{"question": "\uc0f4\ud30c\ub294 \ube44\ub8e8\uc2a4\uac00 \uc9c0\uad6c\uc5d0\uc11c \uac00\uc838\uc628 \ubb34\uc5c7\ub4e4\uc744 \uc120\ub9dd\ud588\ub294\uac00?", "paragraph": "\ud504\ub9ac\uc800\uc640\uc758 \uc7ac\ub300\uacb0 \uc774\ud6c4, \uc624\uacf5\uacfc \ubc30\uc9c0\ud130\ub294 \ube44\ub8e8\uc2a4 \ud589\uc131\uc5d0\uc11c \uc218\ud589\uc744 \uacc4\uc18d\ud55c\ub2e4. \uadf8\ub7ec\ub358 \uc911 \uc774\ubbf8 \ud55c\ucc28\ub840 \ub5a1\ubc25 \ub4f1\uc7a5\ud558\uc600\ub358 \uc0f4\ud30c\uc640 \ubc14\ub85c\uc2a4\uac00 \uc774 \uacf3\uc744 \ubc29\ubb38\ud55c\ub2e4. \uc0ac\uc2e4 \ube44\ub8e8\uc2a4\uc640 \uc0f4\ud30c\ub294 \uc30d\ub465\uc774\uace0, \uc6b0\uc774\uc2a4\uc640 \ubc14\ub85c\uc2a4\ub3c4 \uc624\ub204\uc774\uc600\ub2e4. \ucd1d 12\uac1c\uc758 \uc6b0\uc8fc\uac00 \uc788\ub294\ub370, \uc11c\ub85c \ub300\uce6d\ub418\ub294 \uc6b0\uc8fc\ub07c\ub9ac \uc11c\ub85c \uc720\uc0ac\ud55c \uc874\uc7ac\ub97c \uac00\uc9c0\uace0 \uc788\uace0, \ube44\ub8e8\uc2a4\ub294 7\uc6b0\uc8fc, \uc0f4\ud30c\ub294 6\uc6b0\uc8fc\ub97c \uad00\uc7a5\ud558\ub294 \ud30c\uad34\uc2e0\uc774\ub780 \uc124\uc815\uc774\ub2e4. \uc0f4\ud30c\ub294 \ube44\ub8e8\uc2a4\uac00 \uc790\ub791\uce5c \uc9c0\uad6c\uc5d0\uc11c \uac00\uc838\uc628 \uc74c\uc2dd\ub4e4\uc744 \ubd80\ub7ec\uc6cc\ud588\ub294\ub370, \uc790\uae30 \ucabd \uc6b0\uc8fc \uc9c0\uad6c\uc5d0 \ud574\ub2f9\ud558\ub294 \ud589\uc131\uc5d0\uc120 \uc778\ub958 \ubb38\uba85\uc774 \uc774\ubbf8 \uba78\ub9dd\ud588\ub2e4\ub294 \uc0ac\uc2e4\uc744 \uc54c\uc790, 7\uc6b0\uc8fc \ucabd \uc9c0\uad6c\ub97c \uac00\uc9c0\uace0 \ub0b4\uae30\ub97c \ud558\uae30\ub85c \ud55c\ub2e4. \uc0f4\ud30c\ub294 \uc790\uc2e0\uc774 \uc288\ud37c \ub4dc\ub798\uace4\ubcfc 6\uac1c\ub97c \uc774\ubbf8 \ubaa8\uc558\ub2e4\uba74\uc11c, \ube44\ub8e8\uc2a4 \ub124\uac00 \uc774\uae30\uba74 \uc774\uac78 \uc591\ubcf4\ud560\uaebc\uace0, \uc790\uc2e0\uc774 \uc774\uae30\uba74 \uc774 \uc288\ud37c \ub4dc\ub798\uace4\ubcfc\uc758 \ud798\uc73c\ub85c \uc11c\ub85c\uc758 \uc9c0\uad6c\ub97c \ub9de\ubc14\uafb8\uaca0\ub2e8 \ub0b4\uc6a9\uc758 \ub0b4\uae30\uc600\ub2e4. \ub450 \uc0ac\ub78c\uc774 \uc9c1\uc811 \uc2f8\uc6b0\uba74 \uc6b0\uc8fc\uac00 \ud30c\uad34\ub420\uc9c0\ub3c4 \ubaa8\ub974\ub294 \uc0c1\ud669\uc774\uc5c8\uae30\uc5d0 \uac01\uc790 5\uba85\uc758 \ub300\ub9ac\uc778\uc744 \ub0b4\uc138\uc6cc \uaca9\ud22c \ub300\ud68c\ub97c \ubc8c\uc774\uae30\ub85c \ud558\uc600\ub2e4. \ube44\ub8e8\uc2a4\ub294 \ub531\ud788 \uc6d0\ud558\ub358 \uc18c\uc6d0\uc774 \uc5c6\uc5c8\uc9c0\ub9cc \uc624\uacf5\uc774 \uc6d0\ud558\uc600\uace0, \uacbd\uc7c1\uc2ec\uc5d0 \uc7ac\ubbf8\uc0bc\uc544 \uc81c\uc548\uc744 \ubc1b\uc544\ub4e4\uc774\uae30\ub85c \ud55c\ub2e4. \uc774\ud6c4 5\uba85\uc758 \ub300\ub9ac\uc778\uc740 \uc790\uc2e0\uc774 \uc18d\ud55c \uc6b0\uc8fc\ub97c \uc9c0\ud0a4\uae30 \uc704\ud574 \uaca9\ud22c \ub300\ud68c\uc5d0\uc11c \ubd84\uc804\ud55c\ub2e4. \uc774\ud6c4 \uaca9\ud22c \ub300\ud68c\uc758 \ub0b4\uc6a9\uc5d0 \ub9cc\uc871\ud55c \uacc4\uc655\uc2e0 \uc77c\ud589\uc740 \ub450 \uc6b0\uc8fc\ub97c \uc874\uc18d\uc2dc\ud0a4\uae30\ub85c \ud569\uc758\ud55c\ub2e4.", "answer": "\uc74c\uc2dd", "sentence": "\uc0f4\ud30c\ub294 \ube44\ub8e8\uc2a4\uac00 \uc790\ub791\uce5c \uc9c0\uad6c\uc5d0\uc11c \uac00\uc838\uc628 \uc74c\uc2dd \ub4e4\uc744 \ubd80\ub7ec\uc6cc\ud588\ub294\ub370, \uc790\uae30 \ucabd \uc6b0\uc8fc \uc9c0\uad6c\uc5d0 \ud574\ub2f9\ud558\ub294 \ud589\uc131\uc5d0\uc120 \uc778\ub958 \ubb38\uba85\uc774 \uc774\ubbf8 \uba78\ub9dd\ud588\ub2e4\ub294 \uc0ac\uc2e4\uc744 \uc54c\uc790, 7\uc6b0\uc8fc \ucabd \uc9c0\uad6c\ub97c \uac00\uc9c0\uace0 \ub0b4\uae30\ub97c \ud558\uae30\ub85c \ud55c\ub2e4.", "paragraph_sentence": "\ud504\ub9ac\uc800\uc640\uc758 \uc7ac\ub300\uacb0 \uc774\ud6c4, \uc624\uacf5\uacfc \ubc30\uc9c0\ud130\ub294 \ube44\ub8e8\uc2a4 \ud589\uc131\uc5d0\uc11c \uc218\ud589\uc744 \uacc4\uc18d\ud55c\ub2e4. \uadf8\ub7ec\ub358 \uc911 \uc774\ubbf8 \ud55c\ucc28\ub840 \ub5a1\ubc25 \ub4f1\uc7a5\ud558\uc600\ub358 \uc0f4\ud30c\uc640 \ubc14\ub85c\uc2a4\uac00 \uc774 \uacf3\uc744 \ubc29\ubb38\ud55c\ub2e4. \uc0ac\uc2e4 \ube44\ub8e8\uc2a4\uc640 \uc0f4\ud30c\ub294 \uc30d\ub465\uc774\uace0, \uc6b0\uc774\uc2a4\uc640 \ubc14\ub85c\uc2a4\ub3c4 \uc624\ub204\uc774\uc600\ub2e4. \ucd1d 12\uac1c\uc758 \uc6b0\uc8fc\uac00 \uc788\ub294\ub370, \uc11c\ub85c \ub300\uce6d\ub418\ub294 \uc6b0\uc8fc\ub07c\ub9ac \uc11c\ub85c \uc720\uc0ac\ud55c \uc874\uc7ac\ub97c \uac00\uc9c0\uace0 \uc788\uace0, \ube44\ub8e8\uc2a4\ub294 7\uc6b0\uc8fc, \uc0f4\ud30c\ub294 6\uc6b0\uc8fc\ub97c \uad00\uc7a5\ud558\ub294 \ud30c\uad34\uc2e0\uc774\ub780 \uc124\uc815\uc774\ub2e4. \uc0f4\ud30c\ub294 \ube44\ub8e8\uc2a4\uac00 \uc790\ub791\uce5c \uc9c0\uad6c\uc5d0\uc11c \uac00\uc838\uc628 \uc74c\uc2dd \ub4e4\uc744 \ubd80\ub7ec\uc6cc\ud588\ub294\ub370, \uc790\uae30 \ucabd \uc6b0\uc8fc \uc9c0\uad6c\uc5d0 \ud574\ub2f9\ud558\ub294 \ud589\uc131\uc5d0\uc120 \uc778\ub958 \ubb38\uba85\uc774 \uc774\ubbf8 \uba78\ub9dd\ud588\ub2e4\ub294 \uc0ac\uc2e4\uc744 \uc54c\uc790, 7\uc6b0\uc8fc \ucabd \uc9c0\uad6c\ub97c \uac00\uc9c0\uace0 \ub0b4\uae30\ub97c \ud558\uae30\ub85c \ud55c\ub2e4. \uc0f4\ud30c\ub294 \uc790\uc2e0\uc774 \uc288\ud37c \ub4dc\ub798\uace4\ubcfc 6\uac1c\ub97c \uc774\ubbf8 \ubaa8\uc558\ub2e4\uba74\uc11c, \ube44\ub8e8\uc2a4 \ub124\uac00 \uc774\uae30\uba74 \uc774\uac78 \uc591\ubcf4\ud560\uaebc\uace0, \uc790\uc2e0\uc774 \uc774\uae30\uba74 \uc774 \uc288\ud37c \ub4dc\ub798\uace4\ubcfc\uc758 \ud798\uc73c\ub85c \uc11c\ub85c\uc758 \uc9c0\uad6c\ub97c \ub9de\ubc14\uafb8\uaca0\ub2e8 \ub0b4\uc6a9\uc758 \ub0b4\uae30\uc600\ub2e4. \ub450 \uc0ac\ub78c\uc774 \uc9c1\uc811 \uc2f8\uc6b0\uba74 \uc6b0\uc8fc\uac00 \ud30c\uad34\ub420\uc9c0\ub3c4 \ubaa8\ub974\ub294 \uc0c1\ud669\uc774\uc5c8\uae30\uc5d0 \uac01\uc790 5\uba85\uc758 \ub300\ub9ac\uc778\uc744 \ub0b4\uc138\uc6cc \uaca9\ud22c \ub300\ud68c\ub97c \ubc8c\uc774\uae30\ub85c \ud558\uc600\ub2e4. \ube44\ub8e8\uc2a4\ub294 \ub531\ud788 \uc6d0\ud558\ub358 \uc18c\uc6d0\uc774 \uc5c6\uc5c8\uc9c0\ub9cc \uc624\uacf5\uc774 \uc6d0\ud558\uc600\uace0, \uacbd\uc7c1\uc2ec\uc5d0 \uc7ac\ubbf8\uc0bc\uc544 \uc81c\uc548\uc744 \ubc1b\uc544\ub4e4\uc774\uae30\ub85c \ud55c\ub2e4. \uc774\ud6c4 5\uba85\uc758 \ub300\ub9ac\uc778\uc740 \uc790\uc2e0\uc774 \uc18d\ud55c \uc6b0\uc8fc\ub97c \uc9c0\ud0a4\uae30 \uc704\ud574 \uaca9\ud22c \ub300\ud68c\uc5d0\uc11c \ubd84\uc804\ud55c\ub2e4. \uc774\ud6c4 \uaca9\ud22c \ub300\ud68c\uc758 \ub0b4\uc6a9\uc5d0 \ub9cc\uc871\ud55c \uacc4\uc655\uc2e0 \uc77c\ud589\uc740 \ub450 \uc6b0\uc8fc\ub97c \uc874\uc18d\uc2dc\ud0a4\uae30\ub85c \ud569\uc758\ud55c\ub2e4.", "paragraph_answer": "\ud504\ub9ac\uc800\uc640\uc758 \uc7ac\ub300\uacb0 \uc774\ud6c4, \uc624\uacf5\uacfc \ubc30\uc9c0\ud130\ub294 \ube44\ub8e8\uc2a4 \ud589\uc131\uc5d0\uc11c \uc218\ud589\uc744 \uacc4\uc18d\ud55c\ub2e4. \uadf8\ub7ec\ub358 \uc911 \uc774\ubbf8 \ud55c\ucc28\ub840 \ub5a1\ubc25 \ub4f1\uc7a5\ud558\uc600\ub358 \uc0f4\ud30c\uc640 \ubc14\ub85c\uc2a4\uac00 \uc774 \uacf3\uc744 \ubc29\ubb38\ud55c\ub2e4. \uc0ac\uc2e4 \ube44\ub8e8\uc2a4\uc640 \uc0f4\ud30c\ub294 \uc30d\ub465\uc774\uace0, \uc6b0\uc774\uc2a4\uc640 \ubc14\ub85c\uc2a4\ub3c4 \uc624\ub204\uc774\uc600\ub2e4. \ucd1d 12\uac1c\uc758 \uc6b0\uc8fc\uac00 \uc788\ub294\ub370, \uc11c\ub85c \ub300\uce6d\ub418\ub294 \uc6b0\uc8fc\ub07c\ub9ac \uc11c\ub85c \uc720\uc0ac\ud55c \uc874\uc7ac\ub97c \uac00\uc9c0\uace0 \uc788\uace0, \ube44\ub8e8\uc2a4\ub294 7\uc6b0\uc8fc, \uc0f4\ud30c\ub294 6\uc6b0\uc8fc\ub97c \uad00\uc7a5\ud558\ub294 \ud30c\uad34\uc2e0\uc774\ub780 \uc124\uc815\uc774\ub2e4. \uc0f4\ud30c\ub294 \ube44\ub8e8\uc2a4\uac00 \uc790\ub791\uce5c \uc9c0\uad6c\uc5d0\uc11c \uac00\uc838\uc628 \uc74c\uc2dd \ub4e4\uc744 \ubd80\ub7ec\uc6cc\ud588\ub294\ub370, \uc790\uae30 \ucabd \uc6b0\uc8fc \uc9c0\uad6c\uc5d0 \ud574\ub2f9\ud558\ub294 \ud589\uc131\uc5d0\uc120 \uc778\ub958 \ubb38\uba85\uc774 \uc774\ubbf8 \uba78\ub9dd\ud588\ub2e4\ub294 \uc0ac\uc2e4\uc744 \uc54c\uc790, 7\uc6b0\uc8fc \ucabd \uc9c0\uad6c\ub97c \uac00\uc9c0\uace0 \ub0b4\uae30\ub97c \ud558\uae30\ub85c \ud55c\ub2e4. \uc0f4\ud30c\ub294 \uc790\uc2e0\uc774 \uc288\ud37c \ub4dc\ub798\uace4\ubcfc 6\uac1c\ub97c \uc774\ubbf8 \ubaa8\uc558\ub2e4\uba74\uc11c, \ube44\ub8e8\uc2a4 \ub124\uac00 \uc774\uae30\uba74 \uc774\uac78 \uc591\ubcf4\ud560\uaebc\uace0, \uc790\uc2e0\uc774 \uc774\uae30\uba74 \uc774 \uc288\ud37c \ub4dc\ub798\uace4\ubcfc\uc758 \ud798\uc73c\ub85c \uc11c\ub85c\uc758 \uc9c0\uad6c\ub97c \ub9de\ubc14\uafb8\uaca0\ub2e8 \ub0b4\uc6a9\uc758 \ub0b4\uae30\uc600\ub2e4. \ub450 \uc0ac\ub78c\uc774 \uc9c1\uc811 \uc2f8\uc6b0\uba74 \uc6b0\uc8fc\uac00 \ud30c\uad34\ub420\uc9c0\ub3c4 \ubaa8\ub974\ub294 \uc0c1\ud669\uc774\uc5c8\uae30\uc5d0 \uac01\uc790 5\uba85\uc758 \ub300\ub9ac\uc778\uc744 \ub0b4\uc138\uc6cc \uaca9\ud22c \ub300\ud68c\ub97c \ubc8c\uc774\uae30\ub85c \ud558\uc600\ub2e4. \ube44\ub8e8\uc2a4\ub294 \ub531\ud788 \uc6d0\ud558\ub358 \uc18c\uc6d0\uc774 \uc5c6\uc5c8\uc9c0\ub9cc \uc624\uacf5\uc774 \uc6d0\ud558\uc600\uace0, \uacbd\uc7c1\uc2ec\uc5d0 \uc7ac\ubbf8\uc0bc\uc544 \uc81c\uc548\uc744 \ubc1b\uc544\ub4e4\uc774\uae30\ub85c \ud55c\ub2e4. \uc774\ud6c4 5\uba85\uc758 \ub300\ub9ac\uc778\uc740 \uc790\uc2e0\uc774 \uc18d\ud55c \uc6b0\uc8fc\ub97c \uc9c0\ud0a4\uae30 \uc704\ud574 \uaca9\ud22c \ub300\ud68c\uc5d0\uc11c \ubd84\uc804\ud55c\ub2e4. \uc774\ud6c4 \uaca9\ud22c \ub300\ud68c\uc758 \ub0b4\uc6a9\uc5d0 \ub9cc\uc871\ud55c \uacc4\uc655\uc2e0 \uc77c\ud589\uc740 \ub450 \uc6b0\uc8fc\ub97c \uc874\uc18d\uc2dc\ud0a4\uae30\ub85c \ud569\uc758\ud55c\ub2e4.", "sentence_answer": "\uc0f4\ud30c\ub294 \ube44\ub8e8\uc2a4\uac00 \uc790\ub791\uce5c \uc9c0\uad6c\uc5d0\uc11c \uac00\uc838\uc628 \uc74c\uc2dd \ub4e4\uc744 \ubd80\ub7ec\uc6cc\ud588\ub294\ub370, \uc790\uae30 \ucabd \uc6b0\uc8fc \uc9c0\uad6c\uc5d0 \ud574\ub2f9\ud558\ub294 \ud589\uc131\uc5d0\uc120 \uc778\ub958 \ubb38\uba85\uc774 \uc774\ubbf8 \uba78\ub9dd\ud588\ub2e4\ub294 \uc0ac\uc2e4\uc744 \uc54c\uc790, 7\uc6b0\uc8fc \ucabd \uc9c0\uad6c\ub97c \uac00\uc9c0\uace0 \ub0b4\uae30\ub97c \ud558\uae30\ub85c \ud55c\ub2e4.", "paragraph_id": "6492643-4-1", "paragraph_question": "question: \uc0f4\ud30c\ub294 \ube44\ub8e8\uc2a4\uac00 \uc9c0\uad6c\uc5d0\uc11c \uac00\uc838\uc628 \ubb34\uc5c7\ub4e4\uc744 \uc120\ub9dd\ud588\ub294\uac00?, context: \ud504\ub9ac\uc800\uc640\uc758 \uc7ac\ub300\uacb0 \uc774\ud6c4, \uc624\uacf5\uacfc \ubc30\uc9c0\ud130\ub294 \ube44\ub8e8\uc2a4 \ud589\uc131\uc5d0\uc11c \uc218\ud589\uc744 \uacc4\uc18d\ud55c\ub2e4. \uadf8\ub7ec\ub358 \uc911 \uc774\ubbf8 \ud55c\ucc28\ub840 \ub5a1\ubc25 \ub4f1\uc7a5\ud558\uc600\ub358 \uc0f4\ud30c\uc640 \ubc14\ub85c\uc2a4\uac00 \uc774 \uacf3\uc744 \ubc29\ubb38\ud55c\ub2e4. \uc0ac\uc2e4 \ube44\ub8e8\uc2a4\uc640 \uc0f4\ud30c\ub294 \uc30d\ub465\uc774\uace0, \uc6b0\uc774\uc2a4\uc640 \ubc14\ub85c\uc2a4\ub3c4 \uc624\ub204\uc774\uc600\ub2e4. \ucd1d 12\uac1c\uc758 \uc6b0\uc8fc\uac00 \uc788\ub294\ub370, \uc11c\ub85c \ub300\uce6d\ub418\ub294 \uc6b0\uc8fc\ub07c\ub9ac \uc11c\ub85c \uc720\uc0ac\ud55c \uc874\uc7ac\ub97c \uac00\uc9c0\uace0 \uc788\uace0, \ube44\ub8e8\uc2a4\ub294 7\uc6b0\uc8fc, \uc0f4\ud30c\ub294 6\uc6b0\uc8fc\ub97c \uad00\uc7a5\ud558\ub294 \ud30c\uad34\uc2e0\uc774\ub780 \uc124\uc815\uc774\ub2e4. \uc0f4\ud30c\ub294 \ube44\ub8e8\uc2a4\uac00 \uc790\ub791\uce5c \uc9c0\uad6c\uc5d0\uc11c \uac00\uc838\uc628 \uc74c\uc2dd\ub4e4\uc744 \ubd80\ub7ec\uc6cc\ud588\ub294\ub370, \uc790\uae30 \ucabd \uc6b0\uc8fc \uc9c0\uad6c\uc5d0 \ud574\ub2f9\ud558\ub294 \ud589\uc131\uc5d0\uc120 \uc778\ub958 \ubb38\uba85\uc774 \uc774\ubbf8 \uba78\ub9dd\ud588\ub2e4\ub294 \uc0ac\uc2e4\uc744 \uc54c\uc790, 7\uc6b0\uc8fc \ucabd \uc9c0\uad6c\ub97c \uac00\uc9c0\uace0 \ub0b4\uae30\ub97c \ud558\uae30\ub85c \ud55c\ub2e4. \uc0f4\ud30c\ub294 \uc790\uc2e0\uc774 \uc288\ud37c \ub4dc\ub798\uace4\ubcfc 6\uac1c\ub97c \uc774\ubbf8 \ubaa8\uc558\ub2e4\uba74\uc11c, \ube44\ub8e8\uc2a4 \ub124\uac00 \uc774\uae30\uba74 \uc774\uac78 \uc591\ubcf4\ud560\uaebc\uace0, \uc790\uc2e0\uc774 \uc774\uae30\uba74 \uc774 \uc288\ud37c \ub4dc\ub798\uace4\ubcfc\uc758 \ud798\uc73c\ub85c \uc11c\ub85c\uc758 \uc9c0\uad6c\ub97c \ub9de\ubc14\uafb8\uaca0\ub2e8 \ub0b4\uc6a9\uc758 \ub0b4\uae30\uc600\ub2e4. \ub450 \uc0ac\ub78c\uc774 \uc9c1\uc811 \uc2f8\uc6b0\uba74 \uc6b0\uc8fc\uac00 \ud30c\uad34\ub420\uc9c0\ub3c4 \ubaa8\ub974\ub294 \uc0c1\ud669\uc774\uc5c8\uae30\uc5d0 \uac01\uc790 5\uba85\uc758 \ub300\ub9ac\uc778\uc744 \ub0b4\uc138\uc6cc \uaca9\ud22c \ub300\ud68c\ub97c \ubc8c\uc774\uae30\ub85c \ud558\uc600\ub2e4. \ube44\ub8e8\uc2a4\ub294 \ub531\ud788 \uc6d0\ud558\ub358 \uc18c\uc6d0\uc774 \uc5c6\uc5c8\uc9c0\ub9cc \uc624\uacf5\uc774 \uc6d0\ud558\uc600\uace0, \uacbd\uc7c1\uc2ec\uc5d0 \uc7ac\ubbf8\uc0bc\uc544 \uc81c\uc548\uc744 \ubc1b\uc544\ub4e4\uc774\uae30\ub85c \ud55c\ub2e4. \uc774\ud6c4 5\uba85\uc758 \ub300\ub9ac\uc778\uc740 \uc790\uc2e0\uc774 \uc18d\ud55c \uc6b0\uc8fc\ub97c \uc9c0\ud0a4\uae30 \uc704\ud574 \uaca9\ud22c \ub300\ud68c\uc5d0\uc11c \ubd84\uc804\ud55c\ub2e4. \uc774\ud6c4 \uaca9\ud22c \ub300\ud68c\uc758 \ub0b4\uc6a9\uc5d0 \ub9cc\uc871\ud55c \uacc4\uc655\uc2e0 \uc77c\ud589\uc740 \ub450 \uc6b0\uc8fc\ub97c \uc874\uc18d\uc2dc\ud0a4\uae30\ub85c \ud569\uc758\ud55c\ub2e4."} {"question": "\ud504\ub791\uc2a4\uc758 \uc815\uad50\ubd84\ub9ac \uc6d0\uce59\uc744 \uc774\ub974\ub294 \ub9d0\uc74c?", "paragraph": "\ub4dc\ub808\ud4cc\uc2a4 \uc0ac\uac74(\ud504\ub791\uc2a4\uc5b4: L'affaire Dreyfus)\uc740 19\uc138\uae30 \ud6c4\ubc18\uc758 \uc218\ub144 \ub3d9\uc548 \ud504\ub791\uc2a4\ub97c \ud729\uc4f8\uc5c8\ub358 \ubc18\uc720\ub300\uc8fc\uc758\uc640 \uc774 \ub54c\ubb38\uc5d0 \ud76c\uc0dd\ub41c \ubb34\uace0\ud55c \ub4dc\ub808\ud4cc\uc2a4\uc758 \ubb34\uc8c4 \uc5ec\ubd80\ub97c \ub193\uace0 \ub85c\ub9c8 \uac00\ud1a8\ub9ad\uad50\ud68c\uc640 \uad70\ubd80 \ub4f1 \ubcf4\uc218 \uc138\ub825\uacfc \uc9c4\ubcf4 \uc138\ub825\uc774 \uaca9\ub3cc\ud588\ub358 \uc0ac\uac74\uc774\ub2e4. \uc5d0\ubc00 \uc878\ub77c(Emile Zola)\ub97c \ube44\ub86f\ud55c \ub4dc\ub808\ud4cc\uc2a4\uc758 \ubb34\uc8c4\ub97c \uc8fc\uc7a5\ud55c \ub4dc\ub808\ud4cc\uc2a4\ud30c\ub4e4\uc5d0 \ubc18\ud558\uc5ec \ub4dc\ub808\ud4cc\uc2a4\uc758 \uc720\uc8c4\ub97c \uc8fc\uc7a5\ud588\ub358 \ubc18 \ub4dc\ub808\ud4cc\uc2a4\ud30c\ub4e4\uc740 \uc870\uc9c1\uc801\uc778 \ubc18\ub300\uc6b4\ub3d9\uc744 \ubc8c\ub9ac\uae30 \uc704\ud558\uc5ec '\ud504\ub791\uc2a4 \uc870\uad6d \uc5f0\ub9f9'(Ligue de la Patrie Fran\u00e7aise)\uc744 \uacb0\uc131\ud558\uc600\ub2e4. \ud504\ub791\uc2a4 \ud559\uc220\uc6d0(Acad\u00e9mie fran\u00e7aise)\uc758 \uac00\ud1a8\ub9ad \ud68c\uc6d0\ub4e4\uc740 \uc774 \ub2e8\uccb4\uc5d0 \uc77c\uad04\uc801\uc73c\ub85c \uac00\uc785\ud588\ub2e4. \ub85c\ub9c8 \uac00\ud1a8\ub9ad\uad50\ud68c\uc758 \uc801\uadf9\uc801\uc778 \ubc18\ub4dc\ub808\ud4cc\uc2a4 \uc6b4\ub3d9\uc740 \uacb0\uad6d \ub4dc\ub808\ud4cc\uc2a4\uc758 \ubb34\uc8c4\ub85c \ub05d\ub098\uba70 \ud504\ub791\uc2a4\uc758 \uc815\uad50\ubd84\ub9ac \uc6d0\uce59\uc778 \ub77c\uc774\uc2dc\ud14c \uc6d0\uce59\uc744 \uc218\ub9bd\ud558\ub294 \uacc4\uae30\uac00 \ub418\uc5c8\ub2e4. \ub354 \ub098\uc544\uac00 \ub4dc\ub808\ud4cc\uc2a4 \uc9c0\uc9c0\ud30c\ub4e4\uc758 \uc774\ub150\uacfc \uc804\ub7b5\ub4e4\uc740 \ud3ec\ub974\ud22c\uac08\uacfc \uc2a4\ud398\uc778 \uadf8\ub9ac\uace0 \ub77c\ud2f4\uc544\uba54\ub9ac\uce74 \uc804\uc5ed\uc5d0\uc11c \ubc18\uad50\uad8c\uc8fc\uc758 \uc6b4\ub3d9\uc758 \ubaa8\ub378\uc774 \ub418\uc5c8\ub2e4.", "answer": "\ub77c\uc774\uc2dc\ud14c \uc6d0\uce59", "sentence": "\ub85c\ub9c8 \uac00\ud1a8\ub9ad\uad50\ud68c\uc758 \uc801\uadf9\uc801\uc778 \ubc18\ub4dc\ub808\ud4cc\uc2a4 \uc6b4\ub3d9\uc740 \uacb0\uad6d \ub4dc\ub808\ud4cc\uc2a4\uc758 \ubb34\uc8c4\ub85c \ub05d\ub098\uba70 \ud504\ub791\uc2a4\uc758 \uc815\uad50\ubd84\ub9ac \uc6d0\uce59\uc778 \ub77c\uc774\uc2dc\ud14c \uc6d0\uce59 \uc744 \uc218\ub9bd\ud558\ub294 \uacc4\uae30\uac00 \ub418\uc5c8\ub2e4.", "paragraph_sentence": "\ub4dc\ub808\ud4cc\uc2a4 \uc0ac\uac74(\ud504\ub791\uc2a4\uc5b4: L'affaire Dreyfus)\uc740 19\uc138\uae30 \ud6c4\ubc18\uc758 \uc218\ub144 \ub3d9\uc548 \ud504\ub791\uc2a4\ub97c \ud729\uc4f8\uc5c8\ub358 \ubc18\uc720\ub300\uc8fc\uc758\uc640 \uc774 \ub54c\ubb38\uc5d0 \ud76c\uc0dd\ub41c \ubb34\uace0\ud55c \ub4dc\ub808\ud4cc\uc2a4\uc758 \ubb34\uc8c4 \uc5ec\ubd80\ub97c \ub193\uace0 \ub85c\ub9c8 \uac00\ud1a8\ub9ad\uad50\ud68c\uc640 \uad70\ubd80 \ub4f1 \ubcf4\uc218 \uc138\ub825\uacfc \uc9c4\ubcf4 \uc138\ub825\uc774 \uaca9\ub3cc\ud588\ub358 \uc0ac\uac74\uc774\ub2e4. \uc5d0\ubc00 \uc878\ub77c(Emile Zola)\ub97c \ube44\ub86f\ud55c \ub4dc\ub808\ud4cc\uc2a4\uc758 \ubb34\uc8c4\ub97c \uc8fc\uc7a5\ud55c \ub4dc\ub808\ud4cc\uc2a4\ud30c\ub4e4\uc5d0 \ubc18\ud558\uc5ec \ub4dc\ub808\ud4cc\uc2a4\uc758 \uc720\uc8c4\ub97c \uc8fc\uc7a5\ud588\ub358 \ubc18 \ub4dc\ub808\ud4cc\uc2a4\ud30c\ub4e4\uc740 \uc870\uc9c1\uc801\uc778 \ubc18\ub300\uc6b4\ub3d9\uc744 \ubc8c\ub9ac\uae30 \uc704\ud558\uc5ec '\ud504\ub791\uc2a4 \uc870\uad6d \uc5f0\ub9f9'(Ligue de la Patrie Fran\u00e7aise)\uc744 \uacb0\uc131\ud558\uc600\ub2e4. \ud504\ub791\uc2a4 \ud559\uc220\uc6d0(Acad\u00e9mie fran\u00e7aise)\uc758 \uac00\ud1a8\ub9ad \ud68c\uc6d0\ub4e4\uc740 \uc774 \ub2e8\uccb4\uc5d0 \uc77c\uad04\uc801\uc73c\ub85c \uac00\uc785\ud588\ub2e4. \ub85c\ub9c8 \uac00\ud1a8\ub9ad\uad50\ud68c\uc758 \uc801\uadf9\uc801\uc778 \ubc18\ub4dc\ub808\ud4cc\uc2a4 \uc6b4\ub3d9\uc740 \uacb0\uad6d \ub4dc\ub808\ud4cc\uc2a4\uc758 \ubb34\uc8c4\ub85c \ub05d\ub098\uba70 \ud504\ub791\uc2a4\uc758 \uc815\uad50\ubd84\ub9ac \uc6d0\uce59\uc778 \ub77c\uc774\uc2dc\ud14c \uc6d0\uce59 \uc744 \uc218\ub9bd\ud558\ub294 \uacc4\uae30\uac00 \ub418\uc5c8\ub2e4. \ub354 \ub098\uc544\uac00 \ub4dc\ub808\ud4cc\uc2a4 \uc9c0\uc9c0\ud30c\ub4e4\uc758 \uc774\ub150\uacfc \uc804\ub7b5\ub4e4\uc740 \ud3ec\ub974\ud22c\uac08\uacfc \uc2a4\ud398\uc778 \uadf8\ub9ac\uace0 \ub77c\ud2f4\uc544\uba54\ub9ac\uce74 \uc804\uc5ed\uc5d0\uc11c \ubc18\uad50\uad8c\uc8fc\uc758 \uc6b4\ub3d9\uc758 \ubaa8\ub378\uc774 \ub418\uc5c8\ub2e4.", "paragraph_answer": "\ub4dc\ub808\ud4cc\uc2a4 \uc0ac\uac74(\ud504\ub791\uc2a4\uc5b4: L'affaire Dreyfus)\uc740 19\uc138\uae30 \ud6c4\ubc18\uc758 \uc218\ub144 \ub3d9\uc548 \ud504\ub791\uc2a4\ub97c \ud729\uc4f8\uc5c8\ub358 \ubc18\uc720\ub300\uc8fc\uc758\uc640 \uc774 \ub54c\ubb38\uc5d0 \ud76c\uc0dd\ub41c \ubb34\uace0\ud55c \ub4dc\ub808\ud4cc\uc2a4\uc758 \ubb34\uc8c4 \uc5ec\ubd80\ub97c \ub193\uace0 \ub85c\ub9c8 \uac00\ud1a8\ub9ad\uad50\ud68c\uc640 \uad70\ubd80 \ub4f1 \ubcf4\uc218 \uc138\ub825\uacfc \uc9c4\ubcf4 \uc138\ub825\uc774 \uaca9\ub3cc\ud588\ub358 \uc0ac\uac74\uc774\ub2e4. \uc5d0\ubc00 \uc878\ub77c(Emile Zola)\ub97c \ube44\ub86f\ud55c \ub4dc\ub808\ud4cc\uc2a4\uc758 \ubb34\uc8c4\ub97c \uc8fc\uc7a5\ud55c \ub4dc\ub808\ud4cc\uc2a4\ud30c\ub4e4\uc5d0 \ubc18\ud558\uc5ec \ub4dc\ub808\ud4cc\uc2a4\uc758 \uc720\uc8c4\ub97c \uc8fc\uc7a5\ud588\ub358 \ubc18 \ub4dc\ub808\ud4cc\uc2a4\ud30c\ub4e4\uc740 \uc870\uc9c1\uc801\uc778 \ubc18\ub300\uc6b4\ub3d9\uc744 \ubc8c\ub9ac\uae30 \uc704\ud558\uc5ec '\ud504\ub791\uc2a4 \uc870\uad6d \uc5f0\ub9f9'(Ligue de la Patrie Fran\u00e7aise)\uc744 \uacb0\uc131\ud558\uc600\ub2e4. \ud504\ub791\uc2a4 \ud559\uc220\uc6d0(Acad\u00e9mie fran\u00e7aise)\uc758 \uac00\ud1a8\ub9ad \ud68c\uc6d0\ub4e4\uc740 \uc774 \ub2e8\uccb4\uc5d0 \uc77c\uad04\uc801\uc73c\ub85c \uac00\uc785\ud588\ub2e4. \ub85c\ub9c8 \uac00\ud1a8\ub9ad\uad50\ud68c\uc758 \uc801\uadf9\uc801\uc778 \ubc18\ub4dc\ub808\ud4cc\uc2a4 \uc6b4\ub3d9\uc740 \uacb0\uad6d \ub4dc\ub808\ud4cc\uc2a4\uc758 \ubb34\uc8c4\ub85c \ub05d\ub098\uba70 \ud504\ub791\uc2a4\uc758 \uc815\uad50\ubd84\ub9ac \uc6d0\uce59\uc778 \ub77c\uc774\uc2dc\ud14c \uc6d0\uce59 \uc744 \uc218\ub9bd\ud558\ub294 \uacc4\uae30\uac00 \ub418\uc5c8\ub2e4. \ub354 \ub098\uc544\uac00 \ub4dc\ub808\ud4cc\uc2a4 \uc9c0\uc9c0\ud30c\ub4e4\uc758 \uc774\ub150\uacfc \uc804\ub7b5\ub4e4\uc740 \ud3ec\ub974\ud22c\uac08\uacfc \uc2a4\ud398\uc778 \uadf8\ub9ac\uace0 \ub77c\ud2f4\uc544\uba54\ub9ac\uce74 \uc804\uc5ed\uc5d0\uc11c \ubc18\uad50\uad8c\uc8fc\uc758 \uc6b4\ub3d9\uc758 \ubaa8\ub378\uc774 \ub418\uc5c8\ub2e4.", "sentence_answer": "\ub85c\ub9c8 \uac00\ud1a8\ub9ad\uad50\ud68c\uc758 \uc801\uadf9\uc801\uc778 \ubc18\ub4dc\ub808\ud4cc\uc2a4 \uc6b4\ub3d9\uc740 \uacb0\uad6d \ub4dc\ub808\ud4cc\uc2a4\uc758 \ubb34\uc8c4\ub85c \ub05d\ub098\uba70 \ud504\ub791\uc2a4\uc758 \uc815\uad50\ubd84\ub9ac \uc6d0\uce59\uc778 \ub77c\uc774\uc2dc\ud14c \uc6d0\uce59 \uc744 \uc218\ub9bd\ud558\ub294 \uacc4\uae30\uac00 \ub418\uc5c8\ub2e4.", "paragraph_id": "6048895-0-1", "paragraph_question": "question: \ud504\ub791\uc2a4\uc758 \uc815\uad50\ubd84\ub9ac \uc6d0\uce59\uc744 \uc774\ub974\ub294 \ub9d0\uc74c?, context: \ub4dc\ub808\ud4cc\uc2a4 \uc0ac\uac74(\ud504\ub791\uc2a4\uc5b4: L'affaire Dreyfus)\uc740 19\uc138\uae30 \ud6c4\ubc18\uc758 \uc218\ub144 \ub3d9\uc548 \ud504\ub791\uc2a4\ub97c \ud729\uc4f8\uc5c8\ub358 \ubc18\uc720\ub300\uc8fc\uc758\uc640 \uc774 \ub54c\ubb38\uc5d0 \ud76c\uc0dd\ub41c \ubb34\uace0\ud55c \ub4dc\ub808\ud4cc\uc2a4\uc758 \ubb34\uc8c4 \uc5ec\ubd80\ub97c \ub193\uace0 \ub85c\ub9c8 \uac00\ud1a8\ub9ad\uad50\ud68c\uc640 \uad70\ubd80 \ub4f1 \ubcf4\uc218 \uc138\ub825\uacfc \uc9c4\ubcf4 \uc138\ub825\uc774 \uaca9\ub3cc\ud588\ub358 \uc0ac\uac74\uc774\ub2e4. \uc5d0\ubc00 \uc878\ub77c(Emile Zola)\ub97c \ube44\ub86f\ud55c \ub4dc\ub808\ud4cc\uc2a4\uc758 \ubb34\uc8c4\ub97c \uc8fc\uc7a5\ud55c \ub4dc\ub808\ud4cc\uc2a4\ud30c\ub4e4\uc5d0 \ubc18\ud558\uc5ec \ub4dc\ub808\ud4cc\uc2a4\uc758 \uc720\uc8c4\ub97c \uc8fc\uc7a5\ud588\ub358 \ubc18 \ub4dc\ub808\ud4cc\uc2a4\ud30c\ub4e4\uc740 \uc870\uc9c1\uc801\uc778 \ubc18\ub300\uc6b4\ub3d9\uc744 \ubc8c\ub9ac\uae30 \uc704\ud558\uc5ec '\ud504\ub791\uc2a4 \uc870\uad6d \uc5f0\ub9f9'(Ligue de la Patrie Fran\u00e7aise)\uc744 \uacb0\uc131\ud558\uc600\ub2e4. \ud504\ub791\uc2a4 \ud559\uc220\uc6d0(Acad\u00e9mie fran\u00e7aise)\uc758 \uac00\ud1a8\ub9ad \ud68c\uc6d0\ub4e4\uc740 \uc774 \ub2e8\uccb4\uc5d0 \uc77c\uad04\uc801\uc73c\ub85c \uac00\uc785\ud588\ub2e4. \ub85c\ub9c8 \uac00\ud1a8\ub9ad\uad50\ud68c\uc758 \uc801\uadf9\uc801\uc778 \ubc18\ub4dc\ub808\ud4cc\uc2a4 \uc6b4\ub3d9\uc740 \uacb0\uad6d \ub4dc\ub808\ud4cc\uc2a4\uc758 \ubb34\uc8c4\ub85c \ub05d\ub098\uba70 \ud504\ub791\uc2a4\uc758 \uc815\uad50\ubd84\ub9ac \uc6d0\uce59\uc778 \ub77c\uc774\uc2dc\ud14c \uc6d0\uce59\uc744 \uc218\ub9bd\ud558\ub294 \uacc4\uae30\uac00 \ub418\uc5c8\ub2e4. \ub354 \ub098\uc544\uac00 \ub4dc\ub808\ud4cc\uc2a4 \uc9c0\uc9c0\ud30c\ub4e4\uc758 \uc774\ub150\uacfc \uc804\ub7b5\ub4e4\uc740 \ud3ec\ub974\ud22c\uac08\uacfc \uc2a4\ud398\uc778 \uadf8\ub9ac\uace0 \ub77c\ud2f4\uc544\uba54\ub9ac\uce74 \uc804\uc5ed\uc5d0\uc11c \ubc18\uad50\uad8c\uc8fc\uc758 \uc6b4\ub3d9\uc758 \ubaa8\ub378\uc774 \ub418\uc5c8\ub2e4."} {"question": "\ud328\ub7ec\ub2e4\uc784\uc758 \uc804\ud658\uc5d0 \ub300\ud574 \uc774\uc57c\uae30\ud55c \uc0ac\ub78c\uc740?", "paragraph": "19\uc138\uae30\uae4c\uc9c0 \uacfc\ud559\uc801 \uc9c0\uc2dd\uc740 \uc790\uc5f0\uc8fc\uc758\uc640 \uc2e4\uc99d\uc8fc\uc758\ub97c \ubc14\ud0d5\uc73c\ub85c \uac1d\uad00\uc801 \uc9c4\ub9ac\ub85c \uc5ec\uaca8\uc838 \uc654\ub2e4. \uadf8\ub7ec\ub098 20\uc138\uae30\uc5d0 \ub4e4\uc5b4, \uacfc\ud559\ucca0\ud559\uc740 \uacfc\ud559\uc801 \uc9c0\uc2dd\uc5d0 \ub300\ud55c \uae30\uc874\uc758 \uc808\ub300\uc801 \uae30\uc900\uc744 \ube44\ud310\ud558\uc600\ub2e4. \uacfc\ud559\uc801 \uc9c0\uc2dd\uc740 \uc774\ub860\uc5d0 \uc758\uc874\ud558\uace0, \uad00\ucc30 \uc5ed\uc2dc \uc774\ub860\uc5d0 \uae30\ucd08\ud558\ubbc0\ub85c, \uad00\ucc30 \uacb0\uacfc\ub97c \uc808\ub300\uc801\uc73c\ub85c \uc2e0\ub8b0\ud558\ub294 \ud589\ub3d9\uc8fc\uc758\ub294 \uc633\uc9c0 \uc54a\ub2e4\ub294 \ube44\ud310\uc774 \uc788\uc5c8\ub2e4. \ub610\ud55c, \ube44\ud2b8\uac90\uc288\ud0c0\uc778 \ub4f1\uc758 \uc5b8\uc5b4\ucca0\ud559\uc790\ub4e4\uc740 \ub17c\ub9ac\uc801 \ucd94\ub860 \uc790\uccb4\uac00 \uc5b8\uc5b4 \uac8c\uc784\uc758 \uc0b0\ubb3c\ub85c\uc11c \ubcf4\ud3b8\uc801\uc778 \uacfc\ud559\uc801 \ubc29\ubc95\uc758 \ud655\ub9bd\uc5d0\ub294 \ud55c\uacc4\uac00 \uc788\ub2e4\ub294 \uc810\uc744 \uc9c0\uc801\ud558\uc600\ub2e4. \ud1a0\uba38\uc2a4 \uc0c8\ubba4\uc5bc \ucfe4\uc740 \uacfc\ud559 \uc9c0\uc2dd\uc758 \ubc1c\uc804\uacfc \ubcc0\ud654\ub97c \ud328\ub7ec\ub2e4\uc784\uc758 \uc804\ud658\uc73c\ub85c \ubcf4\uc558\ub2e4. \uadf8\ub7ec\ub098, \uacfc\ud559\uc801 \uc9c0\uc2dd\uc740 \uc774\ub7ec\ud55c \uc57d\uc810\uc5d0\ub3c4 \ubd88\uad6c\ud558\uace0, \uc2a4\uc2a4\ub85c\uc758 \uc624\ub958\ub97c \uc218\uc815\ud558\ub294 \uad6c\uc870\ub97c \ud1b5\ud574 \uc0c8\ub85c\uc6b4 \uacb0\uacfc\ub85c \uc790\uae30 \uc790\uc2e0\uc744 \uc7ac\uad6c\uc131\ud568\uc73c\ub85c\uc368, \ubcf4\ub2e4 \ubcf4\ud3b8\ud0c0\ub2f9\ud55c \uc9c4\ub9ac\uc5d0 \uc811\uadfc\ud558\uac8c \ub41c\ub2e4. \uacfc\ud559\uc801 \ubc29\ubc95\uc740 \uc774\ub7ec\ud55c \uacfc\uc815\uc744 \ud1b5\ud574 \uc0c1\ub300\uc801 \uc9c4\ub9ac\uc5d0 \uc811\uadfc\ud558\ub294 \uacfc\uc815\uc774\ub2e4.", "answer": "\ud1a0\uba38\uc2a4 \uc0c8\ubba4\uc5bc \ucfe4", "sentence": "\ud1a0\uba38\uc2a4 \uc0c8\ubba4\uc5bc \ucfe4 \uc740 \uacfc\ud559 \uc9c0\uc2dd\uc758 \ubc1c\uc804\uacfc \ubcc0\ud654\ub97c \ud328\ub7ec\ub2e4\uc784\uc758 \uc804\ud658\uc73c\ub85c \ubcf4\uc558\ub2e4.", "paragraph_sentence": "19\uc138\uae30\uae4c\uc9c0 \uacfc\ud559\uc801 \uc9c0\uc2dd\uc740 \uc790\uc5f0\uc8fc\uc758\uc640 \uc2e4\uc99d\uc8fc\uc758\ub97c \ubc14\ud0d5\uc73c\ub85c \uac1d\uad00\uc801 \uc9c4\ub9ac\ub85c \uc5ec\uaca8\uc838 \uc654\ub2e4. \uadf8\ub7ec\ub098 20\uc138\uae30\uc5d0 \ub4e4\uc5b4, \uacfc\ud559\ucca0\ud559\uc740 \uacfc\ud559\uc801 \uc9c0\uc2dd\uc5d0 \ub300\ud55c \uae30\uc874\uc758 \uc808\ub300\uc801 \uae30\uc900\uc744 \ube44\ud310\ud558\uc600\ub2e4. \uacfc\ud559\uc801 \uc9c0\uc2dd\uc740 \uc774\ub860\uc5d0 \uc758\uc874\ud558\uace0, \uad00\ucc30 \uc5ed\uc2dc \uc774\ub860\uc5d0 \uae30\ucd08\ud558\ubbc0\ub85c, \uad00\ucc30 \uacb0\uacfc\ub97c \uc808\ub300\uc801\uc73c\ub85c \uc2e0\ub8b0\ud558\ub294 \ud589\ub3d9\uc8fc\uc758\ub294 \uc633\uc9c0 \uc54a\ub2e4\ub294 \ube44\ud310\uc774 \uc788\uc5c8\ub2e4. \ub610\ud55c, \ube44\ud2b8\uac90\uc288\ud0c0\uc778 \ub4f1\uc758 \uc5b8\uc5b4\ucca0\ud559\uc790\ub4e4\uc740 \ub17c\ub9ac\uc801 \ucd94\ub860 \uc790\uccb4\uac00 \uc5b8\uc5b4 \uac8c\uc784\uc758 \uc0b0\ubb3c\ub85c\uc11c \ubcf4\ud3b8\uc801\uc778 \uacfc\ud559\uc801 \ubc29\ubc95\uc758 \ud655\ub9bd\uc5d0\ub294 \ud55c\uacc4\uac00 \uc788\ub2e4\ub294 \uc810\uc744 \uc9c0\uc801\ud558\uc600\ub2e4. \ud1a0\uba38\uc2a4 \uc0c8\ubba4\uc5bc \ucfe4 \uc740 \uacfc\ud559 \uc9c0\uc2dd\uc758 \ubc1c\uc804\uacfc \ubcc0\ud654\ub97c \ud328\ub7ec\ub2e4\uc784\uc758 \uc804\ud658\uc73c\ub85c \ubcf4\uc558\ub2e4. \uadf8\ub7ec\ub098, \uacfc\ud559\uc801 \uc9c0\uc2dd\uc740 \uc774\ub7ec\ud55c \uc57d\uc810\uc5d0\ub3c4 \ubd88\uad6c\ud558\uace0, \uc2a4\uc2a4\ub85c\uc758 \uc624\ub958\ub97c \uc218\uc815\ud558\ub294 \uad6c\uc870\ub97c \ud1b5\ud574 \uc0c8\ub85c\uc6b4 \uacb0\uacfc\ub85c \uc790\uae30 \uc790\uc2e0\uc744 \uc7ac\uad6c\uc131\ud568\uc73c\ub85c\uc368, \ubcf4\ub2e4 \ubcf4\ud3b8\ud0c0\ub2f9\ud55c \uc9c4\ub9ac\uc5d0 \uc811\uadfc\ud558\uac8c \ub41c\ub2e4. \uacfc\ud559\uc801 \ubc29\ubc95\uc740 \uc774\ub7ec\ud55c \uacfc\uc815\uc744 \ud1b5\ud574 \uc0c1\ub300\uc801 \uc9c4\ub9ac\uc5d0 \uc811\uadfc\ud558\ub294 \uacfc\uc815\uc774\ub2e4.", "paragraph_answer": "19\uc138\uae30\uae4c\uc9c0 \uacfc\ud559\uc801 \uc9c0\uc2dd\uc740 \uc790\uc5f0\uc8fc\uc758\uc640 \uc2e4\uc99d\uc8fc\uc758\ub97c \ubc14\ud0d5\uc73c\ub85c \uac1d\uad00\uc801 \uc9c4\ub9ac\ub85c \uc5ec\uaca8\uc838 \uc654\ub2e4. \uadf8\ub7ec\ub098 20\uc138\uae30\uc5d0 \ub4e4\uc5b4, \uacfc\ud559\ucca0\ud559\uc740 \uacfc\ud559\uc801 \uc9c0\uc2dd\uc5d0 \ub300\ud55c \uae30\uc874\uc758 \uc808\ub300\uc801 \uae30\uc900\uc744 \ube44\ud310\ud558\uc600\ub2e4. \uacfc\ud559\uc801 \uc9c0\uc2dd\uc740 \uc774\ub860\uc5d0 \uc758\uc874\ud558\uace0, \uad00\ucc30 \uc5ed\uc2dc \uc774\ub860\uc5d0 \uae30\ucd08\ud558\ubbc0\ub85c, \uad00\ucc30 \uacb0\uacfc\ub97c \uc808\ub300\uc801\uc73c\ub85c \uc2e0\ub8b0\ud558\ub294 \ud589\ub3d9\uc8fc\uc758\ub294 \uc633\uc9c0 \uc54a\ub2e4\ub294 \ube44\ud310\uc774 \uc788\uc5c8\ub2e4. \ub610\ud55c, \ube44\ud2b8\uac90\uc288\ud0c0\uc778 \ub4f1\uc758 \uc5b8\uc5b4\ucca0\ud559\uc790\ub4e4\uc740 \ub17c\ub9ac\uc801 \ucd94\ub860 \uc790\uccb4\uac00 \uc5b8\uc5b4 \uac8c\uc784\uc758 \uc0b0\ubb3c\ub85c\uc11c \ubcf4\ud3b8\uc801\uc778 \uacfc\ud559\uc801 \ubc29\ubc95\uc758 \ud655\ub9bd\uc5d0\ub294 \ud55c\uacc4\uac00 \uc788\ub2e4\ub294 \uc810\uc744 \uc9c0\uc801\ud558\uc600\ub2e4. \ud1a0\uba38\uc2a4 \uc0c8\ubba4\uc5bc \ucfe4 \uc740 \uacfc\ud559 \uc9c0\uc2dd\uc758 \ubc1c\uc804\uacfc \ubcc0\ud654\ub97c \ud328\ub7ec\ub2e4\uc784\uc758 \uc804\ud658\uc73c\ub85c \ubcf4\uc558\ub2e4. \uadf8\ub7ec\ub098, \uacfc\ud559\uc801 \uc9c0\uc2dd\uc740 \uc774\ub7ec\ud55c \uc57d\uc810\uc5d0\ub3c4 \ubd88\uad6c\ud558\uace0, \uc2a4\uc2a4\ub85c\uc758 \uc624\ub958\ub97c \uc218\uc815\ud558\ub294 \uad6c\uc870\ub97c \ud1b5\ud574 \uc0c8\ub85c\uc6b4 \uacb0\uacfc\ub85c \uc790\uae30 \uc790\uc2e0\uc744 \uc7ac\uad6c\uc131\ud568\uc73c\ub85c\uc368, \ubcf4\ub2e4 \ubcf4\ud3b8\ud0c0\ub2f9\ud55c \uc9c4\ub9ac\uc5d0 \uc811\uadfc\ud558\uac8c \ub41c\ub2e4. \uacfc\ud559\uc801 \ubc29\ubc95\uc740 \uc774\ub7ec\ud55c \uacfc\uc815\uc744 \ud1b5\ud574 \uc0c1\ub300\uc801 \uc9c4\ub9ac\uc5d0 \uc811\uadfc\ud558\ub294 \uacfc\uc815\uc774\ub2e4.", "sentence_answer": " \ud1a0\uba38\uc2a4 \uc0c8\ubba4\uc5bc \ucfe4 \uc740 \uacfc\ud559 \uc9c0\uc2dd\uc758 \ubc1c\uc804\uacfc \ubcc0\ud654\ub97c \ud328\ub7ec\ub2e4\uc784\uc758 \uc804\ud658\uc73c\ub85c \ubcf4\uc558\ub2e4.", "paragraph_id": "6601496-2-2", "paragraph_question": "question: \ud328\ub7ec\ub2e4\uc784\uc758 \uc804\ud658\uc5d0 \ub300\ud574 \uc774\uc57c\uae30\ud55c \uc0ac\ub78c\uc740?, context: 19\uc138\uae30\uae4c\uc9c0 \uacfc\ud559\uc801 \uc9c0\uc2dd\uc740 \uc790\uc5f0\uc8fc\uc758\uc640 \uc2e4\uc99d\uc8fc\uc758\ub97c \ubc14\ud0d5\uc73c\ub85c \uac1d\uad00\uc801 \uc9c4\ub9ac\ub85c \uc5ec\uaca8\uc838 \uc654\ub2e4. \uadf8\ub7ec\ub098 20\uc138\uae30\uc5d0 \ub4e4\uc5b4, \uacfc\ud559\ucca0\ud559\uc740 \uacfc\ud559\uc801 \uc9c0\uc2dd\uc5d0 \ub300\ud55c \uae30\uc874\uc758 \uc808\ub300\uc801 \uae30\uc900\uc744 \ube44\ud310\ud558\uc600\ub2e4. \uacfc\ud559\uc801 \uc9c0\uc2dd\uc740 \uc774\ub860\uc5d0 \uc758\uc874\ud558\uace0, \uad00\ucc30 \uc5ed\uc2dc \uc774\ub860\uc5d0 \uae30\ucd08\ud558\ubbc0\ub85c, \uad00\ucc30 \uacb0\uacfc\ub97c \uc808\ub300\uc801\uc73c\ub85c \uc2e0\ub8b0\ud558\ub294 \ud589\ub3d9\uc8fc\uc758\ub294 \uc633\uc9c0 \uc54a\ub2e4\ub294 \ube44\ud310\uc774 \uc788\uc5c8\ub2e4. \ub610\ud55c, \ube44\ud2b8\uac90\uc288\ud0c0\uc778 \ub4f1\uc758 \uc5b8\uc5b4\ucca0\ud559\uc790\ub4e4\uc740 \ub17c\ub9ac\uc801 \ucd94\ub860 \uc790\uccb4\uac00 \uc5b8\uc5b4 \uac8c\uc784\uc758 \uc0b0\ubb3c\ub85c\uc11c \ubcf4\ud3b8\uc801\uc778 \uacfc\ud559\uc801 \ubc29\ubc95\uc758 \ud655\ub9bd\uc5d0\ub294 \ud55c\uacc4\uac00 \uc788\ub2e4\ub294 \uc810\uc744 \uc9c0\uc801\ud558\uc600\ub2e4. \ud1a0\uba38\uc2a4 \uc0c8\ubba4\uc5bc \ucfe4\uc740 \uacfc\ud559 \uc9c0\uc2dd\uc758 \ubc1c\uc804\uacfc \ubcc0\ud654\ub97c \ud328\ub7ec\ub2e4\uc784\uc758 \uc804\ud658\uc73c\ub85c \ubcf4\uc558\ub2e4. \uadf8\ub7ec\ub098, \uacfc\ud559\uc801 \uc9c0\uc2dd\uc740 \uc774\ub7ec\ud55c \uc57d\uc810\uc5d0\ub3c4 \ubd88\uad6c\ud558\uace0, \uc2a4\uc2a4\ub85c\uc758 \uc624\ub958\ub97c \uc218\uc815\ud558\ub294 \uad6c\uc870\ub97c \ud1b5\ud574 \uc0c8\ub85c\uc6b4 \uacb0\uacfc\ub85c \uc790\uae30 \uc790\uc2e0\uc744 \uc7ac\uad6c\uc131\ud568\uc73c\ub85c\uc368, \ubcf4\ub2e4 \ubcf4\ud3b8\ud0c0\ub2f9\ud55c \uc9c4\ub9ac\uc5d0 \uc811\uadfc\ud558\uac8c \ub41c\ub2e4. \uacfc\ud559\uc801 \ubc29\ubc95\uc740 \uc774\ub7ec\ud55c \uacfc\uc815\uc744 \ud1b5\ud574 \uc0c1\ub300\uc801 \uc9c4\ub9ac\uc5d0 \uc811\uadfc\ud558\ub294 \uacfc\uc815\uc774\ub2e4."} {"question": "AKA\uac00 \ubc95\uc778\uc73c\ub85c \uc778\uc815\ubc1b\uc740 \ud574\ub294 \uc5b8\uc81c\uc778\uac00?", "paragraph": "\uc54c\ud30c \uce74\ud30c \uc54c\ud30c(\uadf8\ub9ac\uc2a4\uc5b4: \u0386\u039a\u0386, \uc601\uc5b4: Alpha Kappa Alpha, \uc774\ud558 AKA)\ub294 \uc544\ud504\ub9ac\uce74\uacc4 \ubbf8\uad6d\uc778 \uc5ec\ub300\uc0dd\ub4e4\uc774 \uc124\ub9bd\ud558\uace0 \ubc95\uc778\uc73c\ub85c \ub9cc\ub4e0 \uc5ec\uc131\ud68c\ub85c\uc11c, \ucd5c\ucd08\uc758 \uadf8\ub9ac\uc2a4 \ubb38\uc790\ub85c \uc774\ub984\uc9c0\uc5b4\uc9c4 \uc5ec\uc131\ud68c\uc774\ub2e4. \uc5d0\uc124 \ud5e4\uc9c0\uba3c \ub77c\uc77c\uc774 \uc8fc\ub3c4\ud55c 9\uba85\uc758 \ud559\uc0dd \ubaa8\uc784\uc5d0 \uc758\ud574 \uc6cc\uc2f1\ud134 D.C.\uc758 \ud558\uc6cc\ub4dc \ub300\ud559\uad50\uc5d0\uc11c 1908\ub144 1\uc6d4 15\uc77c \uc124\ub9bd\ub418\uc5c8\ub2e4. \uc5ec\uc131\ud68c\ub97c \uacb0\uc131\ud558\ub294 \uac83\uc740, 20\uc138\uae30 \ucd08 \uc18c\uc218\ubbfc\uc871 \ubc0f \uc5ec\uc131\ub4e4\uc774 \uae30\ud68c\uc758 \ubd88\ud3c9\ub4f1\uc73c\ub85c \uc778\ud558\uc5ec \ud798\uc774\ub098 \uad8c\uc704\uac00 \uac70\uc758 \uc5c6\ub358 \uc9c0\uc5ed\ub4e4\uc5d0\uc11c \uc544\ud504\ub9ac\uce74\uacc4 \ubbf8\uad6d\uc778 \uc5ec\uc131\uc5d0 \ub300\ud55c \uc7a5\ubcbd\uc744 \ubb34\ub108\ub728\ub9ac\ub294 \ub370 \uae30\uc5ec\ud588\ub2e4. AKA\ub294 1913\ub144 1\uc6d4 29\uc77c \ubc95\uc778\uc774 \ub418\uc5c8\ub2e4. \uc544\ud504\ub9ac\uce74\uacc4, \uce74\ud504\uce74\uc2a4\uacc4, \uc544\uc2dc\uc544\uacc4, \ud788\uc2a4\ud328\ub2c9\uc758 \uc5ec\ub300\uc0dd\ub4e4\ub85c \uad6c\uc131\ub41c \uc774 \uc5ec\uc131\ud68c\uc5d0\ub294 \ubbf8\uad6d \ubc0f \uc5ec\ub7ec \ub098\ub77c\uc5d0 975\uac1c \uc774\uc0c1\uc758 \uc9c0\ubd80\uac00 \uc788\uc73c\uba70, 20\ub9cc\uba85 \uc774\uc0c1\uc758 \ud68c\uc6d0\ub4e4\uc774 \uc788\ub2e4. \uc5ec\uc131\ub4e4\uc740 \ub300\ud559 \uc9c0\ubd80 \ub610\ub294 \ub300\ud559\uc6d0 \uc9c0\ubd80\ub97c \ud1b5\ud574 \uac00\uc785\ud560 \uc218 \uc788\ub2e4.", "answer": "1913\ub144", "sentence": "AKA\ub294 1913\ub144 1\uc6d4 29\uc77c \ubc95\uc778\uc774 \ub418\uc5c8\ub2e4.", "paragraph_sentence": "\uc54c\ud30c \uce74\ud30c \uc54c\ud30c(\uadf8\ub9ac\uc2a4\uc5b4: \u0386\u039a\u0386, \uc601\uc5b4: Alpha Kappa Alpha, \uc774\ud558 AKA)\ub294 \uc544\ud504\ub9ac\uce74\uacc4 \ubbf8\uad6d\uc778 \uc5ec\ub300\uc0dd\ub4e4\uc774 \uc124\ub9bd\ud558\uace0 \ubc95\uc778\uc73c\ub85c \ub9cc\ub4e0 \uc5ec\uc131\ud68c\ub85c\uc11c, \ucd5c\ucd08\uc758 \uadf8\ub9ac\uc2a4 \ubb38\uc790\ub85c \uc774\ub984\uc9c0\uc5b4\uc9c4 \uc5ec\uc131\ud68c\uc774\ub2e4. \uc5d0\uc124 \ud5e4\uc9c0\uba3c \ub77c\uc77c\uc774 \uc8fc\ub3c4\ud55c 9\uba85\uc758 \ud559\uc0dd \ubaa8\uc784\uc5d0 \uc758\ud574 \uc6cc\uc2f1\ud134 D.C.\uc758 \ud558\uc6cc\ub4dc \ub300\ud559\uad50\uc5d0\uc11c 1908\ub144 1\uc6d4 15\uc77c \uc124\ub9bd\ub418\uc5c8\ub2e4. \uc5ec\uc131\ud68c\ub97c \uacb0\uc131\ud558\ub294 \uac83\uc740, 20\uc138\uae30 \ucd08 \uc18c\uc218\ubbfc\uc871 \ubc0f \uc5ec\uc131\ub4e4\uc774 \uae30\ud68c\uc758 \ubd88\ud3c9\ub4f1\uc73c\ub85c \uc778\ud558\uc5ec \ud798\uc774\ub098 \uad8c\uc704\uac00 \uac70\uc758 \uc5c6\ub358 \uc9c0\uc5ed\ub4e4\uc5d0\uc11c \uc544\ud504\ub9ac\uce74\uacc4 \ubbf8\uad6d\uc778 \uc5ec\uc131\uc5d0 \ub300\ud55c \uc7a5\ubcbd\uc744 \ubb34\ub108\ub728\ub9ac\ub294 \ub370 \uae30\uc5ec\ud588\ub2e4. AKA\ub294 1913\ub144 1\uc6d4 29\uc77c \ubc95\uc778\uc774 \ub418\uc5c8\ub2e4. \uc544\ud504\ub9ac\uce74\uacc4, \uce74\ud504\uce74\uc2a4\uacc4, \uc544\uc2dc\uc544\uacc4, \ud788\uc2a4\ud328\ub2c9\uc758 \uc5ec\ub300\uc0dd\ub4e4\ub85c \uad6c\uc131\ub41c \uc774 \uc5ec\uc131\ud68c\uc5d0\ub294 \ubbf8\uad6d \ubc0f \uc5ec\ub7ec \ub098\ub77c\uc5d0 975\uac1c \uc774\uc0c1\uc758 \uc9c0\ubd80\uac00 \uc788\uc73c\uba70, 20\ub9cc\uba85 \uc774\uc0c1\uc758 \ud68c\uc6d0\ub4e4\uc774 \uc788\ub2e4. \uc5ec\uc131\ub4e4\uc740 \ub300\ud559 \uc9c0\ubd80 \ub610\ub294 \ub300\ud559\uc6d0 \uc9c0\ubd80\ub97c \ud1b5\ud574 \uac00\uc785\ud560 \uc218 \uc788\ub2e4.", "paragraph_answer": "\uc54c\ud30c \uce74\ud30c \uc54c\ud30c(\uadf8\ub9ac\uc2a4\uc5b4: \u0386\u039a\u0386, \uc601\uc5b4: Alpha Kappa Alpha, \uc774\ud558 AKA)\ub294 \uc544\ud504\ub9ac\uce74\uacc4 \ubbf8\uad6d\uc778 \uc5ec\ub300\uc0dd\ub4e4\uc774 \uc124\ub9bd\ud558\uace0 \ubc95\uc778\uc73c\ub85c \ub9cc\ub4e0 \uc5ec\uc131\ud68c\ub85c\uc11c, \ucd5c\ucd08\uc758 \uadf8\ub9ac\uc2a4 \ubb38\uc790\ub85c \uc774\ub984\uc9c0\uc5b4\uc9c4 \uc5ec\uc131\ud68c\uc774\ub2e4. \uc5d0\uc124 \ud5e4\uc9c0\uba3c \ub77c\uc77c\uc774 \uc8fc\ub3c4\ud55c 9\uba85\uc758 \ud559\uc0dd \ubaa8\uc784\uc5d0 \uc758\ud574 \uc6cc\uc2f1\ud134 D.C.\uc758 \ud558\uc6cc\ub4dc \ub300\ud559\uad50\uc5d0\uc11c 1908\ub144 1\uc6d4 15\uc77c \uc124\ub9bd\ub418\uc5c8\ub2e4. \uc5ec\uc131\ud68c\ub97c \uacb0\uc131\ud558\ub294 \uac83\uc740, 20\uc138\uae30 \ucd08 \uc18c\uc218\ubbfc\uc871 \ubc0f \uc5ec\uc131\ub4e4\uc774 \uae30\ud68c\uc758 \ubd88\ud3c9\ub4f1\uc73c\ub85c \uc778\ud558\uc5ec \ud798\uc774\ub098 \uad8c\uc704\uac00 \uac70\uc758 \uc5c6\ub358 \uc9c0\uc5ed\ub4e4\uc5d0\uc11c \uc544\ud504\ub9ac\uce74\uacc4 \ubbf8\uad6d\uc778 \uc5ec\uc131\uc5d0 \ub300\ud55c \uc7a5\ubcbd\uc744 \ubb34\ub108\ub728\ub9ac\ub294 \ub370 \uae30\uc5ec\ud588\ub2e4. AKA\ub294 1913\ub144 1\uc6d4 29\uc77c \ubc95\uc778\uc774 \ub418\uc5c8\ub2e4. \uc544\ud504\ub9ac\uce74\uacc4, \uce74\ud504\uce74\uc2a4\uacc4, \uc544\uc2dc\uc544\uacc4, \ud788\uc2a4\ud328\ub2c9\uc758 \uc5ec\ub300\uc0dd\ub4e4\ub85c \uad6c\uc131\ub41c \uc774 \uc5ec\uc131\ud68c\uc5d0\ub294 \ubbf8\uad6d \ubc0f \uc5ec\ub7ec \ub098\ub77c\uc5d0 975\uac1c \uc774\uc0c1\uc758 \uc9c0\ubd80\uac00 \uc788\uc73c\uba70, 20\ub9cc\uba85 \uc774\uc0c1\uc758 \ud68c\uc6d0\ub4e4\uc774 \uc788\ub2e4. \uc5ec\uc131\ub4e4\uc740 \ub300\ud559 \uc9c0\ubd80 \ub610\ub294 \ub300\ud559\uc6d0 \uc9c0\ubd80\ub97c \ud1b5\ud574 \uac00\uc785\ud560 \uc218 \uc788\ub2e4.", "sentence_answer": "AKA\ub294 1913\ub144 1\uc6d4 29\uc77c \ubc95\uc778\uc774 \ub418\uc5c8\ub2e4.", "paragraph_id": "6604746-0-2", "paragraph_question": "question: AKA\uac00 \ubc95\uc778\uc73c\ub85c \uc778\uc815\ubc1b\uc740 \ud574\ub294 \uc5b8\uc81c\uc778\uac00?, context: \uc54c\ud30c \uce74\ud30c \uc54c\ud30c(\uadf8\ub9ac\uc2a4\uc5b4: \u0386\u039a\u0386, \uc601\uc5b4: Alpha Kappa Alpha, \uc774\ud558 AKA)\ub294 \uc544\ud504\ub9ac\uce74\uacc4 \ubbf8\uad6d\uc778 \uc5ec\ub300\uc0dd\ub4e4\uc774 \uc124\ub9bd\ud558\uace0 \ubc95\uc778\uc73c\ub85c \ub9cc\ub4e0 \uc5ec\uc131\ud68c\ub85c\uc11c, \ucd5c\ucd08\uc758 \uadf8\ub9ac\uc2a4 \ubb38\uc790\ub85c \uc774\ub984\uc9c0\uc5b4\uc9c4 \uc5ec\uc131\ud68c\uc774\ub2e4. \uc5d0\uc124 \ud5e4\uc9c0\uba3c \ub77c\uc77c\uc774 \uc8fc\ub3c4\ud55c 9\uba85\uc758 \ud559\uc0dd \ubaa8\uc784\uc5d0 \uc758\ud574 \uc6cc\uc2f1\ud134 D.C.\uc758 \ud558\uc6cc\ub4dc \ub300\ud559\uad50\uc5d0\uc11c 1908\ub144 1\uc6d4 15\uc77c \uc124\ub9bd\ub418\uc5c8\ub2e4. \uc5ec\uc131\ud68c\ub97c \uacb0\uc131\ud558\ub294 \uac83\uc740, 20\uc138\uae30 \ucd08 \uc18c\uc218\ubbfc\uc871 \ubc0f \uc5ec\uc131\ub4e4\uc774 \uae30\ud68c\uc758 \ubd88\ud3c9\ub4f1\uc73c\ub85c \uc778\ud558\uc5ec \ud798\uc774\ub098 \uad8c\uc704\uac00 \uac70\uc758 \uc5c6\ub358 \uc9c0\uc5ed\ub4e4\uc5d0\uc11c \uc544\ud504\ub9ac\uce74\uacc4 \ubbf8\uad6d\uc778 \uc5ec\uc131\uc5d0 \ub300\ud55c \uc7a5\ubcbd\uc744 \ubb34\ub108\ub728\ub9ac\ub294 \ub370 \uae30\uc5ec\ud588\ub2e4. AKA\ub294 1913\ub144 1\uc6d4 29\uc77c \ubc95\uc778\uc774 \ub418\uc5c8\ub2e4. \uc544\ud504\ub9ac\uce74\uacc4, \uce74\ud504\uce74\uc2a4\uacc4, \uc544\uc2dc\uc544\uacc4, \ud788\uc2a4\ud328\ub2c9\uc758 \uc5ec\ub300\uc0dd\ub4e4\ub85c \uad6c\uc131\ub41c \uc774 \uc5ec\uc131\ud68c\uc5d0\ub294 \ubbf8\uad6d \ubc0f \uc5ec\ub7ec \ub098\ub77c\uc5d0 975\uac1c \uc774\uc0c1\uc758 \uc9c0\ubd80\uac00 \uc788\uc73c\uba70, 20\ub9cc\uba85 \uc774\uc0c1\uc758 \ud68c\uc6d0\ub4e4\uc774 \uc788\ub2e4. \uc5ec\uc131\ub4e4\uc740 \ub300\ud559 \uc9c0\ubd80 \ub610\ub294 \ub300\ud559\uc6d0 \uc9c0\ubd80\ub97c \ud1b5\ud574 \uac00\uc785\ud560 \uc218 \uc788\ub2e4."} {"question": "\uc7ad \ucff0\uc2a4\uac00 \uac10\ub3c5\uc73c\ub85c \uc120\uc784\ub418\uc5c8\ub358 \ud300\uc758 \uc774\ub984\uc740?", "paragraph": "1918\ub144 12\uc6d4 17\uc77c, \uc7ad \ucff0\uc2a4\ub294 \ud544\ub77c\ub378\ud53c\uc544 \ud544\ub9ac\uc2a4\uc758 \uc0c8\ub85c\uc6b4 \uac10\ub3c5\uc73c\ub85c \uc120\uc784\ub418\uc5c8\ub2e4. \uc7ad \ucff0\uc2a4\uac00 \uc774\ub044\ub294 1919\ub144\uc758 \ud544\ub9ac\uc2a4\ub294 \uc2dc\uc98c \uc911\ubc18\uae4c\uc9c0 18\uc2b9 44\ud328\ub97c \uac70\ub450\uba70, 3\ud560\uc5d0\ub3c4 \ubbf8\uce58\uc9c0 \ubabb\ud558\ub294 \uc2b9\ub960\uc744 \uae30\ub85d\ud588\ub2e4. \uacb0\uad6d 7\uc6d4 9\uc77c \uac10\ub3c5\uc9c1\uc5d0\uc11c \uc0ac\uc784\ud588\ub2e4. \uc0ac\uc784 \ub2f9\uc2dc \uadf8\ub294 \"\ub098\ub294 \ub0b4\uac00 \ud560 \uc218 \uc788\ub294 \ud55c\uc5d0\uc11c \ubaa8\ub4e0 \uac83\uc744 \ud558\ub824\uace0 \ub178\ub825\ud588\ub2e4. \u2026 \ud300\uc744 \ub354\uc6b1 \uac15\ud558\uac8c \ub9cc\ub4e4\uae30 \uc704\ud574 \uba87 \ucc28\ub840\uc758 \uc2dc\ub3c4\ub97c \ud574\ubd24\uc9c0\ub9cc \ud5c8\uc0ac\uc600\ub2e4. \uc774 \ubaa8\ub4e0 \uac83\uc774 \ub0b4 \uc798\ubabb\uc774\ub77c\uace0\ub294 \uc0dd\uac01\ud558\uc9c0 \uc54a\ub294\ub2e4.\"\ub77c\uace0 \uc774\uc57c\uae30\ud558\uba70 \uc544\uc26c\uc6c0\uc744 \ub098\ud0c0\ub0c8\ub2e4. \ud558\uc9c0\ub9cc \uadf8\ud574 12\uc6d4, \uadf8\ub294 \ub2e4\uc2dc \ub514\ud2b8\ub85c\uc774\ud2b8 \ud0c0\uc774\uac70\uc2a4\uc758 \ubd80\ub984\uc744 \ubc1b\uace0 \ud22c\uc218 \ucf54\uce58\ub85c \ubd80\uc784\ud588\ub2e4. 1920\ub144 6\uc6d4 30\uc77c, \ud0c0\uc774\uac70\uc2a4\uc640 \ud654\uc774\ud2b8\uc0ad\uc2a4\uc758 \uacbd\uae30\uc5d0\uc11c 4\ud68c\uae4c\uc9c0 \uc810\uc218\uac00 0-12\ub85c \uc0c1\ub300\ud3b8\uc5d0\uac8c \uae30\uc6b8\uc790 \uc7ad \ucff0\uc2a4\ub294 5\ud68c \ud328\uc804 \ucc98\ub9ac\ub85c \ub4f1\ud310\ud588\ub2e4. 21\uac1c\uc6d4\ub9cc\uc758 \ub4f1\ud310\uc784\uc5d0\ub3c4 \ubd88\uad6c\ud558\uace0, 5\uc774\ub2dd \ub3d9\uc548 5\ud53c\uc548\ud0c0 2\uc2e4\uc810\uc73c\ub85c \ub9c9\uc558\ub2e4. 7\uc6d4 18\uc77c, \uc138\ub108\ud130\uc2a4\uc640\uc758 \uacbd\uae30\uc5d0\uc11c \ub2e4\uc2dc \ud328\uc804 \ucc98\ub9ac\ub85c \ub4f1\ud310\ud574 \u20443\uc774\ub2dd \ub3d9\uc548 2\ud53c\uc548\ud0c0 1\ubcfc\ub137 3\uc2e4\uc810\ud588\ub2e4. \uc774 \uacbd\uae30\uac00 \uc7ad \ucff0\uc2a4\uc758 \uba54\uc774\uc800 \ub9ac\uadf8 \ub9c8\uc9c0\ub9c9 \uacbd\uae30\uac00 \ub418\uc5c8\ub2e4.", "answer": "\ud544\ub77c\ub378\ud53c\uc544 \ud544\ub9ac\uc2a4", "sentence": "1918\ub144 12\uc6d4 17\uc77c, \uc7ad \ucff0\uc2a4\ub294 \ud544\ub77c\ub378\ud53c\uc544 \ud544\ub9ac\uc2a4 \uc758 \uc0c8\ub85c\uc6b4 \uac10\ub3c5\uc73c\ub85c \uc120\uc784\ub418\uc5c8\ub2e4.", "paragraph_sentence": " 1918\ub144 12\uc6d4 17\uc77c, \uc7ad \ucff0\uc2a4\ub294 \ud544\ub77c\ub378\ud53c\uc544 \ud544\ub9ac\uc2a4 \uc758 \uc0c8\ub85c\uc6b4 \uac10\ub3c5\uc73c\ub85c \uc120\uc784\ub418\uc5c8\ub2e4. \uc7ad \ucff0\uc2a4\uac00 \uc774\ub044\ub294 1919\ub144\uc758 \ud544\ub9ac\uc2a4\ub294 \uc2dc\uc98c \uc911\ubc18\uae4c\uc9c0 18\uc2b9 44\ud328\ub97c \uac70\ub450\uba70, 3\ud560\uc5d0\ub3c4 \ubbf8\uce58\uc9c0 \ubabb\ud558\ub294 \uc2b9\ub960\uc744 \uae30\ub85d\ud588\ub2e4. \uacb0\uad6d 7\uc6d4 9\uc77c \uac10\ub3c5\uc9c1\uc5d0\uc11c \uc0ac\uc784\ud588\ub2e4. \uc0ac\uc784 \ub2f9\uc2dc \uadf8\ub294 \"\ub098\ub294 \ub0b4\uac00 \ud560 \uc218 \uc788\ub294 \ud55c\uc5d0\uc11c \ubaa8\ub4e0 \uac83\uc744 \ud558\ub824\uace0 \ub178\ub825\ud588\ub2e4. \u2026 \ud300\uc744 \ub354\uc6b1 \uac15\ud558\uac8c \ub9cc\ub4e4\uae30 \uc704\ud574 \uba87 \ucc28\ub840\uc758 \uc2dc\ub3c4\ub97c \ud574\ubd24\uc9c0\ub9cc \ud5c8\uc0ac\uc600\ub2e4. \uc774 \ubaa8\ub4e0 \uac83\uc774 \ub0b4 \uc798\ubabb\uc774\ub77c\uace0\ub294 \uc0dd\uac01\ud558\uc9c0 \uc54a\ub294\ub2e4. \"\ub77c\uace0 \uc774\uc57c\uae30\ud558\uba70 \uc544\uc26c\uc6c0\uc744 \ub098\ud0c0\ub0c8\ub2e4. \ud558\uc9c0\ub9cc \uadf8\ud574 12\uc6d4, \uadf8\ub294 \ub2e4\uc2dc \ub514\ud2b8\ub85c\uc774\ud2b8 \ud0c0\uc774\uac70\uc2a4\uc758 \ubd80\ub984\uc744 \ubc1b\uace0 \ud22c\uc218 \ucf54\uce58\ub85c \ubd80\uc784\ud588\ub2e4. 1920\ub144 6\uc6d4 30\uc77c, \ud0c0\uc774\uac70\uc2a4\uc640 \ud654\uc774\ud2b8\uc0ad\uc2a4\uc758 \uacbd\uae30\uc5d0\uc11c 4\ud68c\uae4c\uc9c0 \uc810\uc218\uac00 0-12\ub85c \uc0c1\ub300\ud3b8\uc5d0\uac8c \uae30\uc6b8\uc790 \uc7ad \ucff0\uc2a4\ub294 5\ud68c \ud328\uc804 \ucc98\ub9ac\ub85c \ub4f1\ud310\ud588\ub2e4. 21\uac1c\uc6d4\ub9cc\uc758 \ub4f1\ud310\uc784\uc5d0\ub3c4 \ubd88\uad6c\ud558\uace0, 5\uc774\ub2dd \ub3d9\uc548 5\ud53c\uc548\ud0c0 2\uc2e4\uc810\uc73c\ub85c \ub9c9\uc558\ub2e4. 7\uc6d4 18\uc77c, \uc138\ub108\ud130\uc2a4\uc640\uc758 \uacbd\uae30\uc5d0\uc11c \ub2e4\uc2dc \ud328\uc804 \ucc98\ub9ac\ub85c \ub4f1\ud310\ud574 \u20443\uc774\ub2dd \ub3d9\uc548 2\ud53c\uc548\ud0c0 1\ubcfc\ub137 3\uc2e4\uc810\ud588\ub2e4. \uc774 \uacbd\uae30\uac00 \uc7ad \ucff0\uc2a4\uc758 \uba54\uc774\uc800 \ub9ac\uadf8 \ub9c8\uc9c0\ub9c9 \uacbd\uae30\uac00 \ub418\uc5c8\ub2e4.", "paragraph_answer": "1918\ub144 12\uc6d4 17\uc77c, \uc7ad \ucff0\uc2a4\ub294 \ud544\ub77c\ub378\ud53c\uc544 \ud544\ub9ac\uc2a4 \uc758 \uc0c8\ub85c\uc6b4 \uac10\ub3c5\uc73c\ub85c \uc120\uc784\ub418\uc5c8\ub2e4. \uc7ad \ucff0\uc2a4\uac00 \uc774\ub044\ub294 1919\ub144\uc758 \ud544\ub9ac\uc2a4\ub294 \uc2dc\uc98c \uc911\ubc18\uae4c\uc9c0 18\uc2b9 44\ud328\ub97c \uac70\ub450\uba70, 3\ud560\uc5d0\ub3c4 \ubbf8\uce58\uc9c0 \ubabb\ud558\ub294 \uc2b9\ub960\uc744 \uae30\ub85d\ud588\ub2e4. \uacb0\uad6d 7\uc6d4 9\uc77c \uac10\ub3c5\uc9c1\uc5d0\uc11c \uc0ac\uc784\ud588\ub2e4. \uc0ac\uc784 \ub2f9\uc2dc \uadf8\ub294 \"\ub098\ub294 \ub0b4\uac00 \ud560 \uc218 \uc788\ub294 \ud55c\uc5d0\uc11c \ubaa8\ub4e0 \uac83\uc744 \ud558\ub824\uace0 \ub178\ub825\ud588\ub2e4. \u2026 \ud300\uc744 \ub354\uc6b1 \uac15\ud558\uac8c \ub9cc\ub4e4\uae30 \uc704\ud574 \uba87 \ucc28\ub840\uc758 \uc2dc\ub3c4\ub97c \ud574\ubd24\uc9c0\ub9cc \ud5c8\uc0ac\uc600\ub2e4. \uc774 \ubaa8\ub4e0 \uac83\uc774 \ub0b4 \uc798\ubabb\uc774\ub77c\uace0\ub294 \uc0dd\uac01\ud558\uc9c0 \uc54a\ub294\ub2e4.\"\ub77c\uace0 \uc774\uc57c\uae30\ud558\uba70 \uc544\uc26c\uc6c0\uc744 \ub098\ud0c0\ub0c8\ub2e4. \ud558\uc9c0\ub9cc \uadf8\ud574 12\uc6d4, \uadf8\ub294 \ub2e4\uc2dc \ub514\ud2b8\ub85c\uc774\ud2b8 \ud0c0\uc774\uac70\uc2a4\uc758 \ubd80\ub984\uc744 \ubc1b\uace0 \ud22c\uc218 \ucf54\uce58\ub85c \ubd80\uc784\ud588\ub2e4. 1920\ub144 6\uc6d4 30\uc77c, \ud0c0\uc774\uac70\uc2a4\uc640 \ud654\uc774\ud2b8\uc0ad\uc2a4\uc758 \uacbd\uae30\uc5d0\uc11c 4\ud68c\uae4c\uc9c0 \uc810\uc218\uac00 0-12\ub85c \uc0c1\ub300\ud3b8\uc5d0\uac8c \uae30\uc6b8\uc790 \uc7ad \ucff0\uc2a4\ub294 5\ud68c \ud328\uc804 \ucc98\ub9ac\ub85c \ub4f1\ud310\ud588\ub2e4. 21\uac1c\uc6d4\ub9cc\uc758 \ub4f1\ud310\uc784\uc5d0\ub3c4 \ubd88\uad6c\ud558\uace0, 5\uc774\ub2dd \ub3d9\uc548 5\ud53c\uc548\ud0c0 2\uc2e4\uc810\uc73c\ub85c \ub9c9\uc558\ub2e4. 7\uc6d4 18\uc77c, \uc138\ub108\ud130\uc2a4\uc640\uc758 \uacbd\uae30\uc5d0\uc11c \ub2e4\uc2dc \ud328\uc804 \ucc98\ub9ac\ub85c \ub4f1\ud310\ud574 \u20443\uc774\ub2dd \ub3d9\uc548 2\ud53c\uc548\ud0c0 1\ubcfc\ub137 3\uc2e4\uc810\ud588\ub2e4. \uc774 \uacbd\uae30\uac00 \uc7ad \ucff0\uc2a4\uc758 \uba54\uc774\uc800 \ub9ac\uadf8 \ub9c8\uc9c0\ub9c9 \uacbd\uae30\uac00 \ub418\uc5c8\ub2e4.", "sentence_answer": "1918\ub144 12\uc6d4 17\uc77c, \uc7ad \ucff0\uc2a4\ub294 \ud544\ub77c\ub378\ud53c\uc544 \ud544\ub9ac\uc2a4 \uc758 \uc0c8\ub85c\uc6b4 \uac10\ub3c5\uc73c\ub85c \uc120\uc784\ub418\uc5c8\ub2e4.", "paragraph_id": "5860426-13-0", "paragraph_question": "question: \uc7ad \ucff0\uc2a4\uac00 \uac10\ub3c5\uc73c\ub85c \uc120\uc784\ub418\uc5c8\ub358 \ud300\uc758 \uc774\ub984\uc740?, context: 1918\ub144 12\uc6d4 17\uc77c, \uc7ad \ucff0\uc2a4\ub294 \ud544\ub77c\ub378\ud53c\uc544 \ud544\ub9ac\uc2a4\uc758 \uc0c8\ub85c\uc6b4 \uac10\ub3c5\uc73c\ub85c \uc120\uc784\ub418\uc5c8\ub2e4. \uc7ad \ucff0\uc2a4\uac00 \uc774\ub044\ub294 1919\ub144\uc758 \ud544\ub9ac\uc2a4\ub294 \uc2dc\uc98c \uc911\ubc18\uae4c\uc9c0 18\uc2b9 44\ud328\ub97c \uac70\ub450\uba70, 3\ud560\uc5d0\ub3c4 \ubbf8\uce58\uc9c0 \ubabb\ud558\ub294 \uc2b9\ub960\uc744 \uae30\ub85d\ud588\ub2e4. \uacb0\uad6d 7\uc6d4 9\uc77c \uac10\ub3c5\uc9c1\uc5d0\uc11c \uc0ac\uc784\ud588\ub2e4. \uc0ac\uc784 \ub2f9\uc2dc \uadf8\ub294 \"\ub098\ub294 \ub0b4\uac00 \ud560 \uc218 \uc788\ub294 \ud55c\uc5d0\uc11c \ubaa8\ub4e0 \uac83\uc744 \ud558\ub824\uace0 \ub178\ub825\ud588\ub2e4. \u2026 \ud300\uc744 \ub354\uc6b1 \uac15\ud558\uac8c \ub9cc\ub4e4\uae30 \uc704\ud574 \uba87 \ucc28\ub840\uc758 \uc2dc\ub3c4\ub97c \ud574\ubd24\uc9c0\ub9cc \ud5c8\uc0ac\uc600\ub2e4. \uc774 \ubaa8\ub4e0 \uac83\uc774 \ub0b4 \uc798\ubabb\uc774\ub77c\uace0\ub294 \uc0dd\uac01\ud558\uc9c0 \uc54a\ub294\ub2e4.\"\ub77c\uace0 \uc774\uc57c\uae30\ud558\uba70 \uc544\uc26c\uc6c0\uc744 \ub098\ud0c0\ub0c8\ub2e4. \ud558\uc9c0\ub9cc \uadf8\ud574 12\uc6d4, \uadf8\ub294 \ub2e4\uc2dc \ub514\ud2b8\ub85c\uc774\ud2b8 \ud0c0\uc774\uac70\uc2a4\uc758 \ubd80\ub984\uc744 \ubc1b\uace0 \ud22c\uc218 \ucf54\uce58\ub85c \ubd80\uc784\ud588\ub2e4. 1920\ub144 6\uc6d4 30\uc77c, \ud0c0\uc774\uac70\uc2a4\uc640 \ud654\uc774\ud2b8\uc0ad\uc2a4\uc758 \uacbd\uae30\uc5d0\uc11c 4\ud68c\uae4c\uc9c0 \uc810\uc218\uac00 0-12\ub85c \uc0c1\ub300\ud3b8\uc5d0\uac8c \uae30\uc6b8\uc790 \uc7ad \ucff0\uc2a4\ub294 5\ud68c \ud328\uc804 \ucc98\ub9ac\ub85c \ub4f1\ud310\ud588\ub2e4. 21\uac1c\uc6d4\ub9cc\uc758 \ub4f1\ud310\uc784\uc5d0\ub3c4 \ubd88\uad6c\ud558\uace0, 5\uc774\ub2dd \ub3d9\uc548 5\ud53c\uc548\ud0c0 2\uc2e4\uc810\uc73c\ub85c \ub9c9\uc558\ub2e4. 7\uc6d4 18\uc77c, \uc138\ub108\ud130\uc2a4\uc640\uc758 \uacbd\uae30\uc5d0\uc11c \ub2e4\uc2dc \ud328\uc804 \ucc98\ub9ac\ub85c \ub4f1\ud310\ud574 \u20443\uc774\ub2dd \ub3d9\uc548 2\ud53c\uc548\ud0c0 1\ubcfc\ub137 3\uc2e4\uc810\ud588\ub2e4. \uc774 \uacbd\uae30\uac00 \uc7ad \ucff0\uc2a4\uc758 \uba54\uc774\uc800 \ub9ac\uadf8 \ub9c8\uc9c0\ub9c9 \uacbd\uae30\uac00 \ub418\uc5c8\ub2e4."} {"question": "\ubabd\uace0\uc758 \ub3d9\ucabd \uc601\ud1a0 \uad6c\ubd84\uc744 \uc9d3\ub294 \uac15\uc758 \uc774\ub984\uc740?", "paragraph": "\uadf8\ub294 \ubabd\uace8\uc9c0\uc5ed\ub3c4 \ub2f5\uc0ac\ud558\uace0 \uadf8\uac83 \uc5ed\uc2dc \ud568\uaed8 \uae30\ub85d\uc73c\ub85c \ub0a8\uacbc\ub2e4. '\uc694\ud558 \ub3d9\ucabd\uc5d0 \uc7a5\uad70(\u5c07\u8ecd) \uc14b\uc744 \ubc30\uce58\ud588\ub294\ub370 \ud558\ub098\ub294 \ubd09\ucc9c(\u5949\u5929) \ub4f1\uc9c0\ub97c \uc9c4\uc218(\u93ad\u5b88)\ud558\uba74\uc11c \uc2ec\uc591\uc5d0 \uc8fc\ub454\ud558\uace0, \ud558\ub098\ub294 \uc601\uace0\ud0d1(\u5be7\u53e4\u5854) \uc9c0\ubc29\uc744 \uc9c4\uc218\ud558\uba74\uc11c \uc120\ucc3d(\u8239\u5ee0)\uc5d0 \uc8fc\ub454\ud558\uace0, \ud558\ub098\ub294 \ud751\ub8e1\uac15 \uc9c0\ubc29\uc744 \uc9c4\uc218\ud558\uba74\uc11c \uc560\ud638(\u827e\u6ef8)\uc5d0 \uc8fc\ub454\ud558\uc5ec, \uac01\uae30 \ub9cc\uc8fc \uc9c0\uc5ed\uc744 \ud1b5\uc194\ud558\ub294 \uace0\uc0b0\ub300\uc0ac\uc6d0(\u56fa\u5c71\u5927\u56db\u54e1)\uc774\ub2e4. \uac15\ud76c(\u5eb7\u7199) \ub9d0\uae30\uc5d0 \ud751\ub8e1\uac15 \ubd81\ucabd\uc758 \ubabd\uace0\ub97c \uac00\uc7a5 \uc5fc\ub824\ud558\uc5ec \ub2e4\uc2dc \ubc31\ub3c4\ub20c \uc7a5\uad70(\u767d\u5ea6\u8a25\u5c07\u8ecd) \ud55c \uba85\uc744 \ub354 \ubc30\uce58\ud558\uc600\ub2e4 \ud55c\ub2e4. \ubabd\uace0\uc758 48\uac1c \ubd80\uc871 \uac00\uc6b4\ub370\uc11c \ub3d9\ubd81 \uc9c0\ubc29\uc5d0 \uac70\uc8fc\ud55c \uc871\uc18d\uc774 \uac00\uc7a5 \uac15\uc131\ud558\uc5ec \ub300\ube44\ub2ec\uc790(\u5927\u9f3b\u3e9a\u5b50)\ub294 \ud751\ub989\uac15 \ubd81\ucabd\uc5d0 \uc788\ub2e4. \uadf8\ub9ac\ud558\uc5ec \ub3d9\ucabd\uc73c\ub85c \ud751\ub8e1\uac15\uc5d0\uc11c\ubd80\ud130 \uc7a5\uc131 \ubc16\uc5d0\uae4c\uc9c0 \ubd81\ucabd\uc774\ub098 \uc11c\ucabd\uc774 \ubaa8\ub450 \ubabd\uace0\uc758 \uc601\ud1a0\uc774\uba70, \uadf8 \ub113\uc774\ub294 \uc911\uad6d\uc758 \uba87 \uac11\uc808\uc774\ub098 \ub41c\ub2e4. \uc11c\ub85c \uac15\ub300\ud55c \uc138\ub825\uc744 \uac00\uc9c0\uace0 \ud55c \uc9c0\uc5ed\uc744 \ucc28\uc9c0\ud558\uace0 \uc788\uc73c\uba70 \ub3d9\u318d\uc11c\u318d\ub0a8\u318d\ubd81\uc758 \ud669\uc81c\ub77c\uace0 \ubd80\ub978\ub2e4. \ud669\ud0dc\uadf9(\u9ec3\u592a\u6975)\uacfc \uccad\ud0dc\uadf9(\u9751\u592a\u6975)\uc740 \uc911\uad6d \uc11c\ub0a8\ucabd\uc5d0 \uc788\uc73c\uba70, \uc561\ub77c\uc0ac(\u5384\u7f85\u65af)\ub77c\ub294 \uac83\uc740 \uace7 \ub300\ube44(\u5927\u9f3b)\uc774\uace0, \uac1d\uc774\uac1d(\u5580\u723e\u5580)\uc774\ub77c\ub294 \uac83\uc740 \ub3d9\ubd81\ucabd\uc5d0 \uc788\ub2e4\uace0 \ud55c\ub2e4.'\uba70 \ub300\ub7b5\uc758 \uc704\uce58\ub97c \uae30\ub85d\ud558\uc600\ub2e4.", "answer": "\ud751\ub8e1\uac15", "sentence": "\ud558\ub098\ub294 \ud751\ub8e1\uac15 \uc9c0\ubc29\uc744 \uc9c4\uc218\ud558\uba74\uc11c \uc560\ud638(\u827e\u6ef8)\uc5d0 \uc8fc\ub454\ud558\uc5ec, \uac01\uae30 \ub9cc\uc8fc \uc9c0\uc5ed\uc744 \ud1b5\uc194\ud558\ub294 \uace0\uc0b0\ub300\uc0ac\uc6d0(\u56fa\u5c71\u5927\u56db\u54e1)\uc774\ub2e4.", "paragraph_sentence": "\uadf8\ub294 \ubabd\uace8\uc9c0\uc5ed\ub3c4 \ub2f5\uc0ac\ud558\uace0 \uadf8\uac83 \uc5ed\uc2dc \ud568\uaed8 \uae30\ub85d\uc73c\ub85c \ub0a8\uacbc\ub2e4. '\uc694\ud558 \ub3d9\ucabd\uc5d0 \uc7a5\uad70(\u5c07\u8ecd) \uc14b\uc744 \ubc30\uce58\ud588\ub294\ub370 \ud558\ub098\ub294 \ubd09\ucc9c(\u5949\u5929) \ub4f1\uc9c0\ub97c \uc9c4\uc218(\u93ad\u5b88)\ud558\uba74\uc11c \uc2ec\uc591\uc5d0 \uc8fc\ub454\ud558\uace0, \ud558\ub098\ub294 \uc601\uace0\ud0d1(\u5be7\u53e4\u5854) \uc9c0\ubc29\uc744 \uc9c4\uc218\ud558\uba74\uc11c \uc120\ucc3d(\u8239\u5ee0)\uc5d0 \uc8fc\ub454\ud558\uace0, \ud558\ub098\ub294 \ud751\ub8e1\uac15 \uc9c0\ubc29\uc744 \uc9c4\uc218\ud558\uba74\uc11c \uc560\ud638(\u827e\u6ef8)\uc5d0 \uc8fc\ub454\ud558\uc5ec, \uac01\uae30 \ub9cc\uc8fc \uc9c0\uc5ed\uc744 \ud1b5\uc194\ud558\ub294 \uace0\uc0b0\ub300\uc0ac\uc6d0(\u56fa\u5c71\u5927\u56db\u54e1)\uc774\ub2e4. \uac15\ud76c(\u5eb7\u7199) \ub9d0\uae30\uc5d0 \ud751\ub8e1\uac15 \ubd81\ucabd\uc758 \ubabd\uace0\ub97c \uac00\uc7a5 \uc5fc\ub824\ud558\uc5ec \ub2e4\uc2dc \ubc31\ub3c4\ub20c \uc7a5\uad70(\u767d\u5ea6\u8a25\u5c07\u8ecd) \ud55c \uba85\uc744 \ub354 \ubc30\uce58\ud558\uc600\ub2e4 \ud55c\ub2e4. \ubabd\uace0\uc758 48\uac1c \ubd80\uc871 \uac00\uc6b4\ub370\uc11c \ub3d9\ubd81 \uc9c0\ubc29\uc5d0 \uac70\uc8fc\ud55c \uc871\uc18d\uc774 \uac00\uc7a5 \uac15\uc131\ud558\uc5ec \ub300\ube44\ub2ec\uc790(\u5927\u9f3b\u3e9a\u5b50)\ub294 \ud751\ub989\uac15 \ubd81\ucabd\uc5d0 \uc788\ub2e4. \uadf8\ub9ac\ud558\uc5ec \ub3d9\ucabd\uc73c\ub85c \ud751\ub8e1\uac15\uc5d0\uc11c\ubd80\ud130 \uc7a5\uc131 \ubc16\uc5d0\uae4c\uc9c0 \ubd81\ucabd\uc774\ub098 \uc11c\ucabd\uc774 \ubaa8\ub450 \ubabd\uace0\uc758 \uc601\ud1a0\uc774\uba70, \uadf8 \ub113\uc774\ub294 \uc911\uad6d\uc758 \uba87 \uac11\uc808\uc774\ub098 \ub41c\ub2e4. \uc11c\ub85c \uac15\ub300\ud55c \uc138\ub825\uc744 \uac00\uc9c0\uace0 \ud55c \uc9c0\uc5ed\uc744 \ucc28\uc9c0\ud558\uace0 \uc788\uc73c\uba70 \ub3d9\u318d\uc11c\u318d\ub0a8\u318d\ubd81\uc758 \ud669\uc81c\ub77c\uace0 \ubd80\ub978\ub2e4. \ud669\ud0dc\uadf9(\u9ec3\u592a\u6975)\uacfc \uccad\ud0dc\uadf9(\u9751\u592a\u6975)\uc740 \uc911\uad6d \uc11c\ub0a8\ucabd\uc5d0 \uc788\uc73c\uba70, \uc561\ub77c\uc0ac(\u5384\u7f85\u65af)\ub77c\ub294 \uac83\uc740 \uace7 \ub300\ube44(\u5927\u9f3b)\uc774\uace0, \uac1d\uc774\uac1d(\u5580\u723e\u5580)\uc774\ub77c\ub294 \uac83\uc740 \ub3d9\ubd81\ucabd\uc5d0 \uc788\ub2e4\uace0 \ud55c\ub2e4. '\uba70 \ub300\ub7b5\uc758 \uc704\uce58\ub97c \uae30\ub85d\ud558\uc600\ub2e4.", "paragraph_answer": "\uadf8\ub294 \ubabd\uace8\uc9c0\uc5ed\ub3c4 \ub2f5\uc0ac\ud558\uace0 \uadf8\uac83 \uc5ed\uc2dc \ud568\uaed8 \uae30\ub85d\uc73c\ub85c \ub0a8\uacbc\ub2e4. '\uc694\ud558 \ub3d9\ucabd\uc5d0 \uc7a5\uad70(\u5c07\u8ecd) \uc14b\uc744 \ubc30\uce58\ud588\ub294\ub370 \ud558\ub098\ub294 \ubd09\ucc9c(\u5949\u5929) \ub4f1\uc9c0\ub97c \uc9c4\uc218(\u93ad\u5b88)\ud558\uba74\uc11c \uc2ec\uc591\uc5d0 \uc8fc\ub454\ud558\uace0, \ud558\ub098\ub294 \uc601\uace0\ud0d1(\u5be7\u53e4\u5854) \uc9c0\ubc29\uc744 \uc9c4\uc218\ud558\uba74\uc11c \uc120\ucc3d(\u8239\u5ee0)\uc5d0 \uc8fc\ub454\ud558\uace0, \ud558\ub098\ub294 \ud751\ub8e1\uac15 \uc9c0\ubc29\uc744 \uc9c4\uc218\ud558\uba74\uc11c \uc560\ud638(\u827e\u6ef8)\uc5d0 \uc8fc\ub454\ud558\uc5ec, \uac01\uae30 \ub9cc\uc8fc \uc9c0\uc5ed\uc744 \ud1b5\uc194\ud558\ub294 \uace0\uc0b0\ub300\uc0ac\uc6d0(\u56fa\u5c71\u5927\u56db\u54e1)\uc774\ub2e4. \uac15\ud76c(\u5eb7\u7199) \ub9d0\uae30\uc5d0 \ud751\ub8e1\uac15 \ubd81\ucabd\uc758 \ubabd\uace0\ub97c \uac00\uc7a5 \uc5fc\ub824\ud558\uc5ec \ub2e4\uc2dc \ubc31\ub3c4\ub20c \uc7a5\uad70(\u767d\u5ea6\u8a25\u5c07\u8ecd) \ud55c \uba85\uc744 \ub354 \ubc30\uce58\ud558\uc600\ub2e4 \ud55c\ub2e4. \ubabd\uace0\uc758 48\uac1c \ubd80\uc871 \uac00\uc6b4\ub370\uc11c \ub3d9\ubd81 \uc9c0\ubc29\uc5d0 \uac70\uc8fc\ud55c \uc871\uc18d\uc774 \uac00\uc7a5 \uac15\uc131\ud558\uc5ec \ub300\ube44\ub2ec\uc790(\u5927\u9f3b\u3e9a\u5b50)\ub294 \ud751\ub989\uac15 \ubd81\ucabd\uc5d0 \uc788\ub2e4. \uadf8\ub9ac\ud558\uc5ec \ub3d9\ucabd\uc73c\ub85c \ud751\ub8e1\uac15\uc5d0\uc11c\ubd80\ud130 \uc7a5\uc131 \ubc16\uc5d0\uae4c\uc9c0 \ubd81\ucabd\uc774\ub098 \uc11c\ucabd\uc774 \ubaa8\ub450 \ubabd\uace0\uc758 \uc601\ud1a0\uc774\uba70, \uadf8 \ub113\uc774\ub294 \uc911\uad6d\uc758 \uba87 \uac11\uc808\uc774\ub098 \ub41c\ub2e4. \uc11c\ub85c \uac15\ub300\ud55c \uc138\ub825\uc744 \uac00\uc9c0\uace0 \ud55c \uc9c0\uc5ed\uc744 \ucc28\uc9c0\ud558\uace0 \uc788\uc73c\uba70 \ub3d9\u318d\uc11c\u318d\ub0a8\u318d\ubd81\uc758 \ud669\uc81c\ub77c\uace0 \ubd80\ub978\ub2e4. \ud669\ud0dc\uadf9(\u9ec3\u592a\u6975)\uacfc \uccad\ud0dc\uadf9(\u9751\u592a\u6975)\uc740 \uc911\uad6d \uc11c\ub0a8\ucabd\uc5d0 \uc788\uc73c\uba70, \uc561\ub77c\uc0ac(\u5384\u7f85\u65af)\ub77c\ub294 \uac83\uc740 \uace7 \ub300\ube44(\u5927\u9f3b)\uc774\uace0, \uac1d\uc774\uac1d(\u5580\u723e\u5580)\uc774\ub77c\ub294 \uac83\uc740 \ub3d9\ubd81\ucabd\uc5d0 \uc788\ub2e4\uace0 \ud55c\ub2e4.'\uba70 \ub300\ub7b5\uc758 \uc704\uce58\ub97c \uae30\ub85d\ud558\uc600\ub2e4.", "sentence_answer": "\ud558\ub098\ub294 \ud751\ub8e1\uac15 \uc9c0\ubc29\uc744 \uc9c4\uc218\ud558\uba74\uc11c \uc560\ud638(\u827e\u6ef8)\uc5d0 \uc8fc\ub454\ud558\uc5ec, \uac01\uae30 \ub9cc\uc8fc \uc9c0\uc5ed\uc744 \ud1b5\uc194\ud558\ub294 \uace0\uc0b0\ub300\uc0ac\uc6d0(\u56fa\u5c71\u5927\u56db\u54e1)\uc774\ub2e4.", "paragraph_id": "5835710-5-1", "paragraph_question": "question: \ubabd\uace0\uc758 \ub3d9\ucabd \uc601\ud1a0 \uad6c\ubd84\uc744 \uc9d3\ub294 \uac15\uc758 \uc774\ub984\uc740?, context: \uadf8\ub294 \ubabd\uace8\uc9c0\uc5ed\ub3c4 \ub2f5\uc0ac\ud558\uace0 \uadf8\uac83 \uc5ed\uc2dc \ud568\uaed8 \uae30\ub85d\uc73c\ub85c \ub0a8\uacbc\ub2e4. '\uc694\ud558 \ub3d9\ucabd\uc5d0 \uc7a5\uad70(\u5c07\u8ecd) \uc14b\uc744 \ubc30\uce58\ud588\ub294\ub370 \ud558\ub098\ub294 \ubd09\ucc9c(\u5949\u5929) \ub4f1\uc9c0\ub97c \uc9c4\uc218(\u93ad\u5b88)\ud558\uba74\uc11c \uc2ec\uc591\uc5d0 \uc8fc\ub454\ud558\uace0, \ud558\ub098\ub294 \uc601\uace0\ud0d1(\u5be7\u53e4\u5854) \uc9c0\ubc29\uc744 \uc9c4\uc218\ud558\uba74\uc11c \uc120\ucc3d(\u8239\u5ee0)\uc5d0 \uc8fc\ub454\ud558\uace0, \ud558\ub098\ub294 \ud751\ub8e1\uac15 \uc9c0\ubc29\uc744 \uc9c4\uc218\ud558\uba74\uc11c \uc560\ud638(\u827e\u6ef8)\uc5d0 \uc8fc\ub454\ud558\uc5ec, \uac01\uae30 \ub9cc\uc8fc \uc9c0\uc5ed\uc744 \ud1b5\uc194\ud558\ub294 \uace0\uc0b0\ub300\uc0ac\uc6d0(\u56fa\u5c71\u5927\u56db\u54e1)\uc774\ub2e4. \uac15\ud76c(\u5eb7\u7199) \ub9d0\uae30\uc5d0 \ud751\ub8e1\uac15 \ubd81\ucabd\uc758 \ubabd\uace0\ub97c \uac00\uc7a5 \uc5fc\ub824\ud558\uc5ec \ub2e4\uc2dc \ubc31\ub3c4\ub20c \uc7a5\uad70(\u767d\u5ea6\u8a25\u5c07\u8ecd) \ud55c \uba85\uc744 \ub354 \ubc30\uce58\ud558\uc600\ub2e4 \ud55c\ub2e4. \ubabd\uace0\uc758 48\uac1c \ubd80\uc871 \uac00\uc6b4\ub370\uc11c \ub3d9\ubd81 \uc9c0\ubc29\uc5d0 \uac70\uc8fc\ud55c \uc871\uc18d\uc774 \uac00\uc7a5 \uac15\uc131\ud558\uc5ec \ub300\ube44\ub2ec\uc790(\u5927\u9f3b\u3e9a\u5b50)\ub294 \ud751\ub989\uac15 \ubd81\ucabd\uc5d0 \uc788\ub2e4. \uadf8\ub9ac\ud558\uc5ec \ub3d9\ucabd\uc73c\ub85c \ud751\ub8e1\uac15\uc5d0\uc11c\ubd80\ud130 \uc7a5\uc131 \ubc16\uc5d0\uae4c\uc9c0 \ubd81\ucabd\uc774\ub098 \uc11c\ucabd\uc774 \ubaa8\ub450 \ubabd\uace0\uc758 \uc601\ud1a0\uc774\uba70, \uadf8 \ub113\uc774\ub294 \uc911\uad6d\uc758 \uba87 \uac11\uc808\uc774\ub098 \ub41c\ub2e4. \uc11c\ub85c \uac15\ub300\ud55c \uc138\ub825\uc744 \uac00\uc9c0\uace0 \ud55c \uc9c0\uc5ed\uc744 \ucc28\uc9c0\ud558\uace0 \uc788\uc73c\uba70 \ub3d9\u318d\uc11c\u318d\ub0a8\u318d\ubd81\uc758 \ud669\uc81c\ub77c\uace0 \ubd80\ub978\ub2e4. \ud669\ud0dc\uadf9(\u9ec3\u592a\u6975)\uacfc \uccad\ud0dc\uadf9(\u9751\u592a\u6975)\uc740 \uc911\uad6d \uc11c\ub0a8\ucabd\uc5d0 \uc788\uc73c\uba70, \uc561\ub77c\uc0ac(\u5384\u7f85\u65af)\ub77c\ub294 \uac83\uc740 \uace7 \ub300\ube44(\u5927\u9f3b)\uc774\uace0, \uac1d\uc774\uac1d(\u5580\u723e\u5580)\uc774\ub77c\ub294 \uac83\uc740 \ub3d9\ubd81\ucabd\uc5d0 \uc788\ub2e4\uace0 \ud55c\ub2e4.'\uba70 \ub300\ub7b5\uc758 \uc704\uce58\ub97c \uae30\ub85d\ud558\uc600\ub2e4."} {"question": "\uc601\ub77d\uc81c\uc758 \ub9cc\ub144 \uce58\uc138\uc5d0 \ud070 \ud0c0\uaca9\uc744 \uc8fc\uace0 \ubbfc\uc911\ub4e4 \uc0ac\uc774\uc5d0\uc11c \ubbfc\ub2f4\uacfc \uc804\uc124\uc758 \uc18c\uc7ac\uac00 \ub41c \uc778\ubb3c\uc740 \ub204\uad6c\uc778\uac00?", "paragraph": "\ub098\uc911\uc5d0 \ub2f9\uc0c8\uc544\ub294 \uccb4\ud3ec\ub418\uc5b4 \ubaa9\uacfc \uc190\ubc1c\uc5d0 \ud615\uad6c\ub97c \uc50c\uc6b0\uace0 \uad75\uc740 \ucca0\uc0ac\ub85c \ubb36\uc5b4 \ub193\uc558\ub294\ub370, \ub9e4\ubcf5\ud55c \ub2f9\uc0c8\uc544\uc758 \uc794\ub2f9\uc774 \uadf8\ub140\ub97c \ud0c8\uc625\uc2dc\ucf30\ub2e4. \uc774\uc5d0 \uc2dc\uc911\uc5d0\ub294 \ub2f9\uc0c8\uc544\uac00 \uc694\uc220(\u5996\u8853)\uc744 \ubd80\ub824 \ubaa8\ub450 \ubc97\uc5b4 \ub358\uc9c0\uace0 \ub2ec\uc544\ub0ac\ub2e4\ub294 \uc804\uc124\uc774 \ub098\uc624\uac8c \ub418\uc5c8\ub2e4. \ud654\uac00 \ub09c \uc601\ub77d\uc81c\ub294 \"\uc0ad\ubc1c\ud558\uace0 \uc911\uc774 \ub418\uc5c8\uac70\ub098 \uc5ec\ub3c4\uc0ac(\u5973\u9053\u58eb) \ubb34\ub9ac\uc5d0 \ub2f9\uc0c8\uc544\uac00 \uc228\uc5b4 \uc788\uc744\uc9c0 \ubaa8\ub974\ub2c8 \uc0b0\ub465\uacfc \ub09c\uc9d5, \ubca0\uc774\uc9d5\uc758 \ube44\uad6c\ub2c8\ub4e4\uacfc \ucd9c\uac00\ud55c \ubd80\ub140\uc790\ub4e4\uc744 \ubaa8\uc870\ub9ac \uc7a1\uc544\ub4e4\uc774\ub77c\"\ub294 \uc870\uce59\uc744 \ub0b4\ub838\ub2e4. \uba85\ub098\ub77c \uac01\uc9c0\uc5d0\uc11c \uc218 \ub9cc\uba85\uc758 \ube44\uad6c\ub2c8 \uc5ec\uc2b9\uacfc \uc5ec\ub3c4\uc0ac\ub4e4\uc774 \uc7a1\ud600\uc654\uc73c\ub098 \uacb0\uad6d \ub2f9\uc0c8\uc544\ub294 \uc7a1\uc9c0 \ubabb\ud588\ub2e4. \uc774\ub294 \uadf8\uc758 \ub9cc\ub144 \uce58\uc138\uc5d0 \ud070 \ud0c0\uaca9\uc744 \uc8fc\uc5c8\uace0, \uc774\ub294 \ubbfc\uc911\ub4e4 \uc0ac\uc774\uc5d0 \ud68c\uc790\ud654\ub418\uc5b4 \ubbfc\ub2f4\uacfc \uc804\uc124\uc758 \uc18c\uc7ac\uac00 \ub418\uc5c8\ub2e4. \ud6c4\uc77c \uccad\ub098\ub77c \ub54c\uc758 \ubc31\ub828\uad50\ub3c4\uc758 \ub09c\uc740 \ub2f9\uc0c8\uc544\uc758 \ub09c\uacfc \ud64d\uac74\uc801\uc758 \ub09c\uc744 \ucc38\uace0\ud558\uc600\uc73c\uba70, \uccad\ub098\ub77c \ub54c\uc758 \uc791\uac00 \uc5ec\ub2a5(\u5442\u80fd)\uc740 \ub2f9\uc0c8\uc544\uc758 \ubc18\ub780\uc744 \uc18c\uc7ac\ub85c \ud558\uc5ec \uc18c\uc124 '\uc5ec\uc120\uc678\uc0ac'(\u5973\u4ed9\u5916\u53f2)\ub97c \uc4f0\uae30\ub3c4 \ud588\ub2e4. \ub2f9\uc0c8\uc544\uc758 \ub09c\uc740 \uc870\uc120\uc5d0\ub3c4 \uc54c\ub824\uc838 \uc778\uc870\uc2e4\ub85d \ub4f1\uc5d0\ub3c4 \uace0\uc0ac\ub85c \uc18c\uac1c\ub418\uae30\ub3c4 \ud558\uc600\ub2e4.", "answer": "\ub2f9\uc0c8\uc544", "sentence": "\ub098\uc911\uc5d0 \ub2f9\uc0c8\uc544 \ub294 \uccb4\ud3ec\ub418\uc5b4 \ubaa9\uacfc \uc190\ubc1c\uc5d0 \ud615\uad6c\ub97c \uc50c\uc6b0\uace0 \uad75\uc740 \ucca0\uc0ac\ub85c \ubb36\uc5b4 \ub193\uc558\ub294\ub370, \ub9e4\ubcf5\ud55c \ub2f9\uc0c8\uc544\uc758 \uc794\ub2f9\uc774 \uadf8\ub140\ub97c \ud0c8\uc625\uc2dc\ucf30\ub2e4.", "paragraph_sentence": " \ub098\uc911\uc5d0 \ub2f9\uc0c8\uc544 \ub294 \uccb4\ud3ec\ub418\uc5b4 \ubaa9\uacfc \uc190\ubc1c\uc5d0 \ud615\uad6c\ub97c \uc50c\uc6b0\uace0 \uad75\uc740 \ucca0\uc0ac\ub85c \ubb36\uc5b4 \ub193\uc558\ub294\ub370, \ub9e4\ubcf5\ud55c \ub2f9\uc0c8\uc544\uc758 \uc794\ub2f9\uc774 \uadf8\ub140\ub97c \ud0c8\uc625\uc2dc\ucf30\ub2e4. \uc774\uc5d0 \uc2dc\uc911\uc5d0\ub294 \ub2f9\uc0c8\uc544\uac00 \uc694\uc220(\u5996\u8853)\uc744 \ubd80\ub824 \ubaa8\ub450 \ubc97\uc5b4 \ub358\uc9c0\uace0 \ub2ec\uc544\ub0ac\ub2e4\ub294 \uc804\uc124\uc774 \ub098\uc624\uac8c \ub418\uc5c8\ub2e4. \ud654\uac00 \ub09c \uc601\ub77d\uc81c\ub294 \"\uc0ad\ubc1c\ud558\uace0 \uc911\uc774 \ub418\uc5c8\uac70\ub098 \uc5ec\ub3c4\uc0ac(\u5973\u9053\u58eb) \ubb34\ub9ac\uc5d0 \ub2f9\uc0c8\uc544\uac00 \uc228\uc5b4 \uc788\uc744\uc9c0 \ubaa8\ub974\ub2c8 \uc0b0\ub465\uacfc \ub09c\uc9d5, \ubca0\uc774\uc9d5\uc758 \ube44\uad6c\ub2c8\ub4e4\uacfc \ucd9c\uac00\ud55c \ubd80\ub140\uc790\ub4e4\uc744 \ubaa8\uc870\ub9ac \uc7a1\uc544\ub4e4\uc774\ub77c\"\ub294 \uc870\uce59\uc744 \ub0b4\ub838\ub2e4. \uba85\ub098\ub77c \uac01\uc9c0\uc5d0\uc11c \uc218 \ub9cc\uba85\uc758 \ube44\uad6c\ub2c8 \uc5ec\uc2b9\uacfc \uc5ec\ub3c4\uc0ac\ub4e4\uc774 \uc7a1\ud600\uc654\uc73c\ub098 \uacb0\uad6d \ub2f9\uc0c8\uc544\ub294 \uc7a1\uc9c0 \ubabb\ud588\ub2e4. \uc774\ub294 \uadf8\uc758 \ub9cc\ub144 \uce58\uc138\uc5d0 \ud070 \ud0c0\uaca9\uc744 \uc8fc\uc5c8\uace0, \uc774\ub294 \ubbfc\uc911\ub4e4 \uc0ac\uc774\uc5d0 \ud68c\uc790\ud654\ub418\uc5b4 \ubbfc\ub2f4\uacfc \uc804\uc124\uc758 \uc18c\uc7ac\uac00 \ub418\uc5c8\ub2e4. \ud6c4\uc77c \uccad\ub098\ub77c \ub54c\uc758 \ubc31\ub828\uad50\ub3c4\uc758 \ub09c\uc740 \ub2f9\uc0c8\uc544\uc758 \ub09c\uacfc \ud64d\uac74\uc801\uc758 \ub09c\uc744 \ucc38\uace0\ud558\uc600\uc73c\uba70, \uccad\ub098\ub77c \ub54c\uc758 \uc791\uac00 \uc5ec\ub2a5(\u5442\u80fd)\uc740 \ub2f9\uc0c8\uc544\uc758 \ubc18\ub780\uc744 \uc18c\uc7ac\ub85c \ud558\uc5ec \uc18c\uc124 '\uc5ec\uc120\uc678\uc0ac'(\u5973\u4ed9\u5916\u53f2)\ub97c \uc4f0\uae30\ub3c4 \ud588\ub2e4. \ub2f9\uc0c8\uc544\uc758 \ub09c\uc740 \uc870\uc120\uc5d0\ub3c4 \uc54c\ub824\uc838 \uc778\uc870\uc2e4\ub85d \ub4f1\uc5d0\ub3c4 \uace0\uc0ac\ub85c \uc18c\uac1c\ub418\uae30\ub3c4 \ud558\uc600\ub2e4.", "paragraph_answer": "\ub098\uc911\uc5d0 \ub2f9\uc0c8\uc544 \ub294 \uccb4\ud3ec\ub418\uc5b4 \ubaa9\uacfc \uc190\ubc1c\uc5d0 \ud615\uad6c\ub97c \uc50c\uc6b0\uace0 \uad75\uc740 \ucca0\uc0ac\ub85c \ubb36\uc5b4 \ub193\uc558\ub294\ub370, \ub9e4\ubcf5\ud55c \ub2f9\uc0c8\uc544\uc758 \uc794\ub2f9\uc774 \uadf8\ub140\ub97c \ud0c8\uc625\uc2dc\ucf30\ub2e4. \uc774\uc5d0 \uc2dc\uc911\uc5d0\ub294 \ub2f9\uc0c8\uc544\uac00 \uc694\uc220(\u5996\u8853)\uc744 \ubd80\ub824 \ubaa8\ub450 \ubc97\uc5b4 \ub358\uc9c0\uace0 \ub2ec\uc544\ub0ac\ub2e4\ub294 \uc804\uc124\uc774 \ub098\uc624\uac8c \ub418\uc5c8\ub2e4. \ud654\uac00 \ub09c \uc601\ub77d\uc81c\ub294 \"\uc0ad\ubc1c\ud558\uace0 \uc911\uc774 \ub418\uc5c8\uac70\ub098 \uc5ec\ub3c4\uc0ac(\u5973\u9053\u58eb) \ubb34\ub9ac\uc5d0 \ub2f9\uc0c8\uc544\uac00 \uc228\uc5b4 \uc788\uc744\uc9c0 \ubaa8\ub974\ub2c8 \uc0b0\ub465\uacfc \ub09c\uc9d5, \ubca0\uc774\uc9d5\uc758 \ube44\uad6c\ub2c8\ub4e4\uacfc \ucd9c\uac00\ud55c \ubd80\ub140\uc790\ub4e4\uc744 \ubaa8\uc870\ub9ac \uc7a1\uc544\ub4e4\uc774\ub77c\"\ub294 \uc870\uce59\uc744 \ub0b4\ub838\ub2e4. \uba85\ub098\ub77c \uac01\uc9c0\uc5d0\uc11c \uc218 \ub9cc\uba85\uc758 \ube44\uad6c\ub2c8 \uc5ec\uc2b9\uacfc \uc5ec\ub3c4\uc0ac\ub4e4\uc774 \uc7a1\ud600\uc654\uc73c\ub098 \uacb0\uad6d \ub2f9\uc0c8\uc544\ub294 \uc7a1\uc9c0 \ubabb\ud588\ub2e4. \uc774\ub294 \uadf8\uc758 \ub9cc\ub144 \uce58\uc138\uc5d0 \ud070 \ud0c0\uaca9\uc744 \uc8fc\uc5c8\uace0, \uc774\ub294 \ubbfc\uc911\ub4e4 \uc0ac\uc774\uc5d0 \ud68c\uc790\ud654\ub418\uc5b4 \ubbfc\ub2f4\uacfc \uc804\uc124\uc758 \uc18c\uc7ac\uac00 \ub418\uc5c8\ub2e4. \ud6c4\uc77c \uccad\ub098\ub77c \ub54c\uc758 \ubc31\ub828\uad50\ub3c4\uc758 \ub09c\uc740 \ub2f9\uc0c8\uc544\uc758 \ub09c\uacfc \ud64d\uac74\uc801\uc758 \ub09c\uc744 \ucc38\uace0\ud558\uc600\uc73c\uba70, \uccad\ub098\ub77c \ub54c\uc758 \uc791\uac00 \uc5ec\ub2a5(\u5442\u80fd)\uc740 \ub2f9\uc0c8\uc544\uc758 \ubc18\ub780\uc744 \uc18c\uc7ac\ub85c \ud558\uc5ec \uc18c\uc124 '\uc5ec\uc120\uc678\uc0ac'(\u5973\u4ed9\u5916\u53f2)\ub97c \uc4f0\uae30\ub3c4 \ud588\ub2e4. \ub2f9\uc0c8\uc544\uc758 \ub09c\uc740 \uc870\uc120\uc5d0\ub3c4 \uc54c\ub824\uc838 \uc778\uc870\uc2e4\ub85d \ub4f1\uc5d0\ub3c4 \uace0\uc0ac\ub85c \uc18c\uac1c\ub418\uae30\ub3c4 \ud558\uc600\ub2e4.", "sentence_answer": "\ub098\uc911\uc5d0 \ub2f9\uc0c8\uc544 \ub294 \uccb4\ud3ec\ub418\uc5b4 \ubaa9\uacfc \uc190\ubc1c\uc5d0 \ud615\uad6c\ub97c \uc50c\uc6b0\uace0 \uad75\uc740 \ucca0\uc0ac\ub85c \ubb36\uc5b4 \ub193\uc558\ub294\ub370, \ub9e4\ubcf5\ud55c \ub2f9\uc0c8\uc544\uc758 \uc794\ub2f9\uc774 \uadf8\ub140\ub97c \ud0c8\uc625\uc2dc\ucf30\ub2e4.", "paragraph_id": "6524266-5-0", "paragraph_question": "question: \uc601\ub77d\uc81c\uc758 \ub9cc\ub144 \uce58\uc138\uc5d0 \ud070 \ud0c0\uaca9\uc744 \uc8fc\uace0 \ubbfc\uc911\ub4e4 \uc0ac\uc774\uc5d0\uc11c \ubbfc\ub2f4\uacfc \uc804\uc124\uc758 \uc18c\uc7ac\uac00 \ub41c \uc778\ubb3c\uc740 \ub204\uad6c\uc778\uac00?, context: \ub098\uc911\uc5d0 \ub2f9\uc0c8\uc544\ub294 \uccb4\ud3ec\ub418\uc5b4 \ubaa9\uacfc \uc190\ubc1c\uc5d0 \ud615\uad6c\ub97c \uc50c\uc6b0\uace0 \uad75\uc740 \ucca0\uc0ac\ub85c \ubb36\uc5b4 \ub193\uc558\ub294\ub370, \ub9e4\ubcf5\ud55c \ub2f9\uc0c8\uc544\uc758 \uc794\ub2f9\uc774 \uadf8\ub140\ub97c \ud0c8\uc625\uc2dc\ucf30\ub2e4. \uc774\uc5d0 \uc2dc\uc911\uc5d0\ub294 \ub2f9\uc0c8\uc544\uac00 \uc694\uc220(\u5996\u8853)\uc744 \ubd80\ub824 \ubaa8\ub450 \ubc97\uc5b4 \ub358\uc9c0\uace0 \ub2ec\uc544\ub0ac\ub2e4\ub294 \uc804\uc124\uc774 \ub098\uc624\uac8c \ub418\uc5c8\ub2e4. \ud654\uac00 \ub09c \uc601\ub77d\uc81c\ub294 \"\uc0ad\ubc1c\ud558\uace0 \uc911\uc774 \ub418\uc5c8\uac70\ub098 \uc5ec\ub3c4\uc0ac(\u5973\u9053\u58eb) \ubb34\ub9ac\uc5d0 \ub2f9\uc0c8\uc544\uac00 \uc228\uc5b4 \uc788\uc744\uc9c0 \ubaa8\ub974\ub2c8 \uc0b0\ub465\uacfc \ub09c\uc9d5, \ubca0\uc774\uc9d5\uc758 \ube44\uad6c\ub2c8\ub4e4\uacfc \ucd9c\uac00\ud55c \ubd80\ub140\uc790\ub4e4\uc744 \ubaa8\uc870\ub9ac \uc7a1\uc544\ub4e4\uc774\ub77c\"\ub294 \uc870\uce59\uc744 \ub0b4\ub838\ub2e4. \uba85\ub098\ub77c \uac01\uc9c0\uc5d0\uc11c \uc218 \ub9cc\uba85\uc758 \ube44\uad6c\ub2c8 \uc5ec\uc2b9\uacfc \uc5ec\ub3c4\uc0ac\ub4e4\uc774 \uc7a1\ud600\uc654\uc73c\ub098 \uacb0\uad6d \ub2f9\uc0c8\uc544\ub294 \uc7a1\uc9c0 \ubabb\ud588\ub2e4. \uc774\ub294 \uadf8\uc758 \ub9cc\ub144 \uce58\uc138\uc5d0 \ud070 \ud0c0\uaca9\uc744 \uc8fc\uc5c8\uace0, \uc774\ub294 \ubbfc\uc911\ub4e4 \uc0ac\uc774\uc5d0 \ud68c\uc790\ud654\ub418\uc5b4 \ubbfc\ub2f4\uacfc \uc804\uc124\uc758 \uc18c\uc7ac\uac00 \ub418\uc5c8\ub2e4. \ud6c4\uc77c \uccad\ub098\ub77c \ub54c\uc758 \ubc31\ub828\uad50\ub3c4\uc758 \ub09c\uc740 \ub2f9\uc0c8\uc544\uc758 \ub09c\uacfc \ud64d\uac74\uc801\uc758 \ub09c\uc744 \ucc38\uace0\ud558\uc600\uc73c\uba70, \uccad\ub098\ub77c \ub54c\uc758 \uc791\uac00 \uc5ec\ub2a5(\u5442\u80fd)\uc740 \ub2f9\uc0c8\uc544\uc758 \ubc18\ub780\uc744 \uc18c\uc7ac\ub85c \ud558\uc5ec \uc18c\uc124 '\uc5ec\uc120\uc678\uc0ac'(\u5973\u4ed9\u5916\u53f2)\ub97c \uc4f0\uae30\ub3c4 \ud588\ub2e4. \ub2f9\uc0c8\uc544\uc758 \ub09c\uc740 \uc870\uc120\uc5d0\ub3c4 \uc54c\ub824\uc838 \uc778\uc870\uc2e4\ub85d \ub4f1\uc5d0\ub3c4 \uace0\uc0ac\ub85c \uc18c\uac1c\ub418\uae30\ub3c4 \ud558\uc600\ub2e4."} {"question": "\uac74\uc124\ud604\uc7a5 \uc0ac\uace0\ub85c \ubc15\uc6d0\uc21c\uc2dc\uc7a5\uacfc \uccad\uc6d0\uacbd\ucc30\uc758 \uac70\ubd80\ud589\uc704\ub85c \uc778\ud574 \ucda9\ub3cc\uc774 \uc788\uc5c8\ub358 \ub144\ub3c4\ub294?", "paragraph": "\uc0c8\ub204\ub9ac\ub2f9 \uad00\uacc4\uc790\ub4e4\uc740 2013\ub144 8\uc6d4 2\uc77c\uc5d0 \uc11c\uc6b8\ud2b9\ubcc4\uc2dc\uccad\uc744 \ubc29\ubb38\ud558\uace0 \ubc15\uc6d0\uc21c \uc2dc\uc7a5\uc5d0\uac8c \uac74\uc124\ud604\uc7a5 \uc0ac\uace0\uc5d0 \uad00\ud55c \ucc45\uc784\uc744 \ubb3b\ub294 \ud56d\uc758 \uc11c\ud55c\uc744 \uc81c\ucd9c\ud558\ub824\uace0 \ud558\uc600\uc73c\ub098, \ubc15\uc6d0\uc21c\uacfc \uccad\uc6d0\uacbd\ucc30\uc758 \uac70\ubd80\ud589\uc704\ub85c \uc778\ud55c \ucda9\ub3cc\uc774 \uc788\uc5c8\ub2e4. \ud76c\uc0dd\uc790 \uc720\uac00\uc871\ub4e4\uc744 \ube44\ub86f\ud55c \uc0c8\ub204\ub9ac\ub2f9 \uad00\uacc4\uc790\ub4e4\uc740 \ubc15\uc6d0\uc21c\uc5d0\uac8c \ud56d\uc758 \uc11c\ud55c\uc744 \uc804\ub2ec\ud558\ub824\ub294 \ubaa9\uc801\uc774 \uc788\uc5c8\uc73c\ub098, \uc774\ub97c \uac70\ubd80\ud55c \uc11c\uc6b8\uc2dc\uccad \uce21\uc740 \uccad\uc6d0\uacbd\ucc30\uc744 \ub3d9\uc6d0\ud558\uc5ec \ubc29\ubb38\uc744 \ub9c9\uc558\uace0 \uc774 \uacfc\uc815\uc5d0\uc11c \ubab8\uc2f8\uc6c0\uc774 \ubc8c\uc5b4\uc838 \uccad\uc6d0\uacbd\ucc30\uc774 \ubd80\uc0c1\uc744 \uc785\uc5c8\ub2e4. \uc774\uc5d0 \ub300\ud574 \uc11c\uc6b8\uc2dc \uce21\uc740, \uc11c\uc6b8\uc2dc \uc9c1\uc6d0\uc774 \uc0c1\ud574\ub97c \uc785\uc740 \uac83\uc5d0 \ub300\ud574 \uc720\uac10\uc758 \ub73b\uc744 \ud45c\uba85\ud558\uc600\ub2e4. \ubc18\uba74 \uae40\uc131\ud0dc \uc758\uc6d0\uc2e4 \uce21\uc740, \uc11c\uc6b8\uc2dc\ubbfc\uc758 \uc548\uc804\uc744 \uc9c0\ud0a4\uc9c0 \ubabb\ud558\uace0 \uc720\uac00\uc871\uacfc \ubbfc\uc6d0\uc778\uc758 \ud56d\uc758 \ubc29\ubb38\uc744 \ud68c\ud53c\ud55c \ubc15\uc6d0\uc21c \uc2dc\uc7a5\uc774 \ucc45\uc784\uc744 \uc838\uc57c\ud55c\ub2e4\uace0 \uc8fc\uc7a5\ud558\uc600\ub2e4. \uc774\ub7ec\ud55c \uc77c\ub828\uc758 \uc0ac\uac74 \uc911 \uae30\ud0c0 \uc2e0\ubb38\uae30\uc0ac\ub098 \uc778\ud130\ub137 \ub4f1\uc9c0\uc5d0\uc11c \uc0c8\ub204\ub9ac\ub2f9 \uad00\uacc4\uc790\ub97c \uae40\uc131\ud0dc \uc758\uc6d0\uc2e4 \uad00\uacc4\uc790\ub85c \uc798\ubabb \ud45c\uae30\ud558\ub294 \uc624\ubcf4\uac00 \ubc1c\uc0dd\ud588\ub2e4.", "answer": "2013\ub144", "sentence": "\uc0c8\ub204\ub9ac\ub2f9 \uad00\uacc4\uc790\ub4e4\uc740 2013\ub144 8\uc6d4 2\uc77c\uc5d0 \uc11c\uc6b8\ud2b9\ubcc4\uc2dc\uccad\uc744 \ubc29\ubb38\ud558\uace0 \ubc15\uc6d0\uc21c \uc2dc\uc7a5\uc5d0\uac8c \uac74\uc124\ud604\uc7a5 \uc0ac\uace0\uc5d0 \uad00\ud55c \ucc45\uc784\uc744 \ubb3b\ub294 \ud56d\uc758 \uc11c\ud55c\uc744 \uc81c\ucd9c\ud558\ub824\uace0 \ud558\uc600\uc73c\ub098, \ubc15\uc6d0\uc21c\uacfc \uccad\uc6d0\uacbd\ucc30\uc758 \uac70\ubd80\ud589\uc704\ub85c \uc778\ud55c \ucda9\ub3cc\uc774 \uc788\uc5c8\ub2e4.", "paragraph_sentence": " \uc0c8\ub204\ub9ac\ub2f9 \uad00\uacc4\uc790\ub4e4\uc740 2013\ub144 8\uc6d4 2\uc77c\uc5d0 \uc11c\uc6b8\ud2b9\ubcc4\uc2dc\uccad\uc744 \ubc29\ubb38\ud558\uace0 \ubc15\uc6d0\uc21c \uc2dc\uc7a5\uc5d0\uac8c \uac74\uc124\ud604\uc7a5 \uc0ac\uace0\uc5d0 \uad00\ud55c \ucc45\uc784\uc744 \ubb3b\ub294 \ud56d\uc758 \uc11c\ud55c\uc744 \uc81c\ucd9c\ud558\ub824\uace0 \ud558\uc600\uc73c\ub098, \ubc15\uc6d0\uc21c\uacfc \uccad\uc6d0\uacbd\ucc30\uc758 \uac70\ubd80\ud589\uc704\ub85c \uc778\ud55c \ucda9\ub3cc\uc774 \uc788\uc5c8\ub2e4. \ud76c\uc0dd\uc790 \uc720\uac00\uc871\ub4e4\uc744 \ube44\ub86f\ud55c \uc0c8\ub204\ub9ac\ub2f9 \uad00\uacc4\uc790\ub4e4\uc740 \ubc15\uc6d0\uc21c\uc5d0\uac8c \ud56d\uc758 \uc11c\ud55c\uc744 \uc804\ub2ec\ud558\ub824\ub294 \ubaa9\uc801\uc774 \uc788\uc5c8\uc73c\ub098, \uc774\ub97c \uac70\ubd80\ud55c \uc11c\uc6b8\uc2dc\uccad \uce21\uc740 \uccad\uc6d0\uacbd\ucc30\uc744 \ub3d9\uc6d0\ud558\uc5ec \ubc29\ubb38\uc744 \ub9c9\uc558\uace0 \uc774 \uacfc\uc815\uc5d0\uc11c \ubab8\uc2f8\uc6c0\uc774 \ubc8c\uc5b4\uc838 \uccad\uc6d0\uacbd\ucc30\uc774 \ubd80\uc0c1\uc744 \uc785\uc5c8\ub2e4. \uc774\uc5d0 \ub300\ud574 \uc11c\uc6b8\uc2dc \uce21\uc740, \uc11c\uc6b8\uc2dc \uc9c1\uc6d0\uc774 \uc0c1\ud574\ub97c \uc785\uc740 \uac83\uc5d0 \ub300\ud574 \uc720\uac10\uc758 \ub73b\uc744 \ud45c\uba85\ud558\uc600\ub2e4. \ubc18\uba74 \uae40\uc131\ud0dc \uc758\uc6d0\uc2e4 \uce21\uc740, \uc11c\uc6b8\uc2dc\ubbfc\uc758 \uc548\uc804\uc744 \uc9c0\ud0a4\uc9c0 \ubabb\ud558\uace0 \uc720\uac00\uc871\uacfc \ubbfc\uc6d0\uc778\uc758 \ud56d\uc758 \ubc29\ubb38\uc744 \ud68c\ud53c\ud55c \ubc15\uc6d0\uc21c \uc2dc\uc7a5\uc774 \ucc45\uc784\uc744 \uc838\uc57c\ud55c\ub2e4\uace0 \uc8fc\uc7a5\ud558\uc600\ub2e4. \uc774\ub7ec\ud55c \uc77c\ub828\uc758 \uc0ac\uac74 \uc911 \uae30\ud0c0 \uc2e0\ubb38\uae30\uc0ac\ub098 \uc778\ud130\ub137 \ub4f1\uc9c0\uc5d0\uc11c \uc0c8\ub204\ub9ac\ub2f9 \uad00\uacc4\uc790\ub97c \uae40\uc131\ud0dc \uc758\uc6d0\uc2e4 \uad00\uacc4\uc790\ub85c \uc798\ubabb \ud45c\uae30\ud558\ub294 \uc624\ubcf4\uac00 \ubc1c\uc0dd\ud588\ub2e4.", "paragraph_answer": "\uc0c8\ub204\ub9ac\ub2f9 \uad00\uacc4\uc790\ub4e4\uc740 2013\ub144 8\uc6d4 2\uc77c\uc5d0 \uc11c\uc6b8\ud2b9\ubcc4\uc2dc\uccad\uc744 \ubc29\ubb38\ud558\uace0 \ubc15\uc6d0\uc21c \uc2dc\uc7a5\uc5d0\uac8c \uac74\uc124\ud604\uc7a5 \uc0ac\uace0\uc5d0 \uad00\ud55c \ucc45\uc784\uc744 \ubb3b\ub294 \ud56d\uc758 \uc11c\ud55c\uc744 \uc81c\ucd9c\ud558\ub824\uace0 \ud558\uc600\uc73c\ub098, \ubc15\uc6d0\uc21c\uacfc \uccad\uc6d0\uacbd\ucc30\uc758 \uac70\ubd80\ud589\uc704\ub85c \uc778\ud55c \ucda9\ub3cc\uc774 \uc788\uc5c8\ub2e4. \ud76c\uc0dd\uc790 \uc720\uac00\uc871\ub4e4\uc744 \ube44\ub86f\ud55c \uc0c8\ub204\ub9ac\ub2f9 \uad00\uacc4\uc790\ub4e4\uc740 \ubc15\uc6d0\uc21c\uc5d0\uac8c \ud56d\uc758 \uc11c\ud55c\uc744 \uc804\ub2ec\ud558\ub824\ub294 \ubaa9\uc801\uc774 \uc788\uc5c8\uc73c\ub098, \uc774\ub97c \uac70\ubd80\ud55c \uc11c\uc6b8\uc2dc\uccad \uce21\uc740 \uccad\uc6d0\uacbd\ucc30\uc744 \ub3d9\uc6d0\ud558\uc5ec \ubc29\ubb38\uc744 \ub9c9\uc558\uace0 \uc774 \uacfc\uc815\uc5d0\uc11c \ubab8\uc2f8\uc6c0\uc774 \ubc8c\uc5b4\uc838 \uccad\uc6d0\uacbd\ucc30\uc774 \ubd80\uc0c1\uc744 \uc785\uc5c8\ub2e4. \uc774\uc5d0 \ub300\ud574 \uc11c\uc6b8\uc2dc \uce21\uc740, \uc11c\uc6b8\uc2dc \uc9c1\uc6d0\uc774 \uc0c1\ud574\ub97c \uc785\uc740 \uac83\uc5d0 \ub300\ud574 \uc720\uac10\uc758 \ub73b\uc744 \ud45c\uba85\ud558\uc600\ub2e4. \ubc18\uba74 \uae40\uc131\ud0dc \uc758\uc6d0\uc2e4 \uce21\uc740, \uc11c\uc6b8\uc2dc\ubbfc\uc758 \uc548\uc804\uc744 \uc9c0\ud0a4\uc9c0 \ubabb\ud558\uace0 \uc720\uac00\uc871\uacfc \ubbfc\uc6d0\uc778\uc758 \ud56d\uc758 \ubc29\ubb38\uc744 \ud68c\ud53c\ud55c \ubc15\uc6d0\uc21c \uc2dc\uc7a5\uc774 \ucc45\uc784\uc744 \uc838\uc57c\ud55c\ub2e4\uace0 \uc8fc\uc7a5\ud558\uc600\ub2e4. \uc774\ub7ec\ud55c \uc77c\ub828\uc758 \uc0ac\uac74 \uc911 \uae30\ud0c0 \uc2e0\ubb38\uae30\uc0ac\ub098 \uc778\ud130\ub137 \ub4f1\uc9c0\uc5d0\uc11c \uc0c8\ub204\ub9ac\ub2f9 \uad00\uacc4\uc790\ub97c \uae40\uc131\ud0dc \uc758\uc6d0\uc2e4 \uad00\uacc4\uc790\ub85c \uc798\ubabb \ud45c\uae30\ud558\ub294 \uc624\ubcf4\uac00 \ubc1c\uc0dd\ud588\ub2e4.", "sentence_answer": "\uc0c8\ub204\ub9ac\ub2f9 \uad00\uacc4\uc790\ub4e4\uc740 2013\ub144 8\uc6d4 2\uc77c\uc5d0 \uc11c\uc6b8\ud2b9\ubcc4\uc2dc\uccad\uc744 \ubc29\ubb38\ud558\uace0 \ubc15\uc6d0\uc21c \uc2dc\uc7a5\uc5d0\uac8c \uac74\uc124\ud604\uc7a5 \uc0ac\uace0\uc5d0 \uad00\ud55c \ucc45\uc784\uc744 \ubb3b\ub294 \ud56d\uc758 \uc11c\ud55c\uc744 \uc81c\ucd9c\ud558\ub824\uace0 \ud558\uc600\uc73c\ub098, \ubc15\uc6d0\uc21c\uacfc \uccad\uc6d0\uacbd\ucc30\uc758 \uac70\ubd80\ud589\uc704\ub85c \uc778\ud55c \ucda9\ub3cc\uc774 \uc788\uc5c8\ub2e4.", "paragraph_id": "6577067-1-2", "paragraph_question": "question: \uac74\uc124\ud604\uc7a5 \uc0ac\uace0\ub85c \ubc15\uc6d0\uc21c\uc2dc\uc7a5\uacfc \uccad\uc6d0\uacbd\ucc30\uc758 \uac70\ubd80\ud589\uc704\ub85c \uc778\ud574 \ucda9\ub3cc\uc774 \uc788\uc5c8\ub358 \ub144\ub3c4\ub294?, context: \uc0c8\ub204\ub9ac\ub2f9 \uad00\uacc4\uc790\ub4e4\uc740 2013\ub144 8\uc6d4 2\uc77c\uc5d0 \uc11c\uc6b8\ud2b9\ubcc4\uc2dc\uccad\uc744 \ubc29\ubb38\ud558\uace0 \ubc15\uc6d0\uc21c \uc2dc\uc7a5\uc5d0\uac8c \uac74\uc124\ud604\uc7a5 \uc0ac\uace0\uc5d0 \uad00\ud55c \ucc45\uc784\uc744 \ubb3b\ub294 \ud56d\uc758 \uc11c\ud55c\uc744 \uc81c\ucd9c\ud558\ub824\uace0 \ud558\uc600\uc73c\ub098, \ubc15\uc6d0\uc21c\uacfc \uccad\uc6d0\uacbd\ucc30\uc758 \uac70\ubd80\ud589\uc704\ub85c \uc778\ud55c \ucda9\ub3cc\uc774 \uc788\uc5c8\ub2e4. \ud76c\uc0dd\uc790 \uc720\uac00\uc871\ub4e4\uc744 \ube44\ub86f\ud55c \uc0c8\ub204\ub9ac\ub2f9 \uad00\uacc4\uc790\ub4e4\uc740 \ubc15\uc6d0\uc21c\uc5d0\uac8c \ud56d\uc758 \uc11c\ud55c\uc744 \uc804\ub2ec\ud558\ub824\ub294 \ubaa9\uc801\uc774 \uc788\uc5c8\uc73c\ub098, \uc774\ub97c \uac70\ubd80\ud55c \uc11c\uc6b8\uc2dc\uccad \uce21\uc740 \uccad\uc6d0\uacbd\ucc30\uc744 \ub3d9\uc6d0\ud558\uc5ec \ubc29\ubb38\uc744 \ub9c9\uc558\uace0 \uc774 \uacfc\uc815\uc5d0\uc11c \ubab8\uc2f8\uc6c0\uc774 \ubc8c\uc5b4\uc838 \uccad\uc6d0\uacbd\ucc30\uc774 \ubd80\uc0c1\uc744 \uc785\uc5c8\ub2e4. \uc774\uc5d0 \ub300\ud574 \uc11c\uc6b8\uc2dc \uce21\uc740, \uc11c\uc6b8\uc2dc \uc9c1\uc6d0\uc774 \uc0c1\ud574\ub97c \uc785\uc740 \uac83\uc5d0 \ub300\ud574 \uc720\uac10\uc758 \ub73b\uc744 \ud45c\uba85\ud558\uc600\ub2e4. \ubc18\uba74 \uae40\uc131\ud0dc \uc758\uc6d0\uc2e4 \uce21\uc740, \uc11c\uc6b8\uc2dc\ubbfc\uc758 \uc548\uc804\uc744 \uc9c0\ud0a4\uc9c0 \ubabb\ud558\uace0 \uc720\uac00\uc871\uacfc \ubbfc\uc6d0\uc778\uc758 \ud56d\uc758 \ubc29\ubb38\uc744 \ud68c\ud53c\ud55c \ubc15\uc6d0\uc21c \uc2dc\uc7a5\uc774 \ucc45\uc784\uc744 \uc838\uc57c\ud55c\ub2e4\uace0 \uc8fc\uc7a5\ud558\uc600\ub2e4. \uc774\ub7ec\ud55c \uc77c\ub828\uc758 \uc0ac\uac74 \uc911 \uae30\ud0c0 \uc2e0\ubb38\uae30\uc0ac\ub098 \uc778\ud130\ub137 \ub4f1\uc9c0\uc5d0\uc11c \uc0c8\ub204\ub9ac\ub2f9 \uad00\uacc4\uc790\ub97c \uae40\uc131\ud0dc \uc758\uc6d0\uc2e4 \uad00\uacc4\uc790\ub85c \uc798\ubabb \ud45c\uae30\ud558\ub294 \uc624\ubcf4\uac00 \ubc1c\uc0dd\ud588\ub2e4."} {"question": "\ucc9c\ub3c4 \ucc45\uc784\uc790\ub85c \uc624\ud1a0\ubaa8 \uc528\uc5d0\uac8c \uc554\uc0b4\ub2f9\ud55c \uc778\ubb3c\uc740?", "paragraph": "\uc57c\uce74\ubaa8\uce58\ub294 \ub374\uc624(\u5929\u5fdc) 2\ub144(782\ub144)\uc5d0 \ubc1c\uc0dd\ud55c \ud788\uce74\ubbf8\ub178 \uac00\uc640\uc4f0\uae30(\u6c37\u4e0a\u5ddd\u7d99)\uc758 \ub09c\uc5d0 \uc5f0\uc88c\ub2f9\ud574 \uad00\uc9c1\uc5d0\uc11c \ud574\uc784\ub418\uae30\ub3c4 \ud588\uc9c0\ub9cc, \ucd5c\uc885\uc801\uc73c\ub85c\ub294 \uac04\ubb34(\u6853\u6b66) \ucd08\uae30\uc5d0 \uc911\ub0a9\uc5b8(\u4e2d\u7d0d\u8a00)\uae4c\uc9c0 \uc62c\ub790\ub2e4. \uadf8\ub7ec\ub098 \uc5d4\ub7b4\ucfe0(\u5ef6\u66a6) 3\ub144(784\ub144) \ucc9c\ud669\uc774 \ub098\uac00\uc624\uce74\ucfc4(\u9577\u5ca1\u4eac) \ucc9c\ub3c4\ub97c \uc2e4\ud589\uc5d0 \uc62e\uae30\ub294\ub370 \ubd88\ub9cc\uc744 \ud488\uc740 \uc624\ud1a0\ubaa8 \uc528\ub294 \ucc9c\ub3c4 \ucc45\uc784\uc790\uc600\ub358 \uc911\ub0a9\uc5b8 \ud6c4\uc9c0\uc640\ub77c\ub178 \ub2e4\ub124\uc4f0\uad6c(\u85e4\u539f\u7a2e\u7d99)\ub97c \uc554\uc0b4\ud558\ub294 \uc0ac\uac74\uc744 \uc77c\uc73c\ud0a8\ub2e4. \uc0ac\uac74 \ub4a4\uc5d0 \uc624\ud1a0\ubaa8\ub178 \uace0\ub9c8\ub85c\uc758 \uc544\ub4e4\uc778 \uc4f0\uad6c\ud788\ud1a0(\u7d99\u4eba)\ub294 \ub09c\uc758 \uc8fc\ubaa8\uc790\ub85c\uc368 \ucc98\ud615\ub2f9\ud588\uace0, \uc9c1\uc804\uc5d0 \uc0ac\ub9dd\ud588\ub358 \uc57c\uce74\ubaa8\uce58\ub3c4 \uc81c\uba85\ub2f9\ud588\ub2e4. \uadf8\ub7f0 \ud55c\ud3b8\uc73c\ub85c \ud5e4\uc774\uc548 \ucd08\uae30\uc5d0 \uac04\ubb34 \ucc9c\ud669\uc758 \uba85\uc744 \ubc1b\ub4e4\uc5b4 \uc138\uc774\uc774\ud0c0\uc774\uc1fc\uad70(\u5f81\u5937\u5927\u5c06\u8ecd)\uc774 \ub418\uc5b4 \uc5d0\ubbf8\uc2dc(\u8766\u5937) \uc815\ubc8c\uc5d0 \uacf5\uc744 \uc138\uc6b0\uace0 \uc8853\uc704 \uad00\uc704\ub97c \ubc1b\uc740 \uc624\ud1a0\ubaa8\ub178 \uc624\ud1a0\ub9c8\ub85c(\u5927\u4f34\u5f1f\u9ebb\u5442)\ub098, \ud6c4\uc9c0\uc640\ub77c\ub178 \ub2e4\ub124\uc4f0\uad6c \uc554\uc0b4 \uc0ac\uac74\uc758 \uc8fc\ubaa8\uc790\ub85c \ucc98\ud615\ub41c \uc4f0\uad6c\ud788\ud1a0\uc758 \uc544\ub4e4\ub85c \uc5b4\ub9b0 \ub098\uc774\uc5d0 \uc720\ubc30\ub418\uc5c8\ub2e4\uac00 \uc0ac\uba74\ub41c \ub4a4 \uad00\uc9c1\uc5d0 \uc62c\ub77c \uc5c5\uc801\uc744 \uc138\uc6b0\uace0 \uc0b0\uae30(\u53c2\u8b70)\uac00 \ub41c \uc624\ud1a0\ubaa8\ub178 \uad6c\ub2c8\ubbf8\uce58(\u5927\u4f34\u56fd\u9053) \ub4f1\uc758 \uad6c\uad50\uac00 \ubc30\ucd9c\ub418\ub294 \ub4f1 \uc9d1\uc548\uc758 \uc9c0\uc704\ub294 \ubcf4\uc874\ub418\uc5c8\ub2e4.", "answer": "\ud6c4\uc9c0\uc640\ub77c\ub178 \ub2e4\ub124\uc4f0\uad6c", "sentence": "\uadf8\ub7ec\ub098 \uc5d4\ub7b4\ucfe0(\u5ef6\u66a6) 3\ub144(784\ub144) \ucc9c\ud669\uc774 \ub098\uac00\uc624\uce74\ucfc4(\u9577\u5ca1\u4eac) \ucc9c\ub3c4\ub97c \uc2e4\ud589\uc5d0 \uc62e\uae30\ub294\ub370 \ubd88\ub9cc\uc744 \ud488\uc740 \uc624\ud1a0\ubaa8 \uc528\ub294 \ucc9c\ub3c4 \ucc45\uc784\uc790\uc600\ub358 \uc911\ub0a9\uc5b8 \ud6c4\uc9c0\uc640\ub77c\ub178 \ub2e4\ub124\uc4f0\uad6c (\u85e4\u539f\u7a2e\u7d99)\ub97c \uc554\uc0b4\ud558\ub294 \uc0ac\uac74\uc744 \uc77c\uc73c\ud0a8\ub2e4.", "paragraph_sentence": "\uc57c\uce74\ubaa8\uce58\ub294 \ub374\uc624(\u5929\u5fdc) 2\ub144(782\ub144)\uc5d0 \ubc1c\uc0dd\ud55c \ud788\uce74\ubbf8\ub178 \uac00\uc640\uc4f0\uae30(\u6c37\u4e0a\u5ddd\u7d99)\uc758 \ub09c\uc5d0 \uc5f0\uc88c\ub2f9\ud574 \uad00\uc9c1\uc5d0\uc11c \ud574\uc784\ub418\uae30\ub3c4 \ud588\uc9c0\ub9cc, \ucd5c\uc885\uc801\uc73c\ub85c\ub294 \uac04\ubb34(\u6853\u6b66) \ucd08\uae30\uc5d0 \uc911\ub0a9\uc5b8(\u4e2d\u7d0d\u8a00)\uae4c\uc9c0 \uc62c\ub790\ub2e4. \uadf8\ub7ec\ub098 \uc5d4\ub7b4\ucfe0(\u5ef6\u66a6) 3\ub144(784\ub144) \ucc9c\ud669\uc774 \ub098\uac00\uc624\uce74\ucfc4(\u9577\u5ca1\u4eac) \ucc9c\ub3c4\ub97c \uc2e4\ud589\uc5d0 \uc62e\uae30\ub294\ub370 \ubd88\ub9cc\uc744 \ud488\uc740 \uc624\ud1a0\ubaa8 \uc528\ub294 \ucc9c\ub3c4 \ucc45\uc784\uc790\uc600\ub358 \uc911\ub0a9\uc5b8 \ud6c4\uc9c0\uc640\ub77c\ub178 \ub2e4\ub124\uc4f0\uad6c (\u85e4\u539f\u7a2e\u7d99)\ub97c \uc554\uc0b4\ud558\ub294 \uc0ac\uac74\uc744 \uc77c\uc73c\ud0a8\ub2e4. \uc0ac\uac74 \ub4a4\uc5d0 \uc624\ud1a0\ubaa8\ub178 \uace0\ub9c8\ub85c\uc758 \uc544\ub4e4\uc778 \uc4f0\uad6c\ud788\ud1a0(\u7d99\u4eba)\ub294 \ub09c\uc758 \uc8fc\ubaa8\uc790\ub85c\uc368 \ucc98\ud615\ub2f9\ud588\uace0, \uc9c1\uc804\uc5d0 \uc0ac\ub9dd\ud588\ub358 \uc57c\uce74\ubaa8\uce58\ub3c4 \uc81c\uba85\ub2f9\ud588\ub2e4. \uadf8\ub7f0 \ud55c\ud3b8\uc73c\ub85c \ud5e4\uc774\uc548 \ucd08\uae30\uc5d0 \uac04\ubb34 \ucc9c\ud669\uc758 \uba85\uc744 \ubc1b\ub4e4\uc5b4 \uc138\uc774\uc774\ud0c0\uc774\uc1fc\uad70(\u5f81\u5937\u5927\u5c06\u8ecd)\uc774 \ub418\uc5b4 \uc5d0\ubbf8\uc2dc(\u8766\u5937) \uc815\ubc8c\uc5d0 \uacf5\uc744 \uc138\uc6b0\uace0 \uc8853\uc704 \uad00\uc704\ub97c \ubc1b\uc740 \uc624\ud1a0\ubaa8\ub178 \uc624\ud1a0\ub9c8\ub85c(\u5927\u4f34\u5f1f\u9ebb\u5442)\ub098, \ud6c4\uc9c0\uc640\ub77c\ub178 \ub2e4\ub124\uc4f0\uad6c \uc554\uc0b4 \uc0ac\uac74\uc758 \uc8fc\ubaa8\uc790\ub85c \ucc98\ud615\ub41c \uc4f0\uad6c\ud788\ud1a0\uc758 \uc544\ub4e4\ub85c \uc5b4\ub9b0 \ub098\uc774\uc5d0 \uc720\ubc30\ub418\uc5c8\ub2e4\uac00 \uc0ac\uba74\ub41c \ub4a4 \uad00\uc9c1\uc5d0 \uc62c\ub77c \uc5c5\uc801\uc744 \uc138\uc6b0\uace0 \uc0b0\uae30(\u53c2\u8b70)\uac00 \ub41c \uc624\ud1a0\ubaa8\ub178 \uad6c\ub2c8\ubbf8\uce58(\u5927\u4f34\u56fd\u9053) \ub4f1\uc758 \uad6c\uad50\uac00 \ubc30\ucd9c\ub418\ub294 \ub4f1 \uc9d1\uc548\uc758 \uc9c0\uc704\ub294 \ubcf4\uc874\ub418\uc5c8\ub2e4.", "paragraph_answer": "\uc57c\uce74\ubaa8\uce58\ub294 \ub374\uc624(\u5929\u5fdc) 2\ub144(782\ub144)\uc5d0 \ubc1c\uc0dd\ud55c \ud788\uce74\ubbf8\ub178 \uac00\uc640\uc4f0\uae30(\u6c37\u4e0a\u5ddd\u7d99)\uc758 \ub09c\uc5d0 \uc5f0\uc88c\ub2f9\ud574 \uad00\uc9c1\uc5d0\uc11c \ud574\uc784\ub418\uae30\ub3c4 \ud588\uc9c0\ub9cc, \ucd5c\uc885\uc801\uc73c\ub85c\ub294 \uac04\ubb34(\u6853\u6b66) \ucd08\uae30\uc5d0 \uc911\ub0a9\uc5b8(\u4e2d\u7d0d\u8a00)\uae4c\uc9c0 \uc62c\ub790\ub2e4. \uadf8\ub7ec\ub098 \uc5d4\ub7b4\ucfe0(\u5ef6\u66a6) 3\ub144(784\ub144) \ucc9c\ud669\uc774 \ub098\uac00\uc624\uce74\ucfc4(\u9577\u5ca1\u4eac) \ucc9c\ub3c4\ub97c \uc2e4\ud589\uc5d0 \uc62e\uae30\ub294\ub370 \ubd88\ub9cc\uc744 \ud488\uc740 \uc624\ud1a0\ubaa8 \uc528\ub294 \ucc9c\ub3c4 \ucc45\uc784\uc790\uc600\ub358 \uc911\ub0a9\uc5b8 \ud6c4\uc9c0\uc640\ub77c\ub178 \ub2e4\ub124\uc4f0\uad6c (\u85e4\u539f\u7a2e\u7d99)\ub97c \uc554\uc0b4\ud558\ub294 \uc0ac\uac74\uc744 \uc77c\uc73c\ud0a8\ub2e4. \uc0ac\uac74 \ub4a4\uc5d0 \uc624\ud1a0\ubaa8\ub178 \uace0\ub9c8\ub85c\uc758 \uc544\ub4e4\uc778 \uc4f0\uad6c\ud788\ud1a0(\u7d99\u4eba)\ub294 \ub09c\uc758 \uc8fc\ubaa8\uc790\ub85c\uc368 \ucc98\ud615\ub2f9\ud588\uace0, \uc9c1\uc804\uc5d0 \uc0ac\ub9dd\ud588\ub358 \uc57c\uce74\ubaa8\uce58\ub3c4 \uc81c\uba85\ub2f9\ud588\ub2e4. \uadf8\ub7f0 \ud55c\ud3b8\uc73c\ub85c \ud5e4\uc774\uc548 \ucd08\uae30\uc5d0 \uac04\ubb34 \ucc9c\ud669\uc758 \uba85\uc744 \ubc1b\ub4e4\uc5b4 \uc138\uc774\uc774\ud0c0\uc774\uc1fc\uad70(\u5f81\u5937\u5927\u5c06\u8ecd)\uc774 \ub418\uc5b4 \uc5d0\ubbf8\uc2dc(\u8766\u5937) \uc815\ubc8c\uc5d0 \uacf5\uc744 \uc138\uc6b0\uace0 \uc8853\uc704 \uad00\uc704\ub97c \ubc1b\uc740 \uc624\ud1a0\ubaa8\ub178 \uc624\ud1a0\ub9c8\ub85c(\u5927\u4f34\u5f1f\u9ebb\u5442)\ub098, \ud6c4\uc9c0\uc640\ub77c\ub178 \ub2e4\ub124\uc4f0\uad6c \uc554\uc0b4 \uc0ac\uac74\uc758 \uc8fc\ubaa8\uc790\ub85c \ucc98\ud615\ub41c \uc4f0\uad6c\ud788\ud1a0\uc758 \uc544\ub4e4\ub85c \uc5b4\ub9b0 \ub098\uc774\uc5d0 \uc720\ubc30\ub418\uc5c8\ub2e4\uac00 \uc0ac\uba74\ub41c \ub4a4 \uad00\uc9c1\uc5d0 \uc62c\ub77c \uc5c5\uc801\uc744 \uc138\uc6b0\uace0 \uc0b0\uae30(\u53c2\u8b70)\uac00 \ub41c \uc624\ud1a0\ubaa8\ub178 \uad6c\ub2c8\ubbf8\uce58(\u5927\u4f34\u56fd\u9053) \ub4f1\uc758 \uad6c\uad50\uac00 \ubc30\ucd9c\ub418\ub294 \ub4f1 \uc9d1\uc548\uc758 \uc9c0\uc704\ub294 \ubcf4\uc874\ub418\uc5c8\ub2e4.", "sentence_answer": "\uadf8\ub7ec\ub098 \uc5d4\ub7b4\ucfe0(\u5ef6\u66a6) 3\ub144(784\ub144) \ucc9c\ud669\uc774 \ub098\uac00\uc624\uce74\ucfc4(\u9577\u5ca1\u4eac) \ucc9c\ub3c4\ub97c \uc2e4\ud589\uc5d0 \uc62e\uae30\ub294\ub370 \ubd88\ub9cc\uc744 \ud488\uc740 \uc624\ud1a0\ubaa8 \uc528\ub294 \ucc9c\ub3c4 \ucc45\uc784\uc790\uc600\ub358 \uc911\ub0a9\uc5b8 \ud6c4\uc9c0\uc640\ub77c\ub178 \ub2e4\ub124\uc4f0\uad6c (\u85e4\u539f\u7a2e\u7d99)\ub97c \uc554\uc0b4\ud558\ub294 \uc0ac\uac74\uc744 \uc77c\uc73c\ud0a8\ub2e4.", "paragraph_id": "6657437-2-1", "paragraph_question": "question: \ucc9c\ub3c4 \ucc45\uc784\uc790\ub85c \uc624\ud1a0\ubaa8 \uc528\uc5d0\uac8c \uc554\uc0b4\ub2f9\ud55c \uc778\ubb3c\uc740?, context: \uc57c\uce74\ubaa8\uce58\ub294 \ub374\uc624(\u5929\u5fdc) 2\ub144(782\ub144)\uc5d0 \ubc1c\uc0dd\ud55c \ud788\uce74\ubbf8\ub178 \uac00\uc640\uc4f0\uae30(\u6c37\u4e0a\u5ddd\u7d99)\uc758 \ub09c\uc5d0 \uc5f0\uc88c\ub2f9\ud574 \uad00\uc9c1\uc5d0\uc11c \ud574\uc784\ub418\uae30\ub3c4 \ud588\uc9c0\ub9cc, \ucd5c\uc885\uc801\uc73c\ub85c\ub294 \uac04\ubb34(\u6853\u6b66) \ucd08\uae30\uc5d0 \uc911\ub0a9\uc5b8(\u4e2d\u7d0d\u8a00)\uae4c\uc9c0 \uc62c\ub790\ub2e4. \uadf8\ub7ec\ub098 \uc5d4\ub7b4\ucfe0(\u5ef6\u66a6) 3\ub144(784\ub144) \ucc9c\ud669\uc774 \ub098\uac00\uc624\uce74\ucfc4(\u9577\u5ca1\u4eac) \ucc9c\ub3c4\ub97c \uc2e4\ud589\uc5d0 \uc62e\uae30\ub294\ub370 \ubd88\ub9cc\uc744 \ud488\uc740 \uc624\ud1a0\ubaa8 \uc528\ub294 \ucc9c\ub3c4 \ucc45\uc784\uc790\uc600\ub358 \uc911\ub0a9\uc5b8 \ud6c4\uc9c0\uc640\ub77c\ub178 \ub2e4\ub124\uc4f0\uad6c(\u85e4\u539f\u7a2e\u7d99)\ub97c \uc554\uc0b4\ud558\ub294 \uc0ac\uac74\uc744 \uc77c\uc73c\ud0a8\ub2e4. \uc0ac\uac74 \ub4a4\uc5d0 \uc624\ud1a0\ubaa8\ub178 \uace0\ub9c8\ub85c\uc758 \uc544\ub4e4\uc778 \uc4f0\uad6c\ud788\ud1a0(\u7d99\u4eba)\ub294 \ub09c\uc758 \uc8fc\ubaa8\uc790\ub85c\uc368 \ucc98\ud615\ub2f9\ud588\uace0, \uc9c1\uc804\uc5d0 \uc0ac\ub9dd\ud588\ub358 \uc57c\uce74\ubaa8\uce58\ub3c4 \uc81c\uba85\ub2f9\ud588\ub2e4. \uadf8\ub7f0 \ud55c\ud3b8\uc73c\ub85c \ud5e4\uc774\uc548 \ucd08\uae30\uc5d0 \uac04\ubb34 \ucc9c\ud669\uc758 \uba85\uc744 \ubc1b\ub4e4\uc5b4 \uc138\uc774\uc774\ud0c0\uc774\uc1fc\uad70(\u5f81\u5937\u5927\u5c06\u8ecd)\uc774 \ub418\uc5b4 \uc5d0\ubbf8\uc2dc(\u8766\u5937) \uc815\ubc8c\uc5d0 \uacf5\uc744 \uc138\uc6b0\uace0 \uc8853\uc704 \uad00\uc704\ub97c \ubc1b\uc740 \uc624\ud1a0\ubaa8\ub178 \uc624\ud1a0\ub9c8\ub85c(\u5927\u4f34\u5f1f\u9ebb\u5442)\ub098, \ud6c4\uc9c0\uc640\ub77c\ub178 \ub2e4\ub124\uc4f0\uad6c \uc554\uc0b4 \uc0ac\uac74\uc758 \uc8fc\ubaa8\uc790\ub85c \ucc98\ud615\ub41c \uc4f0\uad6c\ud788\ud1a0\uc758 \uc544\ub4e4\ub85c \uc5b4\ub9b0 \ub098\uc774\uc5d0 \uc720\ubc30\ub418\uc5c8\ub2e4\uac00 \uc0ac\uba74\ub41c \ub4a4 \uad00\uc9c1\uc5d0 \uc62c\ub77c \uc5c5\uc801\uc744 \uc138\uc6b0\uace0 \uc0b0\uae30(\u53c2\u8b70)\uac00 \ub41c \uc624\ud1a0\ubaa8\ub178 \uad6c\ub2c8\ubbf8\uce58(\u5927\u4f34\u56fd\u9053) \ub4f1\uc758 \uad6c\uad50\uac00 \ubc30\ucd9c\ub418\ub294 \ub4f1 \uc9d1\uc548\uc758 \uc9c0\uc704\ub294 \ubcf4\uc874\ub418\uc5c8\ub2e4."} {"question": "1989\ub144 \uac00\uc11d\ubc29\ub41c \uc815\uc778\uc219\uc758 \uc624\ube60\uc774\uba70 \uc815\uc131\uc77c\uc774 \uc815\uc77c\uad8c\uc758 \uc544\ub4e4\uc774\ub77c\uace0 \ub9d0\ud55c \uc0ac\ub78c\uc740?", "paragraph": "1989\ub144 \uac00\uc11d\ubc29\ub41c \uc815\uc778\uc219\uc758 \uc624\ube60 \uc815\uc885\uc6b1\uc740 \uc815\uc131\uc77c\uc774 \uc815\uc77c\uad8c\uc758 \uc544\ub4e4\uc774 \ud2c0\ub9bc\uc5c6\ub2e4\uace0 \ub9d0\ud55c\ub2e4. \uadf8\uc5d0 \uc758\ud558\uba74 '\uc815\uc77c\uad8c\uc774 \ud55c \ub2ec\uc5d0 \ud55c \ubc88 \uaf34\ub85c \uc11c\uad50\ub3d9 \uc9d1\uc73c\ub85c \ucc3e\uc544\uc654\uc9c0\uc694. \uadf8\ub9ac\uace0 \ub3d9\uc0dd\uc774 '\uadf8\uc774\uac00 \uc544\ub4e4 \ud558\ub098\ub9cc \ub0b3\uc544\ub2ec\ub77c\uace0 \ud55c\ub2e4'\uba70 \uc0c1\uc758\ub97c \ud574 \ucc98\uc74c\uc5d4 \uac00\uc871\ub4e4\uc774 \uadf9\uad6c \ubc18\ub300\ub97c \ud588\uc9c0\uc694. \ucd9c\uc0b0 \ud6c4\uc5d0\ub294 \uc77c \uc8fc\uc77c\uc5d0 \ud55c\ubc88 \uaf34\ub85c \ucc3e\uc544\uc640 \uc131\uc77c\uc774\ub97c \uc548\uace0 \uc990\uac70\uc6cc\ud588\uc73c\uba70, \ub298\uadf8\ub9c9\uc5d0 \uc544\ub4e4\uc744 \uc5bb\uc5b4 \uc18c\uc6d0\uc744 \uc774\ub918\ub2e4\ub294 \ub73b\uc5d0\uc11c \uc131\uc77c\uc774\ub780 \uc774\ub984\uae4c\uc9c0 \uc9c1\uc811 \uc9c0\uc5b4\uc654\uc5b4\uc694. \uc131\uc77c\uc774\ub3c4 \uc790\uae30 \uc544\ube60\uac00 TV\uc5d0 \ube44\uce58\uba74 '\uc544\ube60'\ub77c\uace0 \uc18c\ub9ac\uce58\uba70 \uc88b\uc544\ud588\uc9c0\uc694.'\ub77c\ub294 \uac83\uc774\ub2e4. \uc815\uc885\uc6b1\uc740 \ud615\uc744 \ub9c8\uce58\uace0 \uad50\ub3c4\uc18c\uc5d0\uc11c \ucd9c\uac10\ud55c \ub4a4\uc5d0 \uae30\uc790 \uc778\ud130\ubdf0\uc5d0\uc11c \uc790\uc2e0\uc740 \ubc94\uc778\uc774 \uc544\ub2c8\ub77c\uba70 \uc5b5\uc6b8\ud568\uc744 \ud638\uc18c\ud558\uc600\ub2e4. \"\ub098\ub294 \ubc94\uc778\uc774 \uc544\ub2c8\ub2e4. \uc544\ubb34\ub9ac \uadf8\ub798\ub3c4 \ub0b4\uac00 \ub0b4 \ub3d9\uc0dd\uc744 \uc8fd\uc77c \uc218\ub294 \uc5c6\ub2e4. \uadf8\ub9ac\uace0 \ub0b4 \ub3d9\uc0dd \uc544\ub4e4 \uc131\uc77c\uc774\ub294 \uc815\uc77c\uad8c \uc804 \uad6d\ubb34\ucd1d\ub9ac\uc758 \uc544\ub4e4\uc774 \ud655\uc2e4\ud558\ub2e4. \ub098\ub294 \ubb34\uc2a8 \uc218\ub97c \uc368\uc11c\ub77c\ub3c4 \ub0b4 \ub204\uba85\uc744 \ubc97\uc744 \uac83\uc774\ub2e4.\"\ub294 \uac83\uc774\ub2e4.", "answer": "\uc815\uc885\uc6b1", "sentence": "1989\ub144 \uac00\uc11d\ubc29\ub41c \uc815\uc778\uc219\uc758 \uc624\ube60 \uc815\uc885\uc6b1 \uc740 \uc815\uc131\uc77c\uc774 \uc815\uc77c\uad8c\uc758 \uc544\ub4e4\uc774 \ud2c0\ub9bc\uc5c6\ub2e4\uace0 \ub9d0\ud55c\ub2e4.", "paragraph_sentence": " 1989\ub144 \uac00\uc11d\ubc29\ub41c \uc815\uc778\uc219\uc758 \uc624\ube60 \uc815\uc885\uc6b1 \uc740 \uc815\uc131\uc77c\uc774 \uc815\uc77c\uad8c\uc758 \uc544\ub4e4\uc774 \ud2c0\ub9bc\uc5c6\ub2e4\uace0 \ub9d0\ud55c\ub2e4. \uadf8\uc5d0 \uc758\ud558\uba74 '\uc815\uc77c\uad8c\uc774 \ud55c \ub2ec\uc5d0 \ud55c \ubc88 \uaf34\ub85c \uc11c\uad50\ub3d9 \uc9d1\uc73c\ub85c \ucc3e\uc544\uc654\uc9c0\uc694. \uadf8\ub9ac\uace0 \ub3d9\uc0dd\uc774 '\uadf8\uc774\uac00 \uc544\ub4e4 \ud558\ub098\ub9cc \ub0b3\uc544\ub2ec\ub77c\uace0 \ud55c\ub2e4'\uba70 \uc0c1\uc758\ub97c \ud574 \ucc98\uc74c\uc5d4 \uac00\uc871\ub4e4\uc774 \uadf9\uad6c \ubc18\ub300\ub97c \ud588\uc9c0\uc694. \ucd9c\uc0b0 \ud6c4\uc5d0\ub294 \uc77c \uc8fc\uc77c\uc5d0 \ud55c\ubc88 \uaf34\ub85c \ucc3e\uc544\uc640 \uc131\uc77c\uc774\ub97c \uc548\uace0 \uc990\uac70\uc6cc\ud588\uc73c\uba70, \ub298\uadf8\ub9c9\uc5d0 \uc544\ub4e4\uc744 \uc5bb\uc5b4 \uc18c\uc6d0\uc744 \uc774\ub918\ub2e4\ub294 \ub73b\uc5d0\uc11c \uc131\uc77c\uc774\ub780 \uc774\ub984\uae4c\uc9c0 \uc9c1\uc811 \uc9c0\uc5b4\uc654\uc5b4\uc694. \uc131\uc77c\uc774\ub3c4 \uc790\uae30 \uc544\ube60\uac00 TV\uc5d0 \ube44\uce58\uba74 '\uc544\ube60'\ub77c\uace0 \uc18c\ub9ac\uce58\uba70 \uc88b\uc544\ud588\uc9c0\uc694. '\ub77c\ub294 \uac83\uc774\ub2e4. \uc815\uc885\uc6b1\uc740 \ud615\uc744 \ub9c8\uce58\uace0 \uad50\ub3c4\uc18c\uc5d0\uc11c \ucd9c\uac10\ud55c \ub4a4\uc5d0 \uae30\uc790 \uc778\ud130\ubdf0\uc5d0\uc11c \uc790\uc2e0\uc740 \ubc94\uc778\uc774 \uc544\ub2c8\ub77c\uba70 \uc5b5\uc6b8\ud568\uc744 \ud638\uc18c\ud558\uc600\ub2e4. \"\ub098\ub294 \ubc94\uc778\uc774 \uc544\ub2c8\ub2e4. \uc544\ubb34\ub9ac \uadf8\ub798\ub3c4 \ub0b4\uac00 \ub0b4 \ub3d9\uc0dd\uc744 \uc8fd\uc77c \uc218\ub294 \uc5c6\ub2e4. \uadf8\ub9ac\uace0 \ub0b4 \ub3d9\uc0dd \uc544\ub4e4 \uc131\uc77c\uc774\ub294 \uc815\uc77c\uad8c \uc804 \uad6d\ubb34\ucd1d\ub9ac\uc758 \uc544\ub4e4\uc774 \ud655\uc2e4\ud558\ub2e4. \ub098\ub294 \ubb34\uc2a8 \uc218\ub97c \uc368\uc11c\ub77c\ub3c4 \ub0b4 \ub204\uba85\uc744 \ubc97\uc744 \uac83\uc774\ub2e4. \"\ub294 \uac83\uc774\ub2e4.", "paragraph_answer": "1989\ub144 \uac00\uc11d\ubc29\ub41c \uc815\uc778\uc219\uc758 \uc624\ube60 \uc815\uc885\uc6b1 \uc740 \uc815\uc131\uc77c\uc774 \uc815\uc77c\uad8c\uc758 \uc544\ub4e4\uc774 \ud2c0\ub9bc\uc5c6\ub2e4\uace0 \ub9d0\ud55c\ub2e4. \uadf8\uc5d0 \uc758\ud558\uba74 '\uc815\uc77c\uad8c\uc774 \ud55c \ub2ec\uc5d0 \ud55c \ubc88 \uaf34\ub85c \uc11c\uad50\ub3d9 \uc9d1\uc73c\ub85c \ucc3e\uc544\uc654\uc9c0\uc694. \uadf8\ub9ac\uace0 \ub3d9\uc0dd\uc774 '\uadf8\uc774\uac00 \uc544\ub4e4 \ud558\ub098\ub9cc \ub0b3\uc544\ub2ec\ub77c\uace0 \ud55c\ub2e4'\uba70 \uc0c1\uc758\ub97c \ud574 \ucc98\uc74c\uc5d4 \uac00\uc871\ub4e4\uc774 \uadf9\uad6c \ubc18\ub300\ub97c \ud588\uc9c0\uc694. \ucd9c\uc0b0 \ud6c4\uc5d0\ub294 \uc77c \uc8fc\uc77c\uc5d0 \ud55c\ubc88 \uaf34\ub85c \ucc3e\uc544\uc640 \uc131\uc77c\uc774\ub97c \uc548\uace0 \uc990\uac70\uc6cc\ud588\uc73c\uba70, \ub298\uadf8\ub9c9\uc5d0 \uc544\ub4e4\uc744 \uc5bb\uc5b4 \uc18c\uc6d0\uc744 \uc774\ub918\ub2e4\ub294 \ub73b\uc5d0\uc11c \uc131\uc77c\uc774\ub780 \uc774\ub984\uae4c\uc9c0 \uc9c1\uc811 \uc9c0\uc5b4\uc654\uc5b4\uc694. \uc131\uc77c\uc774\ub3c4 \uc790\uae30 \uc544\ube60\uac00 TV\uc5d0 \ube44\uce58\uba74 '\uc544\ube60'\ub77c\uace0 \uc18c\ub9ac\uce58\uba70 \uc88b\uc544\ud588\uc9c0\uc694.'\ub77c\ub294 \uac83\uc774\ub2e4. \uc815\uc885\uc6b1\uc740 \ud615\uc744 \ub9c8\uce58\uace0 \uad50\ub3c4\uc18c\uc5d0\uc11c \ucd9c\uac10\ud55c \ub4a4\uc5d0 \uae30\uc790 \uc778\ud130\ubdf0\uc5d0\uc11c \uc790\uc2e0\uc740 \ubc94\uc778\uc774 \uc544\ub2c8\ub77c\uba70 \uc5b5\uc6b8\ud568\uc744 \ud638\uc18c\ud558\uc600\ub2e4. \"\ub098\ub294 \ubc94\uc778\uc774 \uc544\ub2c8\ub2e4. \uc544\ubb34\ub9ac \uadf8\ub798\ub3c4 \ub0b4\uac00 \ub0b4 \ub3d9\uc0dd\uc744 \uc8fd\uc77c \uc218\ub294 \uc5c6\ub2e4. \uadf8\ub9ac\uace0 \ub0b4 \ub3d9\uc0dd \uc544\ub4e4 \uc131\uc77c\uc774\ub294 \uc815\uc77c\uad8c \uc804 \uad6d\ubb34\ucd1d\ub9ac\uc758 \uc544\ub4e4\uc774 \ud655\uc2e4\ud558\ub2e4. \ub098\ub294 \ubb34\uc2a8 \uc218\ub97c \uc368\uc11c\ub77c\ub3c4 \ub0b4 \ub204\uba85\uc744 \ubc97\uc744 \uac83\uc774\ub2e4.\"\ub294 \uac83\uc774\ub2e4.", "sentence_answer": "1989\ub144 \uac00\uc11d\ubc29\ub41c \uc815\uc778\uc219\uc758 \uc624\ube60 \uc815\uc885\uc6b1 \uc740 \uc815\uc131\uc77c\uc774 \uc815\uc77c\uad8c\uc758 \uc544\ub4e4\uc774 \ud2c0\ub9bc\uc5c6\ub2e4\uace0 \ub9d0\ud55c\ub2e4.", "paragraph_id": "6594051-9-0", "paragraph_question": "question: 1989\ub144 \uac00\uc11d\ubc29\ub41c \uc815\uc778\uc219\uc758 \uc624\ube60\uc774\uba70 \uc815\uc131\uc77c\uc774 \uc815\uc77c\uad8c\uc758 \uc544\ub4e4\uc774\ub77c\uace0 \ub9d0\ud55c \uc0ac\ub78c\uc740?, context: 1989\ub144 \uac00\uc11d\ubc29\ub41c \uc815\uc778\uc219\uc758 \uc624\ube60 \uc815\uc885\uc6b1\uc740 \uc815\uc131\uc77c\uc774 \uc815\uc77c\uad8c\uc758 \uc544\ub4e4\uc774 \ud2c0\ub9bc\uc5c6\ub2e4\uace0 \ub9d0\ud55c\ub2e4. \uadf8\uc5d0 \uc758\ud558\uba74 '\uc815\uc77c\uad8c\uc774 \ud55c \ub2ec\uc5d0 \ud55c \ubc88 \uaf34\ub85c \uc11c\uad50\ub3d9 \uc9d1\uc73c\ub85c \ucc3e\uc544\uc654\uc9c0\uc694. \uadf8\ub9ac\uace0 \ub3d9\uc0dd\uc774 '\uadf8\uc774\uac00 \uc544\ub4e4 \ud558\ub098\ub9cc \ub0b3\uc544\ub2ec\ub77c\uace0 \ud55c\ub2e4'\uba70 \uc0c1\uc758\ub97c \ud574 \ucc98\uc74c\uc5d4 \uac00\uc871\ub4e4\uc774 \uadf9\uad6c \ubc18\ub300\ub97c \ud588\uc9c0\uc694. \ucd9c\uc0b0 \ud6c4\uc5d0\ub294 \uc77c \uc8fc\uc77c\uc5d0 \ud55c\ubc88 \uaf34\ub85c \ucc3e\uc544\uc640 \uc131\uc77c\uc774\ub97c \uc548\uace0 \uc990\uac70\uc6cc\ud588\uc73c\uba70, \ub298\uadf8\ub9c9\uc5d0 \uc544\ub4e4\uc744 \uc5bb\uc5b4 \uc18c\uc6d0\uc744 \uc774\ub918\ub2e4\ub294 \ub73b\uc5d0\uc11c \uc131\uc77c\uc774\ub780 \uc774\ub984\uae4c\uc9c0 \uc9c1\uc811 \uc9c0\uc5b4\uc654\uc5b4\uc694. \uc131\uc77c\uc774\ub3c4 \uc790\uae30 \uc544\ube60\uac00 TV\uc5d0 \ube44\uce58\uba74 '\uc544\ube60'\ub77c\uace0 \uc18c\ub9ac\uce58\uba70 \uc88b\uc544\ud588\uc9c0\uc694.'\ub77c\ub294 \uac83\uc774\ub2e4. \uc815\uc885\uc6b1\uc740 \ud615\uc744 \ub9c8\uce58\uace0 \uad50\ub3c4\uc18c\uc5d0\uc11c \ucd9c\uac10\ud55c \ub4a4\uc5d0 \uae30\uc790 \uc778\ud130\ubdf0\uc5d0\uc11c \uc790\uc2e0\uc740 \ubc94\uc778\uc774 \uc544\ub2c8\ub77c\uba70 \uc5b5\uc6b8\ud568\uc744 \ud638\uc18c\ud558\uc600\ub2e4. \"\ub098\ub294 \ubc94\uc778\uc774 \uc544\ub2c8\ub2e4. \uc544\ubb34\ub9ac \uadf8\ub798\ub3c4 \ub0b4\uac00 \ub0b4 \ub3d9\uc0dd\uc744 \uc8fd\uc77c \uc218\ub294 \uc5c6\ub2e4. \uadf8\ub9ac\uace0 \ub0b4 \ub3d9\uc0dd \uc544\ub4e4 \uc131\uc77c\uc774\ub294 \uc815\uc77c\uad8c \uc804 \uad6d\ubb34\ucd1d\ub9ac\uc758 \uc544\ub4e4\uc774 \ud655\uc2e4\ud558\ub2e4. \ub098\ub294 \ubb34\uc2a8 \uc218\ub97c \uc368\uc11c\ub77c\ub3c4 \ub0b4 \ub204\uba85\uc744 \ubc97\uc744 \uac83\uc774\ub2e4.\"\ub294 \uac83\uc774\ub2e4."} {"question": "\ub2c8\ucf54\ud3f4\ub9ac\uc2a4 \uc804\ud22c\uc758 \ud328\ubc30\ub85c \uc5b4\ub514\uc5d0 \ub300\ud55c \uc11c\uc720\ub7fd\uc758 \uc6d0\uc815\uc740 \ub05d\uc774\ub0ac\ub294\uac00?", "paragraph": "9\uc6d4 26\uc77c \ubc14\uc608\uc9c0\ub4dc\ub294 \uc804\ud22c \uc9c1\uc804 \ub77c\ud638\ubcf4\uc758 \uc624\uc2a4\ub9cc \uad70 \ud3ec\ub85c\ub4e4\uc744 \ud504\ub791\uc2a4\uc778\ub4e4\uc774 \uc8fd\uc778\uac83\uc5d0 \ub300\ud55c \ubcf4\ubcf5\uc73c\ub85c 3,000\uba85\uc5d0\uc11c 10,000\uba85\uc5d0 \ub2ec\ud558\ub294 \ud3ec\ub85c\ub4e4\uc744 \uc8fd\uc77c \uac83\uc744 \uba85\ud55c\ub2e4. \uadf8\ub294 \ub610 \uadf8\uac00 \uc555\ub3c4\uc801\uc778 \uc2b9\ub9ac\ub97c \uac70\ub450\uc5c8\uc74c\uc5d0\ub3c4 \ubd88\uad6c\ud558\uace0 \uc804\ud22c \ucd08\ubc18\uc5d0 \ub9ce\uc740 \ubcd1\uc0ac\ub4e4\uc744 \uc783\uc740 \uac83\uc5d0 \ub300\ud558\uc5ec \ubd84\ub178\ud558\uc600\ub2e4. \uadf8\ub294 \uc5b4\ub9b0 \ud3ec\ub85c\ub4e4\uc744 \uc790\uc2e0\uc758 \uad70\ub300\uc5d0 \ud3b8\uc785\uc2dc\ucf30\ub2e4. \uace0\ud5a5\uc73c\ub85c \ub3cc\uc544\uac00\ub294 \uc640\uc911\uc5d0 \ub9ce\uc740 \uace0\uc0dd\uc744 \ud558\uc9c0\ub9cc \uacb0\uad6d \ub3cc\uc544\uac04 \uc774\ub4e4\ub3c4 \uc788\uc5c8\ub2e4. \uc9c0\uae30\uc2a4\ubb38\ud2b8\ub294 \ub2c8\ucf5c\ub77c \uace0\ub974\uc794\uc2a4\ud0a4\uc640 \uc2e4\ub9ac\uc758 \ud5e4\ub974\ub9cc\uc758 \ub3c4\uc6c0\uc73c\ub85c \ub3c4\ub9dd\uce60 \uc218 \uc788\uc5c8\ub2e4. \uadf8\ub294 \ubca0\ub124\uce58\uc544 \ubc30\ub97c \ud0c0 \ud751\ud574, \uc5d0\uac8c \ud574, \uadf8\ub9ac\uace0 \uc9c0\uc911\ud574\ub97c \ud1b5\ud574 \ud574\ub85c\ub85c \uace0\ud5a5\uc73c\ub85c \ub3cc\uc544\uac00\ub294 \uae38\uc744 \ud0dd\ud588\uace0 \uc774 \uc640\uc911\uc5d0 \uc648\ub77c\ud0a4\uc544\uc778\ub4e4\uc758 \ubbff\uc744 \uc218 \uc5c6\ub294 \uc131\ud488\uc744 \uc54c\uc544\ucc28\ub9b0\ub2e4. \uc0e4\ub97c 6\uc138\uc5d0\uac8c \ud328\ubc30\uc18c\uc2dd\uc774 \ud06c\ub9ac\uc2a4\ub9c8\uc2a4\uc5d0 \uc54c\ub824\uc84c\ub2e4. \uc11c\uc720\ub7fd\uc758 \uae30\uc0ac\ub4e4\uc740 \uace7 \uc2ed\uc790\uad70\uc5d0 \ub300\ud55c \uc5f4\uc815\uc744 \uc783\uc5b4\ubc84\ub838\ub2e4. \uc2f8\uc6c0\uc740 \uc2a4\ud398\uc778\uacfc \uc9c0\uc911\ud574, \uadf8\ub9ac\uace0 \ubd81\uc720\ub7fd\uc758 \uc774\uad50\ub3c4\uc5d0 \ub300\ud558\uc5ec \uacc4\uc18d\ub418\uace0 \uc788\uc5c8\ub2e4. \uadf8\ub7ec\ub098 \uc774 \uc804\ud22c \uc774\ud6c4\uc5d0 \ubc1c\uce78 \ubc18\ub3c4\uc5d0 \ub300\ud55c \uc624\uc2a4\ub9cc \ud22c\ub974\ud06c\uc758 \uc804\uc9c4\uc744 \ub9c9\uae30 \uc704\ud574 \uc11c\uc720\ub7fd\uc5d0\uc11c \uad6c\uc131\ub41c \uc6d0\uc815\uc740 \ub354 \uc774\uc0c1 \uc5c6\uc5c8\ub2e4. \ub974\ub124\uc0c1\uc2a4\uc2dc\ub300\uac00 \ub2e4\uac00\uc62c \ub54c\uae4c\uc9c0\uc600\ub2e4.", "answer": "\ubc1c\uce78 \ubc18\ub3c4", "sentence": "\uc774 \uc804\ud22c \uc774\ud6c4\uc5d0 \ubc1c\uce78 \ubc18\ub3c4 \uc5d0 \ub300\ud55c \uc624\uc2a4\ub9cc \ud22c\ub974\ud06c\uc758 \uc804\uc9c4\uc744 \ub9c9\uae30 \uc704\ud574 \uc11c\uc720\ub7fd\uc5d0\uc11c \uad6c\uc131\ub41c \uc6d0\uc815\uc740 \ub354 \uc774\uc0c1 \uc5c6\uc5c8\ub2e4.", "paragraph_sentence": "9\uc6d4 26\uc77c \ubc14\uc608\uc9c0\ub4dc\ub294 \uc804\ud22c \uc9c1\uc804 \ub77c\ud638\ubcf4\uc758 \uc624\uc2a4\ub9cc \uad70 \ud3ec\ub85c\ub4e4\uc744 \ud504\ub791\uc2a4\uc778\ub4e4\uc774 \uc8fd\uc778\uac83\uc5d0 \ub300\ud55c \ubcf4\ubcf5\uc73c\ub85c 3,000\uba85\uc5d0\uc11c 10,000\uba85\uc5d0 \ub2ec\ud558\ub294 \ud3ec\ub85c\ub4e4\uc744 \uc8fd\uc77c \uac83\uc744 \uba85\ud55c\ub2e4. \uadf8\ub294 \ub610 \uadf8\uac00 \uc555\ub3c4\uc801\uc778 \uc2b9\ub9ac\ub97c \uac70\ub450\uc5c8\uc74c\uc5d0\ub3c4 \ubd88\uad6c\ud558\uace0 \uc804\ud22c \ucd08\ubc18\uc5d0 \ub9ce\uc740 \ubcd1\uc0ac\ub4e4\uc744 \uc783\uc740 \uac83\uc5d0 \ub300\ud558\uc5ec \ubd84\ub178\ud558\uc600\ub2e4. \uadf8\ub294 \uc5b4\ub9b0 \ud3ec\ub85c\ub4e4\uc744 \uc790\uc2e0\uc758 \uad70\ub300\uc5d0 \ud3b8\uc785\uc2dc\ucf30\ub2e4. \uace0\ud5a5\uc73c\ub85c \ub3cc\uc544\uac00\ub294 \uc640\uc911\uc5d0 \ub9ce\uc740 \uace0\uc0dd\uc744 \ud558\uc9c0\ub9cc \uacb0\uad6d \ub3cc\uc544\uac04 \uc774\ub4e4\ub3c4 \uc788\uc5c8\ub2e4. \uc9c0\uae30\uc2a4\ubb38\ud2b8\ub294 \ub2c8\ucf5c\ub77c \uace0\ub974\uc794\uc2a4\ud0a4\uc640 \uc2e4\ub9ac\uc758 \ud5e4\ub974\ub9cc\uc758 \ub3c4\uc6c0\uc73c\ub85c \ub3c4\ub9dd\uce60 \uc218 \uc788\uc5c8\ub2e4. \uadf8\ub294 \ubca0\ub124\uce58\uc544 \ubc30\ub97c \ud0c0 \ud751\ud574, \uc5d0\uac8c \ud574, \uadf8\ub9ac\uace0 \uc9c0\uc911\ud574\ub97c \ud1b5\ud574 \ud574\ub85c\ub85c \uace0\ud5a5\uc73c\ub85c \ub3cc\uc544\uac00\ub294 \uae38\uc744 \ud0dd\ud588\uace0 \uc774 \uc640\uc911\uc5d0 \uc648\ub77c\ud0a4\uc544\uc778\ub4e4\uc758 \ubbff\uc744 \uc218 \uc5c6\ub294 \uc131\ud488\uc744 \uc54c\uc544\ucc28\ub9b0\ub2e4. \uc0e4\ub97c 6\uc138\uc5d0\uac8c \ud328\ubc30\uc18c\uc2dd\uc774 \ud06c\ub9ac\uc2a4\ub9c8\uc2a4\uc5d0 \uc54c\ub824\uc84c\ub2e4. \uc11c\uc720\ub7fd\uc758 \uae30\uc0ac\ub4e4\uc740 \uace7 \uc2ed\uc790\uad70\uc5d0 \ub300\ud55c \uc5f4\uc815\uc744 \uc783\uc5b4\ubc84\ub838\ub2e4. \uc2f8\uc6c0\uc740 \uc2a4\ud398\uc778\uacfc \uc9c0\uc911\ud574, \uadf8\ub9ac\uace0 \ubd81\uc720\ub7fd\uc758 \uc774\uad50\ub3c4\uc5d0 \ub300\ud558\uc5ec \uacc4\uc18d\ub418\uace0 \uc788\uc5c8\ub2e4. \uadf8\ub7ec\ub098 \uc774 \uc804\ud22c \uc774\ud6c4\uc5d0 \ubc1c\uce78 \ubc18\ub3c4 \uc5d0 \ub300\ud55c \uc624\uc2a4\ub9cc \ud22c\ub974\ud06c\uc758 \uc804\uc9c4\uc744 \ub9c9\uae30 \uc704\ud574 \uc11c\uc720\ub7fd\uc5d0\uc11c \uad6c\uc131\ub41c \uc6d0\uc815\uc740 \ub354 \uc774\uc0c1 \uc5c6\uc5c8\ub2e4. \ub974\ub124\uc0c1\uc2a4\uc2dc\ub300\uac00 \ub2e4\uac00\uc62c \ub54c\uae4c\uc9c0\uc600\ub2e4.", "paragraph_answer": "9\uc6d4 26\uc77c \ubc14\uc608\uc9c0\ub4dc\ub294 \uc804\ud22c \uc9c1\uc804 \ub77c\ud638\ubcf4\uc758 \uc624\uc2a4\ub9cc \uad70 \ud3ec\ub85c\ub4e4\uc744 \ud504\ub791\uc2a4\uc778\ub4e4\uc774 \uc8fd\uc778\uac83\uc5d0 \ub300\ud55c \ubcf4\ubcf5\uc73c\ub85c 3,000\uba85\uc5d0\uc11c 10,000\uba85\uc5d0 \ub2ec\ud558\ub294 \ud3ec\ub85c\ub4e4\uc744 \uc8fd\uc77c \uac83\uc744 \uba85\ud55c\ub2e4. \uadf8\ub294 \ub610 \uadf8\uac00 \uc555\ub3c4\uc801\uc778 \uc2b9\ub9ac\ub97c \uac70\ub450\uc5c8\uc74c\uc5d0\ub3c4 \ubd88\uad6c\ud558\uace0 \uc804\ud22c \ucd08\ubc18\uc5d0 \ub9ce\uc740 \ubcd1\uc0ac\ub4e4\uc744 \uc783\uc740 \uac83\uc5d0 \ub300\ud558\uc5ec \ubd84\ub178\ud558\uc600\ub2e4. \uadf8\ub294 \uc5b4\ub9b0 \ud3ec\ub85c\ub4e4\uc744 \uc790\uc2e0\uc758 \uad70\ub300\uc5d0 \ud3b8\uc785\uc2dc\ucf30\ub2e4. \uace0\ud5a5\uc73c\ub85c \ub3cc\uc544\uac00\ub294 \uc640\uc911\uc5d0 \ub9ce\uc740 \uace0\uc0dd\uc744 \ud558\uc9c0\ub9cc \uacb0\uad6d \ub3cc\uc544\uac04 \uc774\ub4e4\ub3c4 \uc788\uc5c8\ub2e4. \uc9c0\uae30\uc2a4\ubb38\ud2b8\ub294 \ub2c8\ucf5c\ub77c \uace0\ub974\uc794\uc2a4\ud0a4\uc640 \uc2e4\ub9ac\uc758 \ud5e4\ub974\ub9cc\uc758 \ub3c4\uc6c0\uc73c\ub85c \ub3c4\ub9dd\uce60 \uc218 \uc788\uc5c8\ub2e4. \uadf8\ub294 \ubca0\ub124\uce58\uc544 \ubc30\ub97c \ud0c0 \ud751\ud574, \uc5d0\uac8c \ud574, \uadf8\ub9ac\uace0 \uc9c0\uc911\ud574\ub97c \ud1b5\ud574 \ud574\ub85c\ub85c \uace0\ud5a5\uc73c\ub85c \ub3cc\uc544\uac00\ub294 \uae38\uc744 \ud0dd\ud588\uace0 \uc774 \uc640\uc911\uc5d0 \uc648\ub77c\ud0a4\uc544\uc778\ub4e4\uc758 \ubbff\uc744 \uc218 \uc5c6\ub294 \uc131\ud488\uc744 \uc54c\uc544\ucc28\ub9b0\ub2e4. \uc0e4\ub97c 6\uc138\uc5d0\uac8c \ud328\ubc30\uc18c\uc2dd\uc774 \ud06c\ub9ac\uc2a4\ub9c8\uc2a4\uc5d0 \uc54c\ub824\uc84c\ub2e4. \uc11c\uc720\ub7fd\uc758 \uae30\uc0ac\ub4e4\uc740 \uace7 \uc2ed\uc790\uad70\uc5d0 \ub300\ud55c \uc5f4\uc815\uc744 \uc783\uc5b4\ubc84\ub838\ub2e4. \uc2f8\uc6c0\uc740 \uc2a4\ud398\uc778\uacfc \uc9c0\uc911\ud574, \uadf8\ub9ac\uace0 \ubd81\uc720\ub7fd\uc758 \uc774\uad50\ub3c4\uc5d0 \ub300\ud558\uc5ec \uacc4\uc18d\ub418\uace0 \uc788\uc5c8\ub2e4. \uadf8\ub7ec\ub098 \uc774 \uc804\ud22c \uc774\ud6c4\uc5d0 \ubc1c\uce78 \ubc18\ub3c4 \uc5d0 \ub300\ud55c \uc624\uc2a4\ub9cc \ud22c\ub974\ud06c\uc758 \uc804\uc9c4\uc744 \ub9c9\uae30 \uc704\ud574 \uc11c\uc720\ub7fd\uc5d0\uc11c \uad6c\uc131\ub41c \uc6d0\uc815\uc740 \ub354 \uc774\uc0c1 \uc5c6\uc5c8\ub2e4. \ub974\ub124\uc0c1\uc2a4\uc2dc\ub300\uac00 \ub2e4\uac00\uc62c \ub54c\uae4c\uc9c0\uc600\ub2e4.", "sentence_answer": "\uc774 \uc804\ud22c \uc774\ud6c4\uc5d0 \ubc1c\uce78 \ubc18\ub3c4 \uc5d0 \ub300\ud55c \uc624\uc2a4\ub9cc \ud22c\ub974\ud06c\uc758 \uc804\uc9c4\uc744 \ub9c9\uae30 \uc704\ud574 \uc11c\uc720\ub7fd\uc5d0\uc11c \uad6c\uc131\ub41c \uc6d0\uc815\uc740 \ub354 \uc774\uc0c1 \uc5c6\uc5c8\ub2e4.", "paragraph_id": "6484822-5-2", "paragraph_question": "question: \ub2c8\ucf54\ud3f4\ub9ac\uc2a4 \uc804\ud22c\uc758 \ud328\ubc30\ub85c \uc5b4\ub514\uc5d0 \ub300\ud55c \uc11c\uc720\ub7fd\uc758 \uc6d0\uc815\uc740 \ub05d\uc774\ub0ac\ub294\uac00?, context: 9\uc6d4 26\uc77c \ubc14\uc608\uc9c0\ub4dc\ub294 \uc804\ud22c \uc9c1\uc804 \ub77c\ud638\ubcf4\uc758 \uc624\uc2a4\ub9cc \uad70 \ud3ec\ub85c\ub4e4\uc744 \ud504\ub791\uc2a4\uc778\ub4e4\uc774 \uc8fd\uc778\uac83\uc5d0 \ub300\ud55c \ubcf4\ubcf5\uc73c\ub85c 3,000\uba85\uc5d0\uc11c 10,000\uba85\uc5d0 \ub2ec\ud558\ub294 \ud3ec\ub85c\ub4e4\uc744 \uc8fd\uc77c \uac83\uc744 \uba85\ud55c\ub2e4. \uadf8\ub294 \ub610 \uadf8\uac00 \uc555\ub3c4\uc801\uc778 \uc2b9\ub9ac\ub97c \uac70\ub450\uc5c8\uc74c\uc5d0\ub3c4 \ubd88\uad6c\ud558\uace0 \uc804\ud22c \ucd08\ubc18\uc5d0 \ub9ce\uc740 \ubcd1\uc0ac\ub4e4\uc744 \uc783\uc740 \uac83\uc5d0 \ub300\ud558\uc5ec \ubd84\ub178\ud558\uc600\ub2e4. \uadf8\ub294 \uc5b4\ub9b0 \ud3ec\ub85c\ub4e4\uc744 \uc790\uc2e0\uc758 \uad70\ub300\uc5d0 \ud3b8\uc785\uc2dc\ucf30\ub2e4. \uace0\ud5a5\uc73c\ub85c \ub3cc\uc544\uac00\ub294 \uc640\uc911\uc5d0 \ub9ce\uc740 \uace0\uc0dd\uc744 \ud558\uc9c0\ub9cc \uacb0\uad6d \ub3cc\uc544\uac04 \uc774\ub4e4\ub3c4 \uc788\uc5c8\ub2e4. \uc9c0\uae30\uc2a4\ubb38\ud2b8\ub294 \ub2c8\ucf5c\ub77c \uace0\ub974\uc794\uc2a4\ud0a4\uc640 \uc2e4\ub9ac\uc758 \ud5e4\ub974\ub9cc\uc758 \ub3c4\uc6c0\uc73c\ub85c \ub3c4\ub9dd\uce60 \uc218 \uc788\uc5c8\ub2e4. \uadf8\ub294 \ubca0\ub124\uce58\uc544 \ubc30\ub97c \ud0c0 \ud751\ud574, \uc5d0\uac8c \ud574, \uadf8\ub9ac\uace0 \uc9c0\uc911\ud574\ub97c \ud1b5\ud574 \ud574\ub85c\ub85c \uace0\ud5a5\uc73c\ub85c \ub3cc\uc544\uac00\ub294 \uae38\uc744 \ud0dd\ud588\uace0 \uc774 \uc640\uc911\uc5d0 \uc648\ub77c\ud0a4\uc544\uc778\ub4e4\uc758 \ubbff\uc744 \uc218 \uc5c6\ub294 \uc131\ud488\uc744 \uc54c\uc544\ucc28\ub9b0\ub2e4. \uc0e4\ub97c 6\uc138\uc5d0\uac8c \ud328\ubc30\uc18c\uc2dd\uc774 \ud06c\ub9ac\uc2a4\ub9c8\uc2a4\uc5d0 \uc54c\ub824\uc84c\ub2e4. \uc11c\uc720\ub7fd\uc758 \uae30\uc0ac\ub4e4\uc740 \uace7 \uc2ed\uc790\uad70\uc5d0 \ub300\ud55c \uc5f4\uc815\uc744 \uc783\uc5b4\ubc84\ub838\ub2e4. \uc2f8\uc6c0\uc740 \uc2a4\ud398\uc778\uacfc \uc9c0\uc911\ud574, \uadf8\ub9ac\uace0 \ubd81\uc720\ub7fd\uc758 \uc774\uad50\ub3c4\uc5d0 \ub300\ud558\uc5ec \uacc4\uc18d\ub418\uace0 \uc788\uc5c8\ub2e4. \uadf8\ub7ec\ub098 \uc774 \uc804\ud22c \uc774\ud6c4\uc5d0 \ubc1c\uce78 \ubc18\ub3c4\uc5d0 \ub300\ud55c \uc624\uc2a4\ub9cc \ud22c\ub974\ud06c\uc758 \uc804\uc9c4\uc744 \ub9c9\uae30 \uc704\ud574 \uc11c\uc720\ub7fd\uc5d0\uc11c \uad6c\uc131\ub41c \uc6d0\uc815\uc740 \ub354 \uc774\uc0c1 \uc5c6\uc5c8\ub2e4. \ub974\ub124\uc0c1\uc2a4\uc2dc\ub300\uac00 \ub2e4\uac00\uc62c \ub54c\uae4c\uc9c0\uc600\ub2e4."} {"question": "\ubc45\ud06c\uc0ac\uc774\ub4dc \ubc1c\uc804\uc18c\ub294 \uc5b4\ub5a4 \ubb38\uc81c\ub85c 1981\ub144 \ubb38\uc744 \ub2eb\uc558\ub098?", "paragraph": "2000\ub144 5\uc6d4 12\uc77c \uac1c\uad00\ud55c \ud14c\uc774\ud2b8 \ubaa8\ub358\uc740 \uc601\uad6d \uc815\ubd80\uc758 \ubc00\ub808\ub2c8\uc5c4 \ud504\ub85c\uc81d\ud2b8\uc758 \uc77c\ud658\uc73c\ub85c \ud15c\uc988\uac15\ubcc0\uc758 \ubc45\ud06c\uc0ac\uc774\ub4dc(Bankside) \ubc1c\uc804\uc18c\ub97c \uc0c8\ub86d\uac8c \ub9ac\ubaa8\ub378\ub9c1\ud55c \uacf3\uc5d0 \ub4e4\uc5b4\uc130\ub2e4. \ubc45\ud06c\uc0ac\uc774\ub4dc \ubc1c\uc804\uc18c\ub294 2\ucc28 \uc138\uacc4\ub300\uc804 \uc9c1\ud6c4 \ub7f0\ub358 \uc911\uc2ec\ubd80\uc5d0 \uc804\ub825\uc744 \uacf5\uae09\ud558\uae30 \uc704\ud574 \uc138\uc6cc\uc84c\ub358 \ud654\ub825\ubc1c\uc804\uc18c\ub85c \uc601\uad6d\uc758 \ube68\uac04 \uacf5\uc911\uc804\ud654 \ubc15\uc2a4 \ub514\uc790\uc778\uc73c\ub85c\ub3c4 \uc720\uba85\ud55c \uac74\ucd95\uac00 \uae38\ubc84\ud2b8 \uc2a4\ucf54\ud2b8(Giles Gilbert Scott)\uc5d0 \uc758\ud574 \uc9c0\uc5b4\uc84c\uc73c\uba70 \uacf5\ud574\ubb38\uc81c\ub85c \uc774\uc804\ud55c \uc774\ud6c4 1981\ub144 \ubb38\uc744 \ub2eb\uc740 \uc0c1\ud0dc\uc600\ub2e4. \uc601\uad6d \uc815\ubd80\uc640 \ud14c\uc774\ud2b8 \uc7ac\ub2e8\uc740 \ud15c\uc988\uac15\ubcc0\uc5d0 \uc790\ub9ac\ud558\uace0 \uc788\uc73c\uba74\uc11c \ub113\uc740 \uac74\ubb3c\uba74\uc801\uacfc \uc9c0\ud558\ucca0\uc5ed\uc5d0\uc11c\ub3c4 \uac00\uae4c\uc6b4 \uc774 \ubc1c\uc804\uc18c\ub97c \ud604\ub300\ubbf8\uc220\uad00\uc744 \uc9c0\uc744 \uc7a5\uc18c\ub85c \ub099\uc810\ud558\uace0 \uad6d\uc81c \uac74\ucd95 \uacf5\ubaa8\uc804\uc744 \ud1b5\ud574 \uc120\uc815\ub41c \uc2a4\uc704\uc2a4 \uac74\ucd95\ud68c\uc0ac \ud5e4\ub974\ucd08\ud06c & \ub4dc \ubafc\ub871(Herzog\uc640 de Meuron)\uc774 \ud14c\uc774\ud2b8\ubaa8\ub358 \ud504\ub85c\uc81d\ud2b8\uc5d0 \ucc38\uc5ec\ud558\uac8c \ub41c\ub2e4. \uc57d 8\ub144\uc5ec \uac04\uc758 \uacf5\uc0ac\uae30\uac04 \ub05d\uc5d0 \uc9c0\uc5b4\uc9c4 \ubcf8 \uac74\ubb3c\uc740 \uae30\uc874\uc758 \uc678\uad00\uc740 \ucd5c\ub300\ud55c \uc190\ub300\uc9c0 \uc54a\uace0 \ub0b4\ubd80\ub294 \ubbf8\uc220\uad00\uc758 \uae30\ub2a5\uc5d0 \ub9de\ucdb0 \uc644\uc804\ud788 \uc0c8\ub85c\uc6b4 \uad6c\uc870\ub85c \ubc14\uafb8\ub294 \ubc29\uc2dd\uc73c\ub85c \uac1c\uc870\ub418\uc5c8\ub2e4. \ucd1d \ub192\uc774 99m\uc9c1\uc721\uba74\uccb4 \uc678\ud615\uc758 \uc6c5\uc7a5\ud55c \ud14c\uc774\ud2b8 \ubaa8\ub358\uc740 \ubaa8\ub450 7\uce35\uc73c\ub85c \uad6c\uc131\ub418\uc5b4 \uc788\uc73c\uba70 \uac74\ubb3c \ud55c\uac00\uc6b4\ub370 \uc6d0\ub798 \ubc1c\uc804\uc18c\uc6a9\uc73c\ub85c \uc0ac\uc6a9\ud558\ub358 \ub192\uc774 99m\uc758 \uad74\ub69d\uc774 \uadf8\ub300\ub85c \uc19f\uc544 \uc788\ub294\ub370, \ubc18\ud22c\uba85 \ud328\ub110\uc744 \uc0ac\uc6a9\ud558\uc5ec \ubc24\uc774\uba74 \ub4f1\ub300\ucc98\ub7fc \ube5b\uc744 \ub0b4\ub3c4\ub85d \uac1c\uc870\ud558\uc5ec \uc774 \uad74\ub69d\uc740 \uc624\ub298\ub0a0 \ud14c\uc774\ud2b8 \ubaa8\ub358\uc758 \uc0c1\uc9d5\uc774 \ub418\uc5c8\ub2e4. \ubbf8\uc220\uad00 \uac74\ubb3c \uc790\uccb4\ub9cc\uc73c\ub85c\ub3c4 \ubcfc \uac70\ub9ac\uac00 \ub41c \ud14c\uc774\ud2b8 \ubaa8\ub358\uc740 \ud55c\ud574 400\ub9cc\uba85 \uc774\uc0c1\uc758 \uad00\uad11\uac1d\uc774 \ucc3e\ub294 \ub7f0\ub358\uc758 \uc0c8\ub85c\uc6b4 \uad00\uad11\uba85\uc18c\uac00 \ub418\uc5c8\ub2e4.", "answer": "\uacf5\ud574\ubb38\uc81c", "sentence": "\ubc45\ud06c\uc0ac\uc774\ub4dc \ubc1c\uc804\uc18c\ub294 2\ucc28 \uc138\uacc4\ub300\uc804 \uc9c1\ud6c4 \ub7f0\ub358 \uc911\uc2ec\ubd80\uc5d0 \uc804\ub825\uc744 \uacf5\uae09\ud558\uae30 \uc704\ud574 \uc138\uc6cc\uc84c\ub358 \ud654\ub825\ubc1c\uc804\uc18c\ub85c \uc601\uad6d\uc758 \ube68\uac04 \uacf5\uc911\uc804\ud654 \ubc15\uc2a4 \ub514\uc790\uc778\uc73c\ub85c\ub3c4 \uc720\uba85\ud55c \uac74\ucd95\uac00 \uae38\ubc84\ud2b8 \uc2a4\ucf54\ud2b8(Giles Gilbert Scott)\uc5d0 \uc758\ud574 \uc9c0\uc5b4\uc84c\uc73c\uba70 \uacf5\ud574\ubb38\uc81c \ub85c \uc774\uc804\ud55c \uc774\ud6c4 1981\ub144 \ubb38\uc744 \ub2eb\uc740 \uc0c1\ud0dc\uc600\ub2e4.", "paragraph_sentence": "2000\ub144 5\uc6d4 12\uc77c \uac1c\uad00\ud55c \ud14c\uc774\ud2b8 \ubaa8\ub358\uc740 \uc601\uad6d \uc815\ubd80\uc758 \ubc00\ub808\ub2c8\uc5c4 \ud504\ub85c\uc81d\ud2b8\uc758 \uc77c\ud658\uc73c\ub85c \ud15c\uc988\uac15\ubcc0\uc758 \ubc45\ud06c\uc0ac\uc774\ub4dc(Bankside) \ubc1c\uc804\uc18c\ub97c \uc0c8\ub86d\uac8c \ub9ac\ubaa8\ub378\ub9c1\ud55c \uacf3\uc5d0 \ub4e4\uc5b4\uc130\ub2e4. \ubc45\ud06c\uc0ac\uc774\ub4dc \ubc1c\uc804\uc18c\ub294 2\ucc28 \uc138\uacc4\ub300\uc804 \uc9c1\ud6c4 \ub7f0\ub358 \uc911\uc2ec\ubd80\uc5d0 \uc804\ub825\uc744 \uacf5\uae09\ud558\uae30 \uc704\ud574 \uc138\uc6cc\uc84c\ub358 \ud654\ub825\ubc1c\uc804\uc18c\ub85c \uc601\uad6d\uc758 \ube68\uac04 \uacf5\uc911\uc804\ud654 \ubc15\uc2a4 \ub514\uc790\uc778\uc73c\ub85c\ub3c4 \uc720\uba85\ud55c \uac74\ucd95\uac00 \uae38\ubc84\ud2b8 \uc2a4\ucf54\ud2b8(Giles Gilbert Scott)\uc5d0 \uc758\ud574 \uc9c0\uc5b4\uc84c\uc73c\uba70 \uacf5\ud574\ubb38\uc81c \ub85c \uc774\uc804\ud55c \uc774\ud6c4 1981\ub144 \ubb38\uc744 \ub2eb\uc740 \uc0c1\ud0dc\uc600\ub2e4. \uc601\uad6d \uc815\ubd80\uc640 \ud14c\uc774\ud2b8 \uc7ac\ub2e8\uc740 \ud15c\uc988\uac15\ubcc0\uc5d0 \uc790\ub9ac\ud558\uace0 \uc788\uc73c\uba74\uc11c \ub113\uc740 \uac74\ubb3c\uba74\uc801\uacfc \uc9c0\ud558\ucca0\uc5ed\uc5d0\uc11c\ub3c4 \uac00\uae4c\uc6b4 \uc774 \ubc1c\uc804\uc18c\ub97c \ud604\ub300\ubbf8\uc220\uad00\uc744 \uc9c0\uc744 \uc7a5\uc18c\ub85c \ub099\uc810\ud558\uace0 \uad6d\uc81c \uac74\ucd95 \uacf5\ubaa8\uc804\uc744 \ud1b5\ud574 \uc120\uc815\ub41c \uc2a4\uc704\uc2a4 \uac74\ucd95\ud68c\uc0ac \ud5e4\ub974\ucd08\ud06c & \ub4dc \ubafc\ub871(Herzog\uc640 de Meuron)\uc774 \ud14c\uc774\ud2b8\ubaa8\ub358 \ud504\ub85c\uc81d\ud2b8\uc5d0 \ucc38\uc5ec\ud558\uac8c \ub41c\ub2e4. \uc57d 8\ub144\uc5ec \uac04\uc758 \uacf5\uc0ac\uae30\uac04 \ub05d\uc5d0 \uc9c0\uc5b4\uc9c4 \ubcf8 \uac74\ubb3c\uc740 \uae30\uc874\uc758 \uc678\uad00\uc740 \ucd5c\ub300\ud55c \uc190\ub300\uc9c0 \uc54a\uace0 \ub0b4\ubd80\ub294 \ubbf8\uc220\uad00\uc758 \uae30\ub2a5\uc5d0 \ub9de\ucdb0 \uc644\uc804\ud788 \uc0c8\ub85c\uc6b4 \uad6c\uc870\ub85c \ubc14\uafb8\ub294 \ubc29\uc2dd\uc73c\ub85c \uac1c\uc870\ub418\uc5c8\ub2e4. \ucd1d \ub192\uc774 99m\uc9c1\uc721\uba74\uccb4 \uc678\ud615\uc758 \uc6c5\uc7a5\ud55c \ud14c\uc774\ud2b8 \ubaa8\ub358\uc740 \ubaa8\ub450 7\uce35\uc73c\ub85c \uad6c\uc131\ub418\uc5b4 \uc788\uc73c\uba70 \uac74\ubb3c \ud55c\uac00\uc6b4\ub370 \uc6d0\ub798 \ubc1c\uc804\uc18c\uc6a9\uc73c\ub85c \uc0ac\uc6a9\ud558\ub358 \ub192\uc774 99m\uc758 \uad74\ub69d\uc774 \uadf8\ub300\ub85c \uc19f\uc544 \uc788\ub294\ub370, \ubc18\ud22c\uba85 \ud328\ub110\uc744 \uc0ac\uc6a9\ud558\uc5ec \ubc24\uc774\uba74 \ub4f1\ub300\ucc98\ub7fc \ube5b\uc744 \ub0b4\ub3c4\ub85d \uac1c\uc870\ud558\uc5ec \uc774 \uad74\ub69d\uc740 \uc624\ub298\ub0a0 \ud14c\uc774\ud2b8 \ubaa8\ub358\uc758 \uc0c1\uc9d5\uc774 \ub418\uc5c8\ub2e4. \ubbf8\uc220\uad00 \uac74\ubb3c \uc790\uccb4\ub9cc\uc73c\ub85c\ub3c4 \ubcfc \uac70\ub9ac\uac00 \ub41c \ud14c\uc774\ud2b8 \ubaa8\ub358\uc740 \ud55c\ud574 400\ub9cc\uba85 \uc774\uc0c1\uc758 \uad00\uad11\uac1d\uc774 \ucc3e\ub294 \ub7f0\ub358\uc758 \uc0c8\ub85c\uc6b4 \uad00\uad11\uba85\uc18c\uac00 \ub418\uc5c8\ub2e4.", "paragraph_answer": "2000\ub144 5\uc6d4 12\uc77c \uac1c\uad00\ud55c \ud14c\uc774\ud2b8 \ubaa8\ub358\uc740 \uc601\uad6d \uc815\ubd80\uc758 \ubc00\ub808\ub2c8\uc5c4 \ud504\ub85c\uc81d\ud2b8\uc758 \uc77c\ud658\uc73c\ub85c \ud15c\uc988\uac15\ubcc0\uc758 \ubc45\ud06c\uc0ac\uc774\ub4dc(Bankside) \ubc1c\uc804\uc18c\ub97c \uc0c8\ub86d\uac8c \ub9ac\ubaa8\ub378\ub9c1\ud55c \uacf3\uc5d0 \ub4e4\uc5b4\uc130\ub2e4. \ubc45\ud06c\uc0ac\uc774\ub4dc \ubc1c\uc804\uc18c\ub294 2\ucc28 \uc138\uacc4\ub300\uc804 \uc9c1\ud6c4 \ub7f0\ub358 \uc911\uc2ec\ubd80\uc5d0 \uc804\ub825\uc744 \uacf5\uae09\ud558\uae30 \uc704\ud574 \uc138\uc6cc\uc84c\ub358 \ud654\ub825\ubc1c\uc804\uc18c\ub85c \uc601\uad6d\uc758 \ube68\uac04 \uacf5\uc911\uc804\ud654 \ubc15\uc2a4 \ub514\uc790\uc778\uc73c\ub85c\ub3c4 \uc720\uba85\ud55c \uac74\ucd95\uac00 \uae38\ubc84\ud2b8 \uc2a4\ucf54\ud2b8(Giles Gilbert Scott)\uc5d0 \uc758\ud574 \uc9c0\uc5b4\uc84c\uc73c\uba70 \uacf5\ud574\ubb38\uc81c \ub85c \uc774\uc804\ud55c \uc774\ud6c4 1981\ub144 \ubb38\uc744 \ub2eb\uc740 \uc0c1\ud0dc\uc600\ub2e4. \uc601\uad6d \uc815\ubd80\uc640 \ud14c\uc774\ud2b8 \uc7ac\ub2e8\uc740 \ud15c\uc988\uac15\ubcc0\uc5d0 \uc790\ub9ac\ud558\uace0 \uc788\uc73c\uba74\uc11c \ub113\uc740 \uac74\ubb3c\uba74\uc801\uacfc \uc9c0\ud558\ucca0\uc5ed\uc5d0\uc11c\ub3c4 \uac00\uae4c\uc6b4 \uc774 \ubc1c\uc804\uc18c\ub97c \ud604\ub300\ubbf8\uc220\uad00\uc744 \uc9c0\uc744 \uc7a5\uc18c\ub85c \ub099\uc810\ud558\uace0 \uad6d\uc81c \uac74\ucd95 \uacf5\ubaa8\uc804\uc744 \ud1b5\ud574 \uc120\uc815\ub41c \uc2a4\uc704\uc2a4 \uac74\ucd95\ud68c\uc0ac \ud5e4\ub974\ucd08\ud06c & \ub4dc \ubafc\ub871(Herzog\uc640 de Meuron)\uc774 \ud14c\uc774\ud2b8\ubaa8\ub358 \ud504\ub85c\uc81d\ud2b8\uc5d0 \ucc38\uc5ec\ud558\uac8c \ub41c\ub2e4. \uc57d 8\ub144\uc5ec \uac04\uc758 \uacf5\uc0ac\uae30\uac04 \ub05d\uc5d0 \uc9c0\uc5b4\uc9c4 \ubcf8 \uac74\ubb3c\uc740 \uae30\uc874\uc758 \uc678\uad00\uc740 \ucd5c\ub300\ud55c \uc190\ub300\uc9c0 \uc54a\uace0 \ub0b4\ubd80\ub294 \ubbf8\uc220\uad00\uc758 \uae30\ub2a5\uc5d0 \ub9de\ucdb0 \uc644\uc804\ud788 \uc0c8\ub85c\uc6b4 \uad6c\uc870\ub85c \ubc14\uafb8\ub294 \ubc29\uc2dd\uc73c\ub85c \uac1c\uc870\ub418\uc5c8\ub2e4. \ucd1d \ub192\uc774 99m\uc9c1\uc721\uba74\uccb4 \uc678\ud615\uc758 \uc6c5\uc7a5\ud55c \ud14c\uc774\ud2b8 \ubaa8\ub358\uc740 \ubaa8\ub450 7\uce35\uc73c\ub85c \uad6c\uc131\ub418\uc5b4 \uc788\uc73c\uba70 \uac74\ubb3c \ud55c\uac00\uc6b4\ub370 \uc6d0\ub798 \ubc1c\uc804\uc18c\uc6a9\uc73c\ub85c \uc0ac\uc6a9\ud558\ub358 \ub192\uc774 99m\uc758 \uad74\ub69d\uc774 \uadf8\ub300\ub85c \uc19f\uc544 \uc788\ub294\ub370, \ubc18\ud22c\uba85 \ud328\ub110\uc744 \uc0ac\uc6a9\ud558\uc5ec \ubc24\uc774\uba74 \ub4f1\ub300\ucc98\ub7fc \ube5b\uc744 \ub0b4\ub3c4\ub85d \uac1c\uc870\ud558\uc5ec \uc774 \uad74\ub69d\uc740 \uc624\ub298\ub0a0 \ud14c\uc774\ud2b8 \ubaa8\ub358\uc758 \uc0c1\uc9d5\uc774 \ub418\uc5c8\ub2e4. \ubbf8\uc220\uad00 \uac74\ubb3c \uc790\uccb4\ub9cc\uc73c\ub85c\ub3c4 \ubcfc \uac70\ub9ac\uac00 \ub41c \ud14c\uc774\ud2b8 \ubaa8\ub358\uc740 \ud55c\ud574 400\ub9cc\uba85 \uc774\uc0c1\uc758 \uad00\uad11\uac1d\uc774 \ucc3e\ub294 \ub7f0\ub358\uc758 \uc0c8\ub85c\uc6b4 \uad00\uad11\uba85\uc18c\uac00 \ub418\uc5c8\ub2e4.", "sentence_answer": "\ubc45\ud06c\uc0ac\uc774\ub4dc \ubc1c\uc804\uc18c\ub294 2\ucc28 \uc138\uacc4\ub300\uc804 \uc9c1\ud6c4 \ub7f0\ub358 \uc911\uc2ec\ubd80\uc5d0 \uc804\ub825\uc744 \uacf5\uae09\ud558\uae30 \uc704\ud574 \uc138\uc6cc\uc84c\ub358 \ud654\ub825\ubc1c\uc804\uc18c\ub85c \uc601\uad6d\uc758 \ube68\uac04 \uacf5\uc911\uc804\ud654 \ubc15\uc2a4 \ub514\uc790\uc778\uc73c\ub85c\ub3c4 \uc720\uba85\ud55c \uac74\ucd95\uac00 \uae38\ubc84\ud2b8 \uc2a4\ucf54\ud2b8(Giles Gilbert Scott)\uc5d0 \uc758\ud574 \uc9c0\uc5b4\uc84c\uc73c\uba70 \uacf5\ud574\ubb38\uc81c \ub85c \uc774\uc804\ud55c \uc774\ud6c4 1981\ub144 \ubb38\uc744 \ub2eb\uc740 \uc0c1\ud0dc\uc600\ub2e4.", "paragraph_id": "6565931-0-0", "paragraph_question": "question: \ubc45\ud06c\uc0ac\uc774\ub4dc \ubc1c\uc804\uc18c\ub294 \uc5b4\ub5a4 \ubb38\uc81c\ub85c 1981\ub144 \ubb38\uc744 \ub2eb\uc558\ub098?, context: 2000\ub144 5\uc6d4 12\uc77c \uac1c\uad00\ud55c \ud14c\uc774\ud2b8 \ubaa8\ub358\uc740 \uc601\uad6d \uc815\ubd80\uc758 \ubc00\ub808\ub2c8\uc5c4 \ud504\ub85c\uc81d\ud2b8\uc758 \uc77c\ud658\uc73c\ub85c \ud15c\uc988\uac15\ubcc0\uc758 \ubc45\ud06c\uc0ac\uc774\ub4dc(Bankside) \ubc1c\uc804\uc18c\ub97c \uc0c8\ub86d\uac8c \ub9ac\ubaa8\ub378\ub9c1\ud55c \uacf3\uc5d0 \ub4e4\uc5b4\uc130\ub2e4. \ubc45\ud06c\uc0ac\uc774\ub4dc \ubc1c\uc804\uc18c\ub294 2\ucc28 \uc138\uacc4\ub300\uc804 \uc9c1\ud6c4 \ub7f0\ub358 \uc911\uc2ec\ubd80\uc5d0 \uc804\ub825\uc744 \uacf5\uae09\ud558\uae30 \uc704\ud574 \uc138\uc6cc\uc84c\ub358 \ud654\ub825\ubc1c\uc804\uc18c\ub85c \uc601\uad6d\uc758 \ube68\uac04 \uacf5\uc911\uc804\ud654 \ubc15\uc2a4 \ub514\uc790\uc778\uc73c\ub85c\ub3c4 \uc720\uba85\ud55c \uac74\ucd95\uac00 \uae38\ubc84\ud2b8 \uc2a4\ucf54\ud2b8(Giles Gilbert Scott)\uc5d0 \uc758\ud574 \uc9c0\uc5b4\uc84c\uc73c\uba70 \uacf5\ud574\ubb38\uc81c\ub85c \uc774\uc804\ud55c \uc774\ud6c4 1981\ub144 \ubb38\uc744 \ub2eb\uc740 \uc0c1\ud0dc\uc600\ub2e4. \uc601\uad6d \uc815\ubd80\uc640 \ud14c\uc774\ud2b8 \uc7ac\ub2e8\uc740 \ud15c\uc988\uac15\ubcc0\uc5d0 \uc790\ub9ac\ud558\uace0 \uc788\uc73c\uba74\uc11c \ub113\uc740 \uac74\ubb3c\uba74\uc801\uacfc \uc9c0\ud558\ucca0\uc5ed\uc5d0\uc11c\ub3c4 \uac00\uae4c\uc6b4 \uc774 \ubc1c\uc804\uc18c\ub97c \ud604\ub300\ubbf8\uc220\uad00\uc744 \uc9c0\uc744 \uc7a5\uc18c\ub85c \ub099\uc810\ud558\uace0 \uad6d\uc81c \uac74\ucd95 \uacf5\ubaa8\uc804\uc744 \ud1b5\ud574 \uc120\uc815\ub41c \uc2a4\uc704\uc2a4 \uac74\ucd95\ud68c\uc0ac \ud5e4\ub974\ucd08\ud06c & \ub4dc \ubafc\ub871(Herzog\uc640 de Meuron)\uc774 \ud14c\uc774\ud2b8\ubaa8\ub358 \ud504\ub85c\uc81d\ud2b8\uc5d0 \ucc38\uc5ec\ud558\uac8c \ub41c\ub2e4. \uc57d 8\ub144\uc5ec \uac04\uc758 \uacf5\uc0ac\uae30\uac04 \ub05d\uc5d0 \uc9c0\uc5b4\uc9c4 \ubcf8 \uac74\ubb3c\uc740 \uae30\uc874\uc758 \uc678\uad00\uc740 \ucd5c\ub300\ud55c \uc190\ub300\uc9c0 \uc54a\uace0 \ub0b4\ubd80\ub294 \ubbf8\uc220\uad00\uc758 \uae30\ub2a5\uc5d0 \ub9de\ucdb0 \uc644\uc804\ud788 \uc0c8\ub85c\uc6b4 \uad6c\uc870\ub85c \ubc14\uafb8\ub294 \ubc29\uc2dd\uc73c\ub85c \uac1c\uc870\ub418\uc5c8\ub2e4. \ucd1d \ub192\uc774 99m\uc9c1\uc721\uba74\uccb4 \uc678\ud615\uc758 \uc6c5\uc7a5\ud55c \ud14c\uc774\ud2b8 \ubaa8\ub358\uc740 \ubaa8\ub450 7\uce35\uc73c\ub85c \uad6c\uc131\ub418\uc5b4 \uc788\uc73c\uba70 \uac74\ubb3c \ud55c\uac00\uc6b4\ub370 \uc6d0\ub798 \ubc1c\uc804\uc18c\uc6a9\uc73c\ub85c \uc0ac\uc6a9\ud558\ub358 \ub192\uc774 99m\uc758 \uad74\ub69d\uc774 \uadf8\ub300\ub85c \uc19f\uc544 \uc788\ub294\ub370, \ubc18\ud22c\uba85 \ud328\ub110\uc744 \uc0ac\uc6a9\ud558\uc5ec \ubc24\uc774\uba74 \ub4f1\ub300\ucc98\ub7fc \ube5b\uc744 \ub0b4\ub3c4\ub85d \uac1c\uc870\ud558\uc5ec \uc774 \uad74\ub69d\uc740 \uc624\ub298\ub0a0 \ud14c\uc774\ud2b8 \ubaa8\ub358\uc758 \uc0c1\uc9d5\uc774 \ub418\uc5c8\ub2e4. \ubbf8\uc220\uad00 \uac74\ubb3c \uc790\uccb4\ub9cc\uc73c\ub85c\ub3c4 \ubcfc \uac70\ub9ac\uac00 \ub41c \ud14c\uc774\ud2b8 \ubaa8\ub358\uc740 \ud55c\ud574 400\ub9cc\uba85 \uc774\uc0c1\uc758 \uad00\uad11\uac1d\uc774 \ucc3e\ub294 \ub7f0\ub358\uc758 \uc0c8\ub85c\uc6b4 \uad00\uad11\uba85\uc18c\uac00 \ub418\uc5c8\ub2e4."} {"question": "\uc7a5\uc774\uba38\uc6b0\uac00 \uc911\ud559\uc0dd \ub54c \uac00\ub2f4\ud55c \uac83\uc740 \ubb34\uc5c7\uc778\uac00?", "paragraph": "\uc7a5\uc774\uba38\uc6b0\ub294 1950\ub144 11\uc6d4 14\uc77c \uc911\uad6d \uc2dc\uc548(\u897f\u5b89)\uc5d0\uc11c \ud0dc\uc5b4\ub0ac\ub2e4. \uc7a5\uc774\uba38\uc6b0\uc758 \uc131\uc7a5\uae30\ub294 \uc21c\ud0c4\ud558\uc9c0 \uc54a\uc558\ub2e4. \uad6d\uacf5 \ub0b4\uc804 \ub2f9\uc2dc \uc911\uad6d \uad6d\ubbfc\ub2f9 \uc18c\uc18d \uc7a5\uad50\uc600\ub358 \uc544\ubc84\uc9c0\ub294 \uc911\uad6d \uacf5\uc0b0\ub2f9\uc774 \uc911\ud654\uc778\ubbfc\uacf5\ud654\uad6d\uc744 \uc138\uc6b0\uba74\uc11c \ud0c0\uc774\uc644\uc73c\ub85c \ub3c4\ud53c\ud558\uc600\ub2e4. \uc5b4\uba38\ub2c8 \ub610\ud55c \uc758\uc0ac\uc600\uae30\uc5d0 \uadf8\ub85c \uc778\ud558\uc5ec \uc7a5\uc774\uba38\uc6b0\uc758 \uac00\uc871\uc740 \ubb38\ud654\ud601\uba85 \uae30\uac04\ub3d9\uc548 \ube44\ud310\uc744 \ubc1b\uace0 \ubca0\uc774\uc9d5\uc5d0\uc11c \uba40\ub9ac \uc88c\ucc9c\ub418\uc5c8\ub2e4. \uc0ac\ud68c\uc801\uc73c\ub85c \ubd88\uc548\ud558\uc600\ub358 \ubb38\ud654\ud601\uba85(1965~1969)\uc2dc\uae30, \uc911\ud559\uc0dd\uc774\uc5c8\ub358 \uc7a5\uc774\uba38\uc6b0\ub294 \uacf5\ubd80\ub97c \uc911\uc9c0\ud558\ub3c4\ub85d \uac15\uc694\ubc1b\uc558\uace0, \ud64d\uc704\ubcd1 \uc0dd\ud65c\uc5d0 \uac00\ub2f4\ud558\uace0 \uc2dc\uace8\uc5d0\uc11c \uc77c\uc744 \ud558\uae30\ub3c4 \ud558\uc600\ub2e4. \ubb38\ud654\ud601\uba85\uc774 \ub05d\ub0a0 \ubb34\ub835\uc778 1968\ub144\uc5d0\ub294 \uc7a5\uc774\uba38\uc6b0\ub294 \uc7ac\uad50\uc721\uc744 \uc704\ud574 3\ub144\ub3d9\uc548 \uc0e8\uc2dc \uc9c0\ubc29\uc758 \ub18d\uc7a5\uc5d0\uc11c \uc77c\uc744 \ud558\uc600\ub2e4. \uadf8 \ub4a4\ub85c\ub294 1978\ub144\uae4c\uc9c0 \ubc29\uc9c1 \uacf5\uc7a5\uc5d0\uc11c \ub178\ub3d9\uc790\ub85c\uc11c 7\ub144\uac04 \uc77c\uc744 \ud558\uc600\ub2e4. \ubc29\uc9c1 \uacf5\uc7a5\uc5d0\uc11c 7\ub144\ub3d9\uc548 \uc7a5\uc774\uba38\uc6b0\ub294 \ub9c8\uc624\uc758 \ucd08\uc0c1\ud654\ub97c \uadf8\ub9ac\uae30\ub3c4 \ud558\uba70 \uc791\uac00\ub85c\uc11c\uc758 \uadf8\uc758 \uc7ac\ub2a5\uc744 \ud0a4\uc6cc\ub098\uac14\uace0 \uc0ac\uc9c4\uc5d0 \ub300\ud55c \uad00\uc2ec\uc744 \uac16\uac8c \ub418\uc5c8\ub2e4. \uc7a5\uc774\uba38\uc6b0\uac00 \uccab \uce74\uba54\ub77c\ub97c \uc0ac\uae30 \uc704\ud574 \uc790\uc2e0\uc758 \ud53c\ub97c \ud314\uc544\uc11c 5\uac1c\uc6d4 \uac04 \ub3c8\uc744 \ubaa8\uc73c\uae30\ub3c4 \ud558\uc600\ub2e4. \uc7a5\uc774\uba38\uc6b0\ub294 \ubb38\ud654\ud601\uba85 \uc774\uc804\uc5d0 \ucd9c\ud310\ub41c \ucc45\ub4e4\uc744 \ubcf4\uace0 \ud544\ub984\uacfc \uc0ac\uc9c4\uc740 \uac04\uc774 \uc554\uc2e4\uc5d0\uc11c \ud604\uc0c1\ud558\uba74\uc11c \uc0ac\uc9c4\uc220\uc744 \ub3c5\ud559\ud558\uc600\ub2e4. \ub2f9\uc2dc\uc758 \uc7a5\uc774\uba38\uc6b0\uc758 \uc0ac\uc9c4\ub4e4\uc740 \uc2e0\ubb38\uc5d0 \uc2e4\ub9ac\uae30\ub3c4 \ud558\uc600\ub2e4.", "answer": "\ud64d\uc704\ubcd1", "sentence": "\uc0ac\ud68c\uc801\uc73c\ub85c \ubd88\uc548\ud558\uc600\ub358 \ubb38\ud654\ud601\uba85(1965~1969)\uc2dc\uae30, \uc911\ud559\uc0dd\uc774\uc5c8\ub358 \uc7a5\uc774\uba38\uc6b0\ub294 \uacf5\ubd80\ub97c \uc911\uc9c0\ud558\ub3c4\ub85d \uac15\uc694\ubc1b\uc558\uace0, \ud64d\uc704\ubcd1 \uc0dd\ud65c\uc5d0 \uac00\ub2f4\ud558\uace0 \uc2dc\uace8\uc5d0\uc11c \uc77c\uc744 \ud558\uae30\ub3c4 \ud558\uc600\ub2e4.", "paragraph_sentence": "\uc7a5\uc774\uba38\uc6b0\ub294 1950\ub144 11\uc6d4 14\uc77c \uc911\uad6d \uc2dc\uc548(\u897f\u5b89)\uc5d0\uc11c \ud0dc\uc5b4\ub0ac\ub2e4. \uc7a5\uc774\uba38\uc6b0\uc758 \uc131\uc7a5\uae30\ub294 \uc21c\ud0c4\ud558\uc9c0 \uc54a\uc558\ub2e4. \uad6d\uacf5 \ub0b4\uc804 \ub2f9\uc2dc \uc911\uad6d \uad6d\ubbfc\ub2f9 \uc18c\uc18d \uc7a5\uad50\uc600\ub358 \uc544\ubc84\uc9c0\ub294 \uc911\uad6d \uacf5\uc0b0\ub2f9\uc774 \uc911\ud654\uc778\ubbfc\uacf5\ud654\uad6d\uc744 \uc138\uc6b0\uba74\uc11c \ud0c0\uc774\uc644\uc73c\ub85c \ub3c4\ud53c\ud558\uc600\ub2e4. \uc5b4\uba38\ub2c8 \ub610\ud55c \uc758\uc0ac\uc600\uae30\uc5d0 \uadf8\ub85c \uc778\ud558\uc5ec \uc7a5\uc774\uba38\uc6b0\uc758 \uac00\uc871\uc740 \ubb38\ud654\ud601\uba85 \uae30\uac04\ub3d9\uc548 \ube44\ud310\uc744 \ubc1b\uace0 \ubca0\uc774\uc9d5\uc5d0\uc11c \uba40\ub9ac \uc88c\ucc9c\ub418\uc5c8\ub2e4. \uc0ac\ud68c\uc801\uc73c\ub85c \ubd88\uc548\ud558\uc600\ub358 \ubb38\ud654\ud601\uba85(1965~1969)\uc2dc\uae30, \uc911\ud559\uc0dd\uc774\uc5c8\ub358 \uc7a5\uc774\uba38\uc6b0\ub294 \uacf5\ubd80\ub97c \uc911\uc9c0\ud558\ub3c4\ub85d \uac15\uc694\ubc1b\uc558\uace0, \ud64d\uc704\ubcd1 \uc0dd\ud65c\uc5d0 \uac00\ub2f4\ud558\uace0 \uc2dc\uace8\uc5d0\uc11c \uc77c\uc744 \ud558\uae30\ub3c4 \ud558\uc600\ub2e4. \ubb38\ud654\ud601\uba85\uc774 \ub05d\ub0a0 \ubb34\ub835\uc778 1968\ub144\uc5d0\ub294 \uc7a5\uc774\uba38\uc6b0\ub294 \uc7ac\uad50\uc721\uc744 \uc704\ud574 3\ub144\ub3d9\uc548 \uc0e8\uc2dc \uc9c0\ubc29\uc758 \ub18d\uc7a5\uc5d0\uc11c \uc77c\uc744 \ud558\uc600\ub2e4. \uadf8 \ub4a4\ub85c\ub294 1978\ub144\uae4c\uc9c0 \ubc29\uc9c1 \uacf5\uc7a5\uc5d0\uc11c \ub178\ub3d9\uc790\ub85c\uc11c 7\ub144\uac04 \uc77c\uc744 \ud558\uc600\ub2e4. \ubc29\uc9c1 \uacf5\uc7a5\uc5d0\uc11c 7\ub144\ub3d9\uc548 \uc7a5\uc774\uba38\uc6b0\ub294 \ub9c8\uc624\uc758 \ucd08\uc0c1\ud654\ub97c \uadf8\ub9ac\uae30\ub3c4 \ud558\uba70 \uc791\uac00\ub85c\uc11c\uc758 \uadf8\uc758 \uc7ac\ub2a5\uc744 \ud0a4\uc6cc\ub098\uac14\uace0 \uc0ac\uc9c4\uc5d0 \ub300\ud55c \uad00\uc2ec\uc744 \uac16\uac8c \ub418\uc5c8\ub2e4. \uc7a5\uc774\uba38\uc6b0\uac00 \uccab \uce74\uba54\ub77c\ub97c \uc0ac\uae30 \uc704\ud574 \uc790\uc2e0\uc758 \ud53c\ub97c \ud314\uc544\uc11c 5\uac1c\uc6d4 \uac04 \ub3c8\uc744 \ubaa8\uc73c\uae30\ub3c4 \ud558\uc600\ub2e4. \uc7a5\uc774\uba38\uc6b0\ub294 \ubb38\ud654\ud601\uba85 \uc774\uc804\uc5d0 \ucd9c\ud310\ub41c \ucc45\ub4e4\uc744 \ubcf4\uace0 \ud544\ub984\uacfc \uc0ac\uc9c4\uc740 \uac04\uc774 \uc554\uc2e4\uc5d0\uc11c \ud604\uc0c1\ud558\uba74\uc11c \uc0ac\uc9c4\uc220\uc744 \ub3c5\ud559\ud558\uc600\ub2e4. \ub2f9\uc2dc\uc758 \uc7a5\uc774\uba38\uc6b0\uc758 \uc0ac\uc9c4\ub4e4\uc740 \uc2e0\ubb38\uc5d0 \uc2e4\ub9ac\uae30\ub3c4 \ud558\uc600\ub2e4.", "paragraph_answer": "\uc7a5\uc774\uba38\uc6b0\ub294 1950\ub144 11\uc6d4 14\uc77c \uc911\uad6d \uc2dc\uc548(\u897f\u5b89)\uc5d0\uc11c \ud0dc\uc5b4\ub0ac\ub2e4. \uc7a5\uc774\uba38\uc6b0\uc758 \uc131\uc7a5\uae30\ub294 \uc21c\ud0c4\ud558\uc9c0 \uc54a\uc558\ub2e4. \uad6d\uacf5 \ub0b4\uc804 \ub2f9\uc2dc \uc911\uad6d \uad6d\ubbfc\ub2f9 \uc18c\uc18d \uc7a5\uad50\uc600\ub358 \uc544\ubc84\uc9c0\ub294 \uc911\uad6d \uacf5\uc0b0\ub2f9\uc774 \uc911\ud654\uc778\ubbfc\uacf5\ud654\uad6d\uc744 \uc138\uc6b0\uba74\uc11c \ud0c0\uc774\uc644\uc73c\ub85c \ub3c4\ud53c\ud558\uc600\ub2e4. \uc5b4\uba38\ub2c8 \ub610\ud55c \uc758\uc0ac\uc600\uae30\uc5d0 \uadf8\ub85c \uc778\ud558\uc5ec \uc7a5\uc774\uba38\uc6b0\uc758 \uac00\uc871\uc740 \ubb38\ud654\ud601\uba85 \uae30\uac04\ub3d9\uc548 \ube44\ud310\uc744 \ubc1b\uace0 \ubca0\uc774\uc9d5\uc5d0\uc11c \uba40\ub9ac \uc88c\ucc9c\ub418\uc5c8\ub2e4. \uc0ac\ud68c\uc801\uc73c\ub85c \ubd88\uc548\ud558\uc600\ub358 \ubb38\ud654\ud601\uba85(1965~1969)\uc2dc\uae30, \uc911\ud559\uc0dd\uc774\uc5c8\ub358 \uc7a5\uc774\uba38\uc6b0\ub294 \uacf5\ubd80\ub97c \uc911\uc9c0\ud558\ub3c4\ub85d \uac15\uc694\ubc1b\uc558\uace0, \ud64d\uc704\ubcd1 \uc0dd\ud65c\uc5d0 \uac00\ub2f4\ud558\uace0 \uc2dc\uace8\uc5d0\uc11c \uc77c\uc744 \ud558\uae30\ub3c4 \ud558\uc600\ub2e4. \ubb38\ud654\ud601\uba85\uc774 \ub05d\ub0a0 \ubb34\ub835\uc778 1968\ub144\uc5d0\ub294 \uc7a5\uc774\uba38\uc6b0\ub294 \uc7ac\uad50\uc721\uc744 \uc704\ud574 3\ub144\ub3d9\uc548 \uc0e8\uc2dc \uc9c0\ubc29\uc758 \ub18d\uc7a5\uc5d0\uc11c \uc77c\uc744 \ud558\uc600\ub2e4. \uadf8 \ub4a4\ub85c\ub294 1978\ub144\uae4c\uc9c0 \ubc29\uc9c1 \uacf5\uc7a5\uc5d0\uc11c \ub178\ub3d9\uc790\ub85c\uc11c 7\ub144\uac04 \uc77c\uc744 \ud558\uc600\ub2e4. \ubc29\uc9c1 \uacf5\uc7a5\uc5d0\uc11c 7\ub144\ub3d9\uc548 \uc7a5\uc774\uba38\uc6b0\ub294 \ub9c8\uc624\uc758 \ucd08\uc0c1\ud654\ub97c \uadf8\ub9ac\uae30\ub3c4 \ud558\uba70 \uc791\uac00\ub85c\uc11c\uc758 \uadf8\uc758 \uc7ac\ub2a5\uc744 \ud0a4\uc6cc\ub098\uac14\uace0 \uc0ac\uc9c4\uc5d0 \ub300\ud55c \uad00\uc2ec\uc744 \uac16\uac8c \ub418\uc5c8\ub2e4. \uc7a5\uc774\uba38\uc6b0\uac00 \uccab \uce74\uba54\ub77c\ub97c \uc0ac\uae30 \uc704\ud574 \uc790\uc2e0\uc758 \ud53c\ub97c \ud314\uc544\uc11c 5\uac1c\uc6d4 \uac04 \ub3c8\uc744 \ubaa8\uc73c\uae30\ub3c4 \ud558\uc600\ub2e4. \uc7a5\uc774\uba38\uc6b0\ub294 \ubb38\ud654\ud601\uba85 \uc774\uc804\uc5d0 \ucd9c\ud310\ub41c \ucc45\ub4e4\uc744 \ubcf4\uace0 \ud544\ub984\uacfc \uc0ac\uc9c4\uc740 \uac04\uc774 \uc554\uc2e4\uc5d0\uc11c \ud604\uc0c1\ud558\uba74\uc11c \uc0ac\uc9c4\uc220\uc744 \ub3c5\ud559\ud558\uc600\ub2e4. \ub2f9\uc2dc\uc758 \uc7a5\uc774\uba38\uc6b0\uc758 \uc0ac\uc9c4\ub4e4\uc740 \uc2e0\ubb38\uc5d0 \uc2e4\ub9ac\uae30\ub3c4 \ud558\uc600\ub2e4.", "sentence_answer": "\uc0ac\ud68c\uc801\uc73c\ub85c \ubd88\uc548\ud558\uc600\ub358 \ubb38\ud654\ud601\uba85(1965~1969)\uc2dc\uae30, \uc911\ud559\uc0dd\uc774\uc5c8\ub358 \uc7a5\uc774\uba38\uc6b0\ub294 \uacf5\ubd80\ub97c \uc911\uc9c0\ud558\ub3c4\ub85d \uac15\uc694\ubc1b\uc558\uace0, \ud64d\uc704\ubcd1 \uc0dd\ud65c\uc5d0 \uac00\ub2f4\ud558\uace0 \uc2dc\uace8\uc5d0\uc11c \uc77c\uc744 \ud558\uae30\ub3c4 \ud558\uc600\ub2e4.", "paragraph_id": "5858434-2-1", "paragraph_question": "question: \uc7a5\uc774\uba38\uc6b0\uac00 \uc911\ud559\uc0dd \ub54c \uac00\ub2f4\ud55c \uac83\uc740 \ubb34\uc5c7\uc778\uac00?, context: \uc7a5\uc774\uba38\uc6b0\ub294 1950\ub144 11\uc6d4 14\uc77c \uc911\uad6d \uc2dc\uc548(\u897f\u5b89)\uc5d0\uc11c \ud0dc\uc5b4\ub0ac\ub2e4. \uc7a5\uc774\uba38\uc6b0\uc758 \uc131\uc7a5\uae30\ub294 \uc21c\ud0c4\ud558\uc9c0 \uc54a\uc558\ub2e4. \uad6d\uacf5 \ub0b4\uc804 \ub2f9\uc2dc \uc911\uad6d \uad6d\ubbfc\ub2f9 \uc18c\uc18d \uc7a5\uad50\uc600\ub358 \uc544\ubc84\uc9c0\ub294 \uc911\uad6d \uacf5\uc0b0\ub2f9\uc774 \uc911\ud654\uc778\ubbfc\uacf5\ud654\uad6d\uc744 \uc138\uc6b0\uba74\uc11c \ud0c0\uc774\uc644\uc73c\ub85c \ub3c4\ud53c\ud558\uc600\ub2e4. \uc5b4\uba38\ub2c8 \ub610\ud55c \uc758\uc0ac\uc600\uae30\uc5d0 \uadf8\ub85c \uc778\ud558\uc5ec \uc7a5\uc774\uba38\uc6b0\uc758 \uac00\uc871\uc740 \ubb38\ud654\ud601\uba85 \uae30\uac04\ub3d9\uc548 \ube44\ud310\uc744 \ubc1b\uace0 \ubca0\uc774\uc9d5\uc5d0\uc11c \uba40\ub9ac \uc88c\ucc9c\ub418\uc5c8\ub2e4. \uc0ac\ud68c\uc801\uc73c\ub85c \ubd88\uc548\ud558\uc600\ub358 \ubb38\ud654\ud601\uba85(1965~1969)\uc2dc\uae30, \uc911\ud559\uc0dd\uc774\uc5c8\ub358 \uc7a5\uc774\uba38\uc6b0\ub294 \uacf5\ubd80\ub97c \uc911\uc9c0\ud558\ub3c4\ub85d \uac15\uc694\ubc1b\uc558\uace0, \ud64d\uc704\ubcd1 \uc0dd\ud65c\uc5d0 \uac00\ub2f4\ud558\uace0 \uc2dc\uace8\uc5d0\uc11c \uc77c\uc744 \ud558\uae30\ub3c4 \ud558\uc600\ub2e4. \ubb38\ud654\ud601\uba85\uc774 \ub05d\ub0a0 \ubb34\ub835\uc778 1968\ub144\uc5d0\ub294 \uc7a5\uc774\uba38\uc6b0\ub294 \uc7ac\uad50\uc721\uc744 \uc704\ud574 3\ub144\ub3d9\uc548 \uc0e8\uc2dc \uc9c0\ubc29\uc758 \ub18d\uc7a5\uc5d0\uc11c \uc77c\uc744 \ud558\uc600\ub2e4. \uadf8 \ub4a4\ub85c\ub294 1978\ub144\uae4c\uc9c0 \ubc29\uc9c1 \uacf5\uc7a5\uc5d0\uc11c \ub178\ub3d9\uc790\ub85c\uc11c 7\ub144\uac04 \uc77c\uc744 \ud558\uc600\ub2e4. \ubc29\uc9c1 \uacf5\uc7a5\uc5d0\uc11c 7\ub144\ub3d9\uc548 \uc7a5\uc774\uba38\uc6b0\ub294 \ub9c8\uc624\uc758 \ucd08\uc0c1\ud654\ub97c \uadf8\ub9ac\uae30\ub3c4 \ud558\uba70 \uc791\uac00\ub85c\uc11c\uc758 \uadf8\uc758 \uc7ac\ub2a5\uc744 \ud0a4\uc6cc\ub098\uac14\uace0 \uc0ac\uc9c4\uc5d0 \ub300\ud55c \uad00\uc2ec\uc744 \uac16\uac8c \ub418\uc5c8\ub2e4. \uc7a5\uc774\uba38\uc6b0\uac00 \uccab \uce74\uba54\ub77c\ub97c \uc0ac\uae30 \uc704\ud574 \uc790\uc2e0\uc758 \ud53c\ub97c \ud314\uc544\uc11c 5\uac1c\uc6d4 \uac04 \ub3c8\uc744 \ubaa8\uc73c\uae30\ub3c4 \ud558\uc600\ub2e4. \uc7a5\uc774\uba38\uc6b0\ub294 \ubb38\ud654\ud601\uba85 \uc774\uc804\uc5d0 \ucd9c\ud310\ub41c \ucc45\ub4e4\uc744 \ubcf4\uace0 \ud544\ub984\uacfc \uc0ac\uc9c4\uc740 \uac04\uc774 \uc554\uc2e4\uc5d0\uc11c \ud604\uc0c1\ud558\uba74\uc11c \uc0ac\uc9c4\uc220\uc744 \ub3c5\ud559\ud558\uc600\ub2e4. \ub2f9\uc2dc\uc758 \uc7a5\uc774\uba38\uc6b0\uc758 \uc0ac\uc9c4\ub4e4\uc740 \uc2e0\ubb38\uc5d0 \uc2e4\ub9ac\uae30\ub3c4 \ud558\uc600\ub2e4."} {"question": "\ub300\ud1b5\ub839\uc740 \uad6d\ubbfc\uc758 \uba38\uc2b4\uc774\uc790 \ub300\ub9ac\uc778\uc774\ub2e4\uace0 \ubc1c\uc5b8\ud55c \uc131\ub0a8\uc2dc\uc7a5\uc740 \ub204\uad6c\uc778\uac00?", "paragraph": "\uc774\ud6c4 \uc815\uce58\uc778 \ub300\ud45c\ub85c\uc11c \ub178\ud68c\ucc2c \uc815\uc758\ub2f9 \uc6d0\ub0b4\ub300\ud45c, \uae40\uc885\ud6c8 \ubb34\uc18c\uc18d \uc758\uc6d0, \uc774\uc7ac\uba85 \uc131\ub0a8\uc2dc\uc7a5\ub3c4 \ubc1c\uc5b8\uc5d0 \ub098\uc130\ub2e4. \ub178\ud68c\ucc2c \ub300\ud45c\ub294 \"\uc9c0\ub09c 3\ub144 8\uac1c\uc6d4\uac04 \ubd80\uc815\uc744 \uc800\uc9c0\ub978 \ubc15 \ub300\ud1b5\ub839\uc740 \uc5b4\ub5bb\uac8c \ud574\uc57c \ud558\ub290\ub0d0\"\uba70 \"\uc218\uc0ac\uac00 \ubc15 \ub300\ud1b5\ub839\uacfc \ucd5c\uc21c\uc2e4\uc5d0\uac8c \uacf5\uc720\ub418\uace0 \uc788\ub294\ub370 \ub9d0\uc774 \ub418\ub290\ub0d0, \ubc15 \ub300\ud1b5\ub839\uc774 \uc788\ub294 \ud55c \uc9c4\uc2e4\uaddc\uba85\uc774 \uc548 \ub41c\ub2e4. \ubc15 \ub300\ud1b5\ub839\uc774 \ub5a0\ub098\uc57c \uc6b0\ub9ac \uad6d\ubbfc\ub4e4\uc774 \ub2e4\uc2dc \ud798\uc744 \uc5bb\uc744 \uc218 \uc788\ub2e4\u201c\uace0 \uc8fc\uc7a5\ud558\uc600\ub2e4. \uae40\uc885\ud6c8 \ubb34\uc18c\uc18d \uc758\uc6d0\uc740 \"\uc9c0\uc5ed\uad6c\uc778 \uc6b8\uc0b0\uc5d0\uc11c \uc8fc\ubbfc\ub4e4\uc774 \uc11c\uc6b8\uc5d0 \uc62c\ub77c\uac00\ub294 \uc800\uc5d0\uac8c '\ub300\ud55c\ubbfc\uad6d\uc774 \ubd80\ub044\ub7fd\ub2e4', '\uad6d\ubbfc\ub4e4\uc744 \ubd80\ub044\ub7fd\uac8c \ub9cc\ub4e4\uace0 \uc788\ub2e4'\ub77c\ub294 \ub9d0\uc744 \uc804\ud574\ub2ec\ub77c\uace0 \ud588\ub2e4\"\uba74\uc11c, \"\uad6d\ubbfc\uc758 \ub9c8\uc74c\uc740 \ub300\ud1b5\ub839 \ud558\uc57c\ub2e4\", \"\ub300\ud1b5\ub839\uc774 \ud558\uc57c\ud560 \ub54c\uae4c\uc9c0 \ucd1b\ubd88\uc744 \ub4e4\uc790\"\ub77c\uace0 \uc678\ucce4\ub2e4. \uc774\uc7ac\uba85 \uc131\ub0a8\uc2dc\uc7a5\uc740 \"\ub300\ud1b5\ub839\uc740 \ub098\ub77c\uc758 \uc9c0\ubc30\uc790\uac00 \uc544\ub2c8\ub77c \uad6d\ubbfc\uc758 \uba38\uc2b4\uc774\uace0 \ub300\ub9ac\uc778\uc77c \ubfd0\"\uc774\ub77c\uba70 \"\uadf8\ub7f0 \uadf8\uac00 \ub9c8\uce58 \uc9c0\ubc30\uc790\uc778 \uc591, \uc5ec\uc655\uc778 \uc591 \uc0c1\ud669 \ucd5c\uc21c\uc2e4\uc744 \ub07c\uace0 \ub300\ud55c\ubbfc\uad6d\uc744, \ubbfc\uc8fc\uacf5\ud654\uad6d\uc744 \uc6b0\ub871\ud558\uace0 \uc788\ub2e4\"\uace0 \ubc1c\uc5b8\ud558\uc600\ub2e4. \uc138 \uc0ac\ub78c \uc678\uc5d0\ub3c4 \ub354\ubd88\uc5b4\ubbfc\uc8fc\ub2f9\uc758 \ud45c\ucc3d\uc6d0, \uc1a1\uc601\uae38, \ubc15\uc8fc\ubbfc, \uc815\ucd98\uc219 \uc758\uc6d0, \uc815\uc758\ub2f9\uc758 \uc774\uc815\ubbf8, \uae40\uc885\ub300 \uc758\uc6d0 \ub4f1 \uc57c\ub2f9 \uc815\uce58\uc778\ub4e4\uc774 \uc9d1\ud68c\uc5d0 \ucc38\uc5ec\ud558\uc600\ub2e4. \uc774\ub0a0 \uccad\uacc4\ucc9c \ucd1b\ubd88\uc9d1\ud68c\ub294 \ud329\ud2b8TV (\ud648\ud398\uc774\uc9c0, \uc720\ud29c\ube0c, \uc544\ud504\ub9ac\uce74TV)\uc640 \uc624\ub9c8\uc774TV (\uc720\ud29c\ube0c, \ud398\uc774\uc2a4\ubd81)\uc5d0\uc11c \uc0dd\uc911\uacc4\ud558\uc600\ub2e4.", "answer": "\uc774\uc7ac\uba85", "sentence": "\uc774\ud6c4 \uc815\uce58\uc778 \ub300\ud45c\ub85c\uc11c \ub178\ud68c\ucc2c \uc815\uc758\ub2f9 \uc6d0\ub0b4\ub300\ud45c, \uae40\uc885\ud6c8 \ubb34\uc18c\uc18d \uc758\uc6d0, \uc774\uc7ac\uba85 \uc131\ub0a8\uc2dc\uc7a5\ub3c4 \ubc1c\uc5b8\uc5d0 \ub098\uc130\ub2e4.", "paragraph_sentence": " \uc774\ud6c4 \uc815\uce58\uc778 \ub300\ud45c\ub85c\uc11c \ub178\ud68c\ucc2c \uc815\uc758\ub2f9 \uc6d0\ub0b4\ub300\ud45c, \uae40\uc885\ud6c8 \ubb34\uc18c\uc18d \uc758\uc6d0, \uc774\uc7ac\uba85 \uc131\ub0a8\uc2dc\uc7a5\ub3c4 \ubc1c\uc5b8\uc5d0 \ub098\uc130\ub2e4. \ub178\ud68c\ucc2c \ub300\ud45c\ub294 \"\uc9c0\ub09c 3\ub144 8\uac1c\uc6d4\uac04 \ubd80\uc815\uc744 \uc800\uc9c0\ub978 \ubc15 \ub300\ud1b5\ub839\uc740 \uc5b4\ub5bb\uac8c \ud574\uc57c \ud558\ub290\ub0d0\"\uba70 \"\uc218\uc0ac\uac00 \ubc15 \ub300\ud1b5\ub839\uacfc \ucd5c\uc21c\uc2e4\uc5d0\uac8c \uacf5\uc720\ub418\uace0 \uc788\ub294\ub370 \ub9d0\uc774 \ub418\ub290\ub0d0, \ubc15 \ub300\ud1b5\ub839\uc774 \uc788\ub294 \ud55c \uc9c4\uc2e4\uaddc\uba85\uc774 \uc548 \ub41c\ub2e4. \ubc15 \ub300\ud1b5\ub839\uc774 \ub5a0\ub098\uc57c \uc6b0\ub9ac \uad6d\ubbfc\ub4e4\uc774 \ub2e4\uc2dc \ud798\uc744 \uc5bb\uc744 \uc218 \uc788\ub2e4\u201c\uace0 \uc8fc\uc7a5\ud558\uc600\ub2e4. \uae40\uc885\ud6c8 \ubb34\uc18c\uc18d \uc758\uc6d0\uc740 \"\uc9c0\uc5ed\uad6c\uc778 \uc6b8\uc0b0\uc5d0\uc11c \uc8fc\ubbfc\ub4e4\uc774 \uc11c\uc6b8\uc5d0 \uc62c\ub77c\uac00\ub294 \uc800\uc5d0\uac8c '\ub300\ud55c\ubbfc\uad6d\uc774 \ubd80\ub044\ub7fd\ub2e4', '\uad6d\ubbfc\ub4e4\uc744 \ubd80\ub044\ub7fd\uac8c \ub9cc\ub4e4\uace0 \uc788\ub2e4'\ub77c\ub294 \ub9d0\uc744 \uc804\ud574\ub2ec\ub77c\uace0 \ud588\ub2e4\"\uba74\uc11c, \"\uad6d\ubbfc\uc758 \ub9c8\uc74c\uc740 \ub300\ud1b5\ub839 \ud558\uc57c\ub2e4\", \"\ub300\ud1b5\ub839\uc774 \ud558\uc57c\ud560 \ub54c\uae4c\uc9c0 \ucd1b\ubd88\uc744 \ub4e4\uc790\"\ub77c\uace0 \uc678\ucce4\ub2e4. \uc774\uc7ac\uba85 \uc131\ub0a8\uc2dc\uc7a5\uc740 \"\ub300\ud1b5\ub839\uc740 \ub098\ub77c\uc758 \uc9c0\ubc30\uc790\uac00 \uc544\ub2c8\ub77c \uad6d\ubbfc\uc758 \uba38\uc2b4\uc774\uace0 \ub300\ub9ac\uc778\uc77c \ubfd0\"\uc774\ub77c\uba70 \"\uadf8\ub7f0 \uadf8\uac00 \ub9c8\uce58 \uc9c0\ubc30\uc790\uc778 \uc591, \uc5ec\uc655\uc778 \uc591 \uc0c1\ud669 \ucd5c\uc21c\uc2e4\uc744 \ub07c\uace0 \ub300\ud55c\ubbfc\uad6d\uc744, \ubbfc\uc8fc\uacf5\ud654\uad6d\uc744 \uc6b0\ub871\ud558\uace0 \uc788\ub2e4\"\uace0 \ubc1c\uc5b8\ud558\uc600\ub2e4. \uc138 \uc0ac\ub78c \uc678\uc5d0\ub3c4 \ub354\ubd88\uc5b4\ubbfc\uc8fc\ub2f9\uc758 \ud45c\ucc3d\uc6d0, \uc1a1\uc601\uae38, \ubc15\uc8fc\ubbfc, \uc815\ucd98\uc219 \uc758\uc6d0, \uc815\uc758\ub2f9\uc758 \uc774\uc815\ubbf8, \uae40\uc885\ub300 \uc758\uc6d0 \ub4f1 \uc57c\ub2f9 \uc815\uce58\uc778\ub4e4\uc774 \uc9d1\ud68c\uc5d0 \ucc38\uc5ec\ud558\uc600\ub2e4. \uc774\ub0a0 \uccad\uacc4\ucc9c \ucd1b\ubd88\uc9d1\ud68c\ub294 \ud329\ud2b8TV (\ud648\ud398\uc774\uc9c0, \uc720\ud29c\ube0c, \uc544\ud504\ub9ac\uce74TV)\uc640 \uc624\ub9c8\uc774TV (\uc720\ud29c\ube0c, \ud398\uc774\uc2a4\ubd81)\uc5d0\uc11c \uc0dd\uc911\uacc4\ud558\uc600\ub2e4.", "paragraph_answer": "\uc774\ud6c4 \uc815\uce58\uc778 \ub300\ud45c\ub85c\uc11c \ub178\ud68c\ucc2c \uc815\uc758\ub2f9 \uc6d0\ub0b4\ub300\ud45c, \uae40\uc885\ud6c8 \ubb34\uc18c\uc18d \uc758\uc6d0, \uc774\uc7ac\uba85 \uc131\ub0a8\uc2dc\uc7a5\ub3c4 \ubc1c\uc5b8\uc5d0 \ub098\uc130\ub2e4. \ub178\ud68c\ucc2c \ub300\ud45c\ub294 \"\uc9c0\ub09c 3\ub144 8\uac1c\uc6d4\uac04 \ubd80\uc815\uc744 \uc800\uc9c0\ub978 \ubc15 \ub300\ud1b5\ub839\uc740 \uc5b4\ub5bb\uac8c \ud574\uc57c \ud558\ub290\ub0d0\"\uba70 \"\uc218\uc0ac\uac00 \ubc15 \ub300\ud1b5\ub839\uacfc \ucd5c\uc21c\uc2e4\uc5d0\uac8c \uacf5\uc720\ub418\uace0 \uc788\ub294\ub370 \ub9d0\uc774 \ub418\ub290\ub0d0, \ubc15 \ub300\ud1b5\ub839\uc774 \uc788\ub294 \ud55c \uc9c4\uc2e4\uaddc\uba85\uc774 \uc548 \ub41c\ub2e4. \ubc15 \ub300\ud1b5\ub839\uc774 \ub5a0\ub098\uc57c \uc6b0\ub9ac \uad6d\ubbfc\ub4e4\uc774 \ub2e4\uc2dc \ud798\uc744 \uc5bb\uc744 \uc218 \uc788\ub2e4\u201c\uace0 \uc8fc\uc7a5\ud558\uc600\ub2e4. \uae40\uc885\ud6c8 \ubb34\uc18c\uc18d \uc758\uc6d0\uc740 \"\uc9c0\uc5ed\uad6c\uc778 \uc6b8\uc0b0\uc5d0\uc11c \uc8fc\ubbfc\ub4e4\uc774 \uc11c\uc6b8\uc5d0 \uc62c\ub77c\uac00\ub294 \uc800\uc5d0\uac8c '\ub300\ud55c\ubbfc\uad6d\uc774 \ubd80\ub044\ub7fd\ub2e4', '\uad6d\ubbfc\ub4e4\uc744 \ubd80\ub044\ub7fd\uac8c \ub9cc\ub4e4\uace0 \uc788\ub2e4'\ub77c\ub294 \ub9d0\uc744 \uc804\ud574\ub2ec\ub77c\uace0 \ud588\ub2e4\"\uba74\uc11c, \"\uad6d\ubbfc\uc758 \ub9c8\uc74c\uc740 \ub300\ud1b5\ub839 \ud558\uc57c\ub2e4\", \"\ub300\ud1b5\ub839\uc774 \ud558\uc57c\ud560 \ub54c\uae4c\uc9c0 \ucd1b\ubd88\uc744 \ub4e4\uc790\"\ub77c\uace0 \uc678\ucce4\ub2e4. \uc774\uc7ac\uba85 \uc131\ub0a8\uc2dc\uc7a5\uc740 \"\ub300\ud1b5\ub839\uc740 \ub098\ub77c\uc758 \uc9c0\ubc30\uc790\uac00 \uc544\ub2c8\ub77c \uad6d\ubbfc\uc758 \uba38\uc2b4\uc774\uace0 \ub300\ub9ac\uc778\uc77c \ubfd0\"\uc774\ub77c\uba70 \"\uadf8\ub7f0 \uadf8\uac00 \ub9c8\uce58 \uc9c0\ubc30\uc790\uc778 \uc591, \uc5ec\uc655\uc778 \uc591 \uc0c1\ud669 \ucd5c\uc21c\uc2e4\uc744 \ub07c\uace0 \ub300\ud55c\ubbfc\uad6d\uc744, \ubbfc\uc8fc\uacf5\ud654\uad6d\uc744 \uc6b0\ub871\ud558\uace0 \uc788\ub2e4\"\uace0 \ubc1c\uc5b8\ud558\uc600\ub2e4. \uc138 \uc0ac\ub78c \uc678\uc5d0\ub3c4 \ub354\ubd88\uc5b4\ubbfc\uc8fc\ub2f9\uc758 \ud45c\ucc3d\uc6d0, \uc1a1\uc601\uae38, \ubc15\uc8fc\ubbfc, \uc815\ucd98\uc219 \uc758\uc6d0, \uc815\uc758\ub2f9\uc758 \uc774\uc815\ubbf8, \uae40\uc885\ub300 \uc758\uc6d0 \ub4f1 \uc57c\ub2f9 \uc815\uce58\uc778\ub4e4\uc774 \uc9d1\ud68c\uc5d0 \ucc38\uc5ec\ud558\uc600\ub2e4. \uc774\ub0a0 \uccad\uacc4\ucc9c \ucd1b\ubd88\uc9d1\ud68c\ub294 \ud329\ud2b8TV (\ud648\ud398\uc774\uc9c0, \uc720\ud29c\ube0c, \uc544\ud504\ub9ac\uce74TV)\uc640 \uc624\ub9c8\uc774TV (\uc720\ud29c\ube0c, \ud398\uc774\uc2a4\ubd81)\uc5d0\uc11c \uc0dd\uc911\uacc4\ud558\uc600\ub2e4.", "sentence_answer": "\uc774\ud6c4 \uc815\uce58\uc778 \ub300\ud45c\ub85c\uc11c \ub178\ud68c\ucc2c \uc815\uc758\ub2f9 \uc6d0\ub0b4\ub300\ud45c, \uae40\uc885\ud6c8 \ubb34\uc18c\uc18d \uc758\uc6d0, \uc774\uc7ac\uba85 \uc131\ub0a8\uc2dc\uc7a5\ub3c4 \ubc1c\uc5b8\uc5d0 \ub098\uc130\ub2e4.", "paragraph_id": "6546893-11-1", "paragraph_question": "question: \ub300\ud1b5\ub839\uc740 \uad6d\ubbfc\uc758 \uba38\uc2b4\uc774\uc790 \ub300\ub9ac\uc778\uc774\ub2e4\uace0 \ubc1c\uc5b8\ud55c \uc131\ub0a8\uc2dc\uc7a5\uc740 \ub204\uad6c\uc778\uac00?, context: \uc774\ud6c4 \uc815\uce58\uc778 \ub300\ud45c\ub85c\uc11c \ub178\ud68c\ucc2c \uc815\uc758\ub2f9 \uc6d0\ub0b4\ub300\ud45c, \uae40\uc885\ud6c8 \ubb34\uc18c\uc18d \uc758\uc6d0, \uc774\uc7ac\uba85 \uc131\ub0a8\uc2dc\uc7a5\ub3c4 \ubc1c\uc5b8\uc5d0 \ub098\uc130\ub2e4. \ub178\ud68c\ucc2c \ub300\ud45c\ub294 \"\uc9c0\ub09c 3\ub144 8\uac1c\uc6d4\uac04 \ubd80\uc815\uc744 \uc800\uc9c0\ub978 \ubc15 \ub300\ud1b5\ub839\uc740 \uc5b4\ub5bb\uac8c \ud574\uc57c \ud558\ub290\ub0d0\"\uba70 \"\uc218\uc0ac\uac00 \ubc15 \ub300\ud1b5\ub839\uacfc \ucd5c\uc21c\uc2e4\uc5d0\uac8c \uacf5\uc720\ub418\uace0 \uc788\ub294\ub370 \ub9d0\uc774 \ub418\ub290\ub0d0, \ubc15 \ub300\ud1b5\ub839\uc774 \uc788\ub294 \ud55c \uc9c4\uc2e4\uaddc\uba85\uc774 \uc548 \ub41c\ub2e4. \ubc15 \ub300\ud1b5\ub839\uc774 \ub5a0\ub098\uc57c \uc6b0\ub9ac \uad6d\ubbfc\ub4e4\uc774 \ub2e4\uc2dc \ud798\uc744 \uc5bb\uc744 \uc218 \uc788\ub2e4\u201c\uace0 \uc8fc\uc7a5\ud558\uc600\ub2e4. \uae40\uc885\ud6c8 \ubb34\uc18c\uc18d \uc758\uc6d0\uc740 \"\uc9c0\uc5ed\uad6c\uc778 \uc6b8\uc0b0\uc5d0\uc11c \uc8fc\ubbfc\ub4e4\uc774 \uc11c\uc6b8\uc5d0 \uc62c\ub77c\uac00\ub294 \uc800\uc5d0\uac8c '\ub300\ud55c\ubbfc\uad6d\uc774 \ubd80\ub044\ub7fd\ub2e4', '\uad6d\ubbfc\ub4e4\uc744 \ubd80\ub044\ub7fd\uac8c \ub9cc\ub4e4\uace0 \uc788\ub2e4'\ub77c\ub294 \ub9d0\uc744 \uc804\ud574\ub2ec\ub77c\uace0 \ud588\ub2e4\"\uba74\uc11c, \"\uad6d\ubbfc\uc758 \ub9c8\uc74c\uc740 \ub300\ud1b5\ub839 \ud558\uc57c\ub2e4\", \"\ub300\ud1b5\ub839\uc774 \ud558\uc57c\ud560 \ub54c\uae4c\uc9c0 \ucd1b\ubd88\uc744 \ub4e4\uc790\"\ub77c\uace0 \uc678\ucce4\ub2e4. \uc774\uc7ac\uba85 \uc131\ub0a8\uc2dc\uc7a5\uc740 \"\ub300\ud1b5\ub839\uc740 \ub098\ub77c\uc758 \uc9c0\ubc30\uc790\uac00 \uc544\ub2c8\ub77c \uad6d\ubbfc\uc758 \uba38\uc2b4\uc774\uace0 \ub300\ub9ac\uc778\uc77c \ubfd0\"\uc774\ub77c\uba70 \"\uadf8\ub7f0 \uadf8\uac00 \ub9c8\uce58 \uc9c0\ubc30\uc790\uc778 \uc591, \uc5ec\uc655\uc778 \uc591 \uc0c1\ud669 \ucd5c\uc21c\uc2e4\uc744 \ub07c\uace0 \ub300\ud55c\ubbfc\uad6d\uc744, \ubbfc\uc8fc\uacf5\ud654\uad6d\uc744 \uc6b0\ub871\ud558\uace0 \uc788\ub2e4\"\uace0 \ubc1c\uc5b8\ud558\uc600\ub2e4. \uc138 \uc0ac\ub78c \uc678\uc5d0\ub3c4 \ub354\ubd88\uc5b4\ubbfc\uc8fc\ub2f9\uc758 \ud45c\ucc3d\uc6d0, \uc1a1\uc601\uae38, \ubc15\uc8fc\ubbfc, \uc815\ucd98\uc219 \uc758\uc6d0, \uc815\uc758\ub2f9\uc758 \uc774\uc815\ubbf8, \uae40\uc885\ub300 \uc758\uc6d0 \ub4f1 \uc57c\ub2f9 \uc815\uce58\uc778\ub4e4\uc774 \uc9d1\ud68c\uc5d0 \ucc38\uc5ec\ud558\uc600\ub2e4. \uc774\ub0a0 \uccad\uacc4\ucc9c \ucd1b\ubd88\uc9d1\ud68c\ub294 \ud329\ud2b8TV (\ud648\ud398\uc774\uc9c0, \uc720\ud29c\ube0c, \uc544\ud504\ub9ac\uce74TV)\uc640 \uc624\ub9c8\uc774TV (\uc720\ud29c\ube0c, \ud398\uc774\uc2a4\ubd81)\uc5d0\uc11c \uc0dd\uc911\uacc4\ud558\uc600\ub2e4."} {"question": "\ud55c\uc640 \uc120 \uc9c1\ud1b5 \ucf8c\uc18d\uc774 \uc0dd\uaca8\ub09c \ud574\ub294?", "paragraph": "\uc9c1\ud1b5 \ucf8c\uc18d\uc740 2008\ub144 3\uc6d4 15\uc77c\uc758 \uc2dc\uac04\ud45c \uac1c\uc815\uc73c\ub85c \ud3c9\uc77c \uc544\uce68 \ucd9c\uadfc \uc2dc\uac04\ub300\uc5d0 \uc2e0\uc124\ub41c \uc624\uc0ac\uce74 \uc21c\ud658\uc120 \uc9c1\ud1b5 \uc5f4\ucc28\uc774\ub2e4. \uc815\ucc28\uc5ed\uc740 \ud55c\uc640 \uc120 \ub0b4\uc5d0\uc11c\ub294 \ucf8c\uc18d\uacfc \ub3d9\uc77c\ud558\uba70 \uc624\uc0ac\uce74 \uc21c\ud658\uc120 \uc548\uc5d0\uc11c\ub294 \uac01\uc5ed\uc5d0 \uc815\ucc28\ud55c\ub2e4. \ucc28\ub7c9\uc740 223\uacc4\ub97c \uc0ac\uc6a9\ud558\uace0 \uc788\ub2e4. \uc0c1\ud589\uc740 10\ud3b8\uc774 \uc624\uc0ac\uce74 \uc21c\ud658\uc120 \uc678\uc120\uc744 \uacbd\uc720\ud574 \uc624\uc0ac\uce74 \uc5ed\u00b7\uad50\ubc14\uc2dc \ubc29\uba74\uc73c\ub85c \uc6b4\uc804\ub41c\ub2e4. \uc624\uc0ac\uce74 \uc21c\ud658\uc120\uc744 \uc77c\uc8fc\ud558\ub294 \uc5f4\ucc28 \ub4f1\uacfc \uac19\uc774 \ub3c4\uc911\uc5ed\uc73c\ub85c\uc11c \uc77c\ub2e8 \ub374\ub178\uc9c0 \uc5ed\uc5d0 \uc815\ucc28\ud558\ub294 \uac83\uc5d0\uc11c \uc624\uc0ac\uce74 \ubc29\uba74\uc73c\ub85c \uc6b4\ud589\ud558\uc9c0 \uc54a\ub294\ub2e4\ub294 \uc624\ud574\ub97c \ud53c\ud558\uae30 \uc704\ud574 \uad50\ubc14\uc2dc \ud589\uc744 \uc81c\uc678\ud558\uace0 \uc624\uc0ac\uce74 \uc5ed\uae4c\uc9c0\ub294 \uc624\uc0ac\uce74 \uc21c\ud658\uc120\uc73c\ub85c \uc548\ub0b4\ud558\uace0 \uc788\ub2e4. \ud558\ud589\uc740 \uc624\uc0ac\uce74 \uc21c\ud658\uc120 \ub374\ub178\uc9c0 \uc5ed\uc5d0\uc11c \uad50\ubc14\uc2dc \uc5ed\uc744 \uacbd\uc720\ud574 \uc624\ud1a0\ub9ac \uc5ed\uc774\ub098 \uc640\uce74\uc57c\ub9c8 \uc5ed \ud589\uc774 \uac01\uac01 1\ud3b8\uc529 \uc6b4\ud589\ud558\uace0 \uc788\ub2e4.", "answer": "2008\ub144", "sentence": "\uc9c1\ud1b5 \ucf8c\uc18d\uc740 2008\ub144 3\uc6d4 15\uc77c\uc758 \uc2dc\uac04\ud45c \uac1c\uc815\uc73c\ub85c \ud3c9\uc77c \uc544\uce68 \ucd9c\uadfc \uc2dc\uac04\ub300\uc5d0 \uc2e0\uc124\ub41c \uc624\uc0ac\uce74 \uc21c\ud658\uc120 \uc9c1\ud1b5 \uc5f4\ucc28\uc774\ub2e4.", "paragraph_sentence": " \uc9c1\ud1b5 \ucf8c\uc18d\uc740 2008\ub144 3\uc6d4 15\uc77c\uc758 \uc2dc\uac04\ud45c \uac1c\uc815\uc73c\ub85c \ud3c9\uc77c \uc544\uce68 \ucd9c\uadfc \uc2dc\uac04\ub300\uc5d0 \uc2e0\uc124\ub41c \uc624\uc0ac\uce74 \uc21c\ud658\uc120 \uc9c1\ud1b5 \uc5f4\ucc28\uc774\ub2e4. \uc815\ucc28\uc5ed\uc740 \ud55c\uc640 \uc120 \ub0b4\uc5d0\uc11c\ub294 \ucf8c\uc18d\uacfc \ub3d9\uc77c\ud558\uba70 \uc624\uc0ac\uce74 \uc21c\ud658\uc120 \uc548\uc5d0\uc11c\ub294 \uac01\uc5ed\uc5d0 \uc815\ucc28\ud55c\ub2e4. \ucc28\ub7c9\uc740 223\uacc4\ub97c \uc0ac\uc6a9\ud558\uace0 \uc788\ub2e4. \uc0c1\ud589\uc740 10\ud3b8\uc774 \uc624\uc0ac\uce74 \uc21c\ud658\uc120 \uc678\uc120\uc744 \uacbd\uc720\ud574 \uc624\uc0ac\uce74 \uc5ed\u00b7\uad50\ubc14\uc2dc \ubc29\uba74\uc73c\ub85c \uc6b4\uc804\ub41c\ub2e4. \uc624\uc0ac\uce74 \uc21c\ud658\uc120\uc744 \uc77c\uc8fc\ud558\ub294 \uc5f4\ucc28 \ub4f1\uacfc \uac19\uc774 \ub3c4\uc911\uc5ed\uc73c\ub85c\uc11c \uc77c\ub2e8 \ub374\ub178\uc9c0 \uc5ed\uc5d0 \uc815\ucc28\ud558\ub294 \uac83\uc5d0\uc11c \uc624\uc0ac\uce74 \ubc29\uba74\uc73c\ub85c \uc6b4\ud589\ud558\uc9c0 \uc54a\ub294\ub2e4\ub294 \uc624\ud574\ub97c \ud53c\ud558\uae30 \uc704\ud574 \uad50\ubc14\uc2dc \ud589\uc744 \uc81c\uc678\ud558\uace0 \uc624\uc0ac\uce74 \uc5ed\uae4c\uc9c0\ub294 \uc624\uc0ac\uce74 \uc21c\ud658\uc120\uc73c\ub85c \uc548\ub0b4\ud558\uace0 \uc788\ub2e4. \ud558\ud589\uc740 \uc624\uc0ac\uce74 \uc21c\ud658\uc120 \ub374\ub178\uc9c0 \uc5ed\uc5d0\uc11c \uad50\ubc14\uc2dc \uc5ed\uc744 \uacbd\uc720\ud574 \uc624\ud1a0\ub9ac \uc5ed\uc774\ub098 \uc640\uce74\uc57c\ub9c8 \uc5ed \ud589\uc774 \uac01\uac01 1\ud3b8\uc529 \uc6b4\ud589\ud558\uace0 \uc788\ub2e4.", "paragraph_answer": "\uc9c1\ud1b5 \ucf8c\uc18d\uc740 2008\ub144 3\uc6d4 15\uc77c\uc758 \uc2dc\uac04\ud45c \uac1c\uc815\uc73c\ub85c \ud3c9\uc77c \uc544\uce68 \ucd9c\uadfc \uc2dc\uac04\ub300\uc5d0 \uc2e0\uc124\ub41c \uc624\uc0ac\uce74 \uc21c\ud658\uc120 \uc9c1\ud1b5 \uc5f4\ucc28\uc774\ub2e4. \uc815\ucc28\uc5ed\uc740 \ud55c\uc640 \uc120 \ub0b4\uc5d0\uc11c\ub294 \ucf8c\uc18d\uacfc \ub3d9\uc77c\ud558\uba70 \uc624\uc0ac\uce74 \uc21c\ud658\uc120 \uc548\uc5d0\uc11c\ub294 \uac01\uc5ed\uc5d0 \uc815\ucc28\ud55c\ub2e4. \ucc28\ub7c9\uc740 223\uacc4\ub97c \uc0ac\uc6a9\ud558\uace0 \uc788\ub2e4. \uc0c1\ud589\uc740 10\ud3b8\uc774 \uc624\uc0ac\uce74 \uc21c\ud658\uc120 \uc678\uc120\uc744 \uacbd\uc720\ud574 \uc624\uc0ac\uce74 \uc5ed\u00b7\uad50\ubc14\uc2dc \ubc29\uba74\uc73c\ub85c \uc6b4\uc804\ub41c\ub2e4. \uc624\uc0ac\uce74 \uc21c\ud658\uc120\uc744 \uc77c\uc8fc\ud558\ub294 \uc5f4\ucc28 \ub4f1\uacfc \uac19\uc774 \ub3c4\uc911\uc5ed\uc73c\ub85c\uc11c \uc77c\ub2e8 \ub374\ub178\uc9c0 \uc5ed\uc5d0 \uc815\ucc28\ud558\ub294 \uac83\uc5d0\uc11c \uc624\uc0ac\uce74 \ubc29\uba74\uc73c\ub85c \uc6b4\ud589\ud558\uc9c0 \uc54a\ub294\ub2e4\ub294 \uc624\ud574\ub97c \ud53c\ud558\uae30 \uc704\ud574 \uad50\ubc14\uc2dc \ud589\uc744 \uc81c\uc678\ud558\uace0 \uc624\uc0ac\uce74 \uc5ed\uae4c\uc9c0\ub294 \uc624\uc0ac\uce74 \uc21c\ud658\uc120\uc73c\ub85c \uc548\ub0b4\ud558\uace0 \uc788\ub2e4. \ud558\ud589\uc740 \uc624\uc0ac\uce74 \uc21c\ud658\uc120 \ub374\ub178\uc9c0 \uc5ed\uc5d0\uc11c \uad50\ubc14\uc2dc \uc5ed\uc744 \uacbd\uc720\ud574 \uc624\ud1a0\ub9ac \uc5ed\uc774\ub098 \uc640\uce74\uc57c\ub9c8 \uc5ed \ud589\uc774 \uac01\uac01 1\ud3b8\uc529 \uc6b4\ud589\ud558\uace0 \uc788\ub2e4.", "sentence_answer": "\uc9c1\ud1b5 \ucf8c\uc18d\uc740 2008\ub144 3\uc6d4 15\uc77c\uc758 \uc2dc\uac04\ud45c \uac1c\uc815\uc73c\ub85c \ud3c9\uc77c \uc544\uce68 \ucd9c\uadfc \uc2dc\uac04\ub300\uc5d0 \uc2e0\uc124\ub41c \uc624\uc0ac\uce74 \uc21c\ud658\uc120 \uc9c1\ud1b5 \uc5f4\ucc28\uc774\ub2e4.", "paragraph_id": "6570133-7-0", "paragraph_question": "question: \ud55c\uc640 \uc120 \uc9c1\ud1b5 \ucf8c\uc18d\uc774 \uc0dd\uaca8\ub09c \ud574\ub294?, context: \uc9c1\ud1b5 \ucf8c\uc18d\uc740 2008\ub144 3\uc6d4 15\uc77c\uc758 \uc2dc\uac04\ud45c \uac1c\uc815\uc73c\ub85c \ud3c9\uc77c \uc544\uce68 \ucd9c\uadfc \uc2dc\uac04\ub300\uc5d0 \uc2e0\uc124\ub41c \uc624\uc0ac\uce74 \uc21c\ud658\uc120 \uc9c1\ud1b5 \uc5f4\ucc28\uc774\ub2e4. \uc815\ucc28\uc5ed\uc740 \ud55c\uc640 \uc120 \ub0b4\uc5d0\uc11c\ub294 \ucf8c\uc18d\uacfc \ub3d9\uc77c\ud558\uba70 \uc624\uc0ac\uce74 \uc21c\ud658\uc120 \uc548\uc5d0\uc11c\ub294 \uac01\uc5ed\uc5d0 \uc815\ucc28\ud55c\ub2e4. \ucc28\ub7c9\uc740 223\uacc4\ub97c \uc0ac\uc6a9\ud558\uace0 \uc788\ub2e4. \uc0c1\ud589\uc740 10\ud3b8\uc774 \uc624\uc0ac\uce74 \uc21c\ud658\uc120 \uc678\uc120\uc744 \uacbd\uc720\ud574 \uc624\uc0ac\uce74 \uc5ed\u00b7\uad50\ubc14\uc2dc \ubc29\uba74\uc73c\ub85c \uc6b4\uc804\ub41c\ub2e4. \uc624\uc0ac\uce74 \uc21c\ud658\uc120\uc744 \uc77c\uc8fc\ud558\ub294 \uc5f4\ucc28 \ub4f1\uacfc \uac19\uc774 \ub3c4\uc911\uc5ed\uc73c\ub85c\uc11c \uc77c\ub2e8 \ub374\ub178\uc9c0 \uc5ed\uc5d0 \uc815\ucc28\ud558\ub294 \uac83\uc5d0\uc11c \uc624\uc0ac\uce74 \ubc29\uba74\uc73c\ub85c \uc6b4\ud589\ud558\uc9c0 \uc54a\ub294\ub2e4\ub294 \uc624\ud574\ub97c \ud53c\ud558\uae30 \uc704\ud574 \uad50\ubc14\uc2dc \ud589\uc744 \uc81c\uc678\ud558\uace0 \uc624\uc0ac\uce74 \uc5ed\uae4c\uc9c0\ub294 \uc624\uc0ac\uce74 \uc21c\ud658\uc120\uc73c\ub85c \uc548\ub0b4\ud558\uace0 \uc788\ub2e4. \ud558\ud589\uc740 \uc624\uc0ac\uce74 \uc21c\ud658\uc120 \ub374\ub178\uc9c0 \uc5ed\uc5d0\uc11c \uad50\ubc14\uc2dc \uc5ed\uc744 \uacbd\uc720\ud574 \uc624\ud1a0\ub9ac \uc5ed\uc774\ub098 \uc640\uce74\uc57c\ub9c8 \uc5ed \ud589\uc774 \uac01\uac01 1\ud3b8\uc529 \uc6b4\ud589\ud558\uace0 \uc788\ub2e4."} {"question": "\ud310\uc720\ub9ac \uc81c\uc870\uc758 \uc8fc\uc5ed\uc740 \ubb34\uc5c7\uc778\uac00\uc694?", "paragraph": "\ud310\uc720\ub9ac \ubfd0 \uc544\ub2c8\ub77c \ubcd1\uc774\ub098 \ucef5\u00b7\uc804\uad6c \ub4f1 \uc5ec\ub7ec \uac00\uc9c0 \uc720\ub9ac\uc81c\ud488\uc740 \uace0\uc628\uc73c\ub85c \ud558\uba74 \ubb3c\ub801\ubb3c\ub801\ud574\uc9c0\uace0 \uc790\uc720\ub85c\uc774 \ubaa8\uc591\uc744 \ubcc0\uacbd\ud560 \uc218 \uc788\ub2e4\ub294 \uc720\ub9ac\uc758 \uc131\uc9c8\uc744 \uc774\uc6a9\ud558\uc5ec, \uace0\uc628\uc73c\ub85c \uac00\uc5f4\ud558\uc5ec \uc131\ud615\ud55c\ub2e4. \ud310\uc720\ub9ac \uc81c\uc870\ub294 \ub864\ub7ec\uac00 \uadf8 \uc8fc\uc5ed\uc774\uc9c0\ub9cc \ubcd1\uc774\ub098 \uadf8 \ubc16\uc758 \uc6a9\uae30\uc758 \uacbd\uc6b0\ub294 \ucde8\uac04(\u5439\u687f)\uacfc \ud615(\u578b)\uc774 \uc8fc\uc5ed\uc774 \ub41c\ub2e4. \uc774 \ubc29\ubc95\uc740 \uc624\ub79c \uc61b\ub0a0\ubd80\ud130 \uc0ac\uc6a9\ub418\uc5c8\uc73c\uba70 \uae30\uc6d0\uc804 3\ucc9c\ub144\uacbd\uc758 \uc774\uc9d1\ud2b8 \ubcbd\ud654\uc5d0\ub3c4 \uc785\uc73c\ub85c \ubd88\uc5b4 \uc720\ub9ac\ub97c \uc131\ud615\ud558\ub294 \ubaa8\uc2b5\uc774 \uadf8\ub824\uc838 \uc788\ub2e4. \ud604\uc7ac \uc2dc\ud589\ub418\ub294 \ubc29\ubc95\uc740 \uacf5\uc5c5\uc801\uc73c\ub85c \ub300\ub7c9\uc0dd\uc0b0\uc774 \uac00\ub2a5\ud558\ub3c4\ub85d \uc5f0\uad6c\ub41c \uac83\uc774\uc9c0\ub9cc \uae30\ubcf8\uc6d0\ub9ac\ub294 \uadf8\uc640 \ub2e4\ub984\uc774 \uc5c6\ub2e4. \ub610 \uc6a9\uc735\uc720\ub9ac\ub97c \uc720\ub9ac\uad00\uc73c\ub85c \ub9cc\ub4e4\uc5b4 \uc774 \uad00\uc744 \ubb3c\ub801\ubb3c\ub801\ud558\uac8c \ub9cc\ub4e4\uc5b4 \uc5ec\ub7ec \uac00\uc9c0 \ubaa8\uc591\uc744 \ub9cc\ub4e4 \uc218 \uc788\uc73c\uba70, \ud654\ud559\uc2e4\ud5d8\uc6a9\uc758 \uae30\uad6c\ub3c4 \uc8fc\ub85c \uc774\uc640 \uac19\uc740 \ubc29\ubc95\uc73c\ub85c \ub9cc\ub4e0\ub2e4.", "answer": "\ub864\ub7ec", "sentence": "\uc81c\uc870\ub294 \ub864\ub7ec \uac00 \uadf8 \uc8fc\uc5ed\uc774\uc9c0\ub9cc \ubcd1\uc774\ub098 \uadf8 \ubc16\uc758 \uc6a9\uae30\uc758 \uacbd\uc6b0\ub294 \ucde8\uac04(\u5439\u687f)\uacfc \ud615(\u578b)\uc774 \uc8fc\uc5ed\uc774 \ub41c\ub2e4.", "paragraph_sentence": "\ud310\uc720\ub9ac \ubfd0 \uc544\ub2c8\ub77c \ubcd1\uc774\ub098 \ucef5\u00b7\uc804\uad6c \ub4f1 \uc5ec\ub7ec \uac00\uc9c0 \uc720\ub9ac\uc81c\ud488\uc740 \uace0\uc628\uc73c\ub85c \ud558\uba74 \ubb3c\ub801\ubb3c\ub801\ud574\uc9c0\uace0 \uc790\uc720\ub85c\uc774 \ubaa8\uc591\uc744 \ubcc0\uacbd\ud560 \uc218 \uc788\ub2e4\ub294 \uc720\ub9ac\uc758 \uc131\uc9c8\uc744 \uc774\uc6a9\ud558\uc5ec, \uace0\uc628\uc73c\ub85c \uac00\uc5f4\ud558\uc5ec \uc131\ud615\ud55c\ub2e4. \ud310\uc720\ub9ac \uc81c\uc870\ub294 \ub864\ub7ec \uac00 \uadf8 \uc8fc\uc5ed\uc774\uc9c0\ub9cc \ubcd1\uc774\ub098 \uadf8 \ubc16\uc758 \uc6a9\uae30\uc758 \uacbd\uc6b0\ub294 \ucde8\uac04(\u5439\u687f)\uacfc \ud615(\u578b)\uc774 \uc8fc\uc5ed\uc774 \ub41c\ub2e4. \uc774 \ubc29\ubc95\uc740 \uc624\ub79c \uc61b\ub0a0\ubd80\ud130 \uc0ac\uc6a9\ub418\uc5c8\uc73c\uba70 \uae30\uc6d0\uc804 3\ucc9c\ub144\uacbd\uc758 \uc774\uc9d1\ud2b8 \ubcbd\ud654\uc5d0\ub3c4 \uc785\uc73c\ub85c \ubd88\uc5b4 \uc720\ub9ac\ub97c \uc131\ud615\ud558\ub294 \ubaa8\uc2b5\uc774 \uadf8\ub824\uc838 \uc788\ub2e4. \ud604\uc7ac \uc2dc\ud589\ub418\ub294 \ubc29\ubc95\uc740 \uacf5\uc5c5\uc801\uc73c\ub85c \ub300\ub7c9\uc0dd\uc0b0\uc774 \uac00\ub2a5\ud558\ub3c4\ub85d \uc5f0\uad6c\ub41c \uac83\uc774\uc9c0\ub9cc \uae30\ubcf8\uc6d0\ub9ac\ub294 \uadf8\uc640 \ub2e4\ub984\uc774 \uc5c6\ub2e4. \ub610 \uc6a9\uc735\uc720\ub9ac\ub97c \uc720\ub9ac\uad00\uc73c\ub85c \ub9cc\ub4e4\uc5b4 \uc774 \uad00\uc744 \ubb3c\ub801\ubb3c\ub801\ud558\uac8c \ub9cc\ub4e4\uc5b4 \uc5ec\ub7ec \uac00\uc9c0 \ubaa8\uc591\uc744 \ub9cc\ub4e4 \uc218 \uc788\uc73c\uba70, \ud654\ud559\uc2e4\ud5d8\uc6a9\uc758 \uae30\uad6c\ub3c4 \uc8fc\ub85c \uc774\uc640 \uac19\uc740 \ubc29\ubc95\uc73c\ub85c \ub9cc\ub4e0\ub2e4.", "paragraph_answer": "\ud310\uc720\ub9ac \ubfd0 \uc544\ub2c8\ub77c \ubcd1\uc774\ub098 \ucef5\u00b7\uc804\uad6c \ub4f1 \uc5ec\ub7ec \uac00\uc9c0 \uc720\ub9ac\uc81c\ud488\uc740 \uace0\uc628\uc73c\ub85c \ud558\uba74 \ubb3c\ub801\ubb3c\ub801\ud574\uc9c0\uace0 \uc790\uc720\ub85c\uc774 \ubaa8\uc591\uc744 \ubcc0\uacbd\ud560 \uc218 \uc788\ub2e4\ub294 \uc720\ub9ac\uc758 \uc131\uc9c8\uc744 \uc774\uc6a9\ud558\uc5ec, \uace0\uc628\uc73c\ub85c \uac00\uc5f4\ud558\uc5ec \uc131\ud615\ud55c\ub2e4. \ud310\uc720\ub9ac \uc81c\uc870\ub294 \ub864\ub7ec \uac00 \uadf8 \uc8fc\uc5ed\uc774\uc9c0\ub9cc \ubcd1\uc774\ub098 \uadf8 \ubc16\uc758 \uc6a9\uae30\uc758 \uacbd\uc6b0\ub294 \ucde8\uac04(\u5439\u687f)\uacfc \ud615(\u578b)\uc774 \uc8fc\uc5ed\uc774 \ub41c\ub2e4. \uc774 \ubc29\ubc95\uc740 \uc624\ub79c \uc61b\ub0a0\ubd80\ud130 \uc0ac\uc6a9\ub418\uc5c8\uc73c\uba70 \uae30\uc6d0\uc804 3\ucc9c\ub144\uacbd\uc758 \uc774\uc9d1\ud2b8 \ubcbd\ud654\uc5d0\ub3c4 \uc785\uc73c\ub85c \ubd88\uc5b4 \uc720\ub9ac\ub97c \uc131\ud615\ud558\ub294 \ubaa8\uc2b5\uc774 \uadf8\ub824\uc838 \uc788\ub2e4. \ud604\uc7ac \uc2dc\ud589\ub418\ub294 \ubc29\ubc95\uc740 \uacf5\uc5c5\uc801\uc73c\ub85c \ub300\ub7c9\uc0dd\uc0b0\uc774 \uac00\ub2a5\ud558\ub3c4\ub85d \uc5f0\uad6c\ub41c \uac83\uc774\uc9c0\ub9cc \uae30\ubcf8\uc6d0\ub9ac\ub294 \uadf8\uc640 \ub2e4\ub984\uc774 \uc5c6\ub2e4. \ub610 \uc6a9\uc735\uc720\ub9ac\ub97c \uc720\ub9ac\uad00\uc73c\ub85c \ub9cc\ub4e4\uc5b4 \uc774 \uad00\uc744 \ubb3c\ub801\ubb3c\ub801\ud558\uac8c \ub9cc\ub4e4\uc5b4 \uc5ec\ub7ec \uac00\uc9c0 \ubaa8\uc591\uc744 \ub9cc\ub4e4 \uc218 \uc788\uc73c\uba70, \ud654\ud559\uc2e4\ud5d8\uc6a9\uc758 \uae30\uad6c\ub3c4 \uc8fc\ub85c \uc774\uc640 \uac19\uc740 \ubc29\ubc95\uc73c\ub85c \ub9cc\ub4e0\ub2e4.", "sentence_answer": "\uc81c\uc870\ub294 \ub864\ub7ec \uac00 \uadf8 \uc8fc\uc5ed\uc774\uc9c0\ub9cc \ubcd1\uc774\ub098 \uadf8 \ubc16\uc758 \uc6a9\uae30\uc758 \uacbd\uc6b0\ub294 \ucde8\uac04(\u5439\u687f)\uacfc \ud615(\u578b)\uc774 \uc8fc\uc5ed\uc774 \ub41c\ub2e4.", "paragraph_id": "6542977-2-0", "paragraph_question": "question: \ud310\uc720\ub9ac \uc81c\uc870\uc758 \uc8fc\uc5ed\uc740 \ubb34\uc5c7\uc778\uac00\uc694?, context: \ud310\uc720\ub9ac \ubfd0 \uc544\ub2c8\ub77c \ubcd1\uc774\ub098 \ucef5\u00b7\uc804\uad6c \ub4f1 \uc5ec\ub7ec \uac00\uc9c0 \uc720\ub9ac\uc81c\ud488\uc740 \uace0\uc628\uc73c\ub85c \ud558\uba74 \ubb3c\ub801\ubb3c\ub801\ud574\uc9c0\uace0 \uc790\uc720\ub85c\uc774 \ubaa8\uc591\uc744 \ubcc0\uacbd\ud560 \uc218 \uc788\ub2e4\ub294 \uc720\ub9ac\uc758 \uc131\uc9c8\uc744 \uc774\uc6a9\ud558\uc5ec, \uace0\uc628\uc73c\ub85c \uac00\uc5f4\ud558\uc5ec \uc131\ud615\ud55c\ub2e4. \ud310\uc720\ub9ac \uc81c\uc870\ub294 \ub864\ub7ec\uac00 \uadf8 \uc8fc\uc5ed\uc774\uc9c0\ub9cc \ubcd1\uc774\ub098 \uadf8 \ubc16\uc758 \uc6a9\uae30\uc758 \uacbd\uc6b0\ub294 \ucde8\uac04(\u5439\u687f)\uacfc \ud615(\u578b)\uc774 \uc8fc\uc5ed\uc774 \ub41c\ub2e4. \uc774 \ubc29\ubc95\uc740 \uc624\ub79c \uc61b\ub0a0\ubd80\ud130 \uc0ac\uc6a9\ub418\uc5c8\uc73c\uba70 \uae30\uc6d0\uc804 3\ucc9c\ub144\uacbd\uc758 \uc774\uc9d1\ud2b8 \ubcbd\ud654\uc5d0\ub3c4 \uc785\uc73c\ub85c \ubd88\uc5b4 \uc720\ub9ac\ub97c \uc131\ud615\ud558\ub294 \ubaa8\uc2b5\uc774 \uadf8\ub824\uc838 \uc788\ub2e4. \ud604\uc7ac \uc2dc\ud589\ub418\ub294 \ubc29\ubc95\uc740 \uacf5\uc5c5\uc801\uc73c\ub85c \ub300\ub7c9\uc0dd\uc0b0\uc774 \uac00\ub2a5\ud558\ub3c4\ub85d \uc5f0\uad6c\ub41c \uac83\uc774\uc9c0\ub9cc \uae30\ubcf8\uc6d0\ub9ac\ub294 \uadf8\uc640 \ub2e4\ub984\uc774 \uc5c6\ub2e4. \ub610 \uc6a9\uc735\uc720\ub9ac\ub97c \uc720\ub9ac\uad00\uc73c\ub85c \ub9cc\ub4e4\uc5b4 \uc774 \uad00\uc744 \ubb3c\ub801\ubb3c\ub801\ud558\uac8c \ub9cc\ub4e4\uc5b4 \uc5ec\ub7ec \uac00\uc9c0 \ubaa8\uc591\uc744 \ub9cc\ub4e4 \uc218 \uc788\uc73c\uba70, \ud654\ud559\uc2e4\ud5d8\uc6a9\uc758 \uae30\uad6c\ub3c4 \uc8fc\ub85c \uc774\uc640 \uac19\uc740 \ubc29\ubc95\uc73c\ub85c \ub9cc\ub4e0\ub2e4."} {"question": "\uc870\uc9c0\uc544 \uc9c0\uc5ed\uc758 \uc0ac\ud68c\uc8fc\uc758 \ubbfc\uc871\uc8fc\uc758\uc790 \uc6b4\ub3d9 \uc81c3\uadf8\ub8f9\uc5d0 \uc18d\ud574 \uc788\ub358 \uc774\uc624\uc138\ube0c \uc8fc\uac00\uc288\ube4c\ub9ac\uac00 \ubc14\uafbc \uc774\ub984\uc740?", "paragraph": "\uc774\ud6c4 \uc870\uc9c0\uc544 \uc9c0\uc5ed\uc758 \uc0ac\ud68c\uc8fc\uc758 \ubbfc\uc871\uc8fc\uc758\uc790 \uc6b4\ub3d9\uc740 \uc81c1\uadf8\ub8f9, \uc81c2\uadf8\ub8f9, \uc81c3\uadf8\ub8f9\uc73c\ub85c \ub098\ub258\uc5b4 \uc788\uc5c8\uc73c\uba70 \uac00\uc7a5 \uc9c4\ubcf4\uc801\uc778 \uc81c3\uadf8\ub8f9\uc740 \uc774\uc624\uc138\ube0c \uc8fc\uac00\uc288\ube4c\ub9ac\uac00 \uc18d\ud574 \uc788\uc5c8\ub294\ub370 \uadf8\ub294 \ub098\uc911\uc5d0 \uc774\ub984\uc744 \ub7ec\uc2dc\uc544\uc2dd\uc73c\ub85c \"\ucca0\uc758 \ub0a8\uc790\"\ub85c \ubc14\uafbc \uc774\uc624\uc2dc\ud504 \uc2a4\ud0c8\ub9b0\uc774\ub2e4. 1917\ub144 \ud601\uba85\uc73c\ub85c \ub7ec\uc2dc\uc544 \uc81c\uad6d\uc774 \ubd95\uad34\ub418\uc790 \uc870\uc9c0\uc544\ub294 \uc870\uc9c0\uc544 \ubbfc\uc8fc \uacf5\ud654\uad6d\uc744 \uc138\uc6b0\uace0 \ub3c5\ub9bd\uc744 \uc120\uc5b8\ud588\ub2e4. \uacf5\ud654\uad6d\uc740 \uad6d\uc81c\uc801\uc73c\ub85c \ub3c5\ub9bd\uc744 \uc778\uc815\ubc1b\uc558\uace0, \ub7ec\uc2dc\uc544\ub3c4 \ub3c5\ub9bd\uc744 \uc2b9\uc778\ud588\ub2e4. \uadf8\ub7ec\ub098 \uc18c\ub828\uc758 \ubd89\uc740 \uad70\ub300\ub294 \ub2e4\uc2dc \uc870\uc9c0\uc544\ub97c \uc810\ub839\ud558\uc5ec \uc18c\ub828\uc5d0 \ud3b8\uc785\uc2dc\ud0a4\uace0 \uc544\ub974\uba54\ub2c8\uc544, \uc544\uc81c\ub974\ubc14\uc774\uc794\uacfc \ud568\uaed8 \uc790\uce85\uce74\uc2a4 \uc5f0\ubc29\uc744 \uc218\ub9bd\ud574\uc11c \uc18c\ube44\uc5d0\ud2b8 \uc5f0\ubc29\uc744 \uad6c\uc131\ud558\ub294 \uacf5\ud654\uad6d\uc73c\ub85c \ub9cc\ub4e4\uc5c8\ub2e4. \ubcf8\ub798 \uc870\uc9c0\uc544 \ucd9c\uc2e0\uc774\uc5c8\ub358 \uc2a4\ud0c8\ub9b0\uc740 \uc870\uc9c0\uc544 \ubbfc\uc871\uc8fc\uc758\uc790 \uc2ed\ub9cc \uba85 \uc774\uc0c1\uc744 \uc2dc\ubca0\ub9ac\uc544 \uac15\uc81c\uc218\uc6a9\uc18c\ub85c \ubcf4\ub0c8\ub2e4. 1936\ub144 \uc790\uce85\uce74\uc2a4 \uc5f0\ubc29\uc744 \ud574\uccb4\ud558\uc5ec \uac01\uac01 \uadf8\ub8e8\uc9c0\uc57c, \uc544\ub974\uba54\ub2c8\uc544, \uc544\uc81c\ub974\ubc14\uc774\uc794\uc73c\ub85c \ubd84\ub9ac\ub418\uc5b4 \uc870\uc9c0\uc544\uc758 \uad6d\uba85\uc744 \ub418\ucc3e\uc558\uc9c0\ub9cc \ub3c5\ub9bd\ud558\uc9c0 \ubabb\ud558\uace0 \uacc4\uc18d \uc18c\ub828\uc744 \uad6c\uc131\ud558\ub294 \uacf5\ud654\uad6d\uc758 \ud558\ub098\ub85c \ub0a8\uc544 \uc788\uc5c8\ub2e4.", "answer": "\uc774\uc624\uc2dc\ud504 \uc2a4\ud0c8\ub9b0", "sentence": "\uc774\ud6c4 \uc870\uc9c0\uc544 \uc9c0\uc5ed\uc758 \uc0ac\ud68c\uc8fc\uc758 \ubbfc\uc871\uc8fc\uc758\uc790 \uc6b4\ub3d9\uc740 \uc81c1\uadf8\ub8f9, \uc81c2\uadf8\ub8f9, \uc81c3\uadf8\ub8f9\uc73c\ub85c \ub098\ub258\uc5b4 \uc788\uc5c8\uc73c\uba70 \uac00\uc7a5 \uc9c4\ubcf4\uc801\uc778 \uc81c3\uadf8\ub8f9\uc740 \uc774\uc624\uc138\ube0c \uc8fc\uac00\uc288\ube4c\ub9ac\uac00 \uc18d\ud574 \uc788\uc5c8\ub294\ub370 \uadf8\ub294 \ub098\uc911\uc5d0 \uc774\ub984\uc744 \ub7ec\uc2dc\uc544\uc2dd\uc73c\ub85c \"\ucca0\uc758 \ub0a8\uc790\"\ub85c \ubc14\uafbc \uc774\uc624\uc2dc\ud504 \uc2a4\ud0c8\ub9b0 \uc774\ub2e4.", "paragraph_sentence": " \uc774\ud6c4 \uc870\uc9c0\uc544 \uc9c0\uc5ed\uc758 \uc0ac\ud68c\uc8fc\uc758 \ubbfc\uc871\uc8fc\uc758\uc790 \uc6b4\ub3d9\uc740 \uc81c1\uadf8\ub8f9, \uc81c2\uadf8\ub8f9, \uc81c3\uadf8\ub8f9\uc73c\ub85c \ub098\ub258\uc5b4 \uc788\uc5c8\uc73c\uba70 \uac00\uc7a5 \uc9c4\ubcf4\uc801\uc778 \uc81c3\uadf8\ub8f9\uc740 \uc774\uc624\uc138\ube0c \uc8fc\uac00\uc288\ube4c\ub9ac\uac00 \uc18d\ud574 \uc788\uc5c8\ub294\ub370 \uadf8\ub294 \ub098\uc911\uc5d0 \uc774\ub984\uc744 \ub7ec\uc2dc\uc544\uc2dd\uc73c\ub85c \"\ucca0\uc758 \ub0a8\uc790\"\ub85c \ubc14\uafbc \uc774\uc624\uc2dc\ud504 \uc2a4\ud0c8\ub9b0 \uc774\ub2e4. 1917\ub144 \ud601\uba85\uc73c\ub85c \ub7ec\uc2dc\uc544 \uc81c\uad6d\uc774 \ubd95\uad34\ub418\uc790 \uc870\uc9c0\uc544\ub294 \uc870\uc9c0\uc544 \ubbfc\uc8fc \uacf5\ud654\uad6d\uc744 \uc138\uc6b0\uace0 \ub3c5\ub9bd\uc744 \uc120\uc5b8\ud588\ub2e4. \uacf5\ud654\uad6d\uc740 \uad6d\uc81c\uc801\uc73c\ub85c \ub3c5\ub9bd\uc744 \uc778\uc815\ubc1b\uc558\uace0, \ub7ec\uc2dc\uc544\ub3c4 \ub3c5\ub9bd\uc744 \uc2b9\uc778\ud588\ub2e4. \uadf8\ub7ec\ub098 \uc18c\ub828\uc758 \ubd89\uc740 \uad70\ub300\ub294 \ub2e4\uc2dc \uc870\uc9c0\uc544\ub97c \uc810\ub839\ud558\uc5ec \uc18c\ub828\uc5d0 \ud3b8\uc785\uc2dc\ud0a4\uace0 \uc544\ub974\uba54\ub2c8\uc544, \uc544\uc81c\ub974\ubc14\uc774\uc794\uacfc \ud568\uaed8 \uc790\uce85\uce74\uc2a4 \uc5f0\ubc29\uc744 \uc218\ub9bd\ud574\uc11c \uc18c\ube44\uc5d0\ud2b8 \uc5f0\ubc29\uc744 \uad6c\uc131\ud558\ub294 \uacf5\ud654\uad6d\uc73c\ub85c \ub9cc\ub4e4\uc5c8\ub2e4. \ubcf8\ub798 \uc870\uc9c0\uc544 \ucd9c\uc2e0\uc774\uc5c8\ub358 \uc2a4\ud0c8\ub9b0\uc740 \uc870\uc9c0\uc544 \ubbfc\uc871\uc8fc\uc758\uc790 \uc2ed\ub9cc \uba85 \uc774\uc0c1\uc744 \uc2dc\ubca0\ub9ac\uc544 \uac15\uc81c\uc218\uc6a9\uc18c\ub85c \ubcf4\ub0c8\ub2e4. 1936\ub144 \uc790\uce85\uce74\uc2a4 \uc5f0\ubc29\uc744 \ud574\uccb4\ud558\uc5ec \uac01\uac01 \uadf8\ub8e8\uc9c0\uc57c, \uc544\ub974\uba54\ub2c8\uc544, \uc544\uc81c\ub974\ubc14\uc774\uc794\uc73c\ub85c \ubd84\ub9ac\ub418\uc5b4 \uc870\uc9c0\uc544\uc758 \uad6d\uba85\uc744 \ub418\ucc3e\uc558\uc9c0\ub9cc \ub3c5\ub9bd\ud558\uc9c0 \ubabb\ud558\uace0 \uacc4\uc18d \uc18c\ub828\uc744 \uad6c\uc131\ud558\ub294 \uacf5\ud654\uad6d\uc758 \ud558\ub098\ub85c \ub0a8\uc544 \uc788\uc5c8\ub2e4.", "paragraph_answer": "\uc774\ud6c4 \uc870\uc9c0\uc544 \uc9c0\uc5ed\uc758 \uc0ac\ud68c\uc8fc\uc758 \ubbfc\uc871\uc8fc\uc758\uc790 \uc6b4\ub3d9\uc740 \uc81c1\uadf8\ub8f9, \uc81c2\uadf8\ub8f9, \uc81c3\uadf8\ub8f9\uc73c\ub85c \ub098\ub258\uc5b4 \uc788\uc5c8\uc73c\uba70 \uac00\uc7a5 \uc9c4\ubcf4\uc801\uc778 \uc81c3\uadf8\ub8f9\uc740 \uc774\uc624\uc138\ube0c \uc8fc\uac00\uc288\ube4c\ub9ac\uac00 \uc18d\ud574 \uc788\uc5c8\ub294\ub370 \uadf8\ub294 \ub098\uc911\uc5d0 \uc774\ub984\uc744 \ub7ec\uc2dc\uc544\uc2dd\uc73c\ub85c \"\ucca0\uc758 \ub0a8\uc790\"\ub85c \ubc14\uafbc \uc774\uc624\uc2dc\ud504 \uc2a4\ud0c8\ub9b0 \uc774\ub2e4. 1917\ub144 \ud601\uba85\uc73c\ub85c \ub7ec\uc2dc\uc544 \uc81c\uad6d\uc774 \ubd95\uad34\ub418\uc790 \uc870\uc9c0\uc544\ub294 \uc870\uc9c0\uc544 \ubbfc\uc8fc \uacf5\ud654\uad6d\uc744 \uc138\uc6b0\uace0 \ub3c5\ub9bd\uc744 \uc120\uc5b8\ud588\ub2e4. \uacf5\ud654\uad6d\uc740 \uad6d\uc81c\uc801\uc73c\ub85c \ub3c5\ub9bd\uc744 \uc778\uc815\ubc1b\uc558\uace0, \ub7ec\uc2dc\uc544\ub3c4 \ub3c5\ub9bd\uc744 \uc2b9\uc778\ud588\ub2e4. \uadf8\ub7ec\ub098 \uc18c\ub828\uc758 \ubd89\uc740 \uad70\ub300\ub294 \ub2e4\uc2dc \uc870\uc9c0\uc544\ub97c \uc810\ub839\ud558\uc5ec \uc18c\ub828\uc5d0 \ud3b8\uc785\uc2dc\ud0a4\uace0 \uc544\ub974\uba54\ub2c8\uc544, \uc544\uc81c\ub974\ubc14\uc774\uc794\uacfc \ud568\uaed8 \uc790\uce85\uce74\uc2a4 \uc5f0\ubc29\uc744 \uc218\ub9bd\ud574\uc11c \uc18c\ube44\uc5d0\ud2b8 \uc5f0\ubc29\uc744 \uad6c\uc131\ud558\ub294 \uacf5\ud654\uad6d\uc73c\ub85c \ub9cc\ub4e4\uc5c8\ub2e4. \ubcf8\ub798 \uc870\uc9c0\uc544 \ucd9c\uc2e0\uc774\uc5c8\ub358 \uc2a4\ud0c8\ub9b0\uc740 \uc870\uc9c0\uc544 \ubbfc\uc871\uc8fc\uc758\uc790 \uc2ed\ub9cc \uba85 \uc774\uc0c1\uc744 \uc2dc\ubca0\ub9ac\uc544 \uac15\uc81c\uc218\uc6a9\uc18c\ub85c \ubcf4\ub0c8\ub2e4. 1936\ub144 \uc790\uce85\uce74\uc2a4 \uc5f0\ubc29\uc744 \ud574\uccb4\ud558\uc5ec \uac01\uac01 \uadf8\ub8e8\uc9c0\uc57c, \uc544\ub974\uba54\ub2c8\uc544, \uc544\uc81c\ub974\ubc14\uc774\uc794\uc73c\ub85c \ubd84\ub9ac\ub418\uc5b4 \uc870\uc9c0\uc544\uc758 \uad6d\uba85\uc744 \ub418\ucc3e\uc558\uc9c0\ub9cc \ub3c5\ub9bd\ud558\uc9c0 \ubabb\ud558\uace0 \uacc4\uc18d \uc18c\ub828\uc744 \uad6c\uc131\ud558\ub294 \uacf5\ud654\uad6d\uc758 \ud558\ub098\ub85c \ub0a8\uc544 \uc788\uc5c8\ub2e4.", "sentence_answer": "\uc774\ud6c4 \uc870\uc9c0\uc544 \uc9c0\uc5ed\uc758 \uc0ac\ud68c\uc8fc\uc758 \ubbfc\uc871\uc8fc\uc758\uc790 \uc6b4\ub3d9\uc740 \uc81c1\uadf8\ub8f9, \uc81c2\uadf8\ub8f9, \uc81c3\uadf8\ub8f9\uc73c\ub85c \ub098\ub258\uc5b4 \uc788\uc5c8\uc73c\uba70 \uac00\uc7a5 \uc9c4\ubcf4\uc801\uc778 \uc81c3\uadf8\ub8f9\uc740 \uc774\uc624\uc138\ube0c \uc8fc\uac00\uc288\ube4c\ub9ac\uac00 \uc18d\ud574 \uc788\uc5c8\ub294\ub370 \uadf8\ub294 \ub098\uc911\uc5d0 \uc774\ub984\uc744 \ub7ec\uc2dc\uc544\uc2dd\uc73c\ub85c \"\ucca0\uc758 \ub0a8\uc790\"\ub85c \ubc14\uafbc \uc774\uc624\uc2dc\ud504 \uc2a4\ud0c8\ub9b0 \uc774\ub2e4.", "paragraph_id": "6595129-1-0", "paragraph_question": "question: \uc870\uc9c0\uc544 \uc9c0\uc5ed\uc758 \uc0ac\ud68c\uc8fc\uc758 \ubbfc\uc871\uc8fc\uc758\uc790 \uc6b4\ub3d9 \uc81c3\uadf8\ub8f9\uc5d0 \uc18d\ud574 \uc788\ub358 \uc774\uc624\uc138\ube0c \uc8fc\uac00\uc288\ube4c\ub9ac\uac00 \ubc14\uafbc \uc774\ub984\uc740?, context: \uc774\ud6c4 \uc870\uc9c0\uc544 \uc9c0\uc5ed\uc758 \uc0ac\ud68c\uc8fc\uc758 \ubbfc\uc871\uc8fc\uc758\uc790 \uc6b4\ub3d9\uc740 \uc81c1\uadf8\ub8f9, \uc81c2\uadf8\ub8f9, \uc81c3\uadf8\ub8f9\uc73c\ub85c \ub098\ub258\uc5b4 \uc788\uc5c8\uc73c\uba70 \uac00\uc7a5 \uc9c4\ubcf4\uc801\uc778 \uc81c3\uadf8\ub8f9\uc740 \uc774\uc624\uc138\ube0c \uc8fc\uac00\uc288\ube4c\ub9ac\uac00 \uc18d\ud574 \uc788\uc5c8\ub294\ub370 \uadf8\ub294 \ub098\uc911\uc5d0 \uc774\ub984\uc744 \ub7ec\uc2dc\uc544\uc2dd\uc73c\ub85c \"\ucca0\uc758 \ub0a8\uc790\"\ub85c \ubc14\uafbc \uc774\uc624\uc2dc\ud504 \uc2a4\ud0c8\ub9b0\uc774\ub2e4. 1917\ub144 \ud601\uba85\uc73c\ub85c \ub7ec\uc2dc\uc544 \uc81c\uad6d\uc774 \ubd95\uad34\ub418\uc790 \uc870\uc9c0\uc544\ub294 \uc870\uc9c0\uc544 \ubbfc\uc8fc \uacf5\ud654\uad6d\uc744 \uc138\uc6b0\uace0 \ub3c5\ub9bd\uc744 \uc120\uc5b8\ud588\ub2e4. \uacf5\ud654\uad6d\uc740 \uad6d\uc81c\uc801\uc73c\ub85c \ub3c5\ub9bd\uc744 \uc778\uc815\ubc1b\uc558\uace0, \ub7ec\uc2dc\uc544\ub3c4 \ub3c5\ub9bd\uc744 \uc2b9\uc778\ud588\ub2e4. \uadf8\ub7ec\ub098 \uc18c\ub828\uc758 \ubd89\uc740 \uad70\ub300\ub294 \ub2e4\uc2dc \uc870\uc9c0\uc544\ub97c \uc810\ub839\ud558\uc5ec \uc18c\ub828\uc5d0 \ud3b8\uc785\uc2dc\ud0a4\uace0 \uc544\ub974\uba54\ub2c8\uc544, \uc544\uc81c\ub974\ubc14\uc774\uc794\uacfc \ud568\uaed8 \uc790\uce85\uce74\uc2a4 \uc5f0\ubc29\uc744 \uc218\ub9bd\ud574\uc11c \uc18c\ube44\uc5d0\ud2b8 \uc5f0\ubc29\uc744 \uad6c\uc131\ud558\ub294 \uacf5\ud654\uad6d\uc73c\ub85c \ub9cc\ub4e4\uc5c8\ub2e4. \ubcf8\ub798 \uc870\uc9c0\uc544 \ucd9c\uc2e0\uc774\uc5c8\ub358 \uc2a4\ud0c8\ub9b0\uc740 \uc870\uc9c0\uc544 \ubbfc\uc871\uc8fc\uc758\uc790 \uc2ed\ub9cc \uba85 \uc774\uc0c1\uc744 \uc2dc\ubca0\ub9ac\uc544 \uac15\uc81c\uc218\uc6a9\uc18c\ub85c \ubcf4\ub0c8\ub2e4. 1936\ub144 \uc790\uce85\uce74\uc2a4 \uc5f0\ubc29\uc744 \ud574\uccb4\ud558\uc5ec \uac01\uac01 \uadf8\ub8e8\uc9c0\uc57c, \uc544\ub974\uba54\ub2c8\uc544, \uc544\uc81c\ub974\ubc14\uc774\uc794\uc73c\ub85c \ubd84\ub9ac\ub418\uc5b4 \uc870\uc9c0\uc544\uc758 \uad6d\uba85\uc744 \ub418\ucc3e\uc558\uc9c0\ub9cc \ub3c5\ub9bd\ud558\uc9c0 \ubabb\ud558\uace0 \uacc4\uc18d \uc18c\ub828\uc744 \uad6c\uc131\ud558\ub294 \uacf5\ud654\uad6d\uc758 \ud558\ub098\ub85c \ub0a8\uc544 \uc788\uc5c8\ub2e4."} {"question": "\uacfc\ud68c\uc0ac\ud68c\ud559\uc774 \uae09\uaca9\ud788 \uc131\uc7a5\ud55c \uc2dc\uae30\ub294?", "paragraph": "1957\ub144\uc744 \uae30\uc810\uc73c\ub85c \ud558\uc5ec \uacfc\ud559\uc0ac\ud68c\ud559\uc740 \uae09\uaca9\ud788 \uc131\uc7a5\ud558\uc600\ub2e4. 1950\ub144\ub300 \ucd08\ubc18\ubd80\ud130 \ubbf8\uad6d \uc0b0\uc5c5\uc0ac\ud68c\ud559\uc790\ub4e4\uc774 \ub3c5\uc790\uc801\uc73c\ub85c \uc9c4\ud589\ud574 \uc628, \uae30\uc5c5\u00b7 \uc815\ubd80 \ucd9c\uc5f0 \uc5f0\uad6c\uc18c\uc5d0 \uc18c\uc18d\ub41c \uacfc\ud559\uc790\uc758 \uc0dd\uc0b0\uc131\uc774\ub098 \ubcf5\ub9ac\ud6c4\uc0dd\u00b7 \uacbd\ub825\ud615\uc131\uc5d0 \ub300\ud55c \uc5f0\uad6c\uc5d0 \uacfc\ud559\uc0ac\ud68c\ud559\uc774 \uc811\ubaa9\ub418\uc5c8\ub2e4. 1950\ub144\ub300 \ud6c4\ubc18\uc5d0 \ub4f1\uc7a5\ud55c \uacfc\ud559\uc790\uc758 \uc758\uc0ac\uc18c\ud1b5\uacfc \uc9d1\ub2e8\ud615\uc131\uc5d0 \uad00\ud55c \uc5f0\uad6c\uc5d0\ub3c4 \uc5ed\uc2dc \uacfc\ud559\uc0ac\ud68c\ud559\uc758 \uc5ec\ub7ec \uc774\ub860\ub4e4\uc774 \uc0ac\uc6a9\ub418\uc5c8\uace0, \uacfc\ud559\uc0ac\ud559\uc790 \ub4dc \uc194\ub77c \ud504\ub77c\uc774\uc2a4\uac00 \uc774\ub044\ub294 \uacc4\ub7c9\uc801 \uacfc\ud559\uc5f0\uad6c\uc640\ub3c4 \uc601\ud5a5\uc744 \uc8fc\uace0\ubc1b\uae30 \uc2dc\uc791\ud588\ub2e4. \uc774\uc640 \uac19\uc740 \uc131\uc7a5\uc758 \ubc30\uacbd\uc5d0\ub294 \uba38\ud134\uc774 1957\ub144\uc5d0 \ubbf8\uad6d \uc0ac\ud68c\ud559\ud68c \ud68c\uc7a5\uc5d0 \ucde8\uc784\ud588\ub2e4\ub294 \uc0ac\uc2e4\uc774\ub098 \uacfc\ud559\uc0ac\ud68c\ud559\uc758 \uae30\ubcf8\uc801\uc778 \ud328\ub7ec\ub2e4\uc784\uc774 \uac16\ucdb0\uc9c4 \uac83, \uadf8\ub9ac\uace0 \uc18c\ub828\uc774 \uc2a4\ud478\ud2b8\ub2c8\ud06c 1\ud638\ub97c \ubc1c\uc0ac\ud568\uc73c\ub85c\uc368 \uacfc\ud559\uc758 \uc0ac\ud68c\uc801 \uc5ed\ud560\uc5d0 \ub300\ud574 \uad00\uc2ec\uc774 \ub9ce\uc544\uc9c4 \uac83 \ub4f1\uc744 \uc608\ub85c \ub4e4 \uc218 \uc788\uc744 \uac83\uc774\ub2e4.", "answer": "1957\ub144", "sentence": "1957\ub144 \uc744 \uae30\uc810\uc73c\ub85c \ud558\uc5ec \uacfc\ud559\uc0ac\ud68c\ud559\uc740 \uae09\uaca9\ud788 \uc131\uc7a5\ud558\uc600\ub2e4.", "paragraph_sentence": " 1957\ub144 \uc744 \uae30\uc810\uc73c\ub85c \ud558\uc5ec \uacfc\ud559\uc0ac\ud68c\ud559\uc740 \uae09\uaca9\ud788 \uc131\uc7a5\ud558\uc600\ub2e4. 1950\ub144\ub300 \ucd08\ubc18\ubd80\ud130 \ubbf8\uad6d \uc0b0\uc5c5\uc0ac\ud68c\ud559\uc790\ub4e4\uc774 \ub3c5\uc790\uc801\uc73c\ub85c \uc9c4\ud589\ud574 \uc628, \uae30\uc5c5\u00b7 \uc815\ubd80 \ucd9c\uc5f0 \uc5f0\uad6c\uc18c\uc5d0 \uc18c\uc18d\ub41c \uacfc\ud559\uc790\uc758 \uc0dd\uc0b0\uc131\uc774\ub098 \ubcf5\ub9ac\ud6c4\uc0dd\u00b7 \uacbd\ub825\ud615\uc131\uc5d0 \ub300\ud55c \uc5f0\uad6c\uc5d0 \uacfc\ud559\uc0ac\ud68c\ud559\uc774 \uc811\ubaa9\ub418\uc5c8\ub2e4. 1950\ub144\ub300 \ud6c4\ubc18\uc5d0 \ub4f1\uc7a5\ud55c \uacfc\ud559\uc790\uc758 \uc758\uc0ac\uc18c\ud1b5\uacfc \uc9d1\ub2e8\ud615\uc131\uc5d0 \uad00\ud55c \uc5f0\uad6c\uc5d0\ub3c4 \uc5ed\uc2dc \uacfc\ud559\uc0ac\ud68c\ud559\uc758 \uc5ec\ub7ec \uc774\ub860\ub4e4\uc774 \uc0ac\uc6a9\ub418\uc5c8\uace0, \uacfc\ud559\uc0ac\ud559\uc790 \ub4dc \uc194\ub77c \ud504\ub77c\uc774\uc2a4\uac00 \uc774\ub044\ub294 \uacc4\ub7c9\uc801 \uacfc\ud559\uc5f0\uad6c\uc640\ub3c4 \uc601\ud5a5\uc744 \uc8fc\uace0\ubc1b\uae30 \uc2dc\uc791\ud588\ub2e4. \uc774\uc640 \uac19\uc740 \uc131\uc7a5\uc758 \ubc30\uacbd\uc5d0\ub294 \uba38\ud134\uc774 1957\ub144\uc5d0 \ubbf8\uad6d \uc0ac\ud68c\ud559\ud68c \ud68c\uc7a5\uc5d0 \ucde8\uc784\ud588\ub2e4\ub294 \uc0ac\uc2e4\uc774\ub098 \uacfc\ud559\uc0ac\ud68c\ud559\uc758 \uae30\ubcf8\uc801\uc778 \ud328\ub7ec\ub2e4\uc784\uc774 \uac16\ucdb0\uc9c4 \uac83, \uadf8\ub9ac\uace0 \uc18c\ub828\uc774 \uc2a4\ud478\ud2b8\ub2c8\ud06c 1\ud638\ub97c \ubc1c\uc0ac\ud568\uc73c\ub85c\uc368 \uacfc\ud559\uc758 \uc0ac\ud68c\uc801 \uc5ed\ud560\uc5d0 \ub300\ud574 \uad00\uc2ec\uc774 \ub9ce\uc544\uc9c4 \uac83 \ub4f1\uc744 \uc608\ub85c \ub4e4 \uc218 \uc788\uc744 \uac83\uc774\ub2e4.", "paragraph_answer": " 1957\ub144 \uc744 \uae30\uc810\uc73c\ub85c \ud558\uc5ec \uacfc\ud559\uc0ac\ud68c\ud559\uc740 \uae09\uaca9\ud788 \uc131\uc7a5\ud558\uc600\ub2e4. 1950\ub144\ub300 \ucd08\ubc18\ubd80\ud130 \ubbf8\uad6d \uc0b0\uc5c5\uc0ac\ud68c\ud559\uc790\ub4e4\uc774 \ub3c5\uc790\uc801\uc73c\ub85c \uc9c4\ud589\ud574 \uc628, \uae30\uc5c5\u00b7 \uc815\ubd80 \ucd9c\uc5f0 \uc5f0\uad6c\uc18c\uc5d0 \uc18c\uc18d\ub41c \uacfc\ud559\uc790\uc758 \uc0dd\uc0b0\uc131\uc774\ub098 \ubcf5\ub9ac\ud6c4\uc0dd\u00b7 \uacbd\ub825\ud615\uc131\uc5d0 \ub300\ud55c \uc5f0\uad6c\uc5d0 \uacfc\ud559\uc0ac\ud68c\ud559\uc774 \uc811\ubaa9\ub418\uc5c8\ub2e4. 1950\ub144\ub300 \ud6c4\ubc18\uc5d0 \ub4f1\uc7a5\ud55c \uacfc\ud559\uc790\uc758 \uc758\uc0ac\uc18c\ud1b5\uacfc \uc9d1\ub2e8\ud615\uc131\uc5d0 \uad00\ud55c \uc5f0\uad6c\uc5d0\ub3c4 \uc5ed\uc2dc \uacfc\ud559\uc0ac\ud68c\ud559\uc758 \uc5ec\ub7ec \uc774\ub860\ub4e4\uc774 \uc0ac\uc6a9\ub418\uc5c8\uace0, \uacfc\ud559\uc0ac\ud559\uc790 \ub4dc \uc194\ub77c \ud504\ub77c\uc774\uc2a4\uac00 \uc774\ub044\ub294 \uacc4\ub7c9\uc801 \uacfc\ud559\uc5f0\uad6c\uc640\ub3c4 \uc601\ud5a5\uc744 \uc8fc\uace0\ubc1b\uae30 \uc2dc\uc791\ud588\ub2e4. \uc774\uc640 \uac19\uc740 \uc131\uc7a5\uc758 \ubc30\uacbd\uc5d0\ub294 \uba38\ud134\uc774 1957\ub144\uc5d0 \ubbf8\uad6d \uc0ac\ud68c\ud559\ud68c \ud68c\uc7a5\uc5d0 \ucde8\uc784\ud588\ub2e4\ub294 \uc0ac\uc2e4\uc774\ub098 \uacfc\ud559\uc0ac\ud68c\ud559\uc758 \uae30\ubcf8\uc801\uc778 \ud328\ub7ec\ub2e4\uc784\uc774 \uac16\ucdb0\uc9c4 \uac83, \uadf8\ub9ac\uace0 \uc18c\ub828\uc774 \uc2a4\ud478\ud2b8\ub2c8\ud06c 1\ud638\ub97c \ubc1c\uc0ac\ud568\uc73c\ub85c\uc368 \uacfc\ud559\uc758 \uc0ac\ud68c\uc801 \uc5ed\ud560\uc5d0 \ub300\ud574 \uad00\uc2ec\uc774 \ub9ce\uc544\uc9c4 \uac83 \ub4f1\uc744 \uc608\ub85c \ub4e4 \uc218 \uc788\uc744 \uac83\uc774\ub2e4.", "sentence_answer": " 1957\ub144 \uc744 \uae30\uc810\uc73c\ub85c \ud558\uc5ec \uacfc\ud559\uc0ac\ud68c\ud559\uc740 \uae09\uaca9\ud788 \uc131\uc7a5\ud558\uc600\ub2e4.", "paragraph_id": "5959284-14-0", "paragraph_question": "question: \uacfc\ud68c\uc0ac\ud68c\ud559\uc774 \uae09\uaca9\ud788 \uc131\uc7a5\ud55c \uc2dc\uae30\ub294?, context: 1957\ub144\uc744 \uae30\uc810\uc73c\ub85c \ud558\uc5ec \uacfc\ud559\uc0ac\ud68c\ud559\uc740 \uae09\uaca9\ud788 \uc131\uc7a5\ud558\uc600\ub2e4. 1950\ub144\ub300 \ucd08\ubc18\ubd80\ud130 \ubbf8\uad6d \uc0b0\uc5c5\uc0ac\ud68c\ud559\uc790\ub4e4\uc774 \ub3c5\uc790\uc801\uc73c\ub85c \uc9c4\ud589\ud574 \uc628, \uae30\uc5c5\u00b7 \uc815\ubd80 \ucd9c\uc5f0 \uc5f0\uad6c\uc18c\uc5d0 \uc18c\uc18d\ub41c \uacfc\ud559\uc790\uc758 \uc0dd\uc0b0\uc131\uc774\ub098 \ubcf5\ub9ac\ud6c4\uc0dd\u00b7 \uacbd\ub825\ud615\uc131\uc5d0 \ub300\ud55c \uc5f0\uad6c\uc5d0 \uacfc\ud559\uc0ac\ud68c\ud559\uc774 \uc811\ubaa9\ub418\uc5c8\ub2e4. 1950\ub144\ub300 \ud6c4\ubc18\uc5d0 \ub4f1\uc7a5\ud55c \uacfc\ud559\uc790\uc758 \uc758\uc0ac\uc18c\ud1b5\uacfc \uc9d1\ub2e8\ud615\uc131\uc5d0 \uad00\ud55c \uc5f0\uad6c\uc5d0\ub3c4 \uc5ed\uc2dc \uacfc\ud559\uc0ac\ud68c\ud559\uc758 \uc5ec\ub7ec \uc774\ub860\ub4e4\uc774 \uc0ac\uc6a9\ub418\uc5c8\uace0, \uacfc\ud559\uc0ac\ud559\uc790 \ub4dc \uc194\ub77c \ud504\ub77c\uc774\uc2a4\uac00 \uc774\ub044\ub294 \uacc4\ub7c9\uc801 \uacfc\ud559\uc5f0\uad6c\uc640\ub3c4 \uc601\ud5a5\uc744 \uc8fc\uace0\ubc1b\uae30 \uc2dc\uc791\ud588\ub2e4. \uc774\uc640 \uac19\uc740 \uc131\uc7a5\uc758 \ubc30\uacbd\uc5d0\ub294 \uba38\ud134\uc774 1957\ub144\uc5d0 \ubbf8\uad6d \uc0ac\ud68c\ud559\ud68c \ud68c\uc7a5\uc5d0 \ucde8\uc784\ud588\ub2e4\ub294 \uc0ac\uc2e4\uc774\ub098 \uacfc\ud559\uc0ac\ud68c\ud559\uc758 \uae30\ubcf8\uc801\uc778 \ud328\ub7ec\ub2e4\uc784\uc774 \uac16\ucdb0\uc9c4 \uac83, \uadf8\ub9ac\uace0 \uc18c\ub828\uc774 \uc2a4\ud478\ud2b8\ub2c8\ud06c 1\ud638\ub97c \ubc1c\uc0ac\ud568\uc73c\ub85c\uc368 \uacfc\ud559\uc758 \uc0ac\ud68c\uc801 \uc5ed\ud560\uc5d0 \ub300\ud574 \uad00\uc2ec\uc774 \ub9ce\uc544\uc9c4 \uac83 \ub4f1\uc744 \uc608\ub85c \ub4e4 \uc218 \uc788\uc744 \uac83\uc774\ub2e4."} {"question": "\uc544\uce74\ub370\uc774 \uc2dc\uc0c1\uc5d0\uc11c \uba87 \uac1c\uc758 \ubd80\ubb38\uc5d0 \ub178\ubbf8\ub124\uc774\ud2b8\uac00 \ub418\uc5c8\ub098?", "paragraph": "1962\ub144\uc5d0\ub294 \uc601\ud654\ud654\ub418\uc5b4 \uc544\uce74\ub370\ubbf8 \uc2dc\uc0c1\uc2dd\uc5d0\uc11c 8\uac1c \ubd80\ubb38\uc5d0 \ub178\ubbf8\ub124\uc774\ud2b8\ub418\ub294 \ucf8c\uac70\ub97c \uc774\ub8e9\ud588\uace0 \uc560\ud2f0\ucee4\uc2a4 \ud540\uce58 \ubcc0\ud638\uc0ac \uc5ed\ud560\uc744 \ub9e1\uc558\ub358 \uadf8\ub808\uace0\ub9ac \ud399\uc740 \ub0a8\uc6b0\uc8fc\uc5f0\uc0c1\uc744 \uc218\uc0c1\ud588\ub2e4. 2001\ub144\uc5d0\ub294 \uc2dc\uce74\uace0 '\ud55c \ub3c4\uc2dc \ud55c \ucc45' \uc6b4\ub3d9\uc758 \ub3c4\uc11c\ub85c \uc120\uc815\ub418\uc5b4 \ub2f9\uc2dc \uadf8\uacf3\uc758 \ud070 \ubb38\uc81c\uc600\ub358 \uc778\uc885 \ucc28\ubcc4\uc5d0 \ub300\ud55c \ud1a0\ub860\uc758 \uc7a5\uc744 \ub9c8\ub828\ud558\uace0 \uc2dc\ubbfc\ub4e4\uc758 \uc758\uc2dd\uc744 \ubcc0\ud654\uc2dc\ucf30\ub2e4. 1991\ub144 \ubbf8\uad6d \uad6d\ud68c \ub3c4\uc11c\uad00 \uc120\uc815 \uac00\uc7a5 \uc601\ud5a5\ub825 \uc788\ub294 \ucc45 2\uc704 (1\uc704\ub294 \uc131\uacbd), 1998\ub144 \ubbf8\uad6d \u300a\ub77c\uc774\ube0c\ub7ec\ub9ac \uc800\ub110\u300b \uc120\uc815 20\uc138\uae30 \uac00\uc7a5 \uc601\ud5a5\ub825 \uc788\ub294 \uc18c\uc124 1\uc704, 1999\ub144\uc5d0\ub294 \ubb38\ud559 \uc800\ub110\uc758 \uc5ec\ub860 \uc870\uc0ac \uacb0\uacfc \uae08\uc138\uae30 \ucd5c\uace0\uc758 \uc18c\uc124, 2006\ub144 \uc601\uad6d \ub3c4\uc11c\uad00-\ubc15\ubb3c\uad00 \uc544\uce74\uc774\ube0c \ud611\uc758\ud68c \uc120\uc815 \uc601\uad6d \uc0ac\uc11c\ub4e4\uc774 \uaf3d\uc740 \ubaa8\ub450\uac00 \uc77d\uc5b4\uc57c \ud560 \ucc45 1\uc704, 2008\ub144 \uc601\uad6d \uc628\ub77c\uc778 \uc11c\uc810 \ud50c\ub808\uc774\ub2f7\ucef4 \uc120\uc815 \uc601\uad6d\uc778\ub4e4\uc774 \uaf3d\uc740 \uc5ed\uc0ac\uc0c1 \ucd5c\uace0\uc758 \uc18c\uc124 1\uc704, 2008\ub144 \ub974\ub124\uc0c1\uc2a4 \ub7ec\ub2dd \uc870\uc0ac \ubbf8\uad6d \uace0\ub4f1\ud559\uc0dd\ub4e4\uc774 \uac00\uc7a5 \ub110\ub9ac \uc77d\ub294 \ucc45 1\uc704, \uad7f\ub9ac\uc988\ub2f7\ucef4 \uc120\uc815 20\uc138\uae30 \ub3c5\uc790\uc5d0\uac8c \uac00\uc7a5 \uc0ac\ub791 \ubc1b\uc740 \ucc45 1\uc704\uc5d0 \ubf51\ud614\uace0, \uc9c0\uae08\uae4c\uc9c0 \uc804 \uc138\uacc4 4\ucc9c\ub9cc \ubd80 \uc774\uc0c1\uc758 \ud310\ub9e4\uace0\ub97c \uc62c\ub9b0 \uac83\uc73c\ub85c \uc54c\ub824\uc84c\ub2e4. \ud55c\uad6d\uc5d0\uc11c\ub294 \ubb38\uc608\ucd9c\ud310\uc0ac\uc5d0\uc11c \ucd9c\uac04\ud558\ub2e4\uac00, 2015\ub144 7\uc6d4 \ucd9c\ud310\uc0ac \uc5f4\ub9b0\ucc45\ub4e4\uc5d0\uc11c \ubc88\uc5ed\uc744 \uc0c8\ub86d\uac8c \ub2e4\ub4ec\uc740 \uac1c\uc5ed\ud310\uc744 \ucd9c\uac04\ud588\ub2e4.", "answer": "8\uac1c", "sentence": "1962\ub144\uc5d0\ub294 \uc601\ud654\ud654\ub418\uc5b4 \uc544\uce74\ub370\ubbf8 \uc2dc\uc0c1\uc2dd\uc5d0\uc11c 8\uac1c \ubd80\ubb38\uc5d0 \ub178\ubbf8\ub124\uc774\ud2b8\ub418\ub294 \ucf8c\uac70\ub97c \uc774\ub8e9\ud588\uace0 \uc560\ud2f0\ucee4\uc2a4 \ud540\uce58 \ubcc0\ud638\uc0ac \uc5ed\ud560\uc744 \ub9e1\uc558\ub358 \uadf8\ub808\uace0\ub9ac \ud399\uc740 \ub0a8\uc6b0\uc8fc\uc5f0\uc0c1\uc744 \uc218\uc0c1\ud588\ub2e4.", "paragraph_sentence": " 1962\ub144\uc5d0\ub294 \uc601\ud654\ud654\ub418\uc5b4 \uc544\uce74\ub370\ubbf8 \uc2dc\uc0c1\uc2dd\uc5d0\uc11c 8\uac1c \ubd80\ubb38\uc5d0 \ub178\ubbf8\ub124\uc774\ud2b8\ub418\ub294 \ucf8c\uac70\ub97c \uc774\ub8e9\ud588\uace0 \uc560\ud2f0\ucee4\uc2a4 \ud540\uce58 \ubcc0\ud638\uc0ac \uc5ed\ud560\uc744 \ub9e1\uc558\ub358 \uadf8\ub808\uace0\ub9ac \ud399\uc740 \ub0a8\uc6b0\uc8fc\uc5f0\uc0c1\uc744 \uc218\uc0c1\ud588\ub2e4. 2001\ub144\uc5d0\ub294 \uc2dc\uce74\uace0 '\ud55c \ub3c4\uc2dc \ud55c \ucc45' \uc6b4\ub3d9\uc758 \ub3c4\uc11c\ub85c \uc120\uc815\ub418\uc5b4 \ub2f9\uc2dc \uadf8\uacf3\uc758 \ud070 \ubb38\uc81c\uc600\ub358 \uc778\uc885 \ucc28\ubcc4\uc5d0 \ub300\ud55c \ud1a0\ub860\uc758 \uc7a5\uc744 \ub9c8\ub828\ud558\uace0 \uc2dc\ubbfc\ub4e4\uc758 \uc758\uc2dd\uc744 \ubcc0\ud654\uc2dc\ucf30\ub2e4. 1991\ub144 \ubbf8\uad6d \uad6d\ud68c \ub3c4\uc11c\uad00 \uc120\uc815 \uac00\uc7a5 \uc601\ud5a5\ub825 \uc788\ub294 \ucc45 2\uc704 (1\uc704\ub294 \uc131\uacbd), 1998\ub144 \ubbf8\uad6d \u300a\ub77c\uc774\ube0c\ub7ec\ub9ac \uc800\ub110\u300b \uc120\uc815 20\uc138\uae30 \uac00\uc7a5 \uc601\ud5a5\ub825 \uc788\ub294 \uc18c\uc124 1\uc704, 1999\ub144\uc5d0\ub294 \ubb38\ud559 \uc800\ub110\uc758 \uc5ec\ub860 \uc870\uc0ac \uacb0\uacfc \uae08\uc138\uae30 \ucd5c\uace0\uc758 \uc18c\uc124, 2006\ub144 \uc601\uad6d \ub3c4\uc11c\uad00-\ubc15\ubb3c\uad00 \uc544\uce74\uc774\ube0c \ud611\uc758\ud68c \uc120\uc815 \uc601\uad6d \uc0ac\uc11c\ub4e4\uc774 \uaf3d\uc740 \ubaa8\ub450\uac00 \uc77d\uc5b4\uc57c \ud560 \ucc45 1\uc704, 2008\ub144 \uc601\uad6d \uc628\ub77c\uc778 \uc11c\uc810 \ud50c\ub808\uc774\ub2f7\ucef4 \uc120\uc815 \uc601\uad6d\uc778\ub4e4\uc774 \uaf3d\uc740 \uc5ed\uc0ac\uc0c1 \ucd5c\uace0\uc758 \uc18c\uc124 1\uc704, 2008\ub144 \ub974\ub124\uc0c1\uc2a4 \ub7ec\ub2dd \uc870\uc0ac \ubbf8\uad6d \uace0\ub4f1\ud559\uc0dd\ub4e4\uc774 \uac00\uc7a5 \ub110\ub9ac \uc77d\ub294 \ucc45 1\uc704, \uad7f\ub9ac\uc988\ub2f7\ucef4 \uc120\uc815 20\uc138\uae30 \ub3c5\uc790\uc5d0\uac8c \uac00\uc7a5 \uc0ac\ub791 \ubc1b\uc740 \ucc45 1\uc704\uc5d0 \ubf51\ud614\uace0, \uc9c0\uae08\uae4c\uc9c0 \uc804 \uc138\uacc4 4\ucc9c\ub9cc \ubd80 \uc774\uc0c1\uc758 \ud310\ub9e4\uace0\ub97c \uc62c\ub9b0 \uac83\uc73c\ub85c \uc54c\ub824\uc84c\ub2e4. \ud55c\uad6d\uc5d0\uc11c\ub294 \ubb38\uc608\ucd9c\ud310\uc0ac\uc5d0\uc11c \ucd9c\uac04\ud558\ub2e4\uac00, 2015\ub144 7\uc6d4 \ucd9c\ud310\uc0ac \uc5f4\ub9b0\ucc45\ub4e4\uc5d0\uc11c \ubc88\uc5ed\uc744 \uc0c8\ub86d\uac8c \ub2e4\ub4ec\uc740 \uac1c\uc5ed\ud310\uc744 \ucd9c\uac04\ud588\ub2e4.", "paragraph_answer": "1962\ub144\uc5d0\ub294 \uc601\ud654\ud654\ub418\uc5b4 \uc544\uce74\ub370\ubbf8 \uc2dc\uc0c1\uc2dd\uc5d0\uc11c 8\uac1c \ubd80\ubb38\uc5d0 \ub178\ubbf8\ub124\uc774\ud2b8\ub418\ub294 \ucf8c\uac70\ub97c \uc774\ub8e9\ud588\uace0 \uc560\ud2f0\ucee4\uc2a4 \ud540\uce58 \ubcc0\ud638\uc0ac \uc5ed\ud560\uc744 \ub9e1\uc558\ub358 \uadf8\ub808\uace0\ub9ac \ud399\uc740 \ub0a8\uc6b0\uc8fc\uc5f0\uc0c1\uc744 \uc218\uc0c1\ud588\ub2e4. 2001\ub144\uc5d0\ub294 \uc2dc\uce74\uace0 '\ud55c \ub3c4\uc2dc \ud55c \ucc45' \uc6b4\ub3d9\uc758 \ub3c4\uc11c\ub85c \uc120\uc815\ub418\uc5b4 \ub2f9\uc2dc \uadf8\uacf3\uc758 \ud070 \ubb38\uc81c\uc600\ub358 \uc778\uc885 \ucc28\ubcc4\uc5d0 \ub300\ud55c \ud1a0\ub860\uc758 \uc7a5\uc744 \ub9c8\ub828\ud558\uace0 \uc2dc\ubbfc\ub4e4\uc758 \uc758\uc2dd\uc744 \ubcc0\ud654\uc2dc\ucf30\ub2e4. 1991\ub144 \ubbf8\uad6d \uad6d\ud68c \ub3c4\uc11c\uad00 \uc120\uc815 \uac00\uc7a5 \uc601\ud5a5\ub825 \uc788\ub294 \ucc45 2\uc704 (1\uc704\ub294 \uc131\uacbd), 1998\ub144 \ubbf8\uad6d \u300a\ub77c\uc774\ube0c\ub7ec\ub9ac \uc800\ub110\u300b \uc120\uc815 20\uc138\uae30 \uac00\uc7a5 \uc601\ud5a5\ub825 \uc788\ub294 \uc18c\uc124 1\uc704, 1999\ub144\uc5d0\ub294 \ubb38\ud559 \uc800\ub110\uc758 \uc5ec\ub860 \uc870\uc0ac \uacb0\uacfc \uae08\uc138\uae30 \ucd5c\uace0\uc758 \uc18c\uc124, 2006\ub144 \uc601\uad6d \ub3c4\uc11c\uad00-\ubc15\ubb3c\uad00 \uc544\uce74\uc774\ube0c \ud611\uc758\ud68c \uc120\uc815 \uc601\uad6d \uc0ac\uc11c\ub4e4\uc774 \uaf3d\uc740 \ubaa8\ub450\uac00 \uc77d\uc5b4\uc57c \ud560 \ucc45 1\uc704, 2008\ub144 \uc601\uad6d \uc628\ub77c\uc778 \uc11c\uc810 \ud50c\ub808\uc774\ub2f7\ucef4 \uc120\uc815 \uc601\uad6d\uc778\ub4e4\uc774 \uaf3d\uc740 \uc5ed\uc0ac\uc0c1 \ucd5c\uace0\uc758 \uc18c\uc124 1\uc704, 2008\ub144 \ub974\ub124\uc0c1\uc2a4 \ub7ec\ub2dd \uc870\uc0ac \ubbf8\uad6d \uace0\ub4f1\ud559\uc0dd\ub4e4\uc774 \uac00\uc7a5 \ub110\ub9ac \uc77d\ub294 \ucc45 1\uc704, \uad7f\ub9ac\uc988\ub2f7\ucef4 \uc120\uc815 20\uc138\uae30 \ub3c5\uc790\uc5d0\uac8c \uac00\uc7a5 \uc0ac\ub791 \ubc1b\uc740 \ucc45 1\uc704\uc5d0 \ubf51\ud614\uace0, \uc9c0\uae08\uae4c\uc9c0 \uc804 \uc138\uacc4 4\ucc9c\ub9cc \ubd80 \uc774\uc0c1\uc758 \ud310\ub9e4\uace0\ub97c \uc62c\ub9b0 \uac83\uc73c\ub85c \uc54c\ub824\uc84c\ub2e4. \ud55c\uad6d\uc5d0\uc11c\ub294 \ubb38\uc608\ucd9c\ud310\uc0ac\uc5d0\uc11c \ucd9c\uac04\ud558\ub2e4\uac00, 2015\ub144 7\uc6d4 \ucd9c\ud310\uc0ac \uc5f4\ub9b0\ucc45\ub4e4\uc5d0\uc11c \ubc88\uc5ed\uc744 \uc0c8\ub86d\uac8c \ub2e4\ub4ec\uc740 \uac1c\uc5ed\ud310\uc744 \ucd9c\uac04\ud588\ub2e4.", "sentence_answer": "1962\ub144\uc5d0\ub294 \uc601\ud654\ud654\ub418\uc5b4 \uc544\uce74\ub370\ubbf8 \uc2dc\uc0c1\uc2dd\uc5d0\uc11c 8\uac1c \ubd80\ubb38\uc5d0 \ub178\ubbf8\ub124\uc774\ud2b8\ub418\ub294 \ucf8c\uac70\ub97c \uc774\ub8e9\ud588\uace0 \uc560\ud2f0\ucee4\uc2a4 \ud540\uce58 \ubcc0\ud638\uc0ac \uc5ed\ud560\uc744 \ub9e1\uc558\ub358 \uadf8\ub808\uace0\ub9ac \ud399\uc740 \ub0a8\uc6b0\uc8fc\uc5f0\uc0c1\uc744 \uc218\uc0c1\ud588\ub2e4.", "paragraph_id": "5935500-1-2", "paragraph_question": "question: \uc544\uce74\ub370\uc774 \uc2dc\uc0c1\uc5d0\uc11c \uba87 \uac1c\uc758 \ubd80\ubb38\uc5d0 \ub178\ubbf8\ub124\uc774\ud2b8\uac00 \ub418\uc5c8\ub098?, context: 1962\ub144\uc5d0\ub294 \uc601\ud654\ud654\ub418\uc5b4 \uc544\uce74\ub370\ubbf8 \uc2dc\uc0c1\uc2dd\uc5d0\uc11c 8\uac1c \ubd80\ubb38\uc5d0 \ub178\ubbf8\ub124\uc774\ud2b8\ub418\ub294 \ucf8c\uac70\ub97c \uc774\ub8e9\ud588\uace0 \uc560\ud2f0\ucee4\uc2a4 \ud540\uce58 \ubcc0\ud638\uc0ac \uc5ed\ud560\uc744 \ub9e1\uc558\ub358 \uadf8\ub808\uace0\ub9ac \ud399\uc740 \ub0a8\uc6b0\uc8fc\uc5f0\uc0c1\uc744 \uc218\uc0c1\ud588\ub2e4. 2001\ub144\uc5d0\ub294 \uc2dc\uce74\uace0 '\ud55c \ub3c4\uc2dc \ud55c \ucc45' \uc6b4\ub3d9\uc758 \ub3c4\uc11c\ub85c \uc120\uc815\ub418\uc5b4 \ub2f9\uc2dc \uadf8\uacf3\uc758 \ud070 \ubb38\uc81c\uc600\ub358 \uc778\uc885 \ucc28\ubcc4\uc5d0 \ub300\ud55c \ud1a0\ub860\uc758 \uc7a5\uc744 \ub9c8\ub828\ud558\uace0 \uc2dc\ubbfc\ub4e4\uc758 \uc758\uc2dd\uc744 \ubcc0\ud654\uc2dc\ucf30\ub2e4. 1991\ub144 \ubbf8\uad6d \uad6d\ud68c \ub3c4\uc11c\uad00 \uc120\uc815 \uac00\uc7a5 \uc601\ud5a5\ub825 \uc788\ub294 \ucc45 2\uc704 (1\uc704\ub294 \uc131\uacbd), 1998\ub144 \ubbf8\uad6d \u300a\ub77c\uc774\ube0c\ub7ec\ub9ac \uc800\ub110\u300b \uc120\uc815 20\uc138\uae30 \uac00\uc7a5 \uc601\ud5a5\ub825 \uc788\ub294 \uc18c\uc124 1\uc704, 1999\ub144\uc5d0\ub294 \ubb38\ud559 \uc800\ub110\uc758 \uc5ec\ub860 \uc870\uc0ac \uacb0\uacfc \uae08\uc138\uae30 \ucd5c\uace0\uc758 \uc18c\uc124, 2006\ub144 \uc601\uad6d \ub3c4\uc11c\uad00-\ubc15\ubb3c\uad00 \uc544\uce74\uc774\ube0c \ud611\uc758\ud68c \uc120\uc815 \uc601\uad6d \uc0ac\uc11c\ub4e4\uc774 \uaf3d\uc740 \ubaa8\ub450\uac00 \uc77d\uc5b4\uc57c \ud560 \ucc45 1\uc704, 2008\ub144 \uc601\uad6d \uc628\ub77c\uc778 \uc11c\uc810 \ud50c\ub808\uc774\ub2f7\ucef4 \uc120\uc815 \uc601\uad6d\uc778\ub4e4\uc774 \uaf3d\uc740 \uc5ed\uc0ac\uc0c1 \ucd5c\uace0\uc758 \uc18c\uc124 1\uc704, 2008\ub144 \ub974\ub124\uc0c1\uc2a4 \ub7ec\ub2dd \uc870\uc0ac \ubbf8\uad6d \uace0\ub4f1\ud559\uc0dd\ub4e4\uc774 \uac00\uc7a5 \ub110\ub9ac \uc77d\ub294 \ucc45 1\uc704, \uad7f\ub9ac\uc988\ub2f7\ucef4 \uc120\uc815 20\uc138\uae30 \ub3c5\uc790\uc5d0\uac8c \uac00\uc7a5 \uc0ac\ub791 \ubc1b\uc740 \ucc45 1\uc704\uc5d0 \ubf51\ud614\uace0, \uc9c0\uae08\uae4c\uc9c0 \uc804 \uc138\uacc4 4\ucc9c\ub9cc \ubd80 \uc774\uc0c1\uc758 \ud310\ub9e4\uace0\ub97c \uc62c\ub9b0 \uac83\uc73c\ub85c \uc54c\ub824\uc84c\ub2e4. \ud55c\uad6d\uc5d0\uc11c\ub294 \ubb38\uc608\ucd9c\ud310\uc0ac\uc5d0\uc11c \ucd9c\uac04\ud558\ub2e4\uac00, 2015\ub144 7\uc6d4 \ucd9c\ud310\uc0ac \uc5f4\ub9b0\ucc45\ub4e4\uc5d0\uc11c \ubc88\uc5ed\uc744 \uc0c8\ub86d\uac8c \ub2e4\ub4ec\uc740 \uac1c\uc5ed\ud310\uc744 \ucd9c\uac04\ud588\ub2e4."} {"question": "\uc5ec\uc131 \ucc38\uc815\uad8c \uc6b4\ub3d9\uc744 \ubc18\ub300\ud55c \uc0ac\ub78c\uc740?", "paragraph": "\uc5ec\uc131 \ucc38\uc815\uad8c \uc694\uad6c, \uac01\uc885 \ubb38\ud654\uc801 \uc0ac\ud68c\uc801 \ucc28\ubcc4\uc5d0 \ub300\ud55c \uc800\ud56d \ub4f1\uc740 \uc911\uc0b0\uce35 \uc774\uc0c1 \ub0a8\uc131\uacfc \uacbd\uc81c\uc801\uc73c\ub85c \ub3d9\ub4f1\ud558\uc9c0\ub9cc \ubb38\ud654\uc801 \uc0ac\ud68c\uc801\uc73c\ub85c\ub294 \ubd88\ud3c9\ub4f1\ud55c \uc911\uc0b0\uce35 \uc774\uc0c1 \uc5ec\uc131\uc758 \uc774\ud574\uad00\uacc4\ub97c \uc704\ud55c \uac83\uc774\uba70, \ucf5c\ub860\ud0c0\uc774\uc5d0 \uc758\ud558\uba74 \uc774\ub7ec\ud55c \uc0ac\uc2e4\uc740 \uc5ec\uc131 \ucc38\uc815\uad8c \uc694\uad6c\uac00 \ub178\ub3d9\uacc4\uae09 \uc5ec\uc131\uc5d0\uac8c\uae4c\uc9c0 \ud61c\ud0dd\uc774 \ub3cc\uc544\uac00\ub294 \uc5ec\uc131\uc758 \ubcf4\ud1b5\uc120\uac70\uad8c\uc73c\ub85c \uc774\uc5b4\uc9c0\uc9c0 \ubabb\ud588\uc74c\uc5d0 \uc758\ud574 \uc99d\uac70\ub41c\ub2e4. \uc2ec\uc9c0\uc5b4 \ub9c8\ub354 \uc874\uc2a4\ub294 \uc5ec\uc131\uc8fc\uc758\uac00 \uacc4\uae09\uc6b4\ub3d9\uc744 \uc800\ud574\ud560 \uc218 \uc788\ub294 \uc720\ud55c\uc5ec\uc131\ub4e4\uc758 \ub9d0\uc794\uce58\ub77c\uace0 \ube44\ud310\ud558\uba70 \uc5ec\uc131 \ucc38\uc815\uad8c \uc6b4\ub3d9\ub3c4 \ubc18\ub300\ud588\ub2e4. \uc874\uc2a4\ub294 \ucf5c\ub85c\ub77c\ub3c4 \uc8fc\uc5d0\uc11c \uc5ec\uc131 \ud22c\ud45c\uad8c\uc774 \uc0dd\uacbc\uc74c\uc5d0\ub3c4 \ub178\ub3d9 \ud3ed\ub825\uc744 \ub9c9\ub294\ub370\ub294 \ud558\ub098\ub3c4 \uae30\uc5ec\ud558\uc9c0 \ubabb\ud588\uc74c\uc744 \uc9c0\uc801\ud558\uace0, 1925\ub144 \uc5ec\uc131 \ud22c\ud45c\uad8c\uc774 \uc81c\uc815\ub41c \ub4a4\uc5d0\ub294 \uc774\uac83\uc744 \u201c\uae08\uad8c\uc7ac\ubc8c(plutocrats) \uc5ec\uc131\uc758 \uc870\uc9c1\ud654\u201d\ub77c\uace0 \ud588\ub2e4.", "answer": "\ub9c8\ub354 \uc874\uc2a4", "sentence": "\uc2ec\uc9c0\uc5b4 \ub9c8\ub354 \uc874\uc2a4 \ub294 \uc5ec\uc131\uc8fc\uc758\uac00 \uacc4\uae09\uc6b4\ub3d9\uc744 \uc800\ud574\ud560 \uc218 \uc788\ub294 \uc720\ud55c\uc5ec\uc131\ub4e4\uc758 \ub9d0\uc794\uce58\ub77c\uace0 \ube44\ud310\ud558\uba70 \uc5ec\uc131 \ucc38\uc815\uad8c \uc6b4\ub3d9\ub3c4 \ubc18\ub300\ud588\ub2e4.", "paragraph_sentence": "\uc5ec\uc131 \ucc38\uc815\uad8c \uc694\uad6c, \uac01\uc885 \ubb38\ud654\uc801 \uc0ac\ud68c\uc801 \ucc28\ubcc4\uc5d0 \ub300\ud55c \uc800\ud56d \ub4f1\uc740 \uc911\uc0b0\uce35 \uc774\uc0c1 \ub0a8\uc131\uacfc \uacbd\uc81c\uc801\uc73c\ub85c \ub3d9\ub4f1\ud558\uc9c0\ub9cc \ubb38\ud654\uc801 \uc0ac\ud68c\uc801\uc73c\ub85c\ub294 \ubd88\ud3c9\ub4f1\ud55c \uc911\uc0b0\uce35 \uc774\uc0c1 \uc5ec\uc131\uc758 \uc774\ud574\uad00\uacc4\ub97c \uc704\ud55c \uac83\uc774\uba70, \ucf5c\ub860\ud0c0\uc774\uc5d0 \uc758\ud558\uba74 \uc774\ub7ec\ud55c \uc0ac\uc2e4\uc740 \uc5ec\uc131 \ucc38\uc815\uad8c \uc694\uad6c\uac00 \ub178\ub3d9\uacc4\uae09 \uc5ec\uc131\uc5d0\uac8c\uae4c\uc9c0 \ud61c\ud0dd\uc774 \ub3cc\uc544\uac00\ub294 \uc5ec\uc131\uc758 \ubcf4\ud1b5\uc120\uac70\uad8c\uc73c\ub85c \uc774\uc5b4\uc9c0\uc9c0 \ubabb\ud588\uc74c\uc5d0 \uc758\ud574 \uc99d\uac70\ub41c\ub2e4. \uc2ec\uc9c0\uc5b4 \ub9c8\ub354 \uc874\uc2a4 \ub294 \uc5ec\uc131\uc8fc\uc758\uac00 \uacc4\uae09\uc6b4\ub3d9\uc744 \uc800\ud574\ud560 \uc218 \uc788\ub294 \uc720\ud55c\uc5ec\uc131\ub4e4\uc758 \ub9d0\uc794\uce58\ub77c\uace0 \ube44\ud310\ud558\uba70 \uc5ec\uc131 \ucc38\uc815\uad8c \uc6b4\ub3d9\ub3c4 \ubc18\ub300\ud588\ub2e4. \uc874\uc2a4\ub294 \ucf5c\ub85c\ub77c\ub3c4 \uc8fc\uc5d0\uc11c \uc5ec\uc131 \ud22c\ud45c\uad8c\uc774 \uc0dd\uacbc\uc74c\uc5d0\ub3c4 \ub178\ub3d9 \ud3ed\ub825\uc744 \ub9c9\ub294\ub370\ub294 \ud558\ub098\ub3c4 \uae30\uc5ec\ud558\uc9c0 \ubabb\ud588\uc74c\uc744 \uc9c0\uc801\ud558\uace0, 1925\ub144 \uc5ec\uc131 \ud22c\ud45c\uad8c\uc774 \uc81c\uc815\ub41c \ub4a4\uc5d0\ub294 \uc774\uac83\uc744 \u201c\uae08\uad8c\uc7ac\ubc8c(plutocrats) \uc5ec\uc131\uc758 \uc870\uc9c1\ud654\u201d\ub77c\uace0 \ud588\ub2e4.", "paragraph_answer": "\uc5ec\uc131 \ucc38\uc815\uad8c \uc694\uad6c, \uac01\uc885 \ubb38\ud654\uc801 \uc0ac\ud68c\uc801 \ucc28\ubcc4\uc5d0 \ub300\ud55c \uc800\ud56d \ub4f1\uc740 \uc911\uc0b0\uce35 \uc774\uc0c1 \ub0a8\uc131\uacfc \uacbd\uc81c\uc801\uc73c\ub85c \ub3d9\ub4f1\ud558\uc9c0\ub9cc \ubb38\ud654\uc801 \uc0ac\ud68c\uc801\uc73c\ub85c\ub294 \ubd88\ud3c9\ub4f1\ud55c \uc911\uc0b0\uce35 \uc774\uc0c1 \uc5ec\uc131\uc758 \uc774\ud574\uad00\uacc4\ub97c \uc704\ud55c \uac83\uc774\uba70, \ucf5c\ub860\ud0c0\uc774\uc5d0 \uc758\ud558\uba74 \uc774\ub7ec\ud55c \uc0ac\uc2e4\uc740 \uc5ec\uc131 \ucc38\uc815\uad8c \uc694\uad6c\uac00 \ub178\ub3d9\uacc4\uae09 \uc5ec\uc131\uc5d0\uac8c\uae4c\uc9c0 \ud61c\ud0dd\uc774 \ub3cc\uc544\uac00\ub294 \uc5ec\uc131\uc758 \ubcf4\ud1b5\uc120\uac70\uad8c\uc73c\ub85c \uc774\uc5b4\uc9c0\uc9c0 \ubabb\ud588\uc74c\uc5d0 \uc758\ud574 \uc99d\uac70\ub41c\ub2e4. \uc2ec\uc9c0\uc5b4 \ub9c8\ub354 \uc874\uc2a4 \ub294 \uc5ec\uc131\uc8fc\uc758\uac00 \uacc4\uae09\uc6b4\ub3d9\uc744 \uc800\ud574\ud560 \uc218 \uc788\ub294 \uc720\ud55c\uc5ec\uc131\ub4e4\uc758 \ub9d0\uc794\uce58\ub77c\uace0 \ube44\ud310\ud558\uba70 \uc5ec\uc131 \ucc38\uc815\uad8c \uc6b4\ub3d9\ub3c4 \ubc18\ub300\ud588\ub2e4. \uc874\uc2a4\ub294 \ucf5c\ub85c\ub77c\ub3c4 \uc8fc\uc5d0\uc11c \uc5ec\uc131 \ud22c\ud45c\uad8c\uc774 \uc0dd\uacbc\uc74c\uc5d0\ub3c4 \ub178\ub3d9 \ud3ed\ub825\uc744 \ub9c9\ub294\ub370\ub294 \ud558\ub098\ub3c4 \uae30\uc5ec\ud558\uc9c0 \ubabb\ud588\uc74c\uc744 \uc9c0\uc801\ud558\uace0, 1925\ub144 \uc5ec\uc131 \ud22c\ud45c\uad8c\uc774 \uc81c\uc815\ub41c \ub4a4\uc5d0\ub294 \uc774\uac83\uc744 \u201c\uae08\uad8c\uc7ac\ubc8c(plutocrats) \uc5ec\uc131\uc758 \uc870\uc9c1\ud654\u201d\ub77c\uace0 \ud588\ub2e4.", "sentence_answer": "\uc2ec\uc9c0\uc5b4 \ub9c8\ub354 \uc874\uc2a4 \ub294 \uc5ec\uc131\uc8fc\uc758\uac00 \uacc4\uae09\uc6b4\ub3d9\uc744 \uc800\ud574\ud560 \uc218 \uc788\ub294 \uc720\ud55c\uc5ec\uc131\ub4e4\uc758 \ub9d0\uc794\uce58\ub77c\uace0 \ube44\ud310\ud558\uba70 \uc5ec\uc131 \ucc38\uc815\uad8c \uc6b4\ub3d9\ub3c4 \ubc18\ub300\ud588\ub2e4.", "paragraph_id": "6529197-4-0", "paragraph_question": "question: \uc5ec\uc131 \ucc38\uc815\uad8c \uc6b4\ub3d9\uc744 \ubc18\ub300\ud55c \uc0ac\ub78c\uc740?, context: \uc5ec\uc131 \ucc38\uc815\uad8c \uc694\uad6c, \uac01\uc885 \ubb38\ud654\uc801 \uc0ac\ud68c\uc801 \ucc28\ubcc4\uc5d0 \ub300\ud55c \uc800\ud56d \ub4f1\uc740 \uc911\uc0b0\uce35 \uc774\uc0c1 \ub0a8\uc131\uacfc \uacbd\uc81c\uc801\uc73c\ub85c \ub3d9\ub4f1\ud558\uc9c0\ub9cc \ubb38\ud654\uc801 \uc0ac\ud68c\uc801\uc73c\ub85c\ub294 \ubd88\ud3c9\ub4f1\ud55c \uc911\uc0b0\uce35 \uc774\uc0c1 \uc5ec\uc131\uc758 \uc774\ud574\uad00\uacc4\ub97c \uc704\ud55c \uac83\uc774\uba70, \ucf5c\ub860\ud0c0\uc774\uc5d0 \uc758\ud558\uba74 \uc774\ub7ec\ud55c \uc0ac\uc2e4\uc740 \uc5ec\uc131 \ucc38\uc815\uad8c \uc694\uad6c\uac00 \ub178\ub3d9\uacc4\uae09 \uc5ec\uc131\uc5d0\uac8c\uae4c\uc9c0 \ud61c\ud0dd\uc774 \ub3cc\uc544\uac00\ub294 \uc5ec\uc131\uc758 \ubcf4\ud1b5\uc120\uac70\uad8c\uc73c\ub85c \uc774\uc5b4\uc9c0\uc9c0 \ubabb\ud588\uc74c\uc5d0 \uc758\ud574 \uc99d\uac70\ub41c\ub2e4. \uc2ec\uc9c0\uc5b4 \ub9c8\ub354 \uc874\uc2a4\ub294 \uc5ec\uc131\uc8fc\uc758\uac00 \uacc4\uae09\uc6b4\ub3d9\uc744 \uc800\ud574\ud560 \uc218 \uc788\ub294 \uc720\ud55c\uc5ec\uc131\ub4e4\uc758 \ub9d0\uc794\uce58\ub77c\uace0 \ube44\ud310\ud558\uba70 \uc5ec\uc131 \ucc38\uc815\uad8c \uc6b4\ub3d9\ub3c4 \ubc18\ub300\ud588\ub2e4. \uc874\uc2a4\ub294 \ucf5c\ub85c\ub77c\ub3c4 \uc8fc\uc5d0\uc11c \uc5ec\uc131 \ud22c\ud45c\uad8c\uc774 \uc0dd\uacbc\uc74c\uc5d0\ub3c4 \ub178\ub3d9 \ud3ed\ub825\uc744 \ub9c9\ub294\ub370\ub294 \ud558\ub098\ub3c4 \uae30\uc5ec\ud558\uc9c0 \ubabb\ud588\uc74c\uc744 \uc9c0\uc801\ud558\uace0, 1925\ub144 \uc5ec\uc131 \ud22c\ud45c\uad8c\uc774 \uc81c\uc815\ub41c \ub4a4\uc5d0\ub294 \uc774\uac83\uc744 \u201c\uae08\uad8c\uc7ac\ubc8c(plutocrats) \uc5ec\uc131\uc758 \uc870\uc9c1\ud654\u201d\ub77c\uace0 \ud588\ub2e4."} {"question": "\uc774\uc601\ud559\uc758 \ubd80\uce5c\uc740 \uc5b4\ub290 \uc9c0\uc5ed\uc758 \uc7ac\ub825\uac00\uc600\ub294\uac00?", "paragraph": "\uc774\uc601\ud559\uc740 \uacbd\uae30\ub3c4 \uc758\uc815\ubd80\uc2dc\uc5d0\uc11c \uc911\ud559\uad50\ub97c \ub2e4\ub154\ub2e4. \uc774\uc601\ud559\uc758 \uc911\ud559\uad50 \uad50\uc0ac\ub294 \"1996\ub144 \uc774\uc601\ud559\uc774 \uc911\ud559\uad50 2\ud559\ub144 \ub54c \ubab8\uc5d0 \ud53c\ub97c \ubb3b\ud788\uace0 \ub4f1\uad50\ud574 \ud53c\ud574 \uc5ec\ud559\uc0dd\uc758 \ud53c\ub77c\uba74\uc11c \ub3d9\uae09\uc0dd\ub4e4\uc5d0\uac8c \uc5ec\ud559\uc0dd \uc131\ud3ed\ud589 \uc0ac\uc2e4\uc744 \uc790\ub791\ud558\uace0 \ub2e4\ub140 \uc870\uc0ac\ud588\ub354\ub2c8 \uc2dc\uc778\ud588\ub2e4\"\uba70 \"\ud1f4\ud559\uc744 \uc2dc\ud0a4\ub824 \ud588\uc73c\ub098 \uad50\uc7a5\uc758 \ubc18\ub300\ub85c \uacbd\ubbf8\ud55c \uc9d5\uacc4\uc5d0 \uadf8\ucce4\ub2e4\"\uace0 \ud68c\uc0c1\ud588\ub2e4. \"\uc774\uc601\ud559\uc740 \ubb34\ub2e8\uacb0\uc11d \uc77c\uc218\uac00 \uc218\uc5c5\uc77c\uc218 1/3\uc744 \ucd08\uacfc\ud574 \uc878\uc5c5\uc774 \ubd88\uac00\ud55c \uc0c1\ud669\uc774\uc5c8\ub294\ub370 \uad50\uc7a5\uc774 \uc9c1\uad8c\uc73c\ub85c \uc0c1\uc704\ubc95\uc744 \uc704\ubc18\ud558\uba74\uc11c \uc740\ud3d0\ud574 \uc878\uc5c5\uc2dc\ucf30\ub2e4\"\uba70 \"\ub2f9\uc2dc \uc774\uc601\ud559\uc758 \ubd80\uce5c\uc774 \uc758\uc815\ubd80\uc758 \uc7ac\ub825\uac00\uc600\ub294\ub370 \uc5b4\uba38\ub2c8\uc758 \uce58\ub9db\ubc14\ub78c\uc774 \uc14c\ub2e4\"\uace0 \ub367\ubd99\uc600\ub2e4. \uc774\uc601\ud559\uc758 \uc911\ud559\uad50 \ub3d9\ucc3d\ub4e4\uc740 \"\uc774\uc601\ud559\uc774 \ub610\ub798 \uce5c\uad6c 3\uba85\uacfc \ud55c \uc5ec\uc131\uc744 \uc131\ud3ed\ud589\ud55c \ubaa8\uc2b5\uc744 \uc9c1\uc811 \ubaa9\uaca9\ud588\ub2e4\"\uba70 \ucd08\ub4f1\ud559\uc0dd\uc5d0\uac8c \uacfc\uc790\ub97c \uc8fc\uba70 \ub370\ub824\uac00 \uc131\ud3ed\ud589\ud588\ub2e4\uace0\ub3c4 \ub9d0\ud588\ub2e4. \uc774\uc601\ud559\uc758 \uc5b4\uba38\ub2c8 \uae40\ubaa8\uc528\ub294 \uc774\uc601\ud559\uc758 \uc544\ubc84\uc9c0\uac00 \uc0ac\uc5c5\uc5d0 \uc2e4\ud328\ud558\uba74\uc11c \uc774\ud63c\ud588\ub2e4.", "answer": "\uc758\uc815\ubd80", "sentence": "\uc774\uc601\ud559\uc740 \uacbd\uae30\ub3c4 \uc758\uc815\ubd80 \uc2dc\uc5d0\uc11c \uc911\ud559\uad50\ub97c \ub2e4\ub154\ub2e4.", "paragraph_sentence": " \uc774\uc601\ud559\uc740 \uacbd\uae30\ub3c4 \uc758\uc815\ubd80 \uc2dc\uc5d0\uc11c \uc911\ud559\uad50\ub97c \ub2e4\ub154\ub2e4. \uc774\uc601\ud559\uc758 \uc911\ud559\uad50 \uad50\uc0ac\ub294 \"1996\ub144 \uc774\uc601\ud559\uc774 \uc911\ud559\uad50 2\ud559\ub144 \ub54c \ubab8\uc5d0 \ud53c\ub97c \ubb3b\ud788\uace0 \ub4f1\uad50\ud574 \ud53c\ud574 \uc5ec\ud559\uc0dd\uc758 \ud53c\ub77c\uba74\uc11c \ub3d9\uae09\uc0dd\ub4e4\uc5d0\uac8c \uc5ec\ud559\uc0dd \uc131\ud3ed\ud589 \uc0ac\uc2e4\uc744 \uc790\ub791\ud558\uace0 \ub2e4\ub140 \uc870\uc0ac\ud588\ub354\ub2c8 \uc2dc\uc778\ud588\ub2e4\"\uba70 \"\ud1f4\ud559\uc744 \uc2dc\ud0a4\ub824 \ud588\uc73c\ub098 \uad50\uc7a5\uc758 \ubc18\ub300\ub85c \uacbd\ubbf8\ud55c \uc9d5\uacc4\uc5d0 \uadf8\ucce4\ub2e4\"\uace0 \ud68c\uc0c1\ud588\ub2e4. \"\uc774\uc601\ud559\uc740 \ubb34\ub2e8\uacb0\uc11d \uc77c\uc218\uac00 \uc218\uc5c5\uc77c\uc218 1/3\uc744 \ucd08\uacfc\ud574 \uc878\uc5c5\uc774 \ubd88\uac00\ud55c \uc0c1\ud669\uc774\uc5c8\ub294\ub370 \uad50\uc7a5\uc774 \uc9c1\uad8c\uc73c\ub85c \uc0c1\uc704\ubc95\uc744 \uc704\ubc18\ud558\uba74\uc11c \uc740\ud3d0\ud574 \uc878\uc5c5\uc2dc\ucf30\ub2e4\"\uba70 \"\ub2f9\uc2dc \uc774\uc601\ud559\uc758 \ubd80\uce5c\uc774 \uc758\uc815\ubd80\uc758 \uc7ac\ub825\uac00\uc600\ub294\ub370 \uc5b4\uba38\ub2c8\uc758 \uce58\ub9db\ubc14\ub78c\uc774 \uc14c\ub2e4\"\uace0 \ub367\ubd99\uc600\ub2e4. \uc774\uc601\ud559\uc758 \uc911\ud559\uad50 \ub3d9\ucc3d\ub4e4\uc740 \"\uc774\uc601\ud559\uc774 \ub610\ub798 \uce5c\uad6c 3\uba85\uacfc \ud55c \uc5ec\uc131\uc744 \uc131\ud3ed\ud589\ud55c \ubaa8\uc2b5\uc744 \uc9c1\uc811 \ubaa9\uaca9\ud588\ub2e4\"\uba70 \ucd08\ub4f1\ud559\uc0dd\uc5d0\uac8c \uacfc\uc790\ub97c \uc8fc\uba70 \ub370\ub824\uac00 \uc131\ud3ed\ud589\ud588\ub2e4\uace0\ub3c4 \ub9d0\ud588\ub2e4. \uc774\uc601\ud559\uc758 \uc5b4\uba38\ub2c8 \uae40\ubaa8\uc528\ub294 \uc774\uc601\ud559\uc758 \uc544\ubc84\uc9c0\uac00 \uc0ac\uc5c5\uc5d0 \uc2e4\ud328\ud558\uba74\uc11c \uc774\ud63c\ud588\ub2e4.", "paragraph_answer": "\uc774\uc601\ud559\uc740 \uacbd\uae30\ub3c4 \uc758\uc815\ubd80 \uc2dc\uc5d0\uc11c \uc911\ud559\uad50\ub97c \ub2e4\ub154\ub2e4. \uc774\uc601\ud559\uc758 \uc911\ud559\uad50 \uad50\uc0ac\ub294 \"1996\ub144 \uc774\uc601\ud559\uc774 \uc911\ud559\uad50 2\ud559\ub144 \ub54c \ubab8\uc5d0 \ud53c\ub97c \ubb3b\ud788\uace0 \ub4f1\uad50\ud574 \ud53c\ud574 \uc5ec\ud559\uc0dd\uc758 \ud53c\ub77c\uba74\uc11c \ub3d9\uae09\uc0dd\ub4e4\uc5d0\uac8c \uc5ec\ud559\uc0dd \uc131\ud3ed\ud589 \uc0ac\uc2e4\uc744 \uc790\ub791\ud558\uace0 \ub2e4\ub140 \uc870\uc0ac\ud588\ub354\ub2c8 \uc2dc\uc778\ud588\ub2e4\"\uba70 \"\ud1f4\ud559\uc744 \uc2dc\ud0a4\ub824 \ud588\uc73c\ub098 \uad50\uc7a5\uc758 \ubc18\ub300\ub85c \uacbd\ubbf8\ud55c \uc9d5\uacc4\uc5d0 \uadf8\ucce4\ub2e4\"\uace0 \ud68c\uc0c1\ud588\ub2e4. \"\uc774\uc601\ud559\uc740 \ubb34\ub2e8\uacb0\uc11d \uc77c\uc218\uac00 \uc218\uc5c5\uc77c\uc218 1/3\uc744 \ucd08\uacfc\ud574 \uc878\uc5c5\uc774 \ubd88\uac00\ud55c \uc0c1\ud669\uc774\uc5c8\ub294\ub370 \uad50\uc7a5\uc774 \uc9c1\uad8c\uc73c\ub85c \uc0c1\uc704\ubc95\uc744 \uc704\ubc18\ud558\uba74\uc11c \uc740\ud3d0\ud574 \uc878\uc5c5\uc2dc\ucf30\ub2e4\"\uba70 \"\ub2f9\uc2dc \uc774\uc601\ud559\uc758 \ubd80\uce5c\uc774 \uc758\uc815\ubd80\uc758 \uc7ac\ub825\uac00\uc600\ub294\ub370 \uc5b4\uba38\ub2c8\uc758 \uce58\ub9db\ubc14\ub78c\uc774 \uc14c\ub2e4\"\uace0 \ub367\ubd99\uc600\ub2e4. \uc774\uc601\ud559\uc758 \uc911\ud559\uad50 \ub3d9\ucc3d\ub4e4\uc740 \"\uc774\uc601\ud559\uc774 \ub610\ub798 \uce5c\uad6c 3\uba85\uacfc \ud55c \uc5ec\uc131\uc744 \uc131\ud3ed\ud589\ud55c \ubaa8\uc2b5\uc744 \uc9c1\uc811 \ubaa9\uaca9\ud588\ub2e4\"\uba70 \ucd08\ub4f1\ud559\uc0dd\uc5d0\uac8c \uacfc\uc790\ub97c \uc8fc\uba70 \ub370\ub824\uac00 \uc131\ud3ed\ud589\ud588\ub2e4\uace0\ub3c4 \ub9d0\ud588\ub2e4. \uc774\uc601\ud559\uc758 \uc5b4\uba38\ub2c8 \uae40\ubaa8\uc528\ub294 \uc774\uc601\ud559\uc758 \uc544\ubc84\uc9c0\uac00 \uc0ac\uc5c5\uc5d0 \uc2e4\ud328\ud558\uba74\uc11c \uc774\ud63c\ud588\ub2e4.", "sentence_answer": "\uc774\uc601\ud559\uc740 \uacbd\uae30\ub3c4 \uc758\uc815\ubd80 \uc2dc\uc5d0\uc11c \uc911\ud559\uad50\ub97c \ub2e4\ub154\ub2e4.", "paragraph_id": "6545802-0-1", "paragraph_question": "question: \uc774\uc601\ud559\uc758 \ubd80\uce5c\uc740 \uc5b4\ub290 \uc9c0\uc5ed\uc758 \uc7ac\ub825\uac00\uc600\ub294\uac00?, context: \uc774\uc601\ud559\uc740 \uacbd\uae30\ub3c4 \uc758\uc815\ubd80\uc2dc\uc5d0\uc11c \uc911\ud559\uad50\ub97c \ub2e4\ub154\ub2e4. \uc774\uc601\ud559\uc758 \uc911\ud559\uad50 \uad50\uc0ac\ub294 \"1996\ub144 \uc774\uc601\ud559\uc774 \uc911\ud559\uad50 2\ud559\ub144 \ub54c \ubab8\uc5d0 \ud53c\ub97c \ubb3b\ud788\uace0 \ub4f1\uad50\ud574 \ud53c\ud574 \uc5ec\ud559\uc0dd\uc758 \ud53c\ub77c\uba74\uc11c \ub3d9\uae09\uc0dd\ub4e4\uc5d0\uac8c \uc5ec\ud559\uc0dd \uc131\ud3ed\ud589 \uc0ac\uc2e4\uc744 \uc790\ub791\ud558\uace0 \ub2e4\ub140 \uc870\uc0ac\ud588\ub354\ub2c8 \uc2dc\uc778\ud588\ub2e4\"\uba70 \"\ud1f4\ud559\uc744 \uc2dc\ud0a4\ub824 \ud588\uc73c\ub098 \uad50\uc7a5\uc758 \ubc18\ub300\ub85c \uacbd\ubbf8\ud55c \uc9d5\uacc4\uc5d0 \uadf8\ucce4\ub2e4\"\uace0 \ud68c\uc0c1\ud588\ub2e4. \"\uc774\uc601\ud559\uc740 \ubb34\ub2e8\uacb0\uc11d \uc77c\uc218\uac00 \uc218\uc5c5\uc77c\uc218 1/3\uc744 \ucd08\uacfc\ud574 \uc878\uc5c5\uc774 \ubd88\uac00\ud55c \uc0c1\ud669\uc774\uc5c8\ub294\ub370 \uad50\uc7a5\uc774 \uc9c1\uad8c\uc73c\ub85c \uc0c1\uc704\ubc95\uc744 \uc704\ubc18\ud558\uba74\uc11c \uc740\ud3d0\ud574 \uc878\uc5c5\uc2dc\ucf30\ub2e4\"\uba70 \"\ub2f9\uc2dc \uc774\uc601\ud559\uc758 \ubd80\uce5c\uc774 \uc758\uc815\ubd80\uc758 \uc7ac\ub825\uac00\uc600\ub294\ub370 \uc5b4\uba38\ub2c8\uc758 \uce58\ub9db\ubc14\ub78c\uc774 \uc14c\ub2e4\"\uace0 \ub367\ubd99\uc600\ub2e4. \uc774\uc601\ud559\uc758 \uc911\ud559\uad50 \ub3d9\ucc3d\ub4e4\uc740 \"\uc774\uc601\ud559\uc774 \ub610\ub798 \uce5c\uad6c 3\uba85\uacfc \ud55c \uc5ec\uc131\uc744 \uc131\ud3ed\ud589\ud55c \ubaa8\uc2b5\uc744 \uc9c1\uc811 \ubaa9\uaca9\ud588\ub2e4\"\uba70 \ucd08\ub4f1\ud559\uc0dd\uc5d0\uac8c \uacfc\uc790\ub97c \uc8fc\uba70 \ub370\ub824\uac00 \uc131\ud3ed\ud589\ud588\ub2e4\uace0\ub3c4 \ub9d0\ud588\ub2e4. \uc774\uc601\ud559\uc758 \uc5b4\uba38\ub2c8 \uae40\ubaa8\uc528\ub294 \uc774\uc601\ud559\uc758 \uc544\ubc84\uc9c0\uac00 \uc0ac\uc5c5\uc5d0 \uc2e4\ud328\ud558\uba74\uc11c \uc774\ud63c\ud588\ub2e4."} {"question": "2001\ub144 3\uc6d4, \uc2a4\ud06c\ub958\uc5b4\ud0dd\uacfc \ud568\uaed8 \uc568\ubc94\uc744 \ubc1c\ub9e4 \ud55c \ube44\uc2b7\ud55c \ub958\uc758 \ud391\ud06c\ubc34\ub4dc\uc758 \uc774\ub984\uc740?", "paragraph": "99\ub144\ub9d0 \ubc15\ud0dc\uc2dd \ud0c8\ud1f4\ud6c4 \"\ub4e4\ub72c\ub9c8\uc74c\uac00\ub77c\uc549\ud788\uace0\"(\ud604 popstore)\uc758 \ub4dc\ub7ec\uba38 \uc804\ubbfc\ud655 \uacfc 2000\ub144\ucd08 \uae30\ud0c0 \uc774\uad11\uc7ac(ex - \uc1e0\ud30c\uc774\ud504, \ud604 Hollow Jan)\ub97c\uc601\uc785 4\uc778\uc870 \ub77c\uc778\uc5c5\uc73c\ub85c \uad50\uccb4 \uc11c\uc6b8,\ub300\uad6c \ub4f1 \uc5ec\ub7ec\uc9c0\ubc29 \ud22c\uc5b4\ud558\uba70 \uc218\ub9ce\uc740 \ub77c\uc774\ube0c\ub97c \ub77c\uc774\ube0c\ub97c \uac00\uc9d0. \uadf8\ub9ac\uace0 2001\ub144 3\uc6d4 \ube44\uc2b7\ud55c \ub958\uc758 \ud391\ud06c\ubc34\ub4dc Propeller 21\uacfc \uadf8\ub4e4\uc758 \uccab \uc2f1\uae00 \uc774\uc790 6\uace1\uc9dc\ub9ac \uc2a4\ud50c\ub9bf \uc568\ubc94 Skrew Attack/Propeller21 - In store \ub97c \uc790\uc2e0\uc758 \ub808\uc774\ube14 Beach And Bitch Records \uc5d0\uc11c \ubc1c\ub9e4. \uadf8\ud6c4\ub85c \uc870\ucc9c\ud638 \uc804\ubbfc\ud655 \uad70\ub300\uc5d0 \ub04c\ub824\uac10. \ub4dc\ub7ec\uba38 \uc870\uc0c1\ud604[ex)Propeller21\ud604,RUX]\uc744 \uc601\uc785 2001\ub144\ub9d0 \uc774\uad11\uc7ac \ud0c8\ud1f4.2002\ub144\ucd08 \uc774\uad11\uc7ac \uc7ac\uc601\uc785(2002\ub144\uc911\uc21c \uc774\uad11\uc7ac\ub294 \uacb0\uad6d \ud0c8\ud1f4)", "answer": "Propeller 21", "sentence": "\uadf8\ub9ac\uace0 2001\ub144 3\uc6d4 \ube44\uc2b7\ud55c \ub958\uc758 \ud391\ud06c\ubc34\ub4dc Propeller 21 \uacfc \uadf8\ub4e4\uc758 \uccab \uc2f1\uae00 \uc774\uc790 6\uace1\uc9dc\ub9ac \uc2a4\ud50c\ub9bf \uc568\ubc94 Skrew Attack/Propeller21 - In store \ub97c \uc790\uc2e0\uc758 \ub808\uc774\ube14 Beach And Bitch Records \uc5d0\uc11c \ubc1c\ub9e4.", "paragraph_sentence": "99\ub144\ub9d0 \ubc15\ud0dc\uc2dd \ud0c8\ud1f4\ud6c4 \"\ub4e4\ub72c\ub9c8\uc74c\uac00\ub77c\uc549\ud788\uace0\"(\ud604 popstore)\uc758 \ub4dc\ub7ec\uba38 \uc804\ubbfc\ud655 \uacfc 2000\ub144\ucd08 \uae30\ud0c0 \uc774\uad11\uc7ac(ex - \uc1e0\ud30c\uc774\ud504, \ud604 Hollow Jan)\ub97c\uc601\uc785 4\uc778\uc870 \ub77c\uc778\uc5c5\uc73c\ub85c \uad50\uccb4 \uc11c\uc6b8,\ub300\uad6c \ub4f1 \uc5ec\ub7ec\uc9c0\ubc29 \ud22c\uc5b4\ud558\uba70 \uc218\ub9ce\uc740 \ub77c\uc774\ube0c\ub97c \ub77c\uc774\ube0c\ub97c \uac00\uc9d0. \uadf8\ub9ac\uace0 2001\ub144 3\uc6d4 \ube44\uc2b7\ud55c \ub958\uc758 \ud391\ud06c\ubc34\ub4dc Propeller 21 \uacfc \uadf8\ub4e4\uc758 \uccab \uc2f1\uae00 \uc774\uc790 6\uace1\uc9dc\ub9ac \uc2a4\ud50c\ub9bf \uc568\ubc94 Skrew Attack/Propeller21 - In store \ub97c \uc790\uc2e0\uc758 \ub808\uc774\ube14 Beach And Bitch Records \uc5d0\uc11c \ubc1c\ub9e4. \uadf8\ud6c4\ub85c \uc870\ucc9c\ud638 \uc804\ubbfc\ud655 \uad70\ub300\uc5d0 \ub04c\ub824\uac10. \ub4dc\ub7ec\uba38 \uc870\uc0c1\ud604[ex)Propeller21\ud604,RUX]\uc744 \uc601\uc785 2001\ub144\ub9d0 \uc774\uad11\uc7ac \ud0c8\ud1f4.2002\ub144\ucd08 \uc774\uad11\uc7ac \uc7ac\uc601\uc785(2002\ub144\uc911\uc21c \uc774\uad11\uc7ac\ub294 \uacb0\uad6d \ud0c8\ud1f4)", "paragraph_answer": "99\ub144\ub9d0 \ubc15\ud0dc\uc2dd \ud0c8\ud1f4\ud6c4 \"\ub4e4\ub72c\ub9c8\uc74c\uac00\ub77c\uc549\ud788\uace0\"(\ud604 popstore)\uc758 \ub4dc\ub7ec\uba38 \uc804\ubbfc\ud655 \uacfc 2000\ub144\ucd08 \uae30\ud0c0 \uc774\uad11\uc7ac(ex - \uc1e0\ud30c\uc774\ud504, \ud604 Hollow Jan)\ub97c\uc601\uc785 4\uc778\uc870 \ub77c\uc778\uc5c5\uc73c\ub85c \uad50\uccb4 \uc11c\uc6b8,\ub300\uad6c \ub4f1 \uc5ec\ub7ec\uc9c0\ubc29 \ud22c\uc5b4\ud558\uba70 \uc218\ub9ce\uc740 \ub77c\uc774\ube0c\ub97c \ub77c\uc774\ube0c\ub97c \uac00\uc9d0. \uadf8\ub9ac\uace0 2001\ub144 3\uc6d4 \ube44\uc2b7\ud55c \ub958\uc758 \ud391\ud06c\ubc34\ub4dc Propeller 21 \uacfc \uadf8\ub4e4\uc758 \uccab \uc2f1\uae00 \uc774\uc790 6\uace1\uc9dc\ub9ac \uc2a4\ud50c\ub9bf \uc568\ubc94 Skrew Attack/Propeller21 - In store \ub97c \uc790\uc2e0\uc758 \ub808\uc774\ube14 Beach And Bitch Records \uc5d0\uc11c \ubc1c\ub9e4. \uadf8\ud6c4\ub85c \uc870\ucc9c\ud638 \uc804\ubbfc\ud655 \uad70\ub300\uc5d0 \ub04c\ub824\uac10. \ub4dc\ub7ec\uba38 \uc870\uc0c1\ud604[ex)Propeller21\ud604,RUX]\uc744 \uc601\uc785 2001\ub144\ub9d0 \uc774\uad11\uc7ac \ud0c8\ud1f4.2002\ub144\ucd08 \uc774\uad11\uc7ac \uc7ac\uc601\uc785(2002\ub144\uc911\uc21c \uc774\uad11\uc7ac\ub294 \uacb0\uad6d \ud0c8\ud1f4)", "sentence_answer": "\uadf8\ub9ac\uace0 2001\ub144 3\uc6d4 \ube44\uc2b7\ud55c \ub958\uc758 \ud391\ud06c\ubc34\ub4dc Propeller 21 \uacfc \uadf8\ub4e4\uc758 \uccab \uc2f1\uae00 \uc774\uc790 6\uace1\uc9dc\ub9ac \uc2a4\ud50c\ub9bf \uc568\ubc94 Skrew Attack/Propeller21 - In store \ub97c \uc790\uc2e0\uc758 \ub808\uc774\ube14 Beach And Bitch Records \uc5d0\uc11c \ubc1c\ub9e4.", "paragraph_id": "6457289-0-1", "paragraph_question": "question: 2001\ub144 3\uc6d4, \uc2a4\ud06c\ub958\uc5b4\ud0dd\uacfc \ud568\uaed8 \uc568\ubc94\uc744 \ubc1c\ub9e4 \ud55c \ube44\uc2b7\ud55c \ub958\uc758 \ud391\ud06c\ubc34\ub4dc\uc758 \uc774\ub984\uc740?, context: 99\ub144\ub9d0 \ubc15\ud0dc\uc2dd \ud0c8\ud1f4\ud6c4 \"\ub4e4\ub72c\ub9c8\uc74c\uac00\ub77c\uc549\ud788\uace0\"(\ud604 popstore)\uc758 \ub4dc\ub7ec\uba38 \uc804\ubbfc\ud655 \uacfc 2000\ub144\ucd08 \uae30\ud0c0 \uc774\uad11\uc7ac(ex - \uc1e0\ud30c\uc774\ud504, \ud604 Hollow Jan)\ub97c\uc601\uc785 4\uc778\uc870 \ub77c\uc778\uc5c5\uc73c\ub85c \uad50\uccb4 \uc11c\uc6b8,\ub300\uad6c \ub4f1 \uc5ec\ub7ec\uc9c0\ubc29 \ud22c\uc5b4\ud558\uba70 \uc218\ub9ce\uc740 \ub77c\uc774\ube0c\ub97c \ub77c\uc774\ube0c\ub97c \uac00\uc9d0. \uadf8\ub9ac\uace0 2001\ub144 3\uc6d4 \ube44\uc2b7\ud55c \ub958\uc758 \ud391\ud06c\ubc34\ub4dc Propeller 21\uacfc \uadf8\ub4e4\uc758 \uccab \uc2f1\uae00 \uc774\uc790 6\uace1\uc9dc\ub9ac \uc2a4\ud50c\ub9bf \uc568\ubc94 Skrew Attack/Propeller21 - In store \ub97c \uc790\uc2e0\uc758 \ub808\uc774\ube14 Beach And Bitch Records \uc5d0\uc11c \ubc1c\ub9e4. \uadf8\ud6c4\ub85c \uc870\ucc9c\ud638 \uc804\ubbfc\ud655 \uad70\ub300\uc5d0 \ub04c\ub824\uac10. \ub4dc\ub7ec\uba38 \uc870\uc0c1\ud604[ex)Propeller21\ud604,RUX]\uc744 \uc601\uc785 2001\ub144\ub9d0 \uc774\uad11\uc7ac \ud0c8\ud1f4.2002\ub144\ucd08 \uc774\uad11\uc7ac \uc7ac\uc601\uc785(2002\ub144\uc911\uc21c \uc774\uad11\uc7ac\ub294 \uacb0\uad6d \ud0c8\ud1f4)"} {"question": "\uc548\ub9c8\ub2f9\uc5d0 \uc218\uc591\ubc84\ub4e4\uc744 \uc2ec\uc5c8\ub358 \uad6d\uac00\ub294?", "paragraph": "\uc911\uad6d\uc5d0\uc11c\ub294 \uc218\uc591\ubc84\ub4e4\uc744 \uc548\ub9c8\ub2f9\uc5d0\ub3c4 \uc2ec\ub294\ub370, \uc774\ub294 \ud55c\uad6d\uc5d0\uc11c \ub9c8\ub2f9\uc5d0\ub294 \uc218\uc591\ubc84\ub4e4\uc744 \uc2ec\uc9c0 \uc54a\ub294 \uac83\uacfc \ub300\uc870\ub41c\ub2e4. \ub3d9\uc9c4\uc758 \uc2dc\uc778\uc778 \ub3c4\uc5f0\uba85\uc740 \uad00\uc9c1\uc744 \ubc84\ub9ac\uace0 \uadc0\ud5a5\ud574 \uc9d1 \uc55e\uc5d0 \uc218\uc591\ubc84\ub4e4 \ub2e4\uc12f \uadf8\ub8e8\ub97c \uc2ec\uc740 \ub4a4, \uc2a4\uc2a4\ub85c\ub97c \uc774\ub7ec\ud55c \ub2e4\uc12f \uc218\uc758 \uc218\uc591\ubc84\ub4e4\uc744 \ub73b\ud558\ub294\u3008\uc624\ub958(\u4e94\u67f3) \uc120\uc0dd\u3009\uc774\ub77c \uce6d\ud588\ub2e4. \uadf8\ub9ac\uace0 \uc911\uad6d\uc758 \uc804\uc871 \ubb38\ud654\uc5d0\uc11c\ub294 \uccad\ub098\ub77c \ub54c \ubc29\ud604(\u65b9\u7d62)\uc774\ub77c\ub294 \uc0ac\ub78c\uc774\u300a\ud488\uc870\u300b(\u54c1\u85fb)\ub77c\ub294 \ucc45\uc744 \ud1b5\ud574 \uc804\uc871\uc758 \ud615\uc2dd\uc744 \uc5f4\uc5ec\ub35f \uac00\uc9c0\ub85c \ub098\ub204\uace0 \uc774\ub97c \ub2e4\uc2dc 9\ud488\uc73c\ub85c \uc138\uc138\ud788 \ub098\ub204\uc5c8\ub294\ub370, \uc774\ub7ec\ud55c 9\ud488 \uc911 '\ubb18\ud488\uc0c1\uc911'(\u5999\u54c1\u4e0a\u4e2d)\uc774\ub77c\ub294 \ud488\uacc4\ub294 \"\ub098\uc57d\ud558\uace0 \uac00\ub298\uae30\uac00 \uadf8\uc9c0\uc5c6\ub2e4. \ubc14\ub78c\uc5d0 \uae30\ub304 \uc218\uc591\ubc84\ub4e4\ucc98\ub7fc \uac00\ub0d8\ud37c \ub204\uad70\uac00\uc758 \ubd80\ucd95\uc774 \ud544\uc694\ud558\ub2e4.\u2026\"\ub77c\ub294 \ub73b\uc73c\ub85c, \uc804\uc871\uc758 \ubaa8\uc591\uc744 \uc218\uc591\ubc84\ub4e4\uc5d0 \ube57\ub300 \ubb18\uc0ac\ud558\uc600\ub2e4. \uc911\uad6d\uc5d0\uc11c\ub294 \uc61b\ubd80\ud130\u3008\u6298\u67f3\u76f8\u9001\u3009\ub77c\ub294 \ub9d0\uc774 \uc4f0\uc774\ub294\ub370, \ub0a8\ubd81\uc870 \uc2dc\ub300\uc5d0 \uc5ec\ud589\uc744 \ub5a0\ub098\ub294 \uc0ac\ub78c\uc5d0\uac8c \uc218\uc591\ub098\ubb34\uc758 \uac00\uc9c0\ub97c \uad6c\ubd80\ub824 \ubb36\uc5b4 \uac74\ub124\ub294 \ud48d\uc2b5\uc5d0\uc11c \uc720\ub798\ud588\ub2e4. \uc774\ub294 \ubc84\ub4dc\ub098\ubb34\ub97c \ub73b\ud558\ub294 \uc911\uad6d\uc5b4\uc778 '\u67f3'\uc758 \ubc1c\uc74c([ li\u01d4 ])\uacfc \uba38\ubb34\ub974\ub2e4, \uc8fc\uc758\ud558\ub2e4\ub97c \ub73b\ud558\ub294 \uc911\uad6d\uc5b4\uc778 '\u7559'\uc758 \ubc1c\uc74c([ li\u00fa ])\uc774 \uc720\uc0ac\ud558\uae30 \ub54c\ubb38\uc774\ub2e4. \uc774\uc640 \uac19\uc774 \uc218\uc591\ubc84\ub4e4\uc740 \uc911\uad6d\uc5d0\uc11c\ub294 \ub5a0\ub098\ub294 \uc0ac\ub78c\uacfc \uad00\uacc4\ub418\ub294 '\uc774\ubcc4'\uc758 \uc758\ubbf8\uc640 \uc5f0\uad00\uc774 \uc788\uc73c\uba70, \uc911\uad6d \ubb38\ud559 \uc18d\uc5d0\uc11c\ub3c4 \uc774\ubcc4\uc744 \uc758\ubbf8\ud558\ub294 \ub370\uc5d0 \ub9ce\uc774 \uc4f0\uc600\ub2e4. \ub610\ud55c \uc911\uad6d \ubb38\ud559\uc5d0\uc11c\ub294 \ubd04\uc774 \uc654\uc74c\uc744 \uc0c1\uc9d5\ud558\ub294 \uc774\ubbf8\uc9c0\ub85c \u2018\uc218\uc591\ubc84\ub4e4\uc758 \ubd80\ub4dc\ub7ec\uc6b4 \uac00\uc9c0\u2019\ub97c \ub4dc\ub294 \uacbd\uc6b0\ub3c4 \ub9ce\uc558\ub2e4.", "answer": "\uc911\uad6d", "sentence": "\uc911\uad6d \uc5d0\uc11c\ub294 \uc218\uc591\ubc84\ub4e4\uc744 \uc548\ub9c8\ub2f9\uc5d0\ub3c4 \uc2ec\ub294\ub370, \uc774\ub294 \ud55c\uad6d\uc5d0\uc11c \ub9c8\ub2f9\uc5d0\ub294 \uc218\uc591\ubc84\ub4e4\uc744 \uc2ec\uc9c0 \uc54a\ub294 \uac83\uacfc \ub300\uc870\ub41c\ub2e4.", "paragraph_sentence": " \uc911\uad6d \uc5d0\uc11c\ub294 \uc218\uc591\ubc84\ub4e4\uc744 \uc548\ub9c8\ub2f9\uc5d0\ub3c4 \uc2ec\ub294\ub370, \uc774\ub294 \ud55c\uad6d\uc5d0\uc11c \ub9c8\ub2f9\uc5d0\ub294 \uc218\uc591\ubc84\ub4e4\uc744 \uc2ec\uc9c0 \uc54a\ub294 \uac83\uacfc \ub300\uc870\ub41c\ub2e4. \ub3d9\uc9c4\uc758 \uc2dc\uc778\uc778 \ub3c4\uc5f0\uba85\uc740 \uad00\uc9c1\uc744 \ubc84\ub9ac\uace0 \uadc0\ud5a5\ud574 \uc9d1 \uc55e\uc5d0 \uc218\uc591\ubc84\ub4e4 \ub2e4\uc12f \uadf8\ub8e8\ub97c \uc2ec\uc740 \ub4a4, \uc2a4\uc2a4\ub85c\ub97c \uc774\ub7ec\ud55c \ub2e4\uc12f \uc218\uc758 \uc218\uc591\ubc84\ub4e4\uc744 \ub73b\ud558\ub294\u3008\uc624\ub958(\u4e94\u67f3) \uc120\uc0dd\u3009\uc774\ub77c \uce6d\ud588\ub2e4. \uadf8\ub9ac\uace0 \uc911\uad6d\uc758 \uc804\uc871 \ubb38\ud654\uc5d0\uc11c\ub294 \uccad\ub098\ub77c \ub54c \ubc29\ud604(\u65b9\u7d62)\uc774\ub77c\ub294 \uc0ac\ub78c\uc774\u300a\ud488\uc870\u300b(\u54c1\u85fb)\ub77c\ub294 \ucc45\uc744 \ud1b5\ud574 \uc804\uc871\uc758 \ud615\uc2dd\uc744 \uc5f4\uc5ec\ub35f \uac00\uc9c0\ub85c \ub098\ub204\uace0 \uc774\ub97c \ub2e4\uc2dc 9\ud488\uc73c\ub85c \uc138\uc138\ud788 \ub098\ub204\uc5c8\ub294\ub370, \uc774\ub7ec\ud55c 9\ud488 \uc911 '\ubb18\ud488\uc0c1\uc911'(\u5999\u54c1\u4e0a\u4e2d)\uc774\ub77c\ub294 \ud488\uacc4\ub294 \"\ub098\uc57d\ud558\uace0 \uac00\ub298\uae30\uac00 \uadf8\uc9c0\uc5c6\ub2e4. \ubc14\ub78c\uc5d0 \uae30\ub304 \uc218\uc591\ubc84\ub4e4\ucc98\ub7fc \uac00\ub0d8\ud37c \ub204\uad70\uac00\uc758 \ubd80\ucd95\uc774 \ud544\uc694\ud558\ub2e4.\u2026\"\ub77c\ub294 \ub73b\uc73c\ub85c, \uc804\uc871\uc758 \ubaa8\uc591\uc744 \uc218\uc591\ubc84\ub4e4\uc5d0 \ube57\ub300 \ubb18\uc0ac\ud558\uc600\ub2e4. \uc911\uad6d\uc5d0\uc11c\ub294 \uc61b\ubd80\ud130\u3008\u6298\u67f3\u76f8\u9001\u3009\ub77c\ub294 \ub9d0\uc774 \uc4f0\uc774\ub294\ub370, \ub0a8\ubd81\uc870 \uc2dc\ub300\uc5d0 \uc5ec\ud589\uc744 \ub5a0\ub098\ub294 \uc0ac\ub78c\uc5d0\uac8c \uc218\uc591\ub098\ubb34\uc758 \uac00\uc9c0\ub97c \uad6c\ubd80\ub824 \ubb36\uc5b4 \uac74\ub124\ub294 \ud48d\uc2b5\uc5d0\uc11c \uc720\ub798\ud588\ub2e4. \uc774\ub294 \ubc84\ub4dc\ub098\ubb34\ub97c \ub73b\ud558\ub294 \uc911\uad6d\uc5b4\uc778 '\u67f3'\uc758 \ubc1c\uc74c([ li\u01d4 ])\uacfc \uba38\ubb34\ub974\ub2e4, \uc8fc\uc758\ud558\ub2e4\ub97c \ub73b\ud558\ub294 \uc911\uad6d\uc5b4\uc778 '\u7559'\uc758 \ubc1c\uc74c([ li\u00fa ])\uc774 \uc720\uc0ac\ud558\uae30 \ub54c\ubb38\uc774\ub2e4. \uc774\uc640 \uac19\uc774 \uc218\uc591\ubc84\ub4e4\uc740 \uc911\uad6d\uc5d0\uc11c\ub294 \ub5a0\ub098\ub294 \uc0ac\ub78c\uacfc \uad00\uacc4\ub418\ub294 '\uc774\ubcc4'\uc758 \uc758\ubbf8\uc640 \uc5f0\uad00\uc774 \uc788\uc73c\uba70, \uc911\uad6d \ubb38\ud559 \uc18d\uc5d0\uc11c\ub3c4 \uc774\ubcc4\uc744 \uc758\ubbf8\ud558\ub294 \ub370\uc5d0 \ub9ce\uc774 \uc4f0\uc600\ub2e4. \ub610\ud55c \uc911\uad6d \ubb38\ud559\uc5d0\uc11c\ub294 \ubd04\uc774 \uc654\uc74c\uc744 \uc0c1\uc9d5\ud558\ub294 \uc774\ubbf8\uc9c0\ub85c \u2018\uc218\uc591\ubc84\ub4e4\uc758 \ubd80\ub4dc\ub7ec\uc6b4 \uac00\uc9c0\u2019\ub97c \ub4dc\ub294 \uacbd\uc6b0\ub3c4 \ub9ce\uc558\ub2e4.", "paragraph_answer": " \uc911\uad6d \uc5d0\uc11c\ub294 \uc218\uc591\ubc84\ub4e4\uc744 \uc548\ub9c8\ub2f9\uc5d0\ub3c4 \uc2ec\ub294\ub370, \uc774\ub294 \ud55c\uad6d\uc5d0\uc11c \ub9c8\ub2f9\uc5d0\ub294 \uc218\uc591\ubc84\ub4e4\uc744 \uc2ec\uc9c0 \uc54a\ub294 \uac83\uacfc \ub300\uc870\ub41c\ub2e4. \ub3d9\uc9c4\uc758 \uc2dc\uc778\uc778 \ub3c4\uc5f0\uba85\uc740 \uad00\uc9c1\uc744 \ubc84\ub9ac\uace0 \uadc0\ud5a5\ud574 \uc9d1 \uc55e\uc5d0 \uc218\uc591\ubc84\ub4e4 \ub2e4\uc12f \uadf8\ub8e8\ub97c \uc2ec\uc740 \ub4a4, \uc2a4\uc2a4\ub85c\ub97c \uc774\ub7ec\ud55c \ub2e4\uc12f \uc218\uc758 \uc218\uc591\ubc84\ub4e4\uc744 \ub73b\ud558\ub294\u3008\uc624\ub958(\u4e94\u67f3) \uc120\uc0dd\u3009\uc774\ub77c \uce6d\ud588\ub2e4. \uadf8\ub9ac\uace0 \uc911\uad6d\uc758 \uc804\uc871 \ubb38\ud654\uc5d0\uc11c\ub294 \uccad\ub098\ub77c \ub54c \ubc29\ud604(\u65b9\u7d62)\uc774\ub77c\ub294 \uc0ac\ub78c\uc774\u300a\ud488\uc870\u300b(\u54c1\u85fb)\ub77c\ub294 \ucc45\uc744 \ud1b5\ud574 \uc804\uc871\uc758 \ud615\uc2dd\uc744 \uc5f4\uc5ec\ub35f \uac00\uc9c0\ub85c \ub098\ub204\uace0 \uc774\ub97c \ub2e4\uc2dc 9\ud488\uc73c\ub85c \uc138\uc138\ud788 \ub098\ub204\uc5c8\ub294\ub370, \uc774\ub7ec\ud55c 9\ud488 \uc911 '\ubb18\ud488\uc0c1\uc911'(\u5999\u54c1\u4e0a\u4e2d)\uc774\ub77c\ub294 \ud488\uacc4\ub294 \"\ub098\uc57d\ud558\uace0 \uac00\ub298\uae30\uac00 \uadf8\uc9c0\uc5c6\ub2e4. \ubc14\ub78c\uc5d0 \uae30\ub304 \uc218\uc591\ubc84\ub4e4\ucc98\ub7fc \uac00\ub0d8\ud37c \ub204\uad70\uac00\uc758 \ubd80\ucd95\uc774 \ud544\uc694\ud558\ub2e4.\u2026\"\ub77c\ub294 \ub73b\uc73c\ub85c, \uc804\uc871\uc758 \ubaa8\uc591\uc744 \uc218\uc591\ubc84\ub4e4\uc5d0 \ube57\ub300 \ubb18\uc0ac\ud558\uc600\ub2e4. \uc911\uad6d\uc5d0\uc11c\ub294 \uc61b\ubd80\ud130\u3008\u6298\u67f3\u76f8\u9001\u3009\ub77c\ub294 \ub9d0\uc774 \uc4f0\uc774\ub294\ub370, \ub0a8\ubd81\uc870 \uc2dc\ub300\uc5d0 \uc5ec\ud589\uc744 \ub5a0\ub098\ub294 \uc0ac\ub78c\uc5d0\uac8c \uc218\uc591\ub098\ubb34\uc758 \uac00\uc9c0\ub97c \uad6c\ubd80\ub824 \ubb36\uc5b4 \uac74\ub124\ub294 \ud48d\uc2b5\uc5d0\uc11c \uc720\ub798\ud588\ub2e4. \uc774\ub294 \ubc84\ub4dc\ub098\ubb34\ub97c \ub73b\ud558\ub294 \uc911\uad6d\uc5b4\uc778 '\u67f3'\uc758 \ubc1c\uc74c([ li\u01d4 ])\uacfc \uba38\ubb34\ub974\ub2e4, \uc8fc\uc758\ud558\ub2e4\ub97c \ub73b\ud558\ub294 \uc911\uad6d\uc5b4\uc778 '\u7559'\uc758 \ubc1c\uc74c([ li\u00fa ])\uc774 \uc720\uc0ac\ud558\uae30 \ub54c\ubb38\uc774\ub2e4. \uc774\uc640 \uac19\uc774 \uc218\uc591\ubc84\ub4e4\uc740 \uc911\uad6d\uc5d0\uc11c\ub294 \ub5a0\ub098\ub294 \uc0ac\ub78c\uacfc \uad00\uacc4\ub418\ub294 '\uc774\ubcc4'\uc758 \uc758\ubbf8\uc640 \uc5f0\uad00\uc774 \uc788\uc73c\uba70, \uc911\uad6d \ubb38\ud559 \uc18d\uc5d0\uc11c\ub3c4 \uc774\ubcc4\uc744 \uc758\ubbf8\ud558\ub294 \ub370\uc5d0 \ub9ce\uc774 \uc4f0\uc600\ub2e4. \ub610\ud55c \uc911\uad6d \ubb38\ud559\uc5d0\uc11c\ub294 \ubd04\uc774 \uc654\uc74c\uc744 \uc0c1\uc9d5\ud558\ub294 \uc774\ubbf8\uc9c0\ub85c \u2018\uc218\uc591\ubc84\ub4e4\uc758 \ubd80\ub4dc\ub7ec\uc6b4 \uac00\uc9c0\u2019\ub97c \ub4dc\ub294 \uacbd\uc6b0\ub3c4 \ub9ce\uc558\ub2e4.", "sentence_answer": " \uc911\uad6d \uc5d0\uc11c\ub294 \uc218\uc591\ubc84\ub4e4\uc744 \uc548\ub9c8\ub2f9\uc5d0\ub3c4 \uc2ec\ub294\ub370, \uc774\ub294 \ud55c\uad6d\uc5d0\uc11c \ub9c8\ub2f9\uc5d0\ub294 \uc218\uc591\ubc84\ub4e4\uc744 \uc2ec\uc9c0 \uc54a\ub294 \uac83\uacfc \ub300\uc870\ub41c\ub2e4.", "paragraph_id": "6518768-6-0", "paragraph_question": "question: \uc548\ub9c8\ub2f9\uc5d0 \uc218\uc591\ubc84\ub4e4\uc744 \uc2ec\uc5c8\ub358 \uad6d\uac00\ub294?, context: \uc911\uad6d\uc5d0\uc11c\ub294 \uc218\uc591\ubc84\ub4e4\uc744 \uc548\ub9c8\ub2f9\uc5d0\ub3c4 \uc2ec\ub294\ub370, \uc774\ub294 \ud55c\uad6d\uc5d0\uc11c \ub9c8\ub2f9\uc5d0\ub294 \uc218\uc591\ubc84\ub4e4\uc744 \uc2ec\uc9c0 \uc54a\ub294 \uac83\uacfc \ub300\uc870\ub41c\ub2e4. \ub3d9\uc9c4\uc758 \uc2dc\uc778\uc778 \ub3c4\uc5f0\uba85\uc740 \uad00\uc9c1\uc744 \ubc84\ub9ac\uace0 \uadc0\ud5a5\ud574 \uc9d1 \uc55e\uc5d0 \uc218\uc591\ubc84\ub4e4 \ub2e4\uc12f \uadf8\ub8e8\ub97c \uc2ec\uc740 \ub4a4, \uc2a4\uc2a4\ub85c\ub97c \uc774\ub7ec\ud55c \ub2e4\uc12f \uc218\uc758 \uc218\uc591\ubc84\ub4e4\uc744 \ub73b\ud558\ub294\u3008\uc624\ub958(\u4e94\u67f3) \uc120\uc0dd\u3009\uc774\ub77c \uce6d\ud588\ub2e4. \uadf8\ub9ac\uace0 \uc911\uad6d\uc758 \uc804\uc871 \ubb38\ud654\uc5d0\uc11c\ub294 \uccad\ub098\ub77c \ub54c \ubc29\ud604(\u65b9\u7d62)\uc774\ub77c\ub294 \uc0ac\ub78c\uc774\u300a\ud488\uc870\u300b(\u54c1\u85fb)\ub77c\ub294 \ucc45\uc744 \ud1b5\ud574 \uc804\uc871\uc758 \ud615\uc2dd\uc744 \uc5f4\uc5ec\ub35f \uac00\uc9c0\ub85c \ub098\ub204\uace0 \uc774\ub97c \ub2e4\uc2dc 9\ud488\uc73c\ub85c \uc138\uc138\ud788 \ub098\ub204\uc5c8\ub294\ub370, \uc774\ub7ec\ud55c 9\ud488 \uc911 '\ubb18\ud488\uc0c1\uc911'(\u5999\u54c1\u4e0a\u4e2d)\uc774\ub77c\ub294 \ud488\uacc4\ub294 \"\ub098\uc57d\ud558\uace0 \uac00\ub298\uae30\uac00 \uadf8\uc9c0\uc5c6\ub2e4. \ubc14\ub78c\uc5d0 \uae30\ub304 \uc218\uc591\ubc84\ub4e4\ucc98\ub7fc \uac00\ub0d8\ud37c \ub204\uad70\uac00\uc758 \ubd80\ucd95\uc774 \ud544\uc694\ud558\ub2e4.\u2026\"\ub77c\ub294 \ub73b\uc73c\ub85c, \uc804\uc871\uc758 \ubaa8\uc591\uc744 \uc218\uc591\ubc84\ub4e4\uc5d0 \ube57\ub300 \ubb18\uc0ac\ud558\uc600\ub2e4. \uc911\uad6d\uc5d0\uc11c\ub294 \uc61b\ubd80\ud130\u3008\u6298\u67f3\u76f8\u9001\u3009\ub77c\ub294 \ub9d0\uc774 \uc4f0\uc774\ub294\ub370, \ub0a8\ubd81\uc870 \uc2dc\ub300\uc5d0 \uc5ec\ud589\uc744 \ub5a0\ub098\ub294 \uc0ac\ub78c\uc5d0\uac8c \uc218\uc591\ub098\ubb34\uc758 \uac00\uc9c0\ub97c \uad6c\ubd80\ub824 \ubb36\uc5b4 \uac74\ub124\ub294 \ud48d\uc2b5\uc5d0\uc11c \uc720\ub798\ud588\ub2e4. \uc774\ub294 \ubc84\ub4dc\ub098\ubb34\ub97c \ub73b\ud558\ub294 \uc911\uad6d\uc5b4\uc778 '\u67f3'\uc758 \ubc1c\uc74c([ li\u01d4 ])\uacfc \uba38\ubb34\ub974\ub2e4, \uc8fc\uc758\ud558\ub2e4\ub97c \ub73b\ud558\ub294 \uc911\uad6d\uc5b4\uc778 '\u7559'\uc758 \ubc1c\uc74c([ li\u00fa ])\uc774 \uc720\uc0ac\ud558\uae30 \ub54c\ubb38\uc774\ub2e4. \uc774\uc640 \uac19\uc774 \uc218\uc591\ubc84\ub4e4\uc740 \uc911\uad6d\uc5d0\uc11c\ub294 \ub5a0\ub098\ub294 \uc0ac\ub78c\uacfc \uad00\uacc4\ub418\ub294 '\uc774\ubcc4'\uc758 \uc758\ubbf8\uc640 \uc5f0\uad00\uc774 \uc788\uc73c\uba70, \uc911\uad6d \ubb38\ud559 \uc18d\uc5d0\uc11c\ub3c4 \uc774\ubcc4\uc744 \uc758\ubbf8\ud558\ub294 \ub370\uc5d0 \ub9ce\uc774 \uc4f0\uc600\ub2e4. \ub610\ud55c \uc911\uad6d \ubb38\ud559\uc5d0\uc11c\ub294 \ubd04\uc774 \uc654\uc74c\uc744 \uc0c1\uc9d5\ud558\ub294 \uc774\ubbf8\uc9c0\ub85c \u2018\uc218\uc591\ubc84\ub4e4\uc758 \ubd80\ub4dc\ub7ec\uc6b4 \uac00\uc9c0\u2019\ub97c \ub4dc\ub294 \uacbd\uc6b0\ub3c4 \ub9ce\uc558\ub2e4."} {"question": "\uc544\uc6b0\uad6c\uc2a4\ud22c\uc2a4\ub294 \uc81c1\uc758 \uc2dc\ubbfc\uc774\ub77c\ub294 \ub73b\uc758 \ubb34\uc5c7\uc774\ub77c\uace0 \ubcf8\uc778\uc744 \ubd88\ub800\ub294\uac00?", "paragraph": "\ub0b4\uc804\uc5d0\uc11c \uc2b9\ub9ac\ud55c \uc625\ud0c0\ube44\uc544\ub204\uc2a4\ub294 \uc544\uc6b0\uad6c\uc2a4\ud22c\uc2a4\uc758 \uce6d\ud638\ub97c \ubc1b\uace0 \uac70\uc758 \uc808\ub300\uc801\uc778 \uad8c\ub825\uc744 \uc7a1\uc558\uc9c0\ub9cc, \uacf5\ud654\uc815\uc744 \uac00\uc7a5\ud588\ub2e4. \uc774\ub54c\uc758 \ud1b5\uce58\uccb4\uc81c\ub97c \uc6d0\uc218\uc815\uc774\ub77c \ud558\uba70(\uc0ac\uc2e4\uc0c1 \uc81c\uc815\uc774\ub2e4), \uc544\uc6b0\uad6c\uc2a4\ud22c\uc2a4\ub294 \uc790\uc2e0\uc744 \ud504\ub9b0\ucf11\uc2a4(\uc81c1\uc758 \uc2dc\ubbfc)\ub77c \uce6d\ud55c\ub2e4(\uc774\ub807\uac8c \uc544\uc6b0\uad6c\uc2a4\ud22c\uc2a4\uac00 \ub3c5\uc7ac\uad00\uc744 \uac70\ubd80\ud558\ub294 \ub4f1 \uc870\uc2ec\uc2a4\ub7fd\uac8c \ud589\ub3d9\ud55c \uac83\uc740 \uc591\uc544\ubc84\uc9c0\uc600\ub358 \uc728\ub9ac\uc6b0\uc2a4 \uce74\uc774\uc0ac\ub974\uc758 \uc554\uc0b4 \ub54c\ubb38\uc774\uae30\ub3c4 \ud558\ub2e4). \uc544\uc6b0\uad6c\uc2a4\ud22c\uc2a4\uc758 \ud6c4\uacc4\uc790 \ud2f0\ubca0\ub9ac\uc6b0\uc2a4\ub294 \uc2ec\uac01\ud55c \ubc18\ub300 \uc5c6\uc774 \uad8c\uc88c\uc5d0 \uc62c\ub77c \uc544\uc6b0\uad6c\uc2a4\ud22c\uc2a4\uc5d0 \uc774\uc5b4 \ubcf8\uaca9\uc801\uc778 \uc728\ub9ac\uc6b0\uc2a4-\ud074\ub77c\uc6b0\ub514\uc6b0\uc2a4 \uc655\uc870\ub97c \uc774\ub8e8\uc5c8\uace0, \uc774 \uc655\uc870\ub294 \ud2f0\ubca0\ub9ac\uc6b0\uc2a4 \uc0ac\ud6c4 \uce7c\ub9ac\uad74\ub77c\uc640 \ud074\ub77c\uc6b0\ub514\uc6b0\uc2a4\uc758 \uce58\uc138\ub97c \uac70\uccd0 \uae30\uc6d0\ud6c4 68\ub144 \ub124\ub85c\uac00 \uc790\uc0b4\ud560 \ub54c\uae4c\uc9c0 \uc774\uc5b4\uc9c4\ub2e4. \uc774\ud6c4 1\ub144\uac04 \ub85c\ub9c8\ub294 \ub0b4\uc804\uc0c1\ud0dc\uc5d0 \ub3cc\uc785\ud558\uba74\uc11c \uac08\ubc14, \uc624\ud1a0, \ube44\ud154\ub9ac\uc6b0\uc2a4\uac00 \uc989\uc704\ud55c\ub2e4. \ud6c4\uc5d0 \uc2dc\ub9ac\uc544 \uc8fc\ub454\uad70 \uc0ac\ub839\uad00\uc774\uc5c8\ub358 \ubca0\uc2a4\ud30c\uc2dc\uc544\ub204\uc2a4\uac00 \uc790\uae30 \uad70\ub2e8\uc758 \uc9c0\uc9c0\ub97c \ubc1b\uc544 \ub85c\ub9c8\uc5d0 \uc785\uc131\ud558\uc5ec \ube44\ud154\ub9ac\uc6b0\uc2a4\ub97c \uc0b4\ud574\ud558\uace0 \uc655\uc704\uc5d0 \uc624\ub974\uba74\uc11c \ud50c\ub77c\ube44\uc6b0\uc2a4 \uc655\uc870\uc758 \uc2dc\ub300\ub97c \uac1c\ucc3d\ud55c\ub2e4. \uc81c\uc815 \uc2dc\ub300\uc5d0\ub3c4 \uc601\ud1a0 \ud655\uc7a5\uc740 \uc774\uc5b4\uc84c\uc73c\uba70 \uc548\uc815\uc744 \uc720\uc9c0\ud588\uc9c0\ub9cc, \ud754\ud788 \uc800\uc5f4\ud558\uace0 \ud0c0\ub77d\ud588\ub2e4\uace0 \ud3c9\uac00\ubc1b\ub294 \ud669\uc81c\ub3c4 \uba87\uba87 \uc788\uc5c8\ub2e4(\uac00\ub839 \ud639\uc790\ub294 \uce7c\ub9ac\uad74\ub77c\ub97c \uc815\uc2e0\uc774\uc0c1\uc790\ub85c \ubcf4\uae30\ub3c4 \ud558\uba70(\uc989\uc704 \ud6c4 \uc5bc\ub9c8\uc9c0\ub098\uc9c0 \uc54a\uc544 \uc815\uc2e0\ubcd1\uc774 \uac78\ub824 \ud3ed\uc815\uc744 \uc77c\uc0bc\uc558\ub2e4). \ub124\ub85c\ub294 \uad6d\uac00 \uc815\uc0ac\ubcf4\ub2e4 \uac1c\uc778 \uad00\uc2ec\uc0ac\uc5d0 \ub354 \ubab0\ub450\ud558\uba70 \uc794\uc778\ud588\ub2e4\ub294 \ud3c9\ud310\uc744 \ubc1b\ub294\ub2e4. \ud558\uc9c0\ub9cc \ud604\ub300\uc5d0 \uc774\ub974\ub7ec\uc11c \ub124\ub85c\uc5d0 \ub300\ud574\uc11c\ub294 \uc5ed\uc0ac\uc801\uc778 \uc7ac\ud3c9\uac00\uac00 \uc774\ub8e8\uc5b4\uc9c0\uace0 \uc788\ub2e4.) \uadf8\ub9ac\uace0 \ud50c\ub77c\ube44\uc6b0\uc2a4 \uc655\uc870\uac00 \ub4a4\ub97c \uc787\ub294\ub2e4.(AD 69 - AD 96) \ubca0\uc2a4\ud30c\uc2dc\uc544\ub204\uc2a4, \ud2f0\ud22c\uc2a4, \ub3c4\ubbf8\ud2f0\uc544\ub204\uc2a4. \ub3c4\ubbf8\ud2f0\uc544\ub204\uc2a4 \ud669\uc81c\uc758 \uc554\uc0b4\uc774\ud6c4\uc5d0 \ub85c\ub9c8 \ubc88\uc601\uc758 \uc2dc\uc791\uc744 \uc54c\ub9b0 \uc624\ud604\uc81c \uc2dc\ub300(\ub124\ub974\ubc14-\uc548\ud1a0\ub2c8\ub204\uc2a4 \uc655\uc870(\ub124\ub974\ubc14, \ud2b8\ub77c\uc57c\ub204\uc2a4, \ud558\ub4dc\ub9ac\uc544\ub204\uc2a4, \uc548\ud1a0\ub2c8\ub204\uc2a4 \ud53c\uc6b0\uc2a4, \ub9c8\ub974\ucfe0\uc2a4 \uc544\uc6b0\ub810\ub9ac\uc6b0\uc2a4)\uc5d0\uc11c \ucf64\ubaa8\ub450\uc2a4)\uc5d0 \uc81c\uad6d\uc740 \uc601\ud1a0\ub098 \uacbd\uc81c\u318d\ubb38\ud654\uba74\uc5d0\uc11c \uc815\uc810\uc744 \uc774\ub8e8\uc5c8\ub2e4. \ub124\ub974\ubc14-\uc548\ud1a0\ub2c8\ub204\uc2a4 \uc655\uc870\ub294 \ub300\ub7b5 80\ub144\uac04 \uc9c0\uc18d\ub418\uba74\uc11c \uc774\uc2dc\uae30\uc5d0 \ub85c\ub9c8\ub294 \ub300\ub0b4\uc678\uc801\uc73c\ub85c \uc548\uc815\uc744 \uc774\ub8e8\uc5c8\uace0, \ub85c\ub9c8\uc758 \ud3c9\ud654(\ud30d\uc2a4 \ub85c\ub9c8\ub098) \uc2dc\ub300\uc5d0 \uc81c\uad6d\uc740 \ubc88\uc601\ud588\ub2e4. \ud2b8\ub77c\uc57c\ub204\uc2a4 \uc7ac\uc704\uc2dc\uc5d0\ub294 \ub2e4\ud0a4\uc544(\uc624\ub298\ub0a0\uc758 \ub8e8\ub9c8\ub2c8\uc544)\uc758 \uc815\ubcf5\uacfc, \ud30c\ub974\ud2f0\uc544\uc640\uc758 \uc804\uc7c1\uc744 \ud1b5\ud558\uc5ec \uc81c\uad6d\uc758 \uc601\ud1a0\ub294 \ucd5c\ub300\uac00 \ub418\uba70, \uadf8 \uc9c0\ubc30 \uc9c0\uc5ed\uc774 650\ub9cckm\u00b2\uc5d0 \uc774\ub974\ub800\ub2e4. \ud2b8\ub77c\uc57c\ub204\uc2a4 \uc0ac\ud6c4 \uc989\uc704\ud55c \ud558\ub4dc\ub9ac\uc544\ub204\uc2a4\ub294 \uc815\ubcf5\uc804\uc7c1\uc744 \uc911\ub2e8\ud558\uace0 \uc81c\uad6d\uc744 \uc21c\ucc30\ud558\uba74\uc11c \uad6d\uacbd \uc218\ube44\ub97c \uac15\ud654\ud558\ub294\ub370 \ud798\uc4f4\ub2e4.", "answer": "\ud504\ub9b0\ucf11\uc2a4", "sentence": "\uc544\uc6b0\uad6c\uc2a4\ud22c\uc2a4\ub294 \uc790\uc2e0\uc744 \ud504\ub9b0\ucf11\uc2a4 (\uc81c1\uc758 \uc2dc\ubbfc)\ub77c \uce6d\ud55c\ub2e4(\uc774\ub807\uac8c \uc544\uc6b0\uad6c\uc2a4\ud22c\uc2a4\uac00 \ub3c5\uc7ac\uad00\uc744 \uac70\ubd80\ud558\ub294 \ub4f1 \uc870\uc2ec\uc2a4\ub7fd\uac8c \ud589\ub3d9\ud55c \uac83\uc740 \uc591\uc544\ubc84\uc9c0\uc600\ub358 \uc728\ub9ac\uc6b0\uc2a4 \uce74\uc774\uc0ac\ub974\uc758 \uc554\uc0b4 \ub54c\ubb38\uc774\uae30\ub3c4 \ud558\ub2e4).", "paragraph_sentence": "\ub0b4\uc804\uc5d0\uc11c \uc2b9\ub9ac\ud55c \uc625\ud0c0\ube44\uc544\ub204\uc2a4\ub294 \uc544\uc6b0\uad6c\uc2a4\ud22c\uc2a4\uc758 \uce6d\ud638\ub97c \ubc1b\uace0 \uac70\uc758 \uc808\ub300\uc801\uc778 \uad8c\ub825\uc744 \uc7a1\uc558\uc9c0\ub9cc, \uacf5\ud654\uc815\uc744 \uac00\uc7a5\ud588\ub2e4. \uc774\ub54c\uc758 \ud1b5\uce58\uccb4\uc81c\ub97c \uc6d0\uc218\uc815\uc774\ub77c \ud558\uba70(\uc0ac\uc2e4\uc0c1 \uc81c\uc815\uc774\ub2e4), \uc544\uc6b0\uad6c\uc2a4\ud22c\uc2a4\ub294 \uc790\uc2e0\uc744 \ud504\ub9b0\ucf11\uc2a4 (\uc81c1\uc758 \uc2dc\ubbfc)\ub77c \uce6d\ud55c\ub2e4(\uc774\ub807\uac8c \uc544\uc6b0\uad6c\uc2a4\ud22c\uc2a4\uac00 \ub3c5\uc7ac\uad00\uc744 \uac70\ubd80\ud558\ub294 \ub4f1 \uc870\uc2ec\uc2a4\ub7fd\uac8c \ud589\ub3d9\ud55c \uac83\uc740 \uc591\uc544\ubc84\uc9c0\uc600\ub358 \uc728\ub9ac\uc6b0\uc2a4 \uce74\uc774\uc0ac\ub974\uc758 \uc554\uc0b4 \ub54c\ubb38\uc774\uae30\ub3c4 \ud558\ub2e4). \uc544\uc6b0\uad6c\uc2a4\ud22c\uc2a4\uc758 \ud6c4\uacc4\uc790 \ud2f0\ubca0\ub9ac\uc6b0\uc2a4\ub294 \uc2ec\uac01\ud55c \ubc18\ub300 \uc5c6\uc774 \uad8c\uc88c\uc5d0 \uc62c\ub77c \uc544\uc6b0\uad6c\uc2a4\ud22c\uc2a4\uc5d0 \uc774\uc5b4 \ubcf8\uaca9\uc801\uc778 \uc728\ub9ac\uc6b0\uc2a4-\ud074\ub77c\uc6b0\ub514\uc6b0\uc2a4 \uc655\uc870\ub97c \uc774\ub8e8\uc5c8\uace0, \uc774 \uc655\uc870\ub294 \ud2f0\ubca0\ub9ac\uc6b0\uc2a4 \uc0ac\ud6c4 \uce7c\ub9ac\uad74\ub77c\uc640 \ud074\ub77c\uc6b0\ub514\uc6b0\uc2a4\uc758 \uce58\uc138\ub97c \uac70\uccd0 \uae30\uc6d0\ud6c4 68\ub144 \ub124\ub85c\uac00 \uc790\uc0b4\ud560 \ub54c\uae4c\uc9c0 \uc774\uc5b4\uc9c4\ub2e4. \uc774\ud6c4 1\ub144\uac04 \ub85c\ub9c8\ub294 \ub0b4\uc804\uc0c1\ud0dc\uc5d0 \ub3cc\uc785\ud558\uba74\uc11c \uac08\ubc14, \uc624\ud1a0, \ube44\ud154\ub9ac\uc6b0\uc2a4\uac00 \uc989\uc704\ud55c\ub2e4. \ud6c4\uc5d0 \uc2dc\ub9ac\uc544 \uc8fc\ub454\uad70 \uc0ac\ub839\uad00\uc774\uc5c8\ub358 \ubca0\uc2a4\ud30c\uc2dc\uc544\ub204\uc2a4\uac00 \uc790\uae30 \uad70\ub2e8\uc758 \uc9c0\uc9c0\ub97c \ubc1b\uc544 \ub85c\ub9c8\uc5d0 \uc785\uc131\ud558\uc5ec \ube44\ud154\ub9ac\uc6b0\uc2a4\ub97c \uc0b4\ud574\ud558\uace0 \uc655\uc704\uc5d0 \uc624\ub974\uba74\uc11c \ud50c\ub77c\ube44\uc6b0\uc2a4 \uc655\uc870\uc758 \uc2dc\ub300\ub97c \uac1c\ucc3d\ud55c\ub2e4. \uc81c\uc815 \uc2dc\ub300\uc5d0\ub3c4 \uc601\ud1a0 \ud655\uc7a5\uc740 \uc774\uc5b4\uc84c\uc73c\uba70 \uc548\uc815\uc744 \uc720\uc9c0\ud588\uc9c0\ub9cc, \ud754\ud788 \uc800\uc5f4\ud558\uace0 \ud0c0\ub77d\ud588\ub2e4\uace0 \ud3c9\uac00\ubc1b\ub294 \ud669\uc81c\ub3c4 \uba87\uba87 \uc788\uc5c8\ub2e4(\uac00\ub839 \ud639\uc790\ub294 \uce7c\ub9ac\uad74\ub77c\ub97c \uc815\uc2e0\uc774\uc0c1\uc790\ub85c \ubcf4\uae30\ub3c4 \ud558\uba70 (\uc989\uc704 \ud6c4 \uc5bc\ub9c8\uc9c0\ub098\uc9c0 \uc54a\uc544 \uc815\uc2e0\ubcd1\uc774 \uac78\ub824 \ud3ed\uc815\uc744 \uc77c\uc0bc\uc558\ub2e4). \ub124\ub85c\ub294 \uad6d\uac00 \uc815\uc0ac\ubcf4\ub2e4 \uac1c\uc778 \uad00\uc2ec\uc0ac\uc5d0 \ub354 \ubab0\ub450\ud558\uba70 \uc794\uc778\ud588\ub2e4\ub294 \ud3c9\ud310\uc744 \ubc1b\ub294\ub2e4. \ud558\uc9c0\ub9cc \ud604\ub300\uc5d0 \uc774\ub974\ub7ec\uc11c \ub124\ub85c\uc5d0 \ub300\ud574\uc11c\ub294 \uc5ed\uc0ac\uc801\uc778 \uc7ac\ud3c9\uac00\uac00 \uc774\ub8e8\uc5b4\uc9c0\uace0 \uc788\ub2e4. ) \uadf8\ub9ac\uace0 \ud50c\ub77c\ube44\uc6b0\uc2a4 \uc655\uc870\uac00 \ub4a4\ub97c \uc787\ub294\ub2e4.(AD 69 - AD 96) \ubca0\uc2a4\ud30c\uc2dc\uc544\ub204\uc2a4, \ud2f0\ud22c\uc2a4, \ub3c4\ubbf8\ud2f0\uc544\ub204\uc2a4. \ub3c4\ubbf8\ud2f0\uc544\ub204\uc2a4 \ud669\uc81c\uc758 \uc554\uc0b4\uc774\ud6c4\uc5d0 \ub85c\ub9c8 \ubc88\uc601\uc758 \uc2dc\uc791\uc744 \uc54c\ub9b0 \uc624\ud604\uc81c \uc2dc\ub300(\ub124\ub974\ubc14-\uc548\ud1a0\ub2c8\ub204\uc2a4 \uc655\uc870(\ub124\ub974\ubc14, \ud2b8\ub77c\uc57c\ub204\uc2a4, \ud558\ub4dc\ub9ac\uc544\ub204\uc2a4, \uc548\ud1a0\ub2c8\ub204\uc2a4 \ud53c\uc6b0\uc2a4, \ub9c8\ub974\ucfe0\uc2a4 \uc544\uc6b0\ub810\ub9ac\uc6b0\uc2a4)\uc5d0\uc11c \ucf64\ubaa8\ub450\uc2a4)\uc5d0 \uc81c\uad6d\uc740 \uc601\ud1a0\ub098 \uacbd\uc81c\u318d\ubb38\ud654\uba74\uc5d0\uc11c \uc815\uc810\uc744 \uc774\ub8e8\uc5c8\ub2e4. \ub124\ub974\ubc14-\uc548\ud1a0\ub2c8\ub204\uc2a4 \uc655\uc870\ub294 \ub300\ub7b5 80\ub144\uac04 \uc9c0\uc18d\ub418\uba74\uc11c \uc774\uc2dc\uae30\uc5d0 \ub85c\ub9c8\ub294 \ub300\ub0b4\uc678\uc801\uc73c\ub85c \uc548\uc815\uc744 \uc774\ub8e8\uc5c8\uace0, \ub85c\ub9c8\uc758 \ud3c9\ud654(\ud30d\uc2a4 \ub85c\ub9c8\ub098) \uc2dc\ub300\uc5d0 \uc81c\uad6d\uc740 \ubc88\uc601\ud588\ub2e4. \ud2b8\ub77c\uc57c\ub204\uc2a4 \uc7ac\uc704\uc2dc\uc5d0\ub294 \ub2e4\ud0a4\uc544(\uc624\ub298\ub0a0\uc758 \ub8e8\ub9c8\ub2c8\uc544)\uc758 \uc815\ubcf5\uacfc, \ud30c\ub974\ud2f0\uc544\uc640\uc758 \uc804\uc7c1\uc744 \ud1b5\ud558\uc5ec \uc81c\uad6d\uc758 \uc601\ud1a0\ub294 \ucd5c\ub300\uac00 \ub418\uba70, \uadf8 \uc9c0\ubc30 \uc9c0\uc5ed\uc774 650\ub9cckm\u00b2\uc5d0 \uc774\ub974\ub800\ub2e4. \ud2b8\ub77c\uc57c\ub204\uc2a4 \uc0ac\ud6c4 \uc989\uc704\ud55c \ud558\ub4dc\ub9ac\uc544\ub204\uc2a4\ub294 \uc815\ubcf5\uc804\uc7c1\uc744 \uc911\ub2e8\ud558\uace0 \uc81c\uad6d\uc744 \uc21c\ucc30\ud558\uba74\uc11c \uad6d\uacbd \uc218\ube44\ub97c \uac15\ud654\ud558\ub294\ub370 \ud798\uc4f4\ub2e4.", "paragraph_answer": "\ub0b4\uc804\uc5d0\uc11c \uc2b9\ub9ac\ud55c \uc625\ud0c0\ube44\uc544\ub204\uc2a4\ub294 \uc544\uc6b0\uad6c\uc2a4\ud22c\uc2a4\uc758 \uce6d\ud638\ub97c \ubc1b\uace0 \uac70\uc758 \uc808\ub300\uc801\uc778 \uad8c\ub825\uc744 \uc7a1\uc558\uc9c0\ub9cc, \uacf5\ud654\uc815\uc744 \uac00\uc7a5\ud588\ub2e4. \uc774\ub54c\uc758 \ud1b5\uce58\uccb4\uc81c\ub97c \uc6d0\uc218\uc815\uc774\ub77c \ud558\uba70(\uc0ac\uc2e4\uc0c1 \uc81c\uc815\uc774\ub2e4), \uc544\uc6b0\uad6c\uc2a4\ud22c\uc2a4\ub294 \uc790\uc2e0\uc744 \ud504\ub9b0\ucf11\uc2a4 (\uc81c1\uc758 \uc2dc\ubbfc)\ub77c \uce6d\ud55c\ub2e4(\uc774\ub807\uac8c \uc544\uc6b0\uad6c\uc2a4\ud22c\uc2a4\uac00 \ub3c5\uc7ac\uad00\uc744 \uac70\ubd80\ud558\ub294 \ub4f1 \uc870\uc2ec\uc2a4\ub7fd\uac8c \ud589\ub3d9\ud55c \uac83\uc740 \uc591\uc544\ubc84\uc9c0\uc600\ub358 \uc728\ub9ac\uc6b0\uc2a4 \uce74\uc774\uc0ac\ub974\uc758 \uc554\uc0b4 \ub54c\ubb38\uc774\uae30\ub3c4 \ud558\ub2e4). \uc544\uc6b0\uad6c\uc2a4\ud22c\uc2a4\uc758 \ud6c4\uacc4\uc790 \ud2f0\ubca0\ub9ac\uc6b0\uc2a4\ub294 \uc2ec\uac01\ud55c \ubc18\ub300 \uc5c6\uc774 \uad8c\uc88c\uc5d0 \uc62c\ub77c \uc544\uc6b0\uad6c\uc2a4\ud22c\uc2a4\uc5d0 \uc774\uc5b4 \ubcf8\uaca9\uc801\uc778 \uc728\ub9ac\uc6b0\uc2a4-\ud074\ub77c\uc6b0\ub514\uc6b0\uc2a4 \uc655\uc870\ub97c \uc774\ub8e8\uc5c8\uace0, \uc774 \uc655\uc870\ub294 \ud2f0\ubca0\ub9ac\uc6b0\uc2a4 \uc0ac\ud6c4 \uce7c\ub9ac\uad74\ub77c\uc640 \ud074\ub77c\uc6b0\ub514\uc6b0\uc2a4\uc758 \uce58\uc138\ub97c \uac70\uccd0 \uae30\uc6d0\ud6c4 68\ub144 \ub124\ub85c\uac00 \uc790\uc0b4\ud560 \ub54c\uae4c\uc9c0 \uc774\uc5b4\uc9c4\ub2e4. \uc774\ud6c4 1\ub144\uac04 \ub85c\ub9c8\ub294 \ub0b4\uc804\uc0c1\ud0dc\uc5d0 \ub3cc\uc785\ud558\uba74\uc11c \uac08\ubc14, \uc624\ud1a0, \ube44\ud154\ub9ac\uc6b0\uc2a4\uac00 \uc989\uc704\ud55c\ub2e4. \ud6c4\uc5d0 \uc2dc\ub9ac\uc544 \uc8fc\ub454\uad70 \uc0ac\ub839\uad00\uc774\uc5c8\ub358 \ubca0\uc2a4\ud30c\uc2dc\uc544\ub204\uc2a4\uac00 \uc790\uae30 \uad70\ub2e8\uc758 \uc9c0\uc9c0\ub97c \ubc1b\uc544 \ub85c\ub9c8\uc5d0 \uc785\uc131\ud558\uc5ec \ube44\ud154\ub9ac\uc6b0\uc2a4\ub97c \uc0b4\ud574\ud558\uace0 \uc655\uc704\uc5d0 \uc624\ub974\uba74\uc11c \ud50c\ub77c\ube44\uc6b0\uc2a4 \uc655\uc870\uc758 \uc2dc\ub300\ub97c \uac1c\ucc3d\ud55c\ub2e4. \uc81c\uc815 \uc2dc\ub300\uc5d0\ub3c4 \uc601\ud1a0 \ud655\uc7a5\uc740 \uc774\uc5b4\uc84c\uc73c\uba70 \uc548\uc815\uc744 \uc720\uc9c0\ud588\uc9c0\ub9cc, \ud754\ud788 \uc800\uc5f4\ud558\uace0 \ud0c0\ub77d\ud588\ub2e4\uace0 \ud3c9\uac00\ubc1b\ub294 \ud669\uc81c\ub3c4 \uba87\uba87 \uc788\uc5c8\ub2e4(\uac00\ub839 \ud639\uc790\ub294 \uce7c\ub9ac\uad74\ub77c\ub97c \uc815\uc2e0\uc774\uc0c1\uc790\ub85c \ubcf4\uae30\ub3c4 \ud558\uba70(\uc989\uc704 \ud6c4 \uc5bc\ub9c8\uc9c0\ub098\uc9c0 \uc54a\uc544 \uc815\uc2e0\ubcd1\uc774 \uac78\ub824 \ud3ed\uc815\uc744 \uc77c\uc0bc\uc558\ub2e4). \ub124\ub85c\ub294 \uad6d\uac00 \uc815\uc0ac\ubcf4\ub2e4 \uac1c\uc778 \uad00\uc2ec\uc0ac\uc5d0 \ub354 \ubab0\ub450\ud558\uba70 \uc794\uc778\ud588\ub2e4\ub294 \ud3c9\ud310\uc744 \ubc1b\ub294\ub2e4. \ud558\uc9c0\ub9cc \ud604\ub300\uc5d0 \uc774\ub974\ub7ec\uc11c \ub124\ub85c\uc5d0 \ub300\ud574\uc11c\ub294 \uc5ed\uc0ac\uc801\uc778 \uc7ac\ud3c9\uac00\uac00 \uc774\ub8e8\uc5b4\uc9c0\uace0 \uc788\ub2e4.) \uadf8\ub9ac\uace0 \ud50c\ub77c\ube44\uc6b0\uc2a4 \uc655\uc870\uac00 \ub4a4\ub97c \uc787\ub294\ub2e4.(AD 69 - AD 96) \ubca0\uc2a4\ud30c\uc2dc\uc544\ub204\uc2a4, \ud2f0\ud22c\uc2a4, \ub3c4\ubbf8\ud2f0\uc544\ub204\uc2a4. \ub3c4\ubbf8\ud2f0\uc544\ub204\uc2a4 \ud669\uc81c\uc758 \uc554\uc0b4\uc774\ud6c4\uc5d0 \ub85c\ub9c8 \ubc88\uc601\uc758 \uc2dc\uc791\uc744 \uc54c\ub9b0 \uc624\ud604\uc81c \uc2dc\ub300(\ub124\ub974\ubc14-\uc548\ud1a0\ub2c8\ub204\uc2a4 \uc655\uc870(\ub124\ub974\ubc14, \ud2b8\ub77c\uc57c\ub204\uc2a4, \ud558\ub4dc\ub9ac\uc544\ub204\uc2a4, \uc548\ud1a0\ub2c8\ub204\uc2a4 \ud53c\uc6b0\uc2a4, \ub9c8\ub974\ucfe0\uc2a4 \uc544\uc6b0\ub810\ub9ac\uc6b0\uc2a4)\uc5d0\uc11c \ucf64\ubaa8\ub450\uc2a4)\uc5d0 \uc81c\uad6d\uc740 \uc601\ud1a0\ub098 \uacbd\uc81c\u318d\ubb38\ud654\uba74\uc5d0\uc11c \uc815\uc810\uc744 \uc774\ub8e8\uc5c8\ub2e4. \ub124\ub974\ubc14-\uc548\ud1a0\ub2c8\ub204\uc2a4 \uc655\uc870\ub294 \ub300\ub7b5 80\ub144\uac04 \uc9c0\uc18d\ub418\uba74\uc11c \uc774\uc2dc\uae30\uc5d0 \ub85c\ub9c8\ub294 \ub300\ub0b4\uc678\uc801\uc73c\ub85c \uc548\uc815\uc744 \uc774\ub8e8\uc5c8\uace0, \ub85c\ub9c8\uc758 \ud3c9\ud654(\ud30d\uc2a4 \ub85c\ub9c8\ub098) \uc2dc\ub300\uc5d0 \uc81c\uad6d\uc740 \ubc88\uc601\ud588\ub2e4. \ud2b8\ub77c\uc57c\ub204\uc2a4 \uc7ac\uc704\uc2dc\uc5d0\ub294 \ub2e4\ud0a4\uc544(\uc624\ub298\ub0a0\uc758 \ub8e8\ub9c8\ub2c8\uc544)\uc758 \uc815\ubcf5\uacfc, \ud30c\ub974\ud2f0\uc544\uc640\uc758 \uc804\uc7c1\uc744 \ud1b5\ud558\uc5ec \uc81c\uad6d\uc758 \uc601\ud1a0\ub294 \ucd5c\ub300\uac00 \ub418\uba70, \uadf8 \uc9c0\ubc30 \uc9c0\uc5ed\uc774 650\ub9cckm\u00b2\uc5d0 \uc774\ub974\ub800\ub2e4. \ud2b8\ub77c\uc57c\ub204\uc2a4 \uc0ac\ud6c4 \uc989\uc704\ud55c \ud558\ub4dc\ub9ac\uc544\ub204\uc2a4\ub294 \uc815\ubcf5\uc804\uc7c1\uc744 \uc911\ub2e8\ud558\uace0 \uc81c\uad6d\uc744 \uc21c\ucc30\ud558\uba74\uc11c \uad6d\uacbd \uc218\ube44\ub97c \uac15\ud654\ud558\ub294\ub370 \ud798\uc4f4\ub2e4.", "sentence_answer": "\uc544\uc6b0\uad6c\uc2a4\ud22c\uc2a4\ub294 \uc790\uc2e0\uc744 \ud504\ub9b0\ucf11\uc2a4 (\uc81c1\uc758 \uc2dc\ubbfc)\ub77c \uce6d\ud55c\ub2e4(\uc774\ub807\uac8c \uc544\uc6b0\uad6c\uc2a4\ud22c\uc2a4\uac00 \ub3c5\uc7ac\uad00\uc744 \uac70\ubd80\ud558\ub294 \ub4f1 \uc870\uc2ec\uc2a4\ub7fd\uac8c \ud589\ub3d9\ud55c \uac83\uc740 \uc591\uc544\ubc84\uc9c0\uc600\ub358 \uc728\ub9ac\uc6b0\uc2a4 \uce74\uc774\uc0ac\ub974\uc758 \uc554\uc0b4 \ub54c\ubb38\uc774\uae30\ub3c4 \ud558\ub2e4).", "paragraph_id": "6466082-5-0", "paragraph_question": "question: \uc544\uc6b0\uad6c\uc2a4\ud22c\uc2a4\ub294 \uc81c1\uc758 \uc2dc\ubbfc\uc774\ub77c\ub294 \ub73b\uc758 \ubb34\uc5c7\uc774\ub77c\uace0 \ubcf8\uc778\uc744 \ubd88\ub800\ub294\uac00?, context: \ub0b4\uc804\uc5d0\uc11c \uc2b9\ub9ac\ud55c \uc625\ud0c0\ube44\uc544\ub204\uc2a4\ub294 \uc544\uc6b0\uad6c\uc2a4\ud22c\uc2a4\uc758 \uce6d\ud638\ub97c \ubc1b\uace0 \uac70\uc758 \uc808\ub300\uc801\uc778 \uad8c\ub825\uc744 \uc7a1\uc558\uc9c0\ub9cc, \uacf5\ud654\uc815\uc744 \uac00\uc7a5\ud588\ub2e4. \uc774\ub54c\uc758 \ud1b5\uce58\uccb4\uc81c\ub97c \uc6d0\uc218\uc815\uc774\ub77c \ud558\uba70(\uc0ac\uc2e4\uc0c1 \uc81c\uc815\uc774\ub2e4), \uc544\uc6b0\uad6c\uc2a4\ud22c\uc2a4\ub294 \uc790\uc2e0\uc744 \ud504\ub9b0\ucf11\uc2a4(\uc81c1\uc758 \uc2dc\ubbfc)\ub77c \uce6d\ud55c\ub2e4(\uc774\ub807\uac8c \uc544\uc6b0\uad6c\uc2a4\ud22c\uc2a4\uac00 \ub3c5\uc7ac\uad00\uc744 \uac70\ubd80\ud558\ub294 \ub4f1 \uc870\uc2ec\uc2a4\ub7fd\uac8c \ud589\ub3d9\ud55c \uac83\uc740 \uc591\uc544\ubc84\uc9c0\uc600\ub358 \uc728\ub9ac\uc6b0\uc2a4 \uce74\uc774\uc0ac\ub974\uc758 \uc554\uc0b4 \ub54c\ubb38\uc774\uae30\ub3c4 \ud558\ub2e4). \uc544\uc6b0\uad6c\uc2a4\ud22c\uc2a4\uc758 \ud6c4\uacc4\uc790 \ud2f0\ubca0\ub9ac\uc6b0\uc2a4\ub294 \uc2ec\uac01\ud55c \ubc18\ub300 \uc5c6\uc774 \uad8c\uc88c\uc5d0 \uc62c\ub77c \uc544\uc6b0\uad6c\uc2a4\ud22c\uc2a4\uc5d0 \uc774\uc5b4 \ubcf8\uaca9\uc801\uc778 \uc728\ub9ac\uc6b0\uc2a4-\ud074\ub77c\uc6b0\ub514\uc6b0\uc2a4 \uc655\uc870\ub97c \uc774\ub8e8\uc5c8\uace0, \uc774 \uc655\uc870\ub294 \ud2f0\ubca0\ub9ac\uc6b0\uc2a4 \uc0ac\ud6c4 \uce7c\ub9ac\uad74\ub77c\uc640 \ud074\ub77c\uc6b0\ub514\uc6b0\uc2a4\uc758 \uce58\uc138\ub97c \uac70\uccd0 \uae30\uc6d0\ud6c4 68\ub144 \ub124\ub85c\uac00 \uc790\uc0b4\ud560 \ub54c\uae4c\uc9c0 \uc774\uc5b4\uc9c4\ub2e4. \uc774\ud6c4 1\ub144\uac04 \ub85c\ub9c8\ub294 \ub0b4\uc804\uc0c1\ud0dc\uc5d0 \ub3cc\uc785\ud558\uba74\uc11c \uac08\ubc14, \uc624\ud1a0, \ube44\ud154\ub9ac\uc6b0\uc2a4\uac00 \uc989\uc704\ud55c\ub2e4. \ud6c4\uc5d0 \uc2dc\ub9ac\uc544 \uc8fc\ub454\uad70 \uc0ac\ub839\uad00\uc774\uc5c8\ub358 \ubca0\uc2a4\ud30c\uc2dc\uc544\ub204\uc2a4\uac00 \uc790\uae30 \uad70\ub2e8\uc758 \uc9c0\uc9c0\ub97c \ubc1b\uc544 \ub85c\ub9c8\uc5d0 \uc785\uc131\ud558\uc5ec \ube44\ud154\ub9ac\uc6b0\uc2a4\ub97c \uc0b4\ud574\ud558\uace0 \uc655\uc704\uc5d0 \uc624\ub974\uba74\uc11c \ud50c\ub77c\ube44\uc6b0\uc2a4 \uc655\uc870\uc758 \uc2dc\ub300\ub97c \uac1c\ucc3d\ud55c\ub2e4. \uc81c\uc815 \uc2dc\ub300\uc5d0\ub3c4 \uc601\ud1a0 \ud655\uc7a5\uc740 \uc774\uc5b4\uc84c\uc73c\uba70 \uc548\uc815\uc744 \uc720\uc9c0\ud588\uc9c0\ub9cc, \ud754\ud788 \uc800\uc5f4\ud558\uace0 \ud0c0\ub77d\ud588\ub2e4\uace0 \ud3c9\uac00\ubc1b\ub294 \ud669\uc81c\ub3c4 \uba87\uba87 \uc788\uc5c8\ub2e4(\uac00\ub839 \ud639\uc790\ub294 \uce7c\ub9ac\uad74\ub77c\ub97c \uc815\uc2e0\uc774\uc0c1\uc790\ub85c \ubcf4\uae30\ub3c4 \ud558\uba70(\uc989\uc704 \ud6c4 \uc5bc\ub9c8\uc9c0\ub098\uc9c0 \uc54a\uc544 \uc815\uc2e0\ubcd1\uc774 \uac78\ub824 \ud3ed\uc815\uc744 \uc77c\uc0bc\uc558\ub2e4). \ub124\ub85c\ub294 \uad6d\uac00 \uc815\uc0ac\ubcf4\ub2e4 \uac1c\uc778 \uad00\uc2ec\uc0ac\uc5d0 \ub354 \ubab0\ub450\ud558\uba70 \uc794\uc778\ud588\ub2e4\ub294 \ud3c9\ud310\uc744 \ubc1b\ub294\ub2e4. \ud558\uc9c0\ub9cc \ud604\ub300\uc5d0 \uc774\ub974\ub7ec\uc11c \ub124\ub85c\uc5d0 \ub300\ud574\uc11c\ub294 \uc5ed\uc0ac\uc801\uc778 \uc7ac\ud3c9\uac00\uac00 \uc774\ub8e8\uc5b4\uc9c0\uace0 \uc788\ub2e4.) \uadf8\ub9ac\uace0 \ud50c\ub77c\ube44\uc6b0\uc2a4 \uc655\uc870\uac00 \ub4a4\ub97c \uc787\ub294\ub2e4.(AD 69 - AD 96) \ubca0\uc2a4\ud30c\uc2dc\uc544\ub204\uc2a4, \ud2f0\ud22c\uc2a4, \ub3c4\ubbf8\ud2f0\uc544\ub204\uc2a4. \ub3c4\ubbf8\ud2f0\uc544\ub204\uc2a4 \ud669\uc81c\uc758 \uc554\uc0b4\uc774\ud6c4\uc5d0 \ub85c\ub9c8 \ubc88\uc601\uc758 \uc2dc\uc791\uc744 \uc54c\ub9b0 \uc624\ud604\uc81c \uc2dc\ub300(\ub124\ub974\ubc14-\uc548\ud1a0\ub2c8\ub204\uc2a4 \uc655\uc870(\ub124\ub974\ubc14, \ud2b8\ub77c\uc57c\ub204\uc2a4, \ud558\ub4dc\ub9ac\uc544\ub204\uc2a4, \uc548\ud1a0\ub2c8\ub204\uc2a4 \ud53c\uc6b0\uc2a4, \ub9c8\ub974\ucfe0\uc2a4 \uc544\uc6b0\ub810\ub9ac\uc6b0\uc2a4)\uc5d0\uc11c \ucf64\ubaa8\ub450\uc2a4)\uc5d0 \uc81c\uad6d\uc740 \uc601\ud1a0\ub098 \uacbd\uc81c\u318d\ubb38\ud654\uba74\uc5d0\uc11c \uc815\uc810\uc744 \uc774\ub8e8\uc5c8\ub2e4. \ub124\ub974\ubc14-\uc548\ud1a0\ub2c8\ub204\uc2a4 \uc655\uc870\ub294 \ub300\ub7b5 80\ub144\uac04 \uc9c0\uc18d\ub418\uba74\uc11c \uc774\uc2dc\uae30\uc5d0 \ub85c\ub9c8\ub294 \ub300\ub0b4\uc678\uc801\uc73c\ub85c \uc548\uc815\uc744 \uc774\ub8e8\uc5c8\uace0, \ub85c\ub9c8\uc758 \ud3c9\ud654(\ud30d\uc2a4 \ub85c\ub9c8\ub098) \uc2dc\ub300\uc5d0 \uc81c\uad6d\uc740 \ubc88\uc601\ud588\ub2e4. \ud2b8\ub77c\uc57c\ub204\uc2a4 \uc7ac\uc704\uc2dc\uc5d0\ub294 \ub2e4\ud0a4\uc544(\uc624\ub298\ub0a0\uc758 \ub8e8\ub9c8\ub2c8\uc544)\uc758 \uc815\ubcf5\uacfc, \ud30c\ub974\ud2f0\uc544\uc640\uc758 \uc804\uc7c1\uc744 \ud1b5\ud558\uc5ec \uc81c\uad6d\uc758 \uc601\ud1a0\ub294 \ucd5c\ub300\uac00 \ub418\uba70, \uadf8 \uc9c0\ubc30 \uc9c0\uc5ed\uc774 650\ub9cckm\u00b2\uc5d0 \uc774\ub974\ub800\ub2e4. \ud2b8\ub77c\uc57c\ub204\uc2a4 \uc0ac\ud6c4 \uc989\uc704\ud55c \ud558\ub4dc\ub9ac\uc544\ub204\uc2a4\ub294 \uc815\ubcf5\uc804\uc7c1\uc744 \uc911\ub2e8\ud558\uace0 \uc81c\uad6d\uc744 \uc21c\ucc30\ud558\uba74\uc11c \uad6d\uacbd \uc218\ube44\ub97c \uac15\ud654\ud558\ub294\ub370 \ud798\uc4f4\ub2e4."} {"question": "\uc784\uc218\ud601\uc758 \ub0a0 \uc740 \uba87\uc6d4 \uba87\uc77c\uc778\uac00?", "paragraph": "2001\ub144\uc774 \ub418\uc790 1\ub144\uc774\ub098 \uc9c0\ub09c \uc784\uc218\ud601 \uc0ac\uac74\uc740 \uc11c\uc11c\ud788 \ubb3b\ud788\uae30 \uc2dc\uc791\ud558\uc600\uace0, \uad00\ub828\ub41c \uae30\uc0ac\ub3c4 \uc904\uc5b4\ub4e4\uae30 \uc2dc\uc791\ud558\uc600\ub2e4. \uadf8\ub7ec\ub098 \uc740\ud1f4\uc2dd\uc744 \uce58\ub974\ub294 \ud604\ub300 \uc720\ub2c8\ucf58\uc2a4 \uc18c\uc18d \ub0b4\uc57c\uc218 \uc5fc\uacbd\uc5fd(\ud604 SK \uc640\uc774\ubc88\uc2a4 \ub2e8\uc7a5)\uc774 \uc790\uc2e0\uc758 \ud648\ud398\uc774\uc9c0\ub97c \ud1b5\ud574 \uc57c\uad6c\uc6a9\ud488 \uacbd\ub9e4\ub85c \ubaa8\uc740 \uc218\uc775\uae08 \uc911 500\ub9cc \uc6d0\uc744 \ubcd1\uc0c1\uc5d0 \ub204\uc6cc \uc788\ub294 \uc784\uc218\ud601\uc758 \uce58\ub8cc\ube44\ub85c \uc804\ub2ec\ud574 \ud654\uc81c\uac00 \ub418\uc5c8\uace0, \uc791\uc740 \ub3c4\uc6c0\uc774\ub77c\ub3c4 \uc8fc\uae30 \uc704\ud574, \ub86f\ub370 \uc790\uc774\uc5b8\uce20\ub294 \uadf8\uac00 \uc4f0\ub7ec\uc9c4 \ub0a0\uc778 4\uc6d4 18\uc77c\uc744 \uc784\uc218\ud601\uc758 \ub0a0\ub85c \uc815\ud558\uace0 4\uc6d4 18\uc77c\uc5d0 \uc5f4\ub9ac\ub294 \uacbd\uae30\uc758 \uc218\uc775\uae08\uc758 \uc804\uc561\uc744 \uc784\uc218\ud601\uc758 \uce58\ub8cc\ube44\ub85c \uc804\ud558\uace0, \uc784\uc218\ud601\uc758 \uc544\ub4e4\uc774 \uc2dc\uad6c\ub97c \ud558\uac8c \ub418\uc5c8\ub2e4. \ubd80\uc0b0\uc5d0\uc11c \ub86f\ub370 \uc790\uc774\uc5b8\uce20\uc640 \uac19\uc740 \uc5f0\uace0\uc9c0\ub97c \uc4f0\ub294 \ucd95\uad6c \ud300 \ubd80\uc0b0 \uc544\uc774\ud30c\ud06c\ub294 \uc784\uc218\ud601\uc758 \ub0a0\uc744 \ub9de\uc544 \uc784\uc218\ud601\uc744 \uc704\ud574 \uce58\ub8cc\ube44\ub97c \ubcf4\ub0b4\uae30\ub85c \ud588\ub2e4. \uc774\ud6c4, \ub2f9\uc2dc \ub86f\ub370 \uc790\uc774\uc5b8\uce20\uc758 \uac10\ub3c5\uc774\uc5c8\ub358 \uace0 \uae40\uba85\uc131 \uac10\ub3c5\uc774 \uc62c\uc2a4\ud0c0\uc804 \ud589\uc0ac\uac00 \ud55c\ucc3d\uc774\ub358 \uc7a0\uc2e4\uc57c\uad6c\uc7a5\uc5d0\uc11c \ub86f\ub370 \uc790\uc774\uc5b8\uce20 \ud32c\ud074\ub7fd\uc774 \uc758\uc2dd \ubd88\uba85\uc73c\ub85c \ubcd1\uc0c1\uc5d0 \ub204\uc6cc \uc788\ub294 \uc784\uc218\ud601\uc744 \uaca9\ub824\ud558\uae30 \uc704\ud55c \uc11c\uba85\uc744 \ud588\uc744 \ub2f9\uc2dc '\uc784\uc218\ud601 \uc120\uc218, \uc5b8\uc81c \uadf8\ub77c\uc6b4\ub4dc\uc5d0\uc11c \ub9cc\ub098\uac8c \ub420\uae4c\uc694, \uadf8 \ub0a0\uc744 \uae30\ub2e4\ub9ac\uaca0\uc18c'\ub77c\uace0 \uc801\uc740 \uc801\uc774 \uc788\uc5c8\ub2e4. \ud558\uc9c0\ub9cc \uadf8 \ud574 7\uc6d4 24\uc77c \uae40\uba85\uc131 \uac10\ub3c5\uc774 \uc2ec\uc7a5\ubcd1\uc73c\ub85c \ud0c0\uacc4\ud55c \ub4a4, \uace0 \uae40\uba85\uc131 \uac10\ub3c5\uacfc \uc784\uc218\ud601\uc774 \ud55c \uc57d\uc18d\uc740 \uc9c0\ucf1c\uc9c0\uc9c0 \uc54a\uac8c \ub418\uc5c8\uace0, \uc784\uc218\ud601\uc744 \uc704\ud55c \uce58\ub8cc\uae08\uc744 \ubaa8\uc73c\uc790\ub294 \uc6b4\ub3d9\uc774 \uc7a0\uc2dc \uc77c\uc5b4\ub0ac\uc5c8\ub2e4.", "answer": "4\uc6d4 18\uc77c", "sentence": "\ub0a0\uc778 4\uc6d4 18\uc77c \uc744 \uc784\uc218\ud601\uc758 \ub0a0\ub85c \uc815\ud558\uace0 4\uc6d4 18\uc77c\uc5d0 \uc5f4\ub9ac\ub294 \uacbd\uae30\uc758 \uc218\uc775\uae08\uc758 \uc804\uc561\uc744 \uc784\uc218\ud601\uc758 \uce58\ub8cc\ube44\ub85c \uc804\ud558\uace0, \uc784\uc218\ud601\uc758 \uc544\ub4e4\uc774 \uc2dc\uad6c\ub97c \ud558\uac8c \ub418\uc5c8\ub2e4.", "paragraph_sentence": "2001\ub144\uc774 \ub418\uc790 1\ub144\uc774\ub098 \uc9c0\ub09c \uc784\uc218\ud601 \uc0ac\uac74\uc740 \uc11c\uc11c\ud788 \ubb3b\ud788\uae30 \uc2dc\uc791\ud558\uc600\uace0, \uad00\ub828\ub41c \uae30\uc0ac\ub3c4 \uc904\uc5b4\ub4e4\uae30 \uc2dc\uc791\ud558\uc600\ub2e4. \uadf8\ub7ec\ub098 \uc740\ud1f4\uc2dd\uc744 \uce58\ub974\ub294 \ud604\ub300 \uc720\ub2c8\ucf58\uc2a4 \uc18c\uc18d \ub0b4\uc57c\uc218 \uc5fc\uacbd\uc5fd(\ud604 SK \uc640\uc774\ubc88\uc2a4 \ub2e8\uc7a5)\uc774 \uc790\uc2e0\uc758 \ud648\ud398\uc774\uc9c0\ub97c \ud1b5\ud574 \uc57c\uad6c\uc6a9\ud488 \uacbd\ub9e4\ub85c \ubaa8\uc740 \uc218\uc775\uae08 \uc911 500\ub9cc \uc6d0\uc744 \ubcd1\uc0c1\uc5d0 \ub204\uc6cc \uc788\ub294 \uc784\uc218\ud601\uc758 \uce58\ub8cc\ube44\ub85c \uc804\ub2ec\ud574 \ud654\uc81c\uac00 \ub418\uc5c8\uace0, \uc791\uc740 \ub3c4\uc6c0\uc774\ub77c\ub3c4 \uc8fc\uae30 \uc704\ud574, \ub86f\ub370 \uc790\uc774\uc5b8\uce20\ub294 \uadf8\uac00 \uc4f0\ub7ec\uc9c4 \ub0a0\uc778 4\uc6d4 18\uc77c \uc744 \uc784\uc218\ud601\uc758 \ub0a0\ub85c \uc815\ud558\uace0 4\uc6d4 18\uc77c\uc5d0 \uc5f4\ub9ac\ub294 \uacbd\uae30\uc758 \uc218\uc775\uae08\uc758 \uc804\uc561\uc744 \uc784\uc218\ud601\uc758 \uce58\ub8cc\ube44\ub85c \uc804\ud558\uace0, \uc784\uc218\ud601\uc758 \uc544\ub4e4\uc774 \uc2dc\uad6c\ub97c \ud558\uac8c \ub418\uc5c8\ub2e4. \ubd80\uc0b0\uc5d0\uc11c \ub86f\ub370 \uc790\uc774\uc5b8\uce20\uc640 \uac19\uc740 \uc5f0\uace0\uc9c0\ub97c \uc4f0\ub294 \ucd95\uad6c \ud300 \ubd80\uc0b0 \uc544\uc774\ud30c\ud06c\ub294 \uc784\uc218\ud601\uc758 \ub0a0\uc744 \ub9de\uc544 \uc784\uc218\ud601\uc744 \uc704\ud574 \uce58\ub8cc\ube44\ub97c \ubcf4\ub0b4\uae30\ub85c \ud588\ub2e4. \uc774\ud6c4, \ub2f9\uc2dc \ub86f\ub370 \uc790\uc774\uc5b8\uce20\uc758 \uac10\ub3c5\uc774\uc5c8\ub358 \uace0 \uae40\uba85\uc131 \uac10\ub3c5\uc774 \uc62c\uc2a4\ud0c0\uc804 \ud589\uc0ac\uac00 \ud55c\ucc3d\uc774\ub358 \uc7a0\uc2e4\uc57c\uad6c\uc7a5\uc5d0\uc11c \ub86f\ub370 \uc790\uc774\uc5b8\uce20 \ud32c\ud074\ub7fd\uc774 \uc758\uc2dd \ubd88\uba85\uc73c\ub85c \ubcd1\uc0c1\uc5d0 \ub204\uc6cc \uc788\ub294 \uc784\uc218\ud601\uc744 \uaca9\ub824\ud558\uae30 \uc704\ud55c \uc11c\uba85\uc744 \ud588\uc744 \ub2f9\uc2dc '\uc784\uc218\ud601 \uc120\uc218, \uc5b8\uc81c \uadf8\ub77c\uc6b4\ub4dc\uc5d0\uc11c \ub9cc\ub098\uac8c \ub420\uae4c\uc694, \uadf8 \ub0a0\uc744 \uae30\ub2e4\ub9ac\uaca0\uc18c'\ub77c\uace0 \uc801\uc740 \uc801\uc774 \uc788\uc5c8\ub2e4. \ud558\uc9c0\ub9cc \uadf8 \ud574 7\uc6d4 24\uc77c \uae40\uba85\uc131 \uac10\ub3c5\uc774 \uc2ec\uc7a5\ubcd1\uc73c\ub85c \ud0c0\uacc4\ud55c \ub4a4, \uace0 \uae40\uba85\uc131 \uac10\ub3c5\uacfc \uc784\uc218\ud601\uc774 \ud55c \uc57d\uc18d\uc740 \uc9c0\ucf1c\uc9c0\uc9c0 \uc54a\uac8c \ub418\uc5c8\uace0, \uc784\uc218\ud601\uc744 \uc704\ud55c \uce58\ub8cc\uae08\uc744 \ubaa8\uc73c\uc790\ub294 \uc6b4\ub3d9\uc774 \uc7a0\uc2dc \uc77c\uc5b4\ub0ac\uc5c8\ub2e4.", "paragraph_answer": "2001\ub144\uc774 \ub418\uc790 1\ub144\uc774\ub098 \uc9c0\ub09c \uc784\uc218\ud601 \uc0ac\uac74\uc740 \uc11c\uc11c\ud788 \ubb3b\ud788\uae30 \uc2dc\uc791\ud558\uc600\uace0, \uad00\ub828\ub41c \uae30\uc0ac\ub3c4 \uc904\uc5b4\ub4e4\uae30 \uc2dc\uc791\ud558\uc600\ub2e4. \uadf8\ub7ec\ub098 \uc740\ud1f4\uc2dd\uc744 \uce58\ub974\ub294 \ud604\ub300 \uc720\ub2c8\ucf58\uc2a4 \uc18c\uc18d \ub0b4\uc57c\uc218 \uc5fc\uacbd\uc5fd(\ud604 SK \uc640\uc774\ubc88\uc2a4 \ub2e8\uc7a5)\uc774 \uc790\uc2e0\uc758 \ud648\ud398\uc774\uc9c0\ub97c \ud1b5\ud574 \uc57c\uad6c\uc6a9\ud488 \uacbd\ub9e4\ub85c \ubaa8\uc740 \uc218\uc775\uae08 \uc911 500\ub9cc \uc6d0\uc744 \ubcd1\uc0c1\uc5d0 \ub204\uc6cc \uc788\ub294 \uc784\uc218\ud601\uc758 \uce58\ub8cc\ube44\ub85c \uc804\ub2ec\ud574 \ud654\uc81c\uac00 \ub418\uc5c8\uace0, \uc791\uc740 \ub3c4\uc6c0\uc774\ub77c\ub3c4 \uc8fc\uae30 \uc704\ud574, \ub86f\ub370 \uc790\uc774\uc5b8\uce20\ub294 \uadf8\uac00 \uc4f0\ub7ec\uc9c4 \ub0a0\uc778 4\uc6d4 18\uc77c \uc744 \uc784\uc218\ud601\uc758 \ub0a0\ub85c \uc815\ud558\uace0 4\uc6d4 18\uc77c\uc5d0 \uc5f4\ub9ac\ub294 \uacbd\uae30\uc758 \uc218\uc775\uae08\uc758 \uc804\uc561\uc744 \uc784\uc218\ud601\uc758 \uce58\ub8cc\ube44\ub85c \uc804\ud558\uace0, \uc784\uc218\ud601\uc758 \uc544\ub4e4\uc774 \uc2dc\uad6c\ub97c \ud558\uac8c \ub418\uc5c8\ub2e4. \ubd80\uc0b0\uc5d0\uc11c \ub86f\ub370 \uc790\uc774\uc5b8\uce20\uc640 \uac19\uc740 \uc5f0\uace0\uc9c0\ub97c \uc4f0\ub294 \ucd95\uad6c \ud300 \ubd80\uc0b0 \uc544\uc774\ud30c\ud06c\ub294 \uc784\uc218\ud601\uc758 \ub0a0\uc744 \ub9de\uc544 \uc784\uc218\ud601\uc744 \uc704\ud574 \uce58\ub8cc\ube44\ub97c \ubcf4\ub0b4\uae30\ub85c \ud588\ub2e4. \uc774\ud6c4, \ub2f9\uc2dc \ub86f\ub370 \uc790\uc774\uc5b8\uce20\uc758 \uac10\ub3c5\uc774\uc5c8\ub358 \uace0 \uae40\uba85\uc131 \uac10\ub3c5\uc774 \uc62c\uc2a4\ud0c0\uc804 \ud589\uc0ac\uac00 \ud55c\ucc3d\uc774\ub358 \uc7a0\uc2e4\uc57c\uad6c\uc7a5\uc5d0\uc11c \ub86f\ub370 \uc790\uc774\uc5b8\uce20 \ud32c\ud074\ub7fd\uc774 \uc758\uc2dd \ubd88\uba85\uc73c\ub85c \ubcd1\uc0c1\uc5d0 \ub204\uc6cc \uc788\ub294 \uc784\uc218\ud601\uc744 \uaca9\ub824\ud558\uae30 \uc704\ud55c \uc11c\uba85\uc744 \ud588\uc744 \ub2f9\uc2dc '\uc784\uc218\ud601 \uc120\uc218, \uc5b8\uc81c \uadf8\ub77c\uc6b4\ub4dc\uc5d0\uc11c \ub9cc\ub098\uac8c \ub420\uae4c\uc694, \uadf8 \ub0a0\uc744 \uae30\ub2e4\ub9ac\uaca0\uc18c'\ub77c\uace0 \uc801\uc740 \uc801\uc774 \uc788\uc5c8\ub2e4. \ud558\uc9c0\ub9cc \uadf8 \ud574 7\uc6d4 24\uc77c \uae40\uba85\uc131 \uac10\ub3c5\uc774 \uc2ec\uc7a5\ubcd1\uc73c\ub85c \ud0c0\uacc4\ud55c \ub4a4, \uace0 \uae40\uba85\uc131 \uac10\ub3c5\uacfc \uc784\uc218\ud601\uc774 \ud55c \uc57d\uc18d\uc740 \uc9c0\ucf1c\uc9c0\uc9c0 \uc54a\uac8c \ub418\uc5c8\uace0, \uc784\uc218\ud601\uc744 \uc704\ud55c \uce58\ub8cc\uae08\uc744 \ubaa8\uc73c\uc790\ub294 \uc6b4\ub3d9\uc774 \uc7a0\uc2dc \uc77c\uc5b4\ub0ac\uc5c8\ub2e4.", "sentence_answer": "\ub0a0\uc778 4\uc6d4 18\uc77c \uc744 \uc784\uc218\ud601\uc758 \ub0a0\ub85c \uc815\ud558\uace0 4\uc6d4 18\uc77c\uc5d0 \uc5f4\ub9ac\ub294 \uacbd\uae30\uc758 \uc218\uc775\uae08\uc758 \uc804\uc561\uc744 \uc784\uc218\ud601\uc758 \uce58\ub8cc\ube44\ub85c \uc804\ud558\uace0, \uc784\uc218\ud601\uc758 \uc544\ub4e4\uc774 \uc2dc\uad6c\ub97c \ud558\uac8c \ub418\uc5c8\ub2e4.", "paragraph_id": "6176094-3-0", "paragraph_question": "question: \uc784\uc218\ud601\uc758 \ub0a0 \uc740 \uba87\uc6d4 \uba87\uc77c\uc778\uac00?, context: 2001\ub144\uc774 \ub418\uc790 1\ub144\uc774\ub098 \uc9c0\ub09c \uc784\uc218\ud601 \uc0ac\uac74\uc740 \uc11c\uc11c\ud788 \ubb3b\ud788\uae30 \uc2dc\uc791\ud558\uc600\uace0, \uad00\ub828\ub41c \uae30\uc0ac\ub3c4 \uc904\uc5b4\ub4e4\uae30 \uc2dc\uc791\ud558\uc600\ub2e4. \uadf8\ub7ec\ub098 \uc740\ud1f4\uc2dd\uc744 \uce58\ub974\ub294 \ud604\ub300 \uc720\ub2c8\ucf58\uc2a4 \uc18c\uc18d \ub0b4\uc57c\uc218 \uc5fc\uacbd\uc5fd(\ud604 SK \uc640\uc774\ubc88\uc2a4 \ub2e8\uc7a5)\uc774 \uc790\uc2e0\uc758 \ud648\ud398\uc774\uc9c0\ub97c \ud1b5\ud574 \uc57c\uad6c\uc6a9\ud488 \uacbd\ub9e4\ub85c \ubaa8\uc740 \uc218\uc775\uae08 \uc911 500\ub9cc \uc6d0\uc744 \ubcd1\uc0c1\uc5d0 \ub204\uc6cc \uc788\ub294 \uc784\uc218\ud601\uc758 \uce58\ub8cc\ube44\ub85c \uc804\ub2ec\ud574 \ud654\uc81c\uac00 \ub418\uc5c8\uace0, \uc791\uc740 \ub3c4\uc6c0\uc774\ub77c\ub3c4 \uc8fc\uae30 \uc704\ud574, \ub86f\ub370 \uc790\uc774\uc5b8\uce20\ub294 \uadf8\uac00 \uc4f0\ub7ec\uc9c4 \ub0a0\uc778 4\uc6d4 18\uc77c\uc744 \uc784\uc218\ud601\uc758 \ub0a0\ub85c \uc815\ud558\uace0 4\uc6d4 18\uc77c\uc5d0 \uc5f4\ub9ac\ub294 \uacbd\uae30\uc758 \uc218\uc775\uae08\uc758 \uc804\uc561\uc744 \uc784\uc218\ud601\uc758 \uce58\ub8cc\ube44\ub85c \uc804\ud558\uace0, \uc784\uc218\ud601\uc758 \uc544\ub4e4\uc774 \uc2dc\uad6c\ub97c \ud558\uac8c \ub418\uc5c8\ub2e4. \ubd80\uc0b0\uc5d0\uc11c \ub86f\ub370 \uc790\uc774\uc5b8\uce20\uc640 \uac19\uc740 \uc5f0\uace0\uc9c0\ub97c \uc4f0\ub294 \ucd95\uad6c \ud300 \ubd80\uc0b0 \uc544\uc774\ud30c\ud06c\ub294 \uc784\uc218\ud601\uc758 \ub0a0\uc744 \ub9de\uc544 \uc784\uc218\ud601\uc744 \uc704\ud574 \uce58\ub8cc\ube44\ub97c \ubcf4\ub0b4\uae30\ub85c \ud588\ub2e4. \uc774\ud6c4, \ub2f9\uc2dc \ub86f\ub370 \uc790\uc774\uc5b8\uce20\uc758 \uac10\ub3c5\uc774\uc5c8\ub358 \uace0 \uae40\uba85\uc131 \uac10\ub3c5\uc774 \uc62c\uc2a4\ud0c0\uc804 \ud589\uc0ac\uac00 \ud55c\ucc3d\uc774\ub358 \uc7a0\uc2e4\uc57c\uad6c\uc7a5\uc5d0\uc11c \ub86f\ub370 \uc790\uc774\uc5b8\uce20 \ud32c\ud074\ub7fd\uc774 \uc758\uc2dd \ubd88\uba85\uc73c\ub85c \ubcd1\uc0c1\uc5d0 \ub204\uc6cc \uc788\ub294 \uc784\uc218\ud601\uc744 \uaca9\ub824\ud558\uae30 \uc704\ud55c \uc11c\uba85\uc744 \ud588\uc744 \ub2f9\uc2dc '\uc784\uc218\ud601 \uc120\uc218, \uc5b8\uc81c \uadf8\ub77c\uc6b4\ub4dc\uc5d0\uc11c \ub9cc\ub098\uac8c \ub420\uae4c\uc694, \uadf8 \ub0a0\uc744 \uae30\ub2e4\ub9ac\uaca0\uc18c'\ub77c\uace0 \uc801\uc740 \uc801\uc774 \uc788\uc5c8\ub2e4. \ud558\uc9c0\ub9cc \uadf8 \ud574 7\uc6d4 24\uc77c \uae40\uba85\uc131 \uac10\ub3c5\uc774 \uc2ec\uc7a5\ubcd1\uc73c\ub85c \ud0c0\uacc4\ud55c \ub4a4, \uace0 \uae40\uba85\uc131 \uac10\ub3c5\uacfc \uc784\uc218\ud601\uc774 \ud55c \uc57d\uc18d\uc740 \uc9c0\ucf1c\uc9c0\uc9c0 \uc54a\uac8c \ub418\uc5c8\uace0, \uc784\uc218\ud601\uc744 \uc704\ud55c \uce58\ub8cc\uae08\uc744 \ubaa8\uc73c\uc790\ub294 \uc6b4\ub3d9\uc774 \uc7a0\uc2dc \uc77c\uc5b4\ub0ac\uc5c8\ub2e4."} {"question": "2010\ub144 \ubaa8\uc2a4\ud06c\ubc14 \uc9c0\ud558\ucca0 \ud3ed\ud0c4 \ud14c\ub7ec \ub2f9\uc2dc \ub7ec\uc2dc\uc544\uc758 \ucd1d\ub9ac\ub294?", "paragraph": "\ub7ec\uc2dc\uc544 \uac80\ucc30\ub2f9\uad6d\uc740 \ub8e8\ubc50\uce74 \uc5ed \ud3ed\ubc1c\uc5d0\uc11c 4kg, \ud30c\ub974\ud06c \ucfe8\ud22c\ub9ac \uc5ed \ud3ed\ubc1c\uc5d0\uc11c 1.5~2kg \uc815\ub3c4\uc758 \ud50c\ub77c\uc2a4\ud2f1 \ud3ed\ud0c4\uc774 \uc0ac\uc6a9\ub418\uc5c8\uc744 \uac83\uc774\ub77c\uace0 \ubc1c\ud45c\ud588\ub2e4. \uc0ac\uac74 \ub2f9\uc77c \uc804\ucca0\uc5ed \ubc29\ubc94\uce74\uba54\ub77c\uc5d0 \uc6a9\uc758\uc790\uc758 \ubaa8\uc2b5\uc774 \ud3ec\ucc29\ub418\uc5c8\ub2e4. \ub7ec\uc2dc\uc544 \uac80\ucc30\uc774 \uc870\uc0ac\ub97c \uc2dc\uc791\ud588\uace0, \uadf8 \uacb0\uacfc \uc758\ud55c \uc790\ud3ed\ud14c\ub7ec\uc5c8\ub358 \uac83\uc774 \ubc1d\ud600\uc84c\ub2e4. \uadf8\ub9ac\uace0 \uc54c\ub809\uc0b0\ub4dc\ub974 \ubcf4\ub974\ud2b8\ub2c8\ucf54\ud504 \uc5f0\ubc29\ubcf4\uc548\uad6d \uad6d\uc7a5\uc740 \ub4dc\ubbf8\ud2b8\ub9ac \uba54\ub4dc\ubca0\ub370\ud504 \ub300\ud1b5\ub839\uc5d0\uac8c \"\ubd81\uce85\uce74\uc2a4 \uc9c0\uc5ed\uacfc \uad00\ub828\ub41c \uc5ec\uc131 \uc790\ud3ed \ud14c\ub7ec\"\ub77c\uace0 \ubcf4\uace0\ud588\ub2e4. \uba54\ub4dc\ubca0\ub370\ud504 \ub300\ud1b5\ub839\uc740 \"\ud14c\ub7ec\ub9ac\uc2a4\ud2b8\uc5d0 \ub300\ud55c \uc791\uc804\uc740 \ub9c8\uc9c0\ub9c9\uae4c\uc9c0 \uae34\uc7a5\uc744 \ub2a6\ucd94\uc9c0 \ub9c8\ub77c\"\uba70 \ud14c\ub7ec \ud65c\ub3d9 \uac10\uc2dc\ub97c \uac15\ud654\ud560 \uac83\uc744 \uc9c0\uc2dc\ud588\ub2e4. \uba54\ub4dc\ubca0\ub370\ud504 \ub300\ud1b5\ub839\uc740 \uc0ac\uac74 \ubc1c\uc0dd \ub2f9\uc77c \uc800\ub141\uc5d0 \uc0ac\uac74\ud604\uc7a5\uc778 \ub8e8\ubc50\uce74 \uc5ed\uc744 \ubc29\ubb38\ud574 \uc601\uc804\uc5d0 \uaf43\uc744 \uc62c\ub9ac\uba70 \"\uadf8\ub4e4\uc744 \ucd94\uc801\ud574\uc11c \ub9d0\uc0b4\uc2dc\ud0a4\uaca0\ub2e4\"\ub77c\uace0 \ub367\ubd99\uc600\ub2e4. \uba54\ub4dc\ubca0\ub370\ud504 \ub300\ud1b5\ub839\uc740 \ub7ec\uc2dc\uc544 \uc804\uc5ed\uc758 \uad50\ud1b5\ub9dd\uc5d0 \ub300\ud55c \ud14c\ub7ec\ubc29\uc9c0\ub97c \uc704\ud574 \uacbd\ube44\ub97c \uac15\ud654\ud558\ub294 \ub300\ud1b5\ub839\ubc95\uc5d0 \uc11c\uba85\ud558\uace0 \ub2f9\uad6d\uc758 \uc808\ucc98\ud55c \uc870\uc0ac\ub97c \uc9c0\uc2dc\ud588\ub2e4. \ub610\ud55c \ub3d9\uc544\uc2dc\uc544\ub97c \ubc29\ubb38\ud558\uace0 \uc788\uc5c8\ub358 \ube14\ub77c\ub514\ubbf8\ub974 \ud478\ud2f4 \ucd1d\ub9ac\ub294 \ud14c\ub7ec \uc18c\uc2dd\uc744 \ubcf4\uace0\ubc1b\uace0 \uc11c\ub458\ub7ec \ubaa8\uc2a4\ud06c\ubc14\ub85c \ub3cc\uc544\uac14\ub2e4.", "answer": "\ud478\ud2f4", "sentence": "\ub610\ud55c \ub3d9\uc544\uc2dc\uc544\ub97c \ubc29\ubb38\ud558\uace0 \uc788\uc5c8\ub358 \ube14\ub77c\ub514\ubbf8\ub974 \ud478\ud2f4 \ucd1d\ub9ac\ub294 \ud14c\ub7ec \uc18c\uc2dd\uc744 \ubcf4\uace0\ubc1b\uace0 \uc11c\ub458\ub7ec \ubaa8\uc2a4\ud06c\ubc14\ub85c \ub3cc\uc544\uac14\ub2e4.", "paragraph_sentence": "\ub7ec\uc2dc\uc544 \uac80\ucc30\ub2f9\uad6d\uc740 \ub8e8\ubc50\uce74 \uc5ed \ud3ed\ubc1c\uc5d0\uc11c 4kg, \ud30c\ub974\ud06c \ucfe8\ud22c\ub9ac \uc5ed \ud3ed\ubc1c\uc5d0\uc11c 1.5~2kg \uc815\ub3c4\uc758 \ud50c\ub77c\uc2a4\ud2f1 \ud3ed\ud0c4\uc774 \uc0ac\uc6a9\ub418\uc5c8\uc744 \uac83\uc774\ub77c\uace0 \ubc1c\ud45c\ud588\ub2e4. \uc0ac\uac74 \ub2f9\uc77c \uc804\ucca0\uc5ed \ubc29\ubc94\uce74\uba54\ub77c\uc5d0 \uc6a9\uc758\uc790\uc758 \ubaa8\uc2b5\uc774 \ud3ec\ucc29\ub418\uc5c8\ub2e4. \ub7ec\uc2dc\uc544 \uac80\ucc30\uc774 \uc870\uc0ac\ub97c \uc2dc\uc791\ud588\uace0, \uadf8 \uacb0\uacfc \uc758\ud55c \uc790\ud3ed\ud14c\ub7ec\uc5c8\ub358 \uac83\uc774 \ubc1d\ud600\uc84c\ub2e4. \uadf8\ub9ac\uace0 \uc54c\ub809\uc0b0\ub4dc\ub974 \ubcf4\ub974\ud2b8\ub2c8\ucf54\ud504 \uc5f0\ubc29\ubcf4\uc548\uad6d \uad6d\uc7a5\uc740 \ub4dc\ubbf8\ud2b8\ub9ac \uba54\ub4dc\ubca0\ub370\ud504 \ub300\ud1b5\ub839\uc5d0\uac8c \"\ubd81\uce85\uce74\uc2a4 \uc9c0\uc5ed\uacfc \uad00\ub828\ub41c \uc5ec\uc131 \uc790\ud3ed \ud14c\ub7ec\"\ub77c\uace0 \ubcf4\uace0\ud588\ub2e4. \uba54\ub4dc\ubca0\ub370\ud504 \ub300\ud1b5\ub839\uc740 \"\ud14c\ub7ec\ub9ac\uc2a4\ud2b8\uc5d0 \ub300\ud55c \uc791\uc804\uc740 \ub9c8\uc9c0\ub9c9\uae4c\uc9c0 \uae34\uc7a5\uc744 \ub2a6\ucd94\uc9c0 \ub9c8\ub77c\"\uba70 \ud14c\ub7ec \ud65c\ub3d9 \uac10\uc2dc\ub97c \uac15\ud654\ud560 \uac83\uc744 \uc9c0\uc2dc\ud588\ub2e4. \uba54\ub4dc\ubca0\ub370\ud504 \ub300\ud1b5\ub839\uc740 \uc0ac\uac74 \ubc1c\uc0dd \ub2f9\uc77c \uc800\ub141\uc5d0 \uc0ac\uac74\ud604\uc7a5\uc778 \ub8e8\ubc50\uce74 \uc5ed\uc744 \ubc29\ubb38\ud574 \uc601\uc804\uc5d0 \uaf43\uc744 \uc62c\ub9ac\uba70 \"\uadf8\ub4e4\uc744 \ucd94\uc801\ud574\uc11c \ub9d0\uc0b4\uc2dc\ud0a4\uaca0\ub2e4\"\ub77c\uace0 \ub367\ubd99\uc600\ub2e4. \uba54\ub4dc\ubca0\ub370\ud504 \ub300\ud1b5\ub839\uc740 \ub7ec\uc2dc\uc544 \uc804\uc5ed\uc758 \uad50\ud1b5\ub9dd\uc5d0 \ub300\ud55c \ud14c\ub7ec\ubc29\uc9c0\ub97c \uc704\ud574 \uacbd\ube44\ub97c \uac15\ud654\ud558\ub294 \ub300\ud1b5\ub839\ubc95\uc5d0 \uc11c\uba85\ud558\uace0 \ub2f9\uad6d\uc758 \uc808\ucc98\ud55c \uc870\uc0ac\ub97c \uc9c0\uc2dc\ud588\ub2e4. \ub610\ud55c \ub3d9\uc544\uc2dc\uc544\ub97c \ubc29\ubb38\ud558\uace0 \uc788\uc5c8\ub358 \ube14\ub77c\ub514\ubbf8\ub974 \ud478\ud2f4 \ucd1d\ub9ac\ub294 \ud14c\ub7ec \uc18c\uc2dd\uc744 \ubcf4\uace0\ubc1b\uace0 \uc11c\ub458\ub7ec \ubaa8\uc2a4\ud06c\ubc14\ub85c \ub3cc\uc544\uac14\ub2e4. ", "paragraph_answer": "\ub7ec\uc2dc\uc544 \uac80\ucc30\ub2f9\uad6d\uc740 \ub8e8\ubc50\uce74 \uc5ed \ud3ed\ubc1c\uc5d0\uc11c 4kg, \ud30c\ub974\ud06c \ucfe8\ud22c\ub9ac \uc5ed \ud3ed\ubc1c\uc5d0\uc11c 1.5~2kg \uc815\ub3c4\uc758 \ud50c\ub77c\uc2a4\ud2f1 \ud3ed\ud0c4\uc774 \uc0ac\uc6a9\ub418\uc5c8\uc744 \uac83\uc774\ub77c\uace0 \ubc1c\ud45c\ud588\ub2e4. \uc0ac\uac74 \ub2f9\uc77c \uc804\ucca0\uc5ed \ubc29\ubc94\uce74\uba54\ub77c\uc5d0 \uc6a9\uc758\uc790\uc758 \ubaa8\uc2b5\uc774 \ud3ec\ucc29\ub418\uc5c8\ub2e4. \ub7ec\uc2dc\uc544 \uac80\ucc30\uc774 \uc870\uc0ac\ub97c \uc2dc\uc791\ud588\uace0, \uadf8 \uacb0\uacfc \uc758\ud55c \uc790\ud3ed\ud14c\ub7ec\uc5c8\ub358 \uac83\uc774 \ubc1d\ud600\uc84c\ub2e4. \uadf8\ub9ac\uace0 \uc54c\ub809\uc0b0\ub4dc\ub974 \ubcf4\ub974\ud2b8\ub2c8\ucf54\ud504 \uc5f0\ubc29\ubcf4\uc548\uad6d \uad6d\uc7a5\uc740 \ub4dc\ubbf8\ud2b8\ub9ac \uba54\ub4dc\ubca0\ub370\ud504 \ub300\ud1b5\ub839\uc5d0\uac8c \"\ubd81\uce85\uce74\uc2a4 \uc9c0\uc5ed\uacfc \uad00\ub828\ub41c \uc5ec\uc131 \uc790\ud3ed \ud14c\ub7ec\"\ub77c\uace0 \ubcf4\uace0\ud588\ub2e4. \uba54\ub4dc\ubca0\ub370\ud504 \ub300\ud1b5\ub839\uc740 \"\ud14c\ub7ec\ub9ac\uc2a4\ud2b8\uc5d0 \ub300\ud55c \uc791\uc804\uc740 \ub9c8\uc9c0\ub9c9\uae4c\uc9c0 \uae34\uc7a5\uc744 \ub2a6\ucd94\uc9c0 \ub9c8\ub77c\"\uba70 \ud14c\ub7ec \ud65c\ub3d9 \uac10\uc2dc\ub97c \uac15\ud654\ud560 \uac83\uc744 \uc9c0\uc2dc\ud588\ub2e4. \uba54\ub4dc\ubca0\ub370\ud504 \ub300\ud1b5\ub839\uc740 \uc0ac\uac74 \ubc1c\uc0dd \ub2f9\uc77c \uc800\ub141\uc5d0 \uc0ac\uac74\ud604\uc7a5\uc778 \ub8e8\ubc50\uce74 \uc5ed\uc744 \ubc29\ubb38\ud574 \uc601\uc804\uc5d0 \uaf43\uc744 \uc62c\ub9ac\uba70 \"\uadf8\ub4e4\uc744 \ucd94\uc801\ud574\uc11c \ub9d0\uc0b4\uc2dc\ud0a4\uaca0\ub2e4\"\ub77c\uace0 \ub367\ubd99\uc600\ub2e4. \uba54\ub4dc\ubca0\ub370\ud504 \ub300\ud1b5\ub839\uc740 \ub7ec\uc2dc\uc544 \uc804\uc5ed\uc758 \uad50\ud1b5\ub9dd\uc5d0 \ub300\ud55c \ud14c\ub7ec\ubc29\uc9c0\ub97c \uc704\ud574 \uacbd\ube44\ub97c \uac15\ud654\ud558\ub294 \ub300\ud1b5\ub839\ubc95\uc5d0 \uc11c\uba85\ud558\uace0 \ub2f9\uad6d\uc758 \uc808\ucc98\ud55c \uc870\uc0ac\ub97c \uc9c0\uc2dc\ud588\ub2e4. \ub610\ud55c \ub3d9\uc544\uc2dc\uc544\ub97c \ubc29\ubb38\ud558\uace0 \uc788\uc5c8\ub358 \ube14\ub77c\ub514\ubbf8\ub974 \ud478\ud2f4 \ucd1d\ub9ac\ub294 \ud14c\ub7ec \uc18c\uc2dd\uc744 \ubcf4\uace0\ubc1b\uace0 \uc11c\ub458\ub7ec \ubaa8\uc2a4\ud06c\ubc14\ub85c \ub3cc\uc544\uac14\ub2e4.", "sentence_answer": "\ub610\ud55c \ub3d9\uc544\uc2dc\uc544\ub97c \ubc29\ubb38\ud558\uace0 \uc788\uc5c8\ub358 \ube14\ub77c\ub514\ubbf8\ub974 \ud478\ud2f4 \ucd1d\ub9ac\ub294 \ud14c\ub7ec \uc18c\uc2dd\uc744 \ubcf4\uace0\ubc1b\uace0 \uc11c\ub458\ub7ec \ubaa8\uc2a4\ud06c\ubc14\ub85c \ub3cc\uc544\uac14\ub2e4.", "paragraph_id": "6500129-2-2", "paragraph_question": "question: 2010\ub144 \ubaa8\uc2a4\ud06c\ubc14 \uc9c0\ud558\ucca0 \ud3ed\ud0c4 \ud14c\ub7ec \ub2f9\uc2dc \ub7ec\uc2dc\uc544\uc758 \ucd1d\ub9ac\ub294?, context: \ub7ec\uc2dc\uc544 \uac80\ucc30\ub2f9\uad6d\uc740 \ub8e8\ubc50\uce74 \uc5ed \ud3ed\ubc1c\uc5d0\uc11c 4kg, \ud30c\ub974\ud06c \ucfe8\ud22c\ub9ac \uc5ed \ud3ed\ubc1c\uc5d0\uc11c 1.5~2kg \uc815\ub3c4\uc758 \ud50c\ub77c\uc2a4\ud2f1 \ud3ed\ud0c4\uc774 \uc0ac\uc6a9\ub418\uc5c8\uc744 \uac83\uc774\ub77c\uace0 \ubc1c\ud45c\ud588\ub2e4. \uc0ac\uac74 \ub2f9\uc77c \uc804\ucca0\uc5ed \ubc29\ubc94\uce74\uba54\ub77c\uc5d0 \uc6a9\uc758\uc790\uc758 \ubaa8\uc2b5\uc774 \ud3ec\ucc29\ub418\uc5c8\ub2e4. \ub7ec\uc2dc\uc544 \uac80\ucc30\uc774 \uc870\uc0ac\ub97c \uc2dc\uc791\ud588\uace0, \uadf8 \uacb0\uacfc \uc758\ud55c \uc790\ud3ed\ud14c\ub7ec\uc5c8\ub358 \uac83\uc774 \ubc1d\ud600\uc84c\ub2e4. \uadf8\ub9ac\uace0 \uc54c\ub809\uc0b0\ub4dc\ub974 \ubcf4\ub974\ud2b8\ub2c8\ucf54\ud504 \uc5f0\ubc29\ubcf4\uc548\uad6d \uad6d\uc7a5\uc740 \ub4dc\ubbf8\ud2b8\ub9ac \uba54\ub4dc\ubca0\ub370\ud504 \ub300\ud1b5\ub839\uc5d0\uac8c \"\ubd81\uce85\uce74\uc2a4 \uc9c0\uc5ed\uacfc \uad00\ub828\ub41c \uc5ec\uc131 \uc790\ud3ed \ud14c\ub7ec\"\ub77c\uace0 \ubcf4\uace0\ud588\ub2e4. \uba54\ub4dc\ubca0\ub370\ud504 \ub300\ud1b5\ub839\uc740 \"\ud14c\ub7ec\ub9ac\uc2a4\ud2b8\uc5d0 \ub300\ud55c \uc791\uc804\uc740 \ub9c8\uc9c0\ub9c9\uae4c\uc9c0 \uae34\uc7a5\uc744 \ub2a6\ucd94\uc9c0 \ub9c8\ub77c\"\uba70 \ud14c\ub7ec \ud65c\ub3d9 \uac10\uc2dc\ub97c \uac15\ud654\ud560 \uac83\uc744 \uc9c0\uc2dc\ud588\ub2e4. \uba54\ub4dc\ubca0\ub370\ud504 \ub300\ud1b5\ub839\uc740 \uc0ac\uac74 \ubc1c\uc0dd \ub2f9\uc77c \uc800\ub141\uc5d0 \uc0ac\uac74\ud604\uc7a5\uc778 \ub8e8\ubc50\uce74 \uc5ed\uc744 \ubc29\ubb38\ud574 \uc601\uc804\uc5d0 \uaf43\uc744 \uc62c\ub9ac\uba70 \"\uadf8\ub4e4\uc744 \ucd94\uc801\ud574\uc11c \ub9d0\uc0b4\uc2dc\ud0a4\uaca0\ub2e4\"\ub77c\uace0 \ub367\ubd99\uc600\ub2e4. \uba54\ub4dc\ubca0\ub370\ud504 \ub300\ud1b5\ub839\uc740 \ub7ec\uc2dc\uc544 \uc804\uc5ed\uc758 \uad50\ud1b5\ub9dd\uc5d0 \ub300\ud55c \ud14c\ub7ec\ubc29\uc9c0\ub97c \uc704\ud574 \uacbd\ube44\ub97c \uac15\ud654\ud558\ub294 \ub300\ud1b5\ub839\ubc95\uc5d0 \uc11c\uba85\ud558\uace0 \ub2f9\uad6d\uc758 \uc808\ucc98\ud55c \uc870\uc0ac\ub97c \uc9c0\uc2dc\ud588\ub2e4. \ub610\ud55c \ub3d9\uc544\uc2dc\uc544\ub97c \ubc29\ubb38\ud558\uace0 \uc788\uc5c8\ub358 \ube14\ub77c\ub514\ubbf8\ub974 \ud478\ud2f4 \ucd1d\ub9ac\ub294 \ud14c\ub7ec \uc18c\uc2dd\uc744 \ubcf4\uace0\ubc1b\uace0 \uc11c\ub458\ub7ec \ubaa8\uc2a4\ud06c\ubc14\ub85c \ub3cc\uc544\uac14\ub2e4."} {"question": "\uc911\uc138 \ucd5c\ud6c4\uc758 \ub300\uaddc\ubaa8 \uc2ed\uc790\uad70\uc744 \uc774\ub974\ub294 \ub9d0\uc740?", "paragraph": "\ub2c8\ucf54\ud3f4\ub9ac\uc2a4 \uc804\ud22c(\ubd88\uac00\ub9ac\uc544\uc5b4: \u0411\u0438\u0442\u043a\u0430 \u043f\u0440\u0438 \u041d\u0438\u043a\u043e\u043f\u043e\u043b, Bitka pri Nikopol;\ud130\ud0a4\uc5b4: Ni\u011fbolu Sava\u015f\u0131;\ud5dd\uac00\ub9ac\uc5b4: Nik\u00e1polyi Csata;\ub8e8\ub9c8\ub2c8\uc544\uc5b4: B\u0103t\u0103lia de la Nicopole)\ub294 1396\ub144 9\uc6d4 25\uc77c(\ud639\uc740 9\uc6d4 28\uc77c\uc774\ub780 \ub9d0\ub3c4 \uc788\ub2e4)\uc5d0 \ub3c4\ub098\uc6b0\uac15\ubcc0\uc758 \ub2c8\ucf54\ud3f4\ub9ac\uc2a4\uc5d0\uc11c \uc624\uc2a4\ub9cc \uc81c\uad6d\uc758 \ubc14\uc608\uc9c0\ub4dc 1\uc138(\uc81c\uc704:1389\ub144~1402\ub144)\uc640 \ud5dd\uac00\ub9ac \uc655 \uc9c0\uae30\uc2a4\ubb38\ud2b8\uac00 \uc774\ub044\ub294 \uc720\ub7fd\uc5f0\ud569(\ucc38\uac00\uc138\ub825:\ud5dd\uac00\ub9ac \uc655\uad6d, \uc2e0\uc131\ub85c\ub9c8\uc81c\uad6d, \ud504\ub791\uc2a4, \uc648\ub77c\ud0a4\uc544, \ud3f4\ub780\ub4dc, \uc789\uae00\ub79c\ub4dc, \uc2a4\ucf54\ud2c0\ub79c\ub4dc \uc655\uad6d, \uad6c \uc2a4\uc704\uc2a4 \uc5f0\ud569, \ud29c\ud2bc \uae30\uc0ac\ub2e8, \ubca0\ub124\uce58\uc544 \uacf5\ud654\uad6d, \uc81c\ub178\ubc14 \uacf5\ud654\uad6d, \uc131 \uc694\ud55c \uae30\uc0ac\ub2e8)\uc0ac\uc774\uc5d0\uc11c \uc77c\uc5b4\ub09c \uc804\ud22c\uc774\ub2e4. \ud754\ud788 \ub2c8\ucf54\ud3f4\ub9ac\uc2a4 \uc2ed\uc790\uad70\uc774\ub77c\uace0 \ubd88\ub9ac\uba70, \uc911\uc138 \ucd5c\ud6c4\uc758 \ub300\uaddc\ubaa8 \uc2ed\uc790\uad70\uc774\uc5c8\ub2e4. \uc804\ud22c \uacb0\uacfc\ub294 \uc624\uc2a4\ub9cc \uc81c\uad6d\uc758 \uacb0\uc815\uc801 \uc2b9\ub9ac\ub85c \ub05d\ub0ac\uace0, \ubc14\uc608\uc9c0\ub4dc 1\uc138\ub294 \uce74\uc774\ub85c\uc758 \ub9d8\ub8e8\ud06c \uc655\uc870 \ubcf4\ud638\ud558\uc5d0 \uc788\ub358 \uce7c\ub9ac\ud30c\ub85c\ubd80\ud130 \ub192\uc740 \ud3c9\uac00\ub97c \ubc1b\uc544 \uc220\ud0c4\uc758 \uce6d\ud638\ub97c \ud558\uc0ac\ubc1b\uc558\ub2e4.", "answer": "\ub2c8\ucf54\ud3f4\ub9ac\uc2a4 \uc2ed\uc790\uad70", "sentence": "\ud754\ud788 \ub2c8\ucf54\ud3f4\ub9ac\uc2a4 \uc2ed\uc790\uad70 \uc774\ub77c\uace0 \ubd88\ub9ac\uba70, \uc911\uc138 \ucd5c\ud6c4\uc758 \ub300\uaddc\ubaa8 \uc2ed\uc790\uad70\uc774\uc5c8\ub2e4.", "paragraph_sentence": "\ub2c8\ucf54\ud3f4\ub9ac\uc2a4 \uc804\ud22c(\ubd88\uac00\ub9ac\uc544\uc5b4: \u0411\u0438\u0442\u043a\u0430 \u043f\u0440\u0438 \u041d\u0438\u043a\u043e\u043f\u043e\u043b, Bitka pri Nikopol;\ud130\ud0a4\uc5b4: Ni\u011fbolu Sava\u015f\u0131;\ud5dd\uac00\ub9ac\uc5b4: Nik\u00e1polyi Csata;\ub8e8\ub9c8\ub2c8\uc544\uc5b4: B\u0103t\u0103lia de la Nicopole)\ub294 1396\ub144 9\uc6d4 25\uc77c(\ud639\uc740 9\uc6d4 28\uc77c\uc774\ub780 \ub9d0\ub3c4 \uc788\ub2e4)\uc5d0 \ub3c4\ub098\uc6b0\uac15\ubcc0\uc758 \ub2c8\ucf54\ud3f4\ub9ac\uc2a4\uc5d0\uc11c \uc624\uc2a4\ub9cc \uc81c\uad6d\uc758 \ubc14\uc608\uc9c0\ub4dc 1\uc138(\uc81c\uc704:1389\ub144~1402\ub144)\uc640 \ud5dd\uac00\ub9ac \uc655 \uc9c0\uae30\uc2a4\ubb38\ud2b8\uac00 \uc774\ub044\ub294 \uc720\ub7fd\uc5f0\ud569(\ucc38\uac00\uc138\ub825:\ud5dd\uac00\ub9ac \uc655\uad6d, \uc2e0\uc131\ub85c\ub9c8\uc81c\uad6d, \ud504\ub791\uc2a4, \uc648\ub77c\ud0a4\uc544, \ud3f4\ub780\ub4dc, \uc789\uae00\ub79c\ub4dc, \uc2a4\ucf54\ud2c0\ub79c\ub4dc \uc655\uad6d, \uad6c \uc2a4\uc704\uc2a4 \uc5f0\ud569, \ud29c\ud2bc \uae30\uc0ac\ub2e8, \ubca0\ub124\uce58\uc544 \uacf5\ud654\uad6d, \uc81c\ub178\ubc14 \uacf5\ud654\uad6d, \uc131 \uc694\ud55c \uae30\uc0ac\ub2e8)\uc0ac\uc774\uc5d0\uc11c \uc77c\uc5b4\ub09c \uc804\ud22c\uc774\ub2e4. \ud754\ud788 \ub2c8\ucf54\ud3f4\ub9ac\uc2a4 \uc2ed\uc790\uad70 \uc774\ub77c\uace0 \ubd88\ub9ac\uba70, \uc911\uc138 \ucd5c\ud6c4\uc758 \ub300\uaddc\ubaa8 \uc2ed\uc790\uad70\uc774\uc5c8\ub2e4. \uc804\ud22c \uacb0\uacfc\ub294 \uc624\uc2a4\ub9cc \uc81c\uad6d\uc758 \uacb0\uc815\uc801 \uc2b9\ub9ac\ub85c \ub05d\ub0ac\uace0, \ubc14\uc608\uc9c0\ub4dc 1\uc138\ub294 \uce74\uc774\ub85c\uc758 \ub9d8\ub8e8\ud06c \uc655\uc870 \ubcf4\ud638\ud558\uc5d0 \uc788\ub358 \uce7c\ub9ac\ud30c\ub85c\ubd80\ud130 \ub192\uc740 \ud3c9\uac00\ub97c \ubc1b\uc544 \uc220\ud0c4\uc758 \uce6d\ud638\ub97c \ud558\uc0ac\ubc1b\uc558\ub2e4.", "paragraph_answer": "\ub2c8\ucf54\ud3f4\ub9ac\uc2a4 \uc804\ud22c(\ubd88\uac00\ub9ac\uc544\uc5b4: \u0411\u0438\u0442\u043a\u0430 \u043f\u0440\u0438 \u041d\u0438\u043a\u043e\u043f\u043e\u043b, Bitka pri Nikopol;\ud130\ud0a4\uc5b4: Ni\u011fbolu Sava\u015f\u0131;\ud5dd\uac00\ub9ac\uc5b4: Nik\u00e1polyi Csata;\ub8e8\ub9c8\ub2c8\uc544\uc5b4: B\u0103t\u0103lia de la Nicopole)\ub294 1396\ub144 9\uc6d4 25\uc77c(\ud639\uc740 9\uc6d4 28\uc77c\uc774\ub780 \ub9d0\ub3c4 \uc788\ub2e4)\uc5d0 \ub3c4\ub098\uc6b0\uac15\ubcc0\uc758 \ub2c8\ucf54\ud3f4\ub9ac\uc2a4\uc5d0\uc11c \uc624\uc2a4\ub9cc \uc81c\uad6d\uc758 \ubc14\uc608\uc9c0\ub4dc 1\uc138(\uc81c\uc704:1389\ub144~1402\ub144)\uc640 \ud5dd\uac00\ub9ac \uc655 \uc9c0\uae30\uc2a4\ubb38\ud2b8\uac00 \uc774\ub044\ub294 \uc720\ub7fd\uc5f0\ud569(\ucc38\uac00\uc138\ub825:\ud5dd\uac00\ub9ac \uc655\uad6d, \uc2e0\uc131\ub85c\ub9c8\uc81c\uad6d, \ud504\ub791\uc2a4, \uc648\ub77c\ud0a4\uc544, \ud3f4\ub780\ub4dc, \uc789\uae00\ub79c\ub4dc, \uc2a4\ucf54\ud2c0\ub79c\ub4dc \uc655\uad6d, \uad6c \uc2a4\uc704\uc2a4 \uc5f0\ud569, \ud29c\ud2bc \uae30\uc0ac\ub2e8, \ubca0\ub124\uce58\uc544 \uacf5\ud654\uad6d, \uc81c\ub178\ubc14 \uacf5\ud654\uad6d, \uc131 \uc694\ud55c \uae30\uc0ac\ub2e8)\uc0ac\uc774\uc5d0\uc11c \uc77c\uc5b4\ub09c \uc804\ud22c\uc774\ub2e4. \ud754\ud788 \ub2c8\ucf54\ud3f4\ub9ac\uc2a4 \uc2ed\uc790\uad70 \uc774\ub77c\uace0 \ubd88\ub9ac\uba70, \uc911\uc138 \ucd5c\ud6c4\uc758 \ub300\uaddc\ubaa8 \uc2ed\uc790\uad70\uc774\uc5c8\ub2e4. \uc804\ud22c \uacb0\uacfc\ub294 \uc624\uc2a4\ub9cc \uc81c\uad6d\uc758 \uacb0\uc815\uc801 \uc2b9\ub9ac\ub85c \ub05d\ub0ac\uace0, \ubc14\uc608\uc9c0\ub4dc 1\uc138\ub294 \uce74\uc774\ub85c\uc758 \ub9d8\ub8e8\ud06c \uc655\uc870 \ubcf4\ud638\ud558\uc5d0 \uc788\ub358 \uce7c\ub9ac\ud30c\ub85c\ubd80\ud130 \ub192\uc740 \ud3c9\uac00\ub97c \ubc1b\uc544 \uc220\ud0c4\uc758 \uce6d\ud638\ub97c \ud558\uc0ac\ubc1b\uc558\ub2e4.", "sentence_answer": "\ud754\ud788 \ub2c8\ucf54\ud3f4\ub9ac\uc2a4 \uc2ed\uc790\uad70 \uc774\ub77c\uace0 \ubd88\ub9ac\uba70, \uc911\uc138 \ucd5c\ud6c4\uc758 \ub300\uaddc\ubaa8 \uc2ed\uc790\uad70\uc774\uc5c8\ub2e4.", "paragraph_id": "6484822-0-1", "paragraph_question": "question: \uc911\uc138 \ucd5c\ud6c4\uc758 \ub300\uaddc\ubaa8 \uc2ed\uc790\uad70\uc744 \uc774\ub974\ub294 \ub9d0\uc740?, context: \ub2c8\ucf54\ud3f4\ub9ac\uc2a4 \uc804\ud22c(\ubd88\uac00\ub9ac\uc544\uc5b4: \u0411\u0438\u0442\u043a\u0430 \u043f\u0440\u0438 \u041d\u0438\u043a\u043e\u043f\u043e\u043b, Bitka pri Nikopol;\ud130\ud0a4\uc5b4: Ni\u011fbolu Sava\u015f\u0131;\ud5dd\uac00\ub9ac\uc5b4: Nik\u00e1polyi Csata;\ub8e8\ub9c8\ub2c8\uc544\uc5b4: B\u0103t\u0103lia de la Nicopole)\ub294 1396\ub144 9\uc6d4 25\uc77c(\ud639\uc740 9\uc6d4 28\uc77c\uc774\ub780 \ub9d0\ub3c4 \uc788\ub2e4)\uc5d0 \ub3c4\ub098\uc6b0\uac15\ubcc0\uc758 \ub2c8\ucf54\ud3f4\ub9ac\uc2a4\uc5d0\uc11c \uc624\uc2a4\ub9cc \uc81c\uad6d\uc758 \ubc14\uc608\uc9c0\ub4dc 1\uc138(\uc81c\uc704:1389\ub144~1402\ub144)\uc640 \ud5dd\uac00\ub9ac \uc655 \uc9c0\uae30\uc2a4\ubb38\ud2b8\uac00 \uc774\ub044\ub294 \uc720\ub7fd\uc5f0\ud569(\ucc38\uac00\uc138\ub825:\ud5dd\uac00\ub9ac \uc655\uad6d, \uc2e0\uc131\ub85c\ub9c8\uc81c\uad6d, \ud504\ub791\uc2a4, \uc648\ub77c\ud0a4\uc544, \ud3f4\ub780\ub4dc, \uc789\uae00\ub79c\ub4dc, \uc2a4\ucf54\ud2c0\ub79c\ub4dc \uc655\uad6d, \uad6c \uc2a4\uc704\uc2a4 \uc5f0\ud569, \ud29c\ud2bc \uae30\uc0ac\ub2e8, \ubca0\ub124\uce58\uc544 \uacf5\ud654\uad6d, \uc81c\ub178\ubc14 \uacf5\ud654\uad6d, \uc131 \uc694\ud55c \uae30\uc0ac\ub2e8)\uc0ac\uc774\uc5d0\uc11c \uc77c\uc5b4\ub09c \uc804\ud22c\uc774\ub2e4. \ud754\ud788 \ub2c8\ucf54\ud3f4\ub9ac\uc2a4 \uc2ed\uc790\uad70\uc774\ub77c\uace0 \ubd88\ub9ac\uba70, \uc911\uc138 \ucd5c\ud6c4\uc758 \ub300\uaddc\ubaa8 \uc2ed\uc790\uad70\uc774\uc5c8\ub2e4. \uc804\ud22c \uacb0\uacfc\ub294 \uc624\uc2a4\ub9cc \uc81c\uad6d\uc758 \uacb0\uc815\uc801 \uc2b9\ub9ac\ub85c \ub05d\ub0ac\uace0, \ubc14\uc608\uc9c0\ub4dc 1\uc138\ub294 \uce74\uc774\ub85c\uc758 \ub9d8\ub8e8\ud06c \uc655\uc870 \ubcf4\ud638\ud558\uc5d0 \uc788\ub358 \uce7c\ub9ac\ud30c\ub85c\ubd80\ud130 \ub192\uc740 \ud3c9\uac00\ub97c \ubc1b\uc544 \uc220\ud0c4\uc758 \uce6d\ud638\ub97c \ud558\uc0ac\ubc1b\uc558\ub2e4."} {"question": "\uc624\uc2a4\ud2b8\ub9ac\uc544\uc758 \uc57c\uc804\uad70\uc744 \uad6c\ud558\uc9c0 \ubabb\ud55c \ub7ec\uc2dc\uc544\uad70\uc744 \uac70\ub290\ub838\ub358 \uc7a5\uad70\uc740?", "paragraph": "\uc774\ub7ec\ub294 \ub3d9\uc548 \ucfe0\ud22c\uc870\ud504(Kutuzov) \ud718\ud558\uc758 \ub7ec\uc2dc\uc544\uad70\uc740 \ub3c4\ucc29\uc774 \ub2a6\uc5b4, \uc624\uc2a4\ud2b8\ub9ac\uc544\uc758 \uc57c\uc804\uad70\uc744 \uad6c\ud558\ub294 \ub370 \uc2e4\ud328\ud588\uace0, \uc774\uc5d0 \ub530\ub77c \ub7ec\uc2dc\uc544\uad70\uc740 \ubd81\ucabd\uc73c\ub85c \ud1f4\uac01\ud558\uc5ec, \uc9c0\uc6d0\uad70\uc744 \uae30\ub2e4\ub9bc\uacfc \ub3d9\uc2dc\uc5d0 \ub0a8\uc544\uc788\ub294 \uc624\uc2a4\ud2b8\ub9ac\uc544\uad70\uacfc \ud569\ub958\ud558\ub824 \ud558\uc600\ub2e4. \ud504\ub791\uc2a4\uad70\uc740 \ub7ec\uc2dc\uc544\uad70\uc744 \ub530\ub77c\uac00\ub2e4\uac00, \uace7 \uadf8\ub4e4\uc774 \uace8\uce58 \uc544\ud508 \uc0c1\ud669\uc5d0 \ub193\uc600\ub2e4\ub294 \uac83\uc744 \uc54c\uc544\ucc28\ub838\ub2e4. \ud504\ub85c\uc774\uc13c\uc758 \uc6c0\uc9c1\uc784\uc740 \uc544\uc9c1 \ub4dc\ub7ec\ub098\uc9c0 \uc54a\uc558\uc9c0\ub9cc, \ucda9\ubd84\ud788 \uc801\ub300\uc801\uc77c \uc218 \uc788\uace0, \ub7ec\uc2dc\uc544\uc640 \uc624\uc2a4\ud2b8\ub9ac\uc544\uad70\uc740 \uc774\uc81c \ud569\ub958\ud558\uc600\ub2e4. \ub354 \uc808\ub9dd\uc801\uc774\uac8c\ub3c4 \ub098\ud3f4\ub808\uc639\uc758 \uc5f0\ub77d\ub9dd\uc740 \uc9c0\ub098\uce58\uac8c \uae38\uc5b4\uc838\uc11c, \uc774 \uc5f0\ub77d\ub9dd\uc744 \uc9c0\ud0a4\uae30 \uc704\ud574\uc11c\ub294 \uac15\ud55c \uc218\ube44\ubcd1\ub4e4\uc744 \ubc30\uce58\ud574\uc57c \ud588\ub2e4. \ub098\ud3f4\ub808\uc639\uc740 \uc774\uc81c \uc6b8\ub984\uc758 \uc2b9\ub9ac\ub97c \ud6a8\uacfc\uc801\uc73c\ub85c \uc774\uc6a9\ud560 \uc720\uc77c\ud55c \ubc29\ubc95\uc740 \ub3d9\ub9f9\uad70\uc5d0\uac8c \uc804\ud22c\ub97c \uac15\uc694\ud558\uc5ec, \uc774\ub4e4\uc744 \uaca9\ud30c\uc2dc\ud0a4\ub294 \uac83\uc774\uc5c8\ub2e4. \ub098\ud3f4\ub808\uc639\uc5d0\uac8c\ub294 \ub2e4\ud589\uc2a4\ub7fd\uac8c\ub3c4 \ub7ec\uc2dc\uc544\uc758 \ucc28\ub974\ub294 \uc804\ud22c\ub97c \uc6d0\ud558\uace0 \uc788\uc5c8\ub2e4.", "answer": "\ucfe0\ud22c\uc870\ud504", "sentence": "\uc774\ub7ec\ub294 \ub3d9\uc548 \ucfe0\ud22c\uc870\ud504 (Kutuzov)", "paragraph_sentence": " \uc774\ub7ec\ub294 \ub3d9\uc548 \ucfe0\ud22c\uc870\ud504 (Kutuzov) \ud718\ud558\uc758 \ub7ec\uc2dc\uc544\uad70\uc740 \ub3c4\ucc29\uc774 \ub2a6\uc5b4, \uc624\uc2a4\ud2b8\ub9ac\uc544\uc758 \uc57c\uc804\uad70\uc744 \uad6c\ud558\ub294 \ub370 \uc2e4\ud328\ud588\uace0, \uc774\uc5d0 \ub530\ub77c \ub7ec\uc2dc\uc544\uad70\uc740 \ubd81\ucabd\uc73c\ub85c \ud1f4\uac01\ud558\uc5ec, \uc9c0\uc6d0\uad70\uc744 \uae30\ub2e4\ub9bc\uacfc \ub3d9\uc2dc\uc5d0 \ub0a8\uc544\uc788\ub294 \uc624\uc2a4\ud2b8\ub9ac\uc544\uad70\uacfc \ud569\ub958\ud558\ub824 \ud558\uc600\ub2e4. \ud504\ub791\uc2a4\uad70\uc740 \ub7ec\uc2dc\uc544\uad70\uc744 \ub530\ub77c\uac00\ub2e4\uac00, \uace7 \uadf8\ub4e4\uc774 \uace8\uce58 \uc544\ud508 \uc0c1\ud669\uc5d0 \ub193\uc600\ub2e4\ub294 \uac83\uc744 \uc54c\uc544\ucc28\ub838\ub2e4. \ud504\ub85c\uc774\uc13c\uc758 \uc6c0\uc9c1\uc784\uc740 \uc544\uc9c1 \ub4dc\ub7ec\ub098\uc9c0 \uc54a\uc558\uc9c0\ub9cc, \ucda9\ubd84\ud788 \uc801\ub300\uc801\uc77c \uc218 \uc788\uace0, \ub7ec\uc2dc\uc544\uc640 \uc624\uc2a4\ud2b8\ub9ac\uc544\uad70\uc740 \uc774\uc81c \ud569\ub958\ud558\uc600\ub2e4. \ub354 \uc808\ub9dd\uc801\uc774\uac8c\ub3c4 \ub098\ud3f4\ub808\uc639\uc758 \uc5f0\ub77d\ub9dd\uc740 \uc9c0\ub098\uce58\uac8c \uae38\uc5b4\uc838\uc11c, \uc774 \uc5f0\ub77d\ub9dd\uc744 \uc9c0\ud0a4\uae30 \uc704\ud574\uc11c\ub294 \uac15\ud55c \uc218\ube44\ubcd1\ub4e4\uc744 \ubc30\uce58\ud574\uc57c \ud588\ub2e4. \ub098\ud3f4\ub808\uc639\uc740 \uc774\uc81c \uc6b8\ub984\uc758 \uc2b9\ub9ac\ub97c \ud6a8\uacfc\uc801\uc73c\ub85c \uc774\uc6a9\ud560 \uc720\uc77c\ud55c \ubc29\ubc95\uc740 \ub3d9\ub9f9\uad70\uc5d0\uac8c \uc804\ud22c\ub97c \uac15\uc694\ud558\uc5ec, \uc774\ub4e4\uc744 \uaca9\ud30c\uc2dc\ud0a4\ub294 \uac83\uc774\uc5c8\ub2e4. \ub098\ud3f4\ub808\uc639\uc5d0\uac8c\ub294 \ub2e4\ud589\uc2a4\ub7fd\uac8c\ub3c4 \ub7ec\uc2dc\uc544\uc758 \ucc28\ub974\ub294 \uc804\ud22c\ub97c \uc6d0\ud558\uace0 \uc788\uc5c8\ub2e4.", "paragraph_answer": "\uc774\ub7ec\ub294 \ub3d9\uc548 \ucfe0\ud22c\uc870\ud504 (Kutuzov) \ud718\ud558\uc758 \ub7ec\uc2dc\uc544\uad70\uc740 \ub3c4\ucc29\uc774 \ub2a6\uc5b4, \uc624\uc2a4\ud2b8\ub9ac\uc544\uc758 \uc57c\uc804\uad70\uc744 \uad6c\ud558\ub294 \ub370 \uc2e4\ud328\ud588\uace0, \uc774\uc5d0 \ub530\ub77c \ub7ec\uc2dc\uc544\uad70\uc740 \ubd81\ucabd\uc73c\ub85c \ud1f4\uac01\ud558\uc5ec, \uc9c0\uc6d0\uad70\uc744 \uae30\ub2e4\ub9bc\uacfc \ub3d9\uc2dc\uc5d0 \ub0a8\uc544\uc788\ub294 \uc624\uc2a4\ud2b8\ub9ac\uc544\uad70\uacfc \ud569\ub958\ud558\ub824 \ud558\uc600\ub2e4. \ud504\ub791\uc2a4\uad70\uc740 \ub7ec\uc2dc\uc544\uad70\uc744 \ub530\ub77c\uac00\ub2e4\uac00, \uace7 \uadf8\ub4e4\uc774 \uace8\uce58 \uc544\ud508 \uc0c1\ud669\uc5d0 \ub193\uc600\ub2e4\ub294 \uac83\uc744 \uc54c\uc544\ucc28\ub838\ub2e4. \ud504\ub85c\uc774\uc13c\uc758 \uc6c0\uc9c1\uc784\uc740 \uc544\uc9c1 \ub4dc\ub7ec\ub098\uc9c0 \uc54a\uc558\uc9c0\ub9cc, \ucda9\ubd84\ud788 \uc801\ub300\uc801\uc77c \uc218 \uc788\uace0, \ub7ec\uc2dc\uc544\uc640 \uc624\uc2a4\ud2b8\ub9ac\uc544\uad70\uc740 \uc774\uc81c \ud569\ub958\ud558\uc600\ub2e4. \ub354 \uc808\ub9dd\uc801\uc774\uac8c\ub3c4 \ub098\ud3f4\ub808\uc639\uc758 \uc5f0\ub77d\ub9dd\uc740 \uc9c0\ub098\uce58\uac8c \uae38\uc5b4\uc838\uc11c, \uc774 \uc5f0\ub77d\ub9dd\uc744 \uc9c0\ud0a4\uae30 \uc704\ud574\uc11c\ub294 \uac15\ud55c \uc218\ube44\ubcd1\ub4e4\uc744 \ubc30\uce58\ud574\uc57c \ud588\ub2e4. \ub098\ud3f4\ub808\uc639\uc740 \uc774\uc81c \uc6b8\ub984\uc758 \uc2b9\ub9ac\ub97c \ud6a8\uacfc\uc801\uc73c\ub85c \uc774\uc6a9\ud560 \uc720\uc77c\ud55c \ubc29\ubc95\uc740 \ub3d9\ub9f9\uad70\uc5d0\uac8c \uc804\ud22c\ub97c \uac15\uc694\ud558\uc5ec, \uc774\ub4e4\uc744 \uaca9\ud30c\uc2dc\ud0a4\ub294 \uac83\uc774\uc5c8\ub2e4. \ub098\ud3f4\ub808\uc639\uc5d0\uac8c\ub294 \ub2e4\ud589\uc2a4\ub7fd\uac8c\ub3c4 \ub7ec\uc2dc\uc544\uc758 \ucc28\ub974\ub294 \uc804\ud22c\ub97c \uc6d0\ud558\uace0 \uc788\uc5c8\ub2e4.", "sentence_answer": "\uc774\ub7ec\ub294 \ub3d9\uc548 \ucfe0\ud22c\uc870\ud504 (Kutuzov)", "paragraph_id": "6520935-7-1", "paragraph_question": "question: \uc624\uc2a4\ud2b8\ub9ac\uc544\uc758 \uc57c\uc804\uad70\uc744 \uad6c\ud558\uc9c0 \ubabb\ud55c \ub7ec\uc2dc\uc544\uad70\uc744 \uac70\ub290\ub838\ub358 \uc7a5\uad70\uc740?, context: \uc774\ub7ec\ub294 \ub3d9\uc548 \ucfe0\ud22c\uc870\ud504(Kutuzov) \ud718\ud558\uc758 \ub7ec\uc2dc\uc544\uad70\uc740 \ub3c4\ucc29\uc774 \ub2a6\uc5b4, \uc624\uc2a4\ud2b8\ub9ac\uc544\uc758 \uc57c\uc804\uad70\uc744 \uad6c\ud558\ub294 \ub370 \uc2e4\ud328\ud588\uace0, \uc774\uc5d0 \ub530\ub77c \ub7ec\uc2dc\uc544\uad70\uc740 \ubd81\ucabd\uc73c\ub85c \ud1f4\uac01\ud558\uc5ec, \uc9c0\uc6d0\uad70\uc744 \uae30\ub2e4\ub9bc\uacfc \ub3d9\uc2dc\uc5d0 \ub0a8\uc544\uc788\ub294 \uc624\uc2a4\ud2b8\ub9ac\uc544\uad70\uacfc \ud569\ub958\ud558\ub824 \ud558\uc600\ub2e4. \ud504\ub791\uc2a4\uad70\uc740 \ub7ec\uc2dc\uc544\uad70\uc744 \ub530\ub77c\uac00\ub2e4\uac00, \uace7 \uadf8\ub4e4\uc774 \uace8\uce58 \uc544\ud508 \uc0c1\ud669\uc5d0 \ub193\uc600\ub2e4\ub294 \uac83\uc744 \uc54c\uc544\ucc28\ub838\ub2e4. \ud504\ub85c\uc774\uc13c\uc758 \uc6c0\uc9c1\uc784\uc740 \uc544\uc9c1 \ub4dc\ub7ec\ub098\uc9c0 \uc54a\uc558\uc9c0\ub9cc, \ucda9\ubd84\ud788 \uc801\ub300\uc801\uc77c \uc218 \uc788\uace0, \ub7ec\uc2dc\uc544\uc640 \uc624\uc2a4\ud2b8\ub9ac\uc544\uad70\uc740 \uc774\uc81c \ud569\ub958\ud558\uc600\ub2e4. \ub354 \uc808\ub9dd\uc801\uc774\uac8c\ub3c4 \ub098\ud3f4\ub808\uc639\uc758 \uc5f0\ub77d\ub9dd\uc740 \uc9c0\ub098\uce58\uac8c \uae38\uc5b4\uc838\uc11c, \uc774 \uc5f0\ub77d\ub9dd\uc744 \uc9c0\ud0a4\uae30 \uc704\ud574\uc11c\ub294 \uac15\ud55c \uc218\ube44\ubcd1\ub4e4\uc744 \ubc30\uce58\ud574\uc57c \ud588\ub2e4. \ub098\ud3f4\ub808\uc639\uc740 \uc774\uc81c \uc6b8\ub984\uc758 \uc2b9\ub9ac\ub97c \ud6a8\uacfc\uc801\uc73c\ub85c \uc774\uc6a9\ud560 \uc720\uc77c\ud55c \ubc29\ubc95\uc740 \ub3d9\ub9f9\uad70\uc5d0\uac8c \uc804\ud22c\ub97c \uac15\uc694\ud558\uc5ec, \uc774\ub4e4\uc744 \uaca9\ud30c\uc2dc\ud0a4\ub294 \uac83\uc774\uc5c8\ub2e4. \ub098\ud3f4\ub808\uc639\uc5d0\uac8c\ub294 \ub2e4\ud589\uc2a4\ub7fd\uac8c\ub3c4 \ub7ec\uc2dc\uc544\uc758 \ucc28\ub974\ub294 \uc804\ud22c\ub97c \uc6d0\ud558\uace0 \uc788\uc5c8\ub2e4."} {"question": "1840\ub144 6\uc70c\uc5d0 \ucc3d\uac04\ud558\uaca0\ub2e4\uace0 \ubc1c\ud45c\ud55c \uac83\uc740?", "paragraph": "1840\ub144 6\uc6d4, \ud3ec\ub294 \uc790\uae30 \uc18c\uc720\uc758 \ubb38\uc608\uc9c0 \u300a\uc2a4\ud0c0\uc77c\ub7ec\uc2a4\u300b\ub97c \ucc3d\uac04\ud558\uaca0\ub2e4\ub294 \uacc4\ud68d\uc548\uc744 \ubc1c\ud45c\ud588\ub2e4. \uc6d0\ub798 \ud3ec\uac00 \uc5fc\ub450\uc5d0 \ub454 \uc7a1\uc9c0 \uc774\ub984\uc740 \u300a\ud39c\u300b(The Penn)\uc774\uc5c8\uc73c\uba70, \ud544\ub77c\ub378\ud53c\uc544\uc5d0 \uac70\uc810\uc744 \ub450\uace0 \uc0ac\uc5c5\uc744 \ud560 \uc0dd\uac01\uc774\uc5c8\ub2e4. \ud3ec\ub294 \ud544\ub77c\ub378\ud53c\uc544\uc758 \uc11d\uac04\uc9c0 \u300a\uc0c8\ud130\ub370\uc774 \uc774\ube0c\ub2dd \ud3ec\uc2a4\ud2b8\u300b 1840\ub144 6\uc6d4 6\uc77c \uc790\uc758 \uad11\uace0\ub780 \ud558\ub098\ub97c \uc0ac\uc11c \uc790\uc2e0\uc758 \uacc4\ud68d\uc744 \ub2e4\uc74c\uacfc \uac19\uc774 \ud64d\ubcf4\ud588\ub2e4. \u201c\uc5d0\ub4dc\uac70 A. \ud3ec\uac00 \ud544\ub77c\ub378\ud53c\uc544 \uc2dc\uc5d0\uc11c \ud3b8\uc9d1 \ubc0f \ubc1c\ud589\ud560 \uc6d4\uac04 \ubb38\uc608\uc9c0 \u2018\ud39c \ub9e4\uac70\uc9c4\u2019\uc758 \uacc4\ud68d\uc548.\u201d \uadf8\ub7ec\ub098 \ud3ec\ub294 \uc8fd\uae30 \uc804\uae4c\uc9c0 \uc774 \uacc4\ud68d\uc744 \uc2e4\ud604\ud558\uc9c0 \ubabb\ud588\ub2e4. \uc774\ub54c\ucbe4 \ud3ec\ub294 \ud718\uadf8\ub2f9 \ub2f9\uc6d0\uc744 \uc790\ucc98\ud558\uba70 \ud0c0\uc77c\ub7ec \ud589\uc815\ubd80\uc758 \uacf5\uc9c1\uc744 \ud55c \uc790\ub9ac \uc5bb\uc5b4\ubcf4\ub824\uace0 \uae30\uc6c3\uac70\ub838\ub2e4. \ud0c0\uc77c\ub7ec \ub300\ud1b5\ub839\uc758 \uc544\ub4e4 \ub85c\ubc84\ud2b8 \ud0c0\uc77c\ub7ec\uac00 \ud3ec\uc758 \uce5c\uad6c \ud504\ub808\ub354\ub9ad \ud1a0\uba38\uc2a4(Frederick Thomas)\uc758 \uc9c0\uc778\uc774\uc5c8\ub294\ub370, \ud3ec\ub294 \ub85c\ubc84\ud2b8\uc758 \ub3c4\uc6c0\uc744 \ube4c\ub824 \ud544\ub77c\ub378\ud53c\uc544 \uc138\uad00\uc5d0 \ucde8\uc9c1\ud558\uace0\uc790 \ud588\ub2e4. \uadf8\ub7ec\ub098 \ud3ec\ub294 \uadf8 \uac74\uacfc \uad00\ub828\ud574 \ud1a0\uba38\uc2a4\uc640 \uc0c1\uc758\ud558\uae30\ub85c \ud55c \uc57d\uc18d \uc77c\uc790(1842\ub144 9\uc6d4)\uc5d0 \ub098\uc624\uc9c0 \ubabb\ud588\ub2e4. \ud3ec\ub294 \uc790\uae30\uac00 \uc544\ud320\ub2e4\uace0 \ud588\uc9c0\ub9cc, \ud1a0\uba38\uc2a4\ub294 \ud3ec\uac00 \uc220\uc5d0 \ucde8\ud574\uc11c \ubabb \ub098\uc628 \uac83\uc73c\ub85c \uc0dd\uac01\ud588\ub2e4. \ud3ec\ub294 \uc5ec\ub7ec \ucc28\ub840 \uc5fd\uad00 \uccad\ud0c1\uc744 \ud574\uc11c \uc57d\uc18d\uc744 \ubc1b\uc558\uc9c0\ub9cc \ubaa8\ub4e0 \uac10\ud22c\ub294 \ubc88\ubc88\uc774 \ub2e4\ub978 \uc0ac\ub78c\ub4e4\uc5d0\uac8c \ub3cc\uc544\uac14\ub2e4.", "answer": "\uc2a4\ud0c0\uc77c\ub7ec\uc2a4", "sentence": "1840\ub144 6\uc6d4, \ud3ec\ub294 \uc790\uae30 \uc18c\uc720\uc758 \ubb38\uc608\uc9c0 \u300a \uc2a4\ud0c0\uc77c\ub7ec\uc2a4 \u300b\ub97c \ucc3d\uac04\ud558\uaca0\ub2e4\ub294 \uacc4\ud68d\uc548\uc744 \ubc1c\ud45c\ud588\ub2e4.", "paragraph_sentence": " 1840\ub144 6\uc6d4, \ud3ec\ub294 \uc790\uae30 \uc18c\uc720\uc758 \ubb38\uc608\uc9c0 \u300a \uc2a4\ud0c0\uc77c\ub7ec\uc2a4 \u300b\ub97c \ucc3d\uac04\ud558\uaca0\ub2e4\ub294 \uacc4\ud68d\uc548\uc744 \ubc1c\ud45c\ud588\ub2e4. \uc6d0\ub798 \ud3ec\uac00 \uc5fc\ub450\uc5d0 \ub454 \uc7a1\uc9c0 \uc774\ub984\uc740 \u300a\ud39c\u300b(The Penn)\uc774\uc5c8\uc73c\uba70, \ud544\ub77c\ub378\ud53c\uc544\uc5d0 \uac70\uc810\uc744 \ub450\uace0 \uc0ac\uc5c5\uc744 \ud560 \uc0dd\uac01\uc774\uc5c8\ub2e4. \ud3ec\ub294 \ud544\ub77c\ub378\ud53c\uc544\uc758 \uc11d\uac04\uc9c0 \u300a\uc0c8\ud130\ub370\uc774 \uc774\ube0c\ub2dd \ud3ec\uc2a4\ud2b8\u300b 1840\ub144 6\uc6d4 6\uc77c \uc790\uc758 \uad11\uace0\ub780 \ud558\ub098\ub97c \uc0ac\uc11c \uc790\uc2e0\uc758 \uacc4\ud68d\uc744 \ub2e4\uc74c\uacfc \uac19\uc774 \ud64d\ubcf4\ud588\ub2e4. \u201c\uc5d0\ub4dc\uac70 A. \ud3ec\uac00 \ud544\ub77c\ub378\ud53c\uc544 \uc2dc\uc5d0\uc11c \ud3b8\uc9d1 \ubc0f \ubc1c\ud589\ud560 \uc6d4\uac04 \ubb38\uc608\uc9c0 \u2018\ud39c \ub9e4\uac70\uc9c4\u2019\uc758 \uacc4\ud68d\uc548. \u201d \uadf8\ub7ec\ub098 \ud3ec\ub294 \uc8fd\uae30 \uc804\uae4c\uc9c0 \uc774 \uacc4\ud68d\uc744 \uc2e4\ud604\ud558\uc9c0 \ubabb\ud588\ub2e4. \uc774\ub54c\ucbe4 \ud3ec\ub294 \ud718\uadf8\ub2f9 \ub2f9\uc6d0\uc744 \uc790\ucc98\ud558\uba70 \ud0c0\uc77c\ub7ec \ud589\uc815\ubd80\uc758 \uacf5\uc9c1\uc744 \ud55c \uc790\ub9ac \uc5bb\uc5b4\ubcf4\ub824\uace0 \uae30\uc6c3\uac70\ub838\ub2e4. \ud0c0\uc77c\ub7ec \ub300\ud1b5\ub839\uc758 \uc544\ub4e4 \ub85c\ubc84\ud2b8 \ud0c0\uc77c\ub7ec\uac00 \ud3ec\uc758 \uce5c\uad6c \ud504\ub808\ub354\ub9ad \ud1a0\uba38\uc2a4(Frederick Thomas)\uc758 \uc9c0\uc778\uc774\uc5c8\ub294\ub370, \ud3ec\ub294 \ub85c\ubc84\ud2b8\uc758 \ub3c4\uc6c0\uc744 \ube4c\ub824 \ud544\ub77c\ub378\ud53c\uc544 \uc138\uad00\uc5d0 \ucde8\uc9c1\ud558\uace0\uc790 \ud588\ub2e4. \uadf8\ub7ec\ub098 \ud3ec\ub294 \uadf8 \uac74\uacfc \uad00\ub828\ud574 \ud1a0\uba38\uc2a4\uc640 \uc0c1\uc758\ud558\uae30\ub85c \ud55c \uc57d\uc18d \uc77c\uc790(1842\ub144 9\uc6d4)\uc5d0 \ub098\uc624\uc9c0 \ubabb\ud588\ub2e4. \ud3ec\ub294 \uc790\uae30\uac00 \uc544\ud320\ub2e4\uace0 \ud588\uc9c0\ub9cc, \ud1a0\uba38\uc2a4\ub294 \ud3ec\uac00 \uc220\uc5d0 \ucde8\ud574\uc11c \ubabb \ub098\uc628 \uac83\uc73c\ub85c \uc0dd\uac01\ud588\ub2e4. \ud3ec\ub294 \uc5ec\ub7ec \ucc28\ub840 \uc5fd\uad00 \uccad\ud0c1\uc744 \ud574\uc11c \uc57d\uc18d\uc744 \ubc1b\uc558\uc9c0\ub9cc \ubaa8\ub4e0 \uac10\ud22c\ub294 \ubc88\ubc88\uc774 \ub2e4\ub978 \uc0ac\ub78c\ub4e4\uc5d0\uac8c \ub3cc\uc544\uac14\ub2e4.", "paragraph_answer": "1840\ub144 6\uc6d4, \ud3ec\ub294 \uc790\uae30 \uc18c\uc720\uc758 \ubb38\uc608\uc9c0 \u300a \uc2a4\ud0c0\uc77c\ub7ec\uc2a4 \u300b\ub97c \ucc3d\uac04\ud558\uaca0\ub2e4\ub294 \uacc4\ud68d\uc548\uc744 \ubc1c\ud45c\ud588\ub2e4. \uc6d0\ub798 \ud3ec\uac00 \uc5fc\ub450\uc5d0 \ub454 \uc7a1\uc9c0 \uc774\ub984\uc740 \u300a\ud39c\u300b(The Penn)\uc774\uc5c8\uc73c\uba70, \ud544\ub77c\ub378\ud53c\uc544\uc5d0 \uac70\uc810\uc744 \ub450\uace0 \uc0ac\uc5c5\uc744 \ud560 \uc0dd\uac01\uc774\uc5c8\ub2e4. \ud3ec\ub294 \ud544\ub77c\ub378\ud53c\uc544\uc758 \uc11d\uac04\uc9c0 \u300a\uc0c8\ud130\ub370\uc774 \uc774\ube0c\ub2dd \ud3ec\uc2a4\ud2b8\u300b 1840\ub144 6\uc6d4 6\uc77c \uc790\uc758 \uad11\uace0\ub780 \ud558\ub098\ub97c \uc0ac\uc11c \uc790\uc2e0\uc758 \uacc4\ud68d\uc744 \ub2e4\uc74c\uacfc \uac19\uc774 \ud64d\ubcf4\ud588\ub2e4. \u201c\uc5d0\ub4dc\uac70 A. \ud3ec\uac00 \ud544\ub77c\ub378\ud53c\uc544 \uc2dc\uc5d0\uc11c \ud3b8\uc9d1 \ubc0f \ubc1c\ud589\ud560 \uc6d4\uac04 \ubb38\uc608\uc9c0 \u2018\ud39c \ub9e4\uac70\uc9c4\u2019\uc758 \uacc4\ud68d\uc548.\u201d \uadf8\ub7ec\ub098 \ud3ec\ub294 \uc8fd\uae30 \uc804\uae4c\uc9c0 \uc774 \uacc4\ud68d\uc744 \uc2e4\ud604\ud558\uc9c0 \ubabb\ud588\ub2e4. \uc774\ub54c\ucbe4 \ud3ec\ub294 \ud718\uadf8\ub2f9 \ub2f9\uc6d0\uc744 \uc790\ucc98\ud558\uba70 \ud0c0\uc77c\ub7ec \ud589\uc815\ubd80\uc758 \uacf5\uc9c1\uc744 \ud55c \uc790\ub9ac \uc5bb\uc5b4\ubcf4\ub824\uace0 \uae30\uc6c3\uac70\ub838\ub2e4. \ud0c0\uc77c\ub7ec \ub300\ud1b5\ub839\uc758 \uc544\ub4e4 \ub85c\ubc84\ud2b8 \ud0c0\uc77c\ub7ec\uac00 \ud3ec\uc758 \uce5c\uad6c \ud504\ub808\ub354\ub9ad \ud1a0\uba38\uc2a4(Frederick Thomas)\uc758 \uc9c0\uc778\uc774\uc5c8\ub294\ub370, \ud3ec\ub294 \ub85c\ubc84\ud2b8\uc758 \ub3c4\uc6c0\uc744 \ube4c\ub824 \ud544\ub77c\ub378\ud53c\uc544 \uc138\uad00\uc5d0 \ucde8\uc9c1\ud558\uace0\uc790 \ud588\ub2e4. \uadf8\ub7ec\ub098 \ud3ec\ub294 \uadf8 \uac74\uacfc \uad00\ub828\ud574 \ud1a0\uba38\uc2a4\uc640 \uc0c1\uc758\ud558\uae30\ub85c \ud55c \uc57d\uc18d \uc77c\uc790(1842\ub144 9\uc6d4)\uc5d0 \ub098\uc624\uc9c0 \ubabb\ud588\ub2e4. \ud3ec\ub294 \uc790\uae30\uac00 \uc544\ud320\ub2e4\uace0 \ud588\uc9c0\ub9cc, \ud1a0\uba38\uc2a4\ub294 \ud3ec\uac00 \uc220\uc5d0 \ucde8\ud574\uc11c \ubabb \ub098\uc628 \uac83\uc73c\ub85c \uc0dd\uac01\ud588\ub2e4. \ud3ec\ub294 \uc5ec\ub7ec \ucc28\ub840 \uc5fd\uad00 \uccad\ud0c1\uc744 \ud574\uc11c \uc57d\uc18d\uc744 \ubc1b\uc558\uc9c0\ub9cc \ubaa8\ub4e0 \uac10\ud22c\ub294 \ubc88\ubc88\uc774 \ub2e4\ub978 \uc0ac\ub78c\ub4e4\uc5d0\uac8c \ub3cc\uc544\uac14\ub2e4.", "sentence_answer": "1840\ub144 6\uc6d4, \ud3ec\ub294 \uc790\uae30 \uc18c\uc720\uc758 \ubb38\uc608\uc9c0 \u300a \uc2a4\ud0c0\uc77c\ub7ec\uc2a4 \u300b\ub97c \ucc3d\uac04\ud558\uaca0\ub2e4\ub294 \uacc4\ud68d\uc548\uc744 \ubc1c\ud45c\ud588\ub2e4.", "paragraph_id": "6523010-8-0", "paragraph_question": "question: 1840\ub144 6\uc70c\uc5d0 \ucc3d\uac04\ud558\uaca0\ub2e4\uace0 \ubc1c\ud45c\ud55c \uac83\uc740?, context: 1840\ub144 6\uc6d4, \ud3ec\ub294 \uc790\uae30 \uc18c\uc720\uc758 \ubb38\uc608\uc9c0 \u300a\uc2a4\ud0c0\uc77c\ub7ec\uc2a4\u300b\ub97c \ucc3d\uac04\ud558\uaca0\ub2e4\ub294 \uacc4\ud68d\uc548\uc744 \ubc1c\ud45c\ud588\ub2e4. \uc6d0\ub798 \ud3ec\uac00 \uc5fc\ub450\uc5d0 \ub454 \uc7a1\uc9c0 \uc774\ub984\uc740 \u300a\ud39c\u300b(The Penn)\uc774\uc5c8\uc73c\uba70, \ud544\ub77c\ub378\ud53c\uc544\uc5d0 \uac70\uc810\uc744 \ub450\uace0 \uc0ac\uc5c5\uc744 \ud560 \uc0dd\uac01\uc774\uc5c8\ub2e4. \ud3ec\ub294 \ud544\ub77c\ub378\ud53c\uc544\uc758 \uc11d\uac04\uc9c0 \u300a\uc0c8\ud130\ub370\uc774 \uc774\ube0c\ub2dd \ud3ec\uc2a4\ud2b8\u300b 1840\ub144 6\uc6d4 6\uc77c \uc790\uc758 \uad11\uace0\ub780 \ud558\ub098\ub97c \uc0ac\uc11c \uc790\uc2e0\uc758 \uacc4\ud68d\uc744 \ub2e4\uc74c\uacfc \uac19\uc774 \ud64d\ubcf4\ud588\ub2e4. \u201c\uc5d0\ub4dc\uac70 A. \ud3ec\uac00 \ud544\ub77c\ub378\ud53c\uc544 \uc2dc\uc5d0\uc11c \ud3b8\uc9d1 \ubc0f \ubc1c\ud589\ud560 \uc6d4\uac04 \ubb38\uc608\uc9c0 \u2018\ud39c \ub9e4\uac70\uc9c4\u2019\uc758 \uacc4\ud68d\uc548.\u201d \uadf8\ub7ec\ub098 \ud3ec\ub294 \uc8fd\uae30 \uc804\uae4c\uc9c0 \uc774 \uacc4\ud68d\uc744 \uc2e4\ud604\ud558\uc9c0 \ubabb\ud588\ub2e4. \uc774\ub54c\ucbe4 \ud3ec\ub294 \ud718\uadf8\ub2f9 \ub2f9\uc6d0\uc744 \uc790\ucc98\ud558\uba70 \ud0c0\uc77c\ub7ec \ud589\uc815\ubd80\uc758 \uacf5\uc9c1\uc744 \ud55c \uc790\ub9ac \uc5bb\uc5b4\ubcf4\ub824\uace0 \uae30\uc6c3\uac70\ub838\ub2e4. \ud0c0\uc77c\ub7ec \ub300\ud1b5\ub839\uc758 \uc544\ub4e4 \ub85c\ubc84\ud2b8 \ud0c0\uc77c\ub7ec\uac00 \ud3ec\uc758 \uce5c\uad6c \ud504\ub808\ub354\ub9ad \ud1a0\uba38\uc2a4(Frederick Thomas)\uc758 \uc9c0\uc778\uc774\uc5c8\ub294\ub370, \ud3ec\ub294 \ub85c\ubc84\ud2b8\uc758 \ub3c4\uc6c0\uc744 \ube4c\ub824 \ud544\ub77c\ub378\ud53c\uc544 \uc138\uad00\uc5d0 \ucde8\uc9c1\ud558\uace0\uc790 \ud588\ub2e4. \uadf8\ub7ec\ub098 \ud3ec\ub294 \uadf8 \uac74\uacfc \uad00\ub828\ud574 \ud1a0\uba38\uc2a4\uc640 \uc0c1\uc758\ud558\uae30\ub85c \ud55c \uc57d\uc18d \uc77c\uc790(1842\ub144 9\uc6d4)\uc5d0 \ub098\uc624\uc9c0 \ubabb\ud588\ub2e4. \ud3ec\ub294 \uc790\uae30\uac00 \uc544\ud320\ub2e4\uace0 \ud588\uc9c0\ub9cc, \ud1a0\uba38\uc2a4\ub294 \ud3ec\uac00 \uc220\uc5d0 \ucde8\ud574\uc11c \ubabb \ub098\uc628 \uac83\uc73c\ub85c \uc0dd\uac01\ud588\ub2e4. \ud3ec\ub294 \uc5ec\ub7ec \ucc28\ub840 \uc5fd\uad00 \uccad\ud0c1\uc744 \ud574\uc11c \uc57d\uc18d\uc744 \ubc1b\uc558\uc9c0\ub9cc \ubaa8\ub4e0 \uac10\ud22c\ub294 \ubc88\ubc88\uc774 \ub2e4\ub978 \uc0ac\ub78c\ub4e4\uc5d0\uac8c \ub3cc\uc544\uac14\ub2e4."} {"question": "\uc601\ud654 \uc18d \ud5e4\uc774\uc990\uacfc \uc5b4\uac70\uc2a4\ud130\uc2a4\uac00 \uc0dd\uae30\uac00 \uc788\ub2e4\uace0 \ud3c9\ud55c \ud0c0\uc784\uc758 \uc18c\uc18d\uc778\uc740?", "paragraph": "\ud0c0\uc784\uc758 \ub9ac\ucc98\ub4dc \ucf5c\ub9ac\uc2a4\ub294 \"\uc601\ud654 \uc18d \ud5e4\uc774\uc990\uacfc \uc5b4\uac70\uc2a4\ud130\uc2a4\ub294 \ub208 \uc55e\uc5d0 \ubcf4\uc774\ub294 \uac83\ub4e4\uc744 \uc0dd\uac01\ud558\uba70, \uc0dd\uae30\uac00 \uc788\ub2e4. \uc774 \uc810\uc740 \ub450 \uc0ac\ub78c\uc744 \uc5f0\uae30\ud558\uace0 \uc788\ub294 \ubc30\uc6b0\uac00 \uc80a\uae30 \ub54c\ubb38\uc77c \uac83\uc774\ub2e4.\"\uace0 \ub9d0\ud588\ub2e4. \ub308\ub7ec\uc2a4 \ubaa8\ub2dd \ub274\uc2a4\uc758 \ud06c\ub9ac\uc2a4 \ubd09\uac70\ub294 B+\ub77c\ub294 \ud3c9\uac00\ub97c \ub0b4\ub838\ub2e4 \"\u300a\uc548\ub155, \ud5e4\uc774\uc990\u300b\uc740 \uc7ac\uce58\uc788\uace0 \uc0dd\uae30 \uc788\ub2e4. \ub2e8\uc9c0 \uc870\uae08 \uac10\uc0c1\uc801\uc774\ub2e4.\"\uace0 \ud588\ub2e4. USA \ud22c\ub370\uc774\uc758 \ud074\ub77c\uc6b0\ub514\uc544 \ud478\uc774\uadf8\ub294 4\uc810 \uc911 3.5\uc810\uc744 \uc8fc\uace0, \"\uc88b\uc740 \uac01\ubcf8\uc774\uba70 \uc5f0\uae30\ub3c4 \uc88b\ub2e4. \uc7ac\ubbf8\ub3c4 \uc788\uace0 \ud55c\ud3b8\uc73c\ub85c\ub294 \ud1b5\ucc30\ub825\uc774 \uc788\ub2e4.\"\uace0 \ud3c9\ud558\uc600\ub2e4. \uc5d4\ud130\ud14c\uc778\uba3c\ud2b8 \uc704\ud074\ub9ac\uc758 \ud06c\ub9ac\uc2a4 \ub0b4\uc154\uc6e8\uc774\ud2f0\ub294 \"\uc7ac\ubbf8\uc788\uace0 \ub2ec\ucf64\ud55c \ub9e4\uc6b0 \uac10\ub3d9\uc2a4\ub7ec\uc6b4 \uc601\ud654\"\ub77c\uba70 B \ud3c9\uac00\ub97c \ub0b4\ub838\ub2e4. \uae00\ub85c\ube0c \uc564 \uba54\uc77c\uc758 \uc81c\ud504 \ud398\ubca0\ub808\ub294 \"\uc774 \uc791\ud488\uc740 \uad00\uac1d\uc774 \uc2e4\uc81c \uc0dd\ud65c\uc5d0\uc11c \uac00\uc9c0\ub294 \uae30\ub300\ub97c \ubc30\ubc18\ud560\uc9c0\ub3c4 \ubaa8\ub978\ub2e4. \ud558\uc9c0\ub9cc \uccad\uc18c\ub144\uc744 \uc704\ud55c \uc601\ud654\uc758 \uad00\uc2b5\uc744 \uc2f9\ub451 \uc790\ub974\uace0 \uc788\ub2e4.\"\uace0 \ud558\uc600\ub2e4. \ub9c8\uc774\uc560\ubbf8 \ud5e4\ub7f4\ub4dc\uc758 \ucf54\ub2c8 \uc624\uae00\uc740 \"\uac11\uc790\uae30 \ub530\ub73b\ud574\uc9c0\uac70\ub098 \uc720\uba38\uac00 \uc788\ub294 \ub2ec\ucf64\ud55c \ub85c\ub9e8\uc2a4 \uc601\ud654\ub2e4.\"\ub77c\uace0 \ud558\uc600\ub2e4. \ub514 A.V. \ud074\ub7fd\uc758 A. A. \ub2e4\uc6b0\ub4dc\ub294 B \ud3c9\uac00\ub97c \ub0b4\ub9ac\uba70, \"\ubc88\uac1c \uac19\uc740 \uc704\ud2b8\uac00 \uc788\uc73c\uba70, \uac1c\uc131 \ud48d\ubd80\ud558\uace0 \uc7ac\ub2a5\uc788\ub294 \ubc30\uc6b0\ub4e4\uc5d0 \uc758\ud574 \ub9e4\ub825\uc774 \uc99d\uac00\ud55c\ub2e4.\"\uace0 \ub9d0\ud588\ub2e4. \ub864\ub9c1 \uc2a4\ud1a4\uc758 \ud53c\ud130 \ud2b8\ub798\ubc84\uc2a4\ub294 \"\uc2e0\uc120\ud558\uace0 \uc0dd\uc0dd\ud55c \ub85c\ub9e8\uc2a4 \uc601\ud654\uc774\ub2e4. \uac10\ub3d9\uacfc \uc720\uba38\uac00 \ub118\uce58\uace0 \uc170\uc77c\ub9b0 \uc6b0\ub4e4\ub9ac\uac00 \uc5f0\uae30\ub85c \uc9c8\uc744 \ub192\uc774\uace0 \uc788\ub2e4. \uc170\uc77c\ub9b0 \uc6b0\ub4e4\ub9ac\ub294 \uce74\uba54\ub77c \uc55e\uc5d0\uc11c \uc2e4\uc218\ub97c \ud560 \uc218 \uc5c6\ub2e4\uace0 \ud560 \uc815\ub3c4\ub85c \uc88b\uc740 \ubc30\uc6b0\uc774\ub2e4.\"\ub77c\uba70 \uc170\uc77c\ub9b0 \uc6b0\ub4e4\ub9ac\ub97c \uce6d\ucc2c\ud558\uace0 4\uc810 \uc911 3\uc810\uc744 \uc8fc\uc5c8\ub2e4.", "answer": "\ub9ac\ucc98\ub4dc \ucf5c\ub9ac\uc2a4", "sentence": "\ud0c0\uc784\uc758 \ub9ac\ucc98\ub4dc \ucf5c\ub9ac\uc2a4 \ub294 \"\uc601\ud654 \uc18d \ud5e4\uc774\uc990\uacfc \uc5b4\uac70\uc2a4\ud130\uc2a4\ub294 \ub208 \uc55e\uc5d0 \ubcf4\uc774\ub294 \uac83\ub4e4\uc744 \uc0dd\uac01\ud558\uba70, \uc0dd\uae30\uac00 \uc788\ub2e4.", "paragraph_sentence": " \ud0c0\uc784\uc758 \ub9ac\ucc98\ub4dc \ucf5c\ub9ac\uc2a4 \ub294 \"\uc601\ud654 \uc18d \ud5e4\uc774\uc990\uacfc \uc5b4\uac70\uc2a4\ud130\uc2a4\ub294 \ub208 \uc55e\uc5d0 \ubcf4\uc774\ub294 \uac83\ub4e4\uc744 \uc0dd\uac01\ud558\uba70, \uc0dd\uae30\uac00 \uc788\ub2e4. \uc774 \uc810\uc740 \ub450 \uc0ac\ub78c\uc744 \uc5f0\uae30\ud558\uace0 \uc788\ub294 \ubc30\uc6b0\uac00 \uc80a\uae30 \ub54c\ubb38\uc77c \uac83\uc774\ub2e4. \"\uace0 \ub9d0\ud588\ub2e4. \ub308\ub7ec\uc2a4 \ubaa8\ub2dd \ub274\uc2a4\uc758 \ud06c\ub9ac\uc2a4 \ubd09\uac70\ub294 B+\ub77c\ub294 \ud3c9\uac00\ub97c \ub0b4\ub838\ub2e4 \"\u300a\uc548\ub155, \ud5e4\uc774\uc990\u300b\uc740 \uc7ac\uce58\uc788\uace0 \uc0dd\uae30 \uc788\ub2e4. \ub2e8\uc9c0 \uc870\uae08 \uac10\uc0c1\uc801\uc774\ub2e4. \"\uace0 \ud588\ub2e4. USA \ud22c\ub370\uc774\uc758 \ud074\ub77c\uc6b0\ub514\uc544 \ud478\uc774\uadf8\ub294 4\uc810 \uc911 3.5\uc810\uc744 \uc8fc\uace0, \"\uc88b\uc740 \uac01\ubcf8\uc774\uba70 \uc5f0\uae30\ub3c4 \uc88b\ub2e4. \uc7ac\ubbf8\ub3c4 \uc788\uace0 \ud55c\ud3b8\uc73c\ub85c\ub294 \ud1b5\ucc30\ub825\uc774 \uc788\ub2e4. \"\uace0 \ud3c9\ud558\uc600\ub2e4. \uc5d4\ud130\ud14c\uc778\uba3c\ud2b8 \uc704\ud074\ub9ac\uc758 \ud06c\ub9ac\uc2a4 \ub0b4\uc154\uc6e8\uc774\ud2f0\ub294 \"\uc7ac\ubbf8\uc788\uace0 \ub2ec\ucf64\ud55c \ub9e4\uc6b0 \uac10\ub3d9\uc2a4\ub7ec\uc6b4 \uc601\ud654\"\ub77c\uba70 B \ud3c9\uac00\ub97c \ub0b4\ub838\ub2e4. \uae00\ub85c\ube0c \uc564 \uba54\uc77c\uc758 \uc81c\ud504 \ud398\ubca0\ub808\ub294 \"\uc774 \uc791\ud488\uc740 \uad00\uac1d\uc774 \uc2e4\uc81c \uc0dd\ud65c\uc5d0\uc11c \uac00\uc9c0\ub294 \uae30\ub300\ub97c \ubc30\ubc18\ud560\uc9c0\ub3c4 \ubaa8\ub978\ub2e4. \ud558\uc9c0\ub9cc \uccad\uc18c\ub144\uc744 \uc704\ud55c \uc601\ud654\uc758 \uad00\uc2b5\uc744 \uc2f9\ub451 \uc790\ub974\uace0 \uc788\ub2e4. \"\uace0 \ud558\uc600\ub2e4. \ub9c8\uc774\uc560\ubbf8 \ud5e4\ub7f4\ub4dc\uc758 \ucf54\ub2c8 \uc624\uae00\uc740 \"\uac11\uc790\uae30 \ub530\ub73b\ud574\uc9c0\uac70\ub098 \uc720\uba38\uac00 \uc788\ub294 \ub2ec\ucf64\ud55c \ub85c\ub9e8\uc2a4 \uc601\ud654\ub2e4. \"\ub77c\uace0 \ud558\uc600\ub2e4. \ub514 A.V. \ud074\ub7fd\uc758 A. A. \ub2e4\uc6b0\ub4dc\ub294 B \ud3c9\uac00\ub97c \ub0b4\ub9ac\uba70, \"\ubc88\uac1c \uac19\uc740 \uc704\ud2b8\uac00 \uc788\uc73c\uba70, \uac1c\uc131 \ud48d\ubd80\ud558\uace0 \uc7ac\ub2a5\uc788\ub294 \ubc30\uc6b0\ub4e4\uc5d0 \uc758\ud574 \ub9e4\ub825\uc774 \uc99d\uac00\ud55c\ub2e4. \"\uace0 \ub9d0\ud588\ub2e4. \ub864\ub9c1 \uc2a4\ud1a4\uc758 \ud53c\ud130 \ud2b8\ub798\ubc84\uc2a4\ub294 \"\uc2e0\uc120\ud558\uace0 \uc0dd\uc0dd\ud55c \ub85c\ub9e8\uc2a4 \uc601\ud654\uc774\ub2e4. \uac10\ub3d9\uacfc \uc720\uba38\uac00 \ub118\uce58\uace0 \uc170\uc77c\ub9b0 \uc6b0\ub4e4\ub9ac\uac00 \uc5f0\uae30\ub85c \uc9c8\uc744 \ub192\uc774\uace0 \uc788\ub2e4. \uc170\uc77c\ub9b0 \uc6b0\ub4e4\ub9ac\ub294 \uce74\uba54\ub77c \uc55e\uc5d0\uc11c \uc2e4\uc218\ub97c \ud560 \uc218 \uc5c6\ub2e4\uace0 \ud560 \uc815\ub3c4\ub85c \uc88b\uc740 \ubc30\uc6b0\uc774\ub2e4. \"\ub77c\uba70 \uc170\uc77c\ub9b0 \uc6b0\ub4e4\ub9ac\ub97c \uce6d\ucc2c\ud558\uace0 4\uc810 \uc911 3\uc810\uc744 \uc8fc\uc5c8\ub2e4.", "paragraph_answer": "\ud0c0\uc784\uc758 \ub9ac\ucc98\ub4dc \ucf5c\ub9ac\uc2a4 \ub294 \"\uc601\ud654 \uc18d \ud5e4\uc774\uc990\uacfc \uc5b4\uac70\uc2a4\ud130\uc2a4\ub294 \ub208 \uc55e\uc5d0 \ubcf4\uc774\ub294 \uac83\ub4e4\uc744 \uc0dd\uac01\ud558\uba70, \uc0dd\uae30\uac00 \uc788\ub2e4. \uc774 \uc810\uc740 \ub450 \uc0ac\ub78c\uc744 \uc5f0\uae30\ud558\uace0 \uc788\ub294 \ubc30\uc6b0\uac00 \uc80a\uae30 \ub54c\ubb38\uc77c \uac83\uc774\ub2e4.\"\uace0 \ub9d0\ud588\ub2e4. \ub308\ub7ec\uc2a4 \ubaa8\ub2dd \ub274\uc2a4\uc758 \ud06c\ub9ac\uc2a4 \ubd09\uac70\ub294 B+\ub77c\ub294 \ud3c9\uac00\ub97c \ub0b4\ub838\ub2e4 \"\u300a\uc548\ub155, \ud5e4\uc774\uc990\u300b\uc740 \uc7ac\uce58\uc788\uace0 \uc0dd\uae30 \uc788\ub2e4. \ub2e8\uc9c0 \uc870\uae08 \uac10\uc0c1\uc801\uc774\ub2e4.\"\uace0 \ud588\ub2e4. USA \ud22c\ub370\uc774\uc758 \ud074\ub77c\uc6b0\ub514\uc544 \ud478\uc774\uadf8\ub294 4\uc810 \uc911 3.5\uc810\uc744 \uc8fc\uace0, \"\uc88b\uc740 \uac01\ubcf8\uc774\uba70 \uc5f0\uae30\ub3c4 \uc88b\ub2e4. \uc7ac\ubbf8\ub3c4 \uc788\uace0 \ud55c\ud3b8\uc73c\ub85c\ub294 \ud1b5\ucc30\ub825\uc774 \uc788\ub2e4.\"\uace0 \ud3c9\ud558\uc600\ub2e4. \uc5d4\ud130\ud14c\uc778\uba3c\ud2b8 \uc704\ud074\ub9ac\uc758 \ud06c\ub9ac\uc2a4 \ub0b4\uc154\uc6e8\uc774\ud2f0\ub294 \"\uc7ac\ubbf8\uc788\uace0 \ub2ec\ucf64\ud55c \ub9e4\uc6b0 \uac10\ub3d9\uc2a4\ub7ec\uc6b4 \uc601\ud654\"\ub77c\uba70 B \ud3c9\uac00\ub97c \ub0b4\ub838\ub2e4. \uae00\ub85c\ube0c \uc564 \uba54\uc77c\uc758 \uc81c\ud504 \ud398\ubca0\ub808\ub294 \"\uc774 \uc791\ud488\uc740 \uad00\uac1d\uc774 \uc2e4\uc81c \uc0dd\ud65c\uc5d0\uc11c \uac00\uc9c0\ub294 \uae30\ub300\ub97c \ubc30\ubc18\ud560\uc9c0\ub3c4 \ubaa8\ub978\ub2e4. \ud558\uc9c0\ub9cc \uccad\uc18c\ub144\uc744 \uc704\ud55c \uc601\ud654\uc758 \uad00\uc2b5\uc744 \uc2f9\ub451 \uc790\ub974\uace0 \uc788\ub2e4.\"\uace0 \ud558\uc600\ub2e4. \ub9c8\uc774\uc560\ubbf8 \ud5e4\ub7f4\ub4dc\uc758 \ucf54\ub2c8 \uc624\uae00\uc740 \"\uac11\uc790\uae30 \ub530\ub73b\ud574\uc9c0\uac70\ub098 \uc720\uba38\uac00 \uc788\ub294 \ub2ec\ucf64\ud55c \ub85c\ub9e8\uc2a4 \uc601\ud654\ub2e4.\"\ub77c\uace0 \ud558\uc600\ub2e4. \ub514 A.V. \ud074\ub7fd\uc758 A. A. \ub2e4\uc6b0\ub4dc\ub294 B \ud3c9\uac00\ub97c \ub0b4\ub9ac\uba70, \"\ubc88\uac1c \uac19\uc740 \uc704\ud2b8\uac00 \uc788\uc73c\uba70, \uac1c\uc131 \ud48d\ubd80\ud558\uace0 \uc7ac\ub2a5\uc788\ub294 \ubc30\uc6b0\ub4e4\uc5d0 \uc758\ud574 \ub9e4\ub825\uc774 \uc99d\uac00\ud55c\ub2e4.\"\uace0 \ub9d0\ud588\ub2e4. \ub864\ub9c1 \uc2a4\ud1a4\uc758 \ud53c\ud130 \ud2b8\ub798\ubc84\uc2a4\ub294 \"\uc2e0\uc120\ud558\uace0 \uc0dd\uc0dd\ud55c \ub85c\ub9e8\uc2a4 \uc601\ud654\uc774\ub2e4. \uac10\ub3d9\uacfc \uc720\uba38\uac00 \ub118\uce58\uace0 \uc170\uc77c\ub9b0 \uc6b0\ub4e4\ub9ac\uac00 \uc5f0\uae30\ub85c \uc9c8\uc744 \ub192\uc774\uace0 \uc788\ub2e4. \uc170\uc77c\ub9b0 \uc6b0\ub4e4\ub9ac\ub294 \uce74\uba54\ub77c \uc55e\uc5d0\uc11c \uc2e4\uc218\ub97c \ud560 \uc218 \uc5c6\ub2e4\uace0 \ud560 \uc815\ub3c4\ub85c \uc88b\uc740 \ubc30\uc6b0\uc774\ub2e4.\"\ub77c\uba70 \uc170\uc77c\ub9b0 \uc6b0\ub4e4\ub9ac\ub97c \uce6d\ucc2c\ud558\uace0 4\uc810 \uc911 3\uc810\uc744 \uc8fc\uc5c8\ub2e4.", "sentence_answer": "\ud0c0\uc784\uc758 \ub9ac\ucc98\ub4dc \ucf5c\ub9ac\uc2a4 \ub294 \"\uc601\ud654 \uc18d \ud5e4\uc774\uc990\uacfc \uc5b4\uac70\uc2a4\ud130\uc2a4\ub294 \ub208 \uc55e\uc5d0 \ubcf4\uc774\ub294 \uac83\ub4e4\uc744 \uc0dd\uac01\ud558\uba70, \uc0dd\uae30\uac00 \uc788\ub2e4.", "paragraph_id": "6604655-9-1", "paragraph_question": "question: \uc601\ud654 \uc18d \ud5e4\uc774\uc990\uacfc \uc5b4\uac70\uc2a4\ud130\uc2a4\uac00 \uc0dd\uae30\uac00 \uc788\ub2e4\uace0 \ud3c9\ud55c \ud0c0\uc784\uc758 \uc18c\uc18d\uc778\uc740?, context: \ud0c0\uc784\uc758 \ub9ac\ucc98\ub4dc \ucf5c\ub9ac\uc2a4\ub294 \"\uc601\ud654 \uc18d \ud5e4\uc774\uc990\uacfc \uc5b4\uac70\uc2a4\ud130\uc2a4\ub294 \ub208 \uc55e\uc5d0 \ubcf4\uc774\ub294 \uac83\ub4e4\uc744 \uc0dd\uac01\ud558\uba70, \uc0dd\uae30\uac00 \uc788\ub2e4. \uc774 \uc810\uc740 \ub450 \uc0ac\ub78c\uc744 \uc5f0\uae30\ud558\uace0 \uc788\ub294 \ubc30\uc6b0\uac00 \uc80a\uae30 \ub54c\ubb38\uc77c \uac83\uc774\ub2e4.\"\uace0 \ub9d0\ud588\ub2e4. \ub308\ub7ec\uc2a4 \ubaa8\ub2dd \ub274\uc2a4\uc758 \ud06c\ub9ac\uc2a4 \ubd09\uac70\ub294 B+\ub77c\ub294 \ud3c9\uac00\ub97c \ub0b4\ub838\ub2e4 \"\u300a\uc548\ub155, \ud5e4\uc774\uc990\u300b\uc740 \uc7ac\uce58\uc788\uace0 \uc0dd\uae30 \uc788\ub2e4. \ub2e8\uc9c0 \uc870\uae08 \uac10\uc0c1\uc801\uc774\ub2e4.\"\uace0 \ud588\ub2e4. USA \ud22c\ub370\uc774\uc758 \ud074\ub77c\uc6b0\ub514\uc544 \ud478\uc774\uadf8\ub294 4\uc810 \uc911 3.5\uc810\uc744 \uc8fc\uace0, \"\uc88b\uc740 \uac01\ubcf8\uc774\uba70 \uc5f0\uae30\ub3c4 \uc88b\ub2e4. \uc7ac\ubbf8\ub3c4 \uc788\uace0 \ud55c\ud3b8\uc73c\ub85c\ub294 \ud1b5\ucc30\ub825\uc774 \uc788\ub2e4.\"\uace0 \ud3c9\ud558\uc600\ub2e4. \uc5d4\ud130\ud14c\uc778\uba3c\ud2b8 \uc704\ud074\ub9ac\uc758 \ud06c\ub9ac\uc2a4 \ub0b4\uc154\uc6e8\uc774\ud2f0\ub294 \"\uc7ac\ubbf8\uc788\uace0 \ub2ec\ucf64\ud55c \ub9e4\uc6b0 \uac10\ub3d9\uc2a4\ub7ec\uc6b4 \uc601\ud654\"\ub77c\uba70 B \ud3c9\uac00\ub97c \ub0b4\ub838\ub2e4. \uae00\ub85c\ube0c \uc564 \uba54\uc77c\uc758 \uc81c\ud504 \ud398\ubca0\ub808\ub294 \"\uc774 \uc791\ud488\uc740 \uad00\uac1d\uc774 \uc2e4\uc81c \uc0dd\ud65c\uc5d0\uc11c \uac00\uc9c0\ub294 \uae30\ub300\ub97c \ubc30\ubc18\ud560\uc9c0\ub3c4 \ubaa8\ub978\ub2e4. \ud558\uc9c0\ub9cc \uccad\uc18c\ub144\uc744 \uc704\ud55c \uc601\ud654\uc758 \uad00\uc2b5\uc744 \uc2f9\ub451 \uc790\ub974\uace0 \uc788\ub2e4.\"\uace0 \ud558\uc600\ub2e4. \ub9c8\uc774\uc560\ubbf8 \ud5e4\ub7f4\ub4dc\uc758 \ucf54\ub2c8 \uc624\uae00\uc740 \"\uac11\uc790\uae30 \ub530\ub73b\ud574\uc9c0\uac70\ub098 \uc720\uba38\uac00 \uc788\ub294 \ub2ec\ucf64\ud55c \ub85c\ub9e8\uc2a4 \uc601\ud654\ub2e4.\"\ub77c\uace0 \ud558\uc600\ub2e4. \ub514 A.V. \ud074\ub7fd\uc758 A. A. \ub2e4\uc6b0\ub4dc\ub294 B \ud3c9\uac00\ub97c \ub0b4\ub9ac\uba70, \"\ubc88\uac1c \uac19\uc740 \uc704\ud2b8\uac00 \uc788\uc73c\uba70, \uac1c\uc131 \ud48d\ubd80\ud558\uace0 \uc7ac\ub2a5\uc788\ub294 \ubc30\uc6b0\ub4e4\uc5d0 \uc758\ud574 \ub9e4\ub825\uc774 \uc99d\uac00\ud55c\ub2e4.\"\uace0 \ub9d0\ud588\ub2e4. \ub864\ub9c1 \uc2a4\ud1a4\uc758 \ud53c\ud130 \ud2b8\ub798\ubc84\uc2a4\ub294 \"\uc2e0\uc120\ud558\uace0 \uc0dd\uc0dd\ud55c \ub85c\ub9e8\uc2a4 \uc601\ud654\uc774\ub2e4. \uac10\ub3d9\uacfc \uc720\uba38\uac00 \ub118\uce58\uace0 \uc170\uc77c\ub9b0 \uc6b0\ub4e4\ub9ac\uac00 \uc5f0\uae30\ub85c \uc9c8\uc744 \ub192\uc774\uace0 \uc788\ub2e4. \uc170\uc77c\ub9b0 \uc6b0\ub4e4\ub9ac\ub294 \uce74\uba54\ub77c \uc55e\uc5d0\uc11c \uc2e4\uc218\ub97c \ud560 \uc218 \uc5c6\ub2e4\uace0 \ud560 \uc815\ub3c4\ub85c \uc88b\uc740 \ubc30\uc6b0\uc774\ub2e4.\"\ub77c\uba70 \uc170\uc77c\ub9b0 \uc6b0\ub4e4\ub9ac\ub97c \uce6d\ucc2c\ud558\uace0 4\uc810 \uc911 3\uc810\uc744 \uc8fc\uc5c8\ub2e4."} {"question": "\ud398\ud2b8\ub77c\ub974\uce74\ub294 \uace0\ub300 \uadf8\ub9ac\uc2a4 \ub85c\ub9c8\uc758 \uc720\uc0b0\uc744 \uc7ac\ubc1c\uacac\ud55c \ub4a4 \ubb34\uc5c7\ud558\uc5ec\uc57c \ud55c\ub2e4\uace0 \uc0dd\uac01\ud588\ub294\uac00?", "paragraph": "\ub2e8\ud14c\ubcf4\ub2e4 \ud6c4\ub300 \uc0ac\ub78c\uc778 \ud398\ud2b8\ub77c\ub974\uce74\ub294 \ub85c\ub9c8 \uc81c\uad6d \uc2dc\ub300\uc5d0 \uc778\uac04\uc774 \ucd5c\uace0\uc758 \uac83\uc744 \uc131\ucde8\ud588\uace0 \uadf8 \uc774\ud6c4 \uc810\ucc28 \ubd80\ud328\ud558\uc5ec \uc911\uc138 \uc554\ud751\uc2dc\ub300\uae4c\uc9c0 \uc774\ub974\ub800\ub2e4\uace0 \ubcf4\uc558\ub2e4. \uadf8\ub294 \uc5ed\uc0ac\ub97c \uc885\uad50\uc801 \uc0ac\uac74\uc758 \uc5f0\uc18d\uc774 \uc544\ub2c8\ub77c \uc0ac\ud68c \ubb38\ud654\uc801\uc778 \uc9c4\ubcf4\ub85c \uac04\uc8fc\ud588\uc73c\ubbc0\ub85c \uadf8\ub294 \uace0\ub300 \uadf8\ub9ac\uc2a4 \ub85c\ub9c8\uc758 \uc720\uc0b0\uc744 \uc7ac\ubc1c\uacac\ud558\uc5ec \"\uc7ac\uc0dd\"\uc2dc\ucf1c\uc57c \ud55c\ub2e4\uace0 \uc0dd\uac01\ud588\ub2e4. \uadf8\ub294 \uace0\uc804\ubb38\ud5cc\uc744 \ubaa8\uc73c\uace0 \ub77c\ud2f4\uc5b4\ub85c \uc2dc\uc640 \ucc45\uc744 \uc4f0\ub2e4\uac00 \uc774\ub7f0\uc2dd\uc73c\ub85c \uace0\uc804 \uad50\uc591\uc744 \ubaa8\uc544 \uc778\uac04\uc758 \uc5b4\ub5bb\uac8c \uc0b4\uc544\uc57c \ud558\ub294\uac00\ub97c \uc0ac\uc0c9\ud558\ub294 \ubc29\uc2dd\uc744 '\uc778\ubb38\uc8fc\uc758'\ub77c\uace0 \ubd88\ub800\ub2e4. \uadf8\ub294 \uc18d\uc5b4\ub85c \ucc45\uc744 \uc4f0\uae30 \uc2dc\uc791\ud588\uc73c\uba70 \uc774\ub7ec\ud55c \uc628\uace0\uc9c0\uc2e0\uc801\uc778 \ud0dc\ub3c4\ub294 \uc774\ud6c4\uc758 \uc608\uc220, \uacfc\ud559 \ub4f1 \uc5ec\ub7ec \ubd84\uc57c\uc5d0 \ud070 \uc601\ud5a5\uc744 \ubbf8\ucce4\ub2e4. \ud398\ud2b8\ub77c\ub974\uce74\ub294 '\ub9ac\ube44\uc6b0\uc2a4'\uc758 \uc5ed\uc0ac\uc640 '\ud0a4\ucf00\ub85c'\uc758 \ub3c4\ub355\ucca0\ud559\uc5d0 \uad00\uc2ec\uc744 \ubcf4\uc600\uace0 \ucd5c\ucd08\uc758 \uc778\ubb38\uc8fc\uc758\uc790\ub77c\ub294 \ud3c9\uac00\ub97c \ubc1b\uace0 \uc788\ub2e4.", "answer": "\uc7ac\uc0dd", "sentence": "\uadf8\ub294 \uc5ed\uc0ac\ub97c \uc885\uad50\uc801 \uc0ac\uac74\uc758 \uc5f0\uc18d\uc774 \uc544\ub2c8\ub77c \uc0ac\ud68c \ubb38\ud654\uc801\uc778 \uc9c4\ubcf4\ub85c \uac04\uc8fc\ud588\uc73c\ubbc0\ub85c \uadf8\ub294 \uace0\ub300 \uadf8\ub9ac\uc2a4 \ub85c\ub9c8\uc758 \uc720\uc0b0\uc744 \uc7ac\ubc1c\uacac\ud558\uc5ec \" \uc7ac\uc0dd \"\uc2dc\ucf1c\uc57c \ud55c\ub2e4\uace0 \uc0dd\uac01\ud588\ub2e4.", "paragraph_sentence": "\ub2e8\ud14c\ubcf4\ub2e4 \ud6c4\ub300 \uc0ac\ub78c\uc778 \ud398\ud2b8\ub77c\ub974\uce74\ub294 \ub85c\ub9c8 \uc81c\uad6d \uc2dc\ub300\uc5d0 \uc778\uac04\uc774 \ucd5c\uace0\uc758 \uac83\uc744 \uc131\ucde8\ud588\uace0 \uadf8 \uc774\ud6c4 \uc810\ucc28 \ubd80\ud328\ud558\uc5ec \uc911\uc138 \uc554\ud751\uc2dc\ub300\uae4c\uc9c0 \uc774\ub974\ub800\ub2e4\uace0 \ubcf4\uc558\ub2e4. \uadf8\ub294 \uc5ed\uc0ac\ub97c \uc885\uad50\uc801 \uc0ac\uac74\uc758 \uc5f0\uc18d\uc774 \uc544\ub2c8\ub77c \uc0ac\ud68c \ubb38\ud654\uc801\uc778 \uc9c4\ubcf4\ub85c \uac04\uc8fc\ud588\uc73c\ubbc0\ub85c \uadf8\ub294 \uace0\ub300 \uadf8\ub9ac\uc2a4 \ub85c\ub9c8\uc758 \uc720\uc0b0\uc744 \uc7ac\ubc1c\uacac\ud558\uc5ec \" \uc7ac\uc0dd \"\uc2dc\ucf1c\uc57c \ud55c\ub2e4\uace0 \uc0dd\uac01\ud588\ub2e4. \uadf8\ub294 \uace0\uc804\ubb38\ud5cc\uc744 \ubaa8\uc73c\uace0 \ub77c\ud2f4\uc5b4\ub85c \uc2dc\uc640 \ucc45\uc744 \uc4f0\ub2e4\uac00 \uc774\ub7f0\uc2dd\uc73c\ub85c \uace0\uc804 \uad50\uc591\uc744 \ubaa8\uc544 \uc778\uac04\uc758 \uc5b4\ub5bb\uac8c \uc0b4\uc544\uc57c \ud558\ub294\uac00\ub97c \uc0ac\uc0c9\ud558\ub294 \ubc29\uc2dd\uc744 '\uc778\ubb38\uc8fc\uc758'\ub77c\uace0 \ubd88\ub800\ub2e4. \uadf8\ub294 \uc18d\uc5b4\ub85c \ucc45\uc744 \uc4f0\uae30 \uc2dc\uc791\ud588\uc73c\uba70 \uc774\ub7ec\ud55c \uc628\uace0\uc9c0\uc2e0\uc801\uc778 \ud0dc\ub3c4\ub294 \uc774\ud6c4\uc758 \uc608\uc220, \uacfc\ud559 \ub4f1 \uc5ec\ub7ec \ubd84\uc57c\uc5d0 \ud070 \uc601\ud5a5\uc744 \ubbf8\ucce4\ub2e4. \ud398\ud2b8\ub77c\ub974\uce74\ub294 '\ub9ac\ube44\uc6b0\uc2a4'\uc758 \uc5ed\uc0ac\uc640 '\ud0a4\ucf00\ub85c'\uc758 \ub3c4\ub355\ucca0\ud559\uc5d0 \uad00\uc2ec\uc744 \ubcf4\uc600\uace0 \ucd5c\ucd08\uc758 \uc778\ubb38\uc8fc\uc758\uc790\ub77c\ub294 \ud3c9\uac00\ub97c \ubc1b\uace0 \uc788\ub2e4.", "paragraph_answer": "\ub2e8\ud14c\ubcf4\ub2e4 \ud6c4\ub300 \uc0ac\ub78c\uc778 \ud398\ud2b8\ub77c\ub974\uce74\ub294 \ub85c\ub9c8 \uc81c\uad6d \uc2dc\ub300\uc5d0 \uc778\uac04\uc774 \ucd5c\uace0\uc758 \uac83\uc744 \uc131\ucde8\ud588\uace0 \uadf8 \uc774\ud6c4 \uc810\ucc28 \ubd80\ud328\ud558\uc5ec \uc911\uc138 \uc554\ud751\uc2dc\ub300\uae4c\uc9c0 \uc774\ub974\ub800\ub2e4\uace0 \ubcf4\uc558\ub2e4. \uadf8\ub294 \uc5ed\uc0ac\ub97c \uc885\uad50\uc801 \uc0ac\uac74\uc758 \uc5f0\uc18d\uc774 \uc544\ub2c8\ub77c \uc0ac\ud68c \ubb38\ud654\uc801\uc778 \uc9c4\ubcf4\ub85c \uac04\uc8fc\ud588\uc73c\ubbc0\ub85c \uadf8\ub294 \uace0\ub300 \uadf8\ub9ac\uc2a4 \ub85c\ub9c8\uc758 \uc720\uc0b0\uc744 \uc7ac\ubc1c\uacac\ud558\uc5ec \" \uc7ac\uc0dd \"\uc2dc\ucf1c\uc57c \ud55c\ub2e4\uace0 \uc0dd\uac01\ud588\ub2e4. \uadf8\ub294 \uace0\uc804\ubb38\ud5cc\uc744 \ubaa8\uc73c\uace0 \ub77c\ud2f4\uc5b4\ub85c \uc2dc\uc640 \ucc45\uc744 \uc4f0\ub2e4\uac00 \uc774\ub7f0\uc2dd\uc73c\ub85c \uace0\uc804 \uad50\uc591\uc744 \ubaa8\uc544 \uc778\uac04\uc758 \uc5b4\ub5bb\uac8c \uc0b4\uc544\uc57c \ud558\ub294\uac00\ub97c \uc0ac\uc0c9\ud558\ub294 \ubc29\uc2dd\uc744 '\uc778\ubb38\uc8fc\uc758'\ub77c\uace0 \ubd88\ub800\ub2e4. \uadf8\ub294 \uc18d\uc5b4\ub85c \ucc45\uc744 \uc4f0\uae30 \uc2dc\uc791\ud588\uc73c\uba70 \uc774\ub7ec\ud55c \uc628\uace0\uc9c0\uc2e0\uc801\uc778 \ud0dc\ub3c4\ub294 \uc774\ud6c4\uc758 \uc608\uc220, \uacfc\ud559 \ub4f1 \uc5ec\ub7ec \ubd84\uc57c\uc5d0 \ud070 \uc601\ud5a5\uc744 \ubbf8\ucce4\ub2e4. \ud398\ud2b8\ub77c\ub974\uce74\ub294 '\ub9ac\ube44\uc6b0\uc2a4'\uc758 \uc5ed\uc0ac\uc640 '\ud0a4\ucf00\ub85c'\uc758 \ub3c4\ub355\ucca0\ud559\uc5d0 \uad00\uc2ec\uc744 \ubcf4\uc600\uace0 \ucd5c\ucd08\uc758 \uc778\ubb38\uc8fc\uc758\uc790\ub77c\ub294 \ud3c9\uac00\ub97c \ubc1b\uace0 \uc788\ub2e4.", "sentence_answer": "\uadf8\ub294 \uc5ed\uc0ac\ub97c \uc885\uad50\uc801 \uc0ac\uac74\uc758 \uc5f0\uc18d\uc774 \uc544\ub2c8\ub77c \uc0ac\ud68c \ubb38\ud654\uc801\uc778 \uc9c4\ubcf4\ub85c \uac04\uc8fc\ud588\uc73c\ubbc0\ub85c \uadf8\ub294 \uace0\ub300 \uadf8\ub9ac\uc2a4 \ub85c\ub9c8\uc758 \uc720\uc0b0\uc744 \uc7ac\ubc1c\uacac\ud558\uc5ec \" \uc7ac\uc0dd \"\uc2dc\ucf1c\uc57c \ud55c\ub2e4\uace0 \uc0dd\uac01\ud588\ub2e4.", "paragraph_id": "6656981-1-2", "paragraph_question": "question: \ud398\ud2b8\ub77c\ub974\uce74\ub294 \uace0\ub300 \uadf8\ub9ac\uc2a4 \ub85c\ub9c8\uc758 \uc720\uc0b0\uc744 \uc7ac\ubc1c\uacac\ud55c \ub4a4 \ubb34\uc5c7\ud558\uc5ec\uc57c \ud55c\ub2e4\uace0 \uc0dd\uac01\ud588\ub294\uac00?, context: \ub2e8\ud14c\ubcf4\ub2e4 \ud6c4\ub300 \uc0ac\ub78c\uc778 \ud398\ud2b8\ub77c\ub974\uce74\ub294 \ub85c\ub9c8 \uc81c\uad6d \uc2dc\ub300\uc5d0 \uc778\uac04\uc774 \ucd5c\uace0\uc758 \uac83\uc744 \uc131\ucde8\ud588\uace0 \uadf8 \uc774\ud6c4 \uc810\ucc28 \ubd80\ud328\ud558\uc5ec \uc911\uc138 \uc554\ud751\uc2dc\ub300\uae4c\uc9c0 \uc774\ub974\ub800\ub2e4\uace0 \ubcf4\uc558\ub2e4. \uadf8\ub294 \uc5ed\uc0ac\ub97c \uc885\uad50\uc801 \uc0ac\uac74\uc758 \uc5f0\uc18d\uc774 \uc544\ub2c8\ub77c \uc0ac\ud68c \ubb38\ud654\uc801\uc778 \uc9c4\ubcf4\ub85c \uac04\uc8fc\ud588\uc73c\ubbc0\ub85c \uadf8\ub294 \uace0\ub300 \uadf8\ub9ac\uc2a4 \ub85c\ub9c8\uc758 \uc720\uc0b0\uc744 \uc7ac\ubc1c\uacac\ud558\uc5ec \"\uc7ac\uc0dd\"\uc2dc\ucf1c\uc57c \ud55c\ub2e4\uace0 \uc0dd\uac01\ud588\ub2e4. \uadf8\ub294 \uace0\uc804\ubb38\ud5cc\uc744 \ubaa8\uc73c\uace0 \ub77c\ud2f4\uc5b4\ub85c \uc2dc\uc640 \ucc45\uc744 \uc4f0\ub2e4\uac00 \uc774\ub7f0\uc2dd\uc73c\ub85c \uace0\uc804 \uad50\uc591\uc744 \ubaa8\uc544 \uc778\uac04\uc758 \uc5b4\ub5bb\uac8c \uc0b4\uc544\uc57c \ud558\ub294\uac00\ub97c \uc0ac\uc0c9\ud558\ub294 \ubc29\uc2dd\uc744 '\uc778\ubb38\uc8fc\uc758'\ub77c\uace0 \ubd88\ub800\ub2e4. \uadf8\ub294 \uc18d\uc5b4\ub85c \ucc45\uc744 \uc4f0\uae30 \uc2dc\uc791\ud588\uc73c\uba70 \uc774\ub7ec\ud55c \uc628\uace0\uc9c0\uc2e0\uc801\uc778 \ud0dc\ub3c4\ub294 \uc774\ud6c4\uc758 \uc608\uc220, \uacfc\ud559 \ub4f1 \uc5ec\ub7ec \ubd84\uc57c\uc5d0 \ud070 \uc601\ud5a5\uc744 \ubbf8\ucce4\ub2e4. \ud398\ud2b8\ub77c\ub974\uce74\ub294 '\ub9ac\ube44\uc6b0\uc2a4'\uc758 \uc5ed\uc0ac\uc640 '\ud0a4\ucf00\ub85c'\uc758 \ub3c4\ub355\ucca0\ud559\uc5d0 \uad00\uc2ec\uc744 \ubcf4\uc600\uace0 \ucd5c\ucd08\uc758 \uc778\ubb38\uc8fc\uc758\uc790\ub77c\ub294 \ud3c9\uac00\ub97c \ubc1b\uace0 \uc788\ub2e4."} {"question": "1411\ub144 \ud3f4\ub780\ub4dc\uc640 \ub3c5\uc77c \uae30\uc0ac\ub2e8 \uc0ac\uc774\uc5d0 \uc774\ub8e8\uc5b4\uc9c4 \ud3c9\ud654 \ud611\uc815\uc774 \uccb4\uacb0\ub41c \uc7a5\uc18c\ub294 \uc5b4\ub514\uc778\uac00?", "paragraph": "200\ub144 \ub3d9\uc548 \ud3f4\ub780\ub4dc-\ub9ac\ud22c\uc544\ub2c8\uc544 \uc591\uad6d\uc740 \uc57c\uae30\uc5d0\uc6b0\uc6cc \uc655\uc870\uc5d0 \uc758\ud574\uc11c \uacb0\ud569\ub418\uc5c8\ub2e4. \ud3f4\ub780\ub4dc \uc655\uc740 \ub3d9\uc2dc\uc5d0 \ub9ac\ud22c\uc544\ub2c8\uc544\uc758 \ub300\uacf5\uc774\uc5c8\ub2e4. \uadf8\ub7ec\ub098 \ub9ac\ud22c\uc544\ub2c8\uc544\uc640 \ud3f4\ub780\ub4dc\uac00 \ud558\ub098\uc758 \uad6d\uac00\ub85c \ud1b5\ud569\ub418\uc9c0\ub294 \uc54a\uc558\ub2e4. \ud3f4\ub780\ub4dc-\ub9ac\ud22c\uc544\ub2c8\uc544\uac04\uc5d0 \ud611\uc815\uc774 \uccb4\uacb0\ub428\uc73c\ub85c\uc368 \ub3c5\uc77c \uae30\uc0ac\ub2e8\uacfc\uc758 \uc804\uc7c1\uc774 \ubd88\uac00\ud53c\ud558\uac8c \ub418\uc5c8\ub2e4. \uc804\uc7c1 \uc911 \uadf8\ub8ec\ubc1c\ud2b8(Grunwald)\uc5d0\uc11c \ud3f4\ub780\ub4dc-\ub9ac\ud22c\uc544\ub2c8\uc544\uc758 \uc2b9\ub9ac\ub294 \ub300\ub2e8\ud788 \ud070 \uc758\ubbf8\ub97c \uac00\uc9c0\ub294 \uc5ed\uc0ac\uc801\uc778 \ub300\uc0ac\uac74\uc774\ub2e4. \ub3c5\uc77c \uae30\uc0ac\ub2e8\uc740 \ud3f4\ub780\ub4dc \ub545\uc5d0\uc11c \ucc98\uc74c\uc73c\ub85c \uc774\uc640 \uac19\uc740 \ud328\ubc30\ub97c \uacaa\uac8c \ub418\uc5c8\ub358 \uac83\uc774\ub2e4. \uc774 \ud328\ubc30\ub97c \uae30\uc810\uc73c\ub85c \ub3c5\uc77c \uae30\uc0ac\ub2e8\uc740 \uc11c\uc11c\ud788 \uba78\ub9dd\ud558\uae30 \uc2dc\uc791\ud588\ub2e4. \uadf8\ub7ec\ub2e4\uac00 1411\ub144 \ud1a0\ub8ec\uc5d0\uc11c \ud3f4\ub780\ub4dc\uc640 \ub3c5\uc77c \uae30\uc0ac\ub2e8 \uc0ac\uc774\uc5d0 \ud3c9\ud654 \ud611\uc815\uc774 \uccb4\uacb0\ub418\uc5c8\ub2e4. \uc774 \ud611\uc815\uc5d0 \ub530\ub77c\uc11c \uc988\ubb34\uce58\ub294 \ub9ac\ud22c\uc544\ub2c8\uc544\uc5d0 \uc591\ub3c4\ub418\uc5c8\ub2e4. \uadf8\ub7ec\ub098 \uadf8\uac83\ub3c4 \uc57c\uae30\uc5d0\uc624\uac00 \uc0b4\uc544 \uc788\ub294 \ub3d9\uc548\uc5d0\ub9cc \ub9ac\ud22c\uc544\ub2c8\uc544\uc5d0 \uc18d\ud558\uace0 \uc57c\uae30\uc5d0\uc624\uac00 \uc0ac\ub9dd\ud55c \ud6c4\uc5d0\ub294 \ub2e4\uc2dc \ub3c5\uc77c \uae30\uc0ac\ub2e8\uc5d0\uac8c \ub118\uaca8 \uc8fc\uae30\ub85c \ud569\uc758\ub418\uc5c8\ub2e4. \ube44\ub85d \ud3f4\ub780\ub4dc\ub294 \uc804\uc7c1\uc5d0\uc11c\ub294 \ud06c\uac8c \uc2b9\ub9ac\ud588\uc9c0\ub9cc \uc815\uce58\uc801\uc73c\ub85c \uadf8\uac83\uc744 \uc81c\ub300\ub85c \uc774\uc6a9\ud558\uc9c0 \ubabb\ud588\ub2e4.", "answer": "\ud1a0\ub8ec", "sentence": "\uadf8\ub7ec\ub2e4\uac00 1411\ub144 \ud1a0\ub8ec \uc5d0\uc11c \ud3f4\ub780\ub4dc\uc640 \ub3c5\uc77c \uae30\uc0ac\ub2e8 \uc0ac\uc774\uc5d0 \ud3c9\ud654 \ud611\uc815\uc774 \uccb4\uacb0\ub418\uc5c8\ub2e4.", "paragraph_sentence": "200\ub144 \ub3d9\uc548 \ud3f4\ub780\ub4dc-\ub9ac\ud22c\uc544\ub2c8\uc544 \uc591\uad6d\uc740 \uc57c\uae30\uc5d0\uc6b0\uc6cc \uc655\uc870\uc5d0 \uc758\ud574\uc11c \uacb0\ud569\ub418\uc5c8\ub2e4. \ud3f4\ub780\ub4dc \uc655\uc740 \ub3d9\uc2dc\uc5d0 \ub9ac\ud22c\uc544\ub2c8\uc544\uc758 \ub300\uacf5\uc774\uc5c8\ub2e4. \uadf8\ub7ec\ub098 \ub9ac\ud22c\uc544\ub2c8\uc544\uc640 \ud3f4\ub780\ub4dc\uac00 \ud558\ub098\uc758 \uad6d\uac00\ub85c \ud1b5\ud569\ub418\uc9c0\ub294 \uc54a\uc558\ub2e4. \ud3f4\ub780\ub4dc-\ub9ac\ud22c\uc544\ub2c8\uc544\uac04\uc5d0 \ud611\uc815\uc774 \uccb4\uacb0\ub428\uc73c\ub85c\uc368 \ub3c5\uc77c \uae30\uc0ac\ub2e8\uacfc\uc758 \uc804\uc7c1\uc774 \ubd88\uac00\ud53c\ud558\uac8c \ub418\uc5c8\ub2e4. \uc804\uc7c1 \uc911 \uadf8\ub8ec\ubc1c\ud2b8(Grunwald)\uc5d0\uc11c \ud3f4\ub780\ub4dc-\ub9ac\ud22c\uc544\ub2c8\uc544\uc758 \uc2b9\ub9ac\ub294 \ub300\ub2e8\ud788 \ud070 \uc758\ubbf8\ub97c \uac00\uc9c0\ub294 \uc5ed\uc0ac\uc801\uc778 \ub300\uc0ac\uac74\uc774\ub2e4. \ub3c5\uc77c \uae30\uc0ac\ub2e8\uc740 \ud3f4\ub780\ub4dc \ub545\uc5d0\uc11c \ucc98\uc74c\uc73c\ub85c \uc774\uc640 \uac19\uc740 \ud328\ubc30\ub97c \uacaa\uac8c \ub418\uc5c8\ub358 \uac83\uc774\ub2e4. \uc774 \ud328\ubc30\ub97c \uae30\uc810\uc73c\ub85c \ub3c5\uc77c \uae30\uc0ac\ub2e8\uc740 \uc11c\uc11c\ud788 \uba78\ub9dd\ud558\uae30 \uc2dc\uc791\ud588\ub2e4. \uadf8\ub7ec\ub2e4\uac00 1411\ub144 \ud1a0\ub8ec \uc5d0\uc11c \ud3f4\ub780\ub4dc\uc640 \ub3c5\uc77c \uae30\uc0ac\ub2e8 \uc0ac\uc774\uc5d0 \ud3c9\ud654 \ud611\uc815\uc774 \uccb4\uacb0\ub418\uc5c8\ub2e4. \uc774 \ud611\uc815\uc5d0 \ub530\ub77c\uc11c \uc988\ubb34\uce58\ub294 \ub9ac\ud22c\uc544\ub2c8\uc544\uc5d0 \uc591\ub3c4\ub418\uc5c8\ub2e4. \uadf8\ub7ec\ub098 \uadf8\uac83\ub3c4 \uc57c\uae30\uc5d0\uc624\uac00 \uc0b4\uc544 \uc788\ub294 \ub3d9\uc548\uc5d0\ub9cc \ub9ac\ud22c\uc544\ub2c8\uc544\uc5d0 \uc18d\ud558\uace0 \uc57c\uae30\uc5d0\uc624\uac00 \uc0ac\ub9dd\ud55c \ud6c4\uc5d0\ub294 \ub2e4\uc2dc \ub3c5\uc77c \uae30\uc0ac\ub2e8\uc5d0\uac8c \ub118\uaca8 \uc8fc\uae30\ub85c \ud569\uc758\ub418\uc5c8\ub2e4. \ube44\ub85d \ud3f4\ub780\ub4dc\ub294 \uc804\uc7c1\uc5d0\uc11c\ub294 \ud06c\uac8c \uc2b9\ub9ac\ud588\uc9c0\ub9cc \uc815\uce58\uc801\uc73c\ub85c \uadf8\uac83\uc744 \uc81c\ub300\ub85c \uc774\uc6a9\ud558\uc9c0 \ubabb\ud588\ub2e4.", "paragraph_answer": "200\ub144 \ub3d9\uc548 \ud3f4\ub780\ub4dc-\ub9ac\ud22c\uc544\ub2c8\uc544 \uc591\uad6d\uc740 \uc57c\uae30\uc5d0\uc6b0\uc6cc \uc655\uc870\uc5d0 \uc758\ud574\uc11c \uacb0\ud569\ub418\uc5c8\ub2e4. \ud3f4\ub780\ub4dc \uc655\uc740 \ub3d9\uc2dc\uc5d0 \ub9ac\ud22c\uc544\ub2c8\uc544\uc758 \ub300\uacf5\uc774\uc5c8\ub2e4. \uadf8\ub7ec\ub098 \ub9ac\ud22c\uc544\ub2c8\uc544\uc640 \ud3f4\ub780\ub4dc\uac00 \ud558\ub098\uc758 \uad6d\uac00\ub85c \ud1b5\ud569\ub418\uc9c0\ub294 \uc54a\uc558\ub2e4. \ud3f4\ub780\ub4dc-\ub9ac\ud22c\uc544\ub2c8\uc544\uac04\uc5d0 \ud611\uc815\uc774 \uccb4\uacb0\ub428\uc73c\ub85c\uc368 \ub3c5\uc77c \uae30\uc0ac\ub2e8\uacfc\uc758 \uc804\uc7c1\uc774 \ubd88\uac00\ud53c\ud558\uac8c \ub418\uc5c8\ub2e4. \uc804\uc7c1 \uc911 \uadf8\ub8ec\ubc1c\ud2b8(Grunwald)\uc5d0\uc11c \ud3f4\ub780\ub4dc-\ub9ac\ud22c\uc544\ub2c8\uc544\uc758 \uc2b9\ub9ac\ub294 \ub300\ub2e8\ud788 \ud070 \uc758\ubbf8\ub97c \uac00\uc9c0\ub294 \uc5ed\uc0ac\uc801\uc778 \ub300\uc0ac\uac74\uc774\ub2e4. \ub3c5\uc77c \uae30\uc0ac\ub2e8\uc740 \ud3f4\ub780\ub4dc \ub545\uc5d0\uc11c \ucc98\uc74c\uc73c\ub85c \uc774\uc640 \uac19\uc740 \ud328\ubc30\ub97c \uacaa\uac8c \ub418\uc5c8\ub358 \uac83\uc774\ub2e4. \uc774 \ud328\ubc30\ub97c \uae30\uc810\uc73c\ub85c \ub3c5\uc77c \uae30\uc0ac\ub2e8\uc740 \uc11c\uc11c\ud788 \uba78\ub9dd\ud558\uae30 \uc2dc\uc791\ud588\ub2e4. \uadf8\ub7ec\ub2e4\uac00 1411\ub144 \ud1a0\ub8ec \uc5d0\uc11c \ud3f4\ub780\ub4dc\uc640 \ub3c5\uc77c \uae30\uc0ac\ub2e8 \uc0ac\uc774\uc5d0 \ud3c9\ud654 \ud611\uc815\uc774 \uccb4\uacb0\ub418\uc5c8\ub2e4. \uc774 \ud611\uc815\uc5d0 \ub530\ub77c\uc11c \uc988\ubb34\uce58\ub294 \ub9ac\ud22c\uc544\ub2c8\uc544\uc5d0 \uc591\ub3c4\ub418\uc5c8\ub2e4. \uadf8\ub7ec\ub098 \uadf8\uac83\ub3c4 \uc57c\uae30\uc5d0\uc624\uac00 \uc0b4\uc544 \uc788\ub294 \ub3d9\uc548\uc5d0\ub9cc \ub9ac\ud22c\uc544\ub2c8\uc544\uc5d0 \uc18d\ud558\uace0 \uc57c\uae30\uc5d0\uc624\uac00 \uc0ac\ub9dd\ud55c \ud6c4\uc5d0\ub294 \ub2e4\uc2dc \ub3c5\uc77c \uae30\uc0ac\ub2e8\uc5d0\uac8c \ub118\uaca8 \uc8fc\uae30\ub85c \ud569\uc758\ub418\uc5c8\ub2e4. \ube44\ub85d \ud3f4\ub780\ub4dc\ub294 \uc804\uc7c1\uc5d0\uc11c\ub294 \ud06c\uac8c \uc2b9\ub9ac\ud588\uc9c0\ub9cc \uc815\uce58\uc801\uc73c\ub85c \uadf8\uac83\uc744 \uc81c\ub300\ub85c \uc774\uc6a9\ud558\uc9c0 \ubabb\ud588\ub2e4.", "sentence_answer": "\uadf8\ub7ec\ub2e4\uac00 1411\ub144 \ud1a0\ub8ec \uc5d0\uc11c \ud3f4\ub780\ub4dc\uc640 \ub3c5\uc77c \uae30\uc0ac\ub2e8 \uc0ac\uc774\uc5d0 \ud3c9\ud654 \ud611\uc815\uc774 \uccb4\uacb0\ub418\uc5c8\ub2e4.", "paragraph_id": "6568268-7-0", "paragraph_question": "question: 1411\ub144 \ud3f4\ub780\ub4dc\uc640 \ub3c5\uc77c \uae30\uc0ac\ub2e8 \uc0ac\uc774\uc5d0 \uc774\ub8e8\uc5b4\uc9c4 \ud3c9\ud654 \ud611\uc815\uc774 \uccb4\uacb0\ub41c \uc7a5\uc18c\ub294 \uc5b4\ub514\uc778\uac00?, context: 200\ub144 \ub3d9\uc548 \ud3f4\ub780\ub4dc-\ub9ac\ud22c\uc544\ub2c8\uc544 \uc591\uad6d\uc740 \uc57c\uae30\uc5d0\uc6b0\uc6cc \uc655\uc870\uc5d0 \uc758\ud574\uc11c \uacb0\ud569\ub418\uc5c8\ub2e4. \ud3f4\ub780\ub4dc \uc655\uc740 \ub3d9\uc2dc\uc5d0 \ub9ac\ud22c\uc544\ub2c8\uc544\uc758 \ub300\uacf5\uc774\uc5c8\ub2e4. \uadf8\ub7ec\ub098 \ub9ac\ud22c\uc544\ub2c8\uc544\uc640 \ud3f4\ub780\ub4dc\uac00 \ud558\ub098\uc758 \uad6d\uac00\ub85c \ud1b5\ud569\ub418\uc9c0\ub294 \uc54a\uc558\ub2e4. \ud3f4\ub780\ub4dc-\ub9ac\ud22c\uc544\ub2c8\uc544\uac04\uc5d0 \ud611\uc815\uc774 \uccb4\uacb0\ub428\uc73c\ub85c\uc368 \ub3c5\uc77c \uae30\uc0ac\ub2e8\uacfc\uc758 \uc804\uc7c1\uc774 \ubd88\uac00\ud53c\ud558\uac8c \ub418\uc5c8\ub2e4. \uc804\uc7c1 \uc911 \uadf8\ub8ec\ubc1c\ud2b8(Grunwald)\uc5d0\uc11c \ud3f4\ub780\ub4dc-\ub9ac\ud22c\uc544\ub2c8\uc544\uc758 \uc2b9\ub9ac\ub294 \ub300\ub2e8\ud788 \ud070 \uc758\ubbf8\ub97c \uac00\uc9c0\ub294 \uc5ed\uc0ac\uc801\uc778 \ub300\uc0ac\uac74\uc774\ub2e4. \ub3c5\uc77c \uae30\uc0ac\ub2e8\uc740 \ud3f4\ub780\ub4dc \ub545\uc5d0\uc11c \ucc98\uc74c\uc73c\ub85c \uc774\uc640 \uac19\uc740 \ud328\ubc30\ub97c \uacaa\uac8c \ub418\uc5c8\ub358 \uac83\uc774\ub2e4. \uc774 \ud328\ubc30\ub97c \uae30\uc810\uc73c\ub85c \ub3c5\uc77c \uae30\uc0ac\ub2e8\uc740 \uc11c\uc11c\ud788 \uba78\ub9dd\ud558\uae30 \uc2dc\uc791\ud588\ub2e4. \uadf8\ub7ec\ub2e4\uac00 1411\ub144 \ud1a0\ub8ec\uc5d0\uc11c \ud3f4\ub780\ub4dc\uc640 \ub3c5\uc77c \uae30\uc0ac\ub2e8 \uc0ac\uc774\uc5d0 \ud3c9\ud654 \ud611\uc815\uc774 \uccb4\uacb0\ub418\uc5c8\ub2e4. \uc774 \ud611\uc815\uc5d0 \ub530\ub77c\uc11c \uc988\ubb34\uce58\ub294 \ub9ac\ud22c\uc544\ub2c8\uc544\uc5d0 \uc591\ub3c4\ub418\uc5c8\ub2e4. \uadf8\ub7ec\ub098 \uadf8\uac83\ub3c4 \uc57c\uae30\uc5d0\uc624\uac00 \uc0b4\uc544 \uc788\ub294 \ub3d9\uc548\uc5d0\ub9cc \ub9ac\ud22c\uc544\ub2c8\uc544\uc5d0 \uc18d\ud558\uace0 \uc57c\uae30\uc5d0\uc624\uac00 \uc0ac\ub9dd\ud55c \ud6c4\uc5d0\ub294 \ub2e4\uc2dc \ub3c5\uc77c \uae30\uc0ac\ub2e8\uc5d0\uac8c \ub118\uaca8 \uc8fc\uae30\ub85c \ud569\uc758\ub418\uc5c8\ub2e4. \ube44\ub85d \ud3f4\ub780\ub4dc\ub294 \uc804\uc7c1\uc5d0\uc11c\ub294 \ud06c\uac8c \uc2b9\ub9ac\ud588\uc9c0\ub9cc \uc815\uce58\uc801\uc73c\ub85c \uadf8\uac83\uc744 \uc81c\ub300\ub85c \uc774\uc6a9\ud558\uc9c0 \ubabb\ud588\ub2e4."} {"question": "\uc624\uacf5\uc774 \ub3cc\uc544\uc654\uc744 \ub54c \ubb34\ucc9c\ub3c4\uc0ac\uc758 \uc870\uc218\ub85c \ub4e4\uc5b4\uc628 \uc790\ub294?", "paragraph": "\ub4dc\ub798\uace4\ubcfc\uc740 \ud769\uc5b4\uc9c0\uace0 \ud53c\ub77c\ud6c4\ub97c \ud63c\ub0b4\uc900 \uc190\uc624\uacf5 \uc77c\ud589\uc740 1\ub144 \ub4a4\uc5d0 \ub2e4\uc2dc \ud55c \ubc88 \ub4dc\ub798\uace4\ubcfc\uc744 \ubaa8\uc73c\ub294 \uc5ec\ud589\uc744 \ub5a0\ub098\uae30\ub85c \ud558\uace0 \ud769\uc5b4\uc9c4\ub2e4. \uc624\uacf5\uc740 \ubb34\ucc9c\ub3c4\uc0ac\uc640\uc758 \uc57d\uc18d\uc744 \uae30\uc5b5\ud558\uace0 \uadf8\uc5d0\uac8c \uc218\ub828\uc744 \ubc1b\uc73c\ub7ec \uac04\ub2e4. \ubb34\ucc9c\ub3c4\uc0ac\uc758 \uc2dc\ud5d8\uc744 \ud1b5\uacfc\ud558\uc5ec \uc5ed\uc2dc \uc218\ub828\uc744 \ubc1b\uc73c\ub7ec \uc628 \ud06c\ub9ac\ub9c1, \ubb34\ucc9c\ub3c4\uc0ac\uc758 \uc870\uc218\ub85c \ub4e4\uc5b4\uc628 \ub7f0\uce58\uc640 \ud568\uaed8 \uc218\ub828\uc774 \uc2dc\uc791\ub41c\ub2e4. \ubb34\ucc9c\ub3c4\uc0ac\ub294 \ud3c9\uc18c\uc5d0\ub294 \ubc1d\ud788\ub294 \ub178\uc778\ub124\uc5d0 \uc9c0\ub098\uc9c0 \uc54a\uc558\uc73c\ub098 \uae30\uc0c1\ucc9c\uc678\ud55c \uc218\ub828\ubc95\uc73c\ub85c \uc624\uacf5\uacfc \ud06c\ub9ac\ub9c1\uc744 \ucc29\uc2e4\ud558\uac8c \ud6c8\ub828\uc2dc\ucf30\ub2e4. \ub9c8\uce68\ub0b4 \uc77c\ub144\uc774 \uc9c0\ub098\uace0. \uc624\uacf5\uacfc \ud06c\ub9ac\ub9c1\uc740 \uc218\ub828\uc758 \uc131\uacfc\ub97c \ud655\uc778\ud558\uae30 \uc704\ud574 \ucc9c\ud558\uc81c\uc77c\ubb34\uc220\ub300\ud68c\uc5d0 \ucd9c\uc804\ud55c\ub2e4. \ud68c\uc7a5\uc5d0\uc11c \ubd80\ub974\ub9c8, \uc57c\ubb34\uce58, \uc624\ub8e1 \ub4f1\uacfc \uc7ac\ud68c\ud558\uace0, \uc624\uacf5\uc740 \uac15\uc801\uacfc\uc758 \ub300\uacb0\uc744 \ud6cc\ub96d\ud558\uac8c \uc774\uaca8\ub0b8\ub2e4. \uacb0\uc2b9 \uc0c1\ub300\ub294 \uc758\ubb38\uc758 \ub178\uc778 \uc7ad\ud0a4 \ucd98. \uadf8\ub294 \uc0ac\uc2e4 \uc81c\uc790\ub4e4\uc744 \uc2dc\ud5d8\ud558\uae30 \uc704\ud574 \ubcc0\uc7a5\ud55c \ubb34\ucc9c\ub3c4\uc0ac\uc600\ub2e4.", "answer": "\ub7f0\uce58", "sentence": "\ubb34\ucc9c\ub3c4\uc0ac\uc758 \uc2dc\ud5d8\uc744 \ud1b5\uacfc\ud558\uc5ec \uc5ed\uc2dc \uc218\ub828\uc744 \ubc1b\uc73c\ub7ec \uc628 \ud06c\ub9ac\ub9c1, \ubb34\ucc9c\ub3c4\uc0ac\uc758 \uc870\uc218\ub85c \ub4e4\uc5b4\uc628 \ub7f0\uce58 \uc640 \ud568\uaed8 \uc218\ub828\uc774 \uc2dc\uc791\ub41c\ub2e4.", "paragraph_sentence": "\ub4dc\ub798\uace4\ubcfc\uc740 \ud769\uc5b4\uc9c0\uace0 \ud53c\ub77c\ud6c4\ub97c \ud63c\ub0b4\uc900 \uc190\uc624\uacf5 \uc77c\ud589\uc740 1\ub144 \ub4a4\uc5d0 \ub2e4\uc2dc \ud55c \ubc88 \ub4dc\ub798\uace4\ubcfc\uc744 \ubaa8\uc73c\ub294 \uc5ec\ud589\uc744 \ub5a0\ub098\uae30\ub85c \ud558\uace0 \ud769\uc5b4\uc9c4\ub2e4. \uc624\uacf5\uc740 \ubb34\ucc9c\ub3c4\uc0ac\uc640\uc758 \uc57d\uc18d\uc744 \uae30\uc5b5\ud558\uace0 \uadf8\uc5d0\uac8c \uc218\ub828\uc744 \ubc1b\uc73c\ub7ec \uac04\ub2e4. \ubb34\ucc9c\ub3c4\uc0ac\uc758 \uc2dc\ud5d8\uc744 \ud1b5\uacfc\ud558\uc5ec \uc5ed\uc2dc \uc218\ub828\uc744 \ubc1b\uc73c\ub7ec \uc628 \ud06c\ub9ac\ub9c1, \ubb34\ucc9c\ub3c4\uc0ac\uc758 \uc870\uc218\ub85c \ub4e4\uc5b4\uc628 \ub7f0\uce58 \uc640 \ud568\uaed8 \uc218\ub828\uc774 \uc2dc\uc791\ub41c\ub2e4. \ubb34\ucc9c\ub3c4\uc0ac\ub294 \ud3c9\uc18c\uc5d0\ub294 \ubc1d\ud788\ub294 \ub178\uc778\ub124\uc5d0 \uc9c0\ub098\uc9c0 \uc54a\uc558\uc73c\ub098 \uae30\uc0c1\ucc9c\uc678\ud55c \uc218\ub828\ubc95\uc73c\ub85c \uc624\uacf5\uacfc \ud06c\ub9ac\ub9c1\uc744 \ucc29\uc2e4\ud558\uac8c \ud6c8\ub828\uc2dc\ucf30\ub2e4. \ub9c8\uce68\ub0b4 \uc77c\ub144\uc774 \uc9c0\ub098\uace0. \uc624\uacf5\uacfc \ud06c\ub9ac\ub9c1\uc740 \uc218\ub828\uc758 \uc131\uacfc\ub97c \ud655\uc778\ud558\uae30 \uc704\ud574 \ucc9c\ud558\uc81c\uc77c\ubb34\uc220\ub300\ud68c\uc5d0 \ucd9c\uc804\ud55c\ub2e4. \ud68c\uc7a5\uc5d0\uc11c \ubd80\ub974\ub9c8, \uc57c\ubb34\uce58, \uc624\ub8e1 \ub4f1\uacfc \uc7ac\ud68c\ud558\uace0, \uc624\uacf5\uc740 \uac15\uc801\uacfc\uc758 \ub300\uacb0\uc744 \ud6cc\ub96d\ud558\uac8c \uc774\uaca8\ub0b8\ub2e4. \uacb0\uc2b9 \uc0c1\ub300\ub294 \uc758\ubb38\uc758 \ub178\uc778 \uc7ad\ud0a4 \ucd98. \uadf8\ub294 \uc0ac\uc2e4 \uc81c\uc790\ub4e4\uc744 \uc2dc\ud5d8\ud558\uae30 \uc704\ud574 \ubcc0\uc7a5\ud55c \ubb34\ucc9c\ub3c4\uc0ac\uc600\ub2e4.", "paragraph_answer": "\ub4dc\ub798\uace4\ubcfc\uc740 \ud769\uc5b4\uc9c0\uace0 \ud53c\ub77c\ud6c4\ub97c \ud63c\ub0b4\uc900 \uc190\uc624\uacf5 \uc77c\ud589\uc740 1\ub144 \ub4a4\uc5d0 \ub2e4\uc2dc \ud55c \ubc88 \ub4dc\ub798\uace4\ubcfc\uc744 \ubaa8\uc73c\ub294 \uc5ec\ud589\uc744 \ub5a0\ub098\uae30\ub85c \ud558\uace0 \ud769\uc5b4\uc9c4\ub2e4. \uc624\uacf5\uc740 \ubb34\ucc9c\ub3c4\uc0ac\uc640\uc758 \uc57d\uc18d\uc744 \uae30\uc5b5\ud558\uace0 \uadf8\uc5d0\uac8c \uc218\ub828\uc744 \ubc1b\uc73c\ub7ec \uac04\ub2e4. \ubb34\ucc9c\ub3c4\uc0ac\uc758 \uc2dc\ud5d8\uc744 \ud1b5\uacfc\ud558\uc5ec \uc5ed\uc2dc \uc218\ub828\uc744 \ubc1b\uc73c\ub7ec \uc628 \ud06c\ub9ac\ub9c1, \ubb34\ucc9c\ub3c4\uc0ac\uc758 \uc870\uc218\ub85c \ub4e4\uc5b4\uc628 \ub7f0\uce58 \uc640 \ud568\uaed8 \uc218\ub828\uc774 \uc2dc\uc791\ub41c\ub2e4. \ubb34\ucc9c\ub3c4\uc0ac\ub294 \ud3c9\uc18c\uc5d0\ub294 \ubc1d\ud788\ub294 \ub178\uc778\ub124\uc5d0 \uc9c0\ub098\uc9c0 \uc54a\uc558\uc73c\ub098 \uae30\uc0c1\ucc9c\uc678\ud55c \uc218\ub828\ubc95\uc73c\ub85c \uc624\uacf5\uacfc \ud06c\ub9ac\ub9c1\uc744 \ucc29\uc2e4\ud558\uac8c \ud6c8\ub828\uc2dc\ucf30\ub2e4. \ub9c8\uce68\ub0b4 \uc77c\ub144\uc774 \uc9c0\ub098\uace0. \uc624\uacf5\uacfc \ud06c\ub9ac\ub9c1\uc740 \uc218\ub828\uc758 \uc131\uacfc\ub97c \ud655\uc778\ud558\uae30 \uc704\ud574 \ucc9c\ud558\uc81c\uc77c\ubb34\uc220\ub300\ud68c\uc5d0 \ucd9c\uc804\ud55c\ub2e4. \ud68c\uc7a5\uc5d0\uc11c \ubd80\ub974\ub9c8, \uc57c\ubb34\uce58, \uc624\ub8e1 \ub4f1\uacfc \uc7ac\ud68c\ud558\uace0, \uc624\uacf5\uc740 \uac15\uc801\uacfc\uc758 \ub300\uacb0\uc744 \ud6cc\ub96d\ud558\uac8c \uc774\uaca8\ub0b8\ub2e4. \uacb0\uc2b9 \uc0c1\ub300\ub294 \uc758\ubb38\uc758 \ub178\uc778 \uc7ad\ud0a4 \ucd98. \uadf8\ub294 \uc0ac\uc2e4 \uc81c\uc790\ub4e4\uc744 \uc2dc\ud5d8\ud558\uae30 \uc704\ud574 \ubcc0\uc7a5\ud55c \ubb34\ucc9c\ub3c4\uc0ac\uc600\ub2e4.", "sentence_answer": "\ubb34\ucc9c\ub3c4\uc0ac\uc758 \uc2dc\ud5d8\uc744 \ud1b5\uacfc\ud558\uc5ec \uc5ed\uc2dc \uc218\ub828\uc744 \ubc1b\uc73c\ub7ec \uc628 \ud06c\ub9ac\ub9c1, \ubb34\ucc9c\ub3c4\uc0ac\uc758 \uc870\uc218\ub85c \ub4e4\uc5b4\uc628 \ub7f0\uce58 \uc640 \ud568\uaed8 \uc218\ub828\uc774 \uc2dc\uc791\ub41c\ub2e4.", "paragraph_id": "6492643-0-2", "paragraph_question": "question: \uc624\uacf5\uc774 \ub3cc\uc544\uc654\uc744 \ub54c \ubb34\ucc9c\ub3c4\uc0ac\uc758 \uc870\uc218\ub85c \ub4e4\uc5b4\uc628 \uc790\ub294?, context: \ub4dc\ub798\uace4\ubcfc\uc740 \ud769\uc5b4\uc9c0\uace0 \ud53c\ub77c\ud6c4\ub97c \ud63c\ub0b4\uc900 \uc190\uc624\uacf5 \uc77c\ud589\uc740 1\ub144 \ub4a4\uc5d0 \ub2e4\uc2dc \ud55c \ubc88 \ub4dc\ub798\uace4\ubcfc\uc744 \ubaa8\uc73c\ub294 \uc5ec\ud589\uc744 \ub5a0\ub098\uae30\ub85c \ud558\uace0 \ud769\uc5b4\uc9c4\ub2e4. \uc624\uacf5\uc740 \ubb34\ucc9c\ub3c4\uc0ac\uc640\uc758 \uc57d\uc18d\uc744 \uae30\uc5b5\ud558\uace0 \uadf8\uc5d0\uac8c \uc218\ub828\uc744 \ubc1b\uc73c\ub7ec \uac04\ub2e4. \ubb34\ucc9c\ub3c4\uc0ac\uc758 \uc2dc\ud5d8\uc744 \ud1b5\uacfc\ud558\uc5ec \uc5ed\uc2dc \uc218\ub828\uc744 \ubc1b\uc73c\ub7ec \uc628 \ud06c\ub9ac\ub9c1, \ubb34\ucc9c\ub3c4\uc0ac\uc758 \uc870\uc218\ub85c \ub4e4\uc5b4\uc628 \ub7f0\uce58\uc640 \ud568\uaed8 \uc218\ub828\uc774 \uc2dc\uc791\ub41c\ub2e4. \ubb34\ucc9c\ub3c4\uc0ac\ub294 \ud3c9\uc18c\uc5d0\ub294 \ubc1d\ud788\ub294 \ub178\uc778\ub124\uc5d0 \uc9c0\ub098\uc9c0 \uc54a\uc558\uc73c\ub098 \uae30\uc0c1\ucc9c\uc678\ud55c \uc218\ub828\ubc95\uc73c\ub85c \uc624\uacf5\uacfc \ud06c\ub9ac\ub9c1\uc744 \ucc29\uc2e4\ud558\uac8c \ud6c8\ub828\uc2dc\ucf30\ub2e4. \ub9c8\uce68\ub0b4 \uc77c\ub144\uc774 \uc9c0\ub098\uace0. \uc624\uacf5\uacfc \ud06c\ub9ac\ub9c1\uc740 \uc218\ub828\uc758 \uc131\uacfc\ub97c \ud655\uc778\ud558\uae30 \uc704\ud574 \ucc9c\ud558\uc81c\uc77c\ubb34\uc220\ub300\ud68c\uc5d0 \ucd9c\uc804\ud55c\ub2e4. \ud68c\uc7a5\uc5d0\uc11c \ubd80\ub974\ub9c8, \uc57c\ubb34\uce58, \uc624\ub8e1 \ub4f1\uacfc \uc7ac\ud68c\ud558\uace0, \uc624\uacf5\uc740 \uac15\uc801\uacfc\uc758 \ub300\uacb0\uc744 \ud6cc\ub96d\ud558\uac8c \uc774\uaca8\ub0b8\ub2e4. \uacb0\uc2b9 \uc0c1\ub300\ub294 \uc758\ubb38\uc758 \ub178\uc778 \uc7ad\ud0a4 \ucd98. \uadf8\ub294 \uc0ac\uc2e4 \uc81c\uc790\ub4e4\uc744 \uc2dc\ud5d8\ud558\uae30 \uc704\ud574 \ubcc0\uc7a5\ud55c \ubb34\ucc9c\ub3c4\uc0ac\uc600\ub2e4."} {"question": "\ubbfc\uc0ac\uc18c\uc1a1 \uc0c1\uace0 \uae30\uac04\uc740 \ud328\uc18c\ud55c \ub2f9\uc0ac\uc790\uac00 2\uc2ec \ud310\uacb0\uc11c\ub97c \uc1a1\ub2ec\ubc1b\uc740 \ub0a0\ubd80\ud130 \uba87\uc8fc \uc774\ub0b4\uc778\uac00?", "paragraph": "\ud328\uc18c\ud55c \ub2f9\uc0ac\uc790\uac00 2\uc2ec \ud310\uacb0\uc11c\ub97c \uc1a1\ub2ec\ubc1b\uc740 \ub0a0\ubd80\ud130 2\uc8fc \uc774\ub0b4\uac00 \ubbfc\uc0ac\uc18c\uc1a1 \uc0c1\uace0 \uae30\uac04\uc778\ub370, \uc2e0\ud55c\uc740 2\uc2ec \ud310\uacb0\uc5d0 \ub300\ud574 \uc0c1\uace0\ud558\uc600\uace0 2009\ub144 5\uc6d4 28\uc77c\uc5d0 \ub300\ubc95\uc6d0 \uc81c3\ubd80(\uc8fc\uc2ec \ubc15\uc2dc\ud658 \ub300\ubc95\uad00)\ub294 \"\uc790\uc2e0\uc5d0\uac8c \ucc45\uc784\uc774 \uc5c6\ub294 \uc77c\ub85c \uc774\ubbf8\uc9c0\uac00 \uc190\uc0c1\ub418\ub294 \uc0c1\ud669\uc774 \ubc1c\uc0dd\ud588\ub354\ub77c\ub3c4, \uadf8 \uc190\uc0c1\uc744 \ucd5c\ub300\ud55c \uc904\uc5ec\uc57c \ud558\ub294 \uac83\uc774 \ubaa8\ub378\uc758 \uc758\ubb34\uc778\ub370, \ucd5c\uc9c4\uc2e4\uc740 \uba4d\ub4e0 \uc5bc\uad74\uacfc \ucda9\ub3cc \ud604\uc7a5 \ub4f1\uc744 \uc5b8\ub860\uc774 \ucd2c\uc601\ud558\ub3c4\ub85d \ud5c8\ub77d\ud568\uc73c\ub85c\uc368 \ud488\uc704 \uc720\uc9c0 \uc57d\uc815\uc744 \uc704\ubc18\ud588\ub2e4\"\ub77c\uace0 \ud310\ub2e8\ud558\uc5ec \ud30c\uae30\ud658\uc1a1 \ud310\uacb0\uc744 \ub0b4\ub838\uace0 \uc774 \uc0ac\uac74\uc744 \uc11c\uc6b8\uace0\ub4f1\ubc95\uc6d0\uc73c\ub85c \ub3cc\ub824\ubcf4\ub0c8\ub2e4. \uc774\ub294 \uc704\ubc95\ud589\uc704 \ud53c\ud574\uc790\uc5d0\uac8c '\uc774\ubbf8\uc9c0' \uc6b4\uc6b4 \ud558\ub294 \ub9cc\ud589\uc5d0 \ub300\ud574 \uad6d\uac00\uae30\uad00\uc774 \uc190\uc744 \ub4e4\uc5b4\uc8fc\ub294 \ud589\ud0dc\uc600\ub2e4. \uc774\uc5d0 \uc704\ubc95\ud589\uc704 \uac00\ud574\uc790 \uc870\uc131\ubbfc, \uae30\uc5c5 \uc2e0\ud55c, \uae30\uc790 \uc11c\ubcd1\uae30, \ubc95\uad00 \ubc15\uc2dc\ud658\uc740 \ud55c \ud328\uc784\uc744 \ubcf4\uc5ec\uc8fc\uc5c8\ub2e4.", "answer": "2\uc8fc", "sentence": "\ud328\uc18c\ud55c \ub2f9\uc0ac\uc790\uac00 2\uc2ec \ud310\uacb0\uc11c\ub97c \uc1a1\ub2ec\ubc1b\uc740 \ub0a0\ubd80\ud130 2\uc8fc \uc774\ub0b4\uac00 \ubbfc\uc0ac\uc18c\uc1a1 \uc0c1\uace0 \uae30\uac04\uc778\ub370, \uc2e0\ud55c\uc740 2\uc2ec \ud310\uacb0\uc5d0 \ub300\ud574 \uc0c1\uace0\ud558\uc600\uace0 2009\ub144 5\uc6d4 28\uc77c\uc5d0 \ub300\ubc95\uc6d0 \uc81c3\ubd80(\uc8fc\uc2ec \ubc15\uc2dc\ud658 \ub300\ubc95\uad00)\ub294 \"\uc790\uc2e0\uc5d0\uac8c \ucc45\uc784\uc774 \uc5c6\ub294 \uc77c\ub85c \uc774\ubbf8\uc9c0\uac00 \uc190\uc0c1\ub418\ub294 \uc0c1\ud669\uc774 \ubc1c\uc0dd\ud588\ub354\ub77c\ub3c4, \uadf8 \uc190\uc0c1\uc744 \ucd5c\ub300\ud55c \uc904\uc5ec\uc57c \ud558\ub294 \uac83\uc774 \ubaa8\ub378\uc758 \uc758\ubb34\uc778\ub370, \ucd5c\uc9c4\uc2e4\uc740 \uba4d\ub4e0 \uc5bc\uad74\uacfc \ucda9\ub3cc \ud604\uc7a5 \ub4f1\uc744 \uc5b8\ub860\uc774 \ucd2c\uc601\ud558\ub3c4\ub85d \ud5c8\ub77d\ud568\uc73c\ub85c\uc368 \ud488\uc704 \uc720\uc9c0 \uc57d\uc815\uc744 \uc704\ubc18\ud588\ub2e4\"\ub77c\uace0 \ud310\ub2e8\ud558\uc5ec \ud30c\uae30\ud658\uc1a1 \ud310\uacb0\uc744 \ub0b4\ub838\uace0 \uc774 \uc0ac\uac74\uc744 \uc11c\uc6b8\uace0\ub4f1\ubc95\uc6d0\uc73c\ub85c \ub3cc\ub824\ubcf4\ub0c8\ub2e4.", "paragraph_sentence": " \ud328\uc18c\ud55c \ub2f9\uc0ac\uc790\uac00 2\uc2ec \ud310\uacb0\uc11c\ub97c \uc1a1\ub2ec\ubc1b\uc740 \ub0a0\ubd80\ud130 2\uc8fc \uc774\ub0b4\uac00 \ubbfc\uc0ac\uc18c\uc1a1 \uc0c1\uace0 \uae30\uac04\uc778\ub370, \uc2e0\ud55c\uc740 2\uc2ec \ud310\uacb0\uc5d0 \ub300\ud574 \uc0c1\uace0\ud558\uc600\uace0 2009\ub144 5\uc6d4 28\uc77c\uc5d0 \ub300\ubc95\uc6d0 \uc81c3\ubd80(\uc8fc\uc2ec \ubc15\uc2dc\ud658 \ub300\ubc95\uad00)\ub294 \"\uc790\uc2e0\uc5d0\uac8c \ucc45\uc784\uc774 \uc5c6\ub294 \uc77c\ub85c \uc774\ubbf8\uc9c0\uac00 \uc190\uc0c1\ub418\ub294 \uc0c1\ud669\uc774 \ubc1c\uc0dd\ud588\ub354\ub77c\ub3c4, \uadf8 \uc190\uc0c1\uc744 \ucd5c\ub300\ud55c \uc904\uc5ec\uc57c \ud558\ub294 \uac83\uc774 \ubaa8\ub378\uc758 \uc758\ubb34\uc778\ub370, \ucd5c\uc9c4\uc2e4\uc740 \uba4d\ub4e0 \uc5bc\uad74\uacfc \ucda9\ub3cc \ud604\uc7a5 \ub4f1\uc744 \uc5b8\ub860\uc774 \ucd2c\uc601\ud558\ub3c4\ub85d \ud5c8\ub77d\ud568\uc73c\ub85c\uc368 \ud488\uc704 \uc720\uc9c0 \uc57d\uc815\uc744 \uc704\ubc18\ud588\ub2e4\"\ub77c\uace0 \ud310\ub2e8\ud558\uc5ec \ud30c\uae30\ud658\uc1a1 \ud310\uacb0\uc744 \ub0b4\ub838\uace0 \uc774 \uc0ac\uac74\uc744 \uc11c\uc6b8\uace0\ub4f1\ubc95\uc6d0\uc73c\ub85c \ub3cc\ub824\ubcf4\ub0c8\ub2e4. \uc774\ub294 \uc704\ubc95\ud589\uc704 \ud53c\ud574\uc790\uc5d0\uac8c '\uc774\ubbf8\uc9c0' \uc6b4\uc6b4 \ud558\ub294 \ub9cc\ud589\uc5d0 \ub300\ud574 \uad6d\uac00\uae30\uad00\uc774 \uc190\uc744 \ub4e4\uc5b4\uc8fc\ub294 \ud589\ud0dc\uc600\ub2e4. \uc774\uc5d0 \uc704\ubc95\ud589\uc704 \uac00\ud574\uc790 \uc870\uc131\ubbfc, \uae30\uc5c5 \uc2e0\ud55c, \uae30\uc790 \uc11c\ubcd1\uae30, \ubc95\uad00 \ubc15\uc2dc\ud658\uc740 \ud55c \ud328\uc784\uc744 \ubcf4\uc5ec\uc8fc\uc5c8\ub2e4.", "paragraph_answer": "\ud328\uc18c\ud55c \ub2f9\uc0ac\uc790\uac00 2\uc2ec \ud310\uacb0\uc11c\ub97c \uc1a1\ub2ec\ubc1b\uc740 \ub0a0\ubd80\ud130 2\uc8fc \uc774\ub0b4\uac00 \ubbfc\uc0ac\uc18c\uc1a1 \uc0c1\uace0 \uae30\uac04\uc778\ub370, \uc2e0\ud55c\uc740 2\uc2ec \ud310\uacb0\uc5d0 \ub300\ud574 \uc0c1\uace0\ud558\uc600\uace0 2009\ub144 5\uc6d4 28\uc77c\uc5d0 \ub300\ubc95\uc6d0 \uc81c3\ubd80(\uc8fc\uc2ec \ubc15\uc2dc\ud658 \ub300\ubc95\uad00)\ub294 \"\uc790\uc2e0\uc5d0\uac8c \ucc45\uc784\uc774 \uc5c6\ub294 \uc77c\ub85c \uc774\ubbf8\uc9c0\uac00 \uc190\uc0c1\ub418\ub294 \uc0c1\ud669\uc774 \ubc1c\uc0dd\ud588\ub354\ub77c\ub3c4, \uadf8 \uc190\uc0c1\uc744 \ucd5c\ub300\ud55c \uc904\uc5ec\uc57c \ud558\ub294 \uac83\uc774 \ubaa8\ub378\uc758 \uc758\ubb34\uc778\ub370, \ucd5c\uc9c4\uc2e4\uc740 \uba4d\ub4e0 \uc5bc\uad74\uacfc \ucda9\ub3cc \ud604\uc7a5 \ub4f1\uc744 \uc5b8\ub860\uc774 \ucd2c\uc601\ud558\ub3c4\ub85d \ud5c8\ub77d\ud568\uc73c\ub85c\uc368 \ud488\uc704 \uc720\uc9c0 \uc57d\uc815\uc744 \uc704\ubc18\ud588\ub2e4\"\ub77c\uace0 \ud310\ub2e8\ud558\uc5ec \ud30c\uae30\ud658\uc1a1 \ud310\uacb0\uc744 \ub0b4\ub838\uace0 \uc774 \uc0ac\uac74\uc744 \uc11c\uc6b8\uace0\ub4f1\ubc95\uc6d0\uc73c\ub85c \ub3cc\ub824\ubcf4\ub0c8\ub2e4. \uc774\ub294 \uc704\ubc95\ud589\uc704 \ud53c\ud574\uc790\uc5d0\uac8c '\uc774\ubbf8\uc9c0' \uc6b4\uc6b4 \ud558\ub294 \ub9cc\ud589\uc5d0 \ub300\ud574 \uad6d\uac00\uae30\uad00\uc774 \uc190\uc744 \ub4e4\uc5b4\uc8fc\ub294 \ud589\ud0dc\uc600\ub2e4. \uc774\uc5d0 \uc704\ubc95\ud589\uc704 \uac00\ud574\uc790 \uc870\uc131\ubbfc, \uae30\uc5c5 \uc2e0\ud55c, \uae30\uc790 \uc11c\ubcd1\uae30, \ubc95\uad00 \ubc15\uc2dc\ud658\uc740 \ud55c \ud328\uc784\uc744 \ubcf4\uc5ec\uc8fc\uc5c8\ub2e4.", "sentence_answer": "\ud328\uc18c\ud55c \ub2f9\uc0ac\uc790\uac00 2\uc2ec \ud310\uacb0\uc11c\ub97c \uc1a1\ub2ec\ubc1b\uc740 \ub0a0\ubd80\ud130 2\uc8fc \uc774\ub0b4\uac00 \ubbfc\uc0ac\uc18c\uc1a1 \uc0c1\uace0 \uae30\uac04\uc778\ub370, \uc2e0\ud55c\uc740 2\uc2ec \ud310\uacb0\uc5d0 \ub300\ud574 \uc0c1\uace0\ud558\uc600\uace0 2009\ub144 5\uc6d4 28\uc77c\uc5d0 \ub300\ubc95\uc6d0 \uc81c3\ubd80(\uc8fc\uc2ec \ubc15\uc2dc\ud658 \ub300\ubc95\uad00)\ub294 \"\uc790\uc2e0\uc5d0\uac8c \ucc45\uc784\uc774 \uc5c6\ub294 \uc77c\ub85c \uc774\ubbf8\uc9c0\uac00 \uc190\uc0c1\ub418\ub294 \uc0c1\ud669\uc774 \ubc1c\uc0dd\ud588\ub354\ub77c\ub3c4, \uadf8 \uc190\uc0c1\uc744 \ucd5c\ub300\ud55c \uc904\uc5ec\uc57c \ud558\ub294 \uac83\uc774 \ubaa8\ub378\uc758 \uc758\ubb34\uc778\ub370, \ucd5c\uc9c4\uc2e4\uc740 \uba4d\ub4e0 \uc5bc\uad74\uacfc \ucda9\ub3cc \ud604\uc7a5 \ub4f1\uc744 \uc5b8\ub860\uc774 \ucd2c\uc601\ud558\ub3c4\ub85d \ud5c8\ub77d\ud568\uc73c\ub85c\uc368 \ud488\uc704 \uc720\uc9c0 \uc57d\uc815\uc744 \uc704\ubc18\ud588\ub2e4\"\ub77c\uace0 \ud310\ub2e8\ud558\uc5ec \ud30c\uae30\ud658\uc1a1 \ud310\uacb0\uc744 \ub0b4\ub838\uace0 \uc774 \uc0ac\uac74\uc744 \uc11c\uc6b8\uace0\ub4f1\ubc95\uc6d0\uc73c\ub85c \ub3cc\ub824\ubcf4\ub0c8\ub2e4.", "paragraph_id": "6560245-19-0", "paragraph_question": "question: \ubbfc\uc0ac\uc18c\uc1a1 \uc0c1\uace0 \uae30\uac04\uc740 \ud328\uc18c\ud55c \ub2f9\uc0ac\uc790\uac00 2\uc2ec \ud310\uacb0\uc11c\ub97c \uc1a1\ub2ec\ubc1b\uc740 \ub0a0\ubd80\ud130 \uba87\uc8fc \uc774\ub0b4\uc778\uac00?, context: \ud328\uc18c\ud55c \ub2f9\uc0ac\uc790\uac00 2\uc2ec \ud310\uacb0\uc11c\ub97c \uc1a1\ub2ec\ubc1b\uc740 \ub0a0\ubd80\ud130 2\uc8fc \uc774\ub0b4\uac00 \ubbfc\uc0ac\uc18c\uc1a1 \uc0c1\uace0 \uae30\uac04\uc778\ub370, \uc2e0\ud55c\uc740 2\uc2ec \ud310\uacb0\uc5d0 \ub300\ud574 \uc0c1\uace0\ud558\uc600\uace0 2009\ub144 5\uc6d4 28\uc77c\uc5d0 \ub300\ubc95\uc6d0 \uc81c3\ubd80(\uc8fc\uc2ec \ubc15\uc2dc\ud658 \ub300\ubc95\uad00)\ub294 \"\uc790\uc2e0\uc5d0\uac8c \ucc45\uc784\uc774 \uc5c6\ub294 \uc77c\ub85c \uc774\ubbf8\uc9c0\uac00 \uc190\uc0c1\ub418\ub294 \uc0c1\ud669\uc774 \ubc1c\uc0dd\ud588\ub354\ub77c\ub3c4, \uadf8 \uc190\uc0c1\uc744 \ucd5c\ub300\ud55c \uc904\uc5ec\uc57c \ud558\ub294 \uac83\uc774 \ubaa8\ub378\uc758 \uc758\ubb34\uc778\ub370, \ucd5c\uc9c4\uc2e4\uc740 \uba4d\ub4e0 \uc5bc\uad74\uacfc \ucda9\ub3cc \ud604\uc7a5 \ub4f1\uc744 \uc5b8\ub860\uc774 \ucd2c\uc601\ud558\ub3c4\ub85d \ud5c8\ub77d\ud568\uc73c\ub85c\uc368 \ud488\uc704 \uc720\uc9c0 \uc57d\uc815\uc744 \uc704\ubc18\ud588\ub2e4\"\ub77c\uace0 \ud310\ub2e8\ud558\uc5ec \ud30c\uae30\ud658\uc1a1 \ud310\uacb0\uc744 \ub0b4\ub838\uace0 \uc774 \uc0ac\uac74\uc744 \uc11c\uc6b8\uace0\ub4f1\ubc95\uc6d0\uc73c\ub85c \ub3cc\ub824\ubcf4\ub0c8\ub2e4. \uc774\ub294 \uc704\ubc95\ud589\uc704 \ud53c\ud574\uc790\uc5d0\uac8c '\uc774\ubbf8\uc9c0' \uc6b4\uc6b4 \ud558\ub294 \ub9cc\ud589\uc5d0 \ub300\ud574 \uad6d\uac00\uae30\uad00\uc774 \uc190\uc744 \ub4e4\uc5b4\uc8fc\ub294 \ud589\ud0dc\uc600\ub2e4. \uc774\uc5d0 \uc704\ubc95\ud589\uc704 \uac00\ud574\uc790 \uc870\uc131\ubbfc, \uae30\uc5c5 \uc2e0\ud55c, \uae30\uc790 \uc11c\ubcd1\uae30, \ubc95\uad00 \ubc15\uc2dc\ud658\uc740 \ud55c \ud328\uc784\uc744 \ubcf4\uc5ec\uc8fc\uc5c8\ub2e4."} {"question": "Paperback Writer\ub97c \uc0dd\uc0b0\ud55c \ud68c\uc0ac\ub294?", "paragraph": "\ubbf8\uad6d\uc5d0\uc11c\ub294 \uce90\ud53c\ud2c0 \ub808\ucf54\ub4dc\uac00 \u300aYesterday and Today\u300b\uc5d0\uc11c \uc0ac\uc6a9\ub41c \uc0ac\uc9c4\uacfc \uac19\uc740 \uac83\uc744 \uc0ac\uc6a9\ud55c \u3008Paperback Writer\u3009\ub97c \uc57d 750,000\uc7a5 \uc0dd\uc0b0\ud588\ub2e4. \uc774 \uc74c\ubc18\ub4e4\uc740 \uce90\ud53c\ud2c0\uc0ac \uacf5\uc7a5\uc774 \uc790\ub9ac\uc7a1\uc740 \ubbf8\uad6d\uc758 \ub124 \uac1c \ub3c4\uc2dc\uc778 \ub85c\uc2a4\uc564\uc824\ub808\uc2a4, \uc2a4\ud06c\ub79c\ud134, \uc708\uccb4\uc2a4\ud130, \uc7ad\uc2a8\ube4c\uc5d0\uc11c \uc0dd\uc0b0\ub418\uc5c8\ub2e4. \ucee4\ubc84\uac00 \uc5b4\ub514\uc11c \ub098\uc654\ub294\uc9c0 \uc9c0\uc815\ud558\ub294 \ubc88\ud638\ub294 RIAA \uc2ec\ube14 \uadfc\ucc98\uc5d0 \uc788\ub2e4. \uc608\ub97c \ub4e4\uc5b4 \ub85c\uc2a4\uc564\uc824\ub808\uc2a4 \uacf5\uc7a5\uc758 \uc2a4\ud14c\ub808\uc624 \uc74c\ubc18\uc740 \"5\"\ub85c \uc9c0\uc815, \ub85c\uc2a4\uc564\uc824\ub808\uc2a4 \ubaa8\ub178 \ubc84\uc804\uc740 \"6\"\uc73c\ub85c \ud45c\uae30\ub418\uc5b4 \uc788\ub2e4. \ubaa8\ub178\uc640 \ube44\uad50\ud588\uc744 \ub54c \uc2a4\ud14c\ub808\uc624\ub294 10:1\uc758 \ube44\uc728\ub85c \uadf8 \uc218\uac00 \uc801\uc5b4 \uc218\uc9d1\uac00 \uc0ac\uc774\uc5d0\uc11c \ub354 \ud76c\uadc0, \uac00\uce58\uc788\uac8c \ud574\uc900\ub2e4. \uc801\uc740 \uc218\uc758 \uc74c\ubc18\uc774 DJ\uc640 \ud3c9\ub860\uac00\uc5d0\uac8c \ubcf4\ub0b4\uc84c\uace0 \uc989\uac01\uc801\uc73c\ub85c \ube44\ub09c\uc774 \uc1c4\ub3c4\ud558\uc790 \uce90\ud53c\ud2c0\uc758 \ubaa8\ud68c\uc0ac\uc778 EMI\uc758 \ud68c\uc7a5 \uc870\uc149 \ub77d\uc6b0\ub4dc \uacbd\uc740 \uc989\uc2dc \ud68c\uc218\ud560 \uac83\uc744 \uba85\ud588\ub2e4.", "answer": "\uce90\ud53c\ud2c0 \ub808\ucf54\ub4dc", "sentence": "\ubbf8\uad6d\uc5d0\uc11c\ub294 \uce90\ud53c\ud2c0 \ub808\ucf54\ub4dc \uac00 \u300aYesterday and Today\u300b\uc5d0\uc11c \uc0ac\uc6a9\ub41c \uc0ac\uc9c4\uacfc \uac19\uc740 \uac83\uc744 \uc0ac\uc6a9\ud55c \u3008Paperback Writer\u3009\ub97c \uc57d 750,000\uc7a5 \uc0dd\uc0b0\ud588\ub2e4.", "paragraph_sentence": " \ubbf8\uad6d\uc5d0\uc11c\ub294 \uce90\ud53c\ud2c0 \ub808\ucf54\ub4dc \uac00 \u300aYesterday and Today\u300b\uc5d0\uc11c \uc0ac\uc6a9\ub41c \uc0ac\uc9c4\uacfc \uac19\uc740 \uac83\uc744 \uc0ac\uc6a9\ud55c \u3008Paperback Writer\u3009\ub97c \uc57d 750,000\uc7a5 \uc0dd\uc0b0\ud588\ub2e4. \uc774 \uc74c\ubc18\ub4e4\uc740 \uce90\ud53c\ud2c0\uc0ac \uacf5\uc7a5\uc774 \uc790\ub9ac\uc7a1\uc740 \ubbf8\uad6d\uc758 \ub124 \uac1c \ub3c4\uc2dc\uc778 \ub85c\uc2a4\uc564\uc824\ub808\uc2a4, \uc2a4\ud06c\ub79c\ud134, \uc708\uccb4\uc2a4\ud130, \uc7ad\uc2a8\ube4c\uc5d0\uc11c \uc0dd\uc0b0\ub418\uc5c8\ub2e4. \ucee4\ubc84\uac00 \uc5b4\ub514\uc11c \ub098\uc654\ub294\uc9c0 \uc9c0\uc815\ud558\ub294 \ubc88\ud638\ub294 RIAA \uc2ec\ube14 \uadfc\ucc98\uc5d0 \uc788\ub2e4. \uc608\ub97c \ub4e4\uc5b4 \ub85c\uc2a4\uc564\uc824\ub808\uc2a4 \uacf5\uc7a5\uc758 \uc2a4\ud14c\ub808\uc624 \uc74c\ubc18\uc740 \"5\"\ub85c \uc9c0\uc815, \ub85c\uc2a4\uc564\uc824\ub808\uc2a4 \ubaa8\ub178 \ubc84\uc804\uc740 \"6\"\uc73c\ub85c \ud45c\uae30\ub418\uc5b4 \uc788\ub2e4. \ubaa8\ub178\uc640 \ube44\uad50\ud588\uc744 \ub54c \uc2a4\ud14c\ub808\uc624\ub294 10:1\uc758 \ube44\uc728\ub85c \uadf8 \uc218\uac00 \uc801\uc5b4 \uc218\uc9d1\uac00 \uc0ac\uc774\uc5d0\uc11c \ub354 \ud76c\uadc0, \uac00\uce58\uc788\uac8c \ud574\uc900\ub2e4. \uc801\uc740 \uc218\uc758 \uc74c\ubc18\uc774 DJ\uc640 \ud3c9\ub860\uac00\uc5d0\uac8c \ubcf4\ub0b4\uc84c\uace0 \uc989\uac01\uc801\uc73c\ub85c \ube44\ub09c\uc774 \uc1c4\ub3c4\ud558\uc790 \uce90\ud53c\ud2c0\uc758 \ubaa8\ud68c\uc0ac\uc778 EMI\uc758 \ud68c\uc7a5 \uc870\uc149 \ub77d\uc6b0\ub4dc \uacbd\uc740 \uc989\uc2dc \ud68c\uc218\ud560 \uac83\uc744 \uba85\ud588\ub2e4.", "paragraph_answer": "\ubbf8\uad6d\uc5d0\uc11c\ub294 \uce90\ud53c\ud2c0 \ub808\ucf54\ub4dc \uac00 \u300aYesterday and Today\u300b\uc5d0\uc11c \uc0ac\uc6a9\ub41c \uc0ac\uc9c4\uacfc \uac19\uc740 \uac83\uc744 \uc0ac\uc6a9\ud55c \u3008Paperback Writer\u3009\ub97c \uc57d 750,000\uc7a5 \uc0dd\uc0b0\ud588\ub2e4. \uc774 \uc74c\ubc18\ub4e4\uc740 \uce90\ud53c\ud2c0\uc0ac \uacf5\uc7a5\uc774 \uc790\ub9ac\uc7a1\uc740 \ubbf8\uad6d\uc758 \ub124 \uac1c \ub3c4\uc2dc\uc778 \ub85c\uc2a4\uc564\uc824\ub808\uc2a4, \uc2a4\ud06c\ub79c\ud134, \uc708\uccb4\uc2a4\ud130, \uc7ad\uc2a8\ube4c\uc5d0\uc11c \uc0dd\uc0b0\ub418\uc5c8\ub2e4. \ucee4\ubc84\uac00 \uc5b4\ub514\uc11c \ub098\uc654\ub294\uc9c0 \uc9c0\uc815\ud558\ub294 \ubc88\ud638\ub294 RIAA \uc2ec\ube14 \uadfc\ucc98\uc5d0 \uc788\ub2e4. \uc608\ub97c \ub4e4\uc5b4 \ub85c\uc2a4\uc564\uc824\ub808\uc2a4 \uacf5\uc7a5\uc758 \uc2a4\ud14c\ub808\uc624 \uc74c\ubc18\uc740 \"5\"\ub85c \uc9c0\uc815, \ub85c\uc2a4\uc564\uc824\ub808\uc2a4 \ubaa8\ub178 \ubc84\uc804\uc740 \"6\"\uc73c\ub85c \ud45c\uae30\ub418\uc5b4 \uc788\ub2e4. \ubaa8\ub178\uc640 \ube44\uad50\ud588\uc744 \ub54c \uc2a4\ud14c\ub808\uc624\ub294 10:1\uc758 \ube44\uc728\ub85c \uadf8 \uc218\uac00 \uc801\uc5b4 \uc218\uc9d1\uac00 \uc0ac\uc774\uc5d0\uc11c \ub354 \ud76c\uadc0, \uac00\uce58\uc788\uac8c \ud574\uc900\ub2e4. \uc801\uc740 \uc218\uc758 \uc74c\ubc18\uc774 DJ\uc640 \ud3c9\ub860\uac00\uc5d0\uac8c \ubcf4\ub0b4\uc84c\uace0 \uc989\uac01\uc801\uc73c\ub85c \ube44\ub09c\uc774 \uc1c4\ub3c4\ud558\uc790 \uce90\ud53c\ud2c0\uc758 \ubaa8\ud68c\uc0ac\uc778 EMI\uc758 \ud68c\uc7a5 \uc870\uc149 \ub77d\uc6b0\ub4dc \uacbd\uc740 \uc989\uc2dc \ud68c\uc218\ud560 \uac83\uc744 \uba85\ud588\ub2e4.", "sentence_answer": "\ubbf8\uad6d\uc5d0\uc11c\ub294 \uce90\ud53c\ud2c0 \ub808\ucf54\ub4dc \uac00 \u300aYesterday and Today\u300b\uc5d0\uc11c \uc0ac\uc6a9\ub41c \uc0ac\uc9c4\uacfc \uac19\uc740 \uac83\uc744 \uc0ac\uc6a9\ud55c \u3008Paperback Writer\u3009\ub97c \uc57d 750,000\uc7a5 \uc0dd\uc0b0\ud588\ub2e4.", "paragraph_id": "5782942-1-1", "paragraph_question": "question: Paperback Writer\ub97c \uc0dd\uc0b0\ud55c \ud68c\uc0ac\ub294?, context: \ubbf8\uad6d\uc5d0\uc11c\ub294 \uce90\ud53c\ud2c0 \ub808\ucf54\ub4dc\uac00 \u300aYesterday and Today\u300b\uc5d0\uc11c \uc0ac\uc6a9\ub41c \uc0ac\uc9c4\uacfc \uac19\uc740 \uac83\uc744 \uc0ac\uc6a9\ud55c \u3008Paperback Writer\u3009\ub97c \uc57d 750,000\uc7a5 \uc0dd\uc0b0\ud588\ub2e4. \uc774 \uc74c\ubc18\ub4e4\uc740 \uce90\ud53c\ud2c0\uc0ac \uacf5\uc7a5\uc774 \uc790\ub9ac\uc7a1\uc740 \ubbf8\uad6d\uc758 \ub124 \uac1c \ub3c4\uc2dc\uc778 \ub85c\uc2a4\uc564\uc824\ub808\uc2a4, \uc2a4\ud06c\ub79c\ud134, \uc708\uccb4\uc2a4\ud130, \uc7ad\uc2a8\ube4c\uc5d0\uc11c \uc0dd\uc0b0\ub418\uc5c8\ub2e4. \ucee4\ubc84\uac00 \uc5b4\ub514\uc11c \ub098\uc654\ub294\uc9c0 \uc9c0\uc815\ud558\ub294 \ubc88\ud638\ub294 RIAA \uc2ec\ube14 \uadfc\ucc98\uc5d0 \uc788\ub2e4. \uc608\ub97c \ub4e4\uc5b4 \ub85c\uc2a4\uc564\uc824\ub808\uc2a4 \uacf5\uc7a5\uc758 \uc2a4\ud14c\ub808\uc624 \uc74c\ubc18\uc740 \"5\"\ub85c \uc9c0\uc815, \ub85c\uc2a4\uc564\uc824\ub808\uc2a4 \ubaa8\ub178 \ubc84\uc804\uc740 \"6\"\uc73c\ub85c \ud45c\uae30\ub418\uc5b4 \uc788\ub2e4. \ubaa8\ub178\uc640 \ube44\uad50\ud588\uc744 \ub54c \uc2a4\ud14c\ub808\uc624\ub294 10:1\uc758 \ube44\uc728\ub85c \uadf8 \uc218\uac00 \uc801\uc5b4 \uc218\uc9d1\uac00 \uc0ac\uc774\uc5d0\uc11c \ub354 \ud76c\uadc0, \uac00\uce58\uc788\uac8c \ud574\uc900\ub2e4. \uc801\uc740 \uc218\uc758 \uc74c\ubc18\uc774 DJ\uc640 \ud3c9\ub860\uac00\uc5d0\uac8c \ubcf4\ub0b4\uc84c\uace0 \uc989\uac01\uc801\uc73c\ub85c \ube44\ub09c\uc774 \uc1c4\ub3c4\ud558\uc790 \uce90\ud53c\ud2c0\uc758 \ubaa8\ud68c\uc0ac\uc778 EMI\uc758 \ud68c\uc7a5 \uc870\uc149 \ub77d\uc6b0\ub4dc \uacbd\uc740 \uc989\uc2dc \ud68c\uc218\ud560 \uac83\uc744 \uba85\ud588\ub2e4."} {"question": "\uc54c\ub85c\uc0ac\uc6b0\ub8e8\uc2a4\uc758 \ucc99\ucd94\uace8 \uac2f\uc218\uac00 50\uac1c\ub77c\uace0 \uc608\uc0c1\ud55c \uc0ac\ub78c\uc740?", "paragraph": "\uc54c\ub85c\uc0ac\uc6b0\ub8e8\uc2a4\ub294 \ubaa9\uc5d0 9\uac1c, \ub4f1\uc5d0 14\uac1c, \ucc9c\uace8\uc5d0 5\uac1c\uc758 \ucc99\ucd94\uace8\uc744 \uac00\uc9c0\uace0 \uc788\uc5c8\ub2e4. \uaf2c\ub9ac\uc758 \ucc99\ucd94\uace8 \uc218\ub294 \ubd88\uba85\uc774\uace0 \uac1c\uac1c\uc758 \ud06c\uae30\ub3c4 \ub2e4\uc591\ud588\ub2e4. \uc81c\uc784\uc2a4 \ub9e4\ub4dc\uc2a8\uc740 50\uac1c \uac00\ub7c9\uc73c\ub85c \uc608\uc0c1\ud588\uace0, \uac8c\uc624\ub974\uae30 \ud3f4\uc740 \uc774\uac83\ub3c4 \ub9ce\ub2e4\uace0 \uc5ec\uaca8\uc11c 45\uac1c\ub098 \uadf8 \uc774\ud558\ub85c \uc608\uce21\ud588\ub2e4. \ubaa9\uacfc \ub4f1\uc758 \ucc99\ucd94\uace8 \uc804\ubc29\uc5d0\ub294 \ube48 \uacf5\uac04\uc774 \uc788\uc5c8\uace0, \ud604\uc0dd \ub3d9\ubb3c\ub4e4(\uc0c8\ub77c\ub4e0\uc9c0)\uc5d0\uc11c\ub3c4 \ubc1c\uacac\ub418\ub294 \uc774\ub7f0 \uacf5\uac04\ub4e4\uc740 \ud638\ud761\uc5d0 \uc4f0\uc778 \uacf5\uae30\uc8fc\uba38\ub2c8\ub85c \ubcf4\uc778\ub2e4. \uac00\uc2b4\uc740 \ub113\uace0 \uc6d0\ud1b5\ud615 \ud749\uacfd\uc774\uc5c8\uace0, \ubcf5\ubd80 \ub291\uace8(\ubc30\uc758 \uac08\ube44)\ub97c \uac00\uc9c0\uace0 \uc788\uc5c8\uc9c0\ub9cc \uc774\uac83\ub4e4\uc740 \ud754\ud558\uac8c \ubc1c\uacac\ub418\ub294 \uac83\uc774 \uc544\ub2c8\uba70, \uc57d\uac04\uc758 \uace8\ud654 \ud604\uc0c1\uc774 \uc788\uc5c8\uc744 \uc218\ub3c4 \uc788\ub2e4. \ud55c \ubc1c\ud45c\uc5d0 \uc758\ud558\uba74, \ubcf5\ub291\uace8\uc740 \uac1c\uccb4\uac00 \uc0b4\uc544 \uc788\uc5c8\uc744 \ub54c \uc0c1\ud574\ub97c \uc785\uc5c8\ub2e4\ub294 \uc99d\uac70\uac00 \ub420 \uc218 \uc788\ub2e4. \ucc3d\uc0ac\uace8 (\uc77c\uba85 \uc704\uc2dc\ubcf8) \ub610\ud55c \uc874\uc7ac\ud588\uc9c0\ub9cc, 1996\ub144\uc774 \ub418\uc5b4\uc11c\uc57c \uc778\uc815\ubc1b\uc558\ub2e4. \uba87\uba87 \uacbd\uc6b0\uc5d0 \ucc3d\uc0ac\uace8\uc774 \ubcf5\ub291\uace8\uacfc \ud63c\ub3d9\ub418\uae30\ub3c4 \ud588\ub2e4. \uc5c9\ub369\uc774\uc758 \uc911\uc2ec \ubf08\uc778 \uc7a5\uace8\uc740 \uc721\uc911\ud588\uace0, \uce58\uace8\uc5d0\ub294 \ud280\uc5b4\ub098\uc628 \ub05d\ub2e8\uc774 \uc788\uc5c8\ub294\ub370 \uc774\ub294 \uadfc\uc721\uc744 \uc5f0\uacb0\uc2dc\ucf1c \uc8fc\ub294 \ub3d9\uc2dc\uc5d0 \ub2e4\ub9ac\uac00 \ub545\uc744 \ub51b\uace0 \uc11c \uc788\uc744 \uc218 \uc788\uac8c \ud574 \uc8fc\ub294 \ubc1b\uce68\ub300 \uc5ed\ud560\uc744 \ud588\ub2e4. \ub9e4\ub4dc\uc2a8\uc740 \ud074\ub9ac\ube14\ub79c\ub4dc \ub85c\uc774\ub4dc \ud0d0\uc0ac\uc5d0\uc11c \ubc1c\uacac\ub41c \uac1c\uccb4\ub4e4 \uc911 \uc57d \uc808\ubc18 \uc815\ub3c4\uac00 \ud06c\uae30\uc5d0 \uad00\uacc4\uc5c6\uc774 \ubc1c \ub05d\ubd80\ubd84\uc758 \uc5f0\ubaa8\uac00 \uc11c\ub85c \uacb0\ud569\ub418\uc5b4 \uc788\uc9c0 \uc54a\uc558\ub2e4\uace0 \uae30\uc220\ud588\ub2e4. \ub9e4\ub4dc\uc2a8\uc740 \uc774\uac83\uc774 \uc131\uc801\uc778 \ud2b9\uc9d5\uc77c \uac00\ub2a5\uc131\uc774 \uc788\uc73c\uba70, \uc774\ub294 \uc554\ucef7\uc774 \uc54c\uc744 \ud488\uae30 \uc27d\ub3c4\ub85d \ubf08\uac00 \ubd99\uc5b4 \uc788\uc9c0 \uc54a\uc740 \uac83\uc774\ub77c\uace0 \uc8fc\uc7a5\ud588\ub2e4. \uadf8\ub7ec\ub098 \uc774 \uc0dd\uac01\uc740 \uc774\ud6c4 \uc9c0\uc9c0\ub97c \ubcc4\ub85c \ubc1b\uc9c0 \ubabb\ud588\ub2e4.", "answer": "\uc81c\uc784\uc2a4 \ub9e4\ub4dc\uc2a8", "sentence": "\uc81c\uc784\uc2a4 \ub9e4\ub4dc\uc2a8 \uc740 50\uac1c \uac00\ub7c9\uc73c\ub85c \uc608\uc0c1\ud588\uace0, \uac8c\uc624\ub974\uae30 \ud3f4\uc740 \uc774\uac83\ub3c4 \ub9ce\ub2e4\uace0 \uc5ec\uaca8\uc11c 45\uac1c\ub098 \uadf8 \uc774\ud558\ub85c \uc608\uce21\ud588\ub2e4.", "paragraph_sentence": "\uc54c\ub85c\uc0ac\uc6b0\ub8e8\uc2a4\ub294 \ubaa9\uc5d0 9\uac1c, \ub4f1\uc5d0 14\uac1c, \ucc9c\uace8\uc5d0 5\uac1c\uc758 \ucc99\ucd94\uace8\uc744 \uac00\uc9c0\uace0 \uc788\uc5c8\ub2e4. \uaf2c\ub9ac\uc758 \ucc99\ucd94\uace8 \uc218\ub294 \ubd88\uba85\uc774\uace0 \uac1c\uac1c\uc758 \ud06c\uae30\ub3c4 \ub2e4\uc591\ud588\ub2e4. \uc81c\uc784\uc2a4 \ub9e4\ub4dc\uc2a8 \uc740 50\uac1c \uac00\ub7c9\uc73c\ub85c \uc608\uc0c1\ud588\uace0, \uac8c\uc624\ub974\uae30 \ud3f4\uc740 \uc774\uac83\ub3c4 \ub9ce\ub2e4\uace0 \uc5ec\uaca8\uc11c 45\uac1c\ub098 \uadf8 \uc774\ud558\ub85c \uc608\uce21\ud588\ub2e4. \ubaa9\uacfc \ub4f1\uc758 \ucc99\ucd94\uace8 \uc804\ubc29\uc5d0\ub294 \ube48 \uacf5\uac04\uc774 \uc788\uc5c8\uace0, \ud604\uc0dd \ub3d9\ubb3c\ub4e4(\uc0c8\ub77c\ub4e0\uc9c0)\uc5d0\uc11c\ub3c4 \ubc1c\uacac\ub418\ub294 \uc774\ub7f0 \uacf5\uac04\ub4e4\uc740 \ud638\ud761\uc5d0 \uc4f0\uc778 \uacf5\uae30\uc8fc\uba38\ub2c8\ub85c \ubcf4\uc778\ub2e4. \uac00\uc2b4\uc740 \ub113\uace0 \uc6d0\ud1b5\ud615 \ud749\uacfd\uc774\uc5c8\uace0, \ubcf5\ubd80 \ub291\uace8(\ubc30\uc758 \uac08\ube44)\ub97c \uac00\uc9c0\uace0 \uc788\uc5c8\uc9c0\ub9cc \uc774\uac83\ub4e4\uc740 \ud754\ud558\uac8c \ubc1c\uacac\ub418\ub294 \uac83\uc774 \uc544\ub2c8\uba70, \uc57d\uac04\uc758 \uace8\ud654 \ud604\uc0c1\uc774 \uc788\uc5c8\uc744 \uc218\ub3c4 \uc788\ub2e4. \ud55c \ubc1c\ud45c\uc5d0 \uc758\ud558\uba74, \ubcf5\ub291\uace8\uc740 \uac1c\uccb4\uac00 \uc0b4\uc544 \uc788\uc5c8\uc744 \ub54c \uc0c1\ud574\ub97c \uc785\uc5c8\ub2e4\ub294 \uc99d\uac70\uac00 \ub420 \uc218 \uc788\ub2e4. \ucc3d\uc0ac\uace8 (\uc77c\uba85 \uc704\uc2dc\ubcf8) \ub610\ud55c \uc874\uc7ac\ud588\uc9c0\ub9cc, 1996\ub144\uc774 \ub418\uc5b4\uc11c\uc57c \uc778\uc815\ubc1b\uc558\ub2e4. \uba87\uba87 \uacbd\uc6b0\uc5d0 \ucc3d\uc0ac\uace8\uc774 \ubcf5\ub291\uace8\uacfc \ud63c\ub3d9\ub418\uae30\ub3c4 \ud588\ub2e4. \uc5c9\ub369\uc774\uc758 \uc911\uc2ec \ubf08\uc778 \uc7a5\uace8\uc740 \uc721\uc911\ud588\uace0, \uce58\uace8\uc5d0\ub294 \ud280\uc5b4\ub098\uc628 \ub05d\ub2e8\uc774 \uc788\uc5c8\ub294\ub370 \uc774\ub294 \uadfc\uc721\uc744 \uc5f0\uacb0\uc2dc\ucf1c \uc8fc\ub294 \ub3d9\uc2dc\uc5d0 \ub2e4\ub9ac\uac00 \ub545\uc744 \ub51b\uace0 \uc11c \uc788\uc744 \uc218 \uc788\uac8c \ud574 \uc8fc\ub294 \ubc1b\uce68\ub300 \uc5ed\ud560\uc744 \ud588\ub2e4. \ub9e4\ub4dc\uc2a8\uc740 \ud074\ub9ac\ube14\ub79c\ub4dc \ub85c\uc774\ub4dc \ud0d0\uc0ac\uc5d0\uc11c \ubc1c\uacac\ub41c \uac1c\uccb4\ub4e4 \uc911 \uc57d \uc808\ubc18 \uc815\ub3c4\uac00 \ud06c\uae30\uc5d0 \uad00\uacc4\uc5c6\uc774 \ubc1c \ub05d\ubd80\ubd84\uc758 \uc5f0\ubaa8\uac00 \uc11c\ub85c \uacb0\ud569\ub418\uc5b4 \uc788\uc9c0 \uc54a\uc558\ub2e4\uace0 \uae30\uc220\ud588\ub2e4. \ub9e4\ub4dc\uc2a8\uc740 \uc774\uac83\uc774 \uc131\uc801\uc778 \ud2b9\uc9d5\uc77c \uac00\ub2a5\uc131\uc774 \uc788\uc73c\uba70, \uc774\ub294 \uc554\ucef7\uc774 \uc54c\uc744 \ud488\uae30 \uc27d\ub3c4\ub85d \ubf08\uac00 \ubd99\uc5b4 \uc788\uc9c0 \uc54a\uc740 \uac83\uc774\ub77c\uace0 \uc8fc\uc7a5\ud588\ub2e4. \uadf8\ub7ec\ub098 \uc774 \uc0dd\uac01\uc740 \uc774\ud6c4 \uc9c0\uc9c0\ub97c \ubcc4\ub85c \ubc1b\uc9c0 \ubabb\ud588\ub2e4.", "paragraph_answer": "\uc54c\ub85c\uc0ac\uc6b0\ub8e8\uc2a4\ub294 \ubaa9\uc5d0 9\uac1c, \ub4f1\uc5d0 14\uac1c, \ucc9c\uace8\uc5d0 5\uac1c\uc758 \ucc99\ucd94\uace8\uc744 \uac00\uc9c0\uace0 \uc788\uc5c8\ub2e4. \uaf2c\ub9ac\uc758 \ucc99\ucd94\uace8 \uc218\ub294 \ubd88\uba85\uc774\uace0 \uac1c\uac1c\uc758 \ud06c\uae30\ub3c4 \ub2e4\uc591\ud588\ub2e4. \uc81c\uc784\uc2a4 \ub9e4\ub4dc\uc2a8 \uc740 50\uac1c \uac00\ub7c9\uc73c\ub85c \uc608\uc0c1\ud588\uace0, \uac8c\uc624\ub974\uae30 \ud3f4\uc740 \uc774\uac83\ub3c4 \ub9ce\ub2e4\uace0 \uc5ec\uaca8\uc11c 45\uac1c\ub098 \uadf8 \uc774\ud558\ub85c \uc608\uce21\ud588\ub2e4. \ubaa9\uacfc \ub4f1\uc758 \ucc99\ucd94\uace8 \uc804\ubc29\uc5d0\ub294 \ube48 \uacf5\uac04\uc774 \uc788\uc5c8\uace0, \ud604\uc0dd \ub3d9\ubb3c\ub4e4(\uc0c8\ub77c\ub4e0\uc9c0)\uc5d0\uc11c\ub3c4 \ubc1c\uacac\ub418\ub294 \uc774\ub7f0 \uacf5\uac04\ub4e4\uc740 \ud638\ud761\uc5d0 \uc4f0\uc778 \uacf5\uae30\uc8fc\uba38\ub2c8\ub85c \ubcf4\uc778\ub2e4. \uac00\uc2b4\uc740 \ub113\uace0 \uc6d0\ud1b5\ud615 \ud749\uacfd\uc774\uc5c8\uace0, \ubcf5\ubd80 \ub291\uace8(\ubc30\uc758 \uac08\ube44)\ub97c \uac00\uc9c0\uace0 \uc788\uc5c8\uc9c0\ub9cc \uc774\uac83\ub4e4\uc740 \ud754\ud558\uac8c \ubc1c\uacac\ub418\ub294 \uac83\uc774 \uc544\ub2c8\uba70, \uc57d\uac04\uc758 \uace8\ud654 \ud604\uc0c1\uc774 \uc788\uc5c8\uc744 \uc218\ub3c4 \uc788\ub2e4. \ud55c \ubc1c\ud45c\uc5d0 \uc758\ud558\uba74, \ubcf5\ub291\uace8\uc740 \uac1c\uccb4\uac00 \uc0b4\uc544 \uc788\uc5c8\uc744 \ub54c \uc0c1\ud574\ub97c \uc785\uc5c8\ub2e4\ub294 \uc99d\uac70\uac00 \ub420 \uc218 \uc788\ub2e4. \ucc3d\uc0ac\uace8 (\uc77c\uba85 \uc704\uc2dc\ubcf8) \ub610\ud55c \uc874\uc7ac\ud588\uc9c0\ub9cc, 1996\ub144\uc774 \ub418\uc5b4\uc11c\uc57c \uc778\uc815\ubc1b\uc558\ub2e4. \uba87\uba87 \uacbd\uc6b0\uc5d0 \ucc3d\uc0ac\uace8\uc774 \ubcf5\ub291\uace8\uacfc \ud63c\ub3d9\ub418\uae30\ub3c4 \ud588\ub2e4. \uc5c9\ub369\uc774\uc758 \uc911\uc2ec \ubf08\uc778 \uc7a5\uace8\uc740 \uc721\uc911\ud588\uace0, \uce58\uace8\uc5d0\ub294 \ud280\uc5b4\ub098\uc628 \ub05d\ub2e8\uc774 \uc788\uc5c8\ub294\ub370 \uc774\ub294 \uadfc\uc721\uc744 \uc5f0\uacb0\uc2dc\ucf1c \uc8fc\ub294 \ub3d9\uc2dc\uc5d0 \ub2e4\ub9ac\uac00 \ub545\uc744 \ub51b\uace0 \uc11c \uc788\uc744 \uc218 \uc788\uac8c \ud574 \uc8fc\ub294 \ubc1b\uce68\ub300 \uc5ed\ud560\uc744 \ud588\ub2e4. \ub9e4\ub4dc\uc2a8\uc740 \ud074\ub9ac\ube14\ub79c\ub4dc \ub85c\uc774\ub4dc \ud0d0\uc0ac\uc5d0\uc11c \ubc1c\uacac\ub41c \uac1c\uccb4\ub4e4 \uc911 \uc57d \uc808\ubc18 \uc815\ub3c4\uac00 \ud06c\uae30\uc5d0 \uad00\uacc4\uc5c6\uc774 \ubc1c \ub05d\ubd80\ubd84\uc758 \uc5f0\ubaa8\uac00 \uc11c\ub85c \uacb0\ud569\ub418\uc5b4 \uc788\uc9c0 \uc54a\uc558\ub2e4\uace0 \uae30\uc220\ud588\ub2e4. \ub9e4\ub4dc\uc2a8\uc740 \uc774\uac83\uc774 \uc131\uc801\uc778 \ud2b9\uc9d5\uc77c \uac00\ub2a5\uc131\uc774 \uc788\uc73c\uba70, \uc774\ub294 \uc554\ucef7\uc774 \uc54c\uc744 \ud488\uae30 \uc27d\ub3c4\ub85d \ubf08\uac00 \ubd99\uc5b4 \uc788\uc9c0 \uc54a\uc740 \uac83\uc774\ub77c\uace0 \uc8fc\uc7a5\ud588\ub2e4. \uadf8\ub7ec\ub098 \uc774 \uc0dd\uac01\uc740 \uc774\ud6c4 \uc9c0\uc9c0\ub97c \ubcc4\ub85c \ubc1b\uc9c0 \ubabb\ud588\ub2e4.", "sentence_answer": " \uc81c\uc784\uc2a4 \ub9e4\ub4dc\uc2a8 \uc740 50\uac1c \uac00\ub7c9\uc73c\ub85c \uc608\uc0c1\ud588\uace0, \uac8c\uc624\ub974\uae30 \ud3f4\uc740 \uc774\uac83\ub3c4 \ub9ce\ub2e4\uace0 \uc5ec\uaca8\uc11c 45\uac1c\ub098 \uadf8 \uc774\ud558\ub85c \uc608\uce21\ud588\ub2e4.", "paragraph_id": "6465441-3-1", "paragraph_question": "question: \uc54c\ub85c\uc0ac\uc6b0\ub8e8\uc2a4\uc758 \ucc99\ucd94\uace8 \uac2f\uc218\uac00 50\uac1c\ub77c\uace0 \uc608\uc0c1\ud55c \uc0ac\ub78c\uc740?, context: \uc54c\ub85c\uc0ac\uc6b0\ub8e8\uc2a4\ub294 \ubaa9\uc5d0 9\uac1c, \ub4f1\uc5d0 14\uac1c, \ucc9c\uace8\uc5d0 5\uac1c\uc758 \ucc99\ucd94\uace8\uc744 \uac00\uc9c0\uace0 \uc788\uc5c8\ub2e4. \uaf2c\ub9ac\uc758 \ucc99\ucd94\uace8 \uc218\ub294 \ubd88\uba85\uc774\uace0 \uac1c\uac1c\uc758 \ud06c\uae30\ub3c4 \ub2e4\uc591\ud588\ub2e4. \uc81c\uc784\uc2a4 \ub9e4\ub4dc\uc2a8\uc740 50\uac1c \uac00\ub7c9\uc73c\ub85c \uc608\uc0c1\ud588\uace0, \uac8c\uc624\ub974\uae30 \ud3f4\uc740 \uc774\uac83\ub3c4 \ub9ce\ub2e4\uace0 \uc5ec\uaca8\uc11c 45\uac1c\ub098 \uadf8 \uc774\ud558\ub85c \uc608\uce21\ud588\ub2e4. \ubaa9\uacfc \ub4f1\uc758 \ucc99\ucd94\uace8 \uc804\ubc29\uc5d0\ub294 \ube48 \uacf5\uac04\uc774 \uc788\uc5c8\uace0, \ud604\uc0dd \ub3d9\ubb3c\ub4e4(\uc0c8\ub77c\ub4e0\uc9c0)\uc5d0\uc11c\ub3c4 \ubc1c\uacac\ub418\ub294 \uc774\ub7f0 \uacf5\uac04\ub4e4\uc740 \ud638\ud761\uc5d0 \uc4f0\uc778 \uacf5\uae30\uc8fc\uba38\ub2c8\ub85c \ubcf4\uc778\ub2e4. \uac00\uc2b4\uc740 \ub113\uace0 \uc6d0\ud1b5\ud615 \ud749\uacfd\uc774\uc5c8\uace0, \ubcf5\ubd80 \ub291\uace8(\ubc30\uc758 \uac08\ube44)\ub97c \uac00\uc9c0\uace0 \uc788\uc5c8\uc9c0\ub9cc \uc774\uac83\ub4e4\uc740 \ud754\ud558\uac8c \ubc1c\uacac\ub418\ub294 \uac83\uc774 \uc544\ub2c8\uba70, \uc57d\uac04\uc758 \uace8\ud654 \ud604\uc0c1\uc774 \uc788\uc5c8\uc744 \uc218\ub3c4 \uc788\ub2e4. \ud55c \ubc1c\ud45c\uc5d0 \uc758\ud558\uba74, \ubcf5\ub291\uace8\uc740 \uac1c\uccb4\uac00 \uc0b4\uc544 \uc788\uc5c8\uc744 \ub54c \uc0c1\ud574\ub97c \uc785\uc5c8\ub2e4\ub294 \uc99d\uac70\uac00 \ub420 \uc218 \uc788\ub2e4. \ucc3d\uc0ac\uace8 (\uc77c\uba85 \uc704\uc2dc\ubcf8) \ub610\ud55c \uc874\uc7ac\ud588\uc9c0\ub9cc, 1996\ub144\uc774 \ub418\uc5b4\uc11c\uc57c \uc778\uc815\ubc1b\uc558\ub2e4. \uba87\uba87 \uacbd\uc6b0\uc5d0 \ucc3d\uc0ac\uace8\uc774 \ubcf5\ub291\uace8\uacfc \ud63c\ub3d9\ub418\uae30\ub3c4 \ud588\ub2e4. \uc5c9\ub369\uc774\uc758 \uc911\uc2ec \ubf08\uc778 \uc7a5\uace8\uc740 \uc721\uc911\ud588\uace0, \uce58\uace8\uc5d0\ub294 \ud280\uc5b4\ub098\uc628 \ub05d\ub2e8\uc774 \uc788\uc5c8\ub294\ub370 \uc774\ub294 \uadfc\uc721\uc744 \uc5f0\uacb0\uc2dc\ucf1c \uc8fc\ub294 \ub3d9\uc2dc\uc5d0 \ub2e4\ub9ac\uac00 \ub545\uc744 \ub51b\uace0 \uc11c \uc788\uc744 \uc218 \uc788\uac8c \ud574 \uc8fc\ub294 \ubc1b\uce68\ub300 \uc5ed\ud560\uc744 \ud588\ub2e4. \ub9e4\ub4dc\uc2a8\uc740 \ud074\ub9ac\ube14\ub79c\ub4dc \ub85c\uc774\ub4dc \ud0d0\uc0ac\uc5d0\uc11c \ubc1c\uacac\ub41c \uac1c\uccb4\ub4e4 \uc911 \uc57d \uc808\ubc18 \uc815\ub3c4\uac00 \ud06c\uae30\uc5d0 \uad00\uacc4\uc5c6\uc774 \ubc1c \ub05d\ubd80\ubd84\uc758 \uc5f0\ubaa8\uac00 \uc11c\ub85c \uacb0\ud569\ub418\uc5b4 \uc788\uc9c0 \uc54a\uc558\ub2e4\uace0 \uae30\uc220\ud588\ub2e4. \ub9e4\ub4dc\uc2a8\uc740 \uc774\uac83\uc774 \uc131\uc801\uc778 \ud2b9\uc9d5\uc77c \uac00\ub2a5\uc131\uc774 \uc788\uc73c\uba70, \uc774\ub294 \uc554\ucef7\uc774 \uc54c\uc744 \ud488\uae30 \uc27d\ub3c4\ub85d \ubf08\uac00 \ubd99\uc5b4 \uc788\uc9c0 \uc54a\uc740 \uac83\uc774\ub77c\uace0 \uc8fc\uc7a5\ud588\ub2e4. \uadf8\ub7ec\ub098 \uc774 \uc0dd\uac01\uc740 \uc774\ud6c4 \uc9c0\uc9c0\ub97c \ubcc4\ub85c \ubc1b\uc9c0 \ubabb\ud588\ub2e4."} {"question": "\uc1fc\ud2b8\ud2b8\ub799\uc120\uc218 \uae40\uc544\ub791\uc774 \ud0dc\uc5b4\ub09c \uacf3\uc740?", "paragraph": "\uc804\uc8fc\uc5d0\uc11c \ud0dc\uc5b4\ub09c \uae40\uc544\ub791\uc740 \ucd08\ub4f1\ud559\uad50 \uc2dc\uc808 \uc624\ube60\ub97c \ub530\ub77c \uc2a4\ucf00\uc774\ud2b8\uc7a5\uc744 \ub2e4\ub2c8\ub2e4\uac00 \uc2a4\ucf00\uc774\ud2b8 \uad50\uc0ac\uc758 \ub208\uc5d0 \ub744\uc5b4 \uc6b4\ub3d9\uc744 \uc2dc\uc791\ud558\uc600\ub2e4. \uc774\ud6c4 \uccb4\uacc4\uc801\uc778 \ud6c8\ub828\uc744 \uc704\ud558\uc5ec \uc11c\uc6b8\uc758 \uc911\ud559\uad50\ub85c \uc804\ud559\ud55c \uae40\uc544\ub791\uc740 \uc11c\uc6b8\uc758 \ub300\ud45c\ud300 \uc120\ubc30 \uc9d1\uc5d0\uc11c \ud558\uc219 \uc0dd\ud65c\uc744 \uc2dc\uc791\ud558\uc600\ub294\ub370 \uc5b4\ub9b0 \ub098\uc774\uc758 \uae40\uc544\ub791\uc744 \ud63c\uc790 \uc11c\uc6b8\ub85c \ubcf4\ub0b4\uace0 \uae40\uc544\ub791\uc758 \ubd80\ubaa8\ub294 \uc18d\uc0c1\ud55c \uc801\uc774 \ub9ce\uc558\uc5c8\ub2e4\uace0 \ud55c\ub2e4. \uc804\uc9c0\ud6c8\ub828\uacfc \uc7a5\ube44\uc5d0 \uc218\uc6d4\uce58 \uc54a\uc740 \ube44\uc6a9\uc774 \ub4e4\uc5c8\uc73c\uba70 \ud558\uc219\ube44\ub098 \ub808\uc2a8\ube44\ub3c4 \uba87 \ub2ec\uc529 \ubc00\ub9b0 \uc801\uc774 \uc788\uc5c8\ub294\ub370 \uadf8\ub7ec\ud55c \uacbd\uc81c\uc801\uc778 \uc5b4\ub824\uc6c0\uc73c\ub85c \uc1fc\ud2b8\ud2b8\ub799\uc744 \uadf8\ub9cc\ub450\ub77c\uace0 \ub9d0\ud558\uace0 \uc2f6\uc5c8\ub358 \uc801\ub3c4 \uc788\uc5c8\ub2e4\uace0 \ud55c\ub2e4. \uae40\uc544\ub791\uc740 \ucc98\uc74c\uc5d0 \uc1fc\ud2b8\ud2b8\ub799 \uacbd\uae30 \uc911 \ubab8\uc2f8\uc6c0\uc744 \ud53c\ud574\uc11c \uc21c\ub465\uc774\ub77c\uace0 \uc2a4\ucf00\uc774\ud2b8\uc7a5\uc5d0\uc11c \ubd88\ub838\uc9c0\ub9cc, \uac1d\uc9c0 \uc0dd\ud65c\uc740 \uadf8\ub97c \ub3c5\ud558\uac8c \ub9cc\ub4e4\uc5b4\uc11c \ubab8\uc2f8\uc6c0\uc5d0\uc11c\ub3c4 \ubc00\ub9ac\uc9c0 \uc54a\uace0 \uacfc\uac10\ud55c \ud50c\ub808\uc774\ub97c \ubcf4\uc5ec\uc8fc\uae30 \uc2dc\uc791\ud558\uc600\ub2e4. \uc21c\ud558\uac8c \ubcf4\uc774\ub358 \uae40\uc544\ub791\uc774 \uc720\ub3c5 \uc2a4\ucf00\uc774\ud2b8\uc7a5\uc5d0\uc11c\ub294 \ub2e4\ub978 \ubaa8\uc2b5\uc744 \ubcf4\uc5ec\uc8fc\uae30 \uc2dc\uc791\ud55c \uac83\uc774\uc5c8\ub2e4.", "answer": "\uc804\uc8fc", "sentence": "\uc804\uc8fc \uc5d0\uc11c \ud0dc\uc5b4\ub09c \uae40\uc544\ub791\uc740 \ucd08\ub4f1\ud559\uad50 \uc2dc\uc808 \uc624\ube60\ub97c \ub530\ub77c \uc2a4\ucf00\uc774\ud2b8\uc7a5\uc744 \ub2e4\ub2c8\ub2e4\uac00 \uc2a4\ucf00\uc774\ud2b8 \uad50\uc0ac\uc758 \ub208\uc5d0 \ub744\uc5b4 \uc6b4\ub3d9\uc744 \uc2dc\uc791\ud558\uc600\ub2e4.", "paragraph_sentence": " \uc804\uc8fc \uc5d0\uc11c \ud0dc\uc5b4\ub09c \uae40\uc544\ub791\uc740 \ucd08\ub4f1\ud559\uad50 \uc2dc\uc808 \uc624\ube60\ub97c \ub530\ub77c \uc2a4\ucf00\uc774\ud2b8\uc7a5\uc744 \ub2e4\ub2c8\ub2e4\uac00 \uc2a4\ucf00\uc774\ud2b8 \uad50\uc0ac\uc758 \ub208\uc5d0 \ub744\uc5b4 \uc6b4\ub3d9\uc744 \uc2dc\uc791\ud558\uc600\ub2e4. \uc774\ud6c4 \uccb4\uacc4\uc801\uc778 \ud6c8\ub828\uc744 \uc704\ud558\uc5ec \uc11c\uc6b8\uc758 \uc911\ud559\uad50\ub85c \uc804\ud559\ud55c \uae40\uc544\ub791\uc740 \uc11c\uc6b8\uc758 \ub300\ud45c\ud300 \uc120\ubc30 \uc9d1\uc5d0\uc11c \ud558\uc219 \uc0dd\ud65c\uc744 \uc2dc\uc791\ud558\uc600\ub294\ub370 \uc5b4\ub9b0 \ub098\uc774\uc758 \uae40\uc544\ub791\uc744 \ud63c\uc790 \uc11c\uc6b8\ub85c \ubcf4\ub0b4\uace0 \uae40\uc544\ub791\uc758 \ubd80\ubaa8\ub294 \uc18d\uc0c1\ud55c \uc801\uc774 \ub9ce\uc558\uc5c8\ub2e4\uace0 \ud55c\ub2e4. \uc804\uc9c0\ud6c8\ub828\uacfc \uc7a5\ube44\uc5d0 \uc218\uc6d4\uce58 \uc54a\uc740 \ube44\uc6a9\uc774 \ub4e4\uc5c8\uc73c\uba70 \ud558\uc219\ube44\ub098 \ub808\uc2a8\ube44\ub3c4 \uba87 \ub2ec\uc529 \ubc00\ub9b0 \uc801\uc774 \uc788\uc5c8\ub294\ub370 \uadf8\ub7ec\ud55c \uacbd\uc81c\uc801\uc778 \uc5b4\ub824\uc6c0\uc73c\ub85c \uc1fc\ud2b8\ud2b8\ub799\uc744 \uadf8\ub9cc\ub450\ub77c\uace0 \ub9d0\ud558\uace0 \uc2f6\uc5c8\ub358 \uc801\ub3c4 \uc788\uc5c8\ub2e4\uace0 \ud55c\ub2e4. \uae40\uc544\ub791\uc740 \ucc98\uc74c\uc5d0 \uc1fc\ud2b8\ud2b8\ub799 \uacbd\uae30 \uc911 \ubab8\uc2f8\uc6c0\uc744 \ud53c\ud574\uc11c \uc21c\ub465\uc774\ub77c\uace0 \uc2a4\ucf00\uc774\ud2b8\uc7a5\uc5d0\uc11c \ubd88\ub838\uc9c0\ub9cc, \uac1d\uc9c0 \uc0dd\ud65c\uc740 \uadf8\ub97c \ub3c5\ud558\uac8c \ub9cc\ub4e4\uc5b4\uc11c \ubab8\uc2f8\uc6c0\uc5d0\uc11c\ub3c4 \ubc00\ub9ac\uc9c0 \uc54a\uace0 \uacfc\uac10\ud55c \ud50c\ub808\uc774\ub97c \ubcf4\uc5ec\uc8fc\uae30 \uc2dc\uc791\ud558\uc600\ub2e4. \uc21c\ud558\uac8c \ubcf4\uc774\ub358 \uae40\uc544\ub791\uc774 \uc720\ub3c5 \uc2a4\ucf00\uc774\ud2b8\uc7a5\uc5d0\uc11c\ub294 \ub2e4\ub978 \ubaa8\uc2b5\uc744 \ubcf4\uc5ec\uc8fc\uae30 \uc2dc\uc791\ud55c \uac83\uc774\uc5c8\ub2e4.", "paragraph_answer": " \uc804\uc8fc \uc5d0\uc11c \ud0dc\uc5b4\ub09c \uae40\uc544\ub791\uc740 \ucd08\ub4f1\ud559\uad50 \uc2dc\uc808 \uc624\ube60\ub97c \ub530\ub77c \uc2a4\ucf00\uc774\ud2b8\uc7a5\uc744 \ub2e4\ub2c8\ub2e4\uac00 \uc2a4\ucf00\uc774\ud2b8 \uad50\uc0ac\uc758 \ub208\uc5d0 \ub744\uc5b4 \uc6b4\ub3d9\uc744 \uc2dc\uc791\ud558\uc600\ub2e4. \uc774\ud6c4 \uccb4\uacc4\uc801\uc778 \ud6c8\ub828\uc744 \uc704\ud558\uc5ec \uc11c\uc6b8\uc758 \uc911\ud559\uad50\ub85c \uc804\ud559\ud55c \uae40\uc544\ub791\uc740 \uc11c\uc6b8\uc758 \ub300\ud45c\ud300 \uc120\ubc30 \uc9d1\uc5d0\uc11c \ud558\uc219 \uc0dd\ud65c\uc744 \uc2dc\uc791\ud558\uc600\ub294\ub370 \uc5b4\ub9b0 \ub098\uc774\uc758 \uae40\uc544\ub791\uc744 \ud63c\uc790 \uc11c\uc6b8\ub85c \ubcf4\ub0b4\uace0 \uae40\uc544\ub791\uc758 \ubd80\ubaa8\ub294 \uc18d\uc0c1\ud55c \uc801\uc774 \ub9ce\uc558\uc5c8\ub2e4\uace0 \ud55c\ub2e4. \uc804\uc9c0\ud6c8\ub828\uacfc \uc7a5\ube44\uc5d0 \uc218\uc6d4\uce58 \uc54a\uc740 \ube44\uc6a9\uc774 \ub4e4\uc5c8\uc73c\uba70 \ud558\uc219\ube44\ub098 \ub808\uc2a8\ube44\ub3c4 \uba87 \ub2ec\uc529 \ubc00\ub9b0 \uc801\uc774 \uc788\uc5c8\ub294\ub370 \uadf8\ub7ec\ud55c \uacbd\uc81c\uc801\uc778 \uc5b4\ub824\uc6c0\uc73c\ub85c \uc1fc\ud2b8\ud2b8\ub799\uc744 \uadf8\ub9cc\ub450\ub77c\uace0 \ub9d0\ud558\uace0 \uc2f6\uc5c8\ub358 \uc801\ub3c4 \uc788\uc5c8\ub2e4\uace0 \ud55c\ub2e4. \uae40\uc544\ub791\uc740 \ucc98\uc74c\uc5d0 \uc1fc\ud2b8\ud2b8\ub799 \uacbd\uae30 \uc911 \ubab8\uc2f8\uc6c0\uc744 \ud53c\ud574\uc11c \uc21c\ub465\uc774\ub77c\uace0 \uc2a4\ucf00\uc774\ud2b8\uc7a5\uc5d0\uc11c \ubd88\ub838\uc9c0\ub9cc, \uac1d\uc9c0 \uc0dd\ud65c\uc740 \uadf8\ub97c \ub3c5\ud558\uac8c \ub9cc\ub4e4\uc5b4\uc11c \ubab8\uc2f8\uc6c0\uc5d0\uc11c\ub3c4 \ubc00\ub9ac\uc9c0 \uc54a\uace0 \uacfc\uac10\ud55c \ud50c\ub808\uc774\ub97c \ubcf4\uc5ec\uc8fc\uae30 \uc2dc\uc791\ud558\uc600\ub2e4. \uc21c\ud558\uac8c \ubcf4\uc774\ub358 \uae40\uc544\ub791\uc774 \uc720\ub3c5 \uc2a4\ucf00\uc774\ud2b8\uc7a5\uc5d0\uc11c\ub294 \ub2e4\ub978 \ubaa8\uc2b5\uc744 \ubcf4\uc5ec\uc8fc\uae30 \uc2dc\uc791\ud55c \uac83\uc774\uc5c8\ub2e4.", "sentence_answer": " \uc804\uc8fc \uc5d0\uc11c \ud0dc\uc5b4\ub09c \uae40\uc544\ub791\uc740 \ucd08\ub4f1\ud559\uad50 \uc2dc\uc808 \uc624\ube60\ub97c \ub530\ub77c \uc2a4\ucf00\uc774\ud2b8\uc7a5\uc744 \ub2e4\ub2c8\ub2e4\uac00 \uc2a4\ucf00\uc774\ud2b8 \uad50\uc0ac\uc758 \ub208\uc5d0 \ub744\uc5b4 \uc6b4\ub3d9\uc744 \uc2dc\uc791\ud558\uc600\ub2e4.", "paragraph_id": "5998192-0-0", "paragraph_question": "question: \uc1fc\ud2b8\ud2b8\ub799\uc120\uc218 \uae40\uc544\ub791\uc774 \ud0dc\uc5b4\ub09c \uacf3\uc740?, context: \uc804\uc8fc\uc5d0\uc11c \ud0dc\uc5b4\ub09c \uae40\uc544\ub791\uc740 \ucd08\ub4f1\ud559\uad50 \uc2dc\uc808 \uc624\ube60\ub97c \ub530\ub77c \uc2a4\ucf00\uc774\ud2b8\uc7a5\uc744 \ub2e4\ub2c8\ub2e4\uac00 \uc2a4\ucf00\uc774\ud2b8 \uad50\uc0ac\uc758 \ub208\uc5d0 \ub744\uc5b4 \uc6b4\ub3d9\uc744 \uc2dc\uc791\ud558\uc600\ub2e4. \uc774\ud6c4 \uccb4\uacc4\uc801\uc778 \ud6c8\ub828\uc744 \uc704\ud558\uc5ec \uc11c\uc6b8\uc758 \uc911\ud559\uad50\ub85c \uc804\ud559\ud55c \uae40\uc544\ub791\uc740 \uc11c\uc6b8\uc758 \ub300\ud45c\ud300 \uc120\ubc30 \uc9d1\uc5d0\uc11c \ud558\uc219 \uc0dd\ud65c\uc744 \uc2dc\uc791\ud558\uc600\ub294\ub370 \uc5b4\ub9b0 \ub098\uc774\uc758 \uae40\uc544\ub791\uc744 \ud63c\uc790 \uc11c\uc6b8\ub85c \ubcf4\ub0b4\uace0 \uae40\uc544\ub791\uc758 \ubd80\ubaa8\ub294 \uc18d\uc0c1\ud55c \uc801\uc774 \ub9ce\uc558\uc5c8\ub2e4\uace0 \ud55c\ub2e4. \uc804\uc9c0\ud6c8\ub828\uacfc \uc7a5\ube44\uc5d0 \uc218\uc6d4\uce58 \uc54a\uc740 \ube44\uc6a9\uc774 \ub4e4\uc5c8\uc73c\uba70 \ud558\uc219\ube44\ub098 \ub808\uc2a8\ube44\ub3c4 \uba87 \ub2ec\uc529 \ubc00\ub9b0 \uc801\uc774 \uc788\uc5c8\ub294\ub370 \uadf8\ub7ec\ud55c \uacbd\uc81c\uc801\uc778 \uc5b4\ub824\uc6c0\uc73c\ub85c \uc1fc\ud2b8\ud2b8\ub799\uc744 \uadf8\ub9cc\ub450\ub77c\uace0 \ub9d0\ud558\uace0 \uc2f6\uc5c8\ub358 \uc801\ub3c4 \uc788\uc5c8\ub2e4\uace0 \ud55c\ub2e4. \uae40\uc544\ub791\uc740 \ucc98\uc74c\uc5d0 \uc1fc\ud2b8\ud2b8\ub799 \uacbd\uae30 \uc911 \ubab8\uc2f8\uc6c0\uc744 \ud53c\ud574\uc11c \uc21c\ub465\uc774\ub77c\uace0 \uc2a4\ucf00\uc774\ud2b8\uc7a5\uc5d0\uc11c \ubd88\ub838\uc9c0\ub9cc, \uac1d\uc9c0 \uc0dd\ud65c\uc740 \uadf8\ub97c \ub3c5\ud558\uac8c \ub9cc\ub4e4\uc5b4\uc11c \ubab8\uc2f8\uc6c0\uc5d0\uc11c\ub3c4 \ubc00\ub9ac\uc9c0 \uc54a\uace0 \uacfc\uac10\ud55c \ud50c\ub808\uc774\ub97c \ubcf4\uc5ec\uc8fc\uae30 \uc2dc\uc791\ud558\uc600\ub2e4. \uc21c\ud558\uac8c \ubcf4\uc774\ub358 \uae40\uc544\ub791\uc774 \uc720\ub3c5 \uc2a4\ucf00\uc774\ud2b8\uc7a5\uc5d0\uc11c\ub294 \ub2e4\ub978 \ubaa8\uc2b5\uc744 \ubcf4\uc5ec\uc8fc\uae30 \uc2dc\uc791\ud55c \uac83\uc774\uc5c8\ub2e4."} {"question": "\uc77c\ubcf8\uc758 \uc2e0\ubb38\uc911 \uac00\uc7a5 \ub9ce\uc740 \ubc1c\ud589 \ubd80\uc218\ub97c \uc790\ub791\ud558\ub294 \uc2e0\ubb38 \uc774\ub984\uc740?", "paragraph": "\uc77c\ubcf8\uc758 \uc77c\uc77c \ubc1c\ud589\ubd80\uc218\uac00 \ub9ce\uc740 \uc720\ub825\ud55c \uc2e0\ubb38\uc73c\ub85c\ub294 \uc694\ubbf8\uc6b0\ub9ac \uc2e0\ubb38, \uc544\uc0ac\ud788 \uc2e0\ubb38, \ub9c8\uc774\ub2c8\uce58 \uc2e0\ubb38, \ub2c8\ud63c\ucf00\uc774\uc790\uc774 \uc2e0\ubb38, \uc0b0\ucf00\uc774 \uc2e0\ubb38\uc774 \uc788\uc73c\uba70 \uc774 \uc911 \uc694\ubbf8\uc6b0\ub9ac \uc2e0\ubb38\uc774 \uac00\uc7a5 \ub9ce\uc740 \ubc1c\ud589 \ubd80\uc218\ub97c \uc790\ub791\ud55c\ub2e4. \uc77c\ubcf8\uc758 \uc2e0\ubb38\uc740 19\uc138\uae30\uc5d0 \uc815\uce58\uc801\uc778 \uc8fc\uc7a5(\ud2b9\ud788 \uc815\ubd80\uc5d0 \ub300\ud55c \ube44\ud310)\uc744 \ubaa9\uc801\uc73c\ub85c \ud558\uc5ec \ubc1c\uc871\ud558\uc600\uc73c\uba70 \uc81c2\ucc28 \uc138\uacc4 \ub300\uc804 \ub3c4\uc911\uc5d0\ub294 \uc5b4\uc6a9\ud654\ub418\uae30\ub3c4 \ud558\uc600\uc9c0\ub9cc \ud604\uc7ac\uc5d0\ub294 \ub300\uc911\uc758 \uc778\uae30\uc5d0 \uc601\ud569\ud558\ub824\ub294 \uacbd\ud5a5\uc774 \uac15\ud574\uc84c\ub2e4\ub294 \ube44\ud310\uc744 \ubc1b\uae30\ub3c4 \ud55c\ub2e4. \uc77c\ubcf8\uc5d0\uc11c \uc2e0\ubb38\uc5d0 \ub300\ud55c \ud1b5\uc81c\ub294 \uc5c6\uc9c0\ub9cc \uc2e0\ubb38\uc0ac \ub0b4\uc5d0\uc11c \uc790\uc728\uc801\uc73c\ub85c \uc77c\ubcf8 \ucc9c\ud669\uc5d0 \ub300\ud55c \ube44\ud310 \uae30\uc0ac\uc640 \uac19\uc774 \ubbfc\uac10\ud55c \uae30\uc0ac\ub294 \uac8c\uc7ac\ub418\uc9c0 \ubabb\ud558\ub3c4\ub85d \ud1b5\uc81c\ud558\uace0 \uc788\ub2e4. \ucd5c\uadfc\uc5d0\ub294 \uc778\ud130\ub137 \uc2e0\ubb38\uc0ac\ub4e4\uc774 \uacc4\uc18d \uc0dd\uaca8\ub098\uace0 \uc788\uc9c0\ub9cc, \uc5ed\ub7c9 \ubd80\uc871\uacfc \uc800\uc870\ud55c \uad6c\ub3c5\ub960\ub85c \uc778\ud574 \uc624\ub798 \uac00\uc9c0 \ubabb\ud558\ub294 \uc2e0\ubb38\uc0ac\ub4e4\uc774 \ub300\ub2e4\uc218\uc774\ub2e4.", "answer": "\uc694\ubbf8\uc6b0\ub9ac \uc2e0\ubb38", "sentence": "\uc77c\ubcf8\uc758 \uc77c\uc77c \ubc1c\ud589\ubd80\uc218\uac00 \ub9ce\uc740 \uc720\ub825\ud55c \uc2e0\ubb38\uc73c\ub85c\ub294 \uc694\ubbf8\uc6b0\ub9ac \uc2e0\ubb38 ,", "paragraph_sentence": " \uc77c\ubcf8\uc758 \uc77c\uc77c \ubc1c\ud589\ubd80\uc218\uac00 \ub9ce\uc740 \uc720\ub825\ud55c \uc2e0\ubb38\uc73c\ub85c\ub294 \uc694\ubbf8\uc6b0\ub9ac \uc2e0\ubb38 , \uc544\uc0ac\ud788 \uc2e0\ubb38, \ub9c8\uc774\ub2c8\uce58 \uc2e0\ubb38, \ub2c8\ud63c\ucf00\uc774\uc790\uc774 \uc2e0\ubb38, \uc0b0\ucf00\uc774 \uc2e0\ubb38\uc774 \uc788\uc73c\uba70 \uc774 \uc911 \uc694\ubbf8\uc6b0\ub9ac \uc2e0\ubb38\uc774 \uac00\uc7a5 \ub9ce\uc740 \ubc1c\ud589 \ubd80\uc218\ub97c \uc790\ub791\ud55c\ub2e4. \uc77c\ubcf8\uc758 \uc2e0\ubb38\uc740 19\uc138\uae30\uc5d0 \uc815\uce58\uc801\uc778 \uc8fc\uc7a5(\ud2b9\ud788 \uc815\ubd80\uc5d0 \ub300\ud55c \ube44\ud310)\uc744 \ubaa9\uc801\uc73c\ub85c \ud558\uc5ec \ubc1c\uc871\ud558\uc600\uc73c\uba70 \uc81c2\ucc28 \uc138\uacc4 \ub300\uc804 \ub3c4\uc911\uc5d0\ub294 \uc5b4\uc6a9\ud654\ub418\uae30\ub3c4 \ud558\uc600\uc9c0\ub9cc \ud604\uc7ac\uc5d0\ub294 \ub300\uc911\uc758 \uc778\uae30\uc5d0 \uc601\ud569\ud558\ub824\ub294 \uacbd\ud5a5\uc774 \uac15\ud574\uc84c\ub2e4\ub294 \ube44\ud310\uc744 \ubc1b\uae30\ub3c4 \ud55c\ub2e4. \uc77c\ubcf8\uc5d0\uc11c \uc2e0\ubb38\uc5d0 \ub300\ud55c \ud1b5\uc81c\ub294 \uc5c6\uc9c0\ub9cc \uc2e0\ubb38\uc0ac \ub0b4\uc5d0\uc11c \uc790\uc728\uc801\uc73c\ub85c \uc77c\ubcf8 \ucc9c\ud669\uc5d0 \ub300\ud55c \ube44\ud310 \uae30\uc0ac\uc640 \uac19\uc774 \ubbfc\uac10\ud55c \uae30\uc0ac\ub294 \uac8c\uc7ac\ub418\uc9c0 \ubabb\ud558\ub3c4\ub85d \ud1b5\uc81c\ud558\uace0 \uc788\ub2e4. \ucd5c\uadfc\uc5d0\ub294 \uc778\ud130\ub137 \uc2e0\ubb38\uc0ac\ub4e4\uc774 \uacc4\uc18d \uc0dd\uaca8\ub098\uace0 \uc788\uc9c0\ub9cc, \uc5ed\ub7c9 \ubd80\uc871\uacfc \uc800\uc870\ud55c \uad6c\ub3c5\ub960\ub85c \uc778\ud574 \uc624\ub798 \uac00\uc9c0 \ubabb\ud558\ub294 \uc2e0\ubb38\uc0ac\ub4e4\uc774 \ub300\ub2e4\uc218\uc774\ub2e4.", "paragraph_answer": "\uc77c\ubcf8\uc758 \uc77c\uc77c \ubc1c\ud589\ubd80\uc218\uac00 \ub9ce\uc740 \uc720\ub825\ud55c \uc2e0\ubb38\uc73c\ub85c\ub294 \uc694\ubbf8\uc6b0\ub9ac \uc2e0\ubb38 , \uc544\uc0ac\ud788 \uc2e0\ubb38, \ub9c8\uc774\ub2c8\uce58 \uc2e0\ubb38, \ub2c8\ud63c\ucf00\uc774\uc790\uc774 \uc2e0\ubb38, \uc0b0\ucf00\uc774 \uc2e0\ubb38\uc774 \uc788\uc73c\uba70 \uc774 \uc911 \uc694\ubbf8\uc6b0\ub9ac \uc2e0\ubb38\uc774 \uac00\uc7a5 \ub9ce\uc740 \ubc1c\ud589 \ubd80\uc218\ub97c \uc790\ub791\ud55c\ub2e4. \uc77c\ubcf8\uc758 \uc2e0\ubb38\uc740 19\uc138\uae30\uc5d0 \uc815\uce58\uc801\uc778 \uc8fc\uc7a5(\ud2b9\ud788 \uc815\ubd80\uc5d0 \ub300\ud55c \ube44\ud310)\uc744 \ubaa9\uc801\uc73c\ub85c \ud558\uc5ec \ubc1c\uc871\ud558\uc600\uc73c\uba70 \uc81c2\ucc28 \uc138\uacc4 \ub300\uc804 \ub3c4\uc911\uc5d0\ub294 \uc5b4\uc6a9\ud654\ub418\uae30\ub3c4 \ud558\uc600\uc9c0\ub9cc \ud604\uc7ac\uc5d0\ub294 \ub300\uc911\uc758 \uc778\uae30\uc5d0 \uc601\ud569\ud558\ub824\ub294 \uacbd\ud5a5\uc774 \uac15\ud574\uc84c\ub2e4\ub294 \ube44\ud310\uc744 \ubc1b\uae30\ub3c4 \ud55c\ub2e4. \uc77c\ubcf8\uc5d0\uc11c \uc2e0\ubb38\uc5d0 \ub300\ud55c \ud1b5\uc81c\ub294 \uc5c6\uc9c0\ub9cc \uc2e0\ubb38\uc0ac \ub0b4\uc5d0\uc11c \uc790\uc728\uc801\uc73c\ub85c \uc77c\ubcf8 \ucc9c\ud669\uc5d0 \ub300\ud55c \ube44\ud310 \uae30\uc0ac\uc640 \uac19\uc774 \ubbfc\uac10\ud55c \uae30\uc0ac\ub294 \uac8c\uc7ac\ub418\uc9c0 \ubabb\ud558\ub3c4\ub85d \ud1b5\uc81c\ud558\uace0 \uc788\ub2e4. \ucd5c\uadfc\uc5d0\ub294 \uc778\ud130\ub137 \uc2e0\ubb38\uc0ac\ub4e4\uc774 \uacc4\uc18d \uc0dd\uaca8\ub098\uace0 \uc788\uc9c0\ub9cc, \uc5ed\ub7c9 \ubd80\uc871\uacfc \uc800\uc870\ud55c \uad6c\ub3c5\ub960\ub85c \uc778\ud574 \uc624\ub798 \uac00\uc9c0 \ubabb\ud558\ub294 \uc2e0\ubb38\uc0ac\ub4e4\uc774 \ub300\ub2e4\uc218\uc774\ub2e4.", "sentence_answer": "\uc77c\ubcf8\uc758 \uc77c\uc77c \ubc1c\ud589\ubd80\uc218\uac00 \ub9ce\uc740 \uc720\ub825\ud55c \uc2e0\ubb38\uc73c\ub85c\ub294 \uc694\ubbf8\uc6b0\ub9ac \uc2e0\ubb38 ,", "paragraph_id": "6173085-29-0", "paragraph_question": "question: \uc77c\ubcf8\uc758 \uc2e0\ubb38\uc911 \uac00\uc7a5 \ub9ce\uc740 \ubc1c\ud589 \ubd80\uc218\ub97c \uc790\ub791\ud558\ub294 \uc2e0\ubb38 \uc774\ub984\uc740?, context: \uc77c\ubcf8\uc758 \uc77c\uc77c \ubc1c\ud589\ubd80\uc218\uac00 \ub9ce\uc740 \uc720\ub825\ud55c \uc2e0\ubb38\uc73c\ub85c\ub294 \uc694\ubbf8\uc6b0\ub9ac \uc2e0\ubb38, \uc544\uc0ac\ud788 \uc2e0\ubb38, \ub9c8\uc774\ub2c8\uce58 \uc2e0\ubb38, \ub2c8\ud63c\ucf00\uc774\uc790\uc774 \uc2e0\ubb38, \uc0b0\ucf00\uc774 \uc2e0\ubb38\uc774 \uc788\uc73c\uba70 \uc774 \uc911 \uc694\ubbf8\uc6b0\ub9ac \uc2e0\ubb38\uc774 \uac00\uc7a5 \ub9ce\uc740 \ubc1c\ud589 \ubd80\uc218\ub97c \uc790\ub791\ud55c\ub2e4. \uc77c\ubcf8\uc758 \uc2e0\ubb38\uc740 19\uc138\uae30\uc5d0 \uc815\uce58\uc801\uc778 \uc8fc\uc7a5(\ud2b9\ud788 \uc815\ubd80\uc5d0 \ub300\ud55c \ube44\ud310)\uc744 \ubaa9\uc801\uc73c\ub85c \ud558\uc5ec \ubc1c\uc871\ud558\uc600\uc73c\uba70 \uc81c2\ucc28 \uc138\uacc4 \ub300\uc804 \ub3c4\uc911\uc5d0\ub294 \uc5b4\uc6a9\ud654\ub418\uae30\ub3c4 \ud558\uc600\uc9c0\ub9cc \ud604\uc7ac\uc5d0\ub294 \ub300\uc911\uc758 \uc778\uae30\uc5d0 \uc601\ud569\ud558\ub824\ub294 \uacbd\ud5a5\uc774 \uac15\ud574\uc84c\ub2e4\ub294 \ube44\ud310\uc744 \ubc1b\uae30\ub3c4 \ud55c\ub2e4. \uc77c\ubcf8\uc5d0\uc11c \uc2e0\ubb38\uc5d0 \ub300\ud55c \ud1b5\uc81c\ub294 \uc5c6\uc9c0\ub9cc \uc2e0\ubb38\uc0ac \ub0b4\uc5d0\uc11c \uc790\uc728\uc801\uc73c\ub85c \uc77c\ubcf8 \ucc9c\ud669\uc5d0 \ub300\ud55c \ube44\ud310 \uae30\uc0ac\uc640 \uac19\uc774 \ubbfc\uac10\ud55c \uae30\uc0ac\ub294 \uac8c\uc7ac\ub418\uc9c0 \ubabb\ud558\ub3c4\ub85d \ud1b5\uc81c\ud558\uace0 \uc788\ub2e4. \ucd5c\uadfc\uc5d0\ub294 \uc778\ud130\ub137 \uc2e0\ubb38\uc0ac\ub4e4\uc774 \uacc4\uc18d \uc0dd\uaca8\ub098\uace0 \uc788\uc9c0\ub9cc, \uc5ed\ub7c9 \ubd80\uc871\uacfc \uc800\uc870\ud55c \uad6c\ub3c5\ub960\ub85c \uc778\ud574 \uc624\ub798 \uac00\uc9c0 \ubabb\ud558\ub294 \uc2e0\ubb38\uc0ac\ub4e4\uc774 \ub300\ub2e4\uc218\uc774\ub2e4."} {"question": "\uae30\uc6d0\uc804 5\uc138\uae30 \ub9d0 \ubb34\ub835 \uc2dc\uc778\ub4e4\uc740 \uc2e0\ub4e4\uacfc \uc804\uc124\uc801\uc778 \uc778\ubb3c\ub4e4\uc5d0\uac8c \uc5d0\ub85c\uba54\ub178\uc2a4\ub97c \ud55c\uba85\uc529 \uc815\ud574 \ub450\uc5c8\ub294\ub370 \ubc18\ud574 \uc5d0\ub85c\uba54\ub178\uc2a4\ub97c \ub450\uc9c0 \uc54a\uc740 \uc778\ubb3c\uc740 \ub204\uad6c\uc778\uac00?", "paragraph": "\uace0\ub300 \uc911\ubc18 \uc774\ud6c4\uc5d0\ub294 \ub0a8\uc2e0\uacfc \ub0a8\uc131 \uc601\uc6c5 \uac04\uc758 \uad00\uacc4\uc5d0 \ub300\ud55c \uc2e0\ud654\uac00 \ub354\uc6b1 \ube48\ubc88\ud574\uc9c0\uae30 \uc2dc\uc791\ud588\uc73c\uba70, \uc774\uac83\uc740 \uae30\uc6d0\uc804 630\ub144 \uacbd\uc5d0 \ud655\uc0b0\ub418\uc5c8\ub2e4\uace0 \uc5ec\uaca8\uc9c0\ub294 \uad50\uc721\uc801 \ub3d9\uc131\uc560(Eros paidikos, \u03c0\u03b1\u03b9\u03b4\u03b9\u03ba\u03cc\u03c2 \u1f14\u03c1\u03c9\u03c2)\uc758 \ub3d9\uc2dc\uc801 \ubc1c\uc804\uacfc\ub3c4 \uad00\uacc4\uac00 \uae4a\ub2e4. \uae30\uc6d0\uc804 5\uc138\uae30\uac00 \ub05d\ub0a0 \ubb34\ub835, \uc2dc\uc778\ub4e4\uc740 \uc544\ub808\uc2a4\ub97c \uc81c\uc678\ud55c \ubaa8\ub4e0 \uc911\uc694\ud55c \uc2e0\uacfc \ub9ce\uc740 \uc804\uc124\uc801 \uc778\ubb3c\uc5d0\uac8c \uc801\uc5b4\ub3c4 \ud55c \uba85\uc758 \uc5d0\ub85c\uba54\ub178\uc2a4(\uc131\uc801\uc778 \uad00\uacc4\uc758 \uc5b4\ub9b0 \uc18c\ub144)\ub97c \uc815\ud574\ub450\uc5c8\ub2e4. \uc544\ud0ac\ub808\uc6b0\uc2a4\uc640 \ud30c\ud2b8\ub85c\ud074\ub85c\uc2a4\ucc98\ub7fc \uc774\uc804\uc5d0 \uc874\uc7ac\ud558\ub358 \uc2e0\ud654 \ub610\ud55c \uc554\uc2dc\uc801\uc778 \ub3d9\uc131\uc560 \uad00\uacc4\uac00 \uac00\ubbf8\ub418\uc5c8\ub2e4. \uc54c\ub809\uc0b0\ub4dc\ub9ac\uc544\uc758 \uc2dc\uc778\ub4e4\uc740 \ucc98\uc74c\uc73c\ub85c \uc790\uc2e0\ub4e4\uc758 \uc720\ud589\uc5d0 \ub9de\ucd94\uc5b4 \uadf8\ub9ac\uc2a4\uc758 \uc2e0\ud654\uc801 \uc778\ubb3c\uc758 \uc774\uc57c\uae30\ub97c \uac01\uc0c9\ud558\uc600\uc73c\uba70, \ucd08\uae30 \ub85c\ub9c8 \uc81c\uad6d\uc758 \ubb38\ud559 \uc2e0\ud654 \uc218\uc9d1\uac00\ub4e4\uc5d0\uac8c\ub294 \uc774\uac83\uc774 \ub354\uc6b1 \uc77c\ubc18\uc801\uc778 \ud604\uc0c1\uc774 \ub418\uc5c8\ub2e4.", "answer": "\uc544\ub808\uc2a4", "sentence": "\uae30\uc6d0\uc804 5\uc138\uae30\uac00 \ub05d\ub0a0 \ubb34\ub835, \uc2dc\uc778\ub4e4\uc740 \uc544\ub808\uc2a4 \ub97c \uc81c\uc678\ud55c \ubaa8\ub4e0 \uc911\uc694\ud55c \uc2e0\uacfc \ub9ce\uc740 \uc804\uc124\uc801 \uc778\ubb3c\uc5d0\uac8c \uc801\uc5b4\ub3c4 \ud55c \uba85\uc758 \uc5d0\ub85c\uba54\ub178\uc2a4(\uc131\uc801\uc778 \uad00\uacc4\uc758 \uc5b4\ub9b0 \uc18c\ub144)\ub97c \uc815\ud574\ub450\uc5c8\ub2e4.", "paragraph_sentence": "\uace0\ub300 \uc911\ubc18 \uc774\ud6c4\uc5d0\ub294 \ub0a8\uc2e0\uacfc \ub0a8\uc131 \uc601\uc6c5 \uac04\uc758 \uad00\uacc4\uc5d0 \ub300\ud55c \uc2e0\ud654\uac00 \ub354\uc6b1 \ube48\ubc88\ud574\uc9c0\uae30 \uc2dc\uc791\ud588\uc73c\uba70, \uc774\uac83\uc740 \uae30\uc6d0\uc804 630\ub144 \uacbd\uc5d0 \ud655\uc0b0\ub418\uc5c8\ub2e4\uace0 \uc5ec\uaca8\uc9c0\ub294 \uad50\uc721\uc801 \ub3d9\uc131\uc560(Eros paidikos, \u03c0\u03b1\u03b9\u03b4\u03b9\u03ba\u03cc\u03c2 \u1f14\u03c1\u03c9\u03c2)\uc758 \ub3d9\uc2dc\uc801 \ubc1c\uc804\uacfc\ub3c4 \uad00\uacc4\uac00 \uae4a\ub2e4. \uae30\uc6d0\uc804 5\uc138\uae30\uac00 \ub05d\ub0a0 \ubb34\ub835, \uc2dc\uc778\ub4e4\uc740 \uc544\ub808\uc2a4 \ub97c \uc81c\uc678\ud55c \ubaa8\ub4e0 \uc911\uc694\ud55c \uc2e0\uacfc \ub9ce\uc740 \uc804\uc124\uc801 \uc778\ubb3c\uc5d0\uac8c \uc801\uc5b4\ub3c4 \ud55c \uba85\uc758 \uc5d0\ub85c\uba54\ub178\uc2a4(\uc131\uc801\uc778 \uad00\uacc4\uc758 \uc5b4\ub9b0 \uc18c\ub144)\ub97c \uc815\ud574\ub450\uc5c8\ub2e4. \uc544\ud0ac\ub808\uc6b0\uc2a4\uc640 \ud30c\ud2b8\ub85c\ud074\ub85c\uc2a4\ucc98\ub7fc \uc774\uc804\uc5d0 \uc874\uc7ac\ud558\ub358 \uc2e0\ud654 \ub610\ud55c \uc554\uc2dc\uc801\uc778 \ub3d9\uc131\uc560 \uad00\uacc4\uac00 \uac00\ubbf8\ub418\uc5c8\ub2e4. \uc54c\ub809\uc0b0\ub4dc\ub9ac\uc544\uc758 \uc2dc\uc778\ub4e4\uc740 \ucc98\uc74c\uc73c\ub85c \uc790\uc2e0\ub4e4\uc758 \uc720\ud589\uc5d0 \ub9de\ucd94\uc5b4 \uadf8\ub9ac\uc2a4\uc758 \uc2e0\ud654\uc801 \uc778\ubb3c\uc758 \uc774\uc57c\uae30\ub97c \uac01\uc0c9\ud558\uc600\uc73c\uba70, \ucd08\uae30 \ub85c\ub9c8 \uc81c\uad6d\uc758 \ubb38\ud559 \uc2e0\ud654 \uc218\uc9d1\uac00\ub4e4\uc5d0\uac8c\ub294 \uc774\uac83\uc774 \ub354\uc6b1 \uc77c\ubc18\uc801\uc778 \ud604\uc0c1\uc774 \ub418\uc5c8\ub2e4.", "paragraph_answer": "\uace0\ub300 \uc911\ubc18 \uc774\ud6c4\uc5d0\ub294 \ub0a8\uc2e0\uacfc \ub0a8\uc131 \uc601\uc6c5 \uac04\uc758 \uad00\uacc4\uc5d0 \ub300\ud55c \uc2e0\ud654\uac00 \ub354\uc6b1 \ube48\ubc88\ud574\uc9c0\uae30 \uc2dc\uc791\ud588\uc73c\uba70, \uc774\uac83\uc740 \uae30\uc6d0\uc804 630\ub144 \uacbd\uc5d0 \ud655\uc0b0\ub418\uc5c8\ub2e4\uace0 \uc5ec\uaca8\uc9c0\ub294 \uad50\uc721\uc801 \ub3d9\uc131\uc560(Eros paidikos, \u03c0\u03b1\u03b9\u03b4\u03b9\u03ba\u03cc\u03c2 \u1f14\u03c1\u03c9\u03c2)\uc758 \ub3d9\uc2dc\uc801 \ubc1c\uc804\uacfc\ub3c4 \uad00\uacc4\uac00 \uae4a\ub2e4. \uae30\uc6d0\uc804 5\uc138\uae30\uac00 \ub05d\ub0a0 \ubb34\ub835, \uc2dc\uc778\ub4e4\uc740 \uc544\ub808\uc2a4 \ub97c \uc81c\uc678\ud55c \ubaa8\ub4e0 \uc911\uc694\ud55c \uc2e0\uacfc \ub9ce\uc740 \uc804\uc124\uc801 \uc778\ubb3c\uc5d0\uac8c \uc801\uc5b4\ub3c4 \ud55c \uba85\uc758 \uc5d0\ub85c\uba54\ub178\uc2a4(\uc131\uc801\uc778 \uad00\uacc4\uc758 \uc5b4\ub9b0 \uc18c\ub144)\ub97c \uc815\ud574\ub450\uc5c8\ub2e4. \uc544\ud0ac\ub808\uc6b0\uc2a4\uc640 \ud30c\ud2b8\ub85c\ud074\ub85c\uc2a4\ucc98\ub7fc \uc774\uc804\uc5d0 \uc874\uc7ac\ud558\ub358 \uc2e0\ud654 \ub610\ud55c \uc554\uc2dc\uc801\uc778 \ub3d9\uc131\uc560 \uad00\uacc4\uac00 \uac00\ubbf8\ub418\uc5c8\ub2e4. \uc54c\ub809\uc0b0\ub4dc\ub9ac\uc544\uc758 \uc2dc\uc778\ub4e4\uc740 \ucc98\uc74c\uc73c\ub85c \uc790\uc2e0\ub4e4\uc758 \uc720\ud589\uc5d0 \ub9de\ucd94\uc5b4 \uadf8\ub9ac\uc2a4\uc758 \uc2e0\ud654\uc801 \uc778\ubb3c\uc758 \uc774\uc57c\uae30\ub97c \uac01\uc0c9\ud558\uc600\uc73c\uba70, \ucd08\uae30 \ub85c\ub9c8 \uc81c\uad6d\uc758 \ubb38\ud559 \uc2e0\ud654 \uc218\uc9d1\uac00\ub4e4\uc5d0\uac8c\ub294 \uc774\uac83\uc774 \ub354\uc6b1 \uc77c\ubc18\uc801\uc778 \ud604\uc0c1\uc774 \ub418\uc5c8\ub2e4.", "sentence_answer": "\uae30\uc6d0\uc804 5\uc138\uae30\uac00 \ub05d\ub0a0 \ubb34\ub835, \uc2dc\uc778\ub4e4\uc740 \uc544\ub808\uc2a4 \ub97c \uc81c\uc678\ud55c \ubaa8\ub4e0 \uc911\uc694\ud55c \uc2e0\uacfc \ub9ce\uc740 \uc804\uc124\uc801 \uc778\ubb3c\uc5d0\uac8c \uc801\uc5b4\ub3c4 \ud55c \uba85\uc758 \uc5d0\ub85c\uba54\ub178\uc2a4(\uc131\uc801\uc778 \uad00\uacc4\uc758 \uc5b4\ub9b0 \uc18c\ub144)\ub97c \uc815\ud574\ub450\uc5c8\ub2e4.", "paragraph_id": "6464905-5-1", "paragraph_question": "question: \uae30\uc6d0\uc804 5\uc138\uae30 \ub9d0 \ubb34\ub835 \uc2dc\uc778\ub4e4\uc740 \uc2e0\ub4e4\uacfc \uc804\uc124\uc801\uc778 \uc778\ubb3c\ub4e4\uc5d0\uac8c \uc5d0\ub85c\uba54\ub178\uc2a4\ub97c \ud55c\uba85\uc529 \uc815\ud574 \ub450\uc5c8\ub294\ub370 \ubc18\ud574 \uc5d0\ub85c\uba54\ub178\uc2a4\ub97c \ub450\uc9c0 \uc54a\uc740 \uc778\ubb3c\uc740 \ub204\uad6c\uc778\uac00?, context: \uace0\ub300 \uc911\ubc18 \uc774\ud6c4\uc5d0\ub294 \ub0a8\uc2e0\uacfc \ub0a8\uc131 \uc601\uc6c5 \uac04\uc758 \uad00\uacc4\uc5d0 \ub300\ud55c \uc2e0\ud654\uac00 \ub354\uc6b1 \ube48\ubc88\ud574\uc9c0\uae30 \uc2dc\uc791\ud588\uc73c\uba70, \uc774\uac83\uc740 \uae30\uc6d0\uc804 630\ub144 \uacbd\uc5d0 \ud655\uc0b0\ub418\uc5c8\ub2e4\uace0 \uc5ec\uaca8\uc9c0\ub294 \uad50\uc721\uc801 \ub3d9\uc131\uc560(Eros paidikos, \u03c0\u03b1\u03b9\u03b4\u03b9\u03ba\u03cc\u03c2 \u1f14\u03c1\u03c9\u03c2)\uc758 \ub3d9\uc2dc\uc801 \ubc1c\uc804\uacfc\ub3c4 \uad00\uacc4\uac00 \uae4a\ub2e4. \uae30\uc6d0\uc804 5\uc138\uae30\uac00 \ub05d\ub0a0 \ubb34\ub835, \uc2dc\uc778\ub4e4\uc740 \uc544\ub808\uc2a4\ub97c \uc81c\uc678\ud55c \ubaa8\ub4e0 \uc911\uc694\ud55c \uc2e0\uacfc \ub9ce\uc740 \uc804\uc124\uc801 \uc778\ubb3c\uc5d0\uac8c \uc801\uc5b4\ub3c4 \ud55c \uba85\uc758 \uc5d0\ub85c\uba54\ub178\uc2a4(\uc131\uc801\uc778 \uad00\uacc4\uc758 \uc5b4\ub9b0 \uc18c\ub144)\ub97c \uc815\ud574\ub450\uc5c8\ub2e4. \uc544\ud0ac\ub808\uc6b0\uc2a4\uc640 \ud30c\ud2b8\ub85c\ud074\ub85c\uc2a4\ucc98\ub7fc \uc774\uc804\uc5d0 \uc874\uc7ac\ud558\ub358 \uc2e0\ud654 \ub610\ud55c \uc554\uc2dc\uc801\uc778 \ub3d9\uc131\uc560 \uad00\uacc4\uac00 \uac00\ubbf8\ub418\uc5c8\ub2e4. \uc54c\ub809\uc0b0\ub4dc\ub9ac\uc544\uc758 \uc2dc\uc778\ub4e4\uc740 \ucc98\uc74c\uc73c\ub85c \uc790\uc2e0\ub4e4\uc758 \uc720\ud589\uc5d0 \ub9de\ucd94\uc5b4 \uadf8\ub9ac\uc2a4\uc758 \uc2e0\ud654\uc801 \uc778\ubb3c\uc758 \uc774\uc57c\uae30\ub97c \uac01\uc0c9\ud558\uc600\uc73c\uba70, \ucd08\uae30 \ub85c\ub9c8 \uc81c\uad6d\uc758 \ubb38\ud559 \uc2e0\ud654 \uc218\uc9d1\uac00\ub4e4\uc5d0\uac8c\ub294 \uc774\uac83\uc774 \ub354\uc6b1 \uc77c\ubc18\uc801\uc778 \ud604\uc0c1\uc774 \ub418\uc5c8\ub2e4."} {"question": "\ud56d\uc131\uc774 \uc0dd\uc131\ud558\ub294 \uc790\uae30\uc7a5\uacfc \ud56d\uc131\ud48d\uc740 \ubcc4\uc758 \ubb34\uc5c7\uc5d0 \uc601\ud5a5\uc744 \uc8fc\ub294\uac00?", "paragraph": "\ud56d\uc131\uc758 \uc790\uc804 \uc8fc\uae30\ub294 \ubd84\uad11\ud559\uc801\uc778 \uce21\uc815\ubc95\uc744 \uc4f0\uac70\ub098 \ud56d\uc131\ud45c\uba74 \ud751\uc810\uc774 \uc774\ub3d9\ud558\ub294 \uc18d\ub3c4\ub97c \uce21\uc815\ud568\uc73c\ub85c\uc368 \uac12\uc744 \uad6c\ud560 \uc218 \uc788\ub2e4. \uc80a\uc740 \ubcc4\uc758 \uc801\ub3c4 \uc790\uc804 \uc18d\ub3c4\ub294 \ucd08\uc18d 100\ud0ac\ub85c\ubbf8\ud130\ub97c \ub118\ub294\ub2e4. \uc608\ub97c \ub4e4\uba74 \ubd84\uad11\ud615 B\uc758 \uccad\uc0c9 \uc8fc\uacc4\uc5f4\uc131 \uc544\ucf00\ub974\ub098\ub974\uc758 \uacbd\uc6b0 \uc801\ub3c4 \uc790\uc804 \uc18d\ub3c4\ub294 \ucd08\ub2f9 225\ud0ac\ub85c\ubbf8\ud130 \uc774\uc0c1\uc5d0 \uc774\ub978\ub2e4. \uc774 \ubcc4\uc740 \uc790\uc804 \uc18d\ub3c4\uac00 \ub108\ubb34 \ube68\ub77c\uc11c \uc801\ub3c4 \ucabd\uc774 \ubd80\ud480\uc5b4 \uc624\ub978 \ud68c\uc804 \ud0c0\uc6d0\uccb4 \ubaa8\uc591\uc774\ub2e4. \ub9cc\uc57d \uc774 \ubcc4\uc774 \uc870\uae08 \ub354 \ube60\ub974\uac8c \uc790\uc804\ud574\uc11c \ucd08\ub2f9 300\ud0ac\ub85c\ubbf8\ud130\uc5d0 \uc774\ub974\uba74 \ubcc4\uc740 \uc0b0\uc0b0\uc870\uac01\uc73c\ub85c \ucc22\uaca8 \ub098\uac14\uc744 \uac83\uc774\ub2e4. \ubc18\ub300\ub85c \ud0dc\uc591\uc758 \uc790\uc804 \uc8fc\uae30\ub294 25\uc77c~35\uc77c \uc815\ub3c4\ub85c \uc801\ub3c4\uc5d0\uc11c\uc758 \uacf5\uc804 \uc18d\ub3c4\ub294 \ucd08\ub2f9 1.994\ud0ac\ub85c\ubbf8\ud130\uc5d0 \ubd88\uacfc\ud558\ub2e4. \ud56d\uc131\uc774 \uc0dd\uc131\ud558\ub294 \uc790\uae30\uc7a5 \ubc0f \ud56d\uc131\ud48d\uc740 \uc8fc\uacc4\uc5f4 \uae30\uac04\uc5d0 \uba38\ubb34\ub974\ub294 \ub3d9\uc548 \ubcc4\uc758 \uc790\uc804 \uc18d\ub3c4\ub97c \ud06c\uac8c \ub0ae\ucd98\ub2e4.", "answer": "\uc790\uc804 \uc18d\ub3c4", "sentence": "\uc80a\uc740 \ubcc4\uc758 \uc801\ub3c4 \uc790\uc804 \uc18d\ub3c4 \ub294 \ucd08\uc18d 100\ud0ac\ub85c\ubbf8\ud130\ub97c \ub118\ub294\ub2e4.", "paragraph_sentence": "\ud56d\uc131\uc758 \uc790\uc804 \uc8fc\uae30\ub294 \ubd84\uad11\ud559\uc801\uc778 \uce21\uc815\ubc95\uc744 \uc4f0\uac70\ub098 \ud56d\uc131\ud45c\uba74 \ud751\uc810\uc774 \uc774\ub3d9\ud558\ub294 \uc18d\ub3c4\ub97c \uce21\uc815\ud568\uc73c\ub85c\uc368 \uac12\uc744 \uad6c\ud560 \uc218 \uc788\ub2e4. \uc80a\uc740 \ubcc4\uc758 \uc801\ub3c4 \uc790\uc804 \uc18d\ub3c4 \ub294 \ucd08\uc18d 100\ud0ac\ub85c\ubbf8\ud130\ub97c \ub118\ub294\ub2e4. \uc608\ub97c \ub4e4\uba74 \ubd84\uad11\ud615 B\uc758 \uccad\uc0c9 \uc8fc\uacc4\uc5f4\uc131 \uc544\ucf00\ub974\ub098\ub974\uc758 \uacbd\uc6b0 \uc801\ub3c4 \uc790\uc804 \uc18d\ub3c4\ub294 \ucd08\ub2f9 225\ud0ac\ub85c\ubbf8\ud130 \uc774\uc0c1\uc5d0 \uc774\ub978\ub2e4. \uc774 \ubcc4\uc740 \uc790\uc804 \uc18d\ub3c4\uac00 \ub108\ubb34 \ube68\ub77c\uc11c \uc801\ub3c4 \ucabd\uc774 \ubd80\ud480\uc5b4 \uc624\ub978 \ud68c\uc804 \ud0c0\uc6d0\uccb4 \ubaa8\uc591\uc774\ub2e4. \ub9cc\uc57d \uc774 \ubcc4\uc774 \uc870\uae08 \ub354 \ube60\ub974\uac8c \uc790\uc804\ud574\uc11c \ucd08\ub2f9 300\ud0ac\ub85c\ubbf8\ud130\uc5d0 \uc774\ub974\uba74 \ubcc4\uc740 \uc0b0\uc0b0\uc870\uac01\uc73c\ub85c \ucc22\uaca8 \ub098\uac14\uc744 \uac83\uc774\ub2e4. \ubc18\ub300\ub85c \ud0dc\uc591\uc758 \uc790\uc804 \uc8fc\uae30\ub294 25\uc77c~35\uc77c \uc815\ub3c4\ub85c \uc801\ub3c4\uc5d0\uc11c\uc758 \uacf5\uc804 \uc18d\ub3c4\ub294 \ucd08\ub2f9 1.994\ud0ac\ub85c\ubbf8\ud130\uc5d0 \ubd88\uacfc\ud558\ub2e4. \ud56d\uc131\uc774 \uc0dd\uc131\ud558\ub294 \uc790\uae30\uc7a5 \ubc0f \ud56d\uc131\ud48d\uc740 \uc8fc\uacc4\uc5f4 \uae30\uac04\uc5d0 \uba38\ubb34\ub974\ub294 \ub3d9\uc548 \ubcc4\uc758 \uc790\uc804 \uc18d\ub3c4\ub97c \ud06c\uac8c \ub0ae\ucd98\ub2e4.", "paragraph_answer": "\ud56d\uc131\uc758 \uc790\uc804 \uc8fc\uae30\ub294 \ubd84\uad11\ud559\uc801\uc778 \uce21\uc815\ubc95\uc744 \uc4f0\uac70\ub098 \ud56d\uc131\ud45c\uba74 \ud751\uc810\uc774 \uc774\ub3d9\ud558\ub294 \uc18d\ub3c4\ub97c \uce21\uc815\ud568\uc73c\ub85c\uc368 \uac12\uc744 \uad6c\ud560 \uc218 \uc788\ub2e4. \uc80a\uc740 \ubcc4\uc758 \uc801\ub3c4 \uc790\uc804 \uc18d\ub3c4 \ub294 \ucd08\uc18d 100\ud0ac\ub85c\ubbf8\ud130\ub97c \ub118\ub294\ub2e4. \uc608\ub97c \ub4e4\uba74 \ubd84\uad11\ud615 B\uc758 \uccad\uc0c9 \uc8fc\uacc4\uc5f4\uc131 \uc544\ucf00\ub974\ub098\ub974\uc758 \uacbd\uc6b0 \uc801\ub3c4 \uc790\uc804 \uc18d\ub3c4\ub294 \ucd08\ub2f9 225\ud0ac\ub85c\ubbf8\ud130 \uc774\uc0c1\uc5d0 \uc774\ub978\ub2e4. \uc774 \ubcc4\uc740 \uc790\uc804 \uc18d\ub3c4\uac00 \ub108\ubb34 \ube68\ub77c\uc11c \uc801\ub3c4 \ucabd\uc774 \ubd80\ud480\uc5b4 \uc624\ub978 \ud68c\uc804 \ud0c0\uc6d0\uccb4 \ubaa8\uc591\uc774\ub2e4. \ub9cc\uc57d \uc774 \ubcc4\uc774 \uc870\uae08 \ub354 \ube60\ub974\uac8c \uc790\uc804\ud574\uc11c \ucd08\ub2f9 300\ud0ac\ub85c\ubbf8\ud130\uc5d0 \uc774\ub974\uba74 \ubcc4\uc740 \uc0b0\uc0b0\uc870\uac01\uc73c\ub85c \ucc22\uaca8 \ub098\uac14\uc744 \uac83\uc774\ub2e4. \ubc18\ub300\ub85c \ud0dc\uc591\uc758 \uc790\uc804 \uc8fc\uae30\ub294 25\uc77c~35\uc77c \uc815\ub3c4\ub85c \uc801\ub3c4\uc5d0\uc11c\uc758 \uacf5\uc804 \uc18d\ub3c4\ub294 \ucd08\ub2f9 1.994\ud0ac\ub85c\ubbf8\ud130\uc5d0 \ubd88\uacfc\ud558\ub2e4. \ud56d\uc131\uc774 \uc0dd\uc131\ud558\ub294 \uc790\uae30\uc7a5 \ubc0f \ud56d\uc131\ud48d\uc740 \uc8fc\uacc4\uc5f4 \uae30\uac04\uc5d0 \uba38\ubb34\ub974\ub294 \ub3d9\uc548 \ubcc4\uc758 \uc790\uc804 \uc18d\ub3c4\ub97c \ud06c\uac8c \ub0ae\ucd98\ub2e4.", "sentence_answer": "\uc80a\uc740 \ubcc4\uc758 \uc801\ub3c4 \uc790\uc804 \uc18d\ub3c4 \ub294 \ucd08\uc18d 100\ud0ac\ub85c\ubbf8\ud130\ub97c \ub118\ub294\ub2e4.", "paragraph_id": "6570250-7-1", "paragraph_question": "question: \ud56d\uc131\uc774 \uc0dd\uc131\ud558\ub294 \uc790\uae30\uc7a5\uacfc \ud56d\uc131\ud48d\uc740 \ubcc4\uc758 \ubb34\uc5c7\uc5d0 \uc601\ud5a5\uc744 \uc8fc\ub294\uac00?, context: \ud56d\uc131\uc758 \uc790\uc804 \uc8fc\uae30\ub294 \ubd84\uad11\ud559\uc801\uc778 \uce21\uc815\ubc95\uc744 \uc4f0\uac70\ub098 \ud56d\uc131\ud45c\uba74 \ud751\uc810\uc774 \uc774\ub3d9\ud558\ub294 \uc18d\ub3c4\ub97c \uce21\uc815\ud568\uc73c\ub85c\uc368 \uac12\uc744 \uad6c\ud560 \uc218 \uc788\ub2e4. \uc80a\uc740 \ubcc4\uc758 \uc801\ub3c4 \uc790\uc804 \uc18d\ub3c4\ub294 \ucd08\uc18d 100\ud0ac\ub85c\ubbf8\ud130\ub97c \ub118\ub294\ub2e4. \uc608\ub97c \ub4e4\uba74 \ubd84\uad11\ud615 B\uc758 \uccad\uc0c9 \uc8fc\uacc4\uc5f4\uc131 \uc544\ucf00\ub974\ub098\ub974\uc758 \uacbd\uc6b0 \uc801\ub3c4 \uc790\uc804 \uc18d\ub3c4\ub294 \ucd08\ub2f9 225\ud0ac\ub85c\ubbf8\ud130 \uc774\uc0c1\uc5d0 \uc774\ub978\ub2e4. \uc774 \ubcc4\uc740 \uc790\uc804 \uc18d\ub3c4\uac00 \ub108\ubb34 \ube68\ub77c\uc11c \uc801\ub3c4 \ucabd\uc774 \ubd80\ud480\uc5b4 \uc624\ub978 \ud68c\uc804 \ud0c0\uc6d0\uccb4 \ubaa8\uc591\uc774\ub2e4. \ub9cc\uc57d \uc774 \ubcc4\uc774 \uc870\uae08 \ub354 \ube60\ub974\uac8c \uc790\uc804\ud574\uc11c \ucd08\ub2f9 300\ud0ac\ub85c\ubbf8\ud130\uc5d0 \uc774\ub974\uba74 \ubcc4\uc740 \uc0b0\uc0b0\uc870\uac01\uc73c\ub85c \ucc22\uaca8 \ub098\uac14\uc744 \uac83\uc774\ub2e4. \ubc18\ub300\ub85c \ud0dc\uc591\uc758 \uc790\uc804 \uc8fc\uae30\ub294 25\uc77c~35\uc77c \uc815\ub3c4\ub85c \uc801\ub3c4\uc5d0\uc11c\uc758 \uacf5\uc804 \uc18d\ub3c4\ub294 \ucd08\ub2f9 1.994\ud0ac\ub85c\ubbf8\ud130\uc5d0 \ubd88\uacfc\ud558\ub2e4. \ud56d\uc131\uc774 \uc0dd\uc131\ud558\ub294 \uc790\uae30\uc7a5 \ubc0f \ud56d\uc131\ud48d\uc740 \uc8fc\uacc4\uc5f4 \uae30\uac04\uc5d0 \uba38\ubb34\ub974\ub294 \ub3d9\uc548 \ubcc4\uc758 \uc790\uc804 \uc18d\ub3c4\ub97c \ud06c\uac8c \ub0ae\ucd98\ub2e4."} {"question": "GOD \ucd1d\uc0ac\ub839\uad00\uc73c\ub85c \ubd80\ud130 \ubb34\uc5c7\uc73c\ub85c \uc5ec\uaca8\uc84c\ub294\uac00?", "paragraph": "5\ud654\ubd80\ud130\ud130 \ub4f1\uc7a5. \ub370\uc2a4\ud2b8\ub860 \uad34\uba78 \ud6c4, \ucc3b\uc9d1\uc744 \uacbd\uc601\ud558\uba70 \ud3c9\ud654\ub85c\uc6b4 \ub0a0\ub4e4\uc744 \ubcf4\ub0b4\uace0 \uc788\uc5c8\uc9c0\ub9cc, \uc9c4 \ucf00\uc774\uc2a4\ucf00\uc640 \ub9cc\ub098 \uc0c8\ub85c\uc6b4 \uc2f8\uc6c0\uc5d0 \ucc38\uac00\ud558\uac8c \ub41c\ub2e4. \uacbd\uc704\ub098 \ud0dc\uc0dd\uc740 GOD\uc5d0 \uc774\uc804\ubd80\ud130 \uc54c\ub824\uc838 \uc788\uc5b4 GOD \ucd1d\uc0ac\ub839\uad00\uc73c\ub85c\ubd80\ud130\ub3c4 \u300c\uc694\uc8fc\uc758 \uc778\ubb3c\u300d\uc774\ub77c\uace0 \uc5ec\uaca8\uc9c0\uace0 \uc788\uc5c8\ub2e4. \uc804\uc791 \uc774\uc0c1\uc73c\ub85c \uc804\uc120\uc73c\ub85c \ud5a5\ud558\uace0 \uc788\ub2e4. \ubd80\uce5c\uc744 \uc783\uc740 \ucf00\uc774\uc2a4\ucf00\uc5d0 \uc788\uc5b4\uc11c\ub294 \uc2e0\ub8b0 \ud560 \uc218 \uc788\ub294 \uc874\uc7ac\ub85c, \uc5ed\ub300\uc758 \ub77c\uc774\ub354\ub85c\ubd80\ud130\ub3c4 \uc874\uacbd\ubc1b\uace0 \uc788\ub2e4. \uadf9\uc7a5 \uc791\ud488\uc5d0\uc11c 5\uc778 \ub77c\uc774\ub354\uac00 \ubaa8\uc600\uc744 \ub54c\uc5d0\ub3c4, \ud06c\uac8c \uae30\ubed0\ud558\ub294 \ubaa8\uc2b5\uc774 \uadf8\ub824\uc838 \uc788\ub2e4. \uc9d1\uc774\uc790 \ucf00\uc774\uc2a4\ucf00\uc640\uc758 \uc5f0\ub77d\uc7a5\uc774\uae30\ub3c4 \ud55c \u300cCOL\u300d\uc740 \uc544\ud3f4\ub85c\uac00\uc774\uc2a4\ud2b8 \ub4f1\uc774 \uc778\uac04 \ud615\ud0dc\ub85c \ub4e4\uc5b4\uc62c \uc815\ub3c4\ub85c, \ud0dc\uc0dd\uc774 GOD\uc5d0 \uc54c\ub824\uc9c0\uace0 \uc788\uc5b4 \uacbd\uc6b0\uc5d0 \ub530\ub77c\uc11c\ub294 \uc804\uc7a5\uc774 \ub418\ub294 \uc77c\ub3c4 \uc788\uc5c8\ub2e4(\uadf9\uc7a5 \uc791\ud488).", "answer": "\uc694\uc8fc\uc758 \uc778\ubb3c", "sentence": "\uacbd\uc704\ub098 \ud0dc\uc0dd\uc740 GOD\uc5d0 \uc774\uc804\ubd80\ud130 \uc54c\ub824\uc838 \uc788\uc5b4 GOD \ucd1d\uc0ac\ub839\uad00\uc73c\ub85c\ubd80\ud130\ub3c4 \u300c \uc694\uc8fc\uc758 \uc778\ubb3c \u300d\uc774\ub77c\uace0 \uc5ec\uaca8\uc9c0\uace0 \uc788\uc5c8\ub2e4.", "paragraph_sentence": "5\ud654\ubd80\ud130\ud130 \ub4f1\uc7a5. \ub370\uc2a4\ud2b8\ub860 \uad34\uba78 \ud6c4, \ucc3b\uc9d1\uc744 \uacbd\uc601\ud558\uba70 \ud3c9\ud654\ub85c\uc6b4 \ub0a0\ub4e4\uc744 \ubcf4\ub0b4\uace0 \uc788\uc5c8\uc9c0\ub9cc, \uc9c4 \ucf00\uc774\uc2a4\ucf00\uc640 \ub9cc\ub098 \uc0c8\ub85c\uc6b4 \uc2f8\uc6c0\uc5d0 \ucc38\uac00\ud558\uac8c \ub41c\ub2e4. \uacbd\uc704\ub098 \ud0dc\uc0dd\uc740 GOD\uc5d0 \uc774\uc804\ubd80\ud130 \uc54c\ub824\uc838 \uc788\uc5b4 GOD \ucd1d\uc0ac\ub839\uad00\uc73c\ub85c\ubd80\ud130\ub3c4 \u300c \uc694\uc8fc\uc758 \uc778\ubb3c \u300d\uc774\ub77c\uace0 \uc5ec\uaca8\uc9c0\uace0 \uc788\uc5c8\ub2e4. \uc804\uc791 \uc774\uc0c1\uc73c\ub85c \uc804\uc120\uc73c\ub85c \ud5a5\ud558\uace0 \uc788\ub2e4. \ubd80\uce5c\uc744 \uc783\uc740 \ucf00\uc774\uc2a4\ucf00\uc5d0 \uc788\uc5b4\uc11c\ub294 \uc2e0\ub8b0 \ud560 \uc218 \uc788\ub294 \uc874\uc7ac\ub85c, \uc5ed\ub300\uc758 \ub77c\uc774\ub354\ub85c\ubd80\ud130\ub3c4 \uc874\uacbd\ubc1b\uace0 \uc788\ub2e4. \uadf9\uc7a5 \uc791\ud488\uc5d0\uc11c 5\uc778 \ub77c\uc774\ub354\uac00 \ubaa8\uc600\uc744 \ub54c\uc5d0\ub3c4, \ud06c\uac8c \uae30\ubed0\ud558\ub294 \ubaa8\uc2b5\uc774 \uadf8\ub824\uc838 \uc788\ub2e4. \uc9d1\uc774\uc790 \ucf00\uc774\uc2a4\ucf00\uc640\uc758 \uc5f0\ub77d\uc7a5\uc774\uae30\ub3c4 \ud55c \u300cCOL\u300d\uc740 \uc544\ud3f4\ub85c\uac00\uc774\uc2a4\ud2b8 \ub4f1\uc774 \uc778\uac04 \ud615\ud0dc\ub85c \ub4e4\uc5b4\uc62c \uc815\ub3c4\ub85c, \ud0dc\uc0dd\uc774 GOD\uc5d0 \uc54c\ub824\uc9c0\uace0 \uc788\uc5b4 \uacbd\uc6b0\uc5d0 \ub530\ub77c\uc11c\ub294 \uc804\uc7a5\uc774 \ub418\ub294 \uc77c\ub3c4 \uc788\uc5c8\ub2e4(\uadf9\uc7a5 \uc791\ud488).", "paragraph_answer": "5\ud654\ubd80\ud130\ud130 \ub4f1\uc7a5. \ub370\uc2a4\ud2b8\ub860 \uad34\uba78 \ud6c4, \ucc3b\uc9d1\uc744 \uacbd\uc601\ud558\uba70 \ud3c9\ud654\ub85c\uc6b4 \ub0a0\ub4e4\uc744 \ubcf4\ub0b4\uace0 \uc788\uc5c8\uc9c0\ub9cc, \uc9c4 \ucf00\uc774\uc2a4\ucf00\uc640 \ub9cc\ub098 \uc0c8\ub85c\uc6b4 \uc2f8\uc6c0\uc5d0 \ucc38\uac00\ud558\uac8c \ub41c\ub2e4. \uacbd\uc704\ub098 \ud0dc\uc0dd\uc740 GOD\uc5d0 \uc774\uc804\ubd80\ud130 \uc54c\ub824\uc838 \uc788\uc5b4 GOD \ucd1d\uc0ac\ub839\uad00\uc73c\ub85c\ubd80\ud130\ub3c4 \u300c \uc694\uc8fc\uc758 \uc778\ubb3c \u300d\uc774\ub77c\uace0 \uc5ec\uaca8\uc9c0\uace0 \uc788\uc5c8\ub2e4. \uc804\uc791 \uc774\uc0c1\uc73c\ub85c \uc804\uc120\uc73c\ub85c \ud5a5\ud558\uace0 \uc788\ub2e4. \ubd80\uce5c\uc744 \uc783\uc740 \ucf00\uc774\uc2a4\ucf00\uc5d0 \uc788\uc5b4\uc11c\ub294 \uc2e0\ub8b0 \ud560 \uc218 \uc788\ub294 \uc874\uc7ac\ub85c, \uc5ed\ub300\uc758 \ub77c\uc774\ub354\ub85c\ubd80\ud130\ub3c4 \uc874\uacbd\ubc1b\uace0 \uc788\ub2e4. \uadf9\uc7a5 \uc791\ud488\uc5d0\uc11c 5\uc778 \ub77c\uc774\ub354\uac00 \ubaa8\uc600\uc744 \ub54c\uc5d0\ub3c4, \ud06c\uac8c \uae30\ubed0\ud558\ub294 \ubaa8\uc2b5\uc774 \uadf8\ub824\uc838 \uc788\ub2e4. \uc9d1\uc774\uc790 \ucf00\uc774\uc2a4\ucf00\uc640\uc758 \uc5f0\ub77d\uc7a5\uc774\uae30\ub3c4 \ud55c \u300cCOL\u300d\uc740 \uc544\ud3f4\ub85c\uac00\uc774\uc2a4\ud2b8 \ub4f1\uc774 \uc778\uac04 \ud615\ud0dc\ub85c \ub4e4\uc5b4\uc62c \uc815\ub3c4\ub85c, \ud0dc\uc0dd\uc774 GOD\uc5d0 \uc54c\ub824\uc9c0\uace0 \uc788\uc5b4 \uacbd\uc6b0\uc5d0 \ub530\ub77c\uc11c\ub294 \uc804\uc7a5\uc774 \ub418\ub294 \uc77c\ub3c4 \uc788\uc5c8\ub2e4(\uadf9\uc7a5 \uc791\ud488).", "sentence_answer": "\uacbd\uc704\ub098 \ud0dc\uc0dd\uc740 GOD\uc5d0 \uc774\uc804\ubd80\ud130 \uc54c\ub824\uc838 \uc788\uc5b4 GOD \ucd1d\uc0ac\ub839\uad00\uc73c\ub85c\ubd80\ud130\ub3c4 \u300c \uc694\uc8fc\uc758 \uc778\ubb3c \u300d\uc774\ub77c\uace0 \uc5ec\uaca8\uc9c0\uace0 \uc788\uc5c8\ub2e4.", "paragraph_id": "6588881-3-0", "paragraph_question": "question: GOD \ucd1d\uc0ac\ub839\uad00\uc73c\ub85c \ubd80\ud130 \ubb34\uc5c7\uc73c\ub85c \uc5ec\uaca8\uc84c\ub294\uac00?, context: 5\ud654\ubd80\ud130\ud130 \ub4f1\uc7a5. \ub370\uc2a4\ud2b8\ub860 \uad34\uba78 \ud6c4, \ucc3b\uc9d1\uc744 \uacbd\uc601\ud558\uba70 \ud3c9\ud654\ub85c\uc6b4 \ub0a0\ub4e4\uc744 \ubcf4\ub0b4\uace0 \uc788\uc5c8\uc9c0\ub9cc, \uc9c4 \ucf00\uc774\uc2a4\ucf00\uc640 \ub9cc\ub098 \uc0c8\ub85c\uc6b4 \uc2f8\uc6c0\uc5d0 \ucc38\uac00\ud558\uac8c \ub41c\ub2e4. \uacbd\uc704\ub098 \ud0dc\uc0dd\uc740 GOD\uc5d0 \uc774\uc804\ubd80\ud130 \uc54c\ub824\uc838 \uc788\uc5b4 GOD \ucd1d\uc0ac\ub839\uad00\uc73c\ub85c\ubd80\ud130\ub3c4 \u300c\uc694\uc8fc\uc758 \uc778\ubb3c\u300d\uc774\ub77c\uace0 \uc5ec\uaca8\uc9c0\uace0 \uc788\uc5c8\ub2e4. \uc804\uc791 \uc774\uc0c1\uc73c\ub85c \uc804\uc120\uc73c\ub85c \ud5a5\ud558\uace0 \uc788\ub2e4. \ubd80\uce5c\uc744 \uc783\uc740 \ucf00\uc774\uc2a4\ucf00\uc5d0 \uc788\uc5b4\uc11c\ub294 \uc2e0\ub8b0 \ud560 \uc218 \uc788\ub294 \uc874\uc7ac\ub85c, \uc5ed\ub300\uc758 \ub77c\uc774\ub354\ub85c\ubd80\ud130\ub3c4 \uc874\uacbd\ubc1b\uace0 \uc788\ub2e4. \uadf9\uc7a5 \uc791\ud488\uc5d0\uc11c 5\uc778 \ub77c\uc774\ub354\uac00 \ubaa8\uc600\uc744 \ub54c\uc5d0\ub3c4, \ud06c\uac8c \uae30\ubed0\ud558\ub294 \ubaa8\uc2b5\uc774 \uadf8\ub824\uc838 \uc788\ub2e4. \uc9d1\uc774\uc790 \ucf00\uc774\uc2a4\ucf00\uc640\uc758 \uc5f0\ub77d\uc7a5\uc774\uae30\ub3c4 \ud55c \u300cCOL\u300d\uc740 \uc544\ud3f4\ub85c\uac00\uc774\uc2a4\ud2b8 \ub4f1\uc774 \uc778\uac04 \ud615\ud0dc\ub85c \ub4e4\uc5b4\uc62c \uc815\ub3c4\ub85c, \ud0dc\uc0dd\uc774 GOD\uc5d0 \uc54c\ub824\uc9c0\uace0 \uc788\uc5b4 \uacbd\uc6b0\uc5d0 \ub530\ub77c\uc11c\ub294 \uc804\uc7a5\uc774 \ub418\ub294 \uc77c\ub3c4 \uc788\uc5c8\ub2e4(\uadf9\uc7a5 \uc791\ud488)."} {"question": "\ud658\uc790\ub4e4\uc5d0\uac8c\uc11c \uae30\ub85d\ud55c \uac00\uc7a5 \ub192\uc740 \uc218\uc740 \ub808\ubca8\uc740 ?", "paragraph": "1959\ub144 2\uc6d4\uc5d0 \ubbf8\ub098\ub9c8\ud0c0 \ub9cc\uc758 \uc218\uc740 \ubd84\ud3ec\uac00 \uc870\uc0ac\ub418\uc5c8\ub294\ub370 \uadf8 \uacb0\uacfc\ub294 \ucda9\uaca9\uc801\uc774\uc5c8\ub2e4. \ub9ce\uc740 \uc591\uc758 \uc218\uc740\uc774 \ubb3c\uace0\uae30\uc640 \uac11\uac01\ub958 \uadf8\ub9ac\uace0 \ub9cc\uc758 \uc9c4\ud759\uc5d0\uc11c \ubc1c\uacac\ub418\uc5c8\ub2e4. \uce6b\uc18c \uacf5\uc7a5 \ud3d0\uc218\ub3c4\uad00 \uc548\uc5d0\uc11c \uac00\uc7a5 \ub18d\ub3c4\uac00 \ub192\uc558\uace0 \ubc14\ub2e4\ub85c \ub098\uac08\uc218\ub85d \ub18d\ub3c4\uac00 \uac10\uc18c\ud568\uc73c\ub85c\uc368 \uacf5\uc7a5\uc774 \uc624\uc5fc\uc758 \uc6d0\uc778\uc73c\ub85c \uba85\ud655\ud788 \ud655\uc778\ub418\uc5c8\ub2e4. \ud3d0\uc218\ub3c4\uad00 \uc785\uad6c\uc758 \uc624\uc5fc\uc774 \ub108\ubb34 \uc2ec\ud588\uc73c\uba70, \uce68\uc804\ubb3c 1\ud1a4 \ub2f9 \uc218\uc740\uc774 2 kg \uce21\uc815\ub418\uc5c8\ub2e4. \uce6b\uc18c\ub294 \ud6c4\uc5d0 \uc790\ud68c\uc0ac\ub4e4\uc744 \uc2dc\ucf1c \uc218\uc740\uc774 \uac80\ucd9c\ub41c \uc9c4\ud759\uc744 \ub9e4\ub9bd\ud558\uc600\ub2e4. \ubbf8\ub098\ub9c8\ud0c0 \ubcd1\uc758 \ud76c\uc0dd\uc790\ub4e4\uacfc \uc77c\ubc18\uc801\uc778 \ubbf8\ub098\ub9c8\ud0c0 \uc2dc\ubbfc\ub4e4\uc5d0\uac8c \uba38\ub9ac\uce74\ub77d \uc0d8\ud50c\uc744 \ucc44\ucde8\ud588\ub2e4. \ud658\uc790\ub4e4\uc5d0\uac8c\uc11c\ub294 \uac00\uc7a5 \ub192\uc740 \uc218\uc740 \ub808\ubca8\uc774 705ppm\uc744 \uae30\ub85d\ud558\uc600\uace0 \uc99d\uc0c1\uc774 \uc5c6\ub294 \ubbf8\ub098\ub9c8\ud0c0 \uc2dc \uac70\uc8fc\uc790\ub4e4\uc5d0\uac8c\uc11c\ub294 191ppm\uc774 \uae30\ub85d\ub418\uc5c8\ub2e4. \uc774\uac83\uc740 \ubbf8\ub098\ub9c8\ud0c0 \uc2dc \ubc14\uae65\uc5d0 \uc0ac\ub294 \uc0ac\ub78c\ub4e4\uc758 \ud3c9\uade0\uc778 4ppm\uacfc \ub300\uc870\ub41c\ub2e4. 1959\ub144 11\uc6d4 12\uc77c, \ubbf8\ub098\ub9c8\ud0c0 \uc2dd\uc911\ub3c5 \ubd84\uacfc \uc704\uc6d0\ud68c\ub294, \ubbf8\ub098\ub9c8\ud0c0 \ubcd1\uc740 \uc8fc\ub85c \uc911\ucd94\uc2e0\uacbd\uacc4\uc5d0 \uc601\ud5a5\uc744 \uc8fc\ub294 \uc911\ub3c5\uc99d\uc774\uba70, \ubbf8\ub098\ub9c8\ud0c0 \ub9cc\uacfc \uadf8 \uc8fc\uc704\uc5d0 \uc0ac\ub294 \uc5b4\ud328\ub958\ub97c \ub2e4\ub7c9 \uc18c\ube44\ud55c \uac83\uc774 \uc6d0\uc778\uc774\uace0, \uac00\uc7a5 \uc8fc\ub41c \uc6d0\uc778 \ubb3c\uc9c8\uc740 \uc720\uae30 \uc218\uc740 \ud654\ud569\ubb3c\uc774\ub77c\uace0 \ubc1c\ud45c\ud588\ub2e4.", "answer": "705ppm", "sentence": "\ud658\uc790\ub4e4\uc5d0\uac8c\uc11c\ub294 \uac00\uc7a5 \ub192\uc740 \uc218\uc740 \ub808\ubca8\uc774 705ppm \uc744 \uae30\ub85d\ud558\uc600\uace0 \uc99d\uc0c1\uc774 \uc5c6\ub294 \ubbf8\ub098\ub9c8\ud0c0 \uc2dc \uac70\uc8fc\uc790\ub4e4\uc5d0\uac8c\uc11c\ub294 191ppm\uc774 \uae30\ub85d\ub418\uc5c8\ub2e4.", "paragraph_sentence": "1959\ub144 2\uc6d4\uc5d0 \ubbf8\ub098\ub9c8\ud0c0 \ub9cc\uc758 \uc218\uc740 \ubd84\ud3ec\uac00 \uc870\uc0ac\ub418\uc5c8\ub294\ub370 \uadf8 \uacb0\uacfc\ub294 \ucda9\uaca9\uc801\uc774\uc5c8\ub2e4. \ub9ce\uc740 \uc591\uc758 \uc218\uc740\uc774 \ubb3c\uace0\uae30\uc640 \uac11\uac01\ub958 \uadf8\ub9ac\uace0 \ub9cc\uc758 \uc9c4\ud759\uc5d0\uc11c \ubc1c\uacac\ub418\uc5c8\ub2e4. \uce6b\uc18c \uacf5\uc7a5 \ud3d0\uc218\ub3c4\uad00 \uc548\uc5d0\uc11c \uac00\uc7a5 \ub18d\ub3c4\uac00 \ub192\uc558\uace0 \ubc14\ub2e4\ub85c \ub098\uac08\uc218\ub85d \ub18d\ub3c4\uac00 \uac10\uc18c\ud568\uc73c\ub85c\uc368 \uacf5\uc7a5\uc774 \uc624\uc5fc\uc758 \uc6d0\uc778\uc73c\ub85c \uba85\ud655\ud788 \ud655\uc778\ub418\uc5c8\ub2e4. \ud3d0\uc218\ub3c4\uad00 \uc785\uad6c\uc758 \uc624\uc5fc\uc774 \ub108\ubb34 \uc2ec\ud588\uc73c\uba70, \uce68\uc804\ubb3c 1\ud1a4 \ub2f9 \uc218\uc740\uc774 2 kg \uce21\uc815\ub418\uc5c8\ub2e4. \uce6b\uc18c\ub294 \ud6c4\uc5d0 \uc790\ud68c\uc0ac\ub4e4\uc744 \uc2dc\ucf1c \uc218\uc740\uc774 \uac80\ucd9c\ub41c \uc9c4\ud759\uc744 \ub9e4\ub9bd\ud558\uc600\ub2e4. \ubbf8\ub098\ub9c8\ud0c0 \ubcd1\uc758 \ud76c\uc0dd\uc790\ub4e4\uacfc \uc77c\ubc18\uc801\uc778 \ubbf8\ub098\ub9c8\ud0c0 \uc2dc\ubbfc\ub4e4\uc5d0\uac8c \uba38\ub9ac\uce74\ub77d \uc0d8\ud50c\uc744 \ucc44\ucde8\ud588\ub2e4. \ud658\uc790\ub4e4\uc5d0\uac8c\uc11c\ub294 \uac00\uc7a5 \ub192\uc740 \uc218\uc740 \ub808\ubca8\uc774 705ppm \uc744 \uae30\ub85d\ud558\uc600\uace0 \uc99d\uc0c1\uc774 \uc5c6\ub294 \ubbf8\ub098\ub9c8\ud0c0 \uc2dc \uac70\uc8fc\uc790\ub4e4\uc5d0\uac8c\uc11c\ub294 191ppm\uc774 \uae30\ub85d\ub418\uc5c8\ub2e4. \uc774\uac83\uc740 \ubbf8\ub098\ub9c8\ud0c0 \uc2dc \ubc14\uae65\uc5d0 \uc0ac\ub294 \uc0ac\ub78c\ub4e4\uc758 \ud3c9\uade0\uc778 4ppm\uacfc \ub300\uc870\ub41c\ub2e4. 1959\ub144 11\uc6d4 12\uc77c, \ubbf8\ub098\ub9c8\ud0c0 \uc2dd\uc911\ub3c5 \ubd84\uacfc \uc704\uc6d0\ud68c\ub294, \ubbf8\ub098\ub9c8\ud0c0 \ubcd1\uc740 \uc8fc\ub85c \uc911\ucd94\uc2e0\uacbd\uacc4\uc5d0 \uc601\ud5a5\uc744 \uc8fc\ub294 \uc911\ub3c5\uc99d\uc774\uba70, \ubbf8\ub098\ub9c8\ud0c0 \ub9cc\uacfc \uadf8 \uc8fc\uc704\uc5d0 \uc0ac\ub294 \uc5b4\ud328\ub958\ub97c \ub2e4\ub7c9 \uc18c\ube44\ud55c \uac83\uc774 \uc6d0\uc778\uc774\uace0, \uac00\uc7a5 \uc8fc\ub41c \uc6d0\uc778 \ubb3c\uc9c8\uc740 \uc720\uae30 \uc218\uc740 \ud654\ud569\ubb3c\uc774\ub77c\uace0 \ubc1c\ud45c\ud588\ub2e4.", "paragraph_answer": "1959\ub144 2\uc6d4\uc5d0 \ubbf8\ub098\ub9c8\ud0c0 \ub9cc\uc758 \uc218\uc740 \ubd84\ud3ec\uac00 \uc870\uc0ac\ub418\uc5c8\ub294\ub370 \uadf8 \uacb0\uacfc\ub294 \ucda9\uaca9\uc801\uc774\uc5c8\ub2e4. \ub9ce\uc740 \uc591\uc758 \uc218\uc740\uc774 \ubb3c\uace0\uae30\uc640 \uac11\uac01\ub958 \uadf8\ub9ac\uace0 \ub9cc\uc758 \uc9c4\ud759\uc5d0\uc11c \ubc1c\uacac\ub418\uc5c8\ub2e4. \uce6b\uc18c \uacf5\uc7a5 \ud3d0\uc218\ub3c4\uad00 \uc548\uc5d0\uc11c \uac00\uc7a5 \ub18d\ub3c4\uac00 \ub192\uc558\uace0 \ubc14\ub2e4\ub85c \ub098\uac08\uc218\ub85d \ub18d\ub3c4\uac00 \uac10\uc18c\ud568\uc73c\ub85c\uc368 \uacf5\uc7a5\uc774 \uc624\uc5fc\uc758 \uc6d0\uc778\uc73c\ub85c \uba85\ud655\ud788 \ud655\uc778\ub418\uc5c8\ub2e4. \ud3d0\uc218\ub3c4\uad00 \uc785\uad6c\uc758 \uc624\uc5fc\uc774 \ub108\ubb34 \uc2ec\ud588\uc73c\uba70, \uce68\uc804\ubb3c 1\ud1a4 \ub2f9 \uc218\uc740\uc774 2 kg \uce21\uc815\ub418\uc5c8\ub2e4. \uce6b\uc18c\ub294 \ud6c4\uc5d0 \uc790\ud68c\uc0ac\ub4e4\uc744 \uc2dc\ucf1c \uc218\uc740\uc774 \uac80\ucd9c\ub41c \uc9c4\ud759\uc744 \ub9e4\ub9bd\ud558\uc600\ub2e4. \ubbf8\ub098\ub9c8\ud0c0 \ubcd1\uc758 \ud76c\uc0dd\uc790\ub4e4\uacfc \uc77c\ubc18\uc801\uc778 \ubbf8\ub098\ub9c8\ud0c0 \uc2dc\ubbfc\ub4e4\uc5d0\uac8c \uba38\ub9ac\uce74\ub77d \uc0d8\ud50c\uc744 \ucc44\ucde8\ud588\ub2e4. \ud658\uc790\ub4e4\uc5d0\uac8c\uc11c\ub294 \uac00\uc7a5 \ub192\uc740 \uc218\uc740 \ub808\ubca8\uc774 705ppm \uc744 \uae30\ub85d\ud558\uc600\uace0 \uc99d\uc0c1\uc774 \uc5c6\ub294 \ubbf8\ub098\ub9c8\ud0c0 \uc2dc \uac70\uc8fc\uc790\ub4e4\uc5d0\uac8c\uc11c\ub294 191ppm\uc774 \uae30\ub85d\ub418\uc5c8\ub2e4. \uc774\uac83\uc740 \ubbf8\ub098\ub9c8\ud0c0 \uc2dc \ubc14\uae65\uc5d0 \uc0ac\ub294 \uc0ac\ub78c\ub4e4\uc758 \ud3c9\uade0\uc778 4ppm\uacfc \ub300\uc870\ub41c\ub2e4. 1959\ub144 11\uc6d4 12\uc77c, \ubbf8\ub098\ub9c8\ud0c0 \uc2dd\uc911\ub3c5 \ubd84\uacfc \uc704\uc6d0\ud68c\ub294, \ubbf8\ub098\ub9c8\ud0c0 \ubcd1\uc740 \uc8fc\ub85c \uc911\ucd94\uc2e0\uacbd\uacc4\uc5d0 \uc601\ud5a5\uc744 \uc8fc\ub294 \uc911\ub3c5\uc99d\uc774\uba70, \ubbf8\ub098\ub9c8\ud0c0 \ub9cc\uacfc \uadf8 \uc8fc\uc704\uc5d0 \uc0ac\ub294 \uc5b4\ud328\ub958\ub97c \ub2e4\ub7c9 \uc18c\ube44\ud55c \uac83\uc774 \uc6d0\uc778\uc774\uace0, \uac00\uc7a5 \uc8fc\ub41c \uc6d0\uc778 \ubb3c\uc9c8\uc740 \uc720\uae30 \uc218\uc740 \ud654\ud569\ubb3c\uc774\ub77c\uace0 \ubc1c\ud45c\ud588\ub2e4.", "sentence_answer": "\ud658\uc790\ub4e4\uc5d0\uac8c\uc11c\ub294 \uac00\uc7a5 \ub192\uc740 \uc218\uc740 \ub808\ubca8\uc774 705ppm \uc744 \uae30\ub85d\ud558\uc600\uace0 \uc99d\uc0c1\uc774 \uc5c6\ub294 \ubbf8\ub098\ub9c8\ud0c0 \uc2dc \uac70\uc8fc\uc790\ub4e4\uc5d0\uac8c\uc11c\ub294 191ppm\uc774 \uae30\ub85d\ub418\uc5c8\ub2e4.", "paragraph_id": "6546972-6-1", "paragraph_question": "question: \ud658\uc790\ub4e4\uc5d0\uac8c\uc11c \uae30\ub85d\ud55c \uac00\uc7a5 \ub192\uc740 \uc218\uc740 \ub808\ubca8\uc740 ?, context: 1959\ub144 2\uc6d4\uc5d0 \ubbf8\ub098\ub9c8\ud0c0 \ub9cc\uc758 \uc218\uc740 \ubd84\ud3ec\uac00 \uc870\uc0ac\ub418\uc5c8\ub294\ub370 \uadf8 \uacb0\uacfc\ub294 \ucda9\uaca9\uc801\uc774\uc5c8\ub2e4. \ub9ce\uc740 \uc591\uc758 \uc218\uc740\uc774 \ubb3c\uace0\uae30\uc640 \uac11\uac01\ub958 \uadf8\ub9ac\uace0 \ub9cc\uc758 \uc9c4\ud759\uc5d0\uc11c \ubc1c\uacac\ub418\uc5c8\ub2e4. \uce6b\uc18c \uacf5\uc7a5 \ud3d0\uc218\ub3c4\uad00 \uc548\uc5d0\uc11c \uac00\uc7a5 \ub18d\ub3c4\uac00 \ub192\uc558\uace0 \ubc14\ub2e4\ub85c \ub098\uac08\uc218\ub85d \ub18d\ub3c4\uac00 \uac10\uc18c\ud568\uc73c\ub85c\uc368 \uacf5\uc7a5\uc774 \uc624\uc5fc\uc758 \uc6d0\uc778\uc73c\ub85c \uba85\ud655\ud788 \ud655\uc778\ub418\uc5c8\ub2e4. \ud3d0\uc218\ub3c4\uad00 \uc785\uad6c\uc758 \uc624\uc5fc\uc774 \ub108\ubb34 \uc2ec\ud588\uc73c\uba70, \uce68\uc804\ubb3c 1\ud1a4 \ub2f9 \uc218\uc740\uc774 2 kg \uce21\uc815\ub418\uc5c8\ub2e4. \uce6b\uc18c\ub294 \ud6c4\uc5d0 \uc790\ud68c\uc0ac\ub4e4\uc744 \uc2dc\ucf1c \uc218\uc740\uc774 \uac80\ucd9c\ub41c \uc9c4\ud759\uc744 \ub9e4\ub9bd\ud558\uc600\ub2e4. \ubbf8\ub098\ub9c8\ud0c0 \ubcd1\uc758 \ud76c\uc0dd\uc790\ub4e4\uacfc \uc77c\ubc18\uc801\uc778 \ubbf8\ub098\ub9c8\ud0c0 \uc2dc\ubbfc\ub4e4\uc5d0\uac8c \uba38\ub9ac\uce74\ub77d \uc0d8\ud50c\uc744 \ucc44\ucde8\ud588\ub2e4. \ud658\uc790\ub4e4\uc5d0\uac8c\uc11c\ub294 \uac00\uc7a5 \ub192\uc740 \uc218\uc740 \ub808\ubca8\uc774 705ppm\uc744 \uae30\ub85d\ud558\uc600\uace0 \uc99d\uc0c1\uc774 \uc5c6\ub294 \ubbf8\ub098\ub9c8\ud0c0 \uc2dc \uac70\uc8fc\uc790\ub4e4\uc5d0\uac8c\uc11c\ub294 191ppm\uc774 \uae30\ub85d\ub418\uc5c8\ub2e4. \uc774\uac83\uc740 \ubbf8\ub098\ub9c8\ud0c0 \uc2dc \ubc14\uae65\uc5d0 \uc0ac\ub294 \uc0ac\ub78c\ub4e4\uc758 \ud3c9\uade0\uc778 4ppm\uacfc \ub300\uc870\ub41c\ub2e4. 1959\ub144 11\uc6d4 12\uc77c, \ubbf8\ub098\ub9c8\ud0c0 \uc2dd\uc911\ub3c5 \ubd84\uacfc \uc704\uc6d0\ud68c\ub294, \ubbf8\ub098\ub9c8\ud0c0 \ubcd1\uc740 \uc8fc\ub85c \uc911\ucd94\uc2e0\uacbd\uacc4\uc5d0 \uc601\ud5a5\uc744 \uc8fc\ub294 \uc911\ub3c5\uc99d\uc774\uba70, \ubbf8\ub098\ub9c8\ud0c0 \ub9cc\uacfc \uadf8 \uc8fc\uc704\uc5d0 \uc0ac\ub294 \uc5b4\ud328\ub958\ub97c \ub2e4\ub7c9 \uc18c\ube44\ud55c \uac83\uc774 \uc6d0\uc778\uc774\uace0, \uac00\uc7a5 \uc8fc\ub41c \uc6d0\uc778 \ubb3c\uc9c8\uc740 \uc720\uae30 \uc218\uc740 \ud654\ud569\ubb3c\uc774\ub77c\uace0 \ubc1c\ud45c\ud588\ub2e4."} {"question": "1923\ub144 \uc789\uae00\ub79c\ub4dc FA\ucef5 \uacb0\uc2b9\uc804\uc774 \uc8fc\ub85c \ubd88\ub9ac\ub294 \ubcc4\uce6d\uc740?", "paragraph": "\ud558\uc580 \ub9d0 \ube4c\ub9ac\ub294 \uc601\uad6d \ucd95\uad6c\uc758 \uc5ed\uc0ac \uc18d \uc720\uba85\ud55c \uc7a5\uba74 \uc911 \ud558\ub098\ub85c \ub0a8\uc544\uc788\uc5b4\uc11c \uc774 \uacbd\uae30\ub294 \uc8fc\ub85c \"\ubc31\ub9c8 \uacb0\uc2b9\uc804\"(\"The White Horse Final\")\ub85c \ubd88\ub9b0\ub2e4. 2005\ub144\uc5d0 \uc6f8\ube14\ub9ac \uacbd\uae30\uc7a5 \uadfc\ucc98\uc5d0 \uc9c0\uc5b4\uc9c8 \uc0c8\ub85c\uc6b4 \ub2e4\ub9ac \uc774\ub984\uc744 \ud32c\ub4e4\uc758 \ud22c\ud45c\ub85c \ubf51\uc558\ub294\ub370, \uadf8 \uc911\uc5d0\uc11c \ubc31\ub9c8\uad50(White Horse Bridge)\ub85c \ud558\uc790\ub294 \uc758\uacac\uc774 \uac00\uc7a5 \ub9ce\uc558\ub2e4. \uc774 \ud22c\ud45c\ub97c \uc8fc\uad00\ud588\ub358 \ub7f0\ub358 \uac1c\ubc1c\uccad\uc758 \uc0c1\uc784 \uc774\uc0ac\ub294 \uc774 \ud22c\ud45c \uacb0\uacfc\ub85c \ubf51\ud78c \uc774\ub984\uc774 \ub9d0\ucc98\ub7fc \uc6f8\ube14\ub9ac \uc548\uc758 \uc0ac\ub78c\ub4e4\uc5d0\uac8c \uc548\uc804\uc744 \uc81c\uacf5\ud558\ub77c\ub294 \ub73b\uc73c\ub85c\uc11c \ub2e4\ub9ac\uc758 \uc774\ub984\uc73c\ub85c \uc801\ud569\ud558\ub2e4\uace0 \ubc1c\ud45c\ud588\ub2e4. \uc2a4\ucf54\ud2c0\ub79c\ub4dc \uc2e0\ubb38\uc778 \u300a\ub370\uc77c\ub9ac \ub808\ucf54\ub4dc\u300b\ub294 \uc2a4\ucf54\ud2c0\ub79c\ub4dc\uc758 \uad6d\uac00\ub300\ud45c\ud300\uc758 \ud32c\ub4e4\uc774 \"\ubc31\ub9c8\"(White Horse)\ub97c \uc9c0\uc9c0\ud588\ub294\ub370 \uadf8 \uc774\uc720\ub294 \uc804\uc9c1 \uc789\uae00\ub79c\ub4dc \ucd95\uad6c \uc120\uc218\ub098 \uac10\ub3c5\uc758 \uc774\ub984\uc774 \ubd99\uc5ec\uc838 \uc601\uad11\uc744 \uc548\uac8c \ub418\ub294 \uac83\uc744 \ub9c9\uc73c\ub824\uace0 \ud588\uae30 \ub54c\ubb38\uc774\uace0, \ub610\ud55c \"White Horse\"\ub77c\ub294 \uc774\ub984\uc774 \uc704\uc2a4\ud0a4 \ube0c\ub79c\ub4dc \uc774\ub984\uc774\uc5c8\ub2e4\ub294 \uc810\ub3c4 \uadf8 \uc774\ub984\uc744 \uc9c0\uc9c0\ud55c \uc774\uc720\uc600\ub2e4\uace0 \ubc1d\ud614\ub2e4. \ube4c\ub9ac\uc758 \uc8fc\uc778\uc778 \uc870\uc9c0 \uc2a4\ucf54\ub9ac\ub294 \uc789\uae00\ub79c\ub4dc \ucd95\uad6c\ud611\ud68c\ub85c\ubd80\ud130 \uc774\ud6c4\uc758 \ubaa8\ub4e0 \uacb0\uc2b9\uc804 \uacbd\uae30\ub97c \uad00\ub78c\ud560 \uc218 \uc788\ub294 \ubb34\ub8cc \ud2f0\ucf13\uc744 \ubc1b\uc558\uc9c0\ub9cc \ucd95\uad6c\uc5d0 \uad00\uc2ec\uc774 \uc5c6\uc5b4\uc11c \ud55c \ucc28\ub840\ub3c4 \uacbd\uae30\uc7a5\uc744 \ucc3e\uc9c0 \uc54a\uc558\ub2e4. 2007\ub144\uc5d0\ub294 \uacb0\uc2b9\uc804\uc5d0\uc11c \ubc8c\uc5b4\uc9c4 \uc77c\ub4e4\uc5d0 \ub300\ud55c \ubcfc\ud134 \uc2dc\ubbfc\ub4e4\uc758 \ubc18\uc751\ub4e4\uc744 \uc5f0\uae30\ub85c \ub2f4\uc740 \uc5f0\uadf9\uc774 \ubcfc\ud134\uc758 \uc625\ud0c0\uace4 \uadf9\uc7a5\uc5d0\uc11c \uc0c1\uc601\ub418\uc5c8\ub2e4.", "answer": "\"\ubc31\ub9c8 \uacb0\uc2b9\uc804\"(\"The White Horse Final\")", "sentence": "\ud558\uc580 \ub9d0 \ube4c\ub9ac\ub294 \uc601\uad6d \ucd95\uad6c\uc758 \uc5ed\uc0ac \uc18d \uc720\uba85\ud55c \uc7a5\uba74 \uc911 \ud558\ub098\ub85c \ub0a8\uc544\uc788\uc5b4\uc11c \uc774 \uacbd\uae30\ub294 \uc8fc\ub85c \"\ubc31\ub9c8 \uacb0\uc2b9\uc804\"(\"The White Horse Final\") \ub85c \ubd88\ub9b0\ub2e4.", "paragraph_sentence": " \ud558\uc580 \ub9d0 \ube4c\ub9ac\ub294 \uc601\uad6d \ucd95\uad6c\uc758 \uc5ed\uc0ac \uc18d \uc720\uba85\ud55c \uc7a5\uba74 \uc911 \ud558\ub098\ub85c \ub0a8\uc544\uc788\uc5b4\uc11c \uc774 \uacbd\uae30\ub294 \uc8fc\ub85c \"\ubc31\ub9c8 \uacb0\uc2b9\uc804\"(\"The White Horse Final\") \ub85c \ubd88\ub9b0\ub2e4. 2005\ub144\uc5d0 \uc6f8\ube14\ub9ac \uacbd\uae30\uc7a5 \uadfc\ucc98\uc5d0 \uc9c0\uc5b4\uc9c8 \uc0c8\ub85c\uc6b4 \ub2e4\ub9ac \uc774\ub984\uc744 \ud32c\ub4e4\uc758 \ud22c\ud45c\ub85c \ubf51\uc558\ub294\ub370, \uadf8 \uc911\uc5d0\uc11c \ubc31\ub9c8\uad50(White Horse Bridge)\ub85c \ud558\uc790\ub294 \uc758\uacac\uc774 \uac00\uc7a5 \ub9ce\uc558\ub2e4. \uc774 \ud22c\ud45c\ub97c \uc8fc\uad00\ud588\ub358 \ub7f0\ub358 \uac1c\ubc1c\uccad\uc758 \uc0c1\uc784 \uc774\uc0ac\ub294 \uc774 \ud22c\ud45c \uacb0\uacfc\ub85c \ubf51\ud78c \uc774\ub984\uc774 \ub9d0\ucc98\ub7fc \uc6f8\ube14\ub9ac \uc548\uc758 \uc0ac\ub78c\ub4e4\uc5d0\uac8c \uc548\uc804\uc744 \uc81c\uacf5\ud558\ub77c\ub294 \ub73b\uc73c\ub85c\uc11c \ub2e4\ub9ac\uc758 \uc774\ub984\uc73c\ub85c \uc801\ud569\ud558\ub2e4\uace0 \ubc1c\ud45c\ud588\ub2e4. \uc2a4\ucf54\ud2c0\ub79c\ub4dc \uc2e0\ubb38\uc778 \u300a\ub370\uc77c\ub9ac \ub808\ucf54\ub4dc\u300b\ub294 \uc2a4\ucf54\ud2c0\ub79c\ub4dc\uc758 \uad6d\uac00\ub300\ud45c\ud300\uc758 \ud32c\ub4e4\uc774 \"\ubc31\ub9c8\"(White Horse)\ub97c \uc9c0\uc9c0\ud588\ub294\ub370 \uadf8 \uc774\uc720\ub294 \uc804\uc9c1 \uc789\uae00\ub79c\ub4dc \ucd95\uad6c \uc120\uc218\ub098 \uac10\ub3c5\uc758 \uc774\ub984\uc774 \ubd99\uc5ec\uc838 \uc601\uad11\uc744 \uc548\uac8c \ub418\ub294 \uac83\uc744 \ub9c9\uc73c\ub824\uace0 \ud588\uae30 \ub54c\ubb38\uc774\uace0, \ub610\ud55c \"White Horse\"\ub77c\ub294 \uc774\ub984\uc774 \uc704\uc2a4\ud0a4 \ube0c\ub79c\ub4dc \uc774\ub984\uc774\uc5c8\ub2e4\ub294 \uc810\ub3c4 \uadf8 \uc774\ub984\uc744 \uc9c0\uc9c0\ud55c \uc774\uc720\uc600\ub2e4\uace0 \ubc1d\ud614\ub2e4. \ube4c\ub9ac\uc758 \uc8fc\uc778\uc778 \uc870\uc9c0 \uc2a4\ucf54\ub9ac\ub294 \uc789\uae00\ub79c\ub4dc \ucd95\uad6c\ud611\ud68c\ub85c\ubd80\ud130 \uc774\ud6c4\uc758 \ubaa8\ub4e0 \uacb0\uc2b9\uc804 \uacbd\uae30\ub97c \uad00\ub78c\ud560 \uc218 \uc788\ub294 \ubb34\ub8cc \ud2f0\ucf13\uc744 \ubc1b\uc558\uc9c0\ub9cc \ucd95\uad6c\uc5d0 \uad00\uc2ec\uc774 \uc5c6\uc5b4\uc11c \ud55c \ucc28\ub840\ub3c4 \uacbd\uae30\uc7a5\uc744 \ucc3e\uc9c0 \uc54a\uc558\ub2e4. 2007\ub144\uc5d0\ub294 \uacb0\uc2b9\uc804\uc5d0\uc11c \ubc8c\uc5b4\uc9c4 \uc77c\ub4e4\uc5d0 \ub300\ud55c \ubcfc\ud134 \uc2dc\ubbfc\ub4e4\uc758 \ubc18\uc751\ub4e4\uc744 \uc5f0\uae30\ub85c \ub2f4\uc740 \uc5f0\uadf9\uc774 \ubcfc\ud134\uc758 \uc625\ud0c0\uace4 \uadf9\uc7a5\uc5d0\uc11c \uc0c1\uc601\ub418\uc5c8\ub2e4.", "paragraph_answer": "\ud558\uc580 \ub9d0 \ube4c\ub9ac\ub294 \uc601\uad6d \ucd95\uad6c\uc758 \uc5ed\uc0ac \uc18d \uc720\uba85\ud55c \uc7a5\uba74 \uc911 \ud558\ub098\ub85c \ub0a8\uc544\uc788\uc5b4\uc11c \uc774 \uacbd\uae30\ub294 \uc8fc\ub85c \"\ubc31\ub9c8 \uacb0\uc2b9\uc804\"(\"The White Horse Final\") \ub85c \ubd88\ub9b0\ub2e4. 2005\ub144\uc5d0 \uc6f8\ube14\ub9ac \uacbd\uae30\uc7a5 \uadfc\ucc98\uc5d0 \uc9c0\uc5b4\uc9c8 \uc0c8\ub85c\uc6b4 \ub2e4\ub9ac \uc774\ub984\uc744 \ud32c\ub4e4\uc758 \ud22c\ud45c\ub85c \ubf51\uc558\ub294\ub370, \uadf8 \uc911\uc5d0\uc11c \ubc31\ub9c8\uad50(White Horse Bridge)\ub85c \ud558\uc790\ub294 \uc758\uacac\uc774 \uac00\uc7a5 \ub9ce\uc558\ub2e4. \uc774 \ud22c\ud45c\ub97c \uc8fc\uad00\ud588\ub358 \ub7f0\ub358 \uac1c\ubc1c\uccad\uc758 \uc0c1\uc784 \uc774\uc0ac\ub294 \uc774 \ud22c\ud45c \uacb0\uacfc\ub85c \ubf51\ud78c \uc774\ub984\uc774 \ub9d0\ucc98\ub7fc \uc6f8\ube14\ub9ac \uc548\uc758 \uc0ac\ub78c\ub4e4\uc5d0\uac8c \uc548\uc804\uc744 \uc81c\uacf5\ud558\ub77c\ub294 \ub73b\uc73c\ub85c\uc11c \ub2e4\ub9ac\uc758 \uc774\ub984\uc73c\ub85c \uc801\ud569\ud558\ub2e4\uace0 \ubc1c\ud45c\ud588\ub2e4. \uc2a4\ucf54\ud2c0\ub79c\ub4dc \uc2e0\ubb38\uc778 \u300a\ub370\uc77c\ub9ac \ub808\ucf54\ub4dc\u300b\ub294 \uc2a4\ucf54\ud2c0\ub79c\ub4dc\uc758 \uad6d\uac00\ub300\ud45c\ud300\uc758 \ud32c\ub4e4\uc774 \"\ubc31\ub9c8\"(White Horse)\ub97c \uc9c0\uc9c0\ud588\ub294\ub370 \uadf8 \uc774\uc720\ub294 \uc804\uc9c1 \uc789\uae00\ub79c\ub4dc \ucd95\uad6c \uc120\uc218\ub098 \uac10\ub3c5\uc758 \uc774\ub984\uc774 \ubd99\uc5ec\uc838 \uc601\uad11\uc744 \uc548\uac8c \ub418\ub294 \uac83\uc744 \ub9c9\uc73c\ub824\uace0 \ud588\uae30 \ub54c\ubb38\uc774\uace0, \ub610\ud55c \"White Horse\"\ub77c\ub294 \uc774\ub984\uc774 \uc704\uc2a4\ud0a4 \ube0c\ub79c\ub4dc \uc774\ub984\uc774\uc5c8\ub2e4\ub294 \uc810\ub3c4 \uadf8 \uc774\ub984\uc744 \uc9c0\uc9c0\ud55c \uc774\uc720\uc600\ub2e4\uace0 \ubc1d\ud614\ub2e4. \ube4c\ub9ac\uc758 \uc8fc\uc778\uc778 \uc870\uc9c0 \uc2a4\ucf54\ub9ac\ub294 \uc789\uae00\ub79c\ub4dc \ucd95\uad6c\ud611\ud68c\ub85c\ubd80\ud130 \uc774\ud6c4\uc758 \ubaa8\ub4e0 \uacb0\uc2b9\uc804 \uacbd\uae30\ub97c \uad00\ub78c\ud560 \uc218 \uc788\ub294 \ubb34\ub8cc \ud2f0\ucf13\uc744 \ubc1b\uc558\uc9c0\ub9cc \ucd95\uad6c\uc5d0 \uad00\uc2ec\uc774 \uc5c6\uc5b4\uc11c \ud55c \ucc28\ub840\ub3c4 \uacbd\uae30\uc7a5\uc744 \ucc3e\uc9c0 \uc54a\uc558\ub2e4. 2007\ub144\uc5d0\ub294 \uacb0\uc2b9\uc804\uc5d0\uc11c \ubc8c\uc5b4\uc9c4 \uc77c\ub4e4\uc5d0 \ub300\ud55c \ubcfc\ud134 \uc2dc\ubbfc\ub4e4\uc758 \ubc18\uc751\ub4e4\uc744 \uc5f0\uae30\ub85c \ub2f4\uc740 \uc5f0\uadf9\uc774 \ubcfc\ud134\uc758 \uc625\ud0c0\uace4 \uadf9\uc7a5\uc5d0\uc11c \uc0c1\uc601\ub418\uc5c8\ub2e4.", "sentence_answer": "\ud558\uc580 \ub9d0 \ube4c\ub9ac\ub294 \uc601\uad6d \ucd95\uad6c\uc758 \uc5ed\uc0ac \uc18d \uc720\uba85\ud55c \uc7a5\uba74 \uc911 \ud558\ub098\ub85c \ub0a8\uc544\uc788\uc5b4\uc11c \uc774 \uacbd\uae30\ub294 \uc8fc\ub85c \"\ubc31\ub9c8 \uacb0\uc2b9\uc804\"(\"The White Horse Final\") \ub85c \ubd88\ub9b0\ub2e4.", "paragraph_id": "5782571-10-0", "paragraph_question": "question: 1923\ub144 \uc789\uae00\ub79c\ub4dc FA\ucef5 \uacb0\uc2b9\uc804\uc774 \uc8fc\ub85c \ubd88\ub9ac\ub294 \ubcc4\uce6d\uc740?, context: \ud558\uc580 \ub9d0 \ube4c\ub9ac\ub294 \uc601\uad6d \ucd95\uad6c\uc758 \uc5ed\uc0ac \uc18d \uc720\uba85\ud55c \uc7a5\uba74 \uc911 \ud558\ub098\ub85c \ub0a8\uc544\uc788\uc5b4\uc11c \uc774 \uacbd\uae30\ub294 \uc8fc\ub85c \"\ubc31\ub9c8 \uacb0\uc2b9\uc804\"(\"The White Horse Final\")\ub85c \ubd88\ub9b0\ub2e4. 2005\ub144\uc5d0 \uc6f8\ube14\ub9ac \uacbd\uae30\uc7a5 \uadfc\ucc98\uc5d0 \uc9c0\uc5b4\uc9c8 \uc0c8\ub85c\uc6b4 \ub2e4\ub9ac \uc774\ub984\uc744 \ud32c\ub4e4\uc758 \ud22c\ud45c\ub85c \ubf51\uc558\ub294\ub370, \uadf8 \uc911\uc5d0\uc11c \ubc31\ub9c8\uad50(White Horse Bridge)\ub85c \ud558\uc790\ub294 \uc758\uacac\uc774 \uac00\uc7a5 \ub9ce\uc558\ub2e4. \uc774 \ud22c\ud45c\ub97c \uc8fc\uad00\ud588\ub358 \ub7f0\ub358 \uac1c\ubc1c\uccad\uc758 \uc0c1\uc784 \uc774\uc0ac\ub294 \uc774 \ud22c\ud45c \uacb0\uacfc\ub85c \ubf51\ud78c \uc774\ub984\uc774 \ub9d0\ucc98\ub7fc \uc6f8\ube14\ub9ac \uc548\uc758 \uc0ac\ub78c\ub4e4\uc5d0\uac8c \uc548\uc804\uc744 \uc81c\uacf5\ud558\ub77c\ub294 \ub73b\uc73c\ub85c\uc11c \ub2e4\ub9ac\uc758 \uc774\ub984\uc73c\ub85c \uc801\ud569\ud558\ub2e4\uace0 \ubc1c\ud45c\ud588\ub2e4. \uc2a4\ucf54\ud2c0\ub79c\ub4dc \uc2e0\ubb38\uc778 \u300a\ub370\uc77c\ub9ac \ub808\ucf54\ub4dc\u300b\ub294 \uc2a4\ucf54\ud2c0\ub79c\ub4dc\uc758 \uad6d\uac00\ub300\ud45c\ud300\uc758 \ud32c\ub4e4\uc774 \"\ubc31\ub9c8\"(White Horse)\ub97c \uc9c0\uc9c0\ud588\ub294\ub370 \uadf8 \uc774\uc720\ub294 \uc804\uc9c1 \uc789\uae00\ub79c\ub4dc \ucd95\uad6c \uc120\uc218\ub098 \uac10\ub3c5\uc758 \uc774\ub984\uc774 \ubd99\uc5ec\uc838 \uc601\uad11\uc744 \uc548\uac8c \ub418\ub294 \uac83\uc744 \ub9c9\uc73c\ub824\uace0 \ud588\uae30 \ub54c\ubb38\uc774\uace0, \ub610\ud55c \"White Horse\"\ub77c\ub294 \uc774\ub984\uc774 \uc704\uc2a4\ud0a4 \ube0c\ub79c\ub4dc \uc774\ub984\uc774\uc5c8\ub2e4\ub294 \uc810\ub3c4 \uadf8 \uc774\ub984\uc744 \uc9c0\uc9c0\ud55c \uc774\uc720\uc600\ub2e4\uace0 \ubc1d\ud614\ub2e4. \ube4c\ub9ac\uc758 \uc8fc\uc778\uc778 \uc870\uc9c0 \uc2a4\ucf54\ub9ac\ub294 \uc789\uae00\ub79c\ub4dc \ucd95\uad6c\ud611\ud68c\ub85c\ubd80\ud130 \uc774\ud6c4\uc758 \ubaa8\ub4e0 \uacb0\uc2b9\uc804 \uacbd\uae30\ub97c \uad00\ub78c\ud560 \uc218 \uc788\ub294 \ubb34\ub8cc \ud2f0\ucf13\uc744 \ubc1b\uc558\uc9c0\ub9cc \ucd95\uad6c\uc5d0 \uad00\uc2ec\uc774 \uc5c6\uc5b4\uc11c \ud55c \ucc28\ub840\ub3c4 \uacbd\uae30\uc7a5\uc744 \ucc3e\uc9c0 \uc54a\uc558\ub2e4. 2007\ub144\uc5d0\ub294 \uacb0\uc2b9\uc804\uc5d0\uc11c \ubc8c\uc5b4\uc9c4 \uc77c\ub4e4\uc5d0 \ub300\ud55c \ubcfc\ud134 \uc2dc\ubbfc\ub4e4\uc758 \ubc18\uc751\ub4e4\uc744 \uc5f0\uae30\ub85c \ub2f4\uc740 \uc5f0\uadf9\uc774 \ubcfc\ud134\uc758 \uc625\ud0c0\uace4 \uadf9\uc7a5\uc5d0\uc11c \uc0c1\uc601\ub418\uc5c8\ub2e4."} {"question": "1812\ub144 \uc804\uc7c1\uc758 \ubc1c\ubc1c\ub85c \ubaa8\ub85c\uc5d0\uac8c \ubbf8\uad6d \uad70\ub300\uc758 \uc0ac\ub839\uad00\uc9c1\uc744 \uc81c\uc758\ud55c \uc0ac\ub78c\uc740 \ub204\uad6c\uc778\uac00?", "paragraph": "\ubaa8\ub85c\ub294 \uc544\ub0b4\uc640 \ud568\uaed8 12\uc6d4 \ub274\uc695\uc2dc\uc5d0 \ub3c4\ucc29\ud588\ub2e4. \ubaa8\ub85c\ub294 \uc774 \uc9c0\uc5ed\uc5d0\uc11c \uc5f4\uc815\uc744 \uac00\uc9c0\uace0 \uc77c\uc744 \uc5bb\uc73c\ub824\uace0 \ud588\uc73c\ub098 \ubaa8\ub4e0 \uc9c1\uc7a5\uc5d0\uc11c \uadfc\ubb34\ub97c \uac70\uc808\ub2f9\ud588\ub2e4. \ud55c\ub3d9\uc548 \uc5ec\ud589\uc744 \ud558\uba70 \ubcf4\ub0b4\ub358 \ubaa8\ub85c\ub294 \ud2b8\ub80c\ud134(Trenton)\uc5d0\uc11c \uba87\ub9c8\uc77c \ub5a8\uc5b4\uc9c4 \ub378\ub77c\uc6e8\uc5b4 \uac15(Delaware River) \uadfc\ucc98 \uc7a5\uc6d0\uc744 \uad6c\uc785\ud574 1806\ub144 \ub274\uc800\uc9c0\uc5d0 \uc815\ucc29\ud558\uac8c \ub418\uc5c8\ub2e4. \ubaa8\ub85c\ub294 1813\ub144\uae4c\uc9c0 \uadf8\uacf3\uc5d0\uc11c \uc0b4\uba74\uc11c \ub09a\uc2dc, \uc0ac\ub0e5, \uce5c\ubaa9 \uad50\uc81c\ub85c \uc2dc\uac04\uc744 \ubcf4\ub0c8\ub2e4. \ubaa8\ub85c\uc758 \uac70\uc8fc\uc9c0\ub294 \ubaa8\ub4e0 \uc815\uce58\uc801 \uad6d\uc678 \ub9dd\uba85\uc790\ub4e4\uc758 \ud53c\ub09c\ucc98\uac00 \ub418\uc5c8\uace0, \uc678\uad6d\uc758 \ud798\uc744 \ube4c\ub824 \ub098\ud3f4\ub808\uc639\uc744 \uc0c1\ub300\ub85c \ubaa8\ub85c\uac00 \uce7c\uc744 \ub4e4\ub3c4\ub85d \uad8c\uc720\ud558\uc600\ub2e4. 1812\ub144 \uc804\uc7c1\uc758 \ubc1c\ubc1c\ub85c \ubaa8\ub85c\ub294 \ub9e4\ub514\uc2a8 \ub300\ud1b5\ub839\uc73c\ub85c\ubd80\ud130 \ubbf8\uad6d \uad70\ub300\uc758 \uc0ac\ub839\uad00\uc9c1\uc744 \uc81c\uc758\ubc1b\uc558\ub2e4. \ubaa8\ub85c\ub294 \uae30\uaebc\uc774 \ubc1b\uc544\ub4e4\uc600\uc9c0\ub9cc \uace7 \ub7ec\uc2dc\uc544\uc5d0\uc11c \ub300\uc721\uad70(Grande Arm\u00e9e)\uc774 \ud30c\uba78\ud588\ub2e4\ub294 \uc18c\uc2dd\uc744 \uc811\ud558\uace0 \uc720\ub7fd\uc73c\ub85c \ub3cc\uc544\uac00\uae30\ub85c \uacb0\uc2ec\ud588\ub2e4.", "answer": "\ub9e4\ub514\uc2a8 \ub300\ud1b5\ub839", "sentence": "1812\ub144 \uc804\uc7c1\uc758 \ubc1c\ubc1c\ub85c \ubaa8\ub85c\ub294 \ub9e4\ub514\uc2a8 \ub300\ud1b5\ub839 \uc73c\ub85c\ubd80\ud130 \ubbf8\uad6d \uad70\ub300\uc758 \uc0ac\ub839\uad00\uc9c1\uc744 \uc81c\uc758\ubc1b\uc558\ub2e4.", "paragraph_sentence": "\ubaa8\ub85c\ub294 \uc544\ub0b4\uc640 \ud568\uaed8 12\uc6d4 \ub274\uc695\uc2dc\uc5d0 \ub3c4\ucc29\ud588\ub2e4. \ubaa8\ub85c\ub294 \uc774 \uc9c0\uc5ed\uc5d0\uc11c \uc5f4\uc815\uc744 \uac00\uc9c0\uace0 \uc77c\uc744 \uc5bb\uc73c\ub824\uace0 \ud588\uc73c\ub098 \ubaa8\ub4e0 \uc9c1\uc7a5\uc5d0\uc11c \uadfc\ubb34\ub97c \uac70\uc808\ub2f9\ud588\ub2e4. \ud55c\ub3d9\uc548 \uc5ec\ud589\uc744 \ud558\uba70 \ubcf4\ub0b4\ub358 \ubaa8\ub85c\ub294 \ud2b8\ub80c\ud134(Trenton)\uc5d0\uc11c \uba87\ub9c8\uc77c \ub5a8\uc5b4\uc9c4 \ub378\ub77c\uc6e8\uc5b4 \uac15(Delaware River) \uadfc\ucc98 \uc7a5\uc6d0\uc744 \uad6c\uc785\ud574 1806\ub144 \ub274\uc800\uc9c0\uc5d0 \uc815\ucc29\ud558\uac8c \ub418\uc5c8\ub2e4. \ubaa8\ub85c\ub294 1813\ub144\uae4c\uc9c0 \uadf8\uacf3\uc5d0\uc11c \uc0b4\uba74\uc11c \ub09a\uc2dc, \uc0ac\ub0e5, \uce5c\ubaa9 \uad50\uc81c\ub85c \uc2dc\uac04\uc744 \ubcf4\ub0c8\ub2e4. \ubaa8\ub85c\uc758 \uac70\uc8fc\uc9c0\ub294 \ubaa8\ub4e0 \uc815\uce58\uc801 \uad6d\uc678 \ub9dd\uba85\uc790\ub4e4\uc758 \ud53c\ub09c\ucc98\uac00 \ub418\uc5c8\uace0, \uc678\uad6d\uc758 \ud798\uc744 \ube4c\ub824 \ub098\ud3f4\ub808\uc639\uc744 \uc0c1\ub300\ub85c \ubaa8\ub85c\uac00 \uce7c\uc744 \ub4e4\ub3c4\ub85d \uad8c\uc720\ud558\uc600\ub2e4. 1812\ub144 \uc804\uc7c1\uc758 \ubc1c\ubc1c\ub85c \ubaa8\ub85c\ub294 \ub9e4\ub514\uc2a8 \ub300\ud1b5\ub839 \uc73c\ub85c\ubd80\ud130 \ubbf8\uad6d \uad70\ub300\uc758 \uc0ac\ub839\uad00\uc9c1\uc744 \uc81c\uc758\ubc1b\uc558\ub2e4. \ubaa8\ub85c\ub294 \uae30\uaebc\uc774 \ubc1b\uc544\ub4e4\uc600\uc9c0\ub9cc \uace7 \ub7ec\uc2dc\uc544\uc5d0\uc11c \ub300\uc721\uad70(Grande Arm\u00e9e)\uc774 \ud30c\uba78\ud588\ub2e4\ub294 \uc18c\uc2dd\uc744 \uc811\ud558\uace0 \uc720\ub7fd\uc73c\ub85c \ub3cc\uc544\uac00\uae30\ub85c \uacb0\uc2ec\ud588\ub2e4.", "paragraph_answer": "\ubaa8\ub85c\ub294 \uc544\ub0b4\uc640 \ud568\uaed8 12\uc6d4 \ub274\uc695\uc2dc\uc5d0 \ub3c4\ucc29\ud588\ub2e4. \ubaa8\ub85c\ub294 \uc774 \uc9c0\uc5ed\uc5d0\uc11c \uc5f4\uc815\uc744 \uac00\uc9c0\uace0 \uc77c\uc744 \uc5bb\uc73c\ub824\uace0 \ud588\uc73c\ub098 \ubaa8\ub4e0 \uc9c1\uc7a5\uc5d0\uc11c \uadfc\ubb34\ub97c \uac70\uc808\ub2f9\ud588\ub2e4. \ud55c\ub3d9\uc548 \uc5ec\ud589\uc744 \ud558\uba70 \ubcf4\ub0b4\ub358 \ubaa8\ub85c\ub294 \ud2b8\ub80c\ud134(Trenton)\uc5d0\uc11c \uba87\ub9c8\uc77c \ub5a8\uc5b4\uc9c4 \ub378\ub77c\uc6e8\uc5b4 \uac15(Delaware River) \uadfc\ucc98 \uc7a5\uc6d0\uc744 \uad6c\uc785\ud574 1806\ub144 \ub274\uc800\uc9c0\uc5d0 \uc815\ucc29\ud558\uac8c \ub418\uc5c8\ub2e4. \ubaa8\ub85c\ub294 1813\ub144\uae4c\uc9c0 \uadf8\uacf3\uc5d0\uc11c \uc0b4\uba74\uc11c \ub09a\uc2dc, \uc0ac\ub0e5, \uce5c\ubaa9 \uad50\uc81c\ub85c \uc2dc\uac04\uc744 \ubcf4\ub0c8\ub2e4. \ubaa8\ub85c\uc758 \uac70\uc8fc\uc9c0\ub294 \ubaa8\ub4e0 \uc815\uce58\uc801 \uad6d\uc678 \ub9dd\uba85\uc790\ub4e4\uc758 \ud53c\ub09c\ucc98\uac00 \ub418\uc5c8\uace0, \uc678\uad6d\uc758 \ud798\uc744 \ube4c\ub824 \ub098\ud3f4\ub808\uc639\uc744 \uc0c1\ub300\ub85c \ubaa8\ub85c\uac00 \uce7c\uc744 \ub4e4\ub3c4\ub85d \uad8c\uc720\ud558\uc600\ub2e4. 1812\ub144 \uc804\uc7c1\uc758 \ubc1c\ubc1c\ub85c \ubaa8\ub85c\ub294 \ub9e4\ub514\uc2a8 \ub300\ud1b5\ub839 \uc73c\ub85c\ubd80\ud130 \ubbf8\uad6d \uad70\ub300\uc758 \uc0ac\ub839\uad00\uc9c1\uc744 \uc81c\uc758\ubc1b\uc558\ub2e4. \ubaa8\ub85c\ub294 \uae30\uaebc\uc774 \ubc1b\uc544\ub4e4\uc600\uc9c0\ub9cc \uace7 \ub7ec\uc2dc\uc544\uc5d0\uc11c \ub300\uc721\uad70(Grande Arm\u00e9e)\uc774 \ud30c\uba78\ud588\ub2e4\ub294 \uc18c\uc2dd\uc744 \uc811\ud558\uace0 \uc720\ub7fd\uc73c\ub85c \ub3cc\uc544\uac00\uae30\ub85c \uacb0\uc2ec\ud588\ub2e4.", "sentence_answer": "1812\ub144 \uc804\uc7c1\uc758 \ubc1c\ubc1c\ub85c \ubaa8\ub85c\ub294 \ub9e4\ub514\uc2a8 \ub300\ud1b5\ub839 \uc73c\ub85c\ubd80\ud130 \ubbf8\uad6d \uad70\ub300\uc758 \uc0ac\ub839\uad00\uc9c1\uc744 \uc81c\uc758\ubc1b\uc558\ub2e4.", "paragraph_id": "6548931-3-2", "paragraph_question": "question: 1812\ub144 \uc804\uc7c1\uc758 \ubc1c\ubc1c\ub85c \ubaa8\ub85c\uc5d0\uac8c \ubbf8\uad6d \uad70\ub300\uc758 \uc0ac\ub839\uad00\uc9c1\uc744 \uc81c\uc758\ud55c \uc0ac\ub78c\uc740 \ub204\uad6c\uc778\uac00?, context: \ubaa8\ub85c\ub294 \uc544\ub0b4\uc640 \ud568\uaed8 12\uc6d4 \ub274\uc695\uc2dc\uc5d0 \ub3c4\ucc29\ud588\ub2e4. \ubaa8\ub85c\ub294 \uc774 \uc9c0\uc5ed\uc5d0\uc11c \uc5f4\uc815\uc744 \uac00\uc9c0\uace0 \uc77c\uc744 \uc5bb\uc73c\ub824\uace0 \ud588\uc73c\ub098 \ubaa8\ub4e0 \uc9c1\uc7a5\uc5d0\uc11c \uadfc\ubb34\ub97c \uac70\uc808\ub2f9\ud588\ub2e4. \ud55c\ub3d9\uc548 \uc5ec\ud589\uc744 \ud558\uba70 \ubcf4\ub0b4\ub358 \ubaa8\ub85c\ub294 \ud2b8\ub80c\ud134(Trenton)\uc5d0\uc11c \uba87\ub9c8\uc77c \ub5a8\uc5b4\uc9c4 \ub378\ub77c\uc6e8\uc5b4 \uac15(Delaware River) \uadfc\ucc98 \uc7a5\uc6d0\uc744 \uad6c\uc785\ud574 1806\ub144 \ub274\uc800\uc9c0\uc5d0 \uc815\ucc29\ud558\uac8c \ub418\uc5c8\ub2e4. \ubaa8\ub85c\ub294 1813\ub144\uae4c\uc9c0 \uadf8\uacf3\uc5d0\uc11c \uc0b4\uba74\uc11c \ub09a\uc2dc, \uc0ac\ub0e5, \uce5c\ubaa9 \uad50\uc81c\ub85c \uc2dc\uac04\uc744 \ubcf4\ub0c8\ub2e4. \ubaa8\ub85c\uc758 \uac70\uc8fc\uc9c0\ub294 \ubaa8\ub4e0 \uc815\uce58\uc801 \uad6d\uc678 \ub9dd\uba85\uc790\ub4e4\uc758 \ud53c\ub09c\ucc98\uac00 \ub418\uc5c8\uace0, \uc678\uad6d\uc758 \ud798\uc744 \ube4c\ub824 \ub098\ud3f4\ub808\uc639\uc744 \uc0c1\ub300\ub85c \ubaa8\ub85c\uac00 \uce7c\uc744 \ub4e4\ub3c4\ub85d \uad8c\uc720\ud558\uc600\ub2e4. 1812\ub144 \uc804\uc7c1\uc758 \ubc1c\ubc1c\ub85c \ubaa8\ub85c\ub294 \ub9e4\ub514\uc2a8 \ub300\ud1b5\ub839\uc73c\ub85c\ubd80\ud130 \ubbf8\uad6d \uad70\ub300\uc758 \uc0ac\ub839\uad00\uc9c1\uc744 \uc81c\uc758\ubc1b\uc558\ub2e4. \ubaa8\ub85c\ub294 \uae30\uaebc\uc774 \ubc1b\uc544\ub4e4\uc600\uc9c0\ub9cc \uace7 \ub7ec\uc2dc\uc544\uc5d0\uc11c \ub300\uc721\uad70(Grande Arm\u00e9e)\uc774 \ud30c\uba78\ud588\ub2e4\ub294 \uc18c\uc2dd\uc744 \uc811\ud558\uace0 \uc720\ub7fd\uc73c\ub85c \ub3cc\uc544\uac00\uae30\ub85c \uacb0\uc2ec\ud588\ub2e4."} {"question": "2010\ub144 12\uc6d4 16\uc77c \ubd88\uad50, \uac1c\uc2e0\uad50, \uac00\ud1a8\ub9ad, \uc6d0\ubd88\uad50 \ub4f1\uc774 \uc5f0\ud569\ud558\uc5ec \ubc18\ub300\uc6b4\ub3d9\uc744 \ubc8c\uc778 \ub2e8\uccb4\uc758 \uc774\ub984\uc740?", "paragraph": "4\ub300\uac15 \uc0ac\uc5c5 \ubc18\ub300 \uc6b4\ub3d9\uc740 \uc8fc\ub85c \ud658\uacbd\ub2e8\uccb4\uc640, \uc9c4\ubcf4\uc131\ud5a5\uc758 \uc2dc\ubbfc\ub2e8\uccb4 \uadf8\ub9ac\uace0 \ubd88\uad50\uc640, \ucc9c\uc8fc\uad50\ub97c \uc911\uc2ec\uc73c\ub85c \uad11\ubc94\uc704\ud558\uac8c \uc774\ub8e8\uc5b4\uc9c0\uace0 \uc788\ub2e4. 2010\ub144 6\uc6d4 1\uc77c \ubb38\uc218 \uc2a4\ub2d8\uc740 4\ub300\uac15\uc744 \ubc18\ub300\ud558\uba70 \ubab8\uc5d0 \ubd88\uc744 \uc9c8\ub7ec \uc18c\uc2e0\uacf5\uc591\uc744 \ud558\uc600\uc73c\uba70, \ubd88\uad50 \ucd5c\ub300 \uc885\ud30c\uc778 \uc870\uacc4\uc885\ub3c4 4\ub300\uac15 \uc0ac\uc5c5 \ubc18\ub300\uc785\uc7a5\uc744 \ubc1d\ud614\ub2e4. \uc870\uacc4\uc885\uc740 \ub610\ud55c 2010\ub144 12\uc6d4 8\uc77c, 2011\ub144 \uc0c8\ud574 \uc608\uc0b0\uc548\uc774 \uad6d\ud68c\uc758\uc7a5\uc758 \uc9c8\uc11c\uc720\uc9c0\uad8c\uc774 \ubc1c\ub3d9\ub41c \uc0c1\ud669\uc5d0\uc11c \ucc98\ub9ac\ub418\uc5b4 4\ub300\uac15 \uc608\uc0b0\uc774 \ud3ec\ud568\ub418\uc5b4 \uc0ac\uc5c5\uc774 \uac15\ud589\ub41c\ub2e4\ub294 \uac83\uc5d0 \ub300\ud574 \ubc18\ubc1c\ud558\uba70 \uc815\ubd80\u00b7\uc5ec\ub2f9\uc5d0 \ub300\ud574 \uc804\uad6d \uc0ac\ucc30\uc758 \ucd9c\uc785 \uae08\uc9c0\ub839\uc744 \ub0b4\ub838\ub2e4. \uc870\uacc4\uc885\uc740 \"(\uc815\ubd80 \uc5ec\ub2f9 \uc778\uc0ac\ub97c)\uc0ac\uc801\uc73c\ub85c\ub77c\ub3c4 \ub9cc\ub098\uac70\ub098 \ud1b5\ud654\ud558\uba74 \uc0ac\uc9c1\uc11c\ub97c \uc81c\ucd9c\ud558\ub77c\"\uba70 \ud22c\uc7c1 \uc758\uc9c0\ub97c \ubc1d\ud614\ub2e4. \ub610\ud55c \ucc9c\uc8fc\uad50\ub294 \uc815\uc758\uad6c\ud604\uc0ac\uc81c\ub2e8\uc774 \ubc18\ub300\uc5d0 \uc801\uadf9\uc801\uc774\uba70, \ucc9c\uc8fc\uad50\uc5f0\ub300\ub3c4 4\ub300\uac15\uc744 \ubc18\ub300\ud558\ub294 \uc2dc\uad6d \ubbf8\uc0ac\ub97c \uc5f4\uae30\ub3c4 \ud558\uc600\ub2e4. 10\uc6d4 27\uc77c \ud55c\uad6d \ucc9c\uc8fc\uad50 \uc8fc\uad50\ud68c\uc758\uac00 4\ub300\uac15 \uc0ac\uc5c5\uc744 \"\ub09c\uac1c\ubc1c\"\uc774\ub77c\uace0 \uc9c0\uc801\ud558\uc600\ub2e4. \uc77c\ubd80 \uac1c\uc2e0\uad50\uc640 \uc6d0\ubd88\uad50\uc5d0\uc11c\ub3c4 '\uc885\uad50\ud658\uacbd\ud68c\uc758'\ub97c \uad6c\uc131\ud558\uc5ec 4\ub300\uac15\uc5d0 \ub300\ud55c \ubc18\ub300 \uc785\uc7a5\uc744 \ubc1d\ud788\uae30\ub3c4 \ud588\ub2e4. 2010\ub144 7\uc6d4 8\uc77c\uc5d0\ub294 \uc870\uacc4\uc885 \uc2b9\ub824 13,000\uc5ec\uba85 \uac00\uc6b4\ub370 1/3 \uac00\ub7c9\uc758 \uc2a4\ub2d8 4,000\uc5ec\uba85\uc774 4\ub300\uac15 \uc0ac\uc5c5 \ubc18\ub300 \uc11c\uba85\uc5d0 \ub3d9\ucc38\ud588\ub2e4. \uc774\ub294 \uc870\uacc4\uc885 \uc0ac\uc0c1 \uc720\ub840\uac00 \uc5c6\ub294 \uc77c\ub85c \uae30\ub85d\ub41c\ub2e4. \uc131\uc6c5 \uc9c1\uc9c0\uc0ac \uc8fc\uc9c0\ub294 \"\ud3b8\ud611\ud55c \uc774\uae30\uc2ec, \uac1c\ubc1c\uc774\uc775\uc5d0 \ub300\ud55c \uc695\ub9dd\uc73c\ub85c \uc6b0\ub9ac\uc758 \ubb38\ud654\uc720\uc0b0\uacfc \uc815\uc2e0\uc744 \ubb3b\uc5b4\uac00\uba74\uc11c, \uc5b4\ub5bb\uac8c \uad6d\ubbfc\ud1b5\ud569\uc744 \uc774\ub904\uc9c8\uc218 \uc788\uaca0\ub294\uac00\"\ub77c\uba70 4\ub300\uac15 \uc0ac\uc5c5\uc758 \uc911\ub2e8\uc744 \ucd09\uad6c\ud588\ub2e4. \uacb0\uad6d 2010\ub144 12\uc6d4 16\uc77c \ubd88\uad50, \uac1c\uc2e0\uad50, \uac00\ud1a8\ub9ad, \uc6d0\ubd88\uad50 \ub4f1\uc774 \uc5f0\ud569\ud558\uc5ec '4\ub300\uac15 \uac1c\ubc1c\uc800\uc9c0 4\ub300\uc885\ub2e8 \uc5f0\ub300\ud68c\uc758'\ub97c \uad6c\uc131\ud558\uc5ec \ubc18\ub300\uc6b4\ub3d9\uc744 \ubc8c\uc774\uae30\uc5d0 \uc774\ub974\ub800\ub2e4. \ud658\uacbd\ub2e8\uccb4\ub4e4\uc740 \ucc98\uc74c \ub17c\uc758\uac00 \uc2dc\uc791\ub420\ub54c\ubd80\ud130 \uafb8\uc900\ud788 \ubc18\ub300 \uc785\uc7a5\uc744 \ubc1d\ud600\uc654\ub2e4. \uc774\ub7ec\ud55c \ubc18\ub300\uc5d0\ub3c4 \ubd88\uad6c\ud558\uace0 2009\ub144 12\uc6d4, \ud55c\ub098\ub77c\ub2f9\uc740 3\uc870 5\ucc9c\uc5b5\uc6d0\uc5d0 \ub2ec\ud558\ub294 \uc608\uc0b0\uc744 \uad6d\ud68c \uc0c1\uc784\uc704\uc5d0\uc11c \uae30\uc2b5\uc801\uc73c\ub85c \ud1b5\uacfc\uc2dc\ud0a4\uae30\ub3c4 \ud588\ub2e4.", "answer": "4\ub300\uac15 \uac1c\ubc1c\uc800\uc9c0 4\ub300\uc885\ub2e8 \uc5f0\ub300\ud68c\uc758", "sentence": "\uacb0\uad6d 2010\ub144 12\uc6d4 16\uc77c \ubd88\uad50, \uac1c\uc2e0\uad50, \uac00\ud1a8\ub9ad, \uc6d0\ubd88\uad50 \ub4f1\uc774 \uc5f0\ud569\ud558\uc5ec ' 4\ub300\uac15 \uac1c\ubc1c\uc800\uc9c0 4\ub300\uc885\ub2e8 \uc5f0\ub300\ud68c\uc758 '\ub97c \uad6c\uc131\ud558\uc5ec \ubc18\ub300\uc6b4\ub3d9\uc744 \ubc8c\uc774\uae30\uc5d0 \uc774\ub974\ub800\ub2e4.", "paragraph_sentence": "4\ub300\uac15 \uc0ac\uc5c5 \ubc18\ub300 \uc6b4\ub3d9\uc740 \uc8fc\ub85c \ud658\uacbd\ub2e8\uccb4\uc640, \uc9c4\ubcf4\uc131\ud5a5\uc758 \uc2dc\ubbfc\ub2e8\uccb4 \uadf8\ub9ac\uace0 \ubd88\uad50\uc640, \ucc9c\uc8fc\uad50\ub97c \uc911\uc2ec\uc73c\ub85c \uad11\ubc94\uc704\ud558\uac8c \uc774\ub8e8\uc5b4\uc9c0\uace0 \uc788\ub2e4. 2010\ub144 6\uc6d4 1\uc77c \ubb38\uc218 \uc2a4\ub2d8\uc740 4\ub300\uac15\uc744 \ubc18\ub300\ud558\uba70 \ubab8\uc5d0 \ubd88\uc744 \uc9c8\ub7ec \uc18c\uc2e0\uacf5\uc591\uc744 \ud558\uc600\uc73c\uba70, \ubd88\uad50 \ucd5c\ub300 \uc885\ud30c\uc778 \uc870\uacc4\uc885\ub3c4 4\ub300\uac15 \uc0ac\uc5c5 \ubc18\ub300\uc785\uc7a5\uc744 \ubc1d\ud614\ub2e4. \uc870\uacc4\uc885\uc740 \ub610\ud55c 2010\ub144 12\uc6d4 8\uc77c, 2011\ub144 \uc0c8\ud574 \uc608\uc0b0\uc548\uc774 \uad6d\ud68c\uc758\uc7a5\uc758 \uc9c8\uc11c\uc720\uc9c0\uad8c\uc774 \ubc1c\ub3d9\ub41c \uc0c1\ud669\uc5d0\uc11c \ucc98\ub9ac\ub418\uc5b4 4\ub300\uac15 \uc608\uc0b0\uc774 \ud3ec\ud568\ub418\uc5b4 \uc0ac\uc5c5\uc774 \uac15\ud589\ub41c\ub2e4\ub294 \uac83\uc5d0 \ub300\ud574 \ubc18\ubc1c\ud558\uba70 \uc815\ubd80\u00b7\uc5ec\ub2f9\uc5d0 \ub300\ud574 \uc804\uad6d \uc0ac\ucc30\uc758 \ucd9c\uc785 \uae08\uc9c0\ub839\uc744 \ub0b4\ub838\ub2e4. \uc870\uacc4\uc885\uc740 \"(\uc815\ubd80 \uc5ec\ub2f9 \uc778\uc0ac\ub97c)\uc0ac\uc801\uc73c\ub85c\ub77c\ub3c4 \ub9cc\ub098\uac70\ub098 \ud1b5\ud654\ud558\uba74 \uc0ac\uc9c1\uc11c\ub97c \uc81c\ucd9c\ud558\ub77c\"\uba70 \ud22c\uc7c1 \uc758\uc9c0\ub97c \ubc1d\ud614\ub2e4. \ub610\ud55c \ucc9c\uc8fc\uad50\ub294 \uc815\uc758\uad6c\ud604\uc0ac\uc81c\ub2e8\uc774 \ubc18\ub300\uc5d0 \uc801\uadf9\uc801\uc774\uba70, \ucc9c\uc8fc\uad50\uc5f0\ub300\ub3c4 4\ub300\uac15\uc744 \ubc18\ub300\ud558\ub294 \uc2dc\uad6d \ubbf8\uc0ac\ub97c \uc5f4\uae30\ub3c4 \ud558\uc600\ub2e4. 10\uc6d4 27\uc77c \ud55c\uad6d \ucc9c\uc8fc\uad50 \uc8fc\uad50\ud68c\uc758\uac00 4\ub300\uac15 \uc0ac\uc5c5\uc744 \"\ub09c\uac1c\ubc1c\"\uc774\ub77c\uace0 \uc9c0\uc801\ud558\uc600\ub2e4. \uc77c\ubd80 \uac1c\uc2e0\uad50\uc640 \uc6d0\ubd88\uad50\uc5d0\uc11c\ub3c4 '\uc885\uad50\ud658\uacbd\ud68c\uc758'\ub97c \uad6c\uc131\ud558\uc5ec 4\ub300\uac15\uc5d0 \ub300\ud55c \ubc18\ub300 \uc785\uc7a5\uc744 \ubc1d\ud788\uae30\ub3c4 \ud588\ub2e4. 2010\ub144 7\uc6d4 8\uc77c\uc5d0\ub294 \uc870\uacc4\uc885 \uc2b9\ub824 13,000\uc5ec\uba85 \uac00\uc6b4\ub370 1/3 \uac00\ub7c9\uc758 \uc2a4\ub2d8 4,000\uc5ec\uba85\uc774 4\ub300\uac15 \uc0ac\uc5c5 \ubc18\ub300 \uc11c\uba85\uc5d0 \ub3d9\ucc38\ud588\ub2e4. \uc774\ub294 \uc870\uacc4\uc885 \uc0ac\uc0c1 \uc720\ub840\uac00 \uc5c6\ub294 \uc77c\ub85c \uae30\ub85d\ub41c\ub2e4. \uc131\uc6c5 \uc9c1\uc9c0\uc0ac \uc8fc\uc9c0\ub294 \"\ud3b8\ud611\ud55c \uc774\uae30\uc2ec, \uac1c\ubc1c\uc774\uc775\uc5d0 \ub300\ud55c \uc695\ub9dd\uc73c\ub85c \uc6b0\ub9ac\uc758 \ubb38\ud654\uc720\uc0b0\uacfc \uc815\uc2e0\uc744 \ubb3b\uc5b4\uac00\uba74\uc11c, \uc5b4\ub5bb\uac8c \uad6d\ubbfc\ud1b5\ud569\uc744 \uc774\ub904\uc9c8\uc218 \uc788\uaca0\ub294\uac00\"\ub77c\uba70 4\ub300\uac15 \uc0ac\uc5c5\uc758 \uc911\ub2e8\uc744 \ucd09\uad6c\ud588\ub2e4. \uacb0\uad6d 2010\ub144 12\uc6d4 16\uc77c \ubd88\uad50, \uac1c\uc2e0\uad50, \uac00\ud1a8\ub9ad, \uc6d0\ubd88\uad50 \ub4f1\uc774 \uc5f0\ud569\ud558\uc5ec ' 4\ub300\uac15 \uac1c\ubc1c\uc800\uc9c0 4\ub300\uc885\ub2e8 \uc5f0\ub300\ud68c\uc758 '\ub97c \uad6c\uc131\ud558\uc5ec \ubc18\ub300\uc6b4\ub3d9\uc744 \ubc8c\uc774\uae30\uc5d0 \uc774\ub974\ub800\ub2e4. \ud658\uacbd\ub2e8\uccb4\ub4e4\uc740 \ucc98\uc74c \ub17c\uc758\uac00 \uc2dc\uc791\ub420\ub54c\ubd80\ud130 \uafb8\uc900\ud788 \ubc18\ub300 \uc785\uc7a5\uc744 \ubc1d\ud600\uc654\ub2e4. \uc774\ub7ec\ud55c \ubc18\ub300\uc5d0\ub3c4 \ubd88\uad6c\ud558\uace0 2009\ub144 12\uc6d4, \ud55c\ub098\ub77c\ub2f9\uc740 3\uc870 5\ucc9c\uc5b5\uc6d0\uc5d0 \ub2ec\ud558\ub294 \uc608\uc0b0\uc744 \uad6d\ud68c \uc0c1\uc784\uc704\uc5d0\uc11c \uae30\uc2b5\uc801\uc73c\ub85c \ud1b5\uacfc\uc2dc\ud0a4\uae30\ub3c4 \ud588\ub2e4.", "paragraph_answer": "4\ub300\uac15 \uc0ac\uc5c5 \ubc18\ub300 \uc6b4\ub3d9\uc740 \uc8fc\ub85c \ud658\uacbd\ub2e8\uccb4\uc640, \uc9c4\ubcf4\uc131\ud5a5\uc758 \uc2dc\ubbfc\ub2e8\uccb4 \uadf8\ub9ac\uace0 \ubd88\uad50\uc640, \ucc9c\uc8fc\uad50\ub97c \uc911\uc2ec\uc73c\ub85c \uad11\ubc94\uc704\ud558\uac8c \uc774\ub8e8\uc5b4\uc9c0\uace0 \uc788\ub2e4. 2010\ub144 6\uc6d4 1\uc77c \ubb38\uc218 \uc2a4\ub2d8\uc740 4\ub300\uac15\uc744 \ubc18\ub300\ud558\uba70 \ubab8\uc5d0 \ubd88\uc744 \uc9c8\ub7ec \uc18c\uc2e0\uacf5\uc591\uc744 \ud558\uc600\uc73c\uba70, \ubd88\uad50 \ucd5c\ub300 \uc885\ud30c\uc778 \uc870\uacc4\uc885\ub3c4 4\ub300\uac15 \uc0ac\uc5c5 \ubc18\ub300\uc785\uc7a5\uc744 \ubc1d\ud614\ub2e4. \uc870\uacc4\uc885\uc740 \ub610\ud55c 2010\ub144 12\uc6d4 8\uc77c, 2011\ub144 \uc0c8\ud574 \uc608\uc0b0\uc548\uc774 \uad6d\ud68c\uc758\uc7a5\uc758 \uc9c8\uc11c\uc720\uc9c0\uad8c\uc774 \ubc1c\ub3d9\ub41c \uc0c1\ud669\uc5d0\uc11c \ucc98\ub9ac\ub418\uc5b4 4\ub300\uac15 \uc608\uc0b0\uc774 \ud3ec\ud568\ub418\uc5b4 \uc0ac\uc5c5\uc774 \uac15\ud589\ub41c\ub2e4\ub294 \uac83\uc5d0 \ub300\ud574 \ubc18\ubc1c\ud558\uba70 \uc815\ubd80\u00b7\uc5ec\ub2f9\uc5d0 \ub300\ud574 \uc804\uad6d \uc0ac\ucc30\uc758 \ucd9c\uc785 \uae08\uc9c0\ub839\uc744 \ub0b4\ub838\ub2e4. \uc870\uacc4\uc885\uc740 \"(\uc815\ubd80 \uc5ec\ub2f9 \uc778\uc0ac\ub97c)\uc0ac\uc801\uc73c\ub85c\ub77c\ub3c4 \ub9cc\ub098\uac70\ub098 \ud1b5\ud654\ud558\uba74 \uc0ac\uc9c1\uc11c\ub97c \uc81c\ucd9c\ud558\ub77c\"\uba70 \ud22c\uc7c1 \uc758\uc9c0\ub97c \ubc1d\ud614\ub2e4. \ub610\ud55c \ucc9c\uc8fc\uad50\ub294 \uc815\uc758\uad6c\ud604\uc0ac\uc81c\ub2e8\uc774 \ubc18\ub300\uc5d0 \uc801\uadf9\uc801\uc774\uba70, \ucc9c\uc8fc\uad50\uc5f0\ub300\ub3c4 4\ub300\uac15\uc744 \ubc18\ub300\ud558\ub294 \uc2dc\uad6d \ubbf8\uc0ac\ub97c \uc5f4\uae30\ub3c4 \ud558\uc600\ub2e4. 10\uc6d4 27\uc77c \ud55c\uad6d \ucc9c\uc8fc\uad50 \uc8fc\uad50\ud68c\uc758\uac00 4\ub300\uac15 \uc0ac\uc5c5\uc744 \"\ub09c\uac1c\ubc1c\"\uc774\ub77c\uace0 \uc9c0\uc801\ud558\uc600\ub2e4. \uc77c\ubd80 \uac1c\uc2e0\uad50\uc640 \uc6d0\ubd88\uad50\uc5d0\uc11c\ub3c4 '\uc885\uad50\ud658\uacbd\ud68c\uc758'\ub97c \uad6c\uc131\ud558\uc5ec 4\ub300\uac15\uc5d0 \ub300\ud55c \ubc18\ub300 \uc785\uc7a5\uc744 \ubc1d\ud788\uae30\ub3c4 \ud588\ub2e4. 2010\ub144 7\uc6d4 8\uc77c\uc5d0\ub294 \uc870\uacc4\uc885 \uc2b9\ub824 13,000\uc5ec\uba85 \uac00\uc6b4\ub370 1/3 \uac00\ub7c9\uc758 \uc2a4\ub2d8 4,000\uc5ec\uba85\uc774 4\ub300\uac15 \uc0ac\uc5c5 \ubc18\ub300 \uc11c\uba85\uc5d0 \ub3d9\ucc38\ud588\ub2e4. \uc774\ub294 \uc870\uacc4\uc885 \uc0ac\uc0c1 \uc720\ub840\uac00 \uc5c6\ub294 \uc77c\ub85c \uae30\ub85d\ub41c\ub2e4. \uc131\uc6c5 \uc9c1\uc9c0\uc0ac \uc8fc\uc9c0\ub294 \"\ud3b8\ud611\ud55c \uc774\uae30\uc2ec, \uac1c\ubc1c\uc774\uc775\uc5d0 \ub300\ud55c \uc695\ub9dd\uc73c\ub85c \uc6b0\ub9ac\uc758 \ubb38\ud654\uc720\uc0b0\uacfc \uc815\uc2e0\uc744 \ubb3b\uc5b4\uac00\uba74\uc11c, \uc5b4\ub5bb\uac8c \uad6d\ubbfc\ud1b5\ud569\uc744 \uc774\ub904\uc9c8\uc218 \uc788\uaca0\ub294\uac00\"\ub77c\uba70 4\ub300\uac15 \uc0ac\uc5c5\uc758 \uc911\ub2e8\uc744 \ucd09\uad6c\ud588\ub2e4. \uacb0\uad6d 2010\ub144 12\uc6d4 16\uc77c \ubd88\uad50, \uac1c\uc2e0\uad50, \uac00\ud1a8\ub9ad, \uc6d0\ubd88\uad50 \ub4f1\uc774 \uc5f0\ud569\ud558\uc5ec ' 4\ub300\uac15 \uac1c\ubc1c\uc800\uc9c0 4\ub300\uc885\ub2e8 \uc5f0\ub300\ud68c\uc758 '\ub97c \uad6c\uc131\ud558\uc5ec \ubc18\ub300\uc6b4\ub3d9\uc744 \ubc8c\uc774\uae30\uc5d0 \uc774\ub974\ub800\ub2e4. \ud658\uacbd\ub2e8\uccb4\ub4e4\uc740 \ucc98\uc74c \ub17c\uc758\uac00 \uc2dc\uc791\ub420\ub54c\ubd80\ud130 \uafb8\uc900\ud788 \ubc18\ub300 \uc785\uc7a5\uc744 \ubc1d\ud600\uc654\ub2e4. \uc774\ub7ec\ud55c \ubc18\ub300\uc5d0\ub3c4 \ubd88\uad6c\ud558\uace0 2009\ub144 12\uc6d4, \ud55c\ub098\ub77c\ub2f9\uc740 3\uc870 5\ucc9c\uc5b5\uc6d0\uc5d0 \ub2ec\ud558\ub294 \uc608\uc0b0\uc744 \uad6d\ud68c \uc0c1\uc784\uc704\uc5d0\uc11c \uae30\uc2b5\uc801\uc73c\ub85c \ud1b5\uacfc\uc2dc\ud0a4\uae30\ub3c4 \ud588\ub2e4.", "sentence_answer": "\uacb0\uad6d 2010\ub144 12\uc6d4 16\uc77c \ubd88\uad50, \uac1c\uc2e0\uad50, \uac00\ud1a8\ub9ad, \uc6d0\ubd88\uad50 \ub4f1\uc774 \uc5f0\ud569\ud558\uc5ec ' 4\ub300\uac15 \uac1c\ubc1c\uc800\uc9c0 4\ub300\uc885\ub2e8 \uc5f0\ub300\ud68c\uc758 '\ub97c \uad6c\uc131\ud558\uc5ec \ubc18\ub300\uc6b4\ub3d9\uc744 \ubc8c\uc774\uae30\uc5d0 \uc774\ub974\ub800\ub2e4.", "paragraph_id": "5780412-0-1", "paragraph_question": "question: 2010\ub144 12\uc6d4 16\uc77c \ubd88\uad50, \uac1c\uc2e0\uad50, \uac00\ud1a8\ub9ad, \uc6d0\ubd88\uad50 \ub4f1\uc774 \uc5f0\ud569\ud558\uc5ec \ubc18\ub300\uc6b4\ub3d9\uc744 \ubc8c\uc778 \ub2e8\uccb4\uc758 \uc774\ub984\uc740?, context: 4\ub300\uac15 \uc0ac\uc5c5 \ubc18\ub300 \uc6b4\ub3d9\uc740 \uc8fc\ub85c \ud658\uacbd\ub2e8\uccb4\uc640, \uc9c4\ubcf4\uc131\ud5a5\uc758 \uc2dc\ubbfc\ub2e8\uccb4 \uadf8\ub9ac\uace0 \ubd88\uad50\uc640, \ucc9c\uc8fc\uad50\ub97c \uc911\uc2ec\uc73c\ub85c \uad11\ubc94\uc704\ud558\uac8c \uc774\ub8e8\uc5b4\uc9c0\uace0 \uc788\ub2e4. 2010\ub144 6\uc6d4 1\uc77c \ubb38\uc218 \uc2a4\ub2d8\uc740 4\ub300\uac15\uc744 \ubc18\ub300\ud558\uba70 \ubab8\uc5d0 \ubd88\uc744 \uc9c8\ub7ec \uc18c\uc2e0\uacf5\uc591\uc744 \ud558\uc600\uc73c\uba70, \ubd88\uad50 \ucd5c\ub300 \uc885\ud30c\uc778 \uc870\uacc4\uc885\ub3c4 4\ub300\uac15 \uc0ac\uc5c5 \ubc18\ub300\uc785\uc7a5\uc744 \ubc1d\ud614\ub2e4. \uc870\uacc4\uc885\uc740 \ub610\ud55c 2010\ub144 12\uc6d4 8\uc77c, 2011\ub144 \uc0c8\ud574 \uc608\uc0b0\uc548\uc774 \uad6d\ud68c\uc758\uc7a5\uc758 \uc9c8\uc11c\uc720\uc9c0\uad8c\uc774 \ubc1c\ub3d9\ub41c \uc0c1\ud669\uc5d0\uc11c \ucc98\ub9ac\ub418\uc5b4 4\ub300\uac15 \uc608\uc0b0\uc774 \ud3ec\ud568\ub418\uc5b4 \uc0ac\uc5c5\uc774 \uac15\ud589\ub41c\ub2e4\ub294 \uac83\uc5d0 \ub300\ud574 \ubc18\ubc1c\ud558\uba70 \uc815\ubd80\u00b7\uc5ec\ub2f9\uc5d0 \ub300\ud574 \uc804\uad6d \uc0ac\ucc30\uc758 \ucd9c\uc785 \uae08\uc9c0\ub839\uc744 \ub0b4\ub838\ub2e4. \uc870\uacc4\uc885\uc740 \"(\uc815\ubd80 \uc5ec\ub2f9 \uc778\uc0ac\ub97c)\uc0ac\uc801\uc73c\ub85c\ub77c\ub3c4 \ub9cc\ub098\uac70\ub098 \ud1b5\ud654\ud558\uba74 \uc0ac\uc9c1\uc11c\ub97c \uc81c\ucd9c\ud558\ub77c\"\uba70 \ud22c\uc7c1 \uc758\uc9c0\ub97c \ubc1d\ud614\ub2e4. \ub610\ud55c \ucc9c\uc8fc\uad50\ub294 \uc815\uc758\uad6c\ud604\uc0ac\uc81c\ub2e8\uc774 \ubc18\ub300\uc5d0 \uc801\uadf9\uc801\uc774\uba70, \ucc9c\uc8fc\uad50\uc5f0\ub300\ub3c4 4\ub300\uac15\uc744 \ubc18\ub300\ud558\ub294 \uc2dc\uad6d \ubbf8\uc0ac\ub97c \uc5f4\uae30\ub3c4 \ud558\uc600\ub2e4. 10\uc6d4 27\uc77c \ud55c\uad6d \ucc9c\uc8fc\uad50 \uc8fc\uad50\ud68c\uc758\uac00 4\ub300\uac15 \uc0ac\uc5c5\uc744 \"\ub09c\uac1c\ubc1c\"\uc774\ub77c\uace0 \uc9c0\uc801\ud558\uc600\ub2e4. \uc77c\ubd80 \uac1c\uc2e0\uad50\uc640 \uc6d0\ubd88\uad50\uc5d0\uc11c\ub3c4 '\uc885\uad50\ud658\uacbd\ud68c\uc758'\ub97c \uad6c\uc131\ud558\uc5ec 4\ub300\uac15\uc5d0 \ub300\ud55c \ubc18\ub300 \uc785\uc7a5\uc744 \ubc1d\ud788\uae30\ub3c4 \ud588\ub2e4. 2010\ub144 7\uc6d4 8\uc77c\uc5d0\ub294 \uc870\uacc4\uc885 \uc2b9\ub824 13,000\uc5ec\uba85 \uac00\uc6b4\ub370 1/3 \uac00\ub7c9\uc758 \uc2a4\ub2d8 4,000\uc5ec\uba85\uc774 4\ub300\uac15 \uc0ac\uc5c5 \ubc18\ub300 \uc11c\uba85\uc5d0 \ub3d9\ucc38\ud588\ub2e4. \uc774\ub294 \uc870\uacc4\uc885 \uc0ac\uc0c1 \uc720\ub840\uac00 \uc5c6\ub294 \uc77c\ub85c \uae30\ub85d\ub41c\ub2e4. \uc131\uc6c5 \uc9c1\uc9c0\uc0ac \uc8fc\uc9c0\ub294 \"\ud3b8\ud611\ud55c \uc774\uae30\uc2ec, \uac1c\ubc1c\uc774\uc775\uc5d0 \ub300\ud55c \uc695\ub9dd\uc73c\ub85c \uc6b0\ub9ac\uc758 \ubb38\ud654\uc720\uc0b0\uacfc \uc815\uc2e0\uc744 \ubb3b\uc5b4\uac00\uba74\uc11c, \uc5b4\ub5bb\uac8c \uad6d\ubbfc\ud1b5\ud569\uc744 \uc774\ub904\uc9c8\uc218 \uc788\uaca0\ub294\uac00\"\ub77c\uba70 4\ub300\uac15 \uc0ac\uc5c5\uc758 \uc911\ub2e8\uc744 \ucd09\uad6c\ud588\ub2e4. \uacb0\uad6d 2010\ub144 12\uc6d4 16\uc77c \ubd88\uad50, \uac1c\uc2e0\uad50, \uac00\ud1a8\ub9ad, \uc6d0\ubd88\uad50 \ub4f1\uc774 \uc5f0\ud569\ud558\uc5ec '4\ub300\uac15 \uac1c\ubc1c\uc800\uc9c0 4\ub300\uc885\ub2e8 \uc5f0\ub300\ud68c\uc758'\ub97c \uad6c\uc131\ud558\uc5ec \ubc18\ub300\uc6b4\ub3d9\uc744 \ubc8c\uc774\uae30\uc5d0 \uc774\ub974\ub800\ub2e4. \ud658\uacbd\ub2e8\uccb4\ub4e4\uc740 \ucc98\uc74c \ub17c\uc758\uac00 \uc2dc\uc791\ub420\ub54c\ubd80\ud130 \uafb8\uc900\ud788 \ubc18\ub300 \uc785\uc7a5\uc744 \ubc1d\ud600\uc654\ub2e4. \uc774\ub7ec\ud55c \ubc18\ub300\uc5d0\ub3c4 \ubd88\uad6c\ud558\uace0 2009\ub144 12\uc6d4, \ud55c\ub098\ub77c\ub2f9\uc740 3\uc870 5\ucc9c\uc5b5\uc6d0\uc5d0 \ub2ec\ud558\ub294 \uc608\uc0b0\uc744 \uad6d\ud68c \uc0c1\uc784\uc704\uc5d0\uc11c \uae30\uc2b5\uc801\uc73c\ub85c \ud1b5\uacfc\uc2dc\ud0a4\uae30\ub3c4 \ud588\ub2e4."} {"question": "\ube0c\ub9ac\ud2b8\ub2c8\uc758 \uccab \ubc88\uc9f8 \ud22c\uc5b4\uac00 \uc2dc\uc791\ub41c \ub0a0\uc740?", "paragraph": "1999\ub144 6\uc6d4 28\uc77c, \ube0c\ub9ac\ud2b8\ub2c8\uc758 \uccab \ubc88\uc9f8 \ud22c\uc5b4\uc778 ...Baby One More Time Tour\uac00 \ubd81\uc544\uba54\ub9ac\uce74\uc5d0\uc11c \uc2dc\uc791\ub418\uc5c8\uc744 \ub54c \ube44\ud3c9\uac00\ub4e4\uc740 \ud638\ud3c9\uc744 \ub0b4\ub193\uc558\uc73c\ub098, \ud55c\ud3b8\uc5d0\uc120 \uc120\uc815\uc801\uc778 \uc758\uc0c1\uc73c\ub85c \ube44\ud310\ud558\uae30\ub3c4 \ud588\ub2e4. 2000\ub144 \ud22c\uc5b4\uac00 \ub9c9\ubc14\uc9c0\uc5d0 \uc774\ub974\ub800\uc744 \ub54c, Crazy 2k\ub77c\ub294 \uc81c\ubaa9\uc73c\ub85c \ud22c\uc5b4\ub97c \uc774\uc5b4\ub098\uac14\ub2e4. \uc774 \ud22c\uc5b4\uc5d0\uc11c \ube0c\ub9ac\ud2b8\ub2c8\ub294 \ub450 \ubc88\uc9f8 \uc74c\ubc18\uc758 \uc218\ub85d\uace1\ub4e4\uc744 \uc120\ubcf4\uc600\ub2e4. 2000\ub144 5\uc6d4, \ube0c\ub9ac\ud2b8\ub2c8\uc758 \ub450 \ubc88\uc9f8 \uc815\uaddc \uc74c\ubc18 Oops!...I Did It Again\uc774 \ubc1c\ub9e4\ub41c\ub2e4. \uc774 \uc74c\ubc18\uc740 \ubbf8\uad6d\uc5d0\uc11c \ubc1c\ub9e4 \uccab \uc8fc 1,300,000\ub9cc \uc7a5\uc744 \ud314\uc544\uce58\uc6b0\uba70 \uc0ac\uc6b4\ub4dc\uc2a4\uce94 \uc5ed\uc0ac\uc0c1 \uc194\ub85c \uac00\uc218 \uc911 \uac00\uc7a5 \ub192\uc740 \uccab \uc8fc \ud310\ub9e4\ub7c9\uc774\ub77c\ub294 \uae30\ub85d\uc744 \uc138\uc6e0\ub2e4. \uc568\ubc94\uc740 \uc138\uacc4\uc801\uc73c\ub85c 2,000\ub9cc \uc7a5 \uc774\uc0c1\uc758 \ud310\ub9e4\uace0\ub97c \uc62c\ub838\ub2e4. \uc74c\ubc18\uc758 \ub9ac\ub4dc \uc2f1\uae00 \"Oops!... I Did It Again\"\uc740 \ud638\uc8fc, \ub274\uc9c8\ub79c\ub4dc, \uc601\uad6d \ubc0f \uc5ec\ub7ec \uc720\ub7fd \uad6d\uac00\uc5d0\uc11c \uc815\uc0c1\uc744 \ucc28\uc9c0\ud588\ub2e4. \uc774 \ub178\ub798\ub294 \uadf8\ub798\ubbf8\uc0c1 \ucd5c\uc6b0\uc218 \uc5ec\uc790 \ud31d \ubcf4\uceec \ud37c\ud3ec\uba3c\uc2a4 \ubd80\ubb38\uacfc \uc74c\ubc18\uc740 \ucd5c\uc6b0\uc218 \ud31d \ubcf4\uceec \uc568\ubc94 \ubd80\ubb38\uc5d0 \uac01\uac01 \ud6c4\ubcf4\ub85c \uc62c\ub790\uc9c0\ub9cc, \uc218\uc0c1\uc740 \ud558\uc9c0 \ubabb\ud588\ub2e4.", "answer": "1999\ub144 6\uc6d4 28\uc77c", "sentence": "1999\ub144 6\uc6d4 28\uc77c ,", "paragraph_sentence": " 1999\ub144 6\uc6d4 28\uc77c , \ube0c\ub9ac\ud2b8\ub2c8\uc758 \uccab \ubc88\uc9f8 \ud22c\uc5b4\uc778 ...Baby One More Time Tour\uac00 \ubd81\uc544\uba54\ub9ac\uce74\uc5d0\uc11c \uc2dc\uc791\ub418\uc5c8\uc744 \ub54c \ube44\ud3c9\uac00\ub4e4\uc740 \ud638\ud3c9\uc744 \ub0b4\ub193\uc558\uc73c\ub098, \ud55c\ud3b8\uc5d0\uc120 \uc120\uc815\uc801\uc778 \uc758\uc0c1\uc73c\ub85c \ube44\ud310\ud558\uae30\ub3c4 \ud588\ub2e4. 2000\ub144 \ud22c\uc5b4\uac00 \ub9c9\ubc14\uc9c0\uc5d0 \uc774\ub974\ub800\uc744 \ub54c, Crazy 2k\ub77c\ub294 \uc81c\ubaa9\uc73c\ub85c \ud22c\uc5b4\ub97c \uc774\uc5b4\ub098\uac14\ub2e4. \uc774 \ud22c\uc5b4\uc5d0\uc11c \ube0c\ub9ac\ud2b8\ub2c8\ub294 \ub450 \ubc88\uc9f8 \uc74c\ubc18\uc758 \uc218\ub85d\uace1\ub4e4\uc744 \uc120\ubcf4\uc600\ub2e4. 2000\ub144 5\uc6d4, \ube0c\ub9ac\ud2b8\ub2c8\uc758 \ub450 \ubc88\uc9f8 \uc815\uaddc \uc74c\ubc18 Oops!...I Did It Again\uc774 \ubc1c\ub9e4\ub41c\ub2e4. \uc774 \uc74c\ubc18\uc740 \ubbf8\uad6d\uc5d0\uc11c \ubc1c\ub9e4 \uccab \uc8fc 1,300,000\ub9cc \uc7a5\uc744 \ud314\uc544\uce58\uc6b0\uba70 \uc0ac\uc6b4\ub4dc\uc2a4\uce94 \uc5ed\uc0ac\uc0c1 \uc194\ub85c \uac00\uc218 \uc911 \uac00\uc7a5 \ub192\uc740 \uccab \uc8fc \ud310\ub9e4\ub7c9\uc774\ub77c\ub294 \uae30\ub85d\uc744 \uc138\uc6e0\ub2e4. \uc568\ubc94\uc740 \uc138\uacc4\uc801\uc73c\ub85c 2,000\ub9cc \uc7a5 \uc774\uc0c1\uc758 \ud310\ub9e4\uace0\ub97c \uc62c\ub838\ub2e4. \uc74c\ubc18\uc758 \ub9ac\ub4dc \uc2f1\uae00 \"Oops! ... I Did It Again\"\uc740 \ud638\uc8fc, \ub274\uc9c8\ub79c\ub4dc, \uc601\uad6d \ubc0f \uc5ec\ub7ec \uc720\ub7fd \uad6d\uac00\uc5d0\uc11c \uc815\uc0c1\uc744 \ucc28\uc9c0\ud588\ub2e4. \uc774 \ub178\ub798\ub294 \uadf8\ub798\ubbf8\uc0c1 \ucd5c\uc6b0\uc218 \uc5ec\uc790 \ud31d \ubcf4\uceec \ud37c\ud3ec\uba3c\uc2a4 \ubd80\ubb38\uacfc \uc74c\ubc18\uc740 \ucd5c\uc6b0\uc218 \ud31d \ubcf4\uceec \uc568\ubc94 \ubd80\ubb38\uc5d0 \uac01\uac01 \ud6c4\ubcf4\ub85c \uc62c\ub790\uc9c0\ub9cc, \uc218\uc0c1\uc740 \ud558\uc9c0 \ubabb\ud588\ub2e4.", "paragraph_answer": " 1999\ub144 6\uc6d4 28\uc77c , \ube0c\ub9ac\ud2b8\ub2c8\uc758 \uccab \ubc88\uc9f8 \ud22c\uc5b4\uc778 ...Baby One More Time Tour\uac00 \ubd81\uc544\uba54\ub9ac\uce74\uc5d0\uc11c \uc2dc\uc791\ub418\uc5c8\uc744 \ub54c \ube44\ud3c9\uac00\ub4e4\uc740 \ud638\ud3c9\uc744 \ub0b4\ub193\uc558\uc73c\ub098, \ud55c\ud3b8\uc5d0\uc120 \uc120\uc815\uc801\uc778 \uc758\uc0c1\uc73c\ub85c \ube44\ud310\ud558\uae30\ub3c4 \ud588\ub2e4. 2000\ub144 \ud22c\uc5b4\uac00 \ub9c9\ubc14\uc9c0\uc5d0 \uc774\ub974\ub800\uc744 \ub54c, Crazy 2k\ub77c\ub294 \uc81c\ubaa9\uc73c\ub85c \ud22c\uc5b4\ub97c \uc774\uc5b4\ub098\uac14\ub2e4. \uc774 \ud22c\uc5b4\uc5d0\uc11c \ube0c\ub9ac\ud2b8\ub2c8\ub294 \ub450 \ubc88\uc9f8 \uc74c\ubc18\uc758 \uc218\ub85d\uace1\ub4e4\uc744 \uc120\ubcf4\uc600\ub2e4. 2000\ub144 5\uc6d4, \ube0c\ub9ac\ud2b8\ub2c8\uc758 \ub450 \ubc88\uc9f8 \uc815\uaddc \uc74c\ubc18 Oops!...I Did It Again\uc774 \ubc1c\ub9e4\ub41c\ub2e4. \uc774 \uc74c\ubc18\uc740 \ubbf8\uad6d\uc5d0\uc11c \ubc1c\ub9e4 \uccab \uc8fc 1,300,000\ub9cc \uc7a5\uc744 \ud314\uc544\uce58\uc6b0\uba70 \uc0ac\uc6b4\ub4dc\uc2a4\uce94 \uc5ed\uc0ac\uc0c1 \uc194\ub85c \uac00\uc218 \uc911 \uac00\uc7a5 \ub192\uc740 \uccab \uc8fc \ud310\ub9e4\ub7c9\uc774\ub77c\ub294 \uae30\ub85d\uc744 \uc138\uc6e0\ub2e4. \uc568\ubc94\uc740 \uc138\uacc4\uc801\uc73c\ub85c 2,000\ub9cc \uc7a5 \uc774\uc0c1\uc758 \ud310\ub9e4\uace0\ub97c \uc62c\ub838\ub2e4. \uc74c\ubc18\uc758 \ub9ac\ub4dc \uc2f1\uae00 \"Oops!... I Did It Again\"\uc740 \ud638\uc8fc, \ub274\uc9c8\ub79c\ub4dc, \uc601\uad6d \ubc0f \uc5ec\ub7ec \uc720\ub7fd \uad6d\uac00\uc5d0\uc11c \uc815\uc0c1\uc744 \ucc28\uc9c0\ud588\ub2e4. \uc774 \ub178\ub798\ub294 \uadf8\ub798\ubbf8\uc0c1 \ucd5c\uc6b0\uc218 \uc5ec\uc790 \ud31d \ubcf4\uceec \ud37c\ud3ec\uba3c\uc2a4 \ubd80\ubb38\uacfc \uc74c\ubc18\uc740 \ucd5c\uc6b0\uc218 \ud31d \ubcf4\uceec \uc568\ubc94 \ubd80\ubb38\uc5d0 \uac01\uac01 \ud6c4\ubcf4\ub85c \uc62c\ub790\uc9c0\ub9cc, \uc218\uc0c1\uc740 \ud558\uc9c0 \ubabb\ud588\ub2e4.", "sentence_answer": " 1999\ub144 6\uc6d4 28\uc77c ,", "paragraph_id": "6509455-6-0", "paragraph_question": "question: \ube0c\ub9ac\ud2b8\ub2c8\uc758 \uccab \ubc88\uc9f8 \ud22c\uc5b4\uac00 \uc2dc\uc791\ub41c \ub0a0\uc740?, context: 1999\ub144 6\uc6d4 28\uc77c, \ube0c\ub9ac\ud2b8\ub2c8\uc758 \uccab \ubc88\uc9f8 \ud22c\uc5b4\uc778 ...Baby One More Time Tour\uac00 \ubd81\uc544\uba54\ub9ac\uce74\uc5d0\uc11c \uc2dc\uc791\ub418\uc5c8\uc744 \ub54c \ube44\ud3c9\uac00\ub4e4\uc740 \ud638\ud3c9\uc744 \ub0b4\ub193\uc558\uc73c\ub098, \ud55c\ud3b8\uc5d0\uc120 \uc120\uc815\uc801\uc778 \uc758\uc0c1\uc73c\ub85c \ube44\ud310\ud558\uae30\ub3c4 \ud588\ub2e4. 2000\ub144 \ud22c\uc5b4\uac00 \ub9c9\ubc14\uc9c0\uc5d0 \uc774\ub974\ub800\uc744 \ub54c, Crazy 2k\ub77c\ub294 \uc81c\ubaa9\uc73c\ub85c \ud22c\uc5b4\ub97c \uc774\uc5b4\ub098\uac14\ub2e4. \uc774 \ud22c\uc5b4\uc5d0\uc11c \ube0c\ub9ac\ud2b8\ub2c8\ub294 \ub450 \ubc88\uc9f8 \uc74c\ubc18\uc758 \uc218\ub85d\uace1\ub4e4\uc744 \uc120\ubcf4\uc600\ub2e4. 2000\ub144 5\uc6d4, \ube0c\ub9ac\ud2b8\ub2c8\uc758 \ub450 \ubc88\uc9f8 \uc815\uaddc \uc74c\ubc18 Oops!...I Did It Again\uc774 \ubc1c\ub9e4\ub41c\ub2e4. \uc774 \uc74c\ubc18\uc740 \ubbf8\uad6d\uc5d0\uc11c \ubc1c\ub9e4 \uccab \uc8fc 1,300,000\ub9cc \uc7a5\uc744 \ud314\uc544\uce58\uc6b0\uba70 \uc0ac\uc6b4\ub4dc\uc2a4\uce94 \uc5ed\uc0ac\uc0c1 \uc194\ub85c \uac00\uc218 \uc911 \uac00\uc7a5 \ub192\uc740 \uccab \uc8fc \ud310\ub9e4\ub7c9\uc774\ub77c\ub294 \uae30\ub85d\uc744 \uc138\uc6e0\ub2e4. \uc568\ubc94\uc740 \uc138\uacc4\uc801\uc73c\ub85c 2,000\ub9cc \uc7a5 \uc774\uc0c1\uc758 \ud310\ub9e4\uace0\ub97c \uc62c\ub838\ub2e4. \uc74c\ubc18\uc758 \ub9ac\ub4dc \uc2f1\uae00 \"Oops!... I Did It Again\"\uc740 \ud638\uc8fc, \ub274\uc9c8\ub79c\ub4dc, \uc601\uad6d \ubc0f \uc5ec\ub7ec \uc720\ub7fd \uad6d\uac00\uc5d0\uc11c \uc815\uc0c1\uc744 \ucc28\uc9c0\ud588\ub2e4. \uc774 \ub178\ub798\ub294 \uadf8\ub798\ubbf8\uc0c1 \ucd5c\uc6b0\uc218 \uc5ec\uc790 \ud31d \ubcf4\uceec \ud37c\ud3ec\uba3c\uc2a4 \ubd80\ubb38\uacfc \uc74c\ubc18\uc740 \ucd5c\uc6b0\uc218 \ud31d \ubcf4\uceec \uc568\ubc94 \ubd80\ubb38\uc5d0 \uac01\uac01 \ud6c4\ubcf4\ub85c \uc62c\ub790\uc9c0\ub9cc, \uc218\uc0c1\uc740 \ud558\uc9c0 \ubabb\ud588\ub2e4."} {"question": "\ubb38\uc815\uc778\uc740 \uc5bc\ub9c8\ub098 \ub9ce\uc740 \uad6d,\uc601\ubb38 \uc800\uc11c\ub97c \ud3b8\uc800\ud588\ub098\uc694?", "paragraph": "\u201cThe Future of East Asia\u201d (Palgrave-Macmillan, 2018) co-edited with Peter Hayes, \u201dDenuclearization and peace on the Korean Peninsula\u201c(Yonsei University Press, 2015), co-edited with Peter Hayes. \u201d\ub2e4\ubcf4\uc2a4 \uc774\uc57c\uae30\u201c (\uc640\uc774\uc988\ubc30\ub9ac 2014), \u201c\uc77c\ubcf8\uc740 \uc9c0\uae08 \ubb34\uc5c7\uc744 \uc0dd\uac01\ud558\ub294\uac00?\"(\ud55c\uad6d\ud310 \uc0bc\uc131\uacbd\uc81c\uc5f0\uad6c\uc18c 2013\ub144, \uc911\uad6d\ud310 \uc0ac\ud68c\uacfc\ud559\uc6d0 \ubb38\ud5cc\ucd9c\ud310\uc0ac 2017), \u201cThe Sunshine Policy\"(Yonsei University Press, 2012\ub144)\uc640 \"\uc911\uad6d\uc758 \ub0b4\uc77c\uc744 \ubb3b\ub2e4\" (\ud55c\uad6d\ud310 \uc0bc\uc131\uacbd\uc81c\uc5f0\uad6c\uc18c 2010\ub144, \uc911\uad6d\ud310 \uc138\uacc4\uc9c0\uc2dd\ucd9c\ud310\uc0ac 2012\ub144) \ub4f1 60\uad8c\uc758 \uad6d, \uc601\ubb38 \uc800\uc11c\uc640 \ud3b8\uc800\uac00 \uc788\uace0 World Politics, International Studies Quarterly, World Development \ub4f1 \uad6d\uc81c \uc800\uba85 \ud559\uc220\uc9c0\uc640 \uac01\uc885 \ub17c\ubb38\uc9d1\uc5d0 300\uc5ec \ud3b8\uc758 \ud559\uc220 \ub17c\ubb38\uc744 \uac8c\uc7ac\ud55c \ubc14 \uc788\ub2e4.", "answer": "60\uad8c", "sentence": "\u201d\ub2e4\ubcf4\uc2a4 \uc774\uc57c\uae30\u201c (\uc640\uc774\uc988\ubc30\ub9ac 2014), \u201c\uc77c\ubcf8\uc740 \uc9c0\uae08 \ubb34\uc5c7\uc744 \uc0dd\uac01\ud558\ub294\uac00?\"(\ud55c\uad6d\ud310 \uc0bc\uc131\uacbd\uc81c\uc5f0\uad6c\uc18c 2013\ub144, \uc911\uad6d\ud310 \uc0ac\ud68c\uacfc\ud559\uc6d0 \ubb38\ud5cc\ucd9c\ud310\uc0ac 2017), \u201cThe Sunshine Policy\"(Yonsei University Press, 2012\ub144)\uc640 \"\uc911\uad6d\uc758 \ub0b4\uc77c\uc744 \ubb3b\ub2e4\" (\ud55c\uad6d\ud310 \uc0bc\uc131\uacbd\uc81c\uc5f0\uad6c\uc18c 2010\ub144, \uc911\uad6d\ud310 \uc138\uacc4\uc9c0\uc2dd\ucd9c\ud310\uc0ac 2012\ub144) \ub4f1 60\uad8c \uc758 \uad6d, \uc601\ubb38 \uc800\uc11c\uc640 \ud3b8\uc800\uac00 \uc788\uace0 World Politics, International Studies Quarterly, World Development \ub4f1 \uad6d\uc81c \uc800\uba85 \ud559\uc220\uc9c0\uc640 \uac01\uc885 \ub17c\ubb38\uc9d1\uc5d0 300\uc5ec \ud3b8\uc758 \ud559\uc220 \ub17c\ubb38\uc744 \uac8c\uc7ac\ud55c \ubc14 \uc788\ub2e4.", "paragraph_sentence": "\u201cThe Future of East Asia\u201d (Palgrave-Macmillan, 2018) co-edited with Peter Hayes, \u201dDenuclearization and peace on the Korean Peninsula\u201c(Yonsei University Press, 2015), co-edited with Peter Hayes. \u201d\ub2e4\ubcf4\uc2a4 \uc774\uc57c\uae30\u201c (\uc640\uc774\uc988\ubc30\ub9ac 2014), \u201c\uc77c\ubcf8\uc740 \uc9c0\uae08 \ubb34\uc5c7\uc744 \uc0dd\uac01\ud558\ub294\uac00?\"(\ud55c\uad6d\ud310 \uc0bc\uc131\uacbd\uc81c\uc5f0\uad6c\uc18c 2013\ub144, \uc911\uad6d\ud310 \uc0ac\ud68c\uacfc\ud559\uc6d0 \ubb38\ud5cc\ucd9c\ud310\uc0ac 2017), \u201cThe Sunshine Policy\"(Yonsei University Press, 2012\ub144)\uc640 \"\uc911\uad6d\uc758 \ub0b4\uc77c\uc744 \ubb3b\ub2e4\" (\ud55c\uad6d\ud310 \uc0bc\uc131\uacbd\uc81c\uc5f0\uad6c\uc18c 2010\ub144, \uc911\uad6d\ud310 \uc138\uacc4\uc9c0\uc2dd\ucd9c\ud310\uc0ac 2012\ub144) \ub4f1 60\uad8c \uc758 \uad6d, \uc601\ubb38 \uc800\uc11c\uc640 \ud3b8\uc800\uac00 \uc788\uace0 World Politics, International Studies Quarterly, World Development \ub4f1 \uad6d\uc81c \uc800\uba85 \ud559\uc220\uc9c0\uc640 \uac01\uc885 \ub17c\ubb38\uc9d1\uc5d0 300\uc5ec \ud3b8\uc758 \ud559\uc220 \ub17c\ubb38\uc744 \uac8c\uc7ac\ud55c \ubc14 \uc788\ub2e4. ", "paragraph_answer": "\u201cThe Future of East Asia\u201d (Palgrave-Macmillan, 2018) co-edited with Peter Hayes, \u201dDenuclearization and peace on the Korean Peninsula\u201c(Yonsei University Press, 2015), co-edited with Peter Hayes. \u201d\ub2e4\ubcf4\uc2a4 \uc774\uc57c\uae30\u201c (\uc640\uc774\uc988\ubc30\ub9ac 2014), \u201c\uc77c\ubcf8\uc740 \uc9c0\uae08 \ubb34\uc5c7\uc744 \uc0dd\uac01\ud558\ub294\uac00?\"(\ud55c\uad6d\ud310 \uc0bc\uc131\uacbd\uc81c\uc5f0\uad6c\uc18c 2013\ub144, \uc911\uad6d\ud310 \uc0ac\ud68c\uacfc\ud559\uc6d0 \ubb38\ud5cc\ucd9c\ud310\uc0ac 2017), \u201cThe Sunshine Policy\"(Yonsei University Press, 2012\ub144)\uc640 \"\uc911\uad6d\uc758 \ub0b4\uc77c\uc744 \ubb3b\ub2e4\" (\ud55c\uad6d\ud310 \uc0bc\uc131\uacbd\uc81c\uc5f0\uad6c\uc18c 2010\ub144, \uc911\uad6d\ud310 \uc138\uacc4\uc9c0\uc2dd\ucd9c\ud310\uc0ac 2012\ub144) \ub4f1 60\uad8c \uc758 \uad6d, \uc601\ubb38 \uc800\uc11c\uc640 \ud3b8\uc800\uac00 \uc788\uace0 World Politics, International Studies Quarterly, World Development \ub4f1 \uad6d\uc81c \uc800\uba85 \ud559\uc220\uc9c0\uc640 \uac01\uc885 \ub17c\ubb38\uc9d1\uc5d0 300\uc5ec \ud3b8\uc758 \ud559\uc220 \ub17c\ubb38\uc744 \uac8c\uc7ac\ud55c \ubc14 \uc788\ub2e4.", "sentence_answer": "\u201d\ub2e4\ubcf4\uc2a4 \uc774\uc57c\uae30\u201c (\uc640\uc774\uc988\ubc30\ub9ac 2014), \u201c\uc77c\ubcf8\uc740 \uc9c0\uae08 \ubb34\uc5c7\uc744 \uc0dd\uac01\ud558\ub294\uac00?\"(\ud55c\uad6d\ud310 \uc0bc\uc131\uacbd\uc81c\uc5f0\uad6c\uc18c 2013\ub144, \uc911\uad6d\ud310 \uc0ac\ud68c\uacfc\ud559\uc6d0 \ubb38\ud5cc\ucd9c\ud310\uc0ac 2017), \u201cThe Sunshine Policy\"(Yonsei University Press, 2012\ub144)\uc640 \"\uc911\uad6d\uc758 \ub0b4\uc77c\uc744 \ubb3b\ub2e4\" (\ud55c\uad6d\ud310 \uc0bc\uc131\uacbd\uc81c\uc5f0\uad6c\uc18c 2010\ub144, \uc911\uad6d\ud310 \uc138\uacc4\uc9c0\uc2dd\ucd9c\ud310\uc0ac 2012\ub144) \ub4f1 60\uad8c \uc758 \uad6d, \uc601\ubb38 \uc800\uc11c\uc640 \ud3b8\uc800\uac00 \uc788\uace0 World Politics, International Studies Quarterly, World Development \ub4f1 \uad6d\uc81c \uc800\uba85 \ud559\uc220\uc9c0\uc640 \uac01\uc885 \ub17c\ubb38\uc9d1\uc5d0 300\uc5ec \ud3b8\uc758 \ud559\uc220 \ub17c\ubb38\uc744 \uac8c\uc7ac\ud55c \ubc14 \uc788\ub2e4.", "paragraph_id": "6090144-2-1", "paragraph_question": "question: \ubb38\uc815\uc778\uc740 \uc5bc\ub9c8\ub098 \ub9ce\uc740 \uad6d,\uc601\ubb38 \uc800\uc11c\ub97c \ud3b8\uc800\ud588\ub098\uc694?, context: \u201cThe Future of East Asia\u201d (Palgrave-Macmillan, 2018) co-edited with Peter Hayes, \u201dDenuclearization and peace on the Korean Peninsula\u201c(Yonsei University Press, 2015), co-edited with Peter Hayes. \u201d\ub2e4\ubcf4\uc2a4 \uc774\uc57c\uae30\u201c (\uc640\uc774\uc988\ubc30\ub9ac 2014), \u201c\uc77c\ubcf8\uc740 \uc9c0\uae08 \ubb34\uc5c7\uc744 \uc0dd\uac01\ud558\ub294\uac00?\"(\ud55c\uad6d\ud310 \uc0bc\uc131\uacbd\uc81c\uc5f0\uad6c\uc18c 2013\ub144, \uc911\uad6d\ud310 \uc0ac\ud68c\uacfc\ud559\uc6d0 \ubb38\ud5cc\ucd9c\ud310\uc0ac 2017), \u201cThe Sunshine Policy\"(Yonsei University Press, 2012\ub144)\uc640 \"\uc911\uad6d\uc758 \ub0b4\uc77c\uc744 \ubb3b\ub2e4\" (\ud55c\uad6d\ud310 \uc0bc\uc131\uacbd\uc81c\uc5f0\uad6c\uc18c 2010\ub144, \uc911\uad6d\ud310 \uc138\uacc4\uc9c0\uc2dd\ucd9c\ud310\uc0ac 2012\ub144) \ub4f1 60\uad8c\uc758 \uad6d, \uc601\ubb38 \uc800\uc11c\uc640 \ud3b8\uc800\uac00 \uc788\uace0 World Politics, International Studies Quarterly, World Development \ub4f1 \uad6d\uc81c \uc800\uba85 \ud559\uc220\uc9c0\uc640 \uac01\uc885 \ub17c\ubb38\uc9d1\uc5d0 300\uc5ec \ud3b8\uc758 \ud559\uc220 \ub17c\ubb38\uc744 \uac8c\uc7ac\ud55c \ubc14 \uc788\ub2e4."} {"question": "\uc2a4\ud0e0\ub9ac \uc564\uc11c\ub2c8 \ucf54\ubca8\ub808\uc2a4\ud0a4\uac00 MLB\uc5d0\uc11c \ud65c\uc57d\ud55c \ud3ec\uc9c0\uc158\uc740?", "paragraph": "\uc2a4\ud0e0\ub9ac \uc564\uc11c\ub2c8 \ucf54\ubca8\ub808\uc2a4\ud0a4(\uc601\uc5b4: Stanley Anthony Coveleski, \uc6d0\ub798 \uc774\ub984\uc740 \uc2a4\ud130\ub2c8\uc2ac\ub85c\uc2a4 \ucf54\uc648\ub808\ube0c\uc2a4\ud0a4(\uc601\uc5b4: Stanislaus Kowalewski), 1889\ub144 7\uc6d4 13\uc77c ~ 1984\ub144 3\uc6d4 20\uc77c)\ub294 \ubbf8\uad6d\uc758 \uc804 \ud504\ub85c \uc57c\uad6c \uc120\uc218\ub85c, 1910\ub144\ub300\uc640 20\ub144\ub300\uc5d0 \uba54\uc774\uc800 \ub9ac\uadf8 \ubca0\uc774\uc2a4\ubcfc(MLB)\uc5d0\uc11c \uc2a4\ud54f\ubcfc \ud22c\uc218\ub85c \ud65c\uc57d\ud558\uc600\ub2e4. 14\uc2dc\uc98c \ub3d9\uc548 \uc544\uba54\ub9ac\uce78 \ub9ac\uadf8(AL)\uc758 \ud544\ub77c\ub378\ud53c\uc544 \uc560\uc2ac\ub808\ud2f1\uc2a4, \ud074\ub9ac\ube14\ub79c\ub4dc \uc778\ub514\uc5b8\uc2a4, \uc6cc\uc2f1\ud134 \uc138\ub108\ud130\uc2a4, \ub274\uc695 \uc591\ud0a4\uc2a4\uc5d0\uc11c \ub6f0\uc5c8\ub2e4. \ud1b5\uc0b0 450\uacbd\uae30\uc5d0 \ucd9c\uc804\ud574 3,082\uc774\ub2dd\uc744 \ub358\uc9c0\uba70 215\uc2b9 142\ud328, 224\uc644\ud22c, 38\uc644\ubd09, 2.89\uc758 \ud3c9\uade0\uc790\ucc45\uc810\uc744 \uae30\ub85d\ud588\ub2e4. 1969\ub144 \uc57c\uad6c \uba85\uc608\uc758 \uc804\ub2f9\uc5d0 \uc785\uc131\ud588\ub2e4.", "answer": "\uc2a4\ud54f\ubcfc \ud22c\uc218", "sentence": "\uc2a4\ud0e0\ub9ac \uc564\uc11c\ub2c8 \ucf54\ubca8\ub808\uc2a4\ud0a4(\uc601\uc5b4: Stanley Anthony Coveleski, \uc6d0\ub798 \uc774\ub984\uc740 \uc2a4\ud130\ub2c8\uc2ac\ub85c\uc2a4 \ucf54\uc648\ub808\ube0c\uc2a4\ud0a4(\uc601\uc5b4: Stanislaus Kowalewski), 1889\ub144 7\uc6d4 13\uc77c ~ 1984\ub144 3\uc6d4 20\uc77c)\ub294 \ubbf8\uad6d\uc758 \uc804 \ud504\ub85c \uc57c\uad6c \uc120\uc218\ub85c, 1910\ub144\ub300\uc640 20\ub144\ub300\uc5d0 \uba54\uc774\uc800 \ub9ac\uadf8 \ubca0\uc774\uc2a4\ubcfc(MLB)\uc5d0\uc11c \uc2a4\ud54f\ubcfc \ud22c\uc218 \ub85c \ud65c\uc57d\ud558\uc600\ub2e4.", "paragraph_sentence": " \uc2a4\ud0e0\ub9ac \uc564\uc11c\ub2c8 \ucf54\ubca8\ub808\uc2a4\ud0a4(\uc601\uc5b4: Stanley Anthony Coveleski, \uc6d0\ub798 \uc774\ub984\uc740 \uc2a4\ud130\ub2c8\uc2ac\ub85c\uc2a4 \ucf54\uc648\ub808\ube0c\uc2a4\ud0a4(\uc601\uc5b4: Stanislaus Kowalewski), 1889\ub144 7\uc6d4 13\uc77c ~ 1984\ub144 3\uc6d4 20\uc77c)\ub294 \ubbf8\uad6d\uc758 \uc804 \ud504\ub85c \uc57c\uad6c \uc120\uc218\ub85c, 1910\ub144\ub300\uc640 20\ub144\ub300\uc5d0 \uba54\uc774\uc800 \ub9ac\uadf8 \ubca0\uc774\uc2a4\ubcfc(MLB)\uc5d0\uc11c \uc2a4\ud54f\ubcfc \ud22c\uc218 \ub85c \ud65c\uc57d\ud558\uc600\ub2e4. 14\uc2dc\uc98c \ub3d9\uc548 \uc544\uba54\ub9ac\uce78 \ub9ac\uadf8(AL)\uc758 \ud544\ub77c\ub378\ud53c\uc544 \uc560\uc2ac\ub808\ud2f1\uc2a4, \ud074\ub9ac\ube14\ub79c\ub4dc \uc778\ub514\uc5b8\uc2a4, \uc6cc\uc2f1\ud134 \uc138\ub108\ud130\uc2a4, \ub274\uc695 \uc591\ud0a4\uc2a4\uc5d0\uc11c \ub6f0\uc5c8\ub2e4. \ud1b5\uc0b0 450\uacbd\uae30\uc5d0 \ucd9c\uc804\ud574 3,082\uc774\ub2dd\uc744 \ub358\uc9c0\uba70 215\uc2b9 142\ud328, 224\uc644\ud22c, 38\uc644\ubd09, 2.89\uc758 \ud3c9\uade0\uc790\ucc45\uc810\uc744 \uae30\ub85d\ud588\ub2e4. 1969\ub144 \uc57c\uad6c \uba85\uc608\uc758 \uc804\ub2f9\uc5d0 \uc785\uc131\ud588\ub2e4.", "paragraph_answer": "\uc2a4\ud0e0\ub9ac \uc564\uc11c\ub2c8 \ucf54\ubca8\ub808\uc2a4\ud0a4(\uc601\uc5b4: Stanley Anthony Coveleski, \uc6d0\ub798 \uc774\ub984\uc740 \uc2a4\ud130\ub2c8\uc2ac\ub85c\uc2a4 \ucf54\uc648\ub808\ube0c\uc2a4\ud0a4(\uc601\uc5b4: Stanislaus Kowalewski), 1889\ub144 7\uc6d4 13\uc77c ~ 1984\ub144 3\uc6d4 20\uc77c)\ub294 \ubbf8\uad6d\uc758 \uc804 \ud504\ub85c \uc57c\uad6c \uc120\uc218\ub85c, 1910\ub144\ub300\uc640 20\ub144\ub300\uc5d0 \uba54\uc774\uc800 \ub9ac\uadf8 \ubca0\uc774\uc2a4\ubcfc(MLB)\uc5d0\uc11c \uc2a4\ud54f\ubcfc \ud22c\uc218 \ub85c \ud65c\uc57d\ud558\uc600\ub2e4. 14\uc2dc\uc98c \ub3d9\uc548 \uc544\uba54\ub9ac\uce78 \ub9ac\uadf8(AL)\uc758 \ud544\ub77c\ub378\ud53c\uc544 \uc560\uc2ac\ub808\ud2f1\uc2a4, \ud074\ub9ac\ube14\ub79c\ub4dc \uc778\ub514\uc5b8\uc2a4, \uc6cc\uc2f1\ud134 \uc138\ub108\ud130\uc2a4, \ub274\uc695 \uc591\ud0a4\uc2a4\uc5d0\uc11c \ub6f0\uc5c8\ub2e4. \ud1b5\uc0b0 450\uacbd\uae30\uc5d0 \ucd9c\uc804\ud574 3,082\uc774\ub2dd\uc744 \ub358\uc9c0\uba70 215\uc2b9 142\ud328, 224\uc644\ud22c, 38\uc644\ubd09, 2.89\uc758 \ud3c9\uade0\uc790\ucc45\uc810\uc744 \uae30\ub85d\ud588\ub2e4. 1969\ub144 \uc57c\uad6c \uba85\uc608\uc758 \uc804\ub2f9\uc5d0 \uc785\uc131\ud588\ub2e4.", "sentence_answer": "\uc2a4\ud0e0\ub9ac \uc564\uc11c\ub2c8 \ucf54\ubca8\ub808\uc2a4\ud0a4(\uc601\uc5b4: Stanley Anthony Coveleski, \uc6d0\ub798 \uc774\ub984\uc740 \uc2a4\ud130\ub2c8\uc2ac\ub85c\uc2a4 \ucf54\uc648\ub808\ube0c\uc2a4\ud0a4(\uc601\uc5b4: Stanislaus Kowalewski), 1889\ub144 7\uc6d4 13\uc77c ~ 1984\ub144 3\uc6d4 20\uc77c)\ub294 \ubbf8\uad6d\uc758 \uc804 \ud504\ub85c \uc57c\uad6c \uc120\uc218\ub85c, 1910\ub144\ub300\uc640 20\ub144\ub300\uc5d0 \uba54\uc774\uc800 \ub9ac\uadf8 \ubca0\uc774\uc2a4\ubcfc(MLB)\uc5d0\uc11c \uc2a4\ud54f\ubcfc \ud22c\uc218 \ub85c \ud65c\uc57d\ud558\uc600\ub2e4.", "paragraph_id": "6457302-0-1", "paragraph_question": "question: \uc2a4\ud0e0\ub9ac \uc564\uc11c\ub2c8 \ucf54\ubca8\ub808\uc2a4\ud0a4\uac00 MLB\uc5d0\uc11c \ud65c\uc57d\ud55c \ud3ec\uc9c0\uc158\uc740?, context: \uc2a4\ud0e0\ub9ac \uc564\uc11c\ub2c8 \ucf54\ubca8\ub808\uc2a4\ud0a4(\uc601\uc5b4: Stanley Anthony Coveleski, \uc6d0\ub798 \uc774\ub984\uc740 \uc2a4\ud130\ub2c8\uc2ac\ub85c\uc2a4 \ucf54\uc648\ub808\ube0c\uc2a4\ud0a4(\uc601\uc5b4: Stanislaus Kowalewski), 1889\ub144 7\uc6d4 13\uc77c ~ 1984\ub144 3\uc6d4 20\uc77c)\ub294 \ubbf8\uad6d\uc758 \uc804 \ud504\ub85c \uc57c\uad6c \uc120\uc218\ub85c, 1910\ub144\ub300\uc640 20\ub144\ub300\uc5d0 \uba54\uc774\uc800 \ub9ac\uadf8 \ubca0\uc774\uc2a4\ubcfc(MLB)\uc5d0\uc11c \uc2a4\ud54f\ubcfc \ud22c\uc218\ub85c \ud65c\uc57d\ud558\uc600\ub2e4. 14\uc2dc\uc98c \ub3d9\uc548 \uc544\uba54\ub9ac\uce78 \ub9ac\uadf8(AL)\uc758 \ud544\ub77c\ub378\ud53c\uc544 \uc560\uc2ac\ub808\ud2f1\uc2a4, \ud074\ub9ac\ube14\ub79c\ub4dc \uc778\ub514\uc5b8\uc2a4, \uc6cc\uc2f1\ud134 \uc138\ub108\ud130\uc2a4, \ub274\uc695 \uc591\ud0a4\uc2a4\uc5d0\uc11c \ub6f0\uc5c8\ub2e4. \ud1b5\uc0b0 450\uacbd\uae30\uc5d0 \ucd9c\uc804\ud574 3,082\uc774\ub2dd\uc744 \ub358\uc9c0\uba70 215\uc2b9 142\ud328, 224\uc644\ud22c, 38\uc644\ubd09, 2.89\uc758 \ud3c9\uade0\uc790\ucc45\uc810\uc744 \uae30\ub85d\ud588\ub2e4. 1969\ub144 \uc57c\uad6c \uba85\uc608\uc758 \uc804\ub2f9\uc5d0 \uc785\uc131\ud588\ub2e4."} {"question": "\uc57c\ub9c8\uc790\ud0a4 \ub9c8\uc0ac\uc624 \ubd80\uc0ac\uc7a5\uc774 \uc0ac\uc7a5\uc73c\ub85c \uc2b9\uaca9\ub41c \ub0a0\uc740?", "paragraph": "\uc0ac\uace0\uac00 \ub108\ubb34 \uc2ec\uac01\ud588\uae30 \ub54c\ubb38\uc5d0 \uacbd\uc601\uc9c4\uc758 \uc778\ucc45 \uc0ac\uc784\uc740 \ubd88\uac00\ud53c\ud55c \uac83\uc73c\ub85c \uc5ec\uaca8\uc84c\uc9c0\ub9cc \ud6c4\uacc4\uc778\uc0ac\ub294 \ub09c\ud56d\uc744 \uacaa\uc5c8\ub2e4. \uacb0\uad6d 2006\ub144 2\uc6d4 1\uc77c\uc790\ub85c \ubbf8\ub098\ubbf8\ud0c0\ub2c8 \uc1fc\uc9c0\ub85c\ud68c\uc7a5\uacfc \uce74\ud0a4\uc6b0\uce58 \uce20\uc694\uc2dc \uc0ac\uc7a5\uc774 \ud1f4\uc784\ud558\uace0 \uc57c\ub9c8\uc790\ud0a4 \ub9c8\uc0ac\uc624 \ubd80\uc0ac\uc7a5\uc774 \uc0ac\uc7a5\uc73c\ub85c \uc2b9\uaca9\ub418\uc5c8\ub2e4. \uc678\ubd80(\uc2a4\ubbf8\ud1a0\ubaa8 \uc804\uae30\uacf5\uc5c5)\ub85c\ubd80\ud130 \uace0\ubb38\uc73c\ub85c \uad6c\ub77c\uc6b0\uce58 \ub178\ub9ac\ud0c0\uce74\ub97c \ub9de\uc774\ud558\uac8c \ub418\uc5c8\ub2e4. \ub367\ubd99\uc5ec \uc0c1\ub2f4\uc5ed\uc774\uc790 \uad6d\ucca0 \ubbfc\uc601\ud654\uc758 \uc8fc\uc778\uacf5\uc73c\ub85c\uc11c JR\uc11c\uc77c\ubcf8\uc5d0 \ub300\ud55c \uc601\ud5a5\ub825\uc774 \uac15\ud588\ub358 \uc774\ub370 \ub9c8\uc0ac\ud0c0\uce74\ub3c4 \ubb3c\ub7ec\ub0ac\ub2e4. \uadf8\ub7ec\ub098 \uadf8 \ud6c4\uc5d0 \uc774\ub370 \ub9c8\uc0ac\ud0c0\uce74\ub098 \ub2f9\uc2dc\uc758 \uc624\uc0ac\uce74 \uc9c0\uc0ac\uc7a5\uc778 \ud558\uc2dc\ubaa8\ud1a0 \ud788\uce74\ub8e8 \ub4f1 \uc0ac\uace0\uc5d0 \uc758\ud574\uc11c \uc778\ucc45 \uc0ac\uc784\ud55c 10\uba85\uc774 JR\uc11c\uc77c\ubcf8 \uc790\ud68c\uc0ac\uc758 \uac04\ubd80\ub85c \ub099\ud558\uc0b0 \uc778\uc0ac\ub85c\uc11c \uc784\uba85\ub418\uace0 \uc788\uc5c8\ub358 \uac83\uc774 \ubc1d\ud600\uc838 \uc720\uc871\uc774 \uac15\ud558\uac8c \ubc18\ubc1c\ud588\ub2e4.", "answer": "2006\ub144 2\uc6d4 1\uc77c", "sentence": "\uacb0\uad6d 2006\ub144 2\uc6d4 1\uc77c \uc790\ub85c \ubbf8\ub098\ubbf8\ud0c0\ub2c8 \uc1fc\uc9c0\ub85c\ud68c\uc7a5\uacfc \uce74\ud0a4\uc6b0\uce58 \uce20\uc694\uc2dc \uc0ac\uc7a5\uc774 \ud1f4\uc784\ud558\uace0 \uc57c\ub9c8\uc790\ud0a4 \ub9c8\uc0ac\uc624 \ubd80\uc0ac\uc7a5\uc774 \uc0ac\uc7a5\uc73c\ub85c \uc2b9\uaca9\ub418\uc5c8\ub2e4.", "paragraph_sentence": "\uc0ac\uace0\uac00 \ub108\ubb34 \uc2ec\uac01\ud588\uae30 \ub54c\ubb38\uc5d0 \uacbd\uc601\uc9c4\uc758 \uc778\ucc45 \uc0ac\uc784\uc740 \ubd88\uac00\ud53c\ud55c \uac83\uc73c\ub85c \uc5ec\uaca8\uc84c\uc9c0\ub9cc \ud6c4\uacc4\uc778\uc0ac\ub294 \ub09c\ud56d\uc744 \uacaa\uc5c8\ub2e4. \uacb0\uad6d 2006\ub144 2\uc6d4 1\uc77c \uc790\ub85c \ubbf8\ub098\ubbf8\ud0c0\ub2c8 \uc1fc\uc9c0\ub85c\ud68c\uc7a5\uacfc \uce74\ud0a4\uc6b0\uce58 \uce20\uc694\uc2dc \uc0ac\uc7a5\uc774 \ud1f4\uc784\ud558\uace0 \uc57c\ub9c8\uc790\ud0a4 \ub9c8\uc0ac\uc624 \ubd80\uc0ac\uc7a5\uc774 \uc0ac\uc7a5\uc73c\ub85c \uc2b9\uaca9\ub418\uc5c8\ub2e4. \uc678\ubd80(\uc2a4\ubbf8\ud1a0\ubaa8 \uc804\uae30\uacf5\uc5c5)\ub85c\ubd80\ud130 \uace0\ubb38\uc73c\ub85c \uad6c\ub77c\uc6b0\uce58 \ub178\ub9ac\ud0c0\uce74\ub97c \ub9de\uc774\ud558\uac8c \ub418\uc5c8\ub2e4. \ub367\ubd99\uc5ec \uc0c1\ub2f4\uc5ed\uc774\uc790 \uad6d\ucca0 \ubbfc\uc601\ud654\uc758 \uc8fc\uc778\uacf5\uc73c\ub85c\uc11c JR\uc11c\uc77c\ubcf8\uc5d0 \ub300\ud55c \uc601\ud5a5\ub825\uc774 \uac15\ud588\ub358 \uc774\ub370 \ub9c8\uc0ac\ud0c0\uce74\ub3c4 \ubb3c\ub7ec\ub0ac\ub2e4. \uadf8\ub7ec\ub098 \uadf8 \ud6c4\uc5d0 \uc774\ub370 \ub9c8\uc0ac\ud0c0\uce74\ub098 \ub2f9\uc2dc\uc758 \uc624\uc0ac\uce74 \uc9c0\uc0ac\uc7a5\uc778 \ud558\uc2dc\ubaa8\ud1a0 \ud788\uce74\ub8e8 \ub4f1 \uc0ac\uace0\uc5d0 \uc758\ud574\uc11c \uc778\ucc45 \uc0ac\uc784\ud55c 10\uba85\uc774 JR\uc11c\uc77c\ubcf8 \uc790\ud68c\uc0ac\uc758 \uac04\ubd80\ub85c \ub099\ud558\uc0b0 \uc778\uc0ac\ub85c\uc11c \uc784\uba85\ub418\uace0 \uc788\uc5c8\ub358 \uac83\uc774 \ubc1d\ud600\uc838 \uc720\uc871\uc774 \uac15\ud558\uac8c \ubc18\ubc1c\ud588\ub2e4.", "paragraph_answer": "\uc0ac\uace0\uac00 \ub108\ubb34 \uc2ec\uac01\ud588\uae30 \ub54c\ubb38\uc5d0 \uacbd\uc601\uc9c4\uc758 \uc778\ucc45 \uc0ac\uc784\uc740 \ubd88\uac00\ud53c\ud55c \uac83\uc73c\ub85c \uc5ec\uaca8\uc84c\uc9c0\ub9cc \ud6c4\uacc4\uc778\uc0ac\ub294 \ub09c\ud56d\uc744 \uacaa\uc5c8\ub2e4. \uacb0\uad6d 2006\ub144 2\uc6d4 1\uc77c \uc790\ub85c \ubbf8\ub098\ubbf8\ud0c0\ub2c8 \uc1fc\uc9c0\ub85c\ud68c\uc7a5\uacfc \uce74\ud0a4\uc6b0\uce58 \uce20\uc694\uc2dc \uc0ac\uc7a5\uc774 \ud1f4\uc784\ud558\uace0 \uc57c\ub9c8\uc790\ud0a4 \ub9c8\uc0ac\uc624 \ubd80\uc0ac\uc7a5\uc774 \uc0ac\uc7a5\uc73c\ub85c \uc2b9\uaca9\ub418\uc5c8\ub2e4. \uc678\ubd80(\uc2a4\ubbf8\ud1a0\ubaa8 \uc804\uae30\uacf5\uc5c5)\ub85c\ubd80\ud130 \uace0\ubb38\uc73c\ub85c \uad6c\ub77c\uc6b0\uce58 \ub178\ub9ac\ud0c0\uce74\ub97c \ub9de\uc774\ud558\uac8c \ub418\uc5c8\ub2e4. \ub367\ubd99\uc5ec \uc0c1\ub2f4\uc5ed\uc774\uc790 \uad6d\ucca0 \ubbfc\uc601\ud654\uc758 \uc8fc\uc778\uacf5\uc73c\ub85c\uc11c JR\uc11c\uc77c\ubcf8\uc5d0 \ub300\ud55c \uc601\ud5a5\ub825\uc774 \uac15\ud588\ub358 \uc774\ub370 \ub9c8\uc0ac\ud0c0\uce74\ub3c4 \ubb3c\ub7ec\ub0ac\ub2e4. \uadf8\ub7ec\ub098 \uadf8 \ud6c4\uc5d0 \uc774\ub370 \ub9c8\uc0ac\ud0c0\uce74\ub098 \ub2f9\uc2dc\uc758 \uc624\uc0ac\uce74 \uc9c0\uc0ac\uc7a5\uc778 \ud558\uc2dc\ubaa8\ud1a0 \ud788\uce74\ub8e8 \ub4f1 \uc0ac\uace0\uc5d0 \uc758\ud574\uc11c \uc778\ucc45 \uc0ac\uc784\ud55c 10\uba85\uc774 JR\uc11c\uc77c\ubcf8 \uc790\ud68c\uc0ac\uc758 \uac04\ubd80\ub85c \ub099\ud558\uc0b0 \uc778\uc0ac\ub85c\uc11c \uc784\uba85\ub418\uace0 \uc788\uc5c8\ub358 \uac83\uc774 \ubc1d\ud600\uc838 \uc720\uc871\uc774 \uac15\ud558\uac8c \ubc18\ubc1c\ud588\ub2e4.", "sentence_answer": "\uacb0\uad6d 2006\ub144 2\uc6d4 1\uc77c \uc790\ub85c \ubbf8\ub098\ubbf8\ud0c0\ub2c8 \uc1fc\uc9c0\ub85c\ud68c\uc7a5\uacfc \uce74\ud0a4\uc6b0\uce58 \uce20\uc694\uc2dc \uc0ac\uc7a5\uc774 \ud1f4\uc784\ud558\uace0 \uc57c\ub9c8\uc790\ud0a4 \ub9c8\uc0ac\uc624 \ubd80\uc0ac\uc7a5\uc774 \uc0ac\uc7a5\uc73c\ub85c \uc2b9\uaca9\ub418\uc5c8\ub2e4.", "paragraph_id": "6470376-4-0", "paragraph_question": "question: \uc57c\ub9c8\uc790\ud0a4 \ub9c8\uc0ac\uc624 \ubd80\uc0ac\uc7a5\uc774 \uc0ac\uc7a5\uc73c\ub85c \uc2b9\uaca9\ub41c \ub0a0\uc740?, context: \uc0ac\uace0\uac00 \ub108\ubb34 \uc2ec\uac01\ud588\uae30 \ub54c\ubb38\uc5d0 \uacbd\uc601\uc9c4\uc758 \uc778\ucc45 \uc0ac\uc784\uc740 \ubd88\uac00\ud53c\ud55c \uac83\uc73c\ub85c \uc5ec\uaca8\uc84c\uc9c0\ub9cc \ud6c4\uacc4\uc778\uc0ac\ub294 \ub09c\ud56d\uc744 \uacaa\uc5c8\ub2e4. \uacb0\uad6d 2006\ub144 2\uc6d4 1\uc77c\uc790\ub85c \ubbf8\ub098\ubbf8\ud0c0\ub2c8 \uc1fc\uc9c0\ub85c\ud68c\uc7a5\uacfc \uce74\ud0a4\uc6b0\uce58 \uce20\uc694\uc2dc \uc0ac\uc7a5\uc774 \ud1f4\uc784\ud558\uace0 \uc57c\ub9c8\uc790\ud0a4 \ub9c8\uc0ac\uc624 \ubd80\uc0ac\uc7a5\uc774 \uc0ac\uc7a5\uc73c\ub85c \uc2b9\uaca9\ub418\uc5c8\ub2e4. \uc678\ubd80(\uc2a4\ubbf8\ud1a0\ubaa8 \uc804\uae30\uacf5\uc5c5)\ub85c\ubd80\ud130 \uace0\ubb38\uc73c\ub85c \uad6c\ub77c\uc6b0\uce58 \ub178\ub9ac\ud0c0\uce74\ub97c \ub9de\uc774\ud558\uac8c \ub418\uc5c8\ub2e4. \ub367\ubd99\uc5ec \uc0c1\ub2f4\uc5ed\uc774\uc790 \uad6d\ucca0 \ubbfc\uc601\ud654\uc758 \uc8fc\uc778\uacf5\uc73c\ub85c\uc11c JR\uc11c\uc77c\ubcf8\uc5d0 \ub300\ud55c \uc601\ud5a5\ub825\uc774 \uac15\ud588\ub358 \uc774\ub370 \ub9c8\uc0ac\ud0c0\uce74\ub3c4 \ubb3c\ub7ec\ub0ac\ub2e4. \uadf8\ub7ec\ub098 \uadf8 \ud6c4\uc5d0 \uc774\ub370 \ub9c8\uc0ac\ud0c0\uce74\ub098 \ub2f9\uc2dc\uc758 \uc624\uc0ac\uce74 \uc9c0\uc0ac\uc7a5\uc778 \ud558\uc2dc\ubaa8\ud1a0 \ud788\uce74\ub8e8 \ub4f1 \uc0ac\uace0\uc5d0 \uc758\ud574\uc11c \uc778\ucc45 \uc0ac\uc784\ud55c 10\uba85\uc774 JR\uc11c\uc77c\ubcf8 \uc790\ud68c\uc0ac\uc758 \uac04\ubd80\ub85c \ub099\ud558\uc0b0 \uc778\uc0ac\ub85c\uc11c \uc784\uba85\ub418\uace0 \uc788\uc5c8\ub358 \uac83\uc774 \ubc1d\ud600\uc838 \uc720\uc871\uc774 \uac15\ud558\uac8c \ubc18\ubc1c\ud588\ub2e4."} {"question": "\ub3c4\ub07c\uac00 9\uc6d422\uc77c \ud0c0\uc774\uac70JK\uc640 \ud568\uaed8 \uacb0\uc131\ud55c\uac83\uc740?", "paragraph": "\u300aShow Me The Money 5\u300b \ucd2c\uc601\uc744 \ub9c8\uce5c \ud6c4 7\uc6d4 19\uc77c, \uacbd\uc5f0\uace1\uc774\uc5c8\ub358 \"Beverly 1lls\"\ub97c \ub354\ucf70\uc774\uc5c7\uacfc \ub9ac\ubbf9\uc2a4\ud558\uc5ec \ubc1c\ub9e4\ud558\uc600\uace0 \ubd81\ubbf8 \ud22c\uc5b4 \ud6c4 \uc77c\ub9ac\ub124\uc5b4 \ub808\ucf54\uc988\ub85c\uc11c 5\uc8fc\ub144 \uc5ec\ub984 \ucf58\uc11c\ud2b8 \ubc0f \"ILLIONAIRE EVERYDAY' \uacf5\uac1c\uace1\uc5d0 \ucc38\uc5ec\ud558\uc600\ub2e4. 9\uc6d4 10\uc77c\uc5d0\ub294 \uc2f1\uae00 \"Just 1llin'\"\uc744 \ubc1c\ub9e4\ud558\uc600\ub2e4. 9\uc6d4 22\uc77c\uc5d4 \ud0c0\uc774\uac70JK\uc640 \ud568\uaed8 \uad7f \ub77c\uc774\ud504 \ud06c\ub8e8\ub97c \uacb0\uc131\ud558\uc600\uc73c\uba70, \uc288\ud37c\ube44, \uba74\ub3c4, \uc8fc\ub178\ud50c\ub85c\uac00 \ucc38\uc5ec\ud558\uc600\ub2e4. 29\uc77c\uc5d0\ub294 \uc790\uc2e0\uc758 \uc778\uc2a4\ud0c0\uadf8\ub7a8\uc744 \ud1b5\ud574 \ub354\ucf70\uc774\uc5c7\uacfc \ud568\uaed8 \uc77c\ub9ac\ub124\uc5b4 \ub808\ucf54\uc988\uc758 \ud558\uc704 \ub808\uc774\ube14 \uaca9\uc778 \uc570\ube44\uc158 \ubba4\uc9c1\uc744 \uc124\ub9bd\ud558\uc600\ub2e4\uace0 \ubc1c\ud45c\ud558\uc600\ub2e4. 10\uc6d4 9\uc77c\uc5d0\ub294 \uc570\ube44\uc158 \ubba4\uc9c1 \uc124\ub9bd \uae30\ub150 \ud30c\ud2f0\ub97c \uac00\uc84c\ub2e4. 11\uc6d4 5\uc77c\uc5d0\ub294 \uc790\uc2e0\uc758 \ub2e8\ub3c5 \ucf58\uc11c\ud2b8 \u300aYOUNG KING YOUNG BOSS\u300b\ub97c \uc9c4\ud589\ud558\uc600\uace0 11\uc6d4 12\uc77c\uc5d0\ub294 \u300a\ubb34\ud55c\ub3c4\uc804\u300b \u3008\uc704\ub300\ud55c \uc720\uc0b0\u3009 \ud3b8\uc5d0 \ucd9c\uc5f0, \uc720\uc7ac\uc11d, \uc774\ud558\uc774, \ucc3d\ubaa8\uacfc \ud568\uaed8 \ubb34\ub300\ub97c \uc120\ubcf4\uc600\ub2e4.", "answer": "\uad7f \ub77c\uc774\ud504 \ud06c\ub8e8", "sentence": "9\uc6d4 22\uc77c\uc5d4 \ud0c0\uc774\uac70JK\uc640 \ud568\uaed8 \uad7f \ub77c\uc774\ud504 \ud06c\ub8e8 \ub97c \uacb0\uc131\ud558\uc600\uc73c\uba70, \uc288\ud37c\ube44, \uba74\ub3c4, \uc8fc\ub178\ud50c\ub85c\uac00 \ucc38\uc5ec\ud558\uc600\ub2e4.", "paragraph_sentence": "\u300aShow Me The Money 5\u300b \ucd2c\uc601\uc744 \ub9c8\uce5c \ud6c4 7\uc6d4 19\uc77c, \uacbd\uc5f0\uace1\uc774\uc5c8\ub358 \"Beverly 1lls\"\ub97c \ub354\ucf70\uc774\uc5c7\uacfc \ub9ac\ubbf9\uc2a4\ud558\uc5ec \ubc1c\ub9e4\ud558\uc600\uace0 \ubd81\ubbf8 \ud22c\uc5b4 \ud6c4 \uc77c\ub9ac\ub124\uc5b4 \ub808\ucf54\uc988\ub85c\uc11c 5\uc8fc\ub144 \uc5ec\ub984 \ucf58\uc11c\ud2b8 \ubc0f \"ILLIONAIRE EVERYDAY' \uacf5\uac1c\uace1\uc5d0 \ucc38\uc5ec\ud558\uc600\ub2e4. 9\uc6d4 10\uc77c\uc5d0\ub294 \uc2f1\uae00 \"Just 1llin'\"\uc744 \ubc1c\ub9e4\ud558\uc600\ub2e4. 9\uc6d4 22\uc77c\uc5d4 \ud0c0\uc774\uac70JK\uc640 \ud568\uaed8 \uad7f \ub77c\uc774\ud504 \ud06c\ub8e8 \ub97c \uacb0\uc131\ud558\uc600\uc73c\uba70, \uc288\ud37c\ube44, \uba74\ub3c4, \uc8fc\ub178\ud50c\ub85c\uac00 \ucc38\uc5ec\ud558\uc600\ub2e4. 29\uc77c\uc5d0\ub294 \uc790\uc2e0\uc758 \uc778\uc2a4\ud0c0\uadf8\ub7a8\uc744 \ud1b5\ud574 \ub354\ucf70\uc774\uc5c7\uacfc \ud568\uaed8 \uc77c\ub9ac\ub124\uc5b4 \ub808\ucf54\uc988\uc758 \ud558\uc704 \ub808\uc774\ube14 \uaca9\uc778 \uc570\ube44\uc158 \ubba4\uc9c1\uc744 \uc124\ub9bd\ud558\uc600\ub2e4\uace0 \ubc1c\ud45c\ud558\uc600\ub2e4. 10\uc6d4 9\uc77c\uc5d0\ub294 \uc570\ube44\uc158 \ubba4\uc9c1 \uc124\ub9bd \uae30\ub150 \ud30c\ud2f0\ub97c \uac00\uc84c\ub2e4. 11\uc6d4 5\uc77c\uc5d0\ub294 \uc790\uc2e0\uc758 \ub2e8\ub3c5 \ucf58\uc11c\ud2b8 \u300aYOUNG KING YOUNG BOSS\u300b \ub97c \uc9c4\ud589\ud558\uc600\uace0 11\uc6d4 12\uc77c\uc5d0\ub294 \u300a\ubb34\ud55c\ub3c4\uc804\u300b \u3008\uc704\ub300\ud55c \uc720\uc0b0\u3009 \ud3b8\uc5d0 \ucd9c\uc5f0, \uc720\uc7ac\uc11d, \uc774\ud558\uc774, \ucc3d\ubaa8\uacfc \ud568\uaed8 \ubb34\ub300\ub97c \uc120\ubcf4\uc600\ub2e4.", "paragraph_answer": "\u300aShow Me The Money 5\u300b \ucd2c\uc601\uc744 \ub9c8\uce5c \ud6c4 7\uc6d4 19\uc77c, \uacbd\uc5f0\uace1\uc774\uc5c8\ub358 \"Beverly 1lls\"\ub97c \ub354\ucf70\uc774\uc5c7\uacfc \ub9ac\ubbf9\uc2a4\ud558\uc5ec \ubc1c\ub9e4\ud558\uc600\uace0 \ubd81\ubbf8 \ud22c\uc5b4 \ud6c4 \uc77c\ub9ac\ub124\uc5b4 \ub808\ucf54\uc988\ub85c\uc11c 5\uc8fc\ub144 \uc5ec\ub984 \ucf58\uc11c\ud2b8 \ubc0f \"ILLIONAIRE EVERYDAY' \uacf5\uac1c\uace1\uc5d0 \ucc38\uc5ec\ud558\uc600\ub2e4. 9\uc6d4 10\uc77c\uc5d0\ub294 \uc2f1\uae00 \"Just 1llin'\"\uc744 \ubc1c\ub9e4\ud558\uc600\ub2e4. 9\uc6d4 22\uc77c\uc5d4 \ud0c0\uc774\uac70JK\uc640 \ud568\uaed8 \uad7f \ub77c\uc774\ud504 \ud06c\ub8e8 \ub97c \uacb0\uc131\ud558\uc600\uc73c\uba70, \uc288\ud37c\ube44, \uba74\ub3c4, \uc8fc\ub178\ud50c\ub85c\uac00 \ucc38\uc5ec\ud558\uc600\ub2e4. 29\uc77c\uc5d0\ub294 \uc790\uc2e0\uc758 \uc778\uc2a4\ud0c0\uadf8\ub7a8\uc744 \ud1b5\ud574 \ub354\ucf70\uc774\uc5c7\uacfc \ud568\uaed8 \uc77c\ub9ac\ub124\uc5b4 \ub808\ucf54\uc988\uc758 \ud558\uc704 \ub808\uc774\ube14 \uaca9\uc778 \uc570\ube44\uc158 \ubba4\uc9c1\uc744 \uc124\ub9bd\ud558\uc600\ub2e4\uace0 \ubc1c\ud45c\ud558\uc600\ub2e4. 10\uc6d4 9\uc77c\uc5d0\ub294 \uc570\ube44\uc158 \ubba4\uc9c1 \uc124\ub9bd \uae30\ub150 \ud30c\ud2f0\ub97c \uac00\uc84c\ub2e4. 11\uc6d4 5\uc77c\uc5d0\ub294 \uc790\uc2e0\uc758 \ub2e8\ub3c5 \ucf58\uc11c\ud2b8 \u300aYOUNG KING YOUNG BOSS\u300b\ub97c \uc9c4\ud589\ud558\uc600\uace0 11\uc6d4 12\uc77c\uc5d0\ub294 \u300a\ubb34\ud55c\ub3c4\uc804\u300b \u3008\uc704\ub300\ud55c \uc720\uc0b0\u3009 \ud3b8\uc5d0 \ucd9c\uc5f0, \uc720\uc7ac\uc11d, \uc774\ud558\uc774, \ucc3d\ubaa8\uacfc \ud568\uaed8 \ubb34\ub300\ub97c \uc120\ubcf4\uc600\ub2e4.", "sentence_answer": "9\uc6d4 22\uc77c\uc5d4 \ud0c0\uc774\uac70JK\uc640 \ud568\uaed8 \uad7f \ub77c\uc774\ud504 \ud06c\ub8e8 \ub97c \uacb0\uc131\ud558\uc600\uc73c\uba70, \uc288\ud37c\ube44, \uba74\ub3c4, \uc8fc\ub178\ud50c\ub85c\uac00 \ucc38\uc5ec\ud558\uc600\ub2e4.", "paragraph_id": "6043008-8-0", "paragraph_question": "question: \ub3c4\ub07c\uac00 9\uc6d422\uc77c \ud0c0\uc774\uac70JK\uc640 \ud568\uaed8 \uacb0\uc131\ud55c\uac83\uc740?, context: \u300aShow Me The Money 5\u300b \ucd2c\uc601\uc744 \ub9c8\uce5c \ud6c4 7\uc6d4 19\uc77c, \uacbd\uc5f0\uace1\uc774\uc5c8\ub358 \"Beverly 1lls\"\ub97c \ub354\ucf70\uc774\uc5c7\uacfc \ub9ac\ubbf9\uc2a4\ud558\uc5ec \ubc1c\ub9e4\ud558\uc600\uace0 \ubd81\ubbf8 \ud22c\uc5b4 \ud6c4 \uc77c\ub9ac\ub124\uc5b4 \ub808\ucf54\uc988\ub85c\uc11c 5\uc8fc\ub144 \uc5ec\ub984 \ucf58\uc11c\ud2b8 \ubc0f \"ILLIONAIRE EVERYDAY' \uacf5\uac1c\uace1\uc5d0 \ucc38\uc5ec\ud558\uc600\ub2e4. 9\uc6d4 10\uc77c\uc5d0\ub294 \uc2f1\uae00 \"Just 1llin'\"\uc744 \ubc1c\ub9e4\ud558\uc600\ub2e4. 9\uc6d4 22\uc77c\uc5d4 \ud0c0\uc774\uac70JK\uc640 \ud568\uaed8 \uad7f \ub77c\uc774\ud504 \ud06c\ub8e8\ub97c \uacb0\uc131\ud558\uc600\uc73c\uba70, \uc288\ud37c\ube44, \uba74\ub3c4, \uc8fc\ub178\ud50c\ub85c\uac00 \ucc38\uc5ec\ud558\uc600\ub2e4. 29\uc77c\uc5d0\ub294 \uc790\uc2e0\uc758 \uc778\uc2a4\ud0c0\uadf8\ub7a8\uc744 \ud1b5\ud574 \ub354\ucf70\uc774\uc5c7\uacfc \ud568\uaed8 \uc77c\ub9ac\ub124\uc5b4 \ub808\ucf54\uc988\uc758 \ud558\uc704 \ub808\uc774\ube14 \uaca9\uc778 \uc570\ube44\uc158 \ubba4\uc9c1\uc744 \uc124\ub9bd\ud558\uc600\ub2e4\uace0 \ubc1c\ud45c\ud558\uc600\ub2e4. 10\uc6d4 9\uc77c\uc5d0\ub294 \uc570\ube44\uc158 \ubba4\uc9c1 \uc124\ub9bd \uae30\ub150 \ud30c\ud2f0\ub97c \uac00\uc84c\ub2e4. 11\uc6d4 5\uc77c\uc5d0\ub294 \uc790\uc2e0\uc758 \ub2e8\ub3c5 \ucf58\uc11c\ud2b8 \u300aYOUNG KING YOUNG BOSS\u300b\ub97c \uc9c4\ud589\ud558\uc600\uace0 11\uc6d4 12\uc77c\uc5d0\ub294 \u300a\ubb34\ud55c\ub3c4\uc804\u300b \u3008\uc704\ub300\ud55c \uc720\uc0b0\u3009 \ud3b8\uc5d0 \ucd9c\uc5f0, \uc720\uc7ac\uc11d, \uc774\ud558\uc774, \ucc3d\ubaa8\uacfc \ud568\uaed8 \ubb34\ub300\ub97c \uc120\ubcf4\uc600\ub2e4."} {"question": "\uc544\ub374 \ub9cc \uc5ec\uba85 \uc791\uc804\uc744 \uc704\ud574 \uc5b8\ub860\uc758 \ubcf4\ub3c4 \uc790\uc81c\ub97c \uc694\uccad\ud55c \uc2dc\uae30\ub294 \uc5b8\uc81c\uc778\uac00?", "paragraph": "2011\ub144 1\uc6d4 24\uc77c \uad6d\ubc29\ubd80\ub294 \uc18c\ub9d0\ub9ac\uc544 \ud574\uc801\ub4e4\uc5d0\uac8c \ub0a9\uce58\ub41c \uc0bc\ud638\uc8fc\uc5bc\ub9ac\ud638\uc758 \uc120\uc6d0\ub4e4\uc744 \uad6c\ucd9c\ud558\uae30 \uc704\ud55c \u2018\uc544\ub374 \ub9cc \uc5ec\uba85 \uc791\uc804\u2019\uc744 \uc218\ud589\ud558\uba74\uc11c \uc120\uc6d0\ub4e4\uc758 \uc548\uc804\uc744 \uace0\ub824\ud558\uc5ec \ucd9c\uc785 \uae30\uc790\ub2e8\uc5d0\uac8c \ubcf4\ub3c4\ub97c \uc720\uc608\ud560 \uac83\uc744 \uc694\uccad\ud558\uc600\ub2e4. 1\uc6d4 18\uc77c 1\ucc28 \uad6c\ucd9c \uc791\uc804\uc774 \uc2e4\ud328\ud558\uc790 20\uc77c\uc5d0 \ubd80\uc0b0\uc77c\ubcf4\ub294 \uc5e0\ubc14\uace0\ub97c \ud30c\uae30\ud558\uace0 \ud574\ub2f9 \uc0ac\uc2e4\uc744 \ubcf4\ub3c4\ud558\uc600\uace0 \ubbf8\ub514\uc5b4\uc624\ub298\uc774 \uc774\ub97c \uc778\uc6a9\ud558\uc600\ub2e4. 21\uc77c\uc5d0\ub294 \uc544\uc2dc\uc544\ud22c\ub370\uc774\ub3c4 \ud574\ub2f9 \uc0ac\uc2e4\uc744 \ubcf4\ub3c4\ud558\uc600\ub2e4. \uad6d\ubc29\ubd80\ub294 \ud574\ub2f9 \uae30\uc0ac\uc758 \uc0ad\uc81c\ub97c \uc694\uccad\ud558\uc600\uace0 \ubd80\uc0b0\uc77c\ubcf4\ub294 \uc6f9\uc0ac\uc774\ud2b8\uc5d0 \uac8c\uc7ac\ub41c \uae30\uc0ac\ub97c \uc0ad\uc81c\ud558\uc600\uc73c\ub098 \ubbf8\ub514\uc5b4\uc624\ub298\uacfc \uc544\uc2dc\uc544\ud22c\ub370\uc774\ub294 \uae30\uc0ac\uc758 \uc0ad\uc81c\ub97c \uac70\ubd80\ud558\uc600\ub2e4. \uad6d\ubc29\ubd80\ub294 \uc5e0\ubc14\uace0\ub97c \ud30c\uae30\ud55c \uc5b8\ub860\uc0ac\uc5d0 \uc18c\uc18d\ub41c \uae30\uc790\uc758 \uae30\uc790\uc2e4 \ucd9c\uc785\uc744 \ubb34\uae30\ud55c \uc815\uc9c0\ud558\uc600\uace0 \ubcf4\ub3c4 \uc790\ub8cc \ubc30\ud3ec\ub3c4 \uc911\ub2e8\ud558\uc600\ub2e4. \ub610\ud55c \ub2e4\ub978 \ubd80\ucc98\uc5d0\ub3c4 \uc81c\uc7ac\ub97c \uc694\uccad\ud558\ub294 \uacf5\ubb38\uc744 \ubcf4\ub0c8\ub2e4. \uc774\uc5d0 \uccad\uc640\ub300\ub294 \uc544\uc2dc\uc544\ud22c\ub370\uc774, \ubbf8\ub514\uc5b4\uc624\ub298\uc5d0 \uc18c\uc18d\ub41c \uae30\uc790\uc758 \ucd9c\uc785 \uae30\uc790 \ub4f1\ub85d\uc744 \ucde8\uc18c\ud558\uace0, \ubd80\uc0b0\uc77c\ubcf4\uc5d0 \uc18c\uc18d\ub41c \uae30\uc790\uc758 \uae30\uc790\uc2e4 \ucd9c\uc785\uc744 1\uac1c\uc6d4 \ub3d9\uc548 \uae08\uc9c0\ud558\uc600\ub2e4. \uc544\uc2dc\uc544\ud22c\ub370\uc774\ub294 \uccad\uc640\ub300\uc758 \uc870\uce58\uc5d0 \ubc18\ubc1c\ud558\uc5ec \uc11c\uc6b8\ud589\uc815\ubc95\uc6d0\uc5d0 \ucd9c\uc785 \uae30\uc790 \ub4f1\ub85d \ucde8\uc18c\uc5d0 \ub300\ud55c \ucde8\uc18c \uccad\uad6c\uc640 \ud6a8\ub825 \uc815\uc9c0 \uac00\ucc98\ubd84 \uc2e0\uccad\uc744 \uc81c\uae30\ud558\uc600\ub2e4. \uccad\uc640\ub300\uc640 \uc544\uc2dc\uc544 \ud22c\ub370\uc774\ub294 \ubc95\uc6d0\uc758 \uc870\uc815 \uad8c\uace0\ub97c \uc218\ub77d\ud558\uc5ec \uc544\uc2dc\uc544 \ud22c\ub370\uc774 \uc18c\uc18d \uae30\uc790\uc758 \ucd9c\uc785\uc744 2\uac1c\uc6d4 \ub3d9\uc548 \uc815\uc9c0\ud558\ub294 \uac83\uc73c\ub85c \uac10\uacbd\ud558\uc600\ub2e4.", "answer": "2011\ub144 1\uc6d4 24\uc77c", "sentence": "2011\ub144 1\uc6d4 24\uc77c \uad6d\ubc29\ubd80\ub294 \uc18c\ub9d0\ub9ac\uc544 \ud574\uc801\ub4e4\uc5d0\uac8c \ub0a9\uce58\ub41c \uc0bc\ud638\uc8fc\uc5bc\ub9ac\ud638\uc758 \uc120\uc6d0\ub4e4\uc744 \uad6c\ucd9c\ud558\uae30 \uc704\ud55c \u2018\uc544\ub374 \ub9cc \uc5ec\uba85 \uc791\uc804\u2019\uc744 \uc218\ud589\ud558\uba74\uc11c \uc120\uc6d0\ub4e4\uc758 \uc548\uc804\uc744 \uace0\ub824\ud558\uc5ec \ucd9c\uc785 \uae30\uc790\ub2e8\uc5d0\uac8c \ubcf4\ub3c4\ub97c \uc720\uc608\ud560 \uac83\uc744 \uc694\uccad\ud558\uc600\ub2e4.", "paragraph_sentence": " 2011\ub144 1\uc6d4 24\uc77c \uad6d\ubc29\ubd80\ub294 \uc18c\ub9d0\ub9ac\uc544 \ud574\uc801\ub4e4\uc5d0\uac8c \ub0a9\uce58\ub41c \uc0bc\ud638\uc8fc\uc5bc\ub9ac\ud638\uc758 \uc120\uc6d0\ub4e4\uc744 \uad6c\ucd9c\ud558\uae30 \uc704\ud55c \u2018\uc544\ub374 \ub9cc \uc5ec\uba85 \uc791\uc804\u2019\uc744 \uc218\ud589\ud558\uba74\uc11c \uc120\uc6d0\ub4e4\uc758 \uc548\uc804\uc744 \uace0\ub824\ud558\uc5ec \ucd9c\uc785 \uae30\uc790\ub2e8\uc5d0\uac8c \ubcf4\ub3c4\ub97c \uc720\uc608\ud560 \uac83\uc744 \uc694\uccad\ud558\uc600\ub2e4. 1\uc6d4 18\uc77c 1\ucc28 \uad6c\ucd9c \uc791\uc804\uc774 \uc2e4\ud328\ud558\uc790 20\uc77c\uc5d0 \ubd80\uc0b0\uc77c\ubcf4\ub294 \uc5e0\ubc14\uace0\ub97c \ud30c\uae30\ud558\uace0 \ud574\ub2f9 \uc0ac\uc2e4\uc744 \ubcf4\ub3c4\ud558\uc600\uace0 \ubbf8\ub514\uc5b4\uc624\ub298\uc774 \uc774\ub97c \uc778\uc6a9\ud558\uc600\ub2e4. 21\uc77c\uc5d0\ub294 \uc544\uc2dc\uc544\ud22c\ub370\uc774\ub3c4 \ud574\ub2f9 \uc0ac\uc2e4\uc744 \ubcf4\ub3c4\ud558\uc600\ub2e4. \uad6d\ubc29\ubd80\ub294 \ud574\ub2f9 \uae30\uc0ac\uc758 \uc0ad\uc81c\ub97c \uc694\uccad\ud558\uc600\uace0 \ubd80\uc0b0\uc77c\ubcf4\ub294 \uc6f9\uc0ac\uc774\ud2b8\uc5d0 \uac8c\uc7ac\ub41c \uae30\uc0ac\ub97c \uc0ad\uc81c\ud558\uc600\uc73c\ub098 \ubbf8\ub514\uc5b4\uc624\ub298\uacfc \uc544\uc2dc\uc544\ud22c\ub370\uc774\ub294 \uae30\uc0ac\uc758 \uc0ad\uc81c\ub97c \uac70\ubd80\ud558\uc600\ub2e4. \uad6d\ubc29\ubd80\ub294 \uc5e0\ubc14\uace0\ub97c \ud30c\uae30\ud55c \uc5b8\ub860\uc0ac\uc5d0 \uc18c\uc18d\ub41c \uae30\uc790\uc758 \uae30\uc790\uc2e4 \ucd9c\uc785\uc744 \ubb34\uae30\ud55c \uc815\uc9c0\ud558\uc600\uace0 \ubcf4\ub3c4 \uc790\ub8cc \ubc30\ud3ec\ub3c4 \uc911\ub2e8\ud558\uc600\ub2e4. \ub610\ud55c \ub2e4\ub978 \ubd80\ucc98\uc5d0\ub3c4 \uc81c\uc7ac\ub97c \uc694\uccad\ud558\ub294 \uacf5\ubb38\uc744 \ubcf4\ub0c8\ub2e4. \uc774\uc5d0 \uccad\uc640\ub300\ub294 \uc544\uc2dc\uc544\ud22c\ub370\uc774, \ubbf8\ub514\uc5b4\uc624\ub298\uc5d0 \uc18c\uc18d\ub41c \uae30\uc790\uc758 \ucd9c\uc785 \uae30\uc790 \ub4f1\ub85d\uc744 \ucde8\uc18c\ud558\uace0, \ubd80\uc0b0\uc77c\ubcf4\uc5d0 \uc18c\uc18d\ub41c \uae30\uc790\uc758 \uae30\uc790\uc2e4 \ucd9c\uc785\uc744 1\uac1c\uc6d4 \ub3d9\uc548 \uae08\uc9c0\ud558\uc600\ub2e4. \uc544\uc2dc\uc544\ud22c\ub370\uc774\ub294 \uccad\uc640\ub300\uc758 \uc870\uce58\uc5d0 \ubc18\ubc1c\ud558\uc5ec \uc11c\uc6b8\ud589\uc815\ubc95\uc6d0\uc5d0 \ucd9c\uc785 \uae30\uc790 \ub4f1\ub85d \ucde8\uc18c\uc5d0 \ub300\ud55c \ucde8\uc18c \uccad\uad6c\uc640 \ud6a8\ub825 \uc815\uc9c0 \uac00\ucc98\ubd84 \uc2e0\uccad\uc744 \uc81c\uae30\ud558\uc600\ub2e4. \uccad\uc640\ub300\uc640 \uc544\uc2dc\uc544 \ud22c\ub370\uc774\ub294 \ubc95\uc6d0\uc758 \uc870\uc815 \uad8c\uace0\ub97c \uc218\ub77d\ud558\uc5ec \uc544\uc2dc\uc544 \ud22c\ub370\uc774 \uc18c\uc18d \uae30\uc790\uc758 \ucd9c\uc785\uc744 2\uac1c\uc6d4 \ub3d9\uc548 \uc815\uc9c0\ud558\ub294 \uac83\uc73c\ub85c \uac10\uacbd\ud558\uc600\ub2e4.", "paragraph_answer": " 2011\ub144 1\uc6d4 24\uc77c \uad6d\ubc29\ubd80\ub294 \uc18c\ub9d0\ub9ac\uc544 \ud574\uc801\ub4e4\uc5d0\uac8c \ub0a9\uce58\ub41c \uc0bc\ud638\uc8fc\uc5bc\ub9ac\ud638\uc758 \uc120\uc6d0\ub4e4\uc744 \uad6c\ucd9c\ud558\uae30 \uc704\ud55c \u2018\uc544\ub374 \ub9cc \uc5ec\uba85 \uc791\uc804\u2019\uc744 \uc218\ud589\ud558\uba74\uc11c \uc120\uc6d0\ub4e4\uc758 \uc548\uc804\uc744 \uace0\ub824\ud558\uc5ec \ucd9c\uc785 \uae30\uc790\ub2e8\uc5d0\uac8c \ubcf4\ub3c4\ub97c \uc720\uc608\ud560 \uac83\uc744 \uc694\uccad\ud558\uc600\ub2e4. 1\uc6d4 18\uc77c 1\ucc28 \uad6c\ucd9c \uc791\uc804\uc774 \uc2e4\ud328\ud558\uc790 20\uc77c\uc5d0 \ubd80\uc0b0\uc77c\ubcf4\ub294 \uc5e0\ubc14\uace0\ub97c \ud30c\uae30\ud558\uace0 \ud574\ub2f9 \uc0ac\uc2e4\uc744 \ubcf4\ub3c4\ud558\uc600\uace0 \ubbf8\ub514\uc5b4\uc624\ub298\uc774 \uc774\ub97c \uc778\uc6a9\ud558\uc600\ub2e4. 21\uc77c\uc5d0\ub294 \uc544\uc2dc\uc544\ud22c\ub370\uc774\ub3c4 \ud574\ub2f9 \uc0ac\uc2e4\uc744 \ubcf4\ub3c4\ud558\uc600\ub2e4. \uad6d\ubc29\ubd80\ub294 \ud574\ub2f9 \uae30\uc0ac\uc758 \uc0ad\uc81c\ub97c \uc694\uccad\ud558\uc600\uace0 \ubd80\uc0b0\uc77c\ubcf4\ub294 \uc6f9\uc0ac\uc774\ud2b8\uc5d0 \uac8c\uc7ac\ub41c \uae30\uc0ac\ub97c \uc0ad\uc81c\ud558\uc600\uc73c\ub098 \ubbf8\ub514\uc5b4\uc624\ub298\uacfc \uc544\uc2dc\uc544\ud22c\ub370\uc774\ub294 \uae30\uc0ac\uc758 \uc0ad\uc81c\ub97c \uac70\ubd80\ud558\uc600\ub2e4. \uad6d\ubc29\ubd80\ub294 \uc5e0\ubc14\uace0\ub97c \ud30c\uae30\ud55c \uc5b8\ub860\uc0ac\uc5d0 \uc18c\uc18d\ub41c \uae30\uc790\uc758 \uae30\uc790\uc2e4 \ucd9c\uc785\uc744 \ubb34\uae30\ud55c \uc815\uc9c0\ud558\uc600\uace0 \ubcf4\ub3c4 \uc790\ub8cc \ubc30\ud3ec\ub3c4 \uc911\ub2e8\ud558\uc600\ub2e4. \ub610\ud55c \ub2e4\ub978 \ubd80\ucc98\uc5d0\ub3c4 \uc81c\uc7ac\ub97c \uc694\uccad\ud558\ub294 \uacf5\ubb38\uc744 \ubcf4\ub0c8\ub2e4. \uc774\uc5d0 \uccad\uc640\ub300\ub294 \uc544\uc2dc\uc544\ud22c\ub370\uc774, \ubbf8\ub514\uc5b4\uc624\ub298\uc5d0 \uc18c\uc18d\ub41c \uae30\uc790\uc758 \ucd9c\uc785 \uae30\uc790 \ub4f1\ub85d\uc744 \ucde8\uc18c\ud558\uace0, \ubd80\uc0b0\uc77c\ubcf4\uc5d0 \uc18c\uc18d\ub41c \uae30\uc790\uc758 \uae30\uc790\uc2e4 \ucd9c\uc785\uc744 1\uac1c\uc6d4 \ub3d9\uc548 \uae08\uc9c0\ud558\uc600\ub2e4. \uc544\uc2dc\uc544\ud22c\ub370\uc774\ub294 \uccad\uc640\ub300\uc758 \uc870\uce58\uc5d0 \ubc18\ubc1c\ud558\uc5ec \uc11c\uc6b8\ud589\uc815\ubc95\uc6d0\uc5d0 \ucd9c\uc785 \uae30\uc790 \ub4f1\ub85d \ucde8\uc18c\uc5d0 \ub300\ud55c \ucde8\uc18c \uccad\uad6c\uc640 \ud6a8\ub825 \uc815\uc9c0 \uac00\ucc98\ubd84 \uc2e0\uccad\uc744 \uc81c\uae30\ud558\uc600\ub2e4. \uccad\uc640\ub300\uc640 \uc544\uc2dc\uc544 \ud22c\ub370\uc774\ub294 \ubc95\uc6d0\uc758 \uc870\uc815 \uad8c\uace0\ub97c \uc218\ub77d\ud558\uc5ec \uc544\uc2dc\uc544 \ud22c\ub370\uc774 \uc18c\uc18d \uae30\uc790\uc758 \ucd9c\uc785\uc744 2\uac1c\uc6d4 \ub3d9\uc548 \uc815\uc9c0\ud558\ub294 \uac83\uc73c\ub85c \uac10\uacbd\ud558\uc600\ub2e4.", "sentence_answer": " 2011\ub144 1\uc6d4 24\uc77c \uad6d\ubc29\ubd80\ub294 \uc18c\ub9d0\ub9ac\uc544 \ud574\uc801\ub4e4\uc5d0\uac8c \ub0a9\uce58\ub41c \uc0bc\ud638\uc8fc\uc5bc\ub9ac\ud638\uc758 \uc120\uc6d0\ub4e4\uc744 \uad6c\ucd9c\ud558\uae30 \uc704\ud55c \u2018\uc544\ub374 \ub9cc \uc5ec\uba85 \uc791\uc804\u2019\uc744 \uc218\ud589\ud558\uba74\uc11c \uc120\uc6d0\ub4e4\uc758 \uc548\uc804\uc744 \uace0\ub824\ud558\uc5ec \ucd9c\uc785 \uae30\uc790\ub2e8\uc5d0\uac8c \ubcf4\ub3c4\ub97c \uc720\uc608\ud560 \uac83\uc744 \uc694\uccad\ud558\uc600\ub2e4.", "paragraph_id": "6656840-3-0", "paragraph_question": "question: \uc544\ub374 \ub9cc \uc5ec\uba85 \uc791\uc804\uc744 \uc704\ud574 \uc5b8\ub860\uc758 \ubcf4\ub3c4 \uc790\uc81c\ub97c \uc694\uccad\ud55c \uc2dc\uae30\ub294 \uc5b8\uc81c\uc778\uac00?, context: 2011\ub144 1\uc6d4 24\uc77c \uad6d\ubc29\ubd80\ub294 \uc18c\ub9d0\ub9ac\uc544 \ud574\uc801\ub4e4\uc5d0\uac8c \ub0a9\uce58\ub41c \uc0bc\ud638\uc8fc\uc5bc\ub9ac\ud638\uc758 \uc120\uc6d0\ub4e4\uc744 \uad6c\ucd9c\ud558\uae30 \uc704\ud55c \u2018\uc544\ub374 \ub9cc \uc5ec\uba85 \uc791\uc804\u2019\uc744 \uc218\ud589\ud558\uba74\uc11c \uc120\uc6d0\ub4e4\uc758 \uc548\uc804\uc744 \uace0\ub824\ud558\uc5ec \ucd9c\uc785 \uae30\uc790\ub2e8\uc5d0\uac8c \ubcf4\ub3c4\ub97c \uc720\uc608\ud560 \uac83\uc744 \uc694\uccad\ud558\uc600\ub2e4. 1\uc6d4 18\uc77c 1\ucc28 \uad6c\ucd9c \uc791\uc804\uc774 \uc2e4\ud328\ud558\uc790 20\uc77c\uc5d0 \ubd80\uc0b0\uc77c\ubcf4\ub294 \uc5e0\ubc14\uace0\ub97c \ud30c\uae30\ud558\uace0 \ud574\ub2f9 \uc0ac\uc2e4\uc744 \ubcf4\ub3c4\ud558\uc600\uace0 \ubbf8\ub514\uc5b4\uc624\ub298\uc774 \uc774\ub97c \uc778\uc6a9\ud558\uc600\ub2e4. 21\uc77c\uc5d0\ub294 \uc544\uc2dc\uc544\ud22c\ub370\uc774\ub3c4 \ud574\ub2f9 \uc0ac\uc2e4\uc744 \ubcf4\ub3c4\ud558\uc600\ub2e4. \uad6d\ubc29\ubd80\ub294 \ud574\ub2f9 \uae30\uc0ac\uc758 \uc0ad\uc81c\ub97c \uc694\uccad\ud558\uc600\uace0 \ubd80\uc0b0\uc77c\ubcf4\ub294 \uc6f9\uc0ac\uc774\ud2b8\uc5d0 \uac8c\uc7ac\ub41c \uae30\uc0ac\ub97c \uc0ad\uc81c\ud558\uc600\uc73c\ub098 \ubbf8\ub514\uc5b4\uc624\ub298\uacfc \uc544\uc2dc\uc544\ud22c\ub370\uc774\ub294 \uae30\uc0ac\uc758 \uc0ad\uc81c\ub97c \uac70\ubd80\ud558\uc600\ub2e4. \uad6d\ubc29\ubd80\ub294 \uc5e0\ubc14\uace0\ub97c \ud30c\uae30\ud55c \uc5b8\ub860\uc0ac\uc5d0 \uc18c\uc18d\ub41c \uae30\uc790\uc758 \uae30\uc790\uc2e4 \ucd9c\uc785\uc744 \ubb34\uae30\ud55c \uc815\uc9c0\ud558\uc600\uace0 \ubcf4\ub3c4 \uc790\ub8cc \ubc30\ud3ec\ub3c4 \uc911\ub2e8\ud558\uc600\ub2e4. \ub610\ud55c \ub2e4\ub978 \ubd80\ucc98\uc5d0\ub3c4 \uc81c\uc7ac\ub97c \uc694\uccad\ud558\ub294 \uacf5\ubb38\uc744 \ubcf4\ub0c8\ub2e4. \uc774\uc5d0 \uccad\uc640\ub300\ub294 \uc544\uc2dc\uc544\ud22c\ub370\uc774, \ubbf8\ub514\uc5b4\uc624\ub298\uc5d0 \uc18c\uc18d\ub41c \uae30\uc790\uc758 \ucd9c\uc785 \uae30\uc790 \ub4f1\ub85d\uc744 \ucde8\uc18c\ud558\uace0, \ubd80\uc0b0\uc77c\ubcf4\uc5d0 \uc18c\uc18d\ub41c \uae30\uc790\uc758 \uae30\uc790\uc2e4 \ucd9c\uc785\uc744 1\uac1c\uc6d4 \ub3d9\uc548 \uae08\uc9c0\ud558\uc600\ub2e4. \uc544\uc2dc\uc544\ud22c\ub370\uc774\ub294 \uccad\uc640\ub300\uc758 \uc870\uce58\uc5d0 \ubc18\ubc1c\ud558\uc5ec \uc11c\uc6b8\ud589\uc815\ubc95\uc6d0\uc5d0 \ucd9c\uc785 \uae30\uc790 \ub4f1\ub85d \ucde8\uc18c\uc5d0 \ub300\ud55c \ucde8\uc18c \uccad\uad6c\uc640 \ud6a8\ub825 \uc815\uc9c0 \uac00\ucc98\ubd84 \uc2e0\uccad\uc744 \uc81c\uae30\ud558\uc600\ub2e4. \uccad\uc640\ub300\uc640 \uc544\uc2dc\uc544 \ud22c\ub370\uc774\ub294 \ubc95\uc6d0\uc758 \uc870\uc815 \uad8c\uace0\ub97c \uc218\ub77d\ud558\uc5ec \uc544\uc2dc\uc544 \ud22c\ub370\uc774 \uc18c\uc18d \uae30\uc790\uc758 \ucd9c\uc785\uc744 2\uac1c\uc6d4 \ub3d9\uc548 \uc815\uc9c0\ud558\ub294 \uac83\uc73c\ub85c \uac10\uacbd\ud558\uc600\ub2e4."} {"question": "2011\ub144 \uc2dc\uc98c \ub3c4\uc911 \ud1f4\uc9c4\ud55c \ub450\uc0b0 \ubca0\uc5b4\uc2a4 \uac10\ub3c5\uc740?", "paragraph": "\uc2dc\uc98c \ucd08\ubc18 \ud0c0\uaca9 \ubd80\uc9c4\uc5d0 \uc2dc\ub2ec\ub9ac\uae30\ub3c4 \ud588\uc73c\ub098, 4\uc6d4 \uc911\ubc18\ubd80\ud130 \uc0c1\uc2b9\uc138\ub97c \ubcf4\uc774\uba70 \ud0c0\uc728\uc744 \ub04c\uc5b4\uc62c\ub838\ub2e4. \uadf8\ub7ec\ub358 \uc911 4\uc6d4 24\uc77c \ud55c\ud654 \uc774\uae00\uc2a4\uc804\uc5d0\uc11c \ud22c\uc218 \uc55e \ub545\ubcfc\uc744 \uce58\uace0 1\ub8e8\uc5d0 \ud5e4\ub4dc\ud37c\uc2a4\ud2b8 \uc2ac\ub77c\uc774\ub529\uc744 \uc2dc\ub3c4\ud558\ub2e4 \ubca0\uc774\uc2a4\uc5d0 \uc67c \uc5c4\uc9c0\uc190\uac00\ub77d\uc744 \ubd80\ub52a\ud614\uc73c\ub098 \ud070 \ubd80\uc0c1\uc774 \uc544\ub2cc \uac83\uc73c\ub85c \ubc1d\ud600\uc84c\ub2e4. \uadf8\ub7ec\ub098 \ubd80\uc0c1 \ud68c\ubcf5\uc774 \ub354\ub38c\uc9c0\uba74\uc11c \uc7a0\uc2dc \uacbd\uae30\uc5d0 \ucd9c\uc804\ud558\uc9c0 \uc54a\uc558\ub2e4. \ud558\uc9c0\ub9cc \uadf8\ub97c \ub300\uccb4\ud560 \uc120\uc218\uac00 \uc5c6\uace0 \ud300\ub3c4 \ubd80\uc9c4\ud574 5\uc6d4 \ucd08\ubc18 \ub2e4\uc2dc \uc120\ubc1c \ub77c\uc778\uc5c5\uc5d0 \ubcf5\uadc0\ud588\ub2e4. \ubcf5\uadc0 \uc774\ud6c4 \ud0c0\uaca9 \ubd80\uc9c4\uc5d0 \uc2dc\ub2ec\ub824 \uba87 \uacbd\uae30 \ud558\uc704 \ud0c0\uc120\uc5d0 \ubc30\uce58\ub418\uae30\ub3c4 \ud588\uc73c\ub098, \ub2e4\uc2dc \uc608\ub144 \uac19\uc740 \ud65c\uc57d\uc744 \ud3bc\ucce4\ub2e4. \uc774 \uc2dc\uc98c\uc5d0\ub294 \ud300 \uc131\uc801 \ubd80\uc9c4 \ub4f1\uc73c\ub85c \uc62c\uc2a4\ud0c0\uc804 \ud32c\ud22c\ud45c\uc5d0 \ubf51\ud788\uc9c0 \ubabb\ud588\uace0, \uac10\ub3c5 \ucd94\ucc9c \uc120\uc218\uc5d0\ub3c4 \ub4e4\uc9c0 \ubabb\ud558\uba74\uc11c 2006\ub144 \uc774\ud6c4 \ucc98\uc74c\uc73c\ub85c \uc62c\uc2a4\ud0c0\uc804\uc5d0 \ucc38\uac00\ud558\uc9c0 \ubabb\ud588\ub2e4. 8\uc6d4 5\uc77c, 6\uc77c\uc5d0\ub294 \ub125\uc13c \ud788\uc5b4\ub85c\uc988\ub97c \uc0c1\ub300\ub85c \ub370\ubdd4 \uccab \ub450 \uacbd\uae30 \uc5f0\uc18d \ud648\ub7f0\uc744 \uccd0\ub0c8\uc73c\uba70, \ud2b9\ud788 6\uc77c\uc5d0\ub294 \ub370\ubdd4 \uccab 3\uc810 \uc774\uc0c1\uc758 \ud648\ub7f0\uc744 \ubf51\uc544\ub0c8\ub2e4. \uadf8\ub294 \uc774\ubc88 \uc2dc\uc98c \uc7a6\uc740 \ubd80\uc0c1\uc73c\ub85c \uace0\uc5ed\uc744 \uce58\ub800\uace0, \ud0c0\uc728 9\uc704\ub97c \uae30\ub85d\ud588\uc73c\uba70, \uac1c\uc778 \uc2dc\uc98c \ucd5c\uc18c \ub3c4\ub8e8\uc778 20\ub3c4\ub8e8\ub97c \uae30\ub85d\ud588\ub2e4. \uc790\uc2e0\uc740 \uc678\uc57c \uc218\ube44 \ub54c\ub098 \ub3c4\ub8e8 \uc2dc\ub3c4 \ub4f1 \uc8fc\ub8e8\ud50c\ub808\uc774 \ub54c \ub2e4\uc18c \uc18c\uadf9\uc801\uc73c\ub85c \ud50c\ub808\uc774\ud574\uc11c, \uc790\uc2e0\uc758 \uc7a5\uc810\uc778 \ud5c8\uc2ac\ud50c\ub808\uc774\ub97c \ub9d8\uaecf \ubcf4\uc5ec\uc8fc\uc9c0 \ubabb\ud588\ub2e4\ub294 \uc544\uc26c\uc6c0\uc774 \ucef8\ub2e4\uace0 \ub9d0\ud588\ub2e4. \ub450\uc0b0 \ubca0\uc5b4\uc2a4\ub294 \ub2f9\uc2dc \uac10\ub3c5\uc774\uc5c8\ub358 \uae40\uacbd\ubb38\uc758 \uc2dc\uc98c \ub3c4\uc911 \ud1f4\uc9c4 \ub4f1\uc758 \uc6b0\uc5ec\uace1\uc808\uc744 \uacaa\uc73c\uba70 \uc2dc\uc98c\uc744 5\uc704\ub85c \ub9c8\ubb34\ub9ac\uc9c0\uc5c8\uace0 \ud3ec\uc2a4\ud2b8\uc2dc\uc98c \uc9c4\ucd9c\uc5d0\ub294 \uc2e4\ud328\ud588\ub2e4. \uadf8\ub294 \uace8\ub4e0 \uae00\ub7ec\ube0c \uc678\uc57c\uc218 \ubd80\ubb38 \ud6c4\ubcf4\uc5d0 \uc62c\ub790\uc73c\ub098, \uc804\uccb4 306\ud45c 17\ud45c \ub9cc\uc744 \ub4dd\ud45c\ud558\uba74\uc11c, \uc218\uc0c1\uc5d0\ub294 \uc2e4\ud328\ud588\ub2e4.2011\ub144 \uc2dc\uc98c\ubcf4\ub2e4 7% \uc778\uc0c1\ub41c 2\uc5b5 500\ub9cc\uc6d0\uc5d0 2012 \uc2dc\uc98c \uc5f0\ubd09 \uc7ac\uacc4\uc57d\uc744 \ud558\uba74\uc11c, \ub370\ubdd4 \ucc98\uc74c\uc73c\ub85c \uc5f0\ubd09 2\uc5b5\uc6d0\ub300\ub97c \ub3cc\ud30c\ud588\ub2e4.", "answer": "\uae40\uacbd\ubb38", "sentence": "\ub450\uc0b0 \ubca0\uc5b4\uc2a4\ub294 \ub2f9\uc2dc \uac10\ub3c5\uc774\uc5c8\ub358 \uae40\uacbd\ubb38 \uc758 \uc2dc\uc98c \ub3c4\uc911 \ud1f4\uc9c4 \ub4f1\uc758 \uc6b0\uc5ec\uace1\uc808\uc744 \uacaa\uc73c\uba70 \uc2dc\uc98c\uc744 5\uc704\ub85c \ub9c8\ubb34\ub9ac\uc9c0\uc5c8\uace0 \ud3ec\uc2a4\ud2b8\uc2dc\uc98c \uc9c4\ucd9c\uc5d0\ub294 \uc2e4\ud328\ud588\ub2e4.", "paragraph_sentence": "\uc2dc\uc98c \ucd08\ubc18 \ud0c0\uaca9 \ubd80\uc9c4\uc5d0 \uc2dc\ub2ec\ub9ac\uae30\ub3c4 \ud588\uc73c\ub098, 4\uc6d4 \uc911\ubc18\ubd80\ud130 \uc0c1\uc2b9\uc138\ub97c \ubcf4\uc774\uba70 \ud0c0\uc728\uc744 \ub04c\uc5b4\uc62c\ub838\ub2e4. \uadf8\ub7ec\ub358 \uc911 4\uc6d4 24\uc77c \ud55c\ud654 \uc774\uae00\uc2a4\uc804\uc5d0\uc11c \ud22c\uc218 \uc55e \ub545\ubcfc\uc744 \uce58\uace0 1\ub8e8\uc5d0 \ud5e4\ub4dc\ud37c\uc2a4\ud2b8 \uc2ac\ub77c\uc774\ub529\uc744 \uc2dc\ub3c4\ud558\ub2e4 \ubca0\uc774\uc2a4\uc5d0 \uc67c \uc5c4\uc9c0\uc190\uac00\ub77d\uc744 \ubd80\ub52a\ud614\uc73c\ub098 \ud070 \ubd80\uc0c1\uc774 \uc544\ub2cc \uac83\uc73c\ub85c \ubc1d\ud600\uc84c\ub2e4. \uadf8\ub7ec\ub098 \ubd80\uc0c1 \ud68c\ubcf5\uc774 \ub354\ub38c\uc9c0\uba74\uc11c \uc7a0\uc2dc \uacbd\uae30\uc5d0 \ucd9c\uc804\ud558\uc9c0 \uc54a\uc558\ub2e4. \ud558\uc9c0\ub9cc \uadf8\ub97c \ub300\uccb4\ud560 \uc120\uc218\uac00 \uc5c6\uace0 \ud300\ub3c4 \ubd80\uc9c4\ud574 5\uc6d4 \ucd08\ubc18 \ub2e4\uc2dc \uc120\ubc1c \ub77c\uc778\uc5c5\uc5d0 \ubcf5\uadc0\ud588\ub2e4. \ubcf5\uadc0 \uc774\ud6c4 \ud0c0\uaca9 \ubd80\uc9c4\uc5d0 \uc2dc\ub2ec\ub824 \uba87 \uacbd\uae30 \ud558\uc704 \ud0c0\uc120\uc5d0 \ubc30\uce58\ub418\uae30\ub3c4 \ud588\uc73c\ub098, \ub2e4\uc2dc \uc608\ub144 \uac19\uc740 \ud65c\uc57d\uc744 \ud3bc\ucce4\ub2e4. \uc774 \uc2dc\uc98c\uc5d0\ub294 \ud300 \uc131\uc801 \ubd80\uc9c4 \ub4f1\uc73c\ub85c \uc62c\uc2a4\ud0c0\uc804 \ud32c\ud22c\ud45c\uc5d0 \ubf51\ud788\uc9c0 \ubabb\ud588\uace0, \uac10\ub3c5 \ucd94\ucc9c \uc120\uc218\uc5d0\ub3c4 \ub4e4\uc9c0 \ubabb\ud558\uba74\uc11c 2006\ub144 \uc774\ud6c4 \ucc98\uc74c\uc73c\ub85c \uc62c\uc2a4\ud0c0\uc804\uc5d0 \ucc38\uac00\ud558\uc9c0 \ubabb\ud588\ub2e4. 8\uc6d4 5\uc77c, 6\uc77c\uc5d0\ub294 \ub125\uc13c \ud788\uc5b4\ub85c\uc988\ub97c \uc0c1\ub300\ub85c \ub370\ubdd4 \uccab \ub450 \uacbd\uae30 \uc5f0\uc18d \ud648\ub7f0\uc744 \uccd0\ub0c8\uc73c\uba70, \ud2b9\ud788 6\uc77c\uc5d0\ub294 \ub370\ubdd4 \uccab 3\uc810 \uc774\uc0c1\uc758 \ud648\ub7f0\uc744 \ubf51\uc544\ub0c8\ub2e4. \uadf8\ub294 \uc774\ubc88 \uc2dc\uc98c \uc7a6\uc740 \ubd80\uc0c1\uc73c\ub85c \uace0\uc5ed\uc744 \uce58\ub800\uace0, \ud0c0\uc728 9\uc704\ub97c \uae30\ub85d\ud588\uc73c\uba70, \uac1c\uc778 \uc2dc\uc98c \ucd5c\uc18c \ub3c4\ub8e8\uc778 20\ub3c4\ub8e8\ub97c \uae30\ub85d\ud588\ub2e4. \uc790\uc2e0\uc740 \uc678\uc57c \uc218\ube44 \ub54c\ub098 \ub3c4\ub8e8 \uc2dc\ub3c4 \ub4f1 \uc8fc\ub8e8\ud50c\ub808\uc774 \ub54c \ub2e4\uc18c \uc18c\uadf9\uc801\uc73c\ub85c \ud50c\ub808\uc774\ud574\uc11c, \uc790\uc2e0\uc758 \uc7a5\uc810\uc778 \ud5c8\uc2ac\ud50c\ub808\uc774\ub97c \ub9d8\uaecf \ubcf4\uc5ec\uc8fc\uc9c0 \ubabb\ud588\ub2e4\ub294 \uc544\uc26c\uc6c0\uc774 \ucef8\ub2e4\uace0 \ub9d0\ud588\ub2e4. \ub450\uc0b0 \ubca0\uc5b4\uc2a4\ub294 \ub2f9\uc2dc \uac10\ub3c5\uc774\uc5c8\ub358 \uae40\uacbd\ubb38 \uc758 \uc2dc\uc98c \ub3c4\uc911 \ud1f4\uc9c4 \ub4f1\uc758 \uc6b0\uc5ec\uace1\uc808\uc744 \uacaa\uc73c\uba70 \uc2dc\uc98c\uc744 5\uc704\ub85c \ub9c8\ubb34\ub9ac\uc9c0\uc5c8\uace0 \ud3ec\uc2a4\ud2b8\uc2dc\uc98c \uc9c4\ucd9c\uc5d0\ub294 \uc2e4\ud328\ud588\ub2e4. \uadf8\ub294 \uace8\ub4e0 \uae00\ub7ec\ube0c \uc678\uc57c\uc218 \ubd80\ubb38 \ud6c4\ubcf4\uc5d0 \uc62c\ub790\uc73c\ub098, \uc804\uccb4 306\ud45c 17\ud45c \ub9cc\uc744 \ub4dd\ud45c\ud558\uba74\uc11c, \uc218\uc0c1\uc5d0\ub294 \uc2e4\ud328\ud588\ub2e4.2011\ub144 \uc2dc\uc98c\ubcf4\ub2e4 7% \uc778\uc0c1\ub41c 2\uc5b5 500\ub9cc\uc6d0\uc5d0 2012 \uc2dc\uc98c \uc5f0\ubd09 \uc7ac\uacc4\uc57d\uc744 \ud558\uba74\uc11c, \ub370\ubdd4 \ucc98\uc74c\uc73c\ub85c \uc5f0\ubd09 2\uc5b5\uc6d0\ub300\ub97c \ub3cc\ud30c\ud588\ub2e4.", "paragraph_answer": "\uc2dc\uc98c \ucd08\ubc18 \ud0c0\uaca9 \ubd80\uc9c4\uc5d0 \uc2dc\ub2ec\ub9ac\uae30\ub3c4 \ud588\uc73c\ub098, 4\uc6d4 \uc911\ubc18\ubd80\ud130 \uc0c1\uc2b9\uc138\ub97c \ubcf4\uc774\uba70 \ud0c0\uc728\uc744 \ub04c\uc5b4\uc62c\ub838\ub2e4. \uadf8\ub7ec\ub358 \uc911 4\uc6d4 24\uc77c \ud55c\ud654 \uc774\uae00\uc2a4\uc804\uc5d0\uc11c \ud22c\uc218 \uc55e \ub545\ubcfc\uc744 \uce58\uace0 1\ub8e8\uc5d0 \ud5e4\ub4dc\ud37c\uc2a4\ud2b8 \uc2ac\ub77c\uc774\ub529\uc744 \uc2dc\ub3c4\ud558\ub2e4 \ubca0\uc774\uc2a4\uc5d0 \uc67c \uc5c4\uc9c0\uc190\uac00\ub77d\uc744 \ubd80\ub52a\ud614\uc73c\ub098 \ud070 \ubd80\uc0c1\uc774 \uc544\ub2cc \uac83\uc73c\ub85c \ubc1d\ud600\uc84c\ub2e4. \uadf8\ub7ec\ub098 \ubd80\uc0c1 \ud68c\ubcf5\uc774 \ub354\ub38c\uc9c0\uba74\uc11c \uc7a0\uc2dc \uacbd\uae30\uc5d0 \ucd9c\uc804\ud558\uc9c0 \uc54a\uc558\ub2e4. \ud558\uc9c0\ub9cc \uadf8\ub97c \ub300\uccb4\ud560 \uc120\uc218\uac00 \uc5c6\uace0 \ud300\ub3c4 \ubd80\uc9c4\ud574 5\uc6d4 \ucd08\ubc18 \ub2e4\uc2dc \uc120\ubc1c \ub77c\uc778\uc5c5\uc5d0 \ubcf5\uadc0\ud588\ub2e4. \ubcf5\uadc0 \uc774\ud6c4 \ud0c0\uaca9 \ubd80\uc9c4\uc5d0 \uc2dc\ub2ec\ub824 \uba87 \uacbd\uae30 \ud558\uc704 \ud0c0\uc120\uc5d0 \ubc30\uce58\ub418\uae30\ub3c4 \ud588\uc73c\ub098, \ub2e4\uc2dc \uc608\ub144 \uac19\uc740 \ud65c\uc57d\uc744 \ud3bc\ucce4\ub2e4. \uc774 \uc2dc\uc98c\uc5d0\ub294 \ud300 \uc131\uc801 \ubd80\uc9c4 \ub4f1\uc73c\ub85c \uc62c\uc2a4\ud0c0\uc804 \ud32c\ud22c\ud45c\uc5d0 \ubf51\ud788\uc9c0 \ubabb\ud588\uace0, \uac10\ub3c5 \ucd94\ucc9c \uc120\uc218\uc5d0\ub3c4 \ub4e4\uc9c0 \ubabb\ud558\uba74\uc11c 2006\ub144 \uc774\ud6c4 \ucc98\uc74c\uc73c\ub85c \uc62c\uc2a4\ud0c0\uc804\uc5d0 \ucc38\uac00\ud558\uc9c0 \ubabb\ud588\ub2e4. 8\uc6d4 5\uc77c, 6\uc77c\uc5d0\ub294 \ub125\uc13c \ud788\uc5b4\ub85c\uc988\ub97c \uc0c1\ub300\ub85c \ub370\ubdd4 \uccab \ub450 \uacbd\uae30 \uc5f0\uc18d \ud648\ub7f0\uc744 \uccd0\ub0c8\uc73c\uba70, \ud2b9\ud788 6\uc77c\uc5d0\ub294 \ub370\ubdd4 \uccab 3\uc810 \uc774\uc0c1\uc758 \ud648\ub7f0\uc744 \ubf51\uc544\ub0c8\ub2e4. \uadf8\ub294 \uc774\ubc88 \uc2dc\uc98c \uc7a6\uc740 \ubd80\uc0c1\uc73c\ub85c \uace0\uc5ed\uc744 \uce58\ub800\uace0, \ud0c0\uc728 9\uc704\ub97c \uae30\ub85d\ud588\uc73c\uba70, \uac1c\uc778 \uc2dc\uc98c \ucd5c\uc18c \ub3c4\ub8e8\uc778 20\ub3c4\ub8e8\ub97c \uae30\ub85d\ud588\ub2e4. \uc790\uc2e0\uc740 \uc678\uc57c \uc218\ube44 \ub54c\ub098 \ub3c4\ub8e8 \uc2dc\ub3c4 \ub4f1 \uc8fc\ub8e8\ud50c\ub808\uc774 \ub54c \ub2e4\uc18c \uc18c\uadf9\uc801\uc73c\ub85c \ud50c\ub808\uc774\ud574\uc11c, \uc790\uc2e0\uc758 \uc7a5\uc810\uc778 \ud5c8\uc2ac\ud50c\ub808\uc774\ub97c \ub9d8\uaecf \ubcf4\uc5ec\uc8fc\uc9c0 \ubabb\ud588\ub2e4\ub294 \uc544\uc26c\uc6c0\uc774 \ucef8\ub2e4\uace0 \ub9d0\ud588\ub2e4. \ub450\uc0b0 \ubca0\uc5b4\uc2a4\ub294 \ub2f9\uc2dc \uac10\ub3c5\uc774\uc5c8\ub358 \uae40\uacbd\ubb38 \uc758 \uc2dc\uc98c \ub3c4\uc911 \ud1f4\uc9c4 \ub4f1\uc758 \uc6b0\uc5ec\uace1\uc808\uc744 \uacaa\uc73c\uba70 \uc2dc\uc98c\uc744 5\uc704\ub85c \ub9c8\ubb34\ub9ac\uc9c0\uc5c8\uace0 \ud3ec\uc2a4\ud2b8\uc2dc\uc98c \uc9c4\ucd9c\uc5d0\ub294 \uc2e4\ud328\ud588\ub2e4. \uadf8\ub294 \uace8\ub4e0 \uae00\ub7ec\ube0c \uc678\uc57c\uc218 \ubd80\ubb38 \ud6c4\ubcf4\uc5d0 \uc62c\ub790\uc73c\ub098, \uc804\uccb4 306\ud45c 17\ud45c \ub9cc\uc744 \ub4dd\ud45c\ud558\uba74\uc11c, \uc218\uc0c1\uc5d0\ub294 \uc2e4\ud328\ud588\ub2e4.2011\ub144 \uc2dc\uc98c\ubcf4\ub2e4 7% \uc778\uc0c1\ub41c 2\uc5b5 500\ub9cc\uc6d0\uc5d0 2012 \uc2dc\uc98c \uc5f0\ubd09 \uc7ac\uacc4\uc57d\uc744 \ud558\uba74\uc11c, \ub370\ubdd4 \ucc98\uc74c\uc73c\ub85c \uc5f0\ubd09 2\uc5b5\uc6d0\ub300\ub97c \ub3cc\ud30c\ud588\ub2e4.", "sentence_answer": "\ub450\uc0b0 \ubca0\uc5b4\uc2a4\ub294 \ub2f9\uc2dc \uac10\ub3c5\uc774\uc5c8\ub358 \uae40\uacbd\ubb38 \uc758 \uc2dc\uc98c \ub3c4\uc911 \ud1f4\uc9c4 \ub4f1\uc758 \uc6b0\uc5ec\uace1\uc808\uc744 \uacaa\uc73c\uba70 \uc2dc\uc98c\uc744 5\uc704\ub85c \ub9c8\ubb34\ub9ac\uc9c0\uc5c8\uace0 \ud3ec\uc2a4\ud2b8\uc2dc\uc98c \uc9c4\ucd9c\uc5d0\ub294 \uc2e4\ud328\ud588\ub2e4.", "paragraph_id": "6547838-6-2", "paragraph_question": "question: 2011\ub144 \uc2dc\uc98c \ub3c4\uc911 \ud1f4\uc9c4\ud55c \ub450\uc0b0 \ubca0\uc5b4\uc2a4 \uac10\ub3c5\uc740?, context: \uc2dc\uc98c \ucd08\ubc18 \ud0c0\uaca9 \ubd80\uc9c4\uc5d0 \uc2dc\ub2ec\ub9ac\uae30\ub3c4 \ud588\uc73c\ub098, 4\uc6d4 \uc911\ubc18\ubd80\ud130 \uc0c1\uc2b9\uc138\ub97c \ubcf4\uc774\uba70 \ud0c0\uc728\uc744 \ub04c\uc5b4\uc62c\ub838\ub2e4. \uadf8\ub7ec\ub358 \uc911 4\uc6d4 24\uc77c \ud55c\ud654 \uc774\uae00\uc2a4\uc804\uc5d0\uc11c \ud22c\uc218 \uc55e \ub545\ubcfc\uc744 \uce58\uace0 1\ub8e8\uc5d0 \ud5e4\ub4dc\ud37c\uc2a4\ud2b8 \uc2ac\ub77c\uc774\ub529\uc744 \uc2dc\ub3c4\ud558\ub2e4 \ubca0\uc774\uc2a4\uc5d0 \uc67c \uc5c4\uc9c0\uc190\uac00\ub77d\uc744 \ubd80\ub52a\ud614\uc73c\ub098 \ud070 \ubd80\uc0c1\uc774 \uc544\ub2cc \uac83\uc73c\ub85c \ubc1d\ud600\uc84c\ub2e4. \uadf8\ub7ec\ub098 \ubd80\uc0c1 \ud68c\ubcf5\uc774 \ub354\ub38c\uc9c0\uba74\uc11c \uc7a0\uc2dc \uacbd\uae30\uc5d0 \ucd9c\uc804\ud558\uc9c0 \uc54a\uc558\ub2e4. \ud558\uc9c0\ub9cc \uadf8\ub97c \ub300\uccb4\ud560 \uc120\uc218\uac00 \uc5c6\uace0 \ud300\ub3c4 \ubd80\uc9c4\ud574 5\uc6d4 \ucd08\ubc18 \ub2e4\uc2dc \uc120\ubc1c \ub77c\uc778\uc5c5\uc5d0 \ubcf5\uadc0\ud588\ub2e4. \ubcf5\uadc0 \uc774\ud6c4 \ud0c0\uaca9 \ubd80\uc9c4\uc5d0 \uc2dc\ub2ec\ub824 \uba87 \uacbd\uae30 \ud558\uc704 \ud0c0\uc120\uc5d0 \ubc30\uce58\ub418\uae30\ub3c4 \ud588\uc73c\ub098, \ub2e4\uc2dc \uc608\ub144 \uac19\uc740 \ud65c\uc57d\uc744 \ud3bc\ucce4\ub2e4. \uc774 \uc2dc\uc98c\uc5d0\ub294 \ud300 \uc131\uc801 \ubd80\uc9c4 \ub4f1\uc73c\ub85c \uc62c\uc2a4\ud0c0\uc804 \ud32c\ud22c\ud45c\uc5d0 \ubf51\ud788\uc9c0 \ubabb\ud588\uace0, \uac10\ub3c5 \ucd94\ucc9c \uc120\uc218\uc5d0\ub3c4 \ub4e4\uc9c0 \ubabb\ud558\uba74\uc11c 2006\ub144 \uc774\ud6c4 \ucc98\uc74c\uc73c\ub85c \uc62c\uc2a4\ud0c0\uc804\uc5d0 \ucc38\uac00\ud558\uc9c0 \ubabb\ud588\ub2e4. 8\uc6d4 5\uc77c, 6\uc77c\uc5d0\ub294 \ub125\uc13c \ud788\uc5b4\ub85c\uc988\ub97c \uc0c1\ub300\ub85c \ub370\ubdd4 \uccab \ub450 \uacbd\uae30 \uc5f0\uc18d \ud648\ub7f0\uc744 \uccd0\ub0c8\uc73c\uba70, \ud2b9\ud788 6\uc77c\uc5d0\ub294 \ub370\ubdd4 \uccab 3\uc810 \uc774\uc0c1\uc758 \ud648\ub7f0\uc744 \ubf51\uc544\ub0c8\ub2e4. \uadf8\ub294 \uc774\ubc88 \uc2dc\uc98c \uc7a6\uc740 \ubd80\uc0c1\uc73c\ub85c \uace0\uc5ed\uc744 \uce58\ub800\uace0, \ud0c0\uc728 9\uc704\ub97c \uae30\ub85d\ud588\uc73c\uba70, \uac1c\uc778 \uc2dc\uc98c \ucd5c\uc18c \ub3c4\ub8e8\uc778 20\ub3c4\ub8e8\ub97c \uae30\ub85d\ud588\ub2e4. \uc790\uc2e0\uc740 \uc678\uc57c \uc218\ube44 \ub54c\ub098 \ub3c4\ub8e8 \uc2dc\ub3c4 \ub4f1 \uc8fc\ub8e8\ud50c\ub808\uc774 \ub54c \ub2e4\uc18c \uc18c\uadf9\uc801\uc73c\ub85c \ud50c\ub808\uc774\ud574\uc11c, \uc790\uc2e0\uc758 \uc7a5\uc810\uc778 \ud5c8\uc2ac\ud50c\ub808\uc774\ub97c \ub9d8\uaecf \ubcf4\uc5ec\uc8fc\uc9c0 \ubabb\ud588\ub2e4\ub294 \uc544\uc26c\uc6c0\uc774 \ucef8\ub2e4\uace0 \ub9d0\ud588\ub2e4. \ub450\uc0b0 \ubca0\uc5b4\uc2a4\ub294 \ub2f9\uc2dc \uac10\ub3c5\uc774\uc5c8\ub358 \uae40\uacbd\ubb38\uc758 \uc2dc\uc98c \ub3c4\uc911 \ud1f4\uc9c4 \ub4f1\uc758 \uc6b0\uc5ec\uace1\uc808\uc744 \uacaa\uc73c\uba70 \uc2dc\uc98c\uc744 5\uc704\ub85c \ub9c8\ubb34\ub9ac\uc9c0\uc5c8\uace0 \ud3ec\uc2a4\ud2b8\uc2dc\uc98c \uc9c4\ucd9c\uc5d0\ub294 \uc2e4\ud328\ud588\ub2e4. \uadf8\ub294 \uace8\ub4e0 \uae00\ub7ec\ube0c \uc678\uc57c\uc218 \ubd80\ubb38 \ud6c4\ubcf4\uc5d0 \uc62c\ub790\uc73c\ub098, \uc804\uccb4 306\ud45c 17\ud45c \ub9cc\uc744 \ub4dd\ud45c\ud558\uba74\uc11c, \uc218\uc0c1\uc5d0\ub294 \uc2e4\ud328\ud588\ub2e4.2011\ub144 \uc2dc\uc98c\ubcf4\ub2e4 7% \uc778\uc0c1\ub41c 2\uc5b5 500\ub9cc\uc6d0\uc5d0 2012 \uc2dc\uc98c \uc5f0\ubd09 \uc7ac\uacc4\uc57d\uc744 \ud558\uba74\uc11c, \ub370\ubdd4 \ucc98\uc74c\uc73c\ub85c \uc5f0\ubd09 2\uc5b5\uc6d0\ub300\ub97c \ub3cc\ud30c\ud588\ub2e4."} {"question": "\uc870\uc911\ud6c8 \uc77c\uac00\uac00 100\ub9cc \ub2ec\ub7ec\ub97c \ucd9c\uc790\ud574 \uc124\ub9bd\ud55c KA\uc0ac\uac00 \uc788\ub294 \uc870\uc138 \ud68c\ud53c \uc9c0\uc5ed\uc740?", "paragraph": "1991\ub144 \ubd80\ud130 1998\ub144\uae4c\uc9c0 \uc678\uad6d \uae30\uc5c5 \ub450 \uacf3\uc758 \ud56d\uacf5\uae30\ub97c \uad6c\ub9e4\ud560\ub54c \ud2b9\uc815 \ud68c\uc0ac\uc758 \uc5d4\uc9c4\uc744 \uc7a5\ucc29\ud558\ub294 \uc870\uac74\uc73c\ub85c \ub9ac\ubca0\uc774\ud2b8\ub97c \ubc1b\uc558\ub2e4. \uc774 \uc911 \uc77c\ubd80\uc778 1\ucc9c685\uc5b5\uc6d0\uc744 \uad6d\ub0b4\ub85c \ub4e4\uc5ec\uc640 \uc870\uc911\ud6c8 \uba85\uc608 \ud68c\uc7a5\uacfc \uc870\uc591\ud638 \ud68c\uc7a5 \ub4f1\uc774 \uac1c\uc778\uc801\uc73c\ub85c \uc0ac\uc6a9\ud588\ub2e4. \uc9c0\ub09c 1997\ub144 11\uc6d4 26\uc77c 600\ub9cc\ub2ec\ub7ec\uc758 \ub9ac\ubca0\uc774\ud2b8\ub97c \uad6d\ub0b4\ub85c \ubc18\uc785\ud558\uace0 1998\ub144 7\uc6d4 29\uc77c\uc5d0 \uc774 \uc911 18\ub9cc\ub2ec\ub7ec (2\uc5b5 5\ucc9c\ub9cc\uc6d0)\ub97c \uac1c\uc778 \ubaa9\uc801\uc73c\ub85c \uc0ac\uc6a9\ud558\uae30 \uc704\ud574 3\uac1c\ub97c \ub2f9\uc88c\uc218\ud45c\ub85c \ubd84\uc0b0\ud558\uc5ec \ud604\uae08\ud654\ud55c\uac83\uc73c\ub85c \ubc1d\ud600\uc84c\ub2e4. \ub610, \uc870\uc911\ud6c8 \uc77c\uac00\ub294 \uc774\ub54c \ubc1b\uc740 \ub9ac\ubca0\uc774\ud2b8\ub97c \uc870\uc138 \ud68c\ud53c \uc9c0\uc5ed\uc778 '\uc544\uc77c\ub79c\ub4dc \ub354\ube14\ub9b0' \uc9c0\uc5ed\uc5d0 100\ub9cc\ub2ec\ub7ec\ub97c \ucd9c\uc790\ud574 \uc124\ub9bd\ud55c \ud604\uc9c0 \ubc95\uc778 KA\uc0ac\uc5d0 \uc774\uc804\ud558\uc5ec 1\uc5b5 8\ucc9c 400\ub9cc \ub2ec\ub7ec\uac00 \ud574\uc678\ub85c \uc720\ucd9c\ub410\uace0, \uc774\ub85c \uc778\ud574 814\uc5b5\uc6d0\uc744 \ud0c8\uc138\ud55c \ud610\uc758\uac00 \ubc1d\ud600\uc84c\ub2e4.", "answer": "\uc544\uc77c\ub79c\ub4dc \ub354\ube14\ub9b0", "sentence": "\ub610, \uc870\uc911\ud6c8 \uc77c\uac00\ub294 \uc774\ub54c \ubc1b\uc740 \ub9ac\ubca0\uc774\ud2b8\ub97c \uc870\uc138 \ud68c\ud53c \uc9c0\uc5ed\uc778 ' \uc544\uc77c\ub79c\ub4dc \ub354\ube14\ub9b0 ' \uc9c0\uc5ed\uc5d0 100\ub9cc\ub2ec\ub7ec\ub97c \ucd9c\uc790\ud574 \uc124\ub9bd\ud55c \ud604\uc9c0 \ubc95\uc778 KA\uc0ac\uc5d0 \uc774\uc804\ud558\uc5ec 1\uc5b5 8\ucc9c 400\ub9cc \ub2ec\ub7ec\uac00 \ud574\uc678\ub85c \uc720\ucd9c\ub410\uace0, \uc774\ub85c \uc778\ud574 814\uc5b5\uc6d0\uc744 \ud0c8\uc138\ud55c \ud610\uc758\uac00 \ubc1d\ud600\uc84c\ub2e4.", "paragraph_sentence": "1991\ub144 \ubd80\ud130 1998\ub144\uae4c\uc9c0 \uc678\uad6d \uae30\uc5c5 \ub450 \uacf3\uc758 \ud56d\uacf5\uae30\ub97c \uad6c\ub9e4\ud560\ub54c \ud2b9\uc815 \ud68c\uc0ac\uc758 \uc5d4\uc9c4\uc744 \uc7a5\ucc29\ud558\ub294 \uc870\uac74\uc73c\ub85c \ub9ac\ubca0\uc774\ud2b8\ub97c \ubc1b\uc558\ub2e4. \uc774 \uc911 \uc77c\ubd80\uc778 1\ucc9c685\uc5b5\uc6d0\uc744 \uad6d\ub0b4\ub85c \ub4e4\uc5ec\uc640 \uc870\uc911\ud6c8 \uba85\uc608 \ud68c\uc7a5\uacfc \uc870\uc591\ud638 \ud68c\uc7a5 \ub4f1\uc774 \uac1c\uc778\uc801\uc73c\ub85c \uc0ac\uc6a9\ud588\ub2e4. \uc9c0\ub09c 1997\ub144 11\uc6d4 26\uc77c 600\ub9cc\ub2ec\ub7ec\uc758 \ub9ac\ubca0\uc774\ud2b8\ub97c \uad6d\ub0b4\ub85c \ubc18\uc785\ud558\uace0 1998\ub144 7\uc6d4 29\uc77c\uc5d0 \uc774 \uc911 18\ub9cc\ub2ec\ub7ec (2\uc5b5 5\ucc9c\ub9cc\uc6d0)\ub97c \uac1c\uc778 \ubaa9\uc801\uc73c\ub85c \uc0ac\uc6a9\ud558\uae30 \uc704\ud574 3\uac1c\ub97c \ub2f9\uc88c\uc218\ud45c\ub85c \ubd84\uc0b0\ud558\uc5ec \ud604\uae08\ud654\ud55c\uac83\uc73c\ub85c \ubc1d\ud600\uc84c\ub2e4. \ub610, \uc870\uc911\ud6c8 \uc77c\uac00\ub294 \uc774\ub54c \ubc1b\uc740 \ub9ac\ubca0\uc774\ud2b8\ub97c \uc870\uc138 \ud68c\ud53c \uc9c0\uc5ed\uc778 ' \uc544\uc77c\ub79c\ub4dc \ub354\ube14\ub9b0 ' \uc9c0\uc5ed\uc5d0 100\ub9cc\ub2ec\ub7ec\ub97c \ucd9c\uc790\ud574 \uc124\ub9bd\ud55c \ud604\uc9c0 \ubc95\uc778 KA\uc0ac\uc5d0 \uc774\uc804\ud558\uc5ec 1\uc5b5 8\ucc9c 400\ub9cc \ub2ec\ub7ec\uac00 \ud574\uc678\ub85c \uc720\ucd9c\ub410\uace0, \uc774\ub85c \uc778\ud574 814\uc5b5\uc6d0\uc744 \ud0c8\uc138\ud55c \ud610\uc758\uac00 \ubc1d\ud600\uc84c\ub2e4. ", "paragraph_answer": "1991\ub144 \ubd80\ud130 1998\ub144\uae4c\uc9c0 \uc678\uad6d \uae30\uc5c5 \ub450 \uacf3\uc758 \ud56d\uacf5\uae30\ub97c \uad6c\ub9e4\ud560\ub54c \ud2b9\uc815 \ud68c\uc0ac\uc758 \uc5d4\uc9c4\uc744 \uc7a5\ucc29\ud558\ub294 \uc870\uac74\uc73c\ub85c \ub9ac\ubca0\uc774\ud2b8\ub97c \ubc1b\uc558\ub2e4. \uc774 \uc911 \uc77c\ubd80\uc778 1\ucc9c685\uc5b5\uc6d0\uc744 \uad6d\ub0b4\ub85c \ub4e4\uc5ec\uc640 \uc870\uc911\ud6c8 \uba85\uc608 \ud68c\uc7a5\uacfc \uc870\uc591\ud638 \ud68c\uc7a5 \ub4f1\uc774 \uac1c\uc778\uc801\uc73c\ub85c \uc0ac\uc6a9\ud588\ub2e4. \uc9c0\ub09c 1997\ub144 11\uc6d4 26\uc77c 600\ub9cc\ub2ec\ub7ec\uc758 \ub9ac\ubca0\uc774\ud2b8\ub97c \uad6d\ub0b4\ub85c \ubc18\uc785\ud558\uace0 1998\ub144 7\uc6d4 29\uc77c\uc5d0 \uc774 \uc911 18\ub9cc\ub2ec\ub7ec (2\uc5b5 5\ucc9c\ub9cc\uc6d0)\ub97c \uac1c\uc778 \ubaa9\uc801\uc73c\ub85c \uc0ac\uc6a9\ud558\uae30 \uc704\ud574 3\uac1c\ub97c \ub2f9\uc88c\uc218\ud45c\ub85c \ubd84\uc0b0\ud558\uc5ec \ud604\uae08\ud654\ud55c\uac83\uc73c\ub85c \ubc1d\ud600\uc84c\ub2e4. \ub610, \uc870\uc911\ud6c8 \uc77c\uac00\ub294 \uc774\ub54c \ubc1b\uc740 \ub9ac\ubca0\uc774\ud2b8\ub97c \uc870\uc138 \ud68c\ud53c \uc9c0\uc5ed\uc778 ' \uc544\uc77c\ub79c\ub4dc \ub354\ube14\ub9b0 ' \uc9c0\uc5ed\uc5d0 100\ub9cc\ub2ec\ub7ec\ub97c \ucd9c\uc790\ud574 \uc124\ub9bd\ud55c \ud604\uc9c0 \ubc95\uc778 KA\uc0ac\uc5d0 \uc774\uc804\ud558\uc5ec 1\uc5b5 8\ucc9c 400\ub9cc \ub2ec\ub7ec\uac00 \ud574\uc678\ub85c \uc720\ucd9c\ub410\uace0, \uc774\ub85c \uc778\ud574 814\uc5b5\uc6d0\uc744 \ud0c8\uc138\ud55c \ud610\uc758\uac00 \ubc1d\ud600\uc84c\ub2e4.", "sentence_answer": "\ub610, \uc870\uc911\ud6c8 \uc77c\uac00\ub294 \uc774\ub54c \ubc1b\uc740 \ub9ac\ubca0\uc774\ud2b8\ub97c \uc870\uc138 \ud68c\ud53c \uc9c0\uc5ed\uc778 ' \uc544\uc77c\ub79c\ub4dc \ub354\ube14\ub9b0 ' \uc9c0\uc5ed\uc5d0 100\ub9cc\ub2ec\ub7ec\ub97c \ucd9c\uc790\ud574 \uc124\ub9bd\ud55c \ud604\uc9c0 \ubc95\uc778 KA\uc0ac\uc5d0 \uc774\uc804\ud558\uc5ec 1\uc5b5 8\ucc9c 400\ub9cc \ub2ec\ub7ec\uac00 \ud574\uc678\ub85c \uc720\ucd9c\ub410\uace0, \uc774\ub85c \uc778\ud574 814\uc5b5\uc6d0\uc744 \ud0c8\uc138\ud55c \ud610\uc758\uac00 \ubc1d\ud600\uc84c\ub2e4.", "paragraph_id": "6553326-0-0", "paragraph_question": "question: \uc870\uc911\ud6c8 \uc77c\uac00\uac00 100\ub9cc \ub2ec\ub7ec\ub97c \ucd9c\uc790\ud574 \uc124\ub9bd\ud55c KA\uc0ac\uac00 \uc788\ub294 \uc870\uc138 \ud68c\ud53c \uc9c0\uc5ed\uc740?, context: 1991\ub144 \ubd80\ud130 1998\ub144\uae4c\uc9c0 \uc678\uad6d \uae30\uc5c5 \ub450 \uacf3\uc758 \ud56d\uacf5\uae30\ub97c \uad6c\ub9e4\ud560\ub54c \ud2b9\uc815 \ud68c\uc0ac\uc758 \uc5d4\uc9c4\uc744 \uc7a5\ucc29\ud558\ub294 \uc870\uac74\uc73c\ub85c \ub9ac\ubca0\uc774\ud2b8\ub97c \ubc1b\uc558\ub2e4. \uc774 \uc911 \uc77c\ubd80\uc778 1\ucc9c685\uc5b5\uc6d0\uc744 \uad6d\ub0b4\ub85c \ub4e4\uc5ec\uc640 \uc870\uc911\ud6c8 \uba85\uc608 \ud68c\uc7a5\uacfc \uc870\uc591\ud638 \ud68c\uc7a5 \ub4f1\uc774 \uac1c\uc778\uc801\uc73c\ub85c \uc0ac\uc6a9\ud588\ub2e4. \uc9c0\ub09c 1997\ub144 11\uc6d4 26\uc77c 600\ub9cc\ub2ec\ub7ec\uc758 \ub9ac\ubca0\uc774\ud2b8\ub97c \uad6d\ub0b4\ub85c \ubc18\uc785\ud558\uace0 1998\ub144 7\uc6d4 29\uc77c\uc5d0 \uc774 \uc911 18\ub9cc\ub2ec\ub7ec (2\uc5b5 5\ucc9c\ub9cc\uc6d0)\ub97c \uac1c\uc778 \ubaa9\uc801\uc73c\ub85c \uc0ac\uc6a9\ud558\uae30 \uc704\ud574 3\uac1c\ub97c \ub2f9\uc88c\uc218\ud45c\ub85c \ubd84\uc0b0\ud558\uc5ec \ud604\uae08\ud654\ud55c\uac83\uc73c\ub85c \ubc1d\ud600\uc84c\ub2e4. \ub610, \uc870\uc911\ud6c8 \uc77c\uac00\ub294 \uc774\ub54c \ubc1b\uc740 \ub9ac\ubca0\uc774\ud2b8\ub97c \uc870\uc138 \ud68c\ud53c \uc9c0\uc5ed\uc778 '\uc544\uc77c\ub79c\ub4dc \ub354\ube14\ub9b0' \uc9c0\uc5ed\uc5d0 100\ub9cc\ub2ec\ub7ec\ub97c \ucd9c\uc790\ud574 \uc124\ub9bd\ud55c \ud604\uc9c0 \ubc95\uc778 KA\uc0ac\uc5d0 \uc774\uc804\ud558\uc5ec 1\uc5b5 8\ucc9c 400\ub9cc \ub2ec\ub7ec\uac00 \ud574\uc678\ub85c \uc720\ucd9c\ub410\uace0, \uc774\ub85c \uc778\ud574 814\uc5b5\uc6d0\uc744 \ud0c8\uc138\ud55c \ud610\uc758\uac00 \ubc1d\ud600\uc84c\ub2e4."} {"question": "\uc2a4\ud398\uc778\uc5d0 \ud30c\uacac\ub41c \uad50\ud669 \ud2b9\uc0ac \uc790\ub12c\ub85c\uac00 \uc870\uc0ac\ud55c \uac83\uc740?", "paragraph": "924\ub144 \uc694\ud55c 10\uc138\ub294 \ubaa8\uc790\ub77c\ube45 \uc804\ub840\ub97c \uc870\uc0ac\ud558\uae30 \uc704\ud574 \uad50\ud669 \ud2b9\uc0ac \uc790\ub12c\ub85c\ub97c \uc2a4\ud398\uc778\uc5d0 \ud30c\uacac\ud558\uc600\ub2e4. \uc2dc\ucc30\uc5d0\uc11c \ub3cc\uc544\uc628 \uc790\ub12c\ub85c\ub294 \ubaa8\uc790\ub77c\ube45 \uc804\ub840\uc5d0 \ub300\ud574 \uae0d\uc815\uc801\uc73c\ub85c \ub9d0\ud588\uc73c\uba70, \uc774\uc5d0 \uc694\ud55c 10\uc138\ub294 \ubaa8\uc790\ub77c\ube45 \uc804\ub840\ub97c \uc2b9\uc778\ud558\uc600\ub2e4. \ub2e4\ub9cc \uc131\ubcc0\ud654 \ubd80\ubd84\ub9cc\uc740 \ub85c\ub9c8 \uc804\ub840\ubb38 \uc591\uc2dd\uacfc \ub3d9\uc77c\ud558\uac8c \ubc14\uafb8\ub3c4\ub85d \ud588\ub2e4. \uc694\ud55c 10\uc138\uac00 \uc7ac\uc784\ud55c \uc2dc\uc808\uc5d0\ub294 \ub9ce\uc740 \uc789\uae00\ub79c\ub4dc\uc778\uc774 \ub85c\ub9c8\ub85c \uc131\uc9c0 \uc21c\ub840\ub97c \ud558\ub7ec \uc654\uc73c\uba70, \uadf8 \uc911\uc5d0\ub294 927\ub144 \ub85c\ub9c8\ub97c \ucc38\ubc30\ud558\uae30 \uc704\ud574 \uc628 \uce94\ud130\ubca0\ub9ac \ub300\uc8fc\uad50 \uc6b8\ud3a0\ub984\ub3c4 \uc788\uc5c8\ub2e4. 3\ub144 \uc804\uc778 924\ub144\uc5d0 \uc789\uae00\ub79c\ub4dc \uc655 \uc560\uc140\uc2a4\ud134\uc740 \uadc0\uc871 \uc911 \ud55c \uba85\uc778 \uc54c\ud504\ub808\ub4dc\uac00 \uc790\uc2e0\uc758 \ub208\uc744 \ubf51\uc73c\ub824\ub294 \uc74c\ubaa8\ub97c \uafb8\uba84\ub2e4\ub294 \ud610\uc758\ub85c \ub85c\ub9c8\ub85c \ubcf4\ub0c8\ub294\ub370, \uc54c\ud504\ub808\ub4dc\ub294 \uc694\ud55c 10\uc138 \uc55e\uc5d0 \ucd9c\ub450\ud558\uc5ec \uc790\uc2e0\uc758 \ud610\uc758\ub97c \ubd80\uc778\ud558\uae30\ub85c \ud558\uc600\uc73c\ub098 \ub85c\ub9c8\uc5d0 \ub3c4\ucc29\ud55c \uc9c1\ud6c4 \ubc14\ub85c \uc0ac\ub9dd\ud588\ub2e4. \ub610\ud55c 917\ub144 \uc694\ud55c 10\uc138\ub294 \ube0c\ub808\uba58\uc758 \ub300\uc8fc\uad50\uc5d0\uac8c \uc2a4\uc6e8\ub374\uacfc \ub374\ub9c8\ud06c, \ub178\ub974\uc6e8\uc774, \uc544\uc774\uc2ac\ub780\ub4dc, \uadf8\ub9b0\ub780\ub4dc\uc758 \uc8fc\uad50\ub4e4\uc5d0 \ub300\ud55c \uad00\ud560\uad8c\uc744 \ubd80\uc5ec\ud588\ub2e4.", "answer": "\ubaa8\uc790\ub77c\ube45 \uc804\ub840", "sentence": "924\ub144 \uc694\ud55c 10\uc138\ub294 \ubaa8\uc790\ub77c\ube45 \uc804\ub840 \ub97c \uc870\uc0ac\ud558\uae30 \uc704\ud574 \uad50\ud669 \ud2b9\uc0ac \uc790\ub12c\ub85c\ub97c \uc2a4\ud398\uc778\uc5d0 \ud30c\uacac\ud558\uc600\ub2e4.", "paragraph_sentence": " 924\ub144 \uc694\ud55c 10\uc138\ub294 \ubaa8\uc790\ub77c\ube45 \uc804\ub840 \ub97c \uc870\uc0ac\ud558\uae30 \uc704\ud574 \uad50\ud669 \ud2b9\uc0ac \uc790\ub12c\ub85c\ub97c \uc2a4\ud398\uc778\uc5d0 \ud30c\uacac\ud558\uc600\ub2e4. \uc2dc\ucc30\uc5d0\uc11c \ub3cc\uc544\uc628 \uc790\ub12c\ub85c\ub294 \ubaa8\uc790\ub77c\ube45 \uc804\ub840\uc5d0 \ub300\ud574 \uae0d\uc815\uc801\uc73c\ub85c \ub9d0\ud588\uc73c\uba70, \uc774\uc5d0 \uc694\ud55c 10\uc138\ub294 \ubaa8\uc790\ub77c\ube45 \uc804\ub840\ub97c \uc2b9\uc778\ud558\uc600\ub2e4. \ub2e4\ub9cc \uc131\ubcc0\ud654 \ubd80\ubd84\ub9cc\uc740 \ub85c\ub9c8 \uc804\ub840\ubb38 \uc591\uc2dd\uacfc \ub3d9\uc77c\ud558\uac8c \ubc14\uafb8\ub3c4\ub85d \ud588\ub2e4. \uc694\ud55c 10\uc138\uac00 \uc7ac\uc784\ud55c \uc2dc\uc808\uc5d0\ub294 \ub9ce\uc740 \uc789\uae00\ub79c\ub4dc\uc778\uc774 \ub85c\ub9c8\ub85c \uc131\uc9c0 \uc21c\ub840\ub97c \ud558\ub7ec \uc654\uc73c\uba70, \uadf8 \uc911\uc5d0\ub294 927\ub144 \ub85c\ub9c8\ub97c \ucc38\ubc30\ud558\uae30 \uc704\ud574 \uc628 \uce94\ud130\ubca0\ub9ac \ub300\uc8fc\uad50 \uc6b8\ud3a0\ub984\ub3c4 \uc788\uc5c8\ub2e4. 3\ub144 \uc804\uc778 924\ub144\uc5d0 \uc789\uae00\ub79c\ub4dc \uc655 \uc560\uc140\uc2a4\ud134\uc740 \uadc0\uc871 \uc911 \ud55c \uba85\uc778 \uc54c\ud504\ub808\ub4dc\uac00 \uc790\uc2e0\uc758 \ub208\uc744 \ubf51\uc73c\ub824\ub294 \uc74c\ubaa8\ub97c \uafb8\uba84\ub2e4\ub294 \ud610\uc758\ub85c \ub85c\ub9c8\ub85c \ubcf4\ub0c8\ub294\ub370, \uc54c\ud504\ub808\ub4dc\ub294 \uc694\ud55c 10\uc138 \uc55e\uc5d0 \ucd9c\ub450\ud558\uc5ec \uc790\uc2e0\uc758 \ud610\uc758\ub97c \ubd80\uc778\ud558\uae30\ub85c \ud558\uc600\uc73c\ub098 \ub85c\ub9c8\uc5d0 \ub3c4\ucc29\ud55c \uc9c1\ud6c4 \ubc14\ub85c \uc0ac\ub9dd\ud588\ub2e4. \ub610\ud55c 917\ub144 \uc694\ud55c 10\uc138\ub294 \ube0c\ub808\uba58\uc758 \ub300\uc8fc\uad50\uc5d0\uac8c \uc2a4\uc6e8\ub374\uacfc \ub374\ub9c8\ud06c, \ub178\ub974\uc6e8\uc774, \uc544\uc774\uc2ac\ub780\ub4dc, \uadf8\ub9b0\ub780\ub4dc\uc758 \uc8fc\uad50\ub4e4\uc5d0 \ub300\ud55c \uad00\ud560\uad8c\uc744 \ubd80\uc5ec\ud588\ub2e4.", "paragraph_answer": "924\ub144 \uc694\ud55c 10\uc138\ub294 \ubaa8\uc790\ub77c\ube45 \uc804\ub840 \ub97c \uc870\uc0ac\ud558\uae30 \uc704\ud574 \uad50\ud669 \ud2b9\uc0ac \uc790\ub12c\ub85c\ub97c \uc2a4\ud398\uc778\uc5d0 \ud30c\uacac\ud558\uc600\ub2e4. \uc2dc\ucc30\uc5d0\uc11c \ub3cc\uc544\uc628 \uc790\ub12c\ub85c\ub294 \ubaa8\uc790\ub77c\ube45 \uc804\ub840\uc5d0 \ub300\ud574 \uae0d\uc815\uc801\uc73c\ub85c \ub9d0\ud588\uc73c\uba70, \uc774\uc5d0 \uc694\ud55c 10\uc138\ub294 \ubaa8\uc790\ub77c\ube45 \uc804\ub840\ub97c \uc2b9\uc778\ud558\uc600\ub2e4. \ub2e4\ub9cc \uc131\ubcc0\ud654 \ubd80\ubd84\ub9cc\uc740 \ub85c\ub9c8 \uc804\ub840\ubb38 \uc591\uc2dd\uacfc \ub3d9\uc77c\ud558\uac8c \ubc14\uafb8\ub3c4\ub85d \ud588\ub2e4. \uc694\ud55c 10\uc138\uac00 \uc7ac\uc784\ud55c \uc2dc\uc808\uc5d0\ub294 \ub9ce\uc740 \uc789\uae00\ub79c\ub4dc\uc778\uc774 \ub85c\ub9c8\ub85c \uc131\uc9c0 \uc21c\ub840\ub97c \ud558\ub7ec \uc654\uc73c\uba70, \uadf8 \uc911\uc5d0\ub294 927\ub144 \ub85c\ub9c8\ub97c \ucc38\ubc30\ud558\uae30 \uc704\ud574 \uc628 \uce94\ud130\ubca0\ub9ac \ub300\uc8fc\uad50 \uc6b8\ud3a0\ub984\ub3c4 \uc788\uc5c8\ub2e4. 3\ub144 \uc804\uc778 924\ub144\uc5d0 \uc789\uae00\ub79c\ub4dc \uc655 \uc560\uc140\uc2a4\ud134\uc740 \uadc0\uc871 \uc911 \ud55c \uba85\uc778 \uc54c\ud504\ub808\ub4dc\uac00 \uc790\uc2e0\uc758 \ub208\uc744 \ubf51\uc73c\ub824\ub294 \uc74c\ubaa8\ub97c \uafb8\uba84\ub2e4\ub294 \ud610\uc758\ub85c \ub85c\ub9c8\ub85c \ubcf4\ub0c8\ub294\ub370, \uc54c\ud504\ub808\ub4dc\ub294 \uc694\ud55c 10\uc138 \uc55e\uc5d0 \ucd9c\ub450\ud558\uc5ec \uc790\uc2e0\uc758 \ud610\uc758\ub97c \ubd80\uc778\ud558\uae30\ub85c \ud558\uc600\uc73c\ub098 \ub85c\ub9c8\uc5d0 \ub3c4\ucc29\ud55c \uc9c1\ud6c4 \ubc14\ub85c \uc0ac\ub9dd\ud588\ub2e4. \ub610\ud55c 917\ub144 \uc694\ud55c 10\uc138\ub294 \ube0c\ub808\uba58\uc758 \ub300\uc8fc\uad50\uc5d0\uac8c \uc2a4\uc6e8\ub374\uacfc \ub374\ub9c8\ud06c, \ub178\ub974\uc6e8\uc774, \uc544\uc774\uc2ac\ub780\ub4dc, \uadf8\ub9b0\ub780\ub4dc\uc758 \uc8fc\uad50\ub4e4\uc5d0 \ub300\ud55c \uad00\ud560\uad8c\uc744 \ubd80\uc5ec\ud588\ub2e4.", "sentence_answer": "924\ub144 \uc694\ud55c 10\uc138\ub294 \ubaa8\uc790\ub77c\ube45 \uc804\ub840 \ub97c \uc870\uc0ac\ud558\uae30 \uc704\ud574 \uad50\ud669 \ud2b9\uc0ac \uc790\ub12c\ub85c\ub97c \uc2a4\ud398\uc778\uc5d0 \ud30c\uacac\ud558\uc600\ub2e4.", "paragraph_id": "6511685-7-0", "paragraph_question": "question: \uc2a4\ud398\uc778\uc5d0 \ud30c\uacac\ub41c \uad50\ud669 \ud2b9\uc0ac \uc790\ub12c\ub85c\uac00 \uc870\uc0ac\ud55c \uac83\uc740?, context: 924\ub144 \uc694\ud55c 10\uc138\ub294 \ubaa8\uc790\ub77c\ube45 \uc804\ub840\ub97c \uc870\uc0ac\ud558\uae30 \uc704\ud574 \uad50\ud669 \ud2b9\uc0ac \uc790\ub12c\ub85c\ub97c \uc2a4\ud398\uc778\uc5d0 \ud30c\uacac\ud558\uc600\ub2e4. \uc2dc\ucc30\uc5d0\uc11c \ub3cc\uc544\uc628 \uc790\ub12c\ub85c\ub294 \ubaa8\uc790\ub77c\ube45 \uc804\ub840\uc5d0 \ub300\ud574 \uae0d\uc815\uc801\uc73c\ub85c \ub9d0\ud588\uc73c\uba70, \uc774\uc5d0 \uc694\ud55c 10\uc138\ub294 \ubaa8\uc790\ub77c\ube45 \uc804\ub840\ub97c \uc2b9\uc778\ud558\uc600\ub2e4. \ub2e4\ub9cc \uc131\ubcc0\ud654 \ubd80\ubd84\ub9cc\uc740 \ub85c\ub9c8 \uc804\ub840\ubb38 \uc591\uc2dd\uacfc \ub3d9\uc77c\ud558\uac8c \ubc14\uafb8\ub3c4\ub85d \ud588\ub2e4. \uc694\ud55c 10\uc138\uac00 \uc7ac\uc784\ud55c \uc2dc\uc808\uc5d0\ub294 \ub9ce\uc740 \uc789\uae00\ub79c\ub4dc\uc778\uc774 \ub85c\ub9c8\ub85c \uc131\uc9c0 \uc21c\ub840\ub97c \ud558\ub7ec \uc654\uc73c\uba70, \uadf8 \uc911\uc5d0\ub294 927\ub144 \ub85c\ub9c8\ub97c \ucc38\ubc30\ud558\uae30 \uc704\ud574 \uc628 \uce94\ud130\ubca0\ub9ac \ub300\uc8fc\uad50 \uc6b8\ud3a0\ub984\ub3c4 \uc788\uc5c8\ub2e4. 3\ub144 \uc804\uc778 924\ub144\uc5d0 \uc789\uae00\ub79c\ub4dc \uc655 \uc560\uc140\uc2a4\ud134\uc740 \uadc0\uc871 \uc911 \ud55c \uba85\uc778 \uc54c\ud504\ub808\ub4dc\uac00 \uc790\uc2e0\uc758 \ub208\uc744 \ubf51\uc73c\ub824\ub294 \uc74c\ubaa8\ub97c \uafb8\uba84\ub2e4\ub294 \ud610\uc758\ub85c \ub85c\ub9c8\ub85c \ubcf4\ub0c8\ub294\ub370, \uc54c\ud504\ub808\ub4dc\ub294 \uc694\ud55c 10\uc138 \uc55e\uc5d0 \ucd9c\ub450\ud558\uc5ec \uc790\uc2e0\uc758 \ud610\uc758\ub97c \ubd80\uc778\ud558\uae30\ub85c \ud558\uc600\uc73c\ub098 \ub85c\ub9c8\uc5d0 \ub3c4\ucc29\ud55c \uc9c1\ud6c4 \ubc14\ub85c \uc0ac\ub9dd\ud588\ub2e4. \ub610\ud55c 917\ub144 \uc694\ud55c 10\uc138\ub294 \ube0c\ub808\uba58\uc758 \ub300\uc8fc\uad50\uc5d0\uac8c \uc2a4\uc6e8\ub374\uacfc \ub374\ub9c8\ud06c, \ub178\ub974\uc6e8\uc774, \uc544\uc774\uc2ac\ub780\ub4dc, \uadf8\ub9b0\ub780\ub4dc\uc758 \uc8fc\uad50\ub4e4\uc5d0 \ub300\ud55c \uad00\ud560\uad8c\uc744 \ubd80\uc5ec\ud588\ub2e4."} {"question": "\ud1b5\uc81c\ubd88\uac00, \uc5c9\ub9dd\uc9c4\ucc3d\uc778 \uad00\uc911\ub4e4\uc758 \ud0dc\ub3c4\uac00 \uae09\ubcc0\ud55c \uac83\uc740 \ub204\uac00 \ub3c4\ucc29\ud588\uc744 \ub54c\uc778\uac00?", "paragraph": "\ucd9c\uc785\ubb38\uc740 \uc54c\ub824\uc9c4 \ub300\ub85c \uacbd\uae30 \uc2dc\uc791 \uc2dc\uac04 3\uc2dc\uac04 30\ubd84 \uc804\uc778 \uc624\uc80411\uc2dc 30\ubd84\uc5d0 \uc5f4\ub838\uace0 \uc624\ud6c41\uc2dc\uae4c\uc9c0 \ubab0\ub824\ub4e0 \uad00\uc911\ub4e4\uc740 \uc9c8\uc11c\uc788\uac8c \uacbd\uae30\uc7a5\uc5d0 \uc785\uc7a5\ud588\ub2e4. \uadf8\ub7ec\ub098 1\uc2dc\uae4c\uc9c0 \ubab0\ub824\ub4e0 \uad00\uc911\ub4e4\uc774 \uac10\ub2f9\ud558\uc9c0 \ubabb\ud560 \uc815\ub3c4\ub85c \ubd88\uc5b4\ub098\uc790 \uacbd\uae30\uc7a5 \uad00\ub9ac\uce21\uc758 \uc870\uc0ac \ud6c4\uc5d0 \uc624\ud6c4 1\uc2dc 45\ubd84\uc5d0 \ucd9c\uc785\uad6c\ub97c \ub2eb\uace0 \uad00\uc911\uc758 \uc785\uc7a5\uc744 \ub9c9\uc558\ub2e4. \uad00\uc911\uc774\uc5c8\ub358 \uc70c\ub9ac\uc5c4 \ub85c\uc2a4\ub294 \uc62c\ub9bc\ud53d\uae38(Olympic Way)\uc774 \"\uc0ac\ub78c\ub4e4\ub85c \ub4e4\ub053\uc5c8\"\uace0 \"\uacbd\uae30\uc7a5\uc5d0 \uac00\uae4c\uc6cc\uc9c8\uc218\ub85d \uadf8 \uc815\ub3c4\uac00 \uc2ec\ud574\uc9c0\ub2e4 \uacb0\uad6d \uacbd\uae30\uc7a5\uc5d0 \ub3c4\ucc29\ud588\uc744 \ub54c\ub294 \ud68c\uc804\uc2dd \ubb38\uc774 \ub2eb\ud600\uc788\uc5c8\ub2e4\"\ub77c\uace0 \ub098\uc911\uc5d0 \ud68c\uace0\ud558\uae30\ub3c4 \ud588\ub2e4. \uacbd\uae30\uc7a5 \uc0c1\ud0dc\uc5d0 \ub300\ud55c \uc815\ubcf4\uac00 \uc5ec\ub7ec \uae30\ucc28\uc5ed\uc5d0 \uc804\ub2ec\ub418\uc5c8\uc9c0\ub9cc, \uba87 \ucc9c \uba85\uc758 \uc0ac\ub78c\ub4e4\uc774 \uacc4\uc18d \uacbd\uae30\uc7a5\uc5d0 \ub3c4\ucc29\ud574 \ucd9c\uc785\uad6c \uc8fc\ubcc0\uc5d0 \ud070 \ud63c\ub780\uc774 \uc77c\uc5b4\ub0ac\ub2e4. \uacbd\uae30\uc7a5\uc5d0\uc11c\uc758 \uacbd\uae30 \uc900\ube44\ub294 \ubd80\uc2e4\ud588\uc5c8\ub294\ub370 \u300a\ub370\uc77c\ub9ac \uba54\uc77c\u300b\uc758 \uae30\uc790\ub294 \uacbd\uae30\uc7a5\uc758 \uc0c1\ud0dc\uc5d0 \ub300\ud574 \uad00\ub9ac\uc778\ub4e4\uc740 \"\uc4f8\ubaa8\uac00 \uc5c6\uc5c8\"\uace0 \uacbd\uae30\uc7a5 \uce21 \uc0ac\ub78c\ub4e4\uc740 \uacbd\uae30\uc7a5 \ubc16\uacfc \uc548\uc758 \uc0c1\ud669\uc5d0 \ub300\ud574 \"\uc544\ubb34\uac83\ub3c4 \ubaa8\ub974\ub294 \uac83 \ucc98\ub7fc \ubcf4\uc600\ub2e4\"\uace0 \uc790\uc2e0\uc758 \ub9ac\ud3ec\ud2b8\uc5d0 \uc801\uae30\ub3c4 \ud588\ub2e4. \uacbd\uae30\uc7a5\uc744 \ucc3e\uc740 \ud32c\ub4e4\uc740 \uc544\ubb34\ub7f0 \ub9c8\ub828\ub41c \uc7a5\uc18c\ub97c \uc18c\uac1c\ubc1b\uc9c0 \ubabb\ud588\uace0, \uad00\uc911\uc11d\uc758 \uc544\ub798\uce35\uc744 \uc704\uce35\ubcf4\ub2e4 \uba87 \ubc30\ub098 \ube60\ub974\uac8c \ucc44\uc6e0\ub2e4. \uacbd\uae30\uc7a5 \ubc16\uc758 \uc778\ud30c\uac00 \uc810\uc810 \ub298\uc5b4\ub098\uc11c \uc9c0\uc5ed \uacbd\ucc30\uae4c\uc9c0 \ub3d9\uc6d0\ub418\uc5c8\uc9c0\ub9cc \uc774\ubbf8 \uc778\ud30c\uac00 \ub108\ubb34 \ucee4\uc838\ubc84\ub9ac\ub294 \ubc14\ub78c\uc5d0 \uacbd\ucc30\uc740 \ucde8\ud560 \uc218 \uc788\ub294 \ud589\ub3d9\uc774 \uc544\ubb34\uac83\ub3c4 \uc5c6\ub2e4\uace0 \ud310\ub2e8\ud588\ub2e4. \uc624\ud6c42\uc2dc 15\ubd84, \uacbd\uae30\uc7a5 \ubc16\uc758 \uc0ac\ub78c\ub4e4\uc740 \uacbd\uae30\uc7a5\uc758 \ubcbd\uc744 \ud5a5\ud574 \ub3cc\uc9c4\ud574\uc11c \uc5b5\uc9c0\ub85c \uacbd\uae30\uc7a5\uc5d0 \uc9c4\uc785\ud558\ub824\uace0 \ud588\ub2e4. \uc544\ub798\uce35\uc5d0 \uc788\ub358 \uad00\uc911\ub4e4\uc740 \uc774 \uad70\uc911\uc744 \ud53c\ud558\uae30 \uc704\ud574 \uc774 \ud39c\uc2a4\ub97c \ub118\uc5b4\uc11c \uadf8\ub77c\uc6b4\ub4dc\uae4c\uc9c0 \ub4e4\uc5b4\uac00\uac8c \ub418\uc5c8\ub2e4. \ub2f9\uc2dc \uad00\uc911 \ud14c\ub9ac \ud788\ud0a4\ub294 \"\uc904\uc5ec\uc11c \ub9d0\ud558\uc790\uba74, \ubaa8\ub4e0 \uac83\ub4e4\uc774 \uc9dc\uc99d\ub098\ub294 \ub09c\uc7a5\ud310\uc774\uc5c8\ub2e4.\"\ub77c\uba70 \ud6d7\ub0a0 \ub2f9\uc2dc \uc0c1\ud669\uc758 \uadf9\ub2e8\uc131\uc5d0 \ub300\ud574\uc11c \uc124\uba85\ud558\uae30\ub3c4 \ud588\ub2e4. \uacf5\uc2dd\uc801\uc778 \uad00\uc911 \uc218\ub294 126,047\uba85\uc73c\ub85c \uc9d1\uacc4\ub418\uc5c8\uc9c0\ub9cc \uc2e4\uc81c\ub85c \uc785\uc7a5\ud55c \uad00\uc911 \uc218\ub294 15\ub9cc\uba85\uc5d0\uc11c 30\ub9cc\uba85\uae4c\uc9c0 \uc774\ub97c \uac83\uc73c\ub85c \ucd94\uce21\ub41c\ub2e4. \uc789\uae00\ub79c\ub4dc \ucd95\uad6c\ud611\ud68c\ub294 \uc785\uc7a5\uad8c\uc744 \uc608\ub9e4\ud588\uc73c\ub098 \uc9c0\uc815\ub41c \uc88c\uc11d\uc5d0 \uc549\uc744 \uc218 \uc5c6\uc5c8\ub358 \uad00\uc911\ub4e4\uc5d0\uac8c \ud45c \uac12\uc758 10%\ub97c \ud658\ubd88\ud588\ub2e4. \uacbd\uae30\uc7a5 \uc8fc\ubcc0 \ub3c4\ub85c\ub4e4\uc774 \ubaa8\ub450 \ucc28\ub2e8\ub418\ub294 \ubc14\ub78c\uc5d0 \ubcfc\ud134\uc758 \uc120\uc218\ub4e4\uc740 \uacbd\uae30\uc7a5\uc5d0\uc11c 1\ub9c8\uc77c \ub5a8\uc5b4\uc9c4 \uacf3\uc5d0\uc11c \ubd80\ud130 \ucf54\uce58\uc640 \ub530\ub85c \ub5a8\uc5b4\uc838 \uc778\ud30c \uc18d\uc744 \ub6ab\uace0 \uacbd\uae30\uc7a5\uc5d0 \uc9c4\uc785\ud588\ub2e4. \u300a\ud0c0\uc784\u300b\uc740 \ud55c \ub54c \uacbd\uae30\uac00 \uc5f4\ub9ac\ub294 \uac83 \uc790\uccb4\uac00 \ubd88\uac00\ub2a5\ud574\ubcf4\uc600\uc73c\ub098 \uc601\uad6d\uc758 \uc655 \uc870\uc9c0 5\uc138\uac00 \ub3c4\ucc29\ud558\uc790 \uad00\uc911\ub4e4 \uc0ac\uc774\uc758 \ubd84\uc704\uae30\uac00 \ubc14\ub00c\uc5c8\ub2e4\uace0 \uae30\ub85d\ud588\ub2e4. \uad00\uc911\ub4e4\uc740 \uc5f4\ub82c\ud788 \uc601\uad6d\uc758 \uad6d\uac00\uc778 \u3008\ud558\ub290\ub2d8, \uad6d\uc655 \ud3d0\ud558\ub97c \uc9c0\ucf1c\uc8fc\uc18c\uc11c\u3009\ub97c \ub530\ub77c \ubd80\ub978 \ub4a4\uc5d0 \uc120\uc218\ub4e4\uc774 \uacbd\uae30\ud558\ub294 \uacf5\uac04\uc744 \uc815\ub9ac\ud558\ub294 \uac83\uc744 \ub3d5\uae30 \uc2dc\uc791\ud588\ub2e4.", "answer": "\uc601\uad6d\uc758 \uc655 \uc870\uc9c0 5\uc138", "sentence": "\u300a\ud0c0\uc784\u300b\uc740 \ud55c \ub54c \uacbd\uae30\uac00 \uc5f4\ub9ac\ub294 \uac83 \uc790\uccb4\uac00 \ubd88\uac00\ub2a5\ud574\ubcf4\uc600\uc73c\ub098 \uc601\uad6d\uc758 \uc655 \uc870\uc9c0 5\uc138 \uac00 \ub3c4\ucc29\ud558\uc790 \uad00\uc911\ub4e4 \uc0ac\uc774\uc758 \ubd84\uc704\uae30\uac00 \ubc14\ub00c\uc5c8\ub2e4\uace0 \uae30\ub85d\ud588\ub2e4.", "paragraph_sentence": "\ucd9c\uc785\ubb38\uc740 \uc54c\ub824\uc9c4 \ub300\ub85c \uacbd\uae30 \uc2dc\uc791 \uc2dc\uac04 3\uc2dc\uac04 30\ubd84 \uc804\uc778 \uc624\uc80411\uc2dc 30\ubd84\uc5d0 \uc5f4\ub838\uace0 \uc624\ud6c41\uc2dc\uae4c\uc9c0 \ubab0\ub824\ub4e0 \uad00\uc911\ub4e4\uc740 \uc9c8\uc11c\uc788\uac8c \uacbd\uae30\uc7a5\uc5d0 \uc785\uc7a5\ud588\ub2e4. \uadf8\ub7ec\ub098 1\uc2dc\uae4c\uc9c0 \ubab0\ub824\ub4e0 \uad00\uc911\ub4e4\uc774 \uac10\ub2f9\ud558\uc9c0 \ubabb\ud560 \uc815\ub3c4\ub85c \ubd88\uc5b4\ub098\uc790 \uacbd\uae30\uc7a5 \uad00\ub9ac\uce21\uc758 \uc870\uc0ac \ud6c4\uc5d0 \uc624\ud6c4 1\uc2dc 45\ubd84\uc5d0 \ucd9c\uc785\uad6c\ub97c \ub2eb\uace0 \uad00\uc911\uc758 \uc785\uc7a5\uc744 \ub9c9\uc558\ub2e4. \uad00\uc911\uc774\uc5c8\ub358 \uc70c\ub9ac\uc5c4 \ub85c\uc2a4\ub294 \uc62c\ub9bc\ud53d\uae38(Olympic Way)\uc774 \"\uc0ac\ub78c\ub4e4\ub85c \ub4e4\ub053\uc5c8\"\uace0 \"\uacbd\uae30\uc7a5\uc5d0 \uac00\uae4c\uc6cc\uc9c8\uc218\ub85d \uadf8 \uc815\ub3c4\uac00 \uc2ec\ud574\uc9c0\ub2e4 \uacb0\uad6d \uacbd\uae30\uc7a5\uc5d0 \ub3c4\ucc29\ud588\uc744 \ub54c\ub294 \ud68c\uc804\uc2dd \ubb38\uc774 \ub2eb\ud600\uc788\uc5c8\ub2e4\"\ub77c\uace0 \ub098\uc911\uc5d0 \ud68c\uace0\ud558\uae30\ub3c4 \ud588\ub2e4. \uacbd\uae30\uc7a5 \uc0c1\ud0dc\uc5d0 \ub300\ud55c \uc815\ubcf4\uac00 \uc5ec\ub7ec \uae30\ucc28\uc5ed\uc5d0 \uc804\ub2ec\ub418\uc5c8\uc9c0\ub9cc, \uba87 \ucc9c \uba85\uc758 \uc0ac\ub78c\ub4e4\uc774 \uacc4\uc18d \uacbd\uae30\uc7a5\uc5d0 \ub3c4\ucc29\ud574 \ucd9c\uc785\uad6c \uc8fc\ubcc0\uc5d0 \ud070 \ud63c\ub780\uc774 \uc77c\uc5b4\ub0ac\ub2e4. \uacbd\uae30\uc7a5\uc5d0\uc11c\uc758 \uacbd\uae30 \uc900\ube44\ub294 \ubd80\uc2e4\ud588\uc5c8\ub294\ub370 \u300a\ub370\uc77c\ub9ac \uba54\uc77c\u300b\uc758 \uae30\uc790\ub294 \uacbd\uae30\uc7a5\uc758 \uc0c1\ud0dc\uc5d0 \ub300\ud574 \uad00\ub9ac\uc778\ub4e4\uc740 \"\uc4f8\ubaa8\uac00 \uc5c6\uc5c8\"\uace0 \uacbd\uae30\uc7a5 \uce21 \uc0ac\ub78c\ub4e4\uc740 \uacbd\uae30\uc7a5 \ubc16\uacfc \uc548\uc758 \uc0c1\ud669\uc5d0 \ub300\ud574 \"\uc544\ubb34\uac83\ub3c4 \ubaa8\ub974\ub294 \uac83 \ucc98\ub7fc \ubcf4\uc600\ub2e4\"\uace0 \uc790\uc2e0\uc758 \ub9ac\ud3ec\ud2b8\uc5d0 \uc801\uae30\ub3c4 \ud588\ub2e4. \uacbd\uae30\uc7a5\uc744 \ucc3e\uc740 \ud32c\ub4e4\uc740 \uc544\ubb34\ub7f0 \ub9c8\ub828\ub41c \uc7a5\uc18c\ub97c \uc18c\uac1c\ubc1b\uc9c0 \ubabb\ud588\uace0, \uad00\uc911\uc11d\uc758 \uc544\ub798\uce35\uc744 \uc704\uce35\ubcf4\ub2e4 \uba87 \ubc30\ub098 \ube60\ub974\uac8c \ucc44\uc6e0\ub2e4. \uacbd\uae30\uc7a5 \ubc16\uc758 \uc778\ud30c\uac00 \uc810\uc810 \ub298\uc5b4\ub098\uc11c \uc9c0\uc5ed \uacbd\ucc30\uae4c\uc9c0 \ub3d9\uc6d0\ub418\uc5c8\uc9c0\ub9cc \uc774\ubbf8 \uc778\ud30c\uac00 \ub108\ubb34 \ucee4\uc838\ubc84\ub9ac\ub294 \ubc14\ub78c\uc5d0 \uacbd\ucc30\uc740 \ucde8\ud560 \uc218 \uc788\ub294 \ud589\ub3d9\uc774 \uc544\ubb34\uac83\ub3c4 \uc5c6\ub2e4\uace0 \ud310\ub2e8\ud588\ub2e4. \uc624\ud6c42\uc2dc 15\ubd84, \uacbd\uae30\uc7a5 \ubc16\uc758 \uc0ac\ub78c\ub4e4\uc740 \uacbd\uae30\uc7a5\uc758 \ubcbd\uc744 \ud5a5\ud574 \ub3cc\uc9c4\ud574\uc11c \uc5b5\uc9c0\ub85c \uacbd\uae30\uc7a5\uc5d0 \uc9c4\uc785\ud558\ub824\uace0 \ud588\ub2e4. \uc544\ub798\uce35\uc5d0 \uc788\ub358 \uad00\uc911\ub4e4\uc740 \uc774 \uad70\uc911\uc744 \ud53c\ud558\uae30 \uc704\ud574 \uc774 \ud39c\uc2a4\ub97c \ub118\uc5b4\uc11c \uadf8\ub77c\uc6b4\ub4dc\uae4c\uc9c0 \ub4e4\uc5b4\uac00\uac8c \ub418\uc5c8\ub2e4. \ub2f9\uc2dc \uad00\uc911 \ud14c\ub9ac \ud788\ud0a4\ub294 \"\uc904\uc5ec\uc11c \ub9d0\ud558\uc790\uba74, \ubaa8\ub4e0 \uac83\ub4e4\uc774 \uc9dc\uc99d\ub098\ub294 \ub09c\uc7a5\ud310\uc774\uc5c8\ub2e4. \"\ub77c\uba70 \ud6d7\ub0a0 \ub2f9\uc2dc \uc0c1\ud669\uc758 \uadf9\ub2e8\uc131\uc5d0 \ub300\ud574\uc11c \uc124\uba85\ud558\uae30\ub3c4 \ud588\ub2e4. \uacf5\uc2dd\uc801\uc778 \uad00\uc911 \uc218\ub294 126,047\uba85\uc73c\ub85c \uc9d1\uacc4\ub418\uc5c8\uc9c0\ub9cc \uc2e4\uc81c\ub85c \uc785\uc7a5\ud55c \uad00\uc911 \uc218\ub294 15\ub9cc\uba85\uc5d0\uc11c 30\ub9cc\uba85\uae4c\uc9c0 \uc774\ub97c \uac83\uc73c\ub85c \ucd94\uce21\ub41c\ub2e4. \uc789\uae00\ub79c\ub4dc \ucd95\uad6c\ud611\ud68c\ub294 \uc785\uc7a5\uad8c\uc744 \uc608\ub9e4\ud588\uc73c\ub098 \uc9c0\uc815\ub41c \uc88c\uc11d\uc5d0 \uc549\uc744 \uc218 \uc5c6\uc5c8\ub358 \uad00\uc911\ub4e4\uc5d0\uac8c \ud45c \uac12\uc758 10%\ub97c \ud658\ubd88\ud588\ub2e4. \uacbd\uae30\uc7a5 \uc8fc\ubcc0 \ub3c4\ub85c\ub4e4\uc774 \ubaa8\ub450 \ucc28\ub2e8\ub418\ub294 \ubc14\ub78c\uc5d0 \ubcfc\ud134\uc758 \uc120\uc218\ub4e4\uc740 \uacbd\uae30\uc7a5\uc5d0\uc11c 1\ub9c8\uc77c \ub5a8\uc5b4\uc9c4 \uacf3\uc5d0\uc11c \ubd80\ud130 \ucf54\uce58\uc640 \ub530\ub85c \ub5a8\uc5b4\uc838 \uc778\ud30c \uc18d\uc744 \ub6ab\uace0 \uacbd\uae30\uc7a5\uc5d0 \uc9c4\uc785\ud588\ub2e4. \u300a\ud0c0\uc784\u300b\uc740 \ud55c \ub54c \uacbd\uae30\uac00 \uc5f4\ub9ac\ub294 \uac83 \uc790\uccb4\uac00 \ubd88\uac00\ub2a5\ud574\ubcf4\uc600\uc73c\ub098 \uc601\uad6d\uc758 \uc655 \uc870\uc9c0 5\uc138 \uac00 \ub3c4\ucc29\ud558\uc790 \uad00\uc911\ub4e4 \uc0ac\uc774\uc758 \ubd84\uc704\uae30\uac00 \ubc14\ub00c\uc5c8\ub2e4\uace0 \uae30\ub85d\ud588\ub2e4. \uad00\uc911\ub4e4\uc740 \uc5f4\ub82c\ud788 \uc601\uad6d\uc758 \uad6d\uac00\uc778 \u3008\ud558\ub290\ub2d8, \uad6d\uc655 \ud3d0\ud558\ub97c \uc9c0\ucf1c\uc8fc\uc18c\uc11c\u3009\ub97c \ub530\ub77c \ubd80\ub978 \ub4a4\uc5d0 \uc120\uc218\ub4e4\uc774 \uacbd\uae30\ud558\ub294 \uacf5\uac04\uc744 \uc815\ub9ac\ud558\ub294 \uac83\uc744 \ub3d5\uae30 \uc2dc\uc791\ud588\ub2e4.", "paragraph_answer": "\ucd9c\uc785\ubb38\uc740 \uc54c\ub824\uc9c4 \ub300\ub85c \uacbd\uae30 \uc2dc\uc791 \uc2dc\uac04 3\uc2dc\uac04 30\ubd84 \uc804\uc778 \uc624\uc80411\uc2dc 30\ubd84\uc5d0 \uc5f4\ub838\uace0 \uc624\ud6c41\uc2dc\uae4c\uc9c0 \ubab0\ub824\ub4e0 \uad00\uc911\ub4e4\uc740 \uc9c8\uc11c\uc788\uac8c \uacbd\uae30\uc7a5\uc5d0 \uc785\uc7a5\ud588\ub2e4. \uadf8\ub7ec\ub098 1\uc2dc\uae4c\uc9c0 \ubab0\ub824\ub4e0 \uad00\uc911\ub4e4\uc774 \uac10\ub2f9\ud558\uc9c0 \ubabb\ud560 \uc815\ub3c4\ub85c \ubd88\uc5b4\ub098\uc790 \uacbd\uae30\uc7a5 \uad00\ub9ac\uce21\uc758 \uc870\uc0ac \ud6c4\uc5d0 \uc624\ud6c4 1\uc2dc 45\ubd84\uc5d0 \ucd9c\uc785\uad6c\ub97c \ub2eb\uace0 \uad00\uc911\uc758 \uc785\uc7a5\uc744 \ub9c9\uc558\ub2e4. \uad00\uc911\uc774\uc5c8\ub358 \uc70c\ub9ac\uc5c4 \ub85c\uc2a4\ub294 \uc62c\ub9bc\ud53d\uae38(Olympic Way)\uc774 \"\uc0ac\ub78c\ub4e4\ub85c \ub4e4\ub053\uc5c8\"\uace0 \"\uacbd\uae30\uc7a5\uc5d0 \uac00\uae4c\uc6cc\uc9c8\uc218\ub85d \uadf8 \uc815\ub3c4\uac00 \uc2ec\ud574\uc9c0\ub2e4 \uacb0\uad6d \uacbd\uae30\uc7a5\uc5d0 \ub3c4\ucc29\ud588\uc744 \ub54c\ub294 \ud68c\uc804\uc2dd \ubb38\uc774 \ub2eb\ud600\uc788\uc5c8\ub2e4\"\ub77c\uace0 \ub098\uc911\uc5d0 \ud68c\uace0\ud558\uae30\ub3c4 \ud588\ub2e4. \uacbd\uae30\uc7a5 \uc0c1\ud0dc\uc5d0 \ub300\ud55c \uc815\ubcf4\uac00 \uc5ec\ub7ec \uae30\ucc28\uc5ed\uc5d0 \uc804\ub2ec\ub418\uc5c8\uc9c0\ub9cc, \uba87 \ucc9c \uba85\uc758 \uc0ac\ub78c\ub4e4\uc774 \uacc4\uc18d \uacbd\uae30\uc7a5\uc5d0 \ub3c4\ucc29\ud574 \ucd9c\uc785\uad6c \uc8fc\ubcc0\uc5d0 \ud070 \ud63c\ub780\uc774 \uc77c\uc5b4\ub0ac\ub2e4. \uacbd\uae30\uc7a5\uc5d0\uc11c\uc758 \uacbd\uae30 \uc900\ube44\ub294 \ubd80\uc2e4\ud588\uc5c8\ub294\ub370 \u300a\ub370\uc77c\ub9ac \uba54\uc77c\u300b\uc758 \uae30\uc790\ub294 \uacbd\uae30\uc7a5\uc758 \uc0c1\ud0dc\uc5d0 \ub300\ud574 \uad00\ub9ac\uc778\ub4e4\uc740 \"\uc4f8\ubaa8\uac00 \uc5c6\uc5c8\"\uace0 \uacbd\uae30\uc7a5 \uce21 \uc0ac\ub78c\ub4e4\uc740 \uacbd\uae30\uc7a5 \ubc16\uacfc \uc548\uc758 \uc0c1\ud669\uc5d0 \ub300\ud574 \"\uc544\ubb34\uac83\ub3c4 \ubaa8\ub974\ub294 \uac83 \ucc98\ub7fc \ubcf4\uc600\ub2e4\"\uace0 \uc790\uc2e0\uc758 \ub9ac\ud3ec\ud2b8\uc5d0 \uc801\uae30\ub3c4 \ud588\ub2e4. \uacbd\uae30\uc7a5\uc744 \ucc3e\uc740 \ud32c\ub4e4\uc740 \uc544\ubb34\ub7f0 \ub9c8\ub828\ub41c \uc7a5\uc18c\ub97c \uc18c\uac1c\ubc1b\uc9c0 \ubabb\ud588\uace0, \uad00\uc911\uc11d\uc758 \uc544\ub798\uce35\uc744 \uc704\uce35\ubcf4\ub2e4 \uba87 \ubc30\ub098 \ube60\ub974\uac8c \ucc44\uc6e0\ub2e4. \uacbd\uae30\uc7a5 \ubc16\uc758 \uc778\ud30c\uac00 \uc810\uc810 \ub298\uc5b4\ub098\uc11c \uc9c0\uc5ed \uacbd\ucc30\uae4c\uc9c0 \ub3d9\uc6d0\ub418\uc5c8\uc9c0\ub9cc \uc774\ubbf8 \uc778\ud30c\uac00 \ub108\ubb34 \ucee4\uc838\ubc84\ub9ac\ub294 \ubc14\ub78c\uc5d0 \uacbd\ucc30\uc740 \ucde8\ud560 \uc218 \uc788\ub294 \ud589\ub3d9\uc774 \uc544\ubb34\uac83\ub3c4 \uc5c6\ub2e4\uace0 \ud310\ub2e8\ud588\ub2e4. \uc624\ud6c42\uc2dc 15\ubd84, \uacbd\uae30\uc7a5 \ubc16\uc758 \uc0ac\ub78c\ub4e4\uc740 \uacbd\uae30\uc7a5\uc758 \ubcbd\uc744 \ud5a5\ud574 \ub3cc\uc9c4\ud574\uc11c \uc5b5\uc9c0\ub85c \uacbd\uae30\uc7a5\uc5d0 \uc9c4\uc785\ud558\ub824\uace0 \ud588\ub2e4. \uc544\ub798\uce35\uc5d0 \uc788\ub358 \uad00\uc911\ub4e4\uc740 \uc774 \uad70\uc911\uc744 \ud53c\ud558\uae30 \uc704\ud574 \uc774 \ud39c\uc2a4\ub97c \ub118\uc5b4\uc11c \uadf8\ub77c\uc6b4\ub4dc\uae4c\uc9c0 \ub4e4\uc5b4\uac00\uac8c \ub418\uc5c8\ub2e4. \ub2f9\uc2dc \uad00\uc911 \ud14c\ub9ac \ud788\ud0a4\ub294 \"\uc904\uc5ec\uc11c \ub9d0\ud558\uc790\uba74, \ubaa8\ub4e0 \uac83\ub4e4\uc774 \uc9dc\uc99d\ub098\ub294 \ub09c\uc7a5\ud310\uc774\uc5c8\ub2e4.\"\ub77c\uba70 \ud6d7\ub0a0 \ub2f9\uc2dc \uc0c1\ud669\uc758 \uadf9\ub2e8\uc131\uc5d0 \ub300\ud574\uc11c \uc124\uba85\ud558\uae30\ub3c4 \ud588\ub2e4. \uacf5\uc2dd\uc801\uc778 \uad00\uc911 \uc218\ub294 126,047\uba85\uc73c\ub85c \uc9d1\uacc4\ub418\uc5c8\uc9c0\ub9cc \uc2e4\uc81c\ub85c \uc785\uc7a5\ud55c \uad00\uc911 \uc218\ub294 15\ub9cc\uba85\uc5d0\uc11c 30\ub9cc\uba85\uae4c\uc9c0 \uc774\ub97c \uac83\uc73c\ub85c \ucd94\uce21\ub41c\ub2e4. \uc789\uae00\ub79c\ub4dc \ucd95\uad6c\ud611\ud68c\ub294 \uc785\uc7a5\uad8c\uc744 \uc608\ub9e4\ud588\uc73c\ub098 \uc9c0\uc815\ub41c \uc88c\uc11d\uc5d0 \uc549\uc744 \uc218 \uc5c6\uc5c8\ub358 \uad00\uc911\ub4e4\uc5d0\uac8c \ud45c \uac12\uc758 10%\ub97c \ud658\ubd88\ud588\ub2e4. \uacbd\uae30\uc7a5 \uc8fc\ubcc0 \ub3c4\ub85c\ub4e4\uc774 \ubaa8\ub450 \ucc28\ub2e8\ub418\ub294 \ubc14\ub78c\uc5d0 \ubcfc\ud134\uc758 \uc120\uc218\ub4e4\uc740 \uacbd\uae30\uc7a5\uc5d0\uc11c 1\ub9c8\uc77c \ub5a8\uc5b4\uc9c4 \uacf3\uc5d0\uc11c \ubd80\ud130 \ucf54\uce58\uc640 \ub530\ub85c \ub5a8\uc5b4\uc838 \uc778\ud30c \uc18d\uc744 \ub6ab\uace0 \uacbd\uae30\uc7a5\uc5d0 \uc9c4\uc785\ud588\ub2e4. \u300a\ud0c0\uc784\u300b\uc740 \ud55c \ub54c \uacbd\uae30\uac00 \uc5f4\ub9ac\ub294 \uac83 \uc790\uccb4\uac00 \ubd88\uac00\ub2a5\ud574\ubcf4\uc600\uc73c\ub098 \uc601\uad6d\uc758 \uc655 \uc870\uc9c0 5\uc138 \uac00 \ub3c4\ucc29\ud558\uc790 \uad00\uc911\ub4e4 \uc0ac\uc774\uc758 \ubd84\uc704\uae30\uac00 \ubc14\ub00c\uc5c8\ub2e4\uace0 \uae30\ub85d\ud588\ub2e4. \uad00\uc911\ub4e4\uc740 \uc5f4\ub82c\ud788 \uc601\uad6d\uc758 \uad6d\uac00\uc778 \u3008\ud558\ub290\ub2d8, \uad6d\uc655 \ud3d0\ud558\ub97c \uc9c0\ucf1c\uc8fc\uc18c\uc11c\u3009\ub97c \ub530\ub77c \ubd80\ub978 \ub4a4\uc5d0 \uc120\uc218\ub4e4\uc774 \uacbd\uae30\ud558\ub294 \uacf5\uac04\uc744 \uc815\ub9ac\ud558\ub294 \uac83\uc744 \ub3d5\uae30 \uc2dc\uc791\ud588\ub2e4.", "sentence_answer": "\u300a\ud0c0\uc784\u300b\uc740 \ud55c \ub54c \uacbd\uae30\uac00 \uc5f4\ub9ac\ub294 \uac83 \uc790\uccb4\uac00 \ubd88\uac00\ub2a5\ud574\ubcf4\uc600\uc73c\ub098 \uc601\uad6d\uc758 \uc655 \uc870\uc9c0 5\uc138 \uac00 \ub3c4\ucc29\ud558\uc790 \uad00\uc911\ub4e4 \uc0ac\uc774\uc758 \ubd84\uc704\uae30\uac00 \ubc14\ub00c\uc5c8\ub2e4\uace0 \uae30\ub85d\ud588\ub2e4.", "paragraph_id": "6489739-4-2", "paragraph_question": "question: \ud1b5\uc81c\ubd88\uac00, \uc5c9\ub9dd\uc9c4\ucc3d\uc778 \uad00\uc911\ub4e4\uc758 \ud0dc\ub3c4\uac00 \uae09\ubcc0\ud55c \uac83\uc740 \ub204\uac00 \ub3c4\ucc29\ud588\uc744 \ub54c\uc778\uac00?, context: \ucd9c\uc785\ubb38\uc740 \uc54c\ub824\uc9c4 \ub300\ub85c \uacbd\uae30 \uc2dc\uc791 \uc2dc\uac04 3\uc2dc\uac04 30\ubd84 \uc804\uc778 \uc624\uc80411\uc2dc 30\ubd84\uc5d0 \uc5f4\ub838\uace0 \uc624\ud6c41\uc2dc\uae4c\uc9c0 \ubab0\ub824\ub4e0 \uad00\uc911\ub4e4\uc740 \uc9c8\uc11c\uc788\uac8c \uacbd\uae30\uc7a5\uc5d0 \uc785\uc7a5\ud588\ub2e4. \uadf8\ub7ec\ub098 1\uc2dc\uae4c\uc9c0 \ubab0\ub824\ub4e0 \uad00\uc911\ub4e4\uc774 \uac10\ub2f9\ud558\uc9c0 \ubabb\ud560 \uc815\ub3c4\ub85c \ubd88\uc5b4\ub098\uc790 \uacbd\uae30\uc7a5 \uad00\ub9ac\uce21\uc758 \uc870\uc0ac \ud6c4\uc5d0 \uc624\ud6c4 1\uc2dc 45\ubd84\uc5d0 \ucd9c\uc785\uad6c\ub97c \ub2eb\uace0 \uad00\uc911\uc758 \uc785\uc7a5\uc744 \ub9c9\uc558\ub2e4. \uad00\uc911\uc774\uc5c8\ub358 \uc70c\ub9ac\uc5c4 \ub85c\uc2a4\ub294 \uc62c\ub9bc\ud53d\uae38(Olympic Way)\uc774 \"\uc0ac\ub78c\ub4e4\ub85c \ub4e4\ub053\uc5c8\"\uace0 \"\uacbd\uae30\uc7a5\uc5d0 \uac00\uae4c\uc6cc\uc9c8\uc218\ub85d \uadf8 \uc815\ub3c4\uac00 \uc2ec\ud574\uc9c0\ub2e4 \uacb0\uad6d \uacbd\uae30\uc7a5\uc5d0 \ub3c4\ucc29\ud588\uc744 \ub54c\ub294 \ud68c\uc804\uc2dd \ubb38\uc774 \ub2eb\ud600\uc788\uc5c8\ub2e4\"\ub77c\uace0 \ub098\uc911\uc5d0 \ud68c\uace0\ud558\uae30\ub3c4 \ud588\ub2e4. \uacbd\uae30\uc7a5 \uc0c1\ud0dc\uc5d0 \ub300\ud55c \uc815\ubcf4\uac00 \uc5ec\ub7ec \uae30\ucc28\uc5ed\uc5d0 \uc804\ub2ec\ub418\uc5c8\uc9c0\ub9cc, \uba87 \ucc9c \uba85\uc758 \uc0ac\ub78c\ub4e4\uc774 \uacc4\uc18d \uacbd\uae30\uc7a5\uc5d0 \ub3c4\ucc29\ud574 \ucd9c\uc785\uad6c \uc8fc\ubcc0\uc5d0 \ud070 \ud63c\ub780\uc774 \uc77c\uc5b4\ub0ac\ub2e4. \uacbd\uae30\uc7a5\uc5d0\uc11c\uc758 \uacbd\uae30 \uc900\ube44\ub294 \ubd80\uc2e4\ud588\uc5c8\ub294\ub370 \u300a\ub370\uc77c\ub9ac \uba54\uc77c\u300b\uc758 \uae30\uc790\ub294 \uacbd\uae30\uc7a5\uc758 \uc0c1\ud0dc\uc5d0 \ub300\ud574 \uad00\ub9ac\uc778\ub4e4\uc740 \"\uc4f8\ubaa8\uac00 \uc5c6\uc5c8\"\uace0 \uacbd\uae30\uc7a5 \uce21 \uc0ac\ub78c\ub4e4\uc740 \uacbd\uae30\uc7a5 \ubc16\uacfc \uc548\uc758 \uc0c1\ud669\uc5d0 \ub300\ud574 \"\uc544\ubb34\uac83\ub3c4 \ubaa8\ub974\ub294 \uac83 \ucc98\ub7fc \ubcf4\uc600\ub2e4\"\uace0 \uc790\uc2e0\uc758 \ub9ac\ud3ec\ud2b8\uc5d0 \uc801\uae30\ub3c4 \ud588\ub2e4. \uacbd\uae30\uc7a5\uc744 \ucc3e\uc740 \ud32c\ub4e4\uc740 \uc544\ubb34\ub7f0 \ub9c8\ub828\ub41c \uc7a5\uc18c\ub97c \uc18c\uac1c\ubc1b\uc9c0 \ubabb\ud588\uace0, \uad00\uc911\uc11d\uc758 \uc544\ub798\uce35\uc744 \uc704\uce35\ubcf4\ub2e4 \uba87 \ubc30\ub098 \ube60\ub974\uac8c \ucc44\uc6e0\ub2e4. \uacbd\uae30\uc7a5 \ubc16\uc758 \uc778\ud30c\uac00 \uc810\uc810 \ub298\uc5b4\ub098\uc11c \uc9c0\uc5ed \uacbd\ucc30\uae4c\uc9c0 \ub3d9\uc6d0\ub418\uc5c8\uc9c0\ub9cc \uc774\ubbf8 \uc778\ud30c\uac00 \ub108\ubb34 \ucee4\uc838\ubc84\ub9ac\ub294 \ubc14\ub78c\uc5d0 \uacbd\ucc30\uc740 \ucde8\ud560 \uc218 \uc788\ub294 \ud589\ub3d9\uc774 \uc544\ubb34\uac83\ub3c4 \uc5c6\ub2e4\uace0 \ud310\ub2e8\ud588\ub2e4. \uc624\ud6c42\uc2dc 15\ubd84, \uacbd\uae30\uc7a5 \ubc16\uc758 \uc0ac\ub78c\ub4e4\uc740 \uacbd\uae30\uc7a5\uc758 \ubcbd\uc744 \ud5a5\ud574 \ub3cc\uc9c4\ud574\uc11c \uc5b5\uc9c0\ub85c \uacbd\uae30\uc7a5\uc5d0 \uc9c4\uc785\ud558\ub824\uace0 \ud588\ub2e4. \uc544\ub798\uce35\uc5d0 \uc788\ub358 \uad00\uc911\ub4e4\uc740 \uc774 \uad70\uc911\uc744 \ud53c\ud558\uae30 \uc704\ud574 \uc774 \ud39c\uc2a4\ub97c \ub118\uc5b4\uc11c \uadf8\ub77c\uc6b4\ub4dc\uae4c\uc9c0 \ub4e4\uc5b4\uac00\uac8c \ub418\uc5c8\ub2e4. \ub2f9\uc2dc \uad00\uc911 \ud14c\ub9ac \ud788\ud0a4\ub294 \"\uc904\uc5ec\uc11c \ub9d0\ud558\uc790\uba74, \ubaa8\ub4e0 \uac83\ub4e4\uc774 \uc9dc\uc99d\ub098\ub294 \ub09c\uc7a5\ud310\uc774\uc5c8\ub2e4.\"\ub77c\uba70 \ud6d7\ub0a0 \ub2f9\uc2dc \uc0c1\ud669\uc758 \uadf9\ub2e8\uc131\uc5d0 \ub300\ud574\uc11c \uc124\uba85\ud558\uae30\ub3c4 \ud588\ub2e4. \uacf5\uc2dd\uc801\uc778 \uad00\uc911 \uc218\ub294 126,047\uba85\uc73c\ub85c \uc9d1\uacc4\ub418\uc5c8\uc9c0\ub9cc \uc2e4\uc81c\ub85c \uc785\uc7a5\ud55c \uad00\uc911 \uc218\ub294 15\ub9cc\uba85\uc5d0\uc11c 30\ub9cc\uba85\uae4c\uc9c0 \uc774\ub97c \uac83\uc73c\ub85c \ucd94\uce21\ub41c\ub2e4. \uc789\uae00\ub79c\ub4dc \ucd95\uad6c\ud611\ud68c\ub294 \uc785\uc7a5\uad8c\uc744 \uc608\ub9e4\ud588\uc73c\ub098 \uc9c0\uc815\ub41c \uc88c\uc11d\uc5d0 \uc549\uc744 \uc218 \uc5c6\uc5c8\ub358 \uad00\uc911\ub4e4\uc5d0\uac8c \ud45c \uac12\uc758 10%\ub97c \ud658\ubd88\ud588\ub2e4. \uacbd\uae30\uc7a5 \uc8fc\ubcc0 \ub3c4\ub85c\ub4e4\uc774 \ubaa8\ub450 \ucc28\ub2e8\ub418\ub294 \ubc14\ub78c\uc5d0 \ubcfc\ud134\uc758 \uc120\uc218\ub4e4\uc740 \uacbd\uae30\uc7a5\uc5d0\uc11c 1\ub9c8\uc77c \ub5a8\uc5b4\uc9c4 \uacf3\uc5d0\uc11c \ubd80\ud130 \ucf54\uce58\uc640 \ub530\ub85c \ub5a8\uc5b4\uc838 \uc778\ud30c \uc18d\uc744 \ub6ab\uace0 \uacbd\uae30\uc7a5\uc5d0 \uc9c4\uc785\ud588\ub2e4. \u300a\ud0c0\uc784\u300b\uc740 \ud55c \ub54c \uacbd\uae30\uac00 \uc5f4\ub9ac\ub294 \uac83 \uc790\uccb4\uac00 \ubd88\uac00\ub2a5\ud574\ubcf4\uc600\uc73c\ub098 \uc601\uad6d\uc758 \uc655 \uc870\uc9c0 5\uc138\uac00 \ub3c4\ucc29\ud558\uc790 \uad00\uc911\ub4e4 \uc0ac\uc774\uc758 \ubd84\uc704\uae30\uac00 \ubc14\ub00c\uc5c8\ub2e4\uace0 \uae30\ub85d\ud588\ub2e4. \uad00\uc911\ub4e4\uc740 \uc5f4\ub82c\ud788 \uc601\uad6d\uc758 \uad6d\uac00\uc778 \u3008\ud558\ub290\ub2d8, \uad6d\uc655 \ud3d0\ud558\ub97c \uc9c0\ucf1c\uc8fc\uc18c\uc11c\u3009\ub97c \ub530\ub77c \ubd80\ub978 \ub4a4\uc5d0 \uc120\uc218\ub4e4\uc774 \uacbd\uae30\ud558\ub294 \uacf5\uac04\uc744 \uc815\ub9ac\ud558\ub294 \uac83\uc744 \ub3d5\uae30 \uc2dc\uc791\ud588\ub2e4."} {"question": "\uc2a4\ud0e0\ub9ac \uc564\uc11c\ub2c8 \ucf54\ubca8\ub808\uc2a4\ud0a4\uac00 \uc57c\uad6c \uba85\uc608\uc758 \uc804\ub2f9\uc5d0 \uc785\uc131\ud55c \ub144\ub3c4\ub294?", "paragraph": "\uc2a4\ud0e0\ub9ac \uc564\uc11c\ub2c8 \ucf54\ubca8\ub808\uc2a4\ud0a4(\uc601\uc5b4: Stanley Anthony Coveleski, \uc6d0\ub798 \uc774\ub984\uc740 \uc2a4\ud130\ub2c8\uc2ac\ub85c\uc2a4 \ucf54\uc648\ub808\ube0c\uc2a4\ud0a4(\uc601\uc5b4: Stanislaus Kowalewski), 1889\ub144 7\uc6d4 13\uc77c ~ 1984\ub144 3\uc6d4 20\uc77c)\ub294 \ubbf8\uad6d\uc758 \uc804 \ud504\ub85c \uc57c\uad6c \uc120\uc218\ub85c, 1910\ub144\ub300\uc640 20\ub144\ub300\uc5d0 \uba54\uc774\uc800 \ub9ac\uadf8 \ubca0\uc774\uc2a4\ubcfc(MLB)\uc5d0\uc11c \uc2a4\ud54f\ubcfc \ud22c\uc218\ub85c \ud65c\uc57d\ud558\uc600\ub2e4. 14\uc2dc\uc98c \ub3d9\uc548 \uc544\uba54\ub9ac\uce78 \ub9ac\uadf8(AL)\uc758 \ud544\ub77c\ub378\ud53c\uc544 \uc560\uc2ac\ub808\ud2f1\uc2a4, \ud074\ub9ac\ube14\ub79c\ub4dc \uc778\ub514\uc5b8\uc2a4, \uc6cc\uc2f1\ud134 \uc138\ub108\ud130\uc2a4, \ub274\uc695 \uc591\ud0a4\uc2a4\uc5d0\uc11c \ub6f0\uc5c8\ub2e4. \ud1b5\uc0b0 450\uacbd\uae30\uc5d0 \ucd9c\uc804\ud574 3,082\uc774\ub2dd\uc744 \ub358\uc9c0\uba70 215\uc2b9 142\ud328, 224\uc644\ud22c, 38\uc644\ubd09, 2.89\uc758 \ud3c9\uade0\uc790\ucc45\uc810\uc744 \uae30\ub85d\ud588\ub2e4. 1969\ub144 \uc57c\uad6c \uba85\uc608\uc758 \uc804\ub2f9\uc5d0 \uc785\uc131\ud588\ub2e4.", "answer": "1969\ub144", "sentence": "1969\ub144 \uc57c\uad6c \uba85\uc608\uc758 \uc804\ub2f9\uc5d0 \uc785\uc131\ud588\ub2e4.", "paragraph_sentence": "\uc2a4\ud0e0\ub9ac \uc564\uc11c\ub2c8 \ucf54\ubca8\ub808\uc2a4\ud0a4(\uc601\uc5b4: Stanley Anthony Coveleski, \uc6d0\ub798 \uc774\ub984\uc740 \uc2a4\ud130\ub2c8\uc2ac\ub85c\uc2a4 \ucf54\uc648\ub808\ube0c\uc2a4\ud0a4(\uc601\uc5b4: Stanislaus Kowalewski), 1889\ub144 7\uc6d4 13\uc77c ~ 1984\ub144 3\uc6d4 20\uc77c)\ub294 \ubbf8\uad6d\uc758 \uc804 \ud504\ub85c \uc57c\uad6c \uc120\uc218\ub85c, 1910\ub144\ub300\uc640 20\ub144\ub300\uc5d0 \uba54\uc774\uc800 \ub9ac\uadf8 \ubca0\uc774\uc2a4\ubcfc(MLB)\uc5d0\uc11c \uc2a4\ud54f\ubcfc \ud22c\uc218\ub85c \ud65c\uc57d\ud558\uc600\ub2e4. 14\uc2dc\uc98c \ub3d9\uc548 \uc544\uba54\ub9ac\uce78 \ub9ac\uadf8(AL)\uc758 \ud544\ub77c\ub378\ud53c\uc544 \uc560\uc2ac\ub808\ud2f1\uc2a4, \ud074\ub9ac\ube14\ub79c\ub4dc \uc778\ub514\uc5b8\uc2a4, \uc6cc\uc2f1\ud134 \uc138\ub108\ud130\uc2a4, \ub274\uc695 \uc591\ud0a4\uc2a4\uc5d0\uc11c \ub6f0\uc5c8\ub2e4. \ud1b5\uc0b0 450\uacbd\uae30\uc5d0 \ucd9c\uc804\ud574 3,082\uc774\ub2dd\uc744 \ub358\uc9c0\uba70 215\uc2b9 142\ud328, 224\uc644\ud22c, 38\uc644\ubd09, 2.89\uc758 \ud3c9\uade0\uc790\ucc45\uc810\uc744 \uae30\ub85d\ud588\ub2e4. 1969\ub144 \uc57c\uad6c \uba85\uc608\uc758 \uc804\ub2f9\uc5d0 \uc785\uc131\ud588\ub2e4. ", "paragraph_answer": "\uc2a4\ud0e0\ub9ac \uc564\uc11c\ub2c8 \ucf54\ubca8\ub808\uc2a4\ud0a4(\uc601\uc5b4: Stanley Anthony Coveleski, \uc6d0\ub798 \uc774\ub984\uc740 \uc2a4\ud130\ub2c8\uc2ac\ub85c\uc2a4 \ucf54\uc648\ub808\ube0c\uc2a4\ud0a4(\uc601\uc5b4: Stanislaus Kowalewski), 1889\ub144 7\uc6d4 13\uc77c ~ 1984\ub144 3\uc6d4 20\uc77c)\ub294 \ubbf8\uad6d\uc758 \uc804 \ud504\ub85c \uc57c\uad6c \uc120\uc218\ub85c, 1910\ub144\ub300\uc640 20\ub144\ub300\uc5d0 \uba54\uc774\uc800 \ub9ac\uadf8 \ubca0\uc774\uc2a4\ubcfc(MLB)\uc5d0\uc11c \uc2a4\ud54f\ubcfc \ud22c\uc218\ub85c \ud65c\uc57d\ud558\uc600\ub2e4. 14\uc2dc\uc98c \ub3d9\uc548 \uc544\uba54\ub9ac\uce78 \ub9ac\uadf8(AL)\uc758 \ud544\ub77c\ub378\ud53c\uc544 \uc560\uc2ac\ub808\ud2f1\uc2a4, \ud074\ub9ac\ube14\ub79c\ub4dc \uc778\ub514\uc5b8\uc2a4, \uc6cc\uc2f1\ud134 \uc138\ub108\ud130\uc2a4, \ub274\uc695 \uc591\ud0a4\uc2a4\uc5d0\uc11c \ub6f0\uc5c8\ub2e4. \ud1b5\uc0b0 450\uacbd\uae30\uc5d0 \ucd9c\uc804\ud574 3,082\uc774\ub2dd\uc744 \ub358\uc9c0\uba70 215\uc2b9 142\ud328, 224\uc644\ud22c, 38\uc644\ubd09, 2.89\uc758 \ud3c9\uade0\uc790\ucc45\uc810\uc744 \uae30\ub85d\ud588\ub2e4. 1969\ub144 \uc57c\uad6c \uba85\uc608\uc758 \uc804\ub2f9\uc5d0 \uc785\uc131\ud588\ub2e4.", "sentence_answer": " 1969\ub144 \uc57c\uad6c \uba85\uc608\uc758 \uc804\ub2f9\uc5d0 \uc785\uc131\ud588\ub2e4.", "paragraph_id": "6457302-0-0", "paragraph_question": "question: \uc2a4\ud0e0\ub9ac \uc564\uc11c\ub2c8 \ucf54\ubca8\ub808\uc2a4\ud0a4\uac00 \uc57c\uad6c \uba85\uc608\uc758 \uc804\ub2f9\uc5d0 \uc785\uc131\ud55c \ub144\ub3c4\ub294?, context: \uc2a4\ud0e0\ub9ac \uc564\uc11c\ub2c8 \ucf54\ubca8\ub808\uc2a4\ud0a4(\uc601\uc5b4: Stanley Anthony Coveleski, \uc6d0\ub798 \uc774\ub984\uc740 \uc2a4\ud130\ub2c8\uc2ac\ub85c\uc2a4 \ucf54\uc648\ub808\ube0c\uc2a4\ud0a4(\uc601\uc5b4: Stanislaus Kowalewski), 1889\ub144 7\uc6d4 13\uc77c ~ 1984\ub144 3\uc6d4 20\uc77c)\ub294 \ubbf8\uad6d\uc758 \uc804 \ud504\ub85c \uc57c\uad6c \uc120\uc218\ub85c, 1910\ub144\ub300\uc640 20\ub144\ub300\uc5d0 \uba54\uc774\uc800 \ub9ac\uadf8 \ubca0\uc774\uc2a4\ubcfc(MLB)\uc5d0\uc11c \uc2a4\ud54f\ubcfc \ud22c\uc218\ub85c \ud65c\uc57d\ud558\uc600\ub2e4. 14\uc2dc\uc98c \ub3d9\uc548 \uc544\uba54\ub9ac\uce78 \ub9ac\uadf8(AL)\uc758 \ud544\ub77c\ub378\ud53c\uc544 \uc560\uc2ac\ub808\ud2f1\uc2a4, \ud074\ub9ac\ube14\ub79c\ub4dc \uc778\ub514\uc5b8\uc2a4, \uc6cc\uc2f1\ud134 \uc138\ub108\ud130\uc2a4, \ub274\uc695 \uc591\ud0a4\uc2a4\uc5d0\uc11c \ub6f0\uc5c8\ub2e4. \ud1b5\uc0b0 450\uacbd\uae30\uc5d0 \ucd9c\uc804\ud574 3,082\uc774\ub2dd\uc744 \ub358\uc9c0\uba70 215\uc2b9 142\ud328, 224\uc644\ud22c, 38\uc644\ubd09, 2.89\uc758 \ud3c9\uade0\uc790\ucc45\uc810\uc744 \uae30\ub85d\ud588\ub2e4. 1969\ub144 \uc57c\uad6c \uba85\uc608\uc758 \uc804\ub2f9\uc5d0 \uc785\uc131\ud588\ub2e4."} {"question": "\uacc4\ub7c9\ud568\uc218\ub294 \uae30\ud558\uc5d0\uc11c \uc54c\uace0\uc790 \ud558\ub294 \ubb34\uc5c7\uc744 \uacc4\uc0b0\ud560 \uc218 \uc788\ub294\uac00?", "paragraph": "\uc9c0\uad6c\uc640 \uac19\uc740 \uad6c\ud615 \ud45c\uba74\uc740 \ub2e8\uc21c\ud55c \uc0ac\ub840\ub97c \uc81c\uacf5\ud55c\ub2e4. \ud45c\uba74\uc0c1\uc758 \ubaa8\ub4e0 \uc810\uc758 \uc704\uce58\ub294 \uc704\ub3c4\uc640 \uacbd\ub3c4\ub77c\ub294 \ub450 \uac1c\uc758 \uc88c\ud45c\ub85c \uae30\uc220\ub420 \uc218 \uc788\ub2e4. \ud3c9\uba74\uc0c1\uc758 \uc9c1\uad50\uc88c\ud45c\uacc4\uc640 \ub2ec\ub9ac, \uc88c\ud45c\uc758 \ucc28\uc774\ub294 \ud45c\uba74\uc0c1 \uac70\ub9ac\uc758 \ucc28\uc774\uc640 \uac19\uc740 \uac83\uc774 \uc544\ub2c8\uba70, \uc774\ub294 \uc624\ub978\ucabd \uadf8\ub9bc\uc744 \ubcf4\uba74 \uc27d\uac8c \uc774\ud574\ub41c\ub2e4. \uc801\ub3c4\uc5d0 \uc788\ub294 \uc0ac\ub78c\uc774 \uc11c\ucabd\uc744 \ud5a5\ud574 \uacbd\ub3c4 30 \ub3c4\ub97c \uc774\ub3d9\ud588\uc744 \ub54c \uc774\ub3d9 \uac70\ub9ac(\uc790\uc8fc\uc0c9 \uc120)\ub294 \uc57d 3300 \ud0ac\ub85c\ubbf8\ud130\uc774\ub2e4. \ubc18\uba74 \ubd81\uc704 55\ub3c4\uc5d0 \uc788\ub294 \uc0ac\ub78c\uc774 \uac19\uc740 \ubc29\ud5a5\uc73c\ub85c \uac19\uc740 \uacbd\ub3c4\ub9cc\ud07c \uc774\ub3d9\ud588\uc744 \ub54c \uc774\ub3d9 \uac70\ub9ac(\ud30c\ub780\uc0c9 \uc120)\ub294 \uc57d 1900 \ud0ac\ub85c\ubbf8\ud130\uc774\ub2e4. \uace0\ub85c \uad6c\ud615 \ud45c\uba74\uc0c1\uc758 \uae30\ud558\ub97c \uc124\uba85\ud558\uae30\uc5d0 \uc774 \ub450 \uc88c\ud45c\ub294 \ucda9\ubd84\ud55c \uc815\ubcf4\ub97c \uc81c\uacf5\ud574\uc8fc\uc9c0 \ubabb\ud558\uba70, \uad6c\ud615 \ud45c\uba74\ubcf4\ub2e4 \ub354 \ubcf5\uc7a1\ud55c \uacf5\uac04 \ub610\ub294 \uc2dc\uacf5\uac04\uc5d0 \ub300\ud574\uc11c\ub294 \ub9d0\ud560 \uac83\ub3c4 \uc5c6\ub2e4. \uc774\ub54c \ubaa8\uc790\ub780 \uc815\ubcf4\uac00 \ubc14\ub85c \uacc4\ub7c9 \uc18d\uc5d0 \uc815\ubc00\ud558\uac8c \ud568\ucd95\ub418\uc5b4 \uc788\ub2e4. \uacc4\ub7c9\uc740 \ud45c\uba74(\ub610\ub294 \uacf5\uac04, \ub610\ub294 \uc2dc\uacf5\uac04)\uc0c1\uc758 \uac01 \uc810\uc5d0\uc11c \uc815\uc758\ub418\ub294 \ud568\uc218\ub85c\uc11c \uc88c\ud45c\uc758 \ucc28\uc774\ub97c \uac70\ub9ac\uc758 \ucc28\uc774\uc640 \uad00\uacc4\uc9d3\ub294\ub2e4. \uae30\ud558\uc5d0\uc11c \uc54c\uace0\uc790 \ud558\ub294 \ub2e4\ub978 \ubaa8\ub4e0 \ubb3c\ub9ac\ub7c9, \uc608\ucee8\ub300 \uc784\uc758\uc758 \uace1\uc120\uc758 \uae38\uc774 \ub610\ub294 \ub450 \uace1\uc120\uc774 \ub9cc\ub098\ub294 \uac01\ub3c4 \ub4f1\uc740 \uc774 \uacc4\ub7c9 \ud568\uc218\ub85c \uacc4\uc0b0\ub420 \uc218 \uc788\ub2e4.", "answer": "\ubb3c\ub9ac\ub7c9", "sentence": "\uae30\ud558\uc5d0\uc11c \uc54c\uace0\uc790 \ud558\ub294 \ub2e4\ub978 \ubaa8\ub4e0 \ubb3c\ub9ac\ub7c9 ,", "paragraph_sentence": "\uc9c0\uad6c\uc640 \uac19\uc740 \uad6c\ud615 \ud45c\uba74\uc740 \ub2e8\uc21c\ud55c \uc0ac\ub840\ub97c \uc81c\uacf5\ud55c\ub2e4. \ud45c\uba74\uc0c1\uc758 \ubaa8\ub4e0 \uc810\uc758 \uc704\uce58\ub294 \uc704\ub3c4\uc640 \uacbd\ub3c4\ub77c\ub294 \ub450 \uac1c\uc758 \uc88c\ud45c\ub85c \uae30\uc220\ub420 \uc218 \uc788\ub2e4. \ud3c9\uba74\uc0c1\uc758 \uc9c1\uad50\uc88c\ud45c\uacc4\uc640 \ub2ec\ub9ac, \uc88c\ud45c\uc758 \ucc28\uc774\ub294 \ud45c\uba74\uc0c1 \uac70\ub9ac\uc758 \ucc28\uc774\uc640 \uac19\uc740 \uac83\uc774 \uc544\ub2c8\uba70, \uc774\ub294 \uc624\ub978\ucabd \uadf8\ub9bc\uc744 \ubcf4\uba74 \uc27d\uac8c \uc774\ud574\ub41c\ub2e4. \uc801\ub3c4\uc5d0 \uc788\ub294 \uc0ac\ub78c\uc774 \uc11c\ucabd\uc744 \ud5a5\ud574 \uacbd\ub3c4 30 \ub3c4\ub97c \uc774\ub3d9\ud588\uc744 \ub54c \uc774\ub3d9 \uac70\ub9ac(\uc790\uc8fc\uc0c9 \uc120)\ub294 \uc57d 3300 \ud0ac\ub85c\ubbf8\ud130\uc774\ub2e4. \ubc18\uba74 \ubd81\uc704 55\ub3c4\uc5d0 \uc788\ub294 \uc0ac\ub78c\uc774 \uac19\uc740 \ubc29\ud5a5\uc73c\ub85c \uac19\uc740 \uacbd\ub3c4\ub9cc\ud07c \uc774\ub3d9\ud588\uc744 \ub54c \uc774\ub3d9 \uac70\ub9ac(\ud30c\ub780\uc0c9 \uc120)\ub294 \uc57d 1900 \ud0ac\ub85c\ubbf8\ud130\uc774\ub2e4. \uace0\ub85c \uad6c\ud615 \ud45c\uba74\uc0c1\uc758 \uae30\ud558\ub97c \uc124\uba85\ud558\uae30\uc5d0 \uc774 \ub450 \uc88c\ud45c\ub294 \ucda9\ubd84\ud55c \uc815\ubcf4\ub97c \uc81c\uacf5\ud574\uc8fc\uc9c0 \ubabb\ud558\uba70, \uad6c\ud615 \ud45c\uba74\ubcf4\ub2e4 \ub354 \ubcf5\uc7a1\ud55c \uacf5\uac04 \ub610\ub294 \uc2dc\uacf5\uac04\uc5d0 \ub300\ud574\uc11c\ub294 \ub9d0\ud560 \uac83\ub3c4 \uc5c6\ub2e4. \uc774\ub54c \ubaa8\uc790\ub780 \uc815\ubcf4\uac00 \ubc14\ub85c \uacc4\ub7c9 \uc18d\uc5d0 \uc815\ubc00\ud558\uac8c \ud568\ucd95\ub418\uc5b4 \uc788\ub2e4. \uacc4\ub7c9\uc740 \ud45c\uba74(\ub610\ub294 \uacf5\uac04, \ub610\ub294 \uc2dc\uacf5\uac04)\uc0c1\uc758 \uac01 \uc810\uc5d0\uc11c \uc815\uc758\ub418\ub294 \ud568\uc218\ub85c\uc11c \uc88c\ud45c\uc758 \ucc28\uc774\ub97c \uac70\ub9ac\uc758 \ucc28\uc774\uc640 \uad00\uacc4\uc9d3\ub294\ub2e4. \uae30\ud558\uc5d0\uc11c \uc54c\uace0\uc790 \ud558\ub294 \ub2e4\ub978 \ubaa8\ub4e0 \ubb3c\ub9ac\ub7c9 , \uc608\ucee8\ub300 \uc784\uc758\uc758 \uace1\uc120\uc758 \uae38\uc774 \ub610\ub294 \ub450 \uace1\uc120\uc774 \ub9cc\ub098\ub294 \uac01\ub3c4 \ub4f1\uc740 \uc774 \uacc4\ub7c9 \ud568\uc218\ub85c \uacc4\uc0b0\ub420 \uc218 \uc788\ub2e4.", "paragraph_answer": "\uc9c0\uad6c\uc640 \uac19\uc740 \uad6c\ud615 \ud45c\uba74\uc740 \ub2e8\uc21c\ud55c \uc0ac\ub840\ub97c \uc81c\uacf5\ud55c\ub2e4. \ud45c\uba74\uc0c1\uc758 \ubaa8\ub4e0 \uc810\uc758 \uc704\uce58\ub294 \uc704\ub3c4\uc640 \uacbd\ub3c4\ub77c\ub294 \ub450 \uac1c\uc758 \uc88c\ud45c\ub85c \uae30\uc220\ub420 \uc218 \uc788\ub2e4. \ud3c9\uba74\uc0c1\uc758 \uc9c1\uad50\uc88c\ud45c\uacc4\uc640 \ub2ec\ub9ac, \uc88c\ud45c\uc758 \ucc28\uc774\ub294 \ud45c\uba74\uc0c1 \uac70\ub9ac\uc758 \ucc28\uc774\uc640 \uac19\uc740 \uac83\uc774 \uc544\ub2c8\uba70, \uc774\ub294 \uc624\ub978\ucabd \uadf8\ub9bc\uc744 \ubcf4\uba74 \uc27d\uac8c \uc774\ud574\ub41c\ub2e4. \uc801\ub3c4\uc5d0 \uc788\ub294 \uc0ac\ub78c\uc774 \uc11c\ucabd\uc744 \ud5a5\ud574 \uacbd\ub3c4 30 \ub3c4\ub97c \uc774\ub3d9\ud588\uc744 \ub54c \uc774\ub3d9 \uac70\ub9ac(\uc790\uc8fc\uc0c9 \uc120)\ub294 \uc57d 3300 \ud0ac\ub85c\ubbf8\ud130\uc774\ub2e4. \ubc18\uba74 \ubd81\uc704 55\ub3c4\uc5d0 \uc788\ub294 \uc0ac\ub78c\uc774 \uac19\uc740 \ubc29\ud5a5\uc73c\ub85c \uac19\uc740 \uacbd\ub3c4\ub9cc\ud07c \uc774\ub3d9\ud588\uc744 \ub54c \uc774\ub3d9 \uac70\ub9ac(\ud30c\ub780\uc0c9 \uc120)\ub294 \uc57d 1900 \ud0ac\ub85c\ubbf8\ud130\uc774\ub2e4. \uace0\ub85c \uad6c\ud615 \ud45c\uba74\uc0c1\uc758 \uae30\ud558\ub97c \uc124\uba85\ud558\uae30\uc5d0 \uc774 \ub450 \uc88c\ud45c\ub294 \ucda9\ubd84\ud55c \uc815\ubcf4\ub97c \uc81c\uacf5\ud574\uc8fc\uc9c0 \ubabb\ud558\uba70, \uad6c\ud615 \ud45c\uba74\ubcf4\ub2e4 \ub354 \ubcf5\uc7a1\ud55c \uacf5\uac04 \ub610\ub294 \uc2dc\uacf5\uac04\uc5d0 \ub300\ud574\uc11c\ub294 \ub9d0\ud560 \uac83\ub3c4 \uc5c6\ub2e4. \uc774\ub54c \ubaa8\uc790\ub780 \uc815\ubcf4\uac00 \ubc14\ub85c \uacc4\ub7c9 \uc18d\uc5d0 \uc815\ubc00\ud558\uac8c \ud568\ucd95\ub418\uc5b4 \uc788\ub2e4. \uacc4\ub7c9\uc740 \ud45c\uba74(\ub610\ub294 \uacf5\uac04, \ub610\ub294 \uc2dc\uacf5\uac04)\uc0c1\uc758 \uac01 \uc810\uc5d0\uc11c \uc815\uc758\ub418\ub294 \ud568\uc218\ub85c\uc11c \uc88c\ud45c\uc758 \ucc28\uc774\ub97c \uac70\ub9ac\uc758 \ucc28\uc774\uc640 \uad00\uacc4\uc9d3\ub294\ub2e4. \uae30\ud558\uc5d0\uc11c \uc54c\uace0\uc790 \ud558\ub294 \ub2e4\ub978 \ubaa8\ub4e0 \ubb3c\ub9ac\ub7c9 , \uc608\ucee8\ub300 \uc784\uc758\uc758 \uace1\uc120\uc758 \uae38\uc774 \ub610\ub294 \ub450 \uace1\uc120\uc774 \ub9cc\ub098\ub294 \uac01\ub3c4 \ub4f1\uc740 \uc774 \uacc4\ub7c9 \ud568\uc218\ub85c \uacc4\uc0b0\ub420 \uc218 \uc788\ub2e4.", "sentence_answer": "\uae30\ud558\uc5d0\uc11c \uc54c\uace0\uc790 \ud558\ub294 \ub2e4\ub978 \ubaa8\ub4e0 \ubb3c\ub9ac\ub7c9 ,", "paragraph_id": "6171950-8-2", "paragraph_question": "question: \uacc4\ub7c9\ud568\uc218\ub294 \uae30\ud558\uc5d0\uc11c \uc54c\uace0\uc790 \ud558\ub294 \ubb34\uc5c7\uc744 \uacc4\uc0b0\ud560 \uc218 \uc788\ub294\uac00?, context: \uc9c0\uad6c\uc640 \uac19\uc740 \uad6c\ud615 \ud45c\uba74\uc740 \ub2e8\uc21c\ud55c \uc0ac\ub840\ub97c \uc81c\uacf5\ud55c\ub2e4. \ud45c\uba74\uc0c1\uc758 \ubaa8\ub4e0 \uc810\uc758 \uc704\uce58\ub294 \uc704\ub3c4\uc640 \uacbd\ub3c4\ub77c\ub294 \ub450 \uac1c\uc758 \uc88c\ud45c\ub85c \uae30\uc220\ub420 \uc218 \uc788\ub2e4. \ud3c9\uba74\uc0c1\uc758 \uc9c1\uad50\uc88c\ud45c\uacc4\uc640 \ub2ec\ub9ac, \uc88c\ud45c\uc758 \ucc28\uc774\ub294 \ud45c\uba74\uc0c1 \uac70\ub9ac\uc758 \ucc28\uc774\uc640 \uac19\uc740 \uac83\uc774 \uc544\ub2c8\uba70, \uc774\ub294 \uc624\ub978\ucabd \uadf8\ub9bc\uc744 \ubcf4\uba74 \uc27d\uac8c \uc774\ud574\ub41c\ub2e4. \uc801\ub3c4\uc5d0 \uc788\ub294 \uc0ac\ub78c\uc774 \uc11c\ucabd\uc744 \ud5a5\ud574 \uacbd\ub3c4 30 \ub3c4\ub97c \uc774\ub3d9\ud588\uc744 \ub54c \uc774\ub3d9 \uac70\ub9ac(\uc790\uc8fc\uc0c9 \uc120)\ub294 \uc57d 3300 \ud0ac\ub85c\ubbf8\ud130\uc774\ub2e4. \ubc18\uba74 \ubd81\uc704 55\ub3c4\uc5d0 \uc788\ub294 \uc0ac\ub78c\uc774 \uac19\uc740 \ubc29\ud5a5\uc73c\ub85c \uac19\uc740 \uacbd\ub3c4\ub9cc\ud07c \uc774\ub3d9\ud588\uc744 \ub54c \uc774\ub3d9 \uac70\ub9ac(\ud30c\ub780\uc0c9 \uc120)\ub294 \uc57d 1900 \ud0ac\ub85c\ubbf8\ud130\uc774\ub2e4. \uace0\ub85c \uad6c\ud615 \ud45c\uba74\uc0c1\uc758 \uae30\ud558\ub97c \uc124\uba85\ud558\uae30\uc5d0 \uc774 \ub450 \uc88c\ud45c\ub294 \ucda9\ubd84\ud55c \uc815\ubcf4\ub97c \uc81c\uacf5\ud574\uc8fc\uc9c0 \ubabb\ud558\uba70, \uad6c\ud615 \ud45c\uba74\ubcf4\ub2e4 \ub354 \ubcf5\uc7a1\ud55c \uacf5\uac04 \ub610\ub294 \uc2dc\uacf5\uac04\uc5d0 \ub300\ud574\uc11c\ub294 \ub9d0\ud560 \uac83\ub3c4 \uc5c6\ub2e4. \uc774\ub54c \ubaa8\uc790\ub780 \uc815\ubcf4\uac00 \ubc14\ub85c \uacc4\ub7c9 \uc18d\uc5d0 \uc815\ubc00\ud558\uac8c \ud568\ucd95\ub418\uc5b4 \uc788\ub2e4. \uacc4\ub7c9\uc740 \ud45c\uba74(\ub610\ub294 \uacf5\uac04, \ub610\ub294 \uc2dc\uacf5\uac04)\uc0c1\uc758 \uac01 \uc810\uc5d0\uc11c \uc815\uc758\ub418\ub294 \ud568\uc218\ub85c\uc11c \uc88c\ud45c\uc758 \ucc28\uc774\ub97c \uac70\ub9ac\uc758 \ucc28\uc774\uc640 \uad00\uacc4\uc9d3\ub294\ub2e4. \uae30\ud558\uc5d0\uc11c \uc54c\uace0\uc790 \ud558\ub294 \ub2e4\ub978 \ubaa8\ub4e0 \ubb3c\ub9ac\ub7c9, \uc608\ucee8\ub300 \uc784\uc758\uc758 \uace1\uc120\uc758 \uae38\uc774 \ub610\ub294 \ub450 \uace1\uc120\uc774 \ub9cc\ub098\ub294 \uac01\ub3c4 \ub4f1\uc740 \uc774 \uacc4\ub7c9 \ud568\uc218\ub85c \uacc4\uc0b0\ub420 \uc218 \uc788\ub2e4."} {"question": "1622\ub144\uae4c\uc9c0 \ud2b8\ub808\ubcf8 \ub9c8\uc744\uc744 \uc0ac\uc218\ud55c \uc5ec\ub2e8\uc740?", "paragraph": "\uc774\ub7ec\ud55c \uc5f4\uc545\ud55c \uc0c1\ud669\uc5d0\ub3c4 \ubd88\uad6c\ud558\uace0 \ubc18\ub780\uc740 \ubcf4\ud5e4\ubbf8\uc544\uc778\ub4e4\uc5d0\uac8c \uc720\ub9ac\ud558\uac8c \ub3cc\uc544\uac14\ub2e4. \uadc0\uc871\ub4e4 \ub300\ubd80\ubd84\uc774 \ub8e8\ud130\ud30c\uc640 \uce7c\ubc45\ud30c\uc600\ub358 \uc624\ubc84\uc678\uc2a4\ud130\ub77c\uc774\ud788\uac00 \ubc18\ub780\uc5d0 \ub3d9\ucc38\ud588\uace0, \uace7 \ub2c8\ub354\uc678\uc2a4\ud130\ub77c\uc774\ud788\uac00 \ubc18\ub780\uc744 \uc77c\uc73c\ucf30\ub2e4. 1619\ub144\uc5d0\ub294 \uc9c4\ud2b8\ub9ac\ud788 \ub9c8\ud0c0\uc57c\uc2a4 \ud2b8\ub7f0\uc774 \ube44\uc5d4\ub098 \uc131\ubcbd\uc744 \ud5a5\ud574 \uacf5\uaca9\uc744 \uac00\ud588\ub2e4. \ub354\uc6b1\uc774 \uc601\uad6d \uc81c\ub3c4\uc5d0\uc11c\ub294 \ud504\ub9ac\ub4dc\ub9ac\ud788 5\uc138\uc758 \ubc18\ub780\uc774 \uc720\ub7fd\uc758 \uc870\uc5d8\ub9ac\ub77c \ubb18\uc0ac\ub418\ub294 \uc5d8\ub9ac\uc790\ubca0\uc2a4 \uc2a4\ud29c\uc5b4\ud2b8\uc758 \ubb38\uc81c\ub85c \ube44\ucdb0\uc9c0\uac8c \ub418\uc5c8\ub2e4. \uc774\ub294 30\ub144 \uc804\uc7c1 \uc804\ubc18\uc5d0 \uac78\uccd0 \uc218\ucc9c \uba85\uc758 \uc9c0\uc9c0\uc790\ub4e4\uc774 \uc5d8\ub9ac\uc790\ubca0\uc2a4 \uc2a4\ud29c\uc5b4\ud2b8\uac00 \uc804\uc7c1\uc758 \uc6d0\uc778\uc774\ub77c\ub294 \ud750\ub984\uc744 \ub0b4\ub193\uae30\ub3c4 \ud588\ub2e4. \ud638\ub808\uc774\uc2a4 \ubca0\ub808\uac00 \uc774\ub044\ub294 \uc601\uad6d-\ub124\ub35c\ub780\ub4dc \uc5f0\ud569\uad70\uc774 \uc804\uc7c1 \ucd08\uae30\uc5d0 \ud314\uce20 \uc120\uc81c\ud6c4 \uc9c0\uc5ed\uc73c\ub85c \uc774\ub3d9\ud588\uace0 \uc874 \uc138\ud1a4\uc774 \uc774\ub044\ub294 \uc2a4\ucf54\ud2c0\ub79c\ub4dc-\ub124\ub35c\ub780\ub4dc \uc5f0\ud569\uad70\uc774 \ubcf4\ud5e4\ubbf8\uc544\ub85c \uc774\ub3d9\ud588\ub2e4. \uadf8\ub9ac\uace0 \ud6c4\uc5d0 \uc774\ub4e4\uc740 \ud63c\uc131 \ubd80\ub300\uc778 \ube0c\ub974\ud0c0\ub274 \uc5ec\ub2e8\uc5d0 \ucc38\uc5ec\ud558\uac8c \ub41c\ub2e4. \uc138\ud1a4\uc758 \uc5ec\ub2e8\uc740 1622\ub144\uae4c\uc9c0 \ud2b8\ub808\ubcf8 \ub9c8\uc744\uc744 \uc0ac\uc218\ud55c \uc5ec\ub2e8\uc73c\ub85c, \ubcf4\ud5e4\ubbf8\uc544\uc5d0\uc11c\ub294 \ucd5c\ud6c4\uc758 \uac1c\uc2e0\uad50 \ub3d9\ub9f9\uc774\uc5c8\ub2e4.", "answer": "\uc138\ud1a4\uc758 \uc5ec\ub2e8", "sentence": "\uc138\ud1a4\uc758 \uc5ec\ub2e8 \uc740 1622\ub144\uae4c\uc9c0 \ud2b8\ub808\ubcf8 \ub9c8\uc744\uc744 \uc0ac\uc218\ud55c \uc5ec\ub2e8\uc73c\ub85c, \ubcf4\ud5e4\ubbf8\uc544\uc5d0\uc11c\ub294 \ucd5c\ud6c4\uc758 \uac1c\uc2e0\uad50 \ub3d9\ub9f9\uc774\uc5c8\ub2e4.", "paragraph_sentence": "\uc774\ub7ec\ud55c \uc5f4\uc545\ud55c \uc0c1\ud669\uc5d0\ub3c4 \ubd88\uad6c\ud558\uace0 \ubc18\ub780\uc740 \ubcf4\ud5e4\ubbf8\uc544\uc778\ub4e4\uc5d0\uac8c \uc720\ub9ac\ud558\uac8c \ub3cc\uc544\uac14\ub2e4. \uadc0\uc871\ub4e4 \ub300\ubd80\ubd84\uc774 \ub8e8\ud130\ud30c\uc640 \uce7c\ubc45\ud30c\uc600\ub358 \uc624\ubc84\uc678\uc2a4\ud130\ub77c\uc774\ud788\uac00 \ubc18\ub780\uc5d0 \ub3d9\ucc38\ud588\uace0, \uace7 \ub2c8\ub354\uc678\uc2a4\ud130\ub77c\uc774\ud788\uac00 \ubc18\ub780\uc744 \uc77c\uc73c\ucf30\ub2e4. 1619\ub144\uc5d0\ub294 \uc9c4\ud2b8\ub9ac\ud788 \ub9c8\ud0c0\uc57c\uc2a4 \ud2b8\ub7f0\uc774 \ube44\uc5d4\ub098 \uc131\ubcbd\uc744 \ud5a5\ud574 \uacf5\uaca9\uc744 \uac00\ud588\ub2e4. \ub354\uc6b1\uc774 \uc601\uad6d \uc81c\ub3c4\uc5d0\uc11c\ub294 \ud504\ub9ac\ub4dc\ub9ac\ud788 5\uc138\uc758 \ubc18\ub780\uc774 \uc720\ub7fd\uc758 \uc870\uc5d8\ub9ac\ub77c \ubb18\uc0ac\ub418\ub294 \uc5d8\ub9ac\uc790\ubca0\uc2a4 \uc2a4\ud29c\uc5b4\ud2b8\uc758 \ubb38\uc81c\ub85c \ube44\ucdb0\uc9c0\uac8c \ub418\uc5c8\ub2e4. \uc774\ub294 30\ub144 \uc804\uc7c1 \uc804\ubc18\uc5d0 \uac78\uccd0 \uc218\ucc9c \uba85\uc758 \uc9c0\uc9c0\uc790\ub4e4\uc774 \uc5d8\ub9ac\uc790\ubca0\uc2a4 \uc2a4\ud29c\uc5b4\ud2b8\uac00 \uc804\uc7c1\uc758 \uc6d0\uc778\uc774\ub77c\ub294 \ud750\ub984\uc744 \ub0b4\ub193\uae30\ub3c4 \ud588\ub2e4. \ud638\ub808\uc774\uc2a4 \ubca0\ub808\uac00 \uc774\ub044\ub294 \uc601\uad6d-\ub124\ub35c\ub780\ub4dc \uc5f0\ud569\uad70\uc774 \uc804\uc7c1 \ucd08\uae30\uc5d0 \ud314\uce20 \uc120\uc81c\ud6c4 \uc9c0\uc5ed\uc73c\ub85c \uc774\ub3d9\ud588\uace0 \uc874 \uc138\ud1a4\uc774 \uc774\ub044\ub294 \uc2a4\ucf54\ud2c0\ub79c\ub4dc-\ub124\ub35c\ub780\ub4dc \uc5f0\ud569\uad70\uc774 \ubcf4\ud5e4\ubbf8\uc544\ub85c \uc774\ub3d9\ud588\ub2e4. \uadf8\ub9ac\uace0 \ud6c4\uc5d0 \uc774\ub4e4\uc740 \ud63c\uc131 \ubd80\ub300\uc778 \ube0c\ub974\ud0c0\ub274 \uc5ec\ub2e8\uc5d0 \ucc38\uc5ec\ud558\uac8c \ub41c\ub2e4. \uc138\ud1a4\uc758 \uc5ec\ub2e8 \uc740 1622\ub144\uae4c\uc9c0 \ud2b8\ub808\ubcf8 \ub9c8\uc744\uc744 \uc0ac\uc218\ud55c \uc5ec\ub2e8\uc73c\ub85c, \ubcf4\ud5e4\ubbf8\uc544\uc5d0\uc11c\ub294 \ucd5c\ud6c4\uc758 \uac1c\uc2e0\uad50 \ub3d9\ub9f9\uc774\uc5c8\ub2e4. ", "paragraph_answer": "\uc774\ub7ec\ud55c \uc5f4\uc545\ud55c \uc0c1\ud669\uc5d0\ub3c4 \ubd88\uad6c\ud558\uace0 \ubc18\ub780\uc740 \ubcf4\ud5e4\ubbf8\uc544\uc778\ub4e4\uc5d0\uac8c \uc720\ub9ac\ud558\uac8c \ub3cc\uc544\uac14\ub2e4. \uadc0\uc871\ub4e4 \ub300\ubd80\ubd84\uc774 \ub8e8\ud130\ud30c\uc640 \uce7c\ubc45\ud30c\uc600\ub358 \uc624\ubc84\uc678\uc2a4\ud130\ub77c\uc774\ud788\uac00 \ubc18\ub780\uc5d0 \ub3d9\ucc38\ud588\uace0, \uace7 \ub2c8\ub354\uc678\uc2a4\ud130\ub77c\uc774\ud788\uac00 \ubc18\ub780\uc744 \uc77c\uc73c\ucf30\ub2e4. 1619\ub144\uc5d0\ub294 \uc9c4\ud2b8\ub9ac\ud788 \ub9c8\ud0c0\uc57c\uc2a4 \ud2b8\ub7f0\uc774 \ube44\uc5d4\ub098 \uc131\ubcbd\uc744 \ud5a5\ud574 \uacf5\uaca9\uc744 \uac00\ud588\ub2e4. \ub354\uc6b1\uc774 \uc601\uad6d \uc81c\ub3c4\uc5d0\uc11c\ub294 \ud504\ub9ac\ub4dc\ub9ac\ud788 5\uc138\uc758 \ubc18\ub780\uc774 \uc720\ub7fd\uc758 \uc870\uc5d8\ub9ac\ub77c \ubb18\uc0ac\ub418\ub294 \uc5d8\ub9ac\uc790\ubca0\uc2a4 \uc2a4\ud29c\uc5b4\ud2b8\uc758 \ubb38\uc81c\ub85c \ube44\ucdb0\uc9c0\uac8c \ub418\uc5c8\ub2e4. \uc774\ub294 30\ub144 \uc804\uc7c1 \uc804\ubc18\uc5d0 \uac78\uccd0 \uc218\ucc9c \uba85\uc758 \uc9c0\uc9c0\uc790\ub4e4\uc774 \uc5d8\ub9ac\uc790\ubca0\uc2a4 \uc2a4\ud29c\uc5b4\ud2b8\uac00 \uc804\uc7c1\uc758 \uc6d0\uc778\uc774\ub77c\ub294 \ud750\ub984\uc744 \ub0b4\ub193\uae30\ub3c4 \ud588\ub2e4. \ud638\ub808\uc774\uc2a4 \ubca0\ub808\uac00 \uc774\ub044\ub294 \uc601\uad6d-\ub124\ub35c\ub780\ub4dc \uc5f0\ud569\uad70\uc774 \uc804\uc7c1 \ucd08\uae30\uc5d0 \ud314\uce20 \uc120\uc81c\ud6c4 \uc9c0\uc5ed\uc73c\ub85c \uc774\ub3d9\ud588\uace0 \uc874 \uc138\ud1a4\uc774 \uc774\ub044\ub294 \uc2a4\ucf54\ud2c0\ub79c\ub4dc-\ub124\ub35c\ub780\ub4dc \uc5f0\ud569\uad70\uc774 \ubcf4\ud5e4\ubbf8\uc544\ub85c \uc774\ub3d9\ud588\ub2e4. \uadf8\ub9ac\uace0 \ud6c4\uc5d0 \uc774\ub4e4\uc740 \ud63c\uc131 \ubd80\ub300\uc778 \ube0c\ub974\ud0c0\ub274 \uc5ec\ub2e8\uc5d0 \ucc38\uc5ec\ud558\uac8c \ub41c\ub2e4. \uc138\ud1a4\uc758 \uc5ec\ub2e8 \uc740 1622\ub144\uae4c\uc9c0 \ud2b8\ub808\ubcf8 \ub9c8\uc744\uc744 \uc0ac\uc218\ud55c \uc5ec\ub2e8\uc73c\ub85c, \ubcf4\ud5e4\ubbf8\uc544\uc5d0\uc11c\ub294 \ucd5c\ud6c4\uc758 \uac1c\uc2e0\uad50 \ub3d9\ub9f9\uc774\uc5c8\ub2e4.", "sentence_answer": " \uc138\ud1a4\uc758 \uc5ec\ub2e8 \uc740 1622\ub144\uae4c\uc9c0 \ud2b8\ub808\ubcf8 \ub9c8\uc744\uc744 \uc0ac\uc218\ud55c \uc5ec\ub2e8\uc73c\ub85c, \ubcf4\ud5e4\ubbf8\uc544\uc5d0\uc11c\ub294 \ucd5c\ud6c4\uc758 \uac1c\uc2e0\uad50 \ub3d9\ub9f9\uc774\uc5c8\ub2e4.", "paragraph_id": "6499337-12-0", "paragraph_question": "question: 1622\ub144\uae4c\uc9c0 \ud2b8\ub808\ubcf8 \ub9c8\uc744\uc744 \uc0ac\uc218\ud55c \uc5ec\ub2e8\uc740?, context: \uc774\ub7ec\ud55c \uc5f4\uc545\ud55c \uc0c1\ud669\uc5d0\ub3c4 \ubd88\uad6c\ud558\uace0 \ubc18\ub780\uc740 \ubcf4\ud5e4\ubbf8\uc544\uc778\ub4e4\uc5d0\uac8c \uc720\ub9ac\ud558\uac8c \ub3cc\uc544\uac14\ub2e4. \uadc0\uc871\ub4e4 \ub300\ubd80\ubd84\uc774 \ub8e8\ud130\ud30c\uc640 \uce7c\ubc45\ud30c\uc600\ub358 \uc624\ubc84\uc678\uc2a4\ud130\ub77c\uc774\ud788\uac00 \ubc18\ub780\uc5d0 \ub3d9\ucc38\ud588\uace0, \uace7 \ub2c8\ub354\uc678\uc2a4\ud130\ub77c\uc774\ud788\uac00 \ubc18\ub780\uc744 \uc77c\uc73c\ucf30\ub2e4. 1619\ub144\uc5d0\ub294 \uc9c4\ud2b8\ub9ac\ud788 \ub9c8\ud0c0\uc57c\uc2a4 \ud2b8\ub7f0\uc774 \ube44\uc5d4\ub098 \uc131\ubcbd\uc744 \ud5a5\ud574 \uacf5\uaca9\uc744 \uac00\ud588\ub2e4. \ub354\uc6b1\uc774 \uc601\uad6d \uc81c\ub3c4\uc5d0\uc11c\ub294 \ud504\ub9ac\ub4dc\ub9ac\ud788 5\uc138\uc758 \ubc18\ub780\uc774 \uc720\ub7fd\uc758 \uc870\uc5d8\ub9ac\ub77c \ubb18\uc0ac\ub418\ub294 \uc5d8\ub9ac\uc790\ubca0\uc2a4 \uc2a4\ud29c\uc5b4\ud2b8\uc758 \ubb38\uc81c\ub85c \ube44\ucdb0\uc9c0\uac8c \ub418\uc5c8\ub2e4. \uc774\ub294 30\ub144 \uc804\uc7c1 \uc804\ubc18\uc5d0 \uac78\uccd0 \uc218\ucc9c \uba85\uc758 \uc9c0\uc9c0\uc790\ub4e4\uc774 \uc5d8\ub9ac\uc790\ubca0\uc2a4 \uc2a4\ud29c\uc5b4\ud2b8\uac00 \uc804\uc7c1\uc758 \uc6d0\uc778\uc774\ub77c\ub294 \ud750\ub984\uc744 \ub0b4\ub193\uae30\ub3c4 \ud588\ub2e4. \ud638\ub808\uc774\uc2a4 \ubca0\ub808\uac00 \uc774\ub044\ub294 \uc601\uad6d-\ub124\ub35c\ub780\ub4dc \uc5f0\ud569\uad70\uc774 \uc804\uc7c1 \ucd08\uae30\uc5d0 \ud314\uce20 \uc120\uc81c\ud6c4 \uc9c0\uc5ed\uc73c\ub85c \uc774\ub3d9\ud588\uace0 \uc874 \uc138\ud1a4\uc774 \uc774\ub044\ub294 \uc2a4\ucf54\ud2c0\ub79c\ub4dc-\ub124\ub35c\ub780\ub4dc \uc5f0\ud569\uad70\uc774 \ubcf4\ud5e4\ubbf8\uc544\ub85c \uc774\ub3d9\ud588\ub2e4. \uadf8\ub9ac\uace0 \ud6c4\uc5d0 \uc774\ub4e4\uc740 \ud63c\uc131 \ubd80\ub300\uc778 \ube0c\ub974\ud0c0\ub274 \uc5ec\ub2e8\uc5d0 \ucc38\uc5ec\ud558\uac8c \ub41c\ub2e4. \uc138\ud1a4\uc758 \uc5ec\ub2e8\uc740 1622\ub144\uae4c\uc9c0 \ud2b8\ub808\ubcf8 \ub9c8\uc744\uc744 \uc0ac\uc218\ud55c \uc5ec\ub2e8\uc73c\ub85c, \ubcf4\ud5e4\ubbf8\uc544\uc5d0\uc11c\ub294 \ucd5c\ud6c4\uc758 \uac1c\uc2e0\uad50 \ub3d9\ub9f9\uc774\uc5c8\ub2e4."} {"question": "\uc774\ud718\uc18c\uc758 \uc0dd\uc560\ub97c \uc8fc\uc81c\ub85c \ud55c \uc18c\uc124 \ubb34\uad81\ud654 \uaf43\uc774 \ud53c\uc5c8\uc2b5\ub2c8\ub2e4\ub97c \ucd9c\ud310\ud55c \uc18c\uc124\uac00\ub294?", "paragraph": "\uc774\ud718\uc18c\ub294 1977\ub144 10\uc6d4\uc5d0 \uad6d\ubbfc\ud6c8\uc7a5 \ub3d9\ubc31\uc7a5\uc5d0 \ucd94\uc11c\ub410\uc73c\uba70, 2006\ub144\uc5d0\ub294 \ud55c\uad6d\uacfc\ud559\uae30\uc220\ud55c\ub9bc\uc6d0\uc5d0 \uc758\ud574 \ud55c\uad6d\uacfc\ud559\uae30\uc220\uc778 \uba85\uc608\uc758 \uc804\ub2f9\uc5d0 \ud5cc\uc815(2005\ub144\ub3c4 \ud5cc\uc815 \ub300\uc0c1\uc790)\ub410\ub2e4. \uc774\ud718\uc18c\uc758 \uc0ac\ud6c4, \uc18c\uc124\uac00 \uae40\uc9c4\uba85\uc5d0 \uc758\ud558\uc5ec \uc774\ud718\uc18c\uc758 \uc0dd\uc560\ub97c \uc8fc\uc81c\ub85c \ud55c \uc18c\uc124 \u300a\ubb34\uad81\ud654 \uaf43\uc774 \ud53c\uc5c8\uc2b5\ub2c8\ub2e4\u300b\uac00 \ucd9c\ud310\ub410\ub294\ub370, \uc774\uc5d0 \ub300\ud574 \uc774\ud718\uc18c\uc758 \ubbf8\ub9dd\uc778(Marianne Mun Ching Sim, \uc911\uad6d\uc5b4: \u6c88\u66fc\u83c1) \ub4f1 \uc720\uc871\uc5d0 \uc758\ud558\uc5ec \uc18c\uc124\uc5d0\uc11c \uc774\ud718\uc18c \ubc15\uc0ac\uc758 \uc77c\uae30, \ud3b8\uc9c0 \ub4f1\uc744 \ubb34\ub2e8 \uc804\uc7ac\ud558\uac70\ub098 \uc778\uc6a9\ud558\uc5ec \uc800\uc791\uad8c\uacfc \ud504\ub77c\uc774\ubc84\uc2dc\uac00 \uce68\ud574\ub410\uace0, \uc774\ud718\uc18c \ubc15\uc0ac\uac00 \uad50\ud1b5\uc0ac\uace0\ub85c \uc0ac\ub9dd\ud558\uc600\uc74c\uc5d0\ub3c4 \uc18c\uc124\uc5d0\uc11c \ub300\ud55c\ubbfc\uad6d\uc758 \ud575\uac1c\ubc1c\uacfc \uad00\ub828\ud558\uc5ec \ubbf8\uad6d\uc758 \uc815\ubcf4\uae30\uad00\uc5d0 \uc758\ud55c \uacf5\uc791\uc5d0 \uc758\ud558\uc5ec \uc0b4\ud574\ub41c \uac83\uc73c\ub85c \ubb18\uc0ac\ud558\uc5ec \uace0\uc778\uc758 \uba85\uc608\uac00 \ud6fc\uc190\ub410\ub2e4\ub294 \uc810\uc744 \uc774\uc720\ub85c \uc18c\uc124\uc758 \ucd9c\ud310 \ubc0f \ud310\ub9e4 \uae08\uc9c0 \ub4f1 \uac00\ucc98\ubd84\uc2e0\uccad\uc774 \uc788\uc5c8\uc73c\ub098, \ubc95\uc6d0\uc5d0\uc11c\ub294 \uc6d0\uace0\uc758 \uccad\uad6c\ub97c \uae30\uac01\ud558\uc600\ub2e4.", "answer": "\uae40\uc9c4\uba85", "sentence": "\uc774\ud718\uc18c\uc758 \uc0ac\ud6c4, \uc18c\uc124\uac00 \uae40\uc9c4\uba85 \uc5d0 \uc758\ud558\uc5ec \uc774\ud718\uc18c\uc758 \uc0dd\uc560\ub97c \uc8fc\uc81c\ub85c \ud55c \uc18c\uc124 \u300a\ubb34\uad81\ud654 \uaf43\uc774 \ud53c\uc5c8\uc2b5\ub2c8\ub2e4\u300b\uac00 \ucd9c\ud310\ub410\ub294\ub370, \uc774\uc5d0 \ub300\ud574 \uc774\ud718\uc18c\uc758 \ubbf8\ub9dd\uc778(Marianne Mun Ching Sim, \uc911\uad6d\uc5b4: \u6c88\u66fc\u83c1) \ub4f1 \uc720\uc871\uc5d0 \uc758\ud558\uc5ec \uc18c\uc124\uc5d0\uc11c \uc774\ud718\uc18c \ubc15\uc0ac\uc758 \uc77c\uae30, \ud3b8\uc9c0 \ub4f1\uc744 \ubb34\ub2e8 \uc804\uc7ac\ud558\uac70\ub098 \uc778\uc6a9\ud558\uc5ec \uc800\uc791\uad8c\uacfc \ud504\ub77c\uc774\ubc84\uc2dc\uac00 \uce68\ud574\ub410\uace0, \uc774\ud718\uc18c \ubc15\uc0ac\uac00 \uad50\ud1b5\uc0ac\uace0\ub85c \uc0ac\ub9dd\ud558\uc600\uc74c\uc5d0\ub3c4 \uc18c\uc124\uc5d0\uc11c \ub300\ud55c\ubbfc\uad6d\uc758 \ud575\uac1c\ubc1c\uacfc \uad00\ub828\ud558\uc5ec \ubbf8\uad6d\uc758 \uc815\ubcf4\uae30\uad00\uc5d0 \uc758\ud55c \uacf5\uc791\uc5d0 \uc758\ud558\uc5ec \uc0b4\ud574\ub41c \uac83\uc73c\ub85c \ubb18\uc0ac\ud558\uc5ec \uace0\uc778\uc758 \uba85\uc608\uac00 \ud6fc\uc190\ub410\ub2e4\ub294 \uc810\uc744 \uc774\uc720\ub85c \uc18c\uc124\uc758 \ucd9c\ud310 \ubc0f \ud310\ub9e4 \uae08\uc9c0 \ub4f1 \uac00\ucc98\ubd84\uc2e0\uccad\uc774 \uc788\uc5c8\uc73c\ub098, \ubc95\uc6d0\uc5d0\uc11c\ub294 \uc6d0\uace0\uc758 \uccad\uad6c\ub97c \uae30\uac01\ud558\uc600\ub2e4.", "paragraph_sentence": "\uc774\ud718\uc18c\ub294 1977\ub144 10\uc6d4\uc5d0 \uad6d\ubbfc\ud6c8\uc7a5 \ub3d9\ubc31\uc7a5\uc5d0 \ucd94\uc11c\ub410\uc73c\uba70, 2006\ub144\uc5d0\ub294 \ud55c\uad6d\uacfc\ud559\uae30\uc220\ud55c\ub9bc\uc6d0\uc5d0 \uc758\ud574 \ud55c\uad6d\uacfc\ud559\uae30\uc220\uc778 \uba85\uc608\uc758 \uc804\ub2f9\uc5d0 \ud5cc\uc815(2005\ub144\ub3c4 \ud5cc\uc815 \ub300\uc0c1\uc790)\ub410\ub2e4. \uc774\ud718\uc18c\uc758 \uc0ac\ud6c4, \uc18c\uc124\uac00 \uae40\uc9c4\uba85 \uc5d0 \uc758\ud558\uc5ec \uc774\ud718\uc18c\uc758 \uc0dd\uc560\ub97c \uc8fc\uc81c\ub85c \ud55c \uc18c\uc124 \u300a\ubb34\uad81\ud654 \uaf43\uc774 \ud53c\uc5c8\uc2b5\ub2c8\ub2e4\u300b\uac00 \ucd9c\ud310\ub410\ub294\ub370, \uc774\uc5d0 \ub300\ud574 \uc774\ud718\uc18c\uc758 \ubbf8\ub9dd\uc778(Marianne Mun Ching Sim, \uc911\uad6d\uc5b4: \u6c88\u66fc\u83c1) \ub4f1 \uc720\uc871\uc5d0 \uc758\ud558\uc5ec \uc18c\uc124\uc5d0\uc11c \uc774\ud718\uc18c \ubc15\uc0ac\uc758 \uc77c\uae30, \ud3b8\uc9c0 \ub4f1\uc744 \ubb34\ub2e8 \uc804\uc7ac\ud558\uac70\ub098 \uc778\uc6a9\ud558\uc5ec \uc800\uc791\uad8c\uacfc \ud504\ub77c\uc774\ubc84\uc2dc\uac00 \uce68\ud574\ub410\uace0, \uc774\ud718\uc18c \ubc15\uc0ac\uac00 \uad50\ud1b5\uc0ac\uace0\ub85c \uc0ac\ub9dd\ud558\uc600\uc74c\uc5d0\ub3c4 \uc18c\uc124\uc5d0\uc11c \ub300\ud55c\ubbfc\uad6d\uc758 \ud575\uac1c\ubc1c\uacfc \uad00\ub828\ud558\uc5ec \ubbf8\uad6d\uc758 \uc815\ubcf4\uae30\uad00\uc5d0 \uc758\ud55c \uacf5\uc791\uc5d0 \uc758\ud558\uc5ec \uc0b4\ud574\ub41c \uac83\uc73c\ub85c \ubb18\uc0ac\ud558\uc5ec \uace0\uc778\uc758 \uba85\uc608\uac00 \ud6fc\uc190\ub410\ub2e4\ub294 \uc810\uc744 \uc774\uc720\ub85c \uc18c\uc124\uc758 \ucd9c\ud310 \ubc0f \ud310\ub9e4 \uae08\uc9c0 \ub4f1 \uac00\ucc98\ubd84\uc2e0\uccad\uc774 \uc788\uc5c8\uc73c\ub098, \ubc95\uc6d0\uc5d0\uc11c\ub294 \uc6d0\uace0\uc758 \uccad\uad6c\ub97c \uae30\uac01\ud558\uc600\ub2e4. ", "paragraph_answer": "\uc774\ud718\uc18c\ub294 1977\ub144 10\uc6d4\uc5d0 \uad6d\ubbfc\ud6c8\uc7a5 \ub3d9\ubc31\uc7a5\uc5d0 \ucd94\uc11c\ub410\uc73c\uba70, 2006\ub144\uc5d0\ub294 \ud55c\uad6d\uacfc\ud559\uae30\uc220\ud55c\ub9bc\uc6d0\uc5d0 \uc758\ud574 \ud55c\uad6d\uacfc\ud559\uae30\uc220\uc778 \uba85\uc608\uc758 \uc804\ub2f9\uc5d0 \ud5cc\uc815(2005\ub144\ub3c4 \ud5cc\uc815 \ub300\uc0c1\uc790)\ub410\ub2e4. \uc774\ud718\uc18c\uc758 \uc0ac\ud6c4, \uc18c\uc124\uac00 \uae40\uc9c4\uba85 \uc5d0 \uc758\ud558\uc5ec \uc774\ud718\uc18c\uc758 \uc0dd\uc560\ub97c \uc8fc\uc81c\ub85c \ud55c \uc18c\uc124 \u300a\ubb34\uad81\ud654 \uaf43\uc774 \ud53c\uc5c8\uc2b5\ub2c8\ub2e4\u300b\uac00 \ucd9c\ud310\ub410\ub294\ub370, \uc774\uc5d0 \ub300\ud574 \uc774\ud718\uc18c\uc758 \ubbf8\ub9dd\uc778(Marianne Mun Ching Sim, \uc911\uad6d\uc5b4: \u6c88\u66fc\u83c1) \ub4f1 \uc720\uc871\uc5d0 \uc758\ud558\uc5ec \uc18c\uc124\uc5d0\uc11c \uc774\ud718\uc18c \ubc15\uc0ac\uc758 \uc77c\uae30, \ud3b8\uc9c0 \ub4f1\uc744 \ubb34\ub2e8 \uc804\uc7ac\ud558\uac70\ub098 \uc778\uc6a9\ud558\uc5ec \uc800\uc791\uad8c\uacfc \ud504\ub77c\uc774\ubc84\uc2dc\uac00 \uce68\ud574\ub410\uace0, \uc774\ud718\uc18c \ubc15\uc0ac\uac00 \uad50\ud1b5\uc0ac\uace0\ub85c \uc0ac\ub9dd\ud558\uc600\uc74c\uc5d0\ub3c4 \uc18c\uc124\uc5d0\uc11c \ub300\ud55c\ubbfc\uad6d\uc758 \ud575\uac1c\ubc1c\uacfc \uad00\ub828\ud558\uc5ec \ubbf8\uad6d\uc758 \uc815\ubcf4\uae30\uad00\uc5d0 \uc758\ud55c \uacf5\uc791\uc5d0 \uc758\ud558\uc5ec \uc0b4\ud574\ub41c \uac83\uc73c\ub85c \ubb18\uc0ac\ud558\uc5ec \uace0\uc778\uc758 \uba85\uc608\uac00 \ud6fc\uc190\ub410\ub2e4\ub294 \uc810\uc744 \uc774\uc720\ub85c \uc18c\uc124\uc758 \ucd9c\ud310 \ubc0f \ud310\ub9e4 \uae08\uc9c0 \ub4f1 \uac00\ucc98\ubd84\uc2e0\uccad\uc774 \uc788\uc5c8\uc73c\ub098, \ubc95\uc6d0\uc5d0\uc11c\ub294 \uc6d0\uace0\uc758 \uccad\uad6c\ub97c \uae30\uac01\ud558\uc600\ub2e4.", "sentence_answer": "\uc774\ud718\uc18c\uc758 \uc0ac\ud6c4, \uc18c\uc124\uac00 \uae40\uc9c4\uba85 \uc5d0 \uc758\ud558\uc5ec \uc774\ud718\uc18c\uc758 \uc0dd\uc560\ub97c \uc8fc\uc81c\ub85c \ud55c \uc18c\uc124 \u300a\ubb34\uad81\ud654 \uaf43\uc774 \ud53c\uc5c8\uc2b5\ub2c8\ub2e4\u300b\uac00 \ucd9c\ud310\ub410\ub294\ub370, \uc774\uc5d0 \ub300\ud574 \uc774\ud718\uc18c\uc758 \ubbf8\ub9dd\uc778(Marianne Mun Ching Sim, \uc911\uad6d\uc5b4: \u6c88\u66fc\u83c1) \ub4f1 \uc720\uc871\uc5d0 \uc758\ud558\uc5ec \uc18c\uc124\uc5d0\uc11c \uc774\ud718\uc18c \ubc15\uc0ac\uc758 \uc77c\uae30, \ud3b8\uc9c0 \ub4f1\uc744 \ubb34\ub2e8 \uc804\uc7ac\ud558\uac70\ub098 \uc778\uc6a9\ud558\uc5ec \uc800\uc791\uad8c\uacfc \ud504\ub77c\uc774\ubc84\uc2dc\uac00 \uce68\ud574\ub410\uace0, \uc774\ud718\uc18c \ubc15\uc0ac\uac00 \uad50\ud1b5\uc0ac\uace0\ub85c \uc0ac\ub9dd\ud558\uc600\uc74c\uc5d0\ub3c4 \uc18c\uc124\uc5d0\uc11c \ub300\ud55c\ubbfc\uad6d\uc758 \ud575\uac1c\ubc1c\uacfc \uad00\ub828\ud558\uc5ec \ubbf8\uad6d\uc758 \uc815\ubcf4\uae30\uad00\uc5d0 \uc758\ud55c \uacf5\uc791\uc5d0 \uc758\ud558\uc5ec \uc0b4\ud574\ub41c \uac83\uc73c\ub85c \ubb18\uc0ac\ud558\uc5ec \uace0\uc778\uc758 \uba85\uc608\uac00 \ud6fc\uc190\ub410\ub2e4\ub294 \uc810\uc744 \uc774\uc720\ub85c \uc18c\uc124\uc758 \ucd9c\ud310 \ubc0f \ud310\ub9e4 \uae08\uc9c0 \ub4f1 \uac00\ucc98\ubd84\uc2e0\uccad\uc774 \uc788\uc5c8\uc73c\ub098, \ubc95\uc6d0\uc5d0\uc11c\ub294 \uc6d0\uace0\uc758 \uccad\uad6c\ub97c \uae30\uac01\ud558\uc600\ub2e4.", "paragraph_id": "6329268-15-1", "paragraph_question": "question: \uc774\ud718\uc18c\uc758 \uc0dd\uc560\ub97c \uc8fc\uc81c\ub85c \ud55c \uc18c\uc124 \ubb34\uad81\ud654 \uaf43\uc774 \ud53c\uc5c8\uc2b5\ub2c8\ub2e4\ub97c \ucd9c\ud310\ud55c \uc18c\uc124\uac00\ub294?, context: \uc774\ud718\uc18c\ub294 1977\ub144 10\uc6d4\uc5d0 \uad6d\ubbfc\ud6c8\uc7a5 \ub3d9\ubc31\uc7a5\uc5d0 \ucd94\uc11c\ub410\uc73c\uba70, 2006\ub144\uc5d0\ub294 \ud55c\uad6d\uacfc\ud559\uae30\uc220\ud55c\ub9bc\uc6d0\uc5d0 \uc758\ud574 \ud55c\uad6d\uacfc\ud559\uae30\uc220\uc778 \uba85\uc608\uc758 \uc804\ub2f9\uc5d0 \ud5cc\uc815(2005\ub144\ub3c4 \ud5cc\uc815 \ub300\uc0c1\uc790)\ub410\ub2e4. \uc774\ud718\uc18c\uc758 \uc0ac\ud6c4, \uc18c\uc124\uac00 \uae40\uc9c4\uba85\uc5d0 \uc758\ud558\uc5ec \uc774\ud718\uc18c\uc758 \uc0dd\uc560\ub97c \uc8fc\uc81c\ub85c \ud55c \uc18c\uc124 \u300a\ubb34\uad81\ud654 \uaf43\uc774 \ud53c\uc5c8\uc2b5\ub2c8\ub2e4\u300b\uac00 \ucd9c\ud310\ub410\ub294\ub370, \uc774\uc5d0 \ub300\ud574 \uc774\ud718\uc18c\uc758 \ubbf8\ub9dd\uc778(Marianne Mun Ching Sim, \uc911\uad6d\uc5b4: \u6c88\u66fc\u83c1) \ub4f1 \uc720\uc871\uc5d0 \uc758\ud558\uc5ec \uc18c\uc124\uc5d0\uc11c \uc774\ud718\uc18c \ubc15\uc0ac\uc758 \uc77c\uae30, \ud3b8\uc9c0 \ub4f1\uc744 \ubb34\ub2e8 \uc804\uc7ac\ud558\uac70\ub098 \uc778\uc6a9\ud558\uc5ec \uc800\uc791\uad8c\uacfc \ud504\ub77c\uc774\ubc84\uc2dc\uac00 \uce68\ud574\ub410\uace0, \uc774\ud718\uc18c \ubc15\uc0ac\uac00 \uad50\ud1b5\uc0ac\uace0\ub85c \uc0ac\ub9dd\ud558\uc600\uc74c\uc5d0\ub3c4 \uc18c\uc124\uc5d0\uc11c \ub300\ud55c\ubbfc\uad6d\uc758 \ud575\uac1c\ubc1c\uacfc \uad00\ub828\ud558\uc5ec \ubbf8\uad6d\uc758 \uc815\ubcf4\uae30\uad00\uc5d0 \uc758\ud55c \uacf5\uc791\uc5d0 \uc758\ud558\uc5ec \uc0b4\ud574\ub41c \uac83\uc73c\ub85c \ubb18\uc0ac\ud558\uc5ec \uace0\uc778\uc758 \uba85\uc608\uac00 \ud6fc\uc190\ub410\ub2e4\ub294 \uc810\uc744 \uc774\uc720\ub85c \uc18c\uc124\uc758 \ucd9c\ud310 \ubc0f \ud310\ub9e4 \uae08\uc9c0 \ub4f1 \uac00\ucc98\ubd84\uc2e0\uccad\uc774 \uc788\uc5c8\uc73c\ub098, \ubc95\uc6d0\uc5d0\uc11c\ub294 \uc6d0\uace0\uc758 \uccad\uad6c\ub97c \uae30\uac01\ud558\uc600\ub2e4."} {"question": "\uc720\ub7fd\ucc0c\ub974\ub808\uae30 \ub465\uc9c0\uc5d0\uc11c \uac00\uc7a5 \ud754\ud55c \ubcbc\ub8e9\uc740 \ubb34\uc5c7\uc778\uac00?", "paragraph": "\uc720\ub7fd\ucc0c\ub974\ub808\uae30 \ub465\uc9c0\uc5d0 \uac00\uc7a5 \ud754\ud55c \ubcbc\ub8e9\uc740 \ub2ed\ubcbc\ub8e9(Ceratophyllus gallinae)\uc774\ub2e4. \uc791\uace0 \ud558\uc580 \uc9d1\ucc38\uc0c8\ubcbc\ub8e9(C. fringillae)\ub3c4 \uc885\uc885 \ubcf4\uc774\ub294\ub370, \uc544\ub9c8 \uc720\ub7fd\ucc0c\ub974\ub808\uae30\uac00 \ucc38\uc0c8 \ub465\uc9c0\ub97c \ube7c\uc557\uc740 \uacbd\uc6b0\uc778 \uac83 \uac19\ub2e4. \uc774 \ubcbc\ub8e9\uc740 \ubbf8\uad6d\uc5d0\uc11c \ubc1c\ubcd1\ud55c \uc608\uac00 \uc5c6\uc73c\uba70, \uc2ec\uc9c0\uc5b4 \ubbf8\uad6d\uc758 \uc9d1\ucc38\uc0c8\ub4e4\ub3c4 \ubc1c\ubcd1\ud55c \uc801\uc774 \uc5c6\ub2e4. \uc774\ub294 Menacanthus eurystemus, Brueelia nebulosa, Stumidoecus sturni \ub4f1\uc774 \uae30\uc0dd\ud55c\ub2e4. \uadf8 \uc678\uc5d0 \ubb34\ucc99\ucd94\ub3d9\ubb3c \uae30\uc0dd\ucda9\uc73c\ub85c\ub294 \ucc38\uc9c4\ub4dc\uae30\uc18d(Ixodes)\uc758 \uc9c4\ub4dc\uae30\ub4e4\uc774\ub098 Analgopsis passerinus, Boydaia stumi, \uc591\uacc4\uc7a5\uc751\uc560(Dermanyssus gallinae), Ornithonyssus bursa, O. sylviarum, Proctophyllodes species, Pteronyssoides truncatus, Trouessartia rosteri \ub4f1\uc758 \uc751\uc560\uac00 \uc788\ub2e4. \uc591\uacc4\uc7a5\uc751\uc560(D. gallinae)\ub294 Androlaelaps casalis\ub77c\ub294 \uc721\uc2dd\uc131 \uc751\uc560\uc5d0\uac8c \uc7a1\uc544\uba39\ud78c\ub2e4. \uae30\uc0dd\ucda9\uacc4\uc5d0\ub3c4 \uc774\ub7ec\ud55c \uba39\uc774\uc0ac\uc2ac\uc774 \uc874\uc7ac\ud55c\ub2e4\ub294 \uc0ac\uc2e4\uc774 \uc5b4\uca4c\uba74 \uc0c8\ub4e4\uc774 \ub0a1\uc740 \ub465\uc9c0\ub97c \ub2e4\uc2dc \uc0ac\uc6a9\ud558\ub294 \uac83\uc744 \uc124\uba85\ud574 \uc904 \uc9c0\ub3c4 \ubaa8\ub978\ub2e4.", "answer": "\ub2ed\ubcbc\ub8e9", "sentence": "\uc720\ub7fd\ucc0c\ub974\ub808\uae30 \ub465\uc9c0\uc5d0 \uac00\uc7a5 \ud754\ud55c \ubcbc\ub8e9\uc740 \ub2ed\ubcbc\ub8e9 (Ceratophyllus gallinae)\uc774\ub2e4.", "paragraph_sentence": " \uc720\ub7fd\ucc0c\ub974\ub808\uae30 \ub465\uc9c0\uc5d0 \uac00\uc7a5 \ud754\ud55c \ubcbc\ub8e9\uc740 \ub2ed\ubcbc\ub8e9 (Ceratophyllus gallinae)\uc774\ub2e4. \uc791\uace0 \ud558\uc580 \uc9d1\ucc38\uc0c8\ubcbc\ub8e9(C. fringillae)\ub3c4 \uc885\uc885 \ubcf4\uc774\ub294\ub370, \uc544\ub9c8 \uc720\ub7fd\ucc0c\ub974\ub808\uae30\uac00 \ucc38\uc0c8 \ub465\uc9c0\ub97c \ube7c\uc557\uc740 \uacbd\uc6b0\uc778 \uac83 \uac19\ub2e4. \uc774 \ubcbc\ub8e9\uc740 \ubbf8\uad6d\uc5d0\uc11c \ubc1c\ubcd1\ud55c \uc608\uac00 \uc5c6\uc73c\uba70, \uc2ec\uc9c0\uc5b4 \ubbf8\uad6d\uc758 \uc9d1\ucc38\uc0c8\ub4e4\ub3c4 \ubc1c\ubcd1\ud55c \uc801\uc774 \uc5c6\ub2e4. \uc774\ub294 Menacanthus eurystemus, Brueelia nebulosa, Stumidoecus sturni \ub4f1\uc774 \uae30\uc0dd\ud55c\ub2e4. \uadf8 \uc678\uc5d0 \ubb34\ucc99\ucd94\ub3d9\ubb3c \uae30\uc0dd\ucda9\uc73c\ub85c\ub294 \ucc38\uc9c4\ub4dc\uae30\uc18d(Ixodes)\uc758 \uc9c4\ub4dc\uae30\ub4e4\uc774\ub098 Analgopsis passerinus, Boydaia stumi, \uc591\uacc4\uc7a5\uc751\uc560(Dermanyssus gallinae), Ornithonyssus bursa, O. sylviarum, Proctophyllodes species, Pteronyssoides truncatus, Trouessartia rosteri \ub4f1\uc758 \uc751\uc560\uac00 \uc788\ub2e4. \uc591\uacc4\uc7a5\uc751\uc560(D. gallinae)\ub294 Androlaelaps casalis\ub77c\ub294 \uc721\uc2dd\uc131 \uc751\uc560\uc5d0\uac8c \uc7a1\uc544\uba39\ud78c\ub2e4. \uae30\uc0dd\ucda9\uacc4\uc5d0\ub3c4 \uc774\ub7ec\ud55c \uba39\uc774\uc0ac\uc2ac\uc774 \uc874\uc7ac\ud55c\ub2e4\ub294 \uc0ac\uc2e4\uc774 \uc5b4\uca4c\uba74 \uc0c8\ub4e4\uc774 \ub0a1\uc740 \ub465\uc9c0\ub97c \ub2e4\uc2dc \uc0ac\uc6a9\ud558\ub294 \uac83\uc744 \uc124\uba85\ud574 \uc904 \uc9c0\ub3c4 \ubaa8\ub978\ub2e4.", "paragraph_answer": "\uc720\ub7fd\ucc0c\ub974\ub808\uae30 \ub465\uc9c0\uc5d0 \uac00\uc7a5 \ud754\ud55c \ubcbc\ub8e9\uc740 \ub2ed\ubcbc\ub8e9 (Ceratophyllus gallinae)\uc774\ub2e4. \uc791\uace0 \ud558\uc580 \uc9d1\ucc38\uc0c8\ubcbc\ub8e9(C. fringillae)\ub3c4 \uc885\uc885 \ubcf4\uc774\ub294\ub370, \uc544\ub9c8 \uc720\ub7fd\ucc0c\ub974\ub808\uae30\uac00 \ucc38\uc0c8 \ub465\uc9c0\ub97c \ube7c\uc557\uc740 \uacbd\uc6b0\uc778 \uac83 \uac19\ub2e4. \uc774 \ubcbc\ub8e9\uc740 \ubbf8\uad6d\uc5d0\uc11c \ubc1c\ubcd1\ud55c \uc608\uac00 \uc5c6\uc73c\uba70, \uc2ec\uc9c0\uc5b4 \ubbf8\uad6d\uc758 \uc9d1\ucc38\uc0c8\ub4e4\ub3c4 \ubc1c\ubcd1\ud55c \uc801\uc774 \uc5c6\ub2e4. \uc774\ub294 Menacanthus eurystemus, Brueelia nebulosa, Stumidoecus sturni \ub4f1\uc774 \uae30\uc0dd\ud55c\ub2e4. \uadf8 \uc678\uc5d0 \ubb34\ucc99\ucd94\ub3d9\ubb3c \uae30\uc0dd\ucda9\uc73c\ub85c\ub294 \ucc38\uc9c4\ub4dc\uae30\uc18d(Ixodes)\uc758 \uc9c4\ub4dc\uae30\ub4e4\uc774\ub098 Analgopsis passerinus, Boydaia stumi, \uc591\uacc4\uc7a5\uc751\uc560(Dermanyssus gallinae), Ornithonyssus bursa, O. sylviarum, Proctophyllodes species, Pteronyssoides truncatus, Trouessartia rosteri \ub4f1\uc758 \uc751\uc560\uac00 \uc788\ub2e4. \uc591\uacc4\uc7a5\uc751\uc560(D. gallinae)\ub294 Androlaelaps casalis\ub77c\ub294 \uc721\uc2dd\uc131 \uc751\uc560\uc5d0\uac8c \uc7a1\uc544\uba39\ud78c\ub2e4. \uae30\uc0dd\ucda9\uacc4\uc5d0\ub3c4 \uc774\ub7ec\ud55c \uba39\uc774\uc0ac\uc2ac\uc774 \uc874\uc7ac\ud55c\ub2e4\ub294 \uc0ac\uc2e4\uc774 \uc5b4\uca4c\uba74 \uc0c8\ub4e4\uc774 \ub0a1\uc740 \ub465\uc9c0\ub97c \ub2e4\uc2dc \uc0ac\uc6a9\ud558\ub294 \uac83\uc744 \uc124\uba85\ud574 \uc904 \uc9c0\ub3c4 \ubaa8\ub978\ub2e4.", "sentence_answer": "\uc720\ub7fd\ucc0c\ub974\ub808\uae30 \ub465\uc9c0\uc5d0 \uac00\uc7a5 \ud754\ud55c \ubcbc\ub8e9\uc740 \ub2ed\ubcbc\ub8e9 (Ceratophyllus gallinae)\uc774\ub2e4.", "paragraph_id": "6464640-17-0", "paragraph_question": "question: \uc720\ub7fd\ucc0c\ub974\ub808\uae30 \ub465\uc9c0\uc5d0\uc11c \uac00\uc7a5 \ud754\ud55c \ubcbc\ub8e9\uc740 \ubb34\uc5c7\uc778\uac00?, context: \uc720\ub7fd\ucc0c\ub974\ub808\uae30 \ub465\uc9c0\uc5d0 \uac00\uc7a5 \ud754\ud55c \ubcbc\ub8e9\uc740 \ub2ed\ubcbc\ub8e9(Ceratophyllus gallinae)\uc774\ub2e4. \uc791\uace0 \ud558\uc580 \uc9d1\ucc38\uc0c8\ubcbc\ub8e9(C. fringillae)\ub3c4 \uc885\uc885 \ubcf4\uc774\ub294\ub370, \uc544\ub9c8 \uc720\ub7fd\ucc0c\ub974\ub808\uae30\uac00 \ucc38\uc0c8 \ub465\uc9c0\ub97c \ube7c\uc557\uc740 \uacbd\uc6b0\uc778 \uac83 \uac19\ub2e4. \uc774 \ubcbc\ub8e9\uc740 \ubbf8\uad6d\uc5d0\uc11c \ubc1c\ubcd1\ud55c \uc608\uac00 \uc5c6\uc73c\uba70, \uc2ec\uc9c0\uc5b4 \ubbf8\uad6d\uc758 \uc9d1\ucc38\uc0c8\ub4e4\ub3c4 \ubc1c\ubcd1\ud55c \uc801\uc774 \uc5c6\ub2e4. \uc774\ub294 Menacanthus eurystemus, Brueelia nebulosa, Stumidoecus sturni \ub4f1\uc774 \uae30\uc0dd\ud55c\ub2e4. \uadf8 \uc678\uc5d0 \ubb34\ucc99\ucd94\ub3d9\ubb3c \uae30\uc0dd\ucda9\uc73c\ub85c\ub294 \ucc38\uc9c4\ub4dc\uae30\uc18d(Ixodes)\uc758 \uc9c4\ub4dc\uae30\ub4e4\uc774\ub098 Analgopsis passerinus, Boydaia stumi, \uc591\uacc4\uc7a5\uc751\uc560(Dermanyssus gallinae), Ornithonyssus bursa, O. sylviarum, Proctophyllodes species, Pteronyssoides truncatus, Trouessartia rosteri \ub4f1\uc758 \uc751\uc560\uac00 \uc788\ub2e4. \uc591\uacc4\uc7a5\uc751\uc560(D. gallinae)\ub294 Androlaelaps casalis\ub77c\ub294 \uc721\uc2dd\uc131 \uc751\uc560\uc5d0\uac8c \uc7a1\uc544\uba39\ud78c\ub2e4. \uae30\uc0dd\ucda9\uacc4\uc5d0\ub3c4 \uc774\ub7ec\ud55c \uba39\uc774\uc0ac\uc2ac\uc774 \uc874\uc7ac\ud55c\ub2e4\ub294 \uc0ac\uc2e4\uc774 \uc5b4\uca4c\uba74 \uc0c8\ub4e4\uc774 \ub0a1\uc740 \ub465\uc9c0\ub97c \ub2e4\uc2dc \uc0ac\uc6a9\ud558\ub294 \uac83\uc744 \uc124\uba85\ud574 \uc904 \uc9c0\ub3c4 \ubaa8\ub978\ub2e4."} {"question": "1998\ub144 5\uc6d4 16\uc77c \ucd9c\uc804\ud55c \uacbd\uae30\uc758 \uc0c1\ub300\ud300\uc740?", "paragraph": "\uc774\ub4ec\ud574 1998\ub144\uc5d0\ub294 \uc5b4\uae68 \uc0c1\ud0dc\uac00 \ud68c\ubcf5\ub410\uace0 \uc2a4\ud504\ub9c1 \ucea0\ud504\uc5d0\uc11c \uadfc\ub825 \uac15\ud654\ub098 \uba40\ub9ac \ub358\uc9c0\ub294 \uc5f0\uc2b5\uc744 \uac70\ub4ed\ud588\ub2e4. 5\uc6d4 16\uc77c \uc694\ucf54\ud558\ub9c8\uc804(\ub098\uace0\uc57c \ub3d4)\uc5d0\uc11c \uc120\ubc1c\ud55c 1\ud68c\uc5d0 1\uc810\uc744 \ub0b4\uc92c\uc9c0\ub9cc \uadf8 \ud6c4\uc5d0\ub294 \uc9c1\uad6c\ub85c \ubc00\uc5b4\ub0b4\uace0 5\uc774\ub2dd 1\uc2e4\uc810\uc758 \uc131\uc801\uc73c\ub85c \uc2dc\uc98c 2\uc2b9\uc9f8\ub97c \uac70\ub480\uc9c0\ub9cc \uacb0\uacfc\uc801\uc73c\ub85c \uc774 \uc2b9\ub9ac\uac00 \ud604\uc5ed \uc0dd\ud65c\uc5d0 \uc788\uc5b4\uc11c\uc758 \ub9c8\uc9c0\ub9c9 \uc2b9\ub9ac\uc600\ub2e4. 6\uc6d4 5\uc77c \ub098\uace0\uc57c \ub3d4\uc5d0\uc11c \uc5f4\ub9b0 \uc694\ubbf8\uc6b0\ub9ac\uc804\uc5d0\uc11c\ub294 \ucd5c\uace0 \uc18d\ub3c4 140 km/h\ub97c \uae30\ub85d\ud558\uc5ec 90 km/h\uc758 \uc2ac\ub85c \ucee4\ube0c\ub3c4 \ub418\ub3cc\uc544\uc640\uc11c 8\uc774\ub2dd \ub3d9\uc548 131\uac1c\uc758 \uacf5\uc744 \ub358\uc838 2\ud53c\uc548\ud0c0 1\uc2e4\uc810\uc758 \ud638\ud22c\ub85c \uc2b9\ub9ac\ub97c \uac70\ub450\ub294 \ub4f1 \ubd80\ud65c\uc758 \uc870\uc9d0\uc744 \ubcf4\uc600\ub2e4. \uadf8\ub7ec\ub098 \ud638\uc870\uac00 \uc9c0\uc18d\ub418\uc9c0 \uc54a\uc558\uace0 \uc790\uc2e0\uc758 4\uc5f0\ud328\ub85c \ub9de\uc774\ud55c 7\uc6d4 9\uc77c \uc57c\ucfe0\ub974\ud2b8\uc804\uc5d0 \uc120\ubc1c\ub85c \ub098\uc11c 2/3\uc774\ub2dd\uc5d0 5\ud53c\uc548\ud0c0 4\uc2e4\uc810\uc73c\ub85c \ud328\uc804 \ud22c\uc218\uac00 \ub418\uba74\uc11c 2\uc2b9 8\ud328\ub97c \uae30\ub85d\ud558\uc790, \ud638\uc2dc\ub178 \uc13c\uc774\uce58 \uac10\ub3c5\uc73c\ub85c\ubd80\ud130 \ub2e4\uc2dc \uae30\uad50\ud30c\ub85c \uc804\ud5a5\ud558\ub77c\ub294 \uba85\ub839\uc744 \ubc1b\uace0 1\uad70 \ub4f1\ub85d\uc774 \ub9d0\uc18c\ub410\ub2e4. \uac00\uc744\ubd80\ud130\ub294 \uadf8\ub54c\uae4c\uc9c0 \uac15\ud558\uac8c \uac70\ubd80\ud558\uace0 \uc788\uc5c8\ub358 \uc6e8\uc774\ud2b8 \ud2b8\ub808\uc774\ub2dd\uc5d0\ub3c4 \uc2a4\uc2a4\ub85c \ucc38\uac00\ud558\uc5ec \ub7f0\ub2dd\uc774\ub098 \ube44\ud0c8\uae38 \ub2ec\ub9ac\uae30\ub85c \ud558\ubc18\uc2e0\uc744 \uac15\ud654\ud558\ub294 \ub4f1 \uc790\uc2e0\uc73c\ub85c\uc11c\ub294 \ucc98\uc74c\uc73c\ub85c 12\uc6d4\uc5d0 \uc5f4\ub9b0 \uc790\uc8fc \ud2b8\ub808\uc774\ub2dd\uc5d0\ub3c4 \ucc38\uac00\ud588\ub2e4.", "answer": "\uc694\ucf54\ud558\ub9c8", "sentence": "5\uc6d4 16\uc77c \uc694\ucf54\ud558\ub9c8 \uc804(\ub098\uace0\uc57c \ub3d4)\uc5d0\uc11c \uc120\ubc1c\ud55c 1\ud68c\uc5d0 1\uc810\uc744 \ub0b4\uc92c\uc9c0\ub9cc \uadf8 \ud6c4\uc5d0\ub294 \uc9c1\uad6c\ub85c \ubc00\uc5b4\ub0b4\uace0 5\uc774\ub2dd 1\uc2e4\uc810\uc758 \uc131\uc801\uc73c\ub85c \uc2dc\uc98c 2\uc2b9\uc9f8\ub97c \uac70\ub480\uc9c0\ub9cc \uacb0\uacfc\uc801\uc73c\ub85c \uc774 \uc2b9\ub9ac\uac00 \ud604\uc5ed \uc0dd\ud65c\uc5d0 \uc788\uc5b4\uc11c\uc758 \ub9c8\uc9c0\ub9c9 \uc2b9\ub9ac\uc600\ub2e4.", "paragraph_sentence": "\uc774\ub4ec\ud574 1998\ub144\uc5d0\ub294 \uc5b4\uae68 \uc0c1\ud0dc\uac00 \ud68c\ubcf5\ub410\uace0 \uc2a4\ud504\ub9c1 \ucea0\ud504\uc5d0\uc11c \uadfc\ub825 \uac15\ud654\ub098 \uba40\ub9ac \ub358\uc9c0\ub294 \uc5f0\uc2b5\uc744 \uac70\ub4ed\ud588\ub2e4. 5\uc6d4 16\uc77c \uc694\ucf54\ud558\ub9c8 \uc804(\ub098\uace0\uc57c \ub3d4)\uc5d0\uc11c \uc120\ubc1c\ud55c 1\ud68c\uc5d0 1\uc810\uc744 \ub0b4\uc92c\uc9c0\ub9cc \uadf8 \ud6c4\uc5d0\ub294 \uc9c1\uad6c\ub85c \ubc00\uc5b4\ub0b4\uace0 5\uc774\ub2dd 1\uc2e4\uc810\uc758 \uc131\uc801\uc73c\ub85c \uc2dc\uc98c 2\uc2b9\uc9f8\ub97c \uac70\ub480\uc9c0\ub9cc \uacb0\uacfc\uc801\uc73c\ub85c \uc774 \uc2b9\ub9ac\uac00 \ud604\uc5ed \uc0dd\ud65c\uc5d0 \uc788\uc5b4\uc11c\uc758 \ub9c8\uc9c0\ub9c9 \uc2b9\ub9ac\uc600\ub2e4. 6\uc6d4 5\uc77c \ub098\uace0\uc57c \ub3d4\uc5d0\uc11c \uc5f4\ub9b0 \uc694\ubbf8\uc6b0\ub9ac\uc804\uc5d0\uc11c\ub294 \ucd5c\uace0 \uc18d\ub3c4 140 km/h\ub97c \uae30\ub85d\ud558\uc5ec 90 km/h\uc758 \uc2ac\ub85c \ucee4\ube0c\ub3c4 \ub418\ub3cc\uc544\uc640\uc11c 8\uc774\ub2dd \ub3d9\uc548 131\uac1c\uc758 \uacf5\uc744 \ub358\uc838 2\ud53c\uc548\ud0c0 1\uc2e4\uc810\uc758 \ud638\ud22c\ub85c \uc2b9\ub9ac\ub97c \uac70\ub450\ub294 \ub4f1 \ubd80\ud65c\uc758 \uc870\uc9d0\uc744 \ubcf4\uc600\ub2e4. \uadf8\ub7ec\ub098 \ud638\uc870\uac00 \uc9c0\uc18d\ub418\uc9c0 \uc54a\uc558\uace0 \uc790\uc2e0\uc758 4\uc5f0\ud328\ub85c \ub9de\uc774\ud55c 7\uc6d4 9\uc77c \uc57c\ucfe0\ub974\ud2b8\uc804\uc5d0 \uc120\ubc1c\ub85c \ub098\uc11c 2/3\uc774\ub2dd\uc5d0 5\ud53c\uc548\ud0c0 4\uc2e4\uc810\uc73c\ub85c \ud328\uc804 \ud22c\uc218\uac00 \ub418\uba74\uc11c 2\uc2b9 8\ud328\ub97c \uae30\ub85d\ud558\uc790, \ud638\uc2dc\ub178 \uc13c\uc774\uce58 \uac10\ub3c5\uc73c\ub85c\ubd80\ud130 \ub2e4\uc2dc \uae30\uad50\ud30c\ub85c \uc804\ud5a5\ud558\ub77c\ub294 \uba85\ub839\uc744 \ubc1b\uace0 1\uad70 \ub4f1\ub85d\uc774 \ub9d0\uc18c\ub410\ub2e4. \uac00\uc744\ubd80\ud130\ub294 \uadf8\ub54c\uae4c\uc9c0 \uac15\ud558\uac8c \uac70\ubd80\ud558\uace0 \uc788\uc5c8\ub358 \uc6e8\uc774\ud2b8 \ud2b8\ub808\uc774\ub2dd\uc5d0\ub3c4 \uc2a4\uc2a4\ub85c \ucc38\uac00\ud558\uc5ec \ub7f0\ub2dd\uc774\ub098 \ube44\ud0c8\uae38 \ub2ec\ub9ac\uae30\ub85c \ud558\ubc18\uc2e0\uc744 \uac15\ud654\ud558\ub294 \ub4f1 \uc790\uc2e0\uc73c\ub85c\uc11c\ub294 \ucc98\uc74c\uc73c\ub85c 12\uc6d4\uc5d0 \uc5f4\ub9b0 \uc790\uc8fc \ud2b8\ub808\uc774\ub2dd\uc5d0\ub3c4 \ucc38\uac00\ud588\ub2e4.", "paragraph_answer": "\uc774\ub4ec\ud574 1998\ub144\uc5d0\ub294 \uc5b4\uae68 \uc0c1\ud0dc\uac00 \ud68c\ubcf5\ub410\uace0 \uc2a4\ud504\ub9c1 \ucea0\ud504\uc5d0\uc11c \uadfc\ub825 \uac15\ud654\ub098 \uba40\ub9ac \ub358\uc9c0\ub294 \uc5f0\uc2b5\uc744 \uac70\ub4ed\ud588\ub2e4. 5\uc6d4 16\uc77c \uc694\ucf54\ud558\ub9c8 \uc804(\ub098\uace0\uc57c \ub3d4)\uc5d0\uc11c \uc120\ubc1c\ud55c 1\ud68c\uc5d0 1\uc810\uc744 \ub0b4\uc92c\uc9c0\ub9cc \uadf8 \ud6c4\uc5d0\ub294 \uc9c1\uad6c\ub85c \ubc00\uc5b4\ub0b4\uace0 5\uc774\ub2dd 1\uc2e4\uc810\uc758 \uc131\uc801\uc73c\ub85c \uc2dc\uc98c 2\uc2b9\uc9f8\ub97c \uac70\ub480\uc9c0\ub9cc \uacb0\uacfc\uc801\uc73c\ub85c \uc774 \uc2b9\ub9ac\uac00 \ud604\uc5ed \uc0dd\ud65c\uc5d0 \uc788\uc5b4\uc11c\uc758 \ub9c8\uc9c0\ub9c9 \uc2b9\ub9ac\uc600\ub2e4. 6\uc6d4 5\uc77c \ub098\uace0\uc57c \ub3d4\uc5d0\uc11c \uc5f4\ub9b0 \uc694\ubbf8\uc6b0\ub9ac\uc804\uc5d0\uc11c\ub294 \ucd5c\uace0 \uc18d\ub3c4 140 km/h\ub97c \uae30\ub85d\ud558\uc5ec 90 km/h\uc758 \uc2ac\ub85c \ucee4\ube0c\ub3c4 \ub418\ub3cc\uc544\uc640\uc11c 8\uc774\ub2dd \ub3d9\uc548 131\uac1c\uc758 \uacf5\uc744 \ub358\uc838 2\ud53c\uc548\ud0c0 1\uc2e4\uc810\uc758 \ud638\ud22c\ub85c \uc2b9\ub9ac\ub97c \uac70\ub450\ub294 \ub4f1 \ubd80\ud65c\uc758 \uc870\uc9d0\uc744 \ubcf4\uc600\ub2e4. \uadf8\ub7ec\ub098 \ud638\uc870\uac00 \uc9c0\uc18d\ub418\uc9c0 \uc54a\uc558\uace0 \uc790\uc2e0\uc758 4\uc5f0\ud328\ub85c \ub9de\uc774\ud55c 7\uc6d4 9\uc77c \uc57c\ucfe0\ub974\ud2b8\uc804\uc5d0 \uc120\ubc1c\ub85c \ub098\uc11c 2/3\uc774\ub2dd\uc5d0 5\ud53c\uc548\ud0c0 4\uc2e4\uc810\uc73c\ub85c \ud328\uc804 \ud22c\uc218\uac00 \ub418\uba74\uc11c 2\uc2b9 8\ud328\ub97c \uae30\ub85d\ud558\uc790, \ud638\uc2dc\ub178 \uc13c\uc774\uce58 \uac10\ub3c5\uc73c\ub85c\ubd80\ud130 \ub2e4\uc2dc \uae30\uad50\ud30c\ub85c \uc804\ud5a5\ud558\ub77c\ub294 \uba85\ub839\uc744 \ubc1b\uace0 1\uad70 \ub4f1\ub85d\uc774 \ub9d0\uc18c\ub410\ub2e4. \uac00\uc744\ubd80\ud130\ub294 \uadf8\ub54c\uae4c\uc9c0 \uac15\ud558\uac8c \uac70\ubd80\ud558\uace0 \uc788\uc5c8\ub358 \uc6e8\uc774\ud2b8 \ud2b8\ub808\uc774\ub2dd\uc5d0\ub3c4 \uc2a4\uc2a4\ub85c \ucc38\uac00\ud558\uc5ec \ub7f0\ub2dd\uc774\ub098 \ube44\ud0c8\uae38 \ub2ec\ub9ac\uae30\ub85c \ud558\ubc18\uc2e0\uc744 \uac15\ud654\ud558\ub294 \ub4f1 \uc790\uc2e0\uc73c\ub85c\uc11c\ub294 \ucc98\uc74c\uc73c\ub85c 12\uc6d4\uc5d0 \uc5f4\ub9b0 \uc790\uc8fc \ud2b8\ub808\uc774\ub2dd\uc5d0\ub3c4 \ucc38\uac00\ud588\ub2e4.", "sentence_answer": "5\uc6d4 16\uc77c \uc694\ucf54\ud558\ub9c8 \uc804(\ub098\uace0\uc57c \ub3d4)\uc5d0\uc11c \uc120\ubc1c\ud55c 1\ud68c\uc5d0 1\uc810\uc744 \ub0b4\uc92c\uc9c0\ub9cc \uadf8 \ud6c4\uc5d0\ub294 \uc9c1\uad6c\ub85c \ubc00\uc5b4\ub0b4\uace0 5\uc774\ub2dd 1\uc2e4\uc810\uc758 \uc131\uc801\uc73c\ub85c \uc2dc\uc98c 2\uc2b9\uc9f8\ub97c \uac70\ub480\uc9c0\ub9cc \uacb0\uacfc\uc801\uc73c\ub85c \uc774 \uc2b9\ub9ac\uac00 \ud604\uc5ed \uc0dd\ud65c\uc5d0 \uc788\uc5b4\uc11c\uc758 \ub9c8\uc9c0\ub9c9 \uc2b9\ub9ac\uc600\ub2e4.", "paragraph_id": "6545142-9-0", "paragraph_question": "question: 1998\ub144 5\uc6d4 16\uc77c \ucd9c\uc804\ud55c \uacbd\uae30\uc758 \uc0c1\ub300\ud300\uc740?, context: \uc774\ub4ec\ud574 1998\ub144\uc5d0\ub294 \uc5b4\uae68 \uc0c1\ud0dc\uac00 \ud68c\ubcf5\ub410\uace0 \uc2a4\ud504\ub9c1 \ucea0\ud504\uc5d0\uc11c \uadfc\ub825 \uac15\ud654\ub098 \uba40\ub9ac \ub358\uc9c0\ub294 \uc5f0\uc2b5\uc744 \uac70\ub4ed\ud588\ub2e4. 5\uc6d4 16\uc77c \uc694\ucf54\ud558\ub9c8\uc804(\ub098\uace0\uc57c \ub3d4)\uc5d0\uc11c \uc120\ubc1c\ud55c 1\ud68c\uc5d0 1\uc810\uc744 \ub0b4\uc92c\uc9c0\ub9cc \uadf8 \ud6c4\uc5d0\ub294 \uc9c1\uad6c\ub85c \ubc00\uc5b4\ub0b4\uace0 5\uc774\ub2dd 1\uc2e4\uc810\uc758 \uc131\uc801\uc73c\ub85c \uc2dc\uc98c 2\uc2b9\uc9f8\ub97c \uac70\ub480\uc9c0\ub9cc \uacb0\uacfc\uc801\uc73c\ub85c \uc774 \uc2b9\ub9ac\uac00 \ud604\uc5ed \uc0dd\ud65c\uc5d0 \uc788\uc5b4\uc11c\uc758 \ub9c8\uc9c0\ub9c9 \uc2b9\ub9ac\uc600\ub2e4. 6\uc6d4 5\uc77c \ub098\uace0\uc57c \ub3d4\uc5d0\uc11c \uc5f4\ub9b0 \uc694\ubbf8\uc6b0\ub9ac\uc804\uc5d0\uc11c\ub294 \ucd5c\uace0 \uc18d\ub3c4 140 km/h\ub97c \uae30\ub85d\ud558\uc5ec 90 km/h\uc758 \uc2ac\ub85c \ucee4\ube0c\ub3c4 \ub418\ub3cc\uc544\uc640\uc11c 8\uc774\ub2dd \ub3d9\uc548 131\uac1c\uc758 \uacf5\uc744 \ub358\uc838 2\ud53c\uc548\ud0c0 1\uc2e4\uc810\uc758 \ud638\ud22c\ub85c \uc2b9\ub9ac\ub97c \uac70\ub450\ub294 \ub4f1 \ubd80\ud65c\uc758 \uc870\uc9d0\uc744 \ubcf4\uc600\ub2e4. \uadf8\ub7ec\ub098 \ud638\uc870\uac00 \uc9c0\uc18d\ub418\uc9c0 \uc54a\uc558\uace0 \uc790\uc2e0\uc758 4\uc5f0\ud328\ub85c \ub9de\uc774\ud55c 7\uc6d4 9\uc77c \uc57c\ucfe0\ub974\ud2b8\uc804\uc5d0 \uc120\ubc1c\ub85c \ub098\uc11c 2/3\uc774\ub2dd\uc5d0 5\ud53c\uc548\ud0c0 4\uc2e4\uc810\uc73c\ub85c \ud328\uc804 \ud22c\uc218\uac00 \ub418\uba74\uc11c 2\uc2b9 8\ud328\ub97c \uae30\ub85d\ud558\uc790, \ud638\uc2dc\ub178 \uc13c\uc774\uce58 \uac10\ub3c5\uc73c\ub85c\ubd80\ud130 \ub2e4\uc2dc \uae30\uad50\ud30c\ub85c \uc804\ud5a5\ud558\ub77c\ub294 \uba85\ub839\uc744 \ubc1b\uace0 1\uad70 \ub4f1\ub85d\uc774 \ub9d0\uc18c\ub410\ub2e4. \uac00\uc744\ubd80\ud130\ub294 \uadf8\ub54c\uae4c\uc9c0 \uac15\ud558\uac8c \uac70\ubd80\ud558\uace0 \uc788\uc5c8\ub358 \uc6e8\uc774\ud2b8 \ud2b8\ub808\uc774\ub2dd\uc5d0\ub3c4 \uc2a4\uc2a4\ub85c \ucc38\uac00\ud558\uc5ec \ub7f0\ub2dd\uc774\ub098 \ube44\ud0c8\uae38 \ub2ec\ub9ac\uae30\ub85c \ud558\ubc18\uc2e0\uc744 \uac15\ud654\ud558\ub294 \ub4f1 \uc790\uc2e0\uc73c\ub85c\uc11c\ub294 \ucc98\uc74c\uc73c\ub85c 12\uc6d4\uc5d0 \uc5f4\ub9b0 \uc790\uc8fc \ud2b8\ub808\uc774\ub2dd\uc5d0\ub3c4 \ucc38\uac00\ud588\ub2e4."} {"question": "\uc694\uc220\uc744 \ubd80\ub824 \ucca0\uc0ac \ub4f1\uc744 \ubc97\uc5b4 \ub358\uc9c0\uace0 \ub2ec\uc544\ub0ac\ub2e4\ub294 \uc804\uc124\uc774 \ub098\uc624\uac8c \ub41c \uc778\ubb3c\uc740?", "paragraph": "\ub098\uc911\uc5d0 \ub2f9\uc0c8\uc544\ub294 \uccb4\ud3ec\ub418\uc5b4 \ubaa9\uacfc \uc190\ubc1c\uc5d0 \ud615\uad6c\ub97c \uc50c\uc6b0\uace0 \uad75\uc740 \ucca0\uc0ac\ub85c \ubb36\uc5b4 \ub193\uc558\ub294\ub370, \ub9e4\ubcf5\ud55c \ub2f9\uc0c8\uc544\uc758 \uc794\ub2f9\uc774 \uadf8\ub140\ub97c \ud0c8\uc625\uc2dc\ucf30\ub2e4. \uc774\uc5d0 \uc2dc\uc911\uc5d0\ub294 \ub2f9\uc0c8\uc544\uac00 \uc694\uc220(\u5996\u8853)\uc744 \ubd80\ub824 \ubaa8\ub450 \ubc97\uc5b4 \ub358\uc9c0\uace0 \ub2ec\uc544\ub0ac\ub2e4\ub294 \uc804\uc124\uc774 \ub098\uc624\uac8c \ub418\uc5c8\ub2e4. \ud654\uac00 \ub09c \uc601\ub77d\uc81c\ub294 \"\uc0ad\ubc1c\ud558\uace0 \uc911\uc774 \ub418\uc5c8\uac70\ub098 \uc5ec\ub3c4\uc0ac(\u5973\u9053\u58eb) \ubb34\ub9ac\uc5d0 \ub2f9\uc0c8\uc544\uac00 \uc228\uc5b4 \uc788\uc744\uc9c0 \ubaa8\ub974\ub2c8 \uc0b0\ub465\uacfc \ub09c\uc9d5, \ubca0\uc774\uc9d5\uc758 \ube44\uad6c\ub2c8\ub4e4\uacfc \ucd9c\uac00\ud55c \ubd80\ub140\uc790\ub4e4\uc744 \ubaa8\uc870\ub9ac \uc7a1\uc544\ub4e4\uc774\ub77c\"\ub294 \uc870\uce59\uc744 \ub0b4\ub838\ub2e4. \uba85\ub098\ub77c \uac01\uc9c0\uc5d0\uc11c \uc218 \ub9cc\uba85\uc758 \ube44\uad6c\ub2c8 \uc5ec\uc2b9\uacfc \uc5ec\ub3c4\uc0ac\ub4e4\uc774 \uc7a1\ud600\uc654\uc73c\ub098 \uacb0\uad6d \ub2f9\uc0c8\uc544\ub294 \uc7a1\uc9c0 \ubabb\ud588\ub2e4. \uc774\ub294 \uadf8\uc758 \ub9cc\ub144 \uce58\uc138\uc5d0 \ud070 \ud0c0\uaca9\uc744 \uc8fc\uc5c8\uace0, \uc774\ub294 \ubbfc\uc911\ub4e4 \uc0ac\uc774\uc5d0 \ud68c\uc790\ud654\ub418\uc5b4 \ubbfc\ub2f4\uacfc \uc804\uc124\uc758 \uc18c\uc7ac\uac00 \ub418\uc5c8\ub2e4. \ud6c4\uc77c \uccad\ub098\ub77c \ub54c\uc758 \ubc31\ub828\uad50\ub3c4\uc758 \ub09c\uc740 \ub2f9\uc0c8\uc544\uc758 \ub09c\uacfc \ud64d\uac74\uc801\uc758 \ub09c\uc744 \ucc38\uace0\ud558\uc600\uc73c\uba70, \uccad\ub098\ub77c \ub54c\uc758 \uc791\uac00 \uc5ec\ub2a5(\u5442\u80fd)\uc740 \ub2f9\uc0c8\uc544\uc758 \ubc18\ub780\uc744 \uc18c\uc7ac\ub85c \ud558\uc5ec \uc18c\uc124 '\uc5ec\uc120\uc678\uc0ac'(\u5973\u4ed9\u5916\u53f2)\ub97c \uc4f0\uae30\ub3c4 \ud588\ub2e4. \ub2f9\uc0c8\uc544\uc758 \ub09c\uc740 \uc870\uc120\uc5d0\ub3c4 \uc54c\ub824\uc838 \uc778\uc870\uc2e4\ub85d \ub4f1\uc5d0\ub3c4 \uace0\uc0ac\ub85c \uc18c\uac1c\ub418\uae30\ub3c4 \ud558\uc600\ub2e4.", "answer": "\ub2f9\uc0c8\uc544", "sentence": "\ub098\uc911\uc5d0 \ub2f9\uc0c8\uc544 \ub294 \uccb4\ud3ec\ub418\uc5b4 \ubaa9\uacfc \uc190\ubc1c\uc5d0 \ud615\uad6c\ub97c \uc50c\uc6b0\uace0 \uad75\uc740 \ucca0\uc0ac\ub85c \ubb36\uc5b4 \ub193\uc558\ub294\ub370, \ub9e4\ubcf5\ud55c \ub2f9\uc0c8\uc544\uc758 \uc794\ub2f9\uc774 \uadf8\ub140\ub97c \ud0c8\uc625\uc2dc\ucf30\ub2e4.", "paragraph_sentence": " \ub098\uc911\uc5d0 \ub2f9\uc0c8\uc544 \ub294 \uccb4\ud3ec\ub418\uc5b4 \ubaa9\uacfc \uc190\ubc1c\uc5d0 \ud615\uad6c\ub97c \uc50c\uc6b0\uace0 \uad75\uc740 \ucca0\uc0ac\ub85c \ubb36\uc5b4 \ub193\uc558\ub294\ub370, \ub9e4\ubcf5\ud55c \ub2f9\uc0c8\uc544\uc758 \uc794\ub2f9\uc774 \uadf8\ub140\ub97c \ud0c8\uc625\uc2dc\ucf30\ub2e4. \uc774\uc5d0 \uc2dc\uc911\uc5d0\ub294 \ub2f9\uc0c8\uc544\uac00 \uc694\uc220(\u5996\u8853)\uc744 \ubd80\ub824 \ubaa8\ub450 \ubc97\uc5b4 \ub358\uc9c0\uace0 \ub2ec\uc544\ub0ac\ub2e4\ub294 \uc804\uc124\uc774 \ub098\uc624\uac8c \ub418\uc5c8\ub2e4. \ud654\uac00 \ub09c \uc601\ub77d\uc81c\ub294 \"\uc0ad\ubc1c\ud558\uace0 \uc911\uc774 \ub418\uc5c8\uac70\ub098 \uc5ec\ub3c4\uc0ac(\u5973\u9053\u58eb) \ubb34\ub9ac\uc5d0 \ub2f9\uc0c8\uc544\uac00 \uc228\uc5b4 \uc788\uc744\uc9c0 \ubaa8\ub974\ub2c8 \uc0b0\ub465\uacfc \ub09c\uc9d5, \ubca0\uc774\uc9d5\uc758 \ube44\uad6c\ub2c8\ub4e4\uacfc \ucd9c\uac00\ud55c \ubd80\ub140\uc790\ub4e4\uc744 \ubaa8\uc870\ub9ac \uc7a1\uc544\ub4e4\uc774\ub77c\"\ub294 \uc870\uce59\uc744 \ub0b4\ub838\ub2e4. \uba85\ub098\ub77c \uac01\uc9c0\uc5d0\uc11c \uc218 \ub9cc\uba85\uc758 \ube44\uad6c\ub2c8 \uc5ec\uc2b9\uacfc \uc5ec\ub3c4\uc0ac\ub4e4\uc774 \uc7a1\ud600\uc654\uc73c\ub098 \uacb0\uad6d \ub2f9\uc0c8\uc544\ub294 \uc7a1\uc9c0 \ubabb\ud588\ub2e4. \uc774\ub294 \uadf8\uc758 \ub9cc\ub144 \uce58\uc138\uc5d0 \ud070 \ud0c0\uaca9\uc744 \uc8fc\uc5c8\uace0, \uc774\ub294 \ubbfc\uc911\ub4e4 \uc0ac\uc774\uc5d0 \ud68c\uc790\ud654\ub418\uc5b4 \ubbfc\ub2f4\uacfc \uc804\uc124\uc758 \uc18c\uc7ac\uac00 \ub418\uc5c8\ub2e4. \ud6c4\uc77c \uccad\ub098\ub77c \ub54c\uc758 \ubc31\ub828\uad50\ub3c4\uc758 \ub09c\uc740 \ub2f9\uc0c8\uc544\uc758 \ub09c\uacfc \ud64d\uac74\uc801\uc758 \ub09c\uc744 \ucc38\uace0\ud558\uc600\uc73c\uba70, \uccad\ub098\ub77c \ub54c\uc758 \uc791\uac00 \uc5ec\ub2a5(\u5442\u80fd)\uc740 \ub2f9\uc0c8\uc544\uc758 \ubc18\ub780\uc744 \uc18c\uc7ac\ub85c \ud558\uc5ec \uc18c\uc124 '\uc5ec\uc120\uc678\uc0ac'(\u5973\u4ed9\u5916\u53f2)\ub97c \uc4f0\uae30\ub3c4 \ud588\ub2e4. \ub2f9\uc0c8\uc544\uc758 \ub09c\uc740 \uc870\uc120\uc5d0\ub3c4 \uc54c\ub824\uc838 \uc778\uc870\uc2e4\ub85d \ub4f1\uc5d0\ub3c4 \uace0\uc0ac\ub85c \uc18c\uac1c\ub418\uae30\ub3c4 \ud558\uc600\ub2e4.", "paragraph_answer": "\ub098\uc911\uc5d0 \ub2f9\uc0c8\uc544 \ub294 \uccb4\ud3ec\ub418\uc5b4 \ubaa9\uacfc \uc190\ubc1c\uc5d0 \ud615\uad6c\ub97c \uc50c\uc6b0\uace0 \uad75\uc740 \ucca0\uc0ac\ub85c \ubb36\uc5b4 \ub193\uc558\ub294\ub370, \ub9e4\ubcf5\ud55c \ub2f9\uc0c8\uc544\uc758 \uc794\ub2f9\uc774 \uadf8\ub140\ub97c \ud0c8\uc625\uc2dc\ucf30\ub2e4. \uc774\uc5d0 \uc2dc\uc911\uc5d0\ub294 \ub2f9\uc0c8\uc544\uac00 \uc694\uc220(\u5996\u8853)\uc744 \ubd80\ub824 \ubaa8\ub450 \ubc97\uc5b4 \ub358\uc9c0\uace0 \ub2ec\uc544\ub0ac\ub2e4\ub294 \uc804\uc124\uc774 \ub098\uc624\uac8c \ub418\uc5c8\ub2e4. \ud654\uac00 \ub09c \uc601\ub77d\uc81c\ub294 \"\uc0ad\ubc1c\ud558\uace0 \uc911\uc774 \ub418\uc5c8\uac70\ub098 \uc5ec\ub3c4\uc0ac(\u5973\u9053\u58eb) \ubb34\ub9ac\uc5d0 \ub2f9\uc0c8\uc544\uac00 \uc228\uc5b4 \uc788\uc744\uc9c0 \ubaa8\ub974\ub2c8 \uc0b0\ub465\uacfc \ub09c\uc9d5, \ubca0\uc774\uc9d5\uc758 \ube44\uad6c\ub2c8\ub4e4\uacfc \ucd9c\uac00\ud55c \ubd80\ub140\uc790\ub4e4\uc744 \ubaa8\uc870\ub9ac \uc7a1\uc544\ub4e4\uc774\ub77c\"\ub294 \uc870\uce59\uc744 \ub0b4\ub838\ub2e4. \uba85\ub098\ub77c \uac01\uc9c0\uc5d0\uc11c \uc218 \ub9cc\uba85\uc758 \ube44\uad6c\ub2c8 \uc5ec\uc2b9\uacfc \uc5ec\ub3c4\uc0ac\ub4e4\uc774 \uc7a1\ud600\uc654\uc73c\ub098 \uacb0\uad6d \ub2f9\uc0c8\uc544\ub294 \uc7a1\uc9c0 \ubabb\ud588\ub2e4. \uc774\ub294 \uadf8\uc758 \ub9cc\ub144 \uce58\uc138\uc5d0 \ud070 \ud0c0\uaca9\uc744 \uc8fc\uc5c8\uace0, \uc774\ub294 \ubbfc\uc911\ub4e4 \uc0ac\uc774\uc5d0 \ud68c\uc790\ud654\ub418\uc5b4 \ubbfc\ub2f4\uacfc \uc804\uc124\uc758 \uc18c\uc7ac\uac00 \ub418\uc5c8\ub2e4. \ud6c4\uc77c \uccad\ub098\ub77c \ub54c\uc758 \ubc31\ub828\uad50\ub3c4\uc758 \ub09c\uc740 \ub2f9\uc0c8\uc544\uc758 \ub09c\uacfc \ud64d\uac74\uc801\uc758 \ub09c\uc744 \ucc38\uace0\ud558\uc600\uc73c\uba70, \uccad\ub098\ub77c \ub54c\uc758 \uc791\uac00 \uc5ec\ub2a5(\u5442\u80fd)\uc740 \ub2f9\uc0c8\uc544\uc758 \ubc18\ub780\uc744 \uc18c\uc7ac\ub85c \ud558\uc5ec \uc18c\uc124 '\uc5ec\uc120\uc678\uc0ac'(\u5973\u4ed9\u5916\u53f2)\ub97c \uc4f0\uae30\ub3c4 \ud588\ub2e4. \ub2f9\uc0c8\uc544\uc758 \ub09c\uc740 \uc870\uc120\uc5d0\ub3c4 \uc54c\ub824\uc838 \uc778\uc870\uc2e4\ub85d \ub4f1\uc5d0\ub3c4 \uace0\uc0ac\ub85c \uc18c\uac1c\ub418\uae30\ub3c4 \ud558\uc600\ub2e4.", "sentence_answer": "\ub098\uc911\uc5d0 \ub2f9\uc0c8\uc544 \ub294 \uccb4\ud3ec\ub418\uc5b4 \ubaa9\uacfc \uc190\ubc1c\uc5d0 \ud615\uad6c\ub97c \uc50c\uc6b0\uace0 \uad75\uc740 \ucca0\uc0ac\ub85c \ubb36\uc5b4 \ub193\uc558\ub294\ub370, \ub9e4\ubcf5\ud55c \ub2f9\uc0c8\uc544\uc758 \uc794\ub2f9\uc774 \uadf8\ub140\ub97c \ud0c8\uc625\uc2dc\ucf30\ub2e4.", "paragraph_id": "6524306-5-0", "paragraph_question": "question: \uc694\uc220\uc744 \ubd80\ub824 \ucca0\uc0ac \ub4f1\uc744 \ubc97\uc5b4 \ub358\uc9c0\uace0 \ub2ec\uc544\ub0ac\ub2e4\ub294 \uc804\uc124\uc774 \ub098\uc624\uac8c \ub41c \uc778\ubb3c\uc740?, context: \ub098\uc911\uc5d0 \ub2f9\uc0c8\uc544\ub294 \uccb4\ud3ec\ub418\uc5b4 \ubaa9\uacfc \uc190\ubc1c\uc5d0 \ud615\uad6c\ub97c \uc50c\uc6b0\uace0 \uad75\uc740 \ucca0\uc0ac\ub85c \ubb36\uc5b4 \ub193\uc558\ub294\ub370, \ub9e4\ubcf5\ud55c \ub2f9\uc0c8\uc544\uc758 \uc794\ub2f9\uc774 \uadf8\ub140\ub97c \ud0c8\uc625\uc2dc\ucf30\ub2e4. \uc774\uc5d0 \uc2dc\uc911\uc5d0\ub294 \ub2f9\uc0c8\uc544\uac00 \uc694\uc220(\u5996\u8853)\uc744 \ubd80\ub824 \ubaa8\ub450 \ubc97\uc5b4 \ub358\uc9c0\uace0 \ub2ec\uc544\ub0ac\ub2e4\ub294 \uc804\uc124\uc774 \ub098\uc624\uac8c \ub418\uc5c8\ub2e4. \ud654\uac00 \ub09c \uc601\ub77d\uc81c\ub294 \"\uc0ad\ubc1c\ud558\uace0 \uc911\uc774 \ub418\uc5c8\uac70\ub098 \uc5ec\ub3c4\uc0ac(\u5973\u9053\u58eb) \ubb34\ub9ac\uc5d0 \ub2f9\uc0c8\uc544\uac00 \uc228\uc5b4 \uc788\uc744\uc9c0 \ubaa8\ub974\ub2c8 \uc0b0\ub465\uacfc \ub09c\uc9d5, \ubca0\uc774\uc9d5\uc758 \ube44\uad6c\ub2c8\ub4e4\uacfc \ucd9c\uac00\ud55c \ubd80\ub140\uc790\ub4e4\uc744 \ubaa8\uc870\ub9ac \uc7a1\uc544\ub4e4\uc774\ub77c\"\ub294 \uc870\uce59\uc744 \ub0b4\ub838\ub2e4. \uba85\ub098\ub77c \uac01\uc9c0\uc5d0\uc11c \uc218 \ub9cc\uba85\uc758 \ube44\uad6c\ub2c8 \uc5ec\uc2b9\uacfc \uc5ec\ub3c4\uc0ac\ub4e4\uc774 \uc7a1\ud600\uc654\uc73c\ub098 \uacb0\uad6d \ub2f9\uc0c8\uc544\ub294 \uc7a1\uc9c0 \ubabb\ud588\ub2e4. \uc774\ub294 \uadf8\uc758 \ub9cc\ub144 \uce58\uc138\uc5d0 \ud070 \ud0c0\uaca9\uc744 \uc8fc\uc5c8\uace0, \uc774\ub294 \ubbfc\uc911\ub4e4 \uc0ac\uc774\uc5d0 \ud68c\uc790\ud654\ub418\uc5b4 \ubbfc\ub2f4\uacfc \uc804\uc124\uc758 \uc18c\uc7ac\uac00 \ub418\uc5c8\ub2e4. \ud6c4\uc77c \uccad\ub098\ub77c \ub54c\uc758 \ubc31\ub828\uad50\ub3c4\uc758 \ub09c\uc740 \ub2f9\uc0c8\uc544\uc758 \ub09c\uacfc \ud64d\uac74\uc801\uc758 \ub09c\uc744 \ucc38\uace0\ud558\uc600\uc73c\uba70, \uccad\ub098\ub77c \ub54c\uc758 \uc791\uac00 \uc5ec\ub2a5(\u5442\u80fd)\uc740 \ub2f9\uc0c8\uc544\uc758 \ubc18\ub780\uc744 \uc18c\uc7ac\ub85c \ud558\uc5ec \uc18c\uc124 '\uc5ec\uc120\uc678\uc0ac'(\u5973\u4ed9\u5916\u53f2)\ub97c \uc4f0\uae30\ub3c4 \ud588\ub2e4. \ub2f9\uc0c8\uc544\uc758 \ub09c\uc740 \uc870\uc120\uc5d0\ub3c4 \uc54c\ub824\uc838 \uc778\uc870\uc2e4\ub85d \ub4f1\uc5d0\ub3c4 \uace0\uc0ac\ub85c \uc18c\uac1c\ub418\uae30\ub3c4 \ud558\uc600\ub2e4."} {"question": "\uae34\uae09\uc218\uc785\uc81c\ud55c\uc870\uce58\ub85c \ud2b9\uc815\ud488\ubaa9\uc758 \uc218\uc785\uc774 \uae09\uc99d\ud558\uc5ec \uad6d\ub0b4 \uc5c5\uacc4\uc5d0 \uc911\ub300\ud55c \uc190\uc2e4\uc744 \uc785\ud788\uac70\ub098 \uadf8 \uc6b0\ub824\uac00 \uc788\uc744 \ub54c \ubc1c\ub3d9\ud558\ub294 \uac83\uc740 \ubb34\uc5c7\uc778\uac00?", "paragraph": "\ubbf8\uad6d\uacfc\uc758 \ud1b5\uc0c1 \ub9c8\ucc30\uc740 2018\ub144\ubd80\ud130 \ubcf8\uaca9\ud654\ub410\ub2e4. 2\uc6d4 12\uc77c \ud2b8\ub7fc\ud504\ub294 \"\uc6b0\ub9ac\ub294 \ud55c\uad6d, \uc911\uad6d, \uc77c\ubcf8\uc5d0 \uc5b4\ub9c8\uc5b4\ub9c8\ud55c \ub3c8\uc744 \uc783\uc5c8\ub2e4\"\uba70 \"\uadf8\ub4e4\uc740 25\ub144\uc9f8 \uc81c\uba4b\ub300\ub85c \ud589\ub3d9\ud558\uace0 \uc788\ub2e4\"\uace0 \ube44\ub09c\ud588\ub2e4. \ud2b9\ud788 \ub3d9\ub9f9\uad6d\uc778 \ud55c\uad6d\uacfc \uc77c\ubcf8\uc744 \uaca8\ub0e5\ud574 \"\uc77c\ubd80\ub294 \uc18c\uc704 \ub3d9\ub9f9\uad6d\uc774\uc9c0\ub9cc, \ubb34\uc5ed \uba74\uc5d0\uc11c\ub294 \ub3d9\ub9f9\uad6d\uc774 \uc544\ub2c8\ub2e4\"\ub77c\uba70 \ud638\ud61c\uc138\ub97c \ub3c4\uc785\ud558\uaca0\ub2e4\ub294 \ubc29\uce68\uc744 \ubc1d\ud614\ub2e4. \ud55c\uad6d\uc5d0 \ub300\ud574\uc11c\ub3c4 '\ud55c\uad6d \uc804\uc7c1 \uc9c1\ud6c4 \ud55c\uad6d\uc744 \ub3c4\uc654\uace0 \uadf8\ub4e4\uc740 \ubd80\uc790\uac00 \ub410\uc9c0\ub9cc (\uc6b0\ub9ac\uc5d0\uac8c \ub3c8\uc744 \ub3cc\ub824\uc904 \uc218 \uc788\uc5c8\uc74c\uc5d0\ub3c4) \uc544\ubb34 \uc77c\ub3c4 \uc5c6\uc5c8\ub2e4'\uba70 \ud638\ud61c\uc138 \ubd80\uacfc\uc758 \uc815\ub2f9\uc131\uc744 \ud638\uc18c\ud588\ub2e4. 18\uc77c\uc5d0\ub294 \ubbf8\uad6d \uc0c1\ubb34\ubd80\uac00 \u300c\ubb34\uc5ed\ud655\uc7a5\ubc95\u300d \uc81c232\uc870 \uc870\uc0ac\ubcf4\uace0\uc11c\ub97c \ubc1c\ud45c\ud588\ub294\ub370 \ud55c\uad6d\uacfc \uc911\uad6d \ub4f1 12\uac1c \uad6d\uac00\uc5d0 \ub300\ud574 53%\uc758 \ub192\uc740 \uad00\uc138\ub97c \ubd80\uacfc\ud560 \ub300\uc0c1\uc73c\ub85c \uc9c0\ubaa9\ud588\ub2e4. \ud55c\uad6d \uc815\ubd80\ub294 \ud55c\uad6d\uc774 \ubbf8\uad6d\uc758 \ub3d9\ub9f9\uc774\uae30 \ub54c\ubb38\uc5d0 \ud55c\uad6d\uc0b0 \ucca0\uac15\uc774 \ubbf8\uad6d \uc548\ubcf4\uc5d0 \uc704\ud611\uc774 \ub418\uc9c0 \uc54a\ub294\ub2e4\ub294 \uc810\uc744 \uac15\uc870\ud588\uc9c0\ub9cc \uc124\ub4dd\ub418\uc9c0 \ubabb\ud588\uc73c\uba70, \ub9ce\uc740 \ucca0\uac15\uc744 \uc218\ucd9c\ud558\ub294 \uce90\ub098\ub2e4\u00b7\uc77c\ubcf8\u00b7\ub3c5\uc77c \ub4f1 \uc804\ud1b5\uc801\uc778 \uc6b0\ubc29\uc740 \uace0\uad00\uc138 \ubd80\uacfc \ub300\uc0c1\uc5d0 \ud3ec\ud568\ub418\uc9c0 \uc54a\uc544 \ub17c\ub780\uc744 \uc77c\uc73c\ucf30\ub2e4. \uc0b0\uc5c5\ubd80\ub294 \"\ub300\ubbf8 \uc218\ucd9c\uc774 \ub9ce\uc73c\uba74\uc11c \uc911\uad6d\uc0b0 \ucca0\uac15\uc744 \ub9ce\uc774 \uc218\uc785\ud558\ub294 \uad6d\uac00\ub4e4\uc774 \ud3ec\ud568\ub41c \uac83 \uac19\ub2e4\"\uace0 \ud588\uc73c\uba70 \ubbf8\uad6d \uad6d\ubc29\ubd80\ub3c4 \"\uc774\ub7f0 \uc870\uce58\uac00 \uc911\uad6d\uc758 \uc0dd\uc0b0 \uacfc\uc789\uc744 \ubc14\ub85c\uc7a1\uace0 \uae30\uc874 \ubc18\ub364\ud551 \uad00\uc138\ub97c \uc6b0\ud68c\ud558\ub824\ub294 \uc911\uad6d\uc758 \uc2dc\ub3c4\ub97c \ub9c9\ub294 \ub370 \ucd08\uc810\uc744 \ub9de\ucd98 \uac83\uc774\uc9c0 \ubbf8\uad6d\uacfc\uc758 \uc591\uc790 \uad00\uacc4\uc5d0 \ub9de\ucd98 \uac8c \uc544\ub2c8\ub77c\ub294 \uc810\uc744 \uc6b0\ub9ac\uc758 \ud575\uc2ec \ub3d9\ub9f9\uad6d\ub4e4\uc5d0\uac8c \uac15\uc870\ud558\ub294 \uac8c \ub9e4\uc6b0 \uc911\uc694\ud558\ub2e4\"\uace0 \ubc1d\ud788\uae30\ub3c4 \ud588\ub2e4. \uc138\uc774\ud504\uac00\ub4dc\ub3c4 \ubb38\uc81c\uac00 \ub418\uc5c8\ub2e4. \uc138\uc774\ud504\uac00\ub4dc\ub294 \ud2b9\uc815\ud488\ubaa9\uc758 \uc218\uc785\uc774 \uae09\uc99d\ud558\uc5ec \uad6d\ub0b4 \uc5c5\uacc4\uc5d0 \uc911\ub300\ud55c \uc190\uc2e4\uc774 \ubc1c\uc0dd\ud558\uac70\ub098 \uadf8 \uc6b0\ub824\uac00 \uc788\uc744 \uacbd\uc6b0 \ubc1c\ub3d9\ud558\ub294 \uae34\uae09\uc218\uc785\uc81c\ud55c\uc870\uce58\ub97c \ub9d0\ud558\ub294\ub370 \ubbf8\uad6d\uc774 \ud0dc\uc591\uad11\u00b7\uc138\ud0c1\uae30\uc5d0 \ub300\ud574 \ubb34\uc5ed \uc801\uc790\uc640 \ubbf8\uad6d\ub0b4 \uc81c\uc870\uc5c5\uc790\ub4e4\uc744 \ubcf4\ud638\ud558\uae30 \uc704\ud574 1\uc6d4 22\uc77c \uc138\uc774\ud504\uac00\ub4dc\ub97c \ubc1c\ub3d9\ud55c \uac83\uc774\ub2e4. \uc774\uc5d0 \ud55c\uad6d\uc740 \uc138\uacc4\ubb34\uc5ed\uae30\uad6c \ubd84\uc7c1\ud574\uacb0\uae30\uad6c \uc815\ub840 \ud68c\uc758\uc5d0\uc11c \uc138\ud0c1\uae30 \ubd84\uc7c1\uacfc \uad00\ub828, \ubbf8\uad6d\uc5d0 \ub300\ud55c \uc591\ud5c8\uad00\uc138 \uc815\uc9c0 \uc2b9\uc778\uc744 \uc694\uccad\ud588\uc73c\uba70 \ucca0\uac15\u00b7\ubcc0\uc555\uae30\uc5d0 \uc9d5\ubc8c\uc801 \uad00\uc138\ub97c \ubd80\uacfc\ud55c \uac83 \uc5ed\uc2dc WTO\uc5d0 \uc81c\uc18c\ud558\uae30\ub85c \ud588\ub2e4. \ud3c9\ucc3d \uc62c\ub9bc\ud53d \ub54c, \ud55c\uad6d\uc744 \ucc3e\uc740 \ub9c8\uc774\ud06c \ud39c\uc2a4 \ubbf8\uad6d \ubd80\ud1b5\ub839\uacfc \uc811\uacac\ud560 \ub54c\uc5d0\ub294 \ud55c\uad6d \uc138\ud0c1\uae30\uc5d0 \ub300\ud55c \uc138\uc774\ud504\uac00\ub4dc \uc870\uce58\ub97c \ud480\uc5b4\uc904 \uac83\uc744 \uc694\uccad\ud588\uc73c\uba70, \ud39c\uc2a4\ub294 \"\ubb38 \ub300\ud1b5\ub839\uaed8\uc11c \uc0c1\ub2f9\ud788 \uc26c\uc6b4 \ubb38\uc81c\ub97c \ub358\uc838\uc8fc\uc168\ub2e4\"\uba74\uc11c\ub3c4 \uc989\ub2f5\uc744 \ub0b4\ub193\uc9c0\ub294 \uc54a\uc558\ub2e4.", "answer": "\uc138\uc774\ud504\uac00\ub4dc", "sentence": "\uc138\uc774\ud504\uac00\ub4dc \ub3c4 \ubb38\uc81c\uac00 \ub418\uc5c8\ub2e4.", "paragraph_sentence": "\ubbf8\uad6d\uacfc\uc758 \ud1b5\uc0c1 \ub9c8\ucc30\uc740 2018\ub144\ubd80\ud130 \ubcf8\uaca9\ud654\ub410\ub2e4. 2\uc6d4 12\uc77c \ud2b8\ub7fc\ud504\ub294 \"\uc6b0\ub9ac\ub294 \ud55c\uad6d, \uc911\uad6d, \uc77c\ubcf8\uc5d0 \uc5b4\ub9c8\uc5b4\ub9c8\ud55c \ub3c8\uc744 \uc783\uc5c8\ub2e4\"\uba70 \"\uadf8\ub4e4\uc740 25\ub144\uc9f8 \uc81c\uba4b\ub300\ub85c \ud589\ub3d9\ud558\uace0 \uc788\ub2e4\"\uace0 \ube44\ub09c\ud588\ub2e4. \ud2b9\ud788 \ub3d9\ub9f9\uad6d\uc778 \ud55c\uad6d\uacfc \uc77c\ubcf8\uc744 \uaca8\ub0e5\ud574 \"\uc77c\ubd80\ub294 \uc18c\uc704 \ub3d9\ub9f9\uad6d\uc774\uc9c0\ub9cc, \ubb34\uc5ed \uba74\uc5d0\uc11c\ub294 \ub3d9\ub9f9\uad6d\uc774 \uc544\ub2c8\ub2e4\"\ub77c\uba70 \ud638\ud61c\uc138\ub97c \ub3c4\uc785\ud558\uaca0\ub2e4\ub294 \ubc29\uce68\uc744 \ubc1d\ud614\ub2e4. \ud55c\uad6d\uc5d0 \ub300\ud574\uc11c\ub3c4 '\ud55c\uad6d \uc804\uc7c1 \uc9c1\ud6c4 \ud55c\uad6d\uc744 \ub3c4\uc654\uace0 \uadf8\ub4e4\uc740 \ubd80\uc790\uac00 \ub410\uc9c0\ub9cc (\uc6b0\ub9ac\uc5d0\uac8c \ub3c8\uc744 \ub3cc\ub824\uc904 \uc218 \uc788\uc5c8\uc74c\uc5d0\ub3c4) \uc544\ubb34 \uc77c\ub3c4 \uc5c6\uc5c8\ub2e4'\uba70 \ud638\ud61c\uc138 \ubd80\uacfc\uc758 \uc815\ub2f9\uc131\uc744 \ud638\uc18c\ud588\ub2e4. 18\uc77c\uc5d0\ub294 \ubbf8\uad6d \uc0c1\ubb34\ubd80\uac00 \u300c\ubb34\uc5ed\ud655\uc7a5\ubc95\u300d \uc81c232\uc870 \uc870\uc0ac\ubcf4\uace0\uc11c\ub97c \ubc1c\ud45c\ud588\ub294\ub370 \ud55c\uad6d\uacfc \uc911\uad6d \ub4f1 12\uac1c \uad6d\uac00\uc5d0 \ub300\ud574 53%\uc758 \ub192\uc740 \uad00\uc138\ub97c \ubd80\uacfc\ud560 \ub300\uc0c1\uc73c\ub85c \uc9c0\ubaa9\ud588\ub2e4. \ud55c\uad6d \uc815\ubd80\ub294 \ud55c\uad6d\uc774 \ubbf8\uad6d\uc758 \ub3d9\ub9f9\uc774\uae30 \ub54c\ubb38\uc5d0 \ud55c\uad6d\uc0b0 \ucca0\uac15\uc774 \ubbf8\uad6d \uc548\ubcf4\uc5d0 \uc704\ud611\uc774 \ub418\uc9c0 \uc54a\ub294\ub2e4\ub294 \uc810\uc744 \uac15\uc870\ud588\uc9c0\ub9cc \uc124\ub4dd\ub418\uc9c0 \ubabb\ud588\uc73c\uba70, \ub9ce\uc740 \ucca0\uac15\uc744 \uc218\ucd9c\ud558\ub294 \uce90\ub098\ub2e4\u00b7\uc77c\ubcf8\u00b7\ub3c5\uc77c \ub4f1 \uc804\ud1b5\uc801\uc778 \uc6b0\ubc29\uc740 \uace0\uad00\uc138 \ubd80\uacfc \ub300\uc0c1\uc5d0 \ud3ec\ud568\ub418\uc9c0 \uc54a\uc544 \ub17c\ub780\uc744 \uc77c\uc73c\ucf30\ub2e4. \uc0b0\uc5c5\ubd80\ub294 \"\ub300\ubbf8 \uc218\ucd9c\uc774 \ub9ce\uc73c\uba74\uc11c \uc911\uad6d\uc0b0 \ucca0\uac15\uc744 \ub9ce\uc774 \uc218\uc785\ud558\ub294 \uad6d\uac00\ub4e4\uc774 \ud3ec\ud568\ub41c \uac83 \uac19\ub2e4\"\uace0 \ud588\uc73c\uba70 \ubbf8\uad6d \uad6d\ubc29\ubd80\ub3c4 \"\uc774\ub7f0 \uc870\uce58\uac00 \uc911\uad6d\uc758 \uc0dd\uc0b0 \uacfc\uc789\uc744 \ubc14\ub85c\uc7a1\uace0 \uae30\uc874 \ubc18\ub364\ud551 \uad00\uc138\ub97c \uc6b0\ud68c\ud558\ub824\ub294 \uc911\uad6d\uc758 \uc2dc\ub3c4\ub97c \ub9c9\ub294 \ub370 \ucd08\uc810\uc744 \ub9de\ucd98 \uac83\uc774\uc9c0 \ubbf8\uad6d\uacfc\uc758 \uc591\uc790 \uad00\uacc4\uc5d0 \ub9de\ucd98 \uac8c \uc544\ub2c8\ub77c\ub294 \uc810\uc744 \uc6b0\ub9ac\uc758 \ud575\uc2ec \ub3d9\ub9f9\uad6d\ub4e4\uc5d0\uac8c \uac15\uc870\ud558\ub294 \uac8c \ub9e4\uc6b0 \uc911\uc694\ud558\ub2e4\"\uace0 \ubc1d\ud788\uae30\ub3c4 \ud588\ub2e4. \uc138\uc774\ud504\uac00\ub4dc \ub3c4 \ubb38\uc81c\uac00 \ub418\uc5c8\ub2e4. \uc138\uc774\ud504\uac00\ub4dc\ub294 \ud2b9\uc815\ud488\ubaa9\uc758 \uc218\uc785\uc774 \uae09\uc99d\ud558\uc5ec \uad6d\ub0b4 \uc5c5\uacc4\uc5d0 \uc911\ub300\ud55c \uc190\uc2e4\uc774 \ubc1c\uc0dd\ud558\uac70\ub098 \uadf8 \uc6b0\ub824\uac00 \uc788\uc744 \uacbd\uc6b0 \ubc1c\ub3d9\ud558\ub294 \uae34\uae09\uc218\uc785\uc81c\ud55c\uc870\uce58\ub97c \ub9d0\ud558\ub294\ub370 \ubbf8\uad6d\uc774 \ud0dc\uc591\uad11\u00b7\uc138\ud0c1\uae30\uc5d0 \ub300\ud574 \ubb34\uc5ed \uc801\uc790\uc640 \ubbf8\uad6d\ub0b4 \uc81c\uc870\uc5c5\uc790\ub4e4\uc744 \ubcf4\ud638\ud558\uae30 \uc704\ud574 1\uc6d4 22\uc77c \uc138\uc774\ud504\uac00\ub4dc\ub97c \ubc1c\ub3d9\ud55c \uac83\uc774\ub2e4. \uc774\uc5d0 \ud55c\uad6d\uc740 \uc138\uacc4\ubb34\uc5ed\uae30\uad6c \ubd84\uc7c1\ud574\uacb0\uae30\uad6c \uc815\ub840 \ud68c\uc758\uc5d0\uc11c \uc138\ud0c1\uae30 \ubd84\uc7c1\uacfc \uad00\ub828, \ubbf8\uad6d\uc5d0 \ub300\ud55c \uc591\ud5c8\uad00\uc138 \uc815\uc9c0 \uc2b9\uc778\uc744 \uc694\uccad\ud588\uc73c\uba70 \ucca0\uac15\u00b7\ubcc0\uc555\uae30\uc5d0 \uc9d5\ubc8c\uc801 \uad00\uc138\ub97c \ubd80\uacfc\ud55c \uac83 \uc5ed\uc2dc WTO\uc5d0 \uc81c\uc18c\ud558\uae30\ub85c \ud588\ub2e4. \ud3c9\ucc3d \uc62c\ub9bc\ud53d \ub54c, \ud55c\uad6d\uc744 \ucc3e\uc740 \ub9c8\uc774\ud06c \ud39c\uc2a4 \ubbf8\uad6d \ubd80\ud1b5\ub839\uacfc \uc811\uacac\ud560 \ub54c\uc5d0\ub294 \ud55c\uad6d \uc138\ud0c1\uae30\uc5d0 \ub300\ud55c \uc138\uc774\ud504\uac00\ub4dc \uc870\uce58\ub97c \ud480\uc5b4\uc904 \uac83\uc744 \uc694\uccad\ud588\uc73c\uba70, \ud39c\uc2a4\ub294 \"\ubb38 \ub300\ud1b5\ub839\uaed8\uc11c \uc0c1\ub2f9\ud788 \uc26c\uc6b4 \ubb38\uc81c\ub97c \ub358\uc838\uc8fc\uc168\ub2e4\"\uba74\uc11c\ub3c4 \uc989\ub2f5\uc744 \ub0b4\ub193\uc9c0\ub294 \uc54a\uc558\ub2e4.", "paragraph_answer": "\ubbf8\uad6d\uacfc\uc758 \ud1b5\uc0c1 \ub9c8\ucc30\uc740 2018\ub144\ubd80\ud130 \ubcf8\uaca9\ud654\ub410\ub2e4. 2\uc6d4 12\uc77c \ud2b8\ub7fc\ud504\ub294 \"\uc6b0\ub9ac\ub294 \ud55c\uad6d, \uc911\uad6d, \uc77c\ubcf8\uc5d0 \uc5b4\ub9c8\uc5b4\ub9c8\ud55c \ub3c8\uc744 \uc783\uc5c8\ub2e4\"\uba70 \"\uadf8\ub4e4\uc740 25\ub144\uc9f8 \uc81c\uba4b\ub300\ub85c \ud589\ub3d9\ud558\uace0 \uc788\ub2e4\"\uace0 \ube44\ub09c\ud588\ub2e4. \ud2b9\ud788 \ub3d9\ub9f9\uad6d\uc778 \ud55c\uad6d\uacfc \uc77c\ubcf8\uc744 \uaca8\ub0e5\ud574 \"\uc77c\ubd80\ub294 \uc18c\uc704 \ub3d9\ub9f9\uad6d\uc774\uc9c0\ub9cc, \ubb34\uc5ed \uba74\uc5d0\uc11c\ub294 \ub3d9\ub9f9\uad6d\uc774 \uc544\ub2c8\ub2e4\"\ub77c\uba70 \ud638\ud61c\uc138\ub97c \ub3c4\uc785\ud558\uaca0\ub2e4\ub294 \ubc29\uce68\uc744 \ubc1d\ud614\ub2e4. \ud55c\uad6d\uc5d0 \ub300\ud574\uc11c\ub3c4 '\ud55c\uad6d \uc804\uc7c1 \uc9c1\ud6c4 \ud55c\uad6d\uc744 \ub3c4\uc654\uace0 \uadf8\ub4e4\uc740 \ubd80\uc790\uac00 \ub410\uc9c0\ub9cc (\uc6b0\ub9ac\uc5d0\uac8c \ub3c8\uc744 \ub3cc\ub824\uc904 \uc218 \uc788\uc5c8\uc74c\uc5d0\ub3c4) \uc544\ubb34 \uc77c\ub3c4 \uc5c6\uc5c8\ub2e4'\uba70 \ud638\ud61c\uc138 \ubd80\uacfc\uc758 \uc815\ub2f9\uc131\uc744 \ud638\uc18c\ud588\ub2e4. 18\uc77c\uc5d0\ub294 \ubbf8\uad6d \uc0c1\ubb34\ubd80\uac00 \u300c\ubb34\uc5ed\ud655\uc7a5\ubc95\u300d \uc81c232\uc870 \uc870\uc0ac\ubcf4\uace0\uc11c\ub97c \ubc1c\ud45c\ud588\ub294\ub370 \ud55c\uad6d\uacfc \uc911\uad6d \ub4f1 12\uac1c \uad6d\uac00\uc5d0 \ub300\ud574 53%\uc758 \ub192\uc740 \uad00\uc138\ub97c \ubd80\uacfc\ud560 \ub300\uc0c1\uc73c\ub85c \uc9c0\ubaa9\ud588\ub2e4. \ud55c\uad6d \uc815\ubd80\ub294 \ud55c\uad6d\uc774 \ubbf8\uad6d\uc758 \ub3d9\ub9f9\uc774\uae30 \ub54c\ubb38\uc5d0 \ud55c\uad6d\uc0b0 \ucca0\uac15\uc774 \ubbf8\uad6d \uc548\ubcf4\uc5d0 \uc704\ud611\uc774 \ub418\uc9c0 \uc54a\ub294\ub2e4\ub294 \uc810\uc744 \uac15\uc870\ud588\uc9c0\ub9cc \uc124\ub4dd\ub418\uc9c0 \ubabb\ud588\uc73c\uba70, \ub9ce\uc740 \ucca0\uac15\uc744 \uc218\ucd9c\ud558\ub294 \uce90\ub098\ub2e4\u00b7\uc77c\ubcf8\u00b7\ub3c5\uc77c \ub4f1 \uc804\ud1b5\uc801\uc778 \uc6b0\ubc29\uc740 \uace0\uad00\uc138 \ubd80\uacfc \ub300\uc0c1\uc5d0 \ud3ec\ud568\ub418\uc9c0 \uc54a\uc544 \ub17c\ub780\uc744 \uc77c\uc73c\ucf30\ub2e4. \uc0b0\uc5c5\ubd80\ub294 \"\ub300\ubbf8 \uc218\ucd9c\uc774 \ub9ce\uc73c\uba74\uc11c \uc911\uad6d\uc0b0 \ucca0\uac15\uc744 \ub9ce\uc774 \uc218\uc785\ud558\ub294 \uad6d\uac00\ub4e4\uc774 \ud3ec\ud568\ub41c \uac83 \uac19\ub2e4\"\uace0 \ud588\uc73c\uba70 \ubbf8\uad6d \uad6d\ubc29\ubd80\ub3c4 \"\uc774\ub7f0 \uc870\uce58\uac00 \uc911\uad6d\uc758 \uc0dd\uc0b0 \uacfc\uc789\uc744 \ubc14\ub85c\uc7a1\uace0 \uae30\uc874 \ubc18\ub364\ud551 \uad00\uc138\ub97c \uc6b0\ud68c\ud558\ub824\ub294 \uc911\uad6d\uc758 \uc2dc\ub3c4\ub97c \ub9c9\ub294 \ub370 \ucd08\uc810\uc744 \ub9de\ucd98 \uac83\uc774\uc9c0 \ubbf8\uad6d\uacfc\uc758 \uc591\uc790 \uad00\uacc4\uc5d0 \ub9de\ucd98 \uac8c \uc544\ub2c8\ub77c\ub294 \uc810\uc744 \uc6b0\ub9ac\uc758 \ud575\uc2ec \ub3d9\ub9f9\uad6d\ub4e4\uc5d0\uac8c \uac15\uc870\ud558\ub294 \uac8c \ub9e4\uc6b0 \uc911\uc694\ud558\ub2e4\"\uace0 \ubc1d\ud788\uae30\ub3c4 \ud588\ub2e4. \uc138\uc774\ud504\uac00\ub4dc \ub3c4 \ubb38\uc81c\uac00 \ub418\uc5c8\ub2e4. \uc138\uc774\ud504\uac00\ub4dc\ub294 \ud2b9\uc815\ud488\ubaa9\uc758 \uc218\uc785\uc774 \uae09\uc99d\ud558\uc5ec \uad6d\ub0b4 \uc5c5\uacc4\uc5d0 \uc911\ub300\ud55c \uc190\uc2e4\uc774 \ubc1c\uc0dd\ud558\uac70\ub098 \uadf8 \uc6b0\ub824\uac00 \uc788\uc744 \uacbd\uc6b0 \ubc1c\ub3d9\ud558\ub294 \uae34\uae09\uc218\uc785\uc81c\ud55c\uc870\uce58\ub97c \ub9d0\ud558\ub294\ub370 \ubbf8\uad6d\uc774 \ud0dc\uc591\uad11\u00b7\uc138\ud0c1\uae30\uc5d0 \ub300\ud574 \ubb34\uc5ed \uc801\uc790\uc640 \ubbf8\uad6d\ub0b4 \uc81c\uc870\uc5c5\uc790\ub4e4\uc744 \ubcf4\ud638\ud558\uae30 \uc704\ud574 1\uc6d4 22\uc77c \uc138\uc774\ud504\uac00\ub4dc\ub97c \ubc1c\ub3d9\ud55c \uac83\uc774\ub2e4. \uc774\uc5d0 \ud55c\uad6d\uc740 \uc138\uacc4\ubb34\uc5ed\uae30\uad6c \ubd84\uc7c1\ud574\uacb0\uae30\uad6c \uc815\ub840 \ud68c\uc758\uc5d0\uc11c \uc138\ud0c1\uae30 \ubd84\uc7c1\uacfc \uad00\ub828, \ubbf8\uad6d\uc5d0 \ub300\ud55c \uc591\ud5c8\uad00\uc138 \uc815\uc9c0 \uc2b9\uc778\uc744 \uc694\uccad\ud588\uc73c\uba70 \ucca0\uac15\u00b7\ubcc0\uc555\uae30\uc5d0 \uc9d5\ubc8c\uc801 \uad00\uc138\ub97c \ubd80\uacfc\ud55c \uac83 \uc5ed\uc2dc WTO\uc5d0 \uc81c\uc18c\ud558\uae30\ub85c \ud588\ub2e4. \ud3c9\ucc3d \uc62c\ub9bc\ud53d \ub54c, \ud55c\uad6d\uc744 \ucc3e\uc740 \ub9c8\uc774\ud06c \ud39c\uc2a4 \ubbf8\uad6d \ubd80\ud1b5\ub839\uacfc \uc811\uacac\ud560 \ub54c\uc5d0\ub294 \ud55c\uad6d \uc138\ud0c1\uae30\uc5d0 \ub300\ud55c \uc138\uc774\ud504\uac00\ub4dc \uc870\uce58\ub97c \ud480\uc5b4\uc904 \uac83\uc744 \uc694\uccad\ud588\uc73c\uba70, \ud39c\uc2a4\ub294 \"\ubb38 \ub300\ud1b5\ub839\uaed8\uc11c \uc0c1\ub2f9\ud788 \uc26c\uc6b4 \ubb38\uc81c\ub97c \ub358\uc838\uc8fc\uc168\ub2e4\"\uba74\uc11c\ub3c4 \uc989\ub2f5\uc744 \ub0b4\ub193\uc9c0\ub294 \uc54a\uc558\ub2e4.", "sentence_answer": " \uc138\uc774\ud504\uac00\ub4dc \ub3c4 \ubb38\uc81c\uac00 \ub418\uc5c8\ub2e4.", "paragraph_id": "6580768-78-1", "paragraph_question": "question: \uae34\uae09\uc218\uc785\uc81c\ud55c\uc870\uce58\ub85c \ud2b9\uc815\ud488\ubaa9\uc758 \uc218\uc785\uc774 \uae09\uc99d\ud558\uc5ec \uad6d\ub0b4 \uc5c5\uacc4\uc5d0 \uc911\ub300\ud55c \uc190\uc2e4\uc744 \uc785\ud788\uac70\ub098 \uadf8 \uc6b0\ub824\uac00 \uc788\uc744 \ub54c \ubc1c\ub3d9\ud558\ub294 \uac83\uc740 \ubb34\uc5c7\uc778\uac00?, context: \ubbf8\uad6d\uacfc\uc758 \ud1b5\uc0c1 \ub9c8\ucc30\uc740 2018\ub144\ubd80\ud130 \ubcf8\uaca9\ud654\ub410\ub2e4. 2\uc6d4 12\uc77c \ud2b8\ub7fc\ud504\ub294 \"\uc6b0\ub9ac\ub294 \ud55c\uad6d, \uc911\uad6d, \uc77c\ubcf8\uc5d0 \uc5b4\ub9c8\uc5b4\ub9c8\ud55c \ub3c8\uc744 \uc783\uc5c8\ub2e4\"\uba70 \"\uadf8\ub4e4\uc740 25\ub144\uc9f8 \uc81c\uba4b\ub300\ub85c \ud589\ub3d9\ud558\uace0 \uc788\ub2e4\"\uace0 \ube44\ub09c\ud588\ub2e4. \ud2b9\ud788 \ub3d9\ub9f9\uad6d\uc778 \ud55c\uad6d\uacfc \uc77c\ubcf8\uc744 \uaca8\ub0e5\ud574 \"\uc77c\ubd80\ub294 \uc18c\uc704 \ub3d9\ub9f9\uad6d\uc774\uc9c0\ub9cc, \ubb34\uc5ed \uba74\uc5d0\uc11c\ub294 \ub3d9\ub9f9\uad6d\uc774 \uc544\ub2c8\ub2e4\"\ub77c\uba70 \ud638\ud61c\uc138\ub97c \ub3c4\uc785\ud558\uaca0\ub2e4\ub294 \ubc29\uce68\uc744 \ubc1d\ud614\ub2e4. \ud55c\uad6d\uc5d0 \ub300\ud574\uc11c\ub3c4 '\ud55c\uad6d \uc804\uc7c1 \uc9c1\ud6c4 \ud55c\uad6d\uc744 \ub3c4\uc654\uace0 \uadf8\ub4e4\uc740 \ubd80\uc790\uac00 \ub410\uc9c0\ub9cc (\uc6b0\ub9ac\uc5d0\uac8c \ub3c8\uc744 \ub3cc\ub824\uc904 \uc218 \uc788\uc5c8\uc74c\uc5d0\ub3c4) \uc544\ubb34 \uc77c\ub3c4 \uc5c6\uc5c8\ub2e4'\uba70 \ud638\ud61c\uc138 \ubd80\uacfc\uc758 \uc815\ub2f9\uc131\uc744 \ud638\uc18c\ud588\ub2e4. 18\uc77c\uc5d0\ub294 \ubbf8\uad6d \uc0c1\ubb34\ubd80\uac00 \u300c\ubb34\uc5ed\ud655\uc7a5\ubc95\u300d \uc81c232\uc870 \uc870\uc0ac\ubcf4\uace0\uc11c\ub97c \ubc1c\ud45c\ud588\ub294\ub370 \ud55c\uad6d\uacfc \uc911\uad6d \ub4f1 12\uac1c \uad6d\uac00\uc5d0 \ub300\ud574 53%\uc758 \ub192\uc740 \uad00\uc138\ub97c \ubd80\uacfc\ud560 \ub300\uc0c1\uc73c\ub85c \uc9c0\ubaa9\ud588\ub2e4. \ud55c\uad6d \uc815\ubd80\ub294 \ud55c\uad6d\uc774 \ubbf8\uad6d\uc758 \ub3d9\ub9f9\uc774\uae30 \ub54c\ubb38\uc5d0 \ud55c\uad6d\uc0b0 \ucca0\uac15\uc774 \ubbf8\uad6d \uc548\ubcf4\uc5d0 \uc704\ud611\uc774 \ub418\uc9c0 \uc54a\ub294\ub2e4\ub294 \uc810\uc744 \uac15\uc870\ud588\uc9c0\ub9cc \uc124\ub4dd\ub418\uc9c0 \ubabb\ud588\uc73c\uba70, \ub9ce\uc740 \ucca0\uac15\uc744 \uc218\ucd9c\ud558\ub294 \uce90\ub098\ub2e4\u00b7\uc77c\ubcf8\u00b7\ub3c5\uc77c \ub4f1 \uc804\ud1b5\uc801\uc778 \uc6b0\ubc29\uc740 \uace0\uad00\uc138 \ubd80\uacfc \ub300\uc0c1\uc5d0 \ud3ec\ud568\ub418\uc9c0 \uc54a\uc544 \ub17c\ub780\uc744 \uc77c\uc73c\ucf30\ub2e4. \uc0b0\uc5c5\ubd80\ub294 \"\ub300\ubbf8 \uc218\ucd9c\uc774 \ub9ce\uc73c\uba74\uc11c \uc911\uad6d\uc0b0 \ucca0\uac15\uc744 \ub9ce\uc774 \uc218\uc785\ud558\ub294 \uad6d\uac00\ub4e4\uc774 \ud3ec\ud568\ub41c \uac83 \uac19\ub2e4\"\uace0 \ud588\uc73c\uba70 \ubbf8\uad6d \uad6d\ubc29\ubd80\ub3c4 \"\uc774\ub7f0 \uc870\uce58\uac00 \uc911\uad6d\uc758 \uc0dd\uc0b0 \uacfc\uc789\uc744 \ubc14\ub85c\uc7a1\uace0 \uae30\uc874 \ubc18\ub364\ud551 \uad00\uc138\ub97c \uc6b0\ud68c\ud558\ub824\ub294 \uc911\uad6d\uc758 \uc2dc\ub3c4\ub97c \ub9c9\ub294 \ub370 \ucd08\uc810\uc744 \ub9de\ucd98 \uac83\uc774\uc9c0 \ubbf8\uad6d\uacfc\uc758 \uc591\uc790 \uad00\uacc4\uc5d0 \ub9de\ucd98 \uac8c \uc544\ub2c8\ub77c\ub294 \uc810\uc744 \uc6b0\ub9ac\uc758 \ud575\uc2ec \ub3d9\ub9f9\uad6d\ub4e4\uc5d0\uac8c \uac15\uc870\ud558\ub294 \uac8c \ub9e4\uc6b0 \uc911\uc694\ud558\ub2e4\"\uace0 \ubc1d\ud788\uae30\ub3c4 \ud588\ub2e4. \uc138\uc774\ud504\uac00\ub4dc\ub3c4 \ubb38\uc81c\uac00 \ub418\uc5c8\ub2e4. \uc138\uc774\ud504\uac00\ub4dc\ub294 \ud2b9\uc815\ud488\ubaa9\uc758 \uc218\uc785\uc774 \uae09\uc99d\ud558\uc5ec \uad6d\ub0b4 \uc5c5\uacc4\uc5d0 \uc911\ub300\ud55c \uc190\uc2e4\uc774 \ubc1c\uc0dd\ud558\uac70\ub098 \uadf8 \uc6b0\ub824\uac00 \uc788\uc744 \uacbd\uc6b0 \ubc1c\ub3d9\ud558\ub294 \uae34\uae09\uc218\uc785\uc81c\ud55c\uc870\uce58\ub97c \ub9d0\ud558\ub294\ub370 \ubbf8\uad6d\uc774 \ud0dc\uc591\uad11\u00b7\uc138\ud0c1\uae30\uc5d0 \ub300\ud574 \ubb34\uc5ed \uc801\uc790\uc640 \ubbf8\uad6d\ub0b4 \uc81c\uc870\uc5c5\uc790\ub4e4\uc744 \ubcf4\ud638\ud558\uae30 \uc704\ud574 1\uc6d4 22\uc77c \uc138\uc774\ud504\uac00\ub4dc\ub97c \ubc1c\ub3d9\ud55c \uac83\uc774\ub2e4. \uc774\uc5d0 \ud55c\uad6d\uc740 \uc138\uacc4\ubb34\uc5ed\uae30\uad6c \ubd84\uc7c1\ud574\uacb0\uae30\uad6c \uc815\ub840 \ud68c\uc758\uc5d0\uc11c \uc138\ud0c1\uae30 \ubd84\uc7c1\uacfc \uad00\ub828, \ubbf8\uad6d\uc5d0 \ub300\ud55c \uc591\ud5c8\uad00\uc138 \uc815\uc9c0 \uc2b9\uc778\uc744 \uc694\uccad\ud588\uc73c\uba70 \ucca0\uac15\u00b7\ubcc0\uc555\uae30\uc5d0 \uc9d5\ubc8c\uc801 \uad00\uc138\ub97c \ubd80\uacfc\ud55c \uac83 \uc5ed\uc2dc WTO\uc5d0 \uc81c\uc18c\ud558\uae30\ub85c \ud588\ub2e4. \ud3c9\ucc3d \uc62c\ub9bc\ud53d \ub54c, \ud55c\uad6d\uc744 \ucc3e\uc740 \ub9c8\uc774\ud06c \ud39c\uc2a4 \ubbf8\uad6d \ubd80\ud1b5\ub839\uacfc \uc811\uacac\ud560 \ub54c\uc5d0\ub294 \ud55c\uad6d \uc138\ud0c1\uae30\uc5d0 \ub300\ud55c \uc138\uc774\ud504\uac00\ub4dc \uc870\uce58\ub97c \ud480\uc5b4\uc904 \uac83\uc744 \uc694\uccad\ud588\uc73c\uba70, \ud39c\uc2a4\ub294 \"\ubb38 \ub300\ud1b5\ub839\uaed8\uc11c \uc0c1\ub2f9\ud788 \uc26c\uc6b4 \ubb38\uc81c\ub97c \ub358\uc838\uc8fc\uc168\ub2e4\"\uba74\uc11c\ub3c4 \uc989\ub2f5\uc744 \ub0b4\ub193\uc9c0\ub294 \uc54a\uc558\ub2e4."} {"question": "\ud55c\uc591\ub300\ud559\uad50\ub294 2009\ub144 \ub3d9\uc544 \uad6d\uc545 \ucf69\ucfe0\ub974\uc5d0\uc11c \uba87 \uba85\uc758 \uc218\uc0c1\uc790\ub97c \ubc30\ucd9c\ud558\uc600\ub294\uac00?", "paragraph": "\ud55c\uc591\ub300\ud559\uad50\ub294 \uc815\u00b7\uc7ac\uacc4, \ubc95\uc870\uacc4 \ub4f1 \ub300\ud55c\ubbfc\uad6d \uc0ac\ud68c \uc804\ubc18\uc5d0 \uac78\uccd0 \ub9ce\uc740 \uc778\uc0ac\ub97c \ud3ec\uc9c4\ud558\uace0 \uc788\ub2e4. \ud2b9\ud788 \ub300\ud55c\ubbfc\uad6d\uc758 \uc0b0\uc5c5\ud654, \uadfc\ub300\ud654\uc5d0 \ud070 \uacf5\ud5cc\uc744 \ud588\uc73c\uba70, \ud604\ub300\uadf8\ub8f9\uc758 \uc815\uc8fc\uc601 \ud68c\uc7a5\uc740 \uacf5\uc2dd\uc11d\uc0c1\uc5d0\uc11c \ub300\ud55c\ubbfc\uad6d\uc758 \uadfc\ub300\ud654\uc640 \ud568\uaed8 \ud55c\uc591\ub300\ud559\uad50\ub97c \uc9c1\uc811 \uc5b8\uae09\ud560 \uc815\ub3c4\uc600\uace0 2006\ub144 \ud1b5\uacc4 \uae30\uc900\uc73c\ub85c \uc804\uad6d\ub300\ud559\uc5d0\uc11c \ub450 \ubc88\uc9f8\ub85c \ub9ce\uc740 \ucf54\uc2a4\ub2e5 CEO\ub97c \ubc30\ucd9c\ud558\uc600\uc73c\uba70, 2009\ub144\uc5d0\ub294 \uc77c\ubcf8\uc758 \uc544\uc18c \ub2e4\ub85c \ucd1d\ub9ac\uac00 \ud55c\uc591\ub300\ud559\uad50\ub97c \ubc29\ubb38\ud574 \uc5f0\uad6c\uc9c4\uc744 \uaca9\ub824\ud558\uae30\ub3c4 \ud588\uace0 \uac19\uc740 \ud574 \ucd9c\ud310\ub41c \u300a\ucf54\uc2a4\ub2e5 \uc0c1\uc7a5\ubc95\uc778 \uacbd\uc601 \uc778\uba85\ub85d\u300b\uc5d0 \ub530\ub974\uba74 \ud55c\uc591\ub300\ud559\uad50\ub294 \uc11c\uc6b8\ub300, \uc5f0\uc138\ub300 \ub2e4\uc74c\uc73c\ub85c 3\ubc88\uc9f8 \ub9ce\uc740 \ucf54\uc2a4\ub2e5 \ub4f1\uae30\uc784\uc6d0\uc744 \ubc30\ucd9c\ud574 \ub0c8\ub2e4 \ub610\ud55c 2009\ub144 \uc81c2\ucc28 \uc0ac\ubc95\uc2dc\ud5d8\uc5d0\uc11c\ub294 \ub300\ud55c\ubbfc\uad6d\uc5d0\uc11c 4\ubc88\uc9f8\ub85c \ub9ce\uc740 \ud569\uaca9\uc790\ub97c \ubc30\ucd9c\ud574 \ub0b4\uc5c8\uc73c\uba70 3\ucc28 \uc2dc\ud5d8\uc5d0\ub294 \uc804\uc6d0\uc774 \ud569\uaca9\ud558\uc600\ub2e4. \uac19\uc740 \ud574, \uc74c\uc545\ub300\ud559\uc5d0\uc11c\ub294 \uad6d\uc545\uacfc \uad00\ub828\ud55c \ub300\ud68c\uc778 \ub3d9\uc544 \uad6d\uc545 \ucf69\ucfe0\ub974\uc5d0\uc11c \uae08\uc0c1 5\uba85\uc744 \ud3ec\ud568\ud574, \ucd1d 8\uba85\uc758 \uc218\uc0c1\uc790\ub97c \ub3d9\uc2dc\uc5d0 \ubc30\ucd9c\ud574\ub0b4\ub294 \uc131\uacfc\ub97c \uc774\ub8e8\uae30\ub3c4 \ud588\ub2e4. 2010\ub144 \ud1b5\uacc4\uc5d0 \ub530\ub974\uba74 \ub9e4\ucd9c\uc561 1\ucc9c\ub9cc\uc6d0 \uc774\uc0c1\uc758 \ubca4\ucc98\uae30\uc5c5 CEO\ub97c \ub450\ubc88\uc9f8\ub85c \ub9ce\uc774 \ubc30\ucd9c\ud55c \ub300\ud559\uc73c\ub85c \uc870\uc0ac\ub418\uc5c8\uace0, \uac80\ucc30 \uc8fc\uc694\ubcf4\uc9c1\uc790 \uc870\uc0ac \uacb0\uacfc\uc5d0 \ub530\ub974\uba74 \uc911 3\ubc88\uc9f8\ub85c \ub9ce\uc740 \uacf5\uc9c1\uc790\uac00 \ud55c\uc591\ub300\ud559\uad50 \ucd9c\uc2e0\uc73c\ub85c \ubc30\uce58\ub418\uc5b4\uc788\ub2e4. 2011\ub144 \ud1b5\uacc4 \uae30\uc900, 100\ub300 \uae30\uc5c5(\ub9e4\ucd9c\uc561 \uae30\uc900)\uc758 \uc7ac\uc9c1\uc784\uc6d0 \uc911, \ub300\ud55c\ubbfc\uad6d\uc5d0\uc11c 4\ubc88\uc9f8\ub85c \ub9ce\uc740 \ud55c\uc591\ub300\ud559\uad50 \ucd9c\uc2e0 \uc784\uc6d0\ub4e4\uc774 \ubc30\uce58\ub418\uc5b4\uc788\uc73c\uba70, \uc8fc\ub85c \uae30\uc220\uc9c1 \ubd84\uc57c\uc758 \uc784\uc6d0\uc73c\ub85c \uc885\uc0ac\ud558\uace0 \uc788\ub294\uac83\uc73c\ub85c \uc870\uc0ac\ub418\uc5c8\ub2e4. 2014\ub144 \ud1b5\uacc4\uc5d0\uc11c\ub294, \ub300\ud55c\ubbfc\uad6d\uc5d0\uc11c \uc11c\uc6b8\ub300, \uc5f0\uc138\ub300\uc5d0 \uc774\uc5b4 3\ubc88\uc9f8\ub85c \ucf54\uc2a4\ub2e5 \uc0c1\uc7a5\uc0ac \ucd5c\uace0\uacbd\uc601\uc790(CEO)\ub97c \uac00\uc7a5 \ub9ce\uc774 \ubc30\ucd9c\ud55c \ub300\ud559\uc73c\ub85c \uc870\uc0ac\ub410\uc73c\uba70, \ub300\ud55c\ubbfc\uad6d\uc758 500\ub300 \uae30\uc5c5 \ucd5c\uace0\uacbd\uc601\uc790(CEO)\ub4e4\uc758 \ucd9c\uc2e0\ub300\ud559 \uc870\uc0ac\uc5d0\uc11c\ub294 4\uc704\ub85c \uc870\uc0ac\ub410\ub2e4. \uadf8\ub9ac\uace0 2014\ub144 \uae30\uc900 \uc804\uad6d \ud604\uc9c1\uac80\uc0ac \ucd9c\uc2e0\ub300\ud559 \uc870\uc0ac\uc5d0\uc11c\ub294 4\uc704\ub85c \ub098\ud0c0\ub0ac\ub2e4. \ud55c\ud3b8 2014\ub144\uc5d0 \uce58\ub7ec\uc9c4 \uc81c56\ud68c \uc0ac\ubc95\uc2dc\ud5d8\uc5d0\uc11c\ub294 \uc11c\uc6b8\ub300, \uc5f0\uc138\ub300\uc5d0 \uc774\uc5b4 3\ubc88\uc9f8\ub85c \ub9ce\uc740 \ud569\uaca9\uc790\ub97c \ubc30\ucd9c\ud558\uc600\uc73c\uba70, \uac19\uc740 \ud574\uc758 \ud589\uc815\uace0\uc2dc(5\uae09\uacf5\ucc44 \ud589\uc815\uc9c1)\uc640 \uae30\uc220\uace0\uc2dc(5\uae09\uacf5\ucc44 \uae30\uc220\uc9c1)\uc5d0\uc11c\ub294 \uac01\uac01 4\ubc88\uc9f8\ub85c \ub9ce\uc740 \ud569\uaca9\uc790\ub97c \ubc30\ucd9c\ud558\uc600\ub2e4. 2015\ub144\uc5d0\ub294 \uc911\uc559\uc77c\ubcf4 \ub300\ud559\ud3c9\uac00\uc5d0\uc11c \uad6d\ub0b4\ub300\ud559 \uc911 3\uc704\ub97c \ucc28\uc9c0\ud558\uc600\uace0, \uc138\uacc4\uc801\uc778 \ud1b5\uc2e0\uc0ac \ub85c\uc774\ud130(Reuters)\uac00 \uc120\uc815\ud55c '2015 \uc138\uacc4 \ud601\uc2e0 \ub300\ud559 \ud1b1 100 (The World's Most Innovative Universities)'\uc5d0\uc11c\ub294 \uc138\uacc4 62\uc704, \uad6d\ub0b4 \uc885\ud569\ub300\ud559 \uc911 3\uc704\uc5d0 \ub7ad\ud06c\ub410\ub2e4.", "answer": "8\uba85", "sentence": "\uac19\uc740 \ud574, \uc74c\uc545\ub300\ud559\uc5d0\uc11c\ub294 \uad6d\uc545\uacfc \uad00\ub828\ud55c \ub300\ud68c\uc778 \ub3d9\uc544 \uad6d\uc545 \ucf69\ucfe0\ub974\uc5d0\uc11c \uae08\uc0c1 5\uba85\uc744 \ud3ec\ud568\ud574, \ucd1d 8\uba85 \uc758 \uc218\uc0c1\uc790\ub97c \ub3d9\uc2dc\uc5d0 \ubc30\ucd9c\ud574\ub0b4\ub294 \uc131\uacfc\ub97c \uc774\ub8e8\uae30\ub3c4 \ud588\ub2e4.", "paragraph_sentence": "\ud55c\uc591\ub300\ud559\uad50\ub294 \uc815\u00b7\uc7ac\uacc4, \ubc95\uc870\uacc4 \ub4f1 \ub300\ud55c\ubbfc\uad6d \uc0ac\ud68c \uc804\ubc18\uc5d0 \uac78\uccd0 \ub9ce\uc740 \uc778\uc0ac\ub97c \ud3ec\uc9c4\ud558\uace0 \uc788\ub2e4. \ud2b9\ud788 \ub300\ud55c\ubbfc\uad6d\uc758 \uc0b0\uc5c5\ud654, \uadfc\ub300\ud654\uc5d0 \ud070 \uacf5\ud5cc\uc744 \ud588\uc73c\uba70, \ud604\ub300\uadf8\ub8f9\uc758 \uc815\uc8fc\uc601 \ud68c\uc7a5\uc740 \uacf5\uc2dd\uc11d\uc0c1\uc5d0\uc11c \ub300\ud55c\ubbfc\uad6d\uc758 \uadfc\ub300\ud654\uc640 \ud568\uaed8 \ud55c\uc591\ub300\ud559\uad50\ub97c \uc9c1\uc811 \uc5b8\uae09\ud560 \uc815\ub3c4\uc600\uace0 2006\ub144 \ud1b5\uacc4 \uae30\uc900\uc73c\ub85c \uc804\uad6d\ub300\ud559\uc5d0\uc11c \ub450 \ubc88\uc9f8\ub85c \ub9ce\uc740 \ucf54\uc2a4\ub2e5 CEO\ub97c \ubc30\ucd9c\ud558\uc600\uc73c\uba70, 2009\ub144\uc5d0\ub294 \uc77c\ubcf8\uc758 \uc544\uc18c \ub2e4\ub85c \ucd1d\ub9ac\uac00 \ud55c\uc591\ub300\ud559\uad50\ub97c \ubc29\ubb38\ud574 \uc5f0\uad6c\uc9c4\uc744 \uaca9\ub824\ud558\uae30\ub3c4 \ud588\uace0 \uac19\uc740 \ud574 \ucd9c\ud310\ub41c \u300a\ucf54\uc2a4\ub2e5 \uc0c1\uc7a5\ubc95\uc778 \uacbd\uc601 \uc778\uba85\ub85d\u300b\uc5d0 \ub530\ub974\uba74 \ud55c\uc591\ub300\ud559\uad50\ub294 \uc11c\uc6b8\ub300, \uc5f0\uc138\ub300 \ub2e4\uc74c\uc73c\ub85c 3\ubc88\uc9f8 \ub9ce\uc740 \ucf54\uc2a4\ub2e5 \ub4f1\uae30\uc784\uc6d0\uc744 \ubc30\ucd9c\ud574 \ub0c8\ub2e4 \ub610\ud55c 2009\ub144 \uc81c2\ucc28 \uc0ac\ubc95\uc2dc\ud5d8\uc5d0\uc11c\ub294 \ub300\ud55c\ubbfc\uad6d\uc5d0\uc11c 4\ubc88\uc9f8\ub85c \ub9ce\uc740 \ud569\uaca9\uc790\ub97c \ubc30\ucd9c\ud574 \ub0b4\uc5c8\uc73c\uba70 3\ucc28 \uc2dc\ud5d8\uc5d0\ub294 \uc804\uc6d0\uc774 \ud569\uaca9\ud558\uc600\ub2e4. \uac19\uc740 \ud574, \uc74c\uc545\ub300\ud559\uc5d0\uc11c\ub294 \uad6d\uc545\uacfc \uad00\ub828\ud55c \ub300\ud68c\uc778 \ub3d9\uc544 \uad6d\uc545 \ucf69\ucfe0\ub974\uc5d0\uc11c \uae08\uc0c1 5\uba85\uc744 \ud3ec\ud568\ud574, \ucd1d 8\uba85 \uc758 \uc218\uc0c1\uc790\ub97c \ub3d9\uc2dc\uc5d0 \ubc30\ucd9c\ud574\ub0b4\ub294 \uc131\uacfc\ub97c \uc774\ub8e8\uae30\ub3c4 \ud588\ub2e4. 2010\ub144 \ud1b5\uacc4\uc5d0 \ub530\ub974\uba74 \ub9e4\ucd9c\uc561 1\ucc9c\ub9cc\uc6d0 \uc774\uc0c1\uc758 \ubca4\ucc98\uae30\uc5c5 CEO\ub97c \ub450\ubc88\uc9f8\ub85c \ub9ce\uc774 \ubc30\ucd9c\ud55c \ub300\ud559\uc73c\ub85c \uc870\uc0ac\ub418\uc5c8\uace0, \uac80\ucc30 \uc8fc\uc694\ubcf4\uc9c1\uc790 \uc870\uc0ac \uacb0\uacfc\uc5d0 \ub530\ub974\uba74 \uc911 3\ubc88\uc9f8\ub85c \ub9ce\uc740 \uacf5\uc9c1\uc790\uac00 \ud55c\uc591\ub300\ud559\uad50 \ucd9c\uc2e0\uc73c\ub85c \ubc30\uce58\ub418\uc5b4\uc788\ub2e4. 2011\ub144 \ud1b5\uacc4 \uae30\uc900, 100\ub300 \uae30\uc5c5(\ub9e4\ucd9c\uc561 \uae30\uc900)\uc758 \uc7ac\uc9c1\uc784\uc6d0 \uc911, \ub300\ud55c\ubbfc\uad6d\uc5d0\uc11c 4\ubc88\uc9f8\ub85c \ub9ce\uc740 \ud55c\uc591\ub300\ud559\uad50 \ucd9c\uc2e0 \uc784\uc6d0\ub4e4\uc774 \ubc30\uce58\ub418\uc5b4\uc788\uc73c\uba70, \uc8fc\ub85c \uae30\uc220\uc9c1 \ubd84\uc57c\uc758 \uc784\uc6d0\uc73c\ub85c \uc885\uc0ac\ud558\uace0 \uc788\ub294\uac83\uc73c\ub85c \uc870\uc0ac\ub418\uc5c8\ub2e4. 2014\ub144 \ud1b5\uacc4\uc5d0\uc11c\ub294, \ub300\ud55c\ubbfc\uad6d\uc5d0\uc11c \uc11c\uc6b8\ub300, \uc5f0\uc138\ub300\uc5d0 \uc774\uc5b4 3\ubc88\uc9f8\ub85c \ucf54\uc2a4\ub2e5 \uc0c1\uc7a5\uc0ac \ucd5c\uace0\uacbd\uc601\uc790(CEO)\ub97c \uac00\uc7a5 \ub9ce\uc774 \ubc30\ucd9c\ud55c \ub300\ud559\uc73c\ub85c \uc870\uc0ac\ub410\uc73c\uba70, \ub300\ud55c\ubbfc\uad6d\uc758 500\ub300 \uae30\uc5c5 \ucd5c\uace0\uacbd\uc601\uc790(CEO)\ub4e4\uc758 \ucd9c\uc2e0\ub300\ud559 \uc870\uc0ac\uc5d0\uc11c\ub294 4\uc704\ub85c \uc870\uc0ac\ub410\ub2e4. \uadf8\ub9ac\uace0 2014\ub144 \uae30\uc900 \uc804\uad6d \ud604\uc9c1\uac80\uc0ac \ucd9c\uc2e0\ub300\ud559 \uc870\uc0ac\uc5d0\uc11c\ub294 4\uc704\ub85c \ub098\ud0c0\ub0ac\ub2e4. \ud55c\ud3b8 2014\ub144\uc5d0 \uce58\ub7ec\uc9c4 \uc81c56\ud68c \uc0ac\ubc95\uc2dc\ud5d8\uc5d0\uc11c\ub294 \uc11c\uc6b8\ub300, \uc5f0\uc138\ub300\uc5d0 \uc774\uc5b4 3\ubc88\uc9f8\ub85c \ub9ce\uc740 \ud569\uaca9\uc790\ub97c \ubc30\ucd9c\ud558\uc600\uc73c\uba70, \uac19\uc740 \ud574\uc758 \ud589\uc815\uace0\uc2dc(5\uae09\uacf5\ucc44 \ud589\uc815\uc9c1)\uc640 \uae30\uc220\uace0\uc2dc(5\uae09\uacf5\ucc44 \uae30\uc220\uc9c1)\uc5d0\uc11c\ub294 \uac01\uac01 4\ubc88\uc9f8\ub85c \ub9ce\uc740 \ud569\uaca9\uc790\ub97c \ubc30\ucd9c\ud558\uc600\ub2e4. 2015\ub144\uc5d0\ub294 \uc911\uc559\uc77c\ubcf4 \ub300\ud559\ud3c9\uac00\uc5d0\uc11c \uad6d\ub0b4\ub300\ud559 \uc911 3\uc704\ub97c \ucc28\uc9c0\ud558\uc600\uace0, \uc138\uacc4\uc801\uc778 \ud1b5\uc2e0\uc0ac \ub85c\uc774\ud130(Reuters)\uac00 \uc120\uc815\ud55c '2015 \uc138\uacc4 \ud601\uc2e0 \ub300\ud559 \ud1b1 100 (The World's Most Innovative Universities)'\uc5d0\uc11c\ub294 \uc138\uacc4 62\uc704, \uad6d\ub0b4 \uc885\ud569\ub300\ud559 \uc911 3\uc704\uc5d0 \ub7ad\ud06c\ub410\ub2e4.", "paragraph_answer": "\ud55c\uc591\ub300\ud559\uad50\ub294 \uc815\u00b7\uc7ac\uacc4, \ubc95\uc870\uacc4 \ub4f1 \ub300\ud55c\ubbfc\uad6d \uc0ac\ud68c \uc804\ubc18\uc5d0 \uac78\uccd0 \ub9ce\uc740 \uc778\uc0ac\ub97c \ud3ec\uc9c4\ud558\uace0 \uc788\ub2e4. \ud2b9\ud788 \ub300\ud55c\ubbfc\uad6d\uc758 \uc0b0\uc5c5\ud654, \uadfc\ub300\ud654\uc5d0 \ud070 \uacf5\ud5cc\uc744 \ud588\uc73c\uba70, \ud604\ub300\uadf8\ub8f9\uc758 \uc815\uc8fc\uc601 \ud68c\uc7a5\uc740 \uacf5\uc2dd\uc11d\uc0c1\uc5d0\uc11c \ub300\ud55c\ubbfc\uad6d\uc758 \uadfc\ub300\ud654\uc640 \ud568\uaed8 \ud55c\uc591\ub300\ud559\uad50\ub97c \uc9c1\uc811 \uc5b8\uae09\ud560 \uc815\ub3c4\uc600\uace0 2006\ub144 \ud1b5\uacc4 \uae30\uc900\uc73c\ub85c \uc804\uad6d\ub300\ud559\uc5d0\uc11c \ub450 \ubc88\uc9f8\ub85c \ub9ce\uc740 \ucf54\uc2a4\ub2e5 CEO\ub97c \ubc30\ucd9c\ud558\uc600\uc73c\uba70, 2009\ub144\uc5d0\ub294 \uc77c\ubcf8\uc758 \uc544\uc18c \ub2e4\ub85c \ucd1d\ub9ac\uac00 \ud55c\uc591\ub300\ud559\uad50\ub97c \ubc29\ubb38\ud574 \uc5f0\uad6c\uc9c4\uc744 \uaca9\ub824\ud558\uae30\ub3c4 \ud588\uace0 \uac19\uc740 \ud574 \ucd9c\ud310\ub41c \u300a\ucf54\uc2a4\ub2e5 \uc0c1\uc7a5\ubc95\uc778 \uacbd\uc601 \uc778\uba85\ub85d\u300b\uc5d0 \ub530\ub974\uba74 \ud55c\uc591\ub300\ud559\uad50\ub294 \uc11c\uc6b8\ub300, \uc5f0\uc138\ub300 \ub2e4\uc74c\uc73c\ub85c 3\ubc88\uc9f8 \ub9ce\uc740 \ucf54\uc2a4\ub2e5 \ub4f1\uae30\uc784\uc6d0\uc744 \ubc30\ucd9c\ud574 \ub0c8\ub2e4 \ub610\ud55c 2009\ub144 \uc81c2\ucc28 \uc0ac\ubc95\uc2dc\ud5d8\uc5d0\uc11c\ub294 \ub300\ud55c\ubbfc\uad6d\uc5d0\uc11c 4\ubc88\uc9f8\ub85c \ub9ce\uc740 \ud569\uaca9\uc790\ub97c \ubc30\ucd9c\ud574 \ub0b4\uc5c8\uc73c\uba70 3\ucc28 \uc2dc\ud5d8\uc5d0\ub294 \uc804\uc6d0\uc774 \ud569\uaca9\ud558\uc600\ub2e4. \uac19\uc740 \ud574, \uc74c\uc545\ub300\ud559\uc5d0\uc11c\ub294 \uad6d\uc545\uacfc \uad00\ub828\ud55c \ub300\ud68c\uc778 \ub3d9\uc544 \uad6d\uc545 \ucf69\ucfe0\ub974\uc5d0\uc11c \uae08\uc0c1 5\uba85\uc744 \ud3ec\ud568\ud574, \ucd1d 8\uba85 \uc758 \uc218\uc0c1\uc790\ub97c \ub3d9\uc2dc\uc5d0 \ubc30\ucd9c\ud574\ub0b4\ub294 \uc131\uacfc\ub97c \uc774\ub8e8\uae30\ub3c4 \ud588\ub2e4. 2010\ub144 \ud1b5\uacc4\uc5d0 \ub530\ub974\uba74 \ub9e4\ucd9c\uc561 1\ucc9c\ub9cc\uc6d0 \uc774\uc0c1\uc758 \ubca4\ucc98\uae30\uc5c5 CEO\ub97c \ub450\ubc88\uc9f8\ub85c \ub9ce\uc774 \ubc30\ucd9c\ud55c \ub300\ud559\uc73c\ub85c \uc870\uc0ac\ub418\uc5c8\uace0, \uac80\ucc30 \uc8fc\uc694\ubcf4\uc9c1\uc790 \uc870\uc0ac \uacb0\uacfc\uc5d0 \ub530\ub974\uba74 \uc911 3\ubc88\uc9f8\ub85c \ub9ce\uc740 \uacf5\uc9c1\uc790\uac00 \ud55c\uc591\ub300\ud559\uad50 \ucd9c\uc2e0\uc73c\ub85c \ubc30\uce58\ub418\uc5b4\uc788\ub2e4. 2011\ub144 \ud1b5\uacc4 \uae30\uc900, 100\ub300 \uae30\uc5c5(\ub9e4\ucd9c\uc561 \uae30\uc900)\uc758 \uc7ac\uc9c1\uc784\uc6d0 \uc911, \ub300\ud55c\ubbfc\uad6d\uc5d0\uc11c 4\ubc88\uc9f8\ub85c \ub9ce\uc740 \ud55c\uc591\ub300\ud559\uad50 \ucd9c\uc2e0 \uc784\uc6d0\ub4e4\uc774 \ubc30\uce58\ub418\uc5b4\uc788\uc73c\uba70, \uc8fc\ub85c \uae30\uc220\uc9c1 \ubd84\uc57c\uc758 \uc784\uc6d0\uc73c\ub85c \uc885\uc0ac\ud558\uace0 \uc788\ub294\uac83\uc73c\ub85c \uc870\uc0ac\ub418\uc5c8\ub2e4. 2014\ub144 \ud1b5\uacc4\uc5d0\uc11c\ub294, \ub300\ud55c\ubbfc\uad6d\uc5d0\uc11c \uc11c\uc6b8\ub300, \uc5f0\uc138\ub300\uc5d0 \uc774\uc5b4 3\ubc88\uc9f8\ub85c \ucf54\uc2a4\ub2e5 \uc0c1\uc7a5\uc0ac \ucd5c\uace0\uacbd\uc601\uc790(CEO)\ub97c \uac00\uc7a5 \ub9ce\uc774 \ubc30\ucd9c\ud55c \ub300\ud559\uc73c\ub85c \uc870\uc0ac\ub410\uc73c\uba70, \ub300\ud55c\ubbfc\uad6d\uc758 500\ub300 \uae30\uc5c5 \ucd5c\uace0\uacbd\uc601\uc790(CEO)\ub4e4\uc758 \ucd9c\uc2e0\ub300\ud559 \uc870\uc0ac\uc5d0\uc11c\ub294 4\uc704\ub85c \uc870\uc0ac\ub410\ub2e4. \uadf8\ub9ac\uace0 2014\ub144 \uae30\uc900 \uc804\uad6d \ud604\uc9c1\uac80\uc0ac \ucd9c\uc2e0\ub300\ud559 \uc870\uc0ac\uc5d0\uc11c\ub294 4\uc704\ub85c \ub098\ud0c0\ub0ac\ub2e4. \ud55c\ud3b8 2014\ub144\uc5d0 \uce58\ub7ec\uc9c4 \uc81c56\ud68c \uc0ac\ubc95\uc2dc\ud5d8\uc5d0\uc11c\ub294 \uc11c\uc6b8\ub300, \uc5f0\uc138\ub300\uc5d0 \uc774\uc5b4 3\ubc88\uc9f8\ub85c \ub9ce\uc740 \ud569\uaca9\uc790\ub97c \ubc30\ucd9c\ud558\uc600\uc73c\uba70, \uac19\uc740 \ud574\uc758 \ud589\uc815\uace0\uc2dc(5\uae09\uacf5\ucc44 \ud589\uc815\uc9c1)\uc640 \uae30\uc220\uace0\uc2dc(5\uae09\uacf5\ucc44 \uae30\uc220\uc9c1)\uc5d0\uc11c\ub294 \uac01\uac01 4\ubc88\uc9f8\ub85c \ub9ce\uc740 \ud569\uaca9\uc790\ub97c \ubc30\ucd9c\ud558\uc600\ub2e4. 2015\ub144\uc5d0\ub294 \uc911\uc559\uc77c\ubcf4 \ub300\ud559\ud3c9\uac00\uc5d0\uc11c \uad6d\ub0b4\ub300\ud559 \uc911 3\uc704\ub97c \ucc28\uc9c0\ud558\uc600\uace0, \uc138\uacc4\uc801\uc778 \ud1b5\uc2e0\uc0ac \ub85c\uc774\ud130(Reuters)\uac00 \uc120\uc815\ud55c '2015 \uc138\uacc4 \ud601\uc2e0 \ub300\ud559 \ud1b1 100 (The World's Most Innovative Universities)'\uc5d0\uc11c\ub294 \uc138\uacc4 62\uc704, \uad6d\ub0b4 \uc885\ud569\ub300\ud559 \uc911 3\uc704\uc5d0 \ub7ad\ud06c\ub410\ub2e4.", "sentence_answer": "\uac19\uc740 \ud574, \uc74c\uc545\ub300\ud559\uc5d0\uc11c\ub294 \uad6d\uc545\uacfc \uad00\ub828\ud55c \ub300\ud68c\uc778 \ub3d9\uc544 \uad6d\uc545 \ucf69\ucfe0\ub974\uc5d0\uc11c \uae08\uc0c1 5\uba85\uc744 \ud3ec\ud568\ud574, \ucd1d 8\uba85 \uc758 \uc218\uc0c1\uc790\ub97c \ub3d9\uc2dc\uc5d0 \ubc30\ucd9c\ud574\ub0b4\ub294 \uc131\uacfc\ub97c \uc774\ub8e8\uae30\ub3c4 \ud588\ub2e4.", "paragraph_id": "6570118-7-1", "paragraph_question": "question: \ud55c\uc591\ub300\ud559\uad50\ub294 2009\ub144 \ub3d9\uc544 \uad6d\uc545 \ucf69\ucfe0\ub974\uc5d0\uc11c \uba87 \uba85\uc758 \uc218\uc0c1\uc790\ub97c \ubc30\ucd9c\ud558\uc600\ub294\uac00?, context: \ud55c\uc591\ub300\ud559\uad50\ub294 \uc815\u00b7\uc7ac\uacc4, \ubc95\uc870\uacc4 \ub4f1 \ub300\ud55c\ubbfc\uad6d \uc0ac\ud68c \uc804\ubc18\uc5d0 \uac78\uccd0 \ub9ce\uc740 \uc778\uc0ac\ub97c \ud3ec\uc9c4\ud558\uace0 \uc788\ub2e4. \ud2b9\ud788 \ub300\ud55c\ubbfc\uad6d\uc758 \uc0b0\uc5c5\ud654, \uadfc\ub300\ud654\uc5d0 \ud070 \uacf5\ud5cc\uc744 \ud588\uc73c\uba70, \ud604\ub300\uadf8\ub8f9\uc758 \uc815\uc8fc\uc601 \ud68c\uc7a5\uc740 \uacf5\uc2dd\uc11d\uc0c1\uc5d0\uc11c \ub300\ud55c\ubbfc\uad6d\uc758 \uadfc\ub300\ud654\uc640 \ud568\uaed8 \ud55c\uc591\ub300\ud559\uad50\ub97c \uc9c1\uc811 \uc5b8\uae09\ud560 \uc815\ub3c4\uc600\uace0 2006\ub144 \ud1b5\uacc4 \uae30\uc900\uc73c\ub85c \uc804\uad6d\ub300\ud559\uc5d0\uc11c \ub450 \ubc88\uc9f8\ub85c \ub9ce\uc740 \ucf54\uc2a4\ub2e5 CEO\ub97c \ubc30\ucd9c\ud558\uc600\uc73c\uba70, 2009\ub144\uc5d0\ub294 \uc77c\ubcf8\uc758 \uc544\uc18c \ub2e4\ub85c \ucd1d\ub9ac\uac00 \ud55c\uc591\ub300\ud559\uad50\ub97c \ubc29\ubb38\ud574 \uc5f0\uad6c\uc9c4\uc744 \uaca9\ub824\ud558\uae30\ub3c4 \ud588\uace0 \uac19\uc740 \ud574 \ucd9c\ud310\ub41c \u300a\ucf54\uc2a4\ub2e5 \uc0c1\uc7a5\ubc95\uc778 \uacbd\uc601 \uc778\uba85\ub85d\u300b\uc5d0 \ub530\ub974\uba74 \ud55c\uc591\ub300\ud559\uad50\ub294 \uc11c\uc6b8\ub300, \uc5f0\uc138\ub300 \ub2e4\uc74c\uc73c\ub85c 3\ubc88\uc9f8 \ub9ce\uc740 \ucf54\uc2a4\ub2e5 \ub4f1\uae30\uc784\uc6d0\uc744 \ubc30\ucd9c\ud574 \ub0c8\ub2e4 \ub610\ud55c 2009\ub144 \uc81c2\ucc28 \uc0ac\ubc95\uc2dc\ud5d8\uc5d0\uc11c\ub294 \ub300\ud55c\ubbfc\uad6d\uc5d0\uc11c 4\ubc88\uc9f8\ub85c \ub9ce\uc740 \ud569\uaca9\uc790\ub97c \ubc30\ucd9c\ud574 \ub0b4\uc5c8\uc73c\uba70 3\ucc28 \uc2dc\ud5d8\uc5d0\ub294 \uc804\uc6d0\uc774 \ud569\uaca9\ud558\uc600\ub2e4. \uac19\uc740 \ud574, \uc74c\uc545\ub300\ud559\uc5d0\uc11c\ub294 \uad6d\uc545\uacfc \uad00\ub828\ud55c \ub300\ud68c\uc778 \ub3d9\uc544 \uad6d\uc545 \ucf69\ucfe0\ub974\uc5d0\uc11c \uae08\uc0c1 5\uba85\uc744 \ud3ec\ud568\ud574, \ucd1d 8\uba85\uc758 \uc218\uc0c1\uc790\ub97c \ub3d9\uc2dc\uc5d0 \ubc30\ucd9c\ud574\ub0b4\ub294 \uc131\uacfc\ub97c \uc774\ub8e8\uae30\ub3c4 \ud588\ub2e4. 2010\ub144 \ud1b5\uacc4\uc5d0 \ub530\ub974\uba74 \ub9e4\ucd9c\uc561 1\ucc9c\ub9cc\uc6d0 \uc774\uc0c1\uc758 \ubca4\ucc98\uae30\uc5c5 CEO\ub97c \ub450\ubc88\uc9f8\ub85c \ub9ce\uc774 \ubc30\ucd9c\ud55c \ub300\ud559\uc73c\ub85c \uc870\uc0ac\ub418\uc5c8\uace0, \uac80\ucc30 \uc8fc\uc694\ubcf4\uc9c1\uc790 \uc870\uc0ac \uacb0\uacfc\uc5d0 \ub530\ub974\uba74 \uc911 3\ubc88\uc9f8\ub85c \ub9ce\uc740 \uacf5\uc9c1\uc790\uac00 \ud55c\uc591\ub300\ud559\uad50 \ucd9c\uc2e0\uc73c\ub85c \ubc30\uce58\ub418\uc5b4\uc788\ub2e4. 2011\ub144 \ud1b5\uacc4 \uae30\uc900, 100\ub300 \uae30\uc5c5(\ub9e4\ucd9c\uc561 \uae30\uc900)\uc758 \uc7ac\uc9c1\uc784\uc6d0 \uc911, \ub300\ud55c\ubbfc\uad6d\uc5d0\uc11c 4\ubc88\uc9f8\ub85c \ub9ce\uc740 \ud55c\uc591\ub300\ud559\uad50 \ucd9c\uc2e0 \uc784\uc6d0\ub4e4\uc774 \ubc30\uce58\ub418\uc5b4\uc788\uc73c\uba70, \uc8fc\ub85c \uae30\uc220\uc9c1 \ubd84\uc57c\uc758 \uc784\uc6d0\uc73c\ub85c \uc885\uc0ac\ud558\uace0 \uc788\ub294\uac83\uc73c\ub85c \uc870\uc0ac\ub418\uc5c8\ub2e4. 2014\ub144 \ud1b5\uacc4\uc5d0\uc11c\ub294, \ub300\ud55c\ubbfc\uad6d\uc5d0\uc11c \uc11c\uc6b8\ub300, \uc5f0\uc138\ub300\uc5d0 \uc774\uc5b4 3\ubc88\uc9f8\ub85c \ucf54\uc2a4\ub2e5 \uc0c1\uc7a5\uc0ac \ucd5c\uace0\uacbd\uc601\uc790(CEO)\ub97c \uac00\uc7a5 \ub9ce\uc774 \ubc30\ucd9c\ud55c \ub300\ud559\uc73c\ub85c \uc870\uc0ac\ub410\uc73c\uba70, \ub300\ud55c\ubbfc\uad6d\uc758 500\ub300 \uae30\uc5c5 \ucd5c\uace0\uacbd\uc601\uc790(CEO)\ub4e4\uc758 \ucd9c\uc2e0\ub300\ud559 \uc870\uc0ac\uc5d0\uc11c\ub294 4\uc704\ub85c \uc870\uc0ac\ub410\ub2e4. \uadf8\ub9ac\uace0 2014\ub144 \uae30\uc900 \uc804\uad6d \ud604\uc9c1\uac80\uc0ac \ucd9c\uc2e0\ub300\ud559 \uc870\uc0ac\uc5d0\uc11c\ub294 4\uc704\ub85c \ub098\ud0c0\ub0ac\ub2e4. \ud55c\ud3b8 2014\ub144\uc5d0 \uce58\ub7ec\uc9c4 \uc81c56\ud68c \uc0ac\ubc95\uc2dc\ud5d8\uc5d0\uc11c\ub294 \uc11c\uc6b8\ub300, \uc5f0\uc138\ub300\uc5d0 \uc774\uc5b4 3\ubc88\uc9f8\ub85c \ub9ce\uc740 \ud569\uaca9\uc790\ub97c \ubc30\ucd9c\ud558\uc600\uc73c\uba70, \uac19\uc740 \ud574\uc758 \ud589\uc815\uace0\uc2dc(5\uae09\uacf5\ucc44 \ud589\uc815\uc9c1)\uc640 \uae30\uc220\uace0\uc2dc(5\uae09\uacf5\ucc44 \uae30\uc220\uc9c1)\uc5d0\uc11c\ub294 \uac01\uac01 4\ubc88\uc9f8\ub85c \ub9ce\uc740 \ud569\uaca9\uc790\ub97c \ubc30\ucd9c\ud558\uc600\ub2e4. 2015\ub144\uc5d0\ub294 \uc911\uc559\uc77c\ubcf4 \ub300\ud559\ud3c9\uac00\uc5d0\uc11c \uad6d\ub0b4\ub300\ud559 \uc911 3\uc704\ub97c \ucc28\uc9c0\ud558\uc600\uace0, \uc138\uacc4\uc801\uc778 \ud1b5\uc2e0\uc0ac \ub85c\uc774\ud130(Reuters)\uac00 \uc120\uc815\ud55c '2015 \uc138\uacc4 \ud601\uc2e0 \ub300\ud559 \ud1b1 100 (The World's Most Innovative Universities)'\uc5d0\uc11c\ub294 \uc138\uacc4 62\uc704, \uad6d\ub0b4 \uc885\ud569\ub300\ud559 \uc911 3\uc704\uc5d0 \ub7ad\ud06c\ub410\ub2e4."} {"question": "\uc870\uc601\ub0a8\uc774 1987\ub144 \ucd9c\uc5f0\ud588\ub358 \uc0ac\uadf9 \ub4dc\ub77c\ub9c8\uc758 \uc81c\ubaa9\uc740?", "paragraph": "1982\ub144\uc5d0 \ub300\ud55c\ubbfc\uad6d\uc73c\ub85c \uadc0\uad6d\ud558\uc600\uace0 \uac00\uc218 \uae40\ub3c4\ud5a5\uacfc \ud568\uaed8 \uc74c\uc545\uc791\uc5c5\uc744 \ud558\uba74\uc11c \uadf8\uc640 \ud568\uaed8 \uc74c\ubc18\uc744 \ubc1c\ub9e4\ud558\uba70 \ud65c\ub3d9\uc744 \ud574\uc654\ub2e4. 1987\ub144\uc5d0\ub294 \uc8fc\uc2dc\uacbd \uc120\uc0dd\uc744 \uc874\uacbd\ud55c\ub2e4\ub294 \uc2e0\ub150\uc5d0 \ub530\ub77c \uc0ac\uadf9 MBC \ub4dc\ub77c\ub9c8 \u300a\ud55c\ud78c\uc0d8\u300b(MBC \ud154\ub808\ube44\uc804 \ub4dc\ub77c\ub9c8)\uc5d0\ub3c4 \ucd9c\uc5f0\ud558\uc600\ub2e4. \ud55c\ud3b8 \uc870\uc601\ub0a8\uc740 \uacb0\ud63c \ud6c4 \uc724\uc5ec\uc815 \uc0ac\uc774\uc5d0 2\ub0a8\uc744 \ub450\uc5c8\uc73c\ub098 \uc131\uaca9 \ucc28\uc774\uc640 \ubcf8\uc778\uc774 \uac00\uc815\uc5d0 \ucda9\uc2e4\ud558\uc9c0 \ubabb\ud55c \uc810\uc73c\ub85c \uacb0\uad6d 1987\ub144\uc5d0 \uc774\ud63c\uc744 \ud558\uace0 \ud55c\uad6d\uc73c\ub85c \ub3cc\uc544\uc628 \ud6c4 \uacf5\ubc31\uae30\ub97c \uac00\uc84c\ub2e4. \uacf5\ubc31 \uae30\uac04 \ub3d9\uc548 \uc870\uc601\ub0a8\uacfc \uac19\uc774 \uc9c0\ub0b4\uba70 \uc758\uc9c0\uac00 \ub418\uc5b4\uc8fc\uc5c8\ub358 \uae40\ud55c\uae38\uacfc \ud568\uaed8 \uc9c0\ub0c8\uc744 \ubb34\ub835, \uc5b4\ub290 \ub0a0 \uc2e0\ubb38\uc5d0 \ud654\uac1c\uc7a5\ud130 \uae30\uc0ac\ub97c \ubcf4\uace0 \uc18c\uc124\uac00\ub85c \ud65c\ub3d9\ud558\ub358 \uae40\ud55c\uae38\uc774 \uac00\uc0ac\ub97c \uc4f0\uac8c \ub418\uc5c8\ub2e4. \uadf8 \uac00\uc0ac\uc5d0 \uc870\uc601\ub0a8\uc774 \uc9c1\uc811 \uc791\uace1\ud55c \uba5c\ub85c\ub514\ub85c \ub178\ub798\ub97c \ub9cc\ub4e4\uc5c8\ub294\ub370 \uace1\uba85\uc744 \u3008\ud654\uac1c\uc7a5\ud130\u3009\ub85c \ud558\uace0 \uc568\ubc94\uc744 \ubc1c\ud45c\ud558\uc600\ub2e4. \ub370\ubdd4 \uc774\ud6c4 \ubc88\uc548\uace1\uc774\ub098 \uc131\uac00\ub97c \uc8fc\ub85c \ubd88\ub7ec \uad6d\ubbfc\ub4e4\uc5d0\uac8c \ub110\ub9ac \ubd88\ub9ac\ub294 \ud788\ud2b8\uace1\uc774 \uc5c6\uc5c8\ub294\ub370 \u3008\ud654\uac1c\uc7a5\ud130\u3009\uac00 \ud788\ud2b8\ud558\uba74\uc11c \uc870\uc601\ub0a8\uc744 TOP \uac00\uc218 \ubc18\uc5f4\uc5d0 \uc624\ub974\uac8c \ud558\uc5ec 80\ub144\ub300 \ucd5c\uace0\uc758 \uac00\uc694\ub85c \uc790\ub9ac\uc7a1\uc558\ub2e4. \ub610\ud55c \uc0ac\ub791\ub178\ub798 \u3008\uc0ac\ub791\uc5c6\uc778 \ub09c \ubabb\uc0b4\uc544\uc694\u3009\ub3c4 \uae40\ud55c\uae38\uc774 \uc791\uc0ac\ud558\uc600\uace0 \uc774 \uace1\ub3c4 \ud788\ud2b8\ud558\uc600\ub2e4. 1988\ub144\uc5d0 \ub450 \uace1\uc774\ub098 \ud788\ud2b8\ud558\uac8c \ub418\uc5b4 \uc120\ud48d\uc801\uc778 \uc778\uae30\ub97c \uc5bb\uace0 \uc815\uc0c1\uc5d0 \uc624\ub978 \uc870\uc601\ub0a8\uc740 \uc5ec\ub7ec \uacf5\uc5f0\uacfc \ubb34\ub300\uc5d0 \ucd9c\uc5f0\ud558\uc5ec \uc778\uae30\uac00\ub3c4\ub97c \ub2ec\ub9ac\uac8c \ub418\uc5c8\ub2e4.", "answer": "\ud55c\ud78c\uc0d8", "sentence": "1987\ub144\uc5d0\ub294 \uc8fc\uc2dc\uacbd \uc120\uc0dd\uc744 \uc874\uacbd\ud55c\ub2e4\ub294 \uc2e0\ub150\uc5d0 \ub530\ub77c \uc0ac\uadf9 MBC \ub4dc\ub77c\ub9c8 \u300a \ud55c\ud78c\uc0d8 \u300b(MBC \ud154\ub808\ube44\uc804 \ub4dc\ub77c\ub9c8)\uc5d0\ub3c4 \ucd9c\uc5f0\ud558\uc600\ub2e4.", "paragraph_sentence": "1982\ub144\uc5d0 \ub300\ud55c\ubbfc\uad6d\uc73c\ub85c \uadc0\uad6d\ud558\uc600\uace0 \uac00\uc218 \uae40\ub3c4\ud5a5\uacfc \ud568\uaed8 \uc74c\uc545\uc791\uc5c5\uc744 \ud558\uba74\uc11c \uadf8\uc640 \ud568\uaed8 \uc74c\ubc18\uc744 \ubc1c\ub9e4\ud558\uba70 \ud65c\ub3d9\uc744 \ud574\uc654\ub2e4. 1987\ub144\uc5d0\ub294 \uc8fc\uc2dc\uacbd \uc120\uc0dd\uc744 \uc874\uacbd\ud55c\ub2e4\ub294 \uc2e0\ub150\uc5d0 \ub530\ub77c \uc0ac\uadf9 MBC \ub4dc\ub77c\ub9c8 \u300a \ud55c\ud78c\uc0d8 \u300b(MBC \ud154\ub808\ube44\uc804 \ub4dc\ub77c\ub9c8)\uc5d0\ub3c4 \ucd9c\uc5f0\ud558\uc600\ub2e4. \ud55c\ud3b8 \uc870\uc601\ub0a8\uc740 \uacb0\ud63c \ud6c4 \uc724\uc5ec\uc815 \uc0ac\uc774\uc5d0 2\ub0a8\uc744 \ub450\uc5c8\uc73c\ub098 \uc131\uaca9 \ucc28\uc774\uc640 \ubcf8\uc778\uc774 \uac00\uc815\uc5d0 \ucda9\uc2e4\ud558\uc9c0 \ubabb\ud55c \uc810\uc73c\ub85c \uacb0\uad6d 1987\ub144\uc5d0 \uc774\ud63c\uc744 \ud558\uace0 \ud55c\uad6d\uc73c\ub85c \ub3cc\uc544\uc628 \ud6c4 \uacf5\ubc31\uae30\ub97c \uac00\uc84c\ub2e4. \uacf5\ubc31 \uae30\uac04 \ub3d9\uc548 \uc870\uc601\ub0a8\uacfc \uac19\uc774 \uc9c0\ub0b4\uba70 \uc758\uc9c0\uac00 \ub418\uc5b4\uc8fc\uc5c8\ub358 \uae40\ud55c\uae38\uacfc \ud568\uaed8 \uc9c0\ub0c8\uc744 \ubb34\ub835, \uc5b4\ub290 \ub0a0 \uc2e0\ubb38\uc5d0 \ud654\uac1c\uc7a5\ud130 \uae30\uc0ac\ub97c \ubcf4\uace0 \uc18c\uc124\uac00\ub85c \ud65c\ub3d9\ud558\ub358 \uae40\ud55c\uae38\uc774 \uac00\uc0ac\ub97c \uc4f0\uac8c \ub418\uc5c8\ub2e4. \uadf8 \uac00\uc0ac\uc5d0 \uc870\uc601\ub0a8\uc774 \uc9c1\uc811 \uc791\uace1\ud55c \uba5c\ub85c\ub514\ub85c \ub178\ub798\ub97c \ub9cc\ub4e4\uc5c8\ub294\ub370 \uace1\uba85\uc744 \u3008\ud654\uac1c\uc7a5\ud130\u3009\ub85c \ud558\uace0 \uc568\ubc94\uc744 \ubc1c\ud45c\ud558\uc600\ub2e4. \ub370\ubdd4 \uc774\ud6c4 \ubc88\uc548\uace1\uc774\ub098 \uc131\uac00\ub97c \uc8fc\ub85c \ubd88\ub7ec \uad6d\ubbfc\ub4e4\uc5d0\uac8c \ub110\ub9ac \ubd88\ub9ac\ub294 \ud788\ud2b8\uace1\uc774 \uc5c6\uc5c8\ub294\ub370 \u3008\ud654\uac1c\uc7a5\ud130\u3009\uac00 \ud788\ud2b8\ud558\uba74\uc11c \uc870\uc601\ub0a8\uc744 TOP \uac00\uc218 \ubc18\uc5f4\uc5d0 \uc624\ub974\uac8c \ud558\uc5ec 80\ub144\ub300 \ucd5c\uace0\uc758 \uac00\uc694\ub85c \uc790\ub9ac\uc7a1\uc558\ub2e4. \ub610\ud55c \uc0ac\ub791\ub178\ub798 \u3008\uc0ac\ub791\uc5c6\uc778 \ub09c \ubabb\uc0b4\uc544\uc694\u3009\ub3c4 \uae40\ud55c\uae38\uc774 \uc791\uc0ac\ud558\uc600\uace0 \uc774 \uace1\ub3c4 \ud788\ud2b8\ud558\uc600\ub2e4. 1988\ub144\uc5d0 \ub450 \uace1\uc774\ub098 \ud788\ud2b8\ud558\uac8c \ub418\uc5b4 \uc120\ud48d\uc801\uc778 \uc778\uae30\ub97c \uc5bb\uace0 \uc815\uc0c1\uc5d0 \uc624\ub978 \uc870\uc601\ub0a8\uc740 \uc5ec\ub7ec \uacf5\uc5f0\uacfc \ubb34\ub300\uc5d0 \ucd9c\uc5f0\ud558\uc5ec \uc778\uae30\uac00\ub3c4\ub97c \ub2ec\ub9ac\uac8c \ub418\uc5c8\ub2e4.", "paragraph_answer": "1982\ub144\uc5d0 \ub300\ud55c\ubbfc\uad6d\uc73c\ub85c \uadc0\uad6d\ud558\uc600\uace0 \uac00\uc218 \uae40\ub3c4\ud5a5\uacfc \ud568\uaed8 \uc74c\uc545\uc791\uc5c5\uc744 \ud558\uba74\uc11c \uadf8\uc640 \ud568\uaed8 \uc74c\ubc18\uc744 \ubc1c\ub9e4\ud558\uba70 \ud65c\ub3d9\uc744 \ud574\uc654\ub2e4. 1987\ub144\uc5d0\ub294 \uc8fc\uc2dc\uacbd \uc120\uc0dd\uc744 \uc874\uacbd\ud55c\ub2e4\ub294 \uc2e0\ub150\uc5d0 \ub530\ub77c \uc0ac\uadf9 MBC \ub4dc\ub77c\ub9c8 \u300a \ud55c\ud78c\uc0d8 \u300b(MBC \ud154\ub808\ube44\uc804 \ub4dc\ub77c\ub9c8)\uc5d0\ub3c4 \ucd9c\uc5f0\ud558\uc600\ub2e4. \ud55c\ud3b8 \uc870\uc601\ub0a8\uc740 \uacb0\ud63c \ud6c4 \uc724\uc5ec\uc815 \uc0ac\uc774\uc5d0 2\ub0a8\uc744 \ub450\uc5c8\uc73c\ub098 \uc131\uaca9 \ucc28\uc774\uc640 \ubcf8\uc778\uc774 \uac00\uc815\uc5d0 \ucda9\uc2e4\ud558\uc9c0 \ubabb\ud55c \uc810\uc73c\ub85c \uacb0\uad6d 1987\ub144\uc5d0 \uc774\ud63c\uc744 \ud558\uace0 \ud55c\uad6d\uc73c\ub85c \ub3cc\uc544\uc628 \ud6c4 \uacf5\ubc31\uae30\ub97c \uac00\uc84c\ub2e4. \uacf5\ubc31 \uae30\uac04 \ub3d9\uc548 \uc870\uc601\ub0a8\uacfc \uac19\uc774 \uc9c0\ub0b4\uba70 \uc758\uc9c0\uac00 \ub418\uc5b4\uc8fc\uc5c8\ub358 \uae40\ud55c\uae38\uacfc \ud568\uaed8 \uc9c0\ub0c8\uc744 \ubb34\ub835, \uc5b4\ub290 \ub0a0 \uc2e0\ubb38\uc5d0 \ud654\uac1c\uc7a5\ud130 \uae30\uc0ac\ub97c \ubcf4\uace0 \uc18c\uc124\uac00\ub85c \ud65c\ub3d9\ud558\ub358 \uae40\ud55c\uae38\uc774 \uac00\uc0ac\ub97c \uc4f0\uac8c \ub418\uc5c8\ub2e4. \uadf8 \uac00\uc0ac\uc5d0 \uc870\uc601\ub0a8\uc774 \uc9c1\uc811 \uc791\uace1\ud55c \uba5c\ub85c\ub514\ub85c \ub178\ub798\ub97c \ub9cc\ub4e4\uc5c8\ub294\ub370 \uace1\uba85\uc744 \u3008\ud654\uac1c\uc7a5\ud130\u3009\ub85c \ud558\uace0 \uc568\ubc94\uc744 \ubc1c\ud45c\ud558\uc600\ub2e4. \ub370\ubdd4 \uc774\ud6c4 \ubc88\uc548\uace1\uc774\ub098 \uc131\uac00\ub97c \uc8fc\ub85c \ubd88\ub7ec \uad6d\ubbfc\ub4e4\uc5d0\uac8c \ub110\ub9ac \ubd88\ub9ac\ub294 \ud788\ud2b8\uace1\uc774 \uc5c6\uc5c8\ub294\ub370 \u3008\ud654\uac1c\uc7a5\ud130\u3009\uac00 \ud788\ud2b8\ud558\uba74\uc11c \uc870\uc601\ub0a8\uc744 TOP \uac00\uc218 \ubc18\uc5f4\uc5d0 \uc624\ub974\uac8c \ud558\uc5ec 80\ub144\ub300 \ucd5c\uace0\uc758 \uac00\uc694\ub85c \uc790\ub9ac\uc7a1\uc558\ub2e4. \ub610\ud55c \uc0ac\ub791\ub178\ub798 \u3008\uc0ac\ub791\uc5c6\uc778 \ub09c \ubabb\uc0b4\uc544\uc694\u3009\ub3c4 \uae40\ud55c\uae38\uc774 \uc791\uc0ac\ud558\uc600\uace0 \uc774 \uace1\ub3c4 \ud788\ud2b8\ud558\uc600\ub2e4. 1988\ub144\uc5d0 \ub450 \uace1\uc774\ub098 \ud788\ud2b8\ud558\uac8c \ub418\uc5b4 \uc120\ud48d\uc801\uc778 \uc778\uae30\ub97c \uc5bb\uace0 \uc815\uc0c1\uc5d0 \uc624\ub978 \uc870\uc601\ub0a8\uc740 \uc5ec\ub7ec \uacf5\uc5f0\uacfc \ubb34\ub300\uc5d0 \ucd9c\uc5f0\ud558\uc5ec \uc778\uae30\uac00\ub3c4\ub97c \ub2ec\ub9ac\uac8c \ub418\uc5c8\ub2e4.", "sentence_answer": "1987\ub144\uc5d0\ub294 \uc8fc\uc2dc\uacbd \uc120\uc0dd\uc744 \uc874\uacbd\ud55c\ub2e4\ub294 \uc2e0\ub150\uc5d0 \ub530\ub77c \uc0ac\uadf9 MBC \ub4dc\ub77c\ub9c8 \u300a \ud55c\ud78c\uc0d8 \u300b(MBC \ud154\ub808\ube44\uc804 \ub4dc\ub77c\ub9c8)\uc5d0\ub3c4 \ucd9c\uc5f0\ud558\uc600\ub2e4.", "paragraph_id": "6553421-0-0", "paragraph_question": "question: \uc870\uc601\ub0a8\uc774 1987\ub144 \ucd9c\uc5f0\ud588\ub358 \uc0ac\uadf9 \ub4dc\ub77c\ub9c8\uc758 \uc81c\ubaa9\uc740?, context: 1982\ub144\uc5d0 \ub300\ud55c\ubbfc\uad6d\uc73c\ub85c \uadc0\uad6d\ud558\uc600\uace0 \uac00\uc218 \uae40\ub3c4\ud5a5\uacfc \ud568\uaed8 \uc74c\uc545\uc791\uc5c5\uc744 \ud558\uba74\uc11c \uadf8\uc640 \ud568\uaed8 \uc74c\ubc18\uc744 \ubc1c\ub9e4\ud558\uba70 \ud65c\ub3d9\uc744 \ud574\uc654\ub2e4. 1987\ub144\uc5d0\ub294 \uc8fc\uc2dc\uacbd \uc120\uc0dd\uc744 \uc874\uacbd\ud55c\ub2e4\ub294 \uc2e0\ub150\uc5d0 \ub530\ub77c \uc0ac\uadf9 MBC \ub4dc\ub77c\ub9c8 \u300a\ud55c\ud78c\uc0d8\u300b(MBC \ud154\ub808\ube44\uc804 \ub4dc\ub77c\ub9c8)\uc5d0\ub3c4 \ucd9c\uc5f0\ud558\uc600\ub2e4. \ud55c\ud3b8 \uc870\uc601\ub0a8\uc740 \uacb0\ud63c \ud6c4 \uc724\uc5ec\uc815 \uc0ac\uc774\uc5d0 2\ub0a8\uc744 \ub450\uc5c8\uc73c\ub098 \uc131\uaca9 \ucc28\uc774\uc640 \ubcf8\uc778\uc774 \uac00\uc815\uc5d0 \ucda9\uc2e4\ud558\uc9c0 \ubabb\ud55c \uc810\uc73c\ub85c \uacb0\uad6d 1987\ub144\uc5d0 \uc774\ud63c\uc744 \ud558\uace0 \ud55c\uad6d\uc73c\ub85c \ub3cc\uc544\uc628 \ud6c4 \uacf5\ubc31\uae30\ub97c \uac00\uc84c\ub2e4. \uacf5\ubc31 \uae30\uac04 \ub3d9\uc548 \uc870\uc601\ub0a8\uacfc \uac19\uc774 \uc9c0\ub0b4\uba70 \uc758\uc9c0\uac00 \ub418\uc5b4\uc8fc\uc5c8\ub358 \uae40\ud55c\uae38\uacfc \ud568\uaed8 \uc9c0\ub0c8\uc744 \ubb34\ub835, \uc5b4\ub290 \ub0a0 \uc2e0\ubb38\uc5d0 \ud654\uac1c\uc7a5\ud130 \uae30\uc0ac\ub97c \ubcf4\uace0 \uc18c\uc124\uac00\ub85c \ud65c\ub3d9\ud558\ub358 \uae40\ud55c\uae38\uc774 \uac00\uc0ac\ub97c \uc4f0\uac8c \ub418\uc5c8\ub2e4. \uadf8 \uac00\uc0ac\uc5d0 \uc870\uc601\ub0a8\uc774 \uc9c1\uc811 \uc791\uace1\ud55c \uba5c\ub85c\ub514\ub85c \ub178\ub798\ub97c \ub9cc\ub4e4\uc5c8\ub294\ub370 \uace1\uba85\uc744 \u3008\ud654\uac1c\uc7a5\ud130\u3009\ub85c \ud558\uace0 \uc568\ubc94\uc744 \ubc1c\ud45c\ud558\uc600\ub2e4. \ub370\ubdd4 \uc774\ud6c4 \ubc88\uc548\uace1\uc774\ub098 \uc131\uac00\ub97c \uc8fc\ub85c \ubd88\ub7ec \uad6d\ubbfc\ub4e4\uc5d0\uac8c \ub110\ub9ac \ubd88\ub9ac\ub294 \ud788\ud2b8\uace1\uc774 \uc5c6\uc5c8\ub294\ub370 \u3008\ud654\uac1c\uc7a5\ud130\u3009\uac00 \ud788\ud2b8\ud558\uba74\uc11c \uc870\uc601\ub0a8\uc744 TOP \uac00\uc218 \ubc18\uc5f4\uc5d0 \uc624\ub974\uac8c \ud558\uc5ec 80\ub144\ub300 \ucd5c\uace0\uc758 \uac00\uc694\ub85c \uc790\ub9ac\uc7a1\uc558\ub2e4. \ub610\ud55c \uc0ac\ub791\ub178\ub798 \u3008\uc0ac\ub791\uc5c6\uc778 \ub09c \ubabb\uc0b4\uc544\uc694\u3009\ub3c4 \uae40\ud55c\uae38\uc774 \uc791\uc0ac\ud558\uc600\uace0 \uc774 \uace1\ub3c4 \ud788\ud2b8\ud558\uc600\ub2e4. 1988\ub144\uc5d0 \ub450 \uace1\uc774\ub098 \ud788\ud2b8\ud558\uac8c \ub418\uc5b4 \uc120\ud48d\uc801\uc778 \uc778\uae30\ub97c \uc5bb\uace0 \uc815\uc0c1\uc5d0 \uc624\ub978 \uc870\uc601\ub0a8\uc740 \uc5ec\ub7ec \uacf5\uc5f0\uacfc \ubb34\ub300\uc5d0 \ucd9c\uc5f0\ud558\uc5ec \uc778\uae30\uac00\ub3c4\ub97c \ub2ec\ub9ac\uac8c \ub418\uc5c8\ub2e4."} {"question": "\uc548\ucca0\uc218\uac00 \ubca0\ub080 \uacf5\uc2dd \uc774\ub984\uc740?", "paragraph": "\uc548\ucca0\uc218\uc758 \ubc15\uc0ac\ud559\uc704 \ub17c\ubb38\uc774 \uc548\ucca0\uc218 \ub17c\ubb38\uc5d0 \ube44\ud574 2\ub144 \uc55e\uc11c \ubc15\uc0ac\ud559\uc704\ub97c \ubc1b\uc740 \uc11c\uc6b8\ub300\ud559\uad50 \uc11c \ubaa8 \uad50\uc218\uc758 \ubc15\uc0ac \ub17c\ubb38\uc744 \ud45c\uc808\ud588\ub2e4\ub294 \uc758\ud639\uc774 \uc788\ub2e4. \ubcfc\uce20\ub9cc \uace1\uc120\uc744 \uc720\ub3c4\ud558\ub294 \uc124\uba85\uc5d0\uc11c \uc720\ub3c4\uc2dd\uc744 \uc11c \uad50\uc218 \ub17c\ubb38\uc5d0\uc11c \uac70\uc758 \ubcf5\uc0ac \uc218\uc900\uc73c\ub85c 3\ud398\uc774\uc9c0\uc5d0 \uac78\uccd0 \ubca0\uaf08\ub2e4\ub294 \uac83\uc774\ub2e4. \ub354\uad70\ub2e4\ub098 \uc11c \uad50\uc218\uc758 \ub17c\ubb38\uc5d0\uc11c \uae30\uc220\ud55c \ubcfc\uce20\ub9cc \uacf5\uc2dd\uc740 \ub300\uad04\ud638\uac00 \ud558\ub098 \ube60\uc838\uc11c \uc624\ub958\uac00 \uc788\ub294 \uacf5\uc2dd\uc778\ub370, \uacf5\uad50\ub86d\uac8c\ub3c4 \uc548\ucca0\uc218\uc758 \ub17c\ubb38\uc5d0\uc11c\ub3c4 \uac19\uc740 \uacf5\uc2dd\uc5d0 \ub300\uad04\ud638\uac00 \ube60\uc838 \uc788\uc5c8\ub2e4. \ub530\ub77c\uc11c \ub17c\ubb38\uc744 \ubca0\ub07c\ub2e4\uac00 \uc624\ub958\uae4c\uc9c0 \ubca0\uaf08\ub2e4\ub294 \uc758\ud639\uc774 \uc788\ub2e4. \uadf8\ub7ec\ub098 \uc548\ucca0\uc218 \uce21\uc740 \ubcfc\uce20\ub9cc \uacf5\uc2dd\uc740 \ubcf4\ud3b8\uc801\uc73c\ub85c \uc54c\ub824\uc9c4 \uacf5\uc2dd\uc774\ub77c\uba70 \ub178\ubca8\uc0c1\uc744 \uc218\uc0c1\ud55c \ud649\ud0a8\uacfc \ud5c9\uc2ac\ub9ac\uc758 \ub17c\ubb38\uc5d0\uc11c\ub3c4 \ubcfc\uce20\ub9cc \uacf5\uc2dd\uc744 \uc0ac\uc6a9\ud558\ub294\ub370 \uc788\uc5b4\uc11c \uc6d0\uc800\ub97c \ubc1d\ud788\uc9c0 \uc54a\uc558\uace0, \uadf8\uac83\uc774 \uad00\ud589\uc774\ub77c\uace0 \ubc18\ubc15\ud558\uc600\ub2e4.", "answer": "\ubcfc\uce20\ub9cc \uacf5\uc2dd", "sentence": "\ub354\uad70\ub2e4\ub098 \uc11c \uad50\uc218\uc758 \ub17c\ubb38\uc5d0\uc11c \uae30\uc220\ud55c \ubcfc\uce20\ub9cc \uacf5\uc2dd \uc740 \ub300\uad04\ud638\uac00 \ud558\ub098 \ube60\uc838\uc11c \uc624\ub958\uac00 \uc788\ub294 \uacf5\uc2dd\uc778\ub370, \uacf5\uad50\ub86d\uac8c\ub3c4 \uc548\ucca0\uc218\uc758 \ub17c\ubb38\uc5d0\uc11c\ub3c4 \uac19\uc740 \uacf5\uc2dd\uc5d0 \ub300\uad04\ud638\uac00 \ube60\uc838 \uc788\uc5c8\ub2e4.", "paragraph_sentence": "\uc548\ucca0\uc218\uc758 \ubc15\uc0ac\ud559\uc704 \ub17c\ubb38\uc774 \uc548\ucca0\uc218 \ub17c\ubb38\uc5d0 \ube44\ud574 2\ub144 \uc55e\uc11c \ubc15\uc0ac\ud559\uc704\ub97c \ubc1b\uc740 \uc11c\uc6b8\ub300\ud559\uad50 \uc11c \ubaa8 \uad50\uc218\uc758 \ubc15\uc0ac \ub17c\ubb38\uc744 \ud45c\uc808\ud588\ub2e4\ub294 \uc758\ud639\uc774 \uc788\ub2e4. \ubcfc\uce20\ub9cc \uace1\uc120\uc744 \uc720\ub3c4\ud558\ub294 \uc124\uba85\uc5d0\uc11c \uc720\ub3c4\uc2dd\uc744 \uc11c \uad50\uc218 \ub17c\ubb38\uc5d0\uc11c \uac70\uc758 \ubcf5\uc0ac \uc218\uc900\uc73c\ub85c 3\ud398\uc774\uc9c0\uc5d0 \uac78\uccd0 \ubca0\uaf08\ub2e4\ub294 \uac83\uc774\ub2e4. \ub354\uad70\ub2e4\ub098 \uc11c \uad50\uc218\uc758 \ub17c\ubb38\uc5d0\uc11c \uae30\uc220\ud55c \ubcfc\uce20\ub9cc \uacf5\uc2dd \uc740 \ub300\uad04\ud638\uac00 \ud558\ub098 \ube60\uc838\uc11c \uc624\ub958\uac00 \uc788\ub294 \uacf5\uc2dd\uc778\ub370, \uacf5\uad50\ub86d\uac8c\ub3c4 \uc548\ucca0\uc218\uc758 \ub17c\ubb38\uc5d0\uc11c\ub3c4 \uac19\uc740 \uacf5\uc2dd\uc5d0 \ub300\uad04\ud638\uac00 \ube60\uc838 \uc788\uc5c8\ub2e4. \ub530\ub77c\uc11c \ub17c\ubb38\uc744 \ubca0\ub07c\ub2e4\uac00 \uc624\ub958\uae4c\uc9c0 \ubca0\uaf08\ub2e4\ub294 \uc758\ud639\uc774 \uc788\ub2e4. \uadf8\ub7ec\ub098 \uc548\ucca0\uc218 \uce21\uc740 \ubcfc\uce20\ub9cc \uacf5\uc2dd\uc740 \ubcf4\ud3b8\uc801\uc73c\ub85c \uc54c\ub824\uc9c4 \uacf5\uc2dd\uc774\ub77c\uba70 \ub178\ubca8\uc0c1\uc744 \uc218\uc0c1\ud55c \ud649\ud0a8\uacfc \ud5c9\uc2ac\ub9ac\uc758 \ub17c\ubb38\uc5d0\uc11c\ub3c4 \ubcfc\uce20\ub9cc \uacf5\uc2dd\uc744 \uc0ac\uc6a9\ud558\ub294\ub370 \uc788\uc5b4\uc11c \uc6d0\uc800\ub97c \ubc1d\ud788\uc9c0 \uc54a\uc558\uace0, \uadf8\uac83\uc774 \uad00\ud589\uc774\ub77c\uace0 \ubc18\ubc15\ud558\uc600\ub2e4.", "paragraph_answer": "\uc548\ucca0\uc218\uc758 \ubc15\uc0ac\ud559\uc704 \ub17c\ubb38\uc774 \uc548\ucca0\uc218 \ub17c\ubb38\uc5d0 \ube44\ud574 2\ub144 \uc55e\uc11c \ubc15\uc0ac\ud559\uc704\ub97c \ubc1b\uc740 \uc11c\uc6b8\ub300\ud559\uad50 \uc11c \ubaa8 \uad50\uc218\uc758 \ubc15\uc0ac \ub17c\ubb38\uc744 \ud45c\uc808\ud588\ub2e4\ub294 \uc758\ud639\uc774 \uc788\ub2e4. \ubcfc\uce20\ub9cc \uace1\uc120\uc744 \uc720\ub3c4\ud558\ub294 \uc124\uba85\uc5d0\uc11c \uc720\ub3c4\uc2dd\uc744 \uc11c \uad50\uc218 \ub17c\ubb38\uc5d0\uc11c \uac70\uc758 \ubcf5\uc0ac \uc218\uc900\uc73c\ub85c 3\ud398\uc774\uc9c0\uc5d0 \uac78\uccd0 \ubca0\uaf08\ub2e4\ub294 \uac83\uc774\ub2e4. \ub354\uad70\ub2e4\ub098 \uc11c \uad50\uc218\uc758 \ub17c\ubb38\uc5d0\uc11c \uae30\uc220\ud55c \ubcfc\uce20\ub9cc \uacf5\uc2dd \uc740 \ub300\uad04\ud638\uac00 \ud558\ub098 \ube60\uc838\uc11c \uc624\ub958\uac00 \uc788\ub294 \uacf5\uc2dd\uc778\ub370, \uacf5\uad50\ub86d\uac8c\ub3c4 \uc548\ucca0\uc218\uc758 \ub17c\ubb38\uc5d0\uc11c\ub3c4 \uac19\uc740 \uacf5\uc2dd\uc5d0 \ub300\uad04\ud638\uac00 \ube60\uc838 \uc788\uc5c8\ub2e4. \ub530\ub77c\uc11c \ub17c\ubb38\uc744 \ubca0\ub07c\ub2e4\uac00 \uc624\ub958\uae4c\uc9c0 \ubca0\uaf08\ub2e4\ub294 \uc758\ud639\uc774 \uc788\ub2e4. \uadf8\ub7ec\ub098 \uc548\ucca0\uc218 \uce21\uc740 \ubcfc\uce20\ub9cc \uacf5\uc2dd\uc740 \ubcf4\ud3b8\uc801\uc73c\ub85c \uc54c\ub824\uc9c4 \uacf5\uc2dd\uc774\ub77c\uba70 \ub178\ubca8\uc0c1\uc744 \uc218\uc0c1\ud55c \ud649\ud0a8\uacfc \ud5c9\uc2ac\ub9ac\uc758 \ub17c\ubb38\uc5d0\uc11c\ub3c4 \ubcfc\uce20\ub9cc \uacf5\uc2dd\uc744 \uc0ac\uc6a9\ud558\ub294\ub370 \uc788\uc5b4\uc11c \uc6d0\uc800\ub97c \ubc1d\ud788\uc9c0 \uc54a\uc558\uace0, \uadf8\uac83\uc774 \uad00\ud589\uc774\ub77c\uace0 \ubc18\ubc15\ud558\uc600\ub2e4.", "sentence_answer": "\ub354\uad70\ub2e4\ub098 \uc11c \uad50\uc218\uc758 \ub17c\ubb38\uc5d0\uc11c \uae30\uc220\ud55c \ubcfc\uce20\ub9cc \uacf5\uc2dd \uc740 \ub300\uad04\ud638\uac00 \ud558\ub098 \ube60\uc838\uc11c \uc624\ub958\uac00 \uc788\ub294 \uacf5\uc2dd\uc778\ub370, \uacf5\uad50\ub86d\uac8c\ub3c4 \uc548\ucca0\uc218\uc758 \ub17c\ubb38\uc5d0\uc11c\ub3c4 \uac19\uc740 \uacf5\uc2dd\uc5d0 \ub300\uad04\ud638\uac00 \ube60\uc838 \uc788\uc5c8\ub2e4.", "paragraph_id": "6547507-25-0", "paragraph_question": "question: \uc548\ucca0\uc218\uac00 \ubca0\ub080 \uacf5\uc2dd \uc774\ub984\uc740?, context: \uc548\ucca0\uc218\uc758 \ubc15\uc0ac\ud559\uc704 \ub17c\ubb38\uc774 \uc548\ucca0\uc218 \ub17c\ubb38\uc5d0 \ube44\ud574 2\ub144 \uc55e\uc11c \ubc15\uc0ac\ud559\uc704\ub97c \ubc1b\uc740 \uc11c\uc6b8\ub300\ud559\uad50 \uc11c \ubaa8 \uad50\uc218\uc758 \ubc15\uc0ac \ub17c\ubb38\uc744 \ud45c\uc808\ud588\ub2e4\ub294 \uc758\ud639\uc774 \uc788\ub2e4. \ubcfc\uce20\ub9cc \uace1\uc120\uc744 \uc720\ub3c4\ud558\ub294 \uc124\uba85\uc5d0\uc11c \uc720\ub3c4\uc2dd\uc744 \uc11c \uad50\uc218 \ub17c\ubb38\uc5d0\uc11c \uac70\uc758 \ubcf5\uc0ac \uc218\uc900\uc73c\ub85c 3\ud398\uc774\uc9c0\uc5d0 \uac78\uccd0 \ubca0\uaf08\ub2e4\ub294 \uac83\uc774\ub2e4. \ub354\uad70\ub2e4\ub098 \uc11c \uad50\uc218\uc758 \ub17c\ubb38\uc5d0\uc11c \uae30\uc220\ud55c \ubcfc\uce20\ub9cc \uacf5\uc2dd\uc740 \ub300\uad04\ud638\uac00 \ud558\ub098 \ube60\uc838\uc11c \uc624\ub958\uac00 \uc788\ub294 \uacf5\uc2dd\uc778\ub370, \uacf5\uad50\ub86d\uac8c\ub3c4 \uc548\ucca0\uc218\uc758 \ub17c\ubb38\uc5d0\uc11c\ub3c4 \uac19\uc740 \uacf5\uc2dd\uc5d0 \ub300\uad04\ud638\uac00 \ube60\uc838 \uc788\uc5c8\ub2e4. \ub530\ub77c\uc11c \ub17c\ubb38\uc744 \ubca0\ub07c\ub2e4\uac00 \uc624\ub958\uae4c\uc9c0 \ubca0\uaf08\ub2e4\ub294 \uc758\ud639\uc774 \uc788\ub2e4. \uadf8\ub7ec\ub098 \uc548\ucca0\uc218 \uce21\uc740 \ubcfc\uce20\ub9cc \uacf5\uc2dd\uc740 \ubcf4\ud3b8\uc801\uc73c\ub85c \uc54c\ub824\uc9c4 \uacf5\uc2dd\uc774\ub77c\uba70 \ub178\ubca8\uc0c1\uc744 \uc218\uc0c1\ud55c \ud649\ud0a8\uacfc \ud5c9\uc2ac\ub9ac\uc758 \ub17c\ubb38\uc5d0\uc11c\ub3c4 \ubcfc\uce20\ub9cc \uacf5\uc2dd\uc744 \uc0ac\uc6a9\ud558\ub294\ub370 \uc788\uc5b4\uc11c \uc6d0\uc800\ub97c \ubc1d\ud788\uc9c0 \uc54a\uc558\uace0, \uadf8\uac83\uc774 \uad00\ud589\uc774\ub77c\uace0 \ubc18\ubc15\ud558\uc600\ub2e4."} {"question": "\uc2dc\ubbfc\ud3c9\ud654\ubc95\uc815 \uce21\uc774 \ud2b9\ubcc4\ubc95\uc73c\ub85c \ud3d0\uc9c0\ub97c \uc6d0\ud588\ub358 \uac83\uc740?", "paragraph": "2018\ub144 4\uc6d4 22\uc77c - 23\uc77c \ubbfc\uc8fc\uc0ac\ud68c\ub97c \uc704\ud55c \ubcc0\ud638\uc0ac \ubaa8\uc784\uacfc \ud55c\ubca0\ud3c9\ud654\uc7ac\ub2e8\uc744 \ube44\ub86f\ud55c \ub300\ud55c\ubbfc\uad6d\uc758 \uc5ec\ub7ec \uc2dc\ubbfc\uc0ac\ud68c \ub2e8\uccb4\ub4e4\uc740 \ubca0\ud2b8\ub0a8\uc804\uc7c1 \uc2dc\uae30 \ud55c\uad6d\uad70\uc5d0 \uc758\ud55c \ubbfc\uac04\uc778 \ud559\uc0b4 \uc9c4\uc0c1\uaddc\uba85\uc744 \uc704\ud55c \uc2dc\ubbfc\ud3c9\ud654\ubc95\uc815\uc744 \uac1c\ucd5c\ud558\uc600\ub2e4. \uc774 \uc2dc\ubbfc\ud3c9\ud654\ubc95\uc815\uc740 \ud401\ub2c8\u00b7\ud401\ub11b \ub9c8\uc744 \ud559\uc0b4 \uc0ac\uac74\uc758 \uc0dd\uc874\uc790\uc778 \uc751\uc6b0\uc60c \ud2f0 \ud0c4\uacfc \ud558\ubbf8 \ub9c8\uc744 \ud559\uc0b4 \uc0ac\uac74\uc758 \uc0dd\uc874\uc790 \uc751\uc6b0\uc60c \ud2f0\ud0c4(\ub3d9\uba85\uc774\uc778)\uc774 \uc6d0\uace0\ub85c \ub098\uc11c \ub300\ud55c\ubbfc\uad6d \uc815\ubd80\ub97c \ud53c\uace0\ub85c \ud558\uc5ec \uc190\ud574\ubc30\uc0c1\uc744 \uccad\uad6c\ud558\uc600\uc73c\uba70, \uc2dc\ubbfc\ud3c9\ud654\ubc95\uc815\uc758 \uc7ac\ud310\ubd80\ub294 \uc774\ub4e4\uc758 \uc8fc\uc7a5\uc744 \ubc1b\uc544\ub4e4\uc5ec \uad6d\uac00\ubc30\uc0c1\ubc95\uc5d0 \ub530\ub978 \ubc30\uc0c1\uc744 \ud310\uacb0\ud558\uace0 \ub300\ud55c\ubbfc\uad6d \uc815\ubd80\uc5d0 \uc9c4\uc0c1\uc870\uc0ac\ub97c \uad8c\uace0\ud558\uc600\ub2e4. \uc2dc\ubbfc\ud3c9\ud654\ubc95\uc815\uc740 \ubaa8\uc758\ubc95\uc815\uc73c\ub85c \ubc95\uc801 \uad6c\uc18d\ub825\uc774 \uc5c6\uc73c\ub098 \ubca0\ud2b8\ub0a8 \uc804\uc7c1\uc2dc\uae30 \ud559\uc0b4\uc744 \ubcf4\ud3b8\uc801 \uc778\uad8c\uc758 \ubb38\uc81c\ub85c\uc11c \ucc9c\uba85\ud558\uc600\ub2e4\ub294 \uc758\uc758\uac00 \uc788\ub2e4.\uc2dc\ubbfc\ud3c9\ud654\ubc95\uc815 \uce21\uc740 \uacf5\uc18c \uc2dc\ud6a8\ub97c \uc5c6\uc560\ub294 \ud2b9\ubcc4\ubc95\uc774 \ud544\uc694\ud558\ub2e4\uace0 \uc8fc\uc7a5\ud558\uc600\ub2e4. \ub610\ud55c, \ubbfc\ubcc0\uc740 1968\ub144 \ub2f9\uc2dc \ud401\ub2c8 \ud401\ub11b \ub9c8\uc744 \uc0ac\uac74\uc744 \uc870\uc0ac\ud558\uc600\ub358 \uc911\uc559\uc815\ubcf4\ubd80\uc758 \uc790\ub8cc \uacf5\uac1c\ub97c \uccad\uad6c\ud558\ub294 \uc18c\uc1a1\uc744 \uc900\ube44\uc911\uc774\ub2e4.", "answer": "\uacf5\uc18c \uc2dc\ud6a8", "sentence": "\uc2dc\ubbfc\ud3c9\ud654\ubc95\uc815\uc740 \ubaa8\uc758\ubc95\uc815\uc73c\ub85c \ubc95\uc801 \uad6c\uc18d\ub825\uc774 \uc5c6\uc73c\ub098 \ubca0\ud2b8\ub0a8 \uc804\uc7c1\uc2dc\uae30 \ud559\uc0b4\uc744 \ubcf4\ud3b8\uc801 \uc778\uad8c\uc758 \ubb38\uc81c\ub85c\uc11c \ucc9c\uba85\ud558\uc600\ub2e4\ub294 \uc758\uc758\uac00 \uc788\ub2e4.\uc2dc\ubbfc\ud3c9\ud654\ubc95\uc815 \uce21\uc740 \uacf5\uc18c \uc2dc\ud6a8 \ub97c \uc5c6\uc560\ub294 \ud2b9\ubcc4\ubc95\uc774 \ud544\uc694\ud558\ub2e4\uace0 \uc8fc\uc7a5\ud558\uc600\ub2e4.", "paragraph_sentence": "2018\ub144 4\uc6d4 22\uc77c - 23\uc77c \ubbfc\uc8fc\uc0ac\ud68c\ub97c \uc704\ud55c \ubcc0\ud638\uc0ac \ubaa8\uc784\uacfc \ud55c\ubca0\ud3c9\ud654\uc7ac\ub2e8\uc744 \ube44\ub86f\ud55c \ub300\ud55c\ubbfc\uad6d\uc758 \uc5ec\ub7ec \uc2dc\ubbfc\uc0ac\ud68c \ub2e8\uccb4\ub4e4\uc740 \ubca0\ud2b8\ub0a8\uc804\uc7c1 \uc2dc\uae30 \ud55c\uad6d\uad70\uc5d0 \uc758\ud55c \ubbfc\uac04\uc778 \ud559\uc0b4 \uc9c4\uc0c1\uaddc\uba85\uc744 \uc704\ud55c \uc2dc\ubbfc\ud3c9\ud654\ubc95\uc815\uc744 \uac1c\ucd5c\ud558\uc600\ub2e4. \uc774 \uc2dc\ubbfc\ud3c9\ud654\ubc95\uc815\uc740 \ud401\ub2c8\u00b7\ud401\ub11b \ub9c8\uc744 \ud559\uc0b4 \uc0ac\uac74\uc758 \uc0dd\uc874\uc790\uc778 \uc751\uc6b0\uc60c \ud2f0 \ud0c4\uacfc \ud558\ubbf8 \ub9c8\uc744 \ud559\uc0b4 \uc0ac\uac74\uc758 \uc0dd\uc874\uc790 \uc751\uc6b0\uc60c \ud2f0\ud0c4(\ub3d9\uba85\uc774\uc778)\uc774 \uc6d0\uace0\ub85c \ub098\uc11c \ub300\ud55c\ubbfc\uad6d \uc815\ubd80\ub97c \ud53c\uace0\ub85c \ud558\uc5ec \uc190\ud574\ubc30\uc0c1\uc744 \uccad\uad6c\ud558\uc600\uc73c\uba70, \uc2dc\ubbfc\ud3c9\ud654\ubc95\uc815\uc758 \uc7ac\ud310\ubd80\ub294 \uc774\ub4e4\uc758 \uc8fc\uc7a5\uc744 \ubc1b\uc544\ub4e4\uc5ec \uad6d\uac00\ubc30\uc0c1\ubc95\uc5d0 \ub530\ub978 \ubc30\uc0c1\uc744 \ud310\uacb0\ud558\uace0 \ub300\ud55c\ubbfc\uad6d \uc815\ubd80\uc5d0 \uc9c4\uc0c1\uc870\uc0ac\ub97c \uad8c\uace0\ud558\uc600\ub2e4. \uc2dc\ubbfc\ud3c9\ud654\ubc95\uc815\uc740 \ubaa8\uc758\ubc95\uc815\uc73c\ub85c \ubc95\uc801 \uad6c\uc18d\ub825\uc774 \uc5c6\uc73c\ub098 \ubca0\ud2b8\ub0a8 \uc804\uc7c1\uc2dc\uae30 \ud559\uc0b4\uc744 \ubcf4\ud3b8\uc801 \uc778\uad8c\uc758 \ubb38\uc81c\ub85c\uc11c \ucc9c\uba85\ud558\uc600\ub2e4\ub294 \uc758\uc758\uac00 \uc788\ub2e4.\uc2dc\ubbfc\ud3c9\ud654\ubc95\uc815 \uce21\uc740 \uacf5\uc18c \uc2dc\ud6a8 \ub97c \uc5c6\uc560\ub294 \ud2b9\ubcc4\ubc95\uc774 \ud544\uc694\ud558\ub2e4\uace0 \uc8fc\uc7a5\ud558\uc600\ub2e4. \ub610\ud55c, \ubbfc\ubcc0\uc740 1968\ub144 \ub2f9\uc2dc \ud401\ub2c8 \ud401\ub11b \ub9c8\uc744 \uc0ac\uac74\uc744 \uc870\uc0ac\ud558\uc600\ub358 \uc911\uc559\uc815\ubcf4\ubd80\uc758 \uc790\ub8cc \uacf5\uac1c\ub97c \uccad\uad6c\ud558\ub294 \uc18c\uc1a1\uc744 \uc900\ube44\uc911\uc774\ub2e4.", "paragraph_answer": "2018\ub144 4\uc6d4 22\uc77c - 23\uc77c \ubbfc\uc8fc\uc0ac\ud68c\ub97c \uc704\ud55c \ubcc0\ud638\uc0ac \ubaa8\uc784\uacfc \ud55c\ubca0\ud3c9\ud654\uc7ac\ub2e8\uc744 \ube44\ub86f\ud55c \ub300\ud55c\ubbfc\uad6d\uc758 \uc5ec\ub7ec \uc2dc\ubbfc\uc0ac\ud68c \ub2e8\uccb4\ub4e4\uc740 \ubca0\ud2b8\ub0a8\uc804\uc7c1 \uc2dc\uae30 \ud55c\uad6d\uad70\uc5d0 \uc758\ud55c \ubbfc\uac04\uc778 \ud559\uc0b4 \uc9c4\uc0c1\uaddc\uba85\uc744 \uc704\ud55c \uc2dc\ubbfc\ud3c9\ud654\ubc95\uc815\uc744 \uac1c\ucd5c\ud558\uc600\ub2e4. \uc774 \uc2dc\ubbfc\ud3c9\ud654\ubc95\uc815\uc740 \ud401\ub2c8\u00b7\ud401\ub11b \ub9c8\uc744 \ud559\uc0b4 \uc0ac\uac74\uc758 \uc0dd\uc874\uc790\uc778 \uc751\uc6b0\uc60c \ud2f0 \ud0c4\uacfc \ud558\ubbf8 \ub9c8\uc744 \ud559\uc0b4 \uc0ac\uac74\uc758 \uc0dd\uc874\uc790 \uc751\uc6b0\uc60c \ud2f0\ud0c4(\ub3d9\uba85\uc774\uc778)\uc774 \uc6d0\uace0\ub85c \ub098\uc11c \ub300\ud55c\ubbfc\uad6d \uc815\ubd80\ub97c \ud53c\uace0\ub85c \ud558\uc5ec \uc190\ud574\ubc30\uc0c1\uc744 \uccad\uad6c\ud558\uc600\uc73c\uba70, \uc2dc\ubbfc\ud3c9\ud654\ubc95\uc815\uc758 \uc7ac\ud310\ubd80\ub294 \uc774\ub4e4\uc758 \uc8fc\uc7a5\uc744 \ubc1b\uc544\ub4e4\uc5ec \uad6d\uac00\ubc30\uc0c1\ubc95\uc5d0 \ub530\ub978 \ubc30\uc0c1\uc744 \ud310\uacb0\ud558\uace0 \ub300\ud55c\ubbfc\uad6d \uc815\ubd80\uc5d0 \uc9c4\uc0c1\uc870\uc0ac\ub97c \uad8c\uace0\ud558\uc600\ub2e4. \uc2dc\ubbfc\ud3c9\ud654\ubc95\uc815\uc740 \ubaa8\uc758\ubc95\uc815\uc73c\ub85c \ubc95\uc801 \uad6c\uc18d\ub825\uc774 \uc5c6\uc73c\ub098 \ubca0\ud2b8\ub0a8 \uc804\uc7c1\uc2dc\uae30 \ud559\uc0b4\uc744 \ubcf4\ud3b8\uc801 \uc778\uad8c\uc758 \ubb38\uc81c\ub85c\uc11c \ucc9c\uba85\ud558\uc600\ub2e4\ub294 \uc758\uc758\uac00 \uc788\ub2e4.\uc2dc\ubbfc\ud3c9\ud654\ubc95\uc815 \uce21\uc740 \uacf5\uc18c \uc2dc\ud6a8 \ub97c \uc5c6\uc560\ub294 \ud2b9\ubcc4\ubc95\uc774 \ud544\uc694\ud558\ub2e4\uace0 \uc8fc\uc7a5\ud558\uc600\ub2e4. \ub610\ud55c, \ubbfc\ubcc0\uc740 1968\ub144 \ub2f9\uc2dc \ud401\ub2c8 \ud401\ub11b \ub9c8\uc744 \uc0ac\uac74\uc744 \uc870\uc0ac\ud558\uc600\ub358 \uc911\uc559\uc815\ubcf4\ubd80\uc758 \uc790\ub8cc \uacf5\uac1c\ub97c \uccad\uad6c\ud558\ub294 \uc18c\uc1a1\uc744 \uc900\ube44\uc911\uc774\ub2e4.", "sentence_answer": "\uc2dc\ubbfc\ud3c9\ud654\ubc95\uc815\uc740 \ubaa8\uc758\ubc95\uc815\uc73c\ub85c \ubc95\uc801 \uad6c\uc18d\ub825\uc774 \uc5c6\uc73c\ub098 \ubca0\ud2b8\ub0a8 \uc804\uc7c1\uc2dc\uae30 \ud559\uc0b4\uc744 \ubcf4\ud3b8\uc801 \uc778\uad8c\uc758 \ubb38\uc81c\ub85c\uc11c \ucc9c\uba85\ud558\uc600\ub2e4\ub294 \uc758\uc758\uac00 \uc788\ub2e4.\uc2dc\ubbfc\ud3c9\ud654\ubc95\uc815 \uce21\uc740 \uacf5\uc18c \uc2dc\ud6a8 \ub97c \uc5c6\uc560\ub294 \ud2b9\ubcc4\ubc95\uc774 \ud544\uc694\ud558\ub2e4\uace0 \uc8fc\uc7a5\ud558\uc600\ub2e4.", "paragraph_id": "6221612-4-2", "paragraph_question": "question: \uc2dc\ubbfc\ud3c9\ud654\ubc95\uc815 \uce21\uc774 \ud2b9\ubcc4\ubc95\uc73c\ub85c \ud3d0\uc9c0\ub97c \uc6d0\ud588\ub358 \uac83\uc740?, context: 2018\ub144 4\uc6d4 22\uc77c - 23\uc77c \ubbfc\uc8fc\uc0ac\ud68c\ub97c \uc704\ud55c \ubcc0\ud638\uc0ac \ubaa8\uc784\uacfc \ud55c\ubca0\ud3c9\ud654\uc7ac\ub2e8\uc744 \ube44\ub86f\ud55c \ub300\ud55c\ubbfc\uad6d\uc758 \uc5ec\ub7ec \uc2dc\ubbfc\uc0ac\ud68c \ub2e8\uccb4\ub4e4\uc740 \ubca0\ud2b8\ub0a8\uc804\uc7c1 \uc2dc\uae30 \ud55c\uad6d\uad70\uc5d0 \uc758\ud55c \ubbfc\uac04\uc778 \ud559\uc0b4 \uc9c4\uc0c1\uaddc\uba85\uc744 \uc704\ud55c \uc2dc\ubbfc\ud3c9\ud654\ubc95\uc815\uc744 \uac1c\ucd5c\ud558\uc600\ub2e4. \uc774 \uc2dc\ubbfc\ud3c9\ud654\ubc95\uc815\uc740 \ud401\ub2c8\u00b7\ud401\ub11b \ub9c8\uc744 \ud559\uc0b4 \uc0ac\uac74\uc758 \uc0dd\uc874\uc790\uc778 \uc751\uc6b0\uc60c \ud2f0 \ud0c4\uacfc \ud558\ubbf8 \ub9c8\uc744 \ud559\uc0b4 \uc0ac\uac74\uc758 \uc0dd\uc874\uc790 \uc751\uc6b0\uc60c \ud2f0\ud0c4(\ub3d9\uba85\uc774\uc778)\uc774 \uc6d0\uace0\ub85c \ub098\uc11c \ub300\ud55c\ubbfc\uad6d \uc815\ubd80\ub97c \ud53c\uace0\ub85c \ud558\uc5ec \uc190\ud574\ubc30\uc0c1\uc744 \uccad\uad6c\ud558\uc600\uc73c\uba70, \uc2dc\ubbfc\ud3c9\ud654\ubc95\uc815\uc758 \uc7ac\ud310\ubd80\ub294 \uc774\ub4e4\uc758 \uc8fc\uc7a5\uc744 \ubc1b\uc544\ub4e4\uc5ec \uad6d\uac00\ubc30\uc0c1\ubc95\uc5d0 \ub530\ub978 \ubc30\uc0c1\uc744 \ud310\uacb0\ud558\uace0 \ub300\ud55c\ubbfc\uad6d \uc815\ubd80\uc5d0 \uc9c4\uc0c1\uc870\uc0ac\ub97c \uad8c\uace0\ud558\uc600\ub2e4. \uc2dc\ubbfc\ud3c9\ud654\ubc95\uc815\uc740 \ubaa8\uc758\ubc95\uc815\uc73c\ub85c \ubc95\uc801 \uad6c\uc18d\ub825\uc774 \uc5c6\uc73c\ub098 \ubca0\ud2b8\ub0a8 \uc804\uc7c1\uc2dc\uae30 \ud559\uc0b4\uc744 \ubcf4\ud3b8\uc801 \uc778\uad8c\uc758 \ubb38\uc81c\ub85c\uc11c \ucc9c\uba85\ud558\uc600\ub2e4\ub294 \uc758\uc758\uac00 \uc788\ub2e4.\uc2dc\ubbfc\ud3c9\ud654\ubc95\uc815 \uce21\uc740 \uacf5\uc18c \uc2dc\ud6a8\ub97c \uc5c6\uc560\ub294 \ud2b9\ubcc4\ubc95\uc774 \ud544\uc694\ud558\ub2e4\uace0 \uc8fc\uc7a5\ud558\uc600\ub2e4. \ub610\ud55c, \ubbfc\ubcc0\uc740 1968\ub144 \ub2f9\uc2dc \ud401\ub2c8 \ud401\ub11b \ub9c8\uc744 \uc0ac\uac74\uc744 \uc870\uc0ac\ud558\uc600\ub358 \uc911\uc559\uc815\ubcf4\ubd80\uc758 \uc790\ub8cc \uacf5\uac1c\ub97c \uccad\uad6c\ud558\ub294 \uc18c\uc1a1\uc744 \uc900\ube44\uc911\uc774\ub2e4."} {"question": "\ube14\ub808\uc774\ud06c \ub77c\uc774\ube14\ub9ac\uac00 2012\ub144 \uc140\ub9c8 \ud5e4\uc774\uc5d1\uacfc \uc874 \ud2b8\ub77c\ubcfc\ud0c0 \ub4f1\uacfc \ucd9c\uc5f0\ud55c \uc62c\ub9ac\ubc84 \uc2a4\ud1a4 \uac10\ub3c5\uc791\uc740?", "paragraph": "2011\ub144, \uadf8\ub140\ub294 \ud0c0\uc784\uc988 \ub9e4\uac70\uc9c4 \u2018\uac00\uc7a5 \uc601\ud5a5\ub825 \uc788\ub294 100\uc778\u2019\uc5d0 \uc120\uc815 \ub418\uc5c8\ub2e4. \ub610\ud55c AskMen.com\uc740 \uadf8\ub140\ub97c 2011\ub144 \uac00\uc7a5 \uc139\uc2dc\ud55c \uc5ec\uc131\uc73c\ub85c \uc120\uc815\ud588\uace0, \ud53c\ud50c \ub9e4\uac70\uc9c4\uc5d0\uc11c\ub294 2012\ub144 \uac00\uc7a5 \uc544\ub984\ub2e4\uc6b4 \ubaa8\ub4e0 \ub098\uc774 \uc911 \ud558\ub098\ub85c \uc120\uc815\ud588\ub2e4. 2012\ub144, \uadf8\ub140\ub294 \uc62c\ub9ac\ubc84 \uc2a4\ud1a4 \uac10\ub3c5\uc758 \u300a\ud30c\uad34\uc790\ub4e4\u300b(\uc6d0\uc81c: Savages)\uc5d0\uc11c \ud14c\uc77c\ub7ec \ud0a4\ucde8, \uc544\ub860 \uc874\uc2a8, \uc140\ub9c8 \ud5e4\uc774\uc5d1\uacfc \uc874 \ud2b8\ub77c\ubcfc\ud0c0\uc640 \uacf5\uc5f0\ud588\ub2e4. \ub2f9\ucd08 \uc624\ud544\ub9ac\uc544 \"\uc624\" \uc138\uc774\uc9c0' \uc5ed\ud560\uc5d0\ub294 \uc81c\ub2c8\ud37c \ub85c\ub80c\uc2a4\uac00 \uce90\uc2a4\ud305\ub418\uc5c8\uc9c0\ub9cc \u300a\ud5dd\uac70\uac8c\uc784: \ud310\uc5e0\uc758 \ubd88\uaf43\u300b\uc758 \uc77c\uc815 \ubb38\uc81c\ub85c \ubb34\uc0b0 \ub418\uba74\uc11c \ub77c\uc774\ube14\ub9ac\uac00 \ub300\uccb4\ud588\ub2e4. HitFix\uc758 \uc601\ud654 \ud3c9\ub860\uac00 \ub4dc\ub958 \ub9e5\uc704\ub2c8\ub294 \ub77c\uc774\ube14\ub9ac\uc758 \uc5f0\uae30\ub97c \uce6d\ucc2c\ud558\uba70, \uadf8\ub294 \u201c\uadf8\ub140\uac00 \"\uc624\"\ub97c \uac00\uc2b4 \uc544\ud504\uace0 \uc791\uc740 \uc18c\uc6b8\ub85c \uc5f0\uae30\ud588\uae30\uc5d0 \uc601\ub9ac\ud558\uace0 \ud655\uc2e4\ud788 \uc2ac\ud390\ub2e4\u201d\ub77c\uace0 \ud3c9\ud588\ub2e4. \uac19\uc740 \ud574 \uadf8\ub140\ub294 \uc0c8\ub85c\uc6b4 \uad6c\ucc0c \ud5a5\uc218\uc778 \u2018Gucci Premiere\u2019\uc758 \ubba4\uc988\ub85c \uc120\uc815 \ub418\uc5c8\uace0, \uadf8\ub140\ub294 \ud5a5\uc218\uc5d0 \ub300\ud55c \ub2c8\ucf5c\ub77c\uc2a4 \uc708\ub529 \ub808\ud508 \uac10\ub3c5\uc758 \ub2e8\ud3b8 \uc601\ud654 \uad11\uace0\uc5d0 \ucd9c\uc5f0\ud588\ub2e4.", "answer": "\ud30c\uad34\uc790\ub4e4", "sentence": "2012\ub144, \uadf8\ub140\ub294 \uc62c\ub9ac\ubc84 \uc2a4\ud1a4 \uac10\ub3c5\uc758 \u300a \ud30c\uad34\uc790\ub4e4 \u300b(\uc6d0\uc81c: Savages)\uc5d0\uc11c \ud14c\uc77c\ub7ec \ud0a4\ucde8, \uc544\ub860 \uc874\uc2a8, \uc140\ub9c8 \ud5e4\uc774\uc5d1\uacfc \uc874 \ud2b8\ub77c\ubcfc\ud0c0\uc640 \uacf5\uc5f0\ud588\ub2e4.", "paragraph_sentence": "2011\ub144, \uadf8\ub140\ub294 \ud0c0\uc784\uc988 \ub9e4\uac70\uc9c4 \u2018\uac00\uc7a5 \uc601\ud5a5\ub825 \uc788\ub294 100\uc778\u2019\uc5d0 \uc120\uc815 \ub418\uc5c8\ub2e4. \ub610\ud55c AskMen.com\uc740 \uadf8\ub140\ub97c 2011\ub144 \uac00\uc7a5 \uc139\uc2dc\ud55c \uc5ec\uc131\uc73c\ub85c \uc120\uc815\ud588\uace0, \ud53c\ud50c \ub9e4\uac70\uc9c4\uc5d0\uc11c\ub294 2012\ub144 \uac00\uc7a5 \uc544\ub984\ub2e4\uc6b4 \ubaa8\ub4e0 \ub098\uc774 \uc911 \ud558\ub098\ub85c \uc120\uc815\ud588\ub2e4. 2012\ub144, \uadf8\ub140\ub294 \uc62c\ub9ac\ubc84 \uc2a4\ud1a4 \uac10\ub3c5\uc758 \u300a \ud30c\uad34\uc790\ub4e4 \u300b(\uc6d0\uc81c: Savages)\uc5d0\uc11c \ud14c\uc77c\ub7ec \ud0a4\ucde8, \uc544\ub860 \uc874\uc2a8, \uc140\ub9c8 \ud5e4\uc774\uc5d1\uacfc \uc874 \ud2b8\ub77c\ubcfc\ud0c0\uc640 \uacf5\uc5f0\ud588\ub2e4. \ub2f9\ucd08 \uc624\ud544\ub9ac\uc544 \"\uc624\" \uc138\uc774\uc9c0' \uc5ed\ud560\uc5d0\ub294 \uc81c\ub2c8\ud37c \ub85c\ub80c\uc2a4\uac00 \uce90\uc2a4\ud305\ub418\uc5c8\uc9c0\ub9cc \u300a\ud5dd\uac70\uac8c\uc784: \ud310\uc5e0\uc758 \ubd88\uaf43\u300b\uc758 \uc77c\uc815 \ubb38\uc81c\ub85c \ubb34\uc0b0 \ub418\uba74\uc11c \ub77c\uc774\ube14\ub9ac\uac00 \ub300\uccb4\ud588\ub2e4. HitFix\uc758 \uc601\ud654 \ud3c9\ub860\uac00 \ub4dc\ub958 \ub9e5\uc704\ub2c8\ub294 \ub77c\uc774\ube14\ub9ac\uc758 \uc5f0\uae30\ub97c \uce6d\ucc2c\ud558\uba70, \uadf8\ub294 \u201c\uadf8\ub140\uac00 \"\uc624\"\ub97c \uac00\uc2b4 \uc544\ud504\uace0 \uc791\uc740 \uc18c\uc6b8\ub85c \uc5f0\uae30\ud588\uae30\uc5d0 \uc601\ub9ac\ud558\uace0 \ud655\uc2e4\ud788 \uc2ac\ud390\ub2e4\u201d\ub77c\uace0 \ud3c9\ud588\ub2e4. \uac19\uc740 \ud574 \uadf8\ub140\ub294 \uc0c8\ub85c\uc6b4 \uad6c\ucc0c \ud5a5\uc218\uc778 \u2018Gucci Premiere\u2019\uc758 \ubba4\uc988\ub85c \uc120\uc815 \ub418\uc5c8\uace0, \uadf8\ub140\ub294 \ud5a5\uc218\uc5d0 \ub300\ud55c \ub2c8\ucf5c\ub77c\uc2a4 \uc708\ub529 \ub808\ud508 \uac10\ub3c5\uc758 \ub2e8\ud3b8 \uc601\ud654 \uad11\uace0\uc5d0 \ucd9c\uc5f0\ud588\ub2e4.", "paragraph_answer": "2011\ub144, \uadf8\ub140\ub294 \ud0c0\uc784\uc988 \ub9e4\uac70\uc9c4 \u2018\uac00\uc7a5 \uc601\ud5a5\ub825 \uc788\ub294 100\uc778\u2019\uc5d0 \uc120\uc815 \ub418\uc5c8\ub2e4. \ub610\ud55c AskMen.com\uc740 \uadf8\ub140\ub97c 2011\ub144 \uac00\uc7a5 \uc139\uc2dc\ud55c \uc5ec\uc131\uc73c\ub85c \uc120\uc815\ud588\uace0, \ud53c\ud50c \ub9e4\uac70\uc9c4\uc5d0\uc11c\ub294 2012\ub144 \uac00\uc7a5 \uc544\ub984\ub2e4\uc6b4 \ubaa8\ub4e0 \ub098\uc774 \uc911 \ud558\ub098\ub85c \uc120\uc815\ud588\ub2e4. 2012\ub144, \uadf8\ub140\ub294 \uc62c\ub9ac\ubc84 \uc2a4\ud1a4 \uac10\ub3c5\uc758 \u300a \ud30c\uad34\uc790\ub4e4 \u300b(\uc6d0\uc81c: Savages)\uc5d0\uc11c \ud14c\uc77c\ub7ec \ud0a4\ucde8, \uc544\ub860 \uc874\uc2a8, \uc140\ub9c8 \ud5e4\uc774\uc5d1\uacfc \uc874 \ud2b8\ub77c\ubcfc\ud0c0\uc640 \uacf5\uc5f0\ud588\ub2e4. \ub2f9\ucd08 \uc624\ud544\ub9ac\uc544 \"\uc624\" \uc138\uc774\uc9c0' \uc5ed\ud560\uc5d0\ub294 \uc81c\ub2c8\ud37c \ub85c\ub80c\uc2a4\uac00 \uce90\uc2a4\ud305\ub418\uc5c8\uc9c0\ub9cc \u300a\ud5dd\uac70\uac8c\uc784: \ud310\uc5e0\uc758 \ubd88\uaf43\u300b\uc758 \uc77c\uc815 \ubb38\uc81c\ub85c \ubb34\uc0b0 \ub418\uba74\uc11c \ub77c\uc774\ube14\ub9ac\uac00 \ub300\uccb4\ud588\ub2e4. HitFix\uc758 \uc601\ud654 \ud3c9\ub860\uac00 \ub4dc\ub958 \ub9e5\uc704\ub2c8\ub294 \ub77c\uc774\ube14\ub9ac\uc758 \uc5f0\uae30\ub97c \uce6d\ucc2c\ud558\uba70, \uadf8\ub294 \u201c\uadf8\ub140\uac00 \"\uc624\"\ub97c \uac00\uc2b4 \uc544\ud504\uace0 \uc791\uc740 \uc18c\uc6b8\ub85c \uc5f0\uae30\ud588\uae30\uc5d0 \uc601\ub9ac\ud558\uace0 \ud655\uc2e4\ud788 \uc2ac\ud390\ub2e4\u201d\ub77c\uace0 \ud3c9\ud588\ub2e4. \uac19\uc740 \ud574 \uadf8\ub140\ub294 \uc0c8\ub85c\uc6b4 \uad6c\ucc0c \ud5a5\uc218\uc778 \u2018Gucci Premiere\u2019\uc758 \ubba4\uc988\ub85c \uc120\uc815 \ub418\uc5c8\uace0, \uadf8\ub140\ub294 \ud5a5\uc218\uc5d0 \ub300\ud55c \ub2c8\ucf5c\ub77c\uc2a4 \uc708\ub529 \ub808\ud508 \uac10\ub3c5\uc758 \ub2e8\ud3b8 \uc601\ud654 \uad11\uace0\uc5d0 \ucd9c\uc5f0\ud588\ub2e4.", "sentence_answer": "2012\ub144, \uadf8\ub140\ub294 \uc62c\ub9ac\ubc84 \uc2a4\ud1a4 \uac10\ub3c5\uc758 \u300a \ud30c\uad34\uc790\ub4e4 \u300b(\uc6d0\uc81c: Savages)\uc5d0\uc11c \ud14c\uc77c\ub7ec \ud0a4\ucde8, \uc544\ub860 \uc874\uc2a8, \uc140\ub9c8 \ud5e4\uc774\uc5d1\uacfc \uc874 \ud2b8\ub77c\ubcfc\ud0c0\uc640 \uacf5\uc5f0\ud588\ub2e4.", "paragraph_id": "6530965-3-1", "paragraph_question": "question: \ube14\ub808\uc774\ud06c \ub77c\uc774\ube14\ub9ac\uac00 2012\ub144 \uc140\ub9c8 \ud5e4\uc774\uc5d1\uacfc \uc874 \ud2b8\ub77c\ubcfc\ud0c0 \ub4f1\uacfc \ucd9c\uc5f0\ud55c \uc62c\ub9ac\ubc84 \uc2a4\ud1a4 \uac10\ub3c5\uc791\uc740?, context: 2011\ub144, \uadf8\ub140\ub294 \ud0c0\uc784\uc988 \ub9e4\uac70\uc9c4 \u2018\uac00\uc7a5 \uc601\ud5a5\ub825 \uc788\ub294 100\uc778\u2019\uc5d0 \uc120\uc815 \ub418\uc5c8\ub2e4. \ub610\ud55c AskMen.com\uc740 \uadf8\ub140\ub97c 2011\ub144 \uac00\uc7a5 \uc139\uc2dc\ud55c \uc5ec\uc131\uc73c\ub85c \uc120\uc815\ud588\uace0, \ud53c\ud50c \ub9e4\uac70\uc9c4\uc5d0\uc11c\ub294 2012\ub144 \uac00\uc7a5 \uc544\ub984\ub2e4\uc6b4 \ubaa8\ub4e0 \ub098\uc774 \uc911 \ud558\ub098\ub85c \uc120\uc815\ud588\ub2e4. 2012\ub144, \uadf8\ub140\ub294 \uc62c\ub9ac\ubc84 \uc2a4\ud1a4 \uac10\ub3c5\uc758 \u300a\ud30c\uad34\uc790\ub4e4\u300b(\uc6d0\uc81c: Savages)\uc5d0\uc11c \ud14c\uc77c\ub7ec \ud0a4\ucde8, \uc544\ub860 \uc874\uc2a8, \uc140\ub9c8 \ud5e4\uc774\uc5d1\uacfc \uc874 \ud2b8\ub77c\ubcfc\ud0c0\uc640 \uacf5\uc5f0\ud588\ub2e4. \ub2f9\ucd08 \uc624\ud544\ub9ac\uc544 \"\uc624\" \uc138\uc774\uc9c0' \uc5ed\ud560\uc5d0\ub294 \uc81c\ub2c8\ud37c \ub85c\ub80c\uc2a4\uac00 \uce90\uc2a4\ud305\ub418\uc5c8\uc9c0\ub9cc \u300a\ud5dd\uac70\uac8c\uc784: \ud310\uc5e0\uc758 \ubd88\uaf43\u300b\uc758 \uc77c\uc815 \ubb38\uc81c\ub85c \ubb34\uc0b0 \ub418\uba74\uc11c \ub77c\uc774\ube14\ub9ac\uac00 \ub300\uccb4\ud588\ub2e4. HitFix\uc758 \uc601\ud654 \ud3c9\ub860\uac00 \ub4dc\ub958 \ub9e5\uc704\ub2c8\ub294 \ub77c\uc774\ube14\ub9ac\uc758 \uc5f0\uae30\ub97c \uce6d\ucc2c\ud558\uba70, \uadf8\ub294 \u201c\uadf8\ub140\uac00 \"\uc624\"\ub97c \uac00\uc2b4 \uc544\ud504\uace0 \uc791\uc740 \uc18c\uc6b8\ub85c \uc5f0\uae30\ud588\uae30\uc5d0 \uc601\ub9ac\ud558\uace0 \ud655\uc2e4\ud788 \uc2ac\ud390\ub2e4\u201d\ub77c\uace0 \ud3c9\ud588\ub2e4. \uac19\uc740 \ud574 \uadf8\ub140\ub294 \uc0c8\ub85c\uc6b4 \uad6c\ucc0c \ud5a5\uc218\uc778 \u2018Gucci Premiere\u2019\uc758 \ubba4\uc988\ub85c \uc120\uc815 \ub418\uc5c8\uace0, \uadf8\ub140\ub294 \ud5a5\uc218\uc5d0 \ub300\ud55c \ub2c8\ucf5c\ub77c\uc2a4 \uc708\ub529 \ub808\ud508 \uac10\ub3c5\uc758 \ub2e8\ud3b8 \uc601\ud654 \uad11\uace0\uc5d0 \ucd9c\uc5f0\ud588\ub2e4."} {"question": "\uae40\ub3d9\uc6d0\uc740 \uc5b4\ub5a4 SNS\uc5d0 \ub85c\uadf8\uc778 \ud574\uc11c \ub313\uae00\uc744 \uc870\uc791\ud558\uc600\ub294\uac00?", "paragraph": "2018\ub144 4\uc6d4 19\uc77c \uc11c\uc6b8\uc9c0\ubc29\uacbd\ucc30\uccad\uc5d0 \ub530\ub974\uba74 \uae40\uacbd\uc218\ub294 2016\ub144 11\uc6d4\ubd80\ud130 2018\ub144 3\uc6d4 \uc0ac\uc774\uc5d0 \ud154\ub808\uadf8\ub7a8\uc744 \ud1b5\ud574 \uae40\ub3d9\uc6d0(\ub4dc\ub8e8\ud0b9)\uc5d0\uac8c \uba54\uc2dc\uc9c0 \ucd1d 14\uac74\uc744 \ubcf4\ub0c8\ub2e4. \uba54\uc2dc\uc9c0 14\uac74 \uc911 10\uac74\uc774 \uae30\uc0ac \uc8fc\uc18c\uc600\ub2e4. \uae40\uacbd\uc218\uac00 \uae40\ub3d9\uc6d0(\ub4dc\ub8e8\ud0b9)\uc5d0\uac8c \ubcf4\ub0b8 \uae30\uc0ac\ub294 '\ubb38\uc7ac\uc778 \uc778\ud130\ubdf0', '\ubc18\uae30\ubb38 \ubd09\ud558\u884c\uc5d0 \uce5c\ub178 \ubd88\ud3b8\ud55c \uc2dc\uc120', '\ubb38\uc7ac\uc778\uc774 \uc5ec\uc131 \ud45c\uc2ec\uc5d0 \uc62c\uc778\ud55c\ub2e4' \ub4f1 \ubaa8\ub450 \uc9c1\u00b7\uac04\uc811\uc801\uc73c\ub85c \ubb38\uc7ac\uc778 \ub300\ud1b5\ub839\uacfc \ub354\ubd88\uc5b4\ubbfc\uc8fc\ub2f9 \uc9c0\uc9c0\uc5d0 \uad00\ub828\ub41c \uac83\uc774\uc5c8\ub2e4. \uae40\uacbd\uc218\uac00 \ub313\uae00\uc744 \uc870\uc791\ud55c \uae40\ub3d9\uc6d0(\ub4dc\ub8e8\ud0b9)\uc5d0\uac8c \ubcf4\ub0b8 \ub124\uc774\ubc84 \uae30\uc0ac\uc758 \ub313\uae00 \uc911\uc5d0\uc11c \ube44\uc815\uc0c1\uc801\uc778 SNS\uacc4\uc815\uc774 \ubc1c\uacac\ub418\uae30\ub3c4 \ud588\ub2e4. \ud2b8\uc704\ud130\ub85c \ub85c\uadf8\uc778\ud574 \ub313\uae00\uc744 \ub2e8 \uacc4\uc815 \uc911 SNS \uce5c\uad6c\uac00 \uc0ac\uc2e4\uc0c1 \uc5c6\ub294 \uc0c1\ud0dc\ub85c \ub300\uc120 \uc9c1\uc804\uc778 2017\ub144 4\uc6d4 26\uc77c\ubd80\ud130 5\uc6d4 8\uc77c\uae4c\uc9c0 \ub124\uc774\ubc84\uc5d0 \ub313\uae00\ub9cc \ub2e8 \uacc4\uc815\uc774\uc5c8\ub2e4. \ub313\uae00 \ub0b4\uc6a9\uc740 \ub300\ubd80\ubd84 \ubb38\uc7ac\uc778 \uc9c0\uc9c0 \ub0b4\uc6a9\uc774\uc5c8\ub2e4. \uae40\uacbd\uc218\uac00 \ud2b9\uc815 \uae30\uc0ac \ub9c1\ud06c\ub97c \uc54c\ub824\uc8fc\ub294 \uc801\uadf9\uc801\uc778 \ud589\uc704\ub85c \ub098\uc544\uac04 \uac83\uc774\ub77c\ub294 \uc810\uc5d0\uc11c \uacf5\ubaa8 \ub610\ub294 \uc9c0\uc2dc\u00b7\ubcf4\uace0 \uc5ec\ubd80\ub098 \uc758\uacac \uad50\ud658 \ub4f1\uc744 \ud655\uc778\ud560 \uc8fc\uc694 \uc218\uc0ac \ub2e8\uc11c\uc774\uae30\ub3c4 \ud558\ub2e4.", "answer": "\ud2b8\uc704\ud130", "sentence": "\ud2b8\uc704\ud130 \ub85c \ub85c\uadf8\uc778\ud574 \ub313\uae00\uc744 \ub2e8 \uacc4\uc815 \uc911 SNS \uce5c\uad6c\uac00 \uc0ac\uc2e4\uc0c1 \uc5c6\ub294 \uc0c1\ud0dc\ub85c \ub300\uc120 \uc9c1\uc804\uc778 2017\ub144 4\uc6d4 26\uc77c\ubd80\ud130 5\uc6d4 8\uc77c\uae4c\uc9c0 \ub124\uc774\ubc84\uc5d0 \ub313\uae00\ub9cc \ub2e8 \uacc4\uc815\uc774\uc5c8\ub2e4.", "paragraph_sentence": "2018\ub144 4\uc6d4 19\uc77c \uc11c\uc6b8\uc9c0\ubc29\uacbd\ucc30\uccad\uc5d0 \ub530\ub974\uba74 \uae40\uacbd\uc218\ub294 2016\ub144 11\uc6d4\ubd80\ud130 2018\ub144 3\uc6d4 \uc0ac\uc774\uc5d0 \ud154\ub808\uadf8\ub7a8\uc744 \ud1b5\ud574 \uae40\ub3d9\uc6d0(\ub4dc\ub8e8\ud0b9)\uc5d0\uac8c \uba54\uc2dc\uc9c0 \ucd1d 14\uac74\uc744 \ubcf4\ub0c8\ub2e4. \uba54\uc2dc\uc9c0 14\uac74 \uc911 10\uac74\uc774 \uae30\uc0ac \uc8fc\uc18c\uc600\ub2e4. \uae40\uacbd\uc218\uac00 \uae40\ub3d9\uc6d0(\ub4dc\ub8e8\ud0b9)\uc5d0\uac8c \ubcf4\ub0b8 \uae30\uc0ac\ub294 '\ubb38\uc7ac\uc778 \uc778\ud130\ubdf0', '\ubc18\uae30\ubb38 \ubd09\ud558\u884c\uc5d0 \uce5c\ub178 \ubd88\ud3b8\ud55c \uc2dc\uc120', '\ubb38\uc7ac\uc778\uc774 \uc5ec\uc131 \ud45c\uc2ec\uc5d0 \uc62c\uc778\ud55c\ub2e4' \ub4f1 \ubaa8\ub450 \uc9c1\u00b7\uac04\uc811\uc801\uc73c\ub85c \ubb38\uc7ac\uc778 \ub300\ud1b5\ub839\uacfc \ub354\ubd88\uc5b4\ubbfc\uc8fc\ub2f9 \uc9c0\uc9c0\uc5d0 \uad00\ub828\ub41c \uac83\uc774\uc5c8\ub2e4. \uae40\uacbd\uc218\uac00 \ub313\uae00\uc744 \uc870\uc791\ud55c \uae40\ub3d9\uc6d0(\ub4dc\ub8e8\ud0b9)\uc5d0\uac8c \ubcf4\ub0b8 \ub124\uc774\ubc84 \uae30\uc0ac\uc758 \ub313\uae00 \uc911\uc5d0\uc11c \ube44\uc815\uc0c1\uc801\uc778 SNS\uacc4\uc815\uc774 \ubc1c\uacac\ub418\uae30\ub3c4 \ud588\ub2e4. \ud2b8\uc704\ud130 \ub85c \ub85c\uadf8\uc778\ud574 \ub313\uae00\uc744 \ub2e8 \uacc4\uc815 \uc911 SNS \uce5c\uad6c\uac00 \uc0ac\uc2e4\uc0c1 \uc5c6\ub294 \uc0c1\ud0dc\ub85c \ub300\uc120 \uc9c1\uc804\uc778 2017\ub144 4\uc6d4 26\uc77c\ubd80\ud130 5\uc6d4 8\uc77c\uae4c\uc9c0 \ub124\uc774\ubc84\uc5d0 \ub313\uae00\ub9cc \ub2e8 \uacc4\uc815\uc774\uc5c8\ub2e4. \ub313\uae00 \ub0b4\uc6a9\uc740 \ub300\ubd80\ubd84 \ubb38\uc7ac\uc778 \uc9c0\uc9c0 \ub0b4\uc6a9\uc774\uc5c8\ub2e4. \uae40\uacbd\uc218\uac00 \ud2b9\uc815 \uae30\uc0ac \ub9c1\ud06c\ub97c \uc54c\ub824\uc8fc\ub294 \uc801\uadf9\uc801\uc778 \ud589\uc704\ub85c \ub098\uc544\uac04 \uac83\uc774\ub77c\ub294 \uc810\uc5d0\uc11c \uacf5\ubaa8 \ub610\ub294 \uc9c0\uc2dc\u00b7\ubcf4\uace0 \uc5ec\ubd80\ub098 \uc758\uacac \uad50\ud658 \ub4f1\uc744 \ud655\uc778\ud560 \uc8fc\uc694 \uc218\uc0ac \ub2e8\uc11c\uc774\uae30\ub3c4 \ud558\ub2e4.", "paragraph_answer": "2018\ub144 4\uc6d4 19\uc77c \uc11c\uc6b8\uc9c0\ubc29\uacbd\ucc30\uccad\uc5d0 \ub530\ub974\uba74 \uae40\uacbd\uc218\ub294 2016\ub144 11\uc6d4\ubd80\ud130 2018\ub144 3\uc6d4 \uc0ac\uc774\uc5d0 \ud154\ub808\uadf8\ub7a8\uc744 \ud1b5\ud574 \uae40\ub3d9\uc6d0(\ub4dc\ub8e8\ud0b9)\uc5d0\uac8c \uba54\uc2dc\uc9c0 \ucd1d 14\uac74\uc744 \ubcf4\ub0c8\ub2e4. \uba54\uc2dc\uc9c0 14\uac74 \uc911 10\uac74\uc774 \uae30\uc0ac \uc8fc\uc18c\uc600\ub2e4. \uae40\uacbd\uc218\uac00 \uae40\ub3d9\uc6d0(\ub4dc\ub8e8\ud0b9)\uc5d0\uac8c \ubcf4\ub0b8 \uae30\uc0ac\ub294 '\ubb38\uc7ac\uc778 \uc778\ud130\ubdf0', '\ubc18\uae30\ubb38 \ubd09\ud558\u884c\uc5d0 \uce5c\ub178 \ubd88\ud3b8\ud55c \uc2dc\uc120', '\ubb38\uc7ac\uc778\uc774 \uc5ec\uc131 \ud45c\uc2ec\uc5d0 \uc62c\uc778\ud55c\ub2e4' \ub4f1 \ubaa8\ub450 \uc9c1\u00b7\uac04\uc811\uc801\uc73c\ub85c \ubb38\uc7ac\uc778 \ub300\ud1b5\ub839\uacfc \ub354\ubd88\uc5b4\ubbfc\uc8fc\ub2f9 \uc9c0\uc9c0\uc5d0 \uad00\ub828\ub41c \uac83\uc774\uc5c8\ub2e4. \uae40\uacbd\uc218\uac00 \ub313\uae00\uc744 \uc870\uc791\ud55c \uae40\ub3d9\uc6d0(\ub4dc\ub8e8\ud0b9)\uc5d0\uac8c \ubcf4\ub0b8 \ub124\uc774\ubc84 \uae30\uc0ac\uc758 \ub313\uae00 \uc911\uc5d0\uc11c \ube44\uc815\uc0c1\uc801\uc778 SNS\uacc4\uc815\uc774 \ubc1c\uacac\ub418\uae30\ub3c4 \ud588\ub2e4. \ud2b8\uc704\ud130 \ub85c \ub85c\uadf8\uc778\ud574 \ub313\uae00\uc744 \ub2e8 \uacc4\uc815 \uc911 SNS \uce5c\uad6c\uac00 \uc0ac\uc2e4\uc0c1 \uc5c6\ub294 \uc0c1\ud0dc\ub85c \ub300\uc120 \uc9c1\uc804\uc778 2017\ub144 4\uc6d4 26\uc77c\ubd80\ud130 5\uc6d4 8\uc77c\uae4c\uc9c0 \ub124\uc774\ubc84\uc5d0 \ub313\uae00\ub9cc \ub2e8 \uacc4\uc815\uc774\uc5c8\ub2e4. \ub313\uae00 \ub0b4\uc6a9\uc740 \ub300\ubd80\ubd84 \ubb38\uc7ac\uc778 \uc9c0\uc9c0 \ub0b4\uc6a9\uc774\uc5c8\ub2e4. \uae40\uacbd\uc218\uac00 \ud2b9\uc815 \uae30\uc0ac \ub9c1\ud06c\ub97c \uc54c\ub824\uc8fc\ub294 \uc801\uadf9\uc801\uc778 \ud589\uc704\ub85c \ub098\uc544\uac04 \uac83\uc774\ub77c\ub294 \uc810\uc5d0\uc11c \uacf5\ubaa8 \ub610\ub294 \uc9c0\uc2dc\u00b7\ubcf4\uace0 \uc5ec\ubd80\ub098 \uc758\uacac \uad50\ud658 \ub4f1\uc744 \ud655\uc778\ud560 \uc8fc\uc694 \uc218\uc0ac \ub2e8\uc11c\uc774\uae30\ub3c4 \ud558\ub2e4.", "sentence_answer": " \ud2b8\uc704\ud130 \ub85c \ub85c\uadf8\uc778\ud574 \ub313\uae00\uc744 \ub2e8 \uacc4\uc815 \uc911 SNS \uce5c\uad6c\uac00 \uc0ac\uc2e4\uc0c1 \uc5c6\ub294 \uc0c1\ud0dc\ub85c \ub300\uc120 \uc9c1\uc804\uc778 2017\ub144 4\uc6d4 26\uc77c\ubd80\ud130 5\uc6d4 8\uc77c\uae4c\uc9c0 \ub124\uc774\ubc84\uc5d0 \ub313\uae00\ub9cc \ub2e8 \uacc4\uc815\uc774\uc5c8\ub2e4.", "paragraph_id": "6488117-1-1", "paragraph_question": "question: \uae40\ub3d9\uc6d0\uc740 \uc5b4\ub5a4 SNS\uc5d0 \ub85c\uadf8\uc778 \ud574\uc11c \ub313\uae00\uc744 \uc870\uc791\ud558\uc600\ub294\uac00?, context: 2018\ub144 4\uc6d4 19\uc77c \uc11c\uc6b8\uc9c0\ubc29\uacbd\ucc30\uccad\uc5d0 \ub530\ub974\uba74 \uae40\uacbd\uc218\ub294 2016\ub144 11\uc6d4\ubd80\ud130 2018\ub144 3\uc6d4 \uc0ac\uc774\uc5d0 \ud154\ub808\uadf8\ub7a8\uc744 \ud1b5\ud574 \uae40\ub3d9\uc6d0(\ub4dc\ub8e8\ud0b9)\uc5d0\uac8c \uba54\uc2dc\uc9c0 \ucd1d 14\uac74\uc744 \ubcf4\ub0c8\ub2e4. \uba54\uc2dc\uc9c0 14\uac74 \uc911 10\uac74\uc774 \uae30\uc0ac \uc8fc\uc18c\uc600\ub2e4. \uae40\uacbd\uc218\uac00 \uae40\ub3d9\uc6d0(\ub4dc\ub8e8\ud0b9)\uc5d0\uac8c \ubcf4\ub0b8 \uae30\uc0ac\ub294 '\ubb38\uc7ac\uc778 \uc778\ud130\ubdf0', '\ubc18\uae30\ubb38 \ubd09\ud558\u884c\uc5d0 \uce5c\ub178 \ubd88\ud3b8\ud55c \uc2dc\uc120', '\ubb38\uc7ac\uc778\uc774 \uc5ec\uc131 \ud45c\uc2ec\uc5d0 \uc62c\uc778\ud55c\ub2e4' \ub4f1 \ubaa8\ub450 \uc9c1\u00b7\uac04\uc811\uc801\uc73c\ub85c \ubb38\uc7ac\uc778 \ub300\ud1b5\ub839\uacfc \ub354\ubd88\uc5b4\ubbfc\uc8fc\ub2f9 \uc9c0\uc9c0\uc5d0 \uad00\ub828\ub41c \uac83\uc774\uc5c8\ub2e4. \uae40\uacbd\uc218\uac00 \ub313\uae00\uc744 \uc870\uc791\ud55c \uae40\ub3d9\uc6d0(\ub4dc\ub8e8\ud0b9)\uc5d0\uac8c \ubcf4\ub0b8 \ub124\uc774\ubc84 \uae30\uc0ac\uc758 \ub313\uae00 \uc911\uc5d0\uc11c \ube44\uc815\uc0c1\uc801\uc778 SNS\uacc4\uc815\uc774 \ubc1c\uacac\ub418\uae30\ub3c4 \ud588\ub2e4. \ud2b8\uc704\ud130\ub85c \ub85c\uadf8\uc778\ud574 \ub313\uae00\uc744 \ub2e8 \uacc4\uc815 \uc911 SNS \uce5c\uad6c\uac00 \uc0ac\uc2e4\uc0c1 \uc5c6\ub294 \uc0c1\ud0dc\ub85c \ub300\uc120 \uc9c1\uc804\uc778 2017\ub144 4\uc6d4 26\uc77c\ubd80\ud130 5\uc6d4 8\uc77c\uae4c\uc9c0 \ub124\uc774\ubc84\uc5d0 \ub313\uae00\ub9cc \ub2e8 \uacc4\uc815\uc774\uc5c8\ub2e4. \ub313\uae00 \ub0b4\uc6a9\uc740 \ub300\ubd80\ubd84 \ubb38\uc7ac\uc778 \uc9c0\uc9c0 \ub0b4\uc6a9\uc774\uc5c8\ub2e4. \uae40\uacbd\uc218\uac00 \ud2b9\uc815 \uae30\uc0ac \ub9c1\ud06c\ub97c \uc54c\ub824\uc8fc\ub294 \uc801\uadf9\uc801\uc778 \ud589\uc704\ub85c \ub098\uc544\uac04 \uac83\uc774\ub77c\ub294 \uc810\uc5d0\uc11c \uacf5\ubaa8 \ub610\ub294 \uc9c0\uc2dc\u00b7\ubcf4\uace0 \uc5ec\ubd80\ub098 \uc758\uacac \uad50\ud658 \ub4f1\uc744 \ud655\uc778\ud560 \uc8fc\uc694 \uc218\uc0ac \ub2e8\uc11c\uc774\uae30\ub3c4 \ud558\ub2e4."} {"question": "\ub2c8\ub178\uc774 \uc544\ud0a4\ub178 \uad6d\uc81c\uacf5\ud56d\uc758 2\ud130\ubbf8\ub110\uc744 \ub3c5\uc810\uc73c\ub85c \uc0ac\uc6a9\uc911\uc778 \ud56d\uacf5\uc0ac\ub294?", "paragraph": "1981\ub144\uc5d0 \uac1c\ud56d\ud588\ub2e4. 1983\ub144 \ud544\ub9ac\ud540 \ubbfc\uc8fc\ud654 \uc6b4\ub3d9 \uc9c0\ub3c4\uc790\ub85c \ub2f9\uc2dc \ub3c5\uc7ac\uc790 \ud398\ub974\ub514\ub09c\ub4dc \ub9c8\ub974\ucf54\uc2a4 \uc815\ubd80\uc5d0 \uc800\ud56d\ud588\ub358 \ubca0\ub2c8\uadf8\ub178 \uc544\ud0a4\ub178 2\uc138\uac00 \uc774 \uacf5\ud56d\uc5d0\uc11c \uc554\uc0b4\ub2f9\ud588\ub2e4. \uc774\ub7f0 \uc774\uc720\ub85c \uc774 \uacf5\ud56d\uc740 \ub2c8\ub178\uc774 \uc544\ud0a4\ub178 \uad6d\uc81c\uacf5\ud56d\uc774\ub77c\ub294 \ubcc4\uce6d\uc73c\ub85c \ub354 \ub9ce\uc774 \uc54c\ub824\uc838 \uc788\ub2e4. 1998\ub144\uc5d0 \uc81c 2\ud130\ubbf8\ub110\uc774 \uc644\uacf5\ub418\uc5c8\uace0 2008\ub144\uc5d0 \uc81c 3\ud130\ubbf8\ub110\uc774 \uc644\uacf5\ub418\uc5c8\ub2e4. \uacf5\ud56d\uc5d0 \ubaa8\ub450 3\uac1c\uc758 \ud130\ubbf8\ub110\uc774 \uc788\ub294\ub370, \uc774 \uac00\uc6b4\ub370 2\ud130\ubbf8\ub110\uc740 \ud544\ub9ac\ud540 \ud56d\uacf5\uc774 \ub3c5\uc810\uc801\uc73c\ub85c \uc0ac\uc6a9\ud558\uace0 \uc788\uace0, 3\ud130\ubbf8\ub110\uc740 \ud544\ub9ac\ud540\uc758 \uc800\uac00 \ud56d\uacf5\uc0ac\uc778 \uc138\ubd80 \ud37c\uc2dc\ud53d\uc774 \uc8fc\ub85c \uc0ac\uc6a9\ud558\uace0 \uc788\ub2e4. \ub610\ud55c \ud30c\uc0ac\uc774 \uc9c0\uc5ed\uacfc \ud30c\ub77c\ub0d0\ucf00 \uc9c0\uc5ed \uc811\uacbd\uc5d0 \uc790\ub9ac\uc7a1\uace0 \uc788\uc5b4 \ud30c\uc0ac\uc774 \ud30c\uc774\ube0c \uc2a4\ud0c0 \ubc84\uc2a4 \ud130\ubbf8\ub110\uc5d0\uc11c \ud0dd\uc2dc\ub85c 5\ubd84\ubc16\uc5d0 \uac78\ub9ac\uc9c0 \uc54a\uc744 \uc815\ub3c4\ub85c \ub3c4\uc2ec \uc811\uadfc\uc131\uc774 \ub6f0\uc5b4\ub09c \ud3b8\uc774\ub2e4.", "answer": "\ud544\ub9ac\ud540 \ud56d\uacf5", "sentence": "\uc774 \uac00\uc6b4\ub370 2\ud130\ubbf8\ub110\uc740 \ud544\ub9ac\ud540 \ud56d\uacf5 \uc774 \ub3c5\uc810\uc801\uc73c\ub85c \uc0ac\uc6a9\ud558\uace0 \uc788\uace0, 3\ud130\ubbf8\ub110\uc740 \ud544\ub9ac\ud540\uc758 \uc800\uac00 \ud56d\uacf5\uc0ac\uc778 \uc138\ubd80 \ud37c\uc2dc\ud53d\uc774 \uc8fc\ub85c \uc0ac\uc6a9\ud558\uace0 \uc788\ub2e4.", "paragraph_sentence": "1981\ub144\uc5d0 \uac1c\ud56d\ud588\ub2e4. 1983\ub144 \ud544\ub9ac\ud540 \ubbfc\uc8fc\ud654 \uc6b4\ub3d9 \uc9c0\ub3c4\uc790\ub85c \ub2f9\uc2dc \ub3c5\uc7ac\uc790 \ud398\ub974\ub514\ub09c\ub4dc \ub9c8\ub974\ucf54\uc2a4 \uc815\ubd80\uc5d0 \uc800\ud56d\ud588\ub358 \ubca0\ub2c8\uadf8\ub178 \uc544\ud0a4\ub178 2\uc138\uac00 \uc774 \uacf5\ud56d\uc5d0\uc11c \uc554\uc0b4\ub2f9\ud588\ub2e4. \uc774\ub7f0 \uc774\uc720\ub85c \uc774 \uacf5\ud56d\uc740 \ub2c8\ub178\uc774 \uc544\ud0a4\ub178 \uad6d\uc81c\uacf5\ud56d\uc774\ub77c\ub294 \ubcc4\uce6d\uc73c\ub85c \ub354 \ub9ce\uc774 \uc54c\ub824\uc838 \uc788\ub2e4. 1998\ub144\uc5d0 \uc81c 2\ud130\ubbf8\ub110\uc774 \uc644\uacf5\ub418\uc5c8\uace0 2008\ub144\uc5d0 \uc81c 3\ud130\ubbf8\ub110\uc774 \uc644\uacf5\ub418\uc5c8\ub2e4. \uacf5\ud56d\uc5d0 \ubaa8\ub450 3\uac1c\uc758 \ud130\ubbf8\ub110\uc774 \uc788\ub294\ub370, \uc774 \uac00\uc6b4\ub370 2\ud130\ubbf8\ub110\uc740 \ud544\ub9ac\ud540 \ud56d\uacf5 \uc774 \ub3c5\uc810\uc801\uc73c\ub85c \uc0ac\uc6a9\ud558\uace0 \uc788\uace0, 3\ud130\ubbf8\ub110\uc740 \ud544\ub9ac\ud540\uc758 \uc800\uac00 \ud56d\uacf5\uc0ac\uc778 \uc138\ubd80 \ud37c\uc2dc\ud53d\uc774 \uc8fc\ub85c \uc0ac\uc6a9\ud558\uace0 \uc788\ub2e4. \ub610\ud55c \ud30c\uc0ac\uc774 \uc9c0\uc5ed\uacfc \ud30c\ub77c\ub0d0\ucf00 \uc9c0\uc5ed \uc811\uacbd\uc5d0 \uc790\ub9ac\uc7a1\uace0 \uc788\uc5b4 \ud30c\uc0ac\uc774 \ud30c\uc774\ube0c \uc2a4\ud0c0 \ubc84\uc2a4 \ud130\ubbf8\ub110\uc5d0\uc11c \ud0dd\uc2dc\ub85c 5\ubd84\ubc16\uc5d0 \uac78\ub9ac\uc9c0 \uc54a\uc744 \uc815\ub3c4\ub85c \ub3c4\uc2ec \uc811\uadfc\uc131\uc774 \ub6f0\uc5b4\ub09c \ud3b8\uc774\ub2e4.", "paragraph_answer": "1981\ub144\uc5d0 \uac1c\ud56d\ud588\ub2e4. 1983\ub144 \ud544\ub9ac\ud540 \ubbfc\uc8fc\ud654 \uc6b4\ub3d9 \uc9c0\ub3c4\uc790\ub85c \ub2f9\uc2dc \ub3c5\uc7ac\uc790 \ud398\ub974\ub514\ub09c\ub4dc \ub9c8\ub974\ucf54\uc2a4 \uc815\ubd80\uc5d0 \uc800\ud56d\ud588\ub358 \ubca0\ub2c8\uadf8\ub178 \uc544\ud0a4\ub178 2\uc138\uac00 \uc774 \uacf5\ud56d\uc5d0\uc11c \uc554\uc0b4\ub2f9\ud588\ub2e4. \uc774\ub7f0 \uc774\uc720\ub85c \uc774 \uacf5\ud56d\uc740 \ub2c8\ub178\uc774 \uc544\ud0a4\ub178 \uad6d\uc81c\uacf5\ud56d\uc774\ub77c\ub294 \ubcc4\uce6d\uc73c\ub85c \ub354 \ub9ce\uc774 \uc54c\ub824\uc838 \uc788\ub2e4. 1998\ub144\uc5d0 \uc81c 2\ud130\ubbf8\ub110\uc774 \uc644\uacf5\ub418\uc5c8\uace0 2008\ub144\uc5d0 \uc81c 3\ud130\ubbf8\ub110\uc774 \uc644\uacf5\ub418\uc5c8\ub2e4. \uacf5\ud56d\uc5d0 \ubaa8\ub450 3\uac1c\uc758 \ud130\ubbf8\ub110\uc774 \uc788\ub294\ub370, \uc774 \uac00\uc6b4\ub370 2\ud130\ubbf8\ub110\uc740 \ud544\ub9ac\ud540 \ud56d\uacf5 \uc774 \ub3c5\uc810\uc801\uc73c\ub85c \uc0ac\uc6a9\ud558\uace0 \uc788\uace0, 3\ud130\ubbf8\ub110\uc740 \ud544\ub9ac\ud540\uc758 \uc800\uac00 \ud56d\uacf5\uc0ac\uc778 \uc138\ubd80 \ud37c\uc2dc\ud53d\uc774 \uc8fc\ub85c \uc0ac\uc6a9\ud558\uace0 \uc788\ub2e4. \ub610\ud55c \ud30c\uc0ac\uc774 \uc9c0\uc5ed\uacfc \ud30c\ub77c\ub0d0\ucf00 \uc9c0\uc5ed \uc811\uacbd\uc5d0 \uc790\ub9ac\uc7a1\uace0 \uc788\uc5b4 \ud30c\uc0ac\uc774 \ud30c\uc774\ube0c \uc2a4\ud0c0 \ubc84\uc2a4 \ud130\ubbf8\ub110\uc5d0\uc11c \ud0dd\uc2dc\ub85c 5\ubd84\ubc16\uc5d0 \uac78\ub9ac\uc9c0 \uc54a\uc744 \uc815\ub3c4\ub85c \ub3c4\uc2ec \uc811\uadfc\uc131\uc774 \ub6f0\uc5b4\ub09c \ud3b8\uc774\ub2e4.", "sentence_answer": "\uc774 \uac00\uc6b4\ub370 2\ud130\ubbf8\ub110\uc740 \ud544\ub9ac\ud540 \ud56d\uacf5 \uc774 \ub3c5\uc810\uc801\uc73c\ub85c \uc0ac\uc6a9\ud558\uace0 \uc788\uace0, 3\ud130\ubbf8\ub110\uc740 \ud544\ub9ac\ud540\uc758 \uc800\uac00 \ud56d\uacf5\uc0ac\uc778 \uc138\ubd80 \ud37c\uc2dc\ud53d\uc774 \uc8fc\ub85c \uc0ac\uc6a9\ud558\uace0 \uc788\ub2e4.", "paragraph_id": "6512140-0-2", "paragraph_question": "question: \ub2c8\ub178\uc774 \uc544\ud0a4\ub178 \uad6d\uc81c\uacf5\ud56d\uc758 2\ud130\ubbf8\ub110\uc744 \ub3c5\uc810\uc73c\ub85c \uc0ac\uc6a9\uc911\uc778 \ud56d\uacf5\uc0ac\ub294?, context: 1981\ub144\uc5d0 \uac1c\ud56d\ud588\ub2e4. 1983\ub144 \ud544\ub9ac\ud540 \ubbfc\uc8fc\ud654 \uc6b4\ub3d9 \uc9c0\ub3c4\uc790\ub85c \ub2f9\uc2dc \ub3c5\uc7ac\uc790 \ud398\ub974\ub514\ub09c\ub4dc \ub9c8\ub974\ucf54\uc2a4 \uc815\ubd80\uc5d0 \uc800\ud56d\ud588\ub358 \ubca0\ub2c8\uadf8\ub178 \uc544\ud0a4\ub178 2\uc138\uac00 \uc774 \uacf5\ud56d\uc5d0\uc11c \uc554\uc0b4\ub2f9\ud588\ub2e4. \uc774\ub7f0 \uc774\uc720\ub85c \uc774 \uacf5\ud56d\uc740 \ub2c8\ub178\uc774 \uc544\ud0a4\ub178 \uad6d\uc81c\uacf5\ud56d\uc774\ub77c\ub294 \ubcc4\uce6d\uc73c\ub85c \ub354 \ub9ce\uc774 \uc54c\ub824\uc838 \uc788\ub2e4. 1998\ub144\uc5d0 \uc81c 2\ud130\ubbf8\ub110\uc774 \uc644\uacf5\ub418\uc5c8\uace0 2008\ub144\uc5d0 \uc81c 3\ud130\ubbf8\ub110\uc774 \uc644\uacf5\ub418\uc5c8\ub2e4. \uacf5\ud56d\uc5d0 \ubaa8\ub450 3\uac1c\uc758 \ud130\ubbf8\ub110\uc774 \uc788\ub294\ub370, \uc774 \uac00\uc6b4\ub370 2\ud130\ubbf8\ub110\uc740 \ud544\ub9ac\ud540 \ud56d\uacf5\uc774 \ub3c5\uc810\uc801\uc73c\ub85c \uc0ac\uc6a9\ud558\uace0 \uc788\uace0, 3\ud130\ubbf8\ub110\uc740 \ud544\ub9ac\ud540\uc758 \uc800\uac00 \ud56d\uacf5\uc0ac\uc778 \uc138\ubd80 \ud37c\uc2dc\ud53d\uc774 \uc8fc\ub85c \uc0ac\uc6a9\ud558\uace0 \uc788\ub2e4. \ub610\ud55c \ud30c\uc0ac\uc774 \uc9c0\uc5ed\uacfc \ud30c\ub77c\ub0d0\ucf00 \uc9c0\uc5ed \uc811\uacbd\uc5d0 \uc790\ub9ac\uc7a1\uace0 \uc788\uc5b4 \ud30c\uc0ac\uc774 \ud30c\uc774\ube0c \uc2a4\ud0c0 \ubc84\uc2a4 \ud130\ubbf8\ub110\uc5d0\uc11c \ud0dd\uc2dc\ub85c 5\ubd84\ubc16\uc5d0 \uac78\ub9ac\uc9c0 \uc54a\uc744 \uc815\ub3c4\ub85c \ub3c4\uc2ec \uc811\uadfc\uc131\uc774 \ub6f0\uc5b4\ub09c \ud3b8\uc774\ub2e4."} {"question": "\ub098\uce58\uac00 \uc138\uc6b4 \uad34\ub8b0\uc815\ubd80\uc5d0 \uc758\ud574 \uba87 \uac1c \ub9c8\uc744\uc774 \ubd88\uc5d0 \ud0d4\ub294\uac00?", "paragraph": "\ubca8\ub77c\ub8e8\uc2a4 \uc9c0\uc5ed\uc774 \uac00\uc7a5 \ud070 \ud76c\uc0dd\uc790\uc600\ub2e4. \ub098\uce58\ub294 \ubca8\ub77c\ub8e8\uc2a4\uc5d0 \uad34\ub8b0\uc815\ubd80\ub97c \uc138\uc6b0\uace0 \uc774\ub4e4\ub85c \ud558\uc5ec\uae08 \uc628\uac16 \ub9cc\ud589\uc744 \uc790\ud589\ud1a0\ub85d \ubd80\ucd94\uacbc\ub2e4. \uacb0\uad6d 9\ucc9c\uac1c\uc758 \ub9c8\uc744\uc5d0 \ubd88\uc5d0 \ud0d4\uc73c\uba70 38\ub9cc\uba85\uc774 \uac15\uc81c\ub178\ub3d9\uc744 \uc704\ud574 \uc774\uc1a1\ub418\uc5c8\uace0 \uc218\ub9cc \uba85\uc758 \ubbfc\uac04\uc778\uc774 \ud559\uc0b4\ub2f9\ud588\ub2e4. \uce74\ud2f4(Khatyn)\uacfc \uac19\uc740 \ub9c8\uc744\uc740 \ub9c8\uc744\uc744 \ubd88\ud0dc\uc6b8 \ub54c\uc5d0 \uc8fc\ubbfc\ub4e4\ub3c4 \ubaa8\ub450 \ubd88\uc5d0 \ud0c0\uc11c \uc8fd\uc5c8\ub2e4. \uadf8\ub7f0 \ub9c8\uc744\uc774 \ubca8\ub77c\ub8e8\uc2a4\uc5d0 600\uac1c\ub098 \uc788\uc5c8\ub2e4. \ub3d9\uad6c\uc5d0\uc11c \ubca8\ub77c\ub8e8\uc2a4\ub85c \uc774\uc804\ud55c \uc2ac\ub77c\ube0c\uc778\ub4e4\uc774 \ub9cc\ub4e0 \uc815\ucc29\ucd0c\uc740 5,295\uac1c\uc18c\uac00 \ub098\uce58\uc5d0 \uc758\ud574 \ud30c\uad34\ub418\uc5c8\ub2e4. \uc774\ub4e4 \uc815\ucc29\ucd0c\uc758 \uc8fc\ubbfc\ub4e4\uc740 \uac70\uc758 \ubaa8\ub450\uac00 \uc0b4\ud574\ub418\uc5c8\ub2e4. \uadf8\ub9ac\ud558\uc5ec \ub3c4\ud569 1\ubc31 67\ub9cc\uba85\uc758 \ubbfc\uac04\uc778\uc774 3\ub144 \ub3d9\uc548\uc758 \ub3c5\uc77c \uc810\ub839\uae30\uac04\uc5d0 \uc8fd\uc784\uc744 \ub2f9\ud588\ub2e4. \uc774\ub294 \uc804\uccb4 \uc778\uad6c\uc758 18%\uc5d0 \ud574\ub2f9\ud558\ub294 \uac83\uc774\uc5c8\ub2e4. \uadf8\uc911\uc5d0\uc11c 24\ub9cc 5\ucc9c\uba85\uc740 \uc720\ud0dc\uc778\uc73c\ub85c\uc368 \ubca8\ub77c\ub8e8\uc2a4 \uc2dc\ubbfc\ub4e4\ub85c \uad6c\uc131\ub41c \uc544\uc778\uc790\uce20\uadf8\ub8e8\ud39c(Einsatzgruppen)\uc5d0 \uc758\ud574 \uc0b4\ud574\ub418\uc5c8\ub2e4.", "answer": "9\ucc9c\uac1c", "sentence": "\uacb0\uad6d 9\ucc9c\uac1c \uc758 \ub9c8\uc744\uc5d0 \ubd88\uc5d0 \ud0d4\uc73c\uba70 38\ub9cc\uba85\uc774 \uac15\uc81c\ub178\ub3d9\uc744 \uc704\ud574 \uc774\uc1a1\ub418\uc5c8\uace0 \uc218\ub9cc \uba85\uc758 \ubbfc\uac04\uc778\uc774 \ud559\uc0b4\ub2f9\ud588\ub2e4.", "paragraph_sentence": "\ubca8\ub77c\ub8e8\uc2a4 \uc9c0\uc5ed\uc774 \uac00\uc7a5 \ud070 \ud76c\uc0dd\uc790\uc600\ub2e4. \ub098\uce58\ub294 \ubca8\ub77c\ub8e8\uc2a4\uc5d0 \uad34\ub8b0\uc815\ubd80\ub97c \uc138\uc6b0\uace0 \uc774\ub4e4\ub85c \ud558\uc5ec\uae08 \uc628\uac16 \ub9cc\ud589\uc744 \uc790\ud589\ud1a0\ub85d \ubd80\ucd94\uacbc\ub2e4. \uacb0\uad6d 9\ucc9c\uac1c \uc758 \ub9c8\uc744\uc5d0 \ubd88\uc5d0 \ud0d4\uc73c\uba70 38\ub9cc\uba85\uc774 \uac15\uc81c\ub178\ub3d9\uc744 \uc704\ud574 \uc774\uc1a1\ub418\uc5c8\uace0 \uc218\ub9cc \uba85\uc758 \ubbfc\uac04\uc778\uc774 \ud559\uc0b4\ub2f9\ud588\ub2e4. \uce74\ud2f4(Khatyn)\uacfc \uac19\uc740 \ub9c8\uc744\uc740 \ub9c8\uc744\uc744 \ubd88\ud0dc\uc6b8 \ub54c\uc5d0 \uc8fc\ubbfc\ub4e4\ub3c4 \ubaa8\ub450 \ubd88\uc5d0 \ud0c0\uc11c \uc8fd\uc5c8\ub2e4. \uadf8\ub7f0 \ub9c8\uc744\uc774 \ubca8\ub77c\ub8e8\uc2a4\uc5d0 600\uac1c\ub098 \uc788\uc5c8\ub2e4. \ub3d9\uad6c\uc5d0\uc11c \ubca8\ub77c\ub8e8\uc2a4\ub85c \uc774\uc804\ud55c \uc2ac\ub77c\ube0c\uc778\ub4e4\uc774 \ub9cc\ub4e0 \uc815\ucc29\ucd0c\uc740 5,295\uac1c\uc18c\uac00 \ub098\uce58\uc5d0 \uc758\ud574 \ud30c\uad34\ub418\uc5c8\ub2e4. \uc774\ub4e4 \uc815\ucc29\ucd0c\uc758 \uc8fc\ubbfc\ub4e4\uc740 \uac70\uc758 \ubaa8\ub450\uac00 \uc0b4\ud574\ub418\uc5c8\ub2e4. \uadf8\ub9ac\ud558\uc5ec \ub3c4\ud569 1\ubc31 67\ub9cc\uba85\uc758 \ubbfc\uac04\uc778\uc774 3\ub144 \ub3d9\uc548\uc758 \ub3c5\uc77c \uc810\ub839\uae30\uac04\uc5d0 \uc8fd\uc784\uc744 \ub2f9\ud588\ub2e4. \uc774\ub294 \uc804\uccb4 \uc778\uad6c\uc758 18%\uc5d0 \ud574\ub2f9\ud558\ub294 \uac83\uc774\uc5c8\ub2e4. \uadf8\uc911\uc5d0\uc11c 24\ub9cc 5\ucc9c\uba85\uc740 \uc720\ud0dc\uc778\uc73c\ub85c\uc368 \ubca8\ub77c\ub8e8\uc2a4 \uc2dc\ubbfc\ub4e4\ub85c \uad6c\uc131\ub41c \uc544\uc778\uc790\uce20\uadf8\ub8e8\ud39c(Einsatzgruppen)\uc5d0 \uc758\ud574 \uc0b4\ud574\ub418\uc5c8\ub2e4.", "paragraph_answer": "\ubca8\ub77c\ub8e8\uc2a4 \uc9c0\uc5ed\uc774 \uac00\uc7a5 \ud070 \ud76c\uc0dd\uc790\uc600\ub2e4. \ub098\uce58\ub294 \ubca8\ub77c\ub8e8\uc2a4\uc5d0 \uad34\ub8b0\uc815\ubd80\ub97c \uc138\uc6b0\uace0 \uc774\ub4e4\ub85c \ud558\uc5ec\uae08 \uc628\uac16 \ub9cc\ud589\uc744 \uc790\ud589\ud1a0\ub85d \ubd80\ucd94\uacbc\ub2e4. \uacb0\uad6d 9\ucc9c\uac1c \uc758 \ub9c8\uc744\uc5d0 \ubd88\uc5d0 \ud0d4\uc73c\uba70 38\ub9cc\uba85\uc774 \uac15\uc81c\ub178\ub3d9\uc744 \uc704\ud574 \uc774\uc1a1\ub418\uc5c8\uace0 \uc218\ub9cc \uba85\uc758 \ubbfc\uac04\uc778\uc774 \ud559\uc0b4\ub2f9\ud588\ub2e4. \uce74\ud2f4(Khatyn)\uacfc \uac19\uc740 \ub9c8\uc744\uc740 \ub9c8\uc744\uc744 \ubd88\ud0dc\uc6b8 \ub54c\uc5d0 \uc8fc\ubbfc\ub4e4\ub3c4 \ubaa8\ub450 \ubd88\uc5d0 \ud0c0\uc11c \uc8fd\uc5c8\ub2e4. \uadf8\ub7f0 \ub9c8\uc744\uc774 \ubca8\ub77c\ub8e8\uc2a4\uc5d0 600\uac1c\ub098 \uc788\uc5c8\ub2e4. \ub3d9\uad6c\uc5d0\uc11c \ubca8\ub77c\ub8e8\uc2a4\ub85c \uc774\uc804\ud55c \uc2ac\ub77c\ube0c\uc778\ub4e4\uc774 \ub9cc\ub4e0 \uc815\ucc29\ucd0c\uc740 5,295\uac1c\uc18c\uac00 \ub098\uce58\uc5d0 \uc758\ud574 \ud30c\uad34\ub418\uc5c8\ub2e4. \uc774\ub4e4 \uc815\ucc29\ucd0c\uc758 \uc8fc\ubbfc\ub4e4\uc740 \uac70\uc758 \ubaa8\ub450\uac00 \uc0b4\ud574\ub418\uc5c8\ub2e4. \uadf8\ub9ac\ud558\uc5ec \ub3c4\ud569 1\ubc31 67\ub9cc\uba85\uc758 \ubbfc\uac04\uc778\uc774 3\ub144 \ub3d9\uc548\uc758 \ub3c5\uc77c \uc810\ub839\uae30\uac04\uc5d0 \uc8fd\uc784\uc744 \ub2f9\ud588\ub2e4. \uc774\ub294 \uc804\uccb4 \uc778\uad6c\uc758 18%\uc5d0 \ud574\ub2f9\ud558\ub294 \uac83\uc774\uc5c8\ub2e4. \uadf8\uc911\uc5d0\uc11c 24\ub9cc 5\ucc9c\uba85\uc740 \uc720\ud0dc\uc778\uc73c\ub85c\uc368 \ubca8\ub77c\ub8e8\uc2a4 \uc2dc\ubbfc\ub4e4\ub85c \uad6c\uc131\ub41c \uc544\uc778\uc790\uce20\uadf8\ub8e8\ud39c(Einsatzgruppen)\uc5d0 \uc758\ud574 \uc0b4\ud574\ub418\uc5c8\ub2e4.", "sentence_answer": "\uacb0\uad6d 9\ucc9c\uac1c \uc758 \ub9c8\uc744\uc5d0 \ubd88\uc5d0 \ud0d4\uc73c\uba70 38\ub9cc\uba85\uc774 \uac15\uc81c\ub178\ub3d9\uc744 \uc704\ud574 \uc774\uc1a1\ub418\uc5c8\uace0 \uc218\ub9cc \uba85\uc758 \ubbfc\uac04\uc778\uc774 \ud559\uc0b4\ub2f9\ud588\ub2e4.", "paragraph_id": "6571961-40-2", "paragraph_question": "question: \ub098\uce58\uac00 \uc138\uc6b4 \uad34\ub8b0\uc815\ubd80\uc5d0 \uc758\ud574 \uba87 \uac1c \ub9c8\uc744\uc774 \ubd88\uc5d0 \ud0d4\ub294\uac00?, context: \ubca8\ub77c\ub8e8\uc2a4 \uc9c0\uc5ed\uc774 \uac00\uc7a5 \ud070 \ud76c\uc0dd\uc790\uc600\ub2e4. \ub098\uce58\ub294 \ubca8\ub77c\ub8e8\uc2a4\uc5d0 \uad34\ub8b0\uc815\ubd80\ub97c \uc138\uc6b0\uace0 \uc774\ub4e4\ub85c \ud558\uc5ec\uae08 \uc628\uac16 \ub9cc\ud589\uc744 \uc790\ud589\ud1a0\ub85d \ubd80\ucd94\uacbc\ub2e4. \uacb0\uad6d 9\ucc9c\uac1c\uc758 \ub9c8\uc744\uc5d0 \ubd88\uc5d0 \ud0d4\uc73c\uba70 38\ub9cc\uba85\uc774 \uac15\uc81c\ub178\ub3d9\uc744 \uc704\ud574 \uc774\uc1a1\ub418\uc5c8\uace0 \uc218\ub9cc \uba85\uc758 \ubbfc\uac04\uc778\uc774 \ud559\uc0b4\ub2f9\ud588\ub2e4. \uce74\ud2f4(Khatyn)\uacfc \uac19\uc740 \ub9c8\uc744\uc740 \ub9c8\uc744\uc744 \ubd88\ud0dc\uc6b8 \ub54c\uc5d0 \uc8fc\ubbfc\ub4e4\ub3c4 \ubaa8\ub450 \ubd88\uc5d0 \ud0c0\uc11c \uc8fd\uc5c8\ub2e4. \uadf8\ub7f0 \ub9c8\uc744\uc774 \ubca8\ub77c\ub8e8\uc2a4\uc5d0 600\uac1c\ub098 \uc788\uc5c8\ub2e4. \ub3d9\uad6c\uc5d0\uc11c \ubca8\ub77c\ub8e8\uc2a4\ub85c \uc774\uc804\ud55c \uc2ac\ub77c\ube0c\uc778\ub4e4\uc774 \ub9cc\ub4e0 \uc815\ucc29\ucd0c\uc740 5,295\uac1c\uc18c\uac00 \ub098\uce58\uc5d0 \uc758\ud574 \ud30c\uad34\ub418\uc5c8\ub2e4. \uc774\ub4e4 \uc815\ucc29\ucd0c\uc758 \uc8fc\ubbfc\ub4e4\uc740 \uac70\uc758 \ubaa8\ub450\uac00 \uc0b4\ud574\ub418\uc5c8\ub2e4. \uadf8\ub9ac\ud558\uc5ec \ub3c4\ud569 1\ubc31 67\ub9cc\uba85\uc758 \ubbfc\uac04\uc778\uc774 3\ub144 \ub3d9\uc548\uc758 \ub3c5\uc77c \uc810\ub839\uae30\uac04\uc5d0 \uc8fd\uc784\uc744 \ub2f9\ud588\ub2e4. \uc774\ub294 \uc804\uccb4 \uc778\uad6c\uc758 18%\uc5d0 \ud574\ub2f9\ud558\ub294 \uac83\uc774\uc5c8\ub2e4. \uadf8\uc911\uc5d0\uc11c 24\ub9cc 5\ucc9c\uba85\uc740 \uc720\ud0dc\uc778\uc73c\ub85c\uc368 \ubca8\ub77c\ub8e8\uc2a4 \uc2dc\ubbfc\ub4e4\ub85c \uad6c\uc131\ub41c \uc544\uc778\uc790\uce20\uadf8\ub8e8\ud39c(Einsatzgruppen)\uc5d0 \uc758\ud574 \uc0b4\ud574\ub418\uc5c8\ub2e4."} {"question": "\ucca0\uad6c\uac00 \ubc29\uc1a1 \uc815\uc9c0 \ucc98\ubd84\uc744 \ubc1b\uc740 \uac83\uc740 \uc774\ubc88\uc774 \uba87\ubc88\uc9f8\uc778\uac00?", "paragraph": "\uc9c0\ub09c 22\uc77c \uc544\ud504\ub9ac\uce74TV\ub294 BJ \ucca0\uad6c\uac00 \uc11c\ube44\uc2a4 \uc815\uc9c0 \ucc98\ubd84\uc744 \ubc1b\uc558\uc74c\uc744 \ubc1d\ud614\ub2e4. \uc11c\ube44\uc2a4 \uc815\uc9c0 \ucc98\ubd84\uc744 \uc0ac\uc720\ub294 \ucca0\uad6c\uac00 10\ub300 \uccad\uc18c\ub144\uc5d0\uac8c \uc720\ud574\ud55c \uc7a5\uba74\uc744 \ubc29\uc1a1\uc73c\ub85c \ub0b4\ubcf4\ub0c8\uae30 \ub54c\ubb38\uc774\uc5c8\ub2e4. \ubb38\uc81c\uac00 \ub41c \uc7a5\uba74\uc740 BJ \ucca0\uad6c\uac00 \ubbf8\uc131\ub144\uc790\ub294 \uc2dc\uccad\ud560 \uc218 \uc5c6\uac8c \ud558\ub294 19\uc138 \uc2dc\uccad \uac00\ub2a5 \uc124\uc815\uc744 \ud558\uc9c0 \uc54a\uc740 \ucc44 \ud761\uc5f0\ud558\ub294 \ubaa8\uc2b5\uc744 \uc5ec\uacfc \uc5c6\uc774 \ub4dc\ub7ec\ub0b8 \uc7a5\uba74\uc774\ub2e4. \uc544\ud504\ub9ac\uce74TV\ub294 \uccad\uc18c\ub144 \ubcf4\ud638 \uc815\ucc45\uc758 '\uccad\uc18c\ub144\ub4e4\uc774 \ud574\ub85c\uc6b4 \ud658\uacbd\uc73c\ub85c\ubd80\ud130 \ubcf4\ud638\ubc1b\uc744 \uc218 \uc788\ub3c4\ub85d \uc870\uce58\ud55c\ub2e4'\ub77c\uace0 \uc870\ud56d\uc744 \uadfc\uac70\ub85c \ucca0\uad6c\uc5d0\uac8c \uc11c\ube44\uc2a4 \uc815\uc9c0 \ucc98\ubd84\uc744 \ub0b4\ub838\ub2e4. \ud761\uc5f0 \uc774\uc678\uc5d0 \uc74c\uc8fc \ubc29\uc1a1 \ub4f1\ub3c4 19\uc138 \uc2dc\uccad \uac00\ub2a5 \uc124\uc815\uc744 \ud574\uc57c\ub9cc \ubc29\uc1a1\ud560 \uc218 \uc788\ub2e4. \uac8c\ub2e4\uac00 \ucca0\uad6c\uc758 \ubc29\uc1a1 \uc815\uc9c0 \ucc98\ubd84\uc740 \uc774\ubc88\uc5d0 \ucc98\uc74c\uc774 \uc544\ub2c8\ub77c 16\ubc88\uc9f8\uae30 \ub54c\ubb38\uc5d0 \ub354\uc6b1\ub354 \ub17c\ub780\uc774 \ub418\uace0 \uc788\ub2e4.", "answer": "16\ubc88\uc9f8", "sentence": "\uac8c\ub2e4\uac00 \ucca0\uad6c\uc758 \ubc29\uc1a1 \uc815\uc9c0 \ucc98\ubd84\uc740 \uc774\ubc88\uc5d0 \ucc98\uc74c\uc774 \uc544\ub2c8\ub77c 16\ubc88\uc9f8 \uae30 \ub54c\ubb38\uc5d0 \ub354\uc6b1\ub354 \ub17c\ub780\uc774 \ub418\uace0 \uc788\ub2e4.", "paragraph_sentence": "\uc9c0\ub09c 22\uc77c \uc544\ud504\ub9ac\uce74TV\ub294 BJ \ucca0\uad6c\uac00 \uc11c\ube44\uc2a4 \uc815\uc9c0 \ucc98\ubd84\uc744 \ubc1b\uc558\uc74c\uc744 \ubc1d\ud614\ub2e4. \uc11c\ube44\uc2a4 \uc815\uc9c0 \ucc98\ubd84\uc744 \uc0ac\uc720\ub294 \ucca0\uad6c\uac00 10\ub300 \uccad\uc18c\ub144\uc5d0\uac8c \uc720\ud574\ud55c \uc7a5\uba74\uc744 \ubc29\uc1a1\uc73c\ub85c \ub0b4\ubcf4\ub0c8\uae30 \ub54c\ubb38\uc774\uc5c8\ub2e4. \ubb38\uc81c\uac00 \ub41c \uc7a5\uba74\uc740 BJ \ucca0\uad6c\uac00 \ubbf8\uc131\ub144\uc790\ub294 \uc2dc\uccad\ud560 \uc218 \uc5c6\uac8c \ud558\ub294 19\uc138 \uc2dc\uccad \uac00\ub2a5 \uc124\uc815\uc744 \ud558\uc9c0 \uc54a\uc740 \ucc44 \ud761\uc5f0\ud558\ub294 \ubaa8\uc2b5\uc744 \uc5ec\uacfc \uc5c6\uc774 \ub4dc\ub7ec\ub0b8 \uc7a5\uba74\uc774\ub2e4. \uc544\ud504\ub9ac\uce74TV\ub294 \uccad\uc18c\ub144 \ubcf4\ud638 \uc815\ucc45\uc758 '\uccad\uc18c\ub144\ub4e4\uc774 \ud574\ub85c\uc6b4 \ud658\uacbd\uc73c\ub85c\ubd80\ud130 \ubcf4\ud638\ubc1b\uc744 \uc218 \uc788\ub3c4\ub85d \uc870\uce58\ud55c\ub2e4'\ub77c\uace0 \uc870\ud56d\uc744 \uadfc\uac70\ub85c \ucca0\uad6c\uc5d0\uac8c \uc11c\ube44\uc2a4 \uc815\uc9c0 \ucc98\ubd84\uc744 \ub0b4\ub838\ub2e4. \ud761\uc5f0 \uc774\uc678\uc5d0 \uc74c\uc8fc \ubc29\uc1a1 \ub4f1\ub3c4 19\uc138 \uc2dc\uccad \uac00\ub2a5 \uc124\uc815\uc744 \ud574\uc57c\ub9cc \ubc29\uc1a1\ud560 \uc218 \uc788\ub2e4. \uac8c\ub2e4\uac00 \ucca0\uad6c\uc758 \ubc29\uc1a1 \uc815\uc9c0 \ucc98\ubd84\uc740 \uc774\ubc88\uc5d0 \ucc98\uc74c\uc774 \uc544\ub2c8\ub77c 16\ubc88\uc9f8 \uae30 \ub54c\ubb38\uc5d0 \ub354\uc6b1\ub354 \ub17c\ub780\uc774 \ub418\uace0 \uc788\ub2e4. ", "paragraph_answer": "\uc9c0\ub09c 22\uc77c \uc544\ud504\ub9ac\uce74TV\ub294 BJ \ucca0\uad6c\uac00 \uc11c\ube44\uc2a4 \uc815\uc9c0 \ucc98\ubd84\uc744 \ubc1b\uc558\uc74c\uc744 \ubc1d\ud614\ub2e4. \uc11c\ube44\uc2a4 \uc815\uc9c0 \ucc98\ubd84\uc744 \uc0ac\uc720\ub294 \ucca0\uad6c\uac00 10\ub300 \uccad\uc18c\ub144\uc5d0\uac8c \uc720\ud574\ud55c \uc7a5\uba74\uc744 \ubc29\uc1a1\uc73c\ub85c \ub0b4\ubcf4\ub0c8\uae30 \ub54c\ubb38\uc774\uc5c8\ub2e4. \ubb38\uc81c\uac00 \ub41c \uc7a5\uba74\uc740 BJ \ucca0\uad6c\uac00 \ubbf8\uc131\ub144\uc790\ub294 \uc2dc\uccad\ud560 \uc218 \uc5c6\uac8c \ud558\ub294 19\uc138 \uc2dc\uccad \uac00\ub2a5 \uc124\uc815\uc744 \ud558\uc9c0 \uc54a\uc740 \ucc44 \ud761\uc5f0\ud558\ub294 \ubaa8\uc2b5\uc744 \uc5ec\uacfc \uc5c6\uc774 \ub4dc\ub7ec\ub0b8 \uc7a5\uba74\uc774\ub2e4. \uc544\ud504\ub9ac\uce74TV\ub294 \uccad\uc18c\ub144 \ubcf4\ud638 \uc815\ucc45\uc758 '\uccad\uc18c\ub144\ub4e4\uc774 \ud574\ub85c\uc6b4 \ud658\uacbd\uc73c\ub85c\ubd80\ud130 \ubcf4\ud638\ubc1b\uc744 \uc218 \uc788\ub3c4\ub85d \uc870\uce58\ud55c\ub2e4'\ub77c\uace0 \uc870\ud56d\uc744 \uadfc\uac70\ub85c \ucca0\uad6c\uc5d0\uac8c \uc11c\ube44\uc2a4 \uc815\uc9c0 \ucc98\ubd84\uc744 \ub0b4\ub838\ub2e4. \ud761\uc5f0 \uc774\uc678\uc5d0 \uc74c\uc8fc \ubc29\uc1a1 \ub4f1\ub3c4 19\uc138 \uc2dc\uccad \uac00\ub2a5 \uc124\uc815\uc744 \ud574\uc57c\ub9cc \ubc29\uc1a1\ud560 \uc218 \uc788\ub2e4. \uac8c\ub2e4\uac00 \ucca0\uad6c\uc758 \ubc29\uc1a1 \uc815\uc9c0 \ucc98\ubd84\uc740 \uc774\ubc88\uc5d0 \ucc98\uc74c\uc774 \uc544\ub2c8\ub77c 16\ubc88\uc9f8 \uae30 \ub54c\ubb38\uc5d0 \ub354\uc6b1\ub354 \ub17c\ub780\uc774 \ub418\uace0 \uc788\ub2e4.", "sentence_answer": "\uac8c\ub2e4\uac00 \ucca0\uad6c\uc758 \ubc29\uc1a1 \uc815\uc9c0 \ucc98\ubd84\uc740 \uc774\ubc88\uc5d0 \ucc98\uc74c\uc774 \uc544\ub2c8\ub77c 16\ubc88\uc9f8 \uae30 \ub54c\ubb38\uc5d0 \ub354\uc6b1\ub354 \ub17c\ub780\uc774 \ub418\uace0 \uc788\ub2e4.", "paragraph_id": "6596777-0-1", "paragraph_question": "question: \ucca0\uad6c\uac00 \ubc29\uc1a1 \uc815\uc9c0 \ucc98\ubd84\uc744 \ubc1b\uc740 \uac83\uc740 \uc774\ubc88\uc774 \uba87\ubc88\uc9f8\uc778\uac00?, context: \uc9c0\ub09c 22\uc77c \uc544\ud504\ub9ac\uce74TV\ub294 BJ \ucca0\uad6c\uac00 \uc11c\ube44\uc2a4 \uc815\uc9c0 \ucc98\ubd84\uc744 \ubc1b\uc558\uc74c\uc744 \ubc1d\ud614\ub2e4. \uc11c\ube44\uc2a4 \uc815\uc9c0 \ucc98\ubd84\uc744 \uc0ac\uc720\ub294 \ucca0\uad6c\uac00 10\ub300 \uccad\uc18c\ub144\uc5d0\uac8c \uc720\ud574\ud55c \uc7a5\uba74\uc744 \ubc29\uc1a1\uc73c\ub85c \ub0b4\ubcf4\ub0c8\uae30 \ub54c\ubb38\uc774\uc5c8\ub2e4. \ubb38\uc81c\uac00 \ub41c \uc7a5\uba74\uc740 BJ \ucca0\uad6c\uac00 \ubbf8\uc131\ub144\uc790\ub294 \uc2dc\uccad\ud560 \uc218 \uc5c6\uac8c \ud558\ub294 19\uc138 \uc2dc\uccad \uac00\ub2a5 \uc124\uc815\uc744 \ud558\uc9c0 \uc54a\uc740 \ucc44 \ud761\uc5f0\ud558\ub294 \ubaa8\uc2b5\uc744 \uc5ec\uacfc \uc5c6\uc774 \ub4dc\ub7ec\ub0b8 \uc7a5\uba74\uc774\ub2e4. \uc544\ud504\ub9ac\uce74TV\ub294 \uccad\uc18c\ub144 \ubcf4\ud638 \uc815\ucc45\uc758 '\uccad\uc18c\ub144\ub4e4\uc774 \ud574\ub85c\uc6b4 \ud658\uacbd\uc73c\ub85c\ubd80\ud130 \ubcf4\ud638\ubc1b\uc744 \uc218 \uc788\ub3c4\ub85d \uc870\uce58\ud55c\ub2e4'\ub77c\uace0 \uc870\ud56d\uc744 \uadfc\uac70\ub85c \ucca0\uad6c\uc5d0\uac8c \uc11c\ube44\uc2a4 \uc815\uc9c0 \ucc98\ubd84\uc744 \ub0b4\ub838\ub2e4. \ud761\uc5f0 \uc774\uc678\uc5d0 \uc74c\uc8fc \ubc29\uc1a1 \ub4f1\ub3c4 19\uc138 \uc2dc\uccad \uac00\ub2a5 \uc124\uc815\uc744 \ud574\uc57c\ub9cc \ubc29\uc1a1\ud560 \uc218 \uc788\ub2e4. \uac8c\ub2e4\uac00 \ucca0\uad6c\uc758 \ubc29\uc1a1 \uc815\uc9c0 \ucc98\ubd84\uc740 \uc774\ubc88\uc5d0 \ucc98\uc74c\uc774 \uc544\ub2c8\ub77c 16\ubc88\uc9f8\uae30 \ub54c\ubb38\uc5d0 \ub354\uc6b1\ub354 \ub17c\ub780\uc774 \ub418\uace0 \uc788\ub2e4."} {"question": "\ud3ec\uac94\uc774 \uc4f4 \ub178\uc608\uc81c\ub3c4\uc758 \uacbd\uc81c\uc801 \ud6a8\uc728\uc131\uacfc \ud751\uc778 \ub178\uc608\ub4e4\uc758 \uc0dd\ud65c \uc218\uc900\uc744 \uc5f0\uad6c\ud55c \ucc45\uc758 \uc774\ub984\uc740?", "paragraph": "\uc774 \ub17c\uc7c1\uc740 'Antebellum Puzzle'\uc774\ub77c \ubd88\ub838\ub358 \ubbf8\uad6d \ub0a8\ubd81\uc804\uc7c1 \uc774\uc804 1830~1860\ub144 \ucd9c\uc0dd\uc9d1\ub2e8\uc5d0 \uac78\uccd0 \uad00\uce21\ub41c \ud3c9\uade0 \uc2e0\uc7a5\uc758 \ud558\ub77d \uc6d0\uc778\uc744 \ub458\ub7ec\uc2fc \uc77c\ub828\uc758 \ub17c\uc7c1\uc744 \ub9d0\ud55c\ub2e4. \ub17c\uc7c1\uc758 \uc2dc\uc791\uc740 Fogel and Engerman(1974)\uc774 \uc4f4 \u300aTime on the Cross\u300b\ub77c\ub294 \ucc45\uc774\ub77c\uace0 \uc77c\ubc18\uc801\uc73c\ub85c \uc0dd\uac01\ub41c\ub2e4. \uc774 \ucc45\uc740 \ub178\uc608\uc81c\ub3c4\uc758 \uacbd\uc81c\uc801 \ud6a8\uc728\uc131\uacfc \ud751\uc778 \ub178\uc608\ub4e4\uc758 \uc0dd\ud65c\uc218\uc900\uc744 \uc5f0\uad6c\ud588\ub294\ub370 \uae30\uc874\uc758 \uc815\uc124\uc5d0 \ubc18\ubc15\ud558\uba74\uc11c \ud070 \ubc18\ud5a5\uacfc \ub17c\ub780\uc744 \uc77c\uc73c\ucf30\ub2e4. \ub178\uc608\uc81c \ub17c\uc7c1\uc758 \uacfc\uc815\uc5d0\uc11c \uc218\ub7c9\uacbd\uc81c\uc0ac(cliometrics)\uac00 \ubcf8\uaca9\uc801\uc73c\ub85c \uc2dc\uc791\ub410\uace0 \uc77c\ub828\uc758 \uc5f0\uad6c\ub4e4\uc740 \ud751\uc778 \ub178\uc608\uc758 \ud3c9\uade0\uc18c\ub4dd, \uae30\ub300\uc218\uba85, \ucd9c\uc0b0\uc728, \uc77c\ud3c9\uade0 \uce7c\ub85c\ub9ac \uc12d\ucde8\ub7c9, \uc8fc\uac70 \ubc0f \uc758\ub8cc\ud604\ud669 \ub4f1\uc744 \uc0c1\uc138\ud788 \uadf8\ub9ac\uace0 \uc815\ub7c9\uc801\uc73c\ub85c \uac80\ud1a0\ud558\uc600\ub2e4. Steckel(1979)\uacfc Margo and Steckel(1982)\uc758 \ubbf8\uad6d \ubc31\uc778, \ud751\uc778 \ub178\uc608, \uc601\uad6d \ubc31\uc778\uc758 \uc2e0\uc7a5\ube44\uad50\ub294 \uc774\ud6c4 \uc601\uad6d \uc0dd\ud65c\uc218\uc900 \ub17c\uc7c1\uc5d0 \uc601\ud5a5\uc744 \uc8fc\uc5c8\ub2e4. Fogel(1986) \ub4f1\uc740 \ubbf8\uad6d \ubc31\uc778 \ud3c9\uade0 \uc2e0\uc7a5\uc758 \uc7a5\uae30\ucd94\uc774\ub97c \ucd94\uacc4\ud558\ub294 \uacfc\uc815\uc5d0\uc11c 1830~1860\uc5d0 \uac78\uccd0 \uc2e0\uc7a5 \uac10\uc18c\ub97c \uad00\uce21\ud588\ub2e4. \uc0b0\uc5c5\ud654\uac00 \ubcf8\uaca9\ud654\ub418\uba74\uc11c \uc18c\ub4dd\uacfc \uc0dd\uc0b0\uc131\uc774 \uc99d\uac00\ud588\ub358 \uc2dc\uae30\uc5d0 \ud3c9\uade0 \uc2e0\uc7a5\uc774 \uac10\uc18c\ud588\ub2e4\ub294 \uc0ac\uc2e4\uc740 \ub9ce\uc740 \uacbd\uc81c\ud559\uc790\ub4e4\uc758 \uad00\uc2ec\uc744 \ubaa8\uc73c\uba70 \uc774 \ub17c\uc7c1\uc774 \uc2dc\uc791\ub410\ub2e4.", "answer": "Time on the Cross", "sentence": "\ub17c\uc7c1\uc758 \uc2dc\uc791\uc740 Fogel and Engerman(1974)\uc774 \uc4f4 \u300a Time on the Cross \u300b\ub77c\ub294 \ucc45\uc774\ub77c\uace0 \uc77c\ubc18\uc801\uc73c\ub85c \uc0dd\uac01\ub41c\ub2e4.", "paragraph_sentence": "\uc774 \ub17c\uc7c1\uc740 'Antebellum Puzzle'\uc774\ub77c \ubd88\ub838\ub358 \ubbf8\uad6d \ub0a8\ubd81\uc804\uc7c1 \uc774\uc804 1830~1860\ub144 \ucd9c\uc0dd\uc9d1\ub2e8\uc5d0 \uac78\uccd0 \uad00\uce21\ub41c \ud3c9\uade0 \uc2e0\uc7a5\uc758 \ud558\ub77d \uc6d0\uc778\uc744 \ub458\ub7ec\uc2fc \uc77c\ub828\uc758 \ub17c\uc7c1\uc744 \ub9d0\ud55c\ub2e4. \ub17c\uc7c1\uc758 \uc2dc\uc791\uc740 Fogel and Engerman(1974)\uc774 \uc4f4 \u300a Time on the Cross \u300b\ub77c\ub294 \ucc45\uc774\ub77c\uace0 \uc77c\ubc18\uc801\uc73c\ub85c \uc0dd\uac01\ub41c\ub2e4. \uc774 \ucc45\uc740 \ub178\uc608\uc81c\ub3c4\uc758 \uacbd\uc81c\uc801 \ud6a8\uc728\uc131\uacfc \ud751\uc778 \ub178\uc608\ub4e4\uc758 \uc0dd\ud65c\uc218\uc900\uc744 \uc5f0\uad6c\ud588\ub294\ub370 \uae30\uc874\uc758 \uc815\uc124\uc5d0 \ubc18\ubc15\ud558\uba74\uc11c \ud070 \ubc18\ud5a5\uacfc \ub17c\ub780\uc744 \uc77c\uc73c\ucf30\ub2e4. \ub178\uc608\uc81c \ub17c\uc7c1\uc758 \uacfc\uc815\uc5d0\uc11c \uc218\ub7c9\uacbd\uc81c\uc0ac(cliometrics)\uac00 \ubcf8\uaca9\uc801\uc73c\ub85c \uc2dc\uc791\ub410\uace0 \uc77c\ub828\uc758 \uc5f0\uad6c\ub4e4\uc740 \ud751\uc778 \ub178\uc608\uc758 \ud3c9\uade0\uc18c\ub4dd, \uae30\ub300\uc218\uba85, \ucd9c\uc0b0\uc728, \uc77c\ud3c9\uade0 \uce7c\ub85c\ub9ac \uc12d\ucde8\ub7c9, \uc8fc\uac70 \ubc0f \uc758\ub8cc\ud604\ud669 \ub4f1\uc744 \uc0c1\uc138\ud788 \uadf8\ub9ac\uace0 \uc815\ub7c9\uc801\uc73c\ub85c \uac80\ud1a0\ud558\uc600\ub2e4. Steckel(1979)\uacfc Margo and Steckel(1982)\uc758 \ubbf8\uad6d \ubc31\uc778, \ud751\uc778 \ub178\uc608, \uc601\uad6d \ubc31\uc778\uc758 \uc2e0\uc7a5\ube44\uad50\ub294 \uc774\ud6c4 \uc601\uad6d \uc0dd\ud65c\uc218\uc900 \ub17c\uc7c1\uc5d0 \uc601\ud5a5\uc744 \uc8fc\uc5c8\ub2e4. Fogel(1986) \ub4f1\uc740 \ubbf8\uad6d \ubc31\uc778 \ud3c9\uade0 \uc2e0\uc7a5\uc758 \uc7a5\uae30\ucd94\uc774\ub97c \ucd94\uacc4\ud558\ub294 \uacfc\uc815\uc5d0\uc11c 1830~1860\uc5d0 \uac78\uccd0 \uc2e0\uc7a5 \uac10\uc18c\ub97c \uad00\uce21\ud588\ub2e4. \uc0b0\uc5c5\ud654\uac00 \ubcf8\uaca9\ud654\ub418\uba74\uc11c \uc18c\ub4dd\uacfc \uc0dd\uc0b0\uc131\uc774 \uc99d\uac00\ud588\ub358 \uc2dc\uae30\uc5d0 \ud3c9\uade0 \uc2e0\uc7a5\uc774 \uac10\uc18c\ud588\ub2e4\ub294 \uc0ac\uc2e4\uc740 \ub9ce\uc740 \uacbd\uc81c\ud559\uc790\ub4e4\uc758 \uad00\uc2ec\uc744 \ubaa8\uc73c\uba70 \uc774 \ub17c\uc7c1\uc774 \uc2dc\uc791\ub410\ub2e4.", "paragraph_answer": "\uc774 \ub17c\uc7c1\uc740 'Antebellum Puzzle'\uc774\ub77c \ubd88\ub838\ub358 \ubbf8\uad6d \ub0a8\ubd81\uc804\uc7c1 \uc774\uc804 1830~1860\ub144 \ucd9c\uc0dd\uc9d1\ub2e8\uc5d0 \uac78\uccd0 \uad00\uce21\ub41c \ud3c9\uade0 \uc2e0\uc7a5\uc758 \ud558\ub77d \uc6d0\uc778\uc744 \ub458\ub7ec\uc2fc \uc77c\ub828\uc758 \ub17c\uc7c1\uc744 \ub9d0\ud55c\ub2e4. \ub17c\uc7c1\uc758 \uc2dc\uc791\uc740 Fogel and Engerman(1974)\uc774 \uc4f4 \u300a Time on the Cross \u300b\ub77c\ub294 \ucc45\uc774\ub77c\uace0 \uc77c\ubc18\uc801\uc73c\ub85c \uc0dd\uac01\ub41c\ub2e4. \uc774 \ucc45\uc740 \ub178\uc608\uc81c\ub3c4\uc758 \uacbd\uc81c\uc801 \ud6a8\uc728\uc131\uacfc \ud751\uc778 \ub178\uc608\ub4e4\uc758 \uc0dd\ud65c\uc218\uc900\uc744 \uc5f0\uad6c\ud588\ub294\ub370 \uae30\uc874\uc758 \uc815\uc124\uc5d0 \ubc18\ubc15\ud558\uba74\uc11c \ud070 \ubc18\ud5a5\uacfc \ub17c\ub780\uc744 \uc77c\uc73c\ucf30\ub2e4. \ub178\uc608\uc81c \ub17c\uc7c1\uc758 \uacfc\uc815\uc5d0\uc11c \uc218\ub7c9\uacbd\uc81c\uc0ac(cliometrics)\uac00 \ubcf8\uaca9\uc801\uc73c\ub85c \uc2dc\uc791\ub410\uace0 \uc77c\ub828\uc758 \uc5f0\uad6c\ub4e4\uc740 \ud751\uc778 \ub178\uc608\uc758 \ud3c9\uade0\uc18c\ub4dd, \uae30\ub300\uc218\uba85, \ucd9c\uc0b0\uc728, \uc77c\ud3c9\uade0 \uce7c\ub85c\ub9ac \uc12d\ucde8\ub7c9, \uc8fc\uac70 \ubc0f \uc758\ub8cc\ud604\ud669 \ub4f1\uc744 \uc0c1\uc138\ud788 \uadf8\ub9ac\uace0 \uc815\ub7c9\uc801\uc73c\ub85c \uac80\ud1a0\ud558\uc600\ub2e4. Steckel(1979)\uacfc Margo and Steckel(1982)\uc758 \ubbf8\uad6d \ubc31\uc778, \ud751\uc778 \ub178\uc608, \uc601\uad6d \ubc31\uc778\uc758 \uc2e0\uc7a5\ube44\uad50\ub294 \uc774\ud6c4 \uc601\uad6d \uc0dd\ud65c\uc218\uc900 \ub17c\uc7c1\uc5d0 \uc601\ud5a5\uc744 \uc8fc\uc5c8\ub2e4. Fogel(1986) \ub4f1\uc740 \ubbf8\uad6d \ubc31\uc778 \ud3c9\uade0 \uc2e0\uc7a5\uc758 \uc7a5\uae30\ucd94\uc774\ub97c \ucd94\uacc4\ud558\ub294 \uacfc\uc815\uc5d0\uc11c 1830~1860\uc5d0 \uac78\uccd0 \uc2e0\uc7a5 \uac10\uc18c\ub97c \uad00\uce21\ud588\ub2e4. \uc0b0\uc5c5\ud654\uac00 \ubcf8\uaca9\ud654\ub418\uba74\uc11c \uc18c\ub4dd\uacfc \uc0dd\uc0b0\uc131\uc774 \uc99d\uac00\ud588\ub358 \uc2dc\uae30\uc5d0 \ud3c9\uade0 \uc2e0\uc7a5\uc774 \uac10\uc18c\ud588\ub2e4\ub294 \uc0ac\uc2e4\uc740 \ub9ce\uc740 \uacbd\uc81c\ud559\uc790\ub4e4\uc758 \uad00\uc2ec\uc744 \ubaa8\uc73c\uba70 \uc774 \ub17c\uc7c1\uc774 \uc2dc\uc791\ub410\ub2e4.", "sentence_answer": "\ub17c\uc7c1\uc758 \uc2dc\uc791\uc740 Fogel and Engerman(1974)\uc774 \uc4f4 \u300a Time on the Cross \u300b\ub77c\ub294 \ucc45\uc774\ub77c\uace0 \uc77c\ubc18\uc801\uc73c\ub85c \uc0dd\uac01\ub41c\ub2e4.", "paragraph_id": "6437666-4-0", "paragraph_question": "question: \ud3ec\uac94\uc774 \uc4f4 \ub178\uc608\uc81c\ub3c4\uc758 \uacbd\uc81c\uc801 \ud6a8\uc728\uc131\uacfc \ud751\uc778 \ub178\uc608\ub4e4\uc758 \uc0dd\ud65c \uc218\uc900\uc744 \uc5f0\uad6c\ud55c \ucc45\uc758 \uc774\ub984\uc740?, context: \uc774 \ub17c\uc7c1\uc740 'Antebellum Puzzle'\uc774\ub77c \ubd88\ub838\ub358 \ubbf8\uad6d \ub0a8\ubd81\uc804\uc7c1 \uc774\uc804 1830~1860\ub144 \ucd9c\uc0dd\uc9d1\ub2e8\uc5d0 \uac78\uccd0 \uad00\uce21\ub41c \ud3c9\uade0 \uc2e0\uc7a5\uc758 \ud558\ub77d \uc6d0\uc778\uc744 \ub458\ub7ec\uc2fc \uc77c\ub828\uc758 \ub17c\uc7c1\uc744 \ub9d0\ud55c\ub2e4. \ub17c\uc7c1\uc758 \uc2dc\uc791\uc740 Fogel and Engerman(1974)\uc774 \uc4f4 \u300aTime on the Cross\u300b\ub77c\ub294 \ucc45\uc774\ub77c\uace0 \uc77c\ubc18\uc801\uc73c\ub85c \uc0dd\uac01\ub41c\ub2e4. \uc774 \ucc45\uc740 \ub178\uc608\uc81c\ub3c4\uc758 \uacbd\uc81c\uc801 \ud6a8\uc728\uc131\uacfc \ud751\uc778 \ub178\uc608\ub4e4\uc758 \uc0dd\ud65c\uc218\uc900\uc744 \uc5f0\uad6c\ud588\ub294\ub370 \uae30\uc874\uc758 \uc815\uc124\uc5d0 \ubc18\ubc15\ud558\uba74\uc11c \ud070 \ubc18\ud5a5\uacfc \ub17c\ub780\uc744 \uc77c\uc73c\ucf30\ub2e4. \ub178\uc608\uc81c \ub17c\uc7c1\uc758 \uacfc\uc815\uc5d0\uc11c \uc218\ub7c9\uacbd\uc81c\uc0ac(cliometrics)\uac00 \ubcf8\uaca9\uc801\uc73c\ub85c \uc2dc\uc791\ub410\uace0 \uc77c\ub828\uc758 \uc5f0\uad6c\ub4e4\uc740 \ud751\uc778 \ub178\uc608\uc758 \ud3c9\uade0\uc18c\ub4dd, \uae30\ub300\uc218\uba85, \ucd9c\uc0b0\uc728, \uc77c\ud3c9\uade0 \uce7c\ub85c\ub9ac \uc12d\ucde8\ub7c9, \uc8fc\uac70 \ubc0f \uc758\ub8cc\ud604\ud669 \ub4f1\uc744 \uc0c1\uc138\ud788 \uadf8\ub9ac\uace0 \uc815\ub7c9\uc801\uc73c\ub85c \uac80\ud1a0\ud558\uc600\ub2e4. Steckel(1979)\uacfc Margo and Steckel(1982)\uc758 \ubbf8\uad6d \ubc31\uc778, \ud751\uc778 \ub178\uc608, \uc601\uad6d \ubc31\uc778\uc758 \uc2e0\uc7a5\ube44\uad50\ub294 \uc774\ud6c4 \uc601\uad6d \uc0dd\ud65c\uc218\uc900 \ub17c\uc7c1\uc5d0 \uc601\ud5a5\uc744 \uc8fc\uc5c8\ub2e4. Fogel(1986) \ub4f1\uc740 \ubbf8\uad6d \ubc31\uc778 \ud3c9\uade0 \uc2e0\uc7a5\uc758 \uc7a5\uae30\ucd94\uc774\ub97c \ucd94\uacc4\ud558\ub294 \uacfc\uc815\uc5d0\uc11c 1830~1860\uc5d0 \uac78\uccd0 \uc2e0\uc7a5 \uac10\uc18c\ub97c \uad00\uce21\ud588\ub2e4. \uc0b0\uc5c5\ud654\uac00 \ubcf8\uaca9\ud654\ub418\uba74\uc11c \uc18c\ub4dd\uacfc \uc0dd\uc0b0\uc131\uc774 \uc99d\uac00\ud588\ub358 \uc2dc\uae30\uc5d0 \ud3c9\uade0 \uc2e0\uc7a5\uc774 \uac10\uc18c\ud588\ub2e4\ub294 \uc0ac\uc2e4\uc740 \ub9ce\uc740 \uacbd\uc81c\ud559\uc790\ub4e4\uc758 \uad00\uc2ec\uc744 \ubaa8\uc73c\uba70 \uc774 \ub17c\uc7c1\uc774 \uc2dc\uc791\ub410\ub2e4."} {"question": "\uc1a1\uc2e0\uc601\uc740 \uc5b4\ub5a4 \ud300\uacfc FA\ub97c \uc131\uc0ac\uc2dc\ucf30\ub294\uac00?", "paragraph": "\uc1a1\uc2e0\uc601 \ub610\ud55c \uc774\ud0dd\uadfc\uacfc \ub9c8\ucc2c\uac00\uc9c0\ub85c \ud0c0 \uad6c\ub2e8\uacfc\uc758 \ud611\uc0c1 \ub0a0\uc9dc\uac00 \uc2dc\uc791\ub418\uc790\ub9c8\uc790 \ud55c\ud654 \uc774\uae00\uc2a4\uc640 FA\ub97c \uc131\uc0ac\uc2dc\ucf30\ub2e4. \uacc4\uc57d \uc870\uac74\uc740 \uacc4\uc57d \uae30\uac04 3\ub144 \ub3d9\uc548 \uacc4\uc57d\uae08 4\uc5b5\uc6d0, \uc5f0\ubd09 3\uc5b5\uc6d0\uc5d0 \ucd1d\uc561 13\uc5b5\uc6d0\uc73c\ub85c, \ud50c\ub7ec\uc2a4 \uc635\uc158\uc774 \ubd99\uc5b4\uc788\ub2e4. \ud55c\ud654\uc758 \uc774\uc0c1\uad70 \ud300\uc7a5\uc740 20\uc77c\uc774 \uc790\uc815\uc774 \ub118\uc790\ub9c8\uc790 \uc1a1\uc2e0\uc601\uc744 \ucc3e\uc544\uac00 \ud611\uc0c1\uc744 \ubc8c\uc600\uace0, \uc1a1\uc2e0\uc601\ub3c4 \uc790\uc2e0\uc744 \uc560\ud0c0\uac8c \ucc3e\uc544\uc640 \uc900 \ud55c\ud654 \uad6c\ub2e8\uc758 \uc131\uc758\uc5d0 \uac10\ub3d9\ud574 \uacc4\uc57d\uc774 \uc218\uc6d4\ud788 \uc9c4\ud589\ub418\uc5c8\ub2e4. \uc9c1\uc811 \uad6c\ub2e8\uc5d0 \uc1a1\uc2e0\uc601\uc744 \uc7a1\uc73c\ub77c \uc694\uccad\ud588\uc5c8\ub358 \ud55c\ub300\ud654 \uac10\ub3c5\uc740 \uc774 FA \uc601\uc785\uc5d0 \ud070 \ub9cc\uc871\uac10\uc744 \ub4e4\uc5b4\ub0c8\uace0, \uadf8\ub294 \uc1a1\uc2e0\uc601\uc5d0\uac8c \ubc14\ud2f0\uc2a4\ud0c0, \ubc15\uc815\uc9c4\uacfc \ud568\uaed8 \ud55c\ud654 \ub9c8\uc6b4\ub4dc\uc758 \ud5c8\ub9ac\ub97c \ucc45\uc784\uc9c8 \uc14b\uc5c5\ub9e8 \uc5ed\ud560\uc744 \ub9e1\uacbc\ub2e4. \ud55c\ud3b8, LG\ub294 \uc1a1\uc2e0\uc601\uc5d0 \ub300\ud55c \ubcf4\uc0c1 \uc120\uc218\ub85c \ud3ec\uc218 \ub098\uc131\uc6a9\uc744 \ub370\ub824\uac14\ub2e4.", "answer": "\ud55c\ud654 \uc774\uae00\uc2a4", "sentence": "\uc1a1\uc2e0\uc601 \ub610\ud55c \uc774\ud0dd\uadfc\uacfc \ub9c8\ucc2c\uac00\uc9c0\ub85c \ud0c0 \uad6c\ub2e8\uacfc\uc758 \ud611\uc0c1 \ub0a0\uc9dc\uac00 \uc2dc\uc791\ub418\uc790\ub9c8\uc790 \ud55c\ud654 \uc774\uae00\uc2a4 \uc640 FA\ub97c \uc131\uc0ac\uc2dc\ucf30\ub2e4.", "paragraph_sentence": " \uc1a1\uc2e0\uc601 \ub610\ud55c \uc774\ud0dd\uadfc\uacfc \ub9c8\ucc2c\uac00\uc9c0\ub85c \ud0c0 \uad6c\ub2e8\uacfc\uc758 \ud611\uc0c1 \ub0a0\uc9dc\uac00 \uc2dc\uc791\ub418\uc790\ub9c8\uc790 \ud55c\ud654 \uc774\uae00\uc2a4 \uc640 FA\ub97c \uc131\uc0ac\uc2dc\ucf30\ub2e4. \uacc4\uc57d \uc870\uac74\uc740 \uacc4\uc57d \uae30\uac04 3\ub144 \ub3d9\uc548 \uacc4\uc57d\uae08 4\uc5b5\uc6d0, \uc5f0\ubd09 3\uc5b5\uc6d0\uc5d0 \ucd1d\uc561 13\uc5b5\uc6d0\uc73c\ub85c, \ud50c\ub7ec\uc2a4 \uc635\uc158\uc774 \ubd99\uc5b4\uc788\ub2e4. \ud55c\ud654\uc758 \uc774\uc0c1\uad70 \ud300\uc7a5\uc740 20\uc77c\uc774 \uc790\uc815\uc774 \ub118\uc790\ub9c8\uc790 \uc1a1\uc2e0\uc601\uc744 \ucc3e\uc544\uac00 \ud611\uc0c1\uc744 \ubc8c\uc600\uace0, \uc1a1\uc2e0\uc601\ub3c4 \uc790\uc2e0\uc744 \uc560\ud0c0\uac8c \ucc3e\uc544\uc640 \uc900 \ud55c\ud654 \uad6c\ub2e8\uc758 \uc131\uc758\uc5d0 \uac10\ub3d9\ud574 \uacc4\uc57d\uc774 \uc218\uc6d4\ud788 \uc9c4\ud589\ub418\uc5c8\ub2e4. \uc9c1\uc811 \uad6c\ub2e8\uc5d0 \uc1a1\uc2e0\uc601\uc744 \uc7a1\uc73c\ub77c \uc694\uccad\ud588\uc5c8\ub358 \ud55c\ub300\ud654 \uac10\ub3c5\uc740 \uc774 FA \uc601\uc785\uc5d0 \ud070 \ub9cc\uc871\uac10\uc744 \ub4e4\uc5b4\ub0c8\uace0, \uadf8\ub294 \uc1a1\uc2e0\uc601\uc5d0\uac8c \ubc14\ud2f0\uc2a4\ud0c0, \ubc15\uc815\uc9c4\uacfc \ud568\uaed8 \ud55c\ud654 \ub9c8\uc6b4\ub4dc\uc758 \ud5c8\ub9ac\ub97c \ucc45\uc784\uc9c8 \uc14b\uc5c5\ub9e8 \uc5ed\ud560\uc744 \ub9e1\uacbc\ub2e4. \ud55c\ud3b8, LG\ub294 \uc1a1\uc2e0\uc601\uc5d0 \ub300\ud55c \ubcf4\uc0c1 \uc120\uc218\ub85c \ud3ec\uc218 \ub098\uc131\uc6a9\uc744 \ub370\ub824\uac14\ub2e4.", "paragraph_answer": "\uc1a1\uc2e0\uc601 \ub610\ud55c \uc774\ud0dd\uadfc\uacfc \ub9c8\ucc2c\uac00\uc9c0\ub85c \ud0c0 \uad6c\ub2e8\uacfc\uc758 \ud611\uc0c1 \ub0a0\uc9dc\uac00 \uc2dc\uc791\ub418\uc790\ub9c8\uc790 \ud55c\ud654 \uc774\uae00\uc2a4 \uc640 FA\ub97c \uc131\uc0ac\uc2dc\ucf30\ub2e4. \uacc4\uc57d \uc870\uac74\uc740 \uacc4\uc57d \uae30\uac04 3\ub144 \ub3d9\uc548 \uacc4\uc57d\uae08 4\uc5b5\uc6d0, \uc5f0\ubd09 3\uc5b5\uc6d0\uc5d0 \ucd1d\uc561 13\uc5b5\uc6d0\uc73c\ub85c, \ud50c\ub7ec\uc2a4 \uc635\uc158\uc774 \ubd99\uc5b4\uc788\ub2e4. \ud55c\ud654\uc758 \uc774\uc0c1\uad70 \ud300\uc7a5\uc740 20\uc77c\uc774 \uc790\uc815\uc774 \ub118\uc790\ub9c8\uc790 \uc1a1\uc2e0\uc601\uc744 \ucc3e\uc544\uac00 \ud611\uc0c1\uc744 \ubc8c\uc600\uace0, \uc1a1\uc2e0\uc601\ub3c4 \uc790\uc2e0\uc744 \uc560\ud0c0\uac8c \ucc3e\uc544\uc640 \uc900 \ud55c\ud654 \uad6c\ub2e8\uc758 \uc131\uc758\uc5d0 \uac10\ub3d9\ud574 \uacc4\uc57d\uc774 \uc218\uc6d4\ud788 \uc9c4\ud589\ub418\uc5c8\ub2e4. \uc9c1\uc811 \uad6c\ub2e8\uc5d0 \uc1a1\uc2e0\uc601\uc744 \uc7a1\uc73c\ub77c \uc694\uccad\ud588\uc5c8\ub358 \ud55c\ub300\ud654 \uac10\ub3c5\uc740 \uc774 FA \uc601\uc785\uc5d0 \ud070 \ub9cc\uc871\uac10\uc744 \ub4e4\uc5b4\ub0c8\uace0, \uadf8\ub294 \uc1a1\uc2e0\uc601\uc5d0\uac8c \ubc14\ud2f0\uc2a4\ud0c0, \ubc15\uc815\uc9c4\uacfc \ud568\uaed8 \ud55c\ud654 \ub9c8\uc6b4\ub4dc\uc758 \ud5c8\ub9ac\ub97c \ucc45\uc784\uc9c8 \uc14b\uc5c5\ub9e8 \uc5ed\ud560\uc744 \ub9e1\uacbc\ub2e4. \ud55c\ud3b8, LG\ub294 \uc1a1\uc2e0\uc601\uc5d0 \ub300\ud55c \ubcf4\uc0c1 \uc120\uc218\ub85c \ud3ec\uc218 \ub098\uc131\uc6a9\uc744 \ub370\ub824\uac14\ub2e4.", "sentence_answer": "\uc1a1\uc2e0\uc601 \ub610\ud55c \uc774\ud0dd\uadfc\uacfc \ub9c8\ucc2c\uac00\uc9c0\ub85c \ud0c0 \uad6c\ub2e8\uacfc\uc758 \ud611\uc0c1 \ub0a0\uc9dc\uac00 \uc2dc\uc791\ub418\uc790\ub9c8\uc790 \ud55c\ud654 \uc774\uae00\uc2a4 \uc640 FA\ub97c \uc131\uc0ac\uc2dc\ucf30\ub2e4.", "paragraph_id": "6533929-34-0", "paragraph_question": "question: \uc1a1\uc2e0\uc601\uc740 \uc5b4\ub5a4 \ud300\uacfc FA\ub97c \uc131\uc0ac\uc2dc\ucf30\ub294\uac00?, context: \uc1a1\uc2e0\uc601 \ub610\ud55c \uc774\ud0dd\uadfc\uacfc \ub9c8\ucc2c\uac00\uc9c0\ub85c \ud0c0 \uad6c\ub2e8\uacfc\uc758 \ud611\uc0c1 \ub0a0\uc9dc\uac00 \uc2dc\uc791\ub418\uc790\ub9c8\uc790 \ud55c\ud654 \uc774\uae00\uc2a4\uc640 FA\ub97c \uc131\uc0ac\uc2dc\ucf30\ub2e4. \uacc4\uc57d \uc870\uac74\uc740 \uacc4\uc57d \uae30\uac04 3\ub144 \ub3d9\uc548 \uacc4\uc57d\uae08 4\uc5b5\uc6d0, \uc5f0\ubd09 3\uc5b5\uc6d0\uc5d0 \ucd1d\uc561 13\uc5b5\uc6d0\uc73c\ub85c, \ud50c\ub7ec\uc2a4 \uc635\uc158\uc774 \ubd99\uc5b4\uc788\ub2e4. \ud55c\ud654\uc758 \uc774\uc0c1\uad70 \ud300\uc7a5\uc740 20\uc77c\uc774 \uc790\uc815\uc774 \ub118\uc790\ub9c8\uc790 \uc1a1\uc2e0\uc601\uc744 \ucc3e\uc544\uac00 \ud611\uc0c1\uc744 \ubc8c\uc600\uace0, \uc1a1\uc2e0\uc601\ub3c4 \uc790\uc2e0\uc744 \uc560\ud0c0\uac8c \ucc3e\uc544\uc640 \uc900 \ud55c\ud654 \uad6c\ub2e8\uc758 \uc131\uc758\uc5d0 \uac10\ub3d9\ud574 \uacc4\uc57d\uc774 \uc218\uc6d4\ud788 \uc9c4\ud589\ub418\uc5c8\ub2e4. \uc9c1\uc811 \uad6c\ub2e8\uc5d0 \uc1a1\uc2e0\uc601\uc744 \uc7a1\uc73c\ub77c \uc694\uccad\ud588\uc5c8\ub358 \ud55c\ub300\ud654 \uac10\ub3c5\uc740 \uc774 FA \uc601\uc785\uc5d0 \ud070 \ub9cc\uc871\uac10\uc744 \ub4e4\uc5b4\ub0c8\uace0, \uadf8\ub294 \uc1a1\uc2e0\uc601\uc5d0\uac8c \ubc14\ud2f0\uc2a4\ud0c0, \ubc15\uc815\uc9c4\uacfc \ud568\uaed8 \ud55c\ud654 \ub9c8\uc6b4\ub4dc\uc758 \ud5c8\ub9ac\ub97c \ucc45\uc784\uc9c8 \uc14b\uc5c5\ub9e8 \uc5ed\ud560\uc744 \ub9e1\uacbc\ub2e4. \ud55c\ud3b8, LG\ub294 \uc1a1\uc2e0\uc601\uc5d0 \ub300\ud55c \ubcf4\uc0c1 \uc120\uc218\ub85c \ud3ec\uc218 \ub098\uc131\uc6a9\uc744 \ub370\ub824\uac14\ub2e4."} {"question": "\ub9cc\uc288\ud0c0\uc778\uc774 \uc81c5\ud504\ub85c\uc774\uc13c\ubcf4\ubcd1\uc5f0\ub300 \uc81c6\uc911\ub300 \uc911\ub300\uc7a5\uc73c\ub85c \uc784\uba85\ub41c \ud574\ub294?", "paragraph": "\ub9cc\uc288\ud0c0\uc778\uc740 1\ucc28\ub300\uc804 \uc885\uc804 \uc774\ud6c4\uc5d0\ub3c4 \uad70\uc5d0 \ub0a8\uc558\ub2e4. 1918\ub144 \ube0c\ub808\uc2ac\ub77c\uc6b0(\uc624\ub298\ub0a0\uc758 \ube0c\ub85c\uce20\uc640\ud504)\uc5d0\uc11c \uc804\uc120\ubc29\uc704\uad70 \ucc38\ubaa8\ub85c \uc790\uc6d0\ud588\uace0 1919\ub144\uae4c\uc9c0 \uac70\uae30\uc11c \uadfc\ubb34\ud588\ub2e4. \uc81c2\uad70\ub2e8 \uc0ac\ub839\ubd80\uc5d0 \uc18c\uc18d\ub41c \ub9cc\uc288\ud0c0\uc778\uc740 500,000 \uc5ec\uba85 \uac00\ub7c9\uc758 \ub3c5\uc77c \uc81c\uad6d\uc721\uad70 \ubcd1\ub825\uc744 \uad6d\uac00\ubc29\uc704\uad70 \uc804\ub825\uc73c\ub85c \uad6c\uc870\uc870\uc815\ud558\ub294 \ub370 \ucc38\uc5ec\ud588\ub2e4. \uad6d\uac00\ubc29\uc704\uad70\uc740 \ubc14\uc774\ub9c8\ub974 \uacf5\ud654\uad6d\uc758 \uad70\ub300\ub85c\uc11c, \ubca0\ub974\uc0ac\uc720 \uc870\uc57d\uc5d0 \uc758\ud574 \ubcd1\ub825 \uc218\uac00 100,000 \uba85\uc73c\ub85c \uc81c\ud55c\ub418\uc5b4 \uc788\uc5c8\ub2e4. \uc80a\uc5b4\uc11c\ubd80\ud130 \ub2a5\ub825 \uc788\ub294 \uc9c0\ud718\uad00\uc73c\ub85c \ucd09\ub9dd\ubc1b\ub358 \ub9cc\uc288\ud0c0\uc778\uc740 \ubca0\ub974\uc0ac\uc720 \uc870\uc57d\uc774 \ud5c8\ub77d\ud558\ub294 4,000 \uba85 \uc7a5\uad50 \uc911 \ud55c \uba85\uc73c\ub85c \uc120\ud0dd\ub418\uc5b4 \uad70\uc9c1\uc744 \uc720\uc9c0\ud560 \uc218 \uc788\uc5c8\ub2e4. 1921\ub144 \ub9cc\uc288\ud0c0\uc778\uc740 \uc81c5\ud504\ub85c\uc774\uc13c\ubcf4\ubcd1\uc5f0\ub300 \uc81c6\uc911\ub300 \uc911\ub300\uc7a5\uc73c\ub85c \uc784\uba85\ub418\uc5c8\uace0 \uadf8 \ub4a4\uc5d0\ub294 \uc81c2\uad70\uad00\uad6c\uc0ac\ub839\ubd80 \ubc0f \uc81c4\uad70\uad00\uad6c\uc0ac\ub839\ubd80 \ucc38\ubaa8\uac00 \ub418\uc5b4 1927\ub144\uae4c\uc9c0 \uad70\uc0ac\uc0ac \ubc0f \uad70\uc0ac\uc804\uc220 \uad50\uc721\uc744 \ub9e1\uc558\ub2e4. \uadf8 \ud574\uc5d0 \uc18c\ub839\uc73c\ub85c \uc2b9\uc9c4\ud55c \ub9cc\uc288\ud0c0\uc778\uc740 \ubca0\ub97c\ub9b0\uc758 \uad6d\uac00\ubc29\uc704\uad70 \ubcf8\uccad \uc7a5\uad70\ucc38\ubaa8\uac00 \ub418\uc5c8\uace0, \uac01\uad6d \uad70\uc0ac\uc2dc\uc124\uc744 \ud0d0\ubc29\ud558\uba70 \uc721\uad70 \ub3d9\uc6d0\uacc4\ud68d \uc218\ub9bd\uc744 \uac70\ub4e4\uc5c8\ub2e4. \uc774\ud6c4 \uc911\ub839\uc73c\ub85c \uc9c4\uae09\ud55c \ub9cc\uc288\ud0c0\uc778\uc740 \uc81c4\ubcf4\ubcd1\uc5f0\ub300 \uacbd\ubcf4\ubcd1\ub300\ub300 \ub300\ub300\uc7a5\uc73c\ub85c \uc784\uba85\ub418\uc5b4 1934\ub144\uae4c\uc9c0 \uac70\uae30 \uc788\uc5c8\ub2e4. 1933\ub144 \uad6d\uac00\uc0ac\ud68c\uc8fc\uc758 \ub3c5\uc77c \ub178\ub3d9\uc790\ub2f9(\ub098\uce58\ub2f9)\uc774 \ub3c5\uc77c\uc758 \uad8c\ub825\uc744 \uc7a1\uc74c\uc73c\ub85c\uc368(\ub9c8\ud750\ud130\uadf8\ub77c\uc774\ud48d) \ubc14\uc774\ub9c8\ub974 \uacf5\ud654\uad6d\uc740 \ubd95\uad34\ud588\ub2e4. \uad6d\uac00\ubc29\uc704\uad70\uc740 1920\ub144\ub300\ubd80\ud130 \uc774\ubbf8 \ubca0\ub974\uc0ac\uc720 \uc870\uc57d\uc744 \uc5b4\uae30\uace0 \ubab0\ub798 \uc7ac\ubb34\uc7a5\uc744 \uc2dc\uc791\ud558\uace0 \uc788\uc5c8\uc73c\uba70, \ub098\uce58\ub2f9 \uc2e0\uc815\ubd80\ub294 \uacf5\uc2dd\uc801\uc73c\ub85c \uc870\uc57d \ud30c\uae30\ub97c \uc120\uc5b8\ud558\uace0 \ub300\uaddc\ubaa8 \uc7ac\ubb34\uc7a5\uacfc \uad70\ube44\ud655\uc7a5\uc5d0 \ub3cc\uc785\ud588\ub2e4.", "answer": "1921\ub144", "sentence": "1921\ub144 \ub9cc\uc288\ud0c0\uc778\uc740 \uc81c5\ud504\ub85c\uc774\uc13c\ubcf4\ubcd1\uc5f0\ub300 \uc81c6\uc911\ub300 \uc911\ub300\uc7a5\uc73c\ub85c \uc784\uba85\ub418\uc5c8\uace0 \uadf8 \ub4a4\uc5d0\ub294 \uc81c2\uad70\uad00\uad6c\uc0ac\ub839\ubd80 \ubc0f \uc81c4\uad70\uad00\uad6c\uc0ac\ub839\ubd80 \ucc38\ubaa8\uac00 \ub418\uc5b4 1927\ub144\uae4c\uc9c0 \uad70\uc0ac\uc0ac \ubc0f \uad70\uc0ac\uc804\uc220 \uad50\uc721\uc744 \ub9e1\uc558\ub2e4.", "paragraph_sentence": "\ub9cc\uc288\ud0c0\uc778\uc740 1\ucc28\ub300\uc804 \uc885\uc804 \uc774\ud6c4\uc5d0\ub3c4 \uad70\uc5d0 \ub0a8\uc558\ub2e4. 1918\ub144 \ube0c\ub808\uc2ac\ub77c\uc6b0(\uc624\ub298\ub0a0\uc758 \ube0c\ub85c\uce20\uc640\ud504)\uc5d0\uc11c \uc804\uc120\ubc29\uc704\uad70 \ucc38\ubaa8\ub85c \uc790\uc6d0\ud588\uace0 1919\ub144\uae4c\uc9c0 \uac70\uae30\uc11c \uadfc\ubb34\ud588\ub2e4. \uc81c2\uad70\ub2e8 \uc0ac\ub839\ubd80\uc5d0 \uc18c\uc18d\ub41c \ub9cc\uc288\ud0c0\uc778\uc740 500,000 \uc5ec\uba85 \uac00\ub7c9\uc758 \ub3c5\uc77c \uc81c\uad6d\uc721\uad70 \ubcd1\ub825\uc744 \uad6d\uac00\ubc29\uc704\uad70 \uc804\ub825\uc73c\ub85c \uad6c\uc870\uc870\uc815\ud558\ub294 \ub370 \ucc38\uc5ec\ud588\ub2e4. \uad6d\uac00\ubc29\uc704\uad70\uc740 \ubc14\uc774\ub9c8\ub974 \uacf5\ud654\uad6d\uc758 \uad70\ub300\ub85c\uc11c, \ubca0\ub974\uc0ac\uc720 \uc870\uc57d\uc5d0 \uc758\ud574 \ubcd1\ub825 \uc218\uac00 100,000 \uba85\uc73c\ub85c \uc81c\ud55c\ub418\uc5b4 \uc788\uc5c8\ub2e4. \uc80a\uc5b4\uc11c\ubd80\ud130 \ub2a5\ub825 \uc788\ub294 \uc9c0\ud718\uad00\uc73c\ub85c \ucd09\ub9dd\ubc1b\ub358 \ub9cc\uc288\ud0c0\uc778\uc740 \ubca0\ub974\uc0ac\uc720 \uc870\uc57d\uc774 \ud5c8\ub77d\ud558\ub294 4,000 \uba85 \uc7a5\uad50 \uc911 \ud55c \uba85\uc73c\ub85c \uc120\ud0dd\ub418\uc5b4 \uad70\uc9c1\uc744 \uc720\uc9c0\ud560 \uc218 \uc788\uc5c8\ub2e4. 1921\ub144 \ub9cc\uc288\ud0c0\uc778\uc740 \uc81c5\ud504\ub85c\uc774\uc13c\ubcf4\ubcd1\uc5f0\ub300 \uc81c6\uc911\ub300 \uc911\ub300\uc7a5\uc73c\ub85c \uc784\uba85\ub418\uc5c8\uace0 \uadf8 \ub4a4\uc5d0\ub294 \uc81c2\uad70\uad00\uad6c\uc0ac\ub839\ubd80 \ubc0f \uc81c4\uad70\uad00\uad6c\uc0ac\ub839\ubd80 \ucc38\ubaa8\uac00 \ub418\uc5b4 1927\ub144\uae4c\uc9c0 \uad70\uc0ac\uc0ac \ubc0f \uad70\uc0ac\uc804\uc220 \uad50\uc721\uc744 \ub9e1\uc558\ub2e4. \uadf8 \ud574\uc5d0 \uc18c\ub839\uc73c\ub85c \uc2b9\uc9c4\ud55c \ub9cc\uc288\ud0c0\uc778\uc740 \ubca0\ub97c\ub9b0\uc758 \uad6d\uac00\ubc29\uc704\uad70 \ubcf8\uccad \uc7a5\uad70\ucc38\ubaa8\uac00 \ub418\uc5c8\uace0, \uac01\uad6d \uad70\uc0ac\uc2dc\uc124\uc744 \ud0d0\ubc29\ud558\uba70 \uc721\uad70 \ub3d9\uc6d0\uacc4\ud68d \uc218\ub9bd\uc744 \uac70\ub4e4\uc5c8\ub2e4. \uc774\ud6c4 \uc911\ub839\uc73c\ub85c \uc9c4\uae09\ud55c \ub9cc\uc288\ud0c0\uc778\uc740 \uc81c4\ubcf4\ubcd1\uc5f0\ub300 \uacbd\ubcf4\ubcd1\ub300\ub300 \ub300\ub300\uc7a5\uc73c\ub85c \uc784\uba85\ub418\uc5b4 1934\ub144\uae4c\uc9c0 \uac70\uae30 \uc788\uc5c8\ub2e4. 1933\ub144 \uad6d\uac00\uc0ac\ud68c\uc8fc\uc758 \ub3c5\uc77c \ub178\ub3d9\uc790\ub2f9(\ub098\uce58\ub2f9)\uc774 \ub3c5\uc77c\uc758 \uad8c\ub825\uc744 \uc7a1\uc74c\uc73c\ub85c\uc368(\ub9c8\ud750\ud130\uadf8\ub77c\uc774\ud48d) \ubc14\uc774\ub9c8\ub974 \uacf5\ud654\uad6d\uc740 \ubd95\uad34\ud588\ub2e4. \uad6d\uac00\ubc29\uc704\uad70\uc740 1920\ub144\ub300\ubd80\ud130 \uc774\ubbf8 \ubca0\ub974\uc0ac\uc720 \uc870\uc57d\uc744 \uc5b4\uae30\uace0 \ubab0\ub798 \uc7ac\ubb34\uc7a5\uc744 \uc2dc\uc791\ud558\uace0 \uc788\uc5c8\uc73c\uba70, \ub098\uce58\ub2f9 \uc2e0\uc815\ubd80\ub294 \uacf5\uc2dd\uc801\uc73c\ub85c \uc870\uc57d \ud30c\uae30\ub97c \uc120\uc5b8\ud558\uace0 \ub300\uaddc\ubaa8 \uc7ac\ubb34\uc7a5\uacfc \uad70\ube44\ud655\uc7a5\uc5d0 \ub3cc\uc785\ud588\ub2e4.", "paragraph_answer": "\ub9cc\uc288\ud0c0\uc778\uc740 1\ucc28\ub300\uc804 \uc885\uc804 \uc774\ud6c4\uc5d0\ub3c4 \uad70\uc5d0 \ub0a8\uc558\ub2e4. 1918\ub144 \ube0c\ub808\uc2ac\ub77c\uc6b0(\uc624\ub298\ub0a0\uc758 \ube0c\ub85c\uce20\uc640\ud504)\uc5d0\uc11c \uc804\uc120\ubc29\uc704\uad70 \ucc38\ubaa8\ub85c \uc790\uc6d0\ud588\uace0 1919\ub144\uae4c\uc9c0 \uac70\uae30\uc11c \uadfc\ubb34\ud588\ub2e4. \uc81c2\uad70\ub2e8 \uc0ac\ub839\ubd80\uc5d0 \uc18c\uc18d\ub41c \ub9cc\uc288\ud0c0\uc778\uc740 500,000 \uc5ec\uba85 \uac00\ub7c9\uc758 \ub3c5\uc77c \uc81c\uad6d\uc721\uad70 \ubcd1\ub825\uc744 \uad6d\uac00\ubc29\uc704\uad70 \uc804\ub825\uc73c\ub85c \uad6c\uc870\uc870\uc815\ud558\ub294 \ub370 \ucc38\uc5ec\ud588\ub2e4. \uad6d\uac00\ubc29\uc704\uad70\uc740 \ubc14\uc774\ub9c8\ub974 \uacf5\ud654\uad6d\uc758 \uad70\ub300\ub85c\uc11c, \ubca0\ub974\uc0ac\uc720 \uc870\uc57d\uc5d0 \uc758\ud574 \ubcd1\ub825 \uc218\uac00 100,000 \uba85\uc73c\ub85c \uc81c\ud55c\ub418\uc5b4 \uc788\uc5c8\ub2e4. \uc80a\uc5b4\uc11c\ubd80\ud130 \ub2a5\ub825 \uc788\ub294 \uc9c0\ud718\uad00\uc73c\ub85c \ucd09\ub9dd\ubc1b\ub358 \ub9cc\uc288\ud0c0\uc778\uc740 \ubca0\ub974\uc0ac\uc720 \uc870\uc57d\uc774 \ud5c8\ub77d\ud558\ub294 4,000 \uba85 \uc7a5\uad50 \uc911 \ud55c \uba85\uc73c\ub85c \uc120\ud0dd\ub418\uc5b4 \uad70\uc9c1\uc744 \uc720\uc9c0\ud560 \uc218 \uc788\uc5c8\ub2e4. 1921\ub144 \ub9cc\uc288\ud0c0\uc778\uc740 \uc81c5\ud504\ub85c\uc774\uc13c\ubcf4\ubcd1\uc5f0\ub300 \uc81c6\uc911\ub300 \uc911\ub300\uc7a5\uc73c\ub85c \uc784\uba85\ub418\uc5c8\uace0 \uadf8 \ub4a4\uc5d0\ub294 \uc81c2\uad70\uad00\uad6c\uc0ac\ub839\ubd80 \ubc0f \uc81c4\uad70\uad00\uad6c\uc0ac\ub839\ubd80 \ucc38\ubaa8\uac00 \ub418\uc5b4 1927\ub144\uae4c\uc9c0 \uad70\uc0ac\uc0ac \ubc0f \uad70\uc0ac\uc804\uc220 \uad50\uc721\uc744 \ub9e1\uc558\ub2e4. \uadf8 \ud574\uc5d0 \uc18c\ub839\uc73c\ub85c \uc2b9\uc9c4\ud55c \ub9cc\uc288\ud0c0\uc778\uc740 \ubca0\ub97c\ub9b0\uc758 \uad6d\uac00\ubc29\uc704\uad70 \ubcf8\uccad \uc7a5\uad70\ucc38\ubaa8\uac00 \ub418\uc5c8\uace0, \uac01\uad6d \uad70\uc0ac\uc2dc\uc124\uc744 \ud0d0\ubc29\ud558\uba70 \uc721\uad70 \ub3d9\uc6d0\uacc4\ud68d \uc218\ub9bd\uc744 \uac70\ub4e4\uc5c8\ub2e4. \uc774\ud6c4 \uc911\ub839\uc73c\ub85c \uc9c4\uae09\ud55c \ub9cc\uc288\ud0c0\uc778\uc740 \uc81c4\ubcf4\ubcd1\uc5f0\ub300 \uacbd\ubcf4\ubcd1\ub300\ub300 \ub300\ub300\uc7a5\uc73c\ub85c \uc784\uba85\ub418\uc5b4 1934\ub144\uae4c\uc9c0 \uac70\uae30 \uc788\uc5c8\ub2e4. 1933\ub144 \uad6d\uac00\uc0ac\ud68c\uc8fc\uc758 \ub3c5\uc77c \ub178\ub3d9\uc790\ub2f9(\ub098\uce58\ub2f9)\uc774 \ub3c5\uc77c\uc758 \uad8c\ub825\uc744 \uc7a1\uc74c\uc73c\ub85c\uc368(\ub9c8\ud750\ud130\uadf8\ub77c\uc774\ud48d) \ubc14\uc774\ub9c8\ub974 \uacf5\ud654\uad6d\uc740 \ubd95\uad34\ud588\ub2e4. \uad6d\uac00\ubc29\uc704\uad70\uc740 1920\ub144\ub300\ubd80\ud130 \uc774\ubbf8 \ubca0\ub974\uc0ac\uc720 \uc870\uc57d\uc744 \uc5b4\uae30\uace0 \ubab0\ub798 \uc7ac\ubb34\uc7a5\uc744 \uc2dc\uc791\ud558\uace0 \uc788\uc5c8\uc73c\uba70, \ub098\uce58\ub2f9 \uc2e0\uc815\ubd80\ub294 \uacf5\uc2dd\uc801\uc73c\ub85c \uc870\uc57d \ud30c\uae30\ub97c \uc120\uc5b8\ud558\uace0 \ub300\uaddc\ubaa8 \uc7ac\ubb34\uc7a5\uacfc \uad70\ube44\ud655\uc7a5\uc5d0 \ub3cc\uc785\ud588\ub2e4.", "sentence_answer": " 1921\ub144 \ub9cc\uc288\ud0c0\uc778\uc740 \uc81c5\ud504\ub85c\uc774\uc13c\ubcf4\ubcd1\uc5f0\ub300 \uc81c6\uc911\ub300 \uc911\ub300\uc7a5\uc73c\ub85c \uc784\uba85\ub418\uc5c8\uace0 \uadf8 \ub4a4\uc5d0\ub294 \uc81c2\uad70\uad00\uad6c\uc0ac\ub839\ubd80 \ubc0f \uc81c4\uad70\uad00\uad6c\uc0ac\ub839\ubd80 \ucc38\ubaa8\uac00 \ub418\uc5b4 1927\ub144\uae4c\uc9c0 \uad70\uc0ac\uc0ac \ubc0f \uad70\uc0ac\uc804\uc220 \uad50\uc721\uc744 \ub9e1\uc558\ub2e4.", "paragraph_id": "6540644-6-1", "paragraph_question": "question: \ub9cc\uc288\ud0c0\uc778\uc774 \uc81c5\ud504\ub85c\uc774\uc13c\ubcf4\ubcd1\uc5f0\ub300 \uc81c6\uc911\ub300 \uc911\ub300\uc7a5\uc73c\ub85c \uc784\uba85\ub41c \ud574\ub294?, context: \ub9cc\uc288\ud0c0\uc778\uc740 1\ucc28\ub300\uc804 \uc885\uc804 \uc774\ud6c4\uc5d0\ub3c4 \uad70\uc5d0 \ub0a8\uc558\ub2e4. 1918\ub144 \ube0c\ub808\uc2ac\ub77c\uc6b0(\uc624\ub298\ub0a0\uc758 \ube0c\ub85c\uce20\uc640\ud504)\uc5d0\uc11c \uc804\uc120\ubc29\uc704\uad70 \ucc38\ubaa8\ub85c \uc790\uc6d0\ud588\uace0 1919\ub144\uae4c\uc9c0 \uac70\uae30\uc11c \uadfc\ubb34\ud588\ub2e4. \uc81c2\uad70\ub2e8 \uc0ac\ub839\ubd80\uc5d0 \uc18c\uc18d\ub41c \ub9cc\uc288\ud0c0\uc778\uc740 500,000 \uc5ec\uba85 \uac00\ub7c9\uc758 \ub3c5\uc77c \uc81c\uad6d\uc721\uad70 \ubcd1\ub825\uc744 \uad6d\uac00\ubc29\uc704\uad70 \uc804\ub825\uc73c\ub85c \uad6c\uc870\uc870\uc815\ud558\ub294 \ub370 \ucc38\uc5ec\ud588\ub2e4. \uad6d\uac00\ubc29\uc704\uad70\uc740 \ubc14\uc774\ub9c8\ub974 \uacf5\ud654\uad6d\uc758 \uad70\ub300\ub85c\uc11c, \ubca0\ub974\uc0ac\uc720 \uc870\uc57d\uc5d0 \uc758\ud574 \ubcd1\ub825 \uc218\uac00 100,000 \uba85\uc73c\ub85c \uc81c\ud55c\ub418\uc5b4 \uc788\uc5c8\ub2e4. \uc80a\uc5b4\uc11c\ubd80\ud130 \ub2a5\ub825 \uc788\ub294 \uc9c0\ud718\uad00\uc73c\ub85c \ucd09\ub9dd\ubc1b\ub358 \ub9cc\uc288\ud0c0\uc778\uc740 \ubca0\ub974\uc0ac\uc720 \uc870\uc57d\uc774 \ud5c8\ub77d\ud558\ub294 4,000 \uba85 \uc7a5\uad50 \uc911 \ud55c \uba85\uc73c\ub85c \uc120\ud0dd\ub418\uc5b4 \uad70\uc9c1\uc744 \uc720\uc9c0\ud560 \uc218 \uc788\uc5c8\ub2e4. 1921\ub144 \ub9cc\uc288\ud0c0\uc778\uc740 \uc81c5\ud504\ub85c\uc774\uc13c\ubcf4\ubcd1\uc5f0\ub300 \uc81c6\uc911\ub300 \uc911\ub300\uc7a5\uc73c\ub85c \uc784\uba85\ub418\uc5c8\uace0 \uadf8 \ub4a4\uc5d0\ub294 \uc81c2\uad70\uad00\uad6c\uc0ac\ub839\ubd80 \ubc0f \uc81c4\uad70\uad00\uad6c\uc0ac\ub839\ubd80 \ucc38\ubaa8\uac00 \ub418\uc5b4 1927\ub144\uae4c\uc9c0 \uad70\uc0ac\uc0ac \ubc0f \uad70\uc0ac\uc804\uc220 \uad50\uc721\uc744 \ub9e1\uc558\ub2e4. \uadf8 \ud574\uc5d0 \uc18c\ub839\uc73c\ub85c \uc2b9\uc9c4\ud55c \ub9cc\uc288\ud0c0\uc778\uc740 \ubca0\ub97c\ub9b0\uc758 \uad6d\uac00\ubc29\uc704\uad70 \ubcf8\uccad \uc7a5\uad70\ucc38\ubaa8\uac00 \ub418\uc5c8\uace0, \uac01\uad6d \uad70\uc0ac\uc2dc\uc124\uc744 \ud0d0\ubc29\ud558\uba70 \uc721\uad70 \ub3d9\uc6d0\uacc4\ud68d \uc218\ub9bd\uc744 \uac70\ub4e4\uc5c8\ub2e4. \uc774\ud6c4 \uc911\ub839\uc73c\ub85c \uc9c4\uae09\ud55c \ub9cc\uc288\ud0c0\uc778\uc740 \uc81c4\ubcf4\ubcd1\uc5f0\ub300 \uacbd\ubcf4\ubcd1\ub300\ub300 \ub300\ub300\uc7a5\uc73c\ub85c \uc784\uba85\ub418\uc5b4 1934\ub144\uae4c\uc9c0 \uac70\uae30 \uc788\uc5c8\ub2e4. 1933\ub144 \uad6d\uac00\uc0ac\ud68c\uc8fc\uc758 \ub3c5\uc77c \ub178\ub3d9\uc790\ub2f9(\ub098\uce58\ub2f9)\uc774 \ub3c5\uc77c\uc758 \uad8c\ub825\uc744 \uc7a1\uc74c\uc73c\ub85c\uc368(\ub9c8\ud750\ud130\uadf8\ub77c\uc774\ud48d) \ubc14\uc774\ub9c8\ub974 \uacf5\ud654\uad6d\uc740 \ubd95\uad34\ud588\ub2e4. \uad6d\uac00\ubc29\uc704\uad70\uc740 1920\ub144\ub300\ubd80\ud130 \uc774\ubbf8 \ubca0\ub974\uc0ac\uc720 \uc870\uc57d\uc744 \uc5b4\uae30\uace0 \ubab0\ub798 \uc7ac\ubb34\uc7a5\uc744 \uc2dc\uc791\ud558\uace0 \uc788\uc5c8\uc73c\uba70, \ub098\uce58\ub2f9 \uc2e0\uc815\ubd80\ub294 \uacf5\uc2dd\uc801\uc73c\ub85c \uc870\uc57d \ud30c\uae30\ub97c \uc120\uc5b8\ud558\uace0 \ub300\uaddc\ubaa8 \uc7ac\ubb34\uc7a5\uacfc \uad70\ube44\ud655\uc7a5\uc5d0 \ub3cc\uc785\ud588\ub2e4."} {"question": "\uac15\ubbfc\uaddc \uad50\uac10\uc774 \uc878\uc5c5\ud55c \uc0ac\ubc94\ub300\ub294 \uc5b4\ub290 \ub300\ud559\uc758 \uc0ac\ubc94\ub300\uc778\uac00?", "paragraph": "4\uc6d4 18\uc77c \uc624\ud6c4 4\uc2dc 5\ubd84 \uacbd, \uc218\ud559\uc5ec\ud589 \uc778\uc194\uc790\ub85c \uac14\ub2e4\uac00 \uc0ac\uace0\ub97c \ub2f9\ud558\uace0 \uad6c\uc870\ub41c \uc548\uc0b0 \ub2e8\uc6d0\uace0\ub4f1\ud559\uad50 \uac15\ubbfc\uaddc \uad50\uac10\uc774 \uc804\ub0a8 \uc9c4\ub3c4\uccb4\uc721\uad00 \ub4a4\ud3b8 \uc57c\uc0b0\uc5d0\uc11c \ubaa9\uc744 \ub9e4 \uc228\uc9c4 \ucc44 \ubc1c\uacac\ub418\uc5c8\ub2e4. \uad50\uac10\uc758 \uc9c0\uac11\uc5d0\uc11c\ub294 \uc720\uc11c\uac00 \ubc1c\uacac\ub418\uc5c8\ub2e4. \uc720\uc11c\uc5d0\ub294 \"200\uba85\uc758 \uc0dd\uc0ac\ub97c \uc54c \uc218 \uc5c6\ub294\ub370 \ud63c\uc790 \uc0b4\uae30\uc5d0\ub294 \ud798\uc774 \ubc85\ucc28\ub2e4. \ub098\uc5d0\uac8c \ubaa8\ub4e0 \ucc45\uc784\uc744 \uc9c0\uc6cc \ub2ec\ub77c. \ub0b4\uac00 \uc218\ud559\uc5ec\ud589\uc744 \ucd94\uc9c4\ud588\ub2e4\"\uba70 \uc2dc\uc2e0\uc740 \ud654\uc7a5\ud55c \ub4a4 \uc0ac\uace0 \ud574\uc5ed\uc5d0 \ubfcc\ub824 \ub2ec\ub77c\ub294 \ub0b4\uc6a9\uc774 \ub2f4\uaca8\uc788\ub2e4. \uc774\uc5b4 \"\uc2dc\uc2e0\uc744 \ucc3e\uc9c0 \ubabb\ud558\ub294 \ub140\uc11d\ub4e4\uacfc \ud568\uaed8 \uc800\uc2b9\uc5d0\uc11c\ub3c4 \uc120\uc0dd\uc744 \ud560\uae4c\"\ub77c\ub294 \ub0b4\uc6a9\ub3c4 \uc801\ud600\uc788\uc5c8\ub2e4. \uc790\uc0b4 \uc6d0\uc778 \uc911 \ud558\ub098\ub85c \ucd94\uc815\ub418\ub294 \uac83\uc774 \uc788\ub2e4. \uadf8\ub294 \ud559\uc0dd\ub4e4\uc744 \ub0a8\uaca8\ub450\uace0 \uad6c\uc870\ub41c \uac83\uc5d0 \ub300\ud574 \uc8fc\uc704\uc5d0 \u201c\ub098\ub9cc \ud63c\uc790 \ube60\uc838\ub098\uc628 \uac83 \uac19\uc544 \uad34\ub86d\ub2e4\u201d\uace0 \ud638\uc18c\ud588\uc73c\uba70, \uadf8\ub7f0 \uac00\uc6b4\ub370 \uc5ec\uac1d\uc120 \uce68\ubab0\ub85c \uc0ac\ub9dd\ud558\uac70\ub098 \uc2e4\uc885\ub41c \ub2e8\uc6d0\uace0 \ud559\uc0dd \ud559\ubd80\ubaa8\ub4e4\ub85c\ubd80\ud130 \u201c\uc65c \ub2f9\uc2e0\ub9cc \uc0b4\uc544 \ub3cc\uc544\uc654\ub290\ub0d0\u201d\ub294 \ub4f1 \uac15\ud55c \uc9c8\ud0c0\ub97c \ubc1b\uc740 \uac83\uc73c\ub85c \uc54c\ub824\uc84c\ub2e4. \uac15\ubbfc\uaddc \uad50\uac10\uc740 \uacf5\uc8fc\ub300\ud559\uad50 \uc0ac\ubc94\ub300\ud559\uc744 \uc878\uc5c5 \ud6c4 1987\ub144 \uc911\ub4f1\ud559\uad50 \uad50\uc0ac\ub85c \uc784\uc6a9\ub418\uc5b4 \uc724\ub9ac \uacfc\ubaa9\uc744 \uac00\ub974\ucce4\ub2e4. 2012\ub144 3\uc6d4 \uad50\uac10\uc73c\ub85c \uc2b9\uc9c4\ud558\uc600\uc73c\uba70, 2014\ub144 3\uc6d4 1\uc77c\uc790\ub85c \ub2e8\uc6d0\uace0\ub4f1\ud559\uad50\uc5d0 \ubd80\uc784\ud558\uc600\ub2e4.", "answer": "\uacf5\uc8fc\ub300\ud559\uad50", "sentence": "\uac15\ubbfc\uaddc \uad50\uac10\uc740 \uacf5\uc8fc\ub300\ud559\uad50 \uc0ac\ubc94\ub300\ud559\uc744 \uc878\uc5c5 \ud6c4 1987\ub144 \uc911\ub4f1\ud559\uad50 \uad50\uc0ac\ub85c \uc784\uc6a9\ub418\uc5b4 \uc724\ub9ac \uacfc\ubaa9\uc744 \uac00\ub974\ucce4\ub2e4.", "paragraph_sentence": "4\uc6d4 18\uc77c \uc624\ud6c4 4\uc2dc 5\ubd84 \uacbd, \uc218\ud559\uc5ec\ud589 \uc778\uc194\uc790\ub85c \uac14\ub2e4\uac00 \uc0ac\uace0\ub97c \ub2f9\ud558\uace0 \uad6c\uc870\ub41c \uc548\uc0b0 \ub2e8\uc6d0\uace0\ub4f1\ud559\uad50 \uac15\ubbfc\uaddc \uad50\uac10\uc774 \uc804\ub0a8 \uc9c4\ub3c4\uccb4\uc721\uad00 \ub4a4\ud3b8 \uc57c\uc0b0\uc5d0\uc11c \ubaa9\uc744 \ub9e4 \uc228\uc9c4 \ucc44 \ubc1c\uacac\ub418\uc5c8\ub2e4. \uad50\uac10\uc758 \uc9c0\uac11\uc5d0\uc11c\ub294 \uc720\uc11c\uac00 \ubc1c\uacac\ub418\uc5c8\ub2e4. \uc720\uc11c\uc5d0\ub294 \"200\uba85\uc758 \uc0dd\uc0ac\ub97c \uc54c \uc218 \uc5c6\ub294\ub370 \ud63c\uc790 \uc0b4\uae30\uc5d0\ub294 \ud798\uc774 \ubc85\ucc28\ub2e4. \ub098\uc5d0\uac8c \ubaa8\ub4e0 \ucc45\uc784\uc744 \uc9c0\uc6cc \ub2ec\ub77c. \ub0b4\uac00 \uc218\ud559\uc5ec\ud589\uc744 \ucd94\uc9c4\ud588\ub2e4\"\uba70 \uc2dc\uc2e0\uc740 \ud654\uc7a5\ud55c \ub4a4 \uc0ac\uace0 \ud574\uc5ed\uc5d0 \ubfcc\ub824 \ub2ec\ub77c\ub294 \ub0b4\uc6a9\uc774 \ub2f4\uaca8\uc788\ub2e4. \uc774\uc5b4 \"\uc2dc\uc2e0\uc744 \ucc3e\uc9c0 \ubabb\ud558\ub294 \ub140\uc11d\ub4e4\uacfc \ud568\uaed8 \uc800\uc2b9\uc5d0\uc11c\ub3c4 \uc120\uc0dd\uc744 \ud560\uae4c\"\ub77c\ub294 \ub0b4\uc6a9\ub3c4 \uc801\ud600\uc788\uc5c8\ub2e4. \uc790\uc0b4 \uc6d0\uc778 \uc911 \ud558\ub098\ub85c \ucd94\uc815\ub418\ub294 \uac83\uc774 \uc788\ub2e4. \uadf8\ub294 \ud559\uc0dd\ub4e4\uc744 \ub0a8\uaca8\ub450\uace0 \uad6c\uc870\ub41c \uac83\uc5d0 \ub300\ud574 \uc8fc\uc704\uc5d0 \u201c\ub098\ub9cc \ud63c\uc790 \ube60\uc838\ub098\uc628 \uac83 \uac19\uc544 \uad34\ub86d\ub2e4\u201d\uace0 \ud638\uc18c\ud588\uc73c\uba70, \uadf8\ub7f0 \uac00\uc6b4\ub370 \uc5ec\uac1d\uc120 \uce68\ubab0\ub85c \uc0ac\ub9dd\ud558\uac70\ub098 \uc2e4\uc885\ub41c \ub2e8\uc6d0\uace0 \ud559\uc0dd \ud559\ubd80\ubaa8\ub4e4\ub85c\ubd80\ud130 \u201c\uc65c \ub2f9\uc2e0\ub9cc \uc0b4\uc544 \ub3cc\uc544\uc654\ub290\ub0d0\u201d\ub294 \ub4f1 \uac15\ud55c \uc9c8\ud0c0\ub97c \ubc1b\uc740 \uac83\uc73c\ub85c \uc54c\ub824\uc84c\ub2e4. \uac15\ubbfc\uaddc \uad50\uac10\uc740 \uacf5\uc8fc\ub300\ud559\uad50 \uc0ac\ubc94\ub300\ud559\uc744 \uc878\uc5c5 \ud6c4 1987\ub144 \uc911\ub4f1\ud559\uad50 \uad50\uc0ac\ub85c \uc784\uc6a9\ub418\uc5b4 \uc724\ub9ac \uacfc\ubaa9\uc744 \uac00\ub974\ucce4\ub2e4. 2012\ub144 3\uc6d4 \uad50\uac10\uc73c\ub85c \uc2b9\uc9c4\ud558\uc600\uc73c\uba70, 2014\ub144 3\uc6d4 1\uc77c\uc790\ub85c \ub2e8\uc6d0\uace0\ub4f1\ud559\uad50\uc5d0 \ubd80\uc784\ud558\uc600\ub2e4.", "paragraph_answer": "4\uc6d4 18\uc77c \uc624\ud6c4 4\uc2dc 5\ubd84 \uacbd, \uc218\ud559\uc5ec\ud589 \uc778\uc194\uc790\ub85c \uac14\ub2e4\uac00 \uc0ac\uace0\ub97c \ub2f9\ud558\uace0 \uad6c\uc870\ub41c \uc548\uc0b0 \ub2e8\uc6d0\uace0\ub4f1\ud559\uad50 \uac15\ubbfc\uaddc \uad50\uac10\uc774 \uc804\ub0a8 \uc9c4\ub3c4\uccb4\uc721\uad00 \ub4a4\ud3b8 \uc57c\uc0b0\uc5d0\uc11c \ubaa9\uc744 \ub9e4 \uc228\uc9c4 \ucc44 \ubc1c\uacac\ub418\uc5c8\ub2e4. \uad50\uac10\uc758 \uc9c0\uac11\uc5d0\uc11c\ub294 \uc720\uc11c\uac00 \ubc1c\uacac\ub418\uc5c8\ub2e4. \uc720\uc11c\uc5d0\ub294 \"200\uba85\uc758 \uc0dd\uc0ac\ub97c \uc54c \uc218 \uc5c6\ub294\ub370 \ud63c\uc790 \uc0b4\uae30\uc5d0\ub294 \ud798\uc774 \ubc85\ucc28\ub2e4. \ub098\uc5d0\uac8c \ubaa8\ub4e0 \ucc45\uc784\uc744 \uc9c0\uc6cc \ub2ec\ub77c. \ub0b4\uac00 \uc218\ud559\uc5ec\ud589\uc744 \ucd94\uc9c4\ud588\ub2e4\"\uba70 \uc2dc\uc2e0\uc740 \ud654\uc7a5\ud55c \ub4a4 \uc0ac\uace0 \ud574\uc5ed\uc5d0 \ubfcc\ub824 \ub2ec\ub77c\ub294 \ub0b4\uc6a9\uc774 \ub2f4\uaca8\uc788\ub2e4. \uc774\uc5b4 \"\uc2dc\uc2e0\uc744 \ucc3e\uc9c0 \ubabb\ud558\ub294 \ub140\uc11d\ub4e4\uacfc \ud568\uaed8 \uc800\uc2b9\uc5d0\uc11c\ub3c4 \uc120\uc0dd\uc744 \ud560\uae4c\"\ub77c\ub294 \ub0b4\uc6a9\ub3c4 \uc801\ud600\uc788\uc5c8\ub2e4. \uc790\uc0b4 \uc6d0\uc778 \uc911 \ud558\ub098\ub85c \ucd94\uc815\ub418\ub294 \uac83\uc774 \uc788\ub2e4. \uadf8\ub294 \ud559\uc0dd\ub4e4\uc744 \ub0a8\uaca8\ub450\uace0 \uad6c\uc870\ub41c \uac83\uc5d0 \ub300\ud574 \uc8fc\uc704\uc5d0 \u201c\ub098\ub9cc \ud63c\uc790 \ube60\uc838\ub098\uc628 \uac83 \uac19\uc544 \uad34\ub86d\ub2e4\u201d\uace0 \ud638\uc18c\ud588\uc73c\uba70, \uadf8\ub7f0 \uac00\uc6b4\ub370 \uc5ec\uac1d\uc120 \uce68\ubab0\ub85c \uc0ac\ub9dd\ud558\uac70\ub098 \uc2e4\uc885\ub41c \ub2e8\uc6d0\uace0 \ud559\uc0dd \ud559\ubd80\ubaa8\ub4e4\ub85c\ubd80\ud130 \u201c\uc65c \ub2f9\uc2e0\ub9cc \uc0b4\uc544 \ub3cc\uc544\uc654\ub290\ub0d0\u201d\ub294 \ub4f1 \uac15\ud55c \uc9c8\ud0c0\ub97c \ubc1b\uc740 \uac83\uc73c\ub85c \uc54c\ub824\uc84c\ub2e4. \uac15\ubbfc\uaddc \uad50\uac10\uc740 \uacf5\uc8fc\ub300\ud559\uad50 \uc0ac\ubc94\ub300\ud559\uc744 \uc878\uc5c5 \ud6c4 1987\ub144 \uc911\ub4f1\ud559\uad50 \uad50\uc0ac\ub85c \uc784\uc6a9\ub418\uc5b4 \uc724\ub9ac \uacfc\ubaa9\uc744 \uac00\ub974\ucce4\ub2e4. 2012\ub144 3\uc6d4 \uad50\uac10\uc73c\ub85c \uc2b9\uc9c4\ud558\uc600\uc73c\uba70, 2014\ub144 3\uc6d4 1\uc77c\uc790\ub85c \ub2e8\uc6d0\uace0\ub4f1\ud559\uad50\uc5d0 \ubd80\uc784\ud558\uc600\ub2e4.", "sentence_answer": "\uac15\ubbfc\uaddc \uad50\uac10\uc740 \uacf5\uc8fc\ub300\ud559\uad50 \uc0ac\ubc94\ub300\ud559\uc744 \uc878\uc5c5 \ud6c4 1987\ub144 \uc911\ub4f1\ud559\uad50 \uad50\uc0ac\ub85c \uc784\uc6a9\ub418\uc5b4 \uc724\ub9ac \uacfc\ubaa9\uc744 \uac00\ub974\ucce4\ub2e4.", "paragraph_id": "6603760-16-1", "paragraph_question": "question: \uac15\ubbfc\uaddc \uad50\uac10\uc774 \uc878\uc5c5\ud55c \uc0ac\ubc94\ub300\ub294 \uc5b4\ub290 \ub300\ud559\uc758 \uc0ac\ubc94\ub300\uc778\uac00?, context: 4\uc6d4 18\uc77c \uc624\ud6c4 4\uc2dc 5\ubd84 \uacbd, \uc218\ud559\uc5ec\ud589 \uc778\uc194\uc790\ub85c \uac14\ub2e4\uac00 \uc0ac\uace0\ub97c \ub2f9\ud558\uace0 \uad6c\uc870\ub41c \uc548\uc0b0 \ub2e8\uc6d0\uace0\ub4f1\ud559\uad50 \uac15\ubbfc\uaddc \uad50\uac10\uc774 \uc804\ub0a8 \uc9c4\ub3c4\uccb4\uc721\uad00 \ub4a4\ud3b8 \uc57c\uc0b0\uc5d0\uc11c \ubaa9\uc744 \ub9e4 \uc228\uc9c4 \ucc44 \ubc1c\uacac\ub418\uc5c8\ub2e4. \uad50\uac10\uc758 \uc9c0\uac11\uc5d0\uc11c\ub294 \uc720\uc11c\uac00 \ubc1c\uacac\ub418\uc5c8\ub2e4. \uc720\uc11c\uc5d0\ub294 \"200\uba85\uc758 \uc0dd\uc0ac\ub97c \uc54c \uc218 \uc5c6\ub294\ub370 \ud63c\uc790 \uc0b4\uae30\uc5d0\ub294 \ud798\uc774 \ubc85\ucc28\ub2e4. \ub098\uc5d0\uac8c \ubaa8\ub4e0 \ucc45\uc784\uc744 \uc9c0\uc6cc \ub2ec\ub77c. \ub0b4\uac00 \uc218\ud559\uc5ec\ud589\uc744 \ucd94\uc9c4\ud588\ub2e4\"\uba70 \uc2dc\uc2e0\uc740 \ud654\uc7a5\ud55c \ub4a4 \uc0ac\uace0 \ud574\uc5ed\uc5d0 \ubfcc\ub824 \ub2ec\ub77c\ub294 \ub0b4\uc6a9\uc774 \ub2f4\uaca8\uc788\ub2e4. \uc774\uc5b4 \"\uc2dc\uc2e0\uc744 \ucc3e\uc9c0 \ubabb\ud558\ub294 \ub140\uc11d\ub4e4\uacfc \ud568\uaed8 \uc800\uc2b9\uc5d0\uc11c\ub3c4 \uc120\uc0dd\uc744 \ud560\uae4c\"\ub77c\ub294 \ub0b4\uc6a9\ub3c4 \uc801\ud600\uc788\uc5c8\ub2e4. \uc790\uc0b4 \uc6d0\uc778 \uc911 \ud558\ub098\ub85c \ucd94\uc815\ub418\ub294 \uac83\uc774 \uc788\ub2e4. \uadf8\ub294 \ud559\uc0dd\ub4e4\uc744 \ub0a8\uaca8\ub450\uace0 \uad6c\uc870\ub41c \uac83\uc5d0 \ub300\ud574 \uc8fc\uc704\uc5d0 \u201c\ub098\ub9cc \ud63c\uc790 \ube60\uc838\ub098\uc628 \uac83 \uac19\uc544 \uad34\ub86d\ub2e4\u201d\uace0 \ud638\uc18c\ud588\uc73c\uba70, \uadf8\ub7f0 \uac00\uc6b4\ub370 \uc5ec\uac1d\uc120 \uce68\ubab0\ub85c \uc0ac\ub9dd\ud558\uac70\ub098 \uc2e4\uc885\ub41c \ub2e8\uc6d0\uace0 \ud559\uc0dd \ud559\ubd80\ubaa8\ub4e4\ub85c\ubd80\ud130 \u201c\uc65c \ub2f9\uc2e0\ub9cc \uc0b4\uc544 \ub3cc\uc544\uc654\ub290\ub0d0\u201d\ub294 \ub4f1 \uac15\ud55c \uc9c8\ud0c0\ub97c \ubc1b\uc740 \uac83\uc73c\ub85c \uc54c\ub824\uc84c\ub2e4. \uac15\ubbfc\uaddc \uad50\uac10\uc740 \uacf5\uc8fc\ub300\ud559\uad50 \uc0ac\ubc94\ub300\ud559\uc744 \uc878\uc5c5 \ud6c4 1987\ub144 \uc911\ub4f1\ud559\uad50 \uad50\uc0ac\ub85c \uc784\uc6a9\ub418\uc5b4 \uc724\ub9ac \uacfc\ubaa9\uc744 \uac00\ub974\ucce4\ub2e4. 2012\ub144 3\uc6d4 \uad50\uac10\uc73c\ub85c \uc2b9\uc9c4\ud558\uc600\uc73c\uba70, 2014\ub144 3\uc6d4 1\uc77c\uc790\ub85c \ub2e8\uc6d0\uace0\ub4f1\ud559\uad50\uc5d0 \ubd80\uc784\ud558\uc600\ub2e4."} {"question": "\uc774\ub514 \uc544\ubbfc\uc744 \uc704\ud574 \uc6b0\uac04\ub2e4\uce21\uc5d0 \ud30c\ubcd1\ud55c \ub9ac\ube44\uc544\uc758 \uc778\ubb3c\uc740?", "paragraph": "\uc6b0\uac04\ub2e4\uad70\uc740 \ubc14\ub85c \ud1f4\uac01\uc744 \uac1c\uc2dc\ud588\ub294\ub370, \uac19\uc740 \ub3c5\uc7ac\uc790\uc600\ub358 \uc774\ub514 \uc544\ubbfc\uc744 \uc704\ud574 \ub9ac\ube44\uc544\uc758 \uce74\ub2e4\ud53c\ub294 T-54 \uc804\ucc28\uc640 T-55 \uc804\ucc28, BTR \ubcd1\ub825\uc218\uc1a1\ucc28, BM-21 \ub85c\ucf13\ucd94\uc9c4\ucc28, MiG-21, Tu-22 \ud3ed\uaca9\uae30 \ub4f1\uc73c\ub85c \ubb34\uc7a5\ud55c 2,500\uba85\uc758 \uad70\ub300\ub97c \uc6b0\uac04\ub2e4\uce21\uc5d0 \ud30c\ubcd1\ud588\ub2e4. \uadf8\ub7ec\ub098 \ub9ac\ube44\uc544\uad70\uc774 \uc804\uc120\uc5d0 \ub2e4\ub2e4\ub790\uc744 \ub54c \uc6b0\uac04\ub2e4\uad70\uc740 \uc57d\ud0c8\ud55c \ubb3c\uac74\uc744 \ucc44\uc6b4 \ud2b8\ub7ed\uacfc \ud568\uaed8 \ubc18\ub300\uce21\uc73c\ub85c \ud1f4\uac01\ud574\ubc84\ub838\ub2e4. \ub9ac\ube44\uc544\uad70\uc740 \ub9ac\ube44\uc544 \uc721\uad70\ubd80\ub300\uc640 \uc778\ubbfc \ubbfc\ubcd1\ub300, \uc774\uc2ac\ub78c \uad70\ub2e8 \ub4f1 \uc0ac\ud558\ub77c \uc0ac\ub9c9 \uc774\ub0a8\uc758 \uc544\ud504\ub9ac\uce78 \uc774\uc2ac\ub78c \uc138\ub825\uc744 \uaddc\ud569\ud55c \ud63c\ud569\uad70\uc774\uc5c8\ub2e4. \uc774\ub4e4\uc740 \ud0c4\uc790\ub2c8\uc544\uc640 UNLA\uc5d0 \ubc18\ub300\ud558\ub294 \uc778\uc0ac\ub4e4\uacfc \ud568\uaed8 \uce84\ud314\ub77c\ub85c \ud5a5\ud588\ub294\ub370, \ub8e8\uce74\uc57c\uc758 \ub2aa\uc9c0\ub300\uc5d0\uc11c \uc9c4\uad70\uc774 \uc815\uccb4\ub418\uc5c8\ub2e4. \uc774\uc5d0 \ud0c4\uc790\ub2c8\uc544 \uc815\uc608\uad70\uc774\uc5c8\ub358 \uc81c 208\uc5ec\ub2e8\uc740 \ub2aa \uc11c\ucabd \uac00\uc7a5\uc790\ub9ac\uc758 \ub451\uae38\uc744 \ud1b5\ud574 \uc81c 201 \uc5ec\ub2e8\uc744 \uc6b0\uc120 \ubcf4\ub0c8\ub2e4.", "answer": "\uce74\ub2e4\ud53c", "sentence": "\uc6b0\uac04\ub2e4\uad70\uc740 \ubc14\ub85c \ud1f4\uac01\uc744 \uac1c\uc2dc\ud588\ub294\ub370, \uac19\uc740 \ub3c5\uc7ac\uc790\uc600\ub358 \uc774\ub514 \uc544\ubbfc\uc744 \uc704\ud574 \ub9ac\ube44\uc544\uc758 \uce74\ub2e4\ud53c \ub294 T-54 \uc804\ucc28\uc640 T-55 \uc804\ucc28, BTR \ubcd1\ub825\uc218\uc1a1\ucc28, BM-21 \ub85c\ucf13\ucd94\uc9c4\ucc28, MiG-21, Tu-22 \ud3ed\uaca9\uae30 \ub4f1\uc73c\ub85c \ubb34\uc7a5\ud55c 2,500\uba85\uc758 \uad70\ub300\ub97c \uc6b0\uac04\ub2e4\uce21\uc5d0 \ud30c\ubcd1\ud588\ub2e4.", "paragraph_sentence": " \uc6b0\uac04\ub2e4\uad70\uc740 \ubc14\ub85c \ud1f4\uac01\uc744 \uac1c\uc2dc\ud588\ub294\ub370, \uac19\uc740 \ub3c5\uc7ac\uc790\uc600\ub358 \uc774\ub514 \uc544\ubbfc\uc744 \uc704\ud574 \ub9ac\ube44\uc544\uc758 \uce74\ub2e4\ud53c \ub294 T-54 \uc804\ucc28\uc640 T-55 \uc804\ucc28, BTR \ubcd1\ub825\uc218\uc1a1\ucc28, BM-21 \ub85c\ucf13\ucd94\uc9c4\ucc28, MiG-21, Tu-22 \ud3ed\uaca9\uae30 \ub4f1\uc73c\ub85c \ubb34\uc7a5\ud55c 2,500\uba85\uc758 \uad70\ub300\ub97c \uc6b0\uac04\ub2e4\uce21\uc5d0 \ud30c\ubcd1\ud588\ub2e4. \uadf8\ub7ec\ub098 \ub9ac\ube44\uc544\uad70\uc774 \uc804\uc120\uc5d0 \ub2e4\ub2e4\ub790\uc744 \ub54c \uc6b0\uac04\ub2e4\uad70\uc740 \uc57d\ud0c8\ud55c \ubb3c\uac74\uc744 \ucc44\uc6b4 \ud2b8\ub7ed\uacfc \ud568\uaed8 \ubc18\ub300\uce21\uc73c\ub85c \ud1f4\uac01\ud574\ubc84\ub838\ub2e4. \ub9ac\ube44\uc544\uad70\uc740 \ub9ac\ube44\uc544 \uc721\uad70\ubd80\ub300\uc640 \uc778\ubbfc \ubbfc\ubcd1\ub300, \uc774\uc2ac\ub78c \uad70\ub2e8 \ub4f1 \uc0ac\ud558\ub77c \uc0ac\ub9c9 \uc774\ub0a8\uc758 \uc544\ud504\ub9ac\uce78 \uc774\uc2ac\ub78c \uc138\ub825\uc744 \uaddc\ud569\ud55c \ud63c\ud569\uad70\uc774\uc5c8\ub2e4. \uc774\ub4e4\uc740 \ud0c4\uc790\ub2c8\uc544\uc640 UNLA\uc5d0 \ubc18\ub300\ud558\ub294 \uc778\uc0ac\ub4e4\uacfc \ud568\uaed8 \uce84\ud314\ub77c\ub85c \ud5a5\ud588\ub294\ub370, \ub8e8\uce74\uc57c\uc758 \ub2aa\uc9c0\ub300\uc5d0\uc11c \uc9c4\uad70\uc774 \uc815\uccb4\ub418\uc5c8\ub2e4. \uc774\uc5d0 \ud0c4\uc790\ub2c8\uc544 \uc815\uc608\uad70\uc774\uc5c8\ub358 \uc81c 208\uc5ec\ub2e8\uc740 \ub2aa \uc11c\ucabd \uac00\uc7a5\uc790\ub9ac\uc758 \ub451\uae38\uc744 \ud1b5\ud574 \uc81c 201 \uc5ec\ub2e8\uc744 \uc6b0\uc120 \ubcf4\ub0c8\ub2e4.", "paragraph_answer": "\uc6b0\uac04\ub2e4\uad70\uc740 \ubc14\ub85c \ud1f4\uac01\uc744 \uac1c\uc2dc\ud588\ub294\ub370, \uac19\uc740 \ub3c5\uc7ac\uc790\uc600\ub358 \uc774\ub514 \uc544\ubbfc\uc744 \uc704\ud574 \ub9ac\ube44\uc544\uc758 \uce74\ub2e4\ud53c \ub294 T-54 \uc804\ucc28\uc640 T-55 \uc804\ucc28, BTR \ubcd1\ub825\uc218\uc1a1\ucc28, BM-21 \ub85c\ucf13\ucd94\uc9c4\ucc28, MiG-21, Tu-22 \ud3ed\uaca9\uae30 \ub4f1\uc73c\ub85c \ubb34\uc7a5\ud55c 2,500\uba85\uc758 \uad70\ub300\ub97c \uc6b0\uac04\ub2e4\uce21\uc5d0 \ud30c\ubcd1\ud588\ub2e4. \uadf8\ub7ec\ub098 \ub9ac\ube44\uc544\uad70\uc774 \uc804\uc120\uc5d0 \ub2e4\ub2e4\ub790\uc744 \ub54c \uc6b0\uac04\ub2e4\uad70\uc740 \uc57d\ud0c8\ud55c \ubb3c\uac74\uc744 \ucc44\uc6b4 \ud2b8\ub7ed\uacfc \ud568\uaed8 \ubc18\ub300\uce21\uc73c\ub85c \ud1f4\uac01\ud574\ubc84\ub838\ub2e4. \ub9ac\ube44\uc544\uad70\uc740 \ub9ac\ube44\uc544 \uc721\uad70\ubd80\ub300\uc640 \uc778\ubbfc \ubbfc\ubcd1\ub300, \uc774\uc2ac\ub78c \uad70\ub2e8 \ub4f1 \uc0ac\ud558\ub77c \uc0ac\ub9c9 \uc774\ub0a8\uc758 \uc544\ud504\ub9ac\uce78 \uc774\uc2ac\ub78c \uc138\ub825\uc744 \uaddc\ud569\ud55c \ud63c\ud569\uad70\uc774\uc5c8\ub2e4. \uc774\ub4e4\uc740 \ud0c4\uc790\ub2c8\uc544\uc640 UNLA\uc5d0 \ubc18\ub300\ud558\ub294 \uc778\uc0ac\ub4e4\uacfc \ud568\uaed8 \uce84\ud314\ub77c\ub85c \ud5a5\ud588\ub294\ub370, \ub8e8\uce74\uc57c\uc758 \ub2aa\uc9c0\ub300\uc5d0\uc11c \uc9c4\uad70\uc774 \uc815\uccb4\ub418\uc5c8\ub2e4. \uc774\uc5d0 \ud0c4\uc790\ub2c8\uc544 \uc815\uc608\uad70\uc774\uc5c8\ub358 \uc81c 208\uc5ec\ub2e8\uc740 \ub2aa \uc11c\ucabd \uac00\uc7a5\uc790\ub9ac\uc758 \ub451\uae38\uc744 \ud1b5\ud574 \uc81c 201 \uc5ec\ub2e8\uc744 \uc6b0\uc120 \ubcf4\ub0c8\ub2e4.", "sentence_answer": "\uc6b0\uac04\ub2e4\uad70\uc740 \ubc14\ub85c \ud1f4\uac01\uc744 \uac1c\uc2dc\ud588\ub294\ub370, \uac19\uc740 \ub3c5\uc7ac\uc790\uc600\ub358 \uc774\ub514 \uc544\ubbfc\uc744 \uc704\ud574 \ub9ac\ube44\uc544\uc758 \uce74\ub2e4\ud53c \ub294 T-54 \uc804\ucc28\uc640 T-55 \uc804\ucc28, BTR \ubcd1\ub825\uc218\uc1a1\ucc28, BM-21 \ub85c\ucf13\ucd94\uc9c4\ucc28, MiG-21, Tu-22 \ud3ed\uaca9\uae30 \ub4f1\uc73c\ub85c \ubb34\uc7a5\ud55c 2,500\uba85\uc758 \uad70\ub300\ub97c \uc6b0\uac04\ub2e4\uce21\uc5d0 \ud30c\ubcd1\ud588\ub2e4.", "paragraph_id": "6526192-1-0", "paragraph_question": "question: \uc774\ub514 \uc544\ubbfc\uc744 \uc704\ud574 \uc6b0\uac04\ub2e4\uce21\uc5d0 \ud30c\ubcd1\ud55c \ub9ac\ube44\uc544\uc758 \uc778\ubb3c\uc740?, context: \uc6b0\uac04\ub2e4\uad70\uc740 \ubc14\ub85c \ud1f4\uac01\uc744 \uac1c\uc2dc\ud588\ub294\ub370, \uac19\uc740 \ub3c5\uc7ac\uc790\uc600\ub358 \uc774\ub514 \uc544\ubbfc\uc744 \uc704\ud574 \ub9ac\ube44\uc544\uc758 \uce74\ub2e4\ud53c\ub294 T-54 \uc804\ucc28\uc640 T-55 \uc804\ucc28, BTR \ubcd1\ub825\uc218\uc1a1\ucc28, BM-21 \ub85c\ucf13\ucd94\uc9c4\ucc28, MiG-21, Tu-22 \ud3ed\uaca9\uae30 \ub4f1\uc73c\ub85c \ubb34\uc7a5\ud55c 2,500\uba85\uc758 \uad70\ub300\ub97c \uc6b0\uac04\ub2e4\uce21\uc5d0 \ud30c\ubcd1\ud588\ub2e4. \uadf8\ub7ec\ub098 \ub9ac\ube44\uc544\uad70\uc774 \uc804\uc120\uc5d0 \ub2e4\ub2e4\ub790\uc744 \ub54c \uc6b0\uac04\ub2e4\uad70\uc740 \uc57d\ud0c8\ud55c \ubb3c\uac74\uc744 \ucc44\uc6b4 \ud2b8\ub7ed\uacfc \ud568\uaed8 \ubc18\ub300\uce21\uc73c\ub85c \ud1f4\uac01\ud574\ubc84\ub838\ub2e4. \ub9ac\ube44\uc544\uad70\uc740 \ub9ac\ube44\uc544 \uc721\uad70\ubd80\ub300\uc640 \uc778\ubbfc \ubbfc\ubcd1\ub300, \uc774\uc2ac\ub78c \uad70\ub2e8 \ub4f1 \uc0ac\ud558\ub77c \uc0ac\ub9c9 \uc774\ub0a8\uc758 \uc544\ud504\ub9ac\uce78 \uc774\uc2ac\ub78c \uc138\ub825\uc744 \uaddc\ud569\ud55c \ud63c\ud569\uad70\uc774\uc5c8\ub2e4. \uc774\ub4e4\uc740 \ud0c4\uc790\ub2c8\uc544\uc640 UNLA\uc5d0 \ubc18\ub300\ud558\ub294 \uc778\uc0ac\ub4e4\uacfc \ud568\uaed8 \uce84\ud314\ub77c\ub85c \ud5a5\ud588\ub294\ub370, \ub8e8\uce74\uc57c\uc758 \ub2aa\uc9c0\ub300\uc5d0\uc11c \uc9c4\uad70\uc774 \uc815\uccb4\ub418\uc5c8\ub2e4. \uc774\uc5d0 \ud0c4\uc790\ub2c8\uc544 \uc815\uc608\uad70\uc774\uc5c8\ub358 \uc81c 208\uc5ec\ub2e8\uc740 \ub2aa \uc11c\ucabd \uac00\uc7a5\uc790\ub9ac\uc758 \ub451\uae38\uc744 \ud1b5\ud574 \uc81c 201 \uc5ec\ub2e8\uc744 \uc6b0\uc120 \ubcf4\ub0c8\ub2e4."} {"question": "IOC\uac00 \uaca8\uc6b8\ud310 \uc62c\ub9bc\ud53d\uc744 \uc5f4\uae30\ub85c \ud569\uc758\ud55c \ub54c\ub294?", "paragraph": "\ub3d9\uacc4 \uc62c\ub9bc\ud53d\uc740 \ub208\uacfc \uc5bc\uc74c\uc744 \uc774\uc6a9\ud558\ub294 \uc2a4\ud3ec\uce20\ub4e4\uc744 \ubaa8\uc544 \uc774\ub8e8\uc5b4\uc84c\uc73c\uba70 \ud558\uacc4 \uc62c\ub9bc\ud53d \ub54c \uc2e4\ud589\ud558\uae30 \ubd88\uac00\ub2a5\ud55c \uc885\ubaa9\ub4e4\ub85c \uad6c\uc131\ub418\uc5b4 \uc788\ub2e4. \ud53c\uaca8\uc2a4\ucf00\uc774\ud305, \uc544\uc774\uc2a4\ud558\ud0a4\ub294 \uac01\uac01 1908\ub144\uacfc 1920\ub144\uc5d0 \ud558\uacc4\uc62c\ub9bc\ud53d \uc885\ubaa9\uc73c\ub85c \ub4e4\uc5b4\uac00 \uc788\uc5c8\ub2e4. IOC\ub294 \ub2e4\ub978 \ub3d9\uacc4 \uc2a4\ud3ec\uce20\ub85c \uad6c\uc131\ub41c \uc0c8\ub85c\uc6b4 \ub300\ud68c\ub97c \ub9cc\ub4e4\uace0 \uc2f6\uc5b4 \ud588\uace0, \ub85c\uc794\uc5d0\uc11c \uc5f4\ub9b0 1921\ub144 \uc62c\ub9bc\ud53d \uc758\ud68c\uc5d0\uc11c \uaca8\uc6b8\ud310 \uc62c\ub9bc\ud53d\uc744 \uc5f4\uae30\ub85c \ud569\uc758\ud588\ub2e4. 1\ud68c \ub3d9\uacc4\uc62c\ub9bc\ud53d\uc740 1924\ub144, \ud504\ub791\uc2a4\uc758 \uc0e4\ubaa8\ub2c8\uc5d0\uc11c 11\uc77c\uac04 \uc9c4\ud589\ub418\uc5c8\uace0, 16\uac1c \uc885\ubaa9\uc758 \uacbd\uae30\uac00 \uce58\ub7ec\uc84c\ub2e4. IOC\ub294 \ub3d9\uacc4 \uc62c\ub9bc\ud53d\uc774 4\ub144 \uc8fc\uae30\ub85c \ud558\uacc4 \uc62c\ub9bc\ud53d\uacfc \uac19\uc740 \ub144\ub3c4\uc5d0 \uc5f4\ub9ac\ub3c4\ub85d \ud588\ub2e4. \uc774 \uc804\ud1b5\uc740 \ud504\ub791\uc2a4\uc758 \uc54c\ubca0\ub974\ube4c\uc5d0\uc11c \uc5f4\ub9b0 1992\ub144 \uc62c\ub9bc\ud53d \ub54c \uae4c\uc9c0 \uc9c0\uc18d\ub418\uc5c8\uc73c\ub098, \ub178\ub974\uc6e8\uc774\uc758 \ub9b4\ub808\ud568\uba54\ub974\uc5d0\uc11c \uc5f4\ub9b0 1994\ub144 \uc62c\ub9bc\ud53d\ubd80\ud130 \ub3d9\uacc4 \uc62c\ub9bc\ud53d\uc740 \ud558\uacc4 \uc62c\ub9bc\ud53d\uc774 \ub05d\ub09c\uc9c0 2\ub144\ud6c4\uc5d0 \uac1c\ucd5c\ud558\uc600\ub2e4.", "answer": "1921\ub144", "sentence": "IOC\ub294 \ub2e4\ub978 \ub3d9\uacc4 \uc2a4\ud3ec\uce20\ub85c \uad6c\uc131\ub41c \uc0c8\ub85c\uc6b4 \ub300\ud68c\ub97c \ub9cc\ub4e4\uace0 \uc2f6\uc5b4 \ud588\uace0, \ub85c\uc794\uc5d0\uc11c \uc5f4\ub9b0 1921\ub144 \uc62c\ub9bc\ud53d \uc758\ud68c\uc5d0\uc11c \uaca8\uc6b8\ud310 \uc62c\ub9bc\ud53d\uc744 \uc5f4\uae30\ub85c \ud569\uc758\ud588\ub2e4.", "paragraph_sentence": "\ub3d9\uacc4 \uc62c\ub9bc\ud53d\uc740 \ub208\uacfc \uc5bc\uc74c\uc744 \uc774\uc6a9\ud558\ub294 \uc2a4\ud3ec\uce20\ub4e4\uc744 \ubaa8\uc544 \uc774\ub8e8\uc5b4\uc84c\uc73c\uba70 \ud558\uacc4 \uc62c\ub9bc\ud53d \ub54c \uc2e4\ud589\ud558\uae30 \ubd88\uac00\ub2a5\ud55c \uc885\ubaa9\ub4e4\ub85c \uad6c\uc131\ub418\uc5b4 \uc788\ub2e4. \ud53c\uaca8\uc2a4\ucf00\uc774\ud305, \uc544\uc774\uc2a4\ud558\ud0a4\ub294 \uac01\uac01 1908\ub144\uacfc 1920\ub144\uc5d0 \ud558\uacc4\uc62c\ub9bc\ud53d \uc885\ubaa9\uc73c\ub85c \ub4e4\uc5b4\uac00 \uc788\uc5c8\ub2e4. IOC\ub294 \ub2e4\ub978 \ub3d9\uacc4 \uc2a4\ud3ec\uce20\ub85c \uad6c\uc131\ub41c \uc0c8\ub85c\uc6b4 \ub300\ud68c\ub97c \ub9cc\ub4e4\uace0 \uc2f6\uc5b4 \ud588\uace0, \ub85c\uc794\uc5d0\uc11c \uc5f4\ub9b0 1921\ub144 \uc62c\ub9bc\ud53d \uc758\ud68c\uc5d0\uc11c \uaca8\uc6b8\ud310 \uc62c\ub9bc\ud53d\uc744 \uc5f4\uae30\ub85c \ud569\uc758\ud588\ub2e4. 1\ud68c \ub3d9\uacc4\uc62c\ub9bc\ud53d\uc740 1924\ub144, \ud504\ub791\uc2a4\uc758 \uc0e4\ubaa8\ub2c8\uc5d0\uc11c 11\uc77c\uac04 \uc9c4\ud589\ub418\uc5c8\uace0, 16\uac1c \uc885\ubaa9\uc758 \uacbd\uae30\uac00 \uce58\ub7ec\uc84c\ub2e4. IOC\ub294 \ub3d9\uacc4 \uc62c\ub9bc\ud53d\uc774 4\ub144 \uc8fc\uae30\ub85c \ud558\uacc4 \uc62c\ub9bc\ud53d\uacfc \uac19\uc740 \ub144\ub3c4\uc5d0 \uc5f4\ub9ac\ub3c4\ub85d \ud588\ub2e4. \uc774 \uc804\ud1b5\uc740 \ud504\ub791\uc2a4\uc758 \uc54c\ubca0\ub974\ube4c\uc5d0\uc11c \uc5f4\ub9b0 1992\ub144 \uc62c\ub9bc\ud53d \ub54c \uae4c\uc9c0 \uc9c0\uc18d\ub418\uc5c8\uc73c\ub098, \ub178\ub974\uc6e8\uc774\uc758 \ub9b4\ub808\ud568\uba54\ub974\uc5d0\uc11c \uc5f4\ub9b0 1994\ub144 \uc62c\ub9bc\ud53d\ubd80\ud130 \ub3d9\uacc4 \uc62c\ub9bc\ud53d\uc740 \ud558\uacc4 \uc62c\ub9bc\ud53d\uc774 \ub05d\ub09c\uc9c0 2\ub144\ud6c4\uc5d0 \uac1c\ucd5c\ud558\uc600\ub2e4.", "paragraph_answer": "\ub3d9\uacc4 \uc62c\ub9bc\ud53d\uc740 \ub208\uacfc \uc5bc\uc74c\uc744 \uc774\uc6a9\ud558\ub294 \uc2a4\ud3ec\uce20\ub4e4\uc744 \ubaa8\uc544 \uc774\ub8e8\uc5b4\uc84c\uc73c\uba70 \ud558\uacc4 \uc62c\ub9bc\ud53d \ub54c \uc2e4\ud589\ud558\uae30 \ubd88\uac00\ub2a5\ud55c \uc885\ubaa9\ub4e4\ub85c \uad6c\uc131\ub418\uc5b4 \uc788\ub2e4. \ud53c\uaca8\uc2a4\ucf00\uc774\ud305, \uc544\uc774\uc2a4\ud558\ud0a4\ub294 \uac01\uac01 1908\ub144\uacfc 1920\ub144\uc5d0 \ud558\uacc4\uc62c\ub9bc\ud53d \uc885\ubaa9\uc73c\ub85c \ub4e4\uc5b4\uac00 \uc788\uc5c8\ub2e4. IOC\ub294 \ub2e4\ub978 \ub3d9\uacc4 \uc2a4\ud3ec\uce20\ub85c \uad6c\uc131\ub41c \uc0c8\ub85c\uc6b4 \ub300\ud68c\ub97c \ub9cc\ub4e4\uace0 \uc2f6\uc5b4 \ud588\uace0, \ub85c\uc794\uc5d0\uc11c \uc5f4\ub9b0 1921\ub144 \uc62c\ub9bc\ud53d \uc758\ud68c\uc5d0\uc11c \uaca8\uc6b8\ud310 \uc62c\ub9bc\ud53d\uc744 \uc5f4\uae30\ub85c \ud569\uc758\ud588\ub2e4. 1\ud68c \ub3d9\uacc4\uc62c\ub9bc\ud53d\uc740 1924\ub144, \ud504\ub791\uc2a4\uc758 \uc0e4\ubaa8\ub2c8\uc5d0\uc11c 11\uc77c\uac04 \uc9c4\ud589\ub418\uc5c8\uace0, 16\uac1c \uc885\ubaa9\uc758 \uacbd\uae30\uac00 \uce58\ub7ec\uc84c\ub2e4. IOC\ub294 \ub3d9\uacc4 \uc62c\ub9bc\ud53d\uc774 4\ub144 \uc8fc\uae30\ub85c \ud558\uacc4 \uc62c\ub9bc\ud53d\uacfc \uac19\uc740 \ub144\ub3c4\uc5d0 \uc5f4\ub9ac\ub3c4\ub85d \ud588\ub2e4. \uc774 \uc804\ud1b5\uc740 \ud504\ub791\uc2a4\uc758 \uc54c\ubca0\ub974\ube4c\uc5d0\uc11c \uc5f4\ub9b0 1992\ub144 \uc62c\ub9bc\ud53d \ub54c \uae4c\uc9c0 \uc9c0\uc18d\ub418\uc5c8\uc73c\ub098, \ub178\ub974\uc6e8\uc774\uc758 \ub9b4\ub808\ud568\uba54\ub974\uc5d0\uc11c \uc5f4\ub9b0 1994\ub144 \uc62c\ub9bc\ud53d\ubd80\ud130 \ub3d9\uacc4 \uc62c\ub9bc\ud53d\uc740 \ud558\uacc4 \uc62c\ub9bc\ud53d\uc774 \ub05d\ub09c\uc9c0 2\ub144\ud6c4\uc5d0 \uac1c\ucd5c\ud558\uc600\ub2e4.", "sentence_answer": "IOC\ub294 \ub2e4\ub978 \ub3d9\uacc4 \uc2a4\ud3ec\uce20\ub85c \uad6c\uc131\ub41c \uc0c8\ub85c\uc6b4 \ub300\ud68c\ub97c \ub9cc\ub4e4\uace0 \uc2f6\uc5b4 \ud588\uace0, \ub85c\uc794\uc5d0\uc11c \uc5f4\ub9b0 1921\ub144 \uc62c\ub9bc\ud53d \uc758\ud68c\uc5d0\uc11c \uaca8\uc6b8\ud310 \uc62c\ub9bc\ud53d\uc744 \uc5f4\uae30\ub85c \ud569\uc758\ud588\ub2e4.", "paragraph_id": "6588341-8-2", "paragraph_question": "question: IOC\uac00 \uaca8\uc6b8\ud310 \uc62c\ub9bc\ud53d\uc744 \uc5f4\uae30\ub85c \ud569\uc758\ud55c \ub54c\ub294?, context: \ub3d9\uacc4 \uc62c\ub9bc\ud53d\uc740 \ub208\uacfc \uc5bc\uc74c\uc744 \uc774\uc6a9\ud558\ub294 \uc2a4\ud3ec\uce20\ub4e4\uc744 \ubaa8\uc544 \uc774\ub8e8\uc5b4\uc84c\uc73c\uba70 \ud558\uacc4 \uc62c\ub9bc\ud53d \ub54c \uc2e4\ud589\ud558\uae30 \ubd88\uac00\ub2a5\ud55c \uc885\ubaa9\ub4e4\ub85c \uad6c\uc131\ub418\uc5b4 \uc788\ub2e4. \ud53c\uaca8\uc2a4\ucf00\uc774\ud305, \uc544\uc774\uc2a4\ud558\ud0a4\ub294 \uac01\uac01 1908\ub144\uacfc 1920\ub144\uc5d0 \ud558\uacc4\uc62c\ub9bc\ud53d \uc885\ubaa9\uc73c\ub85c \ub4e4\uc5b4\uac00 \uc788\uc5c8\ub2e4. IOC\ub294 \ub2e4\ub978 \ub3d9\uacc4 \uc2a4\ud3ec\uce20\ub85c \uad6c\uc131\ub41c \uc0c8\ub85c\uc6b4 \ub300\ud68c\ub97c \ub9cc\ub4e4\uace0 \uc2f6\uc5b4 \ud588\uace0, \ub85c\uc794\uc5d0\uc11c \uc5f4\ub9b0 1921\ub144 \uc62c\ub9bc\ud53d \uc758\ud68c\uc5d0\uc11c \uaca8\uc6b8\ud310 \uc62c\ub9bc\ud53d\uc744 \uc5f4\uae30\ub85c \ud569\uc758\ud588\ub2e4. 1\ud68c \ub3d9\uacc4\uc62c\ub9bc\ud53d\uc740 1924\ub144, \ud504\ub791\uc2a4\uc758 \uc0e4\ubaa8\ub2c8\uc5d0\uc11c 11\uc77c\uac04 \uc9c4\ud589\ub418\uc5c8\uace0, 16\uac1c \uc885\ubaa9\uc758 \uacbd\uae30\uac00 \uce58\ub7ec\uc84c\ub2e4. IOC\ub294 \ub3d9\uacc4 \uc62c\ub9bc\ud53d\uc774 4\ub144 \uc8fc\uae30\ub85c \ud558\uacc4 \uc62c\ub9bc\ud53d\uacfc \uac19\uc740 \ub144\ub3c4\uc5d0 \uc5f4\ub9ac\ub3c4\ub85d \ud588\ub2e4. \uc774 \uc804\ud1b5\uc740 \ud504\ub791\uc2a4\uc758 \uc54c\ubca0\ub974\ube4c\uc5d0\uc11c \uc5f4\ub9b0 1992\ub144 \uc62c\ub9bc\ud53d \ub54c \uae4c\uc9c0 \uc9c0\uc18d\ub418\uc5c8\uc73c\ub098, \ub178\ub974\uc6e8\uc774\uc758 \ub9b4\ub808\ud568\uba54\ub974\uc5d0\uc11c \uc5f4\ub9b0 1994\ub144 \uc62c\ub9bc\ud53d\ubd80\ud130 \ub3d9\uacc4 \uc62c\ub9bc\ud53d\uc740 \ud558\uacc4 \uc62c\ub9bc\ud53d\uc774 \ub05d\ub09c\uc9c0 2\ub144\ud6c4\uc5d0 \uac1c\ucd5c\ud558\uc600\ub2e4."} {"question": "18\ub300 \ucd1d\uc120 \ud6c4\ubcf4 \uc120\ucd9c \uacfc\uc815\uc5d0\uc11c \ud55c\ub098\ub77c\ub2f9\uc744 \uc9d1\ub2e8 \ud0c8\ub2f9\ud558\uace0 \ubbf8\ub798\ud55c\uad6d\ub2f9\uc5d0 \uc785\ub2f9\ud55c \uce5c\ubc15\uadfc\ud61c \uacc4\uc5f4 \ub2f9\uc6d0\ub4e4\uc774 \ubc14\uafbc \ub2f9\uba85\uc740?", "paragraph": "\uc6d0\ub798 2007\ub144 \ub300\uc120\uc5d0\uc11c \uc815\uadfc\ubaa8 \ud6c4\ubcf4\ub97c \ub0b4\uc138\uc6e0\ub358 '\ucc38\uc8fc\uc778\uc5f0\ud569'\uc5d0\uc11c \ucd9c\ubc1c\ud588\ub2e4. \ub300\uc120 \uc774\ud6c4 '\ubbf8\ub798\ud55c\uad6d\ub2f9'\uc73c\ub85c \ub2f9\uba85\uc744 \ubc14\uafe8\uc73c\ub098 \uc0ac\uc2e4\uc0c1 \ud750\uc9c0\ubd80\uc9c0\ub41c \uc0c1\ud669\uc5d0\uc11c, \ud55c\ub098\ub77c\ub2f9\uc758 18\ub300 \ucd1d\uc120 \ud6c4\ubcf4 \uc120\ucd9c\uacfc\uc815\uc5d0\uc11c \uce5c \uc774\uba85\ubc15 \uacc4\uc5f4\uc774 \ub9ce\uc774 \uacf5\ucc9c\ub41c \ub370 \ub300\ud55c \ubc18\ubc1c\ub85c, \uce5c \ubc15\uadfc\ud61c \uacc4\uc5f4 \ub2f9\uc6d0\ub4e4\uc774 \ud55c\ub098\ub77c\ub2f9\uc744 \uc9d1\ub2e8 \ud0c8\ub2f9\ud558\uace0 \ubbf8\ub798\ud55c\uad6d\ub2f9\uc5d0 \uc785\ub2f9\ud558\uba74\uc11c \ubc15\uadfc\ud61c\ub97c \uc804\uba74\uc5d0 \ub0b4\uc138\uc6b0\uae30 \uc704\ud574 \ub2f9\uba85\uc744 '\uce5c\ubc15\uc5f0\ub300'\ub85c \ubc14\uafe8\ub2e4. \uc815\uc791, \ubc15\uadfc\ud61c\ub294 \ud55c\ub098\ub77c\ub2f9\uc5d0 \ub0a8\uc544\uc788\uc5c8\ub2e4. \uc774\ub4e4\uc774 \ud55c\ub098\ub77c\ub2f9\uc73c\ub85c \ubcf5\ub2f9\ud558\ub824\ud558\uc790 \uce5c\uc774 \uacc4\uc5f4\uc740 \ubc18\ub300\ud558\uc600\uc73c\ub098 \uacb0\uad6d \ubcf5\ub2f9\uc744 \ubc1b\uc544\ub4e4\uc600\ub2e4. \ub2f9\uba85 \ubcc0\uacbd\uc5d0 \ub300\ud55c \uc694\uad6c\uac00 \ub04a\uc784\uc5c6\uc774 \uc81c\uae30\ub418\uc5b4 \uacf5\ubaa8\ub97c \ud1b5\ud574 2010\ub144 2\uc6d4 12\uc77c \ub2f9\uba85\uc744 \ubbf8\ub798\ud76c\ub9dd\uc5f0\ub300\ub85c \ubcc0\uacbd\ud588\ub2e4. 2010\ub144 7\uc6d4\uc5d0 \uc5f4\ub9b0 \ud55c\ub098\ub77c\ub2f9 \uc804\ub2f9\ub300\ud68c\uc5d0\uc11c \ud55c\ub098\ub77c\ub2f9\uacfc\uc758 \ud569\ub2f9\uc774 \uc758\uacb0\ub418\uc5b4 \ud569\ub2f9 \uc18c\uba78 \ub418\uc5c8\ub2e4. \uc774 \uacfc\uc815\uc5d0\uc11c \uc774\uaddc\ud0dd\uacfc \ube44\ub2f9\uad8c\ud30c\ub294 \ubbf8\ub798\uc5f0\ud569\uc744 \ucc3d\ub2f9\ud558\uc600\ub2e4.", "answer": "\uce5c\ubc15\uc5f0\ub300", "sentence": "\ub300\uc120 \uc774\ud6c4 '\ubbf8\ub798\ud55c\uad6d\ub2f9'\uc73c\ub85c \ub2f9\uba85\uc744 \ubc14\uafe8\uc73c\ub098 \uc0ac\uc2e4\uc0c1 \ud750\uc9c0\ubd80\uc9c0\ub41c \uc0c1\ud669\uc5d0\uc11c, \ud55c\ub098\ub77c\ub2f9\uc758 18\ub300 \ucd1d\uc120 \ud6c4\ubcf4 \uc120\ucd9c\uacfc\uc815\uc5d0\uc11c \uce5c \uc774\uba85\ubc15 \uacc4\uc5f4\uc774 \ub9ce\uc774 \uacf5\ucc9c\ub41c \ub370 \ub300\ud55c \ubc18\ubc1c\ub85c, \uce5c \ubc15\uadfc\ud61c \uacc4\uc5f4 \ub2f9\uc6d0\ub4e4\uc774 \ud55c\ub098\ub77c\ub2f9\uc744 \uc9d1\ub2e8 \ud0c8\ub2f9\ud558\uace0 \ubbf8\ub798\ud55c\uad6d\ub2f9\uc5d0 \uc785\ub2f9\ud558\uba74\uc11c \ubc15\uadfc\ud61c\ub97c \uc804\uba74\uc5d0 \ub0b4\uc138\uc6b0\uae30 \uc704\ud574 \ub2f9\uba85\uc744 ' \uce5c\ubc15\uc5f0\ub300 '\ub85c \ubc14\uafe8\ub2e4.", "paragraph_sentence": "\uc6d0\ub798 2007\ub144 \ub300\uc120\uc5d0\uc11c \uc815\uadfc\ubaa8 \ud6c4\ubcf4\ub97c \ub0b4\uc138\uc6e0\ub358 '\ucc38\uc8fc\uc778\uc5f0\ud569'\uc5d0\uc11c \ucd9c\ubc1c\ud588\ub2e4. \ub300\uc120 \uc774\ud6c4 '\ubbf8\ub798\ud55c\uad6d\ub2f9'\uc73c\ub85c \ub2f9\uba85\uc744 \ubc14\uafe8\uc73c\ub098 \uc0ac\uc2e4\uc0c1 \ud750\uc9c0\ubd80\uc9c0\ub41c \uc0c1\ud669\uc5d0\uc11c, \ud55c\ub098\ub77c\ub2f9\uc758 18\ub300 \ucd1d\uc120 \ud6c4\ubcf4 \uc120\ucd9c\uacfc\uc815\uc5d0\uc11c \uce5c \uc774\uba85\ubc15 \uacc4\uc5f4\uc774 \ub9ce\uc774 \uacf5\ucc9c\ub41c \ub370 \ub300\ud55c \ubc18\ubc1c\ub85c, \uce5c \ubc15\uadfc\ud61c \uacc4\uc5f4 \ub2f9\uc6d0\ub4e4\uc774 \ud55c\ub098\ub77c\ub2f9\uc744 \uc9d1\ub2e8 \ud0c8\ub2f9\ud558\uace0 \ubbf8\ub798\ud55c\uad6d\ub2f9\uc5d0 \uc785\ub2f9\ud558\uba74\uc11c \ubc15\uadfc\ud61c\ub97c \uc804\uba74\uc5d0 \ub0b4\uc138\uc6b0\uae30 \uc704\ud574 \ub2f9\uba85\uc744 ' \uce5c\ubc15\uc5f0\ub300 '\ub85c \ubc14\uafe8\ub2e4. \uc815\uc791, \ubc15\uadfc\ud61c\ub294 \ud55c\ub098\ub77c\ub2f9\uc5d0 \ub0a8\uc544\uc788\uc5c8\ub2e4. \uc774\ub4e4\uc774 \ud55c\ub098\ub77c\ub2f9\uc73c\ub85c \ubcf5\ub2f9\ud558\ub824\ud558\uc790 \uce5c\uc774 \uacc4\uc5f4\uc740 \ubc18\ub300\ud558\uc600\uc73c\ub098 \uacb0\uad6d \ubcf5\ub2f9\uc744 \ubc1b\uc544\ub4e4\uc600\ub2e4. \ub2f9\uba85 \ubcc0\uacbd\uc5d0 \ub300\ud55c \uc694\uad6c\uac00 \ub04a\uc784\uc5c6\uc774 \uc81c\uae30\ub418\uc5b4 \uacf5\ubaa8\ub97c \ud1b5\ud574 2010\ub144 2\uc6d4 12\uc77c \ub2f9\uba85\uc744 \ubbf8\ub798\ud76c\ub9dd\uc5f0\ub300\ub85c \ubcc0\uacbd\ud588\ub2e4. 2010\ub144 7\uc6d4\uc5d0 \uc5f4\ub9b0 \ud55c\ub098\ub77c\ub2f9 \uc804\ub2f9\ub300\ud68c\uc5d0\uc11c \ud55c\ub098\ub77c\ub2f9\uacfc\uc758 \ud569\ub2f9\uc774 \uc758\uacb0\ub418\uc5b4 \ud569\ub2f9 \uc18c\uba78 \ub418\uc5c8\ub2e4. \uc774 \uacfc\uc815\uc5d0\uc11c \uc774\uaddc\ud0dd\uacfc \ube44\ub2f9\uad8c\ud30c\ub294 \ubbf8\ub798\uc5f0\ud569\uc744 \ucc3d\ub2f9\ud558\uc600\ub2e4.", "paragraph_answer": "\uc6d0\ub798 2007\ub144 \ub300\uc120\uc5d0\uc11c \uc815\uadfc\ubaa8 \ud6c4\ubcf4\ub97c \ub0b4\uc138\uc6e0\ub358 '\ucc38\uc8fc\uc778\uc5f0\ud569'\uc5d0\uc11c \ucd9c\ubc1c\ud588\ub2e4. \ub300\uc120 \uc774\ud6c4 '\ubbf8\ub798\ud55c\uad6d\ub2f9'\uc73c\ub85c \ub2f9\uba85\uc744 \ubc14\uafe8\uc73c\ub098 \uc0ac\uc2e4\uc0c1 \ud750\uc9c0\ubd80\uc9c0\ub41c \uc0c1\ud669\uc5d0\uc11c, \ud55c\ub098\ub77c\ub2f9\uc758 18\ub300 \ucd1d\uc120 \ud6c4\ubcf4 \uc120\ucd9c\uacfc\uc815\uc5d0\uc11c \uce5c \uc774\uba85\ubc15 \uacc4\uc5f4\uc774 \ub9ce\uc774 \uacf5\ucc9c\ub41c \ub370 \ub300\ud55c \ubc18\ubc1c\ub85c, \uce5c \ubc15\uadfc\ud61c \uacc4\uc5f4 \ub2f9\uc6d0\ub4e4\uc774 \ud55c\ub098\ub77c\ub2f9\uc744 \uc9d1\ub2e8 \ud0c8\ub2f9\ud558\uace0 \ubbf8\ub798\ud55c\uad6d\ub2f9\uc5d0 \uc785\ub2f9\ud558\uba74\uc11c \ubc15\uadfc\ud61c\ub97c \uc804\uba74\uc5d0 \ub0b4\uc138\uc6b0\uae30 \uc704\ud574 \ub2f9\uba85\uc744 ' \uce5c\ubc15\uc5f0\ub300 '\ub85c \ubc14\uafe8\ub2e4. \uc815\uc791, \ubc15\uadfc\ud61c\ub294 \ud55c\ub098\ub77c\ub2f9\uc5d0 \ub0a8\uc544\uc788\uc5c8\ub2e4. \uc774\ub4e4\uc774 \ud55c\ub098\ub77c\ub2f9\uc73c\ub85c \ubcf5\ub2f9\ud558\ub824\ud558\uc790 \uce5c\uc774 \uacc4\uc5f4\uc740 \ubc18\ub300\ud558\uc600\uc73c\ub098 \uacb0\uad6d \ubcf5\ub2f9\uc744 \ubc1b\uc544\ub4e4\uc600\ub2e4. \ub2f9\uba85 \ubcc0\uacbd\uc5d0 \ub300\ud55c \uc694\uad6c\uac00 \ub04a\uc784\uc5c6\uc774 \uc81c\uae30\ub418\uc5b4 \uacf5\ubaa8\ub97c \ud1b5\ud574 2010\ub144 2\uc6d4 12\uc77c \ub2f9\uba85\uc744 \ubbf8\ub798\ud76c\ub9dd\uc5f0\ub300\ub85c \ubcc0\uacbd\ud588\ub2e4. 2010\ub144 7\uc6d4\uc5d0 \uc5f4\ub9b0 \ud55c\ub098\ub77c\ub2f9 \uc804\ub2f9\ub300\ud68c\uc5d0\uc11c \ud55c\ub098\ub77c\ub2f9\uacfc\uc758 \ud569\ub2f9\uc774 \uc758\uacb0\ub418\uc5b4 \ud569\ub2f9 \uc18c\uba78 \ub418\uc5c8\ub2e4. \uc774 \uacfc\uc815\uc5d0\uc11c \uc774\uaddc\ud0dd\uacfc \ube44\ub2f9\uad8c\ud30c\ub294 \ubbf8\ub798\uc5f0\ud569\uc744 \ucc3d\ub2f9\ud558\uc600\ub2e4.", "sentence_answer": "\ub300\uc120 \uc774\ud6c4 '\ubbf8\ub798\ud55c\uad6d\ub2f9'\uc73c\ub85c \ub2f9\uba85\uc744 \ubc14\uafe8\uc73c\ub098 \uc0ac\uc2e4\uc0c1 \ud750\uc9c0\ubd80\uc9c0\ub41c \uc0c1\ud669\uc5d0\uc11c, \ud55c\ub098\ub77c\ub2f9\uc758 18\ub300 \ucd1d\uc120 \ud6c4\ubcf4 \uc120\ucd9c\uacfc\uc815\uc5d0\uc11c \uce5c \uc774\uba85\ubc15 \uacc4\uc5f4\uc774 \ub9ce\uc774 \uacf5\ucc9c\ub41c \ub370 \ub300\ud55c \ubc18\ubc1c\ub85c, \uce5c \ubc15\uadfc\ud61c \uacc4\uc5f4 \ub2f9\uc6d0\ub4e4\uc774 \ud55c\ub098\ub77c\ub2f9\uc744 \uc9d1\ub2e8 \ud0c8\ub2f9\ud558\uace0 \ubbf8\ub798\ud55c\uad6d\ub2f9\uc5d0 \uc785\ub2f9\ud558\uba74\uc11c \ubc15\uadfc\ud61c\ub97c \uc804\uba74\uc5d0 \ub0b4\uc138\uc6b0\uae30 \uc704\ud574 \ub2f9\uba85\uc744 ' \uce5c\ubc15\uc5f0\ub300 '\ub85c \ubc14\uafe8\ub2e4.", "paragraph_id": "6486249-1-0", "paragraph_question": "question: 18\ub300 \ucd1d\uc120 \ud6c4\ubcf4 \uc120\ucd9c \uacfc\uc815\uc5d0\uc11c \ud55c\ub098\ub77c\ub2f9\uc744 \uc9d1\ub2e8 \ud0c8\ub2f9\ud558\uace0 \ubbf8\ub798\ud55c\uad6d\ub2f9\uc5d0 \uc785\ub2f9\ud55c \uce5c\ubc15\uadfc\ud61c \uacc4\uc5f4 \ub2f9\uc6d0\ub4e4\uc774 \ubc14\uafbc \ub2f9\uba85\uc740?, context: \uc6d0\ub798 2007\ub144 \ub300\uc120\uc5d0\uc11c \uc815\uadfc\ubaa8 \ud6c4\ubcf4\ub97c \ub0b4\uc138\uc6e0\ub358 '\ucc38\uc8fc\uc778\uc5f0\ud569'\uc5d0\uc11c \ucd9c\ubc1c\ud588\ub2e4. \ub300\uc120 \uc774\ud6c4 '\ubbf8\ub798\ud55c\uad6d\ub2f9'\uc73c\ub85c \ub2f9\uba85\uc744 \ubc14\uafe8\uc73c\ub098 \uc0ac\uc2e4\uc0c1 \ud750\uc9c0\ubd80\uc9c0\ub41c \uc0c1\ud669\uc5d0\uc11c, \ud55c\ub098\ub77c\ub2f9\uc758 18\ub300 \ucd1d\uc120 \ud6c4\ubcf4 \uc120\ucd9c\uacfc\uc815\uc5d0\uc11c \uce5c \uc774\uba85\ubc15 \uacc4\uc5f4\uc774 \ub9ce\uc774 \uacf5\ucc9c\ub41c \ub370 \ub300\ud55c \ubc18\ubc1c\ub85c, \uce5c \ubc15\uadfc\ud61c \uacc4\uc5f4 \ub2f9\uc6d0\ub4e4\uc774 \ud55c\ub098\ub77c\ub2f9\uc744 \uc9d1\ub2e8 \ud0c8\ub2f9\ud558\uace0 \ubbf8\ub798\ud55c\uad6d\ub2f9\uc5d0 \uc785\ub2f9\ud558\uba74\uc11c \ubc15\uadfc\ud61c\ub97c \uc804\uba74\uc5d0 \ub0b4\uc138\uc6b0\uae30 \uc704\ud574 \ub2f9\uba85\uc744 '\uce5c\ubc15\uc5f0\ub300'\ub85c \ubc14\uafe8\ub2e4. \uc815\uc791, \ubc15\uadfc\ud61c\ub294 \ud55c\ub098\ub77c\ub2f9\uc5d0 \ub0a8\uc544\uc788\uc5c8\ub2e4. \uc774\ub4e4\uc774 \ud55c\ub098\ub77c\ub2f9\uc73c\ub85c \ubcf5\ub2f9\ud558\ub824\ud558\uc790 \uce5c\uc774 \uacc4\uc5f4\uc740 \ubc18\ub300\ud558\uc600\uc73c\ub098 \uacb0\uad6d \ubcf5\ub2f9\uc744 \ubc1b\uc544\ub4e4\uc600\ub2e4. \ub2f9\uba85 \ubcc0\uacbd\uc5d0 \ub300\ud55c \uc694\uad6c\uac00 \ub04a\uc784\uc5c6\uc774 \uc81c\uae30\ub418\uc5b4 \uacf5\ubaa8\ub97c \ud1b5\ud574 2010\ub144 2\uc6d4 12\uc77c \ub2f9\uba85\uc744 \ubbf8\ub798\ud76c\ub9dd\uc5f0\ub300\ub85c \ubcc0\uacbd\ud588\ub2e4. 2010\ub144 7\uc6d4\uc5d0 \uc5f4\ub9b0 \ud55c\ub098\ub77c\ub2f9 \uc804\ub2f9\ub300\ud68c\uc5d0\uc11c \ud55c\ub098\ub77c\ub2f9\uacfc\uc758 \ud569\ub2f9\uc774 \uc758\uacb0\ub418\uc5b4 \ud569\ub2f9 \uc18c\uba78 \ub418\uc5c8\ub2e4. \uc774 \uacfc\uc815\uc5d0\uc11c \uc774\uaddc\ud0dd\uacfc \ube44\ub2f9\uad8c\ud30c\ub294 \ubbf8\ub798\uc5f0\ud569\uc744 \ucc3d\ub2f9\ud558\uc600\ub2e4."} {"question": "\uc790\uc720\ubbfc\uc8fc\ub2f9\uc774 \uc5ec\ub2f9\uc758 \uc790\ub9ac\ub97c \ud0c8\ud658\ud55c \ub0a0\uc740 \uc5b8\uc81c\uc778\uac00?", "paragraph": "\uc785\ubc95 \uae30\uad00\uc778 \uad6d\ud68c\ub294 \ud5cc\ubc95\uc5d0\uc11c \uba85\uc2dc\ub41c \uad6d\uad8c\uc758 \ucd5c\uace0 \uae30\uad00\uc774\uc790 \ub098\ub77c\uc758 \uc720\uc77c\ud55c \uc785\ubc95 \uae30\uad00\uc73c\ub85c\uc368 \uc601\uad6d\uc2dd \uc758\uc6d0 \ub0b4\uac01\uc81c\ub97c \ucc44\uc6a9\ud588\uae30 \ub54c\ubb38\uc5d0 \uc218\uc0c1\uc758 \uc9c0\uba85\uad8c, \uc911\uc758\uc6d0\uc758 \ub0b4\uac01 \uc2e0\uc784\uacfc \ubd88\uc2e0\uc784\uc758 \uc758\uacb0\uad8c\uc744 \uac00\uc9c4\ub2e4. \uad6d\ud68c\ub294 \uc591\uc6d0\uc81c\ub85c \uc911\uc758\uc6d0 \ubc0f \ucc38\uc758\uc6d0\uc73c\ub85c \uad6c\uc131\ub418\uc5b4 \uc788\ub294\ub370 \uc591\uc6d0\uc740 \uc804 \uad6d\ubbfc\uc758 \ub300\ud45c\ub85c \uc120\ucd9c\ub41c \uc758\uc6d0\uc73c\ub85c \uc870\uc9c1\ub418\uc5b4 \uc788\ub2e4. \uad6d\ud68c\ub294 1955\ub144 \uc774\ud6c4 1993\ub144\ubd80\ud130 1996\ub144\uae4c\uc9c0\uc758 \uc5f0\uc815 \uae30\uac04\uc744 \uc81c\uc678\ud558\uace0 \uc790\uc720\ubbfc\uc8fc\ub2f9\uc774 \uacc4\uc18d\ud574\uc11c \uc5ec\ub2f9\uc758 \uc790\ub9ac\uc5d0 \uc788\uc5c8\uc73c\ub098 2009\ub144 9\uc6d4 15\uc77c \ubbfc\uc8fc\ub2f9\uc758 \ud558\ud1a0\uc57c\ub9c8 \uc720\ud0a4\uc624 \ub0b4\uac01\uc774 \uc2dc\uc791\ub418\uba74\uc11c \ucd5c\ucd08\uc758 \uc815\uad8c \uad50\uccb4\uac00 \uc774\ub8e8\uc5b4\uc84c\ub2e4. 2012\ub144 12\uc6d4 16\uc77c\uc5d0 \uc2e4\uc2dc\ub41c \uc911\uc758\uc6d0 \uc758\uc6d0 \ucd1d\uc120\uac70\uc5d0\uc11c \uc790\uc720\ubbfc\uc8fc\ub2f9\uc774 \ub2e4\uc218\uc758 \uc758\uc11d\uc744 \ucc28\uc9c0\ud558\uc5ec \uc5ec\ub2f9\uc758 \uc790\ub9ac\ub97c \ud0c8\ud658\ud558\uc600\ub2e4.", "answer": "2012\ub144 12\uc6d4 16\uc77c", "sentence": "2012\ub144 12\uc6d4 16\uc77c \uc5d0 \uc2e4\uc2dc\ub41c \uc911\uc758\uc6d0 \uc758\uc6d0 \ucd1d\uc120\uac70\uc5d0\uc11c \uc790\uc720\ubbfc\uc8fc\ub2f9\uc774 \ub2e4\uc218\uc758 \uc758\uc11d\uc744 \ucc28\uc9c0\ud558\uc5ec \uc5ec\ub2f9\uc758 \uc790\ub9ac\ub97c \ud0c8\ud658\ud558\uc600\ub2e4.", "paragraph_sentence": "\uc785\ubc95 \uae30\uad00\uc778 \uad6d\ud68c\ub294 \ud5cc\ubc95\uc5d0\uc11c \uba85\uc2dc\ub41c \uad6d\uad8c\uc758 \ucd5c\uace0 \uae30\uad00\uc774\uc790 \ub098\ub77c\uc758 \uc720\uc77c\ud55c \uc785\ubc95 \uae30\uad00\uc73c\ub85c\uc368 \uc601\uad6d\uc2dd \uc758\uc6d0 \ub0b4\uac01\uc81c\ub97c \ucc44\uc6a9\ud588\uae30 \ub54c\ubb38\uc5d0 \uc218\uc0c1\uc758 \uc9c0\uba85\uad8c, \uc911\uc758\uc6d0\uc758 \ub0b4\uac01 \uc2e0\uc784\uacfc \ubd88\uc2e0\uc784\uc758 \uc758\uacb0\uad8c\uc744 \uac00\uc9c4\ub2e4. \uad6d\ud68c\ub294 \uc591\uc6d0\uc81c\ub85c \uc911\uc758\uc6d0 \ubc0f \ucc38\uc758\uc6d0\uc73c\ub85c \uad6c\uc131\ub418\uc5b4 \uc788\ub294\ub370 \uc591\uc6d0\uc740 \uc804 \uad6d\ubbfc\uc758 \ub300\ud45c\ub85c \uc120\ucd9c\ub41c \uc758\uc6d0\uc73c\ub85c \uc870\uc9c1\ub418\uc5b4 \uc788\ub2e4. \uad6d\ud68c\ub294 1955\ub144 \uc774\ud6c4 1993\ub144\ubd80\ud130 1996\ub144\uae4c\uc9c0\uc758 \uc5f0\uc815 \uae30\uac04\uc744 \uc81c\uc678\ud558\uace0 \uc790\uc720\ubbfc\uc8fc\ub2f9\uc774 \uacc4\uc18d\ud574\uc11c \uc5ec\ub2f9\uc758 \uc790\ub9ac\uc5d0 \uc788\uc5c8\uc73c\ub098 2009\ub144 9\uc6d4 15\uc77c \ubbfc\uc8fc\ub2f9\uc758 \ud558\ud1a0\uc57c\ub9c8 \uc720\ud0a4\uc624 \ub0b4\uac01\uc774 \uc2dc\uc791\ub418\uba74\uc11c \ucd5c\ucd08\uc758 \uc815\uad8c \uad50\uccb4\uac00 \uc774\ub8e8\uc5b4\uc84c\ub2e4. 2012\ub144 12\uc6d4 16\uc77c \uc5d0 \uc2e4\uc2dc\ub41c \uc911\uc758\uc6d0 \uc758\uc6d0 \ucd1d\uc120\uac70\uc5d0\uc11c \uc790\uc720\ubbfc\uc8fc\ub2f9\uc774 \ub2e4\uc218\uc758 \uc758\uc11d\uc744 \ucc28\uc9c0\ud558\uc5ec \uc5ec\ub2f9\uc758 \uc790\ub9ac\ub97c \ud0c8\ud658\ud558\uc600\ub2e4. ", "paragraph_answer": "\uc785\ubc95 \uae30\uad00\uc778 \uad6d\ud68c\ub294 \ud5cc\ubc95\uc5d0\uc11c \uba85\uc2dc\ub41c \uad6d\uad8c\uc758 \ucd5c\uace0 \uae30\uad00\uc774\uc790 \ub098\ub77c\uc758 \uc720\uc77c\ud55c \uc785\ubc95 \uae30\uad00\uc73c\ub85c\uc368 \uc601\uad6d\uc2dd \uc758\uc6d0 \ub0b4\uac01\uc81c\ub97c \ucc44\uc6a9\ud588\uae30 \ub54c\ubb38\uc5d0 \uc218\uc0c1\uc758 \uc9c0\uba85\uad8c, \uc911\uc758\uc6d0\uc758 \ub0b4\uac01 \uc2e0\uc784\uacfc \ubd88\uc2e0\uc784\uc758 \uc758\uacb0\uad8c\uc744 \uac00\uc9c4\ub2e4. \uad6d\ud68c\ub294 \uc591\uc6d0\uc81c\ub85c \uc911\uc758\uc6d0 \ubc0f \ucc38\uc758\uc6d0\uc73c\ub85c \uad6c\uc131\ub418\uc5b4 \uc788\ub294\ub370 \uc591\uc6d0\uc740 \uc804 \uad6d\ubbfc\uc758 \ub300\ud45c\ub85c \uc120\ucd9c\ub41c \uc758\uc6d0\uc73c\ub85c \uc870\uc9c1\ub418\uc5b4 \uc788\ub2e4. \uad6d\ud68c\ub294 1955\ub144 \uc774\ud6c4 1993\ub144\ubd80\ud130 1996\ub144\uae4c\uc9c0\uc758 \uc5f0\uc815 \uae30\uac04\uc744 \uc81c\uc678\ud558\uace0 \uc790\uc720\ubbfc\uc8fc\ub2f9\uc774 \uacc4\uc18d\ud574\uc11c \uc5ec\ub2f9\uc758 \uc790\ub9ac\uc5d0 \uc788\uc5c8\uc73c\ub098 2009\ub144 9\uc6d4 15\uc77c \ubbfc\uc8fc\ub2f9\uc758 \ud558\ud1a0\uc57c\ub9c8 \uc720\ud0a4\uc624 \ub0b4\uac01\uc774 \uc2dc\uc791\ub418\uba74\uc11c \ucd5c\ucd08\uc758 \uc815\uad8c \uad50\uccb4\uac00 \uc774\ub8e8\uc5b4\uc84c\ub2e4. 2012\ub144 12\uc6d4 16\uc77c \uc5d0 \uc2e4\uc2dc\ub41c \uc911\uc758\uc6d0 \uc758\uc6d0 \ucd1d\uc120\uac70\uc5d0\uc11c \uc790\uc720\ubbfc\uc8fc\ub2f9\uc774 \ub2e4\uc218\uc758 \uc758\uc11d\uc744 \ucc28\uc9c0\ud558\uc5ec \uc5ec\ub2f9\uc758 \uc790\ub9ac\ub97c \ud0c8\ud658\ud558\uc600\ub2e4.", "sentence_answer": " 2012\ub144 12\uc6d4 16\uc77c \uc5d0 \uc2e4\uc2dc\ub41c \uc911\uc758\uc6d0 \uc758\uc6d0 \ucd1d\uc120\uac70\uc5d0\uc11c \uc790\uc720\ubbfc\uc8fc\ub2f9\uc774 \ub2e4\uc218\uc758 \uc758\uc11d\uc744 \ucc28\uc9c0\ud558\uc5ec \uc5ec\ub2f9\uc758 \uc790\ub9ac\ub97c \ud0c8\ud658\ud558\uc600\ub2e4.", "paragraph_id": "6591477-7-1", "paragraph_question": "question: \uc790\uc720\ubbfc\uc8fc\ub2f9\uc774 \uc5ec\ub2f9\uc758 \uc790\ub9ac\ub97c \ud0c8\ud658\ud55c \ub0a0\uc740 \uc5b8\uc81c\uc778\uac00?, context: \uc785\ubc95 \uae30\uad00\uc778 \uad6d\ud68c\ub294 \ud5cc\ubc95\uc5d0\uc11c \uba85\uc2dc\ub41c \uad6d\uad8c\uc758 \ucd5c\uace0 \uae30\uad00\uc774\uc790 \ub098\ub77c\uc758 \uc720\uc77c\ud55c \uc785\ubc95 \uae30\uad00\uc73c\ub85c\uc368 \uc601\uad6d\uc2dd \uc758\uc6d0 \ub0b4\uac01\uc81c\ub97c \ucc44\uc6a9\ud588\uae30 \ub54c\ubb38\uc5d0 \uc218\uc0c1\uc758 \uc9c0\uba85\uad8c, \uc911\uc758\uc6d0\uc758 \ub0b4\uac01 \uc2e0\uc784\uacfc \ubd88\uc2e0\uc784\uc758 \uc758\uacb0\uad8c\uc744 \uac00\uc9c4\ub2e4. \uad6d\ud68c\ub294 \uc591\uc6d0\uc81c\ub85c \uc911\uc758\uc6d0 \ubc0f \ucc38\uc758\uc6d0\uc73c\ub85c \uad6c\uc131\ub418\uc5b4 \uc788\ub294\ub370 \uc591\uc6d0\uc740 \uc804 \uad6d\ubbfc\uc758 \ub300\ud45c\ub85c \uc120\ucd9c\ub41c \uc758\uc6d0\uc73c\ub85c \uc870\uc9c1\ub418\uc5b4 \uc788\ub2e4. \uad6d\ud68c\ub294 1955\ub144 \uc774\ud6c4 1993\ub144\ubd80\ud130 1996\ub144\uae4c\uc9c0\uc758 \uc5f0\uc815 \uae30\uac04\uc744 \uc81c\uc678\ud558\uace0 \uc790\uc720\ubbfc\uc8fc\ub2f9\uc774 \uacc4\uc18d\ud574\uc11c \uc5ec\ub2f9\uc758 \uc790\ub9ac\uc5d0 \uc788\uc5c8\uc73c\ub098 2009\ub144 9\uc6d4 15\uc77c \ubbfc\uc8fc\ub2f9\uc758 \ud558\ud1a0\uc57c\ub9c8 \uc720\ud0a4\uc624 \ub0b4\uac01\uc774 \uc2dc\uc791\ub418\uba74\uc11c \ucd5c\ucd08\uc758 \uc815\uad8c \uad50\uccb4\uac00 \uc774\ub8e8\uc5b4\uc84c\ub2e4. 2012\ub144 12\uc6d4 16\uc77c\uc5d0 \uc2e4\uc2dc\ub41c \uc911\uc758\uc6d0 \uc758\uc6d0 \ucd1d\uc120\uac70\uc5d0\uc11c \uc790\uc720\ubbfc\uc8fc\ub2f9\uc774 \ub2e4\uc218\uc758 \uc758\uc11d\uc744 \ucc28\uc9c0\ud558\uc5ec \uc5ec\ub2f9\uc758 \uc790\ub9ac\ub97c \ud0c8\ud658\ud558\uc600\ub2e4."} {"question": "2012\ub144 \uccab \ubbf8\ub2c8 \uc74c\ubc18 \uc81c\ubaa9\uc740?", "paragraph": "\ud55c\ud3b8, \uc544\uc774\ube44\ub294 2009\ub144 \ucd08 \ud3f4\ub77c\ub9ac\uc2a4 \uc5d4\ud130\ud14c\uc778\uba3c\ud2b8\ub85c \uc18c\uc18d\uc0ac\ub97c \uc774\uc801\ud560 \uc608\uc815\uc774\uc5c8\uc9c0\ub9cc, \uc804\uc18d\uad8c\uc740 \uc2a4\ud1b0\uc774\uc564\uc5d0\ud504\uc5d0\uac8c \uc788\ub2e4\ub294 \uc774\uc720\ub85c \ubc95\uc815 \ubd84\uc7c1\uc744 \uc77c\uc73c\ucf30\uace0, \uadf8 \uacb0\uacfc 2011\ub144 9\uc6d4 \uc2b9\uc18c\ud558\uba70 \uc18c\uc18d\uc0ac\ub97c \uc62e\uacbc\ub2e4. 2012\ub144 2\uc6d4, \uc544\uc774\ube44\ub294 2008\ub144 3\uc6d4\ubd80\ud130 \ub9cc\ub0ac\ub358 \uc791\uace1\uac00 \uae40\ud0dc\uc131\uacfc 2011\ub144 \ub9d0\uc5d0 \uacb0\ubcc4\ud588\ub2e4\uace0 \ubc1d\ud614\ub2e4. \uc18c\uc18d\uc0ac\uc640 \ubd84\uc7c1\uc774 \ub05d\ub09c \ud6c4, 2012\ub144 4\uc6d4 27\uc77c \uc544\uc774\ube44\uc758 \uccab \ubbf8\ub2c8 \uc74c\ubc18 INTERVIEW\ub97c \ubc1c\ub9e4\ud588\ub2e4. \uc774\ubc88 \uc568\ubc94\uc740 \uc774\uc804\uc758 \uc568\ubc94\ub4e4\uacfc \ub2ec\ub9ac \ud0c0\uc774\ud2c0\uace1 \u3008\ucc22\uae34 \uac00\uc2b4\u3009\uc744 \ud3ec\ud568\ud574 \ub2e4\uc12f \uace1 \ubaa8\ub450 \ubc1c\ub77c\ub4dc\uc640 \ubbf8\ub514\uc5c4 \ud15c\ud3ec \ub178\ub798\ub4e4\ub85c \uc774\ub8e8\uc5b4\uc84c\ub2e4. 2012\ub144 \uc5ec\ub984\uc5d0\ub294 \ubba4\uc9c0\uceec \uc2dc\uce74\uace0\uc5d0 \uc8fc\uc5f0 \ub85d\uc2dc \ud558\ud2b8\ub85c \ucd9c\uc5f0\ud588\ub2e4. 6\uc6d4 19\uc77c\uc5d0\ub294 \ucd94\uc801\uc790 SBS \ub4dc\ub77c\ub9c8 \u300a\ucd94\uc801\uc790 THE CHASER\u300b\uc758 OST \u3008\uc88b\uc740 \uc0ac\ub78c\u3009\uc744 \ubc1c\ub9e4\ud588\ub2e4. 9\uc6d4\ubd80\ud130\ub294 SBS\uc758 \uc624\ub514\uc158 \ud504\ub85c\uadf8\ub7a8 \u300a\ub0b4 \uc0dd\uc560 \ub9c8\uc9c0\ub9c9 \uc624\ub514\uc158\u300b\uc758 \uc2ec\uc0ac\uc704\uc6d0\uc73c\ub85c \ubc1c\ud0c1 \ub3fc \ucd9c\uc5f0\ud588\ub2e4. \ub610\ud55c KBS \uc608\ub2a5 \u300a\ub0a8\uc790\uc758 \uc790\uaca9\u300b \ud328\ubc00\ub9ac \ud569\ucc3d\ub2e8 \ud3b8\uc5d0 \uc5b4\uba38\ub2c8\uc640 \ud568\uaed8 \uc131\uc545\ub2e8\uc73c\ub85c \ucd9c\uc5f0\ud588\ub2e4.", "answer": "INTERVIEW", "sentence": "\uc18c\uc18d\uc0ac\uc640 \ubd84\uc7c1\uc774 \ub05d\ub09c \ud6c4, 2012\ub144 4\uc6d4 27\uc77c \uc544\uc774\ube44\uc758 \uccab \ubbf8\ub2c8 \uc74c\ubc18 INTERVIEW \ub97c \ubc1c\ub9e4\ud588\ub2e4.", "paragraph_sentence": "\ud55c\ud3b8, \uc544\uc774\ube44\ub294 2009\ub144 \ucd08 \ud3f4\ub77c\ub9ac\uc2a4 \uc5d4\ud130\ud14c\uc778\uba3c\ud2b8\ub85c \uc18c\uc18d\uc0ac\ub97c \uc774\uc801\ud560 \uc608\uc815\uc774\uc5c8\uc9c0\ub9cc, \uc804\uc18d\uad8c\uc740 \uc2a4\ud1b0\uc774\uc564\uc5d0\ud504\uc5d0\uac8c \uc788\ub2e4\ub294 \uc774\uc720\ub85c \ubc95\uc815 \ubd84\uc7c1\uc744 \uc77c\uc73c\ucf30\uace0, \uadf8 \uacb0\uacfc 2011\ub144 9\uc6d4 \uc2b9\uc18c\ud558\uba70 \uc18c\uc18d\uc0ac\ub97c \uc62e\uacbc\ub2e4. 2012\ub144 2\uc6d4, \uc544\uc774\ube44\ub294 2008\ub144 3\uc6d4\ubd80\ud130 \ub9cc\ub0ac\ub358 \uc791\uace1\uac00 \uae40\ud0dc\uc131\uacfc 2011\ub144 \ub9d0\uc5d0 \uacb0\ubcc4\ud588\ub2e4\uace0 \ubc1d\ud614\ub2e4. \uc18c\uc18d\uc0ac\uc640 \ubd84\uc7c1\uc774 \ub05d\ub09c \ud6c4, 2012\ub144 4\uc6d4 27\uc77c \uc544\uc774\ube44\uc758 \uccab \ubbf8\ub2c8 \uc74c\ubc18 INTERVIEW \ub97c \ubc1c\ub9e4\ud588\ub2e4. \uc774\ubc88 \uc568\ubc94\uc740 \uc774\uc804\uc758 \uc568\ubc94\ub4e4\uacfc \ub2ec\ub9ac \ud0c0\uc774\ud2c0\uace1 \u3008\ucc22\uae34 \uac00\uc2b4\u3009\uc744 \ud3ec\ud568\ud574 \ub2e4\uc12f \uace1 \ubaa8\ub450 \ubc1c\ub77c\ub4dc\uc640 \ubbf8\ub514\uc5c4 \ud15c\ud3ec \ub178\ub798\ub4e4\ub85c \uc774\ub8e8\uc5b4\uc84c\ub2e4. 2012\ub144 \uc5ec\ub984\uc5d0\ub294 \ubba4\uc9c0\uceec \uc2dc\uce74\uace0\uc5d0 \uc8fc\uc5f0 \ub85d\uc2dc \ud558\ud2b8\ub85c \ucd9c\uc5f0\ud588\ub2e4. 6\uc6d4 19\uc77c\uc5d0\ub294 \ucd94\uc801\uc790 SBS \ub4dc\ub77c\ub9c8 \u300a\ucd94\uc801\uc790 THE CHASER\u300b\uc758 OST \u3008\uc88b\uc740 \uc0ac\ub78c\u3009\uc744 \ubc1c\ub9e4\ud588\ub2e4. 9\uc6d4\ubd80\ud130\ub294 SBS\uc758 \uc624\ub514\uc158 \ud504\ub85c\uadf8\ub7a8 \u300a\ub0b4 \uc0dd\uc560 \ub9c8\uc9c0\ub9c9 \uc624\ub514\uc158\u300b\uc758 \uc2ec\uc0ac\uc704\uc6d0\uc73c\ub85c \ubc1c\ud0c1 \ub3fc \ucd9c\uc5f0\ud588\ub2e4. \ub610\ud55c KBS \uc608\ub2a5 \u300a\ub0a8\uc790\uc758 \uc790\uaca9\u300b \ud328\ubc00\ub9ac \ud569\ucc3d\ub2e8 \ud3b8\uc5d0 \uc5b4\uba38\ub2c8\uc640 \ud568\uaed8 \uc131\uc545\ub2e8\uc73c\ub85c \ucd9c\uc5f0\ud588\ub2e4.", "paragraph_answer": "\ud55c\ud3b8, \uc544\uc774\ube44\ub294 2009\ub144 \ucd08 \ud3f4\ub77c\ub9ac\uc2a4 \uc5d4\ud130\ud14c\uc778\uba3c\ud2b8\ub85c \uc18c\uc18d\uc0ac\ub97c \uc774\uc801\ud560 \uc608\uc815\uc774\uc5c8\uc9c0\ub9cc, \uc804\uc18d\uad8c\uc740 \uc2a4\ud1b0\uc774\uc564\uc5d0\ud504\uc5d0\uac8c \uc788\ub2e4\ub294 \uc774\uc720\ub85c \ubc95\uc815 \ubd84\uc7c1\uc744 \uc77c\uc73c\ucf30\uace0, \uadf8 \uacb0\uacfc 2011\ub144 9\uc6d4 \uc2b9\uc18c\ud558\uba70 \uc18c\uc18d\uc0ac\ub97c \uc62e\uacbc\ub2e4. 2012\ub144 2\uc6d4, \uc544\uc774\ube44\ub294 2008\ub144 3\uc6d4\ubd80\ud130 \ub9cc\ub0ac\ub358 \uc791\uace1\uac00 \uae40\ud0dc\uc131\uacfc 2011\ub144 \ub9d0\uc5d0 \uacb0\ubcc4\ud588\ub2e4\uace0 \ubc1d\ud614\ub2e4. \uc18c\uc18d\uc0ac\uc640 \ubd84\uc7c1\uc774 \ub05d\ub09c \ud6c4, 2012\ub144 4\uc6d4 27\uc77c \uc544\uc774\ube44\uc758 \uccab \ubbf8\ub2c8 \uc74c\ubc18 INTERVIEW \ub97c \ubc1c\ub9e4\ud588\ub2e4. \uc774\ubc88 \uc568\ubc94\uc740 \uc774\uc804\uc758 \uc568\ubc94\ub4e4\uacfc \ub2ec\ub9ac \ud0c0\uc774\ud2c0\uace1 \u3008\ucc22\uae34 \uac00\uc2b4\u3009\uc744 \ud3ec\ud568\ud574 \ub2e4\uc12f \uace1 \ubaa8\ub450 \ubc1c\ub77c\ub4dc\uc640 \ubbf8\ub514\uc5c4 \ud15c\ud3ec \ub178\ub798\ub4e4\ub85c \uc774\ub8e8\uc5b4\uc84c\ub2e4. 2012\ub144 \uc5ec\ub984\uc5d0\ub294 \ubba4\uc9c0\uceec \uc2dc\uce74\uace0\uc5d0 \uc8fc\uc5f0 \ub85d\uc2dc \ud558\ud2b8\ub85c \ucd9c\uc5f0\ud588\ub2e4. 6\uc6d4 19\uc77c\uc5d0\ub294 \ucd94\uc801\uc790 SBS \ub4dc\ub77c\ub9c8 \u300a\ucd94\uc801\uc790 THE CHASER\u300b\uc758 OST \u3008\uc88b\uc740 \uc0ac\ub78c\u3009\uc744 \ubc1c\ub9e4\ud588\ub2e4. 9\uc6d4\ubd80\ud130\ub294 SBS\uc758 \uc624\ub514\uc158 \ud504\ub85c\uadf8\ub7a8 \u300a\ub0b4 \uc0dd\uc560 \ub9c8\uc9c0\ub9c9 \uc624\ub514\uc158\u300b\uc758 \uc2ec\uc0ac\uc704\uc6d0\uc73c\ub85c \ubc1c\ud0c1 \ub3fc \ucd9c\uc5f0\ud588\ub2e4. \ub610\ud55c KBS \uc608\ub2a5 \u300a\ub0a8\uc790\uc758 \uc790\uaca9\u300b \ud328\ubc00\ub9ac \ud569\ucc3d\ub2e8 \ud3b8\uc5d0 \uc5b4\uba38\ub2c8\uc640 \ud568\uaed8 \uc131\uc545\ub2e8\uc73c\ub85c \ucd9c\uc5f0\ud588\ub2e4.", "sentence_answer": "\uc18c\uc18d\uc0ac\uc640 \ubd84\uc7c1\uc774 \ub05d\ub09c \ud6c4, 2012\ub144 4\uc6d4 27\uc77c \uc544\uc774\ube44\uc758 \uccab \ubbf8\ub2c8 \uc74c\ubc18 INTERVIEW \ub97c \ubc1c\ub9e4\ud588\ub2e4.", "paragraph_id": "6520985-5-0", "paragraph_question": "question: 2012\ub144 \uccab \ubbf8\ub2c8 \uc74c\ubc18 \uc81c\ubaa9\uc740?, context: \ud55c\ud3b8, \uc544\uc774\ube44\ub294 2009\ub144 \ucd08 \ud3f4\ub77c\ub9ac\uc2a4 \uc5d4\ud130\ud14c\uc778\uba3c\ud2b8\ub85c \uc18c\uc18d\uc0ac\ub97c \uc774\uc801\ud560 \uc608\uc815\uc774\uc5c8\uc9c0\ub9cc, \uc804\uc18d\uad8c\uc740 \uc2a4\ud1b0\uc774\uc564\uc5d0\ud504\uc5d0\uac8c \uc788\ub2e4\ub294 \uc774\uc720\ub85c \ubc95\uc815 \ubd84\uc7c1\uc744 \uc77c\uc73c\ucf30\uace0, \uadf8 \uacb0\uacfc 2011\ub144 9\uc6d4 \uc2b9\uc18c\ud558\uba70 \uc18c\uc18d\uc0ac\ub97c \uc62e\uacbc\ub2e4. 2012\ub144 2\uc6d4, \uc544\uc774\ube44\ub294 2008\ub144 3\uc6d4\ubd80\ud130 \ub9cc\ub0ac\ub358 \uc791\uace1\uac00 \uae40\ud0dc\uc131\uacfc 2011\ub144 \ub9d0\uc5d0 \uacb0\ubcc4\ud588\ub2e4\uace0 \ubc1d\ud614\ub2e4. \uc18c\uc18d\uc0ac\uc640 \ubd84\uc7c1\uc774 \ub05d\ub09c \ud6c4, 2012\ub144 4\uc6d4 27\uc77c \uc544\uc774\ube44\uc758 \uccab \ubbf8\ub2c8 \uc74c\ubc18 INTERVIEW\ub97c \ubc1c\ub9e4\ud588\ub2e4. \uc774\ubc88 \uc568\ubc94\uc740 \uc774\uc804\uc758 \uc568\ubc94\ub4e4\uacfc \ub2ec\ub9ac \ud0c0\uc774\ud2c0\uace1 \u3008\ucc22\uae34 \uac00\uc2b4\u3009\uc744 \ud3ec\ud568\ud574 \ub2e4\uc12f \uace1 \ubaa8\ub450 \ubc1c\ub77c\ub4dc\uc640 \ubbf8\ub514\uc5c4 \ud15c\ud3ec \ub178\ub798\ub4e4\ub85c \uc774\ub8e8\uc5b4\uc84c\ub2e4. 2012\ub144 \uc5ec\ub984\uc5d0\ub294 \ubba4\uc9c0\uceec \uc2dc\uce74\uace0\uc5d0 \uc8fc\uc5f0 \ub85d\uc2dc \ud558\ud2b8\ub85c \ucd9c\uc5f0\ud588\ub2e4. 6\uc6d4 19\uc77c\uc5d0\ub294 \ucd94\uc801\uc790 SBS \ub4dc\ub77c\ub9c8 \u300a\ucd94\uc801\uc790 THE CHASER\u300b\uc758 OST \u3008\uc88b\uc740 \uc0ac\ub78c\u3009\uc744 \ubc1c\ub9e4\ud588\ub2e4. 9\uc6d4\ubd80\ud130\ub294 SBS\uc758 \uc624\ub514\uc158 \ud504\ub85c\uadf8\ub7a8 \u300a\ub0b4 \uc0dd\uc560 \ub9c8\uc9c0\ub9c9 \uc624\ub514\uc158\u300b\uc758 \uc2ec\uc0ac\uc704\uc6d0\uc73c\ub85c \ubc1c\ud0c1 \ub3fc \ucd9c\uc5f0\ud588\ub2e4. \ub610\ud55c KBS \uc608\ub2a5 \u300a\ub0a8\uc790\uc758 \uc790\uaca9\u300b \ud328\ubc00\ub9ac \ud569\ucc3d\ub2e8 \ud3b8\uc5d0 \uc5b4\uba38\ub2c8\uc640 \ud568\uaed8 \uc131\uc545\ub2e8\uc73c\ub85c \ucd9c\uc5f0\ud588\ub2e4."} {"question": "\ub9e8\uc2a8 \ud328\ubc00\ub9ac \uc77c\ub2f9\uc740 \ub204\uad6c\uc758 \uc8fc\ub3c4 \ud558\uc5d0 \ub9cc\ub4e4\uc5b4\uc84c\ub294\uac00?", "paragraph": "\uadf8 \ud6c4 1969\ub144\uc5d0 \ub9e8\uc2a8 \uc77c\ub2f9\uc740 \ub85c\ub9cc \ud3f4\ub780\uc2a4\ud0a4\uc758 \uc9d1\uc744 \uc2b5\uaca9\ud558\uc5ec \uadf8\uc758 \uc544\ub0b4\uc774\uc790 \uc720\uba85 \uc601\ud654\ubc30\uc6b0\uc778 \uc0e4\ub860 \ud14c\uc774\ud2b8\ub97c \ud3ec\ud568\ud55c 5\uba85\uc744 \ub09c\ub3c4\uc9c8\ud574 \uc8fd\uac8c \ud588\ub2e4. \uc0ac\uac74 \ub2f9\uc2dc\uc758 \uc0c1\ud669\uc740 \uc9d1 \uc8fc\uc778\uc778 \ub85c\ub9cc \ud3f4\ub780\uc2a4\ud0a4\ub294 \uc601\ud654 \ucd2c\uc601\uc73c\ub85c \uc778\ud558\uc5ec \uc678\ucd9c\uc744 \ud55c \uc0c1\ud0dc\uc600\uc73c\ubbc0\ub85c \uc0e4\ub860 \ud14c\uc774\ud2b8\ub294 \ub0a8\ud3b8\uc774 \uc9d1\uc744 \ube44\uc6b4 \uc0ac\uc774 \uce5c\uad6c\ub4e4\uc744\ub97c \uc9d1\uc73c\ub85c \ucd08\ub300\ud55c \uc0c1\ud0dc\uc600\ub2e4. \uc218\uc794 \uc573\ud0a8\uc2a8\uc774 \uc8fc\ub3d9\uc774 \ub41c \ub9e8\uc2a8 \ud328\ubc00\ub9ac \uc77c\ub2f9\uc740 \uc0e4\ub860 \ud14c\uc774\ud2b8\ub97c \ube44\ub86f\ud558\uc5ec \ud604\uc7a5\uc5d0 \uc788\ub358 5\uba85 \uc804\uc6d0\uc744 \ubab0\uc0b4\ud588\uc73c\uba70 \ud2b9\ud788 \ub2f9\uc2dc \uc784\uc2e0\uc911\uc778 \uc0e4\ub860 \ud14c\uc774\ud2b8\uac00 \ud0dc\uc544\ub9cc\uc774\ub77c\ub3c4 \uc0b4\ub824\ub2ec\ub77c\uace0 \uc560\uc6d0\ud568\uc5d0\ub3c4 \ubd88\uad6c\ud558\uace0 \uc218\uc794 \uc573\ud0a8\uc2a8\uc740 \uc624\ud788\ub824 \uc0e4\ub860 \ud14c\uc774\ud2b8\ub97c \ub354\ub7ec\uc6b4 \ub144\uc774\ub77c \uc695\ud558\uace0 \uc8fd\uc778 \ub4a4 \uadf8 \uc2dc\uccb4\ub97c \ubc8c\uac70\ubc97\uacbc\ub2e4. \uacb0\uad6d \ub098\uc911\uc5d0 \uc218\uc794 \uc573\ud0a8\uc2a8\ub4f1 4\uba85\uc774 \uccb4\ud3ec\ub418\uc5c8\uc73c\uba70 \ucc30\uc2a4 \ub9e8\uc2a8 \uc5ed\uc2dc \ubc30\ud6c4\uc138\ub825\uc73c\ub85c \uc9c0\ubaa9\ubc1b\uc544 \uccb4\ud3ec\ub418\uc5c8\ub2e4. \ucc30\uc2a4 \ub9e8\uc2a8\uc740 \uc218\ub9ce\uc740 \uccb4\ud3ec\ub2f9\ud588\ub358 \uacbd\ud5d8\uc744 \uc0b4\ub824 \uc7ac\ud310\uc744 \uae38\uac8c \ub04c\uc5c8\uc73c\ub098 \uacb0\uad6d 1971\ub144 \uc0ac\ud615\uc744 \uc120\uace0\ubc1b\uc558\ub2e4. \uadf8\ub7ec\ub098 1972\ub144\uc5d0 \ub9e8\uc2a8\uc744 \uccb4\ud3ec\ud55c \uce98\ub9ac\ud3ec\ub2c8\uc544 \uc8fc\uac00 \uc0ac\ud615\uc81c\ub3c4\ub97c \ud3d0\uc9c0\ud558\ub294 \ubc14\ub78c\uc5d0 \ub9e8\uc2a8\uc740 \uc0ac\ud615\uc5d0\uc11c \ubb34\uae30\uc9d5\uc5ed\uc73c\ub85c \uac10\ud615\ub418\uc5b4 \ucd5c\uadfc\uae4c\uc9c0 \uad50\ub3c4\uc18c\uc5d0 \uc218\uac10\ub418\uc5b4 \uc788\ub2e4\uac00 2017\ub144 11\uc6d4 19\uc77c \uc790\uc5f0\uc0ac\ud588\ub2e4.", "answer": "\uc218\uc794 \uc573\ud0a8\uc2a8", "sentence": "\uc218\uc794 \uc573\ud0a8\uc2a8 \uc774 \uc8fc\ub3d9\uc774 \ub41c \ub9e8\uc2a8 \ud328\ubc00\ub9ac \uc77c\ub2f9\uc740 \uc0e4\ub860 \ud14c\uc774\ud2b8\ub97c \ube44\ub86f\ud558\uc5ec \ud604\uc7a5\uc5d0 \uc788\ub358 5\uba85 \uc804\uc6d0\uc744 \ubab0\uc0b4\ud588\uc73c\uba70 \ud2b9\ud788 \ub2f9\uc2dc \uc784\uc2e0\uc911\uc778 \uc0e4\ub860 \ud14c\uc774\ud2b8\uac00 \ud0dc\uc544\ub9cc\uc774\ub77c\ub3c4 \uc0b4\ub824\ub2ec\ub77c\uace0 \uc560\uc6d0\ud568\uc5d0\ub3c4 \ubd88\uad6c\ud558\uace0 \uc218\uc794 \uc573\ud0a8\uc2a8\uc740 \uc624\ud788\ub824 \uc0e4\ub860 \ud14c\uc774\ud2b8\ub97c \ub354\ub7ec\uc6b4 \ub144\uc774\ub77c \uc695\ud558\uace0 \uc8fd\uc778 \ub4a4 \uadf8 \uc2dc\uccb4\ub97c \ubc8c\uac70\ubc97\uacbc\ub2e4.", "paragraph_sentence": "\uadf8 \ud6c4 1969\ub144\uc5d0 \ub9e8\uc2a8 \uc77c\ub2f9\uc740 \ub85c\ub9cc \ud3f4\ub780\uc2a4\ud0a4\uc758 \uc9d1\uc744 \uc2b5\uaca9\ud558\uc5ec \uadf8\uc758 \uc544\ub0b4\uc774\uc790 \uc720\uba85 \uc601\ud654\ubc30\uc6b0\uc778 \uc0e4\ub860 \ud14c\uc774\ud2b8\ub97c \ud3ec\ud568\ud55c 5\uba85\uc744 \ub09c\ub3c4\uc9c8\ud574 \uc8fd\uac8c \ud588\ub2e4. \uc0ac\uac74 \ub2f9\uc2dc\uc758 \uc0c1\ud669\uc740 \uc9d1 \uc8fc\uc778\uc778 \ub85c\ub9cc \ud3f4\ub780\uc2a4\ud0a4\ub294 \uc601\ud654 \ucd2c\uc601\uc73c\ub85c \uc778\ud558\uc5ec \uc678\ucd9c\uc744 \ud55c \uc0c1\ud0dc\uc600\uc73c\ubbc0\ub85c \uc0e4\ub860 \ud14c\uc774\ud2b8\ub294 \ub0a8\ud3b8\uc774 \uc9d1\uc744 \ube44\uc6b4 \uc0ac\uc774 \uce5c\uad6c\ub4e4\uc744\ub97c \uc9d1\uc73c\ub85c \ucd08\ub300\ud55c \uc0c1\ud0dc\uc600\ub2e4. \uc218\uc794 \uc573\ud0a8\uc2a8 \uc774 \uc8fc\ub3d9\uc774 \ub41c \ub9e8\uc2a8 \ud328\ubc00\ub9ac \uc77c\ub2f9\uc740 \uc0e4\ub860 \ud14c\uc774\ud2b8\ub97c \ube44\ub86f\ud558\uc5ec \ud604\uc7a5\uc5d0 \uc788\ub358 5\uba85 \uc804\uc6d0\uc744 \ubab0\uc0b4\ud588\uc73c\uba70 \ud2b9\ud788 \ub2f9\uc2dc \uc784\uc2e0\uc911\uc778 \uc0e4\ub860 \ud14c\uc774\ud2b8\uac00 \ud0dc\uc544\ub9cc\uc774\ub77c\ub3c4 \uc0b4\ub824\ub2ec\ub77c\uace0 \uc560\uc6d0\ud568\uc5d0\ub3c4 \ubd88\uad6c\ud558\uace0 \uc218\uc794 \uc573\ud0a8\uc2a8\uc740 \uc624\ud788\ub824 \uc0e4\ub860 \ud14c\uc774\ud2b8\ub97c \ub354\ub7ec\uc6b4 \ub144\uc774\ub77c \uc695\ud558\uace0 \uc8fd\uc778 \ub4a4 \uadf8 \uc2dc\uccb4\ub97c \ubc8c\uac70\ubc97\uacbc\ub2e4. \uacb0\uad6d \ub098\uc911\uc5d0 \uc218\uc794 \uc573\ud0a8\uc2a8\ub4f1 4\uba85\uc774 \uccb4\ud3ec\ub418\uc5c8\uc73c\uba70 \ucc30\uc2a4 \ub9e8\uc2a8 \uc5ed\uc2dc \ubc30\ud6c4\uc138\ub825\uc73c\ub85c \uc9c0\ubaa9\ubc1b\uc544 \uccb4\ud3ec\ub418\uc5c8\ub2e4. \ucc30\uc2a4 \ub9e8\uc2a8\uc740 \uc218\ub9ce\uc740 \uccb4\ud3ec\ub2f9\ud588\ub358 \uacbd\ud5d8\uc744 \uc0b4\ub824 \uc7ac\ud310\uc744 \uae38\uac8c \ub04c\uc5c8\uc73c\ub098 \uacb0\uad6d 1971\ub144 \uc0ac\ud615\uc744 \uc120\uace0\ubc1b\uc558\ub2e4. \uadf8\ub7ec\ub098 1972\ub144\uc5d0 \ub9e8\uc2a8\uc744 \uccb4\ud3ec\ud55c \uce98\ub9ac\ud3ec\ub2c8\uc544 \uc8fc\uac00 \uc0ac\ud615\uc81c\ub3c4\ub97c \ud3d0\uc9c0\ud558\ub294 \ubc14\ub78c\uc5d0 \ub9e8\uc2a8\uc740 \uc0ac\ud615\uc5d0\uc11c \ubb34\uae30\uc9d5\uc5ed\uc73c\ub85c \uac10\ud615\ub418\uc5b4 \ucd5c\uadfc\uae4c\uc9c0 \uad50\ub3c4\uc18c\uc5d0 \uc218\uac10\ub418\uc5b4 \uc788\ub2e4\uac00 2017\ub144 11\uc6d4 19\uc77c \uc790\uc5f0\uc0ac\ud588\ub2e4.", "paragraph_answer": "\uadf8 \ud6c4 1969\ub144\uc5d0 \ub9e8\uc2a8 \uc77c\ub2f9\uc740 \ub85c\ub9cc \ud3f4\ub780\uc2a4\ud0a4\uc758 \uc9d1\uc744 \uc2b5\uaca9\ud558\uc5ec \uadf8\uc758 \uc544\ub0b4\uc774\uc790 \uc720\uba85 \uc601\ud654\ubc30\uc6b0\uc778 \uc0e4\ub860 \ud14c\uc774\ud2b8\ub97c \ud3ec\ud568\ud55c 5\uba85\uc744 \ub09c\ub3c4\uc9c8\ud574 \uc8fd\uac8c \ud588\ub2e4. \uc0ac\uac74 \ub2f9\uc2dc\uc758 \uc0c1\ud669\uc740 \uc9d1 \uc8fc\uc778\uc778 \ub85c\ub9cc \ud3f4\ub780\uc2a4\ud0a4\ub294 \uc601\ud654 \ucd2c\uc601\uc73c\ub85c \uc778\ud558\uc5ec \uc678\ucd9c\uc744 \ud55c \uc0c1\ud0dc\uc600\uc73c\ubbc0\ub85c \uc0e4\ub860 \ud14c\uc774\ud2b8\ub294 \ub0a8\ud3b8\uc774 \uc9d1\uc744 \ube44\uc6b4 \uc0ac\uc774 \uce5c\uad6c\ub4e4\uc744\ub97c \uc9d1\uc73c\ub85c \ucd08\ub300\ud55c \uc0c1\ud0dc\uc600\ub2e4. \uc218\uc794 \uc573\ud0a8\uc2a8 \uc774 \uc8fc\ub3d9\uc774 \ub41c \ub9e8\uc2a8 \ud328\ubc00\ub9ac \uc77c\ub2f9\uc740 \uc0e4\ub860 \ud14c\uc774\ud2b8\ub97c \ube44\ub86f\ud558\uc5ec \ud604\uc7a5\uc5d0 \uc788\ub358 5\uba85 \uc804\uc6d0\uc744 \ubab0\uc0b4\ud588\uc73c\uba70 \ud2b9\ud788 \ub2f9\uc2dc \uc784\uc2e0\uc911\uc778 \uc0e4\ub860 \ud14c\uc774\ud2b8\uac00 \ud0dc\uc544\ub9cc\uc774\ub77c\ub3c4 \uc0b4\ub824\ub2ec\ub77c\uace0 \uc560\uc6d0\ud568\uc5d0\ub3c4 \ubd88\uad6c\ud558\uace0 \uc218\uc794 \uc573\ud0a8\uc2a8\uc740 \uc624\ud788\ub824 \uc0e4\ub860 \ud14c\uc774\ud2b8\ub97c \ub354\ub7ec\uc6b4 \ub144\uc774\ub77c \uc695\ud558\uace0 \uc8fd\uc778 \ub4a4 \uadf8 \uc2dc\uccb4\ub97c \ubc8c\uac70\ubc97\uacbc\ub2e4. \uacb0\uad6d \ub098\uc911\uc5d0 \uc218\uc794 \uc573\ud0a8\uc2a8\ub4f1 4\uba85\uc774 \uccb4\ud3ec\ub418\uc5c8\uc73c\uba70 \ucc30\uc2a4 \ub9e8\uc2a8 \uc5ed\uc2dc \ubc30\ud6c4\uc138\ub825\uc73c\ub85c \uc9c0\ubaa9\ubc1b\uc544 \uccb4\ud3ec\ub418\uc5c8\ub2e4. \ucc30\uc2a4 \ub9e8\uc2a8\uc740 \uc218\ub9ce\uc740 \uccb4\ud3ec\ub2f9\ud588\ub358 \uacbd\ud5d8\uc744 \uc0b4\ub824 \uc7ac\ud310\uc744 \uae38\uac8c \ub04c\uc5c8\uc73c\ub098 \uacb0\uad6d 1971\ub144 \uc0ac\ud615\uc744 \uc120\uace0\ubc1b\uc558\ub2e4. \uadf8\ub7ec\ub098 1972\ub144\uc5d0 \ub9e8\uc2a8\uc744 \uccb4\ud3ec\ud55c \uce98\ub9ac\ud3ec\ub2c8\uc544 \uc8fc\uac00 \uc0ac\ud615\uc81c\ub3c4\ub97c \ud3d0\uc9c0\ud558\ub294 \ubc14\ub78c\uc5d0 \ub9e8\uc2a8\uc740 \uc0ac\ud615\uc5d0\uc11c \ubb34\uae30\uc9d5\uc5ed\uc73c\ub85c \uac10\ud615\ub418\uc5b4 \ucd5c\uadfc\uae4c\uc9c0 \uad50\ub3c4\uc18c\uc5d0 \uc218\uac10\ub418\uc5b4 \uc788\ub2e4\uac00 2017\ub144 11\uc6d4 19\uc77c \uc790\uc5f0\uc0ac\ud588\ub2e4.", "sentence_answer": " \uc218\uc794 \uc573\ud0a8\uc2a8 \uc774 \uc8fc\ub3d9\uc774 \ub41c \ub9e8\uc2a8 \ud328\ubc00\ub9ac \uc77c\ub2f9\uc740 \uc0e4\ub860 \ud14c\uc774\ud2b8\ub97c \ube44\ub86f\ud558\uc5ec \ud604\uc7a5\uc5d0 \uc788\ub358 5\uba85 \uc804\uc6d0\uc744 \ubab0\uc0b4\ud588\uc73c\uba70 \ud2b9\ud788 \ub2f9\uc2dc \uc784\uc2e0\uc911\uc778 \uc0e4\ub860 \ud14c\uc774\ud2b8\uac00 \ud0dc\uc544\ub9cc\uc774\ub77c\ub3c4 \uc0b4\ub824\ub2ec\ub77c\uace0 \uc560\uc6d0\ud568\uc5d0\ub3c4 \ubd88\uad6c\ud558\uace0 \uc218\uc794 \uc573\ud0a8\uc2a8\uc740 \uc624\ud788\ub824 \uc0e4\ub860 \ud14c\uc774\ud2b8\ub97c \ub354\ub7ec\uc6b4 \ub144\uc774\ub77c \uc695\ud558\uace0 \uc8fd\uc778 \ub4a4 \uadf8 \uc2dc\uccb4\ub97c \ubc8c\uac70\ubc97\uacbc\ub2e4.", "paragraph_id": "6557826-0-1", "paragraph_question": "question: \ub9e8\uc2a8 \ud328\ubc00\ub9ac \uc77c\ub2f9\uc740 \ub204\uad6c\uc758 \uc8fc\ub3c4 \ud558\uc5d0 \ub9cc\ub4e4\uc5b4\uc84c\ub294\uac00?, context: \uadf8 \ud6c4 1969\ub144\uc5d0 \ub9e8\uc2a8 \uc77c\ub2f9\uc740 \ub85c\ub9cc \ud3f4\ub780\uc2a4\ud0a4\uc758 \uc9d1\uc744 \uc2b5\uaca9\ud558\uc5ec \uadf8\uc758 \uc544\ub0b4\uc774\uc790 \uc720\uba85 \uc601\ud654\ubc30\uc6b0\uc778 \uc0e4\ub860 \ud14c\uc774\ud2b8\ub97c \ud3ec\ud568\ud55c 5\uba85\uc744 \ub09c\ub3c4\uc9c8\ud574 \uc8fd\uac8c \ud588\ub2e4. \uc0ac\uac74 \ub2f9\uc2dc\uc758 \uc0c1\ud669\uc740 \uc9d1 \uc8fc\uc778\uc778 \ub85c\ub9cc \ud3f4\ub780\uc2a4\ud0a4\ub294 \uc601\ud654 \ucd2c\uc601\uc73c\ub85c \uc778\ud558\uc5ec \uc678\ucd9c\uc744 \ud55c \uc0c1\ud0dc\uc600\uc73c\ubbc0\ub85c \uc0e4\ub860 \ud14c\uc774\ud2b8\ub294 \ub0a8\ud3b8\uc774 \uc9d1\uc744 \ube44\uc6b4 \uc0ac\uc774 \uce5c\uad6c\ub4e4\uc744\ub97c \uc9d1\uc73c\ub85c \ucd08\ub300\ud55c \uc0c1\ud0dc\uc600\ub2e4. \uc218\uc794 \uc573\ud0a8\uc2a8\uc774 \uc8fc\ub3d9\uc774 \ub41c \ub9e8\uc2a8 \ud328\ubc00\ub9ac \uc77c\ub2f9\uc740 \uc0e4\ub860 \ud14c\uc774\ud2b8\ub97c \ube44\ub86f\ud558\uc5ec \ud604\uc7a5\uc5d0 \uc788\ub358 5\uba85 \uc804\uc6d0\uc744 \ubab0\uc0b4\ud588\uc73c\uba70 \ud2b9\ud788 \ub2f9\uc2dc \uc784\uc2e0\uc911\uc778 \uc0e4\ub860 \ud14c\uc774\ud2b8\uac00 \ud0dc\uc544\ub9cc\uc774\ub77c\ub3c4 \uc0b4\ub824\ub2ec\ub77c\uace0 \uc560\uc6d0\ud568\uc5d0\ub3c4 \ubd88\uad6c\ud558\uace0 \uc218\uc794 \uc573\ud0a8\uc2a8\uc740 \uc624\ud788\ub824 \uc0e4\ub860 \ud14c\uc774\ud2b8\ub97c \ub354\ub7ec\uc6b4 \ub144\uc774\ub77c \uc695\ud558\uace0 \uc8fd\uc778 \ub4a4 \uadf8 \uc2dc\uccb4\ub97c \ubc8c\uac70\ubc97\uacbc\ub2e4. \uacb0\uad6d \ub098\uc911\uc5d0 \uc218\uc794 \uc573\ud0a8\uc2a8\ub4f1 4\uba85\uc774 \uccb4\ud3ec\ub418\uc5c8\uc73c\uba70 \ucc30\uc2a4 \ub9e8\uc2a8 \uc5ed\uc2dc \ubc30\ud6c4\uc138\ub825\uc73c\ub85c \uc9c0\ubaa9\ubc1b\uc544 \uccb4\ud3ec\ub418\uc5c8\ub2e4. \ucc30\uc2a4 \ub9e8\uc2a8\uc740 \uc218\ub9ce\uc740 \uccb4\ud3ec\ub2f9\ud588\ub358 \uacbd\ud5d8\uc744 \uc0b4\ub824 \uc7ac\ud310\uc744 \uae38\uac8c \ub04c\uc5c8\uc73c\ub098 \uacb0\uad6d 1971\ub144 \uc0ac\ud615\uc744 \uc120\uace0\ubc1b\uc558\ub2e4. \uadf8\ub7ec\ub098 1972\ub144\uc5d0 \ub9e8\uc2a8\uc744 \uccb4\ud3ec\ud55c \uce98\ub9ac\ud3ec\ub2c8\uc544 \uc8fc\uac00 \uc0ac\ud615\uc81c\ub3c4\ub97c \ud3d0\uc9c0\ud558\ub294 \ubc14\ub78c\uc5d0 \ub9e8\uc2a8\uc740 \uc0ac\ud615\uc5d0\uc11c \ubb34\uae30\uc9d5\uc5ed\uc73c\ub85c \uac10\ud615\ub418\uc5b4 \ucd5c\uadfc\uae4c\uc9c0 \uad50\ub3c4\uc18c\uc5d0 \uc218\uac10\ub418\uc5b4 \uc788\ub2e4\uac00 2017\ub144 11\uc6d4 19\uc77c \uc790\uc5f0\uc0ac\ud588\ub2e4."} {"question": "\ucd1d\ud1b5\uc73c\ub85c \uc7a5\uc9d5\uad88\uac00 \uc784\uba85\ub41c \uc774\ud6c4 \uc77c\ubd80 \uc628\uac74\uc6b0\ud30c\ub4e4\uc740 \uc5b4\ub5a4 \ub178\uc120\uc73c\ub85c \uc785\uc7a5\uc744 \ubc14\uafb8\uc5c8\ub294\uac00?", "paragraph": "\uad6d\uacf5 \ub0b4\uc804\uc5d0\uc11c \uc911\uad6d \uacf5\uc0b0\ub2f9\uc774 \ub300\ub959\uc744 \uc7a5\uc545\ud558\uac8c \ub41c \ud6c4, \uadf9\uc6b0 \uc138\ub825\ub4e4\uc740 \ud0c0\uc774\uc644\uc73c\ub85c \ud53c\uc2e0\ud558\uc5ec, \ud0c0\uc774\uc644 \uc6d0\uc8fc\ubbfc\ub4e4\uacfc \ud55c\uc778\uc758 \ucc28\ubcc4 \uc815\ucc45\uc744 \uc9c0\uc9c0\ud588\ub2e4. \uc7a5\uc81c\uc2a4\uac00 \uc0ac\ub9dd\ud558\uc600\ub358 1975\ub144\uae4c\uc9c0 \uc774\ub978\ubc14 '\uc804\uc2dc\ubbfc\uc8fc\uccb4\uc81c'\ub77c\ub294 \ubc18\ub3c5\uc7ac\uccb4\uc81c\ub97c \uc720\uc9c0\ud558\uc5ec \uc77c\uc778\uc77c\ud30c \ub3c5\uc7ac\uccb4\uc81c\ub97c \uc644\uace0\ud788 \ub2e4\uc84c\uc5c8\ub2e4. \uc774\ud6c4, \uc911\ud654\ubbfc\uad6d \ub0b4 \uadf9\uc6b0\ub294 \ub300\ub959\uc218\ubcf5\uc774\ub77c\ub294 \uba85\ubd84\uc744 \uac00\uc9c0\uace0 \ubc18\ub300\ud30c\ub4e4\uc744 \uacf5\uc0b0\uc8fc\uc758\ub85c \ubab0\uc544 \ud559\uc0b4\ud588\uc73c\ub098, \uc7a5\uc81c\uc2a4\uac00 \uc8fd\uace0, \uad6d\uc81c\uc801\uc73c\ub85c \uc911\ud654\ubbfc\uad6d\uc774 \uc544\ub2cc \uc911\ud654\uc778\ubbfc\uacf5\ud654\uad6d\uc744 \ud558\ub098\uc758 \uc911\uad6d\uc73c\ub85c \ubcf4\ub3c4\ub85d \uad6d\uc81c\uc801\uc778 \uc778\uc2dd\uc774 \ubc14\ub00c\uace0, \uc7a5\uc9d5\uad88\uac00 \ucd1d\ud1b5\uc774 \ub418\uace0 \ubbfc\uc8fc\ud654\ub41c \uc774\ud6c4\uc5d4 \uc0ac\uc2e4\uc0c1 \ub300\ub959\uc218\ubcf5\uc774 \ubd88\uac00\ub2a5\ud558\uae30\uc5d0, \uc77c\ubd80 \uc628\uac74\uc6b0\ud30c\ub4e4\uc740 \uc911\ud654\uc778\ubbfc\uacf5\ud654\uad6d\uc5d0 \ud611\ub825\ud558\ub294 \uce5c\uc911\ub178\uc120\uc73c\ub85c \ubc14\ub00c\uc5c8\ub2e4. \uc624\ub298\ub0a0\uc758 \uc911\ud654\ubbfc\uad6d \ub0b4 \uadf9\uc6b0\ub294 \ubbfc\uc8fc\uc9c4\ubcf4\ub2f9\uc758 \ud0c0\uc774\uc644 \ub3c5\ub9bd, \ubd84\ub9ac\uc8fc\uc758 \ub178\uc120\uc5d0 \ubc18\ub300\ud558\uba70, \ub3d9\uc2dc\uc5d0 \uc7a5\uc81c\uc2a4\uc2dd \ub300\ub959 \uc218\ubcf5, \uad6d\uc218\uc8fc\uc758, \ubcf4\uc218\uc8fc\uc758, \uad6d\uac00\uc8fc\uc758 \uc131\ud5a5 \ub4f1 \uacfc\uac70\uc758 \uc815\ucc45\uc744 \uc9c0\uc9c0\ud558\ub294 \ud615\ud0dc\ub85c \ub098\ud0c0\ub098\uba70, \uc815\uce58 \ud65c\ub3d9 \uba74\uc5d0\uc11c\ub294 \uc9c1\uc811 \ud589\ub3d9\uc744 \uac15\uc870\ud558\uace0 \uc788\ub2e4.", "answer": "\uce5c\uc911\ub178\uc120", "sentence": "\uc774\ud6c4, \uc911\ud654\ubbfc\uad6d \ub0b4 \uadf9\uc6b0\ub294 \ub300\ub959\uc218\ubcf5\uc774\ub77c\ub294 \uba85\ubd84\uc744 \uac00\uc9c0\uace0 \ubc18\ub300\ud30c\ub4e4\uc744 \uacf5\uc0b0\uc8fc\uc758\ub85c \ubab0\uc544 \ud559\uc0b4\ud588\uc73c\ub098, \uc7a5\uc81c\uc2a4\uac00 \uc8fd\uace0, \uad6d\uc81c\uc801\uc73c\ub85c \uc911\ud654\ubbfc\uad6d\uc774 \uc544\ub2cc \uc911\ud654\uc778\ubbfc\uacf5\ud654\uad6d\uc744 \ud558\ub098\uc758 \uc911\uad6d\uc73c\ub85c \ubcf4\ub3c4\ub85d \uad6d\uc81c\uc801\uc778 \uc778\uc2dd\uc774 \ubc14\ub00c\uace0, \uc7a5\uc9d5\uad88\uac00 \ucd1d\ud1b5\uc774 \ub418\uace0 \ubbfc\uc8fc\ud654\ub41c \uc774\ud6c4\uc5d4 \uc0ac\uc2e4\uc0c1 \ub300\ub959\uc218\ubcf5\uc774 \ubd88\uac00\ub2a5\ud558\uae30\uc5d0, \uc77c\ubd80 \uc628\uac74\uc6b0\ud30c\ub4e4\uc740 \uc911\ud654\uc778\ubbfc\uacf5\ud654\uad6d\uc5d0 \ud611\ub825\ud558\ub294 \uce5c\uc911\ub178\uc120 \uc73c\ub85c \ubc14\ub00c\uc5c8\ub2e4.", "paragraph_sentence": "\uad6d\uacf5 \ub0b4\uc804\uc5d0\uc11c \uc911\uad6d \uacf5\uc0b0\ub2f9\uc774 \ub300\ub959\uc744 \uc7a5\uc545\ud558\uac8c \ub41c \ud6c4, \uadf9\uc6b0 \uc138\ub825\ub4e4\uc740 \ud0c0\uc774\uc644\uc73c\ub85c \ud53c\uc2e0\ud558\uc5ec, \ud0c0\uc774\uc644 \uc6d0\uc8fc\ubbfc\ub4e4\uacfc \ud55c\uc778\uc758 \ucc28\ubcc4 \uc815\ucc45\uc744 \uc9c0\uc9c0\ud588\ub2e4. \uc7a5\uc81c\uc2a4\uac00 \uc0ac\ub9dd\ud558\uc600\ub358 1975\ub144\uae4c\uc9c0 \uc774\ub978\ubc14 '\uc804\uc2dc\ubbfc\uc8fc\uccb4\uc81c'\ub77c\ub294 \ubc18\ub3c5\uc7ac\uccb4\uc81c\ub97c \uc720\uc9c0\ud558\uc5ec \uc77c\uc778\uc77c\ud30c \ub3c5\uc7ac\uccb4\uc81c\ub97c \uc644\uace0\ud788 \ub2e4\uc84c\uc5c8\ub2e4. \uc774\ud6c4, \uc911\ud654\ubbfc\uad6d \ub0b4 \uadf9\uc6b0\ub294 \ub300\ub959\uc218\ubcf5\uc774\ub77c\ub294 \uba85\ubd84\uc744 \uac00\uc9c0\uace0 \ubc18\ub300\ud30c\ub4e4\uc744 \uacf5\uc0b0\uc8fc\uc758\ub85c \ubab0\uc544 \ud559\uc0b4\ud588\uc73c\ub098, \uc7a5\uc81c\uc2a4\uac00 \uc8fd\uace0, \uad6d\uc81c\uc801\uc73c\ub85c \uc911\ud654\ubbfc\uad6d\uc774 \uc544\ub2cc \uc911\ud654\uc778\ubbfc\uacf5\ud654\uad6d\uc744 \ud558\ub098\uc758 \uc911\uad6d\uc73c\ub85c \ubcf4\ub3c4\ub85d \uad6d\uc81c\uc801\uc778 \uc778\uc2dd\uc774 \ubc14\ub00c\uace0, \uc7a5\uc9d5\uad88\uac00 \ucd1d\ud1b5\uc774 \ub418\uace0 \ubbfc\uc8fc\ud654\ub41c \uc774\ud6c4\uc5d4 \uc0ac\uc2e4\uc0c1 \ub300\ub959\uc218\ubcf5\uc774 \ubd88\uac00\ub2a5\ud558\uae30\uc5d0, \uc77c\ubd80 \uc628\uac74\uc6b0\ud30c\ub4e4\uc740 \uc911\ud654\uc778\ubbfc\uacf5\ud654\uad6d\uc5d0 \ud611\ub825\ud558\ub294 \uce5c\uc911\ub178\uc120 \uc73c\ub85c \ubc14\ub00c\uc5c8\ub2e4. \uc624\ub298\ub0a0\uc758 \uc911\ud654\ubbfc\uad6d \ub0b4 \uadf9\uc6b0\ub294 \ubbfc\uc8fc\uc9c4\ubcf4\ub2f9\uc758 \ud0c0\uc774\uc644 \ub3c5\ub9bd, \ubd84\ub9ac\uc8fc\uc758 \ub178\uc120\uc5d0 \ubc18\ub300\ud558\uba70, \ub3d9\uc2dc\uc5d0 \uc7a5\uc81c\uc2a4\uc2dd \ub300\ub959 \uc218\ubcf5, \uad6d\uc218\uc8fc\uc758, \ubcf4\uc218\uc8fc\uc758, \uad6d\uac00\uc8fc\uc758 \uc131\ud5a5 \ub4f1 \uacfc\uac70\uc758 \uc815\ucc45\uc744 \uc9c0\uc9c0\ud558\ub294 \ud615\ud0dc\ub85c \ub098\ud0c0\ub098\uba70, \uc815\uce58 \ud65c\ub3d9 \uba74\uc5d0\uc11c\ub294 \uc9c1\uc811 \ud589\ub3d9\uc744 \uac15\uc870\ud558\uace0 \uc788\ub2e4.", "paragraph_answer": "\uad6d\uacf5 \ub0b4\uc804\uc5d0\uc11c \uc911\uad6d \uacf5\uc0b0\ub2f9\uc774 \ub300\ub959\uc744 \uc7a5\uc545\ud558\uac8c \ub41c \ud6c4, \uadf9\uc6b0 \uc138\ub825\ub4e4\uc740 \ud0c0\uc774\uc644\uc73c\ub85c \ud53c\uc2e0\ud558\uc5ec, \ud0c0\uc774\uc644 \uc6d0\uc8fc\ubbfc\ub4e4\uacfc \ud55c\uc778\uc758 \ucc28\ubcc4 \uc815\ucc45\uc744 \uc9c0\uc9c0\ud588\ub2e4. \uc7a5\uc81c\uc2a4\uac00 \uc0ac\ub9dd\ud558\uc600\ub358 1975\ub144\uae4c\uc9c0 \uc774\ub978\ubc14 '\uc804\uc2dc\ubbfc\uc8fc\uccb4\uc81c'\ub77c\ub294 \ubc18\ub3c5\uc7ac\uccb4\uc81c\ub97c \uc720\uc9c0\ud558\uc5ec \uc77c\uc778\uc77c\ud30c \ub3c5\uc7ac\uccb4\uc81c\ub97c \uc644\uace0\ud788 \ub2e4\uc84c\uc5c8\ub2e4. \uc774\ud6c4, \uc911\ud654\ubbfc\uad6d \ub0b4 \uadf9\uc6b0\ub294 \ub300\ub959\uc218\ubcf5\uc774\ub77c\ub294 \uba85\ubd84\uc744 \uac00\uc9c0\uace0 \ubc18\ub300\ud30c\ub4e4\uc744 \uacf5\uc0b0\uc8fc\uc758\ub85c \ubab0\uc544 \ud559\uc0b4\ud588\uc73c\ub098, \uc7a5\uc81c\uc2a4\uac00 \uc8fd\uace0, \uad6d\uc81c\uc801\uc73c\ub85c \uc911\ud654\ubbfc\uad6d\uc774 \uc544\ub2cc \uc911\ud654\uc778\ubbfc\uacf5\ud654\uad6d\uc744 \ud558\ub098\uc758 \uc911\uad6d\uc73c\ub85c \ubcf4\ub3c4\ub85d \uad6d\uc81c\uc801\uc778 \uc778\uc2dd\uc774 \ubc14\ub00c\uace0, \uc7a5\uc9d5\uad88\uac00 \ucd1d\ud1b5\uc774 \ub418\uace0 \ubbfc\uc8fc\ud654\ub41c \uc774\ud6c4\uc5d4 \uc0ac\uc2e4\uc0c1 \ub300\ub959\uc218\ubcf5\uc774 \ubd88\uac00\ub2a5\ud558\uae30\uc5d0, \uc77c\ubd80 \uc628\uac74\uc6b0\ud30c\ub4e4\uc740 \uc911\ud654\uc778\ubbfc\uacf5\ud654\uad6d\uc5d0 \ud611\ub825\ud558\ub294 \uce5c\uc911\ub178\uc120 \uc73c\ub85c \ubc14\ub00c\uc5c8\ub2e4. \uc624\ub298\ub0a0\uc758 \uc911\ud654\ubbfc\uad6d \ub0b4 \uadf9\uc6b0\ub294 \ubbfc\uc8fc\uc9c4\ubcf4\ub2f9\uc758 \ud0c0\uc774\uc644 \ub3c5\ub9bd, \ubd84\ub9ac\uc8fc\uc758 \ub178\uc120\uc5d0 \ubc18\ub300\ud558\uba70, \ub3d9\uc2dc\uc5d0 \uc7a5\uc81c\uc2a4\uc2dd \ub300\ub959 \uc218\ubcf5, \uad6d\uc218\uc8fc\uc758, \ubcf4\uc218\uc8fc\uc758, \uad6d\uac00\uc8fc\uc758 \uc131\ud5a5 \ub4f1 \uacfc\uac70\uc758 \uc815\ucc45\uc744 \uc9c0\uc9c0\ud558\ub294 \ud615\ud0dc\ub85c \ub098\ud0c0\ub098\uba70, \uc815\uce58 \ud65c\ub3d9 \uba74\uc5d0\uc11c\ub294 \uc9c1\uc811 \ud589\ub3d9\uc744 \uac15\uc870\ud558\uace0 \uc788\ub2e4.", "sentence_answer": "\uc774\ud6c4, \uc911\ud654\ubbfc\uad6d \ub0b4 \uadf9\uc6b0\ub294 \ub300\ub959\uc218\ubcf5\uc774\ub77c\ub294 \uba85\ubd84\uc744 \uac00\uc9c0\uace0 \ubc18\ub300\ud30c\ub4e4\uc744 \uacf5\uc0b0\uc8fc\uc758\ub85c \ubab0\uc544 \ud559\uc0b4\ud588\uc73c\ub098, \uc7a5\uc81c\uc2a4\uac00 \uc8fd\uace0, \uad6d\uc81c\uc801\uc73c\ub85c \uc911\ud654\ubbfc\uad6d\uc774 \uc544\ub2cc \uc911\ud654\uc778\ubbfc\uacf5\ud654\uad6d\uc744 \ud558\ub098\uc758 \uc911\uad6d\uc73c\ub85c \ubcf4\ub3c4\ub85d \uad6d\uc81c\uc801\uc778 \uc778\uc2dd\uc774 \ubc14\ub00c\uace0, \uc7a5\uc9d5\uad88\uac00 \ucd1d\ud1b5\uc774 \ub418\uace0 \ubbfc\uc8fc\ud654\ub41c \uc774\ud6c4\uc5d4 \uc0ac\uc2e4\uc0c1 \ub300\ub959\uc218\ubcf5\uc774 \ubd88\uac00\ub2a5\ud558\uae30\uc5d0, \uc77c\ubd80 \uc628\uac74\uc6b0\ud30c\ub4e4\uc740 \uc911\ud654\uc778\ubbfc\uacf5\ud654\uad6d\uc5d0 \ud611\ub825\ud558\ub294 \uce5c\uc911\ub178\uc120 \uc73c\ub85c \ubc14\ub00c\uc5c8\ub2e4.", "paragraph_id": "6487890-5-1", "paragraph_question": "question: \ucd1d\ud1b5\uc73c\ub85c \uc7a5\uc9d5\uad88\uac00 \uc784\uba85\ub41c \uc774\ud6c4 \uc77c\ubd80 \uc628\uac74\uc6b0\ud30c\ub4e4\uc740 \uc5b4\ub5a4 \ub178\uc120\uc73c\ub85c \uc785\uc7a5\uc744 \ubc14\uafb8\uc5c8\ub294\uac00?, context: \uad6d\uacf5 \ub0b4\uc804\uc5d0\uc11c \uc911\uad6d \uacf5\uc0b0\ub2f9\uc774 \ub300\ub959\uc744 \uc7a5\uc545\ud558\uac8c \ub41c \ud6c4, \uadf9\uc6b0 \uc138\ub825\ub4e4\uc740 \ud0c0\uc774\uc644\uc73c\ub85c \ud53c\uc2e0\ud558\uc5ec, \ud0c0\uc774\uc644 \uc6d0\uc8fc\ubbfc\ub4e4\uacfc \ud55c\uc778\uc758 \ucc28\ubcc4 \uc815\ucc45\uc744 \uc9c0\uc9c0\ud588\ub2e4. \uc7a5\uc81c\uc2a4\uac00 \uc0ac\ub9dd\ud558\uc600\ub358 1975\ub144\uae4c\uc9c0 \uc774\ub978\ubc14 '\uc804\uc2dc\ubbfc\uc8fc\uccb4\uc81c'\ub77c\ub294 \ubc18\ub3c5\uc7ac\uccb4\uc81c\ub97c \uc720\uc9c0\ud558\uc5ec \uc77c\uc778\uc77c\ud30c \ub3c5\uc7ac\uccb4\uc81c\ub97c \uc644\uace0\ud788 \ub2e4\uc84c\uc5c8\ub2e4. \uc774\ud6c4, \uc911\ud654\ubbfc\uad6d \ub0b4 \uadf9\uc6b0\ub294 \ub300\ub959\uc218\ubcf5\uc774\ub77c\ub294 \uba85\ubd84\uc744 \uac00\uc9c0\uace0 \ubc18\ub300\ud30c\ub4e4\uc744 \uacf5\uc0b0\uc8fc\uc758\ub85c \ubab0\uc544 \ud559\uc0b4\ud588\uc73c\ub098, \uc7a5\uc81c\uc2a4\uac00 \uc8fd\uace0, \uad6d\uc81c\uc801\uc73c\ub85c \uc911\ud654\ubbfc\uad6d\uc774 \uc544\ub2cc \uc911\ud654\uc778\ubbfc\uacf5\ud654\uad6d\uc744 \ud558\ub098\uc758 \uc911\uad6d\uc73c\ub85c \ubcf4\ub3c4\ub85d \uad6d\uc81c\uc801\uc778 \uc778\uc2dd\uc774 \ubc14\ub00c\uace0, \uc7a5\uc9d5\uad88\uac00 \ucd1d\ud1b5\uc774 \ub418\uace0 \ubbfc\uc8fc\ud654\ub41c \uc774\ud6c4\uc5d4 \uc0ac\uc2e4\uc0c1 \ub300\ub959\uc218\ubcf5\uc774 \ubd88\uac00\ub2a5\ud558\uae30\uc5d0, \uc77c\ubd80 \uc628\uac74\uc6b0\ud30c\ub4e4\uc740 \uc911\ud654\uc778\ubbfc\uacf5\ud654\uad6d\uc5d0 \ud611\ub825\ud558\ub294 \uce5c\uc911\ub178\uc120\uc73c\ub85c \ubc14\ub00c\uc5c8\ub2e4. \uc624\ub298\ub0a0\uc758 \uc911\ud654\ubbfc\uad6d \ub0b4 \uadf9\uc6b0\ub294 \ubbfc\uc8fc\uc9c4\ubcf4\ub2f9\uc758 \ud0c0\uc774\uc644 \ub3c5\ub9bd, \ubd84\ub9ac\uc8fc\uc758 \ub178\uc120\uc5d0 \ubc18\ub300\ud558\uba70, \ub3d9\uc2dc\uc5d0 \uc7a5\uc81c\uc2a4\uc2dd \ub300\ub959 \uc218\ubcf5, \uad6d\uc218\uc8fc\uc758, \ubcf4\uc218\uc8fc\uc758, \uad6d\uac00\uc8fc\uc758 \uc131\ud5a5 \ub4f1 \uacfc\uac70\uc758 \uc815\ucc45\uc744 \uc9c0\uc9c0\ud558\ub294 \ud615\ud0dc\ub85c \ub098\ud0c0\ub098\uba70, \uc815\uce58 \ud65c\ub3d9 \uba74\uc5d0\uc11c\ub294 \uc9c1\uc811 \ud589\ub3d9\uc744 \uac15\uc870\ud558\uace0 \uc788\ub2e4."} {"question": "\uacbd\uae30\ub3c4 \uc18c\ubc29 \uacf5\ubb34\uc6d0 1\uc778\ub2f9 \ub2f4\ub2f9\ud558\ub294 \uc8fc\ubbfc\uc758 \uc218\ub294 ?", "paragraph": "\ub610\ud55c \uc778\ub825\uc758 \ucd1d\uc6d0 \uc790\uccb4\uac00 \ubd80\uc871\ud558\ub2e4\ub294 \uc9c0\uc801\ub3c4 \uc788\ub2e4. \ub300\ud55c\ubbfc\uad6d\uc758 \uacbd\ucc30\uacf5\ubb34\uc6d0 1\uc778\ub2f9 \ub2f4\ub2f9\ud558\ub294 \uc8fc\ubbfc\uc758 \uc218\ub294 500\uba85\uc778\ub370 \ubc18\ud574, \uc18c\ubc29\uacf5\ubb34\uc6d0\uc740 1\uba85\ub2f9 1306\uba85\uc744 \ub2f4\ub2f9\ud558\uace0 \uc788\ub2e4. \uacbd\uae30\ub3c4\uac00 \uc18c\ubc29\uacf5\ubb34\uc6d0 1\uba85\ub2f9 1989\uba85\uc744 \ub2f4\ub2f9\ud574 \uac00\uc7a5 \uc22b\uc790\uac00 \ubd80\uc871\ud588\uace0, \uac15\uc6d0\ub3c4\uac00 673\uba85\uc73c\ub85c \uac00\uc7a5 \uc591\ud638\ud588\ub2e4. \uc218\uc6d0\uc2dc\uc758 \uacbd\uc6b0 \uacbd\ucc30\uc740 3\uac1c \uacbd\ucc30\uc11c 1600\uba85\uc758 \uc778\ub825\uc774 \uadfc\ubb34\ud558\ub294\ub370 \ubc18\ud574 \uc18c\ubc29\uc740 1\uac1c \uc18c\ubc29\uc11c 370\uba85\uc774 \uadfc\ubb34\ud558\ub294\ub370 \uadf8\ucce4\ub2e4. \uc18c\ubc29\ub825 \ubcf4\uac15\uc744 \uc704\ud574 \uc9c0\ubc29\uc5d0 \uc608\uc0b0\uc744 \ub0b4\ub824\ubcf4\ub0b4\uc9c0\ub9cc \uadf8 \ub3c8\uc744 \ub2e4\ub978 \uacf3\uc73c\ub85c \uc368\ub3c4 \uc5b4\ucc0c\ud560 \ub3c4\ub9ac\uac00 \uc5c6\uace0, \ud2b9\ubcc4 \uad50\ubd80\uc138\uac00 \ub0b4\ub824\uc624\uba74 \uadf8\ub9cc\ud07c\uc744 \uae30\uc874\uc758 \uc608\uc0b0\uc5d0\uc11c \ub5bc\uc5b4 \uc720\uc6a9\ud558\uae30\ub3c4 \ud55c\ub2e4. \uc774\ub7ec\ud55c \uacb0\uacfc\ub85c 2014\ub144 \ub9d0\uc744 \uae30\uc900\uc73c\ub85c \ud604\uc7a5\uc5d0 \ucd9c\ub3d9\ud558\ub294 \uc18c\ubc29\uacf5\ubb34\uc6d0\uc740 5\ub9cc 493\uba85\uc774 \ud544\uc694\ud55c\ub370 \ubc18\ud574 \uc2e4\uc81c \ud604\uc7a5\uc5d0 \ucd9c\ub3d9 \uac00\ub2a5\ud55c \uc18c\ubc29\uacf5\ubb34\uc6d0\uc740 2\ub9cc 9783\uba85\uc73c\ub85c \uae30\uc900\ubcf4\ub2e4 2\ub9cc 710\uba85\uc774 \ubd80\uc871\ud574 \uacb0\uc6d0\uc728\uc774 41%\uc5d0 \ub2ec\ud588\ub2e4. \uc774\ub7ec\ud55c \uc778\ub825 \ubd80\uc871\uc740 \ud604\uc7a5\uc5d0\uc11c\uc758 \ub300\uc751\uacfc \ud3c9\uc2dc\uc758 \uc7a5\ube44 \uad00\ub9ac\uac00 \ubd80\uc2e4\ud574\uc9c0\ub294 \uc6d0\uc778\uc774 \ub420 \uc218 \uc788\ub2e4\ub294 \uc9c0\uc801\uc774 \uc788\ub2e4.", "answer": "1989\uba85", "sentence": "\uacbd\uae30\ub3c4\uac00 \uc18c\ubc29\uacf5\ubb34\uc6d0 1\uba85\ub2f9 1989\uba85 \uc744 \ub2f4\ub2f9\ud574 \uac00\uc7a5 \uc22b\uc790\uac00 \ubd80\uc871\ud588\uace0, \uac15\uc6d0\ub3c4\uac00 673\uba85\uc73c\ub85c \uac00\uc7a5 \uc591\ud638\ud588\ub2e4.", "paragraph_sentence": "\ub610\ud55c \uc778\ub825\uc758 \ucd1d\uc6d0 \uc790\uccb4\uac00 \ubd80\uc871\ud558\ub2e4\ub294 \uc9c0\uc801\ub3c4 \uc788\ub2e4. \ub300\ud55c\ubbfc\uad6d\uc758 \uacbd\ucc30\uacf5\ubb34\uc6d0 1\uc778\ub2f9 \ub2f4\ub2f9\ud558\ub294 \uc8fc\ubbfc\uc758 \uc218\ub294 500\uba85\uc778\ub370 \ubc18\ud574, \uc18c\ubc29\uacf5\ubb34\uc6d0\uc740 1\uba85\ub2f9 1306\uba85\uc744 \ub2f4\ub2f9\ud558\uace0 \uc788\ub2e4. \uacbd\uae30\ub3c4\uac00 \uc18c\ubc29\uacf5\ubb34\uc6d0 1\uba85\ub2f9 1989\uba85 \uc744 \ub2f4\ub2f9\ud574 \uac00\uc7a5 \uc22b\uc790\uac00 \ubd80\uc871\ud588\uace0, \uac15\uc6d0\ub3c4\uac00 673\uba85\uc73c\ub85c \uac00\uc7a5 \uc591\ud638\ud588\ub2e4. \uc218\uc6d0\uc2dc\uc758 \uacbd\uc6b0 \uacbd\ucc30\uc740 3\uac1c \uacbd\ucc30\uc11c 1600\uba85\uc758 \uc778\ub825\uc774 \uadfc\ubb34\ud558\ub294\ub370 \ubc18\ud574 \uc18c\ubc29\uc740 1\uac1c \uc18c\ubc29\uc11c 370\uba85\uc774 \uadfc\ubb34\ud558\ub294\ub370 \uadf8\ucce4\ub2e4. \uc18c\ubc29\ub825 \ubcf4\uac15\uc744 \uc704\ud574 \uc9c0\ubc29\uc5d0 \uc608\uc0b0\uc744 \ub0b4\ub824\ubcf4\ub0b4\uc9c0\ub9cc \uadf8 \ub3c8\uc744 \ub2e4\ub978 \uacf3\uc73c\ub85c \uc368\ub3c4 \uc5b4\ucc0c\ud560 \ub3c4\ub9ac\uac00 \uc5c6\uace0, \ud2b9\ubcc4 \uad50\ubd80\uc138\uac00 \ub0b4\ub824\uc624\uba74 \uadf8\ub9cc\ud07c\uc744 \uae30\uc874\uc758 \uc608\uc0b0\uc5d0\uc11c \ub5bc\uc5b4 \uc720\uc6a9\ud558\uae30\ub3c4 \ud55c\ub2e4. \uc774\ub7ec\ud55c \uacb0\uacfc\ub85c 2014\ub144 \ub9d0\uc744 \uae30\uc900\uc73c\ub85c \ud604\uc7a5\uc5d0 \ucd9c\ub3d9\ud558\ub294 \uc18c\ubc29\uacf5\ubb34\uc6d0\uc740 5\ub9cc 493\uba85\uc774 \ud544\uc694\ud55c\ub370 \ubc18\ud574 \uc2e4\uc81c \ud604\uc7a5\uc5d0 \ucd9c\ub3d9 \uac00\ub2a5\ud55c \uc18c\ubc29\uacf5\ubb34\uc6d0\uc740 2\ub9cc 9783\uba85\uc73c\ub85c \uae30\uc900\ubcf4\ub2e4 2\ub9cc 710\uba85\uc774 \ubd80\uc871\ud574 \uacb0\uc6d0\uc728\uc774 41%\uc5d0 \ub2ec\ud588\ub2e4. \uc774\ub7ec\ud55c \uc778\ub825 \ubd80\uc871\uc740 \ud604\uc7a5\uc5d0\uc11c\uc758 \ub300\uc751\uacfc \ud3c9\uc2dc\uc758 \uc7a5\ube44 \uad00\ub9ac\uac00 \ubd80\uc2e4\ud574\uc9c0\ub294 \uc6d0\uc778\uc774 \ub420 \uc218 \uc788\ub2e4\ub294 \uc9c0\uc801\uc774 \uc788\ub2e4.", "paragraph_answer": "\ub610\ud55c \uc778\ub825\uc758 \ucd1d\uc6d0 \uc790\uccb4\uac00 \ubd80\uc871\ud558\ub2e4\ub294 \uc9c0\uc801\ub3c4 \uc788\ub2e4. \ub300\ud55c\ubbfc\uad6d\uc758 \uacbd\ucc30\uacf5\ubb34\uc6d0 1\uc778\ub2f9 \ub2f4\ub2f9\ud558\ub294 \uc8fc\ubbfc\uc758 \uc218\ub294 500\uba85\uc778\ub370 \ubc18\ud574, \uc18c\ubc29\uacf5\ubb34\uc6d0\uc740 1\uba85\ub2f9 1306\uba85\uc744 \ub2f4\ub2f9\ud558\uace0 \uc788\ub2e4. \uacbd\uae30\ub3c4\uac00 \uc18c\ubc29\uacf5\ubb34\uc6d0 1\uba85\ub2f9 1989\uba85 \uc744 \ub2f4\ub2f9\ud574 \uac00\uc7a5 \uc22b\uc790\uac00 \ubd80\uc871\ud588\uace0, \uac15\uc6d0\ub3c4\uac00 673\uba85\uc73c\ub85c \uac00\uc7a5 \uc591\ud638\ud588\ub2e4. \uc218\uc6d0\uc2dc\uc758 \uacbd\uc6b0 \uacbd\ucc30\uc740 3\uac1c \uacbd\ucc30\uc11c 1600\uba85\uc758 \uc778\ub825\uc774 \uadfc\ubb34\ud558\ub294\ub370 \ubc18\ud574 \uc18c\ubc29\uc740 1\uac1c \uc18c\ubc29\uc11c 370\uba85\uc774 \uadfc\ubb34\ud558\ub294\ub370 \uadf8\ucce4\ub2e4. \uc18c\ubc29\ub825 \ubcf4\uac15\uc744 \uc704\ud574 \uc9c0\ubc29\uc5d0 \uc608\uc0b0\uc744 \ub0b4\ub824\ubcf4\ub0b4\uc9c0\ub9cc \uadf8 \ub3c8\uc744 \ub2e4\ub978 \uacf3\uc73c\ub85c \uc368\ub3c4 \uc5b4\ucc0c\ud560 \ub3c4\ub9ac\uac00 \uc5c6\uace0, \ud2b9\ubcc4 \uad50\ubd80\uc138\uac00 \ub0b4\ub824\uc624\uba74 \uadf8\ub9cc\ud07c\uc744 \uae30\uc874\uc758 \uc608\uc0b0\uc5d0\uc11c \ub5bc\uc5b4 \uc720\uc6a9\ud558\uae30\ub3c4 \ud55c\ub2e4. \uc774\ub7ec\ud55c \uacb0\uacfc\ub85c 2014\ub144 \ub9d0\uc744 \uae30\uc900\uc73c\ub85c \ud604\uc7a5\uc5d0 \ucd9c\ub3d9\ud558\ub294 \uc18c\ubc29\uacf5\ubb34\uc6d0\uc740 5\ub9cc 493\uba85\uc774 \ud544\uc694\ud55c\ub370 \ubc18\ud574 \uc2e4\uc81c \ud604\uc7a5\uc5d0 \ucd9c\ub3d9 \uac00\ub2a5\ud55c \uc18c\ubc29\uacf5\ubb34\uc6d0\uc740 2\ub9cc 9783\uba85\uc73c\ub85c \uae30\uc900\ubcf4\ub2e4 2\ub9cc 710\uba85\uc774 \ubd80\uc871\ud574 \uacb0\uc6d0\uc728\uc774 41%\uc5d0 \ub2ec\ud588\ub2e4. \uc774\ub7ec\ud55c \uc778\ub825 \ubd80\uc871\uc740 \ud604\uc7a5\uc5d0\uc11c\uc758 \ub300\uc751\uacfc \ud3c9\uc2dc\uc758 \uc7a5\ube44 \uad00\ub9ac\uac00 \ubd80\uc2e4\ud574\uc9c0\ub294 \uc6d0\uc778\uc774 \ub420 \uc218 \uc788\ub2e4\ub294 \uc9c0\uc801\uc774 \uc788\ub2e4.", "sentence_answer": "\uacbd\uae30\ub3c4\uac00 \uc18c\ubc29\uacf5\ubb34\uc6d0 1\uba85\ub2f9 1989\uba85 \uc744 \ub2f4\ub2f9\ud574 \uac00\uc7a5 \uc22b\uc790\uac00 \ubd80\uc871\ud588\uace0, \uac15\uc6d0\ub3c4\uac00 673\uba85\uc73c\ub85c \uac00\uc7a5 \uc591\ud638\ud588\ub2e4.", "paragraph_id": "6486393-6-1", "paragraph_question": "question: \uacbd\uae30\ub3c4 \uc18c\ubc29 \uacf5\ubb34\uc6d0 1\uc778\ub2f9 \ub2f4\ub2f9\ud558\ub294 \uc8fc\ubbfc\uc758 \uc218\ub294 ?, context: \ub610\ud55c \uc778\ub825\uc758 \ucd1d\uc6d0 \uc790\uccb4\uac00 \ubd80\uc871\ud558\ub2e4\ub294 \uc9c0\uc801\ub3c4 \uc788\ub2e4. \ub300\ud55c\ubbfc\uad6d\uc758 \uacbd\ucc30\uacf5\ubb34\uc6d0 1\uc778\ub2f9 \ub2f4\ub2f9\ud558\ub294 \uc8fc\ubbfc\uc758 \uc218\ub294 500\uba85\uc778\ub370 \ubc18\ud574, \uc18c\ubc29\uacf5\ubb34\uc6d0\uc740 1\uba85\ub2f9 1306\uba85\uc744 \ub2f4\ub2f9\ud558\uace0 \uc788\ub2e4. \uacbd\uae30\ub3c4\uac00 \uc18c\ubc29\uacf5\ubb34\uc6d0 1\uba85\ub2f9 1989\uba85\uc744 \ub2f4\ub2f9\ud574 \uac00\uc7a5 \uc22b\uc790\uac00 \ubd80\uc871\ud588\uace0, \uac15\uc6d0\ub3c4\uac00 673\uba85\uc73c\ub85c \uac00\uc7a5 \uc591\ud638\ud588\ub2e4. \uc218\uc6d0\uc2dc\uc758 \uacbd\uc6b0 \uacbd\ucc30\uc740 3\uac1c \uacbd\ucc30\uc11c 1600\uba85\uc758 \uc778\ub825\uc774 \uadfc\ubb34\ud558\ub294\ub370 \ubc18\ud574 \uc18c\ubc29\uc740 1\uac1c \uc18c\ubc29\uc11c 370\uba85\uc774 \uadfc\ubb34\ud558\ub294\ub370 \uadf8\ucce4\ub2e4. \uc18c\ubc29\ub825 \ubcf4\uac15\uc744 \uc704\ud574 \uc9c0\ubc29\uc5d0 \uc608\uc0b0\uc744 \ub0b4\ub824\ubcf4\ub0b4\uc9c0\ub9cc \uadf8 \ub3c8\uc744 \ub2e4\ub978 \uacf3\uc73c\ub85c \uc368\ub3c4 \uc5b4\ucc0c\ud560 \ub3c4\ub9ac\uac00 \uc5c6\uace0, \ud2b9\ubcc4 \uad50\ubd80\uc138\uac00 \ub0b4\ub824\uc624\uba74 \uadf8\ub9cc\ud07c\uc744 \uae30\uc874\uc758 \uc608\uc0b0\uc5d0\uc11c \ub5bc\uc5b4 \uc720\uc6a9\ud558\uae30\ub3c4 \ud55c\ub2e4. \uc774\ub7ec\ud55c \uacb0\uacfc\ub85c 2014\ub144 \ub9d0\uc744 \uae30\uc900\uc73c\ub85c \ud604\uc7a5\uc5d0 \ucd9c\ub3d9\ud558\ub294 \uc18c\ubc29\uacf5\ubb34\uc6d0\uc740 5\ub9cc 493\uba85\uc774 \ud544\uc694\ud55c\ub370 \ubc18\ud574 \uc2e4\uc81c \ud604\uc7a5\uc5d0 \ucd9c\ub3d9 \uac00\ub2a5\ud55c \uc18c\ubc29\uacf5\ubb34\uc6d0\uc740 2\ub9cc 9783\uba85\uc73c\ub85c \uae30\uc900\ubcf4\ub2e4 2\ub9cc 710\uba85\uc774 \ubd80\uc871\ud574 \uacb0\uc6d0\uc728\uc774 41%\uc5d0 \ub2ec\ud588\ub2e4. \uc774\ub7ec\ud55c \uc778\ub825 \ubd80\uc871\uc740 \ud604\uc7a5\uc5d0\uc11c\uc758 \ub300\uc751\uacfc \ud3c9\uc2dc\uc758 \uc7a5\ube44 \uad00\ub9ac\uac00 \ubd80\uc2e4\ud574\uc9c0\ub294 \uc6d0\uc778\uc774 \ub420 \uc218 \uc788\ub2e4\ub294 \uc9c0\uc801\uc774 \uc788\ub2e4."} {"question": "\ucc38\uc804 \uc0ac\uc2e4\uc744 \uc228\uae30\uae30 \uc704\ud574 \uc18c\ub828 \uc870\uc885\uc0ac\ub4e4\uc774 \uad50\uc2e0\uc5d0\uc11c \uc4f0\ub3c4\ub85d \uac15\uc694\ubc1b\uc740 \uc5b8\uc5b4\ub294 \ubb34\uc5c7\uc778\uac00?", "paragraph": "\uacf5\uc2dd\uc801\uc73c\ub85c\ub294 \ud55c\uad6d\uc804\uc5d0 \uc18c\ub828\uad70\uc740 \ucc38\uc804\ud558\uc9c0 \uc54a\uc740 \uac83\uc73c\ub85c \uc54c\ub824\uc838 \uc788\uc73c\ub098, \uc2e4\uc81c\ub85c\ub294 7\ub9cc\uc5ec\uba85\uc774 \ucc38\uc804\ud55c \uac83\uc73c\ub85c \ubc1d\ud600\uc84c\uace0, \uc8fc\ub825\uc740 \uacf5\uad70\uc774\uc5c8\ub2e4. \ucc38\uc804 \uc0ac\uc2e4\uc744 \uc228\uae30\uae30 \uc704\ud574 \uc18c\ub828 \uacf5\uad70\uae30\ub294 \uc911\uacf5\uae30\ucc98\ub7fc \ub3c4\uc0c9\uc744 \ubc14\uafb8\uace0, \uc870\uc885\uc0ac\ub4e4\uc758 \uad50\uc2e0\uc5d0\ub3c4 \ub7ec\uc2dc\uc544\uc5b4 \uc544\ub2cc \uc911\uad6d\uc5b4\ub97c \uc4f0\ub3c4\ub85d \uac15\uc694\ubc1b\uc558\ub2e4. \uaca9\ucd94\ub41c \ube44\ud589\uae30 \uc794\ud574\uac00 \ubbf8\uad70 \uc190\uc5d0 \ub4e4\uc5b4\uac00 \ucc38\uc804 \uc0ac\uc2e4\uc774 \ubc1d\ud600\uc9c0\ub294 \uac83\uc744 \ud53c\ud558\uae30 \uc704\ud574 \uc791\uc804 \uad6c\uc5ed\ub3c4 \uc0c1\ub2f9\ud788 \uc81c\ud55c\ub418\uc5c8\ub2e4. \uc18c\ub828 \uacf5\uad70\uae30\ub4e4\uc740 \ubbf8\uad6d \uacf5\uad70\uae30\uc640 \uc5ec\ub7ec \ucc28\ub840 \uacf5\uc911\uc804\uc744 \ubc8c\uc600\uae30 \ub54c\ubb38\uc5d0 \ubbf8\uad6d\ub3c4 \uc18c\ub828\uc758 \ucc38\uc804 \uc0ac\uc2e4\uc744 \uc798 \uc54c\uace0 \uc788\uc5c8\uc9c0\ub9cc, \ub300\uc678\uc801\uc73c\ub85c\ub294 \uc774\ub97c \ube44\ubc00\uc5d0 \ubd80\ucce4\ub2e4. \uc18c\ub828\uc758 \ucc38\uc804 \uc0ac\uc2e4\uc774 \uc54c\ub824\uc9c0\uba74 \ubbf8\uad6d\ub0b4\uc5d0\uc11c \ud655\uc804\uc758 \uc5ec\ub860\uc774 \ub300\ub450\ub420 \uac83\uc744 \uc6b0\ub824\ud55c \uc544\uc774\uc820\ud558\uc6cc \ub300\ud1b5\ub839\uc758 \uc9c0\uc2dc\uc5d0 \ub530\ub77c \ube44\ubc00\uc5d0 \ubd80\ucce4\ub2e4 \ud55c\ub2e4.", "answer": "\uc911\uad6d\uc5b4", "sentence": "\ucc38\uc804 \uc0ac\uc2e4\uc744 \uc228\uae30\uae30 \uc704\ud574 \uc18c\ub828 \uacf5\uad70\uae30\ub294 \uc911\uacf5\uae30\ucc98\ub7fc \ub3c4\uc0c9\uc744 \ubc14\uafb8\uace0, \uc870\uc885\uc0ac\ub4e4\uc758 \uad50\uc2e0\uc5d0\ub3c4 \ub7ec\uc2dc\uc544\uc5b4 \uc544\ub2cc \uc911\uad6d\uc5b4 \ub97c \uc4f0\ub3c4\ub85d \uac15\uc694\ubc1b\uc558\ub2e4.", "paragraph_sentence": "\uacf5\uc2dd\uc801\uc73c\ub85c\ub294 \ud55c\uad6d\uc804\uc5d0 \uc18c\ub828\uad70\uc740 \ucc38\uc804\ud558\uc9c0 \uc54a\uc740 \uac83\uc73c\ub85c \uc54c\ub824\uc838 \uc788\uc73c\ub098, \uc2e4\uc81c\ub85c\ub294 7\ub9cc\uc5ec\uba85\uc774 \ucc38\uc804\ud55c \uac83\uc73c\ub85c \ubc1d\ud600\uc84c\uace0, \uc8fc\ub825\uc740 \uacf5\uad70\uc774\uc5c8\ub2e4. \ucc38\uc804 \uc0ac\uc2e4\uc744 \uc228\uae30\uae30 \uc704\ud574 \uc18c\ub828 \uacf5\uad70\uae30\ub294 \uc911\uacf5\uae30\ucc98\ub7fc \ub3c4\uc0c9\uc744 \ubc14\uafb8\uace0, \uc870\uc885\uc0ac\ub4e4\uc758 \uad50\uc2e0\uc5d0\ub3c4 \ub7ec\uc2dc\uc544\uc5b4 \uc544\ub2cc \uc911\uad6d\uc5b4 \ub97c \uc4f0\ub3c4\ub85d \uac15\uc694\ubc1b\uc558\ub2e4. \uaca9\ucd94\ub41c \ube44\ud589\uae30 \uc794\ud574\uac00 \ubbf8\uad70 \uc190\uc5d0 \ub4e4\uc5b4\uac00 \ucc38\uc804 \uc0ac\uc2e4\uc774 \ubc1d\ud600\uc9c0\ub294 \uac83\uc744 \ud53c\ud558\uae30 \uc704\ud574 \uc791\uc804 \uad6c\uc5ed\ub3c4 \uc0c1\ub2f9\ud788 \uc81c\ud55c\ub418\uc5c8\ub2e4. \uc18c\ub828 \uacf5\uad70\uae30\ub4e4\uc740 \ubbf8\uad6d \uacf5\uad70\uae30\uc640 \uc5ec\ub7ec \ucc28\ub840 \uacf5\uc911\uc804\uc744 \ubc8c\uc600\uae30 \ub54c\ubb38\uc5d0 \ubbf8\uad6d\ub3c4 \uc18c\ub828\uc758 \ucc38\uc804 \uc0ac\uc2e4\uc744 \uc798 \uc54c\uace0 \uc788\uc5c8\uc9c0\ub9cc, \ub300\uc678\uc801\uc73c\ub85c\ub294 \uc774\ub97c \ube44\ubc00\uc5d0 \ubd80\ucce4\ub2e4. \uc18c\ub828\uc758 \ucc38\uc804 \uc0ac\uc2e4\uc774 \uc54c\ub824\uc9c0\uba74 \ubbf8\uad6d\ub0b4\uc5d0\uc11c \ud655\uc804\uc758 \uc5ec\ub860\uc774 \ub300\ub450\ub420 \uac83\uc744 \uc6b0\ub824\ud55c \uc544\uc774\uc820\ud558\uc6cc \ub300\ud1b5\ub839\uc758 \uc9c0\uc2dc\uc5d0 \ub530\ub77c \ube44\ubc00\uc5d0 \ubd80\ucce4\ub2e4 \ud55c\ub2e4.", "paragraph_answer": "\uacf5\uc2dd\uc801\uc73c\ub85c\ub294 \ud55c\uad6d\uc804\uc5d0 \uc18c\ub828\uad70\uc740 \ucc38\uc804\ud558\uc9c0 \uc54a\uc740 \uac83\uc73c\ub85c \uc54c\ub824\uc838 \uc788\uc73c\ub098, \uc2e4\uc81c\ub85c\ub294 7\ub9cc\uc5ec\uba85\uc774 \ucc38\uc804\ud55c \uac83\uc73c\ub85c \ubc1d\ud600\uc84c\uace0, \uc8fc\ub825\uc740 \uacf5\uad70\uc774\uc5c8\ub2e4. \ucc38\uc804 \uc0ac\uc2e4\uc744 \uc228\uae30\uae30 \uc704\ud574 \uc18c\ub828 \uacf5\uad70\uae30\ub294 \uc911\uacf5\uae30\ucc98\ub7fc \ub3c4\uc0c9\uc744 \ubc14\uafb8\uace0, \uc870\uc885\uc0ac\ub4e4\uc758 \uad50\uc2e0\uc5d0\ub3c4 \ub7ec\uc2dc\uc544\uc5b4 \uc544\ub2cc \uc911\uad6d\uc5b4 \ub97c \uc4f0\ub3c4\ub85d \uac15\uc694\ubc1b\uc558\ub2e4. \uaca9\ucd94\ub41c \ube44\ud589\uae30 \uc794\ud574\uac00 \ubbf8\uad70 \uc190\uc5d0 \ub4e4\uc5b4\uac00 \ucc38\uc804 \uc0ac\uc2e4\uc774 \ubc1d\ud600\uc9c0\ub294 \uac83\uc744 \ud53c\ud558\uae30 \uc704\ud574 \uc791\uc804 \uad6c\uc5ed\ub3c4 \uc0c1\ub2f9\ud788 \uc81c\ud55c\ub418\uc5c8\ub2e4. \uc18c\ub828 \uacf5\uad70\uae30\ub4e4\uc740 \ubbf8\uad6d \uacf5\uad70\uae30\uc640 \uc5ec\ub7ec \ucc28\ub840 \uacf5\uc911\uc804\uc744 \ubc8c\uc600\uae30 \ub54c\ubb38\uc5d0 \ubbf8\uad6d\ub3c4 \uc18c\ub828\uc758 \ucc38\uc804 \uc0ac\uc2e4\uc744 \uc798 \uc54c\uace0 \uc788\uc5c8\uc9c0\ub9cc, \ub300\uc678\uc801\uc73c\ub85c\ub294 \uc774\ub97c \ube44\ubc00\uc5d0 \ubd80\ucce4\ub2e4. \uc18c\ub828\uc758 \ucc38\uc804 \uc0ac\uc2e4\uc774 \uc54c\ub824\uc9c0\uba74 \ubbf8\uad6d\ub0b4\uc5d0\uc11c \ud655\uc804\uc758 \uc5ec\ub860\uc774 \ub300\ub450\ub420 \uac83\uc744 \uc6b0\ub824\ud55c \uc544\uc774\uc820\ud558\uc6cc \ub300\ud1b5\ub839\uc758 \uc9c0\uc2dc\uc5d0 \ub530\ub77c \ube44\ubc00\uc5d0 \ubd80\ucce4\ub2e4 \ud55c\ub2e4.", "sentence_answer": "\ucc38\uc804 \uc0ac\uc2e4\uc744 \uc228\uae30\uae30 \uc704\ud574 \uc18c\ub828 \uacf5\uad70\uae30\ub294 \uc911\uacf5\uae30\ucc98\ub7fc \ub3c4\uc0c9\uc744 \ubc14\uafb8\uace0, \uc870\uc885\uc0ac\ub4e4\uc758 \uad50\uc2e0\uc5d0\ub3c4 \ub7ec\uc2dc\uc544\uc5b4 \uc544\ub2cc \uc911\uad6d\uc5b4 \ub97c \uc4f0\ub3c4\ub85d \uac15\uc694\ubc1b\uc558\ub2e4.", "paragraph_id": "6569867-46-1", "paragraph_question": "question: \ucc38\uc804 \uc0ac\uc2e4\uc744 \uc228\uae30\uae30 \uc704\ud574 \uc18c\ub828 \uc870\uc885\uc0ac\ub4e4\uc774 \uad50\uc2e0\uc5d0\uc11c \uc4f0\ub3c4\ub85d \uac15\uc694\ubc1b\uc740 \uc5b8\uc5b4\ub294 \ubb34\uc5c7\uc778\uac00?, context: \uacf5\uc2dd\uc801\uc73c\ub85c\ub294 \ud55c\uad6d\uc804\uc5d0 \uc18c\ub828\uad70\uc740 \ucc38\uc804\ud558\uc9c0 \uc54a\uc740 \uac83\uc73c\ub85c \uc54c\ub824\uc838 \uc788\uc73c\ub098, \uc2e4\uc81c\ub85c\ub294 7\ub9cc\uc5ec\uba85\uc774 \ucc38\uc804\ud55c \uac83\uc73c\ub85c \ubc1d\ud600\uc84c\uace0, \uc8fc\ub825\uc740 \uacf5\uad70\uc774\uc5c8\ub2e4. \ucc38\uc804 \uc0ac\uc2e4\uc744 \uc228\uae30\uae30 \uc704\ud574 \uc18c\ub828 \uacf5\uad70\uae30\ub294 \uc911\uacf5\uae30\ucc98\ub7fc \ub3c4\uc0c9\uc744 \ubc14\uafb8\uace0, \uc870\uc885\uc0ac\ub4e4\uc758 \uad50\uc2e0\uc5d0\ub3c4 \ub7ec\uc2dc\uc544\uc5b4 \uc544\ub2cc \uc911\uad6d\uc5b4\ub97c \uc4f0\ub3c4\ub85d \uac15\uc694\ubc1b\uc558\ub2e4. \uaca9\ucd94\ub41c \ube44\ud589\uae30 \uc794\ud574\uac00 \ubbf8\uad70 \uc190\uc5d0 \ub4e4\uc5b4\uac00 \ucc38\uc804 \uc0ac\uc2e4\uc774 \ubc1d\ud600\uc9c0\ub294 \uac83\uc744 \ud53c\ud558\uae30 \uc704\ud574 \uc791\uc804 \uad6c\uc5ed\ub3c4 \uc0c1\ub2f9\ud788 \uc81c\ud55c\ub418\uc5c8\ub2e4. \uc18c\ub828 \uacf5\uad70\uae30\ub4e4\uc740 \ubbf8\uad6d \uacf5\uad70\uae30\uc640 \uc5ec\ub7ec \ucc28\ub840 \uacf5\uc911\uc804\uc744 \ubc8c\uc600\uae30 \ub54c\ubb38\uc5d0 \ubbf8\uad6d\ub3c4 \uc18c\ub828\uc758 \ucc38\uc804 \uc0ac\uc2e4\uc744 \uc798 \uc54c\uace0 \uc788\uc5c8\uc9c0\ub9cc, \ub300\uc678\uc801\uc73c\ub85c\ub294 \uc774\ub97c \ube44\ubc00\uc5d0 \ubd80\ucce4\ub2e4. \uc18c\ub828\uc758 \ucc38\uc804 \uc0ac\uc2e4\uc774 \uc54c\ub824\uc9c0\uba74 \ubbf8\uad6d\ub0b4\uc5d0\uc11c \ud655\uc804\uc758 \uc5ec\ub860\uc774 \ub300\ub450\ub420 \uac83\uc744 \uc6b0\ub824\ud55c \uc544\uc774\uc820\ud558\uc6cc \ub300\ud1b5\ub839\uc758 \uc9c0\uc2dc\uc5d0 \ub530\ub77c \ube44\ubc00\uc5d0 \ubd80\ucce4\ub2e4 \ud55c\ub2e4."} {"question": "\uc774\ud669\uc758 \ud638\ub294?", "paragraph": "\ub458\uc9f8, \ud1f4\uacc4 \uc774\ud669\uc774\ub77c\ub294 \uc120\ubc30\uac00 \uc788\uc5c8\ub2e4\ub294 \uc810\uc774\ub2e4. \uc728\uace1\uc740 23\uc138 \ub54c \ub3c4\uc0b0\uc11c\uc6d0\uc73c\ub85c \uc9c1\uc811 \ud1f4\uacc4\ub97c \ubc29\ubb38\ud588\uace0, \uadf8 \ub4a4\ub85c\ub3c4 \ud3b8\uc9c0\ub97c \uc8fc\uace0\ubc1b\uc73c\uba70 \ud559\ubb38\uacfc \uac1c\uc778\uc801 \uc2e0\uc0c1\uc5d0 \uad00\ud574 \uc758\uacac\uc744 \ub098\ub234\ub2e4. \uace0\ud5a5\uc73c\ub85c \ub3cc\uc544\uac00\ub824\ub294 \ud1f4\uacc4\ub97c \uadf9\uad6c \ub9cc\ub958\ud588\uace0, \ud1f4\uacc4\uac00 \uc8fd\uace0 \ub098\uc11c\ub294 \ub9cc\uc2dc(\u8f13\u8a69)\ub97c \uc9c0\uc5b4 \uc560\ub3c4\ud588\ub2e4. \ud1f4\uacc4\uc640 \ud568\uaed8 \uc870\uc120 \uc131\ub9ac\ud559\uc758 \ub450 \uc904\uae30\ub97c \ud615\uc131\ud588\ub2e4\ub294 \ud3c9\uac00\ub97c \ubc1b\ub294\ub2e4. \ud6c4\ub300 \ud559\uc790\ub4e4\uc774 \ud559\ud30c\uc640 \ubd95\ub2f9\uc758 \ud615\uc131\uc5d0 \ub530\ub77c \ud1f4\uacc4\uc640 \uc728\uace1 \uc0ac\uc774\uc5d0 \uac74\ub110 \uc218 \uc5c6\ub294 \uac15\uc774 \ud750\ub974\ub294 \uac83\ucc98\ub7fc \ub9cc\ub4e4\uace0, \uadf8\uc5d0 \ub530\ub77c \ud559\ubb38\uc801\uc73c\ub85c\ub098 \uc815\uce58\uc801\uc73c\ub85c\ub098 \uc801\ub300\uc801\uc778 \uad00\uacc4\ucc98\ub7fc \ub418\uc5b4\ubc84\ub838\uc9c0\ub9cc, \ub2f9\ub300\uc5d0\ub294 \uc11c\ub85c\ub97c \uc778\uc815\ud55c \uc88b\uc740 \uc120\ud6c4\ubc30\uc600\ub2e4. \uc728\uace1\uc740 \uadf8\ub7f0 \uc120\ubc30\uc640\uc758 \ud3b8\uc9c0 \uad50\ub958 \ub4f1\uc744 \ud1b5\ud574 \ud559\ubb38\uc801 \uc5f0\ub9c8\ub97c \ud560 \uc218 \uc788\uc5c8\uace0, \uc120\ubc30\uac00 \uc774\ub8e8\uc5b4\ub193\uc740 \uc131\uacfc\ub97c \ub514\ub524\ub3cc\ub85c \ud574\uc11c \uc790\uc2e0\uc758 \ud559\ubb38\uc801 \uc138\uacc4\ub97c \uad6c\ucd95\ud560 \uc218 \uc788\uc5c8\ub2e4.", "answer": "\ud1f4\uacc4", "sentence": "\ub458\uc9f8, \ud1f4\uacc4 \uc774\ud669\uc774\ub77c\ub294 \uc120\ubc30\uac00 \uc788\uc5c8\ub2e4\ub294 \uc810\uc774\ub2e4.", "paragraph_sentence": " \ub458\uc9f8, \ud1f4\uacc4 \uc774\ud669\uc774\ub77c\ub294 \uc120\ubc30\uac00 \uc788\uc5c8\ub2e4\ub294 \uc810\uc774\ub2e4. \uc728\uace1\uc740 23\uc138 \ub54c \ub3c4\uc0b0\uc11c\uc6d0\uc73c\ub85c \uc9c1\uc811 \ud1f4\uacc4\ub97c \ubc29\ubb38\ud588\uace0, \uadf8 \ub4a4\ub85c\ub3c4 \ud3b8\uc9c0\ub97c \uc8fc\uace0\ubc1b\uc73c\uba70 \ud559\ubb38\uacfc \uac1c\uc778\uc801 \uc2e0\uc0c1\uc5d0 \uad00\ud574 \uc758\uacac\uc744 \ub098\ub234\ub2e4. \uace0\ud5a5\uc73c\ub85c \ub3cc\uc544\uac00\ub824\ub294 \ud1f4\uacc4\ub97c \uadf9\uad6c \ub9cc\ub958\ud588\uace0, \ud1f4\uacc4\uac00 \uc8fd\uace0 \ub098\uc11c\ub294 \ub9cc\uc2dc(\u8f13\u8a69)\ub97c \uc9c0\uc5b4 \uc560\ub3c4\ud588\ub2e4. \ud1f4\uacc4\uc640 \ud568\uaed8 \uc870\uc120 \uc131\ub9ac\ud559\uc758 \ub450 \uc904\uae30\ub97c \ud615\uc131\ud588\ub2e4\ub294 \ud3c9\uac00\ub97c \ubc1b\ub294\ub2e4. \ud6c4\ub300 \ud559\uc790\ub4e4\uc774 \ud559\ud30c\uc640 \ubd95\ub2f9\uc758 \ud615\uc131\uc5d0 \ub530\ub77c \ud1f4\uacc4\uc640 \uc728\uace1 \uc0ac\uc774\uc5d0 \uac74\ub110 \uc218 \uc5c6\ub294 \uac15\uc774 \ud750\ub974\ub294 \uac83\ucc98\ub7fc \ub9cc\ub4e4\uace0, \uadf8\uc5d0 \ub530\ub77c \ud559\ubb38\uc801\uc73c\ub85c\ub098 \uc815\uce58\uc801\uc73c\ub85c\ub098 \uc801\ub300\uc801\uc778 \uad00\uacc4\ucc98\ub7fc \ub418\uc5b4\ubc84\ub838\uc9c0\ub9cc, \ub2f9\ub300\uc5d0\ub294 \uc11c\ub85c\ub97c \uc778\uc815\ud55c \uc88b\uc740 \uc120\ud6c4\ubc30\uc600\ub2e4. \uc728\uace1\uc740 \uadf8\ub7f0 \uc120\ubc30\uc640\uc758 \ud3b8\uc9c0 \uad50\ub958 \ub4f1\uc744 \ud1b5\ud574 \ud559\ubb38\uc801 \uc5f0\ub9c8\ub97c \ud560 \uc218 \uc788\uc5c8\uace0, \uc120\ubc30\uac00 \uc774\ub8e8\uc5b4\ub193\uc740 \uc131\uacfc\ub97c \ub514\ub524\ub3cc\ub85c \ud574\uc11c \uc790\uc2e0\uc758 \ud559\ubb38\uc801 \uc138\uacc4\ub97c \uad6c\ucd95\ud560 \uc218 \uc788\uc5c8\ub2e4.", "paragraph_answer": "\ub458\uc9f8, \ud1f4\uacc4 \uc774\ud669\uc774\ub77c\ub294 \uc120\ubc30\uac00 \uc788\uc5c8\ub2e4\ub294 \uc810\uc774\ub2e4. \uc728\uace1\uc740 23\uc138 \ub54c \ub3c4\uc0b0\uc11c\uc6d0\uc73c\ub85c \uc9c1\uc811 \ud1f4\uacc4\ub97c \ubc29\ubb38\ud588\uace0, \uadf8 \ub4a4\ub85c\ub3c4 \ud3b8\uc9c0\ub97c \uc8fc\uace0\ubc1b\uc73c\uba70 \ud559\ubb38\uacfc \uac1c\uc778\uc801 \uc2e0\uc0c1\uc5d0 \uad00\ud574 \uc758\uacac\uc744 \ub098\ub234\ub2e4. \uace0\ud5a5\uc73c\ub85c \ub3cc\uc544\uac00\ub824\ub294 \ud1f4\uacc4\ub97c \uadf9\uad6c \ub9cc\ub958\ud588\uace0, \ud1f4\uacc4\uac00 \uc8fd\uace0 \ub098\uc11c\ub294 \ub9cc\uc2dc(\u8f13\u8a69)\ub97c \uc9c0\uc5b4 \uc560\ub3c4\ud588\ub2e4. \ud1f4\uacc4\uc640 \ud568\uaed8 \uc870\uc120 \uc131\ub9ac\ud559\uc758 \ub450 \uc904\uae30\ub97c \ud615\uc131\ud588\ub2e4\ub294 \ud3c9\uac00\ub97c \ubc1b\ub294\ub2e4. \ud6c4\ub300 \ud559\uc790\ub4e4\uc774 \ud559\ud30c\uc640 \ubd95\ub2f9\uc758 \ud615\uc131\uc5d0 \ub530\ub77c \ud1f4\uacc4\uc640 \uc728\uace1 \uc0ac\uc774\uc5d0 \uac74\ub110 \uc218 \uc5c6\ub294 \uac15\uc774 \ud750\ub974\ub294 \uac83\ucc98\ub7fc \ub9cc\ub4e4\uace0, \uadf8\uc5d0 \ub530\ub77c \ud559\ubb38\uc801\uc73c\ub85c\ub098 \uc815\uce58\uc801\uc73c\ub85c\ub098 \uc801\ub300\uc801\uc778 \uad00\uacc4\ucc98\ub7fc \ub418\uc5b4\ubc84\ub838\uc9c0\ub9cc, \ub2f9\ub300\uc5d0\ub294 \uc11c\ub85c\ub97c \uc778\uc815\ud55c \uc88b\uc740 \uc120\ud6c4\ubc30\uc600\ub2e4. \uc728\uace1\uc740 \uadf8\ub7f0 \uc120\ubc30\uc640\uc758 \ud3b8\uc9c0 \uad50\ub958 \ub4f1\uc744 \ud1b5\ud574 \ud559\ubb38\uc801 \uc5f0\ub9c8\ub97c \ud560 \uc218 \uc788\uc5c8\uace0, \uc120\ubc30\uac00 \uc774\ub8e8\uc5b4\ub193\uc740 \uc131\uacfc\ub97c \ub514\ub524\ub3cc\ub85c \ud574\uc11c \uc790\uc2e0\uc758 \ud559\ubb38\uc801 \uc138\uacc4\ub97c \uad6c\ucd95\ud560 \uc218 \uc788\uc5c8\ub2e4.", "sentence_answer": "\ub458\uc9f8, \ud1f4\uacc4 \uc774\ud669\uc774\ub77c\ub294 \uc120\ubc30\uac00 \uc788\uc5c8\ub2e4\ub294 \uc810\uc774\ub2e4.", "paragraph_id": "6545935-15-1", "paragraph_question": "question: \uc774\ud669\uc758 \ud638\ub294?, context: \ub458\uc9f8, \ud1f4\uacc4 \uc774\ud669\uc774\ub77c\ub294 \uc120\ubc30\uac00 \uc788\uc5c8\ub2e4\ub294 \uc810\uc774\ub2e4. \uc728\uace1\uc740 23\uc138 \ub54c \ub3c4\uc0b0\uc11c\uc6d0\uc73c\ub85c \uc9c1\uc811 \ud1f4\uacc4\ub97c \ubc29\ubb38\ud588\uace0, \uadf8 \ub4a4\ub85c\ub3c4 \ud3b8\uc9c0\ub97c \uc8fc\uace0\ubc1b\uc73c\uba70 \ud559\ubb38\uacfc \uac1c\uc778\uc801 \uc2e0\uc0c1\uc5d0 \uad00\ud574 \uc758\uacac\uc744 \ub098\ub234\ub2e4. \uace0\ud5a5\uc73c\ub85c \ub3cc\uc544\uac00\ub824\ub294 \ud1f4\uacc4\ub97c \uadf9\uad6c \ub9cc\ub958\ud588\uace0, \ud1f4\uacc4\uac00 \uc8fd\uace0 \ub098\uc11c\ub294 \ub9cc\uc2dc(\u8f13\u8a69)\ub97c \uc9c0\uc5b4 \uc560\ub3c4\ud588\ub2e4. \ud1f4\uacc4\uc640 \ud568\uaed8 \uc870\uc120 \uc131\ub9ac\ud559\uc758 \ub450 \uc904\uae30\ub97c \ud615\uc131\ud588\ub2e4\ub294 \ud3c9\uac00\ub97c \ubc1b\ub294\ub2e4. \ud6c4\ub300 \ud559\uc790\ub4e4\uc774 \ud559\ud30c\uc640 \ubd95\ub2f9\uc758 \ud615\uc131\uc5d0 \ub530\ub77c \ud1f4\uacc4\uc640 \uc728\uace1 \uc0ac\uc774\uc5d0 \uac74\ub110 \uc218 \uc5c6\ub294 \uac15\uc774 \ud750\ub974\ub294 \uac83\ucc98\ub7fc \ub9cc\ub4e4\uace0, \uadf8\uc5d0 \ub530\ub77c \ud559\ubb38\uc801\uc73c\ub85c\ub098 \uc815\uce58\uc801\uc73c\ub85c\ub098 \uc801\ub300\uc801\uc778 \uad00\uacc4\ucc98\ub7fc \ub418\uc5b4\ubc84\ub838\uc9c0\ub9cc, \ub2f9\ub300\uc5d0\ub294 \uc11c\ub85c\ub97c \uc778\uc815\ud55c \uc88b\uc740 \uc120\ud6c4\ubc30\uc600\ub2e4. \uc728\uace1\uc740 \uadf8\ub7f0 \uc120\ubc30\uc640\uc758 \ud3b8\uc9c0 \uad50\ub958 \ub4f1\uc744 \ud1b5\ud574 \ud559\ubb38\uc801 \uc5f0\ub9c8\ub97c \ud560 \uc218 \uc788\uc5c8\uace0, \uc120\ubc30\uac00 \uc774\ub8e8\uc5b4\ub193\uc740 \uc131\uacfc\ub97c \ub514\ub524\ub3cc\ub85c \ud574\uc11c \uc790\uc2e0\uc758 \ud559\ubb38\uc801 \uc138\uacc4\ub97c \uad6c\ucd95\ud560 \uc218 \uc788\uc5c8\ub2e4."} {"question": "\ud06c\ub77c\ucfe0\ud504-\ub974\ubd80\ud06c \uac04 \ucca0\ub3c4\uac00 \uac74\uc124\ub41c \uc5f0\ub3c4\ub294?", "paragraph": "1853\ub144 \ud06c\ub9bc \uc804\uc7c1\uc774 \ubc1c\ubc1c\ud558\uc790 \uaebc\uc838\uac00\ub358 \ud3f4\ub780\ub4dc \ubbfc\uc871\uc758 \ud76c\ub9dd\uc774 \ub2e4\uc2dc \uc0b4\uc544\ub098\ub294 \uac83 \uac19\uc558\ub2e4. \uc601\uad6d\uacfc \ud504\ub791\uc2a4, \ud22c\ub974\ud06c\uc640 \ud53c\uc5d0\ubaac\ud14c(Piemonte)\uac00 \ub7ec\uc2dc\uc544\ub97c \uc0c1\ub300\ub85c \uc2f8\uc6b0\uace0 \uc788\uc5c8\uae30 \ub54c\ubb38\uc5d0 \ud3f4\ub780\ub4dc\uc758 \ub9dd\uba85\ub2e8\uccb4\ub4e4\uc740 \ud3f4\ub780\ub4dc \uc758\uc6a9\uad70\uc744 \uc870\uc9c1\ud558\uc5ec \ub300 \ub7ec\uc2dc\uc544 \uc804\uc120\uc5d0 \ud22c\uc785\ud558\ub824\uace0 \uacc4\ud68d\ud588\uc73c\ub098 \ub9dd\uba85\ub2e8\uccb4 \uc9c0\ub3c4\uc790\ub4e4\uc758 \ub0b4\ubd84 \ub54c\ubb38\uc5d0 \uad70\ub300\ub294 \uc870\uc9c1\ub418\uc9c0 \ubabb\ud588\ub2e4. 1850\ub144\ub300 \ucd08\uc5d0\ub294 \uc790\uc5f0\uc801\uc778 \uc7ac\ud574\uae4c\uc9c0 \uacb9\uce58\uace0 \ud749\ub144\uacfc \ud64d\uc218, \uc804\uc5fc\ubcd1\uc73c\ub85c \uac08\ub9ac\uc2dc\uc544 \ub18d\ubbfc\ub4e4\uc758 \uc0dd\ud65c\uc774 \ub354\uc6b1 \ube44\ucc38\ud574\uc84c\ub2e4. \uadf8 \uacb0\uacfc \ub18d\ucd0c\uc5d0\uc11c\uc758 \uc0ac\ub9dd\ub960\uc774 \uc99d\uac00\ud558\uace0 \uc774\uc5d0 \ub530\ub77c \uc778\uad6c\uac00 \uac10\uc18c\ub418\uc5c8\ub2e4. \uc774 \uc9c0\ubc29\uc5d0 \uc0b0\uc5c5\uc774 \uc77c\uc5b4\ub0a0 \uc9d5\uc870\ub294 \uc544\uc9c1 \ub098\ud0c0\ub098\uace0 \uc788\uc9c0 \uc54a\uace0 \uc788\uc5c8\ub2e4. 1856\ub144\uc5d0 \uc774\ub974\ub7ec\uc11c\uc57c \ube44\ub85c\uc18c \ud06c\ub77c\ucfe0\ud504(KrakEw)-\ub974\ubd80\ud504(LwEw)\uac04\uc5d0 \ucca0\ub3c4\uac00 \ubd80\uc124\ub418\uae30 \uc2dc\uc791\ud558\uc600\ub2e4.", "answer": "1856\ub144", "sentence": "1856\ub144 \uc5d0 \uc774\ub974\ub7ec\uc11c\uc57c \ube44\ub85c\uc18c \ud06c\ub77c\ucfe0\ud504(KrakEw)-\ub974\ubd80\ud504(LwEw)\uac04\uc5d0 \ucca0\ub3c4\uac00 \ubd80\uc124\ub418\uae30 \uc2dc\uc791\ud558\uc600\ub2e4.", "paragraph_sentence": "1853\ub144 \ud06c\ub9bc \uc804\uc7c1\uc774 \ubc1c\ubc1c\ud558\uc790 \uaebc\uc838\uac00\ub358 \ud3f4\ub780\ub4dc \ubbfc\uc871\uc758 \ud76c\ub9dd\uc774 \ub2e4\uc2dc \uc0b4\uc544\ub098\ub294 \uac83 \uac19\uc558\ub2e4. \uc601\uad6d\uacfc \ud504\ub791\uc2a4, \ud22c\ub974\ud06c\uc640 \ud53c\uc5d0\ubaac\ud14c(Piemonte)\uac00 \ub7ec\uc2dc\uc544\ub97c \uc0c1\ub300\ub85c \uc2f8\uc6b0\uace0 \uc788\uc5c8\uae30 \ub54c\ubb38\uc5d0 \ud3f4\ub780\ub4dc\uc758 \ub9dd\uba85\ub2e8\uccb4\ub4e4\uc740 \ud3f4\ub780\ub4dc \uc758\uc6a9\uad70\uc744 \uc870\uc9c1\ud558\uc5ec \ub300 \ub7ec\uc2dc\uc544 \uc804\uc120\uc5d0 \ud22c\uc785\ud558\ub824\uace0 \uacc4\ud68d\ud588\uc73c\ub098 \ub9dd\uba85\ub2e8\uccb4 \uc9c0\ub3c4\uc790\ub4e4\uc758 \ub0b4\ubd84 \ub54c\ubb38\uc5d0 \uad70\ub300\ub294 \uc870\uc9c1\ub418\uc9c0 \ubabb\ud588\ub2e4. 1850\ub144\ub300 \ucd08\uc5d0\ub294 \uc790\uc5f0\uc801\uc778 \uc7ac\ud574\uae4c\uc9c0 \uacb9\uce58\uace0 \ud749\ub144\uacfc \ud64d\uc218, \uc804\uc5fc\ubcd1\uc73c\ub85c \uac08\ub9ac\uc2dc\uc544 \ub18d\ubbfc\ub4e4\uc758 \uc0dd\ud65c\uc774 \ub354\uc6b1 \ube44\ucc38\ud574\uc84c\ub2e4. \uadf8 \uacb0\uacfc \ub18d\ucd0c\uc5d0\uc11c\uc758 \uc0ac\ub9dd\ub960\uc774 \uc99d\uac00\ud558\uace0 \uc774\uc5d0 \ub530\ub77c \uc778\uad6c\uac00 \uac10\uc18c\ub418\uc5c8\ub2e4. \uc774 \uc9c0\ubc29\uc5d0 \uc0b0\uc5c5\uc774 \uc77c\uc5b4\ub0a0 \uc9d5\uc870\ub294 \uc544\uc9c1 \ub098\ud0c0\ub098\uace0 \uc788\uc9c0 \uc54a\uace0 \uc788\uc5c8\ub2e4. 1856\ub144 \uc5d0 \uc774\ub974\ub7ec\uc11c\uc57c \ube44\ub85c\uc18c \ud06c\ub77c\ucfe0\ud504(KrakEw)-\ub974\ubd80\ud504(LwEw)\uac04\uc5d0 \ucca0\ub3c4\uac00 \ubd80\uc124\ub418\uae30 \uc2dc\uc791\ud558\uc600\ub2e4. ", "paragraph_answer": "1853\ub144 \ud06c\ub9bc \uc804\uc7c1\uc774 \ubc1c\ubc1c\ud558\uc790 \uaebc\uc838\uac00\ub358 \ud3f4\ub780\ub4dc \ubbfc\uc871\uc758 \ud76c\ub9dd\uc774 \ub2e4\uc2dc \uc0b4\uc544\ub098\ub294 \uac83 \uac19\uc558\ub2e4. \uc601\uad6d\uacfc \ud504\ub791\uc2a4, \ud22c\ub974\ud06c\uc640 \ud53c\uc5d0\ubaac\ud14c(Piemonte)\uac00 \ub7ec\uc2dc\uc544\ub97c \uc0c1\ub300\ub85c \uc2f8\uc6b0\uace0 \uc788\uc5c8\uae30 \ub54c\ubb38\uc5d0 \ud3f4\ub780\ub4dc\uc758 \ub9dd\uba85\ub2e8\uccb4\ub4e4\uc740 \ud3f4\ub780\ub4dc \uc758\uc6a9\uad70\uc744 \uc870\uc9c1\ud558\uc5ec \ub300 \ub7ec\uc2dc\uc544 \uc804\uc120\uc5d0 \ud22c\uc785\ud558\ub824\uace0 \uacc4\ud68d\ud588\uc73c\ub098 \ub9dd\uba85\ub2e8\uccb4 \uc9c0\ub3c4\uc790\ub4e4\uc758 \ub0b4\ubd84 \ub54c\ubb38\uc5d0 \uad70\ub300\ub294 \uc870\uc9c1\ub418\uc9c0 \ubabb\ud588\ub2e4. 1850\ub144\ub300 \ucd08\uc5d0\ub294 \uc790\uc5f0\uc801\uc778 \uc7ac\ud574\uae4c\uc9c0 \uacb9\uce58\uace0 \ud749\ub144\uacfc \ud64d\uc218, \uc804\uc5fc\ubcd1\uc73c\ub85c \uac08\ub9ac\uc2dc\uc544 \ub18d\ubbfc\ub4e4\uc758 \uc0dd\ud65c\uc774 \ub354\uc6b1 \ube44\ucc38\ud574\uc84c\ub2e4. \uadf8 \uacb0\uacfc \ub18d\ucd0c\uc5d0\uc11c\uc758 \uc0ac\ub9dd\ub960\uc774 \uc99d\uac00\ud558\uace0 \uc774\uc5d0 \ub530\ub77c \uc778\uad6c\uac00 \uac10\uc18c\ub418\uc5c8\ub2e4. \uc774 \uc9c0\ubc29\uc5d0 \uc0b0\uc5c5\uc774 \uc77c\uc5b4\ub0a0 \uc9d5\uc870\ub294 \uc544\uc9c1 \ub098\ud0c0\ub098\uace0 \uc788\uc9c0 \uc54a\uace0 \uc788\uc5c8\ub2e4. 1856\ub144 \uc5d0 \uc774\ub974\ub7ec\uc11c\uc57c \ube44\ub85c\uc18c \ud06c\ub77c\ucfe0\ud504(KrakEw)-\ub974\ubd80\ud504(LwEw)\uac04\uc5d0 \ucca0\ub3c4\uac00 \ubd80\uc124\ub418\uae30 \uc2dc\uc791\ud558\uc600\ub2e4.", "sentence_answer": " 1856\ub144 \uc5d0 \uc774\ub974\ub7ec\uc11c\uc57c \ube44\ub85c\uc18c \ud06c\ub77c\ucfe0\ud504(KrakEw)-\ub974\ubd80\ud504(LwEw)\uac04\uc5d0 \ucca0\ub3c4\uac00 \ubd80\uc124\ub418\uae30 \uc2dc\uc791\ud558\uc600\ub2e4.", "paragraph_id": "6600375-13-1", "paragraph_question": "question: \ud06c\ub77c\ucfe0\ud504-\ub974\ubd80\ud06c \uac04 \ucca0\ub3c4\uac00 \uac74\uc124\ub41c \uc5f0\ub3c4\ub294?, context: 1853\ub144 \ud06c\ub9bc \uc804\uc7c1\uc774 \ubc1c\ubc1c\ud558\uc790 \uaebc\uc838\uac00\ub358 \ud3f4\ub780\ub4dc \ubbfc\uc871\uc758 \ud76c\ub9dd\uc774 \ub2e4\uc2dc \uc0b4\uc544\ub098\ub294 \uac83 \uac19\uc558\ub2e4. \uc601\uad6d\uacfc \ud504\ub791\uc2a4, \ud22c\ub974\ud06c\uc640 \ud53c\uc5d0\ubaac\ud14c(Piemonte)\uac00 \ub7ec\uc2dc\uc544\ub97c \uc0c1\ub300\ub85c \uc2f8\uc6b0\uace0 \uc788\uc5c8\uae30 \ub54c\ubb38\uc5d0 \ud3f4\ub780\ub4dc\uc758 \ub9dd\uba85\ub2e8\uccb4\ub4e4\uc740 \ud3f4\ub780\ub4dc \uc758\uc6a9\uad70\uc744 \uc870\uc9c1\ud558\uc5ec \ub300 \ub7ec\uc2dc\uc544 \uc804\uc120\uc5d0 \ud22c\uc785\ud558\ub824\uace0 \uacc4\ud68d\ud588\uc73c\ub098 \ub9dd\uba85\ub2e8\uccb4 \uc9c0\ub3c4\uc790\ub4e4\uc758 \ub0b4\ubd84 \ub54c\ubb38\uc5d0 \uad70\ub300\ub294 \uc870\uc9c1\ub418\uc9c0 \ubabb\ud588\ub2e4. 1850\ub144\ub300 \ucd08\uc5d0\ub294 \uc790\uc5f0\uc801\uc778 \uc7ac\ud574\uae4c\uc9c0 \uacb9\uce58\uace0 \ud749\ub144\uacfc \ud64d\uc218, \uc804\uc5fc\ubcd1\uc73c\ub85c \uac08\ub9ac\uc2dc\uc544 \ub18d\ubbfc\ub4e4\uc758 \uc0dd\ud65c\uc774 \ub354\uc6b1 \ube44\ucc38\ud574\uc84c\ub2e4. \uadf8 \uacb0\uacfc \ub18d\ucd0c\uc5d0\uc11c\uc758 \uc0ac\ub9dd\ub960\uc774 \uc99d\uac00\ud558\uace0 \uc774\uc5d0 \ub530\ub77c \uc778\uad6c\uac00 \uac10\uc18c\ub418\uc5c8\ub2e4. \uc774 \uc9c0\ubc29\uc5d0 \uc0b0\uc5c5\uc774 \uc77c\uc5b4\ub0a0 \uc9d5\uc870\ub294 \uc544\uc9c1 \ub098\ud0c0\ub098\uace0 \uc788\uc9c0 \uc54a\uace0 \uc788\uc5c8\ub2e4. 1856\ub144\uc5d0 \uc774\ub974\ub7ec\uc11c\uc57c \ube44\ub85c\uc18c \ud06c\ub77c\ucfe0\ud504(KrakEw)-\ub974\ubd80\ud504(LwEw)\uac04\uc5d0 \ucca0\ub3c4\uac00 \ubd80\uc124\ub418\uae30 \uc2dc\uc791\ud558\uc600\ub2e4."} {"question": "\ubd81\uc544\ud504\ub9ac\uce74\uc758 \ud669\uae08\uc790\uce7c\uc744 \ud604\uc7ac\uc758 \ub291\ub300 \uac19\uc740 \uac1c\uacfc\ub85c \ubd84\ub958\ud55c \ub3d9\ubb3c\ud559\uc790\ub294 \ub204\uad6c\uc778\uac00?", "paragraph": "2011\ub144\uae4c\uc9c0 \uc544\ud504\ub9ac\uce74\uc5d0 \ud68c\uc0c9\ub291\ub300\uac00 \uc11c\uc2dd\ud588\ub294\uc9c0\ub294 \ub17c\uc7c1\uac70\ub9ac\uc600\ub2e4. \uc544\ub9ac\uc2a4\ud1a0\ud154\ub808\uc2a4\ub294 \uc774\uc9d1\ud2b8\uc5d0\ub294 \uadf8\ub9ac\uc2a4\uc5d0 \uc11c\uc2dd\ud558\ub294 \uac83\ubcf4\ub2e4 \uc870\uae08 \uc791\uc740 \ub291\ub300\uac00 \uc11c\uc2dd\ud55c\ub2e4\uace0 \uc5b8\uae09\ud588\ub2e4. \ub3c5\uc77c\uc758 \uc758\ud559\uc790\uc778 \uac8c\uc624\ub974\uae30 \uc5d8\ubc84\uc2a4(Georg Ebers)\ub294 \ud604\uc7ac\uc758 \uc544\uc2dc\uc6b0\ud2b8, \uace0\ub300 \uc774\uc9d1\ud2b8\uc5d0\uc11c\ub294 \uc544\ub204\ube44\uc2a4\ub77c\ub294 \ub3c4\uc2dc\uac00 \"\ub3c4\uc2dc\uc758 \ub291\ub300\"\ub77c\ub294 \ub73b\uc758 \uc774\ub984\uc774 \ubd99\uc5b4\uc9c4 \uac83\uc744 \uadfc\uac70\ub85c \uc774\uc9d1\ud2b8\uac00 \ub291\ub300\ub97c \uc2e0\uc131\uc2dc\ud588\ub2e4\uace0 \uc8fc\uc7a5\ud588\ub2e4. \ub3d9\ubb3c\ud559\uc790 \uc5d0\ub978\uc2a4\ud2b8 \uc288\uc640\ub974\uce20(Ernst Schwarz)\ub294 \ubd81\uc544\ud504\ub9ac\uce74\uc758 \ud669\uae08\uc790\uce7c\uc744 \ud604\uc7ac\uc758 \ub291\ub300 \uac19\uc740 \uac1c\uacfc\ub85c \ubd84\ub958\ud588\uace0, \uc774\ud6c4 \ud68c\uc0c9\ub291\ub300\uc640 \ud615\ud0dc\ud559\uc801 \ub2ee\uc74c\uc744 \uac04\uacfc\ud55c \uac83\uc73c\ub85c \ube44\ud310\ubc1b\uc558\ub2e4. 2002\ub144 12\uc6d4, \uc5d0\ub9ac\ud2b8\ub808\uc544\uc758 \ub2e4\ub098\ud0ac \uc0ac\ub9c9\uc5d0\uc11c \uac1c\uacfc \ub3d9\ubb3c\uc774 \ubc1c\uacac\ub418\uc5c8\uc73c\uba70 \ud669\uae08\uc790\uce7c \ub610\ub294 \uc8fc\ubcc0 \uc9c0\uc5ed\uc758 6\uc885\uacfc \ube44\uc2b7\ud558\uc9c0 \uc54a\uace0 \ud68c\uc0c9\ub291\ub300\uc640 \ube44\uc2b7\ud55c \uc885\uc744 \ubc1c\uacac\ud588\ub2e4.", "answer": "\uc5d0\ub978\uc2a4\ud2b8 \uc288\uc640\ub974\uce20", "sentence": "\ub3d9\ubb3c\ud559\uc790 \uc5d0\ub978\uc2a4\ud2b8 \uc288\uc640\ub974\uce20 (Ernst Schwarz)", "paragraph_sentence": "2011\ub144\uae4c\uc9c0 \uc544\ud504\ub9ac\uce74\uc5d0 \ud68c\uc0c9\ub291\ub300\uac00 \uc11c\uc2dd\ud588\ub294\uc9c0\ub294 \ub17c\uc7c1\uac70\ub9ac\uc600\ub2e4. \uc544\ub9ac\uc2a4\ud1a0\ud154\ub808\uc2a4\ub294 \uc774\uc9d1\ud2b8\uc5d0\ub294 \uadf8\ub9ac\uc2a4\uc5d0 \uc11c\uc2dd\ud558\ub294 \uac83\ubcf4\ub2e4 \uc870\uae08 \uc791\uc740 \ub291\ub300\uac00 \uc11c\uc2dd\ud55c\ub2e4\uace0 \uc5b8\uae09\ud588\ub2e4. \ub3c5\uc77c\uc758 \uc758\ud559\uc790\uc778 \uac8c\uc624\ub974\uae30 \uc5d8\ubc84\uc2a4(Georg Ebers)\ub294 \ud604\uc7ac\uc758 \uc544\uc2dc\uc6b0\ud2b8, \uace0\ub300 \uc774\uc9d1\ud2b8\uc5d0\uc11c\ub294 \uc544\ub204\ube44\uc2a4\ub77c\ub294 \ub3c4\uc2dc\uac00 \"\ub3c4\uc2dc\uc758 \ub291\ub300\"\ub77c\ub294 \ub73b\uc758 \uc774\ub984\uc774 \ubd99\uc5b4\uc9c4 \uac83\uc744 \uadfc\uac70\ub85c \uc774\uc9d1\ud2b8\uac00 \ub291\ub300\ub97c \uc2e0\uc131\uc2dc\ud588\ub2e4\uace0 \uc8fc\uc7a5\ud588\ub2e4. \ub3d9\ubb3c\ud559\uc790 \uc5d0\ub978\uc2a4\ud2b8 \uc288\uc640\ub974\uce20 (Ernst Schwarz) \ub294 \ubd81\uc544\ud504\ub9ac\uce74\uc758 \ud669\uae08\uc790\uce7c\uc744 \ud604\uc7ac\uc758 \ub291\ub300 \uac19\uc740 \uac1c\uacfc\ub85c \ubd84\ub958\ud588\uace0, \uc774\ud6c4 \ud68c\uc0c9\ub291\ub300\uc640 \ud615\ud0dc\ud559\uc801 \ub2ee\uc74c\uc744 \uac04\uacfc\ud55c \uac83\uc73c\ub85c \ube44\ud310\ubc1b\uc558\ub2e4. 2002\ub144 12\uc6d4, \uc5d0\ub9ac\ud2b8\ub808\uc544\uc758 \ub2e4\ub098\ud0ac \uc0ac\ub9c9\uc5d0\uc11c \uac1c\uacfc \ub3d9\ubb3c\uc774 \ubc1c\uacac\ub418\uc5c8\uc73c\uba70 \ud669\uae08\uc790\uce7c \ub610\ub294 \uc8fc\ubcc0 \uc9c0\uc5ed\uc758 6\uc885\uacfc \ube44\uc2b7\ud558\uc9c0 \uc54a\uace0 \ud68c\uc0c9\ub291\ub300\uc640 \ube44\uc2b7\ud55c \uc885\uc744 \ubc1c\uacac\ud588\ub2e4.", "paragraph_answer": "2011\ub144\uae4c\uc9c0 \uc544\ud504\ub9ac\uce74\uc5d0 \ud68c\uc0c9\ub291\ub300\uac00 \uc11c\uc2dd\ud588\ub294\uc9c0\ub294 \ub17c\uc7c1\uac70\ub9ac\uc600\ub2e4. \uc544\ub9ac\uc2a4\ud1a0\ud154\ub808\uc2a4\ub294 \uc774\uc9d1\ud2b8\uc5d0\ub294 \uadf8\ub9ac\uc2a4\uc5d0 \uc11c\uc2dd\ud558\ub294 \uac83\ubcf4\ub2e4 \uc870\uae08 \uc791\uc740 \ub291\ub300\uac00 \uc11c\uc2dd\ud55c\ub2e4\uace0 \uc5b8\uae09\ud588\ub2e4. \ub3c5\uc77c\uc758 \uc758\ud559\uc790\uc778 \uac8c\uc624\ub974\uae30 \uc5d8\ubc84\uc2a4(Georg Ebers)\ub294 \ud604\uc7ac\uc758 \uc544\uc2dc\uc6b0\ud2b8, \uace0\ub300 \uc774\uc9d1\ud2b8\uc5d0\uc11c\ub294 \uc544\ub204\ube44\uc2a4\ub77c\ub294 \ub3c4\uc2dc\uac00 \"\ub3c4\uc2dc\uc758 \ub291\ub300\"\ub77c\ub294 \ub73b\uc758 \uc774\ub984\uc774 \ubd99\uc5b4\uc9c4 \uac83\uc744 \uadfc\uac70\ub85c \uc774\uc9d1\ud2b8\uac00 \ub291\ub300\ub97c \uc2e0\uc131\uc2dc\ud588\ub2e4\uace0 \uc8fc\uc7a5\ud588\ub2e4. \ub3d9\ubb3c\ud559\uc790 \uc5d0\ub978\uc2a4\ud2b8 \uc288\uc640\ub974\uce20 (Ernst Schwarz)\ub294 \ubd81\uc544\ud504\ub9ac\uce74\uc758 \ud669\uae08\uc790\uce7c\uc744 \ud604\uc7ac\uc758 \ub291\ub300 \uac19\uc740 \uac1c\uacfc\ub85c \ubd84\ub958\ud588\uace0, \uc774\ud6c4 \ud68c\uc0c9\ub291\ub300\uc640 \ud615\ud0dc\ud559\uc801 \ub2ee\uc74c\uc744 \uac04\uacfc\ud55c \uac83\uc73c\ub85c \ube44\ud310\ubc1b\uc558\ub2e4. 2002\ub144 12\uc6d4, \uc5d0\ub9ac\ud2b8\ub808\uc544\uc758 \ub2e4\ub098\ud0ac \uc0ac\ub9c9\uc5d0\uc11c \uac1c\uacfc \ub3d9\ubb3c\uc774 \ubc1c\uacac\ub418\uc5c8\uc73c\uba70 \ud669\uae08\uc790\uce7c \ub610\ub294 \uc8fc\ubcc0 \uc9c0\uc5ed\uc758 6\uc885\uacfc \ube44\uc2b7\ud558\uc9c0 \uc54a\uace0 \ud68c\uc0c9\ub291\ub300\uc640 \ube44\uc2b7\ud55c \uc885\uc744 \ubc1c\uacac\ud588\ub2e4.", "sentence_answer": "\ub3d9\ubb3c\ud559\uc790 \uc5d0\ub978\uc2a4\ud2b8 \uc288\uc640\ub974\uce20 (Ernst Schwarz)", "paragraph_id": "6572454-40-2", "paragraph_question": "question: \ubd81\uc544\ud504\ub9ac\uce74\uc758 \ud669\uae08\uc790\uce7c\uc744 \ud604\uc7ac\uc758 \ub291\ub300 \uac19\uc740 \uac1c\uacfc\ub85c \ubd84\ub958\ud55c \ub3d9\ubb3c\ud559\uc790\ub294 \ub204\uad6c\uc778\uac00?, context: 2011\ub144\uae4c\uc9c0 \uc544\ud504\ub9ac\uce74\uc5d0 \ud68c\uc0c9\ub291\ub300\uac00 \uc11c\uc2dd\ud588\ub294\uc9c0\ub294 \ub17c\uc7c1\uac70\ub9ac\uc600\ub2e4. \uc544\ub9ac\uc2a4\ud1a0\ud154\ub808\uc2a4\ub294 \uc774\uc9d1\ud2b8\uc5d0\ub294 \uadf8\ub9ac\uc2a4\uc5d0 \uc11c\uc2dd\ud558\ub294 \uac83\ubcf4\ub2e4 \uc870\uae08 \uc791\uc740 \ub291\ub300\uac00 \uc11c\uc2dd\ud55c\ub2e4\uace0 \uc5b8\uae09\ud588\ub2e4. \ub3c5\uc77c\uc758 \uc758\ud559\uc790\uc778 \uac8c\uc624\ub974\uae30 \uc5d8\ubc84\uc2a4(Georg Ebers)\ub294 \ud604\uc7ac\uc758 \uc544\uc2dc\uc6b0\ud2b8, \uace0\ub300 \uc774\uc9d1\ud2b8\uc5d0\uc11c\ub294 \uc544\ub204\ube44\uc2a4\ub77c\ub294 \ub3c4\uc2dc\uac00 \"\ub3c4\uc2dc\uc758 \ub291\ub300\"\ub77c\ub294 \ub73b\uc758 \uc774\ub984\uc774 \ubd99\uc5b4\uc9c4 \uac83\uc744 \uadfc\uac70\ub85c \uc774\uc9d1\ud2b8\uac00 \ub291\ub300\ub97c \uc2e0\uc131\uc2dc\ud588\ub2e4\uace0 \uc8fc\uc7a5\ud588\ub2e4. \ub3d9\ubb3c\ud559\uc790 \uc5d0\ub978\uc2a4\ud2b8 \uc288\uc640\ub974\uce20(Ernst Schwarz)\ub294 \ubd81\uc544\ud504\ub9ac\uce74\uc758 \ud669\uae08\uc790\uce7c\uc744 \ud604\uc7ac\uc758 \ub291\ub300 \uac19\uc740 \uac1c\uacfc\ub85c \ubd84\ub958\ud588\uace0, \uc774\ud6c4 \ud68c\uc0c9\ub291\ub300\uc640 \ud615\ud0dc\ud559\uc801 \ub2ee\uc74c\uc744 \uac04\uacfc\ud55c \uac83\uc73c\ub85c \ube44\ud310\ubc1b\uc558\ub2e4. 2002\ub144 12\uc6d4, \uc5d0\ub9ac\ud2b8\ub808\uc544\uc758 \ub2e4\ub098\ud0ac \uc0ac\ub9c9\uc5d0\uc11c \uac1c\uacfc \ub3d9\ubb3c\uc774 \ubc1c\uacac\ub418\uc5c8\uc73c\uba70 \ud669\uae08\uc790\uce7c \ub610\ub294 \uc8fc\ubcc0 \uc9c0\uc5ed\uc758 6\uc885\uacfc \ube44\uc2b7\ud558\uc9c0 \uc54a\uace0 \ud68c\uc0c9\ub291\ub300\uc640 \ube44\uc2b7\ud55c \uc885\uc744 \ubc1c\uacac\ud588\ub2e4."} {"question": "1907\ub144 \uc624\ucfe0\ud558\ub77c \ud5e4\ud0a4\uc6b4\uc774 \uc4f4 \ucc45 \uc774\ub984\uc740?", "paragraph": "\ubc14\ub2e4\uc0ac\uc790(\ub3c5\ub3c4\uac15\uce58)\ub294 \ub3d9\ud574 \uc5f0\uc548\uc5d0 \uc11c\uc2dd\ud558\ub358 \uac15\uce58\uc758 \uc544\uc885\uc774\ub2e4. \ud604\uc7ac \uba78\uc885\ub418\uc5c8\ub2e4\uace0 \uc54c\ub824\uc9c4 \ub3d9\ubb3c\ub85c, \uc870\uc120\uc2dc\ub300 \uc0ac\ub78c\ub4e4\uc740 \uc774\ub4e4\uc744 \"\uac00\uc81c\" \ub610\ub294 \"\uac00\uc9c0\"\ub85c \ubd88\ub800\uc73c\uba70 \uc774 \uc12c\uc744 \uc911\uc2ec\uc73c\ub85c \ub3d9\ud574\uc5d0 \uc218\ub9cc \ub9c8\ub9ac\uac00 \uc11c\uc2dd\ud588\ub2e4\uace0 \ud55c\ub2e4. \uc774\ub4e4\uc774 \uba38\ubb3c\ub800\ub2e4\ub294 \uac00\uc81c\ubc14\uc704\uac00 \ub3c5\ub3c4\uc5d0 \ub0a8\uc544 \uc788\ub2e4. \ub7ec\uc77c \uc804\uc7c1 \uc804\ud6c4\ub85c \uac00\uc8fd\uc744 \uc5bb\uae30 \uc704\ud574 \uc2dc\uc791\ub41c \uc77c\ubcf8\uc778\ub4e4\uc758 \ubb34\ubd84\ubcc4\ud55c \ub0a8\ud68d\uc73c\ub85c \ubc14\ub2e4\uc0ac\uc790\ub294 \uc11c\uc11c\ud788 \uadf8 \ubaa8\uc2b5\uc744 \uac10\ucd94\uc5c8\uc73c\uba70 1974\ub144 \ud64b\uce74\uc774\ub3c4\uc5d0\uc11c \uc0c8\ub07c \uac15\uce58\uac00 \ud655\uc778\ub41c \uc774\ud6c4\ub85c \ubaa9\uaca9\ub418\uc9c0 \uc54a\ub294\ub2e4. 1905\ub144 \uc77c\ubcf8 \uc2dc\ub9c8\ub124\ud604\uc774 \uc774 \uc12c\uc744 \ubb34\ub2e8\uc73c\ub85c \ud3b8\uc785\ud55c \uc774\ud6c4 \uc77c\ubcf8\uc778\ub4e4\uc758 \uc5b4\ud68d\uc774 \uc2dc\uc791\ub418\uc5c8\uc9c0\ub9cc, 1905\ub144 \uc774\uc804\uc5d0 \uc6b8\ub989\ub3c4\uc5d0 \uc0b4\ub358 \ud55c\uad6d\uc778\ub4e4\uc774 1904\ub144\uacfc 1905\ub144\uc5d0 \ub3c5\ub3c4\uc5d0\uc11c \uac15\uce58\ub97c \uc7a1\uc544 \ub9e4\ub144 \uac00\uc8fd 800\uad00(600\uc5d4)\uc529 \uc77c\ubcf8\uc5d0 \uc218\ucd9c\ud55c \uae30\ub85d\uc774 1907\ub144 \uc2dc\ub9c8\ub124\ud604 \ub2e4\ucf00\uc2dc\ub9c8 \uc12c \uc870\uc0ac\ub2e8\uc758 \uc624\ucfe0\ud558\ub77c \ud5e4\ud0a4\uc6b4\uc774 \uc4f4 \ucc45 '\uc8fd\ub3c4 \ubc0f \uc6b8\ub989\ub3c4'\uc5d0 \ub098\uc628\ub2e4.", "answer": "\uc8fd\ub3c4 \ubc0f \uc6b8\ub989\ub3c4", "sentence": "1905\ub144 \uc77c\ubcf8 \uc2dc\ub9c8\ub124\ud604\uc774 \uc774 \uc12c\uc744 \ubb34\ub2e8\uc73c\ub85c \ud3b8\uc785\ud55c \uc774\ud6c4 \uc77c\ubcf8\uc778\ub4e4\uc758 \uc5b4\ud68d\uc774 \uc2dc\uc791\ub418\uc5c8\uc9c0\ub9cc, 1905\ub144 \uc774\uc804\uc5d0 \uc6b8\ub989\ub3c4\uc5d0 \uc0b4\ub358 \ud55c\uad6d\uc778\ub4e4\uc774 1904\ub144\uacfc 1905\ub144\uc5d0 \ub3c5\ub3c4\uc5d0\uc11c \uac15\uce58\ub97c \uc7a1\uc544 \ub9e4\ub144 \uac00\uc8fd 800\uad00(600\uc5d4)\uc529 \uc77c\ubcf8\uc5d0 \uc218\ucd9c\ud55c \uae30\ub85d\uc774 1907\ub144 \uc2dc\ub9c8\ub124\ud604 \ub2e4\ucf00\uc2dc\ub9c8 \uc12c \uc870\uc0ac\ub2e8\uc758 \uc624\ucfe0\ud558\ub77c \ud5e4\ud0a4\uc6b4\uc774 \uc4f4 \ucc45 ' \uc8fd\ub3c4 \ubc0f \uc6b8\ub989\ub3c4 '\uc5d0 \ub098\uc628\ub2e4.", "paragraph_sentence": "\ubc14\ub2e4\uc0ac\uc790(\ub3c5\ub3c4\uac15\uce58)\ub294 \ub3d9\ud574 \uc5f0\uc548\uc5d0 \uc11c\uc2dd\ud558\ub358 \uac15\uce58\uc758 \uc544\uc885\uc774\ub2e4. \ud604\uc7ac \uba78\uc885\ub418\uc5c8\ub2e4\uace0 \uc54c\ub824\uc9c4 \ub3d9\ubb3c\ub85c, \uc870\uc120\uc2dc\ub300 \uc0ac\ub78c\ub4e4\uc740 \uc774\ub4e4\uc744 \"\uac00\uc81c\" \ub610\ub294 \"\uac00\uc9c0\"\ub85c \ubd88\ub800\uc73c\uba70 \uc774 \uc12c\uc744 \uc911\uc2ec\uc73c\ub85c \ub3d9\ud574\uc5d0 \uc218\ub9cc \ub9c8\ub9ac\uac00 \uc11c\uc2dd\ud588\ub2e4\uace0 \ud55c\ub2e4. \uc774\ub4e4\uc774 \uba38\ubb3c\ub800\ub2e4\ub294 \uac00\uc81c\ubc14\uc704\uac00 \ub3c5\ub3c4\uc5d0 \ub0a8\uc544 \uc788\ub2e4. \ub7ec\uc77c \uc804\uc7c1 \uc804\ud6c4\ub85c \uac00\uc8fd\uc744 \uc5bb\uae30 \uc704\ud574 \uc2dc\uc791\ub41c \uc77c\ubcf8\uc778\ub4e4\uc758 \ubb34\ubd84\ubcc4\ud55c \ub0a8\ud68d\uc73c\ub85c \ubc14\ub2e4\uc0ac\uc790\ub294 \uc11c\uc11c\ud788 \uadf8 \ubaa8\uc2b5\uc744 \uac10\ucd94\uc5c8\uc73c\uba70 1974\ub144 \ud64b\uce74\uc774\ub3c4\uc5d0\uc11c \uc0c8\ub07c \uac15\uce58\uac00 \ud655\uc778\ub41c \uc774\ud6c4\ub85c \ubaa9\uaca9\ub418\uc9c0 \uc54a\ub294\ub2e4. 1905\ub144 \uc77c\ubcf8 \uc2dc\ub9c8\ub124\ud604\uc774 \uc774 \uc12c\uc744 \ubb34\ub2e8\uc73c\ub85c \ud3b8\uc785\ud55c \uc774\ud6c4 \uc77c\ubcf8\uc778\ub4e4\uc758 \uc5b4\ud68d\uc774 \uc2dc\uc791\ub418\uc5c8\uc9c0\ub9cc, 1905\ub144 \uc774\uc804\uc5d0 \uc6b8\ub989\ub3c4\uc5d0 \uc0b4\ub358 \ud55c\uad6d\uc778\ub4e4\uc774 1904\ub144\uacfc 1905\ub144\uc5d0 \ub3c5\ub3c4\uc5d0\uc11c \uac15\uce58\ub97c \uc7a1\uc544 \ub9e4\ub144 \uac00\uc8fd 800\uad00(600\uc5d4)\uc529 \uc77c\ubcf8\uc5d0 \uc218\ucd9c\ud55c \uae30\ub85d\uc774 1907\ub144 \uc2dc\ub9c8\ub124\ud604 \ub2e4\ucf00\uc2dc\ub9c8 \uc12c \uc870\uc0ac\ub2e8\uc758 \uc624\ucfe0\ud558\ub77c \ud5e4\ud0a4\uc6b4\uc774 \uc4f4 \ucc45 ' \uc8fd\ub3c4 \ubc0f \uc6b8\ub989\ub3c4 '\uc5d0 \ub098\uc628\ub2e4. ", "paragraph_answer": "\ubc14\ub2e4\uc0ac\uc790(\ub3c5\ub3c4\uac15\uce58)\ub294 \ub3d9\ud574 \uc5f0\uc548\uc5d0 \uc11c\uc2dd\ud558\ub358 \uac15\uce58\uc758 \uc544\uc885\uc774\ub2e4. \ud604\uc7ac \uba78\uc885\ub418\uc5c8\ub2e4\uace0 \uc54c\ub824\uc9c4 \ub3d9\ubb3c\ub85c, \uc870\uc120\uc2dc\ub300 \uc0ac\ub78c\ub4e4\uc740 \uc774\ub4e4\uc744 \"\uac00\uc81c\" \ub610\ub294 \"\uac00\uc9c0\"\ub85c \ubd88\ub800\uc73c\uba70 \uc774 \uc12c\uc744 \uc911\uc2ec\uc73c\ub85c \ub3d9\ud574\uc5d0 \uc218\ub9cc \ub9c8\ub9ac\uac00 \uc11c\uc2dd\ud588\ub2e4\uace0 \ud55c\ub2e4. \uc774\ub4e4\uc774 \uba38\ubb3c\ub800\ub2e4\ub294 \uac00\uc81c\ubc14\uc704\uac00 \ub3c5\ub3c4\uc5d0 \ub0a8\uc544 \uc788\ub2e4. \ub7ec\uc77c \uc804\uc7c1 \uc804\ud6c4\ub85c \uac00\uc8fd\uc744 \uc5bb\uae30 \uc704\ud574 \uc2dc\uc791\ub41c \uc77c\ubcf8\uc778\ub4e4\uc758 \ubb34\ubd84\ubcc4\ud55c \ub0a8\ud68d\uc73c\ub85c \ubc14\ub2e4\uc0ac\uc790\ub294 \uc11c\uc11c\ud788 \uadf8 \ubaa8\uc2b5\uc744 \uac10\ucd94\uc5c8\uc73c\uba70 1974\ub144 \ud64b\uce74\uc774\ub3c4\uc5d0\uc11c \uc0c8\ub07c \uac15\uce58\uac00 \ud655\uc778\ub41c \uc774\ud6c4\ub85c \ubaa9\uaca9\ub418\uc9c0 \uc54a\ub294\ub2e4. 1905\ub144 \uc77c\ubcf8 \uc2dc\ub9c8\ub124\ud604\uc774 \uc774 \uc12c\uc744 \ubb34\ub2e8\uc73c\ub85c \ud3b8\uc785\ud55c \uc774\ud6c4 \uc77c\ubcf8\uc778\ub4e4\uc758 \uc5b4\ud68d\uc774 \uc2dc\uc791\ub418\uc5c8\uc9c0\ub9cc, 1905\ub144 \uc774\uc804\uc5d0 \uc6b8\ub989\ub3c4\uc5d0 \uc0b4\ub358 \ud55c\uad6d\uc778\ub4e4\uc774 1904\ub144\uacfc 1905\ub144\uc5d0 \ub3c5\ub3c4\uc5d0\uc11c \uac15\uce58\ub97c \uc7a1\uc544 \ub9e4\ub144 \uac00\uc8fd 800\uad00(600\uc5d4)\uc529 \uc77c\ubcf8\uc5d0 \uc218\ucd9c\ud55c \uae30\ub85d\uc774 1907\ub144 \uc2dc\ub9c8\ub124\ud604 \ub2e4\ucf00\uc2dc\ub9c8 \uc12c \uc870\uc0ac\ub2e8\uc758 \uc624\ucfe0\ud558\ub77c \ud5e4\ud0a4\uc6b4\uc774 \uc4f4 \ucc45 ' \uc8fd\ub3c4 \ubc0f \uc6b8\ub989\ub3c4 '\uc5d0 \ub098\uc628\ub2e4.", "sentence_answer": "1905\ub144 \uc77c\ubcf8 \uc2dc\ub9c8\ub124\ud604\uc774 \uc774 \uc12c\uc744 \ubb34\ub2e8\uc73c\ub85c \ud3b8\uc785\ud55c \uc774\ud6c4 \uc77c\ubcf8\uc778\ub4e4\uc758 \uc5b4\ud68d\uc774 \uc2dc\uc791\ub418\uc5c8\uc9c0\ub9cc, 1905\ub144 \uc774\uc804\uc5d0 \uc6b8\ub989\ub3c4\uc5d0 \uc0b4\ub358 \ud55c\uad6d\uc778\ub4e4\uc774 1904\ub144\uacfc 1905\ub144\uc5d0 \ub3c5\ub3c4\uc5d0\uc11c \uac15\uce58\ub97c \uc7a1\uc544 \ub9e4\ub144 \uac00\uc8fd 800\uad00(600\uc5d4)\uc529 \uc77c\ubcf8\uc5d0 \uc218\ucd9c\ud55c \uae30\ub85d\uc774 1907\ub144 \uc2dc\ub9c8\ub124\ud604 \ub2e4\ucf00\uc2dc\ub9c8 \uc12c \uc870\uc0ac\ub2e8\uc758 \uc624\ucfe0\ud558\ub77c \ud5e4\ud0a4\uc6b4\uc774 \uc4f4 \ucc45 ' \uc8fd\ub3c4 \ubc0f \uc6b8\ub989\ub3c4 '\uc5d0 \ub098\uc628\ub2e4.", "paragraph_id": "6492102-2-1", "paragraph_question": "question: 1907\ub144 \uc624\ucfe0\ud558\ub77c \ud5e4\ud0a4\uc6b4\uc774 \uc4f4 \ucc45 \uc774\ub984\uc740?, context: \ubc14\ub2e4\uc0ac\uc790(\ub3c5\ub3c4\uac15\uce58)\ub294 \ub3d9\ud574 \uc5f0\uc548\uc5d0 \uc11c\uc2dd\ud558\ub358 \uac15\uce58\uc758 \uc544\uc885\uc774\ub2e4. \ud604\uc7ac \uba78\uc885\ub418\uc5c8\ub2e4\uace0 \uc54c\ub824\uc9c4 \ub3d9\ubb3c\ub85c, \uc870\uc120\uc2dc\ub300 \uc0ac\ub78c\ub4e4\uc740 \uc774\ub4e4\uc744 \"\uac00\uc81c\" \ub610\ub294 \"\uac00\uc9c0\"\ub85c \ubd88\ub800\uc73c\uba70 \uc774 \uc12c\uc744 \uc911\uc2ec\uc73c\ub85c \ub3d9\ud574\uc5d0 \uc218\ub9cc \ub9c8\ub9ac\uac00 \uc11c\uc2dd\ud588\ub2e4\uace0 \ud55c\ub2e4. \uc774\ub4e4\uc774 \uba38\ubb3c\ub800\ub2e4\ub294 \uac00\uc81c\ubc14\uc704\uac00 \ub3c5\ub3c4\uc5d0 \ub0a8\uc544 \uc788\ub2e4. \ub7ec\uc77c \uc804\uc7c1 \uc804\ud6c4\ub85c \uac00\uc8fd\uc744 \uc5bb\uae30 \uc704\ud574 \uc2dc\uc791\ub41c \uc77c\ubcf8\uc778\ub4e4\uc758 \ubb34\ubd84\ubcc4\ud55c \ub0a8\ud68d\uc73c\ub85c \ubc14\ub2e4\uc0ac\uc790\ub294 \uc11c\uc11c\ud788 \uadf8 \ubaa8\uc2b5\uc744 \uac10\ucd94\uc5c8\uc73c\uba70 1974\ub144 \ud64b\uce74\uc774\ub3c4\uc5d0\uc11c \uc0c8\ub07c \uac15\uce58\uac00 \ud655\uc778\ub41c \uc774\ud6c4\ub85c \ubaa9\uaca9\ub418\uc9c0 \uc54a\ub294\ub2e4. 1905\ub144 \uc77c\ubcf8 \uc2dc\ub9c8\ub124\ud604\uc774 \uc774 \uc12c\uc744 \ubb34\ub2e8\uc73c\ub85c \ud3b8\uc785\ud55c \uc774\ud6c4 \uc77c\ubcf8\uc778\ub4e4\uc758 \uc5b4\ud68d\uc774 \uc2dc\uc791\ub418\uc5c8\uc9c0\ub9cc, 1905\ub144 \uc774\uc804\uc5d0 \uc6b8\ub989\ub3c4\uc5d0 \uc0b4\ub358 \ud55c\uad6d\uc778\ub4e4\uc774 1904\ub144\uacfc 1905\ub144\uc5d0 \ub3c5\ub3c4\uc5d0\uc11c \uac15\uce58\ub97c \uc7a1\uc544 \ub9e4\ub144 \uac00\uc8fd 800\uad00(600\uc5d4)\uc529 \uc77c\ubcf8\uc5d0 \uc218\ucd9c\ud55c \uae30\ub85d\uc774 1907\ub144 \uc2dc\ub9c8\ub124\ud604 \ub2e4\ucf00\uc2dc\ub9c8 \uc12c \uc870\uc0ac\ub2e8\uc758 \uc624\ucfe0\ud558\ub77c \ud5e4\ud0a4\uc6b4\uc774 \uc4f4 \ucc45 '\uc8fd\ub3c4 \ubc0f \uc6b8\ub989\ub3c4'\uc5d0 \ub098\uc628\ub2e4."} {"question": "\uc774\ud574\ucc2c\uc774 3.1\uc808 \uae30\ub150\uc2dd\uc5d0 \ucc38\uc11d\ud558\uc9c0 \uc54a\uace0 \uac14\ub358 \uacf3\uc740?", "paragraph": "2006\ub144 3\uc6d4 1\uc77c\uc5d0 \uc138\uc885\ubb38\ud654\ud68c\uad00\uc5d0\uc11c \uc5f4\ub9b0 3.1\uc808 \uae30\ub150\uc2dd\uc5d0\ub294 \ucc38\uc11d\ud558\uc9c0 \uc54a\uace0 \uc774\ud574\ucc2c\uc740 \ubd80\uc0b0\uc9c0\uc5ed \uc0c1\uacf5\uc778\ub4e4\uacfc \uace8\ud504\ubaa8\uc784\uc744 \uac00\uc838 \ub17c\ub780\uc774 \ub418\uc5c8\ub294\ub370, \uc774\ud6c4 \uc57c\ub2f9\uc778 \ud55c\ub098\ub77c\ub2f9\uacfc \ubbfc\uc8fc\ub2f9 \uadf8\ub9ac\uace0 \uc5ec\ub2f9\uc778 \uc5f4\ub9b0\uc6b0\ub9ac\ub2f9 \ub4f1 \uc815\uce58\uad8c\uacfc \uc5b8\ub860\uc758 \uad6c\uc124\uc5d0 \uc62c\ub790\ub2e4. 9\uc77c \ud6c4\uc778 3\uc6d4 10\uc77c\uc5d0\ub294 \uccad\uc640\ub300\uac00 \uace8\ud504 \ud30c\ubb38 \uc758\ud639\uc744 \uc9c1\uc811 \uc870\uc0ac\ud558\uae30\ub3c4 \ud588\uace0, \uc774\ud6c4 3\uc6d4 15\uc77c \ub178\ubb34\ud604 \ub300\ud1b5\ub839\uc774 \uc774\ud574\ucc2c\uc758 \ucd1d\ub9ac\uc9c1 \uc0ac\uc758\ub97c \uc218\uc6a9\ud558\uba74\uc11c \ucd1d\ub9ac\uc9c1\uc5d0\uc11c \ubb3c\ub7ec\ub098\uac8c \ub41c\ub2e4. \uc774 \uc0ac\uac74\uc758 \uc5ec\ud30c\ub85c \uacf5\uc9c1\uc0ac\ud68c \uc77c\ubd80\uc758 \ubc18\ubc1c\uc774 \uc788\uc5c8\uc73c\ub098, \uc811\ub300\uc131 \uace8\ud504 \ubfd0\ub9cc \uc544\ub2c8\ub77c \uce5c\ucc99\uc744 \uc81c\uc678\ud55c \ubaa8\ub4e0 \uc9c1\ubb34\uad00\ub828\uc790\uc640\ub294 \uace8\ud504\ub97c \ud560 \uc218 \uc5c6\ub3c4\ub85d \uaddc\uc81c \uc870\ud56d\uc774 \uc2e0\uc124\ub418\uae30\ub3c4 \ud588\ub2e4. \uc774\ud574\ucc2c\uc740 \uc774\uc640 \ubcc4\ub3c4\ub85c \uc2dc\ubbfc\ub2e8\uccb4 \ud559\uc0ac\ubaa8\ub85c\ubd80\ud130 \uae40\uc9c4\ud45c\uc640 \ud568\uaed8 \uace8\ud504\ud30c\ubb38\uacfc \uad00\ub828\ud558\uc5ec \ud53c\uc18c\ub2f9\ud558\uae30\ub3c4 \ud588\ub2e4. \ub610\ud55c \ucd1d\ub9ac \uc7ac\uc784\uae30\uac04\ub3d9\uc548 \uad6d\ubb34\uc704\uc6d0\ub4e4\uacfc \uacbd\uc81c\ub2e8\uccb4\uc7a5\ub4e4\uc774 \ub9cc\ub098\uc11c \uace8\ud504\uce58\uba74\uc11c \uc815\u00b7\uc7ac\uacc4 \ud604\uc548\uc744 \ub17c\ud558\ub294 \uc774\ub978\ubc14 \uace8\ud504\ud68c\ub3d9\uc774 \uc790\uc8fc \uc774\uc5b4\uc84c\ub2e4.", "answer": "\uace8\ud504\ubaa8\uc784", "sentence": "2006\ub144 3\uc6d4 1\uc77c\uc5d0 \uc138\uc885\ubb38\ud654\ud68c\uad00\uc5d0\uc11c \uc5f4\ub9b0 3.1\uc808 \uae30\ub150\uc2dd\uc5d0\ub294 \ucc38\uc11d\ud558\uc9c0 \uc54a\uace0 \uc774\ud574\ucc2c\uc740 \ubd80\uc0b0\uc9c0\uc5ed \uc0c1\uacf5\uc778\ub4e4\uacfc \uace8\ud504\ubaa8\uc784 \uc744 \uac00\uc838 \ub17c\ub780\uc774 \ub418\uc5c8\ub294\ub370, \uc774\ud6c4 \uc57c\ub2f9\uc778 \ud55c\ub098\ub77c\ub2f9\uacfc \ubbfc\uc8fc\ub2f9 \uadf8\ub9ac\uace0 \uc5ec\ub2f9\uc778 \uc5f4\ub9b0\uc6b0\ub9ac\ub2f9 \ub4f1 \uc815\uce58\uad8c\uacfc \uc5b8\ub860\uc758 \uad6c\uc124\uc5d0 \uc62c\ub790\ub2e4.", "paragraph_sentence": " 2006\ub144 3\uc6d4 1\uc77c\uc5d0 \uc138\uc885\ubb38\ud654\ud68c\uad00\uc5d0\uc11c \uc5f4\ub9b0 3.1\uc808 \uae30\ub150\uc2dd\uc5d0\ub294 \ucc38\uc11d\ud558\uc9c0 \uc54a\uace0 \uc774\ud574\ucc2c\uc740 \ubd80\uc0b0\uc9c0\uc5ed \uc0c1\uacf5\uc778\ub4e4\uacfc \uace8\ud504\ubaa8\uc784 \uc744 \uac00\uc838 \ub17c\ub780\uc774 \ub418\uc5c8\ub294\ub370, \uc774\ud6c4 \uc57c\ub2f9\uc778 \ud55c\ub098\ub77c\ub2f9\uacfc \ubbfc\uc8fc\ub2f9 \uadf8\ub9ac\uace0 \uc5ec\ub2f9\uc778 \uc5f4\ub9b0\uc6b0\ub9ac\ub2f9 \ub4f1 \uc815\uce58\uad8c\uacfc \uc5b8\ub860\uc758 \uad6c\uc124\uc5d0 \uc62c\ub790\ub2e4. 9\uc77c \ud6c4\uc778 3\uc6d4 10\uc77c\uc5d0\ub294 \uccad\uc640\ub300\uac00 \uace8\ud504 \ud30c\ubb38 \uc758\ud639\uc744 \uc9c1\uc811 \uc870\uc0ac\ud558\uae30\ub3c4 \ud588\uace0, \uc774\ud6c4 3\uc6d4 15\uc77c \ub178\ubb34\ud604 \ub300\ud1b5\ub839\uc774 \uc774\ud574\ucc2c\uc758 \ucd1d\ub9ac\uc9c1 \uc0ac\uc758\ub97c \uc218\uc6a9\ud558\uba74\uc11c \ucd1d\ub9ac\uc9c1\uc5d0\uc11c \ubb3c\ub7ec\ub098\uac8c \ub41c\ub2e4. \uc774 \uc0ac\uac74\uc758 \uc5ec\ud30c\ub85c \uacf5\uc9c1\uc0ac\ud68c \uc77c\ubd80\uc758 \ubc18\ubc1c\uc774 \uc788\uc5c8\uc73c\ub098, \uc811\ub300\uc131 \uace8\ud504 \ubfd0\ub9cc \uc544\ub2c8\ub77c \uce5c\ucc99\uc744 \uc81c\uc678\ud55c \ubaa8\ub4e0 \uc9c1\ubb34\uad00\ub828\uc790\uc640\ub294 \uace8\ud504\ub97c \ud560 \uc218 \uc5c6\ub3c4\ub85d \uaddc\uc81c \uc870\ud56d\uc774 \uc2e0\uc124\ub418\uae30\ub3c4 \ud588\ub2e4. \uc774\ud574\ucc2c\uc740 \uc774\uc640 \ubcc4\ub3c4\ub85c \uc2dc\ubbfc\ub2e8\uccb4 \ud559\uc0ac\ubaa8\ub85c\ubd80\ud130 \uae40\uc9c4\ud45c\uc640 \ud568\uaed8 \uace8\ud504\ud30c\ubb38\uacfc \uad00\ub828\ud558\uc5ec \ud53c\uc18c\ub2f9\ud558\uae30\ub3c4 \ud588\ub2e4. \ub610\ud55c \ucd1d\ub9ac \uc7ac\uc784\uae30\uac04\ub3d9\uc548 \uad6d\ubb34\uc704\uc6d0\ub4e4\uacfc \uacbd\uc81c\ub2e8\uccb4\uc7a5\ub4e4\uc774 \ub9cc\ub098\uc11c \uace8\ud504\uce58\uba74\uc11c \uc815\u00b7\uc7ac\uacc4 \ud604\uc548\uc744 \ub17c\ud558\ub294 \uc774\ub978\ubc14 \uace8\ud504\ud68c\ub3d9\uc774 \uc790\uc8fc \uc774\uc5b4\uc84c\ub2e4.", "paragraph_answer": "2006\ub144 3\uc6d4 1\uc77c\uc5d0 \uc138\uc885\ubb38\ud654\ud68c\uad00\uc5d0\uc11c \uc5f4\ub9b0 3.1\uc808 \uae30\ub150\uc2dd\uc5d0\ub294 \ucc38\uc11d\ud558\uc9c0 \uc54a\uace0 \uc774\ud574\ucc2c\uc740 \ubd80\uc0b0\uc9c0\uc5ed \uc0c1\uacf5\uc778\ub4e4\uacfc \uace8\ud504\ubaa8\uc784 \uc744 \uac00\uc838 \ub17c\ub780\uc774 \ub418\uc5c8\ub294\ub370, \uc774\ud6c4 \uc57c\ub2f9\uc778 \ud55c\ub098\ub77c\ub2f9\uacfc \ubbfc\uc8fc\ub2f9 \uadf8\ub9ac\uace0 \uc5ec\ub2f9\uc778 \uc5f4\ub9b0\uc6b0\ub9ac\ub2f9 \ub4f1 \uc815\uce58\uad8c\uacfc \uc5b8\ub860\uc758 \uad6c\uc124\uc5d0 \uc62c\ub790\ub2e4. 9\uc77c \ud6c4\uc778 3\uc6d4 10\uc77c\uc5d0\ub294 \uccad\uc640\ub300\uac00 \uace8\ud504 \ud30c\ubb38 \uc758\ud639\uc744 \uc9c1\uc811 \uc870\uc0ac\ud558\uae30\ub3c4 \ud588\uace0, \uc774\ud6c4 3\uc6d4 15\uc77c \ub178\ubb34\ud604 \ub300\ud1b5\ub839\uc774 \uc774\ud574\ucc2c\uc758 \ucd1d\ub9ac\uc9c1 \uc0ac\uc758\ub97c \uc218\uc6a9\ud558\uba74\uc11c \ucd1d\ub9ac\uc9c1\uc5d0\uc11c \ubb3c\ub7ec\ub098\uac8c \ub41c\ub2e4. \uc774 \uc0ac\uac74\uc758 \uc5ec\ud30c\ub85c \uacf5\uc9c1\uc0ac\ud68c \uc77c\ubd80\uc758 \ubc18\ubc1c\uc774 \uc788\uc5c8\uc73c\ub098, \uc811\ub300\uc131 \uace8\ud504 \ubfd0\ub9cc \uc544\ub2c8\ub77c \uce5c\ucc99\uc744 \uc81c\uc678\ud55c \ubaa8\ub4e0 \uc9c1\ubb34\uad00\ub828\uc790\uc640\ub294 \uace8\ud504\ub97c \ud560 \uc218 \uc5c6\ub3c4\ub85d \uaddc\uc81c \uc870\ud56d\uc774 \uc2e0\uc124\ub418\uae30\ub3c4 \ud588\ub2e4. \uc774\ud574\ucc2c\uc740 \uc774\uc640 \ubcc4\ub3c4\ub85c \uc2dc\ubbfc\ub2e8\uccb4 \ud559\uc0ac\ubaa8\ub85c\ubd80\ud130 \uae40\uc9c4\ud45c\uc640 \ud568\uaed8 \uace8\ud504\ud30c\ubb38\uacfc \uad00\ub828\ud558\uc5ec \ud53c\uc18c\ub2f9\ud558\uae30\ub3c4 \ud588\ub2e4. \ub610\ud55c \ucd1d\ub9ac \uc7ac\uc784\uae30\uac04\ub3d9\uc548 \uad6d\ubb34\uc704\uc6d0\ub4e4\uacfc \uacbd\uc81c\ub2e8\uccb4\uc7a5\ub4e4\uc774 \ub9cc\ub098\uc11c \uace8\ud504\uce58\uba74\uc11c \uc815\u00b7\uc7ac\uacc4 \ud604\uc548\uc744 \ub17c\ud558\ub294 \uc774\ub978\ubc14 \uace8\ud504\ud68c\ub3d9\uc774 \uc790\uc8fc \uc774\uc5b4\uc84c\ub2e4.", "sentence_answer": "2006\ub144 3\uc6d4 1\uc77c\uc5d0 \uc138\uc885\ubb38\ud654\ud68c\uad00\uc5d0\uc11c \uc5f4\ub9b0 3.1\uc808 \uae30\ub150\uc2dd\uc5d0\ub294 \ucc38\uc11d\ud558\uc9c0 \uc54a\uace0 \uc774\ud574\ucc2c\uc740 \ubd80\uc0b0\uc9c0\uc5ed \uc0c1\uacf5\uc778\ub4e4\uacfc \uace8\ud504\ubaa8\uc784 \uc744 \uac00\uc838 \ub17c\ub780\uc774 \ub418\uc5c8\ub294\ub370, \uc774\ud6c4 \uc57c\ub2f9\uc778 \ud55c\ub098\ub77c\ub2f9\uacfc \ubbfc\uc8fc\ub2f9 \uadf8\ub9ac\uace0 \uc5ec\ub2f9\uc778 \uc5f4\ub9b0\uc6b0\ub9ac\ub2f9 \ub4f1 \uc815\uce58\uad8c\uacfc \uc5b8\ub860\uc758 \uad6c\uc124\uc5d0 \uc62c\ub790\ub2e4.", "paragraph_id": "6547929-5-1", "paragraph_question": "question: \uc774\ud574\ucc2c\uc774 3.1\uc808 \uae30\ub150\uc2dd\uc5d0 \ucc38\uc11d\ud558\uc9c0 \uc54a\uace0 \uac14\ub358 \uacf3\uc740?, context: 2006\ub144 3\uc6d4 1\uc77c\uc5d0 \uc138\uc885\ubb38\ud654\ud68c\uad00\uc5d0\uc11c \uc5f4\ub9b0 3.1\uc808 \uae30\ub150\uc2dd\uc5d0\ub294 \ucc38\uc11d\ud558\uc9c0 \uc54a\uace0 \uc774\ud574\ucc2c\uc740 \ubd80\uc0b0\uc9c0\uc5ed \uc0c1\uacf5\uc778\ub4e4\uacfc \uace8\ud504\ubaa8\uc784\uc744 \uac00\uc838 \ub17c\ub780\uc774 \ub418\uc5c8\ub294\ub370, \uc774\ud6c4 \uc57c\ub2f9\uc778 \ud55c\ub098\ub77c\ub2f9\uacfc \ubbfc\uc8fc\ub2f9 \uadf8\ub9ac\uace0 \uc5ec\ub2f9\uc778 \uc5f4\ub9b0\uc6b0\ub9ac\ub2f9 \ub4f1 \uc815\uce58\uad8c\uacfc \uc5b8\ub860\uc758 \uad6c\uc124\uc5d0 \uc62c\ub790\ub2e4. 9\uc77c \ud6c4\uc778 3\uc6d4 10\uc77c\uc5d0\ub294 \uccad\uc640\ub300\uac00 \uace8\ud504 \ud30c\ubb38 \uc758\ud639\uc744 \uc9c1\uc811 \uc870\uc0ac\ud558\uae30\ub3c4 \ud588\uace0, \uc774\ud6c4 3\uc6d4 15\uc77c \ub178\ubb34\ud604 \ub300\ud1b5\ub839\uc774 \uc774\ud574\ucc2c\uc758 \ucd1d\ub9ac\uc9c1 \uc0ac\uc758\ub97c \uc218\uc6a9\ud558\uba74\uc11c \ucd1d\ub9ac\uc9c1\uc5d0\uc11c \ubb3c\ub7ec\ub098\uac8c \ub41c\ub2e4. \uc774 \uc0ac\uac74\uc758 \uc5ec\ud30c\ub85c \uacf5\uc9c1\uc0ac\ud68c \uc77c\ubd80\uc758 \ubc18\ubc1c\uc774 \uc788\uc5c8\uc73c\ub098, \uc811\ub300\uc131 \uace8\ud504 \ubfd0\ub9cc \uc544\ub2c8\ub77c \uce5c\ucc99\uc744 \uc81c\uc678\ud55c \ubaa8\ub4e0 \uc9c1\ubb34\uad00\ub828\uc790\uc640\ub294 \uace8\ud504\ub97c \ud560 \uc218 \uc5c6\ub3c4\ub85d \uaddc\uc81c \uc870\ud56d\uc774 \uc2e0\uc124\ub418\uae30\ub3c4 \ud588\ub2e4. \uc774\ud574\ucc2c\uc740 \uc774\uc640 \ubcc4\ub3c4\ub85c \uc2dc\ubbfc\ub2e8\uccb4 \ud559\uc0ac\ubaa8\ub85c\ubd80\ud130 \uae40\uc9c4\ud45c\uc640 \ud568\uaed8 \uace8\ud504\ud30c\ubb38\uacfc \uad00\ub828\ud558\uc5ec \ud53c\uc18c\ub2f9\ud558\uae30\ub3c4 \ud588\ub2e4. \ub610\ud55c \ucd1d\ub9ac \uc7ac\uc784\uae30\uac04\ub3d9\uc548 \uad6d\ubb34\uc704\uc6d0\ub4e4\uacfc \uacbd\uc81c\ub2e8\uccb4\uc7a5\ub4e4\uc774 \ub9cc\ub098\uc11c \uace8\ud504\uce58\uba74\uc11c \uc815\u00b7\uc7ac\uacc4 \ud604\uc548\uc744 \ub17c\ud558\ub294 \uc774\ub978\ubc14 \uace8\ud504\ud68c\ub3d9\uc774 \uc790\uc8fc \uc774\uc5b4\uc84c\ub2e4."} {"question": "\ud559\uc0dd\ud68c\uad00\uacfc \ub3c4\uc11c\uad00 \uc0ac\uc774\uc5d0 \uc138\uc6cc\uc838 \uc788\ub294 \ub3d9\uc0c1\uc740?", "paragraph": "\ub9e4\uc9c0\ucea0\ud37c\uc2a4\ub294 \uae40\uc6b0\uc911 \uc804 \ub300\uc6b0\uadf8\ub8f9 \ud68c\uc7a5\uc774 \uae30\uc99d\ud55c 165\ub9cc\uc5ecm \ubd80\uc9c0\uc5d0 \ucea0\ud37c\uc2a4\ub97c \uc870\uc131\ud558\uc600\ub2e4. \ub9e4\uc9c0\ucea0\ud37c\uc2a4\uc758 \uad50\ub0b4\ub85c \ub4e4\uc5b4\uc11c\uba74 \uc67c\ud3b8\uc73c\ub85c \ucca8\ub2e8\uc758\ub8cc\uae30\uae30 \ud14c\ud06c\ub178\ud0c0\uc6cc, \ud658\uacbd\uce5c\ud654\uae30\uc220\uc13c\ud130, \uae30\uc219\uc0ac\uc778 \ub9e4\uc9c0 1,2,3\ud559\uc0ac, \uc138\uc5f01,2\ud559\uc0ac, \uccad\uc5f0\ud559\uc0ac\uac00 \uc788\uace0 \ud559\uc0dd\uc2dd\ub2f9, \ud3b8\uc758\uc810, \uc6b0\ub9ac\uc740\ud589, \uc6b0\uccb4\uad6d \ub4f1\uc758 \ud3b8\uc758\uc2dc\uc124\uc774 \uc788\ub294 \uc5f0\uc138\ud504\ub77c\uc790\uac00 \uc788\uc73c\uba70 \uadf8 \uc55e\uc73c\ub85c\ub294 \uc5b8\ub355 \uc704\uc5d0 \ud604\uc6b4\uc7ac\uac00 \ubcf4\uc774\uace0 \uc606\uc73c\ub85c\ub294 \ubb34\uad81\ud654\ub3d9\uc0b0, \uc5f0\uc138\uc2a4\ud3ec\uce20\uc13c\ud130\uc640 \ub300\uc6b4\ub3d9\uc7a5, \ub9e4\uc9c0\ud638\uac00 \uc788\ub2e4. \uc740\ud589\ub098\ubb34 \uac00\ub85c\uc218\uc758 \uae34 \uc9c4\uc785\ub85c\uc5d0\ub294 \ub300\ud559\uad50\ud68c\uc640 \ub300\ud559\ubcf8\ubd80\uac74\ubb3c\uc774 \uc11c\ub85c \uc5f0\uacb0\ub418\uc5b4 \uc788\ub2e4 \uac00\ub85c\uc218\uae38\uc774 \ub05d\ub098\ub294 \uacf3\uc758 \uc55e\uc5d0 \ud559\uc0dd\ud68c\uad00\uacfc \ub3c4\uc11c\uad00\uc774 \ub9c8\uc8fc\ubcf4\uace0 \uc788\uc73c\uba70 \uadf8 \uc0ac\uc774\uc5d4 \uc2e0\ucd0c\ucea0\ubcf4\ub2e4 \ub354 \ud070 \ub3c5\uc218\ub9ac\uc0c1\uc774 \uc138\uc6cc\uc838 \uc788\ub2e4. \ub3c4\uc11c\uad00 \ub4a4\ub85c\ub294 \ubbf8\ub798\uad00, \uadf8 \uc704\ucabd \uc88c\uce21\uc5d0 \uc778\ubb38\uc608\uc220\ub300\ud559\uc774 \uc0ac\uc6a9\ud558\ub294 \uccad\uc1a1\uad00\uacfc \uc815\uacbd\ub300\ud559\uc774 \uc0ac\uc6a9\ud558\ub294 \uc815\uc758\uad00, \uc6b0\uce21\uc5d0 \uacfc\ud559\uae30\uc220\ub300\ud559\uc774 \uc788\ub294 \ucc3d\uc870\uad00, \uadf8 \ub4a4\ud3b8\uc5d0 \ubcf4\uac74\uacfc\ud559\ub300\ud559\uc774 \uc0ac\uc6a9\ud558\ub294 \ubc31\uc6b4\uad00\uc774 \uc788\ub2e4.", "answer": "\ub3c5\uc218\ub9ac\uc0c1", "sentence": "\uc740\ud589\ub098\ubb34 \uac00\ub85c\uc218\uc758 \uae34 \uc9c4\uc785\ub85c\uc5d0\ub294 \ub300\ud559\uad50\ud68c\uc640 \ub300\ud559\ubcf8\ubd80\uac74\ubb3c\uc774 \uc11c\ub85c \uc5f0\uacb0\ub418\uc5b4 \uc788\ub2e4 \uac00\ub85c\uc218\uae38\uc774 \ub05d\ub098\ub294 \uacf3\uc758 \uc55e\uc5d0 \ud559\uc0dd\ud68c\uad00\uacfc \ub3c4\uc11c\uad00\uc774 \ub9c8\uc8fc\ubcf4\uace0 \uc788\uc73c\uba70 \uadf8 \uc0ac\uc774\uc5d4 \uc2e0\ucd0c\ucea0\ubcf4\ub2e4 \ub354 \ud070 \ub3c5\uc218\ub9ac\uc0c1 \uc774 \uc138\uc6cc\uc838 \uc788\ub2e4.", "paragraph_sentence": "\ub9e4\uc9c0\ucea0\ud37c\uc2a4\ub294 \uae40\uc6b0\uc911 \uc804 \ub300\uc6b0\uadf8\ub8f9 \ud68c\uc7a5\uc774 \uae30\uc99d\ud55c 165\ub9cc\uc5ecm \ubd80\uc9c0\uc5d0 \ucea0\ud37c\uc2a4\ub97c \uc870\uc131\ud558\uc600\ub2e4. \ub9e4\uc9c0\ucea0\ud37c\uc2a4\uc758 \uad50\ub0b4\ub85c \ub4e4\uc5b4\uc11c\uba74 \uc67c\ud3b8\uc73c\ub85c \ucca8\ub2e8\uc758\ub8cc\uae30\uae30 \ud14c\ud06c\ub178\ud0c0\uc6cc, \ud658\uacbd\uce5c\ud654\uae30\uc220\uc13c\ud130, \uae30\uc219\uc0ac\uc778 \ub9e4\uc9c0 1,2,3\ud559\uc0ac, \uc138\uc5f01,2\ud559\uc0ac, \uccad\uc5f0\ud559\uc0ac\uac00 \uc788\uace0 \ud559\uc0dd\uc2dd\ub2f9, \ud3b8\uc758\uc810, \uc6b0\ub9ac\uc740\ud589, \uc6b0\uccb4\uad6d \ub4f1\uc758 \ud3b8\uc758\uc2dc\uc124\uc774 \uc788\ub294 \uc5f0\uc138\ud504\ub77c\uc790\uac00 \uc788\uc73c\uba70 \uadf8 \uc55e\uc73c\ub85c\ub294 \uc5b8\ub355 \uc704\uc5d0 \ud604\uc6b4\uc7ac\uac00 \ubcf4\uc774\uace0 \uc606\uc73c\ub85c\ub294 \ubb34\uad81\ud654\ub3d9\uc0b0, \uc5f0\uc138\uc2a4\ud3ec\uce20\uc13c\ud130\uc640 \ub300\uc6b4\ub3d9\uc7a5, \ub9e4\uc9c0\ud638\uac00 \uc788\ub2e4. \uc740\ud589\ub098\ubb34 \uac00\ub85c\uc218\uc758 \uae34 \uc9c4\uc785\ub85c\uc5d0\ub294 \ub300\ud559\uad50\ud68c\uc640 \ub300\ud559\ubcf8\ubd80\uac74\ubb3c\uc774 \uc11c\ub85c \uc5f0\uacb0\ub418\uc5b4 \uc788\ub2e4 \uac00\ub85c\uc218\uae38\uc774 \ub05d\ub098\ub294 \uacf3\uc758 \uc55e\uc5d0 \ud559\uc0dd\ud68c\uad00\uacfc \ub3c4\uc11c\uad00\uc774 \ub9c8\uc8fc\ubcf4\uace0 \uc788\uc73c\uba70 \uadf8 \uc0ac\uc774\uc5d4 \uc2e0\ucd0c\ucea0\ubcf4\ub2e4 \ub354 \ud070 \ub3c5\uc218\ub9ac\uc0c1 \uc774 \uc138\uc6cc\uc838 \uc788\ub2e4. \ub3c4\uc11c\uad00 \ub4a4\ub85c\ub294 \ubbf8\ub798\uad00, \uadf8 \uc704\ucabd \uc88c\uce21\uc5d0 \uc778\ubb38\uc608\uc220\ub300\ud559\uc774 \uc0ac\uc6a9\ud558\ub294 \uccad\uc1a1\uad00\uacfc \uc815\uacbd\ub300\ud559\uc774 \uc0ac\uc6a9\ud558\ub294 \uc815\uc758\uad00, \uc6b0\uce21\uc5d0 \uacfc\ud559\uae30\uc220\ub300\ud559\uc774 \uc788\ub294 \ucc3d\uc870\uad00, \uadf8 \ub4a4\ud3b8\uc5d0 \ubcf4\uac74\uacfc\ud559\ub300\ud559\uc774 \uc0ac\uc6a9\ud558\ub294 \ubc31\uc6b4\uad00\uc774 \uc788\ub2e4.", "paragraph_answer": "\ub9e4\uc9c0\ucea0\ud37c\uc2a4\ub294 \uae40\uc6b0\uc911 \uc804 \ub300\uc6b0\uadf8\ub8f9 \ud68c\uc7a5\uc774 \uae30\uc99d\ud55c 165\ub9cc\uc5ecm \ubd80\uc9c0\uc5d0 \ucea0\ud37c\uc2a4\ub97c \uc870\uc131\ud558\uc600\ub2e4. \ub9e4\uc9c0\ucea0\ud37c\uc2a4\uc758 \uad50\ub0b4\ub85c \ub4e4\uc5b4\uc11c\uba74 \uc67c\ud3b8\uc73c\ub85c \ucca8\ub2e8\uc758\ub8cc\uae30\uae30 \ud14c\ud06c\ub178\ud0c0\uc6cc, \ud658\uacbd\uce5c\ud654\uae30\uc220\uc13c\ud130, \uae30\uc219\uc0ac\uc778 \ub9e4\uc9c0 1,2,3\ud559\uc0ac, \uc138\uc5f01,2\ud559\uc0ac, \uccad\uc5f0\ud559\uc0ac\uac00 \uc788\uace0 \ud559\uc0dd\uc2dd\ub2f9, \ud3b8\uc758\uc810, \uc6b0\ub9ac\uc740\ud589, \uc6b0\uccb4\uad6d \ub4f1\uc758 \ud3b8\uc758\uc2dc\uc124\uc774 \uc788\ub294 \uc5f0\uc138\ud504\ub77c\uc790\uac00 \uc788\uc73c\uba70 \uadf8 \uc55e\uc73c\ub85c\ub294 \uc5b8\ub355 \uc704\uc5d0 \ud604\uc6b4\uc7ac\uac00 \ubcf4\uc774\uace0 \uc606\uc73c\ub85c\ub294 \ubb34\uad81\ud654\ub3d9\uc0b0, \uc5f0\uc138\uc2a4\ud3ec\uce20\uc13c\ud130\uc640 \ub300\uc6b4\ub3d9\uc7a5, \ub9e4\uc9c0\ud638\uac00 \uc788\ub2e4. \uc740\ud589\ub098\ubb34 \uac00\ub85c\uc218\uc758 \uae34 \uc9c4\uc785\ub85c\uc5d0\ub294 \ub300\ud559\uad50\ud68c\uc640 \ub300\ud559\ubcf8\ubd80\uac74\ubb3c\uc774 \uc11c\ub85c \uc5f0\uacb0\ub418\uc5b4 \uc788\ub2e4 \uac00\ub85c\uc218\uae38\uc774 \ub05d\ub098\ub294 \uacf3\uc758 \uc55e\uc5d0 \ud559\uc0dd\ud68c\uad00\uacfc \ub3c4\uc11c\uad00\uc774 \ub9c8\uc8fc\ubcf4\uace0 \uc788\uc73c\uba70 \uadf8 \uc0ac\uc774\uc5d4 \uc2e0\ucd0c\ucea0\ubcf4\ub2e4 \ub354 \ud070 \ub3c5\uc218\ub9ac\uc0c1 \uc774 \uc138\uc6cc\uc838 \uc788\ub2e4. \ub3c4\uc11c\uad00 \ub4a4\ub85c\ub294 \ubbf8\ub798\uad00, \uadf8 \uc704\ucabd \uc88c\uce21\uc5d0 \uc778\ubb38\uc608\uc220\ub300\ud559\uc774 \uc0ac\uc6a9\ud558\ub294 \uccad\uc1a1\uad00\uacfc \uc815\uacbd\ub300\ud559\uc774 \uc0ac\uc6a9\ud558\ub294 \uc815\uc758\uad00, \uc6b0\uce21\uc5d0 \uacfc\ud559\uae30\uc220\ub300\ud559\uc774 \uc788\ub294 \ucc3d\uc870\uad00, \uadf8 \ub4a4\ud3b8\uc5d0 \ubcf4\uac74\uacfc\ud559\ub300\ud559\uc774 \uc0ac\uc6a9\ud558\ub294 \ubc31\uc6b4\uad00\uc774 \uc788\ub2e4.", "sentence_answer": "\uc740\ud589\ub098\ubb34 \uac00\ub85c\uc218\uc758 \uae34 \uc9c4\uc785\ub85c\uc5d0\ub294 \ub300\ud559\uad50\ud68c\uc640 \ub300\ud559\ubcf8\ubd80\uac74\ubb3c\uc774 \uc11c\ub85c \uc5f0\uacb0\ub418\uc5b4 \uc788\ub2e4 \uac00\ub85c\uc218\uae38\uc774 \ub05d\ub098\ub294 \uacf3\uc758 \uc55e\uc5d0 \ud559\uc0dd\ud68c\uad00\uacfc \ub3c4\uc11c\uad00\uc774 \ub9c8\uc8fc\ubcf4\uace0 \uc788\uc73c\uba70 \uadf8 \uc0ac\uc774\uc5d4 \uc2e0\ucd0c\ucea0\ubcf4\ub2e4 \ub354 \ud070 \ub3c5\uc218\ub9ac\uc0c1 \uc774 \uc138\uc6cc\uc838 \uc788\ub2e4.", "paragraph_id": "6458470-9-1", "paragraph_question": "question: \ud559\uc0dd\ud68c\uad00\uacfc \ub3c4\uc11c\uad00 \uc0ac\uc774\uc5d0 \uc138\uc6cc\uc838 \uc788\ub294 \ub3d9\uc0c1\uc740?, context: \ub9e4\uc9c0\ucea0\ud37c\uc2a4\ub294 \uae40\uc6b0\uc911 \uc804 \ub300\uc6b0\uadf8\ub8f9 \ud68c\uc7a5\uc774 \uae30\uc99d\ud55c 165\ub9cc\uc5ecm \ubd80\uc9c0\uc5d0 \ucea0\ud37c\uc2a4\ub97c \uc870\uc131\ud558\uc600\ub2e4. \ub9e4\uc9c0\ucea0\ud37c\uc2a4\uc758 \uad50\ub0b4\ub85c \ub4e4\uc5b4\uc11c\uba74 \uc67c\ud3b8\uc73c\ub85c \ucca8\ub2e8\uc758\ub8cc\uae30\uae30 \ud14c\ud06c\ub178\ud0c0\uc6cc, \ud658\uacbd\uce5c\ud654\uae30\uc220\uc13c\ud130, \uae30\uc219\uc0ac\uc778 \ub9e4\uc9c0 1,2,3\ud559\uc0ac, \uc138\uc5f01,2\ud559\uc0ac, \uccad\uc5f0\ud559\uc0ac\uac00 \uc788\uace0 \ud559\uc0dd\uc2dd\ub2f9, \ud3b8\uc758\uc810, \uc6b0\ub9ac\uc740\ud589, \uc6b0\uccb4\uad6d \ub4f1\uc758 \ud3b8\uc758\uc2dc\uc124\uc774 \uc788\ub294 \uc5f0\uc138\ud504\ub77c\uc790\uac00 \uc788\uc73c\uba70 \uadf8 \uc55e\uc73c\ub85c\ub294 \uc5b8\ub355 \uc704\uc5d0 \ud604\uc6b4\uc7ac\uac00 \ubcf4\uc774\uace0 \uc606\uc73c\ub85c\ub294 \ubb34\uad81\ud654\ub3d9\uc0b0, \uc5f0\uc138\uc2a4\ud3ec\uce20\uc13c\ud130\uc640 \ub300\uc6b4\ub3d9\uc7a5, \ub9e4\uc9c0\ud638\uac00 \uc788\ub2e4. \uc740\ud589\ub098\ubb34 \uac00\ub85c\uc218\uc758 \uae34 \uc9c4\uc785\ub85c\uc5d0\ub294 \ub300\ud559\uad50\ud68c\uc640 \ub300\ud559\ubcf8\ubd80\uac74\ubb3c\uc774 \uc11c\ub85c \uc5f0\uacb0\ub418\uc5b4 \uc788\ub2e4 \uac00\ub85c\uc218\uae38\uc774 \ub05d\ub098\ub294 \uacf3\uc758 \uc55e\uc5d0 \ud559\uc0dd\ud68c\uad00\uacfc \ub3c4\uc11c\uad00\uc774 \ub9c8\uc8fc\ubcf4\uace0 \uc788\uc73c\uba70 \uadf8 \uc0ac\uc774\uc5d4 \uc2e0\ucd0c\ucea0\ubcf4\ub2e4 \ub354 \ud070 \ub3c5\uc218\ub9ac\uc0c1\uc774 \uc138\uc6cc\uc838 \uc788\ub2e4. \ub3c4\uc11c\uad00 \ub4a4\ub85c\ub294 \ubbf8\ub798\uad00, \uadf8 \uc704\ucabd \uc88c\uce21\uc5d0 \uc778\ubb38\uc608\uc220\ub300\ud559\uc774 \uc0ac\uc6a9\ud558\ub294 \uccad\uc1a1\uad00\uacfc \uc815\uacbd\ub300\ud559\uc774 \uc0ac\uc6a9\ud558\ub294 \uc815\uc758\uad00, \uc6b0\uce21\uc5d0 \uacfc\ud559\uae30\uc220\ub300\ud559\uc774 \uc788\ub294 \ucc3d\uc870\uad00, \uadf8 \ub4a4\ud3b8\uc5d0 \ubcf4\uac74\uacfc\ud559\ub300\ud559\uc774 \uc0ac\uc6a9\ud558\ub294 \ubc31\uc6b4\uad00\uc774 \uc788\ub2e4."} {"question": "\uc1a1\uc2e0\uc601\uc758 \ud2b8\ub808\uc774\ub4dc \ub2f9\uc2dc \uc624\uac04 \ud604\uae08\uc740 \uc5bc\ub9c8\uc778\uac00?", "paragraph": "2010\ub144 3\uc6d4\uc5d0 \uc9c4\ud589\ub41c \ub9c8\uc77c\uc601\uc758 \ud604\uae08 \ud2b8\ub808\uc774\ub4dc \uc774\ud6c4 \ub125\uc13c \ud788\uc5b4\ub85c\uc988\ub294 \uacf5\uc2dd\uc801\uc778 \ud604\uae08 \ud2b8\ub808\uc774\ub4dc\ub97c \uc9c4\ud589\ud558\uc9c0 \uc54a\uc558\uc9c0\ub9cc, \uade0\ud615\uc774 \ub9de\uc9c0 \uc54a\ub294 \ud2b8\ub808\uc774\ub4dc\ub97c \uc5ec\ub7ec \ucc28\ub840 \uc9c4\ud589\ud558\uc600\uae30 \ub54c\ubb38\uc5d0 \uc2e4\uc81c\ub85c\ub294 \ud604\uae08\uc774 \ud3ec\ud568\ub41c \ud2b8\ub808\uc774\ub4dc\uc77c \uac83\uc774\ub77c\uace0 \ub9ce\uc740 \ud32c\ub4e4\uacfc \uc804\ubb38\uac00\ub4e4\uc774 \uc608\uc0c1\ud558\uc600\ub2e4. \uc2e4\uc81c\ub85c 2011\ub144 11\uc6d4, \uc911\uc559\uc77c\ubcf4\ub294 \uae30\uc0ac\ub97c \ud1b5\ud574 \uc1a1\uc2e0\uc601 \ud2b8\ub808\uc774\ub4dc \ub2f9\uc2dc 15\uc5b5 \uc6d0\uc774 \ud3ec\ud568\ub418\uc5c8\ub2e4\uace0 \uc5b8\uae09\ud558\uc600\uace0, \uc774\ud6c4 2018\ub144 3\uc6d4\uc5d0\ub294 \uc1a1\uc2e0\uc601\uc774 \uc740\ud1f4 \uc774\ud6c4 \ubaa8\uad50\uc778 \uace0\ub824\ub300\ud559\uad50\uc758 \uc7a1\uc9c0 \u300aSPORTS KU\u300b\uc640 \uc9c4\ud589\ub41c \uc778\ud130\ubdf0\uc5d0\uc11c \ud604\uae08 15\uc5b5 \uc6d0\uc774 \ud3ec\ud568\ub418\uc5c8\ub2e4\uace0 \uc5b8\uae09\ud558\uc600\ub2e4. 2017\ub144 7\uc6d4\uc5d0 \uc9c4\ud589\ub41c \ub450 \ucc28\ub840\uc758 \ud2b8\ub808\uc774\ub4dc\uc5d0\uc11c\ub3c4 \ud604\uae08\uc774 \ud3ec\ud568\ub418\uc5c8\ub2e4\ub294 \uc758\ud639\uc774 \uc81c\uae30\ub418\uc5c8\ub294\ub370, \ub125\uc13c\uc758 \uace0\ud615\uc6b1 \ub2e8\uc7a5\uc740 \uadf8\uc640 \uad00\ub828\ub41c \uc9c8\ubb38\uc5d0 \ud604\uae08\uc774 \ud3ec\ud568\ub418\uc9c0 \uc54a\uc558\ub2e4\uace0 \uc790\uc2e0\uc788\uac8c \ub9d0\ud560 \uc218 \uc788\ub2e4\uba74\uc11c \uad00\ub828\ub41c \uc758\ud639\uc744 \ubd80\uc778\ud558\uc600\ub2e4.", "answer": "15\uc5b5 \uc6d0", "sentence": "\uc2e4\uc81c\ub85c 2011\ub144 11\uc6d4, \uc911\uc559\uc77c\ubcf4\ub294 \uae30\uc0ac\ub97c \ud1b5\ud574 \uc1a1\uc2e0\uc601 \ud2b8\ub808\uc774\ub4dc \ub2f9\uc2dc 15\uc5b5 \uc6d0 \uc774 \ud3ec\ud568\ub418\uc5c8\ub2e4\uace0 \uc5b8\uae09\ud558\uc600\uace0, \uc774\ud6c4 2018\ub144 3\uc6d4\uc5d0\ub294 \uc1a1\uc2e0\uc601\uc774 \uc740\ud1f4 \uc774\ud6c4 \ubaa8\uad50\uc778 \uace0\ub824\ub300\ud559\uad50\uc758 \uc7a1\uc9c0 \u300aSPORTS KU\u300b\uc640 \uc9c4\ud589\ub41c \uc778\ud130\ubdf0\uc5d0\uc11c \ud604\uae08 15\uc5b5 \uc6d0\uc774 \ud3ec\ud568\ub418\uc5c8\ub2e4\uace0 \uc5b8\uae09\ud558\uc600\ub2e4.", "paragraph_sentence": "2010\ub144 3\uc6d4\uc5d0 \uc9c4\ud589\ub41c \ub9c8\uc77c\uc601\uc758 \ud604\uae08 \ud2b8\ub808\uc774\ub4dc \uc774\ud6c4 \ub125\uc13c \ud788\uc5b4\ub85c\uc988\ub294 \uacf5\uc2dd\uc801\uc778 \ud604\uae08 \ud2b8\ub808\uc774\ub4dc\ub97c \uc9c4\ud589\ud558\uc9c0 \uc54a\uc558\uc9c0\ub9cc, \uade0\ud615\uc774 \ub9de\uc9c0 \uc54a\ub294 \ud2b8\ub808\uc774\ub4dc\ub97c \uc5ec\ub7ec \ucc28\ub840 \uc9c4\ud589\ud558\uc600\uae30 \ub54c\ubb38\uc5d0 \uc2e4\uc81c\ub85c\ub294 \ud604\uae08\uc774 \ud3ec\ud568\ub41c \ud2b8\ub808\uc774\ub4dc\uc77c \uac83\uc774\ub77c\uace0 \ub9ce\uc740 \ud32c\ub4e4\uacfc \uc804\ubb38\uac00\ub4e4\uc774 \uc608\uc0c1\ud558\uc600\ub2e4. \uc2e4\uc81c\ub85c 2011\ub144 11\uc6d4, \uc911\uc559\uc77c\ubcf4\ub294 \uae30\uc0ac\ub97c \ud1b5\ud574 \uc1a1\uc2e0\uc601 \ud2b8\ub808\uc774\ub4dc \ub2f9\uc2dc 15\uc5b5 \uc6d0 \uc774 \ud3ec\ud568\ub418\uc5c8\ub2e4\uace0 \uc5b8\uae09\ud558\uc600\uace0, \uc774\ud6c4 2018\ub144 3\uc6d4\uc5d0\ub294 \uc1a1\uc2e0\uc601\uc774 \uc740\ud1f4 \uc774\ud6c4 \ubaa8\uad50\uc778 \uace0\ub824\ub300\ud559\uad50\uc758 \uc7a1\uc9c0 \u300aSPORTS KU\u300b\uc640 \uc9c4\ud589\ub41c \uc778\ud130\ubdf0\uc5d0\uc11c \ud604\uae08 15\uc5b5 \uc6d0\uc774 \ud3ec\ud568\ub418\uc5c8\ub2e4\uace0 \uc5b8\uae09\ud558\uc600\ub2e4. 2017\ub144 7\uc6d4\uc5d0 \uc9c4\ud589\ub41c \ub450 \ucc28\ub840\uc758 \ud2b8\ub808\uc774\ub4dc\uc5d0\uc11c\ub3c4 \ud604\uae08\uc774 \ud3ec\ud568\ub418\uc5c8\ub2e4\ub294 \uc758\ud639\uc774 \uc81c\uae30\ub418\uc5c8\ub294\ub370, \ub125\uc13c\uc758 \uace0\ud615\uc6b1 \ub2e8\uc7a5\uc740 \uadf8\uc640 \uad00\ub828\ub41c \uc9c8\ubb38\uc5d0 \ud604\uae08\uc774 \ud3ec\ud568\ub418\uc9c0 \uc54a\uc558\ub2e4\uace0 \uc790\uc2e0\uc788\uac8c \ub9d0\ud560 \uc218 \uc788\ub2e4\uba74\uc11c \uad00\ub828\ub41c \uc758\ud639\uc744 \ubd80\uc778\ud558\uc600\ub2e4.", "paragraph_answer": "2010\ub144 3\uc6d4\uc5d0 \uc9c4\ud589\ub41c \ub9c8\uc77c\uc601\uc758 \ud604\uae08 \ud2b8\ub808\uc774\ub4dc \uc774\ud6c4 \ub125\uc13c \ud788\uc5b4\ub85c\uc988\ub294 \uacf5\uc2dd\uc801\uc778 \ud604\uae08 \ud2b8\ub808\uc774\ub4dc\ub97c \uc9c4\ud589\ud558\uc9c0 \uc54a\uc558\uc9c0\ub9cc, \uade0\ud615\uc774 \ub9de\uc9c0 \uc54a\ub294 \ud2b8\ub808\uc774\ub4dc\ub97c \uc5ec\ub7ec \ucc28\ub840 \uc9c4\ud589\ud558\uc600\uae30 \ub54c\ubb38\uc5d0 \uc2e4\uc81c\ub85c\ub294 \ud604\uae08\uc774 \ud3ec\ud568\ub41c \ud2b8\ub808\uc774\ub4dc\uc77c \uac83\uc774\ub77c\uace0 \ub9ce\uc740 \ud32c\ub4e4\uacfc \uc804\ubb38\uac00\ub4e4\uc774 \uc608\uc0c1\ud558\uc600\ub2e4. \uc2e4\uc81c\ub85c 2011\ub144 11\uc6d4, \uc911\uc559\uc77c\ubcf4\ub294 \uae30\uc0ac\ub97c \ud1b5\ud574 \uc1a1\uc2e0\uc601 \ud2b8\ub808\uc774\ub4dc \ub2f9\uc2dc 15\uc5b5 \uc6d0 \uc774 \ud3ec\ud568\ub418\uc5c8\ub2e4\uace0 \uc5b8\uae09\ud558\uc600\uace0, \uc774\ud6c4 2018\ub144 3\uc6d4\uc5d0\ub294 \uc1a1\uc2e0\uc601\uc774 \uc740\ud1f4 \uc774\ud6c4 \ubaa8\uad50\uc778 \uace0\ub824\ub300\ud559\uad50\uc758 \uc7a1\uc9c0 \u300aSPORTS KU\u300b\uc640 \uc9c4\ud589\ub41c \uc778\ud130\ubdf0\uc5d0\uc11c \ud604\uae08 15\uc5b5 \uc6d0\uc774 \ud3ec\ud568\ub418\uc5c8\ub2e4\uace0 \uc5b8\uae09\ud558\uc600\ub2e4. 2017\ub144 7\uc6d4\uc5d0 \uc9c4\ud589\ub41c \ub450 \ucc28\ub840\uc758 \ud2b8\ub808\uc774\ub4dc\uc5d0\uc11c\ub3c4 \ud604\uae08\uc774 \ud3ec\ud568\ub418\uc5c8\ub2e4\ub294 \uc758\ud639\uc774 \uc81c\uae30\ub418\uc5c8\ub294\ub370, \ub125\uc13c\uc758 \uace0\ud615\uc6b1 \ub2e8\uc7a5\uc740 \uadf8\uc640 \uad00\ub828\ub41c \uc9c8\ubb38\uc5d0 \ud604\uae08\uc774 \ud3ec\ud568\ub418\uc9c0 \uc54a\uc558\ub2e4\uace0 \uc790\uc2e0\uc788\uac8c \ub9d0\ud560 \uc218 \uc788\ub2e4\uba74\uc11c \uad00\ub828\ub41c \uc758\ud639\uc744 \ubd80\uc778\ud558\uc600\ub2e4.", "sentence_answer": "\uc2e4\uc81c\ub85c 2011\ub144 11\uc6d4, \uc911\uc559\uc77c\ubcf4\ub294 \uae30\uc0ac\ub97c \ud1b5\ud574 \uc1a1\uc2e0\uc601 \ud2b8\ub808\uc774\ub4dc \ub2f9\uc2dc 15\uc5b5 \uc6d0 \uc774 \ud3ec\ud568\ub418\uc5c8\ub2e4\uace0 \uc5b8\uae09\ud558\uc600\uace0, \uc774\ud6c4 2018\ub144 3\uc6d4\uc5d0\ub294 \uc1a1\uc2e0\uc601\uc774 \uc740\ud1f4 \uc774\ud6c4 \ubaa8\uad50\uc778 \uace0\ub824\ub300\ud559\uad50\uc758 \uc7a1\uc9c0 \u300aSPORTS KU\u300b\uc640 \uc9c4\ud589\ub41c \uc778\ud130\ubdf0\uc5d0\uc11c \ud604\uae08 15\uc5b5 \uc6d0\uc774 \ud3ec\ud568\ub418\uc5c8\ub2e4\uace0 \uc5b8\uae09\ud558\uc600\ub2e4.", "paragraph_id": "6488904-19-0", "paragraph_question": "question: \uc1a1\uc2e0\uc601\uc758 \ud2b8\ub808\uc774\ub4dc \ub2f9\uc2dc \uc624\uac04 \ud604\uae08\uc740 \uc5bc\ub9c8\uc778\uac00?, context: 2010\ub144 3\uc6d4\uc5d0 \uc9c4\ud589\ub41c \ub9c8\uc77c\uc601\uc758 \ud604\uae08 \ud2b8\ub808\uc774\ub4dc \uc774\ud6c4 \ub125\uc13c \ud788\uc5b4\ub85c\uc988\ub294 \uacf5\uc2dd\uc801\uc778 \ud604\uae08 \ud2b8\ub808\uc774\ub4dc\ub97c \uc9c4\ud589\ud558\uc9c0 \uc54a\uc558\uc9c0\ub9cc, \uade0\ud615\uc774 \ub9de\uc9c0 \uc54a\ub294 \ud2b8\ub808\uc774\ub4dc\ub97c \uc5ec\ub7ec \ucc28\ub840 \uc9c4\ud589\ud558\uc600\uae30 \ub54c\ubb38\uc5d0 \uc2e4\uc81c\ub85c\ub294 \ud604\uae08\uc774 \ud3ec\ud568\ub41c \ud2b8\ub808\uc774\ub4dc\uc77c \uac83\uc774\ub77c\uace0 \ub9ce\uc740 \ud32c\ub4e4\uacfc \uc804\ubb38\uac00\ub4e4\uc774 \uc608\uc0c1\ud558\uc600\ub2e4. \uc2e4\uc81c\ub85c 2011\ub144 11\uc6d4, \uc911\uc559\uc77c\ubcf4\ub294 \uae30\uc0ac\ub97c \ud1b5\ud574 \uc1a1\uc2e0\uc601 \ud2b8\ub808\uc774\ub4dc \ub2f9\uc2dc 15\uc5b5 \uc6d0\uc774 \ud3ec\ud568\ub418\uc5c8\ub2e4\uace0 \uc5b8\uae09\ud558\uc600\uace0, \uc774\ud6c4 2018\ub144 3\uc6d4\uc5d0\ub294 \uc1a1\uc2e0\uc601\uc774 \uc740\ud1f4 \uc774\ud6c4 \ubaa8\uad50\uc778 \uace0\ub824\ub300\ud559\uad50\uc758 \uc7a1\uc9c0 \u300aSPORTS KU\u300b\uc640 \uc9c4\ud589\ub41c \uc778\ud130\ubdf0\uc5d0\uc11c \ud604\uae08 15\uc5b5 \uc6d0\uc774 \ud3ec\ud568\ub418\uc5c8\ub2e4\uace0 \uc5b8\uae09\ud558\uc600\ub2e4. 2017\ub144 7\uc6d4\uc5d0 \uc9c4\ud589\ub41c \ub450 \ucc28\ub840\uc758 \ud2b8\ub808\uc774\ub4dc\uc5d0\uc11c\ub3c4 \ud604\uae08\uc774 \ud3ec\ud568\ub418\uc5c8\ub2e4\ub294 \uc758\ud639\uc774 \uc81c\uae30\ub418\uc5c8\ub294\ub370, \ub125\uc13c\uc758 \uace0\ud615\uc6b1 \ub2e8\uc7a5\uc740 \uadf8\uc640 \uad00\ub828\ub41c \uc9c8\ubb38\uc5d0 \ud604\uae08\uc774 \ud3ec\ud568\ub418\uc9c0 \uc54a\uc558\ub2e4\uace0 \uc790\uc2e0\uc788\uac8c \ub9d0\ud560 \uc218 \uc788\ub2e4\uba74\uc11c \uad00\ub828\ub41c \uc758\ud639\uc744 \ubd80\uc778\ud558\uc600\ub2e4."} {"question": "\ucc9c\ud669\uc758 \uce59\ud5c8\uc5c6\uc774 \uc9c4\ud589\ub41c \uc774\uc6b0\uc758 \ud63c\uc57d\uc5d0 \ubc18\ubc1c\ud55c \ub2f9\uc2dc \uc655\uacf5\uc871\uc758 \uc0ac\ubb34\ub97c \uad00\uc7a5\ud558\ub358 \uc0ac\ub78c\uc758 \uc774\ub984\uc740?", "paragraph": "\uc758\uce5c\uc655\uc740 \uc544\ub4e4\uc774 \uc77c\ubcf8\uc778\uacfc \uacb0\ud63c\ud558\ub294 \uac83\uc744 \uc6d0\ud558\uc9c0 \uc54a\uc558\uace0 \uc774\uc6b0\ub3c4 \uc870\uc120\uc778\uacfc\uc758 \uacb0\ud63c\uc744 \uace0\uc9d1\ud558\uc600\ub2e4. \uc77c\ubcf8\uc778 \uc2e0\ubd80\ub97c \uac70\ubd80\ud558\uae30 \uc704\ud55c \ubc29\uc548\uc744 \ubaa8\uc0c9\ud558\ub358 \uc911\uc5d0 \uc758\uce5c\uc655\uacfc \uae34\ubc00\ud55c \uad00\uacc4\uc600\ub358 \ubc15\uc601\ud6a8\uc758 \uc11c\uc190\ub140 \ubc15\ucc2c\uc8fc\uac00 \uc774\uc6b0\uc758 \ubc30\uc6b0\uc790\ub85c \ubb3c\ub9dd\uc5d0 \uc624\ub974\uace0 \ud63c\uc57d\uc774 \uc774\ub8e8\uc5b4\uc84c\ub2e4. \uc774\uc6b0\uc758 \uc591\ubaa8 \uad11\uc0b0 \uae40\uc528\ub294 \ubc15\uc601\ud6a8\uac00 \ub0b4\ubb34\ub300\uc2e0\uc73c\ub85c \uc7ac\uc784\ud558\ub358 1895\ub144\uc5d0 \ubc95\ubb34\ud611\ud310 \uae40\ud559\uc6b0 \uc554\uc0b4\uacfc \ubaa8\ubc18 \ud610\uc758\ub85c \uc591\ubd80 \uc774\uc900\uc6a9\uc744 \uc8fd\uc774\ub824 \ud558\uc600\uae30 \ub54c\ubb38\uc5d0 \ubc15\ucc2c\uc8fc\ub97c \ubabb\ub9c8\ub545\ud558\uac8c \uc5ec\uacbc\ub2e4. \uc774\uc6b0\ub294 \ubc15\ucc2c\uc8fc\uc5d0\uac8c \uc0ac\uc8fc\ub2e8\uc790\uc640 \uc57d\ud63c\ubc18\uc9c0\ub97c \ubcf4\ub0b8 \ud6c4\uc5d0 \ub2f9\uc2dc \uc774\uc655\uc9c1 \uc7a5\uad00 \ud55c\ucc3d\uc218\uc5d0\uac8c\ub294 \ud63c\uc57d\uc774 \uc131\ub9bd\ub418\uc5c8\ub2e4\uace0 \uc77c\ubc29\uc801\uc73c\ub85c \ud1b5\uace0\ud558\uc600\ub2e4. \uc655\uacf5\uc871\uc758 \uc0ac\ubb34\ub97c \uad00\uc7a5\ud558\ub358 \uc774\uc655\uc9c1\uc740 \ubc15\ucc2c\uc8fc\uc640\uc758 \ud63c\uc57d\uc774 \ucc9c\ud669\uc758 \uce59\ud5c8\uc5c6\uc774 \uc9c4\ud589\ub418\uc5c8\ub2e4\uace0 \ubc18\ubc1c\ud558\uba70 \uc77c\ubcf8 \ud669\uc871\uacfc \uacb0\ud63c\ud558\ub77c\ub294 \uc555\ub825\uc744 \ud589\uc0ac\ud558\uc600\ub2e4.", "answer": "\uc774\uc655\uc9c1", "sentence": "\uc774\uc6b0\ub294 \ubc15\ucc2c\uc8fc\uc5d0\uac8c \uc0ac\uc8fc\ub2e8\uc790\uc640 \uc57d\ud63c\ubc18\uc9c0\ub97c \ubcf4\ub0b8 \ud6c4\uc5d0 \ub2f9\uc2dc \uc774\uc655\uc9c1 \uc7a5\uad00 \ud55c\ucc3d\uc218\uc5d0\uac8c\ub294 \ud63c\uc57d\uc774 \uc131\ub9bd\ub418\uc5c8\ub2e4\uace0 \uc77c\ubc29\uc801\uc73c\ub85c \ud1b5\uace0\ud558\uc600\ub2e4.", "paragraph_sentence": "\uc758\uce5c\uc655\uc740 \uc544\ub4e4\uc774 \uc77c\ubcf8\uc778\uacfc \uacb0\ud63c\ud558\ub294 \uac83\uc744 \uc6d0\ud558\uc9c0 \uc54a\uc558\uace0 \uc774\uc6b0\ub3c4 \uc870\uc120\uc778\uacfc\uc758 \uacb0\ud63c\uc744 \uace0\uc9d1\ud558\uc600\ub2e4. \uc77c\ubcf8\uc778 \uc2e0\ubd80\ub97c \uac70\ubd80\ud558\uae30 \uc704\ud55c \ubc29\uc548\uc744 \ubaa8\uc0c9\ud558\ub358 \uc911\uc5d0 \uc758\uce5c\uc655\uacfc \uae34\ubc00\ud55c \uad00\uacc4\uc600\ub358 \ubc15\uc601\ud6a8\uc758 \uc11c\uc190\ub140 \ubc15\ucc2c\uc8fc\uac00 \uc774\uc6b0\uc758 \ubc30\uc6b0\uc790\ub85c \ubb3c\ub9dd\uc5d0 \uc624\ub974\uace0 \ud63c\uc57d\uc774 \uc774\ub8e8\uc5b4\uc84c\ub2e4. \uc774\uc6b0\uc758 \uc591\ubaa8 \uad11\uc0b0 \uae40\uc528\ub294 \ubc15\uc601\ud6a8\uac00 \ub0b4\ubb34\ub300\uc2e0\uc73c\ub85c \uc7ac\uc784\ud558\ub358 1895\ub144\uc5d0 \ubc95\ubb34\ud611\ud310 \uae40\ud559\uc6b0 \uc554\uc0b4\uacfc \ubaa8\ubc18 \ud610\uc758\ub85c \uc591\ubd80 \uc774\uc900\uc6a9\uc744 \uc8fd\uc774\ub824 \ud558\uc600\uae30 \ub54c\ubb38\uc5d0 \ubc15\ucc2c\uc8fc\ub97c \ubabb\ub9c8\ub545\ud558\uac8c \uc5ec\uacbc\ub2e4. \uc774\uc6b0\ub294 \ubc15\ucc2c\uc8fc\uc5d0\uac8c \uc0ac\uc8fc\ub2e8\uc790\uc640 \uc57d\ud63c\ubc18\uc9c0\ub97c \ubcf4\ub0b8 \ud6c4\uc5d0 \ub2f9\uc2dc \uc774\uc655\uc9c1 \uc7a5\uad00 \ud55c\ucc3d\uc218\uc5d0\uac8c\ub294 \ud63c\uc57d\uc774 \uc131\ub9bd\ub418\uc5c8\ub2e4\uace0 \uc77c\ubc29\uc801\uc73c\ub85c \ud1b5\uace0\ud558\uc600\ub2e4. \uc655\uacf5\uc871\uc758 \uc0ac\ubb34\ub97c \uad00\uc7a5\ud558\ub358 \uc774\uc655\uc9c1\uc740 \ubc15\ucc2c\uc8fc\uc640\uc758 \ud63c\uc57d\uc774 \ucc9c\ud669\uc758 \uce59\ud5c8\uc5c6\uc774 \uc9c4\ud589\ub418\uc5c8\ub2e4\uace0 \ubc18\ubc1c\ud558\uba70 \uc77c\ubcf8 \ud669\uc871\uacfc \uacb0\ud63c\ud558\ub77c\ub294 \uc555\ub825\uc744 \ud589\uc0ac\ud558\uc600\ub2e4.", "paragraph_answer": "\uc758\uce5c\uc655\uc740 \uc544\ub4e4\uc774 \uc77c\ubcf8\uc778\uacfc \uacb0\ud63c\ud558\ub294 \uac83\uc744 \uc6d0\ud558\uc9c0 \uc54a\uc558\uace0 \uc774\uc6b0\ub3c4 \uc870\uc120\uc778\uacfc\uc758 \uacb0\ud63c\uc744 \uace0\uc9d1\ud558\uc600\ub2e4. \uc77c\ubcf8\uc778 \uc2e0\ubd80\ub97c \uac70\ubd80\ud558\uae30 \uc704\ud55c \ubc29\uc548\uc744 \ubaa8\uc0c9\ud558\ub358 \uc911\uc5d0 \uc758\uce5c\uc655\uacfc \uae34\ubc00\ud55c \uad00\uacc4\uc600\ub358 \ubc15\uc601\ud6a8\uc758 \uc11c\uc190\ub140 \ubc15\ucc2c\uc8fc\uac00 \uc774\uc6b0\uc758 \ubc30\uc6b0\uc790\ub85c \ubb3c\ub9dd\uc5d0 \uc624\ub974\uace0 \ud63c\uc57d\uc774 \uc774\ub8e8\uc5b4\uc84c\ub2e4. \uc774\uc6b0\uc758 \uc591\ubaa8 \uad11\uc0b0 \uae40\uc528\ub294 \ubc15\uc601\ud6a8\uac00 \ub0b4\ubb34\ub300\uc2e0\uc73c\ub85c \uc7ac\uc784\ud558\ub358 1895\ub144\uc5d0 \ubc95\ubb34\ud611\ud310 \uae40\ud559\uc6b0 \uc554\uc0b4\uacfc \ubaa8\ubc18 \ud610\uc758\ub85c \uc591\ubd80 \uc774\uc900\uc6a9\uc744 \uc8fd\uc774\ub824 \ud558\uc600\uae30 \ub54c\ubb38\uc5d0 \ubc15\ucc2c\uc8fc\ub97c \ubabb\ub9c8\ub545\ud558\uac8c \uc5ec\uacbc\ub2e4. \uc774\uc6b0\ub294 \ubc15\ucc2c\uc8fc\uc5d0\uac8c \uc0ac\uc8fc\ub2e8\uc790\uc640 \uc57d\ud63c\ubc18\uc9c0\ub97c \ubcf4\ub0b8 \ud6c4\uc5d0 \ub2f9\uc2dc \uc774\uc655\uc9c1 \uc7a5\uad00 \ud55c\ucc3d\uc218\uc5d0\uac8c\ub294 \ud63c\uc57d\uc774 \uc131\ub9bd\ub418\uc5c8\ub2e4\uace0 \uc77c\ubc29\uc801\uc73c\ub85c \ud1b5\uace0\ud558\uc600\ub2e4. \uc655\uacf5\uc871\uc758 \uc0ac\ubb34\ub97c \uad00\uc7a5\ud558\ub358 \uc774\uc655\uc9c1\uc740 \ubc15\ucc2c\uc8fc\uc640\uc758 \ud63c\uc57d\uc774 \ucc9c\ud669\uc758 \uce59\ud5c8\uc5c6\uc774 \uc9c4\ud589\ub418\uc5c8\ub2e4\uace0 \ubc18\ubc1c\ud558\uba70 \uc77c\ubcf8 \ud669\uc871\uacfc \uacb0\ud63c\ud558\ub77c\ub294 \uc555\ub825\uc744 \ud589\uc0ac\ud558\uc600\ub2e4.", "sentence_answer": "\uc774\uc6b0\ub294 \ubc15\ucc2c\uc8fc\uc5d0\uac8c \uc0ac\uc8fc\ub2e8\uc790\uc640 \uc57d\ud63c\ubc18\uc9c0\ub97c \ubcf4\ub0b8 \ud6c4\uc5d0 \ub2f9\uc2dc \uc774\uc655\uc9c1 \uc7a5\uad00 \ud55c\ucc3d\uc218\uc5d0\uac8c\ub294 \ud63c\uc57d\uc774 \uc131\ub9bd\ub418\uc5c8\ub2e4\uace0 \uc77c\ubc29\uc801\uc73c\ub85c \ud1b5\uace0\ud558\uc600\ub2e4.", "paragraph_id": "6545881-6-1", "paragraph_question": "question: \ucc9c\ud669\uc758 \uce59\ud5c8\uc5c6\uc774 \uc9c4\ud589\ub41c \uc774\uc6b0\uc758 \ud63c\uc57d\uc5d0 \ubc18\ubc1c\ud55c \ub2f9\uc2dc \uc655\uacf5\uc871\uc758 \uc0ac\ubb34\ub97c \uad00\uc7a5\ud558\ub358 \uc0ac\ub78c\uc758 \uc774\ub984\uc740?, context: \uc758\uce5c\uc655\uc740 \uc544\ub4e4\uc774 \uc77c\ubcf8\uc778\uacfc \uacb0\ud63c\ud558\ub294 \uac83\uc744 \uc6d0\ud558\uc9c0 \uc54a\uc558\uace0 \uc774\uc6b0\ub3c4 \uc870\uc120\uc778\uacfc\uc758 \uacb0\ud63c\uc744 \uace0\uc9d1\ud558\uc600\ub2e4. \uc77c\ubcf8\uc778 \uc2e0\ubd80\ub97c \uac70\ubd80\ud558\uae30 \uc704\ud55c \ubc29\uc548\uc744 \ubaa8\uc0c9\ud558\ub358 \uc911\uc5d0 \uc758\uce5c\uc655\uacfc \uae34\ubc00\ud55c \uad00\uacc4\uc600\ub358 \ubc15\uc601\ud6a8\uc758 \uc11c\uc190\ub140 \ubc15\ucc2c\uc8fc\uac00 \uc774\uc6b0\uc758 \ubc30\uc6b0\uc790\ub85c \ubb3c\ub9dd\uc5d0 \uc624\ub974\uace0 \ud63c\uc57d\uc774 \uc774\ub8e8\uc5b4\uc84c\ub2e4. \uc774\uc6b0\uc758 \uc591\ubaa8 \uad11\uc0b0 \uae40\uc528\ub294 \ubc15\uc601\ud6a8\uac00 \ub0b4\ubb34\ub300\uc2e0\uc73c\ub85c \uc7ac\uc784\ud558\ub358 1895\ub144\uc5d0 \ubc95\ubb34\ud611\ud310 \uae40\ud559\uc6b0 \uc554\uc0b4\uacfc \ubaa8\ubc18 \ud610\uc758\ub85c \uc591\ubd80 \uc774\uc900\uc6a9\uc744 \uc8fd\uc774\ub824 \ud558\uc600\uae30 \ub54c\ubb38\uc5d0 \ubc15\ucc2c\uc8fc\ub97c \ubabb\ub9c8\ub545\ud558\uac8c \uc5ec\uacbc\ub2e4. \uc774\uc6b0\ub294 \ubc15\ucc2c\uc8fc\uc5d0\uac8c \uc0ac\uc8fc\ub2e8\uc790\uc640 \uc57d\ud63c\ubc18\uc9c0\ub97c \ubcf4\ub0b8 \ud6c4\uc5d0 \ub2f9\uc2dc \uc774\uc655\uc9c1 \uc7a5\uad00 \ud55c\ucc3d\uc218\uc5d0\uac8c\ub294 \ud63c\uc57d\uc774 \uc131\ub9bd\ub418\uc5c8\ub2e4\uace0 \uc77c\ubc29\uc801\uc73c\ub85c \ud1b5\uace0\ud558\uc600\ub2e4. \uc655\uacf5\uc871\uc758 \uc0ac\ubb34\ub97c \uad00\uc7a5\ud558\ub358 \uc774\uc655\uc9c1\uc740 \ubc15\ucc2c\uc8fc\uc640\uc758 \ud63c\uc57d\uc774 \ucc9c\ud669\uc758 \uce59\ud5c8\uc5c6\uc774 \uc9c4\ud589\ub418\uc5c8\ub2e4\uace0 \ubc18\ubc1c\ud558\uba70 \uc77c\ubcf8 \ud669\uc871\uacfc \uacb0\ud63c\ud558\ub77c\ub294 \uc555\ub825\uc744 \ud589\uc0ac\ud558\uc600\ub2e4."} {"question": "\uc65c\uad00 \ubd81\ucabd\uc5d0 \uc704\uce58\ud55c \ud0c0\uc6d0\ud615 \uc57c\uc0b0\uc758 \uc774\ub984\uc740?", "paragraph": "\ubbf8\uad70\uc5d0\uac8c 303\uace0\uc9c0\ub85c \uc9c0\uc815\ub41c \uc790\uace0\uc0b0\uc740 \uac00\ub298\uace0 \uae38\ucb49\ud55c \ud0c0\uc6d0\ud615\uc758 \uc57c\uc0b0\uc73c\ub85c, \ud0c0\uc6d0 \uc7a5\ucd95\uc740 \ubd81\ub3d9-\ub0a8\uc11c \ubc29\ud5a5\uc73c\ub85c \uc874\uc7ac\ud588\uc73c\uba70 \uadf8 \uae38\uc774\uac00 3.2 \ud0ac\ub85c\ubbf8\ud130(2 \ub9c8\uc77c)\uc774\uace0 \ub192\uc774\ub294 \ud574\ubc1c 303 \ubbf8\ud130(993 \ud53c\ud2b8)\uc774\ub2e4. \uc790\uace0\uc0b0\uc740 \uc65c\uad00 \ubd81\ucabd\uc758 \uccab \ubc88\uc9f8 \uc5b8\ub355\uc73c\ub85c, \ub0a8\ucabd \uc0ac\uba74\uc744 \ud0c0\uace0 \ub0b4\ub824\uc624\uba74 \uc65c\uad00 \uc74d\ub0b4 \uac00\uc7a5\uc790\ub9ac\uc5d0 \ubc14\ub85c \ub2ff\uac8c \ub41c\ub2e4. \uc57c\uc0b0 \uc704\uc5d0 \uc62c\ub77c\uac00\uba74 \uc65c\uad00\uc758 \ub3c4\ub85c\ub9dd\uacfc \ucca0\ub3c4, \uace0\uc18d\ub3c4\ub85c \uad50\ub7c9, \ubd81\uc5d0\uc11c \ub0a8\uc73c\ub85c \ud750\ub974\ub294 \uac15\uc744 \ud55c\ub208\uc5d0 \uc0b4\ud3b4\ubcfc \uc218 \uc788\ub2e4. \uc790\uace0\uc0b0 \uc11c\ucabd \uc0ac\uba74\uc740 \ub099\ub3d9\uac15\uc758 \ub3d9\ucabd \uac15\ubcc0\uacfc \ub2ff\uc544 \uc788\ub2e4. \uc65c\uad00\uc5d0\uc11c \ub0a8\ucabd\uacfc \ubd81\ucabd\uc73c\ub85c \ub099\ub3d9\uac15 \uac15\ubcc0\uc744 \ub530\ub77c \ub3c4\ub85c\uac00 \ubed7\uc5b4 \uc788\uace0, \ubd81\ub3d9\ucabd\uc73c\ub85c \uac00\uba74 \ub300\uad6c\ub85c \ud1b5\ud558\ub294 \ub610\ub2e4\ub978 \uae38\ubaa9\uc778 \ub2e4\ubd80\ub3d9\uc774 \ub098\uc628\ub2e4. \uc790\uace0\uc0b0\uc740 \uadf8 \ubc14\ub85c \uc544\ub798\uc758 \uc65c\uad00 \uc74d\ub0b4 \ubd84 \uc544\ub2c8\ub77c, \uc65c\uad00\uc744 \uc9c0\ub098\ub294 \uacbd\ubd80\uc120 \ucca0\ub3c4\uc640 \ub099\ub3d9\uac15\uc758 \uad50\ub7c9\ub4e4\uc744 \uc7a5\uc545\ud560 \uc218 \uc788\ub294 \uacb0\uc815\uc801 \uc704\uce58\uc600\ub2e4.", "answer": "\uc790\uace0\uc0b0", "sentence": "\ubbf8\uad70\uc5d0\uac8c 303\uace0\uc9c0\ub85c \uc9c0\uc815\ub41c \uc790\uace0\uc0b0 \uc740 \uac00\ub298\uace0 \uae38\ucb49\ud55c \ud0c0\uc6d0\ud615\uc758 \uc57c\uc0b0\uc73c\ub85c, \ud0c0\uc6d0 \uc7a5\ucd95\uc740 \ubd81\ub3d9-\ub0a8\uc11c \ubc29\ud5a5\uc73c\ub85c \uc874\uc7ac\ud588\uc73c\uba70 \uadf8 \uae38\uc774\uac00 3.2 \ud0ac\ub85c\ubbf8\ud130(2 \ub9c8\uc77c)\uc774\uace0 \ub192\uc774\ub294 \ud574\ubc1c 303 \ubbf8\ud130(993 \ud53c\ud2b8)\uc774\ub2e4.", "paragraph_sentence": " \ubbf8\uad70\uc5d0\uac8c 303\uace0\uc9c0\ub85c \uc9c0\uc815\ub41c \uc790\uace0\uc0b0 \uc740 \uac00\ub298\uace0 \uae38\ucb49\ud55c \ud0c0\uc6d0\ud615\uc758 \uc57c\uc0b0\uc73c\ub85c, \ud0c0\uc6d0 \uc7a5\ucd95\uc740 \ubd81\ub3d9-\ub0a8\uc11c \ubc29\ud5a5\uc73c\ub85c \uc874\uc7ac\ud588\uc73c\uba70 \uadf8 \uae38\uc774\uac00 3.2 \ud0ac\ub85c\ubbf8\ud130(2 \ub9c8\uc77c)\uc774\uace0 \ub192\uc774\ub294 \ud574\ubc1c 303 \ubbf8\ud130(993 \ud53c\ud2b8)\uc774\ub2e4. \uc790\uace0\uc0b0\uc740 \uc65c\uad00 \ubd81\ucabd\uc758 \uccab \ubc88\uc9f8 \uc5b8\ub355\uc73c\ub85c, \ub0a8\ucabd \uc0ac\uba74\uc744 \ud0c0\uace0 \ub0b4\ub824\uc624\uba74 \uc65c\uad00 \uc74d\ub0b4 \uac00\uc7a5\uc790\ub9ac\uc5d0 \ubc14\ub85c \ub2ff\uac8c \ub41c\ub2e4. \uc57c\uc0b0 \uc704\uc5d0 \uc62c\ub77c\uac00\uba74 \uc65c\uad00\uc758 \ub3c4\ub85c\ub9dd\uacfc \ucca0\ub3c4, \uace0\uc18d\ub3c4\ub85c \uad50\ub7c9, \ubd81\uc5d0\uc11c \ub0a8\uc73c\ub85c \ud750\ub974\ub294 \uac15\uc744 \ud55c\ub208\uc5d0 \uc0b4\ud3b4\ubcfc \uc218 \uc788\ub2e4. \uc790\uace0\uc0b0 \uc11c\ucabd \uc0ac\uba74\uc740 \ub099\ub3d9\uac15\uc758 \ub3d9\ucabd \uac15\ubcc0\uacfc \ub2ff\uc544 \uc788\ub2e4. \uc65c\uad00\uc5d0\uc11c \ub0a8\ucabd\uacfc \ubd81\ucabd\uc73c\ub85c \ub099\ub3d9\uac15 \uac15\ubcc0\uc744 \ub530\ub77c \ub3c4\ub85c\uac00 \ubed7\uc5b4 \uc788\uace0, \ubd81\ub3d9\ucabd\uc73c\ub85c \uac00\uba74 \ub300\uad6c\ub85c \ud1b5\ud558\ub294 \ub610\ub2e4\ub978 \uae38\ubaa9\uc778 \ub2e4\ubd80\ub3d9\uc774 \ub098\uc628\ub2e4. \uc790\uace0\uc0b0\uc740 \uadf8 \ubc14\ub85c \uc544\ub798\uc758 \uc65c\uad00 \uc74d\ub0b4 \ubd84 \uc544\ub2c8\ub77c, \uc65c\uad00\uc744 \uc9c0\ub098\ub294 \uacbd\ubd80\uc120 \ucca0\ub3c4\uc640 \ub099\ub3d9\uac15\uc758 \uad50\ub7c9\ub4e4\uc744 \uc7a5\uc545\ud560 \uc218 \uc788\ub294 \uacb0\uc815\uc801 \uc704\uce58\uc600\ub2e4.", "paragraph_answer": "\ubbf8\uad70\uc5d0\uac8c 303\uace0\uc9c0\ub85c \uc9c0\uc815\ub41c \uc790\uace0\uc0b0 \uc740 \uac00\ub298\uace0 \uae38\ucb49\ud55c \ud0c0\uc6d0\ud615\uc758 \uc57c\uc0b0\uc73c\ub85c, \ud0c0\uc6d0 \uc7a5\ucd95\uc740 \ubd81\ub3d9-\ub0a8\uc11c \ubc29\ud5a5\uc73c\ub85c \uc874\uc7ac\ud588\uc73c\uba70 \uadf8 \uae38\uc774\uac00 3.2 \ud0ac\ub85c\ubbf8\ud130(2 \ub9c8\uc77c)\uc774\uace0 \ub192\uc774\ub294 \ud574\ubc1c 303 \ubbf8\ud130(993 \ud53c\ud2b8)\uc774\ub2e4. \uc790\uace0\uc0b0\uc740 \uc65c\uad00 \ubd81\ucabd\uc758 \uccab \ubc88\uc9f8 \uc5b8\ub355\uc73c\ub85c, \ub0a8\ucabd \uc0ac\uba74\uc744 \ud0c0\uace0 \ub0b4\ub824\uc624\uba74 \uc65c\uad00 \uc74d\ub0b4 \uac00\uc7a5\uc790\ub9ac\uc5d0 \ubc14\ub85c \ub2ff\uac8c \ub41c\ub2e4. \uc57c\uc0b0 \uc704\uc5d0 \uc62c\ub77c\uac00\uba74 \uc65c\uad00\uc758 \ub3c4\ub85c\ub9dd\uacfc \ucca0\ub3c4, \uace0\uc18d\ub3c4\ub85c \uad50\ub7c9, \ubd81\uc5d0\uc11c \ub0a8\uc73c\ub85c \ud750\ub974\ub294 \uac15\uc744 \ud55c\ub208\uc5d0 \uc0b4\ud3b4\ubcfc \uc218 \uc788\ub2e4. \uc790\uace0\uc0b0 \uc11c\ucabd \uc0ac\uba74\uc740 \ub099\ub3d9\uac15\uc758 \ub3d9\ucabd \uac15\ubcc0\uacfc \ub2ff\uc544 \uc788\ub2e4. \uc65c\uad00\uc5d0\uc11c \ub0a8\ucabd\uacfc \ubd81\ucabd\uc73c\ub85c \ub099\ub3d9\uac15 \uac15\ubcc0\uc744 \ub530\ub77c \ub3c4\ub85c\uac00 \ubed7\uc5b4 \uc788\uace0, \ubd81\ub3d9\ucabd\uc73c\ub85c \uac00\uba74 \ub300\uad6c\ub85c \ud1b5\ud558\ub294 \ub610\ub2e4\ub978 \uae38\ubaa9\uc778 \ub2e4\ubd80\ub3d9\uc774 \ub098\uc628\ub2e4. \uc790\uace0\uc0b0\uc740 \uadf8 \ubc14\ub85c \uc544\ub798\uc758 \uc65c\uad00 \uc74d\ub0b4 \ubd84 \uc544\ub2c8\ub77c, \uc65c\uad00\uc744 \uc9c0\ub098\ub294 \uacbd\ubd80\uc120 \ucca0\ub3c4\uc640 \ub099\ub3d9\uac15\uc758 \uad50\ub7c9\ub4e4\uc744 \uc7a5\uc545\ud560 \uc218 \uc788\ub294 \uacb0\uc815\uc801 \uc704\uce58\uc600\ub2e4.", "sentence_answer": "\ubbf8\uad70\uc5d0\uac8c 303\uace0\uc9c0\ub85c \uc9c0\uc815\ub41c \uc790\uace0\uc0b0 \uc740 \uac00\ub298\uace0 \uae38\ucb49\ud55c \ud0c0\uc6d0\ud615\uc758 \uc57c\uc0b0\uc73c\ub85c, \ud0c0\uc6d0 \uc7a5\ucd95\uc740 \ubd81\ub3d9-\ub0a8\uc11c \ubc29\ud5a5\uc73c\ub85c \uc874\uc7ac\ud588\uc73c\uba70 \uadf8 \uae38\uc774\uac00 3.2 \ud0ac\ub85c\ubbf8\ud130(2 \ub9c8\uc77c)\uc774\uace0 \ub192\uc774\ub294 \ud574\ubc1c 303 \ubbf8\ud130(993 \ud53c\ud2b8)\uc774\ub2e4.", "paragraph_id": "6470861-2-0", "paragraph_question": "question: \uc65c\uad00 \ubd81\ucabd\uc5d0 \uc704\uce58\ud55c \ud0c0\uc6d0\ud615 \uc57c\uc0b0\uc758 \uc774\ub984\uc740?, context: \ubbf8\uad70\uc5d0\uac8c 303\uace0\uc9c0\ub85c \uc9c0\uc815\ub41c \uc790\uace0\uc0b0\uc740 \uac00\ub298\uace0 \uae38\ucb49\ud55c \ud0c0\uc6d0\ud615\uc758 \uc57c\uc0b0\uc73c\ub85c, \ud0c0\uc6d0 \uc7a5\ucd95\uc740 \ubd81\ub3d9-\ub0a8\uc11c \ubc29\ud5a5\uc73c\ub85c \uc874\uc7ac\ud588\uc73c\uba70 \uadf8 \uae38\uc774\uac00 3.2 \ud0ac\ub85c\ubbf8\ud130(2 \ub9c8\uc77c)\uc774\uace0 \ub192\uc774\ub294 \ud574\ubc1c 303 \ubbf8\ud130(993 \ud53c\ud2b8)\uc774\ub2e4. \uc790\uace0\uc0b0\uc740 \uc65c\uad00 \ubd81\ucabd\uc758 \uccab \ubc88\uc9f8 \uc5b8\ub355\uc73c\ub85c, \ub0a8\ucabd \uc0ac\uba74\uc744 \ud0c0\uace0 \ub0b4\ub824\uc624\uba74 \uc65c\uad00 \uc74d\ub0b4 \uac00\uc7a5\uc790\ub9ac\uc5d0 \ubc14\ub85c \ub2ff\uac8c \ub41c\ub2e4. \uc57c\uc0b0 \uc704\uc5d0 \uc62c\ub77c\uac00\uba74 \uc65c\uad00\uc758 \ub3c4\ub85c\ub9dd\uacfc \ucca0\ub3c4, \uace0\uc18d\ub3c4\ub85c \uad50\ub7c9, \ubd81\uc5d0\uc11c \ub0a8\uc73c\ub85c \ud750\ub974\ub294 \uac15\uc744 \ud55c\ub208\uc5d0 \uc0b4\ud3b4\ubcfc \uc218 \uc788\ub2e4. \uc790\uace0\uc0b0 \uc11c\ucabd \uc0ac\uba74\uc740 \ub099\ub3d9\uac15\uc758 \ub3d9\ucabd \uac15\ubcc0\uacfc \ub2ff\uc544 \uc788\ub2e4. \uc65c\uad00\uc5d0\uc11c \ub0a8\ucabd\uacfc \ubd81\ucabd\uc73c\ub85c \ub099\ub3d9\uac15 \uac15\ubcc0\uc744 \ub530\ub77c \ub3c4\ub85c\uac00 \ubed7\uc5b4 \uc788\uace0, \ubd81\ub3d9\ucabd\uc73c\ub85c \uac00\uba74 \ub300\uad6c\ub85c \ud1b5\ud558\ub294 \ub610\ub2e4\ub978 \uae38\ubaa9\uc778 \ub2e4\ubd80\ub3d9\uc774 \ub098\uc628\ub2e4. \uc790\uace0\uc0b0\uc740 \uadf8 \ubc14\ub85c \uc544\ub798\uc758 \uc65c\uad00 \uc74d\ub0b4 \ubd84 \uc544\ub2c8\ub77c, \uc65c\uad00\uc744 \uc9c0\ub098\ub294 \uacbd\ubd80\uc120 \ucca0\ub3c4\uc640 \ub099\ub3d9\uac15\uc758 \uad50\ub7c9\ub4e4\uc744 \uc7a5\uc545\ud560 \uc218 \uc788\ub294 \uacb0\uc815\uc801 \uc704\uce58\uc600\ub2e4."} {"question": "\ud1f4\uac01 \ubcf4\ud2b8\uc5d0 \ud0c4 \uc601\uad6d \uc5ec\uc790\ub4e4\uacfc \uc544\uc774\ub4e4\uc774 \ub04c\ub824\uac00 \uc0b4\ud574\ub41c \uacf3\uc740?", "paragraph": "1857\ub144 6\uc6d4 \ud734\uadf8 \ud720\ub7ec \uc7a5\uad70 \uc18c\uc18d\uc758 \uc138\ud3ec\uc774\ub4e4\uc774 \ubd09\uae30\ud558\uc5ec \uc601\uad6d \uce21 \ubcf4\ub8e8\ub97c \ud3ec\uc704\ud558\uc600\ub2e4. \ud720\ub7ec\ub294 \uc720\ub2a5\ud55c \uad70\uc778\uc774\uc5c8\uc744 \ubfd0\ub9cc\uc544\ub2c8\ub77c \uc0c1\ub958 \uce74\uc2a4\ud2b8 \ucd9c\uc2e0\uc758 \uc544\ub0b4\ub97c \ub9de\uc774\ud558\uc5ec \uc778\ub3c4\uc778 \uc0ac\ud68c\uc5d0\uc11c\ub3c4 \uc601\ud5a5\ub825\uc774 \uc788\ub294 \uc778\ubb3c\uc774\uc5c8\ub2e4. \uc138\ud3ec\uc774 \ubc18\uad70\uc758 \uc9c0\ub3c4\uc790 \ub098\ub098 \uc0ac\ud788\ube0c \uc5ed\uc2dc \ud720\ub7ec\uc640 \uce5c\ucc99\uad00\uacc4\uc5d0 \uc788\uc5c8\ub2e4. \ud3ec\uc704\ub41c \uc9c0 \uc77c\uc8fc\uc77c\uc744 \ub118\uae30\uc790 \uc601\uad6d\uce21\uc740 \uc2dd\ub7c9 \uace0\uac08\uacfc \ubb3c \ubd80\uc871\uc73c\ub85c \uc0ac\uc0c1\uc790\ub97c \ub0b4\uae30 \uc2dc\uc791\ud558\uc600\ub2e4. \uc804\ud22c\uc694\uc6d0 \ubfd0\ub9cc \uc544\ub2c8\ub77c \uc5ec\uc131\uacfc \uc5b4\ub9b0\uc774\ub97c \ud3ec\ud568\ud55c \ubbfc\uac04\uc778\ub4e4\ub3c4 \ud76c\uc0dd\uc790\uac00 \ubc1c\uc0dd\ud558\uc600\ub2e4. \uc0ac\ud0dc\uac00 \uc2ec\uac01\ud574\uc9c0\uc790 6\uc6d4 25\uc77c \uc0ac\ud788\ube0c\ub294 \ud720\ub7ec\uc5d0\uac8c \uc54c\ub77c\ud558\ubc14\ub4dc\ub85c \uc548\uc804\ud558\uac8c \ud1f4\uac01\ud558\ub3c4\ub85d \ud574\uc8fc\uaca0\ub2e4\ub294 \uc81c\uc548\uc744 \ud558\uc600\ub2e4. \uc601\uad6d\uce21\uc740 \uba87 \uc77c \ubd84\ubc16\uc5d0 \ub0a8\uc9c0 \uc54a\uc740 \uc2dd\ub7c9 \uc0ac\uc815\uc744 \uac10\uc548\ud558\uc5ec 27\uc77c \uc544\uce68\uc5d0 \uacbd\ubb34\uc7a5\uc73c\ub85c \ud1f4\uac01\ud558\uaca0\ub2e4\uace0 \ub2f5\ud558\uc600\ub2e4. 27\uc77c \uc0ac\ud788\ube0c\ub294 \uc601\uad6d \uce21\uc5d0 \ubcf4\ud2b8\ub97c \uc81c\uacf5\ud558\uc600\uace0 \uc774\ub4e4\uc740 \uac15\uc744 \ud0c0\uace0 \ub0b4\ub824\uac00 \uc54c\ub77c\ud558\ubc14\ub4dc\ub85c \ud1f4\uac01\ud558\ub824 \ud558\uc600\ub2e4. \uc601\uad6d \uce21\uc5d0 \ub0a8\uc544 \uc788\uc5c8\ub358 \uba87 \uba85\uc758 \uc138\ud3ec\uc774\ub4e4\uc740 \u201c\ubc30\ubc18\uc790\u201d\uc774\uc790 \u201c\uac1c\uc885\uc790\u201d\ub77c\ub294 \uba85\ubaa9\uc73c\ub85c \ubc18\uad70 \uce21\uc5d0 \uc758\ud574 \uc0ac\uc0b4\ub418\uc5c8\ub2e4. \uc601\uad6d \uce21\uc774 \uac90\uc9c0\uc2a4 \uac15\uc5d0 \uc774\ub974\ub7ec \ubcf4\ud2b8\uc5d0 \ud0c0\uae30 \uc2dc\uc791\ud560 \ubb34\ub835 \uac15\uc758 \uc591\ucabd\uc744 \uc138\ud3ec\uc774\ub4e4\uc774 \ud3ec\uc704\ud558\uc600\ub2e4. \ubcf4\ud2b8\uac00 \ucd9c\ubc1c\ud558\uc790 \uc138\ud3ec\uc774\ub4e4\uc740 \ucd1d\uaca9\uc744 \uac00\ud558\uae30 \uc2dc\uc791\ud558\uc600\ub2e4. \uc0b4\uc544\ub0a8\uc740 \uc0ac\ub78c\ub4e4\uc740 \uc138\ud3ec\uc774 \uacbd\uae30\ubcd1\uc774 \ub2ec\ub824\ub4e4\uc5b4 \uc0b4\ud574\ud558\uc600\ub2e4. \uc138\ud3ec\uc774\ub4e4\uc740 \ubaa8\ub4e0 \ub0a8\uc790\ub4e4\uc774 \uc0ac\ub9dd\ud55c \uac83\uc744 \ud655\uc778\ud55c \ud6c4\uc5d0\uc57c \ucd1d\uaca9\uc744 \uba48\ucd94\uc5c8\ub2e4. \uc5ec\uc790\ub4e4\uacfc \uc544\uc774\ub4e4\uc740 \ube44\ube44\uac00\ub974\ub85c \ub04c\ub824\uac14\ub294\ub370 \uacb0\uad6d \uadf8\uacf3\uc5d0\uc11c \uc0b4\ud574\ub418\uc5c8\ub2e4. \ub450 \uba85\uc758 \ubcd1\uc0ac\uc640 \uc18c\uc704 \ud55c \uba85, \uadf8\ub9ac\uace0 \ub300\uc704\uc600\ub358 \ubaa8\ube0c\ub808\uc774 \ud1b0\uc2a8\ub9cc\uc774 \ud0c8\ucd9c\uc5d0 \uc131\uacf5\ud560 \uc218 \uc788\uc5c8\ub2e4. \ud1b0\uc2a8\uc740 \ud6c4\uc77c \u300a\uce78\ud478\ub974 \uc774\uc57c\uae30\u300b\ub97c \ucd9c\uac04\ud558\uc600\ub2e4.", "answer": "\ube44\ube44\uac00\ub974", "sentence": "\uc5ec\uc790\ub4e4\uacfc \uc544\uc774\ub4e4\uc740 \ube44\ube44\uac00\ub974 \ub85c \ub04c\ub824\uac14\ub294\ub370 \uacb0\uad6d \uadf8\uacf3\uc5d0\uc11c \uc0b4\ud574\ub418\uc5c8\ub2e4.", "paragraph_sentence": "1857\ub144 6\uc6d4 \ud734\uadf8 \ud720\ub7ec \uc7a5\uad70 \uc18c\uc18d\uc758 \uc138\ud3ec\uc774\ub4e4\uc774 \ubd09\uae30\ud558\uc5ec \uc601\uad6d \uce21 \ubcf4\ub8e8\ub97c \ud3ec\uc704\ud558\uc600\ub2e4. \ud720\ub7ec\ub294 \uc720\ub2a5\ud55c \uad70\uc778\uc774\uc5c8\uc744 \ubfd0\ub9cc\uc544\ub2c8\ub77c \uc0c1\ub958 \uce74\uc2a4\ud2b8 \ucd9c\uc2e0\uc758 \uc544\ub0b4\ub97c \ub9de\uc774\ud558\uc5ec \uc778\ub3c4\uc778 \uc0ac\ud68c\uc5d0\uc11c\ub3c4 \uc601\ud5a5\ub825\uc774 \uc788\ub294 \uc778\ubb3c\uc774\uc5c8\ub2e4. \uc138\ud3ec\uc774 \ubc18\uad70\uc758 \uc9c0\ub3c4\uc790 \ub098\ub098 \uc0ac\ud788\ube0c \uc5ed\uc2dc \ud720\ub7ec\uc640 \uce5c\ucc99\uad00\uacc4\uc5d0 \uc788\uc5c8\ub2e4. \ud3ec\uc704\ub41c \uc9c0 \uc77c\uc8fc\uc77c\uc744 \ub118\uae30\uc790 \uc601\uad6d\uce21\uc740 \uc2dd\ub7c9 \uace0\uac08\uacfc \ubb3c \ubd80\uc871\uc73c\ub85c \uc0ac\uc0c1\uc790\ub97c \ub0b4\uae30 \uc2dc\uc791\ud558\uc600\ub2e4. \uc804\ud22c\uc694\uc6d0 \ubfd0\ub9cc \uc544\ub2c8\ub77c \uc5ec\uc131\uacfc \uc5b4\ub9b0\uc774\ub97c \ud3ec\ud568\ud55c \ubbfc\uac04\uc778\ub4e4\ub3c4 \ud76c\uc0dd\uc790\uac00 \ubc1c\uc0dd\ud558\uc600\ub2e4. \uc0ac\ud0dc\uac00 \uc2ec\uac01\ud574\uc9c0\uc790 6\uc6d4 25\uc77c \uc0ac\ud788\ube0c\ub294 \ud720\ub7ec\uc5d0\uac8c \uc54c\ub77c\ud558\ubc14\ub4dc\ub85c \uc548\uc804\ud558\uac8c \ud1f4\uac01\ud558\ub3c4\ub85d \ud574\uc8fc\uaca0\ub2e4\ub294 \uc81c\uc548\uc744 \ud558\uc600\ub2e4. \uc601\uad6d\uce21\uc740 \uba87 \uc77c \ubd84\ubc16\uc5d0 \ub0a8\uc9c0 \uc54a\uc740 \uc2dd\ub7c9 \uc0ac\uc815\uc744 \uac10\uc548\ud558\uc5ec 27\uc77c \uc544\uce68\uc5d0 \uacbd\ubb34\uc7a5\uc73c\ub85c \ud1f4\uac01\ud558\uaca0\ub2e4\uace0 \ub2f5\ud558\uc600\ub2e4. 27\uc77c \uc0ac\ud788\ube0c\ub294 \uc601\uad6d \uce21\uc5d0 \ubcf4\ud2b8\ub97c \uc81c\uacf5\ud558\uc600\uace0 \uc774\ub4e4\uc740 \uac15\uc744 \ud0c0\uace0 \ub0b4\ub824\uac00 \uc54c\ub77c\ud558\ubc14\ub4dc\ub85c \ud1f4\uac01\ud558\ub824 \ud558\uc600\ub2e4. \uc601\uad6d \uce21\uc5d0 \ub0a8\uc544 \uc788\uc5c8\ub358 \uba87 \uba85\uc758 \uc138\ud3ec\uc774\ub4e4\uc740 \u201c\ubc30\ubc18\uc790\u201d\uc774\uc790 \u201c\uac1c\uc885\uc790\u201d\ub77c\ub294 \uba85\ubaa9\uc73c\ub85c \ubc18\uad70 \uce21\uc5d0 \uc758\ud574 \uc0ac\uc0b4\ub418\uc5c8\ub2e4. \uc601\uad6d \uce21\uc774 \uac90\uc9c0\uc2a4 \uac15\uc5d0 \uc774\ub974\ub7ec \ubcf4\ud2b8\uc5d0 \ud0c0\uae30 \uc2dc\uc791\ud560 \ubb34\ub835 \uac15\uc758 \uc591\ucabd\uc744 \uc138\ud3ec\uc774\ub4e4\uc774 \ud3ec\uc704\ud558\uc600\ub2e4. \ubcf4\ud2b8\uac00 \ucd9c\ubc1c\ud558\uc790 \uc138\ud3ec\uc774\ub4e4\uc740 \ucd1d\uaca9\uc744 \uac00\ud558\uae30 \uc2dc\uc791\ud558\uc600\ub2e4. \uc0b4\uc544\ub0a8\uc740 \uc0ac\ub78c\ub4e4\uc740 \uc138\ud3ec\uc774 \uacbd\uae30\ubcd1\uc774 \ub2ec\ub824\ub4e4\uc5b4 \uc0b4\ud574\ud558\uc600\ub2e4. \uc138\ud3ec\uc774\ub4e4\uc740 \ubaa8\ub4e0 \ub0a8\uc790\ub4e4\uc774 \uc0ac\ub9dd\ud55c \uac83\uc744 \ud655\uc778\ud55c \ud6c4\uc5d0\uc57c \ucd1d\uaca9\uc744 \uba48\ucd94\uc5c8\ub2e4. \uc5ec\uc790\ub4e4\uacfc \uc544\uc774\ub4e4\uc740 \ube44\ube44\uac00\ub974 \ub85c \ub04c\ub824\uac14\ub294\ub370 \uacb0\uad6d \uadf8\uacf3\uc5d0\uc11c \uc0b4\ud574\ub418\uc5c8\ub2e4. \ub450 \uba85\uc758 \ubcd1\uc0ac\uc640 \uc18c\uc704 \ud55c \uba85, \uadf8\ub9ac\uace0 \ub300\uc704\uc600\ub358 \ubaa8\ube0c\ub808\uc774 \ud1b0\uc2a8\ub9cc\uc774 \ud0c8\ucd9c\uc5d0 \uc131\uacf5\ud560 \uc218 \uc788\uc5c8\ub2e4. \ud1b0\uc2a8\uc740 \ud6c4\uc77c \u300a\uce78\ud478\ub974 \uc774\uc57c\uae30\u300b\ub97c \ucd9c\uac04\ud558\uc600\ub2e4.", "paragraph_answer": "1857\ub144 6\uc6d4 \ud734\uadf8 \ud720\ub7ec \uc7a5\uad70 \uc18c\uc18d\uc758 \uc138\ud3ec\uc774\ub4e4\uc774 \ubd09\uae30\ud558\uc5ec \uc601\uad6d \uce21 \ubcf4\ub8e8\ub97c \ud3ec\uc704\ud558\uc600\ub2e4. \ud720\ub7ec\ub294 \uc720\ub2a5\ud55c \uad70\uc778\uc774\uc5c8\uc744 \ubfd0\ub9cc\uc544\ub2c8\ub77c \uc0c1\ub958 \uce74\uc2a4\ud2b8 \ucd9c\uc2e0\uc758 \uc544\ub0b4\ub97c \ub9de\uc774\ud558\uc5ec \uc778\ub3c4\uc778 \uc0ac\ud68c\uc5d0\uc11c\ub3c4 \uc601\ud5a5\ub825\uc774 \uc788\ub294 \uc778\ubb3c\uc774\uc5c8\ub2e4. \uc138\ud3ec\uc774 \ubc18\uad70\uc758 \uc9c0\ub3c4\uc790 \ub098\ub098 \uc0ac\ud788\ube0c \uc5ed\uc2dc \ud720\ub7ec\uc640 \uce5c\ucc99\uad00\uacc4\uc5d0 \uc788\uc5c8\ub2e4. \ud3ec\uc704\ub41c \uc9c0 \uc77c\uc8fc\uc77c\uc744 \ub118\uae30\uc790 \uc601\uad6d\uce21\uc740 \uc2dd\ub7c9 \uace0\uac08\uacfc \ubb3c \ubd80\uc871\uc73c\ub85c \uc0ac\uc0c1\uc790\ub97c \ub0b4\uae30 \uc2dc\uc791\ud558\uc600\ub2e4. \uc804\ud22c\uc694\uc6d0 \ubfd0\ub9cc \uc544\ub2c8\ub77c \uc5ec\uc131\uacfc \uc5b4\ub9b0\uc774\ub97c \ud3ec\ud568\ud55c \ubbfc\uac04\uc778\ub4e4\ub3c4 \ud76c\uc0dd\uc790\uac00 \ubc1c\uc0dd\ud558\uc600\ub2e4. \uc0ac\ud0dc\uac00 \uc2ec\uac01\ud574\uc9c0\uc790 6\uc6d4 25\uc77c \uc0ac\ud788\ube0c\ub294 \ud720\ub7ec\uc5d0\uac8c \uc54c\ub77c\ud558\ubc14\ub4dc\ub85c \uc548\uc804\ud558\uac8c \ud1f4\uac01\ud558\ub3c4\ub85d \ud574\uc8fc\uaca0\ub2e4\ub294 \uc81c\uc548\uc744 \ud558\uc600\ub2e4. \uc601\uad6d\uce21\uc740 \uba87 \uc77c \ubd84\ubc16\uc5d0 \ub0a8\uc9c0 \uc54a\uc740 \uc2dd\ub7c9 \uc0ac\uc815\uc744 \uac10\uc548\ud558\uc5ec 27\uc77c \uc544\uce68\uc5d0 \uacbd\ubb34\uc7a5\uc73c\ub85c \ud1f4\uac01\ud558\uaca0\ub2e4\uace0 \ub2f5\ud558\uc600\ub2e4. 27\uc77c \uc0ac\ud788\ube0c\ub294 \uc601\uad6d \uce21\uc5d0 \ubcf4\ud2b8\ub97c \uc81c\uacf5\ud558\uc600\uace0 \uc774\ub4e4\uc740 \uac15\uc744 \ud0c0\uace0 \ub0b4\ub824\uac00 \uc54c\ub77c\ud558\ubc14\ub4dc\ub85c \ud1f4\uac01\ud558\ub824 \ud558\uc600\ub2e4. \uc601\uad6d \uce21\uc5d0 \ub0a8\uc544 \uc788\uc5c8\ub358 \uba87 \uba85\uc758 \uc138\ud3ec\uc774\ub4e4\uc740 \u201c\ubc30\ubc18\uc790\u201d\uc774\uc790 \u201c\uac1c\uc885\uc790\u201d\ub77c\ub294 \uba85\ubaa9\uc73c\ub85c \ubc18\uad70 \uce21\uc5d0 \uc758\ud574 \uc0ac\uc0b4\ub418\uc5c8\ub2e4. \uc601\uad6d \uce21\uc774 \uac90\uc9c0\uc2a4 \uac15\uc5d0 \uc774\ub974\ub7ec \ubcf4\ud2b8\uc5d0 \ud0c0\uae30 \uc2dc\uc791\ud560 \ubb34\ub835 \uac15\uc758 \uc591\ucabd\uc744 \uc138\ud3ec\uc774\ub4e4\uc774 \ud3ec\uc704\ud558\uc600\ub2e4. \ubcf4\ud2b8\uac00 \ucd9c\ubc1c\ud558\uc790 \uc138\ud3ec\uc774\ub4e4\uc740 \ucd1d\uaca9\uc744 \uac00\ud558\uae30 \uc2dc\uc791\ud558\uc600\ub2e4. \uc0b4\uc544\ub0a8\uc740 \uc0ac\ub78c\ub4e4\uc740 \uc138\ud3ec\uc774 \uacbd\uae30\ubcd1\uc774 \ub2ec\ub824\ub4e4\uc5b4 \uc0b4\ud574\ud558\uc600\ub2e4. \uc138\ud3ec\uc774\ub4e4\uc740 \ubaa8\ub4e0 \ub0a8\uc790\ub4e4\uc774 \uc0ac\ub9dd\ud55c \uac83\uc744 \ud655\uc778\ud55c \ud6c4\uc5d0\uc57c \ucd1d\uaca9\uc744 \uba48\ucd94\uc5c8\ub2e4. \uc5ec\uc790\ub4e4\uacfc \uc544\uc774\ub4e4\uc740 \ube44\ube44\uac00\ub974 \ub85c \ub04c\ub824\uac14\ub294\ub370 \uacb0\uad6d \uadf8\uacf3\uc5d0\uc11c \uc0b4\ud574\ub418\uc5c8\ub2e4. \ub450 \uba85\uc758 \ubcd1\uc0ac\uc640 \uc18c\uc704 \ud55c \uba85, \uadf8\ub9ac\uace0 \ub300\uc704\uc600\ub358 \ubaa8\ube0c\ub808\uc774 \ud1b0\uc2a8\ub9cc\uc774 \ud0c8\ucd9c\uc5d0 \uc131\uacf5\ud560 \uc218 \uc788\uc5c8\ub2e4. \ud1b0\uc2a8\uc740 \ud6c4\uc77c \u300a\uce78\ud478\ub974 \uc774\uc57c\uae30\u300b\ub97c \ucd9c\uac04\ud558\uc600\ub2e4.", "sentence_answer": "\uc5ec\uc790\ub4e4\uacfc \uc544\uc774\ub4e4\uc740 \ube44\ube44\uac00\ub974 \ub85c \ub04c\ub824\uac14\ub294\ub370 \uacb0\uad6d \uadf8\uacf3\uc5d0\uc11c \uc0b4\ud574\ub418\uc5c8\ub2e4.", "paragraph_id": "6532248-8-2", "paragraph_question": "question: \ud1f4\uac01 \ubcf4\ud2b8\uc5d0 \ud0c4 \uc601\uad6d \uc5ec\uc790\ub4e4\uacfc \uc544\uc774\ub4e4\uc774 \ub04c\ub824\uac00 \uc0b4\ud574\ub41c \uacf3\uc740?, context: 1857\ub144 6\uc6d4 \ud734\uadf8 \ud720\ub7ec \uc7a5\uad70 \uc18c\uc18d\uc758 \uc138\ud3ec\uc774\ub4e4\uc774 \ubd09\uae30\ud558\uc5ec \uc601\uad6d \uce21 \ubcf4\ub8e8\ub97c \ud3ec\uc704\ud558\uc600\ub2e4. \ud720\ub7ec\ub294 \uc720\ub2a5\ud55c \uad70\uc778\uc774\uc5c8\uc744 \ubfd0\ub9cc\uc544\ub2c8\ub77c \uc0c1\ub958 \uce74\uc2a4\ud2b8 \ucd9c\uc2e0\uc758 \uc544\ub0b4\ub97c \ub9de\uc774\ud558\uc5ec \uc778\ub3c4\uc778 \uc0ac\ud68c\uc5d0\uc11c\ub3c4 \uc601\ud5a5\ub825\uc774 \uc788\ub294 \uc778\ubb3c\uc774\uc5c8\ub2e4. \uc138\ud3ec\uc774 \ubc18\uad70\uc758 \uc9c0\ub3c4\uc790 \ub098\ub098 \uc0ac\ud788\ube0c \uc5ed\uc2dc \ud720\ub7ec\uc640 \uce5c\ucc99\uad00\uacc4\uc5d0 \uc788\uc5c8\ub2e4. \ud3ec\uc704\ub41c \uc9c0 \uc77c\uc8fc\uc77c\uc744 \ub118\uae30\uc790 \uc601\uad6d\uce21\uc740 \uc2dd\ub7c9 \uace0\uac08\uacfc \ubb3c \ubd80\uc871\uc73c\ub85c \uc0ac\uc0c1\uc790\ub97c \ub0b4\uae30 \uc2dc\uc791\ud558\uc600\ub2e4. \uc804\ud22c\uc694\uc6d0 \ubfd0\ub9cc \uc544\ub2c8\ub77c \uc5ec\uc131\uacfc \uc5b4\ub9b0\uc774\ub97c \ud3ec\ud568\ud55c \ubbfc\uac04\uc778\ub4e4\ub3c4 \ud76c\uc0dd\uc790\uac00 \ubc1c\uc0dd\ud558\uc600\ub2e4. \uc0ac\ud0dc\uac00 \uc2ec\uac01\ud574\uc9c0\uc790 6\uc6d4 25\uc77c \uc0ac\ud788\ube0c\ub294 \ud720\ub7ec\uc5d0\uac8c \uc54c\ub77c\ud558\ubc14\ub4dc\ub85c \uc548\uc804\ud558\uac8c \ud1f4\uac01\ud558\ub3c4\ub85d \ud574\uc8fc\uaca0\ub2e4\ub294 \uc81c\uc548\uc744 \ud558\uc600\ub2e4. \uc601\uad6d\uce21\uc740 \uba87 \uc77c \ubd84\ubc16\uc5d0 \ub0a8\uc9c0 \uc54a\uc740 \uc2dd\ub7c9 \uc0ac\uc815\uc744 \uac10\uc548\ud558\uc5ec 27\uc77c \uc544\uce68\uc5d0 \uacbd\ubb34\uc7a5\uc73c\ub85c \ud1f4\uac01\ud558\uaca0\ub2e4\uace0 \ub2f5\ud558\uc600\ub2e4. 27\uc77c \uc0ac\ud788\ube0c\ub294 \uc601\uad6d \uce21\uc5d0 \ubcf4\ud2b8\ub97c \uc81c\uacf5\ud558\uc600\uace0 \uc774\ub4e4\uc740 \uac15\uc744 \ud0c0\uace0 \ub0b4\ub824\uac00 \uc54c\ub77c\ud558\ubc14\ub4dc\ub85c \ud1f4\uac01\ud558\ub824 \ud558\uc600\ub2e4. \uc601\uad6d \uce21\uc5d0 \ub0a8\uc544 \uc788\uc5c8\ub358 \uba87 \uba85\uc758 \uc138\ud3ec\uc774\ub4e4\uc740 \u201c\ubc30\ubc18\uc790\u201d\uc774\uc790 \u201c\uac1c\uc885\uc790\u201d\ub77c\ub294 \uba85\ubaa9\uc73c\ub85c \ubc18\uad70 \uce21\uc5d0 \uc758\ud574 \uc0ac\uc0b4\ub418\uc5c8\ub2e4. \uc601\uad6d \uce21\uc774 \uac90\uc9c0\uc2a4 \uac15\uc5d0 \uc774\ub974\ub7ec \ubcf4\ud2b8\uc5d0 \ud0c0\uae30 \uc2dc\uc791\ud560 \ubb34\ub835 \uac15\uc758 \uc591\ucabd\uc744 \uc138\ud3ec\uc774\ub4e4\uc774 \ud3ec\uc704\ud558\uc600\ub2e4. \ubcf4\ud2b8\uac00 \ucd9c\ubc1c\ud558\uc790 \uc138\ud3ec\uc774\ub4e4\uc740 \ucd1d\uaca9\uc744 \uac00\ud558\uae30 \uc2dc\uc791\ud558\uc600\ub2e4. \uc0b4\uc544\ub0a8\uc740 \uc0ac\ub78c\ub4e4\uc740 \uc138\ud3ec\uc774 \uacbd\uae30\ubcd1\uc774 \ub2ec\ub824\ub4e4\uc5b4 \uc0b4\ud574\ud558\uc600\ub2e4. \uc138\ud3ec\uc774\ub4e4\uc740 \ubaa8\ub4e0 \ub0a8\uc790\ub4e4\uc774 \uc0ac\ub9dd\ud55c \uac83\uc744 \ud655\uc778\ud55c \ud6c4\uc5d0\uc57c \ucd1d\uaca9\uc744 \uba48\ucd94\uc5c8\ub2e4. \uc5ec\uc790\ub4e4\uacfc \uc544\uc774\ub4e4\uc740 \ube44\ube44\uac00\ub974\ub85c \ub04c\ub824\uac14\ub294\ub370 \uacb0\uad6d \uadf8\uacf3\uc5d0\uc11c \uc0b4\ud574\ub418\uc5c8\ub2e4. \ub450 \uba85\uc758 \ubcd1\uc0ac\uc640 \uc18c\uc704 \ud55c \uba85, \uadf8\ub9ac\uace0 \ub300\uc704\uc600\ub358 \ubaa8\ube0c\ub808\uc774 \ud1b0\uc2a8\ub9cc\uc774 \ud0c8\ucd9c\uc5d0 \uc131\uacf5\ud560 \uc218 \uc788\uc5c8\ub2e4. \ud1b0\uc2a8\uc740 \ud6c4\uc77c \u300a\uce78\ud478\ub974 \uc774\uc57c\uae30\u300b\ub97c \ucd9c\uac04\ud558\uc600\ub2e4."} {"question": "\uc720\uc2dc\uc9c4\uc774 \ud3c9\ud654\uc720\uc9c0\uad70\uc73c\ub85c \ud30c\ubcd1\ub41c \uacf3\uc740 \uc5b4\ub514\uc778\uac00?", "paragraph": "\uadf8\ub7ec\ub098 \uac15\ubaa8\uc5f0\uc740 \uc720\uc2dc\uc9c4\uacfc \ud5e4\uc5b4\uc9c4 \ud6c4 \uc0dd\uba85\ubcf4\ub2e4 \uc774\uc724\uc744 \ucd94\uad6c\ud558\ub294 \uc758\uc0ac\uac00 \ub418\uc5b4 \ubc84\ub9b0\ub2e4. \uc774\ud6c4 \uc720\uc2dc\uc9c4\uc740 \ud3c9\ud654 \uc720\uc9c0\ub97c \uc704\ud574 \uc6b0\ub974\ud06c\uc5d0 \ud30c\ubcd1\ub418\uace0, \uac15\ubaa8\uc5f0\uc740 \uc758\ub8cc \ubd09\uc0ac\ucc28 \uadf8\uacf3\uc5d0 \uc628\ub2e4. \uc6b0\ub974\ud06c\ub294 2004\ub144 \ud55c\uad6d\uc774 \ud3c9\ud654\uc720\uc9c0\uad70\uc744 \ud30c\ubcd1\ud55c \uc774\ub77c\ud06c\ub97c \uc0c1\uc9d5\ud55c\ub2e4. \ud604\uc2e4\uacfc \ud5c8\uad6c\uac00 \ub4a4\uc11e\uc778 \uc774\uacf3\uc5d0\uc11c \ub458\uc740 \u201c\uc870\uad6d\uc774 \uc9c0\ucf1c\uc57c \ud560 \uad6d\ubbfc\uc758 \uc778\uad8c\u201d\uc774\ub77c\ub294 \uac00\uce58\ub97c \uacf5\uc720\ud558\uba70 \uc0ac\ub791\uc744 \ud0a4\uc6cc \ub098\uac04\ub2e4. \uac15\ubaa8\uc5f0\uc740 \uc6b0\ub974\ud06c\uc5d0\uc11c\uc758 \uacbd\ud5d8\uc73c\ub85c \uc774\uc724\ubcf4\ub2e4 \uc0dd\uba85\uc774 \uc6b0\uc120\uc774\ub77c\ub294 \uc758\uc0ac\ub85c\uc11c\uc758 \ucd08\uc2ec\uc744 \ud68c\ubcf5\ud558\uac8c \ub41c\ub2e4. \uc6b0\ub974\ud06c\uc5d0\uc11c \uc7ac\ub09c\uacfc \ubd09\uc0ac\ub97c \uacbd\ud5d8\ud558\uace0 \uac15\ubaa8\uc5f0\uc740 \ud788\ud3ec\ud06c\ub77c\ud14c\uc2a4 \uc120\uc11c\ub97c \ub5a0\uc62c\ub9b0\ub2e4. \"\ub098\uc758 \uc0dd\uc560\ub97c \uc778\ub958\ubd09\uc0ac\uc5d0 \ubc14\uce60 \uac83\uc744 \uc5c4\uc219\ud788 \uc11c\uc57d\ud558\ub178\ub77c. \ub098\uc758 \ud658\uc790\uc758 \uac74\uac15\uacfc \uc0dd\uba85\uc744 \uccab\uc9f8\ub85c \uc0dd\uac01\ud558\uaca0\ub178\ub77c. \ub098\ub294 \uc778\uc885, \uc885\uad50, \uad6d\uc801, \uc0ac\ud68c\uc801 \uc9c0\uc704\ub97c \ucd08\uc6d4\ud558\uc5ec \uc624\uc9c1 \ud658\uc790\uc5d0 \ub300\ud55c \ub098\uc758 \uc758\ubb34\ub97c \uc9c0\ud0a4\uaca0\ub178\ub77c. \ube44\ub85d \uc704\ud611\uc744 \ub2f9\ud560\uc9c0\ub77c\ub3c4 \ub098\uc758 \uc9c0\uc2dd\uc744 \uc778\ub3c4\uc5d0 \uc5b4\uae0b\ub098\uac8c \uc4f0\uc9c0 \uc54a\uaca0\ub178\ub77c. (6\ud68c)\"", "answer": "\uc6b0\ub974\ud06c", "sentence": "\uc774\ud6c4 \uc720\uc2dc\uc9c4\uc740 \ud3c9\ud654 \uc720\uc9c0\ub97c \uc704\ud574 \uc6b0\ub974\ud06c \uc5d0 \ud30c\ubcd1\ub418\uace0, \uac15\ubaa8\uc5f0\uc740 \uc758\ub8cc \ubd09\uc0ac\ucc28 \uadf8\uacf3\uc5d0 \uc628\ub2e4.", "paragraph_sentence": "\uadf8\ub7ec\ub098 \uac15\ubaa8\uc5f0\uc740 \uc720\uc2dc\uc9c4\uacfc \ud5e4\uc5b4\uc9c4 \ud6c4 \uc0dd\uba85\ubcf4\ub2e4 \uc774\uc724\uc744 \ucd94\uad6c\ud558\ub294 \uc758\uc0ac\uac00 \ub418\uc5b4 \ubc84\ub9b0\ub2e4. \uc774\ud6c4 \uc720\uc2dc\uc9c4\uc740 \ud3c9\ud654 \uc720\uc9c0\ub97c \uc704\ud574 \uc6b0\ub974\ud06c \uc5d0 \ud30c\ubcd1\ub418\uace0, \uac15\ubaa8\uc5f0\uc740 \uc758\ub8cc \ubd09\uc0ac\ucc28 \uadf8\uacf3\uc5d0 \uc628\ub2e4. \uc6b0\ub974\ud06c\ub294 2004\ub144 \ud55c\uad6d\uc774 \ud3c9\ud654\uc720\uc9c0\uad70\uc744 \ud30c\ubcd1\ud55c \uc774\ub77c\ud06c\ub97c \uc0c1\uc9d5\ud55c\ub2e4. \ud604\uc2e4\uacfc \ud5c8\uad6c\uac00 \ub4a4\uc11e\uc778 \uc774\uacf3\uc5d0\uc11c \ub458\uc740 \u201c\uc870\uad6d\uc774 \uc9c0\ucf1c\uc57c \ud560 \uad6d\ubbfc\uc758 \uc778\uad8c\u201d\uc774\ub77c\ub294 \uac00\uce58\ub97c \uacf5\uc720\ud558\uba70 \uc0ac\ub791\uc744 \ud0a4\uc6cc \ub098\uac04\ub2e4. \uac15\ubaa8\uc5f0\uc740 \uc6b0\ub974\ud06c\uc5d0\uc11c\uc758 \uacbd\ud5d8\uc73c\ub85c \uc774\uc724\ubcf4\ub2e4 \uc0dd\uba85\uc774 \uc6b0\uc120\uc774\ub77c\ub294 \uc758\uc0ac\ub85c\uc11c\uc758 \ucd08\uc2ec\uc744 \ud68c\ubcf5\ud558\uac8c \ub41c\ub2e4. \uc6b0\ub974\ud06c\uc5d0\uc11c \uc7ac\ub09c\uacfc \ubd09\uc0ac\ub97c \uacbd\ud5d8\ud558\uace0 \uac15\ubaa8\uc5f0\uc740 \ud788\ud3ec\ud06c\ub77c\ud14c\uc2a4 \uc120\uc11c\ub97c \ub5a0\uc62c\ub9b0\ub2e4. \"\ub098\uc758 \uc0dd\uc560\ub97c \uc778\ub958\ubd09\uc0ac\uc5d0 \ubc14\uce60 \uac83\uc744 \uc5c4\uc219\ud788 \uc11c\uc57d\ud558\ub178\ub77c. \ub098\uc758 \ud658\uc790\uc758 \uac74\uac15\uacfc \uc0dd\uba85\uc744 \uccab\uc9f8\ub85c \uc0dd\uac01\ud558\uaca0\ub178\ub77c. \ub098\ub294 \uc778\uc885, \uc885\uad50, \uad6d\uc801, \uc0ac\ud68c\uc801 \uc9c0\uc704\ub97c \ucd08\uc6d4\ud558\uc5ec \uc624\uc9c1 \ud658\uc790\uc5d0 \ub300\ud55c \ub098\uc758 \uc758\ubb34\ub97c \uc9c0\ud0a4\uaca0\ub178\ub77c. \ube44\ub85d \uc704\ud611\uc744 \ub2f9\ud560\uc9c0\ub77c\ub3c4 \ub098\uc758 \uc9c0\uc2dd\uc744 \uc778\ub3c4\uc5d0 \uc5b4\uae0b\ub098\uac8c \uc4f0\uc9c0 \uc54a\uaca0\ub178\ub77c. (6\ud68c)\"", "paragraph_answer": "\uadf8\ub7ec\ub098 \uac15\ubaa8\uc5f0\uc740 \uc720\uc2dc\uc9c4\uacfc \ud5e4\uc5b4\uc9c4 \ud6c4 \uc0dd\uba85\ubcf4\ub2e4 \uc774\uc724\uc744 \ucd94\uad6c\ud558\ub294 \uc758\uc0ac\uac00 \ub418\uc5b4 \ubc84\ub9b0\ub2e4. \uc774\ud6c4 \uc720\uc2dc\uc9c4\uc740 \ud3c9\ud654 \uc720\uc9c0\ub97c \uc704\ud574 \uc6b0\ub974\ud06c \uc5d0 \ud30c\ubcd1\ub418\uace0, \uac15\ubaa8\uc5f0\uc740 \uc758\ub8cc \ubd09\uc0ac\ucc28 \uadf8\uacf3\uc5d0 \uc628\ub2e4. \uc6b0\ub974\ud06c\ub294 2004\ub144 \ud55c\uad6d\uc774 \ud3c9\ud654\uc720\uc9c0\uad70\uc744 \ud30c\ubcd1\ud55c \uc774\ub77c\ud06c\ub97c \uc0c1\uc9d5\ud55c\ub2e4. \ud604\uc2e4\uacfc \ud5c8\uad6c\uac00 \ub4a4\uc11e\uc778 \uc774\uacf3\uc5d0\uc11c \ub458\uc740 \u201c\uc870\uad6d\uc774 \uc9c0\ucf1c\uc57c \ud560 \uad6d\ubbfc\uc758 \uc778\uad8c\u201d\uc774\ub77c\ub294 \uac00\uce58\ub97c \uacf5\uc720\ud558\uba70 \uc0ac\ub791\uc744 \ud0a4\uc6cc \ub098\uac04\ub2e4. \uac15\ubaa8\uc5f0\uc740 \uc6b0\ub974\ud06c\uc5d0\uc11c\uc758 \uacbd\ud5d8\uc73c\ub85c \uc774\uc724\ubcf4\ub2e4 \uc0dd\uba85\uc774 \uc6b0\uc120\uc774\ub77c\ub294 \uc758\uc0ac\ub85c\uc11c\uc758 \ucd08\uc2ec\uc744 \ud68c\ubcf5\ud558\uac8c \ub41c\ub2e4. \uc6b0\ub974\ud06c\uc5d0\uc11c \uc7ac\ub09c\uacfc \ubd09\uc0ac\ub97c \uacbd\ud5d8\ud558\uace0 \uac15\ubaa8\uc5f0\uc740 \ud788\ud3ec\ud06c\ub77c\ud14c\uc2a4 \uc120\uc11c\ub97c \ub5a0\uc62c\ub9b0\ub2e4. \"\ub098\uc758 \uc0dd\uc560\ub97c \uc778\ub958\ubd09\uc0ac\uc5d0 \ubc14\uce60 \uac83\uc744 \uc5c4\uc219\ud788 \uc11c\uc57d\ud558\ub178\ub77c. \ub098\uc758 \ud658\uc790\uc758 \uac74\uac15\uacfc \uc0dd\uba85\uc744 \uccab\uc9f8\ub85c \uc0dd\uac01\ud558\uaca0\ub178\ub77c. \ub098\ub294 \uc778\uc885, \uc885\uad50, \uad6d\uc801, \uc0ac\ud68c\uc801 \uc9c0\uc704\ub97c \ucd08\uc6d4\ud558\uc5ec \uc624\uc9c1 \ud658\uc790\uc5d0 \ub300\ud55c \ub098\uc758 \uc758\ubb34\ub97c \uc9c0\ud0a4\uaca0\ub178\ub77c. \ube44\ub85d \uc704\ud611\uc744 \ub2f9\ud560\uc9c0\ub77c\ub3c4 \ub098\uc758 \uc9c0\uc2dd\uc744 \uc778\ub3c4\uc5d0 \uc5b4\uae0b\ub098\uac8c \uc4f0\uc9c0 \uc54a\uaca0\ub178\ub77c. (6\ud68c)\"", "sentence_answer": "\uc774\ud6c4 \uc720\uc2dc\uc9c4\uc740 \ud3c9\ud654 \uc720\uc9c0\ub97c \uc704\ud574 \uc6b0\ub974\ud06c \uc5d0 \ud30c\ubcd1\ub418\uace0, \uac15\ubaa8\uc5f0\uc740 \uc758\ub8cc \ubd09\uc0ac\ucc28 \uadf8\uacf3\uc5d0 \uc628\ub2e4.", "paragraph_id": "6598975-2-0", "paragraph_question": "question: \uc720\uc2dc\uc9c4\uc774 \ud3c9\ud654\uc720\uc9c0\uad70\uc73c\ub85c \ud30c\ubcd1\ub41c \uacf3\uc740 \uc5b4\ub514\uc778\uac00?, context: \uadf8\ub7ec\ub098 \uac15\ubaa8\uc5f0\uc740 \uc720\uc2dc\uc9c4\uacfc \ud5e4\uc5b4\uc9c4 \ud6c4 \uc0dd\uba85\ubcf4\ub2e4 \uc774\uc724\uc744 \ucd94\uad6c\ud558\ub294 \uc758\uc0ac\uac00 \ub418\uc5b4 \ubc84\ub9b0\ub2e4. \uc774\ud6c4 \uc720\uc2dc\uc9c4\uc740 \ud3c9\ud654 \uc720\uc9c0\ub97c \uc704\ud574 \uc6b0\ub974\ud06c\uc5d0 \ud30c\ubcd1\ub418\uace0, \uac15\ubaa8\uc5f0\uc740 \uc758\ub8cc \ubd09\uc0ac\ucc28 \uadf8\uacf3\uc5d0 \uc628\ub2e4. \uc6b0\ub974\ud06c\ub294 2004\ub144 \ud55c\uad6d\uc774 \ud3c9\ud654\uc720\uc9c0\uad70\uc744 \ud30c\ubcd1\ud55c \uc774\ub77c\ud06c\ub97c \uc0c1\uc9d5\ud55c\ub2e4. \ud604\uc2e4\uacfc \ud5c8\uad6c\uac00 \ub4a4\uc11e\uc778 \uc774\uacf3\uc5d0\uc11c \ub458\uc740 \u201c\uc870\uad6d\uc774 \uc9c0\ucf1c\uc57c \ud560 \uad6d\ubbfc\uc758 \uc778\uad8c\u201d\uc774\ub77c\ub294 \uac00\uce58\ub97c \uacf5\uc720\ud558\uba70 \uc0ac\ub791\uc744 \ud0a4\uc6cc \ub098\uac04\ub2e4. \uac15\ubaa8\uc5f0\uc740 \uc6b0\ub974\ud06c\uc5d0\uc11c\uc758 \uacbd\ud5d8\uc73c\ub85c \uc774\uc724\ubcf4\ub2e4 \uc0dd\uba85\uc774 \uc6b0\uc120\uc774\ub77c\ub294 \uc758\uc0ac\ub85c\uc11c\uc758 \ucd08\uc2ec\uc744 \ud68c\ubcf5\ud558\uac8c \ub41c\ub2e4. \uc6b0\ub974\ud06c\uc5d0\uc11c \uc7ac\ub09c\uacfc \ubd09\uc0ac\ub97c \uacbd\ud5d8\ud558\uace0 \uac15\ubaa8\uc5f0\uc740 \ud788\ud3ec\ud06c\ub77c\ud14c\uc2a4 \uc120\uc11c\ub97c \ub5a0\uc62c\ub9b0\ub2e4. \"\ub098\uc758 \uc0dd\uc560\ub97c \uc778\ub958\ubd09\uc0ac\uc5d0 \ubc14\uce60 \uac83\uc744 \uc5c4\uc219\ud788 \uc11c\uc57d\ud558\ub178\ub77c. \ub098\uc758 \ud658\uc790\uc758 \uac74\uac15\uacfc \uc0dd\uba85\uc744 \uccab\uc9f8\ub85c \uc0dd\uac01\ud558\uaca0\ub178\ub77c. \ub098\ub294 \uc778\uc885, \uc885\uad50, \uad6d\uc801, \uc0ac\ud68c\uc801 \uc9c0\uc704\ub97c \ucd08\uc6d4\ud558\uc5ec \uc624\uc9c1 \ud658\uc790\uc5d0 \ub300\ud55c \ub098\uc758 \uc758\ubb34\ub97c \uc9c0\ud0a4\uaca0\ub178\ub77c. \ube44\ub85d \uc704\ud611\uc744 \ub2f9\ud560\uc9c0\ub77c\ub3c4 \ub098\uc758 \uc9c0\uc2dd\uc744 \uc778\ub3c4\uc5d0 \uc5b4\uae0b\ub098\uac8c \uc4f0\uc9c0 \uc54a\uaca0\ub178\ub77c. (6\ud68c)\""} {"question": "\uc694\ud55c 10\uc138\uc5d0\uac8c \ub9ac\uc5d0\uc8fc \uc8fc\uad50\uc9c1 \uc2b9\uacc4 \ubb38\uc81c\uc5d0 \uac1c\uc785\ud574 \ub2ec\ub77c\uace0 \uc694\uccad\ud55c \uc778\ubb3c\uc740?", "paragraph": "920\ub144 \uc694\ud55c 10\uc138\ub294 \uc0e4\ub97c 3\uc138\ub85c\ubd80\ud130 \ub9ac\uc5d0\uc8fc \uc8fc\uad50\uc9c1 \uc2b9\uacc4 \ubb38\uc81c\uc5d0 \uac1c\uc785\ud574 \ub2ec\ub77c\ub294 \uc694\uccad\uc744 \ubc1b\uc558\ub2e4. \uc0e4\ub97c 3\uc138\ub294 \ub2f9\ucd08\uc5d0 \ub0b4\uc138\uc6b4 \uc8fc\uad50 \ud6c4\ubcf4\uc790 \ud790\ub450\uc778\uc774 \uadf8\ub97c \ubc30\uc2e0\ud558\uace0 \ud3ed\ub3d9\uc744 \uc77c\uc73c\ud0a8 \ub85c\ud0c0\ub9c1\uae30\uc544 \uacf5\uc791 \uae30\uc140\ubca0\ub974\ud2b8 \ud3b8\uc5d0 \uac00\ub2f4\ud558\uc790, \ub610 \ub2e4\ub978 \ud6c4\ubcf4\uc790\uc778 \ud504\ub94c \uc218\ub3c4\uc6d0\uc758 \ub9ac\ud5e4\ub974\ub85c \uad50\uccb4\ud558\ub824\uace0 \ud588\ub2e4. \uadf8\ub7ec\uc790 \ud790\ub450\uc778\uc740 \ub9ac\ud5e4\ub974\ub97c \uc5b5\ub958\ud558\uc5ec \uc790\uc2e0\uc5d0\uac8c \uc8fc\uad50\uc9c1\uc744 \uc591\ubcf4\ud560 \uac83\uc744 \uac15\uc694\ud588\ub2e4. \uc694\ud55c 10\uc138\ub294 \ud790\ub450\uc778\uacfc \ub9ac\ud5e4\ub974 \ub458 \ub2e4 \ub85c\ub9c8\ub85c \uc18c\ud658\ud558\uc5ec \uc790\ucd08\uc9c0\uc885\uc744 \ub4e4\uc740 \ud6c4, \ub9ac\ud5e4\ub974\uac00 \ub9ac\uc5d0\uc8fc\uc758 \uc8fc\uad50\uc88c\uc5d0 \ucc29\uc88c\ud558\ub294 \uac83\uc744 \uc2b9\uc778\ud558\uace0 \ud790\ub450\uc778\uc744 \ud30c\ubb38\ud558\ub294 \uc870\uce58\ub97c \ucde8\ud558\uc600\ub2e4. 923\ub144 \uc0e4\ub97c 3\uc138\uac00 \ubca0\ub974\ub9dd\ub450\uc544 \ubc31\uc791 \ud5e4\ub974\ubca0\ub974\ud2b8\uc5d0\uac8c \uc0ac\ub85c\uc7a1\ud788\uc790, \uc694\ud55c 10\uc138\ub294 \ub2f9\uc2dc \uc9c0\ub3c4\uc790\ub4e4 \uc911\uc5d0\uc11c \uc720\uc77c\ud558\uac8c \uc0e4\ub97c 3\uc138\uc758 \uad6c\uae08\uc5d0 \ub300\ud574 \ud56d\uc758\ud558\uc600\ub2e4. \uadf8\ub294 \ud5e4\ub974\ubca0\ub974\ud2b8\uc5d0\uac8c \uc0e4\ub97c 3\uc138\ub97c \uc989\uc2dc \ud480\uc5b4\uc8fc\uc9c0 \uc54a\uc73c\uba74 \ud30c\ubb38\ud558\uaca0\ub2e4\uace0 \uacbd\uace0\ud588\uc9c0\ub9cc, \ud5e4\ub974\ubca0\ub974\ud2b8\ub294 \uadf8\uc758 \uacbd\uace0\ub97c \ubb34\uc2dc\ud588\ub2e4. \uad50\ud669\uc758 \uad8c\uc704\ub97c \uacbd\uba78\ud55c \ud5e4\ub974\ubca0\ub974\ud2b8\ub294 925\ub144 \uc790\uc2e0\uc758 5\uc138 \ub41c \uc544\ub4e4 \uc704\uadf8\ub97c \ub7ad\uc2a4\uc758 \ub300\uc8fc\uad50\ub85c \ucc29\uc88c\uc2dc\ucf30\ub2e4. \uc774 \ub54c \ud5e4\ub974\ubca0\ub974\ud2b8\ub294 \uc694\ud55c 10\uc138\uc5d0\uac8c \uc790\uc2e0\uc758 \uc544\ub4e4\uc774 \uc8fc\uad50\ub85c \uc11c\uc784\ub418\ub294 \uac83\uc744 \uc2b9\uc778\ud558\uc9c0 \uc54a\uc744 \uacbd\uc6b0 \uad50\uad6c\ub839\uc744 \ubd84\ud560\ud558\uc5ec \uc790\uc2e0\uc744 \uc9c0\uc9c0\ud558\ub294 \uc5ec\ub7ec \uc138\ub825\uc5d0\uac8c \ubc30\ubd84\ud558\uaca0\ub2e4\uace0 \uc704\ud611\ud558\uc600\ub2e4.", "answer": "\uc0e4\ub97c 3\uc138", "sentence": "920\ub144 \uc694\ud55c 10\uc138\ub294 \uc0e4\ub97c 3\uc138 \ub85c\ubd80\ud130 \ub9ac\uc5d0\uc8fc \uc8fc\uad50\uc9c1 \uc2b9\uacc4 \ubb38\uc81c\uc5d0 \uac1c\uc785\ud574 \ub2ec\ub77c\ub294 \uc694\uccad\uc744 \ubc1b\uc558\ub2e4.", "paragraph_sentence": " 920\ub144 \uc694\ud55c 10\uc138\ub294 \uc0e4\ub97c 3\uc138 \ub85c\ubd80\ud130 \ub9ac\uc5d0\uc8fc \uc8fc\uad50\uc9c1 \uc2b9\uacc4 \ubb38\uc81c\uc5d0 \uac1c\uc785\ud574 \ub2ec\ub77c\ub294 \uc694\uccad\uc744 \ubc1b\uc558\ub2e4. \uc0e4\ub97c 3\uc138\ub294 \ub2f9\ucd08\uc5d0 \ub0b4\uc138\uc6b4 \uc8fc\uad50 \ud6c4\ubcf4\uc790 \ud790\ub450\uc778\uc774 \uadf8\ub97c \ubc30\uc2e0\ud558\uace0 \ud3ed\ub3d9\uc744 \uc77c\uc73c\ud0a8 \ub85c\ud0c0\ub9c1\uae30\uc544 \uacf5\uc791 \uae30\uc140\ubca0\ub974\ud2b8 \ud3b8\uc5d0 \uac00\ub2f4\ud558\uc790, \ub610 \ub2e4\ub978 \ud6c4\ubcf4\uc790\uc778 \ud504\ub94c \uc218\ub3c4\uc6d0\uc758 \ub9ac\ud5e4\ub974\ub85c \uad50\uccb4\ud558\ub824\uace0 \ud588\ub2e4. \uadf8\ub7ec\uc790 \ud790\ub450\uc778\uc740 \ub9ac\ud5e4\ub974\ub97c \uc5b5\ub958\ud558\uc5ec \uc790\uc2e0\uc5d0\uac8c \uc8fc\uad50\uc9c1\uc744 \uc591\ubcf4\ud560 \uac83\uc744 \uac15\uc694\ud588\ub2e4. \uc694\ud55c 10\uc138\ub294 \ud790\ub450\uc778\uacfc \ub9ac\ud5e4\ub974 \ub458 \ub2e4 \ub85c\ub9c8\ub85c \uc18c\ud658\ud558\uc5ec \uc790\ucd08\uc9c0\uc885\uc744 \ub4e4\uc740 \ud6c4, \ub9ac\ud5e4\ub974\uac00 \ub9ac\uc5d0\uc8fc\uc758 \uc8fc\uad50\uc88c\uc5d0 \ucc29\uc88c\ud558\ub294 \uac83\uc744 \uc2b9\uc778\ud558\uace0 \ud790\ub450\uc778\uc744 \ud30c\ubb38\ud558\ub294 \uc870\uce58\ub97c \ucde8\ud558\uc600\ub2e4. 923\ub144 \uc0e4\ub97c 3\uc138\uac00 \ubca0\ub974\ub9dd\ub450\uc544 \ubc31\uc791 \ud5e4\ub974\ubca0\ub974\ud2b8\uc5d0\uac8c \uc0ac\ub85c\uc7a1\ud788\uc790, \uc694\ud55c 10\uc138\ub294 \ub2f9\uc2dc \uc9c0\ub3c4\uc790\ub4e4 \uc911\uc5d0\uc11c \uc720\uc77c\ud558\uac8c \uc0e4\ub97c 3\uc138\uc758 \uad6c\uae08\uc5d0 \ub300\ud574 \ud56d\uc758\ud558\uc600\ub2e4. \uadf8\ub294 \ud5e4\ub974\ubca0\ub974\ud2b8\uc5d0\uac8c \uc0e4\ub97c 3\uc138\ub97c \uc989\uc2dc \ud480\uc5b4\uc8fc\uc9c0 \uc54a\uc73c\uba74 \ud30c\ubb38\ud558\uaca0\ub2e4\uace0 \uacbd\uace0\ud588\uc9c0\ub9cc, \ud5e4\ub974\ubca0\ub974\ud2b8\ub294 \uadf8\uc758 \uacbd\uace0\ub97c \ubb34\uc2dc\ud588\ub2e4. \uad50\ud669\uc758 \uad8c\uc704\ub97c \uacbd\uba78\ud55c \ud5e4\ub974\ubca0\ub974\ud2b8\ub294 925\ub144 \uc790\uc2e0\uc758 5\uc138 \ub41c \uc544\ub4e4 \uc704\uadf8\ub97c \ub7ad\uc2a4\uc758 \ub300\uc8fc\uad50\ub85c \ucc29\uc88c\uc2dc\ucf30\ub2e4. \uc774 \ub54c \ud5e4\ub974\ubca0\ub974\ud2b8\ub294 \uc694\ud55c 10\uc138\uc5d0\uac8c \uc790\uc2e0\uc758 \uc544\ub4e4\uc774 \uc8fc\uad50\ub85c \uc11c\uc784\ub418\ub294 \uac83\uc744 \uc2b9\uc778\ud558\uc9c0 \uc54a\uc744 \uacbd\uc6b0 \uad50\uad6c\ub839\uc744 \ubd84\ud560\ud558\uc5ec \uc790\uc2e0\uc744 \uc9c0\uc9c0\ud558\ub294 \uc5ec\ub7ec \uc138\ub825\uc5d0\uac8c \ubc30\ubd84\ud558\uaca0\ub2e4\uace0 \uc704\ud611\ud558\uc600\ub2e4.", "paragraph_answer": "920\ub144 \uc694\ud55c 10\uc138\ub294 \uc0e4\ub97c 3\uc138 \ub85c\ubd80\ud130 \ub9ac\uc5d0\uc8fc \uc8fc\uad50\uc9c1 \uc2b9\uacc4 \ubb38\uc81c\uc5d0 \uac1c\uc785\ud574 \ub2ec\ub77c\ub294 \uc694\uccad\uc744 \ubc1b\uc558\ub2e4. \uc0e4\ub97c 3\uc138\ub294 \ub2f9\ucd08\uc5d0 \ub0b4\uc138\uc6b4 \uc8fc\uad50 \ud6c4\ubcf4\uc790 \ud790\ub450\uc778\uc774 \uadf8\ub97c \ubc30\uc2e0\ud558\uace0 \ud3ed\ub3d9\uc744 \uc77c\uc73c\ud0a8 \ub85c\ud0c0\ub9c1\uae30\uc544 \uacf5\uc791 \uae30\uc140\ubca0\ub974\ud2b8 \ud3b8\uc5d0 \uac00\ub2f4\ud558\uc790, \ub610 \ub2e4\ub978 \ud6c4\ubcf4\uc790\uc778 \ud504\ub94c \uc218\ub3c4\uc6d0\uc758 \ub9ac\ud5e4\ub974\ub85c \uad50\uccb4\ud558\ub824\uace0 \ud588\ub2e4. \uadf8\ub7ec\uc790 \ud790\ub450\uc778\uc740 \ub9ac\ud5e4\ub974\ub97c \uc5b5\ub958\ud558\uc5ec \uc790\uc2e0\uc5d0\uac8c \uc8fc\uad50\uc9c1\uc744 \uc591\ubcf4\ud560 \uac83\uc744 \uac15\uc694\ud588\ub2e4. \uc694\ud55c 10\uc138\ub294 \ud790\ub450\uc778\uacfc \ub9ac\ud5e4\ub974 \ub458 \ub2e4 \ub85c\ub9c8\ub85c \uc18c\ud658\ud558\uc5ec \uc790\ucd08\uc9c0\uc885\uc744 \ub4e4\uc740 \ud6c4, \ub9ac\ud5e4\ub974\uac00 \ub9ac\uc5d0\uc8fc\uc758 \uc8fc\uad50\uc88c\uc5d0 \ucc29\uc88c\ud558\ub294 \uac83\uc744 \uc2b9\uc778\ud558\uace0 \ud790\ub450\uc778\uc744 \ud30c\ubb38\ud558\ub294 \uc870\uce58\ub97c \ucde8\ud558\uc600\ub2e4. 923\ub144 \uc0e4\ub97c 3\uc138\uac00 \ubca0\ub974\ub9dd\ub450\uc544 \ubc31\uc791 \ud5e4\ub974\ubca0\ub974\ud2b8\uc5d0\uac8c \uc0ac\ub85c\uc7a1\ud788\uc790, \uc694\ud55c 10\uc138\ub294 \ub2f9\uc2dc \uc9c0\ub3c4\uc790\ub4e4 \uc911\uc5d0\uc11c \uc720\uc77c\ud558\uac8c \uc0e4\ub97c 3\uc138\uc758 \uad6c\uae08\uc5d0 \ub300\ud574 \ud56d\uc758\ud558\uc600\ub2e4. \uadf8\ub294 \ud5e4\ub974\ubca0\ub974\ud2b8\uc5d0\uac8c \uc0e4\ub97c 3\uc138\ub97c \uc989\uc2dc \ud480\uc5b4\uc8fc\uc9c0 \uc54a\uc73c\uba74 \ud30c\ubb38\ud558\uaca0\ub2e4\uace0 \uacbd\uace0\ud588\uc9c0\ub9cc, \ud5e4\ub974\ubca0\ub974\ud2b8\ub294 \uadf8\uc758 \uacbd\uace0\ub97c \ubb34\uc2dc\ud588\ub2e4. \uad50\ud669\uc758 \uad8c\uc704\ub97c \uacbd\uba78\ud55c \ud5e4\ub974\ubca0\ub974\ud2b8\ub294 925\ub144 \uc790\uc2e0\uc758 5\uc138 \ub41c \uc544\ub4e4 \uc704\uadf8\ub97c \ub7ad\uc2a4\uc758 \ub300\uc8fc\uad50\ub85c \ucc29\uc88c\uc2dc\ucf30\ub2e4. \uc774 \ub54c \ud5e4\ub974\ubca0\ub974\ud2b8\ub294 \uc694\ud55c 10\uc138\uc5d0\uac8c \uc790\uc2e0\uc758 \uc544\ub4e4\uc774 \uc8fc\uad50\ub85c \uc11c\uc784\ub418\ub294 \uac83\uc744 \uc2b9\uc778\ud558\uc9c0 \uc54a\uc744 \uacbd\uc6b0 \uad50\uad6c\ub839\uc744 \ubd84\ud560\ud558\uc5ec \uc790\uc2e0\uc744 \uc9c0\uc9c0\ud558\ub294 \uc5ec\ub7ec \uc138\ub825\uc5d0\uac8c \ubc30\ubd84\ud558\uaca0\ub2e4\uace0 \uc704\ud611\ud558\uc600\ub2e4.", "sentence_answer": "920\ub144 \uc694\ud55c 10\uc138\ub294 \uc0e4\ub97c 3\uc138 \ub85c\ubd80\ud130 \ub9ac\uc5d0\uc8fc \uc8fc\uad50\uc9c1 \uc2b9\uacc4 \ubb38\uc81c\uc5d0 \uac1c\uc785\ud574 \ub2ec\ub77c\ub294 \uc694\uccad\uc744 \ubc1b\uc558\ub2e4.", "paragraph_id": "6511685-5-0", "paragraph_question": "question: \uc694\ud55c 10\uc138\uc5d0\uac8c \ub9ac\uc5d0\uc8fc \uc8fc\uad50\uc9c1 \uc2b9\uacc4 \ubb38\uc81c\uc5d0 \uac1c\uc785\ud574 \ub2ec\ub77c\uace0 \uc694\uccad\ud55c \uc778\ubb3c\uc740?, context: 920\ub144 \uc694\ud55c 10\uc138\ub294 \uc0e4\ub97c 3\uc138\ub85c\ubd80\ud130 \ub9ac\uc5d0\uc8fc \uc8fc\uad50\uc9c1 \uc2b9\uacc4 \ubb38\uc81c\uc5d0 \uac1c\uc785\ud574 \ub2ec\ub77c\ub294 \uc694\uccad\uc744 \ubc1b\uc558\ub2e4. \uc0e4\ub97c 3\uc138\ub294 \ub2f9\ucd08\uc5d0 \ub0b4\uc138\uc6b4 \uc8fc\uad50 \ud6c4\ubcf4\uc790 \ud790\ub450\uc778\uc774 \uadf8\ub97c \ubc30\uc2e0\ud558\uace0 \ud3ed\ub3d9\uc744 \uc77c\uc73c\ud0a8 \ub85c\ud0c0\ub9c1\uae30\uc544 \uacf5\uc791 \uae30\uc140\ubca0\ub974\ud2b8 \ud3b8\uc5d0 \uac00\ub2f4\ud558\uc790, \ub610 \ub2e4\ub978 \ud6c4\ubcf4\uc790\uc778 \ud504\ub94c \uc218\ub3c4\uc6d0\uc758 \ub9ac\ud5e4\ub974\ub85c \uad50\uccb4\ud558\ub824\uace0 \ud588\ub2e4. \uadf8\ub7ec\uc790 \ud790\ub450\uc778\uc740 \ub9ac\ud5e4\ub974\ub97c \uc5b5\ub958\ud558\uc5ec \uc790\uc2e0\uc5d0\uac8c \uc8fc\uad50\uc9c1\uc744 \uc591\ubcf4\ud560 \uac83\uc744 \uac15\uc694\ud588\ub2e4. \uc694\ud55c 10\uc138\ub294 \ud790\ub450\uc778\uacfc \ub9ac\ud5e4\ub974 \ub458 \ub2e4 \ub85c\ub9c8\ub85c \uc18c\ud658\ud558\uc5ec \uc790\ucd08\uc9c0\uc885\uc744 \ub4e4\uc740 \ud6c4, \ub9ac\ud5e4\ub974\uac00 \ub9ac\uc5d0\uc8fc\uc758 \uc8fc\uad50\uc88c\uc5d0 \ucc29\uc88c\ud558\ub294 \uac83\uc744 \uc2b9\uc778\ud558\uace0 \ud790\ub450\uc778\uc744 \ud30c\ubb38\ud558\ub294 \uc870\uce58\ub97c \ucde8\ud558\uc600\ub2e4. 923\ub144 \uc0e4\ub97c 3\uc138\uac00 \ubca0\ub974\ub9dd\ub450\uc544 \ubc31\uc791 \ud5e4\ub974\ubca0\ub974\ud2b8\uc5d0\uac8c \uc0ac\ub85c\uc7a1\ud788\uc790, \uc694\ud55c 10\uc138\ub294 \ub2f9\uc2dc \uc9c0\ub3c4\uc790\ub4e4 \uc911\uc5d0\uc11c \uc720\uc77c\ud558\uac8c \uc0e4\ub97c 3\uc138\uc758 \uad6c\uae08\uc5d0 \ub300\ud574 \ud56d\uc758\ud558\uc600\ub2e4. \uadf8\ub294 \ud5e4\ub974\ubca0\ub974\ud2b8\uc5d0\uac8c \uc0e4\ub97c 3\uc138\ub97c \uc989\uc2dc \ud480\uc5b4\uc8fc\uc9c0 \uc54a\uc73c\uba74 \ud30c\ubb38\ud558\uaca0\ub2e4\uace0 \uacbd\uace0\ud588\uc9c0\ub9cc, \ud5e4\ub974\ubca0\ub974\ud2b8\ub294 \uadf8\uc758 \uacbd\uace0\ub97c \ubb34\uc2dc\ud588\ub2e4. \uad50\ud669\uc758 \uad8c\uc704\ub97c \uacbd\uba78\ud55c \ud5e4\ub974\ubca0\ub974\ud2b8\ub294 925\ub144 \uc790\uc2e0\uc758 5\uc138 \ub41c \uc544\ub4e4 \uc704\uadf8\ub97c \ub7ad\uc2a4\uc758 \ub300\uc8fc\uad50\ub85c \ucc29\uc88c\uc2dc\ucf30\ub2e4. \uc774 \ub54c \ud5e4\ub974\ubca0\ub974\ud2b8\ub294 \uc694\ud55c 10\uc138\uc5d0\uac8c \uc790\uc2e0\uc758 \uc544\ub4e4\uc774 \uc8fc\uad50\ub85c \uc11c\uc784\ub418\ub294 \uac83\uc744 \uc2b9\uc778\ud558\uc9c0 \uc54a\uc744 \uacbd\uc6b0 \uad50\uad6c\ub839\uc744 \ubd84\ud560\ud558\uc5ec \uc790\uc2e0\uc744 \uc9c0\uc9c0\ud558\ub294 \uc5ec\ub7ec \uc138\ub825\uc5d0\uac8c \ubc30\ubd84\ud558\uaca0\ub2e4\uace0 \uc704\ud611\ud558\uc600\ub2e4."} {"question": "\uc911\uad6d \ubca0\uc774\uc9d5\uc5d0\uc11c \uc5f4\ub9b0 \"\ubca0\uc774\ube44\ubcf5\uc2a4 \uac00\uc694\uc81c\"\uc5d0\uc11c \ubc1c\ud0c1\ub41c SM \uc5d4\ud130\ud14c\uc778\uba3c\ud2b8 \uc18c\uc18d\uc758 \uc911\uad6d\uac00\uc218\ub294 \ub204\uad6c\uc778\uac00?", "paragraph": "\ud55c\ud3b8 \uc911\uad6d \ubca0\uc774\uc9d5\uc5d0\uc11c\ub294 \ubca0\uc774\ube44\ubcf5\uc2a4\uc758 \uc774\ub984\uc744 \ub534 \"\ubca0\uc774\ube44\ubcf5\uc2a4 \uac00\uc694\uc81c\"\uac00 \uac1c\ucd5c\ub418\uc5c8\ub294\ub370 1\ucc28\uc804\uc5d0 \uc57d 4000\uba85\uc774 \ubab0\ub9ac\ub294 \ub300\uc131\ud669\uc744 \uc774\ub8e8\uc5c8\ub2e4. \uadf8\uacf3\uc5d0\uc11c \ubc1c\ud0c1\ub41c SM \uc5d4\ud130\ud14c\uc778\uba3c\ud2b8 \uc18c\uc18d \uc7a5\ub9ac\uc778\uc5d0\uac8c 2007\ub144, DR\ubba4\uc9c1\uc758 \uc724\ub4f1\ub8e1 \ub300\ud45c\ub294 \uc11c\uc6b8 \uc911\uc559\uc9c0\ubc95\uc5d0 \uc7a5\ub9ac\uc778\uacfc \uadf8\uc758 \ud604 \uc18c\uc18d\uc0ac\uc778 SM\uc5d4\ud130\ud14c\uc778\uba3c\ud2b8\ub97c \uc0c1\ub300\ub85c \u201c2\uc5b5\uc6d0\uc758 \uc190\ud574\ub97c \ubc30\uc0c1\ud558\ub77c\u201d\ub294 \ub0b4\uc6a9\uc758 \uc18c\uc7a5\uc744 \uc81c\ucd9c\ud588\ub2e4. \uc7a5\ub9ac\uc778\uc774 \uc18c\uc1a1\uc5d0 \ud718\ub9d0\ub9ac\uac8c \ub41c \uac83\uc740 \uc774\ubbf8 \uc9c0\ub09c 2003\ub144 DR\ubba4\uc9c1\uacfc \ud55c\ucc28\ub840 \uc804\uc18d\uacc4\uc57d\uc744 \uccb4\uacb0\ud55c \ub370\uc11c \ube44\ub86f\ub410\ub2e4. \uc911\uad6d\uacfc \ud55c\uad6d\uc744 \uc624\uac00\uba70 \uac01\uc885 \ud2b8\ub808\uc774\ub2dd\uc744 \uc2dc\ud0a4\uba74\uc11c \uc0c1\ub2f9\ud55c \ube44\uc6a9\uc744 \uc37c\uace0 \uc74c\ubc18 \uc900\ube44\ub3c4 \uc11c\ub450\ub974\uace0 \uc788\uc5c8\ub2e4\u201d\uba74\uc11c \u201c\ud558\uc9c0\ub9cc \uc5b4\uca50 \uc77c\uc778\uc9c0 2003\ub144 8\uc6d4 \uc7a5\ub9ac\uc778\uc758 \uce5c\ubaa8\uac00 \u2018\uacf5\ubd80\ub97c \uacc4\uc18d \uc2dc\ud0a4\uace0 \uc2f6\uace0 \uacf5\ubd80\uac00 \ub05d\ub09c \ud6c4 \uc7ac\uacc4\uc57d\uc744 \uccb4\uacb0\ud558\uaca0\ub2e4\u2019\uace0 \uc694\uccad\ud574\uc640 \uad6d\uc81c\uc801\uc778 \uc2e0\uc758\ub97c \uc0dd\uac01\ud574 \uacc4\uc57d\uc744 \uc77c\uc2dc\uc801\uc73c\ub85c \ud480\uc5b4\uc92c\ub2e4\u201d\uace0 \uc124\uba85\ud588\ub2e4. DR\ubba4\uc9c1\uc740 \uc800\uac04\uc758 \uc0ac\uc815\uc744 \ubc14\ud0d5\uc73c\ub85c \uc9c0\ub09c 1\ub144\uc5ec\uac04 SM\uc5d4\ud130\ud14c\uc778\uba3c\ud2b8\uc640 \uc7a5\ub9ac\uc778\uc744 \uc787\ub2ec\uc544 \uc811\ucd09\ud588\uc73c\ub098 \ud611\uc0c1\uc774 \uc6d0\ud560\uce58 \ubabb\ud574 \uacb0\uad6d \uc18c\uc1a1\uc744 \uc81c\uae30\ud558\uae30\uc5d0 \uc774\ub974\ub800\ub2e4. \ub2f9\uc2dc \uacc4\uc57d\uc11c\uc5d0 \ub530\ub974\uba74 \uc7a5\ub9ac\uc778\uc740 \uacc4\uc57d\ud30c\uae30\uc2dc \uc190\uc2e4\uc561\uacfc \uc704\uc57d\uae08 \ub4f1 \ucd1d 500\ub9cc\uc704\uc548(\uc57d 6\uc5b54000\ub9cc\uc6d0)\uc744 \ubb3c\uc5b4\uc918\uc57c\ud55c\ub2e4\u201d\uba74\uc11c \u201c\ud558\uc9c0\ub9cc \ud55c\uad6d \uc74c\ubc18\uae30\ud68d\uc0ac \uac04\uc758 \uad00\uacc4\ub97c \uace0\ub824\ud574 \uae30\uc874 \uc7a5\ub9ac\uc778\uc5d0\uac8c \ud22c\uc790\ub410\ub358 \ube44\uc6a9\uc5d0 \ud574\ub2f9\ud558\ub294 2\uc5b5\uc6d0\ub9cc \ub3cc\ub824\ubc1b\uc790\ub294 \ucde8\uc9c0\uc5d0\uc11c \uc18c\uc1a1\uc744 \uc9c4\ud589\ud558\uac8c \ub410\ub2e4\u201d\uace0 \ub9d0\ud588\ub2e4.", "answer": "\uc7a5\ub9ac\uc778", "sentence": "\uadf8\uacf3\uc5d0\uc11c \ubc1c\ud0c1\ub41c SM \uc5d4\ud130\ud14c\uc778\uba3c\ud2b8 \uc18c\uc18d \uc7a5\ub9ac\uc778 \uc5d0\uac8c 2007\ub144, DR\ubba4\uc9c1\uc758 \uc724\ub4f1\ub8e1 \ub300\ud45c\ub294 \uc11c\uc6b8 \uc911\uc559\uc9c0\ubc95\uc5d0 \uc7a5\ub9ac\uc778\uacfc \uadf8\uc758 \ud604 \uc18c\uc18d\uc0ac\uc778 SM\uc5d4\ud130\ud14c\uc778\uba3c\ud2b8\ub97c \uc0c1\ub300\ub85c \u201c2\uc5b5\uc6d0\uc758 \uc190\ud574\ub97c \ubc30\uc0c1\ud558\ub77c\u201d\ub294 \ub0b4\uc6a9\uc758 \uc18c\uc7a5\uc744 \uc81c\ucd9c\ud588\ub2e4.", "paragraph_sentence": "\ud55c\ud3b8 \uc911\uad6d \ubca0\uc774\uc9d5\uc5d0\uc11c\ub294 \ubca0\uc774\ube44\ubcf5\uc2a4\uc758 \uc774\ub984\uc744 \ub534 \"\ubca0\uc774\ube44\ubcf5\uc2a4 \uac00\uc694\uc81c\"\uac00 \uac1c\ucd5c\ub418\uc5c8\ub294\ub370 1\ucc28\uc804\uc5d0 \uc57d 4000\uba85\uc774 \ubab0\ub9ac\ub294 \ub300\uc131\ud669\uc744 \uc774\ub8e8\uc5c8\ub2e4. \uadf8\uacf3\uc5d0\uc11c \ubc1c\ud0c1\ub41c SM \uc5d4\ud130\ud14c\uc778\uba3c\ud2b8 \uc18c\uc18d \uc7a5\ub9ac\uc778 \uc5d0\uac8c 2007\ub144, DR\ubba4\uc9c1\uc758 \uc724\ub4f1\ub8e1 \ub300\ud45c\ub294 \uc11c\uc6b8 \uc911\uc559\uc9c0\ubc95\uc5d0 \uc7a5\ub9ac\uc778\uacfc \uadf8\uc758 \ud604 \uc18c\uc18d\uc0ac\uc778 SM\uc5d4\ud130\ud14c\uc778\uba3c\ud2b8\ub97c \uc0c1\ub300\ub85c \u201c2\uc5b5\uc6d0\uc758 \uc190\ud574\ub97c \ubc30\uc0c1\ud558\ub77c\u201d\ub294 \ub0b4\uc6a9\uc758 \uc18c\uc7a5\uc744 \uc81c\ucd9c\ud588\ub2e4. \uc7a5\ub9ac\uc778\uc774 \uc18c\uc1a1\uc5d0 \ud718\ub9d0\ub9ac\uac8c \ub41c \uac83\uc740 \uc774\ubbf8 \uc9c0\ub09c 2003\ub144 DR\ubba4\uc9c1\uacfc \ud55c\ucc28\ub840 \uc804\uc18d\uacc4\uc57d\uc744 \uccb4\uacb0\ud55c \ub370\uc11c \ube44\ub86f\ub410\ub2e4. \uc911\uad6d\uacfc \ud55c\uad6d\uc744 \uc624\uac00\uba70 \uac01\uc885 \ud2b8\ub808\uc774\ub2dd\uc744 \uc2dc\ud0a4\uba74\uc11c \uc0c1\ub2f9\ud55c \ube44\uc6a9\uc744 \uc37c\uace0 \uc74c\ubc18 \uc900\ube44\ub3c4 \uc11c\ub450\ub974\uace0 \uc788\uc5c8\ub2e4\u201d\uba74\uc11c \u201c\ud558\uc9c0\ub9cc \uc5b4\uca50 \uc77c\uc778\uc9c0 2003\ub144 8\uc6d4 \uc7a5\ub9ac\uc778\uc758 \uce5c\ubaa8\uac00 \u2018\uacf5\ubd80\ub97c \uacc4\uc18d \uc2dc\ud0a4\uace0 \uc2f6\uace0 \uacf5\ubd80\uac00 \ub05d\ub09c \ud6c4 \uc7ac\uacc4\uc57d\uc744 \uccb4\uacb0\ud558\uaca0\ub2e4\u2019\uace0 \uc694\uccad\ud574\uc640 \uad6d\uc81c\uc801\uc778 \uc2e0\uc758\ub97c \uc0dd\uac01\ud574 \uacc4\uc57d\uc744 \uc77c\uc2dc\uc801\uc73c\ub85c \ud480\uc5b4\uc92c\ub2e4\u201d\uace0 \uc124\uba85\ud588\ub2e4. DR\ubba4\uc9c1\uc740 \uc800\uac04\uc758 \uc0ac\uc815\uc744 \ubc14\ud0d5\uc73c\ub85c \uc9c0\ub09c 1\ub144\uc5ec\uac04 SM\uc5d4\ud130\ud14c\uc778\uba3c\ud2b8\uc640 \uc7a5\ub9ac\uc778\uc744 \uc787\ub2ec\uc544 \uc811\ucd09\ud588\uc73c\ub098 \ud611\uc0c1\uc774 \uc6d0\ud560\uce58 \ubabb\ud574 \uacb0\uad6d \uc18c\uc1a1\uc744 \uc81c\uae30\ud558\uae30\uc5d0 \uc774\ub974\ub800\ub2e4. \ub2f9\uc2dc \uacc4\uc57d\uc11c\uc5d0 \ub530\ub974\uba74 \uc7a5\ub9ac\uc778\uc740 \uacc4\uc57d\ud30c\uae30\uc2dc \uc190\uc2e4\uc561\uacfc \uc704\uc57d\uae08 \ub4f1 \ucd1d 500\ub9cc\uc704\uc548(\uc57d 6\uc5b54000\ub9cc\uc6d0)\uc744 \ubb3c\uc5b4\uc918\uc57c\ud55c\ub2e4\u201d\uba74\uc11c \u201c\ud558\uc9c0\ub9cc \ud55c\uad6d \uc74c\ubc18\uae30\ud68d\uc0ac \uac04\uc758 \uad00\uacc4\ub97c \uace0\ub824\ud574 \uae30\uc874 \uc7a5\ub9ac\uc778\uc5d0\uac8c \ud22c\uc790\ub410\ub358 \ube44\uc6a9\uc5d0 \ud574\ub2f9\ud558\ub294 2\uc5b5\uc6d0\ub9cc \ub3cc\ub824\ubc1b\uc790\ub294 \ucde8\uc9c0\uc5d0\uc11c \uc18c\uc1a1\uc744 \uc9c4\ud589\ud558\uac8c \ub410\ub2e4\u201d\uace0 \ub9d0\ud588\ub2e4.", "paragraph_answer": "\ud55c\ud3b8 \uc911\uad6d \ubca0\uc774\uc9d5\uc5d0\uc11c\ub294 \ubca0\uc774\ube44\ubcf5\uc2a4\uc758 \uc774\ub984\uc744 \ub534 \"\ubca0\uc774\ube44\ubcf5\uc2a4 \uac00\uc694\uc81c\"\uac00 \uac1c\ucd5c\ub418\uc5c8\ub294\ub370 1\ucc28\uc804\uc5d0 \uc57d 4000\uba85\uc774 \ubab0\ub9ac\ub294 \ub300\uc131\ud669\uc744 \uc774\ub8e8\uc5c8\ub2e4. \uadf8\uacf3\uc5d0\uc11c \ubc1c\ud0c1\ub41c SM \uc5d4\ud130\ud14c\uc778\uba3c\ud2b8 \uc18c\uc18d \uc7a5\ub9ac\uc778 \uc5d0\uac8c 2007\ub144, DR\ubba4\uc9c1\uc758 \uc724\ub4f1\ub8e1 \ub300\ud45c\ub294 \uc11c\uc6b8 \uc911\uc559\uc9c0\ubc95\uc5d0 \uc7a5\ub9ac\uc778\uacfc \uadf8\uc758 \ud604 \uc18c\uc18d\uc0ac\uc778 SM\uc5d4\ud130\ud14c\uc778\uba3c\ud2b8\ub97c \uc0c1\ub300\ub85c \u201c2\uc5b5\uc6d0\uc758 \uc190\ud574\ub97c \ubc30\uc0c1\ud558\ub77c\u201d\ub294 \ub0b4\uc6a9\uc758 \uc18c\uc7a5\uc744 \uc81c\ucd9c\ud588\ub2e4. \uc7a5\ub9ac\uc778\uc774 \uc18c\uc1a1\uc5d0 \ud718\ub9d0\ub9ac\uac8c \ub41c \uac83\uc740 \uc774\ubbf8 \uc9c0\ub09c 2003\ub144 DR\ubba4\uc9c1\uacfc \ud55c\ucc28\ub840 \uc804\uc18d\uacc4\uc57d\uc744 \uccb4\uacb0\ud55c \ub370\uc11c \ube44\ub86f\ub410\ub2e4. \uc911\uad6d\uacfc \ud55c\uad6d\uc744 \uc624\uac00\uba70 \uac01\uc885 \ud2b8\ub808\uc774\ub2dd\uc744 \uc2dc\ud0a4\uba74\uc11c \uc0c1\ub2f9\ud55c \ube44\uc6a9\uc744 \uc37c\uace0 \uc74c\ubc18 \uc900\ube44\ub3c4 \uc11c\ub450\ub974\uace0 \uc788\uc5c8\ub2e4\u201d\uba74\uc11c \u201c\ud558\uc9c0\ub9cc \uc5b4\uca50 \uc77c\uc778\uc9c0 2003\ub144 8\uc6d4 \uc7a5\ub9ac\uc778\uc758 \uce5c\ubaa8\uac00 \u2018\uacf5\ubd80\ub97c \uacc4\uc18d \uc2dc\ud0a4\uace0 \uc2f6\uace0 \uacf5\ubd80\uac00 \ub05d\ub09c \ud6c4 \uc7ac\uacc4\uc57d\uc744 \uccb4\uacb0\ud558\uaca0\ub2e4\u2019\uace0 \uc694\uccad\ud574\uc640 \uad6d\uc81c\uc801\uc778 \uc2e0\uc758\ub97c \uc0dd\uac01\ud574 \uacc4\uc57d\uc744 \uc77c\uc2dc\uc801\uc73c\ub85c \ud480\uc5b4\uc92c\ub2e4\u201d\uace0 \uc124\uba85\ud588\ub2e4. DR\ubba4\uc9c1\uc740 \uc800\uac04\uc758 \uc0ac\uc815\uc744 \ubc14\ud0d5\uc73c\ub85c \uc9c0\ub09c 1\ub144\uc5ec\uac04 SM\uc5d4\ud130\ud14c\uc778\uba3c\ud2b8\uc640 \uc7a5\ub9ac\uc778\uc744 \uc787\ub2ec\uc544 \uc811\ucd09\ud588\uc73c\ub098 \ud611\uc0c1\uc774 \uc6d0\ud560\uce58 \ubabb\ud574 \uacb0\uad6d \uc18c\uc1a1\uc744 \uc81c\uae30\ud558\uae30\uc5d0 \uc774\ub974\ub800\ub2e4. \ub2f9\uc2dc \uacc4\uc57d\uc11c\uc5d0 \ub530\ub974\uba74 \uc7a5\ub9ac\uc778\uc740 \uacc4\uc57d\ud30c\uae30\uc2dc \uc190\uc2e4\uc561\uacfc \uc704\uc57d\uae08 \ub4f1 \ucd1d 500\ub9cc\uc704\uc548(\uc57d 6\uc5b54000\ub9cc\uc6d0)\uc744 \ubb3c\uc5b4\uc918\uc57c\ud55c\ub2e4\u201d\uba74\uc11c \u201c\ud558\uc9c0\ub9cc \ud55c\uad6d \uc74c\ubc18\uae30\ud68d\uc0ac \uac04\uc758 \uad00\uacc4\ub97c \uace0\ub824\ud574 \uae30\uc874 \uc7a5\ub9ac\uc778\uc5d0\uac8c \ud22c\uc790\ub410\ub358 \ube44\uc6a9\uc5d0 \ud574\ub2f9\ud558\ub294 2\uc5b5\uc6d0\ub9cc \ub3cc\ub824\ubc1b\uc790\ub294 \ucde8\uc9c0\uc5d0\uc11c \uc18c\uc1a1\uc744 \uc9c4\ud589\ud558\uac8c \ub410\ub2e4\u201d\uace0 \ub9d0\ud588\ub2e4.", "sentence_answer": "\uadf8\uacf3\uc5d0\uc11c \ubc1c\ud0c1\ub41c SM \uc5d4\ud130\ud14c\uc778\uba3c\ud2b8 \uc18c\uc18d \uc7a5\ub9ac\uc778 \uc5d0\uac8c 2007\ub144, DR\ubba4\uc9c1\uc758 \uc724\ub4f1\ub8e1 \ub300\ud45c\ub294 \uc11c\uc6b8 \uc911\uc559\uc9c0\ubc95\uc5d0 \uc7a5\ub9ac\uc778\uacfc \uadf8\uc758 \ud604 \uc18c\uc18d\uc0ac\uc778 SM\uc5d4\ud130\ud14c\uc778\uba3c\ud2b8\ub97c \uc0c1\ub300\ub85c \u201c2\uc5b5\uc6d0\uc758 \uc190\ud574\ub97c \ubc30\uc0c1\ud558\ub77c\u201d\ub294 \ub0b4\uc6a9\uc758 \uc18c\uc7a5\uc744 \uc81c\ucd9c\ud588\ub2e4.", "paragraph_id": "6110374-18-1", "paragraph_question": "question: \uc911\uad6d \ubca0\uc774\uc9d5\uc5d0\uc11c \uc5f4\ub9b0 \"\ubca0\uc774\ube44\ubcf5\uc2a4 \uac00\uc694\uc81c\"\uc5d0\uc11c \ubc1c\ud0c1\ub41c SM \uc5d4\ud130\ud14c\uc778\uba3c\ud2b8 \uc18c\uc18d\uc758 \uc911\uad6d\uac00\uc218\ub294 \ub204\uad6c\uc778\uac00?, context: \ud55c\ud3b8 \uc911\uad6d \ubca0\uc774\uc9d5\uc5d0\uc11c\ub294 \ubca0\uc774\ube44\ubcf5\uc2a4\uc758 \uc774\ub984\uc744 \ub534 \"\ubca0\uc774\ube44\ubcf5\uc2a4 \uac00\uc694\uc81c\"\uac00 \uac1c\ucd5c\ub418\uc5c8\ub294\ub370 1\ucc28\uc804\uc5d0 \uc57d 4000\uba85\uc774 \ubab0\ub9ac\ub294 \ub300\uc131\ud669\uc744 \uc774\ub8e8\uc5c8\ub2e4. \uadf8\uacf3\uc5d0\uc11c \ubc1c\ud0c1\ub41c SM \uc5d4\ud130\ud14c\uc778\uba3c\ud2b8 \uc18c\uc18d \uc7a5\ub9ac\uc778\uc5d0\uac8c 2007\ub144, DR\ubba4\uc9c1\uc758 \uc724\ub4f1\ub8e1 \ub300\ud45c\ub294 \uc11c\uc6b8 \uc911\uc559\uc9c0\ubc95\uc5d0 \uc7a5\ub9ac\uc778\uacfc \uadf8\uc758 \ud604 \uc18c\uc18d\uc0ac\uc778 SM\uc5d4\ud130\ud14c\uc778\uba3c\ud2b8\ub97c \uc0c1\ub300\ub85c \u201c2\uc5b5\uc6d0\uc758 \uc190\ud574\ub97c \ubc30\uc0c1\ud558\ub77c\u201d\ub294 \ub0b4\uc6a9\uc758 \uc18c\uc7a5\uc744 \uc81c\ucd9c\ud588\ub2e4. \uc7a5\ub9ac\uc778\uc774 \uc18c\uc1a1\uc5d0 \ud718\ub9d0\ub9ac\uac8c \ub41c \uac83\uc740 \uc774\ubbf8 \uc9c0\ub09c 2003\ub144 DR\ubba4\uc9c1\uacfc \ud55c\ucc28\ub840 \uc804\uc18d\uacc4\uc57d\uc744 \uccb4\uacb0\ud55c \ub370\uc11c \ube44\ub86f\ub410\ub2e4. \uc911\uad6d\uacfc \ud55c\uad6d\uc744 \uc624\uac00\uba70 \uac01\uc885 \ud2b8\ub808\uc774\ub2dd\uc744 \uc2dc\ud0a4\uba74\uc11c \uc0c1\ub2f9\ud55c \ube44\uc6a9\uc744 \uc37c\uace0 \uc74c\ubc18 \uc900\ube44\ub3c4 \uc11c\ub450\ub974\uace0 \uc788\uc5c8\ub2e4\u201d\uba74\uc11c \u201c\ud558\uc9c0\ub9cc \uc5b4\uca50 \uc77c\uc778\uc9c0 2003\ub144 8\uc6d4 \uc7a5\ub9ac\uc778\uc758 \uce5c\ubaa8\uac00 \u2018\uacf5\ubd80\ub97c \uacc4\uc18d \uc2dc\ud0a4\uace0 \uc2f6\uace0 \uacf5\ubd80\uac00 \ub05d\ub09c \ud6c4 \uc7ac\uacc4\uc57d\uc744 \uccb4\uacb0\ud558\uaca0\ub2e4\u2019\uace0 \uc694\uccad\ud574\uc640 \uad6d\uc81c\uc801\uc778 \uc2e0\uc758\ub97c \uc0dd\uac01\ud574 \uacc4\uc57d\uc744 \uc77c\uc2dc\uc801\uc73c\ub85c \ud480\uc5b4\uc92c\ub2e4\u201d\uace0 \uc124\uba85\ud588\ub2e4. DR\ubba4\uc9c1\uc740 \uc800\uac04\uc758 \uc0ac\uc815\uc744 \ubc14\ud0d5\uc73c\ub85c \uc9c0\ub09c 1\ub144\uc5ec\uac04 SM\uc5d4\ud130\ud14c\uc778\uba3c\ud2b8\uc640 \uc7a5\ub9ac\uc778\uc744 \uc787\ub2ec\uc544 \uc811\ucd09\ud588\uc73c\ub098 \ud611\uc0c1\uc774 \uc6d0\ud560\uce58 \ubabb\ud574 \uacb0\uad6d \uc18c\uc1a1\uc744 \uc81c\uae30\ud558\uae30\uc5d0 \uc774\ub974\ub800\ub2e4. \ub2f9\uc2dc \uacc4\uc57d\uc11c\uc5d0 \ub530\ub974\uba74 \uc7a5\ub9ac\uc778\uc740 \uacc4\uc57d\ud30c\uae30\uc2dc \uc190\uc2e4\uc561\uacfc \uc704\uc57d\uae08 \ub4f1 \ucd1d 500\ub9cc\uc704\uc548(\uc57d 6\uc5b54000\ub9cc\uc6d0)\uc744 \ubb3c\uc5b4\uc918\uc57c\ud55c\ub2e4\u201d\uba74\uc11c \u201c\ud558\uc9c0\ub9cc \ud55c\uad6d \uc74c\ubc18\uae30\ud68d\uc0ac \uac04\uc758 \uad00\uacc4\ub97c \uace0\ub824\ud574 \uae30\uc874 \uc7a5\ub9ac\uc778\uc5d0\uac8c \ud22c\uc790\ub410\ub358 \ube44\uc6a9\uc5d0 \ud574\ub2f9\ud558\ub294 2\uc5b5\uc6d0\ub9cc \ub3cc\ub824\ubc1b\uc790\ub294 \ucde8\uc9c0\uc5d0\uc11c \uc18c\uc1a1\uc744 \uc9c4\ud589\ud558\uac8c \ub410\ub2e4\u201d\uace0 \ub9d0\ud588\ub2e4."} {"question": "\uc774\uc900\uc6a9\uc5d0\uac8c \ud070\uc77c\uc744 \ub3c4\ubaa8\ud558\uae30 \uc704\ud574\uc11c\ub294 \ub54c\ub97c \ub193\uce58\uc9c0 \ub9d0\uc544\uc544 \ud55c\ub2e4\uace0 \ub9d0\ud55c \uc0ac\ub78c\uc740?", "paragraph": "\ub300\uc6d0\uad70\ud30c\uac00 \ub18d\ubbfc\uad70\uc744 \uc0c1\uacbd\uc2dc\ud0a4\uace0, \uccad\uad6d\uad70\uc744 \ub04c\uc5b4\ub4e4\uc5ec \uc77c\ubcf8\uad70\uc744 \uaca9\ud1f4\ud558\uace0 \uac1c\ud654\ud30c\ub97c \uc81c\uac70\ud558\uc5ec \uc815\uad8c\uc744 \uc7a5\uc545\ud558\ub824\ub294 \uacc4\ud68d\uc740 \ub2f9\ucd08 \uc774\uc900\uc6a9, \uc774\ud0dc\uc6a9, \ubc15\uc900\uc591\uc758 \uc2dc\uad6d\ub300\ucc98 \ub17c\uc758 \uc18d\uc5d0\uc11c \uadf8 \uc724\uacfd\uc774 \uc9dc\uc5ec\uc84c\ub2e4. \ub2f9\uc2dc \ubc15\uc900\uc591\uc740 \uc774\uc900\uc6a9\uc5d0\uac8c \uad00\uc9c1\uc744 \uc26c\uace0 \uc678\uad6d\uc73c\ub85c \ub098\uac00 10\ub144 \ub3d9\uc548 \uacac\ubb38\uc744 \ub113\ud600 \uba85\ub9dd\uc744 \uc5bb\uc740 \uc774\ud6c4\uc5d0 \ub3cc\uc544\uc62c \uac83\uc744 \uad8c\ud558\uc600\ub2e4. \uadf8 \ub3d9\uc548 \uace0\uc885\uc740 \ub178\uc1e0\ud558\uac8c \ub418\uace0 \uc655\uc138\uc790(\uc21c\uc885)\ub3c4 \uadf8\ub2e4\uc9c0 \ud070 \ub355\uc774 \uc5c6\uc73c\ub2c8 \uadf8\ub54c\uc5d0 \uc678\uad6d \uba85\ub9dd\uacfc \uad6d\ub0b4\uc758 \uad00\uc2ec\uc740 \uc790\uc5f0\ud788 \uc774\uc900\uc6a9\uc5d0\uac8c \uc3e0\ub9b4 \uac83\uc774\uace0, \uadf8\ub7ec\uba74 \uadf8\ub2e4\uc9c0 \ub178\ub825\ud558\uc9c0 \uc54a\uace0\ub3c4 \uad8c\ub825\uc744 \uc958 \uc218 \uc788\uc744 \uac83\uc774\ub77c\uace0 \ud558\uc600\ub2e4. \uadf8\ub7ec\ub098 \uc774\ud0dc\uc6a9\uc740 \ud070\uc77c\uc744 \ub3c4\ubaa8\ud560 \uacbd\uc6b0\uc5d0\ub294 \ub54c\ub97c \ub193\uce58\uc9c0 \ub9d0\uc544\uc57c \ud55c\ub2e4\uace0 \ud558\uba74\uc11c, \uc5ec\ub7ec \uc0ac\ub78c\uc758 \uae30\ub300\uac00 \ubaa8\ub450 \ub300\uc6d0\uad70\uc744 \ud5a5\ud558\uace0 \uc788\uace0, \ub354\uc6b1\uc774 \ub3d9\ud559\uad70\uc774 \ub300\uc6d0\uad70\uc744 \ubc1b\ub4e4\uaca0\ub2e4\ub294 \uc8fc\uc7a5\uc744 \ud3b4\uba74\uc11c \ubd09\uae30\ud558\uace0 \uc788\ub294 \uc9c0\uae08, \uadf8\ub4e4\ub85c \ud558\uc5ec\uae08 \uc218\uc2ed\ub9cc \ub300\uc911\uc744 \ub3d9\uc6d0\ud558\uc5ec \uc62c\ub77c\uc624\uac8c \ud55c\ub2e4\uba74 \uc9c4\uc2e4\ub85c \uc0ac\ub78c\ub4e4\uc758 \ubb34\ub9ac\uac00 \ud558\ub298\uc744 \uc774\uae34\ub2e4\uace0 \ud558\ub4ef\uc774 \uc77c\ubcf8\uad70\ub300\uac00 \ube44\ub85d \uc6c0\uc9c1\uc778\ub2e4\uace0 \ud558\ub354\ub77c\ub3c4 \uc5b4\ucc0c\ud560 \uc218 \uc5c6\uc744 \uac83\uc774\ub77c\uace0 \ud558\uc600\ub2e4. \uc774\uc5b4 \uc774\ud0dc\uc6a9\uc740 \uc774\uc900\uc6a9\uc5d0\uac8c \ud55c\ud3b8\uc73c\ub85c \uc77c\ubcf8\uad70\uc744 \ub9cc\ub958\ud558\uc5ec \uc6c0\uc9c1\uc774\uc9c0 \ubabb\ud558\uac8c \ud558\uace0, \ub2e4\ub978 \ud55c\ud3b8\uc73c\ub85c \ubc24\uc744 \uc7ac\ucd09\ud558\uc5ec \uad70\uc911\uc744 \uc62c\ub77c\uc624\uac8c \ud55c\ub2e4\uba74 \uc190\ubc14\ub2e5\uc744 \ub4a4\uc9d1\ub4ef\uc774 \uc77c\uc774 \uc26c\uc6cc\uc9c8 \uac83\uc774\ub77c\uace0 \uc81c\uc548\ud558\uc600\ub2e4.", "answer": "\uc774\ud0dc\uc6a9", "sentence": "\ub300\uc6d0\uad70\ud30c\uac00 \ub18d\ubbfc\uad70\uc744 \uc0c1\uacbd\uc2dc\ud0a4\uace0, \uccad\uad6d\uad70\uc744 \ub04c\uc5b4\ub4e4\uc5ec \uc77c\ubcf8\uad70\uc744 \uaca9\ud1f4\ud558\uace0 \uac1c\ud654\ud30c\ub97c \uc81c\uac70\ud558\uc5ec \uc815\uad8c\uc744 \uc7a5\uc545\ud558\ub824\ub294 \uacc4\ud68d\uc740 \ub2f9\ucd08 \uc774\uc900\uc6a9, \uc774\ud0dc\uc6a9 ,", "paragraph_sentence": " \ub300\uc6d0\uad70\ud30c\uac00 \ub18d\ubbfc\uad70\uc744 \uc0c1\uacbd\uc2dc\ud0a4\uace0, \uccad\uad6d\uad70\uc744 \ub04c\uc5b4\ub4e4\uc5ec \uc77c\ubcf8\uad70\uc744 \uaca9\ud1f4\ud558\uace0 \uac1c\ud654\ud30c\ub97c \uc81c\uac70\ud558\uc5ec \uc815\uad8c\uc744 \uc7a5\uc545\ud558\ub824\ub294 \uacc4\ud68d\uc740 \ub2f9\ucd08 \uc774\uc900\uc6a9, \uc774\ud0dc\uc6a9 , \ubc15\uc900\uc591\uc758 \uc2dc\uad6d\ub300\ucc98 \ub17c\uc758 \uc18d\uc5d0\uc11c \uadf8 \uc724\uacfd\uc774 \uc9dc\uc5ec\uc84c\ub2e4. \ub2f9\uc2dc \ubc15\uc900\uc591\uc740 \uc774\uc900\uc6a9\uc5d0\uac8c \uad00\uc9c1\uc744 \uc26c\uace0 \uc678\uad6d\uc73c\ub85c \ub098\uac00 10\ub144 \ub3d9\uc548 \uacac\ubb38\uc744 \ub113\ud600 \uba85\ub9dd\uc744 \uc5bb\uc740 \uc774\ud6c4\uc5d0 \ub3cc\uc544\uc62c \uac83\uc744 \uad8c\ud558\uc600\ub2e4. \uadf8 \ub3d9\uc548 \uace0\uc885\uc740 \ub178\uc1e0\ud558\uac8c \ub418\uace0 \uc655\uc138\uc790(\uc21c\uc885)\ub3c4 \uadf8\ub2e4\uc9c0 \ud070 \ub355\uc774 \uc5c6\uc73c\ub2c8 \uadf8\ub54c\uc5d0 \uc678\uad6d \uba85\ub9dd\uacfc \uad6d\ub0b4\uc758 \uad00\uc2ec\uc740 \uc790\uc5f0\ud788 \uc774\uc900\uc6a9\uc5d0\uac8c \uc3e0\ub9b4 \uac83\uc774\uace0, \uadf8\ub7ec\uba74 \uadf8\ub2e4\uc9c0 \ub178\ub825\ud558\uc9c0 \uc54a\uace0\ub3c4 \uad8c\ub825\uc744 \uc958 \uc218 \uc788\uc744 \uac83\uc774\ub77c\uace0 \ud558\uc600\ub2e4. \uadf8\ub7ec\ub098 \uc774\ud0dc\uc6a9\uc740 \ud070\uc77c\uc744 \ub3c4\ubaa8\ud560 \uacbd\uc6b0\uc5d0\ub294 \ub54c\ub97c \ub193\uce58\uc9c0 \ub9d0\uc544\uc57c \ud55c\ub2e4\uace0 \ud558\uba74\uc11c, \uc5ec\ub7ec \uc0ac\ub78c\uc758 \uae30\ub300\uac00 \ubaa8\ub450 \ub300\uc6d0\uad70\uc744 \ud5a5\ud558\uace0 \uc788\uace0, \ub354\uc6b1\uc774 \ub3d9\ud559\uad70\uc774 \ub300\uc6d0\uad70\uc744 \ubc1b\ub4e4\uaca0\ub2e4\ub294 \uc8fc\uc7a5\uc744 \ud3b4\uba74\uc11c \ubd09\uae30\ud558\uace0 \uc788\ub294 \uc9c0\uae08, \uadf8\ub4e4\ub85c \ud558\uc5ec\uae08 \uc218\uc2ed\ub9cc \ub300\uc911\uc744 \ub3d9\uc6d0\ud558\uc5ec \uc62c\ub77c\uc624\uac8c \ud55c\ub2e4\uba74 \uc9c4\uc2e4\ub85c \uc0ac\ub78c\ub4e4\uc758 \ubb34\ub9ac\uac00 \ud558\ub298\uc744 \uc774\uae34\ub2e4\uace0 \ud558\ub4ef\uc774 \uc77c\ubcf8\uad70\ub300\uac00 \ube44\ub85d \uc6c0\uc9c1\uc778\ub2e4\uace0 \ud558\ub354\ub77c\ub3c4 \uc5b4\ucc0c\ud560 \uc218 \uc5c6\uc744 \uac83\uc774\ub77c\uace0 \ud558\uc600\ub2e4. \uc774\uc5b4 \uc774\ud0dc\uc6a9\uc740 \uc774\uc900\uc6a9\uc5d0\uac8c \ud55c\ud3b8\uc73c\ub85c \uc77c\ubcf8\uad70\uc744 \ub9cc\ub958\ud558\uc5ec \uc6c0\uc9c1\uc774\uc9c0 \ubabb\ud558\uac8c \ud558\uace0, \ub2e4\ub978 \ud55c\ud3b8\uc73c\ub85c \ubc24\uc744 \uc7ac\ucd09\ud558\uc5ec \uad70\uc911\uc744 \uc62c\ub77c\uc624\uac8c \ud55c\ub2e4\uba74 \uc190\ubc14\ub2e5\uc744 \ub4a4\uc9d1\ub4ef\uc774 \uc77c\uc774 \uc26c\uc6cc\uc9c8 \uac83\uc774\ub77c\uace0 \uc81c\uc548\ud558\uc600\ub2e4.", "paragraph_answer": "\ub300\uc6d0\uad70\ud30c\uac00 \ub18d\ubbfc\uad70\uc744 \uc0c1\uacbd\uc2dc\ud0a4\uace0, \uccad\uad6d\uad70\uc744 \ub04c\uc5b4\ub4e4\uc5ec \uc77c\ubcf8\uad70\uc744 \uaca9\ud1f4\ud558\uace0 \uac1c\ud654\ud30c\ub97c \uc81c\uac70\ud558\uc5ec \uc815\uad8c\uc744 \uc7a5\uc545\ud558\ub824\ub294 \uacc4\ud68d\uc740 \ub2f9\ucd08 \uc774\uc900\uc6a9, \uc774\ud0dc\uc6a9 , \ubc15\uc900\uc591\uc758 \uc2dc\uad6d\ub300\ucc98 \ub17c\uc758 \uc18d\uc5d0\uc11c \uadf8 \uc724\uacfd\uc774 \uc9dc\uc5ec\uc84c\ub2e4. \ub2f9\uc2dc \ubc15\uc900\uc591\uc740 \uc774\uc900\uc6a9\uc5d0\uac8c \uad00\uc9c1\uc744 \uc26c\uace0 \uc678\uad6d\uc73c\ub85c \ub098\uac00 10\ub144 \ub3d9\uc548 \uacac\ubb38\uc744 \ub113\ud600 \uba85\ub9dd\uc744 \uc5bb\uc740 \uc774\ud6c4\uc5d0 \ub3cc\uc544\uc62c \uac83\uc744 \uad8c\ud558\uc600\ub2e4. \uadf8 \ub3d9\uc548 \uace0\uc885\uc740 \ub178\uc1e0\ud558\uac8c \ub418\uace0 \uc655\uc138\uc790(\uc21c\uc885)\ub3c4 \uadf8\ub2e4\uc9c0 \ud070 \ub355\uc774 \uc5c6\uc73c\ub2c8 \uadf8\ub54c\uc5d0 \uc678\uad6d \uba85\ub9dd\uacfc \uad6d\ub0b4\uc758 \uad00\uc2ec\uc740 \uc790\uc5f0\ud788 \uc774\uc900\uc6a9\uc5d0\uac8c \uc3e0\ub9b4 \uac83\uc774\uace0, \uadf8\ub7ec\uba74 \uadf8\ub2e4\uc9c0 \ub178\ub825\ud558\uc9c0 \uc54a\uace0\ub3c4 \uad8c\ub825\uc744 \uc958 \uc218 \uc788\uc744 \uac83\uc774\ub77c\uace0 \ud558\uc600\ub2e4. \uadf8\ub7ec\ub098 \uc774\ud0dc\uc6a9\uc740 \ud070\uc77c\uc744 \ub3c4\ubaa8\ud560 \uacbd\uc6b0\uc5d0\ub294 \ub54c\ub97c \ub193\uce58\uc9c0 \ub9d0\uc544\uc57c \ud55c\ub2e4\uace0 \ud558\uba74\uc11c, \uc5ec\ub7ec \uc0ac\ub78c\uc758 \uae30\ub300\uac00 \ubaa8\ub450 \ub300\uc6d0\uad70\uc744 \ud5a5\ud558\uace0 \uc788\uace0, \ub354\uc6b1\uc774 \ub3d9\ud559\uad70\uc774 \ub300\uc6d0\uad70\uc744 \ubc1b\ub4e4\uaca0\ub2e4\ub294 \uc8fc\uc7a5\uc744 \ud3b4\uba74\uc11c \ubd09\uae30\ud558\uace0 \uc788\ub294 \uc9c0\uae08, \uadf8\ub4e4\ub85c \ud558\uc5ec\uae08 \uc218\uc2ed\ub9cc \ub300\uc911\uc744 \ub3d9\uc6d0\ud558\uc5ec \uc62c\ub77c\uc624\uac8c \ud55c\ub2e4\uba74 \uc9c4\uc2e4\ub85c \uc0ac\ub78c\ub4e4\uc758 \ubb34\ub9ac\uac00 \ud558\ub298\uc744 \uc774\uae34\ub2e4\uace0 \ud558\ub4ef\uc774 \uc77c\ubcf8\uad70\ub300\uac00 \ube44\ub85d \uc6c0\uc9c1\uc778\ub2e4\uace0 \ud558\ub354\ub77c\ub3c4 \uc5b4\ucc0c\ud560 \uc218 \uc5c6\uc744 \uac83\uc774\ub77c\uace0 \ud558\uc600\ub2e4. \uc774\uc5b4 \uc774\ud0dc\uc6a9\uc740 \uc774\uc900\uc6a9\uc5d0\uac8c \ud55c\ud3b8\uc73c\ub85c \uc77c\ubcf8\uad70\uc744 \ub9cc\ub958\ud558\uc5ec \uc6c0\uc9c1\uc774\uc9c0 \ubabb\ud558\uac8c \ud558\uace0, \ub2e4\ub978 \ud55c\ud3b8\uc73c\ub85c \ubc24\uc744 \uc7ac\ucd09\ud558\uc5ec \uad70\uc911\uc744 \uc62c\ub77c\uc624\uac8c \ud55c\ub2e4\uba74 \uc190\ubc14\ub2e5\uc744 \ub4a4\uc9d1\ub4ef\uc774 \uc77c\uc774 \uc26c\uc6cc\uc9c8 \uac83\uc774\ub77c\uace0 \uc81c\uc548\ud558\uc600\ub2e4.", "sentence_answer": "\ub300\uc6d0\uad70\ud30c\uac00 \ub18d\ubbfc\uad70\uc744 \uc0c1\uacbd\uc2dc\ud0a4\uace0, \uccad\uad6d\uad70\uc744 \ub04c\uc5b4\ub4e4\uc5ec \uc77c\ubcf8\uad70\uc744 \uaca9\ud1f4\ud558\uace0 \uac1c\ud654\ud30c\ub97c \uc81c\uac70\ud558\uc5ec \uc815\uad8c\uc744 \uc7a5\uc545\ud558\ub824\ub294 \uacc4\ud68d\uc740 \ub2f9\ucd08 \uc774\uc900\uc6a9, \uc774\ud0dc\uc6a9 ,", "paragraph_id": "6590317-4-1", "paragraph_question": "question: \uc774\uc900\uc6a9\uc5d0\uac8c \ud070\uc77c\uc744 \ub3c4\ubaa8\ud558\uae30 \uc704\ud574\uc11c\ub294 \ub54c\ub97c \ub193\uce58\uc9c0 \ub9d0\uc544\uc544 \ud55c\ub2e4\uace0 \ub9d0\ud55c \uc0ac\ub78c\uc740?, context: \ub300\uc6d0\uad70\ud30c\uac00 \ub18d\ubbfc\uad70\uc744 \uc0c1\uacbd\uc2dc\ud0a4\uace0, \uccad\uad6d\uad70\uc744 \ub04c\uc5b4\ub4e4\uc5ec \uc77c\ubcf8\uad70\uc744 \uaca9\ud1f4\ud558\uace0 \uac1c\ud654\ud30c\ub97c \uc81c\uac70\ud558\uc5ec \uc815\uad8c\uc744 \uc7a5\uc545\ud558\ub824\ub294 \uacc4\ud68d\uc740 \ub2f9\ucd08 \uc774\uc900\uc6a9, \uc774\ud0dc\uc6a9, \ubc15\uc900\uc591\uc758 \uc2dc\uad6d\ub300\ucc98 \ub17c\uc758 \uc18d\uc5d0\uc11c \uadf8 \uc724\uacfd\uc774 \uc9dc\uc5ec\uc84c\ub2e4. \ub2f9\uc2dc \ubc15\uc900\uc591\uc740 \uc774\uc900\uc6a9\uc5d0\uac8c \uad00\uc9c1\uc744 \uc26c\uace0 \uc678\uad6d\uc73c\ub85c \ub098\uac00 10\ub144 \ub3d9\uc548 \uacac\ubb38\uc744 \ub113\ud600 \uba85\ub9dd\uc744 \uc5bb\uc740 \uc774\ud6c4\uc5d0 \ub3cc\uc544\uc62c \uac83\uc744 \uad8c\ud558\uc600\ub2e4. \uadf8 \ub3d9\uc548 \uace0\uc885\uc740 \ub178\uc1e0\ud558\uac8c \ub418\uace0 \uc655\uc138\uc790(\uc21c\uc885)\ub3c4 \uadf8\ub2e4\uc9c0 \ud070 \ub355\uc774 \uc5c6\uc73c\ub2c8 \uadf8\ub54c\uc5d0 \uc678\uad6d \uba85\ub9dd\uacfc \uad6d\ub0b4\uc758 \uad00\uc2ec\uc740 \uc790\uc5f0\ud788 \uc774\uc900\uc6a9\uc5d0\uac8c \uc3e0\ub9b4 \uac83\uc774\uace0, \uadf8\ub7ec\uba74 \uadf8\ub2e4\uc9c0 \ub178\ub825\ud558\uc9c0 \uc54a\uace0\ub3c4 \uad8c\ub825\uc744 \uc958 \uc218 \uc788\uc744 \uac83\uc774\ub77c\uace0 \ud558\uc600\ub2e4. \uadf8\ub7ec\ub098 \uc774\ud0dc\uc6a9\uc740 \ud070\uc77c\uc744 \ub3c4\ubaa8\ud560 \uacbd\uc6b0\uc5d0\ub294 \ub54c\ub97c \ub193\uce58\uc9c0 \ub9d0\uc544\uc57c \ud55c\ub2e4\uace0 \ud558\uba74\uc11c, \uc5ec\ub7ec \uc0ac\ub78c\uc758 \uae30\ub300\uac00 \ubaa8\ub450 \ub300\uc6d0\uad70\uc744 \ud5a5\ud558\uace0 \uc788\uace0, \ub354\uc6b1\uc774 \ub3d9\ud559\uad70\uc774 \ub300\uc6d0\uad70\uc744 \ubc1b\ub4e4\uaca0\ub2e4\ub294 \uc8fc\uc7a5\uc744 \ud3b4\uba74\uc11c \ubd09\uae30\ud558\uace0 \uc788\ub294 \uc9c0\uae08, \uadf8\ub4e4\ub85c \ud558\uc5ec\uae08 \uc218\uc2ed\ub9cc \ub300\uc911\uc744 \ub3d9\uc6d0\ud558\uc5ec \uc62c\ub77c\uc624\uac8c \ud55c\ub2e4\uba74 \uc9c4\uc2e4\ub85c \uc0ac\ub78c\ub4e4\uc758 \ubb34\ub9ac\uac00 \ud558\ub298\uc744 \uc774\uae34\ub2e4\uace0 \ud558\ub4ef\uc774 \uc77c\ubcf8\uad70\ub300\uac00 \ube44\ub85d \uc6c0\uc9c1\uc778\ub2e4\uace0 \ud558\ub354\ub77c\ub3c4 \uc5b4\ucc0c\ud560 \uc218 \uc5c6\uc744 \uac83\uc774\ub77c\uace0 \ud558\uc600\ub2e4. \uc774\uc5b4 \uc774\ud0dc\uc6a9\uc740 \uc774\uc900\uc6a9\uc5d0\uac8c \ud55c\ud3b8\uc73c\ub85c \uc77c\ubcf8\uad70\uc744 \ub9cc\ub958\ud558\uc5ec \uc6c0\uc9c1\uc774\uc9c0 \ubabb\ud558\uac8c \ud558\uace0, \ub2e4\ub978 \ud55c\ud3b8\uc73c\ub85c \ubc24\uc744 \uc7ac\ucd09\ud558\uc5ec \uad70\uc911\uc744 \uc62c\ub77c\uc624\uac8c \ud55c\ub2e4\uba74 \uc190\ubc14\ub2e5\uc744 \ub4a4\uc9d1\ub4ef\uc774 \uc77c\uc774 \uc26c\uc6cc\uc9c8 \uac83\uc774\ub77c\uace0 \uc81c\uc548\ud558\uc600\ub2e4."} {"question": "2013\ub144 11\uc6d4\uc5d0 \ubc1c\ub9e4\ub41c \uc18d\ud3b8\uc740 \ubb34\uc5c7\uc778\uac00?", "paragraph": "\ubcc4\ub3c4\uc758 \uadf8\ub798\ud53d \ub178\ube14 \uc2dc\ub9ac\uc988 \ub610\ud55c \ubc1c\ub9e4\ub418\uc5c8\ub2e4. 2013\ub144 4\uc6d4 \ubc1c\ub9e4\ub41c \ub2e4\ub2c8\uc5d8 \ucf54\ub974\uc138\ud1a0(Danielle Corsetto) \uac01\ubcf8, \uc7ad \uc2a4\ud138\ub9c1(Zack Sterling) \uadf8\ub9bc\uc758 \u300a\uc5b4\ub4dc\ubca4\ucc98 \ud0c0\uc784: \ubd88\uc7a5\ub09c\u300b(Adventure Time: Playing with Fire)\ub294 \ud540\uacfc \uc81c\uc774\ud06c\uc640 \ud568\uaed8 \ubd88\uaf43 \uacf5\uc8fc\uac00 \ubc8c\uc774\ub294 \ubaa8\ud5d8\uc5d0 \ub300\ud574 \ub2e4\ub8e8\uc5c8\ub2e4. \uc18d\ud3b8\uc73c\ub85c\ub294 \ud604\uc7ac\uae4c\uc9c0 \u300a\ud53d\uc140 \uacf5\uc8fc\u300b(Pixel Princesses, 2013\ub144 11\uc6d4 \ubc1c\ub9e4), \u300a\uc2dc\uc789 \ub808\ub4dc\u300b(Seeing Red, 2014\ub144 5\uc6d4), \u300a\ube44\ud130 \uc2a4\uc704\uce20\u300b(Bitter Sweets, 2014\ub144 11\uc6d4), \u300a\uadf8\ub808\uc774\ube14\uc2a4 \uc288\uba54\uc774\ube14\uc2a4\u300b(Graybles Schmaybles, 2015\ub144 5\uc6d4), \u300a\ub9c8\uc2a4\ud06c\ub4dc \uba54\uc774\ud5f4\u300b(Masked Mayhem, 2015\ub144 12\uc6d4), \u300a\ub124 \uac1c\uc758 \uc131\u300b(The Four Castles, 2016\ub144 5\uc6d4), \u300a\ubc84\ube14\uac80 \ub300\ud1b5\ub839\u300b(President Bubblegum, 2016\ub144 9\uc6d4), \u300a\ube0c\ub808\uc778 \ub85c\ubc84\uc2a4\u300b(Brain Robbers, 2017\ub144 3\uc6d4)\uac00 \ubc1c\ub9e4\ub418\uc5c8\ub2e4.", "answer": "\ud53d\uc140 \uacf5\uc8fc", "sentence": "\uc18d\ud3b8\uc73c\ub85c\ub294 \ud604\uc7ac\uae4c\uc9c0 \u300a \ud53d\uc140 \uacf5\uc8fc \u300b(Pixel Princesses, 2013\ub144 11\uc6d4 \ubc1c\ub9e4), \u300a\uc2dc\uc789 \ub808\ub4dc\u300b(Seeing Red, 2014\ub144 5\uc6d4), \u300a\ube44\ud130 \uc2a4\uc704\uce20\u300b(Bitter Sweets, 2014\ub144 11\uc6d4), \u300a\uadf8\ub808\uc774\ube14\uc2a4 \uc288\uba54\uc774\ube14\uc2a4\u300b(Graybles Schmaybles, 2015\ub144 5\uc6d4), \u300a\ub9c8\uc2a4\ud06c\ub4dc \uba54\uc774\ud5f4\u300b(Masked Mayhem, 2015\ub144 12\uc6d4), \u300a\ub124 \uac1c\uc758 \uc131\u300b(The Four Castles, 2016\ub144 5\uc6d4), \u300a\ubc84\ube14\uac80 \ub300\ud1b5\ub839\u300b(President Bubblegum, 2016\ub144 9\uc6d4), \u300a\ube0c\ub808\uc778 \ub85c\ubc84\uc2a4\u300b(Brain Robbers, 2017\ub144 3\uc6d4)\uac00 \ubc1c\ub9e4\ub418\uc5c8\ub2e4.", "paragraph_sentence": "\ubcc4\ub3c4\uc758 \uadf8\ub798\ud53d \ub178\ube14 \uc2dc\ub9ac\uc988 \ub610\ud55c \ubc1c\ub9e4\ub418\uc5c8\ub2e4. 2013\ub144 4\uc6d4 \ubc1c\ub9e4\ub41c \ub2e4\ub2c8\uc5d8 \ucf54\ub974\uc138\ud1a0(Danielle Corsetto) \uac01\ubcf8, \uc7ad \uc2a4\ud138\ub9c1(Zack Sterling) \uadf8\ub9bc\uc758 \u300a\uc5b4\ub4dc\ubca4\ucc98 \ud0c0\uc784: \ubd88\uc7a5\ub09c\u300b(Adventure Time: Playing with Fire)\ub294 \ud540\uacfc \uc81c\uc774\ud06c\uc640 \ud568\uaed8 \ubd88\uaf43 \uacf5\uc8fc\uac00 \ubc8c\uc774\ub294 \ubaa8\ud5d8\uc5d0 \ub300\ud574 \ub2e4\ub8e8\uc5c8\ub2e4. \uc18d\ud3b8\uc73c\ub85c\ub294 \ud604\uc7ac\uae4c\uc9c0 \u300a \ud53d\uc140 \uacf5\uc8fc \u300b(Pixel Princesses, 2013\ub144 11\uc6d4 \ubc1c\ub9e4), \u300a\uc2dc\uc789 \ub808\ub4dc\u300b(Seeing Red, 2014\ub144 5\uc6d4), \u300a\ube44\ud130 \uc2a4\uc704\uce20\u300b(Bitter Sweets, 2014\ub144 11\uc6d4), \u300a\uadf8\ub808\uc774\ube14\uc2a4 \uc288\uba54\uc774\ube14\uc2a4\u300b(Graybles Schmaybles, 2015\ub144 5\uc6d4), \u300a\ub9c8\uc2a4\ud06c\ub4dc \uba54\uc774\ud5f4\u300b(Masked Mayhem, 2015\ub144 12\uc6d4), \u300a\ub124 \uac1c\uc758 \uc131\u300b(The Four Castles, 2016\ub144 5\uc6d4), \u300a\ubc84\ube14\uac80 \ub300\ud1b5\ub839\u300b(President Bubblegum, 2016\ub144 9\uc6d4), \u300a\ube0c\ub808\uc778 \ub85c\ubc84\uc2a4\u300b(Brain Robbers, 2017\ub144 3\uc6d4)\uac00 \ubc1c\ub9e4\ub418\uc5c8\ub2e4. ", "paragraph_answer": "\ubcc4\ub3c4\uc758 \uadf8\ub798\ud53d \ub178\ube14 \uc2dc\ub9ac\uc988 \ub610\ud55c \ubc1c\ub9e4\ub418\uc5c8\ub2e4. 2013\ub144 4\uc6d4 \ubc1c\ub9e4\ub41c \ub2e4\ub2c8\uc5d8 \ucf54\ub974\uc138\ud1a0(Danielle Corsetto) \uac01\ubcf8, \uc7ad \uc2a4\ud138\ub9c1(Zack Sterling) \uadf8\ub9bc\uc758 \u300a\uc5b4\ub4dc\ubca4\ucc98 \ud0c0\uc784: \ubd88\uc7a5\ub09c\u300b(Adventure Time: Playing with Fire)\ub294 \ud540\uacfc \uc81c\uc774\ud06c\uc640 \ud568\uaed8 \ubd88\uaf43 \uacf5\uc8fc\uac00 \ubc8c\uc774\ub294 \ubaa8\ud5d8\uc5d0 \ub300\ud574 \ub2e4\ub8e8\uc5c8\ub2e4. \uc18d\ud3b8\uc73c\ub85c\ub294 \ud604\uc7ac\uae4c\uc9c0 \u300a \ud53d\uc140 \uacf5\uc8fc \u300b(Pixel Princesses, 2013\ub144 11\uc6d4 \ubc1c\ub9e4), \u300a\uc2dc\uc789 \ub808\ub4dc\u300b(Seeing Red, 2014\ub144 5\uc6d4), \u300a\ube44\ud130 \uc2a4\uc704\uce20\u300b(Bitter Sweets, 2014\ub144 11\uc6d4), \u300a\uadf8\ub808\uc774\ube14\uc2a4 \uc288\uba54\uc774\ube14\uc2a4\u300b(Graybles Schmaybles, 2015\ub144 5\uc6d4), \u300a\ub9c8\uc2a4\ud06c\ub4dc \uba54\uc774\ud5f4\u300b(Masked Mayhem, 2015\ub144 12\uc6d4), \u300a\ub124 \uac1c\uc758 \uc131\u300b(The Four Castles, 2016\ub144 5\uc6d4), \u300a\ubc84\ube14\uac80 \ub300\ud1b5\ub839\u300b(President Bubblegum, 2016\ub144 9\uc6d4), \u300a\ube0c\ub808\uc778 \ub85c\ubc84\uc2a4\u300b(Brain Robbers, 2017\ub144 3\uc6d4)\uac00 \ubc1c\ub9e4\ub418\uc5c8\ub2e4.", "sentence_answer": "\uc18d\ud3b8\uc73c\ub85c\ub294 \ud604\uc7ac\uae4c\uc9c0 \u300a \ud53d\uc140 \uacf5\uc8fc \u300b(Pixel Princesses, 2013\ub144 11\uc6d4 \ubc1c\ub9e4), \u300a\uc2dc\uc789 \ub808\ub4dc\u300b(Seeing Red, 2014\ub144 5\uc6d4), \u300a\ube44\ud130 \uc2a4\uc704\uce20\u300b(Bitter Sweets, 2014\ub144 11\uc6d4), \u300a\uadf8\ub808\uc774\ube14\uc2a4 \uc288\uba54\uc774\ube14\uc2a4\u300b(Graybles Schmaybles, 2015\ub144 5\uc6d4), \u300a\ub9c8\uc2a4\ud06c\ub4dc \uba54\uc774\ud5f4\u300b(Masked Mayhem, 2015\ub144 12\uc6d4), \u300a\ub124 \uac1c\uc758 \uc131\u300b(The Four Castles, 2016\ub144 5\uc6d4), \u300a\ubc84\ube14\uac80 \ub300\ud1b5\ub839\u300b(President Bubblegum, 2016\ub144 9\uc6d4), \u300a\ube0c\ub808\uc778 \ub85c\ubc84\uc2a4\u300b(Brain Robbers, 2017\ub144 3\uc6d4)\uac00 \ubc1c\ub9e4\ub418\uc5c8\ub2e4.", "paragraph_id": "6569103-21-1", "paragraph_question": "question: 2013\ub144 11\uc6d4\uc5d0 \ubc1c\ub9e4\ub41c \uc18d\ud3b8\uc740 \ubb34\uc5c7\uc778\uac00?, context: \ubcc4\ub3c4\uc758 \uadf8\ub798\ud53d \ub178\ube14 \uc2dc\ub9ac\uc988 \ub610\ud55c \ubc1c\ub9e4\ub418\uc5c8\ub2e4. 2013\ub144 4\uc6d4 \ubc1c\ub9e4\ub41c \ub2e4\ub2c8\uc5d8 \ucf54\ub974\uc138\ud1a0(Danielle Corsetto) \uac01\ubcf8, \uc7ad \uc2a4\ud138\ub9c1(Zack Sterling) \uadf8\ub9bc\uc758 \u300a\uc5b4\ub4dc\ubca4\ucc98 \ud0c0\uc784: \ubd88\uc7a5\ub09c\u300b(Adventure Time: Playing with Fire)\ub294 \ud540\uacfc \uc81c\uc774\ud06c\uc640 \ud568\uaed8 \ubd88\uaf43 \uacf5\uc8fc\uac00 \ubc8c\uc774\ub294 \ubaa8\ud5d8\uc5d0 \ub300\ud574 \ub2e4\ub8e8\uc5c8\ub2e4. \uc18d\ud3b8\uc73c\ub85c\ub294 \ud604\uc7ac\uae4c\uc9c0 \u300a\ud53d\uc140 \uacf5\uc8fc\u300b(Pixel Princesses, 2013\ub144 11\uc6d4 \ubc1c\ub9e4), \u300a\uc2dc\uc789 \ub808\ub4dc\u300b(Seeing Red, 2014\ub144 5\uc6d4), \u300a\ube44\ud130 \uc2a4\uc704\uce20\u300b(Bitter Sweets, 2014\ub144 11\uc6d4), \u300a\uadf8\ub808\uc774\ube14\uc2a4 \uc288\uba54\uc774\ube14\uc2a4\u300b(Graybles Schmaybles, 2015\ub144 5\uc6d4), \u300a\ub9c8\uc2a4\ud06c\ub4dc \uba54\uc774\ud5f4\u300b(Masked Mayhem, 2015\ub144 12\uc6d4), \u300a\ub124 \uac1c\uc758 \uc131\u300b(The Four Castles, 2016\ub144 5\uc6d4), \u300a\ubc84\ube14\uac80 \ub300\ud1b5\ub839\u300b(President Bubblegum, 2016\ub144 9\uc6d4), \u300a\ube0c\ub808\uc778 \ub85c\ubc84\uc2a4\u300b(Brain Robbers, 2017\ub144 3\uc6d4)\uac00 \ubc1c\ub9e4\ub418\uc5c8\ub2e4."} {"question": "\ubc31\uc120\ud558 \uad50\uc218\ub97c \uc81c\uc678\ud55c \ubaa8\ub4e0 \ub2e4\ub978 \uc0ac\ub78c\ub4e4\uc740 \ubc31\ub0a8\uae30\uc758 \uc0ac\ub9dd \uc6d0\uc778\uc774 \ubb34\uc5c7\uc774\ub77c\uace0 \uc8fc\uc7a5\ud558\uc600\ub294\uac00?", "paragraph": "\ud558\uc9c0\ub9cc \ub300\ud55c\uc758\uc0ac\ud611\ud68c\uc5d0\uc11c\ub294 \uc2ec\ud3d0\uc815\uc9c0\ub294 \"\uc808\ub300\ub85c \uc0ac\ub9dd\uc6d0\uc778\uc774 \ub420 \uc218 \uc5c6\ub2e4\"\ub294 \uacf5\uc2dd\uc785\uc7a5\uc744 \ub0b4\ub193\uc558\ub2e4. \uae30\ub3d9\ud6c8 \ub300\ud55c\uc804\uacf5\uc758\ud611\uc758\ud68c\uc7a5\uc740 \"\ud2b9\uc704\uc5d0\uc11c \ubc31\uc120\ud558 \uad50\uc218\ub97c \uc81c\uc678\ud55c \ubaa8\ub450\uac00 \uc678\uc778\uc0ac\ub77c \ud588\uc74c\uc5d0\ub3c4 \uc0ac\ub9dd\uc9c4\ub2e8\uc11c\ub294 \uadf8\ub300\ub85c \uc720\uc9c0\ub410\ub2e4. \uc8fc\uce58\uc758 \ucca0\ud559\uc774\ub098 \ud2b9\uc218\uc0c1\ud669\uc5d0 \ub300\ud55c \uc9c4\uc815\uc131 \uac19\uc740 \uc18c\ub9ac\ub294 \ube44\uac81\ud55c \ubcc0\uba85\"\uc774\ub77c\uba70 \"\uc11c\uc6b8\ub300\ubcd1\uc6d0\ubfd0 \uc544\ub2c8\ub77c \uc758\ub8cc\uacc4 \uc804\uccb4\uc5d0\uc11c \ubd80\ub044\ub7ec\uc6b4 \uc5ed\uc0ac\ub85c \uae30\ub85d\ub420 \uac83\"\uc774\ub77c\uace0 \uc11c\uc6b8\ub300\ubcd1\uc6d0 \ud2b9\uc704\uc758 \ubc1c\ud45c\ub97c \ube44\ud310\ud588\ub2e4. \ub610\ud55c \uc11c\uc6b8\ub300\ud559\uad50 \ud2b9\ubcc4\uc870\uc0ac\uc704\uc6d0\ud68c \uc704\uc6d0\uc7a5\uc778 \uc774\uc724\uc131 \uad50\uc218\ub294 '\ub098\ub77c\uba74 \uc678\uc778\uc0ac\ub77c \uc37c\uc744 \uac83'\uc774\ub77c\uace0 \ub9d0\ud574 \ub17c\ub780\uc744 \ud0a4\uc6e0\uc73c\uba70 \uc774\ud6c4 JTBC\ub274\uc2a4\ub8f8 \uc778\ud130\ubdf0\uc5d0\uc11c \"\uc11c\uc6b8\ub300\ubcd1\uc6d0 \uc785\uc7a5\uc740 \uc678\uc778\uc0ac\uc778\uac00?\"\ub77c\ub294 \uc190\uc11d\ud76c \uc575\ucee4\uc758 \uc9c8\ubb38\uc5d0 \ub9de\ub2e4\uace0 \ub2f5\ud558\uc600\ub2e4. \uadf8\ub9ac\uace0 \ub2e4\uc74c\ub0a0 CBS \ub77c\ub514\uc624 \uc778\ud130\ubdf0\uc5d0\uc11c \"\ub098\ub294 \ub1cc\uc218\uc220\uc740 \ubc31\uc120\ud558 \uad50\uc218\ud55c\ud14c \ubc1b\uaca0\uc5b4\uc694. \ud558\uc9c0\ub9cc \ub0b4 \uc0ac\ub9dd\uc9c4\ub2e8\uc11c\ub97c \ubc31\uc120\ud558 \uad50\uc218\ud55c\ud14c \ub9e1\uae30\uc9c0\ub294 \uc54a\uaca0\uc2b5\ub2c8\ub2e4.\"\ub77c\uace0 \ub9d0\ud558\uae30\ub3c4 \ud588\ub2e4.", "answer": "\uc678\uc778\uc0ac", "sentence": "\uae30\ub3d9\ud6c8 \ub300\ud55c\uc804\uacf5\uc758\ud611\uc758\ud68c\uc7a5\uc740 \"\ud2b9\uc704\uc5d0\uc11c \ubc31\uc120\ud558 \uad50\uc218\ub97c \uc81c\uc678\ud55c \ubaa8\ub450\uac00 \uc678\uc778\uc0ac \ub77c \ud588\uc74c\uc5d0\ub3c4 \uc0ac\ub9dd\uc9c4\ub2e8\uc11c\ub294 \uadf8\ub300\ub85c \uc720\uc9c0\ub410\ub2e4.", "paragraph_sentence": "\ud558\uc9c0\ub9cc \ub300\ud55c\uc758\uc0ac\ud611\ud68c\uc5d0\uc11c\ub294 \uc2ec\ud3d0\uc815\uc9c0\ub294 \"\uc808\ub300\ub85c \uc0ac\ub9dd\uc6d0\uc778\uc774 \ub420 \uc218 \uc5c6\ub2e4\"\ub294 \uacf5\uc2dd\uc785\uc7a5\uc744 \ub0b4\ub193\uc558\ub2e4. \uae30\ub3d9\ud6c8 \ub300\ud55c\uc804\uacf5\uc758\ud611\uc758\ud68c\uc7a5\uc740 \"\ud2b9\uc704\uc5d0\uc11c \ubc31\uc120\ud558 \uad50\uc218\ub97c \uc81c\uc678\ud55c \ubaa8\ub450\uac00 \uc678\uc778\uc0ac \ub77c \ud588\uc74c\uc5d0\ub3c4 \uc0ac\ub9dd\uc9c4\ub2e8\uc11c\ub294 \uadf8\ub300\ub85c \uc720\uc9c0\ub410\ub2e4. \uc8fc\uce58\uc758 \ucca0\ud559\uc774\ub098 \ud2b9\uc218\uc0c1\ud669\uc5d0 \ub300\ud55c \uc9c4\uc815\uc131 \uac19\uc740 \uc18c\ub9ac\ub294 \ube44\uac81\ud55c \ubcc0\uba85\"\uc774\ub77c\uba70 \"\uc11c\uc6b8\ub300\ubcd1\uc6d0\ubfd0 \uc544\ub2c8\ub77c \uc758\ub8cc\uacc4 \uc804\uccb4\uc5d0\uc11c \ubd80\ub044\ub7ec\uc6b4 \uc5ed\uc0ac\ub85c \uae30\ub85d\ub420 \uac83\"\uc774\ub77c\uace0 \uc11c\uc6b8\ub300\ubcd1\uc6d0 \ud2b9\uc704\uc758 \ubc1c\ud45c\ub97c \ube44\ud310\ud588\ub2e4. \ub610\ud55c \uc11c\uc6b8\ub300\ud559\uad50 \ud2b9\ubcc4\uc870\uc0ac\uc704\uc6d0\ud68c \uc704\uc6d0\uc7a5\uc778 \uc774\uc724\uc131 \uad50\uc218\ub294 '\ub098\ub77c\uba74 \uc678\uc778\uc0ac\ub77c \uc37c\uc744 \uac83'\uc774\ub77c\uace0 \ub9d0\ud574 \ub17c\ub780\uc744 \ud0a4\uc6e0\uc73c\uba70 \uc774\ud6c4 JTBC\ub274\uc2a4\ub8f8 \uc778\ud130\ubdf0\uc5d0\uc11c \"\uc11c\uc6b8\ub300\ubcd1\uc6d0 \uc785\uc7a5\uc740 \uc678\uc778\uc0ac\uc778\uac00?\"\ub77c\ub294 \uc190\uc11d\ud76c \uc575\ucee4\uc758 \uc9c8\ubb38\uc5d0 \ub9de\ub2e4\uace0 \ub2f5\ud558\uc600\ub2e4. \uadf8\ub9ac\uace0 \ub2e4\uc74c\ub0a0 CBS \ub77c\ub514\uc624 \uc778\ud130\ubdf0\uc5d0\uc11c \"\ub098\ub294 \ub1cc\uc218\uc220\uc740 \ubc31\uc120\ud558 \uad50\uc218\ud55c\ud14c \ubc1b\uaca0\uc5b4\uc694. \ud558\uc9c0\ub9cc \ub0b4 \uc0ac\ub9dd\uc9c4\ub2e8\uc11c\ub97c \ubc31\uc120\ud558 \uad50\uc218\ud55c\ud14c \ub9e1\uae30\uc9c0\ub294 \uc54a\uaca0\uc2b5\ub2c8\ub2e4. \"\ub77c\uace0 \ub9d0\ud558\uae30\ub3c4 \ud588\ub2e4.", "paragraph_answer": "\ud558\uc9c0\ub9cc \ub300\ud55c\uc758\uc0ac\ud611\ud68c\uc5d0\uc11c\ub294 \uc2ec\ud3d0\uc815\uc9c0\ub294 \"\uc808\ub300\ub85c \uc0ac\ub9dd\uc6d0\uc778\uc774 \ub420 \uc218 \uc5c6\ub2e4\"\ub294 \uacf5\uc2dd\uc785\uc7a5\uc744 \ub0b4\ub193\uc558\ub2e4. \uae30\ub3d9\ud6c8 \ub300\ud55c\uc804\uacf5\uc758\ud611\uc758\ud68c\uc7a5\uc740 \"\ud2b9\uc704\uc5d0\uc11c \ubc31\uc120\ud558 \uad50\uc218\ub97c \uc81c\uc678\ud55c \ubaa8\ub450\uac00 \uc678\uc778\uc0ac \ub77c \ud588\uc74c\uc5d0\ub3c4 \uc0ac\ub9dd\uc9c4\ub2e8\uc11c\ub294 \uadf8\ub300\ub85c \uc720\uc9c0\ub410\ub2e4. \uc8fc\uce58\uc758 \ucca0\ud559\uc774\ub098 \ud2b9\uc218\uc0c1\ud669\uc5d0 \ub300\ud55c \uc9c4\uc815\uc131 \uac19\uc740 \uc18c\ub9ac\ub294 \ube44\uac81\ud55c \ubcc0\uba85\"\uc774\ub77c\uba70 \"\uc11c\uc6b8\ub300\ubcd1\uc6d0\ubfd0 \uc544\ub2c8\ub77c \uc758\ub8cc\uacc4 \uc804\uccb4\uc5d0\uc11c \ubd80\ub044\ub7ec\uc6b4 \uc5ed\uc0ac\ub85c \uae30\ub85d\ub420 \uac83\"\uc774\ub77c\uace0 \uc11c\uc6b8\ub300\ubcd1\uc6d0 \ud2b9\uc704\uc758 \ubc1c\ud45c\ub97c \ube44\ud310\ud588\ub2e4. \ub610\ud55c \uc11c\uc6b8\ub300\ud559\uad50 \ud2b9\ubcc4\uc870\uc0ac\uc704\uc6d0\ud68c \uc704\uc6d0\uc7a5\uc778 \uc774\uc724\uc131 \uad50\uc218\ub294 '\ub098\ub77c\uba74 \uc678\uc778\uc0ac\ub77c \uc37c\uc744 \uac83'\uc774\ub77c\uace0 \ub9d0\ud574 \ub17c\ub780\uc744 \ud0a4\uc6e0\uc73c\uba70 \uc774\ud6c4 JTBC\ub274\uc2a4\ub8f8 \uc778\ud130\ubdf0\uc5d0\uc11c \"\uc11c\uc6b8\ub300\ubcd1\uc6d0 \uc785\uc7a5\uc740 \uc678\uc778\uc0ac\uc778\uac00?\"\ub77c\ub294 \uc190\uc11d\ud76c \uc575\ucee4\uc758 \uc9c8\ubb38\uc5d0 \ub9de\ub2e4\uace0 \ub2f5\ud558\uc600\ub2e4. \uadf8\ub9ac\uace0 \ub2e4\uc74c\ub0a0 CBS \ub77c\ub514\uc624 \uc778\ud130\ubdf0\uc5d0\uc11c \"\ub098\ub294 \ub1cc\uc218\uc220\uc740 \ubc31\uc120\ud558 \uad50\uc218\ud55c\ud14c \ubc1b\uaca0\uc5b4\uc694. \ud558\uc9c0\ub9cc \ub0b4 \uc0ac\ub9dd\uc9c4\ub2e8\uc11c\ub97c \ubc31\uc120\ud558 \uad50\uc218\ud55c\ud14c \ub9e1\uae30\uc9c0\ub294 \uc54a\uaca0\uc2b5\ub2c8\ub2e4.\"\ub77c\uace0 \ub9d0\ud558\uae30\ub3c4 \ud588\ub2e4.", "sentence_answer": "\uae30\ub3d9\ud6c8 \ub300\ud55c\uc804\uacf5\uc758\ud611\uc758\ud68c\uc7a5\uc740 \"\ud2b9\uc704\uc5d0\uc11c \ubc31\uc120\ud558 \uad50\uc218\ub97c \uc81c\uc678\ud55c \ubaa8\ub450\uac00 \uc678\uc778\uc0ac \ub77c \ud588\uc74c\uc5d0\ub3c4 \uc0ac\ub9dd\uc9c4\ub2e8\uc11c\ub294 \uadf8\ub300\ub85c \uc720\uc9c0\ub410\ub2e4.", "paragraph_id": "6508024-6-1", "paragraph_question": "question: \ubc31\uc120\ud558 \uad50\uc218\ub97c \uc81c\uc678\ud55c \ubaa8\ub4e0 \ub2e4\ub978 \uc0ac\ub78c\ub4e4\uc740 \ubc31\ub0a8\uae30\uc758 \uc0ac\ub9dd \uc6d0\uc778\uc774 \ubb34\uc5c7\uc774\ub77c\uace0 \uc8fc\uc7a5\ud558\uc600\ub294\uac00?, context: \ud558\uc9c0\ub9cc \ub300\ud55c\uc758\uc0ac\ud611\ud68c\uc5d0\uc11c\ub294 \uc2ec\ud3d0\uc815\uc9c0\ub294 \"\uc808\ub300\ub85c \uc0ac\ub9dd\uc6d0\uc778\uc774 \ub420 \uc218 \uc5c6\ub2e4\"\ub294 \uacf5\uc2dd\uc785\uc7a5\uc744 \ub0b4\ub193\uc558\ub2e4. \uae30\ub3d9\ud6c8 \ub300\ud55c\uc804\uacf5\uc758\ud611\uc758\ud68c\uc7a5\uc740 \"\ud2b9\uc704\uc5d0\uc11c \ubc31\uc120\ud558 \uad50\uc218\ub97c \uc81c\uc678\ud55c \ubaa8\ub450\uac00 \uc678\uc778\uc0ac\ub77c \ud588\uc74c\uc5d0\ub3c4 \uc0ac\ub9dd\uc9c4\ub2e8\uc11c\ub294 \uadf8\ub300\ub85c \uc720\uc9c0\ub410\ub2e4. \uc8fc\uce58\uc758 \ucca0\ud559\uc774\ub098 \ud2b9\uc218\uc0c1\ud669\uc5d0 \ub300\ud55c \uc9c4\uc815\uc131 \uac19\uc740 \uc18c\ub9ac\ub294 \ube44\uac81\ud55c \ubcc0\uba85\"\uc774\ub77c\uba70 \"\uc11c\uc6b8\ub300\ubcd1\uc6d0\ubfd0 \uc544\ub2c8\ub77c \uc758\ub8cc\uacc4 \uc804\uccb4\uc5d0\uc11c \ubd80\ub044\ub7ec\uc6b4 \uc5ed\uc0ac\ub85c \uae30\ub85d\ub420 \uac83\"\uc774\ub77c\uace0 \uc11c\uc6b8\ub300\ubcd1\uc6d0 \ud2b9\uc704\uc758 \ubc1c\ud45c\ub97c \ube44\ud310\ud588\ub2e4. \ub610\ud55c \uc11c\uc6b8\ub300\ud559\uad50 \ud2b9\ubcc4\uc870\uc0ac\uc704\uc6d0\ud68c \uc704\uc6d0\uc7a5\uc778 \uc774\uc724\uc131 \uad50\uc218\ub294 '\ub098\ub77c\uba74 \uc678\uc778\uc0ac\ub77c \uc37c\uc744 \uac83'\uc774\ub77c\uace0 \ub9d0\ud574 \ub17c\ub780\uc744 \ud0a4\uc6e0\uc73c\uba70 \uc774\ud6c4 JTBC\ub274\uc2a4\ub8f8 \uc778\ud130\ubdf0\uc5d0\uc11c \"\uc11c\uc6b8\ub300\ubcd1\uc6d0 \uc785\uc7a5\uc740 \uc678\uc778\uc0ac\uc778\uac00?\"\ub77c\ub294 \uc190\uc11d\ud76c \uc575\ucee4\uc758 \uc9c8\ubb38\uc5d0 \ub9de\ub2e4\uace0 \ub2f5\ud558\uc600\ub2e4. \uadf8\ub9ac\uace0 \ub2e4\uc74c\ub0a0 CBS \ub77c\ub514\uc624 \uc778\ud130\ubdf0\uc5d0\uc11c \"\ub098\ub294 \ub1cc\uc218\uc220\uc740 \ubc31\uc120\ud558 \uad50\uc218\ud55c\ud14c \ubc1b\uaca0\uc5b4\uc694. \ud558\uc9c0\ub9cc \ub0b4 \uc0ac\ub9dd\uc9c4\ub2e8\uc11c\ub97c \ubc31\uc120\ud558 \uad50\uc218\ud55c\ud14c \ub9e1\uae30\uc9c0\ub294 \uc54a\uaca0\uc2b5\ub2c8\ub2e4.\"\ub77c\uace0 \ub9d0\ud558\uae30\ub3c4 \ud588\ub2e4."} {"question": "\ub3c5\uc77c \ud63c\uc695 \uc0ac\uc6b0\ub098\uc758 \uae30\uc6d0\uc740 \uc5b4\ub290 \uc2dc\ub300\uc778\uac00?", "paragraph": "\ub3c5\uc77c\uc758 \ud63c\uc695 \uc0ac\uc6b0\ub098\uc758 \uae30\uc6d0\uc740 \uace0\ub300 \ub85c\ub9c8 \uc2dc\ub300\ub85c \uae30\uc6d0\uc744 \ucc3e\uc744 \uc218 \uc788\ub2e4. \uace0\ub300 \ub85c\ub9c8 \uc2dc\ub300\uc5d0 \uc774\ubbf8 \uacf5\uc911\ubaa9\uc695\ud0d5\uc774 \ub4f1\uc7a5\ud558\uae30 \uc2dc\uc791\ud588\ub2e4. \ubcf8\ub798 \ud63c\uc695\uc774 \uae08\uc9c0\ub418\uc5b4 \uc788\uc5c8\uc73c\ub098 \uc810\ucc28 \ub0a8\ud0d5\uacfc \uc5ec\ud0d5\uc758 \uae30\uc900\uc774 \uc5c6\uc5b4\uc84c\uc73c\uba70 \ub2e8\uc21c\ud788 \ubaa9\uc695\uc758 \ubaa9\uc801\uc774 \uc544\ub2cc \ub2f4\ud654\uc2e4, \uc2a4\ud3ec\uce20\uad00\ub78c\uc2e4\uacfc \uac19\uc740 \ub808\uc800 \ud65c\ub3d9\uc744 \uc704\ud55c \uacf5\uac04\uc73c\ub85c \ubcc0\ubaa8\ud558\uc600\ub2e4. \ud2b9\ud788 \ub85c\ub9c8 \uc81c\uc815\uc2dc\ub300\uc5d0 \ud669\uc81c\uc758 \uc778\uae30 \uc720\uc9c0\ub97c \uc704\ud574 \ub2e4\uc218\uc758 \ub300\ud615 \uacf5\uc911\ubaa9\uc695\ud0d5\uc774 \uac74\uc124\ub418\uc5c8\uc73c\uba70 \uac00\ub09c\ud55c \ub85c\ub9c8 \uc2dc\ubbfc\ub3c4 \uacf5\uc911\ubaa9\uc695\ud0d5 \ucd9c\uc785\uc774 \ud5c8\uac00\ub418\uc5c8\ub2e4. \uc774\ub7ec\ud55c \ubaa9\uc695\ud0d5\uc740 \uc2e0\ubd84\uc758 \ucc28\ubcc4 \uc5c6\uc774 \uc624\uc804\uc5d0\ub294 \uc5ec\uc131\ub4e4\uc774, \uc624\ud6c4\uc5d0\ub294 \ub0a8\uc131\ub4e4\uc774 \uc0ac\uc6a9\ud560 \uc218 \uc788\ub3c4\ub85d \uac1c\ubc29\ud558\uc600\uace0 \ucd08\uae30\uc5d0\ub294 \uc131\uc801 \uad6c\ubcc4\uc774 \ub9e4\uc6b0 \uc5c4\uaca9\ud588\ub2e4. \uadf8\ub7ec\ub098 \ud63c\uc695\uc774 \ucc98\uc74c \ud5c8\uc6a9\ub418\uace0 \ub098\uc11c \ubd80\ub3c4\ub355\ud55c \uc0ac\uac74\uc774\ub098 \uad00\uacc4\uac00 \uc99d\uac00\ud558\uc9c0 \uc54a\uc558\ub2e4. \uc695\ud0d5\uc5d0\uc11c\uc758 \uc54c\ubab8\uc740 \uc790\uc2e0\uc774 \uc6d0\ud558\ub358 \ub300\ub85c \uc131\uc758 \ucf8c\ub77d\uc744 \ud0d0\ud558\uace0 \uc788\ub358 \ub300\ubd80\ubd84\uc758 \ub85c\ub9c8 \ub0a8\ub140\ub4e4\uc5d0\uac8c\ub294 \ud2b9\ubcc4\ud55c \uc790\uadf9\uc774 \ub418\uc9c0 \uc54a\uc558\ub358 \uac83\uc774\ub2e4. \uc774\ub7ec\ud55c \ub85c\ub9c8 \uc2dc\ub300\uc758 \ud63c\ud0d5 \ubb38\ud654\uac00 \ub3c5\uc77c \ud63c\uc695 \uc0ac\uc6b0\ub098 \ubb38\ud654\ub85c \uc794\uc7ac\uac00 \ub0a8\uc558\ub2e4\uace0 \ubcfc \uc218 \uc788\ub2e4.", "answer": "\uace0\ub300 \ub85c\ub9c8 \uc2dc\ub300", "sentence": "\ub3c5\uc77c\uc758 \ud63c\uc695 \uc0ac\uc6b0\ub098\uc758 \uae30\uc6d0\uc740 \uace0\ub300 \ub85c\ub9c8 \uc2dc\ub300 \ub85c \uae30\uc6d0\uc744 \ucc3e\uc744 \uc218 \uc788\ub2e4.", "paragraph_sentence": " \ub3c5\uc77c\uc758 \ud63c\uc695 \uc0ac\uc6b0\ub098\uc758 \uae30\uc6d0\uc740 \uace0\ub300 \ub85c\ub9c8 \uc2dc\ub300 \ub85c \uae30\uc6d0\uc744 \ucc3e\uc744 \uc218 \uc788\ub2e4. \uace0\ub300 \ub85c\ub9c8 \uc2dc\ub300\uc5d0 \uc774\ubbf8 \uacf5\uc911\ubaa9\uc695\ud0d5\uc774 \ub4f1\uc7a5\ud558\uae30 \uc2dc\uc791\ud588\ub2e4. \ubcf8\ub798 \ud63c\uc695\uc774 \uae08\uc9c0\ub418\uc5b4 \uc788\uc5c8\uc73c\ub098 \uc810\ucc28 \ub0a8\ud0d5\uacfc \uc5ec\ud0d5\uc758 \uae30\uc900\uc774 \uc5c6\uc5b4\uc84c\uc73c\uba70 \ub2e8\uc21c\ud788 \ubaa9\uc695\uc758 \ubaa9\uc801\uc774 \uc544\ub2cc \ub2f4\ud654\uc2e4, \uc2a4\ud3ec\uce20\uad00\ub78c\uc2e4\uacfc \uac19\uc740 \ub808\uc800 \ud65c\ub3d9\uc744 \uc704\ud55c \uacf5\uac04\uc73c\ub85c \ubcc0\ubaa8\ud558\uc600\ub2e4. \ud2b9\ud788 \ub85c\ub9c8 \uc81c\uc815\uc2dc\ub300\uc5d0 \ud669\uc81c\uc758 \uc778\uae30 \uc720\uc9c0\ub97c \uc704\ud574 \ub2e4\uc218\uc758 \ub300\ud615 \uacf5\uc911\ubaa9\uc695\ud0d5\uc774 \uac74\uc124\ub418\uc5c8\uc73c\uba70 \uac00\ub09c\ud55c \ub85c\ub9c8 \uc2dc\ubbfc\ub3c4 \uacf5\uc911\ubaa9\uc695\ud0d5 \ucd9c\uc785\uc774 \ud5c8\uac00\ub418\uc5c8\ub2e4. \uc774\ub7ec\ud55c \ubaa9\uc695\ud0d5\uc740 \uc2e0\ubd84\uc758 \ucc28\ubcc4 \uc5c6\uc774 \uc624\uc804\uc5d0\ub294 \uc5ec\uc131\ub4e4\uc774, \uc624\ud6c4\uc5d0\ub294 \ub0a8\uc131\ub4e4\uc774 \uc0ac\uc6a9\ud560 \uc218 \uc788\ub3c4\ub85d \uac1c\ubc29\ud558\uc600\uace0 \ucd08\uae30\uc5d0\ub294 \uc131\uc801 \uad6c\ubcc4\uc774 \ub9e4\uc6b0 \uc5c4\uaca9\ud588\ub2e4. \uadf8\ub7ec\ub098 \ud63c\uc695\uc774 \ucc98\uc74c \ud5c8\uc6a9\ub418\uace0 \ub098\uc11c \ubd80\ub3c4\ub355\ud55c \uc0ac\uac74\uc774\ub098 \uad00\uacc4\uac00 \uc99d\uac00\ud558\uc9c0 \uc54a\uc558\ub2e4. \uc695\ud0d5\uc5d0\uc11c\uc758 \uc54c\ubab8\uc740 \uc790\uc2e0\uc774 \uc6d0\ud558\ub358 \ub300\ub85c \uc131\uc758 \ucf8c\ub77d\uc744 \ud0d0\ud558\uace0 \uc788\ub358 \ub300\ubd80\ubd84\uc758 \ub85c\ub9c8 \ub0a8\ub140\ub4e4\uc5d0\uac8c\ub294 \ud2b9\ubcc4\ud55c \uc790\uadf9\uc774 \ub418\uc9c0 \uc54a\uc558\ub358 \uac83\uc774\ub2e4. \uc774\ub7ec\ud55c \ub85c\ub9c8 \uc2dc\ub300\uc758 \ud63c\ud0d5 \ubb38\ud654\uac00 \ub3c5\uc77c \ud63c\uc695 \uc0ac\uc6b0\ub098 \ubb38\ud654\ub85c \uc794\uc7ac\uac00 \ub0a8\uc558\ub2e4\uace0 \ubcfc \uc218 \uc788\ub2e4.", "paragraph_answer": "\ub3c5\uc77c\uc758 \ud63c\uc695 \uc0ac\uc6b0\ub098\uc758 \uae30\uc6d0\uc740 \uace0\ub300 \ub85c\ub9c8 \uc2dc\ub300 \ub85c \uae30\uc6d0\uc744 \ucc3e\uc744 \uc218 \uc788\ub2e4. \uace0\ub300 \ub85c\ub9c8 \uc2dc\ub300\uc5d0 \uc774\ubbf8 \uacf5\uc911\ubaa9\uc695\ud0d5\uc774 \ub4f1\uc7a5\ud558\uae30 \uc2dc\uc791\ud588\ub2e4. \ubcf8\ub798 \ud63c\uc695\uc774 \uae08\uc9c0\ub418\uc5b4 \uc788\uc5c8\uc73c\ub098 \uc810\ucc28 \ub0a8\ud0d5\uacfc \uc5ec\ud0d5\uc758 \uae30\uc900\uc774 \uc5c6\uc5b4\uc84c\uc73c\uba70 \ub2e8\uc21c\ud788 \ubaa9\uc695\uc758 \ubaa9\uc801\uc774 \uc544\ub2cc \ub2f4\ud654\uc2e4, \uc2a4\ud3ec\uce20\uad00\ub78c\uc2e4\uacfc \uac19\uc740 \ub808\uc800 \ud65c\ub3d9\uc744 \uc704\ud55c \uacf5\uac04\uc73c\ub85c \ubcc0\ubaa8\ud558\uc600\ub2e4. \ud2b9\ud788 \ub85c\ub9c8 \uc81c\uc815\uc2dc\ub300\uc5d0 \ud669\uc81c\uc758 \uc778\uae30 \uc720\uc9c0\ub97c \uc704\ud574 \ub2e4\uc218\uc758 \ub300\ud615 \uacf5\uc911\ubaa9\uc695\ud0d5\uc774 \uac74\uc124\ub418\uc5c8\uc73c\uba70 \uac00\ub09c\ud55c \ub85c\ub9c8 \uc2dc\ubbfc\ub3c4 \uacf5\uc911\ubaa9\uc695\ud0d5 \ucd9c\uc785\uc774 \ud5c8\uac00\ub418\uc5c8\ub2e4. \uc774\ub7ec\ud55c \ubaa9\uc695\ud0d5\uc740 \uc2e0\ubd84\uc758 \ucc28\ubcc4 \uc5c6\uc774 \uc624\uc804\uc5d0\ub294 \uc5ec\uc131\ub4e4\uc774, \uc624\ud6c4\uc5d0\ub294 \ub0a8\uc131\ub4e4\uc774 \uc0ac\uc6a9\ud560 \uc218 \uc788\ub3c4\ub85d \uac1c\ubc29\ud558\uc600\uace0 \ucd08\uae30\uc5d0\ub294 \uc131\uc801 \uad6c\ubcc4\uc774 \ub9e4\uc6b0 \uc5c4\uaca9\ud588\ub2e4. \uadf8\ub7ec\ub098 \ud63c\uc695\uc774 \ucc98\uc74c \ud5c8\uc6a9\ub418\uace0 \ub098\uc11c \ubd80\ub3c4\ub355\ud55c \uc0ac\uac74\uc774\ub098 \uad00\uacc4\uac00 \uc99d\uac00\ud558\uc9c0 \uc54a\uc558\ub2e4. \uc695\ud0d5\uc5d0\uc11c\uc758 \uc54c\ubab8\uc740 \uc790\uc2e0\uc774 \uc6d0\ud558\ub358 \ub300\ub85c \uc131\uc758 \ucf8c\ub77d\uc744 \ud0d0\ud558\uace0 \uc788\ub358 \ub300\ubd80\ubd84\uc758 \ub85c\ub9c8 \ub0a8\ub140\ub4e4\uc5d0\uac8c\ub294 \ud2b9\ubcc4\ud55c \uc790\uadf9\uc774 \ub418\uc9c0 \uc54a\uc558\ub358 \uac83\uc774\ub2e4. \uc774\ub7ec\ud55c \ub85c\ub9c8 \uc2dc\ub300\uc758 \ud63c\ud0d5 \ubb38\ud654\uac00 \ub3c5\uc77c \ud63c\uc695 \uc0ac\uc6b0\ub098 \ubb38\ud654\ub85c \uc794\uc7ac\uac00 \ub0a8\uc558\ub2e4\uace0 \ubcfc \uc218 \uc788\ub2e4.", "sentence_answer": "\ub3c5\uc77c\uc758 \ud63c\uc695 \uc0ac\uc6b0\ub098\uc758 \uae30\uc6d0\uc740 \uace0\ub300 \ub85c\ub9c8 \uc2dc\ub300 \ub85c \uae30\uc6d0\uc744 \ucc3e\uc744 \uc218 \uc788\ub2e4.", "paragraph_id": "6578532-0-0", "paragraph_question": "question: \ub3c5\uc77c \ud63c\uc695 \uc0ac\uc6b0\ub098\uc758 \uae30\uc6d0\uc740 \uc5b4\ub290 \uc2dc\ub300\uc778\uac00?, context: \ub3c5\uc77c\uc758 \ud63c\uc695 \uc0ac\uc6b0\ub098\uc758 \uae30\uc6d0\uc740 \uace0\ub300 \ub85c\ub9c8 \uc2dc\ub300\ub85c \uae30\uc6d0\uc744 \ucc3e\uc744 \uc218 \uc788\ub2e4. \uace0\ub300 \ub85c\ub9c8 \uc2dc\ub300\uc5d0 \uc774\ubbf8 \uacf5\uc911\ubaa9\uc695\ud0d5\uc774 \ub4f1\uc7a5\ud558\uae30 \uc2dc\uc791\ud588\ub2e4. \ubcf8\ub798 \ud63c\uc695\uc774 \uae08\uc9c0\ub418\uc5b4 \uc788\uc5c8\uc73c\ub098 \uc810\ucc28 \ub0a8\ud0d5\uacfc \uc5ec\ud0d5\uc758 \uae30\uc900\uc774 \uc5c6\uc5b4\uc84c\uc73c\uba70 \ub2e8\uc21c\ud788 \ubaa9\uc695\uc758 \ubaa9\uc801\uc774 \uc544\ub2cc \ub2f4\ud654\uc2e4, \uc2a4\ud3ec\uce20\uad00\ub78c\uc2e4\uacfc \uac19\uc740 \ub808\uc800 \ud65c\ub3d9\uc744 \uc704\ud55c \uacf5\uac04\uc73c\ub85c \ubcc0\ubaa8\ud558\uc600\ub2e4. \ud2b9\ud788 \ub85c\ub9c8 \uc81c\uc815\uc2dc\ub300\uc5d0 \ud669\uc81c\uc758 \uc778\uae30 \uc720\uc9c0\ub97c \uc704\ud574 \ub2e4\uc218\uc758 \ub300\ud615 \uacf5\uc911\ubaa9\uc695\ud0d5\uc774 \uac74\uc124\ub418\uc5c8\uc73c\uba70 \uac00\ub09c\ud55c \ub85c\ub9c8 \uc2dc\ubbfc\ub3c4 \uacf5\uc911\ubaa9\uc695\ud0d5 \ucd9c\uc785\uc774 \ud5c8\uac00\ub418\uc5c8\ub2e4. \uc774\ub7ec\ud55c \ubaa9\uc695\ud0d5\uc740 \uc2e0\ubd84\uc758 \ucc28\ubcc4 \uc5c6\uc774 \uc624\uc804\uc5d0\ub294 \uc5ec\uc131\ub4e4\uc774, \uc624\ud6c4\uc5d0\ub294 \ub0a8\uc131\ub4e4\uc774 \uc0ac\uc6a9\ud560 \uc218 \uc788\ub3c4\ub85d \uac1c\ubc29\ud558\uc600\uace0 \ucd08\uae30\uc5d0\ub294 \uc131\uc801 \uad6c\ubcc4\uc774 \ub9e4\uc6b0 \uc5c4\uaca9\ud588\ub2e4. \uadf8\ub7ec\ub098 \ud63c\uc695\uc774 \ucc98\uc74c \ud5c8\uc6a9\ub418\uace0 \ub098\uc11c \ubd80\ub3c4\ub355\ud55c \uc0ac\uac74\uc774\ub098 \uad00\uacc4\uac00 \uc99d\uac00\ud558\uc9c0 \uc54a\uc558\ub2e4. \uc695\ud0d5\uc5d0\uc11c\uc758 \uc54c\ubab8\uc740 \uc790\uc2e0\uc774 \uc6d0\ud558\ub358 \ub300\ub85c \uc131\uc758 \ucf8c\ub77d\uc744 \ud0d0\ud558\uace0 \uc788\ub358 \ub300\ubd80\ubd84\uc758 \ub85c\ub9c8 \ub0a8\ub140\ub4e4\uc5d0\uac8c\ub294 \ud2b9\ubcc4\ud55c \uc790\uadf9\uc774 \ub418\uc9c0 \uc54a\uc558\ub358 \uac83\uc774\ub2e4. \uc774\ub7ec\ud55c \ub85c\ub9c8 \uc2dc\ub300\uc758 \ud63c\ud0d5 \ubb38\ud654\uac00 \ub3c5\uc77c \ud63c\uc695 \uc0ac\uc6b0\ub098 \ubb38\ud654\ub85c \uc794\uc7ac\uac00 \ub0a8\uc558\ub2e4\uace0 \ubcfc \uc218 \uc788\ub2e4."} {"question": "\ubca0\ud2b8\ub0a8\uc5d0\uc11c \uac00\uc7a5 \uba3c\uc800 \ubb18\ud638\ub97c \uc0ac\uc6a9\ud55c \uc655\uc870\ub294?", "paragraph": "\ubca0\ud2b8\ub0a8\ub3c4 \uc678\uc655\ub0b4\uc81c\ub97c \uc9c0\ud5a5\ud558\uc5ec \uc548\uc73c\ub85c\ub294 \ud669\uc81c\ub97c \uc790\uce6d\ud558\uc600\uace0, \uc7ac\uc704\ud55c \uad70\uc8fc\ub4e4\uc5d0\uac8c \ubb18\ud638\ub97c \ubd99\uc600\ub2e4. \ubca0\ud2b8\ub0a8\uc5d0\uc11c \uac00\uc7a5 \uba3c\uc800 \ubb18\ud638\ub97c \uc4f0\uae30 \uc2dc\uc791\ud55c \uc655\uc870\ub294 \uc804 \ub808 \uc655\uc870\uc600\uc73c\ub098, \uc2dc\ud638\ub97c \uc62c\ub9ac\uc9c0 \uc54a\uace0 \ubb18\uc81c \ub610\ud55c \uc815\ub9bd\ub418\uc9c0 \uc54a\uc544, \uc81c\ub300\ub85c \uc808\ucc28\ub97c \ubc1f\uc544 \uc62c\ub824\uc9c4 \ubb18\ud638\uac00 \uc544\ub2c8\uc5c8\ub2e4. \uc774\ud6c4 \ubcf8\uaca9\uc801\uc73c\ub85c \ubb18\ud638 \uc81c\ub3c4\uac00 \uc815\ucc29\ub41c \uc655\uc870\ub294 12\uc138\uae30 \uc911\uc5fd\uc5d0 \uc138\uc6cc\uc9c4 \ub9ac \uc655\uc870\ub85c, \uc911\uad6d\uc758 \ub0a8\uc1a1\uc5d0 \uc758\ud574 \uc774\uc804\uc758 \uad50\uc9c0\uad70\uc655(\u4ea4\u8dbe\u90e1\u738b)\uc5d0\uc11c \uc548\ub0a8\uad6d\uc655(\u5b89\u5357\u570b\u738b)\uc73c\ub85c \uaca9\uc0c1\ub418\uc5b4 \ucc45\ubd09\ub418\uba70, \uc678\uc655\ub0b4\uc81c \uccb4\uc81c\ub97c \ud655\ub9bd\ud568\uacfc \ub3d9\uc2dc\uc5d0 \uad70\uc8fc\ub4e4\uc5d0\uac8c \ubb18\ud638\ub97c \ubd99\uc600\ub2e4. \uc911\uac04\uc5d0 \uba85\ub098\ub77c\uac00 \ubca0\ud2b8\ub0a8\uc744 \uce68\ub7b5\ud558\uc5ec \uc548\ub0a8\uad6d \ub300\uc2e0 \uad50\uc9c0\uad6d\uc73c\ub85c \uadf8 \uaca9\uc744 \ub0ae\ucd94\uace0, \uc5ed\uc2dc \ubb18\ud638\uc640 \uc2dc\ud638\ub97c \uc4f0\ub294 \ub4f1\uc758 \ud48d\uc2b5\uc744 \uae08\ud558\uc600\uc73c\ub098, 20\ub144\ub9cc\uc5d0 \ub808 \uc655\uc870\uac00 \ub4e4\uc5b4\uc11c\uba70 \ub2e4\uc2dc \ubb18\uc81c\ub97c \ubd80\ud65c\uc2dc\ucf30\ub2e4. \ubca0\ud2b8\ub0a8\uc778\ub4e4\uc740 \ub9c8\uc9c0\ub9c9 \uc655\uc870\uc778 \uc751\uc6b0\uc60c \uc655\uc870\uae4c\uc9c0 \uc81c\ub3c4\ub97c \uc720\uc9c0\ud558\uc5ec \uad70\uc8fc\ub4e4\uc5d0\uac8c \ubaa8\ub450 \ubb18\ud638\uc640 \uc2dc\ud638\ub97c \uc62c\ub838\ub2e4.", "answer": "\uc804 \ub808 \uc655\uc870", "sentence": "\ubca0\ud2b8\ub0a8\uc5d0\uc11c \uac00\uc7a5 \uba3c\uc800 \ubb18\ud638\ub97c \uc4f0\uae30 \uc2dc\uc791\ud55c \uc655\uc870\ub294 \uc804 \ub808 \uc655\uc870 \uc600\uc73c\ub098, \uc2dc\ud638\ub97c \uc62c\ub9ac\uc9c0 \uc54a\uace0 \ubb18\uc81c \ub610\ud55c \uc815\ub9bd\ub418\uc9c0 \uc54a\uc544, \uc81c\ub300\ub85c \uc808\ucc28\ub97c \ubc1f\uc544 \uc62c\ub824\uc9c4 \ubb18\ud638\uac00 \uc544\ub2c8\uc5c8\ub2e4.", "paragraph_sentence": "\ubca0\ud2b8\ub0a8\ub3c4 \uc678\uc655\ub0b4\uc81c\ub97c \uc9c0\ud5a5\ud558\uc5ec \uc548\uc73c\ub85c\ub294 \ud669\uc81c\ub97c \uc790\uce6d\ud558\uc600\uace0, \uc7ac\uc704\ud55c \uad70\uc8fc\ub4e4\uc5d0\uac8c \ubb18\ud638\ub97c \ubd99\uc600\ub2e4. \ubca0\ud2b8\ub0a8\uc5d0\uc11c \uac00\uc7a5 \uba3c\uc800 \ubb18\ud638\ub97c \uc4f0\uae30 \uc2dc\uc791\ud55c \uc655\uc870\ub294 \uc804 \ub808 \uc655\uc870 \uc600\uc73c\ub098, \uc2dc\ud638\ub97c \uc62c\ub9ac\uc9c0 \uc54a\uace0 \ubb18\uc81c \ub610\ud55c \uc815\ub9bd\ub418\uc9c0 \uc54a\uc544, \uc81c\ub300\ub85c \uc808\ucc28\ub97c \ubc1f\uc544 \uc62c\ub824\uc9c4 \ubb18\ud638\uac00 \uc544\ub2c8\uc5c8\ub2e4. \uc774\ud6c4 \ubcf8\uaca9\uc801\uc73c\ub85c \ubb18\ud638 \uc81c\ub3c4\uac00 \uc815\ucc29\ub41c \uc655\uc870\ub294 12\uc138\uae30 \uc911\uc5fd\uc5d0 \uc138\uc6cc\uc9c4 \ub9ac \uc655\uc870\ub85c, \uc911\uad6d\uc758 \ub0a8\uc1a1\uc5d0 \uc758\ud574 \uc774\uc804\uc758 \uad50\uc9c0\uad70\uc655(\u4ea4\u8dbe\u90e1\u738b)\uc5d0\uc11c \uc548\ub0a8\uad6d\uc655(\u5b89\u5357\u570b\u738b)\uc73c\ub85c \uaca9\uc0c1\ub418\uc5b4 \ucc45\ubd09\ub418\uba70, \uc678\uc655\ub0b4\uc81c \uccb4\uc81c\ub97c \ud655\ub9bd\ud568\uacfc \ub3d9\uc2dc\uc5d0 \uad70\uc8fc\ub4e4\uc5d0\uac8c \ubb18\ud638\ub97c \ubd99\uc600\ub2e4. \uc911\uac04\uc5d0 \uba85\ub098\ub77c\uac00 \ubca0\ud2b8\ub0a8\uc744 \uce68\ub7b5\ud558\uc5ec \uc548\ub0a8\uad6d \ub300\uc2e0 \uad50\uc9c0\uad6d\uc73c\ub85c \uadf8 \uaca9\uc744 \ub0ae\ucd94\uace0, \uc5ed\uc2dc \ubb18\ud638\uc640 \uc2dc\ud638\ub97c \uc4f0\ub294 \ub4f1\uc758 \ud48d\uc2b5\uc744 \uae08\ud558\uc600\uc73c\ub098, 20\ub144\ub9cc\uc5d0 \ub808 \uc655\uc870\uac00 \ub4e4\uc5b4\uc11c\uba70 \ub2e4\uc2dc \ubb18\uc81c\ub97c \ubd80\ud65c\uc2dc\ucf30\ub2e4. \ubca0\ud2b8\ub0a8\uc778\ub4e4\uc740 \ub9c8\uc9c0\ub9c9 \uc655\uc870\uc778 \uc751\uc6b0\uc60c \uc655\uc870\uae4c\uc9c0 \uc81c\ub3c4\ub97c \uc720\uc9c0\ud558\uc5ec \uad70\uc8fc\ub4e4\uc5d0\uac8c \ubaa8\ub450 \ubb18\ud638\uc640 \uc2dc\ud638\ub97c \uc62c\ub838\ub2e4.", "paragraph_answer": "\ubca0\ud2b8\ub0a8\ub3c4 \uc678\uc655\ub0b4\uc81c\ub97c \uc9c0\ud5a5\ud558\uc5ec \uc548\uc73c\ub85c\ub294 \ud669\uc81c\ub97c \uc790\uce6d\ud558\uc600\uace0, \uc7ac\uc704\ud55c \uad70\uc8fc\ub4e4\uc5d0\uac8c \ubb18\ud638\ub97c \ubd99\uc600\ub2e4. \ubca0\ud2b8\ub0a8\uc5d0\uc11c \uac00\uc7a5 \uba3c\uc800 \ubb18\ud638\ub97c \uc4f0\uae30 \uc2dc\uc791\ud55c \uc655\uc870\ub294 \uc804 \ub808 \uc655\uc870 \uc600\uc73c\ub098, \uc2dc\ud638\ub97c \uc62c\ub9ac\uc9c0 \uc54a\uace0 \ubb18\uc81c \ub610\ud55c \uc815\ub9bd\ub418\uc9c0 \uc54a\uc544, \uc81c\ub300\ub85c \uc808\ucc28\ub97c \ubc1f\uc544 \uc62c\ub824\uc9c4 \ubb18\ud638\uac00 \uc544\ub2c8\uc5c8\ub2e4. \uc774\ud6c4 \ubcf8\uaca9\uc801\uc73c\ub85c \ubb18\ud638 \uc81c\ub3c4\uac00 \uc815\ucc29\ub41c \uc655\uc870\ub294 12\uc138\uae30 \uc911\uc5fd\uc5d0 \uc138\uc6cc\uc9c4 \ub9ac \uc655\uc870\ub85c, \uc911\uad6d\uc758 \ub0a8\uc1a1\uc5d0 \uc758\ud574 \uc774\uc804\uc758 \uad50\uc9c0\uad70\uc655(\u4ea4\u8dbe\u90e1\u738b)\uc5d0\uc11c \uc548\ub0a8\uad6d\uc655(\u5b89\u5357\u570b\u738b)\uc73c\ub85c \uaca9\uc0c1\ub418\uc5b4 \ucc45\ubd09\ub418\uba70, \uc678\uc655\ub0b4\uc81c \uccb4\uc81c\ub97c \ud655\ub9bd\ud568\uacfc \ub3d9\uc2dc\uc5d0 \uad70\uc8fc\ub4e4\uc5d0\uac8c \ubb18\ud638\ub97c \ubd99\uc600\ub2e4. \uc911\uac04\uc5d0 \uba85\ub098\ub77c\uac00 \ubca0\ud2b8\ub0a8\uc744 \uce68\ub7b5\ud558\uc5ec \uc548\ub0a8\uad6d \ub300\uc2e0 \uad50\uc9c0\uad6d\uc73c\ub85c \uadf8 \uaca9\uc744 \ub0ae\ucd94\uace0, \uc5ed\uc2dc \ubb18\ud638\uc640 \uc2dc\ud638\ub97c \uc4f0\ub294 \ub4f1\uc758 \ud48d\uc2b5\uc744 \uae08\ud558\uc600\uc73c\ub098, 20\ub144\ub9cc\uc5d0 \ub808 \uc655\uc870\uac00 \ub4e4\uc5b4\uc11c\uba70 \ub2e4\uc2dc \ubb18\uc81c\ub97c \ubd80\ud65c\uc2dc\ucf30\ub2e4. \ubca0\ud2b8\ub0a8\uc778\ub4e4\uc740 \ub9c8\uc9c0\ub9c9 \uc655\uc870\uc778 \uc751\uc6b0\uc60c \uc655\uc870\uae4c\uc9c0 \uc81c\ub3c4\ub97c \uc720\uc9c0\ud558\uc5ec \uad70\uc8fc\ub4e4\uc5d0\uac8c \ubaa8\ub450 \ubb18\ud638\uc640 \uc2dc\ud638\ub97c \uc62c\ub838\ub2e4.", "sentence_answer": "\ubca0\ud2b8\ub0a8\uc5d0\uc11c \uac00\uc7a5 \uba3c\uc800 \ubb18\ud638\ub97c \uc4f0\uae30 \uc2dc\uc791\ud55c \uc655\uc870\ub294 \uc804 \ub808 \uc655\uc870 \uc600\uc73c\ub098, \uc2dc\ud638\ub97c \uc62c\ub9ac\uc9c0 \uc54a\uace0 \ubb18\uc81c \ub610\ud55c \uc815\ub9bd\ub418\uc9c0 \uc54a\uc544, \uc81c\ub300\ub85c \uc808\ucc28\ub97c \ubc1f\uc544 \uc62c\ub824\uc9c4 \ubb18\ud638\uac00 \uc544\ub2c8\uc5c8\ub2e4.", "paragraph_id": "6504692-13-0", "paragraph_question": "question: \ubca0\ud2b8\ub0a8\uc5d0\uc11c \uac00\uc7a5 \uba3c\uc800 \ubb18\ud638\ub97c \uc0ac\uc6a9\ud55c \uc655\uc870\ub294?, context: \ubca0\ud2b8\ub0a8\ub3c4 \uc678\uc655\ub0b4\uc81c\ub97c \uc9c0\ud5a5\ud558\uc5ec \uc548\uc73c\ub85c\ub294 \ud669\uc81c\ub97c \uc790\uce6d\ud558\uc600\uace0, \uc7ac\uc704\ud55c \uad70\uc8fc\ub4e4\uc5d0\uac8c \ubb18\ud638\ub97c \ubd99\uc600\ub2e4. \ubca0\ud2b8\ub0a8\uc5d0\uc11c \uac00\uc7a5 \uba3c\uc800 \ubb18\ud638\ub97c \uc4f0\uae30 \uc2dc\uc791\ud55c \uc655\uc870\ub294 \uc804 \ub808 \uc655\uc870\uc600\uc73c\ub098, \uc2dc\ud638\ub97c \uc62c\ub9ac\uc9c0 \uc54a\uace0 \ubb18\uc81c \ub610\ud55c \uc815\ub9bd\ub418\uc9c0 \uc54a\uc544, \uc81c\ub300\ub85c \uc808\ucc28\ub97c \ubc1f\uc544 \uc62c\ub824\uc9c4 \ubb18\ud638\uac00 \uc544\ub2c8\uc5c8\ub2e4. \uc774\ud6c4 \ubcf8\uaca9\uc801\uc73c\ub85c \ubb18\ud638 \uc81c\ub3c4\uac00 \uc815\ucc29\ub41c \uc655\uc870\ub294 12\uc138\uae30 \uc911\uc5fd\uc5d0 \uc138\uc6cc\uc9c4 \ub9ac \uc655\uc870\ub85c, \uc911\uad6d\uc758 \ub0a8\uc1a1\uc5d0 \uc758\ud574 \uc774\uc804\uc758 \uad50\uc9c0\uad70\uc655(\u4ea4\u8dbe\u90e1\u738b)\uc5d0\uc11c \uc548\ub0a8\uad6d\uc655(\u5b89\u5357\u570b\u738b)\uc73c\ub85c \uaca9\uc0c1\ub418\uc5b4 \ucc45\ubd09\ub418\uba70, \uc678\uc655\ub0b4\uc81c \uccb4\uc81c\ub97c \ud655\ub9bd\ud568\uacfc \ub3d9\uc2dc\uc5d0 \uad70\uc8fc\ub4e4\uc5d0\uac8c \ubb18\ud638\ub97c \ubd99\uc600\ub2e4. \uc911\uac04\uc5d0 \uba85\ub098\ub77c\uac00 \ubca0\ud2b8\ub0a8\uc744 \uce68\ub7b5\ud558\uc5ec \uc548\ub0a8\uad6d \ub300\uc2e0 \uad50\uc9c0\uad6d\uc73c\ub85c \uadf8 \uaca9\uc744 \ub0ae\ucd94\uace0, \uc5ed\uc2dc \ubb18\ud638\uc640 \uc2dc\ud638\ub97c \uc4f0\ub294 \ub4f1\uc758 \ud48d\uc2b5\uc744 \uae08\ud558\uc600\uc73c\ub098, 20\ub144\ub9cc\uc5d0 \ub808 \uc655\uc870\uac00 \ub4e4\uc5b4\uc11c\uba70 \ub2e4\uc2dc \ubb18\uc81c\ub97c \ubd80\ud65c\uc2dc\ucf30\ub2e4. \ubca0\ud2b8\ub0a8\uc778\ub4e4\uc740 \ub9c8\uc9c0\ub9c9 \uc655\uc870\uc778 \uc751\uc6b0\uc60c \uc655\uc870\uae4c\uc9c0 \uc81c\ub3c4\ub97c \uc720\uc9c0\ud558\uc5ec \uad70\uc8fc\ub4e4\uc5d0\uac8c \ubaa8\ub450 \ubb18\ud638\uc640 \uc2dc\ud638\ub97c \uc62c\ub838\ub2e4."} {"question": "\ud604\uc1a1\uc6d4\uc774 \ubc29\ub0a8\ud558\ub294 \ub3d9\uc548 \uacbd\ud638\ub97c \ub9e1\uc740 \uae30\uad00\uc740?", "paragraph": "\uc815\uce58\uad8c\uc5d0\uc11c\ub3c4 \uc815\uc7c1\uc774 \uc77c\uc5b4\ub0ac\ub294\ub370 \uc790\uc720\ud55c\uad6d\ub2f9\uc740 \"\uc21f\uac00\ub77d\ub9cc \ub4e4\uace0 \ub098\ud0c0\ub09c \uc800\ub4e4\uc774 \ud3c9\ucc3d \ub3d9\uacc4\uc62c\ub9bc\ud53d\uc744 \ud3c9\uc591 \uc62c\ub9bc\ud53d\uacfc \uae40\uc815\uc740 \ub3c5\uc7ac \uccb4\uc81c \uc120\uc804\uc7a5\uc73c\ub85c \ub9cc\ub4e4\uace0 \uc788\ub2e4\"\uba70 \"\ucc38\uc73c\ub85c \uc548\ud0c0\uae5d\uace0 \ubd80\ub044\ub7fd\ub2e4\"\uace0 \ube44\ud310\ud588\ub2e4. \uc774\uc5d0 \ub300\ud574 \ub354\ubd88\uc5b4\ubbfc\uc8fc\ub2f9\uc740 \"\ucd5c\uadfc \ud55c\uad6d\ub2f9 \ub300\ud45c\ubd80\ud130 \ub300\ubcc0\uc778\uae4c\uc9c0 \ubc18\uacf5\uc8fc\uc758\ub97c \uc5f0\uc0c1\uc2dc\ud0a4\ub294 \uadf9\uc6b0\uc801 \ubc1c\uc5b8\uc774 \ubaa9\ubd88\uc778\uacac\"\uc774\ub77c\uba70 \"\ud3c9\ucc3d\ud589 \ud3c9\ud654 \uc5f4\ucc28\uc5d0 \uc5b4\ub5bb\uac8c\ub4e0 \uc81c\ub3d9\uc744 \uac78\ub824\ub294 \ud55c\uad6d\ub2f9\uc5d0 \ucc38\uc73c\ub85c \uc720\uac10\"\uc774\ub780 \ub73b\uc744 \ubc1d\ud614\ub2e4. \ud604\uc1a1\uc6d4\uc758 \ubc29\ub0a8 \uacfc\uc815\uc5d0\uc11c\ub3c4 \uc608\uc6b0\uac00 \uc9c0\ub098\ucce4\ub2e4\ub294 \ube44\ud310\uc774 \uc788\uc5c8\ub2e4. \ud604\uc1a1\uc6d4 \uc77c\ud589\uc774 \uc9c0\ub098\ub294 \ub3c4\uc2ec \uc77c\ub300 \ub4f1 \uacf3\uacf3\uc758 \uad50\ud1b5\uc744 \ud1b5\uc81c\ud588\uc73c\uba70 \uc11c\uc6b8\uc5ed\uc5d0\ub294 \uc758\uacbd 720\uba85\uc744 \ubc30\uce58\ud588\ub2e4. \ud604\uc1a1\uc6d4\uc758 \uadfc\uc811\uacbd\ud638\ub294 \uad6d\uac00\uc815\ubcf4\uc6d0\uc774 \ub9e1\uc558\ub294\ub370 \ucde8\uc7ac\uc9c4\uc758 \uc811\uadfc\uc5d0\ub294 \"\ubd88\ud3b8\ud574\ud55c\ub2e4. \uc9c8\ubb38 \uc790\uafb8 \ud558\uc9c0 \ub9d0\ub77c\"\uba70 \uac00\ub85c\ub9c9\uc558\ub2e4. \uc774\uc5d0 \ub300\ud574 \ud55c\uad6d\ub2f9\uc740 \"\ub3d9\uacc4\uc62c\ub9bc\ud53d\uc744 \ud558\uaca0\ub2e4\ub294 \uac74\uc9c0, \ubd81\ud55c \uc608\uc220\ub2e8 \ucd08\uccad \ub3d9\uacc4\ubb38\ud654\uc81c\ub97c \ud558\uaca0\ub2e4\ub294 \uac83\uc778\uc9c0 \uad6c\ubd84\uc774 \ub418\uc9c0 \uc54a\ub294\ub2e4\"\uba70 \"\uc2ec\uc9c0\uc5b4 \uc77c\ubd80 \uc5b8\ub860\uc5d0\uc11c\ub294 \ud604 \ub2e8\uc7a5\uc758 \uc6c0\uc9c1\uc784\uc744 \ud558\ub8e8\uc885\uc77c \uc18d\ubcf4\ub85c \ubc29\uc1a1\uc744 \ud588\ub2e4\"\uba70 \ube44\ud310\ud588\ub2e4.", "answer": "\uad6d\uac00\uc815\ubcf4\uc6d0", "sentence": "\ud604\uc1a1\uc6d4\uc758 \uadfc\uc811\uacbd\ud638\ub294 \uad6d\uac00\uc815\ubcf4\uc6d0 \uc774 \ub9e1\uc558\ub294\ub370 \ucde8\uc7ac\uc9c4\uc758 \uc811\uadfc\uc5d0\ub294 \"\ubd88\ud3b8\ud574\ud55c\ub2e4.", "paragraph_sentence": "\uc815\uce58\uad8c\uc5d0\uc11c\ub3c4 \uc815\uc7c1\uc774 \uc77c\uc5b4\ub0ac\ub294\ub370 \uc790\uc720\ud55c\uad6d\ub2f9\uc740 \"\uc21f\uac00\ub77d\ub9cc \ub4e4\uace0 \ub098\ud0c0\ub09c \uc800\ub4e4\uc774 \ud3c9\ucc3d \ub3d9\uacc4\uc62c\ub9bc\ud53d\uc744 \ud3c9\uc591 \uc62c\ub9bc\ud53d\uacfc \uae40\uc815\uc740 \ub3c5\uc7ac \uccb4\uc81c \uc120\uc804\uc7a5\uc73c\ub85c \ub9cc\ub4e4\uace0 \uc788\ub2e4\"\uba70 \"\ucc38\uc73c\ub85c \uc548\ud0c0\uae5d\uace0 \ubd80\ub044\ub7fd\ub2e4\"\uace0 \ube44\ud310\ud588\ub2e4. \uc774\uc5d0 \ub300\ud574 \ub354\ubd88\uc5b4\ubbfc\uc8fc\ub2f9\uc740 \"\ucd5c\uadfc \ud55c\uad6d\ub2f9 \ub300\ud45c\ubd80\ud130 \ub300\ubcc0\uc778\uae4c\uc9c0 \ubc18\uacf5\uc8fc\uc758\ub97c \uc5f0\uc0c1\uc2dc\ud0a4\ub294 \uadf9\uc6b0\uc801 \ubc1c\uc5b8\uc774 \ubaa9\ubd88\uc778\uacac\"\uc774\ub77c\uba70 \"\ud3c9\ucc3d\ud589 \ud3c9\ud654 \uc5f4\ucc28\uc5d0 \uc5b4\ub5bb\uac8c\ub4e0 \uc81c\ub3d9\uc744 \uac78\ub824\ub294 \ud55c\uad6d\ub2f9\uc5d0 \ucc38\uc73c\ub85c \uc720\uac10\"\uc774\ub780 \ub73b\uc744 \ubc1d\ud614\ub2e4. \ud604\uc1a1\uc6d4\uc758 \ubc29\ub0a8 \uacfc\uc815\uc5d0\uc11c\ub3c4 \uc608\uc6b0\uac00 \uc9c0\ub098\ucce4\ub2e4\ub294 \ube44\ud310\uc774 \uc788\uc5c8\ub2e4. \ud604\uc1a1\uc6d4 \uc77c\ud589\uc774 \uc9c0\ub098\ub294 \ub3c4\uc2ec \uc77c\ub300 \ub4f1 \uacf3\uacf3\uc758 \uad50\ud1b5\uc744 \ud1b5\uc81c\ud588\uc73c\uba70 \uc11c\uc6b8\uc5ed\uc5d0\ub294 \uc758\uacbd 720\uba85\uc744 \ubc30\uce58\ud588\ub2e4. \ud604\uc1a1\uc6d4\uc758 \uadfc\uc811\uacbd\ud638\ub294 \uad6d\uac00\uc815\ubcf4\uc6d0 \uc774 \ub9e1\uc558\ub294\ub370 \ucde8\uc7ac\uc9c4\uc758 \uc811\uadfc\uc5d0\ub294 \"\ubd88\ud3b8\ud574\ud55c\ub2e4. \uc9c8\ubb38 \uc790\uafb8 \ud558\uc9c0 \ub9d0\ub77c\"\uba70 \uac00\ub85c\ub9c9\uc558\ub2e4. \uc774\uc5d0 \ub300\ud574 \ud55c\uad6d\ub2f9\uc740 \"\ub3d9\uacc4\uc62c\ub9bc\ud53d\uc744 \ud558\uaca0\ub2e4\ub294 \uac74\uc9c0, \ubd81\ud55c \uc608\uc220\ub2e8 \ucd08\uccad \ub3d9\uacc4\ubb38\ud654\uc81c\ub97c \ud558\uaca0\ub2e4\ub294 \uac83\uc778\uc9c0 \uad6c\ubd84\uc774 \ub418\uc9c0 \uc54a\ub294\ub2e4\"\uba70 \"\uc2ec\uc9c0\uc5b4 \uc77c\ubd80 \uc5b8\ub860\uc5d0\uc11c\ub294 \ud604 \ub2e8\uc7a5\uc758 \uc6c0\uc9c1\uc784\uc744 \ud558\ub8e8\uc885\uc77c \uc18d\ubcf4\ub85c \ubc29\uc1a1\uc744 \ud588\ub2e4\"\uba70 \ube44\ud310\ud588\ub2e4.", "paragraph_answer": "\uc815\uce58\uad8c\uc5d0\uc11c\ub3c4 \uc815\uc7c1\uc774 \uc77c\uc5b4\ub0ac\ub294\ub370 \uc790\uc720\ud55c\uad6d\ub2f9\uc740 \"\uc21f\uac00\ub77d\ub9cc \ub4e4\uace0 \ub098\ud0c0\ub09c \uc800\ub4e4\uc774 \ud3c9\ucc3d \ub3d9\uacc4\uc62c\ub9bc\ud53d\uc744 \ud3c9\uc591 \uc62c\ub9bc\ud53d\uacfc \uae40\uc815\uc740 \ub3c5\uc7ac \uccb4\uc81c \uc120\uc804\uc7a5\uc73c\ub85c \ub9cc\ub4e4\uace0 \uc788\ub2e4\"\uba70 \"\ucc38\uc73c\ub85c \uc548\ud0c0\uae5d\uace0 \ubd80\ub044\ub7fd\ub2e4\"\uace0 \ube44\ud310\ud588\ub2e4. \uc774\uc5d0 \ub300\ud574 \ub354\ubd88\uc5b4\ubbfc\uc8fc\ub2f9\uc740 \"\ucd5c\uadfc \ud55c\uad6d\ub2f9 \ub300\ud45c\ubd80\ud130 \ub300\ubcc0\uc778\uae4c\uc9c0 \ubc18\uacf5\uc8fc\uc758\ub97c \uc5f0\uc0c1\uc2dc\ud0a4\ub294 \uadf9\uc6b0\uc801 \ubc1c\uc5b8\uc774 \ubaa9\ubd88\uc778\uacac\"\uc774\ub77c\uba70 \"\ud3c9\ucc3d\ud589 \ud3c9\ud654 \uc5f4\ucc28\uc5d0 \uc5b4\ub5bb\uac8c\ub4e0 \uc81c\ub3d9\uc744 \uac78\ub824\ub294 \ud55c\uad6d\ub2f9\uc5d0 \ucc38\uc73c\ub85c \uc720\uac10\"\uc774\ub780 \ub73b\uc744 \ubc1d\ud614\ub2e4. \ud604\uc1a1\uc6d4\uc758 \ubc29\ub0a8 \uacfc\uc815\uc5d0\uc11c\ub3c4 \uc608\uc6b0\uac00 \uc9c0\ub098\ucce4\ub2e4\ub294 \ube44\ud310\uc774 \uc788\uc5c8\ub2e4. \ud604\uc1a1\uc6d4 \uc77c\ud589\uc774 \uc9c0\ub098\ub294 \ub3c4\uc2ec \uc77c\ub300 \ub4f1 \uacf3\uacf3\uc758 \uad50\ud1b5\uc744 \ud1b5\uc81c\ud588\uc73c\uba70 \uc11c\uc6b8\uc5ed\uc5d0\ub294 \uc758\uacbd 720\uba85\uc744 \ubc30\uce58\ud588\ub2e4. \ud604\uc1a1\uc6d4\uc758 \uadfc\uc811\uacbd\ud638\ub294 \uad6d\uac00\uc815\ubcf4\uc6d0 \uc774 \ub9e1\uc558\ub294\ub370 \ucde8\uc7ac\uc9c4\uc758 \uc811\uadfc\uc5d0\ub294 \"\ubd88\ud3b8\ud574\ud55c\ub2e4. \uc9c8\ubb38 \uc790\uafb8 \ud558\uc9c0 \ub9d0\ub77c\"\uba70 \uac00\ub85c\ub9c9\uc558\ub2e4. \uc774\uc5d0 \ub300\ud574 \ud55c\uad6d\ub2f9\uc740 \"\ub3d9\uacc4\uc62c\ub9bc\ud53d\uc744 \ud558\uaca0\ub2e4\ub294 \uac74\uc9c0, \ubd81\ud55c \uc608\uc220\ub2e8 \ucd08\uccad \ub3d9\uacc4\ubb38\ud654\uc81c\ub97c \ud558\uaca0\ub2e4\ub294 \uac83\uc778\uc9c0 \uad6c\ubd84\uc774 \ub418\uc9c0 \uc54a\ub294\ub2e4\"\uba70 \"\uc2ec\uc9c0\uc5b4 \uc77c\ubd80 \uc5b8\ub860\uc5d0\uc11c\ub294 \ud604 \ub2e8\uc7a5\uc758 \uc6c0\uc9c1\uc784\uc744 \ud558\ub8e8\uc885\uc77c \uc18d\ubcf4\ub85c \ubc29\uc1a1\uc744 \ud588\ub2e4\"\uba70 \ube44\ud310\ud588\ub2e4.", "sentence_answer": "\ud604\uc1a1\uc6d4\uc758 \uadfc\uc811\uacbd\ud638\ub294 \uad6d\uac00\uc815\ubcf4\uc6d0 \uc774 \ub9e1\uc558\ub294\ub370 \ucde8\uc7ac\uc9c4\uc758 \uc811\uadfc\uc5d0\ub294 \"\ubd88\ud3b8\ud574\ud55c\ub2e4.", "paragraph_id": "6546926-109-1", "paragraph_question": "question: \ud604\uc1a1\uc6d4\uc774 \ubc29\ub0a8\ud558\ub294 \ub3d9\uc548 \uacbd\ud638\ub97c \ub9e1\uc740 \uae30\uad00\uc740?, context: \uc815\uce58\uad8c\uc5d0\uc11c\ub3c4 \uc815\uc7c1\uc774 \uc77c\uc5b4\ub0ac\ub294\ub370 \uc790\uc720\ud55c\uad6d\ub2f9\uc740 \"\uc21f\uac00\ub77d\ub9cc \ub4e4\uace0 \ub098\ud0c0\ub09c \uc800\ub4e4\uc774 \ud3c9\ucc3d \ub3d9\uacc4\uc62c\ub9bc\ud53d\uc744 \ud3c9\uc591 \uc62c\ub9bc\ud53d\uacfc \uae40\uc815\uc740 \ub3c5\uc7ac \uccb4\uc81c \uc120\uc804\uc7a5\uc73c\ub85c \ub9cc\ub4e4\uace0 \uc788\ub2e4\"\uba70 \"\ucc38\uc73c\ub85c \uc548\ud0c0\uae5d\uace0 \ubd80\ub044\ub7fd\ub2e4\"\uace0 \ube44\ud310\ud588\ub2e4. \uc774\uc5d0 \ub300\ud574 \ub354\ubd88\uc5b4\ubbfc\uc8fc\ub2f9\uc740 \"\ucd5c\uadfc \ud55c\uad6d\ub2f9 \ub300\ud45c\ubd80\ud130 \ub300\ubcc0\uc778\uae4c\uc9c0 \ubc18\uacf5\uc8fc\uc758\ub97c \uc5f0\uc0c1\uc2dc\ud0a4\ub294 \uadf9\uc6b0\uc801 \ubc1c\uc5b8\uc774 \ubaa9\ubd88\uc778\uacac\"\uc774\ub77c\uba70 \"\ud3c9\ucc3d\ud589 \ud3c9\ud654 \uc5f4\ucc28\uc5d0 \uc5b4\ub5bb\uac8c\ub4e0 \uc81c\ub3d9\uc744 \uac78\ub824\ub294 \ud55c\uad6d\ub2f9\uc5d0 \ucc38\uc73c\ub85c \uc720\uac10\"\uc774\ub780 \ub73b\uc744 \ubc1d\ud614\ub2e4. \ud604\uc1a1\uc6d4\uc758 \ubc29\ub0a8 \uacfc\uc815\uc5d0\uc11c\ub3c4 \uc608\uc6b0\uac00 \uc9c0\ub098\ucce4\ub2e4\ub294 \ube44\ud310\uc774 \uc788\uc5c8\ub2e4. \ud604\uc1a1\uc6d4 \uc77c\ud589\uc774 \uc9c0\ub098\ub294 \ub3c4\uc2ec \uc77c\ub300 \ub4f1 \uacf3\uacf3\uc758 \uad50\ud1b5\uc744 \ud1b5\uc81c\ud588\uc73c\uba70 \uc11c\uc6b8\uc5ed\uc5d0\ub294 \uc758\uacbd 720\uba85\uc744 \ubc30\uce58\ud588\ub2e4. \ud604\uc1a1\uc6d4\uc758 \uadfc\uc811\uacbd\ud638\ub294 \uad6d\uac00\uc815\ubcf4\uc6d0\uc774 \ub9e1\uc558\ub294\ub370 \ucde8\uc7ac\uc9c4\uc758 \uc811\uadfc\uc5d0\ub294 \"\ubd88\ud3b8\ud574\ud55c\ub2e4. \uc9c8\ubb38 \uc790\uafb8 \ud558\uc9c0 \ub9d0\ub77c\"\uba70 \uac00\ub85c\ub9c9\uc558\ub2e4. \uc774\uc5d0 \ub300\ud574 \ud55c\uad6d\ub2f9\uc740 \"\ub3d9\uacc4\uc62c\ub9bc\ud53d\uc744 \ud558\uaca0\ub2e4\ub294 \uac74\uc9c0, \ubd81\ud55c \uc608\uc220\ub2e8 \ucd08\uccad \ub3d9\uacc4\ubb38\ud654\uc81c\ub97c \ud558\uaca0\ub2e4\ub294 \uac83\uc778\uc9c0 \uad6c\ubd84\uc774 \ub418\uc9c0 \uc54a\ub294\ub2e4\"\uba70 \"\uc2ec\uc9c0\uc5b4 \uc77c\ubd80 \uc5b8\ub860\uc5d0\uc11c\ub294 \ud604 \ub2e8\uc7a5\uc758 \uc6c0\uc9c1\uc784\uc744 \ud558\ub8e8\uc885\uc77c \uc18d\ubcf4\ub85c \ubc29\uc1a1\uc744 \ud588\ub2e4\"\uba70 \ube44\ud310\ud588\ub2e4."} {"question": "1848\ub144 10\uc6d4\uc5d0 \ud504\ub791\ud06c\ud478\ub974\ud2b8 \uad6d\ubbfc\uc758\ud68c\uc758 \uc9c4\uc601\uc740 \ud06c\uac8c \uba87\uac08\ub798\ub85c \ub098\ub258\uc5b4\uc84c\ub294\uac00?", "paragraph": "1848\ub144\uc740 \ub3c5\uc77c\uc5d0\uc11c \uc815\uce58\uc801 \uc870\uc9c1\ud654\uac00 \uc808\uc815\uc5d0 \uc774\ub978 \uc2dc\uae30\uc600\ub2e4. \ud601\uba85\uc5d0 \ucc38\uc5ec\ud55c \uc5ec\ub7ec \uc138\ub825\ub4e4\uc740 \uac01\uc790\uc758 \uad6d\ubbfc\uc815\uce58\uc801 \ubaa9\ud45c\ub97c \uad00\ucca0\ud558\uae30 \uc704\ud574 \ud074\ub7fd, \ud611\ud68c, \uacb0\uc0ac\uccb4 \ub4f1 \uc9d1\ub2e8\uc744 \uc870\uc9c1\ud574 \ub300\ub9bd\ud558\uc600\ub2e4. \uc774 \uac19\uc740 \uc791\uc5c5\uc740 \uad6d\ubbfc\uc758\ud68c\uc5d0\uc11c\ub3c4 \uc774\ub8e8\uc5b4\uc84c\ub2e4. \uac1c\ud68c \ud68c\uc758\ub294 \uc804\ubc18\uc801\uc73c\ub85c \ub9e4\uc6b0 \ud63c\ub780\uc2a4\ub7fd\uac8c \uc9c4\ud589\ub418\uc5c8\uace0 \uc758\uc6d0\ub4e4\ub3c4 \uc815\uce58\uc801 \uc785\uc7a5\uacfc \uad00\uacc4 \uc5c6\uc774 \ubb34\uc9c8\uc11c\ud558\uac8c \uc549\uc558\uc9c0\ub9cc, \uace7 \uc9c8\uc11c \uc7a1\ud78c \uc758\ud68c \uc791\uc5c5\uc774 \ube60\ub974\uac8c \ud615\uc131\ub418\uc5c8\ub2e4. \uc790\uc720\uc8fc\uc758\uc790, \ubcf4\uc218\uc8fc\uc758\uc790, \uac00\ud1a8\ub9ad\uc8fc\uc758\uc790, \ubbfc\uc8fc\uc8fc\uc758\uc790 \ub4f1\uc73c\ub85c \uad6c\uc131\ub41c \uc758\uc6d0\ub4e4\uc740 \uac01\uc790\uc758 \uc815\uce58\uc801 \uc785\uc7a5\uc5d0 \ub530\ub77c \uc758\ud68c \ubc16\uc758 \ud1a0\ub860\ubaa8\uc784\uc778 \ud074\ub7fd\uc744 \ub9cc\ub4e4\uc5c8\uace0, \uc774\uac83\uc740 \uc815\uce58\uc801 \ub2e4\uc218 \ud615\uc131\uc5d0 \ud544\uc218\uc801\uc778 \ub2f9\ud30c\uc758 \ud615\uc131\uc73c\ub85c \uc774\uc5b4\uc84c\ub2e4. \uc758\uc6d0\ub4e4\uc758 \uc785\uc7a5\uc5d0\uc11c \ub2f9\ud30c \ud65c\ub3d9\uc740 \ud544\uc694\uc545\uc774\uc5c8\uc9c0\ub9cc, \uadf8 \uacb0\uacfc\ub85c \ubbfc\uc8fc\uc801\uc774\uace0 \ub2e4\uc6d0\uc801\uc778 \uc815\ub2f9 \uc81c\ub3c4\uac00 \uc790\ub9ac\uc7a1\uc744 \uc218 \uc788\uc5c8\ub2e4. \uad6d\ubbfc\uc758\ud68c\uc758 \ub2f9\ud30c\ub4e4\uc740 \ud074\ub7fd\uc73c\ub85c \uc778\uc2dd\ub418\uc5c8\uae30 \ub54c\ubb38\uc5d0 \ubcf4\ud1b5 \ubaa8\uc784 \uc7a5\uc18c\uc5d0 \ub530\ub77c\uc11c \uc774\ub984 \uc9c0\uc5b4\uc84c\uace0, \uc804\ubc18\uc801\uc73c\ub85c \ub9e4\uc6b0 \uc720\ub3d9\uc801\uc774\uc5c8\ub2e4. \ub2f9\ud30c\ub4e4\uc740 \ud2b9\ud788 \ud5cc\ubc95, \uc758\ud68c\uc758 \uad8c\ub825, \uc5f0\ubc29 \uad6d\uac00\ub4e4\uacfc \ube44\uad50\ud55c \uc911\uc559 \uad8c\ub825\uc5d0 \ub300\ud55c \uadf8\ub4e4\uc758 \uc785\uc7a5\uc5d0 \ub530\ub77c\uc11c \ud06c\uac8c \uc138 \uac1c\uc758 \uae30\ubcf8 \uc9c4\uc601\uc73c\ub85c \ub098\ub258\uc5c8\ub2e4. 1848\ub144 10\uc6d4\uc5d0 \ub2f9\ud30c\ub4e4\uc740 \ub2e4\uc74c\uacfc \uac19\uc558\ub2e4.", "answer": "\uc138 \uac1c\uc758", "sentence": "\ub2f9\ud30c\ub4e4\uc740 \ud2b9\ud788 \ud5cc\ubc95, \uc758\ud68c\uc758 \uad8c\ub825, \uc5f0\ubc29 \uad6d\uac00\ub4e4\uacfc \ube44\uad50\ud55c \uc911\uc559 \uad8c\ub825\uc5d0 \ub300\ud55c \uadf8\ub4e4\uc758 \uc785\uc7a5\uc5d0 \ub530\ub77c\uc11c \ud06c\uac8c \uc138 \uac1c\uc758 \uae30\ubcf8 \uc9c4\uc601\uc73c\ub85c \ub098\ub258\uc5c8\ub2e4.", "paragraph_sentence": "1848\ub144\uc740 \ub3c5\uc77c\uc5d0\uc11c \uc815\uce58\uc801 \uc870\uc9c1\ud654\uac00 \uc808\uc815\uc5d0 \uc774\ub978 \uc2dc\uae30\uc600\ub2e4. \ud601\uba85\uc5d0 \ucc38\uc5ec\ud55c \uc5ec\ub7ec \uc138\ub825\ub4e4\uc740 \uac01\uc790\uc758 \uad6d\ubbfc\uc815\uce58\uc801 \ubaa9\ud45c\ub97c \uad00\ucca0\ud558\uae30 \uc704\ud574 \ud074\ub7fd, \ud611\ud68c, \uacb0\uc0ac\uccb4 \ub4f1 \uc9d1\ub2e8\uc744 \uc870\uc9c1\ud574 \ub300\ub9bd\ud558\uc600\ub2e4. \uc774 \uac19\uc740 \uc791\uc5c5\uc740 \uad6d\ubbfc\uc758\ud68c\uc5d0\uc11c\ub3c4 \uc774\ub8e8\uc5b4\uc84c\ub2e4. \uac1c\ud68c \ud68c\uc758\ub294 \uc804\ubc18\uc801\uc73c\ub85c \ub9e4\uc6b0 \ud63c\ub780\uc2a4\ub7fd\uac8c \uc9c4\ud589\ub418\uc5c8\uace0 \uc758\uc6d0\ub4e4\ub3c4 \uc815\uce58\uc801 \uc785\uc7a5\uacfc \uad00\uacc4 \uc5c6\uc774 \ubb34\uc9c8\uc11c\ud558\uac8c \uc549\uc558\uc9c0\ub9cc, \uace7 \uc9c8\uc11c \uc7a1\ud78c \uc758\ud68c \uc791\uc5c5\uc774 \ube60\ub974\uac8c \ud615\uc131\ub418\uc5c8\ub2e4. \uc790\uc720\uc8fc\uc758\uc790, \ubcf4\uc218\uc8fc\uc758\uc790, \uac00\ud1a8\ub9ad\uc8fc\uc758\uc790, \ubbfc\uc8fc\uc8fc\uc758\uc790 \ub4f1\uc73c\ub85c \uad6c\uc131\ub41c \uc758\uc6d0\ub4e4\uc740 \uac01\uc790\uc758 \uc815\uce58\uc801 \uc785\uc7a5\uc5d0 \ub530\ub77c \uc758\ud68c \ubc16\uc758 \ud1a0\ub860\ubaa8\uc784\uc778 \ud074\ub7fd\uc744 \ub9cc\ub4e4\uc5c8\uace0, \uc774\uac83\uc740 \uc815\uce58\uc801 \ub2e4\uc218 \ud615\uc131\uc5d0 \ud544\uc218\uc801\uc778 \ub2f9\ud30c\uc758 \ud615\uc131\uc73c\ub85c \uc774\uc5b4\uc84c\ub2e4. \uc758\uc6d0\ub4e4\uc758 \uc785\uc7a5\uc5d0\uc11c \ub2f9\ud30c \ud65c\ub3d9\uc740 \ud544\uc694\uc545\uc774\uc5c8\uc9c0\ub9cc, \uadf8 \uacb0\uacfc\ub85c \ubbfc\uc8fc\uc801\uc774\uace0 \ub2e4\uc6d0\uc801\uc778 \uc815\ub2f9 \uc81c\ub3c4\uac00 \uc790\ub9ac\uc7a1\uc744 \uc218 \uc788\uc5c8\ub2e4. \uad6d\ubbfc\uc758\ud68c\uc758 \ub2f9\ud30c\ub4e4\uc740 \ud074\ub7fd\uc73c\ub85c \uc778\uc2dd\ub418\uc5c8\uae30 \ub54c\ubb38\uc5d0 \ubcf4\ud1b5 \ubaa8\uc784 \uc7a5\uc18c\uc5d0 \ub530\ub77c\uc11c \uc774\ub984 \uc9c0\uc5b4\uc84c\uace0, \uc804\ubc18\uc801\uc73c\ub85c \ub9e4\uc6b0 \uc720\ub3d9\uc801\uc774\uc5c8\ub2e4. \ub2f9\ud30c\ub4e4\uc740 \ud2b9\ud788 \ud5cc\ubc95, \uc758\ud68c\uc758 \uad8c\ub825, \uc5f0\ubc29 \uad6d\uac00\ub4e4\uacfc \ube44\uad50\ud55c \uc911\uc559 \uad8c\ub825\uc5d0 \ub300\ud55c \uadf8\ub4e4\uc758 \uc785\uc7a5\uc5d0 \ub530\ub77c\uc11c \ud06c\uac8c \uc138 \uac1c\uc758 \uae30\ubcf8 \uc9c4\uc601\uc73c\ub85c \ub098\ub258\uc5c8\ub2e4. 1848\ub144 10\uc6d4\uc5d0 \ub2f9\ud30c\ub4e4\uc740 \ub2e4\uc74c\uacfc \uac19\uc558\ub2e4.", "paragraph_answer": "1848\ub144\uc740 \ub3c5\uc77c\uc5d0\uc11c \uc815\uce58\uc801 \uc870\uc9c1\ud654\uac00 \uc808\uc815\uc5d0 \uc774\ub978 \uc2dc\uae30\uc600\ub2e4. \ud601\uba85\uc5d0 \ucc38\uc5ec\ud55c \uc5ec\ub7ec \uc138\ub825\ub4e4\uc740 \uac01\uc790\uc758 \uad6d\ubbfc\uc815\uce58\uc801 \ubaa9\ud45c\ub97c \uad00\ucca0\ud558\uae30 \uc704\ud574 \ud074\ub7fd, \ud611\ud68c, \uacb0\uc0ac\uccb4 \ub4f1 \uc9d1\ub2e8\uc744 \uc870\uc9c1\ud574 \ub300\ub9bd\ud558\uc600\ub2e4. \uc774 \uac19\uc740 \uc791\uc5c5\uc740 \uad6d\ubbfc\uc758\ud68c\uc5d0\uc11c\ub3c4 \uc774\ub8e8\uc5b4\uc84c\ub2e4. \uac1c\ud68c \ud68c\uc758\ub294 \uc804\ubc18\uc801\uc73c\ub85c \ub9e4\uc6b0 \ud63c\ub780\uc2a4\ub7fd\uac8c \uc9c4\ud589\ub418\uc5c8\uace0 \uc758\uc6d0\ub4e4\ub3c4 \uc815\uce58\uc801 \uc785\uc7a5\uacfc \uad00\uacc4 \uc5c6\uc774 \ubb34\uc9c8\uc11c\ud558\uac8c \uc549\uc558\uc9c0\ub9cc, \uace7 \uc9c8\uc11c \uc7a1\ud78c \uc758\ud68c \uc791\uc5c5\uc774 \ube60\ub974\uac8c \ud615\uc131\ub418\uc5c8\ub2e4. \uc790\uc720\uc8fc\uc758\uc790, \ubcf4\uc218\uc8fc\uc758\uc790, \uac00\ud1a8\ub9ad\uc8fc\uc758\uc790, \ubbfc\uc8fc\uc8fc\uc758\uc790 \ub4f1\uc73c\ub85c \uad6c\uc131\ub41c \uc758\uc6d0\ub4e4\uc740 \uac01\uc790\uc758 \uc815\uce58\uc801 \uc785\uc7a5\uc5d0 \ub530\ub77c \uc758\ud68c \ubc16\uc758 \ud1a0\ub860\ubaa8\uc784\uc778 \ud074\ub7fd\uc744 \ub9cc\ub4e4\uc5c8\uace0, \uc774\uac83\uc740 \uc815\uce58\uc801 \ub2e4\uc218 \ud615\uc131\uc5d0 \ud544\uc218\uc801\uc778 \ub2f9\ud30c\uc758 \ud615\uc131\uc73c\ub85c \uc774\uc5b4\uc84c\ub2e4. \uc758\uc6d0\ub4e4\uc758 \uc785\uc7a5\uc5d0\uc11c \ub2f9\ud30c \ud65c\ub3d9\uc740 \ud544\uc694\uc545\uc774\uc5c8\uc9c0\ub9cc, \uadf8 \uacb0\uacfc\ub85c \ubbfc\uc8fc\uc801\uc774\uace0 \ub2e4\uc6d0\uc801\uc778 \uc815\ub2f9 \uc81c\ub3c4\uac00 \uc790\ub9ac\uc7a1\uc744 \uc218 \uc788\uc5c8\ub2e4. \uad6d\ubbfc\uc758\ud68c\uc758 \ub2f9\ud30c\ub4e4\uc740 \ud074\ub7fd\uc73c\ub85c \uc778\uc2dd\ub418\uc5c8\uae30 \ub54c\ubb38\uc5d0 \ubcf4\ud1b5 \ubaa8\uc784 \uc7a5\uc18c\uc5d0 \ub530\ub77c\uc11c \uc774\ub984 \uc9c0\uc5b4\uc84c\uace0, \uc804\ubc18\uc801\uc73c\ub85c \ub9e4\uc6b0 \uc720\ub3d9\uc801\uc774\uc5c8\ub2e4. \ub2f9\ud30c\ub4e4\uc740 \ud2b9\ud788 \ud5cc\ubc95, \uc758\ud68c\uc758 \uad8c\ub825, \uc5f0\ubc29 \uad6d\uac00\ub4e4\uacfc \ube44\uad50\ud55c \uc911\uc559 \uad8c\ub825\uc5d0 \ub300\ud55c \uadf8\ub4e4\uc758 \uc785\uc7a5\uc5d0 \ub530\ub77c\uc11c \ud06c\uac8c \uc138 \uac1c\uc758 \uae30\ubcf8 \uc9c4\uc601\uc73c\ub85c \ub098\ub258\uc5c8\ub2e4. 1848\ub144 10\uc6d4\uc5d0 \ub2f9\ud30c\ub4e4\uc740 \ub2e4\uc74c\uacfc \uac19\uc558\ub2e4.", "sentence_answer": "\ub2f9\ud30c\ub4e4\uc740 \ud2b9\ud788 \ud5cc\ubc95, \uc758\ud68c\uc758 \uad8c\ub825, \uc5f0\ubc29 \uad6d\uac00\ub4e4\uacfc \ube44\uad50\ud55c \uc911\uc559 \uad8c\ub825\uc5d0 \ub300\ud55c \uadf8\ub4e4\uc758 \uc785\uc7a5\uc5d0 \ub530\ub77c\uc11c \ud06c\uac8c \uc138 \uac1c\uc758 \uae30\ubcf8 \uc9c4\uc601\uc73c\ub85c \ub098\ub258\uc5c8\ub2e4.", "paragraph_id": "6465718-15-2", "paragraph_question": "question: 1848\ub144 10\uc6d4\uc5d0 \ud504\ub791\ud06c\ud478\ub974\ud2b8 \uad6d\ubbfc\uc758\ud68c\uc758 \uc9c4\uc601\uc740 \ud06c\uac8c \uba87\uac08\ub798\ub85c \ub098\ub258\uc5b4\uc84c\ub294\uac00?, context: 1848\ub144\uc740 \ub3c5\uc77c\uc5d0\uc11c \uc815\uce58\uc801 \uc870\uc9c1\ud654\uac00 \uc808\uc815\uc5d0 \uc774\ub978 \uc2dc\uae30\uc600\ub2e4. \ud601\uba85\uc5d0 \ucc38\uc5ec\ud55c \uc5ec\ub7ec \uc138\ub825\ub4e4\uc740 \uac01\uc790\uc758 \uad6d\ubbfc\uc815\uce58\uc801 \ubaa9\ud45c\ub97c \uad00\ucca0\ud558\uae30 \uc704\ud574 \ud074\ub7fd, \ud611\ud68c, \uacb0\uc0ac\uccb4 \ub4f1 \uc9d1\ub2e8\uc744 \uc870\uc9c1\ud574 \ub300\ub9bd\ud558\uc600\ub2e4. \uc774 \uac19\uc740 \uc791\uc5c5\uc740 \uad6d\ubbfc\uc758\ud68c\uc5d0\uc11c\ub3c4 \uc774\ub8e8\uc5b4\uc84c\ub2e4. \uac1c\ud68c \ud68c\uc758\ub294 \uc804\ubc18\uc801\uc73c\ub85c \ub9e4\uc6b0 \ud63c\ub780\uc2a4\ub7fd\uac8c \uc9c4\ud589\ub418\uc5c8\uace0 \uc758\uc6d0\ub4e4\ub3c4 \uc815\uce58\uc801 \uc785\uc7a5\uacfc \uad00\uacc4 \uc5c6\uc774 \ubb34\uc9c8\uc11c\ud558\uac8c \uc549\uc558\uc9c0\ub9cc, \uace7 \uc9c8\uc11c \uc7a1\ud78c \uc758\ud68c \uc791\uc5c5\uc774 \ube60\ub974\uac8c \ud615\uc131\ub418\uc5c8\ub2e4. \uc790\uc720\uc8fc\uc758\uc790, \ubcf4\uc218\uc8fc\uc758\uc790, \uac00\ud1a8\ub9ad\uc8fc\uc758\uc790, \ubbfc\uc8fc\uc8fc\uc758\uc790 \ub4f1\uc73c\ub85c \uad6c\uc131\ub41c \uc758\uc6d0\ub4e4\uc740 \uac01\uc790\uc758 \uc815\uce58\uc801 \uc785\uc7a5\uc5d0 \ub530\ub77c \uc758\ud68c \ubc16\uc758 \ud1a0\ub860\ubaa8\uc784\uc778 \ud074\ub7fd\uc744 \ub9cc\ub4e4\uc5c8\uace0, \uc774\uac83\uc740 \uc815\uce58\uc801 \ub2e4\uc218 \ud615\uc131\uc5d0 \ud544\uc218\uc801\uc778 \ub2f9\ud30c\uc758 \ud615\uc131\uc73c\ub85c \uc774\uc5b4\uc84c\ub2e4. \uc758\uc6d0\ub4e4\uc758 \uc785\uc7a5\uc5d0\uc11c \ub2f9\ud30c \ud65c\ub3d9\uc740 \ud544\uc694\uc545\uc774\uc5c8\uc9c0\ub9cc, \uadf8 \uacb0\uacfc\ub85c \ubbfc\uc8fc\uc801\uc774\uace0 \ub2e4\uc6d0\uc801\uc778 \uc815\ub2f9 \uc81c\ub3c4\uac00 \uc790\ub9ac\uc7a1\uc744 \uc218 \uc788\uc5c8\ub2e4. \uad6d\ubbfc\uc758\ud68c\uc758 \ub2f9\ud30c\ub4e4\uc740 \ud074\ub7fd\uc73c\ub85c \uc778\uc2dd\ub418\uc5c8\uae30 \ub54c\ubb38\uc5d0 \ubcf4\ud1b5 \ubaa8\uc784 \uc7a5\uc18c\uc5d0 \ub530\ub77c\uc11c \uc774\ub984 \uc9c0\uc5b4\uc84c\uace0, \uc804\ubc18\uc801\uc73c\ub85c \ub9e4\uc6b0 \uc720\ub3d9\uc801\uc774\uc5c8\ub2e4. \ub2f9\ud30c\ub4e4\uc740 \ud2b9\ud788 \ud5cc\ubc95, \uc758\ud68c\uc758 \uad8c\ub825, \uc5f0\ubc29 \uad6d\uac00\ub4e4\uacfc \ube44\uad50\ud55c \uc911\uc559 \uad8c\ub825\uc5d0 \ub300\ud55c \uadf8\ub4e4\uc758 \uc785\uc7a5\uc5d0 \ub530\ub77c\uc11c \ud06c\uac8c \uc138 \uac1c\uc758 \uae30\ubcf8 \uc9c4\uc601\uc73c\ub85c \ub098\ub258\uc5c8\ub2e4. 1848\ub144 10\uc6d4\uc5d0 \ub2f9\ud30c\ub4e4\uc740 \ub2e4\uc74c\uacfc \uac19\uc558\ub2e4."} {"question": "\ub300\uccb4\ub41c \uc804\uae30\ub3d9\ucc28 \ub0b4\ubd80\uc758 TV\ubc29\uc1a1\uae30\uae30\ub294 \uc5b4\ub5a4 \uc885\ub958\uc758 \ubaa8\ub2c8\ud130\ub97c \uc0ac\uc6a9\ud558\ub294\uac00?", "paragraph": "\uc2e4\ub0b4\ub294 \ud55c\uad6d\ucca0\ub3c4\uacf5\uc0ac\uc5d0\uc11c \uc8fc\ubb38\ud558\uc5ec \ud788\ud0c0\uce58 \uc81c\uc791\uc18c\uc5d0\uc11c \uacf5\uae09\ud55c \uc2dc\ud2b8\ub97c \uc0ac\uc6a9\ud558\uace0 \uc788\uace0 \uc804\uccb4\uc801\uc778 \uc2dc\ud2b8 \ud3ed\uacfc \ud53c\uce58\ub294 \ubb34\uad81\ud654\ud638\ubcf4\ub2e4 \uc57d\uac04 \ub113\uc73c\uba70 \uc911\uac04 \ud314\uac78\uc774 \ubc0f \ud14c\uc774\ube14\uc744 \uac16\ucd94\uace0 \uc788\uc73c\uba70 \ud55c\uad6d\uc778\uc758 \uccb4\ud615\uc5d0 \ub9de\uac8c \uc124\uacc4\ub418\uc5c8\ub2e4. \uc2e4\ub0b4 \ub0c9\ub09c\ubc29 \ubc0f \uacf5\uae30\uc815\ud654\ub97c \uad00\ub9ac\ud558\ub294 HVAC\ub97c \ucc98\uc74c\uc73c\ub85c \ucc44\ud0dd\ud558\uc600\uace0 LCD\ubaa8\ub2c8\ud130\ub97c \uc0ac\uc6a9\ud558\ub294 TV\ubc29\uc1a1\uae30\uae30\ub97c \uac16\ucd94\uace0 \uc788\uc73c\ub098 \ucf54\ub808\uc77c \ub9c8\ud06c\ub9cc \ub744\uc6cc\ub193\uace0 \uc0ac\uc6a9\ud558\uc9c0 \uc54a\ub294\ub2e4. \ud2b9\ud788 \uc120\ub450\ucc28 \uc6b4\uc804\uc2e4\uacfc \uac1d\uc2e4\uac04\uc758 \ubcbd\uc744 \uc804\uae30\uc801 \ud6a8\uacfc\uc5d0 \uc758\ud574 \ud22c\uba85\ud558\uac8c \ud560 \uc218 \uc788\ub294 \ubd88\ud22c\uba85 \uc720\ub9ac\ub97c \ucc44\ud0dd\ud558\uc600\uc73c\ub098, \uc6b4\uc804\uc2e4 \uacf5\uac1c\uc5d0 \uc775\uc219\uce58 \uc54a\uc740 \ud0d3\uc778\uc9c0 \ud3c9\uc18c\uc5d0\ub294 \ud750\ub9ac\uac8c \ud574 \ub450\uace0 \ub2e4\ub2cc\ub2e4. \uc5f4\ucc28 \ub0b4 \ud654\uc7a5\uc2e4, \uc138\uba74\ub300 \uc124\ube44\ub294 \uc788\uc73c\ub098 \uc2dd\ub2f9\ucc28\ub098 \uce74\ud398\uac1d\ucc28, \uc2dd\uc74c\ub8cc \uc790\ub3d9\ud310\ub9e4\uae30 \uc124\ube44\ub294 \ucc28\ub7c9\uc758 \uacf5\uac04\uc774 \uc881\uc544 \uc124\uce58\ud558\uc9c0 \uc54a\uc558\ub2e4.", "answer": "LCD\ubaa8\ub2c8\ud130", "sentence": "\uc2e4\ub0b4 \ub0c9\ub09c\ubc29 \ubc0f \uacf5\uae30\uc815\ud654\ub97c \uad00\ub9ac\ud558\ub294 HVAC\ub97c \ucc98\uc74c\uc73c\ub85c \ucc44\ud0dd\ud558\uc600\uace0 LCD\ubaa8\ub2c8\ud130 \ub97c \uc0ac\uc6a9\ud558\ub294 TV\ubc29\uc1a1\uae30\uae30\ub97c \uac16\ucd94\uace0 \uc788\uc73c\ub098 \ucf54\ub808\uc77c \ub9c8\ud06c\ub9cc \ub744\uc6cc\ub193\uace0 \uc0ac\uc6a9\ud558\uc9c0 \uc54a\ub294\ub2e4.", "paragraph_sentence": "\uc2e4\ub0b4\ub294 \ud55c\uad6d\ucca0\ub3c4\uacf5\uc0ac\uc5d0\uc11c \uc8fc\ubb38\ud558\uc5ec \ud788\ud0c0\uce58 \uc81c\uc791\uc18c\uc5d0\uc11c \uacf5\uae09\ud55c \uc2dc\ud2b8\ub97c \uc0ac\uc6a9\ud558\uace0 \uc788\uace0 \uc804\uccb4\uc801\uc778 \uc2dc\ud2b8 \ud3ed\uacfc \ud53c\uce58\ub294 \ubb34\uad81\ud654\ud638\ubcf4\ub2e4 \uc57d\uac04 \ub113\uc73c\uba70 \uc911\uac04 \ud314\uac78\uc774 \ubc0f \ud14c\uc774\ube14\uc744 \uac16\ucd94\uace0 \uc788\uc73c\uba70 \ud55c\uad6d\uc778\uc758 \uccb4\ud615\uc5d0 \ub9de\uac8c \uc124\uacc4\ub418\uc5c8\ub2e4. \uc2e4\ub0b4 \ub0c9\ub09c\ubc29 \ubc0f \uacf5\uae30\uc815\ud654\ub97c \uad00\ub9ac\ud558\ub294 HVAC\ub97c \ucc98\uc74c\uc73c\ub85c \ucc44\ud0dd\ud558\uc600\uace0 LCD\ubaa8\ub2c8\ud130 \ub97c \uc0ac\uc6a9\ud558\ub294 TV\ubc29\uc1a1\uae30\uae30\ub97c \uac16\ucd94\uace0 \uc788\uc73c\ub098 \ucf54\ub808\uc77c \ub9c8\ud06c\ub9cc \ub744\uc6cc\ub193\uace0 \uc0ac\uc6a9\ud558\uc9c0 \uc54a\ub294\ub2e4. \ud2b9\ud788 \uc120\ub450\ucc28 \uc6b4\uc804\uc2e4\uacfc \uac1d\uc2e4\uac04\uc758 \ubcbd\uc744 \uc804\uae30\uc801 \ud6a8\uacfc\uc5d0 \uc758\ud574 \ud22c\uba85\ud558\uac8c \ud560 \uc218 \uc788\ub294 \ubd88\ud22c\uba85 \uc720\ub9ac\ub97c \ucc44\ud0dd\ud558\uc600\uc73c\ub098, \uc6b4\uc804\uc2e4 \uacf5\uac1c\uc5d0 \uc775\uc219\uce58 \uc54a\uc740 \ud0d3\uc778\uc9c0 \ud3c9\uc18c\uc5d0\ub294 \ud750\ub9ac\uac8c \ud574 \ub450\uace0 \ub2e4\ub2cc\ub2e4. \uc5f4\ucc28 \ub0b4 \ud654\uc7a5\uc2e4, \uc138\uba74\ub300 \uc124\ube44\ub294 \uc788\uc73c\ub098 \uc2dd\ub2f9\ucc28\ub098 \uce74\ud398\uac1d\ucc28, \uc2dd\uc74c\ub8cc \uc790\ub3d9\ud310\ub9e4\uae30 \uc124\ube44\ub294 \ucc28\ub7c9\uc758 \uacf5\uac04\uc774 \uc881\uc544 \uc124\uce58\ud558\uc9c0 \uc54a\uc558\ub2e4.", "paragraph_answer": "\uc2e4\ub0b4\ub294 \ud55c\uad6d\ucca0\ub3c4\uacf5\uc0ac\uc5d0\uc11c \uc8fc\ubb38\ud558\uc5ec \ud788\ud0c0\uce58 \uc81c\uc791\uc18c\uc5d0\uc11c \uacf5\uae09\ud55c \uc2dc\ud2b8\ub97c \uc0ac\uc6a9\ud558\uace0 \uc788\uace0 \uc804\uccb4\uc801\uc778 \uc2dc\ud2b8 \ud3ed\uacfc \ud53c\uce58\ub294 \ubb34\uad81\ud654\ud638\ubcf4\ub2e4 \uc57d\uac04 \ub113\uc73c\uba70 \uc911\uac04 \ud314\uac78\uc774 \ubc0f \ud14c\uc774\ube14\uc744 \uac16\ucd94\uace0 \uc788\uc73c\uba70 \ud55c\uad6d\uc778\uc758 \uccb4\ud615\uc5d0 \ub9de\uac8c \uc124\uacc4\ub418\uc5c8\ub2e4. \uc2e4\ub0b4 \ub0c9\ub09c\ubc29 \ubc0f \uacf5\uae30\uc815\ud654\ub97c \uad00\ub9ac\ud558\ub294 HVAC\ub97c \ucc98\uc74c\uc73c\ub85c \ucc44\ud0dd\ud558\uc600\uace0 LCD\ubaa8\ub2c8\ud130 \ub97c \uc0ac\uc6a9\ud558\ub294 TV\ubc29\uc1a1\uae30\uae30\ub97c \uac16\ucd94\uace0 \uc788\uc73c\ub098 \ucf54\ub808\uc77c \ub9c8\ud06c\ub9cc \ub744\uc6cc\ub193\uace0 \uc0ac\uc6a9\ud558\uc9c0 \uc54a\ub294\ub2e4. \ud2b9\ud788 \uc120\ub450\ucc28 \uc6b4\uc804\uc2e4\uacfc \uac1d\uc2e4\uac04\uc758 \ubcbd\uc744 \uc804\uae30\uc801 \ud6a8\uacfc\uc5d0 \uc758\ud574 \ud22c\uba85\ud558\uac8c \ud560 \uc218 \uc788\ub294 \ubd88\ud22c\uba85 \uc720\ub9ac\ub97c \ucc44\ud0dd\ud558\uc600\uc73c\ub098, \uc6b4\uc804\uc2e4 \uacf5\uac1c\uc5d0 \uc775\uc219\uce58 \uc54a\uc740 \ud0d3\uc778\uc9c0 \ud3c9\uc18c\uc5d0\ub294 \ud750\ub9ac\uac8c \ud574 \ub450\uace0 \ub2e4\ub2cc\ub2e4. \uc5f4\ucc28 \ub0b4 \ud654\uc7a5\uc2e4, \uc138\uba74\ub300 \uc124\ube44\ub294 \uc788\uc73c\ub098 \uc2dd\ub2f9\ucc28\ub098 \uce74\ud398\uac1d\ucc28, \uc2dd\uc74c\ub8cc \uc790\ub3d9\ud310\ub9e4\uae30 \uc124\ube44\ub294 \ucc28\ub7c9\uc758 \uacf5\uac04\uc774 \uc881\uc544 \uc124\uce58\ud558\uc9c0 \uc54a\uc558\ub2e4.", "sentence_answer": "\uc2e4\ub0b4 \ub0c9\ub09c\ubc29 \ubc0f \uacf5\uae30\uc815\ud654\ub97c \uad00\ub9ac\ud558\ub294 HVAC\ub97c \ucc98\uc74c\uc73c\ub85c \ucc44\ud0dd\ud558\uc600\uace0 LCD\ubaa8\ub2c8\ud130 \ub97c \uc0ac\uc6a9\ud558\ub294 TV\ubc29\uc1a1\uae30\uae30\ub97c \uac16\ucd94\uace0 \uc788\uc73c\ub098 \ucf54\ub808\uc77c \ub9c8\ud06c\ub9cc \ub744\uc6cc\ub193\uace0 \uc0ac\uc6a9\ud558\uc9c0 \uc54a\ub294\ub2e4.", "paragraph_id": "6359840-2-1", "paragraph_question": "question: \ub300\uccb4\ub41c \uc804\uae30\ub3d9\ucc28 \ub0b4\ubd80\uc758 TV\ubc29\uc1a1\uae30\uae30\ub294 \uc5b4\ub5a4 \uc885\ub958\uc758 \ubaa8\ub2c8\ud130\ub97c \uc0ac\uc6a9\ud558\ub294\uac00?, context: \uc2e4\ub0b4\ub294 \ud55c\uad6d\ucca0\ub3c4\uacf5\uc0ac\uc5d0\uc11c \uc8fc\ubb38\ud558\uc5ec \ud788\ud0c0\uce58 \uc81c\uc791\uc18c\uc5d0\uc11c \uacf5\uae09\ud55c \uc2dc\ud2b8\ub97c \uc0ac\uc6a9\ud558\uace0 \uc788\uace0 \uc804\uccb4\uc801\uc778 \uc2dc\ud2b8 \ud3ed\uacfc \ud53c\uce58\ub294 \ubb34\uad81\ud654\ud638\ubcf4\ub2e4 \uc57d\uac04 \ub113\uc73c\uba70 \uc911\uac04 \ud314\uac78\uc774 \ubc0f \ud14c\uc774\ube14\uc744 \uac16\ucd94\uace0 \uc788\uc73c\uba70 \ud55c\uad6d\uc778\uc758 \uccb4\ud615\uc5d0 \ub9de\uac8c \uc124\uacc4\ub418\uc5c8\ub2e4. \uc2e4\ub0b4 \ub0c9\ub09c\ubc29 \ubc0f \uacf5\uae30\uc815\ud654\ub97c \uad00\ub9ac\ud558\ub294 HVAC\ub97c \ucc98\uc74c\uc73c\ub85c \ucc44\ud0dd\ud558\uc600\uace0 LCD\ubaa8\ub2c8\ud130\ub97c \uc0ac\uc6a9\ud558\ub294 TV\ubc29\uc1a1\uae30\uae30\ub97c \uac16\ucd94\uace0 \uc788\uc73c\ub098 \ucf54\ub808\uc77c \ub9c8\ud06c\ub9cc \ub744\uc6cc\ub193\uace0 \uc0ac\uc6a9\ud558\uc9c0 \uc54a\ub294\ub2e4. \ud2b9\ud788 \uc120\ub450\ucc28 \uc6b4\uc804\uc2e4\uacfc \uac1d\uc2e4\uac04\uc758 \ubcbd\uc744 \uc804\uae30\uc801 \ud6a8\uacfc\uc5d0 \uc758\ud574 \ud22c\uba85\ud558\uac8c \ud560 \uc218 \uc788\ub294 \ubd88\ud22c\uba85 \uc720\ub9ac\ub97c \ucc44\ud0dd\ud558\uc600\uc73c\ub098, \uc6b4\uc804\uc2e4 \uacf5\uac1c\uc5d0 \uc775\uc219\uce58 \uc54a\uc740 \ud0d3\uc778\uc9c0 \ud3c9\uc18c\uc5d0\ub294 \ud750\ub9ac\uac8c \ud574 \ub450\uace0 \ub2e4\ub2cc\ub2e4. \uc5f4\ucc28 \ub0b4 \ud654\uc7a5\uc2e4, \uc138\uba74\ub300 \uc124\ube44\ub294 \uc788\uc73c\ub098 \uc2dd\ub2f9\ucc28\ub098 \uce74\ud398\uac1d\ucc28, \uc2dd\uc74c\ub8cc \uc790\ub3d9\ud310\ub9e4\uae30 \uc124\ube44\ub294 \ucc28\ub7c9\uc758 \uacf5\uac04\uc774 \uc881\uc544 \uc124\uce58\ud558\uc9c0 \uc54a\uc558\ub2e4."} {"question": "\uc548\ucc3d\ud638\uac00 \uc785\uc6d0\ud55c \ubcd1\uc6d0\uc758 \uc774\ub984\uc740?", "paragraph": "\uc11d\ubc29\ub41c \ub4a4\uc5d0\ub3c4 \uadf8\uc758 \uac74\uac15\uc740 \ub098\uc544\uc9c0\uc9c0 \uc54a\uc558\ub2e4. \uce5c\uad6c\uc778 \uc724\uce58\ud638\uc640 \uadf8\ub97c \uc2a4\uc2b9\ucc98\ub7fc \ub530\ub974\ub358 \uc774\uad11\uc218\uac00 \uace0\uae09 \ud55c\uc57d\uc744 \ubcf4\ub0c8\uc9c0\ub9cc \uc5ec\ub7ec \ubcd1\uc758 \ud569\ubcd1\uc99d\uc73c\ub85c \uace0\uc0dd\ud558\uc5ec \ub0ab\uc9c0 \uc54a\uc558\ub2e4. \uacb0\uad6d \uadf8\ub294 \uacbd\uc131\uc81c\uad6d\ub300\ud559\uad50 \ub300\ud559\ubcd1\uc6d0\uc5d0 \uc785\uc6d0\ud558\uac8c \ub41c\ub2e4. \uadf8\uc758 \uc678\uc870\uce74 \uae40\uc21c\uc6d0\uacfc \uc870\uce74\ub538 \uc548\uc131\uacb0, \uadf8\uc758 \uc81c\uc790\uc778 \ubc15\uc815\ud638 \ub4f1 \uadf8\uc758 \uc785\uc6d0 \uc911 \uc2dc\uc911\uc744 \ub4e4\uc5c8\uc73c\uba70 \ub098\uc911\uc5d0\ub294 \uadf8\uc758 \ub3c5\ub9bd\uc6b4\ub3d9 \ub3d9\uc9c0\uc778 \uc774\uac11\uc758 \ub538 \uc774\uc815\ud76c\uac00 \uac70\ub4e4\uba74\uc11c \ubc88\uac08\uc544\uac00\uba74\uc11c \uc548\ucc3d\ud638\uc758 \uc2dc\uc911\uc744 \ub4e4\uba70 \ub05d\uae4c\uc9c0 \uc9c0\uc131\uaecf \ubcd1\uac04\ud638\ub97c \ud588\ub2e4. \uc724\uce58\ud638\uc640 \uc774\uad11\uc218, \uae40\uc131\uc218 \ub4f1\uc740 \ubcd1\uc6d0\ube44\ub97c \uc9c0\ubd88\ud588\uace0 \uc218\uc2dc\ub85c \uadf8\ub97c \ucc3e\uc544 \ubb38\uc548\ud588\ub2e4. 1938\ub144 \uadf8\ub294 \ud63c\uc218\uc0c1\ud0dc\uc5d0 \ube60\uc84c\ub294\ub370, \uc774\ub54c\uae4c\uc9c0\ub3c4 \uadf8\ub294 \ub098\ub77c\ub97c \uac71\uc815\ud558\uba70 \ubbfc\uc871\uc758 \uc2e4\ub825\uc744 \uae30\ub974\uace0 \ub3c5\ub9bd\uc744 \uc900\ube44\ud574\uc57c \ud568\uc744 \uac15\uc870\ud558\uc600\ub2e4.", "answer": "\uacbd\uc131\uc81c\uad6d\ub300\ud559\uad50 \ub300\ud559\ubcd1\uc6d0", "sentence": "\uacb0\uad6d \uadf8\ub294 \uacbd\uc131\uc81c\uad6d\ub300\ud559\uad50 \ub300\ud559\ubcd1\uc6d0 \uc5d0", "paragraph_sentence": "\uc11d\ubc29\ub41c \ub4a4\uc5d0\ub3c4 \uadf8\uc758 \uac74\uac15\uc740 \ub098\uc544\uc9c0\uc9c0 \uc54a\uc558\ub2e4. \uce5c\uad6c\uc778 \uc724\uce58\ud638\uc640 \uadf8\ub97c \uc2a4\uc2b9\ucc98\ub7fc \ub530\ub974\ub358 \uc774\uad11\uc218\uac00 \uace0\uae09 \ud55c\uc57d\uc744 \ubcf4\ub0c8\uc9c0\ub9cc \uc5ec\ub7ec \ubcd1\uc758 \ud569\ubcd1\uc99d\uc73c\ub85c \uace0\uc0dd\ud558\uc5ec \ub0ab\uc9c0 \uc54a\uc558\ub2e4. \uacb0\uad6d \uadf8\ub294 \uacbd\uc131\uc81c\uad6d\ub300\ud559\uad50 \ub300\ud559\ubcd1\uc6d0 \uc5d0 \uc785\uc6d0\ud558\uac8c \ub41c\ub2e4. \uadf8\uc758 \uc678\uc870\uce74 \uae40\uc21c\uc6d0\uacfc \uc870\uce74\ub538 \uc548\uc131\uacb0, \uadf8\uc758 \uc81c\uc790\uc778 \ubc15\uc815\ud638 \ub4f1 \uadf8\uc758 \uc785\uc6d0 \uc911 \uc2dc\uc911\uc744 \ub4e4\uc5c8\uc73c\uba70 \ub098\uc911\uc5d0\ub294 \uadf8\uc758 \ub3c5\ub9bd\uc6b4\ub3d9 \ub3d9\uc9c0\uc778 \uc774\uac11\uc758 \ub538 \uc774\uc815\ud76c\uac00 \uac70\ub4e4\uba74\uc11c \ubc88\uac08\uc544\uac00\uba74\uc11c \uc548\ucc3d\ud638\uc758 \uc2dc\uc911\uc744 \ub4e4\uba70 \ub05d\uae4c\uc9c0 \uc9c0\uc131\uaecf \ubcd1\uac04\ud638\ub97c \ud588\ub2e4. \uc724\uce58\ud638\uc640 \uc774\uad11\uc218, \uae40\uc131\uc218 \ub4f1\uc740 \ubcd1\uc6d0\ube44\ub97c \uc9c0\ubd88\ud588\uace0 \uc218\uc2dc\ub85c \uadf8\ub97c \ucc3e\uc544 \ubb38\uc548\ud588\ub2e4. 1938\ub144 \uadf8\ub294 \ud63c\uc218\uc0c1\ud0dc\uc5d0 \ube60\uc84c\ub294\ub370, \uc774\ub54c\uae4c\uc9c0\ub3c4 \uadf8\ub294 \ub098\ub77c\ub97c \uac71\uc815\ud558\uba70 \ubbfc\uc871\uc758 \uc2e4\ub825\uc744 \uae30\ub974\uace0 \ub3c5\ub9bd\uc744 \uc900\ube44\ud574\uc57c \ud568\uc744 \uac15\uc870\ud558\uc600\ub2e4.", "paragraph_answer": "\uc11d\ubc29\ub41c \ub4a4\uc5d0\ub3c4 \uadf8\uc758 \uac74\uac15\uc740 \ub098\uc544\uc9c0\uc9c0 \uc54a\uc558\ub2e4. \uce5c\uad6c\uc778 \uc724\uce58\ud638\uc640 \uadf8\ub97c \uc2a4\uc2b9\ucc98\ub7fc \ub530\ub974\ub358 \uc774\uad11\uc218\uac00 \uace0\uae09 \ud55c\uc57d\uc744 \ubcf4\ub0c8\uc9c0\ub9cc \uc5ec\ub7ec \ubcd1\uc758 \ud569\ubcd1\uc99d\uc73c\ub85c \uace0\uc0dd\ud558\uc5ec \ub0ab\uc9c0 \uc54a\uc558\ub2e4. \uacb0\uad6d \uadf8\ub294 \uacbd\uc131\uc81c\uad6d\ub300\ud559\uad50 \ub300\ud559\ubcd1\uc6d0 \uc5d0 \uc785\uc6d0\ud558\uac8c \ub41c\ub2e4. \uadf8\uc758 \uc678\uc870\uce74 \uae40\uc21c\uc6d0\uacfc \uc870\uce74\ub538 \uc548\uc131\uacb0, \uadf8\uc758 \uc81c\uc790\uc778 \ubc15\uc815\ud638 \ub4f1 \uadf8\uc758 \uc785\uc6d0 \uc911 \uc2dc\uc911\uc744 \ub4e4\uc5c8\uc73c\uba70 \ub098\uc911\uc5d0\ub294 \uadf8\uc758 \ub3c5\ub9bd\uc6b4\ub3d9 \ub3d9\uc9c0\uc778 \uc774\uac11\uc758 \ub538 \uc774\uc815\ud76c\uac00 \uac70\ub4e4\uba74\uc11c \ubc88\uac08\uc544\uac00\uba74\uc11c \uc548\ucc3d\ud638\uc758 \uc2dc\uc911\uc744 \ub4e4\uba70 \ub05d\uae4c\uc9c0 \uc9c0\uc131\uaecf \ubcd1\uac04\ud638\ub97c \ud588\ub2e4. \uc724\uce58\ud638\uc640 \uc774\uad11\uc218, \uae40\uc131\uc218 \ub4f1\uc740 \ubcd1\uc6d0\ube44\ub97c \uc9c0\ubd88\ud588\uace0 \uc218\uc2dc\ub85c \uadf8\ub97c \ucc3e\uc544 \ubb38\uc548\ud588\ub2e4. 1938\ub144 \uadf8\ub294 \ud63c\uc218\uc0c1\ud0dc\uc5d0 \ube60\uc84c\ub294\ub370, \uc774\ub54c\uae4c\uc9c0\ub3c4 \uadf8\ub294 \ub098\ub77c\ub97c \uac71\uc815\ud558\uba70 \ubbfc\uc871\uc758 \uc2e4\ub825\uc744 \uae30\ub974\uace0 \ub3c5\ub9bd\uc744 \uc900\ube44\ud574\uc57c \ud568\uc744 \uac15\uc870\ud558\uc600\ub2e4.", "sentence_answer": "\uacb0\uad6d \uadf8\ub294 \uacbd\uc131\uc81c\uad6d\ub300\ud559\uad50 \ub300\ud559\ubcd1\uc6d0 \uc5d0", "paragraph_id": "6538990-14-0", "paragraph_question": "question: \uc548\ucc3d\ud638\uac00 \uc785\uc6d0\ud55c \ubcd1\uc6d0\uc758 \uc774\ub984\uc740?, context: \uc11d\ubc29\ub41c \ub4a4\uc5d0\ub3c4 \uadf8\uc758 \uac74\uac15\uc740 \ub098\uc544\uc9c0\uc9c0 \uc54a\uc558\ub2e4. \uce5c\uad6c\uc778 \uc724\uce58\ud638\uc640 \uadf8\ub97c \uc2a4\uc2b9\ucc98\ub7fc \ub530\ub974\ub358 \uc774\uad11\uc218\uac00 \uace0\uae09 \ud55c\uc57d\uc744 \ubcf4\ub0c8\uc9c0\ub9cc \uc5ec\ub7ec \ubcd1\uc758 \ud569\ubcd1\uc99d\uc73c\ub85c \uace0\uc0dd\ud558\uc5ec \ub0ab\uc9c0 \uc54a\uc558\ub2e4. \uacb0\uad6d \uadf8\ub294 \uacbd\uc131\uc81c\uad6d\ub300\ud559\uad50 \ub300\ud559\ubcd1\uc6d0\uc5d0 \uc785\uc6d0\ud558\uac8c \ub41c\ub2e4. \uadf8\uc758 \uc678\uc870\uce74 \uae40\uc21c\uc6d0\uacfc \uc870\uce74\ub538 \uc548\uc131\uacb0, \uadf8\uc758 \uc81c\uc790\uc778 \ubc15\uc815\ud638 \ub4f1 \uadf8\uc758 \uc785\uc6d0 \uc911 \uc2dc\uc911\uc744 \ub4e4\uc5c8\uc73c\uba70 \ub098\uc911\uc5d0\ub294 \uadf8\uc758 \ub3c5\ub9bd\uc6b4\ub3d9 \ub3d9\uc9c0\uc778 \uc774\uac11\uc758 \ub538 \uc774\uc815\ud76c\uac00 \uac70\ub4e4\uba74\uc11c \ubc88\uac08\uc544\uac00\uba74\uc11c \uc548\ucc3d\ud638\uc758 \uc2dc\uc911\uc744 \ub4e4\uba70 \ub05d\uae4c\uc9c0 \uc9c0\uc131\uaecf \ubcd1\uac04\ud638\ub97c \ud588\ub2e4. \uc724\uce58\ud638\uc640 \uc774\uad11\uc218, \uae40\uc131\uc218 \ub4f1\uc740 \ubcd1\uc6d0\ube44\ub97c \uc9c0\ubd88\ud588\uace0 \uc218\uc2dc\ub85c \uadf8\ub97c \ucc3e\uc544 \ubb38\uc548\ud588\ub2e4. 1938\ub144 \uadf8\ub294 \ud63c\uc218\uc0c1\ud0dc\uc5d0 \ube60\uc84c\ub294\ub370, \uc774\ub54c\uae4c\uc9c0\ub3c4 \uadf8\ub294 \ub098\ub77c\ub97c \uac71\uc815\ud558\uba70 \ubbfc\uc871\uc758 \uc2e4\ub825\uc744 \uae30\ub974\uace0 \ub3c5\ub9bd\uc744 \uc900\ube44\ud574\uc57c \ud568\uc744 \uac15\uc870\ud558\uc600\ub2e4."} {"question": "\ubcf4\uc815\ub300\uc2e0\ub4e4\uc774 \uc5b4\ub9b0 \ud669\uc81c\uac00 \ud6d7\ub0a0 \ud658\uad00\ub4e4\uc5d0\uac8c \ub18d\ub77d\ub420\uae4c \ubd10 \ud3d0\uc9c0\ud55c \uac83\uc740?", "paragraph": "\ubcf4\uc815\ub300\uc2e0\ub4e4\uc740 \uc5b4\ub9b0 \ud669\uc81c\uac00 \ud6d7\ub0a0 \ud658\uad00\ub4e4\uc5d0\uac8c \ub18d\ub77d\ub420\uae4c \ubd10 \uc21c\uce58\uc81c \ub54c \uc124\uce58\ub41c \uba85\ub098\ub77c\uc758 \ub3d9\ucc3d(\u6771\u5ee0, \ud658\uad00\uc758 \uc218\ub1cc\ubd80\uc774\uba70 \ud669\uc81c \uc9c1\uc18d \uc815\ubcf4\uae30\uad00)\uacfc \ube44\uc2b7\ud55c \uae30\uad6c\uc778 \uc2ed\uc0bc\uc544\ubb38(\u5341\u4e09\u8859\u9580)\uc744 \ud3d0\uc9c0\ud558\uc5ec \ud658\uad00\ub4e4\uc744 \uc815\ubb34\uc5d0\uc11c \ucd95\ucd9c\ud558\uace0 \uc6d0\ub798 \uc21c\uce58\uc81c \ub54c \ud3d0\uc9c0\ub41c \ub0b4\ubb34\ubd80\ub97c \ub2e4\uc2dc \uc124\uce58\ud558\uc5ec \ud669\uc81c\uc5d0\uac8c \ucda9\uc131\uc2a4\ub7ec\uc6b4 \ub9cc\uc8fc\uc871 \ucda9\ubcf5\ub4e4\ub85c \ud558\uc5ec\uae08 \ud658\uad00\ub4e4\uc744 \ub300\uc2e0\ud558\uac8c \ud558\uc600\ub2e4. \ubcf4\uc815\ub300\uc2e0\uc740 \ubaa8\ub450 \uaf64 \uc0c1\ub2f9\ud55c \uad8c\ub825\uc744 \ub204\ub838\uc73c\ub098, \uadf8\uc911\uc5d0\uc11c\ub3c4 \ubcd1\ubd80\uc0c1\uc11c \uc624\ubc30\uac00 \uc81c\uc77c \uad8c\ub825\uc774 \ub9c9\uac15\ud558\uc600\ub2e4. \uc624\ubc30\ub294 \ubc31\uc131\ub4e4\uc758 \ub545\uc744 \ubd88\ubc95\uc73c\ub85c \ud68d\ucc45\ud558\ub294 \ub4f1 \uac16\uc740 \uc804\ud6a1\uc744 \uc77c\uc0bc\uc558\ub2e4. \ud558\uc9c0\ub9cc \ubcd1\uad8c\uc744 \ud2c0\uc5b4\uc950\uace0 \uc788\uc5b4\uc11c \uc624\ubc30\uc5d0 \ube44\ud574 \uc138\ub825\uc774 \ubbf8\uc57d\ud55c \ub300\uc2e0\ub4e4\uc774\ub098 \uc544\uc9c1 \uce5c\uc815\uc744 \uc2dc\uc791\ud558\uc9c0 \ubabb\ud55c \uac15\ud76c\uc81c\ub294 \uc5b4\ucc0c\ud560 \ub3c4\ub9ac\uac00 \uc5c6\uc5c8\ub2e4. \uc77c\ub2e8 \uac15\ud76c\uc81c\ub294 \uc624\ubc30\uc758 \ub208 \ubc16\uc5d0 \ub098\uc9c0 \uc54a\uc73c\ub824\uace0 \uc77c\ubd80\ub7ec \uc7a5\uc815\ub4e4\uc744 \ubd88\ub7ec \ubabd\uace8 \uc528\ub984\uc744 \ud558\ub3c4\ub85d \ud558\uc600\ub2e4. \ub610\ud55c, \ud2c8\ud2c8\uc774 \uc81c\uc655\ud559 \uc218\uc5c5\uc744 \uc78a\uc9c0 \uc54a\uace0 \ubc30\uc6e0\ub2e4. \uac15\ud76c\uc81c\uc758 \uadf8 \uc2a4\uc2b9\uc774 \ubc14\ub85c \uba85\ub098\ub77c\uc758 \ub9c8\uc9c0\ub9c9 \uc9c4\uc0ac\uc2dc(\u9032\u58eb\u8a66)\uc5d0\uc11c \uc7a5\uc6d0\uc744 \ud55c \uc81c\uc138(\u6fdf\u4e16)\uc600\ub2e4.", "answer": "\uc2ed\uc0bc\uc544\ubb38", "sentence": "\uc5b4\ub9b0 \ud669\uc81c\uac00 \ud6d7\ub0a0 \ud658\uad00\ub4e4\uc5d0\uac8c \ub18d\ub77d\ub420\uae4c \ubd10 \uc21c\uce58\uc81c \ub54c \uc124\uce58\ub41c \uba85\ub098\ub77c\uc758 \ub3d9\ucc3d(\u6771\u5ee0, \ud658\uad00\uc758 \uc218\ub1cc\ubd80\uc774\uba70 \ud669\uc81c \uc9c1\uc18d \uc815\ubcf4\uae30\uad00)\uacfc \ube44\uc2b7\ud55c \uae30\uad6c\uc778 \uc2ed\uc0bc\uc544\ubb38 (\u5341\u4e09\u8859\u9580)", "paragraph_sentence": "\ubcf4\uc815\ub300\uc2e0\ub4e4\uc740 \uc5b4\ub9b0 \ud669\uc81c\uac00 \ud6d7\ub0a0 \ud658\uad00\ub4e4\uc5d0\uac8c \ub18d\ub77d\ub420\uae4c \ubd10 \uc21c\uce58\uc81c \ub54c \uc124\uce58\ub41c \uba85\ub098\ub77c\uc758 \ub3d9\ucc3d(\u6771\u5ee0, \ud658\uad00\uc758 \uc218\ub1cc\ubd80\uc774\uba70 \ud669\uc81c \uc9c1\uc18d \uc815\ubcf4\uae30\uad00)\uacfc \ube44\uc2b7\ud55c \uae30\uad6c\uc778 \uc2ed\uc0bc\uc544\ubb38 (\u5341\u4e09\u8859\u9580) \uc744 \ud3d0\uc9c0\ud558\uc5ec \ud658\uad00\ub4e4\uc744 \uc815\ubb34\uc5d0\uc11c \ucd95\ucd9c\ud558\uace0 \uc6d0\ub798 \uc21c\uce58\uc81c \ub54c \ud3d0\uc9c0\ub41c \ub0b4\ubb34\ubd80\ub97c \ub2e4\uc2dc \uc124\uce58\ud558\uc5ec \ud669\uc81c\uc5d0\uac8c \ucda9\uc131\uc2a4\ub7ec\uc6b4 \ub9cc\uc8fc\uc871 \ucda9\ubcf5\ub4e4\ub85c \ud558\uc5ec\uae08 \ud658\uad00\ub4e4\uc744 \ub300\uc2e0\ud558\uac8c \ud558\uc600\ub2e4. \ubcf4\uc815\ub300\uc2e0\uc740 \ubaa8\ub450 \uaf64 \uc0c1\ub2f9\ud55c \uad8c\ub825\uc744 \ub204\ub838\uc73c\ub098, \uadf8\uc911\uc5d0\uc11c\ub3c4 \ubcd1\ubd80\uc0c1\uc11c \uc624\ubc30\uac00 \uc81c\uc77c \uad8c\ub825\uc774 \ub9c9\uac15\ud558\uc600\ub2e4. \uc624\ubc30\ub294 \ubc31\uc131\ub4e4\uc758 \ub545\uc744 \ubd88\ubc95\uc73c\ub85c \ud68d\ucc45\ud558\ub294 \ub4f1 \uac16\uc740 \uc804\ud6a1\uc744 \uc77c\uc0bc\uc558\ub2e4. \ud558\uc9c0\ub9cc \ubcd1\uad8c\uc744 \ud2c0\uc5b4\uc950\uace0 \uc788\uc5b4\uc11c \uc624\ubc30\uc5d0 \ube44\ud574 \uc138\ub825\uc774 \ubbf8\uc57d\ud55c \ub300\uc2e0\ub4e4\uc774\ub098 \uc544\uc9c1 \uce5c\uc815\uc744 \uc2dc\uc791\ud558\uc9c0 \ubabb\ud55c \uac15\ud76c\uc81c\ub294 \uc5b4\ucc0c\ud560 \ub3c4\ub9ac\uac00 \uc5c6\uc5c8\ub2e4. \uc77c\ub2e8 \uac15\ud76c\uc81c\ub294 \uc624\ubc30\uc758 \ub208 \ubc16\uc5d0 \ub098\uc9c0 \uc54a\uc73c\ub824\uace0 \uc77c\ubd80\ub7ec \uc7a5\uc815\ub4e4\uc744 \ubd88\ub7ec \ubabd\uace8 \uc528\ub984\uc744 \ud558\ub3c4\ub85d \ud558\uc600\ub2e4. \ub610\ud55c, \ud2c8\ud2c8\uc774 \uc81c\uc655\ud559 \uc218\uc5c5\uc744 \uc78a\uc9c0 \uc54a\uace0 \ubc30\uc6e0\ub2e4. \uac15\ud76c\uc81c\uc758 \uadf8 \uc2a4\uc2b9\uc774 \ubc14\ub85c \uba85\ub098\ub77c\uc758 \ub9c8\uc9c0\ub9c9 \uc9c4\uc0ac\uc2dc(\u9032\u58eb\u8a66)\uc5d0\uc11c \uc7a5\uc6d0\uc744 \ud55c \uc81c\uc138(\u6fdf\u4e16)\uc600\ub2e4.", "paragraph_answer": "\ubcf4\uc815\ub300\uc2e0\ub4e4\uc740 \uc5b4\ub9b0 \ud669\uc81c\uac00 \ud6d7\ub0a0 \ud658\uad00\ub4e4\uc5d0\uac8c \ub18d\ub77d\ub420\uae4c \ubd10 \uc21c\uce58\uc81c \ub54c \uc124\uce58\ub41c \uba85\ub098\ub77c\uc758 \ub3d9\ucc3d(\u6771\u5ee0, \ud658\uad00\uc758 \uc218\ub1cc\ubd80\uc774\uba70 \ud669\uc81c \uc9c1\uc18d \uc815\ubcf4\uae30\uad00)\uacfc \ube44\uc2b7\ud55c \uae30\uad6c\uc778 \uc2ed\uc0bc\uc544\ubb38 (\u5341\u4e09\u8859\u9580)\uc744 \ud3d0\uc9c0\ud558\uc5ec \ud658\uad00\ub4e4\uc744 \uc815\ubb34\uc5d0\uc11c \ucd95\ucd9c\ud558\uace0 \uc6d0\ub798 \uc21c\uce58\uc81c \ub54c \ud3d0\uc9c0\ub41c \ub0b4\ubb34\ubd80\ub97c \ub2e4\uc2dc \uc124\uce58\ud558\uc5ec \ud669\uc81c\uc5d0\uac8c \ucda9\uc131\uc2a4\ub7ec\uc6b4 \ub9cc\uc8fc\uc871 \ucda9\ubcf5\ub4e4\ub85c \ud558\uc5ec\uae08 \ud658\uad00\ub4e4\uc744 \ub300\uc2e0\ud558\uac8c \ud558\uc600\ub2e4. \ubcf4\uc815\ub300\uc2e0\uc740 \ubaa8\ub450 \uaf64 \uc0c1\ub2f9\ud55c \uad8c\ub825\uc744 \ub204\ub838\uc73c\ub098, \uadf8\uc911\uc5d0\uc11c\ub3c4 \ubcd1\ubd80\uc0c1\uc11c \uc624\ubc30\uac00 \uc81c\uc77c \uad8c\ub825\uc774 \ub9c9\uac15\ud558\uc600\ub2e4. \uc624\ubc30\ub294 \ubc31\uc131\ub4e4\uc758 \ub545\uc744 \ubd88\ubc95\uc73c\ub85c \ud68d\ucc45\ud558\ub294 \ub4f1 \uac16\uc740 \uc804\ud6a1\uc744 \uc77c\uc0bc\uc558\ub2e4. \ud558\uc9c0\ub9cc \ubcd1\uad8c\uc744 \ud2c0\uc5b4\uc950\uace0 \uc788\uc5b4\uc11c \uc624\ubc30\uc5d0 \ube44\ud574 \uc138\ub825\uc774 \ubbf8\uc57d\ud55c \ub300\uc2e0\ub4e4\uc774\ub098 \uc544\uc9c1 \uce5c\uc815\uc744 \uc2dc\uc791\ud558\uc9c0 \ubabb\ud55c \uac15\ud76c\uc81c\ub294 \uc5b4\ucc0c\ud560 \ub3c4\ub9ac\uac00 \uc5c6\uc5c8\ub2e4. \uc77c\ub2e8 \uac15\ud76c\uc81c\ub294 \uc624\ubc30\uc758 \ub208 \ubc16\uc5d0 \ub098\uc9c0 \uc54a\uc73c\ub824\uace0 \uc77c\ubd80\ub7ec \uc7a5\uc815\ub4e4\uc744 \ubd88\ub7ec \ubabd\uace8 \uc528\ub984\uc744 \ud558\ub3c4\ub85d \ud558\uc600\ub2e4. \ub610\ud55c, \ud2c8\ud2c8\uc774 \uc81c\uc655\ud559 \uc218\uc5c5\uc744 \uc78a\uc9c0 \uc54a\uace0 \ubc30\uc6e0\ub2e4. \uac15\ud76c\uc81c\uc758 \uadf8 \uc2a4\uc2b9\uc774 \ubc14\ub85c \uba85\ub098\ub77c\uc758 \ub9c8\uc9c0\ub9c9 \uc9c4\uc0ac\uc2dc(\u9032\u58eb\u8a66)\uc5d0\uc11c \uc7a5\uc6d0\uc744 \ud55c \uc81c\uc138(\u6fdf\u4e16)\uc600\ub2e4.", "sentence_answer": "\uc5b4\ub9b0 \ud669\uc81c\uac00 \ud6d7\ub0a0 \ud658\uad00\ub4e4\uc5d0\uac8c \ub18d\ub77d\ub420\uae4c \ubd10 \uc21c\uce58\uc81c \ub54c \uc124\uce58\ub41c \uba85\ub098\ub77c\uc758 \ub3d9\ucc3d(\u6771\u5ee0, \ud658\uad00\uc758 \uc218\ub1cc\ubd80\uc774\uba70 \ud669\uc81c \uc9c1\uc18d \uc815\ubcf4\uae30\uad00)\uacfc \ube44\uc2b7\ud55c \uae30\uad6c\uc778 \uc2ed\uc0bc\uc544\ubb38 (\u5341\u4e09\u8859\u9580)", "paragraph_id": "6478740-7-0", "paragraph_question": "question: \ubcf4\uc815\ub300\uc2e0\ub4e4\uc774 \uc5b4\ub9b0 \ud669\uc81c\uac00 \ud6d7\ub0a0 \ud658\uad00\ub4e4\uc5d0\uac8c \ub18d\ub77d\ub420\uae4c \ubd10 \ud3d0\uc9c0\ud55c \uac83\uc740?, context: \ubcf4\uc815\ub300\uc2e0\ub4e4\uc740 \uc5b4\ub9b0 \ud669\uc81c\uac00 \ud6d7\ub0a0 \ud658\uad00\ub4e4\uc5d0\uac8c \ub18d\ub77d\ub420\uae4c \ubd10 \uc21c\uce58\uc81c \ub54c \uc124\uce58\ub41c \uba85\ub098\ub77c\uc758 \ub3d9\ucc3d(\u6771\u5ee0, \ud658\uad00\uc758 \uc218\ub1cc\ubd80\uc774\uba70 \ud669\uc81c \uc9c1\uc18d \uc815\ubcf4\uae30\uad00)\uacfc \ube44\uc2b7\ud55c \uae30\uad6c\uc778 \uc2ed\uc0bc\uc544\ubb38(\u5341\u4e09\u8859\u9580)\uc744 \ud3d0\uc9c0\ud558\uc5ec \ud658\uad00\ub4e4\uc744 \uc815\ubb34\uc5d0\uc11c \ucd95\ucd9c\ud558\uace0 \uc6d0\ub798 \uc21c\uce58\uc81c \ub54c \ud3d0\uc9c0\ub41c \ub0b4\ubb34\ubd80\ub97c \ub2e4\uc2dc \uc124\uce58\ud558\uc5ec \ud669\uc81c\uc5d0\uac8c \ucda9\uc131\uc2a4\ub7ec\uc6b4 \ub9cc\uc8fc\uc871 \ucda9\ubcf5\ub4e4\ub85c \ud558\uc5ec\uae08 \ud658\uad00\ub4e4\uc744 \ub300\uc2e0\ud558\uac8c \ud558\uc600\ub2e4. \ubcf4\uc815\ub300\uc2e0\uc740 \ubaa8\ub450 \uaf64 \uc0c1\ub2f9\ud55c \uad8c\ub825\uc744 \ub204\ub838\uc73c\ub098, \uadf8\uc911\uc5d0\uc11c\ub3c4 \ubcd1\ubd80\uc0c1\uc11c \uc624\ubc30\uac00 \uc81c\uc77c \uad8c\ub825\uc774 \ub9c9\uac15\ud558\uc600\ub2e4. \uc624\ubc30\ub294 \ubc31\uc131\ub4e4\uc758 \ub545\uc744 \ubd88\ubc95\uc73c\ub85c \ud68d\ucc45\ud558\ub294 \ub4f1 \uac16\uc740 \uc804\ud6a1\uc744 \uc77c\uc0bc\uc558\ub2e4. \ud558\uc9c0\ub9cc \ubcd1\uad8c\uc744 \ud2c0\uc5b4\uc950\uace0 \uc788\uc5b4\uc11c \uc624\ubc30\uc5d0 \ube44\ud574 \uc138\ub825\uc774 \ubbf8\uc57d\ud55c \ub300\uc2e0\ub4e4\uc774\ub098 \uc544\uc9c1 \uce5c\uc815\uc744 \uc2dc\uc791\ud558\uc9c0 \ubabb\ud55c \uac15\ud76c\uc81c\ub294 \uc5b4\ucc0c\ud560 \ub3c4\ub9ac\uac00 \uc5c6\uc5c8\ub2e4. \uc77c\ub2e8 \uac15\ud76c\uc81c\ub294 \uc624\ubc30\uc758 \ub208 \ubc16\uc5d0 \ub098\uc9c0 \uc54a\uc73c\ub824\uace0 \uc77c\ubd80\ub7ec \uc7a5\uc815\ub4e4\uc744 \ubd88\ub7ec \ubabd\uace8 \uc528\ub984\uc744 \ud558\ub3c4\ub85d \ud558\uc600\ub2e4. \ub610\ud55c, \ud2c8\ud2c8\uc774 \uc81c\uc655\ud559 \uc218\uc5c5\uc744 \uc78a\uc9c0 \uc54a\uace0 \ubc30\uc6e0\ub2e4. \uac15\ud76c\uc81c\uc758 \uadf8 \uc2a4\uc2b9\uc774 \ubc14\ub85c \uba85\ub098\ub77c\uc758 \ub9c8\uc9c0\ub9c9 \uc9c4\uc0ac\uc2dc(\u9032\u58eb\u8a66)\uc5d0\uc11c \uc7a5\uc6d0\uc744 \ud55c \uc81c\uc138(\u6fdf\u4e16)\uc600\ub2e4."} {"question": "2017\ub144 7\uc6d4 11\uc77c \uc0ac\ub4dc \uc694\uaca9\uc2e4\ud5d8 \ub2f9\uc2dc \uba85\uc911\ub960\uc740?", "paragraph": "2017\ub144 7\uc6d4 11\uc77c, \ubbf8\uad6d \uad6d\ubc29\ubd80\ub294 \ud558\uc640\uc774\uc5d0\uc11c \ubbf8\uad6d \ubcf8\ud1a0\ub97c \ud5a5\ud574 \ubc1c\uc0ac\ub41c \uc911\uac70\ub9ac\ubbf8\uc0ac\uc77c\uc744 \uc54c\ub798\uc2a4\uce74 \ucf54\ub514\uc545 \uae30\uc9c0\uc5d0\uc11c \uc0ac\ub4dc\ub85c \ub9de\ucd94\ub294 \uc694\uaca9\uc2e4\ud5d8\uc5d0 14\ucc28\ub840 \uc131\uacf5\ud588\ub2e4. \uc911\uac70\ub9ac\ud0c4\ub3c4\ubbf8\uc0ac\uc77c, IRBM\uc758 \uc18d\ub3c4\ub85c \ub0a0\uc544\uac00\ub294 \ube44\ud589\uccb4\ub97c \uba85\uc911\uc2dc\ud0a8 \uac83\uc774\ub2e4. \ubbf8\uad6d \ubbf8\uc0ac\uc77c\ubc29\uc5b4\uad6d\uc740 \uc131\uba85\uc744 \ud1b5\ud574 \"\uc774\ubc88 \uc2dc\ud5d8\uc740 \ud0c4\ub3c4\ubbf8\uc0ac\uc77c\uc744 \uc694\uaca9\ud558\uace0 \ud30c\uad34\ud558\ub294 \uc0ac\ub4dc \ubb34\uae30\uccb4\uacc4\uc758 \ub2a5\ub825\uc744 \ub354\uc6b1 \uc99d\uba85\ud588\ub2e4\"\uace0 \ud3c9\uac00\ud588\ub2e4. \uc0ac\ub4dc\ub294 14\ucc28\ub840\uc758 \uc694\uaca9\uc2dc\ud5d8\uc5d0\uc11c \ubaa8\ub450 \uc131\uacf5\uc744 \uac70\ub46c 100%\uc758 \uba85\uc911\ub960\uc744 \ubcf4\uc774\uace0 \uc788\ub2e4. \uc0ac\ub4dc \uc2dc\ud5d8 \ubc1c\uc0ac\ub294 \ube44\ud589\uccb4\uc758 \ubc1c\uc0ac \uc2dc\uac04\uc744 \ud2b9\uc815\ud558\uc9c0 \uc54a\ub294 \ub4f1 \uc2e4\uc804 \uc0c1\ud669\uacfc \uac19\uc740 \uc870\uac74\uc5d0\uc11c \uc774\ub904\uc84c\ub2e4. \ud558\uc640\uc774 \ubbf8\uc0ac\uc77c \ubc1c\uc0ac \uae30\uc9c0\ub294 \uc5b8\uc81c \uc5b4\ub514\uc11c \ubbf8\uc0ac\uc77c\uc744 \ubc1c\uc0ac\ud560\uc9c0 \uc0ac\uc804\uc5d0 \uc54c\ub9ac\uc9c0 \uc54a\uc558\uc73c\uba70 \uae30\uc2b5\uc801\uc73c\ub85c \uc54c\ub798\uc2a4\uce74\ub97c \ud5a5\ud574 \ubbf8\uc0ac\uc77c\uc744 \ubc1c\uc0ac\ud588\uace0 \uc54c\ub798\uc2a4\uce74 \ucf54\ub514\uc545 \uae30\uc9c0\ub294 \uc2e4\uc804 \uc0c1\ud669\uacfc \uac19\uc740 \uc870\uac74\uc5d0\uc11c \ubbf8\uc0ac\uc77c \uc694\uaca9\uc5d0 \uc131\uacf5\ud588\ub2e4.", "answer": "100%", "sentence": "\uc0ac\ub4dc\ub294 14\ucc28\ub840\uc758 \uc694\uaca9\uc2dc\ud5d8\uc5d0\uc11c \ubaa8\ub450 \uc131\uacf5\uc744 \uac70\ub46c 100% \uc758 \uba85\uc911\ub960\uc744 \ubcf4\uc774\uace0 \uc788\ub2e4.", "paragraph_sentence": "2017\ub144 7\uc6d4 11\uc77c, \ubbf8\uad6d \uad6d\ubc29\ubd80\ub294 \ud558\uc640\uc774\uc5d0\uc11c \ubbf8\uad6d \ubcf8\ud1a0\ub97c \ud5a5\ud574 \ubc1c\uc0ac\ub41c \uc911\uac70\ub9ac\ubbf8\uc0ac\uc77c\uc744 \uc54c\ub798\uc2a4\uce74 \ucf54\ub514\uc545 \uae30\uc9c0\uc5d0\uc11c \uc0ac\ub4dc\ub85c \ub9de\ucd94\ub294 \uc694\uaca9\uc2e4\ud5d8\uc5d0 14\ucc28\ub840 \uc131\uacf5\ud588\ub2e4. \uc911\uac70\ub9ac\ud0c4\ub3c4\ubbf8\uc0ac\uc77c, IRBM\uc758 \uc18d\ub3c4\ub85c \ub0a0\uc544\uac00\ub294 \ube44\ud589\uccb4\ub97c \uba85\uc911\uc2dc\ud0a8 \uac83\uc774\ub2e4. \ubbf8\uad6d \ubbf8\uc0ac\uc77c\ubc29\uc5b4\uad6d\uc740 \uc131\uba85\uc744 \ud1b5\ud574 \"\uc774\ubc88 \uc2dc\ud5d8\uc740 \ud0c4\ub3c4\ubbf8\uc0ac\uc77c\uc744 \uc694\uaca9\ud558\uace0 \ud30c\uad34\ud558\ub294 \uc0ac\ub4dc \ubb34\uae30\uccb4\uacc4\uc758 \ub2a5\ub825\uc744 \ub354\uc6b1 \uc99d\uba85\ud588\ub2e4\"\uace0 \ud3c9\uac00\ud588\ub2e4. \uc0ac\ub4dc\ub294 14\ucc28\ub840\uc758 \uc694\uaca9\uc2dc\ud5d8\uc5d0\uc11c \ubaa8\ub450 \uc131\uacf5\uc744 \uac70\ub46c 100% \uc758 \uba85\uc911\ub960\uc744 \ubcf4\uc774\uace0 \uc788\ub2e4. \uc0ac\ub4dc \uc2dc\ud5d8 \ubc1c\uc0ac\ub294 \ube44\ud589\uccb4\uc758 \ubc1c\uc0ac \uc2dc\uac04\uc744 \ud2b9\uc815\ud558\uc9c0 \uc54a\ub294 \ub4f1 \uc2e4\uc804 \uc0c1\ud669\uacfc \uac19\uc740 \uc870\uac74\uc5d0\uc11c \uc774\ub904\uc84c\ub2e4. \ud558\uc640\uc774 \ubbf8\uc0ac\uc77c \ubc1c\uc0ac \uae30\uc9c0\ub294 \uc5b8\uc81c \uc5b4\ub514\uc11c \ubbf8\uc0ac\uc77c\uc744 \ubc1c\uc0ac\ud560\uc9c0 \uc0ac\uc804\uc5d0 \uc54c\ub9ac\uc9c0 \uc54a\uc558\uc73c\uba70 \uae30\uc2b5\uc801\uc73c\ub85c \uc54c\ub798\uc2a4\uce74\ub97c \ud5a5\ud574 \ubbf8\uc0ac\uc77c\uc744 \ubc1c\uc0ac\ud588\uace0 \uc54c\ub798\uc2a4\uce74 \ucf54\ub514\uc545 \uae30\uc9c0\ub294 \uc2e4\uc804 \uc0c1\ud669\uacfc \uac19\uc740 \uc870\uac74\uc5d0\uc11c \ubbf8\uc0ac\uc77c \uc694\uaca9\uc5d0 \uc131\uacf5\ud588\ub2e4.", "paragraph_answer": "2017\ub144 7\uc6d4 11\uc77c, \ubbf8\uad6d \uad6d\ubc29\ubd80\ub294 \ud558\uc640\uc774\uc5d0\uc11c \ubbf8\uad6d \ubcf8\ud1a0\ub97c \ud5a5\ud574 \ubc1c\uc0ac\ub41c \uc911\uac70\ub9ac\ubbf8\uc0ac\uc77c\uc744 \uc54c\ub798\uc2a4\uce74 \ucf54\ub514\uc545 \uae30\uc9c0\uc5d0\uc11c \uc0ac\ub4dc\ub85c \ub9de\ucd94\ub294 \uc694\uaca9\uc2e4\ud5d8\uc5d0 14\ucc28\ub840 \uc131\uacf5\ud588\ub2e4. \uc911\uac70\ub9ac\ud0c4\ub3c4\ubbf8\uc0ac\uc77c, IRBM\uc758 \uc18d\ub3c4\ub85c \ub0a0\uc544\uac00\ub294 \ube44\ud589\uccb4\ub97c \uba85\uc911\uc2dc\ud0a8 \uac83\uc774\ub2e4. \ubbf8\uad6d \ubbf8\uc0ac\uc77c\ubc29\uc5b4\uad6d\uc740 \uc131\uba85\uc744 \ud1b5\ud574 \"\uc774\ubc88 \uc2dc\ud5d8\uc740 \ud0c4\ub3c4\ubbf8\uc0ac\uc77c\uc744 \uc694\uaca9\ud558\uace0 \ud30c\uad34\ud558\ub294 \uc0ac\ub4dc \ubb34\uae30\uccb4\uacc4\uc758 \ub2a5\ub825\uc744 \ub354\uc6b1 \uc99d\uba85\ud588\ub2e4\"\uace0 \ud3c9\uac00\ud588\ub2e4. \uc0ac\ub4dc\ub294 14\ucc28\ub840\uc758 \uc694\uaca9\uc2dc\ud5d8\uc5d0\uc11c \ubaa8\ub450 \uc131\uacf5\uc744 \uac70\ub46c 100% \uc758 \uba85\uc911\ub960\uc744 \ubcf4\uc774\uace0 \uc788\ub2e4. \uc0ac\ub4dc \uc2dc\ud5d8 \ubc1c\uc0ac\ub294 \ube44\ud589\uccb4\uc758 \ubc1c\uc0ac \uc2dc\uac04\uc744 \ud2b9\uc815\ud558\uc9c0 \uc54a\ub294 \ub4f1 \uc2e4\uc804 \uc0c1\ud669\uacfc \uac19\uc740 \uc870\uac74\uc5d0\uc11c \uc774\ub904\uc84c\ub2e4. \ud558\uc640\uc774 \ubbf8\uc0ac\uc77c \ubc1c\uc0ac \uae30\uc9c0\ub294 \uc5b8\uc81c \uc5b4\ub514\uc11c \ubbf8\uc0ac\uc77c\uc744 \ubc1c\uc0ac\ud560\uc9c0 \uc0ac\uc804\uc5d0 \uc54c\ub9ac\uc9c0 \uc54a\uc558\uc73c\uba70 \uae30\uc2b5\uc801\uc73c\ub85c \uc54c\ub798\uc2a4\uce74\ub97c \ud5a5\ud574 \ubbf8\uc0ac\uc77c\uc744 \ubc1c\uc0ac\ud588\uace0 \uc54c\ub798\uc2a4\uce74 \ucf54\ub514\uc545 \uae30\uc9c0\ub294 \uc2e4\uc804 \uc0c1\ud669\uacfc \uac19\uc740 \uc870\uac74\uc5d0\uc11c \ubbf8\uc0ac\uc77c \uc694\uaca9\uc5d0 \uc131\uacf5\ud588\ub2e4.", "sentence_answer": "\uc0ac\ub4dc\ub294 14\ucc28\ub840\uc758 \uc694\uaca9\uc2dc\ud5d8\uc5d0\uc11c \ubaa8\ub450 \uc131\uacf5\uc744 \uac70\ub46c 100% \uc758 \uba85\uc911\ub960\uc744 \ubcf4\uc774\uace0 \uc788\ub2e4.", "paragraph_id": "6512357-5-1", "paragraph_question": "question: 2017\ub144 7\uc6d4 11\uc77c \uc0ac\ub4dc \uc694\uaca9\uc2e4\ud5d8 \ub2f9\uc2dc \uba85\uc911\ub960\uc740?, context: 2017\ub144 7\uc6d4 11\uc77c, \ubbf8\uad6d \uad6d\ubc29\ubd80\ub294 \ud558\uc640\uc774\uc5d0\uc11c \ubbf8\uad6d \ubcf8\ud1a0\ub97c \ud5a5\ud574 \ubc1c\uc0ac\ub41c \uc911\uac70\ub9ac\ubbf8\uc0ac\uc77c\uc744 \uc54c\ub798\uc2a4\uce74 \ucf54\ub514\uc545 \uae30\uc9c0\uc5d0\uc11c \uc0ac\ub4dc\ub85c \ub9de\ucd94\ub294 \uc694\uaca9\uc2e4\ud5d8\uc5d0 14\ucc28\ub840 \uc131\uacf5\ud588\ub2e4. \uc911\uac70\ub9ac\ud0c4\ub3c4\ubbf8\uc0ac\uc77c, IRBM\uc758 \uc18d\ub3c4\ub85c \ub0a0\uc544\uac00\ub294 \ube44\ud589\uccb4\ub97c \uba85\uc911\uc2dc\ud0a8 \uac83\uc774\ub2e4. \ubbf8\uad6d \ubbf8\uc0ac\uc77c\ubc29\uc5b4\uad6d\uc740 \uc131\uba85\uc744 \ud1b5\ud574 \"\uc774\ubc88 \uc2dc\ud5d8\uc740 \ud0c4\ub3c4\ubbf8\uc0ac\uc77c\uc744 \uc694\uaca9\ud558\uace0 \ud30c\uad34\ud558\ub294 \uc0ac\ub4dc \ubb34\uae30\uccb4\uacc4\uc758 \ub2a5\ub825\uc744 \ub354\uc6b1 \uc99d\uba85\ud588\ub2e4\"\uace0 \ud3c9\uac00\ud588\ub2e4. \uc0ac\ub4dc\ub294 14\ucc28\ub840\uc758 \uc694\uaca9\uc2dc\ud5d8\uc5d0\uc11c \ubaa8\ub450 \uc131\uacf5\uc744 \uac70\ub46c 100%\uc758 \uba85\uc911\ub960\uc744 \ubcf4\uc774\uace0 \uc788\ub2e4. \uc0ac\ub4dc \uc2dc\ud5d8 \ubc1c\uc0ac\ub294 \ube44\ud589\uccb4\uc758 \ubc1c\uc0ac \uc2dc\uac04\uc744 \ud2b9\uc815\ud558\uc9c0 \uc54a\ub294 \ub4f1 \uc2e4\uc804 \uc0c1\ud669\uacfc \uac19\uc740 \uc870\uac74\uc5d0\uc11c \uc774\ub904\uc84c\ub2e4. \ud558\uc640\uc774 \ubbf8\uc0ac\uc77c \ubc1c\uc0ac \uae30\uc9c0\ub294 \uc5b8\uc81c \uc5b4\ub514\uc11c \ubbf8\uc0ac\uc77c\uc744 \ubc1c\uc0ac\ud560\uc9c0 \uc0ac\uc804\uc5d0 \uc54c\ub9ac\uc9c0 \uc54a\uc558\uc73c\uba70 \uae30\uc2b5\uc801\uc73c\ub85c \uc54c\ub798\uc2a4\uce74\ub97c \ud5a5\ud574 \ubbf8\uc0ac\uc77c\uc744 \ubc1c\uc0ac\ud588\uace0 \uc54c\ub798\uc2a4\uce74 \ucf54\ub514\uc545 \uae30\uc9c0\ub294 \uc2e4\uc804 \uc0c1\ud669\uacfc \uac19\uc740 \uc870\uac74\uc5d0\uc11c \ubbf8\uc0ac\uc77c \uc694\uaca9\uc5d0 \uc131\uacf5\ud588\ub2e4."} {"question": "\ud6c8\uc77c\ub4f1 \ud0dc\uadf9\ub300\uc218\uc7a5\uc744 \ud328\uc6a9\ud560 \ub54c \ub300\uc218\ub97c \ub450\ub974\ub294 \ubc29\ud5a5\uc740?", "paragraph": "\ud6c8\uc77c\ub4f1 \ud0dc\uadf9\ub300\uc218\uc7a5\uc740 \uc815\uc7a5(\u6b63\u7ae0)\uacfc \ubd80\uc7a5(\u526f\u7ae0)\uc73c\ub85c \uad6c\uc131\ub41c\ub2e4. \uc815\uc7a5\uc758 \uc7a5(\u7ae0), \uc989 \ubb38\ucc44\uc758 \uc7ac\uc9c8\uc740 \uae08(\u91d1)\uc774\uace0, \uc9c0\ub984\uc740 2\uce58(\u5bf8) 5\ud47c(\u5206)\uc774\uba70, \uae08\uc0c9 \uc120\uc744 \ub450\ub978 \uccad\ud64d\uc0c9\uc758 \ud0dc\uadf9\uc744 \uc911\uc2ec\uc73c\ub85c \uc678\ubcc0\uc5d0\ub294 \ubc31\uc0c9 \uad11\uc120\uc774 \uc774\uc5b4\uc84c\ub2e4. \uaf2d\uc9c0\uc640 \uace0\ub9ac\uc758 \uc7ac\uc9c8\uc740 \uae08\uc774\uba70, \ubc31\uc0c9\uc758 \uc624\uc58f\uaf43\uc744 \uc138 \uc7a5\uc758 \ub179\uc0c9 \uc78e\uc774 \ubc1b\uce58\uace0 \uc788\ub294 \ud615\uc0c1\uc73c\ub85c \uadf8 \ub4b7\uba74\uc5d0\ub294 \uc804\uc11c\uccb4\uc758 \u2018\ud6c8\uacf5\uc77c\ub4f1\u2019(\u52f3\u529f\u4e00\u7b49)\uc774 \uac00\ub85c\uc4f0\uae30\ub85c \uc0c8\uaca8\uc838 \uc788\ub2e4. \ud6c8\uc77c\ub4f1 \ud0dc\uadf9\ub300\uc218\uc7a5\uc744 \ud328\uc6a9\ud560 \ub54c\ub294 \ud64d\uc0c9 \ubc14\ud0d5\uc5d0 \uc30d\uccad\uc120(\u96d9\u9751\u7dda) \uac04\ub3c4\uc9c1(\u9593\u9053\u7e54)\uc73c\ub85c \uc774\ub8e8\uc5b4\uc9c4 \ub300\uc218(\u5927\u7dac)\ub97c \uc624\ub978\ucabd \uc5b4\uae68\uc5d0\uc11c \uc67c\ucabd \uc606\uad6c\ub9ac\uc5d0 \ub450\ub978 \ub2e4\uc74c\uc5d0 \ub05d\ubd80\uc704\uc5d0\uc11c \uad50\ucc28\ud558\uc5ec \uc624\uc58f\uaf43\uc744 \ucc9c\uc73c\ub85c \uc811\uc5b4\ub2ec\uace0 \uadf8 \ubc11\uc5d0 \uc815\uc7a5\uc744 \ub2ec\uc558\uc73c\uba70, \ubd80\uc7a5\uc740 \uc67c\ucabd \uac00\uc2b4\uc5d0 \ub2e8\ub2e4. \uc57d\uc218(\u7565\u7dac)\ub294 \ub300\uc218\uc640 \uc7ac\uc9c8\uc774 \uac19\uace0, \uadf8 \ubaa8\uc591\uc740 \uc6d0\ud615\uc73c\ub85c \uccad\ud64d\uc0c9\uc758 \uc678\ubcc0\uc120\uc744 \ub450\ub978 \uccad\ud64d\uc0c9\uc758 \ud0dc\uadf9\ubb38\uc774\uba70, \ud1b5\uc0c1 \uc608\ubcf5 \ucc29\uc6a9\uc2dc\uc5d0 \uc67c\ucabd \uc637\uae43 \ub2e8\ucd94 \uad6c\uba4d\uc5d0 \uac78\uc5b4\uc11c \ud328\uc6a9\ud55c\ub2e4.", "answer": "\uc624\ub978\ucabd \uc5b4\uae68\uc5d0\uc11c \uc67c\ucabd \uc606\uad6c\ub9ac", "sentence": "\ud6c8\uc77c\ub4f1 \ud0dc\uadf9\ub300\uc218\uc7a5\uc744 \ud328\uc6a9\ud560 \ub54c\ub294 \ud64d\uc0c9 \ubc14\ud0d5\uc5d0 \uc30d\uccad\uc120(\u96d9\u9751\u7dda) \uac04\ub3c4\uc9c1(\u9593\u9053\u7e54)\uc73c\ub85c \uc774\ub8e8\uc5b4\uc9c4 \ub300\uc218(\u5927\u7dac)\ub97c \uc624\ub978\ucabd \uc5b4\uae68\uc5d0\uc11c \uc67c\ucabd \uc606\uad6c\ub9ac \uc5d0 \ub450\ub978 \ub2e4\uc74c\uc5d0 \ub05d\ubd80\uc704\uc5d0\uc11c \uad50\ucc28\ud558\uc5ec \uc624\uc58f\uaf43\uc744 \ucc9c\uc73c\ub85c \uc811\uc5b4\ub2ec\uace0 \uadf8 \ubc11\uc5d0 \uc815\uc7a5\uc744 \ub2ec\uc558\uc73c\uba70, \ubd80\uc7a5\uc740 \uc67c\ucabd \uac00\uc2b4\uc5d0 \ub2e8\ub2e4.", "paragraph_sentence": "\ud6c8\uc77c\ub4f1 \ud0dc\uadf9\ub300\uc218\uc7a5\uc740 \uc815\uc7a5(\u6b63\u7ae0)\uacfc \ubd80\uc7a5(\u526f\u7ae0)\uc73c\ub85c \uad6c\uc131\ub41c\ub2e4. \uc815\uc7a5\uc758 \uc7a5(\u7ae0), \uc989 \ubb38\ucc44\uc758 \uc7ac\uc9c8\uc740 \uae08(\u91d1)\uc774\uace0, \uc9c0\ub984\uc740 2\uce58(\u5bf8) 5\ud47c(\u5206)\uc774\uba70, \uae08\uc0c9 \uc120\uc744 \ub450\ub978 \uccad\ud64d\uc0c9\uc758 \ud0dc\uadf9\uc744 \uc911\uc2ec\uc73c\ub85c \uc678\ubcc0\uc5d0\ub294 \ubc31\uc0c9 \uad11\uc120\uc774 \uc774\uc5b4\uc84c\ub2e4. \uaf2d\uc9c0\uc640 \uace0\ub9ac\uc758 \uc7ac\uc9c8\uc740 \uae08\uc774\uba70, \ubc31\uc0c9\uc758 \uc624\uc58f\uaf43\uc744 \uc138 \uc7a5\uc758 \ub179\uc0c9 \uc78e\uc774 \ubc1b\uce58\uace0 \uc788\ub294 \ud615\uc0c1\uc73c\ub85c \uadf8 \ub4b7\uba74\uc5d0\ub294 \uc804\uc11c\uccb4\uc758 \u2018\ud6c8\uacf5\uc77c\ub4f1\u2019(\u52f3\u529f\u4e00\u7b49)\uc774 \uac00\ub85c\uc4f0\uae30\ub85c \uc0c8\uaca8\uc838 \uc788\ub2e4. \ud6c8\uc77c\ub4f1 \ud0dc\uadf9\ub300\uc218\uc7a5\uc744 \ud328\uc6a9\ud560 \ub54c\ub294 \ud64d\uc0c9 \ubc14\ud0d5\uc5d0 \uc30d\uccad\uc120(\u96d9\u9751\u7dda) \uac04\ub3c4\uc9c1(\u9593\u9053\u7e54)\uc73c\ub85c \uc774\ub8e8\uc5b4\uc9c4 \ub300\uc218(\u5927\u7dac)\ub97c \uc624\ub978\ucabd \uc5b4\uae68\uc5d0\uc11c \uc67c\ucabd \uc606\uad6c\ub9ac \uc5d0 \ub450\ub978 \ub2e4\uc74c\uc5d0 \ub05d\ubd80\uc704\uc5d0\uc11c \uad50\ucc28\ud558\uc5ec \uc624\uc58f\uaf43\uc744 \ucc9c\uc73c\ub85c \uc811\uc5b4\ub2ec\uace0 \uadf8 \ubc11\uc5d0 \uc815\uc7a5\uc744 \ub2ec\uc558\uc73c\uba70, \ubd80\uc7a5\uc740 \uc67c\ucabd \uac00\uc2b4\uc5d0 \ub2e8\ub2e4. \uc57d\uc218(\u7565\u7dac)\ub294 \ub300\uc218\uc640 \uc7ac\uc9c8\uc774 \uac19\uace0, \uadf8 \ubaa8\uc591\uc740 \uc6d0\ud615\uc73c\ub85c \uccad\ud64d\uc0c9\uc758 \uc678\ubcc0\uc120\uc744 \ub450\ub978 \uccad\ud64d\uc0c9\uc758 \ud0dc\uadf9\ubb38\uc774\uba70, \ud1b5\uc0c1 \uc608\ubcf5 \ucc29\uc6a9\uc2dc\uc5d0 \uc67c\ucabd \uc637\uae43 \ub2e8\ucd94 \uad6c\uba4d\uc5d0 \uac78\uc5b4\uc11c \ud328\uc6a9\ud55c\ub2e4.", "paragraph_answer": "\ud6c8\uc77c\ub4f1 \ud0dc\uadf9\ub300\uc218\uc7a5\uc740 \uc815\uc7a5(\u6b63\u7ae0)\uacfc \ubd80\uc7a5(\u526f\u7ae0)\uc73c\ub85c \uad6c\uc131\ub41c\ub2e4. \uc815\uc7a5\uc758 \uc7a5(\u7ae0), \uc989 \ubb38\ucc44\uc758 \uc7ac\uc9c8\uc740 \uae08(\u91d1)\uc774\uace0, \uc9c0\ub984\uc740 2\uce58(\u5bf8) 5\ud47c(\u5206)\uc774\uba70, \uae08\uc0c9 \uc120\uc744 \ub450\ub978 \uccad\ud64d\uc0c9\uc758 \ud0dc\uadf9\uc744 \uc911\uc2ec\uc73c\ub85c \uc678\ubcc0\uc5d0\ub294 \ubc31\uc0c9 \uad11\uc120\uc774 \uc774\uc5b4\uc84c\ub2e4. \uaf2d\uc9c0\uc640 \uace0\ub9ac\uc758 \uc7ac\uc9c8\uc740 \uae08\uc774\uba70, \ubc31\uc0c9\uc758 \uc624\uc58f\uaf43\uc744 \uc138 \uc7a5\uc758 \ub179\uc0c9 \uc78e\uc774 \ubc1b\uce58\uace0 \uc788\ub294 \ud615\uc0c1\uc73c\ub85c \uadf8 \ub4b7\uba74\uc5d0\ub294 \uc804\uc11c\uccb4\uc758 \u2018\ud6c8\uacf5\uc77c\ub4f1\u2019(\u52f3\u529f\u4e00\u7b49)\uc774 \uac00\ub85c\uc4f0\uae30\ub85c \uc0c8\uaca8\uc838 \uc788\ub2e4. \ud6c8\uc77c\ub4f1 \ud0dc\uadf9\ub300\uc218\uc7a5\uc744 \ud328\uc6a9\ud560 \ub54c\ub294 \ud64d\uc0c9 \ubc14\ud0d5\uc5d0 \uc30d\uccad\uc120(\u96d9\u9751\u7dda) \uac04\ub3c4\uc9c1(\u9593\u9053\u7e54)\uc73c\ub85c \uc774\ub8e8\uc5b4\uc9c4 \ub300\uc218(\u5927\u7dac)\ub97c \uc624\ub978\ucabd \uc5b4\uae68\uc5d0\uc11c \uc67c\ucabd \uc606\uad6c\ub9ac \uc5d0 \ub450\ub978 \ub2e4\uc74c\uc5d0 \ub05d\ubd80\uc704\uc5d0\uc11c \uad50\ucc28\ud558\uc5ec \uc624\uc58f\uaf43\uc744 \ucc9c\uc73c\ub85c \uc811\uc5b4\ub2ec\uace0 \uadf8 \ubc11\uc5d0 \uc815\uc7a5\uc744 \ub2ec\uc558\uc73c\uba70, \ubd80\uc7a5\uc740 \uc67c\ucabd \uac00\uc2b4\uc5d0 \ub2e8\ub2e4. \uc57d\uc218(\u7565\u7dac)\ub294 \ub300\uc218\uc640 \uc7ac\uc9c8\uc774 \uac19\uace0, \uadf8 \ubaa8\uc591\uc740 \uc6d0\ud615\uc73c\ub85c \uccad\ud64d\uc0c9\uc758 \uc678\ubcc0\uc120\uc744 \ub450\ub978 \uccad\ud64d\uc0c9\uc758 \ud0dc\uadf9\ubb38\uc774\uba70, \ud1b5\uc0c1 \uc608\ubcf5 \ucc29\uc6a9\uc2dc\uc5d0 \uc67c\ucabd \uc637\uae43 \ub2e8\ucd94 \uad6c\uba4d\uc5d0 \uac78\uc5b4\uc11c \ud328\uc6a9\ud55c\ub2e4.", "sentence_answer": "\ud6c8\uc77c\ub4f1 \ud0dc\uadf9\ub300\uc218\uc7a5\uc744 \ud328\uc6a9\ud560 \ub54c\ub294 \ud64d\uc0c9 \ubc14\ud0d5\uc5d0 \uc30d\uccad\uc120(\u96d9\u9751\u7dda) \uac04\ub3c4\uc9c1(\u9593\u9053\u7e54)\uc73c\ub85c \uc774\ub8e8\uc5b4\uc9c4 \ub300\uc218(\u5927\u7dac)\ub97c \uc624\ub978\ucabd \uc5b4\uae68\uc5d0\uc11c \uc67c\ucabd \uc606\uad6c\ub9ac \uc5d0 \ub450\ub978 \ub2e4\uc74c\uc5d0 \ub05d\ubd80\uc704\uc5d0\uc11c \uad50\ucc28\ud558\uc5ec \uc624\uc58f\uaf43\uc744 \ucc9c\uc73c\ub85c \uc811\uc5b4\ub2ec\uace0 \uadf8 \ubc11\uc5d0 \uc815\uc7a5\uc744 \ub2ec\uc558\uc73c\uba70, \ubd80\uc7a5\uc740 \uc67c\ucabd \uac00\uc2b4\uc5d0 \ub2e8\ub2e4.", "paragraph_id": "6214003-2-1", "paragraph_question": "question: \ud6c8\uc77c\ub4f1 \ud0dc\uadf9\ub300\uc218\uc7a5\uc744 \ud328\uc6a9\ud560 \ub54c \ub300\uc218\ub97c \ub450\ub974\ub294 \ubc29\ud5a5\uc740?, context: \ud6c8\uc77c\ub4f1 \ud0dc\uadf9\ub300\uc218\uc7a5\uc740 \uc815\uc7a5(\u6b63\u7ae0)\uacfc \ubd80\uc7a5(\u526f\u7ae0)\uc73c\ub85c \uad6c\uc131\ub41c\ub2e4. \uc815\uc7a5\uc758 \uc7a5(\u7ae0), \uc989 \ubb38\ucc44\uc758 \uc7ac\uc9c8\uc740 \uae08(\u91d1)\uc774\uace0, \uc9c0\ub984\uc740 2\uce58(\u5bf8) 5\ud47c(\u5206)\uc774\uba70, \uae08\uc0c9 \uc120\uc744 \ub450\ub978 \uccad\ud64d\uc0c9\uc758 \ud0dc\uadf9\uc744 \uc911\uc2ec\uc73c\ub85c \uc678\ubcc0\uc5d0\ub294 \ubc31\uc0c9 \uad11\uc120\uc774 \uc774\uc5b4\uc84c\ub2e4. \uaf2d\uc9c0\uc640 \uace0\ub9ac\uc758 \uc7ac\uc9c8\uc740 \uae08\uc774\uba70, \ubc31\uc0c9\uc758 \uc624\uc58f\uaf43\uc744 \uc138 \uc7a5\uc758 \ub179\uc0c9 \uc78e\uc774 \ubc1b\uce58\uace0 \uc788\ub294 \ud615\uc0c1\uc73c\ub85c \uadf8 \ub4b7\uba74\uc5d0\ub294 \uc804\uc11c\uccb4\uc758 \u2018\ud6c8\uacf5\uc77c\ub4f1\u2019(\u52f3\u529f\u4e00\u7b49)\uc774 \uac00\ub85c\uc4f0\uae30\ub85c \uc0c8\uaca8\uc838 \uc788\ub2e4. \ud6c8\uc77c\ub4f1 \ud0dc\uadf9\ub300\uc218\uc7a5\uc744 \ud328\uc6a9\ud560 \ub54c\ub294 \ud64d\uc0c9 \ubc14\ud0d5\uc5d0 \uc30d\uccad\uc120(\u96d9\u9751\u7dda) \uac04\ub3c4\uc9c1(\u9593\u9053\u7e54)\uc73c\ub85c \uc774\ub8e8\uc5b4\uc9c4 \ub300\uc218(\u5927\u7dac)\ub97c \uc624\ub978\ucabd \uc5b4\uae68\uc5d0\uc11c \uc67c\ucabd \uc606\uad6c\ub9ac\uc5d0 \ub450\ub978 \ub2e4\uc74c\uc5d0 \ub05d\ubd80\uc704\uc5d0\uc11c \uad50\ucc28\ud558\uc5ec \uc624\uc58f\uaf43\uc744 \ucc9c\uc73c\ub85c \uc811\uc5b4\ub2ec\uace0 \uadf8 \ubc11\uc5d0 \uc815\uc7a5\uc744 \ub2ec\uc558\uc73c\uba70, \ubd80\uc7a5\uc740 \uc67c\ucabd \uac00\uc2b4\uc5d0 \ub2e8\ub2e4. \uc57d\uc218(\u7565\u7dac)\ub294 \ub300\uc218\uc640 \uc7ac\uc9c8\uc774 \uac19\uace0, \uadf8 \ubaa8\uc591\uc740 \uc6d0\ud615\uc73c\ub85c \uccad\ud64d\uc0c9\uc758 \uc678\ubcc0\uc120\uc744 \ub450\ub978 \uccad\ud64d\uc0c9\uc758 \ud0dc\uadf9\ubb38\uc774\uba70, \ud1b5\uc0c1 \uc608\ubcf5 \ucc29\uc6a9\uc2dc\uc5d0 \uc67c\ucabd \uc637\uae43 \ub2e8\ucd94 \uad6c\uba4d\uc5d0 \uac78\uc5b4\uc11c \ud328\uc6a9\ud55c\ub2e4."} {"question": "\ub3c4\ub07c\uac00 \ub9c8\uc2a4\ud0c0 \uc6b0\uc758 \uc2f1\uae00\uc744 \ucc38\uc5ec\ud558\uc600\ub294\ub370, \uc81c\ubaa9\uc740?", "paragraph": "\uc74c\ubc18 \ubc1c\ub9e4 \ud6c4 \ud53c\ucc98\ub9c1 \ud65c\ub3d9\uc744 \ud558\ub358 \ub3c4\ub07c\ub294 \ub354\ucf70\uc774\uc5c7 \ucef4\ubc31\uacfc \ud568\uaed8 \uc77c\ub9ac\ub124\uc5b4 \ub808\ucf54\uc988 3\uc8fc\ub144 \ud22c\uc5b4\ub97c \uc804\uac1c\ud558\uc600\ub2e4. \uadf8\ub9ac\uace0 2014\ub144 5\uc6d4 21\uc77c, \uc77c\ub9ac\ub124\uc5b4 \ub808\ucf54\uc988\ub85c\uc11c \ucef4\ud544\ub808\uc774\uc158 \uc74c\ubc18 11:11\uc744 \ubc1c\ub9e4\ud558\uc600\ub2e4. \uc774\ud6c4 \ub354\ucf70\uc774\uc5c7\uacfc \ud568\uaed8 \uc5e0\ub137 \u300aShow Me The Money 3\u300b\uc5d0 \ud504\ub85c\ub4c0\uc11c\ub85c \ucd9c\uc5f0\ud558\uc600\ub294\ub370, \ud300 \ub0b4 \uba64\ubc84\uc600\ub358 \ubc14\ube44\ub97c \uc6b0\uc2b9\uc2dc\ud0a4\uba74\uc11c \uadf8\uc640 \ub3d9\uc2dc\uc5d0 \ucef4\ud544\ub808\uc774\uc158 \uc74c\ubc18\uc758 \uc218\ub85d\uace1\uc774\uc5c8\ub358 \u3008\uc5f0\uacb0\uace0\ub9ac\u3009, \u3008\uac00\u3009\uc758 \uc131\uacf5\uacfc \ud568\uaed8, \ub3c4\ub07c-\ub354\ucf70\uc774\uc5c7\uc758 \ub9cc\ub2f4 \ub4f1\uc73c\ub85c \ub300\uc911\uc5d0\uac8c \ud06c\uac8c \uc778\uc9c0\ub3c4\ub97c \uc313\uc558\ub2e4. \ubc29\uc1a1 \ud65c\ub3d9 \ud6c4\uc5d0\ub3c4 \uc74c\uc545 \ud65c\ub3d9\uc744 \uafb8\uc900\ud558\uac8c \ud558\uc600\ub294\ub370, 8\uc6d4 22\uc77c\uc5d0\ub294 \uc2f1\uae00 \"We Gotta Know\"\ub97c \ubc1c\ub9e4\ud558\uc600\uace0, \uc774\uc5b4 1\uc9d1 \uc218\ub85d\uace1\uc774\uc5c8\ub358 \"My Love\"\uc758 \ud53c\uc544\ub178 \ubc84\uc804, 11\uc6d4 11\uc77c\uc5d0\ub294 \uc2f1\uae00 \u3008\uce58\ud0a4\ucc28\uce74\ucd08\ucf54\ucd08\u3009, \uc77c\ub9ac\ub124\uc5b4 \ub808\ucf54\uc988\uc640 \ud568\uaed8 \"Rockin' with the Best\"\ub97c \ubc1c\ub9e4\ud558\uc600\ub2e4. \ubc29\uc1a1 \ud6c4 \uc778\uc5f0\uc73c\ub85c \ub9c8\uc2a4\ud0c0 \uc6b0\uc758 \uc2f1\uae00 \u3008\uc774\ub9ac\uc640\ubd10\u3009, \ubc14\uc2a4\ucf54\uc758 \uc2f1\uae00 \"DON\"\uc5d0\ub3c4 \ucc38\uc5ec\ud558\uc600\uc73c\uba70 \uadf8 \uc678\uc5d0 \uc18c\uc9c4, \uc804\uc778\uad8c\uacfc \ud568\uaed8 \uc2f1\uae00\uc744 \ubc1c\ub9e4\ud55c \ubc14 \uc788\uace0, \ub354\ucf70\uc774\uc5c7\uacfc \ud568\uaed8 \uc601\ud654 \u300a\ud14c\uc774\ud070 3\u300b\uc758 \uad6d\ub0b4 OST \"Good Luck\"\uc744 \ubc1c\ub9e4\ud558\uc600\ub2e4. \ub610\ud55c 2014\ub144 \uc5e0\ub137 \uc544\uc2dc\uc548 \ubba4\uc9c1 \uc5b4\uc6cc\ub4dc\uc5d0 \ucc38\uc11d\ud574 \ubc14\ube44\uc640 \ub9c8\uc2a4\ud0c0 \uc6b0\uc640 \ud568\uaed8 \uc624\ud504\ub2dd \uace1\uc73c\ub85c \ub77c\uc774\ube0c\ub97c \uc120\ubcf4\uc600\ub2e4.", "answer": "\uc774\ub9ac\uc640\ubd10", "sentence": "\ubc29\uc1a1 \ud6c4 \uc778\uc5f0\uc73c\ub85c \ub9c8\uc2a4\ud0c0 \uc6b0\uc758 \uc2f1\uae00 \u3008 \uc774\ub9ac\uc640\ubd10 \u3009, \ubc14\uc2a4\ucf54\uc758 \uc2f1\uae00 \"DON\"\uc5d0\ub3c4 \ucc38\uc5ec\ud558\uc600\uc73c\uba70 \uadf8 \uc678\uc5d0 \uc18c\uc9c4, \uc804\uc778\uad8c\uacfc \ud568\uaed8 \uc2f1\uae00\uc744 \ubc1c\ub9e4\ud55c \ubc14 \uc788\uace0, \ub354\ucf70\uc774\uc5c7\uacfc \ud568\uaed8 \uc601\ud654 \u300a\ud14c\uc774\ud070 3\u300b\uc758 \uad6d\ub0b4 OST \"Good Luck\"\uc744 \ubc1c\ub9e4\ud558\uc600\ub2e4.", "paragraph_sentence": "\uc74c\ubc18 \ubc1c\ub9e4 \ud6c4 \ud53c\ucc98\ub9c1 \ud65c\ub3d9\uc744 \ud558\ub358 \ub3c4\ub07c\ub294 \ub354\ucf70\uc774\uc5c7 \ucef4\ubc31\uacfc \ud568\uaed8 \uc77c\ub9ac\ub124\uc5b4 \ub808\ucf54\uc988 3\uc8fc\ub144 \ud22c\uc5b4\ub97c \uc804\uac1c\ud558\uc600\ub2e4. \uadf8\ub9ac\uace0 2014\ub144 5\uc6d4 21\uc77c, \uc77c\ub9ac\ub124\uc5b4 \ub808\ucf54\uc988\ub85c\uc11c \ucef4\ud544\ub808\uc774\uc158 \uc74c\ubc18 11:11\uc744 \ubc1c\ub9e4\ud558\uc600\ub2e4. \uc774\ud6c4 \ub354\ucf70\uc774\uc5c7\uacfc \ud568\uaed8 \uc5e0\ub137 \u300aShow Me The Money 3\u300b\uc5d0 \ud504\ub85c\ub4c0\uc11c\ub85c \ucd9c\uc5f0\ud558\uc600\ub294\ub370, \ud300 \ub0b4 \uba64\ubc84\uc600\ub358 \ubc14\ube44\ub97c \uc6b0\uc2b9\uc2dc\ud0a4\uba74\uc11c \uadf8\uc640 \ub3d9\uc2dc\uc5d0 \ucef4\ud544\ub808\uc774\uc158 \uc74c\ubc18\uc758 \uc218\ub85d\uace1\uc774\uc5c8\ub358 \u3008\uc5f0\uacb0\uace0\ub9ac\u3009, \u3008\uac00\u3009\uc758 \uc131\uacf5\uacfc \ud568\uaed8, \ub3c4\ub07c-\ub354\ucf70\uc774\uc5c7\uc758 \ub9cc\ub2f4 \ub4f1\uc73c\ub85c \ub300\uc911\uc5d0\uac8c \ud06c\uac8c \uc778\uc9c0\ub3c4\ub97c \uc313\uc558\ub2e4. \ubc29\uc1a1 \ud65c\ub3d9 \ud6c4\uc5d0\ub3c4 \uc74c\uc545 \ud65c\ub3d9\uc744 \uafb8\uc900\ud558\uac8c \ud558\uc600\ub294\ub370, 8\uc6d4 22\uc77c\uc5d0\ub294 \uc2f1\uae00 \"We Gotta Know\"\ub97c \ubc1c\ub9e4\ud558\uc600\uace0, \uc774\uc5b4 1\uc9d1 \uc218\ub85d\uace1\uc774\uc5c8\ub358 \"My Love\"\uc758 \ud53c\uc544\ub178 \ubc84\uc804, 11\uc6d4 11\uc77c\uc5d0\ub294 \uc2f1\uae00 \u3008\uce58\ud0a4\ucc28\uce74\ucd08\ucf54\ucd08\u3009, \uc77c\ub9ac\ub124\uc5b4 \ub808\ucf54\uc988\uc640 \ud568\uaed8 \"Rockin' with the Best\"\ub97c \ubc1c\ub9e4\ud558\uc600\ub2e4. \ubc29\uc1a1 \ud6c4 \uc778\uc5f0\uc73c\ub85c \ub9c8\uc2a4\ud0c0 \uc6b0\uc758 \uc2f1\uae00 \u3008 \uc774\ub9ac\uc640\ubd10 \u3009, \ubc14\uc2a4\ucf54\uc758 \uc2f1\uae00 \"DON\"\uc5d0\ub3c4 \ucc38\uc5ec\ud558\uc600\uc73c\uba70 \uadf8 \uc678\uc5d0 \uc18c\uc9c4, \uc804\uc778\uad8c\uacfc \ud568\uaed8 \uc2f1\uae00\uc744 \ubc1c\ub9e4\ud55c \ubc14 \uc788\uace0, \ub354\ucf70\uc774\uc5c7\uacfc \ud568\uaed8 \uc601\ud654 \u300a\ud14c\uc774\ud070 3\u300b\uc758 \uad6d\ub0b4 OST \"Good Luck\"\uc744 \ubc1c\ub9e4\ud558\uc600\ub2e4. \ub610\ud55c 2014\ub144 \uc5e0\ub137 \uc544\uc2dc\uc548 \ubba4\uc9c1 \uc5b4\uc6cc\ub4dc\uc5d0 \ucc38\uc11d\ud574 \ubc14\ube44\uc640 \ub9c8\uc2a4\ud0c0 \uc6b0\uc640 \ud568\uaed8 \uc624\ud504\ub2dd \uace1\uc73c\ub85c \ub77c\uc774\ube0c\ub97c \uc120\ubcf4\uc600\ub2e4.", "paragraph_answer": "\uc74c\ubc18 \ubc1c\ub9e4 \ud6c4 \ud53c\ucc98\ub9c1 \ud65c\ub3d9\uc744 \ud558\ub358 \ub3c4\ub07c\ub294 \ub354\ucf70\uc774\uc5c7 \ucef4\ubc31\uacfc \ud568\uaed8 \uc77c\ub9ac\ub124\uc5b4 \ub808\ucf54\uc988 3\uc8fc\ub144 \ud22c\uc5b4\ub97c \uc804\uac1c\ud558\uc600\ub2e4. \uadf8\ub9ac\uace0 2014\ub144 5\uc6d4 21\uc77c, \uc77c\ub9ac\ub124\uc5b4 \ub808\ucf54\uc988\ub85c\uc11c \ucef4\ud544\ub808\uc774\uc158 \uc74c\ubc18 11:11\uc744 \ubc1c\ub9e4\ud558\uc600\ub2e4. \uc774\ud6c4 \ub354\ucf70\uc774\uc5c7\uacfc \ud568\uaed8 \uc5e0\ub137 \u300aShow Me The Money 3\u300b\uc5d0 \ud504\ub85c\ub4c0\uc11c\ub85c \ucd9c\uc5f0\ud558\uc600\ub294\ub370, \ud300 \ub0b4 \uba64\ubc84\uc600\ub358 \ubc14\ube44\ub97c \uc6b0\uc2b9\uc2dc\ud0a4\uba74\uc11c \uadf8\uc640 \ub3d9\uc2dc\uc5d0 \ucef4\ud544\ub808\uc774\uc158 \uc74c\ubc18\uc758 \uc218\ub85d\uace1\uc774\uc5c8\ub358 \u3008\uc5f0\uacb0\uace0\ub9ac\u3009, \u3008\uac00\u3009\uc758 \uc131\uacf5\uacfc \ud568\uaed8, \ub3c4\ub07c-\ub354\ucf70\uc774\uc5c7\uc758 \ub9cc\ub2f4 \ub4f1\uc73c\ub85c \ub300\uc911\uc5d0\uac8c \ud06c\uac8c \uc778\uc9c0\ub3c4\ub97c \uc313\uc558\ub2e4. \ubc29\uc1a1 \ud65c\ub3d9 \ud6c4\uc5d0\ub3c4 \uc74c\uc545 \ud65c\ub3d9\uc744 \uafb8\uc900\ud558\uac8c \ud558\uc600\ub294\ub370, 8\uc6d4 22\uc77c\uc5d0\ub294 \uc2f1\uae00 \"We Gotta Know\"\ub97c \ubc1c\ub9e4\ud558\uc600\uace0, \uc774\uc5b4 1\uc9d1 \uc218\ub85d\uace1\uc774\uc5c8\ub358 \"My Love\"\uc758 \ud53c\uc544\ub178 \ubc84\uc804, 11\uc6d4 11\uc77c\uc5d0\ub294 \uc2f1\uae00 \u3008\uce58\ud0a4\ucc28\uce74\ucd08\ucf54\ucd08\u3009, \uc77c\ub9ac\ub124\uc5b4 \ub808\ucf54\uc988\uc640 \ud568\uaed8 \"Rockin' with the Best\"\ub97c \ubc1c\ub9e4\ud558\uc600\ub2e4. \ubc29\uc1a1 \ud6c4 \uc778\uc5f0\uc73c\ub85c \ub9c8\uc2a4\ud0c0 \uc6b0\uc758 \uc2f1\uae00 \u3008 \uc774\ub9ac\uc640\ubd10 \u3009, \ubc14\uc2a4\ucf54\uc758 \uc2f1\uae00 \"DON\"\uc5d0\ub3c4 \ucc38\uc5ec\ud558\uc600\uc73c\uba70 \uadf8 \uc678\uc5d0 \uc18c\uc9c4, \uc804\uc778\uad8c\uacfc \ud568\uaed8 \uc2f1\uae00\uc744 \ubc1c\ub9e4\ud55c \ubc14 \uc788\uace0, \ub354\ucf70\uc774\uc5c7\uacfc \ud568\uaed8 \uc601\ud654 \u300a\ud14c\uc774\ud070 3\u300b\uc758 \uad6d\ub0b4 OST \"Good Luck\"\uc744 \ubc1c\ub9e4\ud558\uc600\ub2e4. \ub610\ud55c 2014\ub144 \uc5e0\ub137 \uc544\uc2dc\uc548 \ubba4\uc9c1 \uc5b4\uc6cc\ub4dc\uc5d0 \ucc38\uc11d\ud574 \ubc14\ube44\uc640 \ub9c8\uc2a4\ud0c0 \uc6b0\uc640 \ud568\uaed8 \uc624\ud504\ub2dd \uace1\uc73c\ub85c \ub77c\uc774\ube0c\ub97c \uc120\ubcf4\uc600\ub2e4.", "sentence_answer": "\ubc29\uc1a1 \ud6c4 \uc778\uc5f0\uc73c\ub85c \ub9c8\uc2a4\ud0c0 \uc6b0\uc758 \uc2f1\uae00 \u3008 \uc774\ub9ac\uc640\ubd10 \u3009, \ubc14\uc2a4\ucf54\uc758 \uc2f1\uae00 \"DON\"\uc5d0\ub3c4 \ucc38\uc5ec\ud558\uc600\uc73c\uba70 \uadf8 \uc678\uc5d0 \uc18c\uc9c4, \uc804\uc778\uad8c\uacfc \ud568\uaed8 \uc2f1\uae00\uc744 \ubc1c\ub9e4\ud55c \ubc14 \uc788\uace0, \ub354\ucf70\uc774\uc5c7\uacfc \ud568\uaed8 \uc601\ud654 \u300a\ud14c\uc774\ud070 3\u300b\uc758 \uad6d\ub0b4 OST \"Good Luck\"\uc744 \ubc1c\ub9e4\ud558\uc600\ub2e4.", "paragraph_id": "6043008-6-2", "paragraph_question": "question: \ub3c4\ub07c\uac00 \ub9c8\uc2a4\ud0c0 \uc6b0\uc758 \uc2f1\uae00\uc744 \ucc38\uc5ec\ud558\uc600\ub294\ub370, \uc81c\ubaa9\uc740?, context: \uc74c\ubc18 \ubc1c\ub9e4 \ud6c4 \ud53c\ucc98\ub9c1 \ud65c\ub3d9\uc744 \ud558\ub358 \ub3c4\ub07c\ub294 \ub354\ucf70\uc774\uc5c7 \ucef4\ubc31\uacfc \ud568\uaed8 \uc77c\ub9ac\ub124\uc5b4 \ub808\ucf54\uc988 3\uc8fc\ub144 \ud22c\uc5b4\ub97c \uc804\uac1c\ud558\uc600\ub2e4. \uadf8\ub9ac\uace0 2014\ub144 5\uc6d4 21\uc77c, \uc77c\ub9ac\ub124\uc5b4 \ub808\ucf54\uc988\ub85c\uc11c \ucef4\ud544\ub808\uc774\uc158 \uc74c\ubc18 11:11\uc744 \ubc1c\ub9e4\ud558\uc600\ub2e4. \uc774\ud6c4 \ub354\ucf70\uc774\uc5c7\uacfc \ud568\uaed8 \uc5e0\ub137 \u300aShow Me The Money 3\u300b\uc5d0 \ud504\ub85c\ub4c0\uc11c\ub85c \ucd9c\uc5f0\ud558\uc600\ub294\ub370, \ud300 \ub0b4 \uba64\ubc84\uc600\ub358 \ubc14\ube44\ub97c \uc6b0\uc2b9\uc2dc\ud0a4\uba74\uc11c \uadf8\uc640 \ub3d9\uc2dc\uc5d0 \ucef4\ud544\ub808\uc774\uc158 \uc74c\ubc18\uc758 \uc218\ub85d\uace1\uc774\uc5c8\ub358 \u3008\uc5f0\uacb0\uace0\ub9ac\u3009, \u3008\uac00\u3009\uc758 \uc131\uacf5\uacfc \ud568\uaed8, \ub3c4\ub07c-\ub354\ucf70\uc774\uc5c7\uc758 \ub9cc\ub2f4 \ub4f1\uc73c\ub85c \ub300\uc911\uc5d0\uac8c \ud06c\uac8c \uc778\uc9c0\ub3c4\ub97c \uc313\uc558\ub2e4. \ubc29\uc1a1 \ud65c\ub3d9 \ud6c4\uc5d0\ub3c4 \uc74c\uc545 \ud65c\ub3d9\uc744 \uafb8\uc900\ud558\uac8c \ud558\uc600\ub294\ub370, 8\uc6d4 22\uc77c\uc5d0\ub294 \uc2f1\uae00 \"We Gotta Know\"\ub97c \ubc1c\ub9e4\ud558\uc600\uace0, \uc774\uc5b4 1\uc9d1 \uc218\ub85d\uace1\uc774\uc5c8\ub358 \"My Love\"\uc758 \ud53c\uc544\ub178 \ubc84\uc804, 11\uc6d4 11\uc77c\uc5d0\ub294 \uc2f1\uae00 \u3008\uce58\ud0a4\ucc28\uce74\ucd08\ucf54\ucd08\u3009, \uc77c\ub9ac\ub124\uc5b4 \ub808\ucf54\uc988\uc640 \ud568\uaed8 \"Rockin' with the Best\"\ub97c \ubc1c\ub9e4\ud558\uc600\ub2e4. \ubc29\uc1a1 \ud6c4 \uc778\uc5f0\uc73c\ub85c \ub9c8\uc2a4\ud0c0 \uc6b0\uc758 \uc2f1\uae00 \u3008\uc774\ub9ac\uc640\ubd10\u3009, \ubc14\uc2a4\ucf54\uc758 \uc2f1\uae00 \"DON\"\uc5d0\ub3c4 \ucc38\uc5ec\ud558\uc600\uc73c\uba70 \uadf8 \uc678\uc5d0 \uc18c\uc9c4, \uc804\uc778\uad8c\uacfc \ud568\uaed8 \uc2f1\uae00\uc744 \ubc1c\ub9e4\ud55c \ubc14 \uc788\uace0, \ub354\ucf70\uc774\uc5c7\uacfc \ud568\uaed8 \uc601\ud654 \u300a\ud14c\uc774\ud070 3\u300b\uc758 \uad6d\ub0b4 OST \"Good Luck\"\uc744 \ubc1c\ub9e4\ud558\uc600\ub2e4. \ub610\ud55c 2014\ub144 \uc5e0\ub137 \uc544\uc2dc\uc548 \ubba4\uc9c1 \uc5b4\uc6cc\ub4dc\uc5d0 \ucc38\uc11d\ud574 \ubc14\ube44\uc640 \ub9c8\uc2a4\ud0c0 \uc6b0\uc640 \ud568\uaed8 \uc624\ud504\ub2dd \uace1\uc73c\ub85c \ub77c\uc774\ube0c\ub97c \uc120\ubcf4\uc600\ub2e4."} {"question": "\uae40\uc815\uc740\uc774 \ubb34\ub825\uc744 \ud589\uc0ac \ud560\uac83\uc774\ub77c\uace0 \ubcf4\uba74\uc11c \uc774\uc5d0 \ub300\ud574 \uc6b0\ub824\ub97c \ud45c\ud55c \ubbf8\uad6d\uc758 \uad6d\ubc29\uc815\ubcf4\uad6d\uc7a5\uc740 \ub204\uad6c\uc778\uac00?", "paragraph": "\ub0a8\ubd81 \uc815\uc0c1\ud68c\ub2f4 \ud569\uc758\uc640 \ube44\ud575\ud654 \uc758\uc9c0 \ud45c\uba85\uc5d0 \ub300\ud574 \ud2b8\ub7fc\ud504\ub294 \"\ub9ce\uc740 \uc0ac\ub78c\uc774 \uc774\ub7f0 \ub0a0\uc774 \uc624\ub9ac\ub77c\uace0 \uc0dd\uac01\ud558\uc9c0 \ubabb\ud588\ub2e4\"\uace0 \ub9d0\ud558\uba70 \ud658\uc601 \uc785\uc7a5\uc744 \ubcf4\uc600\ub2e4. \ud2b8\ub7fc\ud504\ub294 \ud2b8\uc704\ud130\ub97c \ud1b5\ud574 \"\uc218\ub144 \ub9cc\uc5d0 \ucc98\uc74c\uc73c\ub85c \uc9c4\uc9c0\ud55c \ub178\ub825\uc774 \ubaa8\ub4e0 \uad00\ub828 \ub2f9\uc0ac\uc790\ub4e4\uc5d0 \uc758\ud574 \ud3bc\uccd0\uc9c0\uace0 \uc788\ub2e4\"\uba70 \"\ud5db\ub41c \ud76c\ub9dd\uc77c\uc9c0\ub3c4 \ubaa8\ub978\ub2e4. \uadf8\ub7ec\ub098 \ubbf8\uad6d\uc740 \uc5b4\ub290 \ubc29\ud5a5\uc774 \ub410\ub4e0 \uc5f4\uc2ec\ud788 \uac08 \uc900\ube44\uac00 \ub3fc \uc788\ub2e4\"\uace0 \ud588\ub2e4. \ub610 \"\uadf8\uac83\uc740 \uc804 \uc138\uacc4\ub97c \uc704\ud574 \uc704\ub300\ud55c \uc77c\uc774 \ub420 \uac83\uc774\uace0 \ubd81\ud55c\uc744 \uc704\ud574 \uc704\ub300\ud55c \uc77c\uc774 \ub420 \uac83\uc774\uba70, \ud55c\ubc18\ub3c4\ub97c \uc704\ud574 \uc704\ub300\ud55c \uc77c\uc774 \ub420 \uac83\uc774\uc9c0\ub9cc, \uc6b0\ub9ac\ub294 \ubb34\uc2a8 \uc77c\uc774 \uc77c\uc5b4\ub0a0\uc9c0 \ub450\uace0 \ubcfc \uac83\"\uc774\ub77c\uace0 \ud588\ub2e4. \uc9c0\ub09c \uba87 \ub2ec \uac04 \ubd81\ubbf8 \uc0ac\uc774\uc5d0 '\ub9d0\uc758 \uc804\uc7c1'\uc73c\ub85c \uc704\uae30\uac00 \uace0\uc870\ub410\ub358 \ud55c\ubc18\ub3c4 \uc815\uc138\uac00 \uc911\ub300\ud55c \ubd84\uc218\ub839\uc744 \ub9de\uac8c \ub418\uc5c8\ub2e4\ub294 \ubd84\uc11d\ub3c4 \uc788\ub2e4. \uad6d\ubb34\ubd80\ub3c4 \ub300\ubcc0\uc778 \ube0c\ub9ac\ud551\uc744 \ud1b5\ud574 \ube44\ud575\ud654 \uc758\uc9c0 \ud45c\uba85\uc5d0 \ub300\ud574 \"\uc633\uc740 \ubc29\ud5a5\uc5d0\uc11c \uc774\ub904\uc9c4 \uc870\uce58\ub77c\uace0 \ud655\uc2e0\ud55c\ub2e4\"\uba70 \ub2e4\uc74c \uc870\uce58\ub97c \ub17c\uc758\ud558\uaca0\ub2e4\uace0 \ubc1d\ud614\ub2e4. \u300a\uc6cc\uc2f1\ud134 \ud3ec\uc2a4\ud2b8\u300b\ub294 \"\uae40\uc815\uc740 \ub178\ub3d9\ub2f9 \uc704\uc6d0\uc7a5 \uc2a4\uc2a4\ub85c\uac00 \uba85\ubc31\ud788 \ubcf4\uc99d\ud55c \uadf8 \uc81c\uc548\uc740 \ubbf8\uad6d \ubcf8\ud1a0\ub97c \uc0ac\uac70\ub9ac\uc5d0 \ub450\uc5c8\ub358 \uc218\ub144\uac04\uc758 \ud575\uc2e4\ud5d8\uacfc \ubbf8\uc0ac\uc77c \uae30\uc220\uc758 \uc9c4\uc804 \uc774\ud6c4 \uc911\ub300\ud55c \ubc18\uc804\"\uc774\ub77c\uace0 \uc804\ud588\uc73c\uba70, \u300a\ub274\uc695 \ud0c0\uc784\uc2a4\u300b\ub294 \"\ubb38\uc7ac\uc778 \ub300\ud1b5\ub839\uc758 \ub178\ub825\uc740 \uc911\ub300\ud55c \uc774\uc815\ud45c\ub97c \uc138\uc6e0\ub2e4\"\uace0 \ubcf4\ub3c4\ud588\uc73c\uba70, CNN\uc740 \"\uc62c\ub9bc\ud53d\uc744 \uad00\uacc4 \ud574\ube59\uc758 \uacc4\uae30\ub85c \uc0bc\uc740 \ubb38 \ub300\ud1b5\ub839\uc758 \uc758\ubbf8 \uc788\ub294 \uc678\uad50\uc801 \uc131\uacfc\"\ub77c\uace0 \ud3c9\uac00\ud588\ub2e4. \ud558\uc9c0\ub9cc \ubbf8\uad6d\uc758 \uc815\ubcf4\uae30\uad00 \uc218\uc7a5\ub4e4\uc744 \ud68c\uc758\uc801\uc778 \ubc18\uc751\ub3c4 \ubcf4\uc600\ub2e4. \ub304 \ucf54\uce20 \uad6d\uac00\uc815\ubcf4\uad6d\uc7a5\uc740 \"\uc774 \ubaa8\ub4e0 \uac83\uc5d0 \ub300\ud574 \uc0c1\ub2f9\ud788 \ud68c\uc758\uc801\"\uc774\ub77c\uba70 \"\ub3cc\ud30c\uad6c\uac00 \ub9cc\ub4e4\uc5b4\uc9c4 \uac83\ucc98\ub7fc \ubcf4\uc77c \uc218 \uc788\uc73c\ub098, \uc0c1\ub2f9\ud788 \uc758\uc2ec\uc774 \ub4e0\ub2e4\"\uace0 \ud588\ub2e4.\ub85c\ubc84\ud2b8 \uc560\uc290\ub9ac \uad6d\ubc29\uc815\ubcf4\uad6d\uc7a5 \uc5ed\uc2dc \"\uae40\uc815\uc740\uc740 \ud575\u00b7\ud0c4\ub3c4\ubbf8\uc0ac\uc77c \ud504\ub85c\uadf8\ub7a8\uc744 \ud3ec\uae30\ud560 \uc758\ud5a5\uc744 \ubcf4\uc774\uc9c0 \uc54a\uace0 \uc788\ub2e4\"\uba70 \"\uae40\uc815\uc740\uc740 (\ubb34\ub825) \ucda9\ub3cc\uc5d0 \ub300\ube44\ud55c \uc900\ube44\ud0dc\uc138\ub97c \ub9e4\uc6b0 \uc9c4\uc9c0\ud558\uac8c \uc0dd\uac01\ud558\uace0 \uc788\ub2e4\"\uace0 \ub9d0\ud588\ub2e4.", "answer": "\ub85c\ubc84\ud2b8 \uc560\uc290\ub9ac", "sentence": "\ub85c\ubc84\ud2b8 \uc560\uc290\ub9ac \uad6d\ubc29\uc815\ubcf4\uad6d\uc7a5 \uc5ed\uc2dc \"\uae40\uc815\uc740\uc740 \ud575\u00b7\ud0c4\ub3c4\ubbf8\uc0ac\uc77c \ud504\ub85c\uadf8\ub7a8\uc744 \ud3ec\uae30\ud560 \uc758\ud5a5\uc744 \ubcf4\uc774\uc9c0 \uc54a\uace0 \uc788\ub2e4\"\uba70 \"\uae40\uc815\uc740\uc740 (\ubb34\ub825) \ucda9\ub3cc\uc5d0 \ub300\ube44\ud55c \uc900\ube44\ud0dc\uc138\ub97c \ub9e4\uc6b0 \uc9c4\uc9c0\ud558\uac8c \uc0dd\uac01\ud558\uace0 \uc788\ub2e4\"\uace0 \ub9d0\ud588\ub2e4.", "paragraph_sentence": "\ub0a8\ubd81 \uc815\uc0c1\ud68c\ub2f4 \ud569\uc758\uc640 \ube44\ud575\ud654 \uc758\uc9c0 \ud45c\uba85\uc5d0 \ub300\ud574 \ud2b8\ub7fc\ud504\ub294 \"\ub9ce\uc740 \uc0ac\ub78c\uc774 \uc774\ub7f0 \ub0a0\uc774 \uc624\ub9ac\ub77c\uace0 \uc0dd\uac01\ud558\uc9c0 \ubabb\ud588\ub2e4\"\uace0 \ub9d0\ud558\uba70 \ud658\uc601 \uc785\uc7a5\uc744 \ubcf4\uc600\ub2e4. \ud2b8\ub7fc\ud504\ub294 \ud2b8\uc704\ud130\ub97c \ud1b5\ud574 \"\uc218\ub144 \ub9cc\uc5d0 \ucc98\uc74c\uc73c\ub85c \uc9c4\uc9c0\ud55c \ub178\ub825\uc774 \ubaa8\ub4e0 \uad00\ub828 \ub2f9\uc0ac\uc790\ub4e4\uc5d0 \uc758\ud574 \ud3bc\uccd0\uc9c0\uace0 \uc788\ub2e4\"\uba70 \"\ud5db\ub41c \ud76c\ub9dd\uc77c\uc9c0\ub3c4 \ubaa8\ub978\ub2e4. \uadf8\ub7ec\ub098 \ubbf8\uad6d\uc740 \uc5b4\ub290 \ubc29\ud5a5\uc774 \ub410\ub4e0 \uc5f4\uc2ec\ud788 \uac08 \uc900\ube44\uac00 \ub3fc \uc788\ub2e4\"\uace0 \ud588\ub2e4. \ub610 \"\uadf8\uac83\uc740 \uc804 \uc138\uacc4\ub97c \uc704\ud574 \uc704\ub300\ud55c \uc77c\uc774 \ub420 \uac83\uc774\uace0 \ubd81\ud55c\uc744 \uc704\ud574 \uc704\ub300\ud55c \uc77c\uc774 \ub420 \uac83\uc774\uba70, \ud55c\ubc18\ub3c4\ub97c \uc704\ud574 \uc704\ub300\ud55c \uc77c\uc774 \ub420 \uac83\uc774\uc9c0\ub9cc, \uc6b0\ub9ac\ub294 \ubb34\uc2a8 \uc77c\uc774 \uc77c\uc5b4\ub0a0\uc9c0 \ub450\uace0 \ubcfc \uac83\"\uc774\ub77c\uace0 \ud588\ub2e4. \uc9c0\ub09c \uba87 \ub2ec \uac04 \ubd81\ubbf8 \uc0ac\uc774\uc5d0 '\ub9d0\uc758 \uc804\uc7c1'\uc73c\ub85c \uc704\uae30\uac00 \uace0\uc870\ub410\ub358 \ud55c\ubc18\ub3c4 \uc815\uc138\uac00 \uc911\ub300\ud55c \ubd84\uc218\ub839\uc744 \ub9de\uac8c \ub418\uc5c8\ub2e4\ub294 \ubd84\uc11d\ub3c4 \uc788\ub2e4. \uad6d\ubb34\ubd80\ub3c4 \ub300\ubcc0\uc778 \ube0c\ub9ac\ud551\uc744 \ud1b5\ud574 \ube44\ud575\ud654 \uc758\uc9c0 \ud45c\uba85\uc5d0 \ub300\ud574 \"\uc633\uc740 \ubc29\ud5a5\uc5d0\uc11c \uc774\ub904\uc9c4 \uc870\uce58\ub77c\uace0 \ud655\uc2e0\ud55c\ub2e4\"\uba70 \ub2e4\uc74c \uc870\uce58\ub97c \ub17c\uc758\ud558\uaca0\ub2e4\uace0 \ubc1d\ud614\ub2e4. \u300a\uc6cc\uc2f1\ud134 \ud3ec\uc2a4\ud2b8\u300b\ub294 \"\uae40\uc815\uc740 \ub178\ub3d9\ub2f9 \uc704\uc6d0\uc7a5 \uc2a4\uc2a4\ub85c\uac00 \uba85\ubc31\ud788 \ubcf4\uc99d\ud55c \uadf8 \uc81c\uc548\uc740 \ubbf8\uad6d \ubcf8\ud1a0\ub97c \uc0ac\uac70\ub9ac\uc5d0 \ub450\uc5c8\ub358 \uc218\ub144\uac04\uc758 \ud575\uc2e4\ud5d8\uacfc \ubbf8\uc0ac\uc77c \uae30\uc220\uc758 \uc9c4\uc804 \uc774\ud6c4 \uc911\ub300\ud55c \ubc18\uc804\"\uc774\ub77c\uace0 \uc804\ud588\uc73c\uba70, \u300a\ub274\uc695 \ud0c0\uc784\uc2a4\u300b\ub294 \"\ubb38\uc7ac\uc778 \ub300\ud1b5\ub839\uc758 \ub178\ub825\uc740 \uc911\ub300\ud55c \uc774\uc815\ud45c\ub97c \uc138\uc6e0\ub2e4\"\uace0 \ubcf4\ub3c4\ud588\uc73c\uba70, CNN\uc740 \"\uc62c\ub9bc\ud53d\uc744 \uad00\uacc4 \ud574\ube59\uc758 \uacc4\uae30\ub85c \uc0bc\uc740 \ubb38 \ub300\ud1b5\ub839\uc758 \uc758\ubbf8 \uc788\ub294 \uc678\uad50\uc801 \uc131\uacfc\"\ub77c\uace0 \ud3c9\uac00\ud588\ub2e4. \ud558\uc9c0\ub9cc \ubbf8\uad6d\uc758 \uc815\ubcf4\uae30\uad00 \uc218\uc7a5\ub4e4\uc744 \ud68c\uc758\uc801\uc778 \ubc18\uc751\ub3c4 \ubcf4\uc600\ub2e4. \ub304 \ucf54\uce20 \uad6d\uac00\uc815\ubcf4\uad6d\uc7a5\uc740 \"\uc774 \ubaa8\ub4e0 \uac83\uc5d0 \ub300\ud574 \uc0c1\ub2f9\ud788 \ud68c\uc758\uc801\"\uc774\ub77c\uba70 \"\ub3cc\ud30c\uad6c\uac00 \ub9cc\ub4e4\uc5b4\uc9c4 \uac83\ucc98\ub7fc \ubcf4\uc77c \uc218 \uc788\uc73c\ub098, \uc0c1\ub2f9\ud788 \uc758\uc2ec\uc774 \ub4e0\ub2e4\"\uace0 \ud588\ub2e4. \ub85c\ubc84\ud2b8 \uc560\uc290\ub9ac \uad6d\ubc29\uc815\ubcf4\uad6d\uc7a5 \uc5ed\uc2dc \"\uae40\uc815\uc740\uc740 \ud575\u00b7\ud0c4\ub3c4\ubbf8\uc0ac\uc77c \ud504\ub85c\uadf8\ub7a8\uc744 \ud3ec\uae30\ud560 \uc758\ud5a5\uc744 \ubcf4\uc774\uc9c0 \uc54a\uace0 \uc788\ub2e4\"\uba70 \"\uae40\uc815\uc740\uc740 (\ubb34\ub825) \ucda9\ub3cc\uc5d0 \ub300\ube44\ud55c \uc900\ube44\ud0dc\uc138\ub97c \ub9e4\uc6b0 \uc9c4\uc9c0\ud558\uac8c \uc0dd\uac01\ud558\uace0 \uc788\ub2e4\"\uace0 \ub9d0\ud588\ub2e4. ", "paragraph_answer": "\ub0a8\ubd81 \uc815\uc0c1\ud68c\ub2f4 \ud569\uc758\uc640 \ube44\ud575\ud654 \uc758\uc9c0 \ud45c\uba85\uc5d0 \ub300\ud574 \ud2b8\ub7fc\ud504\ub294 \"\ub9ce\uc740 \uc0ac\ub78c\uc774 \uc774\ub7f0 \ub0a0\uc774 \uc624\ub9ac\ub77c\uace0 \uc0dd\uac01\ud558\uc9c0 \ubabb\ud588\ub2e4\"\uace0 \ub9d0\ud558\uba70 \ud658\uc601 \uc785\uc7a5\uc744 \ubcf4\uc600\ub2e4. \ud2b8\ub7fc\ud504\ub294 \ud2b8\uc704\ud130\ub97c \ud1b5\ud574 \"\uc218\ub144 \ub9cc\uc5d0 \ucc98\uc74c\uc73c\ub85c \uc9c4\uc9c0\ud55c \ub178\ub825\uc774 \ubaa8\ub4e0 \uad00\ub828 \ub2f9\uc0ac\uc790\ub4e4\uc5d0 \uc758\ud574 \ud3bc\uccd0\uc9c0\uace0 \uc788\ub2e4\"\uba70 \"\ud5db\ub41c \ud76c\ub9dd\uc77c\uc9c0\ub3c4 \ubaa8\ub978\ub2e4. \uadf8\ub7ec\ub098 \ubbf8\uad6d\uc740 \uc5b4\ub290 \ubc29\ud5a5\uc774 \ub410\ub4e0 \uc5f4\uc2ec\ud788 \uac08 \uc900\ube44\uac00 \ub3fc \uc788\ub2e4\"\uace0 \ud588\ub2e4. \ub610 \"\uadf8\uac83\uc740 \uc804 \uc138\uacc4\ub97c \uc704\ud574 \uc704\ub300\ud55c \uc77c\uc774 \ub420 \uac83\uc774\uace0 \ubd81\ud55c\uc744 \uc704\ud574 \uc704\ub300\ud55c \uc77c\uc774 \ub420 \uac83\uc774\uba70, \ud55c\ubc18\ub3c4\ub97c \uc704\ud574 \uc704\ub300\ud55c \uc77c\uc774 \ub420 \uac83\uc774\uc9c0\ub9cc, \uc6b0\ub9ac\ub294 \ubb34\uc2a8 \uc77c\uc774 \uc77c\uc5b4\ub0a0\uc9c0 \ub450\uace0 \ubcfc \uac83\"\uc774\ub77c\uace0 \ud588\ub2e4. \uc9c0\ub09c \uba87 \ub2ec \uac04 \ubd81\ubbf8 \uc0ac\uc774\uc5d0 '\ub9d0\uc758 \uc804\uc7c1'\uc73c\ub85c \uc704\uae30\uac00 \uace0\uc870\ub410\ub358 \ud55c\ubc18\ub3c4 \uc815\uc138\uac00 \uc911\ub300\ud55c \ubd84\uc218\ub839\uc744 \ub9de\uac8c \ub418\uc5c8\ub2e4\ub294 \ubd84\uc11d\ub3c4 \uc788\ub2e4. \uad6d\ubb34\ubd80\ub3c4 \ub300\ubcc0\uc778 \ube0c\ub9ac\ud551\uc744 \ud1b5\ud574 \ube44\ud575\ud654 \uc758\uc9c0 \ud45c\uba85\uc5d0 \ub300\ud574 \"\uc633\uc740 \ubc29\ud5a5\uc5d0\uc11c \uc774\ub904\uc9c4 \uc870\uce58\ub77c\uace0 \ud655\uc2e0\ud55c\ub2e4\"\uba70 \ub2e4\uc74c \uc870\uce58\ub97c \ub17c\uc758\ud558\uaca0\ub2e4\uace0 \ubc1d\ud614\ub2e4. \u300a\uc6cc\uc2f1\ud134 \ud3ec\uc2a4\ud2b8\u300b\ub294 \"\uae40\uc815\uc740 \ub178\ub3d9\ub2f9 \uc704\uc6d0\uc7a5 \uc2a4\uc2a4\ub85c\uac00 \uba85\ubc31\ud788 \ubcf4\uc99d\ud55c \uadf8 \uc81c\uc548\uc740 \ubbf8\uad6d \ubcf8\ud1a0\ub97c \uc0ac\uac70\ub9ac\uc5d0 \ub450\uc5c8\ub358 \uc218\ub144\uac04\uc758 \ud575\uc2e4\ud5d8\uacfc \ubbf8\uc0ac\uc77c \uae30\uc220\uc758 \uc9c4\uc804 \uc774\ud6c4 \uc911\ub300\ud55c \ubc18\uc804\"\uc774\ub77c\uace0 \uc804\ud588\uc73c\uba70, \u300a\ub274\uc695 \ud0c0\uc784\uc2a4\u300b\ub294 \"\ubb38\uc7ac\uc778 \ub300\ud1b5\ub839\uc758 \ub178\ub825\uc740 \uc911\ub300\ud55c \uc774\uc815\ud45c\ub97c \uc138\uc6e0\ub2e4\"\uace0 \ubcf4\ub3c4\ud588\uc73c\uba70, CNN\uc740 \"\uc62c\ub9bc\ud53d\uc744 \uad00\uacc4 \ud574\ube59\uc758 \uacc4\uae30\ub85c \uc0bc\uc740 \ubb38 \ub300\ud1b5\ub839\uc758 \uc758\ubbf8 \uc788\ub294 \uc678\uad50\uc801 \uc131\uacfc\"\ub77c\uace0 \ud3c9\uac00\ud588\ub2e4. \ud558\uc9c0\ub9cc \ubbf8\uad6d\uc758 \uc815\ubcf4\uae30\uad00 \uc218\uc7a5\ub4e4\uc744 \ud68c\uc758\uc801\uc778 \ubc18\uc751\ub3c4 \ubcf4\uc600\ub2e4. \ub304 \ucf54\uce20 \uad6d\uac00\uc815\ubcf4\uad6d\uc7a5\uc740 \"\uc774 \ubaa8\ub4e0 \uac83\uc5d0 \ub300\ud574 \uc0c1\ub2f9\ud788 \ud68c\uc758\uc801\"\uc774\ub77c\uba70 \"\ub3cc\ud30c\uad6c\uac00 \ub9cc\ub4e4\uc5b4\uc9c4 \uac83\ucc98\ub7fc \ubcf4\uc77c \uc218 \uc788\uc73c\ub098, \uc0c1\ub2f9\ud788 \uc758\uc2ec\uc774 \ub4e0\ub2e4\"\uace0 \ud588\ub2e4. \ub85c\ubc84\ud2b8 \uc560\uc290\ub9ac \uad6d\ubc29\uc815\ubcf4\uad6d\uc7a5 \uc5ed\uc2dc \"\uae40\uc815\uc740\uc740 \ud575\u00b7\ud0c4\ub3c4\ubbf8\uc0ac\uc77c \ud504\ub85c\uadf8\ub7a8\uc744 \ud3ec\uae30\ud560 \uc758\ud5a5\uc744 \ubcf4\uc774\uc9c0 \uc54a\uace0 \uc788\ub2e4\"\uba70 \"\uae40\uc815\uc740\uc740 (\ubb34\ub825) \ucda9\ub3cc\uc5d0 \ub300\ube44\ud55c \uc900\ube44\ud0dc\uc138\ub97c \ub9e4\uc6b0 \uc9c4\uc9c0\ud558\uac8c \uc0dd\uac01\ud558\uace0 \uc788\ub2e4\"\uace0 \ub9d0\ud588\ub2e4.", "sentence_answer": " \ub85c\ubc84\ud2b8 \uc560\uc290\ub9ac \uad6d\ubc29\uc815\ubcf4\uad6d\uc7a5 \uc5ed\uc2dc \"\uae40\uc815\uc740\uc740 \ud575\u00b7\ud0c4\ub3c4\ubbf8\uc0ac\uc77c \ud504\ub85c\uadf8\ub7a8\uc744 \ud3ec\uae30\ud560 \uc758\ud5a5\uc744 \ubcf4\uc774\uc9c0 \uc54a\uace0 \uc788\ub2e4\"\uba70 \"\uae40\uc815\uc740\uc740 (\ubb34\ub825) \ucda9\ub3cc\uc5d0 \ub300\ube44\ud55c \uc900\ube44\ud0dc\uc138\ub97c \ub9e4\uc6b0 \uc9c4\uc9c0\ud558\uac8c \uc0dd\uac01\ud558\uace0 \uc788\ub2e4\"\uace0 \ub9d0\ud588\ub2e4.", "paragraph_id": "6546926-44-1", "paragraph_question": "question: \uae40\uc815\uc740\uc774 \ubb34\ub825\uc744 \ud589\uc0ac \ud560\uac83\uc774\ub77c\uace0 \ubcf4\uba74\uc11c \uc774\uc5d0 \ub300\ud574 \uc6b0\ub824\ub97c \ud45c\ud55c \ubbf8\uad6d\uc758 \uad6d\ubc29\uc815\ubcf4\uad6d\uc7a5\uc740 \ub204\uad6c\uc778\uac00?, context: \ub0a8\ubd81 \uc815\uc0c1\ud68c\ub2f4 \ud569\uc758\uc640 \ube44\ud575\ud654 \uc758\uc9c0 \ud45c\uba85\uc5d0 \ub300\ud574 \ud2b8\ub7fc\ud504\ub294 \"\ub9ce\uc740 \uc0ac\ub78c\uc774 \uc774\ub7f0 \ub0a0\uc774 \uc624\ub9ac\ub77c\uace0 \uc0dd\uac01\ud558\uc9c0 \ubabb\ud588\ub2e4\"\uace0 \ub9d0\ud558\uba70 \ud658\uc601 \uc785\uc7a5\uc744 \ubcf4\uc600\ub2e4. \ud2b8\ub7fc\ud504\ub294 \ud2b8\uc704\ud130\ub97c \ud1b5\ud574 \"\uc218\ub144 \ub9cc\uc5d0 \ucc98\uc74c\uc73c\ub85c \uc9c4\uc9c0\ud55c \ub178\ub825\uc774 \ubaa8\ub4e0 \uad00\ub828 \ub2f9\uc0ac\uc790\ub4e4\uc5d0 \uc758\ud574 \ud3bc\uccd0\uc9c0\uace0 \uc788\ub2e4\"\uba70 \"\ud5db\ub41c \ud76c\ub9dd\uc77c\uc9c0\ub3c4 \ubaa8\ub978\ub2e4. \uadf8\ub7ec\ub098 \ubbf8\uad6d\uc740 \uc5b4\ub290 \ubc29\ud5a5\uc774 \ub410\ub4e0 \uc5f4\uc2ec\ud788 \uac08 \uc900\ube44\uac00 \ub3fc \uc788\ub2e4\"\uace0 \ud588\ub2e4. \ub610 \"\uadf8\uac83\uc740 \uc804 \uc138\uacc4\ub97c \uc704\ud574 \uc704\ub300\ud55c \uc77c\uc774 \ub420 \uac83\uc774\uace0 \ubd81\ud55c\uc744 \uc704\ud574 \uc704\ub300\ud55c \uc77c\uc774 \ub420 \uac83\uc774\uba70, \ud55c\ubc18\ub3c4\ub97c \uc704\ud574 \uc704\ub300\ud55c \uc77c\uc774 \ub420 \uac83\uc774\uc9c0\ub9cc, \uc6b0\ub9ac\ub294 \ubb34\uc2a8 \uc77c\uc774 \uc77c\uc5b4\ub0a0\uc9c0 \ub450\uace0 \ubcfc \uac83\"\uc774\ub77c\uace0 \ud588\ub2e4. \uc9c0\ub09c \uba87 \ub2ec \uac04 \ubd81\ubbf8 \uc0ac\uc774\uc5d0 '\ub9d0\uc758 \uc804\uc7c1'\uc73c\ub85c \uc704\uae30\uac00 \uace0\uc870\ub410\ub358 \ud55c\ubc18\ub3c4 \uc815\uc138\uac00 \uc911\ub300\ud55c \ubd84\uc218\ub839\uc744 \ub9de\uac8c \ub418\uc5c8\ub2e4\ub294 \ubd84\uc11d\ub3c4 \uc788\ub2e4. \uad6d\ubb34\ubd80\ub3c4 \ub300\ubcc0\uc778 \ube0c\ub9ac\ud551\uc744 \ud1b5\ud574 \ube44\ud575\ud654 \uc758\uc9c0 \ud45c\uba85\uc5d0 \ub300\ud574 \"\uc633\uc740 \ubc29\ud5a5\uc5d0\uc11c \uc774\ub904\uc9c4 \uc870\uce58\ub77c\uace0 \ud655\uc2e0\ud55c\ub2e4\"\uba70 \ub2e4\uc74c \uc870\uce58\ub97c \ub17c\uc758\ud558\uaca0\ub2e4\uace0 \ubc1d\ud614\ub2e4. \u300a\uc6cc\uc2f1\ud134 \ud3ec\uc2a4\ud2b8\u300b\ub294 \"\uae40\uc815\uc740 \ub178\ub3d9\ub2f9 \uc704\uc6d0\uc7a5 \uc2a4\uc2a4\ub85c\uac00 \uba85\ubc31\ud788 \ubcf4\uc99d\ud55c \uadf8 \uc81c\uc548\uc740 \ubbf8\uad6d \ubcf8\ud1a0\ub97c \uc0ac\uac70\ub9ac\uc5d0 \ub450\uc5c8\ub358 \uc218\ub144\uac04\uc758 \ud575\uc2e4\ud5d8\uacfc \ubbf8\uc0ac\uc77c \uae30\uc220\uc758 \uc9c4\uc804 \uc774\ud6c4 \uc911\ub300\ud55c \ubc18\uc804\"\uc774\ub77c\uace0 \uc804\ud588\uc73c\uba70, \u300a\ub274\uc695 \ud0c0\uc784\uc2a4\u300b\ub294 \"\ubb38\uc7ac\uc778 \ub300\ud1b5\ub839\uc758 \ub178\ub825\uc740 \uc911\ub300\ud55c \uc774\uc815\ud45c\ub97c \uc138\uc6e0\ub2e4\"\uace0 \ubcf4\ub3c4\ud588\uc73c\uba70, CNN\uc740 \"\uc62c\ub9bc\ud53d\uc744 \uad00\uacc4 \ud574\ube59\uc758 \uacc4\uae30\ub85c \uc0bc\uc740 \ubb38 \ub300\ud1b5\ub839\uc758 \uc758\ubbf8 \uc788\ub294 \uc678\uad50\uc801 \uc131\uacfc\"\ub77c\uace0 \ud3c9\uac00\ud588\ub2e4. \ud558\uc9c0\ub9cc \ubbf8\uad6d\uc758 \uc815\ubcf4\uae30\uad00 \uc218\uc7a5\ub4e4\uc744 \ud68c\uc758\uc801\uc778 \ubc18\uc751\ub3c4 \ubcf4\uc600\ub2e4. \ub304 \ucf54\uce20 \uad6d\uac00\uc815\ubcf4\uad6d\uc7a5\uc740 \"\uc774 \ubaa8\ub4e0 \uac83\uc5d0 \ub300\ud574 \uc0c1\ub2f9\ud788 \ud68c\uc758\uc801\"\uc774\ub77c\uba70 \"\ub3cc\ud30c\uad6c\uac00 \ub9cc\ub4e4\uc5b4\uc9c4 \uac83\ucc98\ub7fc \ubcf4\uc77c \uc218 \uc788\uc73c\ub098, \uc0c1\ub2f9\ud788 \uc758\uc2ec\uc774 \ub4e0\ub2e4\"\uace0 \ud588\ub2e4.\ub85c\ubc84\ud2b8 \uc560\uc290\ub9ac \uad6d\ubc29\uc815\ubcf4\uad6d\uc7a5 \uc5ed\uc2dc \"\uae40\uc815\uc740\uc740 \ud575\u00b7\ud0c4\ub3c4\ubbf8\uc0ac\uc77c \ud504\ub85c\uadf8\ub7a8\uc744 \ud3ec\uae30\ud560 \uc758\ud5a5\uc744 \ubcf4\uc774\uc9c0 \uc54a\uace0 \uc788\ub2e4\"\uba70 \"\uae40\uc815\uc740\uc740 (\ubb34\ub825) \ucda9\ub3cc\uc5d0 \ub300\ube44\ud55c \uc900\ube44\ud0dc\uc138\ub97c \ub9e4\uc6b0 \uc9c4\uc9c0\ud558\uac8c \uc0dd\uac01\ud558\uace0 \uc788\ub2e4\"\uace0 \ub9d0\ud588\ub2e4."} {"question": "\uc774\uc774\uac00 1569\ub144 \uadc0\uad6d \ud6c4 \ud3b8\ucc2c\uc5d0 \ucc38\uc5ec\ud55c \uc2e4\ub85d\uc740?", "paragraph": "\uadf8\ub294 \uad00\ub8cc\uc0dd\ud65c \uc911\uc5d0\ub3c4 \uafb8\uc900\ud788 \uc774\ud669, \uc870\uc2dd, \uc131\ud63c, \uc815\ucca0 \ub4f1\uacfc \uc11c\uc2e0\uc744 \uc8fc\uace0\ubc1b\uc73c\uba70 \ud559\ubb38\uc744 \uc5f0\uad6c\ud558\uc600\ub2e4. 1568\ub144(\uc120\uc870 1\ub144) \ucc9c\ucd94\uc0ac(\u5343\u79cb\u4f7f)\uac00 \uba85\ub098\ub77c\ub85c \uac08 \ub54c \uc0ac\uc2e0\uc758 \uc11c\uc7a5\uad00(\u66f8\u72c0\u5b98)\uc73c\ub85c \uc5f0\uacbd\uc5d0 \ub3d9\ud589\ud55c \ub4a4, 1569\ub144 \uadc0\uad6d \ud6c4, \ud64d\ubb38\uad00\ubd80\uad50\ub9ac\uac00 \ub418\uc5c8\ub2e4. \uace7 \ud64d\ubb38\uad00\ubd80\uad50\ub9ac\ub85c \ucd98\ucd94\uad00\uae30\uc0ac\uad00\uc5d0 \uacb8\uc784\ub418\uc5c8\uc73c\uba70, \u300a\uba85\uc885\uc2e4\ub85d\u300b\uc758 \ud3b8\ucc2c\uc5d0 \ucc38\uc5ec\ud588\ub2e4. \uc774\ub54c \uadf8\ub294 \ud1f4\uccad\ud6c4 \uc624\ub79c \uce5c\uad6c\uc778 \uc131\ud63c\uacfc '\uc9c0\uc120\uc5ec\uc911(\u81f3\u5584\u8207\u4e2d)' \ubc0f '\uc548\uc790\uaca9\uce58\uc131\uc815\uc9c0\uc124(\u9854\u5b50\u683c\u81f4\u8aa0\u6b63\u4e4b\u8aaa)' \ub4f1 \uc8fc\uc790\ud559\uc758 \uadfc\ubcf8\ubb38\uc81c\ub4e4\uc744 \ub193\uace0 \uc11c\uc2e0\uc73c\ub85c \uc7a5\uae30\uac04 \ud1a0\ub860\ud558\uc600\ub2e4. \uadf8\ud574 \ud64d\ubb38\uad00\uad50\ub9ac\ub97c \uc9c0\ub0c8\uc73c\uba70, 1569\ub144 \uc74c\ub825 9\uc6d4\uc5d0\ub294 \uc1a1\uac15 \uc815\ucca0\uacfc \ud568\uaed8 \u300a\ub3d9\ud638\ubb38\ub2f5\u300b(\u6771\u6e56\u554f\u7b54)\uc774\ub77c\uace0 \ud558\ub294 \ucc45\uc744 \uc368\uc11c \uc120\uc870\uc5d0\uac8c \uc62c\ub838\ub2e4. \uadf8 \ubb34\ub835\uc5d0 \uac00\uc7a5 \uad00\uc2ec \uac16\uace0 \ucd94\uc9c4\ud574\uc57c \ud560 \uc77c\uc774 \ubb34\uc5c7\uc778\uac00\ub97c \u2018\uc2dc\ubb34\u2019(\u6642\u52d9)\uc640 \u2018\ubb34\uc2e4\u2019(\u52d9\u5be6)\uc774\ub77c\ub294 \uc6a9\uc5b4\ub97c \uc0ac\uc6a9\ud558\uba70, \uae09\uc120\ubb34\ub85c \ud574\uacb0\ud574\uc57c \ud558\ub294 \uc815\uce58\uac00 \uc5b4\ub5a4 \uac83\uc778\uac00\ub97c \uba85\ud655\ud788 \ubc1d\ud614\ub2e4. \uae30\ud68c\uac00 \uc788\uc744 \ub54c\ub9c8\ub2e4 \uc774\uc774\ub294 \uacc4\uc18d \u2018\uc2dc\ubb34\u2019\uac00 \uc5b4\ub5a4 \uac83\uc778\uac00\ub97c \uacc4\uc18d\ud558\uc5ec \uc0c1\uc18c\ub85c\ub3c4 \uc62c\ub9ac\uae30\ub3c4 \ud558\uc600\ub2e4.", "answer": "\uba85\uc885\uc2e4\ub85d", "sentence": "\uace7 \ud64d\ubb38\uad00\ubd80\uad50\ub9ac\ub85c \ucd98\ucd94\uad00\uae30\uc0ac\uad00\uc5d0 \uacb8\uc784\ub418\uc5c8\uc73c\uba70, \u300a \uba85\uc885\uc2e4\ub85d \u300b\uc758 \ud3b8\ucc2c\uc5d0 \ucc38\uc5ec\ud588\ub2e4.", "paragraph_sentence": "\uadf8\ub294 \uad00\ub8cc\uc0dd\ud65c \uc911\uc5d0\ub3c4 \uafb8\uc900\ud788 \uc774\ud669, \uc870\uc2dd, \uc131\ud63c, \uc815\ucca0 \ub4f1\uacfc \uc11c\uc2e0\uc744 \uc8fc\uace0\ubc1b\uc73c\uba70 \ud559\ubb38\uc744 \uc5f0\uad6c\ud558\uc600\ub2e4. 1568\ub144(\uc120\uc870 1\ub144) \ucc9c\ucd94\uc0ac(\u5343\u79cb\u4f7f)\uac00 \uba85\ub098\ub77c\ub85c \uac08 \ub54c \uc0ac\uc2e0\uc758 \uc11c\uc7a5\uad00(\u66f8\u72c0\u5b98)\uc73c\ub85c \uc5f0\uacbd\uc5d0 \ub3d9\ud589\ud55c \ub4a4, 1569\ub144 \uadc0\uad6d \ud6c4, \ud64d\ubb38\uad00\ubd80\uad50\ub9ac\uac00 \ub418\uc5c8\ub2e4. \uace7 \ud64d\ubb38\uad00\ubd80\uad50\ub9ac\ub85c \ucd98\ucd94\uad00\uae30\uc0ac\uad00\uc5d0 \uacb8\uc784\ub418\uc5c8\uc73c\uba70, \u300a \uba85\uc885\uc2e4\ub85d \u300b\uc758 \ud3b8\ucc2c\uc5d0 \ucc38\uc5ec\ud588\ub2e4. \uc774\ub54c \uadf8\ub294 \ud1f4\uccad\ud6c4 \uc624\ub79c \uce5c\uad6c\uc778 \uc131\ud63c\uacfc '\uc9c0\uc120\uc5ec\uc911(\u81f3\u5584\u8207\u4e2d)' \ubc0f '\uc548\uc790\uaca9\uce58\uc131\uc815\uc9c0\uc124(\u9854\u5b50\u683c\u81f4\u8aa0\u6b63\u4e4b\u8aaa)' \ub4f1 \uc8fc\uc790\ud559\uc758 \uadfc\ubcf8\ubb38\uc81c\ub4e4\uc744 \ub193\uace0 \uc11c\uc2e0\uc73c\ub85c \uc7a5\uae30\uac04 \ud1a0\ub860\ud558\uc600\ub2e4. \uadf8\ud574 \ud64d\ubb38\uad00\uad50\ub9ac\ub97c \uc9c0\ub0c8\uc73c\uba70, 1569\ub144 \uc74c\ub825 9\uc6d4\uc5d0\ub294 \uc1a1\uac15 \uc815\ucca0\uacfc \ud568\uaed8 \u300a\ub3d9\ud638\ubb38\ub2f5\u300b(\u6771\u6e56\u554f\u7b54)\uc774\ub77c\uace0 \ud558\ub294 \ucc45\uc744 \uc368\uc11c \uc120\uc870\uc5d0\uac8c \uc62c\ub838\ub2e4. \uadf8 \ubb34\ub835\uc5d0 \uac00\uc7a5 \uad00\uc2ec \uac16\uace0 \ucd94\uc9c4\ud574\uc57c \ud560 \uc77c\uc774 \ubb34\uc5c7\uc778\uac00\ub97c \u2018\uc2dc\ubb34\u2019(\u6642\u52d9)\uc640 \u2018\ubb34\uc2e4\u2019(\u52d9\u5be6)\uc774\ub77c\ub294 \uc6a9\uc5b4\ub97c \uc0ac\uc6a9\ud558\uba70, \uae09\uc120\ubb34\ub85c \ud574\uacb0\ud574\uc57c \ud558\ub294 \uc815\uce58\uac00 \uc5b4\ub5a4 \uac83\uc778\uac00\ub97c \uba85\ud655\ud788 \ubc1d\ud614\ub2e4. \uae30\ud68c\uac00 \uc788\uc744 \ub54c\ub9c8\ub2e4 \uc774\uc774\ub294 \uacc4\uc18d \u2018\uc2dc\ubb34\u2019\uac00 \uc5b4\ub5a4 \uac83\uc778\uac00\ub97c \uacc4\uc18d\ud558\uc5ec \uc0c1\uc18c\ub85c\ub3c4 \uc62c\ub9ac\uae30\ub3c4 \ud558\uc600\ub2e4.", "paragraph_answer": "\uadf8\ub294 \uad00\ub8cc\uc0dd\ud65c \uc911\uc5d0\ub3c4 \uafb8\uc900\ud788 \uc774\ud669, \uc870\uc2dd, \uc131\ud63c, \uc815\ucca0 \ub4f1\uacfc \uc11c\uc2e0\uc744 \uc8fc\uace0\ubc1b\uc73c\uba70 \ud559\ubb38\uc744 \uc5f0\uad6c\ud558\uc600\ub2e4. 1568\ub144(\uc120\uc870 1\ub144) \ucc9c\ucd94\uc0ac(\u5343\u79cb\u4f7f)\uac00 \uba85\ub098\ub77c\ub85c \uac08 \ub54c \uc0ac\uc2e0\uc758 \uc11c\uc7a5\uad00(\u66f8\u72c0\u5b98)\uc73c\ub85c \uc5f0\uacbd\uc5d0 \ub3d9\ud589\ud55c \ub4a4, 1569\ub144 \uadc0\uad6d \ud6c4, \ud64d\ubb38\uad00\ubd80\uad50\ub9ac\uac00 \ub418\uc5c8\ub2e4. \uace7 \ud64d\ubb38\uad00\ubd80\uad50\ub9ac\ub85c \ucd98\ucd94\uad00\uae30\uc0ac\uad00\uc5d0 \uacb8\uc784\ub418\uc5c8\uc73c\uba70, \u300a \uba85\uc885\uc2e4\ub85d \u300b\uc758 \ud3b8\ucc2c\uc5d0 \ucc38\uc5ec\ud588\ub2e4. \uc774\ub54c \uadf8\ub294 \ud1f4\uccad\ud6c4 \uc624\ub79c \uce5c\uad6c\uc778 \uc131\ud63c\uacfc '\uc9c0\uc120\uc5ec\uc911(\u81f3\u5584\u8207\u4e2d)' \ubc0f '\uc548\uc790\uaca9\uce58\uc131\uc815\uc9c0\uc124(\u9854\u5b50\u683c\u81f4\u8aa0\u6b63\u4e4b\u8aaa)' \ub4f1 \uc8fc\uc790\ud559\uc758 \uadfc\ubcf8\ubb38\uc81c\ub4e4\uc744 \ub193\uace0 \uc11c\uc2e0\uc73c\ub85c \uc7a5\uae30\uac04 \ud1a0\ub860\ud558\uc600\ub2e4. \uadf8\ud574 \ud64d\ubb38\uad00\uad50\ub9ac\ub97c \uc9c0\ub0c8\uc73c\uba70, 1569\ub144 \uc74c\ub825 9\uc6d4\uc5d0\ub294 \uc1a1\uac15 \uc815\ucca0\uacfc \ud568\uaed8 \u300a\ub3d9\ud638\ubb38\ub2f5\u300b(\u6771\u6e56\u554f\u7b54)\uc774\ub77c\uace0 \ud558\ub294 \ucc45\uc744 \uc368\uc11c \uc120\uc870\uc5d0\uac8c \uc62c\ub838\ub2e4. \uadf8 \ubb34\ub835\uc5d0 \uac00\uc7a5 \uad00\uc2ec \uac16\uace0 \ucd94\uc9c4\ud574\uc57c \ud560 \uc77c\uc774 \ubb34\uc5c7\uc778\uac00\ub97c \u2018\uc2dc\ubb34\u2019(\u6642\u52d9)\uc640 \u2018\ubb34\uc2e4\u2019(\u52d9\u5be6)\uc774\ub77c\ub294 \uc6a9\uc5b4\ub97c \uc0ac\uc6a9\ud558\uba70, \uae09\uc120\ubb34\ub85c \ud574\uacb0\ud574\uc57c \ud558\ub294 \uc815\uce58\uac00 \uc5b4\ub5a4 \uac83\uc778\uac00\ub97c \uba85\ud655\ud788 \ubc1d\ud614\ub2e4. \uae30\ud68c\uac00 \uc788\uc744 \ub54c\ub9c8\ub2e4 \uc774\uc774\ub294 \uacc4\uc18d \u2018\uc2dc\ubb34\u2019\uac00 \uc5b4\ub5a4 \uac83\uc778\uac00\ub97c \uacc4\uc18d\ud558\uc5ec \uc0c1\uc18c\ub85c\ub3c4 \uc62c\ub9ac\uae30\ub3c4 \ud558\uc600\ub2e4.", "sentence_answer": "\uace7 \ud64d\ubb38\uad00\ubd80\uad50\ub9ac\ub85c \ucd98\ucd94\uad00\uae30\uc0ac\uad00\uc5d0 \uacb8\uc784\ub418\uc5c8\uc73c\uba70, \u300a \uba85\uc885\uc2e4\ub85d \u300b\uc758 \ud3b8\ucc2c\uc5d0 \ucc38\uc5ec\ud588\ub2e4.", "paragraph_id": "6545935-2-0", "paragraph_question": "question: \uc774\uc774\uac00 1569\ub144 \uadc0\uad6d \ud6c4 \ud3b8\ucc2c\uc5d0 \ucc38\uc5ec\ud55c \uc2e4\ub85d\uc740?, context: \uadf8\ub294 \uad00\ub8cc\uc0dd\ud65c \uc911\uc5d0\ub3c4 \uafb8\uc900\ud788 \uc774\ud669, \uc870\uc2dd, \uc131\ud63c, \uc815\ucca0 \ub4f1\uacfc \uc11c\uc2e0\uc744 \uc8fc\uace0\ubc1b\uc73c\uba70 \ud559\ubb38\uc744 \uc5f0\uad6c\ud558\uc600\ub2e4. 1568\ub144(\uc120\uc870 1\ub144) \ucc9c\ucd94\uc0ac(\u5343\u79cb\u4f7f)\uac00 \uba85\ub098\ub77c\ub85c \uac08 \ub54c \uc0ac\uc2e0\uc758 \uc11c\uc7a5\uad00(\u66f8\u72c0\u5b98)\uc73c\ub85c \uc5f0\uacbd\uc5d0 \ub3d9\ud589\ud55c \ub4a4, 1569\ub144 \uadc0\uad6d \ud6c4, \ud64d\ubb38\uad00\ubd80\uad50\ub9ac\uac00 \ub418\uc5c8\ub2e4. \uace7 \ud64d\ubb38\uad00\ubd80\uad50\ub9ac\ub85c \ucd98\ucd94\uad00\uae30\uc0ac\uad00\uc5d0 \uacb8\uc784\ub418\uc5c8\uc73c\uba70, \u300a\uba85\uc885\uc2e4\ub85d\u300b\uc758 \ud3b8\ucc2c\uc5d0 \ucc38\uc5ec\ud588\ub2e4. \uc774\ub54c \uadf8\ub294 \ud1f4\uccad\ud6c4 \uc624\ub79c \uce5c\uad6c\uc778 \uc131\ud63c\uacfc '\uc9c0\uc120\uc5ec\uc911(\u81f3\u5584\u8207\u4e2d)' \ubc0f '\uc548\uc790\uaca9\uce58\uc131\uc815\uc9c0\uc124(\u9854\u5b50\u683c\u81f4\u8aa0\u6b63\u4e4b\u8aaa)' \ub4f1 \uc8fc\uc790\ud559\uc758 \uadfc\ubcf8\ubb38\uc81c\ub4e4\uc744 \ub193\uace0 \uc11c\uc2e0\uc73c\ub85c \uc7a5\uae30\uac04 \ud1a0\ub860\ud558\uc600\ub2e4. \uadf8\ud574 \ud64d\ubb38\uad00\uad50\ub9ac\ub97c \uc9c0\ub0c8\uc73c\uba70, 1569\ub144 \uc74c\ub825 9\uc6d4\uc5d0\ub294 \uc1a1\uac15 \uc815\ucca0\uacfc \ud568\uaed8 \u300a\ub3d9\ud638\ubb38\ub2f5\u300b(\u6771\u6e56\u554f\u7b54)\uc774\ub77c\uace0 \ud558\ub294 \ucc45\uc744 \uc368\uc11c \uc120\uc870\uc5d0\uac8c \uc62c\ub838\ub2e4. \uadf8 \ubb34\ub835\uc5d0 \uac00\uc7a5 \uad00\uc2ec \uac16\uace0 \ucd94\uc9c4\ud574\uc57c \ud560 \uc77c\uc774 \ubb34\uc5c7\uc778\uac00\ub97c \u2018\uc2dc\ubb34\u2019(\u6642\u52d9)\uc640 \u2018\ubb34\uc2e4\u2019(\u52d9\u5be6)\uc774\ub77c\ub294 \uc6a9\uc5b4\ub97c \uc0ac\uc6a9\ud558\uba70, \uae09\uc120\ubb34\ub85c \ud574\uacb0\ud574\uc57c \ud558\ub294 \uc815\uce58\uac00 \uc5b4\ub5a4 \uac83\uc778\uac00\ub97c \uba85\ud655\ud788 \ubc1d\ud614\ub2e4. \uae30\ud68c\uac00 \uc788\uc744 \ub54c\ub9c8\ub2e4 \uc774\uc774\ub294 \uacc4\uc18d \u2018\uc2dc\ubb34\u2019\uac00 \uc5b4\ub5a4 \uac83\uc778\uac00\ub97c \uacc4\uc18d\ud558\uc5ec \uc0c1\uc18c\ub85c\ub3c4 \uc62c\ub9ac\uae30\ub3c4 \ud558\uc600\ub2e4."} {"question": "\uc655\uc804\uc758 \uc190\uc790\uc778 \ub9ac\uac00 \uc9c4\uc758 \ubb34\uc131\ud6c4\uac00 \ub418\uc5c8\ub2e4\uace0 \ud55c \ucc45\uc740?", "paragraph": "\u300a\uc2e0\ub2f9\uc11c\u300b(\u65b0\u5510\u66f8) \uc7ac\uc0c1\uc138\uacc4(\u5bb0\u76f8\u4e16\u7cfb) \ud45c2 \uc911\uc5d0\ub294 \uc655\uc804\uc758 \uc190\uc790\uc778 \ub9ac\uac00 \uc9c4\uc758 \ubb34\uc131\ud6c4(\u6b66\u57ce\u4faf)\uac00 \ub418\uc5c8\uace0 \uc6d0(\u5143) \u30fb \uc704(\u5a01)\ub77c\ub294 \uc544\ub4e4\uc744 \ub450\uc5c8\ub2e4\uace0 \ud55c\ub2e4. \uc544\ub4e4\ub4e4\uc740 \uc9c4\uc758 \uc804\ub780\uc744 \ud53c\ud574 \uc0b0\ub465(\u5c71\u6771)\uc73c\ub85c \uc774\uc8fc\ud588\uace0, \uadf8 \uba3c \ud6c4\uc190\ub4e4\uc774 \ud6c4\uc138\uc5d0 \ud55c(\u6f22)\uc758 \uc655\uae38(\u738b\u5409) \u30fb \uc655\uc900(\u738b\u99ff \u30fb \uc655\uc22d(\u738b\u5d07), \uc870\uc704(\u66f9\u9b4f)\uc758 \uc655\uc6c5(\u738b\u96c4), \uc9c4(\u664b)\uc758 \uc655\uc591(\u738b\u7965) \u30fb \uc655\ub3c4(\u738b\u5c0e) \u30fb \uc655\ub3c8(\u738b\u6566) \u30fb \uc655\ud76c\uc9c0(\u738b\u7fb2\u4e4b) \ub4f1\uc758 \uc778\ubb3c\uc744 \ubc30\ucd9c\ud558\uc600\ub2e4. \uc704\uc9c4 \ub0a8\ubd81\uc870 \uc2dc\ub300\uc5d0 \ubb38\ubc8c\ub85c\uc368 \uc774\ub984\uc744 \ub5a8\ucce4\ub358 \ub0ad\uc57c \uc655\uc528(\u746f\u90aa\u738b\u6c0f) \uc9d1\uc548\ub3c4 \uc655\uc804\uc758 \ud6c4\uc190\uc73c\ub85c \uc54c\ub824\uc838 \uc788\ub2e4. \uc989 \ub0ad\uc57c \uc655\uc528\ub294 \uc655\ub9ac\uc758 \uba3c \ud6c4\uc190\uc774 \ub418\ub294 \uac83\uc774\ub2e4. \ub2e4\ub9cc \ud55c\uc11c(\u6f22\u66f8) \uc655\uae38\uc804(\u738b\u5409\u4f1d)\uc5d0\ub294 \uc655\ub9ac\uc640 \uc655\uae38 \uc0ac\uc774\uc5d0 \ub300\ud55c \uad00\ub828\uc744 \uc801\uace0 \uc788\uc9c0 \uc54a\uc544\uc11c \u300a\uc2e0\ub2f9\uc11c\u300b\uc758 \uacc4\ub3c4\uc5d0 \ub300\ud574 \uc2e0\ub8b0\uc131\uc774 \uc758\uc2ec\ubc1b\uace0 \uc788\ub2e4.", "answer": "\uc2e0\ub2f9\uc11c", "sentence": "\u300a \uc2e0\ub2f9\uc11c \u300b(\u65b0\u5510\u66f8) \uc7ac\uc0c1\uc138\uacc4(\u5bb0\u76f8\u4e16\u7cfb) \ud45c2 \uc911\uc5d0\ub294 \uc655\uc804\uc758 \uc190\uc790\uc778 \ub9ac\uac00 \uc9c4\uc758 \ubb34\uc131\ud6c4(\u6b66\u57ce\u4faf)\uac00 \ub418\uc5c8\uace0 \uc6d0(\u5143) \u30fb \uc704(\u5a01)\ub77c\ub294 \uc544\ub4e4\uc744 \ub450\uc5c8\ub2e4\uace0 \ud55c\ub2e4.", "paragraph_sentence": " \u300a \uc2e0\ub2f9\uc11c \u300b(\u65b0\u5510\u66f8) \uc7ac\uc0c1\uc138\uacc4(\u5bb0\u76f8\u4e16\u7cfb) \ud45c2 \uc911\uc5d0\ub294 \uc655\uc804\uc758 \uc190\uc790\uc778 \ub9ac\uac00 \uc9c4\uc758 \ubb34\uc131\ud6c4(\u6b66\u57ce\u4faf)\uac00 \ub418\uc5c8\uace0 \uc6d0(\u5143) \u30fb \uc704(\u5a01)\ub77c\ub294 \uc544\ub4e4\uc744 \ub450\uc5c8\ub2e4\uace0 \ud55c\ub2e4. \uc544\ub4e4\ub4e4\uc740 \uc9c4\uc758 \uc804\ub780\uc744 \ud53c\ud574 \uc0b0\ub465(\u5c71\u6771)\uc73c\ub85c \uc774\uc8fc\ud588\uace0, \uadf8 \uba3c \ud6c4\uc190\ub4e4\uc774 \ud6c4\uc138\uc5d0 \ud55c(\u6f22)\uc758 \uc655\uae38(\u738b\u5409) \u30fb \uc655\uc900(\u738b\u99ff \u30fb \uc655\uc22d(\u738b\u5d07), \uc870\uc704(\u66f9\u9b4f)\uc758 \uc655\uc6c5(\u738b\u96c4), \uc9c4(\u664b)\uc758 \uc655\uc591(\u738b\u7965) \u30fb \uc655\ub3c4(\u738b\u5c0e) \u30fb \uc655\ub3c8(\u738b\u6566) \u30fb \uc655\ud76c\uc9c0(\u738b\u7fb2\u4e4b) \ub4f1\uc758 \uc778\ubb3c\uc744 \ubc30\ucd9c\ud558\uc600\ub2e4. \uc704\uc9c4 \ub0a8\ubd81\uc870 \uc2dc\ub300\uc5d0 \ubb38\ubc8c\ub85c\uc368 \uc774\ub984\uc744 \ub5a8\ucce4\ub358 \ub0ad\uc57c \uc655\uc528(\u746f\u90aa\u738b\u6c0f) \uc9d1\uc548\ub3c4 \uc655\uc804\uc758 \ud6c4\uc190\uc73c\ub85c \uc54c\ub824\uc838 \uc788\ub2e4. \uc989 \ub0ad\uc57c \uc655\uc528\ub294 \uc655\ub9ac\uc758 \uba3c \ud6c4\uc190\uc774 \ub418\ub294 \uac83\uc774\ub2e4. \ub2e4\ub9cc \ud55c\uc11c(\u6f22\u66f8) \uc655\uae38\uc804(\u738b\u5409\u4f1d)\uc5d0\ub294 \uc655\ub9ac\uc640 \uc655\uae38 \uc0ac\uc774\uc5d0 \ub300\ud55c \uad00\ub828\uc744 \uc801\uace0 \uc788\uc9c0 \uc54a\uc544\uc11c \u300a\uc2e0\ub2f9\uc11c\u300b\uc758 \uacc4\ub3c4\uc5d0 \ub300\ud574 \uc2e0\ub8b0\uc131\uc774 \uc758\uc2ec\ubc1b\uace0 \uc788\ub2e4.", "paragraph_answer": "\u300a \uc2e0\ub2f9\uc11c \u300b(\u65b0\u5510\u66f8) \uc7ac\uc0c1\uc138\uacc4(\u5bb0\u76f8\u4e16\u7cfb) \ud45c2 \uc911\uc5d0\ub294 \uc655\uc804\uc758 \uc190\uc790\uc778 \ub9ac\uac00 \uc9c4\uc758 \ubb34\uc131\ud6c4(\u6b66\u57ce\u4faf)\uac00 \ub418\uc5c8\uace0 \uc6d0(\u5143) \u30fb \uc704(\u5a01)\ub77c\ub294 \uc544\ub4e4\uc744 \ub450\uc5c8\ub2e4\uace0 \ud55c\ub2e4. \uc544\ub4e4\ub4e4\uc740 \uc9c4\uc758 \uc804\ub780\uc744 \ud53c\ud574 \uc0b0\ub465(\u5c71\u6771)\uc73c\ub85c \uc774\uc8fc\ud588\uace0, \uadf8 \uba3c \ud6c4\uc190\ub4e4\uc774 \ud6c4\uc138\uc5d0 \ud55c(\u6f22)\uc758 \uc655\uae38(\u738b\u5409) \u30fb \uc655\uc900(\u738b\u99ff \u30fb \uc655\uc22d(\u738b\u5d07), \uc870\uc704(\u66f9\u9b4f)\uc758 \uc655\uc6c5(\u738b\u96c4), \uc9c4(\u664b)\uc758 \uc655\uc591(\u738b\u7965) \u30fb \uc655\ub3c4(\u738b\u5c0e) \u30fb \uc655\ub3c8(\u738b\u6566) \u30fb \uc655\ud76c\uc9c0(\u738b\u7fb2\u4e4b) \ub4f1\uc758 \uc778\ubb3c\uc744 \ubc30\ucd9c\ud558\uc600\ub2e4. \uc704\uc9c4 \ub0a8\ubd81\uc870 \uc2dc\ub300\uc5d0 \ubb38\ubc8c\ub85c\uc368 \uc774\ub984\uc744 \ub5a8\ucce4\ub358 \ub0ad\uc57c \uc655\uc528(\u746f\u90aa\u738b\u6c0f) \uc9d1\uc548\ub3c4 \uc655\uc804\uc758 \ud6c4\uc190\uc73c\ub85c \uc54c\ub824\uc838 \uc788\ub2e4. \uc989 \ub0ad\uc57c \uc655\uc528\ub294 \uc655\ub9ac\uc758 \uba3c \ud6c4\uc190\uc774 \ub418\ub294 \uac83\uc774\ub2e4. \ub2e4\ub9cc \ud55c\uc11c(\u6f22\u66f8) \uc655\uae38\uc804(\u738b\u5409\u4f1d)\uc5d0\ub294 \uc655\ub9ac\uc640 \uc655\uae38 \uc0ac\uc774\uc5d0 \ub300\ud55c \uad00\ub828\uc744 \uc801\uace0 \uc788\uc9c0 \uc54a\uc544\uc11c \u300a\uc2e0\ub2f9\uc11c\u300b\uc758 \uacc4\ub3c4\uc5d0 \ub300\ud574 \uc2e0\ub8b0\uc131\uc774 \uc758\uc2ec\ubc1b\uace0 \uc788\ub2e4.", "sentence_answer": "\u300a \uc2e0\ub2f9\uc11c \u300b(\u65b0\u5510\u66f8) \uc7ac\uc0c1\uc138\uacc4(\u5bb0\u76f8\u4e16\u7cfb) \ud45c2 \uc911\uc5d0\ub294 \uc655\uc804\uc758 \uc190\uc790\uc778 \ub9ac\uac00 \uc9c4\uc758 \ubb34\uc131\ud6c4(\u6b66\u57ce\u4faf)\uac00 \ub418\uc5c8\uace0 \uc6d0(\u5143) \u30fb \uc704(\u5a01)\ub77c\ub294 \uc544\ub4e4\uc744 \ub450\uc5c8\ub2e4\uace0 \ud55c\ub2e4.", "paragraph_id": "6541547-2-1", "paragraph_question": "question: \uc655\uc804\uc758 \uc190\uc790\uc778 \ub9ac\uac00 \uc9c4\uc758 \ubb34\uc131\ud6c4\uac00 \ub418\uc5c8\ub2e4\uace0 \ud55c \ucc45\uc740?, context: \u300a\uc2e0\ub2f9\uc11c\u300b(\u65b0\u5510\u66f8) \uc7ac\uc0c1\uc138\uacc4(\u5bb0\u76f8\u4e16\u7cfb) \ud45c2 \uc911\uc5d0\ub294 \uc655\uc804\uc758 \uc190\uc790\uc778 \ub9ac\uac00 \uc9c4\uc758 \ubb34\uc131\ud6c4(\u6b66\u57ce\u4faf)\uac00 \ub418\uc5c8\uace0 \uc6d0(\u5143) \u30fb \uc704(\u5a01)\ub77c\ub294 \uc544\ub4e4\uc744 \ub450\uc5c8\ub2e4\uace0 \ud55c\ub2e4. \uc544\ub4e4\ub4e4\uc740 \uc9c4\uc758 \uc804\ub780\uc744 \ud53c\ud574 \uc0b0\ub465(\u5c71\u6771)\uc73c\ub85c \uc774\uc8fc\ud588\uace0, \uadf8 \uba3c \ud6c4\uc190\ub4e4\uc774 \ud6c4\uc138\uc5d0 \ud55c(\u6f22)\uc758 \uc655\uae38(\u738b\u5409) \u30fb \uc655\uc900(\u738b\u99ff \u30fb \uc655\uc22d(\u738b\u5d07), \uc870\uc704(\u66f9\u9b4f)\uc758 \uc655\uc6c5(\u738b\u96c4), \uc9c4(\u664b)\uc758 \uc655\uc591(\u738b\u7965) \u30fb \uc655\ub3c4(\u738b\u5c0e) \u30fb \uc655\ub3c8(\u738b\u6566) \u30fb \uc655\ud76c\uc9c0(\u738b\u7fb2\u4e4b) \ub4f1\uc758 \uc778\ubb3c\uc744 \ubc30\ucd9c\ud558\uc600\ub2e4. \uc704\uc9c4 \ub0a8\ubd81\uc870 \uc2dc\ub300\uc5d0 \ubb38\ubc8c\ub85c\uc368 \uc774\ub984\uc744 \ub5a8\ucce4\ub358 \ub0ad\uc57c \uc655\uc528(\u746f\u90aa\u738b\u6c0f) \uc9d1\uc548\ub3c4 \uc655\uc804\uc758 \ud6c4\uc190\uc73c\ub85c \uc54c\ub824\uc838 \uc788\ub2e4. \uc989 \ub0ad\uc57c \uc655\uc528\ub294 \uc655\ub9ac\uc758 \uba3c \ud6c4\uc190\uc774 \ub418\ub294 \uac83\uc774\ub2e4. \ub2e4\ub9cc \ud55c\uc11c(\u6f22\u66f8) \uc655\uae38\uc804(\u738b\u5409\u4f1d)\uc5d0\ub294 \uc655\ub9ac\uc640 \uc655\uae38 \uc0ac\uc774\uc5d0 \ub300\ud55c \uad00\ub828\uc744 \uc801\uace0 \uc788\uc9c0 \uc54a\uc544\uc11c \u300a\uc2e0\ub2f9\uc11c\u300b\uc758 \uacc4\ub3c4\uc5d0 \ub300\ud574 \uc2e0\ub8b0\uc131\uc774 \uc758\uc2ec\ubc1b\uace0 \uc788\ub2e4."} {"question": "\uc870\uc120\uc2dc\ub300 \uc608\uc870\uc5d0 \ub454 \uc885\uc774\ud488 \uad00\uc9c1\uc758 \uba85\uce6d\uc740?", "paragraph": "\uc774\uc900\uc6a9\ub3c4 6\uc6d4 22\uc77c\ubd80\ud130 24\uc77c\uae4c\uc9c0 \uc77c\ubcf8 \uacf5\uc0ac \uc624\ud1a0\ub9ac \uacf5\uc0ac\ub97c \uc124\ub4dd\ud558\uae30 \uc704\ud574 \uc77c\ubcf8\uacf5\uc0ac\uad00\uc744 \ub450 \ucc28\ub840 \ubc29\ubb38\ud558\uc600\ub2e4. \uadf8\ub7ec\ub098 \uc2a4\uae30\ubb34\ub77c \uc11c\uae30\uad00 \ub4f1 \uc77c\ubcf8 \uacf5\uc0ac\uad00 \uc9c1\uc6d0\ub4e4\uc758 \uac15\ub825\ud55c \ubc18\ub300\ub85c \ud765\uc120\ub300\uc6d0\uad70\uacfc \uc774\uc900\uc6a9\uc758 \uc758\ub3c4\ub294 \uc2e4\ud328\ud558\uc600\ub2e4. \ud765\uc120\ub300\uc6d0\uad70\uc740 6\uc6d4 24\uc77c \uc774\uc900\uc6a9\uc744 \ubcc4\uc785\uc9c1\uc5d0 \uc784\uba85\ud558\uc5ec \uace0\uc885\uacfc \uba85\uc131\ud669\ud6c4\uc5d0 \ub300\ud55c \uac10\uc2dc\ub97c \ud55c\uce35 \uac15\ud654\ud558\uc600\ub2e4. 7\uc6d4 \ucd08 \uace0\uc885\uc740 \uac11\uc624\uacbd\uc7a5\uc744 \ub2e8\ud589\ud55c\ub2e4. \uc774\ub54c\uc5d0\ub3c4 \uc774\uc900\uc6a9\uc740 \uafb8\uc900\ud788 \uc77c\ubcf8 \uacf5\uc0ac\uad00\uc744 \ubc29\ubb38\ud558\uc5ec \uba85\uc131\ud669\ud6c4 \ud3d0\ucd9c\uc758 \uc815\ub2f9\uc131\uc744 \uc5ed\uc124\ud558\uba70 \ud611\uc870\ud574\uc904 \uac83\uc744 \ubd80\ud0c1\ud588\ub2e4. 7\uc6d4 12\uc77c \uc608\uc870\ucc38\ud310(\u79ae\u66fa\u53c3\u5224)\uc5d0 \uc784\uba85\ub410\ub2e4. \uac11\uc624\uacbd\uc7a5\uc744 \uc804\ud6c4\ud574\uc11c \ub300\uc6d0\uad70\uacfc \uc774\uc900\uc6a9\uc740 \uba85\uc131\ud669\ud6c4\ub97c \uacf5\uaca9, \ud3d0\uc11c\uc778\ud558\ub824\ub294 \uc74c\ubaa8\ub97c \uafb8\uba84\uc73c\ub098 \uc77c\ubcf8 \uc601\uc0ac\uad00\uc5d0\uc11c \ud638\uc751\ud558\uc9c0 \uc54a\uc544 \uc218\ud3ec\ub85c \ub3cc\uc544\uac00\uace0 \ub9cc\ub2e4.", "answer": "\uc608\uc870\ucc38\ud310", "sentence": "7\uc6d4 12\uc77c \uc608\uc870\ucc38\ud310 (\u79ae\u66fa\u53c3\u5224)", "paragraph_sentence": "\uc774\uc900\uc6a9\ub3c4 6\uc6d4 22\uc77c\ubd80\ud130 24\uc77c\uae4c\uc9c0 \uc77c\ubcf8 \uacf5\uc0ac \uc624\ud1a0\ub9ac \uacf5\uc0ac\ub97c \uc124\ub4dd\ud558\uae30 \uc704\ud574 \uc77c\ubcf8\uacf5\uc0ac\uad00\uc744 \ub450 \ucc28\ub840 \ubc29\ubb38\ud558\uc600\ub2e4. \uadf8\ub7ec\ub098 \uc2a4\uae30\ubb34\ub77c \uc11c\uae30\uad00 \ub4f1 \uc77c\ubcf8 \uacf5\uc0ac\uad00 \uc9c1\uc6d0\ub4e4\uc758 \uac15\ub825\ud55c \ubc18\ub300\ub85c \ud765\uc120\ub300\uc6d0\uad70\uacfc \uc774\uc900\uc6a9\uc758 \uc758\ub3c4\ub294 \uc2e4\ud328\ud558\uc600\ub2e4. \ud765\uc120\ub300\uc6d0\uad70\uc740 6\uc6d4 24\uc77c \uc774\uc900\uc6a9\uc744 \ubcc4\uc785\uc9c1\uc5d0 \uc784\uba85\ud558\uc5ec \uace0\uc885\uacfc \uba85\uc131\ud669\ud6c4\uc5d0 \ub300\ud55c \uac10\uc2dc\ub97c \ud55c\uce35 \uac15\ud654\ud558\uc600\ub2e4. 7\uc6d4 \ucd08 \uace0\uc885\uc740 \uac11\uc624\uacbd\uc7a5\uc744 \ub2e8\ud589\ud55c\ub2e4. \uc774\ub54c\uc5d0\ub3c4 \uc774\uc900\uc6a9\uc740 \uafb8\uc900\ud788 \uc77c\ubcf8 \uacf5\uc0ac\uad00\uc744 \ubc29\ubb38\ud558\uc5ec \uba85\uc131\ud669\ud6c4 \ud3d0\ucd9c\uc758 \uc815\ub2f9\uc131\uc744 \uc5ed\uc124\ud558\uba70 \ud611\uc870\ud574\uc904 \uac83\uc744 \ubd80\ud0c1\ud588\ub2e4. 7\uc6d4 12\uc77c \uc608\uc870\ucc38\ud310 (\u79ae\u66fa\u53c3\u5224) \uc5d0 \uc784\uba85\ub410\ub2e4. \uac11\uc624\uacbd\uc7a5\uc744 \uc804\ud6c4\ud574\uc11c \ub300\uc6d0\uad70\uacfc \uc774\uc900\uc6a9\uc740 \uba85\uc131\ud669\ud6c4\ub97c \uacf5\uaca9, \ud3d0\uc11c\uc778\ud558\ub824\ub294 \uc74c\ubaa8\ub97c \uafb8\uba84\uc73c\ub098 \uc77c\ubcf8 \uc601\uc0ac\uad00\uc5d0\uc11c \ud638\uc751\ud558\uc9c0 \uc54a\uc544 \uc218\ud3ec\ub85c \ub3cc\uc544\uac00\uace0 \ub9cc\ub2e4.", "paragraph_answer": "\uc774\uc900\uc6a9\ub3c4 6\uc6d4 22\uc77c\ubd80\ud130 24\uc77c\uae4c\uc9c0 \uc77c\ubcf8 \uacf5\uc0ac \uc624\ud1a0\ub9ac \uacf5\uc0ac\ub97c \uc124\ub4dd\ud558\uae30 \uc704\ud574 \uc77c\ubcf8\uacf5\uc0ac\uad00\uc744 \ub450 \ucc28\ub840 \ubc29\ubb38\ud558\uc600\ub2e4. \uadf8\ub7ec\ub098 \uc2a4\uae30\ubb34\ub77c \uc11c\uae30\uad00 \ub4f1 \uc77c\ubcf8 \uacf5\uc0ac\uad00 \uc9c1\uc6d0\ub4e4\uc758 \uac15\ub825\ud55c \ubc18\ub300\ub85c \ud765\uc120\ub300\uc6d0\uad70\uacfc \uc774\uc900\uc6a9\uc758 \uc758\ub3c4\ub294 \uc2e4\ud328\ud558\uc600\ub2e4. \ud765\uc120\ub300\uc6d0\uad70\uc740 6\uc6d4 24\uc77c \uc774\uc900\uc6a9\uc744 \ubcc4\uc785\uc9c1\uc5d0 \uc784\uba85\ud558\uc5ec \uace0\uc885\uacfc \uba85\uc131\ud669\ud6c4\uc5d0 \ub300\ud55c \uac10\uc2dc\ub97c \ud55c\uce35 \uac15\ud654\ud558\uc600\ub2e4. 7\uc6d4 \ucd08 \uace0\uc885\uc740 \uac11\uc624\uacbd\uc7a5\uc744 \ub2e8\ud589\ud55c\ub2e4. \uc774\ub54c\uc5d0\ub3c4 \uc774\uc900\uc6a9\uc740 \uafb8\uc900\ud788 \uc77c\ubcf8 \uacf5\uc0ac\uad00\uc744 \ubc29\ubb38\ud558\uc5ec \uba85\uc131\ud669\ud6c4 \ud3d0\ucd9c\uc758 \uc815\ub2f9\uc131\uc744 \uc5ed\uc124\ud558\uba70 \ud611\uc870\ud574\uc904 \uac83\uc744 \ubd80\ud0c1\ud588\ub2e4. 7\uc6d4 12\uc77c \uc608\uc870\ucc38\ud310 (\u79ae\u66fa\u53c3\u5224)\uc5d0 \uc784\uba85\ub410\ub2e4. \uac11\uc624\uacbd\uc7a5\uc744 \uc804\ud6c4\ud574\uc11c \ub300\uc6d0\uad70\uacfc \uc774\uc900\uc6a9\uc740 \uba85\uc131\ud669\ud6c4\ub97c \uacf5\uaca9, \ud3d0\uc11c\uc778\ud558\ub824\ub294 \uc74c\ubaa8\ub97c \uafb8\uba84\uc73c\ub098 \uc77c\ubcf8 \uc601\uc0ac\uad00\uc5d0\uc11c \ud638\uc751\ud558\uc9c0 \uc54a\uc544 \uc218\ud3ec\ub85c \ub3cc\uc544\uac00\uace0 \ub9cc\ub2e4.", "sentence_answer": "7\uc6d4 12\uc77c \uc608\uc870\ucc38\ud310 (\u79ae\u66fa\u53c3\u5224)", "paragraph_id": "6164698-2-1", "paragraph_question": "question: \uc870\uc120\uc2dc\ub300 \uc608\uc870\uc5d0 \ub454 \uc885\uc774\ud488 \uad00\uc9c1\uc758 \uba85\uce6d\uc740?, context: \uc774\uc900\uc6a9\ub3c4 6\uc6d4 22\uc77c\ubd80\ud130 24\uc77c\uae4c\uc9c0 \uc77c\ubcf8 \uacf5\uc0ac \uc624\ud1a0\ub9ac \uacf5\uc0ac\ub97c \uc124\ub4dd\ud558\uae30 \uc704\ud574 \uc77c\ubcf8\uacf5\uc0ac\uad00\uc744 \ub450 \ucc28\ub840 \ubc29\ubb38\ud558\uc600\ub2e4. \uadf8\ub7ec\ub098 \uc2a4\uae30\ubb34\ub77c \uc11c\uae30\uad00 \ub4f1 \uc77c\ubcf8 \uacf5\uc0ac\uad00 \uc9c1\uc6d0\ub4e4\uc758 \uac15\ub825\ud55c \ubc18\ub300\ub85c \ud765\uc120\ub300\uc6d0\uad70\uacfc \uc774\uc900\uc6a9\uc758 \uc758\ub3c4\ub294 \uc2e4\ud328\ud558\uc600\ub2e4. \ud765\uc120\ub300\uc6d0\uad70\uc740 6\uc6d4 24\uc77c \uc774\uc900\uc6a9\uc744 \ubcc4\uc785\uc9c1\uc5d0 \uc784\uba85\ud558\uc5ec \uace0\uc885\uacfc \uba85\uc131\ud669\ud6c4\uc5d0 \ub300\ud55c \uac10\uc2dc\ub97c \ud55c\uce35 \uac15\ud654\ud558\uc600\ub2e4. 7\uc6d4 \ucd08 \uace0\uc885\uc740 \uac11\uc624\uacbd\uc7a5\uc744 \ub2e8\ud589\ud55c\ub2e4. \uc774\ub54c\uc5d0\ub3c4 \uc774\uc900\uc6a9\uc740 \uafb8\uc900\ud788 \uc77c\ubcf8 \uacf5\uc0ac\uad00\uc744 \ubc29\ubb38\ud558\uc5ec \uba85\uc131\ud669\ud6c4 \ud3d0\ucd9c\uc758 \uc815\ub2f9\uc131\uc744 \uc5ed\uc124\ud558\uba70 \ud611\uc870\ud574\uc904 \uac83\uc744 \ubd80\ud0c1\ud588\ub2e4. 7\uc6d4 12\uc77c \uc608\uc870\ucc38\ud310(\u79ae\u66fa\u53c3\u5224)\uc5d0 \uc784\uba85\ub410\ub2e4. \uac11\uc624\uacbd\uc7a5\uc744 \uc804\ud6c4\ud574\uc11c \ub300\uc6d0\uad70\uacfc \uc774\uc900\uc6a9\uc740 \uba85\uc131\ud669\ud6c4\ub97c \uacf5\uaca9, \ud3d0\uc11c\uc778\ud558\ub824\ub294 \uc74c\ubaa8\ub97c \uafb8\uba84\uc73c\ub098 \uc77c\ubcf8 \uc601\uc0ac\uad00\uc5d0\uc11c \ud638\uc751\ud558\uc9c0 \uc54a\uc544 \uc218\ud3ec\ub85c \ub3cc\uc544\uac00\uace0 \ub9cc\ub2e4."} {"question": "4.18 \uae30\ub150 \ub9c8\ub77c\ud1a4\uc774 \ucc98\uc74c \uc2dc\uc791\ub41c \ub144\ub3c4\ub294?", "paragraph": "\ub9e4\ub144 4\uc6d4 18\uc77c\uc5d0\ub294 4\u00b719 \ud601\uba85\uc758 \ub3c4\ud654\uc120\uc774 \ub410\ub358 4\u00b718 \uc758\uac70\ub97c \uae30\ub150\ud558\uae30 \uc704\ud574 4\u00b718 \uae30\ub150 \ub9c8\ub77c\ud1a4\uacfc \uad6c\uad6d\ub300\uc7a5\uc815\uc744 \uc9c4\ud589\ud55c\ub2e4. \ud5cc\ud654 \ud589\uc0ac\uc5d0 \uc774\uc5b4 \uc624\uc804\uc5d0\ub294 \uae30\ub150 \ub9c8\ub77c\ud1a4\uc774, \uc624\ud6c4\uc5d0\ub294 \uad6c\uad6d\ub300\uc7a5\uc815\uc774 \uc9c4\ud589\ub41c\ub2e4. \uae30\ub150 \ub9c8\ub77c\ud1a4\uc740 1969\ub144 \ub2f9\uc2dc \ucd1d\ud559\uc0dd\ud68c\uc5d0\uc11c \ucc98\uc74c\uc73c\ub85c \ub9cc\ub4e0 \uc774\ub798 \ud604\uc7ac\uae4c\uc9c0 \uafb8\uc900\ud788 \uc774\uc5b4\uc838 \ub0b4\ub824\uc624\uace0 \uc788\uc73c\uba70, \ud559\uad50\ub97c \ucd9c\ubc1c\ud558\uc5ec \uad6d\ub9bd 4\u00b719 \ubb18\uc9c0\ub97c \uac70\uccd0 \ub2e4\uc2dc \ud559\uad50\ub85c \ub3cc\uc544\uc624\ub294 \ucd1d 16.2km\uc758 \ucf54\uc2a4\ub85c \uad6c\uc131\ub3fc \uc788\ub2e4. \ucd5c\uc885 \ubaa9\uc801\uc9c0\uac00 \uad6d\ub9bd 4\u00b719 \ubb18\uc9c0\ub85c \uc124\uc815\ub418\uc5b4 \uc788\ub294 \uac83\uc5d0\ub294 4\u00b718\uc774 4\u00b719 \uc815\uc2e0\uc73c\ub85c \uc774\uc5b4\uc9c4\ub2e4\ub294 \uc758\ubbf8\uac00 \ub2f4\uaca8 \uc788\ub2e4. \uc774\uc640 \uac19\uc740 \ud589\uc0ac\ub294 1980\ub144\ub300 \uad70\uc0ac\uc815\uad8c \uc2dc\uc808\uc5d0\ub3c4 \uc774\uc5b4\uc838 \ub9c8\ub77c\ud1a4\uc774\ub77c\ub294 \ud615\uc2dd\uc744 \ube4c\ub824 \ubd88\uc758\uc5d0 \ub9de\uc11c \uc2f8\uc6b0\ub294 \uae30\ub2a5 \ub610\ud55c \ub2f4\ub2f9\ud558\uc600\ub2e4. \uad6c\uad6d\ub300\uc7a5\uc815\uc740 \uac01 \ub2e8\uacfc\ub300\ubcc4\ub85c \ubaa8\uc5ec \uad50\ub0b4 \uc911\uc559\uad11\uc7a5\uc744 \ucd9c\ubc1c\ud558\uc5ec \uc218\uc720\ub3d9 \uad6d\ub9bd 4\u00b719 \ubb18\uc9c0\uae4c\uc9c0 \ud589\uc9c4\ud55c \ub2e4\uc74c, 4\u00b719 \uae30\ub150\ud0d1\uc5d0\uc11c \ucc38\ubc30\ud55c \ud6c4 \ud559\uad50\ub85c \ub2e4\uc2dc \ub3cc\uc544\uc624\ub294 \ud615\uc2dd\uc744 \ub744\ub294 \ud589\uc0ac\ub85c, \uc774 \ud589\uc0ac \ub610\ud55c 1969\ub144\uc5d0 \ucc98\uc74c\uc73c\ub85c \uac1c\ucd5c\ub410\ub2e4. \ud589\uc0ac \ucd08\uae30\uc5d0\ub294 \uc2dc\ubbfc\ub4e4\uc758 \ubd88\ud3b8\uc744 \uc6b0\ub824\ud558\uc5ec \ucc38\uac00\uc790 \uc218\uc5d0 \uc81c\ud55c\uc744 \ub450\uc5c8\uc73c\ub098 1984\ub144\ubd80\ud130 \uc774 \uc81c\ud55c\uc774 \ud3d0\uc9c0\ub410\ub2e4. \uc774 \ud589\uc0ac\uc5d0\uc11c\uc758 \uad6c\ud638\ub294 \ud574\ub2f9\uc5f0\ub3c4\uc758 \uc774\uc288 \ub610\ub294 \uc2dc\ub300\uc801\uc778 \ubcc0\ud654\uac00 \ubc18\uc601\ub41c \uac83\uc73c\ub85c, \uc774\ub97c \uad6c\ud638\ubfd0\ub9cc \uc544\ub2c8\ub77c \ud31c\ud50c\ub81b\uc744 \ud1b5\ud574\uc11c\ub3c4 \uc2dc\ubbfc\ub4e4\uacfc \uacf5\uc720\ud558\uace0 \uc788\ub2e4. \uc120\ubc30\ub4e4\uc758 \uc815\uc2e0\uc744 \uacc4\uc2b9\ud55c\ub2e4\uae30\ubcf4\ub2e4\ub294 \uace0\ub300\uc0dd\uc774 \ub418\uae30 \uc704\ud55c \ud558\ub098\uc758 \uc808\ucc28 \uc815\ub3c4\ub85c \uc778\uc2dd\ub418\ub294 \uac83\uacfc \uad00\ub828\ud558\uc5ec \uc54c\ub9f9\uc774\uac00 \ube60\uc838 \uc788\ub2e4\ub294 \uc758\uacac\ub4e4\uc774 \uc874\uc7ac\ud558\ub294 \ubc18\uba74, \uafb8\uc900\ud788 \ud589\uc0ac\ub97c \uc774\uc5b4\ub098\uac00\uace0 \uc788\ub2e4\ub294 \uc810\uc744 \uae0d\uc815\uc801\uc73c\ub85c \ubcf4\ub294 \uc2dc\uac01 \ub610\ud55c \uc874\uc7ac\ud55c\ub2e4.", "answer": "1969\ub144", "sentence": "\uae30\ub150 \ub9c8\ub77c\ud1a4\uc740 1969\ub144 \ub2f9\uc2dc \ucd1d\ud559\uc0dd\ud68c\uc5d0\uc11c \ucc98\uc74c\uc73c\ub85c \ub9cc\ub4e0 \uc774\ub798 \ud604\uc7ac\uae4c\uc9c0 \uafb8\uc900\ud788 \uc774\uc5b4\uc838 \ub0b4\ub824\uc624\uace0 \uc788\uc73c\uba70, \ud559\uad50\ub97c \ucd9c\ubc1c\ud558\uc5ec \uad6d\ub9bd 4\u00b719 \ubb18\uc9c0\ub97c \uac70\uccd0 \ub2e4\uc2dc \ud559\uad50\ub85c \ub3cc\uc544\uc624\ub294 \ucd1d 16.2km\uc758 \ucf54\uc2a4\ub85c \uad6c\uc131\ub3fc \uc788\ub2e4.", "paragraph_sentence": "\ub9e4\ub144 4\uc6d4 18\uc77c\uc5d0\ub294 4\u00b719 \ud601\uba85\uc758 \ub3c4\ud654\uc120\uc774 \ub410\ub358 4\u00b718 \uc758\uac70\ub97c \uae30\ub150\ud558\uae30 \uc704\ud574 4\u00b718 \uae30\ub150 \ub9c8\ub77c\ud1a4\uacfc \uad6c\uad6d\ub300\uc7a5\uc815\uc744 \uc9c4\ud589\ud55c\ub2e4. \ud5cc\ud654 \ud589\uc0ac\uc5d0 \uc774\uc5b4 \uc624\uc804\uc5d0\ub294 \uae30\ub150 \ub9c8\ub77c\ud1a4\uc774, \uc624\ud6c4\uc5d0\ub294 \uad6c\uad6d\ub300\uc7a5\uc815\uc774 \uc9c4\ud589\ub41c\ub2e4. \uae30\ub150 \ub9c8\ub77c\ud1a4\uc740 1969\ub144 \ub2f9\uc2dc \ucd1d\ud559\uc0dd\ud68c\uc5d0\uc11c \ucc98\uc74c\uc73c\ub85c \ub9cc\ub4e0 \uc774\ub798 \ud604\uc7ac\uae4c\uc9c0 \uafb8\uc900\ud788 \uc774\uc5b4\uc838 \ub0b4\ub824\uc624\uace0 \uc788\uc73c\uba70, \ud559\uad50\ub97c \ucd9c\ubc1c\ud558\uc5ec \uad6d\ub9bd 4\u00b719 \ubb18\uc9c0\ub97c \uac70\uccd0 \ub2e4\uc2dc \ud559\uad50\ub85c \ub3cc\uc544\uc624\ub294 \ucd1d 16.2km\uc758 \ucf54\uc2a4\ub85c \uad6c\uc131\ub3fc \uc788\ub2e4. \ucd5c\uc885 \ubaa9\uc801\uc9c0\uac00 \uad6d\ub9bd 4\u00b719 \ubb18\uc9c0\ub85c \uc124\uc815\ub418\uc5b4 \uc788\ub294 \uac83\uc5d0\ub294 4\u00b718\uc774 4\u00b719 \uc815\uc2e0\uc73c\ub85c \uc774\uc5b4\uc9c4\ub2e4\ub294 \uc758\ubbf8\uac00 \ub2f4\uaca8 \uc788\ub2e4. \uc774\uc640 \uac19\uc740 \ud589\uc0ac\ub294 1980\ub144\ub300 \uad70\uc0ac\uc815\uad8c \uc2dc\uc808\uc5d0\ub3c4 \uc774\uc5b4\uc838 \ub9c8\ub77c\ud1a4\uc774\ub77c\ub294 \ud615\uc2dd\uc744 \ube4c\ub824 \ubd88\uc758\uc5d0 \ub9de\uc11c \uc2f8\uc6b0\ub294 \uae30\ub2a5 \ub610\ud55c \ub2f4\ub2f9\ud558\uc600\ub2e4. \uad6c\uad6d\ub300\uc7a5\uc815\uc740 \uac01 \ub2e8\uacfc\ub300\ubcc4\ub85c \ubaa8\uc5ec \uad50\ub0b4 \uc911\uc559\uad11\uc7a5\uc744 \ucd9c\ubc1c\ud558\uc5ec \uc218\uc720\ub3d9 \uad6d\ub9bd 4\u00b719 \ubb18\uc9c0\uae4c\uc9c0 \ud589\uc9c4\ud55c \ub2e4\uc74c, 4\u00b719 \uae30\ub150\ud0d1\uc5d0\uc11c \ucc38\ubc30\ud55c \ud6c4 \ud559\uad50\ub85c \ub2e4\uc2dc \ub3cc\uc544\uc624\ub294 \ud615\uc2dd\uc744 \ub744\ub294 \ud589\uc0ac\ub85c, \uc774 \ud589\uc0ac \ub610\ud55c 1969\ub144\uc5d0 \ucc98\uc74c\uc73c\ub85c \uac1c\ucd5c\ub410\ub2e4. \ud589\uc0ac \ucd08\uae30\uc5d0\ub294 \uc2dc\ubbfc\ub4e4\uc758 \ubd88\ud3b8\uc744 \uc6b0\ub824\ud558\uc5ec \ucc38\uac00\uc790 \uc218\uc5d0 \uc81c\ud55c\uc744 \ub450\uc5c8\uc73c\ub098 1984\ub144\ubd80\ud130 \uc774 \uc81c\ud55c\uc774 \ud3d0\uc9c0\ub410\ub2e4. \uc774 \ud589\uc0ac\uc5d0\uc11c\uc758 \uad6c\ud638\ub294 \ud574\ub2f9\uc5f0\ub3c4\uc758 \uc774\uc288 \ub610\ub294 \uc2dc\ub300\uc801\uc778 \ubcc0\ud654\uac00 \ubc18\uc601\ub41c \uac83\uc73c\ub85c, \uc774\ub97c \uad6c\ud638\ubfd0\ub9cc \uc544\ub2c8\ub77c \ud31c\ud50c\ub81b\uc744 \ud1b5\ud574\uc11c\ub3c4 \uc2dc\ubbfc\ub4e4\uacfc \uacf5\uc720\ud558\uace0 \uc788\ub2e4. \uc120\ubc30\ub4e4\uc758 \uc815\uc2e0\uc744 \uacc4\uc2b9\ud55c\ub2e4\uae30\ubcf4\ub2e4\ub294 \uace0\ub300\uc0dd\uc774 \ub418\uae30 \uc704\ud55c \ud558\ub098\uc758 \uc808\ucc28 \uc815\ub3c4\ub85c \uc778\uc2dd\ub418\ub294 \uac83\uacfc \uad00\ub828\ud558\uc5ec \uc54c\ub9f9\uc774\uac00 \ube60\uc838 \uc788\ub2e4\ub294 \uc758\uacac\ub4e4\uc774 \uc874\uc7ac\ud558\ub294 \ubc18\uba74, \uafb8\uc900\ud788 \ud589\uc0ac\ub97c \uc774\uc5b4\ub098\uac00\uace0 \uc788\ub2e4\ub294 \uc810\uc744 \uae0d\uc815\uc801\uc73c\ub85c \ubcf4\ub294 \uc2dc\uac01 \ub610\ud55c \uc874\uc7ac\ud55c\ub2e4.", "paragraph_answer": "\ub9e4\ub144 4\uc6d4 18\uc77c\uc5d0\ub294 4\u00b719 \ud601\uba85\uc758 \ub3c4\ud654\uc120\uc774 \ub410\ub358 4\u00b718 \uc758\uac70\ub97c \uae30\ub150\ud558\uae30 \uc704\ud574 4\u00b718 \uae30\ub150 \ub9c8\ub77c\ud1a4\uacfc \uad6c\uad6d\ub300\uc7a5\uc815\uc744 \uc9c4\ud589\ud55c\ub2e4. \ud5cc\ud654 \ud589\uc0ac\uc5d0 \uc774\uc5b4 \uc624\uc804\uc5d0\ub294 \uae30\ub150 \ub9c8\ub77c\ud1a4\uc774, \uc624\ud6c4\uc5d0\ub294 \uad6c\uad6d\ub300\uc7a5\uc815\uc774 \uc9c4\ud589\ub41c\ub2e4. \uae30\ub150 \ub9c8\ub77c\ud1a4\uc740 1969\ub144 \ub2f9\uc2dc \ucd1d\ud559\uc0dd\ud68c\uc5d0\uc11c \ucc98\uc74c\uc73c\ub85c \ub9cc\ub4e0 \uc774\ub798 \ud604\uc7ac\uae4c\uc9c0 \uafb8\uc900\ud788 \uc774\uc5b4\uc838 \ub0b4\ub824\uc624\uace0 \uc788\uc73c\uba70, \ud559\uad50\ub97c \ucd9c\ubc1c\ud558\uc5ec \uad6d\ub9bd 4\u00b719 \ubb18\uc9c0\ub97c \uac70\uccd0 \ub2e4\uc2dc \ud559\uad50\ub85c \ub3cc\uc544\uc624\ub294 \ucd1d 16.2km\uc758 \ucf54\uc2a4\ub85c \uad6c\uc131\ub3fc \uc788\ub2e4. \ucd5c\uc885 \ubaa9\uc801\uc9c0\uac00 \uad6d\ub9bd 4\u00b719 \ubb18\uc9c0\ub85c \uc124\uc815\ub418\uc5b4 \uc788\ub294 \uac83\uc5d0\ub294 4\u00b718\uc774 4\u00b719 \uc815\uc2e0\uc73c\ub85c \uc774\uc5b4\uc9c4\ub2e4\ub294 \uc758\ubbf8\uac00 \ub2f4\uaca8 \uc788\ub2e4. \uc774\uc640 \uac19\uc740 \ud589\uc0ac\ub294 1980\ub144\ub300 \uad70\uc0ac\uc815\uad8c \uc2dc\uc808\uc5d0\ub3c4 \uc774\uc5b4\uc838 \ub9c8\ub77c\ud1a4\uc774\ub77c\ub294 \ud615\uc2dd\uc744 \ube4c\ub824 \ubd88\uc758\uc5d0 \ub9de\uc11c \uc2f8\uc6b0\ub294 \uae30\ub2a5 \ub610\ud55c \ub2f4\ub2f9\ud558\uc600\ub2e4. \uad6c\uad6d\ub300\uc7a5\uc815\uc740 \uac01 \ub2e8\uacfc\ub300\ubcc4\ub85c \ubaa8\uc5ec \uad50\ub0b4 \uc911\uc559\uad11\uc7a5\uc744 \ucd9c\ubc1c\ud558\uc5ec \uc218\uc720\ub3d9 \uad6d\ub9bd 4\u00b719 \ubb18\uc9c0\uae4c\uc9c0 \ud589\uc9c4\ud55c \ub2e4\uc74c, 4\u00b719 \uae30\ub150\ud0d1\uc5d0\uc11c \ucc38\ubc30\ud55c \ud6c4 \ud559\uad50\ub85c \ub2e4\uc2dc \ub3cc\uc544\uc624\ub294 \ud615\uc2dd\uc744 \ub744\ub294 \ud589\uc0ac\ub85c, \uc774 \ud589\uc0ac \ub610\ud55c 1969\ub144\uc5d0 \ucc98\uc74c\uc73c\ub85c \uac1c\ucd5c\ub410\ub2e4. \ud589\uc0ac \ucd08\uae30\uc5d0\ub294 \uc2dc\ubbfc\ub4e4\uc758 \ubd88\ud3b8\uc744 \uc6b0\ub824\ud558\uc5ec \ucc38\uac00\uc790 \uc218\uc5d0 \uc81c\ud55c\uc744 \ub450\uc5c8\uc73c\ub098 1984\ub144\ubd80\ud130 \uc774 \uc81c\ud55c\uc774 \ud3d0\uc9c0\ub410\ub2e4. \uc774 \ud589\uc0ac\uc5d0\uc11c\uc758 \uad6c\ud638\ub294 \ud574\ub2f9\uc5f0\ub3c4\uc758 \uc774\uc288 \ub610\ub294 \uc2dc\ub300\uc801\uc778 \ubcc0\ud654\uac00 \ubc18\uc601\ub41c \uac83\uc73c\ub85c, \uc774\ub97c \uad6c\ud638\ubfd0\ub9cc \uc544\ub2c8\ub77c \ud31c\ud50c\ub81b\uc744 \ud1b5\ud574\uc11c\ub3c4 \uc2dc\ubbfc\ub4e4\uacfc \uacf5\uc720\ud558\uace0 \uc788\ub2e4. \uc120\ubc30\ub4e4\uc758 \uc815\uc2e0\uc744 \uacc4\uc2b9\ud55c\ub2e4\uae30\ubcf4\ub2e4\ub294 \uace0\ub300\uc0dd\uc774 \ub418\uae30 \uc704\ud55c \ud558\ub098\uc758 \uc808\ucc28 \uc815\ub3c4\ub85c \uc778\uc2dd\ub418\ub294 \uac83\uacfc \uad00\ub828\ud558\uc5ec \uc54c\ub9f9\uc774\uac00 \ube60\uc838 \uc788\ub2e4\ub294 \uc758\uacac\ub4e4\uc774 \uc874\uc7ac\ud558\ub294 \ubc18\uba74, \uafb8\uc900\ud788 \ud589\uc0ac\ub97c \uc774\uc5b4\ub098\uac00\uace0 \uc788\ub2e4\ub294 \uc810\uc744 \uae0d\uc815\uc801\uc73c\ub85c \ubcf4\ub294 \uc2dc\uac01 \ub610\ud55c \uc874\uc7ac\ud55c\ub2e4.", "sentence_answer": "\uae30\ub150 \ub9c8\ub77c\ud1a4\uc740 1969\ub144 \ub2f9\uc2dc \ucd1d\ud559\uc0dd\ud68c\uc5d0\uc11c \ucc98\uc74c\uc73c\ub85c \ub9cc\ub4e0 \uc774\ub798 \ud604\uc7ac\uae4c\uc9c0 \uafb8\uc900\ud788 \uc774\uc5b4\uc838 \ub0b4\ub824\uc624\uace0 \uc788\uc73c\uba70, \ud559\uad50\ub97c \ucd9c\ubc1c\ud558\uc5ec \uad6d\ub9bd 4\u00b719 \ubb18\uc9c0\ub97c \uac70\uccd0 \ub2e4\uc2dc \ud559\uad50\ub85c \ub3cc\uc544\uc624\ub294 \ucd1d 16.2km\uc758 \ucf54\uc2a4\ub85c \uad6c\uc131\ub3fc \uc788\ub2e4.", "paragraph_id": "6533861-11-0", "paragraph_question": "question: 4.18 \uae30\ub150 \ub9c8\ub77c\ud1a4\uc774 \ucc98\uc74c \uc2dc\uc791\ub41c \ub144\ub3c4\ub294?, context: \ub9e4\ub144 4\uc6d4 18\uc77c\uc5d0\ub294 4\u00b719 \ud601\uba85\uc758 \ub3c4\ud654\uc120\uc774 \ub410\ub358 4\u00b718 \uc758\uac70\ub97c \uae30\ub150\ud558\uae30 \uc704\ud574 4\u00b718 \uae30\ub150 \ub9c8\ub77c\ud1a4\uacfc \uad6c\uad6d\ub300\uc7a5\uc815\uc744 \uc9c4\ud589\ud55c\ub2e4. \ud5cc\ud654 \ud589\uc0ac\uc5d0 \uc774\uc5b4 \uc624\uc804\uc5d0\ub294 \uae30\ub150 \ub9c8\ub77c\ud1a4\uc774, \uc624\ud6c4\uc5d0\ub294 \uad6c\uad6d\ub300\uc7a5\uc815\uc774 \uc9c4\ud589\ub41c\ub2e4. \uae30\ub150 \ub9c8\ub77c\ud1a4\uc740 1969\ub144 \ub2f9\uc2dc \ucd1d\ud559\uc0dd\ud68c\uc5d0\uc11c \ucc98\uc74c\uc73c\ub85c \ub9cc\ub4e0 \uc774\ub798 \ud604\uc7ac\uae4c\uc9c0 \uafb8\uc900\ud788 \uc774\uc5b4\uc838 \ub0b4\ub824\uc624\uace0 \uc788\uc73c\uba70, \ud559\uad50\ub97c \ucd9c\ubc1c\ud558\uc5ec \uad6d\ub9bd 4\u00b719 \ubb18\uc9c0\ub97c \uac70\uccd0 \ub2e4\uc2dc \ud559\uad50\ub85c \ub3cc\uc544\uc624\ub294 \ucd1d 16.2km\uc758 \ucf54\uc2a4\ub85c \uad6c\uc131\ub3fc \uc788\ub2e4. \ucd5c\uc885 \ubaa9\uc801\uc9c0\uac00 \uad6d\ub9bd 4\u00b719 \ubb18\uc9c0\ub85c \uc124\uc815\ub418\uc5b4 \uc788\ub294 \uac83\uc5d0\ub294 4\u00b718\uc774 4\u00b719 \uc815\uc2e0\uc73c\ub85c \uc774\uc5b4\uc9c4\ub2e4\ub294 \uc758\ubbf8\uac00 \ub2f4\uaca8 \uc788\ub2e4. \uc774\uc640 \uac19\uc740 \ud589\uc0ac\ub294 1980\ub144\ub300 \uad70\uc0ac\uc815\uad8c \uc2dc\uc808\uc5d0\ub3c4 \uc774\uc5b4\uc838 \ub9c8\ub77c\ud1a4\uc774\ub77c\ub294 \ud615\uc2dd\uc744 \ube4c\ub824 \ubd88\uc758\uc5d0 \ub9de\uc11c \uc2f8\uc6b0\ub294 \uae30\ub2a5 \ub610\ud55c \ub2f4\ub2f9\ud558\uc600\ub2e4. \uad6c\uad6d\ub300\uc7a5\uc815\uc740 \uac01 \ub2e8\uacfc\ub300\ubcc4\ub85c \ubaa8\uc5ec \uad50\ub0b4 \uc911\uc559\uad11\uc7a5\uc744 \ucd9c\ubc1c\ud558\uc5ec \uc218\uc720\ub3d9 \uad6d\ub9bd 4\u00b719 \ubb18\uc9c0\uae4c\uc9c0 \ud589\uc9c4\ud55c \ub2e4\uc74c, 4\u00b719 \uae30\ub150\ud0d1\uc5d0\uc11c \ucc38\ubc30\ud55c \ud6c4 \ud559\uad50\ub85c \ub2e4\uc2dc \ub3cc\uc544\uc624\ub294 \ud615\uc2dd\uc744 \ub744\ub294 \ud589\uc0ac\ub85c, \uc774 \ud589\uc0ac \ub610\ud55c 1969\ub144\uc5d0 \ucc98\uc74c\uc73c\ub85c \uac1c\ucd5c\ub410\ub2e4. \ud589\uc0ac \ucd08\uae30\uc5d0\ub294 \uc2dc\ubbfc\ub4e4\uc758 \ubd88\ud3b8\uc744 \uc6b0\ub824\ud558\uc5ec \ucc38\uac00\uc790 \uc218\uc5d0 \uc81c\ud55c\uc744 \ub450\uc5c8\uc73c\ub098 1984\ub144\ubd80\ud130 \uc774 \uc81c\ud55c\uc774 \ud3d0\uc9c0\ub410\ub2e4. \uc774 \ud589\uc0ac\uc5d0\uc11c\uc758 \uad6c\ud638\ub294 \ud574\ub2f9\uc5f0\ub3c4\uc758 \uc774\uc288 \ub610\ub294 \uc2dc\ub300\uc801\uc778 \ubcc0\ud654\uac00 \ubc18\uc601\ub41c \uac83\uc73c\ub85c, \uc774\ub97c \uad6c\ud638\ubfd0\ub9cc \uc544\ub2c8\ub77c \ud31c\ud50c\ub81b\uc744 \ud1b5\ud574\uc11c\ub3c4 \uc2dc\ubbfc\ub4e4\uacfc \uacf5\uc720\ud558\uace0 \uc788\ub2e4. \uc120\ubc30\ub4e4\uc758 \uc815\uc2e0\uc744 \uacc4\uc2b9\ud55c\ub2e4\uae30\ubcf4\ub2e4\ub294 \uace0\ub300\uc0dd\uc774 \ub418\uae30 \uc704\ud55c \ud558\ub098\uc758 \uc808\ucc28 \uc815\ub3c4\ub85c \uc778\uc2dd\ub418\ub294 \uac83\uacfc \uad00\ub828\ud558\uc5ec \uc54c\ub9f9\uc774\uac00 \ube60\uc838 \uc788\ub2e4\ub294 \uc758\uacac\ub4e4\uc774 \uc874\uc7ac\ud558\ub294 \ubc18\uba74, \uafb8\uc900\ud788 \ud589\uc0ac\ub97c \uc774\uc5b4\ub098\uac00\uace0 \uc788\ub2e4\ub294 \uc810\uc744 \uae0d\uc815\uc801\uc73c\ub85c \ubcf4\ub294 \uc2dc\uac01 \ub610\ud55c \uc874\uc7ac\ud55c\ub2e4."} {"question": "\uc5fd\ub85d\uccb4\uc758 \uc138\ud3ec\ub97c \uad00\ucc30\ud588\uc744 \ub54c \ubcf4\uc774\ub294 \uc0c9\uc740?", "paragraph": "\uc5fd\ub85d\uccb4\ub294 \uc5fd\ub85d\uc18c\uac00 \uc788\uc5b4 \uc138\ud3ec\ub97c \uad00\ucc30\ud588\uc744 \ub54c \ub69c\ub837\ud55c \ub179\uc0c9\uc774 \ub098\ud0c0\ub098\ub294 \uac83\uc744 \uc27d\uac8c \ud655\uc778\ud560 \uc218 \uc788\ub2e4. \uc138\ud3ec\ub0b4 \uacf5\uc0dd\uc124\uc5d0 \ub530\ub974\uba74 \uc5fd\ub85d\uccb4\ub294 \ubbf8\ud1a0\ucf58\ub4dc\ub9ac\uc544\uc640 \ud568\uaed8 \ub300\ud45c\uc801\uc778 \uc138\ud3ec\ub0b4 \uacf5\uc0dd\uccb4\uc774\ub2e4. (\ub3c5\uc790\uc801\uc778 DNA\uc640 \ub9ac\ubcf4\uc19c\uc744 \uac00\uc9c0\uace0 \uc788\uc73c\uba70, \ud604\uc7ac\uc758 \ub0a8\uc870\uade0\ub958\uc640 \uacf5\ud1b5 \uc870\uc0c1\uc744 \uac00\uc84c\uc744 \uac83\uc73c\ub85c \ucd94\uce21\ub41c\ub2e4.) \ube5b\uc5d0\ub108\uc9c0\ub97c \uc774\uc6a9\ud574 \ud3ec\ub3c4\ub2f9\uc744 \ud569\uc131\ud558\ub294 \uc138\ud3ec\ub0b4 \uc18c\uae30\uad00\uc73c\ub85c, \ub0b4\ub9c9\uacfc \uc678\ub9c9\uc73c\ub85c \uc774\ub8e8\uc5b4\uc9c4 \ud2b9\uc9d5\uc801\uc778 \ub0b4\ubd80 \uad6c\uc870, \uc989 \ud2f8\ub77c\ucf54\uc774\ub4dc(\ub0b4\ub9c9), \uadf8\ub77c\ub098(\ud2f8\ub77c\ucf54\uc774\ub4dc\ub9c9\uc774 \ub3d9\uc804\ucc98\ub7fc \uc313\uc778 \uad6c\uc870), \uc2a4\ud2b8\ub85c\ub9c8(\uae30\uc9c8)\ub97c \ud604\ubbf8\uacbd\uc73c\ub85c \uad00\ucc30\ud560 \uc218 \uc788\ub2e4. \ub3c5\ub9bd\uc801\uc73c\ub85c \ubd84\uc5f4\ud558\uc5ec \uc99d\uc2dd\ud558\uba70, \ube5b\uc744 \ubc1b\uae30 \uc804\uc5d0\ub294 \uc5fd\ub85d\uc18c\ub97c \ud569\uc131\ud558\uc9c0 \uc54a\uc544 \ubc31\uc0c9(\ubc31\uc0c9\uccb4)\uc774\uc9c0\ub9cc \uac00\uc2dc\uad11\uc120 \ud558\uc5d0\uc11c \uc5fd\ub85d\uc18c\ub97c \ud569\uc131\ud558\uc5ec \ub179\uc0c9\uc744 \ub760\uac8c \ub41c\ub2e4. \uc720\uad00\uc18d \uc2dd\ubb3c\uc758 \uc5fd\ub85d\uccb4\ub294 \ube44\uad11\ud569\uc131 \uc138\ud3ec\uc5d0\uc11c \uc0c9\uc18c\uccb4\ub85c\uc11c \uc874\uc7ac\ud55c\ub2e4. \uc0c9\uc18c\uccb4\uc5d0\ub294, \ub179\ub9d0\uccb4, \uc7a1\uc0c9\uccb4, \ubc31\uc0c9\uccb4 \ub4f1 \ub2e4\uc591\ud55c \uc885\ub958\uac00 \uc788\uc9c0\ub9cc, \ubaa8\ub4e0 \uc0c9\uc18c\uccb4\ub294, \uc774\uc911\ub9c9\uc73c\ub85c \ub458\ub7ec\uc2f8\uc5ec \uc5fd\ub85d\uccb4 DNA\ub97c \uac00\uc9c0\ub294 \uac83\uc774 \ud2b9\uc9d5\uc774\ub2e4.", "answer": "\ub179\uc0c9", "sentence": "\uc138\ud3ec\ub97c \uad00\ucc30\ud588\uc744 \ub54c \ub69c\ub837\ud55c \ub179\uc0c9 \uc774 \ub098\ud0c0\ub098\ub294 \uac83\uc744 \uc27d\uac8c \ud655\uc778\ud560 \uc218 \uc788\ub2e4.", "paragraph_sentence": "\uc5fd\ub85d\uccb4\ub294 \uc5fd\ub85d\uc18c\uac00 \uc788\uc5b4 \uc138\ud3ec\ub97c \uad00\ucc30\ud588\uc744 \ub54c \ub69c\ub837\ud55c \ub179\uc0c9 \uc774 \ub098\ud0c0\ub098\ub294 \uac83\uc744 \uc27d\uac8c \ud655\uc778\ud560 \uc218 \uc788\ub2e4. \uc138\ud3ec\ub0b4 \uacf5\uc0dd\uc124\uc5d0 \ub530\ub974\uba74 \uc5fd\ub85d\uccb4\ub294 \ubbf8\ud1a0\ucf58\ub4dc\ub9ac\uc544\uc640 \ud568\uaed8 \ub300\ud45c\uc801\uc778 \uc138\ud3ec\ub0b4 \uacf5\uc0dd\uccb4\uc774\ub2e4. (\ub3c5\uc790\uc801\uc778 DNA\uc640 \ub9ac\ubcf4\uc19c\uc744 \uac00\uc9c0\uace0 \uc788\uc73c\uba70, \ud604\uc7ac\uc758 \ub0a8\uc870\uade0\ub958\uc640 \uacf5\ud1b5 \uc870\uc0c1\uc744 \uac00\uc84c\uc744 \uac83\uc73c\ub85c \ucd94\uce21\ub41c\ub2e4.) \ube5b\uc5d0\ub108\uc9c0\ub97c \uc774\uc6a9\ud574 \ud3ec\ub3c4\ub2f9\uc744 \ud569\uc131\ud558\ub294 \uc138\ud3ec\ub0b4 \uc18c\uae30\uad00\uc73c\ub85c, \ub0b4\ub9c9\uacfc \uc678\ub9c9\uc73c\ub85c \uc774\ub8e8\uc5b4\uc9c4 \ud2b9\uc9d5\uc801\uc778 \ub0b4\ubd80 \uad6c\uc870, \uc989 \ud2f8\ub77c\ucf54\uc774\ub4dc(\ub0b4\ub9c9), \uadf8\ub77c\ub098(\ud2f8\ub77c\ucf54\uc774\ub4dc\ub9c9\uc774 \ub3d9\uc804\ucc98\ub7fc \uc313\uc778 \uad6c\uc870), \uc2a4\ud2b8\ub85c\ub9c8(\uae30\uc9c8)\ub97c \ud604\ubbf8\uacbd\uc73c\ub85c \uad00\ucc30\ud560 \uc218 \uc788\ub2e4. \ub3c5\ub9bd\uc801\uc73c\ub85c \ubd84\uc5f4\ud558\uc5ec \uc99d\uc2dd\ud558\uba70, \ube5b\uc744 \ubc1b\uae30 \uc804\uc5d0\ub294 \uc5fd\ub85d\uc18c\ub97c \ud569\uc131\ud558\uc9c0 \uc54a\uc544 \ubc31\uc0c9(\ubc31\uc0c9\uccb4)\uc774\uc9c0\ub9cc \uac00\uc2dc\uad11\uc120 \ud558\uc5d0\uc11c \uc5fd\ub85d\uc18c\ub97c \ud569\uc131\ud558\uc5ec \ub179\uc0c9\uc744 \ub760\uac8c \ub41c\ub2e4. \uc720\uad00\uc18d \uc2dd\ubb3c\uc758 \uc5fd\ub85d\uccb4\ub294 \ube44\uad11\ud569\uc131 \uc138\ud3ec\uc5d0\uc11c \uc0c9\uc18c\uccb4\ub85c\uc11c \uc874\uc7ac\ud55c\ub2e4. \uc0c9\uc18c\uccb4\uc5d0\ub294, \ub179\ub9d0\uccb4, \uc7a1\uc0c9\uccb4, \ubc31\uc0c9\uccb4 \ub4f1 \ub2e4\uc591\ud55c \uc885\ub958\uac00 \uc788\uc9c0\ub9cc, \ubaa8\ub4e0 \uc0c9\uc18c\uccb4\ub294, \uc774\uc911\ub9c9\uc73c\ub85c \ub458\ub7ec\uc2f8\uc5ec \uc5fd\ub85d\uccb4 DNA\ub97c \uac00\uc9c0\ub294 \uac83\uc774 \ud2b9\uc9d5\uc774\ub2e4.", "paragraph_answer": "\uc5fd\ub85d\uccb4\ub294 \uc5fd\ub85d\uc18c\uac00 \uc788\uc5b4 \uc138\ud3ec\ub97c \uad00\ucc30\ud588\uc744 \ub54c \ub69c\ub837\ud55c \ub179\uc0c9 \uc774 \ub098\ud0c0\ub098\ub294 \uac83\uc744 \uc27d\uac8c \ud655\uc778\ud560 \uc218 \uc788\ub2e4. \uc138\ud3ec\ub0b4 \uacf5\uc0dd\uc124\uc5d0 \ub530\ub974\uba74 \uc5fd\ub85d\uccb4\ub294 \ubbf8\ud1a0\ucf58\ub4dc\ub9ac\uc544\uc640 \ud568\uaed8 \ub300\ud45c\uc801\uc778 \uc138\ud3ec\ub0b4 \uacf5\uc0dd\uccb4\uc774\ub2e4. (\ub3c5\uc790\uc801\uc778 DNA\uc640 \ub9ac\ubcf4\uc19c\uc744 \uac00\uc9c0\uace0 \uc788\uc73c\uba70, \ud604\uc7ac\uc758 \ub0a8\uc870\uade0\ub958\uc640 \uacf5\ud1b5 \uc870\uc0c1\uc744 \uac00\uc84c\uc744 \uac83\uc73c\ub85c \ucd94\uce21\ub41c\ub2e4.) \ube5b\uc5d0\ub108\uc9c0\ub97c \uc774\uc6a9\ud574 \ud3ec\ub3c4\ub2f9\uc744 \ud569\uc131\ud558\ub294 \uc138\ud3ec\ub0b4 \uc18c\uae30\uad00\uc73c\ub85c, \ub0b4\ub9c9\uacfc \uc678\ub9c9\uc73c\ub85c \uc774\ub8e8\uc5b4\uc9c4 \ud2b9\uc9d5\uc801\uc778 \ub0b4\ubd80 \uad6c\uc870, \uc989 \ud2f8\ub77c\ucf54\uc774\ub4dc(\ub0b4\ub9c9), \uadf8\ub77c\ub098(\ud2f8\ub77c\ucf54\uc774\ub4dc\ub9c9\uc774 \ub3d9\uc804\ucc98\ub7fc \uc313\uc778 \uad6c\uc870), \uc2a4\ud2b8\ub85c\ub9c8(\uae30\uc9c8)\ub97c \ud604\ubbf8\uacbd\uc73c\ub85c \uad00\ucc30\ud560 \uc218 \uc788\ub2e4. \ub3c5\ub9bd\uc801\uc73c\ub85c \ubd84\uc5f4\ud558\uc5ec \uc99d\uc2dd\ud558\uba70, \ube5b\uc744 \ubc1b\uae30 \uc804\uc5d0\ub294 \uc5fd\ub85d\uc18c\ub97c \ud569\uc131\ud558\uc9c0 \uc54a\uc544 \ubc31\uc0c9(\ubc31\uc0c9\uccb4)\uc774\uc9c0\ub9cc \uac00\uc2dc\uad11\uc120 \ud558\uc5d0\uc11c \uc5fd\ub85d\uc18c\ub97c \ud569\uc131\ud558\uc5ec \ub179\uc0c9\uc744 \ub760\uac8c \ub41c\ub2e4. \uc720\uad00\uc18d \uc2dd\ubb3c\uc758 \uc5fd\ub85d\uccb4\ub294 \ube44\uad11\ud569\uc131 \uc138\ud3ec\uc5d0\uc11c \uc0c9\uc18c\uccb4\ub85c\uc11c \uc874\uc7ac\ud55c\ub2e4. \uc0c9\uc18c\uccb4\uc5d0\ub294, \ub179\ub9d0\uccb4, \uc7a1\uc0c9\uccb4, \ubc31\uc0c9\uccb4 \ub4f1 \ub2e4\uc591\ud55c \uc885\ub958\uac00 \uc788\uc9c0\ub9cc, \ubaa8\ub4e0 \uc0c9\uc18c\uccb4\ub294, \uc774\uc911\ub9c9\uc73c\ub85c \ub458\ub7ec\uc2f8\uc5ec \uc5fd\ub85d\uccb4 DNA\ub97c \uac00\uc9c0\ub294 \uac83\uc774 \ud2b9\uc9d5\uc774\ub2e4.", "sentence_answer": "\uc138\ud3ec\ub97c \uad00\ucc30\ud588\uc744 \ub54c \ub69c\ub837\ud55c \ub179\uc0c9 \uc774 \ub098\ud0c0\ub098\ub294 \uac83\uc744 \uc27d\uac8c \ud655\uc778\ud560 \uc218 \uc788\ub2e4.", "paragraph_id": "6587508-0-0", "paragraph_question": "question: \uc5fd\ub85d\uccb4\uc758 \uc138\ud3ec\ub97c \uad00\ucc30\ud588\uc744 \ub54c \ubcf4\uc774\ub294 \uc0c9\uc740?, context: \uc5fd\ub85d\uccb4\ub294 \uc5fd\ub85d\uc18c\uac00 \uc788\uc5b4 \uc138\ud3ec\ub97c \uad00\ucc30\ud588\uc744 \ub54c \ub69c\ub837\ud55c \ub179\uc0c9\uc774 \ub098\ud0c0\ub098\ub294 \uac83\uc744 \uc27d\uac8c \ud655\uc778\ud560 \uc218 \uc788\ub2e4. \uc138\ud3ec\ub0b4 \uacf5\uc0dd\uc124\uc5d0 \ub530\ub974\uba74 \uc5fd\ub85d\uccb4\ub294 \ubbf8\ud1a0\ucf58\ub4dc\ub9ac\uc544\uc640 \ud568\uaed8 \ub300\ud45c\uc801\uc778 \uc138\ud3ec\ub0b4 \uacf5\uc0dd\uccb4\uc774\ub2e4. (\ub3c5\uc790\uc801\uc778 DNA\uc640 \ub9ac\ubcf4\uc19c\uc744 \uac00\uc9c0\uace0 \uc788\uc73c\uba70, \ud604\uc7ac\uc758 \ub0a8\uc870\uade0\ub958\uc640 \uacf5\ud1b5 \uc870\uc0c1\uc744 \uac00\uc84c\uc744 \uac83\uc73c\ub85c \ucd94\uce21\ub41c\ub2e4.) \ube5b\uc5d0\ub108\uc9c0\ub97c \uc774\uc6a9\ud574 \ud3ec\ub3c4\ub2f9\uc744 \ud569\uc131\ud558\ub294 \uc138\ud3ec\ub0b4 \uc18c\uae30\uad00\uc73c\ub85c, \ub0b4\ub9c9\uacfc \uc678\ub9c9\uc73c\ub85c \uc774\ub8e8\uc5b4\uc9c4 \ud2b9\uc9d5\uc801\uc778 \ub0b4\ubd80 \uad6c\uc870, \uc989 \ud2f8\ub77c\ucf54\uc774\ub4dc(\ub0b4\ub9c9), \uadf8\ub77c\ub098(\ud2f8\ub77c\ucf54\uc774\ub4dc\ub9c9\uc774 \ub3d9\uc804\ucc98\ub7fc \uc313\uc778 \uad6c\uc870), \uc2a4\ud2b8\ub85c\ub9c8(\uae30\uc9c8)\ub97c \ud604\ubbf8\uacbd\uc73c\ub85c \uad00\ucc30\ud560 \uc218 \uc788\ub2e4. \ub3c5\ub9bd\uc801\uc73c\ub85c \ubd84\uc5f4\ud558\uc5ec \uc99d\uc2dd\ud558\uba70, \ube5b\uc744 \ubc1b\uae30 \uc804\uc5d0\ub294 \uc5fd\ub85d\uc18c\ub97c \ud569\uc131\ud558\uc9c0 \uc54a\uc544 \ubc31\uc0c9(\ubc31\uc0c9\uccb4)\uc774\uc9c0\ub9cc \uac00\uc2dc\uad11\uc120 \ud558\uc5d0\uc11c \uc5fd\ub85d\uc18c\ub97c \ud569\uc131\ud558\uc5ec \ub179\uc0c9\uc744 \ub760\uac8c \ub41c\ub2e4. \uc720\uad00\uc18d \uc2dd\ubb3c\uc758 \uc5fd\ub85d\uccb4\ub294 \ube44\uad11\ud569\uc131 \uc138\ud3ec\uc5d0\uc11c \uc0c9\uc18c\uccb4\ub85c\uc11c \uc874\uc7ac\ud55c\ub2e4. \uc0c9\uc18c\uccb4\uc5d0\ub294, \ub179\ub9d0\uccb4, \uc7a1\uc0c9\uccb4, \ubc31\uc0c9\uccb4 \ub4f1 \ub2e4\uc591\ud55c \uc885\ub958\uac00 \uc788\uc9c0\ub9cc, \ubaa8\ub4e0 \uc0c9\uc18c\uccb4\ub294, \uc774\uc911\ub9c9\uc73c\ub85c \ub458\ub7ec\uc2f8\uc5ec \uc5fd\ub85d\uccb4 DNA\ub97c \uac00\uc9c0\ub294 \uac83\uc774 \ud2b9\uc9d5\uc774\ub2e4."} {"question": "5.18\uc5d0\uc11c \uc218\ub9ce\uc740 \uc2dc\ubbfc\uc774 \uc601\ucc3d\uc73c\ub85c \ub118\uaca8\uc9c0\uae30 \uc804 \uc628\uac16 \uace0\ubb38\uc744 \ub2f9\ud55c \uacf3\uc740?", "paragraph": "5\u00b718\uc5d0\uc11c 3\ucc9c\uc5ec\uba85\uc5d0 \ub2ec\ud558\ub294 \uc218\ub9ce\uc740 \uc2dc\ubbfc\uc774 \uacc4\uc5c4\uad70\uc5d0 \uc758\ud574 \ud3ed\ud589 \ub2f9\ud558\uace0 \ud2b8\ub7ed\uc5d0 \uc2e4\ub824 \uad11\uc8fc\uad50\ub3c4\uc18c\u00b7\uc0c1\ubb34\ub300\uc5d0 \uc5f0\ud589\ub410\ub2e4. \uc5f0\ud589\uc790\ub294 \uc601\ucc3d\uc73c\ub85c \ub118\uaca8\uc9c0\uae30 \uc804 \ubcf4\uc548\ub300\uc5d0\uc11c \uc628\uac16 \uace0\ubb38\uc744 \ub2f9\ud588\ub2e4. 5\u00b718 \uacfc \uc0ac\uc2e4\uc0c1 \uc5f0\uad00\uc774 \uc5c6\ub294 \uae40\ub300\uc911\uacfc \uad00\ub828\ud558\uc5ec \ub0b4\ub780\uc74c\ubaa8 \uc870\uc791\uc774\ub77c\ub294 \uac01\ubcf8 \uc218\uc0ac\uac00 \uc774\ub8e8\uc5b4\uc84c\ub2e4. \uae40\ub300\uc911\uc73c\ub85c\ubd80\ud130 \uc790\uae08\uc744 \uc5bc\ub9c8 \ubc1b\uc558\ub290\ub0d0\ub294 \ud5c8\uc704\uc790\ubc31\uc744 \uac15\uc694\ud558\uba70, \uc794\uc778\ud55c \uace0\ubb38, \uad6c\ud0c0, \uc2ec\uc9c0\uc5b4 \uac19\uc740 \ub3d9\ub8cc\ub07c\ub9ac \ub54c\ub9ac\uac8c \ud558\ub294 \ube44\uc778\uaca9\uc801 \ubaa8\ub3c5 \ub4f1 \uc774\ub8e8 \ud5e4\uc544\ub9b4 \uc218 \uc5c6\ub294 \ud3ed\uac70\ub97c \uc790\ud589\ud588\ub2e4. \uace0\ubb38\uc774\ub098 \uad6c\ud0c0\ub97c \ub2f9\ud55c \uc0ac\ub78c\ub4e4\uc740 \uc11d\ubc29\uc774 \ub41c \ud6c4\uc5d0\ub3c4 \uc624\ub79c \uc2dc\uc77c \ub3d9\uc548 \ud6c4\uc720\uc99d\uc5d0 \uc2dc\ub2ec\ub824 \uc815\uc0c1\uc801\uc778 \uc0dd\ud65c\uc744 \ud558\uc9c0 \ubabb\ud588\uace0, \uc815\uc2e0\uc9c8\ud658\uc744 \uc553\ub2e4\uac00 \uc0ac\ub9dd\ud588\ub2e4. \uc774\ub4e4\uc740 \ud480\ub824\ub09c \ud6c4\uc5d0\ub3c4 \uc5c4\uccad\ub09c \uacf5\ud3ec\uc640 \ud53c\ud574\uc758\uc2dd\uc5d0 \uc0ac\ub85c\uc7a1\ud600 \uc228\uc8fd\uc774\uba70 \uc0b4\uc544\uc57c \ud588\ub2e4.", "answer": "\ubcf4\uc548\ub300", "sentence": "\uc5f0\ud589\uc790\ub294 \uc601\ucc3d\uc73c\ub85c \ub118\uaca8\uc9c0\uae30 \uc804 \ubcf4\uc548\ub300 \uc5d0\uc11c \uc628\uac16 \uace0\ubb38\uc744 \ub2f9\ud588\ub2e4.", "paragraph_sentence": "5\u00b718\uc5d0\uc11c 3\ucc9c\uc5ec\uba85\uc5d0 \ub2ec\ud558\ub294 \uc218\ub9ce\uc740 \uc2dc\ubbfc\uc774 \uacc4\uc5c4\uad70\uc5d0 \uc758\ud574 \ud3ed\ud589 \ub2f9\ud558\uace0 \ud2b8\ub7ed\uc5d0 \uc2e4\ub824 \uad11\uc8fc\uad50\ub3c4\uc18c\u00b7\uc0c1\ubb34\ub300\uc5d0 \uc5f0\ud589\ub410\ub2e4. \uc5f0\ud589\uc790\ub294 \uc601\ucc3d\uc73c\ub85c \ub118\uaca8\uc9c0\uae30 \uc804 \ubcf4\uc548\ub300 \uc5d0\uc11c \uc628\uac16 \uace0\ubb38\uc744 \ub2f9\ud588\ub2e4. 5\u00b718 \uacfc \uc0ac\uc2e4\uc0c1 \uc5f0\uad00\uc774 \uc5c6\ub294 \uae40\ub300\uc911\uacfc \uad00\ub828\ud558\uc5ec \ub0b4\ub780\uc74c\ubaa8 \uc870\uc791\uc774\ub77c\ub294 \uac01\ubcf8 \uc218\uc0ac\uac00 \uc774\ub8e8\uc5b4\uc84c\ub2e4. \uae40\ub300\uc911\uc73c\ub85c\ubd80\ud130 \uc790\uae08\uc744 \uc5bc\ub9c8 \ubc1b\uc558\ub290\ub0d0\ub294 \ud5c8\uc704\uc790\ubc31\uc744 \uac15\uc694\ud558\uba70, \uc794\uc778\ud55c \uace0\ubb38, \uad6c\ud0c0, \uc2ec\uc9c0\uc5b4 \uac19\uc740 \ub3d9\ub8cc\ub07c\ub9ac \ub54c\ub9ac\uac8c \ud558\ub294 \ube44\uc778\uaca9\uc801 \ubaa8\ub3c5 \ub4f1 \uc774\ub8e8 \ud5e4\uc544\ub9b4 \uc218 \uc5c6\ub294 \ud3ed\uac70\ub97c \uc790\ud589\ud588\ub2e4. \uace0\ubb38\uc774\ub098 \uad6c\ud0c0\ub97c \ub2f9\ud55c \uc0ac\ub78c\ub4e4\uc740 \uc11d\ubc29\uc774 \ub41c \ud6c4\uc5d0\ub3c4 \uc624\ub79c \uc2dc\uc77c \ub3d9\uc548 \ud6c4\uc720\uc99d\uc5d0 \uc2dc\ub2ec\ub824 \uc815\uc0c1\uc801\uc778 \uc0dd\ud65c\uc744 \ud558\uc9c0 \ubabb\ud588\uace0, \uc815\uc2e0\uc9c8\ud658\uc744 \uc553\ub2e4\uac00 \uc0ac\ub9dd\ud588\ub2e4. \uc774\ub4e4\uc740 \ud480\ub824\ub09c \ud6c4\uc5d0\ub3c4 \uc5c4\uccad\ub09c \uacf5\ud3ec\uc640 \ud53c\ud574\uc758\uc2dd\uc5d0 \uc0ac\ub85c\uc7a1\ud600 \uc228\uc8fd\uc774\uba70 \uc0b4\uc544\uc57c \ud588\ub2e4.", "paragraph_answer": "5\u00b718\uc5d0\uc11c 3\ucc9c\uc5ec\uba85\uc5d0 \ub2ec\ud558\ub294 \uc218\ub9ce\uc740 \uc2dc\ubbfc\uc774 \uacc4\uc5c4\uad70\uc5d0 \uc758\ud574 \ud3ed\ud589 \ub2f9\ud558\uace0 \ud2b8\ub7ed\uc5d0 \uc2e4\ub824 \uad11\uc8fc\uad50\ub3c4\uc18c\u00b7\uc0c1\ubb34\ub300\uc5d0 \uc5f0\ud589\ub410\ub2e4. \uc5f0\ud589\uc790\ub294 \uc601\ucc3d\uc73c\ub85c \ub118\uaca8\uc9c0\uae30 \uc804 \ubcf4\uc548\ub300 \uc5d0\uc11c \uc628\uac16 \uace0\ubb38\uc744 \ub2f9\ud588\ub2e4. 5\u00b718 \uacfc \uc0ac\uc2e4\uc0c1 \uc5f0\uad00\uc774 \uc5c6\ub294 \uae40\ub300\uc911\uacfc \uad00\ub828\ud558\uc5ec \ub0b4\ub780\uc74c\ubaa8 \uc870\uc791\uc774\ub77c\ub294 \uac01\ubcf8 \uc218\uc0ac\uac00 \uc774\ub8e8\uc5b4\uc84c\ub2e4. \uae40\ub300\uc911\uc73c\ub85c\ubd80\ud130 \uc790\uae08\uc744 \uc5bc\ub9c8 \ubc1b\uc558\ub290\ub0d0\ub294 \ud5c8\uc704\uc790\ubc31\uc744 \uac15\uc694\ud558\uba70, \uc794\uc778\ud55c \uace0\ubb38, \uad6c\ud0c0, \uc2ec\uc9c0\uc5b4 \uac19\uc740 \ub3d9\ub8cc\ub07c\ub9ac \ub54c\ub9ac\uac8c \ud558\ub294 \ube44\uc778\uaca9\uc801 \ubaa8\ub3c5 \ub4f1 \uc774\ub8e8 \ud5e4\uc544\ub9b4 \uc218 \uc5c6\ub294 \ud3ed\uac70\ub97c \uc790\ud589\ud588\ub2e4. \uace0\ubb38\uc774\ub098 \uad6c\ud0c0\ub97c \ub2f9\ud55c \uc0ac\ub78c\ub4e4\uc740 \uc11d\ubc29\uc774 \ub41c \ud6c4\uc5d0\ub3c4 \uc624\ub79c \uc2dc\uc77c \ub3d9\uc548 \ud6c4\uc720\uc99d\uc5d0 \uc2dc\ub2ec\ub824 \uc815\uc0c1\uc801\uc778 \uc0dd\ud65c\uc744 \ud558\uc9c0 \ubabb\ud588\uace0, \uc815\uc2e0\uc9c8\ud658\uc744 \uc553\ub2e4\uac00 \uc0ac\ub9dd\ud588\ub2e4. \uc774\ub4e4\uc740 \ud480\ub824\ub09c \ud6c4\uc5d0\ub3c4 \uc5c4\uccad\ub09c \uacf5\ud3ec\uc640 \ud53c\ud574\uc758\uc2dd\uc5d0 \uc0ac\ub85c\uc7a1\ud600 \uc228\uc8fd\uc774\uba70 \uc0b4\uc544\uc57c \ud588\ub2e4.", "sentence_answer": "\uc5f0\ud589\uc790\ub294 \uc601\ucc3d\uc73c\ub85c \ub118\uaca8\uc9c0\uae30 \uc804 \ubcf4\uc548\ub300 \uc5d0\uc11c \uc628\uac16 \uace0\ubb38\uc744 \ub2f9\ud588\ub2e4.", "paragraph_id": "6568325-18-0", "paragraph_question": "question: 5.18\uc5d0\uc11c \uc218\ub9ce\uc740 \uc2dc\ubbfc\uc774 \uc601\ucc3d\uc73c\ub85c \ub118\uaca8\uc9c0\uae30 \uc804 \uc628\uac16 \uace0\ubb38\uc744 \ub2f9\ud55c \uacf3\uc740?, context: 5\u00b718\uc5d0\uc11c 3\ucc9c\uc5ec\uba85\uc5d0 \ub2ec\ud558\ub294 \uc218\ub9ce\uc740 \uc2dc\ubbfc\uc774 \uacc4\uc5c4\uad70\uc5d0 \uc758\ud574 \ud3ed\ud589 \ub2f9\ud558\uace0 \ud2b8\ub7ed\uc5d0 \uc2e4\ub824 \uad11\uc8fc\uad50\ub3c4\uc18c\u00b7\uc0c1\ubb34\ub300\uc5d0 \uc5f0\ud589\ub410\ub2e4. \uc5f0\ud589\uc790\ub294 \uc601\ucc3d\uc73c\ub85c \ub118\uaca8\uc9c0\uae30 \uc804 \ubcf4\uc548\ub300\uc5d0\uc11c \uc628\uac16 \uace0\ubb38\uc744 \ub2f9\ud588\ub2e4. 5\u00b718 \uacfc \uc0ac\uc2e4\uc0c1 \uc5f0\uad00\uc774 \uc5c6\ub294 \uae40\ub300\uc911\uacfc \uad00\ub828\ud558\uc5ec \ub0b4\ub780\uc74c\ubaa8 \uc870\uc791\uc774\ub77c\ub294 \uac01\ubcf8 \uc218\uc0ac\uac00 \uc774\ub8e8\uc5b4\uc84c\ub2e4. \uae40\ub300\uc911\uc73c\ub85c\ubd80\ud130 \uc790\uae08\uc744 \uc5bc\ub9c8 \ubc1b\uc558\ub290\ub0d0\ub294 \ud5c8\uc704\uc790\ubc31\uc744 \uac15\uc694\ud558\uba70, \uc794\uc778\ud55c \uace0\ubb38, \uad6c\ud0c0, \uc2ec\uc9c0\uc5b4 \uac19\uc740 \ub3d9\ub8cc\ub07c\ub9ac \ub54c\ub9ac\uac8c \ud558\ub294 \ube44\uc778\uaca9\uc801 \ubaa8\ub3c5 \ub4f1 \uc774\ub8e8 \ud5e4\uc544\ub9b4 \uc218 \uc5c6\ub294 \ud3ed\uac70\ub97c \uc790\ud589\ud588\ub2e4. \uace0\ubb38\uc774\ub098 \uad6c\ud0c0\ub97c \ub2f9\ud55c \uc0ac\ub78c\ub4e4\uc740 \uc11d\ubc29\uc774 \ub41c \ud6c4\uc5d0\ub3c4 \uc624\ub79c \uc2dc\uc77c \ub3d9\uc548 \ud6c4\uc720\uc99d\uc5d0 \uc2dc\ub2ec\ub824 \uc815\uc0c1\uc801\uc778 \uc0dd\ud65c\uc744 \ud558\uc9c0 \ubabb\ud588\uace0, \uc815\uc2e0\uc9c8\ud658\uc744 \uc553\ub2e4\uac00 \uc0ac\ub9dd\ud588\ub2e4. \uc774\ub4e4\uc740 \ud480\ub824\ub09c \ud6c4\uc5d0\ub3c4 \uc5c4\uccad\ub09c \uacf5\ud3ec\uc640 \ud53c\ud574\uc758\uc2dd\uc5d0 \uc0ac\ub85c\uc7a1\ud600 \uc228\uc8fd\uc774\uba70 \uc0b4\uc544\uc57c \ud588\ub2e4."} {"question": "\uc5fd\ub85d\uccb4\uac00 \uc870\uc0c1\uc778 \uc2dc\uc544\ub178\ubc15\ud14c\ub9ac\uc544\ubcf4\ub2e4 \ub2e8\uc21c\ud55c \uac83\uc740?", "paragraph": "\ub0b4\ub9c9\uacfc \uc678\ub9c9\uc0ac\uc774\uc758 \uacf5\uac04\uc744 \ucc44\uc6b0\uace0 \uc788\ub294 \uc561\uccb4\ub97c \uae30\uc9c8(\uc2a4\ud2b8\ub85c\ub9c8)\uc774\ub77c\uace0 \ud558\uba70 \uad11\ud569\uc131\uc138\uade0\uc758 \uc138\ud3ec\uc9c8\uacfc \uac19\uc774 \uc6d0\ud615DNA\uc640 \ub9ac\ubcf4\uc19c\uc744 \uac00\uc9c0\uace0 \uc788\uc5b4 \uc790\uccb4\uc801\uc73c\ub85c \ub2e8\ubc31\uc9c8\uc744 \ub9cc\ub4e4 \uc218 \uc788\ub2e4. \uadf8\ub7ec\ub098 \uad11\ud569\uc131 \uc138\uade0\uacfc\ub294 \ub2e4\ub974\uac8c \uc138\ud3ec \ub0b4\uc5d0\uc11c \ubcf4\ud638\ub97c \ubc1b\uace0 \uc788\ub294 \uc5fd\ub85d\uccb4\ub294 \ud544\uc694\ud55c \ub2e8\ubc31\uc9c8\uc744 \uc790\uccb4\uc5d0\uc11c \ubaa8\ub450 \ub9cc\ub4e4\uc5b4 \ub0b4\uc9c0 \ubabb\ud558\uace0 \ud544\uc694\ud55c \ub2e8\ubc31\uc9c8 \uc911 \uc77c\ubd80\ub97c \ud575\uc758 DNA\ub97c \ud574\ub3c5\ud558\uc5ec \ub9cc\ub4e4\uc5b4\uc9c4 \ub2e8\ubc31\uc9c8\uc5d0 \uc758\uc874\ud55c\ub2e4. \uc774 \ub54c\ubb38\uc5d0 \uc5fd\ub85d\uccb4\ub294 \uc870\uc0c1\uc778 \uc2dc\uc544\ub178\ubc15\ud14c\ub9ac\uc544\ubcf4\ub2e4 \uac8c\ub188(genome)\uc774 \ub2e8\uc21c\ud558\ub2e4. \ub530\ub77c\uc11c \uc5fd\ub85d\uccb4\ub9cc \ubc30\uc591\uc744 \ud560 \uc218 \uc788\ub294 \uac83\uc740 \uac70\uc758 \uc5c6\ub2e4\uace0 \ud560 \uc218 \uc788\uaca0\ub2e4. \ud558\uc9c0\ub9cc \ub4dc\ubb3c\uac8c \uc5fd\ub85d\uccb4\uac00 \uc138\ud3ec\ub85c\ubd80\ud130 \ubd84\ub9ac\ub418\uc5b4 \ubc30\uc591\uac00\ub2a5\ud55c \uc608\ub3c4 \uc788\ub294\ub370, \ub0ad\uc124\ubaa9(\u56ca\u820c\u76ee, Sacoglossa)\uc758 \ud55c \uc885\ub958\ub294 \ubcf4\ud1b5\uc740 \uce58\uc124\ub85c \uac09\uc740 \ud574\uc870(\u6d77\u85fb)\ub97c \ubaa8\ub450 \uc18c\ud654\ud574 \ubc84\ub9ac\uc9c0\ub9cc, \ud574\uc870\uc758 \uc5fd\ub85d\uccb4\ub97c \uc790\uc2e0\uc758 \uc138\ud3ec\uc5d0 \ud3ec\ud68d\ud558\uae30\ub3c4 \ud558\uc5ec \ub3d9\ubb3c\uc784\uc5d0\ub3c4 \ubd88\uad6c\ud558\uace0 \uad11\ud569\uc131\uc744 \ud560 \uc218 \uc788\ub2e4\uace0 \uc54c\ub824\uc838 \uc788\ub2e4.", "answer": "\uac8c\ub188", "sentence": "\uc774 \ub54c\ubb38\uc5d0 \uc5fd\ub85d\uccb4\ub294 \uc870\uc0c1\uc778 \uc2dc\uc544\ub178\ubc15\ud14c\ub9ac\uc544\ubcf4\ub2e4 \uac8c\ub188 (genome)", "paragraph_sentence": "\ub0b4\ub9c9\uacfc \uc678\ub9c9\uc0ac\uc774\uc758 \uacf5\uac04\uc744 \ucc44\uc6b0\uace0 \uc788\ub294 \uc561\uccb4\ub97c \uae30\uc9c8(\uc2a4\ud2b8\ub85c\ub9c8)\uc774\ub77c\uace0 \ud558\uba70 \uad11\ud569\uc131\uc138\uade0\uc758 \uc138\ud3ec\uc9c8\uacfc \uac19\uc774 \uc6d0\ud615DNA\uc640 \ub9ac\ubcf4\uc19c\uc744 \uac00\uc9c0\uace0 \uc788\uc5b4 \uc790\uccb4\uc801\uc73c\ub85c \ub2e8\ubc31\uc9c8\uc744 \ub9cc\ub4e4 \uc218 \uc788\ub2e4. \uadf8\ub7ec\ub098 \uad11\ud569\uc131 \uc138\uade0\uacfc\ub294 \ub2e4\ub974\uac8c \uc138\ud3ec \ub0b4\uc5d0\uc11c \ubcf4\ud638\ub97c \ubc1b\uace0 \uc788\ub294 \uc5fd\ub85d\uccb4\ub294 \ud544\uc694\ud55c \ub2e8\ubc31\uc9c8\uc744 \uc790\uccb4\uc5d0\uc11c \ubaa8\ub450 \ub9cc\ub4e4\uc5b4 \ub0b4\uc9c0 \ubabb\ud558\uace0 \ud544\uc694\ud55c \ub2e8\ubc31\uc9c8 \uc911 \uc77c\ubd80\ub97c \ud575\uc758 DNA\ub97c \ud574\ub3c5\ud558\uc5ec \ub9cc\ub4e4\uc5b4\uc9c4 \ub2e8\ubc31\uc9c8\uc5d0 \uc758\uc874\ud55c\ub2e4. \uc774 \ub54c\ubb38\uc5d0 \uc5fd\ub85d\uccb4\ub294 \uc870\uc0c1\uc778 \uc2dc\uc544\ub178\ubc15\ud14c\ub9ac\uc544\ubcf4\ub2e4 \uac8c\ub188 (genome) \uc774 \ub2e8\uc21c\ud558\ub2e4. \ub530\ub77c\uc11c \uc5fd\ub85d\uccb4\ub9cc \ubc30\uc591\uc744 \ud560 \uc218 \uc788\ub294 \uac83\uc740 \uac70\uc758 \uc5c6\ub2e4\uace0 \ud560 \uc218 \uc788\uaca0\ub2e4. \ud558\uc9c0\ub9cc \ub4dc\ubb3c\uac8c \uc5fd\ub85d\uccb4\uac00 \uc138\ud3ec\ub85c\ubd80\ud130 \ubd84\ub9ac\ub418\uc5b4 \ubc30\uc591\uac00\ub2a5\ud55c \uc608\ub3c4 \uc788\ub294\ub370, \ub0ad\uc124\ubaa9(\u56ca\u820c\u76ee, Sacoglossa)\uc758 \ud55c \uc885\ub958\ub294 \ubcf4\ud1b5\uc740 \uce58\uc124\ub85c \uac09\uc740 \ud574\uc870(\u6d77\u85fb)\ub97c \ubaa8\ub450 \uc18c\ud654\ud574 \ubc84\ub9ac\uc9c0\ub9cc, \ud574\uc870\uc758 \uc5fd\ub85d\uccb4\ub97c \uc790\uc2e0\uc758 \uc138\ud3ec\uc5d0 \ud3ec\ud68d\ud558\uae30\ub3c4 \ud558\uc5ec \ub3d9\ubb3c\uc784\uc5d0\ub3c4 \ubd88\uad6c\ud558\uace0 \uad11\ud569\uc131\uc744 \ud560 \uc218 \uc788\ub2e4\uace0 \uc54c\ub824\uc838 \uc788\ub2e4.", "paragraph_answer": "\ub0b4\ub9c9\uacfc \uc678\ub9c9\uc0ac\uc774\uc758 \uacf5\uac04\uc744 \ucc44\uc6b0\uace0 \uc788\ub294 \uc561\uccb4\ub97c \uae30\uc9c8(\uc2a4\ud2b8\ub85c\ub9c8)\uc774\ub77c\uace0 \ud558\uba70 \uad11\ud569\uc131\uc138\uade0\uc758 \uc138\ud3ec\uc9c8\uacfc \uac19\uc774 \uc6d0\ud615DNA\uc640 \ub9ac\ubcf4\uc19c\uc744 \uac00\uc9c0\uace0 \uc788\uc5b4 \uc790\uccb4\uc801\uc73c\ub85c \ub2e8\ubc31\uc9c8\uc744 \ub9cc\ub4e4 \uc218 \uc788\ub2e4. \uadf8\ub7ec\ub098 \uad11\ud569\uc131 \uc138\uade0\uacfc\ub294 \ub2e4\ub974\uac8c \uc138\ud3ec \ub0b4\uc5d0\uc11c \ubcf4\ud638\ub97c \ubc1b\uace0 \uc788\ub294 \uc5fd\ub85d\uccb4\ub294 \ud544\uc694\ud55c \ub2e8\ubc31\uc9c8\uc744 \uc790\uccb4\uc5d0\uc11c \ubaa8\ub450 \ub9cc\ub4e4\uc5b4 \ub0b4\uc9c0 \ubabb\ud558\uace0 \ud544\uc694\ud55c \ub2e8\ubc31\uc9c8 \uc911 \uc77c\ubd80\ub97c \ud575\uc758 DNA\ub97c \ud574\ub3c5\ud558\uc5ec \ub9cc\ub4e4\uc5b4\uc9c4 \ub2e8\ubc31\uc9c8\uc5d0 \uc758\uc874\ud55c\ub2e4. \uc774 \ub54c\ubb38\uc5d0 \uc5fd\ub85d\uccb4\ub294 \uc870\uc0c1\uc778 \uc2dc\uc544\ub178\ubc15\ud14c\ub9ac\uc544\ubcf4\ub2e4 \uac8c\ub188 (genome)\uc774 \ub2e8\uc21c\ud558\ub2e4. \ub530\ub77c\uc11c \uc5fd\ub85d\uccb4\ub9cc \ubc30\uc591\uc744 \ud560 \uc218 \uc788\ub294 \uac83\uc740 \uac70\uc758 \uc5c6\ub2e4\uace0 \ud560 \uc218 \uc788\uaca0\ub2e4. \ud558\uc9c0\ub9cc \ub4dc\ubb3c\uac8c \uc5fd\ub85d\uccb4\uac00 \uc138\ud3ec\ub85c\ubd80\ud130 \ubd84\ub9ac\ub418\uc5b4 \ubc30\uc591\uac00\ub2a5\ud55c \uc608\ub3c4 \uc788\ub294\ub370, \ub0ad\uc124\ubaa9(\u56ca\u820c\u76ee, Sacoglossa)\uc758 \ud55c \uc885\ub958\ub294 \ubcf4\ud1b5\uc740 \uce58\uc124\ub85c \uac09\uc740 \ud574\uc870(\u6d77\u85fb)\ub97c \ubaa8\ub450 \uc18c\ud654\ud574 \ubc84\ub9ac\uc9c0\ub9cc, \ud574\uc870\uc758 \uc5fd\ub85d\uccb4\ub97c \uc790\uc2e0\uc758 \uc138\ud3ec\uc5d0 \ud3ec\ud68d\ud558\uae30\ub3c4 \ud558\uc5ec \ub3d9\ubb3c\uc784\uc5d0\ub3c4 \ubd88\uad6c\ud558\uace0 \uad11\ud569\uc131\uc744 \ud560 \uc218 \uc788\ub2e4\uace0 \uc54c\ub824\uc838 \uc788\ub2e4.", "sentence_answer": "\uc774 \ub54c\ubb38\uc5d0 \uc5fd\ub85d\uccb4\ub294 \uc870\uc0c1\uc778 \uc2dc\uc544\ub178\ubc15\ud14c\ub9ac\uc544\ubcf4\ub2e4 \uac8c\ub188 (genome)", "paragraph_id": "6524165-3-2", "paragraph_question": "question: \uc5fd\ub85d\uccb4\uac00 \uc870\uc0c1\uc778 \uc2dc\uc544\ub178\ubc15\ud14c\ub9ac\uc544\ubcf4\ub2e4 \ub2e8\uc21c\ud55c \uac83\uc740?, context: \ub0b4\ub9c9\uacfc \uc678\ub9c9\uc0ac\uc774\uc758 \uacf5\uac04\uc744 \ucc44\uc6b0\uace0 \uc788\ub294 \uc561\uccb4\ub97c \uae30\uc9c8(\uc2a4\ud2b8\ub85c\ub9c8)\uc774\ub77c\uace0 \ud558\uba70 \uad11\ud569\uc131\uc138\uade0\uc758 \uc138\ud3ec\uc9c8\uacfc \uac19\uc774 \uc6d0\ud615DNA\uc640 \ub9ac\ubcf4\uc19c\uc744 \uac00\uc9c0\uace0 \uc788\uc5b4 \uc790\uccb4\uc801\uc73c\ub85c \ub2e8\ubc31\uc9c8\uc744 \ub9cc\ub4e4 \uc218 \uc788\ub2e4. \uadf8\ub7ec\ub098 \uad11\ud569\uc131 \uc138\uade0\uacfc\ub294 \ub2e4\ub974\uac8c \uc138\ud3ec \ub0b4\uc5d0\uc11c \ubcf4\ud638\ub97c \ubc1b\uace0 \uc788\ub294 \uc5fd\ub85d\uccb4\ub294 \ud544\uc694\ud55c \ub2e8\ubc31\uc9c8\uc744 \uc790\uccb4\uc5d0\uc11c \ubaa8\ub450 \ub9cc\ub4e4\uc5b4 \ub0b4\uc9c0 \ubabb\ud558\uace0 \ud544\uc694\ud55c \ub2e8\ubc31\uc9c8 \uc911 \uc77c\ubd80\ub97c \ud575\uc758 DNA\ub97c \ud574\ub3c5\ud558\uc5ec \ub9cc\ub4e4\uc5b4\uc9c4 \ub2e8\ubc31\uc9c8\uc5d0 \uc758\uc874\ud55c\ub2e4. \uc774 \ub54c\ubb38\uc5d0 \uc5fd\ub85d\uccb4\ub294 \uc870\uc0c1\uc778 \uc2dc\uc544\ub178\ubc15\ud14c\ub9ac\uc544\ubcf4\ub2e4 \uac8c\ub188(genome)\uc774 \ub2e8\uc21c\ud558\ub2e4. \ub530\ub77c\uc11c \uc5fd\ub85d\uccb4\ub9cc \ubc30\uc591\uc744 \ud560 \uc218 \uc788\ub294 \uac83\uc740 \uac70\uc758 \uc5c6\ub2e4\uace0 \ud560 \uc218 \uc788\uaca0\ub2e4. \ud558\uc9c0\ub9cc \ub4dc\ubb3c\uac8c \uc5fd\ub85d\uccb4\uac00 \uc138\ud3ec\ub85c\ubd80\ud130 \ubd84\ub9ac\ub418\uc5b4 \ubc30\uc591\uac00\ub2a5\ud55c \uc608\ub3c4 \uc788\ub294\ub370, \ub0ad\uc124\ubaa9(\u56ca\u820c\u76ee, Sacoglossa)\uc758 \ud55c \uc885\ub958\ub294 \ubcf4\ud1b5\uc740 \uce58\uc124\ub85c \uac09\uc740 \ud574\uc870(\u6d77\u85fb)\ub97c \ubaa8\ub450 \uc18c\ud654\ud574 \ubc84\ub9ac\uc9c0\ub9cc, \ud574\uc870\uc758 \uc5fd\ub85d\uccb4\ub97c \uc790\uc2e0\uc758 \uc138\ud3ec\uc5d0 \ud3ec\ud68d\ud558\uae30\ub3c4 \ud558\uc5ec \ub3d9\ubb3c\uc784\uc5d0\ub3c4 \ubd88\uad6c\ud558\uace0 \uad11\ud569\uc131\uc744 \ud560 \uc218 \uc788\ub2e4\uace0 \uc54c\ub824\uc838 \uc788\ub2e4."} {"question": "\uc544\ub9ac\uc5d0 \ud63c\uc740 \uc5b4\ub290 \uac00\ubb38\uc758 \uac00\uc8fc\uc778\uac00?", "paragraph": "\uc790\ud558\ub4dc\uc640 \ud568\uaed8 \ud0d1\uc744 \uc62c\ub790\ub358 10\uba85\uc758 \uac00\uc8fc\ub4e4\uc774 \ub9cc\ub4e0 \uac00\ubb38\uc744 \ub73b\ud558\uba70, \ud0dc\uc5b4\ub0a0 \ub54c\ubd80\ud130 \uad00\ub9ac\uc790\uc640 \uacc4\uc57d\uc774 \ub418\uc5b4\uc788\uace0, \uc544\uc8fc \uac15\ub825\ud558\ub2e4. \ud2b9\ud788 \uac00\uc8fc\uc758 \uc790\ub140\ub4e4\uc740 \uc9c1\uacc4\ub77c\uace0 \ud558\uba70 \uac00\uc8fc\uc758 \ub2a5\ub825\uc744 \ubb3c\ub824\ubc1b\uc544 \ud2b9\ud788 \uac15\ud558\ub2e4. \uc544\ub9ac\uc5d0 \uac00\ubb38, \ucfe4 \uac00\ubb38, \ud558 \uac00\ubb38, \uc5f0 \uac00\ubb38, \uc720\ub77c\uc2dc\uc544 \uac00\ubb38, \ud5e8\ub3c4 \uac00\ubb38, \ud3ec \ube44\ub354 \uac00\ubb38, \ud22c \ud398\ub9ac \uac00\ubb38, \ub85c \ud3ec \ube44\uc544 \uac00\ubb38, \uc544\ub9ac \uac00\ubb38\uc774 \uc788\uc73c\uba70, \ucfe4 \uc544\uac8c\ub85c \uc544\uadf8\ub2c8\uc2a4, \ucfe4 \ub780, \ucfe4 \ud558\uce28\ub9c1, \ucfe4 \ub9c8\uc2a4\uccb4\ub2c8 \uc790\ud558\ub4dc(\ub178\ub780 \uc624\uc6d4 \uc18c\uc720), \ucfe4 \uc5d0\ub4dc\uc548(\ucfe4 \uac00\ubb38 \uac00\uc8fc), \uc5f0 \uc774\ud654 (\uc5f0\uac00\ubb38) \ud558 \uc720\ub77c, \ud558 \uc720\ub9ac \uc790\ud558\ub4dc, \ud558 \uc9c4\uc131, \ud558 \uc720\ub9b0(\ud558 \uac00\ubb38 \uac00\uc8fc), \ud3ec \ube44\ub354 \uad6c\uc2a4\ud2b8\uc559, \uc544\ub9ac\uc5d0 \ud63c \ub4f1\uc774 \uc18d\ud55c\ub2e4. \uac00\uc8fc\ub294 \uc544\ub9ac\uc5d0 \uac00\ubb38\uc758 \uac00\uc8fc '\uc544\ub9ac\uc5d0 \ud63c' \ucfe4 \uac00\ubb38\uc758 \uac00\uc8fc '\ucfe4 \uc5d0\ub4dc\uc548' \ud558\uac00\ubb38\uc758 \uac00\uc8fc '\ud558 \uc720\ub9b0' \uc720\ub77c\uc2dc\uc544 \uac00\ubb38\uc758 \uac00\uc8fc '\uc720\ub77c\uc2dc\uc544 \ube14\ub85c\uc12c' \ud578\ub3c4 \uac00\ubb38\uc758 \uac00\uc8fc '\ud578\ub3c4 \ub85d \ube14\ub7ec\ub4dc\uba54\ub7ec' \uc5f0 \uac00\ubb38\uc758 \uac00\uc8fc '\uc5f0 \ud55c\uc544' \ud3ec \ube44\ub354 \uac00\ubb38\uc758 \uac00\uc8fc '\ud3ec \ube44\ub354 \uad6c\uc2a4\ud2b8\uc559' \ud22c \ud398\ub9ac \uac00\ubb38\uc758 \uac00\uc8fc '\ud22c \ud398\ub9ac \ud2b8\ud398\ub9ac' \uc544\ub9ac \uac00\ubb38\uc758 '\uc544\ub9ac \ucc9c'(\uc544\ub9ac \ucc9c\uc740 3\uad70\uc8fc\uc774\uace0 \uc544\ub9ac \ud55c\uc774 \uac00\uc8fc\ub780 \uc18c\ubb38\uc774 \uc788\uc9c0\ub9cc \uc544\ub9ac \ucc9c\uc774 \uac00\uc8fc\uc77c \uac00\ub2a5\uc131\uc774 \ub354 \ub192\ub2e4) \ub85c \ud3ec \ube44\uc544 \uac00\ubb38\uc758 \uac00\uc8fc\ub294 \uc544\uc9c1 \ubc1d\ud600\uc9c0\uc9c0 \uc54a\uc558\ub2e4.", "answer": "\uc544\ub9ac\uc5d0 \uac00\ubb38", "sentence": "\uc544\ub9ac\uc5d0 \uac00\ubb38 ,", "paragraph_sentence": "\uc790\ud558\ub4dc\uc640 \ud568\uaed8 \ud0d1\uc744 \uc62c\ub790\ub358 10\uba85\uc758 \uac00\uc8fc\ub4e4\uc774 \ub9cc\ub4e0 \uac00\ubb38\uc744 \ub73b\ud558\uba70, \ud0dc\uc5b4\ub0a0 \ub54c\ubd80\ud130 \uad00\ub9ac\uc790\uc640 \uacc4\uc57d\uc774 \ub418\uc5b4\uc788\uace0, \uc544\uc8fc \uac15\ub825\ud558\ub2e4. \ud2b9\ud788 \uac00\uc8fc\uc758 \uc790\ub140\ub4e4\uc740 \uc9c1\uacc4\ub77c\uace0 \ud558\uba70 \uac00\uc8fc\uc758 \ub2a5\ub825\uc744 \ubb3c\ub824\ubc1b\uc544 \ud2b9\ud788 \uac15\ud558\ub2e4. \uc544\ub9ac\uc5d0 \uac00\ubb38 , \ucfe4 \uac00\ubb38, \ud558 \uac00\ubb38, \uc5f0 \uac00\ubb38, \uc720\ub77c\uc2dc\uc544 \uac00\ubb38, \ud5e8\ub3c4 \uac00\ubb38, \ud3ec \ube44\ub354 \uac00\ubb38, \ud22c \ud398\ub9ac \uac00\ubb38, \ub85c \ud3ec \ube44\uc544 \uac00\ubb38, \uc544\ub9ac \uac00\ubb38\uc774 \uc788\uc73c\uba70, \ucfe4 \uc544\uac8c\ub85c \uc544\uadf8\ub2c8\uc2a4, \ucfe4 \ub780, \ucfe4 \ud558\uce28\ub9c1, \ucfe4 \ub9c8\uc2a4\uccb4\ub2c8 \uc790\ud558\ub4dc(\ub178\ub780 \uc624\uc6d4 \uc18c\uc720), \ucfe4 \uc5d0\ub4dc\uc548(\ucfe4 \uac00\ubb38 \uac00\uc8fc), \uc5f0 \uc774\ud654 (\uc5f0\uac00\ubb38) \ud558 \uc720\ub77c, \ud558 \uc720\ub9ac \uc790\ud558\ub4dc, \ud558 \uc9c4\uc131, \ud558 \uc720\ub9b0(\ud558 \uac00\ubb38 \uac00\uc8fc), \ud3ec \ube44\ub354 \uad6c\uc2a4\ud2b8\uc559, \uc544\ub9ac\uc5d0 \ud63c \ub4f1\uc774 \uc18d\ud55c\ub2e4. \uac00\uc8fc\ub294 \uc544\ub9ac\uc5d0 \uac00\ubb38\uc758 \uac00\uc8fc '\uc544\ub9ac\uc5d0 \ud63c' \ucfe4 \uac00\ubb38\uc758 \uac00\uc8fc '\ucfe4 \uc5d0\ub4dc\uc548' \ud558\uac00\ubb38\uc758 \uac00\uc8fc '\ud558 \uc720\ub9b0' \uc720\ub77c\uc2dc\uc544 \uac00\ubb38\uc758 \uac00\uc8fc '\uc720\ub77c\uc2dc\uc544 \ube14\ub85c\uc12c' \ud578\ub3c4 \uac00\ubb38\uc758 \uac00\uc8fc '\ud578\ub3c4 \ub85d \ube14\ub7ec\ub4dc\uba54\ub7ec' \uc5f0 \uac00\ubb38\uc758 \uac00\uc8fc '\uc5f0 \ud55c\uc544' \ud3ec \ube44\ub354 \uac00\ubb38\uc758 \uac00\uc8fc '\ud3ec \ube44\ub354 \uad6c\uc2a4\ud2b8\uc559' \ud22c \ud398\ub9ac \uac00\ubb38\uc758 \uac00\uc8fc '\ud22c \ud398\ub9ac \ud2b8\ud398\ub9ac' \uc544\ub9ac \uac00\ubb38\uc758 '\uc544\ub9ac \ucc9c'(\uc544\ub9ac \ucc9c\uc740 3\uad70\uc8fc\uc774\uace0 \uc544\ub9ac \ud55c\uc774 \uac00\uc8fc\ub780 \uc18c\ubb38\uc774 \uc788\uc9c0\ub9cc \uc544\ub9ac \ucc9c\uc774 \uac00\uc8fc\uc77c \uac00\ub2a5\uc131\uc774 \ub354 \ub192\ub2e4) \ub85c \ud3ec \ube44\uc544 \uac00\ubb38\uc758 \uac00\uc8fc\ub294 \uc544\uc9c1 \ubc1d\ud600\uc9c0\uc9c0 \uc54a\uc558\ub2e4.", "paragraph_answer": "\uc790\ud558\ub4dc\uc640 \ud568\uaed8 \ud0d1\uc744 \uc62c\ub790\ub358 10\uba85\uc758 \uac00\uc8fc\ub4e4\uc774 \ub9cc\ub4e0 \uac00\ubb38\uc744 \ub73b\ud558\uba70, \ud0dc\uc5b4\ub0a0 \ub54c\ubd80\ud130 \uad00\ub9ac\uc790\uc640 \uacc4\uc57d\uc774 \ub418\uc5b4\uc788\uace0, \uc544\uc8fc \uac15\ub825\ud558\ub2e4. \ud2b9\ud788 \uac00\uc8fc\uc758 \uc790\ub140\ub4e4\uc740 \uc9c1\uacc4\ub77c\uace0 \ud558\uba70 \uac00\uc8fc\uc758 \ub2a5\ub825\uc744 \ubb3c\ub824\ubc1b\uc544 \ud2b9\ud788 \uac15\ud558\ub2e4. \uc544\ub9ac\uc5d0 \uac00\ubb38 , \ucfe4 \uac00\ubb38, \ud558 \uac00\ubb38, \uc5f0 \uac00\ubb38, \uc720\ub77c\uc2dc\uc544 \uac00\ubb38, \ud5e8\ub3c4 \uac00\ubb38, \ud3ec \ube44\ub354 \uac00\ubb38, \ud22c \ud398\ub9ac \uac00\ubb38, \ub85c \ud3ec \ube44\uc544 \uac00\ubb38, \uc544\ub9ac \uac00\ubb38\uc774 \uc788\uc73c\uba70, \ucfe4 \uc544\uac8c\ub85c \uc544\uadf8\ub2c8\uc2a4, \ucfe4 \ub780, \ucfe4 \ud558\uce28\ub9c1, \ucfe4 \ub9c8\uc2a4\uccb4\ub2c8 \uc790\ud558\ub4dc(\ub178\ub780 \uc624\uc6d4 \uc18c\uc720), \ucfe4 \uc5d0\ub4dc\uc548(\ucfe4 \uac00\ubb38 \uac00\uc8fc), \uc5f0 \uc774\ud654 (\uc5f0\uac00\ubb38) \ud558 \uc720\ub77c, \ud558 \uc720\ub9ac \uc790\ud558\ub4dc, \ud558 \uc9c4\uc131, \ud558 \uc720\ub9b0(\ud558 \uac00\ubb38 \uac00\uc8fc), \ud3ec \ube44\ub354 \uad6c\uc2a4\ud2b8\uc559, \uc544\ub9ac\uc5d0 \ud63c \ub4f1\uc774 \uc18d\ud55c\ub2e4. \uac00\uc8fc\ub294 \uc544\ub9ac\uc5d0 \uac00\ubb38\uc758 \uac00\uc8fc '\uc544\ub9ac\uc5d0 \ud63c' \ucfe4 \uac00\ubb38\uc758 \uac00\uc8fc '\ucfe4 \uc5d0\ub4dc\uc548' \ud558\uac00\ubb38\uc758 \uac00\uc8fc '\ud558 \uc720\ub9b0' \uc720\ub77c\uc2dc\uc544 \uac00\ubb38\uc758 \uac00\uc8fc '\uc720\ub77c\uc2dc\uc544 \ube14\ub85c\uc12c' \ud578\ub3c4 \uac00\ubb38\uc758 \uac00\uc8fc '\ud578\ub3c4 \ub85d \ube14\ub7ec\ub4dc\uba54\ub7ec' \uc5f0 \uac00\ubb38\uc758 \uac00\uc8fc '\uc5f0 \ud55c\uc544' \ud3ec \ube44\ub354 \uac00\ubb38\uc758 \uac00\uc8fc '\ud3ec \ube44\ub354 \uad6c\uc2a4\ud2b8\uc559' \ud22c \ud398\ub9ac \uac00\ubb38\uc758 \uac00\uc8fc '\ud22c \ud398\ub9ac \ud2b8\ud398\ub9ac' \uc544\ub9ac \uac00\ubb38\uc758 '\uc544\ub9ac \ucc9c'(\uc544\ub9ac \ucc9c\uc740 3\uad70\uc8fc\uc774\uace0 \uc544\ub9ac \ud55c\uc774 \uac00\uc8fc\ub780 \uc18c\ubb38\uc774 \uc788\uc9c0\ub9cc \uc544\ub9ac \ucc9c\uc774 \uac00\uc8fc\uc77c \uac00\ub2a5\uc131\uc774 \ub354 \ub192\ub2e4) \ub85c \ud3ec \ube44\uc544 \uac00\ubb38\uc758 \uac00\uc8fc\ub294 \uc544\uc9c1 \ubc1d\ud600\uc9c0\uc9c0 \uc54a\uc558\ub2e4.", "sentence_answer": " \uc544\ub9ac\uc5d0 \uac00\ubb38 ,", "paragraph_id": "6538013-2-1", "paragraph_question": "question: \uc544\ub9ac\uc5d0 \ud63c\uc740 \uc5b4\ub290 \uac00\ubb38\uc758 \uac00\uc8fc\uc778\uac00?, context: \uc790\ud558\ub4dc\uc640 \ud568\uaed8 \ud0d1\uc744 \uc62c\ub790\ub358 10\uba85\uc758 \uac00\uc8fc\ub4e4\uc774 \ub9cc\ub4e0 \uac00\ubb38\uc744 \ub73b\ud558\uba70, \ud0dc\uc5b4\ub0a0 \ub54c\ubd80\ud130 \uad00\ub9ac\uc790\uc640 \uacc4\uc57d\uc774 \ub418\uc5b4\uc788\uace0, \uc544\uc8fc \uac15\ub825\ud558\ub2e4. \ud2b9\ud788 \uac00\uc8fc\uc758 \uc790\ub140\ub4e4\uc740 \uc9c1\uacc4\ub77c\uace0 \ud558\uba70 \uac00\uc8fc\uc758 \ub2a5\ub825\uc744 \ubb3c\ub824\ubc1b\uc544 \ud2b9\ud788 \uac15\ud558\ub2e4. \uc544\ub9ac\uc5d0 \uac00\ubb38, \ucfe4 \uac00\ubb38, \ud558 \uac00\ubb38, \uc5f0 \uac00\ubb38, \uc720\ub77c\uc2dc\uc544 \uac00\ubb38, \ud5e8\ub3c4 \uac00\ubb38, \ud3ec \ube44\ub354 \uac00\ubb38, \ud22c \ud398\ub9ac \uac00\ubb38, \ub85c \ud3ec \ube44\uc544 \uac00\ubb38, \uc544\ub9ac \uac00\ubb38\uc774 \uc788\uc73c\uba70, \ucfe4 \uc544\uac8c\ub85c \uc544\uadf8\ub2c8\uc2a4, \ucfe4 \ub780, \ucfe4 \ud558\uce28\ub9c1, \ucfe4 \ub9c8\uc2a4\uccb4\ub2c8 \uc790\ud558\ub4dc(\ub178\ub780 \uc624\uc6d4 \uc18c\uc720), \ucfe4 \uc5d0\ub4dc\uc548(\ucfe4 \uac00\ubb38 \uac00\uc8fc), \uc5f0 \uc774\ud654 (\uc5f0\uac00\ubb38) \ud558 \uc720\ub77c, \ud558 \uc720\ub9ac \uc790\ud558\ub4dc, \ud558 \uc9c4\uc131, \ud558 \uc720\ub9b0(\ud558 \uac00\ubb38 \uac00\uc8fc), \ud3ec \ube44\ub354 \uad6c\uc2a4\ud2b8\uc559, \uc544\ub9ac\uc5d0 \ud63c \ub4f1\uc774 \uc18d\ud55c\ub2e4. \uac00\uc8fc\ub294 \uc544\ub9ac\uc5d0 \uac00\ubb38\uc758 \uac00\uc8fc '\uc544\ub9ac\uc5d0 \ud63c' \ucfe4 \uac00\ubb38\uc758 \uac00\uc8fc '\ucfe4 \uc5d0\ub4dc\uc548' \ud558\uac00\ubb38\uc758 \uac00\uc8fc '\ud558 \uc720\ub9b0' \uc720\ub77c\uc2dc\uc544 \uac00\ubb38\uc758 \uac00\uc8fc '\uc720\ub77c\uc2dc\uc544 \ube14\ub85c\uc12c' \ud578\ub3c4 \uac00\ubb38\uc758 \uac00\uc8fc '\ud578\ub3c4 \ub85d \ube14\ub7ec\ub4dc\uba54\ub7ec' \uc5f0 \uac00\ubb38\uc758 \uac00\uc8fc '\uc5f0 \ud55c\uc544' \ud3ec \ube44\ub354 \uac00\ubb38\uc758 \uac00\uc8fc '\ud3ec \ube44\ub354 \uad6c\uc2a4\ud2b8\uc559' \ud22c \ud398\ub9ac \uac00\ubb38\uc758 \uac00\uc8fc '\ud22c \ud398\ub9ac \ud2b8\ud398\ub9ac' \uc544\ub9ac \uac00\ubb38\uc758 '\uc544\ub9ac \ucc9c'(\uc544\ub9ac \ucc9c\uc740 3\uad70\uc8fc\uc774\uace0 \uc544\ub9ac \ud55c\uc774 \uac00\uc8fc\ub780 \uc18c\ubb38\uc774 \uc788\uc9c0\ub9cc \uc544\ub9ac \ucc9c\uc774 \uac00\uc8fc\uc77c \uac00\ub2a5\uc131\uc774 \ub354 \ub192\ub2e4) \ub85c \ud3ec \ube44\uc544 \uac00\ubb38\uc758 \uac00\uc8fc\ub294 \uc544\uc9c1 \ubc1d\ud600\uc9c0\uc9c0 \uc54a\uc558\ub2e4."} {"question": "\uc54c\ub85c\uc0ac\uc6b0\ub8e8\uc2a4\uc758 \ucc3d\uc0ac\uace8\uc774 \uc778\uc815 \ubc1b\uc740 \ud574\ub294?", "paragraph": "\uc54c\ub85c\uc0ac\uc6b0\ub8e8\uc2a4\ub294 \ubaa9\uc5d0 9\uac1c, \ub4f1\uc5d0 14\uac1c, \ucc9c\uace8\uc5d0 5\uac1c\uc758 \ucc99\ucd94\uace8\uc744 \uac00\uc9c0\uace0 \uc788\uc5c8\ub2e4. \uaf2c\ub9ac\uc758 \ucc99\ucd94\uace8 \uc218\ub294 \ubd88\uba85\uc774\uace0 \uac1c\uac1c\uc758 \ud06c\uae30\ub3c4 \ub2e4\uc591\ud588\ub2e4. \uc81c\uc784\uc2a4 \ub9e4\ub4dc\uc2a8\uc740 50\uac1c \uac00\ub7c9\uc73c\ub85c \uc608\uc0c1\ud588\uace0, \uac8c\uc624\ub974\uae30 \ud3f4\uc740 \uc774\uac83\ub3c4 \ub9ce\ub2e4\uace0 \uc5ec\uaca8\uc11c 45\uac1c\ub098 \uadf8 \uc774\ud558\ub85c \uc608\uce21\ud588\ub2e4. \ubaa9\uacfc \ub4f1\uc758 \ucc99\ucd94\uace8 \uc804\ubc29\uc5d0\ub294 \ube48 \uacf5\uac04\uc774 \uc788\uc5c8\uace0, \ud604\uc0dd \ub3d9\ubb3c\ub4e4(\uc0c8\ub77c\ub4e0\uc9c0)\uc5d0\uc11c\ub3c4 \ubc1c\uacac\ub418\ub294 \uc774\ub7f0 \uacf5\uac04\ub4e4\uc740 \ud638\ud761\uc5d0 \uc4f0\uc778 \uacf5\uae30\uc8fc\uba38\ub2c8\ub85c \ubcf4\uc778\ub2e4. \uac00\uc2b4\uc740 \ub113\uace0 \uc6d0\ud1b5\ud615 \ud749\uacfd\uc774\uc5c8\uace0, \ubcf5\ubd80 \ub291\uace8(\ubc30\uc758 \uac08\ube44)\ub97c \uac00\uc9c0\uace0 \uc788\uc5c8\uc9c0\ub9cc \uc774\uac83\ub4e4\uc740 \ud754\ud558\uac8c \ubc1c\uacac\ub418\ub294 \uac83\uc774 \uc544\ub2c8\uba70, \uc57d\uac04\uc758 \uace8\ud654 \ud604\uc0c1\uc774 \uc788\uc5c8\uc744 \uc218\ub3c4 \uc788\ub2e4. \ud55c \ubc1c\ud45c\uc5d0 \uc758\ud558\uba74, \ubcf5\ub291\uace8\uc740 \uac1c\uccb4\uac00 \uc0b4\uc544 \uc788\uc5c8\uc744 \ub54c \uc0c1\ud574\ub97c \uc785\uc5c8\ub2e4\ub294 \uc99d\uac70\uac00 \ub420 \uc218 \uc788\ub2e4. \ucc3d\uc0ac\uace8 (\uc77c\uba85 \uc704\uc2dc\ubcf8) \ub610\ud55c \uc874\uc7ac\ud588\uc9c0\ub9cc, 1996\ub144\uc774 \ub418\uc5b4\uc11c\uc57c \uc778\uc815\ubc1b\uc558\ub2e4. \uba87\uba87 \uacbd\uc6b0\uc5d0 \ucc3d\uc0ac\uace8\uc774 \ubcf5\ub291\uace8\uacfc \ud63c\ub3d9\ub418\uae30\ub3c4 \ud588\ub2e4. \uc5c9\ub369\uc774\uc758 \uc911\uc2ec \ubf08\uc778 \uc7a5\uace8\uc740 \uc721\uc911\ud588\uace0, \uce58\uace8\uc5d0\ub294 \ud280\uc5b4\ub098\uc628 \ub05d\ub2e8\uc774 \uc788\uc5c8\ub294\ub370 \uc774\ub294 \uadfc\uc721\uc744 \uc5f0\uacb0\uc2dc\ucf1c \uc8fc\ub294 \ub3d9\uc2dc\uc5d0 \ub2e4\ub9ac\uac00 \ub545\uc744 \ub51b\uace0 \uc11c \uc788\uc744 \uc218 \uc788\uac8c \ud574 \uc8fc\ub294 \ubc1b\uce68\ub300 \uc5ed\ud560\uc744 \ud588\ub2e4. \ub9e4\ub4dc\uc2a8\uc740 \ud074\ub9ac\ube14\ub79c\ub4dc \ub85c\uc774\ub4dc \ud0d0\uc0ac\uc5d0\uc11c \ubc1c\uacac\ub41c \uac1c\uccb4\ub4e4 \uc911 \uc57d \uc808\ubc18 \uc815\ub3c4\uac00 \ud06c\uae30\uc5d0 \uad00\uacc4\uc5c6\uc774 \ubc1c \ub05d\ubd80\ubd84\uc758 \uc5f0\ubaa8\uac00 \uc11c\ub85c \uacb0\ud569\ub418\uc5b4 \uc788\uc9c0 \uc54a\uc558\ub2e4\uace0 \uae30\uc220\ud588\ub2e4. \ub9e4\ub4dc\uc2a8\uc740 \uc774\uac83\uc774 \uc131\uc801\uc778 \ud2b9\uc9d5\uc77c \uac00\ub2a5\uc131\uc774 \uc788\uc73c\uba70, \uc774\ub294 \uc554\ucef7\uc774 \uc54c\uc744 \ud488\uae30 \uc27d\ub3c4\ub85d \ubf08\uac00 \ubd99\uc5b4 \uc788\uc9c0 \uc54a\uc740 \uac83\uc774\ub77c\uace0 \uc8fc\uc7a5\ud588\ub2e4. \uadf8\ub7ec\ub098 \uc774 \uc0dd\uac01\uc740 \uc774\ud6c4 \uc9c0\uc9c0\ub97c \ubcc4\ub85c \ubc1b\uc9c0 \ubabb\ud588\ub2e4.", "answer": "1996\ub144", "sentence": "\ucc3d\uc0ac\uace8 (\uc77c\uba85 \uc704\uc2dc\ubcf8) \ub610\ud55c \uc874\uc7ac\ud588\uc9c0\ub9cc, 1996\ub144 \uc774 \ub418\uc5b4\uc11c\uc57c \uc778\uc815\ubc1b\uc558\ub2e4.", "paragraph_sentence": "\uc54c\ub85c\uc0ac\uc6b0\ub8e8\uc2a4\ub294 \ubaa9\uc5d0 9\uac1c, \ub4f1\uc5d0 14\uac1c, \ucc9c\uace8\uc5d0 5\uac1c\uc758 \ucc99\ucd94\uace8\uc744 \uac00\uc9c0\uace0 \uc788\uc5c8\ub2e4. \uaf2c\ub9ac\uc758 \ucc99\ucd94\uace8 \uc218\ub294 \ubd88\uba85\uc774\uace0 \uac1c\uac1c\uc758 \ud06c\uae30\ub3c4 \ub2e4\uc591\ud588\ub2e4. \uc81c\uc784\uc2a4 \ub9e4\ub4dc\uc2a8\uc740 50\uac1c \uac00\ub7c9\uc73c\ub85c \uc608\uc0c1\ud588\uace0, \uac8c\uc624\ub974\uae30 \ud3f4\uc740 \uc774\uac83\ub3c4 \ub9ce\ub2e4\uace0 \uc5ec\uaca8\uc11c 45\uac1c\ub098 \uadf8 \uc774\ud558\ub85c \uc608\uce21\ud588\ub2e4. \ubaa9\uacfc \ub4f1\uc758 \ucc99\ucd94\uace8 \uc804\ubc29\uc5d0\ub294 \ube48 \uacf5\uac04\uc774 \uc788\uc5c8\uace0, \ud604\uc0dd \ub3d9\ubb3c\ub4e4(\uc0c8\ub77c\ub4e0\uc9c0)\uc5d0\uc11c\ub3c4 \ubc1c\uacac\ub418\ub294 \uc774\ub7f0 \uacf5\uac04\ub4e4\uc740 \ud638\ud761\uc5d0 \uc4f0\uc778 \uacf5\uae30\uc8fc\uba38\ub2c8\ub85c \ubcf4\uc778\ub2e4. \uac00\uc2b4\uc740 \ub113\uace0 \uc6d0\ud1b5\ud615 \ud749\uacfd\uc774\uc5c8\uace0, \ubcf5\ubd80 \ub291\uace8(\ubc30\uc758 \uac08\ube44)\ub97c \uac00\uc9c0\uace0 \uc788\uc5c8\uc9c0\ub9cc \uc774\uac83\ub4e4\uc740 \ud754\ud558\uac8c \ubc1c\uacac\ub418\ub294 \uac83\uc774 \uc544\ub2c8\uba70, \uc57d\uac04\uc758 \uace8\ud654 \ud604\uc0c1\uc774 \uc788\uc5c8\uc744 \uc218\ub3c4 \uc788\ub2e4. \ud55c \ubc1c\ud45c\uc5d0 \uc758\ud558\uba74, \ubcf5\ub291\uace8\uc740 \uac1c\uccb4\uac00 \uc0b4\uc544 \uc788\uc5c8\uc744 \ub54c \uc0c1\ud574\ub97c \uc785\uc5c8\ub2e4\ub294 \uc99d\uac70\uac00 \ub420 \uc218 \uc788\ub2e4. \ucc3d\uc0ac\uace8 (\uc77c\uba85 \uc704\uc2dc\ubcf8) \ub610\ud55c \uc874\uc7ac\ud588\uc9c0\ub9cc, 1996\ub144 \uc774 \ub418\uc5b4\uc11c\uc57c \uc778\uc815\ubc1b\uc558\ub2e4. \uba87\uba87 \uacbd\uc6b0\uc5d0 \ucc3d\uc0ac\uace8\uc774 \ubcf5\ub291\uace8\uacfc \ud63c\ub3d9\ub418\uae30\ub3c4 \ud588\ub2e4. \uc5c9\ub369\uc774\uc758 \uc911\uc2ec \ubf08\uc778 \uc7a5\uace8\uc740 \uc721\uc911\ud588\uace0, \uce58\uace8\uc5d0\ub294 \ud280\uc5b4\ub098\uc628 \ub05d\ub2e8\uc774 \uc788\uc5c8\ub294\ub370 \uc774\ub294 \uadfc\uc721\uc744 \uc5f0\uacb0\uc2dc\ucf1c \uc8fc\ub294 \ub3d9\uc2dc\uc5d0 \ub2e4\ub9ac\uac00 \ub545\uc744 \ub51b\uace0 \uc11c \uc788\uc744 \uc218 \uc788\uac8c \ud574 \uc8fc\ub294 \ubc1b\uce68\ub300 \uc5ed\ud560\uc744 \ud588\ub2e4. \ub9e4\ub4dc\uc2a8\uc740 \ud074\ub9ac\ube14\ub79c\ub4dc \ub85c\uc774\ub4dc \ud0d0\uc0ac\uc5d0\uc11c \ubc1c\uacac\ub41c \uac1c\uccb4\ub4e4 \uc911 \uc57d \uc808\ubc18 \uc815\ub3c4\uac00 \ud06c\uae30\uc5d0 \uad00\uacc4\uc5c6\uc774 \ubc1c \ub05d\ubd80\ubd84\uc758 \uc5f0\ubaa8\uac00 \uc11c\ub85c \uacb0\ud569\ub418\uc5b4 \uc788\uc9c0 \uc54a\uc558\ub2e4\uace0 \uae30\uc220\ud588\ub2e4. \ub9e4\ub4dc\uc2a8\uc740 \uc774\uac83\uc774 \uc131\uc801\uc778 \ud2b9\uc9d5\uc77c \uac00\ub2a5\uc131\uc774 \uc788\uc73c\uba70, \uc774\ub294 \uc554\ucef7\uc774 \uc54c\uc744 \ud488\uae30 \uc27d\ub3c4\ub85d \ubf08\uac00 \ubd99\uc5b4 \uc788\uc9c0 \uc54a\uc740 \uac83\uc774\ub77c\uace0 \uc8fc\uc7a5\ud588\ub2e4. \uadf8\ub7ec\ub098 \uc774 \uc0dd\uac01\uc740 \uc774\ud6c4 \uc9c0\uc9c0\ub97c \ubcc4\ub85c \ubc1b\uc9c0 \ubabb\ud588\ub2e4.", "paragraph_answer": "\uc54c\ub85c\uc0ac\uc6b0\ub8e8\uc2a4\ub294 \ubaa9\uc5d0 9\uac1c, \ub4f1\uc5d0 14\uac1c, \ucc9c\uace8\uc5d0 5\uac1c\uc758 \ucc99\ucd94\uace8\uc744 \uac00\uc9c0\uace0 \uc788\uc5c8\ub2e4. \uaf2c\ub9ac\uc758 \ucc99\ucd94\uace8 \uc218\ub294 \ubd88\uba85\uc774\uace0 \uac1c\uac1c\uc758 \ud06c\uae30\ub3c4 \ub2e4\uc591\ud588\ub2e4. \uc81c\uc784\uc2a4 \ub9e4\ub4dc\uc2a8\uc740 50\uac1c \uac00\ub7c9\uc73c\ub85c \uc608\uc0c1\ud588\uace0, \uac8c\uc624\ub974\uae30 \ud3f4\uc740 \uc774\uac83\ub3c4 \ub9ce\ub2e4\uace0 \uc5ec\uaca8\uc11c 45\uac1c\ub098 \uadf8 \uc774\ud558\ub85c \uc608\uce21\ud588\ub2e4. \ubaa9\uacfc \ub4f1\uc758 \ucc99\ucd94\uace8 \uc804\ubc29\uc5d0\ub294 \ube48 \uacf5\uac04\uc774 \uc788\uc5c8\uace0, \ud604\uc0dd \ub3d9\ubb3c\ub4e4(\uc0c8\ub77c\ub4e0\uc9c0)\uc5d0\uc11c\ub3c4 \ubc1c\uacac\ub418\ub294 \uc774\ub7f0 \uacf5\uac04\ub4e4\uc740 \ud638\ud761\uc5d0 \uc4f0\uc778 \uacf5\uae30\uc8fc\uba38\ub2c8\ub85c \ubcf4\uc778\ub2e4. \uac00\uc2b4\uc740 \ub113\uace0 \uc6d0\ud1b5\ud615 \ud749\uacfd\uc774\uc5c8\uace0, \ubcf5\ubd80 \ub291\uace8(\ubc30\uc758 \uac08\ube44)\ub97c \uac00\uc9c0\uace0 \uc788\uc5c8\uc9c0\ub9cc \uc774\uac83\ub4e4\uc740 \ud754\ud558\uac8c \ubc1c\uacac\ub418\ub294 \uac83\uc774 \uc544\ub2c8\uba70, \uc57d\uac04\uc758 \uace8\ud654 \ud604\uc0c1\uc774 \uc788\uc5c8\uc744 \uc218\ub3c4 \uc788\ub2e4. \ud55c \ubc1c\ud45c\uc5d0 \uc758\ud558\uba74, \ubcf5\ub291\uace8\uc740 \uac1c\uccb4\uac00 \uc0b4\uc544 \uc788\uc5c8\uc744 \ub54c \uc0c1\ud574\ub97c \uc785\uc5c8\ub2e4\ub294 \uc99d\uac70\uac00 \ub420 \uc218 \uc788\ub2e4. \ucc3d\uc0ac\uace8 (\uc77c\uba85 \uc704\uc2dc\ubcf8) \ub610\ud55c \uc874\uc7ac\ud588\uc9c0\ub9cc, 1996\ub144 \uc774 \ub418\uc5b4\uc11c\uc57c \uc778\uc815\ubc1b\uc558\ub2e4. \uba87\uba87 \uacbd\uc6b0\uc5d0 \ucc3d\uc0ac\uace8\uc774 \ubcf5\ub291\uace8\uacfc \ud63c\ub3d9\ub418\uae30\ub3c4 \ud588\ub2e4. \uc5c9\ub369\uc774\uc758 \uc911\uc2ec \ubf08\uc778 \uc7a5\uace8\uc740 \uc721\uc911\ud588\uace0, \uce58\uace8\uc5d0\ub294 \ud280\uc5b4\ub098\uc628 \ub05d\ub2e8\uc774 \uc788\uc5c8\ub294\ub370 \uc774\ub294 \uadfc\uc721\uc744 \uc5f0\uacb0\uc2dc\ucf1c \uc8fc\ub294 \ub3d9\uc2dc\uc5d0 \ub2e4\ub9ac\uac00 \ub545\uc744 \ub51b\uace0 \uc11c \uc788\uc744 \uc218 \uc788\uac8c \ud574 \uc8fc\ub294 \ubc1b\uce68\ub300 \uc5ed\ud560\uc744 \ud588\ub2e4. \ub9e4\ub4dc\uc2a8\uc740 \ud074\ub9ac\ube14\ub79c\ub4dc \ub85c\uc774\ub4dc \ud0d0\uc0ac\uc5d0\uc11c \ubc1c\uacac\ub41c \uac1c\uccb4\ub4e4 \uc911 \uc57d \uc808\ubc18 \uc815\ub3c4\uac00 \ud06c\uae30\uc5d0 \uad00\uacc4\uc5c6\uc774 \ubc1c \ub05d\ubd80\ubd84\uc758 \uc5f0\ubaa8\uac00 \uc11c\ub85c \uacb0\ud569\ub418\uc5b4 \uc788\uc9c0 \uc54a\uc558\ub2e4\uace0 \uae30\uc220\ud588\ub2e4. \ub9e4\ub4dc\uc2a8\uc740 \uc774\uac83\uc774 \uc131\uc801\uc778 \ud2b9\uc9d5\uc77c \uac00\ub2a5\uc131\uc774 \uc788\uc73c\uba70, \uc774\ub294 \uc554\ucef7\uc774 \uc54c\uc744 \ud488\uae30 \uc27d\ub3c4\ub85d \ubf08\uac00 \ubd99\uc5b4 \uc788\uc9c0 \uc54a\uc740 \uac83\uc774\ub77c\uace0 \uc8fc\uc7a5\ud588\ub2e4. \uadf8\ub7ec\ub098 \uc774 \uc0dd\uac01\uc740 \uc774\ud6c4 \uc9c0\uc9c0\ub97c \ubcc4\ub85c \ubc1b\uc9c0 \ubabb\ud588\ub2e4.", "sentence_answer": "\ucc3d\uc0ac\uace8 (\uc77c\uba85 \uc704\uc2dc\ubcf8) \ub610\ud55c \uc874\uc7ac\ud588\uc9c0\ub9cc, 1996\ub144 \uc774 \ub418\uc5b4\uc11c\uc57c \uc778\uc815\ubc1b\uc558\ub2e4.", "paragraph_id": "6539314-3-1", "paragraph_question": "question: \uc54c\ub85c\uc0ac\uc6b0\ub8e8\uc2a4\uc758 \ucc3d\uc0ac\uace8\uc774 \uc778\uc815 \ubc1b\uc740 \ud574\ub294?, context: \uc54c\ub85c\uc0ac\uc6b0\ub8e8\uc2a4\ub294 \ubaa9\uc5d0 9\uac1c, \ub4f1\uc5d0 14\uac1c, \ucc9c\uace8\uc5d0 5\uac1c\uc758 \ucc99\ucd94\uace8\uc744 \uac00\uc9c0\uace0 \uc788\uc5c8\ub2e4. \uaf2c\ub9ac\uc758 \ucc99\ucd94\uace8 \uc218\ub294 \ubd88\uba85\uc774\uace0 \uac1c\uac1c\uc758 \ud06c\uae30\ub3c4 \ub2e4\uc591\ud588\ub2e4. \uc81c\uc784\uc2a4 \ub9e4\ub4dc\uc2a8\uc740 50\uac1c \uac00\ub7c9\uc73c\ub85c \uc608\uc0c1\ud588\uace0, \uac8c\uc624\ub974\uae30 \ud3f4\uc740 \uc774\uac83\ub3c4 \ub9ce\ub2e4\uace0 \uc5ec\uaca8\uc11c 45\uac1c\ub098 \uadf8 \uc774\ud558\ub85c \uc608\uce21\ud588\ub2e4. \ubaa9\uacfc \ub4f1\uc758 \ucc99\ucd94\uace8 \uc804\ubc29\uc5d0\ub294 \ube48 \uacf5\uac04\uc774 \uc788\uc5c8\uace0, \ud604\uc0dd \ub3d9\ubb3c\ub4e4(\uc0c8\ub77c\ub4e0\uc9c0)\uc5d0\uc11c\ub3c4 \ubc1c\uacac\ub418\ub294 \uc774\ub7f0 \uacf5\uac04\ub4e4\uc740 \ud638\ud761\uc5d0 \uc4f0\uc778 \uacf5\uae30\uc8fc\uba38\ub2c8\ub85c \ubcf4\uc778\ub2e4. \uac00\uc2b4\uc740 \ub113\uace0 \uc6d0\ud1b5\ud615 \ud749\uacfd\uc774\uc5c8\uace0, \ubcf5\ubd80 \ub291\uace8(\ubc30\uc758 \uac08\ube44)\ub97c \uac00\uc9c0\uace0 \uc788\uc5c8\uc9c0\ub9cc \uc774\uac83\ub4e4\uc740 \ud754\ud558\uac8c \ubc1c\uacac\ub418\ub294 \uac83\uc774 \uc544\ub2c8\uba70, \uc57d\uac04\uc758 \uace8\ud654 \ud604\uc0c1\uc774 \uc788\uc5c8\uc744 \uc218\ub3c4 \uc788\ub2e4. \ud55c \ubc1c\ud45c\uc5d0 \uc758\ud558\uba74, \ubcf5\ub291\uace8\uc740 \uac1c\uccb4\uac00 \uc0b4\uc544 \uc788\uc5c8\uc744 \ub54c \uc0c1\ud574\ub97c \uc785\uc5c8\ub2e4\ub294 \uc99d\uac70\uac00 \ub420 \uc218 \uc788\ub2e4. \ucc3d\uc0ac\uace8 (\uc77c\uba85 \uc704\uc2dc\ubcf8) \ub610\ud55c \uc874\uc7ac\ud588\uc9c0\ub9cc, 1996\ub144\uc774 \ub418\uc5b4\uc11c\uc57c \uc778\uc815\ubc1b\uc558\ub2e4. \uba87\uba87 \uacbd\uc6b0\uc5d0 \ucc3d\uc0ac\uace8\uc774 \ubcf5\ub291\uace8\uacfc \ud63c\ub3d9\ub418\uae30\ub3c4 \ud588\ub2e4. \uc5c9\ub369\uc774\uc758 \uc911\uc2ec \ubf08\uc778 \uc7a5\uace8\uc740 \uc721\uc911\ud588\uace0, \uce58\uace8\uc5d0\ub294 \ud280\uc5b4\ub098\uc628 \ub05d\ub2e8\uc774 \uc788\uc5c8\ub294\ub370 \uc774\ub294 \uadfc\uc721\uc744 \uc5f0\uacb0\uc2dc\ucf1c \uc8fc\ub294 \ub3d9\uc2dc\uc5d0 \ub2e4\ub9ac\uac00 \ub545\uc744 \ub51b\uace0 \uc11c \uc788\uc744 \uc218 \uc788\uac8c \ud574 \uc8fc\ub294 \ubc1b\uce68\ub300 \uc5ed\ud560\uc744 \ud588\ub2e4. \ub9e4\ub4dc\uc2a8\uc740 \ud074\ub9ac\ube14\ub79c\ub4dc \ub85c\uc774\ub4dc \ud0d0\uc0ac\uc5d0\uc11c \ubc1c\uacac\ub41c \uac1c\uccb4\ub4e4 \uc911 \uc57d \uc808\ubc18 \uc815\ub3c4\uac00 \ud06c\uae30\uc5d0 \uad00\uacc4\uc5c6\uc774 \ubc1c \ub05d\ubd80\ubd84\uc758 \uc5f0\ubaa8\uac00 \uc11c\ub85c \uacb0\ud569\ub418\uc5b4 \uc788\uc9c0 \uc54a\uc558\ub2e4\uace0 \uae30\uc220\ud588\ub2e4. \ub9e4\ub4dc\uc2a8\uc740 \uc774\uac83\uc774 \uc131\uc801\uc778 \ud2b9\uc9d5\uc77c \uac00\ub2a5\uc131\uc774 \uc788\uc73c\uba70, \uc774\ub294 \uc554\ucef7\uc774 \uc54c\uc744 \ud488\uae30 \uc27d\ub3c4\ub85d \ubf08\uac00 \ubd99\uc5b4 \uc788\uc9c0 \uc54a\uc740 \uac83\uc774\ub77c\uace0 \uc8fc\uc7a5\ud588\ub2e4. \uadf8\ub7ec\ub098 \uc774 \uc0dd\uac01\uc740 \uc774\ud6c4 \uc9c0\uc9c0\ub97c \ubcc4\ub85c \ubc1b\uc9c0 \ubabb\ud588\ub2e4."} {"question": "\ucf54\ub9ac\ub3c4\ub77c\uc2a4 \uc544\uc5d0\ub124\uc6b0\uc2a4\ub294 \uc774 \uc774\ub984 \uc774\uc678\uc5d0 \uc5b4\ub5bb\uac8c \ubd88\ub9ac\ub294\uac00?", "paragraph": "\ucf54\ub9ac\ub3c4\ub77c\uc2a4 \uc544\uc5d0\ub124\uc6b0\uc2a4\ub294 \ucf54\ub9ac\ub3c4\ub77c\uc2a4\ub97c \uc798 \ubaa8\ub974\ub294 \uc0ac\ub78c\ub4e4\ub3c4 \uc54c \uc815\ub3c4\ub85c \ub9ce\uc774 \uc54c\ub824\uc838\uc788\uace0 \uac00\uaca9\uc774 \uc2fc \ucf54\ub9ac\ub3c4\ub77c\uc2a4\uc774\uba70, \ub9e4\uc6b0 \ud754\ud558\uae30 \ub54c\ubb38\uc5d0 \ub625\ucf54\ub9ac, \ub9c9\ucf54\ub9ac \ub77c\uace0\ub3c4 \ubd88\ub9b0\ub2e4. \uba39\uc774\ub97c \ucc3e\uc73c\uba74\uc11c \uc544\uac00\ubbf8\ub85c \ubaa8\ub798\ub97c \ubfdc\uc5b4\ub0b4\uae30\ub3c4 \ud558\ub294\ub370, \uadf8 \ubaa8\uc2b5\uc744 \ubcf4\uae30\uc704\ud574\uc11c\ub294 \uc544\uc8fc \uc785\uc790\uac00 \uc791\uc740 \ubc14\ub2e5\uc7ac\ub97c \uc368\uc57c \ud55c\ub2e4. \uc8fc\ub85c \ubaa8\ub4e0 \uc885\ub958\uc758 \ubb3c\uace0\uae30\uc640 \uc0b4 \uc218 \uc788\uc73c\uba70, 6\ub9c8\ub9ac \uc774\uc0c1\uc774 \ubaa8\uc5ec \uc788\uc5b4\uc57c \uc548\uc815\uac10\uc744 \ub290\ub080\ub2e4\uace0 \ud558\ub294 \uc8fc\uc7a5\ub3c4 \uc788\ub2e4.\ubb3c \uc628\ub3c4\ub294 18\u2103~26\u2103\uac00 \uc801\ub2f9\ud558\uba70, \uc57d\uc0b0\uc131, \uc911\uc131, \uc57d\uc5fc\uae30\uc131\uc758 \ubb3c\uc5d0\uc11c \ubb34\ub09c\ud558\uac8c \uc801\uc751\ud55c\ub2e4. \uc228\uc744\uacf3\uc744 \ub9cc\ub4e4\uc5b4\uc8fc\ub294 \uac83\uc774 \uc88b\ub2e4. \uc720\ubaa9\uc744 \ub123\uc5b4\ub3c4 \uc88b\uace0, \ubd80\uc0c1\uc2dd\ubb3c\uc744 \ub744\uc6cc \uc8fc\uba74 \uc57d\uac04 \uc5b4\ub461\uac8c \ub9cc\ub4e4\uc5b4 \uc8fc\uae30 \ub54c\ubb38\uc5d0 \ub3c4\uc6c0\uc774 \ub41c\ub2e4. \uc8fc\ub85c \uc218\uc870\uc758 \ubc11\ubd80\ubd84 \uc5d0\uc11c \uc0dd\ud65c\uc744 \ud558\ub294\ub370, \uc218\ucd08\ub97c \uc2ec\uc740\uc9c0 \uc5bc\ub9c8 \ub418\uc9c0 \uc54a\uc544\uc11c \ubfcc\ub9ac\uac00 \uc790\ub9ac\uc7a1\uae30\uc804\uc774\ub77c\uba74 \ubfcc\ub9ac\ub97c \ubf51\uc544 \ubc84\ub9ac\uae30\ub3c4 \ud558\uae30 \ub54c\ubb38\uc5d0 \uc8fc\uc758\ud574\uc57c \ud55c\ub2e4.", "answer": "\ub625\ucf54\ub9ac, \ub9c9\ucf54\ub9ac", "sentence": "\ucf54\ub9ac\ub3c4\ub77c\uc2a4\ub97c \uc798 \ubaa8\ub974\ub294 \uc0ac\ub78c\ub4e4\ub3c4 \uc54c \uc815\ub3c4\ub85c \ub9ce\uc774 \uc54c\ub824\uc838\uc788\uace0 \uac00\uaca9\uc774 \uc2fc \ucf54\ub9ac\ub3c4\ub77c\uc2a4\uc774\uba70, \ub9e4\uc6b0 \ud754\ud558\uae30 \ub54c\ubb38\uc5d0 \ub625\ucf54\ub9ac, \ub9c9\ucf54\ub9ac \ub77c\uace0\ub3c4 \ubd88\ub9b0\ub2e4.", "paragraph_sentence": "\ucf54\ub9ac\ub3c4\ub77c\uc2a4 \uc544\uc5d0\ub124\uc6b0\uc2a4\ub294 \ucf54\ub9ac\ub3c4\ub77c\uc2a4\ub97c \uc798 \ubaa8\ub974\ub294 \uc0ac\ub78c\ub4e4\ub3c4 \uc54c \uc815\ub3c4\ub85c \ub9ce\uc774 \uc54c\ub824\uc838\uc788\uace0 \uac00\uaca9\uc774 \uc2fc \ucf54\ub9ac\ub3c4\ub77c\uc2a4\uc774\uba70, \ub9e4\uc6b0 \ud754\ud558\uae30 \ub54c\ubb38\uc5d0 \ub625\ucf54\ub9ac, \ub9c9\ucf54\ub9ac \ub77c\uace0\ub3c4 \ubd88\ub9b0\ub2e4. \uba39\uc774\ub97c \ucc3e\uc73c\uba74\uc11c \uc544\uac00\ubbf8\ub85c \ubaa8\ub798\ub97c \ubfdc\uc5b4\ub0b4\uae30\ub3c4 \ud558\ub294\ub370, \uadf8 \ubaa8\uc2b5\uc744 \ubcf4\uae30\uc704\ud574\uc11c\ub294 \uc544\uc8fc \uc785\uc790\uac00 \uc791\uc740 \ubc14\ub2e5\uc7ac\ub97c \uc368\uc57c \ud55c\ub2e4. \uc8fc\ub85c \ubaa8\ub4e0 \uc885\ub958\uc758 \ubb3c\uace0\uae30\uc640 \uc0b4 \uc218 \uc788\uc73c\uba70, 6\ub9c8\ub9ac \uc774\uc0c1\uc774 \ubaa8\uc5ec \uc788\uc5b4\uc57c \uc548\uc815\uac10\uc744 \ub290\ub080\ub2e4\uace0 \ud558\ub294 \uc8fc\uc7a5\ub3c4 \uc788\ub2e4. \ubb3c \uc628\ub3c4\ub294 18\u2103~26\u2103\uac00 \uc801\ub2f9\ud558\uba70, \uc57d\uc0b0\uc131, \uc911\uc131, \uc57d\uc5fc\uae30\uc131\uc758 \ubb3c\uc5d0\uc11c \ubb34\ub09c\ud558\uac8c \uc801\uc751\ud55c\ub2e4. \uc228\uc744\uacf3\uc744 \ub9cc\ub4e4\uc5b4\uc8fc\ub294 \uac83\uc774 \uc88b\ub2e4. \uc720\ubaa9\uc744 \ub123\uc5b4\ub3c4 \uc88b\uace0, \ubd80\uc0c1\uc2dd\ubb3c\uc744 \ub744\uc6cc \uc8fc\uba74 \uc57d\uac04 \uc5b4\ub461\uac8c \ub9cc\ub4e4\uc5b4 \uc8fc\uae30 \ub54c\ubb38\uc5d0 \ub3c4\uc6c0\uc774 \ub41c\ub2e4. \uc8fc\ub85c \uc218\uc870\uc758 \ubc11\ubd80\ubd84 \uc5d0\uc11c \uc0dd\ud65c\uc744 \ud558\ub294\ub370, \uc218\ucd08\ub97c \uc2ec\uc740\uc9c0 \uc5bc\ub9c8 \ub418\uc9c0 \uc54a\uc544\uc11c \ubfcc\ub9ac\uac00 \uc790\ub9ac\uc7a1\uae30\uc804\uc774\ub77c\uba74 \ubfcc\ub9ac\ub97c \ubf51\uc544 \ubc84\ub9ac\uae30\ub3c4 \ud558\uae30 \ub54c\ubb38\uc5d0 \uc8fc\uc758\ud574\uc57c \ud55c\ub2e4.", "paragraph_answer": "\ucf54\ub9ac\ub3c4\ub77c\uc2a4 \uc544\uc5d0\ub124\uc6b0\uc2a4\ub294 \ucf54\ub9ac\ub3c4\ub77c\uc2a4\ub97c \uc798 \ubaa8\ub974\ub294 \uc0ac\ub78c\ub4e4\ub3c4 \uc54c \uc815\ub3c4\ub85c \ub9ce\uc774 \uc54c\ub824\uc838\uc788\uace0 \uac00\uaca9\uc774 \uc2fc \ucf54\ub9ac\ub3c4\ub77c\uc2a4\uc774\uba70, \ub9e4\uc6b0 \ud754\ud558\uae30 \ub54c\ubb38\uc5d0 \ub625\ucf54\ub9ac, \ub9c9\ucf54\ub9ac \ub77c\uace0\ub3c4 \ubd88\ub9b0\ub2e4. \uba39\uc774\ub97c \ucc3e\uc73c\uba74\uc11c \uc544\uac00\ubbf8\ub85c \ubaa8\ub798\ub97c \ubfdc\uc5b4\ub0b4\uae30\ub3c4 \ud558\ub294\ub370, \uadf8 \ubaa8\uc2b5\uc744 \ubcf4\uae30\uc704\ud574\uc11c\ub294 \uc544\uc8fc \uc785\uc790\uac00 \uc791\uc740 \ubc14\ub2e5\uc7ac\ub97c \uc368\uc57c \ud55c\ub2e4. \uc8fc\ub85c \ubaa8\ub4e0 \uc885\ub958\uc758 \ubb3c\uace0\uae30\uc640 \uc0b4 \uc218 \uc788\uc73c\uba70, 6\ub9c8\ub9ac \uc774\uc0c1\uc774 \ubaa8\uc5ec \uc788\uc5b4\uc57c \uc548\uc815\uac10\uc744 \ub290\ub080\ub2e4\uace0 \ud558\ub294 \uc8fc\uc7a5\ub3c4 \uc788\ub2e4.\ubb3c \uc628\ub3c4\ub294 18\u2103~26\u2103\uac00 \uc801\ub2f9\ud558\uba70, \uc57d\uc0b0\uc131, \uc911\uc131, \uc57d\uc5fc\uae30\uc131\uc758 \ubb3c\uc5d0\uc11c \ubb34\ub09c\ud558\uac8c \uc801\uc751\ud55c\ub2e4. \uc228\uc744\uacf3\uc744 \ub9cc\ub4e4\uc5b4\uc8fc\ub294 \uac83\uc774 \uc88b\ub2e4. \uc720\ubaa9\uc744 \ub123\uc5b4\ub3c4 \uc88b\uace0, \ubd80\uc0c1\uc2dd\ubb3c\uc744 \ub744\uc6cc \uc8fc\uba74 \uc57d\uac04 \uc5b4\ub461\uac8c \ub9cc\ub4e4\uc5b4 \uc8fc\uae30 \ub54c\ubb38\uc5d0 \ub3c4\uc6c0\uc774 \ub41c\ub2e4. \uc8fc\ub85c \uc218\uc870\uc758 \ubc11\ubd80\ubd84 \uc5d0\uc11c \uc0dd\ud65c\uc744 \ud558\ub294\ub370, \uc218\ucd08\ub97c \uc2ec\uc740\uc9c0 \uc5bc\ub9c8 \ub418\uc9c0 \uc54a\uc544\uc11c \ubfcc\ub9ac\uac00 \uc790\ub9ac\uc7a1\uae30\uc804\uc774\ub77c\uba74 \ubfcc\ub9ac\ub97c \ubf51\uc544 \ubc84\ub9ac\uae30\ub3c4 \ud558\uae30 \ub54c\ubb38\uc5d0 \uc8fc\uc758\ud574\uc57c \ud55c\ub2e4.", "sentence_answer": "\ucf54\ub9ac\ub3c4\ub77c\uc2a4\ub97c \uc798 \ubaa8\ub974\ub294 \uc0ac\ub78c\ub4e4\ub3c4 \uc54c \uc815\ub3c4\ub85c \ub9ce\uc774 \uc54c\ub824\uc838\uc788\uace0 \uac00\uaca9\uc774 \uc2fc \ucf54\ub9ac\ub3c4\ub77c\uc2a4\uc774\uba70, \ub9e4\uc6b0 \ud754\ud558\uae30 \ub54c\ubb38\uc5d0 \ub625\ucf54\ub9ac, \ub9c9\ucf54\ub9ac \ub77c\uace0\ub3c4 \ubd88\ub9b0\ub2e4.", "paragraph_id": "5906299-1-0", "paragraph_question": "question: \ucf54\ub9ac\ub3c4\ub77c\uc2a4 \uc544\uc5d0\ub124\uc6b0\uc2a4\ub294 \uc774 \uc774\ub984 \uc774\uc678\uc5d0 \uc5b4\ub5bb\uac8c \ubd88\ub9ac\ub294\uac00?, context: \ucf54\ub9ac\ub3c4\ub77c\uc2a4 \uc544\uc5d0\ub124\uc6b0\uc2a4\ub294 \ucf54\ub9ac\ub3c4\ub77c\uc2a4\ub97c \uc798 \ubaa8\ub974\ub294 \uc0ac\ub78c\ub4e4\ub3c4 \uc54c \uc815\ub3c4\ub85c \ub9ce\uc774 \uc54c\ub824\uc838\uc788\uace0 \uac00\uaca9\uc774 \uc2fc \ucf54\ub9ac\ub3c4\ub77c\uc2a4\uc774\uba70, \ub9e4\uc6b0 \ud754\ud558\uae30 \ub54c\ubb38\uc5d0 \ub625\ucf54\ub9ac, \ub9c9\ucf54\ub9ac \ub77c\uace0\ub3c4 \ubd88\ub9b0\ub2e4. \uba39\uc774\ub97c \ucc3e\uc73c\uba74\uc11c \uc544\uac00\ubbf8\ub85c \ubaa8\ub798\ub97c \ubfdc\uc5b4\ub0b4\uae30\ub3c4 \ud558\ub294\ub370, \uadf8 \ubaa8\uc2b5\uc744 \ubcf4\uae30\uc704\ud574\uc11c\ub294 \uc544\uc8fc \uc785\uc790\uac00 \uc791\uc740 \ubc14\ub2e5\uc7ac\ub97c \uc368\uc57c \ud55c\ub2e4. \uc8fc\ub85c \ubaa8\ub4e0 \uc885\ub958\uc758 \ubb3c\uace0\uae30\uc640 \uc0b4 \uc218 \uc788\uc73c\uba70, 6\ub9c8\ub9ac \uc774\uc0c1\uc774 \ubaa8\uc5ec \uc788\uc5b4\uc57c \uc548\uc815\uac10\uc744 \ub290\ub080\ub2e4\uace0 \ud558\ub294 \uc8fc\uc7a5\ub3c4 \uc788\ub2e4.\ubb3c \uc628\ub3c4\ub294 18\u2103~26\u2103\uac00 \uc801\ub2f9\ud558\uba70, \uc57d\uc0b0\uc131, \uc911\uc131, \uc57d\uc5fc\uae30\uc131\uc758 \ubb3c\uc5d0\uc11c \ubb34\ub09c\ud558\uac8c \uc801\uc751\ud55c\ub2e4. \uc228\uc744\uacf3\uc744 \ub9cc\ub4e4\uc5b4\uc8fc\ub294 \uac83\uc774 \uc88b\ub2e4. \uc720\ubaa9\uc744 \ub123\uc5b4\ub3c4 \uc88b\uace0, \ubd80\uc0c1\uc2dd\ubb3c\uc744 \ub744\uc6cc \uc8fc\uba74 \uc57d\uac04 \uc5b4\ub461\uac8c \ub9cc\ub4e4\uc5b4 \uc8fc\uae30 \ub54c\ubb38\uc5d0 \ub3c4\uc6c0\uc774 \ub41c\ub2e4. \uc8fc\ub85c \uc218\uc870\uc758 \ubc11\ubd80\ubd84 \uc5d0\uc11c \uc0dd\ud65c\uc744 \ud558\ub294\ub370, \uc218\ucd08\ub97c \uc2ec\uc740\uc9c0 \uc5bc\ub9c8 \ub418\uc9c0 \uc54a\uc544\uc11c \ubfcc\ub9ac\uac00 \uc790\ub9ac\uc7a1\uae30\uc804\uc774\ub77c\uba74 \ubfcc\ub9ac\ub97c \ubf51\uc544 \ubc84\ub9ac\uae30\ub3c4 \ud558\uae30 \ub54c\ubb38\uc5d0 \uc8fc\uc758\ud574\uc57c \ud55c\ub2e4."} {"question": "\ud669\ubbf8\uc694\uc870 \ud3c9\ub860\uac00\ub294 \uae40\uc740\uc219 \uc791\uac00\uac00 \uc2dc\ud2f0\ud640\ubd80\ud130 \ubb34\uc2a8 \uc138\ub300\uc758 \uc815\uce58\uc801 \uc758\uc2dd\uc744 \ub4dc\ub7ec\ub0b4 \uc654\ub2e4\uace0 \ubd84\uc11d\ud588\ub294\uac00?", "paragraph": "\uadf8\ub9ac\uace0 \uad6d\uac00\uc758 \uac1c\uc778\uc5d0 \ub300\ud55c \ucc45\uc784\uc131\uc774 \ub610\ub2e4\ub978 \uc8fc\uc81c\uc774\ub2e4. \uae40\ub098\ud604 \uae30\uc790\uc5d0 \ub530\ub974\uba74 \uc774\uc804\uc758 \uae40\uc740\uc219 \ub4dc\ub77c\ub9c8\uac00 \ub85c\ub9e8\uc2a4\ub97c \ud1b5\ud574 \uacac\uace0\ud55c \uacc4\uae09 \uc0ac\ud68c\uc5d0 \uade0\uc5f4\uc744 \uc77c\uc73c\ucf1c \uc628 \uac83\uacfc \ub2ec\ub9ac, \u2018\ud0dc\ud6c4\u2019\ub294 \uad6d\uac00\uad00\uc5d0 \ucd08\uc810\uc744 \ub9de\ucdc4\ub2e4\ub294 \uc810\uc774 \ud2b9\uc9d5\uc774\ub2e4. \ud669\ubbf8\uc694\uc870 \ud3c9\ub860\uac00\ub294 \u201c\uc0ac\uc2e4 \uae40 \uc791\uac00\ub294 \u2018\uc2dc\ud2f0\ud640\u2019\ubd80\ud130 \ud3ec\uc2a4\ud2b8 386\uc138\ub300\uc758 \uc815\uce58\uc801 \uc758\uc2dd\uc744 \ub4dc\ub7ec\ub0b4 \uc654\ub2e4. \uad6d\uac00\ub77c\ub294 \uc8fc\uc81c\ub97c \uc9c1\uc811\uc801\uc73c\ub85c \uc5b8\uae09\ud55c \u2018\ud0dc\ud6c4\u2019 \uc5ed\uc2dc \uadf8 \uc5f0\uc7a5\uc120\uc5d0 \uc788\ub2e4.\"\uace0 \ubd84\uc11d\ud588\ub2e4. \uac15\ubaa8\uc5f0\uacfc \uc720\uc2dc\uc9c4\uc740 \u201c\uc870\uad6d\uc774 \uc9c0\ucf1c\uc57c \ud560 \uad6d\ubbfc\uc758 \uc778\uad8c\u201d\uc774\ub77c\ub294 \uac00\uce58\ub97c \uacf5\uc720\ud558\uba70 \uc0ac\ub791\uc744 \ud0a4\uc6cc \ub098\uac04\ub2e4. \u201c\uc544\uc774\uc640 \ub178\uc778\uacfc \ubbf8\uc778\uc740 \ubcf4\ud638\ud574\uc57c \ud55c\ub2e4\ub294 \ubbff\uc74c, \uae38\uac70\ub9ac\uc5d0\uc11c \ub2f4\ubc30 \ud53c\uc6b0\ub294 \u2018\uace0\ub529\u2019\uc744 \ubcf4\uba74 \ubb34\uc12d\uae34 \ud558\uc9c0\ub9cc \ud55c\uc18c\ub9ac \ud560 \uc218 \uc788\ub294 \uc6a9\uae30 (\uc911\ub7b5) \uadf8\ub798\uc11c \uc9c0\ucf1c\uc9c0\ub294 \uad70\uc778\uc758 \uba85\uc608. \ub0b4\uac00 \uc0dd\uac01\ud558\ub294 \uc560\uad6d\uc2ec\uc740 \uadf8\ub7f0 \uac81\ub2c8\ub2e4.\u201d \uc720\uc2dc\uc9c4\uc758 \ub300\uc0ac\ub294 \ud734\uba38\ub2c8\uc998\uc5d0 \uae30\ubc18\uc744 \ub454 \uad6d\uac00\uc801 \uc2e0\ub150\uc744 \ub610\ub837\uc774 \ub4dc\ub7ec\ub0b8\ub2e4. \uc774 \ub300\uc0ac\ub97c \ud1b5\ud574 \ub4dc\ub7ec\ub098\ub4ef \uc720\uc2dc\uc9c4\uc740 \uad70\uc778\uc73c\ub85c\uc11c \ubbfc\uac04\uc778\uc5d0 \ubd09\uc0ac\ud558\ub294 \uc218\ub2e8\uc73c\ub85c\uc11c \uad6d\uac00\ub97c \uc0ac\ub791\ud55c\ub2e4. \uc720\uc2dc\uc9c4\uc774 \ub9d0\ud558\ub294 \"\uc560\uad6d\uc2ec\"\uc740 \uc77c\ubc18\uc801\uc778 \uc758\ubbf8\uac00 \uc544\ub2c8\ub77c \uad70\uc778\uc73c\ub85c\uc11c \ubbfc\uac04\uc778\uc5d0 \ub300\ud55c, \uad6d\uac00\uacf5\ubb34\uc6d0\uc73c\ub85c\uc11c \uc778\uac04\uc5d0 \ub300\ud55c \ucc45\uc784\uc131\uc5d0 \uac00\uae5d\ub2e4.", "answer": "\ud3ec\uc2a4\ud2b8 386\uc138\ub300", "sentence": "\ud669\ubbf8\uc694\uc870 \ud3c9\ub860\uac00\ub294 \u201c\uc0ac\uc2e4 \uae40 \uc791\uac00\ub294 \u2018\uc2dc\ud2f0\ud640\u2019\ubd80\ud130 \ud3ec\uc2a4\ud2b8 386\uc138\ub300 \uc758 \uc815\uce58\uc801 \uc758\uc2dd\uc744 \ub4dc\ub7ec\ub0b4 \uc654\ub2e4.", "paragraph_sentence": "\uadf8\ub9ac\uace0 \uad6d\uac00\uc758 \uac1c\uc778\uc5d0 \ub300\ud55c \ucc45\uc784\uc131\uc774 \ub610\ub2e4\ub978 \uc8fc\uc81c\uc774\ub2e4. \uae40\ub098\ud604 \uae30\uc790\uc5d0 \ub530\ub974\uba74 \uc774\uc804\uc758 \uae40\uc740\uc219 \ub4dc\ub77c\ub9c8\uac00 \ub85c\ub9e8\uc2a4\ub97c \ud1b5\ud574 \uacac\uace0\ud55c \uacc4\uae09 \uc0ac\ud68c\uc5d0 \uade0\uc5f4\uc744 \uc77c\uc73c\ucf1c \uc628 \uac83\uacfc \ub2ec\ub9ac, \u2018\ud0dc\ud6c4\u2019\ub294 \uad6d\uac00\uad00\uc5d0 \ucd08\uc810\uc744 \ub9de\ucdc4\ub2e4\ub294 \uc810\uc774 \ud2b9\uc9d5\uc774\ub2e4. \ud669\ubbf8\uc694\uc870 \ud3c9\ub860\uac00\ub294 \u201c\uc0ac\uc2e4 \uae40 \uc791\uac00\ub294 \u2018\uc2dc\ud2f0\ud640\u2019\ubd80\ud130 \ud3ec\uc2a4\ud2b8 386\uc138\ub300 \uc758 \uc815\uce58\uc801 \uc758\uc2dd\uc744 \ub4dc\ub7ec\ub0b4 \uc654\ub2e4. \uad6d\uac00\ub77c\ub294 \uc8fc\uc81c\ub97c \uc9c1\uc811\uc801\uc73c\ub85c \uc5b8\uae09\ud55c \u2018\ud0dc\ud6c4\u2019 \uc5ed\uc2dc \uadf8 \uc5f0\uc7a5\uc120\uc5d0 \uc788\ub2e4. \"\uace0 \ubd84\uc11d\ud588\ub2e4. \uac15\ubaa8\uc5f0\uacfc \uc720\uc2dc\uc9c4\uc740 \u201c\uc870\uad6d\uc774 \uc9c0\ucf1c\uc57c \ud560 \uad6d\ubbfc\uc758 \uc778\uad8c\u201d\uc774\ub77c\ub294 \uac00\uce58\ub97c \uacf5\uc720\ud558\uba70 \uc0ac\ub791\uc744 \ud0a4\uc6cc \ub098\uac04\ub2e4. \u201c\uc544\uc774\uc640 \ub178\uc778\uacfc \ubbf8\uc778\uc740 \ubcf4\ud638\ud574\uc57c \ud55c\ub2e4\ub294 \ubbff\uc74c, \uae38\uac70\ub9ac\uc5d0\uc11c \ub2f4\ubc30 \ud53c\uc6b0\ub294 \u2018\uace0\ub529\u2019\uc744 \ubcf4\uba74 \ubb34\uc12d\uae34 \ud558\uc9c0\ub9cc \ud55c\uc18c\ub9ac \ud560 \uc218 \uc788\ub294 \uc6a9\uae30 (\uc911\ub7b5) \uadf8\ub798\uc11c \uc9c0\ucf1c\uc9c0\ub294 \uad70\uc778\uc758 \uba85\uc608. \ub0b4\uac00 \uc0dd\uac01\ud558\ub294 \uc560\uad6d\uc2ec\uc740 \uadf8\ub7f0 \uac81\ub2c8\ub2e4. \u201d \uc720\uc2dc\uc9c4\uc758 \ub300\uc0ac\ub294 \ud734\uba38\ub2c8\uc998\uc5d0 \uae30\ubc18\uc744 \ub454 \uad6d\uac00\uc801 \uc2e0\ub150\uc744 \ub610\ub837\uc774 \ub4dc\ub7ec\ub0b8\ub2e4. \uc774 \ub300\uc0ac\ub97c \ud1b5\ud574 \ub4dc\ub7ec\ub098\ub4ef \uc720\uc2dc\uc9c4\uc740 \uad70\uc778\uc73c\ub85c\uc11c \ubbfc\uac04\uc778\uc5d0 \ubd09\uc0ac\ud558\ub294 \uc218\ub2e8\uc73c\ub85c\uc11c \uad6d\uac00\ub97c \uc0ac\ub791\ud55c\ub2e4. \uc720\uc2dc\uc9c4\uc774 \ub9d0\ud558\ub294 \"\uc560\uad6d\uc2ec\"\uc740 \uc77c\ubc18\uc801\uc778 \uc758\ubbf8\uac00 \uc544\ub2c8\ub77c \uad70\uc778\uc73c\ub85c\uc11c \ubbfc\uac04\uc778\uc5d0 \ub300\ud55c, \uad6d\uac00\uacf5\ubb34\uc6d0\uc73c\ub85c\uc11c \uc778\uac04\uc5d0 \ub300\ud55c \ucc45\uc784\uc131\uc5d0 \uac00\uae5d\ub2e4.", "paragraph_answer": "\uadf8\ub9ac\uace0 \uad6d\uac00\uc758 \uac1c\uc778\uc5d0 \ub300\ud55c \ucc45\uc784\uc131\uc774 \ub610\ub2e4\ub978 \uc8fc\uc81c\uc774\ub2e4. \uae40\ub098\ud604 \uae30\uc790\uc5d0 \ub530\ub974\uba74 \uc774\uc804\uc758 \uae40\uc740\uc219 \ub4dc\ub77c\ub9c8\uac00 \ub85c\ub9e8\uc2a4\ub97c \ud1b5\ud574 \uacac\uace0\ud55c \uacc4\uae09 \uc0ac\ud68c\uc5d0 \uade0\uc5f4\uc744 \uc77c\uc73c\ucf1c \uc628 \uac83\uacfc \ub2ec\ub9ac, \u2018\ud0dc\ud6c4\u2019\ub294 \uad6d\uac00\uad00\uc5d0 \ucd08\uc810\uc744 \ub9de\ucdc4\ub2e4\ub294 \uc810\uc774 \ud2b9\uc9d5\uc774\ub2e4. \ud669\ubbf8\uc694\uc870 \ud3c9\ub860\uac00\ub294 \u201c\uc0ac\uc2e4 \uae40 \uc791\uac00\ub294 \u2018\uc2dc\ud2f0\ud640\u2019\ubd80\ud130 \ud3ec\uc2a4\ud2b8 386\uc138\ub300 \uc758 \uc815\uce58\uc801 \uc758\uc2dd\uc744 \ub4dc\ub7ec\ub0b4 \uc654\ub2e4. \uad6d\uac00\ub77c\ub294 \uc8fc\uc81c\ub97c \uc9c1\uc811\uc801\uc73c\ub85c \uc5b8\uae09\ud55c \u2018\ud0dc\ud6c4\u2019 \uc5ed\uc2dc \uadf8 \uc5f0\uc7a5\uc120\uc5d0 \uc788\ub2e4.\"\uace0 \ubd84\uc11d\ud588\ub2e4. \uac15\ubaa8\uc5f0\uacfc \uc720\uc2dc\uc9c4\uc740 \u201c\uc870\uad6d\uc774 \uc9c0\ucf1c\uc57c \ud560 \uad6d\ubbfc\uc758 \uc778\uad8c\u201d\uc774\ub77c\ub294 \uac00\uce58\ub97c \uacf5\uc720\ud558\uba70 \uc0ac\ub791\uc744 \ud0a4\uc6cc \ub098\uac04\ub2e4. \u201c\uc544\uc774\uc640 \ub178\uc778\uacfc \ubbf8\uc778\uc740 \ubcf4\ud638\ud574\uc57c \ud55c\ub2e4\ub294 \ubbff\uc74c, \uae38\uac70\ub9ac\uc5d0\uc11c \ub2f4\ubc30 \ud53c\uc6b0\ub294 \u2018\uace0\ub529\u2019\uc744 \ubcf4\uba74 \ubb34\uc12d\uae34 \ud558\uc9c0\ub9cc \ud55c\uc18c\ub9ac \ud560 \uc218 \uc788\ub294 \uc6a9\uae30 (\uc911\ub7b5) \uadf8\ub798\uc11c \uc9c0\ucf1c\uc9c0\ub294 \uad70\uc778\uc758 \uba85\uc608. \ub0b4\uac00 \uc0dd\uac01\ud558\ub294 \uc560\uad6d\uc2ec\uc740 \uadf8\ub7f0 \uac81\ub2c8\ub2e4.\u201d \uc720\uc2dc\uc9c4\uc758 \ub300\uc0ac\ub294 \ud734\uba38\ub2c8\uc998\uc5d0 \uae30\ubc18\uc744 \ub454 \uad6d\uac00\uc801 \uc2e0\ub150\uc744 \ub610\ub837\uc774 \ub4dc\ub7ec\ub0b8\ub2e4. \uc774 \ub300\uc0ac\ub97c \ud1b5\ud574 \ub4dc\ub7ec\ub098\ub4ef \uc720\uc2dc\uc9c4\uc740 \uad70\uc778\uc73c\ub85c\uc11c \ubbfc\uac04\uc778\uc5d0 \ubd09\uc0ac\ud558\ub294 \uc218\ub2e8\uc73c\ub85c\uc11c \uad6d\uac00\ub97c \uc0ac\ub791\ud55c\ub2e4. \uc720\uc2dc\uc9c4\uc774 \ub9d0\ud558\ub294 \"\uc560\uad6d\uc2ec\"\uc740 \uc77c\ubc18\uc801\uc778 \uc758\ubbf8\uac00 \uc544\ub2c8\ub77c \uad70\uc778\uc73c\ub85c\uc11c \ubbfc\uac04\uc778\uc5d0 \ub300\ud55c, \uad6d\uac00\uacf5\ubb34\uc6d0\uc73c\ub85c\uc11c \uc778\uac04\uc5d0 \ub300\ud55c \ucc45\uc784\uc131\uc5d0 \uac00\uae5d\ub2e4.", "sentence_answer": "\ud669\ubbf8\uc694\uc870 \ud3c9\ub860\uac00\ub294 \u201c\uc0ac\uc2e4 \uae40 \uc791\uac00\ub294 \u2018\uc2dc\ud2f0\ud640\u2019\ubd80\ud130 \ud3ec\uc2a4\ud2b8 386\uc138\ub300 \uc758 \uc815\uce58\uc801 \uc758\uc2dd\uc744 \ub4dc\ub7ec\ub0b4 \uc654\ub2e4.", "paragraph_id": "6565257-4-1", "paragraph_question": "question: \ud669\ubbf8\uc694\uc870 \ud3c9\ub860\uac00\ub294 \uae40\uc740\uc219 \uc791\uac00\uac00 \uc2dc\ud2f0\ud640\ubd80\ud130 \ubb34\uc2a8 \uc138\ub300\uc758 \uc815\uce58\uc801 \uc758\uc2dd\uc744 \ub4dc\ub7ec\ub0b4 \uc654\ub2e4\uace0 \ubd84\uc11d\ud588\ub294\uac00?, context: \uadf8\ub9ac\uace0 \uad6d\uac00\uc758 \uac1c\uc778\uc5d0 \ub300\ud55c \ucc45\uc784\uc131\uc774 \ub610\ub2e4\ub978 \uc8fc\uc81c\uc774\ub2e4. \uae40\ub098\ud604 \uae30\uc790\uc5d0 \ub530\ub974\uba74 \uc774\uc804\uc758 \uae40\uc740\uc219 \ub4dc\ub77c\ub9c8\uac00 \ub85c\ub9e8\uc2a4\ub97c \ud1b5\ud574 \uacac\uace0\ud55c \uacc4\uae09 \uc0ac\ud68c\uc5d0 \uade0\uc5f4\uc744 \uc77c\uc73c\ucf1c \uc628 \uac83\uacfc \ub2ec\ub9ac, \u2018\ud0dc\ud6c4\u2019\ub294 \uad6d\uac00\uad00\uc5d0 \ucd08\uc810\uc744 \ub9de\ucdc4\ub2e4\ub294 \uc810\uc774 \ud2b9\uc9d5\uc774\ub2e4. \ud669\ubbf8\uc694\uc870 \ud3c9\ub860\uac00\ub294 \u201c\uc0ac\uc2e4 \uae40 \uc791\uac00\ub294 \u2018\uc2dc\ud2f0\ud640\u2019\ubd80\ud130 \ud3ec\uc2a4\ud2b8 386\uc138\ub300\uc758 \uc815\uce58\uc801 \uc758\uc2dd\uc744 \ub4dc\ub7ec\ub0b4 \uc654\ub2e4. \uad6d\uac00\ub77c\ub294 \uc8fc\uc81c\ub97c \uc9c1\uc811\uc801\uc73c\ub85c \uc5b8\uae09\ud55c \u2018\ud0dc\ud6c4\u2019 \uc5ed\uc2dc \uadf8 \uc5f0\uc7a5\uc120\uc5d0 \uc788\ub2e4.\"\uace0 \ubd84\uc11d\ud588\ub2e4. \uac15\ubaa8\uc5f0\uacfc \uc720\uc2dc\uc9c4\uc740 \u201c\uc870\uad6d\uc774 \uc9c0\ucf1c\uc57c \ud560 \uad6d\ubbfc\uc758 \uc778\uad8c\u201d\uc774\ub77c\ub294 \uac00\uce58\ub97c \uacf5\uc720\ud558\uba70 \uc0ac\ub791\uc744 \ud0a4\uc6cc \ub098\uac04\ub2e4. \u201c\uc544\uc774\uc640 \ub178\uc778\uacfc \ubbf8\uc778\uc740 \ubcf4\ud638\ud574\uc57c \ud55c\ub2e4\ub294 \ubbff\uc74c, \uae38\uac70\ub9ac\uc5d0\uc11c \ub2f4\ubc30 \ud53c\uc6b0\ub294 \u2018\uace0\ub529\u2019\uc744 \ubcf4\uba74 \ubb34\uc12d\uae34 \ud558\uc9c0\ub9cc \ud55c\uc18c\ub9ac \ud560 \uc218 \uc788\ub294 \uc6a9\uae30 (\uc911\ub7b5) \uadf8\ub798\uc11c \uc9c0\ucf1c\uc9c0\ub294 \uad70\uc778\uc758 \uba85\uc608. \ub0b4\uac00 \uc0dd\uac01\ud558\ub294 \uc560\uad6d\uc2ec\uc740 \uadf8\ub7f0 \uac81\ub2c8\ub2e4.\u201d \uc720\uc2dc\uc9c4\uc758 \ub300\uc0ac\ub294 \ud734\uba38\ub2c8\uc998\uc5d0 \uae30\ubc18\uc744 \ub454 \uad6d\uac00\uc801 \uc2e0\ub150\uc744 \ub610\ub837\uc774 \ub4dc\ub7ec\ub0b8\ub2e4. \uc774 \ub300\uc0ac\ub97c \ud1b5\ud574 \ub4dc\ub7ec\ub098\ub4ef \uc720\uc2dc\uc9c4\uc740 \uad70\uc778\uc73c\ub85c\uc11c \ubbfc\uac04\uc778\uc5d0 \ubd09\uc0ac\ud558\ub294 \uc218\ub2e8\uc73c\ub85c\uc11c \uad6d\uac00\ub97c \uc0ac\ub791\ud55c\ub2e4. \uc720\uc2dc\uc9c4\uc774 \ub9d0\ud558\ub294 \"\uc560\uad6d\uc2ec\"\uc740 \uc77c\ubc18\uc801\uc778 \uc758\ubbf8\uac00 \uc544\ub2c8\ub77c \uad70\uc778\uc73c\ub85c\uc11c \ubbfc\uac04\uc778\uc5d0 \ub300\ud55c, \uad6d\uac00\uacf5\ubb34\uc6d0\uc73c\ub85c\uc11c \uc778\uac04\uc5d0 \ub300\ud55c \ucc45\uc784\uc131\uc5d0 \uac00\uae5d\ub2e4."} {"question": "\ubc15\uc815\ud76c\uc758 \ub538\uc778 \ubc15\uadfc\ub839 \ub0a8\ud3b8\uc758 \uc774\ub984\uc740 \ubb34\uc5c7\uc778\uac00?", "paragraph": "2005\ub144 1\uc6d4 \uc601\ud654 \u300a\uadf8\ub54c \uadf8\uc0ac\ub78c\ub4e4\u300b\uc774 \uc544\ubc84\uc9c0 \ubc15\uc815\ud76c\uc758 \uba85\uc608\ub97c \ud6fc\uc190\ud588\ub2e4\uba70 \uc0c1\uc601\uae08\uc9c0 \uac00\ucc98\ubd84 \uc2e0\uccad\uc744 \ub0c8\uc73c\uba70, \ubc95\uc6d0\uc740 \uc77c\ubd80 \uc7a5\uba74\uc744 \uc0ad\uc81c\ud558\ub77c\uace0 \ud310\uacb0\uc744 \ub0b4\ub838\ub2e4. \uadf8\ub7ec\ub098 \uac19\uc740 \ud574 2\uc6d4 MK\ud53d\ucc98\uc2a4\ub294 \uac00\ucc98\ubd84 \uc774\uc758\uc2e0\uccad\uc18c\uc1a1\uc744 \ub0c8\uc73c\uba70, \ubc15\uc9c0\ub9cc\uc740 \uc601\ud654\uc0c1\uc601\uae08\uc9c0 \ubc0f \uc190\ud574\ubc30\uc0c1 \uccad\uad6c\uc18c\uc1a1\uc744 \uc81c\uae30\ud588\ub2e4. 2006\ub144 \uc11c\uc6b8\uc911\uc559\uc9c0\ubc29\ubc95\uc6d0\uc740 \uc190\ud574\ubc30\uc0c1 \uc18c\uc1a1 \uae08\uc561 5\uc5b5\uc6d0 \uc911 1\uc5b5\uc6d0\uc744 \ubc15\uc9c0\ub9cc\uc5d0\uac8c \uc81c\ucd9c\ud558\ub77c\uace0 \uc6d0\uace0 \uc77c\ubd80\uc2b9\uc18c \ud310\uacb0\uc744 \ub0b4\ub838\uc9c0\ub9cc, \uc0c1\uc601\uae08\uc9c0 \uccad\uad6c\uc5d0 \ub300\ud574\uc11c\ub294 \uae30\uac01\ud558\uc600\ub2e4. 2011\ub144 8\uc6d4 19\uc77c \uc11c\uc6b8\uac15\ub0a8\uacbd\ucc30\uc11c\ub294 \ubc15\uc9c0\ub9cc\uc758 \ub204\ub098\uc778 \ubc15\uadfc\ub839\uc758 \ub0a8\ud3b8 \uc2e0\ub3d9\uc6b1\uc5d0 \ub300\ud574 '2010\ub144 9\uc6d4 \ubc15\uc9c0\ub9cc\uc758 5\ucd0c \uc870\uce74\uc640 \ube44\uc11c\uc2e4\uc7a5\uc774 \uc790\uc2e0\uc744 \uc911\uad6d\uc73c\ub85c \ub0a9\uce58\ud588\uace0, \uc790\uc2e0\uc774 \uc911\uad6d\uc5d0\uc11c \ub9c8\uc57d\uc744 \ud588\ub2e4\ub294 \uc18c\ubb38 \ub4f1\uc744 \ud37c\ub728\ub838\ub2e4\uba70, \ub450 \uc0ac\ub78c\uc744 \uace0\uc18c\ud558\ub294 \ub4f1'\uc758 \ubb34\uace0 \ud610\uc758\ub85c \uad6c\uc18d\ud558\uc600\ub2e4. \uc2e0\ub3d9\uc6b1\uc740 '\ubc15\uc9c0\ub9cc\uc758 \ube44\uc11c\uc2e4\uc7a5 \ub4f1\uc774 2007\ub144 \uc790\uc2e0\uc744 \uc911\uad6d\uc73c\ub85c \ub0a9\uce58\ud558\uc5ec \uc0b4\ud574\ud558\ub824 \ud558\uc600\uace0, \ubc15\uc9c0\ub9cc\uc774 \uc774\ub97c \ub4a4\uc5d0\uc11c \uad50\uc0ac\ud588\ub2e4'\ub294 \uc8fc\uc7a5\ub3c4 \uc81c\uae30\ud558\uc600\uc73c\uba70, \uc5b8\ub860\uacfc\uc758 \uc778\ud130\ubdf0\uc5d0\uc11c \ubc15\uc9c0\ub9cc\uc774 \uc8fd\uc774\ub824\ud588\ub2e4\ub294 \ub179\ucde8\ub85d\uc774 \uc788\ub2e4\uace0 \uc8fc\uc7a5\ud588\uae30 \ub54c\ubb38\uc774\ub2e4.", "answer": "\uc2e0\ub3d9\uc6b1", "sentence": "2011\ub144 8\uc6d4 19\uc77c \uc11c\uc6b8\uac15\ub0a8\uacbd\ucc30\uc11c\ub294 \ubc15\uc9c0\ub9cc\uc758 \ub204\ub098\uc778 \ubc15\uadfc\ub839\uc758 \ub0a8\ud3b8 \uc2e0\ub3d9\uc6b1 \uc5d0", "paragraph_sentence": "2005\ub144 1\uc6d4 \uc601\ud654 \u300a\uadf8\ub54c \uadf8\uc0ac\ub78c\ub4e4\u300b\uc774 \uc544\ubc84\uc9c0 \ubc15\uc815\ud76c\uc758 \uba85\uc608\ub97c \ud6fc\uc190\ud588\ub2e4\uba70 \uc0c1\uc601\uae08\uc9c0 \uac00\ucc98\ubd84 \uc2e0\uccad\uc744 \ub0c8\uc73c\uba70, \ubc95\uc6d0\uc740 \uc77c\ubd80 \uc7a5\uba74\uc744 \uc0ad\uc81c\ud558\ub77c\uace0 \ud310\uacb0\uc744 \ub0b4\ub838\ub2e4. \uadf8\ub7ec\ub098 \uac19\uc740 \ud574 2\uc6d4 MK\ud53d\ucc98\uc2a4\ub294 \uac00\ucc98\ubd84 \uc774\uc758\uc2e0\uccad\uc18c\uc1a1\uc744 \ub0c8\uc73c\uba70, \ubc15\uc9c0\ub9cc\uc740 \uc601\ud654\uc0c1\uc601\uae08\uc9c0 \ubc0f \uc190\ud574\ubc30\uc0c1 \uccad\uad6c\uc18c\uc1a1\uc744 \uc81c\uae30\ud588\ub2e4. 2006\ub144 \uc11c\uc6b8\uc911\uc559\uc9c0\ubc29\ubc95\uc6d0\uc740 \uc190\ud574\ubc30\uc0c1 \uc18c\uc1a1 \uae08\uc561 5\uc5b5\uc6d0 \uc911 1\uc5b5\uc6d0\uc744 \ubc15\uc9c0\ub9cc\uc5d0\uac8c \uc81c\ucd9c\ud558\ub77c\uace0 \uc6d0\uace0 \uc77c\ubd80\uc2b9\uc18c \ud310\uacb0\uc744 \ub0b4\ub838\uc9c0\ub9cc, \uc0c1\uc601\uae08\uc9c0 \uccad\uad6c\uc5d0 \ub300\ud574\uc11c\ub294 \uae30\uac01\ud558\uc600\ub2e4. 2011\ub144 8\uc6d4 19\uc77c \uc11c\uc6b8\uac15\ub0a8\uacbd\ucc30\uc11c\ub294 \ubc15\uc9c0\ub9cc\uc758 \ub204\ub098\uc778 \ubc15\uadfc\ub839\uc758 \ub0a8\ud3b8 \uc2e0\ub3d9\uc6b1 \uc5d0 \ub300\ud574 '2010\ub144 9\uc6d4 \ubc15\uc9c0\ub9cc\uc758 5\ucd0c \uc870\uce74\uc640 \ube44\uc11c\uc2e4\uc7a5\uc774 \uc790\uc2e0\uc744 \uc911\uad6d\uc73c\ub85c \ub0a9\uce58\ud588\uace0, \uc790\uc2e0\uc774 \uc911\uad6d\uc5d0\uc11c \ub9c8\uc57d\uc744 \ud588\ub2e4\ub294 \uc18c\ubb38 \ub4f1\uc744 \ud37c\ub728\ub838\ub2e4\uba70, \ub450 \uc0ac\ub78c\uc744 \uace0\uc18c\ud558\ub294 \ub4f1'\uc758 \ubb34\uace0 \ud610\uc758\ub85c \uad6c\uc18d\ud558\uc600\ub2e4. \uc2e0\ub3d9\uc6b1\uc740 '\ubc15\uc9c0\ub9cc\uc758 \ube44\uc11c\uc2e4\uc7a5 \ub4f1\uc774 2007\ub144 \uc790\uc2e0\uc744 \uc911\uad6d\uc73c\ub85c \ub0a9\uce58\ud558\uc5ec \uc0b4\ud574\ud558\ub824 \ud558\uc600\uace0, \ubc15\uc9c0\ub9cc\uc774 \uc774\ub97c \ub4a4\uc5d0\uc11c \uad50\uc0ac\ud588\ub2e4'\ub294 \uc8fc\uc7a5\ub3c4 \uc81c\uae30\ud558\uc600\uc73c\uba70, \uc5b8\ub860\uacfc\uc758 \uc778\ud130\ubdf0\uc5d0\uc11c \ubc15\uc9c0\ub9cc\uc774 \uc8fd\uc774\ub824\ud588\ub2e4\ub294 \ub179\ucde8\ub85d\uc774 \uc788\ub2e4\uace0 \uc8fc\uc7a5\ud588\uae30 \ub54c\ubb38\uc774\ub2e4.", "paragraph_answer": "2005\ub144 1\uc6d4 \uc601\ud654 \u300a\uadf8\ub54c \uadf8\uc0ac\ub78c\ub4e4\u300b\uc774 \uc544\ubc84\uc9c0 \ubc15\uc815\ud76c\uc758 \uba85\uc608\ub97c \ud6fc\uc190\ud588\ub2e4\uba70 \uc0c1\uc601\uae08\uc9c0 \uac00\ucc98\ubd84 \uc2e0\uccad\uc744 \ub0c8\uc73c\uba70, \ubc95\uc6d0\uc740 \uc77c\ubd80 \uc7a5\uba74\uc744 \uc0ad\uc81c\ud558\ub77c\uace0 \ud310\uacb0\uc744 \ub0b4\ub838\ub2e4. \uadf8\ub7ec\ub098 \uac19\uc740 \ud574 2\uc6d4 MK\ud53d\ucc98\uc2a4\ub294 \uac00\ucc98\ubd84 \uc774\uc758\uc2e0\uccad\uc18c\uc1a1\uc744 \ub0c8\uc73c\uba70, \ubc15\uc9c0\ub9cc\uc740 \uc601\ud654\uc0c1\uc601\uae08\uc9c0 \ubc0f \uc190\ud574\ubc30\uc0c1 \uccad\uad6c\uc18c\uc1a1\uc744 \uc81c\uae30\ud588\ub2e4. 2006\ub144 \uc11c\uc6b8\uc911\uc559\uc9c0\ubc29\ubc95\uc6d0\uc740 \uc190\ud574\ubc30\uc0c1 \uc18c\uc1a1 \uae08\uc561 5\uc5b5\uc6d0 \uc911 1\uc5b5\uc6d0\uc744 \ubc15\uc9c0\ub9cc\uc5d0\uac8c \uc81c\ucd9c\ud558\ub77c\uace0 \uc6d0\uace0 \uc77c\ubd80\uc2b9\uc18c \ud310\uacb0\uc744 \ub0b4\ub838\uc9c0\ub9cc, \uc0c1\uc601\uae08\uc9c0 \uccad\uad6c\uc5d0 \ub300\ud574\uc11c\ub294 \uae30\uac01\ud558\uc600\ub2e4. 2011\ub144 8\uc6d4 19\uc77c \uc11c\uc6b8\uac15\ub0a8\uacbd\ucc30\uc11c\ub294 \ubc15\uc9c0\ub9cc\uc758 \ub204\ub098\uc778 \ubc15\uadfc\ub839\uc758 \ub0a8\ud3b8 \uc2e0\ub3d9\uc6b1 \uc5d0 \ub300\ud574 '2010\ub144 9\uc6d4 \ubc15\uc9c0\ub9cc\uc758 5\ucd0c \uc870\uce74\uc640 \ube44\uc11c\uc2e4\uc7a5\uc774 \uc790\uc2e0\uc744 \uc911\uad6d\uc73c\ub85c \ub0a9\uce58\ud588\uace0, \uc790\uc2e0\uc774 \uc911\uad6d\uc5d0\uc11c \ub9c8\uc57d\uc744 \ud588\ub2e4\ub294 \uc18c\ubb38 \ub4f1\uc744 \ud37c\ub728\ub838\ub2e4\uba70, \ub450 \uc0ac\ub78c\uc744 \uace0\uc18c\ud558\ub294 \ub4f1'\uc758 \ubb34\uace0 \ud610\uc758\ub85c \uad6c\uc18d\ud558\uc600\ub2e4. \uc2e0\ub3d9\uc6b1\uc740 '\ubc15\uc9c0\ub9cc\uc758 \ube44\uc11c\uc2e4\uc7a5 \ub4f1\uc774 2007\ub144 \uc790\uc2e0\uc744 \uc911\uad6d\uc73c\ub85c \ub0a9\uce58\ud558\uc5ec \uc0b4\ud574\ud558\ub824 \ud558\uc600\uace0, \ubc15\uc9c0\ub9cc\uc774 \uc774\ub97c \ub4a4\uc5d0\uc11c \uad50\uc0ac\ud588\ub2e4'\ub294 \uc8fc\uc7a5\ub3c4 \uc81c\uae30\ud558\uc600\uc73c\uba70, \uc5b8\ub860\uacfc\uc758 \uc778\ud130\ubdf0\uc5d0\uc11c \ubc15\uc9c0\ub9cc\uc774 \uc8fd\uc774\ub824\ud588\ub2e4\ub294 \ub179\ucde8\ub85d\uc774 \uc788\ub2e4\uace0 \uc8fc\uc7a5\ud588\uae30 \ub54c\ubb38\uc774\ub2e4.", "sentence_answer": "2011\ub144 8\uc6d4 19\uc77c \uc11c\uc6b8\uac15\ub0a8\uacbd\ucc30\uc11c\ub294 \ubc15\uc9c0\ub9cc\uc758 \ub204\ub098\uc778 \ubc15\uadfc\ub839\uc758 \ub0a8\ud3b8 \uc2e0\ub3d9\uc6b1 \uc5d0", "paragraph_id": "6507726-0-1", "paragraph_question": "question: \ubc15\uc815\ud76c\uc758 \ub538\uc778 \ubc15\uadfc\ub839 \ub0a8\ud3b8\uc758 \uc774\ub984\uc740 \ubb34\uc5c7\uc778\uac00?, context: 2005\ub144 1\uc6d4 \uc601\ud654 \u300a\uadf8\ub54c \uadf8\uc0ac\ub78c\ub4e4\u300b\uc774 \uc544\ubc84\uc9c0 \ubc15\uc815\ud76c\uc758 \uba85\uc608\ub97c \ud6fc\uc190\ud588\ub2e4\uba70 \uc0c1\uc601\uae08\uc9c0 \uac00\ucc98\ubd84 \uc2e0\uccad\uc744 \ub0c8\uc73c\uba70, \ubc95\uc6d0\uc740 \uc77c\ubd80 \uc7a5\uba74\uc744 \uc0ad\uc81c\ud558\ub77c\uace0 \ud310\uacb0\uc744 \ub0b4\ub838\ub2e4. \uadf8\ub7ec\ub098 \uac19\uc740 \ud574 2\uc6d4 MK\ud53d\ucc98\uc2a4\ub294 \uac00\ucc98\ubd84 \uc774\uc758\uc2e0\uccad\uc18c\uc1a1\uc744 \ub0c8\uc73c\uba70, \ubc15\uc9c0\ub9cc\uc740 \uc601\ud654\uc0c1\uc601\uae08\uc9c0 \ubc0f \uc190\ud574\ubc30\uc0c1 \uccad\uad6c\uc18c\uc1a1\uc744 \uc81c\uae30\ud588\ub2e4. 2006\ub144 \uc11c\uc6b8\uc911\uc559\uc9c0\ubc29\ubc95\uc6d0\uc740 \uc190\ud574\ubc30\uc0c1 \uc18c\uc1a1 \uae08\uc561 5\uc5b5\uc6d0 \uc911 1\uc5b5\uc6d0\uc744 \ubc15\uc9c0\ub9cc\uc5d0\uac8c \uc81c\ucd9c\ud558\ub77c\uace0 \uc6d0\uace0 \uc77c\ubd80\uc2b9\uc18c \ud310\uacb0\uc744 \ub0b4\ub838\uc9c0\ub9cc, \uc0c1\uc601\uae08\uc9c0 \uccad\uad6c\uc5d0 \ub300\ud574\uc11c\ub294 \uae30\uac01\ud558\uc600\ub2e4. 2011\ub144 8\uc6d4 19\uc77c \uc11c\uc6b8\uac15\ub0a8\uacbd\ucc30\uc11c\ub294 \ubc15\uc9c0\ub9cc\uc758 \ub204\ub098\uc778 \ubc15\uadfc\ub839\uc758 \ub0a8\ud3b8 \uc2e0\ub3d9\uc6b1\uc5d0 \ub300\ud574 '2010\ub144 9\uc6d4 \ubc15\uc9c0\ub9cc\uc758 5\ucd0c \uc870\uce74\uc640 \ube44\uc11c\uc2e4\uc7a5\uc774 \uc790\uc2e0\uc744 \uc911\uad6d\uc73c\ub85c \ub0a9\uce58\ud588\uace0, \uc790\uc2e0\uc774 \uc911\uad6d\uc5d0\uc11c \ub9c8\uc57d\uc744 \ud588\ub2e4\ub294 \uc18c\ubb38 \ub4f1\uc744 \ud37c\ub728\ub838\ub2e4\uba70, \ub450 \uc0ac\ub78c\uc744 \uace0\uc18c\ud558\ub294 \ub4f1'\uc758 \ubb34\uace0 \ud610\uc758\ub85c \uad6c\uc18d\ud558\uc600\ub2e4. \uc2e0\ub3d9\uc6b1\uc740 '\ubc15\uc9c0\ub9cc\uc758 \ube44\uc11c\uc2e4\uc7a5 \ub4f1\uc774 2007\ub144 \uc790\uc2e0\uc744 \uc911\uad6d\uc73c\ub85c \ub0a9\uce58\ud558\uc5ec \uc0b4\ud574\ud558\ub824 \ud558\uc600\uace0, \ubc15\uc9c0\ub9cc\uc774 \uc774\ub97c \ub4a4\uc5d0\uc11c \uad50\uc0ac\ud588\ub2e4'\ub294 \uc8fc\uc7a5\ub3c4 \uc81c\uae30\ud558\uc600\uc73c\uba70, \uc5b8\ub860\uacfc\uc758 \uc778\ud130\ubdf0\uc5d0\uc11c \ubc15\uc9c0\ub9cc\uc774 \uc8fd\uc774\ub824\ud588\ub2e4\ub294 \ub179\ucde8\ub85d\uc774 \uc788\ub2e4\uace0 \uc8fc\uc7a5\ud588\uae30 \ub54c\ubb38\uc774\ub2e4."} {"question": "\uc5b4\ub290 \ubc45\ud06c\uc2a4\uac00 \uc120\uc218\ub85c\uc11c \uacf5\uc2dd \uc740\ud1f4\ud55c \uc5f0\ub3c4\ub294?", "paragraph": "1968\ub144 \uc5b4\ub2c8 \ubc45\ud06c\uc2a4\ub294 \uadf8 \ub3d9\uc548\uc758 \ud65c\uc57d\uc0c1\uacfc \uacbd\uae30\uc7a5 \uc548\ud30e\uc5d0\uc11c\uc758 \ud6cc\ub96d\ud55c \uc778\ud488\uc73c\ub85c \ub8e8 \uac8c\ub9ad \uba54\ubaa8\ub9ac\uc5bc \uc5b4\uc6cc\ub4dc\ub97c \uc218\uc0c1\ud588\ub2e4. 37\uc138\uc758 \uc5b4\ub2c8 \ubc45\ud06c\uc2a4\ub294 32\ud648\ub7f0\uacfc 83\ud0c0\uc810\uc744 \uae30\ub85d\ud558\uba70 \uc5ec\uc804\ud788 \ud798\uc774 \uc788\uc74c\uc744 \uc99d\uba85\ud588\uc9c0\ub9cc, \ud0c0\uc728\uc740 .246\uc5d0 \uba38\ubb3c\ub800\ub2e4. 1969\ub144 \uc2dc\uc98c\uc740 \ucef5\uc2a4\uac00 \ub0b4\uc154\ub110 \ub9ac\uadf8 \ud398\ub10c\ud2b8 \ub808\uc774\uc2a4 \uc6b0\uc2b9\uc5d0 \uac00\uc7a5 \uadfc\uc811\ud588\ub358 \ud574\uc600\ub2e4. \ucef5\uc2a4\ub294 8\uc6d4\uae4c\uc9c0 2\uc704 \ud300\uacfc 8 \u20442 \uacbd\uae30 \ucc28\ub85c 1\uc704\ub97c \ud558\ub2e4\uac00 \ub9c9\ud310\uc5d0 \ubb34\ub108\uc9c0\uace0 \ub9d0\uc558\ub2e4. \uc774\ubc88 \uc2dc\uc98c\uc5d0\ub3c4 \ubc45\ud06c\uc2a4\ub294 \uc5b4\uae40\uc5c6\uc774 \uc62c\uc2a4\ud0c0\uc804\uc5d0 \ub098\uc130\ub294\ub370 \uc774\ub294 \uadf8\uc758 14\ubc88\uc9f8\uc774\uc790 \ub9c8\uc9c0\ub9c9 \uc62c\uc2a4\ud0c0\uc804 \ucd9c\uc804 \uacbd\uae30\uac00 \ub418\uc5c8\uc73c\uba70, \uacbd\uae30\uc5d0\ub294 \ub300\ud0c0\ub85c\ub9cc \ucd9c\uc804\ud588\ub2e4. 1970\ub144 5\uc6d4 12\uc77c \uc2dc\uce74\uace0\uc5d0 \uc788\ub294 \ud648\uad6c\uc7a5 \ub9ac\uae00\ub9ac \ud544\ub4dc\uc5d0\uc11c \uac1c\uc778 \ud1b5\uc0b0 500\ud648\ub7f0\uacfc 1600\ud0c0\uc810\uc744 \ub3d9\uc2dc\uc5d0 \ub2ec\uc131\ud588\ub2e4. 1971\ub144 12\uc6d4 1\uc77c, \uc5b4\ub2c8 \ubc45\ud06c\uc2a4\ub294 \uc120\uc218\ub85c\uc11c \uacf5\uc2dd\uc801\uc73c\ub85c \uc740\ud1f4\ud588\uc9c0\ub9cc \ucef5\uc2a4\uc5d0\ub294 \ucf54\uce58\ub85c 1973\ub144\uae4c\uc9c0 \ub0a8\uc544 \uc788\uc5c8\ub2e4. \uc774\ud6c4 \ub2e4\uc74c \uc138 \uc2dc\uc98c \ub3d9\uc548 \ub9c8\uc774\ub108 \ub9ac\uadf8\uc5d0\uc11c \uc778\uc2a4\ud2b8\ub7ed\ud130\ub85c \uc120\uc218\ub4e4\uc5d0\uac8c \ube44\ubc95\uc744 \uc804\uc218\ud574\uc8fc\uc5c8\uace0 \ucef5\uc2a4 \ud504\ub7f0\ud2b8\ub85c\ub3c4 \uc77c\ud588\ub2e4.", "answer": "1971\ub144", "sentence": "1971\ub144 12\uc6d4 1\uc77c, \uc5b4\ub2c8 \ubc45\ud06c\uc2a4\ub294 \uc120\uc218\ub85c\uc11c \uacf5\uc2dd\uc801\uc73c\ub85c \uc740\ud1f4\ud588\uc9c0\ub9cc \ucef5\uc2a4\uc5d0\ub294 \ucf54\uce58\ub85c 1973\ub144\uae4c\uc9c0 \ub0a8\uc544 \uc788\uc5c8\ub2e4.", "paragraph_sentence": "1968\ub144 \uc5b4\ub2c8 \ubc45\ud06c\uc2a4\ub294 \uadf8 \ub3d9\uc548\uc758 \ud65c\uc57d\uc0c1\uacfc \uacbd\uae30\uc7a5 \uc548\ud30e\uc5d0\uc11c\uc758 \ud6cc\ub96d\ud55c \uc778\ud488\uc73c\ub85c \ub8e8 \uac8c\ub9ad \uba54\ubaa8\ub9ac\uc5bc \uc5b4\uc6cc\ub4dc\ub97c \uc218\uc0c1\ud588\ub2e4. 37\uc138\uc758 \uc5b4\ub2c8 \ubc45\ud06c\uc2a4\ub294 32\ud648\ub7f0\uacfc 83\ud0c0\uc810\uc744 \uae30\ub85d\ud558\uba70 \uc5ec\uc804\ud788 \ud798\uc774 \uc788\uc74c\uc744 \uc99d\uba85\ud588\uc9c0\ub9cc, \ud0c0\uc728\uc740 .246\uc5d0 \uba38\ubb3c\ub800\ub2e4. 1969\ub144 \uc2dc\uc98c\uc740 \ucef5\uc2a4\uac00 \ub0b4\uc154\ub110 \ub9ac\uadf8 \ud398\ub10c\ud2b8 \ub808\uc774\uc2a4 \uc6b0\uc2b9\uc5d0 \uac00\uc7a5 \uadfc\uc811\ud588\ub358 \ud574\uc600\ub2e4. \ucef5\uc2a4\ub294 8\uc6d4\uae4c\uc9c0 2\uc704 \ud300\uacfc 8 \u20442 \uacbd\uae30 \ucc28\ub85c 1\uc704\ub97c \ud558\ub2e4\uac00 \ub9c9\ud310\uc5d0 \ubb34\ub108\uc9c0\uace0 \ub9d0\uc558\ub2e4. \uc774\ubc88 \uc2dc\uc98c\uc5d0\ub3c4 \ubc45\ud06c\uc2a4\ub294 \uc5b4\uae40\uc5c6\uc774 \uc62c\uc2a4\ud0c0\uc804\uc5d0 \ub098\uc130\ub294\ub370 \uc774\ub294 \uadf8\uc758 14\ubc88\uc9f8\uc774\uc790 \ub9c8\uc9c0\ub9c9 \uc62c\uc2a4\ud0c0\uc804 \ucd9c\uc804 \uacbd\uae30\uac00 \ub418\uc5c8\uc73c\uba70, \uacbd\uae30\uc5d0\ub294 \ub300\ud0c0\ub85c\ub9cc \ucd9c\uc804\ud588\ub2e4. 1970\ub144 5\uc6d4 12\uc77c \uc2dc\uce74\uace0\uc5d0 \uc788\ub294 \ud648\uad6c\uc7a5 \ub9ac\uae00\ub9ac \ud544\ub4dc\uc5d0\uc11c \uac1c\uc778 \ud1b5\uc0b0 500\ud648\ub7f0\uacfc 1600\ud0c0\uc810\uc744 \ub3d9\uc2dc\uc5d0 \ub2ec\uc131\ud588\ub2e4. 1971\ub144 12\uc6d4 1\uc77c, \uc5b4\ub2c8 \ubc45\ud06c\uc2a4\ub294 \uc120\uc218\ub85c\uc11c \uacf5\uc2dd\uc801\uc73c\ub85c \uc740\ud1f4\ud588\uc9c0\ub9cc \ucef5\uc2a4\uc5d0\ub294 \ucf54\uce58\ub85c 1973\ub144\uae4c\uc9c0 \ub0a8\uc544 \uc788\uc5c8\ub2e4. \uc774\ud6c4 \ub2e4\uc74c \uc138 \uc2dc\uc98c \ub3d9\uc548 \ub9c8\uc774\ub108 \ub9ac\uadf8\uc5d0\uc11c \uc778\uc2a4\ud2b8\ub7ed\ud130\ub85c \uc120\uc218\ub4e4\uc5d0\uac8c \ube44\ubc95\uc744 \uc804\uc218\ud574\uc8fc\uc5c8\uace0 \ucef5\uc2a4 \ud504\ub7f0\ud2b8\ub85c\ub3c4 \uc77c\ud588\ub2e4.", "paragraph_answer": "1968\ub144 \uc5b4\ub2c8 \ubc45\ud06c\uc2a4\ub294 \uadf8 \ub3d9\uc548\uc758 \ud65c\uc57d\uc0c1\uacfc \uacbd\uae30\uc7a5 \uc548\ud30e\uc5d0\uc11c\uc758 \ud6cc\ub96d\ud55c \uc778\ud488\uc73c\ub85c \ub8e8 \uac8c\ub9ad \uba54\ubaa8\ub9ac\uc5bc \uc5b4\uc6cc\ub4dc\ub97c \uc218\uc0c1\ud588\ub2e4. 37\uc138\uc758 \uc5b4\ub2c8 \ubc45\ud06c\uc2a4\ub294 32\ud648\ub7f0\uacfc 83\ud0c0\uc810\uc744 \uae30\ub85d\ud558\uba70 \uc5ec\uc804\ud788 \ud798\uc774 \uc788\uc74c\uc744 \uc99d\uba85\ud588\uc9c0\ub9cc, \ud0c0\uc728\uc740 .246\uc5d0 \uba38\ubb3c\ub800\ub2e4. 1969\ub144 \uc2dc\uc98c\uc740 \ucef5\uc2a4\uac00 \ub0b4\uc154\ub110 \ub9ac\uadf8 \ud398\ub10c\ud2b8 \ub808\uc774\uc2a4 \uc6b0\uc2b9\uc5d0 \uac00\uc7a5 \uadfc\uc811\ud588\ub358 \ud574\uc600\ub2e4. \ucef5\uc2a4\ub294 8\uc6d4\uae4c\uc9c0 2\uc704 \ud300\uacfc 8 \u20442 \uacbd\uae30 \ucc28\ub85c 1\uc704\ub97c \ud558\ub2e4\uac00 \ub9c9\ud310\uc5d0 \ubb34\ub108\uc9c0\uace0 \ub9d0\uc558\ub2e4. \uc774\ubc88 \uc2dc\uc98c\uc5d0\ub3c4 \ubc45\ud06c\uc2a4\ub294 \uc5b4\uae40\uc5c6\uc774 \uc62c\uc2a4\ud0c0\uc804\uc5d0 \ub098\uc130\ub294\ub370 \uc774\ub294 \uadf8\uc758 14\ubc88\uc9f8\uc774\uc790 \ub9c8\uc9c0\ub9c9 \uc62c\uc2a4\ud0c0\uc804 \ucd9c\uc804 \uacbd\uae30\uac00 \ub418\uc5c8\uc73c\uba70, \uacbd\uae30\uc5d0\ub294 \ub300\ud0c0\ub85c\ub9cc \ucd9c\uc804\ud588\ub2e4. 1970\ub144 5\uc6d4 12\uc77c \uc2dc\uce74\uace0\uc5d0 \uc788\ub294 \ud648\uad6c\uc7a5 \ub9ac\uae00\ub9ac \ud544\ub4dc\uc5d0\uc11c \uac1c\uc778 \ud1b5\uc0b0 500\ud648\ub7f0\uacfc 1600\ud0c0\uc810\uc744 \ub3d9\uc2dc\uc5d0 \ub2ec\uc131\ud588\ub2e4. 1971\ub144 12\uc6d4 1\uc77c, \uc5b4\ub2c8 \ubc45\ud06c\uc2a4\ub294 \uc120\uc218\ub85c\uc11c \uacf5\uc2dd\uc801\uc73c\ub85c \uc740\ud1f4\ud588\uc9c0\ub9cc \ucef5\uc2a4\uc5d0\ub294 \ucf54\uce58\ub85c 1973\ub144\uae4c\uc9c0 \ub0a8\uc544 \uc788\uc5c8\ub2e4. \uc774\ud6c4 \ub2e4\uc74c \uc138 \uc2dc\uc98c \ub3d9\uc548 \ub9c8\uc774\ub108 \ub9ac\uadf8\uc5d0\uc11c \uc778\uc2a4\ud2b8\ub7ed\ud130\ub85c \uc120\uc218\ub4e4\uc5d0\uac8c \ube44\ubc95\uc744 \uc804\uc218\ud574\uc8fc\uc5c8\uace0 \ucef5\uc2a4 \ud504\ub7f0\ud2b8\ub85c\ub3c4 \uc77c\ud588\ub2e4.", "sentence_answer": " 1971\ub144 12\uc6d4 1\uc77c, \uc5b4\ub2c8 \ubc45\ud06c\uc2a4\ub294 \uc120\uc218\ub85c\uc11c \uacf5\uc2dd\uc801\uc73c\ub85c \uc740\ud1f4\ud588\uc9c0\ub9cc \ucef5\uc2a4\uc5d0\ub294 \ucf54\uce58\ub85c 1973\ub144\uae4c\uc9c0 \ub0a8\uc544 \uc788\uc5c8\ub2e4.", "paragraph_id": "6540574-9-2", "paragraph_question": "question: \uc5b4\ub290 \ubc45\ud06c\uc2a4\uac00 \uc120\uc218\ub85c\uc11c \uacf5\uc2dd \uc740\ud1f4\ud55c \uc5f0\ub3c4\ub294?, context: 1968\ub144 \uc5b4\ub2c8 \ubc45\ud06c\uc2a4\ub294 \uadf8 \ub3d9\uc548\uc758 \ud65c\uc57d\uc0c1\uacfc \uacbd\uae30\uc7a5 \uc548\ud30e\uc5d0\uc11c\uc758 \ud6cc\ub96d\ud55c \uc778\ud488\uc73c\ub85c \ub8e8 \uac8c\ub9ad \uba54\ubaa8\ub9ac\uc5bc \uc5b4\uc6cc\ub4dc\ub97c \uc218\uc0c1\ud588\ub2e4. 37\uc138\uc758 \uc5b4\ub2c8 \ubc45\ud06c\uc2a4\ub294 32\ud648\ub7f0\uacfc 83\ud0c0\uc810\uc744 \uae30\ub85d\ud558\uba70 \uc5ec\uc804\ud788 \ud798\uc774 \uc788\uc74c\uc744 \uc99d\uba85\ud588\uc9c0\ub9cc, \ud0c0\uc728\uc740 .246\uc5d0 \uba38\ubb3c\ub800\ub2e4. 1969\ub144 \uc2dc\uc98c\uc740 \ucef5\uc2a4\uac00 \ub0b4\uc154\ub110 \ub9ac\uadf8 \ud398\ub10c\ud2b8 \ub808\uc774\uc2a4 \uc6b0\uc2b9\uc5d0 \uac00\uc7a5 \uadfc\uc811\ud588\ub358 \ud574\uc600\ub2e4. \ucef5\uc2a4\ub294 8\uc6d4\uae4c\uc9c0 2\uc704 \ud300\uacfc 8 \u20442 \uacbd\uae30 \ucc28\ub85c 1\uc704\ub97c \ud558\ub2e4\uac00 \ub9c9\ud310\uc5d0 \ubb34\ub108\uc9c0\uace0 \ub9d0\uc558\ub2e4. \uc774\ubc88 \uc2dc\uc98c\uc5d0\ub3c4 \ubc45\ud06c\uc2a4\ub294 \uc5b4\uae40\uc5c6\uc774 \uc62c\uc2a4\ud0c0\uc804\uc5d0 \ub098\uc130\ub294\ub370 \uc774\ub294 \uadf8\uc758 14\ubc88\uc9f8\uc774\uc790 \ub9c8\uc9c0\ub9c9 \uc62c\uc2a4\ud0c0\uc804 \ucd9c\uc804 \uacbd\uae30\uac00 \ub418\uc5c8\uc73c\uba70, \uacbd\uae30\uc5d0\ub294 \ub300\ud0c0\ub85c\ub9cc \ucd9c\uc804\ud588\ub2e4. 1970\ub144 5\uc6d4 12\uc77c \uc2dc\uce74\uace0\uc5d0 \uc788\ub294 \ud648\uad6c\uc7a5 \ub9ac\uae00\ub9ac \ud544\ub4dc\uc5d0\uc11c \uac1c\uc778 \ud1b5\uc0b0 500\ud648\ub7f0\uacfc 1600\ud0c0\uc810\uc744 \ub3d9\uc2dc\uc5d0 \ub2ec\uc131\ud588\ub2e4. 1971\ub144 12\uc6d4 1\uc77c, \uc5b4\ub2c8 \ubc45\ud06c\uc2a4\ub294 \uc120\uc218\ub85c\uc11c \uacf5\uc2dd\uc801\uc73c\ub85c \uc740\ud1f4\ud588\uc9c0\ub9cc \ucef5\uc2a4\uc5d0\ub294 \ucf54\uce58\ub85c 1973\ub144\uae4c\uc9c0 \ub0a8\uc544 \uc788\uc5c8\ub2e4. \uc774\ud6c4 \ub2e4\uc74c \uc138 \uc2dc\uc98c \ub3d9\uc548 \ub9c8\uc774\ub108 \ub9ac\uadf8\uc5d0\uc11c \uc778\uc2a4\ud2b8\ub7ed\ud130\ub85c \uc120\uc218\ub4e4\uc5d0\uac8c \ube44\ubc95\uc744 \uc804\uc218\ud574\uc8fc\uc5c8\uace0 \ucef5\uc2a4 \ud504\ub7f0\ud2b8\ub85c\ub3c4 \uc77c\ud588\ub2e4."} {"question": "\uba69\uac94\ub808\uac00 \ub098\uc628 \uace0\ub4f1\ud559\uad50\ub294 \uc5b4\ub514\uc778\uac00?", "paragraph": "1930\ub144 \uadc4\uce20\ubd80\ub974\ud06c \uae40\ub098\uc9c0\uc6c0(\uace0\ub4f1\ud559\uad50)\uc744 \uc878\uc5c5\ud55c \uadf8\ub294 \ubb8c\ud5e8 \ub300\ud559(University of Munich)\uc5d0\uc11c \uc57d\ud559\uacfc \uc778\ub958\ud559\uc744 \uacf5\ubd80\ud558\uc600\uc73c\uba70, \ud14c\uc624\ub3c4\ub974 \ubab0\ub9ac\uc2a8(Theodor Mollison) \uad50\uc218\uc758 \uc9c0\ub3c4\ub85c 1935\ub144 \uc720\ud0dc\uc778 \ud558\uce35\ubbfc\ub4e4\uc758 \uc778\uc885\uc801 \ucc28\uc774\uc810\uc5d0 \ub300\ud55c \ub17c\ubb38\uc744 \uc791\uc131, \uc778\ub958\ud559 \ubc15\uc0ac \ud559\uc704\ub97c \ubc1b\uc558\ub2e4. \ud504\ub791\ud06c\ud478\ub974\ud2b8 \ub300\ud559(Frankfurt University)\uc758 \uc720\uc804 \uc0dd\ubb3c\ud559 \ubc0f \uc778\uc885 \uc704\uc0dd\ud559 \uc5f0\uad6c\uc18c\uc5d0\uc11c \uc624\ud2b8\ub9c8 \ud3f0 \ud398\ub974\uc288\uc5b4(Otmar von Verschuer)\uc640 \ud568\uaed8 \uc5f0\uad6c\ub97c \uc9c4\ud589\ud55c \uadf8\ub294 1938\ub144 '\uac08\ub77c\uc9c4 \uc785\uc220\uacfc \uad6c\uac1c\uc5d0 \uad00\ud55c \uac00\uc871\uc0ac \uc5f0\uad6c'\ub77c\ub294 \ub17c\ubb38\uc73c\ub85c \uc57d\ud559 \ubc15\uc0ac \ud559\uc704\ub97c \ubc1b\uc558\ub2e4. \ub098\uce58 \uc6b0\uc0dd\ud559\uc5d0 \ub300\ud55c \uadf8\uc758 \ubbff\uc74c\uc740 \uc774 \uc2dc\uae30\uc758 \uadf8\uc758 \ub17c\ubb38\uc5d0\uc11c\ub3c4 \ucc3e\uc544\ubcfc \uc218\uac00 \uc788\uc5c8\ub2e4. \ubb8c\ud5e8 \ub300\ud559\uacfc \ud504\ub791\ud06c\ud478\ub974\ud2b8 \ub300\ud559\uc740 \uc804\ud6c4 1964\ub144 \uadf8\uc758 \ud559\uc704\ub97c \ucde8\uc18c\ud558\uc600\ub2e4.", "answer": "\uadc4\uce20\ubd80\ub974\ud06c \uae40\ub098\uc9c0\uc6c0", "sentence": "1930\ub144 \uadc4\uce20\ubd80\ub974\ud06c \uae40\ub098\uc9c0\uc6c0 (\uace0\ub4f1\ud559\uad50)", "paragraph_sentence": " 1930\ub144 \uadc4\uce20\ubd80\ub974\ud06c \uae40\ub098\uc9c0\uc6c0 (\uace0\ub4f1\ud559\uad50) \uc744 \uc878\uc5c5\ud55c \uadf8\ub294 \ubb8c\ud5e8 \ub300\ud559(University of Munich)\uc5d0\uc11c \uc57d\ud559\uacfc \uc778\ub958\ud559\uc744 \uacf5\ubd80\ud558\uc600\uc73c\uba70, \ud14c\uc624\ub3c4\ub974 \ubab0\ub9ac\uc2a8(Theodor Mollison) \uad50\uc218\uc758 \uc9c0\ub3c4\ub85c 1935\ub144 \uc720\ud0dc\uc778 \ud558\uce35\ubbfc\ub4e4\uc758 \uc778\uc885\uc801 \ucc28\uc774\uc810\uc5d0 \ub300\ud55c \ub17c\ubb38\uc744 \uc791\uc131, \uc778\ub958\ud559 \ubc15\uc0ac \ud559\uc704\ub97c \ubc1b\uc558\ub2e4. \ud504\ub791\ud06c\ud478\ub974\ud2b8 \ub300\ud559(Frankfurt University)\uc758 \uc720\uc804 \uc0dd\ubb3c\ud559 \ubc0f \uc778\uc885 \uc704\uc0dd\ud559 \uc5f0\uad6c\uc18c\uc5d0\uc11c \uc624\ud2b8\ub9c8 \ud3f0 \ud398\ub974\uc288\uc5b4(Otmar von Verschuer)\uc640 \ud568\uaed8 \uc5f0\uad6c\ub97c \uc9c4\ud589\ud55c \uadf8\ub294 1938\ub144 '\uac08\ub77c\uc9c4 \uc785\uc220\uacfc \uad6c\uac1c\uc5d0 \uad00\ud55c \uac00\uc871\uc0ac \uc5f0\uad6c'\ub77c\ub294 \ub17c\ubb38\uc73c\ub85c \uc57d\ud559 \ubc15\uc0ac \ud559\uc704\ub97c \ubc1b\uc558\ub2e4. \ub098\uce58 \uc6b0\uc0dd\ud559\uc5d0 \ub300\ud55c \uadf8\uc758 \ubbff\uc74c\uc740 \uc774 \uc2dc\uae30\uc758 \uadf8\uc758 \ub17c\ubb38\uc5d0\uc11c\ub3c4 \ucc3e\uc544\ubcfc \uc218\uac00 \uc788\uc5c8\ub2e4. \ubb8c\ud5e8 \ub300\ud559\uacfc \ud504\ub791\ud06c\ud478\ub974\ud2b8 \ub300\ud559\uc740 \uc804\ud6c4 1964\ub144 \uadf8\uc758 \ud559\uc704\ub97c \ucde8\uc18c\ud558\uc600\ub2e4.", "paragraph_answer": "1930\ub144 \uadc4\uce20\ubd80\ub974\ud06c \uae40\ub098\uc9c0\uc6c0 (\uace0\ub4f1\ud559\uad50)\uc744 \uc878\uc5c5\ud55c \uadf8\ub294 \ubb8c\ud5e8 \ub300\ud559(University of Munich)\uc5d0\uc11c \uc57d\ud559\uacfc \uc778\ub958\ud559\uc744 \uacf5\ubd80\ud558\uc600\uc73c\uba70, \ud14c\uc624\ub3c4\ub974 \ubab0\ub9ac\uc2a8(Theodor Mollison) \uad50\uc218\uc758 \uc9c0\ub3c4\ub85c 1935\ub144 \uc720\ud0dc\uc778 \ud558\uce35\ubbfc\ub4e4\uc758 \uc778\uc885\uc801 \ucc28\uc774\uc810\uc5d0 \ub300\ud55c \ub17c\ubb38\uc744 \uc791\uc131, \uc778\ub958\ud559 \ubc15\uc0ac \ud559\uc704\ub97c \ubc1b\uc558\ub2e4. \ud504\ub791\ud06c\ud478\ub974\ud2b8 \ub300\ud559(Frankfurt University)\uc758 \uc720\uc804 \uc0dd\ubb3c\ud559 \ubc0f \uc778\uc885 \uc704\uc0dd\ud559 \uc5f0\uad6c\uc18c\uc5d0\uc11c \uc624\ud2b8\ub9c8 \ud3f0 \ud398\ub974\uc288\uc5b4(Otmar von Verschuer)\uc640 \ud568\uaed8 \uc5f0\uad6c\ub97c \uc9c4\ud589\ud55c \uadf8\ub294 1938\ub144 '\uac08\ub77c\uc9c4 \uc785\uc220\uacfc \uad6c\uac1c\uc5d0 \uad00\ud55c \uac00\uc871\uc0ac \uc5f0\uad6c'\ub77c\ub294 \ub17c\ubb38\uc73c\ub85c \uc57d\ud559 \ubc15\uc0ac \ud559\uc704\ub97c \ubc1b\uc558\ub2e4. \ub098\uce58 \uc6b0\uc0dd\ud559\uc5d0 \ub300\ud55c \uadf8\uc758 \ubbff\uc74c\uc740 \uc774 \uc2dc\uae30\uc758 \uadf8\uc758 \ub17c\ubb38\uc5d0\uc11c\ub3c4 \ucc3e\uc544\ubcfc \uc218\uac00 \uc788\uc5c8\ub2e4. \ubb8c\ud5e8 \ub300\ud559\uacfc \ud504\ub791\ud06c\ud478\ub974\ud2b8 \ub300\ud559\uc740 \uc804\ud6c4 1964\ub144 \uadf8\uc758 \ud559\uc704\ub97c \ucde8\uc18c\ud558\uc600\ub2e4.", "sentence_answer": "1930\ub144 \uadc4\uce20\ubd80\ub974\ud06c \uae40\ub098\uc9c0\uc6c0 (\uace0\ub4f1\ud559\uad50)", "paragraph_id": "6525997-0-0", "paragraph_question": "question: \uba69\uac94\ub808\uac00 \ub098\uc628 \uace0\ub4f1\ud559\uad50\ub294 \uc5b4\ub514\uc778\uac00?, context: 1930\ub144 \uadc4\uce20\ubd80\ub974\ud06c \uae40\ub098\uc9c0\uc6c0(\uace0\ub4f1\ud559\uad50)\uc744 \uc878\uc5c5\ud55c \uadf8\ub294 \ubb8c\ud5e8 \ub300\ud559(University of Munich)\uc5d0\uc11c \uc57d\ud559\uacfc \uc778\ub958\ud559\uc744 \uacf5\ubd80\ud558\uc600\uc73c\uba70, \ud14c\uc624\ub3c4\ub974 \ubab0\ub9ac\uc2a8(Theodor Mollison) \uad50\uc218\uc758 \uc9c0\ub3c4\ub85c 1935\ub144 \uc720\ud0dc\uc778 \ud558\uce35\ubbfc\ub4e4\uc758 \uc778\uc885\uc801 \ucc28\uc774\uc810\uc5d0 \ub300\ud55c \ub17c\ubb38\uc744 \uc791\uc131, \uc778\ub958\ud559 \ubc15\uc0ac \ud559\uc704\ub97c \ubc1b\uc558\ub2e4. \ud504\ub791\ud06c\ud478\ub974\ud2b8 \ub300\ud559(Frankfurt University)\uc758 \uc720\uc804 \uc0dd\ubb3c\ud559 \ubc0f \uc778\uc885 \uc704\uc0dd\ud559 \uc5f0\uad6c\uc18c\uc5d0\uc11c \uc624\ud2b8\ub9c8 \ud3f0 \ud398\ub974\uc288\uc5b4(Otmar von Verschuer)\uc640 \ud568\uaed8 \uc5f0\uad6c\ub97c \uc9c4\ud589\ud55c \uadf8\ub294 1938\ub144 '\uac08\ub77c\uc9c4 \uc785\uc220\uacfc \uad6c\uac1c\uc5d0 \uad00\ud55c \uac00\uc871\uc0ac \uc5f0\uad6c'\ub77c\ub294 \ub17c\ubb38\uc73c\ub85c \uc57d\ud559 \ubc15\uc0ac \ud559\uc704\ub97c \ubc1b\uc558\ub2e4. \ub098\uce58 \uc6b0\uc0dd\ud559\uc5d0 \ub300\ud55c \uadf8\uc758 \ubbff\uc74c\uc740 \uc774 \uc2dc\uae30\uc758 \uadf8\uc758 \ub17c\ubb38\uc5d0\uc11c\ub3c4 \ucc3e\uc544\ubcfc \uc218\uac00 \uc788\uc5c8\ub2e4. \ubb8c\ud5e8 \ub300\ud559\uacfc \ud504\ub791\ud06c\ud478\ub974\ud2b8 \ub300\ud559\uc740 \uc804\ud6c4 1964\ub144 \uadf8\uc758 \ud559\uc704\ub97c \ucde8\uc18c\ud558\uc600\ub2e4."} {"question": "1926-27 \uc2dc\uc98c \ub9e8\uccb4\uc2a4\ud130 \uc2dc\ud2f0\uc758 \ub9ac\uadf8 \ub9c8\uc9c0\ub9c9 \uacbd\uae30 \uc0c1\ub300\ub294?", "paragraph": "1926\ub144, \ub9e8\uccb4\uc2a4\ud130 \uc2dc\ud2f0\ub294 FA\ucef5 \uacb0\uc2b9\uc5d0\uc11c \ub2e4\uc2dc \ubcfc\ud134 \uc6d0\ub354\ub7ec\uc2a4\uc640 \uc7ac\ud68c\ud558\uc9c0\ub9cc \uc774\ubc88\uc5d4 0-1\ub85c \ud328\ud558\uac8c \ub41c\ub2e4. \ud558\uc9c0\ub9cc \uc774\ud6c4\ubd80\ud134 \uae30\ubcf5\uc774 \uc2ec\ud55c \ud50c\ub808\uc774\ub85c \ub9ac\uadf8\uc5d0\uc11c \ubd80\uc9c4\uc774 \uc787\ub530\ub974\uac8c \ub418\ub294\ub370 \uc2dc\uc98c \ub9c8\uc9c0\ub9c9 \ub0a0, \uac15\ub4f1\uc774 \ub418\uace0 \ub9cc\ub2e4. 1926 - 27\uc2dc\uc98c\uc5d0\ub294 1\ubd80\ub9ac\uadf8 \uc2b9\uaca9\uc744 \ubaa9\uc804\uc5d0 \ub450\uace0 \uce58\uc5f4\ud55c \ub2e4\ud23c\uc774 \ubc8c\uc5b4\uc84c\uc5c8\ub2e4. \uc2b9\uaca9 \uacbd\uc7c1\uc740 \uc2dc\uc98c \ub9c8\uc9c0\ub9c9 \ub0a0\uae4c\uc9c0 \uc774\uc5b4\uc84c\ub294\ub370 \ub9e8\uccb4\uc2a4\ud130 \uc2dc\ud2f0\uc640 \ud3ec\uce20\uba38\uc2a4\uac00 2\uc704\ub97c \ub193\uace0 \ub2e4\ud23c\uc744 \ubc8c\uc774\uace0 \uc788\uc5c8\ub2e4. \ub9e8\uccb4\uc2a4\ud130 \uc2dc\ud2f0\ub294 \ub9c8\uc9c0\ub9c9 \ub0a0 \ube0c\ub798\ub4dc\ud37c\ub4dc \uc2dc\ud2f0\ub97c \ubb34\ub824 8-0\uc73c\ub85c \uaebe\uc5c8\uace0 \ub9e8\uccb4\uc2a4\ud130 \uc2dc\ud2f0 \ud32c\ub4e4\uc740 \uc2b9\uaca9\uc774 \ud655\uc815\ub41c\uac78\ub85c \ubbff\uace0 \uc788\uc5c8\uc9c0\ub9cc \uac19\uc740 \uc2dc\uac01, \ud3ec\uce20\uba38\uc2a4\ub294 15\ubd84\uc774\ub098 \uacbd\uae30\uac00 \uc9c0\uc5f0\ub418\uc5b4 \uc544\uc9c1 \uacbd\uae30\uac00 \uc9c4\ud589 \uc911\uc774\uc5c8\ub2e4. \uacbd\uae30 \ub9c9\ud310\uc5d0 \ud130\uc9c4 \uace8\ub85c 5-1 \uc2b9\ub9ac\ub97c \uac70\ub454 \ud3ec\uce20\uba38\uc2a4\ub294 \uace8 \ud3c9\uade0\ucc28 (\ucd1d \uc2e4\uc810 \uc218\uc640 \ub4dd\uc810 \uc218\ub97c \ub098\ub204\ub294 \ubc29\ubc95\uc73c\ub85c\uc368 \ub2f9\uc2dc\uc5d0\ub294 \uc774 \ub8f0\uc774 \uc801\uc6a9\ub418\uc5c8\uc73c\uba70, \uace8 \ub4dd\uc2e4\ucc28\uac00 \uc801\uc6a9\uc774 \ub41c \uac83\uc740 1970\ub144 \uba55\uc2dc\ucf54 \uc6d4\ub4dc\ucef5, \uc789\uae00\ub79c\ub4dc\uc5d0\ub294 1976 - 77\uc2dc\uc98c\ubd80\ud130\uc600\ub2e4.)\ub85c \uc778\ud574 2\uc704\ub97c \ucc28\uc9c0\ud558\uc5ec \ud3ec\uce20\uba38\uc2a4\uac00 \uc2b9\uaca9\ud558\uac8c \ub418\uc5c8\ub2e4.", "answer": "\ube0c\ub798\ub4dc\ud37c\ub4dc \uc2dc\ud2f0", "sentence": "\ub9e8\uccb4\uc2a4\ud130 \uc2dc\ud2f0\ub294 \ub9c8\uc9c0\ub9c9 \ub0a0 \ube0c\ub798\ub4dc\ud37c\ub4dc \uc2dc\ud2f0 \ub97c \ubb34\ub824 8-0\uc73c\ub85c \uaebe\uc5c8\uace0 \ub9e8\uccb4\uc2a4\ud130 \uc2dc\ud2f0 \ud32c\ub4e4\uc740 \uc2b9\uaca9\uc774 \ud655\uc815\ub41c\uac78\ub85c \ubbff\uace0 \uc788\uc5c8\uc9c0\ub9cc \uac19\uc740 \uc2dc\uac01, \ud3ec\uce20\uba38\uc2a4\ub294 15\ubd84\uc774\ub098 \uacbd\uae30\uac00 \uc9c0\uc5f0\ub418\uc5b4 \uc544\uc9c1 \uacbd\uae30\uac00 \uc9c4\ud589 \uc911\uc774\uc5c8\ub2e4.", "paragraph_sentence": "1926\ub144, \ub9e8\uccb4\uc2a4\ud130 \uc2dc\ud2f0\ub294 FA\ucef5 \uacb0\uc2b9\uc5d0\uc11c \ub2e4\uc2dc \ubcfc\ud134 \uc6d0\ub354\ub7ec\uc2a4\uc640 \uc7ac\ud68c\ud558\uc9c0\ub9cc \uc774\ubc88\uc5d4 0-1\ub85c \ud328\ud558\uac8c \ub41c\ub2e4. \ud558\uc9c0\ub9cc \uc774\ud6c4\ubd80\ud134 \uae30\ubcf5\uc774 \uc2ec\ud55c \ud50c\ub808\uc774\ub85c \ub9ac\uadf8\uc5d0\uc11c \ubd80\uc9c4\uc774 \uc787\ub530\ub974\uac8c \ub418\ub294\ub370 \uc2dc\uc98c \ub9c8\uc9c0\ub9c9 \ub0a0, \uac15\ub4f1\uc774 \ub418\uace0 \ub9cc\ub2e4. 1926 - 27\uc2dc\uc98c\uc5d0\ub294 1\ubd80\ub9ac\uadf8 \uc2b9\uaca9\uc744 \ubaa9\uc804\uc5d0 \ub450\uace0 \uce58\uc5f4\ud55c \ub2e4\ud23c\uc774 \ubc8c\uc5b4\uc84c\uc5c8\ub2e4. \uc2b9\uaca9 \uacbd\uc7c1\uc740 \uc2dc\uc98c \ub9c8\uc9c0\ub9c9 \ub0a0\uae4c\uc9c0 \uc774\uc5b4\uc84c\ub294\ub370 \ub9e8\uccb4\uc2a4\ud130 \uc2dc\ud2f0\uc640 \ud3ec\uce20\uba38\uc2a4\uac00 2\uc704\ub97c \ub193\uace0 \ub2e4\ud23c\uc744 \ubc8c\uc774\uace0 \uc788\uc5c8\ub2e4. \ub9e8\uccb4\uc2a4\ud130 \uc2dc\ud2f0\ub294 \ub9c8\uc9c0\ub9c9 \ub0a0 \ube0c\ub798\ub4dc\ud37c\ub4dc \uc2dc\ud2f0 \ub97c \ubb34\ub824 8-0\uc73c\ub85c \uaebe\uc5c8\uace0 \ub9e8\uccb4\uc2a4\ud130 \uc2dc\ud2f0 \ud32c\ub4e4\uc740 \uc2b9\uaca9\uc774 \ud655\uc815\ub41c\uac78\ub85c \ubbff\uace0 \uc788\uc5c8\uc9c0\ub9cc \uac19\uc740 \uc2dc\uac01, \ud3ec\uce20\uba38\uc2a4\ub294 15\ubd84\uc774\ub098 \uacbd\uae30\uac00 \uc9c0\uc5f0\ub418\uc5b4 \uc544\uc9c1 \uacbd\uae30\uac00 \uc9c4\ud589 \uc911\uc774\uc5c8\ub2e4. \uacbd\uae30 \ub9c9\ud310\uc5d0 \ud130\uc9c4 \uace8\ub85c 5-1 \uc2b9\ub9ac\ub97c \uac70\ub454 \ud3ec\uce20\uba38\uc2a4\ub294 \uace8 \ud3c9\uade0\ucc28 (\ucd1d \uc2e4\uc810 \uc218\uc640 \ub4dd\uc810 \uc218\ub97c \ub098\ub204\ub294 \ubc29\ubc95\uc73c\ub85c\uc368 \ub2f9\uc2dc\uc5d0\ub294 \uc774 \ub8f0\uc774 \uc801\uc6a9\ub418\uc5c8\uc73c\uba70, \uace8 \ub4dd\uc2e4\ucc28\uac00 \uc801\uc6a9\uc774 \ub41c \uac83\uc740 1970\ub144 \uba55\uc2dc\ucf54 \uc6d4\ub4dc\ucef5, \uc789\uae00\ub79c\ub4dc\uc5d0\ub294 1976 - 77\uc2dc\uc98c\ubd80\ud130\uc600\ub2e4.)\ub85c \uc778\ud574 2\uc704\ub97c \ucc28\uc9c0\ud558\uc5ec \ud3ec\uce20\uba38\uc2a4\uac00 \uc2b9\uaca9\ud558\uac8c \ub418\uc5c8\ub2e4.", "paragraph_answer": "1926\ub144, \ub9e8\uccb4\uc2a4\ud130 \uc2dc\ud2f0\ub294 FA\ucef5 \uacb0\uc2b9\uc5d0\uc11c \ub2e4\uc2dc \ubcfc\ud134 \uc6d0\ub354\ub7ec\uc2a4\uc640 \uc7ac\ud68c\ud558\uc9c0\ub9cc \uc774\ubc88\uc5d4 0-1\ub85c \ud328\ud558\uac8c \ub41c\ub2e4. \ud558\uc9c0\ub9cc \uc774\ud6c4\ubd80\ud134 \uae30\ubcf5\uc774 \uc2ec\ud55c \ud50c\ub808\uc774\ub85c \ub9ac\uadf8\uc5d0\uc11c \ubd80\uc9c4\uc774 \uc787\ub530\ub974\uac8c \ub418\ub294\ub370 \uc2dc\uc98c \ub9c8\uc9c0\ub9c9 \ub0a0, \uac15\ub4f1\uc774 \ub418\uace0 \ub9cc\ub2e4. 1926 - 27\uc2dc\uc98c\uc5d0\ub294 1\ubd80\ub9ac\uadf8 \uc2b9\uaca9\uc744 \ubaa9\uc804\uc5d0 \ub450\uace0 \uce58\uc5f4\ud55c \ub2e4\ud23c\uc774 \ubc8c\uc5b4\uc84c\uc5c8\ub2e4. \uc2b9\uaca9 \uacbd\uc7c1\uc740 \uc2dc\uc98c \ub9c8\uc9c0\ub9c9 \ub0a0\uae4c\uc9c0 \uc774\uc5b4\uc84c\ub294\ub370 \ub9e8\uccb4\uc2a4\ud130 \uc2dc\ud2f0\uc640 \ud3ec\uce20\uba38\uc2a4\uac00 2\uc704\ub97c \ub193\uace0 \ub2e4\ud23c\uc744 \ubc8c\uc774\uace0 \uc788\uc5c8\ub2e4. \ub9e8\uccb4\uc2a4\ud130 \uc2dc\ud2f0\ub294 \ub9c8\uc9c0\ub9c9 \ub0a0 \ube0c\ub798\ub4dc\ud37c\ub4dc \uc2dc\ud2f0 \ub97c \ubb34\ub824 8-0\uc73c\ub85c \uaebe\uc5c8\uace0 \ub9e8\uccb4\uc2a4\ud130 \uc2dc\ud2f0 \ud32c\ub4e4\uc740 \uc2b9\uaca9\uc774 \ud655\uc815\ub41c\uac78\ub85c \ubbff\uace0 \uc788\uc5c8\uc9c0\ub9cc \uac19\uc740 \uc2dc\uac01, \ud3ec\uce20\uba38\uc2a4\ub294 15\ubd84\uc774\ub098 \uacbd\uae30\uac00 \uc9c0\uc5f0\ub418\uc5b4 \uc544\uc9c1 \uacbd\uae30\uac00 \uc9c4\ud589 \uc911\uc774\uc5c8\ub2e4. \uacbd\uae30 \ub9c9\ud310\uc5d0 \ud130\uc9c4 \uace8\ub85c 5-1 \uc2b9\ub9ac\ub97c \uac70\ub454 \ud3ec\uce20\uba38\uc2a4\ub294 \uace8 \ud3c9\uade0\ucc28 (\ucd1d \uc2e4\uc810 \uc218\uc640 \ub4dd\uc810 \uc218\ub97c \ub098\ub204\ub294 \ubc29\ubc95\uc73c\ub85c\uc368 \ub2f9\uc2dc\uc5d0\ub294 \uc774 \ub8f0\uc774 \uc801\uc6a9\ub418\uc5c8\uc73c\uba70, \uace8 \ub4dd\uc2e4\ucc28\uac00 \uc801\uc6a9\uc774 \ub41c \uac83\uc740 1970\ub144 \uba55\uc2dc\ucf54 \uc6d4\ub4dc\ucef5, \uc789\uae00\ub79c\ub4dc\uc5d0\ub294 1976 - 77\uc2dc\uc98c\ubd80\ud130\uc600\ub2e4.)\ub85c \uc778\ud574 2\uc704\ub97c \ucc28\uc9c0\ud558\uc5ec \ud3ec\uce20\uba38\uc2a4\uac00 \uc2b9\uaca9\ud558\uac8c \ub418\uc5c8\ub2e4.", "sentence_answer": "\ub9e8\uccb4\uc2a4\ud130 \uc2dc\ud2f0\ub294 \ub9c8\uc9c0\ub9c9 \ub0a0 \ube0c\ub798\ub4dc\ud37c\ub4dc \uc2dc\ud2f0 \ub97c \ubb34\ub824 8-0\uc73c\ub85c \uaebe\uc5c8\uace0 \ub9e8\uccb4\uc2a4\ud130 \uc2dc\ud2f0 \ud32c\ub4e4\uc740 \uc2b9\uaca9\uc774 \ud655\uc815\ub41c\uac78\ub85c \ubbff\uace0 \uc788\uc5c8\uc9c0\ub9cc \uac19\uc740 \uc2dc\uac01, \ud3ec\uce20\uba38\uc2a4\ub294 15\ubd84\uc774\ub098 \uacbd\uae30\uac00 \uc9c0\uc5f0\ub418\uc5b4 \uc544\uc9c1 \uacbd\uae30\uac00 \uc9c4\ud589 \uc911\uc774\uc5c8\ub2e4.", "paragraph_id": "6502151-3-0", "paragraph_question": "question: 1926-27 \uc2dc\uc98c \ub9e8\uccb4\uc2a4\ud130 \uc2dc\ud2f0\uc758 \ub9ac\uadf8 \ub9c8\uc9c0\ub9c9 \uacbd\uae30 \uc0c1\ub300\ub294?, context: 1926\ub144, \ub9e8\uccb4\uc2a4\ud130 \uc2dc\ud2f0\ub294 FA\ucef5 \uacb0\uc2b9\uc5d0\uc11c \ub2e4\uc2dc \ubcfc\ud134 \uc6d0\ub354\ub7ec\uc2a4\uc640 \uc7ac\ud68c\ud558\uc9c0\ub9cc \uc774\ubc88\uc5d4 0-1\ub85c \ud328\ud558\uac8c \ub41c\ub2e4. \ud558\uc9c0\ub9cc \uc774\ud6c4\ubd80\ud134 \uae30\ubcf5\uc774 \uc2ec\ud55c \ud50c\ub808\uc774\ub85c \ub9ac\uadf8\uc5d0\uc11c \ubd80\uc9c4\uc774 \uc787\ub530\ub974\uac8c \ub418\ub294\ub370 \uc2dc\uc98c \ub9c8\uc9c0\ub9c9 \ub0a0, \uac15\ub4f1\uc774 \ub418\uace0 \ub9cc\ub2e4. 1926 - 27\uc2dc\uc98c\uc5d0\ub294 1\ubd80\ub9ac\uadf8 \uc2b9\uaca9\uc744 \ubaa9\uc804\uc5d0 \ub450\uace0 \uce58\uc5f4\ud55c \ub2e4\ud23c\uc774 \ubc8c\uc5b4\uc84c\uc5c8\ub2e4. \uc2b9\uaca9 \uacbd\uc7c1\uc740 \uc2dc\uc98c \ub9c8\uc9c0\ub9c9 \ub0a0\uae4c\uc9c0 \uc774\uc5b4\uc84c\ub294\ub370 \ub9e8\uccb4\uc2a4\ud130 \uc2dc\ud2f0\uc640 \ud3ec\uce20\uba38\uc2a4\uac00 2\uc704\ub97c \ub193\uace0 \ub2e4\ud23c\uc744 \ubc8c\uc774\uace0 \uc788\uc5c8\ub2e4. \ub9e8\uccb4\uc2a4\ud130 \uc2dc\ud2f0\ub294 \ub9c8\uc9c0\ub9c9 \ub0a0 \ube0c\ub798\ub4dc\ud37c\ub4dc \uc2dc\ud2f0\ub97c \ubb34\ub824 8-0\uc73c\ub85c \uaebe\uc5c8\uace0 \ub9e8\uccb4\uc2a4\ud130 \uc2dc\ud2f0 \ud32c\ub4e4\uc740 \uc2b9\uaca9\uc774 \ud655\uc815\ub41c\uac78\ub85c \ubbff\uace0 \uc788\uc5c8\uc9c0\ub9cc \uac19\uc740 \uc2dc\uac01, \ud3ec\uce20\uba38\uc2a4\ub294 15\ubd84\uc774\ub098 \uacbd\uae30\uac00 \uc9c0\uc5f0\ub418\uc5b4 \uc544\uc9c1 \uacbd\uae30\uac00 \uc9c4\ud589 \uc911\uc774\uc5c8\ub2e4. \uacbd\uae30 \ub9c9\ud310\uc5d0 \ud130\uc9c4 \uace8\ub85c 5-1 \uc2b9\ub9ac\ub97c \uac70\ub454 \ud3ec\uce20\uba38\uc2a4\ub294 \uace8 \ud3c9\uade0\ucc28 (\ucd1d \uc2e4\uc810 \uc218\uc640 \ub4dd\uc810 \uc218\ub97c \ub098\ub204\ub294 \ubc29\ubc95\uc73c\ub85c\uc368 \ub2f9\uc2dc\uc5d0\ub294 \uc774 \ub8f0\uc774 \uc801\uc6a9\ub418\uc5c8\uc73c\uba70, \uace8 \ub4dd\uc2e4\ucc28\uac00 \uc801\uc6a9\uc774 \ub41c \uac83\uc740 1970\ub144 \uba55\uc2dc\ucf54 \uc6d4\ub4dc\ucef5, \uc789\uae00\ub79c\ub4dc\uc5d0\ub294 1976 - 77\uc2dc\uc98c\ubd80\ud130\uc600\ub2e4.)\ub85c \uc778\ud574 2\uc704\ub97c \ucc28\uc9c0\ud558\uc5ec \ud3ec\uce20\uba38\uc2a4\uac00 \uc2b9\uaca9\ud558\uac8c \ub418\uc5c8\ub2e4."} {"question": "\uc8fc\uc778\uacfc \uc720\ub300\uac10\uc774 \uc88b\uc740 \ud3ec\uba54\ub77c\ub2c8\uc548\uc744 \ube68\ub9ac \ud63c\uc790\uc11c\ub3c4 \uc2dc\uac04\uc744 \ubcf4\ub0bc \uc218 \uc788\ub3c4\ub85d \ud6c8\ub828\uc744 \uc2dc\ud0a4\uc9c0 \uc54a\uc744 \uacbd\uc6b0 \ubc1c\uc0dd\ud558\ub294 \ubb38\uc81c\uc810\uc740?", "paragraph": "\ud3ec\uba54\ub77c\ub2c8\uc548\uc740 \uc77c\ubc18\uc801\uc73c\ub85c \ub9e4\uc6b0 \uce5c\uadfc\ud558\uace0 \uc6d0\uae30\uc655\uc131\ud55c \uac1c\uc774\ub2e4. \uc774 \uac1c\ub294 \uc8fc\uc778\uc758 \uc8fc\ubcc0\uc5d0 \ud568\uaed8 \uc788\ub294 \uac83\uc744 \uc88b\uc544\ud558\uace0, \ubcf4\ud638\ud574\uc8fc\ub294 \uac83\uc73c\ub85c \uc54c\ub824\uc838 \uc788\ub2e4. \uc774 \ud488\uc885\uc740 \uc8fc\uc778\uacfc \uc720\ub300\uac10\uc774 \ube68\ub9ac \ud615\uc131\ub3fc \ud63c\uc790 \uc2dc\uac04\uc744 \ubcf4\ub0bc \uc218 \uc788\ub3c4\ub85d \ud6c8\ub828 \uc2dc\ud0a4\uc9c0 \uc54a\ub294\ub2e4\uba74, \ubd84\ub9ac \ubd88\uc548\uc5d0 \uc2dc\ub2ec\ub9b4 \uc218 \uc788\ub2e4. \ud3ec\uba54\ub77c\ub2c8\uc548\uc740 \uacbd\uacc4\uc640 \ud658\uacbd\uc758 \ubcc0\ud654\ub97c \uc54c\uc544\ucc44\uace0 \uc0c8\ub85c\uc6b4 \uc790\uadf9\uc5d0 \uc9d6\ub294 \ud589\ub3d9\uc740 \uc5b4\ub5a4 \uc0c1\ud669\uc5d0\uc11c\ub3c4 \uacfc\ub3c4\ud558\uac8c \uc9d6\ub294 \uc2b5\uad00\uc73c\ub85c \uc774\uc5b4 \uc9c8 \uc218 \uc788\ub2e4. \ud3ec\uba54\ub77c\ub2c8\uc548\uc740 \ub611\ub611\ud55c \uac1c\ub85c \ud6c8\ub828 \ubc18\uc751\uc774 \uc88b\uc740\ub370, \uc8fc\uc778\uc5d0\uac8c \uc5b4\ub5bb\uac8c \ud6c8\ub828\uc744 \ubc1b\ub290\ub0d0\uc5d0 \ub530\ub77c \ub9e4\uc6b0 \uc131\uacf5\uc801\uc778 \uc131\uacfc\ub97c \uc5bb\uc744 \uc218 \uc788\ub2e4. \uadf8\ub7ec\ub098 \ud6c8\ub828\uc744 \uc2dc\ud0a4\uc9c0 \uc54a\ub294\ub2e4\uba74, \uc9c0\ubc30\uc801\uc774\uace0 \uacf5\uaca9\uc801\uc73c\ub85c \ub420 \uc218 \uc788\ub2e4. \ub610\ud55c \ud765\ubd84\uc744 \uc798\ud558\uba70 \uae09\ud558\uace0 \ud638\uae30\uc2ec\uc774 \ub9ce\uc544 \ud0c0 \uacac\uc885\uc5d0\uac8c \uad00\uc2ec\uc774 \ub9ce\ub2e4.", "answer": "\ubd84\ub9ac \ubd88\uc548", "sentence": "\uc774 \ud488\uc885\uc740 \uc8fc\uc778\uacfc \uc720\ub300\uac10\uc774 \ube68\ub9ac \ud615\uc131\ub3fc \ud63c\uc790 \uc2dc\uac04\uc744 \ubcf4\ub0bc \uc218 \uc788\ub3c4\ub85d \ud6c8\ub828 \uc2dc\ud0a4\uc9c0 \uc54a\ub294\ub2e4\uba74, \ubd84\ub9ac \ubd88\uc548 \uc5d0 \uc2dc\ub2ec\ub9b4 \uc218 \uc788\ub2e4.", "paragraph_sentence": "\ud3ec\uba54\ub77c\ub2c8\uc548\uc740 \uc77c\ubc18\uc801\uc73c\ub85c \ub9e4\uc6b0 \uce5c\uadfc\ud558\uace0 \uc6d0\uae30\uc655\uc131\ud55c \uac1c\uc774\ub2e4. \uc774 \uac1c\ub294 \uc8fc\uc778\uc758 \uc8fc\ubcc0\uc5d0 \ud568\uaed8 \uc788\ub294 \uac83\uc744 \uc88b\uc544\ud558\uace0, \ubcf4\ud638\ud574\uc8fc\ub294 \uac83\uc73c\ub85c \uc54c\ub824\uc838 \uc788\ub2e4. \uc774 \ud488\uc885\uc740 \uc8fc\uc778\uacfc \uc720\ub300\uac10\uc774 \ube68\ub9ac \ud615\uc131\ub3fc \ud63c\uc790 \uc2dc\uac04\uc744 \ubcf4\ub0bc \uc218 \uc788\ub3c4\ub85d \ud6c8\ub828 \uc2dc\ud0a4\uc9c0 \uc54a\ub294\ub2e4\uba74, \ubd84\ub9ac \ubd88\uc548 \uc5d0 \uc2dc\ub2ec\ub9b4 \uc218 \uc788\ub2e4. \ud3ec\uba54\ub77c\ub2c8\uc548\uc740 \uacbd\uacc4\uc640 \ud658\uacbd\uc758 \ubcc0\ud654\ub97c \uc54c\uc544\ucc44\uace0 \uc0c8\ub85c\uc6b4 \uc790\uadf9\uc5d0 \uc9d6\ub294 \ud589\ub3d9\uc740 \uc5b4\ub5a4 \uc0c1\ud669\uc5d0\uc11c\ub3c4 \uacfc\ub3c4\ud558\uac8c \uc9d6\ub294 \uc2b5\uad00\uc73c\ub85c \uc774\uc5b4 \uc9c8 \uc218 \uc788\ub2e4. \ud3ec\uba54\ub77c\ub2c8\uc548\uc740 \ub611\ub611\ud55c \uac1c\ub85c \ud6c8\ub828 \ubc18\uc751\uc774 \uc88b\uc740\ub370, \uc8fc\uc778\uc5d0\uac8c \uc5b4\ub5bb\uac8c \ud6c8\ub828\uc744 \ubc1b\ub290\ub0d0\uc5d0 \ub530\ub77c \ub9e4\uc6b0 \uc131\uacf5\uc801\uc778 \uc131\uacfc\ub97c \uc5bb\uc744 \uc218 \uc788\ub2e4. \uadf8\ub7ec\ub098 \ud6c8\ub828\uc744 \uc2dc\ud0a4\uc9c0 \uc54a\ub294\ub2e4\uba74, \uc9c0\ubc30\uc801\uc774\uace0 \uacf5\uaca9\uc801\uc73c\ub85c \ub420 \uc218 \uc788\ub2e4. \ub610\ud55c \ud765\ubd84\uc744 \uc798\ud558\uba70 \uae09\ud558\uace0 \ud638\uae30\uc2ec\uc774 \ub9ce\uc544 \ud0c0 \uacac\uc885\uc5d0\uac8c \uad00\uc2ec\uc774 \ub9ce\ub2e4.", "paragraph_answer": "\ud3ec\uba54\ub77c\ub2c8\uc548\uc740 \uc77c\ubc18\uc801\uc73c\ub85c \ub9e4\uc6b0 \uce5c\uadfc\ud558\uace0 \uc6d0\uae30\uc655\uc131\ud55c \uac1c\uc774\ub2e4. \uc774 \uac1c\ub294 \uc8fc\uc778\uc758 \uc8fc\ubcc0\uc5d0 \ud568\uaed8 \uc788\ub294 \uac83\uc744 \uc88b\uc544\ud558\uace0, \ubcf4\ud638\ud574\uc8fc\ub294 \uac83\uc73c\ub85c \uc54c\ub824\uc838 \uc788\ub2e4. \uc774 \ud488\uc885\uc740 \uc8fc\uc778\uacfc \uc720\ub300\uac10\uc774 \ube68\ub9ac \ud615\uc131\ub3fc \ud63c\uc790 \uc2dc\uac04\uc744 \ubcf4\ub0bc \uc218 \uc788\ub3c4\ub85d \ud6c8\ub828 \uc2dc\ud0a4\uc9c0 \uc54a\ub294\ub2e4\uba74, \ubd84\ub9ac \ubd88\uc548 \uc5d0 \uc2dc\ub2ec\ub9b4 \uc218 \uc788\ub2e4. \ud3ec\uba54\ub77c\ub2c8\uc548\uc740 \uacbd\uacc4\uc640 \ud658\uacbd\uc758 \ubcc0\ud654\ub97c \uc54c\uc544\ucc44\uace0 \uc0c8\ub85c\uc6b4 \uc790\uadf9\uc5d0 \uc9d6\ub294 \ud589\ub3d9\uc740 \uc5b4\ub5a4 \uc0c1\ud669\uc5d0\uc11c\ub3c4 \uacfc\ub3c4\ud558\uac8c \uc9d6\ub294 \uc2b5\uad00\uc73c\ub85c \uc774\uc5b4 \uc9c8 \uc218 \uc788\ub2e4. \ud3ec\uba54\ub77c\ub2c8\uc548\uc740 \ub611\ub611\ud55c \uac1c\ub85c \ud6c8\ub828 \ubc18\uc751\uc774 \uc88b\uc740\ub370, \uc8fc\uc778\uc5d0\uac8c \uc5b4\ub5bb\uac8c \ud6c8\ub828\uc744 \ubc1b\ub290\ub0d0\uc5d0 \ub530\ub77c \ub9e4\uc6b0 \uc131\uacf5\uc801\uc778 \uc131\uacfc\ub97c \uc5bb\uc744 \uc218 \uc788\ub2e4. \uadf8\ub7ec\ub098 \ud6c8\ub828\uc744 \uc2dc\ud0a4\uc9c0 \uc54a\ub294\ub2e4\uba74, \uc9c0\ubc30\uc801\uc774\uace0 \uacf5\uaca9\uc801\uc73c\ub85c \ub420 \uc218 \uc788\ub2e4. \ub610\ud55c \ud765\ubd84\uc744 \uc798\ud558\uba70 \uae09\ud558\uace0 \ud638\uae30\uc2ec\uc774 \ub9ce\uc544 \ud0c0 \uacac\uc885\uc5d0\uac8c \uad00\uc2ec\uc774 \ub9ce\ub2e4.", "sentence_answer": "\uc774 \ud488\uc885\uc740 \uc8fc\uc778\uacfc \uc720\ub300\uac10\uc774 \ube68\ub9ac \ud615\uc131\ub3fc \ud63c\uc790 \uc2dc\uac04\uc744 \ubcf4\ub0bc \uc218 \uc788\ub3c4\ub85d \ud6c8\ub828 \uc2dc\ud0a4\uc9c0 \uc54a\ub294\ub2e4\uba74, \ubd84\ub9ac \ubd88\uc548 \uc5d0 \uc2dc\ub2ec\ub9b4 \uc218 \uc788\ub2e4.", "paragraph_id": "6567291-0-1", "paragraph_question": "question: \uc8fc\uc778\uacfc \uc720\ub300\uac10\uc774 \uc88b\uc740 \ud3ec\uba54\ub77c\ub2c8\uc548\uc744 \ube68\ub9ac \ud63c\uc790\uc11c\ub3c4 \uc2dc\uac04\uc744 \ubcf4\ub0bc \uc218 \uc788\ub3c4\ub85d \ud6c8\ub828\uc744 \uc2dc\ud0a4\uc9c0 \uc54a\uc744 \uacbd\uc6b0 \ubc1c\uc0dd\ud558\ub294 \ubb38\uc81c\uc810\uc740?, context: \ud3ec\uba54\ub77c\ub2c8\uc548\uc740 \uc77c\ubc18\uc801\uc73c\ub85c \ub9e4\uc6b0 \uce5c\uadfc\ud558\uace0 \uc6d0\uae30\uc655\uc131\ud55c \uac1c\uc774\ub2e4. \uc774 \uac1c\ub294 \uc8fc\uc778\uc758 \uc8fc\ubcc0\uc5d0 \ud568\uaed8 \uc788\ub294 \uac83\uc744 \uc88b\uc544\ud558\uace0, \ubcf4\ud638\ud574\uc8fc\ub294 \uac83\uc73c\ub85c \uc54c\ub824\uc838 \uc788\ub2e4. \uc774 \ud488\uc885\uc740 \uc8fc\uc778\uacfc \uc720\ub300\uac10\uc774 \ube68\ub9ac \ud615\uc131\ub3fc \ud63c\uc790 \uc2dc\uac04\uc744 \ubcf4\ub0bc \uc218 \uc788\ub3c4\ub85d \ud6c8\ub828 \uc2dc\ud0a4\uc9c0 \uc54a\ub294\ub2e4\uba74, \ubd84\ub9ac \ubd88\uc548\uc5d0 \uc2dc\ub2ec\ub9b4 \uc218 \uc788\ub2e4. \ud3ec\uba54\ub77c\ub2c8\uc548\uc740 \uacbd\uacc4\uc640 \ud658\uacbd\uc758 \ubcc0\ud654\ub97c \uc54c\uc544\ucc44\uace0 \uc0c8\ub85c\uc6b4 \uc790\uadf9\uc5d0 \uc9d6\ub294 \ud589\ub3d9\uc740 \uc5b4\ub5a4 \uc0c1\ud669\uc5d0\uc11c\ub3c4 \uacfc\ub3c4\ud558\uac8c \uc9d6\ub294 \uc2b5\uad00\uc73c\ub85c \uc774\uc5b4 \uc9c8 \uc218 \uc788\ub2e4. \ud3ec\uba54\ub77c\ub2c8\uc548\uc740 \ub611\ub611\ud55c \uac1c\ub85c \ud6c8\ub828 \ubc18\uc751\uc774 \uc88b\uc740\ub370, \uc8fc\uc778\uc5d0\uac8c \uc5b4\ub5bb\uac8c \ud6c8\ub828\uc744 \ubc1b\ub290\ub0d0\uc5d0 \ub530\ub77c \ub9e4\uc6b0 \uc131\uacf5\uc801\uc778 \uc131\uacfc\ub97c \uc5bb\uc744 \uc218 \uc788\ub2e4. \uadf8\ub7ec\ub098 \ud6c8\ub828\uc744 \uc2dc\ud0a4\uc9c0 \uc54a\ub294\ub2e4\uba74, \uc9c0\ubc30\uc801\uc774\uace0 \uacf5\uaca9\uc801\uc73c\ub85c \ub420 \uc218 \uc788\ub2e4. \ub610\ud55c \ud765\ubd84\uc744 \uc798\ud558\uba70 \uae09\ud558\uace0 \ud638\uae30\uc2ec\uc774 \ub9ce\uc544 \ud0c0 \uacac\uc885\uc5d0\uac8c \uad00\uc2ec\uc774 \ub9ce\ub2e4."} {"question": "2013\ub144 4\uc6d4 26\uc77c \ub2f9\uc2dc \ud1b5\uc77c\ubd80 \uc7a5\uad00\uc740?", "paragraph": "2013\ub144 2\uc6d4 12\uc77c\uc5d0 \ud568\uacbd\ubd81\ub3c4 \uae38\uc8fc\uad70\uc5d0\uc11c \ub9ac\ud788\ud130 \uaddc\ubaa8 4.9 (\ubbf8\uad6d \uc9c0\uc9c8\uc870\uc0ac\uad6d\uc740 \ub9ac\ud788\ud130 \uaddc\ubaa8 5.1)\uc758 \ud575\uc2e4\ud5d8\uc744 \uac15\ud589\ud558\uc5ec \ub9ce\uc740 \ub098\ub77c\uc758 \uaddc\ud0c4\uc744 \ubc1b\uc558\ub2e4. 3\uc6d4\uc5d0\ub294 \ub0a8\ubd81\ubd88\uac00\uce68\ud569\uc758\ub97c \ud3d0\uae30\ud558\uaca0\ub2e4\uace0 \ubc1c\ud45c\ud588\uc73c\uba70, \ub610\ud55c \uac01\uc885 \ubbf8\uc0ac\uc77c\uc774 \ud575\ud0c4\ub450\ub97c \uc7a5\ucc29\ud55c \ucc44 \ub300\uae30\uc0c1\ud0dc\uc5d0 \uc788\ub2e4\uace0 \uc704\ud611\ud588\uc73c\uba70, 1991\ub144 \ub0a8\ubd81 \ubd88\uac00\uce68 \ud569\uc758 \ud3d0\uae30\uc640 \ud310\ubb38\uc810 \ub0a8\ubd81 \uc9c1\ud1b5\uc804\ud654 \ub2e8\uc808\uc744 \uc120\uc5b8\ud588\ub2e4. \uc774\uc5d0 \ub300\ud55c\ubbfc\uad6d \uad6d\ubc29\ubd80 \uae40\ubbfc\uc11d \ub300\ubcc0\uc778\uc740 \"\ubd81\ud55c\uc774 \ud575\ubb34\uae30\ub97c \uac00\uc9c0\uace0 \uc6b0\ub9ac \ud55c\uad6d\uc744 \uacf5\uaca9\ud55c\ub2e4\uba74 \uc778\ub958\uc758 \uc758\uc9c0\uff0c\ub300\ud55c\ubbfc\uad6d\uc740 \ub2f9\uc5f0\ud558\uace0 \uc778\ub958\uc758 \uc758\uc9c0\ub85c \uae40\uc815\uc740 \uc815\uad8c\uc740 \uc9c0\uad6c\uc0c1\uc5d0\uc11c \uc18c\uba78\ub420 \uac81\ub2c8\ub2e4.\"\ub77c\uace0 \uc8fc\uc7a5\ud558\uc600\ub2e4. \uad6d\ubc29\ubd80 \ub610\ud55c \uc870\uc120\uc778\ubbfc\uad70\uc774 \ub300\uaddc\ubaa8 \ud6c8\ub828\uc744 \uc704\ud574 \uc9d1\uacb0\ud55c \uc0c1\ud0dc\ub77c\uba70, \ub9cc\uc57d\uc5d0 \uacf5\uaca9\ud560 \uacbd\uc6b0 \ub300\uc751 \uaddc\ubaa8\uc640\ub294 \uc0c1\uad00 \uc5c6\uc774 \uc751\uc9d5\ud558\uaca0\ub2e4\uace0 \uac15\uc870\ud588\ub2e4. \uadf8 \ud6c4 4\uc6d4 26\uc77c \ub300\ud55c\ubbfc\uad6d\uc758 \ub958\uae38\uc7ac \ud1b5\uc77c\ubd80 \uc7a5\uad00\uc774 \uac1c\uc131\uacf5\ub2e8 \ub0b4\uc758 \uc794\ub958\uc778\uc6d0\uc5d0 \ub300\ud55c \ucca0\uc218\ub97c \uacb0\uc815\ud588\uc73c\uba70, \uc774\uc5d0 \ub530\ub77c 27\uc77c \uac1c\uc131\uacf5\ub2e8\uc5d0 \uccb4\ub958\ud558\uace0 \uc788\ub358 126\uba85\uc774 \ucca0\uc218\ud588\uace0, 29\uc77c\uc5d0 \ub098\uba38\uc9c0\ub3c4 \ucca0\uc218\ud558\uae30\ub85c \uacb0\uc815\ud588\ub2e4. \uadf8\ub7ec\ub098 \ub0a8\uc740 \uc794\ub958\uc778\uc6d0 50\uba85 \uc911 43\uba85\ub9cc \uadc0\ud658 \ud5c8\uac00\ub97c \ubc1b\uc558\ub2e4(\ubb3c\ub860 \ub098\uba38\uc9c0 7\uba85\ub3c4 \ud6d7\ub0a0 \uadc0\ud658\ud558\uc600\ub2e4).", "answer": "\ub958\uae38\uc7ac", "sentence": "\uadf8 \ud6c4 4\uc6d4 26\uc77c \ub300\ud55c\ubbfc\uad6d\uc758 \ub958\uae38\uc7ac \ud1b5\uc77c\ubd80 \uc7a5\uad00\uc774 \uac1c\uc131\uacf5\ub2e8 \ub0b4\uc758 \uc794\ub958\uc778\uc6d0\uc5d0 \ub300\ud55c \ucca0\uc218\ub97c \uacb0\uc815\ud588\uc73c\uba70, \uc774\uc5d0 \ub530\ub77c 27\uc77c \uac1c\uc131\uacf5\ub2e8\uc5d0 \uccb4\ub958\ud558\uace0 \uc788\ub358 126\uba85\uc774 \ucca0\uc218\ud588\uace0, 29\uc77c\uc5d0 \ub098\uba38\uc9c0\ub3c4 \ucca0\uc218\ud558\uae30\ub85c \uacb0\uc815\ud588\ub2e4.", "paragraph_sentence": "2013\ub144 2\uc6d4 12\uc77c\uc5d0 \ud568\uacbd\ubd81\ub3c4 \uae38\uc8fc\uad70\uc5d0\uc11c \ub9ac\ud788\ud130 \uaddc\ubaa8 4.9 (\ubbf8\uad6d \uc9c0\uc9c8\uc870\uc0ac\uad6d\uc740 \ub9ac\ud788\ud130 \uaddc\ubaa8 5.1)\uc758 \ud575\uc2e4\ud5d8\uc744 \uac15\ud589\ud558\uc5ec \ub9ce\uc740 \ub098\ub77c\uc758 \uaddc\ud0c4\uc744 \ubc1b\uc558\ub2e4. 3\uc6d4\uc5d0\ub294 \ub0a8\ubd81\ubd88\uac00\uce68\ud569\uc758\ub97c \ud3d0\uae30\ud558\uaca0\ub2e4\uace0 \ubc1c\ud45c\ud588\uc73c\uba70, \ub610\ud55c \uac01\uc885 \ubbf8\uc0ac\uc77c\uc774 \ud575\ud0c4\ub450\ub97c \uc7a5\ucc29\ud55c \ucc44 \ub300\uae30\uc0c1\ud0dc\uc5d0 \uc788\ub2e4\uace0 \uc704\ud611\ud588\uc73c\uba70, 1991\ub144 \ub0a8\ubd81 \ubd88\uac00\uce68 \ud569\uc758 \ud3d0\uae30\uc640 \ud310\ubb38\uc810 \ub0a8\ubd81 \uc9c1\ud1b5\uc804\ud654 \ub2e8\uc808\uc744 \uc120\uc5b8\ud588\ub2e4. \uc774\uc5d0 \ub300\ud55c\ubbfc\uad6d \uad6d\ubc29\ubd80 \uae40\ubbfc\uc11d \ub300\ubcc0\uc778\uc740 \"\ubd81\ud55c\uc774 \ud575\ubb34\uae30\ub97c \uac00\uc9c0\uace0 \uc6b0\ub9ac \ud55c\uad6d\uc744 \uacf5\uaca9\ud55c\ub2e4\uba74 \uc778\ub958\uc758 \uc758\uc9c0\uff0c\ub300\ud55c\ubbfc\uad6d\uc740 \ub2f9\uc5f0\ud558\uace0 \uc778\ub958\uc758 \uc758\uc9c0\ub85c \uae40\uc815\uc740 \uc815\uad8c\uc740 \uc9c0\uad6c\uc0c1\uc5d0\uc11c \uc18c\uba78\ub420 \uac81\ub2c8\ub2e4. \"\ub77c\uace0 \uc8fc\uc7a5\ud558\uc600\ub2e4. \uad6d\ubc29\ubd80 \ub610\ud55c \uc870\uc120\uc778\ubbfc\uad70\uc774 \ub300\uaddc\ubaa8 \ud6c8\ub828\uc744 \uc704\ud574 \uc9d1\uacb0\ud55c \uc0c1\ud0dc\ub77c\uba70, \ub9cc\uc57d\uc5d0 \uacf5\uaca9\ud560 \uacbd\uc6b0 \ub300\uc751 \uaddc\ubaa8\uc640\ub294 \uc0c1\uad00 \uc5c6\uc774 \uc751\uc9d5\ud558\uaca0\ub2e4\uace0 \uac15\uc870\ud588\ub2e4. \uadf8 \ud6c4 4\uc6d4 26\uc77c \ub300\ud55c\ubbfc\uad6d\uc758 \ub958\uae38\uc7ac \ud1b5\uc77c\ubd80 \uc7a5\uad00\uc774 \uac1c\uc131\uacf5\ub2e8 \ub0b4\uc758 \uc794\ub958\uc778\uc6d0\uc5d0 \ub300\ud55c \ucca0\uc218\ub97c \uacb0\uc815\ud588\uc73c\uba70, \uc774\uc5d0 \ub530\ub77c 27\uc77c \uac1c\uc131\uacf5\ub2e8\uc5d0 \uccb4\ub958\ud558\uace0 \uc788\ub358 126\uba85\uc774 \ucca0\uc218\ud588\uace0, 29\uc77c\uc5d0 \ub098\uba38\uc9c0\ub3c4 \ucca0\uc218\ud558\uae30\ub85c \uacb0\uc815\ud588\ub2e4. \uadf8\ub7ec\ub098 \ub0a8\uc740 \uc794\ub958\uc778\uc6d0 50\uba85 \uc911 43\uba85\ub9cc \uadc0\ud658 \ud5c8\uac00\ub97c \ubc1b\uc558\ub2e4(\ubb3c\ub860 \ub098\uba38\uc9c0 7\uba85\ub3c4 \ud6d7\ub0a0 \uadc0\ud658\ud558\uc600\ub2e4).", "paragraph_answer": "2013\ub144 2\uc6d4 12\uc77c\uc5d0 \ud568\uacbd\ubd81\ub3c4 \uae38\uc8fc\uad70\uc5d0\uc11c \ub9ac\ud788\ud130 \uaddc\ubaa8 4.9 (\ubbf8\uad6d \uc9c0\uc9c8\uc870\uc0ac\uad6d\uc740 \ub9ac\ud788\ud130 \uaddc\ubaa8 5.1)\uc758 \ud575\uc2e4\ud5d8\uc744 \uac15\ud589\ud558\uc5ec \ub9ce\uc740 \ub098\ub77c\uc758 \uaddc\ud0c4\uc744 \ubc1b\uc558\ub2e4. 3\uc6d4\uc5d0\ub294 \ub0a8\ubd81\ubd88\uac00\uce68\ud569\uc758\ub97c \ud3d0\uae30\ud558\uaca0\ub2e4\uace0 \ubc1c\ud45c\ud588\uc73c\uba70, \ub610\ud55c \uac01\uc885 \ubbf8\uc0ac\uc77c\uc774 \ud575\ud0c4\ub450\ub97c \uc7a5\ucc29\ud55c \ucc44 \ub300\uae30\uc0c1\ud0dc\uc5d0 \uc788\ub2e4\uace0 \uc704\ud611\ud588\uc73c\uba70, 1991\ub144 \ub0a8\ubd81 \ubd88\uac00\uce68 \ud569\uc758 \ud3d0\uae30\uc640 \ud310\ubb38\uc810 \ub0a8\ubd81 \uc9c1\ud1b5\uc804\ud654 \ub2e8\uc808\uc744 \uc120\uc5b8\ud588\ub2e4. \uc774\uc5d0 \ub300\ud55c\ubbfc\uad6d \uad6d\ubc29\ubd80 \uae40\ubbfc\uc11d \ub300\ubcc0\uc778\uc740 \"\ubd81\ud55c\uc774 \ud575\ubb34\uae30\ub97c \uac00\uc9c0\uace0 \uc6b0\ub9ac \ud55c\uad6d\uc744 \uacf5\uaca9\ud55c\ub2e4\uba74 \uc778\ub958\uc758 \uc758\uc9c0\uff0c\ub300\ud55c\ubbfc\uad6d\uc740 \ub2f9\uc5f0\ud558\uace0 \uc778\ub958\uc758 \uc758\uc9c0\ub85c \uae40\uc815\uc740 \uc815\uad8c\uc740 \uc9c0\uad6c\uc0c1\uc5d0\uc11c \uc18c\uba78\ub420 \uac81\ub2c8\ub2e4.\"\ub77c\uace0 \uc8fc\uc7a5\ud558\uc600\ub2e4. \uad6d\ubc29\ubd80 \ub610\ud55c \uc870\uc120\uc778\ubbfc\uad70\uc774 \ub300\uaddc\ubaa8 \ud6c8\ub828\uc744 \uc704\ud574 \uc9d1\uacb0\ud55c \uc0c1\ud0dc\ub77c\uba70, \ub9cc\uc57d\uc5d0 \uacf5\uaca9\ud560 \uacbd\uc6b0 \ub300\uc751 \uaddc\ubaa8\uc640\ub294 \uc0c1\uad00 \uc5c6\uc774 \uc751\uc9d5\ud558\uaca0\ub2e4\uace0 \uac15\uc870\ud588\ub2e4. \uadf8 \ud6c4 4\uc6d4 26\uc77c \ub300\ud55c\ubbfc\uad6d\uc758 \ub958\uae38\uc7ac \ud1b5\uc77c\ubd80 \uc7a5\uad00\uc774 \uac1c\uc131\uacf5\ub2e8 \ub0b4\uc758 \uc794\ub958\uc778\uc6d0\uc5d0 \ub300\ud55c \ucca0\uc218\ub97c \uacb0\uc815\ud588\uc73c\uba70, \uc774\uc5d0 \ub530\ub77c 27\uc77c \uac1c\uc131\uacf5\ub2e8\uc5d0 \uccb4\ub958\ud558\uace0 \uc788\ub358 126\uba85\uc774 \ucca0\uc218\ud588\uace0, 29\uc77c\uc5d0 \ub098\uba38\uc9c0\ub3c4 \ucca0\uc218\ud558\uae30\ub85c \uacb0\uc815\ud588\ub2e4. \uadf8\ub7ec\ub098 \ub0a8\uc740 \uc794\ub958\uc778\uc6d0 50\uba85 \uc911 43\uba85\ub9cc \uadc0\ud658 \ud5c8\uac00\ub97c \ubc1b\uc558\ub2e4(\ubb3c\ub860 \ub098\uba38\uc9c0 7\uba85\ub3c4 \ud6d7\ub0a0 \uadc0\ud658\ud558\uc600\ub2e4).", "sentence_answer": "\uadf8 \ud6c4 4\uc6d4 26\uc77c \ub300\ud55c\ubbfc\uad6d\uc758 \ub958\uae38\uc7ac \ud1b5\uc77c\ubd80 \uc7a5\uad00\uc774 \uac1c\uc131\uacf5\ub2e8 \ub0b4\uc758 \uc794\ub958\uc778\uc6d0\uc5d0 \ub300\ud55c \ucca0\uc218\ub97c \uacb0\uc815\ud588\uc73c\uba70, \uc774\uc5d0 \ub530\ub77c 27\uc77c \uac1c\uc131\uacf5\ub2e8\uc5d0 \uccb4\ub958\ud558\uace0 \uc788\ub358 126\uba85\uc774 \ucca0\uc218\ud588\uace0, 29\uc77c\uc5d0 \ub098\uba38\uc9c0\ub3c4 \ucca0\uc218\ud558\uae30\ub85c \uacb0\uc815\ud588\ub2e4.", "paragraph_id": "6482362-2-2", "paragraph_question": "question: 2013\ub144 4\uc6d4 26\uc77c \ub2f9\uc2dc \ud1b5\uc77c\ubd80 \uc7a5\uad00\uc740?, context: 2013\ub144 2\uc6d4 12\uc77c\uc5d0 \ud568\uacbd\ubd81\ub3c4 \uae38\uc8fc\uad70\uc5d0\uc11c \ub9ac\ud788\ud130 \uaddc\ubaa8 4.9 (\ubbf8\uad6d \uc9c0\uc9c8\uc870\uc0ac\uad6d\uc740 \ub9ac\ud788\ud130 \uaddc\ubaa8 5.1)\uc758 \ud575\uc2e4\ud5d8\uc744 \uac15\ud589\ud558\uc5ec \ub9ce\uc740 \ub098\ub77c\uc758 \uaddc\ud0c4\uc744 \ubc1b\uc558\ub2e4. 3\uc6d4\uc5d0\ub294 \ub0a8\ubd81\ubd88\uac00\uce68\ud569\uc758\ub97c \ud3d0\uae30\ud558\uaca0\ub2e4\uace0 \ubc1c\ud45c\ud588\uc73c\uba70, \ub610\ud55c \uac01\uc885 \ubbf8\uc0ac\uc77c\uc774 \ud575\ud0c4\ub450\ub97c \uc7a5\ucc29\ud55c \ucc44 \ub300\uae30\uc0c1\ud0dc\uc5d0 \uc788\ub2e4\uace0 \uc704\ud611\ud588\uc73c\uba70, 1991\ub144 \ub0a8\ubd81 \ubd88\uac00\uce68 \ud569\uc758 \ud3d0\uae30\uc640 \ud310\ubb38\uc810 \ub0a8\ubd81 \uc9c1\ud1b5\uc804\ud654 \ub2e8\uc808\uc744 \uc120\uc5b8\ud588\ub2e4. \uc774\uc5d0 \ub300\ud55c\ubbfc\uad6d \uad6d\ubc29\ubd80 \uae40\ubbfc\uc11d \ub300\ubcc0\uc778\uc740 \"\ubd81\ud55c\uc774 \ud575\ubb34\uae30\ub97c \uac00\uc9c0\uace0 \uc6b0\ub9ac \ud55c\uad6d\uc744 \uacf5\uaca9\ud55c\ub2e4\uba74 \uc778\ub958\uc758 \uc758\uc9c0\uff0c\ub300\ud55c\ubbfc\uad6d\uc740 \ub2f9\uc5f0\ud558\uace0 \uc778\ub958\uc758 \uc758\uc9c0\ub85c \uae40\uc815\uc740 \uc815\uad8c\uc740 \uc9c0\uad6c\uc0c1\uc5d0\uc11c \uc18c\uba78\ub420 \uac81\ub2c8\ub2e4.\"\ub77c\uace0 \uc8fc\uc7a5\ud558\uc600\ub2e4. \uad6d\ubc29\ubd80 \ub610\ud55c \uc870\uc120\uc778\ubbfc\uad70\uc774 \ub300\uaddc\ubaa8 \ud6c8\ub828\uc744 \uc704\ud574 \uc9d1\uacb0\ud55c \uc0c1\ud0dc\ub77c\uba70, \ub9cc\uc57d\uc5d0 \uacf5\uaca9\ud560 \uacbd\uc6b0 \ub300\uc751 \uaddc\ubaa8\uc640\ub294 \uc0c1\uad00 \uc5c6\uc774 \uc751\uc9d5\ud558\uaca0\ub2e4\uace0 \uac15\uc870\ud588\ub2e4. \uadf8 \ud6c4 4\uc6d4 26\uc77c \ub300\ud55c\ubbfc\uad6d\uc758 \ub958\uae38\uc7ac \ud1b5\uc77c\ubd80 \uc7a5\uad00\uc774 \uac1c\uc131\uacf5\ub2e8 \ub0b4\uc758 \uc794\ub958\uc778\uc6d0\uc5d0 \ub300\ud55c \ucca0\uc218\ub97c \uacb0\uc815\ud588\uc73c\uba70, \uc774\uc5d0 \ub530\ub77c 27\uc77c \uac1c\uc131\uacf5\ub2e8\uc5d0 \uccb4\ub958\ud558\uace0 \uc788\ub358 126\uba85\uc774 \ucca0\uc218\ud588\uace0, 29\uc77c\uc5d0 \ub098\uba38\uc9c0\ub3c4 \ucca0\uc218\ud558\uae30\ub85c \uacb0\uc815\ud588\ub2e4. \uadf8\ub7ec\ub098 \ub0a8\uc740 \uc794\ub958\uc778\uc6d0 50\uba85 \uc911 43\uba85\ub9cc \uadc0\ud658 \ud5c8\uac00\ub97c \ubc1b\uc558\ub2e4(\ubb3c\ub860 \ub098\uba38\uc9c0 7\uba85\ub3c4 \ud6d7\ub0a0 \uadc0\ud658\ud558\uc600\ub2e4)."} {"question": "\uc624\uc2a4\ub9cc \uc81c\uad6d\uc758 \ubd09\uc2e0\uc774 \ub418\uc5b4 \uc601\ud1a0\uc778 \ube44\ub518\uc744 \ubcf4\uc804\ud55c \uc778\ubb3c\uc740?", "paragraph": "1393\ub144 \ubd88\uac00\ub9ac\uc544\uc758 \uad70\uc8fc \uc774\ubc18 \uc2dc\uc2dc\ub9cc\uc740 \uc784\uc2dc \uc218\ub3c4\uc600\ub358 \ub2c8\ucf54\ud3f4\ub9ac\uc2a4\ub97c \uc624\uc2a4\ub9cc \uc81c\uad6d\uc5d0\uac8c \uacf5\uaca9\ub2f9\ud574 \ube7c\uc557\uacbc\ub2e4. \ud55c\ud3b8 \uadf8\uc758 \ub3d9\uc0dd\uc774\uc5c8\ub358 \uc774\ubc18 \uc2a4\ub77c\uce58\ubbf8\ub974\ub294 \uc624\uc2a4\ub9cc \uc81c\uad6d\uc758 \ubd09\uc2e0\uc774 \ub418\ub294 \ub300\uac00\ub85c \uadf8\uc758 \uc601\ud1a0 \ube44\ub518\uc744 \uc720\uc9c0\ud560 \uc218 \uc788\uc5c8\ub2e4. \ubd88\uac00\ub9ac\uc544 \uadc0\uc871\ub4e4\uacfc \ub2e4\ub978 \ubc1c\uce78\uc758 \ub3c5\ub9bd\uc801\uc778 \uc9c0\ubc30\uc790\ub4e4\uacfc \uad70\uc8fc\ub4e4\uc758 \uc2dc\uac01\uc73c\ub85c \ubcfc \ub54c, \uc774\ub294 \uc624\uc2a4\ub9cc\uc758 \uc815\ubcf5\uacfc\uc815\uc73c\ub85c\ubd80\ud130 \uad6c\uc6d0\ubc1b\uc744 \uc218 \uc788\uace0, \uc774\uc2ac\ub78c\uc758 \uc9c0\ubc30\ub85c\ubd80\ud130 \ubc1c\uce78\uc744 \uc790\uc720\ub86d\uac8c \ud560 \uc218 \uc788\ub294 \uae30\ud68c\uc600\ub2e4. \ub354\ud558\uc5ec, \uc774\uc2ac\ub78c\uacfc \uae30\ub3c5\uad50\uc758 \ub300\uce58\uc120\uc740 \uc11c\uc11c\ud788 \ud5dd\uac00\ub9ac \uc655\uad6d\uc744 \ud5a5\ud558\uc5ec \uc6c0\uc9c1\uc774\uace0 \uc788\uc5c8\ub2e4. \ud5dd\uac00\ub9ac \uc655\uad6d\uc740 \uc774\uc81c \ub3d9\ubd80 \uc720\ub7fd\uc5d0\uc11c \ub450 \uc885\uad50\uc758 \uc0c8\ub85c\uc6b4 \ub300\uce58\uc120\uc774 \ub418\uace0 \uc788\uc5c8\uace0, \ud5dd\uac00\ub9ac\uc778\ub4e4\uc740 \uadf8\ub4e4 \uc790\uc2e0\uc774 \uacf5\uaca9\ubc1b\uc744 \uc704\ud611\uc5d0 \ucc98\ud574 \uc788\uc5c8\ub2e4. \ubca0\ub124\uce58\uc544 \uacf5\ud654\uad6d\uc740 \ubaa8\ub808\uc544\uc758 \uc77c\ubd80 \ubc0f \ub2ec\ub9c8\ud2f0\uc544\ub77c \ubd88\ub9b0 \ubca0\ub124\uce58\uc544\uc758 \uc601\ud1a0\ub97c \ud3ec\ud568\ud55c \ubc1c\uce78\ubc18\ub3c4\uac00 \uc624\uc2a4\ub9cc \uc81c\uad6d\uc5d0\uac8c \uc9c0\ubc30\ub2f9\ud558\uba74 \uc544\ub4dc\ub9ac\uc544 \ud574, \uc774\uc624\ub2c8\uc544 \ud574, \uc5d0\uac8c \ud574\uc5d0\uc11c \ubca0\ub124\uce58\uc544\uc758 \uc601\ud5a5\ub825\uc774 \uac10\uc18c\ud558\ub294 \uac83\uc744 \ub450\ub824\uc6cc\ud588\ub2e4. \ud55c\ud3b8 \uc81c\ub178\ubc14 \uacf5\ud654\uad6d\uc740 \ubc18\uba74\uc5d0 \ub9cc\uc57d \ub3c4\ub098\uc6b0\uac15\uacfc \ubcf4\uc2a4\ud3ec\ub8e8\uc2a4-\ub2e4\ub974\ub2e4\ub12c\uc2a4 \uc591\ucabd \ud574\ud611\uc5d0 \ub300\ud55c \ud1b5\uc81c\uad8c\uc744 \uc624\uc2a4\ub9cc \uc81c\uad6d\uc774 \ud68d\ub4dd\ud55c\ub2e4\uba74, \uce74\ud30c, \uc2dc\ub178\ud398 \ub4f1\uc758 \uc81c\ub178\ubc14\uac00 \uc911\uc694\ud558\uac8c \uc5ec\uae30\ub294 \uc2dd\ubbfc\uc9c0\ub97c \ub2e4\uc218 \ubcf4\uc720\ud558\uace0 \uc788\ub294 \ud751\ud574\uc640 \uc720\ub7fd\uc758 \uad50\uc5ed\ub85c\ub97c \ucd5c\uc885\uc801\uc73c\ub85c \uc624\uc2a4\ub9cc \uc81c\uad6d\uc774 \ub3c5\uc810\ud558\ub294 \uac83\uc744 \uc704\ud5d8\ud558\ub2e4\uace0 \ubcf4\uace0 \uc788\uc5c8\ub2e4. \uc81c\ub178\ubc14\ub294 \ub610\ud55c 1395\ub144 \ubc14\uc608\uc9c0\ub4dc 1\uc138\uac00 \ucf58\uc2a4\ud0c4\ud2f0\ub178\ud3f4\ub9ac\uc2a4\ub97c \ud3ec\uc704\ud560 \ub2f9\uc2dc \uae08\uac01\ub9cc\uc758 \ubd81\ucabd\uc5d0 \uc788\ub358 \uac08\ub77c\ud0c0\uc758 \uc694\uc0c8\ub97c \uac00\uc9c0\uace0 \uc788\uc5c8\ub294\ub370, \uc774\uacf3\uc744 \ubc14\uc57c\uc9c0\ub4dc 1\uc138\uc5d0\uac8c \uae30\uc2b5\uacf5\uaca9\uc744 \ub2f9\ud588\ub2e4.", "answer": "\uc774\ubc18 \uc2a4\ub77c\uce58\ubbf8\ub974", "sentence": "\ud55c\ud3b8 \uadf8\uc758 \ub3d9\uc0dd\uc774\uc5c8\ub358 \uc774\ubc18 \uc2a4\ub77c\uce58\ubbf8\ub974 \ub294 \uc624\uc2a4\ub9cc \uc81c\uad6d\uc758 \ubd09\uc2e0\uc774 \ub418\ub294 \ub300\uac00\ub85c \uadf8\uc758 \uc601\ud1a0 \ube44\ub518\uc744 \uc720\uc9c0\ud560 \uc218 \uc788\uc5c8\ub2e4.", "paragraph_sentence": "1393\ub144 \ubd88\uac00\ub9ac\uc544\uc758 \uad70\uc8fc \uc774\ubc18 \uc2dc\uc2dc\ub9cc\uc740 \uc784\uc2dc \uc218\ub3c4\uc600\ub358 \ub2c8\ucf54\ud3f4\ub9ac\uc2a4\ub97c \uc624\uc2a4\ub9cc \uc81c\uad6d\uc5d0\uac8c \uacf5\uaca9\ub2f9\ud574 \ube7c\uc557\uacbc\ub2e4. \ud55c\ud3b8 \uadf8\uc758 \ub3d9\uc0dd\uc774\uc5c8\ub358 \uc774\ubc18 \uc2a4\ub77c\uce58\ubbf8\ub974 \ub294 \uc624\uc2a4\ub9cc \uc81c\uad6d\uc758 \ubd09\uc2e0\uc774 \ub418\ub294 \ub300\uac00\ub85c \uadf8\uc758 \uc601\ud1a0 \ube44\ub518\uc744 \uc720\uc9c0\ud560 \uc218 \uc788\uc5c8\ub2e4. \ubd88\uac00\ub9ac\uc544 \uadc0\uc871\ub4e4\uacfc \ub2e4\ub978 \ubc1c\uce78\uc758 \ub3c5\ub9bd\uc801\uc778 \uc9c0\ubc30\uc790\ub4e4\uacfc \uad70\uc8fc\ub4e4\uc758 \uc2dc\uac01\uc73c\ub85c \ubcfc \ub54c, \uc774\ub294 \uc624\uc2a4\ub9cc\uc758 \uc815\ubcf5\uacfc\uc815\uc73c\ub85c\ubd80\ud130 \uad6c\uc6d0\ubc1b\uc744 \uc218 \uc788\uace0, \uc774\uc2ac\ub78c\uc758 \uc9c0\ubc30\ub85c\ubd80\ud130 \ubc1c\uce78\uc744 \uc790\uc720\ub86d\uac8c \ud560 \uc218 \uc788\ub294 \uae30\ud68c\uc600\ub2e4. \ub354\ud558\uc5ec, \uc774\uc2ac\ub78c\uacfc \uae30\ub3c5\uad50\uc758 \ub300\uce58\uc120\uc740 \uc11c\uc11c\ud788 \ud5dd\uac00\ub9ac \uc655\uad6d\uc744 \ud5a5\ud558\uc5ec \uc6c0\uc9c1\uc774\uace0 \uc788\uc5c8\ub2e4. \ud5dd\uac00\ub9ac \uc655\uad6d\uc740 \uc774\uc81c \ub3d9\ubd80 \uc720\ub7fd\uc5d0\uc11c \ub450 \uc885\uad50\uc758 \uc0c8\ub85c\uc6b4 \ub300\uce58\uc120\uc774 \ub418\uace0 \uc788\uc5c8\uace0, \ud5dd\uac00\ub9ac\uc778\ub4e4\uc740 \uadf8\ub4e4 \uc790\uc2e0\uc774 \uacf5\uaca9\ubc1b\uc744 \uc704\ud611\uc5d0 \ucc98\ud574 \uc788\uc5c8\ub2e4. \ubca0\ub124\uce58\uc544 \uacf5\ud654\uad6d\uc740 \ubaa8\ub808\uc544\uc758 \uc77c\ubd80 \ubc0f \ub2ec\ub9c8\ud2f0\uc544\ub77c \ubd88\ub9b0 \ubca0\ub124\uce58\uc544\uc758 \uc601\ud1a0\ub97c \ud3ec\ud568\ud55c \ubc1c\uce78\ubc18\ub3c4\uac00 \uc624\uc2a4\ub9cc \uc81c\uad6d\uc5d0\uac8c \uc9c0\ubc30\ub2f9\ud558\uba74 \uc544\ub4dc\ub9ac\uc544 \ud574, \uc774\uc624\ub2c8\uc544 \ud574, \uc5d0\uac8c \ud574\uc5d0\uc11c \ubca0\ub124\uce58\uc544\uc758 \uc601\ud5a5\ub825\uc774 \uac10\uc18c\ud558\ub294 \uac83\uc744 \ub450\ub824\uc6cc\ud588\ub2e4. \ud55c\ud3b8 \uc81c\ub178\ubc14 \uacf5\ud654\uad6d\uc740 \ubc18\uba74\uc5d0 \ub9cc\uc57d \ub3c4\ub098\uc6b0\uac15\uacfc \ubcf4\uc2a4\ud3ec\ub8e8\uc2a4-\ub2e4\ub974\ub2e4\ub12c\uc2a4 \uc591\ucabd \ud574\ud611\uc5d0 \ub300\ud55c \ud1b5\uc81c\uad8c\uc744 \uc624\uc2a4\ub9cc \uc81c\uad6d\uc774 \ud68d\ub4dd\ud55c\ub2e4\uba74, \uce74\ud30c, \uc2dc\ub178\ud398 \ub4f1\uc758 \uc81c\ub178\ubc14\uac00 \uc911\uc694\ud558\uac8c \uc5ec\uae30\ub294 \uc2dd\ubbfc\uc9c0\ub97c \ub2e4\uc218 \ubcf4\uc720\ud558\uace0 \uc788\ub294 \ud751\ud574\uc640 \uc720\ub7fd\uc758 \uad50\uc5ed\ub85c\ub97c \ucd5c\uc885\uc801\uc73c\ub85c \uc624\uc2a4\ub9cc \uc81c\uad6d\uc774 \ub3c5\uc810\ud558\ub294 \uac83\uc744 \uc704\ud5d8\ud558\ub2e4\uace0 \ubcf4\uace0 \uc788\uc5c8\ub2e4. \uc81c\ub178\ubc14\ub294 \ub610\ud55c 1395\ub144 \ubc14\uc608\uc9c0\ub4dc 1\uc138\uac00 \ucf58\uc2a4\ud0c4\ud2f0\ub178\ud3f4\ub9ac\uc2a4\ub97c \ud3ec\uc704\ud560 \ub2f9\uc2dc \uae08\uac01\ub9cc\uc758 \ubd81\ucabd\uc5d0 \uc788\ub358 \uac08\ub77c\ud0c0\uc758 \uc694\uc0c8\ub97c \uac00\uc9c0\uace0 \uc788\uc5c8\ub294\ub370, \uc774\uacf3\uc744 \ubc14\uc57c\uc9c0\ub4dc 1\uc138\uc5d0\uac8c \uae30\uc2b5\uacf5\uaca9\uc744 \ub2f9\ud588\ub2e4.", "paragraph_answer": "1393\ub144 \ubd88\uac00\ub9ac\uc544\uc758 \uad70\uc8fc \uc774\ubc18 \uc2dc\uc2dc\ub9cc\uc740 \uc784\uc2dc \uc218\ub3c4\uc600\ub358 \ub2c8\ucf54\ud3f4\ub9ac\uc2a4\ub97c \uc624\uc2a4\ub9cc \uc81c\uad6d\uc5d0\uac8c \uacf5\uaca9\ub2f9\ud574 \ube7c\uc557\uacbc\ub2e4. \ud55c\ud3b8 \uadf8\uc758 \ub3d9\uc0dd\uc774\uc5c8\ub358 \uc774\ubc18 \uc2a4\ub77c\uce58\ubbf8\ub974 \ub294 \uc624\uc2a4\ub9cc \uc81c\uad6d\uc758 \ubd09\uc2e0\uc774 \ub418\ub294 \ub300\uac00\ub85c \uadf8\uc758 \uc601\ud1a0 \ube44\ub518\uc744 \uc720\uc9c0\ud560 \uc218 \uc788\uc5c8\ub2e4. \ubd88\uac00\ub9ac\uc544 \uadc0\uc871\ub4e4\uacfc \ub2e4\ub978 \ubc1c\uce78\uc758 \ub3c5\ub9bd\uc801\uc778 \uc9c0\ubc30\uc790\ub4e4\uacfc \uad70\uc8fc\ub4e4\uc758 \uc2dc\uac01\uc73c\ub85c \ubcfc \ub54c, \uc774\ub294 \uc624\uc2a4\ub9cc\uc758 \uc815\ubcf5\uacfc\uc815\uc73c\ub85c\ubd80\ud130 \uad6c\uc6d0\ubc1b\uc744 \uc218 \uc788\uace0, \uc774\uc2ac\ub78c\uc758 \uc9c0\ubc30\ub85c\ubd80\ud130 \ubc1c\uce78\uc744 \uc790\uc720\ub86d\uac8c \ud560 \uc218 \uc788\ub294 \uae30\ud68c\uc600\ub2e4. \ub354\ud558\uc5ec, \uc774\uc2ac\ub78c\uacfc \uae30\ub3c5\uad50\uc758 \ub300\uce58\uc120\uc740 \uc11c\uc11c\ud788 \ud5dd\uac00\ub9ac \uc655\uad6d\uc744 \ud5a5\ud558\uc5ec \uc6c0\uc9c1\uc774\uace0 \uc788\uc5c8\ub2e4. \ud5dd\uac00\ub9ac \uc655\uad6d\uc740 \uc774\uc81c \ub3d9\ubd80 \uc720\ub7fd\uc5d0\uc11c \ub450 \uc885\uad50\uc758 \uc0c8\ub85c\uc6b4 \ub300\uce58\uc120\uc774 \ub418\uace0 \uc788\uc5c8\uace0, \ud5dd\uac00\ub9ac\uc778\ub4e4\uc740 \uadf8\ub4e4 \uc790\uc2e0\uc774 \uacf5\uaca9\ubc1b\uc744 \uc704\ud611\uc5d0 \ucc98\ud574 \uc788\uc5c8\ub2e4. \ubca0\ub124\uce58\uc544 \uacf5\ud654\uad6d\uc740 \ubaa8\ub808\uc544\uc758 \uc77c\ubd80 \ubc0f \ub2ec\ub9c8\ud2f0\uc544\ub77c \ubd88\ub9b0 \ubca0\ub124\uce58\uc544\uc758 \uc601\ud1a0\ub97c \ud3ec\ud568\ud55c \ubc1c\uce78\ubc18\ub3c4\uac00 \uc624\uc2a4\ub9cc \uc81c\uad6d\uc5d0\uac8c \uc9c0\ubc30\ub2f9\ud558\uba74 \uc544\ub4dc\ub9ac\uc544 \ud574, \uc774\uc624\ub2c8\uc544 \ud574, \uc5d0\uac8c \ud574\uc5d0\uc11c \ubca0\ub124\uce58\uc544\uc758 \uc601\ud5a5\ub825\uc774 \uac10\uc18c\ud558\ub294 \uac83\uc744 \ub450\ub824\uc6cc\ud588\ub2e4. \ud55c\ud3b8 \uc81c\ub178\ubc14 \uacf5\ud654\uad6d\uc740 \ubc18\uba74\uc5d0 \ub9cc\uc57d \ub3c4\ub098\uc6b0\uac15\uacfc \ubcf4\uc2a4\ud3ec\ub8e8\uc2a4-\ub2e4\ub974\ub2e4\ub12c\uc2a4 \uc591\ucabd \ud574\ud611\uc5d0 \ub300\ud55c \ud1b5\uc81c\uad8c\uc744 \uc624\uc2a4\ub9cc \uc81c\uad6d\uc774 \ud68d\ub4dd\ud55c\ub2e4\uba74, \uce74\ud30c, \uc2dc\ub178\ud398 \ub4f1\uc758 \uc81c\ub178\ubc14\uac00 \uc911\uc694\ud558\uac8c \uc5ec\uae30\ub294 \uc2dd\ubbfc\uc9c0\ub97c \ub2e4\uc218 \ubcf4\uc720\ud558\uace0 \uc788\ub294 \ud751\ud574\uc640 \uc720\ub7fd\uc758 \uad50\uc5ed\ub85c\ub97c \ucd5c\uc885\uc801\uc73c\ub85c \uc624\uc2a4\ub9cc \uc81c\uad6d\uc774 \ub3c5\uc810\ud558\ub294 \uac83\uc744 \uc704\ud5d8\ud558\ub2e4\uace0 \ubcf4\uace0 \uc788\uc5c8\ub2e4. \uc81c\ub178\ubc14\ub294 \ub610\ud55c 1395\ub144 \ubc14\uc608\uc9c0\ub4dc 1\uc138\uac00 \ucf58\uc2a4\ud0c4\ud2f0\ub178\ud3f4\ub9ac\uc2a4\ub97c \ud3ec\uc704\ud560 \ub2f9\uc2dc \uae08\uac01\ub9cc\uc758 \ubd81\ucabd\uc5d0 \uc788\ub358 \uac08\ub77c\ud0c0\uc758 \uc694\uc0c8\ub97c \uac00\uc9c0\uace0 \uc788\uc5c8\ub294\ub370, \uc774\uacf3\uc744 \ubc14\uc57c\uc9c0\ub4dc 1\uc138\uc5d0\uac8c \uae30\uc2b5\uacf5\uaca9\uc744 \ub2f9\ud588\ub2e4.", "sentence_answer": "\ud55c\ud3b8 \uadf8\uc758 \ub3d9\uc0dd\uc774\uc5c8\ub358 \uc774\ubc18 \uc2a4\ub77c\uce58\ubbf8\ub974 \ub294 \uc624\uc2a4\ub9cc \uc81c\uad6d\uc758 \ubd09\uc2e0\uc774 \ub418\ub294 \ub300\uac00\ub85c \uadf8\uc758 \uc601\ud1a0 \ube44\ub518\uc744 \uc720\uc9c0\ud560 \uc218 \uc788\uc5c8\ub2e4.", "paragraph_id": "6484822-1-0", "paragraph_question": "question: \uc624\uc2a4\ub9cc \uc81c\uad6d\uc758 \ubd09\uc2e0\uc774 \ub418\uc5b4 \uc601\ud1a0\uc778 \ube44\ub518\uc744 \ubcf4\uc804\ud55c \uc778\ubb3c\uc740?, context: 1393\ub144 \ubd88\uac00\ub9ac\uc544\uc758 \uad70\uc8fc \uc774\ubc18 \uc2dc\uc2dc\ub9cc\uc740 \uc784\uc2dc \uc218\ub3c4\uc600\ub358 \ub2c8\ucf54\ud3f4\ub9ac\uc2a4\ub97c \uc624\uc2a4\ub9cc \uc81c\uad6d\uc5d0\uac8c \uacf5\uaca9\ub2f9\ud574 \ube7c\uc557\uacbc\ub2e4. \ud55c\ud3b8 \uadf8\uc758 \ub3d9\uc0dd\uc774\uc5c8\ub358 \uc774\ubc18 \uc2a4\ub77c\uce58\ubbf8\ub974\ub294 \uc624\uc2a4\ub9cc \uc81c\uad6d\uc758 \ubd09\uc2e0\uc774 \ub418\ub294 \ub300\uac00\ub85c \uadf8\uc758 \uc601\ud1a0 \ube44\ub518\uc744 \uc720\uc9c0\ud560 \uc218 \uc788\uc5c8\ub2e4. \ubd88\uac00\ub9ac\uc544 \uadc0\uc871\ub4e4\uacfc \ub2e4\ub978 \ubc1c\uce78\uc758 \ub3c5\ub9bd\uc801\uc778 \uc9c0\ubc30\uc790\ub4e4\uacfc \uad70\uc8fc\ub4e4\uc758 \uc2dc\uac01\uc73c\ub85c \ubcfc \ub54c, \uc774\ub294 \uc624\uc2a4\ub9cc\uc758 \uc815\ubcf5\uacfc\uc815\uc73c\ub85c\ubd80\ud130 \uad6c\uc6d0\ubc1b\uc744 \uc218 \uc788\uace0, \uc774\uc2ac\ub78c\uc758 \uc9c0\ubc30\ub85c\ubd80\ud130 \ubc1c\uce78\uc744 \uc790\uc720\ub86d\uac8c \ud560 \uc218 \uc788\ub294 \uae30\ud68c\uc600\ub2e4. \ub354\ud558\uc5ec, \uc774\uc2ac\ub78c\uacfc \uae30\ub3c5\uad50\uc758 \ub300\uce58\uc120\uc740 \uc11c\uc11c\ud788 \ud5dd\uac00\ub9ac \uc655\uad6d\uc744 \ud5a5\ud558\uc5ec \uc6c0\uc9c1\uc774\uace0 \uc788\uc5c8\ub2e4. \ud5dd\uac00\ub9ac \uc655\uad6d\uc740 \uc774\uc81c \ub3d9\ubd80 \uc720\ub7fd\uc5d0\uc11c \ub450 \uc885\uad50\uc758 \uc0c8\ub85c\uc6b4 \ub300\uce58\uc120\uc774 \ub418\uace0 \uc788\uc5c8\uace0, \ud5dd\uac00\ub9ac\uc778\ub4e4\uc740 \uadf8\ub4e4 \uc790\uc2e0\uc774 \uacf5\uaca9\ubc1b\uc744 \uc704\ud611\uc5d0 \ucc98\ud574 \uc788\uc5c8\ub2e4. \ubca0\ub124\uce58\uc544 \uacf5\ud654\uad6d\uc740 \ubaa8\ub808\uc544\uc758 \uc77c\ubd80 \ubc0f \ub2ec\ub9c8\ud2f0\uc544\ub77c \ubd88\ub9b0 \ubca0\ub124\uce58\uc544\uc758 \uc601\ud1a0\ub97c \ud3ec\ud568\ud55c \ubc1c\uce78\ubc18\ub3c4\uac00 \uc624\uc2a4\ub9cc \uc81c\uad6d\uc5d0\uac8c \uc9c0\ubc30\ub2f9\ud558\uba74 \uc544\ub4dc\ub9ac\uc544 \ud574, \uc774\uc624\ub2c8\uc544 \ud574, \uc5d0\uac8c \ud574\uc5d0\uc11c \ubca0\ub124\uce58\uc544\uc758 \uc601\ud5a5\ub825\uc774 \uac10\uc18c\ud558\ub294 \uac83\uc744 \ub450\ub824\uc6cc\ud588\ub2e4. \ud55c\ud3b8 \uc81c\ub178\ubc14 \uacf5\ud654\uad6d\uc740 \ubc18\uba74\uc5d0 \ub9cc\uc57d \ub3c4\ub098\uc6b0\uac15\uacfc \ubcf4\uc2a4\ud3ec\ub8e8\uc2a4-\ub2e4\ub974\ub2e4\ub12c\uc2a4 \uc591\ucabd \ud574\ud611\uc5d0 \ub300\ud55c \ud1b5\uc81c\uad8c\uc744 \uc624\uc2a4\ub9cc \uc81c\uad6d\uc774 \ud68d\ub4dd\ud55c\ub2e4\uba74, \uce74\ud30c, \uc2dc\ub178\ud398 \ub4f1\uc758 \uc81c\ub178\ubc14\uac00 \uc911\uc694\ud558\uac8c \uc5ec\uae30\ub294 \uc2dd\ubbfc\uc9c0\ub97c \ub2e4\uc218 \ubcf4\uc720\ud558\uace0 \uc788\ub294 \ud751\ud574\uc640 \uc720\ub7fd\uc758 \uad50\uc5ed\ub85c\ub97c \ucd5c\uc885\uc801\uc73c\ub85c \uc624\uc2a4\ub9cc \uc81c\uad6d\uc774 \ub3c5\uc810\ud558\ub294 \uac83\uc744 \uc704\ud5d8\ud558\ub2e4\uace0 \ubcf4\uace0 \uc788\uc5c8\ub2e4. \uc81c\ub178\ubc14\ub294 \ub610\ud55c 1395\ub144 \ubc14\uc608\uc9c0\ub4dc 1\uc138\uac00 \ucf58\uc2a4\ud0c4\ud2f0\ub178\ud3f4\ub9ac\uc2a4\ub97c \ud3ec\uc704\ud560 \ub2f9\uc2dc \uae08\uac01\ub9cc\uc758 \ubd81\ucabd\uc5d0 \uc788\ub358 \uac08\ub77c\ud0c0\uc758 \uc694\uc0c8\ub97c \uac00\uc9c0\uace0 \uc788\uc5c8\ub294\ub370, \uc774\uacf3\uc744 \ubc14\uc57c\uc9c0\ub4dc 1\uc138\uc5d0\uac8c \uae30\uc2b5\uacf5\uaca9\uc744 \ub2f9\ud588\ub2e4."} {"question": "\uc815\ud0dc\ud658 \uad50\uc218\ub294 \uace0\ub824\ub300\ud559\uad50 \uc5b4\ub290 \ud559\uacfc \uc18c\uc18d\uc778\uac00?", "paragraph": "\ub9e4\ub144 5\uc6d4 \uc911\uc5d0\ub294 \uc11d\ud0d1\ub300\ub3d9\uc81c(\u77f3\u5854\u5927\u540c\u796d)\ub77c\ub294 \uc774\ub984\uc758 \ud559\uc0dd \ucd95\uc81c\uac00 \uc5f4\ub9b0\ub2e4. 1962\ub144 5\uc6d4 4\uc77c\ubd80\ud130 3\uc77c\uac04 \uac1c\ucd5c\ub41c \u2018\uc11d\ud0d1\ucd95\uc804\u2019\uc774 \uc774 \ud589\uc0ac\uc758 \uc5f0\uc6d0\uc73c\ub85c \uc54c\ub824\uc838 \uc788\ub294\ub370, \uc5ec\uae30\uc11c \u2018\uc11d\ud0d1\u2019\uc774\ub77c\ub294 \uc774\ub984\uc740 \ub2f9\uc2dc \ubcf8\uad50 \uad50\uc218\ub85c \uc7ac\uc9c1 \uc911\uc774\ub358 \uc870\uc9c0\ud6c8\uc774 \uc9c0\uc5c8\ub2e4\uace0 \ud55c\ub2e4. \uc11d\ud0d1\ucd95\uc804\uc774 \u2018\ub300\ub3d9\uc81c\u2019\ub77c\ub294 \uc774\ub984\uc73c\ub85c \ubd88\ub9ac\uae30 \uc2dc\uc791\ud55c \uac83\uc740 1984\ub144\uc758 \uc77c\uc774\ub2e4. \uc0ac\ud68c \ube44\ud310\uc801\uc778 \ud589\uc0ac\ub97c \ud1b5\ud574 \ud559\uc0dd\ub4e4\uc758 \ucc38\uc5ec\uc758\uc2dd\uc744 \uace0\ucde8\ud558\uc600\uc73c\uba70, \ub2f9\uc2dc \ub3c5\uc7ac\uc815\uad8c\uc5d0 \uc758\ud574 \uc758\ubb38\uc758 \uc8fd\uc74c\uc744 \ub2f9\ud55c \ud559\uc0dd\ub4e4\uc744 \ucd94\ubaa8\ud558\ub294 \ud569\ub3d9\uc704\ub839\uad7f \ub4f1\uc744 \uc9c0\ub0b4\uba74\uc11c 3\ub144 \ud6c4\uc778 1987\ub144\uc5d0\ub294 \uc804\uad6d \ub300\ubd80\ubd84\uc758 \ub300\ud559\uc5d0\uc11c \u2018\ub300\ub3d9\uc81c\u2019\ub77c\ub294 \uc774\ub984\uc73c\ub85c \ucd95\uc81c\ub97c \uc5f4 \uc815\ub3c4\ub85c \u2018\ub300\ub3d9\uc81c\u2019\ub77c\ub294 \uc774\ub984\uc740 \ub300\ud559\uac00 \uc804\uc5ed\uc5d0 \ub110\ub9ac \ud37c\uc838\ub098\uac14\ub2e4. \uc778\ubb38\ub300\ud559 \uc0ac\ud68c\ud559\uacfc \uad50\uc218\ub85c \uc7ac\uc9c1 \uc911\uc778 \uc815\ud0dc\ud658\uc740 \u2018\uad50\ub0b4 \uad6c\uc131\uc6d0 \uac04\uc5d0 \uc18c\uc18d\uac10\uacfc \uc77c\uccb4\uac10\uc744 \ub290\ub080\ub2e4\ub294 \uba74\uc5d0\uc11c \uc2a4\ud2b8\ub808\uc2a4 \ud574\ubc29\uad6c \uc774\uc0c1\uc758 \uc758\ubbf8\ub97c \uc9c0\ub2cc\ub2e4\u2019\uba74\uc11c \uad50\ub0b4 \ucd95\uc81c\uc5d0 \uadf8 \uc758\ubbf8\ub97c \ubd80\uc5ec\ud558\uc600\ub2e4. \uc11d\ud0d1\ub300\ub3d9\uc81c\ub294 \uac1c\uad50\uae30\ub150\uc77c\uc758 \ub300\ud45c \ud589\uc0ac\uc600\uc73c\ub098 2000\ub144\ub300 \ub4e4\uc5b4 \uac1c\ucd5c\uae30\uac04\uc774 \uae38\uac8c\ub294 3\uc8fc\uae4c\uc9c0 \ubbf8\ub904\uc9c0\uba70 \uac1c\uad50\uae30\ub150\uc77c\uacfc\uc758 \uac70\ub9ac\uac00 \uba40\uc5b4\uc84c\ub2e4. \ub300\ub3d9\uc81c \uae30\uac04\uc5d0\ub294 \uc5ec\ub7ec \ud559\uacfc/\ubc18 \ud559\uc0dd\ud68c\uc640 \ub3d9\uc544\ub9ac\uc5d0\uc11c \uc8fc\uc810\uc744 \ube44\ub86f\ud55c \uac01\uc885 \uc774\ubca4\ud2b8\ub4e4\uc774 \uad50\ub0b4 \uacf3\uacf3\uc5d0\uc11c \uac1c\ucd5c\ub418\uba70, \uc8fc\uc694 \ud589\uc0ac \uc77c\uc815\uc740 \uad50\ub0b4\uc5d0 \ube44\uce58\ub41c \uace0\ub300\uc2e0\ubb38 \ub4f1\uc744 \ud1b5\ud574 \ud655\uc778\ud560 \uc218 \uc788\ub2e4. \ub300\ub3d9\uc81c \uae30\uac04\uc5d0 \uc560\uae30\ub2a5 \uc9c0\uc5ed\uc5d0\uc11c\ub294 \uc560\uae30\ub2a5\ub3d9\uc544\ub9ac\uc5f0\ud569\ud68c \uc758 \uc8fc\ub3c4\ub85c \ubcc4\ub3c4\uc758 \ud589\uc0ac\uac00 \uc5f4\ub9ac\uba70 2011\ub144\uc758 \uacbd\uc6b0 \uacf5\uacfc\ub300, \uc0dd\uba85\ub300, \uc774\uacfc\ub300, \ubcf4\uacfc\ub300, \uc815\ud1b5\ub300\uc758 5\uac1c \ub2e8\uacfc\ub300\ud559\uc774 \ucc38\uc5ec\ud558\uc5ec \ub178\ubca8\uad11\uc7a5\uc744 \uc8fc \ubb34\ub300\ub85c \u2018Sound Festival\u2019(\uc0ac\uc6b4\ub4dc \ud398\uc2a4\ud2f0\ubc8c)\uacfc \u2018\uc560\uae30\ub2a5 \uc2e4\ub8e8\uc5e3 \uac00\uc694\uc81c\u2019, \u2018\uc560\uae30\ub2a5 \uc694\ub9ac \uacbd\uc5f0\ub300\ud68c\u2019 \ub4f1\uacfc \ub354\ubd88\uc5b4 \uac01\uc885 \uac00\uc218\ub4e4\uc758 \ucd95\ud558\uacf5\uc5f0\uc774 \ud3bc\uccd0\uc84c\ub2e4.", "answer": "\uc0ac\ud68c\ud559\uacfc", "sentence": "\uc778\ubb38\ub300\ud559 \uc0ac\ud68c\ud559\uacfc \uad50\uc218\ub85c \uc7ac\uc9c1 \uc911\uc778 \uc815\ud0dc\ud658\uc740 \u2018\uad50\ub0b4 \uad6c\uc131\uc6d0 \uac04\uc5d0 \uc18c\uc18d\uac10\uacfc \uc77c\uccb4\uac10\uc744 \ub290\ub080\ub2e4\ub294 \uba74\uc5d0\uc11c \uc2a4\ud2b8\ub808\uc2a4 \ud574\ubc29\uad6c \uc774\uc0c1\uc758 \uc758\ubbf8\ub97c \uc9c0\ub2cc\ub2e4\u2019\uba74\uc11c \uad50\ub0b4 \ucd95\uc81c\uc5d0 \uadf8 \uc758\ubbf8\ub97c \ubd80\uc5ec\ud558\uc600\ub2e4.", "paragraph_sentence": "\ub9e4\ub144 5\uc6d4 \uc911\uc5d0\ub294 \uc11d\ud0d1\ub300\ub3d9\uc81c(\u77f3\u5854\u5927\u540c\u796d)\ub77c\ub294 \uc774\ub984\uc758 \ud559\uc0dd \ucd95\uc81c\uac00 \uc5f4\ub9b0\ub2e4. 1962\ub144 5\uc6d4 4\uc77c\ubd80\ud130 3\uc77c\uac04 \uac1c\ucd5c\ub41c \u2018\uc11d\ud0d1\ucd95\uc804\u2019\uc774 \uc774 \ud589\uc0ac\uc758 \uc5f0\uc6d0\uc73c\ub85c \uc54c\ub824\uc838 \uc788\ub294\ub370, \uc5ec\uae30\uc11c \u2018\uc11d\ud0d1\u2019\uc774\ub77c\ub294 \uc774\ub984\uc740 \ub2f9\uc2dc \ubcf8\uad50 \uad50\uc218\ub85c \uc7ac\uc9c1 \uc911\uc774\ub358 \uc870\uc9c0\ud6c8\uc774 \uc9c0\uc5c8\ub2e4\uace0 \ud55c\ub2e4. \uc11d\ud0d1\ucd95\uc804\uc774 \u2018\ub300\ub3d9\uc81c\u2019\ub77c\ub294 \uc774\ub984\uc73c\ub85c \ubd88\ub9ac\uae30 \uc2dc\uc791\ud55c \uac83\uc740 1984\ub144\uc758 \uc77c\uc774\ub2e4. \uc0ac\ud68c \ube44\ud310\uc801\uc778 \ud589\uc0ac\ub97c \ud1b5\ud574 \ud559\uc0dd\ub4e4\uc758 \ucc38\uc5ec\uc758\uc2dd\uc744 \uace0\ucde8\ud558\uc600\uc73c\uba70, \ub2f9\uc2dc \ub3c5\uc7ac\uc815\uad8c\uc5d0 \uc758\ud574 \uc758\ubb38\uc758 \uc8fd\uc74c\uc744 \ub2f9\ud55c \ud559\uc0dd\ub4e4\uc744 \ucd94\ubaa8\ud558\ub294 \ud569\ub3d9\uc704\ub839\uad7f \ub4f1\uc744 \uc9c0\ub0b4\uba74\uc11c 3\ub144 \ud6c4\uc778 1987\ub144\uc5d0\ub294 \uc804\uad6d \ub300\ubd80\ubd84\uc758 \ub300\ud559\uc5d0\uc11c \u2018\ub300\ub3d9\uc81c\u2019\ub77c\ub294 \uc774\ub984\uc73c\ub85c \ucd95\uc81c\ub97c \uc5f4 \uc815\ub3c4\ub85c \u2018\ub300\ub3d9\uc81c\u2019\ub77c\ub294 \uc774\ub984\uc740 \ub300\ud559\uac00 \uc804\uc5ed\uc5d0 \ub110\ub9ac \ud37c\uc838\ub098\uac14\ub2e4. \uc778\ubb38\ub300\ud559 \uc0ac\ud68c\ud559\uacfc \uad50\uc218\ub85c \uc7ac\uc9c1 \uc911\uc778 \uc815\ud0dc\ud658\uc740 \u2018\uad50\ub0b4 \uad6c\uc131\uc6d0 \uac04\uc5d0 \uc18c\uc18d\uac10\uacfc \uc77c\uccb4\uac10\uc744 \ub290\ub080\ub2e4\ub294 \uba74\uc5d0\uc11c \uc2a4\ud2b8\ub808\uc2a4 \ud574\ubc29\uad6c \uc774\uc0c1\uc758 \uc758\ubbf8\ub97c \uc9c0\ub2cc\ub2e4\u2019\uba74\uc11c \uad50\ub0b4 \ucd95\uc81c\uc5d0 \uadf8 \uc758\ubbf8\ub97c \ubd80\uc5ec\ud558\uc600\ub2e4. \uc11d\ud0d1\ub300\ub3d9\uc81c\ub294 \uac1c\uad50\uae30\ub150\uc77c\uc758 \ub300\ud45c \ud589\uc0ac\uc600\uc73c\ub098 2000\ub144\ub300 \ub4e4\uc5b4 \uac1c\ucd5c\uae30\uac04\uc774 \uae38\uac8c\ub294 3\uc8fc\uae4c\uc9c0 \ubbf8\ub904\uc9c0\uba70 \uac1c\uad50\uae30\ub150\uc77c\uacfc\uc758 \uac70\ub9ac\uac00 \uba40\uc5b4\uc84c\ub2e4. \ub300\ub3d9\uc81c \uae30\uac04\uc5d0\ub294 \uc5ec\ub7ec \ud559\uacfc/\ubc18 \ud559\uc0dd\ud68c\uc640 \ub3d9\uc544\ub9ac\uc5d0\uc11c \uc8fc\uc810\uc744 \ube44\ub86f\ud55c \uac01\uc885 \uc774\ubca4\ud2b8\ub4e4\uc774 \uad50\ub0b4 \uacf3\uacf3\uc5d0\uc11c \uac1c\ucd5c\ub418\uba70, \uc8fc\uc694 \ud589\uc0ac \uc77c\uc815\uc740 \uad50\ub0b4\uc5d0 \ube44\uce58\ub41c \uace0\ub300\uc2e0\ubb38 \ub4f1\uc744 \ud1b5\ud574 \ud655\uc778\ud560 \uc218 \uc788\ub2e4. \ub300\ub3d9\uc81c \uae30\uac04\uc5d0 \uc560\uae30\ub2a5 \uc9c0\uc5ed\uc5d0\uc11c\ub294 \uc560\uae30\ub2a5\ub3d9\uc544\ub9ac\uc5f0\ud569\ud68c \uc758 \uc8fc\ub3c4\ub85c \ubcc4\ub3c4\uc758 \ud589\uc0ac\uac00 \uc5f4\ub9ac\uba70 2011\ub144\uc758 \uacbd\uc6b0 \uacf5\uacfc\ub300, \uc0dd\uba85\ub300, \uc774\uacfc\ub300, \ubcf4\uacfc\ub300, \uc815\ud1b5\ub300\uc758 5\uac1c \ub2e8\uacfc\ub300\ud559\uc774 \ucc38\uc5ec\ud558\uc5ec \ub178\ubca8\uad11\uc7a5\uc744 \uc8fc \ubb34\ub300\ub85c \u2018Sound Festival\u2019(\uc0ac\uc6b4\ub4dc \ud398\uc2a4\ud2f0\ubc8c)\uacfc \u2018\uc560\uae30\ub2a5 \uc2e4\ub8e8\uc5e3 \uac00\uc694\uc81c\u2019, \u2018\uc560\uae30\ub2a5 \uc694\ub9ac \uacbd\uc5f0\ub300\ud68c\u2019 \ub4f1\uacfc \ub354\ubd88\uc5b4 \uac01\uc885 \uac00\uc218\ub4e4\uc758 \ucd95\ud558\uacf5\uc5f0\uc774 \ud3bc\uccd0\uc84c\ub2e4.", "paragraph_answer": "\ub9e4\ub144 5\uc6d4 \uc911\uc5d0\ub294 \uc11d\ud0d1\ub300\ub3d9\uc81c(\u77f3\u5854\u5927\u540c\u796d)\ub77c\ub294 \uc774\ub984\uc758 \ud559\uc0dd \ucd95\uc81c\uac00 \uc5f4\ub9b0\ub2e4. 1962\ub144 5\uc6d4 4\uc77c\ubd80\ud130 3\uc77c\uac04 \uac1c\ucd5c\ub41c \u2018\uc11d\ud0d1\ucd95\uc804\u2019\uc774 \uc774 \ud589\uc0ac\uc758 \uc5f0\uc6d0\uc73c\ub85c \uc54c\ub824\uc838 \uc788\ub294\ub370, \uc5ec\uae30\uc11c \u2018\uc11d\ud0d1\u2019\uc774\ub77c\ub294 \uc774\ub984\uc740 \ub2f9\uc2dc \ubcf8\uad50 \uad50\uc218\ub85c \uc7ac\uc9c1 \uc911\uc774\ub358 \uc870\uc9c0\ud6c8\uc774 \uc9c0\uc5c8\ub2e4\uace0 \ud55c\ub2e4. \uc11d\ud0d1\ucd95\uc804\uc774 \u2018\ub300\ub3d9\uc81c\u2019\ub77c\ub294 \uc774\ub984\uc73c\ub85c \ubd88\ub9ac\uae30 \uc2dc\uc791\ud55c \uac83\uc740 1984\ub144\uc758 \uc77c\uc774\ub2e4. \uc0ac\ud68c \ube44\ud310\uc801\uc778 \ud589\uc0ac\ub97c \ud1b5\ud574 \ud559\uc0dd\ub4e4\uc758 \ucc38\uc5ec\uc758\uc2dd\uc744 \uace0\ucde8\ud558\uc600\uc73c\uba70, \ub2f9\uc2dc \ub3c5\uc7ac\uc815\uad8c\uc5d0 \uc758\ud574 \uc758\ubb38\uc758 \uc8fd\uc74c\uc744 \ub2f9\ud55c \ud559\uc0dd\ub4e4\uc744 \ucd94\ubaa8\ud558\ub294 \ud569\ub3d9\uc704\ub839\uad7f \ub4f1\uc744 \uc9c0\ub0b4\uba74\uc11c 3\ub144 \ud6c4\uc778 1987\ub144\uc5d0\ub294 \uc804\uad6d \ub300\ubd80\ubd84\uc758 \ub300\ud559\uc5d0\uc11c \u2018\ub300\ub3d9\uc81c\u2019\ub77c\ub294 \uc774\ub984\uc73c\ub85c \ucd95\uc81c\ub97c \uc5f4 \uc815\ub3c4\ub85c \u2018\ub300\ub3d9\uc81c\u2019\ub77c\ub294 \uc774\ub984\uc740 \ub300\ud559\uac00 \uc804\uc5ed\uc5d0 \ub110\ub9ac \ud37c\uc838\ub098\uac14\ub2e4. \uc778\ubb38\ub300\ud559 \uc0ac\ud68c\ud559\uacfc \uad50\uc218\ub85c \uc7ac\uc9c1 \uc911\uc778 \uc815\ud0dc\ud658\uc740 \u2018\uad50\ub0b4 \uad6c\uc131\uc6d0 \uac04\uc5d0 \uc18c\uc18d\uac10\uacfc \uc77c\uccb4\uac10\uc744 \ub290\ub080\ub2e4\ub294 \uba74\uc5d0\uc11c \uc2a4\ud2b8\ub808\uc2a4 \ud574\ubc29\uad6c \uc774\uc0c1\uc758 \uc758\ubbf8\ub97c \uc9c0\ub2cc\ub2e4\u2019\uba74\uc11c \uad50\ub0b4 \ucd95\uc81c\uc5d0 \uadf8 \uc758\ubbf8\ub97c \ubd80\uc5ec\ud558\uc600\ub2e4. \uc11d\ud0d1\ub300\ub3d9\uc81c\ub294 \uac1c\uad50\uae30\ub150\uc77c\uc758 \ub300\ud45c \ud589\uc0ac\uc600\uc73c\ub098 2000\ub144\ub300 \ub4e4\uc5b4 \uac1c\ucd5c\uae30\uac04\uc774 \uae38\uac8c\ub294 3\uc8fc\uae4c\uc9c0 \ubbf8\ub904\uc9c0\uba70 \uac1c\uad50\uae30\ub150\uc77c\uacfc\uc758 \uac70\ub9ac\uac00 \uba40\uc5b4\uc84c\ub2e4. \ub300\ub3d9\uc81c \uae30\uac04\uc5d0\ub294 \uc5ec\ub7ec \ud559\uacfc/\ubc18 \ud559\uc0dd\ud68c\uc640 \ub3d9\uc544\ub9ac\uc5d0\uc11c \uc8fc\uc810\uc744 \ube44\ub86f\ud55c \uac01\uc885 \uc774\ubca4\ud2b8\ub4e4\uc774 \uad50\ub0b4 \uacf3\uacf3\uc5d0\uc11c \uac1c\ucd5c\ub418\uba70, \uc8fc\uc694 \ud589\uc0ac \uc77c\uc815\uc740 \uad50\ub0b4\uc5d0 \ube44\uce58\ub41c \uace0\ub300\uc2e0\ubb38 \ub4f1\uc744 \ud1b5\ud574 \ud655\uc778\ud560 \uc218 \uc788\ub2e4. \ub300\ub3d9\uc81c \uae30\uac04\uc5d0 \uc560\uae30\ub2a5 \uc9c0\uc5ed\uc5d0\uc11c\ub294 \uc560\uae30\ub2a5\ub3d9\uc544\ub9ac\uc5f0\ud569\ud68c \uc758 \uc8fc\ub3c4\ub85c \ubcc4\ub3c4\uc758 \ud589\uc0ac\uac00 \uc5f4\ub9ac\uba70 2011\ub144\uc758 \uacbd\uc6b0 \uacf5\uacfc\ub300, \uc0dd\uba85\ub300, \uc774\uacfc\ub300, \ubcf4\uacfc\ub300, \uc815\ud1b5\ub300\uc758 5\uac1c \ub2e8\uacfc\ub300\ud559\uc774 \ucc38\uc5ec\ud558\uc5ec \ub178\ubca8\uad11\uc7a5\uc744 \uc8fc \ubb34\ub300\ub85c \u2018Sound Festival\u2019(\uc0ac\uc6b4\ub4dc \ud398\uc2a4\ud2f0\ubc8c)\uacfc \u2018\uc560\uae30\ub2a5 \uc2e4\ub8e8\uc5e3 \uac00\uc694\uc81c\u2019, \u2018\uc560\uae30\ub2a5 \uc694\ub9ac \uacbd\uc5f0\ub300\ud68c\u2019 \ub4f1\uacfc \ub354\ubd88\uc5b4 \uac01\uc885 \uac00\uc218\ub4e4\uc758 \ucd95\ud558\uacf5\uc5f0\uc774 \ud3bc\uccd0\uc84c\ub2e4.", "sentence_answer": "\uc778\ubb38\ub300\ud559 \uc0ac\ud68c\ud559\uacfc \uad50\uc218\ub85c \uc7ac\uc9c1 \uc911\uc778 \uc815\ud0dc\ud658\uc740 \u2018\uad50\ub0b4 \uad6c\uc131\uc6d0 \uac04\uc5d0 \uc18c\uc18d\uac10\uacfc \uc77c\uccb4\uac10\uc744 \ub290\ub080\ub2e4\ub294 \uba74\uc5d0\uc11c \uc2a4\ud2b8\ub808\uc2a4 \ud574\ubc29\uad6c \uc774\uc0c1\uc758 \uc758\ubbf8\ub97c \uc9c0\ub2cc\ub2e4\u2019\uba74\uc11c \uad50\ub0b4 \ucd95\uc81c\uc5d0 \uadf8 \uc758\ubbf8\ub97c \ubd80\uc5ec\ud558\uc600\ub2e4.", "paragraph_id": "6533861-12-2", "paragraph_question": "question: \uc815\ud0dc\ud658 \uad50\uc218\ub294 \uace0\ub824\ub300\ud559\uad50 \uc5b4\ub290 \ud559\uacfc \uc18c\uc18d\uc778\uac00?, context: \ub9e4\ub144 5\uc6d4 \uc911\uc5d0\ub294 \uc11d\ud0d1\ub300\ub3d9\uc81c(\u77f3\u5854\u5927\u540c\u796d)\ub77c\ub294 \uc774\ub984\uc758 \ud559\uc0dd \ucd95\uc81c\uac00 \uc5f4\ub9b0\ub2e4. 1962\ub144 5\uc6d4 4\uc77c\ubd80\ud130 3\uc77c\uac04 \uac1c\ucd5c\ub41c \u2018\uc11d\ud0d1\ucd95\uc804\u2019\uc774 \uc774 \ud589\uc0ac\uc758 \uc5f0\uc6d0\uc73c\ub85c \uc54c\ub824\uc838 \uc788\ub294\ub370, \uc5ec\uae30\uc11c \u2018\uc11d\ud0d1\u2019\uc774\ub77c\ub294 \uc774\ub984\uc740 \ub2f9\uc2dc \ubcf8\uad50 \uad50\uc218\ub85c \uc7ac\uc9c1 \uc911\uc774\ub358 \uc870\uc9c0\ud6c8\uc774 \uc9c0\uc5c8\ub2e4\uace0 \ud55c\ub2e4. \uc11d\ud0d1\ucd95\uc804\uc774 \u2018\ub300\ub3d9\uc81c\u2019\ub77c\ub294 \uc774\ub984\uc73c\ub85c \ubd88\ub9ac\uae30 \uc2dc\uc791\ud55c \uac83\uc740 1984\ub144\uc758 \uc77c\uc774\ub2e4. \uc0ac\ud68c \ube44\ud310\uc801\uc778 \ud589\uc0ac\ub97c \ud1b5\ud574 \ud559\uc0dd\ub4e4\uc758 \ucc38\uc5ec\uc758\uc2dd\uc744 \uace0\ucde8\ud558\uc600\uc73c\uba70, \ub2f9\uc2dc \ub3c5\uc7ac\uc815\uad8c\uc5d0 \uc758\ud574 \uc758\ubb38\uc758 \uc8fd\uc74c\uc744 \ub2f9\ud55c \ud559\uc0dd\ub4e4\uc744 \ucd94\ubaa8\ud558\ub294 \ud569\ub3d9\uc704\ub839\uad7f \ub4f1\uc744 \uc9c0\ub0b4\uba74\uc11c 3\ub144 \ud6c4\uc778 1987\ub144\uc5d0\ub294 \uc804\uad6d \ub300\ubd80\ubd84\uc758 \ub300\ud559\uc5d0\uc11c \u2018\ub300\ub3d9\uc81c\u2019\ub77c\ub294 \uc774\ub984\uc73c\ub85c \ucd95\uc81c\ub97c \uc5f4 \uc815\ub3c4\ub85c \u2018\ub300\ub3d9\uc81c\u2019\ub77c\ub294 \uc774\ub984\uc740 \ub300\ud559\uac00 \uc804\uc5ed\uc5d0 \ub110\ub9ac \ud37c\uc838\ub098\uac14\ub2e4. \uc778\ubb38\ub300\ud559 \uc0ac\ud68c\ud559\uacfc \uad50\uc218\ub85c \uc7ac\uc9c1 \uc911\uc778 \uc815\ud0dc\ud658\uc740 \u2018\uad50\ub0b4 \uad6c\uc131\uc6d0 \uac04\uc5d0 \uc18c\uc18d\uac10\uacfc \uc77c\uccb4\uac10\uc744 \ub290\ub080\ub2e4\ub294 \uba74\uc5d0\uc11c \uc2a4\ud2b8\ub808\uc2a4 \ud574\ubc29\uad6c \uc774\uc0c1\uc758 \uc758\ubbf8\ub97c \uc9c0\ub2cc\ub2e4\u2019\uba74\uc11c \uad50\ub0b4 \ucd95\uc81c\uc5d0 \uadf8 \uc758\ubbf8\ub97c \ubd80\uc5ec\ud558\uc600\ub2e4. \uc11d\ud0d1\ub300\ub3d9\uc81c\ub294 \uac1c\uad50\uae30\ub150\uc77c\uc758 \ub300\ud45c \ud589\uc0ac\uc600\uc73c\ub098 2000\ub144\ub300 \ub4e4\uc5b4 \uac1c\ucd5c\uae30\uac04\uc774 \uae38\uac8c\ub294 3\uc8fc\uae4c\uc9c0 \ubbf8\ub904\uc9c0\uba70 \uac1c\uad50\uae30\ub150\uc77c\uacfc\uc758 \uac70\ub9ac\uac00 \uba40\uc5b4\uc84c\ub2e4. \ub300\ub3d9\uc81c \uae30\uac04\uc5d0\ub294 \uc5ec\ub7ec \ud559\uacfc/\ubc18 \ud559\uc0dd\ud68c\uc640 \ub3d9\uc544\ub9ac\uc5d0\uc11c \uc8fc\uc810\uc744 \ube44\ub86f\ud55c \uac01\uc885 \uc774\ubca4\ud2b8\ub4e4\uc774 \uad50\ub0b4 \uacf3\uacf3\uc5d0\uc11c \uac1c\ucd5c\ub418\uba70, \uc8fc\uc694 \ud589\uc0ac \uc77c\uc815\uc740 \uad50\ub0b4\uc5d0 \ube44\uce58\ub41c \uace0\ub300\uc2e0\ubb38 \ub4f1\uc744 \ud1b5\ud574 \ud655\uc778\ud560 \uc218 \uc788\ub2e4. \ub300\ub3d9\uc81c \uae30\uac04\uc5d0 \uc560\uae30\ub2a5 \uc9c0\uc5ed\uc5d0\uc11c\ub294 \uc560\uae30\ub2a5\ub3d9\uc544\ub9ac\uc5f0\ud569\ud68c \uc758 \uc8fc\ub3c4\ub85c \ubcc4\ub3c4\uc758 \ud589\uc0ac\uac00 \uc5f4\ub9ac\uba70 2011\ub144\uc758 \uacbd\uc6b0 \uacf5\uacfc\ub300, \uc0dd\uba85\ub300, \uc774\uacfc\ub300, \ubcf4\uacfc\ub300, \uc815\ud1b5\ub300\uc758 5\uac1c \ub2e8\uacfc\ub300\ud559\uc774 \ucc38\uc5ec\ud558\uc5ec \ub178\ubca8\uad11\uc7a5\uc744 \uc8fc \ubb34\ub300\ub85c \u2018Sound Festival\u2019(\uc0ac\uc6b4\ub4dc \ud398\uc2a4\ud2f0\ubc8c)\uacfc \u2018\uc560\uae30\ub2a5 \uc2e4\ub8e8\uc5e3 \uac00\uc694\uc81c\u2019, \u2018\uc560\uae30\ub2a5 \uc694\ub9ac \uacbd\uc5f0\ub300\ud68c\u2019 \ub4f1\uacfc \ub354\ubd88\uc5b4 \uac01\uc885 \uac00\uc218\ub4e4\uc758 \ucd95\ud558\uacf5\uc5f0\uc774 \ud3bc\uccd0\uc84c\ub2e4."} {"question": "\ubc15\uadfc\ud61c\uac00 2007\ub144 \ud55c\ub098\ub77c\ub2f9 \ub300\uc120 \uacbd\uc120\uc5d0\uc11c \ub0b4\uc138\uc6b4 \uc815\ucc45\uc740 \ubb34\uc5c7\uc778\uac00?", "paragraph": "\ubc15\uadfc\ud61c\ub294 2007\ub144 \ud55c\ub098\ub77c\ub2f9 \ub300\uc120 \uacbd\uc120\uc5d0 \ub098\uc640 '\uc904\ud478\uc138'\ub77c\ub294 \uc815\ucc45\uc744 \ub0b4\uc138\uc6e0\ub2e4. '\uc138\uae08\uc740 \uc904\uc774\uace0, \uac01\uc885 \uaddc\uc81c\ub294 \ud480\uace0, \ubc95\uae30\uac15\uc740 \uc138\uc6b4\ub2e4'\ub294 \ub73b\uc73c\ub85c \uadf8\uc758 \ub300\ud1b5\ub839\ud6c4\ubcf4 \uacbd\uc120\uc2dc \uc8fc\uc694 \uc815\ucc45\uad6c\ud638\uc600\ub2e4. \uc774 \uc815\ucc45\uc740 \uadf8\ub140\uc758 \ubcf4\uc218\uc801\uc778 \uc131\ud5a5\uc744 \ubcf4\uc5ec\uc8fc\ub294 \ub300\ud45c\uc801 \uc815\ucc45\uc774\ub77c\ub294 \ud3c9\uac00\uac00 \uc788\ub2e4. \uc790\uc720\uc8fc\uc758(liberalism)\uc801\uc774\ub77c\uace0\ub3c4 \ud560 \uc218 \uc788\uc9c0\ub9cc, \uc815\ud655\ud788\ub294 \uc2e0\ubd84\uc81c \ub4f1\uc73c\ub85c\ubd80\ud130 \uc5b5\uc555\ubc1b\ub294 \uac1c\uc778\uc758 \uc790\uc720\ub97c \uc639\ud638\ud558\ub358 '\uace0\uc804\uc801 \uc790\uc720\uc8fc\uc758\uc790(liberalist)'\ub77c\uae30\ubcf4\ub2e4\ub294, \ubd84\ubc30\ubcf4\ub2e4 \uc131\uc7a5\uc744 \uc911\uc2dc\ud558\uace0 \uac01\uc885 \uaddc\uc81c\ub85c\ubd80\ud130 \uc5b5\uc555\ubc1b\ub294 \uc790\ubcf8\uc758 \uc790\uc720\ub85c\uc6b4 \uc774\ub3d9\uc744 \uc639\ud638\ud55c\ub2e4\ub294 \uc810\uc5d0\uc11c '\uc2e0\uc790\uc720\uc8fc\uc758\uc790(neo-liberalist)'\uc5d0 \ub354 \uac00\uae5d\ub2e4\ub294 \ud3c9\uac00\uac00 \uc788\ub2e4. \uc774\ub294 \uc774\ud6c4 \uc81c17\ub300 \uc774\uba85\ubc15\uc815\ubd80\uc758 \uc815\ucc45\uc5d0\ub3c4 \ub9ce\uc774 \ubc18\uc601\ub418\uc5c8\ub2e4.", "answer": "\uc904\ud478\uc138", "sentence": "\ubc15\uadfc\ud61c\ub294 2007\ub144 \ud55c\ub098\ub77c\ub2f9 \ub300\uc120 \uacbd\uc120\uc5d0 \ub098\uc640 ' \uc904\ud478\uc138 '\ub77c\ub294", "paragraph_sentence": " \ubc15\uadfc\ud61c\ub294 2007\ub144 \ud55c\ub098\ub77c\ub2f9 \ub300\uc120 \uacbd\uc120\uc5d0 \ub098\uc640 ' \uc904\ud478\uc138 '\ub77c\ub294 \uc815\ucc45\uc744 \ub0b4\uc138\uc6e0\ub2e4. '\uc138\uae08\uc740 \uc904\uc774\uace0, \uac01\uc885 \uaddc\uc81c\ub294 \ud480\uace0, \ubc95\uae30\uac15\uc740 \uc138\uc6b4\ub2e4'\ub294 \ub73b\uc73c\ub85c \uadf8\uc758 \ub300\ud1b5\ub839\ud6c4\ubcf4 \uacbd\uc120\uc2dc \uc8fc\uc694 \uc815\ucc45\uad6c\ud638\uc600\ub2e4. \uc774 \uc815\ucc45\uc740 \uadf8\ub140\uc758 \ubcf4\uc218\uc801\uc778 \uc131\ud5a5\uc744 \ubcf4\uc5ec\uc8fc\ub294 \ub300\ud45c\uc801 \uc815\ucc45\uc774\ub77c\ub294 \ud3c9\uac00\uac00 \uc788\ub2e4. \uc790\uc720\uc8fc\uc758(liberalism)\uc801\uc774\ub77c\uace0\ub3c4 \ud560 \uc218 \uc788\uc9c0\ub9cc, \uc815\ud655\ud788\ub294 \uc2e0\ubd84\uc81c \ub4f1\uc73c\ub85c\ubd80\ud130 \uc5b5\uc555\ubc1b\ub294 \uac1c\uc778\uc758 \uc790\uc720\ub97c \uc639\ud638\ud558\ub358 '\uace0\uc804\uc801 \uc790\uc720\uc8fc\uc758\uc790(liberalist)'\ub77c\uae30\ubcf4\ub2e4\ub294, \ubd84\ubc30\ubcf4\ub2e4 \uc131\uc7a5\uc744 \uc911\uc2dc\ud558\uace0 \uac01\uc885 \uaddc\uc81c\ub85c\ubd80\ud130 \uc5b5\uc555\ubc1b\ub294 \uc790\ubcf8\uc758 \uc790\uc720\ub85c\uc6b4 \uc774\ub3d9\uc744 \uc639\ud638\ud55c\ub2e4\ub294 \uc810\uc5d0\uc11c '\uc2e0\uc790\uc720\uc8fc\uc758\uc790(neo-liberalist)'\uc5d0 \ub354 \uac00\uae5d\ub2e4\ub294 \ud3c9\uac00\uac00 \uc788\ub2e4. \uc774\ub294 \uc774\ud6c4 \uc81c17\ub300 \uc774\uba85\ubc15\uc815\ubd80\uc758 \uc815\ucc45\uc5d0\ub3c4 \ub9ce\uc774 \ubc18\uc601\ub418\uc5c8\ub2e4.", "paragraph_answer": "\ubc15\uadfc\ud61c\ub294 2007\ub144 \ud55c\ub098\ub77c\ub2f9 \ub300\uc120 \uacbd\uc120\uc5d0 \ub098\uc640 ' \uc904\ud478\uc138 '\ub77c\ub294 \uc815\ucc45\uc744 \ub0b4\uc138\uc6e0\ub2e4. '\uc138\uae08\uc740 \uc904\uc774\uace0, \uac01\uc885 \uaddc\uc81c\ub294 \ud480\uace0, \ubc95\uae30\uac15\uc740 \uc138\uc6b4\ub2e4'\ub294 \ub73b\uc73c\ub85c \uadf8\uc758 \ub300\ud1b5\ub839\ud6c4\ubcf4 \uacbd\uc120\uc2dc \uc8fc\uc694 \uc815\ucc45\uad6c\ud638\uc600\ub2e4. \uc774 \uc815\ucc45\uc740 \uadf8\ub140\uc758 \ubcf4\uc218\uc801\uc778 \uc131\ud5a5\uc744 \ubcf4\uc5ec\uc8fc\ub294 \ub300\ud45c\uc801 \uc815\ucc45\uc774\ub77c\ub294 \ud3c9\uac00\uac00 \uc788\ub2e4. \uc790\uc720\uc8fc\uc758(liberalism)\uc801\uc774\ub77c\uace0\ub3c4 \ud560 \uc218 \uc788\uc9c0\ub9cc, \uc815\ud655\ud788\ub294 \uc2e0\ubd84\uc81c \ub4f1\uc73c\ub85c\ubd80\ud130 \uc5b5\uc555\ubc1b\ub294 \uac1c\uc778\uc758 \uc790\uc720\ub97c \uc639\ud638\ud558\ub358 '\uace0\uc804\uc801 \uc790\uc720\uc8fc\uc758\uc790(liberalist)'\ub77c\uae30\ubcf4\ub2e4\ub294, \ubd84\ubc30\ubcf4\ub2e4 \uc131\uc7a5\uc744 \uc911\uc2dc\ud558\uace0 \uac01\uc885 \uaddc\uc81c\ub85c\ubd80\ud130 \uc5b5\uc555\ubc1b\ub294 \uc790\ubcf8\uc758 \uc790\uc720\ub85c\uc6b4 \uc774\ub3d9\uc744 \uc639\ud638\ud55c\ub2e4\ub294 \uc810\uc5d0\uc11c '\uc2e0\uc790\uc720\uc8fc\uc758\uc790(neo-liberalist)'\uc5d0 \ub354 \uac00\uae5d\ub2e4\ub294 \ud3c9\uac00\uac00 \uc788\ub2e4. \uc774\ub294 \uc774\ud6c4 \uc81c17\ub300 \uc774\uba85\ubc15\uc815\ubd80\uc758 \uc815\ucc45\uc5d0\ub3c4 \ub9ce\uc774 \ubc18\uc601\ub418\uc5c8\ub2e4.", "sentence_answer": "\ubc15\uadfc\ud61c\ub294 2007\ub144 \ud55c\ub098\ub77c\ub2f9 \ub300\uc120 \uacbd\uc120\uc5d0 \ub098\uc640 ' \uc904\ud478\uc138 '\ub77c\ub294", "paragraph_id": "6581158-4-0", "paragraph_question": "question: \ubc15\uadfc\ud61c\uac00 2007\ub144 \ud55c\ub098\ub77c\ub2f9 \ub300\uc120 \uacbd\uc120\uc5d0\uc11c \ub0b4\uc138\uc6b4 \uc815\ucc45\uc740 \ubb34\uc5c7\uc778\uac00?, context: \ubc15\uadfc\ud61c\ub294 2007\ub144 \ud55c\ub098\ub77c\ub2f9 \ub300\uc120 \uacbd\uc120\uc5d0 \ub098\uc640 '\uc904\ud478\uc138'\ub77c\ub294 \uc815\ucc45\uc744 \ub0b4\uc138\uc6e0\ub2e4. '\uc138\uae08\uc740 \uc904\uc774\uace0, \uac01\uc885 \uaddc\uc81c\ub294 \ud480\uace0, \ubc95\uae30\uac15\uc740 \uc138\uc6b4\ub2e4'\ub294 \ub73b\uc73c\ub85c \uadf8\uc758 \ub300\ud1b5\ub839\ud6c4\ubcf4 \uacbd\uc120\uc2dc \uc8fc\uc694 \uc815\ucc45\uad6c\ud638\uc600\ub2e4. \uc774 \uc815\ucc45\uc740 \uadf8\ub140\uc758 \ubcf4\uc218\uc801\uc778 \uc131\ud5a5\uc744 \ubcf4\uc5ec\uc8fc\ub294 \ub300\ud45c\uc801 \uc815\ucc45\uc774\ub77c\ub294 \ud3c9\uac00\uac00 \uc788\ub2e4. \uc790\uc720\uc8fc\uc758(liberalism)\uc801\uc774\ub77c\uace0\ub3c4 \ud560 \uc218 \uc788\uc9c0\ub9cc, \uc815\ud655\ud788\ub294 \uc2e0\ubd84\uc81c \ub4f1\uc73c\ub85c\ubd80\ud130 \uc5b5\uc555\ubc1b\ub294 \uac1c\uc778\uc758 \uc790\uc720\ub97c \uc639\ud638\ud558\ub358 '\uace0\uc804\uc801 \uc790\uc720\uc8fc\uc758\uc790(liberalist)'\ub77c\uae30\ubcf4\ub2e4\ub294, \ubd84\ubc30\ubcf4\ub2e4 \uc131\uc7a5\uc744 \uc911\uc2dc\ud558\uace0 \uac01\uc885 \uaddc\uc81c\ub85c\ubd80\ud130 \uc5b5\uc555\ubc1b\ub294 \uc790\ubcf8\uc758 \uc790\uc720\ub85c\uc6b4 \uc774\ub3d9\uc744 \uc639\ud638\ud55c\ub2e4\ub294 \uc810\uc5d0\uc11c '\uc2e0\uc790\uc720\uc8fc\uc758\uc790(neo-liberalist)'\uc5d0 \ub354 \uac00\uae5d\ub2e4\ub294 \ud3c9\uac00\uac00 \uc788\ub2e4. \uc774\ub294 \uc774\ud6c4 \uc81c17\ub300 \uc774\uba85\ubc15\uc815\ubd80\uc758 \uc815\ucc45\uc5d0\ub3c4 \ub9ce\uc774 \ubc18\uc601\ub418\uc5c8\ub2e4."} {"question": "\uace0\ud2b8\uc170\ud2b8\uc758 \uc2dc\ub3c4\ub97c \uc878\ub82c\ud55c \uc2dc\ub3c4\ub77c\uba70 \ube44\ub09c\ud55c \uc0ac\ub78c\uc740?", "paragraph": "\ub808\uc2f1\uc740 \uace0\ud2b8\uc170\ud2b8\uac00 \ub3c5\uc77c\uc758 \uc0ac\uace0\ubc29\uc2dd\uc744 \uc804\ud600 \uace0\ub824\ud558\uc9c0 \uc54a\uace0 \ubb34\ub300\uc608\uc220\uc778 \ub4dc\ub77c\ub9c8\ub97c \ud504\ub791\uc2a4\uc801\uc778 \ud2c0\uc5d0 \uac15\uc81c\ub85c \ub9de\ucd94\ub824\ud588\ub2e4\uace0 \ud558\uba74\uc11c, \uadf8\uac83\uc740 \uc878\ub82c\ud55c \uc2dc\ub3c4\uc5d0 \ubd88\uacfc\ud558\ub2e4\uace0 \ube44\ub09c\ud588\ub2e4. \ub808\uc2f1\uc740 \ud504\ub791\uc2a4\uc758 \uacc4\ubabd\uc8fc\uc758\uc790 \ub514\ub4dc\ub85c\uc640 \ub354\ubd88\uc5b4 \ud601\uba85\uc801\uc778 \uc2dc\ubbfc\ubbf8\ud559\uc758 \ub300\ud45c\uc790\ub85c\uc11c \uadf9\uc7a5\ubb34\ub300\ub97c \u201c\uc720\uc6a9\uacfc \uad50\uc220 (prodesse et docere)\u201d\uc758 \uc7a5\uc774\ub77c \uaddc\uc815\ud558\uba74\uc11c \uacc4\ubabd\uc8fc\uc758 \ud2b9\uc720\uc758 \uad50\uc220\uc801 \ud6a8\uacfc\ub97c \uac15\uc870\ud588\ub2e4. \uace0\ud2b8\uc170\ud2b8\uac00 \uad50\uc220\ubaa9\uc801\uc744 \ubb38\ud559\uc758 \ubcf8\uc9c8\ub85c \uac15\uc870\ud558\ub294 \uac83\uc774 \ud604\ud559\uc801\uc778\ub370 \ube44\ud574 \ub808\uc2f1\uc740 \uad50\uc220\uc801 \ud6a8\uacfc\ub97c \uc9c0\ub098\uce58\uac8c \uc758\uc2dd\ud558\uc9c0 \uc54a\uace0 \ubb38\ud559\uc758 \uc790\uc5f0\uc2a4\ub7ec\uc6b4 \ubc1c\uc0dd\ud604\uc0c1\uc73c\ub85c \uc774\ud574\ud588\ub2e4. \ub808\uc2f1\uc774 \uc5f0\uadf9\uc758 \uc624\ub77d\uc131\uc744 \uad50\uc220\uc131\uacfc \ub3d9\uc77c\uc120\uc0c1\uc5d0\uc11c \ud574\uc11d\ud558\uace0 \uc788\ub294 \uac83\uc740 \uc5f0\uadf9\uc744 \uc0ac\ud68c\ud604\uc0c1\uc73c\ub85c \uac04\uc8fc\ud558\uae30 \ub54c\ubb38\uc774\ub2e4. \uc989, \uc5f0\uadf9\uc744 \ud1b5\ud574 \uad00\uc911\uc758 \uc778\uac04\uc131\uc744 \ubc1c\uc804\uc2dc\ud0b4\uc73c\ub85c\uc368 \uad50\uc721\uc801\uc778 \uae30\ub2a5\uc744 \uc218\ud589\ud55c\ub2e4\uace0 \ubcf8 \uac83\uc774\ub2e4.", "answer": "\ub808\uc2f1", "sentence": "\ub808\uc2f1 \uc740 \uace0\ud2b8\uc170\ud2b8\uac00 \ub3c5\uc77c\uc758 \uc0ac\uace0\ubc29\uc2dd\uc744 \uc804\ud600 \uace0\ub824\ud558\uc9c0 \uc54a\uace0 \ubb34\ub300\uc608\uc220\uc778 \ub4dc\ub77c\ub9c8\ub97c \ud504\ub791\uc2a4\uc801\uc778 \ud2c0\uc5d0 \uac15\uc81c\ub85c \ub9de\ucd94\ub824\ud588\ub2e4\uace0 \ud558\uba74\uc11c, \uadf8\uac83\uc740 \uc878\ub82c\ud55c \uc2dc\ub3c4\uc5d0 \ubd88\uacfc\ud558\ub2e4\uace0 \ube44\ub09c\ud588\ub2e4.", "paragraph_sentence": " \ub808\uc2f1 \uc740 \uace0\ud2b8\uc170\ud2b8\uac00 \ub3c5\uc77c\uc758 \uc0ac\uace0\ubc29\uc2dd\uc744 \uc804\ud600 \uace0\ub824\ud558\uc9c0 \uc54a\uace0 \ubb34\ub300\uc608\uc220\uc778 \ub4dc\ub77c\ub9c8\ub97c \ud504\ub791\uc2a4\uc801\uc778 \ud2c0\uc5d0 \uac15\uc81c\ub85c \ub9de\ucd94\ub824\ud588\ub2e4\uace0 \ud558\uba74\uc11c, \uadf8\uac83\uc740 \uc878\ub82c\ud55c \uc2dc\ub3c4\uc5d0 \ubd88\uacfc\ud558\ub2e4\uace0 \ube44\ub09c\ud588\ub2e4. \ub808\uc2f1\uc740 \ud504\ub791\uc2a4\uc758 \uacc4\ubabd\uc8fc\uc758\uc790 \ub514\ub4dc\ub85c\uc640 \ub354\ubd88\uc5b4 \ud601\uba85\uc801\uc778 \uc2dc\ubbfc\ubbf8\ud559\uc758 \ub300\ud45c\uc790\ub85c\uc11c \uadf9\uc7a5\ubb34\ub300\ub97c \u201c\uc720\uc6a9\uacfc \uad50\uc220 (prodesse et docere)\u201d\uc758 \uc7a5\uc774\ub77c \uaddc\uc815\ud558\uba74\uc11c \uacc4\ubabd\uc8fc\uc758 \ud2b9\uc720\uc758 \uad50\uc220\uc801 \ud6a8\uacfc\ub97c \uac15\uc870\ud588\ub2e4. \uace0\ud2b8\uc170\ud2b8\uac00 \uad50\uc220\ubaa9\uc801\uc744 \ubb38\ud559\uc758 \ubcf8\uc9c8\ub85c \uac15\uc870\ud558\ub294 \uac83\uc774 \ud604\ud559\uc801\uc778\ub370 \ube44\ud574 \ub808\uc2f1\uc740 \uad50\uc220\uc801 \ud6a8\uacfc\ub97c \uc9c0\ub098\uce58\uac8c \uc758\uc2dd\ud558\uc9c0 \uc54a\uace0 \ubb38\ud559\uc758 \uc790\uc5f0\uc2a4\ub7ec\uc6b4 \ubc1c\uc0dd\ud604\uc0c1\uc73c\ub85c \uc774\ud574\ud588\ub2e4. \ub808\uc2f1\uc774 \uc5f0\uadf9\uc758 \uc624\ub77d\uc131\uc744 \uad50\uc220\uc131\uacfc \ub3d9\uc77c\uc120\uc0c1\uc5d0\uc11c \ud574\uc11d\ud558\uace0 \uc788\ub294 \uac83\uc740 \uc5f0\uadf9\uc744 \uc0ac\ud68c\ud604\uc0c1\uc73c\ub85c \uac04\uc8fc\ud558\uae30 \ub54c\ubb38\uc774\ub2e4. \uc989, \uc5f0\uadf9\uc744 \ud1b5\ud574 \uad00\uc911\uc758 \uc778\uac04\uc131\uc744 \ubc1c\uc804\uc2dc\ud0b4\uc73c\ub85c\uc368 \uad50\uc721\uc801\uc778 \uae30\ub2a5\uc744 \uc218\ud589\ud55c\ub2e4\uace0 \ubcf8 \uac83\uc774\ub2e4.", "paragraph_answer": " \ub808\uc2f1 \uc740 \uace0\ud2b8\uc170\ud2b8\uac00 \ub3c5\uc77c\uc758 \uc0ac\uace0\ubc29\uc2dd\uc744 \uc804\ud600 \uace0\ub824\ud558\uc9c0 \uc54a\uace0 \ubb34\ub300\uc608\uc220\uc778 \ub4dc\ub77c\ub9c8\ub97c \ud504\ub791\uc2a4\uc801\uc778 \ud2c0\uc5d0 \uac15\uc81c\ub85c \ub9de\ucd94\ub824\ud588\ub2e4\uace0 \ud558\uba74\uc11c, \uadf8\uac83\uc740 \uc878\ub82c\ud55c \uc2dc\ub3c4\uc5d0 \ubd88\uacfc\ud558\ub2e4\uace0 \ube44\ub09c\ud588\ub2e4. \ub808\uc2f1\uc740 \ud504\ub791\uc2a4\uc758 \uacc4\ubabd\uc8fc\uc758\uc790 \ub514\ub4dc\ub85c\uc640 \ub354\ubd88\uc5b4 \ud601\uba85\uc801\uc778 \uc2dc\ubbfc\ubbf8\ud559\uc758 \ub300\ud45c\uc790\ub85c\uc11c \uadf9\uc7a5\ubb34\ub300\ub97c \u201c\uc720\uc6a9\uacfc \uad50\uc220 (prodesse et docere)\u201d\uc758 \uc7a5\uc774\ub77c \uaddc\uc815\ud558\uba74\uc11c \uacc4\ubabd\uc8fc\uc758 \ud2b9\uc720\uc758 \uad50\uc220\uc801 \ud6a8\uacfc\ub97c \uac15\uc870\ud588\ub2e4. \uace0\ud2b8\uc170\ud2b8\uac00 \uad50\uc220\ubaa9\uc801\uc744 \ubb38\ud559\uc758 \ubcf8\uc9c8\ub85c \uac15\uc870\ud558\ub294 \uac83\uc774 \ud604\ud559\uc801\uc778\ub370 \ube44\ud574 \ub808\uc2f1\uc740 \uad50\uc220\uc801 \ud6a8\uacfc\ub97c \uc9c0\ub098\uce58\uac8c \uc758\uc2dd\ud558\uc9c0 \uc54a\uace0 \ubb38\ud559\uc758 \uc790\uc5f0\uc2a4\ub7ec\uc6b4 \ubc1c\uc0dd\ud604\uc0c1\uc73c\ub85c \uc774\ud574\ud588\ub2e4. \ub808\uc2f1\uc774 \uc5f0\uadf9\uc758 \uc624\ub77d\uc131\uc744 \uad50\uc220\uc131\uacfc \ub3d9\uc77c\uc120\uc0c1\uc5d0\uc11c \ud574\uc11d\ud558\uace0 \uc788\ub294 \uac83\uc740 \uc5f0\uadf9\uc744 \uc0ac\ud68c\ud604\uc0c1\uc73c\ub85c \uac04\uc8fc\ud558\uae30 \ub54c\ubb38\uc774\ub2e4. \uc989, \uc5f0\uadf9\uc744 \ud1b5\ud574 \uad00\uc911\uc758 \uc778\uac04\uc131\uc744 \ubc1c\uc804\uc2dc\ud0b4\uc73c\ub85c\uc368 \uad50\uc721\uc801\uc778 \uae30\ub2a5\uc744 \uc218\ud589\ud55c\ub2e4\uace0 \ubcf8 \uac83\uc774\ub2e4.", "sentence_answer": " \ub808\uc2f1 \uc740 \uace0\ud2b8\uc170\ud2b8\uac00 \ub3c5\uc77c\uc758 \uc0ac\uace0\ubc29\uc2dd\uc744 \uc804\ud600 \uace0\ub824\ud558\uc9c0 \uc54a\uace0 \ubb34\ub300\uc608\uc220\uc778 \ub4dc\ub77c\ub9c8\ub97c \ud504\ub791\uc2a4\uc801\uc778 \ud2c0\uc5d0 \uac15\uc81c\ub85c \ub9de\ucd94\ub824\ud588\ub2e4\uace0 \ud558\uba74\uc11c, \uadf8\uac83\uc740 \uc878\ub82c\ud55c \uc2dc\ub3c4\uc5d0 \ubd88\uacfc\ud558\ub2e4\uace0 \ube44\ub09c\ud588\ub2e4.", "paragraph_id": "6464112-2-0", "paragraph_question": "question: \uace0\ud2b8\uc170\ud2b8\uc758 \uc2dc\ub3c4\ub97c \uc878\ub82c\ud55c \uc2dc\ub3c4\ub77c\uba70 \ube44\ub09c\ud55c \uc0ac\ub78c\uc740?, context: \ub808\uc2f1\uc740 \uace0\ud2b8\uc170\ud2b8\uac00 \ub3c5\uc77c\uc758 \uc0ac\uace0\ubc29\uc2dd\uc744 \uc804\ud600 \uace0\ub824\ud558\uc9c0 \uc54a\uace0 \ubb34\ub300\uc608\uc220\uc778 \ub4dc\ub77c\ub9c8\ub97c \ud504\ub791\uc2a4\uc801\uc778 \ud2c0\uc5d0 \uac15\uc81c\ub85c \ub9de\ucd94\ub824\ud588\ub2e4\uace0 \ud558\uba74\uc11c, \uadf8\uac83\uc740 \uc878\ub82c\ud55c \uc2dc\ub3c4\uc5d0 \ubd88\uacfc\ud558\ub2e4\uace0 \ube44\ub09c\ud588\ub2e4. \ub808\uc2f1\uc740 \ud504\ub791\uc2a4\uc758 \uacc4\ubabd\uc8fc\uc758\uc790 \ub514\ub4dc\ub85c\uc640 \ub354\ubd88\uc5b4 \ud601\uba85\uc801\uc778 \uc2dc\ubbfc\ubbf8\ud559\uc758 \ub300\ud45c\uc790\ub85c\uc11c \uadf9\uc7a5\ubb34\ub300\ub97c \u201c\uc720\uc6a9\uacfc \uad50\uc220 (prodesse et docere)\u201d\uc758 \uc7a5\uc774\ub77c \uaddc\uc815\ud558\uba74\uc11c \uacc4\ubabd\uc8fc\uc758 \ud2b9\uc720\uc758 \uad50\uc220\uc801 \ud6a8\uacfc\ub97c \uac15\uc870\ud588\ub2e4. \uace0\ud2b8\uc170\ud2b8\uac00 \uad50\uc220\ubaa9\uc801\uc744 \ubb38\ud559\uc758 \ubcf8\uc9c8\ub85c \uac15\uc870\ud558\ub294 \uac83\uc774 \ud604\ud559\uc801\uc778\ub370 \ube44\ud574 \ub808\uc2f1\uc740 \uad50\uc220\uc801 \ud6a8\uacfc\ub97c \uc9c0\ub098\uce58\uac8c \uc758\uc2dd\ud558\uc9c0 \uc54a\uace0 \ubb38\ud559\uc758 \uc790\uc5f0\uc2a4\ub7ec\uc6b4 \ubc1c\uc0dd\ud604\uc0c1\uc73c\ub85c \uc774\ud574\ud588\ub2e4. \ub808\uc2f1\uc774 \uc5f0\uadf9\uc758 \uc624\ub77d\uc131\uc744 \uad50\uc220\uc131\uacfc \ub3d9\uc77c\uc120\uc0c1\uc5d0\uc11c \ud574\uc11d\ud558\uace0 \uc788\ub294 \uac83\uc740 \uc5f0\uadf9\uc744 \uc0ac\ud68c\ud604\uc0c1\uc73c\ub85c \uac04\uc8fc\ud558\uae30 \ub54c\ubb38\uc774\ub2e4. \uc989, \uc5f0\uadf9\uc744 \ud1b5\ud574 \uad00\uc911\uc758 \uc778\uac04\uc131\uc744 \ubc1c\uc804\uc2dc\ud0b4\uc73c\ub85c\uc368 \uad50\uc721\uc801\uc778 \uae30\ub2a5\uc744 \uc218\ud589\ud55c\ub2e4\uace0 \ubcf8 \uac83\uc774\ub2e4."} {"question": "\ub2c8\uc544\ud06c\uac00 \uc0ac\ub9dd\ud560 \ub54c \ub098\uc774\ub294?", "paragraph": "\ud574\ub2ec\uc740 \uc778\uac04\uc5d0\uac8c \uc7a1\ud600 \uc9c0\ub0b4\uba74\uc11c\ub3c4 \uc798 \uc0b4 \uc218 \uc788\uc73c\uba70, \ud604\uc7ac 40\uac1c\uc18c \uc774\uc0c1\uc758 \uc218\uc871\uad00\uacfc \ub3d9\ubb3c\uc6d0\uc5d0\uc11c \ud574\ub2ec\uc744 \uc0ac\uc721\ud558\uace0 \uc788\ub2e4. \uc2dc\uc560\ud2c0 \uc218\uc871\uad00\uc740 1979\ub144\uc5d0 \uac1c\uccb4 \ud2f0\ucd94\ud06c(Tichuk)\uac00 \ud0dc\uc5b4\ub098\uace0, 1980\ub144\ub300\uc5d0 \uc0c8\ub07c\uac00 \uc138 \ub9c8\ub9ac \ub354 \ud0dc\uc5b4\ub0a8\uc73c\ub85c\uc368, \ud574\ub2ec\uc744 \uc218\uc815\ub780\ubd80\ud130 \uc131\uccb4\uae4c\uc9c0 \uc644\uc804\ud788 \uc0ac\uc721\ud55c \ucd5c\ucd08\uc758 \uae30\uad00\uc774 \ub418\uc5c8\ub2e4. 2007\ub144\uc5d0\ub294 \ud574\ub2ec \ub450 \ub9c8\ub9ac\uac00 \uc11c\ub85c \uc190\uc744 \uc7a1\uace0 \ubb3c\uc5d0 \ub5a0\ub2e4\ub2c8\ub294 \uc601\uc0c1\uc774 \uc720\ud29c\ube0c\uc5d0 \uc5c5\ub85c\ub4dc\ub418\uc5b4 2\uc8fc\ub9cc\uc5d0 1\ubc31 5\uc2ed\ub9cc \ud68c\uc758 \uc870\ud68c\uc218\ub97c \ud655\ubcf4\ud588\uace0, 2013\ub144 \ud604\uc7ac\ub294 1\ucc9c 9\ubc31\ub9cc \ud68c\ub97c \ucd08\uacfc\ud558\uace0 \uc788\ub2e4. \ud574\ub2f9 \ub3d9\uc601\uc0c1\uc740 \ubca4\ucfe0\ubc84 \uc218\uc871\uad00\uc5d0\uc11c 5\ub144 \uc55e\uc11c \ucd2c\uc601\ub41c \uac83\uc73c\ub85c, \uc5c5\ub85c\ub4dc\ub418\uc5c8\uc744 \ub2f9\uc2dc \uc720\ud29c\ube0c\uc5d0\uc11c \uac00\uc7a5 \uc778\uae30\uc788\ub294 \ub3d9\ubb3c \uad00\ub828 \ub3d9\uc601\uc0c1\uc774\uc5c8\ub2e4. \ub3d9\uc601\uc0c1\uc5d0\uc11c \ud138 \uc0c9\uae54\uc774 \ubc1d\uc740 \ud574\ub2ec\uc758 \uc774\ub984\uc740 \ub2c8\uc544\ud06c(Nyac)\uc774\uba70, 1989\ub144 \uc5d1\uc190 \ubc1c\ub370\uc2a4 \uae30\ub984 \ub204\ucd9c \uc0ac\uac74 \ub54c \uad6c\uc870\ub41c \ud574\ub2ec\uc774\ub2e4. \ub2c8\uc544\ud06c\ub294 2008\ub144 9\uc6d4\uc5d0 20\uc0b4\ub85c \uc8fd\uc5c8\ub2e4.", "answer": "20\uc0b4", "sentence": "\ub2c8\uc544\ud06c\ub294 2008\ub144 9\uc6d4\uc5d0 20\uc0b4 \ub85c \uc8fd\uc5c8\ub2e4.", "paragraph_sentence": "\ud574\ub2ec\uc740 \uc778\uac04\uc5d0\uac8c \uc7a1\ud600 \uc9c0\ub0b4\uba74\uc11c\ub3c4 \uc798 \uc0b4 \uc218 \uc788\uc73c\uba70, \ud604\uc7ac 40\uac1c\uc18c \uc774\uc0c1\uc758 \uc218\uc871\uad00\uacfc \ub3d9\ubb3c\uc6d0\uc5d0\uc11c \ud574\ub2ec\uc744 \uc0ac\uc721\ud558\uace0 \uc788\ub2e4. \uc2dc\uc560\ud2c0 \uc218\uc871\uad00\uc740 1979\ub144\uc5d0 \uac1c\uccb4 \ud2f0\ucd94\ud06c(Tichuk)\uac00 \ud0dc\uc5b4\ub098\uace0, 1980\ub144\ub300\uc5d0 \uc0c8\ub07c\uac00 \uc138 \ub9c8\ub9ac \ub354 \ud0dc\uc5b4\ub0a8\uc73c\ub85c\uc368, \ud574\ub2ec\uc744 \uc218\uc815\ub780\ubd80\ud130 \uc131\uccb4\uae4c\uc9c0 \uc644\uc804\ud788 \uc0ac\uc721\ud55c \ucd5c\ucd08\uc758 \uae30\uad00\uc774 \ub418\uc5c8\ub2e4. 2007\ub144\uc5d0\ub294 \ud574\ub2ec \ub450 \ub9c8\ub9ac\uac00 \uc11c\ub85c \uc190\uc744 \uc7a1\uace0 \ubb3c\uc5d0 \ub5a0\ub2e4\ub2c8\ub294 \uc601\uc0c1\uc774 \uc720\ud29c\ube0c\uc5d0 \uc5c5\ub85c\ub4dc\ub418\uc5b4 2\uc8fc\ub9cc\uc5d0 1\ubc31 5\uc2ed\ub9cc \ud68c\uc758 \uc870\ud68c\uc218\ub97c \ud655\ubcf4\ud588\uace0, 2013\ub144 \ud604\uc7ac\ub294 1\ucc9c 9\ubc31\ub9cc \ud68c\ub97c \ucd08\uacfc\ud558\uace0 \uc788\ub2e4. \ud574\ub2f9 \ub3d9\uc601\uc0c1\uc740 \ubca4\ucfe0\ubc84 \uc218\uc871\uad00\uc5d0\uc11c 5\ub144 \uc55e\uc11c \ucd2c\uc601\ub41c \uac83\uc73c\ub85c, \uc5c5\ub85c\ub4dc\ub418\uc5c8\uc744 \ub2f9\uc2dc \uc720\ud29c\ube0c\uc5d0\uc11c \uac00\uc7a5 \uc778\uae30\uc788\ub294 \ub3d9\ubb3c \uad00\ub828 \ub3d9\uc601\uc0c1\uc774\uc5c8\ub2e4. \ub3d9\uc601\uc0c1\uc5d0\uc11c \ud138 \uc0c9\uae54\uc774 \ubc1d\uc740 \ud574\ub2ec\uc758 \uc774\ub984\uc740 \ub2c8\uc544\ud06c(Nyac)\uc774\uba70, 1989\ub144 \uc5d1\uc190 \ubc1c\ub370\uc2a4 \uae30\ub984 \ub204\ucd9c \uc0ac\uac74 \ub54c \uad6c\uc870\ub41c \ud574\ub2ec\uc774\ub2e4. \ub2c8\uc544\ud06c\ub294 2008\ub144 9\uc6d4\uc5d0 20\uc0b4 \ub85c \uc8fd\uc5c8\ub2e4. ", "paragraph_answer": "\ud574\ub2ec\uc740 \uc778\uac04\uc5d0\uac8c \uc7a1\ud600 \uc9c0\ub0b4\uba74\uc11c\ub3c4 \uc798 \uc0b4 \uc218 \uc788\uc73c\uba70, \ud604\uc7ac 40\uac1c\uc18c \uc774\uc0c1\uc758 \uc218\uc871\uad00\uacfc \ub3d9\ubb3c\uc6d0\uc5d0\uc11c \ud574\ub2ec\uc744 \uc0ac\uc721\ud558\uace0 \uc788\ub2e4. \uc2dc\uc560\ud2c0 \uc218\uc871\uad00\uc740 1979\ub144\uc5d0 \uac1c\uccb4 \ud2f0\ucd94\ud06c(Tichuk)\uac00 \ud0dc\uc5b4\ub098\uace0, 1980\ub144\ub300\uc5d0 \uc0c8\ub07c\uac00 \uc138 \ub9c8\ub9ac \ub354 \ud0dc\uc5b4\ub0a8\uc73c\ub85c\uc368, \ud574\ub2ec\uc744 \uc218\uc815\ub780\ubd80\ud130 \uc131\uccb4\uae4c\uc9c0 \uc644\uc804\ud788 \uc0ac\uc721\ud55c \ucd5c\ucd08\uc758 \uae30\uad00\uc774 \ub418\uc5c8\ub2e4. 2007\ub144\uc5d0\ub294 \ud574\ub2ec \ub450 \ub9c8\ub9ac\uac00 \uc11c\ub85c \uc190\uc744 \uc7a1\uace0 \ubb3c\uc5d0 \ub5a0\ub2e4\ub2c8\ub294 \uc601\uc0c1\uc774 \uc720\ud29c\ube0c\uc5d0 \uc5c5\ub85c\ub4dc\ub418\uc5b4 2\uc8fc\ub9cc\uc5d0 1\ubc31 5\uc2ed\ub9cc \ud68c\uc758 \uc870\ud68c\uc218\ub97c \ud655\ubcf4\ud588\uace0, 2013\ub144 \ud604\uc7ac\ub294 1\ucc9c 9\ubc31\ub9cc \ud68c\ub97c \ucd08\uacfc\ud558\uace0 \uc788\ub2e4. \ud574\ub2f9 \ub3d9\uc601\uc0c1\uc740 \ubca4\ucfe0\ubc84 \uc218\uc871\uad00\uc5d0\uc11c 5\ub144 \uc55e\uc11c \ucd2c\uc601\ub41c \uac83\uc73c\ub85c, \uc5c5\ub85c\ub4dc\ub418\uc5c8\uc744 \ub2f9\uc2dc \uc720\ud29c\ube0c\uc5d0\uc11c \uac00\uc7a5 \uc778\uae30\uc788\ub294 \ub3d9\ubb3c \uad00\ub828 \ub3d9\uc601\uc0c1\uc774\uc5c8\ub2e4. \ub3d9\uc601\uc0c1\uc5d0\uc11c \ud138 \uc0c9\uae54\uc774 \ubc1d\uc740 \ud574\ub2ec\uc758 \uc774\ub984\uc740 \ub2c8\uc544\ud06c(Nyac)\uc774\uba70, 1989\ub144 \uc5d1\uc190 \ubc1c\ub370\uc2a4 \uae30\ub984 \ub204\ucd9c \uc0ac\uac74 \ub54c \uad6c\uc870\ub41c \ud574\ub2ec\uc774\ub2e4. \ub2c8\uc544\ud06c\ub294 2008\ub144 9\uc6d4\uc5d0 20\uc0b4 \ub85c \uc8fd\uc5c8\ub2e4.", "sentence_answer": "\ub2c8\uc544\ud06c\ub294 2008\ub144 9\uc6d4\uc5d0 20\uc0b4 \ub85c \uc8fd\uc5c8\ub2e4.", "paragraph_id": "6570452-24-1", "paragraph_question": "question: \ub2c8\uc544\ud06c\uac00 \uc0ac\ub9dd\ud560 \ub54c \ub098\uc774\ub294?, context: \ud574\ub2ec\uc740 \uc778\uac04\uc5d0\uac8c \uc7a1\ud600 \uc9c0\ub0b4\uba74\uc11c\ub3c4 \uc798 \uc0b4 \uc218 \uc788\uc73c\uba70, \ud604\uc7ac 40\uac1c\uc18c \uc774\uc0c1\uc758 \uc218\uc871\uad00\uacfc \ub3d9\ubb3c\uc6d0\uc5d0\uc11c \ud574\ub2ec\uc744 \uc0ac\uc721\ud558\uace0 \uc788\ub2e4. \uc2dc\uc560\ud2c0 \uc218\uc871\uad00\uc740 1979\ub144\uc5d0 \uac1c\uccb4 \ud2f0\ucd94\ud06c(Tichuk)\uac00 \ud0dc\uc5b4\ub098\uace0, 1980\ub144\ub300\uc5d0 \uc0c8\ub07c\uac00 \uc138 \ub9c8\ub9ac \ub354 \ud0dc\uc5b4\ub0a8\uc73c\ub85c\uc368, \ud574\ub2ec\uc744 \uc218\uc815\ub780\ubd80\ud130 \uc131\uccb4\uae4c\uc9c0 \uc644\uc804\ud788 \uc0ac\uc721\ud55c \ucd5c\ucd08\uc758 \uae30\uad00\uc774 \ub418\uc5c8\ub2e4. 2007\ub144\uc5d0\ub294 \ud574\ub2ec \ub450 \ub9c8\ub9ac\uac00 \uc11c\ub85c \uc190\uc744 \uc7a1\uace0 \ubb3c\uc5d0 \ub5a0\ub2e4\ub2c8\ub294 \uc601\uc0c1\uc774 \uc720\ud29c\ube0c\uc5d0 \uc5c5\ub85c\ub4dc\ub418\uc5b4 2\uc8fc\ub9cc\uc5d0 1\ubc31 5\uc2ed\ub9cc \ud68c\uc758 \uc870\ud68c\uc218\ub97c \ud655\ubcf4\ud588\uace0, 2013\ub144 \ud604\uc7ac\ub294 1\ucc9c 9\ubc31\ub9cc \ud68c\ub97c \ucd08\uacfc\ud558\uace0 \uc788\ub2e4. \ud574\ub2f9 \ub3d9\uc601\uc0c1\uc740 \ubca4\ucfe0\ubc84 \uc218\uc871\uad00\uc5d0\uc11c 5\ub144 \uc55e\uc11c \ucd2c\uc601\ub41c \uac83\uc73c\ub85c, \uc5c5\ub85c\ub4dc\ub418\uc5c8\uc744 \ub2f9\uc2dc \uc720\ud29c\ube0c\uc5d0\uc11c \uac00\uc7a5 \uc778\uae30\uc788\ub294 \ub3d9\ubb3c \uad00\ub828 \ub3d9\uc601\uc0c1\uc774\uc5c8\ub2e4. \ub3d9\uc601\uc0c1\uc5d0\uc11c \ud138 \uc0c9\uae54\uc774 \ubc1d\uc740 \ud574\ub2ec\uc758 \uc774\ub984\uc740 \ub2c8\uc544\ud06c(Nyac)\uc774\uba70, 1989\ub144 \uc5d1\uc190 \ubc1c\ub370\uc2a4 \uae30\ub984 \ub204\ucd9c \uc0ac\uac74 \ub54c \uad6c\uc870\ub41c \ud574\ub2ec\uc774\ub2e4. \ub2c8\uc544\ud06c\ub294 2008\ub144 9\uc6d4\uc5d0 20\uc0b4\ub85c \uc8fd\uc5c8\ub2e4."} {"question": "\ub178\ub974\uc6e8\uc774\uc5d0\uc11c\ub294 \ubc88\uac1c\ub97c \ubb34\uc5c7\uc774\ub77c\uace0 \ubd88\ub800\ub294\uac00?", "paragraph": "\ud1a0\ub974\uc5d0 \uad00\ud55c \uc774\uc57c\uae30, \ub610\ub294 \ud1a0\ub974 \uc774\uc57c\uae30\uc758 \uc804\ud1b5\uc5d0 \uc601\ud5a5\uc744 \ubc1b\uc740 \uc774\uc57c\uae30\ub294 \uadfc\ub300 \ub4e4\uc5b4\uc11c\ub3c4 \uc2a4\uce78\ub514\ub098\ube44\uc544 \uc9c0\uc5ed\uc5d0\uc11c \uacc4\uc18d \uad6c\uc804\ub418\uc5c8\ub2e4. 19\uc138\uae30\uc5d0 \uc57c\ucf54\ud504 \uadf8\ub9bc\uc740 \uac8c\ub974\ub9cc \uc5b8\uc5b4\uc5d0\uc11c \uc61b \uc2e0\ub4e4\uc758 \uc774\ub984 \ud615\ud0dc\uac00 \ub0a8\uc544 \uc788\ub294 \uc5ec\ub7ec \ud45c\ud604\ub4e4\uc744 \uae30\ub85d\ud588\ub2e4. \ub178\ub974\uc6e8\uc774\uc5d0\uc11c\ub294 \ubc88\uac1c\ub97c \"\ud1a0\ub974\uc758 \uc628\uae30\"\ub77c\ub294 \ub73b\uc758 \"\ud1a0\ub974\uc2a4\ubc14\ub984(Thorsvarme)\uc774\ub77c\uace0 \ubd88\ub800\ub2e4. \uc2a4\uc6e8\ub374\uc5d0\uc11c\ub294 \ubc88\uac1c\ub97c \"\uc120\ud55c \ub178\uc778[\ub3d9\ub8cc]\uc774 \uc804\ucc28\ub97c \ub2ec\ub9b0\ub2e4\"\ub294 \ub73b\uc758 \"\uace0\ub4dc\uad6c\ubca4 \uc544\ud30c\ub974(godgubben \u00e5far)\"\ub77c\uace0 \ud588\uace0, \ucc9c\ub465\uc744 \"\ud1a0\ub974\uc758 \uc6b0\ub974\ub989\"\uc774\ub77c\ub294 \ub73b\uc758 \"\ud1a0\ub974\ub3c8(tord\u00f6n)\uc774\ub77c\uace0 \ud588\ub2e4. \uadf8\ub9bc\uc740 \ub2f9\ub300\uc758 \uc2a4\uce78\ub514\ub098\ube44\uc544\uc778\ub4e4\uc774 \u201c\ub354\uc774\uc0c1 \uadf8 \uc2e0\uc758 \uc9c4\uc9dc \uc774\ub984\uc744 \ubc1c\uc74c\ud558\uac70\ub098 \uadf8\uc758 \uc2e0\uc131\uc744 \ucc2c\ubbf8\ud558\uae30 \uc6d0\ud558\uc9c0 \uc54a\ub294\ub2e4 [...]\u201d\uace0 \ub367\ubd99\uc600\ub2e4.", "answer": "\ud1a0\ub974\uc2a4\ubc14\ub984", "sentence": "\ub178\ub974\uc6e8\uc774\uc5d0\uc11c\ub294 \ubc88\uac1c\ub97c \"\ud1a0\ub974\uc758 \uc628\uae30\"\ub77c\ub294 \ub73b\uc758 \" \ud1a0\ub974\uc2a4\ubc14\ub984 (Thorsvarme)", "paragraph_sentence": "\ud1a0\ub974\uc5d0 \uad00\ud55c \uc774\uc57c\uae30, \ub610\ub294 \ud1a0\ub974 \uc774\uc57c\uae30\uc758 \uc804\ud1b5\uc5d0 \uc601\ud5a5\uc744 \ubc1b\uc740 \uc774\uc57c\uae30\ub294 \uadfc\ub300 \ub4e4\uc5b4\uc11c\ub3c4 \uc2a4\uce78\ub514\ub098\ube44\uc544 \uc9c0\uc5ed\uc5d0\uc11c \uacc4\uc18d \uad6c\uc804\ub418\uc5c8\ub2e4. 19\uc138\uae30\uc5d0 \uc57c\ucf54\ud504 \uadf8\ub9bc\uc740 \uac8c\ub974\ub9cc \uc5b8\uc5b4\uc5d0\uc11c \uc61b \uc2e0\ub4e4\uc758 \uc774\ub984 \ud615\ud0dc\uac00 \ub0a8\uc544 \uc788\ub294 \uc5ec\ub7ec \ud45c\ud604\ub4e4\uc744 \uae30\ub85d\ud588\ub2e4. \ub178\ub974\uc6e8\uc774\uc5d0\uc11c\ub294 \ubc88\uac1c\ub97c \"\ud1a0\ub974\uc758 \uc628\uae30\"\ub77c\ub294 \ub73b\uc758 \" \ud1a0\ub974\uc2a4\ubc14\ub984 (Thorsvarme) \uc774\ub77c\uace0 \ubd88\ub800\ub2e4. \uc2a4\uc6e8\ub374\uc5d0\uc11c\ub294 \ubc88\uac1c\ub97c \"\uc120\ud55c \ub178\uc778[\ub3d9\ub8cc]\uc774 \uc804\ucc28\ub97c \ub2ec\ub9b0\ub2e4\"\ub294 \ub73b\uc758 \"\uace0\ub4dc\uad6c\ubca4 \uc544\ud30c\ub974(godgubben \u00e5far)\"\ub77c\uace0 \ud588\uace0, \ucc9c\ub465\uc744 \"\ud1a0\ub974\uc758 \uc6b0\ub974\ub989\"\uc774\ub77c\ub294 \ub73b\uc758 \"\ud1a0\ub974\ub3c8(tord\u00f6n)\uc774\ub77c\uace0 \ud588\ub2e4. \uadf8\ub9bc\uc740 \ub2f9\ub300\uc758 \uc2a4\uce78\ub514\ub098\ube44\uc544\uc778\ub4e4\uc774 \u201c\ub354\uc774\uc0c1 \uadf8 \uc2e0\uc758 \uc9c4\uc9dc \uc774\ub984\uc744 \ubc1c\uc74c\ud558\uac70\ub098 \uadf8\uc758 \uc2e0\uc131\uc744 \ucc2c\ubbf8\ud558\uae30 \uc6d0\ud558\uc9c0 \uc54a\ub294\ub2e4 [...]\u201d\uace0 \ub367\ubd99\uc600\ub2e4.", "paragraph_answer": "\ud1a0\ub974\uc5d0 \uad00\ud55c \uc774\uc57c\uae30, \ub610\ub294 \ud1a0\ub974 \uc774\uc57c\uae30\uc758 \uc804\ud1b5\uc5d0 \uc601\ud5a5\uc744 \ubc1b\uc740 \uc774\uc57c\uae30\ub294 \uadfc\ub300 \ub4e4\uc5b4\uc11c\ub3c4 \uc2a4\uce78\ub514\ub098\ube44\uc544 \uc9c0\uc5ed\uc5d0\uc11c \uacc4\uc18d \uad6c\uc804\ub418\uc5c8\ub2e4. 19\uc138\uae30\uc5d0 \uc57c\ucf54\ud504 \uadf8\ub9bc\uc740 \uac8c\ub974\ub9cc \uc5b8\uc5b4\uc5d0\uc11c \uc61b \uc2e0\ub4e4\uc758 \uc774\ub984 \ud615\ud0dc\uac00 \ub0a8\uc544 \uc788\ub294 \uc5ec\ub7ec \ud45c\ud604\ub4e4\uc744 \uae30\ub85d\ud588\ub2e4. \ub178\ub974\uc6e8\uc774\uc5d0\uc11c\ub294 \ubc88\uac1c\ub97c \"\ud1a0\ub974\uc758 \uc628\uae30\"\ub77c\ub294 \ub73b\uc758 \" \ud1a0\ub974\uc2a4\ubc14\ub984 (Thorsvarme)\uc774\ub77c\uace0 \ubd88\ub800\ub2e4. \uc2a4\uc6e8\ub374\uc5d0\uc11c\ub294 \ubc88\uac1c\ub97c \"\uc120\ud55c \ub178\uc778[\ub3d9\ub8cc]\uc774 \uc804\ucc28\ub97c \ub2ec\ub9b0\ub2e4\"\ub294 \ub73b\uc758 \"\uace0\ub4dc\uad6c\ubca4 \uc544\ud30c\ub974(godgubben \u00e5far)\"\ub77c\uace0 \ud588\uace0, \ucc9c\ub465\uc744 \"\ud1a0\ub974\uc758 \uc6b0\ub974\ub989\"\uc774\ub77c\ub294 \ub73b\uc758 \"\ud1a0\ub974\ub3c8(tord\u00f6n)\uc774\ub77c\uace0 \ud588\ub2e4. \uadf8\ub9bc\uc740 \ub2f9\ub300\uc758 \uc2a4\uce78\ub514\ub098\ube44\uc544\uc778\ub4e4\uc774 \u201c\ub354\uc774\uc0c1 \uadf8 \uc2e0\uc758 \uc9c4\uc9dc \uc774\ub984\uc744 \ubc1c\uc74c\ud558\uac70\ub098 \uadf8\uc758 \uc2e0\uc131\uc744 \ucc2c\ubbf8\ud558\uae30 \uc6d0\ud558\uc9c0 \uc54a\ub294\ub2e4 [...]\u201d\uace0 \ub367\ubd99\uc600\ub2e4.", "sentence_answer": "\ub178\ub974\uc6e8\uc774\uc5d0\uc11c\ub294 \ubc88\uac1c\ub97c \"\ud1a0\ub974\uc758 \uc628\uae30\"\ub77c\ub294 \ub73b\uc758 \" \ud1a0\ub974\uc2a4\ubc14\ub984 (Thorsvarme)", "paragraph_id": "6566103-8-0", "paragraph_question": "question: \ub178\ub974\uc6e8\uc774\uc5d0\uc11c\ub294 \ubc88\uac1c\ub97c \ubb34\uc5c7\uc774\ub77c\uace0 \ubd88\ub800\ub294\uac00?, context: \ud1a0\ub974\uc5d0 \uad00\ud55c \uc774\uc57c\uae30, \ub610\ub294 \ud1a0\ub974 \uc774\uc57c\uae30\uc758 \uc804\ud1b5\uc5d0 \uc601\ud5a5\uc744 \ubc1b\uc740 \uc774\uc57c\uae30\ub294 \uadfc\ub300 \ub4e4\uc5b4\uc11c\ub3c4 \uc2a4\uce78\ub514\ub098\ube44\uc544 \uc9c0\uc5ed\uc5d0\uc11c \uacc4\uc18d \uad6c\uc804\ub418\uc5c8\ub2e4. 19\uc138\uae30\uc5d0 \uc57c\ucf54\ud504 \uadf8\ub9bc\uc740 \uac8c\ub974\ub9cc \uc5b8\uc5b4\uc5d0\uc11c \uc61b \uc2e0\ub4e4\uc758 \uc774\ub984 \ud615\ud0dc\uac00 \ub0a8\uc544 \uc788\ub294 \uc5ec\ub7ec \ud45c\ud604\ub4e4\uc744 \uae30\ub85d\ud588\ub2e4. \ub178\ub974\uc6e8\uc774\uc5d0\uc11c\ub294 \ubc88\uac1c\ub97c \"\ud1a0\ub974\uc758 \uc628\uae30\"\ub77c\ub294 \ub73b\uc758 \"\ud1a0\ub974\uc2a4\ubc14\ub984(Thorsvarme)\uc774\ub77c\uace0 \ubd88\ub800\ub2e4. \uc2a4\uc6e8\ub374\uc5d0\uc11c\ub294 \ubc88\uac1c\ub97c \"\uc120\ud55c \ub178\uc778[\ub3d9\ub8cc]\uc774 \uc804\ucc28\ub97c \ub2ec\ub9b0\ub2e4\"\ub294 \ub73b\uc758 \"\uace0\ub4dc\uad6c\ubca4 \uc544\ud30c\ub974(godgubben \u00e5far)\"\ub77c\uace0 \ud588\uace0, \ucc9c\ub465\uc744 \"\ud1a0\ub974\uc758 \uc6b0\ub974\ub989\"\uc774\ub77c\ub294 \ub73b\uc758 \"\ud1a0\ub974\ub3c8(tord\u00f6n)\uc774\ub77c\uace0 \ud588\ub2e4. \uadf8\ub9bc\uc740 \ub2f9\ub300\uc758 \uc2a4\uce78\ub514\ub098\ube44\uc544\uc778\ub4e4\uc774 \u201c\ub354\uc774\uc0c1 \uadf8 \uc2e0\uc758 \uc9c4\uc9dc \uc774\ub984\uc744 \ubc1c\uc74c\ud558\uac70\ub098 \uadf8\uc758 \uc2e0\uc131\uc744 \ucc2c\ubbf8\ud558\uae30 \uc6d0\ud558\uc9c0 \uc54a\ub294\ub2e4 [...]\u201d\uace0 \ub367\ubd99\uc600\ub2e4."} {"question": "\uc544\uc774\ube44\uc758 \ucda4 \uc2e4\ub825\uacfc \uc139\uc2dc\ud55c \ubb34\ub300 \ub9e4\ub108\ub97c \ubcf4\uc5ec\uc900 \ub450 \ubc88\uc9f8 \uc2f1\uae00 \uace1 \uc81c\ubaa9\uc740?", "paragraph": "\uc544\uc774\ube44\ub294 JYP \uc5d4\ud130\ud14c\uc778\uba3c\ud2b8\uc758 \uc5f0\uc2b5\uc0dd\uc73c\ub85c \ub370\ubdd4\uc804 2005\ub144 \ub300\ud55c\ubbfc\uad6d\uc758 \uac00\uc218 \uc774\uc218\uc601\uc758 \u3008\uaf43\ub4e4\uc740 \uc9c0\uace0\u3009\uc758 \ubba4\uc9c1\ube44\ub514\uc624\uc5d0 \ucd9c\uc5f0\ud588\ub2e4. \uc544\uc774\ube44\ub294 JYP\uc758 \ubc15\uc9c4\uc601\uacfc \ud32c\ud140 \uc5d4\ud130\ud14c\uc778\uba3c\ud2b8\uac00 \uc190\uc744 \uc7a1\uace0 4\ub144\ub3d9\uc548 \uadf9\ube44\ub9ac\uc5d0 \uc900\ube44\ud574 \ub0b4\uc138\uc6b4 \uc288\ud37c \uc2e0\uc778\uc73c\ub85c\uc11c 2005\ub144 7\uc6d4 19\uc77c \ub4dc\ub514\uc5b4 \uc544\uc774\ube44\uc758 \ub370\ubdd4 \uc568\ubc94 My Sweet and Free Day\uac00 \ubc1c\ub9e4\ub418\uace0 \uccab \uc2f1\uae00 \"\uc624\ub298 \ubc24 \uc77c\"\uc744 \ubc1c\ub9e4\ud558\uba70 \uacf5\uc2dd\uc801\uc73c\ub85c \ub370\ubdd4\ud588\ub2e4. \ud32c\ud140\uc740 \ub2f9\uc2dc \uc544\uc774\ube44\uc5d0\uac8c \ub304\uc2a4, \ubc1c\ub77c\ub4dc \ub4f1 \ubaa8\ub4e0 \uc7a5\ub974\ub97c \uc18c\ud654\ud560 \ubcf4\uceec\uc744 \uc9c0\ub154\uace0 \ucda4\uc2e4\ub825\uae4c\uc9c0 \uacb8\ube44\ud574 \uc131\uc7a5 \uac00\ub2a5\uc131\uc774 \ubb34\uad81\ubb34\uc9c4\ud558\ub2e4\uace0 \uc18c\uac1c\ud558\uae30\ub3c4 \ud588\ub2e4. \uc774\ud6c4 \ub450 \ubc88\uc9f8 \uc2f1\uae00 \"A-ha\"\uac00 \ubc1c\ud45c\ub418\uc5c8\ub2e4. \"A-ha\"\ub294 \uc544\uc774\ube44\uc758 \ucda4 \uc2e4\ub825\uacfc \uc139\uc2dc\ud55c \ubb34\ub300 \ub9e4\ub108\ub97c \ubcf4\uc774\ub294\ub370 \ucda9\ubd84\ud558\uc600\ub2e4. \ud0c0\uc774\ud2c0\uace1 \ud65c\ub3d9\uc774 \ub05d\ub09c \ub4a4, \ud6c4\uc18d\uace1\uc73c\ub85c \"\ubc14\ubcf8\uac00\ubd10\"\ub85c \ud65c\ub3d9\ud558\uc600\ub294\ub370, \ubc1c\ub77c\ub4dc\ud48d\uc758 \ub178\ub798\ub85c \uc544\uc774\ube44\uc758 \uac00\ucc3d\ub825\uc744 \uc785\uc99d\ud558\ub294 \ub370 \ucda9\ubd84\ud558\uc600\uc73c\uba70 \uc628\ub77c\uc778\uc5d0\uc11c \ub3cc\ud48d\uc744 \uc77c\uc73c\ucf1c \uc74c\uc6d0 \ub9e4\ucd9c\ub9cc 10\uc5b5\uc6d0\uc774 \ub118\uc5c8\ub2e4. \ub610\ud55c \uc778\uae30\uc5d0 \ud798\uc785\uc5b4 \uac01\uc885 CF\uacc4\ub97c \uc12d\ub835\ud558\uae30\ub3c4 \ud588\ub294\ub370 \uce90\uc8fc\uc5bc \ube0c\ub79c\ub4dc \ubca0\uc774\uc9c1\ud558\uc6b0\uc2a4\uc640 \uc804\uc18d \ubaa8\ub378 \uacc4\uc57d\uc744 \ud558\uc600\uc73c\uba70 \ud654\uc7a5\ud488 \uce90\uc2dc\ucea3\uacfc\ub3c4 \uac70\uc561\uc758 CF \uacc4\uc57d\uc744 \ud558\uba70 CF\ud038\uc73c\ub85c \uac70\ub4ed\ub0ac\uc5c8\ub2e4. \ud55c\ud3b8 \uc0ac\uc774\uc5b8\uc5d0\uc11c \uc544\uc774\ube44\uc758 \uc774\ub984\uc744 \ub534 '\uc544\uc774\ube44 \ud3f0'\uc774 \ucd9c\uc2dc\ub418\uae30\ub3c4 \ud558\uc600\ub2e4. \ub370\ubdd4\ub97c \uc131\uacf5\uc801\uc73c\ub85c \ub9c8\uce58\uc790 \uc5f0\ub9d0 \uc2dc\uc0c1\uc2dd\uc5d0\uc11c \uac01\uc885 \uc0c1\uc744 \ud729\uc4f8\uc5c8\ub2e4. 2005\ub144 \uace8\ub4e0\ub514\uc2a4\ud06c \uc2dc\uc0c1\uc2dd\uc5d0\uc11c\ub294 \uc2e0\uc778\uc0c1\uacfc \ub300\ud55c\ubbfc\uad6d\uc5f0\uc608\uc608\uc220\uc0c1 \uc2dc\uc0c1\uc2dd\uc5d0\uc120 \ub304\uc2a4\ubd80\ubb38 \uac00\uc218\uc0c1\uc744 \uc218\uc0c1\ud558\uc600\ub2e4. SBS \uac00\uc694\ub300\uc804\uc5d0\uc11c\ub3c4 \uc2e0\uc778\uc0c1\uc744 \uc218\uc0c1\ud558\uba70 \uc704\ub825\uc744 \ubcf4\uc600\ub2e4. \ub610\ud55c \ucf54\ub9ac\uc544\ubca0\uc2a4\ud2b8\ub4dc\ub808\uc11c \uc2dc\uc0c1\uc2dd\uc5d0\uc11c \uac00\uc218\ubd80\ubb38 \ubca0\uc2a4\ud2b8 \ub4dc\ub808\uc11c\uc0c1\uc744 \ubc1b\uc558\ub2e4. MBC 10\ub300 \uac00\uc218\uac00\uc694\uc81c\uc5d0\uc11c\ub3c4 \uc2e0\uc778\uc0c1\uc744 \uc218\uc0c1\ud558\uc600\ub2e4.", "answer": "A-ha", "sentence": "\uc774\ud6c4 \ub450 \ubc88\uc9f8 \uc2f1\uae00 \" A-ha \"\uac00 \ubc1c\ud45c\ub418\uc5c8\ub2e4.", "paragraph_sentence": "\uc544\uc774\ube44\ub294 JYP \uc5d4\ud130\ud14c\uc778\uba3c\ud2b8\uc758 \uc5f0\uc2b5\uc0dd\uc73c\ub85c \ub370\ubdd4\uc804 2005\ub144 \ub300\ud55c\ubbfc\uad6d\uc758 \uac00\uc218 \uc774\uc218\uc601\uc758 \u3008\uaf43\ub4e4\uc740 \uc9c0\uace0\u3009\uc758 \ubba4\uc9c1\ube44\ub514\uc624\uc5d0 \ucd9c\uc5f0\ud588\ub2e4. \uc544\uc774\ube44\ub294 JYP\uc758 \ubc15\uc9c4\uc601\uacfc \ud32c\ud140 \uc5d4\ud130\ud14c\uc778\uba3c\ud2b8\uac00 \uc190\uc744 \uc7a1\uace0 4\ub144\ub3d9\uc548 \uadf9\ube44\ub9ac\uc5d0 \uc900\ube44\ud574 \ub0b4\uc138\uc6b4 \uc288\ud37c \uc2e0\uc778\uc73c\ub85c\uc11c 2005\ub144 7\uc6d4 19\uc77c \ub4dc\ub514\uc5b4 \uc544\uc774\ube44\uc758 \ub370\ubdd4 \uc568\ubc94 My Sweet and Free Day\uac00 \ubc1c\ub9e4\ub418\uace0 \uccab \uc2f1\uae00 \"\uc624\ub298 \ubc24 \uc77c\"\uc744 \ubc1c\ub9e4\ud558\uba70 \uacf5\uc2dd\uc801\uc73c\ub85c \ub370\ubdd4\ud588\ub2e4. \ud32c\ud140\uc740 \ub2f9\uc2dc \uc544\uc774\ube44\uc5d0\uac8c \ub304\uc2a4, \ubc1c\ub77c\ub4dc \ub4f1 \ubaa8\ub4e0 \uc7a5\ub974\ub97c \uc18c\ud654\ud560 \ubcf4\uceec\uc744 \uc9c0\ub154\uace0 \ucda4\uc2e4\ub825\uae4c\uc9c0 \uacb8\ube44\ud574 \uc131\uc7a5 \uac00\ub2a5\uc131\uc774 \ubb34\uad81\ubb34\uc9c4\ud558\ub2e4\uace0 \uc18c\uac1c\ud558\uae30\ub3c4 \ud588\ub2e4. \uc774\ud6c4 \ub450 \ubc88\uc9f8 \uc2f1\uae00 \" A-ha \"\uac00 \ubc1c\ud45c\ub418\uc5c8\ub2e4. \"A-ha\"\ub294 \uc544\uc774\ube44\uc758 \ucda4 \uc2e4\ub825\uacfc \uc139\uc2dc\ud55c \ubb34\ub300 \ub9e4\ub108\ub97c \ubcf4\uc774\ub294\ub370 \ucda9\ubd84\ud558\uc600\ub2e4. \ud0c0\uc774\ud2c0\uace1 \ud65c\ub3d9\uc774 \ub05d\ub09c \ub4a4, \ud6c4\uc18d\uace1\uc73c\ub85c \"\ubc14\ubcf8\uac00\ubd10\"\ub85c \ud65c\ub3d9\ud558\uc600\ub294\ub370, \ubc1c\ub77c\ub4dc\ud48d\uc758 \ub178\ub798\ub85c \uc544\uc774\ube44\uc758 \uac00\ucc3d\ub825\uc744 \uc785\uc99d\ud558\ub294 \ub370 \ucda9\ubd84\ud558\uc600\uc73c\uba70 \uc628\ub77c\uc778\uc5d0\uc11c \ub3cc\ud48d\uc744 \uc77c\uc73c\ucf1c \uc74c\uc6d0 \ub9e4\ucd9c\ub9cc 10\uc5b5\uc6d0\uc774 \ub118\uc5c8\ub2e4. \ub610\ud55c \uc778\uae30\uc5d0 \ud798\uc785\uc5b4 \uac01\uc885 CF\uacc4\ub97c \uc12d\ub835\ud558\uae30\ub3c4 \ud588\ub294\ub370 \uce90\uc8fc\uc5bc \ube0c\ub79c\ub4dc \ubca0\uc774\uc9c1\ud558\uc6b0\uc2a4\uc640 \uc804\uc18d \ubaa8\ub378 \uacc4\uc57d\uc744 \ud558\uc600\uc73c\uba70 \ud654\uc7a5\ud488 \uce90\uc2dc\ucea3\uacfc\ub3c4 \uac70\uc561\uc758 CF \uacc4\uc57d\uc744 \ud558\uba70 CF\ud038\uc73c\ub85c \uac70\ub4ed\ub0ac\uc5c8\ub2e4. \ud55c\ud3b8 \uc0ac\uc774\uc5b8\uc5d0\uc11c \uc544\uc774\ube44\uc758 \uc774\ub984\uc744 \ub534 '\uc544\uc774\ube44 \ud3f0'\uc774 \ucd9c\uc2dc\ub418\uae30\ub3c4 \ud558\uc600\ub2e4. \ub370\ubdd4\ub97c \uc131\uacf5\uc801\uc73c\ub85c \ub9c8\uce58\uc790 \uc5f0\ub9d0 \uc2dc\uc0c1\uc2dd\uc5d0\uc11c \uac01\uc885 \uc0c1\uc744 \ud729\uc4f8\uc5c8\ub2e4. 2005\ub144 \uace8\ub4e0\ub514\uc2a4\ud06c \uc2dc\uc0c1\uc2dd\uc5d0\uc11c\ub294 \uc2e0\uc778\uc0c1\uacfc \ub300\ud55c\ubbfc\uad6d\uc5f0\uc608\uc608\uc220\uc0c1 \uc2dc\uc0c1\uc2dd\uc5d0\uc120 \ub304\uc2a4\ubd80\ubb38 \uac00\uc218\uc0c1\uc744 \uc218\uc0c1\ud558\uc600\ub2e4. SBS \uac00\uc694\ub300\uc804\uc5d0\uc11c\ub3c4 \uc2e0\uc778\uc0c1\uc744 \uc218\uc0c1\ud558\uba70 \uc704\ub825\uc744 \ubcf4\uc600\ub2e4. \ub610\ud55c \ucf54\ub9ac\uc544\ubca0\uc2a4\ud2b8\ub4dc\ub808\uc11c \uc2dc\uc0c1\uc2dd\uc5d0\uc11c \uac00\uc218\ubd80\ubb38 \ubca0\uc2a4\ud2b8 \ub4dc\ub808\uc11c\uc0c1\uc744 \ubc1b\uc558\ub2e4. MBC 10\ub300 \uac00\uc218\uac00\uc694\uc81c\uc5d0\uc11c\ub3c4 \uc2e0\uc778\uc0c1\uc744 \uc218\uc0c1\ud558\uc600\ub2e4.", "paragraph_answer": "\uc544\uc774\ube44\ub294 JYP \uc5d4\ud130\ud14c\uc778\uba3c\ud2b8\uc758 \uc5f0\uc2b5\uc0dd\uc73c\ub85c \ub370\ubdd4\uc804 2005\ub144 \ub300\ud55c\ubbfc\uad6d\uc758 \uac00\uc218 \uc774\uc218\uc601\uc758 \u3008\uaf43\ub4e4\uc740 \uc9c0\uace0\u3009\uc758 \ubba4\uc9c1\ube44\ub514\uc624\uc5d0 \ucd9c\uc5f0\ud588\ub2e4. \uc544\uc774\ube44\ub294 JYP\uc758 \ubc15\uc9c4\uc601\uacfc \ud32c\ud140 \uc5d4\ud130\ud14c\uc778\uba3c\ud2b8\uac00 \uc190\uc744 \uc7a1\uace0 4\ub144\ub3d9\uc548 \uadf9\ube44\ub9ac\uc5d0 \uc900\ube44\ud574 \ub0b4\uc138\uc6b4 \uc288\ud37c \uc2e0\uc778\uc73c\ub85c\uc11c 2005\ub144 7\uc6d4 19\uc77c \ub4dc\ub514\uc5b4 \uc544\uc774\ube44\uc758 \ub370\ubdd4 \uc568\ubc94 My Sweet and Free Day\uac00 \ubc1c\ub9e4\ub418\uace0 \uccab \uc2f1\uae00 \"\uc624\ub298 \ubc24 \uc77c\"\uc744 \ubc1c\ub9e4\ud558\uba70 \uacf5\uc2dd\uc801\uc73c\ub85c \ub370\ubdd4\ud588\ub2e4. \ud32c\ud140\uc740 \ub2f9\uc2dc \uc544\uc774\ube44\uc5d0\uac8c \ub304\uc2a4, \ubc1c\ub77c\ub4dc \ub4f1 \ubaa8\ub4e0 \uc7a5\ub974\ub97c \uc18c\ud654\ud560 \ubcf4\uceec\uc744 \uc9c0\ub154\uace0 \ucda4\uc2e4\ub825\uae4c\uc9c0 \uacb8\ube44\ud574 \uc131\uc7a5 \uac00\ub2a5\uc131\uc774 \ubb34\uad81\ubb34\uc9c4\ud558\ub2e4\uace0 \uc18c\uac1c\ud558\uae30\ub3c4 \ud588\ub2e4. \uc774\ud6c4 \ub450 \ubc88\uc9f8 \uc2f1\uae00 \" A-ha \"\uac00 \ubc1c\ud45c\ub418\uc5c8\ub2e4. \"A-ha\"\ub294 \uc544\uc774\ube44\uc758 \ucda4 \uc2e4\ub825\uacfc \uc139\uc2dc\ud55c \ubb34\ub300 \ub9e4\ub108\ub97c \ubcf4\uc774\ub294\ub370 \ucda9\ubd84\ud558\uc600\ub2e4. \ud0c0\uc774\ud2c0\uace1 \ud65c\ub3d9\uc774 \ub05d\ub09c \ub4a4, \ud6c4\uc18d\uace1\uc73c\ub85c \"\ubc14\ubcf8\uac00\ubd10\"\ub85c \ud65c\ub3d9\ud558\uc600\ub294\ub370, \ubc1c\ub77c\ub4dc\ud48d\uc758 \ub178\ub798\ub85c \uc544\uc774\ube44\uc758 \uac00\ucc3d\ub825\uc744 \uc785\uc99d\ud558\ub294 \ub370 \ucda9\ubd84\ud558\uc600\uc73c\uba70 \uc628\ub77c\uc778\uc5d0\uc11c \ub3cc\ud48d\uc744 \uc77c\uc73c\ucf1c \uc74c\uc6d0 \ub9e4\ucd9c\ub9cc 10\uc5b5\uc6d0\uc774 \ub118\uc5c8\ub2e4. \ub610\ud55c \uc778\uae30\uc5d0 \ud798\uc785\uc5b4 \uac01\uc885 CF\uacc4\ub97c \uc12d\ub835\ud558\uae30\ub3c4 \ud588\ub294\ub370 \uce90\uc8fc\uc5bc \ube0c\ub79c\ub4dc \ubca0\uc774\uc9c1\ud558\uc6b0\uc2a4\uc640 \uc804\uc18d \ubaa8\ub378 \uacc4\uc57d\uc744 \ud558\uc600\uc73c\uba70 \ud654\uc7a5\ud488 \uce90\uc2dc\ucea3\uacfc\ub3c4 \uac70\uc561\uc758 CF \uacc4\uc57d\uc744 \ud558\uba70 CF\ud038\uc73c\ub85c \uac70\ub4ed\ub0ac\uc5c8\ub2e4. \ud55c\ud3b8 \uc0ac\uc774\uc5b8\uc5d0\uc11c \uc544\uc774\ube44\uc758 \uc774\ub984\uc744 \ub534 '\uc544\uc774\ube44 \ud3f0'\uc774 \ucd9c\uc2dc\ub418\uae30\ub3c4 \ud558\uc600\ub2e4. \ub370\ubdd4\ub97c \uc131\uacf5\uc801\uc73c\ub85c \ub9c8\uce58\uc790 \uc5f0\ub9d0 \uc2dc\uc0c1\uc2dd\uc5d0\uc11c \uac01\uc885 \uc0c1\uc744 \ud729\uc4f8\uc5c8\ub2e4. 2005\ub144 \uace8\ub4e0\ub514\uc2a4\ud06c \uc2dc\uc0c1\uc2dd\uc5d0\uc11c\ub294 \uc2e0\uc778\uc0c1\uacfc \ub300\ud55c\ubbfc\uad6d\uc5f0\uc608\uc608\uc220\uc0c1 \uc2dc\uc0c1\uc2dd\uc5d0\uc120 \ub304\uc2a4\ubd80\ubb38 \uac00\uc218\uc0c1\uc744 \uc218\uc0c1\ud558\uc600\ub2e4. SBS \uac00\uc694\ub300\uc804\uc5d0\uc11c\ub3c4 \uc2e0\uc778\uc0c1\uc744 \uc218\uc0c1\ud558\uba70 \uc704\ub825\uc744 \ubcf4\uc600\ub2e4. \ub610\ud55c \ucf54\ub9ac\uc544\ubca0\uc2a4\ud2b8\ub4dc\ub808\uc11c \uc2dc\uc0c1\uc2dd\uc5d0\uc11c \uac00\uc218\ubd80\ubb38 \ubca0\uc2a4\ud2b8 \ub4dc\ub808\uc11c\uc0c1\uc744 \ubc1b\uc558\ub2e4. MBC 10\ub300 \uac00\uc218\uac00\uc694\uc81c\uc5d0\uc11c\ub3c4 \uc2e0\uc778\uc0c1\uc744 \uc218\uc0c1\ud558\uc600\ub2e4.", "sentence_answer": "\uc774\ud6c4 \ub450 \ubc88\uc9f8 \uc2f1\uae00 \" A-ha \"\uac00 \ubc1c\ud45c\ub418\uc5c8\ub2e4.", "paragraph_id": "6520985-2-2", "paragraph_question": "question: \uc544\uc774\ube44\uc758 \ucda4 \uc2e4\ub825\uacfc \uc139\uc2dc\ud55c \ubb34\ub300 \ub9e4\ub108\ub97c \ubcf4\uc5ec\uc900 \ub450 \ubc88\uc9f8 \uc2f1\uae00 \uace1 \uc81c\ubaa9\uc740?, context: \uc544\uc774\ube44\ub294 JYP \uc5d4\ud130\ud14c\uc778\uba3c\ud2b8\uc758 \uc5f0\uc2b5\uc0dd\uc73c\ub85c \ub370\ubdd4\uc804 2005\ub144 \ub300\ud55c\ubbfc\uad6d\uc758 \uac00\uc218 \uc774\uc218\uc601\uc758 \u3008\uaf43\ub4e4\uc740 \uc9c0\uace0\u3009\uc758 \ubba4\uc9c1\ube44\ub514\uc624\uc5d0 \ucd9c\uc5f0\ud588\ub2e4. \uc544\uc774\ube44\ub294 JYP\uc758 \ubc15\uc9c4\uc601\uacfc \ud32c\ud140 \uc5d4\ud130\ud14c\uc778\uba3c\ud2b8\uac00 \uc190\uc744 \uc7a1\uace0 4\ub144\ub3d9\uc548 \uadf9\ube44\ub9ac\uc5d0 \uc900\ube44\ud574 \ub0b4\uc138\uc6b4 \uc288\ud37c \uc2e0\uc778\uc73c\ub85c\uc11c 2005\ub144 7\uc6d4 19\uc77c \ub4dc\ub514\uc5b4 \uc544\uc774\ube44\uc758 \ub370\ubdd4 \uc568\ubc94 My Sweet and Free Day\uac00 \ubc1c\ub9e4\ub418\uace0 \uccab \uc2f1\uae00 \"\uc624\ub298 \ubc24 \uc77c\"\uc744 \ubc1c\ub9e4\ud558\uba70 \uacf5\uc2dd\uc801\uc73c\ub85c \ub370\ubdd4\ud588\ub2e4. \ud32c\ud140\uc740 \ub2f9\uc2dc \uc544\uc774\ube44\uc5d0\uac8c \ub304\uc2a4, \ubc1c\ub77c\ub4dc \ub4f1 \ubaa8\ub4e0 \uc7a5\ub974\ub97c \uc18c\ud654\ud560 \ubcf4\uceec\uc744 \uc9c0\ub154\uace0 \ucda4\uc2e4\ub825\uae4c\uc9c0 \uacb8\ube44\ud574 \uc131\uc7a5 \uac00\ub2a5\uc131\uc774 \ubb34\uad81\ubb34\uc9c4\ud558\ub2e4\uace0 \uc18c\uac1c\ud558\uae30\ub3c4 \ud588\ub2e4. \uc774\ud6c4 \ub450 \ubc88\uc9f8 \uc2f1\uae00 \"A-ha\"\uac00 \ubc1c\ud45c\ub418\uc5c8\ub2e4. \"A-ha\"\ub294 \uc544\uc774\ube44\uc758 \ucda4 \uc2e4\ub825\uacfc \uc139\uc2dc\ud55c \ubb34\ub300 \ub9e4\ub108\ub97c \ubcf4\uc774\ub294\ub370 \ucda9\ubd84\ud558\uc600\ub2e4. \ud0c0\uc774\ud2c0\uace1 \ud65c\ub3d9\uc774 \ub05d\ub09c \ub4a4, \ud6c4\uc18d\uace1\uc73c\ub85c \"\ubc14\ubcf8\uac00\ubd10\"\ub85c \ud65c\ub3d9\ud558\uc600\ub294\ub370, \ubc1c\ub77c\ub4dc\ud48d\uc758 \ub178\ub798\ub85c \uc544\uc774\ube44\uc758 \uac00\ucc3d\ub825\uc744 \uc785\uc99d\ud558\ub294 \ub370 \ucda9\ubd84\ud558\uc600\uc73c\uba70 \uc628\ub77c\uc778\uc5d0\uc11c \ub3cc\ud48d\uc744 \uc77c\uc73c\ucf1c \uc74c\uc6d0 \ub9e4\ucd9c\ub9cc 10\uc5b5\uc6d0\uc774 \ub118\uc5c8\ub2e4. \ub610\ud55c \uc778\uae30\uc5d0 \ud798\uc785\uc5b4 \uac01\uc885 CF\uacc4\ub97c \uc12d\ub835\ud558\uae30\ub3c4 \ud588\ub294\ub370 \uce90\uc8fc\uc5bc \ube0c\ub79c\ub4dc \ubca0\uc774\uc9c1\ud558\uc6b0\uc2a4\uc640 \uc804\uc18d \ubaa8\ub378 \uacc4\uc57d\uc744 \ud558\uc600\uc73c\uba70 \ud654\uc7a5\ud488 \uce90\uc2dc\ucea3\uacfc\ub3c4 \uac70\uc561\uc758 CF \uacc4\uc57d\uc744 \ud558\uba70 CF\ud038\uc73c\ub85c \uac70\ub4ed\ub0ac\uc5c8\ub2e4. \ud55c\ud3b8 \uc0ac\uc774\uc5b8\uc5d0\uc11c \uc544\uc774\ube44\uc758 \uc774\ub984\uc744 \ub534 '\uc544\uc774\ube44 \ud3f0'\uc774 \ucd9c\uc2dc\ub418\uae30\ub3c4 \ud558\uc600\ub2e4. \ub370\ubdd4\ub97c \uc131\uacf5\uc801\uc73c\ub85c \ub9c8\uce58\uc790 \uc5f0\ub9d0 \uc2dc\uc0c1\uc2dd\uc5d0\uc11c \uac01\uc885 \uc0c1\uc744 \ud729\uc4f8\uc5c8\ub2e4. 2005\ub144 \uace8\ub4e0\ub514\uc2a4\ud06c \uc2dc\uc0c1\uc2dd\uc5d0\uc11c\ub294 \uc2e0\uc778\uc0c1\uacfc \ub300\ud55c\ubbfc\uad6d\uc5f0\uc608\uc608\uc220\uc0c1 \uc2dc\uc0c1\uc2dd\uc5d0\uc120 \ub304\uc2a4\ubd80\ubb38 \uac00\uc218\uc0c1\uc744 \uc218\uc0c1\ud558\uc600\ub2e4. SBS \uac00\uc694\ub300\uc804\uc5d0\uc11c\ub3c4 \uc2e0\uc778\uc0c1\uc744 \uc218\uc0c1\ud558\uba70 \uc704\ub825\uc744 \ubcf4\uc600\ub2e4. \ub610\ud55c \ucf54\ub9ac\uc544\ubca0\uc2a4\ud2b8\ub4dc\ub808\uc11c \uc2dc\uc0c1\uc2dd\uc5d0\uc11c \uac00\uc218\ubd80\ubb38 \ubca0\uc2a4\ud2b8 \ub4dc\ub808\uc11c\uc0c1\uc744 \ubc1b\uc558\ub2e4. MBC 10\ub300 \uac00\uc218\uac00\uc694\uc81c\uc5d0\uc11c\ub3c4 \uc2e0\uc778\uc0c1\uc744 \uc218\uc0c1\ud558\uc600\ub2e4."} {"question": "\ub098\ud3f4\ub808\uc639\uc774 \uc544\ucf54 \uacf5\ubc29\uc804\uc5d0 \uc2e4\ud328\ud558\uace0 \ud1f4\uac01\ud55c \uacf3\uc740?", "paragraph": "\ud504\ub791\uc2a4 \ud601\uba85(1789)\uc758 \uc5ec\ud30c\ub97c \ubc29\uc9c0\ud558\uace0\uc790 \uc601\uad6d\uc744 \uc911\uc2ec\uc73c\ub85c \uacb0\uc131\ub41c \uc81c1\ucc28 \ub300\ud504\ub791\uc2a4 \ub3d9\ub9f9\uc774 \uc624\uc2a4\ud2b8\ub9ac\uc544\uc758 \ud0c8\ub77d\uc73c\ub85c \ubd95\uad34(1797)\ub418\uace0, \uc789\uae00\ub79c\ub4dc\ub9cc\uc774 \ud504\ub791\uc2a4\uc640 \uc2f8\uc6b0\ub294 \ud615\uad6d\uc774 \ub418\uc5c8\ub2e4. \uadf8\ub7ec\ub098 \uc81c\ud574\uad8c\uc744 \uc950\uace0 \uc788\ub294 \uc601\uad6d\uc5d0\uac8c \ud504\ub791\uc2a4\ub294 \ud0c0\uaca9\uc744 \uc904 \uc218 \uc5c6\uc5c8\ub2e4. \uadf8\ub798\uc11c \ub098\ud3f4\ub808\uc639 \ubcf4\ub098\ud30c\ub974\ud2b8\ub294 \uc789\uae00\ub79c\ub4dc\uc640 \uc778\ub3c4\uc640\uc758 \uc5f0\uacc4\ub97c \ub04a\uae30 \uc704\ud574, \uc624\uc2a4\ub9cc \uc81c\uad6d\ub839 \uc774\uc9d1\ud2b8\uc5d0 \uc6d0\uc815(1798)\ud558\uc600\ub2e4. \ub098\ud3f4\ub808\uc639\uc774 \uc774\ub044\ub294 \uc774\uc9d1\ud2b8 \uc6d0\uc815\uad70\uc740 \ud234\ub871 \ud56d\uc744 \ucd9c\ubc1c(1798\ub144 5\uc6d4 19\uc77c )\ud558\uc5ec \uc774\uc9d1\ud2b8 \uc544\ubd80\ud0a4\ub974 \ub9cc\uc5d0 \uc0c1\ub959\ud588\ub2e4(7\uc6d4 2\uc77c). \ud53c\ub77c\ubbf8\ub4dc \uc804\ud22c\uc5d0\uc11c \ud604\uc9c0\uad70\uc5d0\uac8c \uc2b9\ub9ac(7\uc6d4 21\uc77c)\ub97c \uac70\ub450\uace0 \uc774\uc5b4 \uce74\uc774\ub85c\uc5d0 \uc785\uc131\ud55c\ub2e4. \uadf8\ub7ec\ub098 10\uc77c \ud6c4 \ub098\uc77c \ud574\uc804(8\uc6d4 1\uc77c)\uc5d0\uc11c \ud5c8\ub808\uc774\uc1fc \ub12c\uc2a8\uc774 \uc774\ub044\ub294 \uc601\uad6d \ud568\ub300\uc5d0 \ud504\ub791\uc2a4 \ud568\ub300\uac00 \ub300\ud328\ud558\uc5ec \ub098\ud3f4\ub808\uc639\uc740 \uc774\uc9d1\ud2b8\uc5d0 \uace0\ub9bd\ub418\uac8c \ub41c\ub2e4. \ub098\ud3f4\ub808\uc639\uc740 \uc2dc\ub9ac\uc544 \ubc29\uba74\uc73c\ub85c \uce68\uacf5\ud558\uc5ec \uc544\ucf54\ub97c \ud3ec\uc704\ud558\uc9c0\ub9cc(\uc544\ucf54 \uacf5\ubc29\uc804, 1799\ub144 3\uc6d4 18\uc77c -5\uc6d4 20\uc77c) \uacf5\ub7b5\uc5d0 \uc2e4\ud328\ud558\uace0 \uc774\uc9d1\ud2b8\ub85c \ud1f4\uac01\ud588\ub2e4.", "answer": "\uc774\uc9d1\ud2b8", "sentence": "\uadf8\ub798\uc11c \ub098\ud3f4\ub808\uc639 \ubcf4\ub098\ud30c\ub974\ud2b8\ub294 \uc789\uae00\ub79c\ub4dc\uc640 \uc778\ub3c4\uc640\uc758 \uc5f0\uacc4\ub97c \ub04a\uae30 \uc704\ud574, \uc624\uc2a4\ub9cc \uc81c\uad6d\ub839 \uc774\uc9d1\ud2b8 \uc5d0 \uc6d0\uc815(1798)\ud558\uc600\ub2e4.", "paragraph_sentence": "\ud504\ub791\uc2a4 \ud601\uba85(1789)\uc758 \uc5ec\ud30c\ub97c \ubc29\uc9c0\ud558\uace0\uc790 \uc601\uad6d\uc744 \uc911\uc2ec\uc73c\ub85c \uacb0\uc131\ub41c \uc81c1\ucc28 \ub300\ud504\ub791\uc2a4 \ub3d9\ub9f9\uc774 \uc624\uc2a4\ud2b8\ub9ac\uc544\uc758 \ud0c8\ub77d\uc73c\ub85c \ubd95\uad34(1797)\ub418\uace0, \uc789\uae00\ub79c\ub4dc\ub9cc\uc774 \ud504\ub791\uc2a4\uc640 \uc2f8\uc6b0\ub294 \ud615\uad6d\uc774 \ub418\uc5c8\ub2e4. \uadf8\ub7ec\ub098 \uc81c\ud574\uad8c\uc744 \uc950\uace0 \uc788\ub294 \uc601\uad6d\uc5d0\uac8c \ud504\ub791\uc2a4\ub294 \ud0c0\uaca9\uc744 \uc904 \uc218 \uc5c6\uc5c8\ub2e4. \uadf8\ub798\uc11c \ub098\ud3f4\ub808\uc639 \ubcf4\ub098\ud30c\ub974\ud2b8\ub294 \uc789\uae00\ub79c\ub4dc\uc640 \uc778\ub3c4\uc640\uc758 \uc5f0\uacc4\ub97c \ub04a\uae30 \uc704\ud574, \uc624\uc2a4\ub9cc \uc81c\uad6d\ub839 \uc774\uc9d1\ud2b8 \uc5d0 \uc6d0\uc815(1798)\ud558\uc600\ub2e4. \ub098\ud3f4\ub808\uc639\uc774 \uc774\ub044\ub294 \uc774\uc9d1\ud2b8 \uc6d0\uc815\uad70\uc740 \ud234\ub871 \ud56d\uc744 \ucd9c\ubc1c(1798\ub144 5\uc6d4 19\uc77c )\ud558\uc5ec \uc774\uc9d1\ud2b8 \uc544\ubd80\ud0a4\ub974 \ub9cc\uc5d0 \uc0c1\ub959\ud588\ub2e4(7\uc6d4 2\uc77c). \ud53c\ub77c\ubbf8\ub4dc \uc804\ud22c\uc5d0\uc11c \ud604\uc9c0\uad70\uc5d0\uac8c \uc2b9\ub9ac(7\uc6d4 21\uc77c)\ub97c \uac70\ub450\uace0 \uc774\uc5b4 \uce74\uc774\ub85c\uc5d0 \uc785\uc131\ud55c\ub2e4. \uadf8\ub7ec\ub098 10\uc77c \ud6c4 \ub098\uc77c \ud574\uc804(8\uc6d4 1\uc77c)\uc5d0\uc11c \ud5c8\ub808\uc774\uc1fc \ub12c\uc2a8\uc774 \uc774\ub044\ub294 \uc601\uad6d \ud568\ub300\uc5d0 \ud504\ub791\uc2a4 \ud568\ub300\uac00 \ub300\ud328\ud558\uc5ec \ub098\ud3f4\ub808\uc639\uc740 \uc774\uc9d1\ud2b8\uc5d0 \uace0\ub9bd\ub418\uac8c \ub41c\ub2e4. \ub098\ud3f4\ub808\uc639\uc740 \uc2dc\ub9ac\uc544 \ubc29\uba74\uc73c\ub85c \uce68\uacf5\ud558\uc5ec \uc544\ucf54\ub97c \ud3ec\uc704\ud558\uc9c0\ub9cc(\uc544\ucf54 \uacf5\ubc29\uc804, 1799\ub144 3\uc6d4 18\uc77c -5\uc6d4 20\uc77c) \uacf5\ub7b5\uc5d0 \uc2e4\ud328\ud558\uace0 \uc774\uc9d1\ud2b8\ub85c \ud1f4\uac01\ud588\ub2e4.", "paragraph_answer": "\ud504\ub791\uc2a4 \ud601\uba85(1789)\uc758 \uc5ec\ud30c\ub97c \ubc29\uc9c0\ud558\uace0\uc790 \uc601\uad6d\uc744 \uc911\uc2ec\uc73c\ub85c \uacb0\uc131\ub41c \uc81c1\ucc28 \ub300\ud504\ub791\uc2a4 \ub3d9\ub9f9\uc774 \uc624\uc2a4\ud2b8\ub9ac\uc544\uc758 \ud0c8\ub77d\uc73c\ub85c \ubd95\uad34(1797)\ub418\uace0, \uc789\uae00\ub79c\ub4dc\ub9cc\uc774 \ud504\ub791\uc2a4\uc640 \uc2f8\uc6b0\ub294 \ud615\uad6d\uc774 \ub418\uc5c8\ub2e4. \uadf8\ub7ec\ub098 \uc81c\ud574\uad8c\uc744 \uc950\uace0 \uc788\ub294 \uc601\uad6d\uc5d0\uac8c \ud504\ub791\uc2a4\ub294 \ud0c0\uaca9\uc744 \uc904 \uc218 \uc5c6\uc5c8\ub2e4. \uadf8\ub798\uc11c \ub098\ud3f4\ub808\uc639 \ubcf4\ub098\ud30c\ub974\ud2b8\ub294 \uc789\uae00\ub79c\ub4dc\uc640 \uc778\ub3c4\uc640\uc758 \uc5f0\uacc4\ub97c \ub04a\uae30 \uc704\ud574, \uc624\uc2a4\ub9cc \uc81c\uad6d\ub839 \uc774\uc9d1\ud2b8 \uc5d0 \uc6d0\uc815(1798)\ud558\uc600\ub2e4. \ub098\ud3f4\ub808\uc639\uc774 \uc774\ub044\ub294 \uc774\uc9d1\ud2b8 \uc6d0\uc815\uad70\uc740 \ud234\ub871 \ud56d\uc744 \ucd9c\ubc1c(1798\ub144 5\uc6d4 19\uc77c )\ud558\uc5ec \uc774\uc9d1\ud2b8 \uc544\ubd80\ud0a4\ub974 \ub9cc\uc5d0 \uc0c1\ub959\ud588\ub2e4(7\uc6d4 2\uc77c). \ud53c\ub77c\ubbf8\ub4dc \uc804\ud22c\uc5d0\uc11c \ud604\uc9c0\uad70\uc5d0\uac8c \uc2b9\ub9ac(7\uc6d4 21\uc77c)\ub97c \uac70\ub450\uace0 \uc774\uc5b4 \uce74\uc774\ub85c\uc5d0 \uc785\uc131\ud55c\ub2e4. \uadf8\ub7ec\ub098 10\uc77c \ud6c4 \ub098\uc77c \ud574\uc804(8\uc6d4 1\uc77c)\uc5d0\uc11c \ud5c8\ub808\uc774\uc1fc \ub12c\uc2a8\uc774 \uc774\ub044\ub294 \uc601\uad6d \ud568\ub300\uc5d0 \ud504\ub791\uc2a4 \ud568\ub300\uac00 \ub300\ud328\ud558\uc5ec \ub098\ud3f4\ub808\uc639\uc740 \uc774\uc9d1\ud2b8\uc5d0 \uace0\ub9bd\ub418\uac8c \ub41c\ub2e4. \ub098\ud3f4\ub808\uc639\uc740 \uc2dc\ub9ac\uc544 \ubc29\uba74\uc73c\ub85c \uce68\uacf5\ud558\uc5ec \uc544\ucf54\ub97c \ud3ec\uc704\ud558\uc9c0\ub9cc(\uc544\ucf54 \uacf5\ubc29\uc804, 1799\ub144 3\uc6d4 18\uc77c -5\uc6d4 20\uc77c) \uacf5\ub7b5\uc5d0 \uc2e4\ud328\ud558\uace0 \uc774\uc9d1\ud2b8\ub85c \ud1f4\uac01\ud588\ub2e4.", "sentence_answer": "\uadf8\ub798\uc11c \ub098\ud3f4\ub808\uc639 \ubcf4\ub098\ud30c\ub974\ud2b8\ub294 \uc789\uae00\ub79c\ub4dc\uc640 \uc778\ub3c4\uc640\uc758 \uc5f0\uacc4\ub97c \ub04a\uae30 \uc704\ud574, \uc624\uc2a4\ub9cc \uc81c\uad6d\ub839 \uc774\uc9d1\ud2b8 \uc5d0 \uc6d0\uc815(1798)\ud558\uc600\ub2e4.", "paragraph_id": "6568624-11-2", "paragraph_question": "question: \ub098\ud3f4\ub808\uc639\uc774 \uc544\ucf54 \uacf5\ubc29\uc804\uc5d0 \uc2e4\ud328\ud558\uace0 \ud1f4\uac01\ud55c \uacf3\uc740?, context: \ud504\ub791\uc2a4 \ud601\uba85(1789)\uc758 \uc5ec\ud30c\ub97c \ubc29\uc9c0\ud558\uace0\uc790 \uc601\uad6d\uc744 \uc911\uc2ec\uc73c\ub85c \uacb0\uc131\ub41c \uc81c1\ucc28 \ub300\ud504\ub791\uc2a4 \ub3d9\ub9f9\uc774 \uc624\uc2a4\ud2b8\ub9ac\uc544\uc758 \ud0c8\ub77d\uc73c\ub85c \ubd95\uad34(1797)\ub418\uace0, \uc789\uae00\ub79c\ub4dc\ub9cc\uc774 \ud504\ub791\uc2a4\uc640 \uc2f8\uc6b0\ub294 \ud615\uad6d\uc774 \ub418\uc5c8\ub2e4. \uadf8\ub7ec\ub098 \uc81c\ud574\uad8c\uc744 \uc950\uace0 \uc788\ub294 \uc601\uad6d\uc5d0\uac8c \ud504\ub791\uc2a4\ub294 \ud0c0\uaca9\uc744 \uc904 \uc218 \uc5c6\uc5c8\ub2e4. \uadf8\ub798\uc11c \ub098\ud3f4\ub808\uc639 \ubcf4\ub098\ud30c\ub974\ud2b8\ub294 \uc789\uae00\ub79c\ub4dc\uc640 \uc778\ub3c4\uc640\uc758 \uc5f0\uacc4\ub97c \ub04a\uae30 \uc704\ud574, \uc624\uc2a4\ub9cc \uc81c\uad6d\ub839 \uc774\uc9d1\ud2b8\uc5d0 \uc6d0\uc815(1798)\ud558\uc600\ub2e4. \ub098\ud3f4\ub808\uc639\uc774 \uc774\ub044\ub294 \uc774\uc9d1\ud2b8 \uc6d0\uc815\uad70\uc740 \ud234\ub871 \ud56d\uc744 \ucd9c\ubc1c(1798\ub144 5\uc6d4 19\uc77c )\ud558\uc5ec \uc774\uc9d1\ud2b8 \uc544\ubd80\ud0a4\ub974 \ub9cc\uc5d0 \uc0c1\ub959\ud588\ub2e4(7\uc6d4 2\uc77c). \ud53c\ub77c\ubbf8\ub4dc \uc804\ud22c\uc5d0\uc11c \ud604\uc9c0\uad70\uc5d0\uac8c \uc2b9\ub9ac(7\uc6d4 21\uc77c)\ub97c \uac70\ub450\uace0 \uc774\uc5b4 \uce74\uc774\ub85c\uc5d0 \uc785\uc131\ud55c\ub2e4. \uadf8\ub7ec\ub098 10\uc77c \ud6c4 \ub098\uc77c \ud574\uc804(8\uc6d4 1\uc77c)\uc5d0\uc11c \ud5c8\ub808\uc774\uc1fc \ub12c\uc2a8\uc774 \uc774\ub044\ub294 \uc601\uad6d \ud568\ub300\uc5d0 \ud504\ub791\uc2a4 \ud568\ub300\uac00 \ub300\ud328\ud558\uc5ec \ub098\ud3f4\ub808\uc639\uc740 \uc774\uc9d1\ud2b8\uc5d0 \uace0\ub9bd\ub418\uac8c \ub41c\ub2e4. \ub098\ud3f4\ub808\uc639\uc740 \uc2dc\ub9ac\uc544 \ubc29\uba74\uc73c\ub85c \uce68\uacf5\ud558\uc5ec \uc544\ucf54\ub97c \ud3ec\uc704\ud558\uc9c0\ub9cc(\uc544\ucf54 \uacf5\ubc29\uc804, 1799\ub144 3\uc6d4 18\uc77c -5\uc6d4 20\uc77c) \uacf5\ub7b5\uc5d0 \uc2e4\ud328\ud558\uace0 \uc774\uc9d1\ud2b8\ub85c \ud1f4\uac01\ud588\ub2e4."} {"question": "\uc870\uc120\uad50\uc721\ub839 \uac1c\uc815 \uc774\ud6c4 \uc870\uc120\uc778 \uc785\ud559 \uacbd\uc7c1\uc774 \uce58\uc5f4\ud574\uc9c4 \uacbd\uc131\uace0\ub4f1\uacf5\uc5c5\ud559\uad50\uc5d0 1935\ub144 \uc870\uc120\uc778 \ud559\uc0dd 1\uba85\ub9cc\uc774 \ud569\uaca9\ud55c \uacfc\ub294?", "paragraph": "\uacbd\uc131\uace0\ub4f1\uacf5\uc5c5\ud559\uad50\ub294 1940\ub144\ub300\uae4c\uc9c0 \uc870\uc120\uc758 \uc720\uc77c\ud55c \uace0\ub4f1\uacf5\uc5c5\uae30\uad00\uc774\uc5c8\uae30 \ub54c\ubb38\uc5d0 \ub2e4\ub978 \uc804\ubb38\ud559\uad50\uc5d0 \ube44\ud574 \ud2b9\uc218\ud55c \ud559\uad50\ub85c \uc5ec\uaca8\uc84c\ub2e4. \uc870\uc120\uad50\uc721\ub839 \uac1c\uc815 \uc774\uc804\uc5d0\ub294 \uc870\uc120\uc778\uc744 \uc804\uccb4\uc815\uc6d0\uc5d0\uc11c 2/3\uac00\ub7c9 \uc120\ubc1c\ud55c\ub2e4\ub294 \ub0b4\uaddc\uac00 \uc788\uc5b4 \uc870\uc120\uc778\uc758 \uc785\ud559 \uacbd\uc7c1\uc774 \uce58\uc5f4\ud558\uc9c0 \uc54a\uc558\uc9c0\ub9cc, \uad50\uc721\ub839 \uac1c\uc815 \uc774\ud6c4 \uadf8\ub7ec\ud55c \ub0b4\uaddc\uac00 \ud3d0\uc9c0\ub418\uc5b4 \uc785\uc2dc \uacbd\uc7c1\ub960\uc740 13:1\uc5d0\uc11c 27:1\uae4c\uc9c0 \ud3ed\uc99d\ud558\uac8c \ub418\uc5c8\uace0 \uc870\uc120\uc778\uc758 \uc785\ud559\uc774 \ub9e4\uc6b0 \uc5b4\ub824\uc6cc\uc84c\ub2e4. \uc774 \ub54c\ubb38\uc5d0 1935\ub144\uc758 \uc785\uc2dc\uc5d0\uc11c\ub294 \ud1a0\ubaa9\uacfc\uc5d0 \uc870\uc120\uc778 \ud559\uc0dd\uc774 \ub2e8 1\uba85\ub9cc \ud569\uaca9\ud558\uae30\ub3c4 \ud588\ub2e4. \ub2f9\uc2dc \uad50\uc721\uacc4\uc5d0\uc11c\ub294 \uacbd\uc131\uace0\ub4f1\uc0c1\uc5c5\ud559\uad50(\uacbd\uc131\uace0\uc0c1)\uc758 \uc870\uc120\uc778 \uc785\ud559\ub09c\uacfc \ud568\uaed8 \uacbd\uc131\uace0\uacf5\uacfc \uacbd\uc131\uace0\uc0c1\uc758 \uc785\uc2dc\uc81c\ub3c4\ub97c \uc870\uc120\uc778\uc5d0\uac8c \uc720\ub9ac\ud558\uac8c \uc2dc\uc815\ud574\uc57c \ud55c\ub2e4\ub294 \uc758\uacac\uc774 \uc788\uc5c8\uc73c\ub098, \uacbd\uc131\uace0\ub4f1\uacf5\uc5c5\ud559\uad50\uc640 \uc870\uc120\ucd1d\ub3c5\ubd80 \ud559\ubb34\uad6d\uc5d0\uc11c \ubc1b\uc544\ub4e4\uc774\uc9c0 \uc54a\uc558\ub2e4.", "answer": "\ud1a0\ubaa9\uacfc", "sentence": "\uc774 \ub54c\ubb38\uc5d0 1935\ub144\uc758 \uc785\uc2dc\uc5d0\uc11c\ub294 \ud1a0\ubaa9\uacfc \uc5d0", "paragraph_sentence": "\uacbd\uc131\uace0\ub4f1\uacf5\uc5c5\ud559\uad50\ub294 1940\ub144\ub300\uae4c\uc9c0 \uc870\uc120\uc758 \uc720\uc77c\ud55c \uace0\ub4f1\uacf5\uc5c5\uae30\uad00\uc774\uc5c8\uae30 \ub54c\ubb38\uc5d0 \ub2e4\ub978 \uc804\ubb38\ud559\uad50\uc5d0 \ube44\ud574 \ud2b9\uc218\ud55c \ud559\uad50\ub85c \uc5ec\uaca8\uc84c\ub2e4. \uc870\uc120\uad50\uc721\ub839 \uac1c\uc815 \uc774\uc804\uc5d0\ub294 \uc870\uc120\uc778\uc744 \uc804\uccb4\uc815\uc6d0\uc5d0\uc11c 2/3\uac00\ub7c9 \uc120\ubc1c\ud55c\ub2e4\ub294 \ub0b4\uaddc\uac00 \uc788\uc5b4 \uc870\uc120\uc778\uc758 \uc785\ud559 \uacbd\uc7c1\uc774 \uce58\uc5f4\ud558\uc9c0 \uc54a\uc558\uc9c0\ub9cc, \uad50\uc721\ub839 \uac1c\uc815 \uc774\ud6c4 \uadf8\ub7ec\ud55c \ub0b4\uaddc\uac00 \ud3d0\uc9c0\ub418\uc5b4 \uc785\uc2dc \uacbd\uc7c1\ub960\uc740 13:1\uc5d0\uc11c 27:1\uae4c\uc9c0 \ud3ed\uc99d\ud558\uac8c \ub418\uc5c8\uace0 \uc870\uc120\uc778\uc758 \uc785\ud559\uc774 \ub9e4\uc6b0 \uc5b4\ub824\uc6cc\uc84c\ub2e4. \uc774 \ub54c\ubb38\uc5d0 1935\ub144\uc758 \uc785\uc2dc\uc5d0\uc11c\ub294 \ud1a0\ubaa9\uacfc \uc5d0 \uc870\uc120\uc778 \ud559\uc0dd\uc774 \ub2e8 1\uba85\ub9cc \ud569\uaca9\ud558\uae30\ub3c4 \ud588\ub2e4. \ub2f9\uc2dc \uad50\uc721\uacc4\uc5d0\uc11c\ub294 \uacbd\uc131\uace0\ub4f1\uc0c1\uc5c5\ud559\uad50(\uacbd\uc131\uace0\uc0c1)\uc758 \uc870\uc120\uc778 \uc785\ud559\ub09c\uacfc \ud568\uaed8 \uacbd\uc131\uace0\uacf5\uacfc \uacbd\uc131\uace0\uc0c1\uc758 \uc785\uc2dc\uc81c\ub3c4\ub97c \uc870\uc120\uc778\uc5d0\uac8c \uc720\ub9ac\ud558\uac8c \uc2dc\uc815\ud574\uc57c \ud55c\ub2e4\ub294 \uc758\uacac\uc774 \uc788\uc5c8\uc73c\ub098, \uacbd\uc131\uace0\ub4f1\uacf5\uc5c5\ud559\uad50\uc640 \uc870\uc120\ucd1d\ub3c5\ubd80 \ud559\ubb34\uad6d\uc5d0\uc11c \ubc1b\uc544\ub4e4\uc774\uc9c0 \uc54a\uc558\ub2e4.", "paragraph_answer": "\uacbd\uc131\uace0\ub4f1\uacf5\uc5c5\ud559\uad50\ub294 1940\ub144\ub300\uae4c\uc9c0 \uc870\uc120\uc758 \uc720\uc77c\ud55c \uace0\ub4f1\uacf5\uc5c5\uae30\uad00\uc774\uc5c8\uae30 \ub54c\ubb38\uc5d0 \ub2e4\ub978 \uc804\ubb38\ud559\uad50\uc5d0 \ube44\ud574 \ud2b9\uc218\ud55c \ud559\uad50\ub85c \uc5ec\uaca8\uc84c\ub2e4. \uc870\uc120\uad50\uc721\ub839 \uac1c\uc815 \uc774\uc804\uc5d0\ub294 \uc870\uc120\uc778\uc744 \uc804\uccb4\uc815\uc6d0\uc5d0\uc11c 2/3\uac00\ub7c9 \uc120\ubc1c\ud55c\ub2e4\ub294 \ub0b4\uaddc\uac00 \uc788\uc5b4 \uc870\uc120\uc778\uc758 \uc785\ud559 \uacbd\uc7c1\uc774 \uce58\uc5f4\ud558\uc9c0 \uc54a\uc558\uc9c0\ub9cc, \uad50\uc721\ub839 \uac1c\uc815 \uc774\ud6c4 \uadf8\ub7ec\ud55c \ub0b4\uaddc\uac00 \ud3d0\uc9c0\ub418\uc5b4 \uc785\uc2dc \uacbd\uc7c1\ub960\uc740 13:1\uc5d0\uc11c 27:1\uae4c\uc9c0 \ud3ed\uc99d\ud558\uac8c \ub418\uc5c8\uace0 \uc870\uc120\uc778\uc758 \uc785\ud559\uc774 \ub9e4\uc6b0 \uc5b4\ub824\uc6cc\uc84c\ub2e4. \uc774 \ub54c\ubb38\uc5d0 1935\ub144\uc758 \uc785\uc2dc\uc5d0\uc11c\ub294 \ud1a0\ubaa9\uacfc \uc5d0 \uc870\uc120\uc778 \ud559\uc0dd\uc774 \ub2e8 1\uba85\ub9cc \ud569\uaca9\ud558\uae30\ub3c4 \ud588\ub2e4. \ub2f9\uc2dc \uad50\uc721\uacc4\uc5d0\uc11c\ub294 \uacbd\uc131\uace0\ub4f1\uc0c1\uc5c5\ud559\uad50(\uacbd\uc131\uace0\uc0c1)\uc758 \uc870\uc120\uc778 \uc785\ud559\ub09c\uacfc \ud568\uaed8 \uacbd\uc131\uace0\uacf5\uacfc \uacbd\uc131\uace0\uc0c1\uc758 \uc785\uc2dc\uc81c\ub3c4\ub97c \uc870\uc120\uc778\uc5d0\uac8c \uc720\ub9ac\ud558\uac8c \uc2dc\uc815\ud574\uc57c \ud55c\ub2e4\ub294 \uc758\uacac\uc774 \uc788\uc5c8\uc73c\ub098, \uacbd\uc131\uace0\ub4f1\uacf5\uc5c5\ud559\uad50\uc640 \uc870\uc120\ucd1d\ub3c5\ubd80 \ud559\ubb34\uad6d\uc5d0\uc11c \ubc1b\uc544\ub4e4\uc774\uc9c0 \uc54a\uc558\ub2e4.", "sentence_answer": "\uc774 \ub54c\ubb38\uc5d0 1935\ub144\uc758 \uc785\uc2dc\uc5d0\uc11c\ub294 \ud1a0\ubaa9\uacfc \uc5d0", "paragraph_id": "6194880-2-1", "paragraph_question": "question: \uc870\uc120\uad50\uc721\ub839 \uac1c\uc815 \uc774\ud6c4 \uc870\uc120\uc778 \uc785\ud559 \uacbd\uc7c1\uc774 \uce58\uc5f4\ud574\uc9c4 \uacbd\uc131\uace0\ub4f1\uacf5\uc5c5\ud559\uad50\uc5d0 1935\ub144 \uc870\uc120\uc778 \ud559\uc0dd 1\uba85\ub9cc\uc774 \ud569\uaca9\ud55c \uacfc\ub294?, context: \uacbd\uc131\uace0\ub4f1\uacf5\uc5c5\ud559\uad50\ub294 1940\ub144\ub300\uae4c\uc9c0 \uc870\uc120\uc758 \uc720\uc77c\ud55c \uace0\ub4f1\uacf5\uc5c5\uae30\uad00\uc774\uc5c8\uae30 \ub54c\ubb38\uc5d0 \ub2e4\ub978 \uc804\ubb38\ud559\uad50\uc5d0 \ube44\ud574 \ud2b9\uc218\ud55c \ud559\uad50\ub85c \uc5ec\uaca8\uc84c\ub2e4. \uc870\uc120\uad50\uc721\ub839 \uac1c\uc815 \uc774\uc804\uc5d0\ub294 \uc870\uc120\uc778\uc744 \uc804\uccb4\uc815\uc6d0\uc5d0\uc11c 2/3\uac00\ub7c9 \uc120\ubc1c\ud55c\ub2e4\ub294 \ub0b4\uaddc\uac00 \uc788\uc5b4 \uc870\uc120\uc778\uc758 \uc785\ud559 \uacbd\uc7c1\uc774 \uce58\uc5f4\ud558\uc9c0 \uc54a\uc558\uc9c0\ub9cc, \uad50\uc721\ub839 \uac1c\uc815 \uc774\ud6c4 \uadf8\ub7ec\ud55c \ub0b4\uaddc\uac00 \ud3d0\uc9c0\ub418\uc5b4 \uc785\uc2dc \uacbd\uc7c1\ub960\uc740 13:1\uc5d0\uc11c 27:1\uae4c\uc9c0 \ud3ed\uc99d\ud558\uac8c \ub418\uc5c8\uace0 \uc870\uc120\uc778\uc758 \uc785\ud559\uc774 \ub9e4\uc6b0 \uc5b4\ub824\uc6cc\uc84c\ub2e4. \uc774 \ub54c\ubb38\uc5d0 1935\ub144\uc758 \uc785\uc2dc\uc5d0\uc11c\ub294 \ud1a0\ubaa9\uacfc\uc5d0 \uc870\uc120\uc778 \ud559\uc0dd\uc774 \ub2e8 1\uba85\ub9cc \ud569\uaca9\ud558\uae30\ub3c4 \ud588\ub2e4. \ub2f9\uc2dc \uad50\uc721\uacc4\uc5d0\uc11c\ub294 \uacbd\uc131\uace0\ub4f1\uc0c1\uc5c5\ud559\uad50(\uacbd\uc131\uace0\uc0c1)\uc758 \uc870\uc120\uc778 \uc785\ud559\ub09c\uacfc \ud568\uaed8 \uacbd\uc131\uace0\uacf5\uacfc \uacbd\uc131\uace0\uc0c1\uc758 \uc785\uc2dc\uc81c\ub3c4\ub97c \uc870\uc120\uc778\uc5d0\uac8c \uc720\ub9ac\ud558\uac8c \uc2dc\uc815\ud574\uc57c \ud55c\ub2e4\ub294 \uc758\uacac\uc774 \uc788\uc5c8\uc73c\ub098, \uacbd\uc131\uace0\ub4f1\uacf5\uc5c5\ud559\uad50\uc640 \uc870\uc120\ucd1d\ub3c5\ubd80 \ud559\ubb34\uad6d\uc5d0\uc11c \ubc1b\uc544\ub4e4\uc774\uc9c0 \uc54a\uc558\ub2e4."} {"question": "2016\ub144 9\uc6d4 12\uc77c \uacbd\uc8fc\uc9c0\uc9c4\uc758 \ubc1c\uc0dd\uc9c0\ub294?", "paragraph": "2016\ub144 \uacbd\uc8fc \uc9c0\uc9c4\uc740 2016\ub144 9\uc6d4 12\uc77c \ub300\ud55c\ubbfc\uad6d \uacbd\uc0c1\ubd81\ub3c4 \uacbd\uc8fc\uc2dc \ub0a8\ub0a8\uc11c\ucabd 8km \uc9c0\uc5ed\uc5d0\uc11c \ubc1c\uc0dd\ud55c \uc9c0\uc9c4\uc774\ub2e4. \ubbf8\uad6d \uc9c0\uc9c8\uc870\uc0ac\uad6d \uc9d1\uacc4 \uae30\uc900\uc73c\ub85c \uc804\uc9c4 \uaddc\ubaa8\ub294 \ub9ac\ud788\ud130 \uaddc\ubaa8 5.1\uc774\uc5c8\uc73c\uba70, \ubcf8\uc9c4\uc758 \uaddc\ubaa8\ub294 \ub9ac\ud788\ud130 \uaddc\ubaa8 5.8\uc774\uc5c8\ub2e4. \uc804\uc9c4\uc758 \ucd5c\ub300 \uc9c4\ub3c4\ub294 V, \ubcf8\uc9c4\uc758 \ucd5c\ub300 \uc9c4\ub3c4\ub294 VI\uc600\ub2e4. \ubcf8 \uc9c0\uc9c4\uc740 1978\ub144 \ub300\ud55c\ubbfc\uad6d \uc9c0\uc9c4 \uad00\uce21 \uc774\ub798 \uc5ed\ub300 \ucd5c\uac15\uc758 \uc9c0\uc9c4\uc774\ub2e4. \ub300\ud55c\ubbfc\uad6d \ub0b4 \uc721\uc0c1\uc9c0\uc9c4\uc73c\ub85c\ub294 1978\ub144 \ucda9\uccad\ubd81\ub3c4 \uc18d\ub9ac\uc0b0\uc5d0\uc11c \uaddc\ubaa8 5.2, \ucda9\uccad\ub0a8\ub3c4 \ud64d\uc131\uad70\uc5d0\uc11c \uaddc\ubaa8 5.0 \uc758 \uc9c0\uc9c4 \ubc1c\uc0dd \ud6c4 38\ub144\ub9cc\uc758 \ub300\ud615\uc9c0\uc9c4\uc774\ub2e4. \ud55c\ubc18\ub3c4\uc5d0\uc11c\ub294 \ud575\uc2e4\ud5d8 \ub4f1\uc5d0 \uc758\ud55c \uc778\uacf5\uc9c0\uc9c4\uc744 \uc81c\uc678\ud558\uba74 1980\ub144 \uc870\uc120\ubbfc\uc8fc\uc8fc\uc758\uc778\ubbfc\uacf5\ud654\uad6d \ud3c9\uc548\ubd81\ub3c4 \uc758\uc8fc-\uc0ad\uc8fc-\uad6c\uc131 \uc9c0\uc5ed\uc5d0\uc11c \uaddc\ubaa8 5.3\uc758 \uc9c0\uc9c4 \ubc1c\uc0dd\ud6c4 36\ub144\ub9cc\uc758 \ub300\ud615\uc9c0\uc9c4\uc774\ub2e4.", "answer": "\uacbd\uc8fc\uc2dc \ub0a8\ub0a8\uc11c\ucabd 8km", "sentence": "2016\ub144 \uacbd\uc8fc \uc9c0\uc9c4\uc740 2016\ub144 9\uc6d4 12\uc77c \ub300\ud55c\ubbfc\uad6d \uacbd\uc0c1\ubd81\ub3c4 \uacbd\uc8fc\uc2dc \ub0a8\ub0a8\uc11c\ucabd 8km \uc9c0\uc5ed\uc5d0\uc11c \ubc1c\uc0dd\ud55c \uc9c0\uc9c4\uc774\ub2e4.", "paragraph_sentence": " 2016\ub144 \uacbd\uc8fc \uc9c0\uc9c4\uc740 2016\ub144 9\uc6d4 12\uc77c \ub300\ud55c\ubbfc\uad6d \uacbd\uc0c1\ubd81\ub3c4 \uacbd\uc8fc\uc2dc \ub0a8\ub0a8\uc11c\ucabd 8km \uc9c0\uc5ed\uc5d0\uc11c \ubc1c\uc0dd\ud55c \uc9c0\uc9c4\uc774\ub2e4. \ubbf8\uad6d \uc9c0\uc9c8\uc870\uc0ac\uad6d \uc9d1\uacc4 \uae30\uc900\uc73c\ub85c \uc804\uc9c4 \uaddc\ubaa8\ub294 \ub9ac\ud788\ud130 \uaddc\ubaa8 5.1\uc774\uc5c8\uc73c\uba70, \ubcf8\uc9c4\uc758 \uaddc\ubaa8\ub294 \ub9ac\ud788\ud130 \uaddc\ubaa8 5.8\uc774\uc5c8\ub2e4. \uc804\uc9c4\uc758 \ucd5c\ub300 \uc9c4\ub3c4\ub294 V, \ubcf8\uc9c4\uc758 \ucd5c\ub300 \uc9c4\ub3c4\ub294 VI\uc600\ub2e4. \ubcf8 \uc9c0\uc9c4\uc740 1978\ub144 \ub300\ud55c\ubbfc\uad6d \uc9c0\uc9c4 \uad00\uce21 \uc774\ub798 \uc5ed\ub300 \ucd5c\uac15\uc758 \uc9c0\uc9c4\uc774\ub2e4. \ub300\ud55c\ubbfc\uad6d \ub0b4 \uc721\uc0c1\uc9c0\uc9c4\uc73c\ub85c\ub294 1978\ub144 \ucda9\uccad\ubd81\ub3c4 \uc18d\ub9ac\uc0b0\uc5d0\uc11c \uaddc\ubaa8 5.2, \ucda9\uccad\ub0a8\ub3c4 \ud64d\uc131\uad70\uc5d0\uc11c \uaddc\ubaa8 5.0 \uc758 \uc9c0\uc9c4 \ubc1c\uc0dd \ud6c4 38\ub144\ub9cc\uc758 \ub300\ud615\uc9c0\uc9c4\uc774\ub2e4. \ud55c\ubc18\ub3c4\uc5d0\uc11c\ub294 \ud575\uc2e4\ud5d8 \ub4f1\uc5d0 \uc758\ud55c \uc778\uacf5\uc9c0\uc9c4\uc744 \uc81c\uc678\ud558\uba74 1980\ub144 \uc870\uc120\ubbfc\uc8fc\uc8fc\uc758\uc778\ubbfc\uacf5\ud654\uad6d \ud3c9\uc548\ubd81\ub3c4 \uc758\uc8fc-\uc0ad\uc8fc-\uad6c\uc131 \uc9c0\uc5ed\uc5d0\uc11c \uaddc\ubaa8 5.3\uc758 \uc9c0\uc9c4 \ubc1c\uc0dd\ud6c4 36\ub144\ub9cc\uc758 \ub300\ud615\uc9c0\uc9c4\uc774\ub2e4.", "paragraph_answer": "2016\ub144 \uacbd\uc8fc \uc9c0\uc9c4\uc740 2016\ub144 9\uc6d4 12\uc77c \ub300\ud55c\ubbfc\uad6d \uacbd\uc0c1\ubd81\ub3c4 \uacbd\uc8fc\uc2dc \ub0a8\ub0a8\uc11c\ucabd 8km \uc9c0\uc5ed\uc5d0\uc11c \ubc1c\uc0dd\ud55c \uc9c0\uc9c4\uc774\ub2e4. \ubbf8\uad6d \uc9c0\uc9c8\uc870\uc0ac\uad6d \uc9d1\uacc4 \uae30\uc900\uc73c\ub85c \uc804\uc9c4 \uaddc\ubaa8\ub294 \ub9ac\ud788\ud130 \uaddc\ubaa8 5.1\uc774\uc5c8\uc73c\uba70, \ubcf8\uc9c4\uc758 \uaddc\ubaa8\ub294 \ub9ac\ud788\ud130 \uaddc\ubaa8 5.8\uc774\uc5c8\ub2e4. \uc804\uc9c4\uc758 \ucd5c\ub300 \uc9c4\ub3c4\ub294 V, \ubcf8\uc9c4\uc758 \ucd5c\ub300 \uc9c4\ub3c4\ub294 VI\uc600\ub2e4. \ubcf8 \uc9c0\uc9c4\uc740 1978\ub144 \ub300\ud55c\ubbfc\uad6d \uc9c0\uc9c4 \uad00\uce21 \uc774\ub798 \uc5ed\ub300 \ucd5c\uac15\uc758 \uc9c0\uc9c4\uc774\ub2e4. \ub300\ud55c\ubbfc\uad6d \ub0b4 \uc721\uc0c1\uc9c0\uc9c4\uc73c\ub85c\ub294 1978\ub144 \ucda9\uccad\ubd81\ub3c4 \uc18d\ub9ac\uc0b0\uc5d0\uc11c \uaddc\ubaa8 5.2, \ucda9\uccad\ub0a8\ub3c4 \ud64d\uc131\uad70\uc5d0\uc11c \uaddc\ubaa8 5.0 \uc758 \uc9c0\uc9c4 \ubc1c\uc0dd \ud6c4 38\ub144\ub9cc\uc758 \ub300\ud615\uc9c0\uc9c4\uc774\ub2e4. \ud55c\ubc18\ub3c4\uc5d0\uc11c\ub294 \ud575\uc2e4\ud5d8 \ub4f1\uc5d0 \uc758\ud55c \uc778\uacf5\uc9c0\uc9c4\uc744 \uc81c\uc678\ud558\uba74 1980\ub144 \uc870\uc120\ubbfc\uc8fc\uc8fc\uc758\uc778\ubbfc\uacf5\ud654\uad6d \ud3c9\uc548\ubd81\ub3c4 \uc758\uc8fc-\uc0ad\uc8fc-\uad6c\uc131 \uc9c0\uc5ed\uc5d0\uc11c \uaddc\ubaa8 5.3\uc758 \uc9c0\uc9c4 \ubc1c\uc0dd\ud6c4 36\ub144\ub9cc\uc758 \ub300\ud615\uc9c0\uc9c4\uc774\ub2e4.", "sentence_answer": "2016\ub144 \uacbd\uc8fc \uc9c0\uc9c4\uc740 2016\ub144 9\uc6d4 12\uc77c \ub300\ud55c\ubbfc\uad6d \uacbd\uc0c1\ubd81\ub3c4 \uacbd\uc8fc\uc2dc \ub0a8\ub0a8\uc11c\ucabd 8km \uc9c0\uc5ed\uc5d0\uc11c \ubc1c\uc0dd\ud55c \uc9c0\uc9c4\uc774\ub2e4.", "paragraph_id": "5780368-0-1", "paragraph_question": "question: 2016\ub144 9\uc6d4 12\uc77c \uacbd\uc8fc\uc9c0\uc9c4\uc758 \ubc1c\uc0dd\uc9c0\ub294?, context: 2016\ub144 \uacbd\uc8fc \uc9c0\uc9c4\uc740 2016\ub144 9\uc6d4 12\uc77c \ub300\ud55c\ubbfc\uad6d \uacbd\uc0c1\ubd81\ub3c4 \uacbd\uc8fc\uc2dc \ub0a8\ub0a8\uc11c\ucabd 8km \uc9c0\uc5ed\uc5d0\uc11c \ubc1c\uc0dd\ud55c \uc9c0\uc9c4\uc774\ub2e4. \ubbf8\uad6d \uc9c0\uc9c8\uc870\uc0ac\uad6d \uc9d1\uacc4 \uae30\uc900\uc73c\ub85c \uc804\uc9c4 \uaddc\ubaa8\ub294 \ub9ac\ud788\ud130 \uaddc\ubaa8 5.1\uc774\uc5c8\uc73c\uba70, \ubcf8\uc9c4\uc758 \uaddc\ubaa8\ub294 \ub9ac\ud788\ud130 \uaddc\ubaa8 5.8\uc774\uc5c8\ub2e4. \uc804\uc9c4\uc758 \ucd5c\ub300 \uc9c4\ub3c4\ub294 V, \ubcf8\uc9c4\uc758 \ucd5c\ub300 \uc9c4\ub3c4\ub294 VI\uc600\ub2e4. \ubcf8 \uc9c0\uc9c4\uc740 1978\ub144 \ub300\ud55c\ubbfc\uad6d \uc9c0\uc9c4 \uad00\uce21 \uc774\ub798 \uc5ed\ub300 \ucd5c\uac15\uc758 \uc9c0\uc9c4\uc774\ub2e4. \ub300\ud55c\ubbfc\uad6d \ub0b4 \uc721\uc0c1\uc9c0\uc9c4\uc73c\ub85c\ub294 1978\ub144 \ucda9\uccad\ubd81\ub3c4 \uc18d\ub9ac\uc0b0\uc5d0\uc11c \uaddc\ubaa8 5.2, \ucda9\uccad\ub0a8\ub3c4 \ud64d\uc131\uad70\uc5d0\uc11c \uaddc\ubaa8 5.0 \uc758 \uc9c0\uc9c4 \ubc1c\uc0dd \ud6c4 38\ub144\ub9cc\uc758 \ub300\ud615\uc9c0\uc9c4\uc774\ub2e4. \ud55c\ubc18\ub3c4\uc5d0\uc11c\ub294 \ud575\uc2e4\ud5d8 \ub4f1\uc5d0 \uc758\ud55c \uc778\uacf5\uc9c0\uc9c4\uc744 \uc81c\uc678\ud558\uba74 1980\ub144 \uc870\uc120\ubbfc\uc8fc\uc8fc\uc758\uc778\ubbfc\uacf5\ud654\uad6d \ud3c9\uc548\ubd81\ub3c4 \uc758\uc8fc-\uc0ad\uc8fc-\uad6c\uc131 \uc9c0\uc5ed\uc5d0\uc11c \uaddc\ubaa8 5.3\uc758 \uc9c0\uc9c4 \ubc1c\uc0dd\ud6c4 36\ub144\ub9cc\uc758 \ub300\ud615\uc9c0\uc9c4\uc774\ub2e4."} {"question": "\uc778\ud130\uc2a4\ud154\ub77c\ub294 \uc77c\ubc18\uc0c1\ub300\uc131\uc774\ub860\uc744 \uc801\uc808\ud558\uac8c \ub179\uc5ec\ub0b8 \uc601\ud654\ub77c\uace0 \ud3c9\uac00\ud55c \uc0ac\ub78c\uc740?", "paragraph": "\uc601\ud654\ud3c9\ub860\uac00 \uc774\ub3d9\uc9c4\uc740 \u201c\uc778\ud130\uc2a4\ud154\ub77c\ub294 \uc6b0\uc8fc\ub97c \uafc8\uafb8\uac8c \ud55c\ub2e4. \ub9cc\uc57d \ucc9c\uccb4\ubb3c\ub9ac\ud559\uc5d0 \uad00\uc2ec\uc774 \uc788\ub294 \uc911\ud559\uc0dd\uc774\ub77c\uba74 \uc774 \uc601\ud654\uac00 \uc778\uc0dd\uc744 \ubc14\uafc0 \uc218\ub3c4 \uc788\ub2e4\u201c\uace0 \ud3c9\uac00\ud588\ub2e4. \uc601\ud654\ud3c9\ub860\uac00 \ud5c8\uc9c0\uc6c5\uc740 \u300a\uc778\ud130\uc2a4\ud154\ub77c\u300b\uc5d0 \ub300\ud574 \u201c\ub2e4\ub978 SF\uc18c\uc124\ub4e4\uc740 \uc778\uac04\uc774 \ud1b5\uc81c\ud560 \uc218 \uc5c6\uace0 \uac00\ub2a0\ud560 \uc218 \uc5c6\ub294 \uac70\ub300\ud55c \ubb38\uc81c\ub97c \ub9cc\ub0ac\uc744 \ub54c \uc678\uacc4 \ubb38\uba85\uc774 \uc640\uc11c \ub3c4\uc640\uc8fc\uac70\ub098 \uc778\uac04\uc774 \uc2e0\uc778\ub958\ub85c \uc9c4\ud654\ud558\uac70\ub098 \uadf8\ub7f0 \uc2dd\uc73c\ub85c \ud480\uc5c8\ub2e4\u201d\uace0 \uc124\uba85\ud558\uba74\uc11c \u201c\uc778\ud130\uc2a4\ud154\ub77c\ub294 \uac00\uc871\uc8fc\uc758\ub97c \ud1b5\ud574\uc11c \ucd08\uc6d4\uc131\uc744 \ud45c\ud604\ud55c\ub2e4. \uadf8\ub7f0 \uce21\uba74\uc5d0\uc11c \ud55c\uad6d \uad00\uac1d\uc5d0\uac8c \uc778\uae30\uac00 \uc788\ub294 \uac83 \uac19\ub2e4\u201d\ub77c\uace0 \ub9d0\ud588\ub2e4. \uc601\ud654\ud3c9\ub860\uac00 \uac15\uc775\ubaa8\ub294 \u201c\ud55c\uc2a4 \uc9d0\uba38\ub294 \ud06c\ub9ac\uc2a4\ud1a0\ud37c \ub180\ub7f0\uc758 \uc601\ud654 \ucca0\ud559\uc744 \uc74c\uc545\uc73c\ub85c \uac00\uc7a5 \uc798 \ud45c\ud604\ud574 \ub0b4\ub294 \uc0ac\ub78c\uc774\ub2e4. \uadf9\uc801\uc778 \uc74c\uc545 \ud6a8\uacfc\ub294 \uacfc\ud559 \uc601\ud654\uc778 \u2018\uc778\ud130\uc2a4\ud154\ub77c \uc18d \uc778\uac04\uc801\uc778 \uac10\uc815\uc744 \ub354\uc6b1 \ub04c\uc5b4\uc62c\ub824\uc8fc\ub294 \uc5ed\ud560\uc744 \ud55c\ub2e4\u201d\uba70 \u201c\ud2b9\ud788, \uc601\ud654 \ub9d0\ubbf8 5\ucc28\uc6d0 \ub3c4\uc11c\uad00 \uc7a5\uba74\uc5d0\uc11c \ud758\ub7ec\ub098\uc624\ub294 \uc74c\uc545\uc740 \uc8fc\uc778\uacf5 \ucfe0\ud37c\uc758 \uc808\uc808\ud55c \ubd80\uc131\uc560\ub97c \ub354\uc6b1 \ubd80\uac01\uc2dc\ucf1c\uc900\ub2e4\u201d\uace0 \ub9d0\ud588\ub2e4. \ubb38\ud654\ud3c9\ub860\uac00 \uae40\uc131\uc218\ub294 \u201c\ud06c\ub9ac\uc2a4\ud1a0\ud37c \ub180\ub7f0 \uac10\ub3c5\uc740 \uc544\ubc84\uc9c0\uac00 \ubb34\uc2a8 \uc77c\uc744 \ud574\uc57c \ud558\ub294\uc9c0\ub97c \uae68\ub2ec\uc740 \uac83 \uac19\ub2e4. \uae30\uc131\uc138\ub300\uac00 \ud560 \uc77c\uc744 \uc815\ud655\ud788 \uaddc\uc815\ud588\ub2e4. \ud55c\uad6d\uc758 \uae30\uc131\uc138\ub300\uc5d0\uac8c \uc774 \uc601\ud654\ub97c \ubcf4\uc5ec\uc8fc\uace0 \uc2f6\ub2e4. \uccad\ub144 \uc2dc\uc808 \uc815\uc758\ub97c \uc8fc\uc7a5\ud588\uc73c\ub098 \uc9c0\uae08\uc740 \uc0dd\uc874\uc774 \uc815\uc758\uac00 \ub41c \ubd80\ub044\ub7ec\uc6b4 \uc5b4\ub978\ub4e4\uc5d0\uac8c \ubcf4\uc5ec\uc8fc\uace0 \uc2f6\ub2e4\u201d\ub77c\uace0 \ub9d0\ud588\ub2e4. \uc11c\uc6b8SF\uc544\uce74\uc774\ube0c \ub300\ud45c\uc774\uc790 SF\ud3c9\ub860\uac00\uc774\uae30\ub3c4 \ud55c \ubc15\uc0c1\uc900\uc740 \"\uc804 \uc138\uacc4\uc801\uc73c\ub85c \ud765\ud589\ud55c SF \uc601\ud654 '\uadf8\ub798\ube44\ud2f0'\ub97c \uc911\u00b7\uace0\ub4f1\ud559\uc0dd\uc6a9\uc774\ub77c\uace0 \uce58\uba74, '\uc778\ud130\uc2a4\ud154\ub77c'\ub294 \ub300\ud559\uc0dd\u00b7\ub300\ud559\uc6d0\uc0dd\uc6a9 \ud558\ub4dc(hard) SF \uc601\ud654\ub2e4.\"\ub77c\uace0 \ub9d0\ud558\uba70 \"\uadf8\ub798\ube44\ud2f0\uac00 \ub274\ud134\uc758 \uc5ed\ud559, \uc989 \ub9cc\uc720\uc778\ub825\uc758 \ubc95\uce59\uc5d0 \ucda9\uc2e4\ud55c \uc791\ud488\uc774\uc5c8\ub2e4\uba74, \uc778\ud130\uc2a4\ud154\ub77c\ub294 \ub274\ud134 \uc774\ud6c4 \uacfc\ud559\uc758 \ud328\ub7ec\ub2e4\uc784\uc744 \ubc14\uafbc \uc544\uc778\uc288\ud0c0\uc778\uc758 \uc77c\ubc18\uc0c1\ub300\uc131 \uc774\ub860\uc744 \uc801\uc808\ud558\uac8c \ub179\uc5ec\ub0b4 SF \uc601\ud654\uc758 \uc9c0\ud3c9\uc744 \ub113\ud614\ub2e4\"\uace0 \ud3c9\uac00\ud588\ub2e4.", "answer": "\ubc15\uc0c1\uc900", "sentence": "\uc11c\uc6b8SF\uc544\uce74\uc774\ube0c \ub300\ud45c\uc774\uc790 SF\ud3c9\ub860\uac00\uc774\uae30\ub3c4 \ud55c \ubc15\uc0c1\uc900 \uc740 \"\uc804 \uc138\uacc4\uc801\uc73c\ub85c \ud765\ud589\ud55c SF \uc601\ud654 '\uadf8\ub798\ube44\ud2f0'\ub97c \uc911\u00b7\uace0\ub4f1\ud559\uc0dd\uc6a9\uc774\ub77c\uace0 \uce58\uba74, '\uc778\ud130\uc2a4\ud154\ub77c'\ub294 \ub300\ud559\uc0dd\u00b7\ub300\ud559\uc6d0\uc0dd\uc6a9 \ud558\ub4dc(hard) SF \uc601\ud654\ub2e4.", "paragraph_sentence": "\uc601\ud654\ud3c9\ub860\uac00 \uc774\ub3d9\uc9c4\uc740 \u201c\uc778\ud130\uc2a4\ud154\ub77c\ub294 \uc6b0\uc8fc\ub97c \uafc8\uafb8\uac8c \ud55c\ub2e4. \ub9cc\uc57d \ucc9c\uccb4\ubb3c\ub9ac\ud559\uc5d0 \uad00\uc2ec\uc774 \uc788\ub294 \uc911\ud559\uc0dd\uc774\ub77c\uba74 \uc774 \uc601\ud654\uac00 \uc778\uc0dd\uc744 \ubc14\uafc0 \uc218\ub3c4 \uc788\ub2e4\u201c\uace0 \ud3c9\uac00\ud588\ub2e4. \uc601\ud654\ud3c9\ub860\uac00 \ud5c8\uc9c0\uc6c5\uc740 \u300a\uc778\ud130\uc2a4\ud154\ub77c\u300b\uc5d0 \ub300\ud574 \u201c\ub2e4\ub978 SF\uc18c\uc124\ub4e4\uc740 \uc778\uac04\uc774 \ud1b5\uc81c\ud560 \uc218 \uc5c6\uace0 \uac00\ub2a0\ud560 \uc218 \uc5c6\ub294 \uac70\ub300\ud55c \ubb38\uc81c\ub97c \ub9cc\ub0ac\uc744 \ub54c \uc678\uacc4 \ubb38\uba85\uc774 \uc640\uc11c \ub3c4\uc640\uc8fc\uac70\ub098 \uc778\uac04\uc774 \uc2e0\uc778\ub958\ub85c \uc9c4\ud654\ud558\uac70\ub098 \uadf8\ub7f0 \uc2dd\uc73c\ub85c \ud480\uc5c8\ub2e4\u201d\uace0 \uc124\uba85\ud558\uba74\uc11c \u201c\uc778\ud130\uc2a4\ud154\ub77c\ub294 \uac00\uc871\uc8fc\uc758\ub97c \ud1b5\ud574\uc11c \ucd08\uc6d4\uc131\uc744 \ud45c\ud604\ud55c\ub2e4. \uadf8\ub7f0 \uce21\uba74\uc5d0\uc11c \ud55c\uad6d \uad00\uac1d\uc5d0\uac8c \uc778\uae30\uac00 \uc788\ub294 \uac83 \uac19\ub2e4\u201d\ub77c\uace0 \ub9d0\ud588\ub2e4. \uc601\ud654\ud3c9\ub860\uac00 \uac15\uc775\ubaa8\ub294 \u201c\ud55c\uc2a4 \uc9d0\uba38\ub294 \ud06c\ub9ac\uc2a4\ud1a0\ud37c \ub180\ub7f0\uc758 \uc601\ud654 \ucca0\ud559\uc744 \uc74c\uc545\uc73c\ub85c \uac00\uc7a5 \uc798 \ud45c\ud604\ud574 \ub0b4\ub294 \uc0ac\ub78c\uc774\ub2e4. \uadf9\uc801\uc778 \uc74c\uc545 \ud6a8\uacfc\ub294 \uacfc\ud559 \uc601\ud654\uc778 \u2018\uc778\ud130\uc2a4\ud154\ub77c \uc18d \uc778\uac04\uc801\uc778 \uac10\uc815\uc744 \ub354\uc6b1 \ub04c\uc5b4\uc62c\ub824\uc8fc\ub294 \uc5ed\ud560\uc744 \ud55c\ub2e4\u201d\uba70 \u201c\ud2b9\ud788, \uc601\ud654 \ub9d0\ubbf8 5\ucc28\uc6d0 \ub3c4\uc11c\uad00 \uc7a5\uba74\uc5d0\uc11c \ud758\ub7ec\ub098\uc624\ub294 \uc74c\uc545\uc740 \uc8fc\uc778\uacf5 \ucfe0\ud37c\uc758 \uc808\uc808\ud55c \ubd80\uc131\uc560\ub97c \ub354\uc6b1 \ubd80\uac01\uc2dc\ucf1c\uc900\ub2e4\u201d\uace0 \ub9d0\ud588\ub2e4. \ubb38\ud654\ud3c9\ub860\uac00 \uae40\uc131\uc218\ub294 \u201c\ud06c\ub9ac\uc2a4\ud1a0\ud37c \ub180\ub7f0 \uac10\ub3c5\uc740 \uc544\ubc84\uc9c0\uac00 \ubb34\uc2a8 \uc77c\uc744 \ud574\uc57c \ud558\ub294\uc9c0\ub97c \uae68\ub2ec\uc740 \uac83 \uac19\ub2e4. \uae30\uc131\uc138\ub300\uac00 \ud560 \uc77c\uc744 \uc815\ud655\ud788 \uaddc\uc815\ud588\ub2e4. \ud55c\uad6d\uc758 \uae30\uc131\uc138\ub300\uc5d0\uac8c \uc774 \uc601\ud654\ub97c \ubcf4\uc5ec\uc8fc\uace0 \uc2f6\ub2e4. \uccad\ub144 \uc2dc\uc808 \uc815\uc758\ub97c \uc8fc\uc7a5\ud588\uc73c\ub098 \uc9c0\uae08\uc740 \uc0dd\uc874\uc774 \uc815\uc758\uac00 \ub41c \ubd80\ub044\ub7ec\uc6b4 \uc5b4\ub978\ub4e4\uc5d0\uac8c \ubcf4\uc5ec\uc8fc\uace0 \uc2f6\ub2e4\u201d\ub77c\uace0 \ub9d0\ud588\ub2e4. \uc11c\uc6b8SF\uc544\uce74\uc774\ube0c \ub300\ud45c\uc774\uc790 SF\ud3c9\ub860\uac00\uc774\uae30\ub3c4 \ud55c \ubc15\uc0c1\uc900 \uc740 \"\uc804 \uc138\uacc4\uc801\uc73c\ub85c \ud765\ud589\ud55c SF \uc601\ud654 '\uadf8\ub798\ube44\ud2f0'\ub97c \uc911\u00b7\uace0\ub4f1\ud559\uc0dd\uc6a9\uc774\ub77c\uace0 \uce58\uba74, '\uc778\ud130\uc2a4\ud154\ub77c'\ub294 \ub300\ud559\uc0dd\u00b7\ub300\ud559\uc6d0\uc0dd\uc6a9 \ud558\ub4dc(hard) SF \uc601\ud654\ub2e4. \"\ub77c\uace0 \ub9d0\ud558\uba70 \"\uadf8\ub798\ube44\ud2f0\uac00 \ub274\ud134\uc758 \uc5ed\ud559, \uc989 \ub9cc\uc720\uc778\ub825\uc758 \ubc95\uce59\uc5d0 \ucda9\uc2e4\ud55c \uc791\ud488\uc774\uc5c8\ub2e4\uba74, \uc778\ud130\uc2a4\ud154\ub77c\ub294 \ub274\ud134 \uc774\ud6c4 \uacfc\ud559\uc758 \ud328\ub7ec\ub2e4\uc784\uc744 \ubc14\uafbc \uc544\uc778\uc288\ud0c0\uc778\uc758 \uc77c\ubc18\uc0c1\ub300\uc131 \uc774\ub860\uc744 \uc801\uc808\ud558\uac8c \ub179\uc5ec\ub0b4 SF \uc601\ud654\uc758 \uc9c0\ud3c9\uc744 \ub113\ud614\ub2e4\"\uace0 \ud3c9\uac00\ud588\ub2e4.", "paragraph_answer": "\uc601\ud654\ud3c9\ub860\uac00 \uc774\ub3d9\uc9c4\uc740 \u201c\uc778\ud130\uc2a4\ud154\ub77c\ub294 \uc6b0\uc8fc\ub97c \uafc8\uafb8\uac8c \ud55c\ub2e4. \ub9cc\uc57d \ucc9c\uccb4\ubb3c\ub9ac\ud559\uc5d0 \uad00\uc2ec\uc774 \uc788\ub294 \uc911\ud559\uc0dd\uc774\ub77c\uba74 \uc774 \uc601\ud654\uac00 \uc778\uc0dd\uc744 \ubc14\uafc0 \uc218\ub3c4 \uc788\ub2e4\u201c\uace0 \ud3c9\uac00\ud588\ub2e4. \uc601\ud654\ud3c9\ub860\uac00 \ud5c8\uc9c0\uc6c5\uc740 \u300a\uc778\ud130\uc2a4\ud154\ub77c\u300b\uc5d0 \ub300\ud574 \u201c\ub2e4\ub978 SF\uc18c\uc124\ub4e4\uc740 \uc778\uac04\uc774 \ud1b5\uc81c\ud560 \uc218 \uc5c6\uace0 \uac00\ub2a0\ud560 \uc218 \uc5c6\ub294 \uac70\ub300\ud55c \ubb38\uc81c\ub97c \ub9cc\ub0ac\uc744 \ub54c \uc678\uacc4 \ubb38\uba85\uc774 \uc640\uc11c \ub3c4\uc640\uc8fc\uac70\ub098 \uc778\uac04\uc774 \uc2e0\uc778\ub958\ub85c \uc9c4\ud654\ud558\uac70\ub098 \uadf8\ub7f0 \uc2dd\uc73c\ub85c \ud480\uc5c8\ub2e4\u201d\uace0 \uc124\uba85\ud558\uba74\uc11c \u201c\uc778\ud130\uc2a4\ud154\ub77c\ub294 \uac00\uc871\uc8fc\uc758\ub97c \ud1b5\ud574\uc11c \ucd08\uc6d4\uc131\uc744 \ud45c\ud604\ud55c\ub2e4. \uadf8\ub7f0 \uce21\uba74\uc5d0\uc11c \ud55c\uad6d \uad00\uac1d\uc5d0\uac8c \uc778\uae30\uac00 \uc788\ub294 \uac83 \uac19\ub2e4\u201d\ub77c\uace0 \ub9d0\ud588\ub2e4. \uc601\ud654\ud3c9\ub860\uac00 \uac15\uc775\ubaa8\ub294 \u201c\ud55c\uc2a4 \uc9d0\uba38\ub294 \ud06c\ub9ac\uc2a4\ud1a0\ud37c \ub180\ub7f0\uc758 \uc601\ud654 \ucca0\ud559\uc744 \uc74c\uc545\uc73c\ub85c \uac00\uc7a5 \uc798 \ud45c\ud604\ud574 \ub0b4\ub294 \uc0ac\ub78c\uc774\ub2e4. \uadf9\uc801\uc778 \uc74c\uc545 \ud6a8\uacfc\ub294 \uacfc\ud559 \uc601\ud654\uc778 \u2018\uc778\ud130\uc2a4\ud154\ub77c \uc18d \uc778\uac04\uc801\uc778 \uac10\uc815\uc744 \ub354\uc6b1 \ub04c\uc5b4\uc62c\ub824\uc8fc\ub294 \uc5ed\ud560\uc744 \ud55c\ub2e4\u201d\uba70 \u201c\ud2b9\ud788, \uc601\ud654 \ub9d0\ubbf8 5\ucc28\uc6d0 \ub3c4\uc11c\uad00 \uc7a5\uba74\uc5d0\uc11c \ud758\ub7ec\ub098\uc624\ub294 \uc74c\uc545\uc740 \uc8fc\uc778\uacf5 \ucfe0\ud37c\uc758 \uc808\uc808\ud55c \ubd80\uc131\uc560\ub97c \ub354\uc6b1 \ubd80\uac01\uc2dc\ucf1c\uc900\ub2e4\u201d\uace0 \ub9d0\ud588\ub2e4. \ubb38\ud654\ud3c9\ub860\uac00 \uae40\uc131\uc218\ub294 \u201c\ud06c\ub9ac\uc2a4\ud1a0\ud37c \ub180\ub7f0 \uac10\ub3c5\uc740 \uc544\ubc84\uc9c0\uac00 \ubb34\uc2a8 \uc77c\uc744 \ud574\uc57c \ud558\ub294\uc9c0\ub97c \uae68\ub2ec\uc740 \uac83 \uac19\ub2e4. \uae30\uc131\uc138\ub300\uac00 \ud560 \uc77c\uc744 \uc815\ud655\ud788 \uaddc\uc815\ud588\ub2e4. \ud55c\uad6d\uc758 \uae30\uc131\uc138\ub300\uc5d0\uac8c \uc774 \uc601\ud654\ub97c \ubcf4\uc5ec\uc8fc\uace0 \uc2f6\ub2e4. \uccad\ub144 \uc2dc\uc808 \uc815\uc758\ub97c \uc8fc\uc7a5\ud588\uc73c\ub098 \uc9c0\uae08\uc740 \uc0dd\uc874\uc774 \uc815\uc758\uac00 \ub41c \ubd80\ub044\ub7ec\uc6b4 \uc5b4\ub978\ub4e4\uc5d0\uac8c \ubcf4\uc5ec\uc8fc\uace0 \uc2f6\ub2e4\u201d\ub77c\uace0 \ub9d0\ud588\ub2e4. \uc11c\uc6b8SF\uc544\uce74\uc774\ube0c \ub300\ud45c\uc774\uc790 SF\ud3c9\ub860\uac00\uc774\uae30\ub3c4 \ud55c \ubc15\uc0c1\uc900 \uc740 \"\uc804 \uc138\uacc4\uc801\uc73c\ub85c \ud765\ud589\ud55c SF \uc601\ud654 '\uadf8\ub798\ube44\ud2f0'\ub97c \uc911\u00b7\uace0\ub4f1\ud559\uc0dd\uc6a9\uc774\ub77c\uace0 \uce58\uba74, '\uc778\ud130\uc2a4\ud154\ub77c'\ub294 \ub300\ud559\uc0dd\u00b7\ub300\ud559\uc6d0\uc0dd\uc6a9 \ud558\ub4dc(hard) SF \uc601\ud654\ub2e4.\"\ub77c\uace0 \ub9d0\ud558\uba70 \"\uadf8\ub798\ube44\ud2f0\uac00 \ub274\ud134\uc758 \uc5ed\ud559, \uc989 \ub9cc\uc720\uc778\ub825\uc758 \ubc95\uce59\uc5d0 \ucda9\uc2e4\ud55c \uc791\ud488\uc774\uc5c8\ub2e4\uba74, \uc778\ud130\uc2a4\ud154\ub77c\ub294 \ub274\ud134 \uc774\ud6c4 \uacfc\ud559\uc758 \ud328\ub7ec\ub2e4\uc784\uc744 \ubc14\uafbc \uc544\uc778\uc288\ud0c0\uc778\uc758 \uc77c\ubc18\uc0c1\ub300\uc131 \uc774\ub860\uc744 \uc801\uc808\ud558\uac8c \ub179\uc5ec\ub0b4 SF \uc601\ud654\uc758 \uc9c0\ud3c9\uc744 \ub113\ud614\ub2e4\"\uace0 \ud3c9\uac00\ud588\ub2e4.", "sentence_answer": "\uc11c\uc6b8SF\uc544\uce74\uc774\ube0c \ub300\ud45c\uc774\uc790 SF\ud3c9\ub860\uac00\uc774\uae30\ub3c4 \ud55c \ubc15\uc0c1\uc900 \uc740 \"\uc804 \uc138\uacc4\uc801\uc73c\ub85c \ud765\ud589\ud55c SF \uc601\ud654 '\uadf8\ub798\ube44\ud2f0'\ub97c \uc911\u00b7\uace0\ub4f1\ud559\uc0dd\uc6a9\uc774\ub77c\uace0 \uce58\uba74, '\uc778\ud130\uc2a4\ud154\ub77c'\ub294 \ub300\ud559\uc0dd\u00b7\ub300\ud559\uc6d0\uc0dd\uc6a9 \ud558\ub4dc(hard) SF \uc601\ud654\ub2e4.", "paragraph_id": "6548270-3-2", "paragraph_question": "question: \uc778\ud130\uc2a4\ud154\ub77c\ub294 \uc77c\ubc18\uc0c1\ub300\uc131\uc774\ub860\uc744 \uc801\uc808\ud558\uac8c \ub179\uc5ec\ub0b8 \uc601\ud654\ub77c\uace0 \ud3c9\uac00\ud55c \uc0ac\ub78c\uc740?, context: \uc601\ud654\ud3c9\ub860\uac00 \uc774\ub3d9\uc9c4\uc740 \u201c\uc778\ud130\uc2a4\ud154\ub77c\ub294 \uc6b0\uc8fc\ub97c \uafc8\uafb8\uac8c \ud55c\ub2e4. \ub9cc\uc57d \ucc9c\uccb4\ubb3c\ub9ac\ud559\uc5d0 \uad00\uc2ec\uc774 \uc788\ub294 \uc911\ud559\uc0dd\uc774\ub77c\uba74 \uc774 \uc601\ud654\uac00 \uc778\uc0dd\uc744 \ubc14\uafc0 \uc218\ub3c4 \uc788\ub2e4\u201c\uace0 \ud3c9\uac00\ud588\ub2e4. \uc601\ud654\ud3c9\ub860\uac00 \ud5c8\uc9c0\uc6c5\uc740 \u300a\uc778\ud130\uc2a4\ud154\ub77c\u300b\uc5d0 \ub300\ud574 \u201c\ub2e4\ub978 SF\uc18c\uc124\ub4e4\uc740 \uc778\uac04\uc774 \ud1b5\uc81c\ud560 \uc218 \uc5c6\uace0 \uac00\ub2a0\ud560 \uc218 \uc5c6\ub294 \uac70\ub300\ud55c \ubb38\uc81c\ub97c \ub9cc\ub0ac\uc744 \ub54c \uc678\uacc4 \ubb38\uba85\uc774 \uc640\uc11c \ub3c4\uc640\uc8fc\uac70\ub098 \uc778\uac04\uc774 \uc2e0\uc778\ub958\ub85c \uc9c4\ud654\ud558\uac70\ub098 \uadf8\ub7f0 \uc2dd\uc73c\ub85c \ud480\uc5c8\ub2e4\u201d\uace0 \uc124\uba85\ud558\uba74\uc11c \u201c\uc778\ud130\uc2a4\ud154\ub77c\ub294 \uac00\uc871\uc8fc\uc758\ub97c \ud1b5\ud574\uc11c \ucd08\uc6d4\uc131\uc744 \ud45c\ud604\ud55c\ub2e4. \uadf8\ub7f0 \uce21\uba74\uc5d0\uc11c \ud55c\uad6d \uad00\uac1d\uc5d0\uac8c \uc778\uae30\uac00 \uc788\ub294 \uac83 \uac19\ub2e4\u201d\ub77c\uace0 \ub9d0\ud588\ub2e4. \uc601\ud654\ud3c9\ub860\uac00 \uac15\uc775\ubaa8\ub294 \u201c\ud55c\uc2a4 \uc9d0\uba38\ub294 \ud06c\ub9ac\uc2a4\ud1a0\ud37c \ub180\ub7f0\uc758 \uc601\ud654 \ucca0\ud559\uc744 \uc74c\uc545\uc73c\ub85c \uac00\uc7a5 \uc798 \ud45c\ud604\ud574 \ub0b4\ub294 \uc0ac\ub78c\uc774\ub2e4. \uadf9\uc801\uc778 \uc74c\uc545 \ud6a8\uacfc\ub294 \uacfc\ud559 \uc601\ud654\uc778 \u2018\uc778\ud130\uc2a4\ud154\ub77c \uc18d \uc778\uac04\uc801\uc778 \uac10\uc815\uc744 \ub354\uc6b1 \ub04c\uc5b4\uc62c\ub824\uc8fc\ub294 \uc5ed\ud560\uc744 \ud55c\ub2e4\u201d\uba70 \u201c\ud2b9\ud788, \uc601\ud654 \ub9d0\ubbf8 5\ucc28\uc6d0 \ub3c4\uc11c\uad00 \uc7a5\uba74\uc5d0\uc11c \ud758\ub7ec\ub098\uc624\ub294 \uc74c\uc545\uc740 \uc8fc\uc778\uacf5 \ucfe0\ud37c\uc758 \uc808\uc808\ud55c \ubd80\uc131\uc560\ub97c \ub354\uc6b1 \ubd80\uac01\uc2dc\ucf1c\uc900\ub2e4\u201d\uace0 \ub9d0\ud588\ub2e4. \ubb38\ud654\ud3c9\ub860\uac00 \uae40\uc131\uc218\ub294 \u201c\ud06c\ub9ac\uc2a4\ud1a0\ud37c \ub180\ub7f0 \uac10\ub3c5\uc740 \uc544\ubc84\uc9c0\uac00 \ubb34\uc2a8 \uc77c\uc744 \ud574\uc57c \ud558\ub294\uc9c0\ub97c \uae68\ub2ec\uc740 \uac83 \uac19\ub2e4. \uae30\uc131\uc138\ub300\uac00 \ud560 \uc77c\uc744 \uc815\ud655\ud788 \uaddc\uc815\ud588\ub2e4. \ud55c\uad6d\uc758 \uae30\uc131\uc138\ub300\uc5d0\uac8c \uc774 \uc601\ud654\ub97c \ubcf4\uc5ec\uc8fc\uace0 \uc2f6\ub2e4. \uccad\ub144 \uc2dc\uc808 \uc815\uc758\ub97c \uc8fc\uc7a5\ud588\uc73c\ub098 \uc9c0\uae08\uc740 \uc0dd\uc874\uc774 \uc815\uc758\uac00 \ub41c \ubd80\ub044\ub7ec\uc6b4 \uc5b4\ub978\ub4e4\uc5d0\uac8c \ubcf4\uc5ec\uc8fc\uace0 \uc2f6\ub2e4\u201d\ub77c\uace0 \ub9d0\ud588\ub2e4. \uc11c\uc6b8SF\uc544\uce74\uc774\ube0c \ub300\ud45c\uc774\uc790 SF\ud3c9\ub860\uac00\uc774\uae30\ub3c4 \ud55c \ubc15\uc0c1\uc900\uc740 \"\uc804 \uc138\uacc4\uc801\uc73c\ub85c \ud765\ud589\ud55c SF \uc601\ud654 '\uadf8\ub798\ube44\ud2f0'\ub97c \uc911\u00b7\uace0\ub4f1\ud559\uc0dd\uc6a9\uc774\ub77c\uace0 \uce58\uba74, '\uc778\ud130\uc2a4\ud154\ub77c'\ub294 \ub300\ud559\uc0dd\u00b7\ub300\ud559\uc6d0\uc0dd\uc6a9 \ud558\ub4dc(hard) SF \uc601\ud654\ub2e4.\"\ub77c\uace0 \ub9d0\ud558\uba70 \"\uadf8\ub798\ube44\ud2f0\uac00 \ub274\ud134\uc758 \uc5ed\ud559, \uc989 \ub9cc\uc720\uc778\ub825\uc758 \ubc95\uce59\uc5d0 \ucda9\uc2e4\ud55c \uc791\ud488\uc774\uc5c8\ub2e4\uba74, \uc778\ud130\uc2a4\ud154\ub77c\ub294 \ub274\ud134 \uc774\ud6c4 \uacfc\ud559\uc758 \ud328\ub7ec\ub2e4\uc784\uc744 \ubc14\uafbc \uc544\uc778\uc288\ud0c0\uc778\uc758 \uc77c\ubc18\uc0c1\ub300\uc131 \uc774\ub860\uc744 \uc801\uc808\ud558\uac8c \ub179\uc5ec\ub0b4 SF \uc601\ud654\uc758 \uc9c0\ud3c9\uc744 \ub113\ud614\ub2e4\"\uace0 \ud3c9\uac00\ud588\ub2e4."} {"question": "\ube44\ud2b8\uac90\uc288\ud0c0\uc778\uc758 \ud615\uc774\uc790 \ud53c\uc544\ub2c8\uc2a4\ud2b8\uc600\ub358 \uc0ac\ub78c\uc740?", "paragraph": "\ube44\ud2b8\uac90\uc288\ud0c0\uc778\uc758 \uc9d1\uc5d0\ub294 \uc694\ud558\ub124\uc2a4 \ube0c\ub78c\uc2a4, \ub9ac\ud558\ub974\ud2b8 \uc288\ud2b8\ub77c\uc6b0\uc2a4, \uad6c\uc2a4\ud0c0\ud504 \ub9d0\ub7ec \uac19\uc740 \uc608\uc220\uac00\uac00 \ub298 \ucd08\ub300\ub418\uc5c8\ub2e4. \uc694\ud558\ub124\uc2a4 \ube0c\ub78c\uc2a4\ub294 \ube44\ud2b8\uac90\uc288\ud0c0\uc778\ubcf4\ub2e4 \ub450 \uc0b4 \uc704\uc758 \ub204\ub098\uc5d0\uac8c \ud53c\uc544\ub178\ub97c \uac00\ub974\ucce4\uace0 \uc694\ud558\ub124\uc2a4 \ube0c\ub78c\uc2a4\uc758 \uc8fc\uc694\ud55c \uc791\ud488 \uac00\uc6b4\ub370 \uba87\uba87\uc744 \ube44\ud2b8\uac90\uc288\ud0c0\uc778 \uc9d1\uc758 \uc74c\uc545\uc2e4\uc5d0\uc11c \ucd08\uc5f0\ud558\uc600\ub2e4. \ube44\ud2b8\uac90\uc288\ud0c0\uc778\uc758 \ud615 \ud30c\uc6b8 \ube44\ud2b8\uac90\uc288\ud0c0\uc778\uc740 \ud6c4\uc77c \uc138\uacc4\uc801\uc73c\ub85c \uc720\uba85\ud55c \ud53c\uc544\ub2c8\uc2a4\ud2b8\uac00 \ub418\uc5c8\ub2e4. \ud30c\uc6b8\uc740 \uc81c1\ucc28 \uc138\uacc4\ub300\uc804 \ucc38\uc804 \uc911 \uc624\ub978\ud314\uc744 \uc783\uace0 \ub7ec\uc2dc\uc544\uc5d0\uc11c \ud3ec\ub85c\uc218\uc6a9\uc18c\uc5d0 \uac07\ud600\uc11c\ub3c4 \uc67c\uc190\ub9cc\uc73c\ub85c \ud53c\uc544\ub178\ub97c \uc5f0\uc2b5\ud558\uc5ec \uba85\uc131\uc744 \uc720\uc9c0\ud558\uc600\ub2e4. \ube44\ud2b8\uac90\uc288\ud0c0\uc778\ub3c4 \uc808\ub300\uc74c\uac10\uc774 \uc788\uc5c8\uace0, \uc74c\uc545\uc744 \ud5a5\ud55c \uc5f4\uc815\uc740 \ube44\ud2b8\uac90\uc288\ud0c0\uc778\uc758 \uc0dd\ud65c\uacfc \uc0ac\uc0c1\uc5d0 \ud070 \uc601\ud5a5\uc744 \uc8fc\uc5c8\ub2e4. \ube44\ud2b8\uac90\uc288\ud0c0\uc778\uc740 \uc790\uc2e0\uc758 \ucca0\ud559 \uc800\uc791\uc5d0\uc11c\ub3c4 \uc74c\uc545\uc5d0 \ube57\ub304 \uc608\ub97c \uc790\uc8fc \uc0ac\uc6a9\ud558\uc600\uace0 \ud074\ub77c\ub9ac\ub137\uc744 \ub2a5\uc219\ud558\uac8c \uc5f0\uc8fc\ud558\uc5ec \uc624\ucf00\uc2a4\ud2b8\ub77c\uc640 \ud611\uc5f0\ud558\uae30\ub3c4 \ud558\uc600\ub2e4. \ub8e8\ud2b8\ube44\ud788 \uc694\uc81c\ud504 \uc694\ud55c \ube44\ud2b8\uac90\uc288\ud0c0\uc778\uc740 \uad50\ud5a5\uace1 \uc804\uccb4\ub97c \uc678\uc6cc \ud718\ud30c\ub78c\uc73c\ub85c \ubd88\ub800\ub2e4.", "answer": "\ud30c\uc6b8 \ube44\ud2b8\uac90\uc288\ud0c0\uc778", "sentence": "\ube44\ud2b8\uac90\uc288\ud0c0\uc778\uc758 \ud615 \ud30c\uc6b8 \ube44\ud2b8\uac90\uc288\ud0c0\uc778 \uc740 \ud6c4\uc77c \uc138\uacc4\uc801\uc73c\ub85c \uc720\uba85\ud55c \ud53c\uc544\ub2c8\uc2a4\ud2b8\uac00 \ub418\uc5c8\ub2e4.", "paragraph_sentence": "\ube44\ud2b8\uac90\uc288\ud0c0\uc778\uc758 \uc9d1\uc5d0\ub294 \uc694\ud558\ub124\uc2a4 \ube0c\ub78c\uc2a4, \ub9ac\ud558\ub974\ud2b8 \uc288\ud2b8\ub77c\uc6b0\uc2a4, \uad6c\uc2a4\ud0c0\ud504 \ub9d0\ub7ec \uac19\uc740 \uc608\uc220\uac00\uac00 \ub298 \ucd08\ub300\ub418\uc5c8\ub2e4. \uc694\ud558\ub124\uc2a4 \ube0c\ub78c\uc2a4\ub294 \ube44\ud2b8\uac90\uc288\ud0c0\uc778\ubcf4\ub2e4 \ub450 \uc0b4 \uc704\uc758 \ub204\ub098\uc5d0\uac8c \ud53c\uc544\ub178\ub97c \uac00\ub974\ucce4\uace0 \uc694\ud558\ub124\uc2a4 \ube0c\ub78c\uc2a4\uc758 \uc8fc\uc694\ud55c \uc791\ud488 \uac00\uc6b4\ub370 \uba87\uba87\uc744 \ube44\ud2b8\uac90\uc288\ud0c0\uc778 \uc9d1\uc758 \uc74c\uc545\uc2e4\uc5d0\uc11c \ucd08\uc5f0\ud558\uc600\ub2e4. \ube44\ud2b8\uac90\uc288\ud0c0\uc778\uc758 \ud615 \ud30c\uc6b8 \ube44\ud2b8\uac90\uc288\ud0c0\uc778 \uc740 \ud6c4\uc77c \uc138\uacc4\uc801\uc73c\ub85c \uc720\uba85\ud55c \ud53c\uc544\ub2c8\uc2a4\ud2b8\uac00 \ub418\uc5c8\ub2e4. \ud30c\uc6b8\uc740 \uc81c1\ucc28 \uc138\uacc4\ub300\uc804 \ucc38\uc804 \uc911 \uc624\ub978\ud314\uc744 \uc783\uace0 \ub7ec\uc2dc\uc544\uc5d0\uc11c \ud3ec\ub85c\uc218\uc6a9\uc18c\uc5d0 \uac07\ud600\uc11c\ub3c4 \uc67c\uc190\ub9cc\uc73c\ub85c \ud53c\uc544\ub178\ub97c \uc5f0\uc2b5\ud558\uc5ec \uba85\uc131\uc744 \uc720\uc9c0\ud558\uc600\ub2e4. \ube44\ud2b8\uac90\uc288\ud0c0\uc778\ub3c4 \uc808\ub300\uc74c\uac10\uc774 \uc788\uc5c8\uace0, \uc74c\uc545\uc744 \ud5a5\ud55c \uc5f4\uc815\uc740 \ube44\ud2b8\uac90\uc288\ud0c0\uc778\uc758 \uc0dd\ud65c\uacfc \uc0ac\uc0c1\uc5d0 \ud070 \uc601\ud5a5\uc744 \uc8fc\uc5c8\ub2e4. \ube44\ud2b8\uac90\uc288\ud0c0\uc778\uc740 \uc790\uc2e0\uc758 \ucca0\ud559 \uc800\uc791\uc5d0\uc11c\ub3c4 \uc74c\uc545\uc5d0 \ube57\ub304 \uc608\ub97c \uc790\uc8fc \uc0ac\uc6a9\ud558\uc600\uace0 \ud074\ub77c\ub9ac\ub137\uc744 \ub2a5\uc219\ud558\uac8c \uc5f0\uc8fc\ud558\uc5ec \uc624\ucf00\uc2a4\ud2b8\ub77c\uc640 \ud611\uc5f0\ud558\uae30\ub3c4 \ud558\uc600\ub2e4. \ub8e8\ud2b8\ube44\ud788 \uc694\uc81c\ud504 \uc694\ud55c \ube44\ud2b8\uac90\uc288\ud0c0\uc778\uc740 \uad50\ud5a5\uace1 \uc804\uccb4\ub97c \uc678\uc6cc \ud718\ud30c\ub78c\uc73c\ub85c \ubd88\ub800\ub2e4.", "paragraph_answer": "\ube44\ud2b8\uac90\uc288\ud0c0\uc778\uc758 \uc9d1\uc5d0\ub294 \uc694\ud558\ub124\uc2a4 \ube0c\ub78c\uc2a4, \ub9ac\ud558\ub974\ud2b8 \uc288\ud2b8\ub77c\uc6b0\uc2a4, \uad6c\uc2a4\ud0c0\ud504 \ub9d0\ub7ec \uac19\uc740 \uc608\uc220\uac00\uac00 \ub298 \ucd08\ub300\ub418\uc5c8\ub2e4. \uc694\ud558\ub124\uc2a4 \ube0c\ub78c\uc2a4\ub294 \ube44\ud2b8\uac90\uc288\ud0c0\uc778\ubcf4\ub2e4 \ub450 \uc0b4 \uc704\uc758 \ub204\ub098\uc5d0\uac8c \ud53c\uc544\ub178\ub97c \uac00\ub974\ucce4\uace0 \uc694\ud558\ub124\uc2a4 \ube0c\ub78c\uc2a4\uc758 \uc8fc\uc694\ud55c \uc791\ud488 \uac00\uc6b4\ub370 \uba87\uba87\uc744 \ube44\ud2b8\uac90\uc288\ud0c0\uc778 \uc9d1\uc758 \uc74c\uc545\uc2e4\uc5d0\uc11c \ucd08\uc5f0\ud558\uc600\ub2e4. \ube44\ud2b8\uac90\uc288\ud0c0\uc778\uc758 \ud615 \ud30c\uc6b8 \ube44\ud2b8\uac90\uc288\ud0c0\uc778 \uc740 \ud6c4\uc77c \uc138\uacc4\uc801\uc73c\ub85c \uc720\uba85\ud55c \ud53c\uc544\ub2c8\uc2a4\ud2b8\uac00 \ub418\uc5c8\ub2e4. \ud30c\uc6b8\uc740 \uc81c1\ucc28 \uc138\uacc4\ub300\uc804 \ucc38\uc804 \uc911 \uc624\ub978\ud314\uc744 \uc783\uace0 \ub7ec\uc2dc\uc544\uc5d0\uc11c \ud3ec\ub85c\uc218\uc6a9\uc18c\uc5d0 \uac07\ud600\uc11c\ub3c4 \uc67c\uc190\ub9cc\uc73c\ub85c \ud53c\uc544\ub178\ub97c \uc5f0\uc2b5\ud558\uc5ec \uba85\uc131\uc744 \uc720\uc9c0\ud558\uc600\ub2e4. \ube44\ud2b8\uac90\uc288\ud0c0\uc778\ub3c4 \uc808\ub300\uc74c\uac10\uc774 \uc788\uc5c8\uace0, \uc74c\uc545\uc744 \ud5a5\ud55c \uc5f4\uc815\uc740 \ube44\ud2b8\uac90\uc288\ud0c0\uc778\uc758 \uc0dd\ud65c\uacfc \uc0ac\uc0c1\uc5d0 \ud070 \uc601\ud5a5\uc744 \uc8fc\uc5c8\ub2e4. \ube44\ud2b8\uac90\uc288\ud0c0\uc778\uc740 \uc790\uc2e0\uc758 \ucca0\ud559 \uc800\uc791\uc5d0\uc11c\ub3c4 \uc74c\uc545\uc5d0 \ube57\ub304 \uc608\ub97c \uc790\uc8fc \uc0ac\uc6a9\ud558\uc600\uace0 \ud074\ub77c\ub9ac\ub137\uc744 \ub2a5\uc219\ud558\uac8c \uc5f0\uc8fc\ud558\uc5ec \uc624\ucf00\uc2a4\ud2b8\ub77c\uc640 \ud611\uc5f0\ud558\uae30\ub3c4 \ud558\uc600\ub2e4. \ub8e8\ud2b8\ube44\ud788 \uc694\uc81c\ud504 \uc694\ud55c \ube44\ud2b8\uac90\uc288\ud0c0\uc778\uc740 \uad50\ud5a5\uace1 \uc804\uccb4\ub97c \uc678\uc6cc \ud718\ud30c\ub78c\uc73c\ub85c \ubd88\ub800\ub2e4.", "sentence_answer": "\ube44\ud2b8\uac90\uc288\ud0c0\uc778\uc758 \ud615 \ud30c\uc6b8 \ube44\ud2b8\uac90\uc288\ud0c0\uc778 \uc740 \ud6c4\uc77c \uc138\uacc4\uc801\uc73c\ub85c \uc720\uba85\ud55c \ud53c\uc544\ub2c8\uc2a4\ud2b8\uac00 \ub418\uc5c8\ub2e4.", "paragraph_id": "6497654-2-0", "paragraph_question": "question: \ube44\ud2b8\uac90\uc288\ud0c0\uc778\uc758 \ud615\uc774\uc790 \ud53c\uc544\ub2c8\uc2a4\ud2b8\uc600\ub358 \uc0ac\ub78c\uc740?, context: \ube44\ud2b8\uac90\uc288\ud0c0\uc778\uc758 \uc9d1\uc5d0\ub294 \uc694\ud558\ub124\uc2a4 \ube0c\ub78c\uc2a4, \ub9ac\ud558\ub974\ud2b8 \uc288\ud2b8\ub77c\uc6b0\uc2a4, \uad6c\uc2a4\ud0c0\ud504 \ub9d0\ub7ec \uac19\uc740 \uc608\uc220\uac00\uac00 \ub298 \ucd08\ub300\ub418\uc5c8\ub2e4. \uc694\ud558\ub124\uc2a4 \ube0c\ub78c\uc2a4\ub294 \ube44\ud2b8\uac90\uc288\ud0c0\uc778\ubcf4\ub2e4 \ub450 \uc0b4 \uc704\uc758 \ub204\ub098\uc5d0\uac8c \ud53c\uc544\ub178\ub97c \uac00\ub974\ucce4\uace0 \uc694\ud558\ub124\uc2a4 \ube0c\ub78c\uc2a4\uc758 \uc8fc\uc694\ud55c \uc791\ud488 \uac00\uc6b4\ub370 \uba87\uba87\uc744 \ube44\ud2b8\uac90\uc288\ud0c0\uc778 \uc9d1\uc758 \uc74c\uc545\uc2e4\uc5d0\uc11c \ucd08\uc5f0\ud558\uc600\ub2e4. \ube44\ud2b8\uac90\uc288\ud0c0\uc778\uc758 \ud615 \ud30c\uc6b8 \ube44\ud2b8\uac90\uc288\ud0c0\uc778\uc740 \ud6c4\uc77c \uc138\uacc4\uc801\uc73c\ub85c \uc720\uba85\ud55c \ud53c\uc544\ub2c8\uc2a4\ud2b8\uac00 \ub418\uc5c8\ub2e4. \ud30c\uc6b8\uc740 \uc81c1\ucc28 \uc138\uacc4\ub300\uc804 \ucc38\uc804 \uc911 \uc624\ub978\ud314\uc744 \uc783\uace0 \ub7ec\uc2dc\uc544\uc5d0\uc11c \ud3ec\ub85c\uc218\uc6a9\uc18c\uc5d0 \uac07\ud600\uc11c\ub3c4 \uc67c\uc190\ub9cc\uc73c\ub85c \ud53c\uc544\ub178\ub97c \uc5f0\uc2b5\ud558\uc5ec \uba85\uc131\uc744 \uc720\uc9c0\ud558\uc600\ub2e4. \ube44\ud2b8\uac90\uc288\ud0c0\uc778\ub3c4 \uc808\ub300\uc74c\uac10\uc774 \uc788\uc5c8\uace0, \uc74c\uc545\uc744 \ud5a5\ud55c \uc5f4\uc815\uc740 \ube44\ud2b8\uac90\uc288\ud0c0\uc778\uc758 \uc0dd\ud65c\uacfc \uc0ac\uc0c1\uc5d0 \ud070 \uc601\ud5a5\uc744 \uc8fc\uc5c8\ub2e4. \ube44\ud2b8\uac90\uc288\ud0c0\uc778\uc740 \uc790\uc2e0\uc758 \ucca0\ud559 \uc800\uc791\uc5d0\uc11c\ub3c4 \uc74c\uc545\uc5d0 \ube57\ub304 \uc608\ub97c \uc790\uc8fc \uc0ac\uc6a9\ud558\uc600\uace0 \ud074\ub77c\ub9ac\ub137\uc744 \ub2a5\uc219\ud558\uac8c \uc5f0\uc8fc\ud558\uc5ec \uc624\ucf00\uc2a4\ud2b8\ub77c\uc640 \ud611\uc5f0\ud558\uae30\ub3c4 \ud558\uc600\ub2e4. \ub8e8\ud2b8\ube44\ud788 \uc694\uc81c\ud504 \uc694\ud55c \ube44\ud2b8\uac90\uc288\ud0c0\uc778\uc740 \uad50\ud5a5\uace1 \uc804\uccb4\ub97c \uc678\uc6cc \ud718\ud30c\ub78c\uc73c\ub85c \ubd88\ub800\ub2e4."} {"question": "1985\ub144 \ucd1d\uc120\uc740 \uba87 \uc6d4\uc5d0 \uce58\ub8e8\uc5b4\uc84c\ub294\uac00?", "paragraph": "\uc5d0\ub4dc\uc6cc\ub4dc \ucf00\ub124\ub514 \ubbf8\uad6d \uc0c1\uc6d0 \uc758\uc6d0\uc740 71\ub144 \ub2f9\uc2dc \uc2e0\ubbfc\ub2f9 \ub300\uc120\ud6c4\ubcf4\ub85c \ubbf8\uad6d\uc744 \ubc29\ubb38\ud55c \uae40\ub300\uc911\uc5d0 \ub300\ud574 \"\ub2f9\uc2e0\uc740 \ud55c\uad6d\uc758 \uc874 F. \ucf00\ub124\ub514\"\ub77c\uba70 \uc9c0\uc9c0\ub97c \ud45c\uba85\ud588\ub2e4. \uadf8\ub294 1980\ub144 \uae40\ub300\uc911\uc774 \uc2e0\uad70\ubd80\uc5d0 \uc758\ud574 \uc0ac\ud615\uc120\uace0\ub97c \ubc1b\uc790 \uad6c\uba85\uc6b4\ub3d9\uc5d0 \uc55e\uc7a5\uc130\uace0 \ubbf8\uad6d \ub9dd\uba85 \uc0dd\ud65c\uacfc \uadc0\uad6d \ub54c\ub3c4 \ud070 \ub3c4\uc6c0\uc744 \uc92c\ub2e4. \ucf00\ub124\ub514 \uc758\uc6d0\uc740 1984\ub144 4\uc6d4 26\uc77c \uae40\ub300\uc911\uc5d0\uac8c \ubcf4\ub0b8 \ud3b8\uc9c0\uc5d0\uc11c \"\ub2f9\uc2e0\uacfc \ub098\ub294 \ubbfc\uc8fc\uc640 \ud3c9\ud654\ub97c \uc9c0\uc9c0\ud558\ub294 \uc785\uc7a5\uc744 \uacf5\uc720\ud558\uace0 \uc788\ub2e4. \ud55c\uad6d\uc758 \ud3c9\ud654\uc801 \uc815\uad8c\uad50\uccb4\ub97c \uc9c0\uc9c0\ud55c\ub2e4\"\ub77c\uace0 \uc37c\ub2e4. 85\ub144 2\u00b712 \ucd1d\uc120 \uc804\uc5d0 \uadc0\uad6d\uc744 \uacb0\uc2ec\ud55c \uae40\ub300\uc911\uc740 1984\ub144 6\uc6d4 \"84\ub144 \ub9d0\uae4c\uc9c0 \uadc0\uad6d\ud558\uae30\ub85c \uacb0\uc815\ud588\ub2e4\"\ub77c\ub294 \ub73b\uc744 \uc11c\uc2e0\uc5d0 \ub2f4\uc544 \uc804\ud588\ub2e4. \ucf00\ub124\ub514 \uc758\uc6d0\uc740 86\ub144 5\uc6d4 \ud55c\uad6d\uc5d0\uc11c \ubbfc\uc8fc\ud654\ub97c \uc704\ud55c \uae40\ub300\uc911\uc758 \ub178\ub825\uc5d0 \uacbd\uc758\ub97c \ud45c\ud55c\ub2e4\ub294 \ub0b4\uc6a9\uc758 \ud3b8\uc9c0\ub97c \ubcf4\ub0c8\ub2e4.", "answer": "2", "sentence": "\ucf00\ub124\ub514 \uc758\uc6d0\uc740 1984\ub144 4\uc6d4 2 6\uc77c \uae40\ub300\uc911\uc5d0\uac8c \ubcf4\ub0b8 \ud3b8\uc9c0\uc5d0\uc11c \"\ub2f9\uc2e0\uacfc \ub098\ub294 \ubbfc\uc8fc\uc640 \ud3c9\ud654\ub97c \uc9c0\uc9c0\ud558\ub294 \uc785\uc7a5\uc744 \uacf5\uc720\ud558\uace0 \uc788\ub2e4.", "paragraph_sentence": "\uc5d0\ub4dc\uc6cc\ub4dc \ucf00\ub124\ub514 \ubbf8\uad6d \uc0c1\uc6d0 \uc758\uc6d0\uc740 71\ub144 \ub2f9\uc2dc \uc2e0\ubbfc\ub2f9 \ub300\uc120\ud6c4\ubcf4\ub85c \ubbf8\uad6d\uc744 \ubc29\ubb38\ud55c \uae40\ub300\uc911\uc5d0 \ub300\ud574 \"\ub2f9\uc2e0\uc740 \ud55c\uad6d\uc758 \uc874 F. \ucf00\ub124\ub514\"\ub77c\uba70 \uc9c0\uc9c0\ub97c \ud45c\uba85\ud588\ub2e4. \uadf8\ub294 1980\ub144 \uae40\ub300\uc911\uc774 \uc2e0\uad70\ubd80\uc5d0 \uc758\ud574 \uc0ac\ud615\uc120\uace0\ub97c \ubc1b\uc790 \uad6c\uba85\uc6b4\ub3d9\uc5d0 \uc55e\uc7a5\uc130\uace0 \ubbf8\uad6d \ub9dd\uba85 \uc0dd\ud65c\uacfc \uadc0\uad6d \ub54c\ub3c4 \ud070 \ub3c4\uc6c0\uc744 \uc92c\ub2e4. \ucf00\ub124\ub514 \uc758\uc6d0\uc740 1984\ub144 4\uc6d4 2 6\uc77c \uae40\ub300\uc911\uc5d0\uac8c \ubcf4\ub0b8 \ud3b8\uc9c0\uc5d0\uc11c \"\ub2f9\uc2e0\uacfc \ub098\ub294 \ubbfc\uc8fc\uc640 \ud3c9\ud654\ub97c \uc9c0\uc9c0\ud558\ub294 \uc785\uc7a5\uc744 \uacf5\uc720\ud558\uace0 \uc788\ub2e4. \ud55c\uad6d\uc758 \ud3c9\ud654\uc801 \uc815\uad8c\uad50\uccb4\ub97c \uc9c0\uc9c0\ud55c\ub2e4\"\ub77c\uace0 \uc37c\ub2e4. 85\ub144 2\u00b712 \ucd1d\uc120 \uc804\uc5d0 \uadc0\uad6d\uc744 \uacb0\uc2ec\ud55c \uae40\ub300\uc911\uc740 1984\ub144 6\uc6d4 \"84\ub144 \ub9d0\uae4c\uc9c0 \uadc0\uad6d\ud558\uae30\ub85c \uacb0\uc815\ud588\ub2e4\"\ub77c\ub294 \ub73b\uc744 \uc11c\uc2e0\uc5d0 \ub2f4\uc544 \uc804\ud588\ub2e4. \ucf00\ub124\ub514 \uc758\uc6d0\uc740 86\ub144 5\uc6d4 \ud55c\uad6d\uc5d0\uc11c \ubbfc\uc8fc\ud654\ub97c \uc704\ud55c \uae40\ub300\uc911\uc758 \ub178\ub825\uc5d0 \uacbd\uc758\ub97c \ud45c\ud55c\ub2e4\ub294 \ub0b4\uc6a9\uc758 \ud3b8\uc9c0\ub97c \ubcf4\ub0c8\ub2e4.", "paragraph_answer": "\uc5d0\ub4dc\uc6cc\ub4dc \ucf00\ub124\ub514 \ubbf8\uad6d \uc0c1\uc6d0 \uc758\uc6d0\uc740 71\ub144 \ub2f9\uc2dc \uc2e0\ubbfc\ub2f9 \ub300\uc120\ud6c4\ubcf4\ub85c \ubbf8\uad6d\uc744 \ubc29\ubb38\ud55c \uae40\ub300\uc911\uc5d0 \ub300\ud574 \"\ub2f9\uc2e0\uc740 \ud55c\uad6d\uc758 \uc874 F. \ucf00\ub124\ub514\"\ub77c\uba70 \uc9c0\uc9c0\ub97c \ud45c\uba85\ud588\ub2e4. \uadf8\ub294 1980\ub144 \uae40\ub300\uc911\uc774 \uc2e0\uad70\ubd80\uc5d0 \uc758\ud574 \uc0ac\ud615\uc120\uace0\ub97c \ubc1b\uc790 \uad6c\uba85\uc6b4\ub3d9\uc5d0 \uc55e\uc7a5\uc130\uace0 \ubbf8\uad6d \ub9dd\uba85 \uc0dd\ud65c\uacfc \uadc0\uad6d \ub54c\ub3c4 \ud070 \ub3c4\uc6c0\uc744 \uc92c\ub2e4. \ucf00\ub124\ub514 \uc758\uc6d0\uc740 1984\ub144 4\uc6d4 2 6\uc77c \uae40\ub300\uc911\uc5d0\uac8c \ubcf4\ub0b8 \ud3b8\uc9c0\uc5d0\uc11c \"\ub2f9\uc2e0\uacfc \ub098\ub294 \ubbfc\uc8fc\uc640 \ud3c9\ud654\ub97c \uc9c0\uc9c0\ud558\ub294 \uc785\uc7a5\uc744 \uacf5\uc720\ud558\uace0 \uc788\ub2e4. \ud55c\uad6d\uc758 \ud3c9\ud654\uc801 \uc815\uad8c\uad50\uccb4\ub97c \uc9c0\uc9c0\ud55c\ub2e4\"\ub77c\uace0 \uc37c\ub2e4. 85\ub144 2\u00b712 \ucd1d\uc120 \uc804\uc5d0 \uadc0\uad6d\uc744 \uacb0\uc2ec\ud55c \uae40\ub300\uc911\uc740 1984\ub144 6\uc6d4 \"84\ub144 \ub9d0\uae4c\uc9c0 \uadc0\uad6d\ud558\uae30\ub85c \uacb0\uc815\ud588\ub2e4\"\ub77c\ub294 \ub73b\uc744 \uc11c\uc2e0\uc5d0 \ub2f4\uc544 \uc804\ud588\ub2e4. \ucf00\ub124\ub514 \uc758\uc6d0\uc740 86\ub144 5\uc6d4 \ud55c\uad6d\uc5d0\uc11c \ubbfc\uc8fc\ud654\ub97c \uc704\ud55c \uae40\ub300\uc911\uc758 \ub178\ub825\uc5d0 \uacbd\uc758\ub97c \ud45c\ud55c\ub2e4\ub294 \ub0b4\uc6a9\uc758 \ud3b8\uc9c0\ub97c \ubcf4\ub0c8\ub2e4.", "sentence_answer": "\ucf00\ub124\ub514 \uc758\uc6d0\uc740 1984\ub144 4\uc6d4 2 6\uc77c \uae40\ub300\uc911\uc5d0\uac8c \ubcf4\ub0b8 \ud3b8\uc9c0\uc5d0\uc11c \"\ub2f9\uc2e0\uacfc \ub098\ub294 \ubbfc\uc8fc\uc640 \ud3c9\ud654\ub97c \uc9c0\uc9c0\ud558\ub294 \uc785\uc7a5\uc744 \uacf5\uc720\ud558\uace0 \uc788\ub2e4.", "paragraph_id": "6488203-18-1", "paragraph_question": "question: 1985\ub144 \ucd1d\uc120\uc740 \uba87 \uc6d4\uc5d0 \uce58\ub8e8\uc5b4\uc84c\ub294\uac00?, context: \uc5d0\ub4dc\uc6cc\ub4dc \ucf00\ub124\ub514 \ubbf8\uad6d \uc0c1\uc6d0 \uc758\uc6d0\uc740 71\ub144 \ub2f9\uc2dc \uc2e0\ubbfc\ub2f9 \ub300\uc120\ud6c4\ubcf4\ub85c \ubbf8\uad6d\uc744 \ubc29\ubb38\ud55c \uae40\ub300\uc911\uc5d0 \ub300\ud574 \"\ub2f9\uc2e0\uc740 \ud55c\uad6d\uc758 \uc874 F. \ucf00\ub124\ub514\"\ub77c\uba70 \uc9c0\uc9c0\ub97c \ud45c\uba85\ud588\ub2e4. \uadf8\ub294 1980\ub144 \uae40\ub300\uc911\uc774 \uc2e0\uad70\ubd80\uc5d0 \uc758\ud574 \uc0ac\ud615\uc120\uace0\ub97c \ubc1b\uc790 \uad6c\uba85\uc6b4\ub3d9\uc5d0 \uc55e\uc7a5\uc130\uace0 \ubbf8\uad6d \ub9dd\uba85 \uc0dd\ud65c\uacfc \uadc0\uad6d \ub54c\ub3c4 \ud070 \ub3c4\uc6c0\uc744 \uc92c\ub2e4. \ucf00\ub124\ub514 \uc758\uc6d0\uc740 1984\ub144 4\uc6d4 26\uc77c \uae40\ub300\uc911\uc5d0\uac8c \ubcf4\ub0b8 \ud3b8\uc9c0\uc5d0\uc11c \"\ub2f9\uc2e0\uacfc \ub098\ub294 \ubbfc\uc8fc\uc640 \ud3c9\ud654\ub97c \uc9c0\uc9c0\ud558\ub294 \uc785\uc7a5\uc744 \uacf5\uc720\ud558\uace0 \uc788\ub2e4. \ud55c\uad6d\uc758 \ud3c9\ud654\uc801 \uc815\uad8c\uad50\uccb4\ub97c \uc9c0\uc9c0\ud55c\ub2e4\"\ub77c\uace0 \uc37c\ub2e4. 85\ub144 2\u00b712 \ucd1d\uc120 \uc804\uc5d0 \uadc0\uad6d\uc744 \uacb0\uc2ec\ud55c \uae40\ub300\uc911\uc740 1984\ub144 6\uc6d4 \"84\ub144 \ub9d0\uae4c\uc9c0 \uadc0\uad6d\ud558\uae30\ub85c \uacb0\uc815\ud588\ub2e4\"\ub77c\ub294 \ub73b\uc744 \uc11c\uc2e0\uc5d0 \ub2f4\uc544 \uc804\ud588\ub2e4. \ucf00\ub124\ub514 \uc758\uc6d0\uc740 86\ub144 5\uc6d4 \ud55c\uad6d\uc5d0\uc11c \ubbfc\uc8fc\ud654\ub97c \uc704\ud55c \uae40\ub300\uc911\uc758 \ub178\ub825\uc5d0 \uacbd\uc758\ub97c \ud45c\ud55c\ub2e4\ub294 \ub0b4\uc6a9\uc758 \ud3b8\uc9c0\ub97c \ubcf4\ub0c8\ub2e4."} {"question": "\uc790\uc0b4\ud55c \ub2e8\uc6d0\uace0 \uad50\uac10\uc740 \uc5b4\ub290 \ub300\ud559\uc744 \uc878\uc5c5\ud558\uc600\ub294\uac00?", "paragraph": "4\uc6d4 18\uc77c \uc624\ud6c4 4\uc2dc 5\ubd84 \uacbd, \uc218\ud559\uc5ec\ud589 \uc778\uc194\uc790\ub85c \uac14\ub2e4\uac00 \uc0ac\uace0\ub97c \ub2f9\ud558\uace0 \uad6c\uc870\ub41c \uc548\uc0b0 \ub2e8\uc6d0\uace0\ub4f1\ud559\uad50 \uac15\ubbfc\uaddc \uad50\uac10\uc774 \uc804\ub0a8 \uc9c4\ub3c4\uccb4\uc721\uad00 \ub4a4\ud3b8 \uc57c\uc0b0\uc5d0\uc11c \ubaa9\uc744 \ub9e4 \uc228\uc9c4 \ucc44 \ubc1c\uacac\ub418\uc5c8\ub2e4. \uad50\uac10\uc758 \uc9c0\uac11\uc5d0\uc11c\ub294 \uc720\uc11c\uac00 \ubc1c\uacac\ub418\uc5c8\ub2e4. \uc720\uc11c\uc5d0\ub294 \"200\uba85\uc758 \uc0dd\uc0ac\ub97c \uc54c \uc218 \uc5c6\ub294\ub370 \ud63c\uc790 \uc0b4\uae30\uc5d0\ub294 \ud798\uc774 \ubc85\ucc28\ub2e4. \ub098\uc5d0\uac8c \ubaa8\ub4e0 \ucc45\uc784\uc744 \uc9c0\uc6cc \ub2ec\ub77c. \ub0b4\uac00 \uc218\ud559\uc5ec\ud589\uc744 \ucd94\uc9c4\ud588\ub2e4\"\uba70 \uc2dc\uc2e0\uc740 \ud654\uc7a5\ud55c \ub4a4 \uc0ac\uace0 \ud574\uc5ed\uc5d0 \ubfcc\ub824 \ub2ec\ub77c\ub294 \ub0b4\uc6a9\uc774 \ub2f4\uaca8\uc788\ub2e4. \uc774\uc5b4 \"\uc2dc\uc2e0\uc744 \ucc3e\uc9c0 \ubabb\ud558\ub294 \ub140\uc11d\ub4e4\uacfc \ud568\uaed8 \uc800\uc2b9\uc5d0\uc11c\ub3c4 \uc120\uc0dd\uc744 \ud560\uae4c\"\ub77c\ub294 \ub0b4\uc6a9\ub3c4 \uc801\ud600\uc788\uc5c8\ub2e4. \uc790\uc0b4 \uc6d0\uc778 \uc911 \ud558\ub098\ub85c \ucd94\uc815\ub418\ub294 \uac83\uc774 \uc788\ub2e4. \uadf8\ub294 \ud559\uc0dd\ub4e4\uc744 \ub0a8\uaca8\ub450\uace0 \uad6c\uc870\ub41c \uac83\uc5d0 \ub300\ud574 \uc8fc\uc704\uc5d0 \u201c\ub098\ub9cc \ud63c\uc790 \ube60\uc838\ub098\uc628 \uac83 \uac19\uc544 \uad34\ub86d\ub2e4\u201d\uace0 \ud638\uc18c\ud588\uc73c\uba70, \uadf8\ub7f0 \uac00\uc6b4\ub370 \uc5ec\uac1d\uc120 \uce68\ubab0\ub85c \uc0ac\ub9dd\ud558\uac70\ub098 \uc2e4\uc885\ub41c \ub2e8\uc6d0\uace0 \ud559\uc0dd \ud559\ubd80\ubaa8\ub4e4\ub85c\ubd80\ud130 \u201c\uc65c \ub2f9\uc2e0\ub9cc \uc0b4\uc544 \ub3cc\uc544\uc654\ub290\ub0d0\u201d\ub294 \ub4f1 \uac15\ud55c \uc9c8\ud0c0\ub97c \ubc1b\uc740 \uac83\uc73c\ub85c \uc54c\ub824\uc84c\ub2e4. \uac15\ubbfc\uaddc \uad50\uac10\uc740 \uacf5\uc8fc\ub300\ud559\uad50 \uc0ac\ubc94\ub300\ud559\uc744 \uc878\uc5c5 \ud6c4 1987\ub144 \uc911\ub4f1\ud559\uad50 \uad50\uc0ac\ub85c \uc784\uc6a9\ub418\uc5b4 \uc724\ub9ac \uacfc\ubaa9\uc744 \uac00\ub974\ucce4\ub2e4. 2012\ub144 3\uc6d4 \uad50\uac10\uc73c\ub85c \uc2b9\uc9c4\ud558\uc600\uc73c\uba70, 2014\ub144 3\uc6d4 1\uc77c\uc790\ub85c \ub2e8\uc6d0\uace0\ub4f1\ud559\uad50\uc5d0 \ubd80\uc784\ud558\uc600\ub2e4.", "answer": "\uacf5\uc8fc\ub300\ud559\uad50 \uc0ac\ubc94\ub300\ud559", "sentence": "\uac15\ubbfc\uaddc \uad50\uac10\uc740 \uacf5\uc8fc\ub300\ud559\uad50 \uc0ac\ubc94\ub300\ud559 \uc744 \uc878\uc5c5 \ud6c4 1987\ub144 \uc911\ub4f1\ud559\uad50 \uad50\uc0ac\ub85c \uc784\uc6a9\ub418\uc5b4 \uc724\ub9ac \uacfc\ubaa9\uc744 \uac00\ub974\ucce4\ub2e4.", "paragraph_sentence": "4\uc6d4 18\uc77c \uc624\ud6c4 4\uc2dc 5\ubd84 \uacbd, \uc218\ud559\uc5ec\ud589 \uc778\uc194\uc790\ub85c \uac14\ub2e4\uac00 \uc0ac\uace0\ub97c \ub2f9\ud558\uace0 \uad6c\uc870\ub41c \uc548\uc0b0 \ub2e8\uc6d0\uace0\ub4f1\ud559\uad50 \uac15\ubbfc\uaddc \uad50\uac10\uc774 \uc804\ub0a8 \uc9c4\ub3c4\uccb4\uc721\uad00 \ub4a4\ud3b8 \uc57c\uc0b0\uc5d0\uc11c \ubaa9\uc744 \ub9e4 \uc228\uc9c4 \ucc44 \ubc1c\uacac\ub418\uc5c8\ub2e4. \uad50\uac10\uc758 \uc9c0\uac11\uc5d0\uc11c\ub294 \uc720\uc11c\uac00 \ubc1c\uacac\ub418\uc5c8\ub2e4. \uc720\uc11c\uc5d0\ub294 \"200\uba85\uc758 \uc0dd\uc0ac\ub97c \uc54c \uc218 \uc5c6\ub294\ub370 \ud63c\uc790 \uc0b4\uae30\uc5d0\ub294 \ud798\uc774 \ubc85\ucc28\ub2e4. \ub098\uc5d0\uac8c \ubaa8\ub4e0 \ucc45\uc784\uc744 \uc9c0\uc6cc \ub2ec\ub77c. \ub0b4\uac00 \uc218\ud559\uc5ec\ud589\uc744 \ucd94\uc9c4\ud588\ub2e4\"\uba70 \uc2dc\uc2e0\uc740 \ud654\uc7a5\ud55c \ub4a4 \uc0ac\uace0 \ud574\uc5ed\uc5d0 \ubfcc\ub824 \ub2ec\ub77c\ub294 \ub0b4\uc6a9\uc774 \ub2f4\uaca8\uc788\ub2e4. \uc774\uc5b4 \"\uc2dc\uc2e0\uc744 \ucc3e\uc9c0 \ubabb\ud558\ub294 \ub140\uc11d\ub4e4\uacfc \ud568\uaed8 \uc800\uc2b9\uc5d0\uc11c\ub3c4 \uc120\uc0dd\uc744 \ud560\uae4c\"\ub77c\ub294 \ub0b4\uc6a9\ub3c4 \uc801\ud600\uc788\uc5c8\ub2e4. \uc790\uc0b4 \uc6d0\uc778 \uc911 \ud558\ub098\ub85c \ucd94\uc815\ub418\ub294 \uac83\uc774 \uc788\ub2e4. \uadf8\ub294 \ud559\uc0dd\ub4e4\uc744 \ub0a8\uaca8\ub450\uace0 \uad6c\uc870\ub41c \uac83\uc5d0 \ub300\ud574 \uc8fc\uc704\uc5d0 \u201c\ub098\ub9cc \ud63c\uc790 \ube60\uc838\ub098\uc628 \uac83 \uac19\uc544 \uad34\ub86d\ub2e4\u201d\uace0 \ud638\uc18c\ud588\uc73c\uba70, \uadf8\ub7f0 \uac00\uc6b4\ub370 \uc5ec\uac1d\uc120 \uce68\ubab0\ub85c \uc0ac\ub9dd\ud558\uac70\ub098 \uc2e4\uc885\ub41c \ub2e8\uc6d0\uace0 \ud559\uc0dd \ud559\ubd80\ubaa8\ub4e4\ub85c\ubd80\ud130 \u201c\uc65c \ub2f9\uc2e0\ub9cc \uc0b4\uc544 \ub3cc\uc544\uc654\ub290\ub0d0\u201d\ub294 \ub4f1 \uac15\ud55c \uc9c8\ud0c0\ub97c \ubc1b\uc740 \uac83\uc73c\ub85c \uc54c\ub824\uc84c\ub2e4. \uac15\ubbfc\uaddc \uad50\uac10\uc740 \uacf5\uc8fc\ub300\ud559\uad50 \uc0ac\ubc94\ub300\ud559 \uc744 \uc878\uc5c5 \ud6c4 1987\ub144 \uc911\ub4f1\ud559\uad50 \uad50\uc0ac\ub85c \uc784\uc6a9\ub418\uc5b4 \uc724\ub9ac \uacfc\ubaa9\uc744 \uac00\ub974\ucce4\ub2e4. 2012\ub144 3\uc6d4 \uad50\uac10\uc73c\ub85c \uc2b9\uc9c4\ud558\uc600\uc73c\uba70, 2014\ub144 3\uc6d4 1\uc77c\uc790\ub85c \ub2e8\uc6d0\uace0\ub4f1\ud559\uad50\uc5d0 \ubd80\uc784\ud558\uc600\ub2e4.", "paragraph_answer": "4\uc6d4 18\uc77c \uc624\ud6c4 4\uc2dc 5\ubd84 \uacbd, \uc218\ud559\uc5ec\ud589 \uc778\uc194\uc790\ub85c \uac14\ub2e4\uac00 \uc0ac\uace0\ub97c \ub2f9\ud558\uace0 \uad6c\uc870\ub41c \uc548\uc0b0 \ub2e8\uc6d0\uace0\ub4f1\ud559\uad50 \uac15\ubbfc\uaddc \uad50\uac10\uc774 \uc804\ub0a8 \uc9c4\ub3c4\uccb4\uc721\uad00 \ub4a4\ud3b8 \uc57c\uc0b0\uc5d0\uc11c \ubaa9\uc744 \ub9e4 \uc228\uc9c4 \ucc44 \ubc1c\uacac\ub418\uc5c8\ub2e4. \uad50\uac10\uc758 \uc9c0\uac11\uc5d0\uc11c\ub294 \uc720\uc11c\uac00 \ubc1c\uacac\ub418\uc5c8\ub2e4. \uc720\uc11c\uc5d0\ub294 \"200\uba85\uc758 \uc0dd\uc0ac\ub97c \uc54c \uc218 \uc5c6\ub294\ub370 \ud63c\uc790 \uc0b4\uae30\uc5d0\ub294 \ud798\uc774 \ubc85\ucc28\ub2e4. \ub098\uc5d0\uac8c \ubaa8\ub4e0 \ucc45\uc784\uc744 \uc9c0\uc6cc \ub2ec\ub77c. \ub0b4\uac00 \uc218\ud559\uc5ec\ud589\uc744 \ucd94\uc9c4\ud588\ub2e4\"\uba70 \uc2dc\uc2e0\uc740 \ud654\uc7a5\ud55c \ub4a4 \uc0ac\uace0 \ud574\uc5ed\uc5d0 \ubfcc\ub824 \ub2ec\ub77c\ub294 \ub0b4\uc6a9\uc774 \ub2f4\uaca8\uc788\ub2e4. \uc774\uc5b4 \"\uc2dc\uc2e0\uc744 \ucc3e\uc9c0 \ubabb\ud558\ub294 \ub140\uc11d\ub4e4\uacfc \ud568\uaed8 \uc800\uc2b9\uc5d0\uc11c\ub3c4 \uc120\uc0dd\uc744 \ud560\uae4c\"\ub77c\ub294 \ub0b4\uc6a9\ub3c4 \uc801\ud600\uc788\uc5c8\ub2e4. \uc790\uc0b4 \uc6d0\uc778 \uc911 \ud558\ub098\ub85c \ucd94\uc815\ub418\ub294 \uac83\uc774 \uc788\ub2e4. \uadf8\ub294 \ud559\uc0dd\ub4e4\uc744 \ub0a8\uaca8\ub450\uace0 \uad6c\uc870\ub41c \uac83\uc5d0 \ub300\ud574 \uc8fc\uc704\uc5d0 \u201c\ub098\ub9cc \ud63c\uc790 \ube60\uc838\ub098\uc628 \uac83 \uac19\uc544 \uad34\ub86d\ub2e4\u201d\uace0 \ud638\uc18c\ud588\uc73c\uba70, \uadf8\ub7f0 \uac00\uc6b4\ub370 \uc5ec\uac1d\uc120 \uce68\ubab0\ub85c \uc0ac\ub9dd\ud558\uac70\ub098 \uc2e4\uc885\ub41c \ub2e8\uc6d0\uace0 \ud559\uc0dd \ud559\ubd80\ubaa8\ub4e4\ub85c\ubd80\ud130 \u201c\uc65c \ub2f9\uc2e0\ub9cc \uc0b4\uc544 \ub3cc\uc544\uc654\ub290\ub0d0\u201d\ub294 \ub4f1 \uac15\ud55c \uc9c8\ud0c0\ub97c \ubc1b\uc740 \uac83\uc73c\ub85c \uc54c\ub824\uc84c\ub2e4. \uac15\ubbfc\uaddc \uad50\uac10\uc740 \uacf5\uc8fc\ub300\ud559\uad50 \uc0ac\ubc94\ub300\ud559 \uc744 \uc878\uc5c5 \ud6c4 1987\ub144 \uc911\ub4f1\ud559\uad50 \uad50\uc0ac\ub85c \uc784\uc6a9\ub418\uc5b4 \uc724\ub9ac \uacfc\ubaa9\uc744 \uac00\ub974\ucce4\ub2e4. 2012\ub144 3\uc6d4 \uad50\uac10\uc73c\ub85c \uc2b9\uc9c4\ud558\uc600\uc73c\uba70, 2014\ub144 3\uc6d4 1\uc77c\uc790\ub85c \ub2e8\uc6d0\uace0\ub4f1\ud559\uad50\uc5d0 \ubd80\uc784\ud558\uc600\ub2e4.", "sentence_answer": "\uac15\ubbfc\uaddc \uad50\uac10\uc740 \uacf5\uc8fc\ub300\ud559\uad50 \uc0ac\ubc94\ub300\ud559 \uc744 \uc878\uc5c5 \ud6c4 1987\ub144 \uc911\ub4f1\ud559\uad50 \uad50\uc0ac\ub85c \uc784\uc6a9\ub418\uc5b4 \uc724\ub9ac \uacfc\ubaa9\uc744 \uac00\ub974\ucce4\ub2e4.", "paragraph_id": "6532130-16-0", "paragraph_question": "question: \uc790\uc0b4\ud55c \ub2e8\uc6d0\uace0 \uad50\uac10\uc740 \uc5b4\ub290 \ub300\ud559\uc744 \uc878\uc5c5\ud558\uc600\ub294\uac00?, context: 4\uc6d4 18\uc77c \uc624\ud6c4 4\uc2dc 5\ubd84 \uacbd, \uc218\ud559\uc5ec\ud589 \uc778\uc194\uc790\ub85c \uac14\ub2e4\uac00 \uc0ac\uace0\ub97c \ub2f9\ud558\uace0 \uad6c\uc870\ub41c \uc548\uc0b0 \ub2e8\uc6d0\uace0\ub4f1\ud559\uad50 \uac15\ubbfc\uaddc \uad50\uac10\uc774 \uc804\ub0a8 \uc9c4\ub3c4\uccb4\uc721\uad00 \ub4a4\ud3b8 \uc57c\uc0b0\uc5d0\uc11c \ubaa9\uc744 \ub9e4 \uc228\uc9c4 \ucc44 \ubc1c\uacac\ub418\uc5c8\ub2e4. \uad50\uac10\uc758 \uc9c0\uac11\uc5d0\uc11c\ub294 \uc720\uc11c\uac00 \ubc1c\uacac\ub418\uc5c8\ub2e4. \uc720\uc11c\uc5d0\ub294 \"200\uba85\uc758 \uc0dd\uc0ac\ub97c \uc54c \uc218 \uc5c6\ub294\ub370 \ud63c\uc790 \uc0b4\uae30\uc5d0\ub294 \ud798\uc774 \ubc85\ucc28\ub2e4. \ub098\uc5d0\uac8c \ubaa8\ub4e0 \ucc45\uc784\uc744 \uc9c0\uc6cc \ub2ec\ub77c. \ub0b4\uac00 \uc218\ud559\uc5ec\ud589\uc744 \ucd94\uc9c4\ud588\ub2e4\"\uba70 \uc2dc\uc2e0\uc740 \ud654\uc7a5\ud55c \ub4a4 \uc0ac\uace0 \ud574\uc5ed\uc5d0 \ubfcc\ub824 \ub2ec\ub77c\ub294 \ub0b4\uc6a9\uc774 \ub2f4\uaca8\uc788\ub2e4. \uc774\uc5b4 \"\uc2dc\uc2e0\uc744 \ucc3e\uc9c0 \ubabb\ud558\ub294 \ub140\uc11d\ub4e4\uacfc \ud568\uaed8 \uc800\uc2b9\uc5d0\uc11c\ub3c4 \uc120\uc0dd\uc744 \ud560\uae4c\"\ub77c\ub294 \ub0b4\uc6a9\ub3c4 \uc801\ud600\uc788\uc5c8\ub2e4. \uc790\uc0b4 \uc6d0\uc778 \uc911 \ud558\ub098\ub85c \ucd94\uc815\ub418\ub294 \uac83\uc774 \uc788\ub2e4. \uadf8\ub294 \ud559\uc0dd\ub4e4\uc744 \ub0a8\uaca8\ub450\uace0 \uad6c\uc870\ub41c \uac83\uc5d0 \ub300\ud574 \uc8fc\uc704\uc5d0 \u201c\ub098\ub9cc \ud63c\uc790 \ube60\uc838\ub098\uc628 \uac83 \uac19\uc544 \uad34\ub86d\ub2e4\u201d\uace0 \ud638\uc18c\ud588\uc73c\uba70, \uadf8\ub7f0 \uac00\uc6b4\ub370 \uc5ec\uac1d\uc120 \uce68\ubab0\ub85c \uc0ac\ub9dd\ud558\uac70\ub098 \uc2e4\uc885\ub41c \ub2e8\uc6d0\uace0 \ud559\uc0dd \ud559\ubd80\ubaa8\ub4e4\ub85c\ubd80\ud130 \u201c\uc65c \ub2f9\uc2e0\ub9cc \uc0b4\uc544 \ub3cc\uc544\uc654\ub290\ub0d0\u201d\ub294 \ub4f1 \uac15\ud55c \uc9c8\ud0c0\ub97c \ubc1b\uc740 \uac83\uc73c\ub85c \uc54c\ub824\uc84c\ub2e4. \uac15\ubbfc\uaddc \uad50\uac10\uc740 \uacf5\uc8fc\ub300\ud559\uad50 \uc0ac\ubc94\ub300\ud559\uc744 \uc878\uc5c5 \ud6c4 1987\ub144 \uc911\ub4f1\ud559\uad50 \uad50\uc0ac\ub85c \uc784\uc6a9\ub418\uc5b4 \uc724\ub9ac \uacfc\ubaa9\uc744 \uac00\ub974\ucce4\ub2e4. 2012\ub144 3\uc6d4 \uad50\uac10\uc73c\ub85c \uc2b9\uc9c4\ud558\uc600\uc73c\uba70, 2014\ub144 3\uc6d4 1\uc77c\uc790\ub85c \ub2e8\uc6d0\uace0\ub4f1\ud559\uad50\uc5d0 \ubd80\uc784\ud558\uc600\ub2e4."} {"question": "\ub85c\uc988\uc758 \uc9d1\uc5d0\uc11c \ubaa8\ub4e0 \uc0ac\ub78c\ub4e4\uc774 \ud3b8\ud558\uac8c \ud574\uc8fc\ub824\uace0 \ud55c \uc0ac\ub78c\uc740?", "paragraph": "\uc5ec\uc790 \uce5c\uad6c\uc640 \ud1b5\ud654\ub97c \ud55c \ud6c4, \ud751\uc778 \ub0a8\uc131 \uc548\ub4dc\ub808\uc774 \ud5e4\uc774\uc6cc\ub4dc\ub294 \ub2a6\uc740 \ubc24 \uad50\uc678\ub97c \uac77\ub358 \ub3c4\uc911 \ub0a9\uce58\ub41c\ub2e4. \uba87 \ub2ec \ud6c4, \ud751\uc778 \uc0ac\uc9c4\uac00\uc778 \ud06c\ub9ac\uc2a4 \uc6cc\uc2f1\ud134\uacfc \uadf8\uc758 \ubc31\uc778 \uc5ec\uc790 \uce5c\uad6c\uc778 \ub85c\uc988 \uc544\ubbf8\ud2f0\uc9c0\ub294 \ub85c\uc988\uc758 \ubd80\ubaa8\uc778 \ub518\uacfc \ubbf8\uc2dc, \uadf8\ub9ac\uace0 \ub85c\uc988\uc758 \ub3d9\uc0dd \uc81c\ub7ec\ubbf8\ub97c \ub9cc\ub098\uae30 \uc704\ud574 \uc5ec\ud589\uc744 \ub5a0\ub09c\ub2e4. \ub85c\uc988\uc758 \uc9d1\uc5d0\uc11c \ubaa8\ub4e0 \uc0ac\ub78c\uc740 \ud06c\ub9ac\uc2a4\ub97c \ud3b8\ud558\uac8c \ud558\uae30 \uc704\ud574 \ub178\ub825\ud558\uc9c0\ub9cc \ud06c\ub9ac\uc2a4\ub294 \ud751\uc778 \uc815\uc6d0\uc0ac\uc640 \uac00\uc815\ubd80\uc758 \uc774\uc0c1\ud55c \ud589\ub3d9\uc5d0 \ubd88\uc548\uac10\uc744 \ub290\ub080\ub2e4. \uadf8\ub0a0 \ubc24 \ud06c\ub9ac\uc2a4\ub294 \ubbf8\uc2dc\uc5d0\uac8c \ube91\uc18c\ub2c8\ub85c \uc8fd\uc740 \uc790\uc2e0\uc758 \uc5b4\uba38\ub2c8\uc5d0 \ub300\ud574 \uc774\uc57c\uae30\ud55c\ub2e4. \uc774\uc57c\uae30\ub97c \ud558\ub358 \uc911, \ubbf8\uc2dc\ub294 \ud06c\ub9ac\uc2a4\uc5d0\uac8c \ucd5c\uba74\uc744 \uac78\uc5b4 \ub9c8\ube44\ub41c \uc0c1\ud0dc\ub97c \ub9cc\ub4e4\uace0 \ud06c\ub9ac\uc2a4\uc758 \uc758\uc2dd\uc744 \uacf5\ud5c8\ub85c \ub358\uc838\ubc84\ub7f0\ub2e4. \ub2e4\uc74c \ub0a0 \uc7a0\uc5d0\uc11c \uae6c \ud06c\ub9ac\uc2a4\ub294 \uc545\ubabd\uc744 \uafb8\uc5c8\ub2e4\uace0 \uc0dd\uac01\ud55c\ub2e4.", "answer": "\ud06c\ub9ac\uc2a4", "sentence": "\uba87 \ub2ec \ud6c4, \ud751\uc778 \uc0ac\uc9c4\uac00\uc778 \ud06c\ub9ac\uc2a4 \uc6cc\uc2f1\ud134\uacfc \uadf8\uc758 \ubc31\uc778 \uc5ec\uc790 \uce5c\uad6c\uc778 \ub85c\uc988 \uc544\ubbf8\ud2f0\uc9c0\ub294 \ub85c\uc988\uc758 \ubd80\ubaa8\uc778 \ub518\uacfc \ubbf8\uc2dc, \uadf8\ub9ac\uace0 \ub85c\uc988\uc758 \ub3d9\uc0dd \uc81c\ub7ec\ubbf8\ub97c \ub9cc\ub098\uae30 \uc704\ud574 \uc5ec\ud589\uc744 \ub5a0\ub09c\ub2e4.", "paragraph_sentence": "\uc5ec\uc790 \uce5c\uad6c\uc640 \ud1b5\ud654\ub97c \ud55c \ud6c4, \ud751\uc778 \ub0a8\uc131 \uc548\ub4dc\ub808\uc774 \ud5e4\uc774\uc6cc\ub4dc\ub294 \ub2a6\uc740 \ubc24 \uad50\uc678\ub97c \uac77\ub358 \ub3c4\uc911 \ub0a9\uce58\ub41c\ub2e4. \uba87 \ub2ec \ud6c4, \ud751\uc778 \uc0ac\uc9c4\uac00\uc778 \ud06c\ub9ac\uc2a4 \uc6cc\uc2f1\ud134\uacfc \uadf8\uc758 \ubc31\uc778 \uc5ec\uc790 \uce5c\uad6c\uc778 \ub85c\uc988 \uc544\ubbf8\ud2f0\uc9c0\ub294 \ub85c\uc988\uc758 \ubd80\ubaa8\uc778 \ub518\uacfc \ubbf8\uc2dc, \uadf8\ub9ac\uace0 \ub85c\uc988\uc758 \ub3d9\uc0dd \uc81c\ub7ec\ubbf8\ub97c \ub9cc\ub098\uae30 \uc704\ud574 \uc5ec\ud589\uc744 \ub5a0\ub09c\ub2e4. \ub85c\uc988\uc758 \uc9d1\uc5d0\uc11c \ubaa8\ub4e0 \uc0ac\ub78c\uc740 \ud06c\ub9ac\uc2a4\ub97c \ud3b8\ud558\uac8c \ud558\uae30 \uc704\ud574 \ub178\ub825\ud558\uc9c0\ub9cc \ud06c\ub9ac\uc2a4\ub294 \ud751\uc778 \uc815\uc6d0\uc0ac\uc640 \uac00\uc815\ubd80\uc758 \uc774\uc0c1\ud55c \ud589\ub3d9\uc5d0 \ubd88\uc548\uac10\uc744 \ub290\ub080\ub2e4. \uadf8\ub0a0 \ubc24 \ud06c\ub9ac\uc2a4\ub294 \ubbf8\uc2dc\uc5d0\uac8c \ube91\uc18c\ub2c8\ub85c \uc8fd\uc740 \uc790\uc2e0\uc758 \uc5b4\uba38\ub2c8\uc5d0 \ub300\ud574 \uc774\uc57c\uae30\ud55c\ub2e4. \uc774\uc57c\uae30\ub97c \ud558\ub358 \uc911, \ubbf8\uc2dc\ub294 \ud06c\ub9ac\uc2a4\uc5d0\uac8c \ucd5c\uba74\uc744 \uac78\uc5b4 \ub9c8\ube44\ub41c \uc0c1\ud0dc\ub97c \ub9cc\ub4e4\uace0 \ud06c\ub9ac\uc2a4\uc758 \uc758\uc2dd\uc744 \uacf5\ud5c8\ub85c \ub358\uc838\ubc84\ub7f0\ub2e4. \ub2e4\uc74c \ub0a0 \uc7a0\uc5d0\uc11c \uae6c \ud06c\ub9ac\uc2a4\ub294 \uc545\ubabd\uc744 \uafb8\uc5c8\ub2e4\uace0 \uc0dd\uac01\ud55c\ub2e4.", "paragraph_answer": "\uc5ec\uc790 \uce5c\uad6c\uc640 \ud1b5\ud654\ub97c \ud55c \ud6c4, \ud751\uc778 \ub0a8\uc131 \uc548\ub4dc\ub808\uc774 \ud5e4\uc774\uc6cc\ub4dc\ub294 \ub2a6\uc740 \ubc24 \uad50\uc678\ub97c \uac77\ub358 \ub3c4\uc911 \ub0a9\uce58\ub41c\ub2e4. \uba87 \ub2ec \ud6c4, \ud751\uc778 \uc0ac\uc9c4\uac00\uc778 \ud06c\ub9ac\uc2a4 \uc6cc\uc2f1\ud134\uacfc \uadf8\uc758 \ubc31\uc778 \uc5ec\uc790 \uce5c\uad6c\uc778 \ub85c\uc988 \uc544\ubbf8\ud2f0\uc9c0\ub294 \ub85c\uc988\uc758 \ubd80\ubaa8\uc778 \ub518\uacfc \ubbf8\uc2dc, \uadf8\ub9ac\uace0 \ub85c\uc988\uc758 \ub3d9\uc0dd \uc81c\ub7ec\ubbf8\ub97c \ub9cc\ub098\uae30 \uc704\ud574 \uc5ec\ud589\uc744 \ub5a0\ub09c\ub2e4. \ub85c\uc988\uc758 \uc9d1\uc5d0\uc11c \ubaa8\ub4e0 \uc0ac\ub78c\uc740 \ud06c\ub9ac\uc2a4\ub97c \ud3b8\ud558\uac8c \ud558\uae30 \uc704\ud574 \ub178\ub825\ud558\uc9c0\ub9cc \ud06c\ub9ac\uc2a4\ub294 \ud751\uc778 \uc815\uc6d0\uc0ac\uc640 \uac00\uc815\ubd80\uc758 \uc774\uc0c1\ud55c \ud589\ub3d9\uc5d0 \ubd88\uc548\uac10\uc744 \ub290\ub080\ub2e4. \uadf8\ub0a0 \ubc24 \ud06c\ub9ac\uc2a4\ub294 \ubbf8\uc2dc\uc5d0\uac8c \ube91\uc18c\ub2c8\ub85c \uc8fd\uc740 \uc790\uc2e0\uc758 \uc5b4\uba38\ub2c8\uc5d0 \ub300\ud574 \uc774\uc57c\uae30\ud55c\ub2e4. \uc774\uc57c\uae30\ub97c \ud558\ub358 \uc911, \ubbf8\uc2dc\ub294 \ud06c\ub9ac\uc2a4\uc5d0\uac8c \ucd5c\uba74\uc744 \uac78\uc5b4 \ub9c8\ube44\ub41c \uc0c1\ud0dc\ub97c \ub9cc\ub4e4\uace0 \ud06c\ub9ac\uc2a4\uc758 \uc758\uc2dd\uc744 \uacf5\ud5c8\ub85c \ub358\uc838\ubc84\ub7f0\ub2e4. \ub2e4\uc74c \ub0a0 \uc7a0\uc5d0\uc11c \uae6c \ud06c\ub9ac\uc2a4\ub294 \uc545\ubabd\uc744 \uafb8\uc5c8\ub2e4\uace0 \uc0dd\uac01\ud55c\ub2e4.", "sentence_answer": "\uba87 \ub2ec \ud6c4, \ud751\uc778 \uc0ac\uc9c4\uac00\uc778 \ud06c\ub9ac\uc2a4 \uc6cc\uc2f1\ud134\uacfc \uadf8\uc758 \ubc31\uc778 \uc5ec\uc790 \uce5c\uad6c\uc778 \ub85c\uc988 \uc544\ubbf8\ud2f0\uc9c0\ub294 \ub85c\uc988\uc758 \ubd80\ubaa8\uc778 \ub518\uacfc \ubbf8\uc2dc, \uadf8\ub9ac\uace0 \ub85c\uc988\uc758 \ub3d9\uc0dd \uc81c\ub7ec\ubbf8\ub97c \ub9cc\ub098\uae30 \uc704\ud574 \uc5ec\ud589\uc744 \ub5a0\ub09c\ub2e4.", "paragraph_id": "6528125-0-0", "paragraph_question": "question: \ub85c\uc988\uc758 \uc9d1\uc5d0\uc11c \ubaa8\ub4e0 \uc0ac\ub78c\ub4e4\uc774 \ud3b8\ud558\uac8c \ud574\uc8fc\ub824\uace0 \ud55c \uc0ac\ub78c\uc740?, context: \uc5ec\uc790 \uce5c\uad6c\uc640 \ud1b5\ud654\ub97c \ud55c \ud6c4, \ud751\uc778 \ub0a8\uc131 \uc548\ub4dc\ub808\uc774 \ud5e4\uc774\uc6cc\ub4dc\ub294 \ub2a6\uc740 \ubc24 \uad50\uc678\ub97c \uac77\ub358 \ub3c4\uc911 \ub0a9\uce58\ub41c\ub2e4. \uba87 \ub2ec \ud6c4, \ud751\uc778 \uc0ac\uc9c4\uac00\uc778 \ud06c\ub9ac\uc2a4 \uc6cc\uc2f1\ud134\uacfc \uadf8\uc758 \ubc31\uc778 \uc5ec\uc790 \uce5c\uad6c\uc778 \ub85c\uc988 \uc544\ubbf8\ud2f0\uc9c0\ub294 \ub85c\uc988\uc758 \ubd80\ubaa8\uc778 \ub518\uacfc \ubbf8\uc2dc, \uadf8\ub9ac\uace0 \ub85c\uc988\uc758 \ub3d9\uc0dd \uc81c\ub7ec\ubbf8\ub97c \ub9cc\ub098\uae30 \uc704\ud574 \uc5ec\ud589\uc744 \ub5a0\ub09c\ub2e4. \ub85c\uc988\uc758 \uc9d1\uc5d0\uc11c \ubaa8\ub4e0 \uc0ac\ub78c\uc740 \ud06c\ub9ac\uc2a4\ub97c \ud3b8\ud558\uac8c \ud558\uae30 \uc704\ud574 \ub178\ub825\ud558\uc9c0\ub9cc \ud06c\ub9ac\uc2a4\ub294 \ud751\uc778 \uc815\uc6d0\uc0ac\uc640 \uac00\uc815\ubd80\uc758 \uc774\uc0c1\ud55c \ud589\ub3d9\uc5d0 \ubd88\uc548\uac10\uc744 \ub290\ub080\ub2e4. \uadf8\ub0a0 \ubc24 \ud06c\ub9ac\uc2a4\ub294 \ubbf8\uc2dc\uc5d0\uac8c \ube91\uc18c\ub2c8\ub85c \uc8fd\uc740 \uc790\uc2e0\uc758 \uc5b4\uba38\ub2c8\uc5d0 \ub300\ud574 \uc774\uc57c\uae30\ud55c\ub2e4. \uc774\uc57c\uae30\ub97c \ud558\ub358 \uc911, \ubbf8\uc2dc\ub294 \ud06c\ub9ac\uc2a4\uc5d0\uac8c \ucd5c\uba74\uc744 \uac78\uc5b4 \ub9c8\ube44\ub41c \uc0c1\ud0dc\ub97c \ub9cc\ub4e4\uace0 \ud06c\ub9ac\uc2a4\uc758 \uc758\uc2dd\uc744 \uacf5\ud5c8\ub85c \ub358\uc838\ubc84\ub7f0\ub2e4. \ub2e4\uc74c \ub0a0 \uc7a0\uc5d0\uc11c \uae6c \ud06c\ub9ac\uc2a4\ub294 \uc545\ubabd\uc744 \uafb8\uc5c8\ub2e4\uace0 \uc0dd\uac01\ud55c\ub2e4."} {"question": "\uc720\uc6b0\uc758 \uc544\ub4e4\uc758 \uc774\ub984\uc740?", "paragraph": "\ud55c\ud3b8 \uc720\uc6b0\ub294 \uc2ec\ubcf5\uc778 \uc804\uc8fc\uc640 \uc120\uc6b0\uc740\uc744 \uc7a5\uc548\uc73c\ub85c \ubcf4\ub0b4 \ud5cc\uc81c\ub97c \uc54c\ud604\ud558\uac8c \ud588\ub2e4. \ud5cc\uc81c\ub294 \uc774\uc5d0 \ud06c\uac8c \uae30\ubed0\ud558\uba70 \uc7a5\uc548\uc5d0 \uc788\ub358 \uc720\uc6b0\uc758 \uc544\ub4e4 \uc720\ud654(\u5289\u548c)\ub97c \uc720\uc6b0\uc5d0\uac8c \ubcf4\ub0b4 \uc720\uc6b0\uac00 \uc790\uc2e0\uc744 \uad6c\uc6d0\ud558\uac8c \ud558\ub77c\ub294 \uba85\ub839\uc744 \ub0b4\ub838\ub2e4. \uc720\ud654\ub294 \uc720\uc8fc\ub85c \ud5a5\ud558\ub358 \uc911 \uc6d0\uc220 \uacc1\uc744 \uc9c0\ub098\uac00\ub2e4\uac00 \uc6d0\uc220\uc5d0\uac8c \uc790\uc2e0\uc758 \ub73b\uc744 \uc124\uba85\ud588\ub294\ub370 \uc6d0\uc220 \uc5ed\uc2dc \ub300\uc678\uc801\uc73c\ub85c\ub294 \ud5cc\uc81c\uc640 \ud669\uc2e4\uc744 \uc639\ud638\ud558\ub294 \uc785\uc7a5\uc744 \ucde8\ud558\uace0 \uc788\uc5c8\uace0 \uc6d0\uc220\uc740 \uc720\uc6b0\ub97c \uc6d0\uad70\uc73c\ub85c \uc0bc\ub294 \ud3b8\uc774 \uc88b\ub2e4\uace0 \uc5ec\uacbc\uc73c\ubbc0\ub85c \uc720\ud654\uc5d0\uac8c \uc790\uc2e0\ub3c4 \ud669\uc81c\uc758 \uad6c\uc6d0\uc5d0 \ud569\ub958\ud558\uac8c \ud574 \ub2ec\ub77c\uace0 \uc694\uad6c\ud558\uba70 \uc720\ud654\ub97c \ubcf4\ub0b4\uc8fc\uc9c0 \uc54a\uace0 \ub300\uc2e0 \uc720\uc6b0\uc5d0\uac8c \uc11c\uc2e0\uc744 \uc4f0\uac8c \ud588\ub2e4. \uc720\ud654\uc758 \uc11c\uc2e0\uc744 \uc811\ud55c \uc720\uc6b0\ub294 \uc720\ud654\uc5d0\uac8c 3\ucc9c\uc758 \uae30\ubcd1\uc744 \ubcf4\ub0c8\uc73c\ub098, \uacf5\uc190\ucc2c\uc740 \ubab0\ub798 \uacf5\uc190\uc6d4\uc744 \ubcf4\ub0b4 \uc6d0\uc220\uacfc \uc190\uc7a1\uace0 \uc720\uc6b0\uac00 \ubcf4\ub0b8 \uad70\uc0ac\ub4e4\uc744 \ube7c\uc557\ub3c4\ub85d \ud588\ub2e4. \uc774\ub85c\uc368 \uc720\uc6b0\uc640 \uacf5\uc190\ucc2c\uc758 \uc0ac\uc774\uac00 \ub354\uc6b1 \ub098\ube60\uc9c0\uac8c \ub418\uc5c8\ub2e4.", "answer": "\uc720\ud654", "sentence": "\ud5cc\uc81c\ub294 \uc774\uc5d0 \ud06c\uac8c \uae30\ubed0\ud558\uba70 \uc7a5\uc548\uc5d0 \uc788\ub358 \uc720\uc6b0\uc758 \uc544\ub4e4 \uc720\ud654 (\u5289\u548c)", "paragraph_sentence": "\ud55c\ud3b8 \uc720\uc6b0\ub294 \uc2ec\ubcf5\uc778 \uc804\uc8fc\uc640 \uc120\uc6b0\uc740\uc744 \uc7a5\uc548\uc73c\ub85c \ubcf4\ub0b4 \ud5cc\uc81c\ub97c \uc54c\ud604\ud558\uac8c \ud588\ub2e4. \ud5cc\uc81c\ub294 \uc774\uc5d0 \ud06c\uac8c \uae30\ubed0\ud558\uba70 \uc7a5\uc548\uc5d0 \uc788\ub358 \uc720\uc6b0\uc758 \uc544\ub4e4 \uc720\ud654 (\u5289\u548c) \ub97c \uc720\uc6b0\uc5d0\uac8c \ubcf4\ub0b4 \uc720\uc6b0\uac00 \uc790\uc2e0\uc744 \uad6c\uc6d0\ud558\uac8c \ud558\ub77c\ub294 \uba85\ub839\uc744 \ub0b4\ub838\ub2e4. \uc720\ud654\ub294 \uc720\uc8fc\ub85c \ud5a5\ud558\ub358 \uc911 \uc6d0\uc220 \uacc1\uc744 \uc9c0\ub098\uac00\ub2e4\uac00 \uc6d0\uc220\uc5d0\uac8c \uc790\uc2e0\uc758 \ub73b\uc744 \uc124\uba85\ud588\ub294\ub370 \uc6d0\uc220 \uc5ed\uc2dc \ub300\uc678\uc801\uc73c\ub85c\ub294 \ud5cc\uc81c\uc640 \ud669\uc2e4\uc744 \uc639\ud638\ud558\ub294 \uc785\uc7a5\uc744 \ucde8\ud558\uace0 \uc788\uc5c8\uace0 \uc6d0\uc220\uc740 \uc720\uc6b0\ub97c \uc6d0\uad70\uc73c\ub85c \uc0bc\ub294 \ud3b8\uc774 \uc88b\ub2e4\uace0 \uc5ec\uacbc\uc73c\ubbc0\ub85c \uc720\ud654\uc5d0\uac8c \uc790\uc2e0\ub3c4 \ud669\uc81c\uc758 \uad6c\uc6d0\uc5d0 \ud569\ub958\ud558\uac8c \ud574 \ub2ec\ub77c\uace0 \uc694\uad6c\ud558\uba70 \uc720\ud654\ub97c \ubcf4\ub0b4\uc8fc\uc9c0 \uc54a\uace0 \ub300\uc2e0 \uc720\uc6b0\uc5d0\uac8c \uc11c\uc2e0\uc744 \uc4f0\uac8c \ud588\ub2e4. \uc720\ud654\uc758 \uc11c\uc2e0\uc744 \uc811\ud55c \uc720\uc6b0\ub294 \uc720\ud654\uc5d0\uac8c 3\ucc9c\uc758 \uae30\ubcd1\uc744 \ubcf4\ub0c8\uc73c\ub098, \uacf5\uc190\ucc2c\uc740 \ubab0\ub798 \uacf5\uc190\uc6d4\uc744 \ubcf4\ub0b4 \uc6d0\uc220\uacfc \uc190\uc7a1\uace0 \uc720\uc6b0\uac00 \ubcf4\ub0b8 \uad70\uc0ac\ub4e4\uc744 \ube7c\uc557\ub3c4\ub85d \ud588\ub2e4. \uc774\ub85c\uc368 \uc720\uc6b0\uc640 \uacf5\uc190\ucc2c\uc758 \uc0ac\uc774\uac00 \ub354\uc6b1 \ub098\ube60\uc9c0\uac8c \ub418\uc5c8\ub2e4.", "paragraph_answer": "\ud55c\ud3b8 \uc720\uc6b0\ub294 \uc2ec\ubcf5\uc778 \uc804\uc8fc\uc640 \uc120\uc6b0\uc740\uc744 \uc7a5\uc548\uc73c\ub85c \ubcf4\ub0b4 \ud5cc\uc81c\ub97c \uc54c\ud604\ud558\uac8c \ud588\ub2e4. \ud5cc\uc81c\ub294 \uc774\uc5d0 \ud06c\uac8c \uae30\ubed0\ud558\uba70 \uc7a5\uc548\uc5d0 \uc788\ub358 \uc720\uc6b0\uc758 \uc544\ub4e4 \uc720\ud654 (\u5289\u548c)\ub97c \uc720\uc6b0\uc5d0\uac8c \ubcf4\ub0b4 \uc720\uc6b0\uac00 \uc790\uc2e0\uc744 \uad6c\uc6d0\ud558\uac8c \ud558\ub77c\ub294 \uba85\ub839\uc744 \ub0b4\ub838\ub2e4. \uc720\ud654\ub294 \uc720\uc8fc\ub85c \ud5a5\ud558\ub358 \uc911 \uc6d0\uc220 \uacc1\uc744 \uc9c0\ub098\uac00\ub2e4\uac00 \uc6d0\uc220\uc5d0\uac8c \uc790\uc2e0\uc758 \ub73b\uc744 \uc124\uba85\ud588\ub294\ub370 \uc6d0\uc220 \uc5ed\uc2dc \ub300\uc678\uc801\uc73c\ub85c\ub294 \ud5cc\uc81c\uc640 \ud669\uc2e4\uc744 \uc639\ud638\ud558\ub294 \uc785\uc7a5\uc744 \ucde8\ud558\uace0 \uc788\uc5c8\uace0 \uc6d0\uc220\uc740 \uc720\uc6b0\ub97c \uc6d0\uad70\uc73c\ub85c \uc0bc\ub294 \ud3b8\uc774 \uc88b\ub2e4\uace0 \uc5ec\uacbc\uc73c\ubbc0\ub85c \uc720\ud654\uc5d0\uac8c \uc790\uc2e0\ub3c4 \ud669\uc81c\uc758 \uad6c\uc6d0\uc5d0 \ud569\ub958\ud558\uac8c \ud574 \ub2ec\ub77c\uace0 \uc694\uad6c\ud558\uba70 \uc720\ud654\ub97c \ubcf4\ub0b4\uc8fc\uc9c0 \uc54a\uace0 \ub300\uc2e0 \uc720\uc6b0\uc5d0\uac8c \uc11c\uc2e0\uc744 \uc4f0\uac8c \ud588\ub2e4. \uc720\ud654\uc758 \uc11c\uc2e0\uc744 \uc811\ud55c \uc720\uc6b0\ub294 \uc720\ud654\uc5d0\uac8c 3\ucc9c\uc758 \uae30\ubcd1\uc744 \ubcf4\ub0c8\uc73c\ub098, \uacf5\uc190\ucc2c\uc740 \ubab0\ub798 \uacf5\uc190\uc6d4\uc744 \ubcf4\ub0b4 \uc6d0\uc220\uacfc \uc190\uc7a1\uace0 \uc720\uc6b0\uac00 \ubcf4\ub0b8 \uad70\uc0ac\ub4e4\uc744 \ube7c\uc557\ub3c4\ub85d \ud588\ub2e4. \uc774\ub85c\uc368 \uc720\uc6b0\uc640 \uacf5\uc190\ucc2c\uc758 \uc0ac\uc774\uac00 \ub354\uc6b1 \ub098\ube60\uc9c0\uac8c \ub418\uc5c8\ub2e4.", "sentence_answer": "\ud5cc\uc81c\ub294 \uc774\uc5d0 \ud06c\uac8c \uae30\ubed0\ud558\uba70 \uc7a5\uc548\uc5d0 \uc788\ub358 \uc720\uc6b0\uc758 \uc544\ub4e4 \uc720\ud654 (\u5289\u548c)", "paragraph_id": "6543340-2-0", "paragraph_question": "question: \uc720\uc6b0\uc758 \uc544\ub4e4\uc758 \uc774\ub984\uc740?, context: \ud55c\ud3b8 \uc720\uc6b0\ub294 \uc2ec\ubcf5\uc778 \uc804\uc8fc\uc640 \uc120\uc6b0\uc740\uc744 \uc7a5\uc548\uc73c\ub85c \ubcf4\ub0b4 \ud5cc\uc81c\ub97c \uc54c\ud604\ud558\uac8c \ud588\ub2e4. \ud5cc\uc81c\ub294 \uc774\uc5d0 \ud06c\uac8c \uae30\ubed0\ud558\uba70 \uc7a5\uc548\uc5d0 \uc788\ub358 \uc720\uc6b0\uc758 \uc544\ub4e4 \uc720\ud654(\u5289\u548c)\ub97c \uc720\uc6b0\uc5d0\uac8c \ubcf4\ub0b4 \uc720\uc6b0\uac00 \uc790\uc2e0\uc744 \uad6c\uc6d0\ud558\uac8c \ud558\ub77c\ub294 \uba85\ub839\uc744 \ub0b4\ub838\ub2e4. \uc720\ud654\ub294 \uc720\uc8fc\ub85c \ud5a5\ud558\ub358 \uc911 \uc6d0\uc220 \uacc1\uc744 \uc9c0\ub098\uac00\ub2e4\uac00 \uc6d0\uc220\uc5d0\uac8c \uc790\uc2e0\uc758 \ub73b\uc744 \uc124\uba85\ud588\ub294\ub370 \uc6d0\uc220 \uc5ed\uc2dc \ub300\uc678\uc801\uc73c\ub85c\ub294 \ud5cc\uc81c\uc640 \ud669\uc2e4\uc744 \uc639\ud638\ud558\ub294 \uc785\uc7a5\uc744 \ucde8\ud558\uace0 \uc788\uc5c8\uace0 \uc6d0\uc220\uc740 \uc720\uc6b0\ub97c \uc6d0\uad70\uc73c\ub85c \uc0bc\ub294 \ud3b8\uc774 \uc88b\ub2e4\uace0 \uc5ec\uacbc\uc73c\ubbc0\ub85c \uc720\ud654\uc5d0\uac8c \uc790\uc2e0\ub3c4 \ud669\uc81c\uc758 \uad6c\uc6d0\uc5d0 \ud569\ub958\ud558\uac8c \ud574 \ub2ec\ub77c\uace0 \uc694\uad6c\ud558\uba70 \uc720\ud654\ub97c \ubcf4\ub0b4\uc8fc\uc9c0 \uc54a\uace0 \ub300\uc2e0 \uc720\uc6b0\uc5d0\uac8c \uc11c\uc2e0\uc744 \uc4f0\uac8c \ud588\ub2e4. \uc720\ud654\uc758 \uc11c\uc2e0\uc744 \uc811\ud55c \uc720\uc6b0\ub294 \uc720\ud654\uc5d0\uac8c 3\ucc9c\uc758 \uae30\ubcd1\uc744 \ubcf4\ub0c8\uc73c\ub098, \uacf5\uc190\ucc2c\uc740 \ubab0\ub798 \uacf5\uc190\uc6d4\uc744 \ubcf4\ub0b4 \uc6d0\uc220\uacfc \uc190\uc7a1\uace0 \uc720\uc6b0\uac00 \ubcf4\ub0b8 \uad70\uc0ac\ub4e4\uc744 \ube7c\uc557\ub3c4\ub85d \ud588\ub2e4. \uc774\ub85c\uc368 \uc720\uc6b0\uc640 \uacf5\uc190\ucc2c\uc758 \uc0ac\uc774\uac00 \ub354\uc6b1 \ub098\ube60\uc9c0\uac8c \ub418\uc5c8\ub2e4."} {"question": "11\uc6d4 22\uc77c \uc0c8 \uc218\uc0c1\uc73c\ub85c \ub204\uac00 \uc120\ucd9c\ub418\uc5c8\ub098?", "paragraph": "9\uc6d4 18\uc77c \ub3c5\uc77c \ucd1d\uc120\uc774 \uc2e4\uc2dc\ub418\uc5c8\ub2e4. \uc120\uac70\uac00 \uc5f4\ub9b0 \ud6c4, \uc544\ubb34\ub3c4 \uc288\ub8b0\ub354\uc758 \uc801\ub179 \uc5f0\ud569\uc774\ub098 \uc559\uac94\ub77c \uba54\ub974\ucf08\uc774 \uc774\ub048 \uae30\ub3c5\uad50\ubbfc\uc8fc\uc5f0\ud569/\ubc14\uc774\uc5d0\ub978 \uae30\ub3c5\uad50\uc0ac\ud68c\uc5f0\ud569\uacfc \uc790\uc720\ubbfc\uc8fc\ub2f9\uc774\ub098 \uc544\ubb34\ub3c4 \uc758\ud68c\uc5d0\uc11c \ub2e4\uc218\ub97c \uc774\ub8e8\uc9c0 \uc54a\uc558\uc73c\ub098 1 \ud37c\uc13c\ud2b8 \ucc28\uc774\ub85c \uae30\ub3c5\uad50\ubbfc\uc8fc\uc5f0\ud569/\ubc14\uc774\uc5d0\ub978 \uae30\ub3c5\uad50\uc0ac\ud68c\uc5f0\ud569\uc774 \ub354\uc6b1 \uac15\ud55c \uc778\uae30\uc801 \uc120\uac70\uc778\ub2e8\uc744 \uac00\uc84c\ub2e4. \uc120\uac70\uac00 \uc5f4\ub9ac\uae30 \uba87\uc8fc \uc804\uc5d0 \uc0ac\ud68c\ubbfc\uc8fc\ub2f9\uc774 15\uc810 \uc774\uc0c1 \ucc28\uc774\ub85c \uae30\ub3c5\uad50\ubbfc\uc8fc\uc5f0\ud569\uc744 \ucd94\uc801\ud55c \uc774\ub798 \uc774 \uacb0\uacfc\ub294 \ub180\ub77c\uc6b4 \uc77c\uc774\uc5c8\uace0, \uc120\uac70 \uc6b4\ub3d9\uc744 \ubc8c\uc774\ub294 \uc790\ub85c\uc11c \uc288\ub8b0\ub354\uc758 \uce74\ub9ac\uc2a4\ub9c8\uc640 \uc6a9\ub9f9\uc5d0 \uadc0\ucc29\uc2dc\ucf30\ub2e4. \ud22c\ud45c\ub294 \uacac\uc2e4\ud788 \uc288\ub8b0\ub354\uac00 \uba54\ub974\ucf08\ubcf4\ub2e4 \ub354\uc6b1 \uc778\uae30\uc788\uc5c8\ub2e4\uace0 \ubcf4\uc600\ub2e4. \uc120\uac70\uc758 \ubc24\uc5d0 \uc288\ub8b0\ub354\uc640 \uba54\ub974\ucf08\uc740 \ub458\ub2e4 \uc2b9\ub9ac\uc640 \uc218\uc0c1\uc9c1\uc744 \uc8fc\uc7a5\ud558\uc600\uc73c\ub098 \uc2dc\ucd08\uc801\uc73c\ub85c \uba54\ub974\ucf08\uacfc \ud568\uaed8 \ub300\uc5f0\ud569\uc744 \uc81c\uc678\ud55c \ud6c4 \uc288\ub8b0\ub354\uc640 \ubb8c\ud14c\ud398\ub9c1\uc740 \ubc14\uc774\uc5d0\ub978 \uae30\ub3c5\uad50\uc0ac\ud68c\uc5f0\ud569\uc758 \uc5d0\ub4dc\ubb38\ud2b8 \uc288\ud1a0\uc774\ubc84\uc640 \ud568\uaed8 \ud611\uc0c1\uc5d0 \ub4e4\uc5b4\uac14\ub2e4. 10\uc6d4 10\uc77c \ub2f9\ub4e4\uc774 \ub300\uc5f0\ud569\uc744 \ud615\uc131\ud558\ub294 \ub370 \ub3d9\uc758\ud558\uc600\ub2e4\uace0 \ubcf4\uace0\ub418\uc5c8\ub2e4. \uc288\ub8b0\ub354\ub294 \uba54\ub974\ucf08\uc5d0\uac8c \uc218\uc0c1\uc9c1\uc744 \uc591\ub3c4\ud558\ub294 \ub370 \ub3d9\uc758\ud558\uc600\uc73c\ub098 \uc0ac\ud68c\ubbfc\uc8fc\ub2f9\uc740 \uc815\ubd80 \uc9c1\uc704\ub4e4\uc758 \ub2e4\uc218\ub97c \ubcf4\uc720\ud558\uace0 \uc815\ubd80 \uc815\ucc45\uc758 \uc219\uace0\uc801\uc778 \ud1b5\uce58\ub97c \uc720\uc9c0\ud558\ub824\uace0 \ud558\uc600\ub2e4. \uba54\ub974\ucf08\uc740 11\uc6d4 22\uc77c \uc0c8 \uc218\uc0c1\uc73c\ub85c \uc120\ucd9c\ub418\uc5c8\ub2e4.", "answer": "\uba54\ub974\ucf08", "sentence": "\uc120\uac70\uac00 \uc5f4\ub9b0 \ud6c4, \uc544\ubb34\ub3c4 \uc288\ub8b0\ub354\uc758 \uc801\ub179 \uc5f0\ud569\uc774\ub098 \uc559\uac94\ub77c \uba54\ub974\ucf08 \uc774 \uc774\ub048 \uae30\ub3c5\uad50\ubbfc\uc8fc\uc5f0\ud569/\ubc14\uc774\uc5d0\ub978 \uae30\ub3c5\uad50\uc0ac\ud68c\uc5f0\ud569\uacfc \uc790\uc720\ubbfc\uc8fc\ub2f9\uc774\ub098 \uc544\ubb34\ub3c4 \uc758\ud68c\uc5d0\uc11c \ub2e4\uc218\ub97c \uc774\ub8e8\uc9c0 \uc54a\uc558\uc73c\ub098 1 \ud37c\uc13c\ud2b8 \ucc28\uc774\ub85c \uae30\ub3c5\uad50\ubbfc\uc8fc\uc5f0\ud569/\ubc14\uc774\uc5d0\ub978 \uae30\ub3c5\uad50\uc0ac\ud68c\uc5f0\ud569\uc774 \ub354\uc6b1 \uac15\ud55c \uc778\uae30\uc801 \uc120\uac70\uc778\ub2e8\uc744 \uac00\uc84c\ub2e4.", "paragraph_sentence": "9\uc6d4 18\uc77c \ub3c5\uc77c \ucd1d\uc120\uc774 \uc2e4\uc2dc\ub418\uc5c8\ub2e4. \uc120\uac70\uac00 \uc5f4\ub9b0 \ud6c4, \uc544\ubb34\ub3c4 \uc288\ub8b0\ub354\uc758 \uc801\ub179 \uc5f0\ud569\uc774\ub098 \uc559\uac94\ub77c \uba54\ub974\ucf08 \uc774 \uc774\ub048 \uae30\ub3c5\uad50\ubbfc\uc8fc\uc5f0\ud569/\ubc14\uc774\uc5d0\ub978 \uae30\ub3c5\uad50\uc0ac\ud68c\uc5f0\ud569\uacfc \uc790\uc720\ubbfc\uc8fc\ub2f9\uc774\ub098 \uc544\ubb34\ub3c4 \uc758\ud68c\uc5d0\uc11c \ub2e4\uc218\ub97c \uc774\ub8e8\uc9c0 \uc54a\uc558\uc73c\ub098 1 \ud37c\uc13c\ud2b8 \ucc28\uc774\ub85c \uae30\ub3c5\uad50\ubbfc\uc8fc\uc5f0\ud569/\ubc14\uc774\uc5d0\ub978 \uae30\ub3c5\uad50\uc0ac\ud68c\uc5f0\ud569\uc774 \ub354\uc6b1 \uac15\ud55c \uc778\uae30\uc801 \uc120\uac70\uc778\ub2e8\uc744 \uac00\uc84c\ub2e4. \uc120\uac70\uac00 \uc5f4\ub9ac\uae30 \uba87\uc8fc \uc804\uc5d0 \uc0ac\ud68c\ubbfc\uc8fc\ub2f9\uc774 15\uc810 \uc774\uc0c1 \ucc28\uc774\ub85c \uae30\ub3c5\uad50\ubbfc\uc8fc\uc5f0\ud569\uc744 \ucd94\uc801\ud55c \uc774\ub798 \uc774 \uacb0\uacfc\ub294 \ub180\ub77c\uc6b4 \uc77c\uc774\uc5c8\uace0, \uc120\uac70 \uc6b4\ub3d9\uc744 \ubc8c\uc774\ub294 \uc790\ub85c\uc11c \uc288\ub8b0\ub354\uc758 \uce74\ub9ac\uc2a4\ub9c8\uc640 \uc6a9\ub9f9\uc5d0 \uadc0\ucc29\uc2dc\ucf30\ub2e4. \ud22c\ud45c\ub294 \uacac\uc2e4\ud788 \uc288\ub8b0\ub354\uac00 \uba54\ub974\ucf08\ubcf4\ub2e4 \ub354\uc6b1 \uc778\uae30\uc788\uc5c8\ub2e4\uace0 \ubcf4\uc600\ub2e4. \uc120\uac70\uc758 \ubc24\uc5d0 \uc288\ub8b0\ub354\uc640 \uba54\ub974\ucf08\uc740 \ub458\ub2e4 \uc2b9\ub9ac\uc640 \uc218\uc0c1\uc9c1\uc744 \uc8fc\uc7a5\ud558\uc600\uc73c\ub098 \uc2dc\ucd08\uc801\uc73c\ub85c \uba54\ub974\ucf08\uacfc \ud568\uaed8 \ub300\uc5f0\ud569\uc744 \uc81c\uc678\ud55c \ud6c4 \uc288\ub8b0\ub354\uc640 \ubb8c\ud14c\ud398\ub9c1\uc740 \ubc14\uc774\uc5d0\ub978 \uae30\ub3c5\uad50\uc0ac\ud68c\uc5f0\ud569\uc758 \uc5d0\ub4dc\ubb38\ud2b8 \uc288\ud1a0\uc774\ubc84\uc640 \ud568\uaed8 \ud611\uc0c1\uc5d0 \ub4e4\uc5b4\uac14\ub2e4. 10\uc6d4 10\uc77c \ub2f9\ub4e4\uc774 \ub300\uc5f0\ud569\uc744 \ud615\uc131\ud558\ub294 \ub370 \ub3d9\uc758\ud558\uc600\ub2e4\uace0 \ubcf4\uace0\ub418\uc5c8\ub2e4. \uc288\ub8b0\ub354\ub294 \uba54\ub974\ucf08\uc5d0\uac8c \uc218\uc0c1\uc9c1\uc744 \uc591\ub3c4\ud558\ub294 \ub370 \ub3d9\uc758\ud558\uc600\uc73c\ub098 \uc0ac\ud68c\ubbfc\uc8fc\ub2f9\uc740 \uc815\ubd80 \uc9c1\uc704\ub4e4\uc758 \ub2e4\uc218\ub97c \ubcf4\uc720\ud558\uace0 \uc815\ubd80 \uc815\ucc45\uc758 \uc219\uace0\uc801\uc778 \ud1b5\uce58\ub97c \uc720\uc9c0\ud558\ub824\uace0 \ud558\uc600\ub2e4. \uba54\ub974\ucf08\uc740 11\uc6d4 22\uc77c \uc0c8 \uc218\uc0c1\uc73c\ub85c \uc120\ucd9c\ub418\uc5c8\ub2e4.", "paragraph_answer": "9\uc6d4 18\uc77c \ub3c5\uc77c \ucd1d\uc120\uc774 \uc2e4\uc2dc\ub418\uc5c8\ub2e4. \uc120\uac70\uac00 \uc5f4\ub9b0 \ud6c4, \uc544\ubb34\ub3c4 \uc288\ub8b0\ub354\uc758 \uc801\ub179 \uc5f0\ud569\uc774\ub098 \uc559\uac94\ub77c \uba54\ub974\ucf08 \uc774 \uc774\ub048 \uae30\ub3c5\uad50\ubbfc\uc8fc\uc5f0\ud569/\ubc14\uc774\uc5d0\ub978 \uae30\ub3c5\uad50\uc0ac\ud68c\uc5f0\ud569\uacfc \uc790\uc720\ubbfc\uc8fc\ub2f9\uc774\ub098 \uc544\ubb34\ub3c4 \uc758\ud68c\uc5d0\uc11c \ub2e4\uc218\ub97c \uc774\ub8e8\uc9c0 \uc54a\uc558\uc73c\ub098 1 \ud37c\uc13c\ud2b8 \ucc28\uc774\ub85c \uae30\ub3c5\uad50\ubbfc\uc8fc\uc5f0\ud569/\ubc14\uc774\uc5d0\ub978 \uae30\ub3c5\uad50\uc0ac\ud68c\uc5f0\ud569\uc774 \ub354\uc6b1 \uac15\ud55c \uc778\uae30\uc801 \uc120\uac70\uc778\ub2e8\uc744 \uac00\uc84c\ub2e4. \uc120\uac70\uac00 \uc5f4\ub9ac\uae30 \uba87\uc8fc \uc804\uc5d0 \uc0ac\ud68c\ubbfc\uc8fc\ub2f9\uc774 15\uc810 \uc774\uc0c1 \ucc28\uc774\ub85c \uae30\ub3c5\uad50\ubbfc\uc8fc\uc5f0\ud569\uc744 \ucd94\uc801\ud55c \uc774\ub798 \uc774 \uacb0\uacfc\ub294 \ub180\ub77c\uc6b4 \uc77c\uc774\uc5c8\uace0, \uc120\uac70 \uc6b4\ub3d9\uc744 \ubc8c\uc774\ub294 \uc790\ub85c\uc11c \uc288\ub8b0\ub354\uc758 \uce74\ub9ac\uc2a4\ub9c8\uc640 \uc6a9\ub9f9\uc5d0 \uadc0\ucc29\uc2dc\ucf30\ub2e4. \ud22c\ud45c\ub294 \uacac\uc2e4\ud788 \uc288\ub8b0\ub354\uac00 \uba54\ub974\ucf08\ubcf4\ub2e4 \ub354\uc6b1 \uc778\uae30\uc788\uc5c8\ub2e4\uace0 \ubcf4\uc600\ub2e4. \uc120\uac70\uc758 \ubc24\uc5d0 \uc288\ub8b0\ub354\uc640 \uba54\ub974\ucf08\uc740 \ub458\ub2e4 \uc2b9\ub9ac\uc640 \uc218\uc0c1\uc9c1\uc744 \uc8fc\uc7a5\ud558\uc600\uc73c\ub098 \uc2dc\ucd08\uc801\uc73c\ub85c \uba54\ub974\ucf08\uacfc \ud568\uaed8 \ub300\uc5f0\ud569\uc744 \uc81c\uc678\ud55c \ud6c4 \uc288\ub8b0\ub354\uc640 \ubb8c\ud14c\ud398\ub9c1\uc740 \ubc14\uc774\uc5d0\ub978 \uae30\ub3c5\uad50\uc0ac\ud68c\uc5f0\ud569\uc758 \uc5d0\ub4dc\ubb38\ud2b8 \uc288\ud1a0\uc774\ubc84\uc640 \ud568\uaed8 \ud611\uc0c1\uc5d0 \ub4e4\uc5b4\uac14\ub2e4. 10\uc6d4 10\uc77c \ub2f9\ub4e4\uc774 \ub300\uc5f0\ud569\uc744 \ud615\uc131\ud558\ub294 \ub370 \ub3d9\uc758\ud558\uc600\ub2e4\uace0 \ubcf4\uace0\ub418\uc5c8\ub2e4. \uc288\ub8b0\ub354\ub294 \uba54\ub974\ucf08\uc5d0\uac8c \uc218\uc0c1\uc9c1\uc744 \uc591\ub3c4\ud558\ub294 \ub370 \ub3d9\uc758\ud558\uc600\uc73c\ub098 \uc0ac\ud68c\ubbfc\uc8fc\ub2f9\uc740 \uc815\ubd80 \uc9c1\uc704\ub4e4\uc758 \ub2e4\uc218\ub97c \ubcf4\uc720\ud558\uace0 \uc815\ubd80 \uc815\ucc45\uc758 \uc219\uace0\uc801\uc778 \ud1b5\uce58\ub97c \uc720\uc9c0\ud558\ub824\uace0 \ud558\uc600\ub2e4. \uba54\ub974\ucf08\uc740 11\uc6d4 22\uc77c \uc0c8 \uc218\uc0c1\uc73c\ub85c \uc120\ucd9c\ub418\uc5c8\ub2e4.", "sentence_answer": "\uc120\uac70\uac00 \uc5f4\ub9b0 \ud6c4, \uc544\ubb34\ub3c4 \uc288\ub8b0\ub354\uc758 \uc801\ub179 \uc5f0\ud569\uc774\ub098 \uc559\uac94\ub77c \uba54\ub974\ucf08 \uc774 \uc774\ub048 \uae30\ub3c5\uad50\ubbfc\uc8fc\uc5f0\ud569/\ubc14\uc774\uc5d0\ub978 \uae30\ub3c5\uad50\uc0ac\ud68c\uc5f0\ud569\uacfc \uc790\uc720\ubbfc\uc8fc\ub2f9\uc774\ub098 \uc544\ubb34\ub3c4 \uc758\ud68c\uc5d0\uc11c \ub2e4\uc218\ub97c \uc774\ub8e8\uc9c0 \uc54a\uc558\uc73c\ub098 1 \ud37c\uc13c\ud2b8 \ucc28\uc774\ub85c \uae30\ub3c5\uad50\ubbfc\uc8fc\uc5f0\ud569/\ubc14\uc774\uc5d0\ub978 \uae30\ub3c5\uad50\uc0ac\ud68c\uc5f0\ud569\uc774 \ub354\uc6b1 \uac15\ud55c \uc778\uae30\uc801 \uc120\uac70\uc778\ub2e8\uc744 \uac00\uc84c\ub2e4.", "paragraph_id": "6472382-1-1", "paragraph_question": "question: 11\uc6d4 22\uc77c \uc0c8 \uc218\uc0c1\uc73c\ub85c \ub204\uac00 \uc120\ucd9c\ub418\uc5c8\ub098?, context: 9\uc6d4 18\uc77c \ub3c5\uc77c \ucd1d\uc120\uc774 \uc2e4\uc2dc\ub418\uc5c8\ub2e4. \uc120\uac70\uac00 \uc5f4\ub9b0 \ud6c4, \uc544\ubb34\ub3c4 \uc288\ub8b0\ub354\uc758 \uc801\ub179 \uc5f0\ud569\uc774\ub098 \uc559\uac94\ub77c \uba54\ub974\ucf08\uc774 \uc774\ub048 \uae30\ub3c5\uad50\ubbfc\uc8fc\uc5f0\ud569/\ubc14\uc774\uc5d0\ub978 \uae30\ub3c5\uad50\uc0ac\ud68c\uc5f0\ud569\uacfc \uc790\uc720\ubbfc\uc8fc\ub2f9\uc774\ub098 \uc544\ubb34\ub3c4 \uc758\ud68c\uc5d0\uc11c \ub2e4\uc218\ub97c \uc774\ub8e8\uc9c0 \uc54a\uc558\uc73c\ub098 1 \ud37c\uc13c\ud2b8 \ucc28\uc774\ub85c \uae30\ub3c5\uad50\ubbfc\uc8fc\uc5f0\ud569/\ubc14\uc774\uc5d0\ub978 \uae30\ub3c5\uad50\uc0ac\ud68c\uc5f0\ud569\uc774 \ub354\uc6b1 \uac15\ud55c \uc778\uae30\uc801 \uc120\uac70\uc778\ub2e8\uc744 \uac00\uc84c\ub2e4. \uc120\uac70\uac00 \uc5f4\ub9ac\uae30 \uba87\uc8fc \uc804\uc5d0 \uc0ac\ud68c\ubbfc\uc8fc\ub2f9\uc774 15\uc810 \uc774\uc0c1 \ucc28\uc774\ub85c \uae30\ub3c5\uad50\ubbfc\uc8fc\uc5f0\ud569\uc744 \ucd94\uc801\ud55c \uc774\ub798 \uc774 \uacb0\uacfc\ub294 \ub180\ub77c\uc6b4 \uc77c\uc774\uc5c8\uace0, \uc120\uac70 \uc6b4\ub3d9\uc744 \ubc8c\uc774\ub294 \uc790\ub85c\uc11c \uc288\ub8b0\ub354\uc758 \uce74\ub9ac\uc2a4\ub9c8\uc640 \uc6a9\ub9f9\uc5d0 \uadc0\ucc29\uc2dc\ucf30\ub2e4. \ud22c\ud45c\ub294 \uacac\uc2e4\ud788 \uc288\ub8b0\ub354\uac00 \uba54\ub974\ucf08\ubcf4\ub2e4 \ub354\uc6b1 \uc778\uae30\uc788\uc5c8\ub2e4\uace0 \ubcf4\uc600\ub2e4. \uc120\uac70\uc758 \ubc24\uc5d0 \uc288\ub8b0\ub354\uc640 \uba54\ub974\ucf08\uc740 \ub458\ub2e4 \uc2b9\ub9ac\uc640 \uc218\uc0c1\uc9c1\uc744 \uc8fc\uc7a5\ud558\uc600\uc73c\ub098 \uc2dc\ucd08\uc801\uc73c\ub85c \uba54\ub974\ucf08\uacfc \ud568\uaed8 \ub300\uc5f0\ud569\uc744 \uc81c\uc678\ud55c \ud6c4 \uc288\ub8b0\ub354\uc640 \ubb8c\ud14c\ud398\ub9c1\uc740 \ubc14\uc774\uc5d0\ub978 \uae30\ub3c5\uad50\uc0ac\ud68c\uc5f0\ud569\uc758 \uc5d0\ub4dc\ubb38\ud2b8 \uc288\ud1a0\uc774\ubc84\uc640 \ud568\uaed8 \ud611\uc0c1\uc5d0 \ub4e4\uc5b4\uac14\ub2e4. 10\uc6d4 10\uc77c \ub2f9\ub4e4\uc774 \ub300\uc5f0\ud569\uc744 \ud615\uc131\ud558\ub294 \ub370 \ub3d9\uc758\ud558\uc600\ub2e4\uace0 \ubcf4\uace0\ub418\uc5c8\ub2e4. \uc288\ub8b0\ub354\ub294 \uba54\ub974\ucf08\uc5d0\uac8c \uc218\uc0c1\uc9c1\uc744 \uc591\ub3c4\ud558\ub294 \ub370 \ub3d9\uc758\ud558\uc600\uc73c\ub098 \uc0ac\ud68c\ubbfc\uc8fc\ub2f9\uc740 \uc815\ubd80 \uc9c1\uc704\ub4e4\uc758 \ub2e4\uc218\ub97c \ubcf4\uc720\ud558\uace0 \uc815\ubd80 \uc815\ucc45\uc758 \uc219\uace0\uc801\uc778 \ud1b5\uce58\ub97c \uc720\uc9c0\ud558\ub824\uace0 \ud558\uc600\ub2e4. \uba54\ub974\ucf08\uc740 11\uc6d4 22\uc77c \uc0c8 \uc218\uc0c1\uc73c\ub85c \uc120\ucd9c\ub418\uc5c8\ub2e4."} {"question": "\ubbf8\uc131\uc219 \uac1c\uccb4\ub97c \uc0ac\ub0e5\ud588\ub2e4\ub294 \uc99d\uac70\uac00 \ubc1c\uacac\ub41c \ud45c\ubcf8\uc740 \ub204\uac00 \ubc1c\uacac\ud588\ub098?", "paragraph": "\uc54c\ub85c\uc0ac\uc6b0\ub8e8\uc2a4\ub294 \ubb34\ub9ac\ub97c \uc9c0\uc5b4 \uc0ac\ub0e5\ud588\uc744 \uac83\uc774\ub098,, \ucd5c\uadfc \uc5f0\uad6c \uacb0\uacfc\uc5d0 \ub530\ub974\uba74 \uadf8\uc640 \ubc18\ub300\ub85c \ub2e4\ub978 \uc774\uad81\uc544\uac15\uc758 \uc885\ucc98\ub7fc \uc11c\ub85c \uacf5\uaca9\uc801\uc778 \ud0dc\ub3c4\ub97c \ubcf4\uc600\uc744 \uac83\uc774\ub77c\ub294 \uacb0\uacfc\uac00 \ub098\uc654\ub2e4. \ud55c \uc5f0\uad6c\uc5d0\uc11c\ub294 \uac1c\uccb4\ubcf4\ub2e4 \ud070 \uac1c\uccb4\ub97c \uc0ac\ub0e5\ud560 \ub54c \ubcf4\uc5ec\uc8fc\ub294 \ud611\ub3d9 \uc791\uc5c5\uc744 \uc8fc\uc2dc\ud588\ub2e4. \ub9ce\uc740 \ud604\uc874 \uc774\uad81\uc544\uac15\uc885\uc740 \uc790\uc2e0\ub9cc\uc758 \ubc94\uc704\ub97c \uc815\ud558\uba70 \uac19\uc740 \uc885\uc774\ub77c\ub3c4 \uc601\uc5ed\uc744 \uce68\ubc94\ud558\uba74 \uc7a1\uc544\uba39\uc744 \uac83\uc774\ub2e4. \uc774\ub294 \ud074\ub9ac\ube14\ub79c\ub4dc-\ub85c\uc774\ub4dc \ud0d0\uc0ac\uc5d0\uc11c \ubc1c\uacac\ub41c \uc54c\ub85c\uc0ac\uc6b0\ub8e8\uc2a4 \ud654\uc11d \ubb34\ub9ac\ub294 \uc8fd\uc740 \ub3d9\ub8cc \uc8fc\uc704\uc5d0 \ubaa8\uc5ec \uc2f8\uc6b0\ub2e4\uac00 \uc11c\uc11c\ud788 \uc8fd\uc5c8\uc744 \uac83\uc774\ub77c\ub294 \uac83\uc744 \ucd94\uc815\ud560 \uc218 \uc788\uac8c \ud55c\ub2e4. \uc774\ub294 \ub610\ud55c \ubbf8\uc131\uc219 \uac1c\uccb4\uac00 \ud604\uc7a5\uc5d0\uc11c \ub9ce\uc740 \uac83\uc744 \uc124\uba85\ud560 \uc218 \uc788\ub2e4. \uc774\ub7ec\ud55c \ud604\uc0c1\uc740 \uc545\uc5b4\ub098 \ucf54\ubaa8\ub3c4\uc655\ub3c4\ub9c8\ubc40\uc774 \uba39\uc774\uac00 \ub9ce\uc740 \uc7a5\uc18c\uc5d0 \ubaa8\uc600\uc744 \ub54c \ubbf8\uc131\uc219 \uac1c\uccb4\uac00 \ub9ce\uc774 \uc8fd\ub294 \uac83\uc5d0\uc11c\ub3c4 \ubcf4\uc778\ub2e4. \uc774\ub294 \ubc14\ucee4\uac00 \ubc1c\uacac\ud55c \ud45c\ubcf8\uc5d0\ub3c4 \uc801\uc6a9\ub420 \uac83\uc774\ub2e4. \uc54c\ub85c\uc0ac\uc6b0\ub8e8\uc2a4\uac00 \ub3d9\uc871\uc744 \uc7a1\uc544\uba39\uc5c8\ub2e4\ub294 \uc99d\uac70\ub294 \uc788\uc73c\uba70, \uac08\ube44\ubf08 \uc870\uac01\uc5d0\uc11c \ubc1c\uacac\ub41c \uc774\ube68, \uacac\uac11\uace8\uc5d0 \ubc1c\uacac\ub41c \uc774\ube68\uc790\uad6d, \uadf8\ub9ac\uace0 \ubc14\ucee4\uac00 \ubc1c\uacac\ud55c \ud604\uc7a5\uc5d0\uc11c \ubc1c\uacac\ub41c \uc11c\ub85c \ud574\ucce4\ub358 \ud754\uc801\uc774 \uc788\ub294 \ubf08 \ub4f1\uc774\ub2e4.", "answer": "\ubc14\ucee4", "sentence": "\uc774\ub294 \ubc14\ucee4 \uac00 \ubc1c\uacac\ud55c \ud45c\ubcf8\uc5d0\ub3c4 \uc801\uc6a9\ub420 \uac83\uc774\ub2e4.", "paragraph_sentence": "\uc54c\ub85c\uc0ac\uc6b0\ub8e8\uc2a4\ub294 \ubb34\ub9ac\ub97c \uc9c0\uc5b4 \uc0ac\ub0e5\ud588\uc744 \uac83\uc774\ub098,, \ucd5c\uadfc \uc5f0\uad6c \uacb0\uacfc\uc5d0 \ub530\ub974\uba74 \uadf8\uc640 \ubc18\ub300\ub85c \ub2e4\ub978 \uc774\uad81\uc544\uac15\uc758 \uc885\ucc98\ub7fc \uc11c\ub85c \uacf5\uaca9\uc801\uc778 \ud0dc\ub3c4\ub97c \ubcf4\uc600\uc744 \uac83\uc774\ub77c\ub294 \uacb0\uacfc\uac00 \ub098\uc654\ub2e4. \ud55c \uc5f0\uad6c\uc5d0\uc11c\ub294 \uac1c\uccb4\ubcf4\ub2e4 \ud070 \uac1c\uccb4\ub97c \uc0ac\ub0e5\ud560 \ub54c \ubcf4\uc5ec\uc8fc\ub294 \ud611\ub3d9 \uc791\uc5c5\uc744 \uc8fc\uc2dc\ud588\ub2e4. \ub9ce\uc740 \ud604\uc874 \uc774\uad81\uc544\uac15\uc885\uc740 \uc790\uc2e0\ub9cc\uc758 \ubc94\uc704\ub97c \uc815\ud558\uba70 \uac19\uc740 \uc885\uc774\ub77c\ub3c4 \uc601\uc5ed\uc744 \uce68\ubc94\ud558\uba74 \uc7a1\uc544\uba39\uc744 \uac83\uc774\ub2e4. \uc774\ub294 \ud074\ub9ac\ube14\ub79c\ub4dc-\ub85c\uc774\ub4dc \ud0d0\uc0ac\uc5d0\uc11c \ubc1c\uacac\ub41c \uc54c\ub85c\uc0ac\uc6b0\ub8e8\uc2a4 \ud654\uc11d \ubb34\ub9ac\ub294 \uc8fd\uc740 \ub3d9\ub8cc \uc8fc\uc704\uc5d0 \ubaa8\uc5ec \uc2f8\uc6b0\ub2e4\uac00 \uc11c\uc11c\ud788 \uc8fd\uc5c8\uc744 \uac83\uc774\ub77c\ub294 \uac83\uc744 \ucd94\uc815\ud560 \uc218 \uc788\uac8c \ud55c\ub2e4. \uc774\ub294 \ub610\ud55c \ubbf8\uc131\uc219 \uac1c\uccb4\uac00 \ud604\uc7a5\uc5d0\uc11c \ub9ce\uc740 \uac83\uc744 \uc124\uba85\ud560 \uc218 \uc788\ub2e4. \uc774\ub7ec\ud55c \ud604\uc0c1\uc740 \uc545\uc5b4\ub098 \ucf54\ubaa8\ub3c4\uc655\ub3c4\ub9c8\ubc40\uc774 \uba39\uc774\uac00 \ub9ce\uc740 \uc7a5\uc18c\uc5d0 \ubaa8\uc600\uc744 \ub54c \ubbf8\uc131\uc219 \uac1c\uccb4\uac00 \ub9ce\uc774 \uc8fd\ub294 \uac83\uc5d0\uc11c\ub3c4 \ubcf4\uc778\ub2e4. \uc774\ub294 \ubc14\ucee4 \uac00 \ubc1c\uacac\ud55c \ud45c\ubcf8\uc5d0\ub3c4 \uc801\uc6a9\ub420 \uac83\uc774\ub2e4. \uc54c\ub85c\uc0ac\uc6b0\ub8e8\uc2a4\uac00 \ub3d9\uc871\uc744 \uc7a1\uc544\uba39\uc5c8\ub2e4\ub294 \uc99d\uac70\ub294 \uc788\uc73c\uba70, \uac08\ube44\ubf08 \uc870\uac01\uc5d0\uc11c \ubc1c\uacac\ub41c \uc774\ube68, \uacac\uac11\uace8\uc5d0 \ubc1c\uacac\ub41c \uc774\ube68\uc790\uad6d, \uadf8\ub9ac\uace0 \ubc14\ucee4\uac00 \ubc1c\uacac\ud55c \ud604\uc7a5\uc5d0\uc11c \ubc1c\uacac\ub41c \uc11c\ub85c \ud574\ucce4\ub358 \ud754\uc801\uc774 \uc788\ub294 \ubf08 \ub4f1\uc774\ub2e4.", "paragraph_answer": "\uc54c\ub85c\uc0ac\uc6b0\ub8e8\uc2a4\ub294 \ubb34\ub9ac\ub97c \uc9c0\uc5b4 \uc0ac\ub0e5\ud588\uc744 \uac83\uc774\ub098,, \ucd5c\uadfc \uc5f0\uad6c \uacb0\uacfc\uc5d0 \ub530\ub974\uba74 \uadf8\uc640 \ubc18\ub300\ub85c \ub2e4\ub978 \uc774\uad81\uc544\uac15\uc758 \uc885\ucc98\ub7fc \uc11c\ub85c \uacf5\uaca9\uc801\uc778 \ud0dc\ub3c4\ub97c \ubcf4\uc600\uc744 \uac83\uc774\ub77c\ub294 \uacb0\uacfc\uac00 \ub098\uc654\ub2e4. \ud55c \uc5f0\uad6c\uc5d0\uc11c\ub294 \uac1c\uccb4\ubcf4\ub2e4 \ud070 \uac1c\uccb4\ub97c \uc0ac\ub0e5\ud560 \ub54c \ubcf4\uc5ec\uc8fc\ub294 \ud611\ub3d9 \uc791\uc5c5\uc744 \uc8fc\uc2dc\ud588\ub2e4. \ub9ce\uc740 \ud604\uc874 \uc774\uad81\uc544\uac15\uc885\uc740 \uc790\uc2e0\ub9cc\uc758 \ubc94\uc704\ub97c \uc815\ud558\uba70 \uac19\uc740 \uc885\uc774\ub77c\ub3c4 \uc601\uc5ed\uc744 \uce68\ubc94\ud558\uba74 \uc7a1\uc544\uba39\uc744 \uac83\uc774\ub2e4. \uc774\ub294 \ud074\ub9ac\ube14\ub79c\ub4dc-\ub85c\uc774\ub4dc \ud0d0\uc0ac\uc5d0\uc11c \ubc1c\uacac\ub41c \uc54c\ub85c\uc0ac\uc6b0\ub8e8\uc2a4 \ud654\uc11d \ubb34\ub9ac\ub294 \uc8fd\uc740 \ub3d9\ub8cc \uc8fc\uc704\uc5d0 \ubaa8\uc5ec \uc2f8\uc6b0\ub2e4\uac00 \uc11c\uc11c\ud788 \uc8fd\uc5c8\uc744 \uac83\uc774\ub77c\ub294 \uac83\uc744 \ucd94\uc815\ud560 \uc218 \uc788\uac8c \ud55c\ub2e4. \uc774\ub294 \ub610\ud55c \ubbf8\uc131\uc219 \uac1c\uccb4\uac00 \ud604\uc7a5\uc5d0\uc11c \ub9ce\uc740 \uac83\uc744 \uc124\uba85\ud560 \uc218 \uc788\ub2e4. \uc774\ub7ec\ud55c \ud604\uc0c1\uc740 \uc545\uc5b4\ub098 \ucf54\ubaa8\ub3c4\uc655\ub3c4\ub9c8\ubc40\uc774 \uba39\uc774\uac00 \ub9ce\uc740 \uc7a5\uc18c\uc5d0 \ubaa8\uc600\uc744 \ub54c \ubbf8\uc131\uc219 \uac1c\uccb4\uac00 \ub9ce\uc774 \uc8fd\ub294 \uac83\uc5d0\uc11c\ub3c4 \ubcf4\uc778\ub2e4. \uc774\ub294 \ubc14\ucee4 \uac00 \ubc1c\uacac\ud55c \ud45c\ubcf8\uc5d0\ub3c4 \uc801\uc6a9\ub420 \uac83\uc774\ub2e4. \uc54c\ub85c\uc0ac\uc6b0\ub8e8\uc2a4\uac00 \ub3d9\uc871\uc744 \uc7a1\uc544\uba39\uc5c8\ub2e4\ub294 \uc99d\uac70\ub294 \uc788\uc73c\uba70, \uac08\ube44\ubf08 \uc870\uac01\uc5d0\uc11c \ubc1c\uacac\ub41c \uc774\ube68, \uacac\uac11\uace8\uc5d0 \ubc1c\uacac\ub41c \uc774\ube68\uc790\uad6d, \uadf8\ub9ac\uace0 \ubc14\ucee4\uac00 \ubc1c\uacac\ud55c \ud604\uc7a5\uc5d0\uc11c \ubc1c\uacac\ub41c \uc11c\ub85c \ud574\ucce4\ub358 \ud754\uc801\uc774 \uc788\ub294 \ubf08 \ub4f1\uc774\ub2e4.", "sentence_answer": "\uc774\ub294 \ubc14\ucee4 \uac00 \ubc1c\uacac\ud55c \ud45c\ubcf8\uc5d0\ub3c4 \uc801\uc6a9\ub420 \uac83\uc774\ub2e4.", "paragraph_id": "6465441-19-0", "paragraph_question": "question: \ubbf8\uc131\uc219 \uac1c\uccb4\ub97c \uc0ac\ub0e5\ud588\ub2e4\ub294 \uc99d\uac70\uac00 \ubc1c\uacac\ub41c \ud45c\ubcf8\uc740 \ub204\uac00 \ubc1c\uacac\ud588\ub098?, context: \uc54c\ub85c\uc0ac\uc6b0\ub8e8\uc2a4\ub294 \ubb34\ub9ac\ub97c \uc9c0\uc5b4 \uc0ac\ub0e5\ud588\uc744 \uac83\uc774\ub098,, \ucd5c\uadfc \uc5f0\uad6c \uacb0\uacfc\uc5d0 \ub530\ub974\uba74 \uadf8\uc640 \ubc18\ub300\ub85c \ub2e4\ub978 \uc774\uad81\uc544\uac15\uc758 \uc885\ucc98\ub7fc \uc11c\ub85c \uacf5\uaca9\uc801\uc778 \ud0dc\ub3c4\ub97c \ubcf4\uc600\uc744 \uac83\uc774\ub77c\ub294 \uacb0\uacfc\uac00 \ub098\uc654\ub2e4. \ud55c \uc5f0\uad6c\uc5d0\uc11c\ub294 \uac1c\uccb4\ubcf4\ub2e4 \ud070 \uac1c\uccb4\ub97c \uc0ac\ub0e5\ud560 \ub54c \ubcf4\uc5ec\uc8fc\ub294 \ud611\ub3d9 \uc791\uc5c5\uc744 \uc8fc\uc2dc\ud588\ub2e4. \ub9ce\uc740 \ud604\uc874 \uc774\uad81\uc544\uac15\uc885\uc740 \uc790\uc2e0\ub9cc\uc758 \ubc94\uc704\ub97c \uc815\ud558\uba70 \uac19\uc740 \uc885\uc774\ub77c\ub3c4 \uc601\uc5ed\uc744 \uce68\ubc94\ud558\uba74 \uc7a1\uc544\uba39\uc744 \uac83\uc774\ub2e4. \uc774\ub294 \ud074\ub9ac\ube14\ub79c\ub4dc-\ub85c\uc774\ub4dc \ud0d0\uc0ac\uc5d0\uc11c \ubc1c\uacac\ub41c \uc54c\ub85c\uc0ac\uc6b0\ub8e8\uc2a4 \ud654\uc11d \ubb34\ub9ac\ub294 \uc8fd\uc740 \ub3d9\ub8cc \uc8fc\uc704\uc5d0 \ubaa8\uc5ec \uc2f8\uc6b0\ub2e4\uac00 \uc11c\uc11c\ud788 \uc8fd\uc5c8\uc744 \uac83\uc774\ub77c\ub294 \uac83\uc744 \ucd94\uc815\ud560 \uc218 \uc788\uac8c \ud55c\ub2e4. \uc774\ub294 \ub610\ud55c \ubbf8\uc131\uc219 \uac1c\uccb4\uac00 \ud604\uc7a5\uc5d0\uc11c \ub9ce\uc740 \uac83\uc744 \uc124\uba85\ud560 \uc218 \uc788\ub2e4. \uc774\ub7ec\ud55c \ud604\uc0c1\uc740 \uc545\uc5b4\ub098 \ucf54\ubaa8\ub3c4\uc655\ub3c4\ub9c8\ubc40\uc774 \uba39\uc774\uac00 \ub9ce\uc740 \uc7a5\uc18c\uc5d0 \ubaa8\uc600\uc744 \ub54c \ubbf8\uc131\uc219 \uac1c\uccb4\uac00 \ub9ce\uc774 \uc8fd\ub294 \uac83\uc5d0\uc11c\ub3c4 \ubcf4\uc778\ub2e4. \uc774\ub294 \ubc14\ucee4\uac00 \ubc1c\uacac\ud55c \ud45c\ubcf8\uc5d0\ub3c4 \uc801\uc6a9\ub420 \uac83\uc774\ub2e4. \uc54c\ub85c\uc0ac\uc6b0\ub8e8\uc2a4\uac00 \ub3d9\uc871\uc744 \uc7a1\uc544\uba39\uc5c8\ub2e4\ub294 \uc99d\uac70\ub294 \uc788\uc73c\uba70, \uac08\ube44\ubf08 \uc870\uac01\uc5d0\uc11c \ubc1c\uacac\ub41c \uc774\ube68, \uacac\uac11\uace8\uc5d0 \ubc1c\uacac\ub41c \uc774\ube68\uc790\uad6d, \uadf8\ub9ac\uace0 \ubc14\ucee4\uac00 \ubc1c\uacac\ud55c \ud604\uc7a5\uc5d0\uc11c \ubc1c\uacac\ub41c \uc11c\ub85c \ud574\ucce4\ub358 \ud754\uc801\uc774 \uc788\ub294 \ubf08 \ub4f1\uc774\ub2e4."} {"question": "\ub3c4\ucfc4\ubb38\ud654\ud559\uc6d0 \ubcf4\uad00 \uc774\ud6c4 2015\ub144 6\uc6d424\uc77c \uc639\uc8fc\uc758 \uc720\ubb3c\uc774 \uc774\uc804\ub41c \uacf3?", "paragraph": "\uc639\uc8fc\uac00 \uc0ac\uc6a9\ud588\ub358 \uc720\ubb3c\uc740 \ud55c\uad6d\uacfc \uc77c\ubcf8\uc5d0\ub3c4 \uadf8\ub9ac \ub9ce\uc740 \uc591\uc740 \ub0a8\uc544\uc788\uc9c0 \uc54a\uc73c\ub098, \uc720\ubb3c\uc774 \uc18c\uc7ac\ud55c \uba87\uba87\uc758 \uc704\uce58\ub97c \uc0b4\ud3b4\ubcf4\uc790\uba74 \ub2e4\uc74c\uacfc \uac19\ub2e4. \uba3c\uc800 \ub3c4\ucfc4\ubb38\ud654\ud559\uc6d0\uc5d0\ub294 \uc870\uc120 \uc655\uac00\ub85c\ubd80\ud130 \uae30\uc99d \ubc1b\uc740 \uc639\uc8fc\uc758 \uc720\ubb3c\uc774 \ub0a8\uc544\uc788\ub294\ub370, \uadf8\ub140\uac00 \uc785\uc5c8\ub358 \ub2f9\uc758\ub098 \uadf8\ub140\uc758 \ub538 \ub9c8\uc0ac\uc5d0\uac00 \uc0ac\uc6a9\ud55c \uc720\uc544\ubcf5\uacfc \ub098\ub9c9\uc2e0 \ub4f1\uc774 \uc18c\uc7a5\ub418\uc5b4 \uc788\ub2e4. \uc774\uc911 \ubcf5\uc2dd 7\uc810(\ub2f9\uc758, \ud64d\uc0c9 \uc2a4\ub780\uce58\ub9c8, \uce58\ub9c8, \uc1a1\ud654\uc0c9 \uc219\uace0\uc0ac \ubc18\ud68c\uc7a5\uc800\uace0\ub9ac, \uc9c4\ubd84\ud64d \uc800\uace0\ub9ac, \ud48d\ucc28\ubc14\uc9c0, \ub2e8\uc18d\uacf3)\uc774 \ucd08\uc804\uc12c\uc720\ud03c\ud2b8\ubc15\ubb3c\uad00 \uad00\uc7a5\uc758 \uc624\ub79c \uc124\ub4dd \ub05d\uc5d0 2015\ub144 6\uc6d4 24\uc77c \ub300\ud55c\ubbfc\uad6d\uc5d0 \ubc18\ud658\ub418\uc5b4 \uad6d\ub9bd\uace0\uad81\ubc15\ubb3c\uad00\uc5d0 \ubcf4\uad00\ub418\uc5c8\ub2e4. \ud050\uc288\uad6d\ub9bd\ubc15\ubb3c\uad00\uc5d0\ub294 \uc639\uc8fc\uc758 \ub0a8\ud3b8\uc774\uc5c8\ub358 \uc18c \ub2e4\ucf00\uc720\ud0a4\uc758 \uc9d1\uc548\uc778 \uc4f0\uc2dc\ub9c8 \uc885\uac00 \uc9d1\uc548\uc5d0\uc11c \uae30\uc99d\ubc1b\uc740 \uc720\ubb3c\uc774 \ub0a8\uc544\uc788\ub294\ub370, \uc639\uc8fc\uac00 \uc0ac\uc6a9\ud55c \uc2dd\uae30, \uc637\uac10, \ud68c\ud654 \ub4f1\uc774 \uc804\uc2dc\ub418\uc5b4 \uc788\ub2e4.", "answer": "\uad6d\ub9bd\uace0\uad81\ubc15\ubb3c\uad00", "sentence": "\uc774\uc911 \ubcf5\uc2dd 7\uc810(\ub2f9\uc758, \ud64d\uc0c9 \uc2a4\ub780\uce58\ub9c8, \uce58\ub9c8, \uc1a1\ud654\uc0c9 \uc219\uace0\uc0ac \ubc18\ud68c\uc7a5\uc800\uace0\ub9ac, \uc9c4\ubd84\ud64d \uc800\uace0\ub9ac, \ud48d\ucc28\ubc14\uc9c0, \ub2e8\uc18d\uacf3)\uc774 \ucd08\uc804\uc12c\uc720\ud03c\ud2b8\ubc15\ubb3c\uad00 \uad00\uc7a5\uc758 \uc624\ub79c \uc124\ub4dd \ub05d\uc5d0 2015\ub144 6\uc6d4 24\uc77c \ub300\ud55c\ubbfc\uad6d\uc5d0 \ubc18\ud658\ub418\uc5b4 \uad6d\ub9bd\uace0\uad81\ubc15\ubb3c\uad00 \uc5d0 \ubcf4\uad00\ub418\uc5c8\ub2e4.", "paragraph_sentence": "\uc639\uc8fc\uac00 \uc0ac\uc6a9\ud588\ub358 \uc720\ubb3c\uc740 \ud55c\uad6d\uacfc \uc77c\ubcf8\uc5d0\ub3c4 \uadf8\ub9ac \ub9ce\uc740 \uc591\uc740 \ub0a8\uc544\uc788\uc9c0 \uc54a\uc73c\ub098, \uc720\ubb3c\uc774 \uc18c\uc7ac\ud55c \uba87\uba87\uc758 \uc704\uce58\ub97c \uc0b4\ud3b4\ubcf4\uc790\uba74 \ub2e4\uc74c\uacfc \uac19\ub2e4. \uba3c\uc800 \ub3c4\ucfc4\ubb38\ud654\ud559\uc6d0\uc5d0\ub294 \uc870\uc120 \uc655\uac00\ub85c\ubd80\ud130 \uae30\uc99d \ubc1b\uc740 \uc639\uc8fc\uc758 \uc720\ubb3c\uc774 \ub0a8\uc544\uc788\ub294\ub370, \uadf8\ub140\uac00 \uc785\uc5c8\ub358 \ub2f9\uc758\ub098 \uadf8\ub140\uc758 \ub538 \ub9c8\uc0ac\uc5d0\uac00 \uc0ac\uc6a9\ud55c \uc720\uc544\ubcf5\uacfc \ub098\ub9c9\uc2e0 \ub4f1\uc774 \uc18c\uc7a5\ub418\uc5b4 \uc788\ub2e4. \uc774\uc911 \ubcf5\uc2dd 7\uc810(\ub2f9\uc758, \ud64d\uc0c9 \uc2a4\ub780\uce58\ub9c8, \uce58\ub9c8, \uc1a1\ud654\uc0c9 \uc219\uace0\uc0ac \ubc18\ud68c\uc7a5\uc800\uace0\ub9ac, \uc9c4\ubd84\ud64d \uc800\uace0\ub9ac, \ud48d\ucc28\ubc14\uc9c0, \ub2e8\uc18d\uacf3)\uc774 \ucd08\uc804\uc12c\uc720\ud03c\ud2b8\ubc15\ubb3c\uad00 \uad00\uc7a5\uc758 \uc624\ub79c \uc124\ub4dd \ub05d\uc5d0 2015\ub144 6\uc6d4 24\uc77c \ub300\ud55c\ubbfc\uad6d\uc5d0 \ubc18\ud658\ub418\uc5b4 \uad6d\ub9bd\uace0\uad81\ubc15\ubb3c\uad00 \uc5d0 \ubcf4\uad00\ub418\uc5c8\ub2e4. \ud050\uc288\uad6d\ub9bd\ubc15\ubb3c\uad00\uc5d0\ub294 \uc639\uc8fc\uc758 \ub0a8\ud3b8\uc774\uc5c8\ub358 \uc18c \ub2e4\ucf00\uc720\ud0a4\uc758 \uc9d1\uc548\uc778 \uc4f0\uc2dc\ub9c8 \uc885\uac00 \uc9d1\uc548\uc5d0\uc11c \uae30\uc99d\ubc1b\uc740 \uc720\ubb3c\uc774 \ub0a8\uc544\uc788\ub294\ub370, \uc639\uc8fc\uac00 \uc0ac\uc6a9\ud55c \uc2dd\uae30, \uc637\uac10, \ud68c\ud654 \ub4f1\uc774 \uc804\uc2dc\ub418\uc5b4 \uc788\ub2e4.", "paragraph_answer": "\uc639\uc8fc\uac00 \uc0ac\uc6a9\ud588\ub358 \uc720\ubb3c\uc740 \ud55c\uad6d\uacfc \uc77c\ubcf8\uc5d0\ub3c4 \uadf8\ub9ac \ub9ce\uc740 \uc591\uc740 \ub0a8\uc544\uc788\uc9c0 \uc54a\uc73c\ub098, \uc720\ubb3c\uc774 \uc18c\uc7ac\ud55c \uba87\uba87\uc758 \uc704\uce58\ub97c \uc0b4\ud3b4\ubcf4\uc790\uba74 \ub2e4\uc74c\uacfc \uac19\ub2e4. \uba3c\uc800 \ub3c4\ucfc4\ubb38\ud654\ud559\uc6d0\uc5d0\ub294 \uc870\uc120 \uc655\uac00\ub85c\ubd80\ud130 \uae30\uc99d \ubc1b\uc740 \uc639\uc8fc\uc758 \uc720\ubb3c\uc774 \ub0a8\uc544\uc788\ub294\ub370, \uadf8\ub140\uac00 \uc785\uc5c8\ub358 \ub2f9\uc758\ub098 \uadf8\ub140\uc758 \ub538 \ub9c8\uc0ac\uc5d0\uac00 \uc0ac\uc6a9\ud55c \uc720\uc544\ubcf5\uacfc \ub098\ub9c9\uc2e0 \ub4f1\uc774 \uc18c\uc7a5\ub418\uc5b4 \uc788\ub2e4. \uc774\uc911 \ubcf5\uc2dd 7\uc810(\ub2f9\uc758, \ud64d\uc0c9 \uc2a4\ub780\uce58\ub9c8, \uce58\ub9c8, \uc1a1\ud654\uc0c9 \uc219\uace0\uc0ac \ubc18\ud68c\uc7a5\uc800\uace0\ub9ac, \uc9c4\ubd84\ud64d \uc800\uace0\ub9ac, \ud48d\ucc28\ubc14\uc9c0, \ub2e8\uc18d\uacf3)\uc774 \ucd08\uc804\uc12c\uc720\ud03c\ud2b8\ubc15\ubb3c\uad00 \uad00\uc7a5\uc758 \uc624\ub79c \uc124\ub4dd \ub05d\uc5d0 2015\ub144 6\uc6d4 24\uc77c \ub300\ud55c\ubbfc\uad6d\uc5d0 \ubc18\ud658\ub418\uc5b4 \uad6d\ub9bd\uace0\uad81\ubc15\ubb3c\uad00 \uc5d0 \ubcf4\uad00\ub418\uc5c8\ub2e4. \ud050\uc288\uad6d\ub9bd\ubc15\ubb3c\uad00\uc5d0\ub294 \uc639\uc8fc\uc758 \ub0a8\ud3b8\uc774\uc5c8\ub358 \uc18c \ub2e4\ucf00\uc720\ud0a4\uc758 \uc9d1\uc548\uc778 \uc4f0\uc2dc\ub9c8 \uc885\uac00 \uc9d1\uc548\uc5d0\uc11c \uae30\uc99d\ubc1b\uc740 \uc720\ubb3c\uc774 \ub0a8\uc544\uc788\ub294\ub370, \uc639\uc8fc\uac00 \uc0ac\uc6a9\ud55c \uc2dd\uae30, \uc637\uac10, \ud68c\ud654 \ub4f1\uc774 \uc804\uc2dc\ub418\uc5b4 \uc788\ub2e4.", "sentence_answer": "\uc774\uc911 \ubcf5\uc2dd 7\uc810(\ub2f9\uc758, \ud64d\uc0c9 \uc2a4\ub780\uce58\ub9c8, \uce58\ub9c8, \uc1a1\ud654\uc0c9 \uc219\uace0\uc0ac \ubc18\ud68c\uc7a5\uc800\uace0\ub9ac, \uc9c4\ubd84\ud64d \uc800\uace0\ub9ac, \ud48d\ucc28\ubc14\uc9c0, \ub2e8\uc18d\uacf3)\uc774 \ucd08\uc804\uc12c\uc720\ud03c\ud2b8\ubc15\ubb3c\uad00 \uad00\uc7a5\uc758 \uc624\ub79c \uc124\ub4dd \ub05d\uc5d0 2015\ub144 6\uc6d4 24\uc77c \ub300\ud55c\ubbfc\uad6d\uc5d0 \ubc18\ud658\ub418\uc5b4 \uad6d\ub9bd\uace0\uad81\ubc15\ubb3c\uad00 \uc5d0 \ubcf4\uad00\ub418\uc5c8\ub2e4.", "paragraph_id": "6292399-6-1", "paragraph_question": "question: \ub3c4\ucfc4\ubb38\ud654\ud559\uc6d0 \ubcf4\uad00 \uc774\ud6c4 2015\ub144 6\uc6d424\uc77c \uc639\uc8fc\uc758 \uc720\ubb3c\uc774 \uc774\uc804\ub41c \uacf3?, context: \uc639\uc8fc\uac00 \uc0ac\uc6a9\ud588\ub358 \uc720\ubb3c\uc740 \ud55c\uad6d\uacfc \uc77c\ubcf8\uc5d0\ub3c4 \uadf8\ub9ac \ub9ce\uc740 \uc591\uc740 \ub0a8\uc544\uc788\uc9c0 \uc54a\uc73c\ub098, \uc720\ubb3c\uc774 \uc18c\uc7ac\ud55c \uba87\uba87\uc758 \uc704\uce58\ub97c \uc0b4\ud3b4\ubcf4\uc790\uba74 \ub2e4\uc74c\uacfc \uac19\ub2e4. \uba3c\uc800 \ub3c4\ucfc4\ubb38\ud654\ud559\uc6d0\uc5d0\ub294 \uc870\uc120 \uc655\uac00\ub85c\ubd80\ud130 \uae30\uc99d \ubc1b\uc740 \uc639\uc8fc\uc758 \uc720\ubb3c\uc774 \ub0a8\uc544\uc788\ub294\ub370, \uadf8\ub140\uac00 \uc785\uc5c8\ub358 \ub2f9\uc758\ub098 \uadf8\ub140\uc758 \ub538 \ub9c8\uc0ac\uc5d0\uac00 \uc0ac\uc6a9\ud55c \uc720\uc544\ubcf5\uacfc \ub098\ub9c9\uc2e0 \ub4f1\uc774 \uc18c\uc7a5\ub418\uc5b4 \uc788\ub2e4. \uc774\uc911 \ubcf5\uc2dd 7\uc810(\ub2f9\uc758, \ud64d\uc0c9 \uc2a4\ub780\uce58\ub9c8, \uce58\ub9c8, \uc1a1\ud654\uc0c9 \uc219\uace0\uc0ac \ubc18\ud68c\uc7a5\uc800\uace0\ub9ac, \uc9c4\ubd84\ud64d \uc800\uace0\ub9ac, \ud48d\ucc28\ubc14\uc9c0, \ub2e8\uc18d\uacf3)\uc774 \ucd08\uc804\uc12c\uc720\ud03c\ud2b8\ubc15\ubb3c\uad00 \uad00\uc7a5\uc758 \uc624\ub79c \uc124\ub4dd \ub05d\uc5d0 2015\ub144 6\uc6d4 24\uc77c \ub300\ud55c\ubbfc\uad6d\uc5d0 \ubc18\ud658\ub418\uc5b4 \uad6d\ub9bd\uace0\uad81\ubc15\ubb3c\uad00\uc5d0 \ubcf4\uad00\ub418\uc5c8\ub2e4. \ud050\uc288\uad6d\ub9bd\ubc15\ubb3c\uad00\uc5d0\ub294 \uc639\uc8fc\uc758 \ub0a8\ud3b8\uc774\uc5c8\ub358 \uc18c \ub2e4\ucf00\uc720\ud0a4\uc758 \uc9d1\uc548\uc778 \uc4f0\uc2dc\ub9c8 \uc885\uac00 \uc9d1\uc548\uc5d0\uc11c \uae30\uc99d\ubc1b\uc740 \uc720\ubb3c\uc774 \ub0a8\uc544\uc788\ub294\ub370, \uc639\uc8fc\uac00 \uc0ac\uc6a9\ud55c \uc2dd\uae30, \uc637\uac10, \ud68c\ud654 \ub4f1\uc774 \uc804\uc2dc\ub418\uc5b4 \uc788\ub2e4."} {"question": "1977\ub144 \uc2dc\uc98c \uc885\ub8cc \ud6c4 \ub09c\uce74\uc774 \ud300\uc5d0\uc11c \ud574\uc784\ub41c \uac10\ub3c5\uc740?", "paragraph": "1977\ub144 \uc2dc\uc98c \uc885\ub8cc \ud6c4 \ub178\ubb34\ub77c\uac00 \ub09c\uce74\uc774 \uac10\ub3c5\uc9c1\uc5d0\uc11c \ud574\uc784\ub418\uc790 \u201c\ub178\ubb34\ub77c \uac10\ub3c5\uc774 \uadf8\ub9cc\ub450\ub294 \uc774\uc0c1 \uc790\uc2e0\ub3c4 \ub0b4\ubcf4\ub0b4\ub2ec\ub77c\u201d\uace0 \ub9d0\ud588\ub2e4. \uacb0\uad6d \ud604\uae08 \ud2b8\ub808\uc774\ub4dc\ub85c \ud788\ub85c\uc2dc\ub9c8 \ub3c4\uc694 \uce74\ud504\uc5d0 \uc774\uc801\ud55c \uc774\ud6c4 \ud788\ub85c\uc2dc\ub9c8 \uad6c\uc6d0 \ud22c\uc218\uc9c4\uc758 \uc5d0\uc774\uc2a4\ub85c \ud65c\uc57d\ud588\ub2e4. \ud788\ub85c\uc2dc\ub9c8 \uc2dc\uc808\uc5d0 \uc5d0\ub098\uc4f0\uc758 \ud22c\uad6c \uae30\uc220\uc740 \ud55c\uce35 \ub354 \uc608\ub9ac\ud574\uc838 \ud0c0\uc790\uc758 \uc2ec\ub9ac\ub97c \uc77d\uc5b4\ub0b8 \uc5f0\ud6c4\uc5d0 \uacf5\uc758 \uc2a4\ud53c\ub4dc\ubfd0\ub9cc \uc544\ub2c8\ub77c \ubaa8\uc158 \ubcc0\ud654\ub85c\ub3c4 \uc644\uae09\uc744 \ub354\ud568\uc73c\ub85c\uc368 \ud0c0\uc790\ub4e4\uc744 \ub18d\ub77d\ud588\ub2e4. \uc774\ub85c\uc368 \ud300\uc774 1979\ub144\uacfc 1980\ub144\uc5d0\ub294 \ud788\ub85c\uc2dc\ub9c8\uc758 2\ub144 \uc5f0\uc18d \uc77c\ubcf8 \uc2dc\ub9ac\uc988 \uc81c\ud328\uc5d0 \ud06c\uac8c \uae30\uc5ec\ub97c \ud588\uace0 \u2018\ube68\uac04 \ud5ec\uba67 \ud669\uae08\uc2dc\ub300\u2019\ub97c \uad6c\ucd95\ud558\ub294 \uc6d0\ub3d9\ub825\uc774 \ub410\ub2e4. 1979\ub144\uc5d0\ub294 \ubcf8\uc778\uc5d0\uac8c \uc788\uc5b4\uc11c\ub294 \ucc98\uc74c\uc774\uc790 \uad6c\uc6d0 \ud22c\uc218\ub85c\ub294 \ucc98\uc74c\uc73c\ub85c \ucd5c\uc6b0\uc218 \uc120\uc218\ub85c \uc120\uc815\ub418\ub294 \uc601\uc608\ub3c4 \uc548\uc558\ub2e4.", "answer": "\ub178\ubb34\ub77c", "sentence": "1977\ub144 \uc2dc\uc98c \uc885\ub8cc \ud6c4 \ub178\ubb34\ub77c \uac00 \ub09c\uce74\uc774 \uac10\ub3c5\uc9c1\uc5d0\uc11c \ud574\uc784\ub418\uc790 \u201c\ub178\ubb34\ub77c \uac10\ub3c5\uc774 \uadf8\ub9cc\ub450\ub294 \uc774\uc0c1 \uc790\uc2e0\ub3c4 \ub0b4\ubcf4\ub0b4\ub2ec\ub77c\u201d\uace0 \ub9d0\ud588\ub2e4.", "paragraph_sentence": " 1977\ub144 \uc2dc\uc98c \uc885\ub8cc \ud6c4 \ub178\ubb34\ub77c \uac00 \ub09c\uce74\uc774 \uac10\ub3c5\uc9c1\uc5d0\uc11c \ud574\uc784\ub418\uc790 \u201c\ub178\ubb34\ub77c \uac10\ub3c5\uc774 \uadf8\ub9cc\ub450\ub294 \uc774\uc0c1 \uc790\uc2e0\ub3c4 \ub0b4\ubcf4\ub0b4\ub2ec\ub77c\u201d\uace0 \ub9d0\ud588\ub2e4. \uacb0\uad6d \ud604\uae08 \ud2b8\ub808\uc774\ub4dc\ub85c \ud788\ub85c\uc2dc\ub9c8 \ub3c4\uc694 \uce74\ud504\uc5d0 \uc774\uc801\ud55c \uc774\ud6c4 \ud788\ub85c\uc2dc\ub9c8 \uad6c\uc6d0 \ud22c\uc218\uc9c4\uc758 \uc5d0\uc774\uc2a4\ub85c \ud65c\uc57d\ud588\ub2e4. \ud788\ub85c\uc2dc\ub9c8 \uc2dc\uc808\uc5d0 \uc5d0\ub098\uc4f0\uc758 \ud22c\uad6c \uae30\uc220\uc740 \ud55c\uce35 \ub354 \uc608\ub9ac\ud574\uc838 \ud0c0\uc790\uc758 \uc2ec\ub9ac\ub97c \uc77d\uc5b4\ub0b8 \uc5f0\ud6c4\uc5d0 \uacf5\uc758 \uc2a4\ud53c\ub4dc\ubfd0\ub9cc \uc544\ub2c8\ub77c \ubaa8\uc158 \ubcc0\ud654\ub85c\ub3c4 \uc644\uae09\uc744 \ub354\ud568\uc73c\ub85c\uc368 \ud0c0\uc790\ub4e4\uc744 \ub18d\ub77d\ud588\ub2e4. \uc774\ub85c\uc368 \ud300\uc774 1979\ub144\uacfc 1980\ub144\uc5d0\ub294 \ud788\ub85c\uc2dc\ub9c8\uc758 2\ub144 \uc5f0\uc18d \uc77c\ubcf8 \uc2dc\ub9ac\uc988 \uc81c\ud328\uc5d0 \ud06c\uac8c \uae30\uc5ec\ub97c \ud588\uace0 \u2018\ube68\uac04 \ud5ec\uba67 \ud669\uae08\uc2dc\ub300\u2019\ub97c \uad6c\ucd95\ud558\ub294 \uc6d0\ub3d9\ub825\uc774 \ub410\ub2e4. 1979\ub144\uc5d0\ub294 \ubcf8\uc778\uc5d0\uac8c \uc788\uc5b4\uc11c\ub294 \ucc98\uc74c\uc774\uc790 \uad6c\uc6d0 \ud22c\uc218\ub85c\ub294 \ucc98\uc74c\uc73c\ub85c \ucd5c\uc6b0\uc218 \uc120\uc218\ub85c \uc120\uc815\ub418\ub294 \uc601\uc608\ub3c4 \uc548\uc558\ub2e4.", "paragraph_answer": "1977\ub144 \uc2dc\uc98c \uc885\ub8cc \ud6c4 \ub178\ubb34\ub77c \uac00 \ub09c\uce74\uc774 \uac10\ub3c5\uc9c1\uc5d0\uc11c \ud574\uc784\ub418\uc790 \u201c\ub178\ubb34\ub77c \uac10\ub3c5\uc774 \uadf8\ub9cc\ub450\ub294 \uc774\uc0c1 \uc790\uc2e0\ub3c4 \ub0b4\ubcf4\ub0b4\ub2ec\ub77c\u201d\uace0 \ub9d0\ud588\ub2e4. \uacb0\uad6d \ud604\uae08 \ud2b8\ub808\uc774\ub4dc\ub85c \ud788\ub85c\uc2dc\ub9c8 \ub3c4\uc694 \uce74\ud504\uc5d0 \uc774\uc801\ud55c \uc774\ud6c4 \ud788\ub85c\uc2dc\ub9c8 \uad6c\uc6d0 \ud22c\uc218\uc9c4\uc758 \uc5d0\uc774\uc2a4\ub85c \ud65c\uc57d\ud588\ub2e4. \ud788\ub85c\uc2dc\ub9c8 \uc2dc\uc808\uc5d0 \uc5d0\ub098\uc4f0\uc758 \ud22c\uad6c \uae30\uc220\uc740 \ud55c\uce35 \ub354 \uc608\ub9ac\ud574\uc838 \ud0c0\uc790\uc758 \uc2ec\ub9ac\ub97c \uc77d\uc5b4\ub0b8 \uc5f0\ud6c4\uc5d0 \uacf5\uc758 \uc2a4\ud53c\ub4dc\ubfd0\ub9cc \uc544\ub2c8\ub77c \ubaa8\uc158 \ubcc0\ud654\ub85c\ub3c4 \uc644\uae09\uc744 \ub354\ud568\uc73c\ub85c\uc368 \ud0c0\uc790\ub4e4\uc744 \ub18d\ub77d\ud588\ub2e4. \uc774\ub85c\uc368 \ud300\uc774 1979\ub144\uacfc 1980\ub144\uc5d0\ub294 \ud788\ub85c\uc2dc\ub9c8\uc758 2\ub144 \uc5f0\uc18d \uc77c\ubcf8 \uc2dc\ub9ac\uc988 \uc81c\ud328\uc5d0 \ud06c\uac8c \uae30\uc5ec\ub97c \ud588\uace0 \u2018\ube68\uac04 \ud5ec\uba67 \ud669\uae08\uc2dc\ub300\u2019\ub97c \uad6c\ucd95\ud558\ub294 \uc6d0\ub3d9\ub825\uc774 \ub410\ub2e4. 1979\ub144\uc5d0\ub294 \ubcf8\uc778\uc5d0\uac8c \uc788\uc5b4\uc11c\ub294 \ucc98\uc74c\uc774\uc790 \uad6c\uc6d0 \ud22c\uc218\ub85c\ub294 \ucc98\uc74c\uc73c\ub85c \ucd5c\uc6b0\uc218 \uc120\uc218\ub85c \uc120\uc815\ub418\ub294 \uc601\uc608\ub3c4 \uc548\uc558\ub2e4.", "sentence_answer": "1977\ub144 \uc2dc\uc98c \uc885\ub8cc \ud6c4 \ub178\ubb34\ub77c \uac00 \ub09c\uce74\uc774 \uac10\ub3c5\uc9c1\uc5d0\uc11c \ud574\uc784\ub418\uc790 \u201c\ub178\ubb34\ub77c \uac10\ub3c5\uc774 \uadf8\ub9cc\ub450\ub294 \uc774\uc0c1 \uc790\uc2e0\ub3c4 \ub0b4\ubcf4\ub0b4\ub2ec\ub77c\u201d\uace0 \ub9d0\ud588\ub2e4.", "paragraph_id": "6522992-10-1", "paragraph_question": "question: 1977\ub144 \uc2dc\uc98c \uc885\ub8cc \ud6c4 \ub09c\uce74\uc774 \ud300\uc5d0\uc11c \ud574\uc784\ub41c \uac10\ub3c5\uc740?, context: 1977\ub144 \uc2dc\uc98c \uc885\ub8cc \ud6c4 \ub178\ubb34\ub77c\uac00 \ub09c\uce74\uc774 \uac10\ub3c5\uc9c1\uc5d0\uc11c \ud574\uc784\ub418\uc790 \u201c\ub178\ubb34\ub77c \uac10\ub3c5\uc774 \uadf8\ub9cc\ub450\ub294 \uc774\uc0c1 \uc790\uc2e0\ub3c4 \ub0b4\ubcf4\ub0b4\ub2ec\ub77c\u201d\uace0 \ub9d0\ud588\ub2e4. \uacb0\uad6d \ud604\uae08 \ud2b8\ub808\uc774\ub4dc\ub85c \ud788\ub85c\uc2dc\ub9c8 \ub3c4\uc694 \uce74\ud504\uc5d0 \uc774\uc801\ud55c \uc774\ud6c4 \ud788\ub85c\uc2dc\ub9c8 \uad6c\uc6d0 \ud22c\uc218\uc9c4\uc758 \uc5d0\uc774\uc2a4\ub85c \ud65c\uc57d\ud588\ub2e4. \ud788\ub85c\uc2dc\ub9c8 \uc2dc\uc808\uc5d0 \uc5d0\ub098\uc4f0\uc758 \ud22c\uad6c \uae30\uc220\uc740 \ud55c\uce35 \ub354 \uc608\ub9ac\ud574\uc838 \ud0c0\uc790\uc758 \uc2ec\ub9ac\ub97c \uc77d\uc5b4\ub0b8 \uc5f0\ud6c4\uc5d0 \uacf5\uc758 \uc2a4\ud53c\ub4dc\ubfd0\ub9cc \uc544\ub2c8\ub77c \ubaa8\uc158 \ubcc0\ud654\ub85c\ub3c4 \uc644\uae09\uc744 \ub354\ud568\uc73c\ub85c\uc368 \ud0c0\uc790\ub4e4\uc744 \ub18d\ub77d\ud588\ub2e4. \uc774\ub85c\uc368 \ud300\uc774 1979\ub144\uacfc 1980\ub144\uc5d0\ub294 \ud788\ub85c\uc2dc\ub9c8\uc758 2\ub144 \uc5f0\uc18d \uc77c\ubcf8 \uc2dc\ub9ac\uc988 \uc81c\ud328\uc5d0 \ud06c\uac8c \uae30\uc5ec\ub97c \ud588\uace0 \u2018\ube68\uac04 \ud5ec\uba67 \ud669\uae08\uc2dc\ub300\u2019\ub97c \uad6c\ucd95\ud558\ub294 \uc6d0\ub3d9\ub825\uc774 \ub410\ub2e4. 1979\ub144\uc5d0\ub294 \ubcf8\uc778\uc5d0\uac8c \uc788\uc5b4\uc11c\ub294 \ucc98\uc74c\uc774\uc790 \uad6c\uc6d0 \ud22c\uc218\ub85c\ub294 \ucc98\uc74c\uc73c\ub85c \ucd5c\uc6b0\uc218 \uc120\uc218\ub85c \uc120\uc815\ub418\ub294 \uc601\uc608\ub3c4 \uc548\uc558\ub2e4."} {"question": "\uc81c2\ucc28 \uc138\uacc4 \ub300\uc804\uc774 \uc77c\uc5b4\ub098\uc790 \uc5f0\ud569\uad6d\uc73c\ub85c \ucc38\uc804\ud55c \ube0c\ub77c\uc9c8\uc774 \uad6d\uad50\ub97c \ub2e8\uc808\ud588\ub358 \ub098\ub77c\ub294?", "paragraph": "\uc5ec\ub7ec \uc8fc \ub3d9\uc548 \ubc30\ub97c \ud0c0\uace0 \ube0c\ub77c\uc9c8\uae4c\uc9c0 \uac74\ub108\uac04 \uc77c\ubcf8\uc778 \uc774\uc8fc\uc790\ub4e4\uc740 \uc0c1\ud30c\uc6b8\ub8e8 \uc8fc\ubcc0\uc758 \ucee4\ud53c \ub18d\uc7a5\uc5d0 \ud2b9\ud788 \uc9d1\uc911\uc801\uc73c\ub85c \uc815\ucc29\ud558\uc600\uace0, \uc77c\ubcf8\uacfc \uc644\uc804\ud788 \ub2e4\ub978 \ud48d\ud1a0\uc5d0 \uc778\uc885\ucc28\ubcc4\uae4c\uc9c0 \ub2f9\ud558\ub294 \ub4f1 \uc545\uc870\uac74 \uc18d\uc5d0\uc11c \uac16\uc740 \uace0\uc0dd\uc744 \ud558\uba70 \uae30\ubc18\uc744 \ub2e6\uc544 \ub098\uac00 \ucc28\uce30 \uacbd\uc81c\uc801\uc73c\ub85c \uc131\uacf5\uc744 \uac70\ub450\uba70, \ube0c\ub77c\uc9c8 \uc774\ubbfc\uc0ac\ud68c\uc758 \ubaa8\ubc94\uc774 \ub418\uc5c8\ub2e4. \uadf8\ub7ec\ub294 \uac00\uc6b4\ub370 \uc81c2\ucc28 \uc138\uacc4 \ub300\uc804\uc774 \uc77c\uc5b4\ub098\uc790 \uc5f0\ud569\uad6d\uc73c\ub85c \ucc38\uc804\ud55c \ube0c\ub77c\uc9c8\uc740 \uc77c\ubcf8\uacfc \uad6d\uad50\ub97c \ub2e8\uc808\ud588\uace0, \uc77c\ubcf8\uc5b4 \uc2e0\ubb38\uc758 \ubc1c\uac04\uacfc \uc77c\ubcf8\uc778 \ud559\uad50\uc758 \uc124\ub9bd\uc744 \uae08\uc9c0\ud558\ub294 \ub4f1 \uc77c\ubcf8\uc778\ub4e4\uc744 \ud0c4\uc555\ud558\uae30\ub3c4 \ud558\uc600\ub2e4. \uc804\ud6c4 \ud0c4\uc555\uc740 \ub05d\ub098\uace0 \uc774\ubbfc\ub3c4 \uc7ac\uac1c\ub418\uc5c8\uc73c\ub098, \uc77c\ubcf8\uc758 \uacbd\uc81c\uc801 \uc131\uc7a5\uc774 \uacc4\uc18d\ub418\uba74\uc11c, \uadf8 \uc774\uc804\ubcf4\ub2e4\ub294 \ud06c\uac8c \uc218\uac00 \uc904\uc5b4\ub4e4\uc5c8\ub2e4. \uadf8\ub7ec\ub098 \ube0c\ub77c\uc9c8\uc5d0 \uc644\uc804\ud788 \uc815\ucc29\ud55c \uc77c\ubcf8\uacc4 \ube0c\ub77c\uc9c8\uc778\ub4e4\uc740 \ud2b9\uc720\uc758 \uadfc\uba74\ud568\uc73c\ub85c \ub354\uc6b1 \uae30\ubc18\uc744 \ub2e6\uc544 \ub098\uac14\uc73c\uba70, \uc77c\ubd80\ub294 \uc544\ub974\ud5e8\ud2f0\ub098 \ub4f1\uc758 \uc8fc\ubcc0\uad6d\uc73c\ub85c\uae4c\uc9c0 \uc9c4\ucd9c\ud558\uae30\ub3c4 \ud558\uc600\ub2e4.", "answer": "\uc77c\ubcf8", "sentence": "\uc5ec\ub7ec \uc8fc \ub3d9\uc548 \ubc30\ub97c \ud0c0\uace0 \ube0c\ub77c\uc9c8\uae4c\uc9c0 \uac74\ub108\uac04 \uc77c\ubcf8 \uc778 \uc774\uc8fc\uc790\ub4e4\uc740 \uc0c1\ud30c\uc6b8\ub8e8 \uc8fc\ubcc0\uc758 \ucee4\ud53c \ub18d\uc7a5\uc5d0 \ud2b9\ud788 \uc9d1\uc911\uc801\uc73c\ub85c \uc815\ucc29\ud558\uc600\uace0, \uc77c\ubcf8\uacfc \uc644\uc804\ud788 \ub2e4\ub978 \ud48d\ud1a0\uc5d0 \uc778\uc885\ucc28\ubcc4\uae4c\uc9c0 \ub2f9\ud558\ub294 \ub4f1 \uc545\uc870\uac74 \uc18d\uc5d0\uc11c \uac16\uc740 \uace0\uc0dd\uc744 \ud558\uba70 \uae30\ubc18\uc744 \ub2e6\uc544 \ub098\uac00 \ucc28\uce30 \uacbd\uc81c\uc801\uc73c\ub85c \uc131\uacf5\uc744 \uac70\ub450\uba70, \ube0c\ub77c\uc9c8 \uc774\ubbfc\uc0ac\ud68c\uc758 \ubaa8\ubc94\uc774 \ub418\uc5c8\ub2e4.", "paragraph_sentence": " \uc5ec\ub7ec \uc8fc \ub3d9\uc548 \ubc30\ub97c \ud0c0\uace0 \ube0c\ub77c\uc9c8\uae4c\uc9c0 \uac74\ub108\uac04 \uc77c\ubcf8 \uc778 \uc774\uc8fc\uc790\ub4e4\uc740 \uc0c1\ud30c\uc6b8\ub8e8 \uc8fc\ubcc0\uc758 \ucee4\ud53c \ub18d\uc7a5\uc5d0 \ud2b9\ud788 \uc9d1\uc911\uc801\uc73c\ub85c \uc815\ucc29\ud558\uc600\uace0, \uc77c\ubcf8\uacfc \uc644\uc804\ud788 \ub2e4\ub978 \ud48d\ud1a0\uc5d0 \uc778\uc885\ucc28\ubcc4\uae4c\uc9c0 \ub2f9\ud558\ub294 \ub4f1 \uc545\uc870\uac74 \uc18d\uc5d0\uc11c \uac16\uc740 \uace0\uc0dd\uc744 \ud558\uba70 \uae30\ubc18\uc744 \ub2e6\uc544 \ub098\uac00 \ucc28\uce30 \uacbd\uc81c\uc801\uc73c\ub85c \uc131\uacf5\uc744 \uac70\ub450\uba70, \ube0c\ub77c\uc9c8 \uc774\ubbfc\uc0ac\ud68c\uc758 \ubaa8\ubc94\uc774 \ub418\uc5c8\ub2e4. \uadf8\ub7ec\ub294 \uac00\uc6b4\ub370 \uc81c2\ucc28 \uc138\uacc4 \ub300\uc804\uc774 \uc77c\uc5b4\ub098\uc790 \uc5f0\ud569\uad6d\uc73c\ub85c \ucc38\uc804\ud55c \ube0c\ub77c\uc9c8\uc740 \uc77c\ubcf8\uacfc \uad6d\uad50\ub97c \ub2e8\uc808\ud588\uace0, \uc77c\ubcf8\uc5b4 \uc2e0\ubb38\uc758 \ubc1c\uac04\uacfc \uc77c\ubcf8\uc778 \ud559\uad50\uc758 \uc124\ub9bd\uc744 \uae08\uc9c0\ud558\ub294 \ub4f1 \uc77c\ubcf8\uc778\ub4e4\uc744 \ud0c4\uc555\ud558\uae30\ub3c4 \ud558\uc600\ub2e4. \uc804\ud6c4 \ud0c4\uc555\uc740 \ub05d\ub098\uace0 \uc774\ubbfc\ub3c4 \uc7ac\uac1c\ub418\uc5c8\uc73c\ub098, \uc77c\ubcf8\uc758 \uacbd\uc81c\uc801 \uc131\uc7a5\uc774 \uacc4\uc18d\ub418\uba74\uc11c, \uadf8 \uc774\uc804\ubcf4\ub2e4\ub294 \ud06c\uac8c \uc218\uac00 \uc904\uc5b4\ub4e4\uc5c8\ub2e4. \uadf8\ub7ec\ub098 \ube0c\ub77c\uc9c8\uc5d0 \uc644\uc804\ud788 \uc815\ucc29\ud55c \uc77c\ubcf8\uacc4 \ube0c\ub77c\uc9c8\uc778\ub4e4\uc740 \ud2b9\uc720\uc758 \uadfc\uba74\ud568\uc73c\ub85c \ub354\uc6b1 \uae30\ubc18\uc744 \ub2e6\uc544 \ub098\uac14\uc73c\uba70, \uc77c\ubd80\ub294 \uc544\ub974\ud5e8\ud2f0\ub098 \ub4f1\uc758 \uc8fc\ubcc0\uad6d\uc73c\ub85c\uae4c\uc9c0 \uc9c4\ucd9c\ud558\uae30\ub3c4 \ud558\uc600\ub2e4.", "paragraph_answer": "\uc5ec\ub7ec \uc8fc \ub3d9\uc548 \ubc30\ub97c \ud0c0\uace0 \ube0c\ub77c\uc9c8\uae4c\uc9c0 \uac74\ub108\uac04 \uc77c\ubcf8 \uc778 \uc774\uc8fc\uc790\ub4e4\uc740 \uc0c1\ud30c\uc6b8\ub8e8 \uc8fc\ubcc0\uc758 \ucee4\ud53c \ub18d\uc7a5\uc5d0 \ud2b9\ud788 \uc9d1\uc911\uc801\uc73c\ub85c \uc815\ucc29\ud558\uc600\uace0, \uc77c\ubcf8\uacfc \uc644\uc804\ud788 \ub2e4\ub978 \ud48d\ud1a0\uc5d0 \uc778\uc885\ucc28\ubcc4\uae4c\uc9c0 \ub2f9\ud558\ub294 \ub4f1 \uc545\uc870\uac74 \uc18d\uc5d0\uc11c \uac16\uc740 \uace0\uc0dd\uc744 \ud558\uba70 \uae30\ubc18\uc744 \ub2e6\uc544 \ub098\uac00 \ucc28\uce30 \uacbd\uc81c\uc801\uc73c\ub85c \uc131\uacf5\uc744 \uac70\ub450\uba70, \ube0c\ub77c\uc9c8 \uc774\ubbfc\uc0ac\ud68c\uc758 \ubaa8\ubc94\uc774 \ub418\uc5c8\ub2e4. \uadf8\ub7ec\ub294 \uac00\uc6b4\ub370 \uc81c2\ucc28 \uc138\uacc4 \ub300\uc804\uc774 \uc77c\uc5b4\ub098\uc790 \uc5f0\ud569\uad6d\uc73c\ub85c \ucc38\uc804\ud55c \ube0c\ub77c\uc9c8\uc740 \uc77c\ubcf8\uacfc \uad6d\uad50\ub97c \ub2e8\uc808\ud588\uace0, \uc77c\ubcf8\uc5b4 \uc2e0\ubb38\uc758 \ubc1c\uac04\uacfc \uc77c\ubcf8\uc778 \ud559\uad50\uc758 \uc124\ub9bd\uc744 \uae08\uc9c0\ud558\ub294 \ub4f1 \uc77c\ubcf8\uc778\ub4e4\uc744 \ud0c4\uc555\ud558\uae30\ub3c4 \ud558\uc600\ub2e4. \uc804\ud6c4 \ud0c4\uc555\uc740 \ub05d\ub098\uace0 \uc774\ubbfc\ub3c4 \uc7ac\uac1c\ub418\uc5c8\uc73c\ub098, \uc77c\ubcf8\uc758 \uacbd\uc81c\uc801 \uc131\uc7a5\uc774 \uacc4\uc18d\ub418\uba74\uc11c, \uadf8 \uc774\uc804\ubcf4\ub2e4\ub294 \ud06c\uac8c \uc218\uac00 \uc904\uc5b4\ub4e4\uc5c8\ub2e4. \uadf8\ub7ec\ub098 \ube0c\ub77c\uc9c8\uc5d0 \uc644\uc804\ud788 \uc815\ucc29\ud55c \uc77c\ubcf8\uacc4 \ube0c\ub77c\uc9c8\uc778\ub4e4\uc740 \ud2b9\uc720\uc758 \uadfc\uba74\ud568\uc73c\ub85c \ub354\uc6b1 \uae30\ubc18\uc744 \ub2e6\uc544 \ub098\uac14\uc73c\uba70, \uc77c\ubd80\ub294 \uc544\ub974\ud5e8\ud2f0\ub098 \ub4f1\uc758 \uc8fc\ubcc0\uad6d\uc73c\ub85c\uae4c\uc9c0 \uc9c4\ucd9c\ud558\uae30\ub3c4 \ud558\uc600\ub2e4.", "sentence_answer": "\uc5ec\ub7ec \uc8fc \ub3d9\uc548 \ubc30\ub97c \ud0c0\uace0 \ube0c\ub77c\uc9c8\uae4c\uc9c0 \uac74\ub108\uac04 \uc77c\ubcf8 \uc778 \uc774\uc8fc\uc790\ub4e4\uc740 \uc0c1\ud30c\uc6b8\ub8e8 \uc8fc\ubcc0\uc758 \ucee4\ud53c \ub18d\uc7a5\uc5d0 \ud2b9\ud788 \uc9d1\uc911\uc801\uc73c\ub85c \uc815\ucc29\ud558\uc600\uace0, \uc77c\ubcf8\uacfc \uc644\uc804\ud788 \ub2e4\ub978 \ud48d\ud1a0\uc5d0 \uc778\uc885\ucc28\ubcc4\uae4c\uc9c0 \ub2f9\ud558\ub294 \ub4f1 \uc545\uc870\uac74 \uc18d\uc5d0\uc11c \uac16\uc740 \uace0\uc0dd\uc744 \ud558\uba70 \uae30\ubc18\uc744 \ub2e6\uc544 \ub098\uac00 \ucc28\uce30 \uacbd\uc81c\uc801\uc73c\ub85c \uc131\uacf5\uc744 \uac70\ub450\uba70, \ube0c\ub77c\uc9c8 \uc774\ubbfc\uc0ac\ud68c\uc758 \ubaa8\ubc94\uc774 \ub418\uc5c8\ub2e4.", "paragraph_id": "6548437-0-1", "paragraph_question": "question: \uc81c2\ucc28 \uc138\uacc4 \ub300\uc804\uc774 \uc77c\uc5b4\ub098\uc790 \uc5f0\ud569\uad6d\uc73c\ub85c \ucc38\uc804\ud55c \ube0c\ub77c\uc9c8\uc774 \uad6d\uad50\ub97c \ub2e8\uc808\ud588\ub358 \ub098\ub77c\ub294?, context: \uc5ec\ub7ec \uc8fc \ub3d9\uc548 \ubc30\ub97c \ud0c0\uace0 \ube0c\ub77c\uc9c8\uae4c\uc9c0 \uac74\ub108\uac04 \uc77c\ubcf8\uc778 \uc774\uc8fc\uc790\ub4e4\uc740 \uc0c1\ud30c\uc6b8\ub8e8 \uc8fc\ubcc0\uc758 \ucee4\ud53c \ub18d\uc7a5\uc5d0 \ud2b9\ud788 \uc9d1\uc911\uc801\uc73c\ub85c \uc815\ucc29\ud558\uc600\uace0, \uc77c\ubcf8\uacfc \uc644\uc804\ud788 \ub2e4\ub978 \ud48d\ud1a0\uc5d0 \uc778\uc885\ucc28\ubcc4\uae4c\uc9c0 \ub2f9\ud558\ub294 \ub4f1 \uc545\uc870\uac74 \uc18d\uc5d0\uc11c \uac16\uc740 \uace0\uc0dd\uc744 \ud558\uba70 \uae30\ubc18\uc744 \ub2e6\uc544 \ub098\uac00 \ucc28\uce30 \uacbd\uc81c\uc801\uc73c\ub85c \uc131\uacf5\uc744 \uac70\ub450\uba70, \ube0c\ub77c\uc9c8 \uc774\ubbfc\uc0ac\ud68c\uc758 \ubaa8\ubc94\uc774 \ub418\uc5c8\ub2e4. \uadf8\ub7ec\ub294 \uac00\uc6b4\ub370 \uc81c2\ucc28 \uc138\uacc4 \ub300\uc804\uc774 \uc77c\uc5b4\ub098\uc790 \uc5f0\ud569\uad6d\uc73c\ub85c \ucc38\uc804\ud55c \ube0c\ub77c\uc9c8\uc740 \uc77c\ubcf8\uacfc \uad6d\uad50\ub97c \ub2e8\uc808\ud588\uace0, \uc77c\ubcf8\uc5b4 \uc2e0\ubb38\uc758 \ubc1c\uac04\uacfc \uc77c\ubcf8\uc778 \ud559\uad50\uc758 \uc124\ub9bd\uc744 \uae08\uc9c0\ud558\ub294 \ub4f1 \uc77c\ubcf8\uc778\ub4e4\uc744 \ud0c4\uc555\ud558\uae30\ub3c4 \ud558\uc600\ub2e4. \uc804\ud6c4 \ud0c4\uc555\uc740 \ub05d\ub098\uace0 \uc774\ubbfc\ub3c4 \uc7ac\uac1c\ub418\uc5c8\uc73c\ub098, \uc77c\ubcf8\uc758 \uacbd\uc81c\uc801 \uc131\uc7a5\uc774 \uacc4\uc18d\ub418\uba74\uc11c, \uadf8 \uc774\uc804\ubcf4\ub2e4\ub294 \ud06c\uac8c \uc218\uac00 \uc904\uc5b4\ub4e4\uc5c8\ub2e4. \uadf8\ub7ec\ub098 \ube0c\ub77c\uc9c8\uc5d0 \uc644\uc804\ud788 \uc815\ucc29\ud55c \uc77c\ubcf8\uacc4 \ube0c\ub77c\uc9c8\uc778\ub4e4\uc740 \ud2b9\uc720\uc758 \uadfc\uba74\ud568\uc73c\ub85c \ub354\uc6b1 \uae30\ubc18\uc744 \ub2e6\uc544 \ub098\uac14\uc73c\uba70, \uc77c\ubd80\ub294 \uc544\ub974\ud5e8\ud2f0\ub098 \ub4f1\uc758 \uc8fc\ubcc0\uad6d\uc73c\ub85c\uae4c\uc9c0 \uc9c4\ucd9c\ud558\uae30\ub3c4 \ud558\uc600\ub2e4."} {"question": "\uc0c8\uc815\uce58\ubbfc\uc8fc\uc5f0\ud569 \ub300\ud45c\ub294?", "paragraph": "\uc8fc\uc2b9\uc6a9 \uc758\uc6d0\uc758 \uc0ac\uacfc\uc218\uc6a9\uc5d0\ub3c4 \ubd88\uad6c\ud558\uace0, \uacf5\uac08\ubc1c\uc5b8 \ud30c\ubb38\uc758 \uacc4\uc18d\ub41c \ud655\uc0b0\uacfc \uc815\uccad\ub798 \ucd5c\uace0\uc704\uc6d0\uc774 \ube44\ub178\uacc4 \ubc15\uc8fc\uc120 \uc758\uc6d0\uc744 SNS\uc0c1\uc5d0\uc11c \uacf5\uaca9\ud558\uba74\uc11c \ub2f9 \uac08\ub4f1\uc5d0 \uae30\ub984\uc744 \ubd80\uc5b4 \uce5c\ub178-\ube44\ub178 \uc2f8\uc6c0\uc758 \uae30\ud3ed\uc81c\uac00 \ub418\uc5c8\ub2e4. \uc774\uc5d0 \ubb38\uc7ac\uc778 \uc0c8\uc815\uce58\ubbfc\uc8fc\uc5f0\ud569 \ub300\ud45c\ub294\u201c\uc74d\ucc38\ub9c8\uc18d\uc758 \uc2ec\uc815\uc73c\ub85c \uc815\uccad\ub798 \ucd5c\uace0\uc704\uc6d0\uc740 \ub2f9\ubd84\uac04 \uc790\uc219\uc758 \uc2dc\uac04\uc744 \uac16\ub3c4\ub85d \ud558\uaca0\ub2e4\u201d\ub77c\uace0 \ubc1d\ud788\uba70, \uc815\uccad\ub798 \ucd5c\uace0\uc704\uc6d0\uc758 \uc790\uc219\uc744 \ubc1d\ud614\uc73c\ub098, \uc7a0\uc2dc \ud6c4 \uc815\uccad\ub798 \uc758\uc6d0\uc758 \ucd5c\uace0\uc704\uc6d0\ud68c \ucd9c\uc11d\uc815\uc9c0 \uc870\uce58\ub97c \ub0b4\ub9ac\uba74\uc11c \uc0ac\uc2e4\uc0c1 \uc815\uccad\ub798\uc758\uc6d0\uc758 \ucd5c\uace0\uc704\uc6d0\uc9c1 \uc9c1\ubb34\uc815\uc9c0\uc870\uce58 \ud558\uc600\ub2e4. \uadf8\ub9ac\uace0 \uc0c8\uc815\uce58\uc5f0\ud569\ub0b4 \ube44\ub178 \uc131\ud5a5\uc758 \ud3c9\ub2f9\uc6d0 10\uc5ec \uba85\uc774 \uacf5\uac08 \ubc1c\uc5b8\uc73c\ub85c \ub17c\ub780\uc744 \uc77c\uc73c\ud0a8 \uc815\uccad\ub798 \ucd5c\uace0\uc704\uc6d0\uc5d0 \ub300\ud574 \ub2f9 \uc724\ub9ac\uc2ec\ud310\uc6d0\uc5d0 \uc81c\uc18c\ud558\uc600\ub2e4. \uc774\uc5d0 \uae40\uc6a9\ubbfc , '\uc720\ubbfc\uc544\ube60' \uae40\uc601\uc624 \ub4f1\uc774 \uc815\uccad\ub798 \uc758\uc6d0\uc744 \uc639\ud638\ud558\uc600\ub2e4. 2015\ub144 5\uc6d4 26\uc77c \uc0c8\uc815\uce58\ubbfc\uc8fc\uc5f0\ud569 \uc724\ub9ac\uc2ec\ud310\uc6d0\uc5d0\uc11c \ub2f9\uc9c1 \uc790\uaca9\uc815\uc9c0 1\ub144\uc774\ub77c\ub294 \uc9d5\uacc4\ub97c \ub0b4\ub9ac\ub294 \uac83\uc73c\ub85c \uacb0\ub860\uc9c0\uc5c8\ub2e4. \ud55c\ud3b8 \ud53c\ud574 \ub2f9\uc0ac\uc790\uc778 \uc8fc\uc2b9\uc6a9\uc758\uc6d0\uc740 \uc120\ucc98\uc758\uc0ac\ub97c \uc724\ub9ac\uc2ec\ud310\uc6d0\uc5d0 \uc804\ub2ec\ud588\uc74c\uc5d0\ub3c4 \uc9d5\uacc4 \uacb0\uacfc\uac00 \ub098\uc624\uc790 \u201c\ub2f9\uc9c1 \uc790\uaca9\uc815\uc9c0\ub97c \ubc1b\ub294\ub2e4\uace0 \ud574\ub3c4 3\uac1c\uc6d4 \uc548\ud30e\uc774 \ub420 \uac70\ub77c\uace0 \uc0dd\uac01\ud588\ub294\ub370 \uc608\uc0c1\ubcf4\ub2e4 \uac15\ud55c \uc870\uce58\uac00 \ub0b4\ub824\uc84c\ub2e4\u201d\uba70 \u201c\uc800\ub85c\uc11c\ub294 \uc815 \ucd5c\uace0\uc704\uc6d0\uc758 \uc0ac\uacfc\ub97c \uc9c4\uc815\uc131 \uc788\uac8c \ubc1b\uc544\ub4e4\uc600\uc5c8\ub294\ub370 \uc548\ud0c0\uae4c\uc6b4 \ub9c8\uc74c\uc774 \ub4e0\ub2e4\u201d\uace0 \ud588\ub2e4.", "answer": "\ubb38\uc7ac\uc778", "sentence": "\uc774\uc5d0 \ubb38\uc7ac\uc778 \uc0c8\uc815\uce58\ubbfc\uc8fc\uc5f0\ud569 \ub300\ud45c\ub294\u201c\uc74d\ucc38\ub9c8\uc18d\uc758 \uc2ec\uc815\uc73c\ub85c \uc815\uccad\ub798 \ucd5c\uace0\uc704\uc6d0\uc740 \ub2f9\ubd84\uac04 \uc790\uc219\uc758 \uc2dc\uac04\uc744 \uac16\ub3c4\ub85d \ud558\uaca0\ub2e4\u201d\ub77c\uace0 \ubc1d\ud788\uba70, \uc815\uccad\ub798 \ucd5c\uace0\uc704\uc6d0\uc758 \uc790\uc219\uc744 \ubc1d\ud614\uc73c\ub098, \uc7a0\uc2dc \ud6c4 \uc815\uccad\ub798 \uc758\uc6d0\uc758 \ucd5c\uace0\uc704\uc6d0\ud68c \ucd9c\uc11d\uc815\uc9c0 \uc870\uce58\ub97c \ub0b4\ub9ac\uba74\uc11c \uc0ac\uc2e4\uc0c1 \uc815\uccad\ub798\uc758\uc6d0\uc758 \ucd5c\uace0\uc704\uc6d0\uc9c1 \uc9c1\ubb34\uc815\uc9c0\uc870\uce58 \ud558\uc600\ub2e4.", "paragraph_sentence": "\uc8fc\uc2b9\uc6a9 \uc758\uc6d0\uc758 \uc0ac\uacfc\uc218\uc6a9\uc5d0\ub3c4 \ubd88\uad6c\ud558\uace0, \uacf5\uac08\ubc1c\uc5b8 \ud30c\ubb38\uc758 \uacc4\uc18d\ub41c \ud655\uc0b0\uacfc \uc815\uccad\ub798 \ucd5c\uace0\uc704\uc6d0\uc774 \ube44\ub178\uacc4 \ubc15\uc8fc\uc120 \uc758\uc6d0\uc744 SNS\uc0c1\uc5d0\uc11c \uacf5\uaca9\ud558\uba74\uc11c \ub2f9 \uac08\ub4f1\uc5d0 \uae30\ub984\uc744 \ubd80\uc5b4 \uce5c\ub178-\ube44\ub178 \uc2f8\uc6c0\uc758 \uae30\ud3ed\uc81c\uac00 \ub418\uc5c8\ub2e4. \uc774\uc5d0 \ubb38\uc7ac\uc778 \uc0c8\uc815\uce58\ubbfc\uc8fc\uc5f0\ud569 \ub300\ud45c\ub294\u201c\uc74d\ucc38\ub9c8\uc18d\uc758 \uc2ec\uc815\uc73c\ub85c \uc815\uccad\ub798 \ucd5c\uace0\uc704\uc6d0\uc740 \ub2f9\ubd84\uac04 \uc790\uc219\uc758 \uc2dc\uac04\uc744 \uac16\ub3c4\ub85d \ud558\uaca0\ub2e4\u201d\ub77c\uace0 \ubc1d\ud788\uba70, \uc815\uccad\ub798 \ucd5c\uace0\uc704\uc6d0\uc758 \uc790\uc219\uc744 \ubc1d\ud614\uc73c\ub098, \uc7a0\uc2dc \ud6c4 \uc815\uccad\ub798 \uc758\uc6d0\uc758 \ucd5c\uace0\uc704\uc6d0\ud68c \ucd9c\uc11d\uc815\uc9c0 \uc870\uce58\ub97c \ub0b4\ub9ac\uba74\uc11c \uc0ac\uc2e4\uc0c1 \uc815\uccad\ub798\uc758\uc6d0\uc758 \ucd5c\uace0\uc704\uc6d0\uc9c1 \uc9c1\ubb34\uc815\uc9c0\uc870\uce58 \ud558\uc600\ub2e4. \uadf8\ub9ac\uace0 \uc0c8\uc815\uce58\uc5f0\ud569\ub0b4 \ube44\ub178 \uc131\ud5a5\uc758 \ud3c9\ub2f9\uc6d0 10\uc5ec \uba85\uc774 \uacf5\uac08 \ubc1c\uc5b8\uc73c\ub85c \ub17c\ub780\uc744 \uc77c\uc73c\ud0a8 \uc815\uccad\ub798 \ucd5c\uace0\uc704\uc6d0\uc5d0 \ub300\ud574 \ub2f9 \uc724\ub9ac\uc2ec\ud310\uc6d0\uc5d0 \uc81c\uc18c\ud558\uc600\ub2e4. \uc774\uc5d0 \uae40\uc6a9\ubbfc , '\uc720\ubbfc\uc544\ube60' \uae40\uc601\uc624 \ub4f1\uc774 \uc815\uccad\ub798 \uc758\uc6d0\uc744 \uc639\ud638\ud558\uc600\ub2e4. 2015\ub144 5\uc6d4 26\uc77c \uc0c8\uc815\uce58\ubbfc\uc8fc\uc5f0\ud569 \uc724\ub9ac\uc2ec\ud310\uc6d0\uc5d0\uc11c \ub2f9\uc9c1 \uc790\uaca9\uc815\uc9c0 1\ub144\uc774\ub77c\ub294 \uc9d5\uacc4\ub97c \ub0b4\ub9ac\ub294 \uac83\uc73c\ub85c \uacb0\ub860\uc9c0\uc5c8\ub2e4. \ud55c\ud3b8 \ud53c\ud574 \ub2f9\uc0ac\uc790\uc778 \uc8fc\uc2b9\uc6a9\uc758\uc6d0\uc740 \uc120\ucc98\uc758\uc0ac\ub97c \uc724\ub9ac\uc2ec\ud310\uc6d0\uc5d0 \uc804\ub2ec\ud588\uc74c\uc5d0\ub3c4 \uc9d5\uacc4 \uacb0\uacfc\uac00 \ub098\uc624\uc790 \u201c\ub2f9\uc9c1 \uc790\uaca9\uc815\uc9c0\ub97c \ubc1b\ub294\ub2e4\uace0 \ud574\ub3c4 3\uac1c\uc6d4 \uc548\ud30e\uc774 \ub420 \uac70\ub77c\uace0 \uc0dd\uac01\ud588\ub294\ub370 \uc608\uc0c1\ubcf4\ub2e4 \uac15\ud55c \uc870\uce58\uac00 \ub0b4\ub824\uc84c\ub2e4\u201d\uba70 \u201c\uc800\ub85c\uc11c\ub294 \uc815 \ucd5c\uace0\uc704\uc6d0\uc758 \uc0ac\uacfc\ub97c \uc9c4\uc815\uc131 \uc788\uac8c \ubc1b\uc544\ub4e4\uc600\uc5c8\ub294\ub370 \uc548\ud0c0\uae4c\uc6b4 \ub9c8\uc74c\uc774 \ub4e0\ub2e4\u201d\uace0 \ud588\ub2e4.", "paragraph_answer": "\uc8fc\uc2b9\uc6a9 \uc758\uc6d0\uc758 \uc0ac\uacfc\uc218\uc6a9\uc5d0\ub3c4 \ubd88\uad6c\ud558\uace0, \uacf5\uac08\ubc1c\uc5b8 \ud30c\ubb38\uc758 \uacc4\uc18d\ub41c \ud655\uc0b0\uacfc \uc815\uccad\ub798 \ucd5c\uace0\uc704\uc6d0\uc774 \ube44\ub178\uacc4 \ubc15\uc8fc\uc120 \uc758\uc6d0\uc744 SNS\uc0c1\uc5d0\uc11c \uacf5\uaca9\ud558\uba74\uc11c \ub2f9 \uac08\ub4f1\uc5d0 \uae30\ub984\uc744 \ubd80\uc5b4 \uce5c\ub178-\ube44\ub178 \uc2f8\uc6c0\uc758 \uae30\ud3ed\uc81c\uac00 \ub418\uc5c8\ub2e4. \uc774\uc5d0 \ubb38\uc7ac\uc778 \uc0c8\uc815\uce58\ubbfc\uc8fc\uc5f0\ud569 \ub300\ud45c\ub294\u201c\uc74d\ucc38\ub9c8\uc18d\uc758 \uc2ec\uc815\uc73c\ub85c \uc815\uccad\ub798 \ucd5c\uace0\uc704\uc6d0\uc740 \ub2f9\ubd84\uac04 \uc790\uc219\uc758 \uc2dc\uac04\uc744 \uac16\ub3c4\ub85d \ud558\uaca0\ub2e4\u201d\ub77c\uace0 \ubc1d\ud788\uba70, \uc815\uccad\ub798 \ucd5c\uace0\uc704\uc6d0\uc758 \uc790\uc219\uc744 \ubc1d\ud614\uc73c\ub098, \uc7a0\uc2dc \ud6c4 \uc815\uccad\ub798 \uc758\uc6d0\uc758 \ucd5c\uace0\uc704\uc6d0\ud68c \ucd9c\uc11d\uc815\uc9c0 \uc870\uce58\ub97c \ub0b4\ub9ac\uba74\uc11c \uc0ac\uc2e4\uc0c1 \uc815\uccad\ub798\uc758\uc6d0\uc758 \ucd5c\uace0\uc704\uc6d0\uc9c1 \uc9c1\ubb34\uc815\uc9c0\uc870\uce58 \ud558\uc600\ub2e4. \uadf8\ub9ac\uace0 \uc0c8\uc815\uce58\uc5f0\ud569\ub0b4 \ube44\ub178 \uc131\ud5a5\uc758 \ud3c9\ub2f9\uc6d0 10\uc5ec \uba85\uc774 \uacf5\uac08 \ubc1c\uc5b8\uc73c\ub85c \ub17c\ub780\uc744 \uc77c\uc73c\ud0a8 \uc815\uccad\ub798 \ucd5c\uace0\uc704\uc6d0\uc5d0 \ub300\ud574 \ub2f9 \uc724\ub9ac\uc2ec\ud310\uc6d0\uc5d0 \uc81c\uc18c\ud558\uc600\ub2e4. \uc774\uc5d0 \uae40\uc6a9\ubbfc , '\uc720\ubbfc\uc544\ube60' \uae40\uc601\uc624 \ub4f1\uc774 \uc815\uccad\ub798 \uc758\uc6d0\uc744 \uc639\ud638\ud558\uc600\ub2e4. 2015\ub144 5\uc6d4 26\uc77c \uc0c8\uc815\uce58\ubbfc\uc8fc\uc5f0\ud569 \uc724\ub9ac\uc2ec\ud310\uc6d0\uc5d0\uc11c \ub2f9\uc9c1 \uc790\uaca9\uc815\uc9c0 1\ub144\uc774\ub77c\ub294 \uc9d5\uacc4\ub97c \ub0b4\ub9ac\ub294 \uac83\uc73c\ub85c \uacb0\ub860\uc9c0\uc5c8\ub2e4. \ud55c\ud3b8 \ud53c\ud574 \ub2f9\uc0ac\uc790\uc778 \uc8fc\uc2b9\uc6a9\uc758\uc6d0\uc740 \uc120\ucc98\uc758\uc0ac\ub97c \uc724\ub9ac\uc2ec\ud310\uc6d0\uc5d0 \uc804\ub2ec\ud588\uc74c\uc5d0\ub3c4 \uc9d5\uacc4 \uacb0\uacfc\uac00 \ub098\uc624\uc790 \u201c\ub2f9\uc9c1 \uc790\uaca9\uc815\uc9c0\ub97c \ubc1b\ub294\ub2e4\uace0 \ud574\ub3c4 3\uac1c\uc6d4 \uc548\ud30e\uc774 \ub420 \uac70\ub77c\uace0 \uc0dd\uac01\ud588\ub294\ub370 \uc608\uc0c1\ubcf4\ub2e4 \uac15\ud55c \uc870\uce58\uac00 \ub0b4\ub824\uc84c\ub2e4\u201d\uba70 \u201c\uc800\ub85c\uc11c\ub294 \uc815 \ucd5c\uace0\uc704\uc6d0\uc758 \uc0ac\uacfc\ub97c \uc9c4\uc815\uc131 \uc788\uac8c \ubc1b\uc544\ub4e4\uc600\uc5c8\ub294\ub370 \uc548\ud0c0\uae4c\uc6b4 \ub9c8\uc74c\uc774 \ub4e0\ub2e4\u201d\uace0 \ud588\ub2e4.", "sentence_answer": "\uc774\uc5d0 \ubb38\uc7ac\uc778 \uc0c8\uc815\uce58\ubbfc\uc8fc\uc5f0\ud569 \ub300\ud45c\ub294\u201c\uc74d\ucc38\ub9c8\uc18d\uc758 \uc2ec\uc815\uc73c\ub85c \uc815\uccad\ub798 \ucd5c\uace0\uc704\uc6d0\uc740 \ub2f9\ubd84\uac04 \uc790\uc219\uc758 \uc2dc\uac04\uc744 \uac16\ub3c4\ub85d \ud558\uaca0\ub2e4\u201d\ub77c\uace0 \ubc1d\ud788\uba70, \uc815\uccad\ub798 \ucd5c\uace0\uc704\uc6d0\uc758 \uc790\uc219\uc744 \ubc1d\ud614\uc73c\ub098, \uc7a0\uc2dc \ud6c4 \uc815\uccad\ub798 \uc758\uc6d0\uc758 \ucd5c\uace0\uc704\uc6d0\ud68c \ucd9c\uc11d\uc815\uc9c0 \uc870\uce58\ub97c \ub0b4\ub9ac\uba74\uc11c \uc0ac\uc2e4\uc0c1 \uc815\uccad\ub798\uc758\uc6d0\uc758 \ucd5c\uace0\uc704\uc6d0\uc9c1 \uc9c1\ubb34\uc815\uc9c0\uc870\uce58 \ud558\uc600\ub2e4.", "paragraph_id": "5869272-2-1", "paragraph_question": "question: \uc0c8\uc815\uce58\ubbfc\uc8fc\uc5f0\ud569 \ub300\ud45c\ub294?, context: \uc8fc\uc2b9\uc6a9 \uc758\uc6d0\uc758 \uc0ac\uacfc\uc218\uc6a9\uc5d0\ub3c4 \ubd88\uad6c\ud558\uace0, \uacf5\uac08\ubc1c\uc5b8 \ud30c\ubb38\uc758 \uacc4\uc18d\ub41c \ud655\uc0b0\uacfc \uc815\uccad\ub798 \ucd5c\uace0\uc704\uc6d0\uc774 \ube44\ub178\uacc4 \ubc15\uc8fc\uc120 \uc758\uc6d0\uc744 SNS\uc0c1\uc5d0\uc11c \uacf5\uaca9\ud558\uba74\uc11c \ub2f9 \uac08\ub4f1\uc5d0 \uae30\ub984\uc744 \ubd80\uc5b4 \uce5c\ub178-\ube44\ub178 \uc2f8\uc6c0\uc758 \uae30\ud3ed\uc81c\uac00 \ub418\uc5c8\ub2e4. \uc774\uc5d0 \ubb38\uc7ac\uc778 \uc0c8\uc815\uce58\ubbfc\uc8fc\uc5f0\ud569 \ub300\ud45c\ub294\u201c\uc74d\ucc38\ub9c8\uc18d\uc758 \uc2ec\uc815\uc73c\ub85c \uc815\uccad\ub798 \ucd5c\uace0\uc704\uc6d0\uc740 \ub2f9\ubd84\uac04 \uc790\uc219\uc758 \uc2dc\uac04\uc744 \uac16\ub3c4\ub85d \ud558\uaca0\ub2e4\u201d\ub77c\uace0 \ubc1d\ud788\uba70, \uc815\uccad\ub798 \ucd5c\uace0\uc704\uc6d0\uc758 \uc790\uc219\uc744 \ubc1d\ud614\uc73c\ub098, \uc7a0\uc2dc \ud6c4 \uc815\uccad\ub798 \uc758\uc6d0\uc758 \ucd5c\uace0\uc704\uc6d0\ud68c \ucd9c\uc11d\uc815\uc9c0 \uc870\uce58\ub97c \ub0b4\ub9ac\uba74\uc11c \uc0ac\uc2e4\uc0c1 \uc815\uccad\ub798\uc758\uc6d0\uc758 \ucd5c\uace0\uc704\uc6d0\uc9c1 \uc9c1\ubb34\uc815\uc9c0\uc870\uce58 \ud558\uc600\ub2e4. \uadf8\ub9ac\uace0 \uc0c8\uc815\uce58\uc5f0\ud569\ub0b4 \ube44\ub178 \uc131\ud5a5\uc758 \ud3c9\ub2f9\uc6d0 10\uc5ec \uba85\uc774 \uacf5\uac08 \ubc1c\uc5b8\uc73c\ub85c \ub17c\ub780\uc744 \uc77c\uc73c\ud0a8 \uc815\uccad\ub798 \ucd5c\uace0\uc704\uc6d0\uc5d0 \ub300\ud574 \ub2f9 \uc724\ub9ac\uc2ec\ud310\uc6d0\uc5d0 \uc81c\uc18c\ud558\uc600\ub2e4. \uc774\uc5d0 \uae40\uc6a9\ubbfc , '\uc720\ubbfc\uc544\ube60' \uae40\uc601\uc624 \ub4f1\uc774 \uc815\uccad\ub798 \uc758\uc6d0\uc744 \uc639\ud638\ud558\uc600\ub2e4. 2015\ub144 5\uc6d4 26\uc77c \uc0c8\uc815\uce58\ubbfc\uc8fc\uc5f0\ud569 \uc724\ub9ac\uc2ec\ud310\uc6d0\uc5d0\uc11c \ub2f9\uc9c1 \uc790\uaca9\uc815\uc9c0 1\ub144\uc774\ub77c\ub294 \uc9d5\uacc4\ub97c \ub0b4\ub9ac\ub294 \uac83\uc73c\ub85c \uacb0\ub860\uc9c0\uc5c8\ub2e4. \ud55c\ud3b8 \ud53c\ud574 \ub2f9\uc0ac\uc790\uc778 \uc8fc\uc2b9\uc6a9\uc758\uc6d0\uc740 \uc120\ucc98\uc758\uc0ac\ub97c \uc724\ub9ac\uc2ec\ud310\uc6d0\uc5d0 \uc804\ub2ec\ud588\uc74c\uc5d0\ub3c4 \uc9d5\uacc4 \uacb0\uacfc\uac00 \ub098\uc624\uc790 \u201c\ub2f9\uc9c1 \uc790\uaca9\uc815\uc9c0\ub97c \ubc1b\ub294\ub2e4\uace0 \ud574\ub3c4 3\uac1c\uc6d4 \uc548\ud30e\uc774 \ub420 \uac70\ub77c\uace0 \uc0dd\uac01\ud588\ub294\ub370 \uc608\uc0c1\ubcf4\ub2e4 \uac15\ud55c \uc870\uce58\uac00 \ub0b4\ub824\uc84c\ub2e4\u201d\uba70 \u201c\uc800\ub85c\uc11c\ub294 \uc815 \ucd5c\uace0\uc704\uc6d0\uc758 \uc0ac\uacfc\ub97c \uc9c4\uc815\uc131 \uc788\uac8c \ubc1b\uc544\ub4e4\uc600\uc5c8\ub294\ub370 \uc548\ud0c0\uae4c\uc6b4 \ub9c8\uc74c\uc774 \ub4e0\ub2e4\u201d\uace0 \ud588\ub2e4."} {"question": "\uc74c\ud575 \uadc0\ub450\ub97c \ud56d\uc0c1 \ubbf8\ub044\ub7fd\uac8c \uc720\uc9c0\ud558\ub294 \uc790\uc5f0\uc801 \ubd84\ube44\ubb3c\uc740?", "paragraph": "\uc74c\ud575\uc758 \ub300\ubd80\ubd84\uc744 \ub36e\uc740 \ud3ec\ud53c\ub97c \uac00\uc9c4 \uc5ec\uc131\ub4e4\uc740 \uc885\uc885 \ud3ec\ud53c\uac00 \ub36e\uc778 \ucc44 \uc790\uc704\ud589\uc704\ub97c \ud558\ub294 \ub0a8\uc131\uacfc \ube44\uc2b7\ud558\uac8c, \uc74c\ud575 \uadc0\ub450\ub97c \ub36e\uc740 \ud3ec\ud53c\ub97c \uc790\uadf9\ud568\uc73c\ub85c\uc368 \uc790\uc704\ud589\uc704\ub97c \ud55c\ub2e4. \ub354 \uc791\uace0 \ub2e8\uc21c\ud55c \uad6c\uc870 \ub54c\ubb38\uc5d0, \uc74c\ud575 \uadc0\ub450\uc640 \ud3ec\ud53c\ub97c \ud55c\ubc88\uc5d0 \ube44\ube44\ub294 \uacbd\ud5a5\uc774 \uc788\ub2e4. \ub9ce\uc740 \uacbd\uc6b0, \uc74c\ud575 \uadc0\ub450 \uc790\uccb4\ub294 \ud3ec\ud53c\ub97c \uc816\ud600 \uc9c1\uc811 \uc790\uadf9\ud558\uae30\uc5d0\ub294 \ub108\ubb34 \ubbfc\uac10\ud558\uae30\ub3c4 \ud558\ub2e4. \uc774\ub7ec\ud55c \uadf9\ub3c4\uc758 \ubbfc\uac10\uc131\uc740 \uc774 \uc870\uc9c1\uc774 \uc6d0\ub798 \uac89\uc73c\ub85c \ub4dc\ub7ec\ub098\uc9c0 \uc54a\uc558\uae30 \ub54c\ubb38\uc778\ub370, \uc774\ub294 \ud3ec\uacbd\uc218\uc220\uc744 \ud558\uc9c0 \uc54a\uc740 \uc5b4\ub9b0 \ub0a8\uc131, \ud2b9\ud788 \uc0ac\ucd98\uae30 \uc774\uc804\uc758 \ud3ec\ud53c\uac00 \uc644\uc804\ud788 \ubc97\uaca8\uc838 \ubcf4\uc9c0 \ubabb\ud55c \ub0a8\uc131\uc5d0\uac8c\uc11c \ub098\ud0c0\ub09c\ub2e4. \uc74c\ud575 \ud3ec\ud53c\ub294 \ub610\ud55c \uc74c\uacbd\uc758 \ud3ec\ud53c\uc640 \ub9c8\ucc2c\uac00\uc9c0\ub85c \uc74c\ud575 \uadc0\ub450\ub97c \ubcf4\ud638\ud558\ub294 \uc5ed\ud560\uc744 \ud558\uae30\ub3c4 \ud55c\ub2e4. \uc131\uc801\uc778 \uc790\uadf9\uc744 \ubc1b\ub294 \uc911\uc5d0, \uce58\uace8\uc744 \ud1b5\ud55c \uac04\uc811\uc801\uc778 \uc790\uadf9 \uc774\uc678\uc5d0 \uc788\uc744 \uc218 \uc788\ub294 \uc74c\uacbd\uacfc \uc74c\ud575 \uadc0\ub450\uac04\uc758 \uc9c1\uc811\uc801\uc778 \uc811\ucd09\uc744 \ubc29\uc9c0\ud574 \uc904 \uc218 \uc788\ub2e4. \uc74c\ud575 \uadc0\ub450\ub294 \ud3ec\ud53c\uc640 \ub9c8\ucc2c\uac00\uc9c0\ub85c, \uc790\uc5f0\uc801\uc73c\ub85c \ubd84\ube44\ub418\ub294 \ud53c\uc9c0(\uce58\uad6c\uc640 \ud568\uaed8 \ubd84\ube44)\uc5d0 \uc758\ud574\uc11c \ud56d\uc0c1 \ubbf8\ub044\ub7fd\uac8c \uc720\uc9c0\ub418\uc5b4 \uc131\uc801\uc778 \uc790\uadf9\uc744 \ubc1b\uc744 \ub54c \ubb34\ub9ac \uc5c6\uc774 \ubbf8\ub044\ub7ec\uc838\uc57c \ud55c\ub2e4. \ub9cc\uc77c \uc5ec\uc131\uc758 \uc74c\ud575 \uadc0\ub450\uac00 \ubbf8\ub044\ub7fd\uc9c0 \uc54a\uc73c\uba74, \ud3ec\ud53c\uc5d0 \uac00\ud574\uc9c0\ub294 \uc131\uc801\uc778 \uc790\uadf9\uc740 \uc74c\ud575 \uadc0\ub450\uc5d0 \uc804\ud574\uc9c0\uc9c0 \uc54a\uac70\ub098, \uc624\ud788\ub824 \ud3ec\ud53c\ub97c \uc0b4\uc9dd\ub9cc \uac74\ub4dc\ub824\ub3c4 \uc5ec\uc131\uc5d0\uac8c \uc74c\ud575 \uadc0\ub450\ub97c \uc0ac\ud3ec\ub85c \ubb38\uc9c0\ub974\ub294 \ub4ef\ud55c \uace0\ud1b5\uc744 \uc548\uaca8\uc8fc\uac8c \ub420 \uac83\uc774\ub2e4.", "answer": "\ud53c\uc9c0", "sentence": "\uc74c\ud575 \uadc0\ub450\ub294 \ud3ec\ud53c\uc640 \ub9c8\ucc2c\uac00\uc9c0\ub85c, \uc790\uc5f0\uc801\uc73c\ub85c \ubd84\ube44\ub418\ub294 \ud53c\uc9c0 (\uce58\uad6c\uc640 \ud568\uaed8 \ubd84\ube44)\uc5d0 \uc758\ud574\uc11c \ud56d\uc0c1 \ubbf8\ub044\ub7fd\uac8c \uc720\uc9c0\ub418\uc5b4 \uc131\uc801\uc778 \uc790\uadf9\uc744 \ubc1b\uc744 \ub54c \ubb34\ub9ac \uc5c6\uc774 \ubbf8\ub044\ub7ec\uc838\uc57c \ud55c\ub2e4.", "paragraph_sentence": "\uc74c\ud575\uc758 \ub300\ubd80\ubd84\uc744 \ub36e\uc740 \ud3ec\ud53c\ub97c \uac00\uc9c4 \uc5ec\uc131\ub4e4\uc740 \uc885\uc885 \ud3ec\ud53c\uac00 \ub36e\uc778 \ucc44 \uc790\uc704\ud589\uc704\ub97c \ud558\ub294 \ub0a8\uc131\uacfc \ube44\uc2b7\ud558\uac8c, \uc74c\ud575 \uadc0\ub450\ub97c \ub36e\uc740 \ud3ec\ud53c\ub97c \uc790\uadf9\ud568\uc73c\ub85c\uc368 \uc790\uc704\ud589\uc704\ub97c \ud55c\ub2e4. \ub354 \uc791\uace0 \ub2e8\uc21c\ud55c \uad6c\uc870 \ub54c\ubb38\uc5d0, \uc74c\ud575 \uadc0\ub450\uc640 \ud3ec\ud53c\ub97c \ud55c\ubc88\uc5d0 \ube44\ube44\ub294 \uacbd\ud5a5\uc774 \uc788\ub2e4. \ub9ce\uc740 \uacbd\uc6b0, \uc74c\ud575 \uadc0\ub450 \uc790\uccb4\ub294 \ud3ec\ud53c\ub97c \uc816\ud600 \uc9c1\uc811 \uc790\uadf9\ud558\uae30\uc5d0\ub294 \ub108\ubb34 \ubbfc\uac10\ud558\uae30\ub3c4 \ud558\ub2e4. \uc774\ub7ec\ud55c \uadf9\ub3c4\uc758 \ubbfc\uac10\uc131\uc740 \uc774 \uc870\uc9c1\uc774 \uc6d0\ub798 \uac89\uc73c\ub85c \ub4dc\ub7ec\ub098\uc9c0 \uc54a\uc558\uae30 \ub54c\ubb38\uc778\ub370, \uc774\ub294 \ud3ec\uacbd\uc218\uc220\uc744 \ud558\uc9c0 \uc54a\uc740 \uc5b4\ub9b0 \ub0a8\uc131, \ud2b9\ud788 \uc0ac\ucd98\uae30 \uc774\uc804\uc758 \ud3ec\ud53c\uac00 \uc644\uc804\ud788 \ubc97\uaca8\uc838 \ubcf4\uc9c0 \ubabb\ud55c \ub0a8\uc131\uc5d0\uac8c\uc11c \ub098\ud0c0\ub09c\ub2e4. \uc74c\ud575 \ud3ec\ud53c\ub294 \ub610\ud55c \uc74c\uacbd\uc758 \ud3ec\ud53c\uc640 \ub9c8\ucc2c\uac00\uc9c0\ub85c \uc74c\ud575 \uadc0\ub450\ub97c \ubcf4\ud638\ud558\ub294 \uc5ed\ud560\uc744 \ud558\uae30\ub3c4 \ud55c\ub2e4. \uc131\uc801\uc778 \uc790\uadf9\uc744 \ubc1b\ub294 \uc911\uc5d0, \uce58\uace8\uc744 \ud1b5\ud55c \uac04\uc811\uc801\uc778 \uc790\uadf9 \uc774\uc678\uc5d0 \uc788\uc744 \uc218 \uc788\ub294 \uc74c\uacbd\uacfc \uc74c\ud575 \uadc0\ub450\uac04\uc758 \uc9c1\uc811\uc801\uc778 \uc811\ucd09\uc744 \ubc29\uc9c0\ud574 \uc904 \uc218 \uc788\ub2e4. \uc74c\ud575 \uadc0\ub450\ub294 \ud3ec\ud53c\uc640 \ub9c8\ucc2c\uac00\uc9c0\ub85c, \uc790\uc5f0\uc801\uc73c\ub85c \ubd84\ube44\ub418\ub294 \ud53c\uc9c0 (\uce58\uad6c\uc640 \ud568\uaed8 \ubd84\ube44)\uc5d0 \uc758\ud574\uc11c \ud56d\uc0c1 \ubbf8\ub044\ub7fd\uac8c \uc720\uc9c0\ub418\uc5b4 \uc131\uc801\uc778 \uc790\uadf9\uc744 \ubc1b\uc744 \ub54c \ubb34\ub9ac \uc5c6\uc774 \ubbf8\ub044\ub7ec\uc838\uc57c \ud55c\ub2e4. \ub9cc\uc77c \uc5ec\uc131\uc758 \uc74c\ud575 \uadc0\ub450\uac00 \ubbf8\ub044\ub7fd\uc9c0 \uc54a\uc73c\uba74, \ud3ec\ud53c\uc5d0 \uac00\ud574\uc9c0\ub294 \uc131\uc801\uc778 \uc790\uadf9\uc740 \uc74c\ud575 \uadc0\ub450\uc5d0 \uc804\ud574\uc9c0\uc9c0 \uc54a\uac70\ub098, \uc624\ud788\ub824 \ud3ec\ud53c\ub97c \uc0b4\uc9dd\ub9cc \uac74\ub4dc\ub824\ub3c4 \uc5ec\uc131\uc5d0\uac8c \uc74c\ud575 \uadc0\ub450\ub97c \uc0ac\ud3ec\ub85c \ubb38\uc9c0\ub974\ub294 \ub4ef\ud55c \uace0\ud1b5\uc744 \uc548\uaca8\uc8fc\uac8c \ub420 \uac83\uc774\ub2e4.", "paragraph_answer": "\uc74c\ud575\uc758 \ub300\ubd80\ubd84\uc744 \ub36e\uc740 \ud3ec\ud53c\ub97c \uac00\uc9c4 \uc5ec\uc131\ub4e4\uc740 \uc885\uc885 \ud3ec\ud53c\uac00 \ub36e\uc778 \ucc44 \uc790\uc704\ud589\uc704\ub97c \ud558\ub294 \ub0a8\uc131\uacfc \ube44\uc2b7\ud558\uac8c, \uc74c\ud575 \uadc0\ub450\ub97c \ub36e\uc740 \ud3ec\ud53c\ub97c \uc790\uadf9\ud568\uc73c\ub85c\uc368 \uc790\uc704\ud589\uc704\ub97c \ud55c\ub2e4. \ub354 \uc791\uace0 \ub2e8\uc21c\ud55c \uad6c\uc870 \ub54c\ubb38\uc5d0, \uc74c\ud575 \uadc0\ub450\uc640 \ud3ec\ud53c\ub97c \ud55c\ubc88\uc5d0 \ube44\ube44\ub294 \uacbd\ud5a5\uc774 \uc788\ub2e4. \ub9ce\uc740 \uacbd\uc6b0, \uc74c\ud575 \uadc0\ub450 \uc790\uccb4\ub294 \ud3ec\ud53c\ub97c \uc816\ud600 \uc9c1\uc811 \uc790\uadf9\ud558\uae30\uc5d0\ub294 \ub108\ubb34 \ubbfc\uac10\ud558\uae30\ub3c4 \ud558\ub2e4. \uc774\ub7ec\ud55c \uadf9\ub3c4\uc758 \ubbfc\uac10\uc131\uc740 \uc774 \uc870\uc9c1\uc774 \uc6d0\ub798 \uac89\uc73c\ub85c \ub4dc\ub7ec\ub098\uc9c0 \uc54a\uc558\uae30 \ub54c\ubb38\uc778\ub370, \uc774\ub294 \ud3ec\uacbd\uc218\uc220\uc744 \ud558\uc9c0 \uc54a\uc740 \uc5b4\ub9b0 \ub0a8\uc131, \ud2b9\ud788 \uc0ac\ucd98\uae30 \uc774\uc804\uc758 \ud3ec\ud53c\uac00 \uc644\uc804\ud788 \ubc97\uaca8\uc838 \ubcf4\uc9c0 \ubabb\ud55c \ub0a8\uc131\uc5d0\uac8c\uc11c \ub098\ud0c0\ub09c\ub2e4. \uc74c\ud575 \ud3ec\ud53c\ub294 \ub610\ud55c \uc74c\uacbd\uc758 \ud3ec\ud53c\uc640 \ub9c8\ucc2c\uac00\uc9c0\ub85c \uc74c\ud575 \uadc0\ub450\ub97c \ubcf4\ud638\ud558\ub294 \uc5ed\ud560\uc744 \ud558\uae30\ub3c4 \ud55c\ub2e4. \uc131\uc801\uc778 \uc790\uadf9\uc744 \ubc1b\ub294 \uc911\uc5d0, \uce58\uace8\uc744 \ud1b5\ud55c \uac04\uc811\uc801\uc778 \uc790\uadf9 \uc774\uc678\uc5d0 \uc788\uc744 \uc218 \uc788\ub294 \uc74c\uacbd\uacfc \uc74c\ud575 \uadc0\ub450\uac04\uc758 \uc9c1\uc811\uc801\uc778 \uc811\ucd09\uc744 \ubc29\uc9c0\ud574 \uc904 \uc218 \uc788\ub2e4. \uc74c\ud575 \uadc0\ub450\ub294 \ud3ec\ud53c\uc640 \ub9c8\ucc2c\uac00\uc9c0\ub85c, \uc790\uc5f0\uc801\uc73c\ub85c \ubd84\ube44\ub418\ub294 \ud53c\uc9c0 (\uce58\uad6c\uc640 \ud568\uaed8 \ubd84\ube44)\uc5d0 \uc758\ud574\uc11c \ud56d\uc0c1 \ubbf8\ub044\ub7fd\uac8c \uc720\uc9c0\ub418\uc5b4 \uc131\uc801\uc778 \uc790\uadf9\uc744 \ubc1b\uc744 \ub54c \ubb34\ub9ac \uc5c6\uc774 \ubbf8\ub044\ub7ec\uc838\uc57c \ud55c\ub2e4. \ub9cc\uc77c \uc5ec\uc131\uc758 \uc74c\ud575 \uadc0\ub450\uac00 \ubbf8\ub044\ub7fd\uc9c0 \uc54a\uc73c\uba74, \ud3ec\ud53c\uc5d0 \uac00\ud574\uc9c0\ub294 \uc131\uc801\uc778 \uc790\uadf9\uc740 \uc74c\ud575 \uadc0\ub450\uc5d0 \uc804\ud574\uc9c0\uc9c0 \uc54a\uac70\ub098, \uc624\ud788\ub824 \ud3ec\ud53c\ub97c \uc0b4\uc9dd\ub9cc \uac74\ub4dc\ub824\ub3c4 \uc5ec\uc131\uc5d0\uac8c \uc74c\ud575 \uadc0\ub450\ub97c \uc0ac\ud3ec\ub85c \ubb38\uc9c0\ub974\ub294 \ub4ef\ud55c \uace0\ud1b5\uc744 \uc548\uaca8\uc8fc\uac8c \ub420 \uac83\uc774\ub2e4.", "sentence_answer": "\uc74c\ud575 \uadc0\ub450\ub294 \ud3ec\ud53c\uc640 \ub9c8\ucc2c\uac00\uc9c0\ub85c, \uc790\uc5f0\uc801\uc73c\ub85c \ubd84\ube44\ub418\ub294 \ud53c\uc9c0 (\uce58\uad6c\uc640 \ud568\uaed8 \ubd84\ube44)\uc5d0 \uc758\ud574\uc11c \ud56d\uc0c1 \ubbf8\ub044\ub7fd\uac8c \uc720\uc9c0\ub418\uc5b4 \uc131\uc801\uc778 \uc790\uadf9\uc744 \ubc1b\uc744 \ub54c \ubb34\ub9ac \uc5c6\uc774 \ubbf8\ub044\ub7ec\uc838\uc57c \ud55c\ub2e4.", "paragraph_id": "6544313-1-1", "paragraph_question": "question: \uc74c\ud575 \uadc0\ub450\ub97c \ud56d\uc0c1 \ubbf8\ub044\ub7fd\uac8c \uc720\uc9c0\ud558\ub294 \uc790\uc5f0\uc801 \ubd84\ube44\ubb3c\uc740?, context: \uc74c\ud575\uc758 \ub300\ubd80\ubd84\uc744 \ub36e\uc740 \ud3ec\ud53c\ub97c \uac00\uc9c4 \uc5ec\uc131\ub4e4\uc740 \uc885\uc885 \ud3ec\ud53c\uac00 \ub36e\uc778 \ucc44 \uc790\uc704\ud589\uc704\ub97c \ud558\ub294 \ub0a8\uc131\uacfc \ube44\uc2b7\ud558\uac8c, \uc74c\ud575 \uadc0\ub450\ub97c \ub36e\uc740 \ud3ec\ud53c\ub97c \uc790\uadf9\ud568\uc73c\ub85c\uc368 \uc790\uc704\ud589\uc704\ub97c \ud55c\ub2e4. \ub354 \uc791\uace0 \ub2e8\uc21c\ud55c \uad6c\uc870 \ub54c\ubb38\uc5d0, \uc74c\ud575 \uadc0\ub450\uc640 \ud3ec\ud53c\ub97c \ud55c\ubc88\uc5d0 \ube44\ube44\ub294 \uacbd\ud5a5\uc774 \uc788\ub2e4. \ub9ce\uc740 \uacbd\uc6b0, \uc74c\ud575 \uadc0\ub450 \uc790\uccb4\ub294 \ud3ec\ud53c\ub97c \uc816\ud600 \uc9c1\uc811 \uc790\uadf9\ud558\uae30\uc5d0\ub294 \ub108\ubb34 \ubbfc\uac10\ud558\uae30\ub3c4 \ud558\ub2e4. \uc774\ub7ec\ud55c \uadf9\ub3c4\uc758 \ubbfc\uac10\uc131\uc740 \uc774 \uc870\uc9c1\uc774 \uc6d0\ub798 \uac89\uc73c\ub85c \ub4dc\ub7ec\ub098\uc9c0 \uc54a\uc558\uae30 \ub54c\ubb38\uc778\ub370, \uc774\ub294 \ud3ec\uacbd\uc218\uc220\uc744 \ud558\uc9c0 \uc54a\uc740 \uc5b4\ub9b0 \ub0a8\uc131, \ud2b9\ud788 \uc0ac\ucd98\uae30 \uc774\uc804\uc758 \ud3ec\ud53c\uac00 \uc644\uc804\ud788 \ubc97\uaca8\uc838 \ubcf4\uc9c0 \ubabb\ud55c \ub0a8\uc131\uc5d0\uac8c\uc11c \ub098\ud0c0\ub09c\ub2e4. \uc74c\ud575 \ud3ec\ud53c\ub294 \ub610\ud55c \uc74c\uacbd\uc758 \ud3ec\ud53c\uc640 \ub9c8\ucc2c\uac00\uc9c0\ub85c \uc74c\ud575 \uadc0\ub450\ub97c \ubcf4\ud638\ud558\ub294 \uc5ed\ud560\uc744 \ud558\uae30\ub3c4 \ud55c\ub2e4. \uc131\uc801\uc778 \uc790\uadf9\uc744 \ubc1b\ub294 \uc911\uc5d0, \uce58\uace8\uc744 \ud1b5\ud55c \uac04\uc811\uc801\uc778 \uc790\uadf9 \uc774\uc678\uc5d0 \uc788\uc744 \uc218 \uc788\ub294 \uc74c\uacbd\uacfc \uc74c\ud575 \uadc0\ub450\uac04\uc758 \uc9c1\uc811\uc801\uc778 \uc811\ucd09\uc744 \ubc29\uc9c0\ud574 \uc904 \uc218 \uc788\ub2e4. \uc74c\ud575 \uadc0\ub450\ub294 \ud3ec\ud53c\uc640 \ub9c8\ucc2c\uac00\uc9c0\ub85c, \uc790\uc5f0\uc801\uc73c\ub85c \ubd84\ube44\ub418\ub294 \ud53c\uc9c0(\uce58\uad6c\uc640 \ud568\uaed8 \ubd84\ube44)\uc5d0 \uc758\ud574\uc11c \ud56d\uc0c1 \ubbf8\ub044\ub7fd\uac8c \uc720\uc9c0\ub418\uc5b4 \uc131\uc801\uc778 \uc790\uadf9\uc744 \ubc1b\uc744 \ub54c \ubb34\ub9ac \uc5c6\uc774 \ubbf8\ub044\ub7ec\uc838\uc57c \ud55c\ub2e4. \ub9cc\uc77c \uc5ec\uc131\uc758 \uc74c\ud575 \uadc0\ub450\uac00 \ubbf8\ub044\ub7fd\uc9c0 \uc54a\uc73c\uba74, \ud3ec\ud53c\uc5d0 \uac00\ud574\uc9c0\ub294 \uc131\uc801\uc778 \uc790\uadf9\uc740 \uc74c\ud575 \uadc0\ub450\uc5d0 \uc804\ud574\uc9c0\uc9c0 \uc54a\uac70\ub098, \uc624\ud788\ub824 \ud3ec\ud53c\ub97c \uc0b4\uc9dd\ub9cc \uac74\ub4dc\ub824\ub3c4 \uc5ec\uc131\uc5d0\uac8c \uc74c\ud575 \uadc0\ub450\ub97c \uc0ac\ud3ec\ub85c \ubb38\uc9c0\ub974\ub294 \ub4ef\ud55c \uace0\ud1b5\uc744 \uc548\uaca8\uc8fc\uac8c \ub420 \uac83\uc774\ub2e4."} {"question": "\ub178\ubb34\ud604\uc774 \uc790\ud0dd \uc606\uc5d0\uc11c \uc0ac\uc2dc \uacf5\ubd80\ub97c \ud588\ub358 \uc0ac\ucc30\uc740?", "paragraph": "\ub178\ubb34\ud604\uc740 \uc790\ud0dd \uc606\uc5d0 \uc788\ub358 \uc815\ud1a0\uc6d0\uc774\ub77c\ub294 \uc0ac\ucc30\uc5d0\uc11c \uc0ac\uc2dc \uacf5\ubd80\ub97c \ud558\uc600\ub2e4. \uc774\ud6c4\uc5d0\ub294 \uae40\ud574\uc2dc \uc7a5\uc720\uba74 \ub300\uccad\ub9ac \uc7a5\uc720\uc554\uc5d0 \uba38\ubb34\ub974\uba74\uc11c \uc0ac\uc2dc\ub97c \uc900\ube44\ud588\uc73c\uba70, \ud2c8\ud2c8\uc774 \ubd88\uad50 \uacbd\uc804\uc744 \ud0d0\ub3c5\ud588\ub2e4. 9\ub144\uac04 \uc0ac\uc2dc \uacf5\ubd80\ub97c \ud558\uc5ec 1975\ub144 4\uc6d4 30\uc138\uc5d0 \uc81c17\ud68c \uc0ac\ubc95\uc2dc\ud5d8\uc5d0 \ud569\uaca9\ud558\uc600\ub2e4. \ud3c9\uc18c \ubd88\uc2ec\uc774 \uae4a\uc740 \uac83\uc73c\ub85c \uc54c\ub824\uc9c4 \uc544\ub0b4 \uad8c\uc591\uc219\uc740 2002\ub144 10\uc6d4 1\uc77c \ub2f9\uc2dc \ub300\uc120\uc744 \uc55e\ub450\uace0 \ud569\ucc9c \ud574\uc778\uc0ac\uc5d0 \uba38\ubb3c\uace0 \uc788\ub294 \uc870\uacc4\uc885 \uc885\uc815\uc778 \ubc95\uc804 \uc2a4\ub2d8\uc73c\ub85c\ubd80\ud130 \ubcf4\uc0b4\uacc4\uc640 '\ub300\ub355\ud654'(\u5927\u5fb7\u82b1)\ub77c\ub294 \ubc95\uba85\uc744 \ubc1b\uc558\ub294\ub370, '\ub300\ub355\ud654'\ub294 \ub300\ud55c\ubbfc\uad6d \uc81c5\u00b76\u00b77\u00b78\u00b79\ub300 \ub300\ud1b5\ub839 \ubc15\uc815\ud76c\uc758 \uc544\ub0b4 \uc721\uc601\uc218\uac00 \ubc1b\uc740 \ubc95\uba85\uacfc \uac19\uc740 \uac83\uc774\uc5c8\ub2e4. \uc774\ub7ec\ud55c \ubc95\uba85 \uc218\uacc4\ub294 \ubd88\uad50\uacc4\uc758 \ubbfc\uc2ec\uc774 \uc2e4\ub9b0 \uac83\uc73c\ub85c \ubcfc \uc218 \uc788\uc73c\uba70, \ubd88\uad50\uacc4\uc758 \uae30\ub300\ub97c \ubc1b\uace0 \uc788\uc74c\uc744 \ubcf4\uc5ec\uc8fc\ub294 \uac83\uc774\ub2e4. \uc774\ud6c4 \ub178\ubb34\ud604\uc740 \uc7ac\uc784 \uae30\uac04 \ub3d9\uc548 \ud574\uc778\uc0ac\ub97c \ubb34\ub824 \uc138 \ucc28\ub840\ub098 \ubc29\ubb38\ud574 \ud604\uc9c1 \ub300\ud1b5\ub839\uc73c\ub85c\uc11c\ub294 \ucd5c\ub2e4 \ubc29\ubb38\uc744 \ud558\uc600\ub2e4. 2003\ub144 12\uc6d4 22\uc77c\uc5d0 \ub178\ubb34\ud604\uc740 \uc544\ub0b4\uc640 \ud568\uaed8 \uacbd\ub0a8 \ud569\ucc9c \ud574\uc778\uc0ac\ub97c \ubd88\uc2dc \ubc29\ubb38\ud574 \uc870\uacc4\uc885 \uc885\uc815 \ubc95\uc804\uc2a4\ub2d8, \ucd1d\ubb34\uc6d0\uc7a5 \ubc95\uc7a5\uc2a4\ub2d8\uacfc \ud658\ub2f4\ud55c \ub4a4 \uc624\ucc2c\uc744 \ud568\uaed8 \ud588\ub2e4. \uc0ac\ud328\uc0b0 \ud130\ub110 \ubb38\uc81c\uc5d0 \ub300\ud55c \uacf5\uc57d\uc744 \uc9c0\ud0a4\uac8c \ubabb\ud558\uac8c \ub418\uc5c8\ub2e4\uba74\uc11c \uc591\ud574\ub97c \uad6c\ud588\ub2e4.", "answer": "\uc815\ud1a0\uc6d0", "sentence": "\ub178\ubb34\ud604\uc740 \uc790\ud0dd \uc606\uc5d0 \uc788\ub358 \uc815\ud1a0\uc6d0 \uc774\ub77c\ub294 \uc0ac\ucc30\uc5d0\uc11c \uc0ac\uc2dc \uacf5\ubd80\ub97c \ud558\uc600\ub2e4.", "paragraph_sentence": " \ub178\ubb34\ud604\uc740 \uc790\ud0dd \uc606\uc5d0 \uc788\ub358 \uc815\ud1a0\uc6d0 \uc774\ub77c\ub294 \uc0ac\ucc30\uc5d0\uc11c \uc0ac\uc2dc \uacf5\ubd80\ub97c \ud558\uc600\ub2e4. \uc774\ud6c4\uc5d0\ub294 \uae40\ud574\uc2dc \uc7a5\uc720\uba74 \ub300\uccad\ub9ac \uc7a5\uc720\uc554\uc5d0 \uba38\ubb34\ub974\uba74\uc11c \uc0ac\uc2dc\ub97c \uc900\ube44\ud588\uc73c\uba70, \ud2c8\ud2c8\uc774 \ubd88\uad50 \uacbd\uc804\uc744 \ud0d0\ub3c5\ud588\ub2e4. 9\ub144\uac04 \uc0ac\uc2dc \uacf5\ubd80\ub97c \ud558\uc5ec 1975\ub144 4\uc6d4 30\uc138\uc5d0 \uc81c17\ud68c \uc0ac\ubc95\uc2dc\ud5d8\uc5d0 \ud569\uaca9\ud558\uc600\ub2e4. \ud3c9\uc18c \ubd88\uc2ec\uc774 \uae4a\uc740 \uac83\uc73c\ub85c \uc54c\ub824\uc9c4 \uc544\ub0b4 \uad8c\uc591\uc219\uc740 2002\ub144 10\uc6d4 1\uc77c \ub2f9\uc2dc \ub300\uc120\uc744 \uc55e\ub450\uace0 \ud569\ucc9c \ud574\uc778\uc0ac\uc5d0 \uba38\ubb3c\uace0 \uc788\ub294 \uc870\uacc4\uc885 \uc885\uc815\uc778 \ubc95\uc804 \uc2a4\ub2d8\uc73c\ub85c\ubd80\ud130 \ubcf4\uc0b4\uacc4\uc640 '\ub300\ub355\ud654'(\u5927\u5fb7\u82b1)\ub77c\ub294 \ubc95\uba85\uc744 \ubc1b\uc558\ub294\ub370, '\ub300\ub355\ud654'\ub294 \ub300\ud55c\ubbfc\uad6d \uc81c5\u00b76\u00b77\u00b78\u00b79\ub300 \ub300\ud1b5\ub839 \ubc15\uc815\ud76c\uc758 \uc544\ub0b4 \uc721\uc601\uc218\uac00 \ubc1b\uc740 \ubc95\uba85\uacfc \uac19\uc740 \uac83\uc774\uc5c8\ub2e4. \uc774\ub7ec\ud55c \ubc95\uba85 \uc218\uacc4\ub294 \ubd88\uad50\uacc4\uc758 \ubbfc\uc2ec\uc774 \uc2e4\ub9b0 \uac83\uc73c\ub85c \ubcfc \uc218 \uc788\uc73c\uba70, \ubd88\uad50\uacc4\uc758 \uae30\ub300\ub97c \ubc1b\uace0 \uc788\uc74c\uc744 \ubcf4\uc5ec\uc8fc\ub294 \uac83\uc774\ub2e4. \uc774\ud6c4 \ub178\ubb34\ud604\uc740 \uc7ac\uc784 \uae30\uac04 \ub3d9\uc548 \ud574\uc778\uc0ac\ub97c \ubb34\ub824 \uc138 \ucc28\ub840\ub098 \ubc29\ubb38\ud574 \ud604\uc9c1 \ub300\ud1b5\ub839\uc73c\ub85c\uc11c\ub294 \ucd5c\ub2e4 \ubc29\ubb38\uc744 \ud558\uc600\ub2e4. 2003\ub144 12\uc6d4 22\uc77c\uc5d0 \ub178\ubb34\ud604\uc740 \uc544\ub0b4\uc640 \ud568\uaed8 \uacbd\ub0a8 \ud569\ucc9c \ud574\uc778\uc0ac\ub97c \ubd88\uc2dc \ubc29\ubb38\ud574 \uc870\uacc4\uc885 \uc885\uc815 \ubc95\uc804\uc2a4\ub2d8, \ucd1d\ubb34\uc6d0\uc7a5 \ubc95\uc7a5\uc2a4\ub2d8\uacfc \ud658\ub2f4\ud55c \ub4a4 \uc624\ucc2c\uc744 \ud568\uaed8 \ud588\ub2e4. \uc0ac\ud328\uc0b0 \ud130\ub110 \ubb38\uc81c\uc5d0 \ub300\ud55c \uacf5\uc57d\uc744 \uc9c0\ud0a4\uac8c \ubabb\ud558\uac8c \ub418\uc5c8\ub2e4\uba74\uc11c \uc591\ud574\ub97c \uad6c\ud588\ub2e4.", "paragraph_answer": "\ub178\ubb34\ud604\uc740 \uc790\ud0dd \uc606\uc5d0 \uc788\ub358 \uc815\ud1a0\uc6d0 \uc774\ub77c\ub294 \uc0ac\ucc30\uc5d0\uc11c \uc0ac\uc2dc \uacf5\ubd80\ub97c \ud558\uc600\ub2e4. \uc774\ud6c4\uc5d0\ub294 \uae40\ud574\uc2dc \uc7a5\uc720\uba74 \ub300\uccad\ub9ac \uc7a5\uc720\uc554\uc5d0 \uba38\ubb34\ub974\uba74\uc11c \uc0ac\uc2dc\ub97c \uc900\ube44\ud588\uc73c\uba70, \ud2c8\ud2c8\uc774 \ubd88\uad50 \uacbd\uc804\uc744 \ud0d0\ub3c5\ud588\ub2e4. 9\ub144\uac04 \uc0ac\uc2dc \uacf5\ubd80\ub97c \ud558\uc5ec 1975\ub144 4\uc6d4 30\uc138\uc5d0 \uc81c17\ud68c \uc0ac\ubc95\uc2dc\ud5d8\uc5d0 \ud569\uaca9\ud558\uc600\ub2e4. \ud3c9\uc18c \ubd88\uc2ec\uc774 \uae4a\uc740 \uac83\uc73c\ub85c \uc54c\ub824\uc9c4 \uc544\ub0b4 \uad8c\uc591\uc219\uc740 2002\ub144 10\uc6d4 1\uc77c \ub2f9\uc2dc \ub300\uc120\uc744 \uc55e\ub450\uace0 \ud569\ucc9c \ud574\uc778\uc0ac\uc5d0 \uba38\ubb3c\uace0 \uc788\ub294 \uc870\uacc4\uc885 \uc885\uc815\uc778 \ubc95\uc804 \uc2a4\ub2d8\uc73c\ub85c\ubd80\ud130 \ubcf4\uc0b4\uacc4\uc640 '\ub300\ub355\ud654'(\u5927\u5fb7\u82b1)\ub77c\ub294 \ubc95\uba85\uc744 \ubc1b\uc558\ub294\ub370, '\ub300\ub355\ud654'\ub294 \ub300\ud55c\ubbfc\uad6d \uc81c5\u00b76\u00b77\u00b78\u00b79\ub300 \ub300\ud1b5\ub839 \ubc15\uc815\ud76c\uc758 \uc544\ub0b4 \uc721\uc601\uc218\uac00 \ubc1b\uc740 \ubc95\uba85\uacfc \uac19\uc740 \uac83\uc774\uc5c8\ub2e4. \uc774\ub7ec\ud55c \ubc95\uba85 \uc218\uacc4\ub294 \ubd88\uad50\uacc4\uc758 \ubbfc\uc2ec\uc774 \uc2e4\ub9b0 \uac83\uc73c\ub85c \ubcfc \uc218 \uc788\uc73c\uba70, \ubd88\uad50\uacc4\uc758 \uae30\ub300\ub97c \ubc1b\uace0 \uc788\uc74c\uc744 \ubcf4\uc5ec\uc8fc\ub294 \uac83\uc774\ub2e4. \uc774\ud6c4 \ub178\ubb34\ud604\uc740 \uc7ac\uc784 \uae30\uac04 \ub3d9\uc548 \ud574\uc778\uc0ac\ub97c \ubb34\ub824 \uc138 \ucc28\ub840\ub098 \ubc29\ubb38\ud574 \ud604\uc9c1 \ub300\ud1b5\ub839\uc73c\ub85c\uc11c\ub294 \ucd5c\ub2e4 \ubc29\ubb38\uc744 \ud558\uc600\ub2e4. 2003\ub144 12\uc6d4 22\uc77c\uc5d0 \ub178\ubb34\ud604\uc740 \uc544\ub0b4\uc640 \ud568\uaed8 \uacbd\ub0a8 \ud569\ucc9c \ud574\uc778\uc0ac\ub97c \ubd88\uc2dc \ubc29\ubb38\ud574 \uc870\uacc4\uc885 \uc885\uc815 \ubc95\uc804\uc2a4\ub2d8, \ucd1d\ubb34\uc6d0\uc7a5 \ubc95\uc7a5\uc2a4\ub2d8\uacfc \ud658\ub2f4\ud55c \ub4a4 \uc624\ucc2c\uc744 \ud568\uaed8 \ud588\ub2e4. \uc0ac\ud328\uc0b0 \ud130\ub110 \ubb38\uc81c\uc5d0 \ub300\ud55c \uacf5\uc57d\uc744 \uc9c0\ud0a4\uac8c \ubabb\ud558\uac8c \ub418\uc5c8\ub2e4\uba74\uc11c \uc591\ud574\ub97c \uad6c\ud588\ub2e4.", "sentence_answer": "\ub178\ubb34\ud604\uc740 \uc790\ud0dd \uc606\uc5d0 \uc788\ub358 \uc815\ud1a0\uc6d0 \uc774\ub77c\ub294 \uc0ac\ucc30\uc5d0\uc11c \uc0ac\uc2dc \uacf5\ubd80\ub97c \ud558\uc600\ub2e4.", "paragraph_id": "6512090-40-0", "paragraph_question": "question: \ub178\ubb34\ud604\uc774 \uc790\ud0dd \uc606\uc5d0\uc11c \uc0ac\uc2dc \uacf5\ubd80\ub97c \ud588\ub358 \uc0ac\ucc30\uc740?, context: \ub178\ubb34\ud604\uc740 \uc790\ud0dd \uc606\uc5d0 \uc788\ub358 \uc815\ud1a0\uc6d0\uc774\ub77c\ub294 \uc0ac\ucc30\uc5d0\uc11c \uc0ac\uc2dc \uacf5\ubd80\ub97c \ud558\uc600\ub2e4. \uc774\ud6c4\uc5d0\ub294 \uae40\ud574\uc2dc \uc7a5\uc720\uba74 \ub300\uccad\ub9ac \uc7a5\uc720\uc554\uc5d0 \uba38\ubb34\ub974\uba74\uc11c \uc0ac\uc2dc\ub97c \uc900\ube44\ud588\uc73c\uba70, \ud2c8\ud2c8\uc774 \ubd88\uad50 \uacbd\uc804\uc744 \ud0d0\ub3c5\ud588\ub2e4. 9\ub144\uac04 \uc0ac\uc2dc \uacf5\ubd80\ub97c \ud558\uc5ec 1975\ub144 4\uc6d4 30\uc138\uc5d0 \uc81c17\ud68c \uc0ac\ubc95\uc2dc\ud5d8\uc5d0 \ud569\uaca9\ud558\uc600\ub2e4. \ud3c9\uc18c \ubd88\uc2ec\uc774 \uae4a\uc740 \uac83\uc73c\ub85c \uc54c\ub824\uc9c4 \uc544\ub0b4 \uad8c\uc591\uc219\uc740 2002\ub144 10\uc6d4 1\uc77c \ub2f9\uc2dc \ub300\uc120\uc744 \uc55e\ub450\uace0 \ud569\ucc9c \ud574\uc778\uc0ac\uc5d0 \uba38\ubb3c\uace0 \uc788\ub294 \uc870\uacc4\uc885 \uc885\uc815\uc778 \ubc95\uc804 \uc2a4\ub2d8\uc73c\ub85c\ubd80\ud130 \ubcf4\uc0b4\uacc4\uc640 '\ub300\ub355\ud654'(\u5927\u5fb7\u82b1)\ub77c\ub294 \ubc95\uba85\uc744 \ubc1b\uc558\ub294\ub370, '\ub300\ub355\ud654'\ub294 \ub300\ud55c\ubbfc\uad6d \uc81c5\u00b76\u00b77\u00b78\u00b79\ub300 \ub300\ud1b5\ub839 \ubc15\uc815\ud76c\uc758 \uc544\ub0b4 \uc721\uc601\uc218\uac00 \ubc1b\uc740 \ubc95\uba85\uacfc \uac19\uc740 \uac83\uc774\uc5c8\ub2e4. \uc774\ub7ec\ud55c \ubc95\uba85 \uc218\uacc4\ub294 \ubd88\uad50\uacc4\uc758 \ubbfc\uc2ec\uc774 \uc2e4\ub9b0 \uac83\uc73c\ub85c \ubcfc \uc218 \uc788\uc73c\uba70, \ubd88\uad50\uacc4\uc758 \uae30\ub300\ub97c \ubc1b\uace0 \uc788\uc74c\uc744 \ubcf4\uc5ec\uc8fc\ub294 \uac83\uc774\ub2e4. \uc774\ud6c4 \ub178\ubb34\ud604\uc740 \uc7ac\uc784 \uae30\uac04 \ub3d9\uc548 \ud574\uc778\uc0ac\ub97c \ubb34\ub824 \uc138 \ucc28\ub840\ub098 \ubc29\ubb38\ud574 \ud604\uc9c1 \ub300\ud1b5\ub839\uc73c\ub85c\uc11c\ub294 \ucd5c\ub2e4 \ubc29\ubb38\uc744 \ud558\uc600\ub2e4. 2003\ub144 12\uc6d4 22\uc77c\uc5d0 \ub178\ubb34\ud604\uc740 \uc544\ub0b4\uc640 \ud568\uaed8 \uacbd\ub0a8 \ud569\ucc9c \ud574\uc778\uc0ac\ub97c \ubd88\uc2dc \ubc29\ubb38\ud574 \uc870\uacc4\uc885 \uc885\uc815 \ubc95\uc804\uc2a4\ub2d8, \ucd1d\ubb34\uc6d0\uc7a5 \ubc95\uc7a5\uc2a4\ub2d8\uacfc \ud658\ub2f4\ud55c \ub4a4 \uc624\ucc2c\uc744 \ud568\uaed8 \ud588\ub2e4. \uc0ac\ud328\uc0b0 \ud130\ub110 \ubb38\uc81c\uc5d0 \ub300\ud55c \uacf5\uc57d\uc744 \uc9c0\ud0a4\uac8c \ubabb\ud558\uac8c \ub418\uc5c8\ub2e4\uba74\uc11c \uc591\ud574\ub97c \uad6c\ud588\ub2e4."} {"question": "\ud589\uc624\ubc84\uc5d0 \uc774\uc5b4 \ube0c\ub798\ub4e4\ub9ac \ucfe0\ud37c\uac00 \ucd9c\uc5f0\ud55c \uc791\ud488\uc73c\ub85c \uc5f0\uae30\uc5d0 \ud639\ud3c9\uc744 \ubc1b\uc558\ub358 \uc601\ud654\ub294?", "paragraph": "2009\ub144 2\uc6d4 7\uc77c, \uc74c\uc545 \uac8c\uc2a4\ud2b8 TV \uc628 \ub354 \ub77c\ub514\uc624\uc640 \ud568\uaed8 \u300a\uc0c8\ud130\ub370\uc774 \ub098\uc774\ud2b8 \ub77c\uc774\ube0c\u300b\uc758 \uc9c4\ud589\uc744 \ub9e1\uc558\ub2e4. \uac19\uc740 \ud574 \uacf5\uac1c\ub41c \u300a\uadf8\ub294 \ub2f9\uc2e0\uc5d0\uac8c \ubc18\ud558\uc9c0 \uc54a\uc558\ub2e4\u300b\uc5d0\uc11c \ubca4 \uac74\ub354\uc2a4 \uc5ed\uc744 \ub9e1\uc544 \uc81c\ub2c8\ud37c \ucf54\ub110\ub9ac\uc640 \uc2a4\uce7c\ub9bf \uc870\ud578\uc2a8\uacfc \ud638\ud761\uc744 \ub9de\ucd94\uc5c8\ub2e4. \ub610\ud55c, \u300a\ud589\uc624\ubc84\u300b\uc5d0\uc11c\ub294 \ud544 \uc6e8\ub125 \uc5ed\uc744 \ub9e1\uc544 \uc5f0\uae30\ud558\uc600\ub2e4. \uc601\ud654\ub294 \ud06c\uac8c \ud765\ud589\uc5d0 \uc131\uacf5\ud558\uc5ec \ucd1d \uc57d 4\uc5b5 6\ucc9c\ub9cc \ub2ec\ub7ec\uc758 \ud765\ud589 \uc218\uc785\uc744 \uc62c\ub838\ub2e4. \ube44\ud3c9 \ub610\ud55c \uc88b\uc740 \ud3c9\uac00\ub97c \ubc1b\uc558\ub2e4. \u300a\ud589\uc624\ubc84\u300b\uc758 \ud765\ud589\uc5d0 \ub300\ud574 \ucfe0\ud37c\ub294 2011\ub144 \u3008\uc170\uc774\ube0c\u3009 \uc7a1\uc9c0\uc640\uc758 \uc778\ud130\ubdf0\uc5d0\uc11c \u201c\uac19\uc740 \uac83\uc774\ub2e4. \uadf8\ub7ec\ub2c8\uae4c, \ub354 \ub9ce\uc740 \ubb38\uc740 \ud655\uc2e4\ud788 \uc5f4\ub824 \uc788\uc9c0\ub9cc, \uc6d4\uc694\uc77c \uc544\uce68\uc5d0 \ub2f4\ubc30\ub97c \ubb3c\uace0 \uc549\uc544 \uc81c\uacf5\ubc1b\uc740 \ub300\ubcf8\uc744 \uc77d\uc740 \uac83\uc740 \uc544\ub2c8\ub2e4.\u201d\ub77c\uace0 \ud558\uc600\ub2e4. \u300a\ud589\uc624\ubc84\u300b\uc5d0 \uc774\uc5b4 \uacf5\uac1c\ub41c \u300a\uc62c \uc5b4\ubc14\uc6c3 \uc2a4\ud2f0\ube0c\u300b\uc5d0\uc11c\ub294 \uc2a4\ud2f0\ube0c \ubc00\ub7ec \uc5ed\uc744 \ub9e1\uc544 \uc0cc\ub4dc\ub77c \ubd88\ub7ed\uacfc \uac19\uc774 \uc5f0\uae30\ud558\uc600\ub2e4. \u300a\ud589\uc624\ubc84\u300b\uc640\ub294 \ub2ec\ub9ac \uc601\ud654\uc758 \uc791\ud488\uc131\uacfc \uc5f0\uae30\uc5d0 \ub300\ud574 \uc88b\uc9c0 \uc54a\uc740 \ud3c9\uc744 \ubc1b\uc558\uc73c\uba70, 2010\ub144 \uace8\ub4e0 \ub798\uc988\ubca0\ub9ac\uc0c1\uc5d0\uc11c \ubd88\ub7ed\uacfc \uac19\uc774 \ucd5c\uc545\uc758 \uc2a4\ud06c\ub9b0 \ucee4\ud50c \uc0c1\uc744 \uc218\uc0c1\ud558\uc600\ub2e4. \ud765\ud589 \uba74\uc5d0\uc11c\ub294 \uc57d 4\ucc9c\ub9cc \ub2ec\ub7ec\uc758 \uc218\uc785\uc744 \uc62c\ub824 \ubbf8\uc57d\ud55c \ud765\ud589\uc744 \ud558\uc600\ub2e4. \uac19\uc740 \ud574 \uacf5\uac1c\ub41c \u300a\ucf00\uc774\uc2a4 39\u300b\uc5d0\uc11c\ub294 \ub354\uae00\ub7ec\uc2a4 J. \uc5d0\uc774\ubbf8\uc2a4 \uc5ed\uc73c\ub85c \ucd9c\uc5f0\ud558\uc600\ub2e4. 2009\ub144 4\uc6d4 \ub77c\uc2a4\ubca0\uc774\uac70\uc2a4\uc5d0\uc11c \uc5f4\ub9b0 \uc1fc\uc6e8\uc2a4\ud2b8 \ucee8\ubca4\uc158\uc5d0\uc11c \uc62c\ud574\uc758 \ucf54\ubbf8\ub514 \uc2a4\ud0c0 \uc0c1\uc744 \uc218\uc0c1\ud558\uc600\uc73c\uba70, 10\uc6d4\uc5d0\ub294 \ud560\ub9ac\uc6b0\ub4dc \uc601\ud654\uc0c1 \ucf54\ubbf8\ub514 \uc601\ud654\uc0c1 \ubd80\ubb38\uc5d0\uc11c \uc218\uc0c1\ud558\uc600\ub2e4.", "answer": "\uc62c \uc5b4\ubc14\uc6c3 \uc2a4\ud2f0\ube0c", "sentence": "\u300a\ud589\uc624\ubc84\u300b\uc5d0 \uc774\uc5b4 \uacf5\uac1c\ub41c \u300a \uc62c \uc5b4\ubc14\uc6c3 \uc2a4\ud2f0\ube0c \u300b\uc5d0\uc11c\ub294 \uc2a4\ud2f0\ube0c \ubc00\ub7ec \uc5ed\uc744 \ub9e1\uc544 \uc0cc\ub4dc\ub77c \ubd88\ub7ed\uacfc \uac19\uc774 \uc5f0\uae30\ud558\uc600\ub2e4.", "paragraph_sentence": "2009\ub144 2\uc6d4 7\uc77c, \uc74c\uc545 \uac8c\uc2a4\ud2b8 TV \uc628 \ub354 \ub77c\ub514\uc624\uc640 \ud568\uaed8 \u300a\uc0c8\ud130\ub370\uc774 \ub098\uc774\ud2b8 \ub77c\uc774\ube0c\u300b\uc758 \uc9c4\ud589\uc744 \ub9e1\uc558\ub2e4. \uac19\uc740 \ud574 \uacf5\uac1c\ub41c \u300a\uadf8\ub294 \ub2f9\uc2e0\uc5d0\uac8c \ubc18\ud558\uc9c0 \uc54a\uc558\ub2e4\u300b\uc5d0\uc11c \ubca4 \uac74\ub354\uc2a4 \uc5ed\uc744 \ub9e1\uc544 \uc81c\ub2c8\ud37c \ucf54\ub110\ub9ac\uc640 \uc2a4\uce7c\ub9bf \uc870\ud578\uc2a8\uacfc \ud638\ud761\uc744 \ub9de\ucd94\uc5c8\ub2e4. \ub610\ud55c, \u300a\ud589\uc624\ubc84\u300b\uc5d0\uc11c\ub294 \ud544 \uc6e8\ub125 \uc5ed\uc744 \ub9e1\uc544 \uc5f0\uae30\ud558\uc600\ub2e4. \uc601\ud654\ub294 \ud06c\uac8c \ud765\ud589\uc5d0 \uc131\uacf5\ud558\uc5ec \ucd1d \uc57d 4\uc5b5 6\ucc9c\ub9cc \ub2ec\ub7ec\uc758 \ud765\ud589 \uc218\uc785\uc744 \uc62c\ub838\ub2e4. \ube44\ud3c9 \ub610\ud55c \uc88b\uc740 \ud3c9\uac00\ub97c \ubc1b\uc558\ub2e4. \u300a\ud589\uc624\ubc84\u300b\uc758 \ud765\ud589\uc5d0 \ub300\ud574 \ucfe0\ud37c\ub294 2011\ub144 \u3008\uc170\uc774\ube0c\u3009 \uc7a1\uc9c0\uc640\uc758 \uc778\ud130\ubdf0\uc5d0\uc11c \u201c\uac19\uc740 \uac83\uc774\ub2e4. \uadf8\ub7ec\ub2c8\uae4c, \ub354 \ub9ce\uc740 \ubb38\uc740 \ud655\uc2e4\ud788 \uc5f4\ub824 \uc788\uc9c0\ub9cc, \uc6d4\uc694\uc77c \uc544\uce68\uc5d0 \ub2f4\ubc30\ub97c \ubb3c\uace0 \uc549\uc544 \uc81c\uacf5\ubc1b\uc740 \ub300\ubcf8\uc744 \uc77d\uc740 \uac83\uc740 \uc544\ub2c8\ub2e4. \u201d\ub77c\uace0 \ud558\uc600\ub2e4. \u300a\ud589\uc624\ubc84\u300b\uc5d0 \uc774\uc5b4 \uacf5\uac1c\ub41c \u300a \uc62c \uc5b4\ubc14\uc6c3 \uc2a4\ud2f0\ube0c \u300b\uc5d0\uc11c\ub294 \uc2a4\ud2f0\ube0c \ubc00\ub7ec \uc5ed\uc744 \ub9e1\uc544 \uc0cc\ub4dc\ub77c \ubd88\ub7ed\uacfc \uac19\uc774 \uc5f0\uae30\ud558\uc600\ub2e4. \u300a\ud589\uc624\ubc84\u300b\uc640\ub294 \ub2ec\ub9ac \uc601\ud654\uc758 \uc791\ud488\uc131\uacfc \uc5f0\uae30\uc5d0 \ub300\ud574 \uc88b\uc9c0 \uc54a\uc740 \ud3c9\uc744 \ubc1b\uc558\uc73c\uba70, 2010\ub144 \uace8\ub4e0 \ub798\uc988\ubca0\ub9ac\uc0c1\uc5d0\uc11c \ubd88\ub7ed\uacfc \uac19\uc774 \ucd5c\uc545\uc758 \uc2a4\ud06c\ub9b0 \ucee4\ud50c \uc0c1\uc744 \uc218\uc0c1\ud558\uc600\ub2e4. \ud765\ud589 \uba74\uc5d0\uc11c\ub294 \uc57d 4\ucc9c\ub9cc \ub2ec\ub7ec\uc758 \uc218\uc785\uc744 \uc62c\ub824 \ubbf8\uc57d\ud55c \ud765\ud589\uc744 \ud558\uc600\ub2e4. \uac19\uc740 \ud574 \uacf5\uac1c\ub41c \u300a\ucf00\uc774\uc2a4 39\u300b\uc5d0\uc11c\ub294 \ub354\uae00\ub7ec\uc2a4 J. \uc5d0\uc774\ubbf8\uc2a4 \uc5ed\uc73c\ub85c \ucd9c\uc5f0\ud558\uc600\ub2e4. 2009\ub144 4\uc6d4 \ub77c\uc2a4\ubca0\uc774\uac70\uc2a4\uc5d0\uc11c \uc5f4\ub9b0 \uc1fc\uc6e8\uc2a4\ud2b8 \ucee8\ubca4\uc158\uc5d0\uc11c \uc62c\ud574\uc758 \ucf54\ubbf8\ub514 \uc2a4\ud0c0 \uc0c1\uc744 \uc218\uc0c1\ud558\uc600\uc73c\uba70, 10\uc6d4\uc5d0\ub294 \ud560\ub9ac\uc6b0\ub4dc \uc601\ud654\uc0c1 \ucf54\ubbf8\ub514 \uc601\ud654\uc0c1 \ubd80\ubb38\uc5d0\uc11c \uc218\uc0c1\ud558\uc600\ub2e4.", "paragraph_answer": "2009\ub144 2\uc6d4 7\uc77c, \uc74c\uc545 \uac8c\uc2a4\ud2b8 TV \uc628 \ub354 \ub77c\ub514\uc624\uc640 \ud568\uaed8 \u300a\uc0c8\ud130\ub370\uc774 \ub098\uc774\ud2b8 \ub77c\uc774\ube0c\u300b\uc758 \uc9c4\ud589\uc744 \ub9e1\uc558\ub2e4. \uac19\uc740 \ud574 \uacf5\uac1c\ub41c \u300a\uadf8\ub294 \ub2f9\uc2e0\uc5d0\uac8c \ubc18\ud558\uc9c0 \uc54a\uc558\ub2e4\u300b\uc5d0\uc11c \ubca4 \uac74\ub354\uc2a4 \uc5ed\uc744 \ub9e1\uc544 \uc81c\ub2c8\ud37c \ucf54\ub110\ub9ac\uc640 \uc2a4\uce7c\ub9bf \uc870\ud578\uc2a8\uacfc \ud638\ud761\uc744 \ub9de\ucd94\uc5c8\ub2e4. \ub610\ud55c, \u300a\ud589\uc624\ubc84\u300b\uc5d0\uc11c\ub294 \ud544 \uc6e8\ub125 \uc5ed\uc744 \ub9e1\uc544 \uc5f0\uae30\ud558\uc600\ub2e4. \uc601\ud654\ub294 \ud06c\uac8c \ud765\ud589\uc5d0 \uc131\uacf5\ud558\uc5ec \ucd1d \uc57d 4\uc5b5 6\ucc9c\ub9cc \ub2ec\ub7ec\uc758 \ud765\ud589 \uc218\uc785\uc744 \uc62c\ub838\ub2e4. \ube44\ud3c9 \ub610\ud55c \uc88b\uc740 \ud3c9\uac00\ub97c \ubc1b\uc558\ub2e4. \u300a\ud589\uc624\ubc84\u300b\uc758 \ud765\ud589\uc5d0 \ub300\ud574 \ucfe0\ud37c\ub294 2011\ub144 \u3008\uc170\uc774\ube0c\u3009 \uc7a1\uc9c0\uc640\uc758 \uc778\ud130\ubdf0\uc5d0\uc11c \u201c\uac19\uc740 \uac83\uc774\ub2e4. \uadf8\ub7ec\ub2c8\uae4c, \ub354 \ub9ce\uc740 \ubb38\uc740 \ud655\uc2e4\ud788 \uc5f4\ub824 \uc788\uc9c0\ub9cc, \uc6d4\uc694\uc77c \uc544\uce68\uc5d0 \ub2f4\ubc30\ub97c \ubb3c\uace0 \uc549\uc544 \uc81c\uacf5\ubc1b\uc740 \ub300\ubcf8\uc744 \uc77d\uc740 \uac83\uc740 \uc544\ub2c8\ub2e4.\u201d\ub77c\uace0 \ud558\uc600\ub2e4. \u300a\ud589\uc624\ubc84\u300b\uc5d0 \uc774\uc5b4 \uacf5\uac1c\ub41c \u300a \uc62c \uc5b4\ubc14\uc6c3 \uc2a4\ud2f0\ube0c \u300b\uc5d0\uc11c\ub294 \uc2a4\ud2f0\ube0c \ubc00\ub7ec \uc5ed\uc744 \ub9e1\uc544 \uc0cc\ub4dc\ub77c \ubd88\ub7ed\uacfc \uac19\uc774 \uc5f0\uae30\ud558\uc600\ub2e4. \u300a\ud589\uc624\ubc84\u300b\uc640\ub294 \ub2ec\ub9ac \uc601\ud654\uc758 \uc791\ud488\uc131\uacfc \uc5f0\uae30\uc5d0 \ub300\ud574 \uc88b\uc9c0 \uc54a\uc740 \ud3c9\uc744 \ubc1b\uc558\uc73c\uba70, 2010\ub144 \uace8\ub4e0 \ub798\uc988\ubca0\ub9ac\uc0c1\uc5d0\uc11c \ubd88\ub7ed\uacfc \uac19\uc774 \ucd5c\uc545\uc758 \uc2a4\ud06c\ub9b0 \ucee4\ud50c \uc0c1\uc744 \uc218\uc0c1\ud558\uc600\ub2e4. \ud765\ud589 \uba74\uc5d0\uc11c\ub294 \uc57d 4\ucc9c\ub9cc \ub2ec\ub7ec\uc758 \uc218\uc785\uc744 \uc62c\ub824 \ubbf8\uc57d\ud55c \ud765\ud589\uc744 \ud558\uc600\ub2e4. \uac19\uc740 \ud574 \uacf5\uac1c\ub41c \u300a\ucf00\uc774\uc2a4 39\u300b\uc5d0\uc11c\ub294 \ub354\uae00\ub7ec\uc2a4 J. \uc5d0\uc774\ubbf8\uc2a4 \uc5ed\uc73c\ub85c \ucd9c\uc5f0\ud558\uc600\ub2e4. 2009\ub144 4\uc6d4 \ub77c\uc2a4\ubca0\uc774\uac70\uc2a4\uc5d0\uc11c \uc5f4\ub9b0 \uc1fc\uc6e8\uc2a4\ud2b8 \ucee8\ubca4\uc158\uc5d0\uc11c \uc62c\ud574\uc758 \ucf54\ubbf8\ub514 \uc2a4\ud0c0 \uc0c1\uc744 \uc218\uc0c1\ud558\uc600\uc73c\uba70, 10\uc6d4\uc5d0\ub294 \ud560\ub9ac\uc6b0\ub4dc \uc601\ud654\uc0c1 \ucf54\ubbf8\ub514 \uc601\ud654\uc0c1 \ubd80\ubb38\uc5d0\uc11c \uc218\uc0c1\ud558\uc600\ub2e4.", "sentence_answer": "\u300a\ud589\uc624\ubc84\u300b\uc5d0 \uc774\uc5b4 \uacf5\uac1c\ub41c \u300a \uc62c \uc5b4\ubc14\uc6c3 \uc2a4\ud2f0\ube0c \u300b\uc5d0\uc11c\ub294 \uc2a4\ud2f0\ube0c \ubc00\ub7ec \uc5ed\uc744 \ub9e1\uc544 \uc0cc\ub4dc\ub77c \ubd88\ub7ed\uacfc \uac19\uc774 \uc5f0\uae30\ud558\uc600\ub2e4.", "paragraph_id": "6509370-3-1", "paragraph_question": "question: \ud589\uc624\ubc84\uc5d0 \uc774\uc5b4 \ube0c\ub798\ub4e4\ub9ac \ucfe0\ud37c\uac00 \ucd9c\uc5f0\ud55c \uc791\ud488\uc73c\ub85c \uc5f0\uae30\uc5d0 \ud639\ud3c9\uc744 \ubc1b\uc558\ub358 \uc601\ud654\ub294?, context: 2009\ub144 2\uc6d4 7\uc77c, \uc74c\uc545 \uac8c\uc2a4\ud2b8 TV \uc628 \ub354 \ub77c\ub514\uc624\uc640 \ud568\uaed8 \u300a\uc0c8\ud130\ub370\uc774 \ub098\uc774\ud2b8 \ub77c\uc774\ube0c\u300b\uc758 \uc9c4\ud589\uc744 \ub9e1\uc558\ub2e4. \uac19\uc740 \ud574 \uacf5\uac1c\ub41c \u300a\uadf8\ub294 \ub2f9\uc2e0\uc5d0\uac8c \ubc18\ud558\uc9c0 \uc54a\uc558\ub2e4\u300b\uc5d0\uc11c \ubca4 \uac74\ub354\uc2a4 \uc5ed\uc744 \ub9e1\uc544 \uc81c\ub2c8\ud37c \ucf54\ub110\ub9ac\uc640 \uc2a4\uce7c\ub9bf \uc870\ud578\uc2a8\uacfc \ud638\ud761\uc744 \ub9de\ucd94\uc5c8\ub2e4. \ub610\ud55c, \u300a\ud589\uc624\ubc84\u300b\uc5d0\uc11c\ub294 \ud544 \uc6e8\ub125 \uc5ed\uc744 \ub9e1\uc544 \uc5f0\uae30\ud558\uc600\ub2e4. \uc601\ud654\ub294 \ud06c\uac8c \ud765\ud589\uc5d0 \uc131\uacf5\ud558\uc5ec \ucd1d \uc57d 4\uc5b5 6\ucc9c\ub9cc \ub2ec\ub7ec\uc758 \ud765\ud589 \uc218\uc785\uc744 \uc62c\ub838\ub2e4. \ube44\ud3c9 \ub610\ud55c \uc88b\uc740 \ud3c9\uac00\ub97c \ubc1b\uc558\ub2e4. \u300a\ud589\uc624\ubc84\u300b\uc758 \ud765\ud589\uc5d0 \ub300\ud574 \ucfe0\ud37c\ub294 2011\ub144 \u3008\uc170\uc774\ube0c\u3009 \uc7a1\uc9c0\uc640\uc758 \uc778\ud130\ubdf0\uc5d0\uc11c \u201c\uac19\uc740 \uac83\uc774\ub2e4. \uadf8\ub7ec\ub2c8\uae4c, \ub354 \ub9ce\uc740 \ubb38\uc740 \ud655\uc2e4\ud788 \uc5f4\ub824 \uc788\uc9c0\ub9cc, \uc6d4\uc694\uc77c \uc544\uce68\uc5d0 \ub2f4\ubc30\ub97c \ubb3c\uace0 \uc549\uc544 \uc81c\uacf5\ubc1b\uc740 \ub300\ubcf8\uc744 \uc77d\uc740 \uac83\uc740 \uc544\ub2c8\ub2e4.\u201d\ub77c\uace0 \ud558\uc600\ub2e4. \u300a\ud589\uc624\ubc84\u300b\uc5d0 \uc774\uc5b4 \uacf5\uac1c\ub41c \u300a\uc62c \uc5b4\ubc14\uc6c3 \uc2a4\ud2f0\ube0c\u300b\uc5d0\uc11c\ub294 \uc2a4\ud2f0\ube0c \ubc00\ub7ec \uc5ed\uc744 \ub9e1\uc544 \uc0cc\ub4dc\ub77c \ubd88\ub7ed\uacfc \uac19\uc774 \uc5f0\uae30\ud558\uc600\ub2e4. \u300a\ud589\uc624\ubc84\u300b\uc640\ub294 \ub2ec\ub9ac \uc601\ud654\uc758 \uc791\ud488\uc131\uacfc \uc5f0\uae30\uc5d0 \ub300\ud574 \uc88b\uc9c0 \uc54a\uc740 \ud3c9\uc744 \ubc1b\uc558\uc73c\uba70, 2010\ub144 \uace8\ub4e0 \ub798\uc988\ubca0\ub9ac\uc0c1\uc5d0\uc11c \ubd88\ub7ed\uacfc \uac19\uc774 \ucd5c\uc545\uc758 \uc2a4\ud06c\ub9b0 \ucee4\ud50c \uc0c1\uc744 \uc218\uc0c1\ud558\uc600\ub2e4. \ud765\ud589 \uba74\uc5d0\uc11c\ub294 \uc57d 4\ucc9c\ub9cc \ub2ec\ub7ec\uc758 \uc218\uc785\uc744 \uc62c\ub824 \ubbf8\uc57d\ud55c \ud765\ud589\uc744 \ud558\uc600\ub2e4. \uac19\uc740 \ud574 \uacf5\uac1c\ub41c \u300a\ucf00\uc774\uc2a4 39\u300b\uc5d0\uc11c\ub294 \ub354\uae00\ub7ec\uc2a4 J. \uc5d0\uc774\ubbf8\uc2a4 \uc5ed\uc73c\ub85c \ucd9c\uc5f0\ud558\uc600\ub2e4. 2009\ub144 4\uc6d4 \ub77c\uc2a4\ubca0\uc774\uac70\uc2a4\uc5d0\uc11c \uc5f4\ub9b0 \uc1fc\uc6e8\uc2a4\ud2b8 \ucee8\ubca4\uc158\uc5d0\uc11c \uc62c\ud574\uc758 \ucf54\ubbf8\ub514 \uc2a4\ud0c0 \uc0c1\uc744 \uc218\uc0c1\ud558\uc600\uc73c\uba70, 10\uc6d4\uc5d0\ub294 \ud560\ub9ac\uc6b0\ub4dc \uc601\ud654\uc0c1 \ucf54\ubbf8\ub514 \uc601\ud654\uc0c1 \ubd80\ubb38\uc5d0\uc11c \uc218\uc0c1\ud558\uc600\ub2e4."} {"question": "\ub525 \uc784\ud329\ud2b8\ub294 \uc5b4\ub5a4 \ucc9c\uccb4\uc758 \ub0b4\ubd80 \uc870\uc131\uc744 \uc5f0\uad6c\ud558\uae30 \uc704\ud55c \uc6b0\uc8fc\uc120\uc774\uc5c8\ub294\ub370 \uc774 \ucc9c\uccb4\ub294?", "paragraph": "\ub525 \uc784\ud329\ud2b8(\uc601\uc5b4: Deep Impact)\ub294 \ucf00\uc774\ud504\ucee4\ub0b4\ubc84\ub7f4 \uacf5\uad70 \uae30\uc9c0\uc5d0\uc11c 2005\ub144 1\uc6d4 12\uc77c 18\uc2dc 47\ubd84(UTC)\uc5d0 \ubc1c\uc0ac\ub41c \ubbf8\uad6d \ud56d\uacf5\uc6b0\uc8fc\uad6d\uc758 \uc6b0\uc8fc \ud0d0\uc0ac\uc120\uc774\ub2e4. \ub525 \uc784\ud329\ud2b8\uc758 \uc784\ubb34\ub294 \ud61c\uc131 \ud15c\ud3a0 1(9P/\ud15c\ud3a0)\uc758 \ub0b4\ubd80 \uc870\uc131\uc744 \uc5f0\uad6c\ud558\uae30 \uc704\ud574 \ucda9\ub3cc\uae30\ub97c \ud61c\uc131\uc73c\ub85c \ub099\ud558\uc2dc\ud0a4\ub294 \uac83\uc774\uc5c8\uace0, 2005\ub144 7\uc6d4 4\uc77c \ud611\uc815 \uc138\uacc4\uc2dc 05:52\uc5d0 \ucda9\ub3cc\uae30\ub294 \uc131\uacf5\uc801\uc73c\ub85c \ud61c\uc131\uc758 \ud575\uc5d0 \ucda9\ub3cc\ud588\ub2e4. \ucda9\ub3cc\uae30\ub294 \ucda9\ub3cc\ud558\uba74\uc11c \ud61c\uc131\uc758 \ud575\uc5d0 \ucda9\ub3cc\uad6c\ub97c \ub9cc\ub4e4\uace0, \ud575\uc758 \uad6c\uc131 \ubb3c\uc9c8\uc744 \ubc16\uc73c\ub85c \ub0a0\ub824 \ubcf4\ub0c8\ub2e4. \uc6b0\uc8fc\uc120\uc5d0\uc11c \ucc0d\uc740, \ubb3c\uc9c8\uc774 \ubd84\ucd9c\ub418\ub294 \uc0ac\uc9c4\uc5d0\uc11c \ud61c\uc131\uc758 \ud575\uc740 \uc608\uc0c1\ubcf4\ub2e4 \uba3c\uc9c0\uac00 \ub9ce\uace0 \uc5bc\uc74c\uc774 \uc801\ub2e4\ub294 \uac83\uc744 \uc54c\uc544\ub0c8\ub294\ub370, \uadf8 \uc774\uc720\ub294 \ucda9\ub3cc\uae30\uac00 \ucda9\ub3cc\ud558\uba74\uc11c \uc608\uae30\uce58 \uc54a\uac8c \ud06c\uace0 \ubc1d\uc740 \uba3c\uc9c0\uad6c\ub984\uc744 \ub9cc\ub4e4\uc5b4, \uc2dc\uc57c\ub97c \uac00\ub838\uae30 \ub54c\ubb38\uc774\ub2e4.", "answer": "\ud61c\uc131", "sentence": "\ub525 \uc784\ud329\ud2b8\uc758 \uc784\ubb34\ub294 \ud61c\uc131 \ud15c\ud3a0 1(9P/\ud15c\ud3a0)\uc758 \ub0b4\ubd80 \uc870\uc131\uc744 \uc5f0\uad6c\ud558\uae30 \uc704\ud574 \ucda9\ub3cc\uae30\ub97c \ud61c\uc131\uc73c\ub85c \ub099\ud558\uc2dc\ud0a4\ub294 \uac83\uc774\uc5c8\uace0, 2005\ub144 7\uc6d4 4\uc77c \ud611\uc815 \uc138\uacc4\uc2dc 05:52\uc5d0 \ucda9\ub3cc\uae30\ub294 \uc131\uacf5\uc801\uc73c\ub85c \ud61c\uc131\uc758 \ud575\uc5d0 \ucda9\ub3cc\ud588\ub2e4.", "paragraph_sentence": "\ub525 \uc784\ud329\ud2b8(\uc601\uc5b4: Deep Impact)\ub294 \ucf00\uc774\ud504\ucee4\ub0b4\ubc84\ub7f4 \uacf5\uad70 \uae30\uc9c0\uc5d0\uc11c 2005\ub144 1\uc6d4 12\uc77c 18\uc2dc 47\ubd84(UTC)\uc5d0 \ubc1c\uc0ac\ub41c \ubbf8\uad6d \ud56d\uacf5\uc6b0\uc8fc\uad6d\uc758 \uc6b0\uc8fc \ud0d0\uc0ac\uc120\uc774\ub2e4. \ub525 \uc784\ud329\ud2b8\uc758 \uc784\ubb34\ub294 \ud61c\uc131 \ud15c\ud3a0 1(9P/\ud15c\ud3a0)\uc758 \ub0b4\ubd80 \uc870\uc131\uc744 \uc5f0\uad6c\ud558\uae30 \uc704\ud574 \ucda9\ub3cc\uae30\ub97c \ud61c\uc131\uc73c\ub85c \ub099\ud558\uc2dc\ud0a4\ub294 \uac83\uc774\uc5c8\uace0, 2005\ub144 7\uc6d4 4\uc77c \ud611\uc815 \uc138\uacc4\uc2dc 05:52\uc5d0 \ucda9\ub3cc\uae30\ub294 \uc131\uacf5\uc801\uc73c\ub85c \ud61c\uc131\uc758 \ud575\uc5d0 \ucda9\ub3cc\ud588\ub2e4. \ucda9\ub3cc\uae30\ub294 \ucda9\ub3cc\ud558\uba74\uc11c \ud61c\uc131\uc758 \ud575\uc5d0 \ucda9\ub3cc\uad6c\ub97c \ub9cc\ub4e4\uace0, \ud575\uc758 \uad6c\uc131 \ubb3c\uc9c8\uc744 \ubc16\uc73c\ub85c \ub0a0\ub824 \ubcf4\ub0c8\ub2e4. \uc6b0\uc8fc\uc120\uc5d0\uc11c \ucc0d\uc740, \ubb3c\uc9c8\uc774 \ubd84\ucd9c\ub418\ub294 \uc0ac\uc9c4\uc5d0\uc11c \ud61c\uc131\uc758 \ud575\uc740 \uc608\uc0c1\ubcf4\ub2e4 \uba3c\uc9c0\uac00 \ub9ce\uace0 \uc5bc\uc74c\uc774 \uc801\ub2e4\ub294 \uac83\uc744 \uc54c\uc544\ub0c8\ub294\ub370, \uadf8 \uc774\uc720\ub294 \ucda9\ub3cc\uae30\uac00 \ucda9\ub3cc\ud558\uba74\uc11c \uc608\uae30\uce58 \uc54a\uac8c \ud06c\uace0 \ubc1d\uc740 \uba3c\uc9c0\uad6c\ub984\uc744 \ub9cc\ub4e4\uc5b4, \uc2dc\uc57c\ub97c \uac00\ub838\uae30 \ub54c\ubb38\uc774\ub2e4.", "paragraph_answer": "\ub525 \uc784\ud329\ud2b8(\uc601\uc5b4: Deep Impact)\ub294 \ucf00\uc774\ud504\ucee4\ub0b4\ubc84\ub7f4 \uacf5\uad70 \uae30\uc9c0\uc5d0\uc11c 2005\ub144 1\uc6d4 12\uc77c 18\uc2dc 47\ubd84(UTC)\uc5d0 \ubc1c\uc0ac\ub41c \ubbf8\uad6d \ud56d\uacf5\uc6b0\uc8fc\uad6d\uc758 \uc6b0\uc8fc \ud0d0\uc0ac\uc120\uc774\ub2e4. \ub525 \uc784\ud329\ud2b8\uc758 \uc784\ubb34\ub294 \ud61c\uc131 \ud15c\ud3a0 1(9P/\ud15c\ud3a0)\uc758 \ub0b4\ubd80 \uc870\uc131\uc744 \uc5f0\uad6c\ud558\uae30 \uc704\ud574 \ucda9\ub3cc\uae30\ub97c \ud61c\uc131\uc73c\ub85c \ub099\ud558\uc2dc\ud0a4\ub294 \uac83\uc774\uc5c8\uace0, 2005\ub144 7\uc6d4 4\uc77c \ud611\uc815 \uc138\uacc4\uc2dc 05:52\uc5d0 \ucda9\ub3cc\uae30\ub294 \uc131\uacf5\uc801\uc73c\ub85c \ud61c\uc131\uc758 \ud575\uc5d0 \ucda9\ub3cc\ud588\ub2e4. \ucda9\ub3cc\uae30\ub294 \ucda9\ub3cc\ud558\uba74\uc11c \ud61c\uc131\uc758 \ud575\uc5d0 \ucda9\ub3cc\uad6c\ub97c \ub9cc\ub4e4\uace0, \ud575\uc758 \uad6c\uc131 \ubb3c\uc9c8\uc744 \ubc16\uc73c\ub85c \ub0a0\ub824 \ubcf4\ub0c8\ub2e4. \uc6b0\uc8fc\uc120\uc5d0\uc11c \ucc0d\uc740, \ubb3c\uc9c8\uc774 \ubd84\ucd9c\ub418\ub294 \uc0ac\uc9c4\uc5d0\uc11c \ud61c\uc131\uc758 \ud575\uc740 \uc608\uc0c1\ubcf4\ub2e4 \uba3c\uc9c0\uac00 \ub9ce\uace0 \uc5bc\uc74c\uc774 \uc801\ub2e4\ub294 \uac83\uc744 \uc54c\uc544\ub0c8\ub294\ub370, \uadf8 \uc774\uc720\ub294 \ucda9\ub3cc\uae30\uac00 \ucda9\ub3cc\ud558\uba74\uc11c \uc608\uae30\uce58 \uc54a\uac8c \ud06c\uace0 \ubc1d\uc740 \uba3c\uc9c0\uad6c\ub984\uc744 \ub9cc\ub4e4\uc5b4, \uc2dc\uc57c\ub97c \uac00\ub838\uae30 \ub54c\ubb38\uc774\ub2e4.", "sentence_answer": "\ub525 \uc784\ud329\ud2b8\uc758 \uc784\ubb34\ub294 \ud61c\uc131 \ud15c\ud3a0 1(9P/\ud15c\ud3a0)\uc758 \ub0b4\ubd80 \uc870\uc131\uc744 \uc5f0\uad6c\ud558\uae30 \uc704\ud574 \ucda9\ub3cc\uae30\ub97c \ud61c\uc131\uc73c\ub85c \ub099\ud558\uc2dc\ud0a4\ub294 \uac83\uc774\uc5c8\uace0, 2005\ub144 7\uc6d4 4\uc77c \ud611\uc815 \uc138\uacc4\uc2dc 05:52\uc5d0 \ucda9\ub3cc\uae30\ub294 \uc131\uacf5\uc801\uc73c\ub85c \ud61c\uc131\uc758 \ud575\uc5d0 \ucda9\ub3cc\ud588\ub2e4.", "paragraph_id": "6049885-0-1", "paragraph_question": "question: \ub525 \uc784\ud329\ud2b8\ub294 \uc5b4\ub5a4 \ucc9c\uccb4\uc758 \ub0b4\ubd80 \uc870\uc131\uc744 \uc5f0\uad6c\ud558\uae30 \uc704\ud55c \uc6b0\uc8fc\uc120\uc774\uc5c8\ub294\ub370 \uc774 \ucc9c\uccb4\ub294?, context: \ub525 \uc784\ud329\ud2b8(\uc601\uc5b4: Deep Impact)\ub294 \ucf00\uc774\ud504\ucee4\ub0b4\ubc84\ub7f4 \uacf5\uad70 \uae30\uc9c0\uc5d0\uc11c 2005\ub144 1\uc6d4 12\uc77c 18\uc2dc 47\ubd84(UTC)\uc5d0 \ubc1c\uc0ac\ub41c \ubbf8\uad6d \ud56d\uacf5\uc6b0\uc8fc\uad6d\uc758 \uc6b0\uc8fc \ud0d0\uc0ac\uc120\uc774\ub2e4. \ub525 \uc784\ud329\ud2b8\uc758 \uc784\ubb34\ub294 \ud61c\uc131 \ud15c\ud3a0 1(9P/\ud15c\ud3a0)\uc758 \ub0b4\ubd80 \uc870\uc131\uc744 \uc5f0\uad6c\ud558\uae30 \uc704\ud574 \ucda9\ub3cc\uae30\ub97c \ud61c\uc131\uc73c\ub85c \ub099\ud558\uc2dc\ud0a4\ub294 \uac83\uc774\uc5c8\uace0, 2005\ub144 7\uc6d4 4\uc77c \ud611\uc815 \uc138\uacc4\uc2dc 05:52\uc5d0 \ucda9\ub3cc\uae30\ub294 \uc131\uacf5\uc801\uc73c\ub85c \ud61c\uc131\uc758 \ud575\uc5d0 \ucda9\ub3cc\ud588\ub2e4. \ucda9\ub3cc\uae30\ub294 \ucda9\ub3cc\ud558\uba74\uc11c \ud61c\uc131\uc758 \ud575\uc5d0 \ucda9\ub3cc\uad6c\ub97c \ub9cc\ub4e4\uace0, \ud575\uc758 \uad6c\uc131 \ubb3c\uc9c8\uc744 \ubc16\uc73c\ub85c \ub0a0\ub824 \ubcf4\ub0c8\ub2e4. \uc6b0\uc8fc\uc120\uc5d0\uc11c \ucc0d\uc740, \ubb3c\uc9c8\uc774 \ubd84\ucd9c\ub418\ub294 \uc0ac\uc9c4\uc5d0\uc11c \ud61c\uc131\uc758 \ud575\uc740 \uc608\uc0c1\ubcf4\ub2e4 \uba3c\uc9c0\uac00 \ub9ce\uace0 \uc5bc\uc74c\uc774 \uc801\ub2e4\ub294 \uac83\uc744 \uc54c\uc544\ub0c8\ub294\ub370, \uadf8 \uc774\uc720\ub294 \ucda9\ub3cc\uae30\uac00 \ucda9\ub3cc\ud558\uba74\uc11c \uc608\uae30\uce58 \uc54a\uac8c \ud06c\uace0 \ubc1d\uc740 \uba3c\uc9c0\uad6c\ub984\uc744 \ub9cc\ub4e4\uc5b4, \uc2dc\uc57c\ub97c \uac00\ub838\uae30 \ub54c\ubb38\uc774\ub2e4."} {"question": "\uc544\ub9c8\ucd94\uc5b4 \uc120\uc218\ub97c \uc774\ub978\ubc14 \uadc0\uc871\uc801\uc778 \uc2e0\uc0ac\ub77c \ubd80\ub974\ub294 \ub9d0\uc740 20\uc138\uae30\uc5d0 \uc774\ub974\ub7ec \ubb34\uc5c7\uc774 \ubd95\uad34\ub418\uba70 \ub4a4\uccd0\uc9c0\ub294 \ub9d0\uc774 \ub418\uc5c8\ub294\uac00?", "paragraph": "20\uc138\uae30\uc5d0 \uc774\ub974\ub7ec\uc11c \uacc4\uae09\uad6c\uc870\uac00 \ubd95\uad34\ub418\uba74\uc11c \uc774\ub978\ubc14 \uadc0\uc871\uc801\uc778 \uc2e0\uc0ac\ub77c\ub294 \uc544\ub9c8\ucd94\uc5b4 \uc120\uc218\uc5d0 \ub300\ud55c \uc815\uc758\ub294 \uc2dc\ub300\uc5d0 \ub4a4\ucc98\uc9c0\ub294 \ub9d0\uc774 \ub418\uac8c \ub41c\ub2e4. \uc77c\ubd80 \uad6d\uac00\ub4e4\uc740 '\uc815\uc2dd \uc544\ub9c8\ucd94\uc5b4 \uc120\uc218'\ub97c '\ud0a4\uc6cc\uc11c' \uc21c\uc218\ud55c \uc544\ub9c8\ucd94\uc5b4 \uc815\uc2e0\uc744 \ubc97\uc5b4\ub098\uace0 \uc788\uc5c8\uace0, \uc790\uc2e0\uc774 \ub0b4\ub294 \ube44\uc6a9\uc73c\ub85c \uc5f0\uc2b5\ud558\ub294 \uc120\uc218\ub4e4\uc758 \ubd88\ub9ac\ud568\uc5d0 \ub300\ud55c \ubaa9\uc18c\ub9ac\uac00 \ub098\uc624\uae30 \uc2dc\uc791\ud588\ub2e4. \ud558\uc9c0\ub9cc IOC\ub294 \uc544\ub9c8\ucd94\uc5b4 \uc815\uc2e0\uc5d0 \uad00\ud55c \uc785\uc7a5\uc744 \uace0\uc218\ud588\ub2e4. 1970\ub144\ub300 \ucd08\uc5d0\ub294 \uc544\ub9c8\ucd94\uc5b4 \uc815\uc2e0\uc774 \uc62c\ub9bc\ud53d\ud5cc\uc7a5\uc5d0\uc11c \ud3d0\uc9c0\ub418\uc5b4\uc57c \ud55c\ub2e4\ub294 \ub9d0\uc774 \ub098\uc624\uae30 \uc2dc\uc791\ud588\ub2e4. \uacb0\uad6d \ud504\ub85c\uc120\uc218\ub4e4\uc758 \ucd9c\uc804\uc740 \uad6d\uc81c\uacbd\uae30\uc5f0\ub9f9(IF)\uc5d0\uc11c \uacb0\uc815\uc9d3\ub3c4\ub85d \ub418\uc5c8\ub2e4. 2008\ub144 \ud604\uc7ac \uc544\ub9c8\ucd94\uc5b4 \uc120\uc218\ub9cc \ucd9c\uc804\ud558\uace0 \uc788\ub294 \uc62c\ub9bc\ud53d \uc885\ubaa9\uc740 \ubcf5\uc2f1\uc774 \uc720\uc77c\ud558\uba70 \ub0a8\uc790 \ucd95\uad6c\uc5d0\uc11c\ub294 \ub098\uc774\uac00 23\uc138 \uc774\uc0c1\uc778 \uc120\uc218\ub97c 3 \uba85\uae4c\uc9c0\ub9cc \uc120\ubc1c\ud560 \uc218 \uc788\ub2e4. \uc774\ub294 \uc544\ub9c8\ucd94\uc5b4 \uc815\uc2e0\uc744 \uc9c0\ud0a4\uae30 \uc704\ud55c \uc77c\ud658\uc73c\ub85c \ubcfc \uc218 \uc788\ub2e4.", "answer": "\uacc4\uae09\uad6c\uc870", "sentence": "20\uc138\uae30\uc5d0 \uc774\ub974\ub7ec\uc11c \uacc4\uae09\uad6c\uc870 \uac00 \ubd95\uad34\ub418\uba74\uc11c \uc774\ub978\ubc14 \uadc0\uc871\uc801\uc778 \uc2e0\uc0ac\ub77c\ub294 \uc544\ub9c8\ucd94\uc5b4 \uc120\uc218\uc5d0 \ub300\ud55c \uc815\uc758\ub294 \uc2dc\ub300\uc5d0 \ub4a4\ucc98\uc9c0\ub294 \ub9d0\uc774 \ub418\uac8c \ub41c\ub2e4.", "paragraph_sentence": " 20\uc138\uae30\uc5d0 \uc774\ub974\ub7ec\uc11c \uacc4\uae09\uad6c\uc870 \uac00 \ubd95\uad34\ub418\uba74\uc11c \uc774\ub978\ubc14 \uadc0\uc871\uc801\uc778 \uc2e0\uc0ac\ub77c\ub294 \uc544\ub9c8\ucd94\uc5b4 \uc120\uc218\uc5d0 \ub300\ud55c \uc815\uc758\ub294 \uc2dc\ub300\uc5d0 \ub4a4\ucc98\uc9c0\ub294 \ub9d0\uc774 \ub418\uac8c \ub41c\ub2e4. \uc77c\ubd80 \uad6d\uac00\ub4e4\uc740 '\uc815\uc2dd \uc544\ub9c8\ucd94\uc5b4 \uc120\uc218'\ub97c '\ud0a4\uc6cc\uc11c' \uc21c\uc218\ud55c \uc544\ub9c8\ucd94\uc5b4 \uc815\uc2e0\uc744 \ubc97\uc5b4\ub098\uace0 \uc788\uc5c8\uace0, \uc790\uc2e0\uc774 \ub0b4\ub294 \ube44\uc6a9\uc73c\ub85c \uc5f0\uc2b5\ud558\ub294 \uc120\uc218\ub4e4\uc758 \ubd88\ub9ac\ud568\uc5d0 \ub300\ud55c \ubaa9\uc18c\ub9ac\uac00 \ub098\uc624\uae30 \uc2dc\uc791\ud588\ub2e4. \ud558\uc9c0\ub9cc IOC\ub294 \uc544\ub9c8\ucd94\uc5b4 \uc815\uc2e0\uc5d0 \uad00\ud55c \uc785\uc7a5\uc744 \uace0\uc218\ud588\ub2e4. 1970\ub144\ub300 \ucd08\uc5d0\ub294 \uc544\ub9c8\ucd94\uc5b4 \uc815\uc2e0\uc774 \uc62c\ub9bc\ud53d\ud5cc\uc7a5\uc5d0\uc11c \ud3d0\uc9c0\ub418\uc5b4\uc57c \ud55c\ub2e4\ub294 \ub9d0\uc774 \ub098\uc624\uae30 \uc2dc\uc791\ud588\ub2e4. \uacb0\uad6d \ud504\ub85c\uc120\uc218\ub4e4\uc758 \ucd9c\uc804\uc740 \uad6d\uc81c\uacbd\uae30\uc5f0\ub9f9(IF)\uc5d0\uc11c \uacb0\uc815\uc9d3\ub3c4\ub85d \ub418\uc5c8\ub2e4. 2008\ub144 \ud604\uc7ac \uc544\ub9c8\ucd94\uc5b4 \uc120\uc218\ub9cc \ucd9c\uc804\ud558\uace0 \uc788\ub294 \uc62c\ub9bc\ud53d \uc885\ubaa9\uc740 \ubcf5\uc2f1\uc774 \uc720\uc77c\ud558\uba70 \ub0a8\uc790 \ucd95\uad6c\uc5d0\uc11c\ub294 \ub098\uc774\uac00 23\uc138 \uc774\uc0c1\uc778 \uc120\uc218\ub97c 3 \uba85\uae4c\uc9c0\ub9cc \uc120\ubc1c\ud560 \uc218 \uc788\ub2e4. \uc774\ub294 \uc544\ub9c8\ucd94\uc5b4 \uc815\uc2e0\uc744 \uc9c0\ud0a4\uae30 \uc704\ud55c \uc77c\ud658\uc73c\ub85c \ubcfc \uc218 \uc788\ub2e4.", "paragraph_answer": "20\uc138\uae30\uc5d0 \uc774\ub974\ub7ec\uc11c \uacc4\uae09\uad6c\uc870 \uac00 \ubd95\uad34\ub418\uba74\uc11c \uc774\ub978\ubc14 \uadc0\uc871\uc801\uc778 \uc2e0\uc0ac\ub77c\ub294 \uc544\ub9c8\ucd94\uc5b4 \uc120\uc218\uc5d0 \ub300\ud55c \uc815\uc758\ub294 \uc2dc\ub300\uc5d0 \ub4a4\ucc98\uc9c0\ub294 \ub9d0\uc774 \ub418\uac8c \ub41c\ub2e4. \uc77c\ubd80 \uad6d\uac00\ub4e4\uc740 '\uc815\uc2dd \uc544\ub9c8\ucd94\uc5b4 \uc120\uc218'\ub97c '\ud0a4\uc6cc\uc11c' \uc21c\uc218\ud55c \uc544\ub9c8\ucd94\uc5b4 \uc815\uc2e0\uc744 \ubc97\uc5b4\ub098\uace0 \uc788\uc5c8\uace0, \uc790\uc2e0\uc774 \ub0b4\ub294 \ube44\uc6a9\uc73c\ub85c \uc5f0\uc2b5\ud558\ub294 \uc120\uc218\ub4e4\uc758 \ubd88\ub9ac\ud568\uc5d0 \ub300\ud55c \ubaa9\uc18c\ub9ac\uac00 \ub098\uc624\uae30 \uc2dc\uc791\ud588\ub2e4. \ud558\uc9c0\ub9cc IOC\ub294 \uc544\ub9c8\ucd94\uc5b4 \uc815\uc2e0\uc5d0 \uad00\ud55c \uc785\uc7a5\uc744 \uace0\uc218\ud588\ub2e4. 1970\ub144\ub300 \ucd08\uc5d0\ub294 \uc544\ub9c8\ucd94\uc5b4 \uc815\uc2e0\uc774 \uc62c\ub9bc\ud53d\ud5cc\uc7a5\uc5d0\uc11c \ud3d0\uc9c0\ub418\uc5b4\uc57c \ud55c\ub2e4\ub294 \ub9d0\uc774 \ub098\uc624\uae30 \uc2dc\uc791\ud588\ub2e4. \uacb0\uad6d \ud504\ub85c\uc120\uc218\ub4e4\uc758 \ucd9c\uc804\uc740 \uad6d\uc81c\uacbd\uae30\uc5f0\ub9f9(IF)\uc5d0\uc11c \uacb0\uc815\uc9d3\ub3c4\ub85d \ub418\uc5c8\ub2e4. 2008\ub144 \ud604\uc7ac \uc544\ub9c8\ucd94\uc5b4 \uc120\uc218\ub9cc \ucd9c\uc804\ud558\uace0 \uc788\ub294 \uc62c\ub9bc\ud53d \uc885\ubaa9\uc740 \ubcf5\uc2f1\uc774 \uc720\uc77c\ud558\uba70 \ub0a8\uc790 \ucd95\uad6c\uc5d0\uc11c\ub294 \ub098\uc774\uac00 23\uc138 \uc774\uc0c1\uc778 \uc120\uc218\ub97c 3 \uba85\uae4c\uc9c0\ub9cc \uc120\ubc1c\ud560 \uc218 \uc788\ub2e4. \uc774\ub294 \uc544\ub9c8\ucd94\uc5b4 \uc815\uc2e0\uc744 \uc9c0\ud0a4\uae30 \uc704\ud55c \uc77c\ud658\uc73c\ub85c \ubcfc \uc218 \uc788\ub2e4.", "sentence_answer": "20\uc138\uae30\uc5d0 \uc774\ub974\ub7ec\uc11c \uacc4\uae09\uad6c\uc870 \uac00 \ubd95\uad34\ub418\uba74\uc11c \uc774\ub978\ubc14 \uadc0\uc871\uc801\uc778 \uc2e0\uc0ac\ub77c\ub294 \uc544\ub9c8\ucd94\uc5b4 \uc120\uc218\uc5d0 \ub300\ud55c \uc815\uc758\ub294 \uc2dc\ub300\uc5d0 \ub4a4\ucc98\uc9c0\ub294 \ub9d0\uc774 \ub418\uac8c \ub41c\ub2e4.", "paragraph_id": "6525547-23-0", "paragraph_question": "question: \uc544\ub9c8\ucd94\uc5b4 \uc120\uc218\ub97c \uc774\ub978\ubc14 \uadc0\uc871\uc801\uc778 \uc2e0\uc0ac\ub77c \ubd80\ub974\ub294 \ub9d0\uc740 20\uc138\uae30\uc5d0 \uc774\ub974\ub7ec \ubb34\uc5c7\uc774 \ubd95\uad34\ub418\uba70 \ub4a4\uccd0\uc9c0\ub294 \ub9d0\uc774 \ub418\uc5c8\ub294\uac00?, context: 20\uc138\uae30\uc5d0 \uc774\ub974\ub7ec\uc11c \uacc4\uae09\uad6c\uc870\uac00 \ubd95\uad34\ub418\uba74\uc11c \uc774\ub978\ubc14 \uadc0\uc871\uc801\uc778 \uc2e0\uc0ac\ub77c\ub294 \uc544\ub9c8\ucd94\uc5b4 \uc120\uc218\uc5d0 \ub300\ud55c \uc815\uc758\ub294 \uc2dc\ub300\uc5d0 \ub4a4\ucc98\uc9c0\ub294 \ub9d0\uc774 \ub418\uac8c \ub41c\ub2e4. \uc77c\ubd80 \uad6d\uac00\ub4e4\uc740 '\uc815\uc2dd \uc544\ub9c8\ucd94\uc5b4 \uc120\uc218'\ub97c '\ud0a4\uc6cc\uc11c' \uc21c\uc218\ud55c \uc544\ub9c8\ucd94\uc5b4 \uc815\uc2e0\uc744 \ubc97\uc5b4\ub098\uace0 \uc788\uc5c8\uace0, \uc790\uc2e0\uc774 \ub0b4\ub294 \ube44\uc6a9\uc73c\ub85c \uc5f0\uc2b5\ud558\ub294 \uc120\uc218\ub4e4\uc758 \ubd88\ub9ac\ud568\uc5d0 \ub300\ud55c \ubaa9\uc18c\ub9ac\uac00 \ub098\uc624\uae30 \uc2dc\uc791\ud588\ub2e4. \ud558\uc9c0\ub9cc IOC\ub294 \uc544\ub9c8\ucd94\uc5b4 \uc815\uc2e0\uc5d0 \uad00\ud55c \uc785\uc7a5\uc744 \uace0\uc218\ud588\ub2e4. 1970\ub144\ub300 \ucd08\uc5d0\ub294 \uc544\ub9c8\ucd94\uc5b4 \uc815\uc2e0\uc774 \uc62c\ub9bc\ud53d\ud5cc\uc7a5\uc5d0\uc11c \ud3d0\uc9c0\ub418\uc5b4\uc57c \ud55c\ub2e4\ub294 \ub9d0\uc774 \ub098\uc624\uae30 \uc2dc\uc791\ud588\ub2e4. \uacb0\uad6d \ud504\ub85c\uc120\uc218\ub4e4\uc758 \ucd9c\uc804\uc740 \uad6d\uc81c\uacbd\uae30\uc5f0\ub9f9(IF)\uc5d0\uc11c \uacb0\uc815\uc9d3\ub3c4\ub85d \ub418\uc5c8\ub2e4. 2008\ub144 \ud604\uc7ac \uc544\ub9c8\ucd94\uc5b4 \uc120\uc218\ub9cc \ucd9c\uc804\ud558\uace0 \uc788\ub294 \uc62c\ub9bc\ud53d \uc885\ubaa9\uc740 \ubcf5\uc2f1\uc774 \uc720\uc77c\ud558\uba70 \ub0a8\uc790 \ucd95\uad6c\uc5d0\uc11c\ub294 \ub098\uc774\uac00 23\uc138 \uc774\uc0c1\uc778 \uc120\uc218\ub97c 3 \uba85\uae4c\uc9c0\ub9cc \uc120\ubc1c\ud560 \uc218 \uc788\ub2e4. \uc774\ub294 \uc544\ub9c8\ucd94\uc5b4 \uc815\uc2e0\uc744 \uc9c0\ud0a4\uae30 \uc704\ud55c \uc77c\ud658\uc73c\ub85c \ubcfc \uc218 \uc788\ub2e4."} {"question": "\ud3ec\ud658 \ub358\uc9c0\uae30\uc5d0\uc11c \ud3ec\ud658\uc744 \ub358\uc9c0\ub294 \uc720\ud6a8 \ubc94\uc704\uc758 \uac01\ub3c4\ub294?", "paragraph": "\ud604\ub300 \uc2a4\ud3ec\uce20\ub85c\uc11c \ud3ec\ud658\ub358\uc9c0\uae30 \uc6b4\ub3d9\uc740 1865\ub144 \uc601\uad6d\uc5d0\uc11c \uc2e4\uc2dc\ub41c \ud6c4 \uc138\uacc4 \uac01\uad6d\uc5d0 \ub110\ub9ac \ubcf4\uae09\ub418\uc5c8\ub2e4. \uc9c0\ub984 2.135m\uc758 \uc11c\ud074 \uc548\uc5d0\uc11c \ube59\uadf8\ub974 \ub3cc\ub2e4\uac00 \ud3ec\ud658\uc744 \ub358\uc9c0\ub294\ub370, \ubc1c\ub9c9\uc774\ub294 \uc11c\ud074\uc758 \uc55e\ucabd\uc5d0 \uace0\uc815\ud558\uba70, \ub358\uc9c0\ub294 \uc720\ud6a8 \ubc94\uc704\uc758 \uac01\ub3c4\ub294 \uc57d 45 \uc774\ub2e4. \ud3ec\ud658\uc758 \ubb34\uac8c\ub294 \ub0a8\uc790\uc6a9 7.260 kg \uc774\uc0c1, \uc5ec\uc790\uc6a9 4 kg \uc774\uc0c1, \uc911\ud559\ubd80 \ub0a8\uc790\uc6a9 4 kg, \uc911\ud559\ubd80 \uc5ec\uc790\uc6a9 2.721kg\uc758 \uac83\uc744 \uc0ac\uc6a9\ud55c\ub2e4. \uc9c0\ub984\uc740 \ub0a8\uc790\uc6a9 110-130mm, \uc5ec\uc790\uc6a9 95-110mm\uc774\ub2e4. \ud3ec\ud658\uc740 \uc5b4\uae68\uc5d0\uc11c \ud55c\ucabd \uc190\ub9cc\uc73c\ub85c \ub358\uc838\uc57c \ud558\uba70, \ub358\uc9c4 \ub2e4\uc74c\uc5d0 \ud30c\uc6b8\uc744 \ub9c9\uae30 \uc704\ud574 \ubc1c\ub9c9\uc774\uc758 \uc548\ucabd\uc744 \uc190\uc73c\ub85c \ubc1b\uccd0\ub3c4 \ub41c\ub2e4. \ud3ec\ud658\ub358\uc9c0\uae30\ub294 \uc628\ubab8\uc758 \ud798\uc744 \ud3ec\ud658\uc5d0 \uc62e\uaca8 \ubc00\uc5b4\ub0b4\uc57c \ud558\ubbc0\ub85c \ud5c8\ub9ac\u00b7\uc5b4\uae68\u00b7\ud314\u00b7\uc190\ubaa9\u00b7\ub2e4\ub9ac\uc758 \ud798\uc744 \uc21c\uac04\uc801\uc73c\ub85c \ud55c \uacf3\uc5d0 \ubaa8\uc544 \ud3ec\ud658\uc5d0 \uc62e\uae30\ub294 \uae30\uc220\uc774 \uc694\uad6c\ub41c\ub2e4.", "answer": "45", "sentence": "\uc57d 45 \uc774\ub2e4.", "paragraph_sentence": "\ud604\ub300 \uc2a4\ud3ec\uce20\ub85c\uc11c \ud3ec\ud658\ub358\uc9c0\uae30 \uc6b4\ub3d9\uc740 1865\ub144 \uc601\uad6d\uc5d0\uc11c \uc2e4\uc2dc\ub41c \ud6c4 \uc138\uacc4 \uac01\uad6d\uc5d0 \ub110\ub9ac \ubcf4\uae09\ub418\uc5c8\ub2e4. \uc9c0\ub984 2.135m\uc758 \uc11c\ud074 \uc548\uc5d0\uc11c \ube59\uadf8\ub974 \ub3cc\ub2e4\uac00 \ud3ec\ud658\uc744 \ub358\uc9c0\ub294\ub370, \ubc1c\ub9c9\uc774\ub294 \uc11c\ud074\uc758 \uc55e\ucabd\uc5d0 \uace0\uc815\ud558\uba70, \ub358\uc9c0\ub294 \uc720\ud6a8 \ubc94\uc704\uc758 \uac01\ub3c4\ub294 \uc57d 45 \uc774\ub2e4. \ud3ec\ud658\uc758 \ubb34\uac8c\ub294 \ub0a8\uc790\uc6a9 7.260 kg \uc774\uc0c1, \uc5ec\uc790\uc6a9 4 kg \uc774\uc0c1, \uc911\ud559\ubd80 \ub0a8\uc790\uc6a9 4 kg, \uc911\ud559\ubd80 \uc5ec\uc790\uc6a9 2.721kg\uc758 \uac83\uc744 \uc0ac\uc6a9\ud55c\ub2e4. \uc9c0\ub984\uc740 \ub0a8\uc790\uc6a9 110-130mm, \uc5ec\uc790\uc6a9 95-110mm\uc774\ub2e4. \ud3ec\ud658\uc740 \uc5b4\uae68\uc5d0\uc11c \ud55c\ucabd \uc190\ub9cc\uc73c\ub85c \ub358\uc838\uc57c \ud558\uba70, \ub358\uc9c4 \ub2e4\uc74c\uc5d0 \ud30c\uc6b8\uc744 \ub9c9\uae30 \uc704\ud574 \ubc1c\ub9c9\uc774\uc758 \uc548\ucabd\uc744 \uc190\uc73c\ub85c \ubc1b\uccd0\ub3c4 \ub41c\ub2e4. \ud3ec\ud658\ub358\uc9c0\uae30\ub294 \uc628\ubab8\uc758 \ud798\uc744 \ud3ec\ud658\uc5d0 \uc62e\uaca8 \ubc00\uc5b4\ub0b4\uc57c \ud558\ubbc0\ub85c \ud5c8\ub9ac\u00b7\uc5b4\uae68\u00b7\ud314\u00b7\uc190\ubaa9\u00b7\ub2e4\ub9ac\uc758 \ud798\uc744 \uc21c\uac04\uc801\uc73c\ub85c \ud55c \uacf3\uc5d0 \ubaa8\uc544 \ud3ec\ud658\uc5d0 \uc62e\uae30\ub294 \uae30\uc220\uc774 \uc694\uad6c\ub41c\ub2e4.", "paragraph_answer": "\ud604\ub300 \uc2a4\ud3ec\uce20\ub85c\uc11c \ud3ec\ud658\ub358\uc9c0\uae30 \uc6b4\ub3d9\uc740 1865\ub144 \uc601\uad6d\uc5d0\uc11c \uc2e4\uc2dc\ub41c \ud6c4 \uc138\uacc4 \uac01\uad6d\uc5d0 \ub110\ub9ac \ubcf4\uae09\ub418\uc5c8\ub2e4. \uc9c0\ub984 2.135m\uc758 \uc11c\ud074 \uc548\uc5d0\uc11c \ube59\uadf8\ub974 \ub3cc\ub2e4\uac00 \ud3ec\ud658\uc744 \ub358\uc9c0\ub294\ub370, \ubc1c\ub9c9\uc774\ub294 \uc11c\ud074\uc758 \uc55e\ucabd\uc5d0 \uace0\uc815\ud558\uba70, \ub358\uc9c0\ub294 \uc720\ud6a8 \ubc94\uc704\uc758 \uac01\ub3c4\ub294 \uc57d 45 \uc774\ub2e4. \ud3ec\ud658\uc758 \ubb34\uac8c\ub294 \ub0a8\uc790\uc6a9 7.260 kg \uc774\uc0c1, \uc5ec\uc790\uc6a9 4 kg \uc774\uc0c1, \uc911\ud559\ubd80 \ub0a8\uc790\uc6a9 4 kg, \uc911\ud559\ubd80 \uc5ec\uc790\uc6a9 2.721kg\uc758 \uac83\uc744 \uc0ac\uc6a9\ud55c\ub2e4. \uc9c0\ub984\uc740 \ub0a8\uc790\uc6a9 110-130mm, \uc5ec\uc790\uc6a9 95-110mm\uc774\ub2e4. \ud3ec\ud658\uc740 \uc5b4\uae68\uc5d0\uc11c \ud55c\ucabd \uc190\ub9cc\uc73c\ub85c \ub358\uc838\uc57c \ud558\uba70, \ub358\uc9c4 \ub2e4\uc74c\uc5d0 \ud30c\uc6b8\uc744 \ub9c9\uae30 \uc704\ud574 \ubc1c\ub9c9\uc774\uc758 \uc548\ucabd\uc744 \uc190\uc73c\ub85c \ubc1b\uccd0\ub3c4 \ub41c\ub2e4. \ud3ec\ud658\ub358\uc9c0\uae30\ub294 \uc628\ubab8\uc758 \ud798\uc744 \ud3ec\ud658\uc5d0 \uc62e\uaca8 \ubc00\uc5b4\ub0b4\uc57c \ud558\ubbc0\ub85c \ud5c8\ub9ac\u00b7\uc5b4\uae68\u00b7\ud314\u00b7\uc190\ubaa9\u00b7\ub2e4\ub9ac\uc758 \ud798\uc744 \uc21c\uac04\uc801\uc73c\ub85c \ud55c \uacf3\uc5d0 \ubaa8\uc544 \ud3ec\ud658\uc5d0 \uc62e\uae30\ub294 \uae30\uc220\uc774 \uc694\uad6c\ub41c\ub2e4.", "sentence_answer": "\uc57d 45 \uc774\ub2e4.", "paragraph_id": "6543641-4-1", "paragraph_question": "question: \ud3ec\ud658 \ub358\uc9c0\uae30\uc5d0\uc11c \ud3ec\ud658\uc744 \ub358\uc9c0\ub294 \uc720\ud6a8 \ubc94\uc704\uc758 \uac01\ub3c4\ub294?, context: \ud604\ub300 \uc2a4\ud3ec\uce20\ub85c\uc11c \ud3ec\ud658\ub358\uc9c0\uae30 \uc6b4\ub3d9\uc740 1865\ub144 \uc601\uad6d\uc5d0\uc11c \uc2e4\uc2dc\ub41c \ud6c4 \uc138\uacc4 \uac01\uad6d\uc5d0 \ub110\ub9ac \ubcf4\uae09\ub418\uc5c8\ub2e4. \uc9c0\ub984 2.135m\uc758 \uc11c\ud074 \uc548\uc5d0\uc11c \ube59\uadf8\ub974 \ub3cc\ub2e4\uac00 \ud3ec\ud658\uc744 \ub358\uc9c0\ub294\ub370, \ubc1c\ub9c9\uc774\ub294 \uc11c\ud074\uc758 \uc55e\ucabd\uc5d0 \uace0\uc815\ud558\uba70, \ub358\uc9c0\ub294 \uc720\ud6a8 \ubc94\uc704\uc758 \uac01\ub3c4\ub294 \uc57d 45 \uc774\ub2e4. \ud3ec\ud658\uc758 \ubb34\uac8c\ub294 \ub0a8\uc790\uc6a9 7.260 kg \uc774\uc0c1, \uc5ec\uc790\uc6a9 4 kg \uc774\uc0c1, \uc911\ud559\ubd80 \ub0a8\uc790\uc6a9 4 kg, \uc911\ud559\ubd80 \uc5ec\uc790\uc6a9 2.721kg\uc758 \uac83\uc744 \uc0ac\uc6a9\ud55c\ub2e4. \uc9c0\ub984\uc740 \ub0a8\uc790\uc6a9 110-130mm, \uc5ec\uc790\uc6a9 95-110mm\uc774\ub2e4. \ud3ec\ud658\uc740 \uc5b4\uae68\uc5d0\uc11c \ud55c\ucabd \uc190\ub9cc\uc73c\ub85c \ub358\uc838\uc57c \ud558\uba70, \ub358\uc9c4 \ub2e4\uc74c\uc5d0 \ud30c\uc6b8\uc744 \ub9c9\uae30 \uc704\ud574 \ubc1c\ub9c9\uc774\uc758 \uc548\ucabd\uc744 \uc190\uc73c\ub85c \ubc1b\uccd0\ub3c4 \ub41c\ub2e4. \ud3ec\ud658\ub358\uc9c0\uae30\ub294 \uc628\ubab8\uc758 \ud798\uc744 \ud3ec\ud658\uc5d0 \uc62e\uaca8 \ubc00\uc5b4\ub0b4\uc57c \ud558\ubbc0\ub85c \ud5c8\ub9ac\u00b7\uc5b4\uae68\u00b7\ud314\u00b7\uc190\ubaa9\u00b7\ub2e4\ub9ac\uc758 \ud798\uc744 \uc21c\uac04\uc801\uc73c\ub85c \ud55c \uacf3\uc5d0 \ubaa8\uc544 \ud3ec\ud658\uc5d0 \uc62e\uae30\ub294 \uae30\uc220\uc774 \uc694\uad6c\ub41c\ub2e4."} {"question": "\ubc15\uadfc\ud76c\ub294 \ubc15\uc815\ud76c\uc758 \uba87 \uc9f8 \ub538\uc778\uac00?", "paragraph": "\ubc15\uc815\ud76c\uc758 \uc14b\uc9f8 \ub538 \ubc15\uadfc\ub839\uc740 2005\ub144 2\uc6d4, \uc774\ub7ec\ud55c \uc8fc\uc7a5\uc744 \ub2f4\uace0 \uc788\ub294 \uc11c\uc801\uc774 \ubd80\uce5c\uc758 \uba85\uc608\ub97c \ud6fc\uc190\ud588\ub2e4\uba70 \uad6d\ub0b4 \ucd9c\ud310\uc0ac \ub300\ud45c\uc778 \uc544\uc774\ud544\ub4dc \ucd9c\ud310\uc0ac \ub300\ud45c \uc720\uc5f0\uc2dd\uc744 \uac80\ucc30\uc5d0 \uace0\uc18c\ud588\uace0 \ub300\ubc95\uc6d0 3\ubd80(\uc8fc\uc2ec \uc548\ub300\ud76c \ub300\ubc95\uad00)\uae4c\uc9c0 \uc62c\ub77c\uac14\uc73c\uba70 1939\ub144, \ubc15\uc815\ud76c\uac00 \uc11c\uba85\ud55c \ubb38\uacbd\uacf5\ub9bd\ubcf4\ud1b5\uc18c\ud559\uad50 \u201c\uc131\uc801\ud1b5\uc9c0\ud45c\u201d\uc640 1940\ub144, \ubc15\uc815\ud76c\uac00 \uad50\uc9c1\uc744 \uc758\uc6d0\uba74\uc9c1\ud588\uc74c\uc744 \ubcf4\uc5ec\uc8fc\ub294 \uad50\uc721 \ub2f9\uad6d\uc758 \uc11c\ub958\ub97c \uc81c\ucd9c\ud558\uc600\ub294\ub370 \uc774\uc640 \uad00\ub828\ub41c \uc7ac\ud310\uc5d0\uc11c \uc548\ub300\ud76c \uc7ac\ud310\uad00 \ub4f1 \uc7ac\ud310\ubd80\ub294 \u201c\uadf8\uc758 \uce5c\uc77c \ud589\uc801 \uc5ec\ubd80\uc5d0 \ub300\ud574 \ub17c\ub780\uc774 \uc788\uace0 \ud2b9\uc124\ubd80\ub300\uc5d0 \uadfc\ubb34\ud588\ub294\uc9c0\ub3c4 \ud55c\uad6d \ud604\ub300\uc0ac\uc758 \uc7c1\uc810\uc73c\ub85c \uacc4\uc18d \uc5f0\uad6c\ub3fc\uc57c \ud55c\ub2e4. \ucc45\uc5d0 \uc801\uc2dc\ub41c \ub0b4\uc6a9\uc774 \uc77c\ubc18\uc801\uc73c\ub85c \uc54c\ub824\uc9c4 \uc0ac\uc2e4\uc5d0 \ubc18\ud55c\ub2e4\uace0 \ud558\ub354\ub77c\ub3c4 \ud53c\uace0\uc778\uc774 \u2018\ud5c8\uc704\u2019\uc784\uc744 \uc778\uc2dd\ud588\ub2e4\uace0 \ub2e8\uc815\ud560 \uc218 \uc5c6\uc5b4 \ubb34\uc8c4\ub97c \uc120\uace0\ud55c \uc6d0\uc2ec\uc740 \uc815\ub2f9\ud558\ub2e4\u201d\ub294 \uc774\uc720\ub97c \ub4e4\uc5b4 \ubb34\uc8c4\ub97c \ud310\uacb0\ud588\ub2e4. \uc774\uc640 \uad00\ub828\ud558\uc5ec \ub370\uc77c\ub9ac\uc548\uc740 \ub2e4\ub978 \uc5b8\ub860\ub4e4\uc774 \ubb34\uc8c4\ud310\uacb0\uc744 \uac00\uc9c0\uace0 \ub958\uc5f0\uc0b0\uc758 \uc8fc\uc7a5\uc744 \uc815\ub2f9\ud654\ud574\uc11c\ub294 \uc548\ub418\uba70, \uc774 \ud310\uacb0\uc740 \ubb34\uc8c4\uac00 \uc8c4\uac00 \uc5c6\uc74c\uc744 \uc758\ubbf8\ud558\ub294 \uac83\uc774 \uc544\ub2c8\ub77c \uc720\uc8c4\uc784\uc744 \ud655\uc99d\ud560 \uadfc\uac70\uac00 \uc5c6\ub2e4\ub294 \ub73b\uc774\ub77c\uace0 \uc8fc\uc7a5\ud588\ub2e4. \ub300\ubc95\uc6d0 3\ubd80\uc5d0\uc11c\ub294 \"\uc5ed\uc0ac\uc801\u00b7\uacf5\uc801 \uc778\ubb3c\uc758 \uacbd\uc6b0 \uc2dc\uac04\uc774 \uacbd\uacfc\ud558\uba74 \ub9dd\uc778\uacfc \uc720\uc871\uc758 \uba85\uc608\ubcf4\ub2e4 \uc5ed\uc0ac\uc801 \uc0ac\uc2e4\uc5d0 \ub300\ud55c \ud45c\ud604\uc758 \uc790\uc720\uac00 \ubcf4\ud638\ub3fc\uc57c \ud558\ubbc0\ub85c \uc0ac\uc790 \uba85\uc608\ud6fc\uc190\uc8c4\uac00 \uc131\ub9bd\ud558\ub824\uba74 \ud5c8\uc704 \uc0ac\uc2e4\uc5d0 \ub300\ud55c \uace0\uc758\uc131\uc744 \uc5c4\uaca9\ud788 \ub530\uc838\uc57c \ud55c\ub2e4\"\uba70 \"\ubc15 \uc804 \ub300\ud1b5\ub839\uc758 \ud2b9\uc124\ubd80\ub300 \uadfc\ubb34\uc124\uc740 \uc5ec\ub7ec \ucc45\uc5d0 \uc5b8\uae09\ub410\uace0 \uc800\uc790 \ub958\uc528\ub294 \uc5ed\uc0ac\ud559\uacc4\uc5d0\uc11c\ub3c4 \uc778\uc9c0\ub3c4\uac00 \uc788\uc5b4, \uac80\uc0ac\uac00 \uc81c\ucd9c\ud55c \uc99d\uac70\ub9cc\uc73c\ub85c\ub294 \uc720\uc8c4\uac00 \uc778\uc815\ub418\uc9c0 \uc54a\ub294\ub2e4\"\uba70 \ubb34\uc8c4\ub97c \uc120\uace0\ud588\ub2e4.", "answer": "\uc14b\uc9f8 \ub538", "sentence": "\ubc15\uc815\ud76c\uc758 \uc14b\uc9f8 \ub538 \ubc15\uadfc\ub839\uc740 2005\ub144 2\uc6d4, \uc774\ub7ec\ud55c \uc8fc\uc7a5\uc744 \ub2f4\uace0 \uc788\ub294 \uc11c\uc801\uc774 \ubd80\uce5c\uc758 \uba85\uc608\ub97c \ud6fc\uc190\ud588\ub2e4\uba70 \uad6d\ub0b4 \ucd9c\ud310\uc0ac \ub300\ud45c\uc778 \uc544\uc774\ud544\ub4dc \ucd9c\ud310\uc0ac \ub300\ud45c \uc720\uc5f0\uc2dd\uc744 \uac80\ucc30\uc5d0 \uace0\uc18c\ud588\uace0 \ub300\ubc95\uc6d0 3\ubd80(\uc8fc\uc2ec \uc548\ub300\ud76c \ub300\ubc95\uad00)\uae4c\uc9c0 \uc62c\ub77c\uac14\uc73c\uba70 1939\ub144, \ubc15\uc815\ud76c\uac00 \uc11c\uba85\ud55c \ubb38\uacbd\uacf5\ub9bd\ubcf4\ud1b5\uc18c\ud559\uad50 \u201c\uc131\uc801\ud1b5\uc9c0\ud45c\u201d\uc640 1940\ub144, \ubc15\uc815\ud76c\uac00 \uad50\uc9c1\uc744 \uc758\uc6d0\uba74\uc9c1\ud588\uc74c\uc744 \ubcf4\uc5ec\uc8fc\ub294 \uad50\uc721 \ub2f9\uad6d\uc758 \uc11c\ub958\ub97c \uc81c\ucd9c\ud558\uc600\ub294\ub370 \uc774\uc640 \uad00\ub828\ub41c \uc7ac\ud310\uc5d0\uc11c \uc548\ub300\ud76c \uc7ac\ud310\uad00 \ub4f1 \uc7ac\ud310\ubd80\ub294 \u201c\uadf8\uc758 \uce5c\uc77c \ud589\uc801 \uc5ec\ubd80\uc5d0 \ub300\ud574 \ub17c\ub780\uc774 \uc788\uace0 \ud2b9\uc124\ubd80\ub300\uc5d0 \uadfc\ubb34\ud588\ub294\uc9c0\ub3c4 \ud55c\uad6d \ud604\ub300\uc0ac\uc758 \uc7c1\uc810\uc73c\ub85c \uacc4\uc18d \uc5f0\uad6c\ub3fc\uc57c \ud55c\ub2e4.", "paragraph_sentence": " \ubc15\uc815\ud76c\uc758 \uc14b\uc9f8 \ub538 \ubc15\uadfc\ub839\uc740 2005\ub144 2\uc6d4, \uc774\ub7ec\ud55c \uc8fc\uc7a5\uc744 \ub2f4\uace0 \uc788\ub294 \uc11c\uc801\uc774 \ubd80\uce5c\uc758 \uba85\uc608\ub97c \ud6fc\uc190\ud588\ub2e4\uba70 \uad6d\ub0b4 \ucd9c\ud310\uc0ac \ub300\ud45c\uc778 \uc544\uc774\ud544\ub4dc \ucd9c\ud310\uc0ac \ub300\ud45c \uc720\uc5f0\uc2dd\uc744 \uac80\ucc30\uc5d0 \uace0\uc18c\ud588\uace0 \ub300\ubc95\uc6d0 3\ubd80(\uc8fc\uc2ec \uc548\ub300\ud76c \ub300\ubc95\uad00)\uae4c\uc9c0 \uc62c\ub77c\uac14\uc73c\uba70 1939\ub144, \ubc15\uc815\ud76c\uac00 \uc11c\uba85\ud55c \ubb38\uacbd\uacf5\ub9bd\ubcf4\ud1b5\uc18c\ud559\uad50 \u201c\uc131\uc801\ud1b5\uc9c0\ud45c\u201d\uc640 1940\ub144, \ubc15\uc815\ud76c\uac00 \uad50\uc9c1\uc744 \uc758\uc6d0\uba74\uc9c1\ud588\uc74c\uc744 \ubcf4\uc5ec\uc8fc\ub294 \uad50\uc721 \ub2f9\uad6d\uc758 \uc11c\ub958\ub97c \uc81c\ucd9c\ud558\uc600\ub294\ub370 \uc774\uc640 \uad00\ub828\ub41c \uc7ac\ud310\uc5d0\uc11c \uc548\ub300\ud76c \uc7ac\ud310\uad00 \ub4f1 \uc7ac\ud310\ubd80\ub294 \u201c\uadf8\uc758 \uce5c\uc77c \ud589\uc801 \uc5ec\ubd80\uc5d0 \ub300\ud574 \ub17c\ub780\uc774 \uc788\uace0 \ud2b9\uc124\ubd80\ub300\uc5d0 \uadfc\ubb34\ud588\ub294\uc9c0\ub3c4 \ud55c\uad6d \ud604\ub300\uc0ac\uc758 \uc7c1\uc810\uc73c\ub85c \uacc4\uc18d \uc5f0\uad6c\ub3fc\uc57c \ud55c\ub2e4. \ucc45\uc5d0 \uc801\uc2dc\ub41c \ub0b4\uc6a9\uc774 \uc77c\ubc18\uc801\uc73c\ub85c \uc54c\ub824\uc9c4 \uc0ac\uc2e4\uc5d0 \ubc18\ud55c\ub2e4\uace0 \ud558\ub354\ub77c\ub3c4 \ud53c\uace0\uc778\uc774 \u2018\ud5c8\uc704\u2019\uc784\uc744 \uc778\uc2dd\ud588\ub2e4\uace0 \ub2e8\uc815\ud560 \uc218 \uc5c6\uc5b4 \ubb34\uc8c4\ub97c \uc120\uace0\ud55c \uc6d0\uc2ec\uc740 \uc815\ub2f9\ud558\ub2e4\u201d\ub294 \uc774\uc720\ub97c \ub4e4\uc5b4 \ubb34\uc8c4\ub97c \ud310\uacb0\ud588\ub2e4. \uc774\uc640 \uad00\ub828\ud558\uc5ec \ub370\uc77c\ub9ac\uc548\uc740 \ub2e4\ub978 \uc5b8\ub860\ub4e4\uc774 \ubb34\uc8c4\ud310\uacb0\uc744 \uac00\uc9c0\uace0 \ub958\uc5f0\uc0b0\uc758 \uc8fc\uc7a5\uc744 \uc815\ub2f9\ud654\ud574\uc11c\ub294 \uc548\ub418\uba70, \uc774 \ud310\uacb0\uc740 \ubb34\uc8c4\uac00 \uc8c4\uac00 \uc5c6\uc74c\uc744 \uc758\ubbf8\ud558\ub294 \uac83\uc774 \uc544\ub2c8\ub77c \uc720\uc8c4\uc784\uc744 \ud655\uc99d\ud560 \uadfc\uac70\uac00 \uc5c6\ub2e4\ub294 \ub73b\uc774\ub77c\uace0 \uc8fc\uc7a5\ud588\ub2e4. \ub300\ubc95\uc6d0 3\ubd80\uc5d0\uc11c\ub294 \"\uc5ed\uc0ac\uc801\u00b7\uacf5\uc801 \uc778\ubb3c\uc758 \uacbd\uc6b0 \uc2dc\uac04\uc774 \uacbd\uacfc\ud558\uba74 \ub9dd\uc778\uacfc \uc720\uc871\uc758 \uba85\uc608\ubcf4\ub2e4 \uc5ed\uc0ac\uc801 \uc0ac\uc2e4\uc5d0 \ub300\ud55c \ud45c\ud604\uc758 \uc790\uc720\uac00 \ubcf4\ud638\ub3fc\uc57c \ud558\ubbc0\ub85c \uc0ac\uc790 \uba85\uc608\ud6fc\uc190\uc8c4\uac00 \uc131\ub9bd\ud558\ub824\uba74 \ud5c8\uc704 \uc0ac\uc2e4\uc5d0 \ub300\ud55c \uace0\uc758\uc131\uc744 \uc5c4\uaca9\ud788 \ub530\uc838\uc57c \ud55c\ub2e4\"\uba70 \"\ubc15 \uc804 \ub300\ud1b5\ub839\uc758 \ud2b9\uc124\ubd80\ub300 \uadfc\ubb34\uc124\uc740 \uc5ec\ub7ec \ucc45\uc5d0 \uc5b8\uae09\ub410\uace0 \uc800\uc790 \ub958\uc528\ub294 \uc5ed\uc0ac\ud559\uacc4\uc5d0\uc11c\ub3c4 \uc778\uc9c0\ub3c4\uac00 \uc788\uc5b4, \uac80\uc0ac\uac00 \uc81c\ucd9c\ud55c \uc99d\uac70\ub9cc\uc73c\ub85c\ub294 \uc720\uc8c4\uac00 \uc778\uc815\ub418\uc9c0 \uc54a\ub294\ub2e4\"\uba70 \ubb34\uc8c4\ub97c \uc120\uace0\ud588\ub2e4.", "paragraph_answer": "\ubc15\uc815\ud76c\uc758 \uc14b\uc9f8 \ub538 \ubc15\uadfc\ub839\uc740 2005\ub144 2\uc6d4, \uc774\ub7ec\ud55c \uc8fc\uc7a5\uc744 \ub2f4\uace0 \uc788\ub294 \uc11c\uc801\uc774 \ubd80\uce5c\uc758 \uba85\uc608\ub97c \ud6fc\uc190\ud588\ub2e4\uba70 \uad6d\ub0b4 \ucd9c\ud310\uc0ac \ub300\ud45c\uc778 \uc544\uc774\ud544\ub4dc \ucd9c\ud310\uc0ac \ub300\ud45c \uc720\uc5f0\uc2dd\uc744 \uac80\ucc30\uc5d0 \uace0\uc18c\ud588\uace0 \ub300\ubc95\uc6d0 3\ubd80(\uc8fc\uc2ec \uc548\ub300\ud76c \ub300\ubc95\uad00)\uae4c\uc9c0 \uc62c\ub77c\uac14\uc73c\uba70 1939\ub144, \ubc15\uc815\ud76c\uac00 \uc11c\uba85\ud55c \ubb38\uacbd\uacf5\ub9bd\ubcf4\ud1b5\uc18c\ud559\uad50 \u201c\uc131\uc801\ud1b5\uc9c0\ud45c\u201d\uc640 1940\ub144, \ubc15\uc815\ud76c\uac00 \uad50\uc9c1\uc744 \uc758\uc6d0\uba74\uc9c1\ud588\uc74c\uc744 \ubcf4\uc5ec\uc8fc\ub294 \uad50\uc721 \ub2f9\uad6d\uc758 \uc11c\ub958\ub97c \uc81c\ucd9c\ud558\uc600\ub294\ub370 \uc774\uc640 \uad00\ub828\ub41c \uc7ac\ud310\uc5d0\uc11c \uc548\ub300\ud76c \uc7ac\ud310\uad00 \ub4f1 \uc7ac\ud310\ubd80\ub294 \u201c\uadf8\uc758 \uce5c\uc77c \ud589\uc801 \uc5ec\ubd80\uc5d0 \ub300\ud574 \ub17c\ub780\uc774 \uc788\uace0 \ud2b9\uc124\ubd80\ub300\uc5d0 \uadfc\ubb34\ud588\ub294\uc9c0\ub3c4 \ud55c\uad6d \ud604\ub300\uc0ac\uc758 \uc7c1\uc810\uc73c\ub85c \uacc4\uc18d \uc5f0\uad6c\ub3fc\uc57c \ud55c\ub2e4. \ucc45\uc5d0 \uc801\uc2dc\ub41c \ub0b4\uc6a9\uc774 \uc77c\ubc18\uc801\uc73c\ub85c \uc54c\ub824\uc9c4 \uc0ac\uc2e4\uc5d0 \ubc18\ud55c\ub2e4\uace0 \ud558\ub354\ub77c\ub3c4 \ud53c\uace0\uc778\uc774 \u2018\ud5c8\uc704\u2019\uc784\uc744 \uc778\uc2dd\ud588\ub2e4\uace0 \ub2e8\uc815\ud560 \uc218 \uc5c6\uc5b4 \ubb34\uc8c4\ub97c \uc120\uace0\ud55c \uc6d0\uc2ec\uc740 \uc815\ub2f9\ud558\ub2e4\u201d\ub294 \uc774\uc720\ub97c \ub4e4\uc5b4 \ubb34\uc8c4\ub97c \ud310\uacb0\ud588\ub2e4. \uc774\uc640 \uad00\ub828\ud558\uc5ec \ub370\uc77c\ub9ac\uc548\uc740 \ub2e4\ub978 \uc5b8\ub860\ub4e4\uc774 \ubb34\uc8c4\ud310\uacb0\uc744 \uac00\uc9c0\uace0 \ub958\uc5f0\uc0b0\uc758 \uc8fc\uc7a5\uc744 \uc815\ub2f9\ud654\ud574\uc11c\ub294 \uc548\ub418\uba70, \uc774 \ud310\uacb0\uc740 \ubb34\uc8c4\uac00 \uc8c4\uac00 \uc5c6\uc74c\uc744 \uc758\ubbf8\ud558\ub294 \uac83\uc774 \uc544\ub2c8\ub77c \uc720\uc8c4\uc784\uc744 \ud655\uc99d\ud560 \uadfc\uac70\uac00 \uc5c6\ub2e4\ub294 \ub73b\uc774\ub77c\uace0 \uc8fc\uc7a5\ud588\ub2e4. \ub300\ubc95\uc6d0 3\ubd80\uc5d0\uc11c\ub294 \"\uc5ed\uc0ac\uc801\u00b7\uacf5\uc801 \uc778\ubb3c\uc758 \uacbd\uc6b0 \uc2dc\uac04\uc774 \uacbd\uacfc\ud558\uba74 \ub9dd\uc778\uacfc \uc720\uc871\uc758 \uba85\uc608\ubcf4\ub2e4 \uc5ed\uc0ac\uc801 \uc0ac\uc2e4\uc5d0 \ub300\ud55c \ud45c\ud604\uc758 \uc790\uc720\uac00 \ubcf4\ud638\ub3fc\uc57c \ud558\ubbc0\ub85c \uc0ac\uc790 \uba85\uc608\ud6fc\uc190\uc8c4\uac00 \uc131\ub9bd\ud558\ub824\uba74 \ud5c8\uc704 \uc0ac\uc2e4\uc5d0 \ub300\ud55c \uace0\uc758\uc131\uc744 \uc5c4\uaca9\ud788 \ub530\uc838\uc57c \ud55c\ub2e4\"\uba70 \"\ubc15 \uc804 \ub300\ud1b5\ub839\uc758 \ud2b9\uc124\ubd80\ub300 \uadfc\ubb34\uc124\uc740 \uc5ec\ub7ec \ucc45\uc5d0 \uc5b8\uae09\ub410\uace0 \uc800\uc790 \ub958\uc528\ub294 \uc5ed\uc0ac\ud559\uacc4\uc5d0\uc11c\ub3c4 \uc778\uc9c0\ub3c4\uac00 \uc788\uc5b4, \uac80\uc0ac\uac00 \uc81c\ucd9c\ud55c \uc99d\uac70\ub9cc\uc73c\ub85c\ub294 \uc720\uc8c4\uac00 \uc778\uc815\ub418\uc9c0 \uc54a\ub294\ub2e4\"\uba70 \ubb34\uc8c4\ub97c \uc120\uace0\ud588\ub2e4.", "sentence_answer": "\ubc15\uc815\ud76c\uc758 \uc14b\uc9f8 \ub538 \ubc15\uadfc\ub839\uc740 2005\ub144 2\uc6d4, \uc774\ub7ec\ud55c \uc8fc\uc7a5\uc744 \ub2f4\uace0 \uc788\ub294 \uc11c\uc801\uc774 \ubd80\uce5c\uc758 \uba85\uc608\ub97c \ud6fc\uc190\ud588\ub2e4\uba70 \uad6d\ub0b4 \ucd9c\ud310\uc0ac \ub300\ud45c\uc778 \uc544\uc774\ud544\ub4dc \ucd9c\ud310\uc0ac \ub300\ud45c \uc720\uc5f0\uc2dd\uc744 \uac80\ucc30\uc5d0 \uace0\uc18c\ud588\uace0 \ub300\ubc95\uc6d0 3\ubd80(\uc8fc\uc2ec \uc548\ub300\ud76c \ub300\ubc95\uad00)\uae4c\uc9c0 \uc62c\ub77c\uac14\uc73c\uba70 1939\ub144, \ubc15\uc815\ud76c\uac00 \uc11c\uba85\ud55c \ubb38\uacbd\uacf5\ub9bd\ubcf4\ud1b5\uc18c\ud559\uad50 \u201c\uc131\uc801\ud1b5\uc9c0\ud45c\u201d\uc640 1940\ub144, \ubc15\uc815\ud76c\uac00 \uad50\uc9c1\uc744 \uc758\uc6d0\uba74\uc9c1\ud588\uc74c\uc744 \ubcf4\uc5ec\uc8fc\ub294 \uad50\uc721 \ub2f9\uad6d\uc758 \uc11c\ub958\ub97c \uc81c\ucd9c\ud558\uc600\ub294\ub370 \uc774\uc640 \uad00\ub828\ub41c \uc7ac\ud310\uc5d0\uc11c \uc548\ub300\ud76c \uc7ac\ud310\uad00 \ub4f1 \uc7ac\ud310\ubd80\ub294 \u201c\uadf8\uc758 \uce5c\uc77c \ud589\uc801 \uc5ec\ubd80\uc5d0 \ub300\ud574 \ub17c\ub780\uc774 \uc788\uace0 \ud2b9\uc124\ubd80\ub300\uc5d0 \uadfc\ubb34\ud588\ub294\uc9c0\ub3c4 \ud55c\uad6d \ud604\ub300\uc0ac\uc758 \uc7c1\uc810\uc73c\ub85c \uacc4\uc18d \uc5f0\uad6c\ub3fc\uc57c \ud55c\ub2e4.", "paragraph_id": "6417222-6-0", "paragraph_question": "question: \ubc15\uadfc\ud76c\ub294 \ubc15\uc815\ud76c\uc758 \uba87 \uc9f8 \ub538\uc778\uac00?, context: \ubc15\uc815\ud76c\uc758 \uc14b\uc9f8 \ub538 \ubc15\uadfc\ub839\uc740 2005\ub144 2\uc6d4, \uc774\ub7ec\ud55c \uc8fc\uc7a5\uc744 \ub2f4\uace0 \uc788\ub294 \uc11c\uc801\uc774 \ubd80\uce5c\uc758 \uba85\uc608\ub97c \ud6fc\uc190\ud588\ub2e4\uba70 \uad6d\ub0b4 \ucd9c\ud310\uc0ac \ub300\ud45c\uc778 \uc544\uc774\ud544\ub4dc \ucd9c\ud310\uc0ac \ub300\ud45c \uc720\uc5f0\uc2dd\uc744 \uac80\ucc30\uc5d0 \uace0\uc18c\ud588\uace0 \ub300\ubc95\uc6d0 3\ubd80(\uc8fc\uc2ec \uc548\ub300\ud76c \ub300\ubc95\uad00)\uae4c\uc9c0 \uc62c\ub77c\uac14\uc73c\uba70 1939\ub144, \ubc15\uc815\ud76c\uac00 \uc11c\uba85\ud55c \ubb38\uacbd\uacf5\ub9bd\ubcf4\ud1b5\uc18c\ud559\uad50 \u201c\uc131\uc801\ud1b5\uc9c0\ud45c\u201d\uc640 1940\ub144, \ubc15\uc815\ud76c\uac00 \uad50\uc9c1\uc744 \uc758\uc6d0\uba74\uc9c1\ud588\uc74c\uc744 \ubcf4\uc5ec\uc8fc\ub294 \uad50\uc721 \ub2f9\uad6d\uc758 \uc11c\ub958\ub97c \uc81c\ucd9c\ud558\uc600\ub294\ub370 \uc774\uc640 \uad00\ub828\ub41c \uc7ac\ud310\uc5d0\uc11c \uc548\ub300\ud76c \uc7ac\ud310\uad00 \ub4f1 \uc7ac\ud310\ubd80\ub294 \u201c\uadf8\uc758 \uce5c\uc77c \ud589\uc801 \uc5ec\ubd80\uc5d0 \ub300\ud574 \ub17c\ub780\uc774 \uc788\uace0 \ud2b9\uc124\ubd80\ub300\uc5d0 \uadfc\ubb34\ud588\ub294\uc9c0\ub3c4 \ud55c\uad6d \ud604\ub300\uc0ac\uc758 \uc7c1\uc810\uc73c\ub85c \uacc4\uc18d \uc5f0\uad6c\ub3fc\uc57c \ud55c\ub2e4. \ucc45\uc5d0 \uc801\uc2dc\ub41c \ub0b4\uc6a9\uc774 \uc77c\ubc18\uc801\uc73c\ub85c \uc54c\ub824\uc9c4 \uc0ac\uc2e4\uc5d0 \ubc18\ud55c\ub2e4\uace0 \ud558\ub354\ub77c\ub3c4 \ud53c\uace0\uc778\uc774 \u2018\ud5c8\uc704\u2019\uc784\uc744 \uc778\uc2dd\ud588\ub2e4\uace0 \ub2e8\uc815\ud560 \uc218 \uc5c6\uc5b4 \ubb34\uc8c4\ub97c \uc120\uace0\ud55c \uc6d0\uc2ec\uc740 \uc815\ub2f9\ud558\ub2e4\u201d\ub294 \uc774\uc720\ub97c \ub4e4\uc5b4 \ubb34\uc8c4\ub97c \ud310\uacb0\ud588\ub2e4. \uc774\uc640 \uad00\ub828\ud558\uc5ec \ub370\uc77c\ub9ac\uc548\uc740 \ub2e4\ub978 \uc5b8\ub860\ub4e4\uc774 \ubb34\uc8c4\ud310\uacb0\uc744 \uac00\uc9c0\uace0 \ub958\uc5f0\uc0b0\uc758 \uc8fc\uc7a5\uc744 \uc815\ub2f9\ud654\ud574\uc11c\ub294 \uc548\ub418\uba70, \uc774 \ud310\uacb0\uc740 \ubb34\uc8c4\uac00 \uc8c4\uac00 \uc5c6\uc74c\uc744 \uc758\ubbf8\ud558\ub294 \uac83\uc774 \uc544\ub2c8\ub77c \uc720\uc8c4\uc784\uc744 \ud655\uc99d\ud560 \uadfc\uac70\uac00 \uc5c6\ub2e4\ub294 \ub73b\uc774\ub77c\uace0 \uc8fc\uc7a5\ud588\ub2e4. \ub300\ubc95\uc6d0 3\ubd80\uc5d0\uc11c\ub294 \"\uc5ed\uc0ac\uc801\u00b7\uacf5\uc801 \uc778\ubb3c\uc758 \uacbd\uc6b0 \uc2dc\uac04\uc774 \uacbd\uacfc\ud558\uba74 \ub9dd\uc778\uacfc \uc720\uc871\uc758 \uba85\uc608\ubcf4\ub2e4 \uc5ed\uc0ac\uc801 \uc0ac\uc2e4\uc5d0 \ub300\ud55c \ud45c\ud604\uc758 \uc790\uc720\uac00 \ubcf4\ud638\ub3fc\uc57c \ud558\ubbc0\ub85c \uc0ac\uc790 \uba85\uc608\ud6fc\uc190\uc8c4\uac00 \uc131\ub9bd\ud558\ub824\uba74 \ud5c8\uc704 \uc0ac\uc2e4\uc5d0 \ub300\ud55c \uace0\uc758\uc131\uc744 \uc5c4\uaca9\ud788 \ub530\uc838\uc57c \ud55c\ub2e4\"\uba70 \"\ubc15 \uc804 \ub300\ud1b5\ub839\uc758 \ud2b9\uc124\ubd80\ub300 \uadfc\ubb34\uc124\uc740 \uc5ec\ub7ec \ucc45\uc5d0 \uc5b8\uae09\ub410\uace0 \uc800\uc790 \ub958\uc528\ub294 \uc5ed\uc0ac\ud559\uacc4\uc5d0\uc11c\ub3c4 \uc778\uc9c0\ub3c4\uac00 \uc788\uc5b4, \uac80\uc0ac\uac00 \uc81c\ucd9c\ud55c \uc99d\uac70\ub9cc\uc73c\ub85c\ub294 \uc720\uc8c4\uac00 \uc778\uc815\ub418\uc9c0 \uc54a\ub294\ub2e4\"\uba70 \ubb34\uc8c4\ub97c \uc120\uace0\ud588\ub2e4."} {"question": "\uc778\uc811\uad6d\uac00\ub85c\ubd80\ud130 \ub178\ub3d9\ub825\uc744 \uc18c\uc704 \uc190\ub2d8\ub178\ub3d9\uc790\ub77c\ub294 \uc774\ub984\uc73c\ub85c \ubd88\ub7ec\ub4e4\uc774\uae30 \uc2dc\uc791\ud55c \ud574\ub294?", "paragraph": "\ub3c5\uc77c\uc740 \uc774\ubbf8 19\uc138\uae30\uc5d0 \uc0b0\uc5c5\ud654\uc5d0 \ub530\ub77c \ub9ce\uc740 \ub18d\ucd0c \uc778\uad6c\ub4e4\uc774 \ub3c4\uc2dc\uc640 \ubbf8\uad6d\uc73c\ub85c \uc774\uc8fc\ud568\uc73c\ub85c\uc368 \ub178\ub3d9\ub825\uc774 \ubd80\uc871\ud558\uac8c \ub418\uc5b4 \uc8fc\ub85c \ub18d\uc5c5\ubd80\ubb38\uc5d0 \uacc4\uc808\ub9c8\ub2e4 \uc774\ud0c8\ub9ac\uc544, \ud3f4\ub780\ub4dc\ub85c\ubd80\ud130 \ub178\ub3d9\ub825\uc744 \ubd88\ub7ec\ub4e4\uc600\ub2e4. \uc81c1\ucc28 \uc138\uacc4\ub300\uc804\uacfc \ub098\uce58\uc2dc\ub300\uc5d0\ub294 \uc678\uad6d\uc778 \ub178\ub3d9\ub825\uc744 \uac15\uc81c \uc9d5\uc6a9\ud558\uc5ec \uc804\uc7c1 \uc0b0\uc5c5\uc5d0 \ud22c\uc785\ud558\uc600\uc73c\uba70, \ub098\uce58 \uc2dc\ub300 \ub54c \uac15\uc81c \uc9d5\uc6a9\ub41c \uc678\uad6d\uc778 \ub178\ub3d9\uc790\uc5d0 \ub300\ud55c \ubc30\uc0c1\ubb38\uc81c\ub294 \uc544\uc9c1\ub3c4 \ud574\uacb0\ub418\uc9c0 \uc54a\uace0 \uc788\ub2e4. \uc81c2\ucc28 \uc138\uacc4\ub300\uc804 \uc774\ud6c4 50\ub144\ub300\uc5d0 \uace0\ub3c4\uc758 \uacbd\uc81c\uc131\uc7a5\uacfc \ud568\uaed8 \uc0b0\uc5c5\ubd80\ubb38\uc5d0 \ub178\ub3d9\ub825\uc774 \ubd80\uc871\ud558\uc5ec 1955\ub144\ubd80\ud130 \uc778\uc811\uad6d\uac00\ub85c\ubd80\ud130 \ub178\ub3d9\ub825\uc744 \uc18c\uc704 '\uc190\ub2d8\ub178\ub3d9\uc790'\ub77c\ub294 \uc774\ub984\uc73c\ub85c \ubd88\ub7ec\ub4e4\uc600\ub2e4. \uc774 \uc190\ub2d8\ub178\ub3d9\uc790\ub4e4\uc740 \ub2e8\uae30\uac04 \ub0b4 \ubaa9\ub3c8 \ub9c8\ub828\uc774 \uc6a9\uc774\ud558\uc9c0 \uc54a\uc544 \uc810\ucc28 \uc815\ucc29\ud558\uac8c \ub418\uc5c8\ub2e4. \ub530\ub77c\uc11c, 2\ub144\ub9c8\ub2e4 \uc0c8\ub85c\uc6b4 \ub178\ub3d9\ub825\uc744 \ubd88\ub7ec\ub4e4\uc778\ub2e4\ub294 \ucc98\uc74c\uc758 'Rotation \uc6d0\uce59'\uc740 \uc2e4\uc6a9\uc131\uc774 \uc5c6\uc5b4 \uace7 \ud3ec\uae30\ud558\uac8c \ub418\uc5c8\ub2e4. \uc2a4\uc704\uc2a4 \uc791\uac00 Max Frisch\uc758 \ub9d0\ucc98\ub7fc \ub178\ub3d9\ub825\uc744 \ubd88\ub800\uc73c\ub098 \uc778\uac04\uc774 \uc628 \uac83\uc774\ub2e4. \ub3c5\uc77c\uc5d0\ub294 \uc57d 7\ubc3134\ub9cc \uba85\uc758 \uc678\uad6d\uc778\uc774 \uc0b4\uace0 \uc788\ub2e4. \uc804\uccb4 \uad6d\ubbfc\uc758 \uc57d 9%\uac00 \uc678\uad6d\uc778\uc778 \uc148\uc774\ub2e4. \ub3c5\uc77c\uc758 \uc778\uad6c\ub294 \uacc4\uc18d \uac10\uc18c\ud558\uace0 \uc788\uc73c\uba70, \uc0c8\ub85c \ud0dc\uc5b4\ub098\ub294 \uc544\uc774\ub4e4\uc758 10 - 15%\uac00 \uc678\uad6d\uc778\uc774\ub2e4. \uc804 \uc678\uad6d\uc778\uc758 \uc57d 15%\ub294 \uc815\uce58\ub9dd\uba85\uc774\ub098 \ub0b4\ub780, \uc804\uc7c1 \ub4f1\uc5d0 \uc758\ud55c \ud53c\ub09c\ubbfc\uc774\uba70 \uc774\ub4e4 \uc911 \uc57d 20%\ub294 \ub3c5\uc77c \uad6d\uac00\ub85c\ubd80\ud130 \ud53c\ub09c\ubbfc\uc774\ub77c\ub294 \uc778\uc815\uc744 \ubc1b\uace0 \uc788\ub2e4. \uacf5\uc2dd\uc801\uc73c\ub85c \uc778\uc815\ubc1b\uc740 \ud53c\ub09c\ubbfc\uc740 \ub3c5\uc77c \uad6d\uac00 \ub0b4\uc5d0\uc11c \uc790\uc720\uc2a4\ub7fd\uac8c \uc6c0\uc9c1\uc77c \uc218 \uc788\uc73c\uba70, \uc9c1\uc5c5\uc744 \uac00\uc9c8 \uc218 \uc788\ub2e4. \ub300\ubd80\ubd84\uc758 \uc678\uad6d\uc778\uc740 \ud070 \ub3c4\uc2dc\uc5d0 \uc9d1\uc911\ud574\uc11c \uac70\uc8fc\ud558\uace0 \uc788\ub2e4. \ud504\ub791\ud06c\ud478\ub974\ud2b8 \uacbd\uc6b0\uc5d0\ub294 \uc2dc\ubbfc\uc758 30% \uc774\uc0c1\uc774, \ubb8c\ud5e8, \uc288\ud22c\ud2b8\uac00\ub974\ud2b8, \ucfb0\ub978\uc740 20% \uc774\uc0c1\uc774 \uc678\uad6d\uc778\uc774\ub2e4", "answer": "1955\ub144", "sentence": "\uc81c2\ucc28 \uc138\uacc4\ub300\uc804 \uc774\ud6c4 50\ub144\ub300\uc5d0 \uace0\ub3c4\uc758 \uacbd\uc81c\uc131\uc7a5\uacfc \ud568\uaed8 \uc0b0\uc5c5\ubd80\ubb38\uc5d0 \ub178\ub3d9\ub825\uc774 \ubd80\uc871\ud558\uc5ec 1955\ub144 \ubd80\ud130 \uc778\uc811\uad6d\uac00\ub85c\ubd80\ud130 \ub178\ub3d9\ub825\uc744 \uc18c\uc704 '\uc190\ub2d8\ub178\ub3d9\uc790'\ub77c\ub294 \uc774\ub984\uc73c\ub85c \ubd88\ub7ec\ub4e4\uc600\ub2e4.", "paragraph_sentence": "\ub3c5\uc77c\uc740 \uc774\ubbf8 19\uc138\uae30\uc5d0 \uc0b0\uc5c5\ud654\uc5d0 \ub530\ub77c \ub9ce\uc740 \ub18d\ucd0c \uc778\uad6c\ub4e4\uc774 \ub3c4\uc2dc\uc640 \ubbf8\uad6d\uc73c\ub85c \uc774\uc8fc\ud568\uc73c\ub85c\uc368 \ub178\ub3d9\ub825\uc774 \ubd80\uc871\ud558\uac8c \ub418\uc5b4 \uc8fc\ub85c \ub18d\uc5c5\ubd80\ubb38\uc5d0 \uacc4\uc808\ub9c8\ub2e4 \uc774\ud0c8\ub9ac\uc544, \ud3f4\ub780\ub4dc\ub85c\ubd80\ud130 \ub178\ub3d9\ub825\uc744 \ubd88\ub7ec\ub4e4\uc600\ub2e4. \uc81c1\ucc28 \uc138\uacc4\ub300\uc804\uacfc \ub098\uce58\uc2dc\ub300\uc5d0\ub294 \uc678\uad6d\uc778 \ub178\ub3d9\ub825\uc744 \uac15\uc81c \uc9d5\uc6a9\ud558\uc5ec \uc804\uc7c1 \uc0b0\uc5c5\uc5d0 \ud22c\uc785\ud558\uc600\uc73c\uba70, \ub098\uce58 \uc2dc\ub300 \ub54c \uac15\uc81c \uc9d5\uc6a9\ub41c \uc678\uad6d\uc778 \ub178\ub3d9\uc790\uc5d0 \ub300\ud55c \ubc30\uc0c1\ubb38\uc81c\ub294 \uc544\uc9c1\ub3c4 \ud574\uacb0\ub418\uc9c0 \uc54a\uace0 \uc788\ub2e4. \uc81c2\ucc28 \uc138\uacc4\ub300\uc804 \uc774\ud6c4 50\ub144\ub300\uc5d0 \uace0\ub3c4\uc758 \uacbd\uc81c\uc131\uc7a5\uacfc \ud568\uaed8 \uc0b0\uc5c5\ubd80\ubb38\uc5d0 \ub178\ub3d9\ub825\uc774 \ubd80\uc871\ud558\uc5ec 1955\ub144 \ubd80\ud130 \uc778\uc811\uad6d\uac00\ub85c\ubd80\ud130 \ub178\ub3d9\ub825\uc744 \uc18c\uc704 '\uc190\ub2d8\ub178\ub3d9\uc790'\ub77c\ub294 \uc774\ub984\uc73c\ub85c \ubd88\ub7ec\ub4e4\uc600\ub2e4. \uc774 \uc190\ub2d8\ub178\ub3d9\uc790\ub4e4\uc740 \ub2e8\uae30\uac04 \ub0b4 \ubaa9\ub3c8 \ub9c8\ub828\uc774 \uc6a9\uc774\ud558\uc9c0 \uc54a\uc544 \uc810\ucc28 \uc815\ucc29\ud558\uac8c \ub418\uc5c8\ub2e4. \ub530\ub77c\uc11c, 2\ub144\ub9c8\ub2e4 \uc0c8\ub85c\uc6b4 \ub178\ub3d9\ub825\uc744 \ubd88\ub7ec\ub4e4\uc778\ub2e4\ub294 \ucc98\uc74c\uc758 'Rotation \uc6d0\uce59'\uc740 \uc2e4\uc6a9\uc131\uc774 \uc5c6\uc5b4 \uace7 \ud3ec\uae30\ud558\uac8c \ub418\uc5c8\ub2e4. \uc2a4\uc704\uc2a4 \uc791\uac00 Max Frisch\uc758 \ub9d0\ucc98\ub7fc \ub178\ub3d9\ub825\uc744 \ubd88\ub800\uc73c\ub098 \uc778\uac04\uc774 \uc628 \uac83\uc774\ub2e4. \ub3c5\uc77c\uc5d0\ub294 \uc57d 7\ubc3134\ub9cc \uba85\uc758 \uc678\uad6d\uc778\uc774 \uc0b4\uace0 \uc788\ub2e4. \uc804\uccb4 \uad6d\ubbfc\uc758 \uc57d 9%\uac00 \uc678\uad6d\uc778\uc778 \uc148\uc774\ub2e4. \ub3c5\uc77c\uc758 \uc778\uad6c\ub294 \uacc4\uc18d \uac10\uc18c\ud558\uace0 \uc788\uc73c\uba70, \uc0c8\ub85c \ud0dc\uc5b4\ub098\ub294 \uc544\uc774\ub4e4\uc758 10 - 15%\uac00 \uc678\uad6d\uc778\uc774\ub2e4. \uc804 \uc678\uad6d\uc778\uc758 \uc57d 15%\ub294 \uc815\uce58\ub9dd\uba85\uc774\ub098 \ub0b4\ub780, \uc804\uc7c1 \ub4f1\uc5d0 \uc758\ud55c \ud53c\ub09c\ubbfc\uc774\uba70 \uc774\ub4e4 \uc911 \uc57d 20%\ub294 \ub3c5\uc77c \uad6d\uac00\ub85c\ubd80\ud130 \ud53c\ub09c\ubbfc\uc774\ub77c\ub294 \uc778\uc815\uc744 \ubc1b\uace0 \uc788\ub2e4. \uacf5\uc2dd\uc801\uc73c\ub85c \uc778\uc815\ubc1b\uc740 \ud53c\ub09c\ubbfc\uc740 \ub3c5\uc77c \uad6d\uac00 \ub0b4\uc5d0\uc11c \uc790\uc720\uc2a4\ub7fd\uac8c \uc6c0\uc9c1\uc77c \uc218 \uc788\uc73c\uba70, \uc9c1\uc5c5\uc744 \uac00\uc9c8 \uc218 \uc788\ub2e4. \ub300\ubd80\ubd84\uc758 \uc678\uad6d\uc778\uc740 \ud070 \ub3c4\uc2dc\uc5d0 \uc9d1\uc911\ud574\uc11c \uac70\uc8fc\ud558\uace0 \uc788\ub2e4. \ud504\ub791\ud06c\ud478\ub974\ud2b8 \uacbd\uc6b0\uc5d0\ub294 \uc2dc\ubbfc\uc758 30% \uc774\uc0c1\uc774, \ubb8c\ud5e8, \uc288\ud22c\ud2b8\uac00\ub974\ud2b8, \ucfb0\ub978\uc740 20% \uc774\uc0c1\uc774 \uc678\uad6d\uc778\uc774\ub2e4", "paragraph_answer": "\ub3c5\uc77c\uc740 \uc774\ubbf8 19\uc138\uae30\uc5d0 \uc0b0\uc5c5\ud654\uc5d0 \ub530\ub77c \ub9ce\uc740 \ub18d\ucd0c \uc778\uad6c\ub4e4\uc774 \ub3c4\uc2dc\uc640 \ubbf8\uad6d\uc73c\ub85c \uc774\uc8fc\ud568\uc73c\ub85c\uc368 \ub178\ub3d9\ub825\uc774 \ubd80\uc871\ud558\uac8c \ub418\uc5b4 \uc8fc\ub85c \ub18d\uc5c5\ubd80\ubb38\uc5d0 \uacc4\uc808\ub9c8\ub2e4 \uc774\ud0c8\ub9ac\uc544, \ud3f4\ub780\ub4dc\ub85c\ubd80\ud130 \ub178\ub3d9\ub825\uc744 \ubd88\ub7ec\ub4e4\uc600\ub2e4. \uc81c1\ucc28 \uc138\uacc4\ub300\uc804\uacfc \ub098\uce58\uc2dc\ub300\uc5d0\ub294 \uc678\uad6d\uc778 \ub178\ub3d9\ub825\uc744 \uac15\uc81c \uc9d5\uc6a9\ud558\uc5ec \uc804\uc7c1 \uc0b0\uc5c5\uc5d0 \ud22c\uc785\ud558\uc600\uc73c\uba70, \ub098\uce58 \uc2dc\ub300 \ub54c \uac15\uc81c \uc9d5\uc6a9\ub41c \uc678\uad6d\uc778 \ub178\ub3d9\uc790\uc5d0 \ub300\ud55c \ubc30\uc0c1\ubb38\uc81c\ub294 \uc544\uc9c1\ub3c4 \ud574\uacb0\ub418\uc9c0 \uc54a\uace0 \uc788\ub2e4. \uc81c2\ucc28 \uc138\uacc4\ub300\uc804 \uc774\ud6c4 50\ub144\ub300\uc5d0 \uace0\ub3c4\uc758 \uacbd\uc81c\uc131\uc7a5\uacfc \ud568\uaed8 \uc0b0\uc5c5\ubd80\ubb38\uc5d0 \ub178\ub3d9\ub825\uc774 \ubd80\uc871\ud558\uc5ec 1955\ub144 \ubd80\ud130 \uc778\uc811\uad6d\uac00\ub85c\ubd80\ud130 \ub178\ub3d9\ub825\uc744 \uc18c\uc704 '\uc190\ub2d8\ub178\ub3d9\uc790'\ub77c\ub294 \uc774\ub984\uc73c\ub85c \ubd88\ub7ec\ub4e4\uc600\ub2e4. \uc774 \uc190\ub2d8\ub178\ub3d9\uc790\ub4e4\uc740 \ub2e8\uae30\uac04 \ub0b4 \ubaa9\ub3c8 \ub9c8\ub828\uc774 \uc6a9\uc774\ud558\uc9c0 \uc54a\uc544 \uc810\ucc28 \uc815\ucc29\ud558\uac8c \ub418\uc5c8\ub2e4. \ub530\ub77c\uc11c, 2\ub144\ub9c8\ub2e4 \uc0c8\ub85c\uc6b4 \ub178\ub3d9\ub825\uc744 \ubd88\ub7ec\ub4e4\uc778\ub2e4\ub294 \ucc98\uc74c\uc758 'Rotation \uc6d0\uce59'\uc740 \uc2e4\uc6a9\uc131\uc774 \uc5c6\uc5b4 \uace7 \ud3ec\uae30\ud558\uac8c \ub418\uc5c8\ub2e4. \uc2a4\uc704\uc2a4 \uc791\uac00 Max Frisch\uc758 \ub9d0\ucc98\ub7fc \ub178\ub3d9\ub825\uc744 \ubd88\ub800\uc73c\ub098 \uc778\uac04\uc774 \uc628 \uac83\uc774\ub2e4. \ub3c5\uc77c\uc5d0\ub294 \uc57d 7\ubc3134\ub9cc \uba85\uc758 \uc678\uad6d\uc778\uc774 \uc0b4\uace0 \uc788\ub2e4. \uc804\uccb4 \uad6d\ubbfc\uc758 \uc57d 9%\uac00 \uc678\uad6d\uc778\uc778 \uc148\uc774\ub2e4. \ub3c5\uc77c\uc758 \uc778\uad6c\ub294 \uacc4\uc18d \uac10\uc18c\ud558\uace0 \uc788\uc73c\uba70, \uc0c8\ub85c \ud0dc\uc5b4\ub098\ub294 \uc544\uc774\ub4e4\uc758 10 - 15%\uac00 \uc678\uad6d\uc778\uc774\ub2e4. \uc804 \uc678\uad6d\uc778\uc758 \uc57d 15%\ub294 \uc815\uce58\ub9dd\uba85\uc774\ub098 \ub0b4\ub780, \uc804\uc7c1 \ub4f1\uc5d0 \uc758\ud55c \ud53c\ub09c\ubbfc\uc774\uba70 \uc774\ub4e4 \uc911 \uc57d 20%\ub294 \ub3c5\uc77c \uad6d\uac00\ub85c\ubd80\ud130 \ud53c\ub09c\ubbfc\uc774\ub77c\ub294 \uc778\uc815\uc744 \ubc1b\uace0 \uc788\ub2e4. \uacf5\uc2dd\uc801\uc73c\ub85c \uc778\uc815\ubc1b\uc740 \ud53c\ub09c\ubbfc\uc740 \ub3c5\uc77c \uad6d\uac00 \ub0b4\uc5d0\uc11c \uc790\uc720\uc2a4\ub7fd\uac8c \uc6c0\uc9c1\uc77c \uc218 \uc788\uc73c\uba70, \uc9c1\uc5c5\uc744 \uac00\uc9c8 \uc218 \uc788\ub2e4. \ub300\ubd80\ubd84\uc758 \uc678\uad6d\uc778\uc740 \ud070 \ub3c4\uc2dc\uc5d0 \uc9d1\uc911\ud574\uc11c \uac70\uc8fc\ud558\uace0 \uc788\ub2e4. \ud504\ub791\ud06c\ud478\ub974\ud2b8 \uacbd\uc6b0\uc5d0\ub294 \uc2dc\ubbfc\uc758 30% \uc774\uc0c1\uc774, \ubb8c\ud5e8, \uc288\ud22c\ud2b8\uac00\ub974\ud2b8, \ucfb0\ub978\uc740 20% \uc774\uc0c1\uc774 \uc678\uad6d\uc778\uc774\ub2e4", "sentence_answer": "\uc81c2\ucc28 \uc138\uacc4\ub300\uc804 \uc774\ud6c4 50\ub144\ub300\uc5d0 \uace0\ub3c4\uc758 \uacbd\uc81c\uc131\uc7a5\uacfc \ud568\uaed8 \uc0b0\uc5c5\ubd80\ubb38\uc5d0 \ub178\ub3d9\ub825\uc774 \ubd80\uc871\ud558\uc5ec 1955\ub144 \ubd80\ud130 \uc778\uc811\uad6d\uac00\ub85c\ubd80\ud130 \ub178\ub3d9\ub825\uc744 \uc18c\uc704 '\uc190\ub2d8\ub178\ub3d9\uc790'\ub77c\ub294 \uc774\ub984\uc73c\ub85c \ubd88\ub7ec\ub4e4\uc600\ub2e4.", "paragraph_id": "6513004-0-1", "paragraph_question": "question: \uc778\uc811\uad6d\uac00\ub85c\ubd80\ud130 \ub178\ub3d9\ub825\uc744 \uc18c\uc704 \uc190\ub2d8\ub178\ub3d9\uc790\ub77c\ub294 \uc774\ub984\uc73c\ub85c \ubd88\ub7ec\ub4e4\uc774\uae30 \uc2dc\uc791\ud55c \ud574\ub294?, context: \ub3c5\uc77c\uc740 \uc774\ubbf8 19\uc138\uae30\uc5d0 \uc0b0\uc5c5\ud654\uc5d0 \ub530\ub77c \ub9ce\uc740 \ub18d\ucd0c \uc778\uad6c\ub4e4\uc774 \ub3c4\uc2dc\uc640 \ubbf8\uad6d\uc73c\ub85c \uc774\uc8fc\ud568\uc73c\ub85c\uc368 \ub178\ub3d9\ub825\uc774 \ubd80\uc871\ud558\uac8c \ub418\uc5b4 \uc8fc\ub85c \ub18d\uc5c5\ubd80\ubb38\uc5d0 \uacc4\uc808\ub9c8\ub2e4 \uc774\ud0c8\ub9ac\uc544, \ud3f4\ub780\ub4dc\ub85c\ubd80\ud130 \ub178\ub3d9\ub825\uc744 \ubd88\ub7ec\ub4e4\uc600\ub2e4. \uc81c1\ucc28 \uc138\uacc4\ub300\uc804\uacfc \ub098\uce58\uc2dc\ub300\uc5d0\ub294 \uc678\uad6d\uc778 \ub178\ub3d9\ub825\uc744 \uac15\uc81c \uc9d5\uc6a9\ud558\uc5ec \uc804\uc7c1 \uc0b0\uc5c5\uc5d0 \ud22c\uc785\ud558\uc600\uc73c\uba70, \ub098\uce58 \uc2dc\ub300 \ub54c \uac15\uc81c \uc9d5\uc6a9\ub41c \uc678\uad6d\uc778 \ub178\ub3d9\uc790\uc5d0 \ub300\ud55c \ubc30\uc0c1\ubb38\uc81c\ub294 \uc544\uc9c1\ub3c4 \ud574\uacb0\ub418\uc9c0 \uc54a\uace0 \uc788\ub2e4. \uc81c2\ucc28 \uc138\uacc4\ub300\uc804 \uc774\ud6c4 50\ub144\ub300\uc5d0 \uace0\ub3c4\uc758 \uacbd\uc81c\uc131\uc7a5\uacfc \ud568\uaed8 \uc0b0\uc5c5\ubd80\ubb38\uc5d0 \ub178\ub3d9\ub825\uc774 \ubd80\uc871\ud558\uc5ec 1955\ub144\ubd80\ud130 \uc778\uc811\uad6d\uac00\ub85c\ubd80\ud130 \ub178\ub3d9\ub825\uc744 \uc18c\uc704 '\uc190\ub2d8\ub178\ub3d9\uc790'\ub77c\ub294 \uc774\ub984\uc73c\ub85c \ubd88\ub7ec\ub4e4\uc600\ub2e4. \uc774 \uc190\ub2d8\ub178\ub3d9\uc790\ub4e4\uc740 \ub2e8\uae30\uac04 \ub0b4 \ubaa9\ub3c8 \ub9c8\ub828\uc774 \uc6a9\uc774\ud558\uc9c0 \uc54a\uc544 \uc810\ucc28 \uc815\ucc29\ud558\uac8c \ub418\uc5c8\ub2e4. \ub530\ub77c\uc11c, 2\ub144\ub9c8\ub2e4 \uc0c8\ub85c\uc6b4 \ub178\ub3d9\ub825\uc744 \ubd88\ub7ec\ub4e4\uc778\ub2e4\ub294 \ucc98\uc74c\uc758 'Rotation \uc6d0\uce59'\uc740 \uc2e4\uc6a9\uc131\uc774 \uc5c6\uc5b4 \uace7 \ud3ec\uae30\ud558\uac8c \ub418\uc5c8\ub2e4. \uc2a4\uc704\uc2a4 \uc791\uac00 Max Frisch\uc758 \ub9d0\ucc98\ub7fc \ub178\ub3d9\ub825\uc744 \ubd88\ub800\uc73c\ub098 \uc778\uac04\uc774 \uc628 \uac83\uc774\ub2e4. \ub3c5\uc77c\uc5d0\ub294 \uc57d 7\ubc3134\ub9cc \uba85\uc758 \uc678\uad6d\uc778\uc774 \uc0b4\uace0 \uc788\ub2e4. \uc804\uccb4 \uad6d\ubbfc\uc758 \uc57d 9%\uac00 \uc678\uad6d\uc778\uc778 \uc148\uc774\ub2e4. \ub3c5\uc77c\uc758 \uc778\uad6c\ub294 \uacc4\uc18d \uac10\uc18c\ud558\uace0 \uc788\uc73c\uba70, \uc0c8\ub85c \ud0dc\uc5b4\ub098\ub294 \uc544\uc774\ub4e4\uc758 10 - 15%\uac00 \uc678\uad6d\uc778\uc774\ub2e4. \uc804 \uc678\uad6d\uc778\uc758 \uc57d 15%\ub294 \uc815\uce58\ub9dd\uba85\uc774\ub098 \ub0b4\ub780, \uc804\uc7c1 \ub4f1\uc5d0 \uc758\ud55c \ud53c\ub09c\ubbfc\uc774\uba70 \uc774\ub4e4 \uc911 \uc57d 20%\ub294 \ub3c5\uc77c \uad6d\uac00\ub85c\ubd80\ud130 \ud53c\ub09c\ubbfc\uc774\ub77c\ub294 \uc778\uc815\uc744 \ubc1b\uace0 \uc788\ub2e4. \uacf5\uc2dd\uc801\uc73c\ub85c \uc778\uc815\ubc1b\uc740 \ud53c\ub09c\ubbfc\uc740 \ub3c5\uc77c \uad6d\uac00 \ub0b4\uc5d0\uc11c \uc790\uc720\uc2a4\ub7fd\uac8c \uc6c0\uc9c1\uc77c \uc218 \uc788\uc73c\uba70, \uc9c1\uc5c5\uc744 \uac00\uc9c8 \uc218 \uc788\ub2e4. \ub300\ubd80\ubd84\uc758 \uc678\uad6d\uc778\uc740 \ud070 \ub3c4\uc2dc\uc5d0 \uc9d1\uc911\ud574\uc11c \uac70\uc8fc\ud558\uace0 \uc788\ub2e4. \ud504\ub791\ud06c\ud478\ub974\ud2b8 \uacbd\uc6b0\uc5d0\ub294 \uc2dc\ubbfc\uc758 30% \uc774\uc0c1\uc774, \ubb8c\ud5e8, \uc288\ud22c\ud2b8\uac00\ub974\ud2b8, \ucfb0\ub978\uc740 20% \uc774\uc0c1\uc774 \uc678\uad6d\uc778\uc774\ub2e4"} {"question": "YG\uc5d4\ud130\ud14c\uc778\uba3c\ud2b8\uc640 \uc7ac\uacc4\uc57d\uc744 \ud55c 2NE1\uc758 \uba64\ubc84\ub294?", "paragraph": "2016\ub144 11\uc6d4 26\uc77c YG\ub294 \ud648\ud398\uc774\uc9c0\uc5d0 \"5\uc6d4 2NE1\uc758 \uc804\uc18d \uacc4\uc57d\uc774 \ub9cc\ub8cc\ub410\uace0, \uacf5\ubbfc\uc9c0 \uc591\uc774 \ud568\uaed8 \ud560 \uc218 \uc5c6\uac8c \ub41c \uc0c1\ud669 \uc18d\uc5d0\uc11c YG\ub294 \ub098\uba38\uc9c0 \uba64\ubc84\ub4e4\uacfc \uc624\ub79c \uc0c1\uc758 \ub05d\uc5d0 2NE1\uc758 \uacf5\uc2dd \ud574\uccb4\ub97c \uacb0\uc815\ud558\uac8c \ub418\uc5c8\uc2b5\ub2c8\ub2e4.\"\ub77c\uba70 2NE1\uc758 \ud574\uccb4 \uc18c\uc2dd\uc744 \uc804\ud588\ub2e4. \ud574\uccb4\uc758 \uc774\uc720\uc5d0 \ub300\ud574\uc11c\ub294 \"\ud65c\ub3d9\uc744 \uc9c0\uc18d\ud574 \ub098\uac00\ub294 \uac83\uc740 \ubb34\ub9ac\uac00 \uc788\ub2e4\uace0 \ud310\ub2e8, \uae30\uc57d \uc5c6\ub294 2NE1\uc758 \ud65c\ub3d9\uc744 \uae30\ub300\ud558\uae30\ubcf4\ub2e4\ub294 \uba64\ubc84\ub4e4\uc758 \uc194\ub85c \ud65c\ub3d9\uc5d0 \ub9e4\uc9c4\ud558\uae30\ub85c \uacb0\uc815\ud558\uac8c \ub41c \uac83\uc785\ub2c8\ub2e4.\" \ub77c\uba70 \ud68c\uc0ac\uc640 \uba64\ubc84\ub4e4\uc758 \uc785\uc7a5\uc744 \uc804\ud588\uace0, \"YG\ub294 \uc9c0\ub09c 5\uc6d4 \uc774\ud6c4 CL\uacfc \uc0b0\ub2e4\ub77c\ubc15\uc758 \uc194\ub85c \uacc4\uc57d\uc744 \uccb4\uacb0\ud588\uc73c\uba70, \uc544\uc27d\uac8c\ub3c4 \ubc15\ubd04 \uc591\uc740 \uc7ac\uacc4\uc57d\uc744 \uccb4\uacb0\ud558\uc9c0 \uc54a\uc558\uc74c\uc744 \uacf5\uc2dd\uc801\uc73c\ub85c \uc54c\ub824\ub4dc\ub9bd\ub2c8\ub2e4\"\ub294 \ub0b4\uc6a9\uc744 \ub367\ubd99\uc774\uba70 YG\uc5d4\ud130\ud14c\uc778\uba3c\ud2b8\uc5d0 \uc0b0\ub2e4\ub77c\ubc15\uacfc \uc528\uc5d8 \ub450 \uba64\ubc84\ub9cc \ub0a8\uac8c\ub41c \uc18c\uc2dd\uc744 \uc804\ud588\ub2e4.", "answer": "CL\uacfc \uc0b0\ub2e4\ub77c\ubc15", "sentence": "\ub77c\uba70 \ud68c\uc0ac\uc640 \uba64\ubc84\ub4e4\uc758 \uc785\uc7a5\uc744 \uc804\ud588\uace0, \"YG\ub294 \uc9c0\ub09c 5\uc6d4 \uc774\ud6c4 CL\uacfc \uc0b0\ub2e4\ub77c\ubc15 \uc758 \uc194\ub85c \uacc4\uc57d\uc744 \uccb4\uacb0\ud588\uc73c\uba70, \uc544\uc27d\uac8c\ub3c4 \ubc15\ubd04 \uc591\uc740 \uc7ac\uacc4\uc57d\uc744 \uccb4\uacb0\ud558\uc9c0 \uc54a\uc558\uc74c\uc744 \uacf5\uc2dd\uc801\uc73c\ub85c \uc54c\ub824\ub4dc\ub9bd\ub2c8\ub2e4\"\ub294 \ub0b4\uc6a9\uc744 \ub367\ubd99\uc774\uba70 YG\uc5d4\ud130\ud14c\uc778\uba3c\ud2b8\uc5d0 \uc0b0\ub2e4\ub77c\ubc15\uacfc \uc528\uc5d8 \ub450 \uba64\ubc84\ub9cc \ub0a8\uac8c\ub41c \uc18c\uc2dd\uc744 \uc804\ud588\ub2e4.", "paragraph_sentence": "2016\ub144 11\uc6d4 26\uc77c YG\ub294 \ud648\ud398\uc774\uc9c0\uc5d0 \"5\uc6d4 2NE1\uc758 \uc804\uc18d \uacc4\uc57d\uc774 \ub9cc\ub8cc\ub410\uace0, \uacf5\ubbfc\uc9c0 \uc591\uc774 \ud568\uaed8 \ud560 \uc218 \uc5c6\uac8c \ub41c \uc0c1\ud669 \uc18d\uc5d0\uc11c YG\ub294 \ub098\uba38\uc9c0 \uba64\ubc84\ub4e4\uacfc \uc624\ub79c \uc0c1\uc758 \ub05d\uc5d0 2NE1\uc758 \uacf5\uc2dd \ud574\uccb4\ub97c \uacb0\uc815\ud558\uac8c \ub418\uc5c8\uc2b5\ub2c8\ub2e4. \"\ub77c\uba70 2NE1\uc758 \ud574\uccb4 \uc18c\uc2dd\uc744 \uc804\ud588\ub2e4. \ud574\uccb4\uc758 \uc774\uc720\uc5d0 \ub300\ud574\uc11c\ub294 \"\ud65c\ub3d9\uc744 \uc9c0\uc18d\ud574 \ub098\uac00\ub294 \uac83\uc740 \ubb34\ub9ac\uac00 \uc788\ub2e4\uace0 \ud310\ub2e8, \uae30\uc57d \uc5c6\ub294 2NE1\uc758 \ud65c\ub3d9\uc744 \uae30\ub300\ud558\uae30\ubcf4\ub2e4\ub294 \uba64\ubc84\ub4e4\uc758 \uc194\ub85c \ud65c\ub3d9\uc5d0 \ub9e4\uc9c4\ud558\uae30\ub85c \uacb0\uc815\ud558\uac8c \ub41c \uac83\uc785\ub2c8\ub2e4. \" \ub77c\uba70 \ud68c\uc0ac\uc640 \uba64\ubc84\ub4e4\uc758 \uc785\uc7a5\uc744 \uc804\ud588\uace0, \"YG\ub294 \uc9c0\ub09c 5\uc6d4 \uc774\ud6c4 CL\uacfc \uc0b0\ub2e4\ub77c\ubc15 \uc758 \uc194\ub85c \uacc4\uc57d\uc744 \uccb4\uacb0\ud588\uc73c\uba70, \uc544\uc27d\uac8c\ub3c4 \ubc15\ubd04 \uc591\uc740 \uc7ac\uacc4\uc57d\uc744 \uccb4\uacb0\ud558\uc9c0 \uc54a\uc558\uc74c\uc744 \uacf5\uc2dd\uc801\uc73c\ub85c \uc54c\ub824\ub4dc\ub9bd\ub2c8\ub2e4\"\ub294 \ub0b4\uc6a9\uc744 \ub367\ubd99\uc774\uba70 YG\uc5d4\ud130\ud14c\uc778\uba3c\ud2b8\uc5d0 \uc0b0\ub2e4\ub77c\ubc15\uacfc \uc528\uc5d8 \ub450 \uba64\ubc84\ub9cc \ub0a8\uac8c\ub41c \uc18c\uc2dd\uc744 \uc804\ud588\ub2e4. ", "paragraph_answer": "2016\ub144 11\uc6d4 26\uc77c YG\ub294 \ud648\ud398\uc774\uc9c0\uc5d0 \"5\uc6d4 2NE1\uc758 \uc804\uc18d \uacc4\uc57d\uc774 \ub9cc\ub8cc\ub410\uace0, \uacf5\ubbfc\uc9c0 \uc591\uc774 \ud568\uaed8 \ud560 \uc218 \uc5c6\uac8c \ub41c \uc0c1\ud669 \uc18d\uc5d0\uc11c YG\ub294 \ub098\uba38\uc9c0 \uba64\ubc84\ub4e4\uacfc \uc624\ub79c \uc0c1\uc758 \ub05d\uc5d0 2NE1\uc758 \uacf5\uc2dd \ud574\uccb4\ub97c \uacb0\uc815\ud558\uac8c \ub418\uc5c8\uc2b5\ub2c8\ub2e4.\"\ub77c\uba70 2NE1\uc758 \ud574\uccb4 \uc18c\uc2dd\uc744 \uc804\ud588\ub2e4. \ud574\uccb4\uc758 \uc774\uc720\uc5d0 \ub300\ud574\uc11c\ub294 \"\ud65c\ub3d9\uc744 \uc9c0\uc18d\ud574 \ub098\uac00\ub294 \uac83\uc740 \ubb34\ub9ac\uac00 \uc788\ub2e4\uace0 \ud310\ub2e8, \uae30\uc57d \uc5c6\ub294 2NE1\uc758 \ud65c\ub3d9\uc744 \uae30\ub300\ud558\uae30\ubcf4\ub2e4\ub294 \uba64\ubc84\ub4e4\uc758 \uc194\ub85c \ud65c\ub3d9\uc5d0 \ub9e4\uc9c4\ud558\uae30\ub85c \uacb0\uc815\ud558\uac8c \ub41c \uac83\uc785\ub2c8\ub2e4.\" \ub77c\uba70 \ud68c\uc0ac\uc640 \uba64\ubc84\ub4e4\uc758 \uc785\uc7a5\uc744 \uc804\ud588\uace0, \"YG\ub294 \uc9c0\ub09c 5\uc6d4 \uc774\ud6c4 CL\uacfc \uc0b0\ub2e4\ub77c\ubc15 \uc758 \uc194\ub85c \uacc4\uc57d\uc744 \uccb4\uacb0\ud588\uc73c\uba70, \uc544\uc27d\uac8c\ub3c4 \ubc15\ubd04 \uc591\uc740 \uc7ac\uacc4\uc57d\uc744 \uccb4\uacb0\ud558\uc9c0 \uc54a\uc558\uc74c\uc744 \uacf5\uc2dd\uc801\uc73c\ub85c \uc54c\ub824\ub4dc\ub9bd\ub2c8\ub2e4\"\ub294 \ub0b4\uc6a9\uc744 \ub367\ubd99\uc774\uba70 YG\uc5d4\ud130\ud14c\uc778\uba3c\ud2b8\uc5d0 \uc0b0\ub2e4\ub77c\ubc15\uacfc \uc528\uc5d8 \ub450 \uba64\ubc84\ub9cc \ub0a8\uac8c\ub41c \uc18c\uc2dd\uc744 \uc804\ud588\ub2e4.", "sentence_answer": "\ub77c\uba70 \ud68c\uc0ac\uc640 \uba64\ubc84\ub4e4\uc758 \uc785\uc7a5\uc744 \uc804\ud588\uace0, \"YG\ub294 \uc9c0\ub09c 5\uc6d4 \uc774\ud6c4 CL\uacfc \uc0b0\ub2e4\ub77c\ubc15 \uc758 \uc194\ub85c \uacc4\uc57d\uc744 \uccb4\uacb0\ud588\uc73c\uba70, \uc544\uc27d\uac8c\ub3c4 \ubc15\ubd04 \uc591\uc740 \uc7ac\uacc4\uc57d\uc744 \uccb4\uacb0\ud558\uc9c0 \uc54a\uc558\uc74c\uc744 \uacf5\uc2dd\uc801\uc73c\ub85c \uc54c\ub824\ub4dc\ub9bd\ub2c8\ub2e4\"\ub294 \ub0b4\uc6a9\uc744 \ub367\ubd99\uc774\uba70 YG\uc5d4\ud130\ud14c\uc778\uba3c\ud2b8\uc5d0 \uc0b0\ub2e4\ub77c\ubc15\uacfc \uc528\uc5d8 \ub450 \uba64\ubc84\ub9cc \ub0a8\uac8c\ub41c \uc18c\uc2dd\uc744 \uc804\ud588\ub2e4.", "paragraph_id": "5782603-10-0", "paragraph_question": "question: YG\uc5d4\ud130\ud14c\uc778\uba3c\ud2b8\uc640 \uc7ac\uacc4\uc57d\uc744 \ud55c 2NE1\uc758 \uba64\ubc84\ub294?, context: 2016\ub144 11\uc6d4 26\uc77c YG\ub294 \ud648\ud398\uc774\uc9c0\uc5d0 \"5\uc6d4 2NE1\uc758 \uc804\uc18d \uacc4\uc57d\uc774 \ub9cc\ub8cc\ub410\uace0, \uacf5\ubbfc\uc9c0 \uc591\uc774 \ud568\uaed8 \ud560 \uc218 \uc5c6\uac8c \ub41c \uc0c1\ud669 \uc18d\uc5d0\uc11c YG\ub294 \ub098\uba38\uc9c0 \uba64\ubc84\ub4e4\uacfc \uc624\ub79c \uc0c1\uc758 \ub05d\uc5d0 2NE1\uc758 \uacf5\uc2dd \ud574\uccb4\ub97c \uacb0\uc815\ud558\uac8c \ub418\uc5c8\uc2b5\ub2c8\ub2e4.\"\ub77c\uba70 2NE1\uc758 \ud574\uccb4 \uc18c\uc2dd\uc744 \uc804\ud588\ub2e4. \ud574\uccb4\uc758 \uc774\uc720\uc5d0 \ub300\ud574\uc11c\ub294 \"\ud65c\ub3d9\uc744 \uc9c0\uc18d\ud574 \ub098\uac00\ub294 \uac83\uc740 \ubb34\ub9ac\uac00 \uc788\ub2e4\uace0 \ud310\ub2e8, \uae30\uc57d \uc5c6\ub294 2NE1\uc758 \ud65c\ub3d9\uc744 \uae30\ub300\ud558\uae30\ubcf4\ub2e4\ub294 \uba64\ubc84\ub4e4\uc758 \uc194\ub85c \ud65c\ub3d9\uc5d0 \ub9e4\uc9c4\ud558\uae30\ub85c \uacb0\uc815\ud558\uac8c \ub41c \uac83\uc785\ub2c8\ub2e4.\" \ub77c\uba70 \ud68c\uc0ac\uc640 \uba64\ubc84\ub4e4\uc758 \uc785\uc7a5\uc744 \uc804\ud588\uace0, \"YG\ub294 \uc9c0\ub09c 5\uc6d4 \uc774\ud6c4 CL\uacfc \uc0b0\ub2e4\ub77c\ubc15\uc758 \uc194\ub85c \uacc4\uc57d\uc744 \uccb4\uacb0\ud588\uc73c\uba70, \uc544\uc27d\uac8c\ub3c4 \ubc15\ubd04 \uc591\uc740 \uc7ac\uacc4\uc57d\uc744 \uccb4\uacb0\ud558\uc9c0 \uc54a\uc558\uc74c\uc744 \uacf5\uc2dd\uc801\uc73c\ub85c \uc54c\ub824\ub4dc\ub9bd\ub2c8\ub2e4\"\ub294 \ub0b4\uc6a9\uc744 \ub367\ubd99\uc774\uba70 YG\uc5d4\ud130\ud14c\uc778\uba3c\ud2b8\uc5d0 \uc0b0\ub2e4\ub77c\ubc15\uacfc \uc528\uc5d8 \ub450 \uba64\ubc84\ub9cc \ub0a8\uac8c\ub41c \uc18c\uc2dd\uc744 \uc804\ud588\ub2e4."} {"question": "\uc2a4\ud14c\uace0\uc0ac\uc6b0\ub8e8\uc2a4\uac00 \ub4b7\ub2e4\ub9ac\ub97c \uc6c0\uc9c1\uc774\ub294 \ub300\uc2e0 \uc55e\ub2e4\ub9ac\ub97c \uc6c0\uc9c1\uc5ec \ud3ec\uc2dd\uc790\uc758 \uacf5\uaca9\uc5d0 \ubbfc\ucca9\ud558\uac8c \ub300\uc751\ud560 \uc218 \uc788\uc744 \uac83\uc774\ub77c\uace0 \ubcf8 \uc0ac\ub78c\uc740?", "paragraph": "\uc2a4\ud14c\uace0\uc0ac\uc6b0\ub8e8\uc2a4\uc758 \uaf2c\ub9ac\uc5d0 \uc788\ub294 \uac00\uc2dc\uac00 1914\ub144\uc5d0 \uae38\ubaa8\uc5b4\uac00 \uc8fc\uc7a5\ud588\ub358 \uac83\ucc98\ub7fc \uacfc\uc2dc\ub9cc\uc744 \uc704\ud55c \uac83\uc774\uc5c8\ub294\uc9c0, \uc544\ub2c8\uba74 \ubb34\uae30\ub85c\ub3c4 \uc0ac\uc6a9\ub418\uc5c8\ub294\uc9c0\uc5d0 \ub300\ud574\uc11c\ub294 \ub17c\uc7c1\uc774 \uc788\uc5b4 \uc654\ub2e4. \ub85c\ubc84\ud2b8 \ubc14\ucee4\ub294 \uc2a4\ud14c\uace0\uc0ac\uc6b0\ub8e8\uc2a4\uc758 \uaf2c\ub9ac\uc758 \ud798\uc904\uc774 \uace8\ud654\ub418\uc9c0 \uc54a\uc544 \ub2e4\ub978 \uacf5\ub8e1\ub4e4\uc758 \uaf2c\ub9ac\uc5d0 \ube44\ud574 \ud6e8\uc52c \uc720\uc5f0\ud588\uc744 \uac83\uc774\ub77c\uace0 \ubcf4\uace0 \uaf2c\ub9ac\uac00 \ubb34\uae30\ub85c \uc0ac\uc6a9\ub418\uc5c8\uc744 \uac00\ub2a5\uc131\uc774 \ub192\ub2e4\uace0 \uc0dd\uac01\ud588\ub2e4. \ud558\uc9c0\ub9cc \uce74\ud39c\ud130\uac00 \uc9c0\uc801\ud55c \uac83\ucc98\ub7fc \uace8\ud310\ub4e4\uc774 \uaf2c\ub9ac \ubd80\uc704\uc758 \ucc99\ucd94\ubf08\uc640 \ub9ce\uc774 \uacb9\uce58\uae30 \ub54c\ubb38\uc5d0 \uc6c0\uc9c1\uc784\uc740 \uc81c\ud55c\uc801\uc774\uc5c8\uc744 \uac83\uc774\ub2e4. \ubc14\ucee4\ub294 \ub610 \uc2a4\ud14c\uace0\uc0ac\uc6b0\ub8e8\uc2a4\uac00 \ucee4\ub2e4\ub780 \ub4b7\ub2e4\ub9ac\ub97c \uc6c0\uc9c1\uc774\uc9c0 \uc54a\uac8c \ub450\uace0, \uc9e7\uc9c0\ub9cc \ub9e4\uc6b0 \uac15\ub825\ud55c \uadfc\uc721\uc744 \uac00\uc9c4 \uc55e\ub2e4\ub9ac\ub85c \ubab8\uc744 \ubc00\uc5b4\ub0b4 \uc5c9\ub369\uc774 \ub4a4\ucabd\uc744 \uc27d\uac8c \uc6c0\uc9c1\uc77c \uc218 \uc788\uc5b4\uc11c \ud3ec\uc2dd\uc790\uc758 \uacf5\uaca9\uc5d0 \ubbfc\ucca9\ud558\uac8c \ub300\uc751\ud560 \uc218 \uc788\uc744 \uac83\uc774\ub77c\uace0 \ubcf4\uc558\ub2e4. \ub354 \ucd5c\uadfc\uc5d0 \ub9e5\ud718\ub2c8\uc640 \ub3d9\ub8cc\ub4e4\uc758 \uaf2c\ub9ac \uac00\uc2dc\uc5d0 \ub300\ud55c \uc5f0\uad6c\uc5d0\uc11c\ub294 \uc678\uc0c1\uc5d0 \uad00\ub828\ub41c \uc0c1\ucc98\uac00 \uc788\ub294 \uacbd\uc6b0\uac00 \ub9ce\uc740 \uac83\uc73c\ub85c \ubcf4\uc544 \uaf2c\ub9ac \uac00\uc2dc\uac00 \uc2e4\uc81c\ub85c \uc2f8\uc6c0\uc5d0 \uc0ac\uc6a9\ub418\uc5c8\uc744 \uac00\ub2a5\uc131\uc774 \ub192\uc740 \uac83\uc73c\ub85c \ub098\ud0c0\ub0ac\ub2e4. \uc774 \uc5f0\uad6c\uc5d0\uc11c \uc870\uc0ac\ub41c \uc2a4\ud14c\uace0\uc0ac\uc6b0\ub8e8\uc2a4\uc758 \ud45c\ubcf8\ub4e4 \uc911 9.8% \uac00 \uaf2c\ub9ac \uac00\uc2dc \ubd80\ubd84\uc5d0 \ubd80\uc0c1\uc744 \uc785\uc740 \uc0c1\ud0dc\uc600\ub2e4. \uc774 \uc0dd\uac01\uc5d0 \ub300\ud574 \uc2e0\ube59\uc131\uc744 \ub354\ud574\uc900 \uac83\uc740 \ud55c \uc54c\ub85c\uc0ac\uc6b0\ub8e8\uc2a4\uc758 \uaf2c\ub9ac\ubf08\uc5d0 \uc2a4\ud14c\uace0\uc0ac\uc6b0\ub8e8\uc2a4\uc758 \uaf2c\ub9ac \uac00\uc2dc\uc640 \uc644\ubcbd\ud558\uac8c \ub9de\uc544 \ub5a8\uc5b4\uc9c0\ub294 \uad6c\uba4d\uc774 \ub098\uc788\ub2e4\ub294 \uc810\uc774\uc5c8\ub2e4.", "answer": "\ubc14\ucee4", "sentence": "\ub85c\ubc84\ud2b8 \ubc14\ucee4 \ub294 \uc2a4\ud14c\uace0\uc0ac\uc6b0\ub8e8\uc2a4\uc758 \uaf2c\ub9ac\uc758 \ud798\uc904\uc774 \uace8\ud654\ub418\uc9c0 \uc54a\uc544 \ub2e4\ub978 \uacf5\ub8e1\ub4e4\uc758 \uaf2c\ub9ac\uc5d0 \ube44\ud574 \ud6e8\uc52c \uc720\uc5f0\ud588\uc744 \uac83\uc774\ub77c\uace0 \ubcf4\uace0 \uaf2c\ub9ac\uac00 \ubb34\uae30\ub85c \uc0ac\uc6a9\ub418\uc5c8\uc744 \uac00\ub2a5\uc131\uc774 \ub192\ub2e4\uace0 \uc0dd\uac01\ud588\ub2e4.", "paragraph_sentence": "\uc2a4\ud14c\uace0\uc0ac\uc6b0\ub8e8\uc2a4\uc758 \uaf2c\ub9ac\uc5d0 \uc788\ub294 \uac00\uc2dc\uac00 1914\ub144\uc5d0 \uae38\ubaa8\uc5b4\uac00 \uc8fc\uc7a5\ud588\ub358 \uac83\ucc98\ub7fc \uacfc\uc2dc\ub9cc\uc744 \uc704\ud55c \uac83\uc774\uc5c8\ub294\uc9c0, \uc544\ub2c8\uba74 \ubb34\uae30\ub85c\ub3c4 \uc0ac\uc6a9\ub418\uc5c8\ub294\uc9c0\uc5d0 \ub300\ud574\uc11c\ub294 \ub17c\uc7c1\uc774 \uc788\uc5b4 \uc654\ub2e4. \ub85c\ubc84\ud2b8 \ubc14\ucee4 \ub294 \uc2a4\ud14c\uace0\uc0ac\uc6b0\ub8e8\uc2a4\uc758 \uaf2c\ub9ac\uc758 \ud798\uc904\uc774 \uace8\ud654\ub418\uc9c0 \uc54a\uc544 \ub2e4\ub978 \uacf5\ub8e1\ub4e4\uc758 \uaf2c\ub9ac\uc5d0 \ube44\ud574 \ud6e8\uc52c \uc720\uc5f0\ud588\uc744 \uac83\uc774\ub77c\uace0 \ubcf4\uace0 \uaf2c\ub9ac\uac00 \ubb34\uae30\ub85c \uc0ac\uc6a9\ub418\uc5c8\uc744 \uac00\ub2a5\uc131\uc774 \ub192\ub2e4\uace0 \uc0dd\uac01\ud588\ub2e4. \ud558\uc9c0\ub9cc \uce74\ud39c\ud130\uac00 \uc9c0\uc801\ud55c \uac83\ucc98\ub7fc \uace8\ud310\ub4e4\uc774 \uaf2c\ub9ac \ubd80\uc704\uc758 \ucc99\ucd94\ubf08\uc640 \ub9ce\uc774 \uacb9\uce58\uae30 \ub54c\ubb38\uc5d0 \uc6c0\uc9c1\uc784\uc740 \uc81c\ud55c\uc801\uc774\uc5c8\uc744 \uac83\uc774\ub2e4. \ubc14\ucee4\ub294 \ub610 \uc2a4\ud14c\uace0\uc0ac\uc6b0\ub8e8\uc2a4\uac00 \ucee4\ub2e4\ub780 \ub4b7\ub2e4\ub9ac\ub97c \uc6c0\uc9c1\uc774\uc9c0 \uc54a\uac8c \ub450\uace0, \uc9e7\uc9c0\ub9cc \ub9e4\uc6b0 \uac15\ub825\ud55c \uadfc\uc721\uc744 \uac00\uc9c4 \uc55e\ub2e4\ub9ac\ub85c \ubab8\uc744 \ubc00\uc5b4\ub0b4 \uc5c9\ub369\uc774 \ub4a4\ucabd\uc744 \uc27d\uac8c \uc6c0\uc9c1\uc77c \uc218 \uc788\uc5b4\uc11c \ud3ec\uc2dd\uc790\uc758 \uacf5\uaca9\uc5d0 \ubbfc\ucca9\ud558\uac8c \ub300\uc751\ud560 \uc218 \uc788\uc744 \uac83\uc774\ub77c\uace0 \ubcf4\uc558\ub2e4. \ub354 \ucd5c\uadfc\uc5d0 \ub9e5\ud718\ub2c8\uc640 \ub3d9\ub8cc\ub4e4\uc758 \uaf2c\ub9ac \uac00\uc2dc\uc5d0 \ub300\ud55c \uc5f0\uad6c\uc5d0\uc11c\ub294 \uc678\uc0c1\uc5d0 \uad00\ub828\ub41c \uc0c1\ucc98\uac00 \uc788\ub294 \uacbd\uc6b0\uac00 \ub9ce\uc740 \uac83\uc73c\ub85c \ubcf4\uc544 \uaf2c\ub9ac \uac00\uc2dc\uac00 \uc2e4\uc81c\ub85c \uc2f8\uc6c0\uc5d0 \uc0ac\uc6a9\ub418\uc5c8\uc744 \uac00\ub2a5\uc131\uc774 \ub192\uc740 \uac83\uc73c\ub85c \ub098\ud0c0\ub0ac\ub2e4. \uc774 \uc5f0\uad6c\uc5d0\uc11c \uc870\uc0ac\ub41c \uc2a4\ud14c\uace0\uc0ac\uc6b0\ub8e8\uc2a4\uc758 \ud45c\ubcf8\ub4e4 \uc911 9.8% \uac00 \uaf2c\ub9ac \uac00\uc2dc \ubd80\ubd84\uc5d0 \ubd80\uc0c1\uc744 \uc785\uc740 \uc0c1\ud0dc\uc600\ub2e4. \uc774 \uc0dd\uac01\uc5d0 \ub300\ud574 \uc2e0\ube59\uc131\uc744 \ub354\ud574\uc900 \uac83\uc740 \ud55c \uc54c\ub85c\uc0ac\uc6b0\ub8e8\uc2a4\uc758 \uaf2c\ub9ac\ubf08\uc5d0 \uc2a4\ud14c\uace0\uc0ac\uc6b0\ub8e8\uc2a4\uc758 \uaf2c\ub9ac \uac00\uc2dc\uc640 \uc644\ubcbd\ud558\uac8c \ub9de\uc544 \ub5a8\uc5b4\uc9c0\ub294 \uad6c\uba4d\uc774 \ub098\uc788\ub2e4\ub294 \uc810\uc774\uc5c8\ub2e4.", "paragraph_answer": "\uc2a4\ud14c\uace0\uc0ac\uc6b0\ub8e8\uc2a4\uc758 \uaf2c\ub9ac\uc5d0 \uc788\ub294 \uac00\uc2dc\uac00 1914\ub144\uc5d0 \uae38\ubaa8\uc5b4\uac00 \uc8fc\uc7a5\ud588\ub358 \uac83\ucc98\ub7fc \uacfc\uc2dc\ub9cc\uc744 \uc704\ud55c \uac83\uc774\uc5c8\ub294\uc9c0, \uc544\ub2c8\uba74 \ubb34\uae30\ub85c\ub3c4 \uc0ac\uc6a9\ub418\uc5c8\ub294\uc9c0\uc5d0 \ub300\ud574\uc11c\ub294 \ub17c\uc7c1\uc774 \uc788\uc5b4 \uc654\ub2e4. \ub85c\ubc84\ud2b8 \ubc14\ucee4 \ub294 \uc2a4\ud14c\uace0\uc0ac\uc6b0\ub8e8\uc2a4\uc758 \uaf2c\ub9ac\uc758 \ud798\uc904\uc774 \uace8\ud654\ub418\uc9c0 \uc54a\uc544 \ub2e4\ub978 \uacf5\ub8e1\ub4e4\uc758 \uaf2c\ub9ac\uc5d0 \ube44\ud574 \ud6e8\uc52c \uc720\uc5f0\ud588\uc744 \uac83\uc774\ub77c\uace0 \ubcf4\uace0 \uaf2c\ub9ac\uac00 \ubb34\uae30\ub85c \uc0ac\uc6a9\ub418\uc5c8\uc744 \uac00\ub2a5\uc131\uc774 \ub192\ub2e4\uace0 \uc0dd\uac01\ud588\ub2e4. \ud558\uc9c0\ub9cc \uce74\ud39c\ud130\uac00 \uc9c0\uc801\ud55c \uac83\ucc98\ub7fc \uace8\ud310\ub4e4\uc774 \uaf2c\ub9ac \ubd80\uc704\uc758 \ucc99\ucd94\ubf08\uc640 \ub9ce\uc774 \uacb9\uce58\uae30 \ub54c\ubb38\uc5d0 \uc6c0\uc9c1\uc784\uc740 \uc81c\ud55c\uc801\uc774\uc5c8\uc744 \uac83\uc774\ub2e4. \ubc14\ucee4\ub294 \ub610 \uc2a4\ud14c\uace0\uc0ac\uc6b0\ub8e8\uc2a4\uac00 \ucee4\ub2e4\ub780 \ub4b7\ub2e4\ub9ac\ub97c \uc6c0\uc9c1\uc774\uc9c0 \uc54a\uac8c \ub450\uace0, \uc9e7\uc9c0\ub9cc \ub9e4\uc6b0 \uac15\ub825\ud55c \uadfc\uc721\uc744 \uac00\uc9c4 \uc55e\ub2e4\ub9ac\ub85c \ubab8\uc744 \ubc00\uc5b4\ub0b4 \uc5c9\ub369\uc774 \ub4a4\ucabd\uc744 \uc27d\uac8c \uc6c0\uc9c1\uc77c \uc218 \uc788\uc5b4\uc11c \ud3ec\uc2dd\uc790\uc758 \uacf5\uaca9\uc5d0 \ubbfc\ucca9\ud558\uac8c \ub300\uc751\ud560 \uc218 \uc788\uc744 \uac83\uc774\ub77c\uace0 \ubcf4\uc558\ub2e4. \ub354 \ucd5c\uadfc\uc5d0 \ub9e5\ud718\ub2c8\uc640 \ub3d9\ub8cc\ub4e4\uc758 \uaf2c\ub9ac \uac00\uc2dc\uc5d0 \ub300\ud55c \uc5f0\uad6c\uc5d0\uc11c\ub294 \uc678\uc0c1\uc5d0 \uad00\ub828\ub41c \uc0c1\ucc98\uac00 \uc788\ub294 \uacbd\uc6b0\uac00 \ub9ce\uc740 \uac83\uc73c\ub85c \ubcf4\uc544 \uaf2c\ub9ac \uac00\uc2dc\uac00 \uc2e4\uc81c\ub85c \uc2f8\uc6c0\uc5d0 \uc0ac\uc6a9\ub418\uc5c8\uc744 \uac00\ub2a5\uc131\uc774 \ub192\uc740 \uac83\uc73c\ub85c \ub098\ud0c0\ub0ac\ub2e4. \uc774 \uc5f0\uad6c\uc5d0\uc11c \uc870\uc0ac\ub41c \uc2a4\ud14c\uace0\uc0ac\uc6b0\ub8e8\uc2a4\uc758 \ud45c\ubcf8\ub4e4 \uc911 9.8% \uac00 \uaf2c\ub9ac \uac00\uc2dc \ubd80\ubd84\uc5d0 \ubd80\uc0c1\uc744 \uc785\uc740 \uc0c1\ud0dc\uc600\ub2e4. \uc774 \uc0dd\uac01\uc5d0 \ub300\ud574 \uc2e0\ube59\uc131\uc744 \ub354\ud574\uc900 \uac83\uc740 \ud55c \uc54c\ub85c\uc0ac\uc6b0\ub8e8\uc2a4\uc758 \uaf2c\ub9ac\ubf08\uc5d0 \uc2a4\ud14c\uace0\uc0ac\uc6b0\ub8e8\uc2a4\uc758 \uaf2c\ub9ac \uac00\uc2dc\uc640 \uc644\ubcbd\ud558\uac8c \ub9de\uc544 \ub5a8\uc5b4\uc9c0\ub294 \uad6c\uba4d\uc774 \ub098\uc788\ub2e4\ub294 \uc810\uc774\uc5c8\ub2e4.", "sentence_answer": "\ub85c\ubc84\ud2b8 \ubc14\ucee4 \ub294 \uc2a4\ud14c\uace0\uc0ac\uc6b0\ub8e8\uc2a4\uc758 \uaf2c\ub9ac\uc758 \ud798\uc904\uc774 \uace8\ud654\ub418\uc9c0 \uc54a\uc544 \ub2e4\ub978 \uacf5\ub8e1\ub4e4\uc758 \uaf2c\ub9ac\uc5d0 \ube44\ud574 \ud6e8\uc52c \uc720\uc5f0\ud588\uc744 \uac83\uc774\ub77c\uace0 \ubcf4\uace0 \uaf2c\ub9ac\uac00 \ubb34\uae30\ub85c \uc0ac\uc6a9\ub418\uc5c8\uc744 \uac00\ub2a5\uc131\uc774 \ub192\ub2e4\uace0 \uc0dd\uac01\ud588\ub2e4.", "paragraph_id": "6532852-10-0", "paragraph_question": "question: \uc2a4\ud14c\uace0\uc0ac\uc6b0\ub8e8\uc2a4\uac00 \ub4b7\ub2e4\ub9ac\ub97c \uc6c0\uc9c1\uc774\ub294 \ub300\uc2e0 \uc55e\ub2e4\ub9ac\ub97c \uc6c0\uc9c1\uc5ec \ud3ec\uc2dd\uc790\uc758 \uacf5\uaca9\uc5d0 \ubbfc\ucca9\ud558\uac8c \ub300\uc751\ud560 \uc218 \uc788\uc744 \uac83\uc774\ub77c\uace0 \ubcf8 \uc0ac\ub78c\uc740?, context: \uc2a4\ud14c\uace0\uc0ac\uc6b0\ub8e8\uc2a4\uc758 \uaf2c\ub9ac\uc5d0 \uc788\ub294 \uac00\uc2dc\uac00 1914\ub144\uc5d0 \uae38\ubaa8\uc5b4\uac00 \uc8fc\uc7a5\ud588\ub358 \uac83\ucc98\ub7fc \uacfc\uc2dc\ub9cc\uc744 \uc704\ud55c \uac83\uc774\uc5c8\ub294\uc9c0, \uc544\ub2c8\uba74 \ubb34\uae30\ub85c\ub3c4 \uc0ac\uc6a9\ub418\uc5c8\ub294\uc9c0\uc5d0 \ub300\ud574\uc11c\ub294 \ub17c\uc7c1\uc774 \uc788\uc5b4 \uc654\ub2e4. \ub85c\ubc84\ud2b8 \ubc14\ucee4\ub294 \uc2a4\ud14c\uace0\uc0ac\uc6b0\ub8e8\uc2a4\uc758 \uaf2c\ub9ac\uc758 \ud798\uc904\uc774 \uace8\ud654\ub418\uc9c0 \uc54a\uc544 \ub2e4\ub978 \uacf5\ub8e1\ub4e4\uc758 \uaf2c\ub9ac\uc5d0 \ube44\ud574 \ud6e8\uc52c \uc720\uc5f0\ud588\uc744 \uac83\uc774\ub77c\uace0 \ubcf4\uace0 \uaf2c\ub9ac\uac00 \ubb34\uae30\ub85c \uc0ac\uc6a9\ub418\uc5c8\uc744 \uac00\ub2a5\uc131\uc774 \ub192\ub2e4\uace0 \uc0dd\uac01\ud588\ub2e4. \ud558\uc9c0\ub9cc \uce74\ud39c\ud130\uac00 \uc9c0\uc801\ud55c \uac83\ucc98\ub7fc \uace8\ud310\ub4e4\uc774 \uaf2c\ub9ac \ubd80\uc704\uc758 \ucc99\ucd94\ubf08\uc640 \ub9ce\uc774 \uacb9\uce58\uae30 \ub54c\ubb38\uc5d0 \uc6c0\uc9c1\uc784\uc740 \uc81c\ud55c\uc801\uc774\uc5c8\uc744 \uac83\uc774\ub2e4. \ubc14\ucee4\ub294 \ub610 \uc2a4\ud14c\uace0\uc0ac\uc6b0\ub8e8\uc2a4\uac00 \ucee4\ub2e4\ub780 \ub4b7\ub2e4\ub9ac\ub97c \uc6c0\uc9c1\uc774\uc9c0 \uc54a\uac8c \ub450\uace0, \uc9e7\uc9c0\ub9cc \ub9e4\uc6b0 \uac15\ub825\ud55c \uadfc\uc721\uc744 \uac00\uc9c4 \uc55e\ub2e4\ub9ac\ub85c \ubab8\uc744 \ubc00\uc5b4\ub0b4 \uc5c9\ub369\uc774 \ub4a4\ucabd\uc744 \uc27d\uac8c \uc6c0\uc9c1\uc77c \uc218 \uc788\uc5b4\uc11c \ud3ec\uc2dd\uc790\uc758 \uacf5\uaca9\uc5d0 \ubbfc\ucca9\ud558\uac8c \ub300\uc751\ud560 \uc218 \uc788\uc744 \uac83\uc774\ub77c\uace0 \ubcf4\uc558\ub2e4. \ub354 \ucd5c\uadfc\uc5d0 \ub9e5\ud718\ub2c8\uc640 \ub3d9\ub8cc\ub4e4\uc758 \uaf2c\ub9ac \uac00\uc2dc\uc5d0 \ub300\ud55c \uc5f0\uad6c\uc5d0\uc11c\ub294 \uc678\uc0c1\uc5d0 \uad00\ub828\ub41c \uc0c1\ucc98\uac00 \uc788\ub294 \uacbd\uc6b0\uac00 \ub9ce\uc740 \uac83\uc73c\ub85c \ubcf4\uc544 \uaf2c\ub9ac \uac00\uc2dc\uac00 \uc2e4\uc81c\ub85c \uc2f8\uc6c0\uc5d0 \uc0ac\uc6a9\ub418\uc5c8\uc744 \uac00\ub2a5\uc131\uc774 \ub192\uc740 \uac83\uc73c\ub85c \ub098\ud0c0\ub0ac\ub2e4. \uc774 \uc5f0\uad6c\uc5d0\uc11c \uc870\uc0ac\ub41c \uc2a4\ud14c\uace0\uc0ac\uc6b0\ub8e8\uc2a4\uc758 \ud45c\ubcf8\ub4e4 \uc911 9.8% \uac00 \uaf2c\ub9ac \uac00\uc2dc \ubd80\ubd84\uc5d0 \ubd80\uc0c1\uc744 \uc785\uc740 \uc0c1\ud0dc\uc600\ub2e4. \uc774 \uc0dd\uac01\uc5d0 \ub300\ud574 \uc2e0\ube59\uc131\uc744 \ub354\ud574\uc900 \uac83\uc740 \ud55c \uc54c\ub85c\uc0ac\uc6b0\ub8e8\uc2a4\uc758 \uaf2c\ub9ac\ubf08\uc5d0 \uc2a4\ud14c\uace0\uc0ac\uc6b0\ub8e8\uc2a4\uc758 \uaf2c\ub9ac \uac00\uc2dc\uc640 \uc644\ubcbd\ud558\uac8c \ub9de\uc544 \ub5a8\uc5b4\uc9c0\ub294 \uad6c\uba4d\uc774 \ub098\uc788\ub2e4\ub294 \uc810\uc774\uc5c8\ub2e4."} {"question": "\uadf8\uc2a4\ud0c0\ube0c \ub77c\ub4dc\ubd80\ub974\ud750\uc5d0 \ub530\ub974\uba74 \uc790\uc720\uc8fc\uc758 \uc0ac\uc0c1\uc740 \ubb34\uc5c7\uc758 \ubcf4\uc7a5\uc73c\ub85c \ucd9c\ubc1c\ub418\ub294\uac00?", "paragraph": "\ub3c5\uc77c\uc758 \ubc95\ud559\uc790 \uad6c\uc2a4\ud0c0\ube0c \ub77c\ub4dc\ubd80\ub974\ud750\ub294 \ubbfc\uc8fc\uc8fc\uc758\ub97c \uc88c\ud30c\uc801 \uc790\uc720\uc8fc\uc758\ub85c \ud574\uc11d\ud55c \uac83\uc740 \uc798\ubabb \ub418\uc5c8\ub2e4\uace0 \ubcf4\uba74\uc11c \uc790\uc720\uc8fc\uc758\uc640 \ubbfc\uc8fc\uc8fc\uc758 \uc0ac\uc774\uc5d0\ub294 \uc815\ub3c4\uc758 \ucc28\uc774\ub9cc\uc774 \uc544\ub2c8\ub77c \uc885\ub958\uc758 \ucc28\uc774\uac00 \uc788\ub2e4\uace0 \ub9d0\ud55c\ub2e4. \uadf8\uc5d0 \ub530\ub974\uba74, \ubbfc\uc8fc\uc8fc\uc758\ub294 \ub2e4\uc218\uc758 \uc758\uc0ac\uc758 \ubb34\uc870\uac74\uc801 \uc9c0\ubc30\ub97c \ub9d0\ud558\uc9c0\ub9cc \uc790\uc720\uc8fc\uc758\ub294 \uac1c\uc778\uc758 \uc758\uc0ac\ub97c \uc704\ud558\uc5ec\uc11c\ub294 (\uacbd\uc6b0\uc5d0 \ub530\ub77c) \ub2e4\uc218\uc758 \uc758\uc0ac\uc5d0 \ub300\ud558\uc5ec \uc790\uae30\uc790\uc2e0\uc744 \uc8fc\uc7a5\ud560 \uac00\ub2a5\uc131\uc744 \uc5f4\uc5b4\ub454\ub2e4. \uadf8\ub9ac\uace0 \uc790\uc720\uc8fc\uc758 \uc0ac\uc0c1\uc758 \ucd9c\ubc1c\uc810\uc740 \uc804(\u524d)\uad6d\uac00\uc801 \uc790\uc720\uc758 \ubcf4\uc7a5\uc778\ub370 \ubc18\ud558\uc5ec \ubbfc\uc8fc\uc8fc\uc758\ub294 \uac1c\uc778\uc758 \uc804\uad6d\uac00\uc801 \uc790\uc720\ub97c \ub2e4\uc218\uc790\uc758 \ucc98\ubd84\uc5d0 \uc704\uc784\ud558\uc5ec \ub300\uc2e0 \ub2e4\uc218 \uc758\uc0ac\uc758 \ud615\uc131\uc5d0 \uad00\uc5ec\ud560 \uac00\ub2a5\uc131\ub9cc\uc744 \uac00\uc9c4\ub2e4\uace0 \ud55c\ub2e4. \uace0\ub85c \uc790\uc720\uc8fc\uc758\ub294 \uc790\uc720\ub97c \ubbfc\uc8fc\uc8fc\uc758\ub294 \ud3c9\ub4f1\uc744 \ub9d0\ud558\uba70 \uc790\uc720\uc8fc\uc758\ub294 \uac1c\uc778\uc5d0\uac8c \ubb34\ud55c\uc758 \uac00\uce58\ub97c \ubbfc\uc8fc\uc8fc\uc758\ub294 \uc720\ud55c\uc758 \uac00\uce58\ub97c \uc900\ub2e4\uace0 \uc774\uc57c\uae30\ud55c\ub2e4.", "answer": "\uc804(\u524d)\uad6d\uac00\uc801 \uc790\uc720", "sentence": "\uadf8\ub9ac\uace0 \uc790\uc720\uc8fc\uc758 \uc0ac\uc0c1\uc758 \ucd9c\ubc1c\uc810\uc740 \uc804(\u524d)\uad6d\uac00\uc801 \uc790\uc720 \uc758 \ubcf4\uc7a5\uc778\ub370 \ubc18\ud558\uc5ec \ubbfc\uc8fc\uc8fc\uc758\ub294 \uac1c\uc778\uc758 \uc804\uad6d\uac00\uc801 \uc790\uc720\ub97c \ub2e4\uc218\uc790\uc758 \ucc98\ubd84\uc5d0 \uc704\uc784\ud558\uc5ec \ub300\uc2e0 \ub2e4\uc218 \uc758\uc0ac\uc758 \ud615\uc131\uc5d0 \uad00\uc5ec\ud560 \uac00\ub2a5\uc131\ub9cc\uc744 \uac00\uc9c4\ub2e4\uace0 \ud55c\ub2e4.", "paragraph_sentence": "\ub3c5\uc77c\uc758 \ubc95\ud559\uc790 \uad6c\uc2a4\ud0c0\ube0c \ub77c\ub4dc\ubd80\ub974\ud750\ub294 \ubbfc\uc8fc\uc8fc\uc758\ub97c \uc88c\ud30c\uc801 \uc790\uc720\uc8fc\uc758\ub85c \ud574\uc11d\ud55c \uac83\uc740 \uc798\ubabb \ub418\uc5c8\ub2e4\uace0 \ubcf4\uba74\uc11c \uc790\uc720\uc8fc\uc758\uc640 \ubbfc\uc8fc\uc8fc\uc758 \uc0ac\uc774\uc5d0\ub294 \uc815\ub3c4\uc758 \ucc28\uc774\ub9cc\uc774 \uc544\ub2c8\ub77c \uc885\ub958\uc758 \ucc28\uc774\uac00 \uc788\ub2e4\uace0 \ub9d0\ud55c\ub2e4. \uadf8\uc5d0 \ub530\ub974\uba74, \ubbfc\uc8fc\uc8fc\uc758\ub294 \ub2e4\uc218\uc758 \uc758\uc0ac\uc758 \ubb34\uc870\uac74\uc801 \uc9c0\ubc30\ub97c \ub9d0\ud558\uc9c0\ub9cc \uc790\uc720\uc8fc\uc758\ub294 \uac1c\uc778\uc758 \uc758\uc0ac\ub97c \uc704\ud558\uc5ec\uc11c\ub294 (\uacbd\uc6b0\uc5d0 \ub530\ub77c) \ub2e4\uc218\uc758 \uc758\uc0ac\uc5d0 \ub300\ud558\uc5ec \uc790\uae30\uc790\uc2e0\uc744 \uc8fc\uc7a5\ud560 \uac00\ub2a5\uc131\uc744 \uc5f4\uc5b4\ub454\ub2e4. \uadf8\ub9ac\uace0 \uc790\uc720\uc8fc\uc758 \uc0ac\uc0c1\uc758 \ucd9c\ubc1c\uc810\uc740 \uc804(\u524d)\uad6d\uac00\uc801 \uc790\uc720 \uc758 \ubcf4\uc7a5\uc778\ub370 \ubc18\ud558\uc5ec \ubbfc\uc8fc\uc8fc\uc758\ub294 \uac1c\uc778\uc758 \uc804\uad6d\uac00\uc801 \uc790\uc720\ub97c \ub2e4\uc218\uc790\uc758 \ucc98\ubd84\uc5d0 \uc704\uc784\ud558\uc5ec \ub300\uc2e0 \ub2e4\uc218 \uc758\uc0ac\uc758 \ud615\uc131\uc5d0 \uad00\uc5ec\ud560 \uac00\ub2a5\uc131\ub9cc\uc744 \uac00\uc9c4\ub2e4\uace0 \ud55c\ub2e4. \uace0\ub85c \uc790\uc720\uc8fc\uc758\ub294 \uc790\uc720\ub97c \ubbfc\uc8fc\uc8fc\uc758\ub294 \ud3c9\ub4f1\uc744 \ub9d0\ud558\uba70 \uc790\uc720\uc8fc\uc758\ub294 \uac1c\uc778\uc5d0\uac8c \ubb34\ud55c\uc758 \uac00\uce58\ub97c \ubbfc\uc8fc\uc8fc\uc758\ub294 \uc720\ud55c\uc758 \uac00\uce58\ub97c \uc900\ub2e4\uace0 \uc774\uc57c\uae30\ud55c\ub2e4.", "paragraph_answer": "\ub3c5\uc77c\uc758 \ubc95\ud559\uc790 \uad6c\uc2a4\ud0c0\ube0c \ub77c\ub4dc\ubd80\ub974\ud750\ub294 \ubbfc\uc8fc\uc8fc\uc758\ub97c \uc88c\ud30c\uc801 \uc790\uc720\uc8fc\uc758\ub85c \ud574\uc11d\ud55c \uac83\uc740 \uc798\ubabb \ub418\uc5c8\ub2e4\uace0 \ubcf4\uba74\uc11c \uc790\uc720\uc8fc\uc758\uc640 \ubbfc\uc8fc\uc8fc\uc758 \uc0ac\uc774\uc5d0\ub294 \uc815\ub3c4\uc758 \ucc28\uc774\ub9cc\uc774 \uc544\ub2c8\ub77c \uc885\ub958\uc758 \ucc28\uc774\uac00 \uc788\ub2e4\uace0 \ub9d0\ud55c\ub2e4. \uadf8\uc5d0 \ub530\ub974\uba74, \ubbfc\uc8fc\uc8fc\uc758\ub294 \ub2e4\uc218\uc758 \uc758\uc0ac\uc758 \ubb34\uc870\uac74\uc801 \uc9c0\ubc30\ub97c \ub9d0\ud558\uc9c0\ub9cc \uc790\uc720\uc8fc\uc758\ub294 \uac1c\uc778\uc758 \uc758\uc0ac\ub97c \uc704\ud558\uc5ec\uc11c\ub294 (\uacbd\uc6b0\uc5d0 \ub530\ub77c) \ub2e4\uc218\uc758 \uc758\uc0ac\uc5d0 \ub300\ud558\uc5ec \uc790\uae30\uc790\uc2e0\uc744 \uc8fc\uc7a5\ud560 \uac00\ub2a5\uc131\uc744 \uc5f4\uc5b4\ub454\ub2e4. \uadf8\ub9ac\uace0 \uc790\uc720\uc8fc\uc758 \uc0ac\uc0c1\uc758 \ucd9c\ubc1c\uc810\uc740 \uc804(\u524d)\uad6d\uac00\uc801 \uc790\uc720 \uc758 \ubcf4\uc7a5\uc778\ub370 \ubc18\ud558\uc5ec \ubbfc\uc8fc\uc8fc\uc758\ub294 \uac1c\uc778\uc758 \uc804\uad6d\uac00\uc801 \uc790\uc720\ub97c \ub2e4\uc218\uc790\uc758 \ucc98\ubd84\uc5d0 \uc704\uc784\ud558\uc5ec \ub300\uc2e0 \ub2e4\uc218 \uc758\uc0ac\uc758 \ud615\uc131\uc5d0 \uad00\uc5ec\ud560 \uac00\ub2a5\uc131\ub9cc\uc744 \uac00\uc9c4\ub2e4\uace0 \ud55c\ub2e4. \uace0\ub85c \uc790\uc720\uc8fc\uc758\ub294 \uc790\uc720\ub97c \ubbfc\uc8fc\uc8fc\uc758\ub294 \ud3c9\ub4f1\uc744 \ub9d0\ud558\uba70 \uc790\uc720\uc8fc\uc758\ub294 \uac1c\uc778\uc5d0\uac8c \ubb34\ud55c\uc758 \uac00\uce58\ub97c \ubbfc\uc8fc\uc8fc\uc758\ub294 \uc720\ud55c\uc758 \uac00\uce58\ub97c \uc900\ub2e4\uace0 \uc774\uc57c\uae30\ud55c\ub2e4.", "sentence_answer": "\uadf8\ub9ac\uace0 \uc790\uc720\uc8fc\uc758 \uc0ac\uc0c1\uc758 \ucd9c\ubc1c\uc810\uc740 \uc804(\u524d)\uad6d\uac00\uc801 \uc790\uc720 \uc758 \ubcf4\uc7a5\uc778\ub370 \ubc18\ud558\uc5ec \ubbfc\uc8fc\uc8fc\uc758\ub294 \uac1c\uc778\uc758 \uc804\uad6d\uac00\uc801 \uc790\uc720\ub97c \ub2e4\uc218\uc790\uc758 \ucc98\ubd84\uc5d0 \uc704\uc784\ud558\uc5ec \ub300\uc2e0 \ub2e4\uc218 \uc758\uc0ac\uc758 \ud615\uc131\uc5d0 \uad00\uc5ec\ud560 \uac00\ub2a5\uc131\ub9cc\uc744 \uac00\uc9c4\ub2e4\uace0 \ud55c\ub2e4.", "paragraph_id": "6657592-2-1", "paragraph_question": "question: \uadf8\uc2a4\ud0c0\ube0c \ub77c\ub4dc\ubd80\ub974\ud750\uc5d0 \ub530\ub974\uba74 \uc790\uc720\uc8fc\uc758 \uc0ac\uc0c1\uc740 \ubb34\uc5c7\uc758 \ubcf4\uc7a5\uc73c\ub85c \ucd9c\ubc1c\ub418\ub294\uac00?, context: \ub3c5\uc77c\uc758 \ubc95\ud559\uc790 \uad6c\uc2a4\ud0c0\ube0c \ub77c\ub4dc\ubd80\ub974\ud750\ub294 \ubbfc\uc8fc\uc8fc\uc758\ub97c \uc88c\ud30c\uc801 \uc790\uc720\uc8fc\uc758\ub85c \ud574\uc11d\ud55c \uac83\uc740 \uc798\ubabb \ub418\uc5c8\ub2e4\uace0 \ubcf4\uba74\uc11c \uc790\uc720\uc8fc\uc758\uc640 \ubbfc\uc8fc\uc8fc\uc758 \uc0ac\uc774\uc5d0\ub294 \uc815\ub3c4\uc758 \ucc28\uc774\ub9cc\uc774 \uc544\ub2c8\ub77c \uc885\ub958\uc758 \ucc28\uc774\uac00 \uc788\ub2e4\uace0 \ub9d0\ud55c\ub2e4. \uadf8\uc5d0 \ub530\ub974\uba74, \ubbfc\uc8fc\uc8fc\uc758\ub294 \ub2e4\uc218\uc758 \uc758\uc0ac\uc758 \ubb34\uc870\uac74\uc801 \uc9c0\ubc30\ub97c \ub9d0\ud558\uc9c0\ub9cc \uc790\uc720\uc8fc\uc758\ub294 \uac1c\uc778\uc758 \uc758\uc0ac\ub97c \uc704\ud558\uc5ec\uc11c\ub294 (\uacbd\uc6b0\uc5d0 \ub530\ub77c) \ub2e4\uc218\uc758 \uc758\uc0ac\uc5d0 \ub300\ud558\uc5ec \uc790\uae30\uc790\uc2e0\uc744 \uc8fc\uc7a5\ud560 \uac00\ub2a5\uc131\uc744 \uc5f4\uc5b4\ub454\ub2e4. \uadf8\ub9ac\uace0 \uc790\uc720\uc8fc\uc758 \uc0ac\uc0c1\uc758 \ucd9c\ubc1c\uc810\uc740 \uc804(\u524d)\uad6d\uac00\uc801 \uc790\uc720\uc758 \ubcf4\uc7a5\uc778\ub370 \ubc18\ud558\uc5ec \ubbfc\uc8fc\uc8fc\uc758\ub294 \uac1c\uc778\uc758 \uc804\uad6d\uac00\uc801 \uc790\uc720\ub97c \ub2e4\uc218\uc790\uc758 \ucc98\ubd84\uc5d0 \uc704\uc784\ud558\uc5ec \ub300\uc2e0 \ub2e4\uc218 \uc758\uc0ac\uc758 \ud615\uc131\uc5d0 \uad00\uc5ec\ud560 \uac00\ub2a5\uc131\ub9cc\uc744 \uac00\uc9c4\ub2e4\uace0 \ud55c\ub2e4. \uace0\ub85c \uc790\uc720\uc8fc\uc758\ub294 \uc790\uc720\ub97c \ubbfc\uc8fc\uc8fc\uc758\ub294 \ud3c9\ub4f1\uc744 \ub9d0\ud558\uba70 \uc790\uc720\uc8fc\uc758\ub294 \uac1c\uc778\uc5d0\uac8c \ubb34\ud55c\uc758 \uac00\uce58\ub97c \ubbfc\uc8fc\uc8fc\uc758\ub294 \uc720\ud55c\uc758 \uac00\uce58\ub97c \uc900\ub2e4\uace0 \uc774\uc57c\uae30\ud55c\ub2e4."} {"question": "\uae40\ud604\uc2dd\uacfc \ubd04\uc5ec\ub984\uac00\uc6b8\uaca8\uc6b8\uc5d0 \uc720\uc7ac\ud558\ub97c \ub300\uc2e0\ud574\uc11c \ub4e4\uc5b4\uc628 \uc0ac\ub78c\uc740?", "paragraph": "1986\ub144 \uae40\uc885\uc9c4, \uc804\ud0dc\uad00, \uc720\uc7ac\ud558, \uc7a5\uae30\ud638\ub294 \uc120\ubc30 \uae40\ud604\uc2dd\uacfc \ud568\uaed8 \"\uae40\ud604\uc2dd\uacfc \ubd04.\uc5ec\ub984.\uac00\uc744.\uaca8\uc6b8\"\uc744 \uacb0\uc131. \uc720\uc7ac\ud558\uc758 \ud0c8\ud1f4 \uc774\ud6c4 \ubc15\uc131\uc2dd\uc758 \ud569\ub958\ub85c \uae40\ud604\uc2dd 3\uc9d1 \"\uae40\ud604\uc2dd\uacfc \ubd04.\uc5ec\ub984.\uac00\uc744.\uaca8\uc6b8\"\uc744 \ubc1c\ud45c. \uae40\uc885\uc9c4\uc774 \uc791\uc0ac, \uc791\uace1\ud55c \u3008\uc4f8\uc4f8\ud55c \uc624\ud6c4\u3009\uac00 \uc774 \uc568\ubc94\uc5d0 \uc2e4\ub824 \uc788\ub2e4. 1988\ub144 \uae40\ud604\uc2dd\uc774 \ub300\ub9c8\ucd08 \uc0ac\uac74\uc73c\ub85c \ud65c\ub3d9\uc744 \ubabb\ud558\uac8c \ub418\uae30 \uc804\uc5d0 \uc720\uc7ac\ud558\ub294 \uadf8\ub8f9\uc744 \ub5a0\ub0ac\uace0, \uc7a5\uae30\ud638\ub294 \uc720\uc7ac\ud558 \uc790\ub9ac\ub97c \uc774\uc5c8\ub358 \ubc15\uc131\uc2dd\uacfc \ud568\uaed8 2\uc778\uc870 \ube5b\uacfc \uc18c\uae08\uc73c\ub85c \ub3c5\ub9bd\ud588\ub2e4. \uae40\uc885\uc9c4\uacfc \uc804\ud0dc\uad00\uc740 2\uc778\uc870 \ubd04\uc5ec\ub984\uac00\uc744\uaca8\uc6b8\ub85c \ubb49\ucce4\ub2e4. \uccab \uc568\ubc94 \u300a\ubd04\uc5ec\ub984\uac00\uc744\uaca8\uc6b8\u300b 1\uc9d1 \ubc1c\ub9e4. \uc218\ub85d\ub41c 10\uace1 \uc911 3\uace1\uc774 \uc5f0\uc8fc\uace1\uc73c\ub85c, \uad6d\ub0b4 \uac00\uc218 \ucd5c\ucd08\ub85c \uc5f0\uc8fc\uace1\uc774 \ud0c0\uc774\ud2c0\uace1 \uc774\uc5b4\uc11c \ud654\uc81c\uac00 \ub418\uc5c8\ub2e4. \ub300\ud45c\uace1\uc73c\ub85c\ub294 \u3008\uc0ac\ub78c\ub4e4\uc740 \ubaa8\ub450 \ubcc0\ud558\ub098\ubd10\u3009\uc640 \u3008\ud56d\uc0c1 \uae30\ubed0\ud558\ub294 \uc0ac\ub78c\ub4e4\u3009\uc774 \uc788\ub2e4.", "answer": "\ubc15\uc131\uc2dd", "sentence": "\uc720\uc7ac\ud558\uc758 \ud0c8\ud1f4 \uc774\ud6c4 \ubc15\uc131\uc2dd \uc758 \ud569\ub958\ub85c \uae40\ud604\uc2dd 3\uc9d1 \"\uae40\ud604\uc2dd\uacfc \ubd04.", "paragraph_sentence": "1986\ub144 \uae40\uc885\uc9c4, \uc804\ud0dc\uad00, \uc720\uc7ac\ud558, \uc7a5\uae30\ud638\ub294 \uc120\ubc30 \uae40\ud604\uc2dd\uacfc \ud568\uaed8 \"\uae40\ud604\uc2dd\uacfc \ubd04. \uc5ec\ub984.\uac00\uc744. \uaca8\uc6b8\"\uc744 \uacb0\uc131. \uc720\uc7ac\ud558\uc758 \ud0c8\ud1f4 \uc774\ud6c4 \ubc15\uc131\uc2dd \uc758 \ud569\ub958\ub85c \uae40\ud604\uc2dd 3\uc9d1 \"\uae40\ud604\uc2dd\uacfc \ubd04. \uc5ec\ub984.\uac00\uc744. \uaca8\uc6b8\"\uc744 \ubc1c\ud45c. \uae40\uc885\uc9c4\uc774 \uc791\uc0ac, \uc791\uace1\ud55c \u3008\uc4f8\uc4f8\ud55c \uc624\ud6c4\u3009\uac00 \uc774 \uc568\ubc94\uc5d0 \uc2e4\ub824 \uc788\ub2e4. 1988\ub144 \uae40\ud604\uc2dd\uc774 \ub300\ub9c8\ucd08 \uc0ac\uac74\uc73c\ub85c \ud65c\ub3d9\uc744 \ubabb\ud558\uac8c \ub418\uae30 \uc804\uc5d0 \uc720\uc7ac\ud558\ub294 \uadf8\ub8f9\uc744 \ub5a0\ub0ac\uace0, \uc7a5\uae30\ud638\ub294 \uc720\uc7ac\ud558 \uc790\ub9ac\ub97c \uc774\uc5c8\ub358 \ubc15\uc131\uc2dd\uacfc \ud568\uaed8 2\uc778\uc870 \ube5b\uacfc \uc18c\uae08\uc73c\ub85c \ub3c5\ub9bd\ud588\ub2e4. \uae40\uc885\uc9c4\uacfc \uc804\ud0dc\uad00\uc740 2\uc778\uc870 \ubd04\uc5ec\ub984\uac00\uc744\uaca8\uc6b8\ub85c \ubb49\ucce4\ub2e4. \uccab \uc568\ubc94 \u300a\ubd04\uc5ec\ub984\uac00\uc744\uaca8\uc6b8\u300b 1\uc9d1 \ubc1c\ub9e4. \uc218\ub85d\ub41c 10\uace1 \uc911 3\uace1\uc774 \uc5f0\uc8fc\uace1\uc73c\ub85c, \uad6d\ub0b4 \uac00\uc218 \ucd5c\ucd08\ub85c \uc5f0\uc8fc\uace1\uc774 \ud0c0\uc774\ud2c0\uace1 \uc774\uc5b4\uc11c \ud654\uc81c\uac00 \ub418\uc5c8\ub2e4. \ub300\ud45c\uace1\uc73c\ub85c\ub294 \u3008\uc0ac\ub78c\ub4e4\uc740 \ubaa8\ub450 \ubcc0\ud558\ub098\ubd10\u3009\uc640 \u3008\ud56d\uc0c1 \uae30\ubed0\ud558\ub294 \uc0ac\ub78c\ub4e4\u3009\uc774 \uc788\ub2e4.", "paragraph_answer": "1986\ub144 \uae40\uc885\uc9c4, \uc804\ud0dc\uad00, \uc720\uc7ac\ud558, \uc7a5\uae30\ud638\ub294 \uc120\ubc30 \uae40\ud604\uc2dd\uacfc \ud568\uaed8 \"\uae40\ud604\uc2dd\uacfc \ubd04.\uc5ec\ub984.\uac00\uc744.\uaca8\uc6b8\"\uc744 \uacb0\uc131. \uc720\uc7ac\ud558\uc758 \ud0c8\ud1f4 \uc774\ud6c4 \ubc15\uc131\uc2dd \uc758 \ud569\ub958\ub85c \uae40\ud604\uc2dd 3\uc9d1 \"\uae40\ud604\uc2dd\uacfc \ubd04.\uc5ec\ub984.\uac00\uc744.\uaca8\uc6b8\"\uc744 \ubc1c\ud45c. \uae40\uc885\uc9c4\uc774 \uc791\uc0ac, \uc791\uace1\ud55c \u3008\uc4f8\uc4f8\ud55c \uc624\ud6c4\u3009\uac00 \uc774 \uc568\ubc94\uc5d0 \uc2e4\ub824 \uc788\ub2e4. 1988\ub144 \uae40\ud604\uc2dd\uc774 \ub300\ub9c8\ucd08 \uc0ac\uac74\uc73c\ub85c \ud65c\ub3d9\uc744 \ubabb\ud558\uac8c \ub418\uae30 \uc804\uc5d0 \uc720\uc7ac\ud558\ub294 \uadf8\ub8f9\uc744 \ub5a0\ub0ac\uace0, \uc7a5\uae30\ud638\ub294 \uc720\uc7ac\ud558 \uc790\ub9ac\ub97c \uc774\uc5c8\ub358 \ubc15\uc131\uc2dd\uacfc \ud568\uaed8 2\uc778\uc870 \ube5b\uacfc \uc18c\uae08\uc73c\ub85c \ub3c5\ub9bd\ud588\ub2e4. \uae40\uc885\uc9c4\uacfc \uc804\ud0dc\uad00\uc740 2\uc778\uc870 \ubd04\uc5ec\ub984\uac00\uc744\uaca8\uc6b8\ub85c \ubb49\ucce4\ub2e4. \uccab \uc568\ubc94 \u300a\ubd04\uc5ec\ub984\uac00\uc744\uaca8\uc6b8\u300b 1\uc9d1 \ubc1c\ub9e4. \uc218\ub85d\ub41c 10\uace1 \uc911 3\uace1\uc774 \uc5f0\uc8fc\uace1\uc73c\ub85c, \uad6d\ub0b4 \uac00\uc218 \ucd5c\ucd08\ub85c \uc5f0\uc8fc\uace1\uc774 \ud0c0\uc774\ud2c0\uace1 \uc774\uc5b4\uc11c \ud654\uc81c\uac00 \ub418\uc5c8\ub2e4. \ub300\ud45c\uace1\uc73c\ub85c\ub294 \u3008\uc0ac\ub78c\ub4e4\uc740 \ubaa8\ub450 \ubcc0\ud558\ub098\ubd10\u3009\uc640 \u3008\ud56d\uc0c1 \uae30\ubed0\ud558\ub294 \uc0ac\ub78c\ub4e4\u3009\uc774 \uc788\ub2e4.", "sentence_answer": "\uc720\uc7ac\ud558\uc758 \ud0c8\ud1f4 \uc774\ud6c4 \ubc15\uc131\uc2dd \uc758 \ud569\ub958\ub85c \uae40\ud604\uc2dd 3\uc9d1 \"\uae40\ud604\uc2dd\uacfc \ubd04.", "paragraph_id": "6508944-0-0", "paragraph_question": "question: \uae40\ud604\uc2dd\uacfc \ubd04\uc5ec\ub984\uac00\uc6b8\uaca8\uc6b8\uc5d0 \uc720\uc7ac\ud558\ub97c \ub300\uc2e0\ud574\uc11c \ub4e4\uc5b4\uc628 \uc0ac\ub78c\uc740?, context: 1986\ub144 \uae40\uc885\uc9c4, \uc804\ud0dc\uad00, \uc720\uc7ac\ud558, \uc7a5\uae30\ud638\ub294 \uc120\ubc30 \uae40\ud604\uc2dd\uacfc \ud568\uaed8 \"\uae40\ud604\uc2dd\uacfc \ubd04.\uc5ec\ub984.\uac00\uc744.\uaca8\uc6b8\"\uc744 \uacb0\uc131. \uc720\uc7ac\ud558\uc758 \ud0c8\ud1f4 \uc774\ud6c4 \ubc15\uc131\uc2dd\uc758 \ud569\ub958\ub85c \uae40\ud604\uc2dd 3\uc9d1 \"\uae40\ud604\uc2dd\uacfc \ubd04.\uc5ec\ub984.\uac00\uc744.\uaca8\uc6b8\"\uc744 \ubc1c\ud45c. \uae40\uc885\uc9c4\uc774 \uc791\uc0ac, \uc791\uace1\ud55c \u3008\uc4f8\uc4f8\ud55c \uc624\ud6c4\u3009\uac00 \uc774 \uc568\ubc94\uc5d0 \uc2e4\ub824 \uc788\ub2e4. 1988\ub144 \uae40\ud604\uc2dd\uc774 \ub300\ub9c8\ucd08 \uc0ac\uac74\uc73c\ub85c \ud65c\ub3d9\uc744 \ubabb\ud558\uac8c \ub418\uae30 \uc804\uc5d0 \uc720\uc7ac\ud558\ub294 \uadf8\ub8f9\uc744 \ub5a0\ub0ac\uace0, \uc7a5\uae30\ud638\ub294 \uc720\uc7ac\ud558 \uc790\ub9ac\ub97c \uc774\uc5c8\ub358 \ubc15\uc131\uc2dd\uacfc \ud568\uaed8 2\uc778\uc870 \ube5b\uacfc \uc18c\uae08\uc73c\ub85c \ub3c5\ub9bd\ud588\ub2e4. \uae40\uc885\uc9c4\uacfc \uc804\ud0dc\uad00\uc740 2\uc778\uc870 \ubd04\uc5ec\ub984\uac00\uc744\uaca8\uc6b8\ub85c \ubb49\ucce4\ub2e4. \uccab \uc568\ubc94 \u300a\ubd04\uc5ec\ub984\uac00\uc744\uaca8\uc6b8\u300b 1\uc9d1 \ubc1c\ub9e4. \uc218\ub85d\ub41c 10\uace1 \uc911 3\uace1\uc774 \uc5f0\uc8fc\uace1\uc73c\ub85c, \uad6d\ub0b4 \uac00\uc218 \ucd5c\ucd08\ub85c \uc5f0\uc8fc\uace1\uc774 \ud0c0\uc774\ud2c0\uace1 \uc774\uc5b4\uc11c \ud654\uc81c\uac00 \ub418\uc5c8\ub2e4. \ub300\ud45c\uace1\uc73c\ub85c\ub294 \u3008\uc0ac\ub78c\ub4e4\uc740 \ubaa8\ub450 \ubcc0\ud558\ub098\ubd10\u3009\uc640 \u3008\ud56d\uc0c1 \uae30\ubed0\ud558\ub294 \uc0ac\ub78c\ub4e4\u3009\uc774 \uc788\ub2e4."} {"question": "\ub77c\uc0ac\ub9ac\uc694\ub294 \uc6b4\uba85\uc744 \uac70\uc5ed\ud558\uc9c0 \uc54a\uace0 \uc5b4\ub5bb\uac8c \ub300\ud558\ub294\uac00?", "paragraph": "\uc774 \uc18c\uc124\uc5d0 \ube44\uce5c \ub77c\uc0ac\ub9ac\uc694\uc758 \ubaa8\uc2b5\uc740 \ud6d7\ub0a0 \ubcf8\uaca9\uc801\uc778 \ud53c\uce74\ub808\uc2a4\ud06c \uc18c\uc124\uc758 \uc8fc\uc778\uacf5\ub4e4\uc774 \ubcf4\uc5ec\uc8fc\ub294 \uc5b4\ub461\uace0 \uc545\ud558\uace0 \ubd80\uc815\uc801\uc778 \ubaa8\uc2b5\uacfc\ub294 \ub2e4\ub974\ub2e4. \ube44\ub85d \uadf8\ub294 \uc778\uc0dd\uc744 \ubc1d\uc740 \ubaa8\uc2b5\uc73c\ub85c \ubcfc \uc218 \uc5c6\ub294, \ub20c\ub9ac\uace0 \uc5b5\uc555\ub2f9\ud558\ub294 \uc704\uce58\uc5d0 \uc788\uc5c8\uc9c0\ub9cc, \uadf8\ub798\uc11c \uc778\uc0dd\uc744 \uc5bc\ub9c8\ub4e0\uc9c0 \ubd80\uc815\uc801\uc73c\ub85c \ubcfc \uc218 \uc788\uc5c8\uc9c0\ub9cc \uadf8\ub294 \uacb0\ucf54 \uc778\uc0dd\uc744 \uc5b4\ub461\uac8c\ub9cc \ubcf4\uc9c0 \uc54a\ub294\ub2e4. \uadf8\ub294 \uadf8\uc5d0\uac8c \ub2e5\uce5c \uc6b4\uba85\uc744 \uac70\uc5ed\ud558\uc9c0 \uc54a\uace0 \ub2f4\ub2f4\ud788 \ub9de\uc774\ud55c\ub2e4. \uc0ac\ubb3c\uc744 \uadf8\ub300\ub85c \uc218\uc6a9\ud558\uba70 \uadf8\ub97c \ub458\ub7ec\uc2f8\uace0 \uc788\ub294 \ud658\uacbd \uc548\uc5d0\uc11c \uc990\uac70\uc6cc\ud558\uace0 \ub9cc\uc871\ud560 \uc904\uc744 \uc548\ub2e4. \uadf8\ub97c \uad34\ub86d\ud788\ub294 \uc6b4\uba85\uc758 \ud6a1\ud3ec\uc5d0 \ub9de\uc11c \uc2f8\uc6b0\uc9c0 \uc54a\uace0, \uc624\ud788\ub824 \uadf8\uac83\uc744 \ub098\ub984\ub300\ub85c \ud130\ub4dd\ud55c \ubc29\ubc95\uc744 \ud1b5\ud558\uc5ec \ubc1b\uc544\ub4e4\uc774\uba74\uc11c \ubb34\ub825\ud654\uc2dc\ud0a8\ub2e4. \ubd88\ud589\uc740 \uadf8\uac83\uc744 \ubd88\ud589\uc73c\ub85c \uc5ec\uae30\ub294 \uc0ac\ub78c\uc5d0\uac8c \ubd88\ud589\uc774\uc9c0 \ubd88\ud589 \uc548\uc5d0\uc11c \ud589\ubcf5\uc744 \ucc3e\uc73c\ub824\uace0 \ud558\ub294 \uc0ac\ub78c\uc5d0\uac8c\ub294 \ub354 \uc774\uc0c1 \ubd88\ud589\uc774 \ub420 \uc218 \uc5c6\ub2e4. \ub77c\uc0ac\ub9ac\uc694\ub294 \uadf8\uac00 \uc0b6\uc744 \ud1b5\ud558\uc5ec \ubc30\uc6b4 \u2018\uc21c\uc751\uc8fc\uc758(conformidad)\u2019\ub97c \ud1b5\ud574 \ubd88\uc6b4\ud55c \ud658\uacbd \uc18d\uc5d0\uc11c\ub3c4 \ubbf8\uc18c\ub97c \uc78a\uc9c0 \uc54a\uace0 \ub610 \ub3c5\uc790\ub4e4\uc744 \uc5b4\ub461\uac8c \ub9cc\ub4e4\uc9c0\ub3c4 \uc54a\ub294\ub2e4. \ub77c\uc0ac\ub9ac\uc694\uc758 \ub099\ucc9c\uc8fc\uc758\uc801 \uc778\uc0dd\uad00\uc774 \ubc14\ub85c \uc5ec\uae30\uc5d0\uc11c \ube44\ub86f\ub41c\ub2e4. \ub05d\uc5c6\uc774 \ubc00\ub824\uc624\ub294 \ubc30\uace0\ud514\uc758 \uace0\ud1b5\uc774 \uc694\uad6c\ud558\ub294 \ube44\ub3c4\ub355\uc801\uc778 \ud589\uc704\ub97c \uc81c\uc678\ud558\uace0\ub294 \uadf8\uc5d0\uac8c\uc11c \ubd80\uc815\uc801\uc778 \ubaa8\uc2b5\uc740 \uc804\ud600 \ucc3e\uc744 \uc218 \uc5c6\ub2e4.", "answer": "\ub2f4\ub2f4\ud788 \ub9de\uc774\ud55c\ub2e4", "sentence": "\uadf8\ub294 \uadf8\uc5d0\uac8c \ub2e5\uce5c \uc6b4\uba85\uc744 \uac70\uc5ed\ud558\uc9c0 \uc54a\uace0 \ub2f4\ub2f4\ud788 \ub9de\uc774\ud55c\ub2e4 .", "paragraph_sentence": "\uc774 \uc18c\uc124\uc5d0 \ube44\uce5c \ub77c\uc0ac\ub9ac\uc694\uc758 \ubaa8\uc2b5\uc740 \ud6d7\ub0a0 \ubcf8\uaca9\uc801\uc778 \ud53c\uce74\ub808\uc2a4\ud06c \uc18c\uc124\uc758 \uc8fc\uc778\uacf5\ub4e4\uc774 \ubcf4\uc5ec\uc8fc\ub294 \uc5b4\ub461\uace0 \uc545\ud558\uace0 \ubd80\uc815\uc801\uc778 \ubaa8\uc2b5\uacfc\ub294 \ub2e4\ub974\ub2e4. \ube44\ub85d \uadf8\ub294 \uc778\uc0dd\uc744 \ubc1d\uc740 \ubaa8\uc2b5\uc73c\ub85c \ubcfc \uc218 \uc5c6\ub294, \ub20c\ub9ac\uace0 \uc5b5\uc555\ub2f9\ud558\ub294 \uc704\uce58\uc5d0 \uc788\uc5c8\uc9c0\ub9cc, \uadf8\ub798\uc11c \uc778\uc0dd\uc744 \uc5bc\ub9c8\ub4e0\uc9c0 \ubd80\uc815\uc801\uc73c\ub85c \ubcfc \uc218 \uc788\uc5c8\uc9c0\ub9cc \uadf8\ub294 \uacb0\ucf54 \uc778\uc0dd\uc744 \uc5b4\ub461\uac8c\ub9cc \ubcf4\uc9c0 \uc54a\ub294\ub2e4. \uadf8\ub294 \uadf8\uc5d0\uac8c \ub2e5\uce5c \uc6b4\uba85\uc744 \uac70\uc5ed\ud558\uc9c0 \uc54a\uace0 \ub2f4\ub2f4\ud788 \ub9de\uc774\ud55c\ub2e4 . \uc0ac\ubb3c\uc744 \uadf8\ub300\ub85c \uc218\uc6a9\ud558\uba70 \uadf8\ub97c \ub458\ub7ec\uc2f8\uace0 \uc788\ub294 \ud658\uacbd \uc548\uc5d0\uc11c \uc990\uac70\uc6cc\ud558\uace0 \ub9cc\uc871\ud560 \uc904\uc744 \uc548\ub2e4. \uadf8\ub97c \uad34\ub86d\ud788\ub294 \uc6b4\uba85\uc758 \ud6a1\ud3ec\uc5d0 \ub9de\uc11c \uc2f8\uc6b0\uc9c0 \uc54a\uace0, \uc624\ud788\ub824 \uadf8\uac83\uc744 \ub098\ub984\ub300\ub85c \ud130\ub4dd\ud55c \ubc29\ubc95\uc744 \ud1b5\ud558\uc5ec \ubc1b\uc544\ub4e4\uc774\uba74\uc11c \ubb34\ub825\ud654\uc2dc\ud0a8\ub2e4. \ubd88\ud589\uc740 \uadf8\uac83\uc744 \ubd88\ud589\uc73c\ub85c \uc5ec\uae30\ub294 \uc0ac\ub78c\uc5d0\uac8c \ubd88\ud589\uc774\uc9c0 \ubd88\ud589 \uc548\uc5d0\uc11c \ud589\ubcf5\uc744 \ucc3e\uc73c\ub824\uace0 \ud558\ub294 \uc0ac\ub78c\uc5d0\uac8c\ub294 \ub354 \uc774\uc0c1 \ubd88\ud589\uc774 \ub420 \uc218 \uc5c6\ub2e4. \ub77c\uc0ac\ub9ac\uc694\ub294 \uadf8\uac00 \uc0b6\uc744 \ud1b5\ud558\uc5ec \ubc30\uc6b4 \u2018\uc21c\uc751\uc8fc\uc758(conformidad)\u2019\ub97c \ud1b5\ud574 \ubd88\uc6b4\ud55c \ud658\uacbd \uc18d\uc5d0\uc11c\ub3c4 \ubbf8\uc18c\ub97c \uc78a\uc9c0 \uc54a\uace0 \ub610 \ub3c5\uc790\ub4e4\uc744 \uc5b4\ub461\uac8c \ub9cc\ub4e4\uc9c0\ub3c4 \uc54a\ub294\ub2e4. \ub77c\uc0ac\ub9ac\uc694\uc758 \ub099\ucc9c\uc8fc\uc758\uc801 \uc778\uc0dd\uad00\uc774 \ubc14\ub85c \uc5ec\uae30\uc5d0\uc11c \ube44\ub86f\ub41c\ub2e4. \ub05d\uc5c6\uc774 \ubc00\ub824\uc624\ub294 \ubc30\uace0\ud514\uc758 \uace0\ud1b5\uc774 \uc694\uad6c\ud558\ub294 \ube44\ub3c4\ub355\uc801\uc778 \ud589\uc704\ub97c \uc81c\uc678\ud558\uace0\ub294 \uadf8\uc5d0\uac8c\uc11c \ubd80\uc815\uc801\uc778 \ubaa8\uc2b5\uc740 \uc804\ud600 \ucc3e\uc744 \uc218 \uc5c6\ub2e4.", "paragraph_answer": "\uc774 \uc18c\uc124\uc5d0 \ube44\uce5c \ub77c\uc0ac\ub9ac\uc694\uc758 \ubaa8\uc2b5\uc740 \ud6d7\ub0a0 \ubcf8\uaca9\uc801\uc778 \ud53c\uce74\ub808\uc2a4\ud06c \uc18c\uc124\uc758 \uc8fc\uc778\uacf5\ub4e4\uc774 \ubcf4\uc5ec\uc8fc\ub294 \uc5b4\ub461\uace0 \uc545\ud558\uace0 \ubd80\uc815\uc801\uc778 \ubaa8\uc2b5\uacfc\ub294 \ub2e4\ub974\ub2e4. \ube44\ub85d \uadf8\ub294 \uc778\uc0dd\uc744 \ubc1d\uc740 \ubaa8\uc2b5\uc73c\ub85c \ubcfc \uc218 \uc5c6\ub294, \ub20c\ub9ac\uace0 \uc5b5\uc555\ub2f9\ud558\ub294 \uc704\uce58\uc5d0 \uc788\uc5c8\uc9c0\ub9cc, \uadf8\ub798\uc11c \uc778\uc0dd\uc744 \uc5bc\ub9c8\ub4e0\uc9c0 \ubd80\uc815\uc801\uc73c\ub85c \ubcfc \uc218 \uc788\uc5c8\uc9c0\ub9cc \uadf8\ub294 \uacb0\ucf54 \uc778\uc0dd\uc744 \uc5b4\ub461\uac8c\ub9cc \ubcf4\uc9c0 \uc54a\ub294\ub2e4. \uadf8\ub294 \uadf8\uc5d0\uac8c \ub2e5\uce5c \uc6b4\uba85\uc744 \uac70\uc5ed\ud558\uc9c0 \uc54a\uace0 \ub2f4\ub2f4\ud788 \ub9de\uc774\ud55c\ub2e4 . \uc0ac\ubb3c\uc744 \uadf8\ub300\ub85c \uc218\uc6a9\ud558\uba70 \uadf8\ub97c \ub458\ub7ec\uc2f8\uace0 \uc788\ub294 \ud658\uacbd \uc548\uc5d0\uc11c \uc990\uac70\uc6cc\ud558\uace0 \ub9cc\uc871\ud560 \uc904\uc744 \uc548\ub2e4. \uadf8\ub97c \uad34\ub86d\ud788\ub294 \uc6b4\uba85\uc758 \ud6a1\ud3ec\uc5d0 \ub9de\uc11c \uc2f8\uc6b0\uc9c0 \uc54a\uace0, \uc624\ud788\ub824 \uadf8\uac83\uc744 \ub098\ub984\ub300\ub85c \ud130\ub4dd\ud55c \ubc29\ubc95\uc744 \ud1b5\ud558\uc5ec \ubc1b\uc544\ub4e4\uc774\uba74\uc11c \ubb34\ub825\ud654\uc2dc\ud0a8\ub2e4. \ubd88\ud589\uc740 \uadf8\uac83\uc744 \ubd88\ud589\uc73c\ub85c \uc5ec\uae30\ub294 \uc0ac\ub78c\uc5d0\uac8c \ubd88\ud589\uc774\uc9c0 \ubd88\ud589 \uc548\uc5d0\uc11c \ud589\ubcf5\uc744 \ucc3e\uc73c\ub824\uace0 \ud558\ub294 \uc0ac\ub78c\uc5d0\uac8c\ub294 \ub354 \uc774\uc0c1 \ubd88\ud589\uc774 \ub420 \uc218 \uc5c6\ub2e4. \ub77c\uc0ac\ub9ac\uc694\ub294 \uadf8\uac00 \uc0b6\uc744 \ud1b5\ud558\uc5ec \ubc30\uc6b4 \u2018\uc21c\uc751\uc8fc\uc758(conformidad)\u2019\ub97c \ud1b5\ud574 \ubd88\uc6b4\ud55c \ud658\uacbd \uc18d\uc5d0\uc11c\ub3c4 \ubbf8\uc18c\ub97c \uc78a\uc9c0 \uc54a\uace0 \ub610 \ub3c5\uc790\ub4e4\uc744 \uc5b4\ub461\uac8c \ub9cc\ub4e4\uc9c0\ub3c4 \uc54a\ub294\ub2e4. \ub77c\uc0ac\ub9ac\uc694\uc758 \ub099\ucc9c\uc8fc\uc758\uc801 \uc778\uc0dd\uad00\uc774 \ubc14\ub85c \uc5ec\uae30\uc5d0\uc11c \ube44\ub86f\ub41c\ub2e4. \ub05d\uc5c6\uc774 \ubc00\ub824\uc624\ub294 \ubc30\uace0\ud514\uc758 \uace0\ud1b5\uc774 \uc694\uad6c\ud558\ub294 \ube44\ub3c4\ub355\uc801\uc778 \ud589\uc704\ub97c \uc81c\uc678\ud558\uace0\ub294 \uadf8\uc5d0\uac8c\uc11c \ubd80\uc815\uc801\uc778 \ubaa8\uc2b5\uc740 \uc804\ud600 \ucc3e\uc744 \uc218 \uc5c6\ub2e4.", "sentence_answer": "\uadf8\ub294 \uadf8\uc5d0\uac8c \ub2e5\uce5c \uc6b4\uba85\uc744 \uac70\uc5ed\ud558\uc9c0 \uc54a\uace0 \ub2f4\ub2f4\ud788 \ub9de\uc774\ud55c\ub2e4 .", "paragraph_id": "6513320-8-1", "paragraph_question": "question: \ub77c\uc0ac\ub9ac\uc694\ub294 \uc6b4\uba85\uc744 \uac70\uc5ed\ud558\uc9c0 \uc54a\uace0 \uc5b4\ub5bb\uac8c \ub300\ud558\ub294\uac00?, context: \uc774 \uc18c\uc124\uc5d0 \ube44\uce5c \ub77c\uc0ac\ub9ac\uc694\uc758 \ubaa8\uc2b5\uc740 \ud6d7\ub0a0 \ubcf8\uaca9\uc801\uc778 \ud53c\uce74\ub808\uc2a4\ud06c \uc18c\uc124\uc758 \uc8fc\uc778\uacf5\ub4e4\uc774 \ubcf4\uc5ec\uc8fc\ub294 \uc5b4\ub461\uace0 \uc545\ud558\uace0 \ubd80\uc815\uc801\uc778 \ubaa8\uc2b5\uacfc\ub294 \ub2e4\ub974\ub2e4. \ube44\ub85d \uadf8\ub294 \uc778\uc0dd\uc744 \ubc1d\uc740 \ubaa8\uc2b5\uc73c\ub85c \ubcfc \uc218 \uc5c6\ub294, \ub20c\ub9ac\uace0 \uc5b5\uc555\ub2f9\ud558\ub294 \uc704\uce58\uc5d0 \uc788\uc5c8\uc9c0\ub9cc, \uadf8\ub798\uc11c \uc778\uc0dd\uc744 \uc5bc\ub9c8\ub4e0\uc9c0 \ubd80\uc815\uc801\uc73c\ub85c \ubcfc \uc218 \uc788\uc5c8\uc9c0\ub9cc \uadf8\ub294 \uacb0\ucf54 \uc778\uc0dd\uc744 \uc5b4\ub461\uac8c\ub9cc \ubcf4\uc9c0 \uc54a\ub294\ub2e4. \uadf8\ub294 \uadf8\uc5d0\uac8c \ub2e5\uce5c \uc6b4\uba85\uc744 \uac70\uc5ed\ud558\uc9c0 \uc54a\uace0 \ub2f4\ub2f4\ud788 \ub9de\uc774\ud55c\ub2e4. \uc0ac\ubb3c\uc744 \uadf8\ub300\ub85c \uc218\uc6a9\ud558\uba70 \uadf8\ub97c \ub458\ub7ec\uc2f8\uace0 \uc788\ub294 \ud658\uacbd \uc548\uc5d0\uc11c \uc990\uac70\uc6cc\ud558\uace0 \ub9cc\uc871\ud560 \uc904\uc744 \uc548\ub2e4. \uadf8\ub97c \uad34\ub86d\ud788\ub294 \uc6b4\uba85\uc758 \ud6a1\ud3ec\uc5d0 \ub9de\uc11c \uc2f8\uc6b0\uc9c0 \uc54a\uace0, \uc624\ud788\ub824 \uadf8\uac83\uc744 \ub098\ub984\ub300\ub85c \ud130\ub4dd\ud55c \ubc29\ubc95\uc744 \ud1b5\ud558\uc5ec \ubc1b\uc544\ub4e4\uc774\uba74\uc11c \ubb34\ub825\ud654\uc2dc\ud0a8\ub2e4. \ubd88\ud589\uc740 \uadf8\uac83\uc744 \ubd88\ud589\uc73c\ub85c \uc5ec\uae30\ub294 \uc0ac\ub78c\uc5d0\uac8c \ubd88\ud589\uc774\uc9c0 \ubd88\ud589 \uc548\uc5d0\uc11c \ud589\ubcf5\uc744 \ucc3e\uc73c\ub824\uace0 \ud558\ub294 \uc0ac\ub78c\uc5d0\uac8c\ub294 \ub354 \uc774\uc0c1 \ubd88\ud589\uc774 \ub420 \uc218 \uc5c6\ub2e4. \ub77c\uc0ac\ub9ac\uc694\ub294 \uadf8\uac00 \uc0b6\uc744 \ud1b5\ud558\uc5ec \ubc30\uc6b4 \u2018\uc21c\uc751\uc8fc\uc758(conformidad)\u2019\ub97c \ud1b5\ud574 \ubd88\uc6b4\ud55c \ud658\uacbd \uc18d\uc5d0\uc11c\ub3c4 \ubbf8\uc18c\ub97c \uc78a\uc9c0 \uc54a\uace0 \ub610 \ub3c5\uc790\ub4e4\uc744 \uc5b4\ub461\uac8c \ub9cc\ub4e4\uc9c0\ub3c4 \uc54a\ub294\ub2e4. \ub77c\uc0ac\ub9ac\uc694\uc758 \ub099\ucc9c\uc8fc\uc758\uc801 \uc778\uc0dd\uad00\uc774 \ubc14\ub85c \uc5ec\uae30\uc5d0\uc11c \ube44\ub86f\ub41c\ub2e4. \ub05d\uc5c6\uc774 \ubc00\ub824\uc624\ub294 \ubc30\uace0\ud514\uc758 \uace0\ud1b5\uc774 \uc694\uad6c\ud558\ub294 \ube44\ub3c4\ub355\uc801\uc778 \ud589\uc704\ub97c \uc81c\uc678\ud558\uace0\ub294 \uadf8\uc5d0\uac8c\uc11c \ubd80\uc815\uc801\uc778 \ubaa8\uc2b5\uc740 \uc804\ud600 \ucc3e\uc744 \uc218 \uc5c6\ub2e4."} {"question": "\uc624\ub2e4\uc640\ub77c \uc815\ubc8c\uc740 \uba87 \ub144\ub3c4\uc5d0 \uc77c\uc5b4\ub0ac\ub294\uac00?", "paragraph": "\ub3c4\uc694\ud1a0\ubbf8 \uac00\ubb38\uc740 1590\ub144 \uc624\ub2e4\uc640\ub77c \uc815\ubc8c\uc744 \uacc4\uae30\ub85c \uc13c\uace0\ucfe0 \uc2dc\ub300\ub97c \uc885\uacb0\uc2dc\ud0a4\uba70 \uc77c\ubcf8\uc744 \uc0ac\uc2e4\uc0c1 \ud1b5\uce58\ud558\ub294 \uac00\ubb38\uc758 \uc704\uce58\uae4c\uc9c0 \uc62c\ub790\ub2e4. \uadf8\ub7ec\ub098 \ub3c4\uc694\ud1a0\ubbf8 \uc528\uc758 \ud655\ub300\ub294 \ud788\ub370\uc694\uc2dc\uac00 \uac1c\uc778\uc758 \uad8c\ub825\uc73c\ub85c \uad00\uc704 \uc11c\uc784\uad8c\uc744 \ub3c5\uc810\ud558\ub294 \ub3d9\uc2dc\uc5d0 \uad00\uc704 \uc11c\uc784 \ubb38\uc11c\uc758 \ub0b4\uc6a9\uc744 \ub9c8\uc74c\ub300\ub85c \uace0\uce60 \uc218\ub3c4 \uc788\uc5c8\ub358 \ub370\uc11c \ube44\ub86f\ub418\uc5c8\uace0, \uac8c\uc774\ucd08(\u6176\u9577) 3\ub144(1598\ub144)\uc5d0 \ud788\ub370\uc694\uc2dc\uac00 \uc8fd\uc740 \ub4a4\uc5d0\ub294 \ub2f9\uc5f0\ud558\uac8c\ub3c4 \uadf8 \uc218\uac00 \uc904\uc5b4\ub4e4\uc5c8\ub2e4. \ub3c4\ucfe0\uac00\uc640 \uc774\uc5d0\uc57c\uc2a4(\u5fb3\u5ddd\u5bb6\u5eb7)\uc640 \uadf8 \uc77c\ubb38\uc774 \u300c\ud558\uc2dc\ubc14\u300d\ub77c\ub294 \ubb18\uc9c0(\u540d\u5b57)\uc640 \u300c\ub3c4\uc694\ud1a0\ubbf8\u300d\ub77c\ub294 \uc6b0\uc9c0\ub97c \uc4f0\ub294 \uac83\uc744 \uadf8\ub9cc\ub450\uace0, \uac8c\uc774\ucd08 8\ub144(1603\ub144)\uc5d0\ub294 \uc774\uc5d0\uc57c\uc2a4\uac00 \u300c\ub2db\ud0c0\u300d(\u65b0\u7530) \u300c\ub3c4\ucfe0\uac00\uc640\u300d \ub4f1\uc758 \ubb18\uc9c0\ub97c \uc0ac\uc6a9\ud574 \u300c\ubbf8\ub098\ubaa8\ud1a0\ub178 \uc544\uc190 \uc774\uc5d0\uc57c\uc2a4\u300d(\u6e90\u671d\u81e3\u5bb6\u5eb7)\ub85c\uc368 \uc138\uc774\uc774\ud0c0\uc774\uc1fc\uad70(\u5f81\u5937\u5927\u5c06\u8ecd)\uc774 \ub41c \uac83\uc740 \ub110\ub9ac \uc54c\ub824\uc838 \uc788\ub2e4. \uadf8\ub7ec\ub098 \uc774\uc5d0\uc57c\uc2a4\ub294 \uc774 \ub2e8\uacc4\uc5d0\uc11c \ub610\ud55c \uc0dd\uc804 \ud788\ub370\uc694\uc2dc\uac00 \ubc30\ud0c0\uc801\uc73c\ub85c \uad00\uc704 \uc11c\uc784\uad8c\uc744 \ub3c5\uc810\ud558\ub358 \uac83\uc744 \ub2f5\uc2b5\ud558\uc9c0\ub294 \uc54a\uc558\uace0, \ud788\ub370\uc694\uc2dc\uc758 \ub4a4\ub97c \uc774\uc5b4 \ud558\uc2dc\ubc14 \uc885\uac00(\u5b97\u5bb6)\uc758 \ub2f9\uc8fc\uac00 \ub41c \ud788\ub370\uc694\ub9ac(\u79c0\u983c)\uac00 \uc624\uc0ac\uce74 \uc131(\u5927\u5742\u57ce)\uc5d0\uc11c \uadf8 \uc790\uc2e0\uc758 \uc9c1\uc18d \uac00\uc2e0\uc5d0 \ub300\ud55c \uad00\uc704 \uc11c\uc784\uc744 \ubcc0\ud568\uc5c6\uc774 \ub3c5\uc790\uc801\uc73c\ub85c \uc774\uc5b4\uac14\ub2e4.", "answer": "1590\ub144", "sentence": "\ub3c4\uc694\ud1a0\ubbf8 \uac00\ubb38\uc740 1590\ub144 \uc624\ub2e4\uc640\ub77c \uc815\ubc8c\uc744 \uacc4\uae30\ub85c \uc13c\uace0\ucfe0 \uc2dc\ub300\ub97c \uc885\uacb0\uc2dc\ud0a4\uba70 \uc77c\ubcf8\uc744 \uc0ac\uc2e4\uc0c1 \ud1b5\uce58\ud558\ub294 \uac00\ubb38\uc758 \uc704\uce58\uae4c\uc9c0 \uc62c\ub790\ub2e4.", "paragraph_sentence": " \ub3c4\uc694\ud1a0\ubbf8 \uac00\ubb38\uc740 1590\ub144 \uc624\ub2e4\uc640\ub77c \uc815\ubc8c\uc744 \uacc4\uae30\ub85c \uc13c\uace0\ucfe0 \uc2dc\ub300\ub97c \uc885\uacb0\uc2dc\ud0a4\uba70 \uc77c\ubcf8\uc744 \uc0ac\uc2e4\uc0c1 \ud1b5\uce58\ud558\ub294 \uac00\ubb38\uc758 \uc704\uce58\uae4c\uc9c0 \uc62c\ub790\ub2e4. \uadf8\ub7ec\ub098 \ub3c4\uc694\ud1a0\ubbf8 \uc528\uc758 \ud655\ub300\ub294 \ud788\ub370\uc694\uc2dc\uac00 \uac1c\uc778\uc758 \uad8c\ub825\uc73c\ub85c \uad00\uc704 \uc11c\uc784\uad8c\uc744 \ub3c5\uc810\ud558\ub294 \ub3d9\uc2dc\uc5d0 \uad00\uc704 \uc11c\uc784 \ubb38\uc11c\uc758 \ub0b4\uc6a9\uc744 \ub9c8\uc74c\ub300\ub85c \uace0\uce60 \uc218\ub3c4 \uc788\uc5c8\ub358 \ub370\uc11c \ube44\ub86f\ub418\uc5c8\uace0, \uac8c\uc774\ucd08(\u6176\u9577) 3\ub144(1598\ub144)\uc5d0 \ud788\ub370\uc694\uc2dc\uac00 \uc8fd\uc740 \ub4a4\uc5d0\ub294 \ub2f9\uc5f0\ud558\uac8c\ub3c4 \uadf8 \uc218\uac00 \uc904\uc5b4\ub4e4\uc5c8\ub2e4. \ub3c4\ucfe0\uac00\uc640 \uc774\uc5d0\uc57c\uc2a4(\u5fb3\u5ddd\u5bb6\u5eb7)\uc640 \uadf8 \uc77c\ubb38\uc774 \u300c\ud558\uc2dc\ubc14\u300d\ub77c\ub294 \ubb18\uc9c0(\u540d\u5b57)\uc640 \u300c\ub3c4\uc694\ud1a0\ubbf8\u300d\ub77c\ub294 \uc6b0\uc9c0\ub97c \uc4f0\ub294 \uac83\uc744 \uadf8\ub9cc\ub450\uace0, \uac8c\uc774\ucd08 8\ub144(1603\ub144)\uc5d0\ub294 \uc774\uc5d0\uc57c\uc2a4\uac00 \u300c\ub2db\ud0c0\u300d(\u65b0\u7530) \u300c\ub3c4\ucfe0\uac00\uc640\u300d \ub4f1\uc758 \ubb18\uc9c0\ub97c \uc0ac\uc6a9\ud574 \u300c\ubbf8\ub098\ubaa8\ud1a0\ub178 \uc544\uc190 \uc774\uc5d0\uc57c\uc2a4\u300d(\u6e90\u671d\u81e3\u5bb6\u5eb7)\ub85c\uc368 \uc138\uc774\uc774\ud0c0\uc774\uc1fc\uad70(\u5f81\u5937\u5927\u5c06\u8ecd)\uc774 \ub41c \uac83\uc740 \ub110\ub9ac \uc54c\ub824\uc838 \uc788\ub2e4. \uadf8\ub7ec\ub098 \uc774\uc5d0\uc57c\uc2a4\ub294 \uc774 \ub2e8\uacc4\uc5d0\uc11c \ub610\ud55c \uc0dd\uc804 \ud788\ub370\uc694\uc2dc\uac00 \ubc30\ud0c0\uc801\uc73c\ub85c \uad00\uc704 \uc11c\uc784\uad8c\uc744 \ub3c5\uc810\ud558\ub358 \uac83\uc744 \ub2f5\uc2b5\ud558\uc9c0\ub294 \uc54a\uc558\uace0, \ud788\ub370\uc694\uc2dc\uc758 \ub4a4\ub97c \uc774\uc5b4 \ud558\uc2dc\ubc14 \uc885\uac00(\u5b97\u5bb6)\uc758 \ub2f9\uc8fc\uac00 \ub41c \ud788\ub370\uc694\ub9ac(\u79c0\u983c)\uac00 \uc624\uc0ac\uce74 \uc131(\u5927\u5742\u57ce)\uc5d0\uc11c \uadf8 \uc790\uc2e0\uc758 \uc9c1\uc18d \uac00\uc2e0\uc5d0 \ub300\ud55c \uad00\uc704 \uc11c\uc784\uc744 \ubcc0\ud568\uc5c6\uc774 \ub3c5\uc790\uc801\uc73c\ub85c \uc774\uc5b4\uac14\ub2e4.", "paragraph_answer": "\ub3c4\uc694\ud1a0\ubbf8 \uac00\ubb38\uc740 1590\ub144 \uc624\ub2e4\uc640\ub77c \uc815\ubc8c\uc744 \uacc4\uae30\ub85c \uc13c\uace0\ucfe0 \uc2dc\ub300\ub97c \uc885\uacb0\uc2dc\ud0a4\uba70 \uc77c\ubcf8\uc744 \uc0ac\uc2e4\uc0c1 \ud1b5\uce58\ud558\ub294 \uac00\ubb38\uc758 \uc704\uce58\uae4c\uc9c0 \uc62c\ub790\ub2e4. \uadf8\ub7ec\ub098 \ub3c4\uc694\ud1a0\ubbf8 \uc528\uc758 \ud655\ub300\ub294 \ud788\ub370\uc694\uc2dc\uac00 \uac1c\uc778\uc758 \uad8c\ub825\uc73c\ub85c \uad00\uc704 \uc11c\uc784\uad8c\uc744 \ub3c5\uc810\ud558\ub294 \ub3d9\uc2dc\uc5d0 \uad00\uc704 \uc11c\uc784 \ubb38\uc11c\uc758 \ub0b4\uc6a9\uc744 \ub9c8\uc74c\ub300\ub85c \uace0\uce60 \uc218\ub3c4 \uc788\uc5c8\ub358 \ub370\uc11c \ube44\ub86f\ub418\uc5c8\uace0, \uac8c\uc774\ucd08(\u6176\u9577) 3\ub144(1598\ub144)\uc5d0 \ud788\ub370\uc694\uc2dc\uac00 \uc8fd\uc740 \ub4a4\uc5d0\ub294 \ub2f9\uc5f0\ud558\uac8c\ub3c4 \uadf8 \uc218\uac00 \uc904\uc5b4\ub4e4\uc5c8\ub2e4. \ub3c4\ucfe0\uac00\uc640 \uc774\uc5d0\uc57c\uc2a4(\u5fb3\u5ddd\u5bb6\u5eb7)\uc640 \uadf8 \uc77c\ubb38\uc774 \u300c\ud558\uc2dc\ubc14\u300d\ub77c\ub294 \ubb18\uc9c0(\u540d\u5b57)\uc640 \u300c\ub3c4\uc694\ud1a0\ubbf8\u300d\ub77c\ub294 \uc6b0\uc9c0\ub97c \uc4f0\ub294 \uac83\uc744 \uadf8\ub9cc\ub450\uace0, \uac8c\uc774\ucd08 8\ub144(1603\ub144)\uc5d0\ub294 \uc774\uc5d0\uc57c\uc2a4\uac00 \u300c\ub2db\ud0c0\u300d(\u65b0\u7530) \u300c\ub3c4\ucfe0\uac00\uc640\u300d \ub4f1\uc758 \ubb18\uc9c0\ub97c \uc0ac\uc6a9\ud574 \u300c\ubbf8\ub098\ubaa8\ud1a0\ub178 \uc544\uc190 \uc774\uc5d0\uc57c\uc2a4\u300d(\u6e90\u671d\u81e3\u5bb6\u5eb7)\ub85c\uc368 \uc138\uc774\uc774\ud0c0\uc774\uc1fc\uad70(\u5f81\u5937\u5927\u5c06\u8ecd)\uc774 \ub41c \uac83\uc740 \ub110\ub9ac \uc54c\ub824\uc838 \uc788\ub2e4. \uadf8\ub7ec\ub098 \uc774\uc5d0\uc57c\uc2a4\ub294 \uc774 \ub2e8\uacc4\uc5d0\uc11c \ub610\ud55c \uc0dd\uc804 \ud788\ub370\uc694\uc2dc\uac00 \ubc30\ud0c0\uc801\uc73c\ub85c \uad00\uc704 \uc11c\uc784\uad8c\uc744 \ub3c5\uc810\ud558\ub358 \uac83\uc744 \ub2f5\uc2b5\ud558\uc9c0\ub294 \uc54a\uc558\uace0, \ud788\ub370\uc694\uc2dc\uc758 \ub4a4\ub97c \uc774\uc5b4 \ud558\uc2dc\ubc14 \uc885\uac00(\u5b97\u5bb6)\uc758 \ub2f9\uc8fc\uac00 \ub41c \ud788\ub370\uc694\ub9ac(\u79c0\u983c)\uac00 \uc624\uc0ac\uce74 \uc131(\u5927\u5742\u57ce)\uc5d0\uc11c \uadf8 \uc790\uc2e0\uc758 \uc9c1\uc18d \uac00\uc2e0\uc5d0 \ub300\ud55c \uad00\uc704 \uc11c\uc784\uc744 \ubcc0\ud568\uc5c6\uc774 \ub3c5\uc790\uc801\uc73c\ub85c \uc774\uc5b4\uac14\ub2e4.", "sentence_answer": "\ub3c4\uc694\ud1a0\ubbf8 \uac00\ubb38\uc740 1590\ub144 \uc624\ub2e4\uc640\ub77c \uc815\ubc8c\uc744 \uacc4\uae30\ub85c \uc13c\uace0\ucfe0 \uc2dc\ub300\ub97c \uc885\uacb0\uc2dc\ud0a4\uba70 \uc77c\ubcf8\uc744 \uc0ac\uc2e4\uc0c1 \ud1b5\uce58\ud558\ub294 \uac00\ubb38\uc758 \uc704\uce58\uae4c\uc9c0 \uc62c\ub790\ub2e4.", "paragraph_id": "6295884-3-0", "paragraph_question": "question: \uc624\ub2e4\uc640\ub77c \uc815\ubc8c\uc740 \uba87 \ub144\ub3c4\uc5d0 \uc77c\uc5b4\ub0ac\ub294\uac00?, context: \ub3c4\uc694\ud1a0\ubbf8 \uac00\ubb38\uc740 1590\ub144 \uc624\ub2e4\uc640\ub77c \uc815\ubc8c\uc744 \uacc4\uae30\ub85c \uc13c\uace0\ucfe0 \uc2dc\ub300\ub97c \uc885\uacb0\uc2dc\ud0a4\uba70 \uc77c\ubcf8\uc744 \uc0ac\uc2e4\uc0c1 \ud1b5\uce58\ud558\ub294 \uac00\ubb38\uc758 \uc704\uce58\uae4c\uc9c0 \uc62c\ub790\ub2e4. \uadf8\ub7ec\ub098 \ub3c4\uc694\ud1a0\ubbf8 \uc528\uc758 \ud655\ub300\ub294 \ud788\ub370\uc694\uc2dc\uac00 \uac1c\uc778\uc758 \uad8c\ub825\uc73c\ub85c \uad00\uc704 \uc11c\uc784\uad8c\uc744 \ub3c5\uc810\ud558\ub294 \ub3d9\uc2dc\uc5d0 \uad00\uc704 \uc11c\uc784 \ubb38\uc11c\uc758 \ub0b4\uc6a9\uc744 \ub9c8\uc74c\ub300\ub85c \uace0\uce60 \uc218\ub3c4 \uc788\uc5c8\ub358 \ub370\uc11c \ube44\ub86f\ub418\uc5c8\uace0, \uac8c\uc774\ucd08(\u6176\u9577) 3\ub144(1598\ub144)\uc5d0 \ud788\ub370\uc694\uc2dc\uac00 \uc8fd\uc740 \ub4a4\uc5d0\ub294 \ub2f9\uc5f0\ud558\uac8c\ub3c4 \uadf8 \uc218\uac00 \uc904\uc5b4\ub4e4\uc5c8\ub2e4. \ub3c4\ucfe0\uac00\uc640 \uc774\uc5d0\uc57c\uc2a4(\u5fb3\u5ddd\u5bb6\u5eb7)\uc640 \uadf8 \uc77c\ubb38\uc774 \u300c\ud558\uc2dc\ubc14\u300d\ub77c\ub294 \ubb18\uc9c0(\u540d\u5b57)\uc640 \u300c\ub3c4\uc694\ud1a0\ubbf8\u300d\ub77c\ub294 \uc6b0\uc9c0\ub97c \uc4f0\ub294 \uac83\uc744 \uadf8\ub9cc\ub450\uace0, \uac8c\uc774\ucd08 8\ub144(1603\ub144)\uc5d0\ub294 \uc774\uc5d0\uc57c\uc2a4\uac00 \u300c\ub2db\ud0c0\u300d(\u65b0\u7530) \u300c\ub3c4\ucfe0\uac00\uc640\u300d \ub4f1\uc758 \ubb18\uc9c0\ub97c \uc0ac\uc6a9\ud574 \u300c\ubbf8\ub098\ubaa8\ud1a0\ub178 \uc544\uc190 \uc774\uc5d0\uc57c\uc2a4\u300d(\u6e90\u671d\u81e3\u5bb6\u5eb7)\ub85c\uc368 \uc138\uc774\uc774\ud0c0\uc774\uc1fc\uad70(\u5f81\u5937\u5927\u5c06\u8ecd)\uc774 \ub41c \uac83\uc740 \ub110\ub9ac \uc54c\ub824\uc838 \uc788\ub2e4. \uadf8\ub7ec\ub098 \uc774\uc5d0\uc57c\uc2a4\ub294 \uc774 \ub2e8\uacc4\uc5d0\uc11c \ub610\ud55c \uc0dd\uc804 \ud788\ub370\uc694\uc2dc\uac00 \ubc30\ud0c0\uc801\uc73c\ub85c \uad00\uc704 \uc11c\uc784\uad8c\uc744 \ub3c5\uc810\ud558\ub358 \uac83\uc744 \ub2f5\uc2b5\ud558\uc9c0\ub294 \uc54a\uc558\uace0, \ud788\ub370\uc694\uc2dc\uc758 \ub4a4\ub97c \uc774\uc5b4 \ud558\uc2dc\ubc14 \uc885\uac00(\u5b97\u5bb6)\uc758 \ub2f9\uc8fc\uac00 \ub41c \ud788\ub370\uc694\ub9ac(\u79c0\u983c)\uac00 \uc624\uc0ac\uce74 \uc131(\u5927\u5742\u57ce)\uc5d0\uc11c \uadf8 \uc790\uc2e0\uc758 \uc9c1\uc18d \uac00\uc2e0\uc5d0 \ub300\ud55c \uad00\uc704 \uc11c\uc784\uc744 \ubcc0\ud568\uc5c6\uc774 \ub3c5\uc790\uc801\uc73c\ub85c \uc774\uc5b4\uac14\ub2e4."} {"question": "\uae40\uadfc\ud0dc\uac00 \uace0\ubb38\uc744 \ub2f9\ud55c \uc77c\uc744 \uc18c\uc7ac\ub85c\ud55c \uc601\ud654 \uc81c\ubaa9\uc740?", "paragraph": "\uad70\uc0ac\ub3c5\uc7ac \uc2dc\uae30 \ub0a8\uc601\ub3d9 \ub300\uacf5\ubd84\uc2e4\uc5d0\uc11c\ub294 \ubbfc\uc8fc\ud654 \uc6b4\ub3d9 \uc778\uc0ac\uc5d0 \ub300\ud55c \uace0\ubb38\uc774 \uc790\ud589\ub418\uc5c8\ub2e4. \ucde8\uc870\uc2e4\uc758 \uad6c\uc870 \uc790\uccb4\uac00 24 \uc2dc\uac04 \ub0b4\ub0b4 \ud53c\uc758\uc790\ub97c \uac10\uc2dc\ud560 \uc218 \uc788\ub294 \uad6c\uc870\ub85c \ub418\uc5b4 \uc788\uace0, \ubcc0\ud638\uc778 \uc811\uacac\uad8c\uacfc \uac19\uc740 \uae30\ubcf8\uc801\uc778 \uc778\uad8c\ub3c4 \uc9c0\ucf1c\uc9c0\uc9c0 \uc54a\uc740 \uc0c1\ud0dc\uc5d0\uc11c \uac00\ud639\ud55c \uace0\ubb38\uc774 \uc774\uc5b4\uc84c\ub2e4. 1985\ub144 9\uc6d4 \ub2f9\uc2dc \ubbfc\uc8fc\ud654\uc6b4\ub3d9\uccad\ub144\uc5f0\ud569\uc758 \uc758\uc7a5\uc774\uc5c8\ub358 \uae40\uadfc\ud0dc\ub294 \ub0a8\uc601\ub3d9 \ub300\uacf5\ubd84\uc2e4 511\ud638\uc5d0\uc11c \uace0\ubb38\uae30\uc220\uc790 \uc774\uadfc\uc548\uc5d0\uac8c \uc804\uae30\uace0\ubb38\uacfc \ubb3c\uace0\ubb38\uc744 \ub2f9\ud558\uc600\ub2e4. \uae40\uadfc\ud0dc\uac00 \uace0\ubb38\uc744 \ub2f9\ud55c \uc77c\uc744 \uc18c\uc7ac\ub85c\ud55c \uc601\ud654 \u300a\ub0a8\uc601\ub3d91985\u300b\uac00 2012\ub144 \uac1c\ubd09\ub418\uc5c8\ub2e4. \ub0a8\uc601\ub3d9 \ub300\uacf5\ubd84\uc2e4\uc774 \uc138\uc0c1\uc5d0 \ub110\ub9ac \uc54c\ub824\uc9c0\uac8c \ub41c \uacc4\uae30\ub294 1987\ub144 1\uc6d4 \uc11c\uc6b8\ub300\ud559\uad50 \ud559\uc0dd\uc774\ub358 \ubc15\uc885\ucca0\uc774 \ucc38\uace0\uc778 \uc9c4\uc220\uc744 \uba85\ubaa9\uc73c\ub85c \uc5f0\ud589\ub418\uc5b4 \uace0\ubb38\uc744 \ubc1b\uace0 \uc0ac\ub9dd\ud55c \uc0ac\uac74 \ub54c\ubb38\uc774\ub2e4. \ubc15\uc885\ucca0\uc740 1987\ub144 1\uc6d4 14\uc77c 509\ud638 \uc870\uc0ac\uc2e4\uc5d0\uc11c \ud3ed\ud589\uacfc \uc804\uae30\uace0\ubb38, \ubb3c\uace0\ubb38\uc744 \ub2f9\ud558\ub2e4 \uc228\uc84c\ub2e4. \ub2f9\uc2dc \uc804\ub450\ud658 \uc815\uad8c\uc740 \uace0\ubb38 \uc0ac\uc2e4\uc744 \uc228\uae30\uae30 \uc704\ud574 \"\ud0c1\ud558\uace0 \uce58\ub2c8 \uc5b5\ud558\uace0 \uc8fd\uc5c8\ub2e4\"\ub294 \ubcc0\uba85\uc744 \ub0b4\ub193\uc544 \ub9ce\uc740 \uc0ac\ub78c\ub4e4\uc774 \ubd84\ub178\ud558\uc600\ub2e4. \uc774 \uc0ac\uac74\uc740 \ubd80\uac80\uc758\uc600\ub358 \uc624\uc5f0\uc0c1\uc774 \ubb3c\uace0\ubb38\uc774 \uc788\uc5c8\uc74c\uc744 \uc554\uc2dc\ud558\ub294 \uc0ac\uc778\uc744 \uae30\uc790\uc5d0\uac8c \uc54c\ub9ac\uba74\uc11c \uc9c4\uc0c1\uc774 \ub4e4\uc5b4\ub098\uae30 \uc2dc\uc791\ud558\uc600\uace0, \uc774\ud6c4 \ub2f9\uad6d\uc774 \ucd95\uc18c \uc740\ud3d0\ub97c \uc2dc\ub3c4\ud558\uc5ec \uc218\uc0ac\uad00 \ub458\ub9cc\uc774 \ub2e8\ub3c5\uc73c\ub85c \uc800\uc9c0\ub978 \uc77c\ub85c \ubab0\uc544\uac00\ub824\uace0 \ud558\uc790 \uc11c\uc6b8\uc601\ub4f1\ud3ec\uad6c\uce58\uc18c \ubcf4\uc548\uacc4\uc7a5 \uc548\uc720\uac00 \ub2f9\uc2dc \uad6c\uce58\uc18c\uc5d0 \uc788\ub358 \uc7ac\uc57c\uc778\uc0ac\uc5d0\uac8c \ud3ed\ub85c\ud558\uc600\ub2e4. \uc548\uc720\uc758 \ud3ed\ub85c\ub294 \uc6b0\uc5ec\uace1\uc808\uc744 \uac70\uccd0 \uc815\uc758\uad6c\ud604\uc0ac\uc81c\ub2e8\uc5d0\uac8c \uc804\ub2ec\ub418\uc5c8\uace0, 1987\ub144 5\uc6d4 18\uc77c \uba85\ub3d9\uc131\ub2f9\uc5d0\uc11c \uc5f4\ub9b0 5\u00b718 \uad11\uc8fc \ubbfc\uc8fc\ud654 \uc6b4\ub3d9 \uae30\ub150\ubbf8\uc0ac\uc5d0\uc11c \u3008\uace0\ubb38 \uacbd\ucc30 \ucd95\uc18c \uc870\uc791\u3009\uc744 \ud3ed\ub85c\ud558\uc5ec 6\uc6d4 \ud56d\uc7c1\uc758 \uae30\ud3ed\uc81c\uac00 \ub418\uc5c8\ub2e4.", "answer": "\ub0a8\uc601\ub3d91985", "sentence": "\uae40\uadfc\ud0dc\uac00 \uace0\ubb38\uc744 \ub2f9\ud55c \uc77c\uc744 \uc18c\uc7ac\ub85c\ud55c \uc601\ud654 \u300a \ub0a8\uc601\ub3d91985 \u300b\uac00 2012\ub144 \uac1c\ubd09\ub418\uc5c8\ub2e4.", "paragraph_sentence": "\uad70\uc0ac\ub3c5\uc7ac \uc2dc\uae30 \ub0a8\uc601\ub3d9 \ub300\uacf5\ubd84\uc2e4\uc5d0\uc11c\ub294 \ubbfc\uc8fc\ud654 \uc6b4\ub3d9 \uc778\uc0ac\uc5d0 \ub300\ud55c \uace0\ubb38\uc774 \uc790\ud589\ub418\uc5c8\ub2e4. \ucde8\uc870\uc2e4\uc758 \uad6c\uc870 \uc790\uccb4\uac00 24 \uc2dc\uac04 \ub0b4\ub0b4 \ud53c\uc758\uc790\ub97c \uac10\uc2dc\ud560 \uc218 \uc788\ub294 \uad6c\uc870\ub85c \ub418\uc5b4 \uc788\uace0, \ubcc0\ud638\uc778 \uc811\uacac\uad8c\uacfc \uac19\uc740 \uae30\ubcf8\uc801\uc778 \uc778\uad8c\ub3c4 \uc9c0\ucf1c\uc9c0\uc9c0 \uc54a\uc740 \uc0c1\ud0dc\uc5d0\uc11c \uac00\ud639\ud55c \uace0\ubb38\uc774 \uc774\uc5b4\uc84c\ub2e4. 1985\ub144 9\uc6d4 \ub2f9\uc2dc \ubbfc\uc8fc\ud654\uc6b4\ub3d9\uccad\ub144\uc5f0\ud569\uc758 \uc758\uc7a5\uc774\uc5c8\ub358 \uae40\uadfc\ud0dc\ub294 \ub0a8\uc601\ub3d9 \ub300\uacf5\ubd84\uc2e4 511\ud638\uc5d0\uc11c \uace0\ubb38\uae30\uc220\uc790 \uc774\uadfc\uc548\uc5d0\uac8c \uc804\uae30\uace0\ubb38\uacfc \ubb3c\uace0\ubb38\uc744 \ub2f9\ud558\uc600\ub2e4. \uae40\uadfc\ud0dc\uac00 \uace0\ubb38\uc744 \ub2f9\ud55c \uc77c\uc744 \uc18c\uc7ac\ub85c\ud55c \uc601\ud654 \u300a \ub0a8\uc601\ub3d91985 \u300b\uac00 2012\ub144 \uac1c\ubd09\ub418\uc5c8\ub2e4. \ub0a8\uc601\ub3d9 \ub300\uacf5\ubd84\uc2e4\uc774 \uc138\uc0c1\uc5d0 \ub110\ub9ac \uc54c\ub824\uc9c0\uac8c \ub41c \uacc4\uae30\ub294 1987\ub144 1\uc6d4 \uc11c\uc6b8\ub300\ud559\uad50 \ud559\uc0dd\uc774\ub358 \ubc15\uc885\ucca0\uc774 \ucc38\uace0\uc778 \uc9c4\uc220\uc744 \uba85\ubaa9\uc73c\ub85c \uc5f0\ud589\ub418\uc5b4 \uace0\ubb38\uc744 \ubc1b\uace0 \uc0ac\ub9dd\ud55c \uc0ac\uac74 \ub54c\ubb38\uc774\ub2e4. \ubc15\uc885\ucca0\uc740 1987\ub144 1\uc6d4 14\uc77c 509\ud638 \uc870\uc0ac\uc2e4\uc5d0\uc11c \ud3ed\ud589\uacfc \uc804\uae30\uace0\ubb38, \ubb3c\uace0\ubb38\uc744 \ub2f9\ud558\ub2e4 \uc228\uc84c\ub2e4. \ub2f9\uc2dc \uc804\ub450\ud658 \uc815\uad8c\uc740 \uace0\ubb38 \uc0ac\uc2e4\uc744 \uc228\uae30\uae30 \uc704\ud574 \"\ud0c1\ud558\uace0 \uce58\ub2c8 \uc5b5\ud558\uace0 \uc8fd\uc5c8\ub2e4\"\ub294 \ubcc0\uba85\uc744 \ub0b4\ub193\uc544 \ub9ce\uc740 \uc0ac\ub78c\ub4e4\uc774 \ubd84\ub178\ud558\uc600\ub2e4. \uc774 \uc0ac\uac74\uc740 \ubd80\uac80\uc758\uc600\ub358 \uc624\uc5f0\uc0c1\uc774 \ubb3c\uace0\ubb38\uc774 \uc788\uc5c8\uc74c\uc744 \uc554\uc2dc\ud558\ub294 \uc0ac\uc778\uc744 \uae30\uc790\uc5d0\uac8c \uc54c\ub9ac\uba74\uc11c \uc9c4\uc0c1\uc774 \ub4e4\uc5b4\ub098\uae30 \uc2dc\uc791\ud558\uc600\uace0, \uc774\ud6c4 \ub2f9\uad6d\uc774 \ucd95\uc18c \uc740\ud3d0\ub97c \uc2dc\ub3c4\ud558\uc5ec \uc218\uc0ac\uad00 \ub458\ub9cc\uc774 \ub2e8\ub3c5\uc73c\ub85c \uc800\uc9c0\ub978 \uc77c\ub85c \ubab0\uc544\uac00\ub824\uace0 \ud558\uc790 \uc11c\uc6b8\uc601\ub4f1\ud3ec\uad6c\uce58\uc18c \ubcf4\uc548\uacc4\uc7a5 \uc548\uc720\uac00 \ub2f9\uc2dc \uad6c\uce58\uc18c\uc5d0 \uc788\ub358 \uc7ac\uc57c\uc778\uc0ac\uc5d0\uac8c \ud3ed\ub85c\ud558\uc600\ub2e4. \uc548\uc720\uc758 \ud3ed\ub85c\ub294 \uc6b0\uc5ec\uace1\uc808\uc744 \uac70\uccd0 \uc815\uc758\uad6c\ud604\uc0ac\uc81c\ub2e8\uc5d0\uac8c \uc804\ub2ec\ub418\uc5c8\uace0, 1987\ub144 5\uc6d4 18\uc77c \uba85\ub3d9\uc131\ub2f9\uc5d0\uc11c \uc5f4\ub9b0 5\u00b718 \uad11\uc8fc \ubbfc\uc8fc\ud654 \uc6b4\ub3d9 \uae30\ub150\ubbf8\uc0ac\uc5d0\uc11c \u3008\uace0\ubb38 \uacbd\ucc30 \ucd95\uc18c \uc870\uc791\u3009\uc744 \ud3ed\ub85c\ud558\uc5ec 6\uc6d4 \ud56d\uc7c1\uc758 \uae30\ud3ed\uc81c\uac00 \ub418\uc5c8\ub2e4.", "paragraph_answer": "\uad70\uc0ac\ub3c5\uc7ac \uc2dc\uae30 \ub0a8\uc601\ub3d9 \ub300\uacf5\ubd84\uc2e4\uc5d0\uc11c\ub294 \ubbfc\uc8fc\ud654 \uc6b4\ub3d9 \uc778\uc0ac\uc5d0 \ub300\ud55c \uace0\ubb38\uc774 \uc790\ud589\ub418\uc5c8\ub2e4. \ucde8\uc870\uc2e4\uc758 \uad6c\uc870 \uc790\uccb4\uac00 24 \uc2dc\uac04 \ub0b4\ub0b4 \ud53c\uc758\uc790\ub97c \uac10\uc2dc\ud560 \uc218 \uc788\ub294 \uad6c\uc870\ub85c \ub418\uc5b4 \uc788\uace0, \ubcc0\ud638\uc778 \uc811\uacac\uad8c\uacfc \uac19\uc740 \uae30\ubcf8\uc801\uc778 \uc778\uad8c\ub3c4 \uc9c0\ucf1c\uc9c0\uc9c0 \uc54a\uc740 \uc0c1\ud0dc\uc5d0\uc11c \uac00\ud639\ud55c \uace0\ubb38\uc774 \uc774\uc5b4\uc84c\ub2e4. 1985\ub144 9\uc6d4 \ub2f9\uc2dc \ubbfc\uc8fc\ud654\uc6b4\ub3d9\uccad\ub144\uc5f0\ud569\uc758 \uc758\uc7a5\uc774\uc5c8\ub358 \uae40\uadfc\ud0dc\ub294 \ub0a8\uc601\ub3d9 \ub300\uacf5\ubd84\uc2e4 511\ud638\uc5d0\uc11c \uace0\ubb38\uae30\uc220\uc790 \uc774\uadfc\uc548\uc5d0\uac8c \uc804\uae30\uace0\ubb38\uacfc \ubb3c\uace0\ubb38\uc744 \ub2f9\ud558\uc600\ub2e4. \uae40\uadfc\ud0dc\uac00 \uace0\ubb38\uc744 \ub2f9\ud55c \uc77c\uc744 \uc18c\uc7ac\ub85c\ud55c \uc601\ud654 \u300a \ub0a8\uc601\ub3d91985 \u300b\uac00 2012\ub144 \uac1c\ubd09\ub418\uc5c8\ub2e4. \ub0a8\uc601\ub3d9 \ub300\uacf5\ubd84\uc2e4\uc774 \uc138\uc0c1\uc5d0 \ub110\ub9ac \uc54c\ub824\uc9c0\uac8c \ub41c \uacc4\uae30\ub294 1987\ub144 1\uc6d4 \uc11c\uc6b8\ub300\ud559\uad50 \ud559\uc0dd\uc774\ub358 \ubc15\uc885\ucca0\uc774 \ucc38\uace0\uc778 \uc9c4\uc220\uc744 \uba85\ubaa9\uc73c\ub85c \uc5f0\ud589\ub418\uc5b4 \uace0\ubb38\uc744 \ubc1b\uace0 \uc0ac\ub9dd\ud55c \uc0ac\uac74 \ub54c\ubb38\uc774\ub2e4. \ubc15\uc885\ucca0\uc740 1987\ub144 1\uc6d4 14\uc77c 509\ud638 \uc870\uc0ac\uc2e4\uc5d0\uc11c \ud3ed\ud589\uacfc \uc804\uae30\uace0\ubb38, \ubb3c\uace0\ubb38\uc744 \ub2f9\ud558\ub2e4 \uc228\uc84c\ub2e4. \ub2f9\uc2dc \uc804\ub450\ud658 \uc815\uad8c\uc740 \uace0\ubb38 \uc0ac\uc2e4\uc744 \uc228\uae30\uae30 \uc704\ud574 \"\ud0c1\ud558\uace0 \uce58\ub2c8 \uc5b5\ud558\uace0 \uc8fd\uc5c8\ub2e4\"\ub294 \ubcc0\uba85\uc744 \ub0b4\ub193\uc544 \ub9ce\uc740 \uc0ac\ub78c\ub4e4\uc774 \ubd84\ub178\ud558\uc600\ub2e4. \uc774 \uc0ac\uac74\uc740 \ubd80\uac80\uc758\uc600\ub358 \uc624\uc5f0\uc0c1\uc774 \ubb3c\uace0\ubb38\uc774 \uc788\uc5c8\uc74c\uc744 \uc554\uc2dc\ud558\ub294 \uc0ac\uc778\uc744 \uae30\uc790\uc5d0\uac8c \uc54c\ub9ac\uba74\uc11c \uc9c4\uc0c1\uc774 \ub4e4\uc5b4\ub098\uae30 \uc2dc\uc791\ud558\uc600\uace0, \uc774\ud6c4 \ub2f9\uad6d\uc774 \ucd95\uc18c \uc740\ud3d0\ub97c \uc2dc\ub3c4\ud558\uc5ec \uc218\uc0ac\uad00 \ub458\ub9cc\uc774 \ub2e8\ub3c5\uc73c\ub85c \uc800\uc9c0\ub978 \uc77c\ub85c \ubab0\uc544\uac00\ub824\uace0 \ud558\uc790 \uc11c\uc6b8\uc601\ub4f1\ud3ec\uad6c\uce58\uc18c \ubcf4\uc548\uacc4\uc7a5 \uc548\uc720\uac00 \ub2f9\uc2dc \uad6c\uce58\uc18c\uc5d0 \uc788\ub358 \uc7ac\uc57c\uc778\uc0ac\uc5d0\uac8c \ud3ed\ub85c\ud558\uc600\ub2e4. \uc548\uc720\uc758 \ud3ed\ub85c\ub294 \uc6b0\uc5ec\uace1\uc808\uc744 \uac70\uccd0 \uc815\uc758\uad6c\ud604\uc0ac\uc81c\ub2e8\uc5d0\uac8c \uc804\ub2ec\ub418\uc5c8\uace0, 1987\ub144 5\uc6d4 18\uc77c \uba85\ub3d9\uc131\ub2f9\uc5d0\uc11c \uc5f4\ub9b0 5\u00b718 \uad11\uc8fc \ubbfc\uc8fc\ud654 \uc6b4\ub3d9 \uae30\ub150\ubbf8\uc0ac\uc5d0\uc11c \u3008\uace0\ubb38 \uacbd\ucc30 \ucd95\uc18c \uc870\uc791\u3009\uc744 \ud3ed\ub85c\ud558\uc5ec 6\uc6d4 \ud56d\uc7c1\uc758 \uae30\ud3ed\uc81c\uac00 \ub418\uc5c8\ub2e4.", "sentence_answer": "\uae40\uadfc\ud0dc\uac00 \uace0\ubb38\uc744 \ub2f9\ud55c \uc77c\uc744 \uc18c\uc7ac\ub85c\ud55c \uc601\ud654 \u300a \ub0a8\uc601\ub3d91985 \u300b\uac00 2012\ub144 \uac1c\ubd09\ub418\uc5c8\ub2e4.", "paragraph_id": "6483514-1-1", "paragraph_question": "question: \uae40\uadfc\ud0dc\uac00 \uace0\ubb38\uc744 \ub2f9\ud55c \uc77c\uc744 \uc18c\uc7ac\ub85c\ud55c \uc601\ud654 \uc81c\ubaa9\uc740?, context: \uad70\uc0ac\ub3c5\uc7ac \uc2dc\uae30 \ub0a8\uc601\ub3d9 \ub300\uacf5\ubd84\uc2e4\uc5d0\uc11c\ub294 \ubbfc\uc8fc\ud654 \uc6b4\ub3d9 \uc778\uc0ac\uc5d0 \ub300\ud55c \uace0\ubb38\uc774 \uc790\ud589\ub418\uc5c8\ub2e4. \ucde8\uc870\uc2e4\uc758 \uad6c\uc870 \uc790\uccb4\uac00 24 \uc2dc\uac04 \ub0b4\ub0b4 \ud53c\uc758\uc790\ub97c \uac10\uc2dc\ud560 \uc218 \uc788\ub294 \uad6c\uc870\ub85c \ub418\uc5b4 \uc788\uace0, \ubcc0\ud638\uc778 \uc811\uacac\uad8c\uacfc \uac19\uc740 \uae30\ubcf8\uc801\uc778 \uc778\uad8c\ub3c4 \uc9c0\ucf1c\uc9c0\uc9c0 \uc54a\uc740 \uc0c1\ud0dc\uc5d0\uc11c \uac00\ud639\ud55c \uace0\ubb38\uc774 \uc774\uc5b4\uc84c\ub2e4. 1985\ub144 9\uc6d4 \ub2f9\uc2dc \ubbfc\uc8fc\ud654\uc6b4\ub3d9\uccad\ub144\uc5f0\ud569\uc758 \uc758\uc7a5\uc774\uc5c8\ub358 \uae40\uadfc\ud0dc\ub294 \ub0a8\uc601\ub3d9 \ub300\uacf5\ubd84\uc2e4 511\ud638\uc5d0\uc11c \uace0\ubb38\uae30\uc220\uc790 \uc774\uadfc\uc548\uc5d0\uac8c \uc804\uae30\uace0\ubb38\uacfc \ubb3c\uace0\ubb38\uc744 \ub2f9\ud558\uc600\ub2e4. \uae40\uadfc\ud0dc\uac00 \uace0\ubb38\uc744 \ub2f9\ud55c \uc77c\uc744 \uc18c\uc7ac\ub85c\ud55c \uc601\ud654 \u300a\ub0a8\uc601\ub3d91985\u300b\uac00 2012\ub144 \uac1c\ubd09\ub418\uc5c8\ub2e4. \ub0a8\uc601\ub3d9 \ub300\uacf5\ubd84\uc2e4\uc774 \uc138\uc0c1\uc5d0 \ub110\ub9ac \uc54c\ub824\uc9c0\uac8c \ub41c \uacc4\uae30\ub294 1987\ub144 1\uc6d4 \uc11c\uc6b8\ub300\ud559\uad50 \ud559\uc0dd\uc774\ub358 \ubc15\uc885\ucca0\uc774 \ucc38\uace0\uc778 \uc9c4\uc220\uc744 \uba85\ubaa9\uc73c\ub85c \uc5f0\ud589\ub418\uc5b4 \uace0\ubb38\uc744 \ubc1b\uace0 \uc0ac\ub9dd\ud55c \uc0ac\uac74 \ub54c\ubb38\uc774\ub2e4. \ubc15\uc885\ucca0\uc740 1987\ub144 1\uc6d4 14\uc77c 509\ud638 \uc870\uc0ac\uc2e4\uc5d0\uc11c \ud3ed\ud589\uacfc \uc804\uae30\uace0\ubb38, \ubb3c\uace0\ubb38\uc744 \ub2f9\ud558\ub2e4 \uc228\uc84c\ub2e4. \ub2f9\uc2dc \uc804\ub450\ud658 \uc815\uad8c\uc740 \uace0\ubb38 \uc0ac\uc2e4\uc744 \uc228\uae30\uae30 \uc704\ud574 \"\ud0c1\ud558\uace0 \uce58\ub2c8 \uc5b5\ud558\uace0 \uc8fd\uc5c8\ub2e4\"\ub294 \ubcc0\uba85\uc744 \ub0b4\ub193\uc544 \ub9ce\uc740 \uc0ac\ub78c\ub4e4\uc774 \ubd84\ub178\ud558\uc600\ub2e4. \uc774 \uc0ac\uac74\uc740 \ubd80\uac80\uc758\uc600\ub358 \uc624\uc5f0\uc0c1\uc774 \ubb3c\uace0\ubb38\uc774 \uc788\uc5c8\uc74c\uc744 \uc554\uc2dc\ud558\ub294 \uc0ac\uc778\uc744 \uae30\uc790\uc5d0\uac8c \uc54c\ub9ac\uba74\uc11c \uc9c4\uc0c1\uc774 \ub4e4\uc5b4\ub098\uae30 \uc2dc\uc791\ud558\uc600\uace0, \uc774\ud6c4 \ub2f9\uad6d\uc774 \ucd95\uc18c \uc740\ud3d0\ub97c \uc2dc\ub3c4\ud558\uc5ec \uc218\uc0ac\uad00 \ub458\ub9cc\uc774 \ub2e8\ub3c5\uc73c\ub85c \uc800\uc9c0\ub978 \uc77c\ub85c \ubab0\uc544\uac00\ub824\uace0 \ud558\uc790 \uc11c\uc6b8\uc601\ub4f1\ud3ec\uad6c\uce58\uc18c \ubcf4\uc548\uacc4\uc7a5 \uc548\uc720\uac00 \ub2f9\uc2dc \uad6c\uce58\uc18c\uc5d0 \uc788\ub358 \uc7ac\uc57c\uc778\uc0ac\uc5d0\uac8c \ud3ed\ub85c\ud558\uc600\ub2e4. \uc548\uc720\uc758 \ud3ed\ub85c\ub294 \uc6b0\uc5ec\uace1\uc808\uc744 \uac70\uccd0 \uc815\uc758\uad6c\ud604\uc0ac\uc81c\ub2e8\uc5d0\uac8c \uc804\ub2ec\ub418\uc5c8\uace0, 1987\ub144 5\uc6d4 18\uc77c \uba85\ub3d9\uc131\ub2f9\uc5d0\uc11c \uc5f4\ub9b0 5\u00b718 \uad11\uc8fc \ubbfc\uc8fc\ud654 \uc6b4\ub3d9 \uae30\ub150\ubbf8\uc0ac\uc5d0\uc11c \u3008\uace0\ubb38 \uacbd\ucc30 \ucd95\uc18c \uc870\uc791\u3009\uc744 \ud3ed\ub85c\ud558\uc5ec 6\uc6d4 \ud56d\uc7c1\uc758 \uae30\ud3ed\uc81c\uac00 \ub418\uc5c8\ub2e4."} {"question": "\ud1a0\uba38\uc2a4 \uba5c\uc11c\uc2a4\uac00 \uc9c0\uc740 \uc720\uba85\ud55c \uc800\uc11c\ub294?", "paragraph": "\ud1a0\uba38\uc2a4 \uba5c\uc11c\uc2a4\uac00 18\uc138\uae30 \uadf8\uc758 \uc720\uba85\ud55c \uc800\uc11c \u300c\uc778\uad6c\ub860\u300d\uc5d0\uc11c \ube44\ud310\ud55c \ubc14\uc640 \uac19\uc774 \uba5c\uc11c\uc2a4 \uc8fc\uc758\uc790\ub4e4\uc740 \ud2b9\uc774\uc810\uc8fc\uc758\uc790\ub4e4\uc774 \ub099\uad00\uc801\uc778 \ubbf8\ub798 \ubc1c\uc804\uc744 \ubbff\ub294 \uac83\uc740 \ucc29\uac01\uc5d0 \ubd88\uacfc\ud558\ub2e4\uace0 \ube44\ud310\ud55c\ub2e4.[10] \uadf8\ub4e4\uc740 \uc778\uad6c\uc758 \uc99d\uac00\ub97c \ubbf8\ub355\uc73c\ub85c \ubcf8 18\uc138\uae30 \ub2f9\uc2dc\uc758 \ud1b5\ub150\uc774 19\uc138\uae30 \uc778\uad6c\uc758 \uae09\uc99d\uc73c\ub85c \ube48\uace4\uc758 \uac00\uc18d\ud654\ub97c \uc774\ub048 \uac83\uacfc \uac19\uc774 \uc815\ubcf4 \uc5d4\ud2b8\ub85c\ud53c\ub294 \ud56d\uc0c1 \uc99d\uac00\ud558\ubbc0\ub85c \ud2b9\uc774\uc810\uc8fc\uc758\uc790\ub4e4\uc774 \uae30\ub300\ud558\ub294 \ud3ed\ubc1c\uc801\uc778 \uae09\uc131\uc7a5\uc740 \uc774\ub8e8\uc5b4 \uc9c8 \uc218 \uc5c6\uc744 \uac83\uc774\ub77c\uace0 \uc124\uba85\ud55c\ub2e4. \ub2e4\uc2dc \ub9d0\ud574 \ud2b9\uc774\uc810\uc8fc\uc758\uc790\ub4e4\uc774 \uc124\uba85\ud558\ub294 \uae30\uc220\uc801 \ud2b9\uc774\uc810\uc73c\ub85c\uc758 \uc131\uc7a5\uc744 \ub4b7\ubc1b\uce68 \ud558\uae30 \uc704\ud55c \uc790\uc6d0\uc740 \uacfc\ub3c4\uae30\uc5d0 \ub2e4\ub2e4\ub97c\uc218\ub85d \ubd80\uc871\ud574\uc9c8 \uac83\uc774\uace0, \uacb0\uacfc\uc801\uc73c\ub85c \uc0ac\uc0c1\uc758 \uc9c0\ud3c9\uc120\uc73c\ub85c \ub118\uc5b4\uac00\uae30 \uc704\ud55c \uacfc\ub3c4\uae30 \ub2e8\uacc4\uc5d0\ub294 \uc790\uc6d0\uc758 \ubd80\uc871\uc73c\ub85c \uc778\ud574 \uc131\uc7a5\uc774 \uba48\ucd94\ub294 \ud2b9\uc774\uc810\uc758 \uc5ed\uc124\uc774 \ubc1c\uc0dd\ud560 \uac83\uc774\ub77c\uace0 \uc608\uce21\ud55c\ub2e4.", "answer": "\uc778\uad6c\ub860", "sentence": "\ud1a0\uba38\uc2a4 \uba5c\uc11c\uc2a4\uac00 18\uc138\uae30 \uadf8\uc758 \uc720\uba85\ud55c \uc800\uc11c \u300c \uc778\uad6c\ub860 \u300d\uc5d0\uc11c \ube44\ud310\ud55c \ubc14\uc640 \uac19\uc774 \uba5c\uc11c\uc2a4 \uc8fc\uc758\uc790\ub4e4\uc740 \ud2b9\uc774\uc810\uc8fc\uc758\uc790\ub4e4\uc774 \ub099\uad00\uc801\uc778 \ubbf8\ub798 \ubc1c\uc804\uc744 \ubbff\ub294 \uac83\uc740 \ucc29\uac01\uc5d0 \ubd88\uacfc\ud558\ub2e4\uace0 \ube44\ud310\ud55c\ub2e4.[10] \uadf8\ub4e4\uc740 \uc778\uad6c\uc758 \uc99d\uac00\ub97c \ubbf8\ub355\uc73c\ub85c \ubcf8 18\uc138\uae30 \ub2f9\uc2dc\uc758 \ud1b5\ub150\uc774 19\uc138\uae30 \uc778\uad6c\uc758 \uae09\uc99d\uc73c\ub85c \ube48\uace4\uc758 \uac00\uc18d\ud654\ub97c \uc774\ub048 \uac83\uacfc \uac19\uc774 \uc815\ubcf4 \uc5d4\ud2b8\ub85c\ud53c\ub294 \ud56d\uc0c1 \uc99d\uac00\ud558\ubbc0\ub85c \ud2b9\uc774\uc810\uc8fc\uc758\uc790\ub4e4\uc774 \uae30\ub300\ud558\ub294 \ud3ed\ubc1c\uc801\uc778 \uae09\uc131\uc7a5\uc740 \uc774\ub8e8\uc5b4 \uc9c8 \uc218 \uc5c6\uc744 \uac83\uc774\ub77c\uace0 \uc124\uba85\ud55c\ub2e4.", "paragraph_sentence": " \ud1a0\uba38\uc2a4 \uba5c\uc11c\uc2a4\uac00 18\uc138\uae30 \uadf8\uc758 \uc720\uba85\ud55c \uc800\uc11c \u300c \uc778\uad6c\ub860 \u300d\uc5d0\uc11c \ube44\ud310\ud55c \ubc14\uc640 \uac19\uc774 \uba5c\uc11c\uc2a4 \uc8fc\uc758\uc790\ub4e4\uc740 \ud2b9\uc774\uc810\uc8fc\uc758\uc790\ub4e4\uc774 \ub099\uad00\uc801\uc778 \ubbf8\ub798 \ubc1c\uc804\uc744 \ubbff\ub294 \uac83\uc740 \ucc29\uac01\uc5d0 \ubd88\uacfc\ud558\ub2e4\uace0 \ube44\ud310\ud55c\ub2e4.[10] \uadf8\ub4e4\uc740 \uc778\uad6c\uc758 \uc99d\uac00\ub97c \ubbf8\ub355\uc73c\ub85c \ubcf8 18\uc138\uae30 \ub2f9\uc2dc\uc758 \ud1b5\ub150\uc774 19\uc138\uae30 \uc778\uad6c\uc758 \uae09\uc99d\uc73c\ub85c \ube48\uace4\uc758 \uac00\uc18d\ud654\ub97c \uc774\ub048 \uac83\uacfc \uac19\uc774 \uc815\ubcf4 \uc5d4\ud2b8\ub85c\ud53c\ub294 \ud56d\uc0c1 \uc99d\uac00\ud558\ubbc0\ub85c \ud2b9\uc774\uc810\uc8fc\uc758\uc790\ub4e4\uc774 \uae30\ub300\ud558\ub294 \ud3ed\ubc1c\uc801\uc778 \uae09\uc131\uc7a5\uc740 \uc774\ub8e8\uc5b4 \uc9c8 \uc218 \uc5c6\uc744 \uac83\uc774\ub77c\uace0 \uc124\uba85\ud55c\ub2e4. \ub2e4\uc2dc \ub9d0\ud574 \ud2b9\uc774\uc810\uc8fc\uc758\uc790\ub4e4\uc774 \uc124\uba85\ud558\ub294 \uae30\uc220\uc801 \ud2b9\uc774\uc810\uc73c\ub85c\uc758 \uc131\uc7a5\uc744 \ub4b7\ubc1b\uce68 \ud558\uae30 \uc704\ud55c \uc790\uc6d0\uc740 \uacfc\ub3c4\uae30\uc5d0 \ub2e4\ub2e4\ub97c\uc218\ub85d \ubd80\uc871\ud574\uc9c8 \uac83\uc774\uace0, \uacb0\uacfc\uc801\uc73c\ub85c \uc0ac\uc0c1\uc758 \uc9c0\ud3c9\uc120\uc73c\ub85c \ub118\uc5b4\uac00\uae30 \uc704\ud55c \uacfc\ub3c4\uae30 \ub2e8\uacc4\uc5d0\ub294 \uc790\uc6d0\uc758 \ubd80\uc871\uc73c\ub85c \uc778\ud574 \uc131\uc7a5\uc774 \uba48\ucd94\ub294 \ud2b9\uc774\uc810\uc758 \uc5ed\uc124\uc774 \ubc1c\uc0dd\ud560 \uac83\uc774\ub77c\uace0 \uc608\uce21\ud55c\ub2e4.", "paragraph_answer": "\ud1a0\uba38\uc2a4 \uba5c\uc11c\uc2a4\uac00 18\uc138\uae30 \uadf8\uc758 \uc720\uba85\ud55c \uc800\uc11c \u300c \uc778\uad6c\ub860 \u300d\uc5d0\uc11c \ube44\ud310\ud55c \ubc14\uc640 \uac19\uc774 \uba5c\uc11c\uc2a4 \uc8fc\uc758\uc790\ub4e4\uc740 \ud2b9\uc774\uc810\uc8fc\uc758\uc790\ub4e4\uc774 \ub099\uad00\uc801\uc778 \ubbf8\ub798 \ubc1c\uc804\uc744 \ubbff\ub294 \uac83\uc740 \ucc29\uac01\uc5d0 \ubd88\uacfc\ud558\ub2e4\uace0 \ube44\ud310\ud55c\ub2e4.[10] \uadf8\ub4e4\uc740 \uc778\uad6c\uc758 \uc99d\uac00\ub97c \ubbf8\ub355\uc73c\ub85c \ubcf8 18\uc138\uae30 \ub2f9\uc2dc\uc758 \ud1b5\ub150\uc774 19\uc138\uae30 \uc778\uad6c\uc758 \uae09\uc99d\uc73c\ub85c \ube48\uace4\uc758 \uac00\uc18d\ud654\ub97c \uc774\ub048 \uac83\uacfc \uac19\uc774 \uc815\ubcf4 \uc5d4\ud2b8\ub85c\ud53c\ub294 \ud56d\uc0c1 \uc99d\uac00\ud558\ubbc0\ub85c \ud2b9\uc774\uc810\uc8fc\uc758\uc790\ub4e4\uc774 \uae30\ub300\ud558\ub294 \ud3ed\ubc1c\uc801\uc778 \uae09\uc131\uc7a5\uc740 \uc774\ub8e8\uc5b4 \uc9c8 \uc218 \uc5c6\uc744 \uac83\uc774\ub77c\uace0 \uc124\uba85\ud55c\ub2e4. \ub2e4\uc2dc \ub9d0\ud574 \ud2b9\uc774\uc810\uc8fc\uc758\uc790\ub4e4\uc774 \uc124\uba85\ud558\ub294 \uae30\uc220\uc801 \ud2b9\uc774\uc810\uc73c\ub85c\uc758 \uc131\uc7a5\uc744 \ub4b7\ubc1b\uce68 \ud558\uae30 \uc704\ud55c \uc790\uc6d0\uc740 \uacfc\ub3c4\uae30\uc5d0 \ub2e4\ub2e4\ub97c\uc218\ub85d \ubd80\uc871\ud574\uc9c8 \uac83\uc774\uace0, \uacb0\uacfc\uc801\uc73c\ub85c \uc0ac\uc0c1\uc758 \uc9c0\ud3c9\uc120\uc73c\ub85c \ub118\uc5b4\uac00\uae30 \uc704\ud55c \uacfc\ub3c4\uae30 \ub2e8\uacc4\uc5d0\ub294 \uc790\uc6d0\uc758 \ubd80\uc871\uc73c\ub85c \uc778\ud574 \uc131\uc7a5\uc774 \uba48\ucd94\ub294 \ud2b9\uc774\uc810\uc758 \uc5ed\uc124\uc774 \ubc1c\uc0dd\ud560 \uac83\uc774\ub77c\uace0 \uc608\uce21\ud55c\ub2e4.", "sentence_answer": "\ud1a0\uba38\uc2a4 \uba5c\uc11c\uc2a4\uac00 18\uc138\uae30 \uadf8\uc758 \uc720\uba85\ud55c \uc800\uc11c \u300c \uc778\uad6c\ub860 \u300d\uc5d0\uc11c \ube44\ud310\ud55c \ubc14\uc640 \uac19\uc774 \uba5c\uc11c\uc2a4 \uc8fc\uc758\uc790\ub4e4\uc740 \ud2b9\uc774\uc810\uc8fc\uc758\uc790\ub4e4\uc774 \ub099\uad00\uc801\uc778 \ubbf8\ub798 \ubc1c\uc804\uc744 \ubbff\ub294 \uac83\uc740 \ucc29\uac01\uc5d0 \ubd88\uacfc\ud558\ub2e4\uace0 \ube44\ud310\ud55c\ub2e4.[10] \uadf8\ub4e4\uc740 \uc778\uad6c\uc758 \uc99d\uac00\ub97c \ubbf8\ub355\uc73c\ub85c \ubcf8 18\uc138\uae30 \ub2f9\uc2dc\uc758 \ud1b5\ub150\uc774 19\uc138\uae30 \uc778\uad6c\uc758 \uae09\uc99d\uc73c\ub85c \ube48\uace4\uc758 \uac00\uc18d\ud654\ub97c \uc774\ub048 \uac83\uacfc \uac19\uc774 \uc815\ubcf4 \uc5d4\ud2b8\ub85c\ud53c\ub294 \ud56d\uc0c1 \uc99d\uac00\ud558\ubbc0\ub85c \ud2b9\uc774\uc810\uc8fc\uc758\uc790\ub4e4\uc774 \uae30\ub300\ud558\ub294 \ud3ed\ubc1c\uc801\uc778 \uae09\uc131\uc7a5\uc740 \uc774\ub8e8\uc5b4 \uc9c8 \uc218 \uc5c6\uc744 \uac83\uc774\ub77c\uace0 \uc124\uba85\ud55c\ub2e4.", "paragraph_id": "6260956-8-1", "paragraph_question": "question: \ud1a0\uba38\uc2a4 \uba5c\uc11c\uc2a4\uac00 \uc9c0\uc740 \uc720\uba85\ud55c \uc800\uc11c\ub294?, context: \ud1a0\uba38\uc2a4 \uba5c\uc11c\uc2a4\uac00 18\uc138\uae30 \uadf8\uc758 \uc720\uba85\ud55c \uc800\uc11c \u300c\uc778\uad6c\ub860\u300d\uc5d0\uc11c \ube44\ud310\ud55c \ubc14\uc640 \uac19\uc774 \uba5c\uc11c\uc2a4 \uc8fc\uc758\uc790\ub4e4\uc740 \ud2b9\uc774\uc810\uc8fc\uc758\uc790\ub4e4\uc774 \ub099\uad00\uc801\uc778 \ubbf8\ub798 \ubc1c\uc804\uc744 \ubbff\ub294 \uac83\uc740 \ucc29\uac01\uc5d0 \ubd88\uacfc\ud558\ub2e4\uace0 \ube44\ud310\ud55c\ub2e4.[10] \uadf8\ub4e4\uc740 \uc778\uad6c\uc758 \uc99d\uac00\ub97c \ubbf8\ub355\uc73c\ub85c \ubcf8 18\uc138\uae30 \ub2f9\uc2dc\uc758 \ud1b5\ub150\uc774 19\uc138\uae30 \uc778\uad6c\uc758 \uae09\uc99d\uc73c\ub85c \ube48\uace4\uc758 \uac00\uc18d\ud654\ub97c \uc774\ub048 \uac83\uacfc \uac19\uc774 \uc815\ubcf4 \uc5d4\ud2b8\ub85c\ud53c\ub294 \ud56d\uc0c1 \uc99d\uac00\ud558\ubbc0\ub85c \ud2b9\uc774\uc810\uc8fc\uc758\uc790\ub4e4\uc774 \uae30\ub300\ud558\ub294 \ud3ed\ubc1c\uc801\uc778 \uae09\uc131\uc7a5\uc740 \uc774\ub8e8\uc5b4 \uc9c8 \uc218 \uc5c6\uc744 \uac83\uc774\ub77c\uace0 \uc124\uba85\ud55c\ub2e4. \ub2e4\uc2dc \ub9d0\ud574 \ud2b9\uc774\uc810\uc8fc\uc758\uc790\ub4e4\uc774 \uc124\uba85\ud558\ub294 \uae30\uc220\uc801 \ud2b9\uc774\uc810\uc73c\ub85c\uc758 \uc131\uc7a5\uc744 \ub4b7\ubc1b\uce68 \ud558\uae30 \uc704\ud55c \uc790\uc6d0\uc740 \uacfc\ub3c4\uae30\uc5d0 \ub2e4\ub2e4\ub97c\uc218\ub85d \ubd80\uc871\ud574\uc9c8 \uac83\uc774\uace0, \uacb0\uacfc\uc801\uc73c\ub85c \uc0ac\uc0c1\uc758 \uc9c0\ud3c9\uc120\uc73c\ub85c \ub118\uc5b4\uac00\uae30 \uc704\ud55c \uacfc\ub3c4\uae30 \ub2e8\uacc4\uc5d0\ub294 \uc790\uc6d0\uc758 \ubd80\uc871\uc73c\ub85c \uc778\ud574 \uc131\uc7a5\uc774 \uba48\ucd94\ub294 \ud2b9\uc774\uc810\uc758 \uc5ed\uc124\uc774 \ubc1c\uc0dd\ud560 \uac83\uc774\ub77c\uace0 \uc608\uce21\ud55c\ub2e4."} {"question": "\ub86f\ub370\uc6d4\ub4dc\ud0c0\uc6cc\uc758 \ub192\uc774\ub294?", "paragraph": "\ub300\ud55c\ubbfc\uad6d\uc758 \ub9c8\ucc9c\ub8e8 \ubaa9\ub85d\uc774\ub2e4. \ub300\ud55c\ubbfc\uad6d\uc5d0\uc11c \uac00\uc7a5 \ub192\uc740 \uac74\ubb3c\uc740 \ud604\uc7ac 123\uce35\uc778 \ub86f\ub370\uc6d4\ub4dc\ud0c0\uc6cc\ub85c 555m(1,821ft) \ub192\uc774\ub85c 2016\ub144 12\uc6d4 22\uc77c\uc5d0 \uc644\uacf5\ub418\uc5c8\uc73c\uba70 \uc138\uacc4\uc5d0\uc11c 5\ubc88\uc9f8\ub85c \ub192\uc740 \uac74\ubb3c\uc774\ub2e4. \ud604\uc7ac \ub300\ud55c\ubbfc\uad6d\uc5d0\ub294 \uac74\uc124 \uc911\uc778 300m\uac00 \ub118\ub294 3\uac1c\uc758 \uac74\ubb3c\uc774 \uc788\ub2e4. \ud574\uc6b4\ub300 \uc5d8\uc2dc\ud2f0 \ub354\uc0f5\uc758 3\uac1c \uac74\ubb3c\uc778 \uc0f5(Sharp)\ub3c4 \ubd80\uc0b0\uc5d0 \uac74\uc124 \uc911\uc774\uba70 412m(1,352ft)\uc5d0 \ub3c4\ub2ec\ud560 \uac83\uc774\ub2e4. 300m\uac00 \ub118\ub294 \ub2e4\uc591\ud55c \ud504\ub85c\uc81d\ud2b8\uac00 \uacc4\ud68d\ub418\uc5b4 \uc788\ub294\ub370, \ub192\uc774\ub294 338m(1,109ft)\uc778 \ud30c\ud06c\uc6d0 \ud0c0\uc6cc(2020\ub144 \uc644\uacf5 \uc608\uc815), \ud604\uc7ac \ubcf4\uc720\ud558\uace0 \uc788\ub294 \ub86f\ub370\ud0c0\uc6b4\ud0c0\uc6cc(510m, 1,673ft) \ubc0f 2021\ub144 \uc644\uacf5 \uc608\uc815\uc778 \ud604\ub300\ucc28 \uae00\ub85c\ubc8c \ube44\uc988\ub2c8\uc2a4 \uc13c\ud130(569m)\uac00 \uac74\uc124\ub420 \uc608\uc815\uc774\ub2e4. \ub300\ud55c\ubbfc\uad6d\uc758 \u300c\ucd08\uace0\uce35 \ubc0f \uc9c0\ud558\uc5f0\uacc4 \ubcf5\ud569\uac74\ucd95\ubb3c \uc7ac\ub09c\uad00\ub9ac\uc5d0 \uad00\ud55c \ud2b9\ubcc4\ubc95\u300d\uc5d0 \ub530\ub974\uba74 '\ucd08\uace0\uce35 \uac74\ucd95\ubb3c' \uc774\ub780 \uce35\uc218\uac00 50\uce35 \uc774\uc0c1 \ub610\ub294 \ub192\uc774\uac00 200m \uc774\uc0c1\uc778 \uac74\ucd95\ubb3c\uc744 \ub9d0\ud55c\ub2e4.", "answer": "555m", "sentence": "\ub300\ud55c\ubbfc\uad6d\uc5d0\uc11c \uac00\uc7a5 \ub192\uc740 \uac74\ubb3c\uc740 \ud604\uc7ac 123\uce35\uc778 \ub86f\ub370\uc6d4\ub4dc\ud0c0\uc6cc\ub85c 555m (1,821ft) \ub192\uc774\ub85c 2016\ub144 12\uc6d4 22\uc77c\uc5d0 \uc644\uacf5\ub418\uc5c8\uc73c\uba70 \uc138\uacc4\uc5d0\uc11c 5\ubc88\uc9f8\ub85c \ub192\uc740 \uac74\ubb3c\uc774\ub2e4.", "paragraph_sentence": "\ub300\ud55c\ubbfc\uad6d\uc758 \ub9c8\ucc9c\ub8e8 \ubaa9\ub85d\uc774\ub2e4. \ub300\ud55c\ubbfc\uad6d\uc5d0\uc11c \uac00\uc7a5 \ub192\uc740 \uac74\ubb3c\uc740 \ud604\uc7ac 123\uce35\uc778 \ub86f\ub370\uc6d4\ub4dc\ud0c0\uc6cc\ub85c 555m (1,821ft) \ub192\uc774\ub85c 2016\ub144 12\uc6d4 22\uc77c\uc5d0 \uc644\uacf5\ub418\uc5c8\uc73c\uba70 \uc138\uacc4\uc5d0\uc11c 5\ubc88\uc9f8\ub85c \ub192\uc740 \uac74\ubb3c\uc774\ub2e4. \ud604\uc7ac \ub300\ud55c\ubbfc\uad6d\uc5d0\ub294 \uac74\uc124 \uc911\uc778 300m\uac00 \ub118\ub294 3\uac1c\uc758 \uac74\ubb3c\uc774 \uc788\ub2e4. \ud574\uc6b4\ub300 \uc5d8\uc2dc\ud2f0 \ub354\uc0f5\uc758 3\uac1c \uac74\ubb3c\uc778 \uc0f5(Sharp)\ub3c4 \ubd80\uc0b0\uc5d0 \uac74\uc124 \uc911\uc774\uba70 412m(1,352ft)\uc5d0 \ub3c4\ub2ec\ud560 \uac83\uc774\ub2e4. 300m\uac00 \ub118\ub294 \ub2e4\uc591\ud55c \ud504\ub85c\uc81d\ud2b8\uac00 \uacc4\ud68d\ub418\uc5b4 \uc788\ub294\ub370, \ub192\uc774\ub294 338m(1,109ft)\uc778 \ud30c\ud06c\uc6d0 \ud0c0\uc6cc(2020\ub144 \uc644\uacf5 \uc608\uc815), \ud604\uc7ac \ubcf4\uc720\ud558\uace0 \uc788\ub294 \ub86f\ub370\ud0c0\uc6b4\ud0c0\uc6cc(510m, 1,673ft) \ubc0f 2021\ub144 \uc644\uacf5 \uc608\uc815\uc778 \ud604\ub300\ucc28 \uae00\ub85c\ubc8c \ube44\uc988\ub2c8\uc2a4 \uc13c\ud130(569m)\uac00 \uac74\uc124\ub420 \uc608\uc815\uc774\ub2e4. \ub300\ud55c\ubbfc\uad6d\uc758 \u300c\ucd08\uace0\uce35 \ubc0f \uc9c0\ud558\uc5f0\uacc4 \ubcf5\ud569\uac74\ucd95\ubb3c \uc7ac\ub09c\uad00\ub9ac\uc5d0 \uad00\ud55c \ud2b9\ubcc4\ubc95\u300d\uc5d0 \ub530\ub974\uba74 '\ucd08\uace0\uce35 \uac74\ucd95\ubb3c' \uc774\ub780 \uce35\uc218\uac00 50\uce35 \uc774\uc0c1 \ub610\ub294 \ub192\uc774\uac00 200m \uc774\uc0c1\uc778 \uac74\ucd95\ubb3c\uc744 \ub9d0\ud55c\ub2e4.", "paragraph_answer": "\ub300\ud55c\ubbfc\uad6d\uc758 \ub9c8\ucc9c\ub8e8 \ubaa9\ub85d\uc774\ub2e4. \ub300\ud55c\ubbfc\uad6d\uc5d0\uc11c \uac00\uc7a5 \ub192\uc740 \uac74\ubb3c\uc740 \ud604\uc7ac 123\uce35\uc778 \ub86f\ub370\uc6d4\ub4dc\ud0c0\uc6cc\ub85c 555m (1,821ft) \ub192\uc774\ub85c 2016\ub144 12\uc6d4 22\uc77c\uc5d0 \uc644\uacf5\ub418\uc5c8\uc73c\uba70 \uc138\uacc4\uc5d0\uc11c 5\ubc88\uc9f8\ub85c \ub192\uc740 \uac74\ubb3c\uc774\ub2e4. \ud604\uc7ac \ub300\ud55c\ubbfc\uad6d\uc5d0\ub294 \uac74\uc124 \uc911\uc778 300m\uac00 \ub118\ub294 3\uac1c\uc758 \uac74\ubb3c\uc774 \uc788\ub2e4. \ud574\uc6b4\ub300 \uc5d8\uc2dc\ud2f0 \ub354\uc0f5\uc758 3\uac1c \uac74\ubb3c\uc778 \uc0f5(Sharp)\ub3c4 \ubd80\uc0b0\uc5d0 \uac74\uc124 \uc911\uc774\uba70 412m(1,352ft)\uc5d0 \ub3c4\ub2ec\ud560 \uac83\uc774\ub2e4. 300m\uac00 \ub118\ub294 \ub2e4\uc591\ud55c \ud504\ub85c\uc81d\ud2b8\uac00 \uacc4\ud68d\ub418\uc5b4 \uc788\ub294\ub370, \ub192\uc774\ub294 338m(1,109ft)\uc778 \ud30c\ud06c\uc6d0 \ud0c0\uc6cc(2020\ub144 \uc644\uacf5 \uc608\uc815), \ud604\uc7ac \ubcf4\uc720\ud558\uace0 \uc788\ub294 \ub86f\ub370\ud0c0\uc6b4\ud0c0\uc6cc(510m, 1,673ft) \ubc0f 2021\ub144 \uc644\uacf5 \uc608\uc815\uc778 \ud604\ub300\ucc28 \uae00\ub85c\ubc8c \ube44\uc988\ub2c8\uc2a4 \uc13c\ud130(569m)\uac00 \uac74\uc124\ub420 \uc608\uc815\uc774\ub2e4. \ub300\ud55c\ubbfc\uad6d\uc758 \u300c\ucd08\uace0\uce35 \ubc0f \uc9c0\ud558\uc5f0\uacc4 \ubcf5\ud569\uac74\ucd95\ubb3c \uc7ac\ub09c\uad00\ub9ac\uc5d0 \uad00\ud55c \ud2b9\ubcc4\ubc95\u300d\uc5d0 \ub530\ub974\uba74 '\ucd08\uace0\uce35 \uac74\ucd95\ubb3c' \uc774\ub780 \uce35\uc218\uac00 50\uce35 \uc774\uc0c1 \ub610\ub294 \ub192\uc774\uac00 200m \uc774\uc0c1\uc778 \uac74\ucd95\ubb3c\uc744 \ub9d0\ud55c\ub2e4.", "sentence_answer": "\ub300\ud55c\ubbfc\uad6d\uc5d0\uc11c \uac00\uc7a5 \ub192\uc740 \uac74\ubb3c\uc740 \ud604\uc7ac 123\uce35\uc778 \ub86f\ub370\uc6d4\ub4dc\ud0c0\uc6cc\ub85c 555m (1,821ft) \ub192\uc774\ub85c 2016\ub144 12\uc6d4 22\uc77c\uc5d0 \uc644\uacf5\ub418\uc5c8\uc73c\uba70 \uc138\uacc4\uc5d0\uc11c 5\ubc88\uc9f8\ub85c \ub192\uc740 \uac74\ubb3c\uc774\ub2e4.", "paragraph_id": "6512270-0-0", "paragraph_question": "question: \ub86f\ub370\uc6d4\ub4dc\ud0c0\uc6cc\uc758 \ub192\uc774\ub294?, context: \ub300\ud55c\ubbfc\uad6d\uc758 \ub9c8\ucc9c\ub8e8 \ubaa9\ub85d\uc774\ub2e4. \ub300\ud55c\ubbfc\uad6d\uc5d0\uc11c \uac00\uc7a5 \ub192\uc740 \uac74\ubb3c\uc740 \ud604\uc7ac 123\uce35\uc778 \ub86f\ub370\uc6d4\ub4dc\ud0c0\uc6cc\ub85c 555m(1,821ft) \ub192\uc774\ub85c 2016\ub144 12\uc6d4 22\uc77c\uc5d0 \uc644\uacf5\ub418\uc5c8\uc73c\uba70 \uc138\uacc4\uc5d0\uc11c 5\ubc88\uc9f8\ub85c \ub192\uc740 \uac74\ubb3c\uc774\ub2e4. \ud604\uc7ac \ub300\ud55c\ubbfc\uad6d\uc5d0\ub294 \uac74\uc124 \uc911\uc778 300m\uac00 \ub118\ub294 3\uac1c\uc758 \uac74\ubb3c\uc774 \uc788\ub2e4. \ud574\uc6b4\ub300 \uc5d8\uc2dc\ud2f0 \ub354\uc0f5\uc758 3\uac1c \uac74\ubb3c\uc778 \uc0f5(Sharp)\ub3c4 \ubd80\uc0b0\uc5d0 \uac74\uc124 \uc911\uc774\uba70 412m(1,352ft)\uc5d0 \ub3c4\ub2ec\ud560 \uac83\uc774\ub2e4. 300m\uac00 \ub118\ub294 \ub2e4\uc591\ud55c \ud504\ub85c\uc81d\ud2b8\uac00 \uacc4\ud68d\ub418\uc5b4 \uc788\ub294\ub370, \ub192\uc774\ub294 338m(1,109ft)\uc778 \ud30c\ud06c\uc6d0 \ud0c0\uc6cc(2020\ub144 \uc644\uacf5 \uc608\uc815), \ud604\uc7ac \ubcf4\uc720\ud558\uace0 \uc788\ub294 \ub86f\ub370\ud0c0\uc6b4\ud0c0\uc6cc(510m, 1,673ft) \ubc0f 2021\ub144 \uc644\uacf5 \uc608\uc815\uc778 \ud604\ub300\ucc28 \uae00\ub85c\ubc8c \ube44\uc988\ub2c8\uc2a4 \uc13c\ud130(569m)\uac00 \uac74\uc124\ub420 \uc608\uc815\uc774\ub2e4. \ub300\ud55c\ubbfc\uad6d\uc758 \u300c\ucd08\uace0\uce35 \ubc0f \uc9c0\ud558\uc5f0\uacc4 \ubcf5\ud569\uac74\ucd95\ubb3c \uc7ac\ub09c\uad00\ub9ac\uc5d0 \uad00\ud55c \ud2b9\ubcc4\ubc95\u300d\uc5d0 \ub530\ub974\uba74 '\ucd08\uace0\uce35 \uac74\ucd95\ubb3c' \uc774\ub780 \uce35\uc218\uac00 50\uce35 \uc774\uc0c1 \ub610\ub294 \ub192\uc774\uac00 200m \uc774\uc0c1\uc778 \uac74\ucd95\ubb3c\uc744 \ub9d0\ud55c\ub2e4."} {"question": "\ub9c8\ub974\ucf54\uac00 \ud504\ub85c\ub4c0\uc2f1 \ubc0f \ub7a9\uc73c\ub85c \ucc38\uc5ec\ud55c \ud799\ud569 \ud06c\ub8e8 \uc9c0\uae30 \ud3a0\ub77c\uc988\uc758 \uc568\ubc94 \uba85\uc740?", "paragraph": "\uadf8 \ud6c4 2008\ub144 \ud55c\ud574 \uadf8\ub294 \uac89\uc73c\ub85c \ub4dc\ub7ec\ub098\ub294 \ud65c\ub3d9\uc740 INC\uc640 M&A \uc568\ubc94 \ucc38\uc5ec \uc815\ub3c4\uc5d0 \uadf8\ucce4\uc73c\ub098 \ucf00\uc774\ube14 TV, \ub77c\ub514\uc624, \uac8c\uc784 CF \uc74c\uc545 \ub4f1\uc744 \uc791\uace1\ud558\uba74\uc11c \uc880 \ub354 \uc791\uace1\uac00\ub85c\uc368\uc758 \uba74\ubaa8\ub97c \ub354\ud588\uace0, \ud2b9\ud788 2008\ub144 12\uc6d4 Jiggy Fellaz\uc758 \uba64\ubc84\uac00 \ub418\uc5c8\uc74c\uc744 \uacf5\ud45c\ud558\uc5ec \ud32c\ub4e4\uc744 \ub180\ub77c\uac8c \ud588\ub2e4. \uace7\ubc14\ub85c \uadf8\ub294 \ubc1c\ub9e4 \uc608\uc815\uc774\uc5c8\ub358 Jiggy Fellaz\uc758 \ucef4\ud544\ub808\uc774\uc158 \uc568\ubc94 The Black Album\uc5d0 \ud504\ub85c\ub4c0\uc2f1 \ubc0f \ub7a9\uc73c\ub85c \ucc38\uc5ec\ud558\uc600\ub2e4. \uc5bc\ub9c8 \uc9c0\ub098\uc9c0 \uc54a\uc544 \uadf8\uc758 \uc0c8 \uc568\ubc94 Premium Edition\uc774 \ubc1c\ub9e4\ub418\uc5c8\ub294\ub370, \uc774 \uc568\ubc94\uc740 \ube44\uc815\uaddc\uc568\ubc94\uc758 \uc131\uaca9\uc744 \ub760\uace0 \uc788\uc5c8\uc73c\uba70, \uc774\ud6c4\ub85c \uc815\uaddc\uae09 \uc568\ubc94\uc5d0 \ub300\ud55c \uacc4\ud68d\uc774 \uc5c6\ub2e4\uace0 \ubc1d\ud600 \ud32c\ub4e4 \uc0ac\uc774\uc5d0\uc11c \ub17c\ub780\uc774 \ub418\uc5c8\ub2e4. \ub354\ubd88\uc5b4 \uc774 \uc568\ubc94\uc740 \uadf8\uac00 \ube45\ub51c \ub808\ucf54\ub4dc\uc5d0\uc11c \ud55c \ub9c8\uc9c0\ub9c9 \ud65c\ub3d9\uc73c\ub85c, 2009\ub144 \ube45\ub51c \uc2a4\ucffc\ub4dc\uc758 \ucc3d\ub2e8 \uc2dc \uadf8\ub294 \ud568\uaed8\ud558\uc9c0 \uc54a\uc73c\uba74\uc11c \ube45\ub51c\uacfc\uc758 \uc778\uc5f0\ub3c4 \ub05d\uc774 \ub0ac\ub2e4.", "answer": "The Black Album", "sentence": "\uace7\ubc14\ub85c \uadf8\ub294 \ubc1c\ub9e4 \uc608\uc815\uc774\uc5c8\ub358 Jiggy Fellaz\uc758 \ucef4\ud544\ub808\uc774\uc158 \uc568\ubc94 The Black Album \uc5d0", "paragraph_sentence": "\uadf8 \ud6c4 2008\ub144 \ud55c\ud574 \uadf8\ub294 \uac89\uc73c\ub85c \ub4dc\ub7ec\ub098\ub294 \ud65c\ub3d9\uc740 INC\uc640 M&A \uc568\ubc94 \ucc38\uc5ec \uc815\ub3c4\uc5d0 \uadf8\ucce4\uc73c\ub098 \ucf00\uc774\ube14 TV, \ub77c\ub514\uc624, \uac8c\uc784 CF \uc74c\uc545 \ub4f1\uc744 \uc791\uace1\ud558\uba74\uc11c \uc880 \ub354 \uc791\uace1\uac00\ub85c\uc368\uc758 \uba74\ubaa8\ub97c \ub354\ud588\uace0, \ud2b9\ud788 2008\ub144 12\uc6d4 Jiggy Fellaz\uc758 \uba64\ubc84\uac00 \ub418\uc5c8\uc74c\uc744 \uacf5\ud45c\ud558\uc5ec \ud32c\ub4e4\uc744 \ub180\ub77c\uac8c \ud588\ub2e4. \uace7\ubc14\ub85c \uadf8\ub294 \ubc1c\ub9e4 \uc608\uc815\uc774\uc5c8\ub358 Jiggy Fellaz\uc758 \ucef4\ud544\ub808\uc774\uc158 \uc568\ubc94 The Black Album \uc5d0 \ud504\ub85c\ub4c0\uc2f1 \ubc0f \ub7a9\uc73c\ub85c \ucc38\uc5ec\ud558\uc600\ub2e4. \uc5bc\ub9c8 \uc9c0\ub098\uc9c0 \uc54a\uc544 \uadf8\uc758 \uc0c8 \uc568\ubc94 Premium Edition\uc774 \ubc1c\ub9e4\ub418\uc5c8\ub294\ub370, \uc774 \uc568\ubc94\uc740 \ube44\uc815\uaddc\uc568\ubc94\uc758 \uc131\uaca9\uc744 \ub760\uace0 \uc788\uc5c8\uc73c\uba70, \uc774\ud6c4\ub85c \uc815\uaddc\uae09 \uc568\ubc94\uc5d0 \ub300\ud55c \uacc4\ud68d\uc774 \uc5c6\ub2e4\uace0 \ubc1d\ud600 \ud32c\ub4e4 \uc0ac\uc774\uc5d0\uc11c \ub17c\ub780\uc774 \ub418\uc5c8\ub2e4. \ub354\ubd88\uc5b4 \uc774 \uc568\ubc94\uc740 \uadf8\uac00 \ube45\ub51c \ub808\ucf54\ub4dc\uc5d0\uc11c \ud55c \ub9c8\uc9c0\ub9c9 \ud65c\ub3d9\uc73c\ub85c, 2009\ub144 \ube45\ub51c \uc2a4\ucffc\ub4dc\uc758 \ucc3d\ub2e8 \uc2dc \uadf8\ub294 \ud568\uaed8\ud558\uc9c0 \uc54a\uc73c\uba74\uc11c \ube45\ub51c\uacfc\uc758 \uc778\uc5f0\ub3c4 \ub05d\uc774 \ub0ac\ub2e4.", "paragraph_answer": "\uadf8 \ud6c4 2008\ub144 \ud55c\ud574 \uadf8\ub294 \uac89\uc73c\ub85c \ub4dc\ub7ec\ub098\ub294 \ud65c\ub3d9\uc740 INC\uc640 M&A \uc568\ubc94 \ucc38\uc5ec \uc815\ub3c4\uc5d0 \uadf8\ucce4\uc73c\ub098 \ucf00\uc774\ube14 TV, \ub77c\ub514\uc624, \uac8c\uc784 CF \uc74c\uc545 \ub4f1\uc744 \uc791\uace1\ud558\uba74\uc11c \uc880 \ub354 \uc791\uace1\uac00\ub85c\uc368\uc758 \uba74\ubaa8\ub97c \ub354\ud588\uace0, \ud2b9\ud788 2008\ub144 12\uc6d4 Jiggy Fellaz\uc758 \uba64\ubc84\uac00 \ub418\uc5c8\uc74c\uc744 \uacf5\ud45c\ud558\uc5ec \ud32c\ub4e4\uc744 \ub180\ub77c\uac8c \ud588\ub2e4. \uace7\ubc14\ub85c \uadf8\ub294 \ubc1c\ub9e4 \uc608\uc815\uc774\uc5c8\ub358 Jiggy Fellaz\uc758 \ucef4\ud544\ub808\uc774\uc158 \uc568\ubc94 The Black Album \uc5d0 \ud504\ub85c\ub4c0\uc2f1 \ubc0f \ub7a9\uc73c\ub85c \ucc38\uc5ec\ud558\uc600\ub2e4. \uc5bc\ub9c8 \uc9c0\ub098\uc9c0 \uc54a\uc544 \uadf8\uc758 \uc0c8 \uc568\ubc94 Premium Edition\uc774 \ubc1c\ub9e4\ub418\uc5c8\ub294\ub370, \uc774 \uc568\ubc94\uc740 \ube44\uc815\uaddc\uc568\ubc94\uc758 \uc131\uaca9\uc744 \ub760\uace0 \uc788\uc5c8\uc73c\uba70, \uc774\ud6c4\ub85c \uc815\uaddc\uae09 \uc568\ubc94\uc5d0 \ub300\ud55c \uacc4\ud68d\uc774 \uc5c6\ub2e4\uace0 \ubc1d\ud600 \ud32c\ub4e4 \uc0ac\uc774\uc5d0\uc11c \ub17c\ub780\uc774 \ub418\uc5c8\ub2e4. \ub354\ubd88\uc5b4 \uc774 \uc568\ubc94\uc740 \uadf8\uac00 \ube45\ub51c \ub808\ucf54\ub4dc\uc5d0\uc11c \ud55c \ub9c8\uc9c0\ub9c9 \ud65c\ub3d9\uc73c\ub85c, 2009\ub144 \ube45\ub51c \uc2a4\ucffc\ub4dc\uc758 \ucc3d\ub2e8 \uc2dc \uadf8\ub294 \ud568\uaed8\ud558\uc9c0 \uc54a\uc73c\uba74\uc11c \ube45\ub51c\uacfc\uc758 \uc778\uc5f0\ub3c4 \ub05d\uc774 \ub0ac\ub2e4.", "sentence_answer": "\uace7\ubc14\ub85c \uadf8\ub294 \ubc1c\ub9e4 \uc608\uc815\uc774\uc5c8\ub358 Jiggy Fellaz\uc758 \ucef4\ud544\ub808\uc774\uc158 \uc568\ubc94 The Black Album \uc5d0", "paragraph_id": "6498982-2-1", "paragraph_question": "question: \ub9c8\ub974\ucf54\uac00 \ud504\ub85c\ub4c0\uc2f1 \ubc0f \ub7a9\uc73c\ub85c \ucc38\uc5ec\ud55c \ud799\ud569 \ud06c\ub8e8 \uc9c0\uae30 \ud3a0\ub77c\uc988\uc758 \uc568\ubc94 \uba85\uc740?, context: \uadf8 \ud6c4 2008\ub144 \ud55c\ud574 \uadf8\ub294 \uac89\uc73c\ub85c \ub4dc\ub7ec\ub098\ub294 \ud65c\ub3d9\uc740 INC\uc640 M&A \uc568\ubc94 \ucc38\uc5ec \uc815\ub3c4\uc5d0 \uadf8\ucce4\uc73c\ub098 \ucf00\uc774\ube14 TV, \ub77c\ub514\uc624, \uac8c\uc784 CF \uc74c\uc545 \ub4f1\uc744 \uc791\uace1\ud558\uba74\uc11c \uc880 \ub354 \uc791\uace1\uac00\ub85c\uc368\uc758 \uba74\ubaa8\ub97c \ub354\ud588\uace0, \ud2b9\ud788 2008\ub144 12\uc6d4 Jiggy Fellaz\uc758 \uba64\ubc84\uac00 \ub418\uc5c8\uc74c\uc744 \uacf5\ud45c\ud558\uc5ec \ud32c\ub4e4\uc744 \ub180\ub77c\uac8c \ud588\ub2e4. \uace7\ubc14\ub85c \uadf8\ub294 \ubc1c\ub9e4 \uc608\uc815\uc774\uc5c8\ub358 Jiggy Fellaz\uc758 \ucef4\ud544\ub808\uc774\uc158 \uc568\ubc94 The Black Album\uc5d0 \ud504\ub85c\ub4c0\uc2f1 \ubc0f \ub7a9\uc73c\ub85c \ucc38\uc5ec\ud558\uc600\ub2e4. \uc5bc\ub9c8 \uc9c0\ub098\uc9c0 \uc54a\uc544 \uadf8\uc758 \uc0c8 \uc568\ubc94 Premium Edition\uc774 \ubc1c\ub9e4\ub418\uc5c8\ub294\ub370, \uc774 \uc568\ubc94\uc740 \ube44\uc815\uaddc\uc568\ubc94\uc758 \uc131\uaca9\uc744 \ub760\uace0 \uc788\uc5c8\uc73c\uba70, \uc774\ud6c4\ub85c \uc815\uaddc\uae09 \uc568\ubc94\uc5d0 \ub300\ud55c \uacc4\ud68d\uc774 \uc5c6\ub2e4\uace0 \ubc1d\ud600 \ud32c\ub4e4 \uc0ac\uc774\uc5d0\uc11c \ub17c\ub780\uc774 \ub418\uc5c8\ub2e4. \ub354\ubd88\uc5b4 \uc774 \uc568\ubc94\uc740 \uadf8\uac00 \ube45\ub51c \ub808\ucf54\ub4dc\uc5d0\uc11c \ud55c \ub9c8\uc9c0\ub9c9 \ud65c\ub3d9\uc73c\ub85c, 2009\ub144 \ube45\ub51c \uc2a4\ucffc\ub4dc\uc758 \ucc3d\ub2e8 \uc2dc \uadf8\ub294 \ud568\uaed8\ud558\uc9c0 \uc54a\uc73c\uba74\uc11c \ube45\ub51c\uacfc\uc758 \uc778\uc5f0\ub3c4 \ub05d\uc774 \ub0ac\ub2e4."} {"question": "\uae09\uc131\uc911\uc774\uc5fc\uc5d0 \uc0ac\uc6a9\ub418\ub294 2\ucc28 \ud56d\uc0dd\uc81c\ub294?", "paragraph": "\ucd08\uae30\uc5d0 \ucda9\ubd84\ud55c \uc591\uc758 \ud56d\uc0dd\uc81c\ub97c \uc0ac\uc6a9\ud558\uac8c \ub418\uba74 \uae09\uc131 \uc911\uc774\uc5fc\uc744 \uce58\uc720\ud558\uace0 \ud569\ubcd1\uc99d\uc744 \ubc29\uc9c0\ud558\ub294\ub370 \ub3c4\uc6c0\uc774 \ub418\ub294 \uac83\uc740 \ub2f9\uc5f0\ud558\uc9c0\ub9cc, \ucd5c\uadfc\uc758 \uce58\ub8cc \ubc29\uce68\uc740 \ud56d\uc0dd\uc81c \ub0b4\uc131\uc744 \ucd5c\uc18c\ud654\ud560 \uc218 \uc788\ub294 \uc804 \uc138\uacc4\uc801 \uacf5\uc870 \uccb4\uc81c\ub97c \uac16\ucd94\ub294 \uac83\uc774\ub2e4. \uadf8\ub7ec\ub098 \ud3d0\ub834\uad6c\uade0\uc758 \ud398\ub2c8\uc2e4\ub9b0 \ub0b4\uc131\uade0 \ube44\uc728\uc740 \ubbf8\uad6d\uc758 \uacbd\uc6b0 10~40%\ub85c \ubcf4\uace0\ub418\ub294 \ub370 \ube44\ud574\uc11c, \ub124\ub35c\ub780\ub4dc\ub294 1% \ubbf8\ub9cc\uc73c\ub85c \ubcf4\uace0\ub418\ub294 \ub4f1, \uad6d\uac00\ubcc4 \uc758\ub8cc \uc2dc\uc2a4\ud15c\uc5d0 \ub530\ub77c \ucc28\uc774\ub97c \ubcf4\uc778\ub2e4. \uc601\ubb38 \uad50\uacfc\uc11c\uc5d0\ub294 1\ucc28 \ud56d\uc0dd\uc81c\uc778 \ud398\ub2c8\uc2e4\ub9b0 \uc0ac\uc6a9\uc774 \uc6d0\uce59\uc774\ub098, \ud55c\uad6d\uc740 \ud3d0\ub834\uad6c\uade0\uc758 \ub0b4\uc131\uc774 80% \uc774\uc0c1\uc73c\ub85c \ubcf4\uace0\ub418\ubbc0\ub85c \ub300\ubd80\ubd84 2\ucc28 \ud56d\uc0dd\uc81c \ub610\ub294 \uace0\uc6a9\ub7c9 \uc544\ubaa9\uc2dc\uc2e4\ub9b0(80~90mg/kg/day) \ubcf5\ud569\uc81c\uac00 \uc0ac\uc6a9\ub41c\ub2e4. \ubb3c\ub860 \ubbf8\uad6d\uc5d0\uc11c\ub3c4 1998\ub144\ubd80\ud130 2004\ub144\uae4c\uc9c0 \uae09\uc131\uc911\uc774\uc5fc\uc73c\ub85c \ud56d\uc0dd\uc81c\ub97c \ucc98\ubc29 \ubc1b\uc740 \ud658\uc790\ub97c \ud6c4\ud5a5 \ubd84\uc11d\ud55c \uacb0\uacfc \ub098\uc774, \uc9c8\ubcd1\uc758 \uc2ec\ud55c \uc815\ub3c4\uc640 \uc0c1\uad00\uc5c6\uc774 \uc810\ucc28 \ub113\uc740 \ubc94\uc704\uc758 \ud56d\uade0\ub825\uc744 \uac00\uc9c0\ub294 \ud56d\uc0dd\uc81c\uc758 \ucc98\ubc29\uc774 \ub298\uace0 \uc788\ub2e4\ub294 \ubb38\uc81c\uc810\uc774 2009\ub144 \ubcf4\uace0\ub41c \ubc14 \uc788\ub2e4. \ub530\ub77c\uc11c \uc138\ud30c\uacc4 \ud56d\uc0dd\uc81c \uc911 Cephalexin, Cefaclor, Cefixime \ub4f1\uc740 \ub0b4\uc131 \ud3d0\ub834\uad6c\uade0\uc5d0 \ud6a8\uacfc\uac00 \uc5c6\ub2e4\uace0 \uc870\uc0ac\ub418\uc5b4 \ud06c\uac8c \uad8c\uc7a5\ub418\uc9c0 \uc54a\uace0 \uc788\uc73c\uba70, Cefdnir, Cefpodoxime, Cefuroxime \ub4f1\uc758 \uc0ac\uc6a9\uc774 \ub298\uc5b4\ub098\uace0 \uc788\ub2e4.", "answer": "\uc544\ubaa9\uc2dc\uc2e4\ub9b0", "sentence": "\ub300\ubd80\ubd84 2\ucc28 \ud56d\uc0dd\uc81c \ub610\ub294 \uace0\uc6a9\ub7c9 \uc544\ubaa9\uc2dc\uc2e4\ub9b0 (80~90mg/kg/day) \ubcf5\ud569\uc81c\uac00 \uc0ac\uc6a9\ub41c\ub2e4.", "paragraph_sentence": "\ucd08\uae30\uc5d0 \ucda9\ubd84\ud55c \uc591\uc758 \ud56d\uc0dd\uc81c\ub97c \uc0ac\uc6a9\ud558\uac8c \ub418\uba74 \uae09\uc131 \uc911\uc774\uc5fc\uc744 \uce58\uc720\ud558\uace0 \ud569\ubcd1\uc99d\uc744 \ubc29\uc9c0\ud558\ub294\ub370 \ub3c4\uc6c0\uc774 \ub418\ub294 \uac83\uc740 \ub2f9\uc5f0\ud558\uc9c0\ub9cc, \ucd5c\uadfc\uc758 \uce58\ub8cc \ubc29\uce68\uc740 \ud56d\uc0dd\uc81c \ub0b4\uc131\uc744 \ucd5c\uc18c\ud654\ud560 \uc218 \uc788\ub294 \uc804 \uc138\uacc4\uc801 \uacf5\uc870 \uccb4\uc81c\ub97c \uac16\ucd94\ub294 \uac83\uc774\ub2e4. \uadf8\ub7ec\ub098 \ud3d0\ub834\uad6c\uade0\uc758 \ud398\ub2c8\uc2e4\ub9b0 \ub0b4\uc131\uade0 \ube44\uc728\uc740 \ubbf8\uad6d\uc758 \uacbd\uc6b0 10~40%\ub85c \ubcf4\uace0\ub418\ub294 \ub370 \ube44\ud574\uc11c, \ub124\ub35c\ub780\ub4dc\ub294 1% \ubbf8\ub9cc\uc73c\ub85c \ubcf4\uace0\ub418\ub294 \ub4f1, \uad6d\uac00\ubcc4 \uc758\ub8cc \uc2dc\uc2a4\ud15c\uc5d0 \ub530\ub77c \ucc28\uc774\ub97c \ubcf4\uc778\ub2e4. \uc601\ubb38 \uad50\uacfc\uc11c\uc5d0\ub294 1\ucc28 \ud56d\uc0dd\uc81c\uc778 \ud398\ub2c8\uc2e4\ub9b0 \uc0ac\uc6a9\uc774 \uc6d0\uce59\uc774\ub098, \ud55c\uad6d\uc740 \ud3d0\ub834\uad6c\uade0\uc758 \ub0b4\uc131\uc774 80% \uc774\uc0c1\uc73c\ub85c \ubcf4\uace0\ub418\ubbc0\ub85c \ub300\ubd80\ubd84 2\ucc28 \ud56d\uc0dd\uc81c \ub610\ub294 \uace0\uc6a9\ub7c9 \uc544\ubaa9\uc2dc\uc2e4\ub9b0 (80~90mg/kg/day) \ubcf5\ud569\uc81c\uac00 \uc0ac\uc6a9\ub41c\ub2e4. \ubb3c\ub860 \ubbf8\uad6d\uc5d0\uc11c\ub3c4 1998\ub144\ubd80\ud130 2004\ub144\uae4c\uc9c0 \uae09\uc131\uc911\uc774\uc5fc\uc73c\ub85c \ud56d\uc0dd\uc81c\ub97c \ucc98\ubc29 \ubc1b\uc740 \ud658\uc790\ub97c \ud6c4\ud5a5 \ubd84\uc11d\ud55c \uacb0\uacfc \ub098\uc774, \uc9c8\ubcd1\uc758 \uc2ec\ud55c \uc815\ub3c4\uc640 \uc0c1\uad00\uc5c6\uc774 \uc810\ucc28 \ub113\uc740 \ubc94\uc704\uc758 \ud56d\uade0\ub825\uc744 \uac00\uc9c0\ub294 \ud56d\uc0dd\uc81c\uc758 \ucc98\ubc29\uc774 \ub298\uace0 \uc788\ub2e4\ub294 \ubb38\uc81c\uc810\uc774 2009\ub144 \ubcf4\uace0\ub41c \ubc14 \uc788\ub2e4. \ub530\ub77c\uc11c \uc138\ud30c\uacc4 \ud56d\uc0dd\uc81c \uc911 Cephalexin, Cefaclor, Cefixime \ub4f1\uc740 \ub0b4\uc131 \ud3d0\ub834\uad6c\uade0\uc5d0 \ud6a8\uacfc\uac00 \uc5c6\ub2e4\uace0 \uc870\uc0ac\ub418\uc5b4 \ud06c\uac8c \uad8c\uc7a5\ub418\uc9c0 \uc54a\uace0 \uc788\uc73c\uba70, Cefdnir, Cefpodoxime, Cefuroxime \ub4f1\uc758 \uc0ac\uc6a9\uc774 \ub298\uc5b4\ub098\uace0 \uc788\ub2e4.", "paragraph_answer": "\ucd08\uae30\uc5d0 \ucda9\ubd84\ud55c \uc591\uc758 \ud56d\uc0dd\uc81c\ub97c \uc0ac\uc6a9\ud558\uac8c \ub418\uba74 \uae09\uc131 \uc911\uc774\uc5fc\uc744 \uce58\uc720\ud558\uace0 \ud569\ubcd1\uc99d\uc744 \ubc29\uc9c0\ud558\ub294\ub370 \ub3c4\uc6c0\uc774 \ub418\ub294 \uac83\uc740 \ub2f9\uc5f0\ud558\uc9c0\ub9cc, \ucd5c\uadfc\uc758 \uce58\ub8cc \ubc29\uce68\uc740 \ud56d\uc0dd\uc81c \ub0b4\uc131\uc744 \ucd5c\uc18c\ud654\ud560 \uc218 \uc788\ub294 \uc804 \uc138\uacc4\uc801 \uacf5\uc870 \uccb4\uc81c\ub97c \uac16\ucd94\ub294 \uac83\uc774\ub2e4. \uadf8\ub7ec\ub098 \ud3d0\ub834\uad6c\uade0\uc758 \ud398\ub2c8\uc2e4\ub9b0 \ub0b4\uc131\uade0 \ube44\uc728\uc740 \ubbf8\uad6d\uc758 \uacbd\uc6b0 10~40%\ub85c \ubcf4\uace0\ub418\ub294 \ub370 \ube44\ud574\uc11c, \ub124\ub35c\ub780\ub4dc\ub294 1% \ubbf8\ub9cc\uc73c\ub85c \ubcf4\uace0\ub418\ub294 \ub4f1, \uad6d\uac00\ubcc4 \uc758\ub8cc \uc2dc\uc2a4\ud15c\uc5d0 \ub530\ub77c \ucc28\uc774\ub97c \ubcf4\uc778\ub2e4. \uc601\ubb38 \uad50\uacfc\uc11c\uc5d0\ub294 1\ucc28 \ud56d\uc0dd\uc81c\uc778 \ud398\ub2c8\uc2e4\ub9b0 \uc0ac\uc6a9\uc774 \uc6d0\uce59\uc774\ub098, \ud55c\uad6d\uc740 \ud3d0\ub834\uad6c\uade0\uc758 \ub0b4\uc131\uc774 80% \uc774\uc0c1\uc73c\ub85c \ubcf4\uace0\ub418\ubbc0\ub85c \ub300\ubd80\ubd84 2\ucc28 \ud56d\uc0dd\uc81c \ub610\ub294 \uace0\uc6a9\ub7c9 \uc544\ubaa9\uc2dc\uc2e4\ub9b0 (80~90mg/kg/day) \ubcf5\ud569\uc81c\uac00 \uc0ac\uc6a9\ub41c\ub2e4. \ubb3c\ub860 \ubbf8\uad6d\uc5d0\uc11c\ub3c4 1998\ub144\ubd80\ud130 2004\ub144\uae4c\uc9c0 \uae09\uc131\uc911\uc774\uc5fc\uc73c\ub85c \ud56d\uc0dd\uc81c\ub97c \ucc98\ubc29 \ubc1b\uc740 \ud658\uc790\ub97c \ud6c4\ud5a5 \ubd84\uc11d\ud55c \uacb0\uacfc \ub098\uc774, \uc9c8\ubcd1\uc758 \uc2ec\ud55c \uc815\ub3c4\uc640 \uc0c1\uad00\uc5c6\uc774 \uc810\ucc28 \ub113\uc740 \ubc94\uc704\uc758 \ud56d\uade0\ub825\uc744 \uac00\uc9c0\ub294 \ud56d\uc0dd\uc81c\uc758 \ucc98\ubc29\uc774 \ub298\uace0 \uc788\ub2e4\ub294 \ubb38\uc81c\uc810\uc774 2009\ub144 \ubcf4\uace0\ub41c \ubc14 \uc788\ub2e4. \ub530\ub77c\uc11c \uc138\ud30c\uacc4 \ud56d\uc0dd\uc81c \uc911 Cephalexin, Cefaclor, Cefixime \ub4f1\uc740 \ub0b4\uc131 \ud3d0\ub834\uad6c\uade0\uc5d0 \ud6a8\uacfc\uac00 \uc5c6\ub2e4\uace0 \uc870\uc0ac\ub418\uc5b4 \ud06c\uac8c \uad8c\uc7a5\ub418\uc9c0 \uc54a\uace0 \uc788\uc73c\uba70, Cefdnir, Cefpodoxime, Cefuroxime \ub4f1\uc758 \uc0ac\uc6a9\uc774 \ub298\uc5b4\ub098\uace0 \uc788\ub2e4.", "sentence_answer": "\ub300\ubd80\ubd84 2\ucc28 \ud56d\uc0dd\uc81c \ub610\ub294 \uace0\uc6a9\ub7c9 \uc544\ubaa9\uc2dc\uc2e4\ub9b0 (80~90mg/kg/day) \ubcf5\ud569\uc81c\uac00 \uc0ac\uc6a9\ub41c\ub2e4.", "paragraph_id": "6490200-0-1", "paragraph_question": "question: \uae09\uc131\uc911\uc774\uc5fc\uc5d0 \uc0ac\uc6a9\ub418\ub294 2\ucc28 \ud56d\uc0dd\uc81c\ub294?, context: \ucd08\uae30\uc5d0 \ucda9\ubd84\ud55c \uc591\uc758 \ud56d\uc0dd\uc81c\ub97c \uc0ac\uc6a9\ud558\uac8c \ub418\uba74 \uae09\uc131 \uc911\uc774\uc5fc\uc744 \uce58\uc720\ud558\uace0 \ud569\ubcd1\uc99d\uc744 \ubc29\uc9c0\ud558\ub294\ub370 \ub3c4\uc6c0\uc774 \ub418\ub294 \uac83\uc740 \ub2f9\uc5f0\ud558\uc9c0\ub9cc, \ucd5c\uadfc\uc758 \uce58\ub8cc \ubc29\uce68\uc740 \ud56d\uc0dd\uc81c \ub0b4\uc131\uc744 \ucd5c\uc18c\ud654\ud560 \uc218 \uc788\ub294 \uc804 \uc138\uacc4\uc801 \uacf5\uc870 \uccb4\uc81c\ub97c \uac16\ucd94\ub294 \uac83\uc774\ub2e4. \uadf8\ub7ec\ub098 \ud3d0\ub834\uad6c\uade0\uc758 \ud398\ub2c8\uc2e4\ub9b0 \ub0b4\uc131\uade0 \ube44\uc728\uc740 \ubbf8\uad6d\uc758 \uacbd\uc6b0 10~40%\ub85c \ubcf4\uace0\ub418\ub294 \ub370 \ube44\ud574\uc11c, \ub124\ub35c\ub780\ub4dc\ub294 1% \ubbf8\ub9cc\uc73c\ub85c \ubcf4\uace0\ub418\ub294 \ub4f1, \uad6d\uac00\ubcc4 \uc758\ub8cc \uc2dc\uc2a4\ud15c\uc5d0 \ub530\ub77c \ucc28\uc774\ub97c \ubcf4\uc778\ub2e4. \uc601\ubb38 \uad50\uacfc\uc11c\uc5d0\ub294 1\ucc28 \ud56d\uc0dd\uc81c\uc778 \ud398\ub2c8\uc2e4\ub9b0 \uc0ac\uc6a9\uc774 \uc6d0\uce59\uc774\ub098, \ud55c\uad6d\uc740 \ud3d0\ub834\uad6c\uade0\uc758 \ub0b4\uc131\uc774 80% \uc774\uc0c1\uc73c\ub85c \ubcf4\uace0\ub418\ubbc0\ub85c \ub300\ubd80\ubd84 2\ucc28 \ud56d\uc0dd\uc81c \ub610\ub294 \uace0\uc6a9\ub7c9 \uc544\ubaa9\uc2dc\uc2e4\ub9b0(80~90mg/kg/day) \ubcf5\ud569\uc81c\uac00 \uc0ac\uc6a9\ub41c\ub2e4. \ubb3c\ub860 \ubbf8\uad6d\uc5d0\uc11c\ub3c4 1998\ub144\ubd80\ud130 2004\ub144\uae4c\uc9c0 \uae09\uc131\uc911\uc774\uc5fc\uc73c\ub85c \ud56d\uc0dd\uc81c\ub97c \ucc98\ubc29 \ubc1b\uc740 \ud658\uc790\ub97c \ud6c4\ud5a5 \ubd84\uc11d\ud55c \uacb0\uacfc \ub098\uc774, \uc9c8\ubcd1\uc758 \uc2ec\ud55c \uc815\ub3c4\uc640 \uc0c1\uad00\uc5c6\uc774 \uc810\ucc28 \ub113\uc740 \ubc94\uc704\uc758 \ud56d\uade0\ub825\uc744 \uac00\uc9c0\ub294 \ud56d\uc0dd\uc81c\uc758 \ucc98\ubc29\uc774 \ub298\uace0 \uc788\ub2e4\ub294 \ubb38\uc81c\uc810\uc774 2009\ub144 \ubcf4\uace0\ub41c \ubc14 \uc788\ub2e4. \ub530\ub77c\uc11c \uc138\ud30c\uacc4 \ud56d\uc0dd\uc81c \uc911 Cephalexin, Cefaclor, Cefixime \ub4f1\uc740 \ub0b4\uc131 \ud3d0\ub834\uad6c\uade0\uc5d0 \ud6a8\uacfc\uac00 \uc5c6\ub2e4\uace0 \uc870\uc0ac\ub418\uc5b4 \ud06c\uac8c \uad8c\uc7a5\ub418\uc9c0 \uc54a\uace0 \uc788\uc73c\uba70, Cefdnir, Cefpodoxime, Cefuroxime \ub4f1\uc758 \uc0ac\uc6a9\uc774 \ub298\uc5b4\ub098\uace0 \uc788\ub2e4."} {"question": "\ubc15\uc815\ud76c\uac00 \uc77c\ubcf8\uc774 \uc81c2\ucc28 \uc138\uacc4\ub300\uc804\uc5d0\uc11c \ud328\ub9dd\ud560 \ub54c\uae4c\uc9c0 \uadfc\ubb34\ud588\ub358 \uacf3\uc740?", "paragraph": "\ubc15\uc815\ud76c\ub294 \ub9cc\uc8fc \ubcf4\ubcd1 \uc81c8\ub2e8\uc5d0\uc11c \uc77c\ubcf8\uc774 \uc81c2\ucc28 \uc138\uacc4\ub300\uc804\uc5d0\uc11c \ud328\ub9dd\ud560 \ub54c\uae4c\uc9c0 \uadfc\ubb34\ud558\uc600\ub2e4. 1945\ub144 8\uc6d4 15\uc77c \uad11\ubcf5\uc774 \ub418\uc790 \uc18c\uc18d \ubd80\ub300\uac00 \uc5c6\uc5b4\uc9c4 \ubc15\uc815\ud76c\ub294 9\uc6d4 21\uc77c \ub3d9\ub8cc\ub4e4\uacfc \ud568\uaed8 \ubca0\uc774\uc9d5 \ucabd\uc73c\ub85c \uac74\ub108\uac00, \uc7a5\uad50 \uacbd\ud5d8\uc790\ub97c \ucc3e\uace0 \uc788\ub358 \ud55c\uad6d\uad11\ubcf5\uad70\uc5d0 \ud3b8\uc785\ub418\uc5b4, \ubd81\uacbd\uc758 \uae40\ud559\uaddc\uac00 \uc9c0\ud718\ud558\ub294 \ud55c\uad6d\uad11\ubcf5\uad70 \uc81c3\uc9c0\ub300 \uc81c1\ub300\ub300 \uc81c2\uc911\ub300\uc7a5\uc5d0 \uc784\uba85\ub418\uc5b4 \uad11\ubcf5\uad70 \uc7a5\uad50\ub85c \ud65c\ub3d9\ud558\ub2e4\uac00 1946\ub144 5\uc6d4 8\uc77c \ubbf8\uad70 \uc218\uc1a1\uc120\uc744 \ud0c0\uace0 \ubd80\uc0b0\ud56d\uc73c\ub85c \uadc0\uad6d\ud558\uc600\ub2e4. \ube48\ud138\ud130\ub9ac \uc0c1\ud0dc\ub85c \ub3cc\uc544\uc628 \uadf8\ub97c \uace0\ud5a5\uc758 \uac00\uc871\ub3c4 \ubc18\uae30\ub294 \ub208\uce58\uac00 \uc544\ub2c8\uc5c8\ub2e4\uace0 \ud55c\ub2e4. \uc14b\uc9f8 \ud615 \ubc15\uc0c1\ud76c(\u6734\u76f8\u7199)\ub294 \u201c\uadf8\ub0e5 \uc120\uc0dd\uc9c8\uc774\ub098 \ud558\uba74 \uc88b\uc558\uc744 \uac78 \uad1c\ud788 \uace0\uc9d1\ub300\ub85c \ud588\ub2e4\uac00 \uac70\uc9c0\uac00 \ub418\uc5b4 \ub3cc\uc544\uc624\uc9c0 \uc54a\uc558\ub290\ub0d0?\u201d\uace0 \uba74\ubc15\uc744 \uc8fc\uc5c8\ub2e4\uace0 \ud55c\ub2e4.", "answer": "\ub9cc\uc8fc \ubcf4\ubcd1 \uc81c8\ub2e8", "sentence": "\ubc15\uc815\ud76c\ub294 \ub9cc\uc8fc \ubcf4\ubcd1 \uc81c8\ub2e8 \uc5d0\uc11c \uc77c\ubcf8\uc774 \uc81c2\ucc28 \uc138\uacc4\ub300\uc804\uc5d0\uc11c \ud328\ub9dd\ud560 \ub54c\uae4c\uc9c0 \uadfc\ubb34\ud558\uc600\ub2e4.", "paragraph_sentence": " \ubc15\uc815\ud76c\ub294 \ub9cc\uc8fc \ubcf4\ubcd1 \uc81c8\ub2e8 \uc5d0\uc11c \uc77c\ubcf8\uc774 \uc81c2\ucc28 \uc138\uacc4\ub300\uc804\uc5d0\uc11c \ud328\ub9dd\ud560 \ub54c\uae4c\uc9c0 \uadfc\ubb34\ud558\uc600\ub2e4. 1945\ub144 8\uc6d4 15\uc77c \uad11\ubcf5\uc774 \ub418\uc790 \uc18c\uc18d \ubd80\ub300\uac00 \uc5c6\uc5b4\uc9c4 \ubc15\uc815\ud76c\ub294 9\uc6d4 21\uc77c \ub3d9\ub8cc\ub4e4\uacfc \ud568\uaed8 \ubca0\uc774\uc9d5 \ucabd\uc73c\ub85c \uac74\ub108\uac00, \uc7a5\uad50 \uacbd\ud5d8\uc790\ub97c \ucc3e\uace0 \uc788\ub358 \ud55c\uad6d\uad11\ubcf5\uad70\uc5d0 \ud3b8\uc785\ub418\uc5b4, \ubd81\uacbd\uc758 \uae40\ud559\uaddc\uac00 \uc9c0\ud718\ud558\ub294 \ud55c\uad6d\uad11\ubcf5\uad70 \uc81c3\uc9c0\ub300 \uc81c1\ub300\ub300 \uc81c2\uc911\ub300\uc7a5\uc5d0 \uc784\uba85\ub418\uc5b4 \uad11\ubcf5\uad70 \uc7a5\uad50\ub85c \ud65c\ub3d9\ud558\ub2e4\uac00 1946\ub144 5\uc6d4 8\uc77c \ubbf8\uad70 \uc218\uc1a1\uc120\uc744 \ud0c0\uace0 \ubd80\uc0b0\ud56d\uc73c\ub85c \uadc0\uad6d\ud558\uc600\ub2e4. \ube48\ud138\ud130\ub9ac \uc0c1\ud0dc\ub85c \ub3cc\uc544\uc628 \uadf8\ub97c \uace0\ud5a5\uc758 \uac00\uc871\ub3c4 \ubc18\uae30\ub294 \ub208\uce58\uac00 \uc544\ub2c8\uc5c8\ub2e4\uace0 \ud55c\ub2e4. \uc14b\uc9f8 \ud615 \ubc15\uc0c1\ud76c(\u6734\u76f8\u7199)\ub294 \u201c\uadf8\ub0e5 \uc120\uc0dd\uc9c8\uc774\ub098 \ud558\uba74 \uc88b\uc558\uc744 \uac78 \uad1c\ud788 \uace0\uc9d1\ub300\ub85c \ud588\ub2e4\uac00 \uac70\uc9c0\uac00 \ub418\uc5b4 \ub3cc\uc544\uc624\uc9c0 \uc54a\uc558\ub290\ub0d0?\u201d\uace0 \uba74\ubc15\uc744 \uc8fc\uc5c8\ub2e4\uace0 \ud55c\ub2e4.", "paragraph_answer": "\ubc15\uc815\ud76c\ub294 \ub9cc\uc8fc \ubcf4\ubcd1 \uc81c8\ub2e8 \uc5d0\uc11c \uc77c\ubcf8\uc774 \uc81c2\ucc28 \uc138\uacc4\ub300\uc804\uc5d0\uc11c \ud328\ub9dd\ud560 \ub54c\uae4c\uc9c0 \uadfc\ubb34\ud558\uc600\ub2e4. 1945\ub144 8\uc6d4 15\uc77c \uad11\ubcf5\uc774 \ub418\uc790 \uc18c\uc18d \ubd80\ub300\uac00 \uc5c6\uc5b4\uc9c4 \ubc15\uc815\ud76c\ub294 9\uc6d4 21\uc77c \ub3d9\ub8cc\ub4e4\uacfc \ud568\uaed8 \ubca0\uc774\uc9d5 \ucabd\uc73c\ub85c \uac74\ub108\uac00, \uc7a5\uad50 \uacbd\ud5d8\uc790\ub97c \ucc3e\uace0 \uc788\ub358 \ud55c\uad6d\uad11\ubcf5\uad70\uc5d0 \ud3b8\uc785\ub418\uc5b4, \ubd81\uacbd\uc758 \uae40\ud559\uaddc\uac00 \uc9c0\ud718\ud558\ub294 \ud55c\uad6d\uad11\ubcf5\uad70 \uc81c3\uc9c0\ub300 \uc81c1\ub300\ub300 \uc81c2\uc911\ub300\uc7a5\uc5d0 \uc784\uba85\ub418\uc5b4 \uad11\ubcf5\uad70 \uc7a5\uad50\ub85c \ud65c\ub3d9\ud558\ub2e4\uac00 1946\ub144 5\uc6d4 8\uc77c \ubbf8\uad70 \uc218\uc1a1\uc120\uc744 \ud0c0\uace0 \ubd80\uc0b0\ud56d\uc73c\ub85c \uadc0\uad6d\ud558\uc600\ub2e4. \ube48\ud138\ud130\ub9ac \uc0c1\ud0dc\ub85c \ub3cc\uc544\uc628 \uadf8\ub97c \uace0\ud5a5\uc758 \uac00\uc871\ub3c4 \ubc18\uae30\ub294 \ub208\uce58\uac00 \uc544\ub2c8\uc5c8\ub2e4\uace0 \ud55c\ub2e4. \uc14b\uc9f8 \ud615 \ubc15\uc0c1\ud76c(\u6734\u76f8\u7199)\ub294 \u201c\uadf8\ub0e5 \uc120\uc0dd\uc9c8\uc774\ub098 \ud558\uba74 \uc88b\uc558\uc744 \uac78 \uad1c\ud788 \uace0\uc9d1\ub300\ub85c \ud588\ub2e4\uac00 \uac70\uc9c0\uac00 \ub418\uc5b4 \ub3cc\uc544\uc624\uc9c0 \uc54a\uc558\ub290\ub0d0?\u201d\uace0 \uba74\ubc15\uc744 \uc8fc\uc5c8\ub2e4\uace0 \ud55c\ub2e4.", "sentence_answer": "\ubc15\uc815\ud76c\ub294 \ub9cc\uc8fc \ubcf4\ubcd1 \uc81c8\ub2e8 \uc5d0\uc11c \uc77c\ubcf8\uc774 \uc81c2\ucc28 \uc138\uacc4\ub300\uc804\uc5d0\uc11c \ud328\ub9dd\ud560 \ub54c\uae4c\uc9c0 \uadfc\ubb34\ud558\uc600\ub2e4.", "paragraph_id": "6530241-13-1", "paragraph_question": "question: \ubc15\uc815\ud76c\uac00 \uc77c\ubcf8\uc774 \uc81c2\ucc28 \uc138\uacc4\ub300\uc804\uc5d0\uc11c \ud328\ub9dd\ud560 \ub54c\uae4c\uc9c0 \uadfc\ubb34\ud588\ub358 \uacf3\uc740?, context: \ubc15\uc815\ud76c\ub294 \ub9cc\uc8fc \ubcf4\ubcd1 \uc81c8\ub2e8\uc5d0\uc11c \uc77c\ubcf8\uc774 \uc81c2\ucc28 \uc138\uacc4\ub300\uc804\uc5d0\uc11c \ud328\ub9dd\ud560 \ub54c\uae4c\uc9c0 \uadfc\ubb34\ud558\uc600\ub2e4. 1945\ub144 8\uc6d4 15\uc77c \uad11\ubcf5\uc774 \ub418\uc790 \uc18c\uc18d \ubd80\ub300\uac00 \uc5c6\uc5b4\uc9c4 \ubc15\uc815\ud76c\ub294 9\uc6d4 21\uc77c \ub3d9\ub8cc\ub4e4\uacfc \ud568\uaed8 \ubca0\uc774\uc9d5 \ucabd\uc73c\ub85c \uac74\ub108\uac00, \uc7a5\uad50 \uacbd\ud5d8\uc790\ub97c \ucc3e\uace0 \uc788\ub358 \ud55c\uad6d\uad11\ubcf5\uad70\uc5d0 \ud3b8\uc785\ub418\uc5b4, \ubd81\uacbd\uc758 \uae40\ud559\uaddc\uac00 \uc9c0\ud718\ud558\ub294 \ud55c\uad6d\uad11\ubcf5\uad70 \uc81c3\uc9c0\ub300 \uc81c1\ub300\ub300 \uc81c2\uc911\ub300\uc7a5\uc5d0 \uc784\uba85\ub418\uc5b4 \uad11\ubcf5\uad70 \uc7a5\uad50\ub85c \ud65c\ub3d9\ud558\ub2e4\uac00 1946\ub144 5\uc6d4 8\uc77c \ubbf8\uad70 \uc218\uc1a1\uc120\uc744 \ud0c0\uace0 \ubd80\uc0b0\ud56d\uc73c\ub85c \uadc0\uad6d\ud558\uc600\ub2e4. \ube48\ud138\ud130\ub9ac \uc0c1\ud0dc\ub85c \ub3cc\uc544\uc628 \uadf8\ub97c \uace0\ud5a5\uc758 \uac00\uc871\ub3c4 \ubc18\uae30\ub294 \ub208\uce58\uac00 \uc544\ub2c8\uc5c8\ub2e4\uace0 \ud55c\ub2e4. \uc14b\uc9f8 \ud615 \ubc15\uc0c1\ud76c(\u6734\u76f8\u7199)\ub294 \u201c\uadf8\ub0e5 \uc120\uc0dd\uc9c8\uc774\ub098 \ud558\uba74 \uc88b\uc558\uc744 \uac78 \uad1c\ud788 \uace0\uc9d1\ub300\ub85c \ud588\ub2e4\uac00 \uac70\uc9c0\uac00 \ub418\uc5b4 \ub3cc\uc544\uc624\uc9c0 \uc54a\uc558\ub290\ub0d0?\u201d\uace0 \uba74\ubc15\uc744 \uc8fc\uc5c8\ub2e4\uace0 \ud55c\ub2e4."} {"question": "\uc131\ud654\uc0c1 \uae08\uc9c0\ub839\uc740 \uc5b8\uc81c \ub0b4\ub824\uc84c\ub098?", "paragraph": "726\ub144\uacbd, \ub808\uc628 3\uc138\ub294 \uc131\ud654\uc0c1 \uc81c\uc791 \ubc0f \uc18c\uc720\uc5d0 \ubc18\ub300\ud55c\ub2e4\ub294 \uc131\ud654\uc0c1 \uae08\uc9c0\ub839\uc744 \ud3ec\uace0\ud558\uc600\ub2e4. \ub808\uc628 3\uc138\ub294 \uc131\ud654\uc0c1 \uae08\uc9c0\ub839\uc744 \ub85c\ub9c8\ub97c \ube44\ub86f\ud55c \uc11c\ubc29\uc5d0\uae4c\uc9c0 \uac15\ud589\ud558\uc9c0\ub294 \uc54a\uc558\uc9c0\ub9cc, \uadf8\ub808\uace0\ub9ac\uc624 2\uc138\ub294 \uc989\uc2dc \uc774 \uce59\ub839\uc744 \uac70\ubd80\ud55c\ub2e4\ub294 \uc758\uc0ac\ub97c \ubc1d\ud614\ub2e4. \ud55c\ud3b8 \ud669\uc81c\uac00 \uc131\ud654\uc0c1 \uae08\uc9c0\ub839\uc744 \ud3ec\uace0\ud588\ub2e4\ub294 \uc18c\uc2dd\uc5d0 \ub77c\ubca4\ub098 \ucd1d\ub3c5\ubd80\uc5d0\uc11c \uc989\uc2dc \ubc18\ub780\uc774 \uc77c\uc5b4\ub0ac\ub2e4. \ub77c\ubca4\ub098\uc640 \ud39c\ud0c0\ud3f4\ub9ac\uc2a4 \uacf5\uad6d\uc758 \ubcd1\uc0ac\ub4e4\uc774 \uaddc\ud569\ud558\uc5ec \ud30c\uc6b8\ub85c\uc2a4 \ucd1d\ub3c5\uacfc \ub808\uc628 3\uc138 \ud669\uc81c\uc5d0 \ubc18\ub300\ud558\uba70 \uadf8\ub4e4\uc5d0\uac8c \ucda9\uc131\uc2a4\ub7ec\uc6b4 \uad00\ub9ac\ub4e4\uc744 \ucad3\uc544\ub0b8 \uac83\uc774\ub2e4. \ud30c\uc6b8\ub85c\uc2a4\ub294 \ud669\uc81c\ud30c\uc758 \ud798\uc744 \uaddc\ud569\ud558\uc5ec \ubc18\ub780\uc744 \uc9c4\uc555\ud558\ub824\uace0 \ud588\uc9c0\ub9cc, \uc0b4\ud574\ub2f9\ud588\ub2e4. \ubc18\ub780\uad70\uc740 \uc790\uccb4\uc801\uc73c\ub85c \uc0c8 \ud669\uc81c\ub97c \uc120\ucd9c\ud558\uace0 \ucf58\uc2a4\ud0c4\ud2f0\ub178\ud3f4\ub9ac\uc2a4\ub85c \uc9c4\uad70\ud558\uace0\uc790 \ud558\uc600\uc9c0\ub9cc, \uadf8\ub808\uace0\ub9ac\uc624 2\uc138\ub294 \ub808\uc628 3\uc138\uc5d0\uac8c \ub300\ud56d\ud558\ub294 \ud589\ub3d9\uc744 \ud558\uc9c0 \ub9d0 \uac83\uc744 \uad8c\uace0\ud558\uc600\ub2e4. \uac19\uc740 \uc2dc\uae30\uc5d0 \uc2a4\uc2a4\ub85c \uacf5\uc791\uc774\ub77c\uace0 \uc790\uce6d\ud55c \uc5d1\uc2a4\ud790\ub77c\ub77c\ud22c\uc2a4\uc640 \uadf8\uc758 \uc544\ub4e4 \ud558\ub4dc\ub9ac\uc544\ub204\uc2a4\uac00 \ub098\ud3f4\ub9ac\uc5d0\uc11c \ubc18\ub780\uc744 \uc77c\uc73c\ud0a8 \ud6c4\uc5d0 \ud669\uc81c\uc758 \ud3b8\uc5d0 \uc11c\uc11c \uadf8\ub808\uace0\ub9ac\uc624 2\uc138\ub97c \uc8fd\uc774\uae30 \uc704\ud574 \ub85c\ub9c8\ub85c \ud5a5\ud588\uc9c0\ub9cc, \uc2dc\ubbfc\uad70\uc5d0 \uc758\ud574 \ud0c0\ub3c4\ub418\uc5b4 \uc0ac\ub9dd\ud588\ub2e4.", "answer": "726\ub144\uacbd", "sentence": "726\ub144\uacbd ,", "paragraph_sentence": " 726\ub144\uacbd , \ub808\uc628 3\uc138\ub294 \uc131\ud654\uc0c1 \uc81c\uc791 \ubc0f \uc18c\uc720\uc5d0 \ubc18\ub300\ud55c\ub2e4\ub294 \uc131\ud654\uc0c1 \uae08\uc9c0\ub839\uc744 \ud3ec\uace0\ud558\uc600\ub2e4. \ub808\uc628 3\uc138\ub294 \uc131\ud654\uc0c1 \uae08\uc9c0\ub839\uc744 \ub85c\ub9c8\ub97c \ube44\ub86f\ud55c \uc11c\ubc29\uc5d0\uae4c\uc9c0 \uac15\ud589\ud558\uc9c0\ub294 \uc54a\uc558\uc9c0\ub9cc, \uadf8\ub808\uace0\ub9ac\uc624 2\uc138\ub294 \uc989\uc2dc \uc774 \uce59\ub839\uc744 \uac70\ubd80\ud55c\ub2e4\ub294 \uc758\uc0ac\ub97c \ubc1d\ud614\ub2e4. \ud55c\ud3b8 \ud669\uc81c\uac00 \uc131\ud654\uc0c1 \uae08\uc9c0\ub839\uc744 \ud3ec\uace0\ud588\ub2e4\ub294 \uc18c\uc2dd\uc5d0 \ub77c\ubca4\ub098 \ucd1d\ub3c5\ubd80\uc5d0\uc11c \uc989\uc2dc \ubc18\ub780\uc774 \uc77c\uc5b4\ub0ac\ub2e4. \ub77c\ubca4\ub098\uc640 \ud39c\ud0c0\ud3f4\ub9ac\uc2a4 \uacf5\uad6d\uc758 \ubcd1\uc0ac\ub4e4\uc774 \uaddc\ud569\ud558\uc5ec \ud30c\uc6b8\ub85c\uc2a4 \ucd1d\ub3c5\uacfc \ub808\uc628 3\uc138 \ud669\uc81c\uc5d0 \ubc18\ub300\ud558\uba70 \uadf8\ub4e4\uc5d0\uac8c \ucda9\uc131\uc2a4\ub7ec\uc6b4 \uad00\ub9ac\ub4e4\uc744 \ucad3\uc544\ub0b8 \uac83\uc774\ub2e4. \ud30c\uc6b8\ub85c\uc2a4\ub294 \ud669\uc81c\ud30c\uc758 \ud798\uc744 \uaddc\ud569\ud558\uc5ec \ubc18\ub780\uc744 \uc9c4\uc555\ud558\ub824\uace0 \ud588\uc9c0\ub9cc, \uc0b4\ud574\ub2f9\ud588\ub2e4. \ubc18\ub780\uad70\uc740 \uc790\uccb4\uc801\uc73c\ub85c \uc0c8 \ud669\uc81c\ub97c \uc120\ucd9c\ud558\uace0 \ucf58\uc2a4\ud0c4\ud2f0\ub178\ud3f4\ub9ac\uc2a4\ub85c \uc9c4\uad70\ud558\uace0\uc790 \ud558\uc600\uc9c0\ub9cc, \uadf8\ub808\uace0\ub9ac\uc624 2\uc138\ub294 \ub808\uc628 3\uc138\uc5d0\uac8c \ub300\ud56d\ud558\ub294 \ud589\ub3d9\uc744 \ud558\uc9c0 \ub9d0 \uac83\uc744 \uad8c\uace0\ud558\uc600\ub2e4. \uac19\uc740 \uc2dc\uae30\uc5d0 \uc2a4\uc2a4\ub85c \uacf5\uc791\uc774\ub77c\uace0 \uc790\uce6d\ud55c \uc5d1\uc2a4\ud790\ub77c\ub77c\ud22c\uc2a4\uc640 \uadf8\uc758 \uc544\ub4e4 \ud558\ub4dc\ub9ac\uc544\ub204\uc2a4\uac00 \ub098\ud3f4\ub9ac\uc5d0\uc11c \ubc18\ub780\uc744 \uc77c\uc73c\ud0a8 \ud6c4\uc5d0 \ud669\uc81c\uc758 \ud3b8\uc5d0 \uc11c\uc11c \uadf8\ub808\uace0\ub9ac\uc624 2\uc138\ub97c \uc8fd\uc774\uae30 \uc704\ud574 \ub85c\ub9c8\ub85c \ud5a5\ud588\uc9c0\ub9cc, \uc2dc\ubbfc\uad70\uc5d0 \uc758\ud574 \ud0c0\ub3c4\ub418\uc5b4 \uc0ac\ub9dd\ud588\ub2e4.", "paragraph_answer": " 726\ub144\uacbd , \ub808\uc628 3\uc138\ub294 \uc131\ud654\uc0c1 \uc81c\uc791 \ubc0f \uc18c\uc720\uc5d0 \ubc18\ub300\ud55c\ub2e4\ub294 \uc131\ud654\uc0c1 \uae08\uc9c0\ub839\uc744 \ud3ec\uace0\ud558\uc600\ub2e4. \ub808\uc628 3\uc138\ub294 \uc131\ud654\uc0c1 \uae08\uc9c0\ub839\uc744 \ub85c\ub9c8\ub97c \ube44\ub86f\ud55c \uc11c\ubc29\uc5d0\uae4c\uc9c0 \uac15\ud589\ud558\uc9c0\ub294 \uc54a\uc558\uc9c0\ub9cc, \uadf8\ub808\uace0\ub9ac\uc624 2\uc138\ub294 \uc989\uc2dc \uc774 \uce59\ub839\uc744 \uac70\ubd80\ud55c\ub2e4\ub294 \uc758\uc0ac\ub97c \ubc1d\ud614\ub2e4. \ud55c\ud3b8 \ud669\uc81c\uac00 \uc131\ud654\uc0c1 \uae08\uc9c0\ub839\uc744 \ud3ec\uace0\ud588\ub2e4\ub294 \uc18c\uc2dd\uc5d0 \ub77c\ubca4\ub098 \ucd1d\ub3c5\ubd80\uc5d0\uc11c \uc989\uc2dc \ubc18\ub780\uc774 \uc77c\uc5b4\ub0ac\ub2e4. \ub77c\ubca4\ub098\uc640 \ud39c\ud0c0\ud3f4\ub9ac\uc2a4 \uacf5\uad6d\uc758 \ubcd1\uc0ac\ub4e4\uc774 \uaddc\ud569\ud558\uc5ec \ud30c\uc6b8\ub85c\uc2a4 \ucd1d\ub3c5\uacfc \ub808\uc628 3\uc138 \ud669\uc81c\uc5d0 \ubc18\ub300\ud558\uba70 \uadf8\ub4e4\uc5d0\uac8c \ucda9\uc131\uc2a4\ub7ec\uc6b4 \uad00\ub9ac\ub4e4\uc744 \ucad3\uc544\ub0b8 \uac83\uc774\ub2e4. \ud30c\uc6b8\ub85c\uc2a4\ub294 \ud669\uc81c\ud30c\uc758 \ud798\uc744 \uaddc\ud569\ud558\uc5ec \ubc18\ub780\uc744 \uc9c4\uc555\ud558\ub824\uace0 \ud588\uc9c0\ub9cc, \uc0b4\ud574\ub2f9\ud588\ub2e4. \ubc18\ub780\uad70\uc740 \uc790\uccb4\uc801\uc73c\ub85c \uc0c8 \ud669\uc81c\ub97c \uc120\ucd9c\ud558\uace0 \ucf58\uc2a4\ud0c4\ud2f0\ub178\ud3f4\ub9ac\uc2a4\ub85c \uc9c4\uad70\ud558\uace0\uc790 \ud558\uc600\uc9c0\ub9cc, \uadf8\ub808\uace0\ub9ac\uc624 2\uc138\ub294 \ub808\uc628 3\uc138\uc5d0\uac8c \ub300\ud56d\ud558\ub294 \ud589\ub3d9\uc744 \ud558\uc9c0 \ub9d0 \uac83\uc744 \uad8c\uace0\ud558\uc600\ub2e4. \uac19\uc740 \uc2dc\uae30\uc5d0 \uc2a4\uc2a4\ub85c \uacf5\uc791\uc774\ub77c\uace0 \uc790\uce6d\ud55c \uc5d1\uc2a4\ud790\ub77c\ub77c\ud22c\uc2a4\uc640 \uadf8\uc758 \uc544\ub4e4 \ud558\ub4dc\ub9ac\uc544\ub204\uc2a4\uac00 \ub098\ud3f4\ub9ac\uc5d0\uc11c \ubc18\ub780\uc744 \uc77c\uc73c\ud0a8 \ud6c4\uc5d0 \ud669\uc81c\uc758 \ud3b8\uc5d0 \uc11c\uc11c \uadf8\ub808\uace0\ub9ac\uc624 2\uc138\ub97c \uc8fd\uc774\uae30 \uc704\ud574 \ub85c\ub9c8\ub85c \ud5a5\ud588\uc9c0\ub9cc, \uc2dc\ubbfc\uad70\uc5d0 \uc758\ud574 \ud0c0\ub3c4\ub418\uc5b4 \uc0ac\ub9dd\ud588\ub2e4.", "sentence_answer": " 726\ub144\uacbd ,", "paragraph_id": "6464899-5-1", "paragraph_question": "question: \uc131\ud654\uc0c1 \uae08\uc9c0\ub839\uc740 \uc5b8\uc81c \ub0b4\ub824\uc84c\ub098?, context: 726\ub144\uacbd, \ub808\uc628 3\uc138\ub294 \uc131\ud654\uc0c1 \uc81c\uc791 \ubc0f \uc18c\uc720\uc5d0 \ubc18\ub300\ud55c\ub2e4\ub294 \uc131\ud654\uc0c1 \uae08\uc9c0\ub839\uc744 \ud3ec\uace0\ud558\uc600\ub2e4. \ub808\uc628 3\uc138\ub294 \uc131\ud654\uc0c1 \uae08\uc9c0\ub839\uc744 \ub85c\ub9c8\ub97c \ube44\ub86f\ud55c \uc11c\ubc29\uc5d0\uae4c\uc9c0 \uac15\ud589\ud558\uc9c0\ub294 \uc54a\uc558\uc9c0\ub9cc, \uadf8\ub808\uace0\ub9ac\uc624 2\uc138\ub294 \uc989\uc2dc \uc774 \uce59\ub839\uc744 \uac70\ubd80\ud55c\ub2e4\ub294 \uc758\uc0ac\ub97c \ubc1d\ud614\ub2e4. \ud55c\ud3b8 \ud669\uc81c\uac00 \uc131\ud654\uc0c1 \uae08\uc9c0\ub839\uc744 \ud3ec\uace0\ud588\ub2e4\ub294 \uc18c\uc2dd\uc5d0 \ub77c\ubca4\ub098 \ucd1d\ub3c5\ubd80\uc5d0\uc11c \uc989\uc2dc \ubc18\ub780\uc774 \uc77c\uc5b4\ub0ac\ub2e4. \ub77c\ubca4\ub098\uc640 \ud39c\ud0c0\ud3f4\ub9ac\uc2a4 \uacf5\uad6d\uc758 \ubcd1\uc0ac\ub4e4\uc774 \uaddc\ud569\ud558\uc5ec \ud30c\uc6b8\ub85c\uc2a4 \ucd1d\ub3c5\uacfc \ub808\uc628 3\uc138 \ud669\uc81c\uc5d0 \ubc18\ub300\ud558\uba70 \uadf8\ub4e4\uc5d0\uac8c \ucda9\uc131\uc2a4\ub7ec\uc6b4 \uad00\ub9ac\ub4e4\uc744 \ucad3\uc544\ub0b8 \uac83\uc774\ub2e4. \ud30c\uc6b8\ub85c\uc2a4\ub294 \ud669\uc81c\ud30c\uc758 \ud798\uc744 \uaddc\ud569\ud558\uc5ec \ubc18\ub780\uc744 \uc9c4\uc555\ud558\ub824\uace0 \ud588\uc9c0\ub9cc, \uc0b4\ud574\ub2f9\ud588\ub2e4. \ubc18\ub780\uad70\uc740 \uc790\uccb4\uc801\uc73c\ub85c \uc0c8 \ud669\uc81c\ub97c \uc120\ucd9c\ud558\uace0 \ucf58\uc2a4\ud0c4\ud2f0\ub178\ud3f4\ub9ac\uc2a4\ub85c \uc9c4\uad70\ud558\uace0\uc790 \ud558\uc600\uc9c0\ub9cc, \uadf8\ub808\uace0\ub9ac\uc624 2\uc138\ub294 \ub808\uc628 3\uc138\uc5d0\uac8c \ub300\ud56d\ud558\ub294 \ud589\ub3d9\uc744 \ud558\uc9c0 \ub9d0 \uac83\uc744 \uad8c\uace0\ud558\uc600\ub2e4. \uac19\uc740 \uc2dc\uae30\uc5d0 \uc2a4\uc2a4\ub85c \uacf5\uc791\uc774\ub77c\uace0 \uc790\uce6d\ud55c \uc5d1\uc2a4\ud790\ub77c\ub77c\ud22c\uc2a4\uc640 \uadf8\uc758 \uc544\ub4e4 \ud558\ub4dc\ub9ac\uc544\ub204\uc2a4\uac00 \ub098\ud3f4\ub9ac\uc5d0\uc11c \ubc18\ub780\uc744 \uc77c\uc73c\ud0a8 \ud6c4\uc5d0 \ud669\uc81c\uc758 \ud3b8\uc5d0 \uc11c\uc11c \uadf8\ub808\uace0\ub9ac\uc624 2\uc138\ub97c \uc8fd\uc774\uae30 \uc704\ud574 \ub85c\ub9c8\ub85c \ud5a5\ud588\uc9c0\ub9cc, \uc2dc\ubbfc\uad70\uc5d0 \uc758\ud574 \ud0c0\ub3c4\ub418\uc5b4 \uc0ac\ub9dd\ud588\ub2e4."} {"question": "\uc77c\ubcf8 \uc81c\uad6d \uc2dc\uc808 \ud669\ub3c4\ud30c \uae30\ud0c0 \uc787\ud0a4\ub85c\ubd80\ud130 \uae30\uc6cc\uc744 \ucc3e\uc744 \uc218 \uc788\ub294 \uac83\uc740?", "paragraph": "\uc77c\ubcf8\uc5d0\uc11c \uadf9\uc6b0 \uc6b4\ub3d9\uacfc \uc0ac\uc0c1\uc740 \uc77c\ubcf8 \uc81c\uad6d \uc2dc\uc808 \ub2f9\uc2dc \uc77c\ubcf8 \uc721\uad70\uc758 \ud30c\ubc8c\uc774\uc5c8\ub358 \ud669\ub3c4\ud30c, \ud1b5\uc81c\ud30c\uc640 \ud30c\uc2dc\uc2a4\ud2b8\uc600\ub358 \uae30\ud0c0 \uc787\ud0a4\ub85c\ubd80\ud130 \uadf8 \uae30\uc6d0\uc744 \ucc3e\uc744 \uc218 \uc788\ub2e4. \ub2f9\uc2dc \ud669\ub3c4\ud30c\ub294 \ubc18\uc790\ubcf8\uc8fc\uc758\uc640 \ud30c\uc2dc\uc998\uc744 \ubaa8\ud1a0\ub85c \ud55c \uad6d\uac00\uc8fc\uc758, \ud33d\ucc3d\uc8fc\uc758\ub97c \uae30\ubc18\uc73c\ub85c \ud588\uace0, \ud1b5\uc81c\ud30c\ub294 \uae30\uc874 \uc77c\ubcf8\uc2dd \uc790\ubcf8\uc8fc\uc758 \uccb4\uc81c\uc640 \uad70\uc8fc\uc81c \uccb4\uc81c\ub97c \uc720\uc9c0\ud558\uba74\uc11c \uc81c\uad6d\uc8fc\uc758\uc640 \uc804\uccb4\uc8fc\uc758\ub97c \uc628\uac74\uc801\uc73c\ub85c \uc2e4\ud589\ud574 \ub098\uac00\uc790\ub294 \uc785\uc7a5\uc774\uc5c8\ub2e4. \ub2e4\uc774\uc1fc \ub370\ubaa8\ud06c\ub77c\uc2dc \uc2dc\ub300\uac00 \ub05d\ub09c \uc774\ud6c4 \uadf9\uc6b0 \uc0ac\uc0c1\uc774 \uc804\uad6d\uc801\uc73c\ub85c \ud37c\uc9c0\uba74\uc11c, \uc774\ub4e4\uc758 \uc0ac\uc0c1\uc5d0 \ud798\uc774 \ub4e4\uc5b4\uac00\uae30 \uc2dc\uc791\ud588\ub2e4. 2\ucc28 \uc138\uacc4\ub300\uc804 \uc885\uc804 \ud6c4, \ub300\ubd80\ubd84\uc758 \uadf9\uc6b0 \uc9c0\uc9c0\uc790\ub4e4\uc740 \ucc98\ubc8c\uc744 \uba74\ud558\uc5ec \uc77c\ubcf8 \uc790\uc720\ub2f9\uc5d0 \uc785\ub2f9\ud558\uc600\uc73c\uba70, \uc624\ub298\ub0a0\uc5d0\ub294 \uadf8\ub4e4\uc758 \uc6d0\ub85c\uc801 \uc5ed\ud560\uc744 \ud588\ub358 \uc815\uce58\uc778\ub4e4\uc774 \uadf9\uc6b0 \uc6b4\ub3d9\uc758 \uc9c0\ub3c4\uc790\uc801 \uc704\uce58\ub97c \ucc28\uc9c0\ud558\uace0\uc788\ub2e4. \ud604\uc7ac \uc77c\ubcf8\uc740 \ub2e4\ub984\uc5c6\uc774 \uadf9\uc6b0\uc5d0 \ub300\ud55c \uc9c0\uc9c0\ub3c4\uac00 \uc0c1\ub2f9\ud788 \ub192\uc73c\uba70, \ud574\ub2f9 \ub2e8\uccb4\ub294 \uc140 \uc218 \uc5c6\uc744 \uc815\ub3c4\ub85c \ub9ce\ub2e4.", "answer": "\uadf9\uc6b0 \uc6b4\ub3d9\uacfc \uc0ac\uc0c1", "sentence": "\uc77c\ubcf8\uc5d0\uc11c \uadf9\uc6b0 \uc6b4\ub3d9\uacfc \uc0ac\uc0c1 \uc740 \uc77c\ubcf8 \uc81c\uad6d \uc2dc\uc808 \ub2f9\uc2dc \uc77c\ubcf8 \uc721\uad70\uc758 \ud30c\ubc8c\uc774\uc5c8\ub358 \ud669\ub3c4\ud30c, \ud1b5\uc81c\ud30c\uc640 \ud30c\uc2dc\uc2a4\ud2b8\uc600\ub358 \uae30\ud0c0 \uc787\ud0a4\ub85c\ubd80\ud130 \uadf8 \uae30\uc6d0\uc744 \ucc3e\uc744 \uc218 \uc788\ub2e4.", "paragraph_sentence": " \uc77c\ubcf8\uc5d0\uc11c \uadf9\uc6b0 \uc6b4\ub3d9\uacfc \uc0ac\uc0c1 \uc740 \uc77c\ubcf8 \uc81c\uad6d \uc2dc\uc808 \ub2f9\uc2dc \uc77c\ubcf8 \uc721\uad70\uc758 \ud30c\ubc8c\uc774\uc5c8\ub358 \ud669\ub3c4\ud30c, \ud1b5\uc81c\ud30c\uc640 \ud30c\uc2dc\uc2a4\ud2b8\uc600\ub358 \uae30\ud0c0 \uc787\ud0a4\ub85c\ubd80\ud130 \uadf8 \uae30\uc6d0\uc744 \ucc3e\uc744 \uc218 \uc788\ub2e4. \ub2f9\uc2dc \ud669\ub3c4\ud30c\ub294 \ubc18\uc790\ubcf8\uc8fc\uc758\uc640 \ud30c\uc2dc\uc998\uc744 \ubaa8\ud1a0\ub85c \ud55c \uad6d\uac00\uc8fc\uc758, \ud33d\ucc3d\uc8fc\uc758\ub97c \uae30\ubc18\uc73c\ub85c \ud588\uace0, \ud1b5\uc81c\ud30c\ub294 \uae30\uc874 \uc77c\ubcf8\uc2dd \uc790\ubcf8\uc8fc\uc758 \uccb4\uc81c\uc640 \uad70\uc8fc\uc81c \uccb4\uc81c\ub97c \uc720\uc9c0\ud558\uba74\uc11c \uc81c\uad6d\uc8fc\uc758\uc640 \uc804\uccb4\uc8fc\uc758\ub97c \uc628\uac74\uc801\uc73c\ub85c \uc2e4\ud589\ud574 \ub098\uac00\uc790\ub294 \uc785\uc7a5\uc774\uc5c8\ub2e4. \ub2e4\uc774\uc1fc \ub370\ubaa8\ud06c\ub77c\uc2dc \uc2dc\ub300\uac00 \ub05d\ub09c \uc774\ud6c4 \uadf9\uc6b0 \uc0ac\uc0c1\uc774 \uc804\uad6d\uc801\uc73c\ub85c \ud37c\uc9c0\uba74\uc11c, \uc774\ub4e4\uc758 \uc0ac\uc0c1\uc5d0 \ud798\uc774 \ub4e4\uc5b4\uac00\uae30 \uc2dc\uc791\ud588\ub2e4. 2\ucc28 \uc138\uacc4\ub300\uc804 \uc885\uc804 \ud6c4, \ub300\ubd80\ubd84\uc758 \uadf9\uc6b0 \uc9c0\uc9c0\uc790\ub4e4\uc740 \ucc98\ubc8c\uc744 \uba74\ud558\uc5ec \uc77c\ubcf8 \uc790\uc720\ub2f9\uc5d0 \uc785\ub2f9\ud558\uc600\uc73c\uba70, \uc624\ub298\ub0a0\uc5d0\ub294 \uadf8\ub4e4\uc758 \uc6d0\ub85c\uc801 \uc5ed\ud560\uc744 \ud588\ub358 \uc815\uce58\uc778\ub4e4\uc774 \uadf9\uc6b0 \uc6b4\ub3d9\uc758 \uc9c0\ub3c4\uc790\uc801 \uc704\uce58\ub97c \ucc28\uc9c0\ud558\uace0\uc788\ub2e4. \ud604\uc7ac \uc77c\ubcf8\uc740 \ub2e4\ub984\uc5c6\uc774 \uadf9\uc6b0\uc5d0 \ub300\ud55c \uc9c0\uc9c0\ub3c4\uac00 \uc0c1\ub2f9\ud788 \ub192\uc73c\uba70, \ud574\ub2f9 \ub2e8\uccb4\ub294 \uc140 \uc218 \uc5c6\uc744 \uc815\ub3c4\ub85c \ub9ce\ub2e4.", "paragraph_answer": "\uc77c\ubcf8\uc5d0\uc11c \uadf9\uc6b0 \uc6b4\ub3d9\uacfc \uc0ac\uc0c1 \uc740 \uc77c\ubcf8 \uc81c\uad6d \uc2dc\uc808 \ub2f9\uc2dc \uc77c\ubcf8 \uc721\uad70\uc758 \ud30c\ubc8c\uc774\uc5c8\ub358 \ud669\ub3c4\ud30c, \ud1b5\uc81c\ud30c\uc640 \ud30c\uc2dc\uc2a4\ud2b8\uc600\ub358 \uae30\ud0c0 \uc787\ud0a4\ub85c\ubd80\ud130 \uadf8 \uae30\uc6d0\uc744 \ucc3e\uc744 \uc218 \uc788\ub2e4. \ub2f9\uc2dc \ud669\ub3c4\ud30c\ub294 \ubc18\uc790\ubcf8\uc8fc\uc758\uc640 \ud30c\uc2dc\uc998\uc744 \ubaa8\ud1a0\ub85c \ud55c \uad6d\uac00\uc8fc\uc758, \ud33d\ucc3d\uc8fc\uc758\ub97c \uae30\ubc18\uc73c\ub85c \ud588\uace0, \ud1b5\uc81c\ud30c\ub294 \uae30\uc874 \uc77c\ubcf8\uc2dd \uc790\ubcf8\uc8fc\uc758 \uccb4\uc81c\uc640 \uad70\uc8fc\uc81c \uccb4\uc81c\ub97c \uc720\uc9c0\ud558\uba74\uc11c \uc81c\uad6d\uc8fc\uc758\uc640 \uc804\uccb4\uc8fc\uc758\ub97c \uc628\uac74\uc801\uc73c\ub85c \uc2e4\ud589\ud574 \ub098\uac00\uc790\ub294 \uc785\uc7a5\uc774\uc5c8\ub2e4. \ub2e4\uc774\uc1fc \ub370\ubaa8\ud06c\ub77c\uc2dc \uc2dc\ub300\uac00 \ub05d\ub09c \uc774\ud6c4 \uadf9\uc6b0 \uc0ac\uc0c1\uc774 \uc804\uad6d\uc801\uc73c\ub85c \ud37c\uc9c0\uba74\uc11c, \uc774\ub4e4\uc758 \uc0ac\uc0c1\uc5d0 \ud798\uc774 \ub4e4\uc5b4\uac00\uae30 \uc2dc\uc791\ud588\ub2e4. 2\ucc28 \uc138\uacc4\ub300\uc804 \uc885\uc804 \ud6c4, \ub300\ubd80\ubd84\uc758 \uadf9\uc6b0 \uc9c0\uc9c0\uc790\ub4e4\uc740 \ucc98\ubc8c\uc744 \uba74\ud558\uc5ec \uc77c\ubcf8 \uc790\uc720\ub2f9\uc5d0 \uc785\ub2f9\ud558\uc600\uc73c\uba70, \uc624\ub298\ub0a0\uc5d0\ub294 \uadf8\ub4e4\uc758 \uc6d0\ub85c\uc801 \uc5ed\ud560\uc744 \ud588\ub358 \uc815\uce58\uc778\ub4e4\uc774 \uadf9\uc6b0 \uc6b4\ub3d9\uc758 \uc9c0\ub3c4\uc790\uc801 \uc704\uce58\ub97c \ucc28\uc9c0\ud558\uace0\uc788\ub2e4. \ud604\uc7ac \uc77c\ubcf8\uc740 \ub2e4\ub984\uc5c6\uc774 \uadf9\uc6b0\uc5d0 \ub300\ud55c \uc9c0\uc9c0\ub3c4\uac00 \uc0c1\ub2f9\ud788 \ub192\uc73c\uba70, \ud574\ub2f9 \ub2e8\uccb4\ub294 \uc140 \uc218 \uc5c6\uc744 \uc815\ub3c4\ub85c \ub9ce\ub2e4.", "sentence_answer": "\uc77c\ubcf8\uc5d0\uc11c \uadf9\uc6b0 \uc6b4\ub3d9\uacfc \uc0ac\uc0c1 \uc740 \uc77c\ubcf8 \uc81c\uad6d \uc2dc\uc808 \ub2f9\uc2dc \uc77c\ubcf8 \uc721\uad70\uc758 \ud30c\ubc8c\uc774\uc5c8\ub358 \ud669\ub3c4\ud30c, \ud1b5\uc81c\ud30c\uc640 \ud30c\uc2dc\uc2a4\ud2b8\uc600\ub358 \uae30\ud0c0 \uc787\ud0a4\ub85c\ubd80\ud130 \uadf8 \uae30\uc6d0\uc744 \ucc3e\uc744 \uc218 \uc788\ub2e4.", "paragraph_id": "5975881-4-0", "paragraph_question": "question: \uc77c\ubcf8 \uc81c\uad6d \uc2dc\uc808 \ud669\ub3c4\ud30c \uae30\ud0c0 \uc787\ud0a4\ub85c\ubd80\ud130 \uae30\uc6cc\uc744 \ucc3e\uc744 \uc218 \uc788\ub294 \uac83\uc740?, context: \uc77c\ubcf8\uc5d0\uc11c \uadf9\uc6b0 \uc6b4\ub3d9\uacfc \uc0ac\uc0c1\uc740 \uc77c\ubcf8 \uc81c\uad6d \uc2dc\uc808 \ub2f9\uc2dc \uc77c\ubcf8 \uc721\uad70\uc758 \ud30c\ubc8c\uc774\uc5c8\ub358 \ud669\ub3c4\ud30c, \ud1b5\uc81c\ud30c\uc640 \ud30c\uc2dc\uc2a4\ud2b8\uc600\ub358 \uae30\ud0c0 \uc787\ud0a4\ub85c\ubd80\ud130 \uadf8 \uae30\uc6d0\uc744 \ucc3e\uc744 \uc218 \uc788\ub2e4. \ub2f9\uc2dc \ud669\ub3c4\ud30c\ub294 \ubc18\uc790\ubcf8\uc8fc\uc758\uc640 \ud30c\uc2dc\uc998\uc744 \ubaa8\ud1a0\ub85c \ud55c \uad6d\uac00\uc8fc\uc758, \ud33d\ucc3d\uc8fc\uc758\ub97c \uae30\ubc18\uc73c\ub85c \ud588\uace0, \ud1b5\uc81c\ud30c\ub294 \uae30\uc874 \uc77c\ubcf8\uc2dd \uc790\ubcf8\uc8fc\uc758 \uccb4\uc81c\uc640 \uad70\uc8fc\uc81c \uccb4\uc81c\ub97c \uc720\uc9c0\ud558\uba74\uc11c \uc81c\uad6d\uc8fc\uc758\uc640 \uc804\uccb4\uc8fc\uc758\ub97c \uc628\uac74\uc801\uc73c\ub85c \uc2e4\ud589\ud574 \ub098\uac00\uc790\ub294 \uc785\uc7a5\uc774\uc5c8\ub2e4. \ub2e4\uc774\uc1fc \ub370\ubaa8\ud06c\ub77c\uc2dc \uc2dc\ub300\uac00 \ub05d\ub09c \uc774\ud6c4 \uadf9\uc6b0 \uc0ac\uc0c1\uc774 \uc804\uad6d\uc801\uc73c\ub85c \ud37c\uc9c0\uba74\uc11c, \uc774\ub4e4\uc758 \uc0ac\uc0c1\uc5d0 \ud798\uc774 \ub4e4\uc5b4\uac00\uae30 \uc2dc\uc791\ud588\ub2e4. 2\ucc28 \uc138\uacc4\ub300\uc804 \uc885\uc804 \ud6c4, \ub300\ubd80\ubd84\uc758 \uadf9\uc6b0 \uc9c0\uc9c0\uc790\ub4e4\uc740 \ucc98\ubc8c\uc744 \uba74\ud558\uc5ec \uc77c\ubcf8 \uc790\uc720\ub2f9\uc5d0 \uc785\ub2f9\ud558\uc600\uc73c\uba70, \uc624\ub298\ub0a0\uc5d0\ub294 \uadf8\ub4e4\uc758 \uc6d0\ub85c\uc801 \uc5ed\ud560\uc744 \ud588\ub358 \uc815\uce58\uc778\ub4e4\uc774 \uadf9\uc6b0 \uc6b4\ub3d9\uc758 \uc9c0\ub3c4\uc790\uc801 \uc704\uce58\ub97c \ucc28\uc9c0\ud558\uace0\uc788\ub2e4. \ud604\uc7ac \uc77c\ubcf8\uc740 \ub2e4\ub984\uc5c6\uc774 \uadf9\uc6b0\uc5d0 \ub300\ud55c \uc9c0\uc9c0\ub3c4\uac00 \uc0c1\ub2f9\ud788 \ub192\uc73c\uba70, \ud574\ub2f9 \ub2e8\uccb4\ub294 \uc140 \uc218 \uc5c6\uc744 \uc815\ub3c4\ub85c \ub9ce\ub2e4."} {"question": "\uc0b0\uc5c5\ud654\uc758 \uacfc\uc815\uc5d0\uc11c \uc5b4\ub5a4 \uc778\ub825\uc5d0 \ub300\ud55c \uc218\uc694\uac00 \uc99d\uac00\ud588\ub294\uac00?", "paragraph": "\uacbd\uc81c\ubc1c\uc804\uc5d0 \ub530\ub978 \uc0b0\uc5c5\ud654\uc758 \uacfc\uc815\uc5d0\uc11c \uc804\ubb38\uc801\uc778 \uc9c0\uc2dd\u00b7\uae30\ub2a5\u00b7\uae30\uc220\uc744 \uac00\uc9c4 \uc778\ub825\uc5d0 \ub300\ud55c \uc218\uc694\uac00 \uc99d\uac00\ud574 \uc654\ub2e4\ub294 \uac83\uc774\ub2e4. \uc0b0\uc5c5\uad6c\uc870\uc758 \uace0\ub3c4\ud654\ub294 \uc0dd\uc0b0\uc9c1\uc5d0\uc11c\uc870\ucc28 \uc911\ub4f1\uad50\uc721 \uc774\uc0c1\uc758 \uccb4\uacc4\uc801\uc778 \uc9c0\uc2dd\uacfc \uc804\ubb38\uc801\uc778 \uae30\uc220\u00b7\uae30\ub2a5\uc744 \uc694\uad6c\ud558\uac8c \ub418\uc5c8\uace0 \uadf8 \uacb0\uacfc \uc0c1\uae09\uc758 \uad50\uc721\uc740 \uc0ac\ud68c\u00b7\uacbd\uc81c\uc801\uc73c\ub85c \uc720\ub9ac\ud55c \ucde8\uc5c5\uc870\uac74\uc774 \ub418\uc5b4 \uad50\uc721\uc758 \uc591\uc801\u00b7\uc9c8\uc801 \uc99d\ub300\ub97c \ucd08\ub798\ud558\uc600\ub2e4. \ub610, \uc804\uccb4\uc778\uad6c\uc758 \uc99d\uac00\ub97c \ub4e4 \uc218 \uc788\ub2e4. \uc778\uad6c\uc758 \uc99d\uac00 \uc790\uccb4\ub294 \ucde8\ud559\ub960\uacfc \ubb34\uad00\ud558\uc9c0\ub9cc \uad50\uc721\uc778\uad6c\uc758 \uc808\ub300\uc218\ub97c \uc99d\uac00\uc2dc\ud0a4\ub294 \uc9c1\uc811\uc801\uc778 \uc694\uc778\uc774\uba70 \ub530\ub77c\uc11c \uad50\uc721\uc758 \uc591\uc801 \uc99d\ub300\ub97c \uc720\ubc1c\ud558\ub294 \uc808\ub300\uc801 \uc694\uc778\uc774 \ub41c\ub2e4. \uad50\uc721\uc758 \uc591\uc801\u00b7\uc9c8\uc801 \uc99d\ub300\ub294 \ub2e4\uc18c\uc758 \ubb38\uc81c\ub294 \uc788\uc73c\ub098 \uc9c0\uadf9\ud788 \ubc14\ub78c\uc9c1\ud55c \uac83\uc774\ub2e4. \uadf8\ub7ec\ub098 \ud55c\uad6d\uc758 \uad50\uc721\uc740 \uad50\uc721\uc7ac\uc815\u00b7\uad50\uc721\ud658\uacbd\uba74\uc5d0\uc11c, \uadf8\ub9ac\uace0 \uad6c\uc870\uc801\uc73c\ub85c \ub9ce\uc740 \ubb38\uc81c\ub97c \uc548\uace0 \uc788\ub2e4. \ub2e8\uc801\uc73c\ub85c 3\ub144\uc758 \uc911\ub4f1\uad50\uc721 \uc758\ubb34\uad50\uc721\uc81c\ub3c4\ub97c \ucc44\ud0dd\ud558\uc600\uc73c\uba74\uc11c\ub3c4 \uc77c\ubd80 \uc2e4\uc2dc\ud560 \uc218\ubc16\uc5d0 \uc5c6\ub294 \uae4c\ub2ed\uc774 \uadf8\uac83\uc774\uba70, \uc77c\ubc18\uacc4 \uace0\ub4f1\ud559\uad50\uc758 \uc9c0\ub098\uce5c \ube44\ub300\uc640 \uc9c1\uc5c5\uad50\uc721\uc758 \ubd80\uc7ac\ub85c \uc7ac\uc218\uc0dd\uc774 \uc99d\uac00\ud558\uace0 \ucde8\uc5c5\ub09c\uc744 \ube5a\uace0 \uc788\ub294 \ud604\uc0c1\uc774 \uadf8\uac83\uc774\ub2e4.", "answer": "\uc804\ubb38\uc801\uc778 \uc9c0\uc2dd\u00b7\uae30\ub2a5\u00b7\uae30\uc220", "sentence": "\uacbd\uc81c\ubc1c\uc804\uc5d0 \ub530\ub978 \uc0b0\uc5c5\ud654\uc758 \uacfc\uc815\uc5d0\uc11c \uc804\ubb38\uc801\uc778 \uc9c0\uc2dd\u00b7\uae30\ub2a5\u00b7\uae30\uc220 \uc744 \uac00\uc9c4 \uc778\ub825\uc5d0 \ub300\ud55c \uc218\uc694\uac00 \uc99d\uac00\ud574 \uc654\ub2e4\ub294 \uac83\uc774\ub2e4.", "paragraph_sentence": " \uacbd\uc81c\ubc1c\uc804\uc5d0 \ub530\ub978 \uc0b0\uc5c5\ud654\uc758 \uacfc\uc815\uc5d0\uc11c \uc804\ubb38\uc801\uc778 \uc9c0\uc2dd\u00b7\uae30\ub2a5\u00b7\uae30\uc220 \uc744 \uac00\uc9c4 \uc778\ub825\uc5d0 \ub300\ud55c \uc218\uc694\uac00 \uc99d\uac00\ud574 \uc654\ub2e4\ub294 \uac83\uc774\ub2e4. \uc0b0\uc5c5\uad6c\uc870\uc758 \uace0\ub3c4\ud654\ub294 \uc0dd\uc0b0\uc9c1\uc5d0\uc11c\uc870\ucc28 \uc911\ub4f1\uad50\uc721 \uc774\uc0c1\uc758 \uccb4\uacc4\uc801\uc778 \uc9c0\uc2dd\uacfc \uc804\ubb38\uc801\uc778 \uae30\uc220\u00b7\uae30\ub2a5\uc744 \uc694\uad6c\ud558\uac8c \ub418\uc5c8\uace0 \uadf8 \uacb0\uacfc \uc0c1\uae09\uc758 \uad50\uc721\uc740 \uc0ac\ud68c\u00b7\uacbd\uc81c\uc801\uc73c\ub85c \uc720\ub9ac\ud55c \ucde8\uc5c5\uc870\uac74\uc774 \ub418\uc5b4 \uad50\uc721\uc758 \uc591\uc801\u00b7\uc9c8\uc801 \uc99d\ub300\ub97c \ucd08\ub798\ud558\uc600\ub2e4. \ub610, \uc804\uccb4\uc778\uad6c\uc758 \uc99d\uac00\ub97c \ub4e4 \uc218 \uc788\ub2e4. \uc778\uad6c\uc758 \uc99d\uac00 \uc790\uccb4\ub294 \ucde8\ud559\ub960\uacfc \ubb34\uad00\ud558\uc9c0\ub9cc \uad50\uc721\uc778\uad6c\uc758 \uc808\ub300\uc218\ub97c \uc99d\uac00\uc2dc\ud0a4\ub294 \uc9c1\uc811\uc801\uc778 \uc694\uc778\uc774\uba70 \ub530\ub77c\uc11c \uad50\uc721\uc758 \uc591\uc801 \uc99d\ub300\ub97c \uc720\ubc1c\ud558\ub294 \uc808\ub300\uc801 \uc694\uc778\uc774 \ub41c\ub2e4. \uad50\uc721\uc758 \uc591\uc801\u00b7\uc9c8\uc801 \uc99d\ub300\ub294 \ub2e4\uc18c\uc758 \ubb38\uc81c\ub294 \uc788\uc73c\ub098 \uc9c0\uadf9\ud788 \ubc14\ub78c\uc9c1\ud55c \uac83\uc774\ub2e4. \uadf8\ub7ec\ub098 \ud55c\uad6d\uc758 \uad50\uc721\uc740 \uad50\uc721\uc7ac\uc815\u00b7\uad50\uc721\ud658\uacbd\uba74\uc5d0\uc11c, \uadf8\ub9ac\uace0 \uad6c\uc870\uc801\uc73c\ub85c \ub9ce\uc740 \ubb38\uc81c\ub97c \uc548\uace0 \uc788\ub2e4. \ub2e8\uc801\uc73c\ub85c 3\ub144\uc758 \uc911\ub4f1\uad50\uc721 \uc758\ubb34\uad50\uc721\uc81c\ub3c4\ub97c \ucc44\ud0dd\ud558\uc600\uc73c\uba74\uc11c\ub3c4 \uc77c\ubd80 \uc2e4\uc2dc\ud560 \uc218\ubc16\uc5d0 \uc5c6\ub294 \uae4c\ub2ed\uc774 \uadf8\uac83\uc774\uba70, \uc77c\ubc18\uacc4 \uace0\ub4f1\ud559\uad50\uc758 \uc9c0\ub098\uce5c \ube44\ub300\uc640 \uc9c1\uc5c5\uad50\uc721\uc758 \ubd80\uc7ac\ub85c \uc7ac\uc218\uc0dd\uc774 \uc99d\uac00\ud558\uace0 \ucde8\uc5c5\ub09c\uc744 \ube5a\uace0 \uc788\ub294 \ud604\uc0c1\uc774 \uadf8\uac83\uc774\ub2e4.", "paragraph_answer": "\uacbd\uc81c\ubc1c\uc804\uc5d0 \ub530\ub978 \uc0b0\uc5c5\ud654\uc758 \uacfc\uc815\uc5d0\uc11c \uc804\ubb38\uc801\uc778 \uc9c0\uc2dd\u00b7\uae30\ub2a5\u00b7\uae30\uc220 \uc744 \uac00\uc9c4 \uc778\ub825\uc5d0 \ub300\ud55c \uc218\uc694\uac00 \uc99d\uac00\ud574 \uc654\ub2e4\ub294 \uac83\uc774\ub2e4. \uc0b0\uc5c5\uad6c\uc870\uc758 \uace0\ub3c4\ud654\ub294 \uc0dd\uc0b0\uc9c1\uc5d0\uc11c\uc870\ucc28 \uc911\ub4f1\uad50\uc721 \uc774\uc0c1\uc758 \uccb4\uacc4\uc801\uc778 \uc9c0\uc2dd\uacfc \uc804\ubb38\uc801\uc778 \uae30\uc220\u00b7\uae30\ub2a5\uc744 \uc694\uad6c\ud558\uac8c \ub418\uc5c8\uace0 \uadf8 \uacb0\uacfc \uc0c1\uae09\uc758 \uad50\uc721\uc740 \uc0ac\ud68c\u00b7\uacbd\uc81c\uc801\uc73c\ub85c \uc720\ub9ac\ud55c \ucde8\uc5c5\uc870\uac74\uc774 \ub418\uc5b4 \uad50\uc721\uc758 \uc591\uc801\u00b7\uc9c8\uc801 \uc99d\ub300\ub97c \ucd08\ub798\ud558\uc600\ub2e4. \ub610, \uc804\uccb4\uc778\uad6c\uc758 \uc99d\uac00\ub97c \ub4e4 \uc218 \uc788\ub2e4. \uc778\uad6c\uc758 \uc99d\uac00 \uc790\uccb4\ub294 \ucde8\ud559\ub960\uacfc \ubb34\uad00\ud558\uc9c0\ub9cc \uad50\uc721\uc778\uad6c\uc758 \uc808\ub300\uc218\ub97c \uc99d\uac00\uc2dc\ud0a4\ub294 \uc9c1\uc811\uc801\uc778 \uc694\uc778\uc774\uba70 \ub530\ub77c\uc11c \uad50\uc721\uc758 \uc591\uc801 \uc99d\ub300\ub97c \uc720\ubc1c\ud558\ub294 \uc808\ub300\uc801 \uc694\uc778\uc774 \ub41c\ub2e4. \uad50\uc721\uc758 \uc591\uc801\u00b7\uc9c8\uc801 \uc99d\ub300\ub294 \ub2e4\uc18c\uc758 \ubb38\uc81c\ub294 \uc788\uc73c\ub098 \uc9c0\uadf9\ud788 \ubc14\ub78c\uc9c1\ud55c \uac83\uc774\ub2e4. \uadf8\ub7ec\ub098 \ud55c\uad6d\uc758 \uad50\uc721\uc740 \uad50\uc721\uc7ac\uc815\u00b7\uad50\uc721\ud658\uacbd\uba74\uc5d0\uc11c, \uadf8\ub9ac\uace0 \uad6c\uc870\uc801\uc73c\ub85c \ub9ce\uc740 \ubb38\uc81c\ub97c \uc548\uace0 \uc788\ub2e4. \ub2e8\uc801\uc73c\ub85c 3\ub144\uc758 \uc911\ub4f1\uad50\uc721 \uc758\ubb34\uad50\uc721\uc81c\ub3c4\ub97c \ucc44\ud0dd\ud558\uc600\uc73c\uba74\uc11c\ub3c4 \uc77c\ubd80 \uc2e4\uc2dc\ud560 \uc218\ubc16\uc5d0 \uc5c6\ub294 \uae4c\ub2ed\uc774 \uadf8\uac83\uc774\uba70, \uc77c\ubc18\uacc4 \uace0\ub4f1\ud559\uad50\uc758 \uc9c0\ub098\uce5c \ube44\ub300\uc640 \uc9c1\uc5c5\uad50\uc721\uc758 \ubd80\uc7ac\ub85c \uc7ac\uc218\uc0dd\uc774 \uc99d\uac00\ud558\uace0 \ucde8\uc5c5\ub09c\uc744 \ube5a\uace0 \uc788\ub294 \ud604\uc0c1\uc774 \uadf8\uac83\uc774\ub2e4.", "sentence_answer": "\uacbd\uc81c\ubc1c\uc804\uc5d0 \ub530\ub978 \uc0b0\uc5c5\ud654\uc758 \uacfc\uc815\uc5d0\uc11c \uc804\ubb38\uc801\uc778 \uc9c0\uc2dd\u00b7\uae30\ub2a5\u00b7\uae30\uc220 \uc744 \uac00\uc9c4 \uc778\ub825\uc5d0 \ub300\ud55c \uc218\uc694\uac00 \uc99d\uac00\ud574 \uc654\ub2e4\ub294 \uac83\uc774\ub2e4.", "paragraph_id": "6038504-2-1", "paragraph_question": "question: \uc0b0\uc5c5\ud654\uc758 \uacfc\uc815\uc5d0\uc11c \uc5b4\ub5a4 \uc778\ub825\uc5d0 \ub300\ud55c \uc218\uc694\uac00 \uc99d\uac00\ud588\ub294\uac00?, context: \uacbd\uc81c\ubc1c\uc804\uc5d0 \ub530\ub978 \uc0b0\uc5c5\ud654\uc758 \uacfc\uc815\uc5d0\uc11c \uc804\ubb38\uc801\uc778 \uc9c0\uc2dd\u00b7\uae30\ub2a5\u00b7\uae30\uc220\uc744 \uac00\uc9c4 \uc778\ub825\uc5d0 \ub300\ud55c \uc218\uc694\uac00 \uc99d\uac00\ud574 \uc654\ub2e4\ub294 \uac83\uc774\ub2e4. \uc0b0\uc5c5\uad6c\uc870\uc758 \uace0\ub3c4\ud654\ub294 \uc0dd\uc0b0\uc9c1\uc5d0\uc11c\uc870\ucc28 \uc911\ub4f1\uad50\uc721 \uc774\uc0c1\uc758 \uccb4\uacc4\uc801\uc778 \uc9c0\uc2dd\uacfc \uc804\ubb38\uc801\uc778 \uae30\uc220\u00b7\uae30\ub2a5\uc744 \uc694\uad6c\ud558\uac8c \ub418\uc5c8\uace0 \uadf8 \uacb0\uacfc \uc0c1\uae09\uc758 \uad50\uc721\uc740 \uc0ac\ud68c\u00b7\uacbd\uc81c\uc801\uc73c\ub85c \uc720\ub9ac\ud55c \ucde8\uc5c5\uc870\uac74\uc774 \ub418\uc5b4 \uad50\uc721\uc758 \uc591\uc801\u00b7\uc9c8\uc801 \uc99d\ub300\ub97c \ucd08\ub798\ud558\uc600\ub2e4. \ub610, \uc804\uccb4\uc778\uad6c\uc758 \uc99d\uac00\ub97c \ub4e4 \uc218 \uc788\ub2e4. \uc778\uad6c\uc758 \uc99d\uac00 \uc790\uccb4\ub294 \ucde8\ud559\ub960\uacfc \ubb34\uad00\ud558\uc9c0\ub9cc \uad50\uc721\uc778\uad6c\uc758 \uc808\ub300\uc218\ub97c \uc99d\uac00\uc2dc\ud0a4\ub294 \uc9c1\uc811\uc801\uc778 \uc694\uc778\uc774\uba70 \ub530\ub77c\uc11c \uad50\uc721\uc758 \uc591\uc801 \uc99d\ub300\ub97c \uc720\ubc1c\ud558\ub294 \uc808\ub300\uc801 \uc694\uc778\uc774 \ub41c\ub2e4. \uad50\uc721\uc758 \uc591\uc801\u00b7\uc9c8\uc801 \uc99d\ub300\ub294 \ub2e4\uc18c\uc758 \ubb38\uc81c\ub294 \uc788\uc73c\ub098 \uc9c0\uadf9\ud788 \ubc14\ub78c\uc9c1\ud55c \uac83\uc774\ub2e4. \uadf8\ub7ec\ub098 \ud55c\uad6d\uc758 \uad50\uc721\uc740 \uad50\uc721\uc7ac\uc815\u00b7\uad50\uc721\ud658\uacbd\uba74\uc5d0\uc11c, \uadf8\ub9ac\uace0 \uad6c\uc870\uc801\uc73c\ub85c \ub9ce\uc740 \ubb38\uc81c\ub97c \uc548\uace0 \uc788\ub2e4. \ub2e8\uc801\uc73c\ub85c 3\ub144\uc758 \uc911\ub4f1\uad50\uc721 \uc758\ubb34\uad50\uc721\uc81c\ub3c4\ub97c \ucc44\ud0dd\ud558\uc600\uc73c\uba74\uc11c\ub3c4 \uc77c\ubd80 \uc2e4\uc2dc\ud560 \uc218\ubc16\uc5d0 \uc5c6\ub294 \uae4c\ub2ed\uc774 \uadf8\uac83\uc774\uba70, \uc77c\ubc18\uacc4 \uace0\ub4f1\ud559\uad50\uc758 \uc9c0\ub098\uce5c \ube44\ub300\uc640 \uc9c1\uc5c5\uad50\uc721\uc758 \ubd80\uc7ac\ub85c \uc7ac\uc218\uc0dd\uc774 \uc99d\uac00\ud558\uace0 \ucde8\uc5c5\ub09c\uc744 \ube5a\uace0 \uc788\ub294 \ud604\uc0c1\uc774 \uadf8\uac83\uc774\ub2e4."} {"question": "\uc5b8\ub354\uc6b0\ub4dc\uad00\uc758 \uc67c\ucabd\uc5d0\uc788\ub294 \uad00\uc740?", "paragraph": "\ubc31\uc591\ub85c\uc758 \ub05d\uc5d0\ub294 \ub300\ud559 \ubcf8\ubd80\uac00 \uc704\uce58\ud55c \uc5b8\ub354\uc6b0\ub4dc\uad00\uc744 \uc911\uc2ec\uc73c\ub85c \uc67c\ud3b8\uc5d0 \uc2a4\ud300\uc2a8\uad00\uacfc \uc624\ub978\ud3b8\uc5d0 \uc544\ud39c\uc824\ub7ec\uad00\uc774 \uc704\uce58\uc5d0 \uc788\uace0 \uc138 \uac74\ubb3c\uc758 \uc911\uc559\uc5d0\ub294 \ud559\uad50\uc758 \ucc3d\uc124\uc790\uc774\uba70 \ucd08\uc11d\uc744 \uc774\ub8ec \uc5b8\ub354\uc6b0\ub4dc\uc758 \ub3d9\uc0c1\uc774 \uc11c \uc788\ub2e4. \uc2a4\ud300\uc2a8\uad00 \ub4a4\ud3b8 \uc5b8\ub355\uc704 \uc67c\ud3b8\uc5d0\ub294 \ud55c\uacbd\uad00\uacfc \ud540\uc2a8\uad00\uc774 \ub098\ub780\ud788 \uc11c \uc788\uace0, \uadf8 \ub4a4\ub85c \uc2e0\ud559\uad00\uc774 \ub3d9\ud5a5\uc744 \ubc14\ub77c\ubcf4\uba70 \uc790\ub9ac \uc7a1\uace0 \uc788\ub2e4. \uc5b8\ub354\uc6b0\ub4dc\uad00\uc758 \ub4a4\ud3b8\uc73c\ub85c \uc0ac\ud68c\uacfc\ud559\ub300\ud559\uc774 \uc790\ub9ac\ud55c \uc5f0\ud76c\uad00\uc774 \uc11c \uc788\uc73c\uba70 \uc624\ub978\ud3b8\uc5d0 \uc131\uc554\uad00, \uc67c\ud3b8\uc5d0 \uc720\uc5b5\uacb8\uae30\ub150\uad00, \uadf8 \ub4a4\ub85c \ub300\uc6b0\uad00\uc774 \uc788\ub2e4. \uc720\uc5b5\uacb8\uae30\ub150\uad00 \ub4a4\ud3b8 \uc624\ub978\ucabd\uc5d0\ub294 \ube4c\ub9c1\uc2ac\ub9ac\uad00\uc774 \uc790\ub9ac \uc7a1\uace0 \uc788\uc73c\uba70 \ub4b7\uc0b0 \uc911\ud131\uc5d0\ub294 \uc885\ud569\uad50\uc2e4\ub2e8\uacfc \uc678\uc194\uad00, \uc704\ub2f9\uad00\uc774 \uc11c \uc788\ub2e4. \uc131\uc554\uad00 \ub4b7\uae38\uc744 \ub530\ub77c \ub3d9\ubb38\ubc29\ud5a5\uc73c\ub85c \uac00\uba74 \uc6b0\uce21\uc5d0 \uc0c8\ucc9c\ub144\uad00, \uad6d\uc81c\ud559\uc0ac, \uc5b8\uc5b4\uc5f0\uad6c\uad50\uc721\uc6d0, \uc81c\uc911\ud559\uc0ac\uac00 \uc788\uace0, \ub300\uc6b0\uad00 \ub4a4\ud3b8 \uace0\uac1c\ub97c \ub118\uc5b4 \ubd81\ubb38\ubc29\ud5a5\uc73c\ub85c \uac00\uba74 \ubb34\uc545\ud559\uc0ac(\ud559\uc0dd\uae30\uc219\uc0ac) 5\uac1c\ub3d9\uc774 \uc790\ub9ac \uc7a1\uace0 \uc788\ub2e4.", "answer": "\uc2a4\ud300\uc2a8\uad00", "sentence": "\ubc31\uc591\ub85c\uc758 \ub05d\uc5d0\ub294 \ub300\ud559 \ubcf8\ubd80\uac00 \uc704\uce58\ud55c \uc5b8\ub354\uc6b0\ub4dc\uad00\uc744 \uc911\uc2ec\uc73c\ub85c \uc67c\ud3b8\uc5d0 \uc2a4\ud300\uc2a8\uad00 \uacfc \uc624\ub978\ud3b8\uc5d0 \uc544\ud39c\uc824\ub7ec\uad00\uc774 \uc704\uce58\uc5d0 \uc788\uace0 \uc138 \uac74\ubb3c\uc758 \uc911\uc559\uc5d0\ub294 \ud559\uad50\uc758 \ucc3d\uc124\uc790\uc774\uba70 \ucd08\uc11d\uc744 \uc774\ub8ec \uc5b8\ub354\uc6b0\ub4dc\uc758 \ub3d9\uc0c1\uc774 \uc11c \uc788\ub2e4.", "paragraph_sentence": " \ubc31\uc591\ub85c\uc758 \ub05d\uc5d0\ub294 \ub300\ud559 \ubcf8\ubd80\uac00 \uc704\uce58\ud55c \uc5b8\ub354\uc6b0\ub4dc\uad00\uc744 \uc911\uc2ec\uc73c\ub85c \uc67c\ud3b8\uc5d0 \uc2a4\ud300\uc2a8\uad00 \uacfc \uc624\ub978\ud3b8\uc5d0 \uc544\ud39c\uc824\ub7ec\uad00\uc774 \uc704\uce58\uc5d0 \uc788\uace0 \uc138 \uac74\ubb3c\uc758 \uc911\uc559\uc5d0\ub294 \ud559\uad50\uc758 \ucc3d\uc124\uc790\uc774\uba70 \ucd08\uc11d\uc744 \uc774\ub8ec \uc5b8\ub354\uc6b0\ub4dc\uc758 \ub3d9\uc0c1\uc774 \uc11c \uc788\ub2e4. \uc2a4\ud300\uc2a8\uad00 \ub4a4\ud3b8 \uc5b8\ub355\uc704 \uc67c\ud3b8\uc5d0\ub294 \ud55c\uacbd\uad00\uacfc \ud540\uc2a8\uad00\uc774 \ub098\ub780\ud788 \uc11c \uc788\uace0, \uadf8 \ub4a4\ub85c \uc2e0\ud559\uad00\uc774 \ub3d9\ud5a5\uc744 \ubc14\ub77c\ubcf4\uba70 \uc790\ub9ac \uc7a1\uace0 \uc788\ub2e4. \uc5b8\ub354\uc6b0\ub4dc\uad00\uc758 \ub4a4\ud3b8\uc73c\ub85c \uc0ac\ud68c\uacfc\ud559\ub300\ud559\uc774 \uc790\ub9ac\ud55c \uc5f0\ud76c\uad00\uc774 \uc11c \uc788\uc73c\uba70 \uc624\ub978\ud3b8\uc5d0 \uc131\uc554\uad00, \uc67c\ud3b8\uc5d0 \uc720\uc5b5\uacb8\uae30\ub150\uad00, \uadf8 \ub4a4\ub85c \ub300\uc6b0\uad00\uc774 \uc788\ub2e4. \uc720\uc5b5\uacb8\uae30\ub150\uad00 \ub4a4\ud3b8 \uc624\ub978\ucabd\uc5d0\ub294 \ube4c\ub9c1\uc2ac\ub9ac\uad00\uc774 \uc790\ub9ac \uc7a1\uace0 \uc788\uc73c\uba70 \ub4b7\uc0b0 \uc911\ud131\uc5d0\ub294 \uc885\ud569\uad50\uc2e4\ub2e8\uacfc \uc678\uc194\uad00, \uc704\ub2f9\uad00\uc774 \uc11c \uc788\ub2e4. \uc131\uc554\uad00 \ub4b7\uae38\uc744 \ub530\ub77c \ub3d9\ubb38\ubc29\ud5a5\uc73c\ub85c \uac00\uba74 \uc6b0\uce21\uc5d0 \uc0c8\ucc9c\ub144\uad00, \uad6d\uc81c\ud559\uc0ac, \uc5b8\uc5b4\uc5f0\uad6c\uad50\uc721\uc6d0, \uc81c\uc911\ud559\uc0ac\uac00 \uc788\uace0, \ub300\uc6b0\uad00 \ub4a4\ud3b8 \uace0\uac1c\ub97c \ub118\uc5b4 \ubd81\ubb38\ubc29\ud5a5\uc73c\ub85c \uac00\uba74 \ubb34\uc545\ud559\uc0ac(\ud559\uc0dd\uae30\uc219\uc0ac) 5\uac1c\ub3d9\uc774 \uc790\ub9ac \uc7a1\uace0 \uc788\ub2e4.", "paragraph_answer": "\ubc31\uc591\ub85c\uc758 \ub05d\uc5d0\ub294 \ub300\ud559 \ubcf8\ubd80\uac00 \uc704\uce58\ud55c \uc5b8\ub354\uc6b0\ub4dc\uad00\uc744 \uc911\uc2ec\uc73c\ub85c \uc67c\ud3b8\uc5d0 \uc2a4\ud300\uc2a8\uad00 \uacfc \uc624\ub978\ud3b8\uc5d0 \uc544\ud39c\uc824\ub7ec\uad00\uc774 \uc704\uce58\uc5d0 \uc788\uace0 \uc138 \uac74\ubb3c\uc758 \uc911\uc559\uc5d0\ub294 \ud559\uad50\uc758 \ucc3d\uc124\uc790\uc774\uba70 \ucd08\uc11d\uc744 \uc774\ub8ec \uc5b8\ub354\uc6b0\ub4dc\uc758 \ub3d9\uc0c1\uc774 \uc11c \uc788\ub2e4. \uc2a4\ud300\uc2a8\uad00 \ub4a4\ud3b8 \uc5b8\ub355\uc704 \uc67c\ud3b8\uc5d0\ub294 \ud55c\uacbd\uad00\uacfc \ud540\uc2a8\uad00\uc774 \ub098\ub780\ud788 \uc11c \uc788\uace0, \uadf8 \ub4a4\ub85c \uc2e0\ud559\uad00\uc774 \ub3d9\ud5a5\uc744 \ubc14\ub77c\ubcf4\uba70 \uc790\ub9ac \uc7a1\uace0 \uc788\ub2e4. \uc5b8\ub354\uc6b0\ub4dc\uad00\uc758 \ub4a4\ud3b8\uc73c\ub85c \uc0ac\ud68c\uacfc\ud559\ub300\ud559\uc774 \uc790\ub9ac\ud55c \uc5f0\ud76c\uad00\uc774 \uc11c \uc788\uc73c\uba70 \uc624\ub978\ud3b8\uc5d0 \uc131\uc554\uad00, \uc67c\ud3b8\uc5d0 \uc720\uc5b5\uacb8\uae30\ub150\uad00, \uadf8 \ub4a4\ub85c \ub300\uc6b0\uad00\uc774 \uc788\ub2e4. \uc720\uc5b5\uacb8\uae30\ub150\uad00 \ub4a4\ud3b8 \uc624\ub978\ucabd\uc5d0\ub294 \ube4c\ub9c1\uc2ac\ub9ac\uad00\uc774 \uc790\ub9ac \uc7a1\uace0 \uc788\uc73c\uba70 \ub4b7\uc0b0 \uc911\ud131\uc5d0\ub294 \uc885\ud569\uad50\uc2e4\ub2e8\uacfc \uc678\uc194\uad00, \uc704\ub2f9\uad00\uc774 \uc11c \uc788\ub2e4. \uc131\uc554\uad00 \ub4b7\uae38\uc744 \ub530\ub77c \ub3d9\ubb38\ubc29\ud5a5\uc73c\ub85c \uac00\uba74 \uc6b0\uce21\uc5d0 \uc0c8\ucc9c\ub144\uad00, \uad6d\uc81c\ud559\uc0ac, \uc5b8\uc5b4\uc5f0\uad6c\uad50\uc721\uc6d0, \uc81c\uc911\ud559\uc0ac\uac00 \uc788\uace0, \ub300\uc6b0\uad00 \ub4a4\ud3b8 \uace0\uac1c\ub97c \ub118\uc5b4 \ubd81\ubb38\ubc29\ud5a5\uc73c\ub85c \uac00\uba74 \ubb34\uc545\ud559\uc0ac(\ud559\uc0dd\uae30\uc219\uc0ac) 5\uac1c\ub3d9\uc774 \uc790\ub9ac \uc7a1\uace0 \uc788\ub2e4.", "sentence_answer": "\ubc31\uc591\ub85c\uc758 \ub05d\uc5d0\ub294 \ub300\ud559 \ubcf8\ubd80\uac00 \uc704\uce58\ud55c \uc5b8\ub354\uc6b0\ub4dc\uad00\uc744 \uc911\uc2ec\uc73c\ub85c \uc67c\ud3b8\uc5d0 \uc2a4\ud300\uc2a8\uad00 \uacfc \uc624\ub978\ud3b8\uc5d0 \uc544\ud39c\uc824\ub7ec\uad00\uc774 \uc704\uce58\uc5d0 \uc788\uace0 \uc138 \uac74\ubb3c\uc758 \uc911\uc559\uc5d0\ub294 \ud559\uad50\uc758 \ucc3d\uc124\uc790\uc774\uba70 \ucd08\uc11d\uc744 \uc774\ub8ec \uc5b8\ub354\uc6b0\ub4dc\uc758 \ub3d9\uc0c1\uc774 \uc11c \uc788\ub2e4.", "paragraph_id": "6458470-6-0", "paragraph_question": "question: \uc5b8\ub354\uc6b0\ub4dc\uad00\uc758 \uc67c\ucabd\uc5d0\uc788\ub294 \uad00\uc740?, context: \ubc31\uc591\ub85c\uc758 \ub05d\uc5d0\ub294 \ub300\ud559 \ubcf8\ubd80\uac00 \uc704\uce58\ud55c \uc5b8\ub354\uc6b0\ub4dc\uad00\uc744 \uc911\uc2ec\uc73c\ub85c \uc67c\ud3b8\uc5d0 \uc2a4\ud300\uc2a8\uad00\uacfc \uc624\ub978\ud3b8\uc5d0 \uc544\ud39c\uc824\ub7ec\uad00\uc774 \uc704\uce58\uc5d0 \uc788\uace0 \uc138 \uac74\ubb3c\uc758 \uc911\uc559\uc5d0\ub294 \ud559\uad50\uc758 \ucc3d\uc124\uc790\uc774\uba70 \ucd08\uc11d\uc744 \uc774\ub8ec \uc5b8\ub354\uc6b0\ub4dc\uc758 \ub3d9\uc0c1\uc774 \uc11c \uc788\ub2e4. \uc2a4\ud300\uc2a8\uad00 \ub4a4\ud3b8 \uc5b8\ub355\uc704 \uc67c\ud3b8\uc5d0\ub294 \ud55c\uacbd\uad00\uacfc \ud540\uc2a8\uad00\uc774 \ub098\ub780\ud788 \uc11c \uc788\uace0, \uadf8 \ub4a4\ub85c \uc2e0\ud559\uad00\uc774 \ub3d9\ud5a5\uc744 \ubc14\ub77c\ubcf4\uba70 \uc790\ub9ac \uc7a1\uace0 \uc788\ub2e4. \uc5b8\ub354\uc6b0\ub4dc\uad00\uc758 \ub4a4\ud3b8\uc73c\ub85c \uc0ac\ud68c\uacfc\ud559\ub300\ud559\uc774 \uc790\ub9ac\ud55c \uc5f0\ud76c\uad00\uc774 \uc11c \uc788\uc73c\uba70 \uc624\ub978\ud3b8\uc5d0 \uc131\uc554\uad00, \uc67c\ud3b8\uc5d0 \uc720\uc5b5\uacb8\uae30\ub150\uad00, \uadf8 \ub4a4\ub85c \ub300\uc6b0\uad00\uc774 \uc788\ub2e4. \uc720\uc5b5\uacb8\uae30\ub150\uad00 \ub4a4\ud3b8 \uc624\ub978\ucabd\uc5d0\ub294 \ube4c\ub9c1\uc2ac\ub9ac\uad00\uc774 \uc790\ub9ac \uc7a1\uace0 \uc788\uc73c\uba70 \ub4b7\uc0b0 \uc911\ud131\uc5d0\ub294 \uc885\ud569\uad50\uc2e4\ub2e8\uacfc \uc678\uc194\uad00, \uc704\ub2f9\uad00\uc774 \uc11c \uc788\ub2e4. \uc131\uc554\uad00 \ub4b7\uae38\uc744 \ub530\ub77c \ub3d9\ubb38\ubc29\ud5a5\uc73c\ub85c \uac00\uba74 \uc6b0\uce21\uc5d0 \uc0c8\ucc9c\ub144\uad00, \uad6d\uc81c\ud559\uc0ac, \uc5b8\uc5b4\uc5f0\uad6c\uad50\uc721\uc6d0, \uc81c\uc911\ud559\uc0ac\uac00 \uc788\uace0, \ub300\uc6b0\uad00 \ub4a4\ud3b8 \uace0\uac1c\ub97c \ub118\uc5b4 \ubd81\ubb38\ubc29\ud5a5\uc73c\ub85c \uac00\uba74 \ubb34\uc545\ud559\uc0ac(\ud559\uc0dd\uae30\uc219\uc0ac) 5\uac1c\ub3d9\uc774 \uc790\ub9ac \uc7a1\uace0 \uc788\ub2e4."} {"question": "\uc18c \ub2e4\ucf00\uc720\ud0a4\uc758 \uc9d1\uc548\uc5d0\uc11c \uae30\uc99d\ud55c \ub355\ud61c\uc639\uc8fc\uc758 \uc720\ubb3c\uc774 \uc788\ub294 \ubc15\ubb3c\uad00\uc740?", "paragraph": "\uc639\uc8fc\uac00 \uc0ac\uc6a9\ud588\ub358 \uc720\ubb3c\uc740 \ud55c\uad6d\uacfc \uc77c\ubcf8\uc5d0\ub3c4 \uadf8\ub9ac \ub9ce\uc740 \uc591\uc740 \ub0a8\uc544\uc788\uc9c0 \uc54a\uc73c\ub098, \uc720\ubb3c\uc774 \uc18c\uc7ac\ud55c \uba87\uba87\uc758 \uc704\uce58\ub97c \uc0b4\ud3b4\ubcf4\uc790\uba74 \ub2e4\uc74c\uacfc \uac19\ub2e4. \uba3c\uc800 \ub3c4\ucfc4\ubb38\ud654\ud559\uc6d0\uc5d0\ub294 \uc870\uc120 \uc655\uac00\ub85c\ubd80\ud130 \uae30\uc99d \ubc1b\uc740 \uc639\uc8fc\uc758 \uc720\ubb3c\uc774 \ub0a8\uc544\uc788\ub294\ub370, \uadf8\ub140\uac00 \uc785\uc5c8\ub358 \ub2f9\uc758\ub098 \uadf8\ub140\uc758 \ub538 \ub9c8\uc0ac\uc5d0\uac00 \uc0ac\uc6a9\ud55c \uc720\uc544\ubcf5\uacfc \ub098\ub9c9\uc2e0 \ub4f1\uc774 \uc18c\uc7a5\ub418\uc5b4 \uc788\ub2e4. \uc774\uc911 \ubcf5\uc2dd 7\uc810(\ub2f9\uc758, \ud64d\uc0c9 \uc2a4\ub780\uce58\ub9c8, \uce58\ub9c8, \uc1a1\ud654\uc0c9 \uc219\uace0\uc0ac \ubc18\ud68c\uc7a5\uc800\uace0\ub9ac, \uc9c4\ubd84\ud64d \uc800\uace0\ub9ac, \ud48d\ucc28\ubc14\uc9c0, \ub2e8\uc18d\uacf3)\uc774 \ucd08\uc804\uc12c\uc720\ud03c\ud2b8\ubc15\ubb3c\uad00 \uad00\uc7a5\uc758 \uc624\ub79c \uc124\ub4dd \ub05d\uc5d0 2015\ub144 6\uc6d4 24\uc77c \ub300\ud55c\ubbfc\uad6d\uc5d0 \ubc18\ud658\ub418\uc5b4 \uad6d\ub9bd\uace0\uad81\ubc15\ubb3c\uad00\uc5d0 \ubcf4\uad00\ub418\uc5c8\ub2e4. \ud050\uc288\uad6d\ub9bd\ubc15\ubb3c\uad00\uc5d0\ub294 \uc639\uc8fc\uc758 \ub0a8\ud3b8\uc774\uc5c8\ub358 \uc18c \ub2e4\ucf00\uc720\ud0a4\uc758 \uc9d1\uc548\uc778 \uc4f0\uc2dc\ub9c8 \uc885\uac00 \uc9d1\uc548\uc5d0\uc11c \uae30\uc99d\ubc1b\uc740 \uc720\ubb3c\uc774 \ub0a8\uc544\uc788\ub294\ub370, \uc639\uc8fc\uac00 \uc0ac\uc6a9\ud55c \uc2dd\uae30, \uc637\uac10, \ud68c\ud654 \ub4f1\uc774 \uc804\uc2dc\ub418\uc5b4 \uc788\ub2e4.", "answer": "\ud050\uc288\uad6d\ub9bd\ubc15\ubb3c\uad00", "sentence": "\ud050\uc288\uad6d\ub9bd\ubc15\ubb3c\uad00 \uc5d0\ub294 \uc639\uc8fc\uc758 \ub0a8\ud3b8\uc774\uc5c8\ub358 \uc18c \ub2e4\ucf00\uc720\ud0a4\uc758 \uc9d1\uc548\uc778 \uc4f0\uc2dc\ub9c8 \uc885\uac00 \uc9d1\uc548\uc5d0\uc11c \uae30\uc99d\ubc1b\uc740 \uc720\ubb3c\uc774 \ub0a8\uc544\uc788\ub294\ub370, \uc639\uc8fc\uac00 \uc0ac\uc6a9\ud55c \uc2dd\uae30, \uc637\uac10, \ud68c\ud654 \ub4f1\uc774 \uc804\uc2dc\ub418\uc5b4 \uc788\ub2e4.", "paragraph_sentence": "\uc639\uc8fc\uac00 \uc0ac\uc6a9\ud588\ub358 \uc720\ubb3c\uc740 \ud55c\uad6d\uacfc \uc77c\ubcf8\uc5d0\ub3c4 \uadf8\ub9ac \ub9ce\uc740 \uc591\uc740 \ub0a8\uc544\uc788\uc9c0 \uc54a\uc73c\ub098, \uc720\ubb3c\uc774 \uc18c\uc7ac\ud55c \uba87\uba87\uc758 \uc704\uce58\ub97c \uc0b4\ud3b4\ubcf4\uc790\uba74 \ub2e4\uc74c\uacfc \uac19\ub2e4. \uba3c\uc800 \ub3c4\ucfc4\ubb38\ud654\ud559\uc6d0\uc5d0\ub294 \uc870\uc120 \uc655\uac00\ub85c\ubd80\ud130 \uae30\uc99d \ubc1b\uc740 \uc639\uc8fc\uc758 \uc720\ubb3c\uc774 \ub0a8\uc544\uc788\ub294\ub370, \uadf8\ub140\uac00 \uc785\uc5c8\ub358 \ub2f9\uc758\ub098 \uadf8\ub140\uc758 \ub538 \ub9c8\uc0ac\uc5d0\uac00 \uc0ac\uc6a9\ud55c \uc720\uc544\ubcf5\uacfc \ub098\ub9c9\uc2e0 \ub4f1\uc774 \uc18c\uc7a5\ub418\uc5b4 \uc788\ub2e4. \uc774\uc911 \ubcf5\uc2dd 7\uc810(\ub2f9\uc758, \ud64d\uc0c9 \uc2a4\ub780\uce58\ub9c8, \uce58\ub9c8, \uc1a1\ud654\uc0c9 \uc219\uace0\uc0ac \ubc18\ud68c\uc7a5\uc800\uace0\ub9ac, \uc9c4\ubd84\ud64d \uc800\uace0\ub9ac, \ud48d\ucc28\ubc14\uc9c0, \ub2e8\uc18d\uacf3)\uc774 \ucd08\uc804\uc12c\uc720\ud03c\ud2b8\ubc15\ubb3c\uad00 \uad00\uc7a5\uc758 \uc624\ub79c \uc124\ub4dd \ub05d\uc5d0 2015\ub144 6\uc6d4 24\uc77c \ub300\ud55c\ubbfc\uad6d\uc5d0 \ubc18\ud658\ub418\uc5b4 \uad6d\ub9bd\uace0\uad81\ubc15\ubb3c\uad00\uc5d0 \ubcf4\uad00\ub418\uc5c8\ub2e4. \ud050\uc288\uad6d\ub9bd\ubc15\ubb3c\uad00 \uc5d0\ub294 \uc639\uc8fc\uc758 \ub0a8\ud3b8\uc774\uc5c8\ub358 \uc18c \ub2e4\ucf00\uc720\ud0a4\uc758 \uc9d1\uc548\uc778 \uc4f0\uc2dc\ub9c8 \uc885\uac00 \uc9d1\uc548\uc5d0\uc11c \uae30\uc99d\ubc1b\uc740 \uc720\ubb3c\uc774 \ub0a8\uc544\uc788\ub294\ub370, \uc639\uc8fc\uac00 \uc0ac\uc6a9\ud55c \uc2dd\uae30, \uc637\uac10, \ud68c\ud654 \ub4f1\uc774 \uc804\uc2dc\ub418\uc5b4 \uc788\ub2e4. ", "paragraph_answer": "\uc639\uc8fc\uac00 \uc0ac\uc6a9\ud588\ub358 \uc720\ubb3c\uc740 \ud55c\uad6d\uacfc \uc77c\ubcf8\uc5d0\ub3c4 \uadf8\ub9ac \ub9ce\uc740 \uc591\uc740 \ub0a8\uc544\uc788\uc9c0 \uc54a\uc73c\ub098, \uc720\ubb3c\uc774 \uc18c\uc7ac\ud55c \uba87\uba87\uc758 \uc704\uce58\ub97c \uc0b4\ud3b4\ubcf4\uc790\uba74 \ub2e4\uc74c\uacfc \uac19\ub2e4. \uba3c\uc800 \ub3c4\ucfc4\ubb38\ud654\ud559\uc6d0\uc5d0\ub294 \uc870\uc120 \uc655\uac00\ub85c\ubd80\ud130 \uae30\uc99d \ubc1b\uc740 \uc639\uc8fc\uc758 \uc720\ubb3c\uc774 \ub0a8\uc544\uc788\ub294\ub370, \uadf8\ub140\uac00 \uc785\uc5c8\ub358 \ub2f9\uc758\ub098 \uadf8\ub140\uc758 \ub538 \ub9c8\uc0ac\uc5d0\uac00 \uc0ac\uc6a9\ud55c \uc720\uc544\ubcf5\uacfc \ub098\ub9c9\uc2e0 \ub4f1\uc774 \uc18c\uc7a5\ub418\uc5b4 \uc788\ub2e4. \uc774\uc911 \ubcf5\uc2dd 7\uc810(\ub2f9\uc758, \ud64d\uc0c9 \uc2a4\ub780\uce58\ub9c8, \uce58\ub9c8, \uc1a1\ud654\uc0c9 \uc219\uace0\uc0ac \ubc18\ud68c\uc7a5\uc800\uace0\ub9ac, \uc9c4\ubd84\ud64d \uc800\uace0\ub9ac, \ud48d\ucc28\ubc14\uc9c0, \ub2e8\uc18d\uacf3)\uc774 \ucd08\uc804\uc12c\uc720\ud03c\ud2b8\ubc15\ubb3c\uad00 \uad00\uc7a5\uc758 \uc624\ub79c \uc124\ub4dd \ub05d\uc5d0 2015\ub144 6\uc6d4 24\uc77c \ub300\ud55c\ubbfc\uad6d\uc5d0 \ubc18\ud658\ub418\uc5b4 \uad6d\ub9bd\uace0\uad81\ubc15\ubb3c\uad00\uc5d0 \ubcf4\uad00\ub418\uc5c8\ub2e4. \ud050\uc288\uad6d\ub9bd\ubc15\ubb3c\uad00 \uc5d0\ub294 \uc639\uc8fc\uc758 \ub0a8\ud3b8\uc774\uc5c8\ub358 \uc18c \ub2e4\ucf00\uc720\ud0a4\uc758 \uc9d1\uc548\uc778 \uc4f0\uc2dc\ub9c8 \uc885\uac00 \uc9d1\uc548\uc5d0\uc11c \uae30\uc99d\ubc1b\uc740 \uc720\ubb3c\uc774 \ub0a8\uc544\uc788\ub294\ub370, \uc639\uc8fc\uac00 \uc0ac\uc6a9\ud55c \uc2dd\uae30, \uc637\uac10, \ud68c\ud654 \ub4f1\uc774 \uc804\uc2dc\ub418\uc5b4 \uc788\ub2e4.", "sentence_answer": " \ud050\uc288\uad6d\ub9bd\ubc15\ubb3c\uad00 \uc5d0\ub294 \uc639\uc8fc\uc758 \ub0a8\ud3b8\uc774\uc5c8\ub358 \uc18c \ub2e4\ucf00\uc720\ud0a4\uc758 \uc9d1\uc548\uc778 \uc4f0\uc2dc\ub9c8 \uc885\uac00 \uc9d1\uc548\uc5d0\uc11c \uae30\uc99d\ubc1b\uc740 \uc720\ubb3c\uc774 \ub0a8\uc544\uc788\ub294\ub370, \uc639\uc8fc\uac00 \uc0ac\uc6a9\ud55c \uc2dd\uae30, \uc637\uac10, \ud68c\ud654 \ub4f1\uc774 \uc804\uc2dc\ub418\uc5b4 \uc788\ub2e4.", "paragraph_id": "6491328-6-1", "paragraph_question": "question: \uc18c \ub2e4\ucf00\uc720\ud0a4\uc758 \uc9d1\uc548\uc5d0\uc11c \uae30\uc99d\ud55c \ub355\ud61c\uc639\uc8fc\uc758 \uc720\ubb3c\uc774 \uc788\ub294 \ubc15\ubb3c\uad00\uc740?, context: \uc639\uc8fc\uac00 \uc0ac\uc6a9\ud588\ub358 \uc720\ubb3c\uc740 \ud55c\uad6d\uacfc \uc77c\ubcf8\uc5d0\ub3c4 \uadf8\ub9ac \ub9ce\uc740 \uc591\uc740 \ub0a8\uc544\uc788\uc9c0 \uc54a\uc73c\ub098, \uc720\ubb3c\uc774 \uc18c\uc7ac\ud55c \uba87\uba87\uc758 \uc704\uce58\ub97c \uc0b4\ud3b4\ubcf4\uc790\uba74 \ub2e4\uc74c\uacfc \uac19\ub2e4. \uba3c\uc800 \ub3c4\ucfc4\ubb38\ud654\ud559\uc6d0\uc5d0\ub294 \uc870\uc120 \uc655\uac00\ub85c\ubd80\ud130 \uae30\uc99d \ubc1b\uc740 \uc639\uc8fc\uc758 \uc720\ubb3c\uc774 \ub0a8\uc544\uc788\ub294\ub370, \uadf8\ub140\uac00 \uc785\uc5c8\ub358 \ub2f9\uc758\ub098 \uadf8\ub140\uc758 \ub538 \ub9c8\uc0ac\uc5d0\uac00 \uc0ac\uc6a9\ud55c \uc720\uc544\ubcf5\uacfc \ub098\ub9c9\uc2e0 \ub4f1\uc774 \uc18c\uc7a5\ub418\uc5b4 \uc788\ub2e4. \uc774\uc911 \ubcf5\uc2dd 7\uc810(\ub2f9\uc758, \ud64d\uc0c9 \uc2a4\ub780\uce58\ub9c8, \uce58\ub9c8, \uc1a1\ud654\uc0c9 \uc219\uace0\uc0ac \ubc18\ud68c\uc7a5\uc800\uace0\ub9ac, \uc9c4\ubd84\ud64d \uc800\uace0\ub9ac, \ud48d\ucc28\ubc14\uc9c0, \ub2e8\uc18d\uacf3)\uc774 \ucd08\uc804\uc12c\uc720\ud03c\ud2b8\ubc15\ubb3c\uad00 \uad00\uc7a5\uc758 \uc624\ub79c \uc124\ub4dd \ub05d\uc5d0 2015\ub144 6\uc6d4 24\uc77c \ub300\ud55c\ubbfc\uad6d\uc5d0 \ubc18\ud658\ub418\uc5b4 \uad6d\ub9bd\uace0\uad81\ubc15\ubb3c\uad00\uc5d0 \ubcf4\uad00\ub418\uc5c8\ub2e4. \ud050\uc288\uad6d\ub9bd\ubc15\ubb3c\uad00\uc5d0\ub294 \uc639\uc8fc\uc758 \ub0a8\ud3b8\uc774\uc5c8\ub358 \uc18c \ub2e4\ucf00\uc720\ud0a4\uc758 \uc9d1\uc548\uc778 \uc4f0\uc2dc\ub9c8 \uc885\uac00 \uc9d1\uc548\uc5d0\uc11c \uae30\uc99d\ubc1b\uc740 \uc720\ubb3c\uc774 \ub0a8\uc544\uc788\ub294\ub370, \uc639\uc8fc\uac00 \uc0ac\uc6a9\ud55c \uc2dd\uae30, \uc637\uac10, \ud68c\ud654 \ub4f1\uc774 \uc804\uc2dc\ub418\uc5b4 \uc788\ub2e4."} {"question": "\uc544\ud06c\ub85c\uce78\ud1a0\uc0ac\uc6b0\ub8e8\uc2a4\uc640 \uc591\ucd94\uc544\ub178\uc0ac\uc6b0\ub8e8\uc2a4\ub294 \uc5b4\ub5a4\uacfc\uc5d0 \uc18d\ud558\ub294 \uacf5\ub8e1\uc778\uac00?", "paragraph": "\ub9e4\ub4dc\uc2a8\uc758 \uc601\ud5a5\ub825 \uc788\ub294 \ub17c\ubb38\uc5d0 \ub530\ub974\uba74, \uc54c\ub85c\uc0ac\uc6b0\ub8e8\uc2a4\ub294 \uc8fc\ub85c \uacfc\ub85c \ubd84\ub958\ub418\uc5c8\uc9c0\ub9cc, \uc54c\ub85c\uc0ac\uc6b0\ub8e8\uc2a4\uacfc\ub294 \ud655\uc2e4\ud788 \uc815\uc758\ub41c \uacfc\ub294 \uc544\ub2c8\uc5c8\ub2e4. \ubcf4\ud1b5 \uc900\uc804\ubb38\uc801\uc778 \uc791\ud488\ub4e4\uc5d0\uc11c\ub294 \u2018\uc54c\ub85c\uc0ac\uc6b0\ub8e8\uc2a4\uacfc\u2019\ub97c \uba54\uac08\ub85c\uc0ac\uc6b0\ub8e8\uc2a4\uacfc\ubcf4\ub2e4 \ud06c\uace0 \uc798 \uc54c\ub824\uc9c4 \ub2e4\uc591\ud55c \uac70\ub300 \uc218\uac01\ub958\ub4e4\uc744 \uac00\ub9ac\ud0a4\ub294 \uc6a9\uc5b4\ub85c \uc0ac\uc6a9\ud588\ub2e4. \uc54c\ub85c\uc0ac\uc6b0\ub8e8\uc2a4\uc640 \uadfc\uc5f0\ub41c\ub2e4\uace0 \uc0dd\uac01\ub418\uc5c8\ub358 \uc804\ud615\uc801\uc778 \uc218\uac01\ub958\ub85c\ub294 \uc778\ub3c4\uc0ac\uc6b0\ub8e8\uc2a4, \ud53c\uc544\ud2b8\ub2c8\uce20\ud0a4\uc0ac\uc6b0\ub8e8\uc2a4, \ud53c\ubca0\ud14c\uc544\uc6b0\uc0ac\uc6b0\ub8e8\uc2a4, \uc591\ucd94\uc544\ub178\uc0ac\uc6b0\ub8e8\uc2a4, \uc544\ud06c\ub85c\uce78\ud1a0\uc0ac\uc6b0\ub8e8\uc2a4, \ud0a4\ub780\ud14c\uc0ac\uc6b0\ub8e8\uc2a4, \ucf64\ud504\uc18c\uc218\ucfe0\uc2a4, \uc2a4\ud1a0\ucf00\uc18c\uc0ac\uc6b0\ub8e8\uc2a4, \uc2a4\uccb4\ucd94\uc544\ub178\uc0ac\uc6b0\ub8e8\uc2a4\uac00 \uc788\ub2e4. \ud604\uc7ac\ub294 \uc218\uac01\ub958\uc758 \ub2e4\uc591\uc131\uc5d0 \ub300\ud55c \ud604\ub300 \uc9c0\uc2dd\uacfc \uc9c4\ud654\uc801 \uc720\uc5f0 \uad00\uacc4\uc758 \uacc4\ud1b5\ubc1c\uc0dd\ud559\uc801 \uc5f0\uad6c\uc758 \ub3c4\ub798\ub85c \uc774\ub4e4 \uc218\uac01\ub958 \uc911 \ub300\ubd80\ubd84\uc740 \uc54c\ub85c\uc0ac\uc6b0\ub8e8\uc2a4\uacfc\ub85c \ubd84\ub958\ub418\uc9c0 \uc54a\ub294\ub2e4. \ub2e4\ub9cc \uc544\ud06c\ub85c\uce78\ud1a0\uc0ac\uc6b0\ub8e8\uc2a4\uc640 \uc591\ucd94\uc544\ub178\uc0ac\uc6b0\ub8e8\uc2a4\ub294 \uadfc\uc5f0\uacfc\uc5d0 \uc18d\ud55c\ub2e4.", "answer": "\uadfc\uc5f0\uacfc", "sentence": "\ub2e4\ub9cc \uc544\ud06c\ub85c\uce78\ud1a0\uc0ac\uc6b0\ub8e8\uc2a4\uc640 \uc591\ucd94\uc544\ub178\uc0ac\uc6b0\ub8e8\uc2a4\ub294 \uadfc\uc5f0\uacfc \uc5d0", "paragraph_sentence": "\ub9e4\ub4dc\uc2a8\uc758 \uc601\ud5a5\ub825 \uc788\ub294 \ub17c\ubb38\uc5d0 \ub530\ub974\uba74, \uc54c\ub85c\uc0ac\uc6b0\ub8e8\uc2a4\ub294 \uc8fc\ub85c \uacfc\ub85c \ubd84\ub958\ub418\uc5c8\uc9c0\ub9cc, \uc54c\ub85c\uc0ac\uc6b0\ub8e8\uc2a4\uacfc\ub294 \ud655\uc2e4\ud788 \uc815\uc758\ub41c \uacfc\ub294 \uc544\ub2c8\uc5c8\ub2e4. \ubcf4\ud1b5 \uc900\uc804\ubb38\uc801\uc778 \uc791\ud488\ub4e4\uc5d0\uc11c\ub294 \u2018\uc54c\ub85c\uc0ac\uc6b0\ub8e8\uc2a4\uacfc\u2019\ub97c \uba54\uac08\ub85c\uc0ac\uc6b0\ub8e8\uc2a4\uacfc\ubcf4\ub2e4 \ud06c\uace0 \uc798 \uc54c\ub824\uc9c4 \ub2e4\uc591\ud55c \uac70\ub300 \uc218\uac01\ub958\ub4e4\uc744 \uac00\ub9ac\ud0a4\ub294 \uc6a9\uc5b4\ub85c \uc0ac\uc6a9\ud588\ub2e4. \uc54c\ub85c\uc0ac\uc6b0\ub8e8\uc2a4\uc640 \uadfc\uc5f0\ub41c\ub2e4\uace0 \uc0dd\uac01\ub418\uc5c8\ub358 \uc804\ud615\uc801\uc778 \uc218\uac01\ub958\ub85c\ub294 \uc778\ub3c4\uc0ac\uc6b0\ub8e8\uc2a4, \ud53c\uc544\ud2b8\ub2c8\uce20\ud0a4\uc0ac\uc6b0\ub8e8\uc2a4, \ud53c\ubca0\ud14c\uc544\uc6b0\uc0ac\uc6b0\ub8e8\uc2a4, \uc591\ucd94\uc544\ub178\uc0ac\uc6b0\ub8e8\uc2a4, \uc544\ud06c\ub85c\uce78\ud1a0\uc0ac\uc6b0\ub8e8\uc2a4, \ud0a4\ub780\ud14c\uc0ac\uc6b0\ub8e8\uc2a4, \ucf64\ud504\uc18c\uc218\ucfe0\uc2a4, \uc2a4\ud1a0\ucf00\uc18c\uc0ac\uc6b0\ub8e8\uc2a4, \uc2a4\uccb4\ucd94\uc544\ub178\uc0ac\uc6b0\ub8e8\uc2a4\uac00 \uc788\ub2e4. \ud604\uc7ac\ub294 \uc218\uac01\ub958\uc758 \ub2e4\uc591\uc131\uc5d0 \ub300\ud55c \ud604\ub300 \uc9c0\uc2dd\uacfc \uc9c4\ud654\uc801 \uc720\uc5f0 \uad00\uacc4\uc758 \uacc4\ud1b5\ubc1c\uc0dd\ud559\uc801 \uc5f0\uad6c\uc758 \ub3c4\ub798\ub85c \uc774\ub4e4 \uc218\uac01\ub958 \uc911 \ub300\ubd80\ubd84\uc740 \uc54c\ub85c\uc0ac\uc6b0\ub8e8\uc2a4\uacfc\ub85c \ubd84\ub958\ub418\uc9c0 \uc54a\ub294\ub2e4. \ub2e4\ub9cc \uc544\ud06c\ub85c\uce78\ud1a0\uc0ac\uc6b0\ub8e8\uc2a4\uc640 \uc591\ucd94\uc544\ub178\uc0ac\uc6b0\ub8e8\uc2a4\ub294 \uadfc\uc5f0\uacfc \uc5d0 \uc18d\ud55c\ub2e4.", "paragraph_answer": "\ub9e4\ub4dc\uc2a8\uc758 \uc601\ud5a5\ub825 \uc788\ub294 \ub17c\ubb38\uc5d0 \ub530\ub974\uba74, \uc54c\ub85c\uc0ac\uc6b0\ub8e8\uc2a4\ub294 \uc8fc\ub85c \uacfc\ub85c \ubd84\ub958\ub418\uc5c8\uc9c0\ub9cc, \uc54c\ub85c\uc0ac\uc6b0\ub8e8\uc2a4\uacfc\ub294 \ud655\uc2e4\ud788 \uc815\uc758\ub41c \uacfc\ub294 \uc544\ub2c8\uc5c8\ub2e4. \ubcf4\ud1b5 \uc900\uc804\ubb38\uc801\uc778 \uc791\ud488\ub4e4\uc5d0\uc11c\ub294 \u2018\uc54c\ub85c\uc0ac\uc6b0\ub8e8\uc2a4\uacfc\u2019\ub97c \uba54\uac08\ub85c\uc0ac\uc6b0\ub8e8\uc2a4\uacfc\ubcf4\ub2e4 \ud06c\uace0 \uc798 \uc54c\ub824\uc9c4 \ub2e4\uc591\ud55c \uac70\ub300 \uc218\uac01\ub958\ub4e4\uc744 \uac00\ub9ac\ud0a4\ub294 \uc6a9\uc5b4\ub85c \uc0ac\uc6a9\ud588\ub2e4. \uc54c\ub85c\uc0ac\uc6b0\ub8e8\uc2a4\uc640 \uadfc\uc5f0\ub41c\ub2e4\uace0 \uc0dd\uac01\ub418\uc5c8\ub358 \uc804\ud615\uc801\uc778 \uc218\uac01\ub958\ub85c\ub294 \uc778\ub3c4\uc0ac\uc6b0\ub8e8\uc2a4, \ud53c\uc544\ud2b8\ub2c8\uce20\ud0a4\uc0ac\uc6b0\ub8e8\uc2a4, \ud53c\ubca0\ud14c\uc544\uc6b0\uc0ac\uc6b0\ub8e8\uc2a4, \uc591\ucd94\uc544\ub178\uc0ac\uc6b0\ub8e8\uc2a4, \uc544\ud06c\ub85c\uce78\ud1a0\uc0ac\uc6b0\ub8e8\uc2a4, \ud0a4\ub780\ud14c\uc0ac\uc6b0\ub8e8\uc2a4, \ucf64\ud504\uc18c\uc218\ucfe0\uc2a4, \uc2a4\ud1a0\ucf00\uc18c\uc0ac\uc6b0\ub8e8\uc2a4, \uc2a4\uccb4\ucd94\uc544\ub178\uc0ac\uc6b0\ub8e8\uc2a4\uac00 \uc788\ub2e4. \ud604\uc7ac\ub294 \uc218\uac01\ub958\uc758 \ub2e4\uc591\uc131\uc5d0 \ub300\ud55c \ud604\ub300 \uc9c0\uc2dd\uacfc \uc9c4\ud654\uc801 \uc720\uc5f0 \uad00\uacc4\uc758 \uacc4\ud1b5\ubc1c\uc0dd\ud559\uc801 \uc5f0\uad6c\uc758 \ub3c4\ub798\ub85c \uc774\ub4e4 \uc218\uac01\ub958 \uc911 \ub300\ubd80\ubd84\uc740 \uc54c\ub85c\uc0ac\uc6b0\ub8e8\uc2a4\uacfc\ub85c \ubd84\ub958\ub418\uc9c0 \uc54a\ub294\ub2e4. \ub2e4\ub9cc \uc544\ud06c\ub85c\uce78\ud1a0\uc0ac\uc6b0\ub8e8\uc2a4\uc640 \uc591\ucd94\uc544\ub178\uc0ac\uc6b0\ub8e8\uc2a4\ub294 \uadfc\uc5f0\uacfc \uc5d0 \uc18d\ud55c\ub2e4.", "sentence_answer": "\ub2e4\ub9cc \uc544\ud06c\ub85c\uce78\ud1a0\uc0ac\uc6b0\ub8e8\uc2a4\uc640 \uc591\ucd94\uc544\ub178\uc0ac\uc6b0\ub8e8\uc2a4\ub294 \uadfc\uc5f0\uacfc \uc5d0", "paragraph_id": "6465441-5-1", "paragraph_question": "question: \uc544\ud06c\ub85c\uce78\ud1a0\uc0ac\uc6b0\ub8e8\uc2a4\uc640 \uc591\ucd94\uc544\ub178\uc0ac\uc6b0\ub8e8\uc2a4\ub294 \uc5b4\ub5a4\uacfc\uc5d0 \uc18d\ud558\ub294 \uacf5\ub8e1\uc778\uac00?, context: \ub9e4\ub4dc\uc2a8\uc758 \uc601\ud5a5\ub825 \uc788\ub294 \ub17c\ubb38\uc5d0 \ub530\ub974\uba74, \uc54c\ub85c\uc0ac\uc6b0\ub8e8\uc2a4\ub294 \uc8fc\ub85c \uacfc\ub85c \ubd84\ub958\ub418\uc5c8\uc9c0\ub9cc, \uc54c\ub85c\uc0ac\uc6b0\ub8e8\uc2a4\uacfc\ub294 \ud655\uc2e4\ud788 \uc815\uc758\ub41c \uacfc\ub294 \uc544\ub2c8\uc5c8\ub2e4. \ubcf4\ud1b5 \uc900\uc804\ubb38\uc801\uc778 \uc791\ud488\ub4e4\uc5d0\uc11c\ub294 \u2018\uc54c\ub85c\uc0ac\uc6b0\ub8e8\uc2a4\uacfc\u2019\ub97c \uba54\uac08\ub85c\uc0ac\uc6b0\ub8e8\uc2a4\uacfc\ubcf4\ub2e4 \ud06c\uace0 \uc798 \uc54c\ub824\uc9c4 \ub2e4\uc591\ud55c \uac70\ub300 \uc218\uac01\ub958\ub4e4\uc744 \uac00\ub9ac\ud0a4\ub294 \uc6a9\uc5b4\ub85c \uc0ac\uc6a9\ud588\ub2e4. \uc54c\ub85c\uc0ac\uc6b0\ub8e8\uc2a4\uc640 \uadfc\uc5f0\ub41c\ub2e4\uace0 \uc0dd\uac01\ub418\uc5c8\ub358 \uc804\ud615\uc801\uc778 \uc218\uac01\ub958\ub85c\ub294 \uc778\ub3c4\uc0ac\uc6b0\ub8e8\uc2a4, \ud53c\uc544\ud2b8\ub2c8\uce20\ud0a4\uc0ac\uc6b0\ub8e8\uc2a4, \ud53c\ubca0\ud14c\uc544\uc6b0\uc0ac\uc6b0\ub8e8\uc2a4, \uc591\ucd94\uc544\ub178\uc0ac\uc6b0\ub8e8\uc2a4, \uc544\ud06c\ub85c\uce78\ud1a0\uc0ac\uc6b0\ub8e8\uc2a4, \ud0a4\ub780\ud14c\uc0ac\uc6b0\ub8e8\uc2a4, \ucf64\ud504\uc18c\uc218\ucfe0\uc2a4, \uc2a4\ud1a0\ucf00\uc18c\uc0ac\uc6b0\ub8e8\uc2a4, \uc2a4\uccb4\ucd94\uc544\ub178\uc0ac\uc6b0\ub8e8\uc2a4\uac00 \uc788\ub2e4. \ud604\uc7ac\ub294 \uc218\uac01\ub958\uc758 \ub2e4\uc591\uc131\uc5d0 \ub300\ud55c \ud604\ub300 \uc9c0\uc2dd\uacfc \uc9c4\ud654\uc801 \uc720\uc5f0 \uad00\uacc4\uc758 \uacc4\ud1b5\ubc1c\uc0dd\ud559\uc801 \uc5f0\uad6c\uc758 \ub3c4\ub798\ub85c \uc774\ub4e4 \uc218\uac01\ub958 \uc911 \ub300\ubd80\ubd84\uc740 \uc54c\ub85c\uc0ac\uc6b0\ub8e8\uc2a4\uacfc\ub85c \ubd84\ub958\ub418\uc9c0 \uc54a\ub294\ub2e4. \ub2e4\ub9cc \uc544\ud06c\ub85c\uce78\ud1a0\uc0ac\uc6b0\ub8e8\uc2a4\uc640 \uc591\ucd94\uc544\ub178\uc0ac\uc6b0\ub8e8\uc2a4\ub294 \uadfc\uc5f0\uacfc\uc5d0 \uc18d\ud55c\ub2e4."} {"question": "\uc624\uc0ac\uce74 \ubc18\ub3c4 \uc55e\ubc14\ub2e4\uc5d0\uc11c \uc2dc\uc791\ub41c \ub2e8\uce35 \ud30c\uad34\ub294 \uc5b4\ub290 \ubc29\ud5a5\uc73c\ub85c \uc774\uc5b4\uc84c\ub098?", "paragraph": "\ub2e8\uce35 \ud30c\uad34\uac00 \ucc98\uc74c\uc73c\ub85c \uc2dc\uc791\ub41c \uc9c4\uc6d0\uc9c0\uc778 \uc0b0\ub9ac\ucfe0 \ub0a8\ubd80 \ud574\uad6c\uba74 \uc9c0\uc5ed\uacfc \uc0b0\ub9ac\ucfe0\uc5d0\uc11c \ubcf4\uc18c \uc5f0\uc548\uae4c\uc9c0 \ud574\uad6c \ucabd \uc9c0\uc5ed\uc758 \uc911\ubd80 \uc9c0\uc5ed\uc5d0\uc11c\ub294 20m \uc774\uc0c1\uc758 \ub9e4\uc6b0 \ud070 \ub2e8\uce35 \uc6b4\ub3d9\uc774 \uc77c\uc5b4\ub0ac\uc73c\uba70, \ub3d9\uc77c\ubcf8\ub300\uc9c0\uc9c4\uc758 \uc9c4\uc6d0\uc5ed\uc778 \ub0a8\ubd81 500km \uae38\uc774, \ub3d9\uc11c 200km \ub113\uc774\uc758 \uc9c0\uc5ed\uc740 \uc0b0\ub9ac\ucfe0 \uc911\ubd80, \ubbf8\uc57c\uae30 \ud604 \uc55e\ubc14\ub2e4, \ud6c4\ucfe0\uc2dc\ub9c8 \ud604 \uc55e\ubc14\ub2e4, \uc774\ubc14\ub77c\ud0a4 \ud604 \uc55e\ubc14\ub2e4 \ub4f1 6\uac1c \uc9c0\uc5ed \uc548\uc5d0 \uc788\ub2e4. \ub2e8\uce35 \ud30c\uad34\ub294 \uc624\uc2dc\uce74 \ubc18\ub3c4 \uc55e\ubc14\ub2e4\uc11c\ubd80\ud130 \ub0a8\ubd81\uc73c\ub85c \ucb48\uc6b1 \uc774\uc5b4\uc84c\uc73c\ub098, \ubd81\uc544\uba54\ub9ac\uce74 \ud310 \uc544\ub798\ub85c \uce68\uac15\ud574 \uc788\ub294 \ud544\ub9ac\ud540\ud574 \ud310 \ubd81\ub3d9\ucabd\uc774 \uc9c0\uac01 \ud30c\uad34\uac00 \ub0a8\ucabd\uc73c\ub85c \ub0b4\ub824\uac00\ub294 \uac83\uc744 \ub9c9\uc544\uc11c \ubcf4\uc18c \uc5f0\uc548 \ubd81\ubd80\uc758 \uc774\ubc14\ub77c\ud0a4 \ud604 \uc55e\ubc14\ub2e4\uc5d0\uc11c \uc9c0\uac01 \ubcc0\ub3d9\uc774 \uba48\ucdc4\ub2e4. \ubd81\ucabd\uc73c\ub85c\ub294 \uc9c4\uc6d0\uc5ed\uc774 1994\ub144 \uc0b0\ub9ac\ucfe0 \uba3c\ubc14\ub2e4 \uc9c0\uc9c4 \ub610\ub294 1968\ub144 \uc0b0\ub9ac\ucfe0 \ubd81\ubd80 \uc9c0\uc9c4\uc758 \uc9c4\uc6d0\uc5ed \ub0a8\ubd80\uc5d0\uc11c \uba48\ucdc4\ub2e4. \uc774 \uc815\ub3c4\ub85c \uc9c4\uc6d0\uc5ed\uc774 \ub113\uc740 \uaddc\ubaa8 M9\uae09 \uc9c0\uc9c4\uc740 \uc9c0\uc9c4\uc870\uc0ac\uc704\uc6d0\ud68c\uc5d0\uc11c\ub3c4 \uac00\uc815\ud558\uc9c0 \ubabb\ud55c \ucd08\uac70\ub300\uc9c0\uc9c4\uc774\uc5c8\ub2e4.", "answer": "\ub0a8\ubd81", "sentence": "\ub2e8\uce35 \ud30c\uad34\uac00 \ucc98\uc74c\uc73c\ub85c \uc2dc\uc791\ub41c \uc9c4\uc6d0\uc9c0\uc778 \uc0b0\ub9ac\ucfe0 \ub0a8\ubd80 \ud574\uad6c\uba74 \uc9c0\uc5ed\uacfc \uc0b0\ub9ac\ucfe0\uc5d0\uc11c \ubcf4\uc18c \uc5f0\uc548\uae4c\uc9c0 \ud574\uad6c \ucabd \uc9c0\uc5ed\uc758 \uc911\ubd80 \uc9c0\uc5ed\uc5d0\uc11c\ub294 20m \uc774\uc0c1\uc758 \ub9e4\uc6b0 \ud070 \ub2e8\uce35 \uc6b4\ub3d9\uc774 \uc77c\uc5b4\ub0ac\uc73c\uba70, \ub3d9\uc77c\ubcf8\ub300\uc9c0\uc9c4\uc758 \uc9c4\uc6d0\uc5ed\uc778 \ub0a8\ubd81 500km \uae38\uc774, \ub3d9\uc11c 200km \ub113\uc774\uc758 \uc9c0\uc5ed\uc740 \uc0b0\ub9ac\ucfe0 \uc911\ubd80, \ubbf8\uc57c\uae30 \ud604 \uc55e\ubc14\ub2e4, \ud6c4\ucfe0\uc2dc\ub9c8 \ud604 \uc55e\ubc14\ub2e4, \uc774\ubc14\ub77c\ud0a4 \ud604 \uc55e\ubc14\ub2e4 \ub4f1 6\uac1c \uc9c0\uc5ed \uc548\uc5d0 \uc788\ub2e4.", "paragraph_sentence": " \ub2e8\uce35 \ud30c\uad34\uac00 \ucc98\uc74c\uc73c\ub85c \uc2dc\uc791\ub41c \uc9c4\uc6d0\uc9c0\uc778 \uc0b0\ub9ac\ucfe0 \ub0a8\ubd80 \ud574\uad6c\uba74 \uc9c0\uc5ed\uacfc \uc0b0\ub9ac\ucfe0\uc5d0\uc11c \ubcf4\uc18c \uc5f0\uc548\uae4c\uc9c0 \ud574\uad6c \ucabd \uc9c0\uc5ed\uc758 \uc911\ubd80 \uc9c0\uc5ed\uc5d0\uc11c\ub294 20m \uc774\uc0c1\uc758 \ub9e4\uc6b0 \ud070 \ub2e8\uce35 \uc6b4\ub3d9\uc774 \uc77c\uc5b4\ub0ac\uc73c\uba70, \ub3d9\uc77c\ubcf8\ub300\uc9c0\uc9c4\uc758 \uc9c4\uc6d0\uc5ed\uc778 \ub0a8\ubd81 500km \uae38\uc774, \ub3d9\uc11c 200km \ub113\uc774\uc758 \uc9c0\uc5ed\uc740 \uc0b0\ub9ac\ucfe0 \uc911\ubd80, \ubbf8\uc57c\uae30 \ud604 \uc55e\ubc14\ub2e4, \ud6c4\ucfe0\uc2dc\ub9c8 \ud604 \uc55e\ubc14\ub2e4, \uc774\ubc14\ub77c\ud0a4 \ud604 \uc55e\ubc14\ub2e4 \ub4f1 6\uac1c \uc9c0\uc5ed \uc548\uc5d0 \uc788\ub2e4. \ub2e8\uce35 \ud30c\uad34\ub294 \uc624\uc2dc\uce74 \ubc18\ub3c4 \uc55e\ubc14\ub2e4\uc11c\ubd80\ud130 \ub0a8\ubd81\uc73c\ub85c \ucb48\uc6b1 \uc774\uc5b4\uc84c\uc73c\ub098, \ubd81\uc544\uba54\ub9ac\uce74 \ud310 \uc544\ub798\ub85c \uce68\uac15\ud574 \uc788\ub294 \ud544\ub9ac\ud540\ud574 \ud310 \ubd81\ub3d9\ucabd\uc774 \uc9c0\uac01 \ud30c\uad34\uac00 \ub0a8\ucabd\uc73c\ub85c \ub0b4\ub824\uac00\ub294 \uac83\uc744 \ub9c9\uc544\uc11c \ubcf4\uc18c \uc5f0\uc548 \ubd81\ubd80\uc758 \uc774\ubc14\ub77c\ud0a4 \ud604 \uc55e\ubc14\ub2e4\uc5d0\uc11c \uc9c0\uac01 \ubcc0\ub3d9\uc774 \uba48\ucdc4\ub2e4. \ubd81\ucabd\uc73c\ub85c\ub294 \uc9c4\uc6d0\uc5ed\uc774 1994\ub144 \uc0b0\ub9ac\ucfe0 \uba3c\ubc14\ub2e4 \uc9c0\uc9c4 \ub610\ub294 1968\ub144 \uc0b0\ub9ac\ucfe0 \ubd81\ubd80 \uc9c0\uc9c4\uc758 \uc9c4\uc6d0\uc5ed \ub0a8\ubd80\uc5d0\uc11c \uba48\ucdc4\ub2e4. \uc774 \uc815\ub3c4\ub85c \uc9c4\uc6d0\uc5ed\uc774 \ub113\uc740 \uaddc\ubaa8 M9\uae09 \uc9c0\uc9c4\uc740 \uc9c0\uc9c4\uc870\uc0ac\uc704\uc6d0\ud68c\uc5d0\uc11c\ub3c4 \uac00\uc815\ud558\uc9c0 \ubabb\ud55c \ucd08\uac70\ub300\uc9c0\uc9c4\uc774\uc5c8\ub2e4.", "paragraph_answer": "\ub2e8\uce35 \ud30c\uad34\uac00 \ucc98\uc74c\uc73c\ub85c \uc2dc\uc791\ub41c \uc9c4\uc6d0\uc9c0\uc778 \uc0b0\ub9ac\ucfe0 \ub0a8\ubd80 \ud574\uad6c\uba74 \uc9c0\uc5ed\uacfc \uc0b0\ub9ac\ucfe0\uc5d0\uc11c \ubcf4\uc18c \uc5f0\uc548\uae4c\uc9c0 \ud574\uad6c \ucabd \uc9c0\uc5ed\uc758 \uc911\ubd80 \uc9c0\uc5ed\uc5d0\uc11c\ub294 20m \uc774\uc0c1\uc758 \ub9e4\uc6b0 \ud070 \ub2e8\uce35 \uc6b4\ub3d9\uc774 \uc77c\uc5b4\ub0ac\uc73c\uba70, \ub3d9\uc77c\ubcf8\ub300\uc9c0\uc9c4\uc758 \uc9c4\uc6d0\uc5ed\uc778 \ub0a8\ubd81 500km \uae38\uc774, \ub3d9\uc11c 200km \ub113\uc774\uc758 \uc9c0\uc5ed\uc740 \uc0b0\ub9ac\ucfe0 \uc911\ubd80, \ubbf8\uc57c\uae30 \ud604 \uc55e\ubc14\ub2e4, \ud6c4\ucfe0\uc2dc\ub9c8 \ud604 \uc55e\ubc14\ub2e4, \uc774\ubc14\ub77c\ud0a4 \ud604 \uc55e\ubc14\ub2e4 \ub4f1 6\uac1c \uc9c0\uc5ed \uc548\uc5d0 \uc788\ub2e4. \ub2e8\uce35 \ud30c\uad34\ub294 \uc624\uc2dc\uce74 \ubc18\ub3c4 \uc55e\ubc14\ub2e4\uc11c\ubd80\ud130 \ub0a8\ubd81\uc73c\ub85c \ucb48\uc6b1 \uc774\uc5b4\uc84c\uc73c\ub098, \ubd81\uc544\uba54\ub9ac\uce74 \ud310 \uc544\ub798\ub85c \uce68\uac15\ud574 \uc788\ub294 \ud544\ub9ac\ud540\ud574 \ud310 \ubd81\ub3d9\ucabd\uc774 \uc9c0\uac01 \ud30c\uad34\uac00 \ub0a8\ucabd\uc73c\ub85c \ub0b4\ub824\uac00\ub294 \uac83\uc744 \ub9c9\uc544\uc11c \ubcf4\uc18c \uc5f0\uc548 \ubd81\ubd80\uc758 \uc774\ubc14\ub77c\ud0a4 \ud604 \uc55e\ubc14\ub2e4\uc5d0\uc11c \uc9c0\uac01 \ubcc0\ub3d9\uc774 \uba48\ucdc4\ub2e4. \ubd81\ucabd\uc73c\ub85c\ub294 \uc9c4\uc6d0\uc5ed\uc774 1994\ub144 \uc0b0\ub9ac\ucfe0 \uba3c\ubc14\ub2e4 \uc9c0\uc9c4 \ub610\ub294 1968\ub144 \uc0b0\ub9ac\ucfe0 \ubd81\ubd80 \uc9c0\uc9c4\uc758 \uc9c4\uc6d0\uc5ed \ub0a8\ubd80\uc5d0\uc11c \uba48\ucdc4\ub2e4. \uc774 \uc815\ub3c4\ub85c \uc9c4\uc6d0\uc5ed\uc774 \ub113\uc740 \uaddc\ubaa8 M9\uae09 \uc9c0\uc9c4\uc740 \uc9c0\uc9c4\uc870\uc0ac\uc704\uc6d0\ud68c\uc5d0\uc11c\ub3c4 \uac00\uc815\ud558\uc9c0 \ubabb\ud55c \ucd08\uac70\ub300\uc9c0\uc9c4\uc774\uc5c8\ub2e4.", "sentence_answer": "\ub2e8\uce35 \ud30c\uad34\uac00 \ucc98\uc74c\uc73c\ub85c \uc2dc\uc791\ub41c \uc9c4\uc6d0\uc9c0\uc778 \uc0b0\ub9ac\ucfe0 \ub0a8\ubd80 \ud574\uad6c\uba74 \uc9c0\uc5ed\uacfc \uc0b0\ub9ac\ucfe0\uc5d0\uc11c \ubcf4\uc18c \uc5f0\uc548\uae4c\uc9c0 \ud574\uad6c \ucabd \uc9c0\uc5ed\uc758 \uc911\ubd80 \uc9c0\uc5ed\uc5d0\uc11c\ub294 20m \uc774\uc0c1\uc758 \ub9e4\uc6b0 \ud070 \ub2e8\uce35 \uc6b4\ub3d9\uc774 \uc77c\uc5b4\ub0ac\uc73c\uba70, \ub3d9\uc77c\ubcf8\ub300\uc9c0\uc9c4\uc758 \uc9c4\uc6d0\uc5ed\uc778 \ub0a8\ubd81 500km \uae38\uc774, \ub3d9\uc11c 200km \ub113\uc774\uc758 \uc9c0\uc5ed\uc740 \uc0b0\ub9ac\ucfe0 \uc911\ubd80, \ubbf8\uc57c\uae30 \ud604 \uc55e\ubc14\ub2e4, \ud6c4\ucfe0\uc2dc\ub9c8 \ud604 \uc55e\ubc14\ub2e4, \uc774\ubc14\ub77c\ud0a4 \ud604 \uc55e\ubc14\ub2e4 \ub4f1 6\uac1c \uc9c0\uc5ed \uc548\uc5d0 \uc788\ub2e4.", "paragraph_id": "6492086-11-0", "paragraph_question": "question: \uc624\uc0ac\uce74 \ubc18\ub3c4 \uc55e\ubc14\ub2e4\uc5d0\uc11c \uc2dc\uc791\ub41c \ub2e8\uce35 \ud30c\uad34\ub294 \uc5b4\ub290 \ubc29\ud5a5\uc73c\ub85c \uc774\uc5b4\uc84c\ub098?, context: \ub2e8\uce35 \ud30c\uad34\uac00 \ucc98\uc74c\uc73c\ub85c \uc2dc\uc791\ub41c \uc9c4\uc6d0\uc9c0\uc778 \uc0b0\ub9ac\ucfe0 \ub0a8\ubd80 \ud574\uad6c\uba74 \uc9c0\uc5ed\uacfc \uc0b0\ub9ac\ucfe0\uc5d0\uc11c \ubcf4\uc18c \uc5f0\uc548\uae4c\uc9c0 \ud574\uad6c \ucabd \uc9c0\uc5ed\uc758 \uc911\ubd80 \uc9c0\uc5ed\uc5d0\uc11c\ub294 20m \uc774\uc0c1\uc758 \ub9e4\uc6b0 \ud070 \ub2e8\uce35 \uc6b4\ub3d9\uc774 \uc77c\uc5b4\ub0ac\uc73c\uba70, \ub3d9\uc77c\ubcf8\ub300\uc9c0\uc9c4\uc758 \uc9c4\uc6d0\uc5ed\uc778 \ub0a8\ubd81 500km \uae38\uc774, \ub3d9\uc11c 200km \ub113\uc774\uc758 \uc9c0\uc5ed\uc740 \uc0b0\ub9ac\ucfe0 \uc911\ubd80, \ubbf8\uc57c\uae30 \ud604 \uc55e\ubc14\ub2e4, \ud6c4\ucfe0\uc2dc\ub9c8 \ud604 \uc55e\ubc14\ub2e4, \uc774\ubc14\ub77c\ud0a4 \ud604 \uc55e\ubc14\ub2e4 \ub4f1 6\uac1c \uc9c0\uc5ed \uc548\uc5d0 \uc788\ub2e4. \ub2e8\uce35 \ud30c\uad34\ub294 \uc624\uc2dc\uce74 \ubc18\ub3c4 \uc55e\ubc14\ub2e4\uc11c\ubd80\ud130 \ub0a8\ubd81\uc73c\ub85c \ucb48\uc6b1 \uc774\uc5b4\uc84c\uc73c\ub098, \ubd81\uc544\uba54\ub9ac\uce74 \ud310 \uc544\ub798\ub85c \uce68\uac15\ud574 \uc788\ub294 \ud544\ub9ac\ud540\ud574 \ud310 \ubd81\ub3d9\ucabd\uc774 \uc9c0\uac01 \ud30c\uad34\uac00 \ub0a8\ucabd\uc73c\ub85c \ub0b4\ub824\uac00\ub294 \uac83\uc744 \ub9c9\uc544\uc11c \ubcf4\uc18c \uc5f0\uc548 \ubd81\ubd80\uc758 \uc774\ubc14\ub77c\ud0a4 \ud604 \uc55e\ubc14\ub2e4\uc5d0\uc11c \uc9c0\uac01 \ubcc0\ub3d9\uc774 \uba48\ucdc4\ub2e4. \ubd81\ucabd\uc73c\ub85c\ub294 \uc9c4\uc6d0\uc5ed\uc774 1994\ub144 \uc0b0\ub9ac\ucfe0 \uba3c\ubc14\ub2e4 \uc9c0\uc9c4 \ub610\ub294 1968\ub144 \uc0b0\ub9ac\ucfe0 \ubd81\ubd80 \uc9c0\uc9c4\uc758 \uc9c4\uc6d0\uc5ed \ub0a8\ubd80\uc5d0\uc11c \uba48\ucdc4\ub2e4. \uc774 \uc815\ub3c4\ub85c \uc9c4\uc6d0\uc5ed\uc774 \ub113\uc740 \uaddc\ubaa8 M9\uae09 \uc9c0\uc9c4\uc740 \uc9c0\uc9c4\uc870\uc0ac\uc704\uc6d0\ud68c\uc5d0\uc11c\ub3c4 \uac00\uc815\ud558\uc9c0 \ubabb\ud55c \ucd08\uac70\ub300\uc9c0\uc9c4\uc774\uc5c8\ub2e4."} {"question": "\ud0dc\uc591\uacc4\uc5d0\uc11c \ubc1c\uacac\ub41c \ub300\ud615 \ucc9c\uccb4 \uc911 \uac00\uc7a5 \uae34 \uc138\ub4dc\ub098\uc758 \uacf5\uc804 \uc8fc\uae30\ub294?", "paragraph": "\uc138\ub4dc\ub098\ub294 \uc9c0\uae08\uae4c\uc9c0 \ud0dc\uc591\uacc4\uc5d0\uc11c \ubc1c\uacac\ub41c \ub300\ud615 \ucc9c\uccb4 \uc911 \uacf5\uc804 \uc8fc\uae30\uac00 \uac00\uc7a5 \uae38\uc5b4, 11,400 \ub144 \uc804\ud6c4\ub85c \uacc4\uc0b0\ub41c\ub2e4. \uadf8 \uada4\ub3c4\ub294 \uc774\uc2ec\ub960\uc774 \uadf9\ub3c4\ub85c \ucee4\uc11c \uc6d0\uc77c\uc810 \uac70\ub9ac\uac00 \uc57d 937 AU\uc774\uace0 \uadfc\uc77c\uc810 \uac70\ub9ac\uac00 \uc57d 76 AU \uc774\ub2e4. \uc774 \uc6d0\uc77c\uc810 \uac70\ub9ac\ub294 2012\ub144 2012 VP113\uc774 \ubc1c\uacac\ub418\uae30 \uc804\uae4c\uc9c0 \ud0dc\uc591\uacc4 \ucc9c\uccb4 \uc911 \uac00\uc7a5 \uba3c \uac70\ub9ac\uc600\ub2e4. \uc138\ub4dc\ub098\uac00 \ubc1c\uacac\ub418\uc5c8\uc744 \ub2f9\uc2dc \ud0dc\uc591\uc5d0\uc11c\uc758 \uac70\ub9ac\ub294 89.6 AU\uc600\uc73c\uba70 \uadfc\uc77c\uc810\uc744 \ud5a5\ud574 \uc6c0\uc9c1\uc774\uace0 \uc788\uc5c8\ub2e4. \uc774\ud6c4 \uac19\uc740 \ud0d0\uc0ac \uacfc\uc815\uc5d0\uc11c \ud0dc\uc591\uc73c\ub85c\ubd80\ud130 97 AU \ub5a8\uc5b4\uc9c4 \uadfc\uc77c\uc810 \uadfc\ucc98\uc5d0\uc11c \uc5d0\ub9ac\uc2a4\uac00 \uac10\uc9c0\ub418\uc5c8\ub2e4. \uc138\ub4dc\ub098\ubcf4\ub2e4 \uada4\ub3c4\uac00 \ub354 \uba3c \ucc9c\uccb4\ub294 \uc77c\ubd80 \uc7a5\uc8fc\uae30 \ud61c\uc131\ub4e4\ubc16\uc5d0 \uc5c6\ub2e4. \uadf8\ub9ac\uace0 \ud61c\uc131\ub4e4\uc740 \ub108\ubb34 \uc5b4\ub450\uc6cc\uc11c \ub0b4\ubd80 \ud0dc\uc591\uacc4\ub85c \ub4e4\uc5b4\uc640 \uadfc\uc77c\uc810\uc744 \ud5a5\ud560 \ub54c\uac00 \uc544\ub2c8\uba74 \ubcf4\uc774\uc9c0\ub3c4 \uc54a\ub294\ub2e4. \uc138\ub4dc\ub098\ub3c4 2076\ub144 \uc911\uc21c\uc5d0 \uadfc\uc77c\uc810\uc5d0 \ub2e4\ub2e4\ub974\uaca0\uc9c0\ub9cc, \ub108\ubb34 \uba40\uc5b4\uc11c \uc721\uc548\uc73c\ub85c\ub294 \uc6d0\ubc18\uc0c1\uc744 \ubcfc \uc218 \uc5c6\uc744 \uac83\uc774\ub2e4. \uc138\ub4dc\ub098 \ud45c\uba74\uc5d0\uc11c \ud558\ub298\uc744 \ubcf4\uba74 \ud0dc\uc591\uc774 \uadf8\uc800 \ub9e4\uc6b0 \ubc1d\uc740(\ubcf4\ub984\ub2ec\uc758 100\ubc30 \ubc1d\uae30) \ud56d\uc131 \uc911 \ud558\ub098\ub85c \ubcf4\uc77c \uac83\uc774\ub2e4. \uc9c0\uad6c\uc5d0\uc11c \ubcf4\uc774\ub294 \ud0dc\uc591\uc758 \ubc1d\uae30\ub294 \ubcf4\ub984\ub2ec\uc758 400,000 \ubc30 \ubc1d\uae30\uc774\ubbc0\ub85c \uc774\ub97c \ub2a5\ud788 \ube44\uad50\ud560 \uc218 \uc788\ub2e4.", "answer": "11,400 \ub144 \uc804\ud6c4", "sentence": "\uc138\ub4dc\ub098\ub294 \uc9c0\uae08\uae4c\uc9c0 \ud0dc\uc591\uacc4\uc5d0\uc11c \ubc1c\uacac\ub41c \ub300\ud615 \ucc9c\uccb4 \uc911 \uacf5\uc804 \uc8fc\uae30\uac00 \uac00\uc7a5 \uae38\uc5b4, 11,400 \ub144 \uc804\ud6c4 \ub85c \uacc4\uc0b0\ub41c\ub2e4.", "paragraph_sentence": " \uc138\ub4dc\ub098\ub294 \uc9c0\uae08\uae4c\uc9c0 \ud0dc\uc591\uacc4\uc5d0\uc11c \ubc1c\uacac\ub41c \ub300\ud615 \ucc9c\uccb4 \uc911 \uacf5\uc804 \uc8fc\uae30\uac00 \uac00\uc7a5 \uae38\uc5b4, 11,400 \ub144 \uc804\ud6c4 \ub85c \uacc4\uc0b0\ub41c\ub2e4. \uadf8 \uada4\ub3c4\ub294 \uc774\uc2ec\ub960\uc774 \uadf9\ub3c4\ub85c \ucee4\uc11c \uc6d0\uc77c\uc810 \uac70\ub9ac\uac00 \uc57d 937 AU\uc774\uace0 \uadfc\uc77c\uc810 \uac70\ub9ac\uac00 \uc57d 76 AU \uc774\ub2e4. \uc774 \uc6d0\uc77c\uc810 \uac70\ub9ac\ub294 2012\ub144 2012 VP113\uc774 \ubc1c\uacac\ub418\uae30 \uc804\uae4c\uc9c0 \ud0dc\uc591\uacc4 \ucc9c\uccb4 \uc911 \uac00\uc7a5 \uba3c \uac70\ub9ac\uc600\ub2e4. \uc138\ub4dc\ub098\uac00 \ubc1c\uacac\ub418\uc5c8\uc744 \ub2f9\uc2dc \ud0dc\uc591\uc5d0\uc11c\uc758 \uac70\ub9ac\ub294 89.6 AU\uc600\uc73c\uba70 \uadfc\uc77c\uc810\uc744 \ud5a5\ud574 \uc6c0\uc9c1\uc774\uace0 \uc788\uc5c8\ub2e4. \uc774\ud6c4 \uac19\uc740 \ud0d0\uc0ac \uacfc\uc815\uc5d0\uc11c \ud0dc\uc591\uc73c\ub85c\ubd80\ud130 97 AU \ub5a8\uc5b4\uc9c4 \uadfc\uc77c\uc810 \uadfc\ucc98\uc5d0\uc11c \uc5d0\ub9ac\uc2a4\uac00 \uac10\uc9c0\ub418\uc5c8\ub2e4. \uc138\ub4dc\ub098\ubcf4\ub2e4 \uada4\ub3c4\uac00 \ub354 \uba3c \ucc9c\uccb4\ub294 \uc77c\ubd80 \uc7a5\uc8fc\uae30 \ud61c\uc131\ub4e4\ubc16\uc5d0 \uc5c6\ub2e4. \uadf8\ub9ac\uace0 \ud61c\uc131\ub4e4\uc740 \ub108\ubb34 \uc5b4\ub450\uc6cc\uc11c \ub0b4\ubd80 \ud0dc\uc591\uacc4\ub85c \ub4e4\uc5b4\uc640 \uadfc\uc77c\uc810\uc744 \ud5a5\ud560 \ub54c\uac00 \uc544\ub2c8\uba74 \ubcf4\uc774\uc9c0\ub3c4 \uc54a\ub294\ub2e4. \uc138\ub4dc\ub098\ub3c4 2076\ub144 \uc911\uc21c\uc5d0 \uadfc\uc77c\uc810\uc5d0 \ub2e4\ub2e4\ub974\uaca0\uc9c0\ub9cc, \ub108\ubb34 \uba40\uc5b4\uc11c \uc721\uc548\uc73c\ub85c\ub294 \uc6d0\ubc18\uc0c1\uc744 \ubcfc \uc218 \uc5c6\uc744 \uac83\uc774\ub2e4. \uc138\ub4dc\ub098 \ud45c\uba74\uc5d0\uc11c \ud558\ub298\uc744 \ubcf4\uba74 \ud0dc\uc591\uc774 \uadf8\uc800 \ub9e4\uc6b0 \ubc1d\uc740(\ubcf4\ub984\ub2ec\uc758 100\ubc30 \ubc1d\uae30) \ud56d\uc131 \uc911 \ud558\ub098\ub85c \ubcf4\uc77c \uac83\uc774\ub2e4. \uc9c0\uad6c\uc5d0\uc11c \ubcf4\uc774\ub294 \ud0dc\uc591\uc758 \ubc1d\uae30\ub294 \ubcf4\ub984\ub2ec\uc758 400,000 \ubc30 \ubc1d\uae30\uc774\ubbc0\ub85c \uc774\ub97c \ub2a5\ud788 \ube44\uad50\ud560 \uc218 \uc788\ub2e4.", "paragraph_answer": "\uc138\ub4dc\ub098\ub294 \uc9c0\uae08\uae4c\uc9c0 \ud0dc\uc591\uacc4\uc5d0\uc11c \ubc1c\uacac\ub41c \ub300\ud615 \ucc9c\uccb4 \uc911 \uacf5\uc804 \uc8fc\uae30\uac00 \uac00\uc7a5 \uae38\uc5b4, 11,400 \ub144 \uc804\ud6c4 \ub85c \uacc4\uc0b0\ub41c\ub2e4. \uadf8 \uada4\ub3c4\ub294 \uc774\uc2ec\ub960\uc774 \uadf9\ub3c4\ub85c \ucee4\uc11c \uc6d0\uc77c\uc810 \uac70\ub9ac\uac00 \uc57d 937 AU\uc774\uace0 \uadfc\uc77c\uc810 \uac70\ub9ac\uac00 \uc57d 76 AU \uc774\ub2e4. \uc774 \uc6d0\uc77c\uc810 \uac70\ub9ac\ub294 2012\ub144 2012 VP113\uc774 \ubc1c\uacac\ub418\uae30 \uc804\uae4c\uc9c0 \ud0dc\uc591\uacc4 \ucc9c\uccb4 \uc911 \uac00\uc7a5 \uba3c \uac70\ub9ac\uc600\ub2e4. \uc138\ub4dc\ub098\uac00 \ubc1c\uacac\ub418\uc5c8\uc744 \ub2f9\uc2dc \ud0dc\uc591\uc5d0\uc11c\uc758 \uac70\ub9ac\ub294 89.6 AU\uc600\uc73c\uba70 \uadfc\uc77c\uc810\uc744 \ud5a5\ud574 \uc6c0\uc9c1\uc774\uace0 \uc788\uc5c8\ub2e4. \uc774\ud6c4 \uac19\uc740 \ud0d0\uc0ac \uacfc\uc815\uc5d0\uc11c \ud0dc\uc591\uc73c\ub85c\ubd80\ud130 97 AU \ub5a8\uc5b4\uc9c4 \uadfc\uc77c\uc810 \uadfc\ucc98\uc5d0\uc11c \uc5d0\ub9ac\uc2a4\uac00 \uac10\uc9c0\ub418\uc5c8\ub2e4. \uc138\ub4dc\ub098\ubcf4\ub2e4 \uada4\ub3c4\uac00 \ub354 \uba3c \ucc9c\uccb4\ub294 \uc77c\ubd80 \uc7a5\uc8fc\uae30 \ud61c\uc131\ub4e4\ubc16\uc5d0 \uc5c6\ub2e4. \uadf8\ub9ac\uace0 \ud61c\uc131\ub4e4\uc740 \ub108\ubb34 \uc5b4\ub450\uc6cc\uc11c \ub0b4\ubd80 \ud0dc\uc591\uacc4\ub85c \ub4e4\uc5b4\uc640 \uadfc\uc77c\uc810\uc744 \ud5a5\ud560 \ub54c\uac00 \uc544\ub2c8\uba74 \ubcf4\uc774\uc9c0\ub3c4 \uc54a\ub294\ub2e4. \uc138\ub4dc\ub098\ub3c4 2076\ub144 \uc911\uc21c\uc5d0 \uadfc\uc77c\uc810\uc5d0 \ub2e4\ub2e4\ub974\uaca0\uc9c0\ub9cc, \ub108\ubb34 \uba40\uc5b4\uc11c \uc721\uc548\uc73c\ub85c\ub294 \uc6d0\ubc18\uc0c1\uc744 \ubcfc \uc218 \uc5c6\uc744 \uac83\uc774\ub2e4. \uc138\ub4dc\ub098 \ud45c\uba74\uc5d0\uc11c \ud558\ub298\uc744 \ubcf4\uba74 \ud0dc\uc591\uc774 \uadf8\uc800 \ub9e4\uc6b0 \ubc1d\uc740(\ubcf4\ub984\ub2ec\uc758 100\ubc30 \ubc1d\uae30) \ud56d\uc131 \uc911 \ud558\ub098\ub85c \ubcf4\uc77c \uac83\uc774\ub2e4. \uc9c0\uad6c\uc5d0\uc11c \ubcf4\uc774\ub294 \ud0dc\uc591\uc758 \ubc1d\uae30\ub294 \ubcf4\ub984\ub2ec\uc758 400,000 \ubc30 \ubc1d\uae30\uc774\ubbc0\ub85c \uc774\ub97c \ub2a5\ud788 \ube44\uad50\ud560 \uc218 \uc788\ub2e4.", "sentence_answer": "\uc138\ub4dc\ub098\ub294 \uc9c0\uae08\uae4c\uc9c0 \ud0dc\uc591\uacc4\uc5d0\uc11c \ubc1c\uacac\ub41c \ub300\ud615 \ucc9c\uccb4 \uc911 \uacf5\uc804 \uc8fc\uae30\uac00 \uac00\uc7a5 \uae38\uc5b4, 11,400 \ub144 \uc804\ud6c4 \ub85c \uacc4\uc0b0\ub41c\ub2e4.", "paragraph_id": "6470281-3-0", "paragraph_question": "question: \ud0dc\uc591\uacc4\uc5d0\uc11c \ubc1c\uacac\ub41c \ub300\ud615 \ucc9c\uccb4 \uc911 \uac00\uc7a5 \uae34 \uc138\ub4dc\ub098\uc758 \uacf5\uc804 \uc8fc\uae30\ub294?, context: \uc138\ub4dc\ub098\ub294 \uc9c0\uae08\uae4c\uc9c0 \ud0dc\uc591\uacc4\uc5d0\uc11c \ubc1c\uacac\ub41c \ub300\ud615 \ucc9c\uccb4 \uc911 \uacf5\uc804 \uc8fc\uae30\uac00 \uac00\uc7a5 \uae38\uc5b4, 11,400 \ub144 \uc804\ud6c4\ub85c \uacc4\uc0b0\ub41c\ub2e4. \uadf8 \uada4\ub3c4\ub294 \uc774\uc2ec\ub960\uc774 \uadf9\ub3c4\ub85c \ucee4\uc11c \uc6d0\uc77c\uc810 \uac70\ub9ac\uac00 \uc57d 937 AU\uc774\uace0 \uadfc\uc77c\uc810 \uac70\ub9ac\uac00 \uc57d 76 AU \uc774\ub2e4. \uc774 \uc6d0\uc77c\uc810 \uac70\ub9ac\ub294 2012\ub144 2012 VP113\uc774 \ubc1c\uacac\ub418\uae30 \uc804\uae4c\uc9c0 \ud0dc\uc591\uacc4 \ucc9c\uccb4 \uc911 \uac00\uc7a5 \uba3c \uac70\ub9ac\uc600\ub2e4. \uc138\ub4dc\ub098\uac00 \ubc1c\uacac\ub418\uc5c8\uc744 \ub2f9\uc2dc \ud0dc\uc591\uc5d0\uc11c\uc758 \uac70\ub9ac\ub294 89.6 AU\uc600\uc73c\uba70 \uadfc\uc77c\uc810\uc744 \ud5a5\ud574 \uc6c0\uc9c1\uc774\uace0 \uc788\uc5c8\ub2e4. \uc774\ud6c4 \uac19\uc740 \ud0d0\uc0ac \uacfc\uc815\uc5d0\uc11c \ud0dc\uc591\uc73c\ub85c\ubd80\ud130 97 AU \ub5a8\uc5b4\uc9c4 \uadfc\uc77c\uc810 \uadfc\ucc98\uc5d0\uc11c \uc5d0\ub9ac\uc2a4\uac00 \uac10\uc9c0\ub418\uc5c8\ub2e4. \uc138\ub4dc\ub098\ubcf4\ub2e4 \uada4\ub3c4\uac00 \ub354 \uba3c \ucc9c\uccb4\ub294 \uc77c\ubd80 \uc7a5\uc8fc\uae30 \ud61c\uc131\ub4e4\ubc16\uc5d0 \uc5c6\ub2e4. \uadf8\ub9ac\uace0 \ud61c\uc131\ub4e4\uc740 \ub108\ubb34 \uc5b4\ub450\uc6cc\uc11c \ub0b4\ubd80 \ud0dc\uc591\uacc4\ub85c \ub4e4\uc5b4\uc640 \uadfc\uc77c\uc810\uc744 \ud5a5\ud560 \ub54c\uac00 \uc544\ub2c8\uba74 \ubcf4\uc774\uc9c0\ub3c4 \uc54a\ub294\ub2e4. \uc138\ub4dc\ub098\ub3c4 2076\ub144 \uc911\uc21c\uc5d0 \uadfc\uc77c\uc810\uc5d0 \ub2e4\ub2e4\ub974\uaca0\uc9c0\ub9cc, \ub108\ubb34 \uba40\uc5b4\uc11c \uc721\uc548\uc73c\ub85c\ub294 \uc6d0\ubc18\uc0c1\uc744 \ubcfc \uc218 \uc5c6\uc744 \uac83\uc774\ub2e4. \uc138\ub4dc\ub098 \ud45c\uba74\uc5d0\uc11c \ud558\ub298\uc744 \ubcf4\uba74 \ud0dc\uc591\uc774 \uadf8\uc800 \ub9e4\uc6b0 \ubc1d\uc740(\ubcf4\ub984\ub2ec\uc758 100\ubc30 \ubc1d\uae30) \ud56d\uc131 \uc911 \ud558\ub098\ub85c \ubcf4\uc77c \uac83\uc774\ub2e4. \uc9c0\uad6c\uc5d0\uc11c \ubcf4\uc774\ub294 \ud0dc\uc591\uc758 \ubc1d\uae30\ub294 \ubcf4\ub984\ub2ec\uc758 400,000 \ubc30 \ubc1d\uae30\uc774\ubbc0\ub85c \uc774\ub97c \ub2a5\ud788 \ube44\uad50\ud560 \uc218 \uc788\ub2e4."} {"question": "\ud504\ub791\ud06c\ud478\ub974\ud2b8 \uad6d\ubbfc\uc758\ud68c\uac00 \uc81c\uad6d\ud5cc\ubc95\uc744 \uac00\uacb0\ud55c \ub0a0\uc9dc\ub294?", "paragraph": "\uae38\uace0 \ub17c\ub780\uc774 \ub9ce\uc740 \ub17c\uc758 \ub05d\uc5d0 \uc758\ud68c\ub294 1849\ub144 3\uc6d4 28\uc77c \uc644\uc804\ud55c \uc81c\uad6d\ud5cc\ubc95\uc744 \uac00\uacb0\ud588\ub2e4. \uc774 \ud5cc\ubc95\uc740 \uc138\uc2b5\ub418\ub294 \uad6d\uac00\uc6d0\uc218\uc9c1\uc744 \uaddc\uc815\ud558\uc600\ub294\ub370, \uc774\ub294 \uac00\uac8c\ub978\uc744 \uc911\uc2ec\uc758 \uc18c\uc704 \u2018\uc138\uc2b5\uad70\uc8fc\uc81c\ud30c\u2019\uac00 \uc6d0\ub798\ub294 \uc138\uc2b5\uad70\uc8fc\uc81c\uc5d0 \ubc18\ub300\ud588\ub358 \ud558\uc778\ub9ac\ud788 \uc9c0\ubaac(August Heinrich Simon) \uc911\uc2ec\uc758 \ubca0\uc2a4\ud150\ud2b8\ud560 \ud30c\uc758 \uc9c0\uc6d0\uc744 \uc5bb\uc5b4 3\uc6d4 27\uc77c \ud22c\ud45c\uc5d0\uc11c 267 \ub300 263\ub85c \ub2e4\uc218\ub97c \ucc28\uc9c0\ud574 \uc774\uae34 \uacb0\uacfc\uc600\ub2e4. \uc774 \uc5f0\ud569\uc740 \ud5cc\ubc95 \ucd08\uc548\uc758 \uccab \ub3c5\ud68c\uac00 \uac01\ud558\ub428\uc5d0 \ub530\ub77c \ub098\uc628 \ud574\uacb0\ucc45\uc774\uc5c8\ub2e4. \uc5ec\ub860 \ubcc0\ud654\uc758 \uc8fc\ub41c \uc774\uc720\ub294, \ub300\uc548\uc73c\ub85c \uc81c\uc2dc\ub41c \uc120\ucd9c\uad70\uc8fc\uc81c\ub098 \uc784\uae30\uc81c \uc758\uc7a5\uc774 \uc788\ub294 \uc9d1\uc815\ubd80\uc81c\uc640 \uac19\uc740 \ub2e4\ub978 \ubaa8\ub4e0 \ubc29\uc548\uc774 \ud6e8\uc52c \uc2e4\ud604 \uac00\ub2a5\uc131\uc774 \uc5c6\uc5c8\uace0, \uae09\uc9c4 \uc88c\ud30c\uac00 \ubbf8\uad6d\uc744 \ubaa8\ub378\ub85c \uc694\uad6c\ud55c \uacf5\ud654\uad6d\ub9cc\ud07c\uc774\ub098 \ubcc4 \uc9c0\uc9c0\ub97c \ubc1b\uc9c0 \ubabb\ud588\uae30 \ub54c\ubb38\uc774\uc5c8\ub2e4.", "answer": "1849\ub144 3\uc6d4 28\uc77c", "sentence": "\uc758\ud68c\ub294 1849\ub144 3\uc6d4 28\uc77c \uc644\uc804\ud55c \uc81c\uad6d\ud5cc\ubc95\uc744 \uac00\uacb0\ud588\ub2e4.", "paragraph_sentence": "\uae38\uace0 \ub17c\ub780\uc774 \ub9ce\uc740 \ub17c\uc758 \ub05d\uc5d0 \uc758\ud68c\ub294 1849\ub144 3\uc6d4 28\uc77c \uc644\uc804\ud55c \uc81c\uad6d\ud5cc\ubc95\uc744 \uac00\uacb0\ud588\ub2e4. \uc774 \ud5cc\ubc95\uc740 \uc138\uc2b5\ub418\ub294 \uad6d\uac00\uc6d0\uc218\uc9c1\uc744 \uaddc\uc815\ud558\uc600\ub294\ub370, \uc774\ub294 \uac00\uac8c\ub978\uc744 \uc911\uc2ec\uc758 \uc18c\uc704 \u2018\uc138\uc2b5\uad70\uc8fc\uc81c\ud30c\u2019\uac00 \uc6d0\ub798\ub294 \uc138\uc2b5\uad70\uc8fc\uc81c\uc5d0 \ubc18\ub300\ud588\ub358 \ud558\uc778\ub9ac\ud788 \uc9c0\ubaac(August Heinrich Simon) \uc911\uc2ec\uc758 \ubca0\uc2a4\ud150\ud2b8\ud560 \ud30c\uc758 \uc9c0\uc6d0\uc744 \uc5bb\uc5b4 3\uc6d4 27\uc77c \ud22c\ud45c\uc5d0\uc11c 267 \ub300 263\ub85c \ub2e4\uc218\ub97c \ucc28\uc9c0\ud574 \uc774\uae34 \uacb0\uacfc\uc600\ub2e4. \uc774 \uc5f0\ud569\uc740 \ud5cc\ubc95 \ucd08\uc548\uc758 \uccab \ub3c5\ud68c\uac00 \uac01\ud558\ub428\uc5d0 \ub530\ub77c \ub098\uc628 \ud574\uacb0\ucc45\uc774\uc5c8\ub2e4. \uc5ec\ub860 \ubcc0\ud654\uc758 \uc8fc\ub41c \uc774\uc720\ub294, \ub300\uc548\uc73c\ub85c \uc81c\uc2dc\ub41c \uc120\ucd9c\uad70\uc8fc\uc81c\ub098 \uc784\uae30\uc81c \uc758\uc7a5\uc774 \uc788\ub294 \uc9d1\uc815\ubd80\uc81c\uc640 \uac19\uc740 \ub2e4\ub978 \ubaa8\ub4e0 \ubc29\uc548\uc774 \ud6e8\uc52c \uc2e4\ud604 \uac00\ub2a5\uc131\uc774 \uc5c6\uc5c8\uace0, \uae09\uc9c4 \uc88c\ud30c\uac00 \ubbf8\uad6d\uc744 \ubaa8\ub378\ub85c \uc694\uad6c\ud55c \uacf5\ud654\uad6d\ub9cc\ud07c\uc774\ub098 \ubcc4 \uc9c0\uc9c0\ub97c \ubc1b\uc9c0 \ubabb\ud588\uae30 \ub54c\ubb38\uc774\uc5c8\ub2e4.", "paragraph_answer": "\uae38\uace0 \ub17c\ub780\uc774 \ub9ce\uc740 \ub17c\uc758 \ub05d\uc5d0 \uc758\ud68c\ub294 1849\ub144 3\uc6d4 28\uc77c \uc644\uc804\ud55c \uc81c\uad6d\ud5cc\ubc95\uc744 \uac00\uacb0\ud588\ub2e4. \uc774 \ud5cc\ubc95\uc740 \uc138\uc2b5\ub418\ub294 \uad6d\uac00\uc6d0\uc218\uc9c1\uc744 \uaddc\uc815\ud558\uc600\ub294\ub370, \uc774\ub294 \uac00\uac8c\ub978\uc744 \uc911\uc2ec\uc758 \uc18c\uc704 \u2018\uc138\uc2b5\uad70\uc8fc\uc81c\ud30c\u2019\uac00 \uc6d0\ub798\ub294 \uc138\uc2b5\uad70\uc8fc\uc81c\uc5d0 \ubc18\ub300\ud588\ub358 \ud558\uc778\ub9ac\ud788 \uc9c0\ubaac(August Heinrich Simon) \uc911\uc2ec\uc758 \ubca0\uc2a4\ud150\ud2b8\ud560 \ud30c\uc758 \uc9c0\uc6d0\uc744 \uc5bb\uc5b4 3\uc6d4 27\uc77c \ud22c\ud45c\uc5d0\uc11c 267 \ub300 263\ub85c \ub2e4\uc218\ub97c \ucc28\uc9c0\ud574 \uc774\uae34 \uacb0\uacfc\uc600\ub2e4. \uc774 \uc5f0\ud569\uc740 \ud5cc\ubc95 \ucd08\uc548\uc758 \uccab \ub3c5\ud68c\uac00 \uac01\ud558\ub428\uc5d0 \ub530\ub77c \ub098\uc628 \ud574\uacb0\ucc45\uc774\uc5c8\ub2e4. \uc5ec\ub860 \ubcc0\ud654\uc758 \uc8fc\ub41c \uc774\uc720\ub294, \ub300\uc548\uc73c\ub85c \uc81c\uc2dc\ub41c \uc120\ucd9c\uad70\uc8fc\uc81c\ub098 \uc784\uae30\uc81c \uc758\uc7a5\uc774 \uc788\ub294 \uc9d1\uc815\ubd80\uc81c\uc640 \uac19\uc740 \ub2e4\ub978 \ubaa8\ub4e0 \ubc29\uc548\uc774 \ud6e8\uc52c \uc2e4\ud604 \uac00\ub2a5\uc131\uc774 \uc5c6\uc5c8\uace0, \uae09\uc9c4 \uc88c\ud30c\uac00 \ubbf8\uad6d\uc744 \ubaa8\ub378\ub85c \uc694\uad6c\ud55c \uacf5\ud654\uad6d\ub9cc\ud07c\uc774\ub098 \ubcc4 \uc9c0\uc9c0\ub97c \ubc1b\uc9c0 \ubabb\ud588\uae30 \ub54c\ubb38\uc774\uc5c8\ub2e4.", "sentence_answer": "\uc758\ud68c\ub294 1849\ub144 3\uc6d4 28\uc77c \uc644\uc804\ud55c \uc81c\uad6d\ud5cc\ubc95\uc744 \uac00\uacb0\ud588\ub2e4.", "paragraph_id": "6465718-24-0", "paragraph_question": "question: \ud504\ub791\ud06c\ud478\ub974\ud2b8 \uad6d\ubbfc\uc758\ud68c\uac00 \uc81c\uad6d\ud5cc\ubc95\uc744 \uac00\uacb0\ud55c \ub0a0\uc9dc\ub294?, context: \uae38\uace0 \ub17c\ub780\uc774 \ub9ce\uc740 \ub17c\uc758 \ub05d\uc5d0 \uc758\ud68c\ub294 1849\ub144 3\uc6d4 28\uc77c \uc644\uc804\ud55c \uc81c\uad6d\ud5cc\ubc95\uc744 \uac00\uacb0\ud588\ub2e4. \uc774 \ud5cc\ubc95\uc740 \uc138\uc2b5\ub418\ub294 \uad6d\uac00\uc6d0\uc218\uc9c1\uc744 \uaddc\uc815\ud558\uc600\ub294\ub370, \uc774\ub294 \uac00\uac8c\ub978\uc744 \uc911\uc2ec\uc758 \uc18c\uc704 \u2018\uc138\uc2b5\uad70\uc8fc\uc81c\ud30c\u2019\uac00 \uc6d0\ub798\ub294 \uc138\uc2b5\uad70\uc8fc\uc81c\uc5d0 \ubc18\ub300\ud588\ub358 \ud558\uc778\ub9ac\ud788 \uc9c0\ubaac(August Heinrich Simon) \uc911\uc2ec\uc758 \ubca0\uc2a4\ud150\ud2b8\ud560 \ud30c\uc758 \uc9c0\uc6d0\uc744 \uc5bb\uc5b4 3\uc6d4 27\uc77c \ud22c\ud45c\uc5d0\uc11c 267 \ub300 263\ub85c \ub2e4\uc218\ub97c \ucc28\uc9c0\ud574 \uc774\uae34 \uacb0\uacfc\uc600\ub2e4. \uc774 \uc5f0\ud569\uc740 \ud5cc\ubc95 \ucd08\uc548\uc758 \uccab \ub3c5\ud68c\uac00 \uac01\ud558\ub428\uc5d0 \ub530\ub77c \ub098\uc628 \ud574\uacb0\ucc45\uc774\uc5c8\ub2e4. \uc5ec\ub860 \ubcc0\ud654\uc758 \uc8fc\ub41c \uc774\uc720\ub294, \ub300\uc548\uc73c\ub85c \uc81c\uc2dc\ub41c \uc120\ucd9c\uad70\uc8fc\uc81c\ub098 \uc784\uae30\uc81c \uc758\uc7a5\uc774 \uc788\ub294 \uc9d1\uc815\ubd80\uc81c\uc640 \uac19\uc740 \ub2e4\ub978 \ubaa8\ub4e0 \ubc29\uc548\uc774 \ud6e8\uc52c \uc2e4\ud604 \uac00\ub2a5\uc131\uc774 \uc5c6\uc5c8\uace0, \uae09\uc9c4 \uc88c\ud30c\uac00 \ubbf8\uad6d\uc744 \ubaa8\ub378\ub85c \uc694\uad6c\ud55c \uacf5\ud654\uad6d\ub9cc\ud07c\uc774\ub098 \ubcc4 \uc9c0\uc9c0\ub97c \ubc1b\uc9c0 \ubabb\ud588\uae30 \ub54c\ubb38\uc774\uc5c8\ub2e4."} {"question": "1930\ub144 \uccab \ubc88\uc9f8 \uc6d4\ub4dc\ucef5\uc774 \uc5f4\ub9b0 \uad6d\uac00\ub294?", "paragraph": "\ub85c\uc2a4\uc564\uc824\ub808\uc2a4\uc5d0\uc11c \uc5f4\ub9b0 1932\ub144 \ud558\uacc4 \uc62c\ub9bc\ud53d\uc5d0\uc11c\ub294 \uac1c\ucd5c\uad6d\uc778 \ubbf8\uad6d\uc5d0\uc11c \ubbf8\uc2dd\ucd95\uad6c\uc758 \uc778\uae30 \ud0d3\uc5d0 \ucd95\uad6c\uc758 \uc778\uae30\uac00 \ub0ae\uc544 \uc815\uc2dd \uc885\ubaa9\uc73c\ub85c \ucc44\ud0dd\ub420 \uacc4\ud68d\uc774 \uc5c6\uc5c8\ub2e4. \ub610\ud55c, FIFA\uc640 IOC\uc758 \uc544\ub9c8\ucd94\uc5b4 \uc120\uc218\uc758 \uc9c0\uc704\uc5d0 \uad00\ud55c \uc758\uacac\uc774 \uc77c\uce58\ud558\uc9c0 \uc54a\uc558\uace0 \uc774\uc5d0 \ub530\ub77c \ucd95\uad6c\ub294 \uc62c\ub9bc\ud53d\uc5d0\uc11c \uc81c\uc678\ub418\uc5c8\ub2e4. \uc774\uc5b4 \ub2f9\uc2dc FIFA \ud68c\uc7a5\uc774\uc5c8\ub358 \uc958 \ub9ac\uba54\ub294 \uccab \ubc88\uc9f8 \uc6d4\ub4dc\ucef5 \ub300\ud68c\ub97c 1930\ub144 \uc6b0\ub8e8\uacfc\uc774\uc5d0\uc11c \uac1c\ucd5c\ud558\uae30\ub85c \ud588\ub2e4. \uac01 \uad6d\uac00\uc758 \ucd95\uad6c \ud611\ud68c\ub4e4\uc740 \ub300\ud68c \ucc38\uac00 \ucd08\uccad\uc744 \ubc1b\uc558\uc9c0\ub9cc, \uc720\ub7fd \uc9c0\uc5ed\uc758 \ud300\ub4e4\uc5d0\ub294 \uc6b0\ub8e8\uacfc\uc774\uac00 \ub300\uc11c\uc591\uc744 \ud6a1\ub2e8\ud574\uc57c \ud558\ub294 \uba3c \ub098\ub77c\uc600\uae30 \ub54c\ubb38\uc5d0, \ud070 \uc9c0\ucd9c\uacfc \uc624\ub79c \uc5ec\uc815\uc774 \ubd88\uac00\ud53c\ud588\ub2e4. \uadf8\ub7f0 \uc774\uc720\uc5d0 \ub530\ub77c \ub300\ud68c 2\ub2ec \uc804\uae4c\uc9c0\ub3c4 \ud300\uc744 \ubcf4\ub0b4\uaca0\ub2e4\ub294 \ud655\uc57d\uc744 \ud55c \uc720\ub7fd \uad6d\uac00\ub294 \uc5c6\uc5c8\ub2e4. \uacb0\uad6d, \uc958 \ub9ac\uba54\ub294 \uc720\ub7fd \ud300\ub4e4\uc744 \ud55c \ud300\uc774\ub77c\ub3c4 \uc124\ub4dd\ud558\uae30 \uc704\ud574 \ub178\ub825\ud588\uc73c\uba70 \uc6b0\ub8e8\uacfc\uc774\ub294 \ubaa8\ub4e0 \uccb4\ub958\ube44\uc640 \ucc28\ube44\ub97c \ucd9c\uc804\ud300\ub4e4\uc5d0\uac8c \uc81c\uacf5\ud558\uaca0\ub2e4\ub294 \uc870\uac74\uc744 \ub0b4\uac74 \ub05d\uc5d0 \uc5ec\ube44 \ubc0f \uccb4\ub958\ube44\uc6a9\uc774 \uacf5\uc9dc\ub77c\ub294 \uc810\uacfc \uc958 \ub9ac\uba54\uc758 \ub208\uce58\ub97c \ubd10\uc11c \ubca8\uae30\uc5d0, \ud504\ub791\uc2a4, \ub8e8\ub9c8\ub2c8\uc544, \uc720\uace0\uc2ac\ub77c\ube44\uc544\uac00 \ub300\ud68c \ucc38\uac00\uc5d0 \uc751\ud588\ub2e4. \uc774\ub9ac\ud558\uc5ec \uc720\ub7fd\uc5d0\uc11c 4\ud300, \ubd81\uc911\ubbf8\uc5d0\uc11c 2\ud300, \ub0a8\ubbf8\uc5d0\uc11c 7\ud300, \ucd1d 13\uac1c \ud300\uc774 \ub300\ud68c\uc5d0 \ucc38\uac00\ud558\uc600\ub2e4.", "answer": "\uc6b0\ub8e8\uacfc\uc774", "sentence": "\uc774\uc5b4 \ub2f9\uc2dc FIFA \ud68c\uc7a5\uc774\uc5c8\ub358 \uc958 \ub9ac\uba54\ub294 \uccab \ubc88\uc9f8 \uc6d4\ub4dc\ucef5 \ub300\ud68c\ub97c 1930\ub144 \uc6b0\ub8e8\uacfc\uc774 \uc5d0\uc11c \uac1c\ucd5c\ud558\uae30\ub85c \ud588\ub2e4.", "paragraph_sentence": "\ub85c\uc2a4\uc564\uc824\ub808\uc2a4\uc5d0\uc11c \uc5f4\ub9b0 1932\ub144 \ud558\uacc4 \uc62c\ub9bc\ud53d\uc5d0\uc11c\ub294 \uac1c\ucd5c\uad6d\uc778 \ubbf8\uad6d\uc5d0\uc11c \ubbf8\uc2dd\ucd95\uad6c\uc758 \uc778\uae30 \ud0d3\uc5d0 \ucd95\uad6c\uc758 \uc778\uae30\uac00 \ub0ae\uc544 \uc815\uc2dd \uc885\ubaa9\uc73c\ub85c \ucc44\ud0dd\ub420 \uacc4\ud68d\uc774 \uc5c6\uc5c8\ub2e4. \ub610\ud55c, FIFA\uc640 IOC\uc758 \uc544\ub9c8\ucd94\uc5b4 \uc120\uc218\uc758 \uc9c0\uc704\uc5d0 \uad00\ud55c \uc758\uacac\uc774 \uc77c\uce58\ud558\uc9c0 \uc54a\uc558\uace0 \uc774\uc5d0 \ub530\ub77c \ucd95\uad6c\ub294 \uc62c\ub9bc\ud53d\uc5d0\uc11c \uc81c\uc678\ub418\uc5c8\ub2e4. \uc774\uc5b4 \ub2f9\uc2dc FIFA \ud68c\uc7a5\uc774\uc5c8\ub358 \uc958 \ub9ac\uba54\ub294 \uccab \ubc88\uc9f8 \uc6d4\ub4dc\ucef5 \ub300\ud68c\ub97c 1930\ub144 \uc6b0\ub8e8\uacfc\uc774 \uc5d0\uc11c \uac1c\ucd5c\ud558\uae30\ub85c \ud588\ub2e4. \uac01 \uad6d\uac00\uc758 \ucd95\uad6c \ud611\ud68c\ub4e4\uc740 \ub300\ud68c \ucc38\uac00 \ucd08\uccad\uc744 \ubc1b\uc558\uc9c0\ub9cc, \uc720\ub7fd \uc9c0\uc5ed\uc758 \ud300\ub4e4\uc5d0\ub294 \uc6b0\ub8e8\uacfc\uc774\uac00 \ub300\uc11c\uc591\uc744 \ud6a1\ub2e8\ud574\uc57c \ud558\ub294 \uba3c \ub098\ub77c\uc600\uae30 \ub54c\ubb38\uc5d0, \ud070 \uc9c0\ucd9c\uacfc \uc624\ub79c \uc5ec\uc815\uc774 \ubd88\uac00\ud53c\ud588\ub2e4. \uadf8\ub7f0 \uc774\uc720\uc5d0 \ub530\ub77c \ub300\ud68c 2\ub2ec \uc804\uae4c\uc9c0\ub3c4 \ud300\uc744 \ubcf4\ub0b4\uaca0\ub2e4\ub294 \ud655\uc57d\uc744 \ud55c \uc720\ub7fd \uad6d\uac00\ub294 \uc5c6\uc5c8\ub2e4. \uacb0\uad6d, \uc958 \ub9ac\uba54\ub294 \uc720\ub7fd \ud300\ub4e4\uc744 \ud55c \ud300\uc774\ub77c\ub3c4 \uc124\ub4dd\ud558\uae30 \uc704\ud574 \ub178\ub825\ud588\uc73c\uba70 \uc6b0\ub8e8\uacfc\uc774\ub294 \ubaa8\ub4e0 \uccb4\ub958\ube44\uc640 \ucc28\ube44\ub97c \ucd9c\uc804\ud300\ub4e4\uc5d0\uac8c \uc81c\uacf5\ud558\uaca0\ub2e4\ub294 \uc870\uac74\uc744 \ub0b4\uac74 \ub05d\uc5d0 \uc5ec\ube44 \ubc0f \uccb4\ub958\ube44\uc6a9\uc774 \uacf5\uc9dc\ub77c\ub294 \uc810\uacfc \uc958 \ub9ac\uba54\uc758 \ub208\uce58\ub97c \ubd10\uc11c \ubca8\uae30\uc5d0, \ud504\ub791\uc2a4, \ub8e8\ub9c8\ub2c8\uc544, \uc720\uace0\uc2ac\ub77c\ube44\uc544\uac00 \ub300\ud68c \ucc38\uac00\uc5d0 \uc751\ud588\ub2e4. \uc774\ub9ac\ud558\uc5ec \uc720\ub7fd\uc5d0\uc11c 4\ud300, \ubd81\uc911\ubbf8\uc5d0\uc11c 2\ud300, \ub0a8\ubbf8\uc5d0\uc11c 7\ud300, \ucd1d 13\uac1c \ud300\uc774 \ub300\ud68c\uc5d0 \ucc38\uac00\ud558\uc600\ub2e4.", "paragraph_answer": "\ub85c\uc2a4\uc564\uc824\ub808\uc2a4\uc5d0\uc11c \uc5f4\ub9b0 1932\ub144 \ud558\uacc4 \uc62c\ub9bc\ud53d\uc5d0\uc11c\ub294 \uac1c\ucd5c\uad6d\uc778 \ubbf8\uad6d\uc5d0\uc11c \ubbf8\uc2dd\ucd95\uad6c\uc758 \uc778\uae30 \ud0d3\uc5d0 \ucd95\uad6c\uc758 \uc778\uae30\uac00 \ub0ae\uc544 \uc815\uc2dd \uc885\ubaa9\uc73c\ub85c \ucc44\ud0dd\ub420 \uacc4\ud68d\uc774 \uc5c6\uc5c8\ub2e4. \ub610\ud55c, FIFA\uc640 IOC\uc758 \uc544\ub9c8\ucd94\uc5b4 \uc120\uc218\uc758 \uc9c0\uc704\uc5d0 \uad00\ud55c \uc758\uacac\uc774 \uc77c\uce58\ud558\uc9c0 \uc54a\uc558\uace0 \uc774\uc5d0 \ub530\ub77c \ucd95\uad6c\ub294 \uc62c\ub9bc\ud53d\uc5d0\uc11c \uc81c\uc678\ub418\uc5c8\ub2e4. \uc774\uc5b4 \ub2f9\uc2dc FIFA \ud68c\uc7a5\uc774\uc5c8\ub358 \uc958 \ub9ac\uba54\ub294 \uccab \ubc88\uc9f8 \uc6d4\ub4dc\ucef5 \ub300\ud68c\ub97c 1930\ub144 \uc6b0\ub8e8\uacfc\uc774 \uc5d0\uc11c \uac1c\ucd5c\ud558\uae30\ub85c \ud588\ub2e4. \uac01 \uad6d\uac00\uc758 \ucd95\uad6c \ud611\ud68c\ub4e4\uc740 \ub300\ud68c \ucc38\uac00 \ucd08\uccad\uc744 \ubc1b\uc558\uc9c0\ub9cc, \uc720\ub7fd \uc9c0\uc5ed\uc758 \ud300\ub4e4\uc5d0\ub294 \uc6b0\ub8e8\uacfc\uc774\uac00 \ub300\uc11c\uc591\uc744 \ud6a1\ub2e8\ud574\uc57c \ud558\ub294 \uba3c \ub098\ub77c\uc600\uae30 \ub54c\ubb38\uc5d0, \ud070 \uc9c0\ucd9c\uacfc \uc624\ub79c \uc5ec\uc815\uc774 \ubd88\uac00\ud53c\ud588\ub2e4. \uadf8\ub7f0 \uc774\uc720\uc5d0 \ub530\ub77c \ub300\ud68c 2\ub2ec \uc804\uae4c\uc9c0\ub3c4 \ud300\uc744 \ubcf4\ub0b4\uaca0\ub2e4\ub294 \ud655\uc57d\uc744 \ud55c \uc720\ub7fd \uad6d\uac00\ub294 \uc5c6\uc5c8\ub2e4. \uacb0\uad6d, \uc958 \ub9ac\uba54\ub294 \uc720\ub7fd \ud300\ub4e4\uc744 \ud55c \ud300\uc774\ub77c\ub3c4 \uc124\ub4dd\ud558\uae30 \uc704\ud574 \ub178\ub825\ud588\uc73c\uba70 \uc6b0\ub8e8\uacfc\uc774\ub294 \ubaa8\ub4e0 \uccb4\ub958\ube44\uc640 \ucc28\ube44\ub97c \ucd9c\uc804\ud300\ub4e4\uc5d0\uac8c \uc81c\uacf5\ud558\uaca0\ub2e4\ub294 \uc870\uac74\uc744 \ub0b4\uac74 \ub05d\uc5d0 \uc5ec\ube44 \ubc0f \uccb4\ub958\ube44\uc6a9\uc774 \uacf5\uc9dc\ub77c\ub294 \uc810\uacfc \uc958 \ub9ac\uba54\uc758 \ub208\uce58\ub97c \ubd10\uc11c \ubca8\uae30\uc5d0, \ud504\ub791\uc2a4, \ub8e8\ub9c8\ub2c8\uc544, \uc720\uace0\uc2ac\ub77c\ube44\uc544\uac00 \ub300\ud68c \ucc38\uac00\uc5d0 \uc751\ud588\ub2e4. \uc774\ub9ac\ud558\uc5ec \uc720\ub7fd\uc5d0\uc11c 4\ud300, \ubd81\uc911\ubbf8\uc5d0\uc11c 2\ud300, \ub0a8\ubbf8\uc5d0\uc11c 7\ud300, \ucd1d 13\uac1c \ud300\uc774 \ub300\ud68c\uc5d0 \ucc38\uac00\ud558\uc600\ub2e4.", "sentence_answer": "\uc774\uc5b4 \ub2f9\uc2dc FIFA \ud68c\uc7a5\uc774\uc5c8\ub358 \uc958 \ub9ac\uba54\ub294 \uccab \ubc88\uc9f8 \uc6d4\ub4dc\ucef5 \ub300\ud68c\ub97c 1930\ub144 \uc6b0\ub8e8\uacfc\uc774 \uc5d0\uc11c \uac1c\ucd5c\ud558\uae30\ub85c \ud588\ub2e4.", "paragraph_id": "6475712-1-1", "paragraph_question": "question: 1930\ub144 \uccab \ubc88\uc9f8 \uc6d4\ub4dc\ucef5\uc774 \uc5f4\ub9b0 \uad6d\uac00\ub294?, context: \ub85c\uc2a4\uc564\uc824\ub808\uc2a4\uc5d0\uc11c \uc5f4\ub9b0 1932\ub144 \ud558\uacc4 \uc62c\ub9bc\ud53d\uc5d0\uc11c\ub294 \uac1c\ucd5c\uad6d\uc778 \ubbf8\uad6d\uc5d0\uc11c \ubbf8\uc2dd\ucd95\uad6c\uc758 \uc778\uae30 \ud0d3\uc5d0 \ucd95\uad6c\uc758 \uc778\uae30\uac00 \ub0ae\uc544 \uc815\uc2dd \uc885\ubaa9\uc73c\ub85c \ucc44\ud0dd\ub420 \uacc4\ud68d\uc774 \uc5c6\uc5c8\ub2e4. \ub610\ud55c, FIFA\uc640 IOC\uc758 \uc544\ub9c8\ucd94\uc5b4 \uc120\uc218\uc758 \uc9c0\uc704\uc5d0 \uad00\ud55c \uc758\uacac\uc774 \uc77c\uce58\ud558\uc9c0 \uc54a\uc558\uace0 \uc774\uc5d0 \ub530\ub77c \ucd95\uad6c\ub294 \uc62c\ub9bc\ud53d\uc5d0\uc11c \uc81c\uc678\ub418\uc5c8\ub2e4. \uc774\uc5b4 \ub2f9\uc2dc FIFA \ud68c\uc7a5\uc774\uc5c8\ub358 \uc958 \ub9ac\uba54\ub294 \uccab \ubc88\uc9f8 \uc6d4\ub4dc\ucef5 \ub300\ud68c\ub97c 1930\ub144 \uc6b0\ub8e8\uacfc\uc774\uc5d0\uc11c \uac1c\ucd5c\ud558\uae30\ub85c \ud588\ub2e4. \uac01 \uad6d\uac00\uc758 \ucd95\uad6c \ud611\ud68c\ub4e4\uc740 \ub300\ud68c \ucc38\uac00 \ucd08\uccad\uc744 \ubc1b\uc558\uc9c0\ub9cc, \uc720\ub7fd \uc9c0\uc5ed\uc758 \ud300\ub4e4\uc5d0\ub294 \uc6b0\ub8e8\uacfc\uc774\uac00 \ub300\uc11c\uc591\uc744 \ud6a1\ub2e8\ud574\uc57c \ud558\ub294 \uba3c \ub098\ub77c\uc600\uae30 \ub54c\ubb38\uc5d0, \ud070 \uc9c0\ucd9c\uacfc \uc624\ub79c \uc5ec\uc815\uc774 \ubd88\uac00\ud53c\ud588\ub2e4. \uadf8\ub7f0 \uc774\uc720\uc5d0 \ub530\ub77c \ub300\ud68c 2\ub2ec \uc804\uae4c\uc9c0\ub3c4 \ud300\uc744 \ubcf4\ub0b4\uaca0\ub2e4\ub294 \ud655\uc57d\uc744 \ud55c \uc720\ub7fd \uad6d\uac00\ub294 \uc5c6\uc5c8\ub2e4. \uacb0\uad6d, \uc958 \ub9ac\uba54\ub294 \uc720\ub7fd \ud300\ub4e4\uc744 \ud55c \ud300\uc774\ub77c\ub3c4 \uc124\ub4dd\ud558\uae30 \uc704\ud574 \ub178\ub825\ud588\uc73c\uba70 \uc6b0\ub8e8\uacfc\uc774\ub294 \ubaa8\ub4e0 \uccb4\ub958\ube44\uc640 \ucc28\ube44\ub97c \ucd9c\uc804\ud300\ub4e4\uc5d0\uac8c \uc81c\uacf5\ud558\uaca0\ub2e4\ub294 \uc870\uac74\uc744 \ub0b4\uac74 \ub05d\uc5d0 \uc5ec\ube44 \ubc0f \uccb4\ub958\ube44\uc6a9\uc774 \uacf5\uc9dc\ub77c\ub294 \uc810\uacfc \uc958 \ub9ac\uba54\uc758 \ub208\uce58\ub97c \ubd10\uc11c \ubca8\uae30\uc5d0, \ud504\ub791\uc2a4, \ub8e8\ub9c8\ub2c8\uc544, \uc720\uace0\uc2ac\ub77c\ube44\uc544\uac00 \ub300\ud68c \ucc38\uac00\uc5d0 \uc751\ud588\ub2e4. \uc774\ub9ac\ud558\uc5ec \uc720\ub7fd\uc5d0\uc11c 4\ud300, \ubd81\uc911\ubbf8\uc5d0\uc11c 2\ud300, \ub0a8\ubbf8\uc5d0\uc11c 7\ud300, \ucd1d 13\uac1c \ud300\uc774 \ub300\ud68c\uc5d0 \ucc38\uac00\ud558\uc600\ub2e4."} {"question": "\uc655\uc774 \uc7ac\ud63c\uc744 \ud558\uace0\uc2f6\uc744\ub54c \ub204\uad6c\uc758 \ud611\uc870\ub97c \uc5bb\uc5b4\uc57c \ud558\ub294\uac00?", "paragraph": "\ub85c\ud0c0\ub974 2\uc138\uc758 \uc774\ud63c \uc2e4\ud328 \uc0ac\uac74\uc740 \ub2e4\ub978 \uad70\uc8fc\ub4e4\uc5d0\uac8c\ub3c4 \uc601\ud5a5\uc744 \uc8fc\uc5c8\ub2e4. \ud569\ubc95\uc801\uc778 \uc544\ub0b4\uac00 \ud6c4\uacc4\uc790\ub97c \ub0b3\uc9c0 \ubabb\ud588\uc744 \ub54c \uc655\uc774 \uc7ac\ud63c\ud558\uace0 \uc2f6\ub2e4 \ud574\ub3c4 \uad50\ud68c\uc758 \ud611\uc870\ub97c \uc5bb\uc9c0 \ubabb\ud55c\ub2e4\uba74 \uacb0\uad6d \ud30c\uba78\uc744 \ubab0\uace0 \uc62c \uc871\uc1c4\uc5d0 \ubb36\uc77c \uc218 \uc788\uc74c\uc744 \ubcf4\uc5ec\uc8fc\uc5c8\ub2e4. \uadc0\uc871\uacfc \uc655\ub4e4\uc740 \ud544\uc694\ud558\ub2e4\uba74 \uacc4\uc18d \uad50\ud68c\uc5d0 \uc800\ud56d\ud588\uc9c0\ub9cc, \uc774\ud63c\uc744 \ub3c4\uc800\ud788 \ud53c\ud560 \uc218 \uc5c6\uc744 \ub54c\uc5d0\ub3c4 \uc77c\ub2e8\uc740 \uc774\ud63c \ubd88\uac00 \uc6d0\uce59\uc744 \ubc1b\uc544\ub4e4\uc774\uace0 \ub300\uc2e0 \uc880 \ub354 \uadf8\ub7f4 \ub4ef\ud55c \ud551\uacc4\ub97c \uc81c\uacf5\ud574\uc904 \uc218 \uc788\ub294 \uc0dd\ud65c \ubc29\uc2dd\uc744 \uc120\ud0dd\ud574 \uad50\ud68c\uc640\uc758 \uc9c1\uc811\uc801\uc778 \ub300\uacb0\uc744 \ud53c\ud558\ub294 \uacbd\uc6b0\uac00 \uc810\uc810 \ub354 \uc99d\uac00\ud558\uc600\ub2e4. \ub85c\ud0c0\ub974 2\uc138\uc758 \ud328\ubc30\ub294 \uc138\uc18d\uc758 \uad70\uc8fc\uac00 \uad50\ud669\ubcf4\ub2e4 \uc544\ub798\ub77c\ub294 \uba54\uc2dc\uc9c0\ub3c4 \ub418\uc5c8\ub2e4. \uadf8\ub7ec\ub098 \uad70\uc8fc\ub4e4\uacfc \uadc0\uc871\ub4e4\uc740 \ubc18\ubc1c\ud588\uc9c0\ub9cc \ub0b4\uc0c9\ud558\uc9c0 \uc54a\uc558\uace0, \ud6c4\ub300\uc5d0 \ud558\uc778\ub9ac\ud788 4\uc138\uc640 \uad50\ud669 \uadf8\ub808\uace0\ub9ac\uc6b0\uc2a4 7\uc138 \uac04\uc758 \uc815\uce58\uc801 \ub300\uacb0\ub85c \ud45c\uba74\ud654\ub418\uc5c8\ub2e4.", "answer": "\uad50\ud68c", "sentence": "\ud569\ubc95\uc801\uc778 \uc544\ub0b4\uac00 \ud6c4\uacc4\uc790\ub97c \ub0b3\uc9c0 \ubabb\ud588\uc744 \ub54c \uc655\uc774 \uc7ac\ud63c\ud558\uace0 \uc2f6\ub2e4 \ud574\ub3c4 \uad50\ud68c \uc758 \ud611\uc870\ub97c \uc5bb\uc9c0 \ubabb\ud55c\ub2e4\uba74 \uacb0\uad6d \ud30c\uba78\uc744 \ubab0\uace0 \uc62c \uc871\uc1c4\uc5d0 \ubb36\uc77c \uc218 \uc788\uc74c\uc744 \ubcf4\uc5ec\uc8fc\uc5c8\ub2e4.", "paragraph_sentence": "\ub85c\ud0c0\ub974 2\uc138\uc758 \uc774\ud63c \uc2e4\ud328 \uc0ac\uac74\uc740 \ub2e4\ub978 \uad70\uc8fc\ub4e4\uc5d0\uac8c\ub3c4 \uc601\ud5a5\uc744 \uc8fc\uc5c8\ub2e4. \ud569\ubc95\uc801\uc778 \uc544\ub0b4\uac00 \ud6c4\uacc4\uc790\ub97c \ub0b3\uc9c0 \ubabb\ud588\uc744 \ub54c \uc655\uc774 \uc7ac\ud63c\ud558\uace0 \uc2f6\ub2e4 \ud574\ub3c4 \uad50\ud68c \uc758 \ud611\uc870\ub97c \uc5bb\uc9c0 \ubabb\ud55c\ub2e4\uba74 \uacb0\uad6d \ud30c\uba78\uc744 \ubab0\uace0 \uc62c \uc871\uc1c4\uc5d0 \ubb36\uc77c \uc218 \uc788\uc74c\uc744 \ubcf4\uc5ec\uc8fc\uc5c8\ub2e4. \uadc0\uc871\uacfc \uc655\ub4e4\uc740 \ud544\uc694\ud558\ub2e4\uba74 \uacc4\uc18d \uad50\ud68c\uc5d0 \uc800\ud56d\ud588\uc9c0\ub9cc, \uc774\ud63c\uc744 \ub3c4\uc800\ud788 \ud53c\ud560 \uc218 \uc5c6\uc744 \ub54c\uc5d0\ub3c4 \uc77c\ub2e8\uc740 \uc774\ud63c \ubd88\uac00 \uc6d0\uce59\uc744 \ubc1b\uc544\ub4e4\uc774\uace0 \ub300\uc2e0 \uc880 \ub354 \uadf8\ub7f4 \ub4ef\ud55c \ud551\uacc4\ub97c \uc81c\uacf5\ud574\uc904 \uc218 \uc788\ub294 \uc0dd\ud65c \ubc29\uc2dd\uc744 \uc120\ud0dd\ud574 \uad50\ud68c\uc640\uc758 \uc9c1\uc811\uc801\uc778 \ub300\uacb0\uc744 \ud53c\ud558\ub294 \uacbd\uc6b0\uac00 \uc810\uc810 \ub354 \uc99d\uac00\ud558\uc600\ub2e4. \ub85c\ud0c0\ub974 2\uc138\uc758 \ud328\ubc30\ub294 \uc138\uc18d\uc758 \uad70\uc8fc\uac00 \uad50\ud669\ubcf4\ub2e4 \uc544\ub798\ub77c\ub294 \uba54\uc2dc\uc9c0\ub3c4 \ub418\uc5c8\ub2e4. \uadf8\ub7ec\ub098 \uad70\uc8fc\ub4e4\uacfc \uadc0\uc871\ub4e4\uc740 \ubc18\ubc1c\ud588\uc9c0\ub9cc \ub0b4\uc0c9\ud558\uc9c0 \uc54a\uc558\uace0, \ud6c4\ub300\uc5d0 \ud558\uc778\ub9ac\ud788 4\uc138\uc640 \uad50\ud669 \uadf8\ub808\uace0\ub9ac\uc6b0\uc2a4 7\uc138 \uac04\uc758 \uc815\uce58\uc801 \ub300\uacb0\ub85c \ud45c\uba74\ud654\ub418\uc5c8\ub2e4.", "paragraph_answer": "\ub85c\ud0c0\ub974 2\uc138\uc758 \uc774\ud63c \uc2e4\ud328 \uc0ac\uac74\uc740 \ub2e4\ub978 \uad70\uc8fc\ub4e4\uc5d0\uac8c\ub3c4 \uc601\ud5a5\uc744 \uc8fc\uc5c8\ub2e4. \ud569\ubc95\uc801\uc778 \uc544\ub0b4\uac00 \ud6c4\uacc4\uc790\ub97c \ub0b3\uc9c0 \ubabb\ud588\uc744 \ub54c \uc655\uc774 \uc7ac\ud63c\ud558\uace0 \uc2f6\ub2e4 \ud574\ub3c4 \uad50\ud68c \uc758 \ud611\uc870\ub97c \uc5bb\uc9c0 \ubabb\ud55c\ub2e4\uba74 \uacb0\uad6d \ud30c\uba78\uc744 \ubab0\uace0 \uc62c \uc871\uc1c4\uc5d0 \ubb36\uc77c \uc218 \uc788\uc74c\uc744 \ubcf4\uc5ec\uc8fc\uc5c8\ub2e4. \uadc0\uc871\uacfc \uc655\ub4e4\uc740 \ud544\uc694\ud558\ub2e4\uba74 \uacc4\uc18d \uad50\ud68c\uc5d0 \uc800\ud56d\ud588\uc9c0\ub9cc, \uc774\ud63c\uc744 \ub3c4\uc800\ud788 \ud53c\ud560 \uc218 \uc5c6\uc744 \ub54c\uc5d0\ub3c4 \uc77c\ub2e8\uc740 \uc774\ud63c \ubd88\uac00 \uc6d0\uce59\uc744 \ubc1b\uc544\ub4e4\uc774\uace0 \ub300\uc2e0 \uc880 \ub354 \uadf8\ub7f4 \ub4ef\ud55c \ud551\uacc4\ub97c \uc81c\uacf5\ud574\uc904 \uc218 \uc788\ub294 \uc0dd\ud65c \ubc29\uc2dd\uc744 \uc120\ud0dd\ud574 \uad50\ud68c\uc640\uc758 \uc9c1\uc811\uc801\uc778 \ub300\uacb0\uc744 \ud53c\ud558\ub294 \uacbd\uc6b0\uac00 \uc810\uc810 \ub354 \uc99d\uac00\ud558\uc600\ub2e4. \ub85c\ud0c0\ub974 2\uc138\uc758 \ud328\ubc30\ub294 \uc138\uc18d\uc758 \uad70\uc8fc\uac00 \uad50\ud669\ubcf4\ub2e4 \uc544\ub798\ub77c\ub294 \uba54\uc2dc\uc9c0\ub3c4 \ub418\uc5c8\ub2e4. \uadf8\ub7ec\ub098 \uad70\uc8fc\ub4e4\uacfc \uadc0\uc871\ub4e4\uc740 \ubc18\ubc1c\ud588\uc9c0\ub9cc \ub0b4\uc0c9\ud558\uc9c0 \uc54a\uc558\uace0, \ud6c4\ub300\uc5d0 \ud558\uc778\ub9ac\ud788 4\uc138\uc640 \uad50\ud669 \uadf8\ub808\uace0\ub9ac\uc6b0\uc2a4 7\uc138 \uac04\uc758 \uc815\uce58\uc801 \ub300\uacb0\ub85c \ud45c\uba74\ud654\ub418\uc5c8\ub2e4.", "sentence_answer": "\ud569\ubc95\uc801\uc778 \uc544\ub0b4\uac00 \ud6c4\uacc4\uc790\ub97c \ub0b3\uc9c0 \ubabb\ud588\uc744 \ub54c \uc655\uc774 \uc7ac\ud63c\ud558\uace0 \uc2f6\ub2e4 \ud574\ub3c4 \uad50\ud68c \uc758 \ud611\uc870\ub97c \uc5bb\uc9c0 \ubabb\ud55c\ub2e4\uba74 \uacb0\uad6d \ud30c\uba78\uc744 \ubab0\uace0 \uc62c \uc871\uc1c4\uc5d0 \ubb36\uc77c \uc218 \uc788\uc74c\uc744 \ubcf4\uc5ec\uc8fc\uc5c8\ub2e4.", "paragraph_id": "6579715-9-0", "paragraph_question": "question: \uc655\uc774 \uc7ac\ud63c\uc744 \ud558\uace0\uc2f6\uc744\ub54c \ub204\uad6c\uc758 \ud611\uc870\ub97c \uc5bb\uc5b4\uc57c \ud558\ub294\uac00?, context: \ub85c\ud0c0\ub974 2\uc138\uc758 \uc774\ud63c \uc2e4\ud328 \uc0ac\uac74\uc740 \ub2e4\ub978 \uad70\uc8fc\ub4e4\uc5d0\uac8c\ub3c4 \uc601\ud5a5\uc744 \uc8fc\uc5c8\ub2e4. \ud569\ubc95\uc801\uc778 \uc544\ub0b4\uac00 \ud6c4\uacc4\uc790\ub97c \ub0b3\uc9c0 \ubabb\ud588\uc744 \ub54c \uc655\uc774 \uc7ac\ud63c\ud558\uace0 \uc2f6\ub2e4 \ud574\ub3c4 \uad50\ud68c\uc758 \ud611\uc870\ub97c \uc5bb\uc9c0 \ubabb\ud55c\ub2e4\uba74 \uacb0\uad6d \ud30c\uba78\uc744 \ubab0\uace0 \uc62c \uc871\uc1c4\uc5d0 \ubb36\uc77c \uc218 \uc788\uc74c\uc744 \ubcf4\uc5ec\uc8fc\uc5c8\ub2e4. \uadc0\uc871\uacfc \uc655\ub4e4\uc740 \ud544\uc694\ud558\ub2e4\uba74 \uacc4\uc18d \uad50\ud68c\uc5d0 \uc800\ud56d\ud588\uc9c0\ub9cc, \uc774\ud63c\uc744 \ub3c4\uc800\ud788 \ud53c\ud560 \uc218 \uc5c6\uc744 \ub54c\uc5d0\ub3c4 \uc77c\ub2e8\uc740 \uc774\ud63c \ubd88\uac00 \uc6d0\uce59\uc744 \ubc1b\uc544\ub4e4\uc774\uace0 \ub300\uc2e0 \uc880 \ub354 \uadf8\ub7f4 \ub4ef\ud55c \ud551\uacc4\ub97c \uc81c\uacf5\ud574\uc904 \uc218 \uc788\ub294 \uc0dd\ud65c \ubc29\uc2dd\uc744 \uc120\ud0dd\ud574 \uad50\ud68c\uc640\uc758 \uc9c1\uc811\uc801\uc778 \ub300\uacb0\uc744 \ud53c\ud558\ub294 \uacbd\uc6b0\uac00 \uc810\uc810 \ub354 \uc99d\uac00\ud558\uc600\ub2e4. \ub85c\ud0c0\ub974 2\uc138\uc758 \ud328\ubc30\ub294 \uc138\uc18d\uc758 \uad70\uc8fc\uac00 \uad50\ud669\ubcf4\ub2e4 \uc544\ub798\ub77c\ub294 \uba54\uc2dc\uc9c0\ub3c4 \ub418\uc5c8\ub2e4. \uadf8\ub7ec\ub098 \uad70\uc8fc\ub4e4\uacfc \uadc0\uc871\ub4e4\uc740 \ubc18\ubc1c\ud588\uc9c0\ub9cc \ub0b4\uc0c9\ud558\uc9c0 \uc54a\uc558\uace0, \ud6c4\ub300\uc5d0 \ud558\uc778\ub9ac\ud788 4\uc138\uc640 \uad50\ud669 \uadf8\ub808\uace0\ub9ac\uc6b0\uc2a4 7\uc138 \uac04\uc758 \uc815\uce58\uc801 \ub300\uacb0\ub85c \ud45c\uba74\ud654\ub418\uc5c8\ub2e4."} {"question": "\ubd80\uc5ec\ub294 \uae30\ub150\ubb3c \uba87 \uc810\uc744 \ubcf4\uc720\ud558\uace0 \uc788\ub294\uac00?", "paragraph": "\ubd80\uc5ec\ub294 \ubc31\uc81c\uc758 \ub9c8\uc9c0\ub9c9 \uc218\ub3c4\ub85c\uc11c \uc774\uc640 \uad00\ub828\ud55c \uc5ed\uc0ac \ud0d0\ubc29\uc774 \ubb38\ud654\uad00\uad11\uc758 \uc8fc\ub41c \uc790\uc6d0\uc774\ub2e4. \uc8fc\uc694 \uc0ac\uc801\uc73c\ub85c\ub294 \ubd80\uc5ec\uad70 \ubd80\uc5ec\uc74d \ub3d9\ub0a8\ub9ac\uc5d0 \uc788\ub294 \ubd80\uc5ec\uc815\ub9bc\uc0ac\uc9c0\uc624\uce35\uc11d\ud0d1\uacfc \ub2a5\uc0b0\ub9ac\uace0\ubd84\uad70, \uad81\ub0a8\uc9c0, \ubd80\uc18c\uc0b0\uc131, \ub099\ud654\uc554, \uace0\ub780\uc0ac, \ubb34\ub7c9\uc0ac \ub4f1\uc774 \uc788\ub2e4. \ub300\ud45c\uc801 \uc720\ubb3c\ub85c\ub294 \uad6d\ubcf4\ub85c \uc9c0\uc815\ub41c \ubc31\uc81c\uae08\ub3d9\ub300\ud5a5\ub85c, \ubd80\uc5ec\uc815\ub9bc\uc0ac\uc9c0\uc624\uce35\uc11d\ud0d1 \ub4f1\uc774 \uc788\ub2e4. \uad6d\uac00\uc9c0\uc815\ubb38\ud654\uc7ac\uc5d0\ub294 \uad6d\ubcf4 4\uc810, \ubcf4\ubb3c 16\uc810, \uc0ac\uc801 21\uacf3, \ubb34\ud615\ubb38\ud654\uc7ac 1\uc810, \ubbfc\uc18d\uc790\ub8cc 4\uc810, \ucc9c\uc5f0\uae30\ub150\ubb3c 1\uc810, \uc0ac\uc801 \ubc0f \uba85\uc2b9 1\uc810\uc744 \ubcf4\uc720\ud558\uace0 \uc788\uc73c\uba70, \ub3c4\uc9c0\uc815\ubb38\ud654\uc7ac\ub85c \uc720\ud615\ubb38\ud654\uc7ac 25\uc810, \uae30\ub150\ubb3c 25\uc810, \ubb34\ud615\ubb38\ud654\uc7ac 5\uc810\uc774 \uc788\ub2e4. \uadf8 \uc678\uc5d0\ub3c4 \ub3c4\ubb38\ud654\uc7ac\uc790\ub8cc 40\uc810, \uad70\ud5a5\ud1a0\uc720\uc87177\uc810\uc73c\ub85c \ub9ce\uc740 \ubb38\ud654\uc7ac\uac00 \uc788\ub2e4. \ub9e4\ud574 \uacf5\uc8fc\uc2dc\uc640 \ud569\ub3d9\uc73c\ub85c 10\uc6d4\uc5d0 \uc885\ud569\uc608\uc220\uc81c\uc778 \ubc31\uc81c\ubb38\ud654\uc81c\ub97c \uac1c\ucd5c\ud55c\ub2e4.", "answer": "25\uc810", "sentence": "\uad6d\uac00\uc9c0\uc815\ubb38\ud654\uc7ac\uc5d0\ub294 \uad6d\ubcf4 4\uc810, \ubcf4\ubb3c 16\uc810, \uc0ac\uc801 21\uacf3, \ubb34\ud615\ubb38\ud654\uc7ac 1\uc810, \ubbfc\uc18d\uc790\ub8cc 4\uc810, \ucc9c\uc5f0\uae30\ub150\ubb3c 1\uc810, \uc0ac\uc801 \ubc0f \uba85\uc2b9 1\uc810\uc744 \ubcf4\uc720\ud558\uace0 \uc788\uc73c\uba70, \ub3c4\uc9c0\uc815\ubb38\ud654\uc7ac\ub85c \uc720\ud615\ubb38\ud654\uc7ac 25\uc810 ,", "paragraph_sentence": "\ubd80\uc5ec\ub294 \ubc31\uc81c\uc758 \ub9c8\uc9c0\ub9c9 \uc218\ub3c4\ub85c\uc11c \uc774\uc640 \uad00\ub828\ud55c \uc5ed\uc0ac \ud0d0\ubc29\uc774 \ubb38\ud654\uad00\uad11\uc758 \uc8fc\ub41c \uc790\uc6d0\uc774\ub2e4. \uc8fc\uc694 \uc0ac\uc801\uc73c\ub85c\ub294 \ubd80\uc5ec\uad70 \ubd80\uc5ec\uc74d \ub3d9\ub0a8\ub9ac\uc5d0 \uc788\ub294 \ubd80\uc5ec\uc815\ub9bc\uc0ac\uc9c0\uc624\uce35\uc11d\ud0d1\uacfc \ub2a5\uc0b0\ub9ac\uace0\ubd84\uad70, \uad81\ub0a8\uc9c0, \ubd80\uc18c\uc0b0\uc131, \ub099\ud654\uc554, \uace0\ub780\uc0ac, \ubb34\ub7c9\uc0ac \ub4f1\uc774 \uc788\ub2e4. \ub300\ud45c\uc801 \uc720\ubb3c\ub85c\ub294 \uad6d\ubcf4\ub85c \uc9c0\uc815\ub41c \ubc31\uc81c\uae08\ub3d9\ub300\ud5a5\ub85c, \ubd80\uc5ec\uc815\ub9bc\uc0ac\uc9c0\uc624\uce35\uc11d\ud0d1 \ub4f1\uc774 \uc788\ub2e4. \uad6d\uac00\uc9c0\uc815\ubb38\ud654\uc7ac\uc5d0\ub294 \uad6d\ubcf4 4\uc810, \ubcf4\ubb3c 16\uc810, \uc0ac\uc801 21\uacf3, \ubb34\ud615\ubb38\ud654\uc7ac 1\uc810, \ubbfc\uc18d\uc790\ub8cc 4\uc810, \ucc9c\uc5f0\uae30\ub150\ubb3c 1\uc810, \uc0ac\uc801 \ubc0f \uba85\uc2b9 1\uc810\uc744 \ubcf4\uc720\ud558\uace0 \uc788\uc73c\uba70, \ub3c4\uc9c0\uc815\ubb38\ud654\uc7ac\ub85c \uc720\ud615\ubb38\ud654\uc7ac 25\uc810 , \uae30\ub150\ubb3c 25\uc810, \ubb34\ud615\ubb38\ud654\uc7ac 5\uc810\uc774 \uc788\ub2e4. \uadf8 \uc678\uc5d0\ub3c4 \ub3c4\ubb38\ud654\uc7ac\uc790\ub8cc 40\uc810, \uad70\ud5a5\ud1a0\uc720\uc87177\uc810\uc73c\ub85c \ub9ce\uc740 \ubb38\ud654\uc7ac\uac00 \uc788\ub2e4. \ub9e4\ud574 \uacf5\uc8fc\uc2dc\uc640 \ud569\ub3d9\uc73c\ub85c 10\uc6d4\uc5d0 \uc885\ud569\uc608\uc220\uc81c\uc778 \ubc31\uc81c\ubb38\ud654\uc81c\ub97c \uac1c\ucd5c\ud55c\ub2e4.", "paragraph_answer": "\ubd80\uc5ec\ub294 \ubc31\uc81c\uc758 \ub9c8\uc9c0\ub9c9 \uc218\ub3c4\ub85c\uc11c \uc774\uc640 \uad00\ub828\ud55c \uc5ed\uc0ac \ud0d0\ubc29\uc774 \ubb38\ud654\uad00\uad11\uc758 \uc8fc\ub41c \uc790\uc6d0\uc774\ub2e4. \uc8fc\uc694 \uc0ac\uc801\uc73c\ub85c\ub294 \ubd80\uc5ec\uad70 \ubd80\uc5ec\uc74d \ub3d9\ub0a8\ub9ac\uc5d0 \uc788\ub294 \ubd80\uc5ec\uc815\ub9bc\uc0ac\uc9c0\uc624\uce35\uc11d\ud0d1\uacfc \ub2a5\uc0b0\ub9ac\uace0\ubd84\uad70, \uad81\ub0a8\uc9c0, \ubd80\uc18c\uc0b0\uc131, \ub099\ud654\uc554, \uace0\ub780\uc0ac, \ubb34\ub7c9\uc0ac \ub4f1\uc774 \uc788\ub2e4. \ub300\ud45c\uc801 \uc720\ubb3c\ub85c\ub294 \uad6d\ubcf4\ub85c \uc9c0\uc815\ub41c \ubc31\uc81c\uae08\ub3d9\ub300\ud5a5\ub85c, \ubd80\uc5ec\uc815\ub9bc\uc0ac\uc9c0\uc624\uce35\uc11d\ud0d1 \ub4f1\uc774 \uc788\ub2e4. \uad6d\uac00\uc9c0\uc815\ubb38\ud654\uc7ac\uc5d0\ub294 \uad6d\ubcf4 4\uc810, \ubcf4\ubb3c 16\uc810, \uc0ac\uc801 21\uacf3, \ubb34\ud615\ubb38\ud654\uc7ac 1\uc810, \ubbfc\uc18d\uc790\ub8cc 4\uc810, \ucc9c\uc5f0\uae30\ub150\ubb3c 1\uc810, \uc0ac\uc801 \ubc0f \uba85\uc2b9 1\uc810\uc744 \ubcf4\uc720\ud558\uace0 \uc788\uc73c\uba70, \ub3c4\uc9c0\uc815\ubb38\ud654\uc7ac\ub85c \uc720\ud615\ubb38\ud654\uc7ac 25\uc810 , \uae30\ub150\ubb3c 25\uc810, \ubb34\ud615\ubb38\ud654\uc7ac 5\uc810\uc774 \uc788\ub2e4. \uadf8 \uc678\uc5d0\ub3c4 \ub3c4\ubb38\ud654\uc7ac\uc790\ub8cc 40\uc810, \uad70\ud5a5\ud1a0\uc720\uc87177\uc810\uc73c\ub85c \ub9ce\uc740 \ubb38\ud654\uc7ac\uac00 \uc788\ub2e4. \ub9e4\ud574 \uacf5\uc8fc\uc2dc\uc640 \ud569\ub3d9\uc73c\ub85c 10\uc6d4\uc5d0 \uc885\ud569\uc608\uc220\uc81c\uc778 \ubc31\uc81c\ubb38\ud654\uc81c\ub97c \uac1c\ucd5c\ud55c\ub2e4.", "sentence_answer": "\uad6d\uac00\uc9c0\uc815\ubb38\ud654\uc7ac\uc5d0\ub294 \uad6d\ubcf4 4\uc810, \ubcf4\ubb3c 16\uc810, \uc0ac\uc801 21\uacf3, \ubb34\ud615\ubb38\ud654\uc7ac 1\uc810, \ubbfc\uc18d\uc790\ub8cc 4\uc810, \ucc9c\uc5f0\uae30\ub150\ubb3c 1\uc810, \uc0ac\uc801 \ubc0f \uba85\uc2b9 1\uc810\uc744 \ubcf4\uc720\ud558\uace0 \uc788\uc73c\uba70, \ub3c4\uc9c0\uc815\ubb38\ud654\uc7ac\ub85c \uc720\ud615\ubb38\ud654\uc7ac 25\uc810 ,", "paragraph_id": "6117753-2-2", "paragraph_question": "question: \ubd80\uc5ec\ub294 \uae30\ub150\ubb3c \uba87 \uc810\uc744 \ubcf4\uc720\ud558\uace0 \uc788\ub294\uac00?, context: \ubd80\uc5ec\ub294 \ubc31\uc81c\uc758 \ub9c8\uc9c0\ub9c9 \uc218\ub3c4\ub85c\uc11c \uc774\uc640 \uad00\ub828\ud55c \uc5ed\uc0ac \ud0d0\ubc29\uc774 \ubb38\ud654\uad00\uad11\uc758 \uc8fc\ub41c \uc790\uc6d0\uc774\ub2e4. \uc8fc\uc694 \uc0ac\uc801\uc73c\ub85c\ub294 \ubd80\uc5ec\uad70 \ubd80\uc5ec\uc74d \ub3d9\ub0a8\ub9ac\uc5d0 \uc788\ub294 \ubd80\uc5ec\uc815\ub9bc\uc0ac\uc9c0\uc624\uce35\uc11d\ud0d1\uacfc \ub2a5\uc0b0\ub9ac\uace0\ubd84\uad70, \uad81\ub0a8\uc9c0, \ubd80\uc18c\uc0b0\uc131, \ub099\ud654\uc554, \uace0\ub780\uc0ac, \ubb34\ub7c9\uc0ac \ub4f1\uc774 \uc788\ub2e4. \ub300\ud45c\uc801 \uc720\ubb3c\ub85c\ub294 \uad6d\ubcf4\ub85c \uc9c0\uc815\ub41c \ubc31\uc81c\uae08\ub3d9\ub300\ud5a5\ub85c, \ubd80\uc5ec\uc815\ub9bc\uc0ac\uc9c0\uc624\uce35\uc11d\ud0d1 \ub4f1\uc774 \uc788\ub2e4. \uad6d\uac00\uc9c0\uc815\ubb38\ud654\uc7ac\uc5d0\ub294 \uad6d\ubcf4 4\uc810, \ubcf4\ubb3c 16\uc810, \uc0ac\uc801 21\uacf3, \ubb34\ud615\ubb38\ud654\uc7ac 1\uc810, \ubbfc\uc18d\uc790\ub8cc 4\uc810, \ucc9c\uc5f0\uae30\ub150\ubb3c 1\uc810, \uc0ac\uc801 \ubc0f \uba85\uc2b9 1\uc810\uc744 \ubcf4\uc720\ud558\uace0 \uc788\uc73c\uba70, \ub3c4\uc9c0\uc815\ubb38\ud654\uc7ac\ub85c \uc720\ud615\ubb38\ud654\uc7ac 25\uc810, \uae30\ub150\ubb3c 25\uc810, \ubb34\ud615\ubb38\ud654\uc7ac 5\uc810\uc774 \uc788\ub2e4. \uadf8 \uc678\uc5d0\ub3c4 \ub3c4\ubb38\ud654\uc7ac\uc790\ub8cc 40\uc810, \uad70\ud5a5\ud1a0\uc720\uc87177\uc810\uc73c\ub85c \ub9ce\uc740 \ubb38\ud654\uc7ac\uac00 \uc788\ub2e4. \ub9e4\ud574 \uacf5\uc8fc\uc2dc\uc640 \ud569\ub3d9\uc73c\ub85c 10\uc6d4\uc5d0 \uc885\ud569\uc608\uc220\uc81c\uc778 \ubc31\uc81c\ubb38\ud654\uc81c\ub97c \uac1c\ucd5c\ud55c\ub2e4."} {"question": "1912\ub144 '\uacf5'\uc758 \uc9c0\uc704\ub97c \uac16\uace0 \uc788\ub358 \uc778\ubb3c\uc778 \uc774\uc900\uc6a9\uc744 \uc81c\uc678\ud55c \uacf5\uc758 \uc9c0\uc704\uc5d0 \uc788\ub358 \uc778\ubb3c\uc740?", "paragraph": "\uc774\uc900\uc6a9\uc740 1912\ub144\uc5d0 \uc0ac\ub9dd\ud55c \uc544\ubc84\uc9c0 \uc774\uc7ac\uba74\uc758 \ub4a4\ub97c \uc774\uc5b4 \u2018\uacf5\u2019(\u516c)\uc758 \uc9c0\uc704\ub97c \uac16\uace0 \uc788\uc5c8\uae30 \ub54c\ubb38\uc5d0 \uc774\uc6b0\ub294 \uc6b4\ud604\uad81\uc758 \uc0c1\uc18d\uc790\ub85c \uacb0\uc815\ub418\uc790 \uacf5\uc704\ub97c \uc138\uc2b5 \ubc1b\uc544 \ud55c\uc815\ub41c \ubc94\uc704\uc758 \uc655\uacf5\uc871\uc5d0\uac8c\ub9cc \uc0ac\uc6a9\ub418\ub358 \u2018\uc804\ud558\u2019(\u6bbf\u4e0b)\uc758 \uacbd\uce6d\uc744 \ubc1b\uac8c \ub418\uc5c8\ub2e4. \ub2f9\uc2dc \uc870\uc120\uc5d0\uc11c \uacf5\uc758 \uc9c0\uc704\uc5d0 \uc788\uc5c8\ub358 \uc778\ubb3c\uc740 \uc774\uac15\uacfc \uc774\uc6b0 \ubc16\uc5d0 \uc5c6\uc5c8\ub2e4. \uc774\uc6b0\uc758 \uc2e0\ubd84\uc778 \uacf5\uc871\uc758 \uacf5\uc740 1910\ub144\uc5d0 \ud55c\uc77c \ubcd1\ud569 \uc870\uc57d\uc774 \uccb4\uacb0\ub41c \uc774\ud6c4 \uc77c\ubcf8 \uc81c\uad6d \uc815\ubd80\uac00 \ub300\ud55c\uc81c\uad6d \ud669\uc2e4\uc744 \uaddc\uc728\ud558\uae30 \uc704\ud574 \uc0c8\ub86d\uac8c \ucc3d\ucd9c\ud55c \uc2e0\ubd84\uc774\ub2e4. \uacf5\uc740 \uc624\ub4f1\uc791\uc758 \uc77c\uc885\uc778 \uc77c\ubcf8 \ud654\uc871\uacfc \uc870\uc120\uadc0\uc871\uc758 \uacf5\uc791\uacfc\ub294 \ub2e4\ub978 \uac1c\ub150\uc73c\ub85c \uc11c\uc5f4\uc0c1 \uc77c\ubcf8 \ud669\uc871\uacfc \uc655\uc871 \uc544\ub798\uc5d0 \uc704\uce58\ud558\uc9c0\ub9cc \ud654\uc871\uacfc \uc870\uc120\uadc0\uc871\ubcf4\ub2e4\ub294 \ub192\uc740 \uc9c0\uc704\uc774\uba70, \uc77c\ubcf8 \ud669\uc871\uc5d0 \ud3b8\uc785\ub41c \uc2e0\ubd84\uc740 \uc544\ub2c8\uc9c0\ub9cc \ubcd1\ud569 \ub2f9\uc2dc \uacf5\ud3ec\ub41c \uba54\uc774\uc9c0 \ucc9c\ud669\uc758 \uc870\uc11c\uc5d0 \ub530\ub77c \ud669\uc871\uc5d0 \uc900\ud558\ub294 \uc608\uc6b0\ub97c \ubc1b\uc558\ub2e4.", "answer": "\uc774\uac15\uacfc \uc774\uc6b0", "sentence": "\ub2f9\uc2dc \uc870\uc120\uc5d0\uc11c \uacf5\uc758 \uc9c0\uc704\uc5d0 \uc788\uc5c8\ub358 \uc778\ubb3c\uc740 \uc774\uac15\uacfc \uc774\uc6b0 \ubc16\uc5d0 \uc5c6\uc5c8\ub2e4.", "paragraph_sentence": "\uc774\uc900\uc6a9\uc740 1912\ub144\uc5d0 \uc0ac\ub9dd\ud55c \uc544\ubc84\uc9c0 \uc774\uc7ac\uba74\uc758 \ub4a4\ub97c \uc774\uc5b4 \u2018\uacf5\u2019(\u516c)\uc758 \uc9c0\uc704\ub97c \uac16\uace0 \uc788\uc5c8\uae30 \ub54c\ubb38\uc5d0 \uc774\uc6b0\ub294 \uc6b4\ud604\uad81\uc758 \uc0c1\uc18d\uc790\ub85c \uacb0\uc815\ub418\uc790 \uacf5\uc704\ub97c \uc138\uc2b5 \ubc1b\uc544 \ud55c\uc815\ub41c \ubc94\uc704\uc758 \uc655\uacf5\uc871\uc5d0\uac8c\ub9cc \uc0ac\uc6a9\ub418\ub358 \u2018\uc804\ud558\u2019(\u6bbf\u4e0b)\uc758 \uacbd\uce6d\uc744 \ubc1b\uac8c \ub418\uc5c8\ub2e4. \ub2f9\uc2dc \uc870\uc120\uc5d0\uc11c \uacf5\uc758 \uc9c0\uc704\uc5d0 \uc788\uc5c8\ub358 \uc778\ubb3c\uc740 \uc774\uac15\uacfc \uc774\uc6b0 \ubc16\uc5d0 \uc5c6\uc5c8\ub2e4. \uc774\uc6b0\uc758 \uc2e0\ubd84\uc778 \uacf5\uc871\uc758 \uacf5\uc740 1910\ub144\uc5d0 \ud55c\uc77c \ubcd1\ud569 \uc870\uc57d\uc774 \uccb4\uacb0\ub41c \uc774\ud6c4 \uc77c\ubcf8 \uc81c\uad6d \uc815\ubd80\uac00 \ub300\ud55c\uc81c\uad6d \ud669\uc2e4\uc744 \uaddc\uc728\ud558\uae30 \uc704\ud574 \uc0c8\ub86d\uac8c \ucc3d\ucd9c\ud55c \uc2e0\ubd84\uc774\ub2e4. \uacf5\uc740 \uc624\ub4f1\uc791\uc758 \uc77c\uc885\uc778 \uc77c\ubcf8 \ud654\uc871\uacfc \uc870\uc120\uadc0\uc871\uc758 \uacf5\uc791\uacfc\ub294 \ub2e4\ub978 \uac1c\ub150\uc73c\ub85c \uc11c\uc5f4\uc0c1 \uc77c\ubcf8 \ud669\uc871\uacfc \uc655\uc871 \uc544\ub798\uc5d0 \uc704\uce58\ud558\uc9c0\ub9cc \ud654\uc871\uacfc \uc870\uc120\uadc0\uc871\ubcf4\ub2e4\ub294 \ub192\uc740 \uc9c0\uc704\uc774\uba70, \uc77c\ubcf8 \ud669\uc871\uc5d0 \ud3b8\uc785\ub41c \uc2e0\ubd84\uc740 \uc544\ub2c8\uc9c0\ub9cc \ubcd1\ud569 \ub2f9\uc2dc \uacf5\ud3ec\ub41c \uba54\uc774\uc9c0 \ucc9c\ud669\uc758 \uc870\uc11c\uc5d0 \ub530\ub77c \ud669\uc871\uc5d0 \uc900\ud558\ub294 \uc608\uc6b0\ub97c \ubc1b\uc558\ub2e4.", "paragraph_answer": "\uc774\uc900\uc6a9\uc740 1912\ub144\uc5d0 \uc0ac\ub9dd\ud55c \uc544\ubc84\uc9c0 \uc774\uc7ac\uba74\uc758 \ub4a4\ub97c \uc774\uc5b4 \u2018\uacf5\u2019(\u516c)\uc758 \uc9c0\uc704\ub97c \uac16\uace0 \uc788\uc5c8\uae30 \ub54c\ubb38\uc5d0 \uc774\uc6b0\ub294 \uc6b4\ud604\uad81\uc758 \uc0c1\uc18d\uc790\ub85c \uacb0\uc815\ub418\uc790 \uacf5\uc704\ub97c \uc138\uc2b5 \ubc1b\uc544 \ud55c\uc815\ub41c \ubc94\uc704\uc758 \uc655\uacf5\uc871\uc5d0\uac8c\ub9cc \uc0ac\uc6a9\ub418\ub358 \u2018\uc804\ud558\u2019(\u6bbf\u4e0b)\uc758 \uacbd\uce6d\uc744 \ubc1b\uac8c \ub418\uc5c8\ub2e4. \ub2f9\uc2dc \uc870\uc120\uc5d0\uc11c \uacf5\uc758 \uc9c0\uc704\uc5d0 \uc788\uc5c8\ub358 \uc778\ubb3c\uc740 \uc774\uac15\uacfc \uc774\uc6b0 \ubc16\uc5d0 \uc5c6\uc5c8\ub2e4. \uc774\uc6b0\uc758 \uc2e0\ubd84\uc778 \uacf5\uc871\uc758 \uacf5\uc740 1910\ub144\uc5d0 \ud55c\uc77c \ubcd1\ud569 \uc870\uc57d\uc774 \uccb4\uacb0\ub41c \uc774\ud6c4 \uc77c\ubcf8 \uc81c\uad6d \uc815\ubd80\uac00 \ub300\ud55c\uc81c\uad6d \ud669\uc2e4\uc744 \uaddc\uc728\ud558\uae30 \uc704\ud574 \uc0c8\ub86d\uac8c \ucc3d\ucd9c\ud55c \uc2e0\ubd84\uc774\ub2e4. \uacf5\uc740 \uc624\ub4f1\uc791\uc758 \uc77c\uc885\uc778 \uc77c\ubcf8 \ud654\uc871\uacfc \uc870\uc120\uadc0\uc871\uc758 \uacf5\uc791\uacfc\ub294 \ub2e4\ub978 \uac1c\ub150\uc73c\ub85c \uc11c\uc5f4\uc0c1 \uc77c\ubcf8 \ud669\uc871\uacfc \uc655\uc871 \uc544\ub798\uc5d0 \uc704\uce58\ud558\uc9c0\ub9cc \ud654\uc871\uacfc \uc870\uc120\uadc0\uc871\ubcf4\ub2e4\ub294 \ub192\uc740 \uc9c0\uc704\uc774\uba70, \uc77c\ubcf8 \ud669\uc871\uc5d0 \ud3b8\uc785\ub41c \uc2e0\ubd84\uc740 \uc544\ub2c8\uc9c0\ub9cc \ubcd1\ud569 \ub2f9\uc2dc \uacf5\ud3ec\ub41c \uba54\uc774\uc9c0 \ucc9c\ud669\uc758 \uc870\uc11c\uc5d0 \ub530\ub77c \ud669\uc871\uc5d0 \uc900\ud558\ub294 \uc608\uc6b0\ub97c \ubc1b\uc558\ub2e4.", "sentence_answer": "\ub2f9\uc2dc \uc870\uc120\uc5d0\uc11c \uacf5\uc758 \uc9c0\uc704\uc5d0 \uc788\uc5c8\ub358 \uc778\ubb3c\uc740 \uc774\uac15\uacfc \uc774\uc6b0 \ubc16\uc5d0 \uc5c6\uc5c8\ub2e4.", "paragraph_id": "6545881-1-0", "paragraph_question": "question: 1912\ub144 '\uacf5'\uc758 \uc9c0\uc704\ub97c \uac16\uace0 \uc788\ub358 \uc778\ubb3c\uc778 \uc774\uc900\uc6a9\uc744 \uc81c\uc678\ud55c \uacf5\uc758 \uc9c0\uc704\uc5d0 \uc788\ub358 \uc778\ubb3c\uc740?, context: \uc774\uc900\uc6a9\uc740 1912\ub144\uc5d0 \uc0ac\ub9dd\ud55c \uc544\ubc84\uc9c0 \uc774\uc7ac\uba74\uc758 \ub4a4\ub97c \uc774\uc5b4 \u2018\uacf5\u2019(\u516c)\uc758 \uc9c0\uc704\ub97c \uac16\uace0 \uc788\uc5c8\uae30 \ub54c\ubb38\uc5d0 \uc774\uc6b0\ub294 \uc6b4\ud604\uad81\uc758 \uc0c1\uc18d\uc790\ub85c \uacb0\uc815\ub418\uc790 \uacf5\uc704\ub97c \uc138\uc2b5 \ubc1b\uc544 \ud55c\uc815\ub41c \ubc94\uc704\uc758 \uc655\uacf5\uc871\uc5d0\uac8c\ub9cc \uc0ac\uc6a9\ub418\ub358 \u2018\uc804\ud558\u2019(\u6bbf\u4e0b)\uc758 \uacbd\uce6d\uc744 \ubc1b\uac8c \ub418\uc5c8\ub2e4. \ub2f9\uc2dc \uc870\uc120\uc5d0\uc11c \uacf5\uc758 \uc9c0\uc704\uc5d0 \uc788\uc5c8\ub358 \uc778\ubb3c\uc740 \uc774\uac15\uacfc \uc774\uc6b0 \ubc16\uc5d0 \uc5c6\uc5c8\ub2e4. \uc774\uc6b0\uc758 \uc2e0\ubd84\uc778 \uacf5\uc871\uc758 \uacf5\uc740 1910\ub144\uc5d0 \ud55c\uc77c \ubcd1\ud569 \uc870\uc57d\uc774 \uccb4\uacb0\ub41c \uc774\ud6c4 \uc77c\ubcf8 \uc81c\uad6d \uc815\ubd80\uac00 \ub300\ud55c\uc81c\uad6d \ud669\uc2e4\uc744 \uaddc\uc728\ud558\uae30 \uc704\ud574 \uc0c8\ub86d\uac8c \ucc3d\ucd9c\ud55c \uc2e0\ubd84\uc774\ub2e4. \uacf5\uc740 \uc624\ub4f1\uc791\uc758 \uc77c\uc885\uc778 \uc77c\ubcf8 \ud654\uc871\uacfc \uc870\uc120\uadc0\uc871\uc758 \uacf5\uc791\uacfc\ub294 \ub2e4\ub978 \uac1c\ub150\uc73c\ub85c \uc11c\uc5f4\uc0c1 \uc77c\ubcf8 \ud669\uc871\uacfc \uc655\uc871 \uc544\ub798\uc5d0 \uc704\uce58\ud558\uc9c0\ub9cc \ud654\uc871\uacfc \uc870\uc120\uadc0\uc871\ubcf4\ub2e4\ub294 \ub192\uc740 \uc9c0\uc704\uc774\uba70, \uc77c\ubcf8 \ud669\uc871\uc5d0 \ud3b8\uc785\ub41c \uc2e0\ubd84\uc740 \uc544\ub2c8\uc9c0\ub9cc \ubcd1\ud569 \ub2f9\uc2dc \uacf5\ud3ec\ub41c \uba54\uc774\uc9c0 \ucc9c\ud669\uc758 \uc870\uc11c\uc5d0 \ub530\ub77c \ud669\uc871\uc5d0 \uc900\ud558\ub294 \uc608\uc6b0\ub97c \ubc1b\uc558\ub2e4."} {"question": "\ub178\ubb34\ud604 \uc804 \ub300\ud1b5\ub839\uc758 \uc720\uc11c\ub294 \ud22c\uc2e0 1\uc2dc\uac04 19\ubd84\uc804\uc778 \uba87 \uc2dc\uc5d0 \uc800\uc7a5\ub41c \ud30c\uc77c\uc774\uc600\ub098?", "paragraph": "\uc544\ub798\ub294 \uadf8\uac00 \ud22c\uc2e0 \uc790\uc0b4\ud558\uae30 \uc804\uc5d0 \ub0a8\uae34 \uac83\uc73c\ub85c \ubcf4\uc774\ub294 \uc720\uc11c \uc804\ubb38\uc774\ub2e4. \uc774 \uc720\uc11c\ub294 \uc0ac\uc800\uc758 \ucef4\ud4e8\ud130\uc5d0 \"\ub098\ub85c \ub9d0\ubbf8\uc554\uc544 \uc5ec\ub7ec \uc0ac\ub78c\uc758 \uace0\ud1b5\uc774 \ub108\ubb34 \ud06c\ub2e4\"\ub77c\ub294 \uc81c\ubaa9\uc758 \ud55c/\uae00 \ud30c\uc77c\ub85c \uc800\uc7a5\ub418\uc5b4 \uc788\uc5c8\ub2e4\uace0 \ud55c\ub2e4. \uae40\uacbd\uc218 \ube44\uc11c\uad00\uc5d0 \ub530\ub974\uba74 \uc774 \uc720\uc11c \ud30c\uc77c\uc774 \uc800\uc7a5\ub41c \uc2dc\uac04\uc740 \ud22c\uc2e0 1\uc2dc\uac04 19\ubd84 \uc804\uc778 \uc624\uc804 5\uc2dc 21\ubd84\uc774\uc5c8\ub2e4\uace0 \ubc1d\ud614\ub2e4. \ud55c\ud3b8 \uc774 \uc720\uc11c\uc5d0\ub294 \ub3c8 \ubb38\uc81c\uc640 \uad00\ub828\ub41c \uc77c\ubd80\ubd84\uc774 \ub204\ub77d\ub418\uc5c8\ub2e4\ub294 \uc8fc\uc7a5\ub3c4 \uc788\uc73c\ub098 \uacbd\ucc30\uc740 \uc870\uc791\uc124\uc744 \uc77c\ucd95\ud588\ub2e4. \uc5f0\ud569\ub274\uc2a4\ub294 \"\uc720\uc11c \uc870\uc791 \uc758\ud639\uc740 \ub178 \uc804 \ub300\ud1b5\ub839\uc758 \uce21\uadfc\uc774 \uc720\uc11c\uc5d0 \ub2f4\uae34 \ub0b4\uc6a9\uc774\ub77c\uba70 \uc804\ud55c \uc774\uc57c\uae30\ub97c \uc77c\ubd80 \ub9e4\uccb4\uac00 \uc81c\ub300\ub85c \ud655\uc778\ud558\uc9c0 \uc54a\uace0 \ubcf4\ub3c4\ud558\uba74\uc11c \ube44\ub86f\ub41c \ud63c\uc120 \ub54c\ubb38\uc774\uc5c8\ub358 \uac83\uc73c\ub85c \ud655\uc778\ub418\uace0 \uc788\ub2e4\"\uba74\uc11c \"\uc0ad\uc81c\ub410\ub2e4\uace0 \uc8fc\uc7a5\ud558\ub294 \ub0b4\uc6a9\uc774 \uacbd\ucc30\uc758 \uacf5\uc2dd \ubc1c\ud45c \uc774\uc804\uc5d0 \uc77c\ubd80 \ub9e4\uccb4\ub4e4\uc774 \ubcf4\ub3c4\ud55c \ub0b4\uc6a9\uacfc \uac70\uc758 \ub3d9\uc77c\ud558\uae30 \ub54c\ubb38\uc774\ub2e4\"\ub77c\uba70 \ud3c9\ud588\ub2e4.", "answer": "\uc624\uc804 5\uc2dc 21\ubd84", "sentence": "\uae40\uacbd\uc218 \ube44\uc11c\uad00\uc5d0 \ub530\ub974\uba74 \uc774 \uc720\uc11c \ud30c\uc77c\uc774 \uc800\uc7a5\ub41c \uc2dc\uac04\uc740 \ud22c\uc2e0 1\uc2dc\uac04 19\ubd84 \uc804\uc778 \uc624\uc804 5\uc2dc 21\ubd84 \uc774\uc5c8\ub2e4\uace0 \ubc1d\ud614\ub2e4.", "paragraph_sentence": "\uc544\ub798\ub294 \uadf8\uac00 \ud22c\uc2e0 \uc790\uc0b4\ud558\uae30 \uc804\uc5d0 \ub0a8\uae34 \uac83\uc73c\ub85c \ubcf4\uc774\ub294 \uc720\uc11c \uc804\ubb38\uc774\ub2e4. \uc774 \uc720\uc11c\ub294 \uc0ac\uc800\uc758 \ucef4\ud4e8\ud130\uc5d0 \"\ub098\ub85c \ub9d0\ubbf8\uc554\uc544 \uc5ec\ub7ec \uc0ac\ub78c\uc758 \uace0\ud1b5\uc774 \ub108\ubb34 \ud06c\ub2e4\"\ub77c\ub294 \uc81c\ubaa9\uc758 \ud55c/\uae00 \ud30c\uc77c\ub85c \uc800\uc7a5\ub418\uc5b4 \uc788\uc5c8\ub2e4\uace0 \ud55c\ub2e4. \uae40\uacbd\uc218 \ube44\uc11c\uad00\uc5d0 \ub530\ub974\uba74 \uc774 \uc720\uc11c \ud30c\uc77c\uc774 \uc800\uc7a5\ub41c \uc2dc\uac04\uc740 \ud22c\uc2e0 1\uc2dc\uac04 19\ubd84 \uc804\uc778 \uc624\uc804 5\uc2dc 21\ubd84 \uc774\uc5c8\ub2e4\uace0 \ubc1d\ud614\ub2e4. \ud55c\ud3b8 \uc774 \uc720\uc11c\uc5d0\ub294 \ub3c8 \ubb38\uc81c\uc640 \uad00\ub828\ub41c \uc77c\ubd80\ubd84\uc774 \ub204\ub77d\ub418\uc5c8\ub2e4\ub294 \uc8fc\uc7a5\ub3c4 \uc788\uc73c\ub098 \uacbd\ucc30\uc740 \uc870\uc791\uc124\uc744 \uc77c\ucd95\ud588\ub2e4. \uc5f0\ud569\ub274\uc2a4\ub294 \"\uc720\uc11c \uc870\uc791 \uc758\ud639\uc740 \ub178 \uc804 \ub300\ud1b5\ub839\uc758 \uce21\uadfc\uc774 \uc720\uc11c\uc5d0 \ub2f4\uae34 \ub0b4\uc6a9\uc774\ub77c\uba70 \uc804\ud55c \uc774\uc57c\uae30\ub97c \uc77c\ubd80 \ub9e4\uccb4\uac00 \uc81c\ub300\ub85c \ud655\uc778\ud558\uc9c0 \uc54a\uace0 \ubcf4\ub3c4\ud558\uba74\uc11c \ube44\ub86f\ub41c \ud63c\uc120 \ub54c\ubb38\uc774\uc5c8\ub358 \uac83\uc73c\ub85c \ud655\uc778\ub418\uace0 \uc788\ub2e4\"\uba74\uc11c \"\uc0ad\uc81c\ub410\ub2e4\uace0 \uc8fc\uc7a5\ud558\ub294 \ub0b4\uc6a9\uc774 \uacbd\ucc30\uc758 \uacf5\uc2dd \ubc1c\ud45c \uc774\uc804\uc5d0 \uc77c\ubd80 \ub9e4\uccb4\ub4e4\uc774 \ubcf4\ub3c4\ud55c \ub0b4\uc6a9\uacfc \uac70\uc758 \ub3d9\uc77c\ud558\uae30 \ub54c\ubb38\uc774\ub2e4\"\ub77c\uba70 \ud3c9\ud588\ub2e4.", "paragraph_answer": "\uc544\ub798\ub294 \uadf8\uac00 \ud22c\uc2e0 \uc790\uc0b4\ud558\uae30 \uc804\uc5d0 \ub0a8\uae34 \uac83\uc73c\ub85c \ubcf4\uc774\ub294 \uc720\uc11c \uc804\ubb38\uc774\ub2e4. \uc774 \uc720\uc11c\ub294 \uc0ac\uc800\uc758 \ucef4\ud4e8\ud130\uc5d0 \"\ub098\ub85c \ub9d0\ubbf8\uc554\uc544 \uc5ec\ub7ec \uc0ac\ub78c\uc758 \uace0\ud1b5\uc774 \ub108\ubb34 \ud06c\ub2e4\"\ub77c\ub294 \uc81c\ubaa9\uc758 \ud55c/\uae00 \ud30c\uc77c\ub85c \uc800\uc7a5\ub418\uc5b4 \uc788\uc5c8\ub2e4\uace0 \ud55c\ub2e4. \uae40\uacbd\uc218 \ube44\uc11c\uad00\uc5d0 \ub530\ub974\uba74 \uc774 \uc720\uc11c \ud30c\uc77c\uc774 \uc800\uc7a5\ub41c \uc2dc\uac04\uc740 \ud22c\uc2e0 1\uc2dc\uac04 19\ubd84 \uc804\uc778 \uc624\uc804 5\uc2dc 21\ubd84 \uc774\uc5c8\ub2e4\uace0 \ubc1d\ud614\ub2e4. \ud55c\ud3b8 \uc774 \uc720\uc11c\uc5d0\ub294 \ub3c8 \ubb38\uc81c\uc640 \uad00\ub828\ub41c \uc77c\ubd80\ubd84\uc774 \ub204\ub77d\ub418\uc5c8\ub2e4\ub294 \uc8fc\uc7a5\ub3c4 \uc788\uc73c\ub098 \uacbd\ucc30\uc740 \uc870\uc791\uc124\uc744 \uc77c\ucd95\ud588\ub2e4. \uc5f0\ud569\ub274\uc2a4\ub294 \"\uc720\uc11c \uc870\uc791 \uc758\ud639\uc740 \ub178 \uc804 \ub300\ud1b5\ub839\uc758 \uce21\uadfc\uc774 \uc720\uc11c\uc5d0 \ub2f4\uae34 \ub0b4\uc6a9\uc774\ub77c\uba70 \uc804\ud55c \uc774\uc57c\uae30\ub97c \uc77c\ubd80 \ub9e4\uccb4\uac00 \uc81c\ub300\ub85c \ud655\uc778\ud558\uc9c0 \uc54a\uace0 \ubcf4\ub3c4\ud558\uba74\uc11c \ube44\ub86f\ub41c \ud63c\uc120 \ub54c\ubb38\uc774\uc5c8\ub358 \uac83\uc73c\ub85c \ud655\uc778\ub418\uace0 \uc788\ub2e4\"\uba74\uc11c \"\uc0ad\uc81c\ub410\ub2e4\uace0 \uc8fc\uc7a5\ud558\ub294 \ub0b4\uc6a9\uc774 \uacbd\ucc30\uc758 \uacf5\uc2dd \ubc1c\ud45c \uc774\uc804\uc5d0 \uc77c\ubd80 \ub9e4\uccb4\ub4e4\uc774 \ubcf4\ub3c4\ud55c \ub0b4\uc6a9\uacfc \uac70\uc758 \ub3d9\uc77c\ud558\uae30 \ub54c\ubb38\uc774\ub2e4\"\ub77c\uba70 \ud3c9\ud588\ub2e4.", "sentence_answer": "\uae40\uacbd\uc218 \ube44\uc11c\uad00\uc5d0 \ub530\ub974\uba74 \uc774 \uc720\uc11c \ud30c\uc77c\uc774 \uc800\uc7a5\ub41c \uc2dc\uac04\uc740 \ud22c\uc2e0 1\uc2dc\uac04 19\ubd84 \uc804\uc778 \uc624\uc804 5\uc2dc 21\ubd84 \uc774\uc5c8\ub2e4\uace0 \ubc1d\ud614\ub2e4.", "paragraph_id": "6483948-27-1", "paragraph_question": "question: \ub178\ubb34\ud604 \uc804 \ub300\ud1b5\ub839\uc758 \uc720\uc11c\ub294 \ud22c\uc2e0 1\uc2dc\uac04 19\ubd84\uc804\uc778 \uba87 \uc2dc\uc5d0 \uc800\uc7a5\ub41c \ud30c\uc77c\uc774\uc600\ub098?, context: \uc544\ub798\ub294 \uadf8\uac00 \ud22c\uc2e0 \uc790\uc0b4\ud558\uae30 \uc804\uc5d0 \ub0a8\uae34 \uac83\uc73c\ub85c \ubcf4\uc774\ub294 \uc720\uc11c \uc804\ubb38\uc774\ub2e4. \uc774 \uc720\uc11c\ub294 \uc0ac\uc800\uc758 \ucef4\ud4e8\ud130\uc5d0 \"\ub098\ub85c \ub9d0\ubbf8\uc554\uc544 \uc5ec\ub7ec \uc0ac\ub78c\uc758 \uace0\ud1b5\uc774 \ub108\ubb34 \ud06c\ub2e4\"\ub77c\ub294 \uc81c\ubaa9\uc758 \ud55c/\uae00 \ud30c\uc77c\ub85c \uc800\uc7a5\ub418\uc5b4 \uc788\uc5c8\ub2e4\uace0 \ud55c\ub2e4. \uae40\uacbd\uc218 \ube44\uc11c\uad00\uc5d0 \ub530\ub974\uba74 \uc774 \uc720\uc11c \ud30c\uc77c\uc774 \uc800\uc7a5\ub41c \uc2dc\uac04\uc740 \ud22c\uc2e0 1\uc2dc\uac04 19\ubd84 \uc804\uc778 \uc624\uc804 5\uc2dc 21\ubd84\uc774\uc5c8\ub2e4\uace0 \ubc1d\ud614\ub2e4. \ud55c\ud3b8 \uc774 \uc720\uc11c\uc5d0\ub294 \ub3c8 \ubb38\uc81c\uc640 \uad00\ub828\ub41c \uc77c\ubd80\ubd84\uc774 \ub204\ub77d\ub418\uc5c8\ub2e4\ub294 \uc8fc\uc7a5\ub3c4 \uc788\uc73c\ub098 \uacbd\ucc30\uc740 \uc870\uc791\uc124\uc744 \uc77c\ucd95\ud588\ub2e4. \uc5f0\ud569\ub274\uc2a4\ub294 \"\uc720\uc11c \uc870\uc791 \uc758\ud639\uc740 \ub178 \uc804 \ub300\ud1b5\ub839\uc758 \uce21\uadfc\uc774 \uc720\uc11c\uc5d0 \ub2f4\uae34 \ub0b4\uc6a9\uc774\ub77c\uba70 \uc804\ud55c \uc774\uc57c\uae30\ub97c \uc77c\ubd80 \ub9e4\uccb4\uac00 \uc81c\ub300\ub85c \ud655\uc778\ud558\uc9c0 \uc54a\uace0 \ubcf4\ub3c4\ud558\uba74\uc11c \ube44\ub86f\ub41c \ud63c\uc120 \ub54c\ubb38\uc774\uc5c8\ub358 \uac83\uc73c\ub85c \ud655\uc778\ub418\uace0 \uc788\ub2e4\"\uba74\uc11c \"\uc0ad\uc81c\ub410\ub2e4\uace0 \uc8fc\uc7a5\ud558\ub294 \ub0b4\uc6a9\uc774 \uacbd\ucc30\uc758 \uacf5\uc2dd \ubc1c\ud45c \uc774\uc804\uc5d0 \uc77c\ubd80 \ub9e4\uccb4\ub4e4\uc774 \ubcf4\ub3c4\ud55c \ub0b4\uc6a9\uacfc \uac70\uc758 \ub3d9\uc77c\ud558\uae30 \ub54c\ubb38\uc774\ub2e4\"\ub77c\uba70 \ud3c9\ud588\ub2e4."} {"question": "\ub300\ud55c\ubbfc\uad6d \ucd5c\ucd08 \uc5ec\uc131 \uc678\uad50\uad00\uc778 \uc804\uc778\ubc94\uc758 \uc5b4\uba38\ub2c8 \uc774\ub984\uc740?", "paragraph": "\uc804\uc778\ubc94\uc740 1958\ub144 9\uc6d4 6\uc77c\uc5d0 \uc11c\uc6b8\ud2b9\ubcc4\uc2dc\uc5d0\uc11c \ud55c\uc758\uc0ac\uc778 \uc544\ubc84\uc9c0 \uc804\uc8fc\ud654\uc640 \uc678\uad50\uad00\uc778 \uc5b4\uba38\ub2c8 \ud64d\uc219\uc790 \uc0ac\uc774\uc5d0\uc11c 1\ub0a8 1\ub140 \uc911 \uc7a5\ub0a8\uc73c\ub85c \ud0dc\uc5b4\ub0ac\ub2e4. \uc720\ub144 \uc2dc\uc808\uc758 \ub300\ubd80\ubd84\uc740 \uc11c\uc6b8\uc5d0 \uc0b4\uc558\uc73c\uba70, 1965\ub144 \ubd80\ubaa8\uc758 \uc774\ud63c \ud6c4 \ub300\ud55c\ubbfc\uad6d \ucd5c\ucd08\uc758 \uc5ec\uc131 \uc678\uad50\uad00\uc778 \ubaa8\uce5c\uc744 \ub530\ub77c \ubbf8\uad6d\uc73c\ub85c \uc774\uc8fc\ud558\uc5ec 4\ub144 \ub3d9\uc548 \ub274\uc695 \ub9e8\ud574\ud2bc\uc5d0\uc11c \uc0dd\ud65c\ud558\uc600\ub2e4. 1969\ub144 \uadc0\uad6d\ud55c \ub4a4\uc5d0\ub3c4 \uadf8\uc758 \ubaa8\uce5c\uc740 \uadf8\uac00 \uc601\uc5b4\ub97c \uc78a\uc9c0 \uc54a\ub3c4\ub85d AFKN\uc744 \uc8fc\ub85c \uc2dc\uccad\ud558\uac8c \ud588\uc73c\uba70 \uc800\ub141\uc2dd\uc0ac \ub54c\ub9cc\ud07c\uc740 \uc601\uc5b4\ub85c \ub300\ud654\ud558\uac8c \ud588\ub2e4. \uadf8 \ub610\ud55c \u300e\ub9ac\ub354\uc2a4 \ub2e4\uc774\uc81c\uc2a4\ud2b8\u300f, \u300e\ub274\uc2a4\uc704\ud06c\u300f, \u300e\ud0c0\uc784\u300f\uacfc \uac19\uc740 \uc2dc\uc0ac \uc7a1\uc9c0\ub97c \ube7c\ub193\uc9c0 \uc54a\uace0 \uc77d\uc73c\uba70 \uc5b4\ub9b4 \ub54c\ubd80\ud130 \uad6d\uc81c\uad00\uacc4\uc5d0 \ub300\ud55c \uac10\uac01\uc744 \uc775\ud600\ub098\uac14\ub2e4. \ub2f9\uc2dc \uc81c2\ucc28 \uc138\uacc4 \ub300\uc804\uc744 \ub2e4\ub8ec \ubbf8\uad6d TV \ud504\ub85c\uadf8\ub7a8 \u300aCombat!\u300b\uc5d0 \ub098\uc624\ub294 \uad70\uc778 \uc0cc\ub354\uc2a4 \uc911\uc0ac\ub97c \ub3d9\uacbd\ud558\ub358 \uadf8\ub294 12\uc0b4 \ub54c \uc678\uc0bc\ucd0c\uc5d0\uac8c \u201c\uad70\uc778\uc774 \ub418\ub824\uba74 \uc721\uad70\uc0ac\uad00\ud559\uad50\uc5d0 \uac00\uc57c \ud55c\ub2e4\u201d\ub294 \ub9d0\uc744 \ub4e3\uace0 \uc721\uc0ac \uc9c4\ud559\uc744 \uafc8\uafb8\uac8c \ub418\uc5c8\ub2e4. \uadf8 \ub4a4 \ub9ac\ub77c\ucd08\ub4f1\ud559\uad50\uc640 \ub300\uacbd\uc911\ud559\uad50\ub97c \uac70\uccd0 1977\ub144 \uacbd\uae30\uace0\ub4f1\ud559\uad50\ub97c \uc878\uc5c5\ud558\uace0 \uc721\uad70\uc0ac\uad00\ud559\uad50 37\uae30\ub85c \uc785\uad50\ud588\ub2e4. \uc721\uc0ac \ub3d9\uae30\uc0dd \uc911 \uc131\uc801\uc774 \uaf34\ucc0c\uc5d0\uc11c \ub450 \ubc88\uc9f8\uc600\uc73c\ub098 \uc601\uc5b4 \uc2dc\ud5d8\uc740 \ub9cc\uc810\uc744 \ubc1b\uc558\ub358 \uadf8\ub294 1981\ub144 4\uc6d4 3\uc77c \uc721\uad70\uc0ac\uad00\ud559\uad50 \uc878\uc5c5\uacfc \ub3d9\uc2dc\uc5d0 \uc721\uad70 \uc18c\uc704\ub85c \uc784\uad00\ud588\ub2e4.", "answer": "\ud64d\uc219\uc790", "sentence": "\uc804\uc778\ubc94\uc740 1958\ub144 9\uc6d4 6\uc77c\uc5d0 \uc11c\uc6b8\ud2b9\ubcc4\uc2dc\uc5d0\uc11c \ud55c\uc758\uc0ac\uc778 \uc544\ubc84\uc9c0 \uc804\uc8fc\ud654\uc640 \uc678\uad50\uad00\uc778 \uc5b4\uba38\ub2c8 \ud64d\uc219\uc790 \uc0ac\uc774\uc5d0\uc11c 1\ub0a8 1\ub140 \uc911 \uc7a5\ub0a8\uc73c\ub85c \ud0dc\uc5b4\ub0ac\ub2e4.", "paragraph_sentence": " \uc804\uc778\ubc94\uc740 1958\ub144 9\uc6d4 6\uc77c\uc5d0 \uc11c\uc6b8\ud2b9\ubcc4\uc2dc\uc5d0\uc11c \ud55c\uc758\uc0ac\uc778 \uc544\ubc84\uc9c0 \uc804\uc8fc\ud654\uc640 \uc678\uad50\uad00\uc778 \uc5b4\uba38\ub2c8 \ud64d\uc219\uc790 \uc0ac\uc774\uc5d0\uc11c 1\ub0a8 1\ub140 \uc911 \uc7a5\ub0a8\uc73c\ub85c \ud0dc\uc5b4\ub0ac\ub2e4. \uc720\ub144 \uc2dc\uc808\uc758 \ub300\ubd80\ubd84\uc740 \uc11c\uc6b8\uc5d0 \uc0b4\uc558\uc73c\uba70, 1965\ub144 \ubd80\ubaa8\uc758 \uc774\ud63c \ud6c4 \ub300\ud55c\ubbfc\uad6d \ucd5c\ucd08\uc758 \uc5ec\uc131 \uc678\uad50\uad00\uc778 \ubaa8\uce5c\uc744 \ub530\ub77c \ubbf8\uad6d\uc73c\ub85c \uc774\uc8fc\ud558\uc5ec 4\ub144 \ub3d9\uc548 \ub274\uc695 \ub9e8\ud574\ud2bc\uc5d0\uc11c \uc0dd\ud65c\ud558\uc600\ub2e4. 1969\ub144 \uadc0\uad6d\ud55c \ub4a4\uc5d0\ub3c4 \uadf8\uc758 \ubaa8\uce5c\uc740 \uadf8\uac00 \uc601\uc5b4\ub97c \uc78a\uc9c0 \uc54a\ub3c4\ub85d AFKN\uc744 \uc8fc\ub85c \uc2dc\uccad\ud558\uac8c \ud588\uc73c\uba70 \uc800\ub141\uc2dd\uc0ac \ub54c\ub9cc\ud07c\uc740 \uc601\uc5b4\ub85c \ub300\ud654\ud558\uac8c \ud588\ub2e4. \uadf8 \ub610\ud55c \u300e\ub9ac\ub354\uc2a4 \ub2e4\uc774\uc81c\uc2a4\ud2b8\u300f, \u300e\ub274\uc2a4\uc704\ud06c\u300f, \u300e\ud0c0\uc784\u300f\uacfc \uac19\uc740 \uc2dc\uc0ac \uc7a1\uc9c0\ub97c \ube7c\ub193\uc9c0 \uc54a\uace0 \uc77d\uc73c\uba70 \uc5b4\ub9b4 \ub54c\ubd80\ud130 \uad6d\uc81c\uad00\uacc4\uc5d0 \ub300\ud55c \uac10\uac01\uc744 \uc775\ud600\ub098\uac14\ub2e4. \ub2f9\uc2dc \uc81c2\ucc28 \uc138\uacc4 \ub300\uc804\uc744 \ub2e4\ub8ec \ubbf8\uad6d TV \ud504\ub85c\uadf8\ub7a8 \u300aCombat!\u300b\uc5d0 \ub098\uc624\ub294 \uad70\uc778 \uc0cc\ub354\uc2a4 \uc911\uc0ac\ub97c \ub3d9\uacbd\ud558\ub358 \uadf8\ub294 12\uc0b4 \ub54c \uc678\uc0bc\ucd0c\uc5d0\uac8c \u201c\uad70\uc778\uc774 \ub418\ub824\uba74 \uc721\uad70\uc0ac\uad00\ud559\uad50\uc5d0 \uac00\uc57c \ud55c\ub2e4\u201d\ub294 \ub9d0\uc744 \ub4e3\uace0 \uc721\uc0ac \uc9c4\ud559\uc744 \uafc8\uafb8\uac8c \ub418\uc5c8\ub2e4. \uadf8 \ub4a4 \ub9ac\ub77c\ucd08\ub4f1\ud559\uad50\uc640 \ub300\uacbd\uc911\ud559\uad50\ub97c \uac70\uccd0 1977\ub144 \uacbd\uae30\uace0\ub4f1\ud559\uad50\ub97c \uc878\uc5c5\ud558\uace0 \uc721\uad70\uc0ac\uad00\ud559\uad50 37\uae30\ub85c \uc785\uad50\ud588\ub2e4. \uc721\uc0ac \ub3d9\uae30\uc0dd \uc911 \uc131\uc801\uc774 \uaf34\ucc0c\uc5d0\uc11c \ub450 \ubc88\uc9f8\uc600\uc73c\ub098 \uc601\uc5b4 \uc2dc\ud5d8\uc740 \ub9cc\uc810\uc744 \ubc1b\uc558\ub358 \uadf8\ub294 1981\ub144 4\uc6d4 3\uc77c \uc721\uad70\uc0ac\uad00\ud559\uad50 \uc878\uc5c5\uacfc \ub3d9\uc2dc\uc5d0 \uc721\uad70 \uc18c\uc704\ub85c \uc784\uad00\ud588\ub2e4.", "paragraph_answer": "\uc804\uc778\ubc94\uc740 1958\ub144 9\uc6d4 6\uc77c\uc5d0 \uc11c\uc6b8\ud2b9\ubcc4\uc2dc\uc5d0\uc11c \ud55c\uc758\uc0ac\uc778 \uc544\ubc84\uc9c0 \uc804\uc8fc\ud654\uc640 \uc678\uad50\uad00\uc778 \uc5b4\uba38\ub2c8 \ud64d\uc219\uc790 \uc0ac\uc774\uc5d0\uc11c 1\ub0a8 1\ub140 \uc911 \uc7a5\ub0a8\uc73c\ub85c \ud0dc\uc5b4\ub0ac\ub2e4. \uc720\ub144 \uc2dc\uc808\uc758 \ub300\ubd80\ubd84\uc740 \uc11c\uc6b8\uc5d0 \uc0b4\uc558\uc73c\uba70, 1965\ub144 \ubd80\ubaa8\uc758 \uc774\ud63c \ud6c4 \ub300\ud55c\ubbfc\uad6d \ucd5c\ucd08\uc758 \uc5ec\uc131 \uc678\uad50\uad00\uc778 \ubaa8\uce5c\uc744 \ub530\ub77c \ubbf8\uad6d\uc73c\ub85c \uc774\uc8fc\ud558\uc5ec 4\ub144 \ub3d9\uc548 \ub274\uc695 \ub9e8\ud574\ud2bc\uc5d0\uc11c \uc0dd\ud65c\ud558\uc600\ub2e4. 1969\ub144 \uadc0\uad6d\ud55c \ub4a4\uc5d0\ub3c4 \uadf8\uc758 \ubaa8\uce5c\uc740 \uadf8\uac00 \uc601\uc5b4\ub97c \uc78a\uc9c0 \uc54a\ub3c4\ub85d AFKN\uc744 \uc8fc\ub85c \uc2dc\uccad\ud558\uac8c \ud588\uc73c\uba70 \uc800\ub141\uc2dd\uc0ac \ub54c\ub9cc\ud07c\uc740 \uc601\uc5b4\ub85c \ub300\ud654\ud558\uac8c \ud588\ub2e4. \uadf8 \ub610\ud55c \u300e\ub9ac\ub354\uc2a4 \ub2e4\uc774\uc81c\uc2a4\ud2b8\u300f, \u300e\ub274\uc2a4\uc704\ud06c\u300f, \u300e\ud0c0\uc784\u300f\uacfc \uac19\uc740 \uc2dc\uc0ac \uc7a1\uc9c0\ub97c \ube7c\ub193\uc9c0 \uc54a\uace0 \uc77d\uc73c\uba70 \uc5b4\ub9b4 \ub54c\ubd80\ud130 \uad6d\uc81c\uad00\uacc4\uc5d0 \ub300\ud55c \uac10\uac01\uc744 \uc775\ud600\ub098\uac14\ub2e4. \ub2f9\uc2dc \uc81c2\ucc28 \uc138\uacc4 \ub300\uc804\uc744 \ub2e4\ub8ec \ubbf8\uad6d TV \ud504\ub85c\uadf8\ub7a8 \u300aCombat!\u300b\uc5d0 \ub098\uc624\ub294 \uad70\uc778 \uc0cc\ub354\uc2a4 \uc911\uc0ac\ub97c \ub3d9\uacbd\ud558\ub358 \uadf8\ub294 12\uc0b4 \ub54c \uc678\uc0bc\ucd0c\uc5d0\uac8c \u201c\uad70\uc778\uc774 \ub418\ub824\uba74 \uc721\uad70\uc0ac\uad00\ud559\uad50\uc5d0 \uac00\uc57c \ud55c\ub2e4\u201d\ub294 \ub9d0\uc744 \ub4e3\uace0 \uc721\uc0ac \uc9c4\ud559\uc744 \uafc8\uafb8\uac8c \ub418\uc5c8\ub2e4. \uadf8 \ub4a4 \ub9ac\ub77c\ucd08\ub4f1\ud559\uad50\uc640 \ub300\uacbd\uc911\ud559\uad50\ub97c \uac70\uccd0 1977\ub144 \uacbd\uae30\uace0\ub4f1\ud559\uad50\ub97c \uc878\uc5c5\ud558\uace0 \uc721\uad70\uc0ac\uad00\ud559\uad50 37\uae30\ub85c \uc785\uad50\ud588\ub2e4. \uc721\uc0ac \ub3d9\uae30\uc0dd \uc911 \uc131\uc801\uc774 \uaf34\ucc0c\uc5d0\uc11c \ub450 \ubc88\uc9f8\uc600\uc73c\ub098 \uc601\uc5b4 \uc2dc\ud5d8\uc740 \ub9cc\uc810\uc744 \ubc1b\uc558\ub358 \uadf8\ub294 1981\ub144 4\uc6d4 3\uc77c \uc721\uad70\uc0ac\uad00\ud559\uad50 \uc878\uc5c5\uacfc \ub3d9\uc2dc\uc5d0 \uc721\uad70 \uc18c\uc704\ub85c \uc784\uad00\ud588\ub2e4.", "sentence_answer": "\uc804\uc778\ubc94\uc740 1958\ub144 9\uc6d4 6\uc77c\uc5d0 \uc11c\uc6b8\ud2b9\ubcc4\uc2dc\uc5d0\uc11c \ud55c\uc758\uc0ac\uc778 \uc544\ubc84\uc9c0 \uc804\uc8fc\ud654\uc640 \uc678\uad50\uad00\uc778 \uc5b4\uba38\ub2c8 \ud64d\uc219\uc790 \uc0ac\uc774\uc5d0\uc11c 1\ub0a8 1\ub140 \uc911 \uc7a5\ub0a8\uc73c\ub85c \ud0dc\uc5b4\ub0ac\ub2e4.", "paragraph_id": "6593261-0-0", "paragraph_question": "question: \ub300\ud55c\ubbfc\uad6d \ucd5c\ucd08 \uc5ec\uc131 \uc678\uad50\uad00\uc778 \uc804\uc778\ubc94\uc758 \uc5b4\uba38\ub2c8 \uc774\ub984\uc740?, context: \uc804\uc778\ubc94\uc740 1958\ub144 9\uc6d4 6\uc77c\uc5d0 \uc11c\uc6b8\ud2b9\ubcc4\uc2dc\uc5d0\uc11c \ud55c\uc758\uc0ac\uc778 \uc544\ubc84\uc9c0 \uc804\uc8fc\ud654\uc640 \uc678\uad50\uad00\uc778 \uc5b4\uba38\ub2c8 \ud64d\uc219\uc790 \uc0ac\uc774\uc5d0\uc11c 1\ub0a8 1\ub140 \uc911 \uc7a5\ub0a8\uc73c\ub85c \ud0dc\uc5b4\ub0ac\ub2e4. \uc720\ub144 \uc2dc\uc808\uc758 \ub300\ubd80\ubd84\uc740 \uc11c\uc6b8\uc5d0 \uc0b4\uc558\uc73c\uba70, 1965\ub144 \ubd80\ubaa8\uc758 \uc774\ud63c \ud6c4 \ub300\ud55c\ubbfc\uad6d \ucd5c\ucd08\uc758 \uc5ec\uc131 \uc678\uad50\uad00\uc778 \ubaa8\uce5c\uc744 \ub530\ub77c \ubbf8\uad6d\uc73c\ub85c \uc774\uc8fc\ud558\uc5ec 4\ub144 \ub3d9\uc548 \ub274\uc695 \ub9e8\ud574\ud2bc\uc5d0\uc11c \uc0dd\ud65c\ud558\uc600\ub2e4. 1969\ub144 \uadc0\uad6d\ud55c \ub4a4\uc5d0\ub3c4 \uadf8\uc758 \ubaa8\uce5c\uc740 \uadf8\uac00 \uc601\uc5b4\ub97c \uc78a\uc9c0 \uc54a\ub3c4\ub85d AFKN\uc744 \uc8fc\ub85c \uc2dc\uccad\ud558\uac8c \ud588\uc73c\uba70 \uc800\ub141\uc2dd\uc0ac \ub54c\ub9cc\ud07c\uc740 \uc601\uc5b4\ub85c \ub300\ud654\ud558\uac8c \ud588\ub2e4. \uadf8 \ub610\ud55c \u300e\ub9ac\ub354\uc2a4 \ub2e4\uc774\uc81c\uc2a4\ud2b8\u300f, \u300e\ub274\uc2a4\uc704\ud06c\u300f, \u300e\ud0c0\uc784\u300f\uacfc \uac19\uc740 \uc2dc\uc0ac \uc7a1\uc9c0\ub97c \ube7c\ub193\uc9c0 \uc54a\uace0 \uc77d\uc73c\uba70 \uc5b4\ub9b4 \ub54c\ubd80\ud130 \uad6d\uc81c\uad00\uacc4\uc5d0 \ub300\ud55c \uac10\uac01\uc744 \uc775\ud600\ub098\uac14\ub2e4. \ub2f9\uc2dc \uc81c2\ucc28 \uc138\uacc4 \ub300\uc804\uc744 \ub2e4\ub8ec \ubbf8\uad6d TV \ud504\ub85c\uadf8\ub7a8 \u300aCombat!\u300b\uc5d0 \ub098\uc624\ub294 \uad70\uc778 \uc0cc\ub354\uc2a4 \uc911\uc0ac\ub97c \ub3d9\uacbd\ud558\ub358 \uadf8\ub294 12\uc0b4 \ub54c \uc678\uc0bc\ucd0c\uc5d0\uac8c \u201c\uad70\uc778\uc774 \ub418\ub824\uba74 \uc721\uad70\uc0ac\uad00\ud559\uad50\uc5d0 \uac00\uc57c \ud55c\ub2e4\u201d\ub294 \ub9d0\uc744 \ub4e3\uace0 \uc721\uc0ac \uc9c4\ud559\uc744 \uafc8\uafb8\uac8c \ub418\uc5c8\ub2e4. \uadf8 \ub4a4 \ub9ac\ub77c\ucd08\ub4f1\ud559\uad50\uc640 \ub300\uacbd\uc911\ud559\uad50\ub97c \uac70\uccd0 1977\ub144 \uacbd\uae30\uace0\ub4f1\ud559\uad50\ub97c \uc878\uc5c5\ud558\uace0 \uc721\uad70\uc0ac\uad00\ud559\uad50 37\uae30\ub85c \uc785\uad50\ud588\ub2e4. \uc721\uc0ac \ub3d9\uae30\uc0dd \uc911 \uc131\uc801\uc774 \uaf34\ucc0c\uc5d0\uc11c \ub450 \ubc88\uc9f8\uc600\uc73c\ub098 \uc601\uc5b4 \uc2dc\ud5d8\uc740 \ub9cc\uc810\uc744 \ubc1b\uc558\ub358 \uadf8\ub294 1981\ub144 4\uc6d4 3\uc77c \uc721\uad70\uc0ac\uad00\ud559\uad50 \uc878\uc5c5\uacfc \ub3d9\uc2dc\uc5d0 \uc721\uad70 \uc18c\uc704\ub85c \uc784\uad00\ud588\ub2e4."} {"question": "\uae30\uc6d0\uc804 22\ub144, \ud64d\uc218\uc640 \uae30\uadfc\uc73c\ub85c \uc778\ud55c \uc0dd\ud65c\uace0 \ud798\ub4e4\uc5b4\uc9c4 \uc0ac\ub78c\ub4e4\uc774 \uc544\uc6b0\uad6c\uc2a4\ud22c\uc2a4\uc5d0\uac8c \uc81c\uc548\ud55c \uad00\uc9c1\uc740 \ubb34\uc5c7\uc778\uac00?", "paragraph": "\uc55e\uc5d0\uc11c \uc5b8\uae09\ud55c \uad8c\ud55c\ub4e4\uc758 \ud68d\ub4dd\uc740 \uc815\uce58\uc801\uc73c\ub85c \ub9e4\uc6b0 \ubbf8\ubb18\ud55c \uc704\uc7a5\uc774\uc5c8\ub2e4. \ud3c9\ubbfc \uacc4\uae09\uc774 \uadf8 \uc758\ubbf8\ub97c \uc81c\ub300\ub85c \uc774\ud574\ud558\uc9c0 \ubabb\ud55c \ub370\uc11c \uc774\ub97c \ubbf8\ub8e8\uc5b4 \uc54c \uc218 \uc788\ub2e4. \uae30\uc6d0\uc804 22\ub144\uc5d0 \ud64d\uc218\uc640 \uae30\uadfc\uc774 \uacb9\uccd0\uc11c \uc77c\ubc18 \uc11c\ubbfc\ub4e4\uc758 \uc0dd\ud65c\uc774 \ud798\ub4e4\uc5b4\uc84c\ub2e4. \uc774\ub4e4\uc740 \uc544\uc6b0\uad6c\uc2a4\ud22c\uc2a4\uc5d0\uac8c \uc601\uad6c \uc9d1\uc815\uad00\uc9c1 \ud639\uc740 \ub3c5\uc7ac\uad00\uc9c1\uc744 \ubd80\uc5ec\ud560 \uac83\uacfc \uadf8\uac00 \uc9c1\uc811 \uac10\ucc30\uad00\uc9c1\uc744 \ub9e1\uc544 \uace1\ubb3c \ub2f4\ub2f9\uad00\uc9c1(cura annonae)\ub97c \ub9e1\uc544\uc904\uac83\uc744 \uac15\ub825\ud558\uac8c \uc694\uad6c\ud588\ub2e4. \uc544\uc6b0\uad6c\uc2a4\ud22c\uc2a4\ub294 \ub3c5\uc7ac\uad00\uc5d0 \ucde8\uc784\ud558\uc9c0\ub294 \uc54a\uc558\uc9c0\ub9cc \uc0ac\ud0dc\ub97c \ud574\uacb0\ud558\uae30 \uc704\ud574 \uc790\ube44\ub97c \ubd80\ub2f4\ud558\uc5ec \uc9e7\uc740 \uc2dc\uac04 \ub0b4\uc5d0 \uc0ac\ud0dc\ub97c \ud574\uacb0\ud558\uc600\ub2e4. 8\ub144\uc5d0 \ub2e4\uc2dc \ubc1c\uc0dd\ud55c \uae30\uadfc \ub54c\uc5d0\ub294 \u201c\uc2dd\ub7c9\uccad \uc7a5\uad00\u201d(\ub77c\ud2f4\uc5b4: praefectus annonae)\uc774\ub77c\ub294 \uad00\uc9c1\uc744 \uc2e0\uc124\ud558\uc5ec \ub85c\ub9c8\uc758 \uc2dd\ub7c9 \uacf5\uae09\uc744 \ucc45\uc784\uc9c0\uac8c \ud558\uc600\ub2e4. \uae30\uc6d0\uc804 19\ub144\uc5d0 \uc6d0\ub85c\uc6d0\uc740 \ubbfc\uc911\uc758 \ubd84\ub178\ub97c \uc0ac\uc9c0 \uc54a\uc73c\ub824\uace0 \uc544\uc6b0\uad6c\uc2a4\ud22c\uc2a4\uac00 \uc9d1\uc815\uad00\uc758 \uc0c1\uc9d5\uc744 \uacf5\uc801\uc778 \uc790\ub9ac\uc5d0\uc11c \uc0ac\uc6a9\ud558\ub294 \uad8c\ub9ac\uc640 \ub450 \uc9d1\uc815\uad00\uc758 \uc0ac\uc774\uc5d0 \uc549\uc744 \uc218 \uc788\ub294 \uad8c\ub9ac\ub97c \ud5c8\ub77d\ud588\ub2e4. \uadf8 \uacb0\uacfc\uad8c\ub825\uc740 \uacf5\uc2dd\uc801\uc73c\ub85c\ub3c4 \uc9d1\uc815\uad00\uc5d0 \ucde8\uc784\ud558\uc9c0 \uc54a\ub354\ub77c\ub3c4 \uc9d1\uc815\uad00\ucc98\ub7fc \uc601\ud5a5\ub825\uc744 \ud589\uc0ac\ud560 \uc218 \uc788\ub294 \ub2e8\uacc4\uc5d0 \ub3c4\ub2ec\ud558\uc600\ub2e4.", "answer": "\uace1\ubb3c \ub2f4\ub2f9\uad00\uc9c1", "sentence": "\uc774\ub4e4\uc740 \uc544\uc6b0\uad6c\uc2a4\ud22c\uc2a4\uc5d0\uac8c \uc601\uad6c \uc9d1\uc815\uad00\uc9c1 \ud639\uc740 \ub3c5\uc7ac\uad00\uc9c1\uc744 \ubd80\uc5ec\ud560 \uac83\uacfc \uadf8\uac00 \uc9c1\uc811 \uac10\ucc30\uad00\uc9c1\uc744 \ub9e1\uc544 \uace1\ubb3c \ub2f4\ub2f9\uad00\uc9c1 (cura annonae)\ub97c \ub9e1\uc544\uc904\uac83\uc744 \uac15\ub825\ud558\uac8c \uc694\uad6c\ud588\ub2e4.", "paragraph_sentence": "\uc55e\uc5d0\uc11c \uc5b8\uae09\ud55c \uad8c\ud55c\ub4e4\uc758 \ud68d\ub4dd\uc740 \uc815\uce58\uc801\uc73c\ub85c \ub9e4\uc6b0 \ubbf8\ubb18\ud55c \uc704\uc7a5\uc774\uc5c8\ub2e4. \ud3c9\ubbfc \uacc4\uae09\uc774 \uadf8 \uc758\ubbf8\ub97c \uc81c\ub300\ub85c \uc774\ud574\ud558\uc9c0 \ubabb\ud55c \ub370\uc11c \uc774\ub97c \ubbf8\ub8e8\uc5b4 \uc54c \uc218 \uc788\ub2e4. \uae30\uc6d0\uc804 22\ub144\uc5d0 \ud64d\uc218\uc640 \uae30\uadfc\uc774 \uacb9\uccd0\uc11c \uc77c\ubc18 \uc11c\ubbfc\ub4e4\uc758 \uc0dd\ud65c\uc774 \ud798\ub4e4\uc5b4\uc84c\ub2e4. \uc774\ub4e4\uc740 \uc544\uc6b0\uad6c\uc2a4\ud22c\uc2a4\uc5d0\uac8c \uc601\uad6c \uc9d1\uc815\uad00\uc9c1 \ud639\uc740 \ub3c5\uc7ac\uad00\uc9c1\uc744 \ubd80\uc5ec\ud560 \uac83\uacfc \uadf8\uac00 \uc9c1\uc811 \uac10\ucc30\uad00\uc9c1\uc744 \ub9e1\uc544 \uace1\ubb3c \ub2f4\ub2f9\uad00\uc9c1 (cura annonae)\ub97c \ub9e1\uc544\uc904\uac83\uc744 \uac15\ub825\ud558\uac8c \uc694\uad6c\ud588\ub2e4. \uc544\uc6b0\uad6c\uc2a4\ud22c\uc2a4\ub294 \ub3c5\uc7ac\uad00\uc5d0 \ucde8\uc784\ud558\uc9c0\ub294 \uc54a\uc558\uc9c0\ub9cc \uc0ac\ud0dc\ub97c \ud574\uacb0\ud558\uae30 \uc704\ud574 \uc790\ube44\ub97c \ubd80\ub2f4\ud558\uc5ec \uc9e7\uc740 \uc2dc\uac04 \ub0b4\uc5d0 \uc0ac\ud0dc\ub97c \ud574\uacb0\ud558\uc600\ub2e4. 8\ub144\uc5d0 \ub2e4\uc2dc \ubc1c\uc0dd\ud55c \uae30\uadfc \ub54c\uc5d0\ub294 \u201c\uc2dd\ub7c9\uccad \uc7a5\uad00\u201d(\ub77c\ud2f4\uc5b4: praefectus annonae)\uc774\ub77c\ub294 \uad00\uc9c1\uc744 \uc2e0\uc124\ud558\uc5ec \ub85c\ub9c8\uc758 \uc2dd\ub7c9 \uacf5\uae09\uc744 \ucc45\uc784\uc9c0\uac8c \ud558\uc600\ub2e4. \uae30\uc6d0\uc804 19\ub144\uc5d0 \uc6d0\ub85c\uc6d0\uc740 \ubbfc\uc911\uc758 \ubd84\ub178\ub97c \uc0ac\uc9c0 \uc54a\uc73c\ub824\uace0 \uc544\uc6b0\uad6c\uc2a4\ud22c\uc2a4\uac00 \uc9d1\uc815\uad00\uc758 \uc0c1\uc9d5\uc744 \uacf5\uc801\uc778 \uc790\ub9ac\uc5d0\uc11c \uc0ac\uc6a9\ud558\ub294 \uad8c\ub9ac\uc640 \ub450 \uc9d1\uc815\uad00\uc758 \uc0ac\uc774\uc5d0 \uc549\uc744 \uc218 \uc788\ub294 \uad8c\ub9ac\ub97c \ud5c8\ub77d\ud588\ub2e4. \uadf8 \uacb0\uacfc\uad8c\ub825\uc740 \uacf5\uc2dd\uc801\uc73c\ub85c\ub3c4 \uc9d1\uc815\uad00\uc5d0 \ucde8\uc784\ud558\uc9c0 \uc54a\ub354\ub77c\ub3c4 \uc9d1\uc815\uad00\ucc98\ub7fc \uc601\ud5a5\ub825\uc744 \ud589\uc0ac\ud560 \uc218 \uc788\ub294 \ub2e8\uacc4\uc5d0 \ub3c4\ub2ec\ud558\uc600\ub2e4.", "paragraph_answer": "\uc55e\uc5d0\uc11c \uc5b8\uae09\ud55c \uad8c\ud55c\ub4e4\uc758 \ud68d\ub4dd\uc740 \uc815\uce58\uc801\uc73c\ub85c \ub9e4\uc6b0 \ubbf8\ubb18\ud55c \uc704\uc7a5\uc774\uc5c8\ub2e4. \ud3c9\ubbfc \uacc4\uae09\uc774 \uadf8 \uc758\ubbf8\ub97c \uc81c\ub300\ub85c \uc774\ud574\ud558\uc9c0 \ubabb\ud55c \ub370\uc11c \uc774\ub97c \ubbf8\ub8e8\uc5b4 \uc54c \uc218 \uc788\ub2e4. \uae30\uc6d0\uc804 22\ub144\uc5d0 \ud64d\uc218\uc640 \uae30\uadfc\uc774 \uacb9\uccd0\uc11c \uc77c\ubc18 \uc11c\ubbfc\ub4e4\uc758 \uc0dd\ud65c\uc774 \ud798\ub4e4\uc5b4\uc84c\ub2e4. \uc774\ub4e4\uc740 \uc544\uc6b0\uad6c\uc2a4\ud22c\uc2a4\uc5d0\uac8c \uc601\uad6c \uc9d1\uc815\uad00\uc9c1 \ud639\uc740 \ub3c5\uc7ac\uad00\uc9c1\uc744 \ubd80\uc5ec\ud560 \uac83\uacfc \uadf8\uac00 \uc9c1\uc811 \uac10\ucc30\uad00\uc9c1\uc744 \ub9e1\uc544 \uace1\ubb3c \ub2f4\ub2f9\uad00\uc9c1 (cura annonae)\ub97c \ub9e1\uc544\uc904\uac83\uc744 \uac15\ub825\ud558\uac8c \uc694\uad6c\ud588\ub2e4. \uc544\uc6b0\uad6c\uc2a4\ud22c\uc2a4\ub294 \ub3c5\uc7ac\uad00\uc5d0 \ucde8\uc784\ud558\uc9c0\ub294 \uc54a\uc558\uc9c0\ub9cc \uc0ac\ud0dc\ub97c \ud574\uacb0\ud558\uae30 \uc704\ud574 \uc790\ube44\ub97c \ubd80\ub2f4\ud558\uc5ec \uc9e7\uc740 \uc2dc\uac04 \ub0b4\uc5d0 \uc0ac\ud0dc\ub97c \ud574\uacb0\ud558\uc600\ub2e4. 8\ub144\uc5d0 \ub2e4\uc2dc \ubc1c\uc0dd\ud55c \uae30\uadfc \ub54c\uc5d0\ub294 \u201c\uc2dd\ub7c9\uccad \uc7a5\uad00\u201d(\ub77c\ud2f4\uc5b4: praefectus annonae)\uc774\ub77c\ub294 \uad00\uc9c1\uc744 \uc2e0\uc124\ud558\uc5ec \ub85c\ub9c8\uc758 \uc2dd\ub7c9 \uacf5\uae09\uc744 \ucc45\uc784\uc9c0\uac8c \ud558\uc600\ub2e4. \uae30\uc6d0\uc804 19\ub144\uc5d0 \uc6d0\ub85c\uc6d0\uc740 \ubbfc\uc911\uc758 \ubd84\ub178\ub97c \uc0ac\uc9c0 \uc54a\uc73c\ub824\uace0 \uc544\uc6b0\uad6c\uc2a4\ud22c\uc2a4\uac00 \uc9d1\uc815\uad00\uc758 \uc0c1\uc9d5\uc744 \uacf5\uc801\uc778 \uc790\ub9ac\uc5d0\uc11c \uc0ac\uc6a9\ud558\ub294 \uad8c\ub9ac\uc640 \ub450 \uc9d1\uc815\uad00\uc758 \uc0ac\uc774\uc5d0 \uc549\uc744 \uc218 \uc788\ub294 \uad8c\ub9ac\ub97c \ud5c8\ub77d\ud588\ub2e4. \uadf8 \uacb0\uacfc\uad8c\ub825\uc740 \uacf5\uc2dd\uc801\uc73c\ub85c\ub3c4 \uc9d1\uc815\uad00\uc5d0 \ucde8\uc784\ud558\uc9c0 \uc54a\ub354\ub77c\ub3c4 \uc9d1\uc815\uad00\ucc98\ub7fc \uc601\ud5a5\ub825\uc744 \ud589\uc0ac\ud560 \uc218 \uc788\ub294 \ub2e8\uacc4\uc5d0 \ub3c4\ub2ec\ud558\uc600\ub2e4.", "sentence_answer": "\uc774\ub4e4\uc740 \uc544\uc6b0\uad6c\uc2a4\ud22c\uc2a4\uc5d0\uac8c \uc601\uad6c \uc9d1\uc815\uad00\uc9c1 \ud639\uc740 \ub3c5\uc7ac\uad00\uc9c1\uc744 \ubd80\uc5ec\ud560 \uac83\uacfc \uadf8\uac00 \uc9c1\uc811 \uac10\ucc30\uad00\uc9c1\uc744 \ub9e1\uc544 \uace1\ubb3c \ub2f4\ub2f9\uad00\uc9c1 (cura annonae)\ub97c \ub9e1\uc544\uc904\uac83\uc744 \uac15\ub825\ud558\uac8c \uc694\uad6c\ud588\ub2e4.", "paragraph_id": "6657258-25-0", "paragraph_question": "question: \uae30\uc6d0\uc804 22\ub144, \ud64d\uc218\uc640 \uae30\uadfc\uc73c\ub85c \uc778\ud55c \uc0dd\ud65c\uace0 \ud798\ub4e4\uc5b4\uc9c4 \uc0ac\ub78c\ub4e4\uc774 \uc544\uc6b0\uad6c\uc2a4\ud22c\uc2a4\uc5d0\uac8c \uc81c\uc548\ud55c \uad00\uc9c1\uc740 \ubb34\uc5c7\uc778\uac00?, context: \uc55e\uc5d0\uc11c \uc5b8\uae09\ud55c \uad8c\ud55c\ub4e4\uc758 \ud68d\ub4dd\uc740 \uc815\uce58\uc801\uc73c\ub85c \ub9e4\uc6b0 \ubbf8\ubb18\ud55c \uc704\uc7a5\uc774\uc5c8\ub2e4. \ud3c9\ubbfc \uacc4\uae09\uc774 \uadf8 \uc758\ubbf8\ub97c \uc81c\ub300\ub85c \uc774\ud574\ud558\uc9c0 \ubabb\ud55c \ub370\uc11c \uc774\ub97c \ubbf8\ub8e8\uc5b4 \uc54c \uc218 \uc788\ub2e4. \uae30\uc6d0\uc804 22\ub144\uc5d0 \ud64d\uc218\uc640 \uae30\uadfc\uc774 \uacb9\uccd0\uc11c \uc77c\ubc18 \uc11c\ubbfc\ub4e4\uc758 \uc0dd\ud65c\uc774 \ud798\ub4e4\uc5b4\uc84c\ub2e4. \uc774\ub4e4\uc740 \uc544\uc6b0\uad6c\uc2a4\ud22c\uc2a4\uc5d0\uac8c \uc601\uad6c \uc9d1\uc815\uad00\uc9c1 \ud639\uc740 \ub3c5\uc7ac\uad00\uc9c1\uc744 \ubd80\uc5ec\ud560 \uac83\uacfc \uadf8\uac00 \uc9c1\uc811 \uac10\ucc30\uad00\uc9c1\uc744 \ub9e1\uc544 \uace1\ubb3c \ub2f4\ub2f9\uad00\uc9c1(cura annonae)\ub97c \ub9e1\uc544\uc904\uac83\uc744 \uac15\ub825\ud558\uac8c \uc694\uad6c\ud588\ub2e4. \uc544\uc6b0\uad6c\uc2a4\ud22c\uc2a4\ub294 \ub3c5\uc7ac\uad00\uc5d0 \ucde8\uc784\ud558\uc9c0\ub294 \uc54a\uc558\uc9c0\ub9cc \uc0ac\ud0dc\ub97c \ud574\uacb0\ud558\uae30 \uc704\ud574 \uc790\ube44\ub97c \ubd80\ub2f4\ud558\uc5ec \uc9e7\uc740 \uc2dc\uac04 \ub0b4\uc5d0 \uc0ac\ud0dc\ub97c \ud574\uacb0\ud558\uc600\ub2e4. 8\ub144\uc5d0 \ub2e4\uc2dc \ubc1c\uc0dd\ud55c \uae30\uadfc \ub54c\uc5d0\ub294 \u201c\uc2dd\ub7c9\uccad \uc7a5\uad00\u201d(\ub77c\ud2f4\uc5b4: praefectus annonae)\uc774\ub77c\ub294 \uad00\uc9c1\uc744 \uc2e0\uc124\ud558\uc5ec \ub85c\ub9c8\uc758 \uc2dd\ub7c9 \uacf5\uae09\uc744 \ucc45\uc784\uc9c0\uac8c \ud558\uc600\ub2e4. \uae30\uc6d0\uc804 19\ub144\uc5d0 \uc6d0\ub85c\uc6d0\uc740 \ubbfc\uc911\uc758 \ubd84\ub178\ub97c \uc0ac\uc9c0 \uc54a\uc73c\ub824\uace0 \uc544\uc6b0\uad6c\uc2a4\ud22c\uc2a4\uac00 \uc9d1\uc815\uad00\uc758 \uc0c1\uc9d5\uc744 \uacf5\uc801\uc778 \uc790\ub9ac\uc5d0\uc11c \uc0ac\uc6a9\ud558\ub294 \uad8c\ub9ac\uc640 \ub450 \uc9d1\uc815\uad00\uc758 \uc0ac\uc774\uc5d0 \uc549\uc744 \uc218 \uc788\ub294 \uad8c\ub9ac\ub97c \ud5c8\ub77d\ud588\ub2e4. \uadf8 \uacb0\uacfc\uad8c\ub825\uc740 \uacf5\uc2dd\uc801\uc73c\ub85c\ub3c4 \uc9d1\uc815\uad00\uc5d0 \ucde8\uc784\ud558\uc9c0 \uc54a\ub354\ub77c\ub3c4 \uc9d1\uc815\uad00\ucc98\ub7fc \uc601\ud5a5\ub825\uc744 \ud589\uc0ac\ud560 \uc218 \uc788\ub294 \ub2e8\uacc4\uc5d0 \ub3c4\ub2ec\ud558\uc600\ub2e4."} {"question": "\ud504\ub791\uc2a4 \uce68\uacf5 \uc81c 1\ub2e8\uacc4 \uacc4\ud68d\uc774\uba70 \ub9cc\uc288\ud0c0\uc778\uc774 \ubd88\ub9cc\uc871\uc2a4\ub7ec\uc6cc \ud588\ub358 \uc791\uc804\uc758 \uc774\ub984\uc740?", "paragraph": "\ud504\ub791\uc2a4 \uce68\uacf5 \uc81c1\ub2e8\uacc4 \uacc4\ud68d\uc778 \ud669\uc0c9 \uc791\uc804(Fall Gelb)\uc740 \uc721\uad70\ucd5c\uace0\uc9c0\ud718\uad00 \ubc1c\ud130 \ud3f0 \ube0c\ub77c\uc6b0\ud788\uce58 \uc0c1\uae09\ub300\uc7a5, \uc7a5\uad70\ucc38\ubaa8\ucd1d\uc7a5 \ud560\ub354 \uc0c1\uae08\ub300\uc7a5\uc744 \ube44\ub86f\ud55c \uc721\uad70\ucd5c\uace0\uc0ac\ub839\ubd80(OKH) \uad6c\uc131\uc6d0\ub4e4\uc774 1939\ub144 10\uc6d4 \ucd08\uc5d0 \ub9c8\ub828\ud55c \uac83\uc774\uc5c8\ub2e4. 1\ucc28\ub300\uc804 \ub54c\uc758 \uc290\ub9ac\ud39c \uacc4\ud68d\uc744 \ub2f5\uc2b5\ud55c \ud669\uc0c9 \uc791\uc804\uc740 \ub124\ub35c\ub780\ub4dc\uc640 \ubca8\uae30\uc5d0\ub97c \ud1b5\ud55c \ud3ec\uc704\uacf5\uaca9\uc744 \ud544\uc694\ub85c \ud588\ub2e4. \ud788\ud2c0\ub7ec\ub294 \uc774\uc5d0 \ub9cc\uc871\uc2a4\ub7ec\uc6cc\ud558\uc9c0 \uc54a\uc558\uace0, \ucc38\ubaa8\ub4e4\uc740 10\uc6d4 \ub0b4\ub0b4 \uc791\uc804 \uc218\uc815\uc5d0 \uc2dc\uac04\uc744 \ubcf4\ub0c8\ub2e4. \uc774 \uc791\uc804\uacc4\ud68d\uc774 \ubd88\ub9cc\uc871\uc2a4\ub7fd\uae30\ub294 \ub9cc\uc288\ud0c0\uc778\ub3c4 \ub9c8\ucc2c\uac00\uc9c0\uc600\ub2e4. \ub9cc\uc288\ud0c0\uc778\uc774 \ubcf4\uae30\uc5d0 \ud669\uc0c9 \uc791\uc804\uc740 \ubd81\uc775\uc5d0 \ubcd1\ub825\uc774 \uc9d1\uc911\ub418\uc5b4 \uc788\uc5b4, \uae30\uc2b5 \uacf5\uc2b5\uc758 \ud6a8\uacfc\uac00 \uc801\uace0, \ub0a8\ucabd\uc5d0\uc11c\uc758 \ubc18\uaca9\uc5d0 \ub178\ucd9c\ub420 \uc218 \uc788\ub2e4\uace0 \uc0dd\uac01\ud588\ub2e4. \ubca8\uae30\uc5d0\uc758 \uc9c0\ud615\uc740 \ud504\ub791\uc2a4 \uacf5\uaca9\uc744 \uc704\ud55c \uc791\uc804\uae30\uc9c0\ub85c \uc4f0\uae30\uc5d0 \ubd80\uc801\ud569\ud588\uace0, \ub9cc\uc288\ud0c0\uc778\uc740 \ud669\uc0c9 \uc791\uc804\uc774 \uc81c1\ucc28 \uc138\uacc4 \ub300\uc804 \ub54c \uadf8\ub7ac\ub358 \uac83\ucc98\ub7fc \uc801\uc758 \uc12c\uba78\uc5d0 \uc2e4\ud328\ud558\uace0 \uc77c\ubd80\uc758 \uc131\uacf5\ub9cc \uac70\ub454 \ub05d\uc5d0 \uc9c0\ub9ac\ud55c \ucc38\ud638\uc804\uc73c\ub85c \uc218\ub834\ud560 \uac83\uc774\ub77c\uace0 \ubcf4\uc558\ub2e4. 10\uc6d4 \ub9d0 \ub9cc\uc288\ud0c0\uc778\uc740 \ub530\ub85c \uc791\uc804\ubc29\uce68\uc744 \uc900\ube44\ud558\uc5ec \uc0c1\uad00 \ub8ec\ud2b8\uc288\ud14c\ud2b8\ub97c \ud1b5\ud574 OKH\uc5d0 \uacc4\ud68d\uc548\uc744 \uc81c\ucd9c\ud588\ub2e4.", "answer": "\ud669\uc0c9 \uc791\uc804", "sentence": "\ud504\ub791\uc2a4 \uce68\uacf5 \uc81c1\ub2e8\uacc4 \uacc4\ud68d\uc778 \ud669\uc0c9 \uc791\uc804 (Fall Gelb)", "paragraph_sentence": " \ud504\ub791\uc2a4 \uce68\uacf5 \uc81c1\ub2e8\uacc4 \uacc4\ud68d\uc778 \ud669\uc0c9 \uc791\uc804 (Fall Gelb) \uc740 \uc721\uad70\ucd5c\uace0\uc9c0\ud718\uad00 \ubc1c\ud130 \ud3f0 \ube0c\ub77c\uc6b0\ud788\uce58 \uc0c1\uae09\ub300\uc7a5, \uc7a5\uad70\ucc38\ubaa8\ucd1d\uc7a5 \ud560\ub354 \uc0c1\uae08\ub300\uc7a5\uc744 \ube44\ub86f\ud55c \uc721\uad70\ucd5c\uace0\uc0ac\ub839\ubd80(OKH) \uad6c\uc131\uc6d0\ub4e4\uc774 1939\ub144 10\uc6d4 \ucd08\uc5d0 \ub9c8\ub828\ud55c \uac83\uc774\uc5c8\ub2e4. 1\ucc28\ub300\uc804 \ub54c\uc758 \uc290\ub9ac\ud39c \uacc4\ud68d\uc744 \ub2f5\uc2b5\ud55c \ud669\uc0c9 \uc791\uc804\uc740 \ub124\ub35c\ub780\ub4dc\uc640 \ubca8\uae30\uc5d0\ub97c \ud1b5\ud55c \ud3ec\uc704\uacf5\uaca9\uc744 \ud544\uc694\ub85c \ud588\ub2e4. \ud788\ud2c0\ub7ec\ub294 \uc774\uc5d0 \ub9cc\uc871\uc2a4\ub7ec\uc6cc\ud558\uc9c0 \uc54a\uc558\uace0, \ucc38\ubaa8\ub4e4\uc740 10\uc6d4 \ub0b4\ub0b4 \uc791\uc804 \uc218\uc815\uc5d0 \uc2dc\uac04\uc744 \ubcf4\ub0c8\ub2e4. \uc774 \uc791\uc804\uacc4\ud68d\uc774 \ubd88\ub9cc\uc871\uc2a4\ub7fd\uae30\ub294 \ub9cc\uc288\ud0c0\uc778\ub3c4 \ub9c8\ucc2c\uac00\uc9c0\uc600\ub2e4. \ub9cc\uc288\ud0c0\uc778\uc774 \ubcf4\uae30\uc5d0 \ud669\uc0c9 \uc791\uc804\uc740 \ubd81\uc775\uc5d0 \ubcd1\ub825\uc774 \uc9d1\uc911\ub418\uc5b4 \uc788\uc5b4, \uae30\uc2b5 \uacf5\uc2b5\uc758 \ud6a8\uacfc\uac00 \uc801\uace0, \ub0a8\ucabd\uc5d0\uc11c\uc758 \ubc18\uaca9\uc5d0 \ub178\ucd9c\ub420 \uc218 \uc788\ub2e4\uace0 \uc0dd\uac01\ud588\ub2e4. \ubca8\uae30\uc5d0\uc758 \uc9c0\ud615\uc740 \ud504\ub791\uc2a4 \uacf5\uaca9\uc744 \uc704\ud55c \uc791\uc804\uae30\uc9c0\ub85c \uc4f0\uae30\uc5d0 \ubd80\uc801\ud569\ud588\uace0, \ub9cc\uc288\ud0c0\uc778\uc740 \ud669\uc0c9 \uc791\uc804\uc774 \uc81c1\ucc28 \uc138\uacc4 \ub300\uc804 \ub54c \uadf8\ub7ac\ub358 \uac83\ucc98\ub7fc \uc801\uc758 \uc12c\uba78\uc5d0 \uc2e4\ud328\ud558\uace0 \uc77c\ubd80\uc758 \uc131\uacf5\ub9cc \uac70\ub454 \ub05d\uc5d0 \uc9c0\ub9ac\ud55c \ucc38\ud638\uc804\uc73c\ub85c \uc218\ub834\ud560 \uac83\uc774\ub77c\uace0 \ubcf4\uc558\ub2e4. 10\uc6d4 \ub9d0 \ub9cc\uc288\ud0c0\uc778\uc740 \ub530\ub85c \uc791\uc804\ubc29\uce68\uc744 \uc900\ube44\ud558\uc5ec \uc0c1\uad00 \ub8ec\ud2b8\uc288\ud14c\ud2b8\ub97c \ud1b5\ud574 OKH\uc5d0 \uacc4\ud68d\uc548\uc744 \uc81c\ucd9c\ud588\ub2e4.", "paragraph_answer": "\ud504\ub791\uc2a4 \uce68\uacf5 \uc81c1\ub2e8\uacc4 \uacc4\ud68d\uc778 \ud669\uc0c9 \uc791\uc804 (Fall Gelb)\uc740 \uc721\uad70\ucd5c\uace0\uc9c0\ud718\uad00 \ubc1c\ud130 \ud3f0 \ube0c\ub77c\uc6b0\ud788\uce58 \uc0c1\uae09\ub300\uc7a5, \uc7a5\uad70\ucc38\ubaa8\ucd1d\uc7a5 \ud560\ub354 \uc0c1\uae08\ub300\uc7a5\uc744 \ube44\ub86f\ud55c \uc721\uad70\ucd5c\uace0\uc0ac\ub839\ubd80(OKH) \uad6c\uc131\uc6d0\ub4e4\uc774 1939\ub144 10\uc6d4 \ucd08\uc5d0 \ub9c8\ub828\ud55c \uac83\uc774\uc5c8\ub2e4. 1\ucc28\ub300\uc804 \ub54c\uc758 \uc290\ub9ac\ud39c \uacc4\ud68d\uc744 \ub2f5\uc2b5\ud55c \ud669\uc0c9 \uc791\uc804\uc740 \ub124\ub35c\ub780\ub4dc\uc640 \ubca8\uae30\uc5d0\ub97c \ud1b5\ud55c \ud3ec\uc704\uacf5\uaca9\uc744 \ud544\uc694\ub85c \ud588\ub2e4. \ud788\ud2c0\ub7ec\ub294 \uc774\uc5d0 \ub9cc\uc871\uc2a4\ub7ec\uc6cc\ud558\uc9c0 \uc54a\uc558\uace0, \ucc38\ubaa8\ub4e4\uc740 10\uc6d4 \ub0b4\ub0b4 \uc791\uc804 \uc218\uc815\uc5d0 \uc2dc\uac04\uc744 \ubcf4\ub0c8\ub2e4. \uc774 \uc791\uc804\uacc4\ud68d\uc774 \ubd88\ub9cc\uc871\uc2a4\ub7fd\uae30\ub294 \ub9cc\uc288\ud0c0\uc778\ub3c4 \ub9c8\ucc2c\uac00\uc9c0\uc600\ub2e4. \ub9cc\uc288\ud0c0\uc778\uc774 \ubcf4\uae30\uc5d0 \ud669\uc0c9 \uc791\uc804\uc740 \ubd81\uc775\uc5d0 \ubcd1\ub825\uc774 \uc9d1\uc911\ub418\uc5b4 \uc788\uc5b4, \uae30\uc2b5 \uacf5\uc2b5\uc758 \ud6a8\uacfc\uac00 \uc801\uace0, \ub0a8\ucabd\uc5d0\uc11c\uc758 \ubc18\uaca9\uc5d0 \ub178\ucd9c\ub420 \uc218 \uc788\ub2e4\uace0 \uc0dd\uac01\ud588\ub2e4. \ubca8\uae30\uc5d0\uc758 \uc9c0\ud615\uc740 \ud504\ub791\uc2a4 \uacf5\uaca9\uc744 \uc704\ud55c \uc791\uc804\uae30\uc9c0\ub85c \uc4f0\uae30\uc5d0 \ubd80\uc801\ud569\ud588\uace0, \ub9cc\uc288\ud0c0\uc778\uc740 \ud669\uc0c9 \uc791\uc804\uc774 \uc81c1\ucc28 \uc138\uacc4 \ub300\uc804 \ub54c \uadf8\ub7ac\ub358 \uac83\ucc98\ub7fc \uc801\uc758 \uc12c\uba78\uc5d0 \uc2e4\ud328\ud558\uace0 \uc77c\ubd80\uc758 \uc131\uacf5\ub9cc \uac70\ub454 \ub05d\uc5d0 \uc9c0\ub9ac\ud55c \ucc38\ud638\uc804\uc73c\ub85c \uc218\ub834\ud560 \uac83\uc774\ub77c\uace0 \ubcf4\uc558\ub2e4. 10\uc6d4 \ub9d0 \ub9cc\uc288\ud0c0\uc778\uc740 \ub530\ub85c \uc791\uc804\ubc29\uce68\uc744 \uc900\ube44\ud558\uc5ec \uc0c1\uad00 \ub8ec\ud2b8\uc288\ud14c\ud2b8\ub97c \ud1b5\ud574 OKH\uc5d0 \uacc4\ud68d\uc548\uc744 \uc81c\ucd9c\ud588\ub2e4.", "sentence_answer": "\ud504\ub791\uc2a4 \uce68\uacf5 \uc81c1\ub2e8\uacc4 \uacc4\ud68d\uc778 \ud669\uc0c9 \uc791\uc804 (Fall Gelb)", "paragraph_id": "6540675-12-0", "paragraph_question": "question: \ud504\ub791\uc2a4 \uce68\uacf5 \uc81c 1\ub2e8\uacc4 \uacc4\ud68d\uc774\uba70 \ub9cc\uc288\ud0c0\uc778\uc774 \ubd88\ub9cc\uc871\uc2a4\ub7ec\uc6cc \ud588\ub358 \uc791\uc804\uc758 \uc774\ub984\uc740?, context: \ud504\ub791\uc2a4 \uce68\uacf5 \uc81c1\ub2e8\uacc4 \uacc4\ud68d\uc778 \ud669\uc0c9 \uc791\uc804(Fall Gelb)\uc740 \uc721\uad70\ucd5c\uace0\uc9c0\ud718\uad00 \ubc1c\ud130 \ud3f0 \ube0c\ub77c\uc6b0\ud788\uce58 \uc0c1\uae09\ub300\uc7a5, \uc7a5\uad70\ucc38\ubaa8\ucd1d\uc7a5 \ud560\ub354 \uc0c1\uae08\ub300\uc7a5\uc744 \ube44\ub86f\ud55c \uc721\uad70\ucd5c\uace0\uc0ac\ub839\ubd80(OKH) \uad6c\uc131\uc6d0\ub4e4\uc774 1939\ub144 10\uc6d4 \ucd08\uc5d0 \ub9c8\ub828\ud55c \uac83\uc774\uc5c8\ub2e4. 1\ucc28\ub300\uc804 \ub54c\uc758 \uc290\ub9ac\ud39c \uacc4\ud68d\uc744 \ub2f5\uc2b5\ud55c \ud669\uc0c9 \uc791\uc804\uc740 \ub124\ub35c\ub780\ub4dc\uc640 \ubca8\uae30\uc5d0\ub97c \ud1b5\ud55c \ud3ec\uc704\uacf5\uaca9\uc744 \ud544\uc694\ub85c \ud588\ub2e4. \ud788\ud2c0\ub7ec\ub294 \uc774\uc5d0 \ub9cc\uc871\uc2a4\ub7ec\uc6cc\ud558\uc9c0 \uc54a\uc558\uace0, \ucc38\ubaa8\ub4e4\uc740 10\uc6d4 \ub0b4\ub0b4 \uc791\uc804 \uc218\uc815\uc5d0 \uc2dc\uac04\uc744 \ubcf4\ub0c8\ub2e4. \uc774 \uc791\uc804\uacc4\ud68d\uc774 \ubd88\ub9cc\uc871\uc2a4\ub7fd\uae30\ub294 \ub9cc\uc288\ud0c0\uc778\ub3c4 \ub9c8\ucc2c\uac00\uc9c0\uc600\ub2e4. \ub9cc\uc288\ud0c0\uc778\uc774 \ubcf4\uae30\uc5d0 \ud669\uc0c9 \uc791\uc804\uc740 \ubd81\uc775\uc5d0 \ubcd1\ub825\uc774 \uc9d1\uc911\ub418\uc5b4 \uc788\uc5b4, \uae30\uc2b5 \uacf5\uc2b5\uc758 \ud6a8\uacfc\uac00 \uc801\uace0, \ub0a8\ucabd\uc5d0\uc11c\uc758 \ubc18\uaca9\uc5d0 \ub178\ucd9c\ub420 \uc218 \uc788\ub2e4\uace0 \uc0dd\uac01\ud588\ub2e4. \ubca8\uae30\uc5d0\uc758 \uc9c0\ud615\uc740 \ud504\ub791\uc2a4 \uacf5\uaca9\uc744 \uc704\ud55c \uc791\uc804\uae30\uc9c0\ub85c \uc4f0\uae30\uc5d0 \ubd80\uc801\ud569\ud588\uace0, \ub9cc\uc288\ud0c0\uc778\uc740 \ud669\uc0c9 \uc791\uc804\uc774 \uc81c1\ucc28 \uc138\uacc4 \ub300\uc804 \ub54c \uadf8\ub7ac\ub358 \uac83\ucc98\ub7fc \uc801\uc758 \uc12c\uba78\uc5d0 \uc2e4\ud328\ud558\uace0 \uc77c\ubd80\uc758 \uc131\uacf5\ub9cc \uac70\ub454 \ub05d\uc5d0 \uc9c0\ub9ac\ud55c \ucc38\ud638\uc804\uc73c\ub85c \uc218\ub834\ud560 \uac83\uc774\ub77c\uace0 \ubcf4\uc558\ub2e4. 10\uc6d4 \ub9d0 \ub9cc\uc288\ud0c0\uc778\uc740 \ub530\ub85c \uc791\uc804\ubc29\uce68\uc744 \uc900\ube44\ud558\uc5ec \uc0c1\uad00 \ub8ec\ud2b8\uc288\ud14c\ud2b8\ub97c \ud1b5\ud574 OKH\uc5d0 \uacc4\ud68d\uc548\uc744 \uc81c\ucd9c\ud588\ub2e4."} {"question": "\uc2dc\ub9ac\uc544\uac00 \ub3c5\ub9bd\ud558\uae30 \uba87 \ub2ec \uc804\uc5d0 \ub7ec\uc2dc\uc544\ub294 \uc5b4\ub5a4 \uc9c0\uc5ed\uc5d0 \uc815\uaddc \ubd80\ub300 \ucc3d\uc124\uacfc \ubb34\uae30\ub97c \uc9c0\uc6d0\ud558\uaca0\ub2e4\uace0 \uc57d\uc18d\ud569\ub2c8\uae4c?", "paragraph": "\ub0c9\uc804 \uc2dc\uae30\ubd80\ud130 \ub7ec\uc2dc\uc544\ub294 \uc2dc\ub9ac\uc544, \uc774\ub780, \uc774\ub77c\ud06c\uc640 \ubc00\uc811\ud55c \ub3d9\ub9f9 \uad00\uacc4\ub97c \uc720\uc9c0\ud588\ub2e4. \ub2f9\uc2dc \uc18c\ube44\uc5d0\ud2b8 \uc5f0\ubc29\uc774\uc5c8\ub358 \ub7ec\uc2dc\uc544\ub294 \ubbf8\uad6d\uc5d0 \ub300\ud56d\ud558\uae30 \uc704\ud574 \uc774\ub4e4\uc744 \uc804\ub7b5\uc801 \ub3d9\ub9f9\uad6d\uc73c\ub85c \uc0bc\uace0\uc790 \ub178\ub825\ud588\ub2e4. \ub300\ud45c\uc801\uc778 \uc608\ub85c \uc2dc\ub9ac\uc544\uac00 \ub3c5\ub9bd\ud558\uae30 \uba87 \ub2ec \uc804\uc778 1946\ub144 2\uc6d4 10\uc77c, \ub7ec\uc2dc\uc544\ub294 \uc2dc\ub9ac\uc544\uc5d0 \uc815\uaddc \ubd80\ub300 \ucc3d\uc124 \ubc0f \ubb34\uae30\ub97c \uc9c0\uc6d0\ud574\uc8fc\uaca0\ub2e4\uace0 \uc57d\uc18d\ud558\uc600\uace0, \uc218\uc5d0\uc988 \uc804\uc7c1 \uc774\ud6c4\uc5d0\ub294 \uc18c\ube44\uc5d0\ud2b8-\uc2dc\ub9ac\uc544 \uac04\uc5d0 \uc218\ub9ce\uc740 \uad70\uc0ac \ubc0f \uacbd\uc81c \ud611\uc815\uc774 \uccb4\uacb0\ub418\uc5c8\ub2e4. 1970\ub144\ub300\uc5d0 \uc18c\ub828\uc740 \uc54c\uc544\uc0ac\ub4dc \uac00\ubb38\uc744 \uc9c0\uc6d0\ud558\uc5ec \uc2dc\ub9ac\uc544\ub97c \uc790\uad6d\uc758 \ub3d9\ub9f9\uad6d\uc73c\ub85c \uc790\ub9ac\ub9e4\uae40\ud558\ub294\ub370 \uae30\uc5ec\ud588\ub2e4. \uc774\ub780\uc758 \uacbd\uc6b0\ub3c4 \ube44\uc2b7\ud588\ub2e4. 1979\ub144 \uc774\ub780 \ud601\uba85 \ubc1c\uc0dd \uc774\ud6c4 \uc0c8\ub85c \ud0c4\uc0dd\ud55c \uc774\ub780 \uc774\uc2ac\ub78c \uacf5\ud654\uad6d\uc744 \ucc98\uc74c \uc778\uc815\ud55c \uad6d\uac00\ub294 \uc18c\ub828\uc774\uc5c8\ub2e4. 1980\ub144 \uc774\ub780-\uc774\ub77c\ud06c \uc804\uc7c1 \ub2f9\uc2dc \uc18c\ub828\uc740 \uc774\ub77c\ud06c\uc5d0 \uc801\uadf9\uc801\uc778 \uc9c0\uc6d0\uc744 \ud558\uc5ec \uc774\ub780\uacfc\uc758 \uad00\uacc4\ub294 \ub0c9\uac01\ub418\uc5c8\uc9c0\ub9cc, \uc18c\ub828 \ubab0\ub77d \uc774\ud6c4 \uc591\uad6d\uc758 \uad00\uacc4\ub294 \ub2e4\uc2dc \ubc1c\uc804\ud558\uc600\uace0, 1990\ub144\ub300 \uc911\ubc18\uc5d0\ub294 \uc774\ub780\uc758 \ud575 \uac1c\ubc1c \ud504\ub85c\uadf8\ub7a8\uc744 \uc9c0\uc6d0\ud558\uc600\ub2e4. \uc774\ub780\uc740 \ub7ec\uc2dc\uc544 \uc8fc\ub3c4\uc758 \uc9d1\ub2e8 \uc548\ubcf4 \uc870\uc57d \uae30\uad6c\uc758 \ucd08\uccad\uad6d\uc774\uba70, \uc0c1\ud558\uc774 \ud611\ub825 \uae30\uad6c\uc758 \uc900\ud68c\uc6d0\uad6d\uc774\uae30\ub3c4 \ud558\ub2e4. \uc774\ub77c\ud06c\uc758 \uacbd\uc6b0, 1972\ub144 \uc774\uc9d1\ud2b8\uac00 \uc18c\ub828\uad70\uc758 \ucca0\uc218\ub97c \uc694\uad6c\ud558\uba74\uc11c \uc591\uad6d \uac04\uc758 \uad00\uacc4\uac00 \ubc1c\uc804\ud558\uae30 \uc2dc\uc791\ud558\uc600\ub2e4. \uc774\ub77c\ud06c\uc640 \uc18c\ub828\uc740 \uac19\uc740 \ud574 \uc6b0\ubc29\uad6d \ub3d9\ub9f9 \uc870\uc57d\uc744 \uccb4\uacb0\ud558\uc5ec \ud0c0\uad6d\uc73c\ub85c\ubd80\ud130\uc758 \uc704\ud611\uc774 \uc788\uc744 \uc2dc \uc0c1\ud638 \uac04 \ubcf4\ud638\ub97c \uc57d\uc18d\ud588\ub2e4. \uc774\ub7ec\ud55c \uc774\uc720\ub85c \uc774\ub780-\uc774\ub77c\ud06c \uc804\uc7c1 \ub2f9\uc2dc \uc18c\ub828\uc740 \uc774\ub77c\ud06c\ub97c \uc9c0\uc6d0\ud588\ub2e4. 1990\ub144 \uac78\ud504 \uc804\uc7c1 \ubc1c\ubc1c \ub2f9\uc2dc \uc18c\ub828\uc740 \ubbf8\uad6d \uc8fc\ub3c4\uc758 \uc5f0\ud569\uad70\uc5d0 \uc9c0\uc6d0\uc744 \ud558\uc5ec \uc774\ub77c\ud06c\uc640\uc758 \uad00\uacc4\ub294 \ub0c9\uac01\ub418\uc5c8\uc73c\ub098, \uc18c\ub828 \ubab0\ub77d \uc774\ud6c4 \uc591\uad6d\uc758 \uad00\uacc4\ub294 \ubc1c\uc804\ud588\ub2e4. \ub7ec\uc2dc\uc544\ub294 \uac78\ud504 \uc804\uc7c1 \uc774\ud6c4 \uc774\ub77c\ud06c\uc5d0 \ub300\ud55c \uc81c\uc81c \ubc0f 2003\ub144 \ubbf8\uad6d\uc758 \uc774\ub77c\ud06c \uce68\uacf5\uc5d0 \ub300\ud574 \ubc18\ub300\ud588\uc73c\uba70, \uc774\ub77c\ud06c\uc640 \ub7ec\uc2dc\uc544 \uc591\uad6d \ubaa8\ub450 \uc774\uc2a4\ub77c\uc5d8\uc758 \uac00\uc790 \uc804\uc7c1\uc5d0 \ubc18\ub300 \ubaa9\uc18c\ub9ac\ub97c \ub0c8\ub2e4. \uc774\ub7ec\ud55c \ub7ec\uc2dc\uc544\uc640 \uc911\ub3d9 \uad6d\uac00 \uac04\uc758 \uad00\uacc4\ub294 \ub3d9\ub9f9\uc758 \ucc3d\ub9bd\uc5d0 \ud070 \uae30\uc5ec\ub97c \ud588\ub2e4\uace0 \ubcfc \uc218 \uc788\ub2e4.", "answer": "\uc2dc\ub9ac\uc544", "sentence": "\ub7ec\uc2dc\uc544\ub294 \uc2dc\ub9ac\uc544 ,", "paragraph_sentence": "\ub0c9\uc804 \uc2dc\uae30\ubd80\ud130 \ub7ec\uc2dc\uc544\ub294 \uc2dc\ub9ac\uc544 , \uc774\ub780, \uc774\ub77c\ud06c\uc640 \ubc00\uc811\ud55c \ub3d9\ub9f9 \uad00\uacc4\ub97c \uc720\uc9c0\ud588\ub2e4. \ub2f9\uc2dc \uc18c\ube44\uc5d0\ud2b8 \uc5f0\ubc29\uc774\uc5c8\ub358 \ub7ec\uc2dc\uc544\ub294 \ubbf8\uad6d\uc5d0 \ub300\ud56d\ud558\uae30 \uc704\ud574 \uc774\ub4e4\uc744 \uc804\ub7b5\uc801 \ub3d9\ub9f9\uad6d\uc73c\ub85c \uc0bc\uace0\uc790 \ub178\ub825\ud588\ub2e4. \ub300\ud45c\uc801\uc778 \uc608\ub85c \uc2dc\ub9ac\uc544\uac00 \ub3c5\ub9bd\ud558\uae30 \uba87 \ub2ec \uc804\uc778 1946\ub144 2\uc6d4 10\uc77c, \ub7ec\uc2dc\uc544\ub294 \uc2dc\ub9ac\uc544\uc5d0 \uc815\uaddc \ubd80\ub300 \ucc3d\uc124 \ubc0f \ubb34\uae30\ub97c \uc9c0\uc6d0\ud574\uc8fc\uaca0\ub2e4\uace0 \uc57d\uc18d\ud558\uc600\uace0, \uc218\uc5d0\uc988 \uc804\uc7c1 \uc774\ud6c4\uc5d0\ub294 \uc18c\ube44\uc5d0\ud2b8-\uc2dc\ub9ac\uc544 \uac04\uc5d0 \uc218\ub9ce\uc740 \uad70\uc0ac \ubc0f \uacbd\uc81c \ud611\uc815\uc774 \uccb4\uacb0\ub418\uc5c8\ub2e4. 1970\ub144\ub300\uc5d0 \uc18c\ub828\uc740 \uc54c\uc544\uc0ac\ub4dc \uac00\ubb38\uc744 \uc9c0\uc6d0\ud558\uc5ec \uc2dc\ub9ac\uc544\ub97c \uc790\uad6d\uc758 \ub3d9\ub9f9\uad6d\uc73c\ub85c \uc790\ub9ac\ub9e4\uae40\ud558\ub294\ub370 \uae30\uc5ec\ud588\ub2e4. \uc774\ub780\uc758 \uacbd\uc6b0\ub3c4 \ube44\uc2b7\ud588\ub2e4. 1979\ub144 \uc774\ub780 \ud601\uba85 \ubc1c\uc0dd \uc774\ud6c4 \uc0c8\ub85c \ud0c4\uc0dd\ud55c \uc774\ub780 \uc774\uc2ac\ub78c \uacf5\ud654\uad6d\uc744 \ucc98\uc74c \uc778\uc815\ud55c \uad6d\uac00\ub294 \uc18c\ub828\uc774\uc5c8\ub2e4. 1980\ub144 \uc774\ub780-\uc774\ub77c\ud06c \uc804\uc7c1 \ub2f9\uc2dc \uc18c\ub828\uc740 \uc774\ub77c\ud06c\uc5d0 \uc801\uadf9\uc801\uc778 \uc9c0\uc6d0\uc744 \ud558\uc5ec \uc774\ub780\uacfc\uc758 \uad00\uacc4\ub294 \ub0c9\uac01\ub418\uc5c8\uc9c0\ub9cc, \uc18c\ub828 \ubab0\ub77d \uc774\ud6c4 \uc591\uad6d\uc758 \uad00\uacc4\ub294 \ub2e4\uc2dc \ubc1c\uc804\ud558\uc600\uace0, 1990\ub144\ub300 \uc911\ubc18\uc5d0\ub294 \uc774\ub780\uc758 \ud575 \uac1c\ubc1c \ud504\ub85c\uadf8\ub7a8\uc744 \uc9c0\uc6d0\ud558\uc600\ub2e4. \uc774\ub780\uc740 \ub7ec\uc2dc\uc544 \uc8fc\ub3c4\uc758 \uc9d1\ub2e8 \uc548\ubcf4 \uc870\uc57d \uae30\uad6c\uc758 \ucd08\uccad\uad6d\uc774\uba70, \uc0c1\ud558\uc774 \ud611\ub825 \uae30\uad6c\uc758 \uc900\ud68c\uc6d0\uad6d\uc774\uae30\ub3c4 \ud558\ub2e4. \uc774\ub77c\ud06c\uc758 \uacbd\uc6b0, 1972\ub144 \uc774\uc9d1\ud2b8\uac00 \uc18c\ub828\uad70\uc758 \ucca0\uc218\ub97c \uc694\uad6c\ud558\uba74\uc11c \uc591\uad6d \uac04\uc758 \uad00\uacc4\uac00 \ubc1c\uc804\ud558\uae30 \uc2dc\uc791\ud558\uc600\ub2e4. \uc774\ub77c\ud06c\uc640 \uc18c\ub828\uc740 \uac19\uc740 \ud574 \uc6b0\ubc29\uad6d \ub3d9\ub9f9 \uc870\uc57d\uc744 \uccb4\uacb0\ud558\uc5ec \ud0c0\uad6d\uc73c\ub85c\ubd80\ud130\uc758 \uc704\ud611\uc774 \uc788\uc744 \uc2dc \uc0c1\ud638 \uac04 \ubcf4\ud638\ub97c \uc57d\uc18d\ud588\ub2e4. \uc774\ub7ec\ud55c \uc774\uc720\ub85c \uc774\ub780-\uc774\ub77c\ud06c \uc804\uc7c1 \ub2f9\uc2dc \uc18c\ub828\uc740 \uc774\ub77c\ud06c\ub97c \uc9c0\uc6d0\ud588\ub2e4. 1990\ub144 \uac78\ud504 \uc804\uc7c1 \ubc1c\ubc1c \ub2f9\uc2dc \uc18c\ub828\uc740 \ubbf8\uad6d \uc8fc\ub3c4\uc758 \uc5f0\ud569\uad70\uc5d0 \uc9c0\uc6d0\uc744 \ud558\uc5ec \uc774\ub77c\ud06c\uc640\uc758 \uad00\uacc4\ub294 \ub0c9\uac01\ub418\uc5c8\uc73c\ub098, \uc18c\ub828 \ubab0\ub77d \uc774\ud6c4 \uc591\uad6d\uc758 \uad00\uacc4\ub294 \ubc1c\uc804\ud588\ub2e4. \ub7ec\uc2dc\uc544\ub294 \uac78\ud504 \uc804\uc7c1 \uc774\ud6c4 \uc774\ub77c\ud06c\uc5d0 \ub300\ud55c \uc81c\uc81c \ubc0f 2003\ub144 \ubbf8\uad6d\uc758 \uc774\ub77c\ud06c \uce68\uacf5\uc5d0 \ub300\ud574 \ubc18\ub300\ud588\uc73c\uba70, \uc774\ub77c\ud06c\uc640 \ub7ec\uc2dc\uc544 \uc591\uad6d \ubaa8\ub450 \uc774\uc2a4\ub77c\uc5d8\uc758 \uac00\uc790 \uc804\uc7c1\uc5d0 \ubc18\ub300 \ubaa9\uc18c\ub9ac\ub97c \ub0c8\ub2e4. \uc774\ub7ec\ud55c \ub7ec\uc2dc\uc544\uc640 \uc911\ub3d9 \uad6d\uac00 \uac04\uc758 \uad00\uacc4\ub294 \ub3d9\ub9f9\uc758 \ucc3d\ub9bd\uc5d0 \ud070 \uae30\uc5ec\ub97c \ud588\ub2e4\uace0 \ubcfc \uc218 \uc788\ub2e4.", "paragraph_answer": "\ub0c9\uc804 \uc2dc\uae30\ubd80\ud130 \ub7ec\uc2dc\uc544\ub294 \uc2dc\ub9ac\uc544 , \uc774\ub780, \uc774\ub77c\ud06c\uc640 \ubc00\uc811\ud55c \ub3d9\ub9f9 \uad00\uacc4\ub97c \uc720\uc9c0\ud588\ub2e4. \ub2f9\uc2dc \uc18c\ube44\uc5d0\ud2b8 \uc5f0\ubc29\uc774\uc5c8\ub358 \ub7ec\uc2dc\uc544\ub294 \ubbf8\uad6d\uc5d0 \ub300\ud56d\ud558\uae30 \uc704\ud574 \uc774\ub4e4\uc744 \uc804\ub7b5\uc801 \ub3d9\ub9f9\uad6d\uc73c\ub85c \uc0bc\uace0\uc790 \ub178\ub825\ud588\ub2e4. \ub300\ud45c\uc801\uc778 \uc608\ub85c \uc2dc\ub9ac\uc544\uac00 \ub3c5\ub9bd\ud558\uae30 \uba87 \ub2ec \uc804\uc778 1946\ub144 2\uc6d4 10\uc77c, \ub7ec\uc2dc\uc544\ub294 \uc2dc\ub9ac\uc544\uc5d0 \uc815\uaddc \ubd80\ub300 \ucc3d\uc124 \ubc0f \ubb34\uae30\ub97c \uc9c0\uc6d0\ud574\uc8fc\uaca0\ub2e4\uace0 \uc57d\uc18d\ud558\uc600\uace0, \uc218\uc5d0\uc988 \uc804\uc7c1 \uc774\ud6c4\uc5d0\ub294 \uc18c\ube44\uc5d0\ud2b8-\uc2dc\ub9ac\uc544 \uac04\uc5d0 \uc218\ub9ce\uc740 \uad70\uc0ac \ubc0f \uacbd\uc81c \ud611\uc815\uc774 \uccb4\uacb0\ub418\uc5c8\ub2e4. 1970\ub144\ub300\uc5d0 \uc18c\ub828\uc740 \uc54c\uc544\uc0ac\ub4dc \uac00\ubb38\uc744 \uc9c0\uc6d0\ud558\uc5ec \uc2dc\ub9ac\uc544\ub97c \uc790\uad6d\uc758 \ub3d9\ub9f9\uad6d\uc73c\ub85c \uc790\ub9ac\ub9e4\uae40\ud558\ub294\ub370 \uae30\uc5ec\ud588\ub2e4. \uc774\ub780\uc758 \uacbd\uc6b0\ub3c4 \ube44\uc2b7\ud588\ub2e4. 1979\ub144 \uc774\ub780 \ud601\uba85 \ubc1c\uc0dd \uc774\ud6c4 \uc0c8\ub85c \ud0c4\uc0dd\ud55c \uc774\ub780 \uc774\uc2ac\ub78c \uacf5\ud654\uad6d\uc744 \ucc98\uc74c \uc778\uc815\ud55c \uad6d\uac00\ub294 \uc18c\ub828\uc774\uc5c8\ub2e4. 1980\ub144 \uc774\ub780-\uc774\ub77c\ud06c \uc804\uc7c1 \ub2f9\uc2dc \uc18c\ub828\uc740 \uc774\ub77c\ud06c\uc5d0 \uc801\uadf9\uc801\uc778 \uc9c0\uc6d0\uc744 \ud558\uc5ec \uc774\ub780\uacfc\uc758 \uad00\uacc4\ub294 \ub0c9\uac01\ub418\uc5c8\uc9c0\ub9cc, \uc18c\ub828 \ubab0\ub77d \uc774\ud6c4 \uc591\uad6d\uc758 \uad00\uacc4\ub294 \ub2e4\uc2dc \ubc1c\uc804\ud558\uc600\uace0, 1990\ub144\ub300 \uc911\ubc18\uc5d0\ub294 \uc774\ub780\uc758 \ud575 \uac1c\ubc1c \ud504\ub85c\uadf8\ub7a8\uc744 \uc9c0\uc6d0\ud558\uc600\ub2e4. \uc774\ub780\uc740 \ub7ec\uc2dc\uc544 \uc8fc\ub3c4\uc758 \uc9d1\ub2e8 \uc548\ubcf4 \uc870\uc57d \uae30\uad6c\uc758 \ucd08\uccad\uad6d\uc774\uba70, \uc0c1\ud558\uc774 \ud611\ub825 \uae30\uad6c\uc758 \uc900\ud68c\uc6d0\uad6d\uc774\uae30\ub3c4 \ud558\ub2e4. \uc774\ub77c\ud06c\uc758 \uacbd\uc6b0, 1972\ub144 \uc774\uc9d1\ud2b8\uac00 \uc18c\ub828\uad70\uc758 \ucca0\uc218\ub97c \uc694\uad6c\ud558\uba74\uc11c \uc591\uad6d \uac04\uc758 \uad00\uacc4\uac00 \ubc1c\uc804\ud558\uae30 \uc2dc\uc791\ud558\uc600\ub2e4. \uc774\ub77c\ud06c\uc640 \uc18c\ub828\uc740 \uac19\uc740 \ud574 \uc6b0\ubc29\uad6d \ub3d9\ub9f9 \uc870\uc57d\uc744 \uccb4\uacb0\ud558\uc5ec \ud0c0\uad6d\uc73c\ub85c\ubd80\ud130\uc758 \uc704\ud611\uc774 \uc788\uc744 \uc2dc \uc0c1\ud638 \uac04 \ubcf4\ud638\ub97c \uc57d\uc18d\ud588\ub2e4. \uc774\ub7ec\ud55c \uc774\uc720\ub85c \uc774\ub780-\uc774\ub77c\ud06c \uc804\uc7c1 \ub2f9\uc2dc \uc18c\ub828\uc740 \uc774\ub77c\ud06c\ub97c \uc9c0\uc6d0\ud588\ub2e4. 1990\ub144 \uac78\ud504 \uc804\uc7c1 \ubc1c\ubc1c \ub2f9\uc2dc \uc18c\ub828\uc740 \ubbf8\uad6d \uc8fc\ub3c4\uc758 \uc5f0\ud569\uad70\uc5d0 \uc9c0\uc6d0\uc744 \ud558\uc5ec \uc774\ub77c\ud06c\uc640\uc758 \uad00\uacc4\ub294 \ub0c9\uac01\ub418\uc5c8\uc73c\ub098, \uc18c\ub828 \ubab0\ub77d \uc774\ud6c4 \uc591\uad6d\uc758 \uad00\uacc4\ub294 \ubc1c\uc804\ud588\ub2e4. \ub7ec\uc2dc\uc544\ub294 \uac78\ud504 \uc804\uc7c1 \uc774\ud6c4 \uc774\ub77c\ud06c\uc5d0 \ub300\ud55c \uc81c\uc81c \ubc0f 2003\ub144 \ubbf8\uad6d\uc758 \uc774\ub77c\ud06c \uce68\uacf5\uc5d0 \ub300\ud574 \ubc18\ub300\ud588\uc73c\uba70, \uc774\ub77c\ud06c\uc640 \ub7ec\uc2dc\uc544 \uc591\uad6d \ubaa8\ub450 \uc774\uc2a4\ub77c\uc5d8\uc758 \uac00\uc790 \uc804\uc7c1\uc5d0 \ubc18\ub300 \ubaa9\uc18c\ub9ac\ub97c \ub0c8\ub2e4. \uc774\ub7ec\ud55c \ub7ec\uc2dc\uc544\uc640 \uc911\ub3d9 \uad6d\uac00 \uac04\uc758 \uad00\uacc4\ub294 \ub3d9\ub9f9\uc758 \ucc3d\ub9bd\uc5d0 \ud070 \uae30\uc5ec\ub97c \ud588\ub2e4\uace0 \ubcfc \uc218 \uc788\ub2e4.", "sentence_answer": "\ub7ec\uc2dc\uc544\ub294 \uc2dc\ub9ac\uc544 ,", "paragraph_id": "6513621-1-0", "paragraph_question": "question: \uc2dc\ub9ac\uc544\uac00 \ub3c5\ub9bd\ud558\uae30 \uba87 \ub2ec \uc804\uc5d0 \ub7ec\uc2dc\uc544\ub294 \uc5b4\ub5a4 \uc9c0\uc5ed\uc5d0 \uc815\uaddc \ubd80\ub300 \ucc3d\uc124\uacfc \ubb34\uae30\ub97c \uc9c0\uc6d0\ud558\uaca0\ub2e4\uace0 \uc57d\uc18d\ud569\ub2c8\uae4c?, context: \ub0c9\uc804 \uc2dc\uae30\ubd80\ud130 \ub7ec\uc2dc\uc544\ub294 \uc2dc\ub9ac\uc544, \uc774\ub780, \uc774\ub77c\ud06c\uc640 \ubc00\uc811\ud55c \ub3d9\ub9f9 \uad00\uacc4\ub97c \uc720\uc9c0\ud588\ub2e4. \ub2f9\uc2dc \uc18c\ube44\uc5d0\ud2b8 \uc5f0\ubc29\uc774\uc5c8\ub358 \ub7ec\uc2dc\uc544\ub294 \ubbf8\uad6d\uc5d0 \ub300\ud56d\ud558\uae30 \uc704\ud574 \uc774\ub4e4\uc744 \uc804\ub7b5\uc801 \ub3d9\ub9f9\uad6d\uc73c\ub85c \uc0bc\uace0\uc790 \ub178\ub825\ud588\ub2e4. \ub300\ud45c\uc801\uc778 \uc608\ub85c \uc2dc\ub9ac\uc544\uac00 \ub3c5\ub9bd\ud558\uae30 \uba87 \ub2ec \uc804\uc778 1946\ub144 2\uc6d4 10\uc77c, \ub7ec\uc2dc\uc544\ub294 \uc2dc\ub9ac\uc544\uc5d0 \uc815\uaddc \ubd80\ub300 \ucc3d\uc124 \ubc0f \ubb34\uae30\ub97c \uc9c0\uc6d0\ud574\uc8fc\uaca0\ub2e4\uace0 \uc57d\uc18d\ud558\uc600\uace0, \uc218\uc5d0\uc988 \uc804\uc7c1 \uc774\ud6c4\uc5d0\ub294 \uc18c\ube44\uc5d0\ud2b8-\uc2dc\ub9ac\uc544 \uac04\uc5d0 \uc218\ub9ce\uc740 \uad70\uc0ac \ubc0f \uacbd\uc81c \ud611\uc815\uc774 \uccb4\uacb0\ub418\uc5c8\ub2e4. 1970\ub144\ub300\uc5d0 \uc18c\ub828\uc740 \uc54c\uc544\uc0ac\ub4dc \uac00\ubb38\uc744 \uc9c0\uc6d0\ud558\uc5ec \uc2dc\ub9ac\uc544\ub97c \uc790\uad6d\uc758 \ub3d9\ub9f9\uad6d\uc73c\ub85c \uc790\ub9ac\ub9e4\uae40\ud558\ub294\ub370 \uae30\uc5ec\ud588\ub2e4. \uc774\ub780\uc758 \uacbd\uc6b0\ub3c4 \ube44\uc2b7\ud588\ub2e4. 1979\ub144 \uc774\ub780 \ud601\uba85 \ubc1c\uc0dd \uc774\ud6c4 \uc0c8\ub85c \ud0c4\uc0dd\ud55c \uc774\ub780 \uc774\uc2ac\ub78c \uacf5\ud654\uad6d\uc744 \ucc98\uc74c \uc778\uc815\ud55c \uad6d\uac00\ub294 \uc18c\ub828\uc774\uc5c8\ub2e4. 1980\ub144 \uc774\ub780-\uc774\ub77c\ud06c \uc804\uc7c1 \ub2f9\uc2dc \uc18c\ub828\uc740 \uc774\ub77c\ud06c\uc5d0 \uc801\uadf9\uc801\uc778 \uc9c0\uc6d0\uc744 \ud558\uc5ec \uc774\ub780\uacfc\uc758 \uad00\uacc4\ub294 \ub0c9\uac01\ub418\uc5c8\uc9c0\ub9cc, \uc18c\ub828 \ubab0\ub77d \uc774\ud6c4 \uc591\uad6d\uc758 \uad00\uacc4\ub294 \ub2e4\uc2dc \ubc1c\uc804\ud558\uc600\uace0, 1990\ub144\ub300 \uc911\ubc18\uc5d0\ub294 \uc774\ub780\uc758 \ud575 \uac1c\ubc1c \ud504\ub85c\uadf8\ub7a8\uc744 \uc9c0\uc6d0\ud558\uc600\ub2e4. \uc774\ub780\uc740 \ub7ec\uc2dc\uc544 \uc8fc\ub3c4\uc758 \uc9d1\ub2e8 \uc548\ubcf4 \uc870\uc57d \uae30\uad6c\uc758 \ucd08\uccad\uad6d\uc774\uba70, \uc0c1\ud558\uc774 \ud611\ub825 \uae30\uad6c\uc758 \uc900\ud68c\uc6d0\uad6d\uc774\uae30\ub3c4 \ud558\ub2e4. \uc774\ub77c\ud06c\uc758 \uacbd\uc6b0, 1972\ub144 \uc774\uc9d1\ud2b8\uac00 \uc18c\ub828\uad70\uc758 \ucca0\uc218\ub97c \uc694\uad6c\ud558\uba74\uc11c \uc591\uad6d \uac04\uc758 \uad00\uacc4\uac00 \ubc1c\uc804\ud558\uae30 \uc2dc\uc791\ud558\uc600\ub2e4. \uc774\ub77c\ud06c\uc640 \uc18c\ub828\uc740 \uac19\uc740 \ud574 \uc6b0\ubc29\uad6d \ub3d9\ub9f9 \uc870\uc57d\uc744 \uccb4\uacb0\ud558\uc5ec \ud0c0\uad6d\uc73c\ub85c\ubd80\ud130\uc758 \uc704\ud611\uc774 \uc788\uc744 \uc2dc \uc0c1\ud638 \uac04 \ubcf4\ud638\ub97c \uc57d\uc18d\ud588\ub2e4. \uc774\ub7ec\ud55c \uc774\uc720\ub85c \uc774\ub780-\uc774\ub77c\ud06c \uc804\uc7c1 \ub2f9\uc2dc \uc18c\ub828\uc740 \uc774\ub77c\ud06c\ub97c \uc9c0\uc6d0\ud588\ub2e4. 1990\ub144 \uac78\ud504 \uc804\uc7c1 \ubc1c\ubc1c \ub2f9\uc2dc \uc18c\ub828\uc740 \ubbf8\uad6d \uc8fc\ub3c4\uc758 \uc5f0\ud569\uad70\uc5d0 \uc9c0\uc6d0\uc744 \ud558\uc5ec \uc774\ub77c\ud06c\uc640\uc758 \uad00\uacc4\ub294 \ub0c9\uac01\ub418\uc5c8\uc73c\ub098, \uc18c\ub828 \ubab0\ub77d \uc774\ud6c4 \uc591\uad6d\uc758 \uad00\uacc4\ub294 \ubc1c\uc804\ud588\ub2e4. \ub7ec\uc2dc\uc544\ub294 \uac78\ud504 \uc804\uc7c1 \uc774\ud6c4 \uc774\ub77c\ud06c\uc5d0 \ub300\ud55c \uc81c\uc81c \ubc0f 2003\ub144 \ubbf8\uad6d\uc758 \uc774\ub77c\ud06c \uce68\uacf5\uc5d0 \ub300\ud574 \ubc18\ub300\ud588\uc73c\uba70, \uc774\ub77c\ud06c\uc640 \ub7ec\uc2dc\uc544 \uc591\uad6d \ubaa8\ub450 \uc774\uc2a4\ub77c\uc5d8\uc758 \uac00\uc790 \uc804\uc7c1\uc5d0 \ubc18\ub300 \ubaa9\uc18c\ub9ac\ub97c \ub0c8\ub2e4. \uc774\ub7ec\ud55c \ub7ec\uc2dc\uc544\uc640 \uc911\ub3d9 \uad6d\uac00 \uac04\uc758 \uad00\uacc4\ub294 \ub3d9\ub9f9\uc758 \ucc3d\ub9bd\uc5d0 \ud070 \uae30\uc5ec\ub97c \ud588\ub2e4\uace0 \ubcfc \uc218 \uc788\ub2e4."} {"question": "\ubc15\uc2e0\ud61c\uac00 \uc131\uc778\uc774 \ub41c \uc774\ud6c4 \ucc98\uc74c \ucd9c\uc5f0\ud55c \ub4dc\ub77c\ub9c8\ub294?", "paragraph": "\ubc15\uc2e0\ud61c\ub294 2009\ub144, \ub4dc\ub77c\ub9c8 \u300a\ubbf8\ub0a8\uc774\uc2dc\ub124\uc694\u300b\ub85c \uc774\ud6c4 1\ub144 \ubc18\ub9cc\uc5d0 \ube0c\ub77c\uc6b4\uad00\uc5d0 \ubcf5\uadc0\ud558\uc600\ub2e4. \uadf9\uc911 \uc18d\uc138\uc640 \ucc28\ub2e8\ub41c \uc218\ub140\uc6d0\uc744 \ub098\uc640 \uc30d\ub465\uc774 \uc624\ube60 \uace0\ubbf8\ub0a8\uc744 \ub300\uc2e0\ud574 \ube44\ubc00\uacfc \uac00\uc2ed, \uacc4\uc0b0\uacfc \uacc4\uc57d\uc73c\ub85c \uc6c0\uc9c1\uc774\ub294 \uc5f0\uc608\uacc4\uc5d0\uc11c \uc0b4\uc544 \ub098\uac00\ub294 \uace0\ubbf8\ub140 \uc5ed\uc744 \uc5f0\uae30\ud558\uba70 \ubc30\uc6b0 \uc778\uc0dd \uc804\ud658\uc810\uc744 \ub9de\uc774\ud588\uace0, \uc77c\ubcf8 \ubc0f \uc911\ud654\uad8c\uc5d0\uc11c\ub3c4 \uc778\uc9c0\ub3c4\ub97c \ub192\ud788\uba70 \ud55c\ub958 \ud038\uc73c\ub85c \ud070 \uc778\uae30\ub97c \ub04c\uae30 \uc2dc\uc791\ud588\ub2e4. \ub098\uc911\uc5d0 \ubc15\uc2e0\ud61c\ub294 \uc778\ud130\ubdf0\uc5d0\uc11c \u201c\uc131\uc778\uc774 \ub3fc \ucc98\uc74c \ud55c \uc791\ud488\uc73c\ub85c, \uc544\uc5ed\uc774 \uc544\ub2cc \u2018\ubc15\uc2e0\ud61c\u2019\ub77c\ub294 \ud55c \uc5ec\ubc30\uc6b0\uc758 \uc2dc\uc791\uc744 \uc54c\ub9ac\uac8c \ud574\uc900 \uc791\ud488\u201d\uc774\ub77c\uace0 \u2018\ubbf8\ub0a8\uc774\uc2dc\ub124\uc694\u2019\uc5d0 \ub300\ud574 \ub9d0\ud588\ub2e4. \uc774\ub4ec \ud574, 2010\ub144 \ubc15\uc2e0\ud61c\ub294 \ub85c\ub9e8\ud2f1 \ucf54\ubbf8\ub514 \uc601\ud654 \u300a\uc2dc\ub77c\ub178; \uc5f0\uc560\uc870\uc791\ub2e8\u300b\uc5d0\uc11c \uc5f0\uc560 \uc870\uc791 \uc694\uc6d0 \uc624\ubbfc\uc601 \uc5ed\uc744 \ub9e1\uc544 \uc5c4\ud0dc\uc6c5, \uc774\ubbfc\uc815, \ucd5c\ub2e4\ub2c8\uc5d8\uacfc \uacf5\uc5f0\ud558\uba70 \uc601\ud654\ub294 260\ub9cc\uba85\uc758 \uad00\uac1d\uc218\ub97c \ub3d9\uc6d0\ud558\uba70 \ub9ce\uc740 \uc0ac\ub791\uc744 \ubc1b\uc558\ub2e4. \ubc15\uc2e0\ud61c\ub294 \uc0ac\uc804 \uc81c\uc791 \ub41c \ub300\ub9cc \ub4dc\ub77c\ub9c8 \u300a\uc120\ud48d\uad00\uac00 \u65cb\u98a8\u7ba1\u5bb6\u300b\uc5d0\uc11c \uc0e4\uc624\uc988 \uc5ed\uc744 \ub9e1\uc544 \ucd9c\uc5f0\ud558\uc600\uace0, 2011\ub144 \ubc29\uc601\ub418\uc5c8\ub2e4.", "answer": "\ubbf8\ub0a8\uc774\uc2dc\ub124\uc694", "sentence": "\ubc15\uc2e0\ud61c\ub294 2009\ub144, \ub4dc\ub77c\ub9c8 \u300a \ubbf8\ub0a8\uc774\uc2dc\ub124\uc694 \u300b\ub85c \uc774\ud6c4 1\ub144 \ubc18\ub9cc\uc5d0 \ube0c\ub77c\uc6b4\uad00\uc5d0 \ubcf5\uadc0\ud558\uc600\ub2e4.", "paragraph_sentence": " \ubc15\uc2e0\ud61c\ub294 2009\ub144, \ub4dc\ub77c\ub9c8 \u300a \ubbf8\ub0a8\uc774\uc2dc\ub124\uc694 \u300b\ub85c \uc774\ud6c4 1\ub144 \ubc18\ub9cc\uc5d0 \ube0c\ub77c\uc6b4\uad00\uc5d0 \ubcf5\uadc0\ud558\uc600\ub2e4. \uadf9\uc911 \uc18d\uc138\uc640 \ucc28\ub2e8\ub41c \uc218\ub140\uc6d0\uc744 \ub098\uc640 \uc30d\ub465\uc774 \uc624\ube60 \uace0\ubbf8\ub0a8\uc744 \ub300\uc2e0\ud574 \ube44\ubc00\uacfc \uac00\uc2ed, \uacc4\uc0b0\uacfc \uacc4\uc57d\uc73c\ub85c \uc6c0\uc9c1\uc774\ub294 \uc5f0\uc608\uacc4\uc5d0\uc11c \uc0b4\uc544 \ub098\uac00\ub294 \uace0\ubbf8\ub140 \uc5ed\uc744 \uc5f0\uae30\ud558\uba70 \ubc30\uc6b0 \uc778\uc0dd \uc804\ud658\uc810\uc744 \ub9de\uc774\ud588\uace0, \uc77c\ubcf8 \ubc0f \uc911\ud654\uad8c\uc5d0\uc11c\ub3c4 \uc778\uc9c0\ub3c4\ub97c \ub192\ud788\uba70 \ud55c\ub958 \ud038\uc73c\ub85c \ud070 \uc778\uae30\ub97c \ub04c\uae30 \uc2dc\uc791\ud588\ub2e4. \ub098\uc911\uc5d0 \ubc15\uc2e0\ud61c\ub294 \uc778\ud130\ubdf0\uc5d0\uc11c \u201c\uc131\uc778\uc774 \ub3fc \ucc98\uc74c \ud55c \uc791\ud488\uc73c\ub85c, \uc544\uc5ed\uc774 \uc544\ub2cc \u2018\ubc15\uc2e0\ud61c\u2019\ub77c\ub294 \ud55c \uc5ec\ubc30\uc6b0\uc758 \uc2dc\uc791\uc744 \uc54c\ub9ac\uac8c \ud574\uc900 \uc791\ud488\u201d\uc774\ub77c\uace0 \u2018\ubbf8\ub0a8\uc774\uc2dc\ub124\uc694\u2019\uc5d0 \ub300\ud574 \ub9d0\ud588\ub2e4. \uc774\ub4ec \ud574, 2010\ub144 \ubc15\uc2e0\ud61c\ub294 \ub85c\ub9e8\ud2f1 \ucf54\ubbf8\ub514 \uc601\ud654 \u300a\uc2dc\ub77c\ub178; \uc5f0\uc560\uc870\uc791\ub2e8\u300b\uc5d0\uc11c \uc5f0\uc560 \uc870\uc791 \uc694\uc6d0 \uc624\ubbfc\uc601 \uc5ed\uc744 \ub9e1\uc544 \uc5c4\ud0dc\uc6c5, \uc774\ubbfc\uc815, \ucd5c\ub2e4\ub2c8\uc5d8\uacfc \uacf5\uc5f0\ud558\uba70 \uc601\ud654\ub294 260\ub9cc\uba85\uc758 \uad00\uac1d\uc218\ub97c \ub3d9\uc6d0\ud558\uba70 \ub9ce\uc740 \uc0ac\ub791\uc744 \ubc1b\uc558\ub2e4. \ubc15\uc2e0\ud61c\ub294 \uc0ac\uc804 \uc81c\uc791 \ub41c \ub300\ub9cc \ub4dc\ub77c\ub9c8 \u300a\uc120\ud48d\uad00\uac00 \u65cb\u98a8\u7ba1\u5bb6\u300b\uc5d0\uc11c \uc0e4\uc624\uc988 \uc5ed\uc744 \ub9e1\uc544 \ucd9c\uc5f0\ud558\uc600\uace0, 2011\ub144 \ubc29\uc601\ub418\uc5c8\ub2e4.", "paragraph_answer": "\ubc15\uc2e0\ud61c\ub294 2009\ub144, \ub4dc\ub77c\ub9c8 \u300a \ubbf8\ub0a8\uc774\uc2dc\ub124\uc694 \u300b\ub85c \uc774\ud6c4 1\ub144 \ubc18\ub9cc\uc5d0 \ube0c\ub77c\uc6b4\uad00\uc5d0 \ubcf5\uadc0\ud558\uc600\ub2e4. \uadf9\uc911 \uc18d\uc138\uc640 \ucc28\ub2e8\ub41c \uc218\ub140\uc6d0\uc744 \ub098\uc640 \uc30d\ub465\uc774 \uc624\ube60 \uace0\ubbf8\ub0a8\uc744 \ub300\uc2e0\ud574 \ube44\ubc00\uacfc \uac00\uc2ed, \uacc4\uc0b0\uacfc \uacc4\uc57d\uc73c\ub85c \uc6c0\uc9c1\uc774\ub294 \uc5f0\uc608\uacc4\uc5d0\uc11c \uc0b4\uc544 \ub098\uac00\ub294 \uace0\ubbf8\ub140 \uc5ed\uc744 \uc5f0\uae30\ud558\uba70 \ubc30\uc6b0 \uc778\uc0dd \uc804\ud658\uc810\uc744 \ub9de\uc774\ud588\uace0, \uc77c\ubcf8 \ubc0f \uc911\ud654\uad8c\uc5d0\uc11c\ub3c4 \uc778\uc9c0\ub3c4\ub97c \ub192\ud788\uba70 \ud55c\ub958 \ud038\uc73c\ub85c \ud070 \uc778\uae30\ub97c \ub04c\uae30 \uc2dc\uc791\ud588\ub2e4. \ub098\uc911\uc5d0 \ubc15\uc2e0\ud61c\ub294 \uc778\ud130\ubdf0\uc5d0\uc11c \u201c\uc131\uc778\uc774 \ub3fc \ucc98\uc74c \ud55c \uc791\ud488\uc73c\ub85c, \uc544\uc5ed\uc774 \uc544\ub2cc \u2018\ubc15\uc2e0\ud61c\u2019\ub77c\ub294 \ud55c \uc5ec\ubc30\uc6b0\uc758 \uc2dc\uc791\uc744 \uc54c\ub9ac\uac8c \ud574\uc900 \uc791\ud488\u201d\uc774\ub77c\uace0 \u2018\ubbf8\ub0a8\uc774\uc2dc\ub124\uc694\u2019\uc5d0 \ub300\ud574 \ub9d0\ud588\ub2e4. \uc774\ub4ec \ud574, 2010\ub144 \ubc15\uc2e0\ud61c\ub294 \ub85c\ub9e8\ud2f1 \ucf54\ubbf8\ub514 \uc601\ud654 \u300a\uc2dc\ub77c\ub178; \uc5f0\uc560\uc870\uc791\ub2e8\u300b\uc5d0\uc11c \uc5f0\uc560 \uc870\uc791 \uc694\uc6d0 \uc624\ubbfc\uc601 \uc5ed\uc744 \ub9e1\uc544 \uc5c4\ud0dc\uc6c5, \uc774\ubbfc\uc815, \ucd5c\ub2e4\ub2c8\uc5d8\uacfc \uacf5\uc5f0\ud558\uba70 \uc601\ud654\ub294 260\ub9cc\uba85\uc758 \uad00\uac1d\uc218\ub97c \ub3d9\uc6d0\ud558\uba70 \ub9ce\uc740 \uc0ac\ub791\uc744 \ubc1b\uc558\ub2e4. \ubc15\uc2e0\ud61c\ub294 \uc0ac\uc804 \uc81c\uc791 \ub41c \ub300\ub9cc \ub4dc\ub77c\ub9c8 \u300a\uc120\ud48d\uad00\uac00 \u65cb\u98a8\u7ba1\u5bb6\u300b\uc5d0\uc11c \uc0e4\uc624\uc988 \uc5ed\uc744 \ub9e1\uc544 \ucd9c\uc5f0\ud558\uc600\uace0, 2011\ub144 \ubc29\uc601\ub418\uc5c8\ub2e4.", "sentence_answer": "\ubc15\uc2e0\ud61c\ub294 2009\ub144, \ub4dc\ub77c\ub9c8 \u300a \ubbf8\ub0a8\uc774\uc2dc\ub124\uc694 \u300b\ub85c \uc774\ud6c4 1\ub144 \ubc18\ub9cc\uc5d0 \ube0c\ub77c\uc6b4\uad00\uc5d0 \ubcf5\uadc0\ud558\uc600\ub2e4.", "paragraph_id": "6507513-2-0", "paragraph_question": "question: \ubc15\uc2e0\ud61c\uac00 \uc131\uc778\uc774 \ub41c \uc774\ud6c4 \ucc98\uc74c \ucd9c\uc5f0\ud55c \ub4dc\ub77c\ub9c8\ub294?, context: \ubc15\uc2e0\ud61c\ub294 2009\ub144, \ub4dc\ub77c\ub9c8 \u300a\ubbf8\ub0a8\uc774\uc2dc\ub124\uc694\u300b\ub85c \uc774\ud6c4 1\ub144 \ubc18\ub9cc\uc5d0 \ube0c\ub77c\uc6b4\uad00\uc5d0 \ubcf5\uadc0\ud558\uc600\ub2e4. \uadf9\uc911 \uc18d\uc138\uc640 \ucc28\ub2e8\ub41c \uc218\ub140\uc6d0\uc744 \ub098\uc640 \uc30d\ub465\uc774 \uc624\ube60 \uace0\ubbf8\ub0a8\uc744 \ub300\uc2e0\ud574 \ube44\ubc00\uacfc \uac00\uc2ed, \uacc4\uc0b0\uacfc \uacc4\uc57d\uc73c\ub85c \uc6c0\uc9c1\uc774\ub294 \uc5f0\uc608\uacc4\uc5d0\uc11c \uc0b4\uc544 \ub098\uac00\ub294 \uace0\ubbf8\ub140 \uc5ed\uc744 \uc5f0\uae30\ud558\uba70 \ubc30\uc6b0 \uc778\uc0dd \uc804\ud658\uc810\uc744 \ub9de\uc774\ud588\uace0, \uc77c\ubcf8 \ubc0f \uc911\ud654\uad8c\uc5d0\uc11c\ub3c4 \uc778\uc9c0\ub3c4\ub97c \ub192\ud788\uba70 \ud55c\ub958 \ud038\uc73c\ub85c \ud070 \uc778\uae30\ub97c \ub04c\uae30 \uc2dc\uc791\ud588\ub2e4. \ub098\uc911\uc5d0 \ubc15\uc2e0\ud61c\ub294 \uc778\ud130\ubdf0\uc5d0\uc11c \u201c\uc131\uc778\uc774 \ub3fc \ucc98\uc74c \ud55c \uc791\ud488\uc73c\ub85c, \uc544\uc5ed\uc774 \uc544\ub2cc \u2018\ubc15\uc2e0\ud61c\u2019\ub77c\ub294 \ud55c \uc5ec\ubc30\uc6b0\uc758 \uc2dc\uc791\uc744 \uc54c\ub9ac\uac8c \ud574\uc900 \uc791\ud488\u201d\uc774\ub77c\uace0 \u2018\ubbf8\ub0a8\uc774\uc2dc\ub124\uc694\u2019\uc5d0 \ub300\ud574 \ub9d0\ud588\ub2e4. \uc774\ub4ec \ud574, 2010\ub144 \ubc15\uc2e0\ud61c\ub294 \ub85c\ub9e8\ud2f1 \ucf54\ubbf8\ub514 \uc601\ud654 \u300a\uc2dc\ub77c\ub178; \uc5f0\uc560\uc870\uc791\ub2e8\u300b\uc5d0\uc11c \uc5f0\uc560 \uc870\uc791 \uc694\uc6d0 \uc624\ubbfc\uc601 \uc5ed\uc744 \ub9e1\uc544 \uc5c4\ud0dc\uc6c5, \uc774\ubbfc\uc815, \ucd5c\ub2e4\ub2c8\uc5d8\uacfc \uacf5\uc5f0\ud558\uba70 \uc601\ud654\ub294 260\ub9cc\uba85\uc758 \uad00\uac1d\uc218\ub97c \ub3d9\uc6d0\ud558\uba70 \ub9ce\uc740 \uc0ac\ub791\uc744 \ubc1b\uc558\ub2e4. \ubc15\uc2e0\ud61c\ub294 \uc0ac\uc804 \uc81c\uc791 \ub41c \ub300\ub9cc \ub4dc\ub77c\ub9c8 \u300a\uc120\ud48d\uad00\uac00 \u65cb\u98a8\u7ba1\u5bb6\u300b\uc5d0\uc11c \uc0e4\uc624\uc988 \uc5ed\uc744 \ub9e1\uc544 \ucd9c\uc5f0\ud558\uc600\uace0, 2011\ub144 \ubc29\uc601\ub418\uc5c8\ub2e4."} {"question": "<\ud5e4\ub77c\ud074\ub808\uc2a4>\uc5d0 \ub300\ud574 '\uc5d0\uc6b0\ub9ac\ud53c\ub370\uc2a4\uc758 \uc5f0\uadf9 \uc5f0\uad6c\uc5d0\uc11c \ub300\ub2e8\ud55c \uc911\uc694\uc131\uc744 \uac00\uc9c4 \uc791\ud488'\uc774\ub77c\uace0 \ud3c9\uac00\ud55c \uc5f0\uad6c\uc790\ub294 \ub204\uad6c\uc778\uac00?", "paragraph": "\uc77c\ubd80 \ucca0\ud559\uc790\ub4e4\uc740 \ud5e4\ub77c\ud074\ub808\uc2a4\uc758 \ubcf5\uc7a1\ud558\uac8c \uc5bd\ud600\uc788\ub294 \uc2e0\ud654\uac00 \uc544\ub974\uace0\uc2a4 \uc655\uad6d\uc758 \uc885\uc18d \uad6d\uac00 \uc9c0\ub3c4\uc790\uc640 \uac19\uc740 \uc2e4\uc874 \uc778\ubb3c\uc774 \ubc14\ud0d5\uc774 \ub418\uc5c8\uc744 \uac83\uc774\ub77c\uace0 \ubcf8\ub2e4. \ub610\ud55c, \uc77c\ubd80 \ucca0\ud559\uc790\ub294 \ud5e4\ub77c\ud074\ub808\uc2a4\uc758 \uc774\uc57c\uae30\uac00 \ud0dc\uc591\uc774 1\ub144\ub3d9\uc548 \ud669\ub3c4 12\uad81\uc744 \ud1b5\uacfc\ud558\ub294 \uac83\uc744 \uc0c1\uc9d5\ud558\ub294 \uac83\uc774\ub77c\uace0 \uc8fc\uc7a5\ud55c\ub2e4. \ub2e4\ub978 \ubb38\ud654\uad8c\uc758 \ucd08\uae30 \uc2e0\ud654\ub85c \ube44\ucd94\uc5b4 \ubcfc \ub54c, \ud5e4\ub77c\ud074\ub808\uc2a4 \uc774\uc57c\uae30\ub294 \uc774\ubbf8 \ud655\ub9bd\ub41c \uc601\uc6c5 \uc2e0\ud654\uc758 \uc9c0\uc5ed\ud654\ub85c\uc11c \ubcf4\uc5ec\uc9c4\ub2e4\ub294 \uc758\uacac\ub3c4 \uc788\ub2e4. \uc804\ud1b5\uc801\uc73c\ub85c, \ud5e4\ub77c\ud074\ub808\uc2a4\ub294 \uc81c\uc6b0\uc2a4\uc640 \ud398\ub974\uc138\uc6b0\uc2a4\uc758 \uc190\ub140 \uc54c\ud06c\uba54\ub124\uc758 \uc544\ub4e4\uc774\uc5c8\ub2e4. \uadf8\uc758 \ud658\uc0c1\uc801\uc774\uace0 \uc720\uc77c\ubb34\uc774\ud55c \uacf5\uc801\uc740 \uadf8\uac83\ub4e4\uc774 \uac16\ub294 \uc124\ud654\uc801 \uc8fc\uc81c\uc640 \ud568\uaed8 \uc778\uae30\uc788\ub294 \uc804\uc124\uc758 \uc18c\uc7ac\ub97c \uc81c\uacf5\ud558\uc600\ub2e4. \uadf8\ub294 \ud76c\uc0dd\uc790\uc774\uc790 \uc81c\ub2e8\uc758 \ucc3d\ub9bd\uc790, \uac8c\uac78\uc2a4\ub7ec\uc6b4 \uba39\ubcf4\ub85c \ubb18\uc0ac\ub418\uac70\ub098 \uc5b8\uae09\ub41c\ub2e4. \ud76c\uadf9\uc5d0\uc11c \ub4f1\uc7a5\ud558\ub294 \uadf8\uc758 \uc774\ub7ec\ud55c \uc5ed\ud560\uc740 \uadf8\uc758 \ube44\ucc38\ud55c \uc8fd\uc74c\uc774 \ub9ce\uc740 \ube44\uadf9\uc801 \uc694\uc18c\ub97c \uac00\uc9c0\uace0 \uc788\ub294 \uac83\uacfc\ub294 \uc0c1\ubc18\uc801\uc774\ub2e4. \u2014 \ud0c8\ub9ac\uc544 \ud30c\ud30c\ub3c4\ud478\ub8e8\ub294 \u300a\ud5e4\ub77c\ud074\ub808\uc2a4\u300b\uc5d0 \ub300\ud574\uc11c \"\uc5d0\uc6b0\ub9ac\ud53c\ub370\uc2a4\uc758 \uc5f0\uadf9 \uc5f0\uad6c\uc5d0\uc11c \ub300\ub2e8\ud55c \uc911\uc694\uc131\uc744 \uac00\uc9c4 \uc791\ud488\"\uc774\ub77c\uace0 \ud3c9\uac00\ud558\uc600\ub2e4. \uc608\uc220\uacfc \ubb38\ud559\uc5d0\uc11c \ud5e4\ub77c\ud074\ub808\uc2a4\ub294 \ubcf4\ud1b5\uc758 \uc778\uac04\ubcf4\ub2e4 \ub9c9\ub300\ud55c \ud798\uc744 \uac00\uc9c4 \ub0a8\uc131\uc73c\ub85c \ub4f1\uc7a5\ud558\uba70, \ud65c\uc744 \ubb34\uae30\ub85c \ub4e4\uace0 \uc788\ub294 \uac83\uc774 \ud2b9\uc9d5\uc778\ub370 \uc885\uc885 \uace4\ubd09\uc73c\ub85c \ubb18\uc0ac\ub418\uae30\ub3c4 \ud558\uc600\ub2e4. \uaf43\ubcd1 \ud1a0\uae30\uc5d0\ub294 \ud5e4\ub77c\ud074\ub808\uc2a4\uc758 \ubb18\uc0ac\uac00 \ub2e4\ub978 \uc18c\uc7ac\uc640 \ube44\ud560 \uc218 \uc5c6\ub294 \uc778\uae30\ub97c \uad6c\uac00\ud558\uc600\ub294\ub370, \ud2b9\ud788 \uadf8\uac00 \uc0ac\uc790\uc640 \uc2f8\uc6b0\ub294 \uc7a5\uba74\uc740 \uc218\ubc31 \uac1c\uc758 \ud1a0\uae30\uc5d0\uc11c \ubc1c\uacac\ub418\uc5c8\ub2e4.", "answer": "\ud0c8\ub9ac\uc544 \ud30c\ud30c\ub3c4\ud478\ub8e8", "sentence": "\u2014 \ud0c8\ub9ac\uc544 \ud30c\ud30c\ub3c4\ud478\ub8e8 \ub294 \u300a\ud5e4\ub77c\ud074\ub808\uc2a4\u300b\uc5d0 \ub300\ud574\uc11c \"\uc5d0\uc6b0\ub9ac\ud53c\ub370\uc2a4\uc758 \uc5f0\uadf9 \uc5f0\uad6c\uc5d0\uc11c \ub300\ub2e8\ud55c \uc911\uc694\uc131\uc744 \uac00\uc9c4 \uc791\ud488\"\uc774\ub77c\uace0 \ud3c9\uac00\ud558\uc600\ub2e4.", "paragraph_sentence": "\uc77c\ubd80 \ucca0\ud559\uc790\ub4e4\uc740 \ud5e4\ub77c\ud074\ub808\uc2a4\uc758 \ubcf5\uc7a1\ud558\uac8c \uc5bd\ud600\uc788\ub294 \uc2e0\ud654\uac00 \uc544\ub974\uace0\uc2a4 \uc655\uad6d\uc758 \uc885\uc18d \uad6d\uac00 \uc9c0\ub3c4\uc790\uc640 \uac19\uc740 \uc2e4\uc874 \uc778\ubb3c\uc774 \ubc14\ud0d5\uc774 \ub418\uc5c8\uc744 \uac83\uc774\ub77c\uace0 \ubcf8\ub2e4. \ub610\ud55c, \uc77c\ubd80 \ucca0\ud559\uc790\ub294 \ud5e4\ub77c\ud074\ub808\uc2a4\uc758 \uc774\uc57c\uae30\uac00 \ud0dc\uc591\uc774 1\ub144\ub3d9\uc548 \ud669\ub3c4 12\uad81\uc744 \ud1b5\uacfc\ud558\ub294 \uac83\uc744 \uc0c1\uc9d5\ud558\ub294 \uac83\uc774\ub77c\uace0 \uc8fc\uc7a5\ud55c\ub2e4. \ub2e4\ub978 \ubb38\ud654\uad8c\uc758 \ucd08\uae30 \uc2e0\ud654\ub85c \ube44\ucd94\uc5b4 \ubcfc \ub54c, \ud5e4\ub77c\ud074\ub808\uc2a4 \uc774\uc57c\uae30\ub294 \uc774\ubbf8 \ud655\ub9bd\ub41c \uc601\uc6c5 \uc2e0\ud654\uc758 \uc9c0\uc5ed\ud654\ub85c\uc11c \ubcf4\uc5ec\uc9c4\ub2e4\ub294 \uc758\uacac\ub3c4 \uc788\ub2e4. \uc804\ud1b5\uc801\uc73c\ub85c, \ud5e4\ub77c\ud074\ub808\uc2a4\ub294 \uc81c\uc6b0\uc2a4\uc640 \ud398\ub974\uc138\uc6b0\uc2a4\uc758 \uc190\ub140 \uc54c\ud06c\uba54\ub124\uc758 \uc544\ub4e4\uc774\uc5c8\ub2e4. \uadf8\uc758 \ud658\uc0c1\uc801\uc774\uace0 \uc720\uc77c\ubb34\uc774\ud55c \uacf5\uc801\uc740 \uadf8\uac83\ub4e4\uc774 \uac16\ub294 \uc124\ud654\uc801 \uc8fc\uc81c\uc640 \ud568\uaed8 \uc778\uae30\uc788\ub294 \uc804\uc124\uc758 \uc18c\uc7ac\ub97c \uc81c\uacf5\ud558\uc600\ub2e4. \uadf8\ub294 \ud76c\uc0dd\uc790\uc774\uc790 \uc81c\ub2e8\uc758 \ucc3d\ub9bd\uc790, \uac8c\uac78\uc2a4\ub7ec\uc6b4 \uba39\ubcf4\ub85c \ubb18\uc0ac\ub418\uac70\ub098 \uc5b8\uae09\ub41c\ub2e4. \ud76c\uadf9\uc5d0\uc11c \ub4f1\uc7a5\ud558\ub294 \uadf8\uc758 \uc774\ub7ec\ud55c \uc5ed\ud560\uc740 \uadf8\uc758 \ube44\ucc38\ud55c \uc8fd\uc74c\uc774 \ub9ce\uc740 \ube44\uadf9\uc801 \uc694\uc18c\ub97c \uac00\uc9c0\uace0 \uc788\ub294 \uac83\uacfc\ub294 \uc0c1\ubc18\uc801\uc774\ub2e4. \u2014 \ud0c8\ub9ac\uc544 \ud30c\ud30c\ub3c4\ud478\ub8e8 \ub294 \u300a\ud5e4\ub77c\ud074\ub808\uc2a4\u300b\uc5d0 \ub300\ud574\uc11c \"\uc5d0\uc6b0\ub9ac\ud53c\ub370\uc2a4\uc758 \uc5f0\uadf9 \uc5f0\uad6c\uc5d0\uc11c \ub300\ub2e8\ud55c \uc911\uc694\uc131\uc744 \uac00\uc9c4 \uc791\ud488\"\uc774\ub77c\uace0 \ud3c9\uac00\ud558\uc600\ub2e4. \uc608\uc220\uacfc \ubb38\ud559\uc5d0\uc11c \ud5e4\ub77c\ud074\ub808\uc2a4\ub294 \ubcf4\ud1b5\uc758 \uc778\uac04\ubcf4\ub2e4 \ub9c9\ub300\ud55c \ud798\uc744 \uac00\uc9c4 \ub0a8\uc131\uc73c\ub85c \ub4f1\uc7a5\ud558\uba70, \ud65c\uc744 \ubb34\uae30\ub85c \ub4e4\uace0 \uc788\ub294 \uac83\uc774 \ud2b9\uc9d5\uc778\ub370 \uc885\uc885 \uace4\ubd09\uc73c\ub85c \ubb18\uc0ac\ub418\uae30\ub3c4 \ud558\uc600\ub2e4. \uaf43\ubcd1 \ud1a0\uae30\uc5d0\ub294 \ud5e4\ub77c\ud074\ub808\uc2a4\uc758 \ubb18\uc0ac\uac00 \ub2e4\ub978 \uc18c\uc7ac\uc640 \ube44\ud560 \uc218 \uc5c6\ub294 \uc778\uae30\ub97c \uad6c\uac00\ud558\uc600\ub294\ub370, \ud2b9\ud788 \uadf8\uac00 \uc0ac\uc790\uc640 \uc2f8\uc6b0\ub294 \uc7a5\uba74\uc740 \uc218\ubc31 \uac1c\uc758 \ud1a0\uae30\uc5d0\uc11c \ubc1c\uacac\ub418\uc5c8\ub2e4.", "paragraph_answer": "\uc77c\ubd80 \ucca0\ud559\uc790\ub4e4\uc740 \ud5e4\ub77c\ud074\ub808\uc2a4\uc758 \ubcf5\uc7a1\ud558\uac8c \uc5bd\ud600\uc788\ub294 \uc2e0\ud654\uac00 \uc544\ub974\uace0\uc2a4 \uc655\uad6d\uc758 \uc885\uc18d \uad6d\uac00 \uc9c0\ub3c4\uc790\uc640 \uac19\uc740 \uc2e4\uc874 \uc778\ubb3c\uc774 \ubc14\ud0d5\uc774 \ub418\uc5c8\uc744 \uac83\uc774\ub77c\uace0 \ubcf8\ub2e4. \ub610\ud55c, \uc77c\ubd80 \ucca0\ud559\uc790\ub294 \ud5e4\ub77c\ud074\ub808\uc2a4\uc758 \uc774\uc57c\uae30\uac00 \ud0dc\uc591\uc774 1\ub144\ub3d9\uc548 \ud669\ub3c4 12\uad81\uc744 \ud1b5\uacfc\ud558\ub294 \uac83\uc744 \uc0c1\uc9d5\ud558\ub294 \uac83\uc774\ub77c\uace0 \uc8fc\uc7a5\ud55c\ub2e4. \ub2e4\ub978 \ubb38\ud654\uad8c\uc758 \ucd08\uae30 \uc2e0\ud654\ub85c \ube44\ucd94\uc5b4 \ubcfc \ub54c, \ud5e4\ub77c\ud074\ub808\uc2a4 \uc774\uc57c\uae30\ub294 \uc774\ubbf8 \ud655\ub9bd\ub41c \uc601\uc6c5 \uc2e0\ud654\uc758 \uc9c0\uc5ed\ud654\ub85c\uc11c \ubcf4\uc5ec\uc9c4\ub2e4\ub294 \uc758\uacac\ub3c4 \uc788\ub2e4. \uc804\ud1b5\uc801\uc73c\ub85c, \ud5e4\ub77c\ud074\ub808\uc2a4\ub294 \uc81c\uc6b0\uc2a4\uc640 \ud398\ub974\uc138\uc6b0\uc2a4\uc758 \uc190\ub140 \uc54c\ud06c\uba54\ub124\uc758 \uc544\ub4e4\uc774\uc5c8\ub2e4. \uadf8\uc758 \ud658\uc0c1\uc801\uc774\uace0 \uc720\uc77c\ubb34\uc774\ud55c \uacf5\uc801\uc740 \uadf8\uac83\ub4e4\uc774 \uac16\ub294 \uc124\ud654\uc801 \uc8fc\uc81c\uc640 \ud568\uaed8 \uc778\uae30\uc788\ub294 \uc804\uc124\uc758 \uc18c\uc7ac\ub97c \uc81c\uacf5\ud558\uc600\ub2e4. \uadf8\ub294 \ud76c\uc0dd\uc790\uc774\uc790 \uc81c\ub2e8\uc758 \ucc3d\ub9bd\uc790, \uac8c\uac78\uc2a4\ub7ec\uc6b4 \uba39\ubcf4\ub85c \ubb18\uc0ac\ub418\uac70\ub098 \uc5b8\uae09\ub41c\ub2e4. \ud76c\uadf9\uc5d0\uc11c \ub4f1\uc7a5\ud558\ub294 \uadf8\uc758 \uc774\ub7ec\ud55c \uc5ed\ud560\uc740 \uadf8\uc758 \ube44\ucc38\ud55c \uc8fd\uc74c\uc774 \ub9ce\uc740 \ube44\uadf9\uc801 \uc694\uc18c\ub97c \uac00\uc9c0\uace0 \uc788\ub294 \uac83\uacfc\ub294 \uc0c1\ubc18\uc801\uc774\ub2e4. \u2014 \ud0c8\ub9ac\uc544 \ud30c\ud30c\ub3c4\ud478\ub8e8 \ub294 \u300a\ud5e4\ub77c\ud074\ub808\uc2a4\u300b\uc5d0 \ub300\ud574\uc11c \"\uc5d0\uc6b0\ub9ac\ud53c\ub370\uc2a4\uc758 \uc5f0\uadf9 \uc5f0\uad6c\uc5d0\uc11c \ub300\ub2e8\ud55c \uc911\uc694\uc131\uc744 \uac00\uc9c4 \uc791\ud488\"\uc774\ub77c\uace0 \ud3c9\uac00\ud558\uc600\ub2e4. \uc608\uc220\uacfc \ubb38\ud559\uc5d0\uc11c \ud5e4\ub77c\ud074\ub808\uc2a4\ub294 \ubcf4\ud1b5\uc758 \uc778\uac04\ubcf4\ub2e4 \ub9c9\ub300\ud55c \ud798\uc744 \uac00\uc9c4 \ub0a8\uc131\uc73c\ub85c \ub4f1\uc7a5\ud558\uba70, \ud65c\uc744 \ubb34\uae30\ub85c \ub4e4\uace0 \uc788\ub294 \uac83\uc774 \ud2b9\uc9d5\uc778\ub370 \uc885\uc885 \uace4\ubd09\uc73c\ub85c \ubb18\uc0ac\ub418\uae30\ub3c4 \ud558\uc600\ub2e4. \uaf43\ubcd1 \ud1a0\uae30\uc5d0\ub294 \ud5e4\ub77c\ud074\ub808\uc2a4\uc758 \ubb18\uc0ac\uac00 \ub2e4\ub978 \uc18c\uc7ac\uc640 \ube44\ud560 \uc218 \uc5c6\ub294 \uc778\uae30\ub97c \uad6c\uac00\ud558\uc600\ub294\ub370, \ud2b9\ud788 \uadf8\uac00 \uc0ac\uc790\uc640 \uc2f8\uc6b0\ub294 \uc7a5\uba74\uc740 \uc218\ubc31 \uac1c\uc758 \ud1a0\uae30\uc5d0\uc11c \ubc1c\uacac\ub418\uc5c8\ub2e4.", "sentence_answer": "\u2014 \ud0c8\ub9ac\uc544 \ud30c\ud30c\ub3c4\ud478\ub8e8 \ub294 \u300a\ud5e4\ub77c\ud074\ub808\uc2a4\u300b\uc5d0 \ub300\ud574\uc11c \"\uc5d0\uc6b0\ub9ac\ud53c\ub370\uc2a4\uc758 \uc5f0\uadf9 \uc5f0\uad6c\uc5d0\uc11c \ub300\ub2e8\ud55c \uc911\uc694\uc131\uc744 \uac00\uc9c4 \uc791\ud488\"\uc774\ub77c\uace0 \ud3c9\uac00\ud558\uc600\ub2e4.", "paragraph_id": "6527377-15-1", "paragraph_question": "question: <\ud5e4\ub77c\ud074\ub808\uc2a4>\uc5d0 \ub300\ud574 '\uc5d0\uc6b0\ub9ac\ud53c\ub370\uc2a4\uc758 \uc5f0\uadf9 \uc5f0\uad6c\uc5d0\uc11c \ub300\ub2e8\ud55c \uc911\uc694\uc131\uc744 \uac00\uc9c4 \uc791\ud488'\uc774\ub77c\uace0 \ud3c9\uac00\ud55c \uc5f0\uad6c\uc790\ub294 \ub204\uad6c\uc778\uac00?, context: \uc77c\ubd80 \ucca0\ud559\uc790\ub4e4\uc740 \ud5e4\ub77c\ud074\ub808\uc2a4\uc758 \ubcf5\uc7a1\ud558\uac8c \uc5bd\ud600\uc788\ub294 \uc2e0\ud654\uac00 \uc544\ub974\uace0\uc2a4 \uc655\uad6d\uc758 \uc885\uc18d \uad6d\uac00 \uc9c0\ub3c4\uc790\uc640 \uac19\uc740 \uc2e4\uc874 \uc778\ubb3c\uc774 \ubc14\ud0d5\uc774 \ub418\uc5c8\uc744 \uac83\uc774\ub77c\uace0 \ubcf8\ub2e4. \ub610\ud55c, \uc77c\ubd80 \ucca0\ud559\uc790\ub294 \ud5e4\ub77c\ud074\ub808\uc2a4\uc758 \uc774\uc57c\uae30\uac00 \ud0dc\uc591\uc774 1\ub144\ub3d9\uc548 \ud669\ub3c4 12\uad81\uc744 \ud1b5\uacfc\ud558\ub294 \uac83\uc744 \uc0c1\uc9d5\ud558\ub294 \uac83\uc774\ub77c\uace0 \uc8fc\uc7a5\ud55c\ub2e4. \ub2e4\ub978 \ubb38\ud654\uad8c\uc758 \ucd08\uae30 \uc2e0\ud654\ub85c \ube44\ucd94\uc5b4 \ubcfc \ub54c, \ud5e4\ub77c\ud074\ub808\uc2a4 \uc774\uc57c\uae30\ub294 \uc774\ubbf8 \ud655\ub9bd\ub41c \uc601\uc6c5 \uc2e0\ud654\uc758 \uc9c0\uc5ed\ud654\ub85c\uc11c \ubcf4\uc5ec\uc9c4\ub2e4\ub294 \uc758\uacac\ub3c4 \uc788\ub2e4. \uc804\ud1b5\uc801\uc73c\ub85c, \ud5e4\ub77c\ud074\ub808\uc2a4\ub294 \uc81c\uc6b0\uc2a4\uc640 \ud398\ub974\uc138\uc6b0\uc2a4\uc758 \uc190\ub140 \uc54c\ud06c\uba54\ub124\uc758 \uc544\ub4e4\uc774\uc5c8\ub2e4. \uadf8\uc758 \ud658\uc0c1\uc801\uc774\uace0 \uc720\uc77c\ubb34\uc774\ud55c \uacf5\uc801\uc740 \uadf8\uac83\ub4e4\uc774 \uac16\ub294 \uc124\ud654\uc801 \uc8fc\uc81c\uc640 \ud568\uaed8 \uc778\uae30\uc788\ub294 \uc804\uc124\uc758 \uc18c\uc7ac\ub97c \uc81c\uacf5\ud558\uc600\ub2e4. \uadf8\ub294 \ud76c\uc0dd\uc790\uc774\uc790 \uc81c\ub2e8\uc758 \ucc3d\ub9bd\uc790, \uac8c\uac78\uc2a4\ub7ec\uc6b4 \uba39\ubcf4\ub85c \ubb18\uc0ac\ub418\uac70\ub098 \uc5b8\uae09\ub41c\ub2e4. \ud76c\uadf9\uc5d0\uc11c \ub4f1\uc7a5\ud558\ub294 \uadf8\uc758 \uc774\ub7ec\ud55c \uc5ed\ud560\uc740 \uadf8\uc758 \ube44\ucc38\ud55c \uc8fd\uc74c\uc774 \ub9ce\uc740 \ube44\uadf9\uc801 \uc694\uc18c\ub97c \uac00\uc9c0\uace0 \uc788\ub294 \uac83\uacfc\ub294 \uc0c1\ubc18\uc801\uc774\ub2e4. \u2014 \ud0c8\ub9ac\uc544 \ud30c\ud30c\ub3c4\ud478\ub8e8\ub294 \u300a\ud5e4\ub77c\ud074\ub808\uc2a4\u300b\uc5d0 \ub300\ud574\uc11c \"\uc5d0\uc6b0\ub9ac\ud53c\ub370\uc2a4\uc758 \uc5f0\uadf9 \uc5f0\uad6c\uc5d0\uc11c \ub300\ub2e8\ud55c \uc911\uc694\uc131\uc744 \uac00\uc9c4 \uc791\ud488\"\uc774\ub77c\uace0 \ud3c9\uac00\ud558\uc600\ub2e4. \uc608\uc220\uacfc \ubb38\ud559\uc5d0\uc11c \ud5e4\ub77c\ud074\ub808\uc2a4\ub294 \ubcf4\ud1b5\uc758 \uc778\uac04\ubcf4\ub2e4 \ub9c9\ub300\ud55c \ud798\uc744 \uac00\uc9c4 \ub0a8\uc131\uc73c\ub85c \ub4f1\uc7a5\ud558\uba70, \ud65c\uc744 \ubb34\uae30\ub85c \ub4e4\uace0 \uc788\ub294 \uac83\uc774 \ud2b9\uc9d5\uc778\ub370 \uc885\uc885 \uace4\ubd09\uc73c\ub85c \ubb18\uc0ac\ub418\uae30\ub3c4 \ud558\uc600\ub2e4. \uaf43\ubcd1 \ud1a0\uae30\uc5d0\ub294 \ud5e4\ub77c\ud074\ub808\uc2a4\uc758 \ubb18\uc0ac\uac00 \ub2e4\ub978 \uc18c\uc7ac\uc640 \ube44\ud560 \uc218 \uc5c6\ub294 \uc778\uae30\ub97c \uad6c\uac00\ud558\uc600\ub294\ub370, \ud2b9\ud788 \uadf8\uac00 \uc0ac\uc790\uc640 \uc2f8\uc6b0\ub294 \uc7a5\uba74\uc740 \uc218\ubc31 \uac1c\uc758 \ud1a0\uae30\uc5d0\uc11c \ubc1c\uacac\ub418\uc5c8\ub2e4."} {"question": "\uc704\ub354\uc218\ud47c\uc774 \uc0ac\uadc0\uae30 \uc2dc\uc791\ud55c \ub3d9\ub8cc\ubc30\uc6b0\uc758 \uc774\ub984\uc740 \ubb34\uc5c7\uc778\uac00?", "paragraph": "1997\ub144 \u300a\ud504\ub9ac\uc6e8\uc774\u300b\uc758 \uc791\uc5c5\uc774 \ubaa8\ub450 \ub05d\ub098\uace0 \ub300\uc911\uc758 \uc8fc\ubaa9\uc744 \ub04c\uac8c \ub41c \uc704\ub354\uc218\ud47c\uc740 1\ub144\uac04 \uc5f0\uae30 \ud65c\ub3d9\uc744 \uc26c\uba74\uc11c, \ub3d9\ub8cc \ubc30\uc6b0\uc600\ub358 2\uc0b4 \uc5f0\uc0c1\uc758 \ub77c\uc774\uc5b8 \ud544\ub9ac\ud53c\uc640 \uc0ac\uadc0\uae30 \uc2dc\uc791\ud588\ub2e4. \uc774\ud6c4 \uc601\ud654 \u300a\ub7ec\ube0c \ud035\u300b, \u300a\ud2b8\uc640\uc774\ub77c\uc787\u300b(1998\ub144)\uc5d0 \ucd9c\uc5f0\ud588\ub2e4. \uc601\ud654 \u300a\ud50c\ub808\uc804\ud2b8\ube4c\u300b\uc5d0\uc11c\ub294 \ud1a0\ube44 \ub9e5\uacfc\uc774\uc5b4\uc640 \ud568\uaed8 \uc5f0\uae30\ud558\uc600\uace0, \uc774 \uc791\ud488\uc5d0\uc11c\ub294 1950\ub144\ub300 \ud154\ub808\ube44\uc804 \uc2dc\ud2b8\ucf64\uc744 1990\ub144\ub300 \uac10\uc131\uc73c\ub85c \ubc14\uafd4\ub193\uc740 10\ub300 \ub0a8\ub9e4\uc758 \uc774\uc57c\uae30\ub85c, \uadf8\ub140\uc758 \uc5ed\ud560\uc740 \uc678\ubaa8\uc640 \uce5c\uad6c\ub4e4, \uc778\uae30 \uc21c\uc704\ub97c \uace0\ubbfc\ud558\ub358 \uc5b4\ub9b0 \uc18c\ub140\uc778 \ub3d9\uc0dd \uc81c\ub2c8\ud37c\uc600\ub2e4. \uc774 \uc5f0\uae30\ub85c \ud560\ub9ac\uc6b0\ub4dc \uc80a\uc740 \uc5f0\uae30\uc790\uc0c1(Young Hollywood Award)\uc744 \uc218\uc0c1\ud588\ub2e4. \uac10\ub3c5\uc774\ub358 \uac8c\ub9ac \ub85c\uc2a4\ub294 \ub9ac\uc988\uac00 \ub6f0\uc5b4\ub09c \uc601\ud654 \ubc30\uc6b0\uac00 \ub420 \uac83\uc744 \uad73\uac74\ud788 \ubbff\uace0 \uc788\uc5c8\ub2e4\uace0 \uce6d\ucc2c\ud588\ub2e4.", "answer": "\ub77c\uc774\uc5b8 \ud544\ub9ac\ud53c", "sentence": "1997\ub144 \u300a\ud504\ub9ac\uc6e8\uc774\u300b\uc758 \uc791\uc5c5\uc774 \ubaa8\ub450 \ub05d\ub098\uace0 \ub300\uc911\uc758 \uc8fc\ubaa9\uc744 \ub04c\uac8c \ub41c \uc704\ub354\uc218\ud47c\uc740 1\ub144\uac04 \uc5f0\uae30 \ud65c\ub3d9\uc744 \uc26c\uba74\uc11c, \ub3d9\ub8cc \ubc30\uc6b0\uc600\ub358 2\uc0b4 \uc5f0\uc0c1\uc758 \ub77c\uc774\uc5b8 \ud544\ub9ac\ud53c \uc640 \uc0ac\uadc0\uae30 \uc2dc\uc791\ud588\ub2e4.", "paragraph_sentence": " 1997\ub144 \u300a\ud504\ub9ac\uc6e8\uc774\u300b\uc758 \uc791\uc5c5\uc774 \ubaa8\ub450 \ub05d\ub098\uace0 \ub300\uc911\uc758 \uc8fc\ubaa9\uc744 \ub04c\uac8c \ub41c \uc704\ub354\uc218\ud47c\uc740 1\ub144\uac04 \uc5f0\uae30 \ud65c\ub3d9\uc744 \uc26c\uba74\uc11c, \ub3d9\ub8cc \ubc30\uc6b0\uc600\ub358 2\uc0b4 \uc5f0\uc0c1\uc758 \ub77c\uc774\uc5b8 \ud544\ub9ac\ud53c \uc640 \uc0ac\uadc0\uae30 \uc2dc\uc791\ud588\ub2e4. \uc774\ud6c4 \uc601\ud654 \u300a\ub7ec\ube0c \ud035\u300b, \u300a\ud2b8\uc640\uc774\ub77c\uc787\u300b(1998\ub144)\uc5d0 \ucd9c\uc5f0\ud588\ub2e4. \uc601\ud654 \u300a\ud50c\ub808\uc804\ud2b8\ube4c\u300b\uc5d0\uc11c\ub294 \ud1a0\ube44 \ub9e5\uacfc\uc774\uc5b4\uc640 \ud568\uaed8 \uc5f0\uae30\ud558\uc600\uace0, \uc774 \uc791\ud488\uc5d0\uc11c\ub294 1950\ub144\ub300 \ud154\ub808\ube44\uc804 \uc2dc\ud2b8\ucf64\uc744 1990\ub144\ub300 \uac10\uc131\uc73c\ub85c \ubc14\uafd4\ub193\uc740 10\ub300 \ub0a8\ub9e4\uc758 \uc774\uc57c\uae30\ub85c, \uadf8\ub140\uc758 \uc5ed\ud560\uc740 \uc678\ubaa8\uc640 \uce5c\uad6c\ub4e4, \uc778\uae30 \uc21c\uc704\ub97c \uace0\ubbfc\ud558\ub358 \uc5b4\ub9b0 \uc18c\ub140\uc778 \ub3d9\uc0dd \uc81c\ub2c8\ud37c\uc600\ub2e4. \uc774 \uc5f0\uae30\ub85c \ud560\ub9ac\uc6b0\ub4dc \uc80a\uc740 \uc5f0\uae30\uc790\uc0c1(Young Hollywood Award)\uc744 \uc218\uc0c1\ud588\ub2e4. \uac10\ub3c5\uc774\ub358 \uac8c\ub9ac \ub85c\uc2a4\ub294 \ub9ac\uc988\uac00 \ub6f0\uc5b4\ub09c \uc601\ud654 \ubc30\uc6b0\uac00 \ub420 \uac83\uc744 \uad73\uac74\ud788 \ubbff\uace0 \uc788\uc5c8\ub2e4\uace0 \uce6d\ucc2c\ud588\ub2e4.", "paragraph_answer": "1997\ub144 \u300a\ud504\ub9ac\uc6e8\uc774\u300b\uc758 \uc791\uc5c5\uc774 \ubaa8\ub450 \ub05d\ub098\uace0 \ub300\uc911\uc758 \uc8fc\ubaa9\uc744 \ub04c\uac8c \ub41c \uc704\ub354\uc218\ud47c\uc740 1\ub144\uac04 \uc5f0\uae30 \ud65c\ub3d9\uc744 \uc26c\uba74\uc11c, \ub3d9\ub8cc \ubc30\uc6b0\uc600\ub358 2\uc0b4 \uc5f0\uc0c1\uc758 \ub77c\uc774\uc5b8 \ud544\ub9ac\ud53c \uc640 \uc0ac\uadc0\uae30 \uc2dc\uc791\ud588\ub2e4. \uc774\ud6c4 \uc601\ud654 \u300a\ub7ec\ube0c \ud035\u300b, \u300a\ud2b8\uc640\uc774\ub77c\uc787\u300b(1998\ub144)\uc5d0 \ucd9c\uc5f0\ud588\ub2e4. \uc601\ud654 \u300a\ud50c\ub808\uc804\ud2b8\ube4c\u300b\uc5d0\uc11c\ub294 \ud1a0\ube44 \ub9e5\uacfc\uc774\uc5b4\uc640 \ud568\uaed8 \uc5f0\uae30\ud558\uc600\uace0, \uc774 \uc791\ud488\uc5d0\uc11c\ub294 1950\ub144\ub300 \ud154\ub808\ube44\uc804 \uc2dc\ud2b8\ucf64\uc744 1990\ub144\ub300 \uac10\uc131\uc73c\ub85c \ubc14\uafd4\ub193\uc740 10\ub300 \ub0a8\ub9e4\uc758 \uc774\uc57c\uae30\ub85c, \uadf8\ub140\uc758 \uc5ed\ud560\uc740 \uc678\ubaa8\uc640 \uce5c\uad6c\ub4e4, \uc778\uae30 \uc21c\uc704\ub97c \uace0\ubbfc\ud558\ub358 \uc5b4\ub9b0 \uc18c\ub140\uc778 \ub3d9\uc0dd \uc81c\ub2c8\ud37c\uc600\ub2e4. \uc774 \uc5f0\uae30\ub85c \ud560\ub9ac\uc6b0\ub4dc \uc80a\uc740 \uc5f0\uae30\uc790\uc0c1(Young Hollywood Award)\uc744 \uc218\uc0c1\ud588\ub2e4. \uac10\ub3c5\uc774\ub358 \uac8c\ub9ac \ub85c\uc2a4\ub294 \ub9ac\uc988\uac00 \ub6f0\uc5b4\ub09c \uc601\ud654 \ubc30\uc6b0\uac00 \ub420 \uac83\uc744 \uad73\uac74\ud788 \ubbff\uace0 \uc788\uc5c8\ub2e4\uace0 \uce6d\ucc2c\ud588\ub2e4.", "sentence_answer": "1997\ub144 \u300a\ud504\ub9ac\uc6e8\uc774\u300b\uc758 \uc791\uc5c5\uc774 \ubaa8\ub450 \ub05d\ub098\uace0 \ub300\uc911\uc758 \uc8fc\ubaa9\uc744 \ub04c\uac8c \ub41c \uc704\ub354\uc218\ud47c\uc740 1\ub144\uac04 \uc5f0\uae30 \ud65c\ub3d9\uc744 \uc26c\uba74\uc11c, \ub3d9\ub8cc \ubc30\uc6b0\uc600\ub358 2\uc0b4 \uc5f0\uc0c1\uc758 \ub77c\uc774\uc5b8 \ud544\ub9ac\ud53c \uc640 \uc0ac\uadc0\uae30 \uc2dc\uc791\ud588\ub2e4.", "paragraph_id": "6534139-2-0", "paragraph_question": "question: \uc704\ub354\uc218\ud47c\uc774 \uc0ac\uadc0\uae30 \uc2dc\uc791\ud55c \ub3d9\ub8cc\ubc30\uc6b0\uc758 \uc774\ub984\uc740 \ubb34\uc5c7\uc778\uac00?, context: 1997\ub144 \u300a\ud504\ub9ac\uc6e8\uc774\u300b\uc758 \uc791\uc5c5\uc774 \ubaa8\ub450 \ub05d\ub098\uace0 \ub300\uc911\uc758 \uc8fc\ubaa9\uc744 \ub04c\uac8c \ub41c \uc704\ub354\uc218\ud47c\uc740 1\ub144\uac04 \uc5f0\uae30 \ud65c\ub3d9\uc744 \uc26c\uba74\uc11c, \ub3d9\ub8cc \ubc30\uc6b0\uc600\ub358 2\uc0b4 \uc5f0\uc0c1\uc758 \ub77c\uc774\uc5b8 \ud544\ub9ac\ud53c\uc640 \uc0ac\uadc0\uae30 \uc2dc\uc791\ud588\ub2e4. \uc774\ud6c4 \uc601\ud654 \u300a\ub7ec\ube0c \ud035\u300b, \u300a\ud2b8\uc640\uc774\ub77c\uc787\u300b(1998\ub144)\uc5d0 \ucd9c\uc5f0\ud588\ub2e4. \uc601\ud654 \u300a\ud50c\ub808\uc804\ud2b8\ube4c\u300b\uc5d0\uc11c\ub294 \ud1a0\ube44 \ub9e5\uacfc\uc774\uc5b4\uc640 \ud568\uaed8 \uc5f0\uae30\ud558\uc600\uace0, \uc774 \uc791\ud488\uc5d0\uc11c\ub294 1950\ub144\ub300 \ud154\ub808\ube44\uc804 \uc2dc\ud2b8\ucf64\uc744 1990\ub144\ub300 \uac10\uc131\uc73c\ub85c \ubc14\uafd4\ub193\uc740 10\ub300 \ub0a8\ub9e4\uc758 \uc774\uc57c\uae30\ub85c, \uadf8\ub140\uc758 \uc5ed\ud560\uc740 \uc678\ubaa8\uc640 \uce5c\uad6c\ub4e4, \uc778\uae30 \uc21c\uc704\ub97c \uace0\ubbfc\ud558\ub358 \uc5b4\ub9b0 \uc18c\ub140\uc778 \ub3d9\uc0dd \uc81c\ub2c8\ud37c\uc600\ub2e4. \uc774 \uc5f0\uae30\ub85c \ud560\ub9ac\uc6b0\ub4dc \uc80a\uc740 \uc5f0\uae30\uc790\uc0c1(Young Hollywood Award)\uc744 \uc218\uc0c1\ud588\ub2e4. \uac10\ub3c5\uc774\ub358 \uac8c\ub9ac \ub85c\uc2a4\ub294 \ub9ac\uc988\uac00 \ub6f0\uc5b4\ub09c \uc601\ud654 \ubc30\uc6b0\uac00 \ub420 \uac83\uc744 \uad73\uac74\ud788 \ubbff\uace0 \uc788\uc5c8\ub2e4\uace0 \uce6d\ucc2c\ud588\ub2e4."} {"question": "\uc77c\ubcf8 \uc815\ubd80\uc5d0\uc11c \ub178\ub3d9\ub825 \ubd80\uc871\uc744 \uc774\uc720\ub85c \uc77c\ubcf8\uacc4 \ube0c\ub77c\uc9c8\uc778\ub4e4\uc744 \ud658\uc601\ud574 \uc774\ubbfc\uc790\uc758 \ud6c4\uc190\uc5d0\uac8c \ubd80\uc5ec\ud558\ub294 \uac83\uc740?", "paragraph": "\uc77c\ubcf8\uacc4 \ube0c\ub77c\uc9c8\uc778\uc758 \uc77c\ubd80\ub294 \uc138\uacc4\uc801\uc778 \uacbd\uc81c\ub300\uad6d\uc774 \ub41c \uc870\uc0c1\uc758 \ub098\ub77c \uc77c\ubcf8\uc73c\ub85c \uacbd\uc81c\uc801\uc778 \ubaa9\uc801\uc73c\ub85c \uac74\ub108\uac00\uace0 \uc788\ub2e4. \uc77c\ubcf8 \uc815\ubd80\uc5d0\uc11c\ub3c4 \ub178\ub3d9\ub825 \ubd80\uc871\uc744 \uc774\uc720\ub85c \uc774\ub4e4\uc744 \ud658\uc601\ud558\uace0 \uc788\uc73c\uba70, \uc774\ubbfc\uc790 \ud6c4\uc190\uc5d0\uac8c \uc601\uc8fc\uad8c\uc744 \ubd80\uc5ec\ud560 \uc218 \uc788\ub3c4\ub85d \ud558\uc600\ub2e4. \uc774\ub4e4\uc758 \uc0c1\ub2f9\uc218\ub294 \uc77c\ubcf8\uc778\ub4e4\uc774 \uae30\ud53c\ud558\ub294 \uc18c\uc704 3D \uc5c5\uc885 \ub4f1\uc758 \ub178\ub3d9\uc5d0 \uc885\uc0ac\ud558\uace0 \uc788\ub2e4. \uc774\ub4e4\uc758 \uc218\ub294 \ucd5c\uadfc \uae09\uaca9\ud788 \uc99d\uac00\ud558\uc5ec \uc77c\ubcf8 \ub0b4\uc5d0 \ubd84\ud3ec\ud558\ub294 \uc678\uad6d\uacc4 \uc8fc\ubbfc \uc911 \ube0c\ub77c\uc9c8\uacc4\uac00 30\ub9cc \uba85\uc5d0 \ub2ec\ud558\uc5ec \ud55c\uad6d\uacc4\uc640 \uc911\uad6d\uacc4 \ub2e4\uc74c\uc73c\ub85c \uadf8 \uc218\uac00 \ub9ce\uc544\uc84c\ub2e4. \ube0c\ub77c\uc9c8\uacc4\uc758 \uc0c1\ub2f9\uc218\ub294 \ubb3c\ub860 \uc77c\ubcf8\uacc4 \ud608\ud1b5\uc744 \uc9c0\ub154\uc73c\uba70, \uc77c\ubcf8\uc73c\ub85c \uac74\ub108\uc628 \uc774\ub4e4 \uc77c\ubcf8\uacc4 \ube0c\ub77c\uc9c8\uc778\uc758 \uc0c1\ub2f9\uc218\ub294 \uc77c\ubcf8\uc5b4\ub97c \ud560 \uc218 \uc5c6\uc5b4 \ucde8\uc5c5\uc5d0 \uc5b4\ub824\uc6c0\uc744 \uacaa\uc744 \uc218\ubc16\uc5d0 \uc5c6\uc73c\uba70, \uc790\ub140\ub4e4\ub3c4 \uc77c\ubcf8 \ud559\uad50\uc5d0 \uce5c\uc219\ud574 \uc9c0\uc9c0 \ubabb\ud558\ub294 \uacbd\uc6b0\uac00 \ub300\ubd80\ubd84\uc774\uace0, \uc544\uc608 \ucde8\ud559\ub3c4 \ud560 \uc218 \uc5c6\ub294 \uacbd\uc6b0\ub3c4 \ub9ce\ub2e4. \uc774\uc5d0 \uc774\ub4e4\uc758 \uc77c\ubcf8\uc5b4 \uad50\uc721 \uae30\ud68c\ub97c \ud655\ub300\ud558\uace0 \uc0ac\ud68c\ubcf4\uc7a5\uc81c\ub3c4\ub97c \uc815\ube44\ud558\ub294 \ub4f1\uc758 \ub178\ub825\uc744 \ud558\uace0 \uc788\uc73c\ub098, \uc774\ub4e4\uc744 \ucc28\ubcc4\ud558\ub294 \uc0ac\ub78c\uae4c\uc9c0 \uc0dd\uae30\ub294 \ub4f1 \uc0ac\ud68c\ubb38\uc81c\ud654\ub418\uace0 \uc788\ub2e4.", "answer": "\uc601\uc8fc\uad8c", "sentence": "\uc774\ubbfc\uc790 \ud6c4\uc190\uc5d0\uac8c \uc601\uc8fc\uad8c \uc744 \ubd80\uc5ec\ud560 \uc218 \uc788\ub3c4\ub85d \ud558\uc600\ub2e4.", "paragraph_sentence": "\uc77c\ubcf8\uacc4 \ube0c\ub77c\uc9c8\uc778\uc758 \uc77c\ubd80\ub294 \uc138\uacc4\uc801\uc778 \uacbd\uc81c\ub300\uad6d\uc774 \ub41c \uc870\uc0c1\uc758 \ub098\ub77c \uc77c\ubcf8\uc73c\ub85c \uacbd\uc81c\uc801\uc778 \ubaa9\uc801\uc73c\ub85c \uac74\ub108\uac00\uace0 \uc788\ub2e4. \uc77c\ubcf8 \uc815\ubd80\uc5d0\uc11c\ub3c4 \ub178\ub3d9\ub825 \ubd80\uc871\uc744 \uc774\uc720\ub85c \uc774\ub4e4\uc744 \ud658\uc601\ud558\uace0 \uc788\uc73c\uba70, \uc774\ubbfc\uc790 \ud6c4\uc190\uc5d0\uac8c \uc601\uc8fc\uad8c \uc744 \ubd80\uc5ec\ud560 \uc218 \uc788\ub3c4\ub85d \ud558\uc600\ub2e4. \uc774\ub4e4\uc758 \uc0c1\ub2f9\uc218\ub294 \uc77c\ubcf8\uc778\ub4e4\uc774 \uae30\ud53c\ud558\ub294 \uc18c\uc704 3D \uc5c5\uc885 \ub4f1\uc758 \ub178\ub3d9\uc5d0 \uc885\uc0ac\ud558\uace0 \uc788\ub2e4. \uc774\ub4e4\uc758 \uc218\ub294 \ucd5c\uadfc \uae09\uaca9\ud788 \uc99d\uac00\ud558\uc5ec \uc77c\ubcf8 \ub0b4\uc5d0 \ubd84\ud3ec\ud558\ub294 \uc678\uad6d\uacc4 \uc8fc\ubbfc \uc911 \ube0c\ub77c\uc9c8\uacc4\uac00 30\ub9cc \uba85\uc5d0 \ub2ec\ud558\uc5ec \ud55c\uad6d\uacc4\uc640 \uc911\uad6d\uacc4 \ub2e4\uc74c\uc73c\ub85c \uadf8 \uc218\uac00 \ub9ce\uc544\uc84c\ub2e4. \ube0c\ub77c\uc9c8\uacc4\uc758 \uc0c1\ub2f9\uc218\ub294 \ubb3c\ub860 \uc77c\ubcf8\uacc4 \ud608\ud1b5\uc744 \uc9c0\ub154\uc73c\uba70, \uc77c\ubcf8\uc73c\ub85c \uac74\ub108\uc628 \uc774\ub4e4 \uc77c\ubcf8\uacc4 \ube0c\ub77c\uc9c8\uc778\uc758 \uc0c1\ub2f9\uc218\ub294 \uc77c\ubcf8\uc5b4\ub97c \ud560 \uc218 \uc5c6\uc5b4 \ucde8\uc5c5\uc5d0 \uc5b4\ub824\uc6c0\uc744 \uacaa\uc744 \uc218\ubc16\uc5d0 \uc5c6\uc73c\uba70, \uc790\ub140\ub4e4\ub3c4 \uc77c\ubcf8 \ud559\uad50\uc5d0 \uce5c\uc219\ud574 \uc9c0\uc9c0 \ubabb\ud558\ub294 \uacbd\uc6b0\uac00 \ub300\ubd80\ubd84\uc774\uace0, \uc544\uc608 \ucde8\ud559\ub3c4 \ud560 \uc218 \uc5c6\ub294 \uacbd\uc6b0\ub3c4 \ub9ce\ub2e4. \uc774\uc5d0 \uc774\ub4e4\uc758 \uc77c\ubcf8\uc5b4 \uad50\uc721 \uae30\ud68c\ub97c \ud655\ub300\ud558\uace0 \uc0ac\ud68c\ubcf4\uc7a5\uc81c\ub3c4\ub97c \uc815\ube44\ud558\ub294 \ub4f1\uc758 \ub178\ub825\uc744 \ud558\uace0 \uc788\uc73c\ub098, \uc774\ub4e4\uc744 \ucc28\ubcc4\ud558\ub294 \uc0ac\ub78c\uae4c\uc9c0 \uc0dd\uae30\ub294 \ub4f1 \uc0ac\ud68c\ubb38\uc81c\ud654\ub418\uace0 \uc788\ub2e4.", "paragraph_answer": "\uc77c\ubcf8\uacc4 \ube0c\ub77c\uc9c8\uc778\uc758 \uc77c\ubd80\ub294 \uc138\uacc4\uc801\uc778 \uacbd\uc81c\ub300\uad6d\uc774 \ub41c \uc870\uc0c1\uc758 \ub098\ub77c \uc77c\ubcf8\uc73c\ub85c \uacbd\uc81c\uc801\uc778 \ubaa9\uc801\uc73c\ub85c \uac74\ub108\uac00\uace0 \uc788\ub2e4. \uc77c\ubcf8 \uc815\ubd80\uc5d0\uc11c\ub3c4 \ub178\ub3d9\ub825 \ubd80\uc871\uc744 \uc774\uc720\ub85c \uc774\ub4e4\uc744 \ud658\uc601\ud558\uace0 \uc788\uc73c\uba70, \uc774\ubbfc\uc790 \ud6c4\uc190\uc5d0\uac8c \uc601\uc8fc\uad8c \uc744 \ubd80\uc5ec\ud560 \uc218 \uc788\ub3c4\ub85d \ud558\uc600\ub2e4. \uc774\ub4e4\uc758 \uc0c1\ub2f9\uc218\ub294 \uc77c\ubcf8\uc778\ub4e4\uc774 \uae30\ud53c\ud558\ub294 \uc18c\uc704 3D \uc5c5\uc885 \ub4f1\uc758 \ub178\ub3d9\uc5d0 \uc885\uc0ac\ud558\uace0 \uc788\ub2e4. \uc774\ub4e4\uc758 \uc218\ub294 \ucd5c\uadfc \uae09\uaca9\ud788 \uc99d\uac00\ud558\uc5ec \uc77c\ubcf8 \ub0b4\uc5d0 \ubd84\ud3ec\ud558\ub294 \uc678\uad6d\uacc4 \uc8fc\ubbfc \uc911 \ube0c\ub77c\uc9c8\uacc4\uac00 30\ub9cc \uba85\uc5d0 \ub2ec\ud558\uc5ec \ud55c\uad6d\uacc4\uc640 \uc911\uad6d\uacc4 \ub2e4\uc74c\uc73c\ub85c \uadf8 \uc218\uac00 \ub9ce\uc544\uc84c\ub2e4. \ube0c\ub77c\uc9c8\uacc4\uc758 \uc0c1\ub2f9\uc218\ub294 \ubb3c\ub860 \uc77c\ubcf8\uacc4 \ud608\ud1b5\uc744 \uc9c0\ub154\uc73c\uba70, \uc77c\ubcf8\uc73c\ub85c \uac74\ub108\uc628 \uc774\ub4e4 \uc77c\ubcf8\uacc4 \ube0c\ub77c\uc9c8\uc778\uc758 \uc0c1\ub2f9\uc218\ub294 \uc77c\ubcf8\uc5b4\ub97c \ud560 \uc218 \uc5c6\uc5b4 \ucde8\uc5c5\uc5d0 \uc5b4\ub824\uc6c0\uc744 \uacaa\uc744 \uc218\ubc16\uc5d0 \uc5c6\uc73c\uba70, \uc790\ub140\ub4e4\ub3c4 \uc77c\ubcf8 \ud559\uad50\uc5d0 \uce5c\uc219\ud574 \uc9c0\uc9c0 \ubabb\ud558\ub294 \uacbd\uc6b0\uac00 \ub300\ubd80\ubd84\uc774\uace0, \uc544\uc608 \ucde8\ud559\ub3c4 \ud560 \uc218 \uc5c6\ub294 \uacbd\uc6b0\ub3c4 \ub9ce\ub2e4. \uc774\uc5d0 \uc774\ub4e4\uc758 \uc77c\ubcf8\uc5b4 \uad50\uc721 \uae30\ud68c\ub97c \ud655\ub300\ud558\uace0 \uc0ac\ud68c\ubcf4\uc7a5\uc81c\ub3c4\ub97c \uc815\ube44\ud558\ub294 \ub4f1\uc758 \ub178\ub825\uc744 \ud558\uace0 \uc788\uc73c\ub098, \uc774\ub4e4\uc744 \ucc28\ubcc4\ud558\ub294 \uc0ac\ub78c\uae4c\uc9c0 \uc0dd\uae30\ub294 \ub4f1 \uc0ac\ud68c\ubb38\uc81c\ud654\ub418\uace0 \uc788\ub2e4.", "sentence_answer": "\uc774\ubbfc\uc790 \ud6c4\uc190\uc5d0\uac8c \uc601\uc8fc\uad8c \uc744 \ubd80\uc5ec\ud560 \uc218 \uc788\ub3c4\ub85d \ud558\uc600\ub2e4.", "paragraph_id": "6548437-2-1", "paragraph_question": "question: \uc77c\ubcf8 \uc815\ubd80\uc5d0\uc11c \ub178\ub3d9\ub825 \ubd80\uc871\uc744 \uc774\uc720\ub85c \uc77c\ubcf8\uacc4 \ube0c\ub77c\uc9c8\uc778\ub4e4\uc744 \ud658\uc601\ud574 \uc774\ubbfc\uc790\uc758 \ud6c4\uc190\uc5d0\uac8c \ubd80\uc5ec\ud558\ub294 \uac83\uc740?, context: \uc77c\ubcf8\uacc4 \ube0c\ub77c\uc9c8\uc778\uc758 \uc77c\ubd80\ub294 \uc138\uacc4\uc801\uc778 \uacbd\uc81c\ub300\uad6d\uc774 \ub41c \uc870\uc0c1\uc758 \ub098\ub77c \uc77c\ubcf8\uc73c\ub85c \uacbd\uc81c\uc801\uc778 \ubaa9\uc801\uc73c\ub85c \uac74\ub108\uac00\uace0 \uc788\ub2e4. \uc77c\ubcf8 \uc815\ubd80\uc5d0\uc11c\ub3c4 \ub178\ub3d9\ub825 \ubd80\uc871\uc744 \uc774\uc720\ub85c \uc774\ub4e4\uc744 \ud658\uc601\ud558\uace0 \uc788\uc73c\uba70, \uc774\ubbfc\uc790 \ud6c4\uc190\uc5d0\uac8c \uc601\uc8fc\uad8c\uc744 \ubd80\uc5ec\ud560 \uc218 \uc788\ub3c4\ub85d \ud558\uc600\ub2e4. \uc774\ub4e4\uc758 \uc0c1\ub2f9\uc218\ub294 \uc77c\ubcf8\uc778\ub4e4\uc774 \uae30\ud53c\ud558\ub294 \uc18c\uc704 3D \uc5c5\uc885 \ub4f1\uc758 \ub178\ub3d9\uc5d0 \uc885\uc0ac\ud558\uace0 \uc788\ub2e4. \uc774\ub4e4\uc758 \uc218\ub294 \ucd5c\uadfc \uae09\uaca9\ud788 \uc99d\uac00\ud558\uc5ec \uc77c\ubcf8 \ub0b4\uc5d0 \ubd84\ud3ec\ud558\ub294 \uc678\uad6d\uacc4 \uc8fc\ubbfc \uc911 \ube0c\ub77c\uc9c8\uacc4\uac00 30\ub9cc \uba85\uc5d0 \ub2ec\ud558\uc5ec \ud55c\uad6d\uacc4\uc640 \uc911\uad6d\uacc4 \ub2e4\uc74c\uc73c\ub85c \uadf8 \uc218\uac00 \ub9ce\uc544\uc84c\ub2e4. \ube0c\ub77c\uc9c8\uacc4\uc758 \uc0c1\ub2f9\uc218\ub294 \ubb3c\ub860 \uc77c\ubcf8\uacc4 \ud608\ud1b5\uc744 \uc9c0\ub154\uc73c\uba70, \uc77c\ubcf8\uc73c\ub85c \uac74\ub108\uc628 \uc774\ub4e4 \uc77c\ubcf8\uacc4 \ube0c\ub77c\uc9c8\uc778\uc758 \uc0c1\ub2f9\uc218\ub294 \uc77c\ubcf8\uc5b4\ub97c \ud560 \uc218 \uc5c6\uc5b4 \ucde8\uc5c5\uc5d0 \uc5b4\ub824\uc6c0\uc744 \uacaa\uc744 \uc218\ubc16\uc5d0 \uc5c6\uc73c\uba70, \uc790\ub140\ub4e4\ub3c4 \uc77c\ubcf8 \ud559\uad50\uc5d0 \uce5c\uc219\ud574 \uc9c0\uc9c0 \ubabb\ud558\ub294 \uacbd\uc6b0\uac00 \ub300\ubd80\ubd84\uc774\uace0, \uc544\uc608 \ucde8\ud559\ub3c4 \ud560 \uc218 \uc5c6\ub294 \uacbd\uc6b0\ub3c4 \ub9ce\ub2e4. \uc774\uc5d0 \uc774\ub4e4\uc758 \uc77c\ubcf8\uc5b4 \uad50\uc721 \uae30\ud68c\ub97c \ud655\ub300\ud558\uace0 \uc0ac\ud68c\ubcf4\uc7a5\uc81c\ub3c4\ub97c \uc815\ube44\ud558\ub294 \ub4f1\uc758 \ub178\ub825\uc744 \ud558\uace0 \uc788\uc73c\ub098, \uc774\ub4e4\uc744 \ucc28\ubcc4\ud558\ub294 \uc0ac\ub78c\uae4c\uc9c0 \uc0dd\uae30\ub294 \ub4f1 \uc0ac\ud68c\ubb38\uc81c\ud654\ub418\uace0 \uc788\ub2e4."} {"question": "\uc800\uc0c1 \uc2b9\uac15\uc7a5\uacfc \uace0\uc0c1 \uc2b9\uc7a5\uac15 \ubaa8\ub450\uc5d0\uc11c \uc2b9\ud558\ucc28\uac00 \uac00\ub2a5\ud558\ub3c4\ub85d \ud0d1\uc7ac\ud55c \uac83\uc740?", "paragraph": "\uc11c\uc6b8-\uc2e0\ucc3d \uac04 \ub178\uc120\uc758 \uacbd\uc6b0, \uc218\ub3c4\uad8c \uc804\ucca0 \uad6c\uac04\uc5d0\uc11c \uc9d1\uc911\uc801\uc73c\ub85c \uc6b4\uc6a9\ub418\uae30 \ub54c\ubb38\uc5d0, \uc800\uc0c1 \uc2b9\uac15\uc7a5\uacfc \uace0\uc0c1 \uc2b9\uac15\uc7a5 \ubaa8\ub450\uc5d0\uc11c \uc2b9\ud558\ucc28\uac00 \uac00\ub2a5\ud558\ub3c4\ub85d \uc720\uc555\uc2dd \ub9ac\ud504\ud2b8 \uc7a5\uce58\ub97c \ud0d1\uc7ac\ud558\uc600\uace0, \ubc30\ub9ac\uc5b4 \ud504\ub9ac\ub97c \uc704\ud574 2\ud638\ucc28\uc5d0\ub294 \uc7a5\uc560\uc778 \ud0d1\uc2b9\uc6a9 \uacbd\uc0ac\ub85c \uc7a5\uce58\ub97c \ud0d1\uc7ac\ud558\uc600\ub2e4. \uc2e0\ucc3d\uc5ed\uacfc \uc11c\uc6b8\uc5ed\uc744 \uc81c\uc678\ud55c \ubaa8\ub4e0 \uc5ed\uc5d0\uc11c \uc800\uc0c1 \uc2b9\uac15\uc7a5\uc744 \uc774\uc6a9\ud558\ub098 \ubaa8\ub4e0 \uc5f4\ucc28\uac00 \uc11c\uc6b8\uc5ed\uc5d0\uc11c \uace0\uc0c1 \uc2b9\uac15\uc7a5\ub9cc\uc744 \uc0ac\uc6a9\ud558\ub294 \uac83\uc740 \uc544\ub2c8\ub2e4. \uc2b9\uac15\uad6c\uc758 \ucc28\ub0b4 \ubd80\ubd84\uc5d0 \ub178\ub780\uc0c9\uc73c\ub85c \ud45c\uc2dc\ub41c \ubd80\ubd84 \uc548\ucabd\uc5d0\ub294 \uc800\uc0c1 \uc2b9\uac15\uc7a5 \uc815\ucc28 \uc2dc \uacc4\ub2e8\uc774 \ub0b4\ub824\uac00\ub3c4\ub85d \ub41c \ubd80\ubd84\uc73c\ub85c, \uc774 \ubd80\ubd84\uc5d0\ub294 \uc13c\uc11c\uac00 \uc788\uc5b4\uc11c \uc800\uc0c1 \uc2b9\uac15\uc7a5 \uc815\ucc28 \uc2dc \uc7a5\uc560\ubb3c\uc774 \uc788\uc744 \uacbd\uc6b0 \ub0b4\ub824\uac00\uc9c0 \uc54a\uc73c\uba70, \ubc18\ub300\ub85c \ubb38\uc744 \ub2eb\uc744 \ub54c\uc5d0\ub3c4 \uc2b9\uac1d \ub4f1\uc774 \uc11c \uc788\uc744 \uacbd\uc6b0 \uacc4\ub2e8\uc744 \uc62c\ub9ac\uace0 \ubb38\uc744 \ub2eb\uc744 \uc218 \uc5c6\ub2e4. \ud55c\ud3b8, \uc77c\ubc18\uc801\uc778 \ud1b5\uadfc\ud615 \uc804\ub3d9\ucc28\uc640 \ucc28\ub7c9 \uae38\uc774\uac00 \ub2e4\ub974\uace0, \uc2a4\ud06c\ub9b0\ub3c4\uc5b4\uc640 \ubb38\uc758 \uc704\uce58\uac00 \ub2ec\ub77c\uc11c \uc2a4\ud06c\ub9b0\ub3c4\uc5b4\uac00 \uc124\uce58\ub41c \uc2b9\uac15\uc7a5\uc5d0\uc11c\ub294 \uc5ec\uac1d \ucde8\uae09\uc774 \ubd88\uac00\ub2a5\ud558\ub2e4.", "answer": "\uc720\uc555\uc2dd \ub9ac\ud504\ud2b8 \uc7a5\uce58", "sentence": "\uc11c\uc6b8-\uc2e0\ucc3d \uac04 \ub178\uc120\uc758 \uacbd\uc6b0, \uc218\ub3c4\uad8c \uc804\ucca0 \uad6c\uac04\uc5d0\uc11c \uc9d1\uc911\uc801\uc73c\ub85c \uc6b4\uc6a9\ub418\uae30 \ub54c\ubb38\uc5d0, \uc800\uc0c1 \uc2b9\uac15\uc7a5\uacfc \uace0\uc0c1 \uc2b9\uac15\uc7a5 \ubaa8\ub450\uc5d0\uc11c \uc2b9\ud558\ucc28\uac00 \uac00\ub2a5\ud558\ub3c4\ub85d \uc720\uc555\uc2dd \ub9ac\ud504\ud2b8 \uc7a5\uce58 \ub97c \ud0d1\uc7ac\ud558\uc600\uace0, \ubc30\ub9ac\uc5b4 \ud504\ub9ac\ub97c \uc704\ud574 2\ud638\ucc28\uc5d0\ub294 \uc7a5\uc560\uc778 \ud0d1\uc2b9\uc6a9 \uacbd\uc0ac\ub85c \uc7a5\uce58\ub97c \ud0d1\uc7ac\ud558\uc600\ub2e4.", "paragraph_sentence": " \uc11c\uc6b8-\uc2e0\ucc3d \uac04 \ub178\uc120\uc758 \uacbd\uc6b0, \uc218\ub3c4\uad8c \uc804\ucca0 \uad6c\uac04\uc5d0\uc11c \uc9d1\uc911\uc801\uc73c\ub85c \uc6b4\uc6a9\ub418\uae30 \ub54c\ubb38\uc5d0, \uc800\uc0c1 \uc2b9\uac15\uc7a5\uacfc \uace0\uc0c1 \uc2b9\uac15\uc7a5 \ubaa8\ub450\uc5d0\uc11c \uc2b9\ud558\ucc28\uac00 \uac00\ub2a5\ud558\ub3c4\ub85d \uc720\uc555\uc2dd \ub9ac\ud504\ud2b8 \uc7a5\uce58 \ub97c \ud0d1\uc7ac\ud558\uc600\uace0, \ubc30\ub9ac\uc5b4 \ud504\ub9ac\ub97c \uc704\ud574 2\ud638\ucc28\uc5d0\ub294 \uc7a5\uc560\uc778 \ud0d1\uc2b9\uc6a9 \uacbd\uc0ac\ub85c \uc7a5\uce58\ub97c \ud0d1\uc7ac\ud558\uc600\ub2e4. \uc2e0\ucc3d\uc5ed\uacfc \uc11c\uc6b8\uc5ed\uc744 \uc81c\uc678\ud55c \ubaa8\ub4e0 \uc5ed\uc5d0\uc11c \uc800\uc0c1 \uc2b9\uac15\uc7a5\uc744 \uc774\uc6a9\ud558\ub098 \ubaa8\ub4e0 \uc5f4\ucc28\uac00 \uc11c\uc6b8\uc5ed\uc5d0\uc11c \uace0\uc0c1 \uc2b9\uac15\uc7a5\ub9cc\uc744 \uc0ac\uc6a9\ud558\ub294 \uac83\uc740 \uc544\ub2c8\ub2e4. \uc2b9\uac15\uad6c\uc758 \ucc28\ub0b4 \ubd80\ubd84\uc5d0 \ub178\ub780\uc0c9\uc73c\ub85c \ud45c\uc2dc\ub41c \ubd80\ubd84 \uc548\ucabd\uc5d0\ub294 \uc800\uc0c1 \uc2b9\uac15\uc7a5 \uc815\ucc28 \uc2dc \uacc4\ub2e8\uc774 \ub0b4\ub824\uac00\ub3c4\ub85d \ub41c \ubd80\ubd84\uc73c\ub85c, \uc774 \ubd80\ubd84\uc5d0\ub294 \uc13c\uc11c\uac00 \uc788\uc5b4\uc11c \uc800\uc0c1 \uc2b9\uac15\uc7a5 \uc815\ucc28 \uc2dc \uc7a5\uc560\ubb3c\uc774 \uc788\uc744 \uacbd\uc6b0 \ub0b4\ub824\uac00\uc9c0 \uc54a\uc73c\uba70, \ubc18\ub300\ub85c \ubb38\uc744 \ub2eb\uc744 \ub54c\uc5d0\ub3c4 \uc2b9\uac1d \ub4f1\uc774 \uc11c \uc788\uc744 \uacbd\uc6b0 \uacc4\ub2e8\uc744 \uc62c\ub9ac\uace0 \ubb38\uc744 \ub2eb\uc744 \uc218 \uc5c6\ub2e4. \ud55c\ud3b8, \uc77c\ubc18\uc801\uc778 \ud1b5\uadfc\ud615 \uc804\ub3d9\ucc28\uc640 \ucc28\ub7c9 \uae38\uc774\uac00 \ub2e4\ub974\uace0, \uc2a4\ud06c\ub9b0\ub3c4\uc5b4\uc640 \ubb38\uc758 \uc704\uce58\uac00 \ub2ec\ub77c\uc11c \uc2a4\ud06c\ub9b0\ub3c4\uc5b4\uac00 \uc124\uce58\ub41c \uc2b9\uac15\uc7a5\uc5d0\uc11c\ub294 \uc5ec\uac1d \ucde8\uae09\uc774 \ubd88\uac00\ub2a5\ud558\ub2e4.", "paragraph_answer": "\uc11c\uc6b8-\uc2e0\ucc3d \uac04 \ub178\uc120\uc758 \uacbd\uc6b0, \uc218\ub3c4\uad8c \uc804\ucca0 \uad6c\uac04\uc5d0\uc11c \uc9d1\uc911\uc801\uc73c\ub85c \uc6b4\uc6a9\ub418\uae30 \ub54c\ubb38\uc5d0, \uc800\uc0c1 \uc2b9\uac15\uc7a5\uacfc \uace0\uc0c1 \uc2b9\uac15\uc7a5 \ubaa8\ub450\uc5d0\uc11c \uc2b9\ud558\ucc28\uac00 \uac00\ub2a5\ud558\ub3c4\ub85d \uc720\uc555\uc2dd \ub9ac\ud504\ud2b8 \uc7a5\uce58 \ub97c \ud0d1\uc7ac\ud558\uc600\uace0, \ubc30\ub9ac\uc5b4 \ud504\ub9ac\ub97c \uc704\ud574 2\ud638\ucc28\uc5d0\ub294 \uc7a5\uc560\uc778 \ud0d1\uc2b9\uc6a9 \uacbd\uc0ac\ub85c \uc7a5\uce58\ub97c \ud0d1\uc7ac\ud558\uc600\ub2e4. \uc2e0\ucc3d\uc5ed\uacfc \uc11c\uc6b8\uc5ed\uc744 \uc81c\uc678\ud55c \ubaa8\ub4e0 \uc5ed\uc5d0\uc11c \uc800\uc0c1 \uc2b9\uac15\uc7a5\uc744 \uc774\uc6a9\ud558\ub098 \ubaa8\ub4e0 \uc5f4\ucc28\uac00 \uc11c\uc6b8\uc5ed\uc5d0\uc11c \uace0\uc0c1 \uc2b9\uac15\uc7a5\ub9cc\uc744 \uc0ac\uc6a9\ud558\ub294 \uac83\uc740 \uc544\ub2c8\ub2e4. \uc2b9\uac15\uad6c\uc758 \ucc28\ub0b4 \ubd80\ubd84\uc5d0 \ub178\ub780\uc0c9\uc73c\ub85c \ud45c\uc2dc\ub41c \ubd80\ubd84 \uc548\ucabd\uc5d0\ub294 \uc800\uc0c1 \uc2b9\uac15\uc7a5 \uc815\ucc28 \uc2dc \uacc4\ub2e8\uc774 \ub0b4\ub824\uac00\ub3c4\ub85d \ub41c \ubd80\ubd84\uc73c\ub85c, \uc774 \ubd80\ubd84\uc5d0\ub294 \uc13c\uc11c\uac00 \uc788\uc5b4\uc11c \uc800\uc0c1 \uc2b9\uac15\uc7a5 \uc815\ucc28 \uc2dc \uc7a5\uc560\ubb3c\uc774 \uc788\uc744 \uacbd\uc6b0 \ub0b4\ub824\uac00\uc9c0 \uc54a\uc73c\uba70, \ubc18\ub300\ub85c \ubb38\uc744 \ub2eb\uc744 \ub54c\uc5d0\ub3c4 \uc2b9\uac1d \ub4f1\uc774 \uc11c \uc788\uc744 \uacbd\uc6b0 \uacc4\ub2e8\uc744 \uc62c\ub9ac\uace0 \ubb38\uc744 \ub2eb\uc744 \uc218 \uc5c6\ub2e4. \ud55c\ud3b8, \uc77c\ubc18\uc801\uc778 \ud1b5\uadfc\ud615 \uc804\ub3d9\ucc28\uc640 \ucc28\ub7c9 \uae38\uc774\uac00 \ub2e4\ub974\uace0, \uc2a4\ud06c\ub9b0\ub3c4\uc5b4\uc640 \ubb38\uc758 \uc704\uce58\uac00 \ub2ec\ub77c\uc11c \uc2a4\ud06c\ub9b0\ub3c4\uc5b4\uac00 \uc124\uce58\ub41c \uc2b9\uac15\uc7a5\uc5d0\uc11c\ub294 \uc5ec\uac1d \ucde8\uae09\uc774 \ubd88\uac00\ub2a5\ud558\ub2e4.", "sentence_answer": "\uc11c\uc6b8-\uc2e0\ucc3d \uac04 \ub178\uc120\uc758 \uacbd\uc6b0, \uc218\ub3c4\uad8c \uc804\ucca0 \uad6c\uac04\uc5d0\uc11c \uc9d1\uc911\uc801\uc73c\ub85c \uc6b4\uc6a9\ub418\uae30 \ub54c\ubb38\uc5d0, \uc800\uc0c1 \uc2b9\uac15\uc7a5\uacfc \uace0\uc0c1 \uc2b9\uac15\uc7a5 \ubaa8\ub450\uc5d0\uc11c \uc2b9\ud558\ucc28\uac00 \uac00\ub2a5\ud558\ub3c4\ub85d \uc720\uc555\uc2dd \ub9ac\ud504\ud2b8 \uc7a5\uce58 \ub97c \ud0d1\uc7ac\ud558\uc600\uace0, \ubc30\ub9ac\uc5b4 \ud504\ub9ac\ub97c \uc704\ud574 2\ud638\ucc28\uc5d0\ub294 \uc7a5\uc560\uc778 \ud0d1\uc2b9\uc6a9 \uacbd\uc0ac\ub85c \uc7a5\uce58\ub97c \ud0d1\uc7ac\ud558\uc600\ub2e4.", "paragraph_id": "6569933-3-0", "paragraph_question": "question: \uc800\uc0c1 \uc2b9\uac15\uc7a5\uacfc \uace0\uc0c1 \uc2b9\uc7a5\uac15 \ubaa8\ub450\uc5d0\uc11c \uc2b9\ud558\ucc28\uac00 \uac00\ub2a5\ud558\ub3c4\ub85d \ud0d1\uc7ac\ud55c \uac83\uc740?, context: \uc11c\uc6b8-\uc2e0\ucc3d \uac04 \ub178\uc120\uc758 \uacbd\uc6b0, \uc218\ub3c4\uad8c \uc804\ucca0 \uad6c\uac04\uc5d0\uc11c \uc9d1\uc911\uc801\uc73c\ub85c \uc6b4\uc6a9\ub418\uae30 \ub54c\ubb38\uc5d0, \uc800\uc0c1 \uc2b9\uac15\uc7a5\uacfc \uace0\uc0c1 \uc2b9\uac15\uc7a5 \ubaa8\ub450\uc5d0\uc11c \uc2b9\ud558\ucc28\uac00 \uac00\ub2a5\ud558\ub3c4\ub85d \uc720\uc555\uc2dd \ub9ac\ud504\ud2b8 \uc7a5\uce58\ub97c \ud0d1\uc7ac\ud558\uc600\uace0, \ubc30\ub9ac\uc5b4 \ud504\ub9ac\ub97c \uc704\ud574 2\ud638\ucc28\uc5d0\ub294 \uc7a5\uc560\uc778 \ud0d1\uc2b9\uc6a9 \uacbd\uc0ac\ub85c \uc7a5\uce58\ub97c \ud0d1\uc7ac\ud558\uc600\ub2e4. \uc2e0\ucc3d\uc5ed\uacfc \uc11c\uc6b8\uc5ed\uc744 \uc81c\uc678\ud55c \ubaa8\ub4e0 \uc5ed\uc5d0\uc11c \uc800\uc0c1 \uc2b9\uac15\uc7a5\uc744 \uc774\uc6a9\ud558\ub098 \ubaa8\ub4e0 \uc5f4\ucc28\uac00 \uc11c\uc6b8\uc5ed\uc5d0\uc11c \uace0\uc0c1 \uc2b9\uac15\uc7a5\ub9cc\uc744 \uc0ac\uc6a9\ud558\ub294 \uac83\uc740 \uc544\ub2c8\ub2e4. \uc2b9\uac15\uad6c\uc758 \ucc28\ub0b4 \ubd80\ubd84\uc5d0 \ub178\ub780\uc0c9\uc73c\ub85c \ud45c\uc2dc\ub41c \ubd80\ubd84 \uc548\ucabd\uc5d0\ub294 \uc800\uc0c1 \uc2b9\uac15\uc7a5 \uc815\ucc28 \uc2dc \uacc4\ub2e8\uc774 \ub0b4\ub824\uac00\ub3c4\ub85d \ub41c \ubd80\ubd84\uc73c\ub85c, \uc774 \ubd80\ubd84\uc5d0\ub294 \uc13c\uc11c\uac00 \uc788\uc5b4\uc11c \uc800\uc0c1 \uc2b9\uac15\uc7a5 \uc815\ucc28 \uc2dc \uc7a5\uc560\ubb3c\uc774 \uc788\uc744 \uacbd\uc6b0 \ub0b4\ub824\uac00\uc9c0 \uc54a\uc73c\uba70, \ubc18\ub300\ub85c \ubb38\uc744 \ub2eb\uc744 \ub54c\uc5d0\ub3c4 \uc2b9\uac1d \ub4f1\uc774 \uc11c \uc788\uc744 \uacbd\uc6b0 \uacc4\ub2e8\uc744 \uc62c\ub9ac\uace0 \ubb38\uc744 \ub2eb\uc744 \uc218 \uc5c6\ub2e4. \ud55c\ud3b8, \uc77c\ubc18\uc801\uc778 \ud1b5\uadfc\ud615 \uc804\ub3d9\ucc28\uc640 \ucc28\ub7c9 \uae38\uc774\uac00 \ub2e4\ub974\uace0, \uc2a4\ud06c\ub9b0\ub3c4\uc5b4\uc640 \ubb38\uc758 \uc704\uce58\uac00 \ub2ec\ub77c\uc11c \uc2a4\ud06c\ub9b0\ub3c4\uc5b4\uac00 \uc124\uce58\ub41c \uc2b9\uac15\uc7a5\uc5d0\uc11c\ub294 \uc5ec\uac1d \ucde8\uae09\uc774 \ubd88\uac00\ub2a5\ud558\ub2e4."} {"question": "\uc774\uba85\ubc15 \uc804 \uc11c\uc6b8\uc2dc\uc7a5\uc774 \uc790\uc2e0\uc758 \uc784\uae30 \uc548\uc5d0 \uccad\uacc4\ucc9c \ubcf5\uc6d0 \uc0ac\uc5c5\uc744 \ub05d\ub9c8\uce58\uae30 \uc704\ud574 \ub0a0\ub9bc\uc73c\ub85c \uc9c4\ud589 \ud588\ub2e4\ub294 \ud3c9\uac00\ub97c \ubc1b\ub294 \uac83\uc740?", "paragraph": "\ubc18\uba74 \ubd80\uc815\uc801 \ud3c9\uac00\ub3c4 \ub9ce\ub2e4. \uacbd\uc81c\ud559\uc790 \uc6b0\uc11d\ud6c8\uc740 \uc790\uc2e0\uc758 \uc800\uc11c\uc5d0\uc11c \ubcf5\uc6d0\ub41c \uccad\uacc4\ucc9c\uc744 \"\uc778\uacf5\ud638\uc218\"\ub77c\uba70 \ud638\ub418\uac8c \ube44\ud310\ud558\uc600\ub2e4. \ub610\ud55c \uae30\uc874\uc5d0 \ud750\ub974\ub294 \ubb3c\uc758 \uc591\uc774 \uc801\uc5b4 \ucde8\uc218\uc7a5\uc5d0\uc11c \uc804\uae30\ub97c \uc774\uc6a9\ud574 \ud55c\uac15\ubb3c\uacfc \uc9c0\ud558\uc218\ub97c \ub04c\uc5b4\ub2e4 \uccad\uacc4\uad11\uc7a5\uc5d0\uc11c \ud758\ub824\ubcf4\ub0b4\ub294\ub370, \uc774\ub54c \ubb3c\uc744 \ub04c\uc5b4\uc62c\ub9ac\ub294 \ub370 \uc4f0\uc774\ub294 \uc804\uae30\uc694\uae08\uc774 \ud55c\ud574 \ud3c9\uade0 8\uc5b5 \uac00\ub7c9\uc73c\ub85c \uc0c1\ub2f9\ud558\uba70 \ub179\uc870 \ub610\ud55c \uc9c0\uc18d\uc801\uc73c\ub85c \ubc1c\uc0dd\ud558\uace0 \uc788\uc5b4 2007 ~ 2009\ub144\uae4c\uc9c0 8000\ub9cc\uc6d0\uc744 \ub179\uc870\uc81c\uac70\ub97c \uc704\ud574 \uc4f0\uc600\ub2e4. \uc774 \uc678\uc5d0\ub3c4 \uc778\uac74\ube44, \uccad\uc18c\u00b7\uacbd\ube44\uc6a9\uc5ed\ube44, \ud1a0\ubaa9\u00b7\uc870\uacbd\uc2dc\uc124 \uc720\uc9c0\uad00\ub9ac\ube44 \ub4f1\uc5d0 2011\ub144\uc5d0\ub9cc 80\uc5b5\uc5ec\uc6d0\uc774 \uc18c\ubaa8\ub418\uc5c8\ub2e4. \uc774\ub294 2005\ub144 37\uc5b5\uc5d0\uc11c 2010\ub144 77\uc5b5\uc6d0\uc5d0 \uc774\uc740 \uac83\uc73c\ub85c \uafb8\uc900\ud788 \ub298\uc5b4\ub098\uace0 \uc788\ub2e4. 2011\ub144 6\uc6d4 \uc870\uc120\uc77c\ubcf4\uc758 \uc124\ubb38 \uc870\uc0ac\uc5d0\uc11c \ucd5c\uc545\uc758 \uac74\ucd95\ubb3c 3\uc704\uc5d0 \ubf51\ud614\ub294\ub370, \"\uccad\uacc4\ucc9c\uc740 \ud55c\uad6d\uc744 \uc0c1\uc9d5\ud558\ub294 \uacf5\uacf5 \uac74\ucd95\ubb3c\uc778\ub370 \uac74\ucd95\uac00\ub3c4 \uc5c6\uc774 \ub2e8\uae30\uac04\uc5d0 \ubd88\ub3c4\uc800\ub85c \ubc00\uc5b4\ubd99\uc774\ub4ef \ub9cc\ub4e4\uc5c8\ub2e4\"\ub294 \uc774\uc720\uc5d0\uc11c\uc600\ub2e4. \ub610 \uc774\uba85\ubc15 \uc804 \uc11c\uc6b8\uc2dc\uc7a5\uc774 \uc790\uc2e0\uc758 \uc784\uae30 \uc548\uc5d0 \uccad\uacc4\ucc9c \ubcf5\uc6d0 \uc0ac\uc5c5\uc744 \ubb34\ub9ac\ud558\uac8c \ub05d\ub9c8\uce58\uae30 \uc704\ud558\uc5ec \uc720\uc801 \ubcf5\uc6d0\uc744 \ub0a0\ub9bc\uc73c\ub85c \uc9c4\ud589\ud588\ub2e4\ub294 \ud3c9\uac00\ub3c4 \uc788\ub294\ub370, \uc2e4\uc81c\ub85c \uc11d\ucd95\u00b7\ud6a8\ub839\uad50\u00b7\ud558\ub791\uad50\u00b7\uc624\uac04\uc218\ubb38\uc740 \ubcf5\uc6d0\ub418\uc9c0 \uc54a\uace0 \uc911\ub791\uad6c \ud558\uc218\uc885\ub9d0\ucc98\ub9ac\uc7a5\uc5d0 \ubcf4\uad00\ub418\uace0 \uc788\ub2e4. \ub610\ud55c 1773\ub144\uc5d0 \uc313\uc740 \ud604 \ubb34\uad50\ub3d9 \uc0ac\uac70\ub9ac\uc5d0\uc11c \uc2dc\uc791\ub418\uba70 \uccad\uacc4\ucc9c\uc744 \ub530\ub77c 17.1\ubbf8\ud130\uc758 \ud3ed\uc744 \ub450\uace0 \uc591\ucabd\uc73c\ub85c \uae38\uac8c \ub298\uc5b4\uc11c \uc788\ub294 \uccad\uacc4\ucc9c \uc11d\ucd95\uc740 \ubc1c\uad74 \ud6c4 \ubcf5\uc6d0\ub418\uc9c0 \ubabb\ud588\ub2e4. \ubcf5\uc6d0 \ub2f9\uc2dc \ubc1c\uacac\ub41c \uc11d\ucd95\uc740 \ub3d9\uc544\uc77c\ubcf4\uc0ac \uc55e\uc758 \uc88c\uc548\uc11d\ucd95\uc774 \uad50\ucc28\ub85c\ub97c \uae30\uc900\uc73c\ub85c \uc11c\ucabd 57m, \ub3d9\ucabd 36m \uc815\ub3c4, \ubc18\ub300\ud3b8 \uc6b0\uc548\uc11d\ucd95\uc740 9m \uc815\ub3c4\ub85c \ubaa8\ub450 100\uc5ec \ubbf8\ud130\ub098 \ub41c\ub2e4. \uc218\ud45c\uad50\ub294 \uc608\uc0b0\uc0c1\uc758 \ubb38\uc81c\ub77c\ub294 \ud45c\uba74\uc801 \uc774\uc720\ub85c \uc7a5\ucda9\ub2e8\uacf5\uc6d0\uc5d0 \ub0a8\uc544 \uc788\uace0, \uad11\ud1b5\uad50\ub294 \uc6d0\ub798\uc640 \ub2e4\ub974\uac8c \ubcf5\uc6d0\ub418\uc5c8\ub2e4. \uccad\uacc4\ucc9c \uc785\uad6c\uc5d0 \uc704\uce58\ud558\ub294 \uc18c\ub77c\ud0d1 \ub610\ud55c \uc77c\uac01\uc5d0\uc11c \ubb38\uc81c\uac00 \ub418\uace0 \uc788\ub2e4. \uc774\uac83\uc740 \uc2a4\uc6e8\ub374 \ucd9c\uc2e0\uc758 \ubbf8\uad6d \ud31d\uc544\ud2f0\uc2a4\ud2b8 \ud074\ub798\uc2a4 \uc62c\ub374\ubc84\uadf8\uac00 \uc124\uacc4\ud55c \uc791\ud488\uc73c\ub85c, KT\uac00 34\uc5b5\uc758 \ube44\uc6a9\uc744 \ub4e4\uc5ec \uc11c\uc6b8\uc2dc\uc5d0 \uae30\uc99d\ud55c \uc791\ud488\uc774\ub2e4. \ub192\uc774 20m, \ud3ed 6m, \ubb34\uac8c 9t\uc774\ub2e4. \uc77c\ubd80\uc5d0\uc11c\ub294 \ud55c\uad6d\uc801\uc778 \ubbf8\uac00 \uacb0\uc5ec\ub418\uc5b4 \uc788\uace0, \uc8fc\ubcc0\uc5d0 \uc5b4\uc6b8\ub9ac\uc9c0\ub3c4 \uc54a\ub294\ub2e4\ub294 \ube44\ud310\uacfc \ud48d\uc218\uc9c0\ub9ac\ud559\uc801\uc73c\ub85c\ub3c4 \uc88b\uc9c0 \ubabb\ud558\ub2e4\ub294 \uc8fc\uc7a5\uc774 \uc81c\uae30\ub418\uace0 \uc788\ub2e4.", "answer": "\uc720\uc801 \ubcf5\uc6d0", "sentence": "\ub610 \uc774\uba85\ubc15 \uc804 \uc11c\uc6b8\uc2dc\uc7a5\uc774 \uc790\uc2e0\uc758 \uc784\uae30 \uc548\uc5d0 \uccad\uacc4\ucc9c \ubcf5\uc6d0 \uc0ac\uc5c5\uc744 \ubb34\ub9ac\ud558\uac8c \ub05d\ub9c8\uce58\uae30 \uc704\ud558\uc5ec \uc720\uc801 \ubcf5\uc6d0 \uc744 \ub0a0\ub9bc\uc73c\ub85c \uc9c4\ud589\ud588\ub2e4\ub294 \ud3c9\uac00\ub3c4 \uc788\ub294\ub370, \uc2e4\uc81c\ub85c \uc11d\ucd95\u00b7\ud6a8\ub839\uad50\u00b7\ud558\ub791\uad50\u00b7\uc624\uac04\uc218\ubb38\uc740 \ubcf5\uc6d0\ub418\uc9c0 \uc54a\uace0 \uc911\ub791\uad6c \ud558\uc218\uc885\ub9d0\ucc98\ub9ac\uc7a5\uc5d0 \ubcf4\uad00\ub418\uace0 \uc788\ub2e4.", "paragraph_sentence": "\ubc18\uba74 \ubd80\uc815\uc801 \ud3c9\uac00\ub3c4 \ub9ce\ub2e4. \uacbd\uc81c\ud559\uc790 \uc6b0\uc11d\ud6c8\uc740 \uc790\uc2e0\uc758 \uc800\uc11c\uc5d0\uc11c \ubcf5\uc6d0\ub41c \uccad\uacc4\ucc9c\uc744 \"\uc778\uacf5\ud638\uc218\"\ub77c\uba70 \ud638\ub418\uac8c \ube44\ud310\ud558\uc600\ub2e4. \ub610\ud55c \uae30\uc874\uc5d0 \ud750\ub974\ub294 \ubb3c\uc758 \uc591\uc774 \uc801\uc5b4 \ucde8\uc218\uc7a5\uc5d0\uc11c \uc804\uae30\ub97c \uc774\uc6a9\ud574 \ud55c\uac15\ubb3c\uacfc \uc9c0\ud558\uc218\ub97c \ub04c\uc5b4\ub2e4 \uccad\uacc4\uad11\uc7a5\uc5d0\uc11c \ud758\ub824\ubcf4\ub0b4\ub294\ub370, \uc774\ub54c \ubb3c\uc744 \ub04c\uc5b4\uc62c\ub9ac\ub294 \ub370 \uc4f0\uc774\ub294 \uc804\uae30\uc694\uae08\uc774 \ud55c\ud574 \ud3c9\uade0 8\uc5b5 \uac00\ub7c9\uc73c\ub85c \uc0c1\ub2f9\ud558\uba70 \ub179\uc870 \ub610\ud55c \uc9c0\uc18d\uc801\uc73c\ub85c \ubc1c\uc0dd\ud558\uace0 \uc788\uc5b4 2007 ~ 2009\ub144\uae4c\uc9c0 8000\ub9cc\uc6d0\uc744 \ub179\uc870\uc81c\uac70\ub97c \uc704\ud574 \uc4f0\uc600\ub2e4. \uc774 \uc678\uc5d0\ub3c4 \uc778\uac74\ube44, \uccad\uc18c\u00b7\uacbd\ube44\uc6a9\uc5ed\ube44, \ud1a0\ubaa9\u00b7\uc870\uacbd\uc2dc\uc124 \uc720\uc9c0\uad00\ub9ac\ube44 \ub4f1\uc5d0 2011\ub144\uc5d0\ub9cc 80\uc5b5\uc5ec\uc6d0\uc774 \uc18c\ubaa8\ub418\uc5c8\ub2e4. \uc774\ub294 2005\ub144 37\uc5b5\uc5d0\uc11c 2010\ub144 77\uc5b5\uc6d0\uc5d0 \uc774\uc740 \uac83\uc73c\ub85c \uafb8\uc900\ud788 \ub298\uc5b4\ub098\uace0 \uc788\ub2e4. 2011\ub144 6\uc6d4 \uc870\uc120\uc77c\ubcf4\uc758 \uc124\ubb38 \uc870\uc0ac\uc5d0\uc11c \ucd5c\uc545\uc758 \uac74\ucd95\ubb3c 3\uc704\uc5d0 \ubf51\ud614\ub294\ub370, \"\uccad\uacc4\ucc9c\uc740 \ud55c\uad6d\uc744 \uc0c1\uc9d5\ud558\ub294 \uacf5\uacf5 \uac74\ucd95\ubb3c\uc778\ub370 \uac74\ucd95\uac00\ub3c4 \uc5c6\uc774 \ub2e8\uae30\uac04\uc5d0 \ubd88\ub3c4\uc800\ub85c \ubc00\uc5b4\ubd99\uc774\ub4ef \ub9cc\ub4e4\uc5c8\ub2e4\"\ub294 \uc774\uc720\uc5d0\uc11c\uc600\ub2e4. \ub610 \uc774\uba85\ubc15 \uc804 \uc11c\uc6b8\uc2dc\uc7a5\uc774 \uc790\uc2e0\uc758 \uc784\uae30 \uc548\uc5d0 \uccad\uacc4\ucc9c \ubcf5\uc6d0 \uc0ac\uc5c5\uc744 \ubb34\ub9ac\ud558\uac8c \ub05d\ub9c8\uce58\uae30 \uc704\ud558\uc5ec \uc720\uc801 \ubcf5\uc6d0 \uc744 \ub0a0\ub9bc\uc73c\ub85c \uc9c4\ud589\ud588\ub2e4\ub294 \ud3c9\uac00\ub3c4 \uc788\ub294\ub370, \uc2e4\uc81c\ub85c \uc11d\ucd95\u00b7\ud6a8\ub839\uad50\u00b7\ud558\ub791\uad50\u00b7\uc624\uac04\uc218\ubb38\uc740 \ubcf5\uc6d0\ub418\uc9c0 \uc54a\uace0 \uc911\ub791\uad6c \ud558\uc218\uc885\ub9d0\ucc98\ub9ac\uc7a5\uc5d0 \ubcf4\uad00\ub418\uace0 \uc788\ub2e4. \ub610\ud55c 1773\ub144\uc5d0 \uc313\uc740 \ud604 \ubb34\uad50\ub3d9 \uc0ac\uac70\ub9ac\uc5d0\uc11c \uc2dc\uc791\ub418\uba70 \uccad\uacc4\ucc9c\uc744 \ub530\ub77c 17.1\ubbf8\ud130\uc758 \ud3ed\uc744 \ub450\uace0 \uc591\ucabd\uc73c\ub85c \uae38\uac8c \ub298\uc5b4\uc11c \uc788\ub294 \uccad\uacc4\ucc9c \uc11d\ucd95\uc740 \ubc1c\uad74 \ud6c4 \ubcf5\uc6d0\ub418\uc9c0 \ubabb\ud588\ub2e4. \ubcf5\uc6d0 \ub2f9\uc2dc \ubc1c\uacac\ub41c \uc11d\ucd95\uc740 \ub3d9\uc544\uc77c\ubcf4\uc0ac \uc55e\uc758 \uc88c\uc548\uc11d\ucd95\uc774 \uad50\ucc28\ub85c\ub97c \uae30\uc900\uc73c\ub85c \uc11c\ucabd 57m, \ub3d9\ucabd 36m \uc815\ub3c4, \ubc18\ub300\ud3b8 \uc6b0\uc548\uc11d\ucd95\uc740 9m \uc815\ub3c4\ub85c \ubaa8\ub450 100\uc5ec \ubbf8\ud130\ub098 \ub41c\ub2e4. \uc218\ud45c\uad50\ub294 \uc608\uc0b0\uc0c1\uc758 \ubb38\uc81c\ub77c\ub294 \ud45c\uba74\uc801 \uc774\uc720\ub85c \uc7a5\ucda9\ub2e8\uacf5\uc6d0\uc5d0 \ub0a8\uc544 \uc788\uace0, \uad11\ud1b5\uad50\ub294 \uc6d0\ub798\uc640 \ub2e4\ub974\uac8c \ubcf5\uc6d0\ub418\uc5c8\ub2e4. \uccad\uacc4\ucc9c \uc785\uad6c\uc5d0 \uc704\uce58\ud558\ub294 \uc18c\ub77c\ud0d1 \ub610\ud55c \uc77c\uac01\uc5d0\uc11c \ubb38\uc81c\uac00 \ub418\uace0 \uc788\ub2e4. \uc774\uac83\uc740 \uc2a4\uc6e8\ub374 \ucd9c\uc2e0\uc758 \ubbf8\uad6d \ud31d\uc544\ud2f0\uc2a4\ud2b8 \ud074\ub798\uc2a4 \uc62c\ub374\ubc84\uadf8\uac00 \uc124\uacc4\ud55c \uc791\ud488\uc73c\ub85c, KT\uac00 34\uc5b5\uc758 \ube44\uc6a9\uc744 \ub4e4\uc5ec \uc11c\uc6b8\uc2dc\uc5d0 \uae30\uc99d\ud55c \uc791\ud488\uc774\ub2e4. \ub192\uc774 20m, \ud3ed 6m, \ubb34\uac8c 9t\uc774\ub2e4. \uc77c\ubd80\uc5d0\uc11c\ub294 \ud55c\uad6d\uc801\uc778 \ubbf8\uac00 \uacb0\uc5ec\ub418\uc5b4 \uc788\uace0, \uc8fc\ubcc0\uc5d0 \uc5b4\uc6b8\ub9ac\uc9c0\ub3c4 \uc54a\ub294\ub2e4\ub294 \ube44\ud310\uacfc \ud48d\uc218\uc9c0\ub9ac\ud559\uc801\uc73c\ub85c\ub3c4 \uc88b\uc9c0 \ubabb\ud558\ub2e4\ub294 \uc8fc\uc7a5\uc774 \uc81c\uae30\ub418\uace0 \uc788\ub2e4.", "paragraph_answer": "\ubc18\uba74 \ubd80\uc815\uc801 \ud3c9\uac00\ub3c4 \ub9ce\ub2e4. \uacbd\uc81c\ud559\uc790 \uc6b0\uc11d\ud6c8\uc740 \uc790\uc2e0\uc758 \uc800\uc11c\uc5d0\uc11c \ubcf5\uc6d0\ub41c \uccad\uacc4\ucc9c\uc744 \"\uc778\uacf5\ud638\uc218\"\ub77c\uba70 \ud638\ub418\uac8c \ube44\ud310\ud558\uc600\ub2e4. \ub610\ud55c \uae30\uc874\uc5d0 \ud750\ub974\ub294 \ubb3c\uc758 \uc591\uc774 \uc801\uc5b4 \ucde8\uc218\uc7a5\uc5d0\uc11c \uc804\uae30\ub97c \uc774\uc6a9\ud574 \ud55c\uac15\ubb3c\uacfc \uc9c0\ud558\uc218\ub97c \ub04c\uc5b4\ub2e4 \uccad\uacc4\uad11\uc7a5\uc5d0\uc11c \ud758\ub824\ubcf4\ub0b4\ub294\ub370, \uc774\ub54c \ubb3c\uc744 \ub04c\uc5b4\uc62c\ub9ac\ub294 \ub370 \uc4f0\uc774\ub294 \uc804\uae30\uc694\uae08\uc774 \ud55c\ud574 \ud3c9\uade0 8\uc5b5 \uac00\ub7c9\uc73c\ub85c \uc0c1\ub2f9\ud558\uba70 \ub179\uc870 \ub610\ud55c \uc9c0\uc18d\uc801\uc73c\ub85c \ubc1c\uc0dd\ud558\uace0 \uc788\uc5b4 2007 ~ 2009\ub144\uae4c\uc9c0 8000\ub9cc\uc6d0\uc744 \ub179\uc870\uc81c\uac70\ub97c \uc704\ud574 \uc4f0\uc600\ub2e4. \uc774 \uc678\uc5d0\ub3c4 \uc778\uac74\ube44, \uccad\uc18c\u00b7\uacbd\ube44\uc6a9\uc5ed\ube44, \ud1a0\ubaa9\u00b7\uc870\uacbd\uc2dc\uc124 \uc720\uc9c0\uad00\ub9ac\ube44 \ub4f1\uc5d0 2011\ub144\uc5d0\ub9cc 80\uc5b5\uc5ec\uc6d0\uc774 \uc18c\ubaa8\ub418\uc5c8\ub2e4. \uc774\ub294 2005\ub144 37\uc5b5\uc5d0\uc11c 2010\ub144 77\uc5b5\uc6d0\uc5d0 \uc774\uc740 \uac83\uc73c\ub85c \uafb8\uc900\ud788 \ub298\uc5b4\ub098\uace0 \uc788\ub2e4. 2011\ub144 6\uc6d4 \uc870\uc120\uc77c\ubcf4\uc758 \uc124\ubb38 \uc870\uc0ac\uc5d0\uc11c \ucd5c\uc545\uc758 \uac74\ucd95\ubb3c 3\uc704\uc5d0 \ubf51\ud614\ub294\ub370, \"\uccad\uacc4\ucc9c\uc740 \ud55c\uad6d\uc744 \uc0c1\uc9d5\ud558\ub294 \uacf5\uacf5 \uac74\ucd95\ubb3c\uc778\ub370 \uac74\ucd95\uac00\ub3c4 \uc5c6\uc774 \ub2e8\uae30\uac04\uc5d0 \ubd88\ub3c4\uc800\ub85c \ubc00\uc5b4\ubd99\uc774\ub4ef \ub9cc\ub4e4\uc5c8\ub2e4\"\ub294 \uc774\uc720\uc5d0\uc11c\uc600\ub2e4. \ub610 \uc774\uba85\ubc15 \uc804 \uc11c\uc6b8\uc2dc\uc7a5\uc774 \uc790\uc2e0\uc758 \uc784\uae30 \uc548\uc5d0 \uccad\uacc4\ucc9c \ubcf5\uc6d0 \uc0ac\uc5c5\uc744 \ubb34\ub9ac\ud558\uac8c \ub05d\ub9c8\uce58\uae30 \uc704\ud558\uc5ec \uc720\uc801 \ubcf5\uc6d0 \uc744 \ub0a0\ub9bc\uc73c\ub85c \uc9c4\ud589\ud588\ub2e4\ub294 \ud3c9\uac00\ub3c4 \uc788\ub294\ub370, \uc2e4\uc81c\ub85c \uc11d\ucd95\u00b7\ud6a8\ub839\uad50\u00b7\ud558\ub791\uad50\u00b7\uc624\uac04\uc218\ubb38\uc740 \ubcf5\uc6d0\ub418\uc9c0 \uc54a\uace0 \uc911\ub791\uad6c \ud558\uc218\uc885\ub9d0\ucc98\ub9ac\uc7a5\uc5d0 \ubcf4\uad00\ub418\uace0 \uc788\ub2e4. \ub610\ud55c 1773\ub144\uc5d0 \uc313\uc740 \ud604 \ubb34\uad50\ub3d9 \uc0ac\uac70\ub9ac\uc5d0\uc11c \uc2dc\uc791\ub418\uba70 \uccad\uacc4\ucc9c\uc744 \ub530\ub77c 17.1\ubbf8\ud130\uc758 \ud3ed\uc744 \ub450\uace0 \uc591\ucabd\uc73c\ub85c \uae38\uac8c \ub298\uc5b4\uc11c \uc788\ub294 \uccad\uacc4\ucc9c \uc11d\ucd95\uc740 \ubc1c\uad74 \ud6c4 \ubcf5\uc6d0\ub418\uc9c0 \ubabb\ud588\ub2e4. \ubcf5\uc6d0 \ub2f9\uc2dc \ubc1c\uacac\ub41c \uc11d\ucd95\uc740 \ub3d9\uc544\uc77c\ubcf4\uc0ac \uc55e\uc758 \uc88c\uc548\uc11d\ucd95\uc774 \uad50\ucc28\ub85c\ub97c \uae30\uc900\uc73c\ub85c \uc11c\ucabd 57m, \ub3d9\ucabd 36m \uc815\ub3c4, \ubc18\ub300\ud3b8 \uc6b0\uc548\uc11d\ucd95\uc740 9m \uc815\ub3c4\ub85c \ubaa8\ub450 100\uc5ec \ubbf8\ud130\ub098 \ub41c\ub2e4. \uc218\ud45c\uad50\ub294 \uc608\uc0b0\uc0c1\uc758 \ubb38\uc81c\ub77c\ub294 \ud45c\uba74\uc801 \uc774\uc720\ub85c \uc7a5\ucda9\ub2e8\uacf5\uc6d0\uc5d0 \ub0a8\uc544 \uc788\uace0, \uad11\ud1b5\uad50\ub294 \uc6d0\ub798\uc640 \ub2e4\ub974\uac8c \ubcf5\uc6d0\ub418\uc5c8\ub2e4. \uccad\uacc4\ucc9c \uc785\uad6c\uc5d0 \uc704\uce58\ud558\ub294 \uc18c\ub77c\ud0d1 \ub610\ud55c \uc77c\uac01\uc5d0\uc11c \ubb38\uc81c\uac00 \ub418\uace0 \uc788\ub2e4. \uc774\uac83\uc740 \uc2a4\uc6e8\ub374 \ucd9c\uc2e0\uc758 \ubbf8\uad6d \ud31d\uc544\ud2f0\uc2a4\ud2b8 \ud074\ub798\uc2a4 \uc62c\ub374\ubc84\uadf8\uac00 \uc124\uacc4\ud55c \uc791\ud488\uc73c\ub85c, KT\uac00 34\uc5b5\uc758 \ube44\uc6a9\uc744 \ub4e4\uc5ec \uc11c\uc6b8\uc2dc\uc5d0 \uae30\uc99d\ud55c \uc791\ud488\uc774\ub2e4. \ub192\uc774 20m, \ud3ed 6m, \ubb34\uac8c 9t\uc774\ub2e4. \uc77c\ubd80\uc5d0\uc11c\ub294 \ud55c\uad6d\uc801\uc778 \ubbf8\uac00 \uacb0\uc5ec\ub418\uc5b4 \uc788\uace0, \uc8fc\ubcc0\uc5d0 \uc5b4\uc6b8\ub9ac\uc9c0\ub3c4 \uc54a\ub294\ub2e4\ub294 \ube44\ud310\uacfc \ud48d\uc218\uc9c0\ub9ac\ud559\uc801\uc73c\ub85c\ub3c4 \uc88b\uc9c0 \ubabb\ud558\ub2e4\ub294 \uc8fc\uc7a5\uc774 \uc81c\uae30\ub418\uace0 \uc788\ub2e4.", "sentence_answer": "\ub610 \uc774\uba85\ubc15 \uc804 \uc11c\uc6b8\uc2dc\uc7a5\uc774 \uc790\uc2e0\uc758 \uc784\uae30 \uc548\uc5d0 \uccad\uacc4\ucc9c \ubcf5\uc6d0 \uc0ac\uc5c5\uc744 \ubb34\ub9ac\ud558\uac8c \ub05d\ub9c8\uce58\uae30 \uc704\ud558\uc5ec \uc720\uc801 \ubcf5\uc6d0 \uc744 \ub0a0\ub9bc\uc73c\ub85c \uc9c4\ud589\ud588\ub2e4\ub294 \ud3c9\uac00\ub3c4 \uc788\ub294\ub370, \uc2e4\uc81c\ub85c \uc11d\ucd95\u00b7\ud6a8\ub839\uad50\u00b7\ud558\ub791\uad50\u00b7\uc624\uac04\uc218\ubb38\uc740 \ubcf5\uc6d0\ub418\uc9c0 \uc54a\uace0 \uc911\ub791\uad6c \ud558\uc218\uc885\ub9d0\ucc98\ub9ac\uc7a5\uc5d0 \ubcf4\uad00\ub418\uace0 \uc788\ub2e4.", "paragraph_id": "6559272-3-1", "paragraph_question": "question: \uc774\uba85\ubc15 \uc804 \uc11c\uc6b8\uc2dc\uc7a5\uc774 \uc790\uc2e0\uc758 \uc784\uae30 \uc548\uc5d0 \uccad\uacc4\ucc9c \ubcf5\uc6d0 \uc0ac\uc5c5\uc744 \ub05d\ub9c8\uce58\uae30 \uc704\ud574 \ub0a0\ub9bc\uc73c\ub85c \uc9c4\ud589 \ud588\ub2e4\ub294 \ud3c9\uac00\ub97c \ubc1b\ub294 \uac83\uc740?, context: \ubc18\uba74 \ubd80\uc815\uc801 \ud3c9\uac00\ub3c4 \ub9ce\ub2e4. \uacbd\uc81c\ud559\uc790 \uc6b0\uc11d\ud6c8\uc740 \uc790\uc2e0\uc758 \uc800\uc11c\uc5d0\uc11c \ubcf5\uc6d0\ub41c \uccad\uacc4\ucc9c\uc744 \"\uc778\uacf5\ud638\uc218\"\ub77c\uba70 \ud638\ub418\uac8c \ube44\ud310\ud558\uc600\ub2e4. \ub610\ud55c \uae30\uc874\uc5d0 \ud750\ub974\ub294 \ubb3c\uc758 \uc591\uc774 \uc801\uc5b4 \ucde8\uc218\uc7a5\uc5d0\uc11c \uc804\uae30\ub97c \uc774\uc6a9\ud574 \ud55c\uac15\ubb3c\uacfc \uc9c0\ud558\uc218\ub97c \ub04c\uc5b4\ub2e4 \uccad\uacc4\uad11\uc7a5\uc5d0\uc11c \ud758\ub824\ubcf4\ub0b4\ub294\ub370, \uc774\ub54c \ubb3c\uc744 \ub04c\uc5b4\uc62c\ub9ac\ub294 \ub370 \uc4f0\uc774\ub294 \uc804\uae30\uc694\uae08\uc774 \ud55c\ud574 \ud3c9\uade0 8\uc5b5 \uac00\ub7c9\uc73c\ub85c \uc0c1\ub2f9\ud558\uba70 \ub179\uc870 \ub610\ud55c \uc9c0\uc18d\uc801\uc73c\ub85c \ubc1c\uc0dd\ud558\uace0 \uc788\uc5b4 2007 ~ 2009\ub144\uae4c\uc9c0 8000\ub9cc\uc6d0\uc744 \ub179\uc870\uc81c\uac70\ub97c \uc704\ud574 \uc4f0\uc600\ub2e4. \uc774 \uc678\uc5d0\ub3c4 \uc778\uac74\ube44, \uccad\uc18c\u00b7\uacbd\ube44\uc6a9\uc5ed\ube44, \ud1a0\ubaa9\u00b7\uc870\uacbd\uc2dc\uc124 \uc720\uc9c0\uad00\ub9ac\ube44 \ub4f1\uc5d0 2011\ub144\uc5d0\ub9cc 80\uc5b5\uc5ec\uc6d0\uc774 \uc18c\ubaa8\ub418\uc5c8\ub2e4. \uc774\ub294 2005\ub144 37\uc5b5\uc5d0\uc11c 2010\ub144 77\uc5b5\uc6d0\uc5d0 \uc774\uc740 \uac83\uc73c\ub85c \uafb8\uc900\ud788 \ub298\uc5b4\ub098\uace0 \uc788\ub2e4. 2011\ub144 6\uc6d4 \uc870\uc120\uc77c\ubcf4\uc758 \uc124\ubb38 \uc870\uc0ac\uc5d0\uc11c \ucd5c\uc545\uc758 \uac74\ucd95\ubb3c 3\uc704\uc5d0 \ubf51\ud614\ub294\ub370, \"\uccad\uacc4\ucc9c\uc740 \ud55c\uad6d\uc744 \uc0c1\uc9d5\ud558\ub294 \uacf5\uacf5 \uac74\ucd95\ubb3c\uc778\ub370 \uac74\ucd95\uac00\ub3c4 \uc5c6\uc774 \ub2e8\uae30\uac04\uc5d0 \ubd88\ub3c4\uc800\ub85c \ubc00\uc5b4\ubd99\uc774\ub4ef \ub9cc\ub4e4\uc5c8\ub2e4\"\ub294 \uc774\uc720\uc5d0\uc11c\uc600\ub2e4. \ub610 \uc774\uba85\ubc15 \uc804 \uc11c\uc6b8\uc2dc\uc7a5\uc774 \uc790\uc2e0\uc758 \uc784\uae30 \uc548\uc5d0 \uccad\uacc4\ucc9c \ubcf5\uc6d0 \uc0ac\uc5c5\uc744 \ubb34\ub9ac\ud558\uac8c \ub05d\ub9c8\uce58\uae30 \uc704\ud558\uc5ec \uc720\uc801 \ubcf5\uc6d0\uc744 \ub0a0\ub9bc\uc73c\ub85c \uc9c4\ud589\ud588\ub2e4\ub294 \ud3c9\uac00\ub3c4 \uc788\ub294\ub370, \uc2e4\uc81c\ub85c \uc11d\ucd95\u00b7\ud6a8\ub839\uad50\u00b7\ud558\ub791\uad50\u00b7\uc624\uac04\uc218\ubb38\uc740 \ubcf5\uc6d0\ub418\uc9c0 \uc54a\uace0 \uc911\ub791\uad6c \ud558\uc218\uc885\ub9d0\ucc98\ub9ac\uc7a5\uc5d0 \ubcf4\uad00\ub418\uace0 \uc788\ub2e4. \ub610\ud55c 1773\ub144\uc5d0 \uc313\uc740 \ud604 \ubb34\uad50\ub3d9 \uc0ac\uac70\ub9ac\uc5d0\uc11c \uc2dc\uc791\ub418\uba70 \uccad\uacc4\ucc9c\uc744 \ub530\ub77c 17.1\ubbf8\ud130\uc758 \ud3ed\uc744 \ub450\uace0 \uc591\ucabd\uc73c\ub85c \uae38\uac8c \ub298\uc5b4\uc11c \uc788\ub294 \uccad\uacc4\ucc9c \uc11d\ucd95\uc740 \ubc1c\uad74 \ud6c4 \ubcf5\uc6d0\ub418\uc9c0 \ubabb\ud588\ub2e4. \ubcf5\uc6d0 \ub2f9\uc2dc \ubc1c\uacac\ub41c \uc11d\ucd95\uc740 \ub3d9\uc544\uc77c\ubcf4\uc0ac \uc55e\uc758 \uc88c\uc548\uc11d\ucd95\uc774 \uad50\ucc28\ub85c\ub97c \uae30\uc900\uc73c\ub85c \uc11c\ucabd 57m, \ub3d9\ucabd 36m \uc815\ub3c4, \ubc18\ub300\ud3b8 \uc6b0\uc548\uc11d\ucd95\uc740 9m \uc815\ub3c4\ub85c \ubaa8\ub450 100\uc5ec \ubbf8\ud130\ub098 \ub41c\ub2e4. \uc218\ud45c\uad50\ub294 \uc608\uc0b0\uc0c1\uc758 \ubb38\uc81c\ub77c\ub294 \ud45c\uba74\uc801 \uc774\uc720\ub85c \uc7a5\ucda9\ub2e8\uacf5\uc6d0\uc5d0 \ub0a8\uc544 \uc788\uace0, \uad11\ud1b5\uad50\ub294 \uc6d0\ub798\uc640 \ub2e4\ub974\uac8c \ubcf5\uc6d0\ub418\uc5c8\ub2e4. \uccad\uacc4\ucc9c \uc785\uad6c\uc5d0 \uc704\uce58\ud558\ub294 \uc18c\ub77c\ud0d1 \ub610\ud55c \uc77c\uac01\uc5d0\uc11c \ubb38\uc81c\uac00 \ub418\uace0 \uc788\ub2e4. \uc774\uac83\uc740 \uc2a4\uc6e8\ub374 \ucd9c\uc2e0\uc758 \ubbf8\uad6d \ud31d\uc544\ud2f0\uc2a4\ud2b8 \ud074\ub798\uc2a4 \uc62c\ub374\ubc84\uadf8\uac00 \uc124\uacc4\ud55c \uc791\ud488\uc73c\ub85c, KT\uac00 34\uc5b5\uc758 \ube44\uc6a9\uc744 \ub4e4\uc5ec \uc11c\uc6b8\uc2dc\uc5d0 \uae30\uc99d\ud55c \uc791\ud488\uc774\ub2e4. \ub192\uc774 20m, \ud3ed 6m, \ubb34\uac8c 9t\uc774\ub2e4. \uc77c\ubd80\uc5d0\uc11c\ub294 \ud55c\uad6d\uc801\uc778 \ubbf8\uac00 \uacb0\uc5ec\ub418\uc5b4 \uc788\uace0, \uc8fc\ubcc0\uc5d0 \uc5b4\uc6b8\ub9ac\uc9c0\ub3c4 \uc54a\ub294\ub2e4\ub294 \ube44\ud310\uacfc \ud48d\uc218\uc9c0\ub9ac\ud559\uc801\uc73c\ub85c\ub3c4 \uc88b\uc9c0 \ubabb\ud558\ub2e4\ub294 \uc8fc\uc7a5\uc774 \uc81c\uae30\ub418\uace0 \uc788\ub2e4."} {"question": "\uc560\ub9ac\uc870\ub098 \ubd81\ubd80\uc5d0 \uc788\ub294 \ud398\uc774\uc9c0\ub97c \uac70\uce58\uba74 \uc5b4\ub514\ub85c \uac08 \uc218 \uc788\ub294\uac00?", "paragraph": "\uadf8\ub79c\ub4dc \uce90\ub2c8\uc5b8 \uad6d\ub9bd\uacf5\uc6d0\uc73c\ub85c \uac00\ub294 \uae38\uc740 \uc0ac\uc6b0\uc2a4\ub9bc(South Rim)\uc758 \uacbd\uc6b0\ub294 I-40\uace0\uc18d \ub3c4\ub85c\ub85c \uc70c\ub9ac\uc5c4\uc2a4(Williams)\uae4c\uc9c0 \uc640\uc11c 64\ubc88 \ub3c4\ub85c\ub85c 100km \uac70\ub9ac\ub97c \uc62c\ub77c\uac00\ub4e0\uc9c0 \ud50c\ub798\uadf8\uc2a4\ud0dc\ud504(Flagstaff)\uc5d0 \uc640\uc11c 180\ubc88 \ub3c4\ub85c\ub85c 130km \uac70\ub9ac\ub97c \uc62c\ub77c\uac00\uc11c \ud22c\uc0ac\uc580(Tusayan) \ub9c8\uc744\uc744 \uac70\uccd0 \ub0a8\ucabd \uc785\uad6c\ub85c \uacf5\uc6d0\uc5d0 \ub4e4\uc5b4\uac00\ub294\uac8c \ub300\ubd80\ubd84\uc758 \uc5ec\ud589\uac1d\uc774 \ud0dd\ud558\ub294 \uae38\uc774\ub2e4. \ud50c\ub798\uadf8\uc2a4\ud0dc\ud504\uc5d0\uc11c 89\ubc88\uacfc 64\ubc88\uc744 \ud1b5\ud574\uc11c \ub3d9\ucabd \uc785\uad6c\ub85c \ub4e4\uc5b4\uac00\ub294 \ubc29\ubc95\ub3c4 \uc788\ub2e4. \ud53c\ub2c9\uc2a4(Phoenix)\uc640 \ud50c\ub798\uadf8\uc2a4\ud0dc\ud504 \uc0ac\uc774 \uac70\ub9ac\ub294 136\ub9c8\uc77c(217km), \ub77c\uc2a4\ubca0\uc774\uac70\uc2a4(Las Vegas)\uc640 \uc70c\ub9ac\uc5c4\uc2a4 \uc0ac\uc774 \uac70\ub9ac\ub294 221\ub9c8\uc77c(353km)\uc774\ub2e4. \ub178\uc2a4\ub9bc\uc73c\ub85c \uac00\ub294 \uae38\uc740 \uc560\ub9ac\uc870\ub098 \ubd81\ubd80\uc5d0 \uc788\ub294 \ud398\uc774\uc9c0(Page)\ub97c \uac70\uce58\ub4e0\uac00 \uc720\ud0c0\uc8fc\uc5d0 \uc788\ub294 \uce74\ub098\ube0c(Kanab)\ub97c \uac70\uccd0 \uc81c\uc774\ucf54\ube0c\ub808\uc774\ud06c(Jacob Lake)\ub97c \uacbd\uc720\ud574\uc11c \ub4e4\uc5b4\uac04\ub2e4. \uc70c\ub9ac\uc5c4\uc2a4\uc640 \uc0ac\uc6b0\uc2a4\ub9bc\uc5d0 \uc788\ub294 \uadf8\ub79c\ub4dc \uce90\ub2c8\uc5b8 \ube4c\ub9ac\uc9c0 \uc0ac\uc774 65\ub9c8\uc77c \uac70\ub9ac\ub97c \ud558\ub8e8\uc5d0 \ud55c\ubc88\uc529 \uc655\ubcf5\ud558\ub294 \uae30\ucc28\ub97c \uc774\uc6a9\ud558\ub294 \uc5ec\ud589\uac1d\ub3c4 \ub9ce\ub2e4. \uc61b\ub0a0\uc758 \ubd84\uc704\uae30\ub97c \ub290\uaef4 \ubcfc\uc218 \uc788\ub294 \uc5ec\ud589 \ubc29\ubc95\uc774\ub2e4. \uacf5\uc6d0 \uc548\uc5d0 \uc788\ub294 \ud638\ud154(Railway Hotel)\uacfc \uae30\ucc28\ud45c\ub97c \ud328\ud0a4\uc9c0\ub85c \uc608\uc57d\ud560 \uc218\ub3c4 \uc788\ub2e4.", "answer": "\ub178\uc2a4\ub9bc", "sentence": "\ub178\uc2a4\ub9bc \uc73c\ub85c \uac00\ub294 \uae38\uc740 \uc560\ub9ac\uc870\ub098 \ubd81\ubd80\uc5d0 \uc788\ub294 \ud398\uc774\uc9c0(Page)\ub97c \uac70\uce58\ub4e0\uac00 \uc720\ud0c0\uc8fc\uc5d0 \uc788\ub294 \uce74\ub098\ube0c(Kanab)\ub97c \uac70\uccd0 \uc81c\uc774\ucf54\ube0c\ub808\uc774\ud06c(Jacob Lake)\ub97c \uacbd\uc720\ud574\uc11c \ub4e4\uc5b4\uac04\ub2e4.", "paragraph_sentence": "\uadf8\ub79c\ub4dc \uce90\ub2c8\uc5b8 \uad6d\ub9bd\uacf5\uc6d0\uc73c\ub85c \uac00\ub294 \uae38\uc740 \uc0ac\uc6b0\uc2a4\ub9bc(South Rim)\uc758 \uacbd\uc6b0\ub294 I-40\uace0\uc18d \ub3c4\ub85c\ub85c \uc70c\ub9ac\uc5c4\uc2a4(Williams)\uae4c\uc9c0 \uc640\uc11c 64\ubc88 \ub3c4\ub85c\ub85c 100km \uac70\ub9ac\ub97c \uc62c\ub77c\uac00\ub4e0\uc9c0 \ud50c\ub798\uadf8\uc2a4\ud0dc\ud504(Flagstaff)\uc5d0 \uc640\uc11c 180\ubc88 \ub3c4\ub85c\ub85c 130km \uac70\ub9ac\ub97c \uc62c\ub77c\uac00\uc11c \ud22c\uc0ac\uc580(Tusayan) \ub9c8\uc744\uc744 \uac70\uccd0 \ub0a8\ucabd \uc785\uad6c\ub85c \uacf5\uc6d0\uc5d0 \ub4e4\uc5b4\uac00\ub294\uac8c \ub300\ubd80\ubd84\uc758 \uc5ec\ud589\uac1d\uc774 \ud0dd\ud558\ub294 \uae38\uc774\ub2e4. \ud50c\ub798\uadf8\uc2a4\ud0dc\ud504\uc5d0\uc11c 89\ubc88\uacfc 64\ubc88\uc744 \ud1b5\ud574\uc11c \ub3d9\ucabd \uc785\uad6c\ub85c \ub4e4\uc5b4\uac00\ub294 \ubc29\ubc95\ub3c4 \uc788\ub2e4. \ud53c\ub2c9\uc2a4(Phoenix)\uc640 \ud50c\ub798\uadf8\uc2a4\ud0dc\ud504 \uc0ac\uc774 \uac70\ub9ac\ub294 136\ub9c8\uc77c(217km), \ub77c\uc2a4\ubca0\uc774\uac70\uc2a4(Las Vegas)\uc640 \uc70c\ub9ac\uc5c4\uc2a4 \uc0ac\uc774 \uac70\ub9ac\ub294 221\ub9c8\uc77c(353km)\uc774\ub2e4. \ub178\uc2a4\ub9bc \uc73c\ub85c \uac00\ub294 \uae38\uc740 \uc560\ub9ac\uc870\ub098 \ubd81\ubd80\uc5d0 \uc788\ub294 \ud398\uc774\uc9c0(Page)\ub97c \uac70\uce58\ub4e0\uac00 \uc720\ud0c0\uc8fc\uc5d0 \uc788\ub294 \uce74\ub098\ube0c(Kanab)\ub97c \uac70\uccd0 \uc81c\uc774\ucf54\ube0c\ub808\uc774\ud06c(Jacob Lake)\ub97c \uacbd\uc720\ud574\uc11c \ub4e4\uc5b4\uac04\ub2e4. \uc70c\ub9ac\uc5c4\uc2a4\uc640 \uc0ac\uc6b0\uc2a4\ub9bc\uc5d0 \uc788\ub294 \uadf8\ub79c\ub4dc \uce90\ub2c8\uc5b8 \ube4c\ub9ac\uc9c0 \uc0ac\uc774 65\ub9c8\uc77c \uac70\ub9ac\ub97c \ud558\ub8e8\uc5d0 \ud55c\ubc88\uc529 \uc655\ubcf5\ud558\ub294 \uae30\ucc28\ub97c \uc774\uc6a9\ud558\ub294 \uc5ec\ud589\uac1d\ub3c4 \ub9ce\ub2e4. \uc61b\ub0a0\uc758 \ubd84\uc704\uae30\ub97c \ub290\uaef4 \ubcfc\uc218 \uc788\ub294 \uc5ec\ud589 \ubc29\ubc95\uc774\ub2e4. \uacf5\uc6d0 \uc548\uc5d0 \uc788\ub294 \ud638\ud154(Railway Hotel)\uacfc \uae30\ucc28\ud45c\ub97c \ud328\ud0a4\uc9c0\ub85c \uc608\uc57d\ud560 \uc218\ub3c4 \uc788\ub2e4.", "paragraph_answer": "\uadf8\ub79c\ub4dc \uce90\ub2c8\uc5b8 \uad6d\ub9bd\uacf5\uc6d0\uc73c\ub85c \uac00\ub294 \uae38\uc740 \uc0ac\uc6b0\uc2a4\ub9bc(South Rim)\uc758 \uacbd\uc6b0\ub294 I-40\uace0\uc18d \ub3c4\ub85c\ub85c \uc70c\ub9ac\uc5c4\uc2a4(Williams)\uae4c\uc9c0 \uc640\uc11c 64\ubc88 \ub3c4\ub85c\ub85c 100km \uac70\ub9ac\ub97c \uc62c\ub77c\uac00\ub4e0\uc9c0 \ud50c\ub798\uadf8\uc2a4\ud0dc\ud504(Flagstaff)\uc5d0 \uc640\uc11c 180\ubc88 \ub3c4\ub85c\ub85c 130km \uac70\ub9ac\ub97c \uc62c\ub77c\uac00\uc11c \ud22c\uc0ac\uc580(Tusayan) \ub9c8\uc744\uc744 \uac70\uccd0 \ub0a8\ucabd \uc785\uad6c\ub85c \uacf5\uc6d0\uc5d0 \ub4e4\uc5b4\uac00\ub294\uac8c \ub300\ubd80\ubd84\uc758 \uc5ec\ud589\uac1d\uc774 \ud0dd\ud558\ub294 \uae38\uc774\ub2e4. \ud50c\ub798\uadf8\uc2a4\ud0dc\ud504\uc5d0\uc11c 89\ubc88\uacfc 64\ubc88\uc744 \ud1b5\ud574\uc11c \ub3d9\ucabd \uc785\uad6c\ub85c \ub4e4\uc5b4\uac00\ub294 \ubc29\ubc95\ub3c4 \uc788\ub2e4. \ud53c\ub2c9\uc2a4(Phoenix)\uc640 \ud50c\ub798\uadf8\uc2a4\ud0dc\ud504 \uc0ac\uc774 \uac70\ub9ac\ub294 136\ub9c8\uc77c(217km), \ub77c\uc2a4\ubca0\uc774\uac70\uc2a4(Las Vegas)\uc640 \uc70c\ub9ac\uc5c4\uc2a4 \uc0ac\uc774 \uac70\ub9ac\ub294 221\ub9c8\uc77c(353km)\uc774\ub2e4. \ub178\uc2a4\ub9bc \uc73c\ub85c \uac00\ub294 \uae38\uc740 \uc560\ub9ac\uc870\ub098 \ubd81\ubd80\uc5d0 \uc788\ub294 \ud398\uc774\uc9c0(Page)\ub97c \uac70\uce58\ub4e0\uac00 \uc720\ud0c0\uc8fc\uc5d0 \uc788\ub294 \uce74\ub098\ube0c(Kanab)\ub97c \uac70\uccd0 \uc81c\uc774\ucf54\ube0c\ub808\uc774\ud06c(Jacob Lake)\ub97c \uacbd\uc720\ud574\uc11c \ub4e4\uc5b4\uac04\ub2e4. \uc70c\ub9ac\uc5c4\uc2a4\uc640 \uc0ac\uc6b0\uc2a4\ub9bc\uc5d0 \uc788\ub294 \uadf8\ub79c\ub4dc \uce90\ub2c8\uc5b8 \ube4c\ub9ac\uc9c0 \uc0ac\uc774 65\ub9c8\uc77c \uac70\ub9ac\ub97c \ud558\ub8e8\uc5d0 \ud55c\ubc88\uc529 \uc655\ubcf5\ud558\ub294 \uae30\ucc28\ub97c \uc774\uc6a9\ud558\ub294 \uc5ec\ud589\uac1d\ub3c4 \ub9ce\ub2e4. \uc61b\ub0a0\uc758 \ubd84\uc704\uae30\ub97c \ub290\uaef4 \ubcfc\uc218 \uc788\ub294 \uc5ec\ud589 \ubc29\ubc95\uc774\ub2e4. \uacf5\uc6d0 \uc548\uc5d0 \uc788\ub294 \ud638\ud154(Railway Hotel)\uacfc \uae30\ucc28\ud45c\ub97c \ud328\ud0a4\uc9c0\ub85c \uc608\uc57d\ud560 \uc218\ub3c4 \uc788\ub2e4.", "sentence_answer": " \ub178\uc2a4\ub9bc \uc73c\ub85c \uac00\ub294 \uae38\uc740 \uc560\ub9ac\uc870\ub098 \ubd81\ubd80\uc5d0 \uc788\ub294 \ud398\uc774\uc9c0(Page)\ub97c \uac70\uce58\ub4e0\uac00 \uc720\ud0c0\uc8fc\uc5d0 \uc788\ub294 \uce74\ub098\ube0c(Kanab)\ub97c \uac70\uccd0 \uc81c\uc774\ucf54\ube0c\ub808\uc774\ud06c(Jacob Lake)\ub97c \uacbd\uc720\ud574\uc11c \ub4e4\uc5b4\uac04\ub2e4.", "paragraph_id": "6546020-2-1", "paragraph_question": "question: \uc560\ub9ac\uc870\ub098 \ubd81\ubd80\uc5d0 \uc788\ub294 \ud398\uc774\uc9c0\ub97c \uac70\uce58\uba74 \uc5b4\ub514\ub85c \uac08 \uc218 \uc788\ub294\uac00?, context: \uadf8\ub79c\ub4dc \uce90\ub2c8\uc5b8 \uad6d\ub9bd\uacf5\uc6d0\uc73c\ub85c \uac00\ub294 \uae38\uc740 \uc0ac\uc6b0\uc2a4\ub9bc(South Rim)\uc758 \uacbd\uc6b0\ub294 I-40\uace0\uc18d \ub3c4\ub85c\ub85c \uc70c\ub9ac\uc5c4\uc2a4(Williams)\uae4c\uc9c0 \uc640\uc11c 64\ubc88 \ub3c4\ub85c\ub85c 100km \uac70\ub9ac\ub97c \uc62c\ub77c\uac00\ub4e0\uc9c0 \ud50c\ub798\uadf8\uc2a4\ud0dc\ud504(Flagstaff)\uc5d0 \uc640\uc11c 180\ubc88 \ub3c4\ub85c\ub85c 130km \uac70\ub9ac\ub97c \uc62c\ub77c\uac00\uc11c \ud22c\uc0ac\uc580(Tusayan) \ub9c8\uc744\uc744 \uac70\uccd0 \ub0a8\ucabd \uc785\uad6c\ub85c \uacf5\uc6d0\uc5d0 \ub4e4\uc5b4\uac00\ub294\uac8c \ub300\ubd80\ubd84\uc758 \uc5ec\ud589\uac1d\uc774 \ud0dd\ud558\ub294 \uae38\uc774\ub2e4. \ud50c\ub798\uadf8\uc2a4\ud0dc\ud504\uc5d0\uc11c 89\ubc88\uacfc 64\ubc88\uc744 \ud1b5\ud574\uc11c \ub3d9\ucabd \uc785\uad6c\ub85c \ub4e4\uc5b4\uac00\ub294 \ubc29\ubc95\ub3c4 \uc788\ub2e4. \ud53c\ub2c9\uc2a4(Phoenix)\uc640 \ud50c\ub798\uadf8\uc2a4\ud0dc\ud504 \uc0ac\uc774 \uac70\ub9ac\ub294 136\ub9c8\uc77c(217km), \ub77c\uc2a4\ubca0\uc774\uac70\uc2a4(Las Vegas)\uc640 \uc70c\ub9ac\uc5c4\uc2a4 \uc0ac\uc774 \uac70\ub9ac\ub294 221\ub9c8\uc77c(353km)\uc774\ub2e4. \ub178\uc2a4\ub9bc\uc73c\ub85c \uac00\ub294 \uae38\uc740 \uc560\ub9ac\uc870\ub098 \ubd81\ubd80\uc5d0 \uc788\ub294 \ud398\uc774\uc9c0(Page)\ub97c \uac70\uce58\ub4e0\uac00 \uc720\ud0c0\uc8fc\uc5d0 \uc788\ub294 \uce74\ub098\ube0c(Kanab)\ub97c \uac70\uccd0 \uc81c\uc774\ucf54\ube0c\ub808\uc774\ud06c(Jacob Lake)\ub97c \uacbd\uc720\ud574\uc11c \ub4e4\uc5b4\uac04\ub2e4. \uc70c\ub9ac\uc5c4\uc2a4\uc640 \uc0ac\uc6b0\uc2a4\ub9bc\uc5d0 \uc788\ub294 \uadf8\ub79c\ub4dc \uce90\ub2c8\uc5b8 \ube4c\ub9ac\uc9c0 \uc0ac\uc774 65\ub9c8\uc77c \uac70\ub9ac\ub97c \ud558\ub8e8\uc5d0 \ud55c\ubc88\uc529 \uc655\ubcf5\ud558\ub294 \uae30\ucc28\ub97c \uc774\uc6a9\ud558\ub294 \uc5ec\ud589\uac1d\ub3c4 \ub9ce\ub2e4. \uc61b\ub0a0\uc758 \ubd84\uc704\uae30\ub97c \ub290\uaef4 \ubcfc\uc218 \uc788\ub294 \uc5ec\ud589 \ubc29\ubc95\uc774\ub2e4. \uacf5\uc6d0 \uc548\uc5d0 \uc788\ub294 \ud638\ud154(Railway Hotel)\uacfc \uae30\ucc28\ud45c\ub97c \ud328\ud0a4\uc9c0\ub85c \uc608\uc57d\ud560 \uc218\ub3c4 \uc788\ub2e4."} {"question": "\ub7ec\uc2dc\uc544\uad70\uc740 \uba87 \uc138\uae30\uc758 \uad00\uc2b5\uc5d0 \ub530\ub77c \uc804\uc5f4\uc744 \uad6c\uc131\ud558\uc600\ub098?", "paragraph": "1805\ub144 \ub7ec\uc2dc\uc544 \uad70\uc740 \uad6c\uccb4\uc81c(\uc559\uc2dc\uc575 \ub808\uc9d0;Ancien R\u00e9gime)\uc801\uc73c\ub85c \uc870\uc9c1\ub418\uc5c8\ub2e4\uace0 \ud2b9\uc9d5\uc9c0\uc744 \uc218 \uc788\ub2e4. \uc77c\ub2e8 \uc5f0\ub300\ub2e8\uc704 \uc774\uc0c1\uc758 \uc601\uad6c\uc801\uc778 \ubd80\ub300\uad6c\uc131\uc774 \uc5c6\uc5c8\uc73c\uba70 \uace0\uc704 \uc9c0\ud718\uad00\ub4e4\uc740 \ub300\ubd80\ubd84 \uadc0\uc871\uc0ac\ud68c\uc5d0\uc11c \ucda9\uc6d0\ub418\uc5c8\ub2e4.(\uadf8\ub9ac\uace0 \uc77c\ubc18\uc801\uc73c\ub85c \ub2a5\ub825\uc774 \uc544\ub2cc \ub9e4\uad00\ub9e4\uc9c1\uc73c\ub85c \uacb0\uc815\ub418\uc5c8\ub2e4.) \uadf8\ub9ac\uace0 \ub7ec\uc2dc\uc544\uad70\uc740 18\uc138\uae30\uc758 \uad00\uc2b5\uc5d0 \ub530\ub77c \uc804\uc5f4\uc744 \uad6c\uc131\ud558\uc600\uc73c\uba70, \uacc4\uc18d\uc801\uc73c\ub85c \"\uc8fc\uc785\uc2dd \ud6c8\ub828\"\uc5d0 \ub530\ub77c \ub450\ub4dc\ub824 \ub9de\uace0 \ubc8c\uc744 \ubc1b\uc558\ub2e4. \uac8c\ub2e4\uac00 \ub9ce\uc740 \ud558\uc704 \uc7a5\uad50\ub4e4\uc740 \uc81c\ub300\ub85c \ud6c8\ub828\ubc1b\uc9c0 \ubabb\ud588\uace0, \uc804\ud22c\uc5d0\uc11c \uc694\uad6c\ub418\ub294 \ubcf5\uc7a1\ud55c \uae30\ub3d9\uc744 \ud560 \ud544\uc694\uac00 \uc788\uc744 \ub54c \ubcd1\uc0ac\ub4e4\uc758 \uc9c4\uc601\uc744 \uad6c\uc131\ud558\ub294 \ub370\uc5d0 \uc5b4\ub824\uc6c0\uc744 \uacaa\uc5c8\ub2e4. \uadf8\ub7fc\uc5d0\ub3c4 \ubd88\uad6c\ud558\uace0, \ub7ec\uc2dc\uc544\uad70\uc740 \ud6cc\ub96d\ud55c \ud3ec\ubcd1\ub300\ub97c \uc9c0\ub2c8\uace0 \uc788\uc5c8\uc73c\uba70, \uc774 \ud3ec\ubcd1\ub300\uc758 \ubcd1\uc0ac\ub4e4\uc740 \uadf8\ub4e4\uc758 \ud3ec\ub97c \uc801\uc758 \uc190\uc5d0 \ube7c\uc557\uae30\uc9c0 \uc54a\uae30 \uc704\ud574 \ud798\uaca8\uc6b4 \uc2f8\uc6c0\uc744 \ubc8c\uc774\uae30\ub3c4 \ud574\uc57c \ud588\ub2e4.", "answer": "18\uc138\uae30", "sentence": "\uadf8\ub9ac\uace0 \ub7ec\uc2dc\uc544\uad70\uc740 18\uc138\uae30 \uc758 \uad00\uc2b5\uc5d0 \ub530\ub77c \uc804\uc5f4\uc744 \uad6c\uc131\ud558\uc600\uc73c\uba70, \uacc4\uc18d\uc801\uc73c\ub85c \"\uc8fc\uc785\uc2dd \ud6c8\ub828\"\uc5d0 \ub530\ub77c \ub450\ub4dc\ub824 \ub9de\uace0 \ubc8c\uc744 \ubc1b\uc558\ub2e4.", "paragraph_sentence": "1805\ub144 \ub7ec\uc2dc\uc544 \uad70\uc740 \uad6c\uccb4\uc81c(\uc559\uc2dc\uc575 \ub808\uc9d0;Ancien R\u00e9gime)\uc801\uc73c\ub85c \uc870\uc9c1\ub418\uc5c8\ub2e4\uace0 \ud2b9\uc9d5\uc9c0\uc744 \uc218 \uc788\ub2e4. \uc77c\ub2e8 \uc5f0\ub300\ub2e8\uc704 \uc774\uc0c1\uc758 \uc601\uad6c\uc801\uc778 \ubd80\ub300\uad6c\uc131\uc774 \uc5c6\uc5c8\uc73c\uba70 \uace0\uc704 \uc9c0\ud718\uad00\ub4e4\uc740 \ub300\ubd80\ubd84 \uadc0\uc871\uc0ac\ud68c\uc5d0\uc11c \ucda9\uc6d0\ub418\uc5c8\ub2e4.(\uadf8\ub9ac\uace0 \uc77c\ubc18\uc801\uc73c\ub85c \ub2a5\ub825\uc774 \uc544\ub2cc \ub9e4\uad00\ub9e4\uc9c1\uc73c\ub85c \uacb0\uc815\ub418\uc5c8\ub2e4. ) \uadf8\ub9ac\uace0 \ub7ec\uc2dc\uc544\uad70\uc740 18\uc138\uae30 \uc758 \uad00\uc2b5\uc5d0 \ub530\ub77c \uc804\uc5f4\uc744 \uad6c\uc131\ud558\uc600\uc73c\uba70, \uacc4\uc18d\uc801\uc73c\ub85c \"\uc8fc\uc785\uc2dd \ud6c8\ub828\"\uc5d0 \ub530\ub77c \ub450\ub4dc\ub824 \ub9de\uace0 \ubc8c\uc744 \ubc1b\uc558\ub2e4. \uac8c\ub2e4\uac00 \ub9ce\uc740 \ud558\uc704 \uc7a5\uad50\ub4e4\uc740 \uc81c\ub300\ub85c \ud6c8\ub828\ubc1b\uc9c0 \ubabb\ud588\uace0, \uc804\ud22c\uc5d0\uc11c \uc694\uad6c\ub418\ub294 \ubcf5\uc7a1\ud55c \uae30\ub3d9\uc744 \ud560 \ud544\uc694\uac00 \uc788\uc744 \ub54c \ubcd1\uc0ac\ub4e4\uc758 \uc9c4\uc601\uc744 \uad6c\uc131\ud558\ub294 \ub370\uc5d0 \uc5b4\ub824\uc6c0\uc744 \uacaa\uc5c8\ub2e4. \uadf8\ub7fc\uc5d0\ub3c4 \ubd88\uad6c\ud558\uace0, \ub7ec\uc2dc\uc544\uad70\uc740 \ud6cc\ub96d\ud55c \ud3ec\ubcd1\ub300\ub97c \uc9c0\ub2c8\uace0 \uc788\uc5c8\uc73c\uba70, \uc774 \ud3ec\ubcd1\ub300\uc758 \ubcd1\uc0ac\ub4e4\uc740 \uadf8\ub4e4\uc758 \ud3ec\ub97c \uc801\uc758 \uc190\uc5d0 \ube7c\uc557\uae30\uc9c0 \uc54a\uae30 \uc704\ud574 \ud798\uaca8\uc6b4 \uc2f8\uc6c0\uc744 \ubc8c\uc774\uae30\ub3c4 \ud574\uc57c \ud588\ub2e4.", "paragraph_answer": "1805\ub144 \ub7ec\uc2dc\uc544 \uad70\uc740 \uad6c\uccb4\uc81c(\uc559\uc2dc\uc575 \ub808\uc9d0;Ancien R\u00e9gime)\uc801\uc73c\ub85c \uc870\uc9c1\ub418\uc5c8\ub2e4\uace0 \ud2b9\uc9d5\uc9c0\uc744 \uc218 \uc788\ub2e4. \uc77c\ub2e8 \uc5f0\ub300\ub2e8\uc704 \uc774\uc0c1\uc758 \uc601\uad6c\uc801\uc778 \ubd80\ub300\uad6c\uc131\uc774 \uc5c6\uc5c8\uc73c\uba70 \uace0\uc704 \uc9c0\ud718\uad00\ub4e4\uc740 \ub300\ubd80\ubd84 \uadc0\uc871\uc0ac\ud68c\uc5d0\uc11c \ucda9\uc6d0\ub418\uc5c8\ub2e4.(\uadf8\ub9ac\uace0 \uc77c\ubc18\uc801\uc73c\ub85c \ub2a5\ub825\uc774 \uc544\ub2cc \ub9e4\uad00\ub9e4\uc9c1\uc73c\ub85c \uacb0\uc815\ub418\uc5c8\ub2e4.) \uadf8\ub9ac\uace0 \ub7ec\uc2dc\uc544\uad70\uc740 18\uc138\uae30 \uc758 \uad00\uc2b5\uc5d0 \ub530\ub77c \uc804\uc5f4\uc744 \uad6c\uc131\ud558\uc600\uc73c\uba70, \uacc4\uc18d\uc801\uc73c\ub85c \"\uc8fc\uc785\uc2dd \ud6c8\ub828\"\uc5d0 \ub530\ub77c \ub450\ub4dc\ub824 \ub9de\uace0 \ubc8c\uc744 \ubc1b\uc558\ub2e4. \uac8c\ub2e4\uac00 \ub9ce\uc740 \ud558\uc704 \uc7a5\uad50\ub4e4\uc740 \uc81c\ub300\ub85c \ud6c8\ub828\ubc1b\uc9c0 \ubabb\ud588\uace0, \uc804\ud22c\uc5d0\uc11c \uc694\uad6c\ub418\ub294 \ubcf5\uc7a1\ud55c \uae30\ub3d9\uc744 \ud560 \ud544\uc694\uac00 \uc788\uc744 \ub54c \ubcd1\uc0ac\ub4e4\uc758 \uc9c4\uc601\uc744 \uad6c\uc131\ud558\ub294 \ub370\uc5d0 \uc5b4\ub824\uc6c0\uc744 \uacaa\uc5c8\ub2e4. \uadf8\ub7fc\uc5d0\ub3c4 \ubd88\uad6c\ud558\uace0, \ub7ec\uc2dc\uc544\uad70\uc740 \ud6cc\ub96d\ud55c \ud3ec\ubcd1\ub300\ub97c \uc9c0\ub2c8\uace0 \uc788\uc5c8\uc73c\uba70, \uc774 \ud3ec\ubcd1\ub300\uc758 \ubcd1\uc0ac\ub4e4\uc740 \uadf8\ub4e4\uc758 \ud3ec\ub97c \uc801\uc758 \uc190\uc5d0 \ube7c\uc557\uae30\uc9c0 \uc54a\uae30 \uc704\ud574 \ud798\uaca8\uc6b4 \uc2f8\uc6c0\uc744 \ubc8c\uc774\uae30\ub3c4 \ud574\uc57c \ud588\ub2e4.", "sentence_answer": "\uadf8\ub9ac\uace0 \ub7ec\uc2dc\uc544\uad70\uc740 18\uc138\uae30 \uc758 \uad00\uc2b5\uc5d0 \ub530\ub77c \uc804\uc5f4\uc744 \uad6c\uc131\ud558\uc600\uc73c\uba70, \uacc4\uc18d\uc801\uc73c\ub85c \"\uc8fc\uc785\uc2dd \ud6c8\ub828\"\uc5d0 \ub530\ub77c \ub450\ub4dc\ub824 \ub9de\uace0 \ubc8c\uc744 \ubc1b\uc558\ub2e4.", "paragraph_id": "6533833-4-0", "paragraph_question": "question: \ub7ec\uc2dc\uc544\uad70\uc740 \uba87 \uc138\uae30\uc758 \uad00\uc2b5\uc5d0 \ub530\ub77c \uc804\uc5f4\uc744 \uad6c\uc131\ud558\uc600\ub098?, context: 1805\ub144 \ub7ec\uc2dc\uc544 \uad70\uc740 \uad6c\uccb4\uc81c(\uc559\uc2dc\uc575 \ub808\uc9d0;Ancien R\u00e9gime)\uc801\uc73c\ub85c \uc870\uc9c1\ub418\uc5c8\ub2e4\uace0 \ud2b9\uc9d5\uc9c0\uc744 \uc218 \uc788\ub2e4. \uc77c\ub2e8 \uc5f0\ub300\ub2e8\uc704 \uc774\uc0c1\uc758 \uc601\uad6c\uc801\uc778 \ubd80\ub300\uad6c\uc131\uc774 \uc5c6\uc5c8\uc73c\uba70 \uace0\uc704 \uc9c0\ud718\uad00\ub4e4\uc740 \ub300\ubd80\ubd84 \uadc0\uc871\uc0ac\ud68c\uc5d0\uc11c \ucda9\uc6d0\ub418\uc5c8\ub2e4.(\uadf8\ub9ac\uace0 \uc77c\ubc18\uc801\uc73c\ub85c \ub2a5\ub825\uc774 \uc544\ub2cc \ub9e4\uad00\ub9e4\uc9c1\uc73c\ub85c \uacb0\uc815\ub418\uc5c8\ub2e4.) \uadf8\ub9ac\uace0 \ub7ec\uc2dc\uc544\uad70\uc740 18\uc138\uae30\uc758 \uad00\uc2b5\uc5d0 \ub530\ub77c \uc804\uc5f4\uc744 \uad6c\uc131\ud558\uc600\uc73c\uba70, \uacc4\uc18d\uc801\uc73c\ub85c \"\uc8fc\uc785\uc2dd \ud6c8\ub828\"\uc5d0 \ub530\ub77c \ub450\ub4dc\ub824 \ub9de\uace0 \ubc8c\uc744 \ubc1b\uc558\ub2e4. \uac8c\ub2e4\uac00 \ub9ce\uc740 \ud558\uc704 \uc7a5\uad50\ub4e4\uc740 \uc81c\ub300\ub85c \ud6c8\ub828\ubc1b\uc9c0 \ubabb\ud588\uace0, \uc804\ud22c\uc5d0\uc11c \uc694\uad6c\ub418\ub294 \ubcf5\uc7a1\ud55c \uae30\ub3d9\uc744 \ud560 \ud544\uc694\uac00 \uc788\uc744 \ub54c \ubcd1\uc0ac\ub4e4\uc758 \uc9c4\uc601\uc744 \uad6c\uc131\ud558\ub294 \ub370\uc5d0 \uc5b4\ub824\uc6c0\uc744 \uacaa\uc5c8\ub2e4. \uadf8\ub7fc\uc5d0\ub3c4 \ubd88\uad6c\ud558\uace0, \ub7ec\uc2dc\uc544\uad70\uc740 \ud6cc\ub96d\ud55c \ud3ec\ubcd1\ub300\ub97c \uc9c0\ub2c8\uace0 \uc788\uc5c8\uc73c\uba70, \uc774 \ud3ec\ubcd1\ub300\uc758 \ubcd1\uc0ac\ub4e4\uc740 \uadf8\ub4e4\uc758 \ud3ec\ub97c \uc801\uc758 \uc190\uc5d0 \ube7c\uc557\uae30\uc9c0 \uc54a\uae30 \uc704\ud574 \ud798\uaca8\uc6b4 \uc2f8\uc6c0\uc744 \ubc8c\uc774\uae30\ub3c4 \ud574\uc57c \ud588\ub2e4."} {"question": "2012\ub144 \ub300\ud1b5\ub839 \uc120\uac70\uc5d0\uc11c \ub204\uad6c\uc5d0\uac8c \ud328\ubc30\ud558\uc600\ub294\uac00?", "paragraph": "\uc774\ud6c4 12\uc6d4 \uc0c8\ub204\ub9ac\ub2f9 \ubc15\uadfc\ud61c \ud6c4\ubcf4\uc640 \ud1b5\ud569\uc9c4\ubcf4\ub2f9 \uc774\uc815\ud76c \ud6c4\ubcf4\uc640 \ud568\uaed8 \ud1a0\ub860\uc5d0 \ucc38\uc11d\ud588\ub2e4. \ubb38\uc7ac\uc778 \uce21\uc740 \uad6d\uac00\uc815\ubcf4\uc6d0\uc5d0\uc11c \uc790\uc2e0\uc744 \ube44\ubc29\ud558\ub294 \uc5ec\ub860\uc870\uc791\uc744 \ubc8c\uc778\ub2e4\uace0 12\uc6d4 11\uc77c \uacf5\uac1c\ud558\uc600\ub2e4. \ubb38\uc7ac\uc778 \uce21\uc740 \uadf8\ub7f0 \ud65c\ub3d9\uc744 \ubc8c\uc778 \uc9c1\uc6d0\uc911 \ud55c\uba85\uc744 \uc9c0\ubaa9\ud588\ub2e4. 3\ucc28 TV \ud1a0\ub860\uc774 \ub05d\ub09c 12\uc6d4 16\uc77c \uc624\ud6c4 11\uc2dc \uc11c\uc6b8\uc9c0\ubc29\uacbd\ucc30\uccad\uc740 \uc99d\uac70\uac00 \ubc1c\uacac\ub418\uc9c0 \uc54a\uc558\ub2e4\uace0 \ubc1c\ud45c\ud588\ub294\ub370, \uc774\ub7ec\ud55c \uacbd\ucc30\uc758 \ubc1c\ud45c\ub294 \ub4a4\uc5d0 \ud5c8\uc704\ub85c \ubc1d\ud600\uc84c\ub2e4. \uc774\ud6c4 2012\ub144 12\uc6d4 19\uc77c \uc120\uac70\uc5d0\uc11c \ud0c8\ub77d \ud6c4\ubcf4 \uc0ac\uc0c1 \ucd5c\ub2e4\uc774\uc790, \uc5ed\ub300 \ub300\ud55c\ubbfc\uad6d \ub300\ud1b5\ub839 \uc120\uac70\uc758 \ud6c4\ubcf4\ub4e4 \uc911 \ub450 \ubc88\uc9f8\ub85c \ub9ce\uc740 14,692,632\ud45c(48.0%)\ub97c \ub4dd\ud45c\ud588\uc9c0\ub9cc \uc0c8\ub204\ub9ac\ub2f9\uc758 \ubc15\uadfc\ud61c \ud6c4\ubcf4\uc5d0\uac8c \uc57d 100\ub9cc \ud45c \ucc28\uc774\ub85c \ud328\ubc30\ud558\uc600\ub2e4. \ub2e4\uc74c\ub0a0 \ubb38\uc7ac\uc778\uc740 \ud328\ubc30\ub97c \uc2b9\ubcf5\ud558\ub294 \uae30\uc790\ud68c\uacac\uc744 \uc5f4\uc5c8\ub2e4.", "answer": "\ubc15\uadfc\ud61c", "sentence": "\uc774\ud6c4 12\uc6d4 \uc0c8\ub204\ub9ac\ub2f9 \ubc15\uadfc\ud61c \ud6c4\ubcf4\uc640 \ud1b5\ud569\uc9c4\ubcf4\ub2f9 \uc774\uc815\ud76c \ud6c4\ubcf4\uc640 \ud568\uaed8 \ud1a0\ub860\uc5d0 \ucc38\uc11d\ud588\ub2e4.", "paragraph_sentence": " \uc774\ud6c4 12\uc6d4 \uc0c8\ub204\ub9ac\ub2f9 \ubc15\uadfc\ud61c \ud6c4\ubcf4\uc640 \ud1b5\ud569\uc9c4\ubcf4\ub2f9 \uc774\uc815\ud76c \ud6c4\ubcf4\uc640 \ud568\uaed8 \ud1a0\ub860\uc5d0 \ucc38\uc11d\ud588\ub2e4. \ubb38\uc7ac\uc778 \uce21\uc740 \uad6d\uac00\uc815\ubcf4\uc6d0\uc5d0\uc11c \uc790\uc2e0\uc744 \ube44\ubc29\ud558\ub294 \uc5ec\ub860\uc870\uc791\uc744 \ubc8c\uc778\ub2e4\uace0 12\uc6d4 11\uc77c \uacf5\uac1c\ud558\uc600\ub2e4. \ubb38\uc7ac\uc778 \uce21\uc740 \uadf8\ub7f0 \ud65c\ub3d9\uc744 \ubc8c\uc778 \uc9c1\uc6d0\uc911 \ud55c\uba85\uc744 \uc9c0\ubaa9\ud588\ub2e4. 3\ucc28 TV \ud1a0\ub860\uc774 \ub05d\ub09c 12\uc6d4 16\uc77c \uc624\ud6c4 11\uc2dc \uc11c\uc6b8\uc9c0\ubc29\uacbd\ucc30\uccad\uc740 \uc99d\uac70\uac00 \ubc1c\uacac\ub418\uc9c0 \uc54a\uc558\ub2e4\uace0 \ubc1c\ud45c\ud588\ub294\ub370, \uc774\ub7ec\ud55c \uacbd\ucc30\uc758 \ubc1c\ud45c\ub294 \ub4a4\uc5d0 \ud5c8\uc704\ub85c \ubc1d\ud600\uc84c\ub2e4. \uc774\ud6c4 2012\ub144 12\uc6d4 19\uc77c \uc120\uac70\uc5d0\uc11c \ud0c8\ub77d \ud6c4\ubcf4 \uc0ac\uc0c1 \ucd5c\ub2e4\uc774\uc790, \uc5ed\ub300 \ub300\ud55c\ubbfc\uad6d \ub300\ud1b5\ub839 \uc120\uac70\uc758 \ud6c4\ubcf4\ub4e4 \uc911 \ub450 \ubc88\uc9f8\ub85c \ub9ce\uc740 14,692,632\ud45c(48.0%)\ub97c \ub4dd\ud45c\ud588\uc9c0\ub9cc \uc0c8\ub204\ub9ac\ub2f9\uc758 \ubc15\uadfc\ud61c \ud6c4\ubcf4\uc5d0\uac8c \uc57d 100\ub9cc \ud45c \ucc28\uc774\ub85c \ud328\ubc30\ud558\uc600\ub2e4. \ub2e4\uc74c\ub0a0 \ubb38\uc7ac\uc778\uc740 \ud328\ubc30\ub97c \uc2b9\ubcf5\ud558\ub294 \uae30\uc790\ud68c\uacac\uc744 \uc5f4\uc5c8\ub2e4.", "paragraph_answer": "\uc774\ud6c4 12\uc6d4 \uc0c8\ub204\ub9ac\ub2f9 \ubc15\uadfc\ud61c \ud6c4\ubcf4\uc640 \ud1b5\ud569\uc9c4\ubcf4\ub2f9 \uc774\uc815\ud76c \ud6c4\ubcf4\uc640 \ud568\uaed8 \ud1a0\ub860\uc5d0 \ucc38\uc11d\ud588\ub2e4. \ubb38\uc7ac\uc778 \uce21\uc740 \uad6d\uac00\uc815\ubcf4\uc6d0\uc5d0\uc11c \uc790\uc2e0\uc744 \ube44\ubc29\ud558\ub294 \uc5ec\ub860\uc870\uc791\uc744 \ubc8c\uc778\ub2e4\uace0 12\uc6d4 11\uc77c \uacf5\uac1c\ud558\uc600\ub2e4. \ubb38\uc7ac\uc778 \uce21\uc740 \uadf8\ub7f0 \ud65c\ub3d9\uc744 \ubc8c\uc778 \uc9c1\uc6d0\uc911 \ud55c\uba85\uc744 \uc9c0\ubaa9\ud588\ub2e4. 3\ucc28 TV \ud1a0\ub860\uc774 \ub05d\ub09c 12\uc6d4 16\uc77c \uc624\ud6c4 11\uc2dc \uc11c\uc6b8\uc9c0\ubc29\uacbd\ucc30\uccad\uc740 \uc99d\uac70\uac00 \ubc1c\uacac\ub418\uc9c0 \uc54a\uc558\ub2e4\uace0 \ubc1c\ud45c\ud588\ub294\ub370, \uc774\ub7ec\ud55c \uacbd\ucc30\uc758 \ubc1c\ud45c\ub294 \ub4a4\uc5d0 \ud5c8\uc704\ub85c \ubc1d\ud600\uc84c\ub2e4. \uc774\ud6c4 2012\ub144 12\uc6d4 19\uc77c \uc120\uac70\uc5d0\uc11c \ud0c8\ub77d \ud6c4\ubcf4 \uc0ac\uc0c1 \ucd5c\ub2e4\uc774\uc790, \uc5ed\ub300 \ub300\ud55c\ubbfc\uad6d \ub300\ud1b5\ub839 \uc120\uac70\uc758 \ud6c4\ubcf4\ub4e4 \uc911 \ub450 \ubc88\uc9f8\ub85c \ub9ce\uc740 14,692,632\ud45c(48.0%)\ub97c \ub4dd\ud45c\ud588\uc9c0\ub9cc \uc0c8\ub204\ub9ac\ub2f9\uc758 \ubc15\uadfc\ud61c \ud6c4\ubcf4\uc5d0\uac8c \uc57d 100\ub9cc \ud45c \ucc28\uc774\ub85c \ud328\ubc30\ud558\uc600\ub2e4. \ub2e4\uc74c\ub0a0 \ubb38\uc7ac\uc778\uc740 \ud328\ubc30\ub97c \uc2b9\ubcf5\ud558\ub294 \uae30\uc790\ud68c\uacac\uc744 \uc5f4\uc5c8\ub2e4.", "sentence_answer": "\uc774\ud6c4 12\uc6d4 \uc0c8\ub204\ub9ac\ub2f9 \ubc15\uadfc\ud61c \ud6c4\ubcf4\uc640 \ud1b5\ud569\uc9c4\ubcf4\ub2f9 \uc774\uc815\ud76c \ud6c4\ubcf4\uc640 \ud568\uaed8 \ud1a0\ub860\uc5d0 \ucc38\uc11d\ud588\ub2e4.", "paragraph_id": "6580716-5-0", "paragraph_question": "question: 2012\ub144 \ub300\ud1b5\ub839 \uc120\uac70\uc5d0\uc11c \ub204\uad6c\uc5d0\uac8c \ud328\ubc30\ud558\uc600\ub294\uac00?, context: \uc774\ud6c4 12\uc6d4 \uc0c8\ub204\ub9ac\ub2f9 \ubc15\uadfc\ud61c \ud6c4\ubcf4\uc640 \ud1b5\ud569\uc9c4\ubcf4\ub2f9 \uc774\uc815\ud76c \ud6c4\ubcf4\uc640 \ud568\uaed8 \ud1a0\ub860\uc5d0 \ucc38\uc11d\ud588\ub2e4. \ubb38\uc7ac\uc778 \uce21\uc740 \uad6d\uac00\uc815\ubcf4\uc6d0\uc5d0\uc11c \uc790\uc2e0\uc744 \ube44\ubc29\ud558\ub294 \uc5ec\ub860\uc870\uc791\uc744 \ubc8c\uc778\ub2e4\uace0 12\uc6d4 11\uc77c \uacf5\uac1c\ud558\uc600\ub2e4. \ubb38\uc7ac\uc778 \uce21\uc740 \uadf8\ub7f0 \ud65c\ub3d9\uc744 \ubc8c\uc778 \uc9c1\uc6d0\uc911 \ud55c\uba85\uc744 \uc9c0\ubaa9\ud588\ub2e4. 3\ucc28 TV \ud1a0\ub860\uc774 \ub05d\ub09c 12\uc6d4 16\uc77c \uc624\ud6c4 11\uc2dc \uc11c\uc6b8\uc9c0\ubc29\uacbd\ucc30\uccad\uc740 \uc99d\uac70\uac00 \ubc1c\uacac\ub418\uc9c0 \uc54a\uc558\ub2e4\uace0 \ubc1c\ud45c\ud588\ub294\ub370, \uc774\ub7ec\ud55c \uacbd\ucc30\uc758 \ubc1c\ud45c\ub294 \ub4a4\uc5d0 \ud5c8\uc704\ub85c \ubc1d\ud600\uc84c\ub2e4. \uc774\ud6c4 2012\ub144 12\uc6d4 19\uc77c \uc120\uac70\uc5d0\uc11c \ud0c8\ub77d \ud6c4\ubcf4 \uc0ac\uc0c1 \ucd5c\ub2e4\uc774\uc790, \uc5ed\ub300 \ub300\ud55c\ubbfc\uad6d \ub300\ud1b5\ub839 \uc120\uac70\uc758 \ud6c4\ubcf4\ub4e4 \uc911 \ub450 \ubc88\uc9f8\ub85c \ub9ce\uc740 14,692,632\ud45c(48.0%)\ub97c \ub4dd\ud45c\ud588\uc9c0\ub9cc \uc0c8\ub204\ub9ac\ub2f9\uc758 \ubc15\uadfc\ud61c \ud6c4\ubcf4\uc5d0\uac8c \uc57d 100\ub9cc \ud45c \ucc28\uc774\ub85c \ud328\ubc30\ud558\uc600\ub2e4. \ub2e4\uc74c\ub0a0 \ubb38\uc7ac\uc778\uc740 \ud328\ubc30\ub97c \uc2b9\ubcf5\ud558\ub294 \uae30\uc790\ud68c\uacac\uc744 \uc5f4\uc5c8\ub2e4."} {"question": "\uc778\ub3c4\uc0ac\uc6b0\ub8e8\uc2a4\ub294 \ubb34\uc2a8 \ub958 \uacf5\ub8e1\uc73c\ub85c \ubd84\ub958\ub418\ub294\uac00?", "paragraph": "\ub9e4\ub4dc\uc2a8\uc758 \uc601\ud5a5\ub825 \uc788\ub294 \ub17c\ubb38\uc5d0 \ub530\ub974\uba74, \uc54c\ub85c\uc0ac\uc6b0\ub8e8\uc2a4\ub294 \uc8fc\ub85c \uacfc\ub85c \ubd84\ub958\ub418\uc5c8\uc9c0\ub9cc, \uc54c\ub85c\uc0ac\uc6b0\ub8e8\uc2a4\uacfc\ub294 \ud655\uc2e4\ud788 \uc815\uc758\ub41c \uacfc\ub294 \uc544\ub2c8\uc5c8\ub2e4. \ubcf4\ud1b5 \uc900\uc804\ubb38\uc801\uc778 \uc791\ud488\ub4e4\uc5d0\uc11c\ub294 \u2018\uc54c\ub85c\uc0ac\uc6b0\ub8e8\uc2a4\uacfc\u2019\ub97c \uba54\uac08\ub85c\uc0ac\uc6b0\ub8e8\uc2a4\uacfc\ubcf4\ub2e4 \ud06c\uace0 \uc798 \uc54c\ub824\uc9c4 \ub2e4\uc591\ud55c \uac70\ub300 \uc218\uac01\ub958\ub4e4\uc744 \uac00\ub9ac\ud0a4\ub294 \uc6a9\uc5b4\ub85c \uc0ac\uc6a9\ud588\ub2e4. \uc54c\ub85c\uc0ac\uc6b0\ub8e8\uc2a4\uc640 \uadfc\uc5f0\ub41c\ub2e4\uace0 \uc0dd\uac01\ub418\uc5c8\ub358 \uc804\ud615\uc801\uc778 \uc218\uac01\ub958\ub85c\ub294 \uc778\ub3c4\uc0ac\uc6b0\ub8e8\uc2a4, \ud53c\uc544\ud2b8\ub2c8\uce20\ud0a4\uc0ac\uc6b0\ub8e8\uc2a4, \ud53c\ubca0\ud14c\uc544\uc6b0\uc0ac\uc6b0\ub8e8\uc2a4, \uc591\ucd94\uc544\ub178\uc0ac\uc6b0\ub8e8\uc2a4, \uc544\ud06c\ub85c\uce78\ud1a0\uc0ac\uc6b0\ub8e8\uc2a4, \ud0a4\ub780\ud14c\uc0ac\uc6b0\ub8e8\uc2a4, \ucf64\ud504\uc18c\uc218\ucfe0\uc2a4, \uc2a4\ud1a0\ucf00\uc18c\uc0ac\uc6b0\ub8e8\uc2a4, \uc2a4\uccb4\ucd94\uc544\ub178\uc0ac\uc6b0\ub8e8\uc2a4\uac00 \uc788\ub2e4. \ud604\uc7ac\ub294 \uc218\uac01\ub958\uc758 \ub2e4\uc591\uc131\uc5d0 \ub300\ud55c \ud604\ub300 \uc9c0\uc2dd\uacfc \uc9c4\ud654\uc801 \uc720\uc5f0 \uad00\uacc4\uc758 \uacc4\ud1b5\ubc1c\uc0dd\ud559\uc801 \uc5f0\uad6c\uc758 \ub3c4\ub798\ub85c \uc774\ub4e4 \uc218\uac01\ub958 \uc911 \ub300\ubd80\ubd84\uc740 \uc54c\ub85c\uc0ac\uc6b0\ub8e8\uc2a4\uacfc\ub85c \ubd84\ub958\ub418\uc9c0 \uc54a\ub294\ub2e4. \ub2e4\ub9cc \uc544\ud06c\ub85c\uce78\ud1a0\uc0ac\uc6b0\ub8e8\uc2a4\uc640 \uc591\ucd94\uc544\ub178\uc0ac\uc6b0\ub8e8\uc2a4\ub294 \uadfc\uc5f0\uacfc\uc5d0 \uc18d\ud55c\ub2e4.", "answer": "\uc218\uac01\ub958", "sentence": "\ubcf4\ud1b5 \uc900\uc804\ubb38\uc801\uc778 \uc791\ud488\ub4e4\uc5d0\uc11c\ub294 \u2018\uc54c\ub85c\uc0ac\uc6b0\ub8e8\uc2a4\uacfc\u2019\ub97c \uba54\uac08\ub85c\uc0ac\uc6b0\ub8e8\uc2a4\uacfc\ubcf4\ub2e4 \ud06c\uace0 \uc798 \uc54c\ub824\uc9c4 \ub2e4\uc591\ud55c \uac70\ub300 \uc218\uac01\ub958 \ub4e4\uc744 \uac00\ub9ac\ud0a4\ub294 \uc6a9\uc5b4\ub85c \uc0ac\uc6a9\ud588\ub2e4.", "paragraph_sentence": "\ub9e4\ub4dc\uc2a8\uc758 \uc601\ud5a5\ub825 \uc788\ub294 \ub17c\ubb38\uc5d0 \ub530\ub974\uba74, \uc54c\ub85c\uc0ac\uc6b0\ub8e8\uc2a4\ub294 \uc8fc\ub85c \uacfc\ub85c \ubd84\ub958\ub418\uc5c8\uc9c0\ub9cc, \uc54c\ub85c\uc0ac\uc6b0\ub8e8\uc2a4\uacfc\ub294 \ud655\uc2e4\ud788 \uc815\uc758\ub41c \uacfc\ub294 \uc544\ub2c8\uc5c8\ub2e4. \ubcf4\ud1b5 \uc900\uc804\ubb38\uc801\uc778 \uc791\ud488\ub4e4\uc5d0\uc11c\ub294 \u2018\uc54c\ub85c\uc0ac\uc6b0\ub8e8\uc2a4\uacfc\u2019\ub97c \uba54\uac08\ub85c\uc0ac\uc6b0\ub8e8\uc2a4\uacfc\ubcf4\ub2e4 \ud06c\uace0 \uc798 \uc54c\ub824\uc9c4 \ub2e4\uc591\ud55c \uac70\ub300 \uc218\uac01\ub958 \ub4e4\uc744 \uac00\ub9ac\ud0a4\ub294 \uc6a9\uc5b4\ub85c \uc0ac\uc6a9\ud588\ub2e4. \uc54c\ub85c\uc0ac\uc6b0\ub8e8\uc2a4\uc640 \uadfc\uc5f0\ub41c\ub2e4\uace0 \uc0dd\uac01\ub418\uc5c8\ub358 \uc804\ud615\uc801\uc778 \uc218\uac01\ub958\ub85c\ub294 \uc778\ub3c4\uc0ac\uc6b0\ub8e8\uc2a4, \ud53c\uc544\ud2b8\ub2c8\uce20\ud0a4\uc0ac\uc6b0\ub8e8\uc2a4, \ud53c\ubca0\ud14c\uc544\uc6b0\uc0ac\uc6b0\ub8e8\uc2a4, \uc591\ucd94\uc544\ub178\uc0ac\uc6b0\ub8e8\uc2a4, \uc544\ud06c\ub85c\uce78\ud1a0\uc0ac\uc6b0\ub8e8\uc2a4, \ud0a4\ub780\ud14c\uc0ac\uc6b0\ub8e8\uc2a4, \ucf64\ud504\uc18c\uc218\ucfe0\uc2a4, \uc2a4\ud1a0\ucf00\uc18c\uc0ac\uc6b0\ub8e8\uc2a4, \uc2a4\uccb4\ucd94\uc544\ub178\uc0ac\uc6b0\ub8e8\uc2a4\uac00 \uc788\ub2e4. \ud604\uc7ac\ub294 \uc218\uac01\ub958\uc758 \ub2e4\uc591\uc131\uc5d0 \ub300\ud55c \ud604\ub300 \uc9c0\uc2dd\uacfc \uc9c4\ud654\uc801 \uc720\uc5f0 \uad00\uacc4\uc758 \uacc4\ud1b5\ubc1c\uc0dd\ud559\uc801 \uc5f0\uad6c\uc758 \ub3c4\ub798\ub85c \uc774\ub4e4 \uc218\uac01\ub958 \uc911 \ub300\ubd80\ubd84\uc740 \uc54c\ub85c\uc0ac\uc6b0\ub8e8\uc2a4\uacfc\ub85c \ubd84\ub958\ub418\uc9c0 \uc54a\ub294\ub2e4. \ub2e4\ub9cc \uc544\ud06c\ub85c\uce78\ud1a0\uc0ac\uc6b0\ub8e8\uc2a4\uc640 \uc591\ucd94\uc544\ub178\uc0ac\uc6b0\ub8e8\uc2a4\ub294 \uadfc\uc5f0\uacfc\uc5d0 \uc18d\ud55c\ub2e4.", "paragraph_answer": "\ub9e4\ub4dc\uc2a8\uc758 \uc601\ud5a5\ub825 \uc788\ub294 \ub17c\ubb38\uc5d0 \ub530\ub974\uba74, \uc54c\ub85c\uc0ac\uc6b0\ub8e8\uc2a4\ub294 \uc8fc\ub85c \uacfc\ub85c \ubd84\ub958\ub418\uc5c8\uc9c0\ub9cc, \uc54c\ub85c\uc0ac\uc6b0\ub8e8\uc2a4\uacfc\ub294 \ud655\uc2e4\ud788 \uc815\uc758\ub41c \uacfc\ub294 \uc544\ub2c8\uc5c8\ub2e4. \ubcf4\ud1b5 \uc900\uc804\ubb38\uc801\uc778 \uc791\ud488\ub4e4\uc5d0\uc11c\ub294 \u2018\uc54c\ub85c\uc0ac\uc6b0\ub8e8\uc2a4\uacfc\u2019\ub97c \uba54\uac08\ub85c\uc0ac\uc6b0\ub8e8\uc2a4\uacfc\ubcf4\ub2e4 \ud06c\uace0 \uc798 \uc54c\ub824\uc9c4 \ub2e4\uc591\ud55c \uac70\ub300 \uc218\uac01\ub958 \ub4e4\uc744 \uac00\ub9ac\ud0a4\ub294 \uc6a9\uc5b4\ub85c \uc0ac\uc6a9\ud588\ub2e4. \uc54c\ub85c\uc0ac\uc6b0\ub8e8\uc2a4\uc640 \uadfc\uc5f0\ub41c\ub2e4\uace0 \uc0dd\uac01\ub418\uc5c8\ub358 \uc804\ud615\uc801\uc778 \uc218\uac01\ub958\ub85c\ub294 \uc778\ub3c4\uc0ac\uc6b0\ub8e8\uc2a4, \ud53c\uc544\ud2b8\ub2c8\uce20\ud0a4\uc0ac\uc6b0\ub8e8\uc2a4, \ud53c\ubca0\ud14c\uc544\uc6b0\uc0ac\uc6b0\ub8e8\uc2a4, \uc591\ucd94\uc544\ub178\uc0ac\uc6b0\ub8e8\uc2a4, \uc544\ud06c\ub85c\uce78\ud1a0\uc0ac\uc6b0\ub8e8\uc2a4, \ud0a4\ub780\ud14c\uc0ac\uc6b0\ub8e8\uc2a4, \ucf64\ud504\uc18c\uc218\ucfe0\uc2a4, \uc2a4\ud1a0\ucf00\uc18c\uc0ac\uc6b0\ub8e8\uc2a4, \uc2a4\uccb4\ucd94\uc544\ub178\uc0ac\uc6b0\ub8e8\uc2a4\uac00 \uc788\ub2e4. \ud604\uc7ac\ub294 \uc218\uac01\ub958\uc758 \ub2e4\uc591\uc131\uc5d0 \ub300\ud55c \ud604\ub300 \uc9c0\uc2dd\uacfc \uc9c4\ud654\uc801 \uc720\uc5f0 \uad00\uacc4\uc758 \uacc4\ud1b5\ubc1c\uc0dd\ud559\uc801 \uc5f0\uad6c\uc758 \ub3c4\ub798\ub85c \uc774\ub4e4 \uc218\uac01\ub958 \uc911 \ub300\ubd80\ubd84\uc740 \uc54c\ub85c\uc0ac\uc6b0\ub8e8\uc2a4\uacfc\ub85c \ubd84\ub958\ub418\uc9c0 \uc54a\ub294\ub2e4. \ub2e4\ub9cc \uc544\ud06c\ub85c\uce78\ud1a0\uc0ac\uc6b0\ub8e8\uc2a4\uc640 \uc591\ucd94\uc544\ub178\uc0ac\uc6b0\ub8e8\uc2a4\ub294 \uadfc\uc5f0\uacfc\uc5d0 \uc18d\ud55c\ub2e4.", "sentence_answer": "\ubcf4\ud1b5 \uc900\uc804\ubb38\uc801\uc778 \uc791\ud488\ub4e4\uc5d0\uc11c\ub294 \u2018\uc54c\ub85c\uc0ac\uc6b0\ub8e8\uc2a4\uacfc\u2019\ub97c \uba54\uac08\ub85c\uc0ac\uc6b0\ub8e8\uc2a4\uacfc\ubcf4\ub2e4 \ud06c\uace0 \uc798 \uc54c\ub824\uc9c4 \ub2e4\uc591\ud55c \uac70\ub300 \uc218\uac01\ub958 \ub4e4\uc744 \uac00\ub9ac\ud0a4\ub294 \uc6a9\uc5b4\ub85c \uc0ac\uc6a9\ud588\ub2e4.", "paragraph_id": "6521806-5-1", "paragraph_question": "question: \uc778\ub3c4\uc0ac\uc6b0\ub8e8\uc2a4\ub294 \ubb34\uc2a8 \ub958 \uacf5\ub8e1\uc73c\ub85c \ubd84\ub958\ub418\ub294\uac00?, context: \ub9e4\ub4dc\uc2a8\uc758 \uc601\ud5a5\ub825 \uc788\ub294 \ub17c\ubb38\uc5d0 \ub530\ub974\uba74, \uc54c\ub85c\uc0ac\uc6b0\ub8e8\uc2a4\ub294 \uc8fc\ub85c \uacfc\ub85c \ubd84\ub958\ub418\uc5c8\uc9c0\ub9cc, \uc54c\ub85c\uc0ac\uc6b0\ub8e8\uc2a4\uacfc\ub294 \ud655\uc2e4\ud788 \uc815\uc758\ub41c \uacfc\ub294 \uc544\ub2c8\uc5c8\ub2e4. \ubcf4\ud1b5 \uc900\uc804\ubb38\uc801\uc778 \uc791\ud488\ub4e4\uc5d0\uc11c\ub294 \u2018\uc54c\ub85c\uc0ac\uc6b0\ub8e8\uc2a4\uacfc\u2019\ub97c \uba54\uac08\ub85c\uc0ac\uc6b0\ub8e8\uc2a4\uacfc\ubcf4\ub2e4 \ud06c\uace0 \uc798 \uc54c\ub824\uc9c4 \ub2e4\uc591\ud55c \uac70\ub300 \uc218\uac01\ub958\ub4e4\uc744 \uac00\ub9ac\ud0a4\ub294 \uc6a9\uc5b4\ub85c \uc0ac\uc6a9\ud588\ub2e4. \uc54c\ub85c\uc0ac\uc6b0\ub8e8\uc2a4\uc640 \uadfc\uc5f0\ub41c\ub2e4\uace0 \uc0dd\uac01\ub418\uc5c8\ub358 \uc804\ud615\uc801\uc778 \uc218\uac01\ub958\ub85c\ub294 \uc778\ub3c4\uc0ac\uc6b0\ub8e8\uc2a4, \ud53c\uc544\ud2b8\ub2c8\uce20\ud0a4\uc0ac\uc6b0\ub8e8\uc2a4, \ud53c\ubca0\ud14c\uc544\uc6b0\uc0ac\uc6b0\ub8e8\uc2a4, \uc591\ucd94\uc544\ub178\uc0ac\uc6b0\ub8e8\uc2a4, \uc544\ud06c\ub85c\uce78\ud1a0\uc0ac\uc6b0\ub8e8\uc2a4, \ud0a4\ub780\ud14c\uc0ac\uc6b0\ub8e8\uc2a4, \ucf64\ud504\uc18c\uc218\ucfe0\uc2a4, \uc2a4\ud1a0\ucf00\uc18c\uc0ac\uc6b0\ub8e8\uc2a4, \uc2a4\uccb4\ucd94\uc544\ub178\uc0ac\uc6b0\ub8e8\uc2a4\uac00 \uc788\ub2e4. \ud604\uc7ac\ub294 \uc218\uac01\ub958\uc758 \ub2e4\uc591\uc131\uc5d0 \ub300\ud55c \ud604\ub300 \uc9c0\uc2dd\uacfc \uc9c4\ud654\uc801 \uc720\uc5f0 \uad00\uacc4\uc758 \uacc4\ud1b5\ubc1c\uc0dd\ud559\uc801 \uc5f0\uad6c\uc758 \ub3c4\ub798\ub85c \uc774\ub4e4 \uc218\uac01\ub958 \uc911 \ub300\ubd80\ubd84\uc740 \uc54c\ub85c\uc0ac\uc6b0\ub8e8\uc2a4\uacfc\ub85c \ubd84\ub958\ub418\uc9c0 \uc54a\ub294\ub2e4. \ub2e4\ub9cc \uc544\ud06c\ub85c\uce78\ud1a0\uc0ac\uc6b0\ub8e8\uc2a4\uc640 \uc591\ucd94\uc544\ub178\uc0ac\uc6b0\ub8e8\uc2a4\ub294 \uadfc\uc5f0\uacfc\uc5d0 \uc18d\ud55c\ub2e4."} {"question": "\uc81c51\uc7a5\uc5d0\uc11c \ub192\uc73c\uc2e0 \ubd84\uc774 \ub77c\uadf8\ub098\ub85c\ud06c\uc758 \uccab \ubc88\uc9f8 \uc9d5\ud6c4\ub77c\uace0 \ud55c \uac83\uc740 \ubb34\uc5c7\uc778\uac00?", "paragraph": "\uc81c51\uc7a5\uc5d0\uc11c\ub294 \u3008\ubb34\ub140\uc758 \uc608\uc5b8\u3009\uc744 \uc778\uc6a9\ud574\uac00\uba74\uc11c \ub77c\uadf8\ub098\ub85c\ud06c\uc758 \uacfc\uc815\uc5d0 \ub300\ud55c \uc0c1\uc138\ud55c \uc124\uba85\uc774 \uc774\ub8e8\uc5b4\uc9c0\uace0, \uc81c52\uc7a5\uacfc \uc81c53\uc7a5\uc5d0\uc11c\ub294 \ub77c\uadf8\ub098\ub85c\ud06c \uc774\ud6c4\uc758 \uc77c\ub4e4\uc774 \uc124\uba85\ub41c\ub2e4. \uc81c51\uc7a5\uc5d0\uc11c \ub192\uc73c\uc2e0 \ubd84\uc740 \ub77c\uadf8\ub098\ub85c\ud06c\uc758 \uccab \ubc88\uc9f8 \uc9d5\ud6c4\ub294 \ud54c\ubd88\ubca0\ud2b8\ub974\ub77c\uace0 \ub77c\uace0 \ud55c\ub2e4. \ud54c\ubd88\ubca0\ud2b8\ub974\ub294 \uc5ec\ub984 \uc5c6\uc774 \uaca8\uc6b8\ub9cc \uc138 \ubc88 \ubc18\ubcf5\ub418\ub294 \uc2dc\uae30\uc774\uba70, \uc774\ub54c \ud0dc\uc591\ub3c4 \uc4f8\ubaa8\uac00 \uc5c6\uc5b4\uc9c8 \uac83\uc774\ub2e4. \ub192\uc73c\uc2e0 \ubd84\uc740 \uadf8 \ub054\ucc0d\ud55c \uaca8\uc6b8\uc774 \uc2dc\uc791\ub418\uae30\uc5d0 \uc55e\uc11c \uc138 \ubc88\uc758 \uaca8\uc6b8\uc774 \uba3c\uc800 \ucc3e\uc544\uc624\uace0, \uc804\uc138\uacc4\uac00 \ub054\ucc0d\ud55c \uc804\uc7c1\uc73c\ub85c \ub4e4\ub053\uc744 \uac83\uc774\ub77c\uace0 \ubd80\uc5f0\ud55c\ub2e4. \uc774\ub54c \ud0d0\uc695\uc73c\ub85c \uc778\ud574 \ud615\uc81c\ub4e4\uc774 \uc8fd\uace0 \uc8fd\uc77c \uac83\uc774\uba70, \uac00\uc815\uc774 \ubb34\ub108\uc9c0\uace0 \uc0ac\ud68c\uac00 \ubb34\ub108\uc838\uc11c \ubd80\ubaa8\uc640 \uc790\uc2dd\ub4e4\uc774 \uace0\ud1b5\uc744 \uacaa\uc744 \uac83\uc774\ub2e4. \ub192\uc73c\uc2e0 \ubd84\uc740 \u3008\ubb34\ub140\uc758 \uc608\uc5b8\u3009 \uc81c45\uc808\uc744 \uc778\uc6a9\ud55c\ub2e4. \uadf8\ub9ac\uace0 \ub192\uc73c\uc2e0 \ubd84\uc740 \uadf8 \ub2e4\uc74c \ub291\ub300\uac00 \ud574\uc640 \ub2ec \ub0a8\ub9e4\ub97c \uc9d1\uc5b4\uc0bc\ud0a4\uace0, \uc778\ub958\ub294 \uc774 \ub09c\uc7a5\ud310 \uc18d\uc5d0\uc11c \ubb34\uc2dc\ubb34\uc2dc\ud55c \uc7ac\uc559\uc774 \uc77c\uc5b4\ub0a0 \uac83\uc774\ub77c\uace0 \uc0dd\uac01\ud558\uac8c \ub41c\ub2e4. \ud558\ub298\uc758 \ubcc4\ub4e4\uc774 \uc0ac\ub77c\uc9c0\uace0, \ub545\uacfc \uc0b0\uc774 \ub108\ubb34 \uc2ec\ud558\uac8c \ud754\ub4e4\ub824 \ub098\ubb34\ubfcc\ub9ac\ub4e4\uc774 \ud759 \ubc16\uc73c\ub85c \ud280\uc5b4\ub098\uc640 \ub4a4\ub525\ud78c\ub2e4. \uc0b0\uc774 \ubb34\ub108\uc9c0\uace0, \ubaa8\ub4e0 \uc18d\ubc15\uc774 \ud480\ub824 \ud39c\ub9ac\ub974\uac00 \uc18d\ubc15\uc5d0\uc11c \ubc97\uc5b4\ub09c\ub2e4.", "answer": "\ud54c\ubd88\ubca0\ud2b8\ub974", "sentence": "\uc81c51\uc7a5\uc5d0\uc11c \ub192\uc73c\uc2e0 \ubd84\uc740 \ub77c\uadf8\ub098\ub85c\ud06c\uc758 \uccab \ubc88\uc9f8 \uc9d5\ud6c4\ub294 \ud54c\ubd88\ubca0\ud2b8\ub974 \ub77c\uace0 \ub77c\uace0 \ud55c\ub2e4.", "paragraph_sentence": "\uc81c51\uc7a5\uc5d0\uc11c\ub294 \u3008\ubb34\ub140\uc758 \uc608\uc5b8\u3009\uc744 \uc778\uc6a9\ud574\uac00\uba74\uc11c \ub77c\uadf8\ub098\ub85c\ud06c\uc758 \uacfc\uc815\uc5d0 \ub300\ud55c \uc0c1\uc138\ud55c \uc124\uba85\uc774 \uc774\ub8e8\uc5b4\uc9c0\uace0, \uc81c52\uc7a5\uacfc \uc81c53\uc7a5\uc5d0\uc11c\ub294 \ub77c\uadf8\ub098\ub85c\ud06c \uc774\ud6c4\uc758 \uc77c\ub4e4\uc774 \uc124\uba85\ub41c\ub2e4. \uc81c51\uc7a5\uc5d0\uc11c \ub192\uc73c\uc2e0 \ubd84\uc740 \ub77c\uadf8\ub098\ub85c\ud06c\uc758 \uccab \ubc88\uc9f8 \uc9d5\ud6c4\ub294 \ud54c\ubd88\ubca0\ud2b8\ub974 \ub77c\uace0 \ub77c\uace0 \ud55c\ub2e4. \ud54c\ubd88\ubca0\ud2b8\ub974\ub294 \uc5ec\ub984 \uc5c6\uc774 \uaca8\uc6b8\ub9cc \uc138 \ubc88 \ubc18\ubcf5\ub418\ub294 \uc2dc\uae30\uc774\uba70, \uc774\ub54c \ud0dc\uc591\ub3c4 \uc4f8\ubaa8\uac00 \uc5c6\uc5b4\uc9c8 \uac83\uc774\ub2e4. \ub192\uc73c\uc2e0 \ubd84\uc740 \uadf8 \ub054\ucc0d\ud55c \uaca8\uc6b8\uc774 \uc2dc\uc791\ub418\uae30\uc5d0 \uc55e\uc11c \uc138 \ubc88\uc758 \uaca8\uc6b8\uc774 \uba3c\uc800 \ucc3e\uc544\uc624\uace0, \uc804\uc138\uacc4\uac00 \ub054\ucc0d\ud55c \uc804\uc7c1\uc73c\ub85c \ub4e4\ub053\uc744 \uac83\uc774\ub77c\uace0 \ubd80\uc5f0\ud55c\ub2e4. \uc774\ub54c \ud0d0\uc695\uc73c\ub85c \uc778\ud574 \ud615\uc81c\ub4e4\uc774 \uc8fd\uace0 \uc8fd\uc77c \uac83\uc774\uba70, \uac00\uc815\uc774 \ubb34\ub108\uc9c0\uace0 \uc0ac\ud68c\uac00 \ubb34\ub108\uc838\uc11c \ubd80\ubaa8\uc640 \uc790\uc2dd\ub4e4\uc774 \uace0\ud1b5\uc744 \uacaa\uc744 \uac83\uc774\ub2e4. \ub192\uc73c\uc2e0 \ubd84\uc740 \u3008\ubb34\ub140\uc758 \uc608\uc5b8\u3009 \uc81c45\uc808\uc744 \uc778\uc6a9\ud55c\ub2e4. \uadf8\ub9ac\uace0 \ub192\uc73c\uc2e0 \ubd84\uc740 \uadf8 \ub2e4\uc74c \ub291\ub300\uac00 \ud574\uc640 \ub2ec \ub0a8\ub9e4\ub97c \uc9d1\uc5b4\uc0bc\ud0a4\uace0, \uc778\ub958\ub294 \uc774 \ub09c\uc7a5\ud310 \uc18d\uc5d0\uc11c \ubb34\uc2dc\ubb34\uc2dc\ud55c \uc7ac\uc559\uc774 \uc77c\uc5b4\ub0a0 \uac83\uc774\ub77c\uace0 \uc0dd\uac01\ud558\uac8c \ub41c\ub2e4. \ud558\ub298\uc758 \ubcc4\ub4e4\uc774 \uc0ac\ub77c\uc9c0\uace0, \ub545\uacfc \uc0b0\uc774 \ub108\ubb34 \uc2ec\ud558\uac8c \ud754\ub4e4\ub824 \ub098\ubb34\ubfcc\ub9ac\ub4e4\uc774 \ud759 \ubc16\uc73c\ub85c \ud280\uc5b4\ub098\uc640 \ub4a4\ub525\ud78c\ub2e4. \uc0b0\uc774 \ubb34\ub108\uc9c0\uace0, \ubaa8\ub4e0 \uc18d\ubc15\uc774 \ud480\ub824 \ud39c\ub9ac\ub974\uac00 \uc18d\ubc15\uc5d0\uc11c \ubc97\uc5b4\ub09c\ub2e4.", "paragraph_answer": "\uc81c51\uc7a5\uc5d0\uc11c\ub294 \u3008\ubb34\ub140\uc758 \uc608\uc5b8\u3009\uc744 \uc778\uc6a9\ud574\uac00\uba74\uc11c \ub77c\uadf8\ub098\ub85c\ud06c\uc758 \uacfc\uc815\uc5d0 \ub300\ud55c \uc0c1\uc138\ud55c \uc124\uba85\uc774 \uc774\ub8e8\uc5b4\uc9c0\uace0, \uc81c52\uc7a5\uacfc \uc81c53\uc7a5\uc5d0\uc11c\ub294 \ub77c\uadf8\ub098\ub85c\ud06c \uc774\ud6c4\uc758 \uc77c\ub4e4\uc774 \uc124\uba85\ub41c\ub2e4. \uc81c51\uc7a5\uc5d0\uc11c \ub192\uc73c\uc2e0 \ubd84\uc740 \ub77c\uadf8\ub098\ub85c\ud06c\uc758 \uccab \ubc88\uc9f8 \uc9d5\ud6c4\ub294 \ud54c\ubd88\ubca0\ud2b8\ub974 \ub77c\uace0 \ub77c\uace0 \ud55c\ub2e4. \ud54c\ubd88\ubca0\ud2b8\ub974\ub294 \uc5ec\ub984 \uc5c6\uc774 \uaca8\uc6b8\ub9cc \uc138 \ubc88 \ubc18\ubcf5\ub418\ub294 \uc2dc\uae30\uc774\uba70, \uc774\ub54c \ud0dc\uc591\ub3c4 \uc4f8\ubaa8\uac00 \uc5c6\uc5b4\uc9c8 \uac83\uc774\ub2e4. \ub192\uc73c\uc2e0 \ubd84\uc740 \uadf8 \ub054\ucc0d\ud55c \uaca8\uc6b8\uc774 \uc2dc\uc791\ub418\uae30\uc5d0 \uc55e\uc11c \uc138 \ubc88\uc758 \uaca8\uc6b8\uc774 \uba3c\uc800 \ucc3e\uc544\uc624\uace0, \uc804\uc138\uacc4\uac00 \ub054\ucc0d\ud55c \uc804\uc7c1\uc73c\ub85c \ub4e4\ub053\uc744 \uac83\uc774\ub77c\uace0 \ubd80\uc5f0\ud55c\ub2e4. \uc774\ub54c \ud0d0\uc695\uc73c\ub85c \uc778\ud574 \ud615\uc81c\ub4e4\uc774 \uc8fd\uace0 \uc8fd\uc77c \uac83\uc774\uba70, \uac00\uc815\uc774 \ubb34\ub108\uc9c0\uace0 \uc0ac\ud68c\uac00 \ubb34\ub108\uc838\uc11c \ubd80\ubaa8\uc640 \uc790\uc2dd\ub4e4\uc774 \uace0\ud1b5\uc744 \uacaa\uc744 \uac83\uc774\ub2e4. \ub192\uc73c\uc2e0 \ubd84\uc740 \u3008\ubb34\ub140\uc758 \uc608\uc5b8\u3009 \uc81c45\uc808\uc744 \uc778\uc6a9\ud55c\ub2e4. \uadf8\ub9ac\uace0 \ub192\uc73c\uc2e0 \ubd84\uc740 \uadf8 \ub2e4\uc74c \ub291\ub300\uac00 \ud574\uc640 \ub2ec \ub0a8\ub9e4\ub97c \uc9d1\uc5b4\uc0bc\ud0a4\uace0, \uc778\ub958\ub294 \uc774 \ub09c\uc7a5\ud310 \uc18d\uc5d0\uc11c \ubb34\uc2dc\ubb34\uc2dc\ud55c \uc7ac\uc559\uc774 \uc77c\uc5b4\ub0a0 \uac83\uc774\ub77c\uace0 \uc0dd\uac01\ud558\uac8c \ub41c\ub2e4. \ud558\ub298\uc758 \ubcc4\ub4e4\uc774 \uc0ac\ub77c\uc9c0\uace0, \ub545\uacfc \uc0b0\uc774 \ub108\ubb34 \uc2ec\ud558\uac8c \ud754\ub4e4\ub824 \ub098\ubb34\ubfcc\ub9ac\ub4e4\uc774 \ud759 \ubc16\uc73c\ub85c \ud280\uc5b4\ub098\uc640 \ub4a4\ub525\ud78c\ub2e4. \uc0b0\uc774 \ubb34\ub108\uc9c0\uace0, \ubaa8\ub4e0 \uc18d\ubc15\uc774 \ud480\ub824 \ud39c\ub9ac\ub974\uac00 \uc18d\ubc15\uc5d0\uc11c \ubc97\uc5b4\ub09c\ub2e4.", "sentence_answer": "\uc81c51\uc7a5\uc5d0\uc11c \ub192\uc73c\uc2e0 \ubd84\uc740 \ub77c\uadf8\ub098\ub85c\ud06c\uc758 \uccab \ubc88\uc9f8 \uc9d5\ud6c4\ub294 \ud54c\ubd88\ubca0\ud2b8\ub974 \ub77c\uace0 \ub77c\uace0 \ud55c\ub2e4.", "paragraph_id": "6493726-5-1", "paragraph_question": "question: \uc81c51\uc7a5\uc5d0\uc11c \ub192\uc73c\uc2e0 \ubd84\uc774 \ub77c\uadf8\ub098\ub85c\ud06c\uc758 \uccab \ubc88\uc9f8 \uc9d5\ud6c4\ub77c\uace0 \ud55c \uac83\uc740 \ubb34\uc5c7\uc778\uac00?, context: \uc81c51\uc7a5\uc5d0\uc11c\ub294 \u3008\ubb34\ub140\uc758 \uc608\uc5b8\u3009\uc744 \uc778\uc6a9\ud574\uac00\uba74\uc11c \ub77c\uadf8\ub098\ub85c\ud06c\uc758 \uacfc\uc815\uc5d0 \ub300\ud55c \uc0c1\uc138\ud55c \uc124\uba85\uc774 \uc774\ub8e8\uc5b4\uc9c0\uace0, \uc81c52\uc7a5\uacfc \uc81c53\uc7a5\uc5d0\uc11c\ub294 \ub77c\uadf8\ub098\ub85c\ud06c \uc774\ud6c4\uc758 \uc77c\ub4e4\uc774 \uc124\uba85\ub41c\ub2e4. \uc81c51\uc7a5\uc5d0\uc11c \ub192\uc73c\uc2e0 \ubd84\uc740 \ub77c\uadf8\ub098\ub85c\ud06c\uc758 \uccab \ubc88\uc9f8 \uc9d5\ud6c4\ub294 \ud54c\ubd88\ubca0\ud2b8\ub974\ub77c\uace0 \ub77c\uace0 \ud55c\ub2e4. \ud54c\ubd88\ubca0\ud2b8\ub974\ub294 \uc5ec\ub984 \uc5c6\uc774 \uaca8\uc6b8\ub9cc \uc138 \ubc88 \ubc18\ubcf5\ub418\ub294 \uc2dc\uae30\uc774\uba70, \uc774\ub54c \ud0dc\uc591\ub3c4 \uc4f8\ubaa8\uac00 \uc5c6\uc5b4\uc9c8 \uac83\uc774\ub2e4. \ub192\uc73c\uc2e0 \ubd84\uc740 \uadf8 \ub054\ucc0d\ud55c \uaca8\uc6b8\uc774 \uc2dc\uc791\ub418\uae30\uc5d0 \uc55e\uc11c \uc138 \ubc88\uc758 \uaca8\uc6b8\uc774 \uba3c\uc800 \ucc3e\uc544\uc624\uace0, \uc804\uc138\uacc4\uac00 \ub054\ucc0d\ud55c \uc804\uc7c1\uc73c\ub85c \ub4e4\ub053\uc744 \uac83\uc774\ub77c\uace0 \ubd80\uc5f0\ud55c\ub2e4. \uc774\ub54c \ud0d0\uc695\uc73c\ub85c \uc778\ud574 \ud615\uc81c\ub4e4\uc774 \uc8fd\uace0 \uc8fd\uc77c \uac83\uc774\uba70, \uac00\uc815\uc774 \ubb34\ub108\uc9c0\uace0 \uc0ac\ud68c\uac00 \ubb34\ub108\uc838\uc11c \ubd80\ubaa8\uc640 \uc790\uc2dd\ub4e4\uc774 \uace0\ud1b5\uc744 \uacaa\uc744 \uac83\uc774\ub2e4. \ub192\uc73c\uc2e0 \ubd84\uc740 \u3008\ubb34\ub140\uc758 \uc608\uc5b8\u3009 \uc81c45\uc808\uc744 \uc778\uc6a9\ud55c\ub2e4. \uadf8\ub9ac\uace0 \ub192\uc73c\uc2e0 \ubd84\uc740 \uadf8 \ub2e4\uc74c \ub291\ub300\uac00 \ud574\uc640 \ub2ec \ub0a8\ub9e4\ub97c \uc9d1\uc5b4\uc0bc\ud0a4\uace0, \uc778\ub958\ub294 \uc774 \ub09c\uc7a5\ud310 \uc18d\uc5d0\uc11c \ubb34\uc2dc\ubb34\uc2dc\ud55c \uc7ac\uc559\uc774 \uc77c\uc5b4\ub0a0 \uac83\uc774\ub77c\uace0 \uc0dd\uac01\ud558\uac8c \ub41c\ub2e4. \ud558\ub298\uc758 \ubcc4\ub4e4\uc774 \uc0ac\ub77c\uc9c0\uace0, \ub545\uacfc \uc0b0\uc774 \ub108\ubb34 \uc2ec\ud558\uac8c \ud754\ub4e4\ub824 \ub098\ubb34\ubfcc\ub9ac\ub4e4\uc774 \ud759 \ubc16\uc73c\ub85c \ud280\uc5b4\ub098\uc640 \ub4a4\ub525\ud78c\ub2e4. \uc0b0\uc774 \ubb34\ub108\uc9c0\uace0, \ubaa8\ub4e0 \uc18d\ubc15\uc774 \ud480\ub824 \ud39c\ub9ac\ub974\uac00 \uc18d\ubc15\uc5d0\uc11c \ubc97\uc5b4\ub09c\ub2e4."} {"question": "\ub77c\ube48\uc744 \uac00\uc7a5 \uc5b4\ub9b0\ub098\uc774\uc5d0 1\uc704\uc5d0 \uc624\ub978 \uc5ec\uc790 \uac00\uc218\ub77c\ub294 \ud0c0\uc774\ud2c0\uc744 \uac00\uc9c0\uac8c \ud574\uc900 \uace1\uc740?", "paragraph": "Let Go\ub294 \uc601\uad6d\uc5d0\uc11c 1\uc704\uc5d0 \uc624\ub974\uba70 \uac00\uc7a5 \uc5b4\ub9b0\ub098\uc774\uc5d0 1\uc704\uc5d0 \uc624\ub978 \uc5ec\uc790 \uac00\uc218\ub77c\ub294 \uae30\ub85d\uc744 \uc138\uc6e0\uace0, \ubbf8\uad6d \uc74c\ubc18 \uc0b0\uc5c5 \ud611\ud68c\uc5d0\uc11c 6x \ud50c\ub798\ud2f0\ub118 \uc778\uc99d\uc744 \ud588\ub2e4. \uc774 \uc74c\ubc18\uc740 2009\ub144\uae4c\uc9c0 \uc138\uacc4\uc801\uc73c\ub85c 1,600\ub9cc \uc7a5\uc758 \ud310\ub9e4\uace0\ub97c \uc62c\ub9ac\uace0\uc788\ub2e4. \ub77c\ube48\uc758 \ub370\ubdd4 \uc2f1\uae00 \"Complicated\" \uc138\uacc4 \uc5ec\ub7ec \uad6d\uac00\uc5d0\uc11c 1\uc704\ub97c \ucc28\uc9c0\ud558\uba70 \uc568\ubc94\uc758 \uc131\uacf5\uc744 \ub3c4\uc654\ub2e4. \ub77c\ube48\uc758 \ub450 \ubc88\uc9f8 \uc815\uaddc \uc74c\ubc18 Under My Skin\ub294 2004\ub144 \ubc1c\ub9e4\ub418\uc5c8\uace0, \ubbf8\uad6d \ube4c\ubcf4\ub4dc 200\uc5d0\uc11c \ucc98\uc74c\uc73c\ub85c 1\uc704\uc5d0 \uc62c\ub790\ub2e4. \uc774 \uc74c\ubc18\uc740 \uc138\uacc4\uc801\uc73c\ub85c 1,000\ub9cc \uc7a5\uc744 \ud314\uc544\uce58\uc6e0\ub2e4. 2007\ub144\uc5d0\ub294 \uc138 \ubc88\uc9f8 \uc815\uaddc \uc74c\ubc18 The Best Damn Thing\uc744 \ubc1c\ub9e4\ud588\uace0, \uc601\uad6d\uc5d0\uc11c 1\uc704\ub97c \ud588\ub2e4. \uc74c\ubc18\uc758 \ub9ac\ub4dc \uc2f1\uae00 \"Girlfriend\"\ub294 \ubbf8\uad6d \ube4c\ubcf4\ub4dc \ud56b 100\uc5d0\uc11c \ucc98\uc74c\uc73c\ub85c 1\uc704\ub97c \ud588\uc73c\uba70, \uc5ec\ub7ec \ub098\ub77c\uc5d0\uc11c 1\uc704\ub97c \ud588\ub2e4. 2011\ub144 3\uc6d4\uc5d0\ub294 \ub77c\ube48\uc758 \ub124 \ubc88\uc9f8 \uc815\uaddc \uc74c\ubc18 Goodbye Lullaby\ub97c \ubc1c\ub9e4\ud588\ub2e4. Goodbye Lullaby\ub294 \uc624\uc2a4\ud2b8\ub808\uc77c\ub9ac\uc544, \uc77c\ubcf8\uc5d0\uc11c 1\uc704\uc5d0 \uc62c\ub790\uace0, \ubbf8\uad6d\uc5d0\uc120 \ucd5c\uace0 4\uc704\uc5d0 \uc62c\ub790\ub2e4.", "answer": "Let Go", "sentence": "Let Go \ub294 \uc601\uad6d\uc5d0\uc11c 1\uc704\uc5d0 \uc624\ub974\uba70 \uac00\uc7a5 \uc5b4\ub9b0\ub098\uc774\uc5d0 1\uc704\uc5d0 \uc624\ub978 \uc5ec\uc790 \uac00\uc218\ub77c\ub294 \uae30\ub85d\uc744 \uc138\uc6e0\uace0, \ubbf8\uad6d \uc74c\ubc18 \uc0b0\uc5c5 \ud611\ud68c\uc5d0\uc11c 6x \ud50c\ub798\ud2f0\ub118 \uc778\uc99d\uc744 \ud588\ub2e4.", "paragraph_sentence": " Let Go \ub294 \uc601\uad6d\uc5d0\uc11c 1\uc704\uc5d0 \uc624\ub974\uba70 \uac00\uc7a5 \uc5b4\ub9b0\ub098\uc774\uc5d0 1\uc704\uc5d0 \uc624\ub978 \uc5ec\uc790 \uac00\uc218\ub77c\ub294 \uae30\ub85d\uc744 \uc138\uc6e0\uace0, \ubbf8\uad6d \uc74c\ubc18 \uc0b0\uc5c5 \ud611\ud68c\uc5d0\uc11c 6x \ud50c\ub798\ud2f0\ub118 \uc778\uc99d\uc744 \ud588\ub2e4. \uc774 \uc74c\ubc18\uc740 2009\ub144\uae4c\uc9c0 \uc138\uacc4\uc801\uc73c\ub85c 1,600\ub9cc \uc7a5\uc758 \ud310\ub9e4\uace0\ub97c \uc62c\ub9ac\uace0\uc788\ub2e4. \ub77c\ube48\uc758 \ub370\ubdd4 \uc2f1\uae00 \"Complicated\" \uc138\uacc4 \uc5ec\ub7ec \uad6d\uac00\uc5d0\uc11c 1\uc704\ub97c \ucc28\uc9c0\ud558\uba70 \uc568\ubc94\uc758 \uc131\uacf5\uc744 \ub3c4\uc654\ub2e4. \ub77c\ube48\uc758 \ub450 \ubc88\uc9f8 \uc815\uaddc \uc74c\ubc18 Under My Skin\ub294 2004\ub144 \ubc1c\ub9e4\ub418\uc5c8\uace0, \ubbf8\uad6d \ube4c\ubcf4\ub4dc 200\uc5d0\uc11c \ucc98\uc74c\uc73c\ub85c 1\uc704\uc5d0 \uc62c\ub790\ub2e4. \uc774 \uc74c\ubc18\uc740 \uc138\uacc4\uc801\uc73c\ub85c 1,000\ub9cc \uc7a5\uc744 \ud314\uc544\uce58\uc6e0\ub2e4. 2007\ub144\uc5d0\ub294 \uc138 \ubc88\uc9f8 \uc815\uaddc \uc74c\ubc18 The Best Damn Thing\uc744 \ubc1c\ub9e4\ud588\uace0, \uc601\uad6d\uc5d0\uc11c 1\uc704\ub97c \ud588\ub2e4. \uc74c\ubc18\uc758 \ub9ac\ub4dc \uc2f1\uae00 \"Girlfriend\"\ub294 \ubbf8\uad6d \ube4c\ubcf4\ub4dc \ud56b 100\uc5d0\uc11c \ucc98\uc74c\uc73c\ub85c 1\uc704\ub97c \ud588\uc73c\uba70, \uc5ec\ub7ec \ub098\ub77c\uc5d0\uc11c 1\uc704\ub97c \ud588\ub2e4. 2011\ub144 3\uc6d4\uc5d0\ub294 \ub77c\ube48\uc758 \ub124 \ubc88\uc9f8 \uc815\uaddc \uc74c\ubc18 Goodbye Lullaby\ub97c \ubc1c\ub9e4\ud588\ub2e4. Goodbye Lullaby\ub294 \uc624\uc2a4\ud2b8\ub808\uc77c\ub9ac\uc544, \uc77c\ubcf8\uc5d0\uc11c 1\uc704\uc5d0 \uc62c\ub790\uace0, \ubbf8\uad6d\uc5d0\uc120 \ucd5c\uace0 4\uc704\uc5d0 \uc62c\ub790\ub2e4.", "paragraph_answer": " Let Go \ub294 \uc601\uad6d\uc5d0\uc11c 1\uc704\uc5d0 \uc624\ub974\uba70 \uac00\uc7a5 \uc5b4\ub9b0\ub098\uc774\uc5d0 1\uc704\uc5d0 \uc624\ub978 \uc5ec\uc790 \uac00\uc218\ub77c\ub294 \uae30\ub85d\uc744 \uc138\uc6e0\uace0, \ubbf8\uad6d \uc74c\ubc18 \uc0b0\uc5c5 \ud611\ud68c\uc5d0\uc11c 6x \ud50c\ub798\ud2f0\ub118 \uc778\uc99d\uc744 \ud588\ub2e4. \uc774 \uc74c\ubc18\uc740 2009\ub144\uae4c\uc9c0 \uc138\uacc4\uc801\uc73c\ub85c 1,600\ub9cc \uc7a5\uc758 \ud310\ub9e4\uace0\ub97c \uc62c\ub9ac\uace0\uc788\ub2e4. \ub77c\ube48\uc758 \ub370\ubdd4 \uc2f1\uae00 \"Complicated\" \uc138\uacc4 \uc5ec\ub7ec \uad6d\uac00\uc5d0\uc11c 1\uc704\ub97c \ucc28\uc9c0\ud558\uba70 \uc568\ubc94\uc758 \uc131\uacf5\uc744 \ub3c4\uc654\ub2e4. \ub77c\ube48\uc758 \ub450 \ubc88\uc9f8 \uc815\uaddc \uc74c\ubc18 Under My Skin\ub294 2004\ub144 \ubc1c\ub9e4\ub418\uc5c8\uace0, \ubbf8\uad6d \ube4c\ubcf4\ub4dc 200\uc5d0\uc11c \ucc98\uc74c\uc73c\ub85c 1\uc704\uc5d0 \uc62c\ub790\ub2e4. \uc774 \uc74c\ubc18\uc740 \uc138\uacc4\uc801\uc73c\ub85c 1,000\ub9cc \uc7a5\uc744 \ud314\uc544\uce58\uc6e0\ub2e4. 2007\ub144\uc5d0\ub294 \uc138 \ubc88\uc9f8 \uc815\uaddc \uc74c\ubc18 The Best Damn Thing\uc744 \ubc1c\ub9e4\ud588\uace0, \uc601\uad6d\uc5d0\uc11c 1\uc704\ub97c \ud588\ub2e4. \uc74c\ubc18\uc758 \ub9ac\ub4dc \uc2f1\uae00 \"Girlfriend\"\ub294 \ubbf8\uad6d \ube4c\ubcf4\ub4dc \ud56b 100\uc5d0\uc11c \ucc98\uc74c\uc73c\ub85c 1\uc704\ub97c \ud588\uc73c\uba70, \uc5ec\ub7ec \ub098\ub77c\uc5d0\uc11c 1\uc704\ub97c \ud588\ub2e4. 2011\ub144 3\uc6d4\uc5d0\ub294 \ub77c\ube48\uc758 \ub124 \ubc88\uc9f8 \uc815\uaddc \uc74c\ubc18 Goodbye Lullaby\ub97c \ubc1c\ub9e4\ud588\ub2e4. Goodbye Lullaby\ub294 \uc624\uc2a4\ud2b8\ub808\uc77c\ub9ac\uc544, \uc77c\ubcf8\uc5d0\uc11c 1\uc704\uc5d0 \uc62c\ub790\uace0, \ubbf8\uad6d\uc5d0\uc120 \ucd5c\uace0 4\uc704\uc5d0 \uc62c\ub790\ub2e4.", "sentence_answer": " Let Go \ub294 \uc601\uad6d\uc5d0\uc11c 1\uc704\uc5d0 \uc624\ub974\uba70 \uac00\uc7a5 \uc5b4\ub9b0\ub098\uc774\uc5d0 1\uc704\uc5d0 \uc624\ub978 \uc5ec\uc790 \uac00\uc218\ub77c\ub294 \uae30\ub85d\uc744 \uc138\uc6e0\uace0, \ubbf8\uad6d \uc74c\ubc18 \uc0b0\uc5c5 \ud611\ud68c\uc5d0\uc11c 6x \ud50c\ub798\ud2f0\ub118 \uc778\uc99d\uc744 \ud588\ub2e4.", "paragraph_id": "6523479-0-0", "paragraph_question": "question: \ub77c\ube48\uc744 \uac00\uc7a5 \uc5b4\ub9b0\ub098\uc774\uc5d0 1\uc704\uc5d0 \uc624\ub978 \uc5ec\uc790 \uac00\uc218\ub77c\ub294 \ud0c0\uc774\ud2c0\uc744 \uac00\uc9c0\uac8c \ud574\uc900 \uace1\uc740?, context: Let Go\ub294 \uc601\uad6d\uc5d0\uc11c 1\uc704\uc5d0 \uc624\ub974\uba70 \uac00\uc7a5 \uc5b4\ub9b0\ub098\uc774\uc5d0 1\uc704\uc5d0 \uc624\ub978 \uc5ec\uc790 \uac00\uc218\ub77c\ub294 \uae30\ub85d\uc744 \uc138\uc6e0\uace0, \ubbf8\uad6d \uc74c\ubc18 \uc0b0\uc5c5 \ud611\ud68c\uc5d0\uc11c 6x \ud50c\ub798\ud2f0\ub118 \uc778\uc99d\uc744 \ud588\ub2e4. \uc774 \uc74c\ubc18\uc740 2009\ub144\uae4c\uc9c0 \uc138\uacc4\uc801\uc73c\ub85c 1,600\ub9cc \uc7a5\uc758 \ud310\ub9e4\uace0\ub97c \uc62c\ub9ac\uace0\uc788\ub2e4. \ub77c\ube48\uc758 \ub370\ubdd4 \uc2f1\uae00 \"Complicated\" \uc138\uacc4 \uc5ec\ub7ec \uad6d\uac00\uc5d0\uc11c 1\uc704\ub97c \ucc28\uc9c0\ud558\uba70 \uc568\ubc94\uc758 \uc131\uacf5\uc744 \ub3c4\uc654\ub2e4. \ub77c\ube48\uc758 \ub450 \ubc88\uc9f8 \uc815\uaddc \uc74c\ubc18 Under My Skin\ub294 2004\ub144 \ubc1c\ub9e4\ub418\uc5c8\uace0, \ubbf8\uad6d \ube4c\ubcf4\ub4dc 200\uc5d0\uc11c \ucc98\uc74c\uc73c\ub85c 1\uc704\uc5d0 \uc62c\ub790\ub2e4. \uc774 \uc74c\ubc18\uc740 \uc138\uacc4\uc801\uc73c\ub85c 1,000\ub9cc \uc7a5\uc744 \ud314\uc544\uce58\uc6e0\ub2e4. 2007\ub144\uc5d0\ub294 \uc138 \ubc88\uc9f8 \uc815\uaddc \uc74c\ubc18 The Best Damn Thing\uc744 \ubc1c\ub9e4\ud588\uace0, \uc601\uad6d\uc5d0\uc11c 1\uc704\ub97c \ud588\ub2e4. \uc74c\ubc18\uc758 \ub9ac\ub4dc \uc2f1\uae00 \"Girlfriend\"\ub294 \ubbf8\uad6d \ube4c\ubcf4\ub4dc \ud56b 100\uc5d0\uc11c \ucc98\uc74c\uc73c\ub85c 1\uc704\ub97c \ud588\uc73c\uba70, \uc5ec\ub7ec \ub098\ub77c\uc5d0\uc11c 1\uc704\ub97c \ud588\ub2e4. 2011\ub144 3\uc6d4\uc5d0\ub294 \ub77c\ube48\uc758 \ub124 \ubc88\uc9f8 \uc815\uaddc \uc74c\ubc18 Goodbye Lullaby\ub97c \ubc1c\ub9e4\ud588\ub2e4. Goodbye Lullaby\ub294 \uc624\uc2a4\ud2b8\ub808\uc77c\ub9ac\uc544, \uc77c\ubcf8\uc5d0\uc11c 1\uc704\uc5d0 \uc62c\ub790\uace0, \ubbf8\uad6d\uc5d0\uc120 \ucd5c\uace0 4\uc704\uc5d0 \uc62c\ub790\ub2e4."} {"question": "\ubca0\uc774\ube44\ubcf5\uc2a4\uc758 \ub9ac\ub4dc \ubcf4\uceec \ucc28\uc720\ubbf8\uac00 \uac11\uc791\uc2a4\ub7fd\uac8c \ubbf8\uad6d\uc73c\ub85c \ub3cc\uc544\uac00\uac8c \ub418\uba74\uc11c \uce90\uc2a4\ud305\ub41c \ub300\ud0c0\uba64\ubc84\uc758 \uc774\ub984\uc740 \ubb34\uc5c7\uc778\uac00?", "paragraph": "<\uba38\ub9ac\ud558\ub294 \ub0a0>\ub85c \ud65c\ub3d9\uc744 \uc774\uc5b4\uac00\ub358 10\uc6d4 \uc911\uc21c, \ub9ac\ub4dc \ubcf4\uceec \ucc28\uc720\ubbf8\uac00 \uc5b4\ub290 \ub0a0 \uac11\uc790\uae30 \ubbf8\uad6d\uc73c\ub85c \ub3cc\uc544\uac00\uba74\uc11c \ud300\uc5d0 \uc704\uae30\uac00 \ucc3e\uc544\uc628\ub2e4. \ud65c\ub3d9\uc5d0 \uce58\uba85\ud0c0\ub97c \uc785\uc740 \ubca0\uc774\ube44\ubcf5\uc2a4\ub294 \uace0\uc815 \ucd9c\uc5f0\uc911\uc774\ub358 SBS <\uae30\uc05c \uc6b0\ub9ac \uc88b\uc740\ub0a0>\uc744 \uc81c\uc678\ud55c \ubaa8\ub4e0 \ubc29\uc1a1 \uc2a4\ucf00\uc904\uc744 \ucde8\uc18c\ud558\uace0 9\uc6d4 \uacbd <\uba38\ub9ac\ud558\ub294 \ub0a0> \ubba4\uc9c1\ube44\ub514\uc624 \ucd2c\uc601 \ud604\uc7a5\uc5d0\uc11c \uce90\uc2a4\ud305\ub418\uc5c8\ub358 \uac04\ubbf8\uc5f0\uc744 \ub300\ud0c0\uba64\ubc84\ub85c \uc601\uc785\uc2dc\ud0a8\ub4a4 \ub450\ub2ec\uac04\uc758 \uc5f0\uc2b5\uae30\uac04\uc744 \uac70\uccd0 11\uc6d4\ubd80\ud130 \uc5b8\ub860\uc5d0 \uba64\ubc84\uad50\uccb4 \uc18c\uc2dd\uc744 \uc54c\ub9ac\uace0 \ud65c\ub3d9\uc744 \uc7ac\uac1c\ud55c\ub2e4. \uac04\ubbf8\uc5f0\uc774 \ud569\ub958\ub418\uace0 \uc560\ucd08\uc5d0 \uae30\ud68d\ud558\ub358 \uc5ec\uc131\ub4e4\uc744 \ub300\ubcc0\ud558\ub294 \uc774\ubbf8\uc9c0\uc640\ub294 \uc644\uc804\ud788 \ub2e4\ub978 \ud2f4\ud31d \uc2a4\ud0c0\uc77c\uc758 \uc2a4\ud0a4\ubcf5\uc744 \uc785\uace0 \ub4f1\uc7a5\ud55c\ub2e4. \uadf8\ub9ac\uace0 \uc548\ubb34\uc801\uc778 \uba74\uc5d0\uc11c\ub3c4 \ub9ce\uc740 \ubcc0\ud654\ub97c \uc8fc\uc5c8\ub294\ub370 \uba64\ubc84 \uc774\ud76c\uc9c4\uacfc \uac04\ubbf8\uc5f0\uc740 \ucee4\ud50c \ub304\uc2a4\ub97c, \uadf8\ub9ac\uace0 \uac1c\uc778\ud30c\ud2b8\uac00 \uc5c6\uc5c8\ub358 \uae40\uc774\uc9c0\ub294 \uba54\uc778\ubcf4\uceec \ucc28\uc720\ubbf8\uc758 \ud30c\ud2b8\ub97c \ub9bd\uc2f1\ud06c\ud558\uba74\uc11c \uc804\ubcf4\ub2e4\ub294 \uc678\uc801\uc778 \ubd80\ubd84\uc73c\ub85c \uc5b4\ud544\ud55c\ub2e4.", "answer": "\uac04\ubbf8\uc5f0", "sentence": "\ud65c\ub3d9\uc5d0 \uce58\uba85\ud0c0\ub97c \uc785\uc740 \ubca0\uc774\ube44\ubcf5\uc2a4\ub294 \uace0\uc815 \ucd9c\uc5f0\uc911\uc774\ub358 SBS <\uae30\uc05c \uc6b0\ub9ac \uc88b\uc740\ub0a0>\uc744 \uc81c\uc678\ud55c \ubaa8\ub4e0 \ubc29\uc1a1 \uc2a4\ucf00\uc904\uc744 \ucde8\uc18c\ud558\uace0 9\uc6d4 \uacbd <\uba38\ub9ac\ud558\ub294 \ub0a0> \ubba4\uc9c1\ube44\ub514\uc624 \ucd2c\uc601 \ud604\uc7a5\uc5d0\uc11c \uce90\uc2a4\ud305\ub418\uc5c8\ub358 \uac04\ubbf8\uc5f0 \uc744 \ub300\ud0c0\uba64\ubc84\ub85c \uc601\uc785\uc2dc\ud0a8\ub4a4 \ub450\ub2ec\uac04\uc758 \uc5f0\uc2b5\uae30\uac04\uc744 \uac70\uccd0 11\uc6d4\ubd80\ud130 \uc5b8\ub860\uc5d0 \uba64\ubc84\uad50\uccb4 \uc18c\uc2dd\uc744 \uc54c\ub9ac\uace0 \ud65c\ub3d9\uc744 \uc7ac\uac1c\ud55c\ub2e4.", "paragraph_sentence": "<\uba38\ub9ac\ud558\ub294 \ub0a0>\ub85c \ud65c\ub3d9\uc744 \uc774\uc5b4\uac00\ub358 10\uc6d4 \uc911\uc21c, \ub9ac\ub4dc \ubcf4\uceec \ucc28\uc720\ubbf8\uac00 \uc5b4\ub290 \ub0a0 \uac11\uc790\uae30 \ubbf8\uad6d\uc73c\ub85c \ub3cc\uc544\uac00\uba74\uc11c \ud300\uc5d0 \uc704\uae30\uac00 \ucc3e\uc544\uc628\ub2e4. \ud65c\ub3d9\uc5d0 \uce58\uba85\ud0c0\ub97c \uc785\uc740 \ubca0\uc774\ube44\ubcf5\uc2a4\ub294 \uace0\uc815 \ucd9c\uc5f0\uc911\uc774\ub358 SBS <\uae30\uc05c \uc6b0\ub9ac \uc88b\uc740\ub0a0>\uc744 \uc81c\uc678\ud55c \ubaa8\ub4e0 \ubc29\uc1a1 \uc2a4\ucf00\uc904\uc744 \ucde8\uc18c\ud558\uace0 9\uc6d4 \uacbd <\uba38\ub9ac\ud558\ub294 \ub0a0> \ubba4\uc9c1\ube44\ub514\uc624 \ucd2c\uc601 \ud604\uc7a5\uc5d0\uc11c \uce90\uc2a4\ud305\ub418\uc5c8\ub358 \uac04\ubbf8\uc5f0 \uc744 \ub300\ud0c0\uba64\ubc84\ub85c \uc601\uc785\uc2dc\ud0a8\ub4a4 \ub450\ub2ec\uac04\uc758 \uc5f0\uc2b5\uae30\uac04\uc744 \uac70\uccd0 11\uc6d4\ubd80\ud130 \uc5b8\ub860\uc5d0 \uba64\ubc84\uad50\uccb4 \uc18c\uc2dd\uc744 \uc54c\ub9ac\uace0 \ud65c\ub3d9\uc744 \uc7ac\uac1c\ud55c\ub2e4. \uac04\ubbf8\uc5f0\uc774 \ud569\ub958\ub418\uace0 \uc560\ucd08\uc5d0 \uae30\ud68d\ud558\ub358 \uc5ec\uc131\ub4e4\uc744 \ub300\ubcc0\ud558\ub294 \uc774\ubbf8\uc9c0\uc640\ub294 \uc644\uc804\ud788 \ub2e4\ub978 \ud2f4\ud31d \uc2a4\ud0c0\uc77c\uc758 \uc2a4\ud0a4\ubcf5\uc744 \uc785\uace0 \ub4f1\uc7a5\ud55c\ub2e4. \uadf8\ub9ac\uace0 \uc548\ubb34\uc801\uc778 \uba74\uc5d0\uc11c\ub3c4 \ub9ce\uc740 \ubcc0\ud654\ub97c \uc8fc\uc5c8\ub294\ub370 \uba64\ubc84 \uc774\ud76c\uc9c4\uacfc \uac04\ubbf8\uc5f0\uc740 \ucee4\ud50c \ub304\uc2a4\ub97c, \uadf8\ub9ac\uace0 \uac1c\uc778\ud30c\ud2b8\uac00 \uc5c6\uc5c8\ub358 \uae40\uc774\uc9c0\ub294 \uba54\uc778\ubcf4\uceec \ucc28\uc720\ubbf8\uc758 \ud30c\ud2b8\ub97c \ub9bd\uc2f1\ud06c\ud558\uba74\uc11c \uc804\ubcf4\ub2e4\ub294 \uc678\uc801\uc778 \ubd80\ubd84\uc73c\ub85c \uc5b4\ud544\ud55c\ub2e4.", "paragraph_answer": "<\uba38\ub9ac\ud558\ub294 \ub0a0>\ub85c \ud65c\ub3d9\uc744 \uc774\uc5b4\uac00\ub358 10\uc6d4 \uc911\uc21c, \ub9ac\ub4dc \ubcf4\uceec \ucc28\uc720\ubbf8\uac00 \uc5b4\ub290 \ub0a0 \uac11\uc790\uae30 \ubbf8\uad6d\uc73c\ub85c \ub3cc\uc544\uac00\uba74\uc11c \ud300\uc5d0 \uc704\uae30\uac00 \ucc3e\uc544\uc628\ub2e4. \ud65c\ub3d9\uc5d0 \uce58\uba85\ud0c0\ub97c \uc785\uc740 \ubca0\uc774\ube44\ubcf5\uc2a4\ub294 \uace0\uc815 \ucd9c\uc5f0\uc911\uc774\ub358 SBS <\uae30\uc05c \uc6b0\ub9ac \uc88b\uc740\ub0a0>\uc744 \uc81c\uc678\ud55c \ubaa8\ub4e0 \ubc29\uc1a1 \uc2a4\ucf00\uc904\uc744 \ucde8\uc18c\ud558\uace0 9\uc6d4 \uacbd <\uba38\ub9ac\ud558\ub294 \ub0a0> \ubba4\uc9c1\ube44\ub514\uc624 \ucd2c\uc601 \ud604\uc7a5\uc5d0\uc11c \uce90\uc2a4\ud305\ub418\uc5c8\ub358 \uac04\ubbf8\uc5f0 \uc744 \ub300\ud0c0\uba64\ubc84\ub85c \uc601\uc785\uc2dc\ud0a8\ub4a4 \ub450\ub2ec\uac04\uc758 \uc5f0\uc2b5\uae30\uac04\uc744 \uac70\uccd0 11\uc6d4\ubd80\ud130 \uc5b8\ub860\uc5d0 \uba64\ubc84\uad50\uccb4 \uc18c\uc2dd\uc744 \uc54c\ub9ac\uace0 \ud65c\ub3d9\uc744 \uc7ac\uac1c\ud55c\ub2e4. \uac04\ubbf8\uc5f0\uc774 \ud569\ub958\ub418\uace0 \uc560\ucd08\uc5d0 \uae30\ud68d\ud558\ub358 \uc5ec\uc131\ub4e4\uc744 \ub300\ubcc0\ud558\ub294 \uc774\ubbf8\uc9c0\uc640\ub294 \uc644\uc804\ud788 \ub2e4\ub978 \ud2f4\ud31d \uc2a4\ud0c0\uc77c\uc758 \uc2a4\ud0a4\ubcf5\uc744 \uc785\uace0 \ub4f1\uc7a5\ud55c\ub2e4. \uadf8\ub9ac\uace0 \uc548\ubb34\uc801\uc778 \uba74\uc5d0\uc11c\ub3c4 \ub9ce\uc740 \ubcc0\ud654\ub97c \uc8fc\uc5c8\ub294\ub370 \uba64\ubc84 \uc774\ud76c\uc9c4\uacfc \uac04\ubbf8\uc5f0\uc740 \ucee4\ud50c \ub304\uc2a4\ub97c, \uadf8\ub9ac\uace0 \uac1c\uc778\ud30c\ud2b8\uac00 \uc5c6\uc5c8\ub358 \uae40\uc774\uc9c0\ub294 \uba54\uc778\ubcf4\uceec \ucc28\uc720\ubbf8\uc758 \ud30c\ud2b8\ub97c \ub9bd\uc2f1\ud06c\ud558\uba74\uc11c \uc804\ubcf4\ub2e4\ub294 \uc678\uc801\uc778 \ubd80\ubd84\uc73c\ub85c \uc5b4\ud544\ud55c\ub2e4.", "sentence_answer": "\ud65c\ub3d9\uc5d0 \uce58\uba85\ud0c0\ub97c \uc785\uc740 \ubca0\uc774\ube44\ubcf5\uc2a4\ub294 \uace0\uc815 \ucd9c\uc5f0\uc911\uc774\ub358 SBS <\uae30\uc05c \uc6b0\ub9ac \uc88b\uc740\ub0a0>\uc744 \uc81c\uc678\ud55c \ubaa8\ub4e0 \ubc29\uc1a1 \uc2a4\ucf00\uc904\uc744 \ucde8\uc18c\ud558\uace0 9\uc6d4 \uacbd <\uba38\ub9ac\ud558\ub294 \ub0a0> \ubba4\uc9c1\ube44\ub514\uc624 \ucd2c\uc601 \ud604\uc7a5\uc5d0\uc11c \uce90\uc2a4\ud305\ub418\uc5c8\ub358 \uac04\ubbf8\uc5f0 \uc744 \ub300\ud0c0\uba64\ubc84\ub85c \uc601\uc785\uc2dc\ud0a8\ub4a4 \ub450\ub2ec\uac04\uc758 \uc5f0\uc2b5\uae30\uac04\uc744 \uac70\uccd0 11\uc6d4\ubd80\ud130 \uc5b8\ub860\uc5d0 \uba64\ubc84\uad50\uccb4 \uc18c\uc2dd\uc744 \uc54c\ub9ac\uace0 \ud65c\ub3d9\uc744 \uc7ac\uac1c\ud55c\ub2e4.", "paragraph_id": "6110374-6-0", "paragraph_question": "question: \ubca0\uc774\ube44\ubcf5\uc2a4\uc758 \ub9ac\ub4dc \ubcf4\uceec \ucc28\uc720\ubbf8\uac00 \uac11\uc791\uc2a4\ub7fd\uac8c \ubbf8\uad6d\uc73c\ub85c \ub3cc\uc544\uac00\uac8c \ub418\uba74\uc11c \uce90\uc2a4\ud305\ub41c \ub300\ud0c0\uba64\ubc84\uc758 \uc774\ub984\uc740 \ubb34\uc5c7\uc778\uac00?, context: <\uba38\ub9ac\ud558\ub294 \ub0a0>\ub85c \ud65c\ub3d9\uc744 \uc774\uc5b4\uac00\ub358 10\uc6d4 \uc911\uc21c, \ub9ac\ub4dc \ubcf4\uceec \ucc28\uc720\ubbf8\uac00 \uc5b4\ub290 \ub0a0 \uac11\uc790\uae30 \ubbf8\uad6d\uc73c\ub85c \ub3cc\uc544\uac00\uba74\uc11c \ud300\uc5d0 \uc704\uae30\uac00 \ucc3e\uc544\uc628\ub2e4. \ud65c\ub3d9\uc5d0 \uce58\uba85\ud0c0\ub97c \uc785\uc740 \ubca0\uc774\ube44\ubcf5\uc2a4\ub294 \uace0\uc815 \ucd9c\uc5f0\uc911\uc774\ub358 SBS <\uae30\uc05c \uc6b0\ub9ac \uc88b\uc740\ub0a0>\uc744 \uc81c\uc678\ud55c \ubaa8\ub4e0 \ubc29\uc1a1 \uc2a4\ucf00\uc904\uc744 \ucde8\uc18c\ud558\uace0 9\uc6d4 \uacbd <\uba38\ub9ac\ud558\ub294 \ub0a0> \ubba4\uc9c1\ube44\ub514\uc624 \ucd2c\uc601 \ud604\uc7a5\uc5d0\uc11c \uce90\uc2a4\ud305\ub418\uc5c8\ub358 \uac04\ubbf8\uc5f0\uc744 \ub300\ud0c0\uba64\ubc84\ub85c \uc601\uc785\uc2dc\ud0a8\ub4a4 \ub450\ub2ec\uac04\uc758 \uc5f0\uc2b5\uae30\uac04\uc744 \uac70\uccd0 11\uc6d4\ubd80\ud130 \uc5b8\ub860\uc5d0 \uba64\ubc84\uad50\uccb4 \uc18c\uc2dd\uc744 \uc54c\ub9ac\uace0 \ud65c\ub3d9\uc744 \uc7ac\uac1c\ud55c\ub2e4. \uac04\ubbf8\uc5f0\uc774 \ud569\ub958\ub418\uace0 \uc560\ucd08\uc5d0 \uae30\ud68d\ud558\ub358 \uc5ec\uc131\ub4e4\uc744 \ub300\ubcc0\ud558\ub294 \uc774\ubbf8\uc9c0\uc640\ub294 \uc644\uc804\ud788 \ub2e4\ub978 \ud2f4\ud31d \uc2a4\ud0c0\uc77c\uc758 \uc2a4\ud0a4\ubcf5\uc744 \uc785\uace0 \ub4f1\uc7a5\ud55c\ub2e4. \uadf8\ub9ac\uace0 \uc548\ubb34\uc801\uc778 \uba74\uc5d0\uc11c\ub3c4 \ub9ce\uc740 \ubcc0\ud654\ub97c \uc8fc\uc5c8\ub294\ub370 \uba64\ubc84 \uc774\ud76c\uc9c4\uacfc \uac04\ubbf8\uc5f0\uc740 \ucee4\ud50c \ub304\uc2a4\ub97c, \uadf8\ub9ac\uace0 \uac1c\uc778\ud30c\ud2b8\uac00 \uc5c6\uc5c8\ub358 \uae40\uc774\uc9c0\ub294 \uba54\uc778\ubcf4\uceec \ucc28\uc720\ubbf8\uc758 \ud30c\ud2b8\ub97c \ub9bd\uc2f1\ud06c\ud558\uba74\uc11c \uc804\ubcf4\ub2e4\ub294 \uc678\uc801\uc778 \ubd80\ubd84\uc73c\ub85c \uc5b4\ud544\ud55c\ub2e4."} {"question": "\uc18c\uc124 \uc644\ubcbd\ud55c \ub098\ubb34\uc640 \uc5f0\uae30 \uace0\ub9ac\ub97c \uc4f4 \uc791\uac00\ub294?", "paragraph": "\uac00\uacf5 \uc138\uacc4\ub4e4\uc5d0\uc11c\ub294 \uc885\uc885, \uacfc\ud559\uc801 \ubc0f \uc885\uad50\uc801 \uc758\ubbf8\uc5d0\uc11c\uc758 \uc6b0\uc8fc\ub860\uc774 \ub4f1\uc7a5\ud55c\ub2e4. \uacfc\ud559 \uc18c\uc124 \uc18d \uc138\uacc4\uc758 \uad6c\uc0c1\uc5d0\ub294, \ud2b9\ud788 \uc6b0\uc8fc \ube44\ud589\uc774 \ub4f1\uc7a5\ud558\ub294 \uc0ac\ud68c\uc5d0\uc11c\ub294, \ubcf4\ud1b5 \ud56d\uc131\uacc4\uc640 \ud589\uc131\ub4e4\uc758 \ucc3d\uc870\uac00 \ub4a4\ub530\ub978\ub2e4. \ub514\uc790\uc774\ub108\uac00 \ud604\uc2e4 \uc138\uacc4\uc758 \ucc9c\ubb38 \ubc95\uce59\uc744 \uc801\uc6a9\ud558\uae30\ub97c \uc6d0\ud55c\ub2e4\uba74, \uadf8\ub294 \ucc9c\uccb4\uc758 \uada4\ub3c4 \ubc0f \ubb3c\ub9ac\uc801 \ud2b9\uc131\uc744 \uacb0\uc815\ud558\uae30 \uc704\ud574 \ucc9c\ubb38\ud559\uc801\uc73c\ub85c \uad6c\uccb4\uc801\uc778 \ubc29\ubc95\uc744 \uc4f8 \uc218 \uc788\ub2e4; \uc774\ub294 \ud558\ub8e8\ub098 \uacc4\uc808\uc758 \uae38\uc774 \ub4f1, \uc5f0\ub300\ud559\uc801 \ucc99\ub3c4\ub97c \ub9cc\ub4e0\ub2e4. \uba87\uba87 \ud56d\uc131\uacc4\ub4e4\uc740 \uc758\ub3c4\uc801\uc73c\ub85c \ud2b9\uc774\ud558\uac8c \ub9cc\ub4e4\uc5b4\uc9c4\ub2e4. \ub798\ub9ac \ub2c8\ube10(Larry Niven)\uc758 \uc18c\uc124 \uc644\ubcbd\ud55c \ub098\ubb34(The Integral Trees)\uc640 \uc5f0\uae30 \uace0\ub9ac(The Smoke Ring)\ub97c \uc704\ud574, \ub2c8\ube10\uc740, \uc911\uc131\uc790\ubcc4 \uc8fc\uc704\ub97c \ub458\ub7ec\uc2fc \uac70\uc8fc \uac00\ub2a5\ud55c \uae30\uc555, \uae30\uc628, \uc131\ubd84\uc744 \uac00\uc9c4 \uae30\uccb4 \ud1a0\ub7ec\uc2a4\uc778, \"\uc790\uc720 \ub099\ud558\ud558\ub294\" \ud658\uacbd\uc744 \uad6c\uc0c1\ud588\ub2e4.", "answer": "\ub798\ub9ac \ub2c8\ube10", "sentence": "\ub798\ub9ac \ub2c8\ube10 (Larry Niven)", "paragraph_sentence": "\uac00\uacf5 \uc138\uacc4\ub4e4\uc5d0\uc11c\ub294 \uc885\uc885, \uacfc\ud559\uc801 \ubc0f \uc885\uad50\uc801 \uc758\ubbf8\uc5d0\uc11c\uc758 \uc6b0\uc8fc\ub860\uc774 \ub4f1\uc7a5\ud55c\ub2e4. \uacfc\ud559 \uc18c\uc124 \uc18d \uc138\uacc4\uc758 \uad6c\uc0c1\uc5d0\ub294, \ud2b9\ud788 \uc6b0\uc8fc \ube44\ud589\uc774 \ub4f1\uc7a5\ud558\ub294 \uc0ac\ud68c\uc5d0\uc11c\ub294, \ubcf4\ud1b5 \ud56d\uc131\uacc4\uc640 \ud589\uc131\ub4e4\uc758 \ucc3d\uc870\uac00 \ub4a4\ub530\ub978\ub2e4. \ub514\uc790\uc774\ub108\uac00 \ud604\uc2e4 \uc138\uacc4\uc758 \ucc9c\ubb38 \ubc95\uce59\uc744 \uc801\uc6a9\ud558\uae30\ub97c \uc6d0\ud55c\ub2e4\uba74, \uadf8\ub294 \ucc9c\uccb4\uc758 \uada4\ub3c4 \ubc0f \ubb3c\ub9ac\uc801 \ud2b9\uc131\uc744 \uacb0\uc815\ud558\uae30 \uc704\ud574 \ucc9c\ubb38\ud559\uc801\uc73c\ub85c \uad6c\uccb4\uc801\uc778 \ubc29\ubc95\uc744 \uc4f8 \uc218 \uc788\ub2e4; \uc774\ub294 \ud558\ub8e8\ub098 \uacc4\uc808\uc758 \uae38\uc774 \ub4f1, \uc5f0\ub300\ud559\uc801 \ucc99\ub3c4\ub97c \ub9cc\ub4e0\ub2e4. \uba87\uba87 \ud56d\uc131\uacc4\ub4e4\uc740 \uc758\ub3c4\uc801\uc73c\ub85c \ud2b9\uc774\ud558\uac8c \ub9cc\ub4e4\uc5b4\uc9c4\ub2e4. \ub798\ub9ac \ub2c8\ube10 (Larry Niven) \uc758 \uc18c\uc124 \uc644\ubcbd\ud55c \ub098\ubb34(The Integral Trees)\uc640 \uc5f0\uae30 \uace0\ub9ac(The Smoke Ring)\ub97c \uc704\ud574, \ub2c8\ube10\uc740, \uc911\uc131\uc790\ubcc4 \uc8fc\uc704\ub97c \ub458\ub7ec\uc2fc \uac70\uc8fc \uac00\ub2a5\ud55c \uae30\uc555, \uae30\uc628, \uc131\ubd84\uc744 \uac00\uc9c4 \uae30\uccb4 \ud1a0\ub7ec\uc2a4\uc778, \"\uc790\uc720 \ub099\ud558\ud558\ub294\" \ud658\uacbd\uc744 \uad6c\uc0c1\ud588\ub2e4.", "paragraph_answer": "\uac00\uacf5 \uc138\uacc4\ub4e4\uc5d0\uc11c\ub294 \uc885\uc885, \uacfc\ud559\uc801 \ubc0f \uc885\uad50\uc801 \uc758\ubbf8\uc5d0\uc11c\uc758 \uc6b0\uc8fc\ub860\uc774 \ub4f1\uc7a5\ud55c\ub2e4. \uacfc\ud559 \uc18c\uc124 \uc18d \uc138\uacc4\uc758 \uad6c\uc0c1\uc5d0\ub294, \ud2b9\ud788 \uc6b0\uc8fc \ube44\ud589\uc774 \ub4f1\uc7a5\ud558\ub294 \uc0ac\ud68c\uc5d0\uc11c\ub294, \ubcf4\ud1b5 \ud56d\uc131\uacc4\uc640 \ud589\uc131\ub4e4\uc758 \ucc3d\uc870\uac00 \ub4a4\ub530\ub978\ub2e4. \ub514\uc790\uc774\ub108\uac00 \ud604\uc2e4 \uc138\uacc4\uc758 \ucc9c\ubb38 \ubc95\uce59\uc744 \uc801\uc6a9\ud558\uae30\ub97c \uc6d0\ud55c\ub2e4\uba74, \uadf8\ub294 \ucc9c\uccb4\uc758 \uada4\ub3c4 \ubc0f \ubb3c\ub9ac\uc801 \ud2b9\uc131\uc744 \uacb0\uc815\ud558\uae30 \uc704\ud574 \ucc9c\ubb38\ud559\uc801\uc73c\ub85c \uad6c\uccb4\uc801\uc778 \ubc29\ubc95\uc744 \uc4f8 \uc218 \uc788\ub2e4; \uc774\ub294 \ud558\ub8e8\ub098 \uacc4\uc808\uc758 \uae38\uc774 \ub4f1, \uc5f0\ub300\ud559\uc801 \ucc99\ub3c4\ub97c \ub9cc\ub4e0\ub2e4. \uba87\uba87 \ud56d\uc131\uacc4\ub4e4\uc740 \uc758\ub3c4\uc801\uc73c\ub85c \ud2b9\uc774\ud558\uac8c \ub9cc\ub4e4\uc5b4\uc9c4\ub2e4. \ub798\ub9ac \ub2c8\ube10 (Larry Niven)\uc758 \uc18c\uc124 \uc644\ubcbd\ud55c \ub098\ubb34(The Integral Trees)\uc640 \uc5f0\uae30 \uace0\ub9ac(The Smoke Ring)\ub97c \uc704\ud574, \ub2c8\ube10\uc740, \uc911\uc131\uc790\ubcc4 \uc8fc\uc704\ub97c \ub458\ub7ec\uc2fc \uac70\uc8fc \uac00\ub2a5\ud55c \uae30\uc555, \uae30\uc628, \uc131\ubd84\uc744 \uac00\uc9c4 \uae30\uccb4 \ud1a0\ub7ec\uc2a4\uc778, \"\uc790\uc720 \ub099\ud558\ud558\ub294\" \ud658\uacbd\uc744 \uad6c\uc0c1\ud588\ub2e4.", "sentence_answer": " \ub798\ub9ac \ub2c8\ube10 (Larry Niven)", "paragraph_id": "6536338-4-1", "paragraph_question": "question: \uc18c\uc124 \uc644\ubcbd\ud55c \ub098\ubb34\uc640 \uc5f0\uae30 \uace0\ub9ac\ub97c \uc4f4 \uc791\uac00\ub294?, context: \uac00\uacf5 \uc138\uacc4\ub4e4\uc5d0\uc11c\ub294 \uc885\uc885, \uacfc\ud559\uc801 \ubc0f \uc885\uad50\uc801 \uc758\ubbf8\uc5d0\uc11c\uc758 \uc6b0\uc8fc\ub860\uc774 \ub4f1\uc7a5\ud55c\ub2e4. \uacfc\ud559 \uc18c\uc124 \uc18d \uc138\uacc4\uc758 \uad6c\uc0c1\uc5d0\ub294, \ud2b9\ud788 \uc6b0\uc8fc \ube44\ud589\uc774 \ub4f1\uc7a5\ud558\ub294 \uc0ac\ud68c\uc5d0\uc11c\ub294, \ubcf4\ud1b5 \ud56d\uc131\uacc4\uc640 \ud589\uc131\ub4e4\uc758 \ucc3d\uc870\uac00 \ub4a4\ub530\ub978\ub2e4. \ub514\uc790\uc774\ub108\uac00 \ud604\uc2e4 \uc138\uacc4\uc758 \ucc9c\ubb38 \ubc95\uce59\uc744 \uc801\uc6a9\ud558\uae30\ub97c \uc6d0\ud55c\ub2e4\uba74, \uadf8\ub294 \ucc9c\uccb4\uc758 \uada4\ub3c4 \ubc0f \ubb3c\ub9ac\uc801 \ud2b9\uc131\uc744 \uacb0\uc815\ud558\uae30 \uc704\ud574 \ucc9c\ubb38\ud559\uc801\uc73c\ub85c \uad6c\uccb4\uc801\uc778 \ubc29\ubc95\uc744 \uc4f8 \uc218 \uc788\ub2e4; \uc774\ub294 \ud558\ub8e8\ub098 \uacc4\uc808\uc758 \uae38\uc774 \ub4f1, \uc5f0\ub300\ud559\uc801 \ucc99\ub3c4\ub97c \ub9cc\ub4e0\ub2e4. \uba87\uba87 \ud56d\uc131\uacc4\ub4e4\uc740 \uc758\ub3c4\uc801\uc73c\ub85c \ud2b9\uc774\ud558\uac8c \ub9cc\ub4e4\uc5b4\uc9c4\ub2e4. \ub798\ub9ac \ub2c8\ube10(Larry Niven)\uc758 \uc18c\uc124 \uc644\ubcbd\ud55c \ub098\ubb34(The Integral Trees)\uc640 \uc5f0\uae30 \uace0\ub9ac(The Smoke Ring)\ub97c \uc704\ud574, \ub2c8\ube10\uc740, \uc911\uc131\uc790\ubcc4 \uc8fc\uc704\ub97c \ub458\ub7ec\uc2fc \uac70\uc8fc \uac00\ub2a5\ud55c \uae30\uc555, \uae30\uc628, \uc131\ubd84\uc744 \uac00\uc9c4 \uae30\uccb4 \ud1a0\ub7ec\uc2a4\uc778, \"\uc790\uc720 \ub099\ud558\ud558\ub294\" \ud658\uacbd\uc744 \uad6c\uc0c1\ud588\ub2e4."} {"question": "7\uc6d4 13\uc77c \ud788\ud2c0\ub7ec\uac00 \ucfe0\ub974\uc2a4\ud06c\uc5d0\uc11c \ucca0\uc218 \uba85\ub839\uc744 \ud558\uac8c\ub41c \uc774\uc720\ub294 \uc5f0\ud569\uad70\uc774 \uc5b4\ub290 \uc9c0\uc5ed\uc744 \uce68\uacf5\ud558\uc600\uae30 \ub54c\ubb38\uc774\uc5c8\ub098?", "paragraph": "\uc131\ucc44 \uc791\uc804\uc740 \ub3d9\ubd80\uc804\uc120 \ub3c5\uc77c \ucd5c\ud6c4\uc758 \uc804\ub7b5\uc801 \uacf5\uc138\uc791\uc804\uc774\uba70, 4\ubc31\ub9cc \uba85 \uc774\uc0c1\uc758 \uad70\uc778\uc774 \ub3d9\uc6d0\ub41c \uc5ed\uc0ac\uc0c1 \uac00\uc7a5 \uac70\ub300\ud55c \uaddc\ubaa8\uc758 \uc804\ud22c \uc911 \ud558\ub098\uc600\ub2e4. \ub3c5\uc77c\uad70\uc774 1943\ub144 7\uc6d4 5\uc77c \ucd5c\ucd08 \uacf5\uaca9\uc744 \uac1c\uc2dc\ud560 \ub54c\ucbe4\uc5d0\ub294 \uc774\ubbf8 \uc18c\ub828\uad70\uc774 \ub3c5\uc77c\uad70\uc744 \uc218\uc801\uc73c\ub85c \uac70\uc758 \uc138 \ubc30 \uac00\ub7c9 \uc555\ub3c4\ud558\uace0 \uc788\uc5c8\ub2e4. \ubc1c\ud130 \ubaa8\ub378\uc758 \uc81c9\uad70\uc774 \ubd81\ucabd\uc5d0\uc11c, \ub9cc\uc288\ud0c0\uc778\uc758 \ub0a8\ubd80 \uc9d1\ub2e8\uad70\uc774 \ub0a8\ucabd\uc5d0\uc11c \ud611\uaca9\ud558\uc5ec \ud611\uacf5\uc744 \uc9c4\ud589\ud588\ub2e4. \uc81c9\uad70\uacfc \ub0a8\ubd80 \uc9d1\ub2e8\uad70 \ubaa8\ub450 \uc804\ucc28\uac00 \uc9c0\ub8b0\ubc2d\uc5d0 \ub0a0\uc544\uac04\ub2e4\uac70\ub098 \uc18c\ub828\uad70\uc758 \uc900\ube44\ub41c \ubc29\uc5b4\uc120\uc5d0 \ub9c9\ud788\ub294 \ub4f1 \uc774\uc720\ub85c \uc778\ud574 \uc9c4\uaca9\uc740 \ub354\ub3a0\ub2e4. 5\uc77c \ubc24\ub0ae\uc744 \uc2f8\uc6b4 \ub4a4 \ubaa8\ub378\uc758 \uc9c4\uaca9\uc774 \uba48\ucd94\uc5c8\ub2e4. \uc774 \uc2dc\uc810\uc5d0\uc11c \uc81c9\uad70\uc758 \uc0ac\uc0c1\uc790\ub294 25,000\uc5ec\uba85\uc5d0 \ub2ec\ud588\ub2e4. 7\uc6d4 13\uc77c \ubaa8\ub378\uc758 \ubd80\ub300\ub294 \uc624\ub810\uc744 \ud5a5\ud574 \ud6c4\ud1f4\ud588\uace0, \uc18c\ub828\uad70\uc740 \ucfe0\ud22c\uc870\ud504 \uc791\uc804\uc744 \uac1c\uc2dc\ud588\ub2e4. \ub9cc\uc288\ud0c0\uc778\uc758 \ubd80\ub300\ub294 \uc18c\ub828\uad70 \uc804\uc5f4\uc744 \ub118\uc5b4\uac08 \uc218 \uc788\uc5c8\uc9c0\ub9cc \uc5ed\uc2dc \uc2ec\uac01\ud55c \uc0ac\uc0c1\uc790\uac00 \ub204\uc801\ub418\uc5c8\ub2e4. \ub9cc\uc288\ud0c0\uc778\uc740 \uc8fc\uc694 \ubaa9\ud45c\ubb3c \uc911 \ud558\ub098\uc778 \ud504\ub85c\ucf54\ub85c\ube0c\uce74\uc5d0 \ub2ff\uae30\ub9cc \ud558\uace0 \uc9c4\uc785\uc774\ub098 \uc810\ub839\uc740 \ud558\uc9c0\ub3c4 \ubabb\ud588\ub2e4. 7\uc6d4 11\uc77c, \ud504\ub85c\ucf54\ub85c\ube0c\uce74 \uc804\ud22c\uc5d0\uc11c \ub3c5\uc77c\uad70\uc740 \uc18c\ub828\uad70\uc5d0\uac8c \ud070 \ud53c\ud574\ub97c \uc785\ud614\ub2e4. \uadf8\ub7ec\ub098 7\uc6d4 13\uc77c \ud788\ud2c0\ub7ec\uac00 \ucfe0\ub974\uc2a4\ud06c \uacf5\uc138\ub294 \uc2e4\ud328\ud588\uc73c\ub2c8 \ucca0\uc218\ud560 \uac83\uc744 \uba85\ub839\ud588\ub2e4. \uc5f0\ud569\uad70\uc774 \uc2dc\uce60\ub9ac\uc544\ub97c \uce68\uacf5\ud55c \uac83\uc774\ub2e4. \ub9cc\uc288\ud0c0\uc778\uc740 \ud56d\uc758\ud588\ub2e4. \ub9cc\uc288\ud0c0\uc778\uc774 \ubcf4\uae30\uc5d0\ub294 \ucfe0\ub974\uc2a4\ud06c\uc758 \uc18c\ub828\uad70\uc740 \ubaa8\ub4e0 \uc608\ube44\ub2a5\ub825\uc744 \uc18c\uc9c4\ud55c \uc0c1\ud0dc\uc600\ub2e4. \uadf8\ub7ec\ub098 \ud788\ud2c0\ub7ec\ub294 \uc791\uc804 \ud3ec\uae30\ub97c \uace0\uc9d1\ud588\ub2e4. \ud604\ub300 \uc5ed\uc0ac\ud559\uc790\ub4e4\uc740 \uc18c\ub828\uad70\uc758 \ud53c\ud574\uac00 \uadf9\uc2ec\ud558\uae30\ub294 \ud588\uc73c\ub098 \ub3c5\uc77c\uad70\uc774 \uacf5\uc138\ub97c \uc131\uacf5\uc801\uc73c\ub85c \uacc4\uc18d\ud588\uc744 \uac00\ub2a5\uc131\uc740 \ub0ae\ub2e4\uace0 \ubcf4\uace0 \uc788\ub2e4.", "answer": "\uc2dc\uce60\ub9ac\uc544", "sentence": "\uc5f0\ud569\uad70\uc774 \uc2dc\uce60\ub9ac\uc544 \ub97c \uce68\uacf5\ud55c \uac83\uc774\ub2e4.", "paragraph_sentence": "\uc131\ucc44 \uc791\uc804\uc740 \ub3d9\ubd80\uc804\uc120 \ub3c5\uc77c \ucd5c\ud6c4\uc758 \uc804\ub7b5\uc801 \uacf5\uc138\uc791\uc804\uc774\uba70, 4\ubc31\ub9cc \uba85 \uc774\uc0c1\uc758 \uad70\uc778\uc774 \ub3d9\uc6d0\ub41c \uc5ed\uc0ac\uc0c1 \uac00\uc7a5 \uac70\ub300\ud55c \uaddc\ubaa8\uc758 \uc804\ud22c \uc911 \ud558\ub098\uc600\ub2e4. \ub3c5\uc77c\uad70\uc774 1943\ub144 7\uc6d4 5\uc77c \ucd5c\ucd08 \uacf5\uaca9\uc744 \uac1c\uc2dc\ud560 \ub54c\ucbe4\uc5d0\ub294 \uc774\ubbf8 \uc18c\ub828\uad70\uc774 \ub3c5\uc77c\uad70\uc744 \uc218\uc801\uc73c\ub85c \uac70\uc758 \uc138 \ubc30 \uac00\ub7c9 \uc555\ub3c4\ud558\uace0 \uc788\uc5c8\ub2e4. \ubc1c\ud130 \ubaa8\ub378\uc758 \uc81c9\uad70\uc774 \ubd81\ucabd\uc5d0\uc11c, \ub9cc\uc288\ud0c0\uc778\uc758 \ub0a8\ubd80 \uc9d1\ub2e8\uad70\uc774 \ub0a8\ucabd\uc5d0\uc11c \ud611\uaca9\ud558\uc5ec \ud611\uacf5\uc744 \uc9c4\ud589\ud588\ub2e4. \uc81c9\uad70\uacfc \ub0a8\ubd80 \uc9d1\ub2e8\uad70 \ubaa8\ub450 \uc804\ucc28\uac00 \uc9c0\ub8b0\ubc2d\uc5d0 \ub0a0\uc544\uac04\ub2e4\uac70\ub098 \uc18c\ub828\uad70\uc758 \uc900\ube44\ub41c \ubc29\uc5b4\uc120\uc5d0 \ub9c9\ud788\ub294 \ub4f1 \uc774\uc720\ub85c \uc778\ud574 \uc9c4\uaca9\uc740 \ub354\ub3a0\ub2e4. 5\uc77c \ubc24\ub0ae\uc744 \uc2f8\uc6b4 \ub4a4 \ubaa8\ub378\uc758 \uc9c4\uaca9\uc774 \uba48\ucd94\uc5c8\ub2e4. \uc774 \uc2dc\uc810\uc5d0\uc11c \uc81c9\uad70\uc758 \uc0ac\uc0c1\uc790\ub294 25,000\uc5ec\uba85\uc5d0 \ub2ec\ud588\ub2e4. 7\uc6d4 13\uc77c \ubaa8\ub378\uc758 \ubd80\ub300\ub294 \uc624\ub810\uc744 \ud5a5\ud574 \ud6c4\ud1f4\ud588\uace0, \uc18c\ub828\uad70\uc740 \ucfe0\ud22c\uc870\ud504 \uc791\uc804\uc744 \uac1c\uc2dc\ud588\ub2e4. \ub9cc\uc288\ud0c0\uc778\uc758 \ubd80\ub300\ub294 \uc18c\ub828\uad70 \uc804\uc5f4\uc744 \ub118\uc5b4\uac08 \uc218 \uc788\uc5c8\uc9c0\ub9cc \uc5ed\uc2dc \uc2ec\uac01\ud55c \uc0ac\uc0c1\uc790\uac00 \ub204\uc801\ub418\uc5c8\ub2e4. \ub9cc\uc288\ud0c0\uc778\uc740 \uc8fc\uc694 \ubaa9\ud45c\ubb3c \uc911 \ud558\ub098\uc778 \ud504\ub85c\ucf54\ub85c\ube0c\uce74\uc5d0 \ub2ff\uae30\ub9cc \ud558\uace0 \uc9c4\uc785\uc774\ub098 \uc810\ub839\uc740 \ud558\uc9c0\ub3c4 \ubabb\ud588\ub2e4. 7\uc6d4 11\uc77c, \ud504\ub85c\ucf54\ub85c\ube0c\uce74 \uc804\ud22c\uc5d0\uc11c \ub3c5\uc77c\uad70\uc740 \uc18c\ub828\uad70\uc5d0\uac8c \ud070 \ud53c\ud574\ub97c \uc785\ud614\ub2e4. \uadf8\ub7ec\ub098 7\uc6d4 13\uc77c \ud788\ud2c0\ub7ec\uac00 \ucfe0\ub974\uc2a4\ud06c \uacf5\uc138\ub294 \uc2e4\ud328\ud588\uc73c\ub2c8 \ucca0\uc218\ud560 \uac83\uc744 \uba85\ub839\ud588\ub2e4. \uc5f0\ud569\uad70\uc774 \uc2dc\uce60\ub9ac\uc544 \ub97c \uce68\uacf5\ud55c \uac83\uc774\ub2e4. \ub9cc\uc288\ud0c0\uc778\uc740 \ud56d\uc758\ud588\ub2e4. \ub9cc\uc288\ud0c0\uc778\uc774 \ubcf4\uae30\uc5d0\ub294 \ucfe0\ub974\uc2a4\ud06c\uc758 \uc18c\ub828\uad70\uc740 \ubaa8\ub4e0 \uc608\ube44\ub2a5\ub825\uc744 \uc18c\uc9c4\ud55c \uc0c1\ud0dc\uc600\ub2e4. \uadf8\ub7ec\ub098 \ud788\ud2c0\ub7ec\ub294 \uc791\uc804 \ud3ec\uae30\ub97c \uace0\uc9d1\ud588\ub2e4. \ud604\ub300 \uc5ed\uc0ac\ud559\uc790\ub4e4\uc740 \uc18c\ub828\uad70\uc758 \ud53c\ud574\uac00 \uadf9\uc2ec\ud558\uae30\ub294 \ud588\uc73c\ub098 \ub3c5\uc77c\uad70\uc774 \uacf5\uc138\ub97c \uc131\uacf5\uc801\uc73c\ub85c \uacc4\uc18d\ud588\uc744 \uac00\ub2a5\uc131\uc740 \ub0ae\ub2e4\uace0 \ubcf4\uace0 \uc788\ub2e4.", "paragraph_answer": "\uc131\ucc44 \uc791\uc804\uc740 \ub3d9\ubd80\uc804\uc120 \ub3c5\uc77c \ucd5c\ud6c4\uc758 \uc804\ub7b5\uc801 \uacf5\uc138\uc791\uc804\uc774\uba70, 4\ubc31\ub9cc \uba85 \uc774\uc0c1\uc758 \uad70\uc778\uc774 \ub3d9\uc6d0\ub41c \uc5ed\uc0ac\uc0c1 \uac00\uc7a5 \uac70\ub300\ud55c \uaddc\ubaa8\uc758 \uc804\ud22c \uc911 \ud558\ub098\uc600\ub2e4. \ub3c5\uc77c\uad70\uc774 1943\ub144 7\uc6d4 5\uc77c \ucd5c\ucd08 \uacf5\uaca9\uc744 \uac1c\uc2dc\ud560 \ub54c\ucbe4\uc5d0\ub294 \uc774\ubbf8 \uc18c\ub828\uad70\uc774 \ub3c5\uc77c\uad70\uc744 \uc218\uc801\uc73c\ub85c \uac70\uc758 \uc138 \ubc30 \uac00\ub7c9 \uc555\ub3c4\ud558\uace0 \uc788\uc5c8\ub2e4. \ubc1c\ud130 \ubaa8\ub378\uc758 \uc81c9\uad70\uc774 \ubd81\ucabd\uc5d0\uc11c, \ub9cc\uc288\ud0c0\uc778\uc758 \ub0a8\ubd80 \uc9d1\ub2e8\uad70\uc774 \ub0a8\ucabd\uc5d0\uc11c \ud611\uaca9\ud558\uc5ec \ud611\uacf5\uc744 \uc9c4\ud589\ud588\ub2e4. \uc81c9\uad70\uacfc \ub0a8\ubd80 \uc9d1\ub2e8\uad70 \ubaa8\ub450 \uc804\ucc28\uac00 \uc9c0\ub8b0\ubc2d\uc5d0 \ub0a0\uc544\uac04\ub2e4\uac70\ub098 \uc18c\ub828\uad70\uc758 \uc900\ube44\ub41c \ubc29\uc5b4\uc120\uc5d0 \ub9c9\ud788\ub294 \ub4f1 \uc774\uc720\ub85c \uc778\ud574 \uc9c4\uaca9\uc740 \ub354\ub3a0\ub2e4. 5\uc77c \ubc24\ub0ae\uc744 \uc2f8\uc6b4 \ub4a4 \ubaa8\ub378\uc758 \uc9c4\uaca9\uc774 \uba48\ucd94\uc5c8\ub2e4. \uc774 \uc2dc\uc810\uc5d0\uc11c \uc81c9\uad70\uc758 \uc0ac\uc0c1\uc790\ub294 25,000\uc5ec\uba85\uc5d0 \ub2ec\ud588\ub2e4. 7\uc6d4 13\uc77c \ubaa8\ub378\uc758 \ubd80\ub300\ub294 \uc624\ub810\uc744 \ud5a5\ud574 \ud6c4\ud1f4\ud588\uace0, \uc18c\ub828\uad70\uc740 \ucfe0\ud22c\uc870\ud504 \uc791\uc804\uc744 \uac1c\uc2dc\ud588\ub2e4. \ub9cc\uc288\ud0c0\uc778\uc758 \ubd80\ub300\ub294 \uc18c\ub828\uad70 \uc804\uc5f4\uc744 \ub118\uc5b4\uac08 \uc218 \uc788\uc5c8\uc9c0\ub9cc \uc5ed\uc2dc \uc2ec\uac01\ud55c \uc0ac\uc0c1\uc790\uac00 \ub204\uc801\ub418\uc5c8\ub2e4. \ub9cc\uc288\ud0c0\uc778\uc740 \uc8fc\uc694 \ubaa9\ud45c\ubb3c \uc911 \ud558\ub098\uc778 \ud504\ub85c\ucf54\ub85c\ube0c\uce74\uc5d0 \ub2ff\uae30\ub9cc \ud558\uace0 \uc9c4\uc785\uc774\ub098 \uc810\ub839\uc740 \ud558\uc9c0\ub3c4 \ubabb\ud588\ub2e4. 7\uc6d4 11\uc77c, \ud504\ub85c\ucf54\ub85c\ube0c\uce74 \uc804\ud22c\uc5d0\uc11c \ub3c5\uc77c\uad70\uc740 \uc18c\ub828\uad70\uc5d0\uac8c \ud070 \ud53c\ud574\ub97c \uc785\ud614\ub2e4. \uadf8\ub7ec\ub098 7\uc6d4 13\uc77c \ud788\ud2c0\ub7ec\uac00 \ucfe0\ub974\uc2a4\ud06c \uacf5\uc138\ub294 \uc2e4\ud328\ud588\uc73c\ub2c8 \ucca0\uc218\ud560 \uac83\uc744 \uba85\ub839\ud588\ub2e4. \uc5f0\ud569\uad70\uc774 \uc2dc\uce60\ub9ac\uc544 \ub97c \uce68\uacf5\ud55c \uac83\uc774\ub2e4. \ub9cc\uc288\ud0c0\uc778\uc740 \ud56d\uc758\ud588\ub2e4. \ub9cc\uc288\ud0c0\uc778\uc774 \ubcf4\uae30\uc5d0\ub294 \ucfe0\ub974\uc2a4\ud06c\uc758 \uc18c\ub828\uad70\uc740 \ubaa8\ub4e0 \uc608\ube44\ub2a5\ub825\uc744 \uc18c\uc9c4\ud55c \uc0c1\ud0dc\uc600\ub2e4. \uadf8\ub7ec\ub098 \ud788\ud2c0\ub7ec\ub294 \uc791\uc804 \ud3ec\uae30\ub97c \uace0\uc9d1\ud588\ub2e4. \ud604\ub300 \uc5ed\uc0ac\ud559\uc790\ub4e4\uc740 \uc18c\ub828\uad70\uc758 \ud53c\ud574\uac00 \uadf9\uc2ec\ud558\uae30\ub294 \ud588\uc73c\ub098 \ub3c5\uc77c\uad70\uc774 \uacf5\uc138\ub97c \uc131\uacf5\uc801\uc73c\ub85c \uacc4\uc18d\ud588\uc744 \uac00\ub2a5\uc131\uc740 \ub0ae\ub2e4\uace0 \ubcf4\uace0 \uc788\ub2e4.", "sentence_answer": "\uc5f0\ud569\uad70\uc774 \uc2dc\uce60\ub9ac\uc544 \ub97c \uce68\uacf5\ud55c \uac83\uc774\ub2e4.", "paragraph_id": "6540675-25-2", "paragraph_question": "question: 7\uc6d4 13\uc77c \ud788\ud2c0\ub7ec\uac00 \ucfe0\ub974\uc2a4\ud06c\uc5d0\uc11c \ucca0\uc218 \uba85\ub839\uc744 \ud558\uac8c\ub41c \uc774\uc720\ub294 \uc5f0\ud569\uad70\uc774 \uc5b4\ub290 \uc9c0\uc5ed\uc744 \uce68\uacf5\ud558\uc600\uae30 \ub54c\ubb38\uc774\uc5c8\ub098?, context: \uc131\ucc44 \uc791\uc804\uc740 \ub3d9\ubd80\uc804\uc120 \ub3c5\uc77c \ucd5c\ud6c4\uc758 \uc804\ub7b5\uc801 \uacf5\uc138\uc791\uc804\uc774\uba70, 4\ubc31\ub9cc \uba85 \uc774\uc0c1\uc758 \uad70\uc778\uc774 \ub3d9\uc6d0\ub41c \uc5ed\uc0ac\uc0c1 \uac00\uc7a5 \uac70\ub300\ud55c \uaddc\ubaa8\uc758 \uc804\ud22c \uc911 \ud558\ub098\uc600\ub2e4. \ub3c5\uc77c\uad70\uc774 1943\ub144 7\uc6d4 5\uc77c \ucd5c\ucd08 \uacf5\uaca9\uc744 \uac1c\uc2dc\ud560 \ub54c\ucbe4\uc5d0\ub294 \uc774\ubbf8 \uc18c\ub828\uad70\uc774 \ub3c5\uc77c\uad70\uc744 \uc218\uc801\uc73c\ub85c \uac70\uc758 \uc138 \ubc30 \uac00\ub7c9 \uc555\ub3c4\ud558\uace0 \uc788\uc5c8\ub2e4. \ubc1c\ud130 \ubaa8\ub378\uc758 \uc81c9\uad70\uc774 \ubd81\ucabd\uc5d0\uc11c, \ub9cc\uc288\ud0c0\uc778\uc758 \ub0a8\ubd80 \uc9d1\ub2e8\uad70\uc774 \ub0a8\ucabd\uc5d0\uc11c \ud611\uaca9\ud558\uc5ec \ud611\uacf5\uc744 \uc9c4\ud589\ud588\ub2e4. \uc81c9\uad70\uacfc \ub0a8\ubd80 \uc9d1\ub2e8\uad70 \ubaa8\ub450 \uc804\ucc28\uac00 \uc9c0\ub8b0\ubc2d\uc5d0 \ub0a0\uc544\uac04\ub2e4\uac70\ub098 \uc18c\ub828\uad70\uc758 \uc900\ube44\ub41c \ubc29\uc5b4\uc120\uc5d0 \ub9c9\ud788\ub294 \ub4f1 \uc774\uc720\ub85c \uc778\ud574 \uc9c4\uaca9\uc740 \ub354\ub3a0\ub2e4. 5\uc77c \ubc24\ub0ae\uc744 \uc2f8\uc6b4 \ub4a4 \ubaa8\ub378\uc758 \uc9c4\uaca9\uc774 \uba48\ucd94\uc5c8\ub2e4. \uc774 \uc2dc\uc810\uc5d0\uc11c \uc81c9\uad70\uc758 \uc0ac\uc0c1\uc790\ub294 25,000\uc5ec\uba85\uc5d0 \ub2ec\ud588\ub2e4. 7\uc6d4 13\uc77c \ubaa8\ub378\uc758 \ubd80\ub300\ub294 \uc624\ub810\uc744 \ud5a5\ud574 \ud6c4\ud1f4\ud588\uace0, \uc18c\ub828\uad70\uc740 \ucfe0\ud22c\uc870\ud504 \uc791\uc804\uc744 \uac1c\uc2dc\ud588\ub2e4. \ub9cc\uc288\ud0c0\uc778\uc758 \ubd80\ub300\ub294 \uc18c\ub828\uad70 \uc804\uc5f4\uc744 \ub118\uc5b4\uac08 \uc218 \uc788\uc5c8\uc9c0\ub9cc \uc5ed\uc2dc \uc2ec\uac01\ud55c \uc0ac\uc0c1\uc790\uac00 \ub204\uc801\ub418\uc5c8\ub2e4. \ub9cc\uc288\ud0c0\uc778\uc740 \uc8fc\uc694 \ubaa9\ud45c\ubb3c \uc911 \ud558\ub098\uc778 \ud504\ub85c\ucf54\ub85c\ube0c\uce74\uc5d0 \ub2ff\uae30\ub9cc \ud558\uace0 \uc9c4\uc785\uc774\ub098 \uc810\ub839\uc740 \ud558\uc9c0\ub3c4 \ubabb\ud588\ub2e4. 7\uc6d4 11\uc77c, \ud504\ub85c\ucf54\ub85c\ube0c\uce74 \uc804\ud22c\uc5d0\uc11c \ub3c5\uc77c\uad70\uc740 \uc18c\ub828\uad70\uc5d0\uac8c \ud070 \ud53c\ud574\ub97c \uc785\ud614\ub2e4. \uadf8\ub7ec\ub098 7\uc6d4 13\uc77c \ud788\ud2c0\ub7ec\uac00 \ucfe0\ub974\uc2a4\ud06c \uacf5\uc138\ub294 \uc2e4\ud328\ud588\uc73c\ub2c8 \ucca0\uc218\ud560 \uac83\uc744 \uba85\ub839\ud588\ub2e4. \uc5f0\ud569\uad70\uc774 \uc2dc\uce60\ub9ac\uc544\ub97c \uce68\uacf5\ud55c \uac83\uc774\ub2e4. \ub9cc\uc288\ud0c0\uc778\uc740 \ud56d\uc758\ud588\ub2e4. \ub9cc\uc288\ud0c0\uc778\uc774 \ubcf4\uae30\uc5d0\ub294 \ucfe0\ub974\uc2a4\ud06c\uc758 \uc18c\ub828\uad70\uc740 \ubaa8\ub4e0 \uc608\ube44\ub2a5\ub825\uc744 \uc18c\uc9c4\ud55c \uc0c1\ud0dc\uc600\ub2e4. \uadf8\ub7ec\ub098 \ud788\ud2c0\ub7ec\ub294 \uc791\uc804 \ud3ec\uae30\ub97c \uace0\uc9d1\ud588\ub2e4. \ud604\ub300 \uc5ed\uc0ac\ud559\uc790\ub4e4\uc740 \uc18c\ub828\uad70\uc758 \ud53c\ud574\uac00 \uadf9\uc2ec\ud558\uae30\ub294 \ud588\uc73c\ub098 \ub3c5\uc77c\uad70\uc774 \uacf5\uc138\ub97c \uc131\uacf5\uc801\uc73c\ub85c \uacc4\uc18d\ud588\uc744 \uac00\ub2a5\uc131\uc740 \ub0ae\ub2e4\uace0 \ubcf4\uace0 \uc788\ub2e4."} {"question": "\ub3c5\ub9bd\ub41c \uc704\uc6d0\ud68c\uc778 \uad6d\uac00\uc778\uad8c\uc704\uc6d0\ud68c\ubc95\uc774 \uc81c\uc815\ub41c \ub0a0\uc740?", "paragraph": "\uad6d\ub0b4\uc5d0\uc11c\ub294 \uc0ac\ud68c \uac01\uacc4\uac01\uce35\uc758 \uc774\ud574\uad00\uacc4\uc5d0 \uc758\ud574 \uc124\uce58 \uacfc\uc815\uc5d0\uc11c\ubd80\ud130 \uc0ac\ubb34\ucc98 \uc124\uce58\uae4c\uc9c0\uc758 \uacfc\uc815\uc774 \uc21c\ud0c4\uce58 \uc54a\uc558\ub2e4. \ub300\ud55c\ubbfc\uad6d\uc5d0\uc11c \uad6d\uac00\uc778\uad8c\uae30\uad6c \uc124\uce58\uc5d0 \uad00\ud55c \ub17c\uc758\ub294 1993\ub144 6\uc6d4 10\uc77c \ube44\uc5d4\ub098\uc5d0\uc11c \uc5f4\ub9b0 \uc138\uacc4\uc778\uad8c\ub300\ud68c\uc5d0 \ubbfc\uac04\ub2e8\uccb4 \uacf5\ub3d9\ub300\ucc45\uc704\uc6d0\ud68c\uac00 \ucc38\uac00\ud558\uc5ec \ub300\ud55c\ubbfc\uad6d \uc815\ubd80\uc5d0 \uad6d\uac00\uc778\uad8c\uae30\uad6c \uc124\uce58\ub97c \uc694\uccad\ud55c \uac83\uc744 \uacc4\uae30\ub85c \uc2dc\uc791\ub418\uc5c8\ub2e4. \uc774\ud6c4 \ubbfc\uac04\ub2e8\uccb4\ub4e4\uc740 \uad6d\ub0b4\uc678 \uc6cc\ud06c\uc20d\uacfc \uc778\uad8c\ub300\ud68c \ub4f1\uc5d0\uc11c \uc778\uad8c\uae30\uad6c \uc124\ub9bd\uc744 \uafb8\uc900\ud788 \uc694\uad6c\ud558\uc600\ub2e4. \uad6d\ub0b4\uc678\uc5d0\uc11c \uc774\ub7ec\ud55c \ub17c\uc758 \uacfc\uc815\uc744 \uac70\uce58\ub294 \uac00\uc6b4\ub370 1997\ub144 11\uc6d4 \uae40\ub300\uc911 \ub300\ud1b5\ub839 \ud6c4\ubcf4\uac00 \uc778\uad8c\ubc95 \uc81c\uc815 \ubc0f \uad6d\ubbfc\uc778\uad8c\uc704\uc6d0\ud68c \uc124\ub9bd\uc744 \ub300\uc120 \uacf5\uc57d\uc73c\ub85c \ubc1c\ud45c\ud588\ub2e4. \uadf8 \ud6c4 1998\ub144 \uae40\ub300\uc911 \uc815\ubd80\uc5d0\uc11c \uad6d\uac00\uc778\uad8c\uc704\uc6d0\ud68c \uc124\uce58\ub97c \uace8\uc790\ub85c \ud55c \uacc4\ud68d\uc744 \ubc1c\ud45c\ud558\uc600\uace0 \ubc95\ubb34\ubd80\uac00 \uad00\ub828 \ubc95\uc548\uc744 \uc81c\ucd9c\ud558\uc600\ub2e4. \ud558\uc9c0\ub9cc \uad6d\uc81c\uc0ac\uba74\uc704\uc6d0\ud68c\uc640 \uac19\uc740 \uc2dc\ubbfc\ub2e8\uccb4\uc5d0\uc11c \ubc95\ubb34\ubd80 \uc18c\uad00 \uae30\uad6c \uc124\uce58 \ubc18\ub300\ub97c \uc9c0\uc18d\uc801\uc73c\ub85c \uc8fc\uc7a5\ud558\uc5ec 2001\ub144 5\uc6d4 24\uc77c \ub3c5\ub9bd\ub41c \uc704\uc6d0\ud68c\ub85c\uc11c\uc758 \uc9c0\uc704\ub97c \uac00\uc9c0\ub294 \uad6d\uac00\uc778\uad8c\uc704\uc6d0\ud68c\ubc95\uc774 \uc81c\uc815\ub418\uc5c8\uace0 \uadf8\ud574 11\uc6d4 25\uc77c \uc815\uc2dd \ucd9c\ubc94\ud558\uc600\ub2e4. \ub2e4\ub9cc \uc0ac\ubb34\ucc98 \uc815\uc6d0\uacfc \uc608\uc0b0\uc5d0 \ub530\ub978 \ud589\uc815\uc790\uce58\ubd80\uc640\uc758 \uac08\ub4f1\uc774 \uc788\uc5b4 \uc774\ub97c \uc870\uc728\ud574 \ub098\uac00\ub294 \uacfc\uc815\uc774 \uae38\uc5b4\uc838 \uacb0\uad6d \uad6d\uac00\uc778\uad8c\uc704\uc6d0\ud68c \uc0ac\ubb34\ucc98\ub294 2002\ub144 4\uc6d4 1\uc77c \ubc1c\uc871\ud558\uc600\ub2e4.", "answer": "2001\ub144 5\uc6d4 24\uc77c", "sentence": "\ud558\uc9c0\ub9cc \uad6d\uc81c\uc0ac\uba74\uc704\uc6d0\ud68c\uc640 \uac19\uc740 \uc2dc\ubbfc\ub2e8\uccb4\uc5d0\uc11c \ubc95\ubb34\ubd80 \uc18c\uad00 \uae30\uad6c \uc124\uce58 \ubc18\ub300\ub97c \uc9c0\uc18d\uc801\uc73c\ub85c \uc8fc\uc7a5\ud558\uc5ec 2001\ub144 5\uc6d4 24\uc77c \ub3c5\ub9bd\ub41c \uc704\uc6d0\ud68c\ub85c\uc11c\uc758 \uc9c0\uc704\ub97c \uac00\uc9c0\ub294 \uad6d\uac00\uc778\uad8c\uc704\uc6d0\ud68c\ubc95\uc774 \uc81c\uc815\ub418\uc5c8\uace0 \uadf8\ud574 11\uc6d4 25\uc77c \uc815\uc2dd \ucd9c\ubc94\ud558\uc600\ub2e4.", "paragraph_sentence": "\uad6d\ub0b4\uc5d0\uc11c\ub294 \uc0ac\ud68c \uac01\uacc4\uac01\uce35\uc758 \uc774\ud574\uad00\uacc4\uc5d0 \uc758\ud574 \uc124\uce58 \uacfc\uc815\uc5d0\uc11c\ubd80\ud130 \uc0ac\ubb34\ucc98 \uc124\uce58\uae4c\uc9c0\uc758 \uacfc\uc815\uc774 \uc21c\ud0c4\uce58 \uc54a\uc558\ub2e4. \ub300\ud55c\ubbfc\uad6d\uc5d0\uc11c \uad6d\uac00\uc778\uad8c\uae30\uad6c \uc124\uce58\uc5d0 \uad00\ud55c \ub17c\uc758\ub294 1993\ub144 6\uc6d4 10\uc77c \ube44\uc5d4\ub098\uc5d0\uc11c \uc5f4\ub9b0 \uc138\uacc4\uc778\uad8c\ub300\ud68c\uc5d0 \ubbfc\uac04\ub2e8\uccb4 \uacf5\ub3d9\ub300\ucc45\uc704\uc6d0\ud68c\uac00 \ucc38\uac00\ud558\uc5ec \ub300\ud55c\ubbfc\uad6d \uc815\ubd80\uc5d0 \uad6d\uac00\uc778\uad8c\uae30\uad6c \uc124\uce58\ub97c \uc694\uccad\ud55c \uac83\uc744 \uacc4\uae30\ub85c \uc2dc\uc791\ub418\uc5c8\ub2e4. \uc774\ud6c4 \ubbfc\uac04\ub2e8\uccb4\ub4e4\uc740 \uad6d\ub0b4\uc678 \uc6cc\ud06c\uc20d\uacfc \uc778\uad8c\ub300\ud68c \ub4f1\uc5d0\uc11c \uc778\uad8c\uae30\uad6c \uc124\ub9bd\uc744 \uafb8\uc900\ud788 \uc694\uad6c\ud558\uc600\ub2e4. \uad6d\ub0b4\uc678\uc5d0\uc11c \uc774\ub7ec\ud55c \ub17c\uc758 \uacfc\uc815\uc744 \uac70\uce58\ub294 \uac00\uc6b4\ub370 1997\ub144 11\uc6d4 \uae40\ub300\uc911 \ub300\ud1b5\ub839 \ud6c4\ubcf4\uac00 \uc778\uad8c\ubc95 \uc81c\uc815 \ubc0f \uad6d\ubbfc\uc778\uad8c\uc704\uc6d0\ud68c \uc124\ub9bd\uc744 \ub300\uc120 \uacf5\uc57d\uc73c\ub85c \ubc1c\ud45c\ud588\ub2e4. \uadf8 \ud6c4 1998\ub144 \uae40\ub300\uc911 \uc815\ubd80\uc5d0\uc11c \uad6d\uac00\uc778\uad8c\uc704\uc6d0\ud68c \uc124\uce58\ub97c \uace8\uc790\ub85c \ud55c \uacc4\ud68d\uc744 \ubc1c\ud45c\ud558\uc600\uace0 \ubc95\ubb34\ubd80\uac00 \uad00\ub828 \ubc95\uc548\uc744 \uc81c\ucd9c\ud558\uc600\ub2e4. \ud558\uc9c0\ub9cc \uad6d\uc81c\uc0ac\uba74\uc704\uc6d0\ud68c\uc640 \uac19\uc740 \uc2dc\ubbfc\ub2e8\uccb4\uc5d0\uc11c \ubc95\ubb34\ubd80 \uc18c\uad00 \uae30\uad6c \uc124\uce58 \ubc18\ub300\ub97c \uc9c0\uc18d\uc801\uc73c\ub85c \uc8fc\uc7a5\ud558\uc5ec 2001\ub144 5\uc6d4 24\uc77c \ub3c5\ub9bd\ub41c \uc704\uc6d0\ud68c\ub85c\uc11c\uc758 \uc9c0\uc704\ub97c \uac00\uc9c0\ub294 \uad6d\uac00\uc778\uad8c\uc704\uc6d0\ud68c\ubc95\uc774 \uc81c\uc815\ub418\uc5c8\uace0 \uadf8\ud574 11\uc6d4 25\uc77c \uc815\uc2dd \ucd9c\ubc94\ud558\uc600\ub2e4. \ub2e4\ub9cc \uc0ac\ubb34\ucc98 \uc815\uc6d0\uacfc \uc608\uc0b0\uc5d0 \ub530\ub978 \ud589\uc815\uc790\uce58\ubd80\uc640\uc758 \uac08\ub4f1\uc774 \uc788\uc5b4 \uc774\ub97c \uc870\uc728\ud574 \ub098\uac00\ub294 \uacfc\uc815\uc774 \uae38\uc5b4\uc838 \uacb0\uad6d \uad6d\uac00\uc778\uad8c\uc704\uc6d0\ud68c \uc0ac\ubb34\ucc98\ub294 2002\ub144 4\uc6d4 1\uc77c \ubc1c\uc871\ud558\uc600\ub2e4.", "paragraph_answer": "\uad6d\ub0b4\uc5d0\uc11c\ub294 \uc0ac\ud68c \uac01\uacc4\uac01\uce35\uc758 \uc774\ud574\uad00\uacc4\uc5d0 \uc758\ud574 \uc124\uce58 \uacfc\uc815\uc5d0\uc11c\ubd80\ud130 \uc0ac\ubb34\ucc98 \uc124\uce58\uae4c\uc9c0\uc758 \uacfc\uc815\uc774 \uc21c\ud0c4\uce58 \uc54a\uc558\ub2e4. \ub300\ud55c\ubbfc\uad6d\uc5d0\uc11c \uad6d\uac00\uc778\uad8c\uae30\uad6c \uc124\uce58\uc5d0 \uad00\ud55c \ub17c\uc758\ub294 1993\ub144 6\uc6d4 10\uc77c \ube44\uc5d4\ub098\uc5d0\uc11c \uc5f4\ub9b0 \uc138\uacc4\uc778\uad8c\ub300\ud68c\uc5d0 \ubbfc\uac04\ub2e8\uccb4 \uacf5\ub3d9\ub300\ucc45\uc704\uc6d0\ud68c\uac00 \ucc38\uac00\ud558\uc5ec \ub300\ud55c\ubbfc\uad6d \uc815\ubd80\uc5d0 \uad6d\uac00\uc778\uad8c\uae30\uad6c \uc124\uce58\ub97c \uc694\uccad\ud55c \uac83\uc744 \uacc4\uae30\ub85c \uc2dc\uc791\ub418\uc5c8\ub2e4. \uc774\ud6c4 \ubbfc\uac04\ub2e8\uccb4\ub4e4\uc740 \uad6d\ub0b4\uc678 \uc6cc\ud06c\uc20d\uacfc \uc778\uad8c\ub300\ud68c \ub4f1\uc5d0\uc11c \uc778\uad8c\uae30\uad6c \uc124\ub9bd\uc744 \uafb8\uc900\ud788 \uc694\uad6c\ud558\uc600\ub2e4. \uad6d\ub0b4\uc678\uc5d0\uc11c \uc774\ub7ec\ud55c \ub17c\uc758 \uacfc\uc815\uc744 \uac70\uce58\ub294 \uac00\uc6b4\ub370 1997\ub144 11\uc6d4 \uae40\ub300\uc911 \ub300\ud1b5\ub839 \ud6c4\ubcf4\uac00 \uc778\uad8c\ubc95 \uc81c\uc815 \ubc0f \uad6d\ubbfc\uc778\uad8c\uc704\uc6d0\ud68c \uc124\ub9bd\uc744 \ub300\uc120 \uacf5\uc57d\uc73c\ub85c \ubc1c\ud45c\ud588\ub2e4. \uadf8 \ud6c4 1998\ub144 \uae40\ub300\uc911 \uc815\ubd80\uc5d0\uc11c \uad6d\uac00\uc778\uad8c\uc704\uc6d0\ud68c \uc124\uce58\ub97c \uace8\uc790\ub85c \ud55c \uacc4\ud68d\uc744 \ubc1c\ud45c\ud558\uc600\uace0 \ubc95\ubb34\ubd80\uac00 \uad00\ub828 \ubc95\uc548\uc744 \uc81c\ucd9c\ud558\uc600\ub2e4. \ud558\uc9c0\ub9cc \uad6d\uc81c\uc0ac\uba74\uc704\uc6d0\ud68c\uc640 \uac19\uc740 \uc2dc\ubbfc\ub2e8\uccb4\uc5d0\uc11c \ubc95\ubb34\ubd80 \uc18c\uad00 \uae30\uad6c \uc124\uce58 \ubc18\ub300\ub97c \uc9c0\uc18d\uc801\uc73c\ub85c \uc8fc\uc7a5\ud558\uc5ec 2001\ub144 5\uc6d4 24\uc77c \ub3c5\ub9bd\ub41c \uc704\uc6d0\ud68c\ub85c\uc11c\uc758 \uc9c0\uc704\ub97c \uac00\uc9c0\ub294 \uad6d\uac00\uc778\uad8c\uc704\uc6d0\ud68c\ubc95\uc774 \uc81c\uc815\ub418\uc5c8\uace0 \uadf8\ud574 11\uc6d4 25\uc77c \uc815\uc2dd \ucd9c\ubc94\ud558\uc600\ub2e4. \ub2e4\ub9cc \uc0ac\ubb34\ucc98 \uc815\uc6d0\uacfc \uc608\uc0b0\uc5d0 \ub530\ub978 \ud589\uc815\uc790\uce58\ubd80\uc640\uc758 \uac08\ub4f1\uc774 \uc788\uc5b4 \uc774\ub97c \uc870\uc728\ud574 \ub098\uac00\ub294 \uacfc\uc815\uc774 \uae38\uc5b4\uc838 \uacb0\uad6d \uad6d\uac00\uc778\uad8c\uc704\uc6d0\ud68c \uc0ac\ubb34\ucc98\ub294 2002\ub144 4\uc6d4 1\uc77c \ubc1c\uc871\ud558\uc600\ub2e4.", "sentence_answer": "\ud558\uc9c0\ub9cc \uad6d\uc81c\uc0ac\uba74\uc704\uc6d0\ud68c\uc640 \uac19\uc740 \uc2dc\ubbfc\ub2e8\uccb4\uc5d0\uc11c \ubc95\ubb34\ubd80 \uc18c\uad00 \uae30\uad6c \uc124\uce58 \ubc18\ub300\ub97c \uc9c0\uc18d\uc801\uc73c\ub85c \uc8fc\uc7a5\ud558\uc5ec 2001\ub144 5\uc6d4 24\uc77c \ub3c5\ub9bd\ub41c \uc704\uc6d0\ud68c\ub85c\uc11c\uc758 \uc9c0\uc704\ub97c \uac00\uc9c0\ub294 \uad6d\uac00\uc778\uad8c\uc704\uc6d0\ud68c\ubc95\uc774 \uc81c\uc815\ub418\uc5c8\uace0 \uadf8\ud574 11\uc6d4 25\uc77c \uc815\uc2dd \ucd9c\ubc94\ud558\uc600\ub2e4.", "paragraph_id": "6577657-1-2", "paragraph_question": "question: \ub3c5\ub9bd\ub41c \uc704\uc6d0\ud68c\uc778 \uad6d\uac00\uc778\uad8c\uc704\uc6d0\ud68c\ubc95\uc774 \uc81c\uc815\ub41c \ub0a0\uc740?, context: \uad6d\ub0b4\uc5d0\uc11c\ub294 \uc0ac\ud68c \uac01\uacc4\uac01\uce35\uc758 \uc774\ud574\uad00\uacc4\uc5d0 \uc758\ud574 \uc124\uce58 \uacfc\uc815\uc5d0\uc11c\ubd80\ud130 \uc0ac\ubb34\ucc98 \uc124\uce58\uae4c\uc9c0\uc758 \uacfc\uc815\uc774 \uc21c\ud0c4\uce58 \uc54a\uc558\ub2e4. \ub300\ud55c\ubbfc\uad6d\uc5d0\uc11c \uad6d\uac00\uc778\uad8c\uae30\uad6c \uc124\uce58\uc5d0 \uad00\ud55c \ub17c\uc758\ub294 1993\ub144 6\uc6d4 10\uc77c \ube44\uc5d4\ub098\uc5d0\uc11c \uc5f4\ub9b0 \uc138\uacc4\uc778\uad8c\ub300\ud68c\uc5d0 \ubbfc\uac04\ub2e8\uccb4 \uacf5\ub3d9\ub300\ucc45\uc704\uc6d0\ud68c\uac00 \ucc38\uac00\ud558\uc5ec \ub300\ud55c\ubbfc\uad6d \uc815\ubd80\uc5d0 \uad6d\uac00\uc778\uad8c\uae30\uad6c \uc124\uce58\ub97c \uc694\uccad\ud55c \uac83\uc744 \uacc4\uae30\ub85c \uc2dc\uc791\ub418\uc5c8\ub2e4. \uc774\ud6c4 \ubbfc\uac04\ub2e8\uccb4\ub4e4\uc740 \uad6d\ub0b4\uc678 \uc6cc\ud06c\uc20d\uacfc \uc778\uad8c\ub300\ud68c \ub4f1\uc5d0\uc11c \uc778\uad8c\uae30\uad6c \uc124\ub9bd\uc744 \uafb8\uc900\ud788 \uc694\uad6c\ud558\uc600\ub2e4. \uad6d\ub0b4\uc678\uc5d0\uc11c \uc774\ub7ec\ud55c \ub17c\uc758 \uacfc\uc815\uc744 \uac70\uce58\ub294 \uac00\uc6b4\ub370 1997\ub144 11\uc6d4 \uae40\ub300\uc911 \ub300\ud1b5\ub839 \ud6c4\ubcf4\uac00 \uc778\uad8c\ubc95 \uc81c\uc815 \ubc0f \uad6d\ubbfc\uc778\uad8c\uc704\uc6d0\ud68c \uc124\ub9bd\uc744 \ub300\uc120 \uacf5\uc57d\uc73c\ub85c \ubc1c\ud45c\ud588\ub2e4. \uadf8 \ud6c4 1998\ub144 \uae40\ub300\uc911 \uc815\ubd80\uc5d0\uc11c \uad6d\uac00\uc778\uad8c\uc704\uc6d0\ud68c \uc124\uce58\ub97c \uace8\uc790\ub85c \ud55c \uacc4\ud68d\uc744 \ubc1c\ud45c\ud558\uc600\uace0 \ubc95\ubb34\ubd80\uac00 \uad00\ub828 \ubc95\uc548\uc744 \uc81c\ucd9c\ud558\uc600\ub2e4. \ud558\uc9c0\ub9cc \uad6d\uc81c\uc0ac\uba74\uc704\uc6d0\ud68c\uc640 \uac19\uc740 \uc2dc\ubbfc\ub2e8\uccb4\uc5d0\uc11c \ubc95\ubb34\ubd80 \uc18c\uad00 \uae30\uad6c \uc124\uce58 \ubc18\ub300\ub97c \uc9c0\uc18d\uc801\uc73c\ub85c \uc8fc\uc7a5\ud558\uc5ec 2001\ub144 5\uc6d4 24\uc77c \ub3c5\ub9bd\ub41c \uc704\uc6d0\ud68c\ub85c\uc11c\uc758 \uc9c0\uc704\ub97c \uac00\uc9c0\ub294 \uad6d\uac00\uc778\uad8c\uc704\uc6d0\ud68c\ubc95\uc774 \uc81c\uc815\ub418\uc5c8\uace0 \uadf8\ud574 11\uc6d4 25\uc77c \uc815\uc2dd \ucd9c\ubc94\ud558\uc600\ub2e4. \ub2e4\ub9cc \uc0ac\ubb34\ucc98 \uc815\uc6d0\uacfc \uc608\uc0b0\uc5d0 \ub530\ub978 \ud589\uc815\uc790\uce58\ubd80\uc640\uc758 \uac08\ub4f1\uc774 \uc788\uc5b4 \uc774\ub97c \uc870\uc728\ud574 \ub098\uac00\ub294 \uacfc\uc815\uc774 \uae38\uc5b4\uc838 \uacb0\uad6d \uad6d\uac00\uc778\uad8c\uc704\uc6d0\ud68c \uc0ac\ubb34\ucc98\ub294 2002\ub144 4\uc6d4 1\uc77c \ubc1c\uc871\ud558\uc600\ub2e4."} {"question": "\ub9e5 \ubbf8\ub2c8 \ubaa8\ub378\uc740 2010\ub144 5\uc6d4 15\uc77c \uc774\ud6c4 \uc5b8\uc81c \uc5c5\ub370\uc774\ud2b8\uac00 \uc774\ub8e8\uc5b4\uc9c0\uac8c \ub418\uc5c8\ub098?", "paragraph": "\ud604\uc7ac \uc0c8\ub85c\uc6b4 \ub9e5 \ubbf8\ub2c8 \ubaa8\ub378\uc740 2010\ub144 5\uc6d4 15\uc77c\uc5d0 \uacf5\uac1c\ub418\uc5c8\uace0, \ub354 \uc587\uc740 \ub514\uc790\uc778\uc744 \uac16\ucd98 \uc54c\ub8e8\ubbf8\ub284 \uc720\ub2c8\ubc14\ub514 \ub514\uc790\uc778\uc774\ub2e4. \uc774\ub54c\ubb38\uc5d0 \ub7a8\uc774\ub098 \ud558\ub4dc\uc640 \uac19\uc740 \ub0b4\ubd80 \ubd80\ud488\uc744 \uad50\uccb4\ud560 \uc218 \uc788\uac8c \ub418\uc5c8\uc73c\uba70, \ud558\ub4dc\uc6e8\uc5b4 \uc5c5\uadf8\ub808\uc774\ub4dc \uc5ed\uc2dc \uac00\ub2a5\ud574\uc84c\ub2e4. \ub610\ud55c \uc5d4\ub514\ube44\uc544 Geforce 320M\uc744 \uac16\ucd94\uc5b4 HDMI \uc785\ub825\uc744 \uc9c0\uc6d0\ud55c\ub2e4. \uc774 \ubaa8\ub378\uc758 \uc5c5\ub370\uc774\ud2b8\ub294 2011\ub144 6\uc6d4 20\uc77c\uc5d0 \uc774\ub904\uc84c\uc73c\uba70 \ub0b4\uc7a5\ud615 CD/DVD \uad11\ud559 \ub4dc\ub77c\uc774\ube0c\uc640, \uc0c8\ub85c\uc6b4 \uc36c\ub354\ubcfc\ud2b8 \ud3ec\ud2b8, \uc778\ud154 \ucf54\uc5b4 i5\ub97c \uac16\ucd94\uace0, \uc778\ud154 HD 3000 \uadf8\ub798\ud53d \uce74\ub4dc\ub97c \uae30\ubcf8\uc73c\ub85c \ud0d1\uc7ac\ud558\uba70, \ucd94\uac00 \uad6c\uc131\uc73c\ub85c \ub77c\ub370\uc628 HD6630M\uc744 \ud0d1\uc7ac\ud560 \uc218 \uc788\ub2e4. \uc11c\ubc84 \ubaa8\ub378 \uc5ed\uc2dc \uc5c5\ub370\uc774\ud2b8 \ub418\uc5b4 \uc778\ud154 \ucffc\ub4dc \ucf54\uc5b4i7\uc744 \ud0d1\uc7ac\ud588\ub2e4. \uc774 \ucf54\uc5b4\ub294 2012\ub144 \ub9d0\uae4c\uc9c0 \uacc4\uc18d \uc0ac\uc6a9\ub418\uc5c8\ub2e4.", "answer": "2011\ub144 6\uc6d4 20\uc77c", "sentence": "\uc774 \ubaa8\ub378\uc758 \uc5c5\ub370\uc774\ud2b8\ub294 2011\ub144 6\uc6d4 20\uc77c \uc5d0 \uc774\ub904\uc84c\uc73c\uba70 \ub0b4\uc7a5\ud615 CD/DVD \uad11\ud559 \ub4dc\ub77c\uc774\ube0c\uc640, \uc0c8\ub85c\uc6b4 \uc36c\ub354\ubcfc\ud2b8 \ud3ec\ud2b8, \uc778\ud154 \ucf54\uc5b4 i5\ub97c \uac16\ucd94\uace0, \uc778\ud154 HD 3000 \uadf8\ub798\ud53d \uce74\ub4dc\ub97c \uae30\ubcf8\uc73c\ub85c \ud0d1\uc7ac\ud558\uba70, \ucd94\uac00 \uad6c\uc131\uc73c\ub85c \ub77c\ub370\uc628 HD6630M\uc744 \ud0d1\uc7ac\ud560 \uc218 \uc788\ub2e4.", "paragraph_sentence": "\ud604\uc7ac \uc0c8\ub85c\uc6b4 \ub9e5 \ubbf8\ub2c8 \ubaa8\ub378\uc740 2010\ub144 5\uc6d4 15\uc77c\uc5d0 \uacf5\uac1c\ub418\uc5c8\uace0, \ub354 \uc587\uc740 \ub514\uc790\uc778\uc744 \uac16\ucd98 \uc54c\ub8e8\ubbf8\ub284 \uc720\ub2c8\ubc14\ub514 \ub514\uc790\uc778\uc774\ub2e4. \uc774\ub54c\ubb38\uc5d0 \ub7a8\uc774\ub098 \ud558\ub4dc\uc640 \uac19\uc740 \ub0b4\ubd80 \ubd80\ud488\uc744 \uad50\uccb4\ud560 \uc218 \uc788\uac8c \ub418\uc5c8\uc73c\uba70, \ud558\ub4dc\uc6e8\uc5b4 \uc5c5\uadf8\ub808\uc774\ub4dc \uc5ed\uc2dc \uac00\ub2a5\ud574\uc84c\ub2e4. \ub610\ud55c \uc5d4\ub514\ube44\uc544 Geforce 320M\uc744 \uac16\ucd94\uc5b4 HDMI \uc785\ub825\uc744 \uc9c0\uc6d0\ud55c\ub2e4. \uc774 \ubaa8\ub378\uc758 \uc5c5\ub370\uc774\ud2b8\ub294 2011\ub144 6\uc6d4 20\uc77c \uc5d0 \uc774\ub904\uc84c\uc73c\uba70 \ub0b4\uc7a5\ud615 CD/DVD \uad11\ud559 \ub4dc\ub77c\uc774\ube0c\uc640, \uc0c8\ub85c\uc6b4 \uc36c\ub354\ubcfc\ud2b8 \ud3ec\ud2b8, \uc778\ud154 \ucf54\uc5b4 i5\ub97c \uac16\ucd94\uace0, \uc778\ud154 HD 3000 \uadf8\ub798\ud53d \uce74\ub4dc\ub97c \uae30\ubcf8\uc73c\ub85c \ud0d1\uc7ac\ud558\uba70, \ucd94\uac00 \uad6c\uc131\uc73c\ub85c \ub77c\ub370\uc628 HD6630M\uc744 \ud0d1\uc7ac\ud560 \uc218 \uc788\ub2e4. \uc11c\ubc84 \ubaa8\ub378 \uc5ed\uc2dc \uc5c5\ub370\uc774\ud2b8 \ub418\uc5b4 \uc778\ud154 \ucffc\ub4dc \ucf54\uc5b4i7\uc744 \ud0d1\uc7ac\ud588\ub2e4. \uc774 \ucf54\uc5b4\ub294 2012\ub144 \ub9d0\uae4c\uc9c0 \uacc4\uc18d \uc0ac\uc6a9\ub418\uc5c8\ub2e4.", "paragraph_answer": "\ud604\uc7ac \uc0c8\ub85c\uc6b4 \ub9e5 \ubbf8\ub2c8 \ubaa8\ub378\uc740 2010\ub144 5\uc6d4 15\uc77c\uc5d0 \uacf5\uac1c\ub418\uc5c8\uace0, \ub354 \uc587\uc740 \ub514\uc790\uc778\uc744 \uac16\ucd98 \uc54c\ub8e8\ubbf8\ub284 \uc720\ub2c8\ubc14\ub514 \ub514\uc790\uc778\uc774\ub2e4. \uc774\ub54c\ubb38\uc5d0 \ub7a8\uc774\ub098 \ud558\ub4dc\uc640 \uac19\uc740 \ub0b4\ubd80 \ubd80\ud488\uc744 \uad50\uccb4\ud560 \uc218 \uc788\uac8c \ub418\uc5c8\uc73c\uba70, \ud558\ub4dc\uc6e8\uc5b4 \uc5c5\uadf8\ub808\uc774\ub4dc \uc5ed\uc2dc \uac00\ub2a5\ud574\uc84c\ub2e4. \ub610\ud55c \uc5d4\ub514\ube44\uc544 Geforce 320M\uc744 \uac16\ucd94\uc5b4 HDMI \uc785\ub825\uc744 \uc9c0\uc6d0\ud55c\ub2e4. \uc774 \ubaa8\ub378\uc758 \uc5c5\ub370\uc774\ud2b8\ub294 2011\ub144 6\uc6d4 20\uc77c \uc5d0 \uc774\ub904\uc84c\uc73c\uba70 \ub0b4\uc7a5\ud615 CD/DVD \uad11\ud559 \ub4dc\ub77c\uc774\ube0c\uc640, \uc0c8\ub85c\uc6b4 \uc36c\ub354\ubcfc\ud2b8 \ud3ec\ud2b8, \uc778\ud154 \ucf54\uc5b4 i5\ub97c \uac16\ucd94\uace0, \uc778\ud154 HD 3000 \uadf8\ub798\ud53d \uce74\ub4dc\ub97c \uae30\ubcf8\uc73c\ub85c \ud0d1\uc7ac\ud558\uba70, \ucd94\uac00 \uad6c\uc131\uc73c\ub85c \ub77c\ub370\uc628 HD6630M\uc744 \ud0d1\uc7ac\ud560 \uc218 \uc788\ub2e4. \uc11c\ubc84 \ubaa8\ub378 \uc5ed\uc2dc \uc5c5\ub370\uc774\ud2b8 \ub418\uc5b4 \uc778\ud154 \ucffc\ub4dc \ucf54\uc5b4i7\uc744 \ud0d1\uc7ac\ud588\ub2e4. \uc774 \ucf54\uc5b4\ub294 2012\ub144 \ub9d0\uae4c\uc9c0 \uacc4\uc18d \uc0ac\uc6a9\ub418\uc5c8\ub2e4.", "sentence_answer": "\uc774 \ubaa8\ub378\uc758 \uc5c5\ub370\uc774\ud2b8\ub294 2011\ub144 6\uc6d4 20\uc77c \uc5d0 \uc774\ub904\uc84c\uc73c\uba70 \ub0b4\uc7a5\ud615 CD/DVD \uad11\ud559 \ub4dc\ub77c\uc774\ube0c\uc640, \uc0c8\ub85c\uc6b4 \uc36c\ub354\ubcfc\ud2b8 \ud3ec\ud2b8, \uc778\ud154 \ucf54\uc5b4 i5\ub97c \uac16\ucd94\uace0, \uc778\ud154 HD 3000 \uadf8\ub798\ud53d \uce74\ub4dc\ub97c \uae30\ubcf8\uc73c\ub85c \ud0d1\uc7ac\ud558\uba70, \ucd94\uac00 \uad6c\uc131\uc73c\ub85c \ub77c\ub370\uc628 HD6630M\uc744 \ud0d1\uc7ac\ud560 \uc218 \uc788\ub2e4.", "paragraph_id": "6075058-1-1", "paragraph_question": "question: \ub9e5 \ubbf8\ub2c8 \ubaa8\ub378\uc740 2010\ub144 5\uc6d4 15\uc77c \uc774\ud6c4 \uc5b8\uc81c \uc5c5\ub370\uc774\ud2b8\uac00 \uc774\ub8e8\uc5b4\uc9c0\uac8c \ub418\uc5c8\ub098?, context: \ud604\uc7ac \uc0c8\ub85c\uc6b4 \ub9e5 \ubbf8\ub2c8 \ubaa8\ub378\uc740 2010\ub144 5\uc6d4 15\uc77c\uc5d0 \uacf5\uac1c\ub418\uc5c8\uace0, \ub354 \uc587\uc740 \ub514\uc790\uc778\uc744 \uac16\ucd98 \uc54c\ub8e8\ubbf8\ub284 \uc720\ub2c8\ubc14\ub514 \ub514\uc790\uc778\uc774\ub2e4. \uc774\ub54c\ubb38\uc5d0 \ub7a8\uc774\ub098 \ud558\ub4dc\uc640 \uac19\uc740 \ub0b4\ubd80 \ubd80\ud488\uc744 \uad50\uccb4\ud560 \uc218 \uc788\uac8c \ub418\uc5c8\uc73c\uba70, \ud558\ub4dc\uc6e8\uc5b4 \uc5c5\uadf8\ub808\uc774\ub4dc \uc5ed\uc2dc \uac00\ub2a5\ud574\uc84c\ub2e4. \ub610\ud55c \uc5d4\ub514\ube44\uc544 Geforce 320M\uc744 \uac16\ucd94\uc5b4 HDMI \uc785\ub825\uc744 \uc9c0\uc6d0\ud55c\ub2e4. \uc774 \ubaa8\ub378\uc758 \uc5c5\ub370\uc774\ud2b8\ub294 2011\ub144 6\uc6d4 20\uc77c\uc5d0 \uc774\ub904\uc84c\uc73c\uba70 \ub0b4\uc7a5\ud615 CD/DVD \uad11\ud559 \ub4dc\ub77c\uc774\ube0c\uc640, \uc0c8\ub85c\uc6b4 \uc36c\ub354\ubcfc\ud2b8 \ud3ec\ud2b8, \uc778\ud154 \ucf54\uc5b4 i5\ub97c \uac16\ucd94\uace0, \uc778\ud154 HD 3000 \uadf8\ub798\ud53d \uce74\ub4dc\ub97c \uae30\ubcf8\uc73c\ub85c \ud0d1\uc7ac\ud558\uba70, \ucd94\uac00 \uad6c\uc131\uc73c\ub85c \ub77c\ub370\uc628 HD6630M\uc744 \ud0d1\uc7ac\ud560 \uc218 \uc788\ub2e4. \uc11c\ubc84 \ubaa8\ub378 \uc5ed\uc2dc \uc5c5\ub370\uc774\ud2b8 \ub418\uc5b4 \uc778\ud154 \ucffc\ub4dc \ucf54\uc5b4i7\uc744 \ud0d1\uc7ac\ud588\ub2e4. \uc774 \ucf54\uc5b4\ub294 2012\ub144 \ub9d0\uae4c\uc9c0 \uacc4\uc18d \uc0ac\uc6a9\ub418\uc5c8\ub2e4."} {"question": "16\uc138\uae30 \uae08\uc740 \uc138\uacf5\uc0ac\uc5d0 \uc758\ud574 \uc0ac\uc6a9\ub41c \uc7a5\uc2dd\uc744 \ud65c\uc6a9\ud55c \uac74\ucd95\uc591\uc2dd\uc740?", "paragraph": "\uac74\ucd95\uc740 \ub2e4\uc74c\uacfc \uac19\uc774 16\uc138\uae30 \uae08\uc740 \uc138\uacf5\uc0ac\uc5d0 \uc758\ud574 \uc0ac\uc6a9\ub41c \uc7a5\uc2dd\uc744 \ud65c\uc6a9\ud55c \uac74\ucd95\uc591\uc2dd \ud50c\ub77c\ud14c\ub808\uc2a4\ucf54(15\uc138\uae30\uc5d0\uc11c-16\uc138\uae30\uc758 1\ubd84\uae30), \uc21c\uc218(\uc815\ud1b5)\uc8fc\uc758 \ubc0f, \uc774\ud0c8\ub9ac\uc544\ud48d (16\uc138\uae30 \uc804\ubc18\uae30), \uc5d0\ub808\ub9ac\uc544\ub178; \uc5d8\uc5d0\uc2a4\uaf2c\ub9ac\uc54c\uc758 \uac74\ucd95\uac00 \ud6c4\uc548 \ub370 \uc5d0\ub808\ub77c\uc5d0\uc11c \uc5f0\uc720\ud55c \uac74\ucd95\uc6a9\uc5b4(1559\ub144\ubd80\ud130 \ub2e4\uc74c\uc138\uae30 \uc911\ubc18) \ud06c\uac8c \uc138 \uc2dc\ub300\ub85c \ubd84\ub958\uac00 \uac00\ub2a5\ud558\ub2e4. \uc774\uc911 \uccab \ubc88\uc9f8\ub294 \ud45c\uba74\uc801\uc778 \ud615\ud0dc\uc5d0\uc11c \ub4dc\ub7ec\ub098\ub294\ub370 \uc804\ubc18\uc801\uc778 \uace0\ub515\uc591\uc2dd\uc744 \uc720\uc9c0\ud558\uba74\uc11c \ud30c\uc0ac\ub4dc\uc758 \ud654\ub824\ud55c \uc7a5\uc2dd\uc744 \ud558\ub294 \uac83\uc73c\ub85c \ub4dc\ub7ec\ub09c\ub2e4. \uc774 \uc7a5\uc2dd\uc740 \uc138\ubc00\ud558\uace0 \ud48d\ubd80\ud55c \ubaa8\uc2b5\uc744 \uc9c0\ub2c8\uace0 \uc788\ub294\ub370 \uc0b4\ub77c\ub9dd\uce74 \ub300\ud559 \uac74\ubb3c\uc758 \ud30c\uc0ac\ub4dc\uc758 \uc7a5\uc2dd\uc774 \ub300\ud45c\uc801\uc778 \uc608\ub77c\uace0 \ud560 \uc218 \uc788\ub2e4. \uc21c\uc218(\uc815\ud1b5)\uc8fc\uc758\ub294 \uc774\ud0c8\ub9ac\uc544\uc758 \uac00\uc7a5 \ud6c4\ubc18\uae30\uc758 \uce21\uba74\uc744 \ubc18\uc601\ud558\uace0 \uc788\ub294\ub370, \uc54c\ub78c\ube0c\ub77c\uc758 \uce74\ub97c\ub85c\uc2a4 5\uc138 \uad81\uc804\uc774 \ub300\ud45c\uc801\uc778 \uc608\uc774\ub2e4.", "answer": "\ud50c\ub77c\ud14c\ub808\uc2a4\ucf54", "sentence": "\uac74\ucd95\uc740 \ub2e4\uc74c\uacfc \uac19\uc774 16\uc138\uae30 \uae08\uc740 \uc138\uacf5\uc0ac\uc5d0 \uc758\ud574 \uc0ac\uc6a9\ub41c \uc7a5\uc2dd\uc744 \ud65c\uc6a9\ud55c \uac74\ucd95\uc591\uc2dd \ud50c\ub77c\ud14c\ub808\uc2a4\ucf54 (15\uc138\uae30\uc5d0\uc11c-16\uc138\uae30\uc758 1\ubd84\uae30), \uc21c\uc218(\uc815\ud1b5)\uc8fc\uc758 \ubc0f, \uc774\ud0c8\ub9ac\uc544\ud48d (16\uc138\uae30 \uc804\ubc18\uae30), \uc5d0\ub808\ub9ac\uc544\ub178; \uc5d8\uc5d0\uc2a4\uaf2c\ub9ac\uc54c\uc758 \uac74\ucd95\uac00 \ud6c4\uc548 \ub370 \uc5d0\ub808\ub77c\uc5d0\uc11c \uc5f0\uc720\ud55c \uac74\ucd95\uc6a9\uc5b4(1559\ub144\ubd80\ud130 \ub2e4\uc74c\uc138\uae30 \uc911\ubc18) \ud06c\uac8c \uc138 \uc2dc\ub300\ub85c \ubd84\ub958\uac00 \uac00\ub2a5\ud558\ub2e4.", "paragraph_sentence": " \uac74\ucd95\uc740 \ub2e4\uc74c\uacfc \uac19\uc774 16\uc138\uae30 \uae08\uc740 \uc138\uacf5\uc0ac\uc5d0 \uc758\ud574 \uc0ac\uc6a9\ub41c \uc7a5\uc2dd\uc744 \ud65c\uc6a9\ud55c \uac74\ucd95\uc591\uc2dd \ud50c\ub77c\ud14c\ub808\uc2a4\ucf54 (15\uc138\uae30\uc5d0\uc11c-16\uc138\uae30\uc758 1\ubd84\uae30), \uc21c\uc218(\uc815\ud1b5)\uc8fc\uc758 \ubc0f, \uc774\ud0c8\ub9ac\uc544\ud48d (16\uc138\uae30 \uc804\ubc18\uae30), \uc5d0\ub808\ub9ac\uc544\ub178; \uc5d8\uc5d0\uc2a4\uaf2c\ub9ac\uc54c\uc758 \uac74\ucd95\uac00 \ud6c4\uc548 \ub370 \uc5d0\ub808\ub77c\uc5d0\uc11c \uc5f0\uc720\ud55c \uac74\ucd95\uc6a9\uc5b4(1559\ub144\ubd80\ud130 \ub2e4\uc74c\uc138\uae30 \uc911\ubc18) \ud06c\uac8c \uc138 \uc2dc\ub300\ub85c \ubd84\ub958\uac00 \uac00\ub2a5\ud558\ub2e4. \uc774\uc911 \uccab \ubc88\uc9f8\ub294 \ud45c\uba74\uc801\uc778 \ud615\ud0dc\uc5d0\uc11c \ub4dc\ub7ec\ub098\ub294\ub370 \uc804\ubc18\uc801\uc778 \uace0\ub515\uc591\uc2dd\uc744 \uc720\uc9c0\ud558\uba74\uc11c \ud30c\uc0ac\ub4dc\uc758 \ud654\ub824\ud55c \uc7a5\uc2dd\uc744 \ud558\ub294 \uac83\uc73c\ub85c \ub4dc\ub7ec\ub09c\ub2e4. \uc774 \uc7a5\uc2dd\uc740 \uc138\ubc00\ud558\uace0 \ud48d\ubd80\ud55c \ubaa8\uc2b5\uc744 \uc9c0\ub2c8\uace0 \uc788\ub294\ub370 \uc0b4\ub77c\ub9dd\uce74 \ub300\ud559 \uac74\ubb3c\uc758 \ud30c\uc0ac\ub4dc\uc758 \uc7a5\uc2dd\uc774 \ub300\ud45c\uc801\uc778 \uc608\ub77c\uace0 \ud560 \uc218 \uc788\ub2e4. \uc21c\uc218(\uc815\ud1b5)\uc8fc\uc758\ub294 \uc774\ud0c8\ub9ac\uc544\uc758 \uac00\uc7a5 \ud6c4\ubc18\uae30\uc758 \uce21\uba74\uc744 \ubc18\uc601\ud558\uace0 \uc788\ub294\ub370, \uc54c\ub78c\ube0c\ub77c\uc758 \uce74\ub97c\ub85c\uc2a4 5\uc138 \uad81\uc804\uc774 \ub300\ud45c\uc801\uc778 \uc608\uc774\ub2e4.", "paragraph_answer": "\uac74\ucd95\uc740 \ub2e4\uc74c\uacfc \uac19\uc774 16\uc138\uae30 \uae08\uc740 \uc138\uacf5\uc0ac\uc5d0 \uc758\ud574 \uc0ac\uc6a9\ub41c \uc7a5\uc2dd\uc744 \ud65c\uc6a9\ud55c \uac74\ucd95\uc591\uc2dd \ud50c\ub77c\ud14c\ub808\uc2a4\ucf54 (15\uc138\uae30\uc5d0\uc11c-16\uc138\uae30\uc758 1\ubd84\uae30), \uc21c\uc218(\uc815\ud1b5)\uc8fc\uc758 \ubc0f, \uc774\ud0c8\ub9ac\uc544\ud48d (16\uc138\uae30 \uc804\ubc18\uae30), \uc5d0\ub808\ub9ac\uc544\ub178; \uc5d8\uc5d0\uc2a4\uaf2c\ub9ac\uc54c\uc758 \uac74\ucd95\uac00 \ud6c4\uc548 \ub370 \uc5d0\ub808\ub77c\uc5d0\uc11c \uc5f0\uc720\ud55c \uac74\ucd95\uc6a9\uc5b4(1559\ub144\ubd80\ud130 \ub2e4\uc74c\uc138\uae30 \uc911\ubc18) \ud06c\uac8c \uc138 \uc2dc\ub300\ub85c \ubd84\ub958\uac00 \uac00\ub2a5\ud558\ub2e4. \uc774\uc911 \uccab \ubc88\uc9f8\ub294 \ud45c\uba74\uc801\uc778 \ud615\ud0dc\uc5d0\uc11c \ub4dc\ub7ec\ub098\ub294\ub370 \uc804\ubc18\uc801\uc778 \uace0\ub515\uc591\uc2dd\uc744 \uc720\uc9c0\ud558\uba74\uc11c \ud30c\uc0ac\ub4dc\uc758 \ud654\ub824\ud55c \uc7a5\uc2dd\uc744 \ud558\ub294 \uac83\uc73c\ub85c \ub4dc\ub7ec\ub09c\ub2e4. \uc774 \uc7a5\uc2dd\uc740 \uc138\ubc00\ud558\uace0 \ud48d\ubd80\ud55c \ubaa8\uc2b5\uc744 \uc9c0\ub2c8\uace0 \uc788\ub294\ub370 \uc0b4\ub77c\ub9dd\uce74 \ub300\ud559 \uac74\ubb3c\uc758 \ud30c\uc0ac\ub4dc\uc758 \uc7a5\uc2dd\uc774 \ub300\ud45c\uc801\uc778 \uc608\ub77c\uace0 \ud560 \uc218 \uc788\ub2e4. \uc21c\uc218(\uc815\ud1b5)\uc8fc\uc758\ub294 \uc774\ud0c8\ub9ac\uc544\uc758 \uac00\uc7a5 \ud6c4\ubc18\uae30\uc758 \uce21\uba74\uc744 \ubc18\uc601\ud558\uace0 \uc788\ub294\ub370, \uc54c\ub78c\ube0c\ub77c\uc758 \uce74\ub97c\ub85c\uc2a4 5\uc138 \uad81\uc804\uc774 \ub300\ud45c\uc801\uc778 \uc608\uc774\ub2e4.", "sentence_answer": "\uac74\ucd95\uc740 \ub2e4\uc74c\uacfc \uac19\uc774 16\uc138\uae30 \uae08\uc740 \uc138\uacf5\uc0ac\uc5d0 \uc758\ud574 \uc0ac\uc6a9\ub41c \uc7a5\uc2dd\uc744 \ud65c\uc6a9\ud55c \uac74\ucd95\uc591\uc2dd \ud50c\ub77c\ud14c\ub808\uc2a4\ucf54 (15\uc138\uae30\uc5d0\uc11c-16\uc138\uae30\uc758 1\ubd84\uae30), \uc21c\uc218(\uc815\ud1b5)\uc8fc\uc758 \ubc0f, \uc774\ud0c8\ub9ac\uc544\ud48d (16\uc138\uae30 \uc804\ubc18\uae30), \uc5d0\ub808\ub9ac\uc544\ub178; \uc5d8\uc5d0\uc2a4\uaf2c\ub9ac\uc54c\uc758 \uac74\ucd95\uac00 \ud6c4\uc548 \ub370 \uc5d0\ub808\ub77c\uc5d0\uc11c \uc5f0\uc720\ud55c \uac74\ucd95\uc6a9\uc5b4(1559\ub144\ubd80\ud130 \ub2e4\uc74c\uc138\uae30 \uc911\ubc18) \ud06c\uac8c \uc138 \uc2dc\ub300\ub85c \ubd84\ub958\uac00 \uac00\ub2a5\ud558\ub2e4.", "paragraph_id": "6656981-4-0", "paragraph_question": "question: 16\uc138\uae30 \uae08\uc740 \uc138\uacf5\uc0ac\uc5d0 \uc758\ud574 \uc0ac\uc6a9\ub41c \uc7a5\uc2dd\uc744 \ud65c\uc6a9\ud55c \uac74\ucd95\uc591\uc2dd\uc740?, context: \uac74\ucd95\uc740 \ub2e4\uc74c\uacfc \uac19\uc774 16\uc138\uae30 \uae08\uc740 \uc138\uacf5\uc0ac\uc5d0 \uc758\ud574 \uc0ac\uc6a9\ub41c \uc7a5\uc2dd\uc744 \ud65c\uc6a9\ud55c \uac74\ucd95\uc591\uc2dd \ud50c\ub77c\ud14c\ub808\uc2a4\ucf54(15\uc138\uae30\uc5d0\uc11c-16\uc138\uae30\uc758 1\ubd84\uae30), \uc21c\uc218(\uc815\ud1b5)\uc8fc\uc758 \ubc0f, \uc774\ud0c8\ub9ac\uc544\ud48d (16\uc138\uae30 \uc804\ubc18\uae30), \uc5d0\ub808\ub9ac\uc544\ub178; \uc5d8\uc5d0\uc2a4\uaf2c\ub9ac\uc54c\uc758 \uac74\ucd95\uac00 \ud6c4\uc548 \ub370 \uc5d0\ub808\ub77c\uc5d0\uc11c \uc5f0\uc720\ud55c \uac74\ucd95\uc6a9\uc5b4(1559\ub144\ubd80\ud130 \ub2e4\uc74c\uc138\uae30 \uc911\ubc18) \ud06c\uac8c \uc138 \uc2dc\ub300\ub85c \ubd84\ub958\uac00 \uac00\ub2a5\ud558\ub2e4. \uc774\uc911 \uccab \ubc88\uc9f8\ub294 \ud45c\uba74\uc801\uc778 \ud615\ud0dc\uc5d0\uc11c \ub4dc\ub7ec\ub098\ub294\ub370 \uc804\ubc18\uc801\uc778 \uace0\ub515\uc591\uc2dd\uc744 \uc720\uc9c0\ud558\uba74\uc11c \ud30c\uc0ac\ub4dc\uc758 \ud654\ub824\ud55c \uc7a5\uc2dd\uc744 \ud558\ub294 \uac83\uc73c\ub85c \ub4dc\ub7ec\ub09c\ub2e4. \uc774 \uc7a5\uc2dd\uc740 \uc138\ubc00\ud558\uace0 \ud48d\ubd80\ud55c \ubaa8\uc2b5\uc744 \uc9c0\ub2c8\uace0 \uc788\ub294\ub370 \uc0b4\ub77c\ub9dd\uce74 \ub300\ud559 \uac74\ubb3c\uc758 \ud30c\uc0ac\ub4dc\uc758 \uc7a5\uc2dd\uc774 \ub300\ud45c\uc801\uc778 \uc608\ub77c\uace0 \ud560 \uc218 \uc788\ub2e4. \uc21c\uc218(\uc815\ud1b5)\uc8fc\uc758\ub294 \uc774\ud0c8\ub9ac\uc544\uc758 \uac00\uc7a5 \ud6c4\ubc18\uae30\uc758 \uce21\uba74\uc744 \ubc18\uc601\ud558\uace0 \uc788\ub294\ub370, \uc54c\ub78c\ube0c\ub77c\uc758 \uce74\ub97c\ub85c\uc2a4 5\uc138 \uad81\uc804\uc774 \ub300\ud45c\uc801\uc778 \uc608\uc774\ub2e4."} {"question": "\ub9cc\uc288\ud0c0\uc778 \ub300\uc704\ub294 \ubca8\ub85c\ube0c\uc758 \ubd80\ud558\ub85c \uc77c\ud558\ub2e4\uac00 \uc5b4\ub290 \uc0ac\ub2e8\uc758 \ucc38\ubaa8\uc7a5\uc73c\ub85c \uc784\uba85\ubc1b\ub098?", "paragraph": "1915\ub144 6\uc6d4 17\uc77c \ub9cc\uc288\ud0c0\uc778\uc740 \ubcf5\uadc0\ud558\uc5ec \ub9c9\uc2a4 \ud3f0 \uac08\ube44\uce20\uac00 \uc9c0\ud718\ud558\ub294 \uc81c10\uad70 \uc791\uc804\ucc38\ubaa8\uc7a5\uad50\uac00 \ub418\uc5c8\ub2e4. \uc5bc\ub9c8 \ub4a4 \ub300\uc704\ub85c \uc2b9\uc9c4\ud55c \ub9cc\uc288\ud0c0\uc778\uc740 \uc81c10\uad70\uc774 \ud3f4\ub780\ub4dc, \ub9ac\ud22c\uc544\ub2c8\uc544, \ubaac\ud14c\ub124\uadf8\ub85c, \uc54c\ubc14\ub2c8\uc544\ub97c \uc131\uacf5\uc801\uc73c\ub85c \uacf5\uaca9\ud558\ub294 \uacfc\uc815\uc5d0\uc11c \uacf5\uc138\uc791\uc804 \uacc4\ud68d \ubc0f \uc218\ud589\uc744 \uacbd\ud5d8\uc73c\ub85c \uccb4\ub4dd\ud588\ub2e4. 1916\ub144 \ucd08 \ubca0\ub974\ub42d\uc5d0\uc11c \uacf5\uc138\uc791\uc804\uc744 \uc218\ud589\ud558\uace0 \uadf8 \ub4a4\uc5d0\ub294 \uc19c \uac15 \uadfc\ucc98\uc758 \uc0ac\ub839\ubd80\uc5d0\uc11c \ud504\ub9ac\uce20 \ud3f0 \ubca8\ub85c\ube0c \uc7a5\uad70\uacfc \ud504\ub9ac\uce20 \ud3f0 \ub85c\uc2a4\ubca0\ub974\ud06c \ucc38\ubaa8\uc7a5 \ud718\ud558\uc5d0 \ubcf4\uae09\uc7a5\uad50\ub85c \uc77c\ud588\ub2e4. \uc774 \uc77c\ub300\ub294 1\ucc28 \ub300\uc804 \ub0b4\ub0b4 \uac00\uc7a5 \uaca9\ub82c\ud55c \uc804\ud22c\uac00 \ubc8c\uc5b4\uc9c4 \uc804\uc7a5\uc774\uc5c8\ub2e4. \uc601\uad6d\uad70\uacfc \ud504\ub791\uc2a4\uad70\uc740 1916\ub144 7\uc6d4\uc5d0\uc11c 11\uc6d4\uc5d0 \uac78\uccd0 \uacf5\uc138\ub97c \ud3bc\uccd0 \uaca8\uc6b8\ucbe4\uc5d0\ub294 \ub3c5\uc77c\uad70\uc744 \ubca0\ub974\ub42d-\ub791\uc2a4 \uc0ac\uc774\uc5d0 \uad6c\ucd95\ub41c \ubc29\uc5b4\uc120\uc778 \ud78c\ub374\ubd80\ub974\ud06c \uc120\uae4c\uc9c0 \ubc00\uc5b4\ubd99\uc600\ub2e4. \ub9cc\uc288\ud0c0\uc778 \ub300\uc704\ub294 1917\ub144 10\uc6d4\uae4c\uc9c0 \uacc4\uc18d \ubca8\ub85c\ube0c \uc218\ud558\ub85c \uc77c\ud558\ub2e4\uac00 \ub9ac\uac00\ub97c \uc810\ub839\uc911\uc778 \uc81c4\uae30\ubcd1\uc0ac\ub2e8 \ucc38\ubaa8\uc7a5\uc73c\ub85c \uc784\uba85\ubc1b\uc544 \uadf8\ub9ac\ub85c \uc62e\uaca8\uac14\ub2e4. 1918\ub144 3\uc6d4 \ube0c\ub808\uc2a4\ud2b8-\ub9ac\ud1a0\ud504\uc2a4\ud06c \uc870\uc57d\uc73c\ub85c \ub3d9\ubd80\uc804\uc120\uc774 \ub3c5\uc77c\uc758 \uc2b9\ub9ac\ub85c \ub05d\ub098\uc790 \ub9cc\uc288\ud0c0\uc778\uc758 \ubd80\ub300\ub294 \ub3d9\ubd80\uc804\uc120\uc5d0 \ub354 \uba38\ubb34\ub97c \ud544\uc694\uac00 \uc5c6\uc5b4\uc84c\uace0, \ub9cc\uc288\ud0c0\uc778\uc740 \ub2e4\uc2dc \ub7ad\uc2a4 \uadfc\uad50\uc758 \uc81c213\ubcf4\ubcd1\uc0ac\ub2e8\uc73c\ub85c \uac70\ucc98\ub97c \uc62e\uacbc\ub2e4. \ub3c5\uc77c\uad70\uc740 \uc11c\ubd80\uc804\uc120\uc5d0\uc11c \uc77c\ubd80 \uc131\uacfc\ub97c \uac70\ub450\uc5c8\uc73c\ub098 \uacb0\uad6d \uc804\uc7c1\uc5d0\uc11c \uc84c\uace0, 1918\ub144 11\uc6d4 11\uc77c \ud734\uc804\ud611\uc815\uc774 \uc870\uc778(1918\ub144 11\uc6d4 11\uc77c \ud734\uc804)\ub41c\ub2e4.", "answer": "\uc81c4\uae30\ubcd1\uc0ac\ub2e8", "sentence": "\ub9cc\uc288\ud0c0\uc778 \ub300\uc704\ub294 1917\ub144 10\uc6d4\uae4c\uc9c0 \uacc4\uc18d \ubca8\ub85c\ube0c \uc218\ud558\ub85c \uc77c\ud558\ub2e4\uac00 \ub9ac\uac00\ub97c \uc810\ub839\uc911\uc778 \uc81c4\uae30\ubcd1\uc0ac\ub2e8 \ucc38\ubaa8\uc7a5\uc73c\ub85c \uc784\uba85\ubc1b\uc544 \uadf8\ub9ac\ub85c \uc62e\uaca8\uac14\ub2e4.", "paragraph_sentence": "1915\ub144 6\uc6d4 17\uc77c \ub9cc\uc288\ud0c0\uc778\uc740 \ubcf5\uadc0\ud558\uc5ec \ub9c9\uc2a4 \ud3f0 \uac08\ube44\uce20\uac00 \uc9c0\ud718\ud558\ub294 \uc81c10\uad70 \uc791\uc804\ucc38\ubaa8\uc7a5\uad50\uac00 \ub418\uc5c8\ub2e4. \uc5bc\ub9c8 \ub4a4 \ub300\uc704\ub85c \uc2b9\uc9c4\ud55c \ub9cc\uc288\ud0c0\uc778\uc740 \uc81c10\uad70\uc774 \ud3f4\ub780\ub4dc, \ub9ac\ud22c\uc544\ub2c8\uc544, \ubaac\ud14c\ub124\uadf8\ub85c, \uc54c\ubc14\ub2c8\uc544\ub97c \uc131\uacf5\uc801\uc73c\ub85c \uacf5\uaca9\ud558\ub294 \uacfc\uc815\uc5d0\uc11c \uacf5\uc138\uc791\uc804 \uacc4\ud68d \ubc0f \uc218\ud589\uc744 \uacbd\ud5d8\uc73c\ub85c \uccb4\ub4dd\ud588\ub2e4. 1916\ub144 \ucd08 \ubca0\ub974\ub42d\uc5d0\uc11c \uacf5\uc138\uc791\uc804\uc744 \uc218\ud589\ud558\uace0 \uadf8 \ub4a4\uc5d0\ub294 \uc19c \uac15 \uadfc\ucc98\uc758 \uc0ac\ub839\ubd80\uc5d0\uc11c \ud504\ub9ac\uce20 \ud3f0 \ubca8\ub85c\ube0c \uc7a5\uad70\uacfc \ud504\ub9ac\uce20 \ud3f0 \ub85c\uc2a4\ubca0\ub974\ud06c \ucc38\ubaa8\uc7a5 \ud718\ud558\uc5d0 \ubcf4\uae09\uc7a5\uad50\ub85c \uc77c\ud588\ub2e4. \uc774 \uc77c\ub300\ub294 1\ucc28 \ub300\uc804 \ub0b4\ub0b4 \uac00\uc7a5 \uaca9\ub82c\ud55c \uc804\ud22c\uac00 \ubc8c\uc5b4\uc9c4 \uc804\uc7a5\uc774\uc5c8\ub2e4. \uc601\uad6d\uad70\uacfc \ud504\ub791\uc2a4\uad70\uc740 1916\ub144 7\uc6d4\uc5d0\uc11c 11\uc6d4\uc5d0 \uac78\uccd0 \uacf5\uc138\ub97c \ud3bc\uccd0 \uaca8\uc6b8\ucbe4\uc5d0\ub294 \ub3c5\uc77c\uad70\uc744 \ubca0\ub974\ub42d-\ub791\uc2a4 \uc0ac\uc774\uc5d0 \uad6c\ucd95\ub41c \ubc29\uc5b4\uc120\uc778 \ud78c\ub374\ubd80\ub974\ud06c \uc120\uae4c\uc9c0 \ubc00\uc5b4\ubd99\uc600\ub2e4. \ub9cc\uc288\ud0c0\uc778 \ub300\uc704\ub294 1917\ub144 10\uc6d4\uae4c\uc9c0 \uacc4\uc18d \ubca8\ub85c\ube0c \uc218\ud558\ub85c \uc77c\ud558\ub2e4\uac00 \ub9ac\uac00\ub97c \uc810\ub839\uc911\uc778 \uc81c4\uae30\ubcd1\uc0ac\ub2e8 \ucc38\ubaa8\uc7a5\uc73c\ub85c \uc784\uba85\ubc1b\uc544 \uadf8\ub9ac\ub85c \uc62e\uaca8\uac14\ub2e4. 1918\ub144 3\uc6d4 \ube0c\ub808\uc2a4\ud2b8-\ub9ac\ud1a0\ud504\uc2a4\ud06c \uc870\uc57d\uc73c\ub85c \ub3d9\ubd80\uc804\uc120\uc774 \ub3c5\uc77c\uc758 \uc2b9\ub9ac\ub85c \ub05d\ub098\uc790 \ub9cc\uc288\ud0c0\uc778\uc758 \ubd80\ub300\ub294 \ub3d9\ubd80\uc804\uc120\uc5d0 \ub354 \uba38\ubb34\ub97c \ud544\uc694\uac00 \uc5c6\uc5b4\uc84c\uace0, \ub9cc\uc288\ud0c0\uc778\uc740 \ub2e4\uc2dc \ub7ad\uc2a4 \uadfc\uad50\uc758 \uc81c213\ubcf4\ubcd1\uc0ac\ub2e8\uc73c\ub85c \uac70\ucc98\ub97c \uc62e\uacbc\ub2e4. \ub3c5\uc77c\uad70\uc740 \uc11c\ubd80\uc804\uc120\uc5d0\uc11c \uc77c\ubd80 \uc131\uacfc\ub97c \uac70\ub450\uc5c8\uc73c\ub098 \uacb0\uad6d \uc804\uc7c1\uc5d0\uc11c \uc84c\uace0, 1918\ub144 11\uc6d4 11\uc77c \ud734\uc804\ud611\uc815\uc774 \uc870\uc778(1918\ub144 11\uc6d4 11\uc77c \ud734\uc804)\ub41c\ub2e4.", "paragraph_answer": "1915\ub144 6\uc6d4 17\uc77c \ub9cc\uc288\ud0c0\uc778\uc740 \ubcf5\uadc0\ud558\uc5ec \ub9c9\uc2a4 \ud3f0 \uac08\ube44\uce20\uac00 \uc9c0\ud718\ud558\ub294 \uc81c10\uad70 \uc791\uc804\ucc38\ubaa8\uc7a5\uad50\uac00 \ub418\uc5c8\ub2e4. \uc5bc\ub9c8 \ub4a4 \ub300\uc704\ub85c \uc2b9\uc9c4\ud55c \ub9cc\uc288\ud0c0\uc778\uc740 \uc81c10\uad70\uc774 \ud3f4\ub780\ub4dc, \ub9ac\ud22c\uc544\ub2c8\uc544, \ubaac\ud14c\ub124\uadf8\ub85c, \uc54c\ubc14\ub2c8\uc544\ub97c \uc131\uacf5\uc801\uc73c\ub85c \uacf5\uaca9\ud558\ub294 \uacfc\uc815\uc5d0\uc11c \uacf5\uc138\uc791\uc804 \uacc4\ud68d \ubc0f \uc218\ud589\uc744 \uacbd\ud5d8\uc73c\ub85c \uccb4\ub4dd\ud588\ub2e4. 1916\ub144 \ucd08 \ubca0\ub974\ub42d\uc5d0\uc11c \uacf5\uc138\uc791\uc804\uc744 \uc218\ud589\ud558\uace0 \uadf8 \ub4a4\uc5d0\ub294 \uc19c \uac15 \uadfc\ucc98\uc758 \uc0ac\ub839\ubd80\uc5d0\uc11c \ud504\ub9ac\uce20 \ud3f0 \ubca8\ub85c\ube0c \uc7a5\uad70\uacfc \ud504\ub9ac\uce20 \ud3f0 \ub85c\uc2a4\ubca0\ub974\ud06c \ucc38\ubaa8\uc7a5 \ud718\ud558\uc5d0 \ubcf4\uae09\uc7a5\uad50\ub85c \uc77c\ud588\ub2e4. \uc774 \uc77c\ub300\ub294 1\ucc28 \ub300\uc804 \ub0b4\ub0b4 \uac00\uc7a5 \uaca9\ub82c\ud55c \uc804\ud22c\uac00 \ubc8c\uc5b4\uc9c4 \uc804\uc7a5\uc774\uc5c8\ub2e4. \uc601\uad6d\uad70\uacfc \ud504\ub791\uc2a4\uad70\uc740 1916\ub144 7\uc6d4\uc5d0\uc11c 11\uc6d4\uc5d0 \uac78\uccd0 \uacf5\uc138\ub97c \ud3bc\uccd0 \uaca8\uc6b8\ucbe4\uc5d0\ub294 \ub3c5\uc77c\uad70\uc744 \ubca0\ub974\ub42d-\ub791\uc2a4 \uc0ac\uc774\uc5d0 \uad6c\ucd95\ub41c \ubc29\uc5b4\uc120\uc778 \ud78c\ub374\ubd80\ub974\ud06c \uc120\uae4c\uc9c0 \ubc00\uc5b4\ubd99\uc600\ub2e4. \ub9cc\uc288\ud0c0\uc778 \ub300\uc704\ub294 1917\ub144 10\uc6d4\uae4c\uc9c0 \uacc4\uc18d \ubca8\ub85c\ube0c \uc218\ud558\ub85c \uc77c\ud558\ub2e4\uac00 \ub9ac\uac00\ub97c \uc810\ub839\uc911\uc778 \uc81c4\uae30\ubcd1\uc0ac\ub2e8 \ucc38\ubaa8\uc7a5\uc73c\ub85c \uc784\uba85\ubc1b\uc544 \uadf8\ub9ac\ub85c \uc62e\uaca8\uac14\ub2e4. 1918\ub144 3\uc6d4 \ube0c\ub808\uc2a4\ud2b8-\ub9ac\ud1a0\ud504\uc2a4\ud06c \uc870\uc57d\uc73c\ub85c \ub3d9\ubd80\uc804\uc120\uc774 \ub3c5\uc77c\uc758 \uc2b9\ub9ac\ub85c \ub05d\ub098\uc790 \ub9cc\uc288\ud0c0\uc778\uc758 \ubd80\ub300\ub294 \ub3d9\ubd80\uc804\uc120\uc5d0 \ub354 \uba38\ubb34\ub97c \ud544\uc694\uac00 \uc5c6\uc5b4\uc84c\uace0, \ub9cc\uc288\ud0c0\uc778\uc740 \ub2e4\uc2dc \ub7ad\uc2a4 \uadfc\uad50\uc758 \uc81c213\ubcf4\ubcd1\uc0ac\ub2e8\uc73c\ub85c \uac70\ucc98\ub97c \uc62e\uacbc\ub2e4. \ub3c5\uc77c\uad70\uc740 \uc11c\ubd80\uc804\uc120\uc5d0\uc11c \uc77c\ubd80 \uc131\uacfc\ub97c \uac70\ub450\uc5c8\uc73c\ub098 \uacb0\uad6d \uc804\uc7c1\uc5d0\uc11c \uc84c\uace0, 1918\ub144 11\uc6d4 11\uc77c \ud734\uc804\ud611\uc815\uc774 \uc870\uc778(1918\ub144 11\uc6d4 11\uc77c \ud734\uc804)\ub41c\ub2e4.", "sentence_answer": "\ub9cc\uc288\ud0c0\uc778 \ub300\uc704\ub294 1917\ub144 10\uc6d4\uae4c\uc9c0 \uacc4\uc18d \ubca8\ub85c\ube0c \uc218\ud558\ub85c \uc77c\ud558\ub2e4\uac00 \ub9ac\uac00\ub97c \uc810\ub839\uc911\uc778 \uc81c4\uae30\ubcd1\uc0ac\ub2e8 \ucc38\ubaa8\uc7a5\uc73c\ub85c \uc784\uba85\ubc1b\uc544 \uadf8\ub9ac\ub85c \uc62e\uaca8\uac14\ub2e4.", "paragraph_id": "6540644-4-2", "paragraph_question": "question: \ub9cc\uc288\ud0c0\uc778 \ub300\uc704\ub294 \ubca8\ub85c\ube0c\uc758 \ubd80\ud558\ub85c \uc77c\ud558\ub2e4\uac00 \uc5b4\ub290 \uc0ac\ub2e8\uc758 \ucc38\ubaa8\uc7a5\uc73c\ub85c \uc784\uba85\ubc1b\ub098?, context: 1915\ub144 6\uc6d4 17\uc77c \ub9cc\uc288\ud0c0\uc778\uc740 \ubcf5\uadc0\ud558\uc5ec \ub9c9\uc2a4 \ud3f0 \uac08\ube44\uce20\uac00 \uc9c0\ud718\ud558\ub294 \uc81c10\uad70 \uc791\uc804\ucc38\ubaa8\uc7a5\uad50\uac00 \ub418\uc5c8\ub2e4. \uc5bc\ub9c8 \ub4a4 \ub300\uc704\ub85c \uc2b9\uc9c4\ud55c \ub9cc\uc288\ud0c0\uc778\uc740 \uc81c10\uad70\uc774 \ud3f4\ub780\ub4dc, \ub9ac\ud22c\uc544\ub2c8\uc544, \ubaac\ud14c\ub124\uadf8\ub85c, \uc54c\ubc14\ub2c8\uc544\ub97c \uc131\uacf5\uc801\uc73c\ub85c \uacf5\uaca9\ud558\ub294 \uacfc\uc815\uc5d0\uc11c \uacf5\uc138\uc791\uc804 \uacc4\ud68d \ubc0f \uc218\ud589\uc744 \uacbd\ud5d8\uc73c\ub85c \uccb4\ub4dd\ud588\ub2e4. 1916\ub144 \ucd08 \ubca0\ub974\ub42d\uc5d0\uc11c \uacf5\uc138\uc791\uc804\uc744 \uc218\ud589\ud558\uace0 \uadf8 \ub4a4\uc5d0\ub294 \uc19c \uac15 \uadfc\ucc98\uc758 \uc0ac\ub839\ubd80\uc5d0\uc11c \ud504\ub9ac\uce20 \ud3f0 \ubca8\ub85c\ube0c \uc7a5\uad70\uacfc \ud504\ub9ac\uce20 \ud3f0 \ub85c\uc2a4\ubca0\ub974\ud06c \ucc38\ubaa8\uc7a5 \ud718\ud558\uc5d0 \ubcf4\uae09\uc7a5\uad50\ub85c \uc77c\ud588\ub2e4. \uc774 \uc77c\ub300\ub294 1\ucc28 \ub300\uc804 \ub0b4\ub0b4 \uac00\uc7a5 \uaca9\ub82c\ud55c \uc804\ud22c\uac00 \ubc8c\uc5b4\uc9c4 \uc804\uc7a5\uc774\uc5c8\ub2e4. \uc601\uad6d\uad70\uacfc \ud504\ub791\uc2a4\uad70\uc740 1916\ub144 7\uc6d4\uc5d0\uc11c 11\uc6d4\uc5d0 \uac78\uccd0 \uacf5\uc138\ub97c \ud3bc\uccd0 \uaca8\uc6b8\ucbe4\uc5d0\ub294 \ub3c5\uc77c\uad70\uc744 \ubca0\ub974\ub42d-\ub791\uc2a4 \uc0ac\uc774\uc5d0 \uad6c\ucd95\ub41c \ubc29\uc5b4\uc120\uc778 \ud78c\ub374\ubd80\ub974\ud06c \uc120\uae4c\uc9c0 \ubc00\uc5b4\ubd99\uc600\ub2e4. \ub9cc\uc288\ud0c0\uc778 \ub300\uc704\ub294 1917\ub144 10\uc6d4\uae4c\uc9c0 \uacc4\uc18d \ubca8\ub85c\ube0c \uc218\ud558\ub85c \uc77c\ud558\ub2e4\uac00 \ub9ac\uac00\ub97c \uc810\ub839\uc911\uc778 \uc81c4\uae30\ubcd1\uc0ac\ub2e8 \ucc38\ubaa8\uc7a5\uc73c\ub85c \uc784\uba85\ubc1b\uc544 \uadf8\ub9ac\ub85c \uc62e\uaca8\uac14\ub2e4. 1918\ub144 3\uc6d4 \ube0c\ub808\uc2a4\ud2b8-\ub9ac\ud1a0\ud504\uc2a4\ud06c \uc870\uc57d\uc73c\ub85c \ub3d9\ubd80\uc804\uc120\uc774 \ub3c5\uc77c\uc758 \uc2b9\ub9ac\ub85c \ub05d\ub098\uc790 \ub9cc\uc288\ud0c0\uc778\uc758 \ubd80\ub300\ub294 \ub3d9\ubd80\uc804\uc120\uc5d0 \ub354 \uba38\ubb34\ub97c \ud544\uc694\uac00 \uc5c6\uc5b4\uc84c\uace0, \ub9cc\uc288\ud0c0\uc778\uc740 \ub2e4\uc2dc \ub7ad\uc2a4 \uadfc\uad50\uc758 \uc81c213\ubcf4\ubcd1\uc0ac\ub2e8\uc73c\ub85c \uac70\ucc98\ub97c \uc62e\uacbc\ub2e4. \ub3c5\uc77c\uad70\uc740 \uc11c\ubd80\uc804\uc120\uc5d0\uc11c \uc77c\ubd80 \uc131\uacfc\ub97c \uac70\ub450\uc5c8\uc73c\ub098 \uacb0\uad6d \uc804\uc7c1\uc5d0\uc11c \uc84c\uace0, 1918\ub144 11\uc6d4 11\uc77c \ud734\uc804\ud611\uc815\uc774 \uc870\uc778(1918\ub144 11\uc6d4 11\uc77c \ud734\uc804)\ub41c\ub2e4."} {"question": "\uc601\uad6d \uce21 \uc694\uc6d0\uc73c\ub85c \ub85c\uc2a4\uc568\ub7ec\ubaa8\uc2a4\uc758 \ud575\uc2ec \ubd84\uc57c\uc5d0 \uadfc\ubb34\ud558\uba74\uc11c \uc18c\ub828\uc5d0 \uac00\uc7a5 \ud070 \uc815\ubcf4\ub97c \ub118\uae34 \ucca9\ubcf4\uc6d0\uc740 \ub204\uad6c\uc778\uac00?", "paragraph": "\uad00\ub828 \uc778\uc6d0\uc774 \ub9e4\uc6b0 \ub9ce\uc558\uae30 \ub54c\ubb38\uc5d0 \ubcf4\uc548 \uc720\uc9c0\ub294 \uc5b4\ub824\uc6b4 \uc77c\uc774\uc5c8\ub2e4. \uacc4\ud68d\uc758 \ubcf4\uc548 \uc720\uc9c0\ub97c \uc704\ud574 \ubc29\ucca9\ub300\uac00 \uc6b4\uc6a9\ub418\uc5c8\ub2e4. 1943\ub144, \uc18c\ub828\uc774 \ub9e8\ud574\ud2bc \uacc4\ud68d\uc758 \uae30\uc220\uc744 \ube7c\ub0b4\ub824 \ud55c\ub2e4\ub294 \uc810\uc774 \ud655\uc2e4\ud574\uc84c\ub2e4. \uc11c\ubd80 \ubc29\uc704 \uc0ac\ub839\ubd80 \uc18c\uc18d\uc758 \ubc29\ucca9\ub300\uc5d0 \uadfc\ubb34\ud558\ub358 \ubcf4\ub9ac\uc2a4 T. \ud328\uc26c \uc18c\ub839\uc740 \uc624\ud39c\ud558\uc774\uba38\uc758 \ucd94\ucc9c\uc73c\ub85c \ubc84\ud074\ub9ac \ub300\ud559\uad50\uc758 \ubc29\uc0ac\uc120\uc5f0\uad6c\uc18c\uc5d0\uc11c \uadfc\ubb34\ud558\ub358 \ud558\ucf58 \uccb4\ubc1c\ub9ac\uc5b4\uac00 \uc18c\ub828\uc5d0\uac8c \uc815\ubcf4\ub97c \uc804\ub2ec\ud588\ub2e4\ub294 \uac83\uc744 \uc801\ubc1c\ud558\uc600\ub2e4. \uc18c\ub828\uc5d0 \uac00\uc7a5 \ud070 \uc815\ubcf4\ub97c \ub118\uae34 \ucca9\ubcf4\uc6d0\uc740 \uc601\uad6d \uce21 \uc694\uc6d0\uc73c\ub85c \ub85c\uc2a4\uc568\ub7ec\ubaa8\uc2a4\uc5d0\uc11c \ud575\uc2ec \ubd84\uc57c\uc5d0 \uadfc\ubb34\ud558\ub358 \ud074\ub77c\uc6b0\uc2a4 \ud37c\uce58\uc2a4\uc600\ub2e4. \ud074\ub77c\uc6b0\uc2a4 \ud37c\uce58\uc2a4\uc758 \uac04\ucca9\ud589\uc704\ub294 1950\ub144\uc5d0 \ud3ed\ub85c\ub418\uc5c8\uace0, \uc774\uac83\uc740 \ubbf8\uad6d, \uc601\uad6d, \uce90\ub098\ub2e4 \uc0ac\uc774\uc758 \ud575\ud611\ub825\uc5d0 \uc7a5\uc560 \uc694\uc778\uc73c\ub85c \uc791\uc6a9\ud558\uc600\ub2e4. \uc774 \uc678\uc5d0\ub3c4 \ud5e8\ub9ac \uace8\ub4dc, \ub370\uc774\ube44\ub4dc \uadf8\ub9b0\uae00\ub798\uc2a4, \uc5d0\ub378 \ub85c\uc820\ubc84\uadf8\uc640 \uc904\ub9ac\uc5b4\uc2a4 \ub85c\uc820\ubc84\uadf8 \ub4f1\uc774 \ucca9\ubcf4\uc6d0\uc73c\ub85c \ubc1d\ud600\uc84c\ub2e4. \uadf8\ub7ec\ub098, \uc870\uc9c0 \ucf54\ubc14\uc640 \uac19\uc740 \ub2e4\ub978 \ucca9\ubcf4\uc6d0\ub4e4\uc740 \uc804\ud6c4\uc5d0\ub3c4 \uc624\ub7ab\ub3d9\uc548 \ubc1c\uac01\ub418\uc9c0 \uc54a\uae30\ub3c4 \ud558\uc600\ub2e4. \uc18c\ub828\uc740 \uc774\ub807\uac8c \ubaa8\uc544\uc9c4 \uc815\ubcf4\ub4e4\uc744 \ubc14\ud0d5\uc73c\ub85c \uc18c\ube44\uc5d0\ud2b8 \ud575\ud3ed\ud0c4 \uacc4\ud68d\uc744 \uc55e\ub2f9\uae38 \uc218 \uc788\uc5c8\ub2e4.", "answer": "\ud074\ub77c\uc6b0\uc2a4 \ud37c\uce58\uc2a4", "sentence": "\uc18c\ub828\uc5d0 \uac00\uc7a5 \ud070 \uc815\ubcf4\ub97c \ub118\uae34 \ucca9\ubcf4\uc6d0\uc740 \uc601\uad6d \uce21 \uc694\uc6d0\uc73c\ub85c \ub85c\uc2a4\uc568\ub7ec\ubaa8\uc2a4\uc5d0\uc11c \ud575\uc2ec \ubd84\uc57c\uc5d0 \uadfc\ubb34\ud558\ub358 \ud074\ub77c\uc6b0\uc2a4 \ud37c\uce58\uc2a4 \uc600\ub2e4.", "paragraph_sentence": "\uad00\ub828 \uc778\uc6d0\uc774 \ub9e4\uc6b0 \ub9ce\uc558\uae30 \ub54c\ubb38\uc5d0 \ubcf4\uc548 \uc720\uc9c0\ub294 \uc5b4\ub824\uc6b4 \uc77c\uc774\uc5c8\ub2e4. \uacc4\ud68d\uc758 \ubcf4\uc548 \uc720\uc9c0\ub97c \uc704\ud574 \ubc29\ucca9\ub300\uac00 \uc6b4\uc6a9\ub418\uc5c8\ub2e4. 1943\ub144, \uc18c\ub828\uc774 \ub9e8\ud574\ud2bc \uacc4\ud68d\uc758 \uae30\uc220\uc744 \ube7c\ub0b4\ub824 \ud55c\ub2e4\ub294 \uc810\uc774 \ud655\uc2e4\ud574\uc84c\ub2e4. \uc11c\ubd80 \ubc29\uc704 \uc0ac\ub839\ubd80 \uc18c\uc18d\uc758 \ubc29\ucca9\ub300\uc5d0 \uadfc\ubb34\ud558\ub358 \ubcf4\ub9ac\uc2a4 T. \ud328\uc26c \uc18c\ub839\uc740 \uc624\ud39c\ud558\uc774\uba38\uc758 \ucd94\ucc9c\uc73c\ub85c \ubc84\ud074\ub9ac \ub300\ud559\uad50\uc758 \ubc29\uc0ac\uc120\uc5f0\uad6c\uc18c\uc5d0\uc11c \uadfc\ubb34\ud558\ub358 \ud558\ucf58 \uccb4\ubc1c\ub9ac\uc5b4\uac00 \uc18c\ub828\uc5d0\uac8c \uc815\ubcf4\ub97c \uc804\ub2ec\ud588\ub2e4\ub294 \uac83\uc744 \uc801\ubc1c\ud558\uc600\ub2e4. \uc18c\ub828\uc5d0 \uac00\uc7a5 \ud070 \uc815\ubcf4\ub97c \ub118\uae34 \ucca9\ubcf4\uc6d0\uc740 \uc601\uad6d \uce21 \uc694\uc6d0\uc73c\ub85c \ub85c\uc2a4\uc568\ub7ec\ubaa8\uc2a4\uc5d0\uc11c \ud575\uc2ec \ubd84\uc57c\uc5d0 \uadfc\ubb34\ud558\ub358 \ud074\ub77c\uc6b0\uc2a4 \ud37c\uce58\uc2a4 \uc600\ub2e4. \ud074\ub77c\uc6b0\uc2a4 \ud37c\uce58\uc2a4\uc758 \uac04\ucca9\ud589\uc704\ub294 1950\ub144\uc5d0 \ud3ed\ub85c\ub418\uc5c8\uace0, \uc774\uac83\uc740 \ubbf8\uad6d, \uc601\uad6d, \uce90\ub098\ub2e4 \uc0ac\uc774\uc758 \ud575\ud611\ub825\uc5d0 \uc7a5\uc560 \uc694\uc778\uc73c\ub85c \uc791\uc6a9\ud558\uc600\ub2e4. \uc774 \uc678\uc5d0\ub3c4 \ud5e8\ub9ac \uace8\ub4dc, \ub370\uc774\ube44\ub4dc \uadf8\ub9b0\uae00\ub798\uc2a4, \uc5d0\ub378 \ub85c\uc820\ubc84\uadf8\uc640 \uc904\ub9ac\uc5b4\uc2a4 \ub85c\uc820\ubc84\uadf8 \ub4f1\uc774 \ucca9\ubcf4\uc6d0\uc73c\ub85c \ubc1d\ud600\uc84c\ub2e4. \uadf8\ub7ec\ub098, \uc870\uc9c0 \ucf54\ubc14\uc640 \uac19\uc740 \ub2e4\ub978 \ucca9\ubcf4\uc6d0\ub4e4\uc740 \uc804\ud6c4\uc5d0\ub3c4 \uc624\ub7ab\ub3d9\uc548 \ubc1c\uac01\ub418\uc9c0 \uc54a\uae30\ub3c4 \ud558\uc600\ub2e4. \uc18c\ub828\uc740 \uc774\ub807\uac8c \ubaa8\uc544\uc9c4 \uc815\ubcf4\ub4e4\uc744 \ubc14\ud0d5\uc73c\ub85c \uc18c\ube44\uc5d0\ud2b8 \ud575\ud3ed\ud0c4 \uacc4\ud68d\uc744 \uc55e\ub2f9\uae38 \uc218 \uc788\uc5c8\ub2e4.", "paragraph_answer": "\uad00\ub828 \uc778\uc6d0\uc774 \ub9e4\uc6b0 \ub9ce\uc558\uae30 \ub54c\ubb38\uc5d0 \ubcf4\uc548 \uc720\uc9c0\ub294 \uc5b4\ub824\uc6b4 \uc77c\uc774\uc5c8\ub2e4. \uacc4\ud68d\uc758 \ubcf4\uc548 \uc720\uc9c0\ub97c \uc704\ud574 \ubc29\ucca9\ub300\uac00 \uc6b4\uc6a9\ub418\uc5c8\ub2e4. 1943\ub144, \uc18c\ub828\uc774 \ub9e8\ud574\ud2bc \uacc4\ud68d\uc758 \uae30\uc220\uc744 \ube7c\ub0b4\ub824 \ud55c\ub2e4\ub294 \uc810\uc774 \ud655\uc2e4\ud574\uc84c\ub2e4. \uc11c\ubd80 \ubc29\uc704 \uc0ac\ub839\ubd80 \uc18c\uc18d\uc758 \ubc29\ucca9\ub300\uc5d0 \uadfc\ubb34\ud558\ub358 \ubcf4\ub9ac\uc2a4 T. \ud328\uc26c \uc18c\ub839\uc740 \uc624\ud39c\ud558\uc774\uba38\uc758 \ucd94\ucc9c\uc73c\ub85c \ubc84\ud074\ub9ac \ub300\ud559\uad50\uc758 \ubc29\uc0ac\uc120\uc5f0\uad6c\uc18c\uc5d0\uc11c \uadfc\ubb34\ud558\ub358 \ud558\ucf58 \uccb4\ubc1c\ub9ac\uc5b4\uac00 \uc18c\ub828\uc5d0\uac8c \uc815\ubcf4\ub97c \uc804\ub2ec\ud588\ub2e4\ub294 \uac83\uc744 \uc801\ubc1c\ud558\uc600\ub2e4. \uc18c\ub828\uc5d0 \uac00\uc7a5 \ud070 \uc815\ubcf4\ub97c \ub118\uae34 \ucca9\ubcf4\uc6d0\uc740 \uc601\uad6d \uce21 \uc694\uc6d0\uc73c\ub85c \ub85c\uc2a4\uc568\ub7ec\ubaa8\uc2a4\uc5d0\uc11c \ud575\uc2ec \ubd84\uc57c\uc5d0 \uadfc\ubb34\ud558\ub358 \ud074\ub77c\uc6b0\uc2a4 \ud37c\uce58\uc2a4 \uc600\ub2e4. \ud074\ub77c\uc6b0\uc2a4 \ud37c\uce58\uc2a4\uc758 \uac04\ucca9\ud589\uc704\ub294 1950\ub144\uc5d0 \ud3ed\ub85c\ub418\uc5c8\uace0, \uc774\uac83\uc740 \ubbf8\uad6d, \uc601\uad6d, \uce90\ub098\ub2e4 \uc0ac\uc774\uc758 \ud575\ud611\ub825\uc5d0 \uc7a5\uc560 \uc694\uc778\uc73c\ub85c \uc791\uc6a9\ud558\uc600\ub2e4. \uc774 \uc678\uc5d0\ub3c4 \ud5e8\ub9ac \uace8\ub4dc, \ub370\uc774\ube44\ub4dc \uadf8\ub9b0\uae00\ub798\uc2a4, \uc5d0\ub378 \ub85c\uc820\ubc84\uadf8\uc640 \uc904\ub9ac\uc5b4\uc2a4 \ub85c\uc820\ubc84\uadf8 \ub4f1\uc774 \ucca9\ubcf4\uc6d0\uc73c\ub85c \ubc1d\ud600\uc84c\ub2e4. \uadf8\ub7ec\ub098, \uc870\uc9c0 \ucf54\ubc14\uc640 \uac19\uc740 \ub2e4\ub978 \ucca9\ubcf4\uc6d0\ub4e4\uc740 \uc804\ud6c4\uc5d0\ub3c4 \uc624\ub7ab\ub3d9\uc548 \ubc1c\uac01\ub418\uc9c0 \uc54a\uae30\ub3c4 \ud558\uc600\ub2e4. \uc18c\ub828\uc740 \uc774\ub807\uac8c \ubaa8\uc544\uc9c4 \uc815\ubcf4\ub4e4\uc744 \ubc14\ud0d5\uc73c\ub85c \uc18c\ube44\uc5d0\ud2b8 \ud575\ud3ed\ud0c4 \uacc4\ud68d\uc744 \uc55e\ub2f9\uae38 \uc218 \uc788\uc5c8\ub2e4.", "sentence_answer": "\uc18c\ub828\uc5d0 \uac00\uc7a5 \ud070 \uc815\ubcf4\ub97c \ub118\uae34 \ucca9\ubcf4\uc6d0\uc740 \uc601\uad6d \uce21 \uc694\uc6d0\uc73c\ub85c \ub85c\uc2a4\uc568\ub7ec\ubaa8\uc2a4\uc5d0\uc11c \ud575\uc2ec \ubd84\uc57c\uc5d0 \uadfc\ubb34\ud558\ub358 \ud074\ub77c\uc6b0\uc2a4 \ud37c\uce58\uc2a4 \uc600\ub2e4.", "paragraph_id": "6075564-21-2", "paragraph_question": "question: \uc601\uad6d \uce21 \uc694\uc6d0\uc73c\ub85c \ub85c\uc2a4\uc568\ub7ec\ubaa8\uc2a4\uc758 \ud575\uc2ec \ubd84\uc57c\uc5d0 \uadfc\ubb34\ud558\uba74\uc11c \uc18c\ub828\uc5d0 \uac00\uc7a5 \ud070 \uc815\ubcf4\ub97c \ub118\uae34 \ucca9\ubcf4\uc6d0\uc740 \ub204\uad6c\uc778\uac00?, context: \uad00\ub828 \uc778\uc6d0\uc774 \ub9e4\uc6b0 \ub9ce\uc558\uae30 \ub54c\ubb38\uc5d0 \ubcf4\uc548 \uc720\uc9c0\ub294 \uc5b4\ub824\uc6b4 \uc77c\uc774\uc5c8\ub2e4. \uacc4\ud68d\uc758 \ubcf4\uc548 \uc720\uc9c0\ub97c \uc704\ud574 \ubc29\ucca9\ub300\uac00 \uc6b4\uc6a9\ub418\uc5c8\ub2e4. 1943\ub144, \uc18c\ub828\uc774 \ub9e8\ud574\ud2bc \uacc4\ud68d\uc758 \uae30\uc220\uc744 \ube7c\ub0b4\ub824 \ud55c\ub2e4\ub294 \uc810\uc774 \ud655\uc2e4\ud574\uc84c\ub2e4. \uc11c\ubd80 \ubc29\uc704 \uc0ac\ub839\ubd80 \uc18c\uc18d\uc758 \ubc29\ucca9\ub300\uc5d0 \uadfc\ubb34\ud558\ub358 \ubcf4\ub9ac\uc2a4 T. \ud328\uc26c \uc18c\ub839\uc740 \uc624\ud39c\ud558\uc774\uba38\uc758 \ucd94\ucc9c\uc73c\ub85c \ubc84\ud074\ub9ac \ub300\ud559\uad50\uc758 \ubc29\uc0ac\uc120\uc5f0\uad6c\uc18c\uc5d0\uc11c \uadfc\ubb34\ud558\ub358 \ud558\ucf58 \uccb4\ubc1c\ub9ac\uc5b4\uac00 \uc18c\ub828\uc5d0\uac8c \uc815\ubcf4\ub97c \uc804\ub2ec\ud588\ub2e4\ub294 \uac83\uc744 \uc801\ubc1c\ud558\uc600\ub2e4. \uc18c\ub828\uc5d0 \uac00\uc7a5 \ud070 \uc815\ubcf4\ub97c \ub118\uae34 \ucca9\ubcf4\uc6d0\uc740 \uc601\uad6d \uce21 \uc694\uc6d0\uc73c\ub85c \ub85c\uc2a4\uc568\ub7ec\ubaa8\uc2a4\uc5d0\uc11c \ud575\uc2ec \ubd84\uc57c\uc5d0 \uadfc\ubb34\ud558\ub358 \ud074\ub77c\uc6b0\uc2a4 \ud37c\uce58\uc2a4\uc600\ub2e4. \ud074\ub77c\uc6b0\uc2a4 \ud37c\uce58\uc2a4\uc758 \uac04\ucca9\ud589\uc704\ub294 1950\ub144\uc5d0 \ud3ed\ub85c\ub418\uc5c8\uace0, \uc774\uac83\uc740 \ubbf8\uad6d, \uc601\uad6d, \uce90\ub098\ub2e4 \uc0ac\uc774\uc758 \ud575\ud611\ub825\uc5d0 \uc7a5\uc560 \uc694\uc778\uc73c\ub85c \uc791\uc6a9\ud558\uc600\ub2e4. \uc774 \uc678\uc5d0\ub3c4 \ud5e8\ub9ac \uace8\ub4dc, \ub370\uc774\ube44\ub4dc \uadf8\ub9b0\uae00\ub798\uc2a4, \uc5d0\ub378 \ub85c\uc820\ubc84\uadf8\uc640 \uc904\ub9ac\uc5b4\uc2a4 \ub85c\uc820\ubc84\uadf8 \ub4f1\uc774 \ucca9\ubcf4\uc6d0\uc73c\ub85c \ubc1d\ud600\uc84c\ub2e4. \uadf8\ub7ec\ub098, \uc870\uc9c0 \ucf54\ubc14\uc640 \uac19\uc740 \ub2e4\ub978 \ucca9\ubcf4\uc6d0\ub4e4\uc740 \uc804\ud6c4\uc5d0\ub3c4 \uc624\ub7ab\ub3d9\uc548 \ubc1c\uac01\ub418\uc9c0 \uc54a\uae30\ub3c4 \ud558\uc600\ub2e4. \uc18c\ub828\uc740 \uc774\ub807\uac8c \ubaa8\uc544\uc9c4 \uc815\ubcf4\ub4e4\uc744 \ubc14\ud0d5\uc73c\ub85c \uc18c\ube44\uc5d0\ud2b8 \ud575\ud3ed\ud0c4 \uacc4\ud68d\uc744 \uc55e\ub2f9\uae38 \uc218 \uc788\uc5c8\ub2e4."} {"question": "\uc77c\ubcf8\uc740 \ub300\ud55c\uc81c\uad6d\uc744 \uba87\ub144\ub3d9\uc548 \ud1b5\uce58\ud558\uc600\ub294\uac00?", "paragraph": "\ud55c\ubc18\ub3c4\uc640 \uc77c\ubcf8 \uc5f4\ub3c4\ub294 \uace0\ub300\ubd80\ud130 \uac01\uc885 \ubb38\ud654\uc758 \uc804\ud30c\uc640 \ubb34\uc5ed\uc744 \ube44\ub86f\ud55c \ub9ce\uc740 \uad50\ub958\ub97c \uc774\uc5b4\uc654\ub2e4. \ud2b9\ud788 \ubc31\uc81c\ub294 \uace0\ub300 \uc77c\ubcf8\uc5d0 \uc911\uad6d \ubb38\ubb3c\uc744 \ub9ce\uc774 \uc804\ud30c\ud558\uc600\ub2e4. \uace0\ub824 \uc2dc\ub300 \ud6c4\ubc18\uacfc \uc870\uc120 \uc2dc\ub300 \uc804\ubc18\uc5d0 \uac78\uccd0\uc11c\ub294 \uc4f0\uc2dc\ub9c8 \uc12c\uacfc \uaddc\uc288\ub97c \uadfc\uac70\uc9c0\ub85c \ud558\ub294 \uc65c\uad6c(\u502d\u5bc7)\uac00 \uac01\uc885 \uc57d\ud0c8\uc744 \uc77c\uc0bc\uc558\uc73c\uba70 \uc774\ub85c \uc778\ud574 \uc870\uc120 \uc2dc\ub300 \uc911\ubc18\uc5d0 \uc784\uc9c4\uc65c\ub780\uc774 \uc77c\uc5b4\ub098\ub294 \ub4f1 \uc911\uc138\uc640 \uadfc\uc138\uae30 \ud55c\uc77c \uad00\uacc4\ub294 \uae09\uc18d\ub3c4\ub85c \ub0c9\uac01\ub418\uae30\ub3c4 \ud558\uc600\uc73c\ub098, \uc784\uc9c4\uc65c\ub780 \uc774\ud6c4 \uc870\uc120\uc758 \ud1b5\uc2e0\uc0ac \ud30c\uacac\uc73c\ub85c \ub2e4\uc2dc \ud55c\uc77c \uad50\ub958\uac00 \uc7ac\uac1c\ub418\uc5c8\ub2e4. \uadf8\ub7ec\ub098 \uba54\uc774\uc9c0 \uc720\uc2e0 \uc774\ud6c4\uc5d0 \ub4f1\uc7a5\ud55c \uc77c\ubcf8 \uc81c\uad6d\uc774 1910\ub144 \ub300\ud55c\uc81c\uad6d\uc744 \ubb34\ub825\uc73c\ub85c \ubcd1\ud569\ud55c \ub4a4 1945\ub144\uae4c\uc9c0 35\ub144 \ub3d9\uc548 \ud3ed\uc545\ud55c \uc2dd\ubbfc \ud1b5\uce58\ub97c \ud589\ud558\uace0 \ud604\ub300\uc5d0\ub3c4 \uc77c\ubcf8\uc758 \uc5ec\ub7ec \uc6b0\uc775 \uc778\uc0ac\ub4e4\uc774 \ud55c\uad6d\uc5d0 \ub300\ud55c \uc815\uce58\uc801 \ub3c4\ubc1c\uc744 \uc8fc\ucc3d\ud558\uba74\uc11c \uc591\uad6d\uc758 \uad6d\ubbfc \uac10\uc815\uc774 \uadf9\ub3c4\ub85c \uc545\ud654\ub418\uc5c8\ub2e4. \uc77c\ubcf8\uc5d0\uc11c\ub3c4 \ubc18\ud55c\uac10\uc815\uc744 \uac00\uc9c4 \uc77c\ubd80 \uc77c\ubcf8\uc778\ub4e4\uc774 \ubc18\ud55c\uc2dc\uc704\ub97c \ud558\uace0 \uc788\uc73c\uba70, \ud604\uc7ac \ub300\ud55c\ubbfc\uad6d\uacfc \uc77c\ubcf8\uc740 \ub3c5\ub3c4 \ubd84\uc7c1\uacfc \ub3d9\ud574\uc758 \uc774\ub984\uc5d0 \ub300\ud55c \ubd84\uc7c1 \ub4f1\uc758 \uacfc\uc81c\ub97c \uc548\uace0 \uc788\ub2e4. 2009\ub144\uc5d0 \uc77c\ubcf8 \ubbfc\uc8fc\ub2f9 \ud558\ud1a0\uc57c\ub9c8 \uc720\ud0a4\uc624 \ub0b4\uac01\uc774 \uc131\ub9bd\ud55c \uc774\ud6c4 \uc774\ub8e8\uc5b4\uc9c4 \uc591\uad6d\ubbfc \uc124\ubb38\uc870\uc0ac\uc5d0\uc11c \uc751\ub2f5\uc790\uc758 \ub2e4\uc218\uac00 \ub354 \uc545\ud654\ub41c \ud55c\uc77c \uad00\uacc4\uc5d0 \ubcc0\ud654\uac00 \uc5c6\uc744 \uac83\uc774\ub77c \uc751\ub2f5\ud55c \uacb0\uacfc\uac00 \ub098\uc624\uae30\ub3c4 \ud558\uc600\ub2e4. \uc704\uc548\ubd80\uc5d0 \ub300\ud55c \uc5ed\uc0ac\uc65c\uace1\uc73c\ub85c \uc77c\ubcf8\uc815\ubd80\ub294 \uc720\uc5d4\uc73c\ub85c\ubd80\ud130 \uacbd\uace0\ub97c \ubc1b\uc558\ub2e4.", "answer": "35\ub144", "sentence": "\uadf8\ub7ec\ub098 \uba54\uc774\uc9c0 \uc720\uc2e0 \uc774\ud6c4\uc5d0 \ub4f1\uc7a5\ud55c \uc77c\ubcf8 \uc81c\uad6d\uc774 1910\ub144 \ub300\ud55c\uc81c\uad6d\uc744 \ubb34\ub825\uc73c\ub85c \ubcd1\ud569\ud55c \ub4a4 1945\ub144\uae4c\uc9c0 35\ub144 \ub3d9\uc548 \ud3ed\uc545\ud55c \uc2dd\ubbfc \ud1b5\uce58\ub97c \ud589\ud558\uace0 \ud604\ub300\uc5d0\ub3c4 \uc77c\ubcf8\uc758 \uc5ec\ub7ec \uc6b0\uc775 \uc778\uc0ac\ub4e4\uc774 \ud55c\uad6d\uc5d0 \ub300\ud55c \uc815\uce58\uc801 \ub3c4\ubc1c\uc744 \uc8fc\ucc3d\ud558\uba74\uc11c \uc591\uad6d\uc758 \uad6d\ubbfc \uac10\uc815\uc774 \uadf9\ub3c4\ub85c \uc545\ud654\ub418\uc5c8\ub2e4.", "paragraph_sentence": "\ud55c\ubc18\ub3c4\uc640 \uc77c\ubcf8 \uc5f4\ub3c4\ub294 \uace0\ub300\ubd80\ud130 \uac01\uc885 \ubb38\ud654\uc758 \uc804\ud30c\uc640 \ubb34\uc5ed\uc744 \ube44\ub86f\ud55c \ub9ce\uc740 \uad50\ub958\ub97c \uc774\uc5b4\uc654\ub2e4. \ud2b9\ud788 \ubc31\uc81c\ub294 \uace0\ub300 \uc77c\ubcf8\uc5d0 \uc911\uad6d \ubb38\ubb3c\uc744 \ub9ce\uc774 \uc804\ud30c\ud558\uc600\ub2e4. \uace0\ub824 \uc2dc\ub300 \ud6c4\ubc18\uacfc \uc870\uc120 \uc2dc\ub300 \uc804\ubc18\uc5d0 \uac78\uccd0\uc11c\ub294 \uc4f0\uc2dc\ub9c8 \uc12c\uacfc \uaddc\uc288\ub97c \uadfc\uac70\uc9c0\ub85c \ud558\ub294 \uc65c\uad6c(\u502d\u5bc7)\uac00 \uac01\uc885 \uc57d\ud0c8\uc744 \uc77c\uc0bc\uc558\uc73c\uba70 \uc774\ub85c \uc778\ud574 \uc870\uc120 \uc2dc\ub300 \uc911\ubc18\uc5d0 \uc784\uc9c4\uc65c\ub780\uc774 \uc77c\uc5b4\ub098\ub294 \ub4f1 \uc911\uc138\uc640 \uadfc\uc138\uae30 \ud55c\uc77c \uad00\uacc4\ub294 \uae09\uc18d\ub3c4\ub85c \ub0c9\uac01\ub418\uae30\ub3c4 \ud558\uc600\uc73c\ub098, \uc784\uc9c4\uc65c\ub780 \uc774\ud6c4 \uc870\uc120\uc758 \ud1b5\uc2e0\uc0ac \ud30c\uacac\uc73c\ub85c \ub2e4\uc2dc \ud55c\uc77c \uad50\ub958\uac00 \uc7ac\uac1c\ub418\uc5c8\ub2e4. \uadf8\ub7ec\ub098 \uba54\uc774\uc9c0 \uc720\uc2e0 \uc774\ud6c4\uc5d0 \ub4f1\uc7a5\ud55c \uc77c\ubcf8 \uc81c\uad6d\uc774 1910\ub144 \ub300\ud55c\uc81c\uad6d\uc744 \ubb34\ub825\uc73c\ub85c \ubcd1\ud569\ud55c \ub4a4 1945\ub144\uae4c\uc9c0 35\ub144 \ub3d9\uc548 \ud3ed\uc545\ud55c \uc2dd\ubbfc \ud1b5\uce58\ub97c \ud589\ud558\uace0 \ud604\ub300\uc5d0\ub3c4 \uc77c\ubcf8\uc758 \uc5ec\ub7ec \uc6b0\uc775 \uc778\uc0ac\ub4e4\uc774 \ud55c\uad6d\uc5d0 \ub300\ud55c \uc815\uce58\uc801 \ub3c4\ubc1c\uc744 \uc8fc\ucc3d\ud558\uba74\uc11c \uc591\uad6d\uc758 \uad6d\ubbfc \uac10\uc815\uc774 \uadf9\ub3c4\ub85c \uc545\ud654\ub418\uc5c8\ub2e4. \uc77c\ubcf8\uc5d0\uc11c\ub3c4 \ubc18\ud55c\uac10\uc815\uc744 \uac00\uc9c4 \uc77c\ubd80 \uc77c\ubcf8\uc778\ub4e4\uc774 \ubc18\ud55c\uc2dc\uc704\ub97c \ud558\uace0 \uc788\uc73c\uba70, \ud604\uc7ac \ub300\ud55c\ubbfc\uad6d\uacfc \uc77c\ubcf8\uc740 \ub3c5\ub3c4 \ubd84\uc7c1\uacfc \ub3d9\ud574\uc758 \uc774\ub984\uc5d0 \ub300\ud55c \ubd84\uc7c1 \ub4f1\uc758 \uacfc\uc81c\ub97c \uc548\uace0 \uc788\ub2e4. 2009\ub144\uc5d0 \uc77c\ubcf8 \ubbfc\uc8fc\ub2f9 \ud558\ud1a0\uc57c\ub9c8 \uc720\ud0a4\uc624 \ub0b4\uac01\uc774 \uc131\ub9bd\ud55c \uc774\ud6c4 \uc774\ub8e8\uc5b4\uc9c4 \uc591\uad6d\ubbfc \uc124\ubb38\uc870\uc0ac\uc5d0\uc11c \uc751\ub2f5\uc790\uc758 \ub2e4\uc218\uac00 \ub354 \uc545\ud654\ub41c \ud55c\uc77c \uad00\uacc4\uc5d0 \ubcc0\ud654\uac00 \uc5c6\uc744 \uac83\uc774\ub77c \uc751\ub2f5\ud55c \uacb0\uacfc\uac00 \ub098\uc624\uae30\ub3c4 \ud558\uc600\ub2e4. \uc704\uc548\ubd80\uc5d0 \ub300\ud55c \uc5ed\uc0ac\uc65c\uace1\uc73c\ub85c \uc77c\ubcf8\uc815\ubd80\ub294 \uc720\uc5d4\uc73c\ub85c\ubd80\ud130 \uacbd\uace0\ub97c \ubc1b\uc558\ub2e4.", "paragraph_answer": "\ud55c\ubc18\ub3c4\uc640 \uc77c\ubcf8 \uc5f4\ub3c4\ub294 \uace0\ub300\ubd80\ud130 \uac01\uc885 \ubb38\ud654\uc758 \uc804\ud30c\uc640 \ubb34\uc5ed\uc744 \ube44\ub86f\ud55c \ub9ce\uc740 \uad50\ub958\ub97c \uc774\uc5b4\uc654\ub2e4. \ud2b9\ud788 \ubc31\uc81c\ub294 \uace0\ub300 \uc77c\ubcf8\uc5d0 \uc911\uad6d \ubb38\ubb3c\uc744 \ub9ce\uc774 \uc804\ud30c\ud558\uc600\ub2e4. \uace0\ub824 \uc2dc\ub300 \ud6c4\ubc18\uacfc \uc870\uc120 \uc2dc\ub300 \uc804\ubc18\uc5d0 \uac78\uccd0\uc11c\ub294 \uc4f0\uc2dc\ub9c8 \uc12c\uacfc \uaddc\uc288\ub97c \uadfc\uac70\uc9c0\ub85c \ud558\ub294 \uc65c\uad6c(\u502d\u5bc7)\uac00 \uac01\uc885 \uc57d\ud0c8\uc744 \uc77c\uc0bc\uc558\uc73c\uba70 \uc774\ub85c \uc778\ud574 \uc870\uc120 \uc2dc\ub300 \uc911\ubc18\uc5d0 \uc784\uc9c4\uc65c\ub780\uc774 \uc77c\uc5b4\ub098\ub294 \ub4f1 \uc911\uc138\uc640 \uadfc\uc138\uae30 \ud55c\uc77c \uad00\uacc4\ub294 \uae09\uc18d\ub3c4\ub85c \ub0c9\uac01\ub418\uae30\ub3c4 \ud558\uc600\uc73c\ub098, \uc784\uc9c4\uc65c\ub780 \uc774\ud6c4 \uc870\uc120\uc758 \ud1b5\uc2e0\uc0ac \ud30c\uacac\uc73c\ub85c \ub2e4\uc2dc \ud55c\uc77c \uad50\ub958\uac00 \uc7ac\uac1c\ub418\uc5c8\ub2e4. \uadf8\ub7ec\ub098 \uba54\uc774\uc9c0 \uc720\uc2e0 \uc774\ud6c4\uc5d0 \ub4f1\uc7a5\ud55c \uc77c\ubcf8 \uc81c\uad6d\uc774 1910\ub144 \ub300\ud55c\uc81c\uad6d\uc744 \ubb34\ub825\uc73c\ub85c \ubcd1\ud569\ud55c \ub4a4 1945\ub144\uae4c\uc9c0 35\ub144 \ub3d9\uc548 \ud3ed\uc545\ud55c \uc2dd\ubbfc \ud1b5\uce58\ub97c \ud589\ud558\uace0 \ud604\ub300\uc5d0\ub3c4 \uc77c\ubcf8\uc758 \uc5ec\ub7ec \uc6b0\uc775 \uc778\uc0ac\ub4e4\uc774 \ud55c\uad6d\uc5d0 \ub300\ud55c \uc815\uce58\uc801 \ub3c4\ubc1c\uc744 \uc8fc\ucc3d\ud558\uba74\uc11c \uc591\uad6d\uc758 \uad6d\ubbfc \uac10\uc815\uc774 \uadf9\ub3c4\ub85c \uc545\ud654\ub418\uc5c8\ub2e4. \uc77c\ubcf8\uc5d0\uc11c\ub3c4 \ubc18\ud55c\uac10\uc815\uc744 \uac00\uc9c4 \uc77c\ubd80 \uc77c\ubcf8\uc778\ub4e4\uc774 \ubc18\ud55c\uc2dc\uc704\ub97c \ud558\uace0 \uc788\uc73c\uba70, \ud604\uc7ac \ub300\ud55c\ubbfc\uad6d\uacfc \uc77c\ubcf8\uc740 \ub3c5\ub3c4 \ubd84\uc7c1\uacfc \ub3d9\ud574\uc758 \uc774\ub984\uc5d0 \ub300\ud55c \ubd84\uc7c1 \ub4f1\uc758 \uacfc\uc81c\ub97c \uc548\uace0 \uc788\ub2e4. 2009\ub144\uc5d0 \uc77c\ubcf8 \ubbfc\uc8fc\ub2f9 \ud558\ud1a0\uc57c\ub9c8 \uc720\ud0a4\uc624 \ub0b4\uac01\uc774 \uc131\ub9bd\ud55c \uc774\ud6c4 \uc774\ub8e8\uc5b4\uc9c4 \uc591\uad6d\ubbfc \uc124\ubb38\uc870\uc0ac\uc5d0\uc11c \uc751\ub2f5\uc790\uc758 \ub2e4\uc218\uac00 \ub354 \uc545\ud654\ub41c \ud55c\uc77c \uad00\uacc4\uc5d0 \ubcc0\ud654\uac00 \uc5c6\uc744 \uac83\uc774\ub77c \uc751\ub2f5\ud55c \uacb0\uacfc\uac00 \ub098\uc624\uae30\ub3c4 \ud558\uc600\ub2e4. \uc704\uc548\ubd80\uc5d0 \ub300\ud55c \uc5ed\uc0ac\uc65c\uace1\uc73c\ub85c \uc77c\ubcf8\uc815\ubd80\ub294 \uc720\uc5d4\uc73c\ub85c\ubd80\ud130 \uacbd\uace0\ub97c \ubc1b\uc558\ub2e4.", "sentence_answer": "\uadf8\ub7ec\ub098 \uba54\uc774\uc9c0 \uc720\uc2e0 \uc774\ud6c4\uc5d0 \ub4f1\uc7a5\ud55c \uc77c\ubcf8 \uc81c\uad6d\uc774 1910\ub144 \ub300\ud55c\uc81c\uad6d\uc744 \ubb34\ub825\uc73c\ub85c \ubcd1\ud569\ud55c \ub4a4 1945\ub144\uae4c\uc9c0 35\ub144 \ub3d9\uc548 \ud3ed\uc545\ud55c \uc2dd\ubbfc \ud1b5\uce58\ub97c \ud589\ud558\uace0 \ud604\ub300\uc5d0\ub3c4 \uc77c\ubcf8\uc758 \uc5ec\ub7ec \uc6b0\uc775 \uc778\uc0ac\ub4e4\uc774 \ud55c\uad6d\uc5d0 \ub300\ud55c \uc815\uce58\uc801 \ub3c4\ubc1c\uc744 \uc8fc\ucc3d\ud558\uba74\uc11c \uc591\uad6d\uc758 \uad6d\ubbfc \uac10\uc815\uc774 \uadf9\ub3c4\ub85c \uc545\ud654\ub418\uc5c8\ub2e4.", "paragraph_id": "6173085-10-1", "paragraph_question": "question: \uc77c\ubcf8\uc740 \ub300\ud55c\uc81c\uad6d\uc744 \uba87\ub144\ub3d9\uc548 \ud1b5\uce58\ud558\uc600\ub294\uac00?, context: \ud55c\ubc18\ub3c4\uc640 \uc77c\ubcf8 \uc5f4\ub3c4\ub294 \uace0\ub300\ubd80\ud130 \uac01\uc885 \ubb38\ud654\uc758 \uc804\ud30c\uc640 \ubb34\uc5ed\uc744 \ube44\ub86f\ud55c \ub9ce\uc740 \uad50\ub958\ub97c \uc774\uc5b4\uc654\ub2e4. \ud2b9\ud788 \ubc31\uc81c\ub294 \uace0\ub300 \uc77c\ubcf8\uc5d0 \uc911\uad6d \ubb38\ubb3c\uc744 \ub9ce\uc774 \uc804\ud30c\ud558\uc600\ub2e4. \uace0\ub824 \uc2dc\ub300 \ud6c4\ubc18\uacfc \uc870\uc120 \uc2dc\ub300 \uc804\ubc18\uc5d0 \uac78\uccd0\uc11c\ub294 \uc4f0\uc2dc\ub9c8 \uc12c\uacfc \uaddc\uc288\ub97c \uadfc\uac70\uc9c0\ub85c \ud558\ub294 \uc65c\uad6c(\u502d\u5bc7)\uac00 \uac01\uc885 \uc57d\ud0c8\uc744 \uc77c\uc0bc\uc558\uc73c\uba70 \uc774\ub85c \uc778\ud574 \uc870\uc120 \uc2dc\ub300 \uc911\ubc18\uc5d0 \uc784\uc9c4\uc65c\ub780\uc774 \uc77c\uc5b4\ub098\ub294 \ub4f1 \uc911\uc138\uc640 \uadfc\uc138\uae30 \ud55c\uc77c \uad00\uacc4\ub294 \uae09\uc18d\ub3c4\ub85c \ub0c9\uac01\ub418\uae30\ub3c4 \ud558\uc600\uc73c\ub098, \uc784\uc9c4\uc65c\ub780 \uc774\ud6c4 \uc870\uc120\uc758 \ud1b5\uc2e0\uc0ac \ud30c\uacac\uc73c\ub85c \ub2e4\uc2dc \ud55c\uc77c \uad50\ub958\uac00 \uc7ac\uac1c\ub418\uc5c8\ub2e4. \uadf8\ub7ec\ub098 \uba54\uc774\uc9c0 \uc720\uc2e0 \uc774\ud6c4\uc5d0 \ub4f1\uc7a5\ud55c \uc77c\ubcf8 \uc81c\uad6d\uc774 1910\ub144 \ub300\ud55c\uc81c\uad6d\uc744 \ubb34\ub825\uc73c\ub85c \ubcd1\ud569\ud55c \ub4a4 1945\ub144\uae4c\uc9c0 35\ub144 \ub3d9\uc548 \ud3ed\uc545\ud55c \uc2dd\ubbfc \ud1b5\uce58\ub97c \ud589\ud558\uace0 \ud604\ub300\uc5d0\ub3c4 \uc77c\ubcf8\uc758 \uc5ec\ub7ec \uc6b0\uc775 \uc778\uc0ac\ub4e4\uc774 \ud55c\uad6d\uc5d0 \ub300\ud55c \uc815\uce58\uc801 \ub3c4\ubc1c\uc744 \uc8fc\ucc3d\ud558\uba74\uc11c \uc591\uad6d\uc758 \uad6d\ubbfc \uac10\uc815\uc774 \uadf9\ub3c4\ub85c \uc545\ud654\ub418\uc5c8\ub2e4. \uc77c\ubcf8\uc5d0\uc11c\ub3c4 \ubc18\ud55c\uac10\uc815\uc744 \uac00\uc9c4 \uc77c\ubd80 \uc77c\ubcf8\uc778\ub4e4\uc774 \ubc18\ud55c\uc2dc\uc704\ub97c \ud558\uace0 \uc788\uc73c\uba70, \ud604\uc7ac \ub300\ud55c\ubbfc\uad6d\uacfc \uc77c\ubcf8\uc740 \ub3c5\ub3c4 \ubd84\uc7c1\uacfc \ub3d9\ud574\uc758 \uc774\ub984\uc5d0 \ub300\ud55c \ubd84\uc7c1 \ub4f1\uc758 \uacfc\uc81c\ub97c \uc548\uace0 \uc788\ub2e4. 2009\ub144\uc5d0 \uc77c\ubcf8 \ubbfc\uc8fc\ub2f9 \ud558\ud1a0\uc57c\ub9c8 \uc720\ud0a4\uc624 \ub0b4\uac01\uc774 \uc131\ub9bd\ud55c \uc774\ud6c4 \uc774\ub8e8\uc5b4\uc9c4 \uc591\uad6d\ubbfc \uc124\ubb38\uc870\uc0ac\uc5d0\uc11c \uc751\ub2f5\uc790\uc758 \ub2e4\uc218\uac00 \ub354 \uc545\ud654\ub41c \ud55c\uc77c \uad00\uacc4\uc5d0 \ubcc0\ud654\uac00 \uc5c6\uc744 \uac83\uc774\ub77c \uc751\ub2f5\ud55c \uacb0\uacfc\uac00 \ub098\uc624\uae30\ub3c4 \ud558\uc600\ub2e4. \uc704\uc548\ubd80\uc5d0 \ub300\ud55c \uc5ed\uc0ac\uc65c\uace1\uc73c\ub85c \uc77c\ubcf8\uc815\ubd80\ub294 \uc720\uc5d4\uc73c\ub85c\ubd80\ud130 \uacbd\uace0\ub97c \ubc1b\uc558\ub2e4."} {"question": "\ud544\ub9ac\ud504 4\uc138\uc758 \uc555\ubc15\uc73c\ub85c \uad50\ud669\ub4e4\uc740 \ud504\ub791\uc2a4 \uc5b4\ub290 \ub3c4\uc2dc\uc5d0 \uac70\uc8fc\ud558\uac8c\ub418\uc5c8\ub098?", "paragraph": "\uc774\ud6c4 \uad50\ud669\uad8c\uc740 \uae09\uaca9\ud788 \uc1e0\ud1f4\ud588\ub2e4. \uc774\ud6c4 \uc120\ucd9c\ub41c \uad50\ud669\ub4e4\uc740 \ud544\ub9ac\ud504 4\uc138\uc758 \uac15\ub825\ud55c \uacac\uc81c\ub97c \ubc1b\ub2e4\uac00, \uacb0\uad6d 1309\ub144 \ud544\ub9ac\ud504 4\uc138\uc758 \uc555\ubc15\uc5d0 \uad74\ubcf5\ud558\uc5ec \ub85c\ub9c8\ub97c \ub5a0\ub098 \ud504\ub791\uc2a4 \ub0a8\ubd80\uc5d0 \uc788\ub294 \ub3c4\uc2dc \uc544\ube44\ub1fd\uc5d0 \uac70\uc8fc\ud558\uc600\ub2e4. 1309\ub144\ubd80\ud130 1377\ub144\uae4c\uc9c0 \uc57d 70\ub144\uac04 \uad50\ud669\ub4e4\uc740 \uc544\ube44\ub1fd\uc5d0 \uac70\uc8fc\ud558\uba70 \ud504\ub791\uc2a4 \uad6d\uc655\uc758 \uac15\ub825\ud55c \uac04\uc12d\uc744 \ubc1b\uc558\ub2e4. \uc544\ube44\ub1fd \uc720\uc218\ub77c\uace0\ub3c4 \ubd80\ub974\ub294 \uc774 \uc2dc\uae30 \ub3d9\uc548 \uad50\ud669\uc740 \ubaa8\ub450 \ud504\ub791\uc2a4\uc778\uc774 \uc120\ucd9c\ub418\uc5c8\uace0, \uad50\ud68c\uc758 \uc694\uc9c1\uc5d0\ub3c4 \ud504\ub791\uc2a4\uc778 \uc131\uc9c1\uc790\uac00 \ub300\uac70 \ucc28\uc9c0\ud558\uc600\ub2e4. 1377\ub144 \uad50\ud669 \uadf8\ub808\uace0\ub9ac\uc624 11\uc138\uac00 \ub85c\ub9c8\ub85c \ubcf5\uadc0\ud588\uc73c\ub098 \uc544\ube44\ub1fd \uc2dc\uc808\uc744 \uacaa\uc73c\uba74\uc11c \ubc1c\uc0dd\ud55c \uad50\ud68c\uc758 \ud63c\ub780\uc744 \ub05d\ub098\uc9c0 \uc54a\uc558\ub2e4. 1378\ub144 \uc774\ud0c8\ub9ac\uc544\uc778\uc774\uc5c8\ub358 \uad50\ud669 \uc6b0\ub974\ubc14\ub178 6\uc138\uac00 \uc0c8 \uad50\ud669\uc5d0 \uc120\ucd9c\ub418\uc790 \ud504\ub791\uc2a4\uc778 \ucd94\uae30\uacbd\ub4e4\uc740 \uc774\uc5d0 \ub300\ud574 \ubb34\ud6a8 \uc120\uc5b8\uc744 \ud558\uace0 \ud504\ub791\uc2a4 \ucd9c\uc2e0\uc778 \ub300\ub9bd \uad50\ud669 \ud074\ub808\uba58\uc2a4 7\uc138\ub97c \uc790\uccb4\uc801\uc73c\ub85c \uc120\ucd9c\ud558\uc5ec \ub0b4\uc138\uc6e0\ub2e4. \uc774\ud6c4 \uad50\ud68c\ub294 40\ub144 \ub3d9\uc548 \uc11c\uad6c \ub300\uc774\uad50 \uc2dc\ub300\ub97c \ub9de\uac8c \ub418\uc5c8\ub2e4. \uad50\ud669\uc740 \ub85c\ub9c8\uc5d0 \uba38\ubb3c\ub800\uc73c\uba70, \ub300\ub9bd \uad50\ud669\uacfc \uadf8\ub97c \ub530\ub974\ub294 \uc774\ub4e4\uc740 \uc544\ube44\ub1fd\uc5d0 \uba38\ubb3c\ub800\ub2e4. \uadf8 \uacb0\uacfc, \uad50\ud68c\uc758 \ubd84\uc5f4\uc740 \uba85\ubc31\ud788 \uad6d\uac00\uc801\uc778 \uc591\uc0c1\uc744 \ub760\uac8c \ub418\uc5c8\ub294\ub370 \uc789\uae00\ub79c\ub4dc\uc640 \ud5dd\uac00\ub9ac, \uc2a4\uce78\ub514\ub098\ube44\uc544, \uc774\ud0c8\ub9ac\uc544\uc758 \ub300\ub2e4\uc218 \ub3c4\uc2dc\uad6d\uac00, \uc2e0\uc131 \ub85c\ub9c8 \uc81c\uad6d \ub4f1\uc740 \ub85c\ub9c8 \uad50\ud669\uc744 \uc9c0\uc9c0\ud588\uc73c\uba70, \ud504\ub791\uc2a4\uc640 \ub098\ud3f4\ub9ac, \uc0ac\ubcf4\uc774\uc544, \uc2a4\ucf54\ud2c0\ub79c\ub4dc, \uc2a4\ud398\uc778, \uc2dc\uce60\ub9ac\uc544 \ub4f1\uc740 \uc544\ube44\ub1fd\uc758 \ub300\ub9bd \uad50\ud669\uc744 \uc9c0\uc9c0\ud558\uc600\ub2e4. \ub098\uc911\uc5d0\ub294 \ub300\ub9bd \uad50\ud669\uc774 \ud55c \uba85 \ub354 \uc0dd\uaca8, \uc138 \uba85\uc758 \uad50\ud669\uc774 \uc11c\ub85c \uc790\uc2e0\uc774 \uc9c4\uc9dc \uad50\ud669\uc774\ub77c\uace0 \uc8fc\uc7a5\ud558\uba70 \uc0c1\ub300\ud3b8\uc744 \uc11c\ub85c \ud30c\ubb38\ud558\uc600\uc73c\uba70, \uad50\ud68c\ub294 \ud63c\ub780\uc5d0 \ube60\uc84c\ub2e4. \uc11c\ubc29 \uad50\ud68c\uc758 \ubd84\uc5f4\uc740 1417\ub144 \uc18c\uc9d1\ub41c \ucf58\uc2a4\ud0c4\uce20 \uacf5\uc758\ud68c\uc5d0\uc11c \ub9c8\ub974\ud2f0\ub178 5\uc138\ub97c \uc0c8 \uad50\ud669\uc73c\ub85c \uc120\ucd9c\ud568\uc73c\ub85c\uc368 \uc885\uacb0\ub418\uc5c8\ub2e4.", "answer": "\uc544\ube44\ub1fd", "sentence": "\uc774\ud6c4 \uc120\ucd9c\ub41c \uad50\ud669\ub4e4\uc740 \ud544\ub9ac\ud504 4\uc138\uc758 \uac15\ub825\ud55c \uacac\uc81c\ub97c \ubc1b\ub2e4\uac00, \uacb0\uad6d 1309\ub144 \ud544\ub9ac\ud504 4\uc138\uc758 \uc555\ubc15\uc5d0 \uad74\ubcf5\ud558\uc5ec \ub85c\ub9c8\ub97c \ub5a0\ub098 \ud504\ub791\uc2a4 \ub0a8\ubd80\uc5d0 \uc788\ub294 \ub3c4\uc2dc \uc544\ube44\ub1fd \uc5d0 \uac70\uc8fc\ud558\uc600\ub2e4.", "paragraph_sentence": "\uc774\ud6c4 \uad50\ud669\uad8c\uc740 \uae09\uaca9\ud788 \uc1e0\ud1f4\ud588\ub2e4. \uc774\ud6c4 \uc120\ucd9c\ub41c \uad50\ud669\ub4e4\uc740 \ud544\ub9ac\ud504 4\uc138\uc758 \uac15\ub825\ud55c \uacac\uc81c\ub97c \ubc1b\ub2e4\uac00, \uacb0\uad6d 1309\ub144 \ud544\ub9ac\ud504 4\uc138\uc758 \uc555\ubc15\uc5d0 \uad74\ubcf5\ud558\uc5ec \ub85c\ub9c8\ub97c \ub5a0\ub098 \ud504\ub791\uc2a4 \ub0a8\ubd80\uc5d0 \uc788\ub294 \ub3c4\uc2dc \uc544\ube44\ub1fd \uc5d0 \uac70\uc8fc\ud558\uc600\ub2e4. 1309\ub144\ubd80\ud130 1377\ub144\uae4c\uc9c0 \uc57d 70\ub144\uac04 \uad50\ud669\ub4e4\uc740 \uc544\ube44\ub1fd\uc5d0 \uac70\uc8fc\ud558\uba70 \ud504\ub791\uc2a4 \uad6d\uc655\uc758 \uac15\ub825\ud55c \uac04\uc12d\uc744 \ubc1b\uc558\ub2e4. \uc544\ube44\ub1fd \uc720\uc218\ub77c\uace0\ub3c4 \ubd80\ub974\ub294 \uc774 \uc2dc\uae30 \ub3d9\uc548 \uad50\ud669\uc740 \ubaa8\ub450 \ud504\ub791\uc2a4\uc778\uc774 \uc120\ucd9c\ub418\uc5c8\uace0, \uad50\ud68c\uc758 \uc694\uc9c1\uc5d0\ub3c4 \ud504\ub791\uc2a4\uc778 \uc131\uc9c1\uc790\uac00 \ub300\uac70 \ucc28\uc9c0\ud558\uc600\ub2e4. 1377\ub144 \uad50\ud669 \uadf8\ub808\uace0\ub9ac\uc624 11\uc138\uac00 \ub85c\ub9c8\ub85c \ubcf5\uadc0\ud588\uc73c\ub098 \uc544\ube44\ub1fd \uc2dc\uc808\uc744 \uacaa\uc73c\uba74\uc11c \ubc1c\uc0dd\ud55c \uad50\ud68c\uc758 \ud63c\ub780\uc744 \ub05d\ub098\uc9c0 \uc54a\uc558\ub2e4. 1378\ub144 \uc774\ud0c8\ub9ac\uc544\uc778\uc774\uc5c8\ub358 \uad50\ud669 \uc6b0\ub974\ubc14\ub178 6\uc138\uac00 \uc0c8 \uad50\ud669\uc5d0 \uc120\ucd9c\ub418\uc790 \ud504\ub791\uc2a4\uc778 \ucd94\uae30\uacbd\ub4e4\uc740 \uc774\uc5d0 \ub300\ud574 \ubb34\ud6a8 \uc120\uc5b8\uc744 \ud558\uace0 \ud504\ub791\uc2a4 \ucd9c\uc2e0\uc778 \ub300\ub9bd \uad50\ud669 \ud074\ub808\uba58\uc2a4 7\uc138\ub97c \uc790\uccb4\uc801\uc73c\ub85c \uc120\ucd9c\ud558\uc5ec \ub0b4\uc138\uc6e0\ub2e4. \uc774\ud6c4 \uad50\ud68c\ub294 40\ub144 \ub3d9\uc548 \uc11c\uad6c \ub300\uc774\uad50 \uc2dc\ub300\ub97c \ub9de\uac8c \ub418\uc5c8\ub2e4. \uad50\ud669\uc740 \ub85c\ub9c8\uc5d0 \uba38\ubb3c\ub800\uc73c\uba70, \ub300\ub9bd \uad50\ud669\uacfc \uadf8\ub97c \ub530\ub974\ub294 \uc774\ub4e4\uc740 \uc544\ube44\ub1fd\uc5d0 \uba38\ubb3c\ub800\ub2e4. \uadf8 \uacb0\uacfc, \uad50\ud68c\uc758 \ubd84\uc5f4\uc740 \uba85\ubc31\ud788 \uad6d\uac00\uc801\uc778 \uc591\uc0c1\uc744 \ub760\uac8c \ub418\uc5c8\ub294\ub370 \uc789\uae00\ub79c\ub4dc\uc640 \ud5dd\uac00\ub9ac, \uc2a4\uce78\ub514\ub098\ube44\uc544, \uc774\ud0c8\ub9ac\uc544\uc758 \ub300\ub2e4\uc218 \ub3c4\uc2dc\uad6d\uac00, \uc2e0\uc131 \ub85c\ub9c8 \uc81c\uad6d \ub4f1\uc740 \ub85c\ub9c8 \uad50\ud669\uc744 \uc9c0\uc9c0\ud588\uc73c\uba70, \ud504\ub791\uc2a4\uc640 \ub098\ud3f4\ub9ac, \uc0ac\ubcf4\uc774\uc544, \uc2a4\ucf54\ud2c0\ub79c\ub4dc, \uc2a4\ud398\uc778, \uc2dc\uce60\ub9ac\uc544 \ub4f1\uc740 \uc544\ube44\ub1fd\uc758 \ub300\ub9bd \uad50\ud669\uc744 \uc9c0\uc9c0\ud558\uc600\ub2e4. \ub098\uc911\uc5d0\ub294 \ub300\ub9bd \uad50\ud669\uc774 \ud55c \uba85 \ub354 \uc0dd\uaca8, \uc138 \uba85\uc758 \uad50\ud669\uc774 \uc11c\ub85c \uc790\uc2e0\uc774 \uc9c4\uc9dc \uad50\ud669\uc774\ub77c\uace0 \uc8fc\uc7a5\ud558\uba70 \uc0c1\ub300\ud3b8\uc744 \uc11c\ub85c \ud30c\ubb38\ud558\uc600\uc73c\uba70, \uad50\ud68c\ub294 \ud63c\ub780\uc5d0 \ube60\uc84c\ub2e4. \uc11c\ubc29 \uad50\ud68c\uc758 \ubd84\uc5f4\uc740 1417\ub144 \uc18c\uc9d1\ub41c \ucf58\uc2a4\ud0c4\uce20 \uacf5\uc758\ud68c\uc5d0\uc11c \ub9c8\ub974\ud2f0\ub178 5\uc138\ub97c \uc0c8 \uad50\ud669\uc73c\ub85c \uc120\ucd9c\ud568\uc73c\ub85c\uc368 \uc885\uacb0\ub418\uc5c8\ub2e4.", "paragraph_answer": "\uc774\ud6c4 \uad50\ud669\uad8c\uc740 \uae09\uaca9\ud788 \uc1e0\ud1f4\ud588\ub2e4. \uc774\ud6c4 \uc120\ucd9c\ub41c \uad50\ud669\ub4e4\uc740 \ud544\ub9ac\ud504 4\uc138\uc758 \uac15\ub825\ud55c \uacac\uc81c\ub97c \ubc1b\ub2e4\uac00, \uacb0\uad6d 1309\ub144 \ud544\ub9ac\ud504 4\uc138\uc758 \uc555\ubc15\uc5d0 \uad74\ubcf5\ud558\uc5ec \ub85c\ub9c8\ub97c \ub5a0\ub098 \ud504\ub791\uc2a4 \ub0a8\ubd80\uc5d0 \uc788\ub294 \ub3c4\uc2dc \uc544\ube44\ub1fd \uc5d0 \uac70\uc8fc\ud558\uc600\ub2e4. 1309\ub144\ubd80\ud130 1377\ub144\uae4c\uc9c0 \uc57d 70\ub144\uac04 \uad50\ud669\ub4e4\uc740 \uc544\ube44\ub1fd\uc5d0 \uac70\uc8fc\ud558\uba70 \ud504\ub791\uc2a4 \uad6d\uc655\uc758 \uac15\ub825\ud55c \uac04\uc12d\uc744 \ubc1b\uc558\ub2e4. \uc544\ube44\ub1fd \uc720\uc218\ub77c\uace0\ub3c4 \ubd80\ub974\ub294 \uc774 \uc2dc\uae30 \ub3d9\uc548 \uad50\ud669\uc740 \ubaa8\ub450 \ud504\ub791\uc2a4\uc778\uc774 \uc120\ucd9c\ub418\uc5c8\uace0, \uad50\ud68c\uc758 \uc694\uc9c1\uc5d0\ub3c4 \ud504\ub791\uc2a4\uc778 \uc131\uc9c1\uc790\uac00 \ub300\uac70 \ucc28\uc9c0\ud558\uc600\ub2e4. 1377\ub144 \uad50\ud669 \uadf8\ub808\uace0\ub9ac\uc624 11\uc138\uac00 \ub85c\ub9c8\ub85c \ubcf5\uadc0\ud588\uc73c\ub098 \uc544\ube44\ub1fd \uc2dc\uc808\uc744 \uacaa\uc73c\uba74\uc11c \ubc1c\uc0dd\ud55c \uad50\ud68c\uc758 \ud63c\ub780\uc744 \ub05d\ub098\uc9c0 \uc54a\uc558\ub2e4. 1378\ub144 \uc774\ud0c8\ub9ac\uc544\uc778\uc774\uc5c8\ub358 \uad50\ud669 \uc6b0\ub974\ubc14\ub178 6\uc138\uac00 \uc0c8 \uad50\ud669\uc5d0 \uc120\ucd9c\ub418\uc790 \ud504\ub791\uc2a4\uc778 \ucd94\uae30\uacbd\ub4e4\uc740 \uc774\uc5d0 \ub300\ud574 \ubb34\ud6a8 \uc120\uc5b8\uc744 \ud558\uace0 \ud504\ub791\uc2a4 \ucd9c\uc2e0\uc778 \ub300\ub9bd \uad50\ud669 \ud074\ub808\uba58\uc2a4 7\uc138\ub97c \uc790\uccb4\uc801\uc73c\ub85c \uc120\ucd9c\ud558\uc5ec \ub0b4\uc138\uc6e0\ub2e4. \uc774\ud6c4 \uad50\ud68c\ub294 40\ub144 \ub3d9\uc548 \uc11c\uad6c \ub300\uc774\uad50 \uc2dc\ub300\ub97c \ub9de\uac8c \ub418\uc5c8\ub2e4. \uad50\ud669\uc740 \ub85c\ub9c8\uc5d0 \uba38\ubb3c\ub800\uc73c\uba70, \ub300\ub9bd \uad50\ud669\uacfc \uadf8\ub97c \ub530\ub974\ub294 \uc774\ub4e4\uc740 \uc544\ube44\ub1fd\uc5d0 \uba38\ubb3c\ub800\ub2e4. \uadf8 \uacb0\uacfc, \uad50\ud68c\uc758 \ubd84\uc5f4\uc740 \uba85\ubc31\ud788 \uad6d\uac00\uc801\uc778 \uc591\uc0c1\uc744 \ub760\uac8c \ub418\uc5c8\ub294\ub370 \uc789\uae00\ub79c\ub4dc\uc640 \ud5dd\uac00\ub9ac, \uc2a4\uce78\ub514\ub098\ube44\uc544, \uc774\ud0c8\ub9ac\uc544\uc758 \ub300\ub2e4\uc218 \ub3c4\uc2dc\uad6d\uac00, \uc2e0\uc131 \ub85c\ub9c8 \uc81c\uad6d \ub4f1\uc740 \ub85c\ub9c8 \uad50\ud669\uc744 \uc9c0\uc9c0\ud588\uc73c\uba70, \ud504\ub791\uc2a4\uc640 \ub098\ud3f4\ub9ac, \uc0ac\ubcf4\uc774\uc544, \uc2a4\ucf54\ud2c0\ub79c\ub4dc, \uc2a4\ud398\uc778, \uc2dc\uce60\ub9ac\uc544 \ub4f1\uc740 \uc544\ube44\ub1fd\uc758 \ub300\ub9bd \uad50\ud669\uc744 \uc9c0\uc9c0\ud558\uc600\ub2e4. \ub098\uc911\uc5d0\ub294 \ub300\ub9bd \uad50\ud669\uc774 \ud55c \uba85 \ub354 \uc0dd\uaca8, \uc138 \uba85\uc758 \uad50\ud669\uc774 \uc11c\ub85c \uc790\uc2e0\uc774 \uc9c4\uc9dc \uad50\ud669\uc774\ub77c\uace0 \uc8fc\uc7a5\ud558\uba70 \uc0c1\ub300\ud3b8\uc744 \uc11c\ub85c \ud30c\ubb38\ud558\uc600\uc73c\uba70, \uad50\ud68c\ub294 \ud63c\ub780\uc5d0 \ube60\uc84c\ub2e4. \uc11c\ubc29 \uad50\ud68c\uc758 \ubd84\uc5f4\uc740 1417\ub144 \uc18c\uc9d1\ub41c \ucf58\uc2a4\ud0c4\uce20 \uacf5\uc758\ud68c\uc5d0\uc11c \ub9c8\ub974\ud2f0\ub178 5\uc138\ub97c \uc0c8 \uad50\ud669\uc73c\ub85c \uc120\ucd9c\ud568\uc73c\ub85c\uc368 \uc885\uacb0\ub418\uc5c8\ub2e4.", "sentence_answer": "\uc774\ud6c4 \uc120\ucd9c\ub41c \uad50\ud669\ub4e4\uc740 \ud544\ub9ac\ud504 4\uc138\uc758 \uac15\ub825\ud55c \uacac\uc81c\ub97c \ubc1b\ub2e4\uac00, \uacb0\uad6d 1309\ub144 \ud544\ub9ac\ud504 4\uc138\uc758 \uc555\ubc15\uc5d0 \uad74\ubcf5\ud558\uc5ec \ub85c\ub9c8\ub97c \ub5a0\ub098 \ud504\ub791\uc2a4 \ub0a8\ubd80\uc5d0 \uc788\ub294 \ub3c4\uc2dc \uc544\ube44\ub1fd \uc5d0 \uac70\uc8fc\ud558\uc600\ub2e4.", "paragraph_id": "6656709-10-0", "paragraph_question": "question: \ud544\ub9ac\ud504 4\uc138\uc758 \uc555\ubc15\uc73c\ub85c \uad50\ud669\ub4e4\uc740 \ud504\ub791\uc2a4 \uc5b4\ub290 \ub3c4\uc2dc\uc5d0 \uac70\uc8fc\ud558\uac8c\ub418\uc5c8\ub098?, context: \uc774\ud6c4 \uad50\ud669\uad8c\uc740 \uae09\uaca9\ud788 \uc1e0\ud1f4\ud588\ub2e4. \uc774\ud6c4 \uc120\ucd9c\ub41c \uad50\ud669\ub4e4\uc740 \ud544\ub9ac\ud504 4\uc138\uc758 \uac15\ub825\ud55c \uacac\uc81c\ub97c \ubc1b\ub2e4\uac00, \uacb0\uad6d 1309\ub144 \ud544\ub9ac\ud504 4\uc138\uc758 \uc555\ubc15\uc5d0 \uad74\ubcf5\ud558\uc5ec \ub85c\ub9c8\ub97c \ub5a0\ub098 \ud504\ub791\uc2a4 \ub0a8\ubd80\uc5d0 \uc788\ub294 \ub3c4\uc2dc \uc544\ube44\ub1fd\uc5d0 \uac70\uc8fc\ud558\uc600\ub2e4. 1309\ub144\ubd80\ud130 1377\ub144\uae4c\uc9c0 \uc57d 70\ub144\uac04 \uad50\ud669\ub4e4\uc740 \uc544\ube44\ub1fd\uc5d0 \uac70\uc8fc\ud558\uba70 \ud504\ub791\uc2a4 \uad6d\uc655\uc758 \uac15\ub825\ud55c \uac04\uc12d\uc744 \ubc1b\uc558\ub2e4. \uc544\ube44\ub1fd \uc720\uc218\ub77c\uace0\ub3c4 \ubd80\ub974\ub294 \uc774 \uc2dc\uae30 \ub3d9\uc548 \uad50\ud669\uc740 \ubaa8\ub450 \ud504\ub791\uc2a4\uc778\uc774 \uc120\ucd9c\ub418\uc5c8\uace0, \uad50\ud68c\uc758 \uc694\uc9c1\uc5d0\ub3c4 \ud504\ub791\uc2a4\uc778 \uc131\uc9c1\uc790\uac00 \ub300\uac70 \ucc28\uc9c0\ud558\uc600\ub2e4. 1377\ub144 \uad50\ud669 \uadf8\ub808\uace0\ub9ac\uc624 11\uc138\uac00 \ub85c\ub9c8\ub85c \ubcf5\uadc0\ud588\uc73c\ub098 \uc544\ube44\ub1fd \uc2dc\uc808\uc744 \uacaa\uc73c\uba74\uc11c \ubc1c\uc0dd\ud55c \uad50\ud68c\uc758 \ud63c\ub780\uc744 \ub05d\ub098\uc9c0 \uc54a\uc558\ub2e4. 1378\ub144 \uc774\ud0c8\ub9ac\uc544\uc778\uc774\uc5c8\ub358 \uad50\ud669 \uc6b0\ub974\ubc14\ub178 6\uc138\uac00 \uc0c8 \uad50\ud669\uc5d0 \uc120\ucd9c\ub418\uc790 \ud504\ub791\uc2a4\uc778 \ucd94\uae30\uacbd\ub4e4\uc740 \uc774\uc5d0 \ub300\ud574 \ubb34\ud6a8 \uc120\uc5b8\uc744 \ud558\uace0 \ud504\ub791\uc2a4 \ucd9c\uc2e0\uc778 \ub300\ub9bd \uad50\ud669 \ud074\ub808\uba58\uc2a4 7\uc138\ub97c \uc790\uccb4\uc801\uc73c\ub85c \uc120\ucd9c\ud558\uc5ec \ub0b4\uc138\uc6e0\ub2e4. \uc774\ud6c4 \uad50\ud68c\ub294 40\ub144 \ub3d9\uc548 \uc11c\uad6c \ub300\uc774\uad50 \uc2dc\ub300\ub97c \ub9de\uac8c \ub418\uc5c8\ub2e4. \uad50\ud669\uc740 \ub85c\ub9c8\uc5d0 \uba38\ubb3c\ub800\uc73c\uba70, \ub300\ub9bd \uad50\ud669\uacfc \uadf8\ub97c \ub530\ub974\ub294 \uc774\ub4e4\uc740 \uc544\ube44\ub1fd\uc5d0 \uba38\ubb3c\ub800\ub2e4. \uadf8 \uacb0\uacfc, \uad50\ud68c\uc758 \ubd84\uc5f4\uc740 \uba85\ubc31\ud788 \uad6d\uac00\uc801\uc778 \uc591\uc0c1\uc744 \ub760\uac8c \ub418\uc5c8\ub294\ub370 \uc789\uae00\ub79c\ub4dc\uc640 \ud5dd\uac00\ub9ac, \uc2a4\uce78\ub514\ub098\ube44\uc544, \uc774\ud0c8\ub9ac\uc544\uc758 \ub300\ub2e4\uc218 \ub3c4\uc2dc\uad6d\uac00, \uc2e0\uc131 \ub85c\ub9c8 \uc81c\uad6d \ub4f1\uc740 \ub85c\ub9c8 \uad50\ud669\uc744 \uc9c0\uc9c0\ud588\uc73c\uba70, \ud504\ub791\uc2a4\uc640 \ub098\ud3f4\ub9ac, \uc0ac\ubcf4\uc774\uc544, \uc2a4\ucf54\ud2c0\ub79c\ub4dc, \uc2a4\ud398\uc778, \uc2dc\uce60\ub9ac\uc544 \ub4f1\uc740 \uc544\ube44\ub1fd\uc758 \ub300\ub9bd \uad50\ud669\uc744 \uc9c0\uc9c0\ud558\uc600\ub2e4. \ub098\uc911\uc5d0\ub294 \ub300\ub9bd \uad50\ud669\uc774 \ud55c \uba85 \ub354 \uc0dd\uaca8, \uc138 \uba85\uc758 \uad50\ud669\uc774 \uc11c\ub85c \uc790\uc2e0\uc774 \uc9c4\uc9dc \uad50\ud669\uc774\ub77c\uace0 \uc8fc\uc7a5\ud558\uba70 \uc0c1\ub300\ud3b8\uc744 \uc11c\ub85c \ud30c\ubb38\ud558\uc600\uc73c\uba70, \uad50\ud68c\ub294 \ud63c\ub780\uc5d0 \ube60\uc84c\ub2e4. \uc11c\ubc29 \uad50\ud68c\uc758 \ubd84\uc5f4\uc740 1417\ub144 \uc18c\uc9d1\ub41c \ucf58\uc2a4\ud0c4\uce20 \uacf5\uc758\ud68c\uc5d0\uc11c \ub9c8\ub974\ud2f0\ub178 5\uc138\ub97c \uc0c8 \uad50\ud669\uc73c\ub85c \uc120\ucd9c\ud568\uc73c\ub85c\uc368 \uc885\uacb0\ub418\uc5c8\ub2e4."} {"question": "\ud5c8\ub9ac\ucf00\uc778 \uc70c\ub9c8\uac00 \uae30\ub85d\uc744 \uae6c \ub54c\ub294?", "paragraph": "\ud5c8\ub9ac\ucf00\uc778 \uae38\ubc84\ud2b8\ub294 \uc790\uba54\uc774\uce74\ub97c \ube60\uc838\ub098\uc628 \ub4a4\uc5d0 \ub2e4\uc2dc \uae09\uaca9\ud558\uac8c \uac15\ud574\uc84c\uc73c\uba70 \ucf00\uc774\ub9e8 \uc81c\ub3c4\uc758 \uac00\uc7a5 \ud070 \uc12c\uc778 \uadf8\ub79c\ub4dc \ucf00\uc774\ub9e8(Grand Cayman)\uc12c\uc758 \uae30\uc0c1\uad00\uce21\uc18c\uc5d0\uc11c\ub294 9\uc6d4 13\uc77c, \uae38\ubc84\ud2b8\uac00 \uc12c \ub0a8\ub3d9\ucabd\uc5d0 \uc704\uce58\ud574\uc788\uc744 \ub54c 252km/h(156mph)\uc5d0 \ub2ec\ud558\ub294 \ub3cc\ud48d\uc774 \uad00\uce21\ub418\uae30\ub3c4 \ud558\uc600\ub2e4. \uae38\ubc84\ud2b8\ub294 \uc911\uc2ec\uae30\uc555\uc774 888hPa\uc5d0 \ub3c4\ub2ec\ud558\uae30\uae4c\uc9c0 \uc26c\uc9c0\uc54a\uace0 \uacc4\uc18d \uac15\ud574\uc84c\uace0 \ub9c8\uce68\ub0b4 \ucd5c\uc131\uae30\ub97c \ub9de\uc774\ud558\uc600\uc744 \ub54c\ub294 \uac15\ud48d\ubc18\uacbd \ub0b4\uc5d0\uc11c 295km/h(185mph)\uc758 \ubc14\ub78c\uc774 \ubd88\uc5c8\ub2e4. \uc774 \uc218\uce58\ub294 24\uc2dc\uac04\ub9cc\uc5d0 72hPa\uac00 \ub5a8\uc5b4\uc9c4 \uac83\uc73c\ub85c \uae30\ub85d\ub418\uc5c8\uace0 \uc774\ub294 \ub2f9\uc2dc \uc804\uc5d0 \uc77c\uc5b4\ub0ac\ub358 \ubaa8\ub4e0 \ubd81\ub300\uc11c\uc591 \ud5c8\ub9ac\ucf00\uc778\uc744 \ud1b5\ud2c0\uc5b4\uc11c \uac00\uc7a5 \uac15\ud55c \ud5c8\ub9ac\ucf00\uc778\uc774\uc5c8\ub2e4. \uadf8\ub7ec\ub098 \uc774 \uae30\ub85d\uc740 2005\ub144\uc5d0 \ud5c8\ub9ac\ucf00\uc778 \uc70c\ub9c8\uac00 6hPa \ub0ae\uc740 882hPa\ub97c \uae30\ub85d\ud558\uba74\uc11c \uae68\uc9c0\uac8c \ub418\uc5c8\ub2e4.", "answer": "2005\ub144", "sentence": "\uadf8\ub7ec\ub098 \uc774 \uae30\ub85d\uc740 2005\ub144 \uc5d0", "paragraph_sentence": "\ud5c8\ub9ac\ucf00\uc778 \uae38\ubc84\ud2b8\ub294 \uc790\uba54\uc774\uce74\ub97c \ube60\uc838\ub098\uc628 \ub4a4\uc5d0 \ub2e4\uc2dc \uae09\uaca9\ud558\uac8c \uac15\ud574\uc84c\uc73c\uba70 \ucf00\uc774\ub9e8 \uc81c\ub3c4\uc758 \uac00\uc7a5 \ud070 \uc12c\uc778 \uadf8\ub79c\ub4dc \ucf00\uc774\ub9e8(Grand Cayman)\uc12c\uc758 \uae30\uc0c1\uad00\uce21\uc18c\uc5d0\uc11c\ub294 9\uc6d4 13\uc77c, \uae38\ubc84\ud2b8\uac00 \uc12c \ub0a8\ub3d9\ucabd\uc5d0 \uc704\uce58\ud574\uc788\uc744 \ub54c 252km/h(156mph)\uc5d0 \ub2ec\ud558\ub294 \ub3cc\ud48d\uc774 \uad00\uce21\ub418\uae30\ub3c4 \ud558\uc600\ub2e4. \uae38\ubc84\ud2b8\ub294 \uc911\uc2ec\uae30\uc555\uc774 888hPa\uc5d0 \ub3c4\ub2ec\ud558\uae30\uae4c\uc9c0 \uc26c\uc9c0\uc54a\uace0 \uacc4\uc18d \uac15\ud574\uc84c\uace0 \ub9c8\uce68\ub0b4 \ucd5c\uc131\uae30\ub97c \ub9de\uc774\ud558\uc600\uc744 \ub54c\ub294 \uac15\ud48d\ubc18\uacbd \ub0b4\uc5d0\uc11c 295km/h(185mph)\uc758 \ubc14\ub78c\uc774 \ubd88\uc5c8\ub2e4. \uc774 \uc218\uce58\ub294 24\uc2dc\uac04\ub9cc\uc5d0 72hPa\uac00 \ub5a8\uc5b4\uc9c4 \uac83\uc73c\ub85c \uae30\ub85d\ub418\uc5c8\uace0 \uc774\ub294 \ub2f9\uc2dc \uc804\uc5d0 \uc77c\uc5b4\ub0ac\ub358 \ubaa8\ub4e0 \ubd81\ub300\uc11c\uc591 \ud5c8\ub9ac\ucf00\uc778\uc744 \ud1b5\ud2c0\uc5b4\uc11c \uac00\uc7a5 \uac15\ud55c \ud5c8\ub9ac\ucf00\uc778\uc774\uc5c8\ub2e4. \uadf8\ub7ec\ub098 \uc774 \uae30\ub85d\uc740 2005\ub144 \uc5d0 \ud5c8\ub9ac\ucf00\uc778 \uc70c\ub9c8\uac00 6hPa \ub0ae\uc740 882hPa\ub97c \uae30\ub85d\ud558\uba74\uc11c \uae68\uc9c0\uac8c \ub418\uc5c8\ub2e4.", "paragraph_answer": "\ud5c8\ub9ac\ucf00\uc778 \uae38\ubc84\ud2b8\ub294 \uc790\uba54\uc774\uce74\ub97c \ube60\uc838\ub098\uc628 \ub4a4\uc5d0 \ub2e4\uc2dc \uae09\uaca9\ud558\uac8c \uac15\ud574\uc84c\uc73c\uba70 \ucf00\uc774\ub9e8 \uc81c\ub3c4\uc758 \uac00\uc7a5 \ud070 \uc12c\uc778 \uadf8\ub79c\ub4dc \ucf00\uc774\ub9e8(Grand Cayman)\uc12c\uc758 \uae30\uc0c1\uad00\uce21\uc18c\uc5d0\uc11c\ub294 9\uc6d4 13\uc77c, \uae38\ubc84\ud2b8\uac00 \uc12c \ub0a8\ub3d9\ucabd\uc5d0 \uc704\uce58\ud574\uc788\uc744 \ub54c 252km/h(156mph)\uc5d0 \ub2ec\ud558\ub294 \ub3cc\ud48d\uc774 \uad00\uce21\ub418\uae30\ub3c4 \ud558\uc600\ub2e4. \uae38\ubc84\ud2b8\ub294 \uc911\uc2ec\uae30\uc555\uc774 888hPa\uc5d0 \ub3c4\ub2ec\ud558\uae30\uae4c\uc9c0 \uc26c\uc9c0\uc54a\uace0 \uacc4\uc18d \uac15\ud574\uc84c\uace0 \ub9c8\uce68\ub0b4 \ucd5c\uc131\uae30\ub97c \ub9de\uc774\ud558\uc600\uc744 \ub54c\ub294 \uac15\ud48d\ubc18\uacbd \ub0b4\uc5d0\uc11c 295km/h(185mph)\uc758 \ubc14\ub78c\uc774 \ubd88\uc5c8\ub2e4. \uc774 \uc218\uce58\ub294 24\uc2dc\uac04\ub9cc\uc5d0 72hPa\uac00 \ub5a8\uc5b4\uc9c4 \uac83\uc73c\ub85c \uae30\ub85d\ub418\uc5c8\uace0 \uc774\ub294 \ub2f9\uc2dc \uc804\uc5d0 \uc77c\uc5b4\ub0ac\ub358 \ubaa8\ub4e0 \ubd81\ub300\uc11c\uc591 \ud5c8\ub9ac\ucf00\uc778\uc744 \ud1b5\ud2c0\uc5b4\uc11c \uac00\uc7a5 \uac15\ud55c \ud5c8\ub9ac\ucf00\uc778\uc774\uc5c8\ub2e4. \uadf8\ub7ec\ub098 \uc774 \uae30\ub85d\uc740 2005\ub144 \uc5d0 \ud5c8\ub9ac\ucf00\uc778 \uc70c\ub9c8\uac00 6hPa \ub0ae\uc740 882hPa\ub97c \uae30\ub85d\ud558\uba74\uc11c \uae68\uc9c0\uac8c \ub418\uc5c8\ub2e4.", "sentence_answer": "\uadf8\ub7ec\ub098 \uc774 \uae30\ub85d\uc740 2005\ub144 \uc5d0", "paragraph_id": "6571114-0-2", "paragraph_question": "question: \ud5c8\ub9ac\ucf00\uc778 \uc70c\ub9c8\uac00 \uae30\ub85d\uc744 \uae6c \ub54c\ub294?, context: \ud5c8\ub9ac\ucf00\uc778 \uae38\ubc84\ud2b8\ub294 \uc790\uba54\uc774\uce74\ub97c \ube60\uc838\ub098\uc628 \ub4a4\uc5d0 \ub2e4\uc2dc \uae09\uaca9\ud558\uac8c \uac15\ud574\uc84c\uc73c\uba70 \ucf00\uc774\ub9e8 \uc81c\ub3c4\uc758 \uac00\uc7a5 \ud070 \uc12c\uc778 \uadf8\ub79c\ub4dc \ucf00\uc774\ub9e8(Grand Cayman)\uc12c\uc758 \uae30\uc0c1\uad00\uce21\uc18c\uc5d0\uc11c\ub294 9\uc6d4 13\uc77c, \uae38\ubc84\ud2b8\uac00 \uc12c \ub0a8\ub3d9\ucabd\uc5d0 \uc704\uce58\ud574\uc788\uc744 \ub54c 252km/h(156mph)\uc5d0 \ub2ec\ud558\ub294 \ub3cc\ud48d\uc774 \uad00\uce21\ub418\uae30\ub3c4 \ud558\uc600\ub2e4. \uae38\ubc84\ud2b8\ub294 \uc911\uc2ec\uae30\uc555\uc774 888hPa\uc5d0 \ub3c4\ub2ec\ud558\uae30\uae4c\uc9c0 \uc26c\uc9c0\uc54a\uace0 \uacc4\uc18d \uac15\ud574\uc84c\uace0 \ub9c8\uce68\ub0b4 \ucd5c\uc131\uae30\ub97c \ub9de\uc774\ud558\uc600\uc744 \ub54c\ub294 \uac15\ud48d\ubc18\uacbd \ub0b4\uc5d0\uc11c 295km/h(185mph)\uc758 \ubc14\ub78c\uc774 \ubd88\uc5c8\ub2e4. \uc774 \uc218\uce58\ub294 24\uc2dc\uac04\ub9cc\uc5d0 72hPa\uac00 \ub5a8\uc5b4\uc9c4 \uac83\uc73c\ub85c \uae30\ub85d\ub418\uc5c8\uace0 \uc774\ub294 \ub2f9\uc2dc \uc804\uc5d0 \uc77c\uc5b4\ub0ac\ub358 \ubaa8\ub4e0 \ubd81\ub300\uc11c\uc591 \ud5c8\ub9ac\ucf00\uc778\uc744 \ud1b5\ud2c0\uc5b4\uc11c \uac00\uc7a5 \uac15\ud55c \ud5c8\ub9ac\ucf00\uc778\uc774\uc5c8\ub2e4. \uadf8\ub7ec\ub098 \uc774 \uae30\ub85d\uc740 2005\ub144\uc5d0 \ud5c8\ub9ac\ucf00\uc778 \uc70c\ub9c8\uac00 6hPa \ub0ae\uc740 882hPa\ub97c \uae30\ub85d\ud558\uba74\uc11c \uae68\uc9c0\uac8c \ub418\uc5c8\ub2e4."} {"question": "\uc0c1\ub300 \ub2f9\uc758 \uc218\ub1cc\ub97c \uacf5\uaca9\ud558\ub294 \ud30c\ub2f9\uc758 \uc804\ud1b5\uc801\uc778 \uad00\ud589\uc744 \uc5c6\uc560\uae30 \uc704\ud55c \uc870\uce58\ub294 \ubb34\uc5c7\uc744 \uc774\uc6a9\ud558\uc600\ub294\uac00?", "paragraph": "\uc601\uc870\uc758 \uc774\ub7ec\ud55c \ub178\ub825\uc73c\ub85c \ud0d5\ud3c9 \uc815\uce58\ub294 \uadf8\uc758 \uc190\uc790\uc778 \uc815\uc870\uc5d0\uac8c\ub85c \uc774\uc5b4\uc9c4\ub2e4. \uc544\ubc84\uc9c0 \uc0ac\ub3c4\uc138\uc790\uc758 \uc8fd\uc74c\uacfc \uc774\ub97c \ub458\ub7ec\uc2fc \uc2dc\ud30c\uc640 \ubcbd\ud30c \uac04\uc758 \uac08\ub4f1\uc744 \uacbd\ud5d8\ud55c \uc815\uc870\ub294 \uc601\uc870\uc758 \ud0d5\ud3c9 \uc815\uce58 \uc758\uc9c0\ub97c \ubc1b\ub4e4\uc5b4 \ub354\uc6b1 \ubc1c\uc804\uc2dc\ucf1c \ub098\uac14\ub2e4. \uc774\ub294 \ub2f9\uc2dc \ub300\uac04\uc744 \uc774\uc6a9\ud558\uc5ec \uc0c1\ub300 \ub2f9\uc758 \uc218\ub1cc\ub97c \uacf5\uaca9\ud558\ub294 \ud30c\ub2f9\uc758 \uc804\ud1b5\uc801\uc778 \uad00\ud589\uc744 \uc5c6\uc560\uae30 \uc704\ud55c \uc870\uce58\uc600\ub2e4. \ub610\ud55c, \ub2f9\uc2dc \ubd95\ub2f9 \uc870\uc131\uc758 \uc8fc\uc694 \ud1b5\ub85c\uc600\ub358 \uc778\uc0ac\uad8c\uc5d0 \uc784\uae08\uc774 \uc9c1\uc811 \uac1c\uc785\ud568\uc73c\ub85c\uc368 \uc870\uc815\uc5d0\uc11c \ub2f9\ud30c\uc758 \uc601\ud5a5\ub825\uc744 \uc904\uc774\uace0 \uc784\uae08\uacfc \uc815\uc2b9\ub4e4\uc774 \uc870\uc815\uc758 \uc8fc\ub3c4\uad8c\uc744 \ud655\ubcf4\ud574\ub098\uac14\ub2e4. \uadf8\ub9ac\uace0 \uc5f0\uc88c\ubc95\uacfc \ub300\uc5ed\uc8c4 \uc801\uc6a9 \ubc94\uc704\ub97c \uc81c\ud55c\ud568\uc73c\ub85c\uc368 \ub300\uc5ed\uc8c4\ub97c \ube59\uc790\ud558\uc5ec \ub2e4\uc218\uc758 \uc0c1\ub300\ub2f9 \uc778\ubb3c\uc744 \uc77c\uc2dc\uc5d0 \ud0c4\ud575\ud558\ub294 \uad00\ud589\uc744 \ucca0\ud3d0\ud588\ub2e4. \ubfd0\ub9cc \uc544\ub2c8\ub77c \uc544\uc608 \uc870\uc815\uc5d0\uc11c \ub300\uc2e0\ub4e4\uc774 \ub2f9\ud30c\ub97c \uc9c0\ubaa9\ud558\uac70\ub098 \ub2f9\ud30c\ub77c\ub294 \uc6a9\uc5b4\ub97c \uc0ac\uc6a9\ud558\ub294 \uac83 \uc790\uccb4\ub97c \uae08\uc9c0\ud568\uc73c\ub85c\uc368 \ud30c\ub2f9\uc758\uc2dd \uc790\uccb4\ub97c \uc5c6\uc560\uace0\uc790 \ud558\uc600\ub2e4. \uc815\uc870\uc758 \uce58\uc138\uc5d0\ub294 \ub178\ub860, \uc18c\ub860, \ub0a8\uc778, \uc18c\ubd81\uc758 \uc0ac\uc0c9\ub2f9\ud30c\uac00 \ubcf4\ud3b8\ud654\ub418\uc5c8\ub2e4.", "answer": "\ub300\uac04", "sentence": "\uc774\ub294 \ub2f9\uc2dc \ub300\uac04 \uc744 \uc774\uc6a9\ud558\uc5ec \uc0c1\ub300 \ub2f9\uc758 \uc218\ub1cc\ub97c \uacf5\uaca9\ud558\ub294 \ud30c\ub2f9\uc758 \uc804\ud1b5\uc801\uc778 \uad00\ud589\uc744 \uc5c6\uc560\uae30 \uc704\ud55c \uc870\uce58\uc600\ub2e4.", "paragraph_sentence": "\uc601\uc870\uc758 \uc774\ub7ec\ud55c \ub178\ub825\uc73c\ub85c \ud0d5\ud3c9 \uc815\uce58\ub294 \uadf8\uc758 \uc190\uc790\uc778 \uc815\uc870\uc5d0\uac8c\ub85c \uc774\uc5b4\uc9c4\ub2e4. \uc544\ubc84\uc9c0 \uc0ac\ub3c4\uc138\uc790\uc758 \uc8fd\uc74c\uacfc \uc774\ub97c \ub458\ub7ec\uc2fc \uc2dc\ud30c\uc640 \ubcbd\ud30c \uac04\uc758 \uac08\ub4f1\uc744 \uacbd\ud5d8\ud55c \uc815\uc870\ub294 \uc601\uc870\uc758 \ud0d5\ud3c9 \uc815\uce58 \uc758\uc9c0\ub97c \ubc1b\ub4e4\uc5b4 \ub354\uc6b1 \ubc1c\uc804\uc2dc\ucf1c \ub098\uac14\ub2e4. \uc774\ub294 \ub2f9\uc2dc \ub300\uac04 \uc744 \uc774\uc6a9\ud558\uc5ec \uc0c1\ub300 \ub2f9\uc758 \uc218\ub1cc\ub97c \uacf5\uaca9\ud558\ub294 \ud30c\ub2f9\uc758 \uc804\ud1b5\uc801\uc778 \uad00\ud589\uc744 \uc5c6\uc560\uae30 \uc704\ud55c \uc870\uce58\uc600\ub2e4. \ub610\ud55c, \ub2f9\uc2dc \ubd95\ub2f9 \uc870\uc131\uc758 \uc8fc\uc694 \ud1b5\ub85c\uc600\ub358 \uc778\uc0ac\uad8c\uc5d0 \uc784\uae08\uc774 \uc9c1\uc811 \uac1c\uc785\ud568\uc73c\ub85c\uc368 \uc870\uc815\uc5d0\uc11c \ub2f9\ud30c\uc758 \uc601\ud5a5\ub825\uc744 \uc904\uc774\uace0 \uc784\uae08\uacfc \uc815\uc2b9\ub4e4\uc774 \uc870\uc815\uc758 \uc8fc\ub3c4\uad8c\uc744 \ud655\ubcf4\ud574\ub098\uac14\ub2e4. \uadf8\ub9ac\uace0 \uc5f0\uc88c\ubc95\uacfc \ub300\uc5ed\uc8c4 \uc801\uc6a9 \ubc94\uc704\ub97c \uc81c\ud55c\ud568\uc73c\ub85c\uc368 \ub300\uc5ed\uc8c4\ub97c \ube59\uc790\ud558\uc5ec \ub2e4\uc218\uc758 \uc0c1\ub300\ub2f9 \uc778\ubb3c\uc744 \uc77c\uc2dc\uc5d0 \ud0c4\ud575\ud558\ub294 \uad00\ud589\uc744 \ucca0\ud3d0\ud588\ub2e4. \ubfd0\ub9cc \uc544\ub2c8\ub77c \uc544\uc608 \uc870\uc815\uc5d0\uc11c \ub300\uc2e0\ub4e4\uc774 \ub2f9\ud30c\ub97c \uc9c0\ubaa9\ud558\uac70\ub098 \ub2f9\ud30c\ub77c\ub294 \uc6a9\uc5b4\ub97c \uc0ac\uc6a9\ud558\ub294 \uac83 \uc790\uccb4\ub97c \uae08\uc9c0\ud568\uc73c\ub85c\uc368 \ud30c\ub2f9\uc758\uc2dd \uc790\uccb4\ub97c \uc5c6\uc560\uace0\uc790 \ud558\uc600\ub2e4. \uc815\uc870\uc758 \uce58\uc138\uc5d0\ub294 \ub178\ub860, \uc18c\ub860, \ub0a8\uc778, \uc18c\ubd81\uc758 \uc0ac\uc0c9\ub2f9\ud30c\uac00 \ubcf4\ud3b8\ud654\ub418\uc5c8\ub2e4.", "paragraph_answer": "\uc601\uc870\uc758 \uc774\ub7ec\ud55c \ub178\ub825\uc73c\ub85c \ud0d5\ud3c9 \uc815\uce58\ub294 \uadf8\uc758 \uc190\uc790\uc778 \uc815\uc870\uc5d0\uac8c\ub85c \uc774\uc5b4\uc9c4\ub2e4. \uc544\ubc84\uc9c0 \uc0ac\ub3c4\uc138\uc790\uc758 \uc8fd\uc74c\uacfc \uc774\ub97c \ub458\ub7ec\uc2fc \uc2dc\ud30c\uc640 \ubcbd\ud30c \uac04\uc758 \uac08\ub4f1\uc744 \uacbd\ud5d8\ud55c \uc815\uc870\ub294 \uc601\uc870\uc758 \ud0d5\ud3c9 \uc815\uce58 \uc758\uc9c0\ub97c \ubc1b\ub4e4\uc5b4 \ub354\uc6b1 \ubc1c\uc804\uc2dc\ucf1c \ub098\uac14\ub2e4. \uc774\ub294 \ub2f9\uc2dc \ub300\uac04 \uc744 \uc774\uc6a9\ud558\uc5ec \uc0c1\ub300 \ub2f9\uc758 \uc218\ub1cc\ub97c \uacf5\uaca9\ud558\ub294 \ud30c\ub2f9\uc758 \uc804\ud1b5\uc801\uc778 \uad00\ud589\uc744 \uc5c6\uc560\uae30 \uc704\ud55c \uc870\uce58\uc600\ub2e4. \ub610\ud55c, \ub2f9\uc2dc \ubd95\ub2f9 \uc870\uc131\uc758 \uc8fc\uc694 \ud1b5\ub85c\uc600\ub358 \uc778\uc0ac\uad8c\uc5d0 \uc784\uae08\uc774 \uc9c1\uc811 \uac1c\uc785\ud568\uc73c\ub85c\uc368 \uc870\uc815\uc5d0\uc11c \ub2f9\ud30c\uc758 \uc601\ud5a5\ub825\uc744 \uc904\uc774\uace0 \uc784\uae08\uacfc \uc815\uc2b9\ub4e4\uc774 \uc870\uc815\uc758 \uc8fc\ub3c4\uad8c\uc744 \ud655\ubcf4\ud574\ub098\uac14\ub2e4. \uadf8\ub9ac\uace0 \uc5f0\uc88c\ubc95\uacfc \ub300\uc5ed\uc8c4 \uc801\uc6a9 \ubc94\uc704\ub97c \uc81c\ud55c\ud568\uc73c\ub85c\uc368 \ub300\uc5ed\uc8c4\ub97c \ube59\uc790\ud558\uc5ec \ub2e4\uc218\uc758 \uc0c1\ub300\ub2f9 \uc778\ubb3c\uc744 \uc77c\uc2dc\uc5d0 \ud0c4\ud575\ud558\ub294 \uad00\ud589\uc744 \ucca0\ud3d0\ud588\ub2e4. \ubfd0\ub9cc \uc544\ub2c8\ub77c \uc544\uc608 \uc870\uc815\uc5d0\uc11c \ub300\uc2e0\ub4e4\uc774 \ub2f9\ud30c\ub97c \uc9c0\ubaa9\ud558\uac70\ub098 \ub2f9\ud30c\ub77c\ub294 \uc6a9\uc5b4\ub97c \uc0ac\uc6a9\ud558\ub294 \uac83 \uc790\uccb4\ub97c \uae08\uc9c0\ud568\uc73c\ub85c\uc368 \ud30c\ub2f9\uc758\uc2dd \uc790\uccb4\ub97c \uc5c6\uc560\uace0\uc790 \ud558\uc600\ub2e4. \uc815\uc870\uc758 \uce58\uc138\uc5d0\ub294 \ub178\ub860, \uc18c\ub860, \ub0a8\uc778, \uc18c\ubd81\uc758 \uc0ac\uc0c9\ub2f9\ud30c\uac00 \ubcf4\ud3b8\ud654\ub418\uc5c8\ub2e4.", "sentence_answer": "\uc774\ub294 \ub2f9\uc2dc \ub300\uac04 \uc744 \uc774\uc6a9\ud558\uc5ec \uc0c1\ub300 \ub2f9\uc758 \uc218\ub1cc\ub97c \uacf5\uaca9\ud558\ub294 \ud30c\ub2f9\uc758 \uc804\ud1b5\uc801\uc778 \uad00\ud589\uc744 \uc5c6\uc560\uae30 \uc704\ud55c \uc870\uce58\uc600\ub2e4.", "paragraph_id": "6187890-11-1", "paragraph_question": "question: \uc0c1\ub300 \ub2f9\uc758 \uc218\ub1cc\ub97c \uacf5\uaca9\ud558\ub294 \ud30c\ub2f9\uc758 \uc804\ud1b5\uc801\uc778 \uad00\ud589\uc744 \uc5c6\uc560\uae30 \uc704\ud55c \uc870\uce58\ub294 \ubb34\uc5c7\uc744 \uc774\uc6a9\ud558\uc600\ub294\uac00?, context: \uc601\uc870\uc758 \uc774\ub7ec\ud55c \ub178\ub825\uc73c\ub85c \ud0d5\ud3c9 \uc815\uce58\ub294 \uadf8\uc758 \uc190\uc790\uc778 \uc815\uc870\uc5d0\uac8c\ub85c \uc774\uc5b4\uc9c4\ub2e4. \uc544\ubc84\uc9c0 \uc0ac\ub3c4\uc138\uc790\uc758 \uc8fd\uc74c\uacfc \uc774\ub97c \ub458\ub7ec\uc2fc \uc2dc\ud30c\uc640 \ubcbd\ud30c \uac04\uc758 \uac08\ub4f1\uc744 \uacbd\ud5d8\ud55c \uc815\uc870\ub294 \uc601\uc870\uc758 \ud0d5\ud3c9 \uc815\uce58 \uc758\uc9c0\ub97c \ubc1b\ub4e4\uc5b4 \ub354\uc6b1 \ubc1c\uc804\uc2dc\ucf1c \ub098\uac14\ub2e4. \uc774\ub294 \ub2f9\uc2dc \ub300\uac04\uc744 \uc774\uc6a9\ud558\uc5ec \uc0c1\ub300 \ub2f9\uc758 \uc218\ub1cc\ub97c \uacf5\uaca9\ud558\ub294 \ud30c\ub2f9\uc758 \uc804\ud1b5\uc801\uc778 \uad00\ud589\uc744 \uc5c6\uc560\uae30 \uc704\ud55c \uc870\uce58\uc600\ub2e4. \ub610\ud55c, \ub2f9\uc2dc \ubd95\ub2f9 \uc870\uc131\uc758 \uc8fc\uc694 \ud1b5\ub85c\uc600\ub358 \uc778\uc0ac\uad8c\uc5d0 \uc784\uae08\uc774 \uc9c1\uc811 \uac1c\uc785\ud568\uc73c\ub85c\uc368 \uc870\uc815\uc5d0\uc11c \ub2f9\ud30c\uc758 \uc601\ud5a5\ub825\uc744 \uc904\uc774\uace0 \uc784\uae08\uacfc \uc815\uc2b9\ub4e4\uc774 \uc870\uc815\uc758 \uc8fc\ub3c4\uad8c\uc744 \ud655\ubcf4\ud574\ub098\uac14\ub2e4. \uadf8\ub9ac\uace0 \uc5f0\uc88c\ubc95\uacfc \ub300\uc5ed\uc8c4 \uc801\uc6a9 \ubc94\uc704\ub97c \uc81c\ud55c\ud568\uc73c\ub85c\uc368 \ub300\uc5ed\uc8c4\ub97c \ube59\uc790\ud558\uc5ec \ub2e4\uc218\uc758 \uc0c1\ub300\ub2f9 \uc778\ubb3c\uc744 \uc77c\uc2dc\uc5d0 \ud0c4\ud575\ud558\ub294 \uad00\ud589\uc744 \ucca0\ud3d0\ud588\ub2e4. \ubfd0\ub9cc \uc544\ub2c8\ub77c \uc544\uc608 \uc870\uc815\uc5d0\uc11c \ub300\uc2e0\ub4e4\uc774 \ub2f9\ud30c\ub97c \uc9c0\ubaa9\ud558\uac70\ub098 \ub2f9\ud30c\ub77c\ub294 \uc6a9\uc5b4\ub97c \uc0ac\uc6a9\ud558\ub294 \uac83 \uc790\uccb4\ub97c \uae08\uc9c0\ud568\uc73c\ub85c\uc368 \ud30c\ub2f9\uc758\uc2dd \uc790\uccb4\ub97c \uc5c6\uc560\uace0\uc790 \ud558\uc600\ub2e4. \uc815\uc870\uc758 \uce58\uc138\uc5d0\ub294 \ub178\ub860, \uc18c\ub860, \ub0a8\uc778, \uc18c\ubd81\uc758 \uc0ac\uc0c9\ub2f9\ud30c\uac00 \ubcf4\ud3b8\ud654\ub418\uc5c8\ub2e4."} {"question": "\uba54\uac74 \ud3ed\uc2a4\uac00 2007\ub144 6\uc6d4 \uc81c\ud504 \ube0c\ub9ac\uc9c0\uc2a4\uc640 \uc0ac\uc774\uba3c \ud398\uadf8 \ub4f1\uacfc \ucd9c\uc5f0\ud55c \uc601\ud654\uc758 \uc81c\ubaa9\uc740?", "paragraph": "2007\ub144 \ub3d9\uba85\uc758 \uc18c\uc124\uc744 \uc6d0\uc791\uc73c\ub85c \ud55c \uc2e4\uc81c\uc561\uc158 \uc601\ud654 \u300a\ud2b8\ub79c\uc2a4\ud3ec\uba38\u300b\uc5d0\uc11c \uc0d8 \uc717\uc705\ud0a4(\uc0e4\uc774\uc544 \ub7ec\ubc84\ud504)\uc758 \uc560\uc815 \uc0c1\ub300 \ubbf8\uce74\uc5d8\ub77c \ubca0\uc778\uc2a4 \uc5ed\uc73c\ub85c \ucd9c\uc5f0\ud55c\ub2e4. \uc601\ud654\ub294 \uc138\uacc4\uc801\uc73c\ub85c 7\uc5b5 970\ub9cc \ub2ec\ub7ec\ub97c \ubc8c\uc5b4\ub4e4\uc774\uba74\uc11c \ud765\ud589\uc5d0 \uc131\uacf5\ud588\uace0, \ud3ed\uc2a4\ub294 \uc139\uc2dc\ud55c \uc774\ubbf8\uc9c0\ub85c \uc138\uacc4\uc801\uc778 \uc2a4\ud0c0\ub364\uc5d0 \uc62c\ub790\ub2e4. MTV \ubb34\ube44 \uc5b4\uc6cc\ub4dc\uc5d0\uc11c \uc2e0\uc778\uc0c1\uacfc \ud2f4 \ucd08\uc774\uc2a4 \uc5b4\uc6cc\ub4dc\uc5d0\uc11c \ucd08\uc774\uc2a4 \uc601\ud654 \uc5ec\ubc30\uc6b0: \uc561\uc158 \uc5b4\ub4dc\ubca4\ucc98, \ucd08\uc774\uc2a4 \uc601\ud654: \uc5ec\uc790 \uc2e0\uc778, \ucd08\uc774\uc2a4 \uc601\ud654: \ub9bd\ub85d \ubd80\ubb38 \ud6c4\ubcf4\uc5d0 \uc62c\ub790\uc9c0\ub9cc, \uc218\uc0c1\uc5d0\ub294 \uc2e4\ud328\ud588\ub2e4. \uc774\ud6c4 \ud3ed\uc2a4\ub294 \u300a\ud2b8\ub79c\uc2a4\ud3ec\uba38\u300b \uc601\ud654 \uc2dc\ub9ac\uc988\uc5d0 \ub450 \ud3b8 \ub354 \ucd9c\uc5f0\ud558\uae30\ub85c \uacc4\uc57d\ud588\ub2e4. \uac19\uc740 \ud574 6\uc6d4 \uc81c\ud504 \ube0c\ub9ac\uc9c0\uc2a4, \uc0ac\uc774\uba3c \ud398\uadf8, \ucee4\uc2a4\ud2f4 \ub358\uc2a4\ud2b8\uc640 \ud568\uaed8 \u300a\ud558\uc6b0 \ud22c \ub8e8\uc988 \ud504\ub80c\uc988\u300b\uc5d0 \uc18c\ud53c \ub9e4\uc2a4 \uc5ed\uc73c\ub85c \ucd9c\uc5f0 \ud588\uc73c\ub098, \ubc15\uc2a4\uc624\ud53c\uc2a4 \ud765\ud589 \uc2e4\ud328\ub85c \ub05d\ub0ac\ub2e4.", "answer": "\ud558\uc6b0 \ud22c \ub8e8\uc988 \ud504\ub80c\uc988", "sentence": "\uac19\uc740 \ud574 6\uc6d4 \uc81c\ud504 \ube0c\ub9ac\uc9c0\uc2a4, \uc0ac\uc774\uba3c \ud398\uadf8, \ucee4\uc2a4\ud2f4 \ub358\uc2a4\ud2b8\uc640 \ud568\uaed8 \u300a \ud558\uc6b0 \ud22c \ub8e8\uc988 \ud504\ub80c\uc988 \u300b\uc5d0 \uc18c\ud53c \ub9e4\uc2a4 \uc5ed\uc73c\ub85c \ucd9c\uc5f0 \ud588\uc73c\ub098, \ubc15\uc2a4\uc624\ud53c\uc2a4 \ud765\ud589 \uc2e4\ud328\ub85c \ub05d\ub0ac\ub2e4.", "paragraph_sentence": "2007\ub144 \ub3d9\uba85\uc758 \uc18c\uc124\uc744 \uc6d0\uc791\uc73c\ub85c \ud55c \uc2e4\uc81c\uc561\uc158 \uc601\ud654 \u300a\ud2b8\ub79c\uc2a4\ud3ec\uba38\u300b\uc5d0\uc11c \uc0d8 \uc717\uc705\ud0a4(\uc0e4\uc774\uc544 \ub7ec\ubc84\ud504)\uc758 \uc560\uc815 \uc0c1\ub300 \ubbf8\uce74\uc5d8\ub77c \ubca0\uc778\uc2a4 \uc5ed\uc73c\ub85c \ucd9c\uc5f0\ud55c\ub2e4. \uc601\ud654\ub294 \uc138\uacc4\uc801\uc73c\ub85c 7\uc5b5 970\ub9cc \ub2ec\ub7ec\ub97c \ubc8c\uc5b4\ub4e4\uc774\uba74\uc11c \ud765\ud589\uc5d0 \uc131\uacf5\ud588\uace0, \ud3ed\uc2a4\ub294 \uc139\uc2dc\ud55c \uc774\ubbf8\uc9c0\ub85c \uc138\uacc4\uc801\uc778 \uc2a4\ud0c0\ub364\uc5d0 \uc62c\ub790\ub2e4. MTV \ubb34\ube44 \uc5b4\uc6cc\ub4dc\uc5d0\uc11c \uc2e0\uc778\uc0c1\uacfc \ud2f4 \ucd08\uc774\uc2a4 \uc5b4\uc6cc\ub4dc\uc5d0\uc11c \ucd08\uc774\uc2a4 \uc601\ud654 \uc5ec\ubc30\uc6b0: \uc561\uc158 \uc5b4\ub4dc\ubca4\ucc98, \ucd08\uc774\uc2a4 \uc601\ud654: \uc5ec\uc790 \uc2e0\uc778, \ucd08\uc774\uc2a4 \uc601\ud654: \ub9bd\ub85d \ubd80\ubb38 \ud6c4\ubcf4\uc5d0 \uc62c\ub790\uc9c0\ub9cc, \uc218\uc0c1\uc5d0\ub294 \uc2e4\ud328\ud588\ub2e4. \uc774\ud6c4 \ud3ed\uc2a4\ub294 \u300a\ud2b8\ub79c\uc2a4\ud3ec\uba38\u300b \uc601\ud654 \uc2dc\ub9ac\uc988\uc5d0 \ub450 \ud3b8 \ub354 \ucd9c\uc5f0\ud558\uae30\ub85c \uacc4\uc57d\ud588\ub2e4. \uac19\uc740 \ud574 6\uc6d4 \uc81c\ud504 \ube0c\ub9ac\uc9c0\uc2a4, \uc0ac\uc774\uba3c \ud398\uadf8, \ucee4\uc2a4\ud2f4 \ub358\uc2a4\ud2b8\uc640 \ud568\uaed8 \u300a \ud558\uc6b0 \ud22c \ub8e8\uc988 \ud504\ub80c\uc988 \u300b\uc5d0 \uc18c\ud53c \ub9e4\uc2a4 \uc5ed\uc73c\ub85c \ucd9c\uc5f0 \ud588\uc73c\ub098, \ubc15\uc2a4\uc624\ud53c\uc2a4 \ud765\ud589 \uc2e4\ud328\ub85c \ub05d\ub0ac\ub2e4. ", "paragraph_answer": "2007\ub144 \ub3d9\uba85\uc758 \uc18c\uc124\uc744 \uc6d0\uc791\uc73c\ub85c \ud55c \uc2e4\uc81c\uc561\uc158 \uc601\ud654 \u300a\ud2b8\ub79c\uc2a4\ud3ec\uba38\u300b\uc5d0\uc11c \uc0d8 \uc717\uc705\ud0a4(\uc0e4\uc774\uc544 \ub7ec\ubc84\ud504)\uc758 \uc560\uc815 \uc0c1\ub300 \ubbf8\uce74\uc5d8\ub77c \ubca0\uc778\uc2a4 \uc5ed\uc73c\ub85c \ucd9c\uc5f0\ud55c\ub2e4. \uc601\ud654\ub294 \uc138\uacc4\uc801\uc73c\ub85c 7\uc5b5 970\ub9cc \ub2ec\ub7ec\ub97c \ubc8c\uc5b4\ub4e4\uc774\uba74\uc11c \ud765\ud589\uc5d0 \uc131\uacf5\ud588\uace0, \ud3ed\uc2a4\ub294 \uc139\uc2dc\ud55c \uc774\ubbf8\uc9c0\ub85c \uc138\uacc4\uc801\uc778 \uc2a4\ud0c0\ub364\uc5d0 \uc62c\ub790\ub2e4. MTV \ubb34\ube44 \uc5b4\uc6cc\ub4dc\uc5d0\uc11c \uc2e0\uc778\uc0c1\uacfc \ud2f4 \ucd08\uc774\uc2a4 \uc5b4\uc6cc\ub4dc\uc5d0\uc11c \ucd08\uc774\uc2a4 \uc601\ud654 \uc5ec\ubc30\uc6b0: \uc561\uc158 \uc5b4\ub4dc\ubca4\ucc98, \ucd08\uc774\uc2a4 \uc601\ud654: \uc5ec\uc790 \uc2e0\uc778, \ucd08\uc774\uc2a4 \uc601\ud654: \ub9bd\ub85d \ubd80\ubb38 \ud6c4\ubcf4\uc5d0 \uc62c\ub790\uc9c0\ub9cc, \uc218\uc0c1\uc5d0\ub294 \uc2e4\ud328\ud588\ub2e4. \uc774\ud6c4 \ud3ed\uc2a4\ub294 \u300a\ud2b8\ub79c\uc2a4\ud3ec\uba38\u300b \uc601\ud654 \uc2dc\ub9ac\uc988\uc5d0 \ub450 \ud3b8 \ub354 \ucd9c\uc5f0\ud558\uae30\ub85c \uacc4\uc57d\ud588\ub2e4. \uac19\uc740 \ud574 6\uc6d4 \uc81c\ud504 \ube0c\ub9ac\uc9c0\uc2a4, \uc0ac\uc774\uba3c \ud398\uadf8, \ucee4\uc2a4\ud2f4 \ub358\uc2a4\ud2b8\uc640 \ud568\uaed8 \u300a \ud558\uc6b0 \ud22c \ub8e8\uc988 \ud504\ub80c\uc988 \u300b\uc5d0 \uc18c\ud53c \ub9e4\uc2a4 \uc5ed\uc73c\ub85c \ucd9c\uc5f0 \ud588\uc73c\ub098, \ubc15\uc2a4\uc624\ud53c\uc2a4 \ud765\ud589 \uc2e4\ud328\ub85c \ub05d\ub0ac\ub2e4.", "sentence_answer": "\uac19\uc740 \ud574 6\uc6d4 \uc81c\ud504 \ube0c\ub9ac\uc9c0\uc2a4, \uc0ac\uc774\uba3c \ud398\uadf8, \ucee4\uc2a4\ud2f4 \ub358\uc2a4\ud2b8\uc640 \ud568\uaed8 \u300a \ud558\uc6b0 \ud22c \ub8e8\uc988 \ud504\ub80c\uc988 \u300b\uc5d0 \uc18c\ud53c \ub9e4\uc2a4 \uc5ed\uc73c\ub85c \ucd9c\uc5f0 \ud588\uc73c\ub098, \ubc15\uc2a4\uc624\ud53c\uc2a4 \ud765\ud589 \uc2e4\ud328\ub85c \ub05d\ub0ac\ub2e4.", "paragraph_id": "6502694-1-1", "paragraph_question": "question: \uba54\uac74 \ud3ed\uc2a4\uac00 2007\ub144 6\uc6d4 \uc81c\ud504 \ube0c\ub9ac\uc9c0\uc2a4\uc640 \uc0ac\uc774\uba3c \ud398\uadf8 \ub4f1\uacfc \ucd9c\uc5f0\ud55c \uc601\ud654\uc758 \uc81c\ubaa9\uc740?, context: 2007\ub144 \ub3d9\uba85\uc758 \uc18c\uc124\uc744 \uc6d0\uc791\uc73c\ub85c \ud55c \uc2e4\uc81c\uc561\uc158 \uc601\ud654 \u300a\ud2b8\ub79c\uc2a4\ud3ec\uba38\u300b\uc5d0\uc11c \uc0d8 \uc717\uc705\ud0a4(\uc0e4\uc774\uc544 \ub7ec\ubc84\ud504)\uc758 \uc560\uc815 \uc0c1\ub300 \ubbf8\uce74\uc5d8\ub77c \ubca0\uc778\uc2a4 \uc5ed\uc73c\ub85c \ucd9c\uc5f0\ud55c\ub2e4. \uc601\ud654\ub294 \uc138\uacc4\uc801\uc73c\ub85c 7\uc5b5 970\ub9cc \ub2ec\ub7ec\ub97c \ubc8c\uc5b4\ub4e4\uc774\uba74\uc11c \ud765\ud589\uc5d0 \uc131\uacf5\ud588\uace0, \ud3ed\uc2a4\ub294 \uc139\uc2dc\ud55c \uc774\ubbf8\uc9c0\ub85c \uc138\uacc4\uc801\uc778 \uc2a4\ud0c0\ub364\uc5d0 \uc62c\ub790\ub2e4. MTV \ubb34\ube44 \uc5b4\uc6cc\ub4dc\uc5d0\uc11c \uc2e0\uc778\uc0c1\uacfc \ud2f4 \ucd08\uc774\uc2a4 \uc5b4\uc6cc\ub4dc\uc5d0\uc11c \ucd08\uc774\uc2a4 \uc601\ud654 \uc5ec\ubc30\uc6b0: \uc561\uc158 \uc5b4\ub4dc\ubca4\ucc98, \ucd08\uc774\uc2a4 \uc601\ud654: \uc5ec\uc790 \uc2e0\uc778, \ucd08\uc774\uc2a4 \uc601\ud654: \ub9bd\ub85d \ubd80\ubb38 \ud6c4\ubcf4\uc5d0 \uc62c\ub790\uc9c0\ub9cc, \uc218\uc0c1\uc5d0\ub294 \uc2e4\ud328\ud588\ub2e4. \uc774\ud6c4 \ud3ed\uc2a4\ub294 \u300a\ud2b8\ub79c\uc2a4\ud3ec\uba38\u300b \uc601\ud654 \uc2dc\ub9ac\uc988\uc5d0 \ub450 \ud3b8 \ub354 \ucd9c\uc5f0\ud558\uae30\ub85c \uacc4\uc57d\ud588\ub2e4. \uac19\uc740 \ud574 6\uc6d4 \uc81c\ud504 \ube0c\ub9ac\uc9c0\uc2a4, \uc0ac\uc774\uba3c \ud398\uadf8, \ucee4\uc2a4\ud2f4 \ub358\uc2a4\ud2b8\uc640 \ud568\uaed8 \u300a\ud558\uc6b0 \ud22c \ub8e8\uc988 \ud504\ub80c\uc988\u300b\uc5d0 \uc18c\ud53c \ub9e4\uc2a4 \uc5ed\uc73c\ub85c \ucd9c\uc5f0 \ud588\uc73c\ub098, \ubc15\uc2a4\uc624\ud53c\uc2a4 \ud765\ud589 \uc2e4\ud328\ub85c \ub05d\ub0ac\ub2e4."} {"question": "\uae30\uc5c5 \ubd80\ub2f4\uc744 \uac00\uc911\uc2dc\ud0a4\uace0 \uacbd\uc81c \uc0c1\ud669\uc744 \uc5b4\ub835\uac8c \ub9cc\ub4e4 \uc218 \uc788\ub2e4\uace0 \ubc18\ubc1c\uc744 \uc0b0 \ubc95\uc740 \ubb34\uc5c7\uc778\uac00?", "paragraph": "2013\ub144 4\uc6d4 30\uc77c\uc5d0 \uae40\uc131\ud0dc\ub294 '\uace0\uc6a9\uc0c1 \uc5f0\ub839\ucc28\ubcc4 \ubc0f \uace0\ub839\uc790 \uace0\uc6a9\ucd09\uc9c4\uc5d0 \uad00\ud55c \ubc95\ub960' \uc77c\ubd80 \uac1c\uc815\uc548\uc744 \ud1b5\uacfc\uc2dc\ucf30\ub2e4. \uc774\ub7ec\ud55c \uc815\ub144\uc5f0\uc7a5\ubc95\uc740 \uae30\uc5c5 \ubd80\ub2f4\uc744 \uac00\uc911\uc2dc\ucf1c \uacbd\uc81c \uc0c1\ud669\uc744 \uc5b4\ub835\uac8c \ub9cc\ub4e4 \uc218 \uc788\ub2e4\ub294 \uacbd\uc81c\ub2e8\uccb4\ub4e4\uc758 \ubc18\ubc1c\uc774 \uc788\uc5c8\uace0, \uad6c\uccb4\uc801\uc778 \uc870\ubb38\uc744 \ub193\uace0 \ubc95\uc0ac\uc704\uc5d0\uc11c\ub3c4 \uc27d\uc9c0 \uc54a\uc740 \ub17c\uc758\uac00 \uc624\uac00\uae30\ub3c4 \ud588\ub2e4\uace0 \ud55c\ub2e4. \uae40\uc131\ud0dc\ub294 \uc815\ub144\uc5f0\uc7a5 \ud1b5\uacfc\uc758 \uc758\ubbf8\uc5d0 \ub300\ud574 \"\uc815\ub144 60\uc138 \uc5f0\uc7a5\uc740 \uc911\uc7a5\ub144\uce35\uc758 \uace0\uc6a9 \uc548\uc815\uc740 \ubb3c\ub860 \uc0c8\uc815\ubd80\uc758 \uad6d\uc815\uacfc\uc81c\uc778 \uace0\uc6a9\ub960 70% \ub2ec\uc131\uc5d0\ub3c4 \ud06c\uac8c \uae30\uc5ec\ud560 \uac83\"\uc774\ub77c\uba70 \"\uc55e\uc73c\ub85c\ub3c4 \uc18c\uc678\ubc1b\ub294 \uc0ac\ud68c\uc801 \uc57d\uc790\uc640 \ucde8\uc57d\uacc4\uce35\uc744 \uc704\ud574 \ub354 \uc18c\uc2e0 \uc788\uac8c \uc758\uc815\ud65c\ub3d9\uc744 \ubc8c\uc774\uaca0\ub2e4\"\uace0 \ub9d0\ud588\ub2e4. \uc774\ucc98\ub7fc \uc0ac\ud68c \uc548\uc804\ub9dd \uad6c\ucd95\uacfc \ub178\ud6c4 \ubcf5\uc9c0\uc758 \ud5a5\uc0c1\uc744 \uc704\ud55c \uc785\ubc95\ud65c\ub3d9\uc774 \uc788\uc5c8\uc73c\ub098 \uccad\ub144 \uc77c\uc790\ub9ac \uac10\uc18c\uc640 \uae30\uc5c5 \uacbd\uc81c\ud65c\ub3d9 \uc704\ucd95\uc5d0 \ub300\ud55c \ub17c\ub780\uc774 \uc788\uc5c8\ub2e4.", "answer": "\uc815\ub144\uc5f0\uc7a5\ubc95", "sentence": "\uc774\ub7ec\ud55c \uc815\ub144\uc5f0\uc7a5\ubc95 \uc740 \uae30\uc5c5 \ubd80\ub2f4\uc744 \uac00\uc911\uc2dc\ucf1c \uacbd\uc81c \uc0c1\ud669\uc744 \uc5b4\ub835\uac8c \ub9cc\ub4e4 \uc218 \uc788\ub2e4\ub294 \uacbd\uc81c\ub2e8\uccb4\ub4e4\uc758 \ubc18\ubc1c\uc774 \uc788\uc5c8\uace0, \uad6c\uccb4\uc801\uc778 \uc870\ubb38\uc744 \ub193\uace0 \ubc95\uc0ac\uc704\uc5d0\uc11c\ub3c4 \uc27d\uc9c0 \uc54a\uc740 \ub17c\uc758\uac00 \uc624\uac00\uae30\ub3c4 \ud588\ub2e4\uace0 \ud55c\ub2e4.", "paragraph_sentence": "2013\ub144 4\uc6d4 30\uc77c\uc5d0 \uae40\uc131\ud0dc\ub294 '\uace0\uc6a9\uc0c1 \uc5f0\ub839\ucc28\ubcc4 \ubc0f \uace0\ub839\uc790 \uace0\uc6a9\ucd09\uc9c4\uc5d0 \uad00\ud55c \ubc95\ub960' \uc77c\ubd80 \uac1c\uc815\uc548\uc744 \ud1b5\uacfc\uc2dc\ucf30\ub2e4. \uc774\ub7ec\ud55c \uc815\ub144\uc5f0\uc7a5\ubc95 \uc740 \uae30\uc5c5 \ubd80\ub2f4\uc744 \uac00\uc911\uc2dc\ucf1c \uacbd\uc81c \uc0c1\ud669\uc744 \uc5b4\ub835\uac8c \ub9cc\ub4e4 \uc218 \uc788\ub2e4\ub294 \uacbd\uc81c\ub2e8\uccb4\ub4e4\uc758 \ubc18\ubc1c\uc774 \uc788\uc5c8\uace0, \uad6c\uccb4\uc801\uc778 \uc870\ubb38\uc744 \ub193\uace0 \ubc95\uc0ac\uc704\uc5d0\uc11c\ub3c4 \uc27d\uc9c0 \uc54a\uc740 \ub17c\uc758\uac00 \uc624\uac00\uae30\ub3c4 \ud588\ub2e4\uace0 \ud55c\ub2e4. \uae40\uc131\ud0dc\ub294 \uc815\ub144\uc5f0\uc7a5 \ud1b5\uacfc\uc758 \uc758\ubbf8\uc5d0 \ub300\ud574 \"\uc815\ub144 60\uc138 \uc5f0\uc7a5\uc740 \uc911\uc7a5\ub144\uce35\uc758 \uace0\uc6a9 \uc548\uc815\uc740 \ubb3c\ub860 \uc0c8\uc815\ubd80\uc758 \uad6d\uc815\uacfc\uc81c\uc778 \uace0\uc6a9\ub960 70% \ub2ec\uc131\uc5d0\ub3c4 \ud06c\uac8c \uae30\uc5ec\ud560 \uac83\"\uc774\ub77c\uba70 \"\uc55e\uc73c\ub85c\ub3c4 \uc18c\uc678\ubc1b\ub294 \uc0ac\ud68c\uc801 \uc57d\uc790\uc640 \ucde8\uc57d\uacc4\uce35\uc744 \uc704\ud574 \ub354 \uc18c\uc2e0 \uc788\uac8c \uc758\uc815\ud65c\ub3d9\uc744 \ubc8c\uc774\uaca0\ub2e4\"\uace0 \ub9d0\ud588\ub2e4. \uc774\ucc98\ub7fc \uc0ac\ud68c \uc548\uc804\ub9dd \uad6c\ucd95\uacfc \ub178\ud6c4 \ubcf5\uc9c0\uc758 \ud5a5\uc0c1\uc744 \uc704\ud55c \uc785\ubc95\ud65c\ub3d9\uc774 \uc788\uc5c8\uc73c\ub098 \uccad\ub144 \uc77c\uc790\ub9ac \uac10\uc18c\uc640 \uae30\uc5c5 \uacbd\uc81c\ud65c\ub3d9 \uc704\ucd95\uc5d0 \ub300\ud55c \ub17c\ub780\uc774 \uc788\uc5c8\ub2e4.", "paragraph_answer": "2013\ub144 4\uc6d4 30\uc77c\uc5d0 \uae40\uc131\ud0dc\ub294 '\uace0\uc6a9\uc0c1 \uc5f0\ub839\ucc28\ubcc4 \ubc0f \uace0\ub839\uc790 \uace0\uc6a9\ucd09\uc9c4\uc5d0 \uad00\ud55c \ubc95\ub960' \uc77c\ubd80 \uac1c\uc815\uc548\uc744 \ud1b5\uacfc\uc2dc\ucf30\ub2e4. \uc774\ub7ec\ud55c \uc815\ub144\uc5f0\uc7a5\ubc95 \uc740 \uae30\uc5c5 \ubd80\ub2f4\uc744 \uac00\uc911\uc2dc\ucf1c \uacbd\uc81c \uc0c1\ud669\uc744 \uc5b4\ub835\uac8c \ub9cc\ub4e4 \uc218 \uc788\ub2e4\ub294 \uacbd\uc81c\ub2e8\uccb4\ub4e4\uc758 \ubc18\ubc1c\uc774 \uc788\uc5c8\uace0, \uad6c\uccb4\uc801\uc778 \uc870\ubb38\uc744 \ub193\uace0 \ubc95\uc0ac\uc704\uc5d0\uc11c\ub3c4 \uc27d\uc9c0 \uc54a\uc740 \ub17c\uc758\uac00 \uc624\uac00\uae30\ub3c4 \ud588\ub2e4\uace0 \ud55c\ub2e4. \uae40\uc131\ud0dc\ub294 \uc815\ub144\uc5f0\uc7a5 \ud1b5\uacfc\uc758 \uc758\ubbf8\uc5d0 \ub300\ud574 \"\uc815\ub144 60\uc138 \uc5f0\uc7a5\uc740 \uc911\uc7a5\ub144\uce35\uc758 \uace0\uc6a9 \uc548\uc815\uc740 \ubb3c\ub860 \uc0c8\uc815\ubd80\uc758 \uad6d\uc815\uacfc\uc81c\uc778 \uace0\uc6a9\ub960 70% \ub2ec\uc131\uc5d0\ub3c4 \ud06c\uac8c \uae30\uc5ec\ud560 \uac83\"\uc774\ub77c\uba70 \"\uc55e\uc73c\ub85c\ub3c4 \uc18c\uc678\ubc1b\ub294 \uc0ac\ud68c\uc801 \uc57d\uc790\uc640 \ucde8\uc57d\uacc4\uce35\uc744 \uc704\ud574 \ub354 \uc18c\uc2e0 \uc788\uac8c \uc758\uc815\ud65c\ub3d9\uc744 \ubc8c\uc774\uaca0\ub2e4\"\uace0 \ub9d0\ud588\ub2e4. \uc774\ucc98\ub7fc \uc0ac\ud68c \uc548\uc804\ub9dd \uad6c\ucd95\uacfc \ub178\ud6c4 \ubcf5\uc9c0\uc758 \ud5a5\uc0c1\uc744 \uc704\ud55c \uc785\ubc95\ud65c\ub3d9\uc774 \uc788\uc5c8\uc73c\ub098 \uccad\ub144 \uc77c\uc790\ub9ac \uac10\uc18c\uc640 \uae30\uc5c5 \uacbd\uc81c\ud65c\ub3d9 \uc704\ucd95\uc5d0 \ub300\ud55c \ub17c\ub780\uc774 \uc788\uc5c8\ub2e4.", "sentence_answer": "\uc774\ub7ec\ud55c \uc815\ub144\uc5f0\uc7a5\ubc95 \uc740 \uae30\uc5c5 \ubd80\ub2f4\uc744 \uac00\uc911\uc2dc\ucf1c \uacbd\uc81c \uc0c1\ud669\uc744 \uc5b4\ub835\uac8c \ub9cc\ub4e4 \uc218 \uc788\ub2e4\ub294 \uacbd\uc81c\ub2e8\uccb4\ub4e4\uc758 \ubc18\ubc1c\uc774 \uc788\uc5c8\uace0, \uad6c\uccb4\uc801\uc778 \uc870\ubb38\uc744 \ub193\uace0 \ubc95\uc0ac\uc704\uc5d0\uc11c\ub3c4 \uc27d\uc9c0 \uc54a\uc740 \ub17c\uc758\uac00 \uc624\uac00\uae30\ub3c4 \ud588\ub2e4\uace0 \ud55c\ub2e4.", "paragraph_id": "6577053-0-1", "paragraph_question": "question: \uae30\uc5c5 \ubd80\ub2f4\uc744 \uac00\uc911\uc2dc\ud0a4\uace0 \uacbd\uc81c \uc0c1\ud669\uc744 \uc5b4\ub835\uac8c \ub9cc\ub4e4 \uc218 \uc788\ub2e4\uace0 \ubc18\ubc1c\uc744 \uc0b0 \ubc95\uc740 \ubb34\uc5c7\uc778\uac00?, context: 2013\ub144 4\uc6d4 30\uc77c\uc5d0 \uae40\uc131\ud0dc\ub294 '\uace0\uc6a9\uc0c1 \uc5f0\ub839\ucc28\ubcc4 \ubc0f \uace0\ub839\uc790 \uace0\uc6a9\ucd09\uc9c4\uc5d0 \uad00\ud55c \ubc95\ub960' \uc77c\ubd80 \uac1c\uc815\uc548\uc744 \ud1b5\uacfc\uc2dc\ucf30\ub2e4. \uc774\ub7ec\ud55c \uc815\ub144\uc5f0\uc7a5\ubc95\uc740 \uae30\uc5c5 \ubd80\ub2f4\uc744 \uac00\uc911\uc2dc\ucf1c \uacbd\uc81c \uc0c1\ud669\uc744 \uc5b4\ub835\uac8c \ub9cc\ub4e4 \uc218 \uc788\ub2e4\ub294 \uacbd\uc81c\ub2e8\uccb4\ub4e4\uc758 \ubc18\ubc1c\uc774 \uc788\uc5c8\uace0, \uad6c\uccb4\uc801\uc778 \uc870\ubb38\uc744 \ub193\uace0 \ubc95\uc0ac\uc704\uc5d0\uc11c\ub3c4 \uc27d\uc9c0 \uc54a\uc740 \ub17c\uc758\uac00 \uc624\uac00\uae30\ub3c4 \ud588\ub2e4\uace0 \ud55c\ub2e4. \uae40\uc131\ud0dc\ub294 \uc815\ub144\uc5f0\uc7a5 \ud1b5\uacfc\uc758 \uc758\ubbf8\uc5d0 \ub300\ud574 \"\uc815\ub144 60\uc138 \uc5f0\uc7a5\uc740 \uc911\uc7a5\ub144\uce35\uc758 \uace0\uc6a9 \uc548\uc815\uc740 \ubb3c\ub860 \uc0c8\uc815\ubd80\uc758 \uad6d\uc815\uacfc\uc81c\uc778 \uace0\uc6a9\ub960 70% \ub2ec\uc131\uc5d0\ub3c4 \ud06c\uac8c \uae30\uc5ec\ud560 \uac83\"\uc774\ub77c\uba70 \"\uc55e\uc73c\ub85c\ub3c4 \uc18c\uc678\ubc1b\ub294 \uc0ac\ud68c\uc801 \uc57d\uc790\uc640 \ucde8\uc57d\uacc4\uce35\uc744 \uc704\ud574 \ub354 \uc18c\uc2e0 \uc788\uac8c \uc758\uc815\ud65c\ub3d9\uc744 \ubc8c\uc774\uaca0\ub2e4\"\uace0 \ub9d0\ud588\ub2e4. \uc774\ucc98\ub7fc \uc0ac\ud68c \uc548\uc804\ub9dd \uad6c\ucd95\uacfc \ub178\ud6c4 \ubcf5\uc9c0\uc758 \ud5a5\uc0c1\uc744 \uc704\ud55c \uc785\ubc95\ud65c\ub3d9\uc774 \uc788\uc5c8\uc73c\ub098 \uccad\ub144 \uc77c\uc790\ub9ac \uac10\uc18c\uc640 \uae30\uc5c5 \uacbd\uc81c\ud65c\ub3d9 \uc704\ucd95\uc5d0 \ub300\ud55c \ub17c\ub780\uc774 \uc788\uc5c8\ub2e4."} {"question": "\ud0c0\ube14\ub77c\ub9ac\ub514\uc2a4\uac00 10\ubd84\ub9cc\uc5d0 \ud1f4\uc7a5\ub2f9\ud55c \uacbd\uae30\uc758 \uc0c1\ub300\ud300\uc740?", "paragraph": "2010\ub144 4\uc6d4 30\uc77c, \ub77c\ub9ac\uc0ac\ub294 29\uc138 \uc13c\ud130\ubc31\uacfc \ud611\uc0c1\uc5d0 \uc131\uacf5\ud558\uc5ec \uc774 \ud574 \uc5ec\ub984\uc5d0 \ub179\uc0c9 \uad70\ub2e8\uacfc\uc758 \uacc4\uc57d\uc774 \ub9cc\ub8cc\ub418\uba74 \uc790\uc720\uc774\uc801\uc73c\ub85c \ud300\uc5d0 \ud569\ub958\ud558\uac8c \ub420 \uac83\uc774\ub77c \ubc1c\ud45c\ud558\uc600\ub2e4. \ud504\ub791\uc2a4\uc5d0\uc11c \ud669\uc18c\uac19\uc740 \ud50c\ub808\uc774\uc2a4\ud0c0\uc77c\ub85c '\ud669\uc18c' (Le Taureau) \ub77c\ub294 \ubcc4\uce6d\uc744 \uc5bb\uc740 \uadf8\ub294 \ub9b4\uacfc \uc0dd\ud14c\ud2f0\uc5d4 \uc120\uc218\ub85c \ud65c\uc57d\ud574 \ub9ac\uadf8 1\uc5d0\uc11c \uc131\uacf5\ud55c \uadf8\ub9ac\uc2a4 \uc120\uc218\uc774\ub098, \uc54c\ub7ad \ud398\ub7ad \uac10\ub3c5\uc5d0 \uc758\ud574 \uc2a4\ud0c0\ub4dc \uc870\ud504\ub9ac \uadc0\uc0e4\ub974 \uc5f0\uace0 \ud300\uc758 \uba85\ub2e8\uc5d0\uc11c \ubc00\ub824\ub0ac\ub2e4. \uc804 \uc62c\ub7ad\ud53c\ud06c \ub9ac\uc639 \uac10\ub3c5\uc774 \uacbd\uc9c8\ub41c \ud6c4, \ud0c0\ube14\ub77c\ub9ac\ub514\uc2a4\ub294 \ud06c\ub9ac\uc2a4\ud1a0\ud504 \uac08\ud2f0\uc5d0\ub974 \uac10\ub3c5 \ud558\uc5d0 \uc778\uc0c1\uc744 \uc904 \uae30\ud68c\ub97c \uc7a1\uc558\uc73c\ub098, \uac08\ud2f0\uc5d0\ub974 \uc784\uae30\uc758 \uccab \uacbd\uae30\uc778 \ub9c8\ub974\uc138\uc720\uc804\uc5d0\uc11c 10\ubd84\ub9cc\uc5d0 \ud1f4\uc7a5\ub2f9\ud558\uba74\uc11c \ub179\uc0c9 \uad70\ub2e8\uc73c\ub85c\uc368\uc758 \u314e\ub880\uc57d\uc774 \uc0ac\uc2e4\uc0c1 \ub05d\ub098\ubc84\ub838\ub2e4. \ub098\uc911\uc5d0 \ud569\ub958\ud560 \uad6c\ub2e8\uc758 \uacf5\uc2dd \uc6f9\uc0ac\uc774\ud2b8\uc5d0\uc11c, \uadf8\ub9ac\uc2a4 \uc778\ud130\ub137 \uc5b8\ub860\uc5d0 \uadf8\ub294 \"\uc800\ub294 \uc81c \uacbd\ub825\uc744 \ud574\uc678\uc5d0\uc11c \uc774\uc5b4\ub098\uac00\ub294 \uac83\uc744 \uc6b0\uc120\uc21c\uc704\ub85c \uc7a1\uc558\uc5c8\uc2b5\ub2c8\ub2e4. \uc800\ub294 \uadf8\ub9ac\uc2a4\uc5d0 \ubcf5\uadc0\ud560 \uc5ec\ubd80\ub97c 4\uc6d4\uae4c\uc9c0 \uacb0\uc815\ud588\uc73c\uba70, 9\ub144\ub9cc\uc758 \ubcf5\uadc0\ub294 \uc81c\uac00 \uac00\uc871\uc744 \uac70\ub290\ub9ac\ub294 \uac83\uc73c\ub85c \uc778\ud574 \ub9cc\ubc18\uc758 \uc900\ube44\ub97c \ud544\uc694\ub85c \ud569\ub2c8\ub2e4. \uc800\ub294 \uadf8\ub9ac\uc2a4\uc758 \uc11c\ub108\uac1c \uad6c\ub2e8\uc758 \ucd94\ucc9c\uc744 \ubc1b\uc558\uc73c\ub098, \uc2ec\uc0ac\uc219\uace0 \ub05d\uc5d0 \uc800\ub294 \ub77c\ub9ac\uc0ac\uc5d0 \uc785\ub2e8\ud558\uae30\ub85c \uacb0\uc815\ud588\uc2b5\ub2c8\ub2e4.\" \ud0c0\ube14\ub77c\ub9ac\ub514\uc2a4\ub294 \uc804\uc5d0 1\uc6d4\uc5d0 \uadf8\ub9ac\uc2a4 \ud074\ub7fd\uc758 \uc9c4\uc2ec\ud55c \uc81c\uc758\ub97c \ubc1b\uc558\uc73c\ub098, \ucc28\ub0a8\uc758 \ucd9c\uc0dd\uc744 \uc9c0\ucf1c\ubcf4\uae30 \uc704\ud574 \uac70\uc808\ud588\uc5c8\ub2e4. \uadf8\ub294 \uc9c0\ub09c \uc2dc\uc98c\uc744 8\uc704\ub85c \ub9c8\uce5c \uc218\ud398\ub974\ub9ac\uac00 \uc5d8\ub77c\ub2e4 \ud074\ub7fd\uacfc 4\ub144 \uacc4\uc57d\uc744 \ub9fa\uc5c8\ub2e4.", "answer": "\ub9c8\ub974\uc138\uc720", "sentence": "\uac08\ud2f0\uc5d0\ub974 \uc784\uae30\uc758 \uccab \uacbd\uae30\uc778 \ub9c8\ub974\uc138\uc720 \uc804\uc5d0\uc11c 10\ubd84\ub9cc\uc5d0 \ud1f4\uc7a5\ub2f9\ud558\uba74\uc11c \ub179\uc0c9 \uad70\ub2e8\uc73c\ub85c\uc368\uc758 \u314e\ub880\uc57d\uc774 \uc0ac\uc2e4\uc0c1 \ub05d\ub098\ubc84\ub838\ub2e4.", "paragraph_sentence": "2010\ub144 4\uc6d4 30\uc77c, \ub77c\ub9ac\uc0ac\ub294 29\uc138 \uc13c\ud130\ubc31\uacfc \ud611\uc0c1\uc5d0 \uc131\uacf5\ud558\uc5ec \uc774 \ud574 \uc5ec\ub984\uc5d0 \ub179\uc0c9 \uad70\ub2e8\uacfc\uc758 \uacc4\uc57d\uc774 \ub9cc\ub8cc\ub418\uba74 \uc790\uc720\uc774\uc801\uc73c\ub85c \ud300\uc5d0 \ud569\ub958\ud558\uac8c \ub420 \uac83\uc774\ub77c \ubc1c\ud45c\ud558\uc600\ub2e4. \ud504\ub791\uc2a4\uc5d0\uc11c \ud669\uc18c\uac19\uc740 \ud50c\ub808\uc774\uc2a4\ud0c0\uc77c\ub85c '\ud669\uc18c' (Le Taureau) \ub77c\ub294 \ubcc4\uce6d\uc744 \uc5bb\uc740 \uadf8\ub294 \ub9b4\uacfc \uc0dd\ud14c\ud2f0\uc5d4 \uc120\uc218\ub85c \ud65c\uc57d\ud574 \ub9ac\uadf8 1\uc5d0\uc11c \uc131\uacf5\ud55c \uadf8\ub9ac\uc2a4 \uc120\uc218\uc774\ub098, \uc54c\ub7ad \ud398\ub7ad \uac10\ub3c5\uc5d0 \uc758\ud574 \uc2a4\ud0c0\ub4dc \uc870\ud504\ub9ac \uadc0\uc0e4\ub974 \uc5f0\uace0 \ud300\uc758 \uba85\ub2e8\uc5d0\uc11c \ubc00\ub824\ub0ac\ub2e4. \uc804 \uc62c\ub7ad\ud53c\ud06c \ub9ac\uc639 \uac10\ub3c5\uc774 \uacbd\uc9c8\ub41c \ud6c4, \ud0c0\ube14\ub77c\ub9ac\ub514\uc2a4\ub294 \ud06c\ub9ac\uc2a4\ud1a0\ud504 \uac08\ud2f0\uc5d0\ub974 \uac10\ub3c5 \ud558\uc5d0 \uc778\uc0c1\uc744 \uc904 \uae30\ud68c\ub97c \uc7a1\uc558\uc73c\ub098, \uac08\ud2f0\uc5d0\ub974 \uc784\uae30\uc758 \uccab \uacbd\uae30\uc778 \ub9c8\ub974\uc138\uc720 \uc804\uc5d0\uc11c 10\ubd84\ub9cc\uc5d0 \ud1f4\uc7a5\ub2f9\ud558\uba74\uc11c \ub179\uc0c9 \uad70\ub2e8\uc73c\ub85c\uc368\uc758 \u314e\ub880\uc57d\uc774 \uc0ac\uc2e4\uc0c1 \ub05d\ub098\ubc84\ub838\ub2e4. \ub098\uc911\uc5d0 \ud569\ub958\ud560 \uad6c\ub2e8\uc758 \uacf5\uc2dd \uc6f9\uc0ac\uc774\ud2b8\uc5d0\uc11c, \uadf8\ub9ac\uc2a4 \uc778\ud130\ub137 \uc5b8\ub860\uc5d0 \uadf8\ub294 \"\uc800\ub294 \uc81c \uacbd\ub825\uc744 \ud574\uc678\uc5d0\uc11c \uc774\uc5b4\ub098\uac00\ub294 \uac83\uc744 \uc6b0\uc120\uc21c\uc704\ub85c \uc7a1\uc558\uc5c8\uc2b5\ub2c8\ub2e4. \uc800\ub294 \uadf8\ub9ac\uc2a4\uc5d0 \ubcf5\uadc0\ud560 \uc5ec\ubd80\ub97c 4\uc6d4\uae4c\uc9c0 \uacb0\uc815\ud588\uc73c\uba70, 9\ub144\ub9cc\uc758 \ubcf5\uadc0\ub294 \uc81c\uac00 \uac00\uc871\uc744 \uac70\ub290\ub9ac\ub294 \uac83\uc73c\ub85c \uc778\ud574 \ub9cc\ubc18\uc758 \uc900\ube44\ub97c \ud544\uc694\ub85c \ud569\ub2c8\ub2e4. \uc800\ub294 \uadf8\ub9ac\uc2a4\uc758 \uc11c\ub108\uac1c \uad6c\ub2e8\uc758 \ucd94\ucc9c\uc744 \ubc1b\uc558\uc73c\ub098, \uc2ec\uc0ac\uc219\uace0 \ub05d\uc5d0 \uc800\ub294 \ub77c\ub9ac\uc0ac\uc5d0 \uc785\ub2e8\ud558\uae30\ub85c \uacb0\uc815\ud588\uc2b5\ub2c8\ub2e4.\" \ud0c0\ube14\ub77c\ub9ac\ub514\uc2a4\ub294 \uc804\uc5d0 1\uc6d4\uc5d0 \uadf8\ub9ac\uc2a4 \ud074\ub7fd\uc758 \uc9c4\uc2ec\ud55c \uc81c\uc758\ub97c \ubc1b\uc558\uc73c\ub098, \ucc28\ub0a8\uc758 \ucd9c\uc0dd\uc744 \uc9c0\ucf1c\ubcf4\uae30 \uc704\ud574 \uac70\uc808\ud588\uc5c8\ub2e4. \uadf8\ub294 \uc9c0\ub09c \uc2dc\uc98c\uc744 8\uc704\ub85c \ub9c8\uce5c \uc218\ud398\ub974\ub9ac\uac00 \uc5d8\ub77c\ub2e4 \ud074\ub7fd\uacfc 4\ub144 \uacc4\uc57d\uc744 \ub9fa\uc5c8\ub2e4.", "paragraph_answer": "2010\ub144 4\uc6d4 30\uc77c, \ub77c\ub9ac\uc0ac\ub294 29\uc138 \uc13c\ud130\ubc31\uacfc \ud611\uc0c1\uc5d0 \uc131\uacf5\ud558\uc5ec \uc774 \ud574 \uc5ec\ub984\uc5d0 \ub179\uc0c9 \uad70\ub2e8\uacfc\uc758 \uacc4\uc57d\uc774 \ub9cc\ub8cc\ub418\uba74 \uc790\uc720\uc774\uc801\uc73c\ub85c \ud300\uc5d0 \ud569\ub958\ud558\uac8c \ub420 \uac83\uc774\ub77c \ubc1c\ud45c\ud558\uc600\ub2e4. \ud504\ub791\uc2a4\uc5d0\uc11c \ud669\uc18c\uac19\uc740 \ud50c\ub808\uc774\uc2a4\ud0c0\uc77c\ub85c '\ud669\uc18c' (Le Taureau) \ub77c\ub294 \ubcc4\uce6d\uc744 \uc5bb\uc740 \uadf8\ub294 \ub9b4\uacfc \uc0dd\ud14c\ud2f0\uc5d4 \uc120\uc218\ub85c \ud65c\uc57d\ud574 \ub9ac\uadf8 1\uc5d0\uc11c \uc131\uacf5\ud55c \uadf8\ub9ac\uc2a4 \uc120\uc218\uc774\ub098, \uc54c\ub7ad \ud398\ub7ad \uac10\ub3c5\uc5d0 \uc758\ud574 \uc2a4\ud0c0\ub4dc \uc870\ud504\ub9ac \uadc0\uc0e4\ub974 \uc5f0\uace0 \ud300\uc758 \uba85\ub2e8\uc5d0\uc11c \ubc00\ub824\ub0ac\ub2e4. \uc804 \uc62c\ub7ad\ud53c\ud06c \ub9ac\uc639 \uac10\ub3c5\uc774 \uacbd\uc9c8\ub41c \ud6c4, \ud0c0\ube14\ub77c\ub9ac\ub514\uc2a4\ub294 \ud06c\ub9ac\uc2a4\ud1a0\ud504 \uac08\ud2f0\uc5d0\ub974 \uac10\ub3c5 \ud558\uc5d0 \uc778\uc0c1\uc744 \uc904 \uae30\ud68c\ub97c \uc7a1\uc558\uc73c\ub098, \uac08\ud2f0\uc5d0\ub974 \uc784\uae30\uc758 \uccab \uacbd\uae30\uc778 \ub9c8\ub974\uc138\uc720 \uc804\uc5d0\uc11c 10\ubd84\ub9cc\uc5d0 \ud1f4\uc7a5\ub2f9\ud558\uba74\uc11c \ub179\uc0c9 \uad70\ub2e8\uc73c\ub85c\uc368\uc758 \u314e\ub880\uc57d\uc774 \uc0ac\uc2e4\uc0c1 \ub05d\ub098\ubc84\ub838\ub2e4. \ub098\uc911\uc5d0 \ud569\ub958\ud560 \uad6c\ub2e8\uc758 \uacf5\uc2dd \uc6f9\uc0ac\uc774\ud2b8\uc5d0\uc11c, \uadf8\ub9ac\uc2a4 \uc778\ud130\ub137 \uc5b8\ub860\uc5d0 \uadf8\ub294 \"\uc800\ub294 \uc81c \uacbd\ub825\uc744 \ud574\uc678\uc5d0\uc11c \uc774\uc5b4\ub098\uac00\ub294 \uac83\uc744 \uc6b0\uc120\uc21c\uc704\ub85c \uc7a1\uc558\uc5c8\uc2b5\ub2c8\ub2e4. \uc800\ub294 \uadf8\ub9ac\uc2a4\uc5d0 \ubcf5\uadc0\ud560 \uc5ec\ubd80\ub97c 4\uc6d4\uae4c\uc9c0 \uacb0\uc815\ud588\uc73c\uba70, 9\ub144\ub9cc\uc758 \ubcf5\uadc0\ub294 \uc81c\uac00 \uac00\uc871\uc744 \uac70\ub290\ub9ac\ub294 \uac83\uc73c\ub85c \uc778\ud574 \ub9cc\ubc18\uc758 \uc900\ube44\ub97c \ud544\uc694\ub85c \ud569\ub2c8\ub2e4. \uc800\ub294 \uadf8\ub9ac\uc2a4\uc758 \uc11c\ub108\uac1c \uad6c\ub2e8\uc758 \ucd94\ucc9c\uc744 \ubc1b\uc558\uc73c\ub098, \uc2ec\uc0ac\uc219\uace0 \ub05d\uc5d0 \uc800\ub294 \ub77c\ub9ac\uc0ac\uc5d0 \uc785\ub2e8\ud558\uae30\ub85c \uacb0\uc815\ud588\uc2b5\ub2c8\ub2e4.\" \ud0c0\ube14\ub77c\ub9ac\ub514\uc2a4\ub294 \uc804\uc5d0 1\uc6d4\uc5d0 \uadf8\ub9ac\uc2a4 \ud074\ub7fd\uc758 \uc9c4\uc2ec\ud55c \uc81c\uc758\ub97c \ubc1b\uc558\uc73c\ub098, \ucc28\ub0a8\uc758 \ucd9c\uc0dd\uc744 \uc9c0\ucf1c\ubcf4\uae30 \uc704\ud574 \uac70\uc808\ud588\uc5c8\ub2e4. \uadf8\ub294 \uc9c0\ub09c \uc2dc\uc98c\uc744 8\uc704\ub85c \ub9c8\uce5c \uc218\ud398\ub974\ub9ac\uac00 \uc5d8\ub77c\ub2e4 \ud074\ub7fd\uacfc 4\ub144 \uacc4\uc57d\uc744 \ub9fa\uc5c8\ub2e4.", "sentence_answer": "\uac08\ud2f0\uc5d0\ub974 \uc784\uae30\uc758 \uccab \uacbd\uae30\uc778 \ub9c8\ub974\uc138\uc720 \uc804\uc5d0\uc11c 10\ubd84\ub9cc\uc5d0 \ud1f4\uc7a5\ub2f9\ud558\uba74\uc11c \ub179\uc0c9 \uad70\ub2e8\uc73c\ub85c\uc368\uc758 \u314e\ub880\uc57d\uc774 \uc0ac\uc2e4\uc0c1 \ub05d\ub098\ubc84\ub838\ub2e4.", "paragraph_id": "6523547-2-1", "paragraph_question": "question: \ud0c0\ube14\ub77c\ub9ac\ub514\uc2a4\uac00 10\ubd84\ub9cc\uc5d0 \ud1f4\uc7a5\ub2f9\ud55c \uacbd\uae30\uc758 \uc0c1\ub300\ud300\uc740?, context: 2010\ub144 4\uc6d4 30\uc77c, \ub77c\ub9ac\uc0ac\ub294 29\uc138 \uc13c\ud130\ubc31\uacfc \ud611\uc0c1\uc5d0 \uc131\uacf5\ud558\uc5ec \uc774 \ud574 \uc5ec\ub984\uc5d0 \ub179\uc0c9 \uad70\ub2e8\uacfc\uc758 \uacc4\uc57d\uc774 \ub9cc\ub8cc\ub418\uba74 \uc790\uc720\uc774\uc801\uc73c\ub85c \ud300\uc5d0 \ud569\ub958\ud558\uac8c \ub420 \uac83\uc774\ub77c \ubc1c\ud45c\ud558\uc600\ub2e4. \ud504\ub791\uc2a4\uc5d0\uc11c \ud669\uc18c\uac19\uc740 \ud50c\ub808\uc774\uc2a4\ud0c0\uc77c\ub85c '\ud669\uc18c' (Le Taureau) \ub77c\ub294 \ubcc4\uce6d\uc744 \uc5bb\uc740 \uadf8\ub294 \ub9b4\uacfc \uc0dd\ud14c\ud2f0\uc5d4 \uc120\uc218\ub85c \ud65c\uc57d\ud574 \ub9ac\uadf8 1\uc5d0\uc11c \uc131\uacf5\ud55c \uadf8\ub9ac\uc2a4 \uc120\uc218\uc774\ub098, \uc54c\ub7ad \ud398\ub7ad \uac10\ub3c5\uc5d0 \uc758\ud574 \uc2a4\ud0c0\ub4dc \uc870\ud504\ub9ac \uadc0\uc0e4\ub974 \uc5f0\uace0 \ud300\uc758 \uba85\ub2e8\uc5d0\uc11c \ubc00\ub824\ub0ac\ub2e4. \uc804 \uc62c\ub7ad\ud53c\ud06c \ub9ac\uc639 \uac10\ub3c5\uc774 \uacbd\uc9c8\ub41c \ud6c4, \ud0c0\ube14\ub77c\ub9ac\ub514\uc2a4\ub294 \ud06c\ub9ac\uc2a4\ud1a0\ud504 \uac08\ud2f0\uc5d0\ub974 \uac10\ub3c5 \ud558\uc5d0 \uc778\uc0c1\uc744 \uc904 \uae30\ud68c\ub97c \uc7a1\uc558\uc73c\ub098, \uac08\ud2f0\uc5d0\ub974 \uc784\uae30\uc758 \uccab \uacbd\uae30\uc778 \ub9c8\ub974\uc138\uc720\uc804\uc5d0\uc11c 10\ubd84\ub9cc\uc5d0 \ud1f4\uc7a5\ub2f9\ud558\uba74\uc11c \ub179\uc0c9 \uad70\ub2e8\uc73c\ub85c\uc368\uc758 \u314e\ub880\uc57d\uc774 \uc0ac\uc2e4\uc0c1 \ub05d\ub098\ubc84\ub838\ub2e4. \ub098\uc911\uc5d0 \ud569\ub958\ud560 \uad6c\ub2e8\uc758 \uacf5\uc2dd \uc6f9\uc0ac\uc774\ud2b8\uc5d0\uc11c, \uadf8\ub9ac\uc2a4 \uc778\ud130\ub137 \uc5b8\ub860\uc5d0 \uadf8\ub294 \"\uc800\ub294 \uc81c \uacbd\ub825\uc744 \ud574\uc678\uc5d0\uc11c \uc774\uc5b4\ub098\uac00\ub294 \uac83\uc744 \uc6b0\uc120\uc21c\uc704\ub85c \uc7a1\uc558\uc5c8\uc2b5\ub2c8\ub2e4. \uc800\ub294 \uadf8\ub9ac\uc2a4\uc5d0 \ubcf5\uadc0\ud560 \uc5ec\ubd80\ub97c 4\uc6d4\uae4c\uc9c0 \uacb0\uc815\ud588\uc73c\uba70, 9\ub144\ub9cc\uc758 \ubcf5\uadc0\ub294 \uc81c\uac00 \uac00\uc871\uc744 \uac70\ub290\ub9ac\ub294 \uac83\uc73c\ub85c \uc778\ud574 \ub9cc\ubc18\uc758 \uc900\ube44\ub97c \ud544\uc694\ub85c \ud569\ub2c8\ub2e4. \uc800\ub294 \uadf8\ub9ac\uc2a4\uc758 \uc11c\ub108\uac1c \uad6c\ub2e8\uc758 \ucd94\ucc9c\uc744 \ubc1b\uc558\uc73c\ub098, \uc2ec\uc0ac\uc219\uace0 \ub05d\uc5d0 \uc800\ub294 \ub77c\ub9ac\uc0ac\uc5d0 \uc785\ub2e8\ud558\uae30\ub85c \uacb0\uc815\ud588\uc2b5\ub2c8\ub2e4.\" \ud0c0\ube14\ub77c\ub9ac\ub514\uc2a4\ub294 \uc804\uc5d0 1\uc6d4\uc5d0 \uadf8\ub9ac\uc2a4 \ud074\ub7fd\uc758 \uc9c4\uc2ec\ud55c \uc81c\uc758\ub97c \ubc1b\uc558\uc73c\ub098, \ucc28\ub0a8\uc758 \ucd9c\uc0dd\uc744 \uc9c0\ucf1c\ubcf4\uae30 \uc704\ud574 \uac70\uc808\ud588\uc5c8\ub2e4. \uadf8\ub294 \uc9c0\ub09c \uc2dc\uc98c\uc744 8\uc704\ub85c \ub9c8\uce5c \uc218\ud398\ub974\ub9ac\uac00 \uc5d8\ub77c\ub2e4 \ud074\ub7fd\uacfc 4\ub144 \uacc4\uc57d\uc744 \ub9fa\uc5c8\ub2e4."} {"question": "\uc774\uc601\uc560\ub294 1979\ub144 \uc5b4\ub514 \ubaa8\ub378\ub85c \uccab \ub370\ubdd4\ub97c \ud558\uc600\ub098?", "paragraph": "\ucd08\ub4f1\ud559\uad50 3\ud559\ub144 \ub54c\uc778 1979\ub144\uc5d0 \ud45c\uc900\uc804\uacfc \ubaa8\ub378\ub85c \uccab \ub370\ubdd4\ud558\uc600\uace0 \uc911\ud559\uad50 2\ud559\ub144\uc774\ub358 1984\ub144\uc5d0 \uc8fc\ub2c8\uc5b4 \uc7a1\uc9c0 \ud45c\uc9c0 \ubaa8\ub378\ub85c \ub370\ubdd4 \ud6c4, 1990\ub144 \uc624\ub9ac\uc628 \ud22c\uc720 \ucd08\ucf5c\ub9bf \uad11\uace0\ub97c \ud1b5\ud574 \uc5f0\uc608 \ud65c\ub3d9\uc744 \uc2dc\uc791\ud588\ub2e4. \uc774\ud6c4 \uc544\ubaa8\ub808\ud37c\uc2dc\ud53d \ud654\uc7a5\ud488 \ub9c8\ubabd\ub4dc\uc758 \uad11\uace0 \ubaa8\ub378\ub85c \uc624\ub79c\uae30\uac04 \ud65c\ub3d9\ud558\uc600\uace0, \"\uc0b0\uc18c\uac19\uc740 \uc5ec\uc790\"\ub77c\ub294 \uc218\uc2dd\uc5b4\uac00 \ub530\ub77c\ubd99\uc73c\uba70 \ub300\uc911\ub4e4\uc758 \uad00\uc2ec\uc744 \ubc1b\uae30 \uc2dc\uc791\ud558\uc5ec, \ub0a8\uc131\ub4e4\uc5d0\uac8c\ub294 \ub85c\ub9dd\uc758 \ub300\uc0c1\uc774 \ub418\uc5c8\ub2e4. 1993\ub144 SBS \ud2b9\ucc44 \ud0e4\ub7f0\ud2b8\ub85c \ubc1c\ud0c1\ub418\uc5b4, \ub4dc\ub77c\ub9c8 \u300a\ub301\uc758 \ub0a8\ud3b8\uc740 \uc5b4\ub5a0\uc2ed\ub2c8\uae4c\u300b\ub85c \ubcf8\uaca9 \uc5f0\uae30\uc790 \ub370\ubdd4\ud588\ub2e4. \uc774\uc601\uc560\ub294 1990\ub144\ub300 \uc911\ubc18\ubd80\ud130 2000\ub144 \ucd08\uae4c\uc9c0 \u300a\uc0ac\ub791\uacfc \uacb0\ud63c\u300b, \u300a\uc11c\uad81\u300b, \u300a\ud30c\ud30c\u300b(1995), \u300a\uadf8\ub4e4\uc758 \ud3ec\uc639\u300b, \u300a\ub3d9\uae30\uac04\u300b(1996), \u300a\uc758\uac00\ud615\uc81c\u300b, \u300a\ub0b4\uac00 \uc0ac\ub294 \uc774\uc720\u300b, \u300a\uc0ac\ub791\ud558\ub2c8\uae4c\u300b(1997), \u300a\ub85c\ub9e8\uc2a4\u300b, \u300a\uc560\ub4dc\ubc84\ud0b7\u300b(1998), \u300a\ud30c\ub3c4\u300b, \u300a\ucd08\ub300\u300b(1999), \u300a\ubd88\uaf43\u300b(2000) \ub4f1 \uc218\ub9ce\uc740 \ub4dc\ub77c\ub9c8\ub97c \ucd9c\uc5f0\ud558\uba70 \ucee4\ub9ac\uc5b4\ub97c \uc313\uc544\uac14\ub2e4.", "answer": "\ud45c\uc900\uc804\uacfc", "sentence": "\ucd08\ub4f1\ud559\uad50 3\ud559\ub144 \ub54c\uc778 1979\ub144\uc5d0 \ud45c\uc900\uc804\uacfc \ubaa8\ub378\ub85c \uccab \ub370\ubdd4\ud558\uc600\uace0 \uc911\ud559\uad50 2\ud559\ub144\uc774\ub358 1984\ub144\uc5d0 \uc8fc\ub2c8\uc5b4 \uc7a1\uc9c0 \ud45c\uc9c0 \ubaa8\ub378\ub85c \ub370\ubdd4 \ud6c4, 1990\ub144 \uc624\ub9ac\uc628 \ud22c\uc720 \ucd08\ucf5c\ub9bf \uad11\uace0\ub97c \ud1b5\ud574 \uc5f0\uc608 \ud65c\ub3d9\uc744 \uc2dc\uc791\ud588\ub2e4.", "paragraph_sentence": " \ucd08\ub4f1\ud559\uad50 3\ud559\ub144 \ub54c\uc778 1979\ub144\uc5d0 \ud45c\uc900\uc804\uacfc \ubaa8\ub378\ub85c \uccab \ub370\ubdd4\ud558\uc600\uace0 \uc911\ud559\uad50 2\ud559\ub144\uc774\ub358 1984\ub144\uc5d0 \uc8fc\ub2c8\uc5b4 \uc7a1\uc9c0 \ud45c\uc9c0 \ubaa8\ub378\ub85c \ub370\ubdd4 \ud6c4, 1990\ub144 \uc624\ub9ac\uc628 \ud22c\uc720 \ucd08\ucf5c\ub9bf \uad11\uace0\ub97c \ud1b5\ud574 \uc5f0\uc608 \ud65c\ub3d9\uc744 \uc2dc\uc791\ud588\ub2e4. \uc774\ud6c4 \uc544\ubaa8\ub808\ud37c\uc2dc\ud53d \ud654\uc7a5\ud488 \ub9c8\ubabd\ub4dc\uc758 \uad11\uace0 \ubaa8\ub378\ub85c \uc624\ub79c\uae30\uac04 \ud65c\ub3d9\ud558\uc600\uace0, \"\uc0b0\uc18c\uac19\uc740 \uc5ec\uc790\"\ub77c\ub294 \uc218\uc2dd\uc5b4\uac00 \ub530\ub77c\ubd99\uc73c\uba70 \ub300\uc911\ub4e4\uc758 \uad00\uc2ec\uc744 \ubc1b\uae30 \uc2dc\uc791\ud558\uc5ec, \ub0a8\uc131\ub4e4\uc5d0\uac8c\ub294 \ub85c\ub9dd\uc758 \ub300\uc0c1\uc774 \ub418\uc5c8\ub2e4. 1993\ub144 SBS \ud2b9\ucc44 \ud0e4\ub7f0\ud2b8\ub85c \ubc1c\ud0c1\ub418\uc5b4, \ub4dc\ub77c\ub9c8 \u300a\ub301\uc758 \ub0a8\ud3b8\uc740 \uc5b4\ub5a0\uc2ed\ub2c8\uae4c\u300b\ub85c \ubcf8\uaca9 \uc5f0\uae30\uc790 \ub370\ubdd4\ud588\ub2e4. \uc774\uc601\uc560\ub294 1990\ub144\ub300 \uc911\ubc18\ubd80\ud130 2000\ub144 \ucd08\uae4c\uc9c0 \u300a\uc0ac\ub791\uacfc \uacb0\ud63c\u300b, \u300a\uc11c\uad81\u300b, \u300a\ud30c\ud30c\u300b(1995), \u300a\uadf8\ub4e4\uc758 \ud3ec\uc639\u300b, \u300a\ub3d9\uae30\uac04\u300b(1996), \u300a\uc758\uac00\ud615\uc81c\u300b, \u300a\ub0b4\uac00 \uc0ac\ub294 \uc774\uc720\u300b, \u300a\uc0ac\ub791\ud558\ub2c8\uae4c\u300b(1997), \u300a\ub85c\ub9e8\uc2a4\u300b, \u300a\uc560\ub4dc\ubc84\ud0b7\u300b(1998), \u300a\ud30c\ub3c4\u300b, \u300a\ucd08\ub300\u300b(1999), \u300a\ubd88\uaf43\u300b(2000) \ub4f1 \uc218\ub9ce\uc740 \ub4dc\ub77c\ub9c8\ub97c \ucd9c\uc5f0\ud558\uba70 \ucee4\ub9ac\uc5b4\ub97c \uc313\uc544\uac14\ub2e4.", "paragraph_answer": "\ucd08\ub4f1\ud559\uad50 3\ud559\ub144 \ub54c\uc778 1979\ub144\uc5d0 \ud45c\uc900\uc804\uacfc \ubaa8\ub378\ub85c \uccab \ub370\ubdd4\ud558\uc600\uace0 \uc911\ud559\uad50 2\ud559\ub144\uc774\ub358 1984\ub144\uc5d0 \uc8fc\ub2c8\uc5b4 \uc7a1\uc9c0 \ud45c\uc9c0 \ubaa8\ub378\ub85c \ub370\ubdd4 \ud6c4, 1990\ub144 \uc624\ub9ac\uc628 \ud22c\uc720 \ucd08\ucf5c\ub9bf \uad11\uace0\ub97c \ud1b5\ud574 \uc5f0\uc608 \ud65c\ub3d9\uc744 \uc2dc\uc791\ud588\ub2e4. \uc774\ud6c4 \uc544\ubaa8\ub808\ud37c\uc2dc\ud53d \ud654\uc7a5\ud488 \ub9c8\ubabd\ub4dc\uc758 \uad11\uace0 \ubaa8\ub378\ub85c \uc624\ub79c\uae30\uac04 \ud65c\ub3d9\ud558\uc600\uace0, \"\uc0b0\uc18c\uac19\uc740 \uc5ec\uc790\"\ub77c\ub294 \uc218\uc2dd\uc5b4\uac00 \ub530\ub77c\ubd99\uc73c\uba70 \ub300\uc911\ub4e4\uc758 \uad00\uc2ec\uc744 \ubc1b\uae30 \uc2dc\uc791\ud558\uc5ec, \ub0a8\uc131\ub4e4\uc5d0\uac8c\ub294 \ub85c\ub9dd\uc758 \ub300\uc0c1\uc774 \ub418\uc5c8\ub2e4. 1993\ub144 SBS \ud2b9\ucc44 \ud0e4\ub7f0\ud2b8\ub85c \ubc1c\ud0c1\ub418\uc5b4, \ub4dc\ub77c\ub9c8 \u300a\ub301\uc758 \ub0a8\ud3b8\uc740 \uc5b4\ub5a0\uc2ed\ub2c8\uae4c\u300b\ub85c \ubcf8\uaca9 \uc5f0\uae30\uc790 \ub370\ubdd4\ud588\ub2e4. \uc774\uc601\uc560\ub294 1990\ub144\ub300 \uc911\ubc18\ubd80\ud130 2000\ub144 \ucd08\uae4c\uc9c0 \u300a\uc0ac\ub791\uacfc \uacb0\ud63c\u300b, \u300a\uc11c\uad81\u300b, \u300a\ud30c\ud30c\u300b(1995), \u300a\uadf8\ub4e4\uc758 \ud3ec\uc639\u300b, \u300a\ub3d9\uae30\uac04\u300b(1996), \u300a\uc758\uac00\ud615\uc81c\u300b, \u300a\ub0b4\uac00 \uc0ac\ub294 \uc774\uc720\u300b, \u300a\uc0ac\ub791\ud558\ub2c8\uae4c\u300b(1997), \u300a\ub85c\ub9e8\uc2a4\u300b, \u300a\uc560\ub4dc\ubc84\ud0b7\u300b(1998), \u300a\ud30c\ub3c4\u300b, \u300a\ucd08\ub300\u300b(1999), \u300a\ubd88\uaf43\u300b(2000) \ub4f1 \uc218\ub9ce\uc740 \ub4dc\ub77c\ub9c8\ub97c \ucd9c\uc5f0\ud558\uba70 \ucee4\ub9ac\uc5b4\ub97c \uc313\uc544\uac14\ub2e4.", "sentence_answer": "\ucd08\ub4f1\ud559\uad50 3\ud559\ub144 \ub54c\uc778 1979\ub144\uc5d0 \ud45c\uc900\uc804\uacfc \ubaa8\ub378\ub85c \uccab \ub370\ubdd4\ud558\uc600\uace0 \uc911\ud559\uad50 2\ud559\ub144\uc774\ub358 1984\ub144\uc5d0 \uc8fc\ub2c8\uc5b4 \uc7a1\uc9c0 \ud45c\uc9c0 \ubaa8\ub378\ub85c \ub370\ubdd4 \ud6c4, 1990\ub144 \uc624\ub9ac\uc628 \ud22c\uc720 \ucd08\ucf5c\ub9bf \uad11\uace0\ub97c \ud1b5\ud574 \uc5f0\uc608 \ud65c\ub3d9\uc744 \uc2dc\uc791\ud588\ub2e4.", "paragraph_id": "6162279-0-0", "paragraph_question": "question: \uc774\uc601\uc560\ub294 1979\ub144 \uc5b4\ub514 \ubaa8\ub378\ub85c \uccab \ub370\ubdd4\ub97c \ud558\uc600\ub098?, context: \ucd08\ub4f1\ud559\uad50 3\ud559\ub144 \ub54c\uc778 1979\ub144\uc5d0 \ud45c\uc900\uc804\uacfc \ubaa8\ub378\ub85c \uccab \ub370\ubdd4\ud558\uc600\uace0 \uc911\ud559\uad50 2\ud559\ub144\uc774\ub358 1984\ub144\uc5d0 \uc8fc\ub2c8\uc5b4 \uc7a1\uc9c0 \ud45c\uc9c0 \ubaa8\ub378\ub85c \ub370\ubdd4 \ud6c4, 1990\ub144 \uc624\ub9ac\uc628 \ud22c\uc720 \ucd08\ucf5c\ub9bf \uad11\uace0\ub97c \ud1b5\ud574 \uc5f0\uc608 \ud65c\ub3d9\uc744 \uc2dc\uc791\ud588\ub2e4. \uc774\ud6c4 \uc544\ubaa8\ub808\ud37c\uc2dc\ud53d \ud654\uc7a5\ud488 \ub9c8\ubabd\ub4dc\uc758 \uad11\uace0 \ubaa8\ub378\ub85c \uc624\ub79c\uae30\uac04 \ud65c\ub3d9\ud558\uc600\uace0, \"\uc0b0\uc18c\uac19\uc740 \uc5ec\uc790\"\ub77c\ub294 \uc218\uc2dd\uc5b4\uac00 \ub530\ub77c\ubd99\uc73c\uba70 \ub300\uc911\ub4e4\uc758 \uad00\uc2ec\uc744 \ubc1b\uae30 \uc2dc\uc791\ud558\uc5ec, \ub0a8\uc131\ub4e4\uc5d0\uac8c\ub294 \ub85c\ub9dd\uc758 \ub300\uc0c1\uc774 \ub418\uc5c8\ub2e4. 1993\ub144 SBS \ud2b9\ucc44 \ud0e4\ub7f0\ud2b8\ub85c \ubc1c\ud0c1\ub418\uc5b4, \ub4dc\ub77c\ub9c8 \u300a\ub301\uc758 \ub0a8\ud3b8\uc740 \uc5b4\ub5a0\uc2ed\ub2c8\uae4c\u300b\ub85c \ubcf8\uaca9 \uc5f0\uae30\uc790 \ub370\ubdd4\ud588\ub2e4. \uc774\uc601\uc560\ub294 1990\ub144\ub300 \uc911\ubc18\ubd80\ud130 2000\ub144 \ucd08\uae4c\uc9c0 \u300a\uc0ac\ub791\uacfc \uacb0\ud63c\u300b, \u300a\uc11c\uad81\u300b, \u300a\ud30c\ud30c\u300b(1995), \u300a\uadf8\ub4e4\uc758 \ud3ec\uc639\u300b, \u300a\ub3d9\uae30\uac04\u300b(1996), \u300a\uc758\uac00\ud615\uc81c\u300b, \u300a\ub0b4\uac00 \uc0ac\ub294 \uc774\uc720\u300b, \u300a\uc0ac\ub791\ud558\ub2c8\uae4c\u300b(1997), \u300a\ub85c\ub9e8\uc2a4\u300b, \u300a\uc560\ub4dc\ubc84\ud0b7\u300b(1998), \u300a\ud30c\ub3c4\u300b, \u300a\ucd08\ub300\u300b(1999), \u300a\ubd88\uaf43\u300b(2000) \ub4f1 \uc218\ub9ce\uc740 \ub4dc\ub77c\ub9c8\ub97c \ucd9c\uc5f0\ud558\uba70 \ucee4\ub9ac\uc5b4\ub97c \uc313\uc544\uac14\ub2e4."} {"question": "\ub85c\uc2a4\uc564\uc824\ub808\uc2a4\ub85c \uc774\uc8fc\ud55c \uc774\ud6c4 \ub9c8\uc2a4\uac00 \ubaa8\ud0c0\uc6b4\uacfc \uacc4\uc57d\uc744 \ub9fa\uc740 \ub144\ub3c4\ub294?", "paragraph": "\ub85c\uc2a4\uc564\uc824\ub808\uc2a4\ub85c \uc774\ub3d9\ud55c \ud6c4, 2004\ub144 \ub9c8\uc2a4\ub294 \ubaa8\ud0c0\uc6b4\uacfc \uacc4\uc57d\uc744 \ub9fa\uc5c8\ub2e4. \ubaa8\ud0c0\uc6b4\uacfc\uc758 \uacc4\uc57d\uc740 \uc131\uacfc\uac00 \uc5c6\uc5c8\uc73c\uba70, \uace7 \uc1a1\ub77c\uc774\ud130\uc774\uc790 \ud504\ub85c\ub4c0\uc11c \ud544\ub9bd \ub85c\ub7f0\uc2a4(Philip Lawrence)\ub97c \ub9cc\ub098\uac8c \ub418\uc5c8\ub2e4. \ub9c8\uc2a4, \ub85c\ub7f0\uc2a4, \uc560\ub9ac \ub7ec\ubc14\uc778(Ari Levine)\uc740 \ud504\ub85c\ub4c0\uc2f1\ud300 \uc2a4\ubbf8\uc9d5\ud134\uc2a4(The Smeezingtons)\ub97c \uacb0\uc131\ud558\uc600\ub2e4. 2006\ub144, \ub85c\ub7f0\uc2a4\ub294 \uc560\ud2c0\ub79c\ud2f1 \ub808\ucf54\ub4dc\uc758 \ubbf8\ub798 \uad00\ub9ac\uc790 \uc5d0\ub7f0 \ubca0\uc774\uc155(Aaron Bay-Schuck)\uc5d0\uac8c \ub9c8\uc2a4\ub97c \uc18c\uac1c\ud574\uc8fc\uc5c8\ub2e4. \ub9c8\uc2a4\ub294 \uadf8\uc5d0\uac8c 2\uace1\uc744 \ub4e4\ub824\uc8fc\uc5c8\uace0, \ubca0\uc774\uc155\uc740 \ub9c8\uc2a4\ub97c \ubc14\ub85c \uacc4\uc57d\ud558\uace0 \uc2f6\uc5b4\ud588\ub2e4. \ud558\uc9c0\ub9cc \uacb0\uad6d, \uacc4\uc57d\ud558\ub294 \ub370 3\ub144\uc774 \uac78\ub838\ub2e4. \ubca0\uc774\uc155\uc740 \ub9c8\uc2a4\uc640 \uc2a4\ubbf8\uc9d5\ud134\uc2a4\ub97c \uc560\ud2c0\ub79c\ud2f1 \ub808\ucf54\ub4dc \uc544\ud2f0\uc2a4\ud2b8\ub97c \uc704\ud55c \uc791\uace1 \ubc0f \ud504\ub85c\ub4c0\uc2f1 \ubd80\ubd84\uc73c\ub85c \uacc4\uc57d\ud558\uc600\ub2e4. \ud79b\ucffc\ud130\uc2a4\uc640\uc758 \uc778\ud130\ubdf0\uc5d0\uc11c \ubca0\uc774\uc155\uc5d0 \ub530\ub974\uba74, \ub9c8\uc2a4\ub294 \uad81\uadf9\uc801\uc778 \ubaa9\ud45c\ub294 \uc194\ub85c \uc544\ud2f0\uc2a4\ud2b8\ub77c\uace0 \ub9d0\ud588\uc73c\uba70, \uadf8\ub294 \uae30\uaebc\uc774 \ub2e4\ub978 \uc544\ud2f0\uc2a4\ud2b8\ub97c \uc704\ud574 \uace1\uc744 \uc4f0\uace0 \ud504\ub85c\ub4c0\uc2a4\ud558\ub294 \uac83\uc5d0 \ub300\ud574 \uc790\uc2e0\uc758 \uc791\uace1 \ub2a5\ub825\uc744 \ud5a5\uc0c1\uc2dc\ud0a4\uace0 \uadf8 \uc544\ud2f0\uc2a4\ud2b8\uc758 \uc720\ud615\uc744 \ubc1c\uacac\ud560 \uc218 \uc788\ub2e4\uace0 \ud558\uc600\ub2e4. \ubca0\uc774\uc155\uc740 \ub9c8\uc2a4\uac00 \ub098\uc911\uc5d0 \uc131\uacf5\ud55c \uac83\uc740 \uc774 \uae30\uac04\uc5d0 \uc758\ud55c \"\uc790\uc544 \ubc1c\uacac\"\uc774 \uc0c1\ub2f9\ud788 \uc601\ud5a5\uc774 \ub07c\ucce4\ub2e4\uace0 \ud558\uc600\ub2e4.", "answer": "2004\ub144", "sentence": "\ub85c\uc2a4\uc564\uc824\ub808\uc2a4\ub85c \uc774\ub3d9\ud55c \ud6c4, 2004\ub144 \ub9c8\uc2a4\ub294 \ubaa8\ud0c0\uc6b4\uacfc \uacc4\uc57d\uc744 \ub9fa\uc5c8\ub2e4.", "paragraph_sentence": " \ub85c\uc2a4\uc564\uc824\ub808\uc2a4\ub85c \uc774\ub3d9\ud55c \ud6c4, 2004\ub144 \ub9c8\uc2a4\ub294 \ubaa8\ud0c0\uc6b4\uacfc \uacc4\uc57d\uc744 \ub9fa\uc5c8\ub2e4. \ubaa8\ud0c0\uc6b4\uacfc\uc758 \uacc4\uc57d\uc740 \uc131\uacfc\uac00 \uc5c6\uc5c8\uc73c\uba70, \uace7 \uc1a1\ub77c\uc774\ud130\uc774\uc790 \ud504\ub85c\ub4c0\uc11c \ud544\ub9bd \ub85c\ub7f0\uc2a4(Philip Lawrence)\ub97c \ub9cc\ub098\uac8c \ub418\uc5c8\ub2e4. \ub9c8\uc2a4, \ub85c\ub7f0\uc2a4, \uc560\ub9ac \ub7ec\ubc14\uc778(Ari Levine)\uc740 \ud504\ub85c\ub4c0\uc2f1\ud300 \uc2a4\ubbf8\uc9d5\ud134\uc2a4(The Smeezingtons)\ub97c \uacb0\uc131\ud558\uc600\ub2e4. 2006\ub144, \ub85c\ub7f0\uc2a4\ub294 \uc560\ud2c0\ub79c\ud2f1 \ub808\ucf54\ub4dc\uc758 \ubbf8\ub798 \uad00\ub9ac\uc790 \uc5d0\ub7f0 \ubca0\uc774\uc155(Aaron Bay-Schuck)\uc5d0\uac8c \ub9c8\uc2a4\ub97c \uc18c\uac1c\ud574\uc8fc\uc5c8\ub2e4. \ub9c8\uc2a4\ub294 \uadf8\uc5d0\uac8c 2\uace1\uc744 \ub4e4\ub824\uc8fc\uc5c8\uace0, \ubca0\uc774\uc155\uc740 \ub9c8\uc2a4\ub97c \ubc14\ub85c \uacc4\uc57d\ud558\uace0 \uc2f6\uc5b4\ud588\ub2e4. \ud558\uc9c0\ub9cc \uacb0\uad6d, \uacc4\uc57d\ud558\ub294 \ub370 3\ub144\uc774 \uac78\ub838\ub2e4. \ubca0\uc774\uc155\uc740 \ub9c8\uc2a4\uc640 \uc2a4\ubbf8\uc9d5\ud134\uc2a4\ub97c \uc560\ud2c0\ub79c\ud2f1 \ub808\ucf54\ub4dc \uc544\ud2f0\uc2a4\ud2b8\ub97c \uc704\ud55c \uc791\uace1 \ubc0f \ud504\ub85c\ub4c0\uc2f1 \ubd80\ubd84\uc73c\ub85c \uacc4\uc57d\ud558\uc600\ub2e4. \ud79b\ucffc\ud130\uc2a4\uc640\uc758 \uc778\ud130\ubdf0\uc5d0\uc11c \ubca0\uc774\uc155\uc5d0 \ub530\ub974\uba74, \ub9c8\uc2a4\ub294 \uad81\uadf9\uc801\uc778 \ubaa9\ud45c\ub294 \uc194\ub85c \uc544\ud2f0\uc2a4\ud2b8\ub77c\uace0 \ub9d0\ud588\uc73c\uba70, \uadf8\ub294 \uae30\uaebc\uc774 \ub2e4\ub978 \uc544\ud2f0\uc2a4\ud2b8\ub97c \uc704\ud574 \uace1\uc744 \uc4f0\uace0 \ud504\ub85c\ub4c0\uc2a4\ud558\ub294 \uac83\uc5d0 \ub300\ud574 \uc790\uc2e0\uc758 \uc791\uace1 \ub2a5\ub825\uc744 \ud5a5\uc0c1\uc2dc\ud0a4\uace0 \uadf8 \uc544\ud2f0\uc2a4\ud2b8\uc758 \uc720\ud615\uc744 \ubc1c\uacac\ud560 \uc218 \uc788\ub2e4\uace0 \ud558\uc600\ub2e4. \ubca0\uc774\uc155\uc740 \ub9c8\uc2a4\uac00 \ub098\uc911\uc5d0 \uc131\uacf5\ud55c \uac83\uc740 \uc774 \uae30\uac04\uc5d0 \uc758\ud55c \"\uc790\uc544 \ubc1c\uacac\"\uc774 \uc0c1\ub2f9\ud788 \uc601\ud5a5\uc774 \ub07c\ucce4\ub2e4\uace0 \ud558\uc600\ub2e4.", "paragraph_answer": "\ub85c\uc2a4\uc564\uc824\ub808\uc2a4\ub85c \uc774\ub3d9\ud55c \ud6c4, 2004\ub144 \ub9c8\uc2a4\ub294 \ubaa8\ud0c0\uc6b4\uacfc \uacc4\uc57d\uc744 \ub9fa\uc5c8\ub2e4. \ubaa8\ud0c0\uc6b4\uacfc\uc758 \uacc4\uc57d\uc740 \uc131\uacfc\uac00 \uc5c6\uc5c8\uc73c\uba70, \uace7 \uc1a1\ub77c\uc774\ud130\uc774\uc790 \ud504\ub85c\ub4c0\uc11c \ud544\ub9bd \ub85c\ub7f0\uc2a4(Philip Lawrence)\ub97c \ub9cc\ub098\uac8c \ub418\uc5c8\ub2e4. \ub9c8\uc2a4, \ub85c\ub7f0\uc2a4, \uc560\ub9ac \ub7ec\ubc14\uc778(Ari Levine)\uc740 \ud504\ub85c\ub4c0\uc2f1\ud300 \uc2a4\ubbf8\uc9d5\ud134\uc2a4(The Smeezingtons)\ub97c \uacb0\uc131\ud558\uc600\ub2e4. 2006\ub144, \ub85c\ub7f0\uc2a4\ub294 \uc560\ud2c0\ub79c\ud2f1 \ub808\ucf54\ub4dc\uc758 \ubbf8\ub798 \uad00\ub9ac\uc790 \uc5d0\ub7f0 \ubca0\uc774\uc155(Aaron Bay-Schuck)\uc5d0\uac8c \ub9c8\uc2a4\ub97c \uc18c\uac1c\ud574\uc8fc\uc5c8\ub2e4. \ub9c8\uc2a4\ub294 \uadf8\uc5d0\uac8c 2\uace1\uc744 \ub4e4\ub824\uc8fc\uc5c8\uace0, \ubca0\uc774\uc155\uc740 \ub9c8\uc2a4\ub97c \ubc14\ub85c \uacc4\uc57d\ud558\uace0 \uc2f6\uc5b4\ud588\ub2e4. \ud558\uc9c0\ub9cc \uacb0\uad6d, \uacc4\uc57d\ud558\ub294 \ub370 3\ub144\uc774 \uac78\ub838\ub2e4. \ubca0\uc774\uc155\uc740 \ub9c8\uc2a4\uc640 \uc2a4\ubbf8\uc9d5\ud134\uc2a4\ub97c \uc560\ud2c0\ub79c\ud2f1 \ub808\ucf54\ub4dc \uc544\ud2f0\uc2a4\ud2b8\ub97c \uc704\ud55c \uc791\uace1 \ubc0f \ud504\ub85c\ub4c0\uc2f1 \ubd80\ubd84\uc73c\ub85c \uacc4\uc57d\ud558\uc600\ub2e4. \ud79b\ucffc\ud130\uc2a4\uc640\uc758 \uc778\ud130\ubdf0\uc5d0\uc11c \ubca0\uc774\uc155\uc5d0 \ub530\ub974\uba74, \ub9c8\uc2a4\ub294 \uad81\uadf9\uc801\uc778 \ubaa9\ud45c\ub294 \uc194\ub85c \uc544\ud2f0\uc2a4\ud2b8\ub77c\uace0 \ub9d0\ud588\uc73c\uba70, \uadf8\ub294 \uae30\uaebc\uc774 \ub2e4\ub978 \uc544\ud2f0\uc2a4\ud2b8\ub97c \uc704\ud574 \uace1\uc744 \uc4f0\uace0 \ud504\ub85c\ub4c0\uc2a4\ud558\ub294 \uac83\uc5d0 \ub300\ud574 \uc790\uc2e0\uc758 \uc791\uace1 \ub2a5\ub825\uc744 \ud5a5\uc0c1\uc2dc\ud0a4\uace0 \uadf8 \uc544\ud2f0\uc2a4\ud2b8\uc758 \uc720\ud615\uc744 \ubc1c\uacac\ud560 \uc218 \uc788\ub2e4\uace0 \ud558\uc600\ub2e4. \ubca0\uc774\uc155\uc740 \ub9c8\uc2a4\uac00 \ub098\uc911\uc5d0 \uc131\uacf5\ud55c \uac83\uc740 \uc774 \uae30\uac04\uc5d0 \uc758\ud55c \"\uc790\uc544 \ubc1c\uacac\"\uc774 \uc0c1\ub2f9\ud788 \uc601\ud5a5\uc774 \ub07c\ucce4\ub2e4\uace0 \ud558\uc600\ub2e4.", "sentence_answer": "\ub85c\uc2a4\uc564\uc824\ub808\uc2a4\ub85c \uc774\ub3d9\ud55c \ud6c4, 2004\ub144 \ub9c8\uc2a4\ub294 \ubaa8\ud0c0\uc6b4\uacfc \uacc4\uc57d\uc744 \ub9fa\uc5c8\ub2e4.", "paragraph_id": "6535231-4-0", "paragraph_question": "question: \ub85c\uc2a4\uc564\uc824\ub808\uc2a4\ub85c \uc774\uc8fc\ud55c \uc774\ud6c4 \ub9c8\uc2a4\uac00 \ubaa8\ud0c0\uc6b4\uacfc \uacc4\uc57d\uc744 \ub9fa\uc740 \ub144\ub3c4\ub294?, context: \ub85c\uc2a4\uc564\uc824\ub808\uc2a4\ub85c \uc774\ub3d9\ud55c \ud6c4, 2004\ub144 \ub9c8\uc2a4\ub294 \ubaa8\ud0c0\uc6b4\uacfc \uacc4\uc57d\uc744 \ub9fa\uc5c8\ub2e4. \ubaa8\ud0c0\uc6b4\uacfc\uc758 \uacc4\uc57d\uc740 \uc131\uacfc\uac00 \uc5c6\uc5c8\uc73c\uba70, \uace7 \uc1a1\ub77c\uc774\ud130\uc774\uc790 \ud504\ub85c\ub4c0\uc11c \ud544\ub9bd \ub85c\ub7f0\uc2a4(Philip Lawrence)\ub97c \ub9cc\ub098\uac8c \ub418\uc5c8\ub2e4. \ub9c8\uc2a4, \ub85c\ub7f0\uc2a4, \uc560\ub9ac \ub7ec\ubc14\uc778(Ari Levine)\uc740 \ud504\ub85c\ub4c0\uc2f1\ud300 \uc2a4\ubbf8\uc9d5\ud134\uc2a4(The Smeezingtons)\ub97c \uacb0\uc131\ud558\uc600\ub2e4. 2006\ub144, \ub85c\ub7f0\uc2a4\ub294 \uc560\ud2c0\ub79c\ud2f1 \ub808\ucf54\ub4dc\uc758 \ubbf8\ub798 \uad00\ub9ac\uc790 \uc5d0\ub7f0 \ubca0\uc774\uc155(Aaron Bay-Schuck)\uc5d0\uac8c \ub9c8\uc2a4\ub97c \uc18c\uac1c\ud574\uc8fc\uc5c8\ub2e4. \ub9c8\uc2a4\ub294 \uadf8\uc5d0\uac8c 2\uace1\uc744 \ub4e4\ub824\uc8fc\uc5c8\uace0, \ubca0\uc774\uc155\uc740 \ub9c8\uc2a4\ub97c \ubc14\ub85c \uacc4\uc57d\ud558\uace0 \uc2f6\uc5b4\ud588\ub2e4. \ud558\uc9c0\ub9cc \uacb0\uad6d, \uacc4\uc57d\ud558\ub294 \ub370 3\ub144\uc774 \uac78\ub838\ub2e4. \ubca0\uc774\uc155\uc740 \ub9c8\uc2a4\uc640 \uc2a4\ubbf8\uc9d5\ud134\uc2a4\ub97c \uc560\ud2c0\ub79c\ud2f1 \ub808\ucf54\ub4dc \uc544\ud2f0\uc2a4\ud2b8\ub97c \uc704\ud55c \uc791\uace1 \ubc0f \ud504\ub85c\ub4c0\uc2f1 \ubd80\ubd84\uc73c\ub85c \uacc4\uc57d\ud558\uc600\ub2e4. \ud79b\ucffc\ud130\uc2a4\uc640\uc758 \uc778\ud130\ubdf0\uc5d0\uc11c \ubca0\uc774\uc155\uc5d0 \ub530\ub974\uba74, \ub9c8\uc2a4\ub294 \uad81\uadf9\uc801\uc778 \ubaa9\ud45c\ub294 \uc194\ub85c \uc544\ud2f0\uc2a4\ud2b8\ub77c\uace0 \ub9d0\ud588\uc73c\uba70, \uadf8\ub294 \uae30\uaebc\uc774 \ub2e4\ub978 \uc544\ud2f0\uc2a4\ud2b8\ub97c \uc704\ud574 \uace1\uc744 \uc4f0\uace0 \ud504\ub85c\ub4c0\uc2a4\ud558\ub294 \uac83\uc5d0 \ub300\ud574 \uc790\uc2e0\uc758 \uc791\uace1 \ub2a5\ub825\uc744 \ud5a5\uc0c1\uc2dc\ud0a4\uace0 \uadf8 \uc544\ud2f0\uc2a4\ud2b8\uc758 \uc720\ud615\uc744 \ubc1c\uacac\ud560 \uc218 \uc788\ub2e4\uace0 \ud558\uc600\ub2e4. \ubca0\uc774\uc155\uc740 \ub9c8\uc2a4\uac00 \ub098\uc911\uc5d0 \uc131\uacf5\ud55c \uac83\uc740 \uc774 \uae30\uac04\uc5d0 \uc758\ud55c \"\uc790\uc544 \ubc1c\uacac\"\uc774 \uc0c1\ub2f9\ud788 \uc601\ud5a5\uc774 \ub07c\ucce4\ub2e4\uace0 \ud558\uc600\ub2e4."} {"question": "\uace0\uc7ac\uc5f4 \uae30\uc790 \uc8fc\ucd5c\uc758 \ube14\ub85c\uac70\uc640\uc758 \ub300\ud654 \uccab \ucd08\ub300 \uba85\uc0ac\uc600\uc73c\uba70 \uace0\uc7ac\uc5f4\uc5d0\uac8c \ub2e4\uc74c \ucd08\ub300 \uc778\ubb3c\ub85c \ucd5c\uc9c4\uc2e4\uc744 \ucd94\ucc9c\ud55c \uc0ac\ub78c\uc740?", "paragraph": "2008\ub144 9\uc6d4 30\uc77c, \uc2dc\uc0acIN\uc758 \uace0\uc7ac\uc5f4 \uae30\uc790 \uc8fc\ucd5c\uc758 \u300a\ube14\ub85c\uac70\uc640\uc758 \ub300\ud654\u300b\uc5d0 \uccab \ucd08\ub300 \uba85\uc0ac\ub85c \ub098\uc628 \ubbfc\uc8fc\ub2f9 \uc18c\uc18d \uad6d\ud68c\uc758\uc6d0 \ucd5c\ubb38\uc21c\uc740 \ube14\ub85c\uac70\ub4e4\uc758 \ub2e4\uc74c \ub300\ud654 \uc0c1\ub300\uc790\ub85c \ucd5c\uc9c4\uc2e4\uc744 \ucd94\ucc9c\ud588\ub2e4. \ucd5c\ubb38\uc21c\uc740 \"\ucd5c\uc9c4\uc2e4\uc740 \uc790\ub140\uc758 \uc131\ubcf8(\u59d3\u672c) \ubcc0\uacbd \uc774\ud6c4 \uc0ac\ud68c\uc758\uc2dd\uc5d0 \ub208\uc744 \ub5b4\ub2e4. \uc5ec\ub7ec \uac00\uc9c0 \ud798\ub4e0 \uc77c\ub4e4\uc744 \uacaa\uc73c\uba74\uc11c \uc0ac\ud68c\ubb38\uc81c\uc5d0 \ub208\uc744 \ub5b4\uace0 \ub208\uc744 \ub72c \uac83\uc744 \ub118\uc5b4\uc11c \uc790\ub140\ub4e4\uc5d0\uac8c \uc790\uc2e0\uc758 \uc131(\u59d3)\uc744 \uc4f0\uac8c \ud560 \ub9cc\ud07c \uc0ac\ud68c \uc9c4\ubcf4\uc758 \uc120\ubd09\uc5d0 \uc120 \uadf8\ub97c \ub9cc\ub098\ubd10\uc57c \ud55c\ub2e4. \uc6b0\ub9ac\uac00 \uc54c\uace0 \uc788\ub294 \ucd5c\uc9c4\uc2e4\uacfc \ub2e4\ub978 \ub610 \ub2e4\ub978 \ucd5c\uc9c4\uc2e4\uc744 \ubc1c\uacac\ud560 \uc218 \uc788\ub2e4\"\ub77c\uace0 \ub9d0\ud558\uba74\uc11c \ucd5c\uc9c4\uc2e4\uc744 \ube14\ub85c\uac70\ub4e4\uc774 \ub9cc\ub098\ubcfc\ub9cc\ud55c \uc778\ubb3c\ub85c \ud3c9\uac00\ud588\ub2e4. \ucd5c\uc9c4\uc2e4\uacfc \ucd5c\ubb38\uc21c\uc740 \ucd5c\ubb38\uc21c\uc774 MBC \uc0ac\uc7a5\uc73c\ub85c \uc7ac\uc9c1\ud558\ub358 \uc2dc\uc808\uc778 2005\ub144\ubd80\ud130 \uce5c\ubd84\uc744 \ub098\ub208 \uc0ac\uc774\uc600\ub2e4. \ucd5c\ubb38\uc21c\uc740 \uace0\uc7ac\uc5f4\uc5d0\uac8c '\uc544\ub294 \uc5f0\uc608\uc778, \ucd5c\uc9c4\uc2e4', '\uce5c\ud55c \uc5f0\uc608\uc778, \ucd5c\uc9c4\uc2e4', '\uc88b\uc544\ud558\ub294 \uc5f0\uc608\uc778, \ucd5c\uc9c4\uc2e4', '\ube14\ub85c\uac70\ub4e4\uc774 \ub9cc\ub098\uc57c \ud560 \uc5f0\uc608\uc778, \ucd5c\uc9c4\uc2e4', '\uc790\uc2e0\uc774 \uc12d\uc678\ud574\uc904 \uc218 \uc788\ub294 \uc5f0\uc608\uc778, \ucd5c\uc9c4\uc2e4', '\u300a\ube14\ub85c\uac70\uc640\uc758 \ub300\ud654\u300b\uc5d0 \uc790\uc2e0\uc774 \ube14\ub85c\uac70 \uc790\uaca9\uc73c\ub85c \ud328\ub110\ub85c \ucc38\uc5ec\ud560 \ub300\uc0c1, \ucd5c\uc9c4\uc2e4' \ub4f1\uc73c\ub85c \ucd5c\uc9c4\uc2e4\uc5d0 \ub300\ud574 \ub9d0\ud588\uace0, \uace0\uc7ac\uc5f4\uc740 \u300a\ube14\ub85c\uac70\uc640\uc758 \ub300\ud654\u300b\ub97c \uc548\ucc29\uc2dc\ud0a4\uae30 \uc704\ud574 \ucd5c\uc9c4\uc2e4\uc744 \ucd5c\ub300\ud55c \ube68\ub9ac \ucd08\ub300\ud558\ub824\uace0 \uacc4\ud68d\ud588\ub2e4.", "answer": "\ucd5c\ubb38\uc21c", "sentence": "2008\ub144 9\uc6d4 30\uc77c, \uc2dc\uc0acIN\uc758 \uace0\uc7ac\uc5f4 \uae30\uc790 \uc8fc\ucd5c\uc758 \u300a\ube14\ub85c\uac70\uc640\uc758 \ub300\ud654\u300b\uc5d0 \uccab \ucd08\ub300 \uba85\uc0ac\ub85c \ub098\uc628 \ubbfc\uc8fc\ub2f9 \uc18c\uc18d \uad6d\ud68c\uc758\uc6d0 \ucd5c\ubb38\uc21c \uc740 \ube14\ub85c\uac70\ub4e4\uc758 \ub2e4\uc74c \ub300\ud654 \uc0c1\ub300\uc790\ub85c \ucd5c\uc9c4\uc2e4\uc744 \ucd94\ucc9c\ud588\ub2e4.", "paragraph_sentence": " 2008\ub144 9\uc6d4 30\uc77c, \uc2dc\uc0acIN\uc758 \uace0\uc7ac\uc5f4 \uae30\uc790 \uc8fc\ucd5c\uc758 \u300a\ube14\ub85c\uac70\uc640\uc758 \ub300\ud654\u300b\uc5d0 \uccab \ucd08\ub300 \uba85\uc0ac\ub85c \ub098\uc628 \ubbfc\uc8fc\ub2f9 \uc18c\uc18d \uad6d\ud68c\uc758\uc6d0 \ucd5c\ubb38\uc21c \uc740 \ube14\ub85c\uac70\ub4e4\uc758 \ub2e4\uc74c \ub300\ud654 \uc0c1\ub300\uc790\ub85c \ucd5c\uc9c4\uc2e4\uc744 \ucd94\ucc9c\ud588\ub2e4. \ucd5c\ubb38\uc21c\uc740 \"\ucd5c\uc9c4\uc2e4\uc740 \uc790\ub140\uc758 \uc131\ubcf8(\u59d3\u672c) \ubcc0\uacbd \uc774\ud6c4 \uc0ac\ud68c\uc758\uc2dd\uc5d0 \ub208\uc744 \ub5b4\ub2e4. \uc5ec\ub7ec \uac00\uc9c0 \ud798\ub4e0 \uc77c\ub4e4\uc744 \uacaa\uc73c\uba74\uc11c \uc0ac\ud68c\ubb38\uc81c\uc5d0 \ub208\uc744 \ub5b4\uace0 \ub208\uc744 \ub72c \uac83\uc744 \ub118\uc5b4\uc11c \uc790\ub140\ub4e4\uc5d0\uac8c \uc790\uc2e0\uc758 \uc131(\u59d3)\uc744 \uc4f0\uac8c \ud560 \ub9cc\ud07c \uc0ac\ud68c \uc9c4\ubcf4\uc758 \uc120\ubd09\uc5d0 \uc120 \uadf8\ub97c \ub9cc\ub098\ubd10\uc57c \ud55c\ub2e4. \uc6b0\ub9ac\uac00 \uc54c\uace0 \uc788\ub294 \ucd5c\uc9c4\uc2e4\uacfc \ub2e4\ub978 \ub610 \ub2e4\ub978 \ucd5c\uc9c4\uc2e4\uc744 \ubc1c\uacac\ud560 \uc218 \uc788\ub2e4\"\ub77c\uace0 \ub9d0\ud558\uba74\uc11c \ucd5c\uc9c4\uc2e4\uc744 \ube14\ub85c\uac70\ub4e4\uc774 \ub9cc\ub098\ubcfc\ub9cc\ud55c \uc778\ubb3c\ub85c \ud3c9\uac00\ud588\ub2e4. \ucd5c\uc9c4\uc2e4\uacfc \ucd5c\ubb38\uc21c\uc740 \ucd5c\ubb38\uc21c\uc774 MBC \uc0ac\uc7a5\uc73c\ub85c \uc7ac\uc9c1\ud558\ub358 \uc2dc\uc808\uc778 2005\ub144\ubd80\ud130 \uce5c\ubd84\uc744 \ub098\ub208 \uc0ac\uc774\uc600\ub2e4. \ucd5c\ubb38\uc21c\uc740 \uace0\uc7ac\uc5f4\uc5d0\uac8c '\uc544\ub294 \uc5f0\uc608\uc778, \ucd5c\uc9c4\uc2e4', '\uce5c\ud55c \uc5f0\uc608\uc778, \ucd5c\uc9c4\uc2e4', '\uc88b\uc544\ud558\ub294 \uc5f0\uc608\uc778, \ucd5c\uc9c4\uc2e4', '\ube14\ub85c\uac70\ub4e4\uc774 \ub9cc\ub098\uc57c \ud560 \uc5f0\uc608\uc778, \ucd5c\uc9c4\uc2e4', '\uc790\uc2e0\uc774 \uc12d\uc678\ud574\uc904 \uc218 \uc788\ub294 \uc5f0\uc608\uc778, \ucd5c\uc9c4\uc2e4', '\u300a\ube14\ub85c\uac70\uc640\uc758 \ub300\ud654\u300b\uc5d0 \uc790\uc2e0\uc774 \ube14\ub85c\uac70 \uc790\uaca9\uc73c\ub85c \ud328\ub110\ub85c \ucc38\uc5ec\ud560 \ub300\uc0c1, \ucd5c\uc9c4\uc2e4' \ub4f1\uc73c\ub85c \ucd5c\uc9c4\uc2e4\uc5d0 \ub300\ud574 \ub9d0\ud588\uace0, \uace0\uc7ac\uc5f4\uc740 \u300a\ube14\ub85c\uac70\uc640\uc758 \ub300\ud654\u300b\ub97c \uc548\ucc29\uc2dc\ud0a4\uae30 \uc704\ud574 \ucd5c\uc9c4\uc2e4\uc744 \ucd5c\ub300\ud55c \ube68\ub9ac \ucd08\ub300\ud558\ub824\uace0 \uacc4\ud68d\ud588\ub2e4.", "paragraph_answer": "2008\ub144 9\uc6d4 30\uc77c, \uc2dc\uc0acIN\uc758 \uace0\uc7ac\uc5f4 \uae30\uc790 \uc8fc\ucd5c\uc758 \u300a\ube14\ub85c\uac70\uc640\uc758 \ub300\ud654\u300b\uc5d0 \uccab \ucd08\ub300 \uba85\uc0ac\ub85c \ub098\uc628 \ubbfc\uc8fc\ub2f9 \uc18c\uc18d \uad6d\ud68c\uc758\uc6d0 \ucd5c\ubb38\uc21c \uc740 \ube14\ub85c\uac70\ub4e4\uc758 \ub2e4\uc74c \ub300\ud654 \uc0c1\ub300\uc790\ub85c \ucd5c\uc9c4\uc2e4\uc744 \ucd94\ucc9c\ud588\ub2e4. \ucd5c\ubb38\uc21c\uc740 \"\ucd5c\uc9c4\uc2e4\uc740 \uc790\ub140\uc758 \uc131\ubcf8(\u59d3\u672c) \ubcc0\uacbd \uc774\ud6c4 \uc0ac\ud68c\uc758\uc2dd\uc5d0 \ub208\uc744 \ub5b4\ub2e4. \uc5ec\ub7ec \uac00\uc9c0 \ud798\ub4e0 \uc77c\ub4e4\uc744 \uacaa\uc73c\uba74\uc11c \uc0ac\ud68c\ubb38\uc81c\uc5d0 \ub208\uc744 \ub5b4\uace0 \ub208\uc744 \ub72c \uac83\uc744 \ub118\uc5b4\uc11c \uc790\ub140\ub4e4\uc5d0\uac8c \uc790\uc2e0\uc758 \uc131(\u59d3)\uc744 \uc4f0\uac8c \ud560 \ub9cc\ud07c \uc0ac\ud68c \uc9c4\ubcf4\uc758 \uc120\ubd09\uc5d0 \uc120 \uadf8\ub97c \ub9cc\ub098\ubd10\uc57c \ud55c\ub2e4. \uc6b0\ub9ac\uac00 \uc54c\uace0 \uc788\ub294 \ucd5c\uc9c4\uc2e4\uacfc \ub2e4\ub978 \ub610 \ub2e4\ub978 \ucd5c\uc9c4\uc2e4\uc744 \ubc1c\uacac\ud560 \uc218 \uc788\ub2e4\"\ub77c\uace0 \ub9d0\ud558\uba74\uc11c \ucd5c\uc9c4\uc2e4\uc744 \ube14\ub85c\uac70\ub4e4\uc774 \ub9cc\ub098\ubcfc\ub9cc\ud55c \uc778\ubb3c\ub85c \ud3c9\uac00\ud588\ub2e4. \ucd5c\uc9c4\uc2e4\uacfc \ucd5c\ubb38\uc21c\uc740 \ucd5c\ubb38\uc21c\uc774 MBC \uc0ac\uc7a5\uc73c\ub85c \uc7ac\uc9c1\ud558\ub358 \uc2dc\uc808\uc778 2005\ub144\ubd80\ud130 \uce5c\ubd84\uc744 \ub098\ub208 \uc0ac\uc774\uc600\ub2e4. \ucd5c\ubb38\uc21c\uc740 \uace0\uc7ac\uc5f4\uc5d0\uac8c '\uc544\ub294 \uc5f0\uc608\uc778, \ucd5c\uc9c4\uc2e4', '\uce5c\ud55c \uc5f0\uc608\uc778, \ucd5c\uc9c4\uc2e4', '\uc88b\uc544\ud558\ub294 \uc5f0\uc608\uc778, \ucd5c\uc9c4\uc2e4', '\ube14\ub85c\uac70\ub4e4\uc774 \ub9cc\ub098\uc57c \ud560 \uc5f0\uc608\uc778, \ucd5c\uc9c4\uc2e4', '\uc790\uc2e0\uc774 \uc12d\uc678\ud574\uc904 \uc218 \uc788\ub294 \uc5f0\uc608\uc778, \ucd5c\uc9c4\uc2e4', '\u300a\ube14\ub85c\uac70\uc640\uc758 \ub300\ud654\u300b\uc5d0 \uc790\uc2e0\uc774 \ube14\ub85c\uac70 \uc790\uaca9\uc73c\ub85c \ud328\ub110\ub85c \ucc38\uc5ec\ud560 \ub300\uc0c1, \ucd5c\uc9c4\uc2e4' \ub4f1\uc73c\ub85c \ucd5c\uc9c4\uc2e4\uc5d0 \ub300\ud574 \ub9d0\ud588\uace0, \uace0\uc7ac\uc5f4\uc740 \u300a\ube14\ub85c\uac70\uc640\uc758 \ub300\ud654\u300b\ub97c \uc548\ucc29\uc2dc\ud0a4\uae30 \uc704\ud574 \ucd5c\uc9c4\uc2e4\uc744 \ucd5c\ub300\ud55c \ube68\ub9ac \ucd08\ub300\ud558\ub824\uace0 \uacc4\ud68d\ud588\ub2e4.", "sentence_answer": "2008\ub144 9\uc6d4 30\uc77c, \uc2dc\uc0acIN\uc758 \uace0\uc7ac\uc5f4 \uae30\uc790 \uc8fc\ucd5c\uc758 \u300a\ube14\ub85c\uac70\uc640\uc758 \ub300\ud654\u300b\uc5d0 \uccab \ucd08\ub300 \uba85\uc0ac\ub85c \ub098\uc628 \ubbfc\uc8fc\ub2f9 \uc18c\uc18d \uad6d\ud68c\uc758\uc6d0 \ucd5c\ubb38\uc21c \uc740 \ube14\ub85c\uac70\ub4e4\uc758 \ub2e4\uc74c \ub300\ud654 \uc0c1\ub300\uc790\ub85c \ucd5c\uc9c4\uc2e4\uc744 \ucd94\ucc9c\ud588\ub2e4.", "paragraph_id": "6596968-19-0", "paragraph_question": "question: \uace0\uc7ac\uc5f4 \uae30\uc790 \uc8fc\ucd5c\uc758 \ube14\ub85c\uac70\uc640\uc758 \ub300\ud654 \uccab \ucd08\ub300 \uba85\uc0ac\uc600\uc73c\uba70 \uace0\uc7ac\uc5f4\uc5d0\uac8c \ub2e4\uc74c \ucd08\ub300 \uc778\ubb3c\ub85c \ucd5c\uc9c4\uc2e4\uc744 \ucd94\ucc9c\ud55c \uc0ac\ub78c\uc740?, context: 2008\ub144 9\uc6d4 30\uc77c, \uc2dc\uc0acIN\uc758 \uace0\uc7ac\uc5f4 \uae30\uc790 \uc8fc\ucd5c\uc758 \u300a\ube14\ub85c\uac70\uc640\uc758 \ub300\ud654\u300b\uc5d0 \uccab \ucd08\ub300 \uba85\uc0ac\ub85c \ub098\uc628 \ubbfc\uc8fc\ub2f9 \uc18c\uc18d \uad6d\ud68c\uc758\uc6d0 \ucd5c\ubb38\uc21c\uc740 \ube14\ub85c\uac70\ub4e4\uc758 \ub2e4\uc74c \ub300\ud654 \uc0c1\ub300\uc790\ub85c \ucd5c\uc9c4\uc2e4\uc744 \ucd94\ucc9c\ud588\ub2e4. \ucd5c\ubb38\uc21c\uc740 \"\ucd5c\uc9c4\uc2e4\uc740 \uc790\ub140\uc758 \uc131\ubcf8(\u59d3\u672c) \ubcc0\uacbd \uc774\ud6c4 \uc0ac\ud68c\uc758\uc2dd\uc5d0 \ub208\uc744 \ub5b4\ub2e4. \uc5ec\ub7ec \uac00\uc9c0 \ud798\ub4e0 \uc77c\ub4e4\uc744 \uacaa\uc73c\uba74\uc11c \uc0ac\ud68c\ubb38\uc81c\uc5d0 \ub208\uc744 \ub5b4\uace0 \ub208\uc744 \ub72c \uac83\uc744 \ub118\uc5b4\uc11c \uc790\ub140\ub4e4\uc5d0\uac8c \uc790\uc2e0\uc758 \uc131(\u59d3)\uc744 \uc4f0\uac8c \ud560 \ub9cc\ud07c \uc0ac\ud68c \uc9c4\ubcf4\uc758 \uc120\ubd09\uc5d0 \uc120 \uadf8\ub97c \ub9cc\ub098\ubd10\uc57c \ud55c\ub2e4. \uc6b0\ub9ac\uac00 \uc54c\uace0 \uc788\ub294 \ucd5c\uc9c4\uc2e4\uacfc \ub2e4\ub978 \ub610 \ub2e4\ub978 \ucd5c\uc9c4\uc2e4\uc744 \ubc1c\uacac\ud560 \uc218 \uc788\ub2e4\"\ub77c\uace0 \ub9d0\ud558\uba74\uc11c \ucd5c\uc9c4\uc2e4\uc744 \ube14\ub85c\uac70\ub4e4\uc774 \ub9cc\ub098\ubcfc\ub9cc\ud55c \uc778\ubb3c\ub85c \ud3c9\uac00\ud588\ub2e4. \ucd5c\uc9c4\uc2e4\uacfc \ucd5c\ubb38\uc21c\uc740 \ucd5c\ubb38\uc21c\uc774 MBC \uc0ac\uc7a5\uc73c\ub85c \uc7ac\uc9c1\ud558\ub358 \uc2dc\uc808\uc778 2005\ub144\ubd80\ud130 \uce5c\ubd84\uc744 \ub098\ub208 \uc0ac\uc774\uc600\ub2e4. \ucd5c\ubb38\uc21c\uc740 \uace0\uc7ac\uc5f4\uc5d0\uac8c '\uc544\ub294 \uc5f0\uc608\uc778, \ucd5c\uc9c4\uc2e4', '\uce5c\ud55c \uc5f0\uc608\uc778, \ucd5c\uc9c4\uc2e4', '\uc88b\uc544\ud558\ub294 \uc5f0\uc608\uc778, \ucd5c\uc9c4\uc2e4', '\ube14\ub85c\uac70\ub4e4\uc774 \ub9cc\ub098\uc57c \ud560 \uc5f0\uc608\uc778, \ucd5c\uc9c4\uc2e4', '\uc790\uc2e0\uc774 \uc12d\uc678\ud574\uc904 \uc218 \uc788\ub294 \uc5f0\uc608\uc778, \ucd5c\uc9c4\uc2e4', '\u300a\ube14\ub85c\uac70\uc640\uc758 \ub300\ud654\u300b\uc5d0 \uc790\uc2e0\uc774 \ube14\ub85c\uac70 \uc790\uaca9\uc73c\ub85c \ud328\ub110\ub85c \ucc38\uc5ec\ud560 \ub300\uc0c1, \ucd5c\uc9c4\uc2e4' \ub4f1\uc73c\ub85c \ucd5c\uc9c4\uc2e4\uc5d0 \ub300\ud574 \ub9d0\ud588\uace0, \uace0\uc7ac\uc5f4\uc740 \u300a\ube14\ub85c\uac70\uc640\uc758 \ub300\ud654\u300b\ub97c \uc548\ucc29\uc2dc\ud0a4\uae30 \uc704\ud574 \ucd5c\uc9c4\uc2e4\uc744 \ucd5c\ub300\ud55c \ube68\ub9ac \ucd08\ub300\ud558\ub824\uace0 \uacc4\ud68d\ud588\ub2e4."} {"question": "\ub300\uad90 \uc548\uc758 \uc911\ub300\ud55c \uc5c5\ubb34\uc5d0 \ub300\ud574 \uac15\ub300\ud55c \uad8c\ud55c\uc744 \uc190\uc548\uc5d0 \uc950\uace0 \uc788\ub358 \uad00\uccad\uc744 \ubb34\uc5c7\uc774\ub77c \ud558\ub294\uac00?", "paragraph": "\uc18d\uc0ac(\u5c6c\u53f8)\ub85c\ub294 \uc0ac\uc778\uc18c(\u820d\u4eba\u6240)\uac00 \uc788\uc5b4\uc11c \ub300\uc18c \uc591\ubc18\uc758 \uc790\uc11c\uc81c\uc9c8(\u5b50\u58fb\u5f1f\u59ea : \uc544\ub4e4\uacfc \uc0ac\uc704\uc640 \uc544\uc6b0\uc640 \uc870\uce74)\uc744 \uc218\uc6a9\ud558\uace0, \uacbd\uc0ac(\u7d93\u53f2)\u00b7\ubcd1\uc11c(\u5175\u66f8)\u00b7\uc728\ubb38(\u5f8b\u6587)\u00b7\uc0b0\uc218(\u7b97\u6578)\u00b7\uc0ac\uc5b4(\u5c04\u5fa1) \ub4f1\uc744 \uac00\ub974\ucce4\uace0, \uc219\uc704(\u5bbf\u885b)\uc5d0\ub3c4 \ucda9\ub2f9\ud558\ub294 \ud55c\ud3b8 \uac01\uc790\uc758 \uc790\uc9c8\uc5d0 \ub530\ub77c \ubc1c\ud0c1 \ub4f1\uc6a9\ud558\uc600\ub2e4. \ub2f9\uc2dc\uc758 \ubb38\ud5cc\uc5d0\ub294 \uc0bc\uad70\ubd80\ub294 \uc8fc\uad00(\u5468\u5b98)\uc758 \uc0ac\ub9c8(\u53f8\u99ac)\uc758 \uc9c1\uc704\ub77c\ub4e0\uac00 \uad70\ubb34\ubcd1\uad8c\uc744 \uc804\uc7a5(\u5c08\u638c)\ud55c\ub2e4\ub294 \ub9d0\uc774 \ubcf4\uc774\uba70, \ub300\uad90 \uc548\uc758 \uc218\uc704, \ub3c4\uc131\uc758 \uc21c\ucc30 \ub4f1\uc744 \ucd1d\uad04\ud558\ub294 \uc678\uc5d0 \uad70\uc815\u00b7\uad70\ubb34\uc5d0 \uad00\ud558\uc5ec\ub3c4 \uac15\ub300\ud55c \uad8c\ud55c\uc744 \uc7a5\uc545\ud558\uace0 \uc788\uc5c8\ub2e4. \ubb34\uc120(\u6b66\u9078 : \uad70 \uc778\uc0ac)\u00b7\ubcd1\uc801(\u5175\u7c4d)\u00b7\uc6b0\uc5ed(\u90f5\u9a5b)\uc744 \uad00\uc7a5\ud558\ub294 \ubcd1\uc870(\u5175\u66f9)\ub294 \uc0ac\ubb34 \ucc98\ub9ac\ub97c \ud568\uc5d0 \uadf8\ucce4\uace0, \uc911\ucd94\uc6d0(\u4e2d\u6a1e\u9662)\uc758 \uc9c1\uc7a5(\u8077\u638c)\uc778 \ubcd1\uae30(\u5175\u6a5f)\u00b7\uad70\uc815(\u8ecd\u653f)\u00b7\uc219\uc704\u00b7\uacbd\ube44 \ub4f1\uc740 \uc0bc\uad70\ubd80\uc5d0 \uc62e\uaca8\uc838 \uc788\uc5c8\ub2e4.", "answer": "\uc0bc\uad70\ubd80", "sentence": "\ub2f9\uc2dc\uc758 \ubb38\ud5cc\uc5d0\ub294 \uc0bc\uad70\ubd80 \ub294 \uc8fc\uad00(\u5468\u5b98)\uc758 \uc0ac\ub9c8(\u53f8\u99ac)\uc758 \uc9c1\uc704\ub77c\ub4e0\uac00 \uad70\ubb34\ubcd1\uad8c\uc744 \uc804\uc7a5(\u5c08\u638c)\ud55c\ub2e4\ub294 \ub9d0\uc774 \ubcf4\uc774\uba70, \ub300\uad90 \uc548\uc758 \uc218\uc704, \ub3c4\uc131\uc758 \uc21c\ucc30 \ub4f1\uc744 \ucd1d\uad04\ud558\ub294 \uc678\uc5d0 \uad70\uc815\u00b7\uad70\ubb34\uc5d0 \uad00\ud558\uc5ec\ub3c4 \uac15\ub300\ud55c \uad8c\ud55c\uc744 \uc7a5\uc545\ud558\uace0 \uc788\uc5c8\ub2e4.", "paragraph_sentence": "\uc18d\uc0ac(\u5c6c\u53f8)\ub85c\ub294 \uc0ac\uc778\uc18c(\u820d\u4eba\u6240)\uac00 \uc788\uc5b4\uc11c \ub300\uc18c \uc591\ubc18\uc758 \uc790\uc11c\uc81c\uc9c8(\u5b50\u58fb\u5f1f\u59ea : \uc544\ub4e4\uacfc \uc0ac\uc704\uc640 \uc544\uc6b0\uc640 \uc870\uce74)\uc744 \uc218\uc6a9\ud558\uace0, \uacbd\uc0ac(\u7d93\u53f2)\u00b7\ubcd1\uc11c(\u5175\u66f8)\u00b7\uc728\ubb38(\u5f8b\u6587)\u00b7\uc0b0\uc218(\u7b97\u6578)\u00b7\uc0ac\uc5b4(\u5c04\u5fa1) \ub4f1\uc744 \uac00\ub974\ucce4\uace0, \uc219\uc704(\u5bbf\u885b)\uc5d0\ub3c4 \ucda9\ub2f9\ud558\ub294 \ud55c\ud3b8 \uac01\uc790\uc758 \uc790\uc9c8\uc5d0 \ub530\ub77c \ubc1c\ud0c1 \ub4f1\uc6a9\ud558\uc600\ub2e4. \ub2f9\uc2dc\uc758 \ubb38\ud5cc\uc5d0\ub294 \uc0bc\uad70\ubd80 \ub294 \uc8fc\uad00(\u5468\u5b98)\uc758 \uc0ac\ub9c8(\u53f8\u99ac)\uc758 \uc9c1\uc704\ub77c\ub4e0\uac00 \uad70\ubb34\ubcd1\uad8c\uc744 \uc804\uc7a5(\u5c08\u638c)\ud55c\ub2e4\ub294 \ub9d0\uc774 \ubcf4\uc774\uba70, \ub300\uad90 \uc548\uc758 \uc218\uc704, \ub3c4\uc131\uc758 \uc21c\ucc30 \ub4f1\uc744 \ucd1d\uad04\ud558\ub294 \uc678\uc5d0 \uad70\uc815\u00b7\uad70\ubb34\uc5d0 \uad00\ud558\uc5ec\ub3c4 \uac15\ub300\ud55c \uad8c\ud55c\uc744 \uc7a5\uc545\ud558\uace0 \uc788\uc5c8\ub2e4. \ubb34\uc120(\u6b66\u9078 : \uad70 \uc778\uc0ac)\u00b7\ubcd1\uc801(\u5175\u7c4d)\u00b7\uc6b0\uc5ed(\u90f5\u9a5b)\uc744 \uad00\uc7a5\ud558\ub294 \ubcd1\uc870(\u5175\u66f9)\ub294 \uc0ac\ubb34 \ucc98\ub9ac\ub97c \ud568\uc5d0 \uadf8\ucce4\uace0, \uc911\ucd94\uc6d0(\u4e2d\u6a1e\u9662)\uc758 \uc9c1\uc7a5(\u8077\u638c)\uc778 \ubcd1\uae30(\u5175\u6a5f)\u00b7\uad70\uc815(\u8ecd\u653f)\u00b7\uc219\uc704\u00b7\uacbd\ube44 \ub4f1\uc740 \uc0bc\uad70\ubd80\uc5d0 \uc62e\uaca8\uc838 \uc788\uc5c8\ub2e4.", "paragraph_answer": "\uc18d\uc0ac(\u5c6c\u53f8)\ub85c\ub294 \uc0ac\uc778\uc18c(\u820d\u4eba\u6240)\uac00 \uc788\uc5b4\uc11c \ub300\uc18c \uc591\ubc18\uc758 \uc790\uc11c\uc81c\uc9c8(\u5b50\u58fb\u5f1f\u59ea : \uc544\ub4e4\uacfc \uc0ac\uc704\uc640 \uc544\uc6b0\uc640 \uc870\uce74)\uc744 \uc218\uc6a9\ud558\uace0, \uacbd\uc0ac(\u7d93\u53f2)\u00b7\ubcd1\uc11c(\u5175\u66f8)\u00b7\uc728\ubb38(\u5f8b\u6587)\u00b7\uc0b0\uc218(\u7b97\u6578)\u00b7\uc0ac\uc5b4(\u5c04\u5fa1) \ub4f1\uc744 \uac00\ub974\ucce4\uace0, \uc219\uc704(\u5bbf\u885b)\uc5d0\ub3c4 \ucda9\ub2f9\ud558\ub294 \ud55c\ud3b8 \uac01\uc790\uc758 \uc790\uc9c8\uc5d0 \ub530\ub77c \ubc1c\ud0c1 \ub4f1\uc6a9\ud558\uc600\ub2e4. \ub2f9\uc2dc\uc758 \ubb38\ud5cc\uc5d0\ub294 \uc0bc\uad70\ubd80 \ub294 \uc8fc\uad00(\u5468\u5b98)\uc758 \uc0ac\ub9c8(\u53f8\u99ac)\uc758 \uc9c1\uc704\ub77c\ub4e0\uac00 \uad70\ubb34\ubcd1\uad8c\uc744 \uc804\uc7a5(\u5c08\u638c)\ud55c\ub2e4\ub294 \ub9d0\uc774 \ubcf4\uc774\uba70, \ub300\uad90 \uc548\uc758 \uc218\uc704, \ub3c4\uc131\uc758 \uc21c\ucc30 \ub4f1\uc744 \ucd1d\uad04\ud558\ub294 \uc678\uc5d0 \uad70\uc815\u00b7\uad70\ubb34\uc5d0 \uad00\ud558\uc5ec\ub3c4 \uac15\ub300\ud55c \uad8c\ud55c\uc744 \uc7a5\uc545\ud558\uace0 \uc788\uc5c8\ub2e4. \ubb34\uc120(\u6b66\u9078 : \uad70 \uc778\uc0ac)\u00b7\ubcd1\uc801(\u5175\u7c4d)\u00b7\uc6b0\uc5ed(\u90f5\u9a5b)\uc744 \uad00\uc7a5\ud558\ub294 \ubcd1\uc870(\u5175\u66f9)\ub294 \uc0ac\ubb34 \ucc98\ub9ac\ub97c \ud568\uc5d0 \uadf8\ucce4\uace0, \uc911\ucd94\uc6d0(\u4e2d\u6a1e\u9662)\uc758 \uc9c1\uc7a5(\u8077\u638c)\uc778 \ubcd1\uae30(\u5175\u6a5f)\u00b7\uad70\uc815(\u8ecd\u653f)\u00b7\uc219\uc704\u00b7\uacbd\ube44 \ub4f1\uc740 \uc0bc\uad70\ubd80\uc5d0 \uc62e\uaca8\uc838 \uc788\uc5c8\ub2e4.", "sentence_answer": "\ub2f9\uc2dc\uc758 \ubb38\ud5cc\uc5d0\ub294 \uc0bc\uad70\ubd80 \ub294 \uc8fc\uad00(\u5468\u5b98)\uc758 \uc0ac\ub9c8(\u53f8\u99ac)\uc758 \uc9c1\uc704\ub77c\ub4e0\uac00 \uad70\ubb34\ubcd1\uad8c\uc744 \uc804\uc7a5(\u5c08\u638c)\ud55c\ub2e4\ub294 \ub9d0\uc774 \ubcf4\uc774\uba70, \ub300\uad90 \uc548\uc758 \uc218\uc704, \ub3c4\uc131\uc758 \uc21c\ucc30 \ub4f1\uc744 \ucd1d\uad04\ud558\ub294 \uc678\uc5d0 \uad70\uc815\u00b7\uad70\ubb34\uc5d0 \uad00\ud558\uc5ec\ub3c4 \uac15\ub300\ud55c \uad8c\ud55c\uc744 \uc7a5\uc545\ud558\uace0 \uc788\uc5c8\ub2e4.", "paragraph_id": "6134794-2-0", "paragraph_question": "question: \ub300\uad90 \uc548\uc758 \uc911\ub300\ud55c \uc5c5\ubb34\uc5d0 \ub300\ud574 \uac15\ub300\ud55c \uad8c\ud55c\uc744 \uc190\uc548\uc5d0 \uc950\uace0 \uc788\ub358 \uad00\uccad\uc744 \ubb34\uc5c7\uc774\ub77c \ud558\ub294\uac00?, context: \uc18d\uc0ac(\u5c6c\u53f8)\ub85c\ub294 \uc0ac\uc778\uc18c(\u820d\u4eba\u6240)\uac00 \uc788\uc5b4\uc11c \ub300\uc18c \uc591\ubc18\uc758 \uc790\uc11c\uc81c\uc9c8(\u5b50\u58fb\u5f1f\u59ea : \uc544\ub4e4\uacfc \uc0ac\uc704\uc640 \uc544\uc6b0\uc640 \uc870\uce74)\uc744 \uc218\uc6a9\ud558\uace0, \uacbd\uc0ac(\u7d93\u53f2)\u00b7\ubcd1\uc11c(\u5175\u66f8)\u00b7\uc728\ubb38(\u5f8b\u6587)\u00b7\uc0b0\uc218(\u7b97\u6578)\u00b7\uc0ac\uc5b4(\u5c04\u5fa1) \ub4f1\uc744 \uac00\ub974\ucce4\uace0, \uc219\uc704(\u5bbf\u885b)\uc5d0\ub3c4 \ucda9\ub2f9\ud558\ub294 \ud55c\ud3b8 \uac01\uc790\uc758 \uc790\uc9c8\uc5d0 \ub530\ub77c \ubc1c\ud0c1 \ub4f1\uc6a9\ud558\uc600\ub2e4. \ub2f9\uc2dc\uc758 \ubb38\ud5cc\uc5d0\ub294 \uc0bc\uad70\ubd80\ub294 \uc8fc\uad00(\u5468\u5b98)\uc758 \uc0ac\ub9c8(\u53f8\u99ac)\uc758 \uc9c1\uc704\ub77c\ub4e0\uac00 \uad70\ubb34\ubcd1\uad8c\uc744 \uc804\uc7a5(\u5c08\u638c)\ud55c\ub2e4\ub294 \ub9d0\uc774 \ubcf4\uc774\uba70, \ub300\uad90 \uc548\uc758 \uc218\uc704, \ub3c4\uc131\uc758 \uc21c\ucc30 \ub4f1\uc744 \ucd1d\uad04\ud558\ub294 \uc678\uc5d0 \uad70\uc815\u00b7\uad70\ubb34\uc5d0 \uad00\ud558\uc5ec\ub3c4 \uac15\ub300\ud55c \uad8c\ud55c\uc744 \uc7a5\uc545\ud558\uace0 \uc788\uc5c8\ub2e4. \ubb34\uc120(\u6b66\u9078 : \uad70 \uc778\uc0ac)\u00b7\ubcd1\uc801(\u5175\u7c4d)\u00b7\uc6b0\uc5ed(\u90f5\u9a5b)\uc744 \uad00\uc7a5\ud558\ub294 \ubcd1\uc870(\u5175\u66f9)\ub294 \uc0ac\ubb34 \ucc98\ub9ac\ub97c \ud568\uc5d0 \uadf8\ucce4\uace0, \uc911\ucd94\uc6d0(\u4e2d\u6a1e\u9662)\uc758 \uc9c1\uc7a5(\u8077\u638c)\uc778 \ubcd1\uae30(\u5175\u6a5f)\u00b7\uad70\uc815(\u8ecd\u653f)\u00b7\uc219\uc704\u00b7\uacbd\ube44 \ub4f1\uc740 \uc0bc\uad70\ubd80\uc5d0 \uc62e\uaca8\uc838 \uc788\uc5c8\ub2e4."} {"question": "2010\ub144 8\uc6d4 5\uc77c \uc559\ub4dc\ub808 \uc544\uc720\ub294 \ub9c8\ub974\uc138\uc720\uc640 \uba87 \ub144\uc758 \uc5f0\uc7a5\uacc4\uc57d\uc744 \uccb4\uacb0\ud558\uc600\ub098?", "paragraph": "\uc544\ub97c \uc544\ube44\ub1fd\uc5d0\uc11c \uc131\uacf5\uc801\uc778 \uc2dc\uc98c\uc744 \ubcf4\ub0b8 \uc544\uc720\ub294 2010\ub144 5\uc6d4 16\uc77c\uc5d0 \ub9c8\ub974\uc138\uc720 \uac10\ub3c5 \ub514\ub514\uc5d0 \ub370\uc0f9\uc73c\ub85c\ubd80\ud130 2010-11 \uc2dc\uc98c \ubcf5\uadc0\ub97c \ud655\uc778\ubc1b\uc558\uace0 \ub354 \uc911\ub300\ud55c \ucc45\uc784\uc744 \ub9e1\uae38 \uac83\uc774\ub77c\uace0 \ubc1c\ud45c\ud558\uc600\ub2e4. 8\uc6d4 5\uc77c, \uc544\uc720\ub294 \ub9c8\ub974\uc138\uc720\uc640 3\ub144 \uc5f0\uc7a5\uacc4\uc57d\uc744 \uccb4\uacb0\ud558\uc600\ub2e4. \uc774 \uacc4\uc57d\uc73c\ub85c \uadf8\ub294 2014\ub144 6\uc6d4\uae4c\uc9c0 \ub9c8\ub974\uc138\uc720 \uc120\uc218 \uc2e0\ubd84\uc73c\ub85c \ub0a8\uac8c \ub418\uc5c8\ub2e4. \ub85c\uc774\ud06c \ub808\ubbf8\uac00 \ud300\uc5d0 \ud569\ub958\ud558\uc600\uc74c\uc5d0\ub3c4 \ubd88\uad6c\ud558\uace0, \ub370\uc0f9 \uac10\ub3c5\uc740 \uc544\uc720\ub97c \uc8fc\uc804\uc73c\ub85c \ub0b4\uc138\uc6e0\ub2e4. \ubc1c\ub791\uc2dc\uc5d4 FC\uc640\uc758 \ub9ac\uadf8 2\ub77c\uc6b4\ub4dc \uacbd\uae30\uc5d0\uc11c, \uadf8\ub294 \uc2dc\uc98c \uccab\uace8\uc744 \uae30\ub85d\ud558\uc600\uc73c\ub098, \ub9c8\ub974\uc138\uc720\ub294 2-3\uc73c\ub85c \ud328\ud558\uc600\ub2e4. 9\uc6d4 \uad6d\uac00\ub300\ud56d\uc804 \uc774\ud6c4, \uadf8\ub294 AC \uc544\ub97c \uc544\ube44\ub1fd\uc744 \uc0c1\ub300\ub85c 2\uace8\uc744 \uae30\ub85d\ud558\uc5ec 3-0 \uc2b9\ub9ac\ub97c \uacac\uc778\ud558\uc600\ub2e4. \ucfe0\ud504 \ub4dc \ub77c \ub9ac\uadf8\uc5d0\uc11c, \uc544\uc720\ub294 16\uac15 \uc0c1\ub300 EA \uac31\uac15\uacfc 8\uac15 \uc0c1\ub300 AS \ubaa8\ub098\ucf54 FC\uc640\uc758 \uacbd\uae30\uc5d0\uc11c \uac01\uac01 \uacb0\uc2b9\uace8\uc744 \uae30\ub85d\ud558\uc600\ub2e4. 2010\ub144 11\uc6d4 20\uc77c, \uadf8\ub294 \ud234\ub8e8\uc988 FC\ub97c \uc0c1\ub300\ub85c \uacb0\uc2b9\uace8\uc744 \uae30\ub85d\ud558\uc600\ub2e4. \uadf8\ub294 \uacbd\uae30\uc885\ub8cc 2\ubd84\uc744 \ub0a8\uaca8\ub193\uace0 \uace8\uc744 \uae30\ub85d\ud558\uc600\ub2e4. 2011\ub144 4\uc6d4 27\uc77c, \uc544\uc720\ub294 OGC \ub2c8\uc2a4\uc804\uc5d0\uc11c \ud504\ub85c\uacc4\uc57d \uc774\ud6c4 \ucc98\uc74c\uc73c\ub85c \ud574\ud2b8\ud2b8\ub9ad\uc744 \ub2ec\uc131\ud558\uc600\uace0, \ub9c8\ub974\uc138\uc720\ub294 4-2 \uc2b9\ub9ac\ub97c \uac70\ub450\uc5c8\ub2e4. \uadf8\uc758 \ub0a8\ub3d9\uc0dd \uc870\ub974\ub2f9\uc740 \uc774 \uacbd\uae30\uc5d0\uc11c \ub098\uba38\uc9c0 \ud55c\uace8\uc744 \ub4dd\uc810\ud558\uc600\ub2e4. \uadf8\uc758 \uc2dc\uc98c \ub0b4\ub0b4 \uc88b\uc740 \uc131\uacfc\ub85c \uc778\ud574, \uc544\uc720\ub294 \ub9ac\uadf8 1 \uc62c\ud574\uc758 \uc2e0\uc778 \uc120\uc218\ub85c \ub9c8\ubc45 \ub9c8\ud0f1, \uc580 \uc74c\ube4c\ub77c\uc640 \ub354\ubd88\uc5b4 \uc120\uc815\ub418\uc5c8\ub2e4. \uc544\uc720\ub294 2010-11 \uc2dc\uc98c \ub9c8\ub974\uc138\uc720\uc758 \ucd5c\uace0 \uc120\uc218\ub85c \uc120\uc815\ub418\uc5c8\ub2e4.", "answer": "3\ub144", "sentence": "\uc544\uc720\ub294 \ub9c8\ub974\uc138\uc720\uc640 3\ub144 \uc5f0\uc7a5\uacc4\uc57d\uc744 \uccb4\uacb0\ud558\uc600\ub2e4.", "paragraph_sentence": "\uc544\ub97c \uc544\ube44\ub1fd\uc5d0\uc11c \uc131\uacf5\uc801\uc778 \uc2dc\uc98c\uc744 \ubcf4\ub0b8 \uc544\uc720\ub294 2010\ub144 5\uc6d4 16\uc77c\uc5d0 \ub9c8\ub974\uc138\uc720 \uac10\ub3c5 \ub514\ub514\uc5d0 \ub370\uc0f9\uc73c\ub85c\ubd80\ud130 2010-11 \uc2dc\uc98c \ubcf5\uadc0\ub97c \ud655\uc778\ubc1b\uc558\uace0 \ub354 \uc911\ub300\ud55c \ucc45\uc784\uc744 \ub9e1\uae38 \uac83\uc774\ub77c\uace0 \ubc1c\ud45c\ud558\uc600\ub2e4. 8\uc6d4 5\uc77c, \uc544\uc720\ub294 \ub9c8\ub974\uc138\uc720\uc640 3\ub144 \uc5f0\uc7a5\uacc4\uc57d\uc744 \uccb4\uacb0\ud558\uc600\ub2e4. \uc774 \uacc4\uc57d\uc73c\ub85c \uadf8\ub294 2014\ub144 6\uc6d4\uae4c\uc9c0 \ub9c8\ub974\uc138\uc720 \uc120\uc218 \uc2e0\ubd84\uc73c\ub85c \ub0a8\uac8c \ub418\uc5c8\ub2e4. \ub85c\uc774\ud06c \ub808\ubbf8\uac00 \ud300\uc5d0 \ud569\ub958\ud558\uc600\uc74c\uc5d0\ub3c4 \ubd88\uad6c\ud558\uace0, \ub370\uc0f9 \uac10\ub3c5\uc740 \uc544\uc720\ub97c \uc8fc\uc804\uc73c\ub85c \ub0b4\uc138\uc6e0\ub2e4. \ubc1c\ub791\uc2dc\uc5d4 FC\uc640\uc758 \ub9ac\uadf8 2\ub77c\uc6b4\ub4dc \uacbd\uae30\uc5d0\uc11c, \uadf8\ub294 \uc2dc\uc98c \uccab\uace8\uc744 \uae30\ub85d\ud558\uc600\uc73c\ub098, \ub9c8\ub974\uc138\uc720\ub294 2-3\uc73c\ub85c \ud328\ud558\uc600\ub2e4. 9\uc6d4 \uad6d\uac00\ub300\ud56d\uc804 \uc774\ud6c4, \uadf8\ub294 AC \uc544\ub97c \uc544\ube44\ub1fd\uc744 \uc0c1\ub300\ub85c 2\uace8\uc744 \uae30\ub85d\ud558\uc5ec 3-0 \uc2b9\ub9ac\ub97c \uacac\uc778\ud558\uc600\ub2e4. \ucfe0\ud504 \ub4dc \ub77c \ub9ac\uadf8\uc5d0\uc11c, \uc544\uc720\ub294 16\uac15 \uc0c1\ub300 EA \uac31\uac15\uacfc 8\uac15 \uc0c1\ub300 AS \ubaa8\ub098\ucf54 FC\uc640\uc758 \uacbd\uae30\uc5d0\uc11c \uac01\uac01 \uacb0\uc2b9\uace8\uc744 \uae30\ub85d\ud558\uc600\ub2e4. 2010\ub144 11\uc6d4 20\uc77c, \uadf8\ub294 \ud234\ub8e8\uc988 FC\ub97c \uc0c1\ub300\ub85c \uacb0\uc2b9\uace8\uc744 \uae30\ub85d\ud558\uc600\ub2e4. \uadf8\ub294 \uacbd\uae30\uc885\ub8cc 2\ubd84\uc744 \ub0a8\uaca8\ub193\uace0 \uace8\uc744 \uae30\ub85d\ud558\uc600\ub2e4. 2011\ub144 4\uc6d4 27\uc77c, \uc544\uc720\ub294 OGC \ub2c8\uc2a4\uc804\uc5d0\uc11c \ud504\ub85c\uacc4\uc57d \uc774\ud6c4 \ucc98\uc74c\uc73c\ub85c \ud574\ud2b8\ud2b8\ub9ad\uc744 \ub2ec\uc131\ud558\uc600\uace0, \ub9c8\ub974\uc138\uc720\ub294 4-2 \uc2b9\ub9ac\ub97c \uac70\ub450\uc5c8\ub2e4. \uadf8\uc758 \ub0a8\ub3d9\uc0dd \uc870\ub974\ub2f9\uc740 \uc774 \uacbd\uae30\uc5d0\uc11c \ub098\uba38\uc9c0 \ud55c\uace8\uc744 \ub4dd\uc810\ud558\uc600\ub2e4. \uadf8\uc758 \uc2dc\uc98c \ub0b4\ub0b4 \uc88b\uc740 \uc131\uacfc\ub85c \uc778\ud574, \uc544\uc720\ub294 \ub9ac\uadf8 1 \uc62c\ud574\uc758 \uc2e0\uc778 \uc120\uc218\ub85c \ub9c8\ubc45 \ub9c8\ud0f1, \uc580 \uc74c\ube4c\ub77c\uc640 \ub354\ubd88\uc5b4 \uc120\uc815\ub418\uc5c8\ub2e4. \uc544\uc720\ub294 2010-11 \uc2dc\uc98c \ub9c8\ub974\uc138\uc720\uc758 \ucd5c\uace0 \uc120\uc218\ub85c \uc120\uc815\ub418\uc5c8\ub2e4.", "paragraph_answer": "\uc544\ub97c \uc544\ube44\ub1fd\uc5d0\uc11c \uc131\uacf5\uc801\uc778 \uc2dc\uc98c\uc744 \ubcf4\ub0b8 \uc544\uc720\ub294 2010\ub144 5\uc6d4 16\uc77c\uc5d0 \ub9c8\ub974\uc138\uc720 \uac10\ub3c5 \ub514\ub514\uc5d0 \ub370\uc0f9\uc73c\ub85c\ubd80\ud130 2010-11 \uc2dc\uc98c \ubcf5\uadc0\ub97c \ud655\uc778\ubc1b\uc558\uace0 \ub354 \uc911\ub300\ud55c \ucc45\uc784\uc744 \ub9e1\uae38 \uac83\uc774\ub77c\uace0 \ubc1c\ud45c\ud558\uc600\ub2e4. 8\uc6d4 5\uc77c, \uc544\uc720\ub294 \ub9c8\ub974\uc138\uc720\uc640 3\ub144 \uc5f0\uc7a5\uacc4\uc57d\uc744 \uccb4\uacb0\ud558\uc600\ub2e4. \uc774 \uacc4\uc57d\uc73c\ub85c \uadf8\ub294 2014\ub144 6\uc6d4\uae4c\uc9c0 \ub9c8\ub974\uc138\uc720 \uc120\uc218 \uc2e0\ubd84\uc73c\ub85c \ub0a8\uac8c \ub418\uc5c8\ub2e4. \ub85c\uc774\ud06c \ub808\ubbf8\uac00 \ud300\uc5d0 \ud569\ub958\ud558\uc600\uc74c\uc5d0\ub3c4 \ubd88\uad6c\ud558\uace0, \ub370\uc0f9 \uac10\ub3c5\uc740 \uc544\uc720\ub97c \uc8fc\uc804\uc73c\ub85c \ub0b4\uc138\uc6e0\ub2e4. \ubc1c\ub791\uc2dc\uc5d4 FC\uc640\uc758 \ub9ac\uadf8 2\ub77c\uc6b4\ub4dc \uacbd\uae30\uc5d0\uc11c, \uadf8\ub294 \uc2dc\uc98c \uccab\uace8\uc744 \uae30\ub85d\ud558\uc600\uc73c\ub098, \ub9c8\ub974\uc138\uc720\ub294 2-3\uc73c\ub85c \ud328\ud558\uc600\ub2e4. 9\uc6d4 \uad6d\uac00\ub300\ud56d\uc804 \uc774\ud6c4, \uadf8\ub294 AC \uc544\ub97c \uc544\ube44\ub1fd\uc744 \uc0c1\ub300\ub85c 2\uace8\uc744 \uae30\ub85d\ud558\uc5ec 3-0 \uc2b9\ub9ac\ub97c \uacac\uc778\ud558\uc600\ub2e4. \ucfe0\ud504 \ub4dc \ub77c \ub9ac\uadf8\uc5d0\uc11c, \uc544\uc720\ub294 16\uac15 \uc0c1\ub300 EA \uac31\uac15\uacfc 8\uac15 \uc0c1\ub300 AS \ubaa8\ub098\ucf54 FC\uc640\uc758 \uacbd\uae30\uc5d0\uc11c \uac01\uac01 \uacb0\uc2b9\uace8\uc744 \uae30\ub85d\ud558\uc600\ub2e4. 2010\ub144 11\uc6d4 20\uc77c, \uadf8\ub294 \ud234\ub8e8\uc988 FC\ub97c \uc0c1\ub300\ub85c \uacb0\uc2b9\uace8\uc744 \uae30\ub85d\ud558\uc600\ub2e4. \uadf8\ub294 \uacbd\uae30\uc885\ub8cc 2\ubd84\uc744 \ub0a8\uaca8\ub193\uace0 \uace8\uc744 \uae30\ub85d\ud558\uc600\ub2e4. 2011\ub144 4\uc6d4 27\uc77c, \uc544\uc720\ub294 OGC \ub2c8\uc2a4\uc804\uc5d0\uc11c \ud504\ub85c\uacc4\uc57d \uc774\ud6c4 \ucc98\uc74c\uc73c\ub85c \ud574\ud2b8\ud2b8\ub9ad\uc744 \ub2ec\uc131\ud558\uc600\uace0, \ub9c8\ub974\uc138\uc720\ub294 4-2 \uc2b9\ub9ac\ub97c \uac70\ub450\uc5c8\ub2e4. \uadf8\uc758 \ub0a8\ub3d9\uc0dd \uc870\ub974\ub2f9\uc740 \uc774 \uacbd\uae30\uc5d0\uc11c \ub098\uba38\uc9c0 \ud55c\uace8\uc744 \ub4dd\uc810\ud558\uc600\ub2e4. \uadf8\uc758 \uc2dc\uc98c \ub0b4\ub0b4 \uc88b\uc740 \uc131\uacfc\ub85c \uc778\ud574, \uc544\uc720\ub294 \ub9ac\uadf8 1 \uc62c\ud574\uc758 \uc2e0\uc778 \uc120\uc218\ub85c \ub9c8\ubc45 \ub9c8\ud0f1, \uc580 \uc74c\ube4c\ub77c\uc640 \ub354\ubd88\uc5b4 \uc120\uc815\ub418\uc5c8\ub2e4. \uc544\uc720\ub294 2010-11 \uc2dc\uc98c \ub9c8\ub974\uc138\uc720\uc758 \ucd5c\uace0 \uc120\uc218\ub85c \uc120\uc815\ub418\uc5c8\ub2e4.", "sentence_answer": "\uc544\uc720\ub294 \ub9c8\ub974\uc138\uc720\uc640 3\ub144 \uc5f0\uc7a5\uacc4\uc57d\uc744 \uccb4\uacb0\ud558\uc600\ub2e4.", "paragraph_id": "6539920-4-0", "paragraph_question": "question: 2010\ub144 8\uc6d4 5\uc77c \uc559\ub4dc\ub808 \uc544\uc720\ub294 \ub9c8\ub974\uc138\uc720\uc640 \uba87 \ub144\uc758 \uc5f0\uc7a5\uacc4\uc57d\uc744 \uccb4\uacb0\ud558\uc600\ub098?, context: \uc544\ub97c \uc544\ube44\ub1fd\uc5d0\uc11c \uc131\uacf5\uc801\uc778 \uc2dc\uc98c\uc744 \ubcf4\ub0b8 \uc544\uc720\ub294 2010\ub144 5\uc6d4 16\uc77c\uc5d0 \ub9c8\ub974\uc138\uc720 \uac10\ub3c5 \ub514\ub514\uc5d0 \ub370\uc0f9\uc73c\ub85c\ubd80\ud130 2010-11 \uc2dc\uc98c \ubcf5\uadc0\ub97c \ud655\uc778\ubc1b\uc558\uace0 \ub354 \uc911\ub300\ud55c \ucc45\uc784\uc744 \ub9e1\uae38 \uac83\uc774\ub77c\uace0 \ubc1c\ud45c\ud558\uc600\ub2e4. 8\uc6d4 5\uc77c, \uc544\uc720\ub294 \ub9c8\ub974\uc138\uc720\uc640 3\ub144 \uc5f0\uc7a5\uacc4\uc57d\uc744 \uccb4\uacb0\ud558\uc600\ub2e4. \uc774 \uacc4\uc57d\uc73c\ub85c \uadf8\ub294 2014\ub144 6\uc6d4\uae4c\uc9c0 \ub9c8\ub974\uc138\uc720 \uc120\uc218 \uc2e0\ubd84\uc73c\ub85c \ub0a8\uac8c \ub418\uc5c8\ub2e4. \ub85c\uc774\ud06c \ub808\ubbf8\uac00 \ud300\uc5d0 \ud569\ub958\ud558\uc600\uc74c\uc5d0\ub3c4 \ubd88\uad6c\ud558\uace0, \ub370\uc0f9 \uac10\ub3c5\uc740 \uc544\uc720\ub97c \uc8fc\uc804\uc73c\ub85c \ub0b4\uc138\uc6e0\ub2e4. \ubc1c\ub791\uc2dc\uc5d4 FC\uc640\uc758 \ub9ac\uadf8 2\ub77c\uc6b4\ub4dc \uacbd\uae30\uc5d0\uc11c, \uadf8\ub294 \uc2dc\uc98c \uccab\uace8\uc744 \uae30\ub85d\ud558\uc600\uc73c\ub098, \ub9c8\ub974\uc138\uc720\ub294 2-3\uc73c\ub85c \ud328\ud558\uc600\ub2e4. 9\uc6d4 \uad6d\uac00\ub300\ud56d\uc804 \uc774\ud6c4, \uadf8\ub294 AC \uc544\ub97c \uc544\ube44\ub1fd\uc744 \uc0c1\ub300\ub85c 2\uace8\uc744 \uae30\ub85d\ud558\uc5ec 3-0 \uc2b9\ub9ac\ub97c \uacac\uc778\ud558\uc600\ub2e4. \ucfe0\ud504 \ub4dc \ub77c \ub9ac\uadf8\uc5d0\uc11c, \uc544\uc720\ub294 16\uac15 \uc0c1\ub300 EA \uac31\uac15\uacfc 8\uac15 \uc0c1\ub300 AS \ubaa8\ub098\ucf54 FC\uc640\uc758 \uacbd\uae30\uc5d0\uc11c \uac01\uac01 \uacb0\uc2b9\uace8\uc744 \uae30\ub85d\ud558\uc600\ub2e4. 2010\ub144 11\uc6d4 20\uc77c, \uadf8\ub294 \ud234\ub8e8\uc988 FC\ub97c \uc0c1\ub300\ub85c \uacb0\uc2b9\uace8\uc744 \uae30\ub85d\ud558\uc600\ub2e4. \uadf8\ub294 \uacbd\uae30\uc885\ub8cc 2\ubd84\uc744 \ub0a8\uaca8\ub193\uace0 \uace8\uc744 \uae30\ub85d\ud558\uc600\ub2e4. 2011\ub144 4\uc6d4 27\uc77c, \uc544\uc720\ub294 OGC \ub2c8\uc2a4\uc804\uc5d0\uc11c \ud504\ub85c\uacc4\uc57d \uc774\ud6c4 \ucc98\uc74c\uc73c\ub85c \ud574\ud2b8\ud2b8\ub9ad\uc744 \ub2ec\uc131\ud558\uc600\uace0, \ub9c8\ub974\uc138\uc720\ub294 4-2 \uc2b9\ub9ac\ub97c \uac70\ub450\uc5c8\ub2e4. \uadf8\uc758 \ub0a8\ub3d9\uc0dd \uc870\ub974\ub2f9\uc740 \uc774 \uacbd\uae30\uc5d0\uc11c \ub098\uba38\uc9c0 \ud55c\uace8\uc744 \ub4dd\uc810\ud558\uc600\ub2e4. \uadf8\uc758 \uc2dc\uc98c \ub0b4\ub0b4 \uc88b\uc740 \uc131\uacfc\ub85c \uc778\ud574, \uc544\uc720\ub294 \ub9ac\uadf8 1 \uc62c\ud574\uc758 \uc2e0\uc778 \uc120\uc218\ub85c \ub9c8\ubc45 \ub9c8\ud0f1, \uc580 \uc74c\ube4c\ub77c\uc640 \ub354\ubd88\uc5b4 \uc120\uc815\ub418\uc5c8\ub2e4. \uc544\uc720\ub294 2010-11 \uc2dc\uc98c \ub9c8\ub974\uc138\uc720\uc758 \ucd5c\uace0 \uc120\uc218\ub85c \uc120\uc815\ub418\uc5c8\ub2e4."} {"question": "\ube44\uc2a4\ub9c8\ub974\ud06c\ub97c \uacbd\uc9c8\ud55c \ub3c5\uc77c \uc81c\uad6d\uc758 \ud669\uc81c\ub294?", "paragraph": "\uadf8\ub7ec\ub098 \ube4c\ud5ec\ub984 2\uc138\ub97c \uc774\uc6a9\ud558\uc5ec \uc790\uc2e0\uc758 \uad8c\ub825\uc744 \uac15\ud654\ud558\uc5ec \uc5ed\uc73c\ub85c \ud504\ub9ac\ub4dc\ub9ac\ud788\uc640 \ube45\ud1a0\ub9ac\uc544\uc758 \uad8c\ud55c\uc744 \uc57d\ud654\uc2dc\ud0a4\ub824 \ud588\ub358 \ube44\uc2a4\ub9c8\ub974\ud06c\uc758 \uacc4\ud68d\uc740 \uacb0\uacfc\uc801\uc73c\ub85c \uadf8\uc758 \ubab0\ub77d\uc744 \uac00\uc838\uc624\uac8c \ub41c\ub2e4. \ube4c\ud5ec\ub984 2\uc138\ub294 \ubd80\ud669\uc778 \ud504\ub9ac\ub4dc\ub9ac\ud788 3\uc138\uc640 \uac19\uc774 \uc81c\uad6d\uc218\uc0c1\uc774 \ub108\ubb34 \uac15\ub825\ud55c \uad8c\ud55c\uc744 \uac00\uc9c0\uace0 \uc788\ub2e4\uace0 \uc0dd\uac01\ud588\uace0 \uadf8 \uad8c\ud55c\uc744 \ud669\uc81c\uac00 \uac00\uc838\uc640 \ub354\uc6b1 \uac15\ub825\ud55c \ud669\uc81c\uad8c\uc744 \ud589\uc0ac\ud558\uc5ec\uc57c \ud55c\ub2e4\uace0 \uc0dd\uac01\ud588\ub2e4. \ube44\uc2a4\ub9c8\ub974\ud06c\ub294 \ube4c\ud5ec\ub984 2\uc138\uac00 \uc790\uc2e0\uc744 \uacbd\uc9c8\ud560 \ub54c \uadf8\ub294 \uc774\ubbf8 \ubaa8\ub4e0 \uad8c\ud55c\uacfc \uadf8\uac00 \uc790\uc2e0\uc758 \uc9d1\uad8c\uc744 \uc704\ud574 \ubf51\uc740 \ub9ce\uc740 \uc778\uc7ac\ud480\uc744 \uc774\ubbf8 \uc783\uc5c8\ub2e4. \ube44\uc2a4\ub9c8\ub974\ud06c\ub294 \ube45\ud1a0\ub9ac\uc544 \ud669\ud0dc\ud6c4\uc5d0\uac8c \uc544\ub4e4\uc778 \ud669\uc81c\ub97c \ub300\uc2e0\ud574 \uc815\uce58\uc801 \uc601\ud5a5\ub825\uc744 \ud589\uc0ac\ud574 \uc904\uac83\uc744 \uc8fc\ubb38\ud558\uc600\uc9c0\ub9cc \ube44\uc2a4\ub9c8\ub974\ud06c\uc758 \ub2f9\uc5ec\ub4e4\uc740 \uc790\uc2e0\ub4e4\uc774 \ube4c\ud5ec\ub984 1\uc138\uc640 \ud504\ub9ac\ub4dc\ub9ac\ud788 3\uc138\uc758 \uc7ac\uc704 \uae30\uac04 \ub0b4\ub0b4 \uc5c5\uc2e0\uc5ec\uae30\ub358 \uc0ac\ub78c\ub4e4\uc5d0\uac8c\uc11c \uc555\ub825\uc744 \ubc1b\uace0 \uc788\uc5c8\uace0, \ube44\uc2a4\ub9c8\ub974\ud06c\ub294 \uacbd\uc9c8 \ub2f9\uc2dc \ubd80\ud558\ub4e4\uc758 \ub450\ud130\uc6b4 \uc2e0\ub9dd\uacfc \ucda9\uc131\uc2ec\uc744 \uc5bb\uc9c0 \ubabb\ud588\ub2e4. \uc601\uad6d\uc758 \ucd1d\ub9ac\ub97c \uc5ed\uc784\ud55c \uc81c3\ub300 \uc194\uc988\ubca0\ub9ac \ud6c4\uc791 \ub85c\ubc84\ud2b8 \uac1c\uc2a4\ucf54\uc778\uc138\uc2e4\uc740 \ube45\ud1a0\ub9ac\uc544 \uc5ec\uc655\uc5d0\uac8c \u201c\ud504\ub9ac\ub4dc\ub9ac\ud788 \ud669\uc81c\uac00 \uc989\uc704\ud560 \ub54c \ube4c\ud5ec\ub984 \ud669\ud0dc\uc790\ub97c \uc55e\uc138\uc6cc \uc2a4\uc2a4\ub85c\uc758 \uad8c\ud55c\uc744 \uac15\ud654\ud558\ub824 \ud588\ub358 \ube44\uc2a4\ub9c8\ub974\ud06c\uc758 \uc815\uce58\uc801\uc778 \uc218\uac00 \uacb0\uad6d \uadf8\uc758 \ud1f4\uc9c4\uc744 \uac00\uc838\uc624\ub294 \ub2a5\ub825\uc774 \ub418\uc5b4\ubc84\ub838\ub2e4\u201d\ub77c\uace0 \ub9d0\ud588\ub2e4. \ub3d9\uc815\uc2ec\uacfc \uc2b9\ub9ac\uac10\uc774 \ub4a4\uc11e\uc778 \uc0c1\ud0dc\uc600\ub358 \ud669\ud0dc\ud6c4\ub294 \ube44\uc2a4\ub9c8\ub974\ud06c\uc5d0\uac8c \ube44\ub85d \uc790\uc2e0\uc774 \ube4c\ud5ec\ub984 \ud669\uc81c\uc5d0\uac8c \uc601\ud5a5\ub825\uc744 \ud589\uc0ac\ud574\ub3c4, \ube44\uc2a4\ub9c8\ub974\ud06c\uac00 \uc2a4\uc2a4\ub85c \ubb34\ub108\ub728\ub9b0 \uac83\ub4e4\ub85c\ubd80\ud130 \uadf8\ub97c \uad6c\ud574\uc904 \uc218 \uc5c6\ub2e4\uace0 \ub2f5\ud588\ub2e4.", "answer": "\ube4c\ud5ec\ub984 2\uc138", "sentence": "\uadf8\ub7ec\ub098 \ube4c\ud5ec\ub984 2\uc138 \ub97c \uc774\uc6a9\ud558\uc5ec \uc790\uc2e0\uc758 \uad8c\ub825\uc744 \uac15\ud654\ud558\uc5ec \uc5ed\uc73c\ub85c \ud504\ub9ac\ub4dc\ub9ac\ud788\uc640 \ube45\ud1a0\ub9ac\uc544\uc758 \uad8c\ud55c\uc744 \uc57d\ud654\uc2dc\ud0a4\ub824 \ud588\ub358 \ube44\uc2a4\ub9c8\ub974\ud06c\uc758 \uacc4\ud68d\uc740 \uacb0\uacfc\uc801\uc73c\ub85c \uadf8\uc758 \ubab0\ub77d\uc744 \uac00\uc838\uc624\uac8c \ub41c\ub2e4.", "paragraph_sentence": " \uadf8\ub7ec\ub098 \ube4c\ud5ec\ub984 2\uc138 \ub97c \uc774\uc6a9\ud558\uc5ec \uc790\uc2e0\uc758 \uad8c\ub825\uc744 \uac15\ud654\ud558\uc5ec \uc5ed\uc73c\ub85c \ud504\ub9ac\ub4dc\ub9ac\ud788\uc640 \ube45\ud1a0\ub9ac\uc544\uc758 \uad8c\ud55c\uc744 \uc57d\ud654\uc2dc\ud0a4\ub824 \ud588\ub358 \ube44\uc2a4\ub9c8\ub974\ud06c\uc758 \uacc4\ud68d\uc740 \uacb0\uacfc\uc801\uc73c\ub85c \uadf8\uc758 \ubab0\ub77d\uc744 \uac00\uc838\uc624\uac8c \ub41c\ub2e4. \ube4c\ud5ec\ub984 2\uc138\ub294 \ubd80\ud669\uc778 \ud504\ub9ac\ub4dc\ub9ac\ud788 3\uc138\uc640 \uac19\uc774 \uc81c\uad6d\uc218\uc0c1\uc774 \ub108\ubb34 \uac15\ub825\ud55c \uad8c\ud55c\uc744 \uac00\uc9c0\uace0 \uc788\ub2e4\uace0 \uc0dd\uac01\ud588\uace0 \uadf8 \uad8c\ud55c\uc744 \ud669\uc81c\uac00 \uac00\uc838\uc640 \ub354\uc6b1 \uac15\ub825\ud55c \ud669\uc81c\uad8c\uc744 \ud589\uc0ac\ud558\uc5ec\uc57c \ud55c\ub2e4\uace0 \uc0dd\uac01\ud588\ub2e4. \ube44\uc2a4\ub9c8\ub974\ud06c\ub294 \ube4c\ud5ec\ub984 2\uc138\uac00 \uc790\uc2e0\uc744 \uacbd\uc9c8\ud560 \ub54c \uadf8\ub294 \uc774\ubbf8 \ubaa8\ub4e0 \uad8c\ud55c\uacfc \uadf8\uac00 \uc790\uc2e0\uc758 \uc9d1\uad8c\uc744 \uc704\ud574 \ubf51\uc740 \ub9ce\uc740 \uc778\uc7ac\ud480\uc744 \uc774\ubbf8 \uc783\uc5c8\ub2e4. \ube44\uc2a4\ub9c8\ub974\ud06c\ub294 \ube45\ud1a0\ub9ac\uc544 \ud669\ud0dc\ud6c4\uc5d0\uac8c \uc544\ub4e4\uc778 \ud669\uc81c\ub97c \ub300\uc2e0\ud574 \uc815\uce58\uc801 \uc601\ud5a5\ub825\uc744 \ud589\uc0ac\ud574 \uc904\uac83\uc744 \uc8fc\ubb38\ud558\uc600\uc9c0\ub9cc \ube44\uc2a4\ub9c8\ub974\ud06c\uc758 \ub2f9\uc5ec\ub4e4\uc740 \uc790\uc2e0\ub4e4\uc774 \ube4c\ud5ec\ub984 1\uc138\uc640 \ud504\ub9ac\ub4dc\ub9ac\ud788 3\uc138\uc758 \uc7ac\uc704 \uae30\uac04 \ub0b4\ub0b4 \uc5c5\uc2e0\uc5ec\uae30\ub358 \uc0ac\ub78c\ub4e4\uc5d0\uac8c\uc11c \uc555\ub825\uc744 \ubc1b\uace0 \uc788\uc5c8\uace0, \ube44\uc2a4\ub9c8\ub974\ud06c\ub294 \uacbd\uc9c8 \ub2f9\uc2dc \ubd80\ud558\ub4e4\uc758 \ub450\ud130\uc6b4 \uc2e0\ub9dd\uacfc \ucda9\uc131\uc2ec\uc744 \uc5bb\uc9c0 \ubabb\ud588\ub2e4. \uc601\uad6d\uc758 \ucd1d\ub9ac\ub97c \uc5ed\uc784\ud55c \uc81c3\ub300 \uc194\uc988\ubca0\ub9ac \ud6c4\uc791 \ub85c\ubc84\ud2b8 \uac1c\uc2a4\ucf54\uc778\uc138\uc2e4\uc740 \ube45\ud1a0\ub9ac\uc544 \uc5ec\uc655\uc5d0\uac8c \u201c\ud504\ub9ac\ub4dc\ub9ac\ud788 \ud669\uc81c\uac00 \uc989\uc704\ud560 \ub54c \ube4c\ud5ec\ub984 \ud669\ud0dc\uc790\ub97c \uc55e\uc138\uc6cc \uc2a4\uc2a4\ub85c\uc758 \uad8c\ud55c\uc744 \uac15\ud654\ud558\ub824 \ud588\ub358 \ube44\uc2a4\ub9c8\ub974\ud06c\uc758 \uc815\uce58\uc801\uc778 \uc218\uac00 \uacb0\uad6d \uadf8\uc758 \ud1f4\uc9c4\uc744 \uac00\uc838\uc624\ub294 \ub2a5\ub825\uc774 \ub418\uc5b4\ubc84\ub838\ub2e4\u201d\ub77c\uace0 \ub9d0\ud588\ub2e4. \ub3d9\uc815\uc2ec\uacfc \uc2b9\ub9ac\uac10\uc774 \ub4a4\uc11e\uc778 \uc0c1\ud0dc\uc600\ub358 \ud669\ud0dc\ud6c4\ub294 \ube44\uc2a4\ub9c8\ub974\ud06c\uc5d0\uac8c \ube44\ub85d \uc790\uc2e0\uc774 \ube4c\ud5ec\ub984 \ud669\uc81c\uc5d0\uac8c \uc601\ud5a5\ub825\uc744 \ud589\uc0ac\ud574\ub3c4, \ube44\uc2a4\ub9c8\ub974\ud06c\uac00 \uc2a4\uc2a4\ub85c \ubb34\ub108\ub728\ub9b0 \uac83\ub4e4\ub85c\ubd80\ud130 \uadf8\ub97c \uad6c\ud574\uc904 \uc218 \uc5c6\ub2e4\uace0 \ub2f5\ud588\ub2e4.", "paragraph_answer": "\uadf8\ub7ec\ub098 \ube4c\ud5ec\ub984 2\uc138 \ub97c \uc774\uc6a9\ud558\uc5ec \uc790\uc2e0\uc758 \uad8c\ub825\uc744 \uac15\ud654\ud558\uc5ec \uc5ed\uc73c\ub85c \ud504\ub9ac\ub4dc\ub9ac\ud788\uc640 \ube45\ud1a0\ub9ac\uc544\uc758 \uad8c\ud55c\uc744 \uc57d\ud654\uc2dc\ud0a4\ub824 \ud588\ub358 \ube44\uc2a4\ub9c8\ub974\ud06c\uc758 \uacc4\ud68d\uc740 \uacb0\uacfc\uc801\uc73c\ub85c \uadf8\uc758 \ubab0\ub77d\uc744 \uac00\uc838\uc624\uac8c \ub41c\ub2e4. \ube4c\ud5ec\ub984 2\uc138\ub294 \ubd80\ud669\uc778 \ud504\ub9ac\ub4dc\ub9ac\ud788 3\uc138\uc640 \uac19\uc774 \uc81c\uad6d\uc218\uc0c1\uc774 \ub108\ubb34 \uac15\ub825\ud55c \uad8c\ud55c\uc744 \uac00\uc9c0\uace0 \uc788\ub2e4\uace0 \uc0dd\uac01\ud588\uace0 \uadf8 \uad8c\ud55c\uc744 \ud669\uc81c\uac00 \uac00\uc838\uc640 \ub354\uc6b1 \uac15\ub825\ud55c \ud669\uc81c\uad8c\uc744 \ud589\uc0ac\ud558\uc5ec\uc57c \ud55c\ub2e4\uace0 \uc0dd\uac01\ud588\ub2e4. \ube44\uc2a4\ub9c8\ub974\ud06c\ub294 \ube4c\ud5ec\ub984 2\uc138\uac00 \uc790\uc2e0\uc744 \uacbd\uc9c8\ud560 \ub54c \uadf8\ub294 \uc774\ubbf8 \ubaa8\ub4e0 \uad8c\ud55c\uacfc \uadf8\uac00 \uc790\uc2e0\uc758 \uc9d1\uad8c\uc744 \uc704\ud574 \ubf51\uc740 \ub9ce\uc740 \uc778\uc7ac\ud480\uc744 \uc774\ubbf8 \uc783\uc5c8\ub2e4. \ube44\uc2a4\ub9c8\ub974\ud06c\ub294 \ube45\ud1a0\ub9ac\uc544 \ud669\ud0dc\ud6c4\uc5d0\uac8c \uc544\ub4e4\uc778 \ud669\uc81c\ub97c \ub300\uc2e0\ud574 \uc815\uce58\uc801 \uc601\ud5a5\ub825\uc744 \ud589\uc0ac\ud574 \uc904\uac83\uc744 \uc8fc\ubb38\ud558\uc600\uc9c0\ub9cc \ube44\uc2a4\ub9c8\ub974\ud06c\uc758 \ub2f9\uc5ec\ub4e4\uc740 \uc790\uc2e0\ub4e4\uc774 \ube4c\ud5ec\ub984 1\uc138\uc640 \ud504\ub9ac\ub4dc\ub9ac\ud788 3\uc138\uc758 \uc7ac\uc704 \uae30\uac04 \ub0b4\ub0b4 \uc5c5\uc2e0\uc5ec\uae30\ub358 \uc0ac\ub78c\ub4e4\uc5d0\uac8c\uc11c \uc555\ub825\uc744 \ubc1b\uace0 \uc788\uc5c8\uace0, \ube44\uc2a4\ub9c8\ub974\ud06c\ub294 \uacbd\uc9c8 \ub2f9\uc2dc \ubd80\ud558\ub4e4\uc758 \ub450\ud130\uc6b4 \uc2e0\ub9dd\uacfc \ucda9\uc131\uc2ec\uc744 \uc5bb\uc9c0 \ubabb\ud588\ub2e4. \uc601\uad6d\uc758 \ucd1d\ub9ac\ub97c \uc5ed\uc784\ud55c \uc81c3\ub300 \uc194\uc988\ubca0\ub9ac \ud6c4\uc791 \ub85c\ubc84\ud2b8 \uac1c\uc2a4\ucf54\uc778\uc138\uc2e4\uc740 \ube45\ud1a0\ub9ac\uc544 \uc5ec\uc655\uc5d0\uac8c \u201c\ud504\ub9ac\ub4dc\ub9ac\ud788 \ud669\uc81c\uac00 \uc989\uc704\ud560 \ub54c \ube4c\ud5ec\ub984 \ud669\ud0dc\uc790\ub97c \uc55e\uc138\uc6cc \uc2a4\uc2a4\ub85c\uc758 \uad8c\ud55c\uc744 \uac15\ud654\ud558\ub824 \ud588\ub358 \ube44\uc2a4\ub9c8\ub974\ud06c\uc758 \uc815\uce58\uc801\uc778 \uc218\uac00 \uacb0\uad6d \uadf8\uc758 \ud1f4\uc9c4\uc744 \uac00\uc838\uc624\ub294 \ub2a5\ub825\uc774 \ub418\uc5b4\ubc84\ub838\ub2e4\u201d\ub77c\uace0 \ub9d0\ud588\ub2e4. \ub3d9\uc815\uc2ec\uacfc \uc2b9\ub9ac\uac10\uc774 \ub4a4\uc11e\uc778 \uc0c1\ud0dc\uc600\ub358 \ud669\ud0dc\ud6c4\ub294 \ube44\uc2a4\ub9c8\ub974\ud06c\uc5d0\uac8c \ube44\ub85d \uc790\uc2e0\uc774 \ube4c\ud5ec\ub984 \ud669\uc81c\uc5d0\uac8c \uc601\ud5a5\ub825\uc744 \ud589\uc0ac\ud574\ub3c4, \ube44\uc2a4\ub9c8\ub974\ud06c\uac00 \uc2a4\uc2a4\ub85c \ubb34\ub108\ub728\ub9b0 \uac83\ub4e4\ub85c\ubd80\ud130 \uadf8\ub97c \uad6c\ud574\uc904 \uc218 \uc5c6\ub2e4\uace0 \ub2f5\ud588\ub2e4.", "sentence_answer": "\uadf8\ub7ec\ub098 \ube4c\ud5ec\ub984 2\uc138 \ub97c \uc774\uc6a9\ud558\uc5ec \uc790\uc2e0\uc758 \uad8c\ub825\uc744 \uac15\ud654\ud558\uc5ec \uc5ed\uc73c\ub85c \ud504\ub9ac\ub4dc\ub9ac\ud788\uc640 \ube45\ud1a0\ub9ac\uc544\uc758 \uad8c\ud55c\uc744 \uc57d\ud654\uc2dc\ud0a4\ub824 \ud588\ub358 \ube44\uc2a4\ub9c8\ub974\ud06c\uc758 \uacc4\ud68d\uc740 \uacb0\uacfc\uc801\uc73c\ub85c \uadf8\uc758 \ubab0\ub77d\uc744 \uac00\uc838\uc624\uac8c \ub41c\ub2e4.", "paragraph_id": "6568749-16-1", "paragraph_question": "question: \ube44\uc2a4\ub9c8\ub974\ud06c\ub97c \uacbd\uc9c8\ud55c \ub3c5\uc77c \uc81c\uad6d\uc758 \ud669\uc81c\ub294?, context: \uadf8\ub7ec\ub098 \ube4c\ud5ec\ub984 2\uc138\ub97c \uc774\uc6a9\ud558\uc5ec \uc790\uc2e0\uc758 \uad8c\ub825\uc744 \uac15\ud654\ud558\uc5ec \uc5ed\uc73c\ub85c \ud504\ub9ac\ub4dc\ub9ac\ud788\uc640 \ube45\ud1a0\ub9ac\uc544\uc758 \uad8c\ud55c\uc744 \uc57d\ud654\uc2dc\ud0a4\ub824 \ud588\ub358 \ube44\uc2a4\ub9c8\ub974\ud06c\uc758 \uacc4\ud68d\uc740 \uacb0\uacfc\uc801\uc73c\ub85c \uadf8\uc758 \ubab0\ub77d\uc744 \uac00\uc838\uc624\uac8c \ub41c\ub2e4. \ube4c\ud5ec\ub984 2\uc138\ub294 \ubd80\ud669\uc778 \ud504\ub9ac\ub4dc\ub9ac\ud788 3\uc138\uc640 \uac19\uc774 \uc81c\uad6d\uc218\uc0c1\uc774 \ub108\ubb34 \uac15\ub825\ud55c \uad8c\ud55c\uc744 \uac00\uc9c0\uace0 \uc788\ub2e4\uace0 \uc0dd\uac01\ud588\uace0 \uadf8 \uad8c\ud55c\uc744 \ud669\uc81c\uac00 \uac00\uc838\uc640 \ub354\uc6b1 \uac15\ub825\ud55c \ud669\uc81c\uad8c\uc744 \ud589\uc0ac\ud558\uc5ec\uc57c \ud55c\ub2e4\uace0 \uc0dd\uac01\ud588\ub2e4. \ube44\uc2a4\ub9c8\ub974\ud06c\ub294 \ube4c\ud5ec\ub984 2\uc138\uac00 \uc790\uc2e0\uc744 \uacbd\uc9c8\ud560 \ub54c \uadf8\ub294 \uc774\ubbf8 \ubaa8\ub4e0 \uad8c\ud55c\uacfc \uadf8\uac00 \uc790\uc2e0\uc758 \uc9d1\uad8c\uc744 \uc704\ud574 \ubf51\uc740 \ub9ce\uc740 \uc778\uc7ac\ud480\uc744 \uc774\ubbf8 \uc783\uc5c8\ub2e4. \ube44\uc2a4\ub9c8\ub974\ud06c\ub294 \ube45\ud1a0\ub9ac\uc544 \ud669\ud0dc\ud6c4\uc5d0\uac8c \uc544\ub4e4\uc778 \ud669\uc81c\ub97c \ub300\uc2e0\ud574 \uc815\uce58\uc801 \uc601\ud5a5\ub825\uc744 \ud589\uc0ac\ud574 \uc904\uac83\uc744 \uc8fc\ubb38\ud558\uc600\uc9c0\ub9cc \ube44\uc2a4\ub9c8\ub974\ud06c\uc758 \ub2f9\uc5ec\ub4e4\uc740 \uc790\uc2e0\ub4e4\uc774 \ube4c\ud5ec\ub984 1\uc138\uc640 \ud504\ub9ac\ub4dc\ub9ac\ud788 3\uc138\uc758 \uc7ac\uc704 \uae30\uac04 \ub0b4\ub0b4 \uc5c5\uc2e0\uc5ec\uae30\ub358 \uc0ac\ub78c\ub4e4\uc5d0\uac8c\uc11c \uc555\ub825\uc744 \ubc1b\uace0 \uc788\uc5c8\uace0, \ube44\uc2a4\ub9c8\ub974\ud06c\ub294 \uacbd\uc9c8 \ub2f9\uc2dc \ubd80\ud558\ub4e4\uc758 \ub450\ud130\uc6b4 \uc2e0\ub9dd\uacfc \ucda9\uc131\uc2ec\uc744 \uc5bb\uc9c0 \ubabb\ud588\ub2e4. \uc601\uad6d\uc758 \ucd1d\ub9ac\ub97c \uc5ed\uc784\ud55c \uc81c3\ub300 \uc194\uc988\ubca0\ub9ac \ud6c4\uc791 \ub85c\ubc84\ud2b8 \uac1c\uc2a4\ucf54\uc778\uc138\uc2e4\uc740 \ube45\ud1a0\ub9ac\uc544 \uc5ec\uc655\uc5d0\uac8c \u201c\ud504\ub9ac\ub4dc\ub9ac\ud788 \ud669\uc81c\uac00 \uc989\uc704\ud560 \ub54c \ube4c\ud5ec\ub984 \ud669\ud0dc\uc790\ub97c \uc55e\uc138\uc6cc \uc2a4\uc2a4\ub85c\uc758 \uad8c\ud55c\uc744 \uac15\ud654\ud558\ub824 \ud588\ub358 \ube44\uc2a4\ub9c8\ub974\ud06c\uc758 \uc815\uce58\uc801\uc778 \uc218\uac00 \uacb0\uad6d \uadf8\uc758 \ud1f4\uc9c4\uc744 \uac00\uc838\uc624\ub294 \ub2a5\ub825\uc774 \ub418\uc5b4\ubc84\ub838\ub2e4\u201d\ub77c\uace0 \ub9d0\ud588\ub2e4. \ub3d9\uc815\uc2ec\uacfc \uc2b9\ub9ac\uac10\uc774 \ub4a4\uc11e\uc778 \uc0c1\ud0dc\uc600\ub358 \ud669\ud0dc\ud6c4\ub294 \ube44\uc2a4\ub9c8\ub974\ud06c\uc5d0\uac8c \ube44\ub85d \uc790\uc2e0\uc774 \ube4c\ud5ec\ub984 \ud669\uc81c\uc5d0\uac8c \uc601\ud5a5\ub825\uc744 \ud589\uc0ac\ud574\ub3c4, \ube44\uc2a4\ub9c8\ub974\ud06c\uac00 \uc2a4\uc2a4\ub85c \ubb34\ub108\ub728\ub9b0 \uac83\ub4e4\ub85c\ubd80\ud130 \uadf8\ub97c \uad6c\ud574\uc904 \uc218 \uc5c6\ub2e4\uace0 \ub2f5\ud588\ub2e4."} {"question": "\uac80\uc0ac\uc640 \uc2dc\uc0ac\uc778 \uac04\uc758 \uc18c\uc1a1\uc5d0 \ub300\ud574 \uc11c\uc6b8\uace0\ub4f1\ubc95\uc6d0\uc740 \ub204\uad6c\uc758 \ud328\uc18c \ud310\uacb0\uc744 \ub0b4\ub838\ub098?", "paragraph": "2007\ub144 12\uc6d4, \uc2dc\uc0acIN\uc740 \uae40\uacbd\uc900\uc758 \uc790\ud544 \uba54\ubaa8\ub97c \uadfc\uac70\ub85c, \"\uae40\uc528\uac00 \uc870\uc0ac \uacfc\uc815\uc5d0\uc11c \uc218\uc0ac \uac80\uc0ac\ub85c\ubd80\ud130 '\uc774\uba85\ubc15 \ud55c\ub098\ub77c\ub2f9 (\ub2f9\uc2dc) \ud6c4\ubcf4\uc5d0\uac8c \uc720\ub9ac\ud55c \uc9c4\uc220\uc744 \ud558\uba74 \uad6c\ud615\ub7c9\uc744 3\ub144\uc73c\ub85c \ub9de\ucdb0\uc8fc\uaca0\ub2e4'\ub294 \ucde8\uc9c0\uc758 \ud68c\uc720\ub97c \ubc1b\uc558\ub2e4\uace0 \uc8fc\uc7a5\ud588\ub2e4\"\ub77c\uace0 \ubcf4\ub3c4\ud588\ub2e4. \uc774\uc5d0 2007\ub144 \ub300\uc120 \ubb34\ub835 `BBK \uc0ac\uac74'\uc744 \uc218\uc0ac\ud588\ub358 \uac80\uc0ac\ub4e4\uc740 \uc2dc\uc0acIN\uc744 \uc0c1\ub300\ub85c, \"\uc2dc\uc0acIN\uc774 \uae40\uc528\uc758 \uc8fc\uc7a5\uc744 \uc77c\ubc29\uc801\uc73c\ub85c \uc18c\uac1c\ud558\uba70 \uc758\ud639\uc744 \uc81c\uae30\ud568\uc73c\ub85c\uc368 \uc790\uc2e0\ub4e4\uc758 \uba85\uc608\uac00 \ud6fc\uc190\ub410\ub2e4\"\uba70 6\uc5b5 \uc6d0\uc758 \uc190\ud574\ubc30\uc0c1 \uccad\uad6c\uc18c\uc1a1\uc744 \uc81c\uae30\ud588\ub2e4. 1\uc2ec\uc740 \"\ud574\ub2f9 \uae30\uc0ac\ub85c \uc778\ud574 \uac80\uc0ac\ub4e4\uc758 \uba85\uc608\uac00 \ud6fc\uc190\ub410\ub2e4\"\uba70 \ucc45\uc784\uc744 \uc77c\ubd80 \uc778\uc815\ud574 3\ucc9c 600\ub9cc \uc6d0\uc744 \ubc30\uc0c1\ud558\ub3c4\ub85d \uc6d0\uace0 \uc77c\ubd80 \uc2b9\uc18c \ud310\uacb0\uc744 \ub0b4\ub838\ub2e4. 2011\ub144 4\uc6d4 21\uc77c, \uc11c\uc6b8\uace0\ub4f1\ubc95\uc6d0\uc740 \uc774 \uc190\ud574\ubc30\uc0c1 \uccad\uad6c\uc18c\uc1a1\uc5d0 \ub300\ud574 \"\ubcf4\ub3c4\ub41c \uae40\uc528 \uc790\ud544\uc758 \uba54\ubaa8\uc9c0\ub098 \ub179\uc74c\ud14c\uc774\ud504 \ub4f1\uc774 \uc0ac\ud6c4 \uc870\uc791\ub41c \uac83\uc73c\ub85c \ubcf4\uc774\uc9c0 \uc54a\ub294 \uc810 \ub4f1\uc5d0 \ube44\ucdb0\ubcfc \ub54c, \uae30\uc0ac\uc758 \ud5c8\uc704\uc131\uc774 \uc778\uc815\ub418\uc9c0 \uc54a\ub294\ub2e4\"\ub77c\uace0 \ubc1d\ud788\uba70 \uc6d0\uace0 \ud328\uc18c \ud310\uacb0\uc744 \ub0b4\ub838\ub2e4.", "answer": "\uc6d0\uace0", "sentence": "1\uc2ec\uc740 \"\ud574\ub2f9 \uae30\uc0ac\ub85c \uc778\ud574 \uac80\uc0ac\ub4e4\uc758 \uba85\uc608\uac00 \ud6fc\uc190\ub410\ub2e4\"\uba70 \ucc45\uc784\uc744 \uc77c\ubd80 \uc778\uc815\ud574 3\ucc9c 600\ub9cc \uc6d0\uc744 \ubc30\uc0c1\ud558\ub3c4\ub85d \uc6d0\uace0 \uc77c\ubd80 \uc2b9\uc18c \ud310\uacb0\uc744 \ub0b4\ub838\ub2e4.", "paragraph_sentence": "2007\ub144 12\uc6d4, \uc2dc\uc0acIN\uc740 \uae40\uacbd\uc900\uc758 \uc790\ud544 \uba54\ubaa8\ub97c \uadfc\uac70\ub85c, \"\uae40\uc528\uac00 \uc870\uc0ac \uacfc\uc815\uc5d0\uc11c \uc218\uc0ac \uac80\uc0ac\ub85c\ubd80\ud130 '\uc774\uba85\ubc15 \ud55c\ub098\ub77c\ub2f9 (\ub2f9\uc2dc) \ud6c4\ubcf4\uc5d0\uac8c \uc720\ub9ac\ud55c \uc9c4\uc220\uc744 \ud558\uba74 \uad6c\ud615\ub7c9\uc744 3\ub144\uc73c\ub85c \ub9de\ucdb0\uc8fc\uaca0\ub2e4'\ub294 \ucde8\uc9c0\uc758 \ud68c\uc720\ub97c \ubc1b\uc558\ub2e4\uace0 \uc8fc\uc7a5\ud588\ub2e4\"\ub77c\uace0 \ubcf4\ub3c4\ud588\ub2e4. \uc774\uc5d0 2007\ub144 \ub300\uc120 \ubb34\ub835 `BBK \uc0ac\uac74'\uc744 \uc218\uc0ac\ud588\ub358 \uac80\uc0ac\ub4e4\uc740 \uc2dc\uc0acIN\uc744 \uc0c1\ub300\ub85c, \"\uc2dc\uc0acIN\uc774 \uae40\uc528\uc758 \uc8fc\uc7a5\uc744 \uc77c\ubc29\uc801\uc73c\ub85c \uc18c\uac1c\ud558\uba70 \uc758\ud639\uc744 \uc81c\uae30\ud568\uc73c\ub85c\uc368 \uc790\uc2e0\ub4e4\uc758 \uba85\uc608\uac00 \ud6fc\uc190\ub410\ub2e4\"\uba70 6\uc5b5 \uc6d0\uc758 \uc190\ud574\ubc30\uc0c1 \uccad\uad6c\uc18c\uc1a1\uc744 \uc81c\uae30\ud588\ub2e4. 1\uc2ec\uc740 \"\ud574\ub2f9 \uae30\uc0ac\ub85c \uc778\ud574 \uac80\uc0ac\ub4e4\uc758 \uba85\uc608\uac00 \ud6fc\uc190\ub410\ub2e4\"\uba70 \ucc45\uc784\uc744 \uc77c\ubd80 \uc778\uc815\ud574 3\ucc9c 600\ub9cc \uc6d0\uc744 \ubc30\uc0c1\ud558\ub3c4\ub85d \uc6d0\uace0 \uc77c\ubd80 \uc2b9\uc18c \ud310\uacb0\uc744 \ub0b4\ub838\ub2e4. 2011\ub144 4\uc6d4 21\uc77c, \uc11c\uc6b8\uace0\ub4f1\ubc95\uc6d0\uc740 \uc774 \uc190\ud574\ubc30\uc0c1 \uccad\uad6c\uc18c\uc1a1\uc5d0 \ub300\ud574 \"\ubcf4\ub3c4\ub41c \uae40\uc528 \uc790\ud544\uc758 \uba54\ubaa8\uc9c0\ub098 \ub179\uc74c\ud14c\uc774\ud504 \ub4f1\uc774 \uc0ac\ud6c4 \uc870\uc791\ub41c \uac83\uc73c\ub85c \ubcf4\uc774\uc9c0 \uc54a\ub294 \uc810 \ub4f1\uc5d0 \ube44\ucdb0\ubcfc \ub54c, \uae30\uc0ac\uc758 \ud5c8\uc704\uc131\uc774 \uc778\uc815\ub418\uc9c0 \uc54a\ub294\ub2e4\"\ub77c\uace0 \ubc1d\ud788\uba70 \uc6d0\uace0 \ud328\uc18c \ud310\uacb0\uc744 \ub0b4\ub838\ub2e4.", "paragraph_answer": "2007\ub144 12\uc6d4, \uc2dc\uc0acIN\uc740 \uae40\uacbd\uc900\uc758 \uc790\ud544 \uba54\ubaa8\ub97c \uadfc\uac70\ub85c, \"\uae40\uc528\uac00 \uc870\uc0ac \uacfc\uc815\uc5d0\uc11c \uc218\uc0ac \uac80\uc0ac\ub85c\ubd80\ud130 '\uc774\uba85\ubc15 \ud55c\ub098\ub77c\ub2f9 (\ub2f9\uc2dc) \ud6c4\ubcf4\uc5d0\uac8c \uc720\ub9ac\ud55c \uc9c4\uc220\uc744 \ud558\uba74 \uad6c\ud615\ub7c9\uc744 3\ub144\uc73c\ub85c \ub9de\ucdb0\uc8fc\uaca0\ub2e4'\ub294 \ucde8\uc9c0\uc758 \ud68c\uc720\ub97c \ubc1b\uc558\ub2e4\uace0 \uc8fc\uc7a5\ud588\ub2e4\"\ub77c\uace0 \ubcf4\ub3c4\ud588\ub2e4. \uc774\uc5d0 2007\ub144 \ub300\uc120 \ubb34\ub835 `BBK \uc0ac\uac74'\uc744 \uc218\uc0ac\ud588\ub358 \uac80\uc0ac\ub4e4\uc740 \uc2dc\uc0acIN\uc744 \uc0c1\ub300\ub85c, \"\uc2dc\uc0acIN\uc774 \uae40\uc528\uc758 \uc8fc\uc7a5\uc744 \uc77c\ubc29\uc801\uc73c\ub85c \uc18c\uac1c\ud558\uba70 \uc758\ud639\uc744 \uc81c\uae30\ud568\uc73c\ub85c\uc368 \uc790\uc2e0\ub4e4\uc758 \uba85\uc608\uac00 \ud6fc\uc190\ub410\ub2e4\"\uba70 6\uc5b5 \uc6d0\uc758 \uc190\ud574\ubc30\uc0c1 \uccad\uad6c\uc18c\uc1a1\uc744 \uc81c\uae30\ud588\ub2e4. 1\uc2ec\uc740 \"\ud574\ub2f9 \uae30\uc0ac\ub85c \uc778\ud574 \uac80\uc0ac\ub4e4\uc758 \uba85\uc608\uac00 \ud6fc\uc190\ub410\ub2e4\"\uba70 \ucc45\uc784\uc744 \uc77c\ubd80 \uc778\uc815\ud574 3\ucc9c 600\ub9cc \uc6d0\uc744 \ubc30\uc0c1\ud558\ub3c4\ub85d \uc6d0\uace0 \uc77c\ubd80 \uc2b9\uc18c \ud310\uacb0\uc744 \ub0b4\ub838\ub2e4. 2011\ub144 4\uc6d4 21\uc77c, \uc11c\uc6b8\uace0\ub4f1\ubc95\uc6d0\uc740 \uc774 \uc190\ud574\ubc30\uc0c1 \uccad\uad6c\uc18c\uc1a1\uc5d0 \ub300\ud574 \"\ubcf4\ub3c4\ub41c \uae40\uc528 \uc790\ud544\uc758 \uba54\ubaa8\uc9c0\ub098 \ub179\uc74c\ud14c\uc774\ud504 \ub4f1\uc774 \uc0ac\ud6c4 \uc870\uc791\ub41c \uac83\uc73c\ub85c \ubcf4\uc774\uc9c0 \uc54a\ub294 \uc810 \ub4f1\uc5d0 \ube44\ucdb0\ubcfc \ub54c, \uae30\uc0ac\uc758 \ud5c8\uc704\uc131\uc774 \uc778\uc815\ub418\uc9c0 \uc54a\ub294\ub2e4\"\ub77c\uace0 \ubc1d\ud788\uba70 \uc6d0\uace0 \ud328\uc18c \ud310\uacb0\uc744 \ub0b4\ub838\ub2e4.", "sentence_answer": "1\uc2ec\uc740 \"\ud574\ub2f9 \uae30\uc0ac\ub85c \uc778\ud574 \uac80\uc0ac\ub4e4\uc758 \uba85\uc608\uac00 \ud6fc\uc190\ub410\ub2e4\"\uba70 \ucc45\uc784\uc744 \uc77c\ubd80 \uc778\uc815\ud574 3\ucc9c 600\ub9cc \uc6d0\uc744 \ubc30\uc0c1\ud558\ub3c4\ub85d \uc6d0\uace0 \uc77c\ubd80 \uc2b9\uc18c \ud310\uacb0\uc744 \ub0b4\ub838\ub2e4.", "paragraph_id": "6500461-3-2", "paragraph_question": "question: \uac80\uc0ac\uc640 \uc2dc\uc0ac\uc778 \uac04\uc758 \uc18c\uc1a1\uc5d0 \ub300\ud574 \uc11c\uc6b8\uace0\ub4f1\ubc95\uc6d0\uc740 \ub204\uad6c\uc758 \ud328\uc18c \ud310\uacb0\uc744 \ub0b4\ub838\ub098?, context: 2007\ub144 12\uc6d4, \uc2dc\uc0acIN\uc740 \uae40\uacbd\uc900\uc758 \uc790\ud544 \uba54\ubaa8\ub97c \uadfc\uac70\ub85c, \"\uae40\uc528\uac00 \uc870\uc0ac \uacfc\uc815\uc5d0\uc11c \uc218\uc0ac \uac80\uc0ac\ub85c\ubd80\ud130 '\uc774\uba85\ubc15 \ud55c\ub098\ub77c\ub2f9 (\ub2f9\uc2dc) \ud6c4\ubcf4\uc5d0\uac8c \uc720\ub9ac\ud55c \uc9c4\uc220\uc744 \ud558\uba74 \uad6c\ud615\ub7c9\uc744 3\ub144\uc73c\ub85c \ub9de\ucdb0\uc8fc\uaca0\ub2e4'\ub294 \ucde8\uc9c0\uc758 \ud68c\uc720\ub97c \ubc1b\uc558\ub2e4\uace0 \uc8fc\uc7a5\ud588\ub2e4\"\ub77c\uace0 \ubcf4\ub3c4\ud588\ub2e4. \uc774\uc5d0 2007\ub144 \ub300\uc120 \ubb34\ub835 `BBK \uc0ac\uac74'\uc744 \uc218\uc0ac\ud588\ub358 \uac80\uc0ac\ub4e4\uc740 \uc2dc\uc0acIN\uc744 \uc0c1\ub300\ub85c, \"\uc2dc\uc0acIN\uc774 \uae40\uc528\uc758 \uc8fc\uc7a5\uc744 \uc77c\ubc29\uc801\uc73c\ub85c \uc18c\uac1c\ud558\uba70 \uc758\ud639\uc744 \uc81c\uae30\ud568\uc73c\ub85c\uc368 \uc790\uc2e0\ub4e4\uc758 \uba85\uc608\uac00 \ud6fc\uc190\ub410\ub2e4\"\uba70 6\uc5b5 \uc6d0\uc758 \uc190\ud574\ubc30\uc0c1 \uccad\uad6c\uc18c\uc1a1\uc744 \uc81c\uae30\ud588\ub2e4. 1\uc2ec\uc740 \"\ud574\ub2f9 \uae30\uc0ac\ub85c \uc778\ud574 \uac80\uc0ac\ub4e4\uc758 \uba85\uc608\uac00 \ud6fc\uc190\ub410\ub2e4\"\uba70 \ucc45\uc784\uc744 \uc77c\ubd80 \uc778\uc815\ud574 3\ucc9c 600\ub9cc \uc6d0\uc744 \ubc30\uc0c1\ud558\ub3c4\ub85d \uc6d0\uace0 \uc77c\ubd80 \uc2b9\uc18c \ud310\uacb0\uc744 \ub0b4\ub838\ub2e4. 2011\ub144 4\uc6d4 21\uc77c, \uc11c\uc6b8\uace0\ub4f1\ubc95\uc6d0\uc740 \uc774 \uc190\ud574\ubc30\uc0c1 \uccad\uad6c\uc18c\uc1a1\uc5d0 \ub300\ud574 \"\ubcf4\ub3c4\ub41c \uae40\uc528 \uc790\ud544\uc758 \uba54\ubaa8\uc9c0\ub098 \ub179\uc74c\ud14c\uc774\ud504 \ub4f1\uc774 \uc0ac\ud6c4 \uc870\uc791\ub41c \uac83\uc73c\ub85c \ubcf4\uc774\uc9c0 \uc54a\ub294 \uc810 \ub4f1\uc5d0 \ube44\ucdb0\ubcfc \ub54c, \uae30\uc0ac\uc758 \ud5c8\uc704\uc131\uc774 \uc778\uc815\ub418\uc9c0 \uc54a\ub294\ub2e4\"\ub77c\uace0 \ubc1d\ud788\uba70 \uc6d0\uace0 \ud328\uc18c \ud310\uacb0\uc744 \ub0b4\ub838\ub2e4."} {"question": "\uc790\uae30 \ucc3d\uc870\uc758 \ub3c4\uc57d\ub3c4 \uac10\ud589\ud558\ub294 \uc0dd\uba85\uc758 \ud750\ub984\uc740 \ubb34\uc5c7\uc73c\ub85c \ubcfc \uc218 \uc788\ub294\uac00?", "paragraph": "\ub3c4\ub355\uc801 \uc758\ubb34\uc758 \ubcf8\uc131\uacfc \ubc1c\uc0dd\uc801 \uc6d0\ucc9c\uc744 \ubd84\uc11d\ud558\uc5ec \ub2eb\ud78c\ub3c4\ub355\uacfc \uc5f4\ub9b0\ub3c4\ub355\uc758 \ubcf8\uc131\uc0c1 \ucc28\uc774\ub97c \uc9c0\uc801\ud558\uace0, \uc804\uc790\uc5d0 \uc758\uc874\ud558\ub294 \ub2eb\ud78c\uc0ac\ud68c\ub85c\ubd80\ud130 \ud6c4\uc790\uc5d0 \uae30\ucd08\ud558\ub294 \uc5f4\ub9b0\uc0ac\ud68c\ub85c\uc758 \uc774\ud589\uc5d0\uc11c \uc815\uc11c\uc801 \uac10\ub3d9\uc758 \uc5ed\ud560\uc774 \uc911\uc694\ud568\uc744 \uc5ed\uc124\ud55c\ub2e4. \uc0ac\ud68c\ub294 \ud558\ub098\uc758 \uac70\ub300\ud55c \uc720\uae30\uccb4\uc640 \uc720\uc0ac\ud558\uc5ec \uc0ac\ud68c\uc758 \uc0dd\uc874\uacfc \uc9c8\uc11c\ub97c \uc720\uc9c0\ud558\uae30 \uc704\ud574\uc11c\ub294 \uc131\uc6d0\ub4e4\uc774 \ubb34\uc758\uc2dd\uc801\uc73c\ub85c \uc218\ud589\ud558\ub294 \uc2b5\uad00\ub4e4\uc758 \uccb4\uacc4\ub97c \ud544\uc694\ub85c \ud55c\ub2e4. \uc778\uac04 \uc9c0\uc131\uc758 \ubc1c\uba85\ud488\uc73c\ub85c \ubcf4\uc774\ub294 \uad00\uc2b5, \ubc95, \uaddc\uc728, \ub3c4\ub355\uc801 \uc758\ubb34\ub4e4\uc758 \ubc1c\uc0dd\uc801 \uadfc\uc6d0\uc5d0\ub294 \uc9d1\ub2e8\uc758 \uc874\uc18d\uc744 \uc720\uc9c0\ud558\uae30 \uc704\ud55c \uc790\uc5f0\uc801 \ubcf8\ub2a5\uc758 \uc694\uad6c\uac00 \ub4e4\uc5b4 \uc788\ub2e4. \uc0ac\ud68c\uc801 \uacb0\uc18d\uc744 \uc9c0\ud5a5\ud558\ub294 \ubcf8\ub2a5\uc5d0 \ub530\ub77c \uc758\ubb34\ub97c \uac15\uc81c\ud558\ub294 \uc5b5\uc555\uc758 \ub3c4\ub355\uc740 \ub2eb\ud78c\uc0ac\ud68c\ub97c \uaca8\ub0e5\ud558\uba70 \uac00\uc871\uc560\uc640 \uc560\uad6d\uc2ec\uc73c\ub85c \ub618\ub618 \ubb49\uccd0 \ub2e4\ub978 \uc0ac\ud68c\uc5d0 \ub300\ud574 \ubc30\ud0c0\uc801\uc778 \uac70\ub9ac\ub97c \ucde8\ud55c\ub2e4. \uadf8\ub7ec\ub098 \uc5f4\ub9b0\uc0ac\ud68c\uc758 \uac00\ub2a5\uc131\uc774 \uc5c6\ub294 \uac83\uc740 \uc544\ub2c8\ub2e4. \uc0ac\ud68c\uc801 \ud615\ud0dc\ub85c \ub4dc\ub7ec\ub09c \uc0dd\uba85\uc758 \ud750\ub984\uc740 \uc790\uae30 \ubcf4\uc874\uc5d0\ub9cc \ubab0\ub450\ud558\ub294 \uac83\uc774 \uc544\ub2c8\ub77c \uc790\uae30 \ucc3d\uc870\uc758 \ub3c4\uc57d\ub3c4 \uac10\ud589\ud558\uae30 \ub54c\ubb38\uc774\ub2e4.", "answer": "\uc5f4\ub9b0\uc0ac\ud68c\uc758 \uac00\ub2a5\uc131", "sentence": "\uadf8\ub7ec\ub098 \uc5f4\ub9b0\uc0ac\ud68c\uc758 \uac00\ub2a5\uc131 \uc774 \uc5c6\ub294 \uac83\uc740 \uc544\ub2c8\ub2e4.", "paragraph_sentence": "\ub3c4\ub355\uc801 \uc758\ubb34\uc758 \ubcf8\uc131\uacfc \ubc1c\uc0dd\uc801 \uc6d0\ucc9c\uc744 \ubd84\uc11d\ud558\uc5ec \ub2eb\ud78c\ub3c4\ub355\uacfc \uc5f4\ub9b0\ub3c4\ub355\uc758 \ubcf8\uc131\uc0c1 \ucc28\uc774\ub97c \uc9c0\uc801\ud558\uace0, \uc804\uc790\uc5d0 \uc758\uc874\ud558\ub294 \ub2eb\ud78c\uc0ac\ud68c\ub85c\ubd80\ud130 \ud6c4\uc790\uc5d0 \uae30\ucd08\ud558\ub294 \uc5f4\ub9b0\uc0ac\ud68c\ub85c\uc758 \uc774\ud589\uc5d0\uc11c \uc815\uc11c\uc801 \uac10\ub3d9\uc758 \uc5ed\ud560\uc774 \uc911\uc694\ud568\uc744 \uc5ed\uc124\ud55c\ub2e4. \uc0ac\ud68c\ub294 \ud558\ub098\uc758 \uac70\ub300\ud55c \uc720\uae30\uccb4\uc640 \uc720\uc0ac\ud558\uc5ec \uc0ac\ud68c\uc758 \uc0dd\uc874\uacfc \uc9c8\uc11c\ub97c \uc720\uc9c0\ud558\uae30 \uc704\ud574\uc11c\ub294 \uc131\uc6d0\ub4e4\uc774 \ubb34\uc758\uc2dd\uc801\uc73c\ub85c \uc218\ud589\ud558\ub294 \uc2b5\uad00\ub4e4\uc758 \uccb4\uacc4\ub97c \ud544\uc694\ub85c \ud55c\ub2e4. \uc778\uac04 \uc9c0\uc131\uc758 \ubc1c\uba85\ud488\uc73c\ub85c \ubcf4\uc774\ub294 \uad00\uc2b5, \ubc95, \uaddc\uc728, \ub3c4\ub355\uc801 \uc758\ubb34\ub4e4\uc758 \ubc1c\uc0dd\uc801 \uadfc\uc6d0\uc5d0\ub294 \uc9d1\ub2e8\uc758 \uc874\uc18d\uc744 \uc720\uc9c0\ud558\uae30 \uc704\ud55c \uc790\uc5f0\uc801 \ubcf8\ub2a5\uc758 \uc694\uad6c\uac00 \ub4e4\uc5b4 \uc788\ub2e4. \uc0ac\ud68c\uc801 \uacb0\uc18d\uc744 \uc9c0\ud5a5\ud558\ub294 \ubcf8\ub2a5\uc5d0 \ub530\ub77c \uc758\ubb34\ub97c \uac15\uc81c\ud558\ub294 \uc5b5\uc555\uc758 \ub3c4\ub355\uc740 \ub2eb\ud78c\uc0ac\ud68c\ub97c \uaca8\ub0e5\ud558\uba70 \uac00\uc871\uc560\uc640 \uc560\uad6d\uc2ec\uc73c\ub85c \ub618\ub618 \ubb49\uccd0 \ub2e4\ub978 \uc0ac\ud68c\uc5d0 \ub300\ud574 \ubc30\ud0c0\uc801\uc778 \uac70\ub9ac\ub97c \ucde8\ud55c\ub2e4. \uadf8\ub7ec\ub098 \uc5f4\ub9b0\uc0ac\ud68c\uc758 \uac00\ub2a5\uc131 \uc774 \uc5c6\ub294 \uac83\uc740 \uc544\ub2c8\ub2e4. \uc0ac\ud68c\uc801 \ud615\ud0dc\ub85c \ub4dc\ub7ec\ub09c \uc0dd\uba85\uc758 \ud750\ub984\uc740 \uc790\uae30 \ubcf4\uc874\uc5d0\ub9cc \ubab0\ub450\ud558\ub294 \uac83\uc774 \uc544\ub2c8\ub77c \uc790\uae30 \ucc3d\uc870\uc758 \ub3c4\uc57d\ub3c4 \uac10\ud589\ud558\uae30 \ub54c\ubb38\uc774\ub2e4.", "paragraph_answer": "\ub3c4\ub355\uc801 \uc758\ubb34\uc758 \ubcf8\uc131\uacfc \ubc1c\uc0dd\uc801 \uc6d0\ucc9c\uc744 \ubd84\uc11d\ud558\uc5ec \ub2eb\ud78c\ub3c4\ub355\uacfc \uc5f4\ub9b0\ub3c4\ub355\uc758 \ubcf8\uc131\uc0c1 \ucc28\uc774\ub97c \uc9c0\uc801\ud558\uace0, \uc804\uc790\uc5d0 \uc758\uc874\ud558\ub294 \ub2eb\ud78c\uc0ac\ud68c\ub85c\ubd80\ud130 \ud6c4\uc790\uc5d0 \uae30\ucd08\ud558\ub294 \uc5f4\ub9b0\uc0ac\ud68c\ub85c\uc758 \uc774\ud589\uc5d0\uc11c \uc815\uc11c\uc801 \uac10\ub3d9\uc758 \uc5ed\ud560\uc774 \uc911\uc694\ud568\uc744 \uc5ed\uc124\ud55c\ub2e4. \uc0ac\ud68c\ub294 \ud558\ub098\uc758 \uac70\ub300\ud55c \uc720\uae30\uccb4\uc640 \uc720\uc0ac\ud558\uc5ec \uc0ac\ud68c\uc758 \uc0dd\uc874\uacfc \uc9c8\uc11c\ub97c \uc720\uc9c0\ud558\uae30 \uc704\ud574\uc11c\ub294 \uc131\uc6d0\ub4e4\uc774 \ubb34\uc758\uc2dd\uc801\uc73c\ub85c \uc218\ud589\ud558\ub294 \uc2b5\uad00\ub4e4\uc758 \uccb4\uacc4\ub97c \ud544\uc694\ub85c \ud55c\ub2e4. \uc778\uac04 \uc9c0\uc131\uc758 \ubc1c\uba85\ud488\uc73c\ub85c \ubcf4\uc774\ub294 \uad00\uc2b5, \ubc95, \uaddc\uc728, \ub3c4\ub355\uc801 \uc758\ubb34\ub4e4\uc758 \ubc1c\uc0dd\uc801 \uadfc\uc6d0\uc5d0\ub294 \uc9d1\ub2e8\uc758 \uc874\uc18d\uc744 \uc720\uc9c0\ud558\uae30 \uc704\ud55c \uc790\uc5f0\uc801 \ubcf8\ub2a5\uc758 \uc694\uad6c\uac00 \ub4e4\uc5b4 \uc788\ub2e4. \uc0ac\ud68c\uc801 \uacb0\uc18d\uc744 \uc9c0\ud5a5\ud558\ub294 \ubcf8\ub2a5\uc5d0 \ub530\ub77c \uc758\ubb34\ub97c \uac15\uc81c\ud558\ub294 \uc5b5\uc555\uc758 \ub3c4\ub355\uc740 \ub2eb\ud78c\uc0ac\ud68c\ub97c \uaca8\ub0e5\ud558\uba70 \uac00\uc871\uc560\uc640 \uc560\uad6d\uc2ec\uc73c\ub85c \ub618\ub618 \ubb49\uccd0 \ub2e4\ub978 \uc0ac\ud68c\uc5d0 \ub300\ud574 \ubc30\ud0c0\uc801\uc778 \uac70\ub9ac\ub97c \ucde8\ud55c\ub2e4. \uadf8\ub7ec\ub098 \uc5f4\ub9b0\uc0ac\ud68c\uc758 \uac00\ub2a5\uc131 \uc774 \uc5c6\ub294 \uac83\uc740 \uc544\ub2c8\ub2e4. \uc0ac\ud68c\uc801 \ud615\ud0dc\ub85c \ub4dc\ub7ec\ub09c \uc0dd\uba85\uc758 \ud750\ub984\uc740 \uc790\uae30 \ubcf4\uc874\uc5d0\ub9cc \ubab0\ub450\ud558\ub294 \uac83\uc774 \uc544\ub2c8\ub77c \uc790\uae30 \ucc3d\uc870\uc758 \ub3c4\uc57d\ub3c4 \uac10\ud589\ud558\uae30 \ub54c\ubb38\uc774\ub2e4.", "sentence_answer": "\uadf8\ub7ec\ub098 \uc5f4\ub9b0\uc0ac\ud68c\uc758 \uac00\ub2a5\uc131 \uc774 \uc5c6\ub294 \uac83\uc740 \uc544\ub2c8\ub2e4.", "paragraph_id": "6491869-0-0", "paragraph_question": "question: \uc790\uae30 \ucc3d\uc870\uc758 \ub3c4\uc57d\ub3c4 \uac10\ud589\ud558\ub294 \uc0dd\uba85\uc758 \ud750\ub984\uc740 \ubb34\uc5c7\uc73c\ub85c \ubcfc \uc218 \uc788\ub294\uac00?, context: \ub3c4\ub355\uc801 \uc758\ubb34\uc758 \ubcf8\uc131\uacfc \ubc1c\uc0dd\uc801 \uc6d0\ucc9c\uc744 \ubd84\uc11d\ud558\uc5ec \ub2eb\ud78c\ub3c4\ub355\uacfc \uc5f4\ub9b0\ub3c4\ub355\uc758 \ubcf8\uc131\uc0c1 \ucc28\uc774\ub97c \uc9c0\uc801\ud558\uace0, \uc804\uc790\uc5d0 \uc758\uc874\ud558\ub294 \ub2eb\ud78c\uc0ac\ud68c\ub85c\ubd80\ud130 \ud6c4\uc790\uc5d0 \uae30\ucd08\ud558\ub294 \uc5f4\ub9b0\uc0ac\ud68c\ub85c\uc758 \uc774\ud589\uc5d0\uc11c \uc815\uc11c\uc801 \uac10\ub3d9\uc758 \uc5ed\ud560\uc774 \uc911\uc694\ud568\uc744 \uc5ed\uc124\ud55c\ub2e4. \uc0ac\ud68c\ub294 \ud558\ub098\uc758 \uac70\ub300\ud55c \uc720\uae30\uccb4\uc640 \uc720\uc0ac\ud558\uc5ec \uc0ac\ud68c\uc758 \uc0dd\uc874\uacfc \uc9c8\uc11c\ub97c \uc720\uc9c0\ud558\uae30 \uc704\ud574\uc11c\ub294 \uc131\uc6d0\ub4e4\uc774 \ubb34\uc758\uc2dd\uc801\uc73c\ub85c \uc218\ud589\ud558\ub294 \uc2b5\uad00\ub4e4\uc758 \uccb4\uacc4\ub97c \ud544\uc694\ub85c \ud55c\ub2e4. \uc778\uac04 \uc9c0\uc131\uc758 \ubc1c\uba85\ud488\uc73c\ub85c \ubcf4\uc774\ub294 \uad00\uc2b5, \ubc95, \uaddc\uc728, \ub3c4\ub355\uc801 \uc758\ubb34\ub4e4\uc758 \ubc1c\uc0dd\uc801 \uadfc\uc6d0\uc5d0\ub294 \uc9d1\ub2e8\uc758 \uc874\uc18d\uc744 \uc720\uc9c0\ud558\uae30 \uc704\ud55c \uc790\uc5f0\uc801 \ubcf8\ub2a5\uc758 \uc694\uad6c\uac00 \ub4e4\uc5b4 \uc788\ub2e4. \uc0ac\ud68c\uc801 \uacb0\uc18d\uc744 \uc9c0\ud5a5\ud558\ub294 \ubcf8\ub2a5\uc5d0 \ub530\ub77c \uc758\ubb34\ub97c \uac15\uc81c\ud558\ub294 \uc5b5\uc555\uc758 \ub3c4\ub355\uc740 \ub2eb\ud78c\uc0ac\ud68c\ub97c \uaca8\ub0e5\ud558\uba70 \uac00\uc871\uc560\uc640 \uc560\uad6d\uc2ec\uc73c\ub85c \ub618\ub618 \ubb49\uccd0 \ub2e4\ub978 \uc0ac\ud68c\uc5d0 \ub300\ud574 \ubc30\ud0c0\uc801\uc778 \uac70\ub9ac\ub97c \ucde8\ud55c\ub2e4. \uadf8\ub7ec\ub098 \uc5f4\ub9b0\uc0ac\ud68c\uc758 \uac00\ub2a5\uc131\uc774 \uc5c6\ub294 \uac83\uc740 \uc544\ub2c8\ub2e4. \uc0ac\ud68c\uc801 \ud615\ud0dc\ub85c \ub4dc\ub7ec\ub09c \uc0dd\uba85\uc758 \ud750\ub984\uc740 \uc790\uae30 \ubcf4\uc874\uc5d0\ub9cc \ubab0\ub450\ud558\ub294 \uac83\uc774 \uc544\ub2c8\ub77c \uc790\uae30 \ucc3d\uc870\uc758 \ub3c4\uc57d\ub3c4 \uac10\ud589\ud558\uae30 \ub54c\ubb38\uc774\ub2e4."} {"question": "\uc77c\ubcf8\uc740 \uba87 \ub144 \ub3d9\uc548 \ub300\ud55c\uc81c\uad6d\uc758 \uc2dd\ubbfc \ud1b5\uce58\ub97c \ud558\uc600\ub294\uac00?", "paragraph": "\ud55c\ubc18\ub3c4\uc640 \uc77c\ubcf8 \uc5f4\ub3c4\ub294 \uace0\ub300\ubd80\ud130 \uac01\uc885 \ubb38\ud654\uc758 \uc804\ud30c\uc640 \ubb34\uc5ed\uc744 \ube44\ub86f\ud55c \ub9ce\uc740 \uad50\ub958\ub97c \uc774\uc5b4\uc654\ub2e4. \ud2b9\ud788 \ubc31\uc81c\ub294 \uace0\ub300 \uc77c\ubcf8\uc5d0 \uc911\uad6d \ubb38\ubb3c\uc744 \ub9ce\uc774 \uc804\ud30c\ud558\uc600\ub2e4. \uace0\ub824 \uc2dc\ub300 \ud6c4\ubc18\uacfc \uc870\uc120 \uc2dc\ub300 \uc804\ubc18\uc5d0 \uac78\uccd0\uc11c\ub294 \uc4f0\uc2dc\ub9c8 \uc12c\uacfc \uaddc\uc288\ub97c \uadfc\uac70\uc9c0\ub85c \ud558\ub294 \uc65c\uad6c(\u502d\u5bc7)\uac00 \uac01\uc885 \uc57d\ud0c8\uc744 \uc77c\uc0bc\uc558\uc73c\uba70 \uc774\ub85c \uc778\ud574 \uc870\uc120 \uc2dc\ub300 \uc911\ubc18\uc5d0 \uc784\uc9c4\uc65c\ub780\uc774 \uc77c\uc5b4\ub098\ub294 \ub4f1 \uc911\uc138\uc640 \uadfc\uc138\uae30 \ud55c\uc77c \uad00\uacc4\ub294 \uae09\uc18d\ub3c4\ub85c \ub0c9\uac01\ub418\uae30\ub3c4 \ud558\uc600\uc73c\ub098, \uc784\uc9c4\uc65c\ub780 \uc774\ud6c4 \uc870\uc120\uc758 \ud1b5\uc2e0\uc0ac \ud30c\uacac\uc73c\ub85c \ub2e4\uc2dc \ud55c\uc77c \uad50\ub958\uac00 \uc7ac\uac1c\ub418\uc5c8\ub2e4. \uadf8\ub7ec\ub098 \uba54\uc774\uc9c0 \uc720\uc2e0 \uc774\ud6c4\uc5d0 \ub4f1\uc7a5\ud55c \uc77c\ubcf8 \uc81c\uad6d\uc774 1910\ub144 \ub300\ud55c\uc81c\uad6d\uc744 \ubb34\ub825\uc73c\ub85c \ubcd1\ud569\ud55c \ub4a4 1945\ub144\uae4c\uc9c0 35\ub144 \ub3d9\uc548 \ud3ed\uc545\ud55c \uc2dd\ubbfc \ud1b5\uce58\ub97c \ud589\ud558\uace0 \ud604\ub300\uc5d0\ub3c4 \uc77c\ubcf8\uc758 \uc5ec\ub7ec \uc6b0\uc775 \uc778\uc0ac\ub4e4\uc774 \ud55c\uad6d\uc5d0 \ub300\ud55c \uc815\uce58\uc801 \ub3c4\ubc1c\uc744 \uc8fc\ucc3d\ud558\uba74\uc11c \uc591\uad6d\uc758 \uad6d\ubbfc \uac10\uc815\uc774 \uadf9\ub3c4\ub85c \uc545\ud654\ub418\uc5c8\ub2e4. \uc77c\ubcf8\uc5d0\uc11c\ub3c4 \ubc18\ud55c\uac10\uc815\uc744 \uac00\uc9c4 \uc77c\ubd80 \uc77c\ubcf8\uc778\ub4e4\uc774 \ubc18\ud55c\uc2dc\uc704\ub97c \ud558\uace0 \uc788\uc73c\uba70, \ud604\uc7ac \ub300\ud55c\ubbfc\uad6d\uacfc \uc77c\ubcf8\uc740 \ub3c5\ub3c4 \ubd84\uc7c1\uacfc \ub3d9\ud574\uc758 \uc774\ub984\uc5d0 \ub300\ud55c \ubd84\uc7c1 \ub4f1\uc758 \uacfc\uc81c\ub97c \uc548\uace0 \uc788\ub2e4. 2009\ub144\uc5d0 \uc77c\ubcf8 \ubbfc\uc8fc\ub2f9 \ud558\ud1a0\uc57c\ub9c8 \uc720\ud0a4\uc624 \ub0b4\uac01\uc774 \uc131\ub9bd\ud55c \uc774\ud6c4 \uc774\ub8e8\uc5b4\uc9c4 \uc591\uad6d\ubbfc \uc124\ubb38\uc870\uc0ac\uc5d0\uc11c \uc751\ub2f5\uc790\uc758 \ub2e4\uc218\uac00 \ub354 \uc545\ud654\ub41c \ud55c\uc77c \uad00\uacc4\uc5d0 \ubcc0\ud654\uac00 \uc5c6\uc744 \uac83\uc774\ub77c \uc751\ub2f5\ud55c \uacb0\uacfc\uac00 \ub098\uc624\uae30\ub3c4 \ud558\uc600\ub2e4. \uc704\uc548\ubd80\uc5d0 \ub300\ud55c \uc5ed\uc0ac\uc65c\uace1\uc73c\ub85c \uc77c\ubcf8\uc815\ubd80\ub294 \uc720\uc5d4\uc73c\ub85c\ubd80\ud130 \uacbd\uace0\ub97c \ubc1b\uc558\ub2e4.", "answer": "35\ub144", "sentence": "\uadf8\ub7ec\ub098 \uba54\uc774\uc9c0 \uc720\uc2e0 \uc774\ud6c4\uc5d0 \ub4f1\uc7a5\ud55c \uc77c\ubcf8 \uc81c\uad6d\uc774 1910\ub144 \ub300\ud55c\uc81c\uad6d\uc744 \ubb34\ub825\uc73c\ub85c \ubcd1\ud569\ud55c \ub4a4 1945\ub144\uae4c\uc9c0 35\ub144 \ub3d9\uc548 \ud3ed\uc545\ud55c \uc2dd\ubbfc \ud1b5\uce58\ub97c \ud589\ud558\uace0 \ud604\ub300\uc5d0\ub3c4 \uc77c\ubcf8\uc758 \uc5ec\ub7ec \uc6b0\uc775 \uc778\uc0ac\ub4e4\uc774 \ud55c\uad6d\uc5d0 \ub300\ud55c \uc815\uce58\uc801 \ub3c4\ubc1c\uc744 \uc8fc\ucc3d\ud558\uba74\uc11c \uc591\uad6d\uc758 \uad6d\ubbfc \uac10\uc815\uc774 \uadf9\ub3c4\ub85c \uc545\ud654\ub418\uc5c8\ub2e4.", "paragraph_sentence": "\ud55c\ubc18\ub3c4\uc640 \uc77c\ubcf8 \uc5f4\ub3c4\ub294 \uace0\ub300\ubd80\ud130 \uac01\uc885 \ubb38\ud654\uc758 \uc804\ud30c\uc640 \ubb34\uc5ed\uc744 \ube44\ub86f\ud55c \ub9ce\uc740 \uad50\ub958\ub97c \uc774\uc5b4\uc654\ub2e4. \ud2b9\ud788 \ubc31\uc81c\ub294 \uace0\ub300 \uc77c\ubcf8\uc5d0 \uc911\uad6d \ubb38\ubb3c\uc744 \ub9ce\uc774 \uc804\ud30c\ud558\uc600\ub2e4. \uace0\ub824 \uc2dc\ub300 \ud6c4\ubc18\uacfc \uc870\uc120 \uc2dc\ub300 \uc804\ubc18\uc5d0 \uac78\uccd0\uc11c\ub294 \uc4f0\uc2dc\ub9c8 \uc12c\uacfc \uaddc\uc288\ub97c \uadfc\uac70\uc9c0\ub85c \ud558\ub294 \uc65c\uad6c(\u502d\u5bc7)\uac00 \uac01\uc885 \uc57d\ud0c8\uc744 \uc77c\uc0bc\uc558\uc73c\uba70 \uc774\ub85c \uc778\ud574 \uc870\uc120 \uc2dc\ub300 \uc911\ubc18\uc5d0 \uc784\uc9c4\uc65c\ub780\uc774 \uc77c\uc5b4\ub098\ub294 \ub4f1 \uc911\uc138\uc640 \uadfc\uc138\uae30 \ud55c\uc77c \uad00\uacc4\ub294 \uae09\uc18d\ub3c4\ub85c \ub0c9\uac01\ub418\uae30\ub3c4 \ud558\uc600\uc73c\ub098, \uc784\uc9c4\uc65c\ub780 \uc774\ud6c4 \uc870\uc120\uc758 \ud1b5\uc2e0\uc0ac \ud30c\uacac\uc73c\ub85c \ub2e4\uc2dc \ud55c\uc77c \uad50\ub958\uac00 \uc7ac\uac1c\ub418\uc5c8\ub2e4. \uadf8\ub7ec\ub098 \uba54\uc774\uc9c0 \uc720\uc2e0 \uc774\ud6c4\uc5d0 \ub4f1\uc7a5\ud55c \uc77c\ubcf8 \uc81c\uad6d\uc774 1910\ub144 \ub300\ud55c\uc81c\uad6d\uc744 \ubb34\ub825\uc73c\ub85c \ubcd1\ud569\ud55c \ub4a4 1945\ub144\uae4c\uc9c0 35\ub144 \ub3d9\uc548 \ud3ed\uc545\ud55c \uc2dd\ubbfc \ud1b5\uce58\ub97c \ud589\ud558\uace0 \ud604\ub300\uc5d0\ub3c4 \uc77c\ubcf8\uc758 \uc5ec\ub7ec \uc6b0\uc775 \uc778\uc0ac\ub4e4\uc774 \ud55c\uad6d\uc5d0 \ub300\ud55c \uc815\uce58\uc801 \ub3c4\ubc1c\uc744 \uc8fc\ucc3d\ud558\uba74\uc11c \uc591\uad6d\uc758 \uad6d\ubbfc \uac10\uc815\uc774 \uadf9\ub3c4\ub85c \uc545\ud654\ub418\uc5c8\ub2e4. \uc77c\ubcf8\uc5d0\uc11c\ub3c4 \ubc18\ud55c\uac10\uc815\uc744 \uac00\uc9c4 \uc77c\ubd80 \uc77c\ubcf8\uc778\ub4e4\uc774 \ubc18\ud55c\uc2dc\uc704\ub97c \ud558\uace0 \uc788\uc73c\uba70, \ud604\uc7ac \ub300\ud55c\ubbfc\uad6d\uacfc \uc77c\ubcf8\uc740 \ub3c5\ub3c4 \ubd84\uc7c1\uacfc \ub3d9\ud574\uc758 \uc774\ub984\uc5d0 \ub300\ud55c \ubd84\uc7c1 \ub4f1\uc758 \uacfc\uc81c\ub97c \uc548\uace0 \uc788\ub2e4. 2009\ub144\uc5d0 \uc77c\ubcf8 \ubbfc\uc8fc\ub2f9 \ud558\ud1a0\uc57c\ub9c8 \uc720\ud0a4\uc624 \ub0b4\uac01\uc774 \uc131\ub9bd\ud55c \uc774\ud6c4 \uc774\ub8e8\uc5b4\uc9c4 \uc591\uad6d\ubbfc \uc124\ubb38\uc870\uc0ac\uc5d0\uc11c \uc751\ub2f5\uc790\uc758 \ub2e4\uc218\uac00 \ub354 \uc545\ud654\ub41c \ud55c\uc77c \uad00\uacc4\uc5d0 \ubcc0\ud654\uac00 \uc5c6\uc744 \uac83\uc774\ub77c \uc751\ub2f5\ud55c \uacb0\uacfc\uac00 \ub098\uc624\uae30\ub3c4 \ud558\uc600\ub2e4. \uc704\uc548\ubd80\uc5d0 \ub300\ud55c \uc5ed\uc0ac\uc65c\uace1\uc73c\ub85c \uc77c\ubcf8\uc815\ubd80\ub294 \uc720\uc5d4\uc73c\ub85c\ubd80\ud130 \uacbd\uace0\ub97c \ubc1b\uc558\ub2e4.", "paragraph_answer": "\ud55c\ubc18\ub3c4\uc640 \uc77c\ubcf8 \uc5f4\ub3c4\ub294 \uace0\ub300\ubd80\ud130 \uac01\uc885 \ubb38\ud654\uc758 \uc804\ud30c\uc640 \ubb34\uc5ed\uc744 \ube44\ub86f\ud55c \ub9ce\uc740 \uad50\ub958\ub97c \uc774\uc5b4\uc654\ub2e4. \ud2b9\ud788 \ubc31\uc81c\ub294 \uace0\ub300 \uc77c\ubcf8\uc5d0 \uc911\uad6d \ubb38\ubb3c\uc744 \ub9ce\uc774 \uc804\ud30c\ud558\uc600\ub2e4. \uace0\ub824 \uc2dc\ub300 \ud6c4\ubc18\uacfc \uc870\uc120 \uc2dc\ub300 \uc804\ubc18\uc5d0 \uac78\uccd0\uc11c\ub294 \uc4f0\uc2dc\ub9c8 \uc12c\uacfc \uaddc\uc288\ub97c \uadfc\uac70\uc9c0\ub85c \ud558\ub294 \uc65c\uad6c(\u502d\u5bc7)\uac00 \uac01\uc885 \uc57d\ud0c8\uc744 \uc77c\uc0bc\uc558\uc73c\uba70 \uc774\ub85c \uc778\ud574 \uc870\uc120 \uc2dc\ub300 \uc911\ubc18\uc5d0 \uc784\uc9c4\uc65c\ub780\uc774 \uc77c\uc5b4\ub098\ub294 \ub4f1 \uc911\uc138\uc640 \uadfc\uc138\uae30 \ud55c\uc77c \uad00\uacc4\ub294 \uae09\uc18d\ub3c4\ub85c \ub0c9\uac01\ub418\uae30\ub3c4 \ud558\uc600\uc73c\ub098, \uc784\uc9c4\uc65c\ub780 \uc774\ud6c4 \uc870\uc120\uc758 \ud1b5\uc2e0\uc0ac \ud30c\uacac\uc73c\ub85c \ub2e4\uc2dc \ud55c\uc77c \uad50\ub958\uac00 \uc7ac\uac1c\ub418\uc5c8\ub2e4. \uadf8\ub7ec\ub098 \uba54\uc774\uc9c0 \uc720\uc2e0 \uc774\ud6c4\uc5d0 \ub4f1\uc7a5\ud55c \uc77c\ubcf8 \uc81c\uad6d\uc774 1910\ub144 \ub300\ud55c\uc81c\uad6d\uc744 \ubb34\ub825\uc73c\ub85c \ubcd1\ud569\ud55c \ub4a4 1945\ub144\uae4c\uc9c0 35\ub144 \ub3d9\uc548 \ud3ed\uc545\ud55c \uc2dd\ubbfc \ud1b5\uce58\ub97c \ud589\ud558\uace0 \ud604\ub300\uc5d0\ub3c4 \uc77c\ubcf8\uc758 \uc5ec\ub7ec \uc6b0\uc775 \uc778\uc0ac\ub4e4\uc774 \ud55c\uad6d\uc5d0 \ub300\ud55c \uc815\uce58\uc801 \ub3c4\ubc1c\uc744 \uc8fc\ucc3d\ud558\uba74\uc11c \uc591\uad6d\uc758 \uad6d\ubbfc \uac10\uc815\uc774 \uadf9\ub3c4\ub85c \uc545\ud654\ub418\uc5c8\ub2e4. \uc77c\ubcf8\uc5d0\uc11c\ub3c4 \ubc18\ud55c\uac10\uc815\uc744 \uac00\uc9c4 \uc77c\ubd80 \uc77c\ubcf8\uc778\ub4e4\uc774 \ubc18\ud55c\uc2dc\uc704\ub97c \ud558\uace0 \uc788\uc73c\uba70, \ud604\uc7ac \ub300\ud55c\ubbfc\uad6d\uacfc \uc77c\ubcf8\uc740 \ub3c5\ub3c4 \ubd84\uc7c1\uacfc \ub3d9\ud574\uc758 \uc774\ub984\uc5d0 \ub300\ud55c \ubd84\uc7c1 \ub4f1\uc758 \uacfc\uc81c\ub97c \uc548\uace0 \uc788\ub2e4. 2009\ub144\uc5d0 \uc77c\ubcf8 \ubbfc\uc8fc\ub2f9 \ud558\ud1a0\uc57c\ub9c8 \uc720\ud0a4\uc624 \ub0b4\uac01\uc774 \uc131\ub9bd\ud55c \uc774\ud6c4 \uc774\ub8e8\uc5b4\uc9c4 \uc591\uad6d\ubbfc \uc124\ubb38\uc870\uc0ac\uc5d0\uc11c \uc751\ub2f5\uc790\uc758 \ub2e4\uc218\uac00 \ub354 \uc545\ud654\ub41c \ud55c\uc77c \uad00\uacc4\uc5d0 \ubcc0\ud654\uac00 \uc5c6\uc744 \uac83\uc774\ub77c \uc751\ub2f5\ud55c \uacb0\uacfc\uac00 \ub098\uc624\uae30\ub3c4 \ud558\uc600\ub2e4. \uc704\uc548\ubd80\uc5d0 \ub300\ud55c \uc5ed\uc0ac\uc65c\uace1\uc73c\ub85c \uc77c\ubcf8\uc815\ubd80\ub294 \uc720\uc5d4\uc73c\ub85c\ubd80\ud130 \uacbd\uace0\ub97c \ubc1b\uc558\ub2e4.", "sentence_answer": "\uadf8\ub7ec\ub098 \uba54\uc774\uc9c0 \uc720\uc2e0 \uc774\ud6c4\uc5d0 \ub4f1\uc7a5\ud55c \uc77c\ubcf8 \uc81c\uad6d\uc774 1910\ub144 \ub300\ud55c\uc81c\uad6d\uc744 \ubb34\ub825\uc73c\ub85c \ubcd1\ud569\ud55c \ub4a4 1945\ub144\uae4c\uc9c0 35\ub144 \ub3d9\uc548 \ud3ed\uc545\ud55c \uc2dd\ubbfc \ud1b5\uce58\ub97c \ud589\ud558\uace0 \ud604\ub300\uc5d0\ub3c4 \uc77c\ubcf8\uc758 \uc5ec\ub7ec \uc6b0\uc775 \uc778\uc0ac\ub4e4\uc774 \ud55c\uad6d\uc5d0 \ub300\ud55c \uc815\uce58\uc801 \ub3c4\ubc1c\uc744 \uc8fc\ucc3d\ud558\uba74\uc11c \uc591\uad6d\uc758 \uad6d\ubbfc \uac10\uc815\uc774 \uadf9\ub3c4\ub85c \uc545\ud654\ub418\uc5c8\ub2e4.", "paragraph_id": "6591477-10-2", "paragraph_question": "question: \uc77c\ubcf8\uc740 \uba87 \ub144 \ub3d9\uc548 \ub300\ud55c\uc81c\uad6d\uc758 \uc2dd\ubbfc \ud1b5\uce58\ub97c \ud558\uc600\ub294\uac00?, context: \ud55c\ubc18\ub3c4\uc640 \uc77c\ubcf8 \uc5f4\ub3c4\ub294 \uace0\ub300\ubd80\ud130 \uac01\uc885 \ubb38\ud654\uc758 \uc804\ud30c\uc640 \ubb34\uc5ed\uc744 \ube44\ub86f\ud55c \ub9ce\uc740 \uad50\ub958\ub97c \uc774\uc5b4\uc654\ub2e4. \ud2b9\ud788 \ubc31\uc81c\ub294 \uace0\ub300 \uc77c\ubcf8\uc5d0 \uc911\uad6d \ubb38\ubb3c\uc744 \ub9ce\uc774 \uc804\ud30c\ud558\uc600\ub2e4. \uace0\ub824 \uc2dc\ub300 \ud6c4\ubc18\uacfc \uc870\uc120 \uc2dc\ub300 \uc804\ubc18\uc5d0 \uac78\uccd0\uc11c\ub294 \uc4f0\uc2dc\ub9c8 \uc12c\uacfc \uaddc\uc288\ub97c \uadfc\uac70\uc9c0\ub85c \ud558\ub294 \uc65c\uad6c(\u502d\u5bc7)\uac00 \uac01\uc885 \uc57d\ud0c8\uc744 \uc77c\uc0bc\uc558\uc73c\uba70 \uc774\ub85c \uc778\ud574 \uc870\uc120 \uc2dc\ub300 \uc911\ubc18\uc5d0 \uc784\uc9c4\uc65c\ub780\uc774 \uc77c\uc5b4\ub098\ub294 \ub4f1 \uc911\uc138\uc640 \uadfc\uc138\uae30 \ud55c\uc77c \uad00\uacc4\ub294 \uae09\uc18d\ub3c4\ub85c \ub0c9\uac01\ub418\uae30\ub3c4 \ud558\uc600\uc73c\ub098, \uc784\uc9c4\uc65c\ub780 \uc774\ud6c4 \uc870\uc120\uc758 \ud1b5\uc2e0\uc0ac \ud30c\uacac\uc73c\ub85c \ub2e4\uc2dc \ud55c\uc77c \uad50\ub958\uac00 \uc7ac\uac1c\ub418\uc5c8\ub2e4. \uadf8\ub7ec\ub098 \uba54\uc774\uc9c0 \uc720\uc2e0 \uc774\ud6c4\uc5d0 \ub4f1\uc7a5\ud55c \uc77c\ubcf8 \uc81c\uad6d\uc774 1910\ub144 \ub300\ud55c\uc81c\uad6d\uc744 \ubb34\ub825\uc73c\ub85c \ubcd1\ud569\ud55c \ub4a4 1945\ub144\uae4c\uc9c0 35\ub144 \ub3d9\uc548 \ud3ed\uc545\ud55c \uc2dd\ubbfc \ud1b5\uce58\ub97c \ud589\ud558\uace0 \ud604\ub300\uc5d0\ub3c4 \uc77c\ubcf8\uc758 \uc5ec\ub7ec \uc6b0\uc775 \uc778\uc0ac\ub4e4\uc774 \ud55c\uad6d\uc5d0 \ub300\ud55c \uc815\uce58\uc801 \ub3c4\ubc1c\uc744 \uc8fc\ucc3d\ud558\uba74\uc11c \uc591\uad6d\uc758 \uad6d\ubbfc \uac10\uc815\uc774 \uadf9\ub3c4\ub85c \uc545\ud654\ub418\uc5c8\ub2e4. \uc77c\ubcf8\uc5d0\uc11c\ub3c4 \ubc18\ud55c\uac10\uc815\uc744 \uac00\uc9c4 \uc77c\ubd80 \uc77c\ubcf8\uc778\ub4e4\uc774 \ubc18\ud55c\uc2dc\uc704\ub97c \ud558\uace0 \uc788\uc73c\uba70, \ud604\uc7ac \ub300\ud55c\ubbfc\uad6d\uacfc \uc77c\ubcf8\uc740 \ub3c5\ub3c4 \ubd84\uc7c1\uacfc \ub3d9\ud574\uc758 \uc774\ub984\uc5d0 \ub300\ud55c \ubd84\uc7c1 \ub4f1\uc758 \uacfc\uc81c\ub97c \uc548\uace0 \uc788\ub2e4. 2009\ub144\uc5d0 \uc77c\ubcf8 \ubbfc\uc8fc\ub2f9 \ud558\ud1a0\uc57c\ub9c8 \uc720\ud0a4\uc624 \ub0b4\uac01\uc774 \uc131\ub9bd\ud55c \uc774\ud6c4 \uc774\ub8e8\uc5b4\uc9c4 \uc591\uad6d\ubbfc \uc124\ubb38\uc870\uc0ac\uc5d0\uc11c \uc751\ub2f5\uc790\uc758 \ub2e4\uc218\uac00 \ub354 \uc545\ud654\ub41c \ud55c\uc77c \uad00\uacc4\uc5d0 \ubcc0\ud654\uac00 \uc5c6\uc744 \uac83\uc774\ub77c \uc751\ub2f5\ud55c \uacb0\uacfc\uac00 \ub098\uc624\uae30\ub3c4 \ud558\uc600\ub2e4. \uc704\uc548\ubd80\uc5d0 \ub300\ud55c \uc5ed\uc0ac\uc65c\uace1\uc73c\ub85c \uc77c\ubcf8\uc815\ubd80\ub294 \uc720\uc5d4\uc73c\ub85c\ubd80\ud130 \uacbd\uace0\ub97c \ubc1b\uc558\ub2e4."} {"question": "\ubc84\ud074\ub9ac\uc758 \ubb3c\ub9ac\ud559\uc790\ub4e4\uc774 \ud575\uc735\ud569\uc73c\ub85c \uc911\uc218\uc18c\ub97c \uc0bc\uc911\uc218\uc18c\ub85c \ubc14\uafd4\uc11c \uc5d0\ub108\uc9c0\ub97c \ub9cc\ub4dc\ub294 \ubc29\ubc95\uc744 \uc54c\uc544\ub0c8\ub294\ub370 \uc774\ub97c \ubb34\uc5c7\uc774\ub77c\uace0 \ubd88\ub800\ub294\uac00?", "paragraph": "\ud575\ubd84\uc5f4\uc744 \uc774\uc6a9\ud55c \ud3ed\ud0c4 \uc81c\uc870\uc5d0 \ub300\ud55c \uc774\ub860\uc801 \uae30\ubc18\uc774 \uc644\uc131\ub420 \ubb34\ub835, \ubc84\ud074\ub9ac\uc758 \ubb3c\ub9ac\ud559\uc790\ub4e4\uc740 \ud575\uc735\ud569\uc744 \ud1b5\ud574 \uc911\uc218\uc18c\ub97c \uc0bc\uc911\uc218\uc18c\ub85c \ubcc0\ud658\ud558\ub294 \uacfc\uc815\uc5d0\uc11c \uac15\ub825\ud55c \uc5d0\ub108\uc9c0\uac00 \ubc1c\uc0dd\ud560 \uc218 \uc788\ub2e4\ub294 \uac83\uc744 \ubc1c\uacac\ud558\uc600\ub2e4. \uadf8\ub4e4\uc740 \uc774\uac83\uc744 \"\uc288\ud37c \uc6d0\uc790 \ud3ed\ud0c4\"\uc774\ub77c \ubd88\ub800\ub294\ub370, \uc774 \uc774\ub860\uc740 \uc804\uc7c1 \uc774\ud6c4 \uc218\uc18c \ud3ed\ud0c4\uc73c\ub85c \ud604\uc2e4\ud654\ub418\uc5c8\ub2e4. \ub2f9\uc2dc\uc5d0 \ud14c\uc77c\ub7ec\ub294 \ud575\uc735\ud569 \ud3ed\ud0c4\uc744 \uacc4\uc18d\ud558\uc5ec \uae30\ud68d\ud558\uc600\uc9c0\ub9cc \ubca0\uc2a4\ub294 \uc774\ub97c \ubc88\ubc88\ud788 \uac70\uc808\ud558\uc600\uace0, \uacb0\uad6d \ud575\ubd84\uc5f4\uc744 \uc774\uc6a9\ud55c \uc6d0\uc790 \ud3ed\ud0c4\uc744 \uc81c\uc791\ud558\uae30\ub85c \uacb0\uc815\ud558\uc600\ub2e4. \ud55c\ud3b8, \ud14c\uc77c\ub7ec\ub294 \uc6d0\uc790 \ud3ed\ud0c4\uc758 \ud3ed\ubc1c\uc744 \uae30\ud3ed\uc81c\ub85c \ud558\uc5ec \uc9c8\uc18c \ud575\uc758 \ud575\uc735\ud569 \ubc18\uc751\uc744 \uc774\ub04c \uc218 \uc788\ub2e4\uace0 \uc8fc\uc7a5\ud558\uc600\ub2e4. \ubca0\uc2a4\ub294 \uadf8\ub7f0 \uc77c\uc774 \uc77c\uc5b4\ub0a0 \uc218 \uc5c6\ub2e4\uace0 \ubcf4\uc558\uace0, \"\uc790\uac00 \uc99d\uc2dd\ud558\ub294 \ud575 \uc5f0\uc1c4 \ubc18\uc751\uc740 \uc2dc\uc791\ub420 \uc218 \uc5c6\ub2e4\"\uace0 \ubcf4\uace0\ud558\uc600\ub2e4. \uc2e4\uc81c \uc218\uc18c \ud3ed\ud0c4\uc740 1952\ub144\uc5d0 \ucc98\uc74c\uc73c\ub85c \uc81c\uc791\ub418\uc5c8\ub2e4.", "answer": "\uc288\ud37c \uc6d0\uc790 \ud3ed\ud0c4", "sentence": "\uadf8\ub4e4\uc740 \uc774\uac83\uc744 \" \uc288\ud37c \uc6d0\uc790 \ud3ed\ud0c4 \"\uc774\ub77c \ubd88\ub800\ub294\ub370, \uc774 \uc774\ub860\uc740 \uc804\uc7c1 \uc774\ud6c4 \uc218\uc18c \ud3ed\ud0c4\uc73c\ub85c \ud604\uc2e4\ud654\ub418\uc5c8\ub2e4.", "paragraph_sentence": "\ud575\ubd84\uc5f4\uc744 \uc774\uc6a9\ud55c \ud3ed\ud0c4 \uc81c\uc870\uc5d0 \ub300\ud55c \uc774\ub860\uc801 \uae30\ubc18\uc774 \uc644\uc131\ub420 \ubb34\ub835, \ubc84\ud074\ub9ac\uc758 \ubb3c\ub9ac\ud559\uc790\ub4e4\uc740 \ud575\uc735\ud569\uc744 \ud1b5\ud574 \uc911\uc218\uc18c\ub97c \uc0bc\uc911\uc218\uc18c\ub85c \ubcc0\ud658\ud558\ub294 \uacfc\uc815\uc5d0\uc11c \uac15\ub825\ud55c \uc5d0\ub108\uc9c0\uac00 \ubc1c\uc0dd\ud560 \uc218 \uc788\ub2e4\ub294 \uac83\uc744 \ubc1c\uacac\ud558\uc600\ub2e4. \uadf8\ub4e4\uc740 \uc774\uac83\uc744 \" \uc288\ud37c \uc6d0\uc790 \ud3ed\ud0c4 \"\uc774\ub77c \ubd88\ub800\ub294\ub370, \uc774 \uc774\ub860\uc740 \uc804\uc7c1 \uc774\ud6c4 \uc218\uc18c \ud3ed\ud0c4\uc73c\ub85c \ud604\uc2e4\ud654\ub418\uc5c8\ub2e4. \ub2f9\uc2dc\uc5d0 \ud14c\uc77c\ub7ec\ub294 \ud575\uc735\ud569 \ud3ed\ud0c4\uc744 \uacc4\uc18d\ud558\uc5ec \uae30\ud68d\ud558\uc600\uc9c0\ub9cc \ubca0\uc2a4\ub294 \uc774\ub97c \ubc88\ubc88\ud788 \uac70\uc808\ud558\uc600\uace0, \uacb0\uad6d \ud575\ubd84\uc5f4\uc744 \uc774\uc6a9\ud55c \uc6d0\uc790 \ud3ed\ud0c4\uc744 \uc81c\uc791\ud558\uae30\ub85c \uacb0\uc815\ud558\uc600\ub2e4. \ud55c\ud3b8, \ud14c\uc77c\ub7ec\ub294 \uc6d0\uc790 \ud3ed\ud0c4\uc758 \ud3ed\ubc1c\uc744 \uae30\ud3ed\uc81c\ub85c \ud558\uc5ec \uc9c8\uc18c \ud575\uc758 \ud575\uc735\ud569 \ubc18\uc751\uc744 \uc774\ub04c \uc218 \uc788\ub2e4\uace0 \uc8fc\uc7a5\ud558\uc600\ub2e4. \ubca0\uc2a4\ub294 \uadf8\ub7f0 \uc77c\uc774 \uc77c\uc5b4\ub0a0 \uc218 \uc5c6\ub2e4\uace0 \ubcf4\uc558\uace0, \"\uc790\uac00 \uc99d\uc2dd\ud558\ub294 \ud575 \uc5f0\uc1c4 \ubc18\uc751\uc740 \uc2dc\uc791\ub420 \uc218 \uc5c6\ub2e4\"\uace0 \ubcf4\uace0\ud558\uc600\ub2e4. \uc2e4\uc81c \uc218\uc18c \ud3ed\ud0c4\uc740 1952\ub144\uc5d0 \ucc98\uc74c\uc73c\ub85c \uc81c\uc791\ub418\uc5c8\ub2e4.", "paragraph_answer": "\ud575\ubd84\uc5f4\uc744 \uc774\uc6a9\ud55c \ud3ed\ud0c4 \uc81c\uc870\uc5d0 \ub300\ud55c \uc774\ub860\uc801 \uae30\ubc18\uc774 \uc644\uc131\ub420 \ubb34\ub835, \ubc84\ud074\ub9ac\uc758 \ubb3c\ub9ac\ud559\uc790\ub4e4\uc740 \ud575\uc735\ud569\uc744 \ud1b5\ud574 \uc911\uc218\uc18c\ub97c \uc0bc\uc911\uc218\uc18c\ub85c \ubcc0\ud658\ud558\ub294 \uacfc\uc815\uc5d0\uc11c \uac15\ub825\ud55c \uc5d0\ub108\uc9c0\uac00 \ubc1c\uc0dd\ud560 \uc218 \uc788\ub2e4\ub294 \uac83\uc744 \ubc1c\uacac\ud558\uc600\ub2e4. \uadf8\ub4e4\uc740 \uc774\uac83\uc744 \" \uc288\ud37c \uc6d0\uc790 \ud3ed\ud0c4 \"\uc774\ub77c \ubd88\ub800\ub294\ub370, \uc774 \uc774\ub860\uc740 \uc804\uc7c1 \uc774\ud6c4 \uc218\uc18c \ud3ed\ud0c4\uc73c\ub85c \ud604\uc2e4\ud654\ub418\uc5c8\ub2e4. \ub2f9\uc2dc\uc5d0 \ud14c\uc77c\ub7ec\ub294 \ud575\uc735\ud569 \ud3ed\ud0c4\uc744 \uacc4\uc18d\ud558\uc5ec \uae30\ud68d\ud558\uc600\uc9c0\ub9cc \ubca0\uc2a4\ub294 \uc774\ub97c \ubc88\ubc88\ud788 \uac70\uc808\ud558\uc600\uace0, \uacb0\uad6d \ud575\ubd84\uc5f4\uc744 \uc774\uc6a9\ud55c \uc6d0\uc790 \ud3ed\ud0c4\uc744 \uc81c\uc791\ud558\uae30\ub85c \uacb0\uc815\ud558\uc600\ub2e4. \ud55c\ud3b8, \ud14c\uc77c\ub7ec\ub294 \uc6d0\uc790 \ud3ed\ud0c4\uc758 \ud3ed\ubc1c\uc744 \uae30\ud3ed\uc81c\ub85c \ud558\uc5ec \uc9c8\uc18c \ud575\uc758 \ud575\uc735\ud569 \ubc18\uc751\uc744 \uc774\ub04c \uc218 \uc788\ub2e4\uace0 \uc8fc\uc7a5\ud558\uc600\ub2e4. \ubca0\uc2a4\ub294 \uadf8\ub7f0 \uc77c\uc774 \uc77c\uc5b4\ub0a0 \uc218 \uc5c6\ub2e4\uace0 \ubcf4\uc558\uace0, \"\uc790\uac00 \uc99d\uc2dd\ud558\ub294 \ud575 \uc5f0\uc1c4 \ubc18\uc751\uc740 \uc2dc\uc791\ub420 \uc218 \uc5c6\ub2e4\"\uace0 \ubcf4\uace0\ud558\uc600\ub2e4. \uc2e4\uc81c \uc218\uc18c \ud3ed\ud0c4\uc740 1952\ub144\uc5d0 \ucc98\uc74c\uc73c\ub85c \uc81c\uc791\ub418\uc5c8\ub2e4.", "sentence_answer": "\uadf8\ub4e4\uc740 \uc774\uac83\uc744 \" \uc288\ud37c \uc6d0\uc790 \ud3ed\ud0c4 \"\uc774\ub77c \ubd88\ub800\ub294\ub370, \uc774 \uc774\ub860\uc740 \uc804\uc7c1 \uc774\ud6c4 \uc218\uc18c \ud3ed\ud0c4\uc73c\ub85c \ud604\uc2e4\ud654\ub418\uc5c8\ub2e4.", "paragraph_id": "6502200-7-1", "paragraph_question": "question: \ubc84\ud074\ub9ac\uc758 \ubb3c\ub9ac\ud559\uc790\ub4e4\uc774 \ud575\uc735\ud569\uc73c\ub85c \uc911\uc218\uc18c\ub97c \uc0bc\uc911\uc218\uc18c\ub85c \ubc14\uafd4\uc11c \uc5d0\ub108\uc9c0\ub97c \ub9cc\ub4dc\ub294 \ubc29\ubc95\uc744 \uc54c\uc544\ub0c8\ub294\ub370 \uc774\ub97c \ubb34\uc5c7\uc774\ub77c\uace0 \ubd88\ub800\ub294\uac00?, context: \ud575\ubd84\uc5f4\uc744 \uc774\uc6a9\ud55c \ud3ed\ud0c4 \uc81c\uc870\uc5d0 \ub300\ud55c \uc774\ub860\uc801 \uae30\ubc18\uc774 \uc644\uc131\ub420 \ubb34\ub835, \ubc84\ud074\ub9ac\uc758 \ubb3c\ub9ac\ud559\uc790\ub4e4\uc740 \ud575\uc735\ud569\uc744 \ud1b5\ud574 \uc911\uc218\uc18c\ub97c \uc0bc\uc911\uc218\uc18c\ub85c \ubcc0\ud658\ud558\ub294 \uacfc\uc815\uc5d0\uc11c \uac15\ub825\ud55c \uc5d0\ub108\uc9c0\uac00 \ubc1c\uc0dd\ud560 \uc218 \uc788\ub2e4\ub294 \uac83\uc744 \ubc1c\uacac\ud558\uc600\ub2e4. \uadf8\ub4e4\uc740 \uc774\uac83\uc744 \"\uc288\ud37c \uc6d0\uc790 \ud3ed\ud0c4\"\uc774\ub77c \ubd88\ub800\ub294\ub370, \uc774 \uc774\ub860\uc740 \uc804\uc7c1 \uc774\ud6c4 \uc218\uc18c \ud3ed\ud0c4\uc73c\ub85c \ud604\uc2e4\ud654\ub418\uc5c8\ub2e4. \ub2f9\uc2dc\uc5d0 \ud14c\uc77c\ub7ec\ub294 \ud575\uc735\ud569 \ud3ed\ud0c4\uc744 \uacc4\uc18d\ud558\uc5ec \uae30\ud68d\ud558\uc600\uc9c0\ub9cc \ubca0\uc2a4\ub294 \uc774\ub97c \ubc88\ubc88\ud788 \uac70\uc808\ud558\uc600\uace0, \uacb0\uad6d \ud575\ubd84\uc5f4\uc744 \uc774\uc6a9\ud55c \uc6d0\uc790 \ud3ed\ud0c4\uc744 \uc81c\uc791\ud558\uae30\ub85c \uacb0\uc815\ud558\uc600\ub2e4. \ud55c\ud3b8, \ud14c\uc77c\ub7ec\ub294 \uc6d0\uc790 \ud3ed\ud0c4\uc758 \ud3ed\ubc1c\uc744 \uae30\ud3ed\uc81c\ub85c \ud558\uc5ec \uc9c8\uc18c \ud575\uc758 \ud575\uc735\ud569 \ubc18\uc751\uc744 \uc774\ub04c \uc218 \uc788\ub2e4\uace0 \uc8fc\uc7a5\ud558\uc600\ub2e4. \ubca0\uc2a4\ub294 \uadf8\ub7f0 \uc77c\uc774 \uc77c\uc5b4\ub0a0 \uc218 \uc5c6\ub2e4\uace0 \ubcf4\uc558\uace0, \"\uc790\uac00 \uc99d\uc2dd\ud558\ub294 \ud575 \uc5f0\uc1c4 \ubc18\uc751\uc740 \uc2dc\uc791\ub420 \uc218 \uc5c6\ub2e4\"\uace0 \ubcf4\uace0\ud558\uc600\ub2e4. \uc2e4\uc81c \uc218\uc18c \ud3ed\ud0c4\uc740 1952\ub144\uc5d0 \ucc98\uc74c\uc73c\ub85c \uc81c\uc791\ub418\uc5c8\ub2e4."} {"question": "\ubd81\uc870\ub294 \ub2f9\uc2dc\uc5d0 \uc5b4\ub514 \ubb38\ud654\uc758 \uc601\ud5a5\uc744 \ubc1b\uc558\ub294\uac00?", "paragraph": "\ub0a8\ubd81\uc870(\u5357\u5317\u671d) \uc2dc\ub300\uc5d0\ub294 \ubd81\ubc29\ubbfc\uc871\uc774 \ud654\ubd81(\u83ef\u5317)\uc744 \uc81c\uc555\ud558\uace0 \ud55c\uc871\uc740 \uc591\ucbd4\uac15 \ub0a8\ucabd\uc73c\ub85c \uc774\ub3d9\ud558\uc5ec \ubaa8\ub450 \uc5ec\ub7ec \uc870(\u671d)\ub97c \uacaa\uc5c8\ub294\ub370, \uadf8\uac04 \ub0a8\uc870\uc758 \ub098\ub77c\ub4e4\uc740 \uc8fc\ub85c \uc911\uad6d \uace0\uc720\uc758 \uc544\uc545(\u96c5\u6a02) \ubc0f \uc18d\uc545(\u4fd7\u6a02, \u6df8\u5546\u4e09\u8abf \ub4f1)\uc744 \uc804\ud558\uace0, \ubd81\uc870\ub294 \ub2f9\uc2dc\uc758 \uc778\ub3c4 \ubc0f \uc774\ub780\uc758 \ubb38\ud654\uc758 \uc601\ud5a5\uc744 \ubc1b\uc544 \ud06c\uac8c \ubc1c\ub2ec\ud55c \uc11c\uc5ed(\u897f\u57df) \uc5ec\ub7ec \ub098\ub77c\uc758 \uc545\ubb34(\u6a02\u821e)\ub97c \uc218\uc785\ud558\uc5ec \uc774\uac83\uc744 \uc911\uad6d \uace0\uc720\uc758 \uc74c\uc545\uc5d0 \uc735\ud569\ud558\uae30\ub97c \ud798\uc37c\ub2e4. \ud6c4\uc5d0 \ubd81\uc870\uc5d0\uc11c \uc77c\uc5b4\ub098 \uc911\uad6d\uc744 \ud1b5\uc77c\ud55c \uc218\ub098\ub77c(581-619)\ub294 \uc774\ub4e4 \uc11c\uc5ed\uc758 \uc74c\uc545\uacfc \uc911\uad6d \uace0\uc720\uc758 \uc74c\uc545\uc744 \ud569\uccd0\uc11c, \ub0b4\uc6a9\uacfc \uc6d0\uc0b0\uc9c0\uc5d0 \ub530\ub77c \uce60\ubd80\uae30(\u4e03\u90e8\u4f0e)\ub85c \ub098\ub204\uc5b4 \uad81\uc911\uc758 \ud5a5\uc5f0\uc545(\u97ff\u5bb4\u6a02)\uc744 \uc0bc\uc558\ub2e4. \ud6c4\uc5d0 \uc774 \uce60\ubd80\uae30\uc5d0 \uc18c\ub975(\u758e\u52d2, \uce74\uc2dc\uac00\ub974)\uacfc \uac15\uad6d(\u5eb7\u570b, \uc0ac\ub9c8\ub974\uce78\ub4dc)\uc758 \uae30\ub97c \ub354\ud558\uc5ec 9\ubd80\uae30(\u4e5d\u90e8\u4f0e)\uac00 \ub428\uc73c\ub85c\uc368 \uadf8 \uad6d\uc81c\uc801 \uc131\uaca9\uc774 \ub354\uc6b1 \uac15\ud558\uac8c \ub418\uc5c8\ub2e4. \uc774\ub9ac\ud558\uc5ec \uc885\ubb18(\u5b97\u5edf)\uc758 \uc81c\uc545(\u796d\u6a02)\uc778 \uc544\uc545\uacfc \ud568\uaed8 \ud5a5\uc5f0\uc545\uc73c\ub85c\uc11c\uc758 9\ubd80\uae30\uac00 \uc735\uc131\ud558\uc600\ub2e4.", "answer": "\uc778\ub3c4 \ubc0f \uc774\ub780\uc758 \ubb38\ud654", "sentence": "\ub2f9\uc2dc\uc758 \uc778\ub3c4 \ubc0f \uc774\ub780\uc758 \ubb38\ud654 \uc758 \uc601\ud5a5\uc744 \ubc1b\uc544 \ud06c\uac8c \ubc1c\ub2ec\ud55c \uc11c\uc5ed(\u897f\u57df) \uc5ec\ub7ec \ub098\ub77c\uc758 \uc545\ubb34(\u6a02\u821e)\ub97c \uc218\uc785\ud558\uc5ec \uc774\uac83\uc744 \uc911\uad6d \uace0\uc720\uc758 \uc74c\uc545\uc5d0 \uc735\ud569\ud558\uae30\ub97c \ud798\uc37c\ub2e4.", "paragraph_sentence": "\ub0a8\ubd81\uc870(\u5357\u5317\u671d) \uc2dc\ub300\uc5d0\ub294 \ubd81\ubc29\ubbfc\uc871\uc774 \ud654\ubd81(\u83ef\u5317)\uc744 \uc81c\uc555\ud558\uace0 \ud55c\uc871\uc740 \uc591\ucbd4\uac15 \ub0a8\ucabd\uc73c\ub85c \uc774\ub3d9\ud558\uc5ec \ubaa8\ub450 \uc5ec\ub7ec \uc870(\u671d)\ub97c \uacaa\uc5c8\ub294\ub370, \uadf8\uac04 \ub0a8\uc870\uc758 \ub098\ub77c\ub4e4\uc740 \uc8fc\ub85c \uc911\uad6d \uace0\uc720\uc758 \uc544\uc545(\u96c5\u6a02) \ubc0f \uc18d\uc545(\u4fd7\u6a02, \u6df8\u5546\u4e09\u8abf \ub4f1)\uc744 \uc804\ud558\uace0, \ubd81\uc870\ub294 \ub2f9\uc2dc\uc758 \uc778\ub3c4 \ubc0f \uc774\ub780\uc758 \ubb38\ud654 \uc758 \uc601\ud5a5\uc744 \ubc1b\uc544 \ud06c\uac8c \ubc1c\ub2ec\ud55c \uc11c\uc5ed(\u897f\u57df) \uc5ec\ub7ec \ub098\ub77c\uc758 \uc545\ubb34(\u6a02\u821e)\ub97c \uc218\uc785\ud558\uc5ec \uc774\uac83\uc744 \uc911\uad6d \uace0\uc720\uc758 \uc74c\uc545\uc5d0 \uc735\ud569\ud558\uae30\ub97c \ud798\uc37c\ub2e4. \ud6c4\uc5d0 \ubd81\uc870\uc5d0\uc11c \uc77c\uc5b4\ub098 \uc911\uad6d\uc744 \ud1b5\uc77c\ud55c \uc218\ub098\ub77c(581-619)\ub294 \uc774\ub4e4 \uc11c\uc5ed\uc758 \uc74c\uc545\uacfc \uc911\uad6d \uace0\uc720\uc758 \uc74c\uc545\uc744 \ud569\uccd0\uc11c, \ub0b4\uc6a9\uacfc \uc6d0\uc0b0\uc9c0\uc5d0 \ub530\ub77c \uce60\ubd80\uae30(\u4e03\u90e8\u4f0e)\ub85c \ub098\ub204\uc5b4 \uad81\uc911\uc758 \ud5a5\uc5f0\uc545(\u97ff\u5bb4\u6a02)\uc744 \uc0bc\uc558\ub2e4. \ud6c4\uc5d0 \uc774 \uce60\ubd80\uae30\uc5d0 \uc18c\ub975(\u758e\u52d2, \uce74\uc2dc\uac00\ub974)\uacfc \uac15\uad6d(\u5eb7\u570b, \uc0ac\ub9c8\ub974\uce78\ub4dc)\uc758 \uae30\ub97c \ub354\ud558\uc5ec 9\ubd80\uae30(\u4e5d\u90e8\u4f0e)\uac00 \ub428\uc73c\ub85c\uc368 \uadf8 \uad6d\uc81c\uc801 \uc131\uaca9\uc774 \ub354\uc6b1 \uac15\ud558\uac8c \ub418\uc5c8\ub2e4. \uc774\ub9ac\ud558\uc5ec \uc885\ubb18(\u5b97\u5edf)\uc758 \uc81c\uc545(\u796d\u6a02)\uc778 \uc544\uc545\uacfc \ud568\uaed8 \ud5a5\uc5f0\uc545\uc73c\ub85c\uc11c\uc758 9\ubd80\uae30\uac00 \uc735\uc131\ud558\uc600\ub2e4.", "paragraph_answer": "\ub0a8\ubd81\uc870(\u5357\u5317\u671d) \uc2dc\ub300\uc5d0\ub294 \ubd81\ubc29\ubbfc\uc871\uc774 \ud654\ubd81(\u83ef\u5317)\uc744 \uc81c\uc555\ud558\uace0 \ud55c\uc871\uc740 \uc591\ucbd4\uac15 \ub0a8\ucabd\uc73c\ub85c \uc774\ub3d9\ud558\uc5ec \ubaa8\ub450 \uc5ec\ub7ec \uc870(\u671d)\ub97c \uacaa\uc5c8\ub294\ub370, \uadf8\uac04 \ub0a8\uc870\uc758 \ub098\ub77c\ub4e4\uc740 \uc8fc\ub85c \uc911\uad6d \uace0\uc720\uc758 \uc544\uc545(\u96c5\u6a02) \ubc0f \uc18d\uc545(\u4fd7\u6a02, \u6df8\u5546\u4e09\u8abf \ub4f1)\uc744 \uc804\ud558\uace0, \ubd81\uc870\ub294 \ub2f9\uc2dc\uc758 \uc778\ub3c4 \ubc0f \uc774\ub780\uc758 \ubb38\ud654 \uc758 \uc601\ud5a5\uc744 \ubc1b\uc544 \ud06c\uac8c \ubc1c\ub2ec\ud55c \uc11c\uc5ed(\u897f\u57df) \uc5ec\ub7ec \ub098\ub77c\uc758 \uc545\ubb34(\u6a02\u821e)\ub97c \uc218\uc785\ud558\uc5ec \uc774\uac83\uc744 \uc911\uad6d \uace0\uc720\uc758 \uc74c\uc545\uc5d0 \uc735\ud569\ud558\uae30\ub97c \ud798\uc37c\ub2e4. \ud6c4\uc5d0 \ubd81\uc870\uc5d0\uc11c \uc77c\uc5b4\ub098 \uc911\uad6d\uc744 \ud1b5\uc77c\ud55c \uc218\ub098\ub77c(581-619)\ub294 \uc774\ub4e4 \uc11c\uc5ed\uc758 \uc74c\uc545\uacfc \uc911\uad6d \uace0\uc720\uc758 \uc74c\uc545\uc744 \ud569\uccd0\uc11c, \ub0b4\uc6a9\uacfc \uc6d0\uc0b0\uc9c0\uc5d0 \ub530\ub77c \uce60\ubd80\uae30(\u4e03\u90e8\u4f0e)\ub85c \ub098\ub204\uc5b4 \uad81\uc911\uc758 \ud5a5\uc5f0\uc545(\u97ff\u5bb4\u6a02)\uc744 \uc0bc\uc558\ub2e4. \ud6c4\uc5d0 \uc774 \uce60\ubd80\uae30\uc5d0 \uc18c\ub975(\u758e\u52d2, \uce74\uc2dc\uac00\ub974)\uacfc \uac15\uad6d(\u5eb7\u570b, \uc0ac\ub9c8\ub974\uce78\ub4dc)\uc758 \uae30\ub97c \ub354\ud558\uc5ec 9\ubd80\uae30(\u4e5d\u90e8\u4f0e)\uac00 \ub428\uc73c\ub85c\uc368 \uadf8 \uad6d\uc81c\uc801 \uc131\uaca9\uc774 \ub354\uc6b1 \uac15\ud558\uac8c \ub418\uc5c8\ub2e4. \uc774\ub9ac\ud558\uc5ec \uc885\ubb18(\u5b97\u5edf)\uc758 \uc81c\uc545(\u796d\u6a02)\uc778 \uc544\uc545\uacfc \ud568\uaed8 \ud5a5\uc5f0\uc545\uc73c\ub85c\uc11c\uc758 9\ubd80\uae30\uac00 \uc735\uc131\ud558\uc600\ub2e4.", "sentence_answer": "\ub2f9\uc2dc\uc758 \uc778\ub3c4 \ubc0f \uc774\ub780\uc758 \ubb38\ud654 \uc758 \uc601\ud5a5\uc744 \ubc1b\uc544 \ud06c\uac8c \ubc1c\ub2ec\ud55c \uc11c\uc5ed(\u897f\u57df) \uc5ec\ub7ec \ub098\ub77c\uc758 \uc545\ubb34(\u6a02\u821e)\ub97c \uc218\uc785\ud558\uc5ec \uc774\uac83\uc744 \uc911\uad6d \uace0\uc720\uc758 \uc74c\uc545\uc5d0 \uc735\ud569\ud558\uae30\ub97c \ud798\uc37c\ub2e4.", "paragraph_id": "6556573-0-0", "paragraph_question": "question: \ubd81\uc870\ub294 \ub2f9\uc2dc\uc5d0 \uc5b4\ub514 \ubb38\ud654\uc758 \uc601\ud5a5\uc744 \ubc1b\uc558\ub294\uac00?, context: \ub0a8\ubd81\uc870(\u5357\u5317\u671d) \uc2dc\ub300\uc5d0\ub294 \ubd81\ubc29\ubbfc\uc871\uc774 \ud654\ubd81(\u83ef\u5317)\uc744 \uc81c\uc555\ud558\uace0 \ud55c\uc871\uc740 \uc591\ucbd4\uac15 \ub0a8\ucabd\uc73c\ub85c \uc774\ub3d9\ud558\uc5ec \ubaa8\ub450 \uc5ec\ub7ec \uc870(\u671d)\ub97c \uacaa\uc5c8\ub294\ub370, \uadf8\uac04 \ub0a8\uc870\uc758 \ub098\ub77c\ub4e4\uc740 \uc8fc\ub85c \uc911\uad6d \uace0\uc720\uc758 \uc544\uc545(\u96c5\u6a02) \ubc0f \uc18d\uc545(\u4fd7\u6a02, \u6df8\u5546\u4e09\u8abf \ub4f1)\uc744 \uc804\ud558\uace0, \ubd81\uc870\ub294 \ub2f9\uc2dc\uc758 \uc778\ub3c4 \ubc0f \uc774\ub780\uc758 \ubb38\ud654\uc758 \uc601\ud5a5\uc744 \ubc1b\uc544 \ud06c\uac8c \ubc1c\ub2ec\ud55c \uc11c\uc5ed(\u897f\u57df) \uc5ec\ub7ec \ub098\ub77c\uc758 \uc545\ubb34(\u6a02\u821e)\ub97c \uc218\uc785\ud558\uc5ec \uc774\uac83\uc744 \uc911\uad6d \uace0\uc720\uc758 \uc74c\uc545\uc5d0 \uc735\ud569\ud558\uae30\ub97c \ud798\uc37c\ub2e4. \ud6c4\uc5d0 \ubd81\uc870\uc5d0\uc11c \uc77c\uc5b4\ub098 \uc911\uad6d\uc744 \ud1b5\uc77c\ud55c \uc218\ub098\ub77c(581-619)\ub294 \uc774\ub4e4 \uc11c\uc5ed\uc758 \uc74c\uc545\uacfc \uc911\uad6d \uace0\uc720\uc758 \uc74c\uc545\uc744 \ud569\uccd0\uc11c, \ub0b4\uc6a9\uacfc \uc6d0\uc0b0\uc9c0\uc5d0 \ub530\ub77c \uce60\ubd80\uae30(\u4e03\u90e8\u4f0e)\ub85c \ub098\ub204\uc5b4 \uad81\uc911\uc758 \ud5a5\uc5f0\uc545(\u97ff\u5bb4\u6a02)\uc744 \uc0bc\uc558\ub2e4. \ud6c4\uc5d0 \uc774 \uce60\ubd80\uae30\uc5d0 \uc18c\ub975(\u758e\u52d2, \uce74\uc2dc\uac00\ub974)\uacfc \uac15\uad6d(\u5eb7\u570b, \uc0ac\ub9c8\ub974\uce78\ub4dc)\uc758 \uae30\ub97c \ub354\ud558\uc5ec 9\ubd80\uae30(\u4e5d\u90e8\u4f0e)\uac00 \ub428\uc73c\ub85c\uc368 \uadf8 \uad6d\uc81c\uc801 \uc131\uaca9\uc774 \ub354\uc6b1 \uac15\ud558\uac8c \ub418\uc5c8\ub2e4. \uc774\ub9ac\ud558\uc5ec \uc885\ubb18(\u5b97\u5edf)\uc758 \uc81c\uc545(\u796d\u6a02)\uc778 \uc544\uc545\uacfc \ud568\uaed8 \ud5a5\uc5f0\uc545\uc73c\ub85c\uc11c\uc758 9\ubd80\uae30\uac00 \uc735\uc131\ud558\uc600\ub2e4."} {"question": "\uc7ad \ucff0\uc2a4\uc758 1910\ub144\ub3c4 6\uc6d4\ub9d0\uae4c\uc9c0 \uc2dc\uc98c \uc2b9\ud328\uc728\uc740?", "paragraph": "1910\ub144 \uc2dc\uc98c \ucd08 \ucff0\uc2a4\ub294 \ub35c \ud480\ub9b0 \ub0a0\uc528\uc758 \uc601\ud5a5\uacfc \uc81c\uad6c \ub09c\uc870 \ub4f1\uc73c\ub85c \uccab 6\uc8fc \ub3d9\uc548 \ub124 \ubc88 \ubc16\uc5d0 \uc120\ubc1c \ucd9c\uc804\ud558\uc9c0 \ubabb\ud588\uace0, \uac1c\ub9c9\ud55c\uc9c0 \ud55c \ub2ec \uac00\uae4c\uc774 \ub418\uc5b4\uc11c\uc57c \uccab \uc2b9\uc744 \uac70\ub450\uc5c8\ub2e4. \ucd08\ubc18 \ubd80\uc9c4\uc5d0\uc11c \ubc97\uc5b4\ub098\uae30 \uc704\ud574\uc11c \uadf8\ub294 \uc544\uc774\ub7ec \ud1a0\uba38\uc2a4 \ub300\uc2e0 \uc7ad \ub7a9\uc744 \uc790\uc2e0\uc758 \uc804\uc6a9 \ud3ec\uc218\ub85c \ub450\uc5b4 \uc81c\uad6c\ub97c \ud5a5\uc0c1\uc2dc\ud0a4\ub824\ub294 \ub3d9\uc2dc\uc5d0, \ub099\ucc28\uac00 \ud070 \ucee4\ube0c\ub97c \uc790\uc8fc \uc0ac\uc6a9\ud558\uba70 \ud0c0\uc790\ub4e4\uc758 \ubc29\ub9dd\uc774\ub97c \uc774\ub04c\uc5b4\ub0b4\ub824\uace0 \ud588\ub2e4. \uadf8 \uacb0\uacfc 6\uc6d4 \ub9d0\uae4c\uc9c0 8\uc2b9 4\ud328\ub97c \uac70\ub450\uc5c8\ub294\ub370, \uadf8\uc911 2\uc2b9\uc740 \uc2dc\uce74\uace0 \ud654\uc774\ud2b8\uc0ad\uc2a4\ub97c \uc0c1\ub300\ub85c \uac01\uac01 1\uc548\ud0c0\uc640 2\uc548\ud0c0\ub9cc\uc744 \ub0b4\uc8fc\uace0 \uc5bb\uc740 \uc2b9\ub9ac\uc600\ub2e4. \uc2dc\uc98c \uc911\ubc18\uc5d0 \uc811\uc5b4\ub4e4\uba74\uc11c \uc8fc\ucd95 \ud22c\uc218\uc600\ub358 \uc5d0\ub514 \ud50c\ub7ad\ud06c, \uce58\ud504 \ubca4\ub354, \uadf8\ub9ac\uace0 \ud574\ub9ac \ud06c\ub77c\uc6b0\uc2a4\uac00 \uac01\uac01 \ubd80\uc0c1\uc73c\ub85c \uc790\ub9ac\ub97c \ube44\uc6b0\ub294 \uc0ac\uc774\uc5d0, \uc7ad \ucff0\uc2a4\ub294 \ud300\uc758 \uac00\uc7a5 \ub9ce\uc740 \uc774\ub2dd\uc744 \uc18c\ud654\ud558\uba70 \uc2b9\uc218\ub97c \uc313\uc544\uac14\ub2e4. 7\uc6d4\uc5d0 \uc774\ud2c0 \uac04\uaca9\uc73c\ub85c \ubaa8\ub450 \uc5f4 \ubc88 \uc120\ubc1c \ub4f1\ud310\ud55c \uacbd\uae30\uc5d0\uc11c \ube60\uc9d0\uc5c6\uc774 \uc644\ubd09\uc2b9\uc744 \uac70\ub450\uc5c8\uc73c\uba70, \uc2dc\uc98c \uc808\ubc18\uc5d0 \uac00\uae4c\uc6b4 \uae30\uac04\uc778 7\uc6d4 12\uc77c\ubd80\ud130 9\uc6d4 30\uc77c \ub3d9\uc548\uc5d0\ub294 53\uc774\ub2dd \uc5f0\uc18d \ubb34\uc2e4\uc810 \uae30\ub85d\uc744 \uc138\uc6e0\ub2e4. 8\uc6d4 4\uc77c, \ucff0\uc2a4\ub294 \ud654\uc774\ud2b8\uc0ad\uc2a4\uc758 \uc5d0\ub4dc \uc6d4\uc2dc\uc640 16\uc774\ub2dd\uae4c\uc9c0 \uac00\ub294 \uc2b9\ubd80\ub97c \ud3bc\ucce4\ub294\ub370 \ub05d\ub0b4 \ub458\uc911 \uc5b4\ub290 \ud300\ub3c4 \uc810\uc218\ub97c \ub0b4\uc9c0 \ubabb\ud558\uba70 \uadf8\ub300\ub85c \uacbd\uae30\uac00 \uc885\ub8cc\ub418\uc5c8\ub2e4. \uadf8\ub294 \uc774 \uacbd\uae30\uc5d0\uc11c 3\uc548\ud0c0\ub9cc \ub0b4\uc8fc\ub294 \ub300\uc2e0 \ud0c8\uc0bc\uc9c4\uc744 18\uac1c \uc7a1\uc544\ub0c8\uc73c\uba70, \uc774 \uacbd\uae30\ub97c \ub450\uace0 \ub098\uc911\uc5d0 \uadf8\ub294 \uc790\uc2e0\uc758 \uc57c\uad6c \uc778\uc0dd\uc5d0\uc11c \ucd5c\uace0\uc758 \uacbd\uae30\uc600\ub2e4\uace0 \uc774\uc57c\uae30\ud588\ub2e4. \uc774\ubc88 \uc2dc\uc98c \ucd5c\uc885 \uae30\ub85d\uc740 45\uacbd\uae30(\uc120\ubc1c 38\uacbd\uae30) 353\uc774\ub2dd, 31\uc2b9 9\ud328, 13\uc644\ubd09, \ud3c9\uade0\uc790\ucc45\uc810 1.30, 224\ud0c8\uc0bc\uc9c4\uc774\uc5c8\ub2e4. \ub4f1\ud310 \uacbd\uae30 \ud69f\uc218\uc640 \uc2b9\uc218, \uc644\ubd09\uc5d0\uc11c 1\uc704\ub97c \ucc28\uc9c0\ud588\uc73c\uba70, \ud3c9\uade0\uc790\ucc45\uc810\uacfc \uc644\ud22c\ub294 2\uc704, \ud22c\uad6c \uc774\ub2dd\uacfc \ud0c8\uc0bc\uc9c4 \ubd80\ubb38\uc5d0\uc11c\ub294 3\uc704\uc5d0 \uc774\ub984\uc744 \uc62c\ub838\ub2e4. \ub610\ud55c \uc774\ubc88 \uc2dc\uc98c \ubb34\ud53c\ud648\ub7f0\uc744 \uae30\ub85d\ud588\ub2e4.", "answer": "8\uc2b9 4\ud328", "sentence": "\uadf8 \uacb0\uacfc 6\uc6d4 \ub9d0\uae4c\uc9c0 8\uc2b9 4\ud328 \ub97c \uac70\ub450\uc5c8\ub294\ub370, \uadf8\uc911 2\uc2b9\uc740 \uc2dc\uce74\uace0 \ud654\uc774\ud2b8\uc0ad\uc2a4\ub97c \uc0c1\ub300\ub85c \uac01\uac01 1\uc548\ud0c0\uc640 2\uc548\ud0c0\ub9cc\uc744 \ub0b4\uc8fc\uace0 \uc5bb\uc740 \uc2b9\ub9ac\uc600\ub2e4.", "paragraph_sentence": "1910\ub144 \uc2dc\uc98c \ucd08 \ucff0\uc2a4\ub294 \ub35c \ud480\ub9b0 \ub0a0\uc528\uc758 \uc601\ud5a5\uacfc \uc81c\uad6c \ub09c\uc870 \ub4f1\uc73c\ub85c \uccab 6\uc8fc \ub3d9\uc548 \ub124 \ubc88 \ubc16\uc5d0 \uc120\ubc1c \ucd9c\uc804\ud558\uc9c0 \ubabb\ud588\uace0, \uac1c\ub9c9\ud55c\uc9c0 \ud55c \ub2ec \uac00\uae4c\uc774 \ub418\uc5b4\uc11c\uc57c \uccab \uc2b9\uc744 \uac70\ub450\uc5c8\ub2e4. \ucd08\ubc18 \ubd80\uc9c4\uc5d0\uc11c \ubc97\uc5b4\ub098\uae30 \uc704\ud574\uc11c \uadf8\ub294 \uc544\uc774\ub7ec \ud1a0\uba38\uc2a4 \ub300\uc2e0 \uc7ad \ub7a9\uc744 \uc790\uc2e0\uc758 \uc804\uc6a9 \ud3ec\uc218\ub85c \ub450\uc5b4 \uc81c\uad6c\ub97c \ud5a5\uc0c1\uc2dc\ud0a4\ub824\ub294 \ub3d9\uc2dc\uc5d0, \ub099\ucc28\uac00 \ud070 \ucee4\ube0c\ub97c \uc790\uc8fc \uc0ac\uc6a9\ud558\uba70 \ud0c0\uc790\ub4e4\uc758 \ubc29\ub9dd\uc774\ub97c \uc774\ub04c\uc5b4\ub0b4\ub824\uace0 \ud588\ub2e4. \uadf8 \uacb0\uacfc 6\uc6d4 \ub9d0\uae4c\uc9c0 8\uc2b9 4\ud328 \ub97c \uac70\ub450\uc5c8\ub294\ub370, \uadf8\uc911 2\uc2b9\uc740 \uc2dc\uce74\uace0 \ud654\uc774\ud2b8\uc0ad\uc2a4\ub97c \uc0c1\ub300\ub85c \uac01\uac01 1\uc548\ud0c0\uc640 2\uc548\ud0c0\ub9cc\uc744 \ub0b4\uc8fc\uace0 \uc5bb\uc740 \uc2b9\ub9ac\uc600\ub2e4. \uc2dc\uc98c \uc911\ubc18\uc5d0 \uc811\uc5b4\ub4e4\uba74\uc11c \uc8fc\ucd95 \ud22c\uc218\uc600\ub358 \uc5d0\ub514 \ud50c\ub7ad\ud06c, \uce58\ud504 \ubca4\ub354, \uadf8\ub9ac\uace0 \ud574\ub9ac \ud06c\ub77c\uc6b0\uc2a4\uac00 \uac01\uac01 \ubd80\uc0c1\uc73c\ub85c \uc790\ub9ac\ub97c \ube44\uc6b0\ub294 \uc0ac\uc774\uc5d0, \uc7ad \ucff0\uc2a4\ub294 \ud300\uc758 \uac00\uc7a5 \ub9ce\uc740 \uc774\ub2dd\uc744 \uc18c\ud654\ud558\uba70 \uc2b9\uc218\ub97c \uc313\uc544\uac14\ub2e4. 7\uc6d4\uc5d0 \uc774\ud2c0 \uac04\uaca9\uc73c\ub85c \ubaa8\ub450 \uc5f4 \ubc88 \uc120\ubc1c \ub4f1\ud310\ud55c \uacbd\uae30\uc5d0\uc11c \ube60\uc9d0\uc5c6\uc774 \uc644\ubd09\uc2b9\uc744 \uac70\ub450\uc5c8\uc73c\uba70, \uc2dc\uc98c \uc808\ubc18\uc5d0 \uac00\uae4c\uc6b4 \uae30\uac04\uc778 7\uc6d4 12\uc77c\ubd80\ud130 9\uc6d4 30\uc77c \ub3d9\uc548\uc5d0\ub294 53\uc774\ub2dd \uc5f0\uc18d \ubb34\uc2e4\uc810 \uae30\ub85d\uc744 \uc138\uc6e0\ub2e4. 8\uc6d4 4\uc77c, \ucff0\uc2a4\ub294 \ud654\uc774\ud2b8\uc0ad\uc2a4\uc758 \uc5d0\ub4dc \uc6d4\uc2dc\uc640 16\uc774\ub2dd\uae4c\uc9c0 \uac00\ub294 \uc2b9\ubd80\ub97c \ud3bc\ucce4\ub294\ub370 \ub05d\ub0b4 \ub458\uc911 \uc5b4\ub290 \ud300\ub3c4 \uc810\uc218\ub97c \ub0b4\uc9c0 \ubabb\ud558\uba70 \uadf8\ub300\ub85c \uacbd\uae30\uac00 \uc885\ub8cc\ub418\uc5c8\ub2e4. \uadf8\ub294 \uc774 \uacbd\uae30\uc5d0\uc11c 3\uc548\ud0c0\ub9cc \ub0b4\uc8fc\ub294 \ub300\uc2e0 \ud0c8\uc0bc\uc9c4\uc744 18\uac1c \uc7a1\uc544\ub0c8\uc73c\uba70, \uc774 \uacbd\uae30\ub97c \ub450\uace0 \ub098\uc911\uc5d0 \uadf8\ub294 \uc790\uc2e0\uc758 \uc57c\uad6c \uc778\uc0dd\uc5d0\uc11c \ucd5c\uace0\uc758 \uacbd\uae30\uc600\ub2e4\uace0 \uc774\uc57c\uae30\ud588\ub2e4. \uc774\ubc88 \uc2dc\uc98c \ucd5c\uc885 \uae30\ub85d\uc740 45\uacbd\uae30(\uc120\ubc1c 38\uacbd\uae30) 353\uc774\ub2dd, 31\uc2b9 9\ud328, 13\uc644\ubd09, \ud3c9\uade0\uc790\ucc45\uc810 1.30, 224\ud0c8\uc0bc\uc9c4\uc774\uc5c8\ub2e4. \ub4f1\ud310 \uacbd\uae30 \ud69f\uc218\uc640 \uc2b9\uc218, \uc644\ubd09\uc5d0\uc11c 1\uc704\ub97c \ucc28\uc9c0\ud588\uc73c\uba70, \ud3c9\uade0\uc790\ucc45\uc810\uacfc \uc644\ud22c\ub294 2\uc704, \ud22c\uad6c \uc774\ub2dd\uacfc \ud0c8\uc0bc\uc9c4 \ubd80\ubb38\uc5d0\uc11c\ub294 3\uc704\uc5d0 \uc774\ub984\uc744 \uc62c\ub838\ub2e4. \ub610\ud55c \uc774\ubc88 \uc2dc\uc98c \ubb34\ud53c\ud648\ub7f0\uc744 \uae30\ub85d\ud588\ub2e4.", "paragraph_answer": "1910\ub144 \uc2dc\uc98c \ucd08 \ucff0\uc2a4\ub294 \ub35c \ud480\ub9b0 \ub0a0\uc528\uc758 \uc601\ud5a5\uacfc \uc81c\uad6c \ub09c\uc870 \ub4f1\uc73c\ub85c \uccab 6\uc8fc \ub3d9\uc548 \ub124 \ubc88 \ubc16\uc5d0 \uc120\ubc1c \ucd9c\uc804\ud558\uc9c0 \ubabb\ud588\uace0, \uac1c\ub9c9\ud55c\uc9c0 \ud55c \ub2ec \uac00\uae4c\uc774 \ub418\uc5b4\uc11c\uc57c \uccab \uc2b9\uc744 \uac70\ub450\uc5c8\ub2e4. \ucd08\ubc18 \ubd80\uc9c4\uc5d0\uc11c \ubc97\uc5b4\ub098\uae30 \uc704\ud574\uc11c \uadf8\ub294 \uc544\uc774\ub7ec \ud1a0\uba38\uc2a4 \ub300\uc2e0 \uc7ad \ub7a9\uc744 \uc790\uc2e0\uc758 \uc804\uc6a9 \ud3ec\uc218\ub85c \ub450\uc5b4 \uc81c\uad6c\ub97c \ud5a5\uc0c1\uc2dc\ud0a4\ub824\ub294 \ub3d9\uc2dc\uc5d0, \ub099\ucc28\uac00 \ud070 \ucee4\ube0c\ub97c \uc790\uc8fc \uc0ac\uc6a9\ud558\uba70 \ud0c0\uc790\ub4e4\uc758 \ubc29\ub9dd\uc774\ub97c \uc774\ub04c\uc5b4\ub0b4\ub824\uace0 \ud588\ub2e4. \uadf8 \uacb0\uacfc 6\uc6d4 \ub9d0\uae4c\uc9c0 8\uc2b9 4\ud328 \ub97c \uac70\ub450\uc5c8\ub294\ub370, \uadf8\uc911 2\uc2b9\uc740 \uc2dc\uce74\uace0 \ud654\uc774\ud2b8\uc0ad\uc2a4\ub97c \uc0c1\ub300\ub85c \uac01\uac01 1\uc548\ud0c0\uc640 2\uc548\ud0c0\ub9cc\uc744 \ub0b4\uc8fc\uace0 \uc5bb\uc740 \uc2b9\ub9ac\uc600\ub2e4. \uc2dc\uc98c \uc911\ubc18\uc5d0 \uc811\uc5b4\ub4e4\uba74\uc11c \uc8fc\ucd95 \ud22c\uc218\uc600\ub358 \uc5d0\ub514 \ud50c\ub7ad\ud06c, \uce58\ud504 \ubca4\ub354, \uadf8\ub9ac\uace0 \ud574\ub9ac \ud06c\ub77c\uc6b0\uc2a4\uac00 \uac01\uac01 \ubd80\uc0c1\uc73c\ub85c \uc790\ub9ac\ub97c \ube44\uc6b0\ub294 \uc0ac\uc774\uc5d0, \uc7ad \ucff0\uc2a4\ub294 \ud300\uc758 \uac00\uc7a5 \ub9ce\uc740 \uc774\ub2dd\uc744 \uc18c\ud654\ud558\uba70 \uc2b9\uc218\ub97c \uc313\uc544\uac14\ub2e4. 7\uc6d4\uc5d0 \uc774\ud2c0 \uac04\uaca9\uc73c\ub85c \ubaa8\ub450 \uc5f4 \ubc88 \uc120\ubc1c \ub4f1\ud310\ud55c \uacbd\uae30\uc5d0\uc11c \ube60\uc9d0\uc5c6\uc774 \uc644\ubd09\uc2b9\uc744 \uac70\ub450\uc5c8\uc73c\uba70, \uc2dc\uc98c \uc808\ubc18\uc5d0 \uac00\uae4c\uc6b4 \uae30\uac04\uc778 7\uc6d4 12\uc77c\ubd80\ud130 9\uc6d4 30\uc77c \ub3d9\uc548\uc5d0\ub294 53\uc774\ub2dd \uc5f0\uc18d \ubb34\uc2e4\uc810 \uae30\ub85d\uc744 \uc138\uc6e0\ub2e4. 8\uc6d4 4\uc77c, \ucff0\uc2a4\ub294 \ud654\uc774\ud2b8\uc0ad\uc2a4\uc758 \uc5d0\ub4dc \uc6d4\uc2dc\uc640 16\uc774\ub2dd\uae4c\uc9c0 \uac00\ub294 \uc2b9\ubd80\ub97c \ud3bc\ucce4\ub294\ub370 \ub05d\ub0b4 \ub458\uc911 \uc5b4\ub290 \ud300\ub3c4 \uc810\uc218\ub97c \ub0b4\uc9c0 \ubabb\ud558\uba70 \uadf8\ub300\ub85c \uacbd\uae30\uac00 \uc885\ub8cc\ub418\uc5c8\ub2e4. \uadf8\ub294 \uc774 \uacbd\uae30\uc5d0\uc11c 3\uc548\ud0c0\ub9cc \ub0b4\uc8fc\ub294 \ub300\uc2e0 \ud0c8\uc0bc\uc9c4\uc744 18\uac1c \uc7a1\uc544\ub0c8\uc73c\uba70, \uc774 \uacbd\uae30\ub97c \ub450\uace0 \ub098\uc911\uc5d0 \uadf8\ub294 \uc790\uc2e0\uc758 \uc57c\uad6c \uc778\uc0dd\uc5d0\uc11c \ucd5c\uace0\uc758 \uacbd\uae30\uc600\ub2e4\uace0 \uc774\uc57c\uae30\ud588\ub2e4. \uc774\ubc88 \uc2dc\uc98c \ucd5c\uc885 \uae30\ub85d\uc740 45\uacbd\uae30(\uc120\ubc1c 38\uacbd\uae30) 353\uc774\ub2dd, 31\uc2b9 9\ud328, 13\uc644\ubd09, \ud3c9\uade0\uc790\ucc45\uc810 1.30, 224\ud0c8\uc0bc\uc9c4\uc774\uc5c8\ub2e4. \ub4f1\ud310 \uacbd\uae30 \ud69f\uc218\uc640 \uc2b9\uc218, \uc644\ubd09\uc5d0\uc11c 1\uc704\ub97c \ucc28\uc9c0\ud588\uc73c\uba70, \ud3c9\uade0\uc790\ucc45\uc810\uacfc \uc644\ud22c\ub294 2\uc704, \ud22c\uad6c \uc774\ub2dd\uacfc \ud0c8\uc0bc\uc9c4 \ubd80\ubb38\uc5d0\uc11c\ub294 3\uc704\uc5d0 \uc774\ub984\uc744 \uc62c\ub838\ub2e4. \ub610\ud55c \uc774\ubc88 \uc2dc\uc98c \ubb34\ud53c\ud648\ub7f0\uc744 \uae30\ub85d\ud588\ub2e4.", "sentence_answer": "\uadf8 \uacb0\uacfc 6\uc6d4 \ub9d0\uae4c\uc9c0 8\uc2b9 4\ud328 \ub97c \uac70\ub450\uc5c8\ub294\ub370, \uadf8\uc911 2\uc2b9\uc740 \uc2dc\uce74\uace0 \ud654\uc774\ud2b8\uc0ad\uc2a4\ub97c \uc0c1\ub300\ub85c \uac01\uac01 1\uc548\ud0c0\uc640 2\uc548\ud0c0\ub9cc\uc744 \ub0b4\uc8fc\uace0 \uc5bb\uc740 \uc2b9\ub9ac\uc600\ub2e4.", "paragraph_id": "6608933-4-0", "paragraph_question": "question: \uc7ad \ucff0\uc2a4\uc758 1910\ub144\ub3c4 6\uc6d4\ub9d0\uae4c\uc9c0 \uc2dc\uc98c \uc2b9\ud328\uc728\uc740?, context: 1910\ub144 \uc2dc\uc98c \ucd08 \ucff0\uc2a4\ub294 \ub35c \ud480\ub9b0 \ub0a0\uc528\uc758 \uc601\ud5a5\uacfc \uc81c\uad6c \ub09c\uc870 \ub4f1\uc73c\ub85c \uccab 6\uc8fc \ub3d9\uc548 \ub124 \ubc88 \ubc16\uc5d0 \uc120\ubc1c \ucd9c\uc804\ud558\uc9c0 \ubabb\ud588\uace0, \uac1c\ub9c9\ud55c\uc9c0 \ud55c \ub2ec \uac00\uae4c\uc774 \ub418\uc5b4\uc11c\uc57c \uccab \uc2b9\uc744 \uac70\ub450\uc5c8\ub2e4. \ucd08\ubc18 \ubd80\uc9c4\uc5d0\uc11c \ubc97\uc5b4\ub098\uae30 \uc704\ud574\uc11c \uadf8\ub294 \uc544\uc774\ub7ec \ud1a0\uba38\uc2a4 \ub300\uc2e0 \uc7ad \ub7a9\uc744 \uc790\uc2e0\uc758 \uc804\uc6a9 \ud3ec\uc218\ub85c \ub450\uc5b4 \uc81c\uad6c\ub97c \ud5a5\uc0c1\uc2dc\ud0a4\ub824\ub294 \ub3d9\uc2dc\uc5d0, \ub099\ucc28\uac00 \ud070 \ucee4\ube0c\ub97c \uc790\uc8fc \uc0ac\uc6a9\ud558\uba70 \ud0c0\uc790\ub4e4\uc758 \ubc29\ub9dd\uc774\ub97c \uc774\ub04c\uc5b4\ub0b4\ub824\uace0 \ud588\ub2e4. \uadf8 \uacb0\uacfc 6\uc6d4 \ub9d0\uae4c\uc9c0 8\uc2b9 4\ud328\ub97c \uac70\ub450\uc5c8\ub294\ub370, \uadf8\uc911 2\uc2b9\uc740 \uc2dc\uce74\uace0 \ud654\uc774\ud2b8\uc0ad\uc2a4\ub97c \uc0c1\ub300\ub85c \uac01\uac01 1\uc548\ud0c0\uc640 2\uc548\ud0c0\ub9cc\uc744 \ub0b4\uc8fc\uace0 \uc5bb\uc740 \uc2b9\ub9ac\uc600\ub2e4. \uc2dc\uc98c \uc911\ubc18\uc5d0 \uc811\uc5b4\ub4e4\uba74\uc11c \uc8fc\ucd95 \ud22c\uc218\uc600\ub358 \uc5d0\ub514 \ud50c\ub7ad\ud06c, \uce58\ud504 \ubca4\ub354, \uadf8\ub9ac\uace0 \ud574\ub9ac \ud06c\ub77c\uc6b0\uc2a4\uac00 \uac01\uac01 \ubd80\uc0c1\uc73c\ub85c \uc790\ub9ac\ub97c \ube44\uc6b0\ub294 \uc0ac\uc774\uc5d0, \uc7ad \ucff0\uc2a4\ub294 \ud300\uc758 \uac00\uc7a5 \ub9ce\uc740 \uc774\ub2dd\uc744 \uc18c\ud654\ud558\uba70 \uc2b9\uc218\ub97c \uc313\uc544\uac14\ub2e4. 7\uc6d4\uc5d0 \uc774\ud2c0 \uac04\uaca9\uc73c\ub85c \ubaa8\ub450 \uc5f4 \ubc88 \uc120\ubc1c \ub4f1\ud310\ud55c \uacbd\uae30\uc5d0\uc11c \ube60\uc9d0\uc5c6\uc774 \uc644\ubd09\uc2b9\uc744 \uac70\ub450\uc5c8\uc73c\uba70, \uc2dc\uc98c \uc808\ubc18\uc5d0 \uac00\uae4c\uc6b4 \uae30\uac04\uc778 7\uc6d4 12\uc77c\ubd80\ud130 9\uc6d4 30\uc77c \ub3d9\uc548\uc5d0\ub294 53\uc774\ub2dd \uc5f0\uc18d \ubb34\uc2e4\uc810 \uae30\ub85d\uc744 \uc138\uc6e0\ub2e4. 8\uc6d4 4\uc77c, \ucff0\uc2a4\ub294 \ud654\uc774\ud2b8\uc0ad\uc2a4\uc758 \uc5d0\ub4dc \uc6d4\uc2dc\uc640 16\uc774\ub2dd\uae4c\uc9c0 \uac00\ub294 \uc2b9\ubd80\ub97c \ud3bc\ucce4\ub294\ub370 \ub05d\ub0b4 \ub458\uc911 \uc5b4\ub290 \ud300\ub3c4 \uc810\uc218\ub97c \ub0b4\uc9c0 \ubabb\ud558\uba70 \uadf8\ub300\ub85c \uacbd\uae30\uac00 \uc885\ub8cc\ub418\uc5c8\ub2e4. \uadf8\ub294 \uc774 \uacbd\uae30\uc5d0\uc11c 3\uc548\ud0c0\ub9cc \ub0b4\uc8fc\ub294 \ub300\uc2e0 \ud0c8\uc0bc\uc9c4\uc744 18\uac1c \uc7a1\uc544\ub0c8\uc73c\uba70, \uc774 \uacbd\uae30\ub97c \ub450\uace0 \ub098\uc911\uc5d0 \uadf8\ub294 \uc790\uc2e0\uc758 \uc57c\uad6c \uc778\uc0dd\uc5d0\uc11c \ucd5c\uace0\uc758 \uacbd\uae30\uc600\ub2e4\uace0 \uc774\uc57c\uae30\ud588\ub2e4. \uc774\ubc88 \uc2dc\uc98c \ucd5c\uc885 \uae30\ub85d\uc740 45\uacbd\uae30(\uc120\ubc1c 38\uacbd\uae30) 353\uc774\ub2dd, 31\uc2b9 9\ud328, 13\uc644\ubd09, \ud3c9\uade0\uc790\ucc45\uc810 1.30, 224\ud0c8\uc0bc\uc9c4\uc774\uc5c8\ub2e4. \ub4f1\ud310 \uacbd\uae30 \ud69f\uc218\uc640 \uc2b9\uc218, \uc644\ubd09\uc5d0\uc11c 1\uc704\ub97c \ucc28\uc9c0\ud588\uc73c\uba70, \ud3c9\uade0\uc790\ucc45\uc810\uacfc \uc644\ud22c\ub294 2\uc704, \ud22c\uad6c \uc774\ub2dd\uacfc \ud0c8\uc0bc\uc9c4 \ubd80\ubb38\uc5d0\uc11c\ub294 3\uc704\uc5d0 \uc774\ub984\uc744 \uc62c\ub838\ub2e4. \ub610\ud55c \uc774\ubc88 \uc2dc\uc98c \ubb34\ud53c\ud648\ub7f0\uc744 \uae30\ub85d\ud588\ub2e4."} {"question": "\uae40\uae30\uc2dd\uc740 \uc784\uae30 \ub9d0 \ubb34\uc5c7\uc744 \ub0a8\uc6a9\ud558\uc600\ub294\uac00?", "paragraph": "2013\ub144 10\uc6d4 22\uc77c \uae40\uae30\uc2dd \uc758\uc6d0\uc740 \uad6d\uac10\uc5d0\uc11c \uc774\uc740\uc7ac \ud55c\uad6d\ud589\uc815\uc5f0\uad6c\uc6d0\uc7a5\uc5d0\uac8c \"\uc2a4\ud0c0\ubc85\uc2a4 \ucee4\ud53c\uc20d\uc5d0\uc11c \uc735\ud569\uacfc \ud601\uc2e0\uc744 \ud1b5\ud55c \uc815\uad00 \ud611\ub825\ubc29\uc548\uc5d0 \uad00\ud55c \ub17c\uc758\ub97c \ud55c\ub2e4\uace0 1\ub9cc 2700\uc6d0\uc744 \uc4f0\uc168\uc5b4\uc694.\u2026\uc774\ub7f0 \uc815\ub3c4\ub294 \uc0ac\ube44\ub85c \uc4f0\uc154\uc57c\uc9c0 \uc774\uac83\uc744 \ubc95\uc778\uce74\ub4dc\ub97c \uc4f0\uc2dc\ub294 \uac83\uc740 \uc801\uc808\uce58 \uc54a\uc9c0\uc694?\"\ub77c\uace0 \uacf5\uaca9\ud588\ub2e4. \ucee4\ud53c\uac12\uc744 \ub193\uace0\ub3c4 \uacf5\uacfc \uc0ac\ub97c \uc5c4\uaca9\ud788 \uad6c\ubd84\ud588\ub358 \uae40\uae30\uc2dd\uc740 \uc784\uae30 \ub9d0 \uc815\uce58 \uc790\uae08\uc744 \ub0a8\uc6a9\ud588\ub2e4. \uc815\uce58\uc790\uae08\uc740 \uc784\uae30 \ub9cc\ub8cc\uc640 \ud568\uaed8 \uad6d\uace0\ub85c \ubc18\ub0a9\ud574\uc57c \ud55c\ub2e4. \ud558\uc9c0\ub9cc \uadf8\ub294 19\ub300 \uad6d\ud68c \uc784\uae30 \uc885\ub8cc \uc9c1\uc804\uc778 2016\ub144 1\uc6d4 1\uc77c\ubd80\ud130 6\uc6d4 10\uc77c\uae4c\uc9c0 \ucd1d 3\uc5b56\ucc9c849\ub9cc\uc6d0 \uac00\ub7c9\uc758 \uc815\uce58\uc790\uae08\uc744 \ub0a8\uc6a9\ud588\ub2e4. \uc774 \uc911 \uae40\uae30\uc2dd\uc774 \uc11c\uc6b8 \uac15\ubd81\uac11 \uc9c0\uc5ed \uacbd\uc120\uc5d0\uc11c \ud0c8\ub77d\ud55c 3\uc6d4 22\uc77c\uae4c\uc9c0\uc758 \uc0ac\uc6a9\uc561\uc740 6\ucc9c115\ub9cc\uc6d0, \uacbd\uc120\uc5d0\uc11c \ud0c8\ub77d\ud55c \ub4a4 6\uc6d4 10\uc77c\uae4c\uc9c0 80\uc77c \ub3d9\uc548\uc758 \uc0ac\uc6a9\uc561\uc740 3\uc5b5734\ub9cc\uc6d0\uc73c\ub85c \uac01\uac01 \uc9d1\uacc4\ub410\ub2e4. 2016\ub144 \uc804\uccb4 \uc0ac\uc6a9\uc561\uc758 83.4%\uac00\ub7c9\uc774 \uc774 80\uc77c\uac04\uc5d0 \uc9d1\uc911\ub418\uc5c8\ub2e4. 2016\ub144 5\uc6d4 20\uc77c \uc5ec\ube44\uc11c\uc640 \ud568\uaed8 \ub3c5\uc77c\u00b7\ub124\ub35c\ub780\ub4dc\u00b7\uc2a4\uc6e8\ub374\uc73c\ub85c \ucd9c\uc7a5\uc744 \uac14\ub2e4. 5\uc6d4 19\uc77c\uc5d4 \uc815\uce58\ud6c4\uc6d0\uae08 5000\ub9cc\uc6d0\uc744 \ubbfc\uc8fc\ub2f9 \ucd08\u00b7\uc7ac\uc120 \uc758\uc6d0\ubaa8\uc784 '\ub354\uc88b\uc740\ubbf8\ub798'\uc5d0 \uc804\ub2ec\ud588\ub2e4. \uc790\uc2e0\uc758 \ubcf4\uc88c\uc9c4\ub4e4\uc5d0\uac8c\ub3c4 \ud1f4\uc9c1\uae08 \uba85\ubaa9\uc73c\ub85c \ubaa8\ub450 2200\ub9cc \uc6d0\uc744 \uacc4\uc88c \uc774\uccb4\ud588\ub2e4.", "answer": "\uc815\uce58 \uc790\uae08", "sentence": "\uae40\uae30\uc2dd\uc740 \uc784\uae30 \ub9d0 \uc815\uce58 \uc790\uae08 \uc744 \ub0a8\uc6a9\ud588\ub2e4.", "paragraph_sentence": "2013\ub144 10\uc6d4 22\uc77c \uae40\uae30\uc2dd \uc758\uc6d0\uc740 \uad6d\uac10\uc5d0\uc11c \uc774\uc740\uc7ac \ud55c\uad6d\ud589\uc815\uc5f0\uad6c\uc6d0\uc7a5\uc5d0\uac8c \"\uc2a4\ud0c0\ubc85\uc2a4 \ucee4\ud53c\uc20d\uc5d0\uc11c \uc735\ud569\uacfc \ud601\uc2e0\uc744 \ud1b5\ud55c \uc815\uad00 \ud611\ub825\ubc29\uc548\uc5d0 \uad00\ud55c \ub17c\uc758\ub97c \ud55c\ub2e4\uace0 1\ub9cc 2700\uc6d0\uc744 \uc4f0\uc168\uc5b4\uc694.\u2026\uc774\ub7f0 \uc815\ub3c4\ub294 \uc0ac\ube44\ub85c \uc4f0\uc154\uc57c\uc9c0 \uc774\uac83\uc744 \ubc95\uc778\uce74\ub4dc\ub97c \uc4f0\uc2dc\ub294 \uac83\uc740 \uc801\uc808\uce58 \uc54a\uc9c0\uc694?\"\ub77c\uace0 \uacf5\uaca9\ud588\ub2e4. \ucee4\ud53c\uac12\uc744 \ub193\uace0\ub3c4 \uacf5\uacfc \uc0ac\ub97c \uc5c4\uaca9\ud788 \uad6c\ubd84\ud588\ub358 \uae40\uae30\uc2dd\uc740 \uc784\uae30 \ub9d0 \uc815\uce58 \uc790\uae08 \uc744 \ub0a8\uc6a9\ud588\ub2e4. \uc815\uce58\uc790\uae08\uc740 \uc784\uae30 \ub9cc\ub8cc\uc640 \ud568\uaed8 \uad6d\uace0\ub85c \ubc18\ub0a9\ud574\uc57c \ud55c\ub2e4. \ud558\uc9c0\ub9cc \uadf8\ub294 19\ub300 \uad6d\ud68c \uc784\uae30 \uc885\ub8cc \uc9c1\uc804\uc778 2016\ub144 1\uc6d4 1\uc77c\ubd80\ud130 6\uc6d4 10\uc77c\uae4c\uc9c0 \ucd1d 3\uc5b56\ucc9c849\ub9cc\uc6d0 \uac00\ub7c9\uc758 \uc815\uce58\uc790\uae08\uc744 \ub0a8\uc6a9\ud588\ub2e4. \uc774 \uc911 \uae40\uae30\uc2dd\uc774 \uc11c\uc6b8 \uac15\ubd81\uac11 \uc9c0\uc5ed \uacbd\uc120\uc5d0\uc11c \ud0c8\ub77d\ud55c 3\uc6d4 22\uc77c\uae4c\uc9c0\uc758 \uc0ac\uc6a9\uc561\uc740 6\ucc9c115\ub9cc\uc6d0, \uacbd\uc120\uc5d0\uc11c \ud0c8\ub77d\ud55c \ub4a4 6\uc6d4 10\uc77c\uae4c\uc9c0 80\uc77c \ub3d9\uc548\uc758 \uc0ac\uc6a9\uc561\uc740 3\uc5b5734\ub9cc\uc6d0\uc73c\ub85c \uac01\uac01 \uc9d1\uacc4\ub410\ub2e4. 2016\ub144 \uc804\uccb4 \uc0ac\uc6a9\uc561\uc758 83.4%\uac00\ub7c9\uc774 \uc774 80\uc77c\uac04\uc5d0 \uc9d1\uc911\ub418\uc5c8\ub2e4. 2016\ub144 5\uc6d4 20\uc77c \uc5ec\ube44\uc11c\uc640 \ud568\uaed8 \ub3c5\uc77c\u00b7\ub124\ub35c\ub780\ub4dc\u00b7\uc2a4\uc6e8\ub374\uc73c\ub85c \ucd9c\uc7a5\uc744 \uac14\ub2e4. 5\uc6d4 19\uc77c\uc5d4 \uc815\uce58\ud6c4\uc6d0\uae08 5000\ub9cc\uc6d0\uc744 \ubbfc\uc8fc\ub2f9 \ucd08\u00b7\uc7ac\uc120 \uc758\uc6d0\ubaa8\uc784 '\ub354\uc88b\uc740\ubbf8\ub798'\uc5d0 \uc804\ub2ec\ud588\ub2e4. \uc790\uc2e0\uc758 \ubcf4\uc88c\uc9c4\ub4e4\uc5d0\uac8c\ub3c4 \ud1f4\uc9c1\uae08 \uba85\ubaa9\uc73c\ub85c \ubaa8\ub450 2200\ub9cc \uc6d0\uc744 \uacc4\uc88c \uc774\uccb4\ud588\ub2e4.", "paragraph_answer": "2013\ub144 10\uc6d4 22\uc77c \uae40\uae30\uc2dd \uc758\uc6d0\uc740 \uad6d\uac10\uc5d0\uc11c \uc774\uc740\uc7ac \ud55c\uad6d\ud589\uc815\uc5f0\uad6c\uc6d0\uc7a5\uc5d0\uac8c \"\uc2a4\ud0c0\ubc85\uc2a4 \ucee4\ud53c\uc20d\uc5d0\uc11c \uc735\ud569\uacfc \ud601\uc2e0\uc744 \ud1b5\ud55c \uc815\uad00 \ud611\ub825\ubc29\uc548\uc5d0 \uad00\ud55c \ub17c\uc758\ub97c \ud55c\ub2e4\uace0 1\ub9cc 2700\uc6d0\uc744 \uc4f0\uc168\uc5b4\uc694.\u2026\uc774\ub7f0 \uc815\ub3c4\ub294 \uc0ac\ube44\ub85c \uc4f0\uc154\uc57c\uc9c0 \uc774\uac83\uc744 \ubc95\uc778\uce74\ub4dc\ub97c \uc4f0\uc2dc\ub294 \uac83\uc740 \uc801\uc808\uce58 \uc54a\uc9c0\uc694?\"\ub77c\uace0 \uacf5\uaca9\ud588\ub2e4. \ucee4\ud53c\uac12\uc744 \ub193\uace0\ub3c4 \uacf5\uacfc \uc0ac\ub97c \uc5c4\uaca9\ud788 \uad6c\ubd84\ud588\ub358 \uae40\uae30\uc2dd\uc740 \uc784\uae30 \ub9d0 \uc815\uce58 \uc790\uae08 \uc744 \ub0a8\uc6a9\ud588\ub2e4. \uc815\uce58\uc790\uae08\uc740 \uc784\uae30 \ub9cc\ub8cc\uc640 \ud568\uaed8 \uad6d\uace0\ub85c \ubc18\ub0a9\ud574\uc57c \ud55c\ub2e4. \ud558\uc9c0\ub9cc \uadf8\ub294 19\ub300 \uad6d\ud68c \uc784\uae30 \uc885\ub8cc \uc9c1\uc804\uc778 2016\ub144 1\uc6d4 1\uc77c\ubd80\ud130 6\uc6d4 10\uc77c\uae4c\uc9c0 \ucd1d 3\uc5b56\ucc9c849\ub9cc\uc6d0 \uac00\ub7c9\uc758 \uc815\uce58\uc790\uae08\uc744 \ub0a8\uc6a9\ud588\ub2e4. \uc774 \uc911 \uae40\uae30\uc2dd\uc774 \uc11c\uc6b8 \uac15\ubd81\uac11 \uc9c0\uc5ed \uacbd\uc120\uc5d0\uc11c \ud0c8\ub77d\ud55c 3\uc6d4 22\uc77c\uae4c\uc9c0\uc758 \uc0ac\uc6a9\uc561\uc740 6\ucc9c115\ub9cc\uc6d0, \uacbd\uc120\uc5d0\uc11c \ud0c8\ub77d\ud55c \ub4a4 6\uc6d4 10\uc77c\uae4c\uc9c0 80\uc77c \ub3d9\uc548\uc758 \uc0ac\uc6a9\uc561\uc740 3\uc5b5734\ub9cc\uc6d0\uc73c\ub85c \uac01\uac01 \uc9d1\uacc4\ub410\ub2e4. 2016\ub144 \uc804\uccb4 \uc0ac\uc6a9\uc561\uc758 83.4%\uac00\ub7c9\uc774 \uc774 80\uc77c\uac04\uc5d0 \uc9d1\uc911\ub418\uc5c8\ub2e4. 2016\ub144 5\uc6d4 20\uc77c \uc5ec\ube44\uc11c\uc640 \ud568\uaed8 \ub3c5\uc77c\u00b7\ub124\ub35c\ub780\ub4dc\u00b7\uc2a4\uc6e8\ub374\uc73c\ub85c \ucd9c\uc7a5\uc744 \uac14\ub2e4. 5\uc6d4 19\uc77c\uc5d4 \uc815\uce58\ud6c4\uc6d0\uae08 5000\ub9cc\uc6d0\uc744 \ubbfc\uc8fc\ub2f9 \ucd08\u00b7\uc7ac\uc120 \uc758\uc6d0\ubaa8\uc784 '\ub354\uc88b\uc740\ubbf8\ub798'\uc5d0 \uc804\ub2ec\ud588\ub2e4. \uc790\uc2e0\uc758 \ubcf4\uc88c\uc9c4\ub4e4\uc5d0\uac8c\ub3c4 \ud1f4\uc9c1\uae08 \uba85\ubaa9\uc73c\ub85c \ubaa8\ub450 2200\ub9cc \uc6d0\uc744 \uacc4\uc88c \uc774\uccb4\ud588\ub2e4.", "sentence_answer": "\uae40\uae30\uc2dd\uc740 \uc784\uae30 \ub9d0 \uc815\uce58 \uc790\uae08 \uc744 \ub0a8\uc6a9\ud588\ub2e4.", "paragraph_id": "6481286-5-0", "paragraph_question": "question: \uae40\uae30\uc2dd\uc740 \uc784\uae30 \ub9d0 \ubb34\uc5c7\uc744 \ub0a8\uc6a9\ud558\uc600\ub294\uac00?, context: 2013\ub144 10\uc6d4 22\uc77c \uae40\uae30\uc2dd \uc758\uc6d0\uc740 \uad6d\uac10\uc5d0\uc11c \uc774\uc740\uc7ac \ud55c\uad6d\ud589\uc815\uc5f0\uad6c\uc6d0\uc7a5\uc5d0\uac8c \"\uc2a4\ud0c0\ubc85\uc2a4 \ucee4\ud53c\uc20d\uc5d0\uc11c \uc735\ud569\uacfc \ud601\uc2e0\uc744 \ud1b5\ud55c \uc815\uad00 \ud611\ub825\ubc29\uc548\uc5d0 \uad00\ud55c \ub17c\uc758\ub97c \ud55c\ub2e4\uace0 1\ub9cc 2700\uc6d0\uc744 \uc4f0\uc168\uc5b4\uc694.\u2026\uc774\ub7f0 \uc815\ub3c4\ub294 \uc0ac\ube44\ub85c \uc4f0\uc154\uc57c\uc9c0 \uc774\uac83\uc744 \ubc95\uc778\uce74\ub4dc\ub97c \uc4f0\uc2dc\ub294 \uac83\uc740 \uc801\uc808\uce58 \uc54a\uc9c0\uc694?\"\ub77c\uace0 \uacf5\uaca9\ud588\ub2e4. \ucee4\ud53c\uac12\uc744 \ub193\uace0\ub3c4 \uacf5\uacfc \uc0ac\ub97c \uc5c4\uaca9\ud788 \uad6c\ubd84\ud588\ub358 \uae40\uae30\uc2dd\uc740 \uc784\uae30 \ub9d0 \uc815\uce58 \uc790\uae08\uc744 \ub0a8\uc6a9\ud588\ub2e4. \uc815\uce58\uc790\uae08\uc740 \uc784\uae30 \ub9cc\ub8cc\uc640 \ud568\uaed8 \uad6d\uace0\ub85c \ubc18\ub0a9\ud574\uc57c \ud55c\ub2e4. \ud558\uc9c0\ub9cc \uadf8\ub294 19\ub300 \uad6d\ud68c \uc784\uae30 \uc885\ub8cc \uc9c1\uc804\uc778 2016\ub144 1\uc6d4 1\uc77c\ubd80\ud130 6\uc6d4 10\uc77c\uae4c\uc9c0 \ucd1d 3\uc5b56\ucc9c849\ub9cc\uc6d0 \uac00\ub7c9\uc758 \uc815\uce58\uc790\uae08\uc744 \ub0a8\uc6a9\ud588\ub2e4. \uc774 \uc911 \uae40\uae30\uc2dd\uc774 \uc11c\uc6b8 \uac15\ubd81\uac11 \uc9c0\uc5ed \uacbd\uc120\uc5d0\uc11c \ud0c8\ub77d\ud55c 3\uc6d4 22\uc77c\uae4c\uc9c0\uc758 \uc0ac\uc6a9\uc561\uc740 6\ucc9c115\ub9cc\uc6d0, \uacbd\uc120\uc5d0\uc11c \ud0c8\ub77d\ud55c \ub4a4 6\uc6d4 10\uc77c\uae4c\uc9c0 80\uc77c \ub3d9\uc548\uc758 \uc0ac\uc6a9\uc561\uc740 3\uc5b5734\ub9cc\uc6d0\uc73c\ub85c \uac01\uac01 \uc9d1\uacc4\ub410\ub2e4. 2016\ub144 \uc804\uccb4 \uc0ac\uc6a9\uc561\uc758 83.4%\uac00\ub7c9\uc774 \uc774 80\uc77c\uac04\uc5d0 \uc9d1\uc911\ub418\uc5c8\ub2e4. 2016\ub144 5\uc6d4 20\uc77c \uc5ec\ube44\uc11c\uc640 \ud568\uaed8 \ub3c5\uc77c\u00b7\ub124\ub35c\ub780\ub4dc\u00b7\uc2a4\uc6e8\ub374\uc73c\ub85c \ucd9c\uc7a5\uc744 \uac14\ub2e4. 5\uc6d4 19\uc77c\uc5d4 \uc815\uce58\ud6c4\uc6d0\uae08 5000\ub9cc\uc6d0\uc744 \ubbfc\uc8fc\ub2f9 \ucd08\u00b7\uc7ac\uc120 \uc758\uc6d0\ubaa8\uc784 '\ub354\uc88b\uc740\ubbf8\ub798'\uc5d0 \uc804\ub2ec\ud588\ub2e4. \uc790\uc2e0\uc758 \ubcf4\uc88c\uc9c4\ub4e4\uc5d0\uac8c\ub3c4 \ud1f4\uc9c1\uae08 \uba85\ubaa9\uc73c\ub85c \ubaa8\ub450 2200\ub9cc \uc6d0\uc744 \uacc4\uc88c \uc774\uccb4\ud588\ub2e4."} {"question": "1982\ub144 \uc778\uae30\uac00 \uc624\ub974\uc9c0\uc54a\ub294 \ub86f\ub370\uc5d0\ub300\ud55c \ubd88\ub9cc\uc73c\ub85c \uc5b4\ub5a4\ud300\uc5d0 \uc774\uc801\uc744 \uc9c0\uc6d0\ud588\ub294\uac00?", "paragraph": "\uadf8\ub7ec\ub098 1982\ub144\uc5d0 \ud314\uafc8\uce58\uc5d0 \uc774\uc0c1\uc774 \uc0dd\uaca8 \uc5ec\ub7ec\uac00\uc9c0 \uce58\ub8cc\ubc95\uc744 \ubcd1\ud589\ud558\uc600\uace0 \ubbf8\uc57c\ubaa8\ud1a0 \ubb34\uc0ac\uc2dc\uc758 \u2018\uc624\ub95c\uc11c\u2019\ub97c \uc560\ub3c5\ud588\ub2e4. \uc640\uce74\uc57c\ub9c8\ud604 \uc2dc\ub77c\ud558\ub9c8 \uc815\uc758 \uc2dd\uc218\uc6d0(\uc57d\uc218\ud130)\uc778 \uc4f0\uc988\ub77c\ud6c4\uce58\uc5d0\uc11c \uc88c\uc120\uc744 \ud558\uba70 \uc2ec\uc57c\uc5d0 \ud770 \uc637\uc744 \uac78\uce58\uace0 \ud3ed\ud3ec\uc218\ub97c \ub9de\uc73c\uba70 \uc218\ud589\uc744 \ud558\uae30\ub3c4 \ud588\ub2e4. \ub610\ud55c \uac19\uc740 \ud574 \uc624\ud504 \uc2dc\uc98c\uc5d0\ub294 \uc778\uae30\uac00 \uc804\ud600 \uc624\ub974\uc9c0 \uc54a\ub294 \ub86f\ub370\uc5d0 \ub300\ud55c \ubd88\ub9cc\uc73c\ub85c \uc13c\ud2b8\ub7f4 \ub9ac\uadf8\uc758 \uc778\uae30 \uad6c\ub2e8\uc778 \ud55c\uc2e0 \ud0c0\uc774\uac70\uc2a4\uc758 \uc774\uc801\uc744 \uc9c0\uc6d0\ud588\uc9c0\ub9cc \ub9de\uad50\ud658\uc5d0 \ud544\uc694\ud55c \uc778\uc6d0\uc774 \uc815\ud574\uc9c0\uc9c0 \uc54a\uc544 \uacb0\uad6d \ub86f\ub370\uc5d0 \uc794\ub958\ud588\ub2e4. \uc774\ub4ec\ud574 1983\ub144\uc5d0\ub3c4 \ud314\uafc8\uce58\uc758 \uc99d\uc0c1\uc774 \ub098\uc544\uc9c0\uc9c0 \uc54a\uc544 \ubbf8\uad6d\uc73c\ub85c \ub3c4\ubbf8\ud574 \uc2a4\ud3ec\uce20 \uc758\ud559\uc758 \uad8c\uc704\uc788\ub294 \ud559\uc790\ub85c \uc54c\ub824\uc9c4 \ud504\ub7ad\ud06c \uc870\ube0c\uc758 \uc9d1\ub3c4\ud558\uc5d0 \uc67c\ud314\uc758 \ud798\uc904\uc744 \uc624\ub978\ucabd \ud314\uafc8\uce58\uc5d0 \uc774\uc2dd\ud558\ub294 \uc218\uc220(\ud1a0\ubbf8 \uc874 \uc218\uc220)\uc744 \ubc1b\uc558\ub2e4. \uc774\ud6c4 2\ub144 \uac04\uc744 \uc7ac\ud65c\ud6c8\ub828\uc5d0\ub9cc \uc804\ub150\ud574 1984\ub144 \uc2dc\uc98c \uc885\ubc18\uc5d0 \ubcf5\uadc0\ud588\ub2e4.", "answer": "\ud0c0\uc774\uac70\uc2a4", "sentence": "\uac19\uc740 \ud574 \uc624\ud504 \uc2dc\uc98c\uc5d0\ub294 \uc778\uae30\uac00 \uc804\ud600 \uc624\ub974\uc9c0 \uc54a\ub294 \ub86f\ub370\uc5d0 \ub300\ud55c \ubd88\ub9cc\uc73c\ub85c \uc13c\ud2b8\ub7f4 \ub9ac\uadf8\uc758 \uc778\uae30 \uad6c\ub2e8\uc778 \ud55c\uc2e0 \ud0c0\uc774\uac70\uc2a4 \uc758 \uc774\uc801\uc744 \uc9c0\uc6d0\ud588\uc9c0\ub9cc \ub9de\uad50\ud658\uc5d0 \ud544\uc694\ud55c \uc778\uc6d0\uc774 \uc815\ud574\uc9c0\uc9c0 \uc54a\uc544 \uacb0\uad6d \ub86f\ub370\uc5d0 \uc794\ub958\ud588\ub2e4.", "paragraph_sentence": "\uadf8\ub7ec\ub098 1982\ub144\uc5d0 \ud314\uafc8\uce58\uc5d0 \uc774\uc0c1\uc774 \uc0dd\uaca8 \uc5ec\ub7ec\uac00\uc9c0 \uce58\ub8cc\ubc95\uc744 \ubcd1\ud589\ud558\uc600\uace0 \ubbf8\uc57c\ubaa8\ud1a0 \ubb34\uc0ac\uc2dc\uc758 \u2018\uc624\ub95c\uc11c\u2019\ub97c \uc560\ub3c5\ud588\ub2e4. \uc640\uce74\uc57c\ub9c8\ud604 \uc2dc\ub77c\ud558\ub9c8 \uc815\uc758 \uc2dd\uc218\uc6d0(\uc57d\uc218\ud130)\uc778 \uc4f0\uc988\ub77c\ud6c4\uce58\uc5d0\uc11c \uc88c\uc120\uc744 \ud558\uba70 \uc2ec\uc57c\uc5d0 \ud770 \uc637\uc744 \uac78\uce58\uace0 \ud3ed\ud3ec\uc218\ub97c \ub9de\uc73c\uba70 \uc218\ud589\uc744 \ud558\uae30\ub3c4 \ud588\ub2e4. \ub610\ud55c \uac19\uc740 \ud574 \uc624\ud504 \uc2dc\uc98c\uc5d0\ub294 \uc778\uae30\uac00 \uc804\ud600 \uc624\ub974\uc9c0 \uc54a\ub294 \ub86f\ub370\uc5d0 \ub300\ud55c \ubd88\ub9cc\uc73c\ub85c \uc13c\ud2b8\ub7f4 \ub9ac\uadf8\uc758 \uc778\uae30 \uad6c\ub2e8\uc778 \ud55c\uc2e0 \ud0c0\uc774\uac70\uc2a4 \uc758 \uc774\uc801\uc744 \uc9c0\uc6d0\ud588\uc9c0\ub9cc \ub9de\uad50\ud658\uc5d0 \ud544\uc694\ud55c \uc778\uc6d0\uc774 \uc815\ud574\uc9c0\uc9c0 \uc54a\uc544 \uacb0\uad6d \ub86f\ub370\uc5d0 \uc794\ub958\ud588\ub2e4. \uc774\ub4ec\ud574 1983\ub144\uc5d0\ub3c4 \ud314\uafc8\uce58\uc758 \uc99d\uc0c1\uc774 \ub098\uc544\uc9c0\uc9c0 \uc54a\uc544 \ubbf8\uad6d\uc73c\ub85c \ub3c4\ubbf8\ud574 \uc2a4\ud3ec\uce20 \uc758\ud559\uc758 \uad8c\uc704\uc788\ub294 \ud559\uc790\ub85c \uc54c\ub824\uc9c4 \ud504\ub7ad\ud06c \uc870\ube0c\uc758 \uc9d1\ub3c4\ud558\uc5d0 \uc67c\ud314\uc758 \ud798\uc904\uc744 \uc624\ub978\ucabd \ud314\uafc8\uce58\uc5d0 \uc774\uc2dd\ud558\ub294 \uc218\uc220(\ud1a0\ubbf8 \uc874 \uc218\uc220)\uc744 \ubc1b\uc558\ub2e4. \uc774\ud6c4 2\ub144 \uac04\uc744 \uc7ac\ud65c\ud6c8\ub828\uc5d0\ub9cc \uc804\ub150\ud574 1984\ub144 \uc2dc\uc98c \uc885\ubc18\uc5d0 \ubcf5\uadc0\ud588\ub2e4.", "paragraph_answer": "\uadf8\ub7ec\ub098 1982\ub144\uc5d0 \ud314\uafc8\uce58\uc5d0 \uc774\uc0c1\uc774 \uc0dd\uaca8 \uc5ec\ub7ec\uac00\uc9c0 \uce58\ub8cc\ubc95\uc744 \ubcd1\ud589\ud558\uc600\uace0 \ubbf8\uc57c\ubaa8\ud1a0 \ubb34\uc0ac\uc2dc\uc758 \u2018\uc624\ub95c\uc11c\u2019\ub97c \uc560\ub3c5\ud588\ub2e4. \uc640\uce74\uc57c\ub9c8\ud604 \uc2dc\ub77c\ud558\ub9c8 \uc815\uc758 \uc2dd\uc218\uc6d0(\uc57d\uc218\ud130)\uc778 \uc4f0\uc988\ub77c\ud6c4\uce58\uc5d0\uc11c \uc88c\uc120\uc744 \ud558\uba70 \uc2ec\uc57c\uc5d0 \ud770 \uc637\uc744 \uac78\uce58\uace0 \ud3ed\ud3ec\uc218\ub97c \ub9de\uc73c\uba70 \uc218\ud589\uc744 \ud558\uae30\ub3c4 \ud588\ub2e4. \ub610\ud55c \uac19\uc740 \ud574 \uc624\ud504 \uc2dc\uc98c\uc5d0\ub294 \uc778\uae30\uac00 \uc804\ud600 \uc624\ub974\uc9c0 \uc54a\ub294 \ub86f\ub370\uc5d0 \ub300\ud55c \ubd88\ub9cc\uc73c\ub85c \uc13c\ud2b8\ub7f4 \ub9ac\uadf8\uc758 \uc778\uae30 \uad6c\ub2e8\uc778 \ud55c\uc2e0 \ud0c0\uc774\uac70\uc2a4 \uc758 \uc774\uc801\uc744 \uc9c0\uc6d0\ud588\uc9c0\ub9cc \ub9de\uad50\ud658\uc5d0 \ud544\uc694\ud55c \uc778\uc6d0\uc774 \uc815\ud574\uc9c0\uc9c0 \uc54a\uc544 \uacb0\uad6d \ub86f\ub370\uc5d0 \uc794\ub958\ud588\ub2e4. \uc774\ub4ec\ud574 1983\ub144\uc5d0\ub3c4 \ud314\uafc8\uce58\uc758 \uc99d\uc0c1\uc774 \ub098\uc544\uc9c0\uc9c0 \uc54a\uc544 \ubbf8\uad6d\uc73c\ub85c \ub3c4\ubbf8\ud574 \uc2a4\ud3ec\uce20 \uc758\ud559\uc758 \uad8c\uc704\uc788\ub294 \ud559\uc790\ub85c \uc54c\ub824\uc9c4 \ud504\ub7ad\ud06c \uc870\ube0c\uc758 \uc9d1\ub3c4\ud558\uc5d0 \uc67c\ud314\uc758 \ud798\uc904\uc744 \uc624\ub978\ucabd \ud314\uafc8\uce58\uc5d0 \uc774\uc2dd\ud558\ub294 \uc218\uc220(\ud1a0\ubbf8 \uc874 \uc218\uc220)\uc744 \ubc1b\uc558\ub2e4. \uc774\ud6c4 2\ub144 \uac04\uc744 \uc7ac\ud65c\ud6c8\ub828\uc5d0\ub9cc \uc804\ub150\ud574 1984\ub144 \uc2dc\uc98c \uc885\ubc18\uc5d0 \ubcf5\uadc0\ud588\ub2e4.", "sentence_answer": "\uac19\uc740 \ud574 \uc624\ud504 \uc2dc\uc98c\uc5d0\ub294 \uc778\uae30\uac00 \uc804\ud600 \uc624\ub974\uc9c0 \uc54a\ub294 \ub86f\ub370\uc5d0 \ub300\ud55c \ubd88\ub9cc\uc73c\ub85c \uc13c\ud2b8\ub7f4 \ub9ac\uadf8\uc758 \uc778\uae30 \uad6c\ub2e8\uc778 \ud55c\uc2e0 \ud0c0\uc774\uac70\uc2a4 \uc758 \uc774\uc801\uc744 \uc9c0\uc6d0\ud588\uc9c0\ub9cc \ub9de\uad50\ud658\uc5d0 \ud544\uc694\ud55c \uc778\uc6d0\uc774 \uc815\ud574\uc9c0\uc9c0 \uc54a\uc544 \uacb0\uad6d \ub86f\ub370\uc5d0 \uc794\ub958\ud588\ub2e4.", "paragraph_id": "6410644-4-1", "paragraph_question": "question: 1982\ub144 \uc778\uae30\uac00 \uc624\ub974\uc9c0\uc54a\ub294 \ub86f\ub370\uc5d0\ub300\ud55c \ubd88\ub9cc\uc73c\ub85c \uc5b4\ub5a4\ud300\uc5d0 \uc774\uc801\uc744 \uc9c0\uc6d0\ud588\ub294\uac00?, context: \uadf8\ub7ec\ub098 1982\ub144\uc5d0 \ud314\uafc8\uce58\uc5d0 \uc774\uc0c1\uc774 \uc0dd\uaca8 \uc5ec\ub7ec\uac00\uc9c0 \uce58\ub8cc\ubc95\uc744 \ubcd1\ud589\ud558\uc600\uace0 \ubbf8\uc57c\ubaa8\ud1a0 \ubb34\uc0ac\uc2dc\uc758 \u2018\uc624\ub95c\uc11c\u2019\ub97c \uc560\ub3c5\ud588\ub2e4. \uc640\uce74\uc57c\ub9c8\ud604 \uc2dc\ub77c\ud558\ub9c8 \uc815\uc758 \uc2dd\uc218\uc6d0(\uc57d\uc218\ud130)\uc778 \uc4f0\uc988\ub77c\ud6c4\uce58\uc5d0\uc11c \uc88c\uc120\uc744 \ud558\uba70 \uc2ec\uc57c\uc5d0 \ud770 \uc637\uc744 \uac78\uce58\uace0 \ud3ed\ud3ec\uc218\ub97c \ub9de\uc73c\uba70 \uc218\ud589\uc744 \ud558\uae30\ub3c4 \ud588\ub2e4. \ub610\ud55c \uac19\uc740 \ud574 \uc624\ud504 \uc2dc\uc98c\uc5d0\ub294 \uc778\uae30\uac00 \uc804\ud600 \uc624\ub974\uc9c0 \uc54a\ub294 \ub86f\ub370\uc5d0 \ub300\ud55c \ubd88\ub9cc\uc73c\ub85c \uc13c\ud2b8\ub7f4 \ub9ac\uadf8\uc758 \uc778\uae30 \uad6c\ub2e8\uc778 \ud55c\uc2e0 \ud0c0\uc774\uac70\uc2a4\uc758 \uc774\uc801\uc744 \uc9c0\uc6d0\ud588\uc9c0\ub9cc \ub9de\uad50\ud658\uc5d0 \ud544\uc694\ud55c \uc778\uc6d0\uc774 \uc815\ud574\uc9c0\uc9c0 \uc54a\uc544 \uacb0\uad6d \ub86f\ub370\uc5d0 \uc794\ub958\ud588\ub2e4. \uc774\ub4ec\ud574 1983\ub144\uc5d0\ub3c4 \ud314\uafc8\uce58\uc758 \uc99d\uc0c1\uc774 \ub098\uc544\uc9c0\uc9c0 \uc54a\uc544 \ubbf8\uad6d\uc73c\ub85c \ub3c4\ubbf8\ud574 \uc2a4\ud3ec\uce20 \uc758\ud559\uc758 \uad8c\uc704\uc788\ub294 \ud559\uc790\ub85c \uc54c\ub824\uc9c4 \ud504\ub7ad\ud06c \uc870\ube0c\uc758 \uc9d1\ub3c4\ud558\uc5d0 \uc67c\ud314\uc758 \ud798\uc904\uc744 \uc624\ub978\ucabd \ud314\uafc8\uce58\uc5d0 \uc774\uc2dd\ud558\ub294 \uc218\uc220(\ud1a0\ubbf8 \uc874 \uc218\uc220)\uc744 \ubc1b\uc558\ub2e4. \uc774\ud6c4 2\ub144 \uac04\uc744 \uc7ac\ud65c\ud6c8\ub828\uc5d0\ub9cc \uc804\ub150\ud574 1984\ub144 \uc2dc\uc98c \uc885\ubc18\uc5d0 \ubcf5\uadc0\ud588\ub2e4."} {"question": "2003\ub144 8\uc6d4 \ud300\uc744 \ud574\uccb4\ud558\uac8c \ub41c \uc774\uc720\ub294?", "paragraph": "2003\ub144 \ud558\ud604\uc6b0, \uc804\uaddc\ud638, \uc774\uc815\uae38\uc740 \ubc34\ub4dc\uba85\uc744 The C.O.M(\ub354 \ucef4)\uc73c\ub85c \ubcc0\uacbd\ud55c\ub2e4. \ubc15\ucca0\uc601\uc744 \uc0c8\ub85c\uc6b4 \ubca0\uc774\uc2dc\uc2a4\ud2b8\ub85c \uc601\uc785\ud558\uace0 \ub77c\uc774\ube0c\ud074\ub7fd \uc2ac\ub7ec\uac70, \uc7ac\uba38\uc2a4 \ub4f1\uc9c0\uc5d0\uc11c \uacf5\uc5f0\uc744 \ud558\uba70 \ud65c\ub3d9\uc744 \uc2dc\uc791\ud55c\ub2e4. \uadf8\ub7ec\ub098 \uadf8\ud574 8\uc6d4, \uad70\uc785\ub300\ubb38\uc81c\ub85c \uc778\ud574 \ud300\uc740 \ud574\uccb4\ud558\uac8c \ub41c\ub2e4. 2003\ub144 10\uc6d4 5\uc77c \uc308\uc9c0 \uc0ac\uc6b4\ub4dc \ud398\uc2a4\ud2f0\ubc8c \uc228\uc740\uace0\uc218\uc5d0 \uc120\uc815\ub41c\ub2e4. \ud300\uc740 \uc774\ubbf8 \ud574\uccb4\ud55c \uc0c1\ud0dc\uc600\uc9c0\ub9cc \uc308\uc9c0 \uc0ac\uc6b4\ub4dc \ud398\uc2a4\ud2f0\ubc8c\uc744 \ub9c8\uc9c0\ub9c9\uc73c\ub85c \ubb34\ub300\uc5d0 \uc624\ub974\uac8c \ub41c\ub2e4. 2004\ub144 \ud558\ud604\uc6b0, \uc774\uc815\uae38\uc740 \uad70\uc785\ub300\ub97c \ud558\uc600\uace0 \uc804\uaddc\ud638\ub294 \uace0\ud5a5\uc778 \uac15\uc6d0\ub3c4\ub85c \ub3cc\uc544\uac04\ub2e4. 2006\ub144 4\uc6d4, \ud558\ud604\uc6b0\ub294 \uad70\uc81c\ub300\ub97c \ud558\uace0 \uc804\uaddc\ud638\ub97c \ub9cc\ub098 \ubc34\ub4dc\ub97c \ub2e4\uc2dc \ud558\uae30\ub85c\ud55c\ub2e4. \uac15\uc6d0\ub3c4\uc5d0 \uc788\ub294 \uc804\uaddc\ud638\uc758 \ubd80\ubaa8\ub2d8\uc774 \uc6b4\uc601\ud558\ub294 \ud39c\uc158\uc5d0\uc11c \ud568\uaed8 \uc0b4\uba74\uc11c 7\uac1c\uc6d4 \ub3d9\uc548 \uace1\uc744 \uc4f0\uac8c \ub41c\ub2e4.", "answer": "\uad70\uc785\ub300\ubb38\uc81c", "sentence": "\uadf8\ub7ec\ub098 \uadf8\ud574 8\uc6d4, \uad70\uc785\ub300\ubb38\uc81c \ub85c \uc778\ud574 \ud300\uc740 \ud574\uccb4\ud558\uac8c \ub41c\ub2e4.", "paragraph_sentence": "2003\ub144 \ud558\ud604\uc6b0, \uc804\uaddc\ud638, \uc774\uc815\uae38\uc740 \ubc34\ub4dc\uba85\uc744 The C.O.M(\ub354 \ucef4)\uc73c\ub85c \ubcc0\uacbd\ud55c\ub2e4. \ubc15\ucca0\uc601\uc744 \uc0c8\ub85c\uc6b4 \ubca0\uc774\uc2dc\uc2a4\ud2b8\ub85c \uc601\uc785\ud558\uace0 \ub77c\uc774\ube0c\ud074\ub7fd \uc2ac\ub7ec\uac70, \uc7ac\uba38\uc2a4 \ub4f1\uc9c0\uc5d0\uc11c \uacf5\uc5f0\uc744 \ud558\uba70 \ud65c\ub3d9\uc744 \uc2dc\uc791\ud55c\ub2e4. \uadf8\ub7ec\ub098 \uadf8\ud574 8\uc6d4, \uad70\uc785\ub300\ubb38\uc81c \ub85c \uc778\ud574 \ud300\uc740 \ud574\uccb4\ud558\uac8c \ub41c\ub2e4. 2003\ub144 10\uc6d4 5\uc77c \uc308\uc9c0 \uc0ac\uc6b4\ub4dc \ud398\uc2a4\ud2f0\ubc8c \uc228\uc740\uace0\uc218\uc5d0 \uc120\uc815\ub41c\ub2e4. \ud300\uc740 \uc774\ubbf8 \ud574\uccb4\ud55c \uc0c1\ud0dc\uc600\uc9c0\ub9cc \uc308\uc9c0 \uc0ac\uc6b4\ub4dc \ud398\uc2a4\ud2f0\ubc8c\uc744 \ub9c8\uc9c0\ub9c9\uc73c\ub85c \ubb34\ub300\uc5d0 \uc624\ub974\uac8c \ub41c\ub2e4. 2004\ub144 \ud558\ud604\uc6b0, \uc774\uc815\uae38\uc740 \uad70\uc785\ub300\ub97c \ud558\uc600\uace0 \uc804\uaddc\ud638\ub294 \uace0\ud5a5\uc778 \uac15\uc6d0\ub3c4\ub85c \ub3cc\uc544\uac04\ub2e4. 2006\ub144 4\uc6d4, \ud558\ud604\uc6b0\ub294 \uad70\uc81c\ub300\ub97c \ud558\uace0 \uc804\uaddc\ud638\ub97c \ub9cc\ub098 \ubc34\ub4dc\ub97c \ub2e4\uc2dc \ud558\uae30\ub85c\ud55c\ub2e4. \uac15\uc6d0\ub3c4\uc5d0 \uc788\ub294 \uc804\uaddc\ud638\uc758 \ubd80\ubaa8\ub2d8\uc774 \uc6b4\uc601\ud558\ub294 \ud39c\uc158\uc5d0\uc11c \ud568\uaed8 \uc0b4\uba74\uc11c 7\uac1c\uc6d4 \ub3d9\uc548 \uace1\uc744 \uc4f0\uac8c \ub41c\ub2e4.", "paragraph_answer": "2003\ub144 \ud558\ud604\uc6b0, \uc804\uaddc\ud638, \uc774\uc815\uae38\uc740 \ubc34\ub4dc\uba85\uc744 The C.O.M(\ub354 \ucef4)\uc73c\ub85c \ubcc0\uacbd\ud55c\ub2e4. \ubc15\ucca0\uc601\uc744 \uc0c8\ub85c\uc6b4 \ubca0\uc774\uc2dc\uc2a4\ud2b8\ub85c \uc601\uc785\ud558\uace0 \ub77c\uc774\ube0c\ud074\ub7fd \uc2ac\ub7ec\uac70, \uc7ac\uba38\uc2a4 \ub4f1\uc9c0\uc5d0\uc11c \uacf5\uc5f0\uc744 \ud558\uba70 \ud65c\ub3d9\uc744 \uc2dc\uc791\ud55c\ub2e4. \uadf8\ub7ec\ub098 \uadf8\ud574 8\uc6d4, \uad70\uc785\ub300\ubb38\uc81c \ub85c \uc778\ud574 \ud300\uc740 \ud574\uccb4\ud558\uac8c \ub41c\ub2e4. 2003\ub144 10\uc6d4 5\uc77c \uc308\uc9c0 \uc0ac\uc6b4\ub4dc \ud398\uc2a4\ud2f0\ubc8c \uc228\uc740\uace0\uc218\uc5d0 \uc120\uc815\ub41c\ub2e4. \ud300\uc740 \uc774\ubbf8 \ud574\uccb4\ud55c \uc0c1\ud0dc\uc600\uc9c0\ub9cc \uc308\uc9c0 \uc0ac\uc6b4\ub4dc \ud398\uc2a4\ud2f0\ubc8c\uc744 \ub9c8\uc9c0\ub9c9\uc73c\ub85c \ubb34\ub300\uc5d0 \uc624\ub974\uac8c \ub41c\ub2e4. 2004\ub144 \ud558\ud604\uc6b0, \uc774\uc815\uae38\uc740 \uad70\uc785\ub300\ub97c \ud558\uc600\uace0 \uc804\uaddc\ud638\ub294 \uace0\ud5a5\uc778 \uac15\uc6d0\ub3c4\ub85c \ub3cc\uc544\uac04\ub2e4. 2006\ub144 4\uc6d4, \ud558\ud604\uc6b0\ub294 \uad70\uc81c\ub300\ub97c \ud558\uace0 \uc804\uaddc\ud638\ub97c \ub9cc\ub098 \ubc34\ub4dc\ub97c \ub2e4\uc2dc \ud558\uae30\ub85c\ud55c\ub2e4. \uac15\uc6d0\ub3c4\uc5d0 \uc788\ub294 \uc804\uaddc\ud638\uc758 \ubd80\ubaa8\ub2d8\uc774 \uc6b4\uc601\ud558\ub294 \ud39c\uc158\uc5d0\uc11c \ud568\uaed8 \uc0b4\uba74\uc11c 7\uac1c\uc6d4 \ub3d9\uc548 \uace1\uc744 \uc4f0\uac8c \ub41c\ub2e4.", "sentence_answer": "\uadf8\ub7ec\ub098 \uadf8\ud574 8\uc6d4, \uad70\uc785\ub300\ubb38\uc81c \ub85c \uc778\ud574 \ud300\uc740 \ud574\uccb4\ud558\uac8c \ub41c\ub2e4.", "paragraph_id": "6480662-1-1", "paragraph_question": "question: 2003\ub144 8\uc6d4 \ud300\uc744 \ud574\uccb4\ud558\uac8c \ub41c \uc774\uc720\ub294?, context: 2003\ub144 \ud558\ud604\uc6b0, \uc804\uaddc\ud638, \uc774\uc815\uae38\uc740 \ubc34\ub4dc\uba85\uc744 The C.O.M(\ub354 \ucef4)\uc73c\ub85c \ubcc0\uacbd\ud55c\ub2e4. \ubc15\ucca0\uc601\uc744 \uc0c8\ub85c\uc6b4 \ubca0\uc774\uc2dc\uc2a4\ud2b8\ub85c \uc601\uc785\ud558\uace0 \ub77c\uc774\ube0c\ud074\ub7fd \uc2ac\ub7ec\uac70, \uc7ac\uba38\uc2a4 \ub4f1\uc9c0\uc5d0\uc11c \uacf5\uc5f0\uc744 \ud558\uba70 \ud65c\ub3d9\uc744 \uc2dc\uc791\ud55c\ub2e4. \uadf8\ub7ec\ub098 \uadf8\ud574 8\uc6d4, \uad70\uc785\ub300\ubb38\uc81c\ub85c \uc778\ud574 \ud300\uc740 \ud574\uccb4\ud558\uac8c \ub41c\ub2e4. 2003\ub144 10\uc6d4 5\uc77c \uc308\uc9c0 \uc0ac\uc6b4\ub4dc \ud398\uc2a4\ud2f0\ubc8c \uc228\uc740\uace0\uc218\uc5d0 \uc120\uc815\ub41c\ub2e4. \ud300\uc740 \uc774\ubbf8 \ud574\uccb4\ud55c \uc0c1\ud0dc\uc600\uc9c0\ub9cc \uc308\uc9c0 \uc0ac\uc6b4\ub4dc \ud398\uc2a4\ud2f0\ubc8c\uc744 \ub9c8\uc9c0\ub9c9\uc73c\ub85c \ubb34\ub300\uc5d0 \uc624\ub974\uac8c \ub41c\ub2e4. 2004\ub144 \ud558\ud604\uc6b0, \uc774\uc815\uae38\uc740 \uad70\uc785\ub300\ub97c \ud558\uc600\uace0 \uc804\uaddc\ud638\ub294 \uace0\ud5a5\uc778 \uac15\uc6d0\ub3c4\ub85c \ub3cc\uc544\uac04\ub2e4. 2006\ub144 4\uc6d4, \ud558\ud604\uc6b0\ub294 \uad70\uc81c\ub300\ub97c \ud558\uace0 \uc804\uaddc\ud638\ub97c \ub9cc\ub098 \ubc34\ub4dc\ub97c \ub2e4\uc2dc \ud558\uae30\ub85c\ud55c\ub2e4. \uac15\uc6d0\ub3c4\uc5d0 \uc788\ub294 \uc804\uaddc\ud638\uc758 \ubd80\ubaa8\ub2d8\uc774 \uc6b4\uc601\ud558\ub294 \ud39c\uc158\uc5d0\uc11c \ud568\uaed8 \uc0b4\uba74\uc11c 7\uac1c\uc6d4 \ub3d9\uc548 \uace1\uc744 \uc4f0\uac8c \ub41c\ub2e4."} {"question": "\uc911\uc138 \uc608\uc220\uc758 \ub9d0\uae30\ub97c \uc7a5\uc2dd\ud558\ub294 \uc790\uc5f0\uc8fc\uc758\ub85c \ub098\uc544\uac04 \uacfc\ud559\uc801\uc774\uace0 \ud569\ub9ac\uc801\uc778 \uc815\uc2e0\uc744 \ubd88\ub7ec\uc77c\uc73c\ud0a4\ub294 \uc0ac\uc0c1\uc740?", "paragraph": "15\uc138\uae30 \ucd08\uc5fd, \uadf8 \ub54c\uae4c\uc9c0 \uadf8 \ubcf8\uc9c8\uc5d0 \uc788\uc5b4\uc11c \uc911\uc138\uc801 \uc591\uc2dd\uc744 \ub2f5\uc2b5\ud558\uace0 \uc788\ub358 \uc774\ud0c8\ub9ac\uc544 \uc870\uac01\uc740 \uc885\ub798\uc5d0 \ubcfc \uc218 \uc5c6\uc5c8\ub358 \ub450 \uac1c\ub150, \uc989 \uace0\uc804 \ud615\uc2dd\uc758 \uc7ac\ud604\uacfc \uc801\uadf9\uc801\uc778 \uc0ac\uc2e4(\u5beb\u5be6) \ud45c\ud604\uc73c\ub85c \ubcc0\ud558\uae30 \uc2dc\uc791\ud558\uc600\ub2e4. \uace0\uc804 \uc608\uc220\uc5d0 \ub300\ud55c \uad00\uc2ec\uc740 \ub974\ub124\uc0c1\uc2a4\uc758 \ud734\uba38\ub2c8\uc998\uc774 \uc778\uac04\uc744 \ub9cc\ubb3c\uc758 \ucc99\ub3c4\ub85c\uc11c \uc911\uc2dc\ud558\ub294 \uace0\ub300 \uc0ac\uc0c1\uacfc \uadf8 \uc720\uc5f0\uc131\uc744 \uac00\uc9c0\uace0 \uc788\ub2e4\ub294 \uc0ac\uc2e4\ub85c\ubd80\ud130 \uc720\ub798\ud55c\ub2e4. \uadf8 \ubc18\uba74\uc5d0 \ud734\uba38\ub2c8\uc998\uc740 \uacfc\ud559\uc801\uc774\uace0 \ud569\ub9ac\uc801\uc778 \uc815\uc2e0\uc744 \ubd88\ub7ec\uc77c\uc73c\ucf1c, \ubd81\ubc29\uc5d0 \uc788\uc5b4\uc11c \uc911\uc138 \uc608\uc220\uc758 \ub9d0\uae30\ub97c \uc7a5\uc2dd\ud558\ub294 \uc790\uc5f0\uc8fc\uc758\ub85c \ub098\uc544\uac14\ub2e4. \uac19\uc740 \uc790\uc5f0\uc8fc\uc758\ub77c \ud560\uc9c0\ub77c\ub3c4 15\uc138\uae30\uc5d0 \uc788\uc5b4\uc11c \uc790\uc5f0\uc5d0\uc758 \uc811\uadfc\uc740 \uc911\uc138\uc640 \ub2ec\ub77c\uc11c \uc9c1\uad00\uc801\uc774\uae30\ubcf4\ub2e4\ub294 \uacfc\ud559\uc801\uc774\uc5c8\uace0, \uc885\ud569\uc801\uc774\uae30\ubcf4\ub2e4 \ubd84\uc11d\uc801\uc774\uba70, \uc2e0\uc758 \uc9c8\uc11c\ub97c \uc0c1\uc9d5\ud558\ub294 \uac83\ubcf4\ub2e4 \uc790\uc5f0 \ubc14\ub85c \uadf8\uac83\uc744 \uc704\ud558\uace0, \uc138\uacc4\uc758 \uae30\uc874 \uc0ac\uc2e4\uc744 \uc5f0\uad6c\ud558\ub294 \uac83\uc774\uc5c8\ub2e4. \ub098\uccb4\uc0c1\uc774 \ub610\ub2e4\uc2dc \uc8fc\uc81c\ub85c \ucc44\uc6a9\ub41c \uac83\uc740 \uadf8 \ud45c\ud604\uc774\ub2e4.", "answer": "\ud734\uba38\ub2c8\uc998", "sentence": "\uace0\uc804 \uc608\uc220\uc5d0 \ub300\ud55c \uad00\uc2ec\uc740 \ub974\ub124\uc0c1\uc2a4\uc758 \ud734\uba38\ub2c8\uc998 \uc774 \uc778\uac04\uc744 \ub9cc\ubb3c\uc758 \ucc99\ub3c4\ub85c\uc11c \uc911\uc2dc\ud558\ub294 \uace0\ub300 \uc0ac\uc0c1\uacfc \uadf8 \uc720\uc5f0\uc131\uc744 \uac00\uc9c0\uace0 \uc788\ub2e4\ub294 \uc0ac\uc2e4\ub85c\ubd80\ud130 \uc720\ub798\ud55c\ub2e4.", "paragraph_sentence": "15\uc138\uae30 \ucd08\uc5fd, \uadf8 \ub54c\uae4c\uc9c0 \uadf8 \ubcf8\uc9c8\uc5d0 \uc788\uc5b4\uc11c \uc911\uc138\uc801 \uc591\uc2dd\uc744 \ub2f5\uc2b5\ud558\uace0 \uc788\ub358 \uc774\ud0c8\ub9ac\uc544 \uc870\uac01\uc740 \uc885\ub798\uc5d0 \ubcfc \uc218 \uc5c6\uc5c8\ub358 \ub450 \uac1c\ub150, \uc989 \uace0\uc804 \ud615\uc2dd\uc758 \uc7ac\ud604\uacfc \uc801\uadf9\uc801\uc778 \uc0ac\uc2e4(\u5beb\u5be6) \ud45c\ud604\uc73c\ub85c \ubcc0\ud558\uae30 \uc2dc\uc791\ud558\uc600\ub2e4. \uace0\uc804 \uc608\uc220\uc5d0 \ub300\ud55c \uad00\uc2ec\uc740 \ub974\ub124\uc0c1\uc2a4\uc758 \ud734\uba38\ub2c8\uc998 \uc774 \uc778\uac04\uc744 \ub9cc\ubb3c\uc758 \ucc99\ub3c4\ub85c\uc11c \uc911\uc2dc\ud558\ub294 \uace0\ub300 \uc0ac\uc0c1\uacfc \uadf8 \uc720\uc5f0\uc131\uc744 \uac00\uc9c0\uace0 \uc788\ub2e4\ub294 \uc0ac\uc2e4\ub85c\ubd80\ud130 \uc720\ub798\ud55c\ub2e4. \uadf8 \ubc18\uba74\uc5d0 \ud734\uba38\ub2c8\uc998\uc740 \uacfc\ud559\uc801\uc774\uace0 \ud569\ub9ac\uc801\uc778 \uc815\uc2e0\uc744 \ubd88\ub7ec\uc77c\uc73c\ucf1c, \ubd81\ubc29\uc5d0 \uc788\uc5b4\uc11c \uc911\uc138 \uc608\uc220\uc758 \ub9d0\uae30\ub97c \uc7a5\uc2dd\ud558\ub294 \uc790\uc5f0\uc8fc\uc758\ub85c \ub098\uc544\uac14\ub2e4. \uac19\uc740 \uc790\uc5f0\uc8fc\uc758\ub77c \ud560\uc9c0\ub77c\ub3c4 15\uc138\uae30\uc5d0 \uc788\uc5b4\uc11c \uc790\uc5f0\uc5d0\uc758 \uc811\uadfc\uc740 \uc911\uc138\uc640 \ub2ec\ub77c\uc11c \uc9c1\uad00\uc801\uc774\uae30\ubcf4\ub2e4\ub294 \uacfc\ud559\uc801\uc774\uc5c8\uace0, \uc885\ud569\uc801\uc774\uae30\ubcf4\ub2e4 \ubd84\uc11d\uc801\uc774\uba70, \uc2e0\uc758 \uc9c8\uc11c\ub97c \uc0c1\uc9d5\ud558\ub294 \uac83\ubcf4\ub2e4 \uc790\uc5f0 \ubc14\ub85c \uadf8\uac83\uc744 \uc704\ud558\uace0, \uc138\uacc4\uc758 \uae30\uc874 \uc0ac\uc2e4\uc744 \uc5f0\uad6c\ud558\ub294 \uac83\uc774\uc5c8\ub2e4. \ub098\uccb4\uc0c1\uc774 \ub610\ub2e4\uc2dc \uc8fc\uc81c\ub85c \ucc44\uc6a9\ub41c \uac83\uc740 \uadf8 \ud45c\ud604\uc774\ub2e4.", "paragraph_answer": "15\uc138\uae30 \ucd08\uc5fd, \uadf8 \ub54c\uae4c\uc9c0 \uadf8 \ubcf8\uc9c8\uc5d0 \uc788\uc5b4\uc11c \uc911\uc138\uc801 \uc591\uc2dd\uc744 \ub2f5\uc2b5\ud558\uace0 \uc788\ub358 \uc774\ud0c8\ub9ac\uc544 \uc870\uac01\uc740 \uc885\ub798\uc5d0 \ubcfc \uc218 \uc5c6\uc5c8\ub358 \ub450 \uac1c\ub150, \uc989 \uace0\uc804 \ud615\uc2dd\uc758 \uc7ac\ud604\uacfc \uc801\uadf9\uc801\uc778 \uc0ac\uc2e4(\u5beb\u5be6) \ud45c\ud604\uc73c\ub85c \ubcc0\ud558\uae30 \uc2dc\uc791\ud558\uc600\ub2e4. \uace0\uc804 \uc608\uc220\uc5d0 \ub300\ud55c \uad00\uc2ec\uc740 \ub974\ub124\uc0c1\uc2a4\uc758 \ud734\uba38\ub2c8\uc998 \uc774 \uc778\uac04\uc744 \ub9cc\ubb3c\uc758 \ucc99\ub3c4\ub85c\uc11c \uc911\uc2dc\ud558\ub294 \uace0\ub300 \uc0ac\uc0c1\uacfc \uadf8 \uc720\uc5f0\uc131\uc744 \uac00\uc9c0\uace0 \uc788\ub2e4\ub294 \uc0ac\uc2e4\ub85c\ubd80\ud130 \uc720\ub798\ud55c\ub2e4. \uadf8 \ubc18\uba74\uc5d0 \ud734\uba38\ub2c8\uc998\uc740 \uacfc\ud559\uc801\uc774\uace0 \ud569\ub9ac\uc801\uc778 \uc815\uc2e0\uc744 \ubd88\ub7ec\uc77c\uc73c\ucf1c, \ubd81\ubc29\uc5d0 \uc788\uc5b4\uc11c \uc911\uc138 \uc608\uc220\uc758 \ub9d0\uae30\ub97c \uc7a5\uc2dd\ud558\ub294 \uc790\uc5f0\uc8fc\uc758\ub85c \ub098\uc544\uac14\ub2e4. \uac19\uc740 \uc790\uc5f0\uc8fc\uc758\ub77c \ud560\uc9c0\ub77c\ub3c4 15\uc138\uae30\uc5d0 \uc788\uc5b4\uc11c \uc790\uc5f0\uc5d0\uc758 \uc811\uadfc\uc740 \uc911\uc138\uc640 \ub2ec\ub77c\uc11c \uc9c1\uad00\uc801\uc774\uae30\ubcf4\ub2e4\ub294 \uacfc\ud559\uc801\uc774\uc5c8\uace0, \uc885\ud569\uc801\uc774\uae30\ubcf4\ub2e4 \ubd84\uc11d\uc801\uc774\uba70, \uc2e0\uc758 \uc9c8\uc11c\ub97c \uc0c1\uc9d5\ud558\ub294 \uac83\ubcf4\ub2e4 \uc790\uc5f0 \ubc14\ub85c \uadf8\uac83\uc744 \uc704\ud558\uace0, \uc138\uacc4\uc758 \uae30\uc874 \uc0ac\uc2e4\uc744 \uc5f0\uad6c\ud558\ub294 \uac83\uc774\uc5c8\ub2e4. \ub098\uccb4\uc0c1\uc774 \ub610\ub2e4\uc2dc \uc8fc\uc81c\ub85c \ucc44\uc6a9\ub41c \uac83\uc740 \uadf8 \ud45c\ud604\uc774\ub2e4.", "sentence_answer": "\uace0\uc804 \uc608\uc220\uc5d0 \ub300\ud55c \uad00\uc2ec\uc740 \ub974\ub124\uc0c1\uc2a4\uc758 \ud734\uba38\ub2c8\uc998 \uc774 \uc778\uac04\uc744 \ub9cc\ubb3c\uc758 \ucc99\ub3c4\ub85c\uc11c \uc911\uc2dc\ud558\ub294 \uace0\ub300 \uc0ac\uc0c1\uacfc \uadf8 \uc720\uc5f0\uc131\uc744 \uac00\uc9c0\uace0 \uc788\ub2e4\ub294 \uc0ac\uc2e4\ub85c\ubd80\ud130 \uc720\ub798\ud55c\ub2e4.", "paragraph_id": "6529061-2-1", "paragraph_question": "question: \uc911\uc138 \uc608\uc220\uc758 \ub9d0\uae30\ub97c \uc7a5\uc2dd\ud558\ub294 \uc790\uc5f0\uc8fc\uc758\ub85c \ub098\uc544\uac04 \uacfc\ud559\uc801\uc774\uace0 \ud569\ub9ac\uc801\uc778 \uc815\uc2e0\uc744 \ubd88\ub7ec\uc77c\uc73c\ud0a4\ub294 \uc0ac\uc0c1\uc740?, context: 15\uc138\uae30 \ucd08\uc5fd, \uadf8 \ub54c\uae4c\uc9c0 \uadf8 \ubcf8\uc9c8\uc5d0 \uc788\uc5b4\uc11c \uc911\uc138\uc801 \uc591\uc2dd\uc744 \ub2f5\uc2b5\ud558\uace0 \uc788\ub358 \uc774\ud0c8\ub9ac\uc544 \uc870\uac01\uc740 \uc885\ub798\uc5d0 \ubcfc \uc218 \uc5c6\uc5c8\ub358 \ub450 \uac1c\ub150, \uc989 \uace0\uc804 \ud615\uc2dd\uc758 \uc7ac\ud604\uacfc \uc801\uadf9\uc801\uc778 \uc0ac\uc2e4(\u5beb\u5be6) \ud45c\ud604\uc73c\ub85c \ubcc0\ud558\uae30 \uc2dc\uc791\ud558\uc600\ub2e4. \uace0\uc804 \uc608\uc220\uc5d0 \ub300\ud55c \uad00\uc2ec\uc740 \ub974\ub124\uc0c1\uc2a4\uc758 \ud734\uba38\ub2c8\uc998\uc774 \uc778\uac04\uc744 \ub9cc\ubb3c\uc758 \ucc99\ub3c4\ub85c\uc11c \uc911\uc2dc\ud558\ub294 \uace0\ub300 \uc0ac\uc0c1\uacfc \uadf8 \uc720\uc5f0\uc131\uc744 \uac00\uc9c0\uace0 \uc788\ub2e4\ub294 \uc0ac\uc2e4\ub85c\ubd80\ud130 \uc720\ub798\ud55c\ub2e4. \uadf8 \ubc18\uba74\uc5d0 \ud734\uba38\ub2c8\uc998\uc740 \uacfc\ud559\uc801\uc774\uace0 \ud569\ub9ac\uc801\uc778 \uc815\uc2e0\uc744 \ubd88\ub7ec\uc77c\uc73c\ucf1c, \ubd81\ubc29\uc5d0 \uc788\uc5b4\uc11c \uc911\uc138 \uc608\uc220\uc758 \ub9d0\uae30\ub97c \uc7a5\uc2dd\ud558\ub294 \uc790\uc5f0\uc8fc\uc758\ub85c \ub098\uc544\uac14\ub2e4. \uac19\uc740 \uc790\uc5f0\uc8fc\uc758\ub77c \ud560\uc9c0\ub77c\ub3c4 15\uc138\uae30\uc5d0 \uc788\uc5b4\uc11c \uc790\uc5f0\uc5d0\uc758 \uc811\uadfc\uc740 \uc911\uc138\uc640 \ub2ec\ub77c\uc11c \uc9c1\uad00\uc801\uc774\uae30\ubcf4\ub2e4\ub294 \uacfc\ud559\uc801\uc774\uc5c8\uace0, \uc885\ud569\uc801\uc774\uae30\ubcf4\ub2e4 \ubd84\uc11d\uc801\uc774\uba70, \uc2e0\uc758 \uc9c8\uc11c\ub97c \uc0c1\uc9d5\ud558\ub294 \uac83\ubcf4\ub2e4 \uc790\uc5f0 \ubc14\ub85c \uadf8\uac83\uc744 \uc704\ud558\uace0, \uc138\uacc4\uc758 \uae30\uc874 \uc0ac\uc2e4\uc744 \uc5f0\uad6c\ud558\ub294 \uac83\uc774\uc5c8\ub2e4. \ub098\uccb4\uc0c1\uc774 \ub610\ub2e4\uc2dc \uc8fc\uc81c\ub85c \ucc44\uc6a9\ub41c \uac83\uc740 \uadf8 \ud45c\ud604\uc774\ub2e4."} {"question": "2010\ub144 FIFA \uc6d4\ub4dc\ucef5\uc5d0 \ucc38\uc5ec\ud55c \uac00\ub098\uc758 8\uac15\uc804 \uc0c1\ub300\uad6d\uc740 \uc5b4\ub290\ub098\ub77c\uc778\uac00?", "paragraph": "\uc544\uc720\uac00 \ucc38\uc5ec\ud55c \ub450 \ubc88\uc9f8 \uba54\uc774\uc800 \uad6d\uac00\ub300\ud56d\uc804 \ub300\ud68c\ub294 2010\ub144 \uc544\ud504\ub9ac\uce74 \ub124\uc774\uc158\uc2a4\ucef5\uc774\uc5c8\ub2e4. 2010\ub144 1\uc6d4 19\uc77c, \ubd80\ub974\ud0a4\ub098\ud30c\uc18c\uc640\uc758 \uc870\ubcc4\ub9ac\uadf8 \ucd5c\uc885\uc804\uc5d0\uc11c, \uadf8\ub294 30\ubd84\uc5d0 \ud5e4\ub529\uc73c\ub85c \uccab \uad6d\uac00\ub300\ud45c\ud300 \ub4dd\uc810\uc744 \ud558\uc600\ub2e4. \uac00\ub098\ub294 \uc774 \uacbd\uae30\uc5d0\uc11c 1-0\uc73c\ub85c \uc2b9\ub9ac\ud558\uace0, \uacb0\uc2b9\uae4c\uc9c0 \uc9c4\ucd9c\ud558\uc600\uc73c\ub098, \uacb0\uc2b9\uc804\uc5d0\uc11c \uc774\uc9d1\ud2b8\uc5d0 0-1\ub85c \ud328\ud558\uc600\ub2e4. \uc544\uc720\ub294 \uc774 \ub300\ud68c\uc758 \uac00\ub098\uac00 \uad00\uc5ec\ud55c 5\uacbd\uae30\uc5d0 \ubaa8\ub450 \ucd9c\uc804\ud558\uc600\ub2e4. 2010\ub144 5\uc6d4 7\uc77c, \uc544\uc720\ub294 \ubc00\ub85c\ubc18 \ub77c\uc608\ube44\uce58 \uac00\ub098 \uac10\ub3c5\uc5d0 \uc758\ud574 2010\ub144 FIFA \uc6d4\ub4dc\ucef5\uc5d0 \ucc38\uc5ec\ud558\ub294 30\uc778 \uba85\ub2e8\uc5d0 \ud3ec\ud568\ub418\uc5b4 \uc788\uc5c8\ub2e4. \uadf8\ub294 \ub098\uc911\uc5d0 \ud615 \uc774\ube0c\ub77c\ud798 \uc544\uc720\uc640 \ud568\uaed8 \ucd5c\uc885 23\uc778 \uc5d4\ud2b8\ub9ac\uc5d0 \ud3ec\ud568\ub418\uc5c8\ub2e4. 6\uc6d4 12\uc77c, \uc544\uc720\ub294 \uc138\ub974\ube44\uc544\ub97c \uc0c1\ub300\ub85c \uacbd\ud5d8\uc774 \ub354 \ub9ce\uc740 \uc124\ub9ac \ubb38\ud0c0\ub9ac\ub97c \ub300\uc2e0\ud558\uc5ec \ucd9c\uc804\ud558\uc600\ub2e4. \uc774\ub85c\uc368, \uadf8\ub294 FIFA \uc6d4\ub4dc\ucef5 \ub370\ubdd4\uc804\uc744 \uce58\ub8e8\uc5c8\ub2e4. \uadf8\ub294 \uc774\ud6c4 \uc624\uc2a4\ud2b8\ub808\uc77c\ub9ac\uc544\uc640 \ub3c5\uc77c\uacfc\uc758 \ub098\uba38\uc9c0 \uc870\ubcc4\ub9ac\uadf8 2\uacbd\uae30\uc5d0 \ucd9c\uc7a5\ud558\uc600\uace0, \uac00\ub098\ub294 \uc8fd\uc74c\uc758 D\uc870\ub97c 2\uc704\ub85c \ud1b5\uacfc\ud558\uc600\ub2e4. \uac00\ub098\uc758 16\uac15 \uc0c1\ub300\ub294 \ubbf8\uad6d\uc73c\ub85c, \uc544\uc720\ub294 \ubbf8\uad6d \uc218\ube44\ub97c \ucc0c\ub974\uba70 \uc544\uc0ac\ubaa8\uc544 \uae30\uc548\uc758 \uacb0\uc2b9\uace8\uc744 \uc5b4\uc2dc\uc2a4\ud2b8\ud558\uc600\ub2e4. \uadf8\uc758 \uacbd\uae30\uc5d0\uc11c\uc758 \uc131\uacfc\ub85c, \uc544\uc720\ub294 FIFA\ub85c\ubd80\ud130 \uacbd\uae30 MVP\ub85c \uc120\uc815\ud558\uc600\ub2e4. \uadf8\ub7ec\ub098, \uc544\uc720\ub294 16\uac15\uc804\uc5d0\uc11c 2\ubc88\uc9f8 \uc610\ub85c\uc6b0 \uce74\ub4dc\ub97c \ubc1b\uc74c\uc5d0 \ub530\ub77c \uc6b0\ub8e8\uacfc\uc774\uc640\uc758 8\uac15\uc804\uc5d0 \ucc38\uac00\ud558\uc9c0 \ubabb\ud558\uc600\uace0, \uac00\ub098\ub294 \uc2b9\ubd80\ucc28\uae30 \ub05d\uc5d0 \ud328\ud558\uc600\ub2e4.", "answer": "\uc6b0\ub8e8\uacfc\uc774", "sentence": "\uadf8\ub7ec\ub098, \uc544\uc720\ub294 16\uac15\uc804\uc5d0\uc11c 2\ubc88\uc9f8 \uc610\ub85c\uc6b0 \uce74\ub4dc\ub97c \ubc1b\uc74c\uc5d0 \ub530\ub77c \uc6b0\ub8e8\uacfc\uc774 \uc640\uc758 8\uac15\uc804\uc5d0 \ucc38\uac00\ud558\uc9c0 \ubabb\ud558\uc600\uace0, \uac00\ub098\ub294 \uc2b9\ubd80\ucc28\uae30 \ub05d\uc5d0 \ud328\ud558\uc600\ub2e4.", "paragraph_sentence": "\uc544\uc720\uac00 \ucc38\uc5ec\ud55c \ub450 \ubc88\uc9f8 \uba54\uc774\uc800 \uad6d\uac00\ub300\ud56d\uc804 \ub300\ud68c\ub294 2010\ub144 \uc544\ud504\ub9ac\uce74 \ub124\uc774\uc158\uc2a4\ucef5\uc774\uc5c8\ub2e4. 2010\ub144 1\uc6d4 19\uc77c, \ubd80\ub974\ud0a4\ub098\ud30c\uc18c\uc640\uc758 \uc870\ubcc4\ub9ac\uadf8 \ucd5c\uc885\uc804\uc5d0\uc11c, \uadf8\ub294 30\ubd84\uc5d0 \ud5e4\ub529\uc73c\ub85c \uccab \uad6d\uac00\ub300\ud45c\ud300 \ub4dd\uc810\uc744 \ud558\uc600\ub2e4. \uac00\ub098\ub294 \uc774 \uacbd\uae30\uc5d0\uc11c 1-0\uc73c\ub85c \uc2b9\ub9ac\ud558\uace0, \uacb0\uc2b9\uae4c\uc9c0 \uc9c4\ucd9c\ud558\uc600\uc73c\ub098, \uacb0\uc2b9\uc804\uc5d0\uc11c \uc774\uc9d1\ud2b8\uc5d0 0-1\ub85c \ud328\ud558\uc600\ub2e4. \uc544\uc720\ub294 \uc774 \ub300\ud68c\uc758 \uac00\ub098\uac00 \uad00\uc5ec\ud55c 5\uacbd\uae30\uc5d0 \ubaa8\ub450 \ucd9c\uc804\ud558\uc600\ub2e4. 2010\ub144 5\uc6d4 7\uc77c, \uc544\uc720\ub294 \ubc00\ub85c\ubc18 \ub77c\uc608\ube44\uce58 \uac00\ub098 \uac10\ub3c5\uc5d0 \uc758\ud574 2010\ub144 FIFA \uc6d4\ub4dc\ucef5\uc5d0 \ucc38\uc5ec\ud558\ub294 30\uc778 \uba85\ub2e8\uc5d0 \ud3ec\ud568\ub418\uc5b4 \uc788\uc5c8\ub2e4. \uadf8\ub294 \ub098\uc911\uc5d0 \ud615 \uc774\ube0c\ub77c\ud798 \uc544\uc720\uc640 \ud568\uaed8 \ucd5c\uc885 23\uc778 \uc5d4\ud2b8\ub9ac\uc5d0 \ud3ec\ud568\ub418\uc5c8\ub2e4. 6\uc6d4 12\uc77c, \uc544\uc720\ub294 \uc138\ub974\ube44\uc544\ub97c \uc0c1\ub300\ub85c \uacbd\ud5d8\uc774 \ub354 \ub9ce\uc740 \uc124\ub9ac \ubb38\ud0c0\ub9ac\ub97c \ub300\uc2e0\ud558\uc5ec \ucd9c\uc804\ud558\uc600\ub2e4. \uc774\ub85c\uc368, \uadf8\ub294 FIFA \uc6d4\ub4dc\ucef5 \ub370\ubdd4\uc804\uc744 \uce58\ub8e8\uc5c8\ub2e4. \uadf8\ub294 \uc774\ud6c4 \uc624\uc2a4\ud2b8\ub808\uc77c\ub9ac\uc544\uc640 \ub3c5\uc77c\uacfc\uc758 \ub098\uba38\uc9c0 \uc870\ubcc4\ub9ac\uadf8 2\uacbd\uae30\uc5d0 \ucd9c\uc7a5\ud558\uc600\uace0, \uac00\ub098\ub294 \uc8fd\uc74c\uc758 D\uc870\ub97c 2\uc704\ub85c \ud1b5\uacfc\ud558\uc600\ub2e4. \uac00\ub098\uc758 16\uac15 \uc0c1\ub300\ub294 \ubbf8\uad6d\uc73c\ub85c, \uc544\uc720\ub294 \ubbf8\uad6d \uc218\ube44\ub97c \ucc0c\ub974\uba70 \uc544\uc0ac\ubaa8\uc544 \uae30\uc548\uc758 \uacb0\uc2b9\uace8\uc744 \uc5b4\uc2dc\uc2a4\ud2b8\ud558\uc600\ub2e4. \uadf8\uc758 \uacbd\uae30\uc5d0\uc11c\uc758 \uc131\uacfc\ub85c, \uc544\uc720\ub294 FIFA\ub85c\ubd80\ud130 \uacbd\uae30 MVP\ub85c \uc120\uc815\ud558\uc600\ub2e4. \uadf8\ub7ec\ub098, \uc544\uc720\ub294 16\uac15\uc804\uc5d0\uc11c 2\ubc88\uc9f8 \uc610\ub85c\uc6b0 \uce74\ub4dc\ub97c \ubc1b\uc74c\uc5d0 \ub530\ub77c \uc6b0\ub8e8\uacfc\uc774 \uc640\uc758 8\uac15\uc804\uc5d0 \ucc38\uac00\ud558\uc9c0 \ubabb\ud558\uc600\uace0, \uac00\ub098\ub294 \uc2b9\ubd80\ucc28\uae30 \ub05d\uc5d0 \ud328\ud558\uc600\ub2e4. ", "paragraph_answer": "\uc544\uc720\uac00 \ucc38\uc5ec\ud55c \ub450 \ubc88\uc9f8 \uba54\uc774\uc800 \uad6d\uac00\ub300\ud56d\uc804 \ub300\ud68c\ub294 2010\ub144 \uc544\ud504\ub9ac\uce74 \ub124\uc774\uc158\uc2a4\ucef5\uc774\uc5c8\ub2e4. 2010\ub144 1\uc6d4 19\uc77c, \ubd80\ub974\ud0a4\ub098\ud30c\uc18c\uc640\uc758 \uc870\ubcc4\ub9ac\uadf8 \ucd5c\uc885\uc804\uc5d0\uc11c, \uadf8\ub294 30\ubd84\uc5d0 \ud5e4\ub529\uc73c\ub85c \uccab \uad6d\uac00\ub300\ud45c\ud300 \ub4dd\uc810\uc744 \ud558\uc600\ub2e4. \uac00\ub098\ub294 \uc774 \uacbd\uae30\uc5d0\uc11c 1-0\uc73c\ub85c \uc2b9\ub9ac\ud558\uace0, \uacb0\uc2b9\uae4c\uc9c0 \uc9c4\ucd9c\ud558\uc600\uc73c\ub098, \uacb0\uc2b9\uc804\uc5d0\uc11c \uc774\uc9d1\ud2b8\uc5d0 0-1\ub85c \ud328\ud558\uc600\ub2e4. \uc544\uc720\ub294 \uc774 \ub300\ud68c\uc758 \uac00\ub098\uac00 \uad00\uc5ec\ud55c 5\uacbd\uae30\uc5d0 \ubaa8\ub450 \ucd9c\uc804\ud558\uc600\ub2e4. 2010\ub144 5\uc6d4 7\uc77c, \uc544\uc720\ub294 \ubc00\ub85c\ubc18 \ub77c\uc608\ube44\uce58 \uac00\ub098 \uac10\ub3c5\uc5d0 \uc758\ud574 2010\ub144 FIFA \uc6d4\ub4dc\ucef5\uc5d0 \ucc38\uc5ec\ud558\ub294 30\uc778 \uba85\ub2e8\uc5d0 \ud3ec\ud568\ub418\uc5b4 \uc788\uc5c8\ub2e4. \uadf8\ub294 \ub098\uc911\uc5d0 \ud615 \uc774\ube0c\ub77c\ud798 \uc544\uc720\uc640 \ud568\uaed8 \ucd5c\uc885 23\uc778 \uc5d4\ud2b8\ub9ac\uc5d0 \ud3ec\ud568\ub418\uc5c8\ub2e4. 6\uc6d4 12\uc77c, \uc544\uc720\ub294 \uc138\ub974\ube44\uc544\ub97c \uc0c1\ub300\ub85c \uacbd\ud5d8\uc774 \ub354 \ub9ce\uc740 \uc124\ub9ac \ubb38\ud0c0\ub9ac\ub97c \ub300\uc2e0\ud558\uc5ec \ucd9c\uc804\ud558\uc600\ub2e4. \uc774\ub85c\uc368, \uadf8\ub294 FIFA \uc6d4\ub4dc\ucef5 \ub370\ubdd4\uc804\uc744 \uce58\ub8e8\uc5c8\ub2e4. \uadf8\ub294 \uc774\ud6c4 \uc624\uc2a4\ud2b8\ub808\uc77c\ub9ac\uc544\uc640 \ub3c5\uc77c\uacfc\uc758 \ub098\uba38\uc9c0 \uc870\ubcc4\ub9ac\uadf8 2\uacbd\uae30\uc5d0 \ucd9c\uc7a5\ud558\uc600\uace0, \uac00\ub098\ub294 \uc8fd\uc74c\uc758 D\uc870\ub97c 2\uc704\ub85c \ud1b5\uacfc\ud558\uc600\ub2e4. \uac00\ub098\uc758 16\uac15 \uc0c1\ub300\ub294 \ubbf8\uad6d\uc73c\ub85c, \uc544\uc720\ub294 \ubbf8\uad6d \uc218\ube44\ub97c \ucc0c\ub974\uba70 \uc544\uc0ac\ubaa8\uc544 \uae30\uc548\uc758 \uacb0\uc2b9\uace8\uc744 \uc5b4\uc2dc\uc2a4\ud2b8\ud558\uc600\ub2e4. \uadf8\uc758 \uacbd\uae30\uc5d0\uc11c\uc758 \uc131\uacfc\ub85c, \uc544\uc720\ub294 FIFA\ub85c\ubd80\ud130 \uacbd\uae30 MVP\ub85c \uc120\uc815\ud558\uc600\ub2e4. \uadf8\ub7ec\ub098, \uc544\uc720\ub294 16\uac15\uc804\uc5d0\uc11c 2\ubc88\uc9f8 \uc610\ub85c\uc6b0 \uce74\ub4dc\ub97c \ubc1b\uc74c\uc5d0 \ub530\ub77c \uc6b0\ub8e8\uacfc\uc774 \uc640\uc758 8\uac15\uc804\uc5d0 \ucc38\uac00\ud558\uc9c0 \ubabb\ud558\uc600\uace0, \uac00\ub098\ub294 \uc2b9\ubd80\ucc28\uae30 \ub05d\uc5d0 \ud328\ud558\uc600\ub2e4.", "sentence_answer": "\uadf8\ub7ec\ub098, \uc544\uc720\ub294 16\uac15\uc804\uc5d0\uc11c 2\ubc88\uc9f8 \uc610\ub85c\uc6b0 \uce74\ub4dc\ub97c \ubc1b\uc74c\uc5d0 \ub530\ub77c \uc6b0\ub8e8\uacfc\uc774 \uc640\uc758 8\uac15\uc804\uc5d0 \ucc38\uac00\ud558\uc9c0 \ubabb\ud558\uc600\uace0, \uac00\ub098\ub294 \uc2b9\ubd80\ucc28\uae30 \ub05d\uc5d0 \ud328\ud558\uc600\ub2e4.", "paragraph_id": "6466047-7-2", "paragraph_question": "question: 2010\ub144 FIFA \uc6d4\ub4dc\ucef5\uc5d0 \ucc38\uc5ec\ud55c \uac00\ub098\uc758 8\uac15\uc804 \uc0c1\ub300\uad6d\uc740 \uc5b4\ub290\ub098\ub77c\uc778\uac00?, context: \uc544\uc720\uac00 \ucc38\uc5ec\ud55c \ub450 \ubc88\uc9f8 \uba54\uc774\uc800 \uad6d\uac00\ub300\ud56d\uc804 \ub300\ud68c\ub294 2010\ub144 \uc544\ud504\ub9ac\uce74 \ub124\uc774\uc158\uc2a4\ucef5\uc774\uc5c8\ub2e4. 2010\ub144 1\uc6d4 19\uc77c, \ubd80\ub974\ud0a4\ub098\ud30c\uc18c\uc640\uc758 \uc870\ubcc4\ub9ac\uadf8 \ucd5c\uc885\uc804\uc5d0\uc11c, \uadf8\ub294 30\ubd84\uc5d0 \ud5e4\ub529\uc73c\ub85c \uccab \uad6d\uac00\ub300\ud45c\ud300 \ub4dd\uc810\uc744 \ud558\uc600\ub2e4. \uac00\ub098\ub294 \uc774 \uacbd\uae30\uc5d0\uc11c 1-0\uc73c\ub85c \uc2b9\ub9ac\ud558\uace0, \uacb0\uc2b9\uae4c\uc9c0 \uc9c4\ucd9c\ud558\uc600\uc73c\ub098, \uacb0\uc2b9\uc804\uc5d0\uc11c \uc774\uc9d1\ud2b8\uc5d0 0-1\ub85c \ud328\ud558\uc600\ub2e4. \uc544\uc720\ub294 \uc774 \ub300\ud68c\uc758 \uac00\ub098\uac00 \uad00\uc5ec\ud55c 5\uacbd\uae30\uc5d0 \ubaa8\ub450 \ucd9c\uc804\ud558\uc600\ub2e4. 2010\ub144 5\uc6d4 7\uc77c, \uc544\uc720\ub294 \ubc00\ub85c\ubc18 \ub77c\uc608\ube44\uce58 \uac00\ub098 \uac10\ub3c5\uc5d0 \uc758\ud574 2010\ub144 FIFA \uc6d4\ub4dc\ucef5\uc5d0 \ucc38\uc5ec\ud558\ub294 30\uc778 \uba85\ub2e8\uc5d0 \ud3ec\ud568\ub418\uc5b4 \uc788\uc5c8\ub2e4. \uadf8\ub294 \ub098\uc911\uc5d0 \ud615 \uc774\ube0c\ub77c\ud798 \uc544\uc720\uc640 \ud568\uaed8 \ucd5c\uc885 23\uc778 \uc5d4\ud2b8\ub9ac\uc5d0 \ud3ec\ud568\ub418\uc5c8\ub2e4. 6\uc6d4 12\uc77c, \uc544\uc720\ub294 \uc138\ub974\ube44\uc544\ub97c \uc0c1\ub300\ub85c \uacbd\ud5d8\uc774 \ub354 \ub9ce\uc740 \uc124\ub9ac \ubb38\ud0c0\ub9ac\ub97c \ub300\uc2e0\ud558\uc5ec \ucd9c\uc804\ud558\uc600\ub2e4. \uc774\ub85c\uc368, \uadf8\ub294 FIFA \uc6d4\ub4dc\ucef5 \ub370\ubdd4\uc804\uc744 \uce58\ub8e8\uc5c8\ub2e4. \uadf8\ub294 \uc774\ud6c4 \uc624\uc2a4\ud2b8\ub808\uc77c\ub9ac\uc544\uc640 \ub3c5\uc77c\uacfc\uc758 \ub098\uba38\uc9c0 \uc870\ubcc4\ub9ac\uadf8 2\uacbd\uae30\uc5d0 \ucd9c\uc7a5\ud558\uc600\uace0, \uac00\ub098\ub294 \uc8fd\uc74c\uc758 D\uc870\ub97c 2\uc704\ub85c \ud1b5\uacfc\ud558\uc600\ub2e4. \uac00\ub098\uc758 16\uac15 \uc0c1\ub300\ub294 \ubbf8\uad6d\uc73c\ub85c, \uc544\uc720\ub294 \ubbf8\uad6d \uc218\ube44\ub97c \ucc0c\ub974\uba70 \uc544\uc0ac\ubaa8\uc544 \uae30\uc548\uc758 \uacb0\uc2b9\uace8\uc744 \uc5b4\uc2dc\uc2a4\ud2b8\ud558\uc600\ub2e4. \uadf8\uc758 \uacbd\uae30\uc5d0\uc11c\uc758 \uc131\uacfc\ub85c, \uc544\uc720\ub294 FIFA\ub85c\ubd80\ud130 \uacbd\uae30 MVP\ub85c \uc120\uc815\ud558\uc600\ub2e4. \uadf8\ub7ec\ub098, \uc544\uc720\ub294 16\uac15\uc804\uc5d0\uc11c 2\ubc88\uc9f8 \uc610\ub85c\uc6b0 \uce74\ub4dc\ub97c \ubc1b\uc74c\uc5d0 \ub530\ub77c \uc6b0\ub8e8\uacfc\uc774\uc640\uc758 8\uac15\uc804\uc5d0 \ucc38\uac00\ud558\uc9c0 \ubabb\ud558\uc600\uace0, \uac00\ub098\ub294 \uc2b9\ubd80\ucc28\uae30 \ub05d\uc5d0 \ud328\ud558\uc600\ub2e4."} {"question": "\ubc14\ub974\uc0e4\ubc14 \uacf5\uad6d\uc774 \uba78\ub9dd\ud55c \ud6c4 \ud589\uc815\uad8c\uc744 \uc591\ub3c4\ubc1b\uc740 \uacf3\uc740 \uc5b4\ub514\uc778\uac00?", "paragraph": "10\ub9cc \uba85\uc758 \ud3f4\ub780\ub4dc\uc778\uc774 \ucc38\uc804\ud588\ub358 \ub098\ud3f4\ub808\uc639\uc758 \ub7ec\uc2dc\uc544 \uc6d0\uc815\uc774 \uc2e4\ud328\ub85c \ub05d\ub098\uc790 \ub098\ud3f4\ub808\uc639\uc5d0 \uac78\uc5c8\ub358 \ud3f4\ub780\ub4dc \ubbfc\uc871\uc758 \ubaa8\ub4e0 \ud76c\ub9dd\ub3c4 \uc88c\uc808\ub418\uc5c8\ub2e4. 1813\ub144 2\uc6d4 \ubc14\ub974\uc0e4\ubc14\ub97c \uc810\ub839\ud55c \ub7ec\uc2dc\uc544 \uad70\ub300\ub294 5\uc6d4\uc5d0 \uc774\ubbf8 \uc804 \uacf5\uad6d\uc758 \uc601\ud1a0\ub97c \uc9c0\ubc30\ud558\uac8c \ub418\uc5c8\ub2e4. \uacf5\uad6d\uc774 \uba78\ub9dd\ud55c \ud6c4\uc5d0\ub3c4 \ud589\uc815\uad8c\uc740 \ub7ec\uc2dc\uc544 \uc810\ub839\uad70\uc5d0\uac8c \ub118\uc5b4\uac00\uc9c0 \uc54a\uc558\uace0 \uc0c8\ub86d\uac8c \uad6c\uc131\ub41c '5\uc778 \ucd5c\uace0\uc704\uc6d0\ud68c'\uc5d0\uac8c \uc591\ub3c4\ub418\uc5c8\ub2e4. 1815\ub144 \ube48 \ud68c\uc758\uc5d0\uc11c \ud3f4\ub780\ub4dc\ub294 \ub7ec\uc2dc\uc544, \ud504\ub85c\uc774\uc13c, \uc624\uc2a4\ud2b8\ub9ac\uc544\uc5d0 \uc758\ud574\uc11c \ub2e4\uc2dc \ubd84\ud560\ub418\uc5c8\ub2e4. 1815\ub144 6\uc6d4 9\uc77c \ube48 \ud68c\uc758 \ucd5c\uc885 \uc758\uc815\uc11c\uc5d0 \ub530\ub77c \ud504\ub85c\uc774\uc13c\uc740 2\ucc28 \ubd84\ud560\uc2dc \ud68d\ub4dd\ud588\ub358 \uc77c\ubd80 \uc9c0\uc5ed\uacfc '\ud3ec\uc988\ub09c \ub300\uacf5\uad6d' \uc744 \uc5bb\uc5c8\uace0, \uc624\uc2a4\ud2b8\ub9ac\uc544\ub294 \ube44\uc5d8\ub9ac\uce20\uce74(Wieliczka) \uc18c\uae08 \uad11\uc0b0\uacfc 1772\ub144 \ubd84\ud560\uc2dc \ucc28\uc9c0\ud588\ub358 \ub300\ubd80\ubd84\uc758 \uc9c0\uc5ed\uc744 \ub2e4\uc2dc \ucc28\uc9c0\ud588\ub2e4. \ud06c\ub77c\ucfe0\ud504\ub294 3\uad6d\uc758 \uac10\uc2dc\ud558\uc5d0 \uc790\uccb4 \ud5cc\ubc95\uc744 \uac00\uc9c4 \uc790\uc720\uc2dc\uac00 \ub418\uc5c8\ub2e4. \ub098\uba38\uc9c0 \ubc14\ub974\uc0e4\ubc14 \uacf5\uad6d\uc740 \ud5cc\ubc95\uc5d0 \uc758\ud574\uc11c \ub7ec\uc2dc\uc544\uc640 \uc601\uc6d0\ud788 \uacb0\ud569\ub418\ub294 \ud3f4\ub780\ub4dc \uc655\uad6d\uc73c\ub85c \uac1c\ud3b8\ub418\uc5c8\ub2e4. \uc774\ub807\uac8c \ub418\uc5b4 \ubc14\ub974\uc0e4\ubc14 \uacf5\uad6d\uc740 \ub7ec\uc2dc\uc544\uc640 \ud504\ub85c\uc774\uc13c\uc5d0 \uc758\ud574\uc11c \ubd84\ud560\ub418\uc5c8\ub2e4.", "answer": "5\uc778 \ucd5c\uace0\uc704\uc6d0\ud68c", "sentence": "\uacf5\uad6d\uc774 \uba78\ub9dd\ud55c \ud6c4\uc5d0\ub3c4 \ud589\uc815\uad8c\uc740 \ub7ec\uc2dc\uc544 \uc810\ub839\uad70\uc5d0\uac8c \ub118\uc5b4\uac00\uc9c0 \uc54a\uc558\uace0 \uc0c8\ub86d\uac8c \uad6c\uc131\ub41c ' 5\uc778 \ucd5c\uace0\uc704\uc6d0\ud68c '\uc5d0\uac8c \uc591\ub3c4\ub418\uc5c8\ub2e4.", "paragraph_sentence": "10\ub9cc \uba85\uc758 \ud3f4\ub780\ub4dc\uc778\uc774 \ucc38\uc804\ud588\ub358 \ub098\ud3f4\ub808\uc639\uc758 \ub7ec\uc2dc\uc544 \uc6d0\uc815\uc774 \uc2e4\ud328\ub85c \ub05d\ub098\uc790 \ub098\ud3f4\ub808\uc639\uc5d0 \uac78\uc5c8\ub358 \ud3f4\ub780\ub4dc \ubbfc\uc871\uc758 \ubaa8\ub4e0 \ud76c\ub9dd\ub3c4 \uc88c\uc808\ub418\uc5c8\ub2e4. 1813\ub144 2\uc6d4 \ubc14\ub974\uc0e4\ubc14\ub97c \uc810\ub839\ud55c \ub7ec\uc2dc\uc544 \uad70\ub300\ub294 5\uc6d4\uc5d0 \uc774\ubbf8 \uc804 \uacf5\uad6d\uc758 \uc601\ud1a0\ub97c \uc9c0\ubc30\ud558\uac8c \ub418\uc5c8\ub2e4. \uacf5\uad6d\uc774 \uba78\ub9dd\ud55c \ud6c4\uc5d0\ub3c4 \ud589\uc815\uad8c\uc740 \ub7ec\uc2dc\uc544 \uc810\ub839\uad70\uc5d0\uac8c \ub118\uc5b4\uac00\uc9c0 \uc54a\uc558\uace0 \uc0c8\ub86d\uac8c \uad6c\uc131\ub41c ' 5\uc778 \ucd5c\uace0\uc704\uc6d0\ud68c '\uc5d0\uac8c \uc591\ub3c4\ub418\uc5c8\ub2e4. 1815\ub144 \ube48 \ud68c\uc758\uc5d0\uc11c \ud3f4\ub780\ub4dc\ub294 \ub7ec\uc2dc\uc544, \ud504\ub85c\uc774\uc13c, \uc624\uc2a4\ud2b8\ub9ac\uc544\uc5d0 \uc758\ud574\uc11c \ub2e4\uc2dc \ubd84\ud560\ub418\uc5c8\ub2e4. 1815\ub144 6\uc6d4 9\uc77c \ube48 \ud68c\uc758 \ucd5c\uc885 \uc758\uc815\uc11c\uc5d0 \ub530\ub77c \ud504\ub85c\uc774\uc13c\uc740 2\ucc28 \ubd84\ud560\uc2dc \ud68d\ub4dd\ud588\ub358 \uc77c\ubd80 \uc9c0\uc5ed\uacfc '\ud3ec\uc988\ub09c \ub300\uacf5\uad6d' \uc744 \uc5bb\uc5c8\uace0, \uc624\uc2a4\ud2b8\ub9ac\uc544\ub294 \ube44\uc5d8\ub9ac\uce20\uce74(Wieliczka) \uc18c\uae08 \uad11\uc0b0\uacfc 1772\ub144 \ubd84\ud560\uc2dc \ucc28\uc9c0\ud588\ub358 \ub300\ubd80\ubd84\uc758 \uc9c0\uc5ed\uc744 \ub2e4\uc2dc \ucc28\uc9c0\ud588\ub2e4. \ud06c\ub77c\ucfe0\ud504\ub294 3\uad6d\uc758 \uac10\uc2dc\ud558\uc5d0 \uc790\uccb4 \ud5cc\ubc95\uc744 \uac00\uc9c4 \uc790\uc720\uc2dc\uac00 \ub418\uc5c8\ub2e4. \ub098\uba38\uc9c0 \ubc14\ub974\uc0e4\ubc14 \uacf5\uad6d\uc740 \ud5cc\ubc95\uc5d0 \uc758\ud574\uc11c \ub7ec\uc2dc\uc544\uc640 \uc601\uc6d0\ud788 \uacb0\ud569\ub418\ub294 \ud3f4\ub780\ub4dc \uc655\uad6d\uc73c\ub85c \uac1c\ud3b8\ub418\uc5c8\ub2e4. \uc774\ub807\uac8c \ub418\uc5b4 \ubc14\ub974\uc0e4\ubc14 \uacf5\uad6d\uc740 \ub7ec\uc2dc\uc544\uc640 \ud504\ub85c\uc774\uc13c\uc5d0 \uc758\ud574\uc11c \ubd84\ud560\ub418\uc5c8\ub2e4.", "paragraph_answer": "10\ub9cc \uba85\uc758 \ud3f4\ub780\ub4dc\uc778\uc774 \ucc38\uc804\ud588\ub358 \ub098\ud3f4\ub808\uc639\uc758 \ub7ec\uc2dc\uc544 \uc6d0\uc815\uc774 \uc2e4\ud328\ub85c \ub05d\ub098\uc790 \ub098\ud3f4\ub808\uc639\uc5d0 \uac78\uc5c8\ub358 \ud3f4\ub780\ub4dc \ubbfc\uc871\uc758 \ubaa8\ub4e0 \ud76c\ub9dd\ub3c4 \uc88c\uc808\ub418\uc5c8\ub2e4. 1813\ub144 2\uc6d4 \ubc14\ub974\uc0e4\ubc14\ub97c \uc810\ub839\ud55c \ub7ec\uc2dc\uc544 \uad70\ub300\ub294 5\uc6d4\uc5d0 \uc774\ubbf8 \uc804 \uacf5\uad6d\uc758 \uc601\ud1a0\ub97c \uc9c0\ubc30\ud558\uac8c \ub418\uc5c8\ub2e4. \uacf5\uad6d\uc774 \uba78\ub9dd\ud55c \ud6c4\uc5d0\ub3c4 \ud589\uc815\uad8c\uc740 \ub7ec\uc2dc\uc544 \uc810\ub839\uad70\uc5d0\uac8c \ub118\uc5b4\uac00\uc9c0 \uc54a\uc558\uace0 \uc0c8\ub86d\uac8c \uad6c\uc131\ub41c ' 5\uc778 \ucd5c\uace0\uc704\uc6d0\ud68c '\uc5d0\uac8c \uc591\ub3c4\ub418\uc5c8\ub2e4. 1815\ub144 \ube48 \ud68c\uc758\uc5d0\uc11c \ud3f4\ub780\ub4dc\ub294 \ub7ec\uc2dc\uc544, \ud504\ub85c\uc774\uc13c, \uc624\uc2a4\ud2b8\ub9ac\uc544\uc5d0 \uc758\ud574\uc11c \ub2e4\uc2dc \ubd84\ud560\ub418\uc5c8\ub2e4. 1815\ub144 6\uc6d4 9\uc77c \ube48 \ud68c\uc758 \ucd5c\uc885 \uc758\uc815\uc11c\uc5d0 \ub530\ub77c \ud504\ub85c\uc774\uc13c\uc740 2\ucc28 \ubd84\ud560\uc2dc \ud68d\ub4dd\ud588\ub358 \uc77c\ubd80 \uc9c0\uc5ed\uacfc '\ud3ec\uc988\ub09c \ub300\uacf5\uad6d' \uc744 \uc5bb\uc5c8\uace0, \uc624\uc2a4\ud2b8\ub9ac\uc544\ub294 \ube44\uc5d8\ub9ac\uce20\uce74(Wieliczka) \uc18c\uae08 \uad11\uc0b0\uacfc 1772\ub144 \ubd84\ud560\uc2dc \ucc28\uc9c0\ud588\ub358 \ub300\ubd80\ubd84\uc758 \uc9c0\uc5ed\uc744 \ub2e4\uc2dc \ucc28\uc9c0\ud588\ub2e4. \ud06c\ub77c\ucfe0\ud504\ub294 3\uad6d\uc758 \uac10\uc2dc\ud558\uc5d0 \uc790\uccb4 \ud5cc\ubc95\uc744 \uac00\uc9c4 \uc790\uc720\uc2dc\uac00 \ub418\uc5c8\ub2e4. \ub098\uba38\uc9c0 \ubc14\ub974\uc0e4\ubc14 \uacf5\uad6d\uc740 \ud5cc\ubc95\uc5d0 \uc758\ud574\uc11c \ub7ec\uc2dc\uc544\uc640 \uc601\uc6d0\ud788 \uacb0\ud569\ub418\ub294 \ud3f4\ub780\ub4dc \uc655\uad6d\uc73c\ub85c \uac1c\ud3b8\ub418\uc5c8\ub2e4. \uc774\ub807\uac8c \ub418\uc5b4 \ubc14\ub974\uc0e4\ubc14 \uacf5\uad6d\uc740 \ub7ec\uc2dc\uc544\uc640 \ud504\ub85c\uc774\uc13c\uc5d0 \uc758\ud574\uc11c \ubd84\ud560\ub418\uc5c8\ub2e4.", "sentence_answer": "\uacf5\uad6d\uc774 \uba78\ub9dd\ud55c \ud6c4\uc5d0\ub3c4 \ud589\uc815\uad8c\uc740 \ub7ec\uc2dc\uc544 \uc810\ub839\uad70\uc5d0\uac8c \ub118\uc5b4\uac00\uc9c0 \uc54a\uc558\uace0 \uc0c8\ub86d\uac8c \uad6c\uc131\ub41c ' 5\uc778 \ucd5c\uace0\uc704\uc6d0\ud68c '\uc5d0\uac8c \uc591\ub3c4\ub418\uc5c8\ub2e4.", "paragraph_id": "6568268-12-0", "paragraph_question": "question: \ubc14\ub974\uc0e4\ubc14 \uacf5\uad6d\uc774 \uba78\ub9dd\ud55c \ud6c4 \ud589\uc815\uad8c\uc744 \uc591\ub3c4\ubc1b\uc740 \uacf3\uc740 \uc5b4\ub514\uc778\uac00?, context: 10\ub9cc \uba85\uc758 \ud3f4\ub780\ub4dc\uc778\uc774 \ucc38\uc804\ud588\ub358 \ub098\ud3f4\ub808\uc639\uc758 \ub7ec\uc2dc\uc544 \uc6d0\uc815\uc774 \uc2e4\ud328\ub85c \ub05d\ub098\uc790 \ub098\ud3f4\ub808\uc639\uc5d0 \uac78\uc5c8\ub358 \ud3f4\ub780\ub4dc \ubbfc\uc871\uc758 \ubaa8\ub4e0 \ud76c\ub9dd\ub3c4 \uc88c\uc808\ub418\uc5c8\ub2e4. 1813\ub144 2\uc6d4 \ubc14\ub974\uc0e4\ubc14\ub97c \uc810\ub839\ud55c \ub7ec\uc2dc\uc544 \uad70\ub300\ub294 5\uc6d4\uc5d0 \uc774\ubbf8 \uc804 \uacf5\uad6d\uc758 \uc601\ud1a0\ub97c \uc9c0\ubc30\ud558\uac8c \ub418\uc5c8\ub2e4. \uacf5\uad6d\uc774 \uba78\ub9dd\ud55c \ud6c4\uc5d0\ub3c4 \ud589\uc815\uad8c\uc740 \ub7ec\uc2dc\uc544 \uc810\ub839\uad70\uc5d0\uac8c \ub118\uc5b4\uac00\uc9c0 \uc54a\uc558\uace0 \uc0c8\ub86d\uac8c \uad6c\uc131\ub41c '5\uc778 \ucd5c\uace0\uc704\uc6d0\ud68c'\uc5d0\uac8c \uc591\ub3c4\ub418\uc5c8\ub2e4. 1815\ub144 \ube48 \ud68c\uc758\uc5d0\uc11c \ud3f4\ub780\ub4dc\ub294 \ub7ec\uc2dc\uc544, \ud504\ub85c\uc774\uc13c, \uc624\uc2a4\ud2b8\ub9ac\uc544\uc5d0 \uc758\ud574\uc11c \ub2e4\uc2dc \ubd84\ud560\ub418\uc5c8\ub2e4. 1815\ub144 6\uc6d4 9\uc77c \ube48 \ud68c\uc758 \ucd5c\uc885 \uc758\uc815\uc11c\uc5d0 \ub530\ub77c \ud504\ub85c\uc774\uc13c\uc740 2\ucc28 \ubd84\ud560\uc2dc \ud68d\ub4dd\ud588\ub358 \uc77c\ubd80 \uc9c0\uc5ed\uacfc '\ud3ec\uc988\ub09c \ub300\uacf5\uad6d' \uc744 \uc5bb\uc5c8\uace0, \uc624\uc2a4\ud2b8\ub9ac\uc544\ub294 \ube44\uc5d8\ub9ac\uce20\uce74(Wieliczka) \uc18c\uae08 \uad11\uc0b0\uacfc 1772\ub144 \ubd84\ud560\uc2dc \ucc28\uc9c0\ud588\ub358 \ub300\ubd80\ubd84\uc758 \uc9c0\uc5ed\uc744 \ub2e4\uc2dc \ucc28\uc9c0\ud588\ub2e4. \ud06c\ub77c\ucfe0\ud504\ub294 3\uad6d\uc758 \uac10\uc2dc\ud558\uc5d0 \uc790\uccb4 \ud5cc\ubc95\uc744 \uac00\uc9c4 \uc790\uc720\uc2dc\uac00 \ub418\uc5c8\ub2e4. \ub098\uba38\uc9c0 \ubc14\ub974\uc0e4\ubc14 \uacf5\uad6d\uc740 \ud5cc\ubc95\uc5d0 \uc758\ud574\uc11c \ub7ec\uc2dc\uc544\uc640 \uc601\uc6d0\ud788 \uacb0\ud569\ub418\ub294 \ud3f4\ub780\ub4dc \uc655\uad6d\uc73c\ub85c \uac1c\ud3b8\ub418\uc5c8\ub2e4. \uc774\ub807\uac8c \ub418\uc5b4 \ubc14\ub974\uc0e4\ubc14 \uacf5\uad6d\uc740 \ub7ec\uc2dc\uc544\uc640 \ud504\ub85c\uc774\uc13c\uc5d0 \uc758\ud574\uc11c \ubd84\ud560\ub418\uc5c8\ub2e4."} {"question": "\ube14\ub77c\ub514\ubbf8\ub974 \ub098\ubcf4\ucf54\ud504\uc758 \uc18c\uc124 \ub864\ub9ac\ud0c0\uc5d0\uc11c \ub9c8\uc131\uc758 \uc18c\ub140 \ub3cc\ub85c\ub808\uc2a4\uc5d0 \uc758\ud574 \ud30c\uba78\ud558\ub294 \uc911\ub144 \ubb38\ud559\uac00\uc758 \uc774\ub984\uc740?", "paragraph": "\ub85c\ub9ac\ud0c0\uc758 \uc5b4\uc6d0\uc740 \ube14\ub77c\ub514\ubbf8\ub974 \ub098\ubcf4\ucf54\ud504\uac00 1958\ub144\uc5d0 \ubc1c\ud45c\ud55c \uc18c\uc124 \ub864\ub9ac\ud0c0\uc5d0 \ub4f1\uc7a5\ud558\ub294 \ub3cc\ub85c\ub808\uc2a4 \ud5e4\uc774\uc988\uc758 \uc560\uce6d(\ub2c9\ub124\uc784) \uc911 \ud558\ub098\uc778 \ub864\ub9ac\ud0c0\uc774\ub2e4. (\uadf8\ub7ec\ubbc0\ub85c \uc791\uc911\uc5d0\uc11c\ub294 \ub3cc\ub9ac, \ub864 \ub4f1\uc73c\ub85c \ubd88\ub9b0\ub2e4.) \uc774 \uc18c\uc124\uc5d0 \ub4f1\uc7a5\ud558\ub294 \uc911\ub144 \ubb38\ud559\uac00\ub294 \ub3cc\ub85c\ub808\uc2a4 \ud5e4\uc774\uc988\uc5d0\uac8c \uccab \ub208\uc5d0 \ubc18\ud558\uace0, \ub3cc\ub85c\ub808\uc2a4 \ud5e4\uc774\uc988\ub294 \uadf8\ub7ec\ud55c \ubb38\ud559\uac00\ub97c \ubc88\ub871\ud558\ub294 12\uc0b4 \ubbf8\uc18c\ub140\uc774\ub2e4. \ub610\ud55c \uc774 \uc791\ud488\uc5d0\uc11c \uc720\ub798\ud558\uc5ec \u201810\ub300 \ucd08\ubc18 \uac00\ub7c9\uc18c\ub140\uc5d0\uac8c \ud2b9\ubcc4\ud55c \uac10\uc815\uc744 \ud488\uc740 \uc0ac\ub78c\u2019\uc744 \ub864\ub9ac\ud0c0 \ucf64\ud50c\ub809\uc2a4\ub77c \ubd80\ub974\uae30\ub3c4 \ud55c\ub2e4. \uc774 \ub3cc\ub85c\ub808\uc2a4\ub77c\ub294 \uc18c\ub140\ub294 \uc18c\ub144\uae30 \ud2b9\uc720 \uc2e0\ube44\ub85c\uc6b4 \ub9e4\ub825\uc744 \uc9c0\ub2cc \ub9c8\uc131\uc758 \uc18c\ub140\ub85c, \uc911\ub144 \ubb38\ud559\uac00 \ud5d8\ubc84\ud2b8 \ud5d8\ubc84\ud2b8\ub97c \ubc88\ub871\ud558\uc5ec \ud30c\uba78\ub85c \uc774\ub048\ub2e4. \ub098\ubcf4\ucf54\ud504\ub294 \ub85c\ub9ac\ud0c0\ub97c \uc5f0\ub839\uc801\uc73c\ub85c \uc5b4\ub9ac\uace0 (10\ub300 \ucd08\ubc18 \uc815\ub3c4), \uc5b8\ub3d9\uc774\ub098 \uc6a9\ubaa8\uc640 \uc790\ud0dc\uac00 \uc18c\uc545\ub9c8\uac19\uace0, \uad50\ud0dc(coquetry)\ud558\uace0 \ub2d8\ud3ab(nymphet)\ud574\uc57c\ud55c\ub2e4\uace0 \uae4c\ub2e4\ub86d\uac8c \uc815\uc758\ud588\uc73c\ub098, \uc77c\ubcf8\uc5d0\uc11c \u2018\ub85c\ub9ac\ud0c0\u2019\ub77c\ub294 \ub9d0\uc740 \ub2d8\ud504\uc640\ub294 \uac70\uc758 \ubc18\ub300\ub85c \u2018\uc774\ubbf8 \uc2e4\uc81c\ub85c \uc5b4\ub978\uc774\uc9c0\ub9cc, \ub3d9\uc548\uc774\ub77c\uc11c \uc21c\uc9c4\ud568\uc73c\ub85c \ubc18\ud558\uac8c \ud558\ub294 \uc5ec\uc131\u2019\uc774\ub098 \u2018\uc815\ub9d0\ub85c \uc544\uc9c1 \uc5d0\ub85c\uc2a4\ub97c \uc694\ub9cc\ud07c\ub3c4 \uc2e0\uccb4\uc5d0 \uc9c0\ub2c8\uc9c0 \uc54a\uc740 \uc18c\ub140\u2019\ub97c \uac00\ub9ac\ud0a8\ub2e4. \ub2e4\ucf00\ubaa8\ud1a0 \ub178\ubc14\ub77c\ub294 \ub85c\ub9ac\ud0c0\ub97c \uc774\uc57c\uae30\ud560 \ub54c \u2018\ub098\ubcf4\ucf54\ud504\uac00 \uc815\uc758\ud55c \ub85c\ub9ac\ud0c0\u2019\uc640, \uac70\uae30\uc5d0\uc11c \ud30c\uc0dd\ud558\uc5ec \ub4a4\uc9d1\ud600\ubc84\ub9b0 \u2018\uc77c\ubcf8 \ud2b9\uc720 \ub85c\ub9ac\ud0c0 \ud574\uc11d\u2019\uc744 \uc815\ub9ac\ud574\uc11c \ubc14\ub85c\uc7a1\uc9c0 \uc54a\uc73c\uba74, \ub17c\uc810\uc774 \uacc4\uc18d \uc5c7\uac08\ub824\uc11c \ud63c\ub780\uc744 \uacaa\uac8c \ub41c\ub2e4\uace0 \ub9d0\ud55c\ub2e4. \uadf8\ub807\uae30 \ub54c\ubb38\uc5d0, \ub2e4\ucf00\ubaa8\ud1a0 \ub178\ubc14\ub77c\ub294 \u2018\uc77c\ubcf8\uc801 \ud574\uc11d\u2019 \ub85c\ub9ac\ud0c0\ub97c \uae30\ubcf8\uc73c\ub85c \uc815\uc758\ud55c\ub2e4\uace0 \ub9d0\ud588\ub2e4. \uc800\uc11c \u300a\ud328\uce58\uc6cc\ud06c\u300b(\u30d1\u30c3\u30c1\u30ef\u30fc\u30af)\uc5d0\uc11c \ub85c\ub9ac\ud0c0 \uc6d0\ub958 \ube0c\ub79c\ub4dc\uc778 \u2018MILK\u2019 \ub514\ub809\ud130 \uc624\uc624\uce74\uc640 \ud788\ud1a0\ubbf8(\u5927\u5ddd\u3072\u3068\u307f)\ub294 \uc774\ub807\uac8c \ub9d0\ud588\ub2e4. \u201c\u2018MILK\u2019\uc758 \ub514\uc790\uc778\uc740 \u2018\uc5ec\uc790 \uc544\uc774(\u5973\u306e\u5b50)\ub2c8\uae4c \uadc0\uc5fd\uace0, \uc870\uae08 \ubd80\ub4dc\ub7fd\uac8c(\u30dd\u30ef\u30fc\u30f3) \ubcf4\uc774\uace0 \ucc9c\uc0ac\uac19\uc740 \uad6c\uc11d\uc5d0, \uc545\ub9c8\uac19\uc740 \ubd80\ubd84\uc744 \uc9c0\ub2cc \uc5ec\uc790 \uc544\uc774\u2019 \ub610\ub294 \u2018\uc5b4\ub9b0\uc774\uac19\uc740 \ubd80\ubd84\uc744 \ub0a8\uaca8\ub454 \uace0\uc6b4 \uc131\uc778 \uc5ec\uc131\u2019 \uc774\ubbf8\uc9c0\ub85c \ub9cc\ub4e4\uc5b4\uc84c\uace0, \u2018\ucc9c\uc0ac \ud0c8\uc744 \uc4f4 \uc545\ub9c8\u2019\ub97c \uc758\uc2dd\ud558\uace0 \uc788\ub2e4.\u201d", "answer": "\ud5d8\ubc84\ud2b8 \ud5d8\ubc84\ud2b8", "sentence": "\uc774 \ub3cc\ub85c\ub808\uc2a4\ub77c\ub294 \uc18c\ub140\ub294 \uc18c\ub144\uae30 \ud2b9\uc720 \uc2e0\ube44\ub85c\uc6b4 \ub9e4\ub825\uc744 \uc9c0\ub2cc \ub9c8\uc131\uc758 \uc18c\ub140\ub85c, \uc911\ub144 \ubb38\ud559\uac00 \ud5d8\ubc84\ud2b8 \ud5d8\ubc84\ud2b8 \ub97c \ubc88\ub871\ud558\uc5ec \ud30c\uba78\ub85c \uc774\ub048\ub2e4.", "paragraph_sentence": "\ub85c\ub9ac\ud0c0\uc758 \uc5b4\uc6d0\uc740 \ube14\ub77c\ub514\ubbf8\ub974 \ub098\ubcf4\ucf54\ud504\uac00 1958\ub144\uc5d0 \ubc1c\ud45c\ud55c \uc18c\uc124 \ub864\ub9ac\ud0c0\uc5d0 \ub4f1\uc7a5\ud558\ub294 \ub3cc\ub85c\ub808\uc2a4 \ud5e4\uc774\uc988\uc758 \uc560\uce6d(\ub2c9\ub124\uc784) \uc911 \ud558\ub098\uc778 \ub864\ub9ac\ud0c0\uc774\ub2e4. (\uadf8\ub7ec\ubbc0\ub85c \uc791\uc911\uc5d0\uc11c\ub294 \ub3cc\ub9ac, \ub864 \ub4f1\uc73c\ub85c \ubd88\ub9b0\ub2e4.) \uc774 \uc18c\uc124\uc5d0 \ub4f1\uc7a5\ud558\ub294 \uc911\ub144 \ubb38\ud559\uac00\ub294 \ub3cc\ub85c\ub808\uc2a4 \ud5e4\uc774\uc988\uc5d0\uac8c \uccab \ub208\uc5d0 \ubc18\ud558\uace0, \ub3cc\ub85c\ub808\uc2a4 \ud5e4\uc774\uc988\ub294 \uadf8\ub7ec\ud55c \ubb38\ud559\uac00\ub97c \ubc88\ub871\ud558\ub294 12\uc0b4 \ubbf8\uc18c\ub140\uc774\ub2e4. \ub610\ud55c \uc774 \uc791\ud488\uc5d0\uc11c \uc720\ub798\ud558\uc5ec \u201810\ub300 \ucd08\ubc18 \uac00\ub7c9\uc18c\ub140\uc5d0\uac8c \ud2b9\ubcc4\ud55c \uac10\uc815\uc744 \ud488\uc740 \uc0ac\ub78c\u2019\uc744 \ub864\ub9ac\ud0c0 \ucf64\ud50c\ub809\uc2a4\ub77c \ubd80\ub974\uae30\ub3c4 \ud55c\ub2e4. \uc774 \ub3cc\ub85c\ub808\uc2a4\ub77c\ub294 \uc18c\ub140\ub294 \uc18c\ub144\uae30 \ud2b9\uc720 \uc2e0\ube44\ub85c\uc6b4 \ub9e4\ub825\uc744 \uc9c0\ub2cc \ub9c8\uc131\uc758 \uc18c\ub140\ub85c, \uc911\ub144 \ubb38\ud559\uac00 \ud5d8\ubc84\ud2b8 \ud5d8\ubc84\ud2b8 \ub97c \ubc88\ub871\ud558\uc5ec \ud30c\uba78\ub85c \uc774\ub048\ub2e4. \ub098\ubcf4\ucf54\ud504\ub294 \ub85c\ub9ac\ud0c0\ub97c \uc5f0\ub839\uc801\uc73c\ub85c \uc5b4\ub9ac\uace0 (10\ub300 \ucd08\ubc18 \uc815\ub3c4), \uc5b8\ub3d9\uc774\ub098 \uc6a9\ubaa8\uc640 \uc790\ud0dc\uac00 \uc18c\uc545\ub9c8\uac19\uace0, \uad50\ud0dc(coquetry)\ud558\uace0 \ub2d8\ud3ab(nymphet)\ud574\uc57c\ud55c\ub2e4\uace0 \uae4c\ub2e4\ub86d\uac8c \uc815\uc758\ud588\uc73c\ub098, \uc77c\ubcf8\uc5d0\uc11c \u2018\ub85c\ub9ac\ud0c0\u2019\ub77c\ub294 \ub9d0\uc740 \ub2d8\ud504\uc640\ub294 \uac70\uc758 \ubc18\ub300\ub85c \u2018\uc774\ubbf8 \uc2e4\uc81c\ub85c \uc5b4\ub978\uc774\uc9c0\ub9cc, \ub3d9\uc548\uc774\ub77c\uc11c \uc21c\uc9c4\ud568\uc73c\ub85c \ubc18\ud558\uac8c \ud558\ub294 \uc5ec\uc131\u2019\uc774\ub098 \u2018\uc815\ub9d0\ub85c \uc544\uc9c1 \uc5d0\ub85c\uc2a4\ub97c \uc694\ub9cc\ud07c\ub3c4 \uc2e0\uccb4\uc5d0 \uc9c0\ub2c8\uc9c0 \uc54a\uc740 \uc18c\ub140\u2019\ub97c \uac00\ub9ac\ud0a8\ub2e4. \ub2e4\ucf00\ubaa8\ud1a0 \ub178\ubc14\ub77c\ub294 \ub85c\ub9ac\ud0c0\ub97c \uc774\uc57c\uae30\ud560 \ub54c \u2018\ub098\ubcf4\ucf54\ud504\uac00 \uc815\uc758\ud55c \ub85c\ub9ac\ud0c0\u2019\uc640, \uac70\uae30\uc5d0\uc11c \ud30c\uc0dd\ud558\uc5ec \ub4a4\uc9d1\ud600\ubc84\ub9b0 \u2018\uc77c\ubcf8 \ud2b9\uc720 \ub85c\ub9ac\ud0c0 \ud574\uc11d\u2019\uc744 \uc815\ub9ac\ud574\uc11c \ubc14\ub85c\uc7a1\uc9c0 \uc54a\uc73c\uba74, \ub17c\uc810\uc774 \uacc4\uc18d \uc5c7\uac08\ub824\uc11c \ud63c\ub780\uc744 \uacaa\uac8c \ub41c\ub2e4\uace0 \ub9d0\ud55c\ub2e4. \uadf8\ub807\uae30 \ub54c\ubb38\uc5d0, \ub2e4\ucf00\ubaa8\ud1a0 \ub178\ubc14\ub77c\ub294 \u2018\uc77c\ubcf8\uc801 \ud574\uc11d\u2019 \ub85c\ub9ac\ud0c0\ub97c \uae30\ubcf8\uc73c\ub85c \uc815\uc758\ud55c\ub2e4\uace0 \ub9d0\ud588\ub2e4. \uc800\uc11c \u300a\ud328\uce58\uc6cc\ud06c\u300b(\u30d1\u30c3\u30c1\u30ef\u30fc\u30af)\uc5d0\uc11c \ub85c\ub9ac\ud0c0 \uc6d0\ub958 \ube0c\ub79c\ub4dc\uc778 \u2018MILK\u2019 \ub514\ub809\ud130 \uc624\uc624\uce74\uc640 \ud788\ud1a0\ubbf8(\u5927\u5ddd\u3072\u3068\u307f)\ub294 \uc774\ub807\uac8c \ub9d0\ud588\ub2e4. \u201c\u2018MILK\u2019\uc758 \ub514\uc790\uc778\uc740 \u2018\uc5ec\uc790 \uc544\uc774(\u5973\u306e\u5b50)\ub2c8\uae4c \uadc0\uc5fd\uace0, \uc870\uae08 \ubd80\ub4dc\ub7fd\uac8c(\u30dd\u30ef\u30fc\u30f3) \ubcf4\uc774\uace0 \ucc9c\uc0ac\uac19\uc740 \uad6c\uc11d\uc5d0, \uc545\ub9c8\uac19\uc740 \ubd80\ubd84\uc744 \uc9c0\ub2cc \uc5ec\uc790 \uc544\uc774\u2019 \ub610\ub294 \u2018\uc5b4\ub9b0\uc774\uac19\uc740 \ubd80\ubd84\uc744 \ub0a8\uaca8\ub454 \uace0\uc6b4 \uc131\uc778 \uc5ec\uc131\u2019 \uc774\ubbf8\uc9c0\ub85c \ub9cc\ub4e4\uc5b4\uc84c\uace0, \u2018\ucc9c\uc0ac \ud0c8\uc744 \uc4f4 \uc545\ub9c8\u2019\ub97c \uc758\uc2dd\ud558\uace0 \uc788\ub2e4. \u201d", "paragraph_answer": "\ub85c\ub9ac\ud0c0\uc758 \uc5b4\uc6d0\uc740 \ube14\ub77c\ub514\ubbf8\ub974 \ub098\ubcf4\ucf54\ud504\uac00 1958\ub144\uc5d0 \ubc1c\ud45c\ud55c \uc18c\uc124 \ub864\ub9ac\ud0c0\uc5d0 \ub4f1\uc7a5\ud558\ub294 \ub3cc\ub85c\ub808\uc2a4 \ud5e4\uc774\uc988\uc758 \uc560\uce6d(\ub2c9\ub124\uc784) \uc911 \ud558\ub098\uc778 \ub864\ub9ac\ud0c0\uc774\ub2e4. (\uadf8\ub7ec\ubbc0\ub85c \uc791\uc911\uc5d0\uc11c\ub294 \ub3cc\ub9ac, \ub864 \ub4f1\uc73c\ub85c \ubd88\ub9b0\ub2e4.) \uc774 \uc18c\uc124\uc5d0 \ub4f1\uc7a5\ud558\ub294 \uc911\ub144 \ubb38\ud559\uac00\ub294 \ub3cc\ub85c\ub808\uc2a4 \ud5e4\uc774\uc988\uc5d0\uac8c \uccab \ub208\uc5d0 \ubc18\ud558\uace0, \ub3cc\ub85c\ub808\uc2a4 \ud5e4\uc774\uc988\ub294 \uadf8\ub7ec\ud55c \ubb38\ud559\uac00\ub97c \ubc88\ub871\ud558\ub294 12\uc0b4 \ubbf8\uc18c\ub140\uc774\ub2e4. \ub610\ud55c \uc774 \uc791\ud488\uc5d0\uc11c \uc720\ub798\ud558\uc5ec \u201810\ub300 \ucd08\ubc18 \uac00\ub7c9\uc18c\ub140\uc5d0\uac8c \ud2b9\ubcc4\ud55c \uac10\uc815\uc744 \ud488\uc740 \uc0ac\ub78c\u2019\uc744 \ub864\ub9ac\ud0c0 \ucf64\ud50c\ub809\uc2a4\ub77c \ubd80\ub974\uae30\ub3c4 \ud55c\ub2e4. \uc774 \ub3cc\ub85c\ub808\uc2a4\ub77c\ub294 \uc18c\ub140\ub294 \uc18c\ub144\uae30 \ud2b9\uc720 \uc2e0\ube44\ub85c\uc6b4 \ub9e4\ub825\uc744 \uc9c0\ub2cc \ub9c8\uc131\uc758 \uc18c\ub140\ub85c, \uc911\ub144 \ubb38\ud559\uac00 \ud5d8\ubc84\ud2b8 \ud5d8\ubc84\ud2b8 \ub97c \ubc88\ub871\ud558\uc5ec \ud30c\uba78\ub85c \uc774\ub048\ub2e4. \ub098\ubcf4\ucf54\ud504\ub294 \ub85c\ub9ac\ud0c0\ub97c \uc5f0\ub839\uc801\uc73c\ub85c \uc5b4\ub9ac\uace0 (10\ub300 \ucd08\ubc18 \uc815\ub3c4), \uc5b8\ub3d9\uc774\ub098 \uc6a9\ubaa8\uc640 \uc790\ud0dc\uac00 \uc18c\uc545\ub9c8\uac19\uace0, \uad50\ud0dc(coquetry)\ud558\uace0 \ub2d8\ud3ab(nymphet)\ud574\uc57c\ud55c\ub2e4\uace0 \uae4c\ub2e4\ub86d\uac8c \uc815\uc758\ud588\uc73c\ub098, \uc77c\ubcf8\uc5d0\uc11c \u2018\ub85c\ub9ac\ud0c0\u2019\ub77c\ub294 \ub9d0\uc740 \ub2d8\ud504\uc640\ub294 \uac70\uc758 \ubc18\ub300\ub85c \u2018\uc774\ubbf8 \uc2e4\uc81c\ub85c \uc5b4\ub978\uc774\uc9c0\ub9cc, \ub3d9\uc548\uc774\ub77c\uc11c \uc21c\uc9c4\ud568\uc73c\ub85c \ubc18\ud558\uac8c \ud558\ub294 \uc5ec\uc131\u2019\uc774\ub098 \u2018\uc815\ub9d0\ub85c \uc544\uc9c1 \uc5d0\ub85c\uc2a4\ub97c \uc694\ub9cc\ud07c\ub3c4 \uc2e0\uccb4\uc5d0 \uc9c0\ub2c8\uc9c0 \uc54a\uc740 \uc18c\ub140\u2019\ub97c \uac00\ub9ac\ud0a8\ub2e4. \ub2e4\ucf00\ubaa8\ud1a0 \ub178\ubc14\ub77c\ub294 \ub85c\ub9ac\ud0c0\ub97c \uc774\uc57c\uae30\ud560 \ub54c \u2018\ub098\ubcf4\ucf54\ud504\uac00 \uc815\uc758\ud55c \ub85c\ub9ac\ud0c0\u2019\uc640, \uac70\uae30\uc5d0\uc11c \ud30c\uc0dd\ud558\uc5ec \ub4a4\uc9d1\ud600\ubc84\ub9b0 \u2018\uc77c\ubcf8 \ud2b9\uc720 \ub85c\ub9ac\ud0c0 \ud574\uc11d\u2019\uc744 \uc815\ub9ac\ud574\uc11c \ubc14\ub85c\uc7a1\uc9c0 \uc54a\uc73c\uba74, \ub17c\uc810\uc774 \uacc4\uc18d \uc5c7\uac08\ub824\uc11c \ud63c\ub780\uc744 \uacaa\uac8c \ub41c\ub2e4\uace0 \ub9d0\ud55c\ub2e4. \uadf8\ub807\uae30 \ub54c\ubb38\uc5d0, \ub2e4\ucf00\ubaa8\ud1a0 \ub178\ubc14\ub77c\ub294 \u2018\uc77c\ubcf8\uc801 \ud574\uc11d\u2019 \ub85c\ub9ac\ud0c0\ub97c \uae30\ubcf8\uc73c\ub85c \uc815\uc758\ud55c\ub2e4\uace0 \ub9d0\ud588\ub2e4. \uc800\uc11c \u300a\ud328\uce58\uc6cc\ud06c\u300b(\u30d1\u30c3\u30c1\u30ef\u30fc\u30af)\uc5d0\uc11c \ub85c\ub9ac\ud0c0 \uc6d0\ub958 \ube0c\ub79c\ub4dc\uc778 \u2018MILK\u2019 \ub514\ub809\ud130 \uc624\uc624\uce74\uc640 \ud788\ud1a0\ubbf8(\u5927\u5ddd\u3072\u3068\u307f)\ub294 \uc774\ub807\uac8c \ub9d0\ud588\ub2e4. \u201c\u2018MILK\u2019\uc758 \ub514\uc790\uc778\uc740 \u2018\uc5ec\uc790 \uc544\uc774(\u5973\u306e\u5b50)\ub2c8\uae4c \uadc0\uc5fd\uace0, \uc870\uae08 \ubd80\ub4dc\ub7fd\uac8c(\u30dd\u30ef\u30fc\u30f3) \ubcf4\uc774\uace0 \ucc9c\uc0ac\uac19\uc740 \uad6c\uc11d\uc5d0, \uc545\ub9c8\uac19\uc740 \ubd80\ubd84\uc744 \uc9c0\ub2cc \uc5ec\uc790 \uc544\uc774\u2019 \ub610\ub294 \u2018\uc5b4\ub9b0\uc774\uac19\uc740 \ubd80\ubd84\uc744 \ub0a8\uaca8\ub454 \uace0\uc6b4 \uc131\uc778 \uc5ec\uc131\u2019 \uc774\ubbf8\uc9c0\ub85c \ub9cc\ub4e4\uc5b4\uc84c\uace0, \u2018\ucc9c\uc0ac \ud0c8\uc744 \uc4f4 \uc545\ub9c8\u2019\ub97c \uc758\uc2dd\ud558\uace0 \uc788\ub2e4.\u201d", "sentence_answer": "\uc774 \ub3cc\ub85c\ub808\uc2a4\ub77c\ub294 \uc18c\ub140\ub294 \uc18c\ub144\uae30 \ud2b9\uc720 \uc2e0\ube44\ub85c\uc6b4 \ub9e4\ub825\uc744 \uc9c0\ub2cc \ub9c8\uc131\uc758 \uc18c\ub140\ub85c, \uc911\ub144 \ubb38\ud559\uac00 \ud5d8\ubc84\ud2b8 \ud5d8\ubc84\ud2b8 \ub97c \ubc88\ub871\ud558\uc5ec \ud30c\uba78\ub85c \uc774\ub048\ub2e4.", "paragraph_id": "6513684-0-2", "paragraph_question": "question: \ube14\ub77c\ub514\ubbf8\ub974 \ub098\ubcf4\ucf54\ud504\uc758 \uc18c\uc124 \ub864\ub9ac\ud0c0\uc5d0\uc11c \ub9c8\uc131\uc758 \uc18c\ub140 \ub3cc\ub85c\ub808\uc2a4\uc5d0 \uc758\ud574 \ud30c\uba78\ud558\ub294 \uc911\ub144 \ubb38\ud559\uac00\uc758 \uc774\ub984\uc740?, context: \ub85c\ub9ac\ud0c0\uc758 \uc5b4\uc6d0\uc740 \ube14\ub77c\ub514\ubbf8\ub974 \ub098\ubcf4\ucf54\ud504\uac00 1958\ub144\uc5d0 \ubc1c\ud45c\ud55c \uc18c\uc124 \ub864\ub9ac\ud0c0\uc5d0 \ub4f1\uc7a5\ud558\ub294 \ub3cc\ub85c\ub808\uc2a4 \ud5e4\uc774\uc988\uc758 \uc560\uce6d(\ub2c9\ub124\uc784) \uc911 \ud558\ub098\uc778 \ub864\ub9ac\ud0c0\uc774\ub2e4. (\uadf8\ub7ec\ubbc0\ub85c \uc791\uc911\uc5d0\uc11c\ub294 \ub3cc\ub9ac, \ub864 \ub4f1\uc73c\ub85c \ubd88\ub9b0\ub2e4.) \uc774 \uc18c\uc124\uc5d0 \ub4f1\uc7a5\ud558\ub294 \uc911\ub144 \ubb38\ud559\uac00\ub294 \ub3cc\ub85c\ub808\uc2a4 \ud5e4\uc774\uc988\uc5d0\uac8c \uccab \ub208\uc5d0 \ubc18\ud558\uace0, \ub3cc\ub85c\ub808\uc2a4 \ud5e4\uc774\uc988\ub294 \uadf8\ub7ec\ud55c \ubb38\ud559\uac00\ub97c \ubc88\ub871\ud558\ub294 12\uc0b4 \ubbf8\uc18c\ub140\uc774\ub2e4. \ub610\ud55c \uc774 \uc791\ud488\uc5d0\uc11c \uc720\ub798\ud558\uc5ec \u201810\ub300 \ucd08\ubc18 \uac00\ub7c9\uc18c\ub140\uc5d0\uac8c \ud2b9\ubcc4\ud55c \uac10\uc815\uc744 \ud488\uc740 \uc0ac\ub78c\u2019\uc744 \ub864\ub9ac\ud0c0 \ucf64\ud50c\ub809\uc2a4\ub77c \ubd80\ub974\uae30\ub3c4 \ud55c\ub2e4. \uc774 \ub3cc\ub85c\ub808\uc2a4\ub77c\ub294 \uc18c\ub140\ub294 \uc18c\ub144\uae30 \ud2b9\uc720 \uc2e0\ube44\ub85c\uc6b4 \ub9e4\ub825\uc744 \uc9c0\ub2cc \ub9c8\uc131\uc758 \uc18c\ub140\ub85c, \uc911\ub144 \ubb38\ud559\uac00 \ud5d8\ubc84\ud2b8 \ud5d8\ubc84\ud2b8\ub97c \ubc88\ub871\ud558\uc5ec \ud30c\uba78\ub85c \uc774\ub048\ub2e4. \ub098\ubcf4\ucf54\ud504\ub294 \ub85c\ub9ac\ud0c0\ub97c \uc5f0\ub839\uc801\uc73c\ub85c \uc5b4\ub9ac\uace0 (10\ub300 \ucd08\ubc18 \uc815\ub3c4), \uc5b8\ub3d9\uc774\ub098 \uc6a9\ubaa8\uc640 \uc790\ud0dc\uac00 \uc18c\uc545\ub9c8\uac19\uace0, \uad50\ud0dc(coquetry)\ud558\uace0 \ub2d8\ud3ab(nymphet)\ud574\uc57c\ud55c\ub2e4\uace0 \uae4c\ub2e4\ub86d\uac8c \uc815\uc758\ud588\uc73c\ub098, \uc77c\ubcf8\uc5d0\uc11c \u2018\ub85c\ub9ac\ud0c0\u2019\ub77c\ub294 \ub9d0\uc740 \ub2d8\ud504\uc640\ub294 \uac70\uc758 \ubc18\ub300\ub85c \u2018\uc774\ubbf8 \uc2e4\uc81c\ub85c \uc5b4\ub978\uc774\uc9c0\ub9cc, \ub3d9\uc548\uc774\ub77c\uc11c \uc21c\uc9c4\ud568\uc73c\ub85c \ubc18\ud558\uac8c \ud558\ub294 \uc5ec\uc131\u2019\uc774\ub098 \u2018\uc815\ub9d0\ub85c \uc544\uc9c1 \uc5d0\ub85c\uc2a4\ub97c \uc694\ub9cc\ud07c\ub3c4 \uc2e0\uccb4\uc5d0 \uc9c0\ub2c8\uc9c0 \uc54a\uc740 \uc18c\ub140\u2019\ub97c \uac00\ub9ac\ud0a8\ub2e4. \ub2e4\ucf00\ubaa8\ud1a0 \ub178\ubc14\ub77c\ub294 \ub85c\ub9ac\ud0c0\ub97c \uc774\uc57c\uae30\ud560 \ub54c \u2018\ub098\ubcf4\ucf54\ud504\uac00 \uc815\uc758\ud55c \ub85c\ub9ac\ud0c0\u2019\uc640, \uac70\uae30\uc5d0\uc11c \ud30c\uc0dd\ud558\uc5ec \ub4a4\uc9d1\ud600\ubc84\ub9b0 \u2018\uc77c\ubcf8 \ud2b9\uc720 \ub85c\ub9ac\ud0c0 \ud574\uc11d\u2019\uc744 \uc815\ub9ac\ud574\uc11c \ubc14\ub85c\uc7a1\uc9c0 \uc54a\uc73c\uba74, \ub17c\uc810\uc774 \uacc4\uc18d \uc5c7\uac08\ub824\uc11c \ud63c\ub780\uc744 \uacaa\uac8c \ub41c\ub2e4\uace0 \ub9d0\ud55c\ub2e4. \uadf8\ub807\uae30 \ub54c\ubb38\uc5d0, \ub2e4\ucf00\ubaa8\ud1a0 \ub178\ubc14\ub77c\ub294 \u2018\uc77c\ubcf8\uc801 \ud574\uc11d\u2019 \ub85c\ub9ac\ud0c0\ub97c \uae30\ubcf8\uc73c\ub85c \uc815\uc758\ud55c\ub2e4\uace0 \ub9d0\ud588\ub2e4. \uc800\uc11c \u300a\ud328\uce58\uc6cc\ud06c\u300b(\u30d1\u30c3\u30c1\u30ef\u30fc\u30af)\uc5d0\uc11c \ub85c\ub9ac\ud0c0 \uc6d0\ub958 \ube0c\ub79c\ub4dc\uc778 \u2018MILK\u2019 \ub514\ub809\ud130 \uc624\uc624\uce74\uc640 \ud788\ud1a0\ubbf8(\u5927\u5ddd\u3072\u3068\u307f)\ub294 \uc774\ub807\uac8c \ub9d0\ud588\ub2e4. \u201c\u2018MILK\u2019\uc758 \ub514\uc790\uc778\uc740 \u2018\uc5ec\uc790 \uc544\uc774(\u5973\u306e\u5b50)\ub2c8\uae4c \uadc0\uc5fd\uace0, \uc870\uae08 \ubd80\ub4dc\ub7fd\uac8c(\u30dd\u30ef\u30fc\u30f3) \ubcf4\uc774\uace0 \ucc9c\uc0ac\uac19\uc740 \uad6c\uc11d\uc5d0, \uc545\ub9c8\uac19\uc740 \ubd80\ubd84\uc744 \uc9c0\ub2cc \uc5ec\uc790 \uc544\uc774\u2019 \ub610\ub294 \u2018\uc5b4\ub9b0\uc774\uac19\uc740 \ubd80\ubd84\uc744 \ub0a8\uaca8\ub454 \uace0\uc6b4 \uc131\uc778 \uc5ec\uc131\u2019 \uc774\ubbf8\uc9c0\ub85c \ub9cc\ub4e4\uc5b4\uc84c\uace0, \u2018\ucc9c\uc0ac \ud0c8\uc744 \uc4f4 \uc545\ub9c8\u2019\ub97c \uc758\uc2dd\ud558\uace0 \uc788\ub2e4.\u201d"} {"question": "\ubd88\ubc95\uc73c\ub85c \uc9c4\uc5d0\uc5b4 \ub4f1\uae30\uc784\uc6d0\uc5d0 \uc62c\ub790\uc5c8\ub358 \uc0ac\ub78c\uc758 \uc774\ub984\uc740?", "paragraph": "\uc870\ud604\ubbfc \uc9c4\uc5d0\uc5b4 \ub9c8\ucf00\ud305\ubcf8\ubd80 \ubcf8\ubd80\uc7a5 (\ubd80\uc0ac\uc7a5)\uc774 2010\ub144\ubd80\ud130 2016\ub144 \uae4c\uc9c0 6\ub144\uac04 \ubd88\ubc95\uc73c\ub85c \uc9c4\uc5d0\uc5b4 \ub4f1\uae30\uc784\uc6d0\uc5d0 \uc62c\ub790\ub358 \uac83\uc73c\ub85c \ud655\uc778\ub418\uc5b4 \ub17c\ub780\uc774 \uc77c\uc5c8\ub2e4. 2018\ub144 '\ubb3c\ubcbc\ub77d \uac11\uc9c8 \ub17c\ub780'\uc73c\ub85c \uc0ac\ud1f4 \uc555\ub825\uc774 \uac70\uc138\uc9c0\ub294 \uac00\uc6b4\ub370 \uc870\ud604\ubbfc \ubd80\uc0ac\uc7a5\uc774 \uc624\ub7ab\ub3d9\uc548 \ubd88\ubc95\uc801\uc778 \uacbd\uc601\uc9c0\uc704\ub97c \ub204\ub9b0 \uc0ac\uc2e4\uc774 \ucd94\uac00\ub85c \ub4dc\ub7ec\ub098\uba74\uc11c \ub610 \ub2e4\ub978 \ub17c\ub780\uc774 \ub418\uace0 \uc788\ub2e4. \uc5f0\ud569\ub274\uc2a4\uac00 \uae08\uc735\uac10\ub3c5\uc6d0 \uc804\uc790\uacf5\uc2dc\uc2dc\uc2a4\ud15c \ub4f1\uc744 \ud1b5\ud574 \ud655\uc778\ud55c \uc9c4\uc5d0\uc5b4 \uad00\ub828 \uacf5\uc2dc\ub97c \uc885\ud569\ud558\uba74 '\uc870 \uc5d0\ubc00\ub9ac \ub9ac'(CHO EMILY LEE)\ub77c\ub294 \uc778\ubb3c\uc774 2010\ub144 3\uc6d4\ubd80\ud130 2016\ub144 3\uc6d4\uae4c\uc9c0 \uc9c4\uc5d0\uc5b4 \uc0ac\ub0b4\uc774\uc0ac\ub85c \ub4f1\uc7ac\ub410\ub2e4. \uc678\uad6d\uc778\uc774 \uad6d\uc801\ud56d\uacf5\uc0ac \ub4f1\uae30\uc784\uc6d0\uc73c\ub85c \uc624\ub978 \uac83\uc740 \uba85\ubc31\ud55c \ubd88\ubc95\uc774\ub2e4. \ud56d\uacf5\uc0ac\uc5c5\ubc95 \uc81c9\uc870\ub294 '\uad6d\ub0b4\u00b7\uad6d\uc81c\ud56d\uacf5\uc6b4\uc1a1\uc0ac\uc5c5 \uba74\ud5c8\uc758 \uacb0\uaca9\uc0ac\uc720' \uc911 \ud558\ub098\ub85c \uc784\uc6d0 \uc911\uc5d0 '\ub300\ud55c\ubbfc\uad6d \uad6d\ubbfc\uc774 \uc544\ub2cc \uc0ac\ub78c'\uc774 \uc788\ub294 \uacbd\uc6b0\ub97c \uaf3d\uace0 \uc788\ub2e4. \uc9c4\uc5d0\uc5b4 \uad00\uacc4\uc790\uc5d0 \ub530\ub974\uba74 \uc870\ud604\ubbfc \ubd80\uc0ac\uc7a5\uc740 \ub2f9\uc2dc \ub17c\ub780\uc758 \uc18c\uc9c0\uac00 \uc788\uc5b4 2016\ub144\uc5d0 \ub4f1\uae30 \uc784\uc6d0\uc744 \uc0ac\uc784\ud55c \uac83\uc73c\ub85c \uc548\ub2e4\uace0 \ubc1d\ud614\ub2e4.", "answer": "\uc870\ud604\ubbfc", "sentence": "\uc870\ud604\ubbfc \uc9c4\uc5d0\uc5b4 \ub9c8\ucf00\ud305\ubcf8\ubd80 \ubcf8\ubd80\uc7a5 (\ubd80\uc0ac\uc7a5)\uc774 2010\ub144\ubd80\ud130 2016\ub144 \uae4c\uc9c0 6\ub144\uac04 \ubd88\ubc95\uc73c\ub85c \uc9c4\uc5d0\uc5b4 \ub4f1\uae30\uc784\uc6d0\uc5d0 \uc62c\ub790\ub358 \uac83\uc73c\ub85c \ud655\uc778\ub418\uc5b4 \ub17c\ub780\uc774 \uc77c\uc5c8\ub2e4.", "paragraph_sentence": " \uc870\ud604\ubbfc \uc9c4\uc5d0\uc5b4 \ub9c8\ucf00\ud305\ubcf8\ubd80 \ubcf8\ubd80\uc7a5 (\ubd80\uc0ac\uc7a5)\uc774 2010\ub144\ubd80\ud130 2016\ub144 \uae4c\uc9c0 6\ub144\uac04 \ubd88\ubc95\uc73c\ub85c \uc9c4\uc5d0\uc5b4 \ub4f1\uae30\uc784\uc6d0\uc5d0 \uc62c\ub790\ub358 \uac83\uc73c\ub85c \ud655\uc778\ub418\uc5b4 \ub17c\ub780\uc774 \uc77c\uc5c8\ub2e4. 2018\ub144 '\ubb3c\ubcbc\ub77d \uac11\uc9c8 \ub17c\ub780'\uc73c\ub85c \uc0ac\ud1f4 \uc555\ub825\uc774 \uac70\uc138\uc9c0\ub294 \uac00\uc6b4\ub370 \uc870\ud604\ubbfc \ubd80\uc0ac\uc7a5\uc774 \uc624\ub7ab\ub3d9\uc548 \ubd88\ubc95\uc801\uc778 \uacbd\uc601\uc9c0\uc704\ub97c \ub204\ub9b0 \uc0ac\uc2e4\uc774 \ucd94\uac00\ub85c \ub4dc\ub7ec\ub098\uba74\uc11c \ub610 \ub2e4\ub978 \ub17c\ub780\uc774 \ub418\uace0 \uc788\ub2e4. \uc5f0\ud569\ub274\uc2a4\uac00 \uae08\uc735\uac10\ub3c5\uc6d0 \uc804\uc790\uacf5\uc2dc\uc2dc\uc2a4\ud15c \ub4f1\uc744 \ud1b5\ud574 \ud655\uc778\ud55c \uc9c4\uc5d0\uc5b4 \uad00\ub828 \uacf5\uc2dc\ub97c \uc885\ud569\ud558\uba74 '\uc870 \uc5d0\ubc00\ub9ac \ub9ac'(CHO EMILY LEE)\ub77c\ub294 \uc778\ubb3c\uc774 2010\ub144 3\uc6d4\ubd80\ud130 2016\ub144 3\uc6d4\uae4c\uc9c0 \uc9c4\uc5d0\uc5b4 \uc0ac\ub0b4\uc774\uc0ac\ub85c \ub4f1\uc7ac\ub410\ub2e4. \uc678\uad6d\uc778\uc774 \uad6d\uc801\ud56d\uacf5\uc0ac \ub4f1\uae30\uc784\uc6d0\uc73c\ub85c \uc624\ub978 \uac83\uc740 \uba85\ubc31\ud55c \ubd88\ubc95\uc774\ub2e4. \ud56d\uacf5\uc0ac\uc5c5\ubc95 \uc81c9\uc870\ub294 '\uad6d\ub0b4\u00b7\uad6d\uc81c\ud56d\uacf5\uc6b4\uc1a1\uc0ac\uc5c5 \uba74\ud5c8\uc758 \uacb0\uaca9\uc0ac\uc720' \uc911 \ud558\ub098\ub85c \uc784\uc6d0 \uc911\uc5d0 '\ub300\ud55c\ubbfc\uad6d \uad6d\ubbfc\uc774 \uc544\ub2cc \uc0ac\ub78c'\uc774 \uc788\ub294 \uacbd\uc6b0\ub97c \uaf3d\uace0 \uc788\ub2e4. \uc9c4\uc5d0\uc5b4 \uad00\uacc4\uc790\uc5d0 \ub530\ub974\uba74 \uc870\ud604\ubbfc \ubd80\uc0ac\uc7a5\uc740 \ub2f9\uc2dc \ub17c\ub780\uc758 \uc18c\uc9c0\uac00 \uc788\uc5b4 2016\ub144\uc5d0 \ub4f1\uae30 \uc784\uc6d0\uc744 \uc0ac\uc784\ud55c \uac83\uc73c\ub85c \uc548\ub2e4\uace0 \ubc1d\ud614\ub2e4.", "paragraph_answer": " \uc870\ud604\ubbfc \uc9c4\uc5d0\uc5b4 \ub9c8\ucf00\ud305\ubcf8\ubd80 \ubcf8\ubd80\uc7a5 (\ubd80\uc0ac\uc7a5)\uc774 2010\ub144\ubd80\ud130 2016\ub144 \uae4c\uc9c0 6\ub144\uac04 \ubd88\ubc95\uc73c\ub85c \uc9c4\uc5d0\uc5b4 \ub4f1\uae30\uc784\uc6d0\uc5d0 \uc62c\ub790\ub358 \uac83\uc73c\ub85c \ud655\uc778\ub418\uc5b4 \ub17c\ub780\uc774 \uc77c\uc5c8\ub2e4. 2018\ub144 '\ubb3c\ubcbc\ub77d \uac11\uc9c8 \ub17c\ub780'\uc73c\ub85c \uc0ac\ud1f4 \uc555\ub825\uc774 \uac70\uc138\uc9c0\ub294 \uac00\uc6b4\ub370 \uc870\ud604\ubbfc \ubd80\uc0ac\uc7a5\uc774 \uc624\ub7ab\ub3d9\uc548 \ubd88\ubc95\uc801\uc778 \uacbd\uc601\uc9c0\uc704\ub97c \ub204\ub9b0 \uc0ac\uc2e4\uc774 \ucd94\uac00\ub85c \ub4dc\ub7ec\ub098\uba74\uc11c \ub610 \ub2e4\ub978 \ub17c\ub780\uc774 \ub418\uace0 \uc788\ub2e4. \uc5f0\ud569\ub274\uc2a4\uac00 \uae08\uc735\uac10\ub3c5\uc6d0 \uc804\uc790\uacf5\uc2dc\uc2dc\uc2a4\ud15c \ub4f1\uc744 \ud1b5\ud574 \ud655\uc778\ud55c \uc9c4\uc5d0\uc5b4 \uad00\ub828 \uacf5\uc2dc\ub97c \uc885\ud569\ud558\uba74 '\uc870 \uc5d0\ubc00\ub9ac \ub9ac'(CHO EMILY LEE)\ub77c\ub294 \uc778\ubb3c\uc774 2010\ub144 3\uc6d4\ubd80\ud130 2016\ub144 3\uc6d4\uae4c\uc9c0 \uc9c4\uc5d0\uc5b4 \uc0ac\ub0b4\uc774\uc0ac\ub85c \ub4f1\uc7ac\ub410\ub2e4. \uc678\uad6d\uc778\uc774 \uad6d\uc801\ud56d\uacf5\uc0ac \ub4f1\uae30\uc784\uc6d0\uc73c\ub85c \uc624\ub978 \uac83\uc740 \uba85\ubc31\ud55c \ubd88\ubc95\uc774\ub2e4. \ud56d\uacf5\uc0ac\uc5c5\ubc95 \uc81c9\uc870\ub294 '\uad6d\ub0b4\u00b7\uad6d\uc81c\ud56d\uacf5\uc6b4\uc1a1\uc0ac\uc5c5 \uba74\ud5c8\uc758 \uacb0\uaca9\uc0ac\uc720' \uc911 \ud558\ub098\ub85c \uc784\uc6d0 \uc911\uc5d0 '\ub300\ud55c\ubbfc\uad6d \uad6d\ubbfc\uc774 \uc544\ub2cc \uc0ac\ub78c'\uc774 \uc788\ub294 \uacbd\uc6b0\ub97c \uaf3d\uace0 \uc788\ub2e4. \uc9c4\uc5d0\uc5b4 \uad00\uacc4\uc790\uc5d0 \ub530\ub974\uba74 \uc870\ud604\ubbfc \ubd80\uc0ac\uc7a5\uc740 \ub2f9\uc2dc \ub17c\ub780\uc758 \uc18c\uc9c0\uac00 \uc788\uc5b4 2016\ub144\uc5d0 \ub4f1\uae30 \uc784\uc6d0\uc744 \uc0ac\uc784\ud55c \uac83\uc73c\ub85c \uc548\ub2e4\uace0 \ubc1d\ud614\ub2e4.", "sentence_answer": " \uc870\ud604\ubbfc \uc9c4\uc5d0\uc5b4 \ub9c8\ucf00\ud305\ubcf8\ubd80 \ubcf8\ubd80\uc7a5 (\ubd80\uc0ac\uc7a5)\uc774 2010\ub144\ubd80\ud130 2016\ub144 \uae4c\uc9c0 6\ub144\uac04 \ubd88\ubc95\uc73c\ub85c \uc9c4\uc5d0\uc5b4 \ub4f1\uae30\uc784\uc6d0\uc5d0 \uc62c\ub790\ub358 \uac83\uc73c\ub85c \ud655\uc778\ub418\uc5b4 \ub17c\ub780\uc774 \uc77c\uc5c8\ub2e4.", "paragraph_id": "6595615-0-0", "paragraph_question": "question: \ubd88\ubc95\uc73c\ub85c \uc9c4\uc5d0\uc5b4 \ub4f1\uae30\uc784\uc6d0\uc5d0 \uc62c\ub790\uc5c8\ub358 \uc0ac\ub78c\uc758 \uc774\ub984\uc740?, context: \uc870\ud604\ubbfc \uc9c4\uc5d0\uc5b4 \ub9c8\ucf00\ud305\ubcf8\ubd80 \ubcf8\ubd80\uc7a5 (\ubd80\uc0ac\uc7a5)\uc774 2010\ub144\ubd80\ud130 2016\ub144 \uae4c\uc9c0 6\ub144\uac04 \ubd88\ubc95\uc73c\ub85c \uc9c4\uc5d0\uc5b4 \ub4f1\uae30\uc784\uc6d0\uc5d0 \uc62c\ub790\ub358 \uac83\uc73c\ub85c \ud655\uc778\ub418\uc5b4 \ub17c\ub780\uc774 \uc77c\uc5c8\ub2e4. 2018\ub144 '\ubb3c\ubcbc\ub77d \uac11\uc9c8 \ub17c\ub780'\uc73c\ub85c \uc0ac\ud1f4 \uc555\ub825\uc774 \uac70\uc138\uc9c0\ub294 \uac00\uc6b4\ub370 \uc870\ud604\ubbfc \ubd80\uc0ac\uc7a5\uc774 \uc624\ub7ab\ub3d9\uc548 \ubd88\ubc95\uc801\uc778 \uacbd\uc601\uc9c0\uc704\ub97c \ub204\ub9b0 \uc0ac\uc2e4\uc774 \ucd94\uac00\ub85c \ub4dc\ub7ec\ub098\uba74\uc11c \ub610 \ub2e4\ub978 \ub17c\ub780\uc774 \ub418\uace0 \uc788\ub2e4. \uc5f0\ud569\ub274\uc2a4\uac00 \uae08\uc735\uac10\ub3c5\uc6d0 \uc804\uc790\uacf5\uc2dc\uc2dc\uc2a4\ud15c \ub4f1\uc744 \ud1b5\ud574 \ud655\uc778\ud55c \uc9c4\uc5d0\uc5b4 \uad00\ub828 \uacf5\uc2dc\ub97c \uc885\ud569\ud558\uba74 '\uc870 \uc5d0\ubc00\ub9ac \ub9ac'(CHO EMILY LEE)\ub77c\ub294 \uc778\ubb3c\uc774 2010\ub144 3\uc6d4\ubd80\ud130 2016\ub144 3\uc6d4\uae4c\uc9c0 \uc9c4\uc5d0\uc5b4 \uc0ac\ub0b4\uc774\uc0ac\ub85c \ub4f1\uc7ac\ub410\ub2e4. \uc678\uad6d\uc778\uc774 \uad6d\uc801\ud56d\uacf5\uc0ac \ub4f1\uae30\uc784\uc6d0\uc73c\ub85c \uc624\ub978 \uac83\uc740 \uba85\ubc31\ud55c \ubd88\ubc95\uc774\ub2e4. \ud56d\uacf5\uc0ac\uc5c5\ubc95 \uc81c9\uc870\ub294 '\uad6d\ub0b4\u00b7\uad6d\uc81c\ud56d\uacf5\uc6b4\uc1a1\uc0ac\uc5c5 \uba74\ud5c8\uc758 \uacb0\uaca9\uc0ac\uc720' \uc911 \ud558\ub098\ub85c \uc784\uc6d0 \uc911\uc5d0 '\ub300\ud55c\ubbfc\uad6d \uad6d\ubbfc\uc774 \uc544\ub2cc \uc0ac\ub78c'\uc774 \uc788\ub294 \uacbd\uc6b0\ub97c \uaf3d\uace0 \uc788\ub2e4. \uc9c4\uc5d0\uc5b4 \uad00\uacc4\uc790\uc5d0 \ub530\ub974\uba74 \uc870\ud604\ubbfc \ubd80\uc0ac\uc7a5\uc740 \ub2f9\uc2dc \ub17c\ub780\uc758 \uc18c\uc9c0\uac00 \uc788\uc5b4 2016\ub144\uc5d0 \ub4f1\uae30 \uc784\uc6d0\uc744 \uc0ac\uc784\ud55c \uac83\uc73c\ub85c \uc548\ub2e4\uace0 \ubc1d\ud614\ub2e4."} {"question": "\ube4c\ub9ac\uc5d0\uac8c \uc218\ubc31 \ud30c\uc6b4\ub4dc\uc758 \uc694\uae08\uc744 \uc694\uad6c\ud55c \uac83\uc740 \ub204\uad6c\uc778\uac00?", "paragraph": "\ube4c\ub9ac\uc758 \uc544\ube60\ub294 \ube4c\ub9ac\ub97c \ub3c4\uc6b8 \uc218 \uc788\ub294 \uc720\uc77c\ud55c \uae38\uc740 \uc77c\ub85c \ub3cc\uc544\uac00\ub294 \uac83 \ubfd0\uc774\ub77c\uace0 \uacb0\uc2ec\ud55c\ub2e4. \ud1a0\ub2c8\uac00 \ud30c\uc5c5 \ubc14\ub9ac\ucf00\uc774\ud2b8\ub97c \ub118\uc5b4\uac00\ub294 \uc544\ube60\uc758 \ubaa8\uc2b5\uc744 \ubcf4\uc558\uc744 \ub54c, \ud1a0\ub2c8\ub294 \uaca9\ub178\ud558\uace0 \uad11\ubd80 \uc870\ud569\uacfc \ube4c\ub9ac\uc758 \uafc8\uc744 \uc704\ud574 \ube4c\ub9ac\ub97c \ub3d5\ub294 \uac83 \uc911 \ub354 \uc911\uc694\ud55c \uac83\uc774 \ubb34\uc5c7\uc778\uc9c0\uc5d0 \ub300\ud574 \uc2f8\uc6b4\ub2e4 (*He could be a Star). \ub17c\uc7c1\uc740 \uacb0\uad6d \uaca9\ud22c\ub97c \ubd80\ub974\uac8c\ub418\uace0 \uadf8 \uacfc\uc815\uc5d0\uc11c \ube4c\ub9ac\uac00 \ub73b\ud558\uc9c0\uc54a\uac8c \ub2e4\uce5c\ub2e4. \ucc28\ub840\ucc28\ub840\ub85c, \uad11\ubd80\ub4e4\uc740 \ube4c\ub9ac\uc758 \ud3b8\uc744 \ub4e4\uc5b4\uc8fc\uace0 \ud30c\uc5c5 \uc218\ub2f9\uc744 \uc8fc\uace0\uc790 \ud55c\ub2e4. \ud558\uc9c0\ub9cc \ube4c\ub9ac\ub294 \ub7f0\ub358\uc73c\ub85c \ud5a5\ud558\ub294 \ubc84\uc2a4 \uc694\uae08\uc744 \ub0bc \uc218 \uc5c6\ub2e4. \ud30c\uc5c5 \ubc29\ud574\uc790\ub294 \ube4c\ub9ac\uc5d0\uac8c \uc218\ubc31 \ud30c\uc6b4\ub4dc\uc758 \uc694\uae08\uc744 \uc694\uad6c\ud558\uace0, \ud654\ub09c \ud1a0\ub2c8\ub294 \ubc29\ud574\ud558\ub824 \ud558\uc9c0\ub9cc, \ube4c\ub9ac\ub294 \uadf8\ub7fc\uc5d0\ub3c4 \ubd88\uad6c\ud558\uace0 \ubc1b\uc544\ub4e4\uc778\ub2e4. \ub2f9\uc7a5 \ud76c\ub9dd\uc774 \uc0ac\ub77c\uc84c\uace0, \ud1a0\ub2c8\ub294 \ub354 \uc774\uc0c1 \ubc29\ubc95\uc774 \uc788\ub294\uc9c0\uc5d0 \ub300\ud574 \uc74c\uc6b8\ud788 \uc2ec\uc0ac\uc219\uace0\ud55c\ub2e4.", "answer": "\ud30c\uc5c5 \ubc29\ud574\uc790", "sentence": "\ud30c\uc5c5 \ubc29\ud574\uc790 \ub294 \ube4c\ub9ac\uc5d0\uac8c \uc218\ubc31 \ud30c\uc6b4\ub4dc\uc758 \uc694\uae08\uc744 \uc694\uad6c\ud558\uace0, \ud654\ub09c \ud1a0\ub2c8\ub294 \ubc29\ud574\ud558\ub824 \ud558\uc9c0\ub9cc, \ube4c\ub9ac\ub294 \uadf8\ub7fc\uc5d0\ub3c4 \ubd88\uad6c\ud558\uace0 \ubc1b\uc544\ub4e4\uc778\ub2e4.", "paragraph_sentence": "\ube4c\ub9ac\uc758 \uc544\ube60\ub294 \ube4c\ub9ac\ub97c \ub3c4\uc6b8 \uc218 \uc788\ub294 \uc720\uc77c\ud55c \uae38\uc740 \uc77c\ub85c \ub3cc\uc544\uac00\ub294 \uac83 \ubfd0\uc774\ub77c\uace0 \uacb0\uc2ec\ud55c\ub2e4. \ud1a0\ub2c8\uac00 \ud30c\uc5c5 \ubc14\ub9ac\ucf00\uc774\ud2b8\ub97c \ub118\uc5b4\uac00\ub294 \uc544\ube60\uc758 \ubaa8\uc2b5\uc744 \ubcf4\uc558\uc744 \ub54c, \ud1a0\ub2c8\ub294 \uaca9\ub178\ud558\uace0 \uad11\ubd80 \uc870\ud569\uacfc \ube4c\ub9ac\uc758 \uafc8\uc744 \uc704\ud574 \ube4c\ub9ac\ub97c \ub3d5\ub294 \uac83 \uc911 \ub354 \uc911\uc694\ud55c \uac83\uc774 \ubb34\uc5c7\uc778\uc9c0\uc5d0 \ub300\ud574 \uc2f8\uc6b4\ub2e4 (*He could be a Star). \ub17c\uc7c1\uc740 \uacb0\uad6d \uaca9\ud22c\ub97c \ubd80\ub974\uac8c\ub418\uace0 \uadf8 \uacfc\uc815\uc5d0\uc11c \ube4c\ub9ac\uac00 \ub73b\ud558\uc9c0\uc54a\uac8c \ub2e4\uce5c\ub2e4. \ucc28\ub840\ucc28\ub840\ub85c, \uad11\ubd80\ub4e4\uc740 \ube4c\ub9ac\uc758 \ud3b8\uc744 \ub4e4\uc5b4\uc8fc\uace0 \ud30c\uc5c5 \uc218\ub2f9\uc744 \uc8fc\uace0\uc790 \ud55c\ub2e4. \ud558\uc9c0\ub9cc \ube4c\ub9ac\ub294 \ub7f0\ub358\uc73c\ub85c \ud5a5\ud558\ub294 \ubc84\uc2a4 \uc694\uae08\uc744 \ub0bc \uc218 \uc5c6\ub2e4. \ud30c\uc5c5 \ubc29\ud574\uc790 \ub294 \ube4c\ub9ac\uc5d0\uac8c \uc218\ubc31 \ud30c\uc6b4\ub4dc\uc758 \uc694\uae08\uc744 \uc694\uad6c\ud558\uace0, \ud654\ub09c \ud1a0\ub2c8\ub294 \ubc29\ud574\ud558\ub824 \ud558\uc9c0\ub9cc, \ube4c\ub9ac\ub294 \uadf8\ub7fc\uc5d0\ub3c4 \ubd88\uad6c\ud558\uace0 \ubc1b\uc544\ub4e4\uc778\ub2e4. \ub2f9\uc7a5 \ud76c\ub9dd\uc774 \uc0ac\ub77c\uc84c\uace0, \ud1a0\ub2c8\ub294 \ub354 \uc774\uc0c1 \ubc29\ubc95\uc774 \uc788\ub294\uc9c0\uc5d0 \ub300\ud574 \uc74c\uc6b8\ud788 \uc2ec\uc0ac\uc219\uace0\ud55c\ub2e4.", "paragraph_answer": "\ube4c\ub9ac\uc758 \uc544\ube60\ub294 \ube4c\ub9ac\ub97c \ub3c4\uc6b8 \uc218 \uc788\ub294 \uc720\uc77c\ud55c \uae38\uc740 \uc77c\ub85c \ub3cc\uc544\uac00\ub294 \uac83 \ubfd0\uc774\ub77c\uace0 \uacb0\uc2ec\ud55c\ub2e4. \ud1a0\ub2c8\uac00 \ud30c\uc5c5 \ubc14\ub9ac\ucf00\uc774\ud2b8\ub97c \ub118\uc5b4\uac00\ub294 \uc544\ube60\uc758 \ubaa8\uc2b5\uc744 \ubcf4\uc558\uc744 \ub54c, \ud1a0\ub2c8\ub294 \uaca9\ub178\ud558\uace0 \uad11\ubd80 \uc870\ud569\uacfc \ube4c\ub9ac\uc758 \uafc8\uc744 \uc704\ud574 \ube4c\ub9ac\ub97c \ub3d5\ub294 \uac83 \uc911 \ub354 \uc911\uc694\ud55c \uac83\uc774 \ubb34\uc5c7\uc778\uc9c0\uc5d0 \ub300\ud574 \uc2f8\uc6b4\ub2e4 (*He could be a Star). \ub17c\uc7c1\uc740 \uacb0\uad6d \uaca9\ud22c\ub97c \ubd80\ub974\uac8c\ub418\uace0 \uadf8 \uacfc\uc815\uc5d0\uc11c \ube4c\ub9ac\uac00 \ub73b\ud558\uc9c0\uc54a\uac8c \ub2e4\uce5c\ub2e4. \ucc28\ub840\ucc28\ub840\ub85c, \uad11\ubd80\ub4e4\uc740 \ube4c\ub9ac\uc758 \ud3b8\uc744 \ub4e4\uc5b4\uc8fc\uace0 \ud30c\uc5c5 \uc218\ub2f9\uc744 \uc8fc\uace0\uc790 \ud55c\ub2e4. \ud558\uc9c0\ub9cc \ube4c\ub9ac\ub294 \ub7f0\ub358\uc73c\ub85c \ud5a5\ud558\ub294 \ubc84\uc2a4 \uc694\uae08\uc744 \ub0bc \uc218 \uc5c6\ub2e4. \ud30c\uc5c5 \ubc29\ud574\uc790 \ub294 \ube4c\ub9ac\uc5d0\uac8c \uc218\ubc31 \ud30c\uc6b4\ub4dc\uc758 \uc694\uae08\uc744 \uc694\uad6c\ud558\uace0, \ud654\ub09c \ud1a0\ub2c8\ub294 \ubc29\ud574\ud558\ub824 \ud558\uc9c0\ub9cc, \ube4c\ub9ac\ub294 \uadf8\ub7fc\uc5d0\ub3c4 \ubd88\uad6c\ud558\uace0 \ubc1b\uc544\ub4e4\uc778\ub2e4. \ub2f9\uc7a5 \ud76c\ub9dd\uc774 \uc0ac\ub77c\uc84c\uace0, \ud1a0\ub2c8\ub294 \ub354 \uc774\uc0c1 \ubc29\ubc95\uc774 \uc788\ub294\uc9c0\uc5d0 \ub300\ud574 \uc74c\uc6b8\ud788 \uc2ec\uc0ac\uc219\uace0\ud55c\ub2e4.", "sentence_answer": " \ud30c\uc5c5 \ubc29\ud574\uc790 \ub294 \ube4c\ub9ac\uc5d0\uac8c \uc218\ubc31 \ud30c\uc6b4\ub4dc\uc758 \uc694\uae08\uc744 \uc694\uad6c\ud558\uace0, \ud654\ub09c \ud1a0\ub2c8\ub294 \ubc29\ud574\ud558\ub824 \ud558\uc9c0\ub9cc, \ube4c\ub9ac\ub294 \uadf8\ub7fc\uc5d0\ub3c4 \ubd88\uad6c\ud558\uace0 \ubc1b\uc544\ub4e4\uc778\ub2e4.", "paragraph_id": "6531194-7-1", "paragraph_question": "question: \ube4c\ub9ac\uc5d0\uac8c \uc218\ubc31 \ud30c\uc6b4\ub4dc\uc758 \uc694\uae08\uc744 \uc694\uad6c\ud55c \uac83\uc740 \ub204\uad6c\uc778\uac00?, context: \ube4c\ub9ac\uc758 \uc544\ube60\ub294 \ube4c\ub9ac\ub97c \ub3c4\uc6b8 \uc218 \uc788\ub294 \uc720\uc77c\ud55c \uae38\uc740 \uc77c\ub85c \ub3cc\uc544\uac00\ub294 \uac83 \ubfd0\uc774\ub77c\uace0 \uacb0\uc2ec\ud55c\ub2e4. \ud1a0\ub2c8\uac00 \ud30c\uc5c5 \ubc14\ub9ac\ucf00\uc774\ud2b8\ub97c \ub118\uc5b4\uac00\ub294 \uc544\ube60\uc758 \ubaa8\uc2b5\uc744 \ubcf4\uc558\uc744 \ub54c, \ud1a0\ub2c8\ub294 \uaca9\ub178\ud558\uace0 \uad11\ubd80 \uc870\ud569\uacfc \ube4c\ub9ac\uc758 \uafc8\uc744 \uc704\ud574 \ube4c\ub9ac\ub97c \ub3d5\ub294 \uac83 \uc911 \ub354 \uc911\uc694\ud55c \uac83\uc774 \ubb34\uc5c7\uc778\uc9c0\uc5d0 \ub300\ud574 \uc2f8\uc6b4\ub2e4 (*He could be a Star). \ub17c\uc7c1\uc740 \uacb0\uad6d \uaca9\ud22c\ub97c \ubd80\ub974\uac8c\ub418\uace0 \uadf8 \uacfc\uc815\uc5d0\uc11c \ube4c\ub9ac\uac00 \ub73b\ud558\uc9c0\uc54a\uac8c \ub2e4\uce5c\ub2e4. \ucc28\ub840\ucc28\ub840\ub85c, \uad11\ubd80\ub4e4\uc740 \ube4c\ub9ac\uc758 \ud3b8\uc744 \ub4e4\uc5b4\uc8fc\uace0 \ud30c\uc5c5 \uc218\ub2f9\uc744 \uc8fc\uace0\uc790 \ud55c\ub2e4. \ud558\uc9c0\ub9cc \ube4c\ub9ac\ub294 \ub7f0\ub358\uc73c\ub85c \ud5a5\ud558\ub294 \ubc84\uc2a4 \uc694\uae08\uc744 \ub0bc \uc218 \uc5c6\ub2e4. \ud30c\uc5c5 \ubc29\ud574\uc790\ub294 \ube4c\ub9ac\uc5d0\uac8c \uc218\ubc31 \ud30c\uc6b4\ub4dc\uc758 \uc694\uae08\uc744 \uc694\uad6c\ud558\uace0, \ud654\ub09c \ud1a0\ub2c8\ub294 \ubc29\ud574\ud558\ub824 \ud558\uc9c0\ub9cc, \ube4c\ub9ac\ub294 \uadf8\ub7fc\uc5d0\ub3c4 \ubd88\uad6c\ud558\uace0 \ubc1b\uc544\ub4e4\uc778\ub2e4. \ub2f9\uc7a5 \ud76c\ub9dd\uc774 \uc0ac\ub77c\uc84c\uace0, \ud1a0\ub2c8\ub294 \ub354 \uc774\uc0c1 \ubc29\ubc95\uc774 \uc788\ub294\uc9c0\uc5d0 \ub300\ud574 \uc74c\uc6b8\ud788 \uc2ec\uc0ac\uc219\uace0\ud55c\ub2e4."} {"question": "\ub85c\ub9c8\uc758 \ub3c4\ub85c\ub97c \ud2bc\ud2bc\ud55c \ud3ec\uc7a5\ub3c4\ub85c\ub85c \ub9cc\ub4e4 \uc218 \uc788\uac8c \ud55c \uac83\uc740?", "paragraph": "\ucf58\ud06c\ub9ac\ud2b8 \ub355\ubd84\uc5d0 \ud2bc\ud2bc\ud55c \ud3ec\uc7a5\ub3c4\ub85c\uc778 \ub85c\ub9c8 \ub3c4\ub85c\ub97c \uae54 \uc218 \uc788\uac8c \ub418\uc5c8\ub294\ub370, \uc0c1\ub2f9\uc218 \ub85c\ub9c8 \ub3c4\ub85c\ub294 \ub85c\ub9c8\uac00 \uba78\ub9dd\ud55c\uc9c0 \ucc9c \ub144 \ub4a4\uc5d0\ub3c4 \uacc4\uc18d \uc4f0\uc600\ub2e4. \uad11\ub300\ud558\uace0 \ud6a8\uc728\uc801\uc778 \uc81c\uad6d \uc804\uc5ed\uc758 \ub3c4\ub85c\ub9dd\uc740 \ub85c\ub9c8\uc758 \ud328\uad8c\uacfc \uc601\ud5a5\ub825\uc744 \ud06c\uac8c \uc99d\ub300\uc2dc\ucf30\ub2e4. \ub85c\ub9c8 \ub3c4\ub85c\ub294 \uc804\uccb4\uc801\uc73c\ub85c 5\ub9cc \ub9c8\uc77c\uc5d0 \ub2ec\ud558\uc600\uace0, \ubcc0\ubc29\uc758 \ub3c4\ub85c\ub294 \uad70\uc0ac\uacf5\ubcd1 \uae30\uc220\uc790\uac00 \ucd95\uc870\ud558\uc600\ub294\ub370 \uc624\ub298\uae4c\uc9c0\ub3c4 \uac74\uc7ac\ud558\ub2e4. \ube44\ud2b8\ub8e8\ube44\uc6b0\uc2a4 \uc800\uc11c 12\uad8c 1\uc7a5\uc5d0 \uc788\ub294 \uacf5\ubc95\uc744 \ubcf4\uba74, \ub9e8 \uc544\ub798 \uae30\ucd08\ub85c \ud070 \ub3cc\uce35\uacfc \ubd80\uc2a4\ub7ec\uae30\uce35\uc744 \ubc88\uac08\uc544 \uae54\uace0, \uadf8 \uc704\uc5d0 \ubaa8\ub798\uce35\uc744 \ud3b4\uace0, \ub9e8 \uc704\uc5d0\ub294 \ub2e4\uac01\ud615 \ud070 \ub3cc\uc758 \uc0c1\ubd80\ub97c \uac08\uc544\uc11c \ud3c9\ud3c9\ud558\uac8c \ub9cc\ub4e4\uc5b4 \ub36e\ub294 \uac83\uc774\ub77c\uace0 \ud55c\ub2e4. \uc6d0\ub798 \ub85c\ub9c8 \ub3c4\ub85c\ub294 \ub85c\ub9c8 \uad70\ub2e8\uc774 \uc2e0\uc18d\ud558\uac8c \uc774\ub3d9\ud558\uae30 \uac74\uc124\ud55c \ub3c4\ub85c\uc600\ub2e4. \uadf8\ub7ec\ub098 \uc774 \ub300\ub85c\ub294 \ub85c\ub9c8\uac00 \uacbd\uc81c\uc801\uc73c\ub85c \ubc88\uc601\ud558\ub294 \ub370\uc5d0\ub3c4 \uc77c\uc775\uc744 \ub2f4\ub2f9\ud588\uc73c\uba70, \uad50\uc5ed\uc758 \uc911\uc2ec\uc9c0\uc778 \ub85c\ub9c8\uc758 \uc704\uc0c1\uc744 \uad73\ud600 \"\ubaa8\ub4e0 \uae38\uc740 \ub85c\ub9c8\ub85c \ud1b5\ud55c\ub2e4\"\ub294 \ub9d0\uc774 \ub098\uc654\ub2e4. \ub85c\ub9c8 \uc815\ubd80\ub294 \uc5ed\ucc38 \uc81c\ub3c4\ub97c \uc6b4\uc601\ud558\uc5ec, \ub3c4\ub85c \uc77c\uc815\ud55c \uac04\uaca9\ub9c8\ub2e4 \uc5ec\ud589\uc790\ub4e4\uc774 \uc274 \uacf3\uc744 \uc81c\uacf5\ud588\uc73c\uba70 \ud544\uc694\ud55c \uc9c0\uc5ed\uc5d0 \ub2e4\ub9ac\ub97c \uac74\uc124\ud558\uc5ec 24\uc2dc\uac04\uc5d0 800km\ub97c \uc774\ub3d9\ud560 \uc218 \uc788\ub294 \ud30c\ubc1c \uccb4\uc81c\ub97c \ub9c8\ub828\ud588\ub2e4.", "answer": "\ucf58\ud06c\ub9ac\ud2b8", "sentence": "\ucf58\ud06c\ub9ac\ud2b8 \ub355\ubd84\uc5d0 \ud2bc\ud2bc\ud55c \ud3ec\uc7a5\ub3c4\ub85c\uc778 \ub85c\ub9c8 \ub3c4\ub85c\ub97c \uae54 \uc218 \uc788\uac8c \ub418\uc5c8\ub294\ub370, \uc0c1\ub2f9\uc218 \ub85c\ub9c8 \ub3c4\ub85c\ub294 \ub85c\ub9c8\uac00 \uba78\ub9dd\ud55c\uc9c0 \ucc9c \ub144 \ub4a4\uc5d0\ub3c4 \uacc4\uc18d \uc4f0\uc600\ub2e4.", "paragraph_sentence": " \ucf58\ud06c\ub9ac\ud2b8 \ub355\ubd84\uc5d0 \ud2bc\ud2bc\ud55c \ud3ec\uc7a5\ub3c4\ub85c\uc778 \ub85c\ub9c8 \ub3c4\ub85c\ub97c \uae54 \uc218 \uc788\uac8c \ub418\uc5c8\ub294\ub370, \uc0c1\ub2f9\uc218 \ub85c\ub9c8 \ub3c4\ub85c\ub294 \ub85c\ub9c8\uac00 \uba78\ub9dd\ud55c\uc9c0 \ucc9c \ub144 \ub4a4\uc5d0\ub3c4 \uacc4\uc18d \uc4f0\uc600\ub2e4. \uad11\ub300\ud558\uace0 \ud6a8\uc728\uc801\uc778 \uc81c\uad6d \uc804\uc5ed\uc758 \ub3c4\ub85c\ub9dd\uc740 \ub85c\ub9c8\uc758 \ud328\uad8c\uacfc \uc601\ud5a5\ub825\uc744 \ud06c\uac8c \uc99d\ub300\uc2dc\ucf30\ub2e4. \ub85c\ub9c8 \ub3c4\ub85c\ub294 \uc804\uccb4\uc801\uc73c\ub85c 5\ub9cc \ub9c8\uc77c\uc5d0 \ub2ec\ud558\uc600\uace0, \ubcc0\ubc29\uc758 \ub3c4\ub85c\ub294 \uad70\uc0ac\uacf5\ubcd1 \uae30\uc220\uc790\uac00 \ucd95\uc870\ud558\uc600\ub294\ub370 \uc624\ub298\uae4c\uc9c0\ub3c4 \uac74\uc7ac\ud558\ub2e4. \ube44\ud2b8\ub8e8\ube44\uc6b0\uc2a4 \uc800\uc11c 12\uad8c 1\uc7a5\uc5d0 \uc788\ub294 \uacf5\ubc95\uc744 \ubcf4\uba74, \ub9e8 \uc544\ub798 \uae30\ucd08\ub85c \ud070 \ub3cc\uce35\uacfc \ubd80\uc2a4\ub7ec\uae30\uce35\uc744 \ubc88\uac08\uc544 \uae54\uace0, \uadf8 \uc704\uc5d0 \ubaa8\ub798\uce35\uc744 \ud3b4\uace0, \ub9e8 \uc704\uc5d0\ub294 \ub2e4\uac01\ud615 \ud070 \ub3cc\uc758 \uc0c1\ubd80\ub97c \uac08\uc544\uc11c \ud3c9\ud3c9\ud558\uac8c \ub9cc\ub4e4\uc5b4 \ub36e\ub294 \uac83\uc774\ub77c\uace0 \ud55c\ub2e4. \uc6d0\ub798 \ub85c\ub9c8 \ub3c4\ub85c\ub294 \ub85c\ub9c8 \uad70\ub2e8\uc774 \uc2e0\uc18d\ud558\uac8c \uc774\ub3d9\ud558\uae30 \uac74\uc124\ud55c \ub3c4\ub85c\uc600\ub2e4. \uadf8\ub7ec\ub098 \uc774 \ub300\ub85c\ub294 \ub85c\ub9c8\uac00 \uacbd\uc81c\uc801\uc73c\ub85c \ubc88\uc601\ud558\ub294 \ub370\uc5d0\ub3c4 \uc77c\uc775\uc744 \ub2f4\ub2f9\ud588\uc73c\uba70, \uad50\uc5ed\uc758 \uc911\uc2ec\uc9c0\uc778 \ub85c\ub9c8\uc758 \uc704\uc0c1\uc744 \uad73\ud600 \"\ubaa8\ub4e0 \uae38\uc740 \ub85c\ub9c8\ub85c \ud1b5\ud55c\ub2e4\"\ub294 \ub9d0\uc774 \ub098\uc654\ub2e4. \ub85c\ub9c8 \uc815\ubd80\ub294 \uc5ed\ucc38 \uc81c\ub3c4\ub97c \uc6b4\uc601\ud558\uc5ec, \ub3c4\ub85c \uc77c\uc815\ud55c \uac04\uaca9\ub9c8\ub2e4 \uc5ec\ud589\uc790\ub4e4\uc774 \uc274 \uacf3\uc744 \uc81c\uacf5\ud588\uc73c\uba70 \ud544\uc694\ud55c \uc9c0\uc5ed\uc5d0 \ub2e4\ub9ac\ub97c \uac74\uc124\ud558\uc5ec 24\uc2dc\uac04\uc5d0 800km\ub97c \uc774\ub3d9\ud560 \uc218 \uc788\ub294 \ud30c\ubc1c \uccb4\uc81c\ub97c \ub9c8\ub828\ud588\ub2e4.", "paragraph_answer": " \ucf58\ud06c\ub9ac\ud2b8 \ub355\ubd84\uc5d0 \ud2bc\ud2bc\ud55c \ud3ec\uc7a5\ub3c4\ub85c\uc778 \ub85c\ub9c8 \ub3c4\ub85c\ub97c \uae54 \uc218 \uc788\uac8c \ub418\uc5c8\ub294\ub370, \uc0c1\ub2f9\uc218 \ub85c\ub9c8 \ub3c4\ub85c\ub294 \ub85c\ub9c8\uac00 \uba78\ub9dd\ud55c\uc9c0 \ucc9c \ub144 \ub4a4\uc5d0\ub3c4 \uacc4\uc18d \uc4f0\uc600\ub2e4. \uad11\ub300\ud558\uace0 \ud6a8\uc728\uc801\uc778 \uc81c\uad6d \uc804\uc5ed\uc758 \ub3c4\ub85c\ub9dd\uc740 \ub85c\ub9c8\uc758 \ud328\uad8c\uacfc \uc601\ud5a5\ub825\uc744 \ud06c\uac8c \uc99d\ub300\uc2dc\ucf30\ub2e4. \ub85c\ub9c8 \ub3c4\ub85c\ub294 \uc804\uccb4\uc801\uc73c\ub85c 5\ub9cc \ub9c8\uc77c\uc5d0 \ub2ec\ud558\uc600\uace0, \ubcc0\ubc29\uc758 \ub3c4\ub85c\ub294 \uad70\uc0ac\uacf5\ubcd1 \uae30\uc220\uc790\uac00 \ucd95\uc870\ud558\uc600\ub294\ub370 \uc624\ub298\uae4c\uc9c0\ub3c4 \uac74\uc7ac\ud558\ub2e4. \ube44\ud2b8\ub8e8\ube44\uc6b0\uc2a4 \uc800\uc11c 12\uad8c 1\uc7a5\uc5d0 \uc788\ub294 \uacf5\ubc95\uc744 \ubcf4\uba74, \ub9e8 \uc544\ub798 \uae30\ucd08\ub85c \ud070 \ub3cc\uce35\uacfc \ubd80\uc2a4\ub7ec\uae30\uce35\uc744 \ubc88\uac08\uc544 \uae54\uace0, \uadf8 \uc704\uc5d0 \ubaa8\ub798\uce35\uc744 \ud3b4\uace0, \ub9e8 \uc704\uc5d0\ub294 \ub2e4\uac01\ud615 \ud070 \ub3cc\uc758 \uc0c1\ubd80\ub97c \uac08\uc544\uc11c \ud3c9\ud3c9\ud558\uac8c \ub9cc\ub4e4\uc5b4 \ub36e\ub294 \uac83\uc774\ub77c\uace0 \ud55c\ub2e4. \uc6d0\ub798 \ub85c\ub9c8 \ub3c4\ub85c\ub294 \ub85c\ub9c8 \uad70\ub2e8\uc774 \uc2e0\uc18d\ud558\uac8c \uc774\ub3d9\ud558\uae30 \uac74\uc124\ud55c \ub3c4\ub85c\uc600\ub2e4. \uadf8\ub7ec\ub098 \uc774 \ub300\ub85c\ub294 \ub85c\ub9c8\uac00 \uacbd\uc81c\uc801\uc73c\ub85c \ubc88\uc601\ud558\ub294 \ub370\uc5d0\ub3c4 \uc77c\uc775\uc744 \ub2f4\ub2f9\ud588\uc73c\uba70, \uad50\uc5ed\uc758 \uc911\uc2ec\uc9c0\uc778 \ub85c\ub9c8\uc758 \uc704\uc0c1\uc744 \uad73\ud600 \"\ubaa8\ub4e0 \uae38\uc740 \ub85c\ub9c8\ub85c \ud1b5\ud55c\ub2e4\"\ub294 \ub9d0\uc774 \ub098\uc654\ub2e4. \ub85c\ub9c8 \uc815\ubd80\ub294 \uc5ed\ucc38 \uc81c\ub3c4\ub97c \uc6b4\uc601\ud558\uc5ec, \ub3c4\ub85c \uc77c\uc815\ud55c \uac04\uaca9\ub9c8\ub2e4 \uc5ec\ud589\uc790\ub4e4\uc774 \uc274 \uacf3\uc744 \uc81c\uacf5\ud588\uc73c\uba70 \ud544\uc694\ud55c \uc9c0\uc5ed\uc5d0 \ub2e4\ub9ac\ub97c \uac74\uc124\ud558\uc5ec 24\uc2dc\uac04\uc5d0 800km\ub97c \uc774\ub3d9\ud560 \uc218 \uc788\ub294 \ud30c\ubc1c \uccb4\uc81c\ub97c \ub9c8\ub828\ud588\ub2e4.", "sentence_answer": " \ucf58\ud06c\ub9ac\ud2b8 \ub355\ubd84\uc5d0 \ud2bc\ud2bc\ud55c \ud3ec\uc7a5\ub3c4\ub85c\uc778 \ub85c\ub9c8 \ub3c4\ub85c\ub97c \uae54 \uc218 \uc788\uac8c \ub418\uc5c8\ub294\ub370, \uc0c1\ub2f9\uc218 \ub85c\ub9c8 \ub3c4\ub85c\ub294 \ub85c\ub9c8\uac00 \uba78\ub9dd\ud55c\uc9c0 \ucc9c \ub144 \ub4a4\uc5d0\ub3c4 \uacc4\uc18d \uc4f0\uc600\ub2e4.", "paragraph_id": "6527248-23-0", "paragraph_question": "question: \ub85c\ub9c8\uc758 \ub3c4\ub85c\ub97c \ud2bc\ud2bc\ud55c \ud3ec\uc7a5\ub3c4\ub85c\ub85c \ub9cc\ub4e4 \uc218 \uc788\uac8c \ud55c \uac83\uc740?, context: \ucf58\ud06c\ub9ac\ud2b8 \ub355\ubd84\uc5d0 \ud2bc\ud2bc\ud55c \ud3ec\uc7a5\ub3c4\ub85c\uc778 \ub85c\ub9c8 \ub3c4\ub85c\ub97c \uae54 \uc218 \uc788\uac8c \ub418\uc5c8\ub294\ub370, \uc0c1\ub2f9\uc218 \ub85c\ub9c8 \ub3c4\ub85c\ub294 \ub85c\ub9c8\uac00 \uba78\ub9dd\ud55c\uc9c0 \ucc9c \ub144 \ub4a4\uc5d0\ub3c4 \uacc4\uc18d \uc4f0\uc600\ub2e4. \uad11\ub300\ud558\uace0 \ud6a8\uc728\uc801\uc778 \uc81c\uad6d \uc804\uc5ed\uc758 \ub3c4\ub85c\ub9dd\uc740 \ub85c\ub9c8\uc758 \ud328\uad8c\uacfc \uc601\ud5a5\ub825\uc744 \ud06c\uac8c \uc99d\ub300\uc2dc\ucf30\ub2e4. \ub85c\ub9c8 \ub3c4\ub85c\ub294 \uc804\uccb4\uc801\uc73c\ub85c 5\ub9cc \ub9c8\uc77c\uc5d0 \ub2ec\ud558\uc600\uace0, \ubcc0\ubc29\uc758 \ub3c4\ub85c\ub294 \uad70\uc0ac\uacf5\ubcd1 \uae30\uc220\uc790\uac00 \ucd95\uc870\ud558\uc600\ub294\ub370 \uc624\ub298\uae4c\uc9c0\ub3c4 \uac74\uc7ac\ud558\ub2e4. \ube44\ud2b8\ub8e8\ube44\uc6b0\uc2a4 \uc800\uc11c 12\uad8c 1\uc7a5\uc5d0 \uc788\ub294 \uacf5\ubc95\uc744 \ubcf4\uba74, \ub9e8 \uc544\ub798 \uae30\ucd08\ub85c \ud070 \ub3cc\uce35\uacfc \ubd80\uc2a4\ub7ec\uae30\uce35\uc744 \ubc88\uac08\uc544 \uae54\uace0, \uadf8 \uc704\uc5d0 \ubaa8\ub798\uce35\uc744 \ud3b4\uace0, \ub9e8 \uc704\uc5d0\ub294 \ub2e4\uac01\ud615 \ud070 \ub3cc\uc758 \uc0c1\ubd80\ub97c \uac08\uc544\uc11c \ud3c9\ud3c9\ud558\uac8c \ub9cc\ub4e4\uc5b4 \ub36e\ub294 \uac83\uc774\ub77c\uace0 \ud55c\ub2e4. \uc6d0\ub798 \ub85c\ub9c8 \ub3c4\ub85c\ub294 \ub85c\ub9c8 \uad70\ub2e8\uc774 \uc2e0\uc18d\ud558\uac8c \uc774\ub3d9\ud558\uae30 \uac74\uc124\ud55c \ub3c4\ub85c\uc600\ub2e4. \uadf8\ub7ec\ub098 \uc774 \ub300\ub85c\ub294 \ub85c\ub9c8\uac00 \uacbd\uc81c\uc801\uc73c\ub85c \ubc88\uc601\ud558\ub294 \ub370\uc5d0\ub3c4 \uc77c\uc775\uc744 \ub2f4\ub2f9\ud588\uc73c\uba70, \uad50\uc5ed\uc758 \uc911\uc2ec\uc9c0\uc778 \ub85c\ub9c8\uc758 \uc704\uc0c1\uc744 \uad73\ud600 \"\ubaa8\ub4e0 \uae38\uc740 \ub85c\ub9c8\ub85c \ud1b5\ud55c\ub2e4\"\ub294 \ub9d0\uc774 \ub098\uc654\ub2e4. \ub85c\ub9c8 \uc815\ubd80\ub294 \uc5ed\ucc38 \uc81c\ub3c4\ub97c \uc6b4\uc601\ud558\uc5ec, \ub3c4\ub85c \uc77c\uc815\ud55c \uac04\uaca9\ub9c8\ub2e4 \uc5ec\ud589\uc790\ub4e4\uc774 \uc274 \uacf3\uc744 \uc81c\uacf5\ud588\uc73c\uba70 \ud544\uc694\ud55c \uc9c0\uc5ed\uc5d0 \ub2e4\ub9ac\ub97c \uac74\uc124\ud558\uc5ec 24\uc2dc\uac04\uc5d0 800km\ub97c \uc774\ub3d9\ud560 \uc218 \uc788\ub294 \ud30c\ubc1c \uccb4\uc81c\ub97c \ub9c8\ub828\ud588\ub2e4."} {"question": "\ub8e8\uc2dc\uc6b0\uc2a4 \uc720\ub2c8\uc6b0\uc2a4 \ube0c\ub8e8\ud22c\uc2a4\uc5d0\uc11c \ub8e8\uc2dc\uc6b0\uc2a4\uc758 \ub73b\uc740?", "paragraph": "\ud584\ub9bf\uacfc \uac19\uc740 \"\ubc14\ubcf4 \uc601\uc6c5\"\uc758 \uc6d0\ud615\uc740 \uc6d0\uc2dc \uc778\ub3c4 \uc720\ub7fd\uc5b4\uc871\uae4c\uc9c0 \uc18c\uae09\ub418\uba70 \uc774\ud0c8\ub9ac\uc544, \uc5d0\uc2a4\ud30c\ub0d0, \uc2a4\uce78\ub2e4\ub098\ube44\uc544 \ub4f1\uc9c0\uc758 \uc5ec\ub7ec \uc2e0\ud654\uc640 \uc774\uc57c\uae30\uc5d0\uc11c \uad11\ubc94\uc704\ud558\uac8c \ub2e4\ub8e8\uc5b4\uc9c0\uace0 \uc788\ub2e4. \ud584\ub9bf \uc774\uc57c\uae30\uc758 \uc6d0\ud615\uc744 \ud615\uc131\ud558\ub294 \uac83\uc73c\ub85c\ub294 \uc791\uac00\ubbf8\uc0c1\uc758 \uc2a4\uce78\ub2e4\ub098\ube44\uc544 \uc11c\uc0ac\uc2dc\uc778 \ud750\ub8b8\ud504\uc2a4 \uc0ac\uac00 \ud06c\ub77c\uce74(Hr\u00f3lfs saga kraka, \ud750\ub8b8\ud504\uc2a4 \ud06c\ub77c\uce74 \uc655\uc758 \uc774\uc57c\uae30)\ub97c \uaf3d\uc744 \uc218 \uc788\ub2e4. \uc774 \uc774\uc57c\uae30\uc5d0\ub294 \uc0b4\ud574\ub2f9\ud55c \uc655\uacfc \uadf8\uc758 \ub450 \uc544\ub4e4\uc774 \ub4f1\uc7a5\ud558\uba70 \ub9ce\uc740 \ubd80\ubd84\uc5d0\uc11c \ud584\ub9bf\uacfc \ube44\uc2b7\ud55c \uc791\ud488 \uad6c\uc131\uc744 \ubcf4\uc778\ub2e4. \ud584\ub9bf\uc758 \ub610 \ub2e4\ub978 \uc6d0\uc804\uc73c\ub85c\ub294 \ub8e8\uc2dc\uc6b0\uc2a4 \uc720\ub2c8\uc6b0\uc2a4 \ube0c\ub8e8\ud22c\uc2a4(Lucius Junius Brutus)\uc758 \uc804\uc124\uc774 \uc788\ub2e4. \uadf8\uc758 \uc774\ub984\uc5d0\uc11c \ub8e8\uc2dc\uc6b0\uc2a4\ub294 \"\ube5b\"\uc744 \ube0c\ub8e8\ud22c\uc2a4\ub294 \"\ubc14\ubcf4\"\ub97c \ub73b\ud55c\ub2e4. \uadf8\ub294 \uc790\uc2e0\uc758 \uc544\ubc84\uc9c0\uc640 \ud615\uc81c\ub97c \uc0b4\ud574\ud55c \ud0c0\ub974\ud034\ub2c8\uc6b0\uc2a4\uc5d0\uac8c \ubcf5\uc218\ud558\uae30 \uc704\ud574 \uc2a4\uc2a4\ub85c \ubbf8\uce5c\ucc99\ud558\uc600\ub2e4. 17\uc138\uae30\uc758 \ub178\ub974\ub9cc \ud559\uc790 \ud1a0\ub974\ud398\uc6b0\uc2a4\ub294 \uc170\uc775\uc2a4\ud53c\uc5b4\uc758 \ud584\ub9bf\uacfc \uc544\uc774\uc2ac\ub79c\ub4dc\uc758 \uc554\ub85c\ub514 \uc774\uc57c\uae30 \ubc0f \uc5d0\uc2a4\ud30c\ub0d0\uc758 \uc554\ubc1c\ub808\uc2a4 \uc655\uc790 \uc774\uc57c\uae30\ub97c \ube44\uad50\ud558\uc600\ub2e4. \uc774\ub4e4 \uc774\uc57c\uae30 \ubaa8\ub450 \uc5b4\uba38\ub2c8\uc758 \uce68\uc2e4\uc5d0 \uc228\uc5b4\ub4e0 \uc7ac\uc0c1\uc744 \uc655\uc73c\ub85c \ucc29\uac01\ud558\uc5ec \uc0b4\ud574\ud558\ub294 \ubd80\ubd84\uc774 \ub4f1\uc7a5\ud55c\ub2e4.", "answer": "\ube5b", "sentence": "\uadf8\uc758 \uc774\ub984\uc5d0\uc11c \ub8e8\uc2dc\uc6b0\uc2a4\ub294 \" \ube5b \"\uc744 \ube0c\ub8e8\ud22c\uc2a4\ub294 \"\ubc14\ubcf4\"\ub97c \ub73b\ud55c\ub2e4.", "paragraph_sentence": "\ud584\ub9bf\uacfc \uac19\uc740 \"\ubc14\ubcf4 \uc601\uc6c5\"\uc758 \uc6d0\ud615\uc740 \uc6d0\uc2dc \uc778\ub3c4 \uc720\ub7fd\uc5b4\uc871\uae4c\uc9c0 \uc18c\uae09\ub418\uba70 \uc774\ud0c8\ub9ac\uc544, \uc5d0\uc2a4\ud30c\ub0d0, \uc2a4\uce78\ub2e4\ub098\ube44\uc544 \ub4f1\uc9c0\uc758 \uc5ec\ub7ec \uc2e0\ud654\uc640 \uc774\uc57c\uae30\uc5d0\uc11c \uad11\ubc94\uc704\ud558\uac8c \ub2e4\ub8e8\uc5b4\uc9c0\uace0 \uc788\ub2e4. \ud584\ub9bf \uc774\uc57c\uae30\uc758 \uc6d0\ud615\uc744 \ud615\uc131\ud558\ub294 \uac83\uc73c\ub85c\ub294 \uc791\uac00\ubbf8\uc0c1\uc758 \uc2a4\uce78\ub2e4\ub098\ube44\uc544 \uc11c\uc0ac\uc2dc\uc778 \ud750\ub8b8\ud504\uc2a4 \uc0ac\uac00 \ud06c\ub77c\uce74(Hr\u00f3lfs saga kraka, \ud750\ub8b8\ud504\uc2a4 \ud06c\ub77c\uce74 \uc655\uc758 \uc774\uc57c\uae30)\ub97c \uaf3d\uc744 \uc218 \uc788\ub2e4. \uc774 \uc774\uc57c\uae30\uc5d0\ub294 \uc0b4\ud574\ub2f9\ud55c \uc655\uacfc \uadf8\uc758 \ub450 \uc544\ub4e4\uc774 \ub4f1\uc7a5\ud558\uba70 \ub9ce\uc740 \ubd80\ubd84\uc5d0\uc11c \ud584\ub9bf\uacfc \ube44\uc2b7\ud55c \uc791\ud488 \uad6c\uc131\uc744 \ubcf4\uc778\ub2e4. \ud584\ub9bf\uc758 \ub610 \ub2e4\ub978 \uc6d0\uc804\uc73c\ub85c\ub294 \ub8e8\uc2dc\uc6b0\uc2a4 \uc720\ub2c8\uc6b0\uc2a4 \ube0c\ub8e8\ud22c\uc2a4(Lucius Junius Brutus)\uc758 \uc804\uc124\uc774 \uc788\ub2e4. \uadf8\uc758 \uc774\ub984\uc5d0\uc11c \ub8e8\uc2dc\uc6b0\uc2a4\ub294 \" \ube5b \"\uc744 \ube0c\ub8e8\ud22c\uc2a4\ub294 \"\ubc14\ubcf4\"\ub97c \ub73b\ud55c\ub2e4. \uadf8\ub294 \uc790\uc2e0\uc758 \uc544\ubc84\uc9c0\uc640 \ud615\uc81c\ub97c \uc0b4\ud574\ud55c \ud0c0\ub974\ud034\ub2c8\uc6b0\uc2a4\uc5d0\uac8c \ubcf5\uc218\ud558\uae30 \uc704\ud574 \uc2a4\uc2a4\ub85c \ubbf8\uce5c\ucc99\ud558\uc600\ub2e4. 17\uc138\uae30\uc758 \ub178\ub974\ub9cc \ud559\uc790 \ud1a0\ub974\ud398\uc6b0\uc2a4\ub294 \uc170\uc775\uc2a4\ud53c\uc5b4\uc758 \ud584\ub9bf\uacfc \uc544\uc774\uc2ac\ub79c\ub4dc\uc758 \uc554\ub85c\ub514 \uc774\uc57c\uae30 \ubc0f \uc5d0\uc2a4\ud30c\ub0d0\uc758 \uc554\ubc1c\ub808\uc2a4 \uc655\uc790 \uc774\uc57c\uae30\ub97c \ube44\uad50\ud558\uc600\ub2e4. \uc774\ub4e4 \uc774\uc57c\uae30 \ubaa8\ub450 \uc5b4\uba38\ub2c8\uc758 \uce68\uc2e4\uc5d0 \uc228\uc5b4\ub4e0 \uc7ac\uc0c1\uc744 \uc655\uc73c\ub85c \ucc29\uac01\ud558\uc5ec \uc0b4\ud574\ud558\ub294 \ubd80\ubd84\uc774 \ub4f1\uc7a5\ud55c\ub2e4.", "paragraph_answer": "\ud584\ub9bf\uacfc \uac19\uc740 \"\ubc14\ubcf4 \uc601\uc6c5\"\uc758 \uc6d0\ud615\uc740 \uc6d0\uc2dc \uc778\ub3c4 \uc720\ub7fd\uc5b4\uc871\uae4c\uc9c0 \uc18c\uae09\ub418\uba70 \uc774\ud0c8\ub9ac\uc544, \uc5d0\uc2a4\ud30c\ub0d0, \uc2a4\uce78\ub2e4\ub098\ube44\uc544 \ub4f1\uc9c0\uc758 \uc5ec\ub7ec \uc2e0\ud654\uc640 \uc774\uc57c\uae30\uc5d0\uc11c \uad11\ubc94\uc704\ud558\uac8c \ub2e4\ub8e8\uc5b4\uc9c0\uace0 \uc788\ub2e4. \ud584\ub9bf \uc774\uc57c\uae30\uc758 \uc6d0\ud615\uc744 \ud615\uc131\ud558\ub294 \uac83\uc73c\ub85c\ub294 \uc791\uac00\ubbf8\uc0c1\uc758 \uc2a4\uce78\ub2e4\ub098\ube44\uc544 \uc11c\uc0ac\uc2dc\uc778 \ud750\ub8b8\ud504\uc2a4 \uc0ac\uac00 \ud06c\ub77c\uce74(Hr\u00f3lfs saga kraka, \ud750\ub8b8\ud504\uc2a4 \ud06c\ub77c\uce74 \uc655\uc758 \uc774\uc57c\uae30)\ub97c \uaf3d\uc744 \uc218 \uc788\ub2e4. \uc774 \uc774\uc57c\uae30\uc5d0\ub294 \uc0b4\ud574\ub2f9\ud55c \uc655\uacfc \uadf8\uc758 \ub450 \uc544\ub4e4\uc774 \ub4f1\uc7a5\ud558\uba70 \ub9ce\uc740 \ubd80\ubd84\uc5d0\uc11c \ud584\ub9bf\uacfc \ube44\uc2b7\ud55c \uc791\ud488 \uad6c\uc131\uc744 \ubcf4\uc778\ub2e4. \ud584\ub9bf\uc758 \ub610 \ub2e4\ub978 \uc6d0\uc804\uc73c\ub85c\ub294 \ub8e8\uc2dc\uc6b0\uc2a4 \uc720\ub2c8\uc6b0\uc2a4 \ube0c\ub8e8\ud22c\uc2a4(Lucius Junius Brutus)\uc758 \uc804\uc124\uc774 \uc788\ub2e4. \uadf8\uc758 \uc774\ub984\uc5d0\uc11c \ub8e8\uc2dc\uc6b0\uc2a4\ub294 \" \ube5b \"\uc744 \ube0c\ub8e8\ud22c\uc2a4\ub294 \"\ubc14\ubcf4\"\ub97c \ub73b\ud55c\ub2e4. \uadf8\ub294 \uc790\uc2e0\uc758 \uc544\ubc84\uc9c0\uc640 \ud615\uc81c\ub97c \uc0b4\ud574\ud55c \ud0c0\ub974\ud034\ub2c8\uc6b0\uc2a4\uc5d0\uac8c \ubcf5\uc218\ud558\uae30 \uc704\ud574 \uc2a4\uc2a4\ub85c \ubbf8\uce5c\ucc99\ud558\uc600\ub2e4. 17\uc138\uae30\uc758 \ub178\ub974\ub9cc \ud559\uc790 \ud1a0\ub974\ud398\uc6b0\uc2a4\ub294 \uc170\uc775\uc2a4\ud53c\uc5b4\uc758 \ud584\ub9bf\uacfc \uc544\uc774\uc2ac\ub79c\ub4dc\uc758 \uc554\ub85c\ub514 \uc774\uc57c\uae30 \ubc0f \uc5d0\uc2a4\ud30c\ub0d0\uc758 \uc554\ubc1c\ub808\uc2a4 \uc655\uc790 \uc774\uc57c\uae30\ub97c \ube44\uad50\ud558\uc600\ub2e4. \uc774\ub4e4 \uc774\uc57c\uae30 \ubaa8\ub450 \uc5b4\uba38\ub2c8\uc758 \uce68\uc2e4\uc5d0 \uc228\uc5b4\ub4e0 \uc7ac\uc0c1\uc744 \uc655\uc73c\ub85c \ucc29\uac01\ud558\uc5ec \uc0b4\ud574\ud558\ub294 \ubd80\ubd84\uc774 \ub4f1\uc7a5\ud55c\ub2e4.", "sentence_answer": "\uadf8\uc758 \uc774\ub984\uc5d0\uc11c \ub8e8\uc2dc\uc6b0\uc2a4\ub294 \" \ube5b \"\uc744 \ube0c\ub8e8\ud22c\uc2a4\ub294 \"\ubc14\ubcf4\"\ub97c \ub73b\ud55c\ub2e4.", "paragraph_id": "6570869-3-1", "paragraph_question": "question: \ub8e8\uc2dc\uc6b0\uc2a4 \uc720\ub2c8\uc6b0\uc2a4 \ube0c\ub8e8\ud22c\uc2a4\uc5d0\uc11c \ub8e8\uc2dc\uc6b0\uc2a4\uc758 \ub73b\uc740?, context: \ud584\ub9bf\uacfc \uac19\uc740 \"\ubc14\ubcf4 \uc601\uc6c5\"\uc758 \uc6d0\ud615\uc740 \uc6d0\uc2dc \uc778\ub3c4 \uc720\ub7fd\uc5b4\uc871\uae4c\uc9c0 \uc18c\uae09\ub418\uba70 \uc774\ud0c8\ub9ac\uc544, \uc5d0\uc2a4\ud30c\ub0d0, \uc2a4\uce78\ub2e4\ub098\ube44\uc544 \ub4f1\uc9c0\uc758 \uc5ec\ub7ec \uc2e0\ud654\uc640 \uc774\uc57c\uae30\uc5d0\uc11c \uad11\ubc94\uc704\ud558\uac8c \ub2e4\ub8e8\uc5b4\uc9c0\uace0 \uc788\ub2e4. \ud584\ub9bf \uc774\uc57c\uae30\uc758 \uc6d0\ud615\uc744 \ud615\uc131\ud558\ub294 \uac83\uc73c\ub85c\ub294 \uc791\uac00\ubbf8\uc0c1\uc758 \uc2a4\uce78\ub2e4\ub098\ube44\uc544 \uc11c\uc0ac\uc2dc\uc778 \ud750\ub8b8\ud504\uc2a4 \uc0ac\uac00 \ud06c\ub77c\uce74(Hr\u00f3lfs saga kraka, \ud750\ub8b8\ud504\uc2a4 \ud06c\ub77c\uce74 \uc655\uc758 \uc774\uc57c\uae30)\ub97c \uaf3d\uc744 \uc218 \uc788\ub2e4. \uc774 \uc774\uc57c\uae30\uc5d0\ub294 \uc0b4\ud574\ub2f9\ud55c \uc655\uacfc \uadf8\uc758 \ub450 \uc544\ub4e4\uc774 \ub4f1\uc7a5\ud558\uba70 \ub9ce\uc740 \ubd80\ubd84\uc5d0\uc11c \ud584\ub9bf\uacfc \ube44\uc2b7\ud55c \uc791\ud488 \uad6c\uc131\uc744 \ubcf4\uc778\ub2e4. \ud584\ub9bf\uc758 \ub610 \ub2e4\ub978 \uc6d0\uc804\uc73c\ub85c\ub294 \ub8e8\uc2dc\uc6b0\uc2a4 \uc720\ub2c8\uc6b0\uc2a4 \ube0c\ub8e8\ud22c\uc2a4(Lucius Junius Brutus)\uc758 \uc804\uc124\uc774 \uc788\ub2e4. \uadf8\uc758 \uc774\ub984\uc5d0\uc11c \ub8e8\uc2dc\uc6b0\uc2a4\ub294 \"\ube5b\"\uc744 \ube0c\ub8e8\ud22c\uc2a4\ub294 \"\ubc14\ubcf4\"\ub97c \ub73b\ud55c\ub2e4. \uadf8\ub294 \uc790\uc2e0\uc758 \uc544\ubc84\uc9c0\uc640 \ud615\uc81c\ub97c \uc0b4\ud574\ud55c \ud0c0\ub974\ud034\ub2c8\uc6b0\uc2a4\uc5d0\uac8c \ubcf5\uc218\ud558\uae30 \uc704\ud574 \uc2a4\uc2a4\ub85c \ubbf8\uce5c\ucc99\ud558\uc600\ub2e4. 17\uc138\uae30\uc758 \ub178\ub974\ub9cc \ud559\uc790 \ud1a0\ub974\ud398\uc6b0\uc2a4\ub294 \uc170\uc775\uc2a4\ud53c\uc5b4\uc758 \ud584\ub9bf\uacfc \uc544\uc774\uc2ac\ub79c\ub4dc\uc758 \uc554\ub85c\ub514 \uc774\uc57c\uae30 \ubc0f \uc5d0\uc2a4\ud30c\ub0d0\uc758 \uc554\ubc1c\ub808\uc2a4 \uc655\uc790 \uc774\uc57c\uae30\ub97c \ube44\uad50\ud558\uc600\ub2e4. \uc774\ub4e4 \uc774\uc57c\uae30 \ubaa8\ub450 \uc5b4\uba38\ub2c8\uc758 \uce68\uc2e4\uc5d0 \uc228\uc5b4\ub4e0 \uc7ac\uc0c1\uc744 \uc655\uc73c\ub85c \ucc29\uac01\ud558\uc5ec \uc0b4\ud574\ud558\ub294 \ubd80\ubd84\uc774 \ub4f1\uc7a5\ud55c\ub2e4."} {"question": "\ub2e5\ud130\ud6c4\uc758 \ucd1d\uad04 \ud504\ub85c\ub4c0\uc11c \uc774\ub984\uc740 \ubb34\uc5c7\uc778\uac00?", "paragraph": "2004\ub144 5\uc6d4 24\uc77c\uc5d0\ub294 \ube4c\ub9ac \ud30c\uc774\ud37c\uac00 \ub85c\uc988 \ud0c0\uc77c\ub7ec \uc5ed\uc73c\ub85c \uce90\uc2a4\ud305\ub418\uc5c8\ub2e4\ub294 \uc18c\uc2dd\uc774 \ubc1c\ud45c\ub410\ub2e4. \ucd1d\uad04 \ud504\ub85c\ub4c0\uc11c\uc778 \uc904\ub9ac \uac00\ub4dc\ub108\uc5d0 \ub530\ub974\uba74, \uc804 \ud31d\uc2a4\ud0c0\uc600\ub358 \ud30c\uc774\ud37c\ub294 \"\ud06c\ub9ac\uc2a4\ud1a0\ud37c \uc5d0\ud074\uc2a4\ud134\uc744 \uc704\ud55c \ub3c5\ud2b9\ud558\uace0 \ud65c\ub825\uc788\ub294 \ub3d9\ubc18\uc790\"\ub85c\uc368 \"\ub4dc\ub77c\ub9c8\uc5d0 \uc644\ubcbd\ud558\uac8c \uc54c\ub9de\ub2e4\"\uace0 \ub9d0\ud588\ub2e4. \ub370\uc774\ube44\uc2a4\ub294 \ub85c\uc988\ub97c \"\ubb54\uac00 \uae30\uc774\ud55c \uac83\uc5d0 \ud718\ub9d0\ub9ac\uace0 \uadf8\uac83\ub4e4\uc758 \ub3d9\ub4f1\ud55c \uac83\uc744 \uc2a4\uc2a4\ub85c \ubc1c\uacac\ud574\ub0b4\ub294 \ud3c9\ubc94\ud55c \uc0ac\ub78c\"\uc73c\ub85c \ubb18\uc0ac\ud588\ub2e4. \ucee4\ubc00 \ucf54\ub450\ub9ac\uc640 \ub178\uc5d8 \ud074\ub77c\ud06c\ub3c4 \uac01\uac01 \ub85c\uc988\uc758 \uc5c4\ub9c8\uc640 \ub0a8\uc790\uce5c\uad6c \uc5ed\uc73c\ub85c \uce90\uc2a4\ud305\ub418\uc5c8\ub2e4. \ub370\uc774\ube44\uc2a4\ub294 \uc774\ub4e4 \ub4f1\uc7a5\uc778\ubb3c\uc774 \"\ub85c\uc988\ub97c \uc9c4\uc2e4\ub418\uac8c \ub9cc\ub4e4\uace0\", \"\ub85c\uc988\uc5d0\uac8c \uc0b6\uc744 \ubc14\uce58\ub294 \uac83\"\uc744 \ub0b4\ud3ec\ud558\uae38 \uc6d0\ud588\ub2e4. \ud55c\ud3b8 \ub85c\uc988\ub124 \uac00\uc871\uc740 \ub4dc\ub77c\ub9c8\uc5d0\uc11c \u3008Rose\u3009 \uc774\uc804\uc758 \ub3d9\ud589\uc790\ub4e4\uc5d0\uac8c\uc11c\ub294 \ub4dc\ubb3c\uac8c \ub098\ud0c0\ub0ac\ub358 \ub178\ub3d9\uacc4\uce35\uc774\ub2e4.", "answer": "\uc904\ub9ac \uac00\ub4dc\ub108", "sentence": "\ucd1d\uad04 \ud504\ub85c\ub4c0\uc11c\uc778 \uc904\ub9ac \uac00\ub4dc\ub108 \uc5d0", "paragraph_sentence": "2004\ub144 5\uc6d4 24\uc77c\uc5d0\ub294 \ube4c\ub9ac \ud30c\uc774\ud37c\uac00 \ub85c\uc988 \ud0c0\uc77c\ub7ec \uc5ed\uc73c\ub85c \uce90\uc2a4\ud305\ub418\uc5c8\ub2e4\ub294 \uc18c\uc2dd\uc774 \ubc1c\ud45c\ub410\ub2e4. \ucd1d\uad04 \ud504\ub85c\ub4c0\uc11c\uc778 \uc904\ub9ac \uac00\ub4dc\ub108 \uc5d0 \ub530\ub974\uba74, \uc804 \ud31d\uc2a4\ud0c0\uc600\ub358 \ud30c\uc774\ud37c\ub294 \"\ud06c\ub9ac\uc2a4\ud1a0\ud37c \uc5d0\ud074\uc2a4\ud134\uc744 \uc704\ud55c \ub3c5\ud2b9\ud558\uace0 \ud65c\ub825\uc788\ub294 \ub3d9\ubc18\uc790\"\ub85c\uc368 \"\ub4dc\ub77c\ub9c8\uc5d0 \uc644\ubcbd\ud558\uac8c \uc54c\ub9de\ub2e4\"\uace0 \ub9d0\ud588\ub2e4. \ub370\uc774\ube44\uc2a4\ub294 \ub85c\uc988\ub97c \"\ubb54\uac00 \uae30\uc774\ud55c \uac83\uc5d0 \ud718\ub9d0\ub9ac\uace0 \uadf8\uac83\ub4e4\uc758 \ub3d9\ub4f1\ud55c \uac83\uc744 \uc2a4\uc2a4\ub85c \ubc1c\uacac\ud574\ub0b4\ub294 \ud3c9\ubc94\ud55c \uc0ac\ub78c\"\uc73c\ub85c \ubb18\uc0ac\ud588\ub2e4. \ucee4\ubc00 \ucf54\ub450\ub9ac\uc640 \ub178\uc5d8 \ud074\ub77c\ud06c\ub3c4 \uac01\uac01 \ub85c\uc988\uc758 \uc5c4\ub9c8\uc640 \ub0a8\uc790\uce5c\uad6c \uc5ed\uc73c\ub85c \uce90\uc2a4\ud305\ub418\uc5c8\ub2e4. \ub370\uc774\ube44\uc2a4\ub294 \uc774\ub4e4 \ub4f1\uc7a5\uc778\ubb3c\uc774 \"\ub85c\uc988\ub97c \uc9c4\uc2e4\ub418\uac8c \ub9cc\ub4e4\uace0\", \"\ub85c\uc988\uc5d0\uac8c \uc0b6\uc744 \ubc14\uce58\ub294 \uac83\"\uc744 \ub0b4\ud3ec\ud558\uae38 \uc6d0\ud588\ub2e4. \ud55c\ud3b8 \ub85c\uc988\ub124 \uac00\uc871\uc740 \ub4dc\ub77c\ub9c8\uc5d0\uc11c \u3008Rose\u3009 \uc774\uc804\uc758 \ub3d9\ud589\uc790\ub4e4\uc5d0\uac8c\uc11c\ub294 \ub4dc\ubb3c\uac8c \ub098\ud0c0\ub0ac\ub358 \ub178\ub3d9\uacc4\uce35\uc774\ub2e4.", "paragraph_answer": "2004\ub144 5\uc6d4 24\uc77c\uc5d0\ub294 \ube4c\ub9ac \ud30c\uc774\ud37c\uac00 \ub85c\uc988 \ud0c0\uc77c\ub7ec \uc5ed\uc73c\ub85c \uce90\uc2a4\ud305\ub418\uc5c8\ub2e4\ub294 \uc18c\uc2dd\uc774 \ubc1c\ud45c\ub410\ub2e4. \ucd1d\uad04 \ud504\ub85c\ub4c0\uc11c\uc778 \uc904\ub9ac \uac00\ub4dc\ub108 \uc5d0 \ub530\ub974\uba74, \uc804 \ud31d\uc2a4\ud0c0\uc600\ub358 \ud30c\uc774\ud37c\ub294 \"\ud06c\ub9ac\uc2a4\ud1a0\ud37c \uc5d0\ud074\uc2a4\ud134\uc744 \uc704\ud55c \ub3c5\ud2b9\ud558\uace0 \ud65c\ub825\uc788\ub294 \ub3d9\ubc18\uc790\"\ub85c\uc368 \"\ub4dc\ub77c\ub9c8\uc5d0 \uc644\ubcbd\ud558\uac8c \uc54c\ub9de\ub2e4\"\uace0 \ub9d0\ud588\ub2e4. \ub370\uc774\ube44\uc2a4\ub294 \ub85c\uc988\ub97c \"\ubb54\uac00 \uae30\uc774\ud55c \uac83\uc5d0 \ud718\ub9d0\ub9ac\uace0 \uadf8\uac83\ub4e4\uc758 \ub3d9\ub4f1\ud55c \uac83\uc744 \uc2a4\uc2a4\ub85c \ubc1c\uacac\ud574\ub0b4\ub294 \ud3c9\ubc94\ud55c \uc0ac\ub78c\"\uc73c\ub85c \ubb18\uc0ac\ud588\ub2e4. \ucee4\ubc00 \ucf54\ub450\ub9ac\uc640 \ub178\uc5d8 \ud074\ub77c\ud06c\ub3c4 \uac01\uac01 \ub85c\uc988\uc758 \uc5c4\ub9c8\uc640 \ub0a8\uc790\uce5c\uad6c \uc5ed\uc73c\ub85c \uce90\uc2a4\ud305\ub418\uc5c8\ub2e4. \ub370\uc774\ube44\uc2a4\ub294 \uc774\ub4e4 \ub4f1\uc7a5\uc778\ubb3c\uc774 \"\ub85c\uc988\ub97c \uc9c4\uc2e4\ub418\uac8c \ub9cc\ub4e4\uace0\", \"\ub85c\uc988\uc5d0\uac8c \uc0b6\uc744 \ubc14\uce58\ub294 \uac83\"\uc744 \ub0b4\ud3ec\ud558\uae38 \uc6d0\ud588\ub2e4. \ud55c\ud3b8 \ub85c\uc988\ub124 \uac00\uc871\uc740 \ub4dc\ub77c\ub9c8\uc5d0\uc11c \u3008Rose\u3009 \uc774\uc804\uc758 \ub3d9\ud589\uc790\ub4e4\uc5d0\uac8c\uc11c\ub294 \ub4dc\ubb3c\uac8c \ub098\ud0c0\ub0ac\ub358 \ub178\ub3d9\uacc4\uce35\uc774\ub2e4.", "sentence_answer": "\ucd1d\uad04 \ud504\ub85c\ub4c0\uc11c\uc778 \uc904\ub9ac \uac00\ub4dc\ub108 \uc5d0", "paragraph_id": "6483060-5-1", "paragraph_question": "question: \ub2e5\ud130\ud6c4\uc758 \ucd1d\uad04 \ud504\ub85c\ub4c0\uc11c \uc774\ub984\uc740 \ubb34\uc5c7\uc778\uac00?, context: 2004\ub144 5\uc6d4 24\uc77c\uc5d0\ub294 \ube4c\ub9ac \ud30c\uc774\ud37c\uac00 \ub85c\uc988 \ud0c0\uc77c\ub7ec \uc5ed\uc73c\ub85c \uce90\uc2a4\ud305\ub418\uc5c8\ub2e4\ub294 \uc18c\uc2dd\uc774 \ubc1c\ud45c\ub410\ub2e4. \ucd1d\uad04 \ud504\ub85c\ub4c0\uc11c\uc778 \uc904\ub9ac \uac00\ub4dc\ub108\uc5d0 \ub530\ub974\uba74, \uc804 \ud31d\uc2a4\ud0c0\uc600\ub358 \ud30c\uc774\ud37c\ub294 \"\ud06c\ub9ac\uc2a4\ud1a0\ud37c \uc5d0\ud074\uc2a4\ud134\uc744 \uc704\ud55c \ub3c5\ud2b9\ud558\uace0 \ud65c\ub825\uc788\ub294 \ub3d9\ubc18\uc790\"\ub85c\uc368 \"\ub4dc\ub77c\ub9c8\uc5d0 \uc644\ubcbd\ud558\uac8c \uc54c\ub9de\ub2e4\"\uace0 \ub9d0\ud588\ub2e4. \ub370\uc774\ube44\uc2a4\ub294 \ub85c\uc988\ub97c \"\ubb54\uac00 \uae30\uc774\ud55c \uac83\uc5d0 \ud718\ub9d0\ub9ac\uace0 \uadf8\uac83\ub4e4\uc758 \ub3d9\ub4f1\ud55c \uac83\uc744 \uc2a4\uc2a4\ub85c \ubc1c\uacac\ud574\ub0b4\ub294 \ud3c9\ubc94\ud55c \uc0ac\ub78c\"\uc73c\ub85c \ubb18\uc0ac\ud588\ub2e4. \ucee4\ubc00 \ucf54\ub450\ub9ac\uc640 \ub178\uc5d8 \ud074\ub77c\ud06c\ub3c4 \uac01\uac01 \ub85c\uc988\uc758 \uc5c4\ub9c8\uc640 \ub0a8\uc790\uce5c\uad6c \uc5ed\uc73c\ub85c \uce90\uc2a4\ud305\ub418\uc5c8\ub2e4. \ub370\uc774\ube44\uc2a4\ub294 \uc774\ub4e4 \ub4f1\uc7a5\uc778\ubb3c\uc774 \"\ub85c\uc988\ub97c \uc9c4\uc2e4\ub418\uac8c \ub9cc\ub4e4\uace0\", \"\ub85c\uc988\uc5d0\uac8c \uc0b6\uc744 \ubc14\uce58\ub294 \uac83\"\uc744 \ub0b4\ud3ec\ud558\uae38 \uc6d0\ud588\ub2e4. \ud55c\ud3b8 \ub85c\uc988\ub124 \uac00\uc871\uc740 \ub4dc\ub77c\ub9c8\uc5d0\uc11c \u3008Rose\u3009 \uc774\uc804\uc758 \ub3d9\ud589\uc790\ub4e4\uc5d0\uac8c\uc11c\ub294 \ub4dc\ubb3c\uac8c \ub098\ud0c0\ub0ac\ub358 \ub178\ub3d9\uacc4\uce35\uc774\ub2e4."} {"question": "\uc815\uad6d\uc774 \uae09\uaca9\ud558\uac8c \ubc14\ub00c\ub294 \ud658\uad6d\uc774 \uc77c\uc5b4\ub098\uba74\uc11c \uacac\uc81c\uc640 \uade0\ud615\uc744 \ud1b5\ud55c \uc0c1\ud638 \uacf5\uc874\uc774\ub77c\ub294 \ubb34\uc5c7\uc774 \ubb34\ub108\uc84c\ub294\uac00?", "paragraph": "1680\ub144\uc758 \uacbd\uc2e0\ud658\uad6d\uc73c\ub85c \uc11c\uc778\uc774 \uad8c\ub825\uc744 \uc7a1\uc740 \ub4a4 \uade0\ud615\uc774 \ubb34\ub108\uc838, \uc11c\uc778\uc740 \ub0a8\uc778\uc744 \ucca0\uc800\ud788 \ud0c4\uc555\ud558\uc600\ub2e4. \uc774\uc5b4 \uc11c\uc778\uc5d0\uc11c \ubd84\uc5f4\ub41c \ub178\ub860\uacfc \uc18c\ub860\uc774 \ub300\ub9bd\ud558\uc600\uace0, \uadf8\uae30\uc57c \uc815\uad6d\uc774 \uae09\uaca9\ud558\uac8c \ubc14\ub00c\ub294 \ud658\uad6d\uc774 \uc77c\uc5b4\ub098\uba74\uc11c \uacac\uc81c\uc640 \uade0\ud615\uc744 \ud1b5\ud55c \uc0c1\ud638 \uacf5\uc874\uc774\ub77c\ub294 '\ubd95\ub2f9 \uc815\uce58\uc758 \uc6d0\uce59'\uc774 \ubb34\ub108\uc9c0\uace0 \ud2b9\uc815\ud55c \ubd95\ub2f9\uc774 \ub2e4\ub978 \uc815\uad8c\uc758 \ubd95\ub2f9\uc744 \ubab0\uc544\ub0b4\uace0 \uc815\uad8c\uc744 \ub3c5\uc810\ud558\ub294 \uc77c\ub2f9 \uc804\uc81c\ud654\uc758 \uacbd\ud5a5\uc744 \ub760\uba74\uc11c \uad6d\uc655\uc774 \uc0ac\uc0ac\ub85c\uc6b4 \uac10\uc815\uc73c\ub85c \uc815\uce58\uc5d0 \uac1c\uc785\uc774 \ub418\uc5b4 \uad6d\uc815\uc774 \ub9c8\ube44\ub420 \uc815\ub3c4\ub85c \ud63c\ub780\uc2a4\ub7ec\uc6e0\uace0, \uc774 \ub54c\ubb38\uc5d0 \uc5ec\ub7ec \ubd95\ub2f9 \uac04\uc5d0 \uac08\ub4f1\uacfc \uc624\ud574\ub85c \ubc8c\uc5b4\uc9c4 \ub2f9\uc7c1\uc73c\ub85c \uc778\ud574 \uc601\uc870\uc758 \uc544\ub4e4\uc778 \uc0ac\ub3c4\uc138\uc790 \uc774\uc120\uc774 \ub4a4\uc8fc \uc548\uc5d0 \uac16\ud600 \uc8fd\ub294 \uc784\uc624\ud654\ubcc0\uc774 \uc77c\uc5b4\ub0ac\ub2e4. \uadf8\ub798\uc11c \uc655\uad8c\uc774 \uc2e0\ud558\ub4e4\uc5d0 \uc758\ud574 \uc57d\ud574\uc9c0\uc790 \ub2f9\uc7c1\uc744 \ub9c9\uae30 \uc704\ud574 \uc601\uc870\uc640 \uc815\uc870\ub294 \ub2f9\ud30c\uc758 \uade0\ud615\uc744 \uace0\ub824\ud55c \uc778\uc7ac\uae30\uc6a9(\ud0d5\ud3c9\ucc45)\uc744 \uc2e4\ucc9c\ud588\ub2e4. \uadf8\ub7ec\ub098 \uc815\uc870\uac00 1800\ub144\uc5d0 \uac11\uc791\uc2a4\ub7fd\uac8c \uc885\uae30\uc758 \uc5fc\uc99d\uc73c\ub85c \uc138\uc0c1\uc744 \ub728\uc790 \ub178\ub860\uc138\ub825\uc774 \ub3c5\uc810\ud558\uc5ec \uc815\uad6d\uc744 \uc774\ub04c\uc5c8\uc9c0\ub9cc, 19\uc138\uae30\uc758 \uc21c\uc870\u00b7\ud5cc\uc885\u00b7\ucca0\uc885 \ub4f1 3\ub300\uc5d0 \uac78\uce5c \uc548\ub3d9 \uae40\uc528\uc640 \ud48d\uc591 \uc870\uc528 \ub4f1 \uc678\ucc99 \uc138\ub825\uc758 \uc138\ub3c4 \uc815\uce58\uac00 60\uc5ec \ub144 \ub3d9\uc548 \uacc4\uc18d\ub418\uc5c8\ub2e4. \uc774\ub4e4 \uc678\ucc99\uac00\ubb38\uc740 \uc655\uc704 \uc784\uba85\uc5d0\ub3c4 \uc190\uc744 \ub300\ub294 \ub4f1 \ubd80\uc815\ubd80\ud328 \uc18d\uc5d0\uc11c\uc758 \uc808\ub300\uad8c\ub825\uc744 \ub204\ub838\ub2e4. \uadf8\ub9ac\uace0 \ud48d\uc591 \uc870\uc528\uc758 \uac00\ubb38\uc758 \ub300\ud45c\uc600\ub358 \uc870\ub300\ube44)\uac00 \ud765\uc120\ub300\uc6d0\uad70\uacfc \uacb0\ud0c1\ud558\uc5ec \ud765\uc120\ub300\uc6d0\uad70\uc758 \ub458\uc9f8\uc544\ub4e4 \uba85\ubcf5(\uace0\uc885)\uc744 \uc870\uc120 \uad6d\uc655 \uc790\ub9ac\uc5d0 \uc62c\ub9b0\ub2e4.", "answer": "\ubd95\ub2f9 \uc815\uce58\uc758 \uc6d0\uce59", "sentence": "\uc774\uc5b4 \uc11c\uc778\uc5d0\uc11c \ubd84\uc5f4\ub41c \ub178\ub860\uacfc \uc18c\ub860\uc774 \ub300\ub9bd\ud558\uc600\uace0, \uadf8\uae30\uc57c \uc815\uad6d\uc774 \uae09\uaca9\ud558\uac8c \ubc14\ub00c\ub294 \ud658\uad6d\uc774 \uc77c\uc5b4\ub098\uba74\uc11c \uacac\uc81c\uc640 \uade0\ud615\uc744 \ud1b5\ud55c \uc0c1\ud638 \uacf5\uc874\uc774\ub77c\ub294 ' \ubd95\ub2f9 \uc815\uce58\uc758 \uc6d0\uce59 '\uc774 \ubb34\ub108\uc9c0\uace0 \ud2b9\uc815\ud55c \ubd95\ub2f9\uc774 \ub2e4\ub978 \uc815\uad8c\uc758 \ubd95\ub2f9\uc744 \ubab0\uc544\ub0b4\uace0 \uc815\uad8c\uc744 \ub3c5\uc810\ud558\ub294 \uc77c\ub2f9 \uc804\uc81c\ud654\uc758 \uacbd\ud5a5\uc744 \ub760\uba74\uc11c \uad6d\uc655\uc774 \uc0ac\uc0ac\ub85c\uc6b4 \uac10\uc815\uc73c\ub85c \uc815\uce58\uc5d0 \uac1c\uc785\uc774 \ub418\uc5b4 \uad6d\uc815\uc774 \ub9c8\ube44\ub420 \uc815\ub3c4\ub85c \ud63c\ub780\uc2a4\ub7ec\uc6e0\uace0, \uc774 \ub54c\ubb38\uc5d0 \uc5ec\ub7ec \ubd95\ub2f9 \uac04\uc5d0 \uac08\ub4f1\uacfc \uc624\ud574\ub85c \ubc8c\uc5b4\uc9c4 \ub2f9\uc7c1\uc73c\ub85c \uc778\ud574 \uc601\uc870\uc758 \uc544\ub4e4\uc778 \uc0ac\ub3c4\uc138\uc790 \uc774\uc120\uc774 \ub4a4\uc8fc \uc548\uc5d0 \uac16\ud600 \uc8fd\ub294 \uc784\uc624\ud654\ubcc0\uc774 \uc77c\uc5b4\ub0ac\ub2e4.", "paragraph_sentence": "1680\ub144\uc758 \uacbd\uc2e0\ud658\uad6d\uc73c\ub85c \uc11c\uc778\uc774 \uad8c\ub825\uc744 \uc7a1\uc740 \ub4a4 \uade0\ud615\uc774 \ubb34\ub108\uc838, \uc11c\uc778\uc740 \ub0a8\uc778\uc744 \ucca0\uc800\ud788 \ud0c4\uc555\ud558\uc600\ub2e4. \uc774\uc5b4 \uc11c\uc778\uc5d0\uc11c \ubd84\uc5f4\ub41c \ub178\ub860\uacfc \uc18c\ub860\uc774 \ub300\ub9bd\ud558\uc600\uace0, \uadf8\uae30\uc57c \uc815\uad6d\uc774 \uae09\uaca9\ud558\uac8c \ubc14\ub00c\ub294 \ud658\uad6d\uc774 \uc77c\uc5b4\ub098\uba74\uc11c \uacac\uc81c\uc640 \uade0\ud615\uc744 \ud1b5\ud55c \uc0c1\ud638 \uacf5\uc874\uc774\ub77c\ub294 ' \ubd95\ub2f9 \uc815\uce58\uc758 \uc6d0\uce59 '\uc774 \ubb34\ub108\uc9c0\uace0 \ud2b9\uc815\ud55c \ubd95\ub2f9\uc774 \ub2e4\ub978 \uc815\uad8c\uc758 \ubd95\ub2f9\uc744 \ubab0\uc544\ub0b4\uace0 \uc815\uad8c\uc744 \ub3c5\uc810\ud558\ub294 \uc77c\ub2f9 \uc804\uc81c\ud654\uc758 \uacbd\ud5a5\uc744 \ub760\uba74\uc11c \uad6d\uc655\uc774 \uc0ac\uc0ac\ub85c\uc6b4 \uac10\uc815\uc73c\ub85c \uc815\uce58\uc5d0 \uac1c\uc785\uc774 \ub418\uc5b4 \uad6d\uc815\uc774 \ub9c8\ube44\ub420 \uc815\ub3c4\ub85c \ud63c\ub780\uc2a4\ub7ec\uc6e0\uace0, \uc774 \ub54c\ubb38\uc5d0 \uc5ec\ub7ec \ubd95\ub2f9 \uac04\uc5d0 \uac08\ub4f1\uacfc \uc624\ud574\ub85c \ubc8c\uc5b4\uc9c4 \ub2f9\uc7c1\uc73c\ub85c \uc778\ud574 \uc601\uc870\uc758 \uc544\ub4e4\uc778 \uc0ac\ub3c4\uc138\uc790 \uc774\uc120\uc774 \ub4a4\uc8fc \uc548\uc5d0 \uac16\ud600 \uc8fd\ub294 \uc784\uc624\ud654\ubcc0\uc774 \uc77c\uc5b4\ub0ac\ub2e4. \uadf8\ub798\uc11c \uc655\uad8c\uc774 \uc2e0\ud558\ub4e4\uc5d0 \uc758\ud574 \uc57d\ud574\uc9c0\uc790 \ub2f9\uc7c1\uc744 \ub9c9\uae30 \uc704\ud574 \uc601\uc870\uc640 \uc815\uc870\ub294 \ub2f9\ud30c\uc758 \uade0\ud615\uc744 \uace0\ub824\ud55c \uc778\uc7ac\uae30\uc6a9(\ud0d5\ud3c9\ucc45)\uc744 \uc2e4\ucc9c\ud588\ub2e4. \uadf8\ub7ec\ub098 \uc815\uc870\uac00 1800\ub144\uc5d0 \uac11\uc791\uc2a4\ub7fd\uac8c \uc885\uae30\uc758 \uc5fc\uc99d\uc73c\ub85c \uc138\uc0c1\uc744 \ub728\uc790 \ub178\ub860\uc138\ub825\uc774 \ub3c5\uc810\ud558\uc5ec \uc815\uad6d\uc744 \uc774\ub04c\uc5c8\uc9c0\ub9cc, 19\uc138\uae30\uc758 \uc21c\uc870\u00b7\ud5cc\uc885\u00b7\ucca0\uc885 \ub4f1 3\ub300\uc5d0 \uac78\uce5c \uc548\ub3d9 \uae40\uc528\uc640 \ud48d\uc591 \uc870\uc528 \ub4f1 \uc678\ucc99 \uc138\ub825\uc758 \uc138\ub3c4 \uc815\uce58\uac00 60\uc5ec \ub144 \ub3d9\uc548 \uacc4\uc18d\ub418\uc5c8\ub2e4. \uc774\ub4e4 \uc678\ucc99\uac00\ubb38\uc740 \uc655\uc704 \uc784\uba85\uc5d0\ub3c4 \uc190\uc744 \ub300\ub294 \ub4f1 \ubd80\uc815\ubd80\ud328 \uc18d\uc5d0\uc11c\uc758 \uc808\ub300\uad8c\ub825\uc744 \ub204\ub838\ub2e4. \uadf8\ub9ac\uace0 \ud48d\uc591 \uc870\uc528\uc758 \uac00\ubb38\uc758 \ub300\ud45c\uc600\ub358 \uc870\ub300\ube44)\uac00 \ud765\uc120\ub300\uc6d0\uad70\uacfc \uacb0\ud0c1\ud558\uc5ec \ud765\uc120\ub300\uc6d0\uad70\uc758 \ub458\uc9f8\uc544\ub4e4 \uba85\ubcf5(\uace0\uc885)\uc744 \uc870\uc120 \uad6d\uc655 \uc790\ub9ac\uc5d0 \uc62c\ub9b0\ub2e4.", "paragraph_answer": "1680\ub144\uc758 \uacbd\uc2e0\ud658\uad6d\uc73c\ub85c \uc11c\uc778\uc774 \uad8c\ub825\uc744 \uc7a1\uc740 \ub4a4 \uade0\ud615\uc774 \ubb34\ub108\uc838, \uc11c\uc778\uc740 \ub0a8\uc778\uc744 \ucca0\uc800\ud788 \ud0c4\uc555\ud558\uc600\ub2e4. \uc774\uc5b4 \uc11c\uc778\uc5d0\uc11c \ubd84\uc5f4\ub41c \ub178\ub860\uacfc \uc18c\ub860\uc774 \ub300\ub9bd\ud558\uc600\uace0, \uadf8\uae30\uc57c \uc815\uad6d\uc774 \uae09\uaca9\ud558\uac8c \ubc14\ub00c\ub294 \ud658\uad6d\uc774 \uc77c\uc5b4\ub098\uba74\uc11c \uacac\uc81c\uc640 \uade0\ud615\uc744 \ud1b5\ud55c \uc0c1\ud638 \uacf5\uc874\uc774\ub77c\ub294 ' \ubd95\ub2f9 \uc815\uce58\uc758 \uc6d0\uce59 '\uc774 \ubb34\ub108\uc9c0\uace0 \ud2b9\uc815\ud55c \ubd95\ub2f9\uc774 \ub2e4\ub978 \uc815\uad8c\uc758 \ubd95\ub2f9\uc744 \ubab0\uc544\ub0b4\uace0 \uc815\uad8c\uc744 \ub3c5\uc810\ud558\ub294 \uc77c\ub2f9 \uc804\uc81c\ud654\uc758 \uacbd\ud5a5\uc744 \ub760\uba74\uc11c \uad6d\uc655\uc774 \uc0ac\uc0ac\ub85c\uc6b4 \uac10\uc815\uc73c\ub85c \uc815\uce58\uc5d0 \uac1c\uc785\uc774 \ub418\uc5b4 \uad6d\uc815\uc774 \ub9c8\ube44\ub420 \uc815\ub3c4\ub85c \ud63c\ub780\uc2a4\ub7ec\uc6e0\uace0, \uc774 \ub54c\ubb38\uc5d0 \uc5ec\ub7ec \ubd95\ub2f9 \uac04\uc5d0 \uac08\ub4f1\uacfc \uc624\ud574\ub85c \ubc8c\uc5b4\uc9c4 \ub2f9\uc7c1\uc73c\ub85c \uc778\ud574 \uc601\uc870\uc758 \uc544\ub4e4\uc778 \uc0ac\ub3c4\uc138\uc790 \uc774\uc120\uc774 \ub4a4\uc8fc \uc548\uc5d0 \uac16\ud600 \uc8fd\ub294 \uc784\uc624\ud654\ubcc0\uc774 \uc77c\uc5b4\ub0ac\ub2e4. \uadf8\ub798\uc11c \uc655\uad8c\uc774 \uc2e0\ud558\ub4e4\uc5d0 \uc758\ud574 \uc57d\ud574\uc9c0\uc790 \ub2f9\uc7c1\uc744 \ub9c9\uae30 \uc704\ud574 \uc601\uc870\uc640 \uc815\uc870\ub294 \ub2f9\ud30c\uc758 \uade0\ud615\uc744 \uace0\ub824\ud55c \uc778\uc7ac\uae30\uc6a9(\ud0d5\ud3c9\ucc45)\uc744 \uc2e4\ucc9c\ud588\ub2e4. \uadf8\ub7ec\ub098 \uc815\uc870\uac00 1800\ub144\uc5d0 \uac11\uc791\uc2a4\ub7fd\uac8c \uc885\uae30\uc758 \uc5fc\uc99d\uc73c\ub85c \uc138\uc0c1\uc744 \ub728\uc790 \ub178\ub860\uc138\ub825\uc774 \ub3c5\uc810\ud558\uc5ec \uc815\uad6d\uc744 \uc774\ub04c\uc5c8\uc9c0\ub9cc, 19\uc138\uae30\uc758 \uc21c\uc870\u00b7\ud5cc\uc885\u00b7\ucca0\uc885 \ub4f1 3\ub300\uc5d0 \uac78\uce5c \uc548\ub3d9 \uae40\uc528\uc640 \ud48d\uc591 \uc870\uc528 \ub4f1 \uc678\ucc99 \uc138\ub825\uc758 \uc138\ub3c4 \uc815\uce58\uac00 60\uc5ec \ub144 \ub3d9\uc548 \uacc4\uc18d\ub418\uc5c8\ub2e4. \uc774\ub4e4 \uc678\ucc99\uac00\ubb38\uc740 \uc655\uc704 \uc784\uba85\uc5d0\ub3c4 \uc190\uc744 \ub300\ub294 \ub4f1 \ubd80\uc815\ubd80\ud328 \uc18d\uc5d0\uc11c\uc758 \uc808\ub300\uad8c\ub825\uc744 \ub204\ub838\ub2e4. \uadf8\ub9ac\uace0 \ud48d\uc591 \uc870\uc528\uc758 \uac00\ubb38\uc758 \ub300\ud45c\uc600\ub358 \uc870\ub300\ube44)\uac00 \ud765\uc120\ub300\uc6d0\uad70\uacfc \uacb0\ud0c1\ud558\uc5ec \ud765\uc120\ub300\uc6d0\uad70\uc758 \ub458\uc9f8\uc544\ub4e4 \uba85\ubcf5(\uace0\uc885)\uc744 \uc870\uc120 \uad6d\uc655 \uc790\ub9ac\uc5d0 \uc62c\ub9b0\ub2e4.", "sentence_answer": "\uc774\uc5b4 \uc11c\uc778\uc5d0\uc11c \ubd84\uc5f4\ub41c \ub178\ub860\uacfc \uc18c\ub860\uc774 \ub300\ub9bd\ud558\uc600\uace0, \uadf8\uae30\uc57c \uc815\uad6d\uc774 \uae09\uaca9\ud558\uac8c \ubc14\ub00c\ub294 \ud658\uad6d\uc774 \uc77c\uc5b4\ub098\uba74\uc11c \uacac\uc81c\uc640 \uade0\ud615\uc744 \ud1b5\ud55c \uc0c1\ud638 \uacf5\uc874\uc774\ub77c\ub294 ' \ubd95\ub2f9 \uc815\uce58\uc758 \uc6d0\uce59 '\uc774 \ubb34\ub108\uc9c0\uace0 \ud2b9\uc815\ud55c \ubd95\ub2f9\uc774 \ub2e4\ub978 \uc815\uad8c\uc758 \ubd95\ub2f9\uc744 \ubab0\uc544\ub0b4\uace0 \uc815\uad8c\uc744 \ub3c5\uc810\ud558\ub294 \uc77c\ub2f9 \uc804\uc81c\ud654\uc758 \uacbd\ud5a5\uc744 \ub760\uba74\uc11c \uad6d\uc655\uc774 \uc0ac\uc0ac\ub85c\uc6b4 \uac10\uc815\uc73c\ub85c \uc815\uce58\uc5d0 \uac1c\uc785\uc774 \ub418\uc5b4 \uad6d\uc815\uc774 \ub9c8\ube44\ub420 \uc815\ub3c4\ub85c \ud63c\ub780\uc2a4\ub7ec\uc6e0\uace0, \uc774 \ub54c\ubb38\uc5d0 \uc5ec\ub7ec \ubd95\ub2f9 \uac04\uc5d0 \uac08\ub4f1\uacfc \uc624\ud574\ub85c \ubc8c\uc5b4\uc9c4 \ub2f9\uc7c1\uc73c\ub85c \uc778\ud574 \uc601\uc870\uc758 \uc544\ub4e4\uc778 \uc0ac\ub3c4\uc138\uc790 \uc774\uc120\uc774 \ub4a4\uc8fc \uc548\uc5d0 \uac16\ud600 \uc8fd\ub294 \uc784\uc624\ud654\ubcc0\uc774 \uc77c\uc5b4\ub0ac\ub2e4.", "paragraph_id": "6187890-2-1", "paragraph_question": "question: \uc815\uad6d\uc774 \uae09\uaca9\ud558\uac8c \ubc14\ub00c\ub294 \ud658\uad6d\uc774 \uc77c\uc5b4\ub098\uba74\uc11c \uacac\uc81c\uc640 \uade0\ud615\uc744 \ud1b5\ud55c \uc0c1\ud638 \uacf5\uc874\uc774\ub77c\ub294 \ubb34\uc5c7\uc774 \ubb34\ub108\uc84c\ub294\uac00?, context: 1680\ub144\uc758 \uacbd\uc2e0\ud658\uad6d\uc73c\ub85c \uc11c\uc778\uc774 \uad8c\ub825\uc744 \uc7a1\uc740 \ub4a4 \uade0\ud615\uc774 \ubb34\ub108\uc838, \uc11c\uc778\uc740 \ub0a8\uc778\uc744 \ucca0\uc800\ud788 \ud0c4\uc555\ud558\uc600\ub2e4. \uc774\uc5b4 \uc11c\uc778\uc5d0\uc11c \ubd84\uc5f4\ub41c \ub178\ub860\uacfc \uc18c\ub860\uc774 \ub300\ub9bd\ud558\uc600\uace0, \uadf8\uae30\uc57c \uc815\uad6d\uc774 \uae09\uaca9\ud558\uac8c \ubc14\ub00c\ub294 \ud658\uad6d\uc774 \uc77c\uc5b4\ub098\uba74\uc11c \uacac\uc81c\uc640 \uade0\ud615\uc744 \ud1b5\ud55c \uc0c1\ud638 \uacf5\uc874\uc774\ub77c\ub294 '\ubd95\ub2f9 \uc815\uce58\uc758 \uc6d0\uce59'\uc774 \ubb34\ub108\uc9c0\uace0 \ud2b9\uc815\ud55c \ubd95\ub2f9\uc774 \ub2e4\ub978 \uc815\uad8c\uc758 \ubd95\ub2f9\uc744 \ubab0\uc544\ub0b4\uace0 \uc815\uad8c\uc744 \ub3c5\uc810\ud558\ub294 \uc77c\ub2f9 \uc804\uc81c\ud654\uc758 \uacbd\ud5a5\uc744 \ub760\uba74\uc11c \uad6d\uc655\uc774 \uc0ac\uc0ac\ub85c\uc6b4 \uac10\uc815\uc73c\ub85c \uc815\uce58\uc5d0 \uac1c\uc785\uc774 \ub418\uc5b4 \uad6d\uc815\uc774 \ub9c8\ube44\ub420 \uc815\ub3c4\ub85c \ud63c\ub780\uc2a4\ub7ec\uc6e0\uace0, \uc774 \ub54c\ubb38\uc5d0 \uc5ec\ub7ec \ubd95\ub2f9 \uac04\uc5d0 \uac08\ub4f1\uacfc \uc624\ud574\ub85c \ubc8c\uc5b4\uc9c4 \ub2f9\uc7c1\uc73c\ub85c \uc778\ud574 \uc601\uc870\uc758 \uc544\ub4e4\uc778 \uc0ac\ub3c4\uc138\uc790 \uc774\uc120\uc774 \ub4a4\uc8fc \uc548\uc5d0 \uac16\ud600 \uc8fd\ub294 \uc784\uc624\ud654\ubcc0\uc774 \uc77c\uc5b4\ub0ac\ub2e4. \uadf8\ub798\uc11c \uc655\uad8c\uc774 \uc2e0\ud558\ub4e4\uc5d0 \uc758\ud574 \uc57d\ud574\uc9c0\uc790 \ub2f9\uc7c1\uc744 \ub9c9\uae30 \uc704\ud574 \uc601\uc870\uc640 \uc815\uc870\ub294 \ub2f9\ud30c\uc758 \uade0\ud615\uc744 \uace0\ub824\ud55c \uc778\uc7ac\uae30\uc6a9(\ud0d5\ud3c9\ucc45)\uc744 \uc2e4\ucc9c\ud588\ub2e4. \uadf8\ub7ec\ub098 \uc815\uc870\uac00 1800\ub144\uc5d0 \uac11\uc791\uc2a4\ub7fd\uac8c \uc885\uae30\uc758 \uc5fc\uc99d\uc73c\ub85c \uc138\uc0c1\uc744 \ub728\uc790 \ub178\ub860\uc138\ub825\uc774 \ub3c5\uc810\ud558\uc5ec \uc815\uad6d\uc744 \uc774\ub04c\uc5c8\uc9c0\ub9cc, 19\uc138\uae30\uc758 \uc21c\uc870\u00b7\ud5cc\uc885\u00b7\ucca0\uc885 \ub4f1 3\ub300\uc5d0 \uac78\uce5c \uc548\ub3d9 \uae40\uc528\uc640 \ud48d\uc591 \uc870\uc528 \ub4f1 \uc678\ucc99 \uc138\ub825\uc758 \uc138\ub3c4 \uc815\uce58\uac00 60\uc5ec \ub144 \ub3d9\uc548 \uacc4\uc18d\ub418\uc5c8\ub2e4. \uc774\ub4e4 \uc678\ucc99\uac00\ubb38\uc740 \uc655\uc704 \uc784\uba85\uc5d0\ub3c4 \uc190\uc744 \ub300\ub294 \ub4f1 \ubd80\uc815\ubd80\ud328 \uc18d\uc5d0\uc11c\uc758 \uc808\ub300\uad8c\ub825\uc744 \ub204\ub838\ub2e4. \uadf8\ub9ac\uace0 \ud48d\uc591 \uc870\uc528\uc758 \uac00\ubb38\uc758 \ub300\ud45c\uc600\ub358 \uc870\ub300\ube44)\uac00 \ud765\uc120\ub300\uc6d0\uad70\uacfc \uacb0\ud0c1\ud558\uc5ec \ud765\uc120\ub300\uc6d0\uad70\uc758 \ub458\uc9f8\uc544\ub4e4 \uba85\ubcf5(\uace0\uc885)\uc744 \uc870\uc120 \uad6d\uc655 \uc790\ub9ac\uc5d0 \uc62c\ub9b0\ub2e4."} {"question": "\ubc15\ud558\uc120\uc758 \uc601\ud654 \ub370\ubdd4\uc791\uc740?", "paragraph": "\uc11c\uc6b8\ud2b9\ubcc4\uc2dc \uc911\ub791\uad6c \ub9dd\uc6b0\ub3d9\uc5d0\uc11c \ud0dc\uc5b4\ub09c \ubc15\ud558\uc120\uc740 \uc11c\uc6b8\uc911\ud654\ucd08\ub4f1\ud559\uad50, \uc601\ub780\uc5ec\uc790\uc911\ud559\uad50, \uc1a1\uace1\uc5ec\uc790\uace0\ub4f1\ud559\uad50\ub97c \uc878\uc5c5\ud588\ub2e4. \ub610\ud55c \ub3d9\uad6d\ub300\ud559\uad50 \uc5f0\uadf9\uc601\ud654\ud559\uacfc\ub97c \uc878\uc5c5\ud588\ub2e4. \uace0\ub4f1\ud559\uc0dd \uc2dc\uc808 KBS\u300a\ub3c4\uc804! \uace8\ub4e0\ubca8\u300b\uc5d0 \ucd9c\uc5f0\ud55c \uc801\uc774 \uc788\ub2e4. \uc601\ud654\u300a\ud0a4\ub2e4\ub9ac \uc544\uc800\uc528\u300b\uc2dc\uc0ac\ud68c\uc758 \ubb34\ub300\uc778\uc0ac\ub97c \uad6c\uacbd\uac14\ub2e4\uac00 \uc2a4\uce74\uc6b0\ud2b8\ub418\uc5b4 \uc5f0\uc608\uacc4\uc5d0 \uc785\ubb38\ud558\uc600\ub2e4. \ubc15\ud558\uc120\uc758 \ud560\uc544\ubc84\uc9c0\ub294 \ub300\ub839 \ucd9c\uc2e0\uc73c\ub85c \ubc15\uc815\ud76c \uc804 \ub300\ud1b5\ub839\uc758 \ud55c \uae30\uc218 \uc544\ub798 \ud6c4\ubc30\ub85c 2012\ub144 \uc5ec\uc131\uc9c0 \u300a'\uc5ec\uc131\uc911\uc559'\u300b5\uc6d4\ud638\ub97c \ud1b5\ud574 \uc790\uc138\ud788 \uc18c\uac1c \ub410\ub2e4\uace0 \ud55c\ub2e4. \ubc15\ud558\uc120\uc740 2005\ub144 SBS \ub4dc\ub77c\ub9c8 \u300a\uc0ac\ub791\uc740 \uae30\uc801\uc774 \ud544\uc694\ud574\u300b\ub85c \ubcf8\uaca9\uc801\uc73c\ub85c \ub4dc\ub77c\ub9c8\uc5d0 \ub370\ubdd4\ud558\uc600\uc73c\uba70, 2006\ub144\u300a\uc544\ud30c\ud2b8\u300b\ub85c \uc601\ud654\uc5d0 \ub370\ubdd4\ud558\uc600\ub2e4. 2007\ub144 KBS \ub4dc\ub77c\ub9c8 \u300a\uacbd\uc131 \uc2a4\uce94\ub4e4\u300b\ub85c \uc0ac\uadf9\uc5d0 \uccab \ucd9c\uc5f0\ud558\uc600\ub2e4. \uadf8\ub9ac\uace0 2017\ub144 1\uc6d4 22\uc77c\uc5d0 \ubc30\uc6b0 \ub958\uc218\uc601\uacfc \uacb0\ud63c\ud588\ub2e4. 2017\ub144 8\uc6d4 9\uc77c\uc5d0 \uac1c\ubd09\ud55c \uc601\ud654\u300a\uccad\ub144\uacbd\ucc30\u300b\uc5d0\uc11c \uce74\ub9ac\uc2a4\ub9c8 \uc788\ub294 \uacbd\ucc30\ub300\ud559 \uc120\ubc30 \uc8fc\ud76c \uc5ed\uc73c\ub85c \ucd9c\uc5f0 \ud558\uc600\uc73c\uba70, 8\uc6d4 30\uc77c\uc5d0\ub294 \uc601\ud654\u300a\ub85c\ub9c8\uc758 \ud734\uc77c\u300b\uc5d0 \ucd9c\uc5f0\ud588\ub2e4.", "answer": "\uc544\ud30c\ud2b8", "sentence": "2006\ub144\u300a \uc544\ud30c\ud2b8 \u300b\ub85c \uc601\ud654\uc5d0 \ub370\ubdd4\ud558\uc600\ub2e4.", "paragraph_sentence": "\uc11c\uc6b8\ud2b9\ubcc4\uc2dc \uc911\ub791\uad6c \ub9dd\uc6b0\ub3d9\uc5d0\uc11c \ud0dc\uc5b4\ub09c \ubc15\ud558\uc120\uc740 \uc11c\uc6b8\uc911\ud654\ucd08\ub4f1\ud559\uad50, \uc601\ub780\uc5ec\uc790\uc911\ud559\uad50, \uc1a1\uace1\uc5ec\uc790\uace0\ub4f1\ud559\uad50\ub97c \uc878\uc5c5\ud588\ub2e4. \ub610\ud55c \ub3d9\uad6d\ub300\ud559\uad50 \uc5f0\uadf9\uc601\ud654\ud559\uacfc\ub97c \uc878\uc5c5\ud588\ub2e4. \uace0\ub4f1\ud559\uc0dd \uc2dc\uc808 KBS\u300a\ub3c4\uc804! \uace8\ub4e0\ubca8\u300b\uc5d0 \ucd9c\uc5f0\ud55c \uc801\uc774 \uc788\ub2e4. \uc601\ud654\u300a\ud0a4\ub2e4\ub9ac \uc544\uc800\uc528\u300b\uc2dc\uc0ac\ud68c\uc758 \ubb34\ub300\uc778\uc0ac\ub97c \uad6c\uacbd\uac14\ub2e4\uac00 \uc2a4\uce74\uc6b0\ud2b8\ub418\uc5b4 \uc5f0\uc608\uacc4\uc5d0 \uc785\ubb38\ud558\uc600\ub2e4. \ubc15\ud558\uc120\uc758 \ud560\uc544\ubc84\uc9c0\ub294 \ub300\ub839 \ucd9c\uc2e0\uc73c\ub85c \ubc15\uc815\ud76c \uc804 \ub300\ud1b5\ub839\uc758 \ud55c \uae30\uc218 \uc544\ub798 \ud6c4\ubc30\ub85c 2012\ub144 \uc5ec\uc131\uc9c0 \u300a'\uc5ec\uc131\uc911\uc559'\u300b5\uc6d4\ud638\ub97c \ud1b5\ud574 \uc790\uc138\ud788 \uc18c\uac1c \ub410\ub2e4\uace0 \ud55c\ub2e4. \ubc15\ud558\uc120\uc740 2005\ub144 SBS \ub4dc\ub77c\ub9c8 \u300a\uc0ac\ub791\uc740 \uae30\uc801\uc774 \ud544\uc694\ud574\u300b\ub85c \ubcf8\uaca9\uc801\uc73c\ub85c \ub4dc\ub77c\ub9c8\uc5d0 \ub370\ubdd4\ud558\uc600\uc73c\uba70, 2006\ub144\u300a \uc544\ud30c\ud2b8 \u300b\ub85c \uc601\ud654\uc5d0 \ub370\ubdd4\ud558\uc600\ub2e4. 2007\ub144 KBS \ub4dc\ub77c\ub9c8 \u300a\uacbd\uc131 \uc2a4\uce94\ub4e4\u300b\ub85c \uc0ac\uadf9\uc5d0 \uccab \ucd9c\uc5f0\ud558\uc600\ub2e4. \uadf8\ub9ac\uace0 2017\ub144 1\uc6d4 22\uc77c\uc5d0 \ubc30\uc6b0 \ub958\uc218\uc601\uacfc \uacb0\ud63c\ud588\ub2e4. 2017\ub144 8\uc6d4 9\uc77c\uc5d0 \uac1c\ubd09\ud55c \uc601\ud654\u300a\uccad\ub144\uacbd\ucc30\u300b\uc5d0\uc11c \uce74\ub9ac\uc2a4\ub9c8 \uc788\ub294 \uacbd\ucc30\ub300\ud559 \uc120\ubc30 \uc8fc\ud76c \uc5ed\uc73c\ub85c \ucd9c\uc5f0 \ud558\uc600\uc73c\uba70, 8\uc6d4 30\uc77c\uc5d0\ub294 \uc601\ud654\u300a\ub85c\ub9c8\uc758 \ud734\uc77c\u300b\uc5d0 \ucd9c\uc5f0\ud588\ub2e4.", "paragraph_answer": "\uc11c\uc6b8\ud2b9\ubcc4\uc2dc \uc911\ub791\uad6c \ub9dd\uc6b0\ub3d9\uc5d0\uc11c \ud0dc\uc5b4\ub09c \ubc15\ud558\uc120\uc740 \uc11c\uc6b8\uc911\ud654\ucd08\ub4f1\ud559\uad50, \uc601\ub780\uc5ec\uc790\uc911\ud559\uad50, \uc1a1\uace1\uc5ec\uc790\uace0\ub4f1\ud559\uad50\ub97c \uc878\uc5c5\ud588\ub2e4. \ub610\ud55c \ub3d9\uad6d\ub300\ud559\uad50 \uc5f0\uadf9\uc601\ud654\ud559\uacfc\ub97c \uc878\uc5c5\ud588\ub2e4. \uace0\ub4f1\ud559\uc0dd \uc2dc\uc808 KBS\u300a\ub3c4\uc804! \uace8\ub4e0\ubca8\u300b\uc5d0 \ucd9c\uc5f0\ud55c \uc801\uc774 \uc788\ub2e4. \uc601\ud654\u300a\ud0a4\ub2e4\ub9ac \uc544\uc800\uc528\u300b\uc2dc\uc0ac\ud68c\uc758 \ubb34\ub300\uc778\uc0ac\ub97c \uad6c\uacbd\uac14\ub2e4\uac00 \uc2a4\uce74\uc6b0\ud2b8\ub418\uc5b4 \uc5f0\uc608\uacc4\uc5d0 \uc785\ubb38\ud558\uc600\ub2e4. \ubc15\ud558\uc120\uc758 \ud560\uc544\ubc84\uc9c0\ub294 \ub300\ub839 \ucd9c\uc2e0\uc73c\ub85c \ubc15\uc815\ud76c \uc804 \ub300\ud1b5\ub839\uc758 \ud55c \uae30\uc218 \uc544\ub798 \ud6c4\ubc30\ub85c 2012\ub144 \uc5ec\uc131\uc9c0 \u300a'\uc5ec\uc131\uc911\uc559'\u300b5\uc6d4\ud638\ub97c \ud1b5\ud574 \uc790\uc138\ud788 \uc18c\uac1c \ub410\ub2e4\uace0 \ud55c\ub2e4. \ubc15\ud558\uc120\uc740 2005\ub144 SBS \ub4dc\ub77c\ub9c8 \u300a\uc0ac\ub791\uc740 \uae30\uc801\uc774 \ud544\uc694\ud574\u300b\ub85c \ubcf8\uaca9\uc801\uc73c\ub85c \ub4dc\ub77c\ub9c8\uc5d0 \ub370\ubdd4\ud558\uc600\uc73c\uba70, 2006\ub144\u300a \uc544\ud30c\ud2b8 \u300b\ub85c \uc601\ud654\uc5d0 \ub370\ubdd4\ud558\uc600\ub2e4. 2007\ub144 KBS \ub4dc\ub77c\ub9c8 \u300a\uacbd\uc131 \uc2a4\uce94\ub4e4\u300b\ub85c \uc0ac\uadf9\uc5d0 \uccab \ucd9c\uc5f0\ud558\uc600\ub2e4. \uadf8\ub9ac\uace0 2017\ub144 1\uc6d4 22\uc77c\uc5d0 \ubc30\uc6b0 \ub958\uc218\uc601\uacfc \uacb0\ud63c\ud588\ub2e4. 2017\ub144 8\uc6d4 9\uc77c\uc5d0 \uac1c\ubd09\ud55c \uc601\ud654\u300a\uccad\ub144\uacbd\ucc30\u300b\uc5d0\uc11c \uce74\ub9ac\uc2a4\ub9c8 \uc788\ub294 \uacbd\ucc30\ub300\ud559 \uc120\ubc30 \uc8fc\ud76c \uc5ed\uc73c\ub85c \ucd9c\uc5f0 \ud558\uc600\uc73c\uba70, 8\uc6d4 30\uc77c\uc5d0\ub294 \uc601\ud654\u300a\ub85c\ub9c8\uc758 \ud734\uc77c\u300b\uc5d0 \ucd9c\uc5f0\ud588\ub2e4.", "sentence_answer": "2006\ub144\u300a \uc544\ud30c\ud2b8 \u300b\ub85c \uc601\ud654\uc5d0 \ub370\ubdd4\ud558\uc600\ub2e4.", "paragraph_id": "6105539-0-2", "paragraph_question": "question: \ubc15\ud558\uc120\uc758 \uc601\ud654 \ub370\ubdd4\uc791\uc740?, context: \uc11c\uc6b8\ud2b9\ubcc4\uc2dc \uc911\ub791\uad6c \ub9dd\uc6b0\ub3d9\uc5d0\uc11c \ud0dc\uc5b4\ub09c \ubc15\ud558\uc120\uc740 \uc11c\uc6b8\uc911\ud654\ucd08\ub4f1\ud559\uad50, \uc601\ub780\uc5ec\uc790\uc911\ud559\uad50, \uc1a1\uace1\uc5ec\uc790\uace0\ub4f1\ud559\uad50\ub97c \uc878\uc5c5\ud588\ub2e4. \ub610\ud55c \ub3d9\uad6d\ub300\ud559\uad50 \uc5f0\uadf9\uc601\ud654\ud559\uacfc\ub97c \uc878\uc5c5\ud588\ub2e4. \uace0\ub4f1\ud559\uc0dd \uc2dc\uc808 KBS\u300a\ub3c4\uc804! \uace8\ub4e0\ubca8\u300b\uc5d0 \ucd9c\uc5f0\ud55c \uc801\uc774 \uc788\ub2e4. \uc601\ud654\u300a\ud0a4\ub2e4\ub9ac \uc544\uc800\uc528\u300b\uc2dc\uc0ac\ud68c\uc758 \ubb34\ub300\uc778\uc0ac\ub97c \uad6c\uacbd\uac14\ub2e4\uac00 \uc2a4\uce74\uc6b0\ud2b8\ub418\uc5b4 \uc5f0\uc608\uacc4\uc5d0 \uc785\ubb38\ud558\uc600\ub2e4. \ubc15\ud558\uc120\uc758 \ud560\uc544\ubc84\uc9c0\ub294 \ub300\ub839 \ucd9c\uc2e0\uc73c\ub85c \ubc15\uc815\ud76c \uc804 \ub300\ud1b5\ub839\uc758 \ud55c \uae30\uc218 \uc544\ub798 \ud6c4\ubc30\ub85c 2012\ub144 \uc5ec\uc131\uc9c0 \u300a'\uc5ec\uc131\uc911\uc559'\u300b5\uc6d4\ud638\ub97c \ud1b5\ud574 \uc790\uc138\ud788 \uc18c\uac1c \ub410\ub2e4\uace0 \ud55c\ub2e4. \ubc15\ud558\uc120\uc740 2005\ub144 SBS \ub4dc\ub77c\ub9c8 \u300a\uc0ac\ub791\uc740 \uae30\uc801\uc774 \ud544\uc694\ud574\u300b\ub85c \ubcf8\uaca9\uc801\uc73c\ub85c \ub4dc\ub77c\ub9c8\uc5d0 \ub370\ubdd4\ud558\uc600\uc73c\uba70, 2006\ub144\u300a\uc544\ud30c\ud2b8\u300b\ub85c \uc601\ud654\uc5d0 \ub370\ubdd4\ud558\uc600\ub2e4. 2007\ub144 KBS \ub4dc\ub77c\ub9c8 \u300a\uacbd\uc131 \uc2a4\uce94\ub4e4\u300b\ub85c \uc0ac\uadf9\uc5d0 \uccab \ucd9c\uc5f0\ud558\uc600\ub2e4. \uadf8\ub9ac\uace0 2017\ub144 1\uc6d4 22\uc77c\uc5d0 \ubc30\uc6b0 \ub958\uc218\uc601\uacfc \uacb0\ud63c\ud588\ub2e4. 2017\ub144 8\uc6d4 9\uc77c\uc5d0 \uac1c\ubd09\ud55c \uc601\ud654\u300a\uccad\ub144\uacbd\ucc30\u300b\uc5d0\uc11c \uce74\ub9ac\uc2a4\ub9c8 \uc788\ub294 \uacbd\ucc30\ub300\ud559 \uc120\ubc30 \uc8fc\ud76c \uc5ed\uc73c\ub85c \ucd9c\uc5f0 \ud558\uc600\uc73c\uba70, 8\uc6d4 30\uc77c\uc5d0\ub294 \uc601\ud654\u300a\ub85c\ub9c8\uc758 \ud734\uc77c\u300b\uc5d0 \ucd9c\uc5f0\ud588\ub2e4."} {"question": "\uc774\uc81c\ud6c8\uc774 \ube44\ubc00\uc758 \ubb38\uc73c\ub85c SBS \uc5f0\uae30\ub300\uc0c1\uc5d0\uc11c \ubc1b\uc740 \uc0c1\uc740?", "paragraph": "\uc904\uace7 \ub2e8\ud3b8\uc601\ud654\uc640 \ub3c5\ub9bd\uc601\ud654\uacc4\uc5d0\uc11c \uc5f0\uae30\ub97c \ud3bc\uce58\ub2e4 2011\ub144 \u300a\ud30c\uc218\uafbc\u300b\uacfc \u300a\uace0\uc9c0\uc804\u300b \ub450 \uc791\ud488\uc73c\ub85c \uc790\uc2e0\uc758 \uc874\uc7ac\ub97c \ucda9\ubb34\ub85c\uc5d0 \uac01\uc778\uc2dc\ucf30\uace0, \uc2e0\uc778\uc0c1 6\uad00\uc655\uc5d0 \uc62c\ub790\ub2e4. \uc774\ud6c4 \uc601\ud654 \u300a\uac74\ucd95\ud559\uac1c\ub860\u300b, \u300a\uc810\uc7c1\uc774\ub4e4\u300b, \u300a\ubd84\ub178\uc758 \uc724\ub9ac\ud559\u300b, \u300a\ud30c\ud30c\ub85c\ud2f0\u300b\uc5d0 \uc8fc\uc5f0\uc73c\ub85c \ucd9c\uc5f0\ud588\uace0, 2012\ub144\ubd80\ud130 2014\ub144\uae4c\uc9c0 \uc11c\uc6b8\uc9c0\ubc29\uacbd\ucc30\uccad\uc5d0\uc11c \uc758\ubb34\uacbd\ucc30\ub85c \ubcf5\ubb34\ud588\ub2e4. 2014\ub144 \u300a\ube44\ubc00\uc758 \ubb38\u300b\uc744 \ud1b5\ud574 \ubcf5\uadc0\ud588\uace0, SBS \uc5f0\uae30\ub300\uc0c1\uc5d0\uc11c \ucd5c\uc6b0\uc218\uc0c1\uc744 \ubc1b\uc558\ub2e4. 2016\ub144\uc5d0\ub294 \u300a\uc2dc\uadf8\ub110\u300b\uc5d0 \ucd9c\uc5f0\ud588\uace0, \uc601\ud654 \ubcf5\uadc0\uc791\uc778 \u300a\ud0d0\uc815 \ud64d\uae38\ub3d9: \uc0ac\ub77c\uc9c4 \ub9c8\uc744\u300b\uc774 \uac1c\ubd09\ud588\ub2e4. \uadf8\ub9ac\uace0 2017\ub144 2\uc6d4 3\uc77c\u300a\ub0b4\uc77c \uadf8\ub300\uc640\u300b\uc5d0\uc11c \uc720\uc18c\uc900 \uc5ed\ud560\uc744 \ub9e1\uc558\uace0, \uadf8 \ud574 6\uc6d4 28\uc77c \uc601\ud654\u300a\ubc15\uc5f4\u300b\ud070 \ud765\ud589\uc744 \uc774\uc5b4\ub098\uac14\ub2e4. 9\uc6d4 21\uc77c \uc601\ud654\u300a\uc544\uc774 \uce94 \uc2a4\ud53c\ud06c\u300b\uc5d0\uc11c \ubc15\ubbfc\uc7ac \uc5ed\uc744 \ub9e1\uc558\ub2e4.", "answer": "\ucd5c\uc6b0\uc218\uc0c1", "sentence": "2014\ub144 \u300a\ube44\ubc00\uc758 \ubb38\u300b\uc744 \ud1b5\ud574 \ubcf5\uadc0\ud588\uace0, SBS \uc5f0\uae30\ub300\uc0c1\uc5d0\uc11c \ucd5c\uc6b0\uc218\uc0c1 \uc744 \ubc1b\uc558\ub2e4.", "paragraph_sentence": "\uc904\uace7 \ub2e8\ud3b8\uc601\ud654\uc640 \ub3c5\ub9bd\uc601\ud654\uacc4\uc5d0\uc11c \uc5f0\uae30\ub97c \ud3bc\uce58\ub2e4 2011\ub144 \u300a\ud30c\uc218\uafbc\u300b\uacfc \u300a\uace0\uc9c0\uc804\u300b \ub450 \uc791\ud488\uc73c\ub85c \uc790\uc2e0\uc758 \uc874\uc7ac\ub97c \ucda9\ubb34\ub85c\uc5d0 \uac01\uc778\uc2dc\ucf30\uace0, \uc2e0\uc778\uc0c1 6\uad00\uc655\uc5d0 \uc62c\ub790\ub2e4. \uc774\ud6c4 \uc601\ud654 \u300a\uac74\ucd95\ud559\uac1c\ub860\u300b, \u300a\uc810\uc7c1\uc774\ub4e4\u300b, \u300a\ubd84\ub178\uc758 \uc724\ub9ac\ud559\u300b, \u300a\ud30c\ud30c\ub85c\ud2f0\u300b\uc5d0 \uc8fc\uc5f0\uc73c\ub85c \ucd9c\uc5f0\ud588\uace0, 2012\ub144\ubd80\ud130 2014\ub144\uae4c\uc9c0 \uc11c\uc6b8\uc9c0\ubc29\uacbd\ucc30\uccad\uc5d0\uc11c \uc758\ubb34\uacbd\ucc30\ub85c \ubcf5\ubb34\ud588\ub2e4. 2014\ub144 \u300a\ube44\ubc00\uc758 \ubb38\u300b\uc744 \ud1b5\ud574 \ubcf5\uadc0\ud588\uace0, SBS \uc5f0\uae30\ub300\uc0c1\uc5d0\uc11c \ucd5c\uc6b0\uc218\uc0c1 \uc744 \ubc1b\uc558\ub2e4. 2016\ub144\uc5d0\ub294 \u300a\uc2dc\uadf8\ub110\u300b\uc5d0 \ucd9c\uc5f0\ud588\uace0, \uc601\ud654 \ubcf5\uadc0\uc791\uc778 \u300a\ud0d0\uc815 \ud64d\uae38\ub3d9: \uc0ac\ub77c\uc9c4 \ub9c8\uc744\u300b\uc774 \uac1c\ubd09\ud588\ub2e4. \uadf8\ub9ac\uace0 2017\ub144 2\uc6d4 3\uc77c\u300a\ub0b4\uc77c \uadf8\ub300\uc640\u300b\uc5d0\uc11c \uc720\uc18c\uc900 \uc5ed\ud560\uc744 \ub9e1\uc558\uace0, \uadf8 \ud574 6\uc6d4 28\uc77c \uc601\ud654\u300a\ubc15\uc5f4\u300b\ud070 \ud765\ud589\uc744 \uc774\uc5b4\ub098\uac14\ub2e4. 9\uc6d4 21\uc77c \uc601\ud654\u300a\uc544\uc774 \uce94 \uc2a4\ud53c\ud06c\u300b\uc5d0\uc11c \ubc15\ubbfc\uc7ac \uc5ed\uc744 \ub9e1\uc558\ub2e4.", "paragraph_answer": "\uc904\uace7 \ub2e8\ud3b8\uc601\ud654\uc640 \ub3c5\ub9bd\uc601\ud654\uacc4\uc5d0\uc11c \uc5f0\uae30\ub97c \ud3bc\uce58\ub2e4 2011\ub144 \u300a\ud30c\uc218\uafbc\u300b\uacfc \u300a\uace0\uc9c0\uc804\u300b \ub450 \uc791\ud488\uc73c\ub85c \uc790\uc2e0\uc758 \uc874\uc7ac\ub97c \ucda9\ubb34\ub85c\uc5d0 \uac01\uc778\uc2dc\ucf30\uace0, \uc2e0\uc778\uc0c1 6\uad00\uc655\uc5d0 \uc62c\ub790\ub2e4. \uc774\ud6c4 \uc601\ud654 \u300a\uac74\ucd95\ud559\uac1c\ub860\u300b, \u300a\uc810\uc7c1\uc774\ub4e4\u300b, \u300a\ubd84\ub178\uc758 \uc724\ub9ac\ud559\u300b, \u300a\ud30c\ud30c\ub85c\ud2f0\u300b\uc5d0 \uc8fc\uc5f0\uc73c\ub85c \ucd9c\uc5f0\ud588\uace0, 2012\ub144\ubd80\ud130 2014\ub144\uae4c\uc9c0 \uc11c\uc6b8\uc9c0\ubc29\uacbd\ucc30\uccad\uc5d0\uc11c \uc758\ubb34\uacbd\ucc30\ub85c \ubcf5\ubb34\ud588\ub2e4. 2014\ub144 \u300a\ube44\ubc00\uc758 \ubb38\u300b\uc744 \ud1b5\ud574 \ubcf5\uadc0\ud588\uace0, SBS \uc5f0\uae30\ub300\uc0c1\uc5d0\uc11c \ucd5c\uc6b0\uc218\uc0c1 \uc744 \ubc1b\uc558\ub2e4. 2016\ub144\uc5d0\ub294 \u300a\uc2dc\uadf8\ub110\u300b\uc5d0 \ucd9c\uc5f0\ud588\uace0, \uc601\ud654 \ubcf5\uadc0\uc791\uc778 \u300a\ud0d0\uc815 \ud64d\uae38\ub3d9: \uc0ac\ub77c\uc9c4 \ub9c8\uc744\u300b\uc774 \uac1c\ubd09\ud588\ub2e4. \uadf8\ub9ac\uace0 2017\ub144 2\uc6d4 3\uc77c\u300a\ub0b4\uc77c \uadf8\ub300\uc640\u300b\uc5d0\uc11c \uc720\uc18c\uc900 \uc5ed\ud560\uc744 \ub9e1\uc558\uace0, \uadf8 \ud574 6\uc6d4 28\uc77c \uc601\ud654\u300a\ubc15\uc5f4\u300b\ud070 \ud765\ud589\uc744 \uc774\uc5b4\ub098\uac14\ub2e4. 9\uc6d4 21\uc77c \uc601\ud654\u300a\uc544\uc774 \uce94 \uc2a4\ud53c\ud06c\u300b\uc5d0\uc11c \ubc15\ubbfc\uc7ac \uc5ed\uc744 \ub9e1\uc558\ub2e4.", "sentence_answer": "2014\ub144 \u300a\ube44\ubc00\uc758 \ubb38\u300b\uc744 \ud1b5\ud574 \ubcf5\uadc0\ud588\uace0, SBS \uc5f0\uae30\ub300\uc0c1\uc5d0\uc11c \ucd5c\uc6b0\uc218\uc0c1 \uc744 \ubc1b\uc558\ub2e4.", "paragraph_id": "6547787-0-1", "paragraph_question": "question: \uc774\uc81c\ud6c8\uc774 \ube44\ubc00\uc758 \ubb38\uc73c\ub85c SBS \uc5f0\uae30\ub300\uc0c1\uc5d0\uc11c \ubc1b\uc740 \uc0c1\uc740?, context: \uc904\uace7 \ub2e8\ud3b8\uc601\ud654\uc640 \ub3c5\ub9bd\uc601\ud654\uacc4\uc5d0\uc11c \uc5f0\uae30\ub97c \ud3bc\uce58\ub2e4 2011\ub144 \u300a\ud30c\uc218\uafbc\u300b\uacfc \u300a\uace0\uc9c0\uc804\u300b \ub450 \uc791\ud488\uc73c\ub85c \uc790\uc2e0\uc758 \uc874\uc7ac\ub97c \ucda9\ubb34\ub85c\uc5d0 \uac01\uc778\uc2dc\ucf30\uace0, \uc2e0\uc778\uc0c1 6\uad00\uc655\uc5d0 \uc62c\ub790\ub2e4. \uc774\ud6c4 \uc601\ud654 \u300a\uac74\ucd95\ud559\uac1c\ub860\u300b, \u300a\uc810\uc7c1\uc774\ub4e4\u300b, \u300a\ubd84\ub178\uc758 \uc724\ub9ac\ud559\u300b, \u300a\ud30c\ud30c\ub85c\ud2f0\u300b\uc5d0 \uc8fc\uc5f0\uc73c\ub85c \ucd9c\uc5f0\ud588\uace0, 2012\ub144\ubd80\ud130 2014\ub144\uae4c\uc9c0 \uc11c\uc6b8\uc9c0\ubc29\uacbd\ucc30\uccad\uc5d0\uc11c \uc758\ubb34\uacbd\ucc30\ub85c \ubcf5\ubb34\ud588\ub2e4. 2014\ub144 \u300a\ube44\ubc00\uc758 \ubb38\u300b\uc744 \ud1b5\ud574 \ubcf5\uadc0\ud588\uace0, SBS \uc5f0\uae30\ub300\uc0c1\uc5d0\uc11c \ucd5c\uc6b0\uc218\uc0c1\uc744 \ubc1b\uc558\ub2e4. 2016\ub144\uc5d0\ub294 \u300a\uc2dc\uadf8\ub110\u300b\uc5d0 \ucd9c\uc5f0\ud588\uace0, \uc601\ud654 \ubcf5\uadc0\uc791\uc778 \u300a\ud0d0\uc815 \ud64d\uae38\ub3d9: \uc0ac\ub77c\uc9c4 \ub9c8\uc744\u300b\uc774 \uac1c\ubd09\ud588\ub2e4. \uadf8\ub9ac\uace0 2017\ub144 2\uc6d4 3\uc77c\u300a\ub0b4\uc77c \uadf8\ub300\uc640\u300b\uc5d0\uc11c \uc720\uc18c\uc900 \uc5ed\ud560\uc744 \ub9e1\uc558\uace0, \uadf8 \ud574 6\uc6d4 28\uc77c \uc601\ud654\u300a\ubc15\uc5f4\u300b\ud070 \ud765\ud589\uc744 \uc774\uc5b4\ub098\uac14\ub2e4. 9\uc6d4 21\uc77c \uc601\ud654\u300a\uc544\uc774 \uce94 \uc2a4\ud53c\ud06c\u300b\uc5d0\uc11c \ubc15\ubbfc\uc7ac \uc5ed\uc744 \ub9e1\uc558\ub2e4."} {"question": "\ubbf8\uad6d\uc5d0\uc11c \uc2e0\ubb38\ubc29\uc1a1\ud559\uc758 \ucd9c\ud604\uc740 \ud504\ub85c\ud398\uc154\ub0a0\ub9ac\uc998\uc758 \ud615\uc131\uacfc \ubb34\uc5c7\uc774 \ub9de\ubb3c\ub824 \uc788\ub294\uac00?", "paragraph": "\ubbf8\uad6d\uc5d0\uc11c \uc2e0\ubb38\ubc29\uc1a1\ud559\uc758 \ucd9c\ud604\uc740 \ud504\ub85c\ud398\uc154\ub0a0\ub9ac\uc998\uc758 \ud615\uc131\uacfc \ubbf8\ub514\uc5b4 \ubc1c\ub2ec\uacfc \ub9de\ubb3c\ub824 \uc788\ub2e4. \uba3c\uc800 \uc2e0\ubb38\uc758 \ubc1c\ub2ec\ub85c \uc778\ud574\uc11c \uc800\ub110\ub9ac\uc998 \uad50\uc721\uc758 \ud544\uc694\uc131\uc774 \uc81c\uae30\ub418\uc5c8\ub2e4. \ubbf8\uad6d\uc0ac\ud68c\uc5d0\uc11c\ub294 19\uc138\ub9d0 \uc758\uacfc\ub300\ud559, \ubc95\uacfc\ub300\ud559, \uad50\uc721\ub300\ud559 \ub4f1\uc774 \uc2e4\ubb34\uc640 \uad50\uc591\uc744 \ud568\uaed8 \uac00\ub974\uce58\ub294 \uc9c1\uc5c5\ud559\uad50\ub85c \ubc1c\ub2ec\ud558\uac8c \ub41c\ub2e4. \ub0a8\ubd81\uc804\uc7c1\uc774\ud6c4 \ub0a8\ubd80\uad70\uc758 \ub85c\ubc84\ud2b8 \ub9ac \uc7a5\uad70\uc774 \ubc84\uc9c0\ub2c8\uc544\uc8fc\uc758 \uc6cc\uc2f1\ud134 \uceec\ub9ac\uc9c0 \ud559\uc7a5\uc73c\ub85c \uc788\uc744 \ub54c, \uc5b8\ub860\uc778 \uad50\uc721\uc758 \ud544\uc694\uc131\uc744 \ub290\ub07c\uc5b4\uc11c 1860\ub144\ub300\ud6c4\ubc18\ubd80\ud130 \uc800\ub110\ub9ac\uc998 \uad50\uc721\uc744 \uac00\ub974\uce58\uae30 \uc2dc\uc791\ud55c \uac83\uc774 \ub300\ud559\uc218\uc900\uc5d0\uc11c \ucd5c\ucd08\uc758 \uc800\ub110\ub9ac\uc998 \uad50\uc721\uc73c\ub85c \ud1b5\uc6a9\ub418\uace0 \uc788\ub2e4 . \uc774\ud6c4 \uc6cc\uc2f1\ud134 \uceec\ub9ac\uc9c0\ub294 \uc6cc\uc2f1\ud134 \uc564 \ub9ac(Washington and Lee University) \ub300\ud559\uc73c\ub85c \ubc1c\ub2ec\ud55c\ub2e4. \ucd08\uae30 \uad50\uc721\uacfc\uc815\uc740 \ud65c\uc790, \uc18d\uae30, \ud68c\uacc4\ud559, \uac04\ub2e8\ud55c \uc800\ub110\ub9ac\uc998 \uc774\ub860 \ub4f1\uc744 \uac00\ub974\ucce4\ub2e4. \ubb3c\ub860 \uc800\ub110\ub9ac\uc998 \uc774\ub860\uc774 \uc544\uc9c1 \ubc1c\ub2ec\ud558\uc9c0 \uc54a\uc544\uc11c \uc774\ub860\uc774\ub77c\uace0 \ub531\ud788 \ubd80\ub97c \uac83\uc774 \uc5c6\ub358 \uc2dc\uae30\uc600\ub2e4.", "answer": "\ubbf8\ub514\uc5b4 \ubc1c\ub2ec", "sentence": "\ubbf8\uad6d\uc5d0\uc11c \uc2e0\ubb38\ubc29\uc1a1\ud559\uc758 \ucd9c\ud604\uc740 \ud504\ub85c\ud398\uc154\ub0a0\ub9ac\uc998\uc758 \ud615\uc131\uacfc \ubbf8\ub514\uc5b4 \ubc1c\ub2ec \uacfc \ub9de\ubb3c\ub824 \uc788\ub2e4.", "paragraph_sentence": " \ubbf8\uad6d\uc5d0\uc11c \uc2e0\ubb38\ubc29\uc1a1\ud559\uc758 \ucd9c\ud604\uc740 \ud504\ub85c\ud398\uc154\ub0a0\ub9ac\uc998\uc758 \ud615\uc131\uacfc \ubbf8\ub514\uc5b4 \ubc1c\ub2ec \uacfc \ub9de\ubb3c\ub824 \uc788\ub2e4. \uba3c\uc800 \uc2e0\ubb38\uc758 \ubc1c\ub2ec\ub85c \uc778\ud574\uc11c \uc800\ub110\ub9ac\uc998 \uad50\uc721\uc758 \ud544\uc694\uc131\uc774 \uc81c\uae30\ub418\uc5c8\ub2e4. \ubbf8\uad6d\uc0ac\ud68c\uc5d0\uc11c\ub294 19\uc138\ub9d0 \uc758\uacfc\ub300\ud559, \ubc95\uacfc\ub300\ud559, \uad50\uc721\ub300\ud559 \ub4f1\uc774 \uc2e4\ubb34\uc640 \uad50\uc591\uc744 \ud568\uaed8 \uac00\ub974\uce58\ub294 \uc9c1\uc5c5\ud559\uad50\ub85c \ubc1c\ub2ec\ud558\uac8c \ub41c\ub2e4. \ub0a8\ubd81\uc804\uc7c1\uc774\ud6c4 \ub0a8\ubd80\uad70\uc758 \ub85c\ubc84\ud2b8 \ub9ac \uc7a5\uad70\uc774 \ubc84\uc9c0\ub2c8\uc544\uc8fc\uc758 \uc6cc\uc2f1\ud134 \uceec\ub9ac\uc9c0 \ud559\uc7a5\uc73c\ub85c \uc788\uc744 \ub54c, \uc5b8\ub860\uc778 \uad50\uc721\uc758 \ud544\uc694\uc131\uc744 \ub290\ub07c\uc5b4\uc11c 1860\ub144\ub300\ud6c4\ubc18\ubd80\ud130 \uc800\ub110\ub9ac\uc998 \uad50\uc721\uc744 \uac00\ub974\uce58\uae30 \uc2dc\uc791\ud55c \uac83\uc774 \ub300\ud559\uc218\uc900\uc5d0\uc11c \ucd5c\ucd08\uc758 \uc800\ub110\ub9ac\uc998 \uad50\uc721\uc73c\ub85c \ud1b5\uc6a9\ub418\uace0 \uc788\ub2e4 . \uc774\ud6c4 \uc6cc\uc2f1\ud134 \uceec\ub9ac\uc9c0\ub294 \uc6cc\uc2f1\ud134 \uc564 \ub9ac(Washington and Lee University) \ub300\ud559\uc73c\ub85c \ubc1c\ub2ec\ud55c\ub2e4. \ucd08\uae30 \uad50\uc721\uacfc\uc815\uc740 \ud65c\uc790, \uc18d\uae30, \ud68c\uacc4\ud559, \uac04\ub2e8\ud55c \uc800\ub110\ub9ac\uc998 \uc774\ub860 \ub4f1\uc744 \uac00\ub974\ucce4\ub2e4. \ubb3c\ub860 \uc800\ub110\ub9ac\uc998 \uc774\ub860\uc774 \uc544\uc9c1 \ubc1c\ub2ec\ud558\uc9c0 \uc54a\uc544\uc11c \uc774\ub860\uc774\ub77c\uace0 \ub531\ud788 \ubd80\ub97c \uac83\uc774 \uc5c6\ub358 \uc2dc\uae30\uc600\ub2e4.", "paragraph_answer": "\ubbf8\uad6d\uc5d0\uc11c \uc2e0\ubb38\ubc29\uc1a1\ud559\uc758 \ucd9c\ud604\uc740 \ud504\ub85c\ud398\uc154\ub0a0\ub9ac\uc998\uc758 \ud615\uc131\uacfc \ubbf8\ub514\uc5b4 \ubc1c\ub2ec \uacfc \ub9de\ubb3c\ub824 \uc788\ub2e4. \uba3c\uc800 \uc2e0\ubb38\uc758 \ubc1c\ub2ec\ub85c \uc778\ud574\uc11c \uc800\ub110\ub9ac\uc998 \uad50\uc721\uc758 \ud544\uc694\uc131\uc774 \uc81c\uae30\ub418\uc5c8\ub2e4. \ubbf8\uad6d\uc0ac\ud68c\uc5d0\uc11c\ub294 19\uc138\ub9d0 \uc758\uacfc\ub300\ud559, \ubc95\uacfc\ub300\ud559, \uad50\uc721\ub300\ud559 \ub4f1\uc774 \uc2e4\ubb34\uc640 \uad50\uc591\uc744 \ud568\uaed8 \uac00\ub974\uce58\ub294 \uc9c1\uc5c5\ud559\uad50\ub85c \ubc1c\ub2ec\ud558\uac8c \ub41c\ub2e4. \ub0a8\ubd81\uc804\uc7c1\uc774\ud6c4 \ub0a8\ubd80\uad70\uc758 \ub85c\ubc84\ud2b8 \ub9ac \uc7a5\uad70\uc774 \ubc84\uc9c0\ub2c8\uc544\uc8fc\uc758 \uc6cc\uc2f1\ud134 \uceec\ub9ac\uc9c0 \ud559\uc7a5\uc73c\ub85c \uc788\uc744 \ub54c, \uc5b8\ub860\uc778 \uad50\uc721\uc758 \ud544\uc694\uc131\uc744 \ub290\ub07c\uc5b4\uc11c 1860\ub144\ub300\ud6c4\ubc18\ubd80\ud130 \uc800\ub110\ub9ac\uc998 \uad50\uc721\uc744 \uac00\ub974\uce58\uae30 \uc2dc\uc791\ud55c \uac83\uc774 \ub300\ud559\uc218\uc900\uc5d0\uc11c \ucd5c\ucd08\uc758 \uc800\ub110\ub9ac\uc998 \uad50\uc721\uc73c\ub85c \ud1b5\uc6a9\ub418\uace0 \uc788\ub2e4 . \uc774\ud6c4 \uc6cc\uc2f1\ud134 \uceec\ub9ac\uc9c0\ub294 \uc6cc\uc2f1\ud134 \uc564 \ub9ac(Washington and Lee University) \ub300\ud559\uc73c\ub85c \ubc1c\ub2ec\ud55c\ub2e4. \ucd08\uae30 \uad50\uc721\uacfc\uc815\uc740 \ud65c\uc790, \uc18d\uae30, \ud68c\uacc4\ud559, \uac04\ub2e8\ud55c \uc800\ub110\ub9ac\uc998 \uc774\ub860 \ub4f1\uc744 \uac00\ub974\ucce4\ub2e4. \ubb3c\ub860 \uc800\ub110\ub9ac\uc998 \uc774\ub860\uc774 \uc544\uc9c1 \ubc1c\ub2ec\ud558\uc9c0 \uc54a\uc544\uc11c \uc774\ub860\uc774\ub77c\uace0 \ub531\ud788 \ubd80\ub97c \uac83\uc774 \uc5c6\ub358 \uc2dc\uae30\uc600\ub2e4.", "sentence_answer": "\ubbf8\uad6d\uc5d0\uc11c \uc2e0\ubb38\ubc29\uc1a1\ud559\uc758 \ucd9c\ud604\uc740 \ud504\ub85c\ud398\uc154\ub0a0\ub9ac\uc998\uc758 \ud615\uc131\uacfc \ubbf8\ub514\uc5b4 \ubc1c\ub2ec \uacfc \ub9de\ubb3c\ub824 \uc788\ub2e4.", "paragraph_id": "6463864-1-0", "paragraph_question": "question: \ubbf8\uad6d\uc5d0\uc11c \uc2e0\ubb38\ubc29\uc1a1\ud559\uc758 \ucd9c\ud604\uc740 \ud504\ub85c\ud398\uc154\ub0a0\ub9ac\uc998\uc758 \ud615\uc131\uacfc \ubb34\uc5c7\uc774 \ub9de\ubb3c\ub824 \uc788\ub294\uac00?, context: \ubbf8\uad6d\uc5d0\uc11c \uc2e0\ubb38\ubc29\uc1a1\ud559\uc758 \ucd9c\ud604\uc740 \ud504\ub85c\ud398\uc154\ub0a0\ub9ac\uc998\uc758 \ud615\uc131\uacfc \ubbf8\ub514\uc5b4 \ubc1c\ub2ec\uacfc \ub9de\ubb3c\ub824 \uc788\ub2e4. \uba3c\uc800 \uc2e0\ubb38\uc758 \ubc1c\ub2ec\ub85c \uc778\ud574\uc11c \uc800\ub110\ub9ac\uc998 \uad50\uc721\uc758 \ud544\uc694\uc131\uc774 \uc81c\uae30\ub418\uc5c8\ub2e4. \ubbf8\uad6d\uc0ac\ud68c\uc5d0\uc11c\ub294 19\uc138\ub9d0 \uc758\uacfc\ub300\ud559, \ubc95\uacfc\ub300\ud559, \uad50\uc721\ub300\ud559 \ub4f1\uc774 \uc2e4\ubb34\uc640 \uad50\uc591\uc744 \ud568\uaed8 \uac00\ub974\uce58\ub294 \uc9c1\uc5c5\ud559\uad50\ub85c \ubc1c\ub2ec\ud558\uac8c \ub41c\ub2e4. \ub0a8\ubd81\uc804\uc7c1\uc774\ud6c4 \ub0a8\ubd80\uad70\uc758 \ub85c\ubc84\ud2b8 \ub9ac \uc7a5\uad70\uc774 \ubc84\uc9c0\ub2c8\uc544\uc8fc\uc758 \uc6cc\uc2f1\ud134 \uceec\ub9ac\uc9c0 \ud559\uc7a5\uc73c\ub85c \uc788\uc744 \ub54c, \uc5b8\ub860\uc778 \uad50\uc721\uc758 \ud544\uc694\uc131\uc744 \ub290\ub07c\uc5b4\uc11c 1860\ub144\ub300\ud6c4\ubc18\ubd80\ud130 \uc800\ub110\ub9ac\uc998 \uad50\uc721\uc744 \uac00\ub974\uce58\uae30 \uc2dc\uc791\ud55c \uac83\uc774 \ub300\ud559\uc218\uc900\uc5d0\uc11c \ucd5c\ucd08\uc758 \uc800\ub110\ub9ac\uc998 \uad50\uc721\uc73c\ub85c \ud1b5\uc6a9\ub418\uace0 \uc788\ub2e4 . \uc774\ud6c4 \uc6cc\uc2f1\ud134 \uceec\ub9ac\uc9c0\ub294 \uc6cc\uc2f1\ud134 \uc564 \ub9ac(Washington and Lee University) \ub300\ud559\uc73c\ub85c \ubc1c\ub2ec\ud55c\ub2e4. \ucd08\uae30 \uad50\uc721\uacfc\uc815\uc740 \ud65c\uc790, \uc18d\uae30, \ud68c\uacc4\ud559, \uac04\ub2e8\ud55c \uc800\ub110\ub9ac\uc998 \uc774\ub860 \ub4f1\uc744 \uac00\ub974\ucce4\ub2e4. \ubb3c\ub860 \uc800\ub110\ub9ac\uc998 \uc774\ub860\uc774 \uc544\uc9c1 \ubc1c\ub2ec\ud558\uc9c0 \uc54a\uc544\uc11c \uc774\ub860\uc774\ub77c\uace0 \ub531\ud788 \ubd80\ub97c \uac83\uc774 \uc5c6\ub358 \uc2dc\uae30\uc600\ub2e4."} {"question": "2009\ub144 12\uc6d4 24\uc77c\ubd80\ud130 2010\ub144 1\uc6d4 31\uc77c \uc0ac\uc774\uc5d0 \ub86f\ub370\uc2dc\ub124\ub9c8\uc758 \uc544\ub974\uc138\uc6b0\uc2a4 \ucd08\uadf9\uc758 \uc2dc\uacf5\uc73c\ub85c \uc0c1\uc601\uad00\uc5d0\uc11c \ubc30\ud3ec\ud55c \ud3ec\ucf13\ubaac\uc758 \uc774\ub984\uc740 \ubb34\uc5c7\uc785\ub2c8\uae4c?", "paragraph": "\ub300\ud55c\ubbfc\uad6d\uc5d0\uc11c\ub294 2009\ub144 12\uc6d4 10\uc77c, \u300a\ud3ec\ucf13\ubaac\uc2a4\ud130 \ud558\ud2b8\uace8\ub4dc\u00b7\uc18c\uc6b8\uc2e4\ubc84\u300b\ub97c 2010\ub144 2\uc6d4 4\uc77c \uc815\uc2dd \ubc1c\ub9e4\ud55c\ub2e4\uace0 \uacf5\uac1c\ud558\uc600\ub2e4. \ud55c\uad6d\ub2cc\ud150\ub3c4\uc5d0\uc11c\ub294 2010\ub144 1\uc6d4 30\uc77c\uacfc 1\uc6d4 31\uc77c \uc81c7\ucc28 \ud3ec\ucf13\ubaac \ubaa8\uc73c\uae30 \uc774\ubca4\ud2b8\ub97c \uac1c\ucd5c\ud558\uc5ec \u300a\ud3ec\ucf13\ubaac\uc2a4\ud130DP \ub514\uc544\ub8e8\uac00\u00b7\ud384\uae30\uc544\u300b, \u300aPt \uae30\ub77c\ud2f0\ub098\u300b \ud50c\ub808\uc774\uc5b4\uc5d0\uac8c \"\uc774\uc0c1\ud55c \uce74\ub4dc\"\ub97c \ud1b5\ud574 \ud3ec\ucf13\ubaac \uc9c0\ub77c\uce58\ub97c \ubc30\ud3ec\ud558\uc600\ub2e4. \uc774 \uc9c0\ub77c\uce58\ub97c \u300a\ud558\ud2b8\uace8\ub4dc\u00b7\uc18c\uc6b8\uc2e4\ubc84\u300b\ub85c \uc62e\uae30\uba74 \ud3ec\ucf13\uc6cc\ucee4\uc5d0 \ucf54\uc2a4\ub9f5 \"\ubc24\ud558\ub298\uc758 \ub05d\"\uc774 \ucd94\uac00\ub41c\ub2e4. 2009\ub144 12\uc6d4 24\uc77c\ubd80\ud130 2010\ub144 1\uc6d4 31\uc77c\uae4c\uc9c0 \ub86f\ub370\uc2dc\ub124\ub9c8\uc758 \u300a\uc544\ub974\uc138\uc6b0\uc2a4 \ucd08\uadf9\uc758 \uc2dc\uacf5\uc73c\ub85c\u300b \uc0c1\uc601\uad00\uc5d0\uc11c \ubc30\ud3ec \ubc1b\uc740 \ud3ec\ucf13\ubaac \uc544\ub974\uc138\uc6b0\uc2a4\ub97c \uac8c\uc784 \ub0b4\ub85c \ub370\ub824\uac00\uba74 \ub514\uc544\ub8e8\uac00, \ud384\uae30\uc544, \uae30\ub77c\ud2f0\ub098 \uc911 \ud55c \ub9c8\ub9ac\ub97c \uc5bb\uc744 \uc218 \uc788\uc73c\uba70, \uc601\ud654 \uc608\ub9e4\uad8c\uc744 \uc9c0\ucc38\ud55c \uc0ac\ub78c\uc5d0 \ud55c\ud574 \uc9c0\uc815\ub41c \uc0c1\uc601\uad00\uc5d0\uc11c \ubc30\ud3ec\ud558\uc600\ub358 \"\ud53c\uce74\uce04 \uceec\ub7ec\uc758 \ud53c\uce04\"\ub294 \uac8c\uc784 \ub0b4 \ub108\ub3c4\ubc24\ub098\ubb34 \uc232\uc73c\ub85c \ub370\ub824\uac00\uba74 \"\uc090\ucb49\uadc0 \ud53c\uce04\"\ub97c \uc5bb\uc744 \uc218 \uc788\ub2e4. \ubc1c\ub9e4 \uae30\ub150\uc73c\ub85c E\ub9c8\ud2b8\uc5d0\uc11c \uac8c\uc784\uc744 \uad6c\ub9e4\ud558\uba74 \uc120\ucc29\uc21c\uc73c\ub85c \uce60\uc0c9\uc870\uc640 \ub8e8\uae30\uc544 \ud53c\uaddc\uc5b4\ub97c \uc99d\uc815\ud558\uc600\uace0, \ucd08\ud68c\ud310\uc5d0\ub294 \ud3ec\ucf13\ubaac \uce74\ub4dc \uac8c\uc784\uc758 \uce60\uc0c9\uc870\uc640 \ub8e8\uae30\uc544 \ud504\ub85c\ubaa8 \uce74\ub4dc\ub97c \uac01\uac01 \ub3d9\ubd09\ud558\uc600\ub2e4.", "answer": "\uc544\ub974\uc138\uc6b0\uc2a4", "sentence": "2009\ub144 12\uc6d4 24\uc77c\ubd80\ud130 2010\ub144 1\uc6d4 31\uc77c\uae4c\uc9c0 \ub86f\ub370\uc2dc\ub124\ub9c8\uc758 \u300a \uc544\ub974\uc138\uc6b0\uc2a4 \ucd08\uadf9\uc758 \uc2dc\uacf5\uc73c\ub85c\u300b \uc0c1\uc601\uad00\uc5d0\uc11c \ubc30\ud3ec \ubc1b\uc740 \ud3ec\ucf13\ubaac \uc544\ub974\uc138\uc6b0\uc2a4\ub97c \uac8c\uc784 \ub0b4\ub85c \ub370\ub824\uac00\uba74 \ub514\uc544\ub8e8\uac00, \ud384\uae30\uc544, \uae30\ub77c\ud2f0\ub098 \uc911 \ud55c \ub9c8\ub9ac\ub97c \uc5bb\uc744 \uc218 \uc788\uc73c\uba70, \uc601\ud654 \uc608\ub9e4\uad8c\uc744 \uc9c0\ucc38\ud55c \uc0ac\ub78c\uc5d0 \ud55c\ud574 \uc9c0\uc815\ub41c \uc0c1\uc601\uad00\uc5d0\uc11c \ubc30\ud3ec\ud558\uc600\ub358 \"\ud53c\uce74\uce04 \uceec\ub7ec\uc758 \ud53c\uce04\"\ub294 \uac8c\uc784 \ub0b4 \ub108\ub3c4\ubc24\ub098\ubb34 \uc232\uc73c\ub85c \ub370\ub824\uac00\uba74 \"\uc090\ucb49\uadc0 \ud53c\uce04\"\ub97c \uc5bb\uc744 \uc218 \uc788\ub2e4.", "paragraph_sentence": "\ub300\ud55c\ubbfc\uad6d\uc5d0\uc11c\ub294 2009\ub144 12\uc6d4 10\uc77c, \u300a\ud3ec\ucf13\ubaac\uc2a4\ud130 \ud558\ud2b8\uace8\ub4dc\u00b7\uc18c\uc6b8\uc2e4\ubc84\u300b\ub97c 2010\ub144 2\uc6d4 4\uc77c \uc815\uc2dd \ubc1c\ub9e4\ud55c\ub2e4\uace0 \uacf5\uac1c\ud558\uc600\ub2e4. \ud55c\uad6d\ub2cc\ud150\ub3c4\uc5d0\uc11c\ub294 2010\ub144 1\uc6d4 30\uc77c\uacfc 1\uc6d4 31\uc77c \uc81c7\ucc28 \ud3ec\ucf13\ubaac \ubaa8\uc73c\uae30 \uc774\ubca4\ud2b8\ub97c \uac1c\ucd5c\ud558\uc5ec \u300a\ud3ec\ucf13\ubaac\uc2a4\ud130DP \ub514\uc544\ub8e8\uac00\u00b7\ud384\uae30\uc544\u300b, \u300aPt \uae30\ub77c\ud2f0\ub098\u300b \ud50c\ub808\uc774\uc5b4\uc5d0\uac8c \"\uc774\uc0c1\ud55c \uce74\ub4dc\"\ub97c \ud1b5\ud574 \ud3ec\ucf13\ubaac \uc9c0\ub77c\uce58\ub97c \ubc30\ud3ec\ud558\uc600\ub2e4. \uc774 \uc9c0\ub77c\uce58\ub97c \u300a\ud558\ud2b8\uace8\ub4dc\u00b7\uc18c\uc6b8\uc2e4\ubc84\u300b\ub85c \uc62e\uae30\uba74 \ud3ec\ucf13\uc6cc\ucee4\uc5d0 \ucf54\uc2a4\ub9f5 \"\ubc24\ud558\ub298\uc758 \ub05d\"\uc774 \ucd94\uac00\ub41c\ub2e4. 2009\ub144 12\uc6d4 24\uc77c\ubd80\ud130 2010\ub144 1\uc6d4 31\uc77c\uae4c\uc9c0 \ub86f\ub370\uc2dc\ub124\ub9c8\uc758 \u300a \uc544\ub974\uc138\uc6b0\uc2a4 \ucd08\uadf9\uc758 \uc2dc\uacf5\uc73c\ub85c\u300b \uc0c1\uc601\uad00\uc5d0\uc11c \ubc30\ud3ec \ubc1b\uc740 \ud3ec\ucf13\ubaac \uc544\ub974\uc138\uc6b0\uc2a4\ub97c \uac8c\uc784 \ub0b4\ub85c \ub370\ub824\uac00\uba74 \ub514\uc544\ub8e8\uac00, \ud384\uae30\uc544, \uae30\ub77c\ud2f0\ub098 \uc911 \ud55c \ub9c8\ub9ac\ub97c \uc5bb\uc744 \uc218 \uc788\uc73c\uba70, \uc601\ud654 \uc608\ub9e4\uad8c\uc744 \uc9c0\ucc38\ud55c \uc0ac\ub78c\uc5d0 \ud55c\ud574 \uc9c0\uc815\ub41c \uc0c1\uc601\uad00\uc5d0\uc11c \ubc30\ud3ec\ud558\uc600\ub358 \"\ud53c\uce74\uce04 \uceec\ub7ec\uc758 \ud53c\uce04\"\ub294 \uac8c\uc784 \ub0b4 \ub108\ub3c4\ubc24\ub098\ubb34 \uc232\uc73c\ub85c \ub370\ub824\uac00\uba74 \"\uc090\ucb49\uadc0 \ud53c\uce04\"\ub97c \uc5bb\uc744 \uc218 \uc788\ub2e4. \ubc1c\ub9e4 \uae30\ub150\uc73c\ub85c E\ub9c8\ud2b8\uc5d0\uc11c \uac8c\uc784\uc744 \uad6c\ub9e4\ud558\uba74 \uc120\ucc29\uc21c\uc73c\ub85c \uce60\uc0c9\uc870\uc640 \ub8e8\uae30\uc544 \ud53c\uaddc\uc5b4\ub97c \uc99d\uc815\ud558\uc600\uace0, \ucd08\ud68c\ud310\uc5d0\ub294 \ud3ec\ucf13\ubaac \uce74\ub4dc \uac8c\uc784\uc758 \uce60\uc0c9\uc870\uc640 \ub8e8\uae30\uc544 \ud504\ub85c\ubaa8 \uce74\ub4dc\ub97c \uac01\uac01 \ub3d9\ubd09\ud558\uc600\ub2e4.", "paragraph_answer": "\ub300\ud55c\ubbfc\uad6d\uc5d0\uc11c\ub294 2009\ub144 12\uc6d4 10\uc77c, \u300a\ud3ec\ucf13\ubaac\uc2a4\ud130 \ud558\ud2b8\uace8\ub4dc\u00b7\uc18c\uc6b8\uc2e4\ubc84\u300b\ub97c 2010\ub144 2\uc6d4 4\uc77c \uc815\uc2dd \ubc1c\ub9e4\ud55c\ub2e4\uace0 \uacf5\uac1c\ud558\uc600\ub2e4. \ud55c\uad6d\ub2cc\ud150\ub3c4\uc5d0\uc11c\ub294 2010\ub144 1\uc6d4 30\uc77c\uacfc 1\uc6d4 31\uc77c \uc81c7\ucc28 \ud3ec\ucf13\ubaac \ubaa8\uc73c\uae30 \uc774\ubca4\ud2b8\ub97c \uac1c\ucd5c\ud558\uc5ec \u300a\ud3ec\ucf13\ubaac\uc2a4\ud130DP \ub514\uc544\ub8e8\uac00\u00b7\ud384\uae30\uc544\u300b, \u300aPt \uae30\ub77c\ud2f0\ub098\u300b \ud50c\ub808\uc774\uc5b4\uc5d0\uac8c \"\uc774\uc0c1\ud55c \uce74\ub4dc\"\ub97c \ud1b5\ud574 \ud3ec\ucf13\ubaac \uc9c0\ub77c\uce58\ub97c \ubc30\ud3ec\ud558\uc600\ub2e4. \uc774 \uc9c0\ub77c\uce58\ub97c \u300a\ud558\ud2b8\uace8\ub4dc\u00b7\uc18c\uc6b8\uc2e4\ubc84\u300b\ub85c \uc62e\uae30\uba74 \ud3ec\ucf13\uc6cc\ucee4\uc5d0 \ucf54\uc2a4\ub9f5 \"\ubc24\ud558\ub298\uc758 \ub05d\"\uc774 \ucd94\uac00\ub41c\ub2e4. 2009\ub144 12\uc6d4 24\uc77c\ubd80\ud130 2010\ub144 1\uc6d4 31\uc77c\uae4c\uc9c0 \ub86f\ub370\uc2dc\ub124\ub9c8\uc758 \u300a \uc544\ub974\uc138\uc6b0\uc2a4 \ucd08\uadf9\uc758 \uc2dc\uacf5\uc73c\ub85c\u300b \uc0c1\uc601\uad00\uc5d0\uc11c \ubc30\ud3ec \ubc1b\uc740 \ud3ec\ucf13\ubaac \uc544\ub974\uc138\uc6b0\uc2a4\ub97c \uac8c\uc784 \ub0b4\ub85c \ub370\ub824\uac00\uba74 \ub514\uc544\ub8e8\uac00, \ud384\uae30\uc544, \uae30\ub77c\ud2f0\ub098 \uc911 \ud55c \ub9c8\ub9ac\ub97c \uc5bb\uc744 \uc218 \uc788\uc73c\uba70, \uc601\ud654 \uc608\ub9e4\uad8c\uc744 \uc9c0\ucc38\ud55c \uc0ac\ub78c\uc5d0 \ud55c\ud574 \uc9c0\uc815\ub41c \uc0c1\uc601\uad00\uc5d0\uc11c \ubc30\ud3ec\ud558\uc600\ub358 \"\ud53c\uce74\uce04 \uceec\ub7ec\uc758 \ud53c\uce04\"\ub294 \uac8c\uc784 \ub0b4 \ub108\ub3c4\ubc24\ub098\ubb34 \uc232\uc73c\ub85c \ub370\ub824\uac00\uba74 \"\uc090\ucb49\uadc0 \ud53c\uce04\"\ub97c \uc5bb\uc744 \uc218 \uc788\ub2e4. \ubc1c\ub9e4 \uae30\ub150\uc73c\ub85c E\ub9c8\ud2b8\uc5d0\uc11c \uac8c\uc784\uc744 \uad6c\ub9e4\ud558\uba74 \uc120\ucc29\uc21c\uc73c\ub85c \uce60\uc0c9\uc870\uc640 \ub8e8\uae30\uc544 \ud53c\uaddc\uc5b4\ub97c \uc99d\uc815\ud558\uc600\uace0, \ucd08\ud68c\ud310\uc5d0\ub294 \ud3ec\ucf13\ubaac \uce74\ub4dc \uac8c\uc784\uc758 \uce60\uc0c9\uc870\uc640 \ub8e8\uae30\uc544 \ud504\ub85c\ubaa8 \uce74\ub4dc\ub97c \uac01\uac01 \ub3d9\ubd09\ud558\uc600\ub2e4.", "sentence_answer": "2009\ub144 12\uc6d4 24\uc77c\ubd80\ud130 2010\ub144 1\uc6d4 31\uc77c\uae4c\uc9c0 \ub86f\ub370\uc2dc\ub124\ub9c8\uc758 \u300a \uc544\ub974\uc138\uc6b0\uc2a4 \ucd08\uadf9\uc758 \uc2dc\uacf5\uc73c\ub85c\u300b \uc0c1\uc601\uad00\uc5d0\uc11c \ubc30\ud3ec \ubc1b\uc740 \ud3ec\ucf13\ubaac \uc544\ub974\uc138\uc6b0\uc2a4\ub97c \uac8c\uc784 \ub0b4\ub85c \ub370\ub824\uac00\uba74 \ub514\uc544\ub8e8\uac00, \ud384\uae30\uc544, \uae30\ub77c\ud2f0\ub098 \uc911 \ud55c \ub9c8\ub9ac\ub97c \uc5bb\uc744 \uc218 \uc788\uc73c\uba70, \uc601\ud654 \uc608\ub9e4\uad8c\uc744 \uc9c0\ucc38\ud55c \uc0ac\ub78c\uc5d0 \ud55c\ud574 \uc9c0\uc815\ub41c \uc0c1\uc601\uad00\uc5d0\uc11c \ubc30\ud3ec\ud558\uc600\ub358 \"\ud53c\uce74\uce04 \uceec\ub7ec\uc758 \ud53c\uce04\"\ub294 \uac8c\uc784 \ub0b4 \ub108\ub3c4\ubc24\ub098\ubb34 \uc232\uc73c\ub85c \ub370\ub824\uac00\uba74 \"\uc090\ucb49\uadc0 \ud53c\uce04\"\ub97c \uc5bb\uc744 \uc218 \uc788\ub2e4.", "paragraph_id": "6220514-5-2", "paragraph_question": "question: 2009\ub144 12\uc6d4 24\uc77c\ubd80\ud130 2010\ub144 1\uc6d4 31\uc77c \uc0ac\uc774\uc5d0 \ub86f\ub370\uc2dc\ub124\ub9c8\uc758 \uc544\ub974\uc138\uc6b0\uc2a4 \ucd08\uadf9\uc758 \uc2dc\uacf5\uc73c\ub85c \uc0c1\uc601\uad00\uc5d0\uc11c \ubc30\ud3ec\ud55c \ud3ec\ucf13\ubaac\uc758 \uc774\ub984\uc740 \ubb34\uc5c7\uc785\ub2c8\uae4c?, context: \ub300\ud55c\ubbfc\uad6d\uc5d0\uc11c\ub294 2009\ub144 12\uc6d4 10\uc77c, \u300a\ud3ec\ucf13\ubaac\uc2a4\ud130 \ud558\ud2b8\uace8\ub4dc\u00b7\uc18c\uc6b8\uc2e4\ubc84\u300b\ub97c 2010\ub144 2\uc6d4 4\uc77c \uc815\uc2dd \ubc1c\ub9e4\ud55c\ub2e4\uace0 \uacf5\uac1c\ud558\uc600\ub2e4. \ud55c\uad6d\ub2cc\ud150\ub3c4\uc5d0\uc11c\ub294 2010\ub144 1\uc6d4 30\uc77c\uacfc 1\uc6d4 31\uc77c \uc81c7\ucc28 \ud3ec\ucf13\ubaac \ubaa8\uc73c\uae30 \uc774\ubca4\ud2b8\ub97c \uac1c\ucd5c\ud558\uc5ec \u300a\ud3ec\ucf13\ubaac\uc2a4\ud130DP \ub514\uc544\ub8e8\uac00\u00b7\ud384\uae30\uc544\u300b, \u300aPt \uae30\ub77c\ud2f0\ub098\u300b \ud50c\ub808\uc774\uc5b4\uc5d0\uac8c \"\uc774\uc0c1\ud55c \uce74\ub4dc\"\ub97c \ud1b5\ud574 \ud3ec\ucf13\ubaac \uc9c0\ub77c\uce58\ub97c \ubc30\ud3ec\ud558\uc600\ub2e4. \uc774 \uc9c0\ub77c\uce58\ub97c \u300a\ud558\ud2b8\uace8\ub4dc\u00b7\uc18c\uc6b8\uc2e4\ubc84\u300b\ub85c \uc62e\uae30\uba74 \ud3ec\ucf13\uc6cc\ucee4\uc5d0 \ucf54\uc2a4\ub9f5 \"\ubc24\ud558\ub298\uc758 \ub05d\"\uc774 \ucd94\uac00\ub41c\ub2e4. 2009\ub144 12\uc6d4 24\uc77c\ubd80\ud130 2010\ub144 1\uc6d4 31\uc77c\uae4c\uc9c0 \ub86f\ub370\uc2dc\ub124\ub9c8\uc758 \u300a\uc544\ub974\uc138\uc6b0\uc2a4 \ucd08\uadf9\uc758 \uc2dc\uacf5\uc73c\ub85c\u300b \uc0c1\uc601\uad00\uc5d0\uc11c \ubc30\ud3ec \ubc1b\uc740 \ud3ec\ucf13\ubaac \uc544\ub974\uc138\uc6b0\uc2a4\ub97c \uac8c\uc784 \ub0b4\ub85c \ub370\ub824\uac00\uba74 \ub514\uc544\ub8e8\uac00, \ud384\uae30\uc544, \uae30\ub77c\ud2f0\ub098 \uc911 \ud55c \ub9c8\ub9ac\ub97c \uc5bb\uc744 \uc218 \uc788\uc73c\uba70, \uc601\ud654 \uc608\ub9e4\uad8c\uc744 \uc9c0\ucc38\ud55c \uc0ac\ub78c\uc5d0 \ud55c\ud574 \uc9c0\uc815\ub41c \uc0c1\uc601\uad00\uc5d0\uc11c \ubc30\ud3ec\ud558\uc600\ub358 \"\ud53c\uce74\uce04 \uceec\ub7ec\uc758 \ud53c\uce04\"\ub294 \uac8c\uc784 \ub0b4 \ub108\ub3c4\ubc24\ub098\ubb34 \uc232\uc73c\ub85c \ub370\ub824\uac00\uba74 \"\uc090\ucb49\uadc0 \ud53c\uce04\"\ub97c \uc5bb\uc744 \uc218 \uc788\ub2e4. \ubc1c\ub9e4 \uae30\ub150\uc73c\ub85c E\ub9c8\ud2b8\uc5d0\uc11c \uac8c\uc784\uc744 \uad6c\ub9e4\ud558\uba74 \uc120\ucc29\uc21c\uc73c\ub85c \uce60\uc0c9\uc870\uc640 \ub8e8\uae30\uc544 \ud53c\uaddc\uc5b4\ub97c \uc99d\uc815\ud558\uc600\uace0, \ucd08\ud68c\ud310\uc5d0\ub294 \ud3ec\ucf13\ubaac \uce74\ub4dc \uac8c\uc784\uc758 \uce60\uc0c9\uc870\uc640 \ub8e8\uae30\uc544 \ud504\ub85c\ubaa8 \uce74\ub4dc\ub97c \uac01\uac01 \ub3d9\ubd09\ud558\uc600\ub2e4."} {"question": "\ud2f0\ubca0\ub9ac\uc6b0\uc2a4\uac00 \uac8c\ub974\ub9c8\ub2c8\uc544\uc5d0\uc11c \ucca0\uc218\ud558\uae30\ub85c \uacb0\uc815\ud558\uace0 \uc5b4\ub290 \uac15\uc744 \ubc29\uc5b4\uc120\uc73c\ub85c \uc0bc\uc558\ub098?", "paragraph": "\ud30c\ub974\ud2f0\uc544\uac00 \uc5b8\uc81c\ub098 \uc704\ud611\uc801\uc778 \uc0c1\ub300\uc774\uae30\ub294 \ud588\uc9c0\ub9cc, \uc2e4\uc81c \uc804\uc7c1\uc740 \uac8c\ub974\ub9cc \uc871\uc744 \uc0c1\ub300\ub85c \ub300\ubd80\ubd84 \ub77c\uc778 \uac15, \ub3c4\ub098\uc6b0 \uac15 \uadfc\uad50\uc5d0\uc11c \ubc8c\uc5b4\uc84c\ub2e4. \uc548\ud1a0\ub2c8\uc6b0\uc2a4\uc640 \ucd5c\uc885 \uc804\ud22c\ub97c \ubc8c\uc774\uae30 \uc804\uc5d0 \ub2ec\ub9c8\ud2f0\uc544\uc758 \ubd80\uc871\ub4e4\uacfc \ubc8c\uc600\ub358 \uc804\uc7c1 \uc774\ud6c4 \ub85c\ub9c8\uad70\uc740 \ucc29\uc2e4\ud788 \ub3c4\ub098\uc6b0 \uac15\uc744 \ud5a5\ud574 \uc9c4\uaca9\ud558\uae30 \uc2dc\uc791\ud588\ub2e4. \ud558\uc9c0\ub9cc \uc5ec\ub7ec \ucc28\ub840 \uc804\uc7c1\uc5d0\uc11c \uc2b9\ub9ac\ud588\uc74c\uc5d0\ub3c4 \ubd88\uad6c\ud558\uace0 \uac8c\ub974\ub9c8\ub2c8\uc544 \uc9c0\uc5ed\uc740 \ub85c\ub9c8\ud654\ud558\ub294 \ub370 \uc2e4\ud328\ud558\uc600\ub2e4. 9\ub144\uc5d0 \ud1a0\uc774\ud1a0\ubd80\ub974\ud06c \uc232\uc5d0\uc11c \ub2f9\ud55c \ucc38\ud328\uac00 \uc774\ub97c \uc0c1\uc9d5\uc801\uc73c\ub85c \ubcf4\uc5ec\uc8fc\ub294 \uc0ac\uac74\uc774\ub2e4. \uac8c\ub974\ub9c8\ub2c8\uc544\uc758 \ucd1d\ub3c5\uc778 \ubc14\ub8e8\uc2a4\uac00 \uc774\ub044\ub294 3\uac1c \uad70\ub2e8\uc774 \ucf00\ub8e8\uc2a4\ud0a4 \uc871 \ucd9c\uc2e0\uc758 \uc544\ub974\ubbf8\ub2c8\uc6b0\uc2a4\uac00 \uc774\ub044\ub294 \uac8c\ub974\ub9cc \uc871\uc5d0\uac8c \uc804\uba78\ud55c\ub2e4. \uc544\uc6b0\uad6c\uc2a4\ud22c\uc2a4\ub294 \uc0ac\ud0dc\ub97c \uc218\uc2b5\ud558\ub824 \ud588\uace0, \ud2f0\ubca0\ub9ac\uc6b0\uc2a4\ub294 \uc774\ud6c4 \uc5ec\ub7ec \ucc28\ub840 \ub77c\uc778\ub780\ud2b8\ub85c \uc9c4\uaca9\ud558\uc5ec \uc2b9\ub9ac\ub97c \uac70\ub450\uc5c8\ub2e4. \ud558\uc9c0\ub9cc \uc544\uc6b0\uad6c\uc2a4\ud22c\uc2a4\uc758 \uc8fd\uc740 \ud6c4\uc5d0 \uacc4\uc2b9\uc790\uc778 \ud2f0\ubca0\ub9ac\uc6b0\uc2a4\ub294 \uac8c\ub974\ub9c8\ub2c8\uc544\uc5d0\uc11c \ucca0\uc218\ud558\uae30\ub85c \uacb0\uc815\ud558\uc600\uace0, \uc774\ud6c4 \ub85c\ub9c8\uad70\uc740 \ub77c\uc778 \uac15\uacfc \ub3c4\ub098\uc6b0 \uac15\uc744 \uc8fc \ubc29\uc5b4\uc120\uc73c\ub85c \uc0bc\uc544 \ubc29\uc704 \uccb4\uacc4\ub97c \uad6c\ucd95\ud55c\ub2e4.", "answer": "\ub77c\uc778 \uac15\uacfc \ub3c4\ub098\uc6b0 \uac15", "sentence": "\uc774\ud6c4 \ub85c\ub9c8\uad70\uc740 \ub77c\uc778 \uac15\uacfc \ub3c4\ub098\uc6b0 \uac15 \uc744 \uc8fc \ubc29\uc5b4\uc120\uc73c\ub85c \uc0bc\uc544 \ubc29\uc704 \uccb4\uacc4\ub97c \uad6c\ucd95\ud55c\ub2e4.", "paragraph_sentence": "\ud30c\ub974\ud2f0\uc544\uac00 \uc5b8\uc81c\ub098 \uc704\ud611\uc801\uc778 \uc0c1\ub300\uc774\uae30\ub294 \ud588\uc9c0\ub9cc, \uc2e4\uc81c \uc804\uc7c1\uc740 \uac8c\ub974\ub9cc \uc871\uc744 \uc0c1\ub300\ub85c \ub300\ubd80\ubd84 \ub77c\uc778 \uac15, \ub3c4\ub098\uc6b0 \uac15 \uadfc\uad50\uc5d0\uc11c \ubc8c\uc5b4\uc84c\ub2e4. \uc548\ud1a0\ub2c8\uc6b0\uc2a4\uc640 \ucd5c\uc885 \uc804\ud22c\ub97c \ubc8c\uc774\uae30 \uc804\uc5d0 \ub2ec\ub9c8\ud2f0\uc544\uc758 \ubd80\uc871\ub4e4\uacfc \ubc8c\uc600\ub358 \uc804\uc7c1 \uc774\ud6c4 \ub85c\ub9c8\uad70\uc740 \ucc29\uc2e4\ud788 \ub3c4\ub098\uc6b0 \uac15\uc744 \ud5a5\ud574 \uc9c4\uaca9\ud558\uae30 \uc2dc\uc791\ud588\ub2e4. \ud558\uc9c0\ub9cc \uc5ec\ub7ec \ucc28\ub840 \uc804\uc7c1\uc5d0\uc11c \uc2b9\ub9ac\ud588\uc74c\uc5d0\ub3c4 \ubd88\uad6c\ud558\uace0 \uac8c\ub974\ub9c8\ub2c8\uc544 \uc9c0\uc5ed\uc740 \ub85c\ub9c8\ud654\ud558\ub294 \ub370 \uc2e4\ud328\ud558\uc600\ub2e4. 9\ub144\uc5d0 \ud1a0\uc774\ud1a0\ubd80\ub974\ud06c \uc232\uc5d0\uc11c \ub2f9\ud55c \ucc38\ud328\uac00 \uc774\ub97c \uc0c1\uc9d5\uc801\uc73c\ub85c \ubcf4\uc5ec\uc8fc\ub294 \uc0ac\uac74\uc774\ub2e4. \uac8c\ub974\ub9c8\ub2c8\uc544\uc758 \ucd1d\ub3c5\uc778 \ubc14\ub8e8\uc2a4\uac00 \uc774\ub044\ub294 3\uac1c \uad70\ub2e8\uc774 \ucf00\ub8e8\uc2a4\ud0a4 \uc871 \ucd9c\uc2e0\uc758 \uc544\ub974\ubbf8\ub2c8\uc6b0\uc2a4\uac00 \uc774\ub044\ub294 \uac8c\ub974\ub9cc \uc871\uc5d0\uac8c \uc804\uba78\ud55c\ub2e4. \uc544\uc6b0\uad6c\uc2a4\ud22c\uc2a4\ub294 \uc0ac\ud0dc\ub97c \uc218\uc2b5\ud558\ub824 \ud588\uace0, \ud2f0\ubca0\ub9ac\uc6b0\uc2a4\ub294 \uc774\ud6c4 \uc5ec\ub7ec \ucc28\ub840 \ub77c\uc778\ub780\ud2b8\ub85c \uc9c4\uaca9\ud558\uc5ec \uc2b9\ub9ac\ub97c \uac70\ub450\uc5c8\ub2e4. \ud558\uc9c0\ub9cc \uc544\uc6b0\uad6c\uc2a4\ud22c\uc2a4\uc758 \uc8fd\uc740 \ud6c4\uc5d0 \uacc4\uc2b9\uc790\uc778 \ud2f0\ubca0\ub9ac\uc6b0\uc2a4\ub294 \uac8c\ub974\ub9c8\ub2c8\uc544\uc5d0\uc11c \ucca0\uc218\ud558\uae30\ub85c \uacb0\uc815\ud558\uc600\uace0, \uc774\ud6c4 \ub85c\ub9c8\uad70\uc740 \ub77c\uc778 \uac15\uacfc \ub3c4\ub098\uc6b0 \uac15 \uc744 \uc8fc \ubc29\uc5b4\uc120\uc73c\ub85c \uc0bc\uc544 \ubc29\uc704 \uccb4\uacc4\ub97c \uad6c\ucd95\ud55c\ub2e4. ", "paragraph_answer": "\ud30c\ub974\ud2f0\uc544\uac00 \uc5b8\uc81c\ub098 \uc704\ud611\uc801\uc778 \uc0c1\ub300\uc774\uae30\ub294 \ud588\uc9c0\ub9cc, \uc2e4\uc81c \uc804\uc7c1\uc740 \uac8c\ub974\ub9cc \uc871\uc744 \uc0c1\ub300\ub85c \ub300\ubd80\ubd84 \ub77c\uc778 \uac15, \ub3c4\ub098\uc6b0 \uac15 \uadfc\uad50\uc5d0\uc11c \ubc8c\uc5b4\uc84c\ub2e4. \uc548\ud1a0\ub2c8\uc6b0\uc2a4\uc640 \ucd5c\uc885 \uc804\ud22c\ub97c \ubc8c\uc774\uae30 \uc804\uc5d0 \ub2ec\ub9c8\ud2f0\uc544\uc758 \ubd80\uc871\ub4e4\uacfc \ubc8c\uc600\ub358 \uc804\uc7c1 \uc774\ud6c4 \ub85c\ub9c8\uad70\uc740 \ucc29\uc2e4\ud788 \ub3c4\ub098\uc6b0 \uac15\uc744 \ud5a5\ud574 \uc9c4\uaca9\ud558\uae30 \uc2dc\uc791\ud588\ub2e4. \ud558\uc9c0\ub9cc \uc5ec\ub7ec \ucc28\ub840 \uc804\uc7c1\uc5d0\uc11c \uc2b9\ub9ac\ud588\uc74c\uc5d0\ub3c4 \ubd88\uad6c\ud558\uace0 \uac8c\ub974\ub9c8\ub2c8\uc544 \uc9c0\uc5ed\uc740 \ub85c\ub9c8\ud654\ud558\ub294 \ub370 \uc2e4\ud328\ud558\uc600\ub2e4. 9\ub144\uc5d0 \ud1a0\uc774\ud1a0\ubd80\ub974\ud06c \uc232\uc5d0\uc11c \ub2f9\ud55c \ucc38\ud328\uac00 \uc774\ub97c \uc0c1\uc9d5\uc801\uc73c\ub85c \ubcf4\uc5ec\uc8fc\ub294 \uc0ac\uac74\uc774\ub2e4. \uac8c\ub974\ub9c8\ub2c8\uc544\uc758 \ucd1d\ub3c5\uc778 \ubc14\ub8e8\uc2a4\uac00 \uc774\ub044\ub294 3\uac1c \uad70\ub2e8\uc774 \ucf00\ub8e8\uc2a4\ud0a4 \uc871 \ucd9c\uc2e0\uc758 \uc544\ub974\ubbf8\ub2c8\uc6b0\uc2a4\uac00 \uc774\ub044\ub294 \uac8c\ub974\ub9cc \uc871\uc5d0\uac8c \uc804\uba78\ud55c\ub2e4. \uc544\uc6b0\uad6c\uc2a4\ud22c\uc2a4\ub294 \uc0ac\ud0dc\ub97c \uc218\uc2b5\ud558\ub824 \ud588\uace0, \ud2f0\ubca0\ub9ac\uc6b0\uc2a4\ub294 \uc774\ud6c4 \uc5ec\ub7ec \ucc28\ub840 \ub77c\uc778\ub780\ud2b8\ub85c \uc9c4\uaca9\ud558\uc5ec \uc2b9\ub9ac\ub97c \uac70\ub450\uc5c8\ub2e4. \ud558\uc9c0\ub9cc \uc544\uc6b0\uad6c\uc2a4\ud22c\uc2a4\uc758 \uc8fd\uc740 \ud6c4\uc5d0 \uacc4\uc2b9\uc790\uc778 \ud2f0\ubca0\ub9ac\uc6b0\uc2a4\ub294 \uac8c\ub974\ub9c8\ub2c8\uc544\uc5d0\uc11c \ucca0\uc218\ud558\uae30\ub85c \uacb0\uc815\ud558\uc600\uace0, \uc774\ud6c4 \ub85c\ub9c8\uad70\uc740 \ub77c\uc778 \uac15\uacfc \ub3c4\ub098\uc6b0 \uac15 \uc744 \uc8fc \ubc29\uc5b4\uc120\uc73c\ub85c \uc0bc\uc544 \ubc29\uc704 \uccb4\uacc4\ub97c \uad6c\ucd95\ud55c\ub2e4.", "sentence_answer": "\uc774\ud6c4 \ub85c\ub9c8\uad70\uc740 \ub77c\uc778 \uac15\uacfc \ub3c4\ub098\uc6b0 \uac15 \uc744 \uc8fc \ubc29\uc5b4\uc120\uc73c\ub85c \uc0bc\uc544 \ubc29\uc704 \uccb4\uacc4\ub97c \uad6c\ucd95\ud55c\ub2e4.", "paragraph_id": "6585254-28-2", "paragraph_question": "question: \ud2f0\ubca0\ub9ac\uc6b0\uc2a4\uac00 \uac8c\ub974\ub9c8\ub2c8\uc544\uc5d0\uc11c \ucca0\uc218\ud558\uae30\ub85c \uacb0\uc815\ud558\uace0 \uc5b4\ub290 \uac15\uc744 \ubc29\uc5b4\uc120\uc73c\ub85c \uc0bc\uc558\ub098?, context: \ud30c\ub974\ud2f0\uc544\uac00 \uc5b8\uc81c\ub098 \uc704\ud611\uc801\uc778 \uc0c1\ub300\uc774\uae30\ub294 \ud588\uc9c0\ub9cc, \uc2e4\uc81c \uc804\uc7c1\uc740 \uac8c\ub974\ub9cc \uc871\uc744 \uc0c1\ub300\ub85c \ub300\ubd80\ubd84 \ub77c\uc778 \uac15, \ub3c4\ub098\uc6b0 \uac15 \uadfc\uad50\uc5d0\uc11c \ubc8c\uc5b4\uc84c\ub2e4. \uc548\ud1a0\ub2c8\uc6b0\uc2a4\uc640 \ucd5c\uc885 \uc804\ud22c\ub97c \ubc8c\uc774\uae30 \uc804\uc5d0 \ub2ec\ub9c8\ud2f0\uc544\uc758 \ubd80\uc871\ub4e4\uacfc \ubc8c\uc600\ub358 \uc804\uc7c1 \uc774\ud6c4 \ub85c\ub9c8\uad70\uc740 \ucc29\uc2e4\ud788 \ub3c4\ub098\uc6b0 \uac15\uc744 \ud5a5\ud574 \uc9c4\uaca9\ud558\uae30 \uc2dc\uc791\ud588\ub2e4. \ud558\uc9c0\ub9cc \uc5ec\ub7ec \ucc28\ub840 \uc804\uc7c1\uc5d0\uc11c \uc2b9\ub9ac\ud588\uc74c\uc5d0\ub3c4 \ubd88\uad6c\ud558\uace0 \uac8c\ub974\ub9c8\ub2c8\uc544 \uc9c0\uc5ed\uc740 \ub85c\ub9c8\ud654\ud558\ub294 \ub370 \uc2e4\ud328\ud558\uc600\ub2e4. 9\ub144\uc5d0 \ud1a0\uc774\ud1a0\ubd80\ub974\ud06c \uc232\uc5d0\uc11c \ub2f9\ud55c \ucc38\ud328\uac00 \uc774\ub97c \uc0c1\uc9d5\uc801\uc73c\ub85c \ubcf4\uc5ec\uc8fc\ub294 \uc0ac\uac74\uc774\ub2e4. \uac8c\ub974\ub9c8\ub2c8\uc544\uc758 \ucd1d\ub3c5\uc778 \ubc14\ub8e8\uc2a4\uac00 \uc774\ub044\ub294 3\uac1c \uad70\ub2e8\uc774 \ucf00\ub8e8\uc2a4\ud0a4 \uc871 \ucd9c\uc2e0\uc758 \uc544\ub974\ubbf8\ub2c8\uc6b0\uc2a4\uac00 \uc774\ub044\ub294 \uac8c\ub974\ub9cc \uc871\uc5d0\uac8c \uc804\uba78\ud55c\ub2e4. \uc544\uc6b0\uad6c\uc2a4\ud22c\uc2a4\ub294 \uc0ac\ud0dc\ub97c \uc218\uc2b5\ud558\ub824 \ud588\uace0, \ud2f0\ubca0\ub9ac\uc6b0\uc2a4\ub294 \uc774\ud6c4 \uc5ec\ub7ec \ucc28\ub840 \ub77c\uc778\ub780\ud2b8\ub85c \uc9c4\uaca9\ud558\uc5ec \uc2b9\ub9ac\ub97c \uac70\ub450\uc5c8\ub2e4. \ud558\uc9c0\ub9cc \uc544\uc6b0\uad6c\uc2a4\ud22c\uc2a4\uc758 \uc8fd\uc740 \ud6c4\uc5d0 \uacc4\uc2b9\uc790\uc778 \ud2f0\ubca0\ub9ac\uc6b0\uc2a4\ub294 \uac8c\ub974\ub9c8\ub2c8\uc544\uc5d0\uc11c \ucca0\uc218\ud558\uae30\ub85c \uacb0\uc815\ud558\uc600\uace0, \uc774\ud6c4 \ub85c\ub9c8\uad70\uc740 \ub77c\uc778 \uac15\uacfc \ub3c4\ub098\uc6b0 \uac15\uc744 \uc8fc \ubc29\uc5b4\uc120\uc73c\ub85c \uc0bc\uc544 \ubc29\uc704 \uccb4\uacc4\ub97c \uad6c\ucd95\ud55c\ub2e4."} {"question": "\uc62c\ucf55\uc740 \uba87\ub144\ub3c4\uc5d0 \uc77c\ubcf8\uc5d0 \ubcf5\uadc0\ud558\uc600\ub294\uac00?", "paragraph": "\uc57d 2\ub144\uac04\uc758 \ud734\uac00\ub97c \ubcf4\ub0b8 \ud6c4 1864\ub144(\uac90\uc9c0 \uc6d0\ub144) \ubd04\uc5d0 \uc77c\ubcf8\uc5d0 \ubcf5\uadc0\ud588\ub2e4. \uadf8\ub7ec\ub098 \uc77c\ubcf8\uc758 \uc591\uc0c1\uc740 \uae09\ubcc0\ud574 \uc788\uc5c8\ub2e4. \uadc0\uad6d \uc911\uc5d0 \ub098\ub9c8\ubb34\uae30 \uc0ac\uac74\uacfc \uadf8\uc5d0 \ub300\ud55c \ubcf4\ubcf5\uc73c\ub85c \uc0ac\uc4f0\uc5d0\uc774 \uc804\uc7c1\uc774 \ubc8c\uc5b4\uc84c\uace0, \uc870\uc288\ubc88\uc774 \uc678\uad6d \uc120\ubc15\uc744 \ud3ec\uaca9\ud588\uae30 \ub54c\ubb38\uc5d0 \uac04\ubaac \ud574\ud611\uc740 \ud56d\ud589 \ubd88\ub2a5\uc774 \ub418\uc5b4 \uc788\ub294 \ub4f1 \uc77c\ubcf8\uc758 \uc591\uc774\uc801 \uacbd\ud5a5\uc774 \uac15\ud574\uc9c0\uace0 \uc788\uc5c8\ub2e4. \ub9c9\ubd80\ub3c4 \uc591\uc774\ud30c\ub97c \ud68c\uc720\ud558\uae30 \uc704\ud574 \uc720\ub7fd\uc5d0 \uc694\ucf54\ud558\ub9c8 \uc1c4\ud56d \ub2f4\ud310 \uc0ac\uc808\ub2e8\uc744 \ud30c\uacac\ud588\ub2e4. \uc62c\ucf55\uc740 \uc774\ub97c \ud0c0\ud30c\ud558\uace0\uc790, \uc2dc\ucf54\ucfe0 \ud568\ub300\ub97c \ub3d9\uc6d0\ud55c \uc2dc\ubaa8\ub178\uc138\ud0a4 \ud3ec\uaca9 \uc0ac\uac74\uc5d0\uc11c \uc8fc\ub3c4\uc801 \uc5ed\ud560\uc744 \ud558\uc600\uace0, \uc774\uac83\ub97c \uc778\uc815\ud558\uc9c0 \uc54a\uc558\ub358 \uc678\ubb34\ubd80 \uc7a5\uad00 \uc874 \ub7ec\uc140\uc5d0 \uc758\ud574 \uc18c\ud658\ub418\uc5c8\ub2e4. \uc8fc\uc77c \uacf5\uc0ac\ub294 \uc77c\ucc0d\uc774 \uccad\ub098\ub77c\uc5d0\uc11c \uc790\uc2e0\uc758 \uc544\ub7ab \uc0ac\ub78c\uc774\uc5c8\ub358 \ud574\ub9ac \ud30c\ud06c\uc2a4(\uc7ac\uc784, 1865 - 1883\ub144)\uc5d0\uac8c \ubb3c\ub824\uc8fc\uc5c8\ub2e4.", "answer": "1864\ub144", "sentence": "\uc57d 2\ub144\uac04\uc758 \ud734\uac00\ub97c \ubcf4\ub0b8 \ud6c4 1864\ub144 (\uac90\uc9c0 \uc6d0\ub144) \ubd04\uc5d0 \uc77c\ubcf8\uc5d0 \ubcf5\uadc0\ud588\ub2e4.", "paragraph_sentence": " \uc57d 2\ub144\uac04\uc758 \ud734\uac00\ub97c \ubcf4\ub0b8 \ud6c4 1864\ub144 (\uac90\uc9c0 \uc6d0\ub144) \ubd04\uc5d0 \uc77c\ubcf8\uc5d0 \ubcf5\uadc0\ud588\ub2e4. \uadf8\ub7ec\ub098 \uc77c\ubcf8\uc758 \uc591\uc0c1\uc740 \uae09\ubcc0\ud574 \uc788\uc5c8\ub2e4. \uadc0\uad6d \uc911\uc5d0 \ub098\ub9c8\ubb34\uae30 \uc0ac\uac74\uacfc \uadf8\uc5d0 \ub300\ud55c \ubcf4\ubcf5\uc73c\ub85c \uc0ac\uc4f0\uc5d0\uc774 \uc804\uc7c1\uc774 \ubc8c\uc5b4\uc84c\uace0, \uc870\uc288\ubc88\uc774 \uc678\uad6d \uc120\ubc15\uc744 \ud3ec\uaca9\ud588\uae30 \ub54c\ubb38\uc5d0 \uac04\ubaac \ud574\ud611\uc740 \ud56d\ud589 \ubd88\ub2a5\uc774 \ub418\uc5b4 \uc788\ub294 \ub4f1 \uc77c\ubcf8\uc758 \uc591\uc774\uc801 \uacbd\ud5a5\uc774 \uac15\ud574\uc9c0\uace0 \uc788\uc5c8\ub2e4. \ub9c9\ubd80\ub3c4 \uc591\uc774\ud30c\ub97c \ud68c\uc720\ud558\uae30 \uc704\ud574 \uc720\ub7fd\uc5d0 \uc694\ucf54\ud558\ub9c8 \uc1c4\ud56d \ub2f4\ud310 \uc0ac\uc808\ub2e8\uc744 \ud30c\uacac\ud588\ub2e4. \uc62c\ucf55\uc740 \uc774\ub97c \ud0c0\ud30c\ud558\uace0\uc790, \uc2dc\ucf54\ucfe0 \ud568\ub300\ub97c \ub3d9\uc6d0\ud55c \uc2dc\ubaa8\ub178\uc138\ud0a4 \ud3ec\uaca9 \uc0ac\uac74\uc5d0\uc11c \uc8fc\ub3c4\uc801 \uc5ed\ud560\uc744 \ud558\uc600\uace0, \uc774\uac83\ub97c \uc778\uc815\ud558\uc9c0 \uc54a\uc558\ub358 \uc678\ubb34\ubd80 \uc7a5\uad00 \uc874 \ub7ec\uc140\uc5d0 \uc758\ud574 \uc18c\ud658\ub418\uc5c8\ub2e4. \uc8fc\uc77c \uacf5\uc0ac\ub294 \uc77c\ucc0d\uc774 \uccad\ub098\ub77c\uc5d0\uc11c \uc790\uc2e0\uc758 \uc544\ub7ab \uc0ac\ub78c\uc774\uc5c8\ub358 \ud574\ub9ac \ud30c\ud06c\uc2a4(\uc7ac\uc784, 1865 - 1883\ub144)\uc5d0\uac8c \ubb3c\ub824\uc8fc\uc5c8\ub2e4.", "paragraph_answer": "\uc57d 2\ub144\uac04\uc758 \ud734\uac00\ub97c \ubcf4\ub0b8 \ud6c4 1864\ub144 (\uac90\uc9c0 \uc6d0\ub144) \ubd04\uc5d0 \uc77c\ubcf8\uc5d0 \ubcf5\uadc0\ud588\ub2e4. \uadf8\ub7ec\ub098 \uc77c\ubcf8\uc758 \uc591\uc0c1\uc740 \uae09\ubcc0\ud574 \uc788\uc5c8\ub2e4. \uadc0\uad6d \uc911\uc5d0 \ub098\ub9c8\ubb34\uae30 \uc0ac\uac74\uacfc \uadf8\uc5d0 \ub300\ud55c \ubcf4\ubcf5\uc73c\ub85c \uc0ac\uc4f0\uc5d0\uc774 \uc804\uc7c1\uc774 \ubc8c\uc5b4\uc84c\uace0, \uc870\uc288\ubc88\uc774 \uc678\uad6d \uc120\ubc15\uc744 \ud3ec\uaca9\ud588\uae30 \ub54c\ubb38\uc5d0 \uac04\ubaac \ud574\ud611\uc740 \ud56d\ud589 \ubd88\ub2a5\uc774 \ub418\uc5b4 \uc788\ub294 \ub4f1 \uc77c\ubcf8\uc758 \uc591\uc774\uc801 \uacbd\ud5a5\uc774 \uac15\ud574\uc9c0\uace0 \uc788\uc5c8\ub2e4. \ub9c9\ubd80\ub3c4 \uc591\uc774\ud30c\ub97c \ud68c\uc720\ud558\uae30 \uc704\ud574 \uc720\ub7fd\uc5d0 \uc694\ucf54\ud558\ub9c8 \uc1c4\ud56d \ub2f4\ud310 \uc0ac\uc808\ub2e8\uc744 \ud30c\uacac\ud588\ub2e4. \uc62c\ucf55\uc740 \uc774\ub97c \ud0c0\ud30c\ud558\uace0\uc790, \uc2dc\ucf54\ucfe0 \ud568\ub300\ub97c \ub3d9\uc6d0\ud55c \uc2dc\ubaa8\ub178\uc138\ud0a4 \ud3ec\uaca9 \uc0ac\uac74\uc5d0\uc11c \uc8fc\ub3c4\uc801 \uc5ed\ud560\uc744 \ud558\uc600\uace0, \uc774\uac83\ub97c \uc778\uc815\ud558\uc9c0 \uc54a\uc558\ub358 \uc678\ubb34\ubd80 \uc7a5\uad00 \uc874 \ub7ec\uc140\uc5d0 \uc758\ud574 \uc18c\ud658\ub418\uc5c8\ub2e4. \uc8fc\uc77c \uacf5\uc0ac\ub294 \uc77c\ucc0d\uc774 \uccad\ub098\ub77c\uc5d0\uc11c \uc790\uc2e0\uc758 \uc544\ub7ab \uc0ac\ub78c\uc774\uc5c8\ub358 \ud574\ub9ac \ud30c\ud06c\uc2a4(\uc7ac\uc784, 1865 - 1883\ub144)\uc5d0\uac8c \ubb3c\ub824\uc8fc\uc5c8\ub2e4.", "sentence_answer": "\uc57d 2\ub144\uac04\uc758 \ud734\uac00\ub97c \ubcf4\ub0b8 \ud6c4 1864\ub144 (\uac90\uc9c0 \uc6d0\ub144) \ubd04\uc5d0 \uc77c\ubcf8\uc5d0 \ubcf5\uadc0\ud588\ub2e4.", "paragraph_id": "6579281-3-1", "paragraph_question": "question: \uc62c\ucf55\uc740 \uba87\ub144\ub3c4\uc5d0 \uc77c\ubcf8\uc5d0 \ubcf5\uadc0\ud558\uc600\ub294\uac00?, context: \uc57d 2\ub144\uac04\uc758 \ud734\uac00\ub97c \ubcf4\ub0b8 \ud6c4 1864\ub144(\uac90\uc9c0 \uc6d0\ub144) \ubd04\uc5d0 \uc77c\ubcf8\uc5d0 \ubcf5\uadc0\ud588\ub2e4. \uadf8\ub7ec\ub098 \uc77c\ubcf8\uc758 \uc591\uc0c1\uc740 \uae09\ubcc0\ud574 \uc788\uc5c8\ub2e4. \uadc0\uad6d \uc911\uc5d0 \ub098\ub9c8\ubb34\uae30 \uc0ac\uac74\uacfc \uadf8\uc5d0 \ub300\ud55c \ubcf4\ubcf5\uc73c\ub85c \uc0ac\uc4f0\uc5d0\uc774 \uc804\uc7c1\uc774 \ubc8c\uc5b4\uc84c\uace0, \uc870\uc288\ubc88\uc774 \uc678\uad6d \uc120\ubc15\uc744 \ud3ec\uaca9\ud588\uae30 \ub54c\ubb38\uc5d0 \uac04\ubaac \ud574\ud611\uc740 \ud56d\ud589 \ubd88\ub2a5\uc774 \ub418\uc5b4 \uc788\ub294 \ub4f1 \uc77c\ubcf8\uc758 \uc591\uc774\uc801 \uacbd\ud5a5\uc774 \uac15\ud574\uc9c0\uace0 \uc788\uc5c8\ub2e4. \ub9c9\ubd80\ub3c4 \uc591\uc774\ud30c\ub97c \ud68c\uc720\ud558\uae30 \uc704\ud574 \uc720\ub7fd\uc5d0 \uc694\ucf54\ud558\ub9c8 \uc1c4\ud56d \ub2f4\ud310 \uc0ac\uc808\ub2e8\uc744 \ud30c\uacac\ud588\ub2e4. \uc62c\ucf55\uc740 \uc774\ub97c \ud0c0\ud30c\ud558\uace0\uc790, \uc2dc\ucf54\ucfe0 \ud568\ub300\ub97c \ub3d9\uc6d0\ud55c \uc2dc\ubaa8\ub178\uc138\ud0a4 \ud3ec\uaca9 \uc0ac\uac74\uc5d0\uc11c \uc8fc\ub3c4\uc801 \uc5ed\ud560\uc744 \ud558\uc600\uace0, \uc774\uac83\ub97c \uc778\uc815\ud558\uc9c0 \uc54a\uc558\ub358 \uc678\ubb34\ubd80 \uc7a5\uad00 \uc874 \ub7ec\uc140\uc5d0 \uc758\ud574 \uc18c\ud658\ub418\uc5c8\ub2e4. \uc8fc\uc77c \uacf5\uc0ac\ub294 \uc77c\ucc0d\uc774 \uccad\ub098\ub77c\uc5d0\uc11c \uc790\uc2e0\uc758 \uc544\ub7ab \uc0ac\ub78c\uc774\uc5c8\ub358 \ud574\ub9ac \ud30c\ud06c\uc2a4(\uc7ac\uc784, 1865 - 1883\ub144)\uc5d0\uac8c \ubb3c\ub824\uc8fc\uc5c8\ub2e4."} {"question": "2006\ub144 \uc5f0\uad6c\uc5d0\uc11c \ub3d9\uc131\uc560\uc758 \uac10\uc815\uc744 \ub290\ub080\uc801\uc774 \uc788\ub2e4\uace0 \uc751\ub2f5\ud55c \uc775\uba85\uc758 \uc751\ub2f5\uc790\ub4e4\uc758 \ube44\uc728\uc740?", "paragraph": "\uc8fc\uc694\ud55c \uc5f0\uad6c\ub4e4\uc5d0 \ub530\ub974\uba74, 2~11%\uc758 \uc0ac\ub78c\uc740 \uc0b4\uba74\uc11c \ub3d9\uc131\uacfc \uc5b4\ub5a4 \ud615\ud0dc\ub85c\ub4e0\uc9c0 \uc131\uc801 \uc811\ucd09\uc744 \ud574\ubcf8 \uc801\uc774 \uc788\ub2e4. \ub3d9\uc131\uc5d0\uac8c \ub9e4\ub825\uc744 \ub290\ub07c\uac70\ub098 \uc131\uc801\uc778 \ud589\ub3d9\uc744 \ud574\ubd24\ub294\uc9c0 \ubb3c\uc5b4\ubd24\uc744 \ub54c\ub294 \uc218\uce58\uac00 16~21%\ub85c \ub192\uc544\uc84c\ub2e4. 2006\ub144\uc758 \uc5f0\uad6c\uc5d0\uc11c, 20%\uc758 \uc775\uba85\uc758 \uc751\ub2f5\uc790\ub4e4\uc740 \ub3d9\uc131\uc560\uc758 \ub290\ub08c\uc744 \ubc1b\uc740 \uc801\uc774 \uc788\ub2e4\uace0 \uc751\ub2f5\ud588\ub2e4. \uc2a4\uc2a4\ub85c\ub97c \ub3d9\uc131\uc560\uc790\ub85c \uc778\uc2dd\ud55c \uacbd\uc6b0\ub294 2~3%\uc774\ub2e4. 1992\ub144\uc758 \uc5f0\uad6c\uc5d0\uc11c, 6.1%\uc758 \uc601\uad6d \ub0a8\uc131\uacfc 4.1%\uc758 \ud504\ub791\uc2a4 \ub0a8\uc131\uc774 \ub3d9\uc131\uc560 \uacbd\ud5d8\uc774 \uc788\ub2e4\uace0 \uc751\ub2f5\ud588\ub2e4. 2008\ub144\uc758 \uc124\ubb38\uc870\uc0ac\uc5d0\uc11c, 13%\uc758 \uc601\uad6d\uc778\uc774 \uc5b4\ub5a4 \ud615\ud0dc\uc758 \ub3d9\uc131 \uac04 \uc131 \uc811\ucd09\uc744 \uacbd\ud5d8\ud588\ub2e4\uace0 \uc751\ub2f5\ud588\uc73c\ub098 \ub3d9\uc131\uc560\uc790\ub098 \uc591\uc131\uc560\uc790\ub85c \uc815\uccb4\ud654\ud55c \uacbd\uc6b0\ub294 6%\uc774\ub2e4. 2010\ub144 \uc601\uad6d \ud1b5\uacc4\uccad\uc758 \uc124\ubb38 \uc870\uc0ac\uc5d0 \ub530\ub974\uba74, 1.5%\uc758 \uc601\uad6d\uc778\uc774 \ub3d9\uc131\uc560\uc790\ub098 \uc591\uc131\uc560\uc790\ub85c \uc815\uccb4\ud654\ud588\ub2e4\uace0 \uc751\ub2f5\ud588\ub2e4. \uc601\uad6d \ud1b5\uacc4\uccad\uc740 \"\ub2e4\ub978 \uc124\ubb38 \uc870\uc0ac\ub4e4\uc774 0.3~3%\uc758 \ube44\uc728\uc744 \ubcf4\uc5ec\uc8fc\ub294 \uac83\uacfc \ube44\uc2b7\ud558\ub2e4\"\uace0 \uc2dc\uc0ac\ud588\ub2e4.", "answer": "20%", "sentence": "2006\ub144\uc758 \uc5f0\uad6c\uc5d0\uc11c, 20% \uc758 \uc775\uba85\uc758 \uc751\ub2f5\uc790\ub4e4\uc740 \ub3d9\uc131\uc560\uc758 \ub290\ub08c\uc744 \ubc1b\uc740 \uc801\uc774 \uc788\ub2e4\uace0 \uc751\ub2f5\ud588\ub2e4.", "paragraph_sentence": "\uc8fc\uc694\ud55c \uc5f0\uad6c\ub4e4\uc5d0 \ub530\ub974\uba74, 2~11%\uc758 \uc0ac\ub78c\uc740 \uc0b4\uba74\uc11c \ub3d9\uc131\uacfc \uc5b4\ub5a4 \ud615\ud0dc\ub85c\ub4e0\uc9c0 \uc131\uc801 \uc811\ucd09\uc744 \ud574\ubcf8 \uc801\uc774 \uc788\ub2e4. \ub3d9\uc131\uc5d0\uac8c \ub9e4\ub825\uc744 \ub290\ub07c\uac70\ub098 \uc131\uc801\uc778 \ud589\ub3d9\uc744 \ud574\ubd24\ub294\uc9c0 \ubb3c\uc5b4\ubd24\uc744 \ub54c\ub294 \uc218\uce58\uac00 16~21%\ub85c \ub192\uc544\uc84c\ub2e4. 2006\ub144\uc758 \uc5f0\uad6c\uc5d0\uc11c, 20% \uc758 \uc775\uba85\uc758 \uc751\ub2f5\uc790\ub4e4\uc740 \ub3d9\uc131\uc560\uc758 \ub290\ub08c\uc744 \ubc1b\uc740 \uc801\uc774 \uc788\ub2e4\uace0 \uc751\ub2f5\ud588\ub2e4. \uc2a4\uc2a4\ub85c\ub97c \ub3d9\uc131\uc560\uc790\ub85c \uc778\uc2dd\ud55c \uacbd\uc6b0\ub294 2~3%\uc774\ub2e4. 1992\ub144\uc758 \uc5f0\uad6c\uc5d0\uc11c, 6.1%\uc758 \uc601\uad6d \ub0a8\uc131\uacfc 4.1%\uc758 \ud504\ub791\uc2a4 \ub0a8\uc131\uc774 \ub3d9\uc131\uc560 \uacbd\ud5d8\uc774 \uc788\ub2e4\uace0 \uc751\ub2f5\ud588\ub2e4. 2008\ub144\uc758 \uc124\ubb38\uc870\uc0ac\uc5d0\uc11c, 13%\uc758 \uc601\uad6d\uc778\uc774 \uc5b4\ub5a4 \ud615\ud0dc\uc758 \ub3d9\uc131 \uac04 \uc131 \uc811\ucd09\uc744 \uacbd\ud5d8\ud588\ub2e4\uace0 \uc751\ub2f5\ud588\uc73c\ub098 \ub3d9\uc131\uc560\uc790\ub098 \uc591\uc131\uc560\uc790\ub85c \uc815\uccb4\ud654\ud55c \uacbd\uc6b0\ub294 6%\uc774\ub2e4. 2010\ub144 \uc601\uad6d \ud1b5\uacc4\uccad\uc758 \uc124\ubb38 \uc870\uc0ac\uc5d0 \ub530\ub974\uba74, 1.5%\uc758 \uc601\uad6d\uc778\uc774 \ub3d9\uc131\uc560\uc790\ub098 \uc591\uc131\uc560\uc790\ub85c \uc815\uccb4\ud654\ud588\ub2e4\uace0 \uc751\ub2f5\ud588\ub2e4. \uc601\uad6d \ud1b5\uacc4\uccad\uc740 \"\ub2e4\ub978 \uc124\ubb38 \uc870\uc0ac\ub4e4\uc774 0.3~3%\uc758 \ube44\uc728\uc744 \ubcf4\uc5ec\uc8fc\ub294 \uac83\uacfc \ube44\uc2b7\ud558\ub2e4\"\uace0 \uc2dc\uc0ac\ud588\ub2e4.", "paragraph_answer": "\uc8fc\uc694\ud55c \uc5f0\uad6c\ub4e4\uc5d0 \ub530\ub974\uba74, 2~11%\uc758 \uc0ac\ub78c\uc740 \uc0b4\uba74\uc11c \ub3d9\uc131\uacfc \uc5b4\ub5a4 \ud615\ud0dc\ub85c\ub4e0\uc9c0 \uc131\uc801 \uc811\ucd09\uc744 \ud574\ubcf8 \uc801\uc774 \uc788\ub2e4. \ub3d9\uc131\uc5d0\uac8c \ub9e4\ub825\uc744 \ub290\ub07c\uac70\ub098 \uc131\uc801\uc778 \ud589\ub3d9\uc744 \ud574\ubd24\ub294\uc9c0 \ubb3c\uc5b4\ubd24\uc744 \ub54c\ub294 \uc218\uce58\uac00 16~21%\ub85c \ub192\uc544\uc84c\ub2e4. 2006\ub144\uc758 \uc5f0\uad6c\uc5d0\uc11c, 20% \uc758 \uc775\uba85\uc758 \uc751\ub2f5\uc790\ub4e4\uc740 \ub3d9\uc131\uc560\uc758 \ub290\ub08c\uc744 \ubc1b\uc740 \uc801\uc774 \uc788\ub2e4\uace0 \uc751\ub2f5\ud588\ub2e4. \uc2a4\uc2a4\ub85c\ub97c \ub3d9\uc131\uc560\uc790\ub85c \uc778\uc2dd\ud55c \uacbd\uc6b0\ub294 2~3%\uc774\ub2e4. 1992\ub144\uc758 \uc5f0\uad6c\uc5d0\uc11c, 6.1%\uc758 \uc601\uad6d \ub0a8\uc131\uacfc 4.1%\uc758 \ud504\ub791\uc2a4 \ub0a8\uc131\uc774 \ub3d9\uc131\uc560 \uacbd\ud5d8\uc774 \uc788\ub2e4\uace0 \uc751\ub2f5\ud588\ub2e4. 2008\ub144\uc758 \uc124\ubb38\uc870\uc0ac\uc5d0\uc11c, 13%\uc758 \uc601\uad6d\uc778\uc774 \uc5b4\ub5a4 \ud615\ud0dc\uc758 \ub3d9\uc131 \uac04 \uc131 \uc811\ucd09\uc744 \uacbd\ud5d8\ud588\ub2e4\uace0 \uc751\ub2f5\ud588\uc73c\ub098 \ub3d9\uc131\uc560\uc790\ub098 \uc591\uc131\uc560\uc790\ub85c \uc815\uccb4\ud654\ud55c \uacbd\uc6b0\ub294 6%\uc774\ub2e4. 2010\ub144 \uc601\uad6d \ud1b5\uacc4\uccad\uc758 \uc124\ubb38 \uc870\uc0ac\uc5d0 \ub530\ub974\uba74, 1.5%\uc758 \uc601\uad6d\uc778\uc774 \ub3d9\uc131\uc560\uc790\ub098 \uc591\uc131\uc560\uc790\ub85c \uc815\uccb4\ud654\ud588\ub2e4\uace0 \uc751\ub2f5\ud588\ub2e4. \uc601\uad6d \ud1b5\uacc4\uccad\uc740 \"\ub2e4\ub978 \uc124\ubb38 \uc870\uc0ac\ub4e4\uc774 0.3~3%\uc758 \ube44\uc728\uc744 \ubcf4\uc5ec\uc8fc\ub294 \uac83\uacfc \ube44\uc2b7\ud558\ub2e4\"\uace0 \uc2dc\uc0ac\ud588\ub2e4.", "sentence_answer": "2006\ub144\uc758 \uc5f0\uad6c\uc5d0\uc11c, 20% \uc758 \uc775\uba85\uc758 \uc751\ub2f5\uc790\ub4e4\uc740 \ub3d9\uc131\uc560\uc758 \ub290\ub08c\uc744 \ubc1b\uc740 \uc801\uc774 \uc788\ub2e4\uace0 \uc751\ub2f5\ud588\ub2e4.", "paragraph_id": "6299767-8-0", "paragraph_question": "question: 2006\ub144 \uc5f0\uad6c\uc5d0\uc11c \ub3d9\uc131\uc560\uc758 \uac10\uc815\uc744 \ub290\ub080\uc801\uc774 \uc788\ub2e4\uace0 \uc751\ub2f5\ud55c \uc775\uba85\uc758 \uc751\ub2f5\uc790\ub4e4\uc758 \ube44\uc728\uc740?, context: \uc8fc\uc694\ud55c \uc5f0\uad6c\ub4e4\uc5d0 \ub530\ub974\uba74, 2~11%\uc758 \uc0ac\ub78c\uc740 \uc0b4\uba74\uc11c \ub3d9\uc131\uacfc \uc5b4\ub5a4 \ud615\ud0dc\ub85c\ub4e0\uc9c0 \uc131\uc801 \uc811\ucd09\uc744 \ud574\ubcf8 \uc801\uc774 \uc788\ub2e4. \ub3d9\uc131\uc5d0\uac8c \ub9e4\ub825\uc744 \ub290\ub07c\uac70\ub098 \uc131\uc801\uc778 \ud589\ub3d9\uc744 \ud574\ubd24\ub294\uc9c0 \ubb3c\uc5b4\ubd24\uc744 \ub54c\ub294 \uc218\uce58\uac00 16~21%\ub85c \ub192\uc544\uc84c\ub2e4. 2006\ub144\uc758 \uc5f0\uad6c\uc5d0\uc11c, 20%\uc758 \uc775\uba85\uc758 \uc751\ub2f5\uc790\ub4e4\uc740 \ub3d9\uc131\uc560\uc758 \ub290\ub08c\uc744 \ubc1b\uc740 \uc801\uc774 \uc788\ub2e4\uace0 \uc751\ub2f5\ud588\ub2e4. \uc2a4\uc2a4\ub85c\ub97c \ub3d9\uc131\uc560\uc790\ub85c \uc778\uc2dd\ud55c \uacbd\uc6b0\ub294 2~3%\uc774\ub2e4. 1992\ub144\uc758 \uc5f0\uad6c\uc5d0\uc11c, 6.1%\uc758 \uc601\uad6d \ub0a8\uc131\uacfc 4.1%\uc758 \ud504\ub791\uc2a4 \ub0a8\uc131\uc774 \ub3d9\uc131\uc560 \uacbd\ud5d8\uc774 \uc788\ub2e4\uace0 \uc751\ub2f5\ud588\ub2e4. 2008\ub144\uc758 \uc124\ubb38\uc870\uc0ac\uc5d0\uc11c, 13%\uc758 \uc601\uad6d\uc778\uc774 \uc5b4\ub5a4 \ud615\ud0dc\uc758 \ub3d9\uc131 \uac04 \uc131 \uc811\ucd09\uc744 \uacbd\ud5d8\ud588\ub2e4\uace0 \uc751\ub2f5\ud588\uc73c\ub098 \ub3d9\uc131\uc560\uc790\ub098 \uc591\uc131\uc560\uc790\ub85c \uc815\uccb4\ud654\ud55c \uacbd\uc6b0\ub294 6%\uc774\ub2e4. 2010\ub144 \uc601\uad6d \ud1b5\uacc4\uccad\uc758 \uc124\ubb38 \uc870\uc0ac\uc5d0 \ub530\ub974\uba74, 1.5%\uc758 \uc601\uad6d\uc778\uc774 \ub3d9\uc131\uc560\uc790\ub098 \uc591\uc131\uc560\uc790\ub85c \uc815\uccb4\ud654\ud588\ub2e4\uace0 \uc751\ub2f5\ud588\ub2e4. \uc601\uad6d \ud1b5\uacc4\uccad\uc740 \"\ub2e4\ub978 \uc124\ubb38 \uc870\uc0ac\ub4e4\uc774 0.3~3%\uc758 \ube44\uc728\uc744 \ubcf4\uc5ec\uc8fc\ub294 \uac83\uacfc \ube44\uc2b7\ud558\ub2e4\"\uace0 \uc2dc\uc0ac\ud588\ub2e4."} {"question": "\uad6c\ubbf8\uad8c\uc758 \uc0ac\uccb4\ubd80\ud65c \uc2e0\uc559\uc740 \ubb34\uc2a8 \uc885\uad50\uc5d0 \uc758\ud55c \uac83\uc778\uac00?", "paragraph": "\uc77c\ubcf8\uc5d0\uc11c\ub294 \uad6c\ubbf8\uad8c\uc758 \ud06c\ub9ac\uc2a4\ud2b8\uad50\uc5d0 \uc758\ud55c \uc0ac\uccb4\uc758 \ubd80\ud65c\uc2e0\uc559\uc774\ub098 \uadf8\uc5d0 \ub530\ub77c \uc874\uc7ac\ud55c \ud654\uc7a5\uc758 \uae08\uae30\u00b7\uc800\ud56d\uac10\uacfc \uac19\uc740 \uac1c\ub150\uc740 \ubd80\uc871\ud55c \uacbd\ud5a5\uc774 \uc788\ub2e4. \ub610 \uc5d0\ub3c4\uae30\uc5d0\ub294 \ub9c8\ucc28\uac00 \uc874\uc7ac\ud558\uc9c0 \uc54a\uc558\uace0, \ub9cc\uc57d \ub9cc\uc77c \uc5ec\ud589\uc9c0\ub098 \uba3c \ubd09\uc0ac\ud560 \uacf3\uc5d0\uc11c \uae09\uc0ac\uc790\uac00 \ub098\uc624\uace0, \uadf8 \uc0ac\uccb4\ub97c \uc6d0\uaca9\uc9c0\uc5d0 \ubc18\uc1a1\ud55c\ub2e4\uba74 \uc2e4\uc9c8\uc801\uc73c\ub85c\ub294 \ub098\uac00\ubaa8\uce58 \ub4f1\uc744 \uc774\uc6a9\ud574 \uc778\ub825\uc5d0 \uc758\uc9c0\ud558\uc9c0 \uc54a\uc558\uace0, \uc77c\ubc18 \uc11c\ubbfc\uc758 \ub808\ubca8\uc5d0\uc11c\ub294 \uc0ac\uccb4\ub97c \uadf8\ub300\ub85c\uc758 \ubaa8\uc2b5\uc73c\ub85c \uc7a5\uac70\ub9ac \uc218\uc1a1\ud55c\ub2e4\ub294 \uc0dd\uac01\ub3c4 \uc120\ud0dd\uc0ac\ud56d\ub3c4 \uc874\uc7ac\ud558\uc9c0 \uc54a\uc558\ub2e4. \uc774 \uc0dd\uac01\uc740 \uad6c\ubbf8\uc778\uc5d0 \uc758\ud574\uc11c \ub9c8\ucc28\uc640 \uacac\uc778\uc6a9\uc758 \uc911\uc885\ub9c8\uac00 \ubc18\uc785\ub41c \uc5d0\ub3c4\ub9c9\ubd80 \ub9d0\uae30\ubd80\ud130 \uba54\uc774\uc9c0\uae30, \uadf8\ub9ac\uace0 \ub3d9\ub825 \uadfc\ub300\ud654\uac00 \uc9c4\ud589\ub41c \uba54\uc774\uc9c0 \ud6c4\uae30 \uc774\ud6c4\ub3c4 \ubcf8\uc9c8\uc801\uc73c\ub85c\ub294 \ub108\ubb34 \ubcc0\ud568\uc5c6\uc774, \uc804\uc2dc\uc911\ub3c4 \uc804\uc0ac\uc790\ub294 \ud604\uc9c0\uc5d0\uc11c \ud654\uc7a5\ub418\uc5b4 \uc804\ud6c4\ub3c4 \ub610 \uc624\ub7ab\ub3d9\uc548, \ub2e4\uc218\uc758 \uc0ac\ub9dd\uc790\uac00 \ubc1c\uc0dd\ud55c \uc7ac\ud574\ub098 \uc0ac\uace0\uc5d0\uc11c\ub294 \ud604\uc9c0\uc5d0\uc11c \ud654\uc7a5\ud5c8\uac00\ub97c \uc5bb\uc5b4 \uc11c\uc11c\ud788 \uc0ac\uccb4\ub97c \ud654\uc7a5\ud568\uc5d0 \uad50\ubd80\ud558\uace0 \uc720\uace8\uc744 \uac00\uc9c0\uace0 \ub3cc\uc544\uac04\ub2e4\ub294 \ud615\ud0dc\uac00 \uc77c\ubc18\uc801\uc774\uc5c8\ub2e4. \uc624\ub7ab\ub3d9\uc548 \ub9e4\uc7a5\uc2b5\uad00\uc774 \ub0a8\uc544 \uc788\ub358 \uc9c0\uc5ed\ub3c4 \ub9ce\uc9c0\ub9cc, \uc774\uac83\ub4e4\uc5d0\uc11c\ub3c4 \ud654\uc7a5\ub3c4 \uc644\uc804\ud788\ub294 \ubd80\uc815\ub418\uc9c0 \uc54a\uace0, \ud654\uc7a5\uc758 \uae30\uc220\uc758 \uc9c4\ubcf4\ub098 \uc2dc\uc124\uc758 \ub3c4\uc785\uc5d0 \uc758\ud574\uc11c \uadfc\ud604\ub300\uc5d0 \uae09\uc18d\ud788 \ub9e4\uc7a5\uc774 \uc1e0\ud1f4\ud588\ub2e4. \uadf8 \uac19\uc740 \uc77c\ub85c\ubd80\ud130, \uc77c\ubcf8\uc5d0\uc11c\ub294 \uad6c\ubbf8\uad8c\uacfc \uac19\uc740 \uc720\uccb4\ubcf4\uc874\uae30\uc220\uc758 \uc2b5\uad00\uc774 \ud37c\uc9c0\uc9c0 \uc54a\uc558\ub2e4.", "answer": "\ud06c\ub9ac\uc2a4\ud2b8\uad50", "sentence": "\uc77c\ubcf8\uc5d0\uc11c\ub294 \uad6c\ubbf8\uad8c\uc758 \ud06c\ub9ac\uc2a4\ud2b8\uad50 \uc5d0 \uc758\ud55c \uc0ac\uccb4\uc758 \ubd80\ud65c\uc2e0\uc559\uc774\ub098 \uadf8\uc5d0 \ub530\ub77c \uc874\uc7ac\ud55c \ud654\uc7a5\uc758 \uae08\uae30\u00b7\uc800\ud56d\uac10\uacfc \uac19\uc740 \uac1c\ub150\uc740 \ubd80\uc871\ud55c \uacbd\ud5a5\uc774 \uc788\ub2e4.", "paragraph_sentence": " \uc77c\ubcf8\uc5d0\uc11c\ub294 \uad6c\ubbf8\uad8c\uc758 \ud06c\ub9ac\uc2a4\ud2b8\uad50 \uc5d0 \uc758\ud55c \uc0ac\uccb4\uc758 \ubd80\ud65c\uc2e0\uc559\uc774\ub098 \uadf8\uc5d0 \ub530\ub77c \uc874\uc7ac\ud55c \ud654\uc7a5\uc758 \uae08\uae30\u00b7\uc800\ud56d\uac10\uacfc \uac19\uc740 \uac1c\ub150\uc740 \ubd80\uc871\ud55c \uacbd\ud5a5\uc774 \uc788\ub2e4. \ub610 \uc5d0\ub3c4\uae30\uc5d0\ub294 \ub9c8\ucc28\uac00 \uc874\uc7ac\ud558\uc9c0 \uc54a\uc558\uace0, \ub9cc\uc57d \ub9cc\uc77c \uc5ec\ud589\uc9c0\ub098 \uba3c \ubd09\uc0ac\ud560 \uacf3\uc5d0\uc11c \uae09\uc0ac\uc790\uac00 \ub098\uc624\uace0, \uadf8 \uc0ac\uccb4\ub97c \uc6d0\uaca9\uc9c0\uc5d0 \ubc18\uc1a1\ud55c\ub2e4\uba74 \uc2e4\uc9c8\uc801\uc73c\ub85c\ub294 \ub098\uac00\ubaa8\uce58 \ub4f1\uc744 \uc774\uc6a9\ud574 \uc778\ub825\uc5d0 \uc758\uc9c0\ud558\uc9c0 \uc54a\uc558\uace0, \uc77c\ubc18 \uc11c\ubbfc\uc758 \ub808\ubca8\uc5d0\uc11c\ub294 \uc0ac\uccb4\ub97c \uadf8\ub300\ub85c\uc758 \ubaa8\uc2b5\uc73c\ub85c \uc7a5\uac70\ub9ac \uc218\uc1a1\ud55c\ub2e4\ub294 \uc0dd\uac01\ub3c4 \uc120\ud0dd\uc0ac\ud56d\ub3c4 \uc874\uc7ac\ud558\uc9c0 \uc54a\uc558\ub2e4. \uc774 \uc0dd\uac01\uc740 \uad6c\ubbf8\uc778\uc5d0 \uc758\ud574\uc11c \ub9c8\ucc28\uc640 \uacac\uc778\uc6a9\uc758 \uc911\uc885\ub9c8\uac00 \ubc18\uc785\ub41c \uc5d0\ub3c4\ub9c9\ubd80 \ub9d0\uae30\ubd80\ud130 \uba54\uc774\uc9c0\uae30, \uadf8\ub9ac\uace0 \ub3d9\ub825 \uadfc\ub300\ud654\uac00 \uc9c4\ud589\ub41c \uba54\uc774\uc9c0 \ud6c4\uae30 \uc774\ud6c4\ub3c4 \ubcf8\uc9c8\uc801\uc73c\ub85c\ub294 \ub108\ubb34 \ubcc0\ud568\uc5c6\uc774, \uc804\uc2dc\uc911\ub3c4 \uc804\uc0ac\uc790\ub294 \ud604\uc9c0\uc5d0\uc11c \ud654\uc7a5\ub418\uc5b4 \uc804\ud6c4\ub3c4 \ub610 \uc624\ub7ab\ub3d9\uc548, \ub2e4\uc218\uc758 \uc0ac\ub9dd\uc790\uac00 \ubc1c\uc0dd\ud55c \uc7ac\ud574\ub098 \uc0ac\uace0\uc5d0\uc11c\ub294 \ud604\uc9c0\uc5d0\uc11c \ud654\uc7a5\ud5c8\uac00\ub97c \uc5bb\uc5b4 \uc11c\uc11c\ud788 \uc0ac\uccb4\ub97c \ud654\uc7a5\ud568\uc5d0 \uad50\ubd80\ud558\uace0 \uc720\uace8\uc744 \uac00\uc9c0\uace0 \ub3cc\uc544\uac04\ub2e4\ub294 \ud615\ud0dc\uac00 \uc77c\ubc18\uc801\uc774\uc5c8\ub2e4. \uc624\ub7ab\ub3d9\uc548 \ub9e4\uc7a5\uc2b5\uad00\uc774 \ub0a8\uc544 \uc788\ub358 \uc9c0\uc5ed\ub3c4 \ub9ce\uc9c0\ub9cc, \uc774\uac83\ub4e4\uc5d0\uc11c\ub3c4 \ud654\uc7a5\ub3c4 \uc644\uc804\ud788\ub294 \ubd80\uc815\ub418\uc9c0 \uc54a\uace0, \ud654\uc7a5\uc758 \uae30\uc220\uc758 \uc9c4\ubcf4\ub098 \uc2dc\uc124\uc758 \ub3c4\uc785\uc5d0 \uc758\ud574\uc11c \uadfc\ud604\ub300\uc5d0 \uae09\uc18d\ud788 \ub9e4\uc7a5\uc774 \uc1e0\ud1f4\ud588\ub2e4. \uadf8 \uac19\uc740 \uc77c\ub85c\ubd80\ud130, \uc77c\ubcf8\uc5d0\uc11c\ub294 \uad6c\ubbf8\uad8c\uacfc \uac19\uc740 \uc720\uccb4\ubcf4\uc874\uae30\uc220\uc758 \uc2b5\uad00\uc774 \ud37c\uc9c0\uc9c0 \uc54a\uc558\ub2e4.", "paragraph_answer": "\uc77c\ubcf8\uc5d0\uc11c\ub294 \uad6c\ubbf8\uad8c\uc758 \ud06c\ub9ac\uc2a4\ud2b8\uad50 \uc5d0 \uc758\ud55c \uc0ac\uccb4\uc758 \ubd80\ud65c\uc2e0\uc559\uc774\ub098 \uadf8\uc5d0 \ub530\ub77c \uc874\uc7ac\ud55c \ud654\uc7a5\uc758 \uae08\uae30\u00b7\uc800\ud56d\uac10\uacfc \uac19\uc740 \uac1c\ub150\uc740 \ubd80\uc871\ud55c \uacbd\ud5a5\uc774 \uc788\ub2e4. \ub610 \uc5d0\ub3c4\uae30\uc5d0\ub294 \ub9c8\ucc28\uac00 \uc874\uc7ac\ud558\uc9c0 \uc54a\uc558\uace0, \ub9cc\uc57d \ub9cc\uc77c \uc5ec\ud589\uc9c0\ub098 \uba3c \ubd09\uc0ac\ud560 \uacf3\uc5d0\uc11c \uae09\uc0ac\uc790\uac00 \ub098\uc624\uace0, \uadf8 \uc0ac\uccb4\ub97c \uc6d0\uaca9\uc9c0\uc5d0 \ubc18\uc1a1\ud55c\ub2e4\uba74 \uc2e4\uc9c8\uc801\uc73c\ub85c\ub294 \ub098\uac00\ubaa8\uce58 \ub4f1\uc744 \uc774\uc6a9\ud574 \uc778\ub825\uc5d0 \uc758\uc9c0\ud558\uc9c0 \uc54a\uc558\uace0, \uc77c\ubc18 \uc11c\ubbfc\uc758 \ub808\ubca8\uc5d0\uc11c\ub294 \uc0ac\uccb4\ub97c \uadf8\ub300\ub85c\uc758 \ubaa8\uc2b5\uc73c\ub85c \uc7a5\uac70\ub9ac \uc218\uc1a1\ud55c\ub2e4\ub294 \uc0dd\uac01\ub3c4 \uc120\ud0dd\uc0ac\ud56d\ub3c4 \uc874\uc7ac\ud558\uc9c0 \uc54a\uc558\ub2e4. \uc774 \uc0dd\uac01\uc740 \uad6c\ubbf8\uc778\uc5d0 \uc758\ud574\uc11c \ub9c8\ucc28\uc640 \uacac\uc778\uc6a9\uc758 \uc911\uc885\ub9c8\uac00 \ubc18\uc785\ub41c \uc5d0\ub3c4\ub9c9\ubd80 \ub9d0\uae30\ubd80\ud130 \uba54\uc774\uc9c0\uae30, \uadf8\ub9ac\uace0 \ub3d9\ub825 \uadfc\ub300\ud654\uac00 \uc9c4\ud589\ub41c \uba54\uc774\uc9c0 \ud6c4\uae30 \uc774\ud6c4\ub3c4 \ubcf8\uc9c8\uc801\uc73c\ub85c\ub294 \ub108\ubb34 \ubcc0\ud568\uc5c6\uc774, \uc804\uc2dc\uc911\ub3c4 \uc804\uc0ac\uc790\ub294 \ud604\uc9c0\uc5d0\uc11c \ud654\uc7a5\ub418\uc5b4 \uc804\ud6c4\ub3c4 \ub610 \uc624\ub7ab\ub3d9\uc548, \ub2e4\uc218\uc758 \uc0ac\ub9dd\uc790\uac00 \ubc1c\uc0dd\ud55c \uc7ac\ud574\ub098 \uc0ac\uace0\uc5d0\uc11c\ub294 \ud604\uc9c0\uc5d0\uc11c \ud654\uc7a5\ud5c8\uac00\ub97c \uc5bb\uc5b4 \uc11c\uc11c\ud788 \uc0ac\uccb4\ub97c \ud654\uc7a5\ud568\uc5d0 \uad50\ubd80\ud558\uace0 \uc720\uace8\uc744 \uac00\uc9c0\uace0 \ub3cc\uc544\uac04\ub2e4\ub294 \ud615\ud0dc\uac00 \uc77c\ubc18\uc801\uc774\uc5c8\ub2e4. \uc624\ub7ab\ub3d9\uc548 \ub9e4\uc7a5\uc2b5\uad00\uc774 \ub0a8\uc544 \uc788\ub358 \uc9c0\uc5ed\ub3c4 \ub9ce\uc9c0\ub9cc, \uc774\uac83\ub4e4\uc5d0\uc11c\ub3c4 \ud654\uc7a5\ub3c4 \uc644\uc804\ud788\ub294 \ubd80\uc815\ub418\uc9c0 \uc54a\uace0, \ud654\uc7a5\uc758 \uae30\uc220\uc758 \uc9c4\ubcf4\ub098 \uc2dc\uc124\uc758 \ub3c4\uc785\uc5d0 \uc758\ud574\uc11c \uadfc\ud604\ub300\uc5d0 \uae09\uc18d\ud788 \ub9e4\uc7a5\uc774 \uc1e0\ud1f4\ud588\ub2e4. \uadf8 \uac19\uc740 \uc77c\ub85c\ubd80\ud130, \uc77c\ubcf8\uc5d0\uc11c\ub294 \uad6c\ubbf8\uad8c\uacfc \uac19\uc740 \uc720\uccb4\ubcf4\uc874\uae30\uc220\uc758 \uc2b5\uad00\uc774 \ud37c\uc9c0\uc9c0 \uc54a\uc558\ub2e4.", "sentence_answer": "\uc77c\ubcf8\uc5d0\uc11c\ub294 \uad6c\ubbf8\uad8c\uc758 \ud06c\ub9ac\uc2a4\ud2b8\uad50 \uc5d0 \uc758\ud55c \uc0ac\uccb4\uc758 \ubd80\ud65c\uc2e0\uc559\uc774\ub098 \uadf8\uc5d0 \ub530\ub77c \uc874\uc7ac\ud55c \ud654\uc7a5\uc758 \uae08\uae30\u00b7\uc800\ud56d\uac10\uacfc \uac19\uc740 \uac1c\ub150\uc740 \ubd80\uc871\ud55c \uacbd\ud5a5\uc774 \uc788\ub2e4.", "paragraph_id": "6589258-0-1", "paragraph_question": "question: \uad6c\ubbf8\uad8c\uc758 \uc0ac\uccb4\ubd80\ud65c \uc2e0\uc559\uc740 \ubb34\uc2a8 \uc885\uad50\uc5d0 \uc758\ud55c \uac83\uc778\uac00?, context: \uc77c\ubcf8\uc5d0\uc11c\ub294 \uad6c\ubbf8\uad8c\uc758 \ud06c\ub9ac\uc2a4\ud2b8\uad50\uc5d0 \uc758\ud55c \uc0ac\uccb4\uc758 \ubd80\ud65c\uc2e0\uc559\uc774\ub098 \uadf8\uc5d0 \ub530\ub77c \uc874\uc7ac\ud55c \ud654\uc7a5\uc758 \uae08\uae30\u00b7\uc800\ud56d\uac10\uacfc \uac19\uc740 \uac1c\ub150\uc740 \ubd80\uc871\ud55c \uacbd\ud5a5\uc774 \uc788\ub2e4. \ub610 \uc5d0\ub3c4\uae30\uc5d0\ub294 \ub9c8\ucc28\uac00 \uc874\uc7ac\ud558\uc9c0 \uc54a\uc558\uace0, \ub9cc\uc57d \ub9cc\uc77c \uc5ec\ud589\uc9c0\ub098 \uba3c \ubd09\uc0ac\ud560 \uacf3\uc5d0\uc11c \uae09\uc0ac\uc790\uac00 \ub098\uc624\uace0, \uadf8 \uc0ac\uccb4\ub97c \uc6d0\uaca9\uc9c0\uc5d0 \ubc18\uc1a1\ud55c\ub2e4\uba74 \uc2e4\uc9c8\uc801\uc73c\ub85c\ub294 \ub098\uac00\ubaa8\uce58 \ub4f1\uc744 \uc774\uc6a9\ud574 \uc778\ub825\uc5d0 \uc758\uc9c0\ud558\uc9c0 \uc54a\uc558\uace0, \uc77c\ubc18 \uc11c\ubbfc\uc758 \ub808\ubca8\uc5d0\uc11c\ub294 \uc0ac\uccb4\ub97c \uadf8\ub300\ub85c\uc758 \ubaa8\uc2b5\uc73c\ub85c \uc7a5\uac70\ub9ac \uc218\uc1a1\ud55c\ub2e4\ub294 \uc0dd\uac01\ub3c4 \uc120\ud0dd\uc0ac\ud56d\ub3c4 \uc874\uc7ac\ud558\uc9c0 \uc54a\uc558\ub2e4. \uc774 \uc0dd\uac01\uc740 \uad6c\ubbf8\uc778\uc5d0 \uc758\ud574\uc11c \ub9c8\ucc28\uc640 \uacac\uc778\uc6a9\uc758 \uc911\uc885\ub9c8\uac00 \ubc18\uc785\ub41c \uc5d0\ub3c4\ub9c9\ubd80 \ub9d0\uae30\ubd80\ud130 \uba54\uc774\uc9c0\uae30, \uadf8\ub9ac\uace0 \ub3d9\ub825 \uadfc\ub300\ud654\uac00 \uc9c4\ud589\ub41c \uba54\uc774\uc9c0 \ud6c4\uae30 \uc774\ud6c4\ub3c4 \ubcf8\uc9c8\uc801\uc73c\ub85c\ub294 \ub108\ubb34 \ubcc0\ud568\uc5c6\uc774, \uc804\uc2dc\uc911\ub3c4 \uc804\uc0ac\uc790\ub294 \ud604\uc9c0\uc5d0\uc11c \ud654\uc7a5\ub418\uc5b4 \uc804\ud6c4\ub3c4 \ub610 \uc624\ub7ab\ub3d9\uc548, \ub2e4\uc218\uc758 \uc0ac\ub9dd\uc790\uac00 \ubc1c\uc0dd\ud55c \uc7ac\ud574\ub098 \uc0ac\uace0\uc5d0\uc11c\ub294 \ud604\uc9c0\uc5d0\uc11c \ud654\uc7a5\ud5c8\uac00\ub97c \uc5bb\uc5b4 \uc11c\uc11c\ud788 \uc0ac\uccb4\ub97c \ud654\uc7a5\ud568\uc5d0 \uad50\ubd80\ud558\uace0 \uc720\uace8\uc744 \uac00\uc9c0\uace0 \ub3cc\uc544\uac04\ub2e4\ub294 \ud615\ud0dc\uac00 \uc77c\ubc18\uc801\uc774\uc5c8\ub2e4. \uc624\ub7ab\ub3d9\uc548 \ub9e4\uc7a5\uc2b5\uad00\uc774 \ub0a8\uc544 \uc788\ub358 \uc9c0\uc5ed\ub3c4 \ub9ce\uc9c0\ub9cc, \uc774\uac83\ub4e4\uc5d0\uc11c\ub3c4 \ud654\uc7a5\ub3c4 \uc644\uc804\ud788\ub294 \ubd80\uc815\ub418\uc9c0 \uc54a\uace0, \ud654\uc7a5\uc758 \uae30\uc220\uc758 \uc9c4\ubcf4\ub098 \uc2dc\uc124\uc758 \ub3c4\uc785\uc5d0 \uc758\ud574\uc11c \uadfc\ud604\ub300\uc5d0 \uae09\uc18d\ud788 \ub9e4\uc7a5\uc774 \uc1e0\ud1f4\ud588\ub2e4. \uadf8 \uac19\uc740 \uc77c\ub85c\ubd80\ud130, \uc77c\ubcf8\uc5d0\uc11c\ub294 \uad6c\ubbf8\uad8c\uacfc \uac19\uc740 \uc720\uccb4\ubcf4\uc874\uae30\uc220\uc758 \uc2b5\uad00\uc774 \ud37c\uc9c0\uc9c0 \uc54a\uc558\ub2e4."} {"question": "\uc774 \uc0ac\uac74\uc774 \ud070 \uc774\uc288\uac00 \ub41c \uc774\uc720\ub294 \uc6b0\ub9ac \uc0ac\ud68c\uc758 \uc5b4\ub5a0\ud55c \ud604\uc2e4 \ub54c\ubb38\uc778\uac00?", "paragraph": "\uc11c\ucc9c\uc11d \uccad\uc18c\ub144\uc815\uc2e0\uacfc \uc804\ubb38\uc758\ub294 \u201c\uc815\uc2e0\ubcd1\uc758 \uc99d\uc0c1\uc740 \uc0ac\ud68c\uc801 \ub9e5\ub77d \uc18d\uc5d0 \uc788\ub2e4\u201d\uba74\uc11c \u201c\ubb38\uc81c\ub294 \uadf8\uac00 \u2018\uc5ec\uc131\ub4e4\uc774 \ub098\ub97c \ubb34\uc2dc\ud574\uc11c\u2019 \ubc94\uc8c4\ub97c \uc800\uc9c8\ub800\ub2e4\uace0 \ub9d0\ud55c \uac83\uc774\ub2e4. \uc774 \ub9d0\uc740 \uc0ac\ud68c\uc801 \ub9e5\ub77d\uc744 \uac16\uace0 \uc788\uace0 \uadf8\uac83\uc740 \u2018\uc5ec\uc131\ud610\uc624\u2019\u201d\ub77c\uace0 \uc9c0\uc801\ud588\ub2e4. \u201c\uacfc\uac70 \uad8c\uc704\uc8fc\uc758 \ub3c5\uc7ac \uc2dc\uc808\uc5d0\ub294 \ub9ce\uc740 \uc870\ud604\ubcd1 \ud658\uc790\ub4e4\uc774 \ud658\uccad\uc744 \ud638\uc18c\ud558\uba74\uc11c \uc911\uc559\uc815\ubcf4\ubd80\uac00 \ub098\ub97c \ubbf8\ud589\ud558\uace0 \ub3c4\uccad\ud558\uace0 \uc788\ub2e4\ub294 \uc774\uc57c\uae30\u201d\ub97c \ud588\uace0, \u201c80\ub144\ub300 \ud6c4\ubc18\uc5d0\ub294 CIA\u201d\uac00, \u201c2000\ub144\ub300 \uc774\ud6c4\uc5d0\ub294 \uc0bc\uc131\uc774 \uc18c\uc7ac\uac00 \ub418\ub294 \uacbd\uc6b0\ub3c4 \uc788\uc5c8\u201d\ub358 \uac83\ucc98\ub7fc \ud604\uc7ac \u2018\uc5ec\uc131\ud610\uc624\u2019\uac00 \ub4f1\uc7a5\ud55c \uac83\uc744 \uc8fc\ubaa9\ud574\uc57c \ud55c\ub2e4\ub294 \uac83\uc774\ub2e4\"\uba74\uc11c \u201c\uc5ec\uc131 \ud610\uc624 \uc758\uc2dd\uc774 \uc815\uc2e0\ubcd1\uc758 \uc99d\uc0c1\uc73c\ub85c\uae4c\uc9c0 \ubc1c\uc804\ud558\uace0 \uc788\ub2e4\uba74 \uadf8 \uc2ec\uac01\uc131\uc744 \uc778\uc815\ud558\uace0, \uc0ac\ud68c \uc804\ubc18\uc5d0\uc11c \uc774\ub7f0 \uc758\uc2dd\uc774 \uc790\ub9ac\uc7a1\uc9c0 \ubabb\ud558\ub3c4\ub85d \uad6c\uc870\uc801 \uac1c\ud601\uc744 \ud558\uace0 \uc758\uc2dd\uc758 \ubcc0\ud654\ub97c \ucd94\uad6c\ud574\uc57c\"\ud55c\ub2e4\uace0 \ud558\uace0 \u201c\u2018\uc815\uc2e0\ubcd1\uc774 \ubc94\uc8c4\uc758 \uc6d0\uc778\uc774\ub0d0? \uc544\ub2c8\uba74 \uc5ec\ud610\uc774 \uc6d0\uc778\uc774\ub0d0?\u2019 \uc774\ub7f0 \uc218\uc900 \ub0ae\uc740 \ub17c\uc7c1\uc740 \uc774\uc820 \uba48\ucdb0\uc57c \ud55c\ub2e4\u201d\uace0 \uc8fc\uc7a5\ud558\uc600\ub2e4. \ub300\uc911\uc758 \ubc18\uc751\uc5d0 \ub300\ud574\uc11c\ub294 \"\uc774 \uc0ac\uac74\uc774 \ud070 \uc774\uc288\uac00 \ub41c \uc774\uc720\ub294 \ud55c \ubc94\uc8c4\uc790\uc758 \ub9d0 \ub54c\ubb38\uc774 \uc544\ub2c8\ub77c \uadf8 \ubc94\uc8c4\uac00 \uc77c\uc5b4\ub09c \uc6b0\ub9ac \uc0ac\ud68c\uc758 \uc704\ud5d8\ud55c \ud604\uc2e4 \ub54c\ubb38\uc774\ub2e4. \uac15\ub825 \uc0ac\uac74\uc758 \ud76c\uc0dd\uc790 \ube44\uc728\uc774 \ub0a8\uc131\uc5d0 \ube44\ud574 \uc5ec\uc131\uc774 8\ubc30\uac00 \ub118\ub294 \ud1b5\uacc4\ub85c \uc54c \uc218 \uc788\ub4ef \uc5ec\uc131\ub4e4\uc774 \uc548\uc804\ud558\uc9c0 \uc54a\uae30 \ub54c\ubb38\uc774\ub2e4.\"\ub77c\uace0 \ubd84\uc11d\ud588\ub2e4.", "answer": "\uc704\ud5d8\ud55c \ud604\uc2e4", "sentence": "\ub300\uc911\uc758 \ubc18\uc751\uc5d0 \ub300\ud574\uc11c\ub294 \"\uc774 \uc0ac\uac74\uc774 \ud070 \uc774\uc288\uac00 \ub41c \uc774\uc720\ub294 \ud55c \ubc94\uc8c4\uc790\uc758 \ub9d0 \ub54c\ubb38\uc774 \uc544\ub2c8\ub77c \uadf8 \ubc94\uc8c4\uac00 \uc77c\uc5b4\ub09c \uc6b0\ub9ac \uc0ac\ud68c\uc758 \uc704\ud5d8\ud55c \ud604\uc2e4 \ub54c\ubb38\uc774\ub2e4.", "paragraph_sentence": "\uc11c\ucc9c\uc11d \uccad\uc18c\ub144\uc815\uc2e0\uacfc \uc804\ubb38\uc758\ub294 \u201c\uc815\uc2e0\ubcd1\uc758 \uc99d\uc0c1\uc740 \uc0ac\ud68c\uc801 \ub9e5\ub77d \uc18d\uc5d0 \uc788\ub2e4\u201d\uba74\uc11c \u201c\ubb38\uc81c\ub294 \uadf8\uac00 \u2018\uc5ec\uc131\ub4e4\uc774 \ub098\ub97c \ubb34\uc2dc\ud574\uc11c\u2019 \ubc94\uc8c4\ub97c \uc800\uc9c8\ub800\ub2e4\uace0 \ub9d0\ud55c \uac83\uc774\ub2e4. \uc774 \ub9d0\uc740 \uc0ac\ud68c\uc801 \ub9e5\ub77d\uc744 \uac16\uace0 \uc788\uace0 \uadf8\uac83\uc740 \u2018\uc5ec\uc131\ud610\uc624\u2019\u201d\ub77c\uace0 \uc9c0\uc801\ud588\ub2e4. \u201c\uacfc\uac70 \uad8c\uc704\uc8fc\uc758 \ub3c5\uc7ac \uc2dc\uc808\uc5d0\ub294 \ub9ce\uc740 \uc870\ud604\ubcd1 \ud658\uc790\ub4e4\uc774 \ud658\uccad\uc744 \ud638\uc18c\ud558\uba74\uc11c \uc911\uc559\uc815\ubcf4\ubd80\uac00 \ub098\ub97c \ubbf8\ud589\ud558\uace0 \ub3c4\uccad\ud558\uace0 \uc788\ub2e4\ub294 \uc774\uc57c\uae30\u201d\ub97c \ud588\uace0, \u201c80\ub144\ub300 \ud6c4\ubc18\uc5d0\ub294 CIA\u201d\uac00, \u201c2000\ub144\ub300 \uc774\ud6c4\uc5d0\ub294 \uc0bc\uc131\uc774 \uc18c\uc7ac\uac00 \ub418\ub294 \uacbd\uc6b0\ub3c4 \uc788\uc5c8\u201d\ub358 \uac83\ucc98\ub7fc \ud604\uc7ac \u2018\uc5ec\uc131\ud610\uc624\u2019\uac00 \ub4f1\uc7a5\ud55c \uac83\uc744 \uc8fc\ubaa9\ud574\uc57c \ud55c\ub2e4\ub294 \uac83\uc774\ub2e4\"\uba74\uc11c \u201c\uc5ec\uc131 \ud610\uc624 \uc758\uc2dd\uc774 \uc815\uc2e0\ubcd1\uc758 \uc99d\uc0c1\uc73c\ub85c\uae4c\uc9c0 \ubc1c\uc804\ud558\uace0 \uc788\ub2e4\uba74 \uadf8 \uc2ec\uac01\uc131\uc744 \uc778\uc815\ud558\uace0, \uc0ac\ud68c \uc804\ubc18\uc5d0\uc11c \uc774\ub7f0 \uc758\uc2dd\uc774 \uc790\ub9ac\uc7a1\uc9c0 \ubabb\ud558\ub3c4\ub85d \uad6c\uc870\uc801 \uac1c\ud601\uc744 \ud558\uace0 \uc758\uc2dd\uc758 \ubcc0\ud654\ub97c \ucd94\uad6c\ud574\uc57c\"\ud55c\ub2e4\uace0 \ud558\uace0 \u201c\u2018\uc815\uc2e0\ubcd1\uc774 \ubc94\uc8c4\uc758 \uc6d0\uc778\uc774\ub0d0? \uc544\ub2c8\uba74 \uc5ec\ud610\uc774 \uc6d0\uc778\uc774\ub0d0? \u2019 \uc774\ub7f0 \uc218\uc900 \ub0ae\uc740 \ub17c\uc7c1\uc740 \uc774\uc820 \uba48\ucdb0\uc57c \ud55c\ub2e4\u201d\uace0 \uc8fc\uc7a5\ud558\uc600\ub2e4. \ub300\uc911\uc758 \ubc18\uc751\uc5d0 \ub300\ud574\uc11c\ub294 \"\uc774 \uc0ac\uac74\uc774 \ud070 \uc774\uc288\uac00 \ub41c \uc774\uc720\ub294 \ud55c \ubc94\uc8c4\uc790\uc758 \ub9d0 \ub54c\ubb38\uc774 \uc544\ub2c8\ub77c \uadf8 \ubc94\uc8c4\uac00 \uc77c\uc5b4\ub09c \uc6b0\ub9ac \uc0ac\ud68c\uc758 \uc704\ud5d8\ud55c \ud604\uc2e4 \ub54c\ubb38\uc774\ub2e4. \uac15\ub825 \uc0ac\uac74\uc758 \ud76c\uc0dd\uc790 \ube44\uc728\uc774 \ub0a8\uc131\uc5d0 \ube44\ud574 \uc5ec\uc131\uc774 8\ubc30\uac00 \ub118\ub294 \ud1b5\uacc4\ub85c \uc54c \uc218 \uc788\ub4ef \uc5ec\uc131\ub4e4\uc774 \uc548\uc804\ud558\uc9c0 \uc54a\uae30 \ub54c\ubb38\uc774\ub2e4. \"\ub77c\uace0 \ubd84\uc11d\ud588\ub2e4.", "paragraph_answer": "\uc11c\ucc9c\uc11d \uccad\uc18c\ub144\uc815\uc2e0\uacfc \uc804\ubb38\uc758\ub294 \u201c\uc815\uc2e0\ubcd1\uc758 \uc99d\uc0c1\uc740 \uc0ac\ud68c\uc801 \ub9e5\ub77d \uc18d\uc5d0 \uc788\ub2e4\u201d\uba74\uc11c \u201c\ubb38\uc81c\ub294 \uadf8\uac00 \u2018\uc5ec\uc131\ub4e4\uc774 \ub098\ub97c \ubb34\uc2dc\ud574\uc11c\u2019 \ubc94\uc8c4\ub97c \uc800\uc9c8\ub800\ub2e4\uace0 \ub9d0\ud55c \uac83\uc774\ub2e4. \uc774 \ub9d0\uc740 \uc0ac\ud68c\uc801 \ub9e5\ub77d\uc744 \uac16\uace0 \uc788\uace0 \uadf8\uac83\uc740 \u2018\uc5ec\uc131\ud610\uc624\u2019\u201d\ub77c\uace0 \uc9c0\uc801\ud588\ub2e4. \u201c\uacfc\uac70 \uad8c\uc704\uc8fc\uc758 \ub3c5\uc7ac \uc2dc\uc808\uc5d0\ub294 \ub9ce\uc740 \uc870\ud604\ubcd1 \ud658\uc790\ub4e4\uc774 \ud658\uccad\uc744 \ud638\uc18c\ud558\uba74\uc11c \uc911\uc559\uc815\ubcf4\ubd80\uac00 \ub098\ub97c \ubbf8\ud589\ud558\uace0 \ub3c4\uccad\ud558\uace0 \uc788\ub2e4\ub294 \uc774\uc57c\uae30\u201d\ub97c \ud588\uace0, \u201c80\ub144\ub300 \ud6c4\ubc18\uc5d0\ub294 CIA\u201d\uac00, \u201c2000\ub144\ub300 \uc774\ud6c4\uc5d0\ub294 \uc0bc\uc131\uc774 \uc18c\uc7ac\uac00 \ub418\ub294 \uacbd\uc6b0\ub3c4 \uc788\uc5c8\u201d\ub358 \uac83\ucc98\ub7fc \ud604\uc7ac \u2018\uc5ec\uc131\ud610\uc624\u2019\uac00 \ub4f1\uc7a5\ud55c \uac83\uc744 \uc8fc\ubaa9\ud574\uc57c \ud55c\ub2e4\ub294 \uac83\uc774\ub2e4\"\uba74\uc11c \u201c\uc5ec\uc131 \ud610\uc624 \uc758\uc2dd\uc774 \uc815\uc2e0\ubcd1\uc758 \uc99d\uc0c1\uc73c\ub85c\uae4c\uc9c0 \ubc1c\uc804\ud558\uace0 \uc788\ub2e4\uba74 \uadf8 \uc2ec\uac01\uc131\uc744 \uc778\uc815\ud558\uace0, \uc0ac\ud68c \uc804\ubc18\uc5d0\uc11c \uc774\ub7f0 \uc758\uc2dd\uc774 \uc790\ub9ac\uc7a1\uc9c0 \ubabb\ud558\ub3c4\ub85d \uad6c\uc870\uc801 \uac1c\ud601\uc744 \ud558\uace0 \uc758\uc2dd\uc758 \ubcc0\ud654\ub97c \ucd94\uad6c\ud574\uc57c\"\ud55c\ub2e4\uace0 \ud558\uace0 \u201c\u2018\uc815\uc2e0\ubcd1\uc774 \ubc94\uc8c4\uc758 \uc6d0\uc778\uc774\ub0d0? \uc544\ub2c8\uba74 \uc5ec\ud610\uc774 \uc6d0\uc778\uc774\ub0d0?\u2019 \uc774\ub7f0 \uc218\uc900 \ub0ae\uc740 \ub17c\uc7c1\uc740 \uc774\uc820 \uba48\ucdb0\uc57c \ud55c\ub2e4\u201d\uace0 \uc8fc\uc7a5\ud558\uc600\ub2e4. \ub300\uc911\uc758 \ubc18\uc751\uc5d0 \ub300\ud574\uc11c\ub294 \"\uc774 \uc0ac\uac74\uc774 \ud070 \uc774\uc288\uac00 \ub41c \uc774\uc720\ub294 \ud55c \ubc94\uc8c4\uc790\uc758 \ub9d0 \ub54c\ubb38\uc774 \uc544\ub2c8\ub77c \uadf8 \ubc94\uc8c4\uac00 \uc77c\uc5b4\ub09c \uc6b0\ub9ac \uc0ac\ud68c\uc758 \uc704\ud5d8\ud55c \ud604\uc2e4 \ub54c\ubb38\uc774\ub2e4. \uac15\ub825 \uc0ac\uac74\uc758 \ud76c\uc0dd\uc790 \ube44\uc728\uc774 \ub0a8\uc131\uc5d0 \ube44\ud574 \uc5ec\uc131\uc774 8\ubc30\uac00 \ub118\ub294 \ud1b5\uacc4\ub85c \uc54c \uc218 \uc788\ub4ef \uc5ec\uc131\ub4e4\uc774 \uc548\uc804\ud558\uc9c0 \uc54a\uae30 \ub54c\ubb38\uc774\ub2e4.\"\ub77c\uace0 \ubd84\uc11d\ud588\ub2e4.", "sentence_answer": "\ub300\uc911\uc758 \ubc18\uc751\uc5d0 \ub300\ud574\uc11c\ub294 \"\uc774 \uc0ac\uac74\uc774 \ud070 \uc774\uc288\uac00 \ub41c \uc774\uc720\ub294 \ud55c \ubc94\uc8c4\uc790\uc758 \ub9d0 \ub54c\ubb38\uc774 \uc544\ub2c8\ub77c \uadf8 \ubc94\uc8c4\uac00 \uc77c\uc5b4\ub09c \uc6b0\ub9ac \uc0ac\ud68c\uc758 \uc704\ud5d8\ud55c \ud604\uc2e4 \ub54c\ubb38\uc774\ub2e4.", "paragraph_id": "6601077-8-2", "paragraph_question": "question: \uc774 \uc0ac\uac74\uc774 \ud070 \uc774\uc288\uac00 \ub41c \uc774\uc720\ub294 \uc6b0\ub9ac \uc0ac\ud68c\uc758 \uc5b4\ub5a0\ud55c \ud604\uc2e4 \ub54c\ubb38\uc778\uac00?, context: \uc11c\ucc9c\uc11d \uccad\uc18c\ub144\uc815\uc2e0\uacfc \uc804\ubb38\uc758\ub294 \u201c\uc815\uc2e0\ubcd1\uc758 \uc99d\uc0c1\uc740 \uc0ac\ud68c\uc801 \ub9e5\ub77d \uc18d\uc5d0 \uc788\ub2e4\u201d\uba74\uc11c \u201c\ubb38\uc81c\ub294 \uadf8\uac00 \u2018\uc5ec\uc131\ub4e4\uc774 \ub098\ub97c \ubb34\uc2dc\ud574\uc11c\u2019 \ubc94\uc8c4\ub97c \uc800\uc9c8\ub800\ub2e4\uace0 \ub9d0\ud55c \uac83\uc774\ub2e4. \uc774 \ub9d0\uc740 \uc0ac\ud68c\uc801 \ub9e5\ub77d\uc744 \uac16\uace0 \uc788\uace0 \uadf8\uac83\uc740 \u2018\uc5ec\uc131\ud610\uc624\u2019\u201d\ub77c\uace0 \uc9c0\uc801\ud588\ub2e4. \u201c\uacfc\uac70 \uad8c\uc704\uc8fc\uc758 \ub3c5\uc7ac \uc2dc\uc808\uc5d0\ub294 \ub9ce\uc740 \uc870\ud604\ubcd1 \ud658\uc790\ub4e4\uc774 \ud658\uccad\uc744 \ud638\uc18c\ud558\uba74\uc11c \uc911\uc559\uc815\ubcf4\ubd80\uac00 \ub098\ub97c \ubbf8\ud589\ud558\uace0 \ub3c4\uccad\ud558\uace0 \uc788\ub2e4\ub294 \uc774\uc57c\uae30\u201d\ub97c \ud588\uace0, \u201c80\ub144\ub300 \ud6c4\ubc18\uc5d0\ub294 CIA\u201d\uac00, \u201c2000\ub144\ub300 \uc774\ud6c4\uc5d0\ub294 \uc0bc\uc131\uc774 \uc18c\uc7ac\uac00 \ub418\ub294 \uacbd\uc6b0\ub3c4 \uc788\uc5c8\u201d\ub358 \uac83\ucc98\ub7fc \ud604\uc7ac \u2018\uc5ec\uc131\ud610\uc624\u2019\uac00 \ub4f1\uc7a5\ud55c \uac83\uc744 \uc8fc\ubaa9\ud574\uc57c \ud55c\ub2e4\ub294 \uac83\uc774\ub2e4\"\uba74\uc11c \u201c\uc5ec\uc131 \ud610\uc624 \uc758\uc2dd\uc774 \uc815\uc2e0\ubcd1\uc758 \uc99d\uc0c1\uc73c\ub85c\uae4c\uc9c0 \ubc1c\uc804\ud558\uace0 \uc788\ub2e4\uba74 \uadf8 \uc2ec\uac01\uc131\uc744 \uc778\uc815\ud558\uace0, \uc0ac\ud68c \uc804\ubc18\uc5d0\uc11c \uc774\ub7f0 \uc758\uc2dd\uc774 \uc790\ub9ac\uc7a1\uc9c0 \ubabb\ud558\ub3c4\ub85d \uad6c\uc870\uc801 \uac1c\ud601\uc744 \ud558\uace0 \uc758\uc2dd\uc758 \ubcc0\ud654\ub97c \ucd94\uad6c\ud574\uc57c\"\ud55c\ub2e4\uace0 \ud558\uace0 \u201c\u2018\uc815\uc2e0\ubcd1\uc774 \ubc94\uc8c4\uc758 \uc6d0\uc778\uc774\ub0d0? \uc544\ub2c8\uba74 \uc5ec\ud610\uc774 \uc6d0\uc778\uc774\ub0d0?\u2019 \uc774\ub7f0 \uc218\uc900 \ub0ae\uc740 \ub17c\uc7c1\uc740 \uc774\uc820 \uba48\ucdb0\uc57c \ud55c\ub2e4\u201d\uace0 \uc8fc\uc7a5\ud558\uc600\ub2e4. \ub300\uc911\uc758 \ubc18\uc751\uc5d0 \ub300\ud574\uc11c\ub294 \"\uc774 \uc0ac\uac74\uc774 \ud070 \uc774\uc288\uac00 \ub41c \uc774\uc720\ub294 \ud55c \ubc94\uc8c4\uc790\uc758 \ub9d0 \ub54c\ubb38\uc774 \uc544\ub2c8\ub77c \uadf8 \ubc94\uc8c4\uac00 \uc77c\uc5b4\ub09c \uc6b0\ub9ac \uc0ac\ud68c\uc758 \uc704\ud5d8\ud55c \ud604\uc2e4 \ub54c\ubb38\uc774\ub2e4. \uac15\ub825 \uc0ac\uac74\uc758 \ud76c\uc0dd\uc790 \ube44\uc728\uc774 \ub0a8\uc131\uc5d0 \ube44\ud574 \uc5ec\uc131\uc774 8\ubc30\uac00 \ub118\ub294 \ud1b5\uacc4\ub85c \uc54c \uc218 \uc788\ub4ef \uc5ec\uc131\ub4e4\uc774 \uc548\uc804\ud558\uc9c0 \uc54a\uae30 \ub54c\ubb38\uc774\ub2e4.\"\ub77c\uace0 \ubd84\uc11d\ud588\ub2e4."} {"question": "2010\ub144 \uc62c\uc2a4\ud0c0\uc804\uc740 \uba87\uc6d4 \uba87\uc77c\uc5d0 \uc62c\ub9b4 \uc608\uc815\uc774\uc5c8\ub294\uac00?", "paragraph": "KBO\ub294 22\uc77c\uc5d0 \uc774 \ud2b8\ub808\uc774\ub4dc\ub97c \ud569\ubc95\uc801\uc778 \ud2b8\ub808\uc774\ub4dc\ub85c \uac04\uc8fc\ud558\uace0 \uc2b9\uc778 \ucc98\ub9ac\ud558\uc600\ub2e4. KBO\ub294 \ub2e4\ub7c9\uc758 \ud2b8\ub808\uc774\ub4dc\ub97c \uc6b0\ub824\ud574 2010 \uc2dc\uc98c\uc774 \ub05d\ub0a0 \ub54c \uae4c\uc9c0 \ub125\uc13c \ud788\uc5b4\ub85c\uc988\uc758 \ud2b8\ub808\uc774\ub4dc\ub97c \uae08\uc9c0\ud558\ub294 \uac01\uc11c\ub97c \ub125\uc13c \uce21\uc5d0\uc11c \uc5bb\uc5b4\ub0b4\uc5c8\ub2e4. 7\uc6d4 24\uc77c\uc5d0 \uc5f4\ub9b4 \uc608\uc815\uc778 2010\ub144 \uc62c\uc2a4\ud0c0\uc804\uc5d0\uc11c \ub125\uc13c \ud788\uc5b4\ub85c\uc988 \uc18c\uc18d\uc73c\ub85c \uc62c\uc2a4\ud0c0\ub85c \ubf51\ud600 \uc6e8\uc2a4\ud134 \ub9ac\uadf8 \uc18c\uc18d\uc73c\ub85c \ucd9c\uc804\ud560 \uc608\uc815\uc774\uc5c8\ub358 \ud669\uc7ac\uade0\uc740 \uc774\uc2a4\ud134 \ub9ac\uadf8\uc5d0 \uc18d\ud574\uc788\ub294 \ub86f\ub370 \uc790\uc774\uc5b8\uce20\ub85c \uc774\uc801\ud568\uc5d0 \ub530\ub77c \uc62c\uc2a4\ud0c0\uc804\uc744 \uc774\uc2a4\ud134 \ub9ac\uadf8 \uc18c\uc18d\uc73c\ub85c \ucd9c\uc804\ud558\uac8c \ub418\uc5c8\ub2e4. 1\uba85\uc774 \ucd94\uac00\ub41c \uc774\uc2a4\ud134 \ub9ac\uadf8\uc5d0 \ube44\ud574 1\uba85\uc774 \ube60\uc9c4 \uc6e8\uc2a4\ud134 \ub9ac\uadf8\ub294 \uae40\uc120\ube48\uacfc \uc624\uc9c0\ud658\uc744 \ucd94\uac00\ud574 \uade0\ud615\uc744 \ub9de\ucd94\uc5c8\ub2e4. \ub86f\ub370\uc5d0 \ud569\ub958\ud574 \uc788\ub358 \ud669\uc7ac\uade0\uc740 \uadf8 \ub0a0 \ubc14\ub85c \ud55c\ud654 \uc774\uae00\uc2a4\uc640\uc758 \uacbd\uae30\uc5d0 2\ubc88 \ud0c0\uc790 3\ub8e8\uc218\ub85c \uc120\ubc1c \ucd9c\uc804\ud558\uc5ec \uc548\ud0c0\ub3c4 \uae30\ub85d\ud558\uc600\ub2e4. \ub85c\uc774\uc2a4\ud130 \uac10\ub3c5\uc740 3\ub8e8\uc218\uc640 \uc720\uaca9\uc218 \uacb8\uc5c5\uc744 \uc2dc\uc0ac\ud558\uc600\ub2e4.", "answer": "7\uc6d4 24\uc77c", "sentence": "7\uc6d4 24\uc77c \uc5d0 \uc5f4\ub9b4 \uc608\uc815\uc778 2010\ub144 \uc62c\uc2a4\ud0c0\uc804\uc5d0\uc11c \ub125\uc13c \ud788\uc5b4\ub85c\uc988 \uc18c\uc18d\uc73c\ub85c \uc62c\uc2a4\ud0c0\ub85c \ubf51\ud600 \uc6e8\uc2a4\ud134 \ub9ac\uadf8 \uc18c\uc18d\uc73c\ub85c \ucd9c\uc804\ud560 \uc608\uc815\uc774\uc5c8\ub358 \ud669\uc7ac\uade0\uc740 \uc774\uc2a4\ud134 \ub9ac\uadf8\uc5d0 \uc18d\ud574\uc788\ub294 \ub86f\ub370 \uc790\uc774\uc5b8\uce20\ub85c \uc774\uc801\ud568\uc5d0 \ub530\ub77c \uc62c\uc2a4\ud0c0\uc804\uc744 \uc774\uc2a4\ud134 \ub9ac\uadf8 \uc18c\uc18d\uc73c\ub85c \ucd9c\uc804\ud558\uac8c \ub418\uc5c8\ub2e4.", "paragraph_sentence": "KBO\ub294 22\uc77c\uc5d0 \uc774 \ud2b8\ub808\uc774\ub4dc\ub97c \ud569\ubc95\uc801\uc778 \ud2b8\ub808\uc774\ub4dc\ub85c \uac04\uc8fc\ud558\uace0 \uc2b9\uc778 \ucc98\ub9ac\ud558\uc600\ub2e4. KBO\ub294 \ub2e4\ub7c9\uc758 \ud2b8\ub808\uc774\ub4dc\ub97c \uc6b0\ub824\ud574 2010 \uc2dc\uc98c\uc774 \ub05d\ub0a0 \ub54c \uae4c\uc9c0 \ub125\uc13c \ud788\uc5b4\ub85c\uc988\uc758 \ud2b8\ub808\uc774\ub4dc\ub97c \uae08\uc9c0\ud558\ub294 \uac01\uc11c\ub97c \ub125\uc13c \uce21\uc5d0\uc11c \uc5bb\uc5b4\ub0b4\uc5c8\ub2e4. 7\uc6d4 24\uc77c \uc5d0 \uc5f4\ub9b4 \uc608\uc815\uc778 2010\ub144 \uc62c\uc2a4\ud0c0\uc804\uc5d0\uc11c \ub125\uc13c \ud788\uc5b4\ub85c\uc988 \uc18c\uc18d\uc73c\ub85c \uc62c\uc2a4\ud0c0\ub85c \ubf51\ud600 \uc6e8\uc2a4\ud134 \ub9ac\uadf8 \uc18c\uc18d\uc73c\ub85c \ucd9c\uc804\ud560 \uc608\uc815\uc774\uc5c8\ub358 \ud669\uc7ac\uade0\uc740 \uc774\uc2a4\ud134 \ub9ac\uadf8\uc5d0 \uc18d\ud574\uc788\ub294 \ub86f\ub370 \uc790\uc774\uc5b8\uce20\ub85c \uc774\uc801\ud568\uc5d0 \ub530\ub77c \uc62c\uc2a4\ud0c0\uc804\uc744 \uc774\uc2a4\ud134 \ub9ac\uadf8 \uc18c\uc18d\uc73c\ub85c \ucd9c\uc804\ud558\uac8c \ub418\uc5c8\ub2e4. 1\uba85\uc774 \ucd94\uac00\ub41c \uc774\uc2a4\ud134 \ub9ac\uadf8\uc5d0 \ube44\ud574 1\uba85\uc774 \ube60\uc9c4 \uc6e8\uc2a4\ud134 \ub9ac\uadf8\ub294 \uae40\uc120\ube48\uacfc \uc624\uc9c0\ud658\uc744 \ucd94\uac00\ud574 \uade0\ud615\uc744 \ub9de\ucd94\uc5c8\ub2e4. \ub86f\ub370\uc5d0 \ud569\ub958\ud574 \uc788\ub358 \ud669\uc7ac\uade0\uc740 \uadf8 \ub0a0 \ubc14\ub85c \ud55c\ud654 \uc774\uae00\uc2a4\uc640\uc758 \uacbd\uae30\uc5d0 2\ubc88 \ud0c0\uc790 3\ub8e8\uc218\ub85c \uc120\ubc1c \ucd9c\uc804\ud558\uc5ec \uc548\ud0c0\ub3c4 \uae30\ub85d\ud558\uc600\ub2e4. \ub85c\uc774\uc2a4\ud130 \uac10\ub3c5\uc740 3\ub8e8\uc218\uc640 \uc720\uaca9\uc218 \uacb8\uc5c5\uc744 \uc2dc\uc0ac\ud558\uc600\ub2e4.", "paragraph_answer": "KBO\ub294 22\uc77c\uc5d0 \uc774 \ud2b8\ub808\uc774\ub4dc\ub97c \ud569\ubc95\uc801\uc778 \ud2b8\ub808\uc774\ub4dc\ub85c \uac04\uc8fc\ud558\uace0 \uc2b9\uc778 \ucc98\ub9ac\ud558\uc600\ub2e4. KBO\ub294 \ub2e4\ub7c9\uc758 \ud2b8\ub808\uc774\ub4dc\ub97c \uc6b0\ub824\ud574 2010 \uc2dc\uc98c\uc774 \ub05d\ub0a0 \ub54c \uae4c\uc9c0 \ub125\uc13c \ud788\uc5b4\ub85c\uc988\uc758 \ud2b8\ub808\uc774\ub4dc\ub97c \uae08\uc9c0\ud558\ub294 \uac01\uc11c\ub97c \ub125\uc13c \uce21\uc5d0\uc11c \uc5bb\uc5b4\ub0b4\uc5c8\ub2e4. 7\uc6d4 24\uc77c \uc5d0 \uc5f4\ub9b4 \uc608\uc815\uc778 2010\ub144 \uc62c\uc2a4\ud0c0\uc804\uc5d0\uc11c \ub125\uc13c \ud788\uc5b4\ub85c\uc988 \uc18c\uc18d\uc73c\ub85c \uc62c\uc2a4\ud0c0\ub85c \ubf51\ud600 \uc6e8\uc2a4\ud134 \ub9ac\uadf8 \uc18c\uc18d\uc73c\ub85c \ucd9c\uc804\ud560 \uc608\uc815\uc774\uc5c8\ub358 \ud669\uc7ac\uade0\uc740 \uc774\uc2a4\ud134 \ub9ac\uadf8\uc5d0 \uc18d\ud574\uc788\ub294 \ub86f\ub370 \uc790\uc774\uc5b8\uce20\ub85c \uc774\uc801\ud568\uc5d0 \ub530\ub77c \uc62c\uc2a4\ud0c0\uc804\uc744 \uc774\uc2a4\ud134 \ub9ac\uadf8 \uc18c\uc18d\uc73c\ub85c \ucd9c\uc804\ud558\uac8c \ub418\uc5c8\ub2e4. 1\uba85\uc774 \ucd94\uac00\ub41c \uc774\uc2a4\ud134 \ub9ac\uadf8\uc5d0 \ube44\ud574 1\uba85\uc774 \ube60\uc9c4 \uc6e8\uc2a4\ud134 \ub9ac\uadf8\ub294 \uae40\uc120\ube48\uacfc \uc624\uc9c0\ud658\uc744 \ucd94\uac00\ud574 \uade0\ud615\uc744 \ub9de\ucd94\uc5c8\ub2e4. \ub86f\ub370\uc5d0 \ud569\ub958\ud574 \uc788\ub358 \ud669\uc7ac\uade0\uc740 \uadf8 \ub0a0 \ubc14\ub85c \ud55c\ud654 \uc774\uae00\uc2a4\uc640\uc758 \uacbd\uae30\uc5d0 2\ubc88 \ud0c0\uc790 3\ub8e8\uc218\ub85c \uc120\ubc1c \ucd9c\uc804\ud558\uc5ec \uc548\ud0c0\ub3c4 \uae30\ub85d\ud558\uc600\ub2e4. \ub85c\uc774\uc2a4\ud130 \uac10\ub3c5\uc740 3\ub8e8\uc218\uc640 \uc720\uaca9\uc218 \uacb8\uc5c5\uc744 \uc2dc\uc0ac\ud558\uc600\ub2e4.", "sentence_answer": " 7\uc6d4 24\uc77c \uc5d0 \uc5f4\ub9b4 \uc608\uc815\uc778 2010\ub144 \uc62c\uc2a4\ud0c0\uc804\uc5d0\uc11c \ub125\uc13c \ud788\uc5b4\ub85c\uc988 \uc18c\uc18d\uc73c\ub85c \uc62c\uc2a4\ud0c0\ub85c \ubf51\ud600 \uc6e8\uc2a4\ud134 \ub9ac\uadf8 \uc18c\uc18d\uc73c\ub85c \ucd9c\uc804\ud560 \uc608\uc815\uc774\uc5c8\ub358 \ud669\uc7ac\uade0\uc740 \uc774\uc2a4\ud134 \ub9ac\uadf8\uc5d0 \uc18d\ud574\uc788\ub294 \ub86f\ub370 \uc790\uc774\uc5b8\uce20\ub85c \uc774\uc801\ud568\uc5d0 \ub530\ub77c \uc62c\uc2a4\ud0c0\uc804\uc744 \uc774\uc2a4\ud134 \ub9ac\uadf8 \uc18c\uc18d\uc73c\ub85c \ucd9c\uc804\ud558\uac8c \ub418\uc5c8\ub2e4.", "paragraph_id": "6533929-12-0", "paragraph_question": "question: 2010\ub144 \uc62c\uc2a4\ud0c0\uc804\uc740 \uba87\uc6d4 \uba87\uc77c\uc5d0 \uc62c\ub9b4 \uc608\uc815\uc774\uc5c8\ub294\uac00?, context: KBO\ub294 22\uc77c\uc5d0 \uc774 \ud2b8\ub808\uc774\ub4dc\ub97c \ud569\ubc95\uc801\uc778 \ud2b8\ub808\uc774\ub4dc\ub85c \uac04\uc8fc\ud558\uace0 \uc2b9\uc778 \ucc98\ub9ac\ud558\uc600\ub2e4. KBO\ub294 \ub2e4\ub7c9\uc758 \ud2b8\ub808\uc774\ub4dc\ub97c \uc6b0\ub824\ud574 2010 \uc2dc\uc98c\uc774 \ub05d\ub0a0 \ub54c \uae4c\uc9c0 \ub125\uc13c \ud788\uc5b4\ub85c\uc988\uc758 \ud2b8\ub808\uc774\ub4dc\ub97c \uae08\uc9c0\ud558\ub294 \uac01\uc11c\ub97c \ub125\uc13c \uce21\uc5d0\uc11c \uc5bb\uc5b4\ub0b4\uc5c8\ub2e4. 7\uc6d4 24\uc77c\uc5d0 \uc5f4\ub9b4 \uc608\uc815\uc778 2010\ub144 \uc62c\uc2a4\ud0c0\uc804\uc5d0\uc11c \ub125\uc13c \ud788\uc5b4\ub85c\uc988 \uc18c\uc18d\uc73c\ub85c \uc62c\uc2a4\ud0c0\ub85c \ubf51\ud600 \uc6e8\uc2a4\ud134 \ub9ac\uadf8 \uc18c\uc18d\uc73c\ub85c \ucd9c\uc804\ud560 \uc608\uc815\uc774\uc5c8\ub358 \ud669\uc7ac\uade0\uc740 \uc774\uc2a4\ud134 \ub9ac\uadf8\uc5d0 \uc18d\ud574\uc788\ub294 \ub86f\ub370 \uc790\uc774\uc5b8\uce20\ub85c \uc774\uc801\ud568\uc5d0 \ub530\ub77c \uc62c\uc2a4\ud0c0\uc804\uc744 \uc774\uc2a4\ud134 \ub9ac\uadf8 \uc18c\uc18d\uc73c\ub85c \ucd9c\uc804\ud558\uac8c \ub418\uc5c8\ub2e4. 1\uba85\uc774 \ucd94\uac00\ub41c \uc774\uc2a4\ud134 \ub9ac\uadf8\uc5d0 \ube44\ud574 1\uba85\uc774 \ube60\uc9c4 \uc6e8\uc2a4\ud134 \ub9ac\uadf8\ub294 \uae40\uc120\ube48\uacfc \uc624\uc9c0\ud658\uc744 \ucd94\uac00\ud574 \uade0\ud615\uc744 \ub9de\ucd94\uc5c8\ub2e4. \ub86f\ub370\uc5d0 \ud569\ub958\ud574 \uc788\ub358 \ud669\uc7ac\uade0\uc740 \uadf8 \ub0a0 \ubc14\ub85c \ud55c\ud654 \uc774\uae00\uc2a4\uc640\uc758 \uacbd\uae30\uc5d0 2\ubc88 \ud0c0\uc790 3\ub8e8\uc218\ub85c \uc120\ubc1c \ucd9c\uc804\ud558\uc5ec \uc548\ud0c0\ub3c4 \uae30\ub85d\ud558\uc600\ub2e4. \ub85c\uc774\uc2a4\ud130 \uac10\ub3c5\uc740 3\ub8e8\uc218\uc640 \uc720\uaca9\uc218 \uacb8\uc5c5\uc744 \uc2dc\uc0ac\ud558\uc600\ub2e4."} {"question": "\ucd1d 32\uba85\uc774 \ud50c\ub808\uc774\ub97c \ud560 \uc218 \uc788\ub294 \ud604\uc874 \uac00\uc7a5 \ud68d\uae30\uc801\uc778 \uc791\ud488\uc774\ub77c \ud3c9\uac00\ubc1b\ub294 \uc628\ub77c\uc778 \uac8c\uc784\uc740 \ubb34\uc5c7\uc778\uac00?", "paragraph": "\ud604\uc874\ud558\ub294 \uc628\ub77c\uc778 \uac8c\uc784 \uc911 \uac00\uc7a5 \ud68d\uae30\uc801\uc778 \uc791\ud488\uc73c\ub85c \ud3c9\uac00\ubc1b\ub294 \ud018\uc774\ud06c\ub294 \uc5f4 \uc5ec\uc12f \uba85\uc758 \ud50c\ub808\uc774\uc5b4\uc640 \ud300\uc744 \uc774\ub8f0 \uc218 \uc788\uc73c\uba70, \ucd1d \uc11c\ub978 \ub450\uba85\uc758 \ud50c\ub808\uc774\uc5b4\uac00 \ub3d9\uc2dc\uc5d0 \uc0c1\ud638\uc791\uc6a9 \ud560 \uc218 \uc788\ub294 \uac8c\uc784\uc774\uae30\ub3c4 \ud558\uc600\ub2e4. \uc774\uc5d0 \ub530\ub77c, \uac8c\uc774\uba38\ub4e4\uc740 \ud074\ub79c\uc774\ub77c\ub294 \uc790\uc2e0\ub4e4\ub9cc\uc758 \uc9d1\ub2e8\uc744 \ud615\uc131\ud558\uae30 \uc2dc\uc791\ud588\ub2e4. \uac01\uac01\uc758 \ud074\ub79c\ub4e4\uc740 \uc790\uc2e0\ub4e4\ub9cc\uc758 \uc815\uccb4\uc131\uc744 \ub9cc\ub4e4\uc5b4\ub0b4\uace0 \ud074\ub79c\uc744 \ud64d\ubcf4\ud558\uae30\ub3c4 \ud558\uba70, \ud074\ub79c \ub0b4\ubd80\uc758 \uccb4\uacc4\ub97c \uad6c\uc131\ud560 \ubfd0\ub9cc \uc544\ub2c8\ub77c \ud074\ub79c\uc744 \uc0c1\uc9d5\ud558\ub294 \ud45c\uc2dc\ub97c \uac89\uc73c\ub85c \ub4dc\ub7ec\ub0b4\uae30\ub3c4 \ud55c\ub2e4. \ud074\ub79c\ub4e4 \ub07c\ub9ac\ub294 \uad50\ub958\ub97c \ub9fa\uae30\ub3c4 \ud558\uba70 \uc801\ub300\uc801\uc778 \uad00\uacc4\ub97c \ud615\uc131\ud558\uae30\ub3c4, \uadf8\ub9ac\uace0 \ub3d9\ub9f9\uc758 \uad00\uacc4\ub97c \ub9fa\ub294 \uacbd\uc6b0\ub3c4 \uc788\ub2e4. \ud074\ub79c\uc6d0\ub4e4 \uac04\uc758 \uc0c1\ud638\uc791\uc6a9\uc774 \uc0dd\uae30\ub294 \uacbd\uc6b0\ub294 \uc8fc\ub85c \uacf5\uc2dd\uc801\uc778 \uacbd\uc7c1\uc774 \ubc1c\uc0dd\ud558\ub294 \uacbd\uae30, \ud639\uc740 \ud074\ub79c\uc6d0\ub4e4\ub07c\ub9ac \uacf5\uac1c \uc11c\ubc84\uc5d0\uc11c \ud568\uaed8 \ubaa8\uc5ec \uc77c\ubc18\uc801\uc778 \ud50c\ub808\uc774\ub97c \uc990\uae30\ub294 \uacbd\uc6b0 \ub4f1\uc774 \uc788\ub2e4 . \ud074\ub79c\uc774 \uad6c\uc131\uc6d0\uc744 \ubaa8\uc9d1\ud558\ub294 \ubc29\uc2dd\uc73c\ub85c\ub294 \ud2b9\uc815 \uc11c\ubc84\uc5d0 \ud65c\ub3d9\ud558\ub294 \ub6f0\uc5b4\ub09c \uc120\uc218\ub4e4\uc744 \ucd08\ub300\ud558\uc5ec \uc785\ub2e8\uc744 \uc704\ud55c \uba74\uc811\uc744 \ubcf4\uac8c \ud558\uac70\ub098 \uc9c1\uc811 \uc785\ub2e8\uc744 \uad8c\uc720 \ud558\uae30\ub3c4 \ud55c\ub2e4.", "answer": "\ud018\uc774\ud06c", "sentence": "\ud604\uc874\ud558\ub294 \uc628\ub77c\uc778 \uac8c\uc784 \uc911 \uac00\uc7a5 \ud68d\uae30\uc801\uc778 \uc791\ud488\uc73c\ub85c \ud3c9\uac00\ubc1b\ub294 \ud018\uc774\ud06c \ub294 \uc5f4 \uc5ec\uc12f \uba85\uc758 \ud50c\ub808\uc774\uc5b4\uc640 \ud300\uc744 \uc774\ub8f0 \uc218 \uc788\uc73c\uba70, \ucd1d \uc11c\ub978 \ub450\uba85\uc758 \ud50c\ub808\uc774\uc5b4\uac00 \ub3d9\uc2dc\uc5d0 \uc0c1\ud638\uc791\uc6a9 \ud560 \uc218 \uc788\ub294 \uac8c\uc784\uc774\uae30\ub3c4 \ud558\uc600\ub2e4.", "paragraph_sentence": " \ud604\uc874\ud558\ub294 \uc628\ub77c\uc778 \uac8c\uc784 \uc911 \uac00\uc7a5 \ud68d\uae30\uc801\uc778 \uc791\ud488\uc73c\ub85c \ud3c9\uac00\ubc1b\ub294 \ud018\uc774\ud06c \ub294 \uc5f4 \uc5ec\uc12f \uba85\uc758 \ud50c\ub808\uc774\uc5b4\uc640 \ud300\uc744 \uc774\ub8f0 \uc218 \uc788\uc73c\uba70, \ucd1d \uc11c\ub978 \ub450\uba85\uc758 \ud50c\ub808\uc774\uc5b4\uac00 \ub3d9\uc2dc\uc5d0 \uc0c1\ud638\uc791\uc6a9 \ud560 \uc218 \uc788\ub294 \uac8c\uc784\uc774\uae30\ub3c4 \ud558\uc600\ub2e4. \uc774\uc5d0 \ub530\ub77c, \uac8c\uc774\uba38\ub4e4\uc740 \ud074\ub79c\uc774\ub77c\ub294 \uc790\uc2e0\ub4e4\ub9cc\uc758 \uc9d1\ub2e8\uc744 \ud615\uc131\ud558\uae30 \uc2dc\uc791\ud588\ub2e4. \uac01\uac01\uc758 \ud074\ub79c\ub4e4\uc740 \uc790\uc2e0\ub4e4\ub9cc\uc758 \uc815\uccb4\uc131\uc744 \ub9cc\ub4e4\uc5b4\ub0b4\uace0 \ud074\ub79c\uc744 \ud64d\ubcf4\ud558\uae30\ub3c4 \ud558\uba70, \ud074\ub79c \ub0b4\ubd80\uc758 \uccb4\uacc4\ub97c \uad6c\uc131\ud560 \ubfd0\ub9cc \uc544\ub2c8\ub77c \ud074\ub79c\uc744 \uc0c1\uc9d5\ud558\ub294 \ud45c\uc2dc\ub97c \uac89\uc73c\ub85c \ub4dc\ub7ec\ub0b4\uae30\ub3c4 \ud55c\ub2e4. \ud074\ub79c\ub4e4 \ub07c\ub9ac\ub294 \uad50\ub958\ub97c \ub9fa\uae30\ub3c4 \ud558\uba70 \uc801\ub300\uc801\uc778 \uad00\uacc4\ub97c \ud615\uc131\ud558\uae30\ub3c4, \uadf8\ub9ac\uace0 \ub3d9\ub9f9\uc758 \uad00\uacc4\ub97c \ub9fa\ub294 \uacbd\uc6b0\ub3c4 \uc788\ub2e4. \ud074\ub79c\uc6d0\ub4e4 \uac04\uc758 \uc0c1\ud638\uc791\uc6a9\uc774 \uc0dd\uae30\ub294 \uacbd\uc6b0\ub294 \uc8fc\ub85c \uacf5\uc2dd\uc801\uc778 \uacbd\uc7c1\uc774 \ubc1c\uc0dd\ud558\ub294 \uacbd\uae30, \ud639\uc740 \ud074\ub79c\uc6d0\ub4e4\ub07c\ub9ac \uacf5\uac1c \uc11c\ubc84\uc5d0\uc11c \ud568\uaed8 \ubaa8\uc5ec \uc77c\ubc18\uc801\uc778 \ud50c\ub808\uc774\ub97c \uc990\uae30\ub294 \uacbd\uc6b0 \ub4f1\uc774 \uc788\ub2e4 . \ud074\ub79c\uc774 \uad6c\uc131\uc6d0\uc744 \ubaa8\uc9d1\ud558\ub294 \ubc29\uc2dd\uc73c\ub85c\ub294 \ud2b9\uc815 \uc11c\ubc84\uc5d0 \ud65c\ub3d9\ud558\ub294 \ub6f0\uc5b4\ub09c \uc120\uc218\ub4e4\uc744 \ucd08\ub300\ud558\uc5ec \uc785\ub2e8\uc744 \uc704\ud55c \uba74\uc811\uc744 \ubcf4\uac8c \ud558\uac70\ub098 \uc9c1\uc811 \uc785\ub2e8\uc744 \uad8c\uc720 \ud558\uae30\ub3c4 \ud55c\ub2e4.", "paragraph_answer": "\ud604\uc874\ud558\ub294 \uc628\ub77c\uc778 \uac8c\uc784 \uc911 \uac00\uc7a5 \ud68d\uae30\uc801\uc778 \uc791\ud488\uc73c\ub85c \ud3c9\uac00\ubc1b\ub294 \ud018\uc774\ud06c \ub294 \uc5f4 \uc5ec\uc12f \uba85\uc758 \ud50c\ub808\uc774\uc5b4\uc640 \ud300\uc744 \uc774\ub8f0 \uc218 \uc788\uc73c\uba70, \ucd1d \uc11c\ub978 \ub450\uba85\uc758 \ud50c\ub808\uc774\uc5b4\uac00 \ub3d9\uc2dc\uc5d0 \uc0c1\ud638\uc791\uc6a9 \ud560 \uc218 \uc788\ub294 \uac8c\uc784\uc774\uae30\ub3c4 \ud558\uc600\ub2e4. \uc774\uc5d0 \ub530\ub77c, \uac8c\uc774\uba38\ub4e4\uc740 \ud074\ub79c\uc774\ub77c\ub294 \uc790\uc2e0\ub4e4\ub9cc\uc758 \uc9d1\ub2e8\uc744 \ud615\uc131\ud558\uae30 \uc2dc\uc791\ud588\ub2e4. \uac01\uac01\uc758 \ud074\ub79c\ub4e4\uc740 \uc790\uc2e0\ub4e4\ub9cc\uc758 \uc815\uccb4\uc131\uc744 \ub9cc\ub4e4\uc5b4\ub0b4\uace0 \ud074\ub79c\uc744 \ud64d\ubcf4\ud558\uae30\ub3c4 \ud558\uba70, \ud074\ub79c \ub0b4\ubd80\uc758 \uccb4\uacc4\ub97c \uad6c\uc131\ud560 \ubfd0\ub9cc \uc544\ub2c8\ub77c \ud074\ub79c\uc744 \uc0c1\uc9d5\ud558\ub294 \ud45c\uc2dc\ub97c \uac89\uc73c\ub85c \ub4dc\ub7ec\ub0b4\uae30\ub3c4 \ud55c\ub2e4. \ud074\ub79c\ub4e4 \ub07c\ub9ac\ub294 \uad50\ub958\ub97c \ub9fa\uae30\ub3c4 \ud558\uba70 \uc801\ub300\uc801\uc778 \uad00\uacc4\ub97c \ud615\uc131\ud558\uae30\ub3c4, \uadf8\ub9ac\uace0 \ub3d9\ub9f9\uc758 \uad00\uacc4\ub97c \ub9fa\ub294 \uacbd\uc6b0\ub3c4 \uc788\ub2e4. \ud074\ub79c\uc6d0\ub4e4 \uac04\uc758 \uc0c1\ud638\uc791\uc6a9\uc774 \uc0dd\uae30\ub294 \uacbd\uc6b0\ub294 \uc8fc\ub85c \uacf5\uc2dd\uc801\uc778 \uacbd\uc7c1\uc774 \ubc1c\uc0dd\ud558\ub294 \uacbd\uae30, \ud639\uc740 \ud074\ub79c\uc6d0\ub4e4\ub07c\ub9ac \uacf5\uac1c \uc11c\ubc84\uc5d0\uc11c \ud568\uaed8 \ubaa8\uc5ec \uc77c\ubc18\uc801\uc778 \ud50c\ub808\uc774\ub97c \uc990\uae30\ub294 \uacbd\uc6b0 \ub4f1\uc774 \uc788\ub2e4 . \ud074\ub79c\uc774 \uad6c\uc131\uc6d0\uc744 \ubaa8\uc9d1\ud558\ub294 \ubc29\uc2dd\uc73c\ub85c\ub294 \ud2b9\uc815 \uc11c\ubc84\uc5d0 \ud65c\ub3d9\ud558\ub294 \ub6f0\uc5b4\ub09c \uc120\uc218\ub4e4\uc744 \ucd08\ub300\ud558\uc5ec \uc785\ub2e8\uc744 \uc704\ud55c \uba74\uc811\uc744 \ubcf4\uac8c \ud558\uac70\ub098 \uc9c1\uc811 \uc785\ub2e8\uc744 \uad8c\uc720 \ud558\uae30\ub3c4 \ud55c\ub2e4.", "sentence_answer": "\ud604\uc874\ud558\ub294 \uc628\ub77c\uc778 \uac8c\uc784 \uc911 \uac00\uc7a5 \ud68d\uae30\uc801\uc778 \uc791\ud488\uc73c\ub85c \ud3c9\uac00\ubc1b\ub294 \ud018\uc774\ud06c \ub294 \uc5f4 \uc5ec\uc12f \uba85\uc758 \ud50c\ub808\uc774\uc5b4\uc640 \ud300\uc744 \uc774\ub8f0 \uc218 \uc788\uc73c\uba70, \ucd1d \uc11c\ub978 \ub450\uba85\uc758 \ud50c\ub808\uc774\uc5b4\uac00 \ub3d9\uc2dc\uc5d0 \uc0c1\ud638\uc791\uc6a9 \ud560 \uc218 \uc788\ub294 \uac8c\uc784\uc774\uae30\ub3c4 \ud558\uc600\ub2e4.", "paragraph_id": "6535282-3-0", "paragraph_question": "question: \ucd1d 32\uba85\uc774 \ud50c\ub808\uc774\ub97c \ud560 \uc218 \uc788\ub294 \ud604\uc874 \uac00\uc7a5 \ud68d\uae30\uc801\uc778 \uc791\ud488\uc774\ub77c \ud3c9\uac00\ubc1b\ub294 \uc628\ub77c\uc778 \uac8c\uc784\uc740 \ubb34\uc5c7\uc778\uac00?, context: \ud604\uc874\ud558\ub294 \uc628\ub77c\uc778 \uac8c\uc784 \uc911 \uac00\uc7a5 \ud68d\uae30\uc801\uc778 \uc791\ud488\uc73c\ub85c \ud3c9\uac00\ubc1b\ub294 \ud018\uc774\ud06c\ub294 \uc5f4 \uc5ec\uc12f \uba85\uc758 \ud50c\ub808\uc774\uc5b4\uc640 \ud300\uc744 \uc774\ub8f0 \uc218 \uc788\uc73c\uba70, \ucd1d \uc11c\ub978 \ub450\uba85\uc758 \ud50c\ub808\uc774\uc5b4\uac00 \ub3d9\uc2dc\uc5d0 \uc0c1\ud638\uc791\uc6a9 \ud560 \uc218 \uc788\ub294 \uac8c\uc784\uc774\uae30\ub3c4 \ud558\uc600\ub2e4. \uc774\uc5d0 \ub530\ub77c, \uac8c\uc774\uba38\ub4e4\uc740 \ud074\ub79c\uc774\ub77c\ub294 \uc790\uc2e0\ub4e4\ub9cc\uc758 \uc9d1\ub2e8\uc744 \ud615\uc131\ud558\uae30 \uc2dc\uc791\ud588\ub2e4. \uac01\uac01\uc758 \ud074\ub79c\ub4e4\uc740 \uc790\uc2e0\ub4e4\ub9cc\uc758 \uc815\uccb4\uc131\uc744 \ub9cc\ub4e4\uc5b4\ub0b4\uace0 \ud074\ub79c\uc744 \ud64d\ubcf4\ud558\uae30\ub3c4 \ud558\uba70, \ud074\ub79c \ub0b4\ubd80\uc758 \uccb4\uacc4\ub97c \uad6c\uc131\ud560 \ubfd0\ub9cc \uc544\ub2c8\ub77c \ud074\ub79c\uc744 \uc0c1\uc9d5\ud558\ub294 \ud45c\uc2dc\ub97c \uac89\uc73c\ub85c \ub4dc\ub7ec\ub0b4\uae30\ub3c4 \ud55c\ub2e4. \ud074\ub79c\ub4e4 \ub07c\ub9ac\ub294 \uad50\ub958\ub97c \ub9fa\uae30\ub3c4 \ud558\uba70 \uc801\ub300\uc801\uc778 \uad00\uacc4\ub97c \ud615\uc131\ud558\uae30\ub3c4, \uadf8\ub9ac\uace0 \ub3d9\ub9f9\uc758 \uad00\uacc4\ub97c \ub9fa\ub294 \uacbd\uc6b0\ub3c4 \uc788\ub2e4. \ud074\ub79c\uc6d0\ub4e4 \uac04\uc758 \uc0c1\ud638\uc791\uc6a9\uc774 \uc0dd\uae30\ub294 \uacbd\uc6b0\ub294 \uc8fc\ub85c \uacf5\uc2dd\uc801\uc778 \uacbd\uc7c1\uc774 \ubc1c\uc0dd\ud558\ub294 \uacbd\uae30, \ud639\uc740 \ud074\ub79c\uc6d0\ub4e4\ub07c\ub9ac \uacf5\uac1c \uc11c\ubc84\uc5d0\uc11c \ud568\uaed8 \ubaa8\uc5ec \uc77c\ubc18\uc801\uc778 \ud50c\ub808\uc774\ub97c \uc990\uae30\ub294 \uacbd\uc6b0 \ub4f1\uc774 \uc788\ub2e4 . \ud074\ub79c\uc774 \uad6c\uc131\uc6d0\uc744 \ubaa8\uc9d1\ud558\ub294 \ubc29\uc2dd\uc73c\ub85c\ub294 \ud2b9\uc815 \uc11c\ubc84\uc5d0 \ud65c\ub3d9\ud558\ub294 \ub6f0\uc5b4\ub09c \uc120\uc218\ub4e4\uc744 \ucd08\ub300\ud558\uc5ec \uc785\ub2e8\uc744 \uc704\ud55c \uba74\uc811\uc744 \ubcf4\uac8c \ud558\uac70\ub098 \uc9c1\uc811 \uc785\ub2e8\uc744 \uad8c\uc720 \ud558\uae30\ub3c4 \ud55c\ub2e4."} {"question": "\uae40\uace0\uc740\uc774 \uc911\uad6d\uc73c\ub85c \uc774\ub3d9\ud55c \ub54c \uba87 \uc0b4\uc774\uc5c8\ub098?", "paragraph": "\uae40\uace0\uc740\uc740 1991\ub144 7\uc6d4 2\uc77c \uc11c\uc6b8\uc5d0\uc11c 1\ub0a8 1\ub140 \uc911 \ub458\uc9f8\ub85c \ud0dc\uc5b4\ub0ac\ub2e4. \uc624\ube60\uc640\ub294 \uc5f0\ub144\uc0dd\uc774\ub2e4. \u2018\uace0\uc740\u2019\uc774\ub77c\ub294 \uc774\ub984\uc740 \uc774\ub984\ucc98\ub7fc \uacf1\uac8c \uc790\ub77c\uae38 \ubc14\ub77c\ub294 \ub9c8\uc74c\uc73c\ub85c \uadf8\ub140\uc758 \uc544\ubc84\uc9c0\uac00 \uc9c0\uc5b4\uc900 \uc774\ub984\uc774\ub77c\uace0 \ud55c\ub2e4. 1994\ub144 \uae40\uace0\uc740\uc774 4\uc0b4\uc774 \ub418\ub358 \ud574 \uc911\uad6d\uc5d0\uc11c \uc0ac\uc5c5\uc744 \ud558\ub294 \uc544\ubc84\uc9c0\ub97c \ub530\ub77c \uac00\uc871\uc774 \uc911\uad6d\uc73c\ub85c \uc774\ub3d9\ud558\uba74\uc11c \uadf8 \uacf3\uc5d0\uc11c 14\uc138\ub54c\uae4c\uc9c0 10\ub144\uac04 \uc0b4\ub2e4\uac00 \uc911\ud559\uad50 1\ud559\ub144 \ub54c \uadc0\uad6d\ud588\ub2e4. \uadf8\ub140\ub294 \ubca0\uc774\uc9d5\uc5d0\uc11c 5\ub144, \ubca0\uc774\uc9d5\uc5d0\uc11c \ucc28\ub85c 1\uc2dc\uac04 30\ubd84\uac00\ub7c9 \ub5a8\uc5b4\uc9c4 \ub3c4\uc2dc\uc5d0\uc11c 5\ub144\uc744 \ubcf4\ub0c8\uace0, \uc870\uae30 \uad50\uc721\uc73c\ub85c \uc778\ud574 \uc911\uad6d\uc5b4\uc640 \uc911\uad6d\ubb38\ud654, \uc815\uc11c\uc5d0 \uc775\uc219\ud558\ub2e4. \uc774 \uc601\ud5a5\uc73c\ub85c \uc911\uad6d\uc5b4\uc5d0\ub3c4 \ub9e4\uc6b0 \ub2a5\ud1b5\ud558\ub2e4\uace0 \ud55c\ub2e4. \uae40\uace0\uc740\uc758 \uc911\uad6d\uc5b4 \uc2e4\ub825\uc740 \uc9c0\ub09c 2014\ub144 \ud64d\ucf69\uc5d0\uc11c \uc5f4\ub9b0 \uc544\uc2dc\uc548 \ud544\ub984 \uc5b4\uc6cc\uc988\uc5d0\uc11c \ud64d\ucf69\uc758 \uc778\uae30 \ubc30\uc6b0 \uacb8 \ud504\ub85c\ub4c0\uc11c \uc784\uac00\ub3d9\uacfc \ud568\uaed8 \uc2dc\uc0c1\uc790\ub85c \ub098\uc11c \uaf64 \uae34 \uc2dc\uac04 \uc911\uad6d\uc5b4\ub85c \ub300\ud654\ub97c \ub098\ub204\uba70 \uc218\uc0c1\uc790\uc5d0\uac8c \uc0c1\uc744 \uac74\ub124\ub294 \ubaa8\uc2b5\uc744 \ud1b5\ud574 \uc54c\ub824\uc84c\ub2e4.", "answer": "4\uc0b4", "sentence": "1994\ub144 \uae40\uace0\uc740\uc774 4\uc0b4 \uc774 \ub418\ub358 \ud574 \uc911\uad6d\uc5d0\uc11c \uc0ac\uc5c5\uc744 \ud558\ub294 \uc544\ubc84\uc9c0\ub97c \ub530\ub77c \uac00\uc871\uc774 \uc911\uad6d\uc73c\ub85c \uc774\ub3d9\ud558\uba74\uc11c \uadf8 \uacf3\uc5d0\uc11c 14\uc138\ub54c\uae4c\uc9c0 10\ub144\uac04 \uc0b4\ub2e4\uac00 \uc911\ud559\uad50 1\ud559\ub144 \ub54c \uadc0\uad6d\ud588\ub2e4.", "paragraph_sentence": "\uae40\uace0\uc740\uc740 1991\ub144 7\uc6d4 2\uc77c \uc11c\uc6b8\uc5d0\uc11c 1\ub0a8 1\ub140 \uc911 \ub458\uc9f8\ub85c \ud0dc\uc5b4\ub0ac\ub2e4. \uc624\ube60\uc640\ub294 \uc5f0\ub144\uc0dd\uc774\ub2e4. \u2018\uace0\uc740\u2019\uc774\ub77c\ub294 \uc774\ub984\uc740 \uc774\ub984\ucc98\ub7fc \uacf1\uac8c \uc790\ub77c\uae38 \ubc14\ub77c\ub294 \ub9c8\uc74c\uc73c\ub85c \uadf8\ub140\uc758 \uc544\ubc84\uc9c0\uac00 \uc9c0\uc5b4\uc900 \uc774\ub984\uc774\ub77c\uace0 \ud55c\ub2e4. 1994\ub144 \uae40\uace0\uc740\uc774 4\uc0b4 \uc774 \ub418\ub358 \ud574 \uc911\uad6d\uc5d0\uc11c \uc0ac\uc5c5\uc744 \ud558\ub294 \uc544\ubc84\uc9c0\ub97c \ub530\ub77c \uac00\uc871\uc774 \uc911\uad6d\uc73c\ub85c \uc774\ub3d9\ud558\uba74\uc11c \uadf8 \uacf3\uc5d0\uc11c 14\uc138\ub54c\uae4c\uc9c0 10\ub144\uac04 \uc0b4\ub2e4\uac00 \uc911\ud559\uad50 1\ud559\ub144 \ub54c \uadc0\uad6d\ud588\ub2e4. \uadf8\ub140\ub294 \ubca0\uc774\uc9d5\uc5d0\uc11c 5\ub144, \ubca0\uc774\uc9d5\uc5d0\uc11c \ucc28\ub85c 1\uc2dc\uac04 30\ubd84\uac00\ub7c9 \ub5a8\uc5b4\uc9c4 \ub3c4\uc2dc\uc5d0\uc11c 5\ub144\uc744 \ubcf4\ub0c8\uace0, \uc870\uae30 \uad50\uc721\uc73c\ub85c \uc778\ud574 \uc911\uad6d\uc5b4\uc640 \uc911\uad6d\ubb38\ud654, \uc815\uc11c\uc5d0 \uc775\uc219\ud558\ub2e4. \uc774 \uc601\ud5a5\uc73c\ub85c \uc911\uad6d\uc5b4\uc5d0\ub3c4 \ub9e4\uc6b0 \ub2a5\ud1b5\ud558\ub2e4\uace0 \ud55c\ub2e4. \uae40\uace0\uc740\uc758 \uc911\uad6d\uc5b4 \uc2e4\ub825\uc740 \uc9c0\ub09c 2014\ub144 \ud64d\ucf69\uc5d0\uc11c \uc5f4\ub9b0 \uc544\uc2dc\uc548 \ud544\ub984 \uc5b4\uc6cc\uc988\uc5d0\uc11c \ud64d\ucf69\uc758 \uc778\uae30 \ubc30\uc6b0 \uacb8 \ud504\ub85c\ub4c0\uc11c \uc784\uac00\ub3d9\uacfc \ud568\uaed8 \uc2dc\uc0c1\uc790\ub85c \ub098\uc11c \uaf64 \uae34 \uc2dc\uac04 \uc911\uad6d\uc5b4\ub85c \ub300\ud654\ub97c \ub098\ub204\uba70 \uc218\uc0c1\uc790\uc5d0\uac8c \uc0c1\uc744 \uac74\ub124\ub294 \ubaa8\uc2b5\uc744 \ud1b5\ud574 \uc54c\ub824\uc84c\ub2e4.", "paragraph_answer": "\uae40\uace0\uc740\uc740 1991\ub144 7\uc6d4 2\uc77c \uc11c\uc6b8\uc5d0\uc11c 1\ub0a8 1\ub140 \uc911 \ub458\uc9f8\ub85c \ud0dc\uc5b4\ub0ac\ub2e4. \uc624\ube60\uc640\ub294 \uc5f0\ub144\uc0dd\uc774\ub2e4. \u2018\uace0\uc740\u2019\uc774\ub77c\ub294 \uc774\ub984\uc740 \uc774\ub984\ucc98\ub7fc \uacf1\uac8c \uc790\ub77c\uae38 \ubc14\ub77c\ub294 \ub9c8\uc74c\uc73c\ub85c \uadf8\ub140\uc758 \uc544\ubc84\uc9c0\uac00 \uc9c0\uc5b4\uc900 \uc774\ub984\uc774\ub77c\uace0 \ud55c\ub2e4. 1994\ub144 \uae40\uace0\uc740\uc774 4\uc0b4 \uc774 \ub418\ub358 \ud574 \uc911\uad6d\uc5d0\uc11c \uc0ac\uc5c5\uc744 \ud558\ub294 \uc544\ubc84\uc9c0\ub97c \ub530\ub77c \uac00\uc871\uc774 \uc911\uad6d\uc73c\ub85c \uc774\ub3d9\ud558\uba74\uc11c \uadf8 \uacf3\uc5d0\uc11c 14\uc138\ub54c\uae4c\uc9c0 10\ub144\uac04 \uc0b4\ub2e4\uac00 \uc911\ud559\uad50 1\ud559\ub144 \ub54c \uadc0\uad6d\ud588\ub2e4. \uadf8\ub140\ub294 \ubca0\uc774\uc9d5\uc5d0\uc11c 5\ub144, \ubca0\uc774\uc9d5\uc5d0\uc11c \ucc28\ub85c 1\uc2dc\uac04 30\ubd84\uac00\ub7c9 \ub5a8\uc5b4\uc9c4 \ub3c4\uc2dc\uc5d0\uc11c 5\ub144\uc744 \ubcf4\ub0c8\uace0, \uc870\uae30 \uad50\uc721\uc73c\ub85c \uc778\ud574 \uc911\uad6d\uc5b4\uc640 \uc911\uad6d\ubb38\ud654, \uc815\uc11c\uc5d0 \uc775\uc219\ud558\ub2e4. \uc774 \uc601\ud5a5\uc73c\ub85c \uc911\uad6d\uc5b4\uc5d0\ub3c4 \ub9e4\uc6b0 \ub2a5\ud1b5\ud558\ub2e4\uace0 \ud55c\ub2e4. \uae40\uace0\uc740\uc758 \uc911\uad6d\uc5b4 \uc2e4\ub825\uc740 \uc9c0\ub09c 2014\ub144 \ud64d\ucf69\uc5d0\uc11c \uc5f4\ub9b0 \uc544\uc2dc\uc548 \ud544\ub984 \uc5b4\uc6cc\uc988\uc5d0\uc11c \ud64d\ucf69\uc758 \uc778\uae30 \ubc30\uc6b0 \uacb8 \ud504\ub85c\ub4c0\uc11c \uc784\uac00\ub3d9\uacfc \ud568\uaed8 \uc2dc\uc0c1\uc790\ub85c \ub098\uc11c \uaf64 \uae34 \uc2dc\uac04 \uc911\uad6d\uc5b4\ub85c \ub300\ud654\ub97c \ub098\ub204\uba70 \uc218\uc0c1\uc790\uc5d0\uac8c \uc0c1\uc744 \uac74\ub124\ub294 \ubaa8\uc2b5\uc744 \ud1b5\ud574 \uc54c\ub824\uc84c\ub2e4.", "sentence_answer": "1994\ub144 \uae40\uace0\uc740\uc774 4\uc0b4 \uc774 \ub418\ub358 \ud574 \uc911\uad6d\uc5d0\uc11c \uc0ac\uc5c5\uc744 \ud558\ub294 \uc544\ubc84\uc9c0\ub97c \ub530\ub77c \uac00\uc871\uc774 \uc911\uad6d\uc73c\ub85c \uc774\ub3d9\ud558\uba74\uc11c \uadf8 \uacf3\uc5d0\uc11c 14\uc138\ub54c\uae4c\uc9c0 10\ub144\uac04 \uc0b4\ub2e4\uac00 \uc911\ud559\uad50 1\ud559\ub144 \ub54c \uadc0\uad6d\ud588\ub2e4.", "paragraph_id": "6481144-0-2", "paragraph_question": "question: \uae40\uace0\uc740\uc774 \uc911\uad6d\uc73c\ub85c \uc774\ub3d9\ud55c \ub54c \uba87 \uc0b4\uc774\uc5c8\ub098?, context: \uae40\uace0\uc740\uc740 1991\ub144 7\uc6d4 2\uc77c \uc11c\uc6b8\uc5d0\uc11c 1\ub0a8 1\ub140 \uc911 \ub458\uc9f8\ub85c \ud0dc\uc5b4\ub0ac\ub2e4. \uc624\ube60\uc640\ub294 \uc5f0\ub144\uc0dd\uc774\ub2e4. \u2018\uace0\uc740\u2019\uc774\ub77c\ub294 \uc774\ub984\uc740 \uc774\ub984\ucc98\ub7fc \uacf1\uac8c \uc790\ub77c\uae38 \ubc14\ub77c\ub294 \ub9c8\uc74c\uc73c\ub85c \uadf8\ub140\uc758 \uc544\ubc84\uc9c0\uac00 \uc9c0\uc5b4\uc900 \uc774\ub984\uc774\ub77c\uace0 \ud55c\ub2e4. 1994\ub144 \uae40\uace0\uc740\uc774 4\uc0b4\uc774 \ub418\ub358 \ud574 \uc911\uad6d\uc5d0\uc11c \uc0ac\uc5c5\uc744 \ud558\ub294 \uc544\ubc84\uc9c0\ub97c \ub530\ub77c \uac00\uc871\uc774 \uc911\uad6d\uc73c\ub85c \uc774\ub3d9\ud558\uba74\uc11c \uadf8 \uacf3\uc5d0\uc11c 14\uc138\ub54c\uae4c\uc9c0 10\ub144\uac04 \uc0b4\ub2e4\uac00 \uc911\ud559\uad50 1\ud559\ub144 \ub54c \uadc0\uad6d\ud588\ub2e4. \uadf8\ub140\ub294 \ubca0\uc774\uc9d5\uc5d0\uc11c 5\ub144, \ubca0\uc774\uc9d5\uc5d0\uc11c \ucc28\ub85c 1\uc2dc\uac04 30\ubd84\uac00\ub7c9 \ub5a8\uc5b4\uc9c4 \ub3c4\uc2dc\uc5d0\uc11c 5\ub144\uc744 \ubcf4\ub0c8\uace0, \uc870\uae30 \uad50\uc721\uc73c\ub85c \uc778\ud574 \uc911\uad6d\uc5b4\uc640 \uc911\uad6d\ubb38\ud654, \uc815\uc11c\uc5d0 \uc775\uc219\ud558\ub2e4. \uc774 \uc601\ud5a5\uc73c\ub85c \uc911\uad6d\uc5b4\uc5d0\ub3c4 \ub9e4\uc6b0 \ub2a5\ud1b5\ud558\ub2e4\uace0 \ud55c\ub2e4. \uae40\uace0\uc740\uc758 \uc911\uad6d\uc5b4 \uc2e4\ub825\uc740 \uc9c0\ub09c 2014\ub144 \ud64d\ucf69\uc5d0\uc11c \uc5f4\ub9b0 \uc544\uc2dc\uc548 \ud544\ub984 \uc5b4\uc6cc\uc988\uc5d0\uc11c \ud64d\ucf69\uc758 \uc778\uae30 \ubc30\uc6b0 \uacb8 \ud504\ub85c\ub4c0\uc11c \uc784\uac00\ub3d9\uacfc \ud568\uaed8 \uc2dc\uc0c1\uc790\ub85c \ub098\uc11c \uaf64 \uae34 \uc2dc\uac04 \uc911\uad6d\uc5b4\ub85c \ub300\ud654\ub97c \ub098\ub204\uba70 \uc218\uc0c1\uc790\uc5d0\uac8c \uc0c1\uc744 \uac74\ub124\ub294 \ubaa8\uc2b5\uc744 \ud1b5\ud574 \uc54c\ub824\uc84c\ub2e4."} {"question": "\uc9c0\uc911\ud574 \ub3d9\ubd80\uc5d0\uc11c \ud0a4\ud504\ub85c\uc2a4 \ub2e4\uc74c\uc73c\ub85c \ud070 \uc12c\uc758 \uba85\uce6d\uc740 \ubb34\uc5c7\uc778\uac00?", "paragraph": "\uc774 \uc12c\uc5d0\ub294 2005\ub144 \uae30\uc900\uc73c\ub85c 650,000\uba85\uc774 \uc0b4\uace0 \uc788\uc73c\uba70, \uadf8\ub9ac\uc2a4\uc758 13 \uad00\uad6c \uc911 \ud55c \uacf3\uc774\ub2e4. \uadf8\ub9ac\uc2a4\uc5d0\uc11c \uac00\uc7a5 \ud070 \uc12c\uc774\uba70, \uc9c0\uc911\ud574 \ub3d9\ubd80\uc5d0\uc11c \ud0a4\ud504\ub85c\uc2a4 \ub2e4\uc74c\uc73c\ub85c \ud070 \uc12c\uc774\uae30\ub3c4 \ud558\ub2e4. \ud06c\ub808\ud0c0\ub294 \ub3d9\uc11c\ub85c \uae38\uac8c \ubed7\uc5b4 \uc788\ub294 \ubaa8\uc2b5\uc778\ub370, \ub3d9\uc5d0\uc11c \uc11c\uae4c\uc9c0 260 km\uc5d0 \uc774\ub974\uc9c0\ub9cc \ub0a8\ubd81\uc73c\ub85c \uadf8 \ud3ed\uc740 60km \uc815\ub3c4\uc774\uba70, \ud3ed\uc774 12km\uc5d0 \ubd88\uacfc\ud55c \ub808\ub77c\ud398\ud2b8\ub77c\ucc98\ub7fc \uc881\uc740 \uc9c0\uc5ed\ub3c4 \uc788\ub2e4. \ud06c\ub808\ud0c0\uc758 \uba74\uc801\uc740 8,336 \ud3c9\ubc29 \ud0ac\ub85c\ubbf8\ud130\uc774\uba70 \ud574\uc548\uc120\uc758 \uae38\uc774\ub294 1,046km\ub85c, \ubd81\ucabd\uc5d0\ub294 \ud06c\ub808\ud0c0 \ud574(\u039a\u03c1\u03b7\u03c4\u03b9\u03ba\u03cc \u03a0\u03ad\u03bb\u03b1\u03b3\u03bf\u03c2)\uac00, \ub0a8\ucabd\uc5d0\ub294 \ub9ac\ube44\uc544 \ud574(\u039b\u03b9\u03b2\u03c5\u03ba\u03cc \u03a0\u03ad\u03bb\u03b1\u03b3\u03bf\u03c2), \uc11c\ucabd\uc73c\ub85c\ub294 \ubbf8\ub974\ud1a0\uc544 \ud574, \ub3d9\ucabd\uc5d0\ub294 \uce74\ub974\ud30c\ud2f0\uc628 \ud574\uac00 \uc788\ub2e4. \uadf8\ub9ac\uc2a4 \ubcf8\ud1a0\uc640\ub294 \ub0a8\ucabd\uc73c\ub85c 160km \uc815\ub3c4 \ub5a8\uc5b4\uc838 \uc788\ub2e4.", "answer": "\ud06c\ub808\ud0c0", "sentence": "\ud06c\ub808\ud0c0 \ub294 \ub3d9\uc11c\ub85c \uae38\uac8c \ubed7\uc5b4 \uc788\ub294 \ubaa8\uc2b5\uc778\ub370, \ub3d9\uc5d0\uc11c \uc11c\uae4c\uc9c0 260 km\uc5d0 \uc774\ub974\uc9c0\ub9cc \ub0a8\ubd81\uc73c\ub85c \uadf8 \ud3ed\uc740 60km \uc815\ub3c4\uc774\uba70, \ud3ed\uc774 12km\uc5d0 \ubd88\uacfc\ud55c \ub808\ub77c\ud398\ud2b8\ub77c\ucc98\ub7fc \uc881\uc740 \uc9c0\uc5ed\ub3c4 \uc788\ub2e4.", "paragraph_sentence": "\uc774 \uc12c\uc5d0\ub294 2005\ub144 \uae30\uc900\uc73c\ub85c 650,000\uba85\uc774 \uc0b4\uace0 \uc788\uc73c\uba70, \uadf8\ub9ac\uc2a4\uc758 13 \uad00\uad6c \uc911 \ud55c \uacf3\uc774\ub2e4. \uadf8\ub9ac\uc2a4\uc5d0\uc11c \uac00\uc7a5 \ud070 \uc12c\uc774\uba70, \uc9c0\uc911\ud574 \ub3d9\ubd80\uc5d0\uc11c \ud0a4\ud504\ub85c\uc2a4 \ub2e4\uc74c\uc73c\ub85c \ud070 \uc12c\uc774\uae30\ub3c4 \ud558\ub2e4. \ud06c\ub808\ud0c0 \ub294 \ub3d9\uc11c\ub85c \uae38\uac8c \ubed7\uc5b4 \uc788\ub294 \ubaa8\uc2b5\uc778\ub370, \ub3d9\uc5d0\uc11c \uc11c\uae4c\uc9c0 260 km\uc5d0 \uc774\ub974\uc9c0\ub9cc \ub0a8\ubd81\uc73c\ub85c \uadf8 \ud3ed\uc740 60km \uc815\ub3c4\uc774\uba70, \ud3ed\uc774 12km\uc5d0 \ubd88\uacfc\ud55c \ub808\ub77c\ud398\ud2b8\ub77c\ucc98\ub7fc \uc881\uc740 \uc9c0\uc5ed\ub3c4 \uc788\ub2e4. \ud06c\ub808\ud0c0\uc758 \uba74\uc801\uc740 8,336 \ud3c9\ubc29 \ud0ac\ub85c\ubbf8\ud130\uc774\uba70 \ud574\uc548\uc120\uc758 \uae38\uc774\ub294 1,046km\ub85c, \ubd81\ucabd\uc5d0\ub294 \ud06c\ub808\ud0c0 \ud574(\u039a\u03c1\u03b7\u03c4\u03b9\u03ba\u03cc \u03a0\u03ad\u03bb\u03b1\u03b3\u03bf\u03c2)\uac00, \ub0a8\ucabd\uc5d0\ub294 \ub9ac\ube44\uc544 \ud574(\u039b\u03b9\u03b2\u03c5\u03ba\u03cc \u03a0\u03ad\u03bb\u03b1\u03b3\u03bf\u03c2), \uc11c\ucabd\uc73c\ub85c\ub294 \ubbf8\ub974\ud1a0\uc544 \ud574, \ub3d9\ucabd\uc5d0\ub294 \uce74\ub974\ud30c\ud2f0\uc628 \ud574\uac00 \uc788\ub2e4. \uadf8\ub9ac\uc2a4 \ubcf8\ud1a0\uc640\ub294 \ub0a8\ucabd\uc73c\ub85c 160km \uc815\ub3c4 \ub5a8\uc5b4\uc838 \uc788\ub2e4.", "paragraph_answer": "\uc774 \uc12c\uc5d0\ub294 2005\ub144 \uae30\uc900\uc73c\ub85c 650,000\uba85\uc774 \uc0b4\uace0 \uc788\uc73c\uba70, \uadf8\ub9ac\uc2a4\uc758 13 \uad00\uad6c \uc911 \ud55c \uacf3\uc774\ub2e4. \uadf8\ub9ac\uc2a4\uc5d0\uc11c \uac00\uc7a5 \ud070 \uc12c\uc774\uba70, \uc9c0\uc911\ud574 \ub3d9\ubd80\uc5d0\uc11c \ud0a4\ud504\ub85c\uc2a4 \ub2e4\uc74c\uc73c\ub85c \ud070 \uc12c\uc774\uae30\ub3c4 \ud558\ub2e4. \ud06c\ub808\ud0c0 \ub294 \ub3d9\uc11c\ub85c \uae38\uac8c \ubed7\uc5b4 \uc788\ub294 \ubaa8\uc2b5\uc778\ub370, \ub3d9\uc5d0\uc11c \uc11c\uae4c\uc9c0 260 km\uc5d0 \uc774\ub974\uc9c0\ub9cc \ub0a8\ubd81\uc73c\ub85c \uadf8 \ud3ed\uc740 60km \uc815\ub3c4\uc774\uba70, \ud3ed\uc774 12km\uc5d0 \ubd88\uacfc\ud55c \ub808\ub77c\ud398\ud2b8\ub77c\ucc98\ub7fc \uc881\uc740 \uc9c0\uc5ed\ub3c4 \uc788\ub2e4. \ud06c\ub808\ud0c0\uc758 \uba74\uc801\uc740 8,336 \ud3c9\ubc29 \ud0ac\ub85c\ubbf8\ud130\uc774\uba70 \ud574\uc548\uc120\uc758 \uae38\uc774\ub294 1,046km\ub85c, \ubd81\ucabd\uc5d0\ub294 \ud06c\ub808\ud0c0 \ud574(\u039a\u03c1\u03b7\u03c4\u03b9\u03ba\u03cc \u03a0\u03ad\u03bb\u03b1\u03b3\u03bf\u03c2)\uac00, \ub0a8\ucabd\uc5d0\ub294 \ub9ac\ube44\uc544 \ud574(\u039b\u03b9\u03b2\u03c5\u03ba\u03cc \u03a0\u03ad\u03bb\u03b1\u03b3\u03bf\u03c2), \uc11c\ucabd\uc73c\ub85c\ub294 \ubbf8\ub974\ud1a0\uc544 \ud574, \ub3d9\ucabd\uc5d0\ub294 \uce74\ub974\ud30c\ud2f0\uc628 \ud574\uac00 \uc788\ub2e4. \uadf8\ub9ac\uc2a4 \ubcf8\ud1a0\uc640\ub294 \ub0a8\ucabd\uc73c\ub85c 160km \uc815\ub3c4 \ub5a8\uc5b4\uc838 \uc788\ub2e4.", "sentence_answer": " \ud06c\ub808\ud0c0 \ub294 \ub3d9\uc11c\ub85c \uae38\uac8c \ubed7\uc5b4 \uc788\ub294 \ubaa8\uc2b5\uc778\ub370, \ub3d9\uc5d0\uc11c \uc11c\uae4c\uc9c0 260 km\uc5d0 \uc774\ub974\uc9c0\ub9cc \ub0a8\ubd81\uc73c\ub85c \uadf8 \ud3ed\uc740 60km \uc815\ub3c4\uc774\uba70, \ud3ed\uc774 12km\uc5d0 \ubd88\uacfc\ud55c \ub808\ub77c\ud398\ud2b8\ub77c\ucc98\ub7fc \uc881\uc740 \uc9c0\uc5ed\ub3c4 \uc788\ub2e4.", "paragraph_id": "6397843-0-0", "paragraph_question": "question: \uc9c0\uc911\ud574 \ub3d9\ubd80\uc5d0\uc11c \ud0a4\ud504\ub85c\uc2a4 \ub2e4\uc74c\uc73c\ub85c \ud070 \uc12c\uc758 \uba85\uce6d\uc740 \ubb34\uc5c7\uc778\uac00?, context: \uc774 \uc12c\uc5d0\ub294 2005\ub144 \uae30\uc900\uc73c\ub85c 650,000\uba85\uc774 \uc0b4\uace0 \uc788\uc73c\uba70, \uadf8\ub9ac\uc2a4\uc758 13 \uad00\uad6c \uc911 \ud55c \uacf3\uc774\ub2e4. \uadf8\ub9ac\uc2a4\uc5d0\uc11c \uac00\uc7a5 \ud070 \uc12c\uc774\uba70, \uc9c0\uc911\ud574 \ub3d9\ubd80\uc5d0\uc11c \ud0a4\ud504\ub85c\uc2a4 \ub2e4\uc74c\uc73c\ub85c \ud070 \uc12c\uc774\uae30\ub3c4 \ud558\ub2e4. \ud06c\ub808\ud0c0\ub294 \ub3d9\uc11c\ub85c \uae38\uac8c \ubed7\uc5b4 \uc788\ub294 \ubaa8\uc2b5\uc778\ub370, \ub3d9\uc5d0\uc11c \uc11c\uae4c\uc9c0 260 km\uc5d0 \uc774\ub974\uc9c0\ub9cc \ub0a8\ubd81\uc73c\ub85c \uadf8 \ud3ed\uc740 60km \uc815\ub3c4\uc774\uba70, \ud3ed\uc774 12km\uc5d0 \ubd88\uacfc\ud55c \ub808\ub77c\ud398\ud2b8\ub77c\ucc98\ub7fc \uc881\uc740 \uc9c0\uc5ed\ub3c4 \uc788\ub2e4. \ud06c\ub808\ud0c0\uc758 \uba74\uc801\uc740 8,336 \ud3c9\ubc29 \ud0ac\ub85c\ubbf8\ud130\uc774\uba70 \ud574\uc548\uc120\uc758 \uae38\uc774\ub294 1,046km\ub85c, \ubd81\ucabd\uc5d0\ub294 \ud06c\ub808\ud0c0 \ud574(\u039a\u03c1\u03b7\u03c4\u03b9\u03ba\u03cc \u03a0\u03ad\u03bb\u03b1\u03b3\u03bf\u03c2)\uac00, \ub0a8\ucabd\uc5d0\ub294 \ub9ac\ube44\uc544 \ud574(\u039b\u03b9\u03b2\u03c5\u03ba\u03cc \u03a0\u03ad\u03bb\u03b1\u03b3\u03bf\u03c2), \uc11c\ucabd\uc73c\ub85c\ub294 \ubbf8\ub974\ud1a0\uc544 \ud574, \ub3d9\ucabd\uc5d0\ub294 \uce74\ub974\ud30c\ud2f0\uc628 \ud574\uac00 \uc788\ub2e4. \uadf8\ub9ac\uc2a4 \ubcf8\ud1a0\uc640\ub294 \ub0a8\ucabd\uc73c\ub85c 160km \uc815\ub3c4 \ub5a8\uc5b4\uc838 \uc788\ub2e4."} {"question": "17\uc138\uae30 \ud504\ub791\uc2a4\uc758 \uc655\ub9bd\uc815\uc6d0 \uc124\uacc4\uc790\ub294?", "paragraph": "\ub8e8\uc774 14\uc138\uac00 \uad6d\uac00\uc758 \ud2c0\uc744 \uc644\ube44\ud558\uba70 \uc655\uad8c\uc744 \uac15\ud654\ud558\uba70 \uc655\uc774 \ubb38\ud654\ub97c \uc8fc\ub3c4\ud558\uace0 \ud6c4\uc6d0\ud558\uba70 \ubc1c\ub2ec\ud558\uac8c \ub41c\ub2e4. \uc774\ud0c8\ub9ac\uc544\uc5d0\uc11c \ud655\ub9bd\ub41c \ucd95 \uc911\uc2ec\uc758 \uc815\uc6d0 \uc124\uacc4 \uae30\ubc95\uc774 \ud504\ub791\uc2a4\uc5d0\uc11c\ub294 \uc0c8\ub85c\uc6b4 \uc2a4\ucf00\uc77c\ub85c \uc801\uc6a9\ub418\uc5c8\uace0 \uc6b0\uc8fc\uc801 \uc2a4\ucf00\uc77c\ub85c \ud655\uc7a5\ud558\uac8c \ub418\uc5c8\ub2e4. 17\uc138\uae30 \ud504\ub791\uc2a4\ub294 \uc559\ub4dc\ub808 \ub974 \ub178\ud2b8\ub85c\ub77c\ub294 \uc655\ub9bd\uc815\uc6d0\uc758 \uc124\uac8c\uac00\uac00 \uc8fc\ub3c4 \ud588\uc73c\uba70 \uc815\uc6d0\uc744 \ud1b5\ud574\uc11c \ud0dc\uc591\uc655\uc758 \uad8c\uc704\uac00 \ub4e4\uc5b4\ub098\ub3c4\ub85d \uac70\ub300\ud55c \uc2a4\ucf00\uc77c\uac10\uc744 \ud1b5\ud574\uc11c \ubcf4\ub294 \uc774\ub85c \ud558\uc5ec\uae08 \uacbd\uc678\uac10\uc774 \uc77c\uac8c \ud558\ub294 \ubc29\ubc95\uc744 \uc0ac\uc6a9\ud588\ub2e4. \uadf8\ub294 \uc7a5\uc5c4\ud568\uc744 \ud45c\uc2dc\ud558\ub294\ub370 \uba74\uc801\uc744 \uc0ac\uc6a9\ud558\uace0 \ubc14\ub85c\ud06c \uc591\uc2dd\uc5d0 \uc6b0\uc544\ud568\uc744 \uac00\ubbf8, \uc18c\ub85c, \ud3c9\uba74\uae30\ud558\ud559, \uc815\uc6d0\uc758 \ub2f4\uc744 \uc0b0\ub9bc\uc73c\ub85c \ucc98\ub9ac\ud558\ub294 \ubc29\ubc95\uc744 \uc0ac\uc6a9\ud588\uace0 \uc774\ub294 18~19\uc138\uae30 \uc804 \uc720\ub7fd\uc5d0 \uc601\ud5a5\uc744 \uc8fc\uc5c8\ub2e4. \uadf8\ub294 \uae30\uc874 \ub974\ub124\uc0c1\uc2a4 \uc815\uc6d0\uc5d0\uc11c \ud0c8\ud53c, \uc7a5\uc5c4\ud558\uba74\uc11c \uc138\ub828\ub41c \uc2a4\ud0c0\uc77c\uc744 \ubcf4\uc5ec\uc8fc\uace0 \uacf5\uac04\uc744 \uae30\ud558\ud559\uc801\uc73c\ub85c \ud30c\uc545, \uc815\uc6d0 \ub514\uc790\uc778\uc758 \uc2a4\ucf00\uc77c\uc744 \ud655\uc7a5\uc2dc\ucf30\ub2e4, \ub610\ud55c \ub370\uce74\ub974\ud2b8\uc758 \uc218\ud559\uc801 \uac1c\ub150\uc744 \uc815\uc6d0 \uc124\uacc4\uc5d0 \uc801\uc6a9\ud558\uc5ec \ub300\uaddc\ubaa8 \uae30\ud558\ud559\uc694\uc18c\ub97c \uc815\uc6d0 \uad6c\uc131\uc758 \ud1a0\ub300\uac00 \ub418\uac8c\ud558\uc600\ub2e4.", "answer": "\uc559\ub4dc\ub808 \ub974 \ub178\ud2b8\ub85c", "sentence": "17\uc138\uae30 \ud504\ub791\uc2a4\ub294 \uc559\ub4dc\ub808 \ub974 \ub178\ud2b8\ub85c \ub77c\ub294 \uc655\ub9bd\uc815\uc6d0\uc758 \uc124\uac8c\uac00\uac00 \uc8fc\ub3c4 \ud588\uc73c\uba70 \uc815\uc6d0\uc744 \ud1b5\ud574\uc11c \ud0dc\uc591\uc655\uc758 \uad8c\uc704\uac00 \ub4e4\uc5b4\ub098\ub3c4\ub85d \uac70\ub300\ud55c \uc2a4\ucf00\uc77c\uac10\uc744 \ud1b5\ud574\uc11c \ubcf4\ub294 \uc774\ub85c \ud558\uc5ec\uae08 \uacbd\uc678\uac10\uc774 \uc77c\uac8c \ud558\ub294 \ubc29\ubc95\uc744 \uc0ac\uc6a9\ud588\ub2e4.", "paragraph_sentence": "\ub8e8\uc774 14\uc138\uac00 \uad6d\uac00\uc758 \ud2c0\uc744 \uc644\ube44\ud558\uba70 \uc655\uad8c\uc744 \uac15\ud654\ud558\uba70 \uc655\uc774 \ubb38\ud654\ub97c \uc8fc\ub3c4\ud558\uace0 \ud6c4\uc6d0\ud558\uba70 \ubc1c\ub2ec\ud558\uac8c \ub41c\ub2e4. \uc774\ud0c8\ub9ac\uc544\uc5d0\uc11c \ud655\ub9bd\ub41c \ucd95 \uc911\uc2ec\uc758 \uc815\uc6d0 \uc124\uacc4 \uae30\ubc95\uc774 \ud504\ub791\uc2a4\uc5d0\uc11c\ub294 \uc0c8\ub85c\uc6b4 \uc2a4\ucf00\uc77c\ub85c \uc801\uc6a9\ub418\uc5c8\uace0 \uc6b0\uc8fc\uc801 \uc2a4\ucf00\uc77c\ub85c \ud655\uc7a5\ud558\uac8c \ub418\uc5c8\ub2e4. 17\uc138\uae30 \ud504\ub791\uc2a4\ub294 \uc559\ub4dc\ub808 \ub974 \ub178\ud2b8\ub85c \ub77c\ub294 \uc655\ub9bd\uc815\uc6d0\uc758 \uc124\uac8c\uac00\uac00 \uc8fc\ub3c4 \ud588\uc73c\uba70 \uc815\uc6d0\uc744 \ud1b5\ud574\uc11c \ud0dc\uc591\uc655\uc758 \uad8c\uc704\uac00 \ub4e4\uc5b4\ub098\ub3c4\ub85d \uac70\ub300\ud55c \uc2a4\ucf00\uc77c\uac10\uc744 \ud1b5\ud574\uc11c \ubcf4\ub294 \uc774\ub85c \ud558\uc5ec\uae08 \uacbd\uc678\uac10\uc774 \uc77c\uac8c \ud558\ub294 \ubc29\ubc95\uc744 \uc0ac\uc6a9\ud588\ub2e4. \uadf8\ub294 \uc7a5\uc5c4\ud568\uc744 \ud45c\uc2dc\ud558\ub294\ub370 \uba74\uc801\uc744 \uc0ac\uc6a9\ud558\uace0 \ubc14\ub85c\ud06c \uc591\uc2dd\uc5d0 \uc6b0\uc544\ud568\uc744 \uac00\ubbf8, \uc18c\ub85c, \ud3c9\uba74\uae30\ud558\ud559, \uc815\uc6d0\uc758 \ub2f4\uc744 \uc0b0\ub9bc\uc73c\ub85c \ucc98\ub9ac\ud558\ub294 \ubc29\ubc95\uc744 \uc0ac\uc6a9\ud588\uace0 \uc774\ub294 18~19\uc138\uae30 \uc804 \uc720\ub7fd\uc5d0 \uc601\ud5a5\uc744 \uc8fc\uc5c8\ub2e4. \uadf8\ub294 \uae30\uc874 \ub974\ub124\uc0c1\uc2a4 \uc815\uc6d0\uc5d0\uc11c \ud0c8\ud53c, \uc7a5\uc5c4\ud558\uba74\uc11c \uc138\ub828\ub41c \uc2a4\ud0c0\uc77c\uc744 \ubcf4\uc5ec\uc8fc\uace0 \uacf5\uac04\uc744 \uae30\ud558\ud559\uc801\uc73c\ub85c \ud30c\uc545, \uc815\uc6d0 \ub514\uc790\uc778\uc758 \uc2a4\ucf00\uc77c\uc744 \ud655\uc7a5\uc2dc\ucf30\ub2e4, \ub610\ud55c \ub370\uce74\ub974\ud2b8\uc758 \uc218\ud559\uc801 \uac1c\ub150\uc744 \uc815\uc6d0 \uc124\uacc4\uc5d0 \uc801\uc6a9\ud558\uc5ec \ub300\uaddc\ubaa8 \uae30\ud558\ud559\uc694\uc18c\ub97c \uc815\uc6d0 \uad6c\uc131\uc758 \ud1a0\ub300\uac00 \ub418\uac8c\ud558\uc600\ub2e4.", "paragraph_answer": "\ub8e8\uc774 14\uc138\uac00 \uad6d\uac00\uc758 \ud2c0\uc744 \uc644\ube44\ud558\uba70 \uc655\uad8c\uc744 \uac15\ud654\ud558\uba70 \uc655\uc774 \ubb38\ud654\ub97c \uc8fc\ub3c4\ud558\uace0 \ud6c4\uc6d0\ud558\uba70 \ubc1c\ub2ec\ud558\uac8c \ub41c\ub2e4. \uc774\ud0c8\ub9ac\uc544\uc5d0\uc11c \ud655\ub9bd\ub41c \ucd95 \uc911\uc2ec\uc758 \uc815\uc6d0 \uc124\uacc4 \uae30\ubc95\uc774 \ud504\ub791\uc2a4\uc5d0\uc11c\ub294 \uc0c8\ub85c\uc6b4 \uc2a4\ucf00\uc77c\ub85c \uc801\uc6a9\ub418\uc5c8\uace0 \uc6b0\uc8fc\uc801 \uc2a4\ucf00\uc77c\ub85c \ud655\uc7a5\ud558\uac8c \ub418\uc5c8\ub2e4. 17\uc138\uae30 \ud504\ub791\uc2a4\ub294 \uc559\ub4dc\ub808 \ub974 \ub178\ud2b8\ub85c \ub77c\ub294 \uc655\ub9bd\uc815\uc6d0\uc758 \uc124\uac8c\uac00\uac00 \uc8fc\ub3c4 \ud588\uc73c\uba70 \uc815\uc6d0\uc744 \ud1b5\ud574\uc11c \ud0dc\uc591\uc655\uc758 \uad8c\uc704\uac00 \ub4e4\uc5b4\ub098\ub3c4\ub85d \uac70\ub300\ud55c \uc2a4\ucf00\uc77c\uac10\uc744 \ud1b5\ud574\uc11c \ubcf4\ub294 \uc774\ub85c \ud558\uc5ec\uae08 \uacbd\uc678\uac10\uc774 \uc77c\uac8c \ud558\ub294 \ubc29\ubc95\uc744 \uc0ac\uc6a9\ud588\ub2e4. \uadf8\ub294 \uc7a5\uc5c4\ud568\uc744 \ud45c\uc2dc\ud558\ub294\ub370 \uba74\uc801\uc744 \uc0ac\uc6a9\ud558\uace0 \ubc14\ub85c\ud06c \uc591\uc2dd\uc5d0 \uc6b0\uc544\ud568\uc744 \uac00\ubbf8, \uc18c\ub85c, \ud3c9\uba74\uae30\ud558\ud559, \uc815\uc6d0\uc758 \ub2f4\uc744 \uc0b0\ub9bc\uc73c\ub85c \ucc98\ub9ac\ud558\ub294 \ubc29\ubc95\uc744 \uc0ac\uc6a9\ud588\uace0 \uc774\ub294 18~19\uc138\uae30 \uc804 \uc720\ub7fd\uc5d0 \uc601\ud5a5\uc744 \uc8fc\uc5c8\ub2e4. \uadf8\ub294 \uae30\uc874 \ub974\ub124\uc0c1\uc2a4 \uc815\uc6d0\uc5d0\uc11c \ud0c8\ud53c, \uc7a5\uc5c4\ud558\uba74\uc11c \uc138\ub828\ub41c \uc2a4\ud0c0\uc77c\uc744 \ubcf4\uc5ec\uc8fc\uace0 \uacf5\uac04\uc744 \uae30\ud558\ud559\uc801\uc73c\ub85c \ud30c\uc545, \uc815\uc6d0 \ub514\uc790\uc778\uc758 \uc2a4\ucf00\uc77c\uc744 \ud655\uc7a5\uc2dc\ucf30\ub2e4, \ub610\ud55c \ub370\uce74\ub974\ud2b8\uc758 \uc218\ud559\uc801 \uac1c\ub150\uc744 \uc815\uc6d0 \uc124\uacc4\uc5d0 \uc801\uc6a9\ud558\uc5ec \ub300\uaddc\ubaa8 \uae30\ud558\ud559\uc694\uc18c\ub97c \uc815\uc6d0 \uad6c\uc131\uc758 \ud1a0\ub300\uac00 \ub418\uac8c\ud558\uc600\ub2e4.", "sentence_answer": "17\uc138\uae30 \ud504\ub791\uc2a4\ub294 \uc559\ub4dc\ub808 \ub974 \ub178\ud2b8\ub85c \ub77c\ub294 \uc655\ub9bd\uc815\uc6d0\uc758 \uc124\uac8c\uac00\uac00 \uc8fc\ub3c4 \ud588\uc73c\uba70 \uc815\uc6d0\uc744 \ud1b5\ud574\uc11c \ud0dc\uc591\uc655\uc758 \uad8c\uc704\uac00 \ub4e4\uc5b4\ub098\ub3c4\ub85d \uac70\ub300\ud55c \uc2a4\ucf00\uc77c\uac10\uc744 \ud1b5\ud574\uc11c \ubcf4\ub294 \uc774\ub85c \ud558\uc5ec\uae08 \uacbd\uc678\uac10\uc774 \uc77c\uac8c \ud558\ub294 \ubc29\ubc95\uc744 \uc0ac\uc6a9\ud588\ub2e4.", "paragraph_id": "5874187-0-0", "paragraph_question": "question: 17\uc138\uae30 \ud504\ub791\uc2a4\uc758 \uc655\ub9bd\uc815\uc6d0 \uc124\uacc4\uc790\ub294?, context: \ub8e8\uc774 14\uc138\uac00 \uad6d\uac00\uc758 \ud2c0\uc744 \uc644\ube44\ud558\uba70 \uc655\uad8c\uc744 \uac15\ud654\ud558\uba70 \uc655\uc774 \ubb38\ud654\ub97c \uc8fc\ub3c4\ud558\uace0 \ud6c4\uc6d0\ud558\uba70 \ubc1c\ub2ec\ud558\uac8c \ub41c\ub2e4. \uc774\ud0c8\ub9ac\uc544\uc5d0\uc11c \ud655\ub9bd\ub41c \ucd95 \uc911\uc2ec\uc758 \uc815\uc6d0 \uc124\uacc4 \uae30\ubc95\uc774 \ud504\ub791\uc2a4\uc5d0\uc11c\ub294 \uc0c8\ub85c\uc6b4 \uc2a4\ucf00\uc77c\ub85c \uc801\uc6a9\ub418\uc5c8\uace0 \uc6b0\uc8fc\uc801 \uc2a4\ucf00\uc77c\ub85c \ud655\uc7a5\ud558\uac8c \ub418\uc5c8\ub2e4. 17\uc138\uae30 \ud504\ub791\uc2a4\ub294 \uc559\ub4dc\ub808 \ub974 \ub178\ud2b8\ub85c\ub77c\ub294 \uc655\ub9bd\uc815\uc6d0\uc758 \uc124\uac8c\uac00\uac00 \uc8fc\ub3c4 \ud588\uc73c\uba70 \uc815\uc6d0\uc744 \ud1b5\ud574\uc11c \ud0dc\uc591\uc655\uc758 \uad8c\uc704\uac00 \ub4e4\uc5b4\ub098\ub3c4\ub85d \uac70\ub300\ud55c \uc2a4\ucf00\uc77c\uac10\uc744 \ud1b5\ud574\uc11c \ubcf4\ub294 \uc774\ub85c \ud558\uc5ec\uae08 \uacbd\uc678\uac10\uc774 \uc77c\uac8c \ud558\ub294 \ubc29\ubc95\uc744 \uc0ac\uc6a9\ud588\ub2e4. \uadf8\ub294 \uc7a5\uc5c4\ud568\uc744 \ud45c\uc2dc\ud558\ub294\ub370 \uba74\uc801\uc744 \uc0ac\uc6a9\ud558\uace0 \ubc14\ub85c\ud06c \uc591\uc2dd\uc5d0 \uc6b0\uc544\ud568\uc744 \uac00\ubbf8, \uc18c\ub85c, \ud3c9\uba74\uae30\ud558\ud559, \uc815\uc6d0\uc758 \ub2f4\uc744 \uc0b0\ub9bc\uc73c\ub85c \ucc98\ub9ac\ud558\ub294 \ubc29\ubc95\uc744 \uc0ac\uc6a9\ud588\uace0 \uc774\ub294 18~19\uc138\uae30 \uc804 \uc720\ub7fd\uc5d0 \uc601\ud5a5\uc744 \uc8fc\uc5c8\ub2e4. \uadf8\ub294 \uae30\uc874 \ub974\ub124\uc0c1\uc2a4 \uc815\uc6d0\uc5d0\uc11c \ud0c8\ud53c, \uc7a5\uc5c4\ud558\uba74\uc11c \uc138\ub828\ub41c \uc2a4\ud0c0\uc77c\uc744 \ubcf4\uc5ec\uc8fc\uace0 \uacf5\uac04\uc744 \uae30\ud558\ud559\uc801\uc73c\ub85c \ud30c\uc545, \uc815\uc6d0 \ub514\uc790\uc778\uc758 \uc2a4\ucf00\uc77c\uc744 \ud655\uc7a5\uc2dc\ucf30\ub2e4, \ub610\ud55c \ub370\uce74\ub974\ud2b8\uc758 \uc218\ud559\uc801 \uac1c\ub150\uc744 \uc815\uc6d0 \uc124\uacc4\uc5d0 \uc801\uc6a9\ud558\uc5ec \ub300\uaddc\ubaa8 \uae30\ud558\ud559\uc694\uc18c\ub97c \uc815\uc6d0 \uad6c\uc131\uc758 \ud1a0\ub300\uac00 \ub418\uac8c\ud558\uc600\ub2e4."} {"question": "2008\ub144 8\uc6d4 \uae30\uc900\uc73c\ub85c \uc9c0\uad6c\uc758 \uc9c8\ub7c9 10\ubc30 \ubbf8\ub9cc \ud589\uc131\uc758 \uac1c\uc218\ub294?", "paragraph": "\uc9c0\uae08\uae4c\uc9c0 \ubc1c\uacac\ub41c \uc678\uacc4 \ud589\uc131 \ub300\ubd80\ubd84\uc740 \ubaa9\uc131 \ub610\ub294 \uadf8 \uc774\uc0c1\uc73c\ub85c \ub9e4\uc6b0 \ubb34\uac70\uc6b4 \uc9c8\ub7c9\uc744 \uc9c0\ub154\ub2e4. 2008\ub144 8\uc6d4 \uae30\uc900\uc73c\ub85c \uc9c0\uad6c \uc9c8\ub7c9 10\ubc30 \ubbf8\ub9cc \ud589\uc131\uc758 \uc22b\uc790\ub294 12\uac1c\uc5d0 \ubd88\uacfc\ud558\ub2e4. \uc801\uc9c0 \uc54a\uc740 \ud589\uc131\ub4e4\uc740 \ubaa9\uc131\ubcf4\ub2e4\ub3c4 \ud6e8\uc52c \ubb34\uac81\ub2e4. \uadf8\ub7ec\ub098 \uc774\ub294 \uc120\ud0dd \ud3b8\ud5a5\uc801\uc778 \uacb0\uacfc\uc5d0 \ubd88\uacfc\ud558\ub2e4. \uc9c0\uae08\uae4c\uc9c0 \uac1c\ubc1c\ub41c \ubc1c\uacac \ubc29\ubc95\uc73c\ub85c\ub294 \uc791\uace0 \uac00\ubcbc\uc6b4 \ud589\uc131\ubcf4\ub2e4\ub294 \ubb34\uac81\uace0 \ub369\uce58 \ud070 \ud589\uc131\ub4e4\uc744 \ubc1c\uacac\ud558\uae30\uac00 \ud6e8\uc52c \uc27d\ub2e4. \uc774\ucc98\ub7fc \uc3e0\ub9b0 \uacb0\uacfc\ub294 \ud1b5\uacc4\uc801 \ubd84\uc11d\uc744 \uc5b4\ub835\uac8c \ub9cc\ub4dc\ub098 \uc2e4\uc81c \uc9c0\uad6c\ucc98\ub7fc \uac00\ubcbc\uc6b4 \ud589\uc131\ub4e4\uc740 \ubb34\uac70\uc6b4 \uac00\uc2a4 \ud589\uc131\ub4e4\ubcf4\ub2e4 \ud754\ud558\uac8c \uc874\uc7ac\ud558\ub294 \uac83\uc73c\ub85c \ucd94\uce21\ub41c\ub2e4. \ub610\ud55c \uad00\uce21 \uae30\uc220\uc774 \uc544\uc9c1 \uc815\uad50\ud558\uc9c0 \ubabb\ud568\uc5d0\ub3c4 \ubd88\uad6c\ud558\uace0 \uc9c0\uad6c\uc9c8\ub7c9 \uc218 \ubc30 \uc218\uc900 \ud589\uc131\ub4e4\uc774 \uc5ec\ub7ff \ubc1c\uacac\ub418\uc5c8\uc74c\uc744 \uace0\ub824\ud558\uba74, \uc791\uc740 \ud589\uc131\ub4e4\uc740 \ub9e4\uc6b0 \ud754\ud558\uac8c \uc6b0\uc8fc\uc5d0 \uc874\uc7ac\ud558\ub294 \uac83\uc73c\ub85c \uc0dd\uac01\ub41c\ub2e4. \uce60\ub808 \uc18c\uc7ac \ub77c \uc2e4\ub77c \ucc9c\ubb38\ub300 HARPS \uc2a4\ud399\ud2b8\ub7fc \uc0ac\uc9c4\uae30 2008\ub144 \uc790\ub8cc\uc5d0 \ub530\ub974\uba74 \ud56d\uc131 14\uac1c \uc911 \ud558\ub098 \uaf34\ub85c \uac00\uc2a4 \ud589\uc131\uc774 \uc788\uc73c\uba70, \ub2e4\uc2dc \uac00\uc2a4 \ud589\uc131\uc774 \uc788\ub294 \ubcc4 \uc14b \uc911 \ud558\ub098 \uaf34\ub85c \uc9c0\uad6c\uc9c8\ub7c9 30\ubc30 \uc774\ud558\uc758 \uc554\uc11d \ud589\uc131\uc744 \uac70\ub290\ub9ac\uace0 \uc788\uc744 \uac00\ub2a5\uc131\uc774 \ub192\uc740 \uac83\uc73c\ub85c \ubc1d\ud600\uc84c\ub2e4.", "answer": "12\uac1c", "sentence": "2008\ub144 8\uc6d4 \uae30\uc900\uc73c\ub85c \uc9c0\uad6c \uc9c8\ub7c9 10\ubc30 \ubbf8\ub9cc \ud589\uc131\uc758 \uc22b\uc790\ub294 12\uac1c \uc5d0 \ubd88\uacfc\ud558\ub2e4.", "paragraph_sentence": "\uc9c0\uae08\uae4c\uc9c0 \ubc1c\uacac\ub41c \uc678\uacc4 \ud589\uc131 \ub300\ubd80\ubd84\uc740 \ubaa9\uc131 \ub610\ub294 \uadf8 \uc774\uc0c1\uc73c\ub85c \ub9e4\uc6b0 \ubb34\uac70\uc6b4 \uc9c8\ub7c9\uc744 \uc9c0\ub154\ub2e4. 2008\ub144 8\uc6d4 \uae30\uc900\uc73c\ub85c \uc9c0\uad6c \uc9c8\ub7c9 10\ubc30 \ubbf8\ub9cc \ud589\uc131\uc758 \uc22b\uc790\ub294 12\uac1c \uc5d0 \ubd88\uacfc\ud558\ub2e4. \uc801\uc9c0 \uc54a\uc740 \ud589\uc131\ub4e4\uc740 \ubaa9\uc131\ubcf4\ub2e4\ub3c4 \ud6e8\uc52c \ubb34\uac81\ub2e4. \uadf8\ub7ec\ub098 \uc774\ub294 \uc120\ud0dd \ud3b8\ud5a5\uc801\uc778 \uacb0\uacfc\uc5d0 \ubd88\uacfc\ud558\ub2e4. \uc9c0\uae08\uae4c\uc9c0 \uac1c\ubc1c\ub41c \ubc1c\uacac \ubc29\ubc95\uc73c\ub85c\ub294 \uc791\uace0 \uac00\ubcbc\uc6b4 \ud589\uc131\ubcf4\ub2e4\ub294 \ubb34\uac81\uace0 \ub369\uce58 \ud070 \ud589\uc131\ub4e4\uc744 \ubc1c\uacac\ud558\uae30\uac00 \ud6e8\uc52c \uc27d\ub2e4. \uc774\ucc98\ub7fc \uc3e0\ub9b0 \uacb0\uacfc\ub294 \ud1b5\uacc4\uc801 \ubd84\uc11d\uc744 \uc5b4\ub835\uac8c \ub9cc\ub4dc\ub098 \uc2e4\uc81c \uc9c0\uad6c\ucc98\ub7fc \uac00\ubcbc\uc6b4 \ud589\uc131\ub4e4\uc740 \ubb34\uac70\uc6b4 \uac00\uc2a4 \ud589\uc131\ub4e4\ubcf4\ub2e4 \ud754\ud558\uac8c \uc874\uc7ac\ud558\ub294 \uac83\uc73c\ub85c \ucd94\uce21\ub41c\ub2e4. \ub610\ud55c \uad00\uce21 \uae30\uc220\uc774 \uc544\uc9c1 \uc815\uad50\ud558\uc9c0 \ubabb\ud568\uc5d0\ub3c4 \ubd88\uad6c\ud558\uace0 \uc9c0\uad6c\uc9c8\ub7c9 \uc218 \ubc30 \uc218\uc900 \ud589\uc131\ub4e4\uc774 \uc5ec\ub7ff \ubc1c\uacac\ub418\uc5c8\uc74c\uc744 \uace0\ub824\ud558\uba74, \uc791\uc740 \ud589\uc131\ub4e4\uc740 \ub9e4\uc6b0 \ud754\ud558\uac8c \uc6b0\uc8fc\uc5d0 \uc874\uc7ac\ud558\ub294 \uac83\uc73c\ub85c \uc0dd\uac01\ub41c\ub2e4. \uce60\ub808 \uc18c\uc7ac \ub77c \uc2e4\ub77c \ucc9c\ubb38\ub300 HARPS \uc2a4\ud399\ud2b8\ub7fc \uc0ac\uc9c4\uae30 2008\ub144 \uc790\ub8cc\uc5d0 \ub530\ub974\uba74 \ud56d\uc131 14\uac1c \uc911 \ud558\ub098 \uaf34\ub85c \uac00\uc2a4 \ud589\uc131\uc774 \uc788\uc73c\uba70, \ub2e4\uc2dc \uac00\uc2a4 \ud589\uc131\uc774 \uc788\ub294 \ubcc4 \uc14b \uc911 \ud558\ub098 \uaf34\ub85c \uc9c0\uad6c\uc9c8\ub7c9 30\ubc30 \uc774\ud558\uc758 \uc554\uc11d \ud589\uc131\uc744 \uac70\ub290\ub9ac\uace0 \uc788\uc744 \uac00\ub2a5\uc131\uc774 \ub192\uc740 \uac83\uc73c\ub85c \ubc1d\ud600\uc84c\ub2e4.", "paragraph_answer": "\uc9c0\uae08\uae4c\uc9c0 \ubc1c\uacac\ub41c \uc678\uacc4 \ud589\uc131 \ub300\ubd80\ubd84\uc740 \ubaa9\uc131 \ub610\ub294 \uadf8 \uc774\uc0c1\uc73c\ub85c \ub9e4\uc6b0 \ubb34\uac70\uc6b4 \uc9c8\ub7c9\uc744 \uc9c0\ub154\ub2e4. 2008\ub144 8\uc6d4 \uae30\uc900\uc73c\ub85c \uc9c0\uad6c \uc9c8\ub7c9 10\ubc30 \ubbf8\ub9cc \ud589\uc131\uc758 \uc22b\uc790\ub294 12\uac1c \uc5d0 \ubd88\uacfc\ud558\ub2e4. \uc801\uc9c0 \uc54a\uc740 \ud589\uc131\ub4e4\uc740 \ubaa9\uc131\ubcf4\ub2e4\ub3c4 \ud6e8\uc52c \ubb34\uac81\ub2e4. \uadf8\ub7ec\ub098 \uc774\ub294 \uc120\ud0dd \ud3b8\ud5a5\uc801\uc778 \uacb0\uacfc\uc5d0 \ubd88\uacfc\ud558\ub2e4. \uc9c0\uae08\uae4c\uc9c0 \uac1c\ubc1c\ub41c \ubc1c\uacac \ubc29\ubc95\uc73c\ub85c\ub294 \uc791\uace0 \uac00\ubcbc\uc6b4 \ud589\uc131\ubcf4\ub2e4\ub294 \ubb34\uac81\uace0 \ub369\uce58 \ud070 \ud589\uc131\ub4e4\uc744 \ubc1c\uacac\ud558\uae30\uac00 \ud6e8\uc52c \uc27d\ub2e4. \uc774\ucc98\ub7fc \uc3e0\ub9b0 \uacb0\uacfc\ub294 \ud1b5\uacc4\uc801 \ubd84\uc11d\uc744 \uc5b4\ub835\uac8c \ub9cc\ub4dc\ub098 \uc2e4\uc81c \uc9c0\uad6c\ucc98\ub7fc \uac00\ubcbc\uc6b4 \ud589\uc131\ub4e4\uc740 \ubb34\uac70\uc6b4 \uac00\uc2a4 \ud589\uc131\ub4e4\ubcf4\ub2e4 \ud754\ud558\uac8c \uc874\uc7ac\ud558\ub294 \uac83\uc73c\ub85c \ucd94\uce21\ub41c\ub2e4. \ub610\ud55c \uad00\uce21 \uae30\uc220\uc774 \uc544\uc9c1 \uc815\uad50\ud558\uc9c0 \ubabb\ud568\uc5d0\ub3c4 \ubd88\uad6c\ud558\uace0 \uc9c0\uad6c\uc9c8\ub7c9 \uc218 \ubc30 \uc218\uc900 \ud589\uc131\ub4e4\uc774 \uc5ec\ub7ff \ubc1c\uacac\ub418\uc5c8\uc74c\uc744 \uace0\ub824\ud558\uba74, \uc791\uc740 \ud589\uc131\ub4e4\uc740 \ub9e4\uc6b0 \ud754\ud558\uac8c \uc6b0\uc8fc\uc5d0 \uc874\uc7ac\ud558\ub294 \uac83\uc73c\ub85c \uc0dd\uac01\ub41c\ub2e4. \uce60\ub808 \uc18c\uc7ac \ub77c \uc2e4\ub77c \ucc9c\ubb38\ub300 HARPS \uc2a4\ud399\ud2b8\ub7fc \uc0ac\uc9c4\uae30 2008\ub144 \uc790\ub8cc\uc5d0 \ub530\ub974\uba74 \ud56d\uc131 14\uac1c \uc911 \ud558\ub098 \uaf34\ub85c \uac00\uc2a4 \ud589\uc131\uc774 \uc788\uc73c\uba70, \ub2e4\uc2dc \uac00\uc2a4 \ud589\uc131\uc774 \uc788\ub294 \ubcc4 \uc14b \uc911 \ud558\ub098 \uaf34\ub85c \uc9c0\uad6c\uc9c8\ub7c9 30\ubc30 \uc774\ud558\uc758 \uc554\uc11d \ud589\uc131\uc744 \uac70\ub290\ub9ac\uace0 \uc788\uc744 \uac00\ub2a5\uc131\uc774 \ub192\uc740 \uac83\uc73c\ub85c \ubc1d\ud600\uc84c\ub2e4.", "sentence_answer": "2008\ub144 8\uc6d4 \uae30\uc900\uc73c\ub85c \uc9c0\uad6c \uc9c8\ub7c9 10\ubc30 \ubbf8\ub9cc \ud589\uc131\uc758 \uc22b\uc790\ub294 12\uac1c \uc5d0 \ubd88\uacfc\ud558\ub2e4.", "paragraph_id": "6525746-5-0", "paragraph_question": "question: 2008\ub144 8\uc6d4 \uae30\uc900\uc73c\ub85c \uc9c0\uad6c\uc758 \uc9c8\ub7c9 10\ubc30 \ubbf8\ub9cc \ud589\uc131\uc758 \uac1c\uc218\ub294?, context: \uc9c0\uae08\uae4c\uc9c0 \ubc1c\uacac\ub41c \uc678\uacc4 \ud589\uc131 \ub300\ubd80\ubd84\uc740 \ubaa9\uc131 \ub610\ub294 \uadf8 \uc774\uc0c1\uc73c\ub85c \ub9e4\uc6b0 \ubb34\uac70\uc6b4 \uc9c8\ub7c9\uc744 \uc9c0\ub154\ub2e4. 2008\ub144 8\uc6d4 \uae30\uc900\uc73c\ub85c \uc9c0\uad6c \uc9c8\ub7c9 10\ubc30 \ubbf8\ub9cc \ud589\uc131\uc758 \uc22b\uc790\ub294 12\uac1c\uc5d0 \ubd88\uacfc\ud558\ub2e4. \uc801\uc9c0 \uc54a\uc740 \ud589\uc131\ub4e4\uc740 \ubaa9\uc131\ubcf4\ub2e4\ub3c4 \ud6e8\uc52c \ubb34\uac81\ub2e4. \uadf8\ub7ec\ub098 \uc774\ub294 \uc120\ud0dd \ud3b8\ud5a5\uc801\uc778 \uacb0\uacfc\uc5d0 \ubd88\uacfc\ud558\ub2e4. \uc9c0\uae08\uae4c\uc9c0 \uac1c\ubc1c\ub41c \ubc1c\uacac \ubc29\ubc95\uc73c\ub85c\ub294 \uc791\uace0 \uac00\ubcbc\uc6b4 \ud589\uc131\ubcf4\ub2e4\ub294 \ubb34\uac81\uace0 \ub369\uce58 \ud070 \ud589\uc131\ub4e4\uc744 \ubc1c\uacac\ud558\uae30\uac00 \ud6e8\uc52c \uc27d\ub2e4. \uc774\ucc98\ub7fc \uc3e0\ub9b0 \uacb0\uacfc\ub294 \ud1b5\uacc4\uc801 \ubd84\uc11d\uc744 \uc5b4\ub835\uac8c \ub9cc\ub4dc\ub098 \uc2e4\uc81c \uc9c0\uad6c\ucc98\ub7fc \uac00\ubcbc\uc6b4 \ud589\uc131\ub4e4\uc740 \ubb34\uac70\uc6b4 \uac00\uc2a4 \ud589\uc131\ub4e4\ubcf4\ub2e4 \ud754\ud558\uac8c \uc874\uc7ac\ud558\ub294 \uac83\uc73c\ub85c \ucd94\uce21\ub41c\ub2e4. \ub610\ud55c \uad00\uce21 \uae30\uc220\uc774 \uc544\uc9c1 \uc815\uad50\ud558\uc9c0 \ubabb\ud568\uc5d0\ub3c4 \ubd88\uad6c\ud558\uace0 \uc9c0\uad6c\uc9c8\ub7c9 \uc218 \ubc30 \uc218\uc900 \ud589\uc131\ub4e4\uc774 \uc5ec\ub7ff \ubc1c\uacac\ub418\uc5c8\uc74c\uc744 \uace0\ub824\ud558\uba74, \uc791\uc740 \ud589\uc131\ub4e4\uc740 \ub9e4\uc6b0 \ud754\ud558\uac8c \uc6b0\uc8fc\uc5d0 \uc874\uc7ac\ud558\ub294 \uac83\uc73c\ub85c \uc0dd\uac01\ub41c\ub2e4. \uce60\ub808 \uc18c\uc7ac \ub77c \uc2e4\ub77c \ucc9c\ubb38\ub300 HARPS \uc2a4\ud399\ud2b8\ub7fc \uc0ac\uc9c4\uae30 2008\ub144 \uc790\ub8cc\uc5d0 \ub530\ub974\uba74 \ud56d\uc131 14\uac1c \uc911 \ud558\ub098 \uaf34\ub85c \uac00\uc2a4 \ud589\uc131\uc774 \uc788\uc73c\uba70, \ub2e4\uc2dc \uac00\uc2a4 \ud589\uc131\uc774 \uc788\ub294 \ubcc4 \uc14b \uc911 \ud558\ub098 \uaf34\ub85c \uc9c0\uad6c\uc9c8\ub7c9 30\ubc30 \uc774\ud558\uc758 \uc554\uc11d \ud589\uc131\uc744 \uac70\ub290\ub9ac\uace0 \uc788\uc744 \uac00\ub2a5\uc131\uc774 \ub192\uc740 \uac83\uc73c\ub85c \ubc1d\ud600\uc84c\ub2e4."} {"question": "\ud074\ub85c\ubc84\ud544\ub4dc\ub294 \uc5b4\ub514\uc5d0\uc11c \ubc1c\uc0dd\ud558\ub294 \uc598\uae30\ub97c \ub2e4\ub978 \uc791\ud488\uc778\uac00\uc694?", "paragraph": "\uc774 \uc601\ud654\ub294 \uc624\ub80c \uc6b0\uc9c0\uc5d8\uc774 \uc6b0\uc8fc \uc815\uac70\uc7a5\uc758 \uc120\uc6d0\ub4e4\uc744 \uc8fc\uc694 \uc904\uac70\ub9ac\ub85c \ub2e4\ub8e8\ub294 \u300a\uac13 \ud30c\ud2f0\ud074\u300b(God Particle)\uc744 \uac01\uc0c9\ud55c \uac83\uc774\uba70, \uc6d0\ub798\ub294 \u300a\ud074\ub85c\ubc84\ud544\ub4dc\u300b\uc640 \uad00\ub828\uc774 \uc5c6\uc5c8\ub2e4. \uc774 \uac01\ubcf8\uc740 2012\ub144\uc5d0 \ud30c\ub77c\ub9c8\uc6b4\ud2b8 \ud53d\uccd0\uc2a4\uc640 \ubc30\ub4dc \ub85c\ubd07 \ud504\ub85c\ub355\uc158\uc2a4\uac00 \ud68d\ub4dd\ud588\ub2e4. \ucd08\uae30\uc5d0\ub294 \ud30c\ub77c\ub9c8\uc6b4\ud2b8\uc758 \uc800\uc608\uc0b0 \ubc30\uae09 \uc81c\uc791\uc0ac Insurge\uc758 \uc791\ud488\uc73c\ub85c \uacc4\ud68d\ud588\uc5c8\uc73c\ub098, \ud574\ub2f9 \uc81c\uc791\uc0ac\uac00 \uc815\ub9ac\ub418\uba74\uc11c, \ud30c\ub77c\ub9c8\uc6b4\ud2b8 \ubc30\uae09 \uc601\ud654\ub85c \ud655\uc7a5\ub418\uc5c8\ub2e4. \uc601\ud654\uac00 \uc81c\uc791\ub418\uace0 \uc788\uc744 \ub3d9\uc548\uc5d0 \uc5d0\uc774\ube0c\ub7fc\uc2a4\ub294 \ud074\ub85c\ub7ec\ud544\ub4dc\uc640 \uc774 \uc601\ud654\ub97c \uc5f0\uacb0\uc2dc\ud0a4\uae30\ub85c \uacb0\uc815\ud558\uba74\uc11c, \u300a\ud074\ub85c\ubc84\ud544\ub4dc 10\ubc88\uc9c0\u300b\uc758 \uc6d0\uc791 \uac01\ubcf8 \uc140\ub7ec \u300aThe Cellar\u300b\uc5d0\uc11c \uac19\uc740 \ubc29\ubc95\uc744 \ud588\ub358\uac70\ucc98\ub7fc \uc6b0\uc9c0\uc5d8\uc758 \uac01\ubcf8\uc744 \uac01\uc0c9\ud558\uace0 \ud074\ub85c\ubc84\ud544\ub4dc\uc640 \uc5f0\uacb0\ub418\ub294 \uc7a5\uba74\uc744 \ucd94\uac00\ud588\ub2e4. \uc5d0\uc774\ube0c\ub7fc\uc2a4\ub294 \uc785\uc790 \uac00\uc18d\uae30 \uc0ac\uace0\uac00 \ubbf8\ub798\uc758 \uc0ac\uac74\ub4e4\uc774 \uacfc\uac70\uc5d0 \ubcc0\ud654\ub97c \uc77c\uc73c\ud0ac \uc218\uc788\ub294 \uc218\ub2e8\uc73c\ub85c \uc0ac\uc6a9\ub420 \uc218 \uc788\uc74c\uc744 \uc788\ub2e4\uace0 \ubcf4\uc558\uace0, \ud568\uaed8 \ud074\ub85c\ubc84\ud544\ub4dc \ud504\ub79c\ucc28\uc774\uc988\uc5d0 \uc11c\uc220\uc801\uc73c\ub85c \uc5f0\uacb0\ub418\ub294 \ubc29\ubc95\uc744 \uad6c\ucd95\ud588\ub2e4.", "answer": "\uc6b0\uc8fc \uc815\uac70\uc7a5", "sentence": "\uc774 \uc601\ud654\ub294 \uc624\ub80c \uc6b0\uc9c0\uc5d8\uc774 \uc6b0\uc8fc \uc815\uac70\uc7a5 \uc758 \uc120\uc6d0\ub4e4\uc744 \uc8fc\uc694 \uc904\uac70\ub9ac\ub85c \ub2e4\ub8e8\ub294 \u300a\uac13 \ud30c\ud2f0\ud074\u300b(God Particle)\uc744 \uac01\uc0c9\ud55c \uac83\uc774\uba70, \uc6d0\ub798\ub294 \u300a\ud074\ub85c\ubc84\ud544\ub4dc\u300b\uc640 \uad00\ub828\uc774 \uc5c6\uc5c8\ub2e4.", "paragraph_sentence": " \uc774 \uc601\ud654\ub294 \uc624\ub80c \uc6b0\uc9c0\uc5d8\uc774 \uc6b0\uc8fc \uc815\uac70\uc7a5 \uc758 \uc120\uc6d0\ub4e4\uc744 \uc8fc\uc694 \uc904\uac70\ub9ac\ub85c \ub2e4\ub8e8\ub294 \u300a\uac13 \ud30c\ud2f0\ud074\u300b(God Particle)\uc744 \uac01\uc0c9\ud55c \uac83\uc774\uba70, \uc6d0\ub798\ub294 \u300a\ud074\ub85c\ubc84\ud544\ub4dc\u300b\uc640 \uad00\ub828\uc774 \uc5c6\uc5c8\ub2e4. \uc774 \uac01\ubcf8\uc740 2012\ub144\uc5d0 \ud30c\ub77c\ub9c8\uc6b4\ud2b8 \ud53d\uccd0\uc2a4\uc640 \ubc30\ub4dc \ub85c\ubd07 \ud504\ub85c\ub355\uc158\uc2a4\uac00 \ud68d\ub4dd\ud588\ub2e4. \ucd08\uae30\uc5d0\ub294 \ud30c\ub77c\ub9c8\uc6b4\ud2b8\uc758 \uc800\uc608\uc0b0 \ubc30\uae09 \uc81c\uc791\uc0ac Insurge\uc758 \uc791\ud488\uc73c\ub85c \uacc4\ud68d\ud588\uc5c8\uc73c\ub098, \ud574\ub2f9 \uc81c\uc791\uc0ac\uac00 \uc815\ub9ac\ub418\uba74\uc11c, \ud30c\ub77c\ub9c8\uc6b4\ud2b8 \ubc30\uae09 \uc601\ud654\ub85c \ud655\uc7a5\ub418\uc5c8\ub2e4. \uc601\ud654\uac00 \uc81c\uc791\ub418\uace0 \uc788\uc744 \ub3d9\uc548\uc5d0 \uc5d0\uc774\ube0c\ub7fc\uc2a4\ub294 \ud074\ub85c\ub7ec\ud544\ub4dc\uc640 \uc774 \uc601\ud654\ub97c \uc5f0\uacb0\uc2dc\ud0a4\uae30\ub85c \uacb0\uc815\ud558\uba74\uc11c, \u300a\ud074\ub85c\ubc84\ud544\ub4dc 10\ubc88\uc9c0\u300b\uc758 \uc6d0\uc791 \uac01\ubcf8 \uc140\ub7ec \u300aThe Cellar\u300b\uc5d0\uc11c \uac19\uc740 \ubc29\ubc95\uc744 \ud588\ub358\uac70\ucc98\ub7fc \uc6b0\uc9c0\uc5d8\uc758 \uac01\ubcf8\uc744 \uac01\uc0c9\ud558\uace0 \ud074\ub85c\ubc84\ud544\ub4dc\uc640 \uc5f0\uacb0\ub418\ub294 \uc7a5\uba74\uc744 \ucd94\uac00\ud588\ub2e4. \uc5d0\uc774\ube0c\ub7fc\uc2a4\ub294 \uc785\uc790 \uac00\uc18d\uae30 \uc0ac\uace0\uac00 \ubbf8\ub798\uc758 \uc0ac\uac74\ub4e4\uc774 \uacfc\uac70\uc5d0 \ubcc0\ud654\ub97c \uc77c\uc73c\ud0ac \uc218\uc788\ub294 \uc218\ub2e8\uc73c\ub85c \uc0ac\uc6a9\ub420 \uc218 \uc788\uc74c\uc744 \uc788\ub2e4\uace0 \ubcf4\uc558\uace0, \ud568\uaed8 \ud074\ub85c\ubc84\ud544\ub4dc \ud504\ub79c\ucc28\uc774\uc988\uc5d0 \uc11c\uc220\uc801\uc73c\ub85c \uc5f0\uacb0\ub418\ub294 \ubc29\ubc95\uc744 \uad6c\ucd95\ud588\ub2e4.", "paragraph_answer": "\uc774 \uc601\ud654\ub294 \uc624\ub80c \uc6b0\uc9c0\uc5d8\uc774 \uc6b0\uc8fc \uc815\uac70\uc7a5 \uc758 \uc120\uc6d0\ub4e4\uc744 \uc8fc\uc694 \uc904\uac70\ub9ac\ub85c \ub2e4\ub8e8\ub294 \u300a\uac13 \ud30c\ud2f0\ud074\u300b(God Particle)\uc744 \uac01\uc0c9\ud55c \uac83\uc774\uba70, \uc6d0\ub798\ub294 \u300a\ud074\ub85c\ubc84\ud544\ub4dc\u300b\uc640 \uad00\ub828\uc774 \uc5c6\uc5c8\ub2e4. \uc774 \uac01\ubcf8\uc740 2012\ub144\uc5d0 \ud30c\ub77c\ub9c8\uc6b4\ud2b8 \ud53d\uccd0\uc2a4\uc640 \ubc30\ub4dc \ub85c\ubd07 \ud504\ub85c\ub355\uc158\uc2a4\uac00 \ud68d\ub4dd\ud588\ub2e4. \ucd08\uae30\uc5d0\ub294 \ud30c\ub77c\ub9c8\uc6b4\ud2b8\uc758 \uc800\uc608\uc0b0 \ubc30\uae09 \uc81c\uc791\uc0ac Insurge\uc758 \uc791\ud488\uc73c\ub85c \uacc4\ud68d\ud588\uc5c8\uc73c\ub098, \ud574\ub2f9 \uc81c\uc791\uc0ac\uac00 \uc815\ub9ac\ub418\uba74\uc11c, \ud30c\ub77c\ub9c8\uc6b4\ud2b8 \ubc30\uae09 \uc601\ud654\ub85c \ud655\uc7a5\ub418\uc5c8\ub2e4. \uc601\ud654\uac00 \uc81c\uc791\ub418\uace0 \uc788\uc744 \ub3d9\uc548\uc5d0 \uc5d0\uc774\ube0c\ub7fc\uc2a4\ub294 \ud074\ub85c\ub7ec\ud544\ub4dc\uc640 \uc774 \uc601\ud654\ub97c \uc5f0\uacb0\uc2dc\ud0a4\uae30\ub85c \uacb0\uc815\ud558\uba74\uc11c, \u300a\ud074\ub85c\ubc84\ud544\ub4dc 10\ubc88\uc9c0\u300b\uc758 \uc6d0\uc791 \uac01\ubcf8 \uc140\ub7ec \u300aThe Cellar\u300b\uc5d0\uc11c \uac19\uc740 \ubc29\ubc95\uc744 \ud588\ub358\uac70\ucc98\ub7fc \uc6b0\uc9c0\uc5d8\uc758 \uac01\ubcf8\uc744 \uac01\uc0c9\ud558\uace0 \ud074\ub85c\ubc84\ud544\ub4dc\uc640 \uc5f0\uacb0\ub418\ub294 \uc7a5\uba74\uc744 \ucd94\uac00\ud588\ub2e4. \uc5d0\uc774\ube0c\ub7fc\uc2a4\ub294 \uc785\uc790 \uac00\uc18d\uae30 \uc0ac\uace0\uac00 \ubbf8\ub798\uc758 \uc0ac\uac74\ub4e4\uc774 \uacfc\uac70\uc5d0 \ubcc0\ud654\ub97c \uc77c\uc73c\ud0ac \uc218\uc788\ub294 \uc218\ub2e8\uc73c\ub85c \uc0ac\uc6a9\ub420 \uc218 \uc788\uc74c\uc744 \uc788\ub2e4\uace0 \ubcf4\uc558\uace0, \ud568\uaed8 \ud074\ub85c\ubc84\ud544\ub4dc \ud504\ub79c\ucc28\uc774\uc988\uc5d0 \uc11c\uc220\uc801\uc73c\ub85c \uc5f0\uacb0\ub418\ub294 \ubc29\ubc95\uc744 \uad6c\ucd95\ud588\ub2e4.", "sentence_answer": "\uc774 \uc601\ud654\ub294 \uc624\ub80c \uc6b0\uc9c0\uc5d8\uc774 \uc6b0\uc8fc \uc815\uac70\uc7a5 \uc758 \uc120\uc6d0\ub4e4\uc744 \uc8fc\uc694 \uc904\uac70\ub9ac\ub85c \ub2e4\ub8e8\ub294 \u300a\uac13 \ud30c\ud2f0\ud074\u300b(God Particle)\uc744 \uac01\uc0c9\ud55c \uac83\uc774\uba70, \uc6d0\ub798\ub294 \u300a\ud074\ub85c\ubc84\ud544\ub4dc\u300b\uc640 \uad00\ub828\uc774 \uc5c6\uc5c8\ub2e4.", "paragraph_id": "6564695-1-1", "paragraph_question": "question: \ud074\ub85c\ubc84\ud544\ub4dc\ub294 \uc5b4\ub514\uc5d0\uc11c \ubc1c\uc0dd\ud558\ub294 \uc598\uae30\ub97c \ub2e4\ub978 \uc791\ud488\uc778\uac00\uc694?, context: \uc774 \uc601\ud654\ub294 \uc624\ub80c \uc6b0\uc9c0\uc5d8\uc774 \uc6b0\uc8fc \uc815\uac70\uc7a5\uc758 \uc120\uc6d0\ub4e4\uc744 \uc8fc\uc694 \uc904\uac70\ub9ac\ub85c \ub2e4\ub8e8\ub294 \u300a\uac13 \ud30c\ud2f0\ud074\u300b(God Particle)\uc744 \uac01\uc0c9\ud55c \uac83\uc774\uba70, \uc6d0\ub798\ub294 \u300a\ud074\ub85c\ubc84\ud544\ub4dc\u300b\uc640 \uad00\ub828\uc774 \uc5c6\uc5c8\ub2e4. \uc774 \uac01\ubcf8\uc740 2012\ub144\uc5d0 \ud30c\ub77c\ub9c8\uc6b4\ud2b8 \ud53d\uccd0\uc2a4\uc640 \ubc30\ub4dc \ub85c\ubd07 \ud504\ub85c\ub355\uc158\uc2a4\uac00 \ud68d\ub4dd\ud588\ub2e4. \ucd08\uae30\uc5d0\ub294 \ud30c\ub77c\ub9c8\uc6b4\ud2b8\uc758 \uc800\uc608\uc0b0 \ubc30\uae09 \uc81c\uc791\uc0ac Insurge\uc758 \uc791\ud488\uc73c\ub85c \uacc4\ud68d\ud588\uc5c8\uc73c\ub098, \ud574\ub2f9 \uc81c\uc791\uc0ac\uac00 \uc815\ub9ac\ub418\uba74\uc11c, \ud30c\ub77c\ub9c8\uc6b4\ud2b8 \ubc30\uae09 \uc601\ud654\ub85c \ud655\uc7a5\ub418\uc5c8\ub2e4. \uc601\ud654\uac00 \uc81c\uc791\ub418\uace0 \uc788\uc744 \ub3d9\uc548\uc5d0 \uc5d0\uc774\ube0c\ub7fc\uc2a4\ub294 \ud074\ub85c\ub7ec\ud544\ub4dc\uc640 \uc774 \uc601\ud654\ub97c \uc5f0\uacb0\uc2dc\ud0a4\uae30\ub85c \uacb0\uc815\ud558\uba74\uc11c, \u300a\ud074\ub85c\ubc84\ud544\ub4dc 10\ubc88\uc9c0\u300b\uc758 \uc6d0\uc791 \uac01\ubcf8 \uc140\ub7ec \u300aThe Cellar\u300b\uc5d0\uc11c \uac19\uc740 \ubc29\ubc95\uc744 \ud588\ub358\uac70\ucc98\ub7fc \uc6b0\uc9c0\uc5d8\uc758 \uac01\ubcf8\uc744 \uac01\uc0c9\ud558\uace0 \ud074\ub85c\ubc84\ud544\ub4dc\uc640 \uc5f0\uacb0\ub418\ub294 \uc7a5\uba74\uc744 \ucd94\uac00\ud588\ub2e4. \uc5d0\uc774\ube0c\ub7fc\uc2a4\ub294 \uc785\uc790 \uac00\uc18d\uae30 \uc0ac\uace0\uac00 \ubbf8\ub798\uc758 \uc0ac\uac74\ub4e4\uc774 \uacfc\uac70\uc5d0 \ubcc0\ud654\ub97c \uc77c\uc73c\ud0ac \uc218\uc788\ub294 \uc218\ub2e8\uc73c\ub85c \uc0ac\uc6a9\ub420 \uc218 \uc788\uc74c\uc744 \uc788\ub2e4\uace0 \ubcf4\uc558\uace0, \ud568\uaed8 \ud074\ub85c\ubc84\ud544\ub4dc \ud504\ub79c\ucc28\uc774\uc988\uc5d0 \uc11c\uc220\uc801\uc73c\ub85c \uc5f0\uacb0\ub418\ub294 \ubc29\ubc95\uc744 \uad6c\ucd95\ud588\ub2e4."} {"question": "1941\ub144 9\uc6d4 17\uc77c\uc5d0 \ub3c4\uc774\uce60\ub780\ud2b8 \uc81c11\uad70\uc758 \uc9c0\ud718\uad00\uc774 \ub41c \uc0ac\ub78c\uc758 \uc774\ub984\uc740 \ubb34\uc5c7\uc778\uac00?", "paragraph": "\ub3c4\uc774\uce60\ub780\ud2b8 \uc721\uad70 \ucd5c\uace0\uc0ac\ub839\ubd80\ub294 \ud06c\ub9bc\ubc18\ub3c4\uac00 \ub8e8\ub9c8\ub2c8\uc544 \uc11d\uc720 \uc2dc\uc124 \uacf5\uaca9\uc744 \ubc29\uc5b4\ud558\ub824\ub294 \uad70\uc0ac\uc0c1 \uc694\ucc98\uc774\uace0 \ucd5c\ub300\ud55c \ube68\ub9ac \uc810\ub839\ud558\ub77c\uace0 \uba85\ub839\ud588\ub2e4. \uc544\ub3cc\ud504 \ud788\ud2c0\ub7ec \ud1b5\ucd1d\uc740 \ub0a8\uc9c4\ud558\ub77c\ub294 \uc790\uc2e0\uc758 \uba85\ub839\uc5d0 \ub300\ud55c \uc7a5\uc560\ub97c \ubabb \uacac\ub514\uace0 8\uc6d4 12\uc77c\uc5d0 \ud06c\ub9bc\ubc18\ub3c4\ub97c \uc989\uc2dc \uc810\ub839\ud558\ub77c\ub294 \ub9d0\uc744 \ubc18\ubcf5\ud588\ub2e4. \ud0a4\uc608\ud504\uc5d0\uc11c \uac70\ub300\ud55c \ud3ec\uc704\uc804\uc774 \uc77c\uc5b4\ub098\ub294 \ud55c \ub2ec\uc5ec\uac04, 9\uc6d4 17\uc77c \ub3c4\uc774\uce60\ub780\ud2b8 \uc81c11\uad70\uc758 \uc9c0\ud718\uad00\uc774 \ub41c \uc0c1\uae09\ub300\uc7a5 \uc5d0\ub9ac\ud788 \ud3f0 \ub9cc\uc288\ud0c0\uc778\uc774 \uc9c0\ud718\ub97c \ub9e1\uc740 \uc9c0 \uc77c\uc8fc\uc77c \ud6c4 \ud06c\ub9bc\ubc18\ub3c4\ub97c \uacf5\uaca9\ud558\uae30 \uc2dc\uc791\ud588\ub2e4. \uce58\uc5f4\ud55c \uc804\ud22c \uc774\ud6c4 \uc5d0\ub9ac\ud788 \ud3f0 \ub9cc\uc288\ud0c0\uc778\uc740 \uc18c\ub828\uc758 \uc5ec\ub7ec \ubc18\uaca9\uc744 \uaca9\ud1f4\ud558\uace0 \ubd95\uad34\uc2dc\ucf30\ub2e4. 11\uc6d4 1\uc77c\uc5d0\ub294 \ud06c\ub9bc\ubc18\ub3c4\uc758 \uc8fc\ub3c4\uc778 \uc2ec\ud398\ub85c\ud3f4\uc744 \uc810\ub839\ud588\uace0 11\uc6d4 16\uc77c\uc740 \ud06c\ub9bc\ubc18\ub3c4 \uc804\uccb4\ub97c \uc77c\uc18c\ud588\ub2e4. \ucf00\ub974\uce58\uac00 11\uc6d4 16\uc77c \ud568\ub77d\ub41c \ud6c4 \uc720\uc77c\ud558\uac8c \ub0a8\uc740 \uc9c0\uc5ed\uc740 \uc138\ubc14\uc2a4\ud1a0\ud3f4\uc774\uc5c8\ub2e4.", "answer": "\uc5d0\ub9ac\ud788 \ud3f0 \ub9cc\uc288\ud0c0\uc778", "sentence": "\ud0a4\uc608\ud504\uc5d0\uc11c \uac70\ub300\ud55c \ud3ec\uc704\uc804\uc774 \uc77c\uc5b4\ub098\ub294 \ud55c \ub2ec\uc5ec\uac04, 9\uc6d4 17\uc77c \ub3c4\uc774\uce60\ub780\ud2b8 \uc81c11\uad70\uc758 \uc9c0\ud718\uad00\uc774 \ub41c \uc0c1\uae09\ub300\uc7a5 \uc5d0\ub9ac\ud788 \ud3f0 \ub9cc\uc288\ud0c0\uc778 \uc774 \uc9c0\ud718\ub97c \ub9e1\uc740 \uc9c0 \uc77c\uc8fc\uc77c \ud6c4 \ud06c\ub9bc\ubc18\ub3c4\ub97c \uacf5\uaca9\ud558\uae30 \uc2dc\uc791\ud588\ub2e4.", "paragraph_sentence": "\ub3c4\uc774\uce60\ub780\ud2b8 \uc721\uad70 \ucd5c\uace0\uc0ac\ub839\ubd80\ub294 \ud06c\ub9bc\ubc18\ub3c4\uac00 \ub8e8\ub9c8\ub2c8\uc544 \uc11d\uc720 \uc2dc\uc124 \uacf5\uaca9\uc744 \ubc29\uc5b4\ud558\ub824\ub294 \uad70\uc0ac\uc0c1 \uc694\ucc98\uc774\uace0 \ucd5c\ub300\ud55c \ube68\ub9ac \uc810\ub839\ud558\ub77c\uace0 \uba85\ub839\ud588\ub2e4. \uc544\ub3cc\ud504 \ud788\ud2c0\ub7ec \ud1b5\ucd1d\uc740 \ub0a8\uc9c4\ud558\ub77c\ub294 \uc790\uc2e0\uc758 \uba85\ub839\uc5d0 \ub300\ud55c \uc7a5\uc560\ub97c \ubabb \uacac\ub514\uace0 8\uc6d4 12\uc77c\uc5d0 \ud06c\ub9bc\ubc18\ub3c4\ub97c \uc989\uc2dc \uc810\ub839\ud558\ub77c\ub294 \ub9d0\uc744 \ubc18\ubcf5\ud588\ub2e4. \ud0a4\uc608\ud504\uc5d0\uc11c \uac70\ub300\ud55c \ud3ec\uc704\uc804\uc774 \uc77c\uc5b4\ub098\ub294 \ud55c \ub2ec\uc5ec\uac04, 9\uc6d4 17\uc77c \ub3c4\uc774\uce60\ub780\ud2b8 \uc81c11\uad70\uc758 \uc9c0\ud718\uad00\uc774 \ub41c \uc0c1\uae09\ub300\uc7a5 \uc5d0\ub9ac\ud788 \ud3f0 \ub9cc\uc288\ud0c0\uc778 \uc774 \uc9c0\ud718\ub97c \ub9e1\uc740 \uc9c0 \uc77c\uc8fc\uc77c \ud6c4 \ud06c\ub9bc\ubc18\ub3c4\ub97c \uacf5\uaca9\ud558\uae30 \uc2dc\uc791\ud588\ub2e4. \uce58\uc5f4\ud55c \uc804\ud22c \uc774\ud6c4 \uc5d0\ub9ac\ud788 \ud3f0 \ub9cc\uc288\ud0c0\uc778\uc740 \uc18c\ub828\uc758 \uc5ec\ub7ec \ubc18\uaca9\uc744 \uaca9\ud1f4\ud558\uace0 \ubd95\uad34\uc2dc\ucf30\ub2e4. 11\uc6d4 1\uc77c\uc5d0\ub294 \ud06c\ub9bc\ubc18\ub3c4\uc758 \uc8fc\ub3c4\uc778 \uc2ec\ud398\ub85c\ud3f4\uc744 \uc810\ub839\ud588\uace0 11\uc6d4 16\uc77c\uc740 \ud06c\ub9bc\ubc18\ub3c4 \uc804\uccb4\ub97c \uc77c\uc18c\ud588\ub2e4. \ucf00\ub974\uce58\uac00 11\uc6d4 16\uc77c \ud568\ub77d\ub41c \ud6c4 \uc720\uc77c\ud558\uac8c \ub0a8\uc740 \uc9c0\uc5ed\uc740 \uc138\ubc14\uc2a4\ud1a0\ud3f4\uc774\uc5c8\ub2e4.", "paragraph_answer": "\ub3c4\uc774\uce60\ub780\ud2b8 \uc721\uad70 \ucd5c\uace0\uc0ac\ub839\ubd80\ub294 \ud06c\ub9bc\ubc18\ub3c4\uac00 \ub8e8\ub9c8\ub2c8\uc544 \uc11d\uc720 \uc2dc\uc124 \uacf5\uaca9\uc744 \ubc29\uc5b4\ud558\ub824\ub294 \uad70\uc0ac\uc0c1 \uc694\ucc98\uc774\uace0 \ucd5c\ub300\ud55c \ube68\ub9ac \uc810\ub839\ud558\ub77c\uace0 \uba85\ub839\ud588\ub2e4. \uc544\ub3cc\ud504 \ud788\ud2c0\ub7ec \ud1b5\ucd1d\uc740 \ub0a8\uc9c4\ud558\ub77c\ub294 \uc790\uc2e0\uc758 \uba85\ub839\uc5d0 \ub300\ud55c \uc7a5\uc560\ub97c \ubabb \uacac\ub514\uace0 8\uc6d4 12\uc77c\uc5d0 \ud06c\ub9bc\ubc18\ub3c4\ub97c \uc989\uc2dc \uc810\ub839\ud558\ub77c\ub294 \ub9d0\uc744 \ubc18\ubcf5\ud588\ub2e4. \ud0a4\uc608\ud504\uc5d0\uc11c \uac70\ub300\ud55c \ud3ec\uc704\uc804\uc774 \uc77c\uc5b4\ub098\ub294 \ud55c \ub2ec\uc5ec\uac04, 9\uc6d4 17\uc77c \ub3c4\uc774\uce60\ub780\ud2b8 \uc81c11\uad70\uc758 \uc9c0\ud718\uad00\uc774 \ub41c \uc0c1\uae09\ub300\uc7a5 \uc5d0\ub9ac\ud788 \ud3f0 \ub9cc\uc288\ud0c0\uc778 \uc774 \uc9c0\ud718\ub97c \ub9e1\uc740 \uc9c0 \uc77c\uc8fc\uc77c \ud6c4 \ud06c\ub9bc\ubc18\ub3c4\ub97c \uacf5\uaca9\ud558\uae30 \uc2dc\uc791\ud588\ub2e4. \uce58\uc5f4\ud55c \uc804\ud22c \uc774\ud6c4 \uc5d0\ub9ac\ud788 \ud3f0 \ub9cc\uc288\ud0c0\uc778\uc740 \uc18c\ub828\uc758 \uc5ec\ub7ec \ubc18\uaca9\uc744 \uaca9\ud1f4\ud558\uace0 \ubd95\uad34\uc2dc\ucf30\ub2e4. 11\uc6d4 1\uc77c\uc5d0\ub294 \ud06c\ub9bc\ubc18\ub3c4\uc758 \uc8fc\ub3c4\uc778 \uc2ec\ud398\ub85c\ud3f4\uc744 \uc810\ub839\ud588\uace0 11\uc6d4 16\uc77c\uc740 \ud06c\ub9bc\ubc18\ub3c4 \uc804\uccb4\ub97c \uc77c\uc18c\ud588\ub2e4. \ucf00\ub974\uce58\uac00 11\uc6d4 16\uc77c \ud568\ub77d\ub41c \ud6c4 \uc720\uc77c\ud558\uac8c \ub0a8\uc740 \uc9c0\uc5ed\uc740 \uc138\ubc14\uc2a4\ud1a0\ud3f4\uc774\uc5c8\ub2e4.", "sentence_answer": "\ud0a4\uc608\ud504\uc5d0\uc11c \uac70\ub300\ud55c \ud3ec\uc704\uc804\uc774 \uc77c\uc5b4\ub098\ub294 \ud55c \ub2ec\uc5ec\uac04, 9\uc6d4 17\uc77c \ub3c4\uc774\uce60\ub780\ud2b8 \uc81c11\uad70\uc758 \uc9c0\ud718\uad00\uc774 \ub41c \uc0c1\uae09\ub300\uc7a5 \uc5d0\ub9ac\ud788 \ud3f0 \ub9cc\uc288\ud0c0\uc778 \uc774 \uc9c0\ud718\ub97c \ub9e1\uc740 \uc9c0 \uc77c\uc8fc\uc77c \ud6c4 \ud06c\ub9bc\ubc18\ub3c4\ub97c \uacf5\uaca9\ud558\uae30 \uc2dc\uc791\ud588\ub2e4.", "paragraph_id": "6517980-2-0", "paragraph_question": "question: 1941\ub144 9\uc6d4 17\uc77c\uc5d0 \ub3c4\uc774\uce60\ub780\ud2b8 \uc81c11\uad70\uc758 \uc9c0\ud718\uad00\uc774 \ub41c \uc0ac\ub78c\uc758 \uc774\ub984\uc740 \ubb34\uc5c7\uc778\uac00?, context: \ub3c4\uc774\uce60\ub780\ud2b8 \uc721\uad70 \ucd5c\uace0\uc0ac\ub839\ubd80\ub294 \ud06c\ub9bc\ubc18\ub3c4\uac00 \ub8e8\ub9c8\ub2c8\uc544 \uc11d\uc720 \uc2dc\uc124 \uacf5\uaca9\uc744 \ubc29\uc5b4\ud558\ub824\ub294 \uad70\uc0ac\uc0c1 \uc694\ucc98\uc774\uace0 \ucd5c\ub300\ud55c \ube68\ub9ac \uc810\ub839\ud558\ub77c\uace0 \uba85\ub839\ud588\ub2e4. \uc544\ub3cc\ud504 \ud788\ud2c0\ub7ec \ud1b5\ucd1d\uc740 \ub0a8\uc9c4\ud558\ub77c\ub294 \uc790\uc2e0\uc758 \uba85\ub839\uc5d0 \ub300\ud55c \uc7a5\uc560\ub97c \ubabb \uacac\ub514\uace0 8\uc6d4 12\uc77c\uc5d0 \ud06c\ub9bc\ubc18\ub3c4\ub97c \uc989\uc2dc \uc810\ub839\ud558\ub77c\ub294 \ub9d0\uc744 \ubc18\ubcf5\ud588\ub2e4. \ud0a4\uc608\ud504\uc5d0\uc11c \uac70\ub300\ud55c \ud3ec\uc704\uc804\uc774 \uc77c\uc5b4\ub098\ub294 \ud55c \ub2ec\uc5ec\uac04, 9\uc6d4 17\uc77c \ub3c4\uc774\uce60\ub780\ud2b8 \uc81c11\uad70\uc758 \uc9c0\ud718\uad00\uc774 \ub41c \uc0c1\uae09\ub300\uc7a5 \uc5d0\ub9ac\ud788 \ud3f0 \ub9cc\uc288\ud0c0\uc778\uc774 \uc9c0\ud718\ub97c \ub9e1\uc740 \uc9c0 \uc77c\uc8fc\uc77c \ud6c4 \ud06c\ub9bc\ubc18\ub3c4\ub97c \uacf5\uaca9\ud558\uae30 \uc2dc\uc791\ud588\ub2e4. \uce58\uc5f4\ud55c \uc804\ud22c \uc774\ud6c4 \uc5d0\ub9ac\ud788 \ud3f0 \ub9cc\uc288\ud0c0\uc778\uc740 \uc18c\ub828\uc758 \uc5ec\ub7ec \ubc18\uaca9\uc744 \uaca9\ud1f4\ud558\uace0 \ubd95\uad34\uc2dc\ucf30\ub2e4. 11\uc6d4 1\uc77c\uc5d0\ub294 \ud06c\ub9bc\ubc18\ub3c4\uc758 \uc8fc\ub3c4\uc778 \uc2ec\ud398\ub85c\ud3f4\uc744 \uc810\ub839\ud588\uace0 11\uc6d4 16\uc77c\uc740 \ud06c\ub9bc\ubc18\ub3c4 \uc804\uccb4\ub97c \uc77c\uc18c\ud588\ub2e4. \ucf00\ub974\uce58\uac00 11\uc6d4 16\uc77c \ud568\ub77d\ub41c \ud6c4 \uc720\uc77c\ud558\uac8c \ub0a8\uc740 \uc9c0\uc5ed\uc740 \uc138\ubc14\uc2a4\ud1a0\ud3f4\uc774\uc5c8\ub2e4."} {"question": "2012\ub144 \ucf00\uc774\ud2f0 \ud398\ub9ac\uac00 \uc2dc\uc791\ud55c \ub124 \ubc88\uc9f8 \uc815\uaddc \uc568\ubc94\uc758 \uc774\ub984\uc740?", "paragraph": "2012\ub144 11\uc6d4 \ud398\ub9ac\ub294 \ub124 \ubc88\uc9f8 \uc815\uaddc \uc568\ubc94 Prism \uc791\uc5c5\uc744 \uc2dc\uc791\ud588\ub2e4. \u300a\ube4c\ubcf4\ub4dc\u300b\uc640\uc758 \uc778\ud130\ubdf0\uc5d0\uc11c \"\ub0b4\uac00 \ub2e4\uc74c\uc5d0 \ub9cc\ub4e4\uace0 \uc2f6\uc740 \uc74c\ubc18\uc744 \uc815\ud655\ud788 \uc54c\uace0\uc788\ub2e4. \uc74c\ubc18 \ucee4\ubc84\uc758 \uc0c9\uac10\uacfc \ud1a4, \uc2ec\uc9c0\uc5b4 \ub2e4\uc74c\uc5d0 \ud574\uc57c \ub420 \ud22c\uc5b4\uc758 \uc720\ud615\uae4c\uc9c0 \uc54c\uace0\uc788\ub2e4. \ub0b4 \uba38\ub9bf\uc18d\uc5d0 \uc788\ub294 \uad6c\uc0c1\uc774 \uc2e4\uc81c\ub85c \uc2e4\ud604\ub41c\ub2e4\uba74 \uc815\ub9d0 \uc88b\uc744 \uac83\uc774\ub2e4\"\uc774\ub77c\uace0 \uc568\ubc94\uc5d0 \ub300\ud574 \ub9d0\ud588\ub2e4. 2012\ub144 6\uc6d4 \u300a\ubcf4\uadf8\u300b\uc640 \uc778\ud130\ubdf0\ub97c \ud1b5\ud574 \ud398\ub9ac\uc758 \uacb0\ud63c \uc0dd\ud65c\uc774 \ub05d\ub0a8\uacfc \ub3d9\uc2dc\uc5d0 \uc0c8 \uc568\ubc94\uc5d0\ub294 \"\uc5b4\ub450\uc6b4 \uc694\uc18c\"\ub4e4\uc774 \uac00\ub4dd\ud560 \uac83\uc774\ub77c\uace0 \uc804\ud588\ub294\ub370, 2013 MTV \ube44\ub514\uc624 \ubba4\uc9c1 \uc5b4\uc6cc\ub4dc \uc911 \uc790\uae30\uc790\uc2e0\uc744 \ubc18\uc131\ud558\ub294 \uae30\uac04 \ub3d9\uc548 \uc568\ubc94\uc758 \ubc29\ud5a5\uc774 \ubcc0\uacbd\ub418\uc5c8\ub2e4\uace0 \ub9d0\ud588\ub2e4. \uc774\uc5b4 \"\ub098\ub294 \uc5c4\uccad\ub09c \ud504\ub9ac\uc998(Prismatic)\uc744 \ub290\uaf08\ub2e4\"\uace0 \ub9d0\ud558\uba70 \uc5ec\uae30\uc11c \uc568\ubc94 \uc81c\ubaa9\uc758 \uc601\uac10\uc744 \uc5bb\uc5c8\ub2e4\uace0 \ud55c\ub2e4. 2013\ub144 8\uc6d4 10\uc77c \uc568\ubc94\uc758 \ub9ac\ub4dc \uc2f1\uae00 \u3008Roar\u3009\uac00 \ubc1c\ub9e4\ub418\uc5c8\ub2e4. MTV \ube44\ub514\uc624 \ubba4\uc9c1 \uc5b4\uc6cc\ub4dc\uc5d0\uc11c \ucc98\uc74c \uacf5\uc5f0\uc744 \uc120\ubcf4\uc600\uc73c\uba70 \ube4c\ubcf4\ub4dc \ud56b 100 1\uc704\uc5d0 \uc62c\ub790\ub2e4. 2013\ub144 10\uc6d4 16\uc77c\uc5d0\ub294 \ub450 \ubc88\uc9f8 \uc2f1\uae00 \u3008Unconditionally\u3009\uc774 \ubc1c\ub9e4\ub418\uc5c8\uace0, \ubbf8\uad6d\uc5d0\uc11c \ucd5c\uace0 14\uc704\uae4c\uc9c0 \uc62c\ub790\ub2e4.", "answer": "Prism", "sentence": "2012\ub144 11\uc6d4 \ud398\ub9ac\ub294 \ub124 \ubc88\uc9f8 \uc815\uaddc \uc568\ubc94 Prism \uc791\uc5c5\uc744 \uc2dc\uc791\ud588\ub2e4.", "paragraph_sentence": " 2012\ub144 11\uc6d4 \ud398\ub9ac\ub294 \ub124 \ubc88\uc9f8 \uc815\uaddc \uc568\ubc94 Prism \uc791\uc5c5\uc744 \uc2dc\uc791\ud588\ub2e4. \u300a\ube4c\ubcf4\ub4dc\u300b\uc640\uc758 \uc778\ud130\ubdf0\uc5d0\uc11c \"\ub0b4\uac00 \ub2e4\uc74c\uc5d0 \ub9cc\ub4e4\uace0 \uc2f6\uc740 \uc74c\ubc18\uc744 \uc815\ud655\ud788 \uc54c\uace0\uc788\ub2e4. \uc74c\ubc18 \ucee4\ubc84\uc758 \uc0c9\uac10\uacfc \ud1a4, \uc2ec\uc9c0\uc5b4 \ub2e4\uc74c\uc5d0 \ud574\uc57c \ub420 \ud22c\uc5b4\uc758 \uc720\ud615\uae4c\uc9c0 \uc54c\uace0\uc788\ub2e4. \ub0b4 \uba38\ub9bf\uc18d\uc5d0 \uc788\ub294 \uad6c\uc0c1\uc774 \uc2e4\uc81c\ub85c \uc2e4\ud604\ub41c\ub2e4\uba74 \uc815\ub9d0 \uc88b\uc744 \uac83\uc774\ub2e4\" \uc774\ub77c\uace0 \uc568\ubc94\uc5d0 \ub300\ud574 \ub9d0\ud588\ub2e4. 2012\ub144 6\uc6d4 \u300a\ubcf4\uadf8\u300b\uc640 \uc778\ud130\ubdf0\ub97c \ud1b5\ud574 \ud398\ub9ac\uc758 \uacb0\ud63c \uc0dd\ud65c\uc774 \ub05d\ub0a8\uacfc \ub3d9\uc2dc\uc5d0 \uc0c8 \uc568\ubc94\uc5d0\ub294 \"\uc5b4\ub450\uc6b4 \uc694\uc18c\"\ub4e4\uc774 \uac00\ub4dd\ud560 \uac83\uc774\ub77c\uace0 \uc804\ud588\ub294\ub370, 2013 MTV \ube44\ub514\uc624 \ubba4\uc9c1 \uc5b4\uc6cc\ub4dc \uc911 \uc790\uae30\uc790\uc2e0\uc744 \ubc18\uc131\ud558\ub294 \uae30\uac04 \ub3d9\uc548 \uc568\ubc94\uc758 \ubc29\ud5a5\uc774 \ubcc0\uacbd\ub418\uc5c8\ub2e4\uace0 \ub9d0\ud588\ub2e4. \uc774\uc5b4 \"\ub098\ub294 \uc5c4\uccad\ub09c \ud504\ub9ac\uc998(Prismatic)\uc744 \ub290\uaf08\ub2e4\"\uace0 \ub9d0\ud558\uba70 \uc5ec\uae30\uc11c \uc568\ubc94 \uc81c\ubaa9\uc758 \uc601\uac10\uc744 \uc5bb\uc5c8\ub2e4\uace0 \ud55c\ub2e4. 2013\ub144 8\uc6d4 10\uc77c \uc568\ubc94\uc758 \ub9ac\ub4dc \uc2f1\uae00 \u3008Roar\u3009\uac00 \ubc1c\ub9e4\ub418\uc5c8\ub2e4. MTV \ube44\ub514\uc624 \ubba4\uc9c1 \uc5b4\uc6cc\ub4dc\uc5d0\uc11c \ucc98\uc74c \uacf5\uc5f0\uc744 \uc120\ubcf4\uc600\uc73c\uba70 \ube4c\ubcf4\ub4dc \ud56b 100 1\uc704\uc5d0 \uc62c\ub790\ub2e4. 2013\ub144 10\uc6d4 16\uc77c\uc5d0\ub294 \ub450 \ubc88\uc9f8 \uc2f1\uae00 \u3008Unconditionally\u3009\uc774 \ubc1c\ub9e4\ub418\uc5c8\uace0, \ubbf8\uad6d\uc5d0\uc11c \ucd5c\uace0 14\uc704\uae4c\uc9c0 \uc62c\ub790\ub2e4.", "paragraph_answer": "2012\ub144 11\uc6d4 \ud398\ub9ac\ub294 \ub124 \ubc88\uc9f8 \uc815\uaddc \uc568\ubc94 Prism \uc791\uc5c5\uc744 \uc2dc\uc791\ud588\ub2e4. \u300a\ube4c\ubcf4\ub4dc\u300b\uc640\uc758 \uc778\ud130\ubdf0\uc5d0\uc11c \"\ub0b4\uac00 \ub2e4\uc74c\uc5d0 \ub9cc\ub4e4\uace0 \uc2f6\uc740 \uc74c\ubc18\uc744 \uc815\ud655\ud788 \uc54c\uace0\uc788\ub2e4. \uc74c\ubc18 \ucee4\ubc84\uc758 \uc0c9\uac10\uacfc \ud1a4, \uc2ec\uc9c0\uc5b4 \ub2e4\uc74c\uc5d0 \ud574\uc57c \ub420 \ud22c\uc5b4\uc758 \uc720\ud615\uae4c\uc9c0 \uc54c\uace0\uc788\ub2e4. \ub0b4 \uba38\ub9bf\uc18d\uc5d0 \uc788\ub294 \uad6c\uc0c1\uc774 \uc2e4\uc81c\ub85c \uc2e4\ud604\ub41c\ub2e4\uba74 \uc815\ub9d0 \uc88b\uc744 \uac83\uc774\ub2e4\"\uc774\ub77c\uace0 \uc568\ubc94\uc5d0 \ub300\ud574 \ub9d0\ud588\ub2e4. 2012\ub144 6\uc6d4 \u300a\ubcf4\uadf8\u300b\uc640 \uc778\ud130\ubdf0\ub97c \ud1b5\ud574 \ud398\ub9ac\uc758 \uacb0\ud63c \uc0dd\ud65c\uc774 \ub05d\ub0a8\uacfc \ub3d9\uc2dc\uc5d0 \uc0c8 \uc568\ubc94\uc5d0\ub294 \"\uc5b4\ub450\uc6b4 \uc694\uc18c\"\ub4e4\uc774 \uac00\ub4dd\ud560 \uac83\uc774\ub77c\uace0 \uc804\ud588\ub294\ub370, 2013 MTV \ube44\ub514\uc624 \ubba4\uc9c1 \uc5b4\uc6cc\ub4dc \uc911 \uc790\uae30\uc790\uc2e0\uc744 \ubc18\uc131\ud558\ub294 \uae30\uac04 \ub3d9\uc548 \uc568\ubc94\uc758 \ubc29\ud5a5\uc774 \ubcc0\uacbd\ub418\uc5c8\ub2e4\uace0 \ub9d0\ud588\ub2e4. \uc774\uc5b4 \"\ub098\ub294 \uc5c4\uccad\ub09c \ud504\ub9ac\uc998(Prismatic)\uc744 \ub290\uaf08\ub2e4\"\uace0 \ub9d0\ud558\uba70 \uc5ec\uae30\uc11c \uc568\ubc94 \uc81c\ubaa9\uc758 \uc601\uac10\uc744 \uc5bb\uc5c8\ub2e4\uace0 \ud55c\ub2e4. 2013\ub144 8\uc6d4 10\uc77c \uc568\ubc94\uc758 \ub9ac\ub4dc \uc2f1\uae00 \u3008Roar\u3009\uac00 \ubc1c\ub9e4\ub418\uc5c8\ub2e4. MTV \ube44\ub514\uc624 \ubba4\uc9c1 \uc5b4\uc6cc\ub4dc\uc5d0\uc11c \ucc98\uc74c \uacf5\uc5f0\uc744 \uc120\ubcf4\uc600\uc73c\uba70 \ube4c\ubcf4\ub4dc \ud56b 100 1\uc704\uc5d0 \uc62c\ub790\ub2e4. 2013\ub144 10\uc6d4 16\uc77c\uc5d0\ub294 \ub450 \ubc88\uc9f8 \uc2f1\uae00 \u3008Unconditionally\u3009\uc774 \ubc1c\ub9e4\ub418\uc5c8\uace0, \ubbf8\uad6d\uc5d0\uc11c \ucd5c\uace0 14\uc704\uae4c\uc9c0 \uc62c\ub790\ub2e4.", "sentence_answer": "2012\ub144 11\uc6d4 \ud398\ub9ac\ub294 \ub124 \ubc88\uc9f8 \uc815\uaddc \uc568\ubc94 Prism \uc791\uc5c5\uc744 \uc2dc\uc791\ud588\ub2e4.", "paragraph_id": "6209726-14-0", "paragraph_question": "question: 2012\ub144 \ucf00\uc774\ud2f0 \ud398\ub9ac\uac00 \uc2dc\uc791\ud55c \ub124 \ubc88\uc9f8 \uc815\uaddc \uc568\ubc94\uc758 \uc774\ub984\uc740?, context: 2012\ub144 11\uc6d4 \ud398\ub9ac\ub294 \ub124 \ubc88\uc9f8 \uc815\uaddc \uc568\ubc94 Prism \uc791\uc5c5\uc744 \uc2dc\uc791\ud588\ub2e4. \u300a\ube4c\ubcf4\ub4dc\u300b\uc640\uc758 \uc778\ud130\ubdf0\uc5d0\uc11c \"\ub0b4\uac00 \ub2e4\uc74c\uc5d0 \ub9cc\ub4e4\uace0 \uc2f6\uc740 \uc74c\ubc18\uc744 \uc815\ud655\ud788 \uc54c\uace0\uc788\ub2e4. \uc74c\ubc18 \ucee4\ubc84\uc758 \uc0c9\uac10\uacfc \ud1a4, \uc2ec\uc9c0\uc5b4 \ub2e4\uc74c\uc5d0 \ud574\uc57c \ub420 \ud22c\uc5b4\uc758 \uc720\ud615\uae4c\uc9c0 \uc54c\uace0\uc788\ub2e4. \ub0b4 \uba38\ub9bf\uc18d\uc5d0 \uc788\ub294 \uad6c\uc0c1\uc774 \uc2e4\uc81c\ub85c \uc2e4\ud604\ub41c\ub2e4\uba74 \uc815\ub9d0 \uc88b\uc744 \uac83\uc774\ub2e4\"\uc774\ub77c\uace0 \uc568\ubc94\uc5d0 \ub300\ud574 \ub9d0\ud588\ub2e4. 2012\ub144 6\uc6d4 \u300a\ubcf4\uadf8\u300b\uc640 \uc778\ud130\ubdf0\ub97c \ud1b5\ud574 \ud398\ub9ac\uc758 \uacb0\ud63c \uc0dd\ud65c\uc774 \ub05d\ub0a8\uacfc \ub3d9\uc2dc\uc5d0 \uc0c8 \uc568\ubc94\uc5d0\ub294 \"\uc5b4\ub450\uc6b4 \uc694\uc18c\"\ub4e4\uc774 \uac00\ub4dd\ud560 \uac83\uc774\ub77c\uace0 \uc804\ud588\ub294\ub370, 2013 MTV \ube44\ub514\uc624 \ubba4\uc9c1 \uc5b4\uc6cc\ub4dc \uc911 \uc790\uae30\uc790\uc2e0\uc744 \ubc18\uc131\ud558\ub294 \uae30\uac04 \ub3d9\uc548 \uc568\ubc94\uc758 \ubc29\ud5a5\uc774 \ubcc0\uacbd\ub418\uc5c8\ub2e4\uace0 \ub9d0\ud588\ub2e4. \uc774\uc5b4 \"\ub098\ub294 \uc5c4\uccad\ub09c \ud504\ub9ac\uc998(Prismatic)\uc744 \ub290\uaf08\ub2e4\"\uace0 \ub9d0\ud558\uba70 \uc5ec\uae30\uc11c \uc568\ubc94 \uc81c\ubaa9\uc758 \uc601\uac10\uc744 \uc5bb\uc5c8\ub2e4\uace0 \ud55c\ub2e4. 2013\ub144 8\uc6d4 10\uc77c \uc568\ubc94\uc758 \ub9ac\ub4dc \uc2f1\uae00 \u3008Roar\u3009\uac00 \ubc1c\ub9e4\ub418\uc5c8\ub2e4. MTV \ube44\ub514\uc624 \ubba4\uc9c1 \uc5b4\uc6cc\ub4dc\uc5d0\uc11c \ucc98\uc74c \uacf5\uc5f0\uc744 \uc120\ubcf4\uc600\uc73c\uba70 \ube4c\ubcf4\ub4dc \ud56b 100 1\uc704\uc5d0 \uc62c\ub790\ub2e4. 2013\ub144 10\uc6d4 16\uc77c\uc5d0\ub294 \ub450 \ubc88\uc9f8 \uc2f1\uae00 \u3008Unconditionally\u3009\uc774 \ubc1c\ub9e4\ub418\uc5c8\uace0, \ubbf8\uad6d\uc5d0\uc11c \ucd5c\uace0 14\uc704\uae4c\uc9c0 \uc62c\ub790\ub2e4."} {"question": "\ud22c\ud22c 2\uc9d1\uc5d0\uc11c 1\uc704 \ud6c4\ubcf4\uc5d0 \uc624\ub978 \uace1\uc740 \ubb34\uc5c7\uc778\uac00\uc694?", "paragraph": "\uc624\uc9c0\ud6c8, \uc720\ud604\uc7ac\uac00 \uc785\ub300\ub97c \ud558\uba74\uc11c \uae40\uc9c0\ud6c8\uc740 1995\ub144 6\uc6d4\uc5d0 \uac1d\uc6d0 \uba64\ubc84 \uae40\uc900, \uc784\uc131\uc740, \ud669\ud61c\uc601\uacfc \ud568\uaed8 \ud22c\ud22c 2\uc9d1\uc758 \uba64\ubc84\ub85c \ud65c\ub3d9\ud558\uc600\ub2e4. \ud22c\ud22c 2\uc9d1 \uc2dc\uae30\uc778 1995\ub144\uc5d0\ub294 \uadf8\ub8f9\uc758 \ub9ac\ub354\uc640 \ubcf4\uceec\ub85c\uc11c \ud65c\ub3d9\ud558\uc600\uace0, \uc5e0\ub137\uc758 '\uace0 \uc5e0\ub137 \uace0'\ub85c \uccab MC \ub370\ubdd4\ub97c \ud558\uac8c \ub418\uc5c8\ub2e4. (2\uc9d1 \uc218\ub85d\uace1 \uc911 \uc8fc\ub41c \ubc29\uc1a1 \ud65c\ub3d9\uace1\uc740 '\ubc14\ub78c\ub09c \uc5ec\uc790'\uc640 '\ub2c8\uac00 \ub0b4\uac83\uc774 \ub418\uac08\uc218\ub85d'\uc774\uba70 2\uc9d1 \ub178\ub798\uc758 \ub300\ubd80\ubd84\uc744 \uae40\uc9c0\ud6c8\uc774 \ubd88\ub800\ub2e4. \ub098\uba38\uc9c0 \uba64\ubc84\ub4e4\uc740 \uac1d\uc6d0 \uba64\ubc84 \ud615\uc2dd\uc73c\ub85c \uc568\ubc94 \ub2f9 1-2\uace1\uc5d0\ub9cc \ucc38\uc5ec\ud558\uc600\ub2e4. \ubc14\ub78c\ub09c \uc5ec\uc790\ub294 1\uc704 \ud6c4\ubcf4\uc5d0\ub3c4 \uc624\ub974\ub294 \ub4f1 \ucd5c\uc0c1\uc704\uad8c\uc744 \uc720\uc9c0\ud558\uc600\ub2e4. \uadf8\ub294 1996\ub144 1\uc6d4 5\uc77c\uc790 MBC \uc778\uae30\uac00\uc694 BEST 50\uc5d0\uc11c '\ub2c8\uac00 \ub0b4\uac83\uc774 \ub418\uac08\uc218\ub85d'\uc758 \ubb34\ub300 \ub97c \ub05d\uc73c\ub85c \ud22c\ud22c \ud65c\ub3d9\uc744 \ub9c8\uce58\uace0 \uad70\ub300\uc5d0 \uc785\ub300\ud558\uac8c \ub418\uc5c8\ub2e4. \ud65c\ub3d9 \uc911\uc5d0 \uc785\ub300\ud55c\ud130\ub77c \uadf8\uc758 \uc785\ub300 \ud6c4\uc5d0\ub3c4 \ub2c8\uac00 \ub0b4\uac83\uc774 \ub418\uac08\uc218\ub85d\uc740 \ud55c\ub3d9\uc548 \uac00\uc694\ud1b110\uc5d0 \uc21c\uc704\uad8c\uc5d0 \uba38\ubb3c\ub800\ub2e4.", "answer": "\ubc14\ub78c\ub09c \uc5ec\uc790", "sentence": "(2\uc9d1 \uc218\ub85d\uace1 \uc911 \uc8fc\ub41c \ubc29\uc1a1 \ud65c\ub3d9\uace1\uc740 ' \ubc14\ub78c\ub09c \uc5ec\uc790 '\uc640 '\ub2c8\uac00 \ub0b4\uac83\uc774 \ub418\uac08\uc218\ub85d'\uc774\uba70 2\uc9d1 \ub178\ub798\uc758 \ub300\ubd80\ubd84\uc744 \uae40\uc9c0\ud6c8\uc774 \ubd88\ub800\ub2e4.", "paragraph_sentence": "\uc624\uc9c0\ud6c8, \uc720\ud604\uc7ac\uac00 \uc785\ub300\ub97c \ud558\uba74\uc11c \uae40\uc9c0\ud6c8\uc740 1995\ub144 6\uc6d4\uc5d0 \uac1d\uc6d0 \uba64\ubc84 \uae40\uc900, \uc784\uc131\uc740, \ud669\ud61c\uc601\uacfc \ud568\uaed8 \ud22c\ud22c 2\uc9d1\uc758 \uba64\ubc84\ub85c \ud65c\ub3d9\ud558\uc600\ub2e4. \ud22c\ud22c 2\uc9d1 \uc2dc\uae30\uc778 1995\ub144\uc5d0\ub294 \uadf8\ub8f9\uc758 \ub9ac\ub354\uc640 \ubcf4\uceec\ub85c\uc11c \ud65c\ub3d9\ud558\uc600\uace0, \uc5e0\ub137\uc758 '\uace0 \uc5e0\ub137 \uace0'\ub85c \uccab MC \ub370\ubdd4\ub97c \ud558\uac8c \ub418\uc5c8\ub2e4. (2\uc9d1 \uc218\ub85d\uace1 \uc911 \uc8fc\ub41c \ubc29\uc1a1 \ud65c\ub3d9\uace1\uc740 ' \ubc14\ub78c\ub09c \uc5ec\uc790 '\uc640 '\ub2c8\uac00 \ub0b4\uac83\uc774 \ub418\uac08\uc218\ub85d'\uc774\uba70 2\uc9d1 \ub178\ub798\uc758 \ub300\ubd80\ubd84\uc744 \uae40\uc9c0\ud6c8\uc774 \ubd88\ub800\ub2e4. \ub098\uba38\uc9c0 \uba64\ubc84\ub4e4\uc740 \uac1d\uc6d0 \uba64\ubc84 \ud615\uc2dd\uc73c\ub85c \uc568\ubc94 \ub2f9 1-2\uace1\uc5d0\ub9cc \ucc38\uc5ec\ud558\uc600\ub2e4. \ubc14\ub78c\ub09c \uc5ec\uc790\ub294 1\uc704 \ud6c4\ubcf4\uc5d0\ub3c4 \uc624\ub974\ub294 \ub4f1 \ucd5c\uc0c1\uc704\uad8c\uc744 \uc720\uc9c0\ud558\uc600\ub2e4. \uadf8\ub294 1996\ub144 1\uc6d4 5\uc77c\uc790 MBC \uc778\uae30\uac00\uc694 BEST 50\uc5d0\uc11c '\ub2c8\uac00 \ub0b4\uac83\uc774 \ub418\uac08\uc218\ub85d'\uc758 \ubb34\ub300 \ub97c \ub05d\uc73c\ub85c \ud22c\ud22c \ud65c\ub3d9\uc744 \ub9c8\uce58\uace0 \uad70\ub300\uc5d0 \uc785\ub300\ud558\uac8c \ub418\uc5c8\ub2e4. \ud65c\ub3d9 \uc911\uc5d0 \uc785\ub300\ud55c\ud130\ub77c \uadf8\uc758 \uc785\ub300 \ud6c4\uc5d0\ub3c4 \ub2c8\uac00 \ub0b4\uac83\uc774 \ub418\uac08\uc218\ub85d\uc740 \ud55c\ub3d9\uc548 \uac00\uc694\ud1b110\uc5d0 \uc21c\uc704\uad8c\uc5d0 \uba38\ubb3c\ub800\ub2e4.", "paragraph_answer": "\uc624\uc9c0\ud6c8, \uc720\ud604\uc7ac\uac00 \uc785\ub300\ub97c \ud558\uba74\uc11c \uae40\uc9c0\ud6c8\uc740 1995\ub144 6\uc6d4\uc5d0 \uac1d\uc6d0 \uba64\ubc84 \uae40\uc900, \uc784\uc131\uc740, \ud669\ud61c\uc601\uacfc \ud568\uaed8 \ud22c\ud22c 2\uc9d1\uc758 \uba64\ubc84\ub85c \ud65c\ub3d9\ud558\uc600\ub2e4. \ud22c\ud22c 2\uc9d1 \uc2dc\uae30\uc778 1995\ub144\uc5d0\ub294 \uadf8\ub8f9\uc758 \ub9ac\ub354\uc640 \ubcf4\uceec\ub85c\uc11c \ud65c\ub3d9\ud558\uc600\uace0, \uc5e0\ub137\uc758 '\uace0 \uc5e0\ub137 \uace0'\ub85c \uccab MC \ub370\ubdd4\ub97c \ud558\uac8c \ub418\uc5c8\ub2e4. (2\uc9d1 \uc218\ub85d\uace1 \uc911 \uc8fc\ub41c \ubc29\uc1a1 \ud65c\ub3d9\uace1\uc740 ' \ubc14\ub78c\ub09c \uc5ec\uc790 '\uc640 '\ub2c8\uac00 \ub0b4\uac83\uc774 \ub418\uac08\uc218\ub85d'\uc774\uba70 2\uc9d1 \ub178\ub798\uc758 \ub300\ubd80\ubd84\uc744 \uae40\uc9c0\ud6c8\uc774 \ubd88\ub800\ub2e4. \ub098\uba38\uc9c0 \uba64\ubc84\ub4e4\uc740 \uac1d\uc6d0 \uba64\ubc84 \ud615\uc2dd\uc73c\ub85c \uc568\ubc94 \ub2f9 1-2\uace1\uc5d0\ub9cc \ucc38\uc5ec\ud558\uc600\ub2e4. \ubc14\ub78c\ub09c \uc5ec\uc790\ub294 1\uc704 \ud6c4\ubcf4\uc5d0\ub3c4 \uc624\ub974\ub294 \ub4f1 \ucd5c\uc0c1\uc704\uad8c\uc744 \uc720\uc9c0\ud558\uc600\ub2e4. \uadf8\ub294 1996\ub144 1\uc6d4 5\uc77c\uc790 MBC \uc778\uae30\uac00\uc694 BEST 50\uc5d0\uc11c '\ub2c8\uac00 \ub0b4\uac83\uc774 \ub418\uac08\uc218\ub85d'\uc758 \ubb34\ub300 \ub97c \ub05d\uc73c\ub85c \ud22c\ud22c \ud65c\ub3d9\uc744 \ub9c8\uce58\uace0 \uad70\ub300\uc5d0 \uc785\ub300\ud558\uac8c \ub418\uc5c8\ub2e4. \ud65c\ub3d9 \uc911\uc5d0 \uc785\ub300\ud55c\ud130\ub77c \uadf8\uc758 \uc785\ub300 \ud6c4\uc5d0\ub3c4 \ub2c8\uac00 \ub0b4\uac83\uc774 \ub418\uac08\uc218\ub85d\uc740 \ud55c\ub3d9\uc548 \uac00\uc694\ud1b110\uc5d0 \uc21c\uc704\uad8c\uc5d0 \uba38\ubb3c\ub800\ub2e4.", "sentence_answer": "(2\uc9d1 \uc218\ub85d\uace1 \uc911 \uc8fc\ub41c \ubc29\uc1a1 \ud65c\ub3d9\uace1\uc740 ' \ubc14\ub78c\ub09c \uc5ec\uc790 '\uc640 '\ub2c8\uac00 \ub0b4\uac83\uc774 \ub418\uac08\uc218\ub85d'\uc774\uba70 2\uc9d1 \ub178\ub798\uc758 \ub300\ubd80\ubd84\uc744 \uae40\uc9c0\ud6c8\uc774 \ubd88\ub800\ub2e4.", "paragraph_id": "6488568-1-2", "paragraph_question": "question: \ud22c\ud22c 2\uc9d1\uc5d0\uc11c 1\uc704 \ud6c4\ubcf4\uc5d0 \uc624\ub978 \uace1\uc740 \ubb34\uc5c7\uc778\uac00\uc694?, context: \uc624\uc9c0\ud6c8, \uc720\ud604\uc7ac\uac00 \uc785\ub300\ub97c \ud558\uba74\uc11c \uae40\uc9c0\ud6c8\uc740 1995\ub144 6\uc6d4\uc5d0 \uac1d\uc6d0 \uba64\ubc84 \uae40\uc900, \uc784\uc131\uc740, \ud669\ud61c\uc601\uacfc \ud568\uaed8 \ud22c\ud22c 2\uc9d1\uc758 \uba64\ubc84\ub85c \ud65c\ub3d9\ud558\uc600\ub2e4. \ud22c\ud22c 2\uc9d1 \uc2dc\uae30\uc778 1995\ub144\uc5d0\ub294 \uadf8\ub8f9\uc758 \ub9ac\ub354\uc640 \ubcf4\uceec\ub85c\uc11c \ud65c\ub3d9\ud558\uc600\uace0, \uc5e0\ub137\uc758 '\uace0 \uc5e0\ub137 \uace0'\ub85c \uccab MC \ub370\ubdd4\ub97c \ud558\uac8c \ub418\uc5c8\ub2e4. (2\uc9d1 \uc218\ub85d\uace1 \uc911 \uc8fc\ub41c \ubc29\uc1a1 \ud65c\ub3d9\uace1\uc740 '\ubc14\ub78c\ub09c \uc5ec\uc790'\uc640 '\ub2c8\uac00 \ub0b4\uac83\uc774 \ub418\uac08\uc218\ub85d'\uc774\uba70 2\uc9d1 \ub178\ub798\uc758 \ub300\ubd80\ubd84\uc744 \uae40\uc9c0\ud6c8\uc774 \ubd88\ub800\ub2e4. \ub098\uba38\uc9c0 \uba64\ubc84\ub4e4\uc740 \uac1d\uc6d0 \uba64\ubc84 \ud615\uc2dd\uc73c\ub85c \uc568\ubc94 \ub2f9 1-2\uace1\uc5d0\ub9cc \ucc38\uc5ec\ud558\uc600\ub2e4. \ubc14\ub78c\ub09c \uc5ec\uc790\ub294 1\uc704 \ud6c4\ubcf4\uc5d0\ub3c4 \uc624\ub974\ub294 \ub4f1 \ucd5c\uc0c1\uc704\uad8c\uc744 \uc720\uc9c0\ud558\uc600\ub2e4. \uadf8\ub294 1996\ub144 1\uc6d4 5\uc77c\uc790 MBC \uc778\uae30\uac00\uc694 BEST 50\uc5d0\uc11c '\ub2c8\uac00 \ub0b4\uac83\uc774 \ub418\uac08\uc218\ub85d'\uc758 \ubb34\ub300 \ub97c \ub05d\uc73c\ub85c \ud22c\ud22c \ud65c\ub3d9\uc744 \ub9c8\uce58\uace0 \uad70\ub300\uc5d0 \uc785\ub300\ud558\uac8c \ub418\uc5c8\ub2e4. \ud65c\ub3d9 \uc911\uc5d0 \uc785\ub300\ud55c\ud130\ub77c \uadf8\uc758 \uc785\ub300 \ud6c4\uc5d0\ub3c4 \ub2c8\uac00 \ub0b4\uac83\uc774 \ub418\uac08\uc218\ub85d\uc740 \ud55c\ub3d9\uc548 \uac00\uc694\ud1b110\uc5d0 \uc21c\uc704\uad8c\uc5d0 \uba38\ubb3c\ub800\ub2e4."} {"question": "\uce74\uc640\ub9ac\uc988\ud30c\ub294 \uc9c0\ud558\ub4dc\ub97c 6\ubc88 \uc9f8 \ubb34\uc5c7\uc73c\ub85c \uc0bc\uc558\uc2b5\ub2c8\uae4c?", "paragraph": "\uce74\uc640\ub9ac\uc988\ud30c\ub294 \uc804\ud22c\uc801\uc778 \ud589\ub3d9\uc8fc\uc758\uc790\uc5ec\uc11c, \uc9c0\ud558\ub4dc(jihad \uc8fc\ub85c \u2018\uc131\uc804\u2019\uc73c\ub85c \ubc88\uc5ed\ud55c\ub2e4)\ub97c \uc5ec\uc12f \ubc88\uc9f8 \u2018\uc2e0\uc559\uc758 \uae30\ub465\u2019\uc73c\ub85c \uc0bc\uc558\ub2e4. 661\ub144 \uc774\ub4e4\uc740 \uc54c\ub9ac\ub97c \uc0b4\ud574\ud588\ub2e4. \uce74\uc640\ub9ac\uc988\ud30c\ub294 \uba54\uce74\uc758 \ubd80\uc871 \uafb8\ub77c\uc774\uc26c\uc758 \uc790\uc190\ub9cc\uc774 \uce7c\ub9ac\ud30c\uac00 \ub420 \uc218 \uc788\ub2e4\ub294 \uc218\ub2c8\ud30c\uc758 \uc804\ud1b5\uc801 \uacac\ud574\ub97c \ube44\ub09c\ud558\uace0, \ub3c5\uc2e4\ud55c \ubb34\uc2ac\ub9bc\uc774\uba74 \ub204\uad6c\ub098 \uce7c\ub9ac\ud30c\uac00 \ub420 \uc218 \uc788\uc5b4\uc57c \ud55c\ub2e4\uace0 \uc8fc\uc7a5\ud588\ub2e4. \ub9cc\ubbfc \ud3c9\ub4f1\uc8fc\uc758\uc801 \uc785\uc7a5\uacfc \uc544\ub78d \uadc0\uc871\uce35\uc5d0 \ub300\ud55c \uc801\uac1c\uc2ec \ub54c\ubb38\uc5d0 \ubca0\ub4dc\uc708\uacfc \ube44\uc544\ub78d\uacc4 \ubb34\uc2ac\ub9bc \ucd94\uc885\uc790\ub97c \uc5bb\uc744 \uc218 \uc788\uc5c8\uc9c0\ub9cc \ub0b4\ubd84\uc73c\ub85c \ub4b7\ub0a0 \uc800\uc808\ub85c \uc57d\ud654\ub410\ub2e4. \ud604\uc7ac\ub294 \uc54c\uc81c\ub9ac\uc640 \ud280\ub2c8\uc9c0\uc758 \ubca0\ub974\ubca0\ub974 \uc9c0\uc5ed, \ub3d9\uc544\ud504\ub9ac\uce74 \ud0c4\uc790\ub2c8\uc544\uc640 \uc544\ub77c\ube44\uc544 \ubc18\ub3c4\uc758 \uc624\ub9cc\uc5d0 \uc870\uae08 \ub0a8\uc544\uc788\uc744 \ubfd0\uc774\ub2e4. \uadf8\ub7ec\ub098 \uc774\ub4e4\uc758 \uccad\uad50\ub3c4\uc801\uc778 \uc815\uc2e0\uc740 18\uc138\uae30 \uc0ac\uc6b0\ub514\uc544\ub77c\ube44\uc544 \uac74\uad6d\uc758 \uae30\ubc18\uc774 \ub41c \uc640\ud558\ube0c(Wahhab) \uc6b4\ub3d9\uc758 \ubc14\ud0d5\uc774 \ub418\uae30\ub3c4 \ud588\ub2e4.", "answer": "\uc2e0\uc559\uc758 \uae30\ub465", "sentence": "\uce74\uc640\ub9ac\uc988\ud30c\ub294 \uc804\ud22c\uc801\uc778 \ud589\ub3d9\uc8fc\uc758\uc790\uc5ec\uc11c, \uc9c0\ud558\ub4dc(jihad \uc8fc\ub85c \u2018\uc131\uc804\u2019\uc73c\ub85c \ubc88\uc5ed\ud55c\ub2e4)\ub97c \uc5ec\uc12f \ubc88\uc9f8 \u2018 \uc2e0\uc559\uc758 \uae30\ub465 \u2019\uc73c\ub85c \uc0bc\uc558\ub2e4.", "paragraph_sentence": " \uce74\uc640\ub9ac\uc988\ud30c\ub294 \uc804\ud22c\uc801\uc778 \ud589\ub3d9\uc8fc\uc758\uc790\uc5ec\uc11c, \uc9c0\ud558\ub4dc(jihad \uc8fc\ub85c \u2018\uc131\uc804\u2019\uc73c\ub85c \ubc88\uc5ed\ud55c\ub2e4)\ub97c \uc5ec\uc12f \ubc88\uc9f8 \u2018 \uc2e0\uc559\uc758 \uae30\ub465 \u2019\uc73c\ub85c \uc0bc\uc558\ub2e4. 661\ub144 \uc774\ub4e4\uc740 \uc54c\ub9ac\ub97c \uc0b4\ud574\ud588\ub2e4. \uce74\uc640\ub9ac\uc988\ud30c\ub294 \uba54\uce74\uc758 \ubd80\uc871 \uafb8\ub77c\uc774\uc26c\uc758 \uc790\uc190\ub9cc\uc774 \uce7c\ub9ac\ud30c\uac00 \ub420 \uc218 \uc788\ub2e4\ub294 \uc218\ub2c8\ud30c\uc758 \uc804\ud1b5\uc801 \uacac\ud574\ub97c \ube44\ub09c\ud558\uace0, \ub3c5\uc2e4\ud55c \ubb34\uc2ac\ub9bc\uc774\uba74 \ub204\uad6c\ub098 \uce7c\ub9ac\ud30c\uac00 \ub420 \uc218 \uc788\uc5b4\uc57c \ud55c\ub2e4\uace0 \uc8fc\uc7a5\ud588\ub2e4. \ub9cc\ubbfc \ud3c9\ub4f1\uc8fc\uc758\uc801 \uc785\uc7a5\uacfc \uc544\ub78d \uadc0\uc871\uce35\uc5d0 \ub300\ud55c \uc801\uac1c\uc2ec \ub54c\ubb38\uc5d0 \ubca0\ub4dc\uc708\uacfc \ube44\uc544\ub78d\uacc4 \ubb34\uc2ac\ub9bc \ucd94\uc885\uc790\ub97c \uc5bb\uc744 \uc218 \uc788\uc5c8\uc9c0\ub9cc \ub0b4\ubd84\uc73c\ub85c \ub4b7\ub0a0 \uc800\uc808\ub85c \uc57d\ud654\ub410\ub2e4. \ud604\uc7ac\ub294 \uc54c\uc81c\ub9ac\uc640 \ud280\ub2c8\uc9c0\uc758 \ubca0\ub974\ubca0\ub974 \uc9c0\uc5ed, \ub3d9\uc544\ud504\ub9ac\uce74 \ud0c4\uc790\ub2c8\uc544\uc640 \uc544\ub77c\ube44\uc544 \ubc18\ub3c4\uc758 \uc624\ub9cc\uc5d0 \uc870\uae08 \ub0a8\uc544\uc788\uc744 \ubfd0\uc774\ub2e4. \uadf8\ub7ec\ub098 \uc774\ub4e4\uc758 \uccad\uad50\ub3c4\uc801\uc778 \uc815\uc2e0\uc740 18\uc138\uae30 \uc0ac\uc6b0\ub514\uc544\ub77c\ube44\uc544 \uac74\uad6d\uc758 \uae30\ubc18\uc774 \ub41c \uc640\ud558\ube0c(Wahhab) \uc6b4\ub3d9\uc758 \ubc14\ud0d5\uc774 \ub418\uae30\ub3c4 \ud588\ub2e4.", "paragraph_answer": "\uce74\uc640\ub9ac\uc988\ud30c\ub294 \uc804\ud22c\uc801\uc778 \ud589\ub3d9\uc8fc\uc758\uc790\uc5ec\uc11c, \uc9c0\ud558\ub4dc(jihad \uc8fc\ub85c \u2018\uc131\uc804\u2019\uc73c\ub85c \ubc88\uc5ed\ud55c\ub2e4)\ub97c \uc5ec\uc12f \ubc88\uc9f8 \u2018 \uc2e0\uc559\uc758 \uae30\ub465 \u2019\uc73c\ub85c \uc0bc\uc558\ub2e4. 661\ub144 \uc774\ub4e4\uc740 \uc54c\ub9ac\ub97c \uc0b4\ud574\ud588\ub2e4. \uce74\uc640\ub9ac\uc988\ud30c\ub294 \uba54\uce74\uc758 \ubd80\uc871 \uafb8\ub77c\uc774\uc26c\uc758 \uc790\uc190\ub9cc\uc774 \uce7c\ub9ac\ud30c\uac00 \ub420 \uc218 \uc788\ub2e4\ub294 \uc218\ub2c8\ud30c\uc758 \uc804\ud1b5\uc801 \uacac\ud574\ub97c \ube44\ub09c\ud558\uace0, \ub3c5\uc2e4\ud55c \ubb34\uc2ac\ub9bc\uc774\uba74 \ub204\uad6c\ub098 \uce7c\ub9ac\ud30c\uac00 \ub420 \uc218 \uc788\uc5b4\uc57c \ud55c\ub2e4\uace0 \uc8fc\uc7a5\ud588\ub2e4. \ub9cc\ubbfc \ud3c9\ub4f1\uc8fc\uc758\uc801 \uc785\uc7a5\uacfc \uc544\ub78d \uadc0\uc871\uce35\uc5d0 \ub300\ud55c \uc801\uac1c\uc2ec \ub54c\ubb38\uc5d0 \ubca0\ub4dc\uc708\uacfc \ube44\uc544\ub78d\uacc4 \ubb34\uc2ac\ub9bc \ucd94\uc885\uc790\ub97c \uc5bb\uc744 \uc218 \uc788\uc5c8\uc9c0\ub9cc \ub0b4\ubd84\uc73c\ub85c \ub4b7\ub0a0 \uc800\uc808\ub85c \uc57d\ud654\ub410\ub2e4. \ud604\uc7ac\ub294 \uc54c\uc81c\ub9ac\uc640 \ud280\ub2c8\uc9c0\uc758 \ubca0\ub974\ubca0\ub974 \uc9c0\uc5ed, \ub3d9\uc544\ud504\ub9ac\uce74 \ud0c4\uc790\ub2c8\uc544\uc640 \uc544\ub77c\ube44\uc544 \ubc18\ub3c4\uc758 \uc624\ub9cc\uc5d0 \uc870\uae08 \ub0a8\uc544\uc788\uc744 \ubfd0\uc774\ub2e4. \uadf8\ub7ec\ub098 \uc774\ub4e4\uc758 \uccad\uad50\ub3c4\uc801\uc778 \uc815\uc2e0\uc740 18\uc138\uae30 \uc0ac\uc6b0\ub514\uc544\ub77c\ube44\uc544 \uac74\uad6d\uc758 \uae30\ubc18\uc774 \ub41c \uc640\ud558\ube0c(Wahhab) \uc6b4\ub3d9\uc758 \ubc14\ud0d5\uc774 \ub418\uae30\ub3c4 \ud588\ub2e4.", "sentence_answer": "\uce74\uc640\ub9ac\uc988\ud30c\ub294 \uc804\ud22c\uc801\uc778 \ud589\ub3d9\uc8fc\uc758\uc790\uc5ec\uc11c, \uc9c0\ud558\ub4dc(jihad \uc8fc\ub85c \u2018\uc131\uc804\u2019\uc73c\ub85c \ubc88\uc5ed\ud55c\ub2e4)\ub97c \uc5ec\uc12f \ubc88\uc9f8 \u2018 \uc2e0\uc559\uc758 \uae30\ub465 \u2019\uc73c\ub85c \uc0bc\uc558\ub2e4.", "paragraph_id": "6545747-4-0", "paragraph_question": "question: \uce74\uc640\ub9ac\uc988\ud30c\ub294 \uc9c0\ud558\ub4dc\ub97c 6\ubc88 \uc9f8 \ubb34\uc5c7\uc73c\ub85c \uc0bc\uc558\uc2b5\ub2c8\uae4c?, context: \uce74\uc640\ub9ac\uc988\ud30c\ub294 \uc804\ud22c\uc801\uc778 \ud589\ub3d9\uc8fc\uc758\uc790\uc5ec\uc11c, \uc9c0\ud558\ub4dc(jihad \uc8fc\ub85c \u2018\uc131\uc804\u2019\uc73c\ub85c \ubc88\uc5ed\ud55c\ub2e4)\ub97c \uc5ec\uc12f \ubc88\uc9f8 \u2018\uc2e0\uc559\uc758 \uae30\ub465\u2019\uc73c\ub85c \uc0bc\uc558\ub2e4. 661\ub144 \uc774\ub4e4\uc740 \uc54c\ub9ac\ub97c \uc0b4\ud574\ud588\ub2e4. \uce74\uc640\ub9ac\uc988\ud30c\ub294 \uba54\uce74\uc758 \ubd80\uc871 \uafb8\ub77c\uc774\uc26c\uc758 \uc790\uc190\ub9cc\uc774 \uce7c\ub9ac\ud30c\uac00 \ub420 \uc218 \uc788\ub2e4\ub294 \uc218\ub2c8\ud30c\uc758 \uc804\ud1b5\uc801 \uacac\ud574\ub97c \ube44\ub09c\ud558\uace0, \ub3c5\uc2e4\ud55c \ubb34\uc2ac\ub9bc\uc774\uba74 \ub204\uad6c\ub098 \uce7c\ub9ac\ud30c\uac00 \ub420 \uc218 \uc788\uc5b4\uc57c \ud55c\ub2e4\uace0 \uc8fc\uc7a5\ud588\ub2e4. \ub9cc\ubbfc \ud3c9\ub4f1\uc8fc\uc758\uc801 \uc785\uc7a5\uacfc \uc544\ub78d \uadc0\uc871\uce35\uc5d0 \ub300\ud55c \uc801\uac1c\uc2ec \ub54c\ubb38\uc5d0 \ubca0\ub4dc\uc708\uacfc \ube44\uc544\ub78d\uacc4 \ubb34\uc2ac\ub9bc \ucd94\uc885\uc790\ub97c \uc5bb\uc744 \uc218 \uc788\uc5c8\uc9c0\ub9cc \ub0b4\ubd84\uc73c\ub85c \ub4b7\ub0a0 \uc800\uc808\ub85c \uc57d\ud654\ub410\ub2e4. \ud604\uc7ac\ub294 \uc54c\uc81c\ub9ac\uc640 \ud280\ub2c8\uc9c0\uc758 \ubca0\ub974\ubca0\ub974 \uc9c0\uc5ed, \ub3d9\uc544\ud504\ub9ac\uce74 \ud0c4\uc790\ub2c8\uc544\uc640 \uc544\ub77c\ube44\uc544 \ubc18\ub3c4\uc758 \uc624\ub9cc\uc5d0 \uc870\uae08 \ub0a8\uc544\uc788\uc744 \ubfd0\uc774\ub2e4. \uadf8\ub7ec\ub098 \uc774\ub4e4\uc758 \uccad\uad50\ub3c4\uc801\uc778 \uc815\uc2e0\uc740 18\uc138\uae30 \uc0ac\uc6b0\ub514\uc544\ub77c\ube44\uc544 \uac74\uad6d\uc758 \uae30\ubc18\uc774 \ub41c \uc640\ud558\ube0c(Wahhab) \uc6b4\ub3d9\uc758 \ubc14\ud0d5\uc774 \ub418\uae30\ub3c4 \ud588\ub2e4."} {"question": "\uc5ec\uc131\ub9cc \uc785\ud559\uc790\uaca9\uc694\uac74\uc73c\ub85c \ud55c \uacf3\uc740?", "paragraph": "\uccad\uad6c\uc778\uc740 \ub0a8\uc131 \ub85c\uc2a4\ucfe8 \uc9c0\uc6d0\uc790\ub4e4\ub85c \ud53c\uccad\uad6c\uc778 \uad50\uc721\ubd80\uc7a5\uad00\uc774 \ud53c\uccad\uad6c\uc778 \uc774\ud68c\ud559\ub2f9\uc5d0\uac8c \ubc95\ud559\uc804\ubb38\ub300\ud559\uc6d0(\ub85c\uc2a4\ucfe8) \uc124\uce58\uc778\uac00\ub97c \ud558\uba74\uc11c \uc81c\ucd9c\ud55c \uc785\ud559\uc804\ud615\uacc4\ud68d \uc911 \uc5ec\uc131\ub9cc\uc744 \uc785\ud559\uc790\uaca9\uc73c\ub85c \ud55c \ubd80\ubd84\uc744 \uc778\uc815\ud558\uace0 \uc774\ud654\ud559\ub2f9\uc740 \uc785\ud559\ubaa8\uc9d1\uc744 \ud558\uba74\uc11c \uc5ec\uc131\ub9cc\uc744 \uc785\ud559\uc790\uaca9\uc694\uac74\uc73c\ub85c \ud558\uc600\ub2e4. \uc774\uc5d0 \ub0a8\uc131\uc778 \uccad\uad6c\uc778\ub4e4\uc758 \ud3c9\ub4f1\uad8c, \uc9c1\uc5c5\uc758 \uc790\uc720 \ubc0f \uad50\uc721\uc744 \ubc1b\uc744 \uad8c\ub9ac\ub97c \uce68\ud574\ud55c\ub2e4\uace0 \uc8fc\uc7a5\ud558\uba74\uc11c \ud5cc\ubc95\uc18c\uc6d0\uc2ec\ud310\uc744 \uccad\uad6c\ud558\uc600\ub2e4. \uccad\uad6c\uc778\uce21\uc740 \"\ubc95\ud559\uc804\ubb38\ub300\ud559\uc6d0 \uc815\uc6d0\uc774 \ucd1d 2,000\uba85\uc778\ub370 \uc774\ub300\uc5d0 \ud560\ub2f9\ub41c \uc815\uc6d0 100\uba85\uc744 \ube7c\uba74 \ub0a8\uc131\uc740 \uc0ac\uc2e4 1,900\uba85\uc758 \uc815\uc6d0\uc744 \ub450\uace0 \uacbd\uc7c1\ud558\ub294 \uac83\uc774\ubbc0\ub85c \ud3c9\ub4f1\uad8c\uacfc \uc9c1\uc5c5\uc758 \uc790\uc720 \ub4f1\uc744 \uce68\ud574\ubc1b\uc558\ub2e4\"\uace0 \uc8fc\uc7a5\ud558\uc600\uace0 \ubc18\uba74 \uc774\ud654\uc5ec\ub300\uce21\uc740 \"\uccad\uad6c\uc778\ub4e4\uc5d0\uac8c\ub294 \uc774\ub300 \ubfd0\ub9cc \uc544\ub2c8\ub77c \ub2e4\ub978 \ubc95\ud559\uc804\ubb38\ub300\ud559\uc6d0\uc5d0 \uc9c4\ud559\ud560 \uae30\ud68c\uac00 \ucda9\ubd84\ud788 \ubcf4\uc7a5\ub3fc \uc788\uc5b4 \uc774\ub300\uc758 \ubaa8\uc9d1\uc694\uac15\uc774 \uccad\uad6c\uc778\ub4e4\uc758 \uae30\ubcf8\uad8c\uc744 \uce68\ud574\ud558\uc9c0 \uc54a\ub294\ub2e4\"\uace0 \uc8fc\uc7a5\ud558\uc600\ub2e4.", "answer": "\uc774\ud654\ud559\ub2f9", "sentence": "\uccad\uad6c\uc778\uc740 \ub0a8\uc131 \ub85c\uc2a4\ucfe8 \uc9c0\uc6d0\uc790\ub4e4\ub85c \ud53c\uccad\uad6c\uc778 \uad50\uc721\ubd80\uc7a5\uad00\uc774 \ud53c\uccad\uad6c\uc778 \uc774\ud68c\ud559\ub2f9\uc5d0\uac8c \ubc95\ud559\uc804\ubb38\ub300\ud559\uc6d0(\ub85c\uc2a4\ucfe8) \uc124\uce58\uc778\uac00\ub97c \ud558\uba74\uc11c \uc81c\ucd9c\ud55c \uc785\ud559\uc804\ud615\uacc4\ud68d \uc911 \uc5ec\uc131\ub9cc\uc744 \uc785\ud559\uc790\uaca9\uc73c\ub85c \ud55c \ubd80\ubd84\uc744 \uc778\uc815\ud558\uace0 \uc774\ud654\ud559\ub2f9 \uc740 \uc785\ud559\ubaa8\uc9d1\uc744 \ud558\uba74\uc11c \uc5ec\uc131\ub9cc\uc744 \uc785\ud559\uc790\uaca9\uc694\uac74\uc73c\ub85c \ud558\uc600\ub2e4.", "paragraph_sentence": " \uccad\uad6c\uc778\uc740 \ub0a8\uc131 \ub85c\uc2a4\ucfe8 \uc9c0\uc6d0\uc790\ub4e4\ub85c \ud53c\uccad\uad6c\uc778 \uad50\uc721\ubd80\uc7a5\uad00\uc774 \ud53c\uccad\uad6c\uc778 \uc774\ud68c\ud559\ub2f9\uc5d0\uac8c \ubc95\ud559\uc804\ubb38\ub300\ud559\uc6d0(\ub85c\uc2a4\ucfe8) \uc124\uce58\uc778\uac00\ub97c \ud558\uba74\uc11c \uc81c\ucd9c\ud55c \uc785\ud559\uc804\ud615\uacc4\ud68d \uc911 \uc5ec\uc131\ub9cc\uc744 \uc785\ud559\uc790\uaca9\uc73c\ub85c \ud55c \ubd80\ubd84\uc744 \uc778\uc815\ud558\uace0 \uc774\ud654\ud559\ub2f9 \uc740 \uc785\ud559\ubaa8\uc9d1\uc744 \ud558\uba74\uc11c \uc5ec\uc131\ub9cc\uc744 \uc785\ud559\uc790\uaca9\uc694\uac74\uc73c\ub85c \ud558\uc600\ub2e4. \uc774\uc5d0 \ub0a8\uc131\uc778 \uccad\uad6c\uc778\ub4e4\uc758 \ud3c9\ub4f1\uad8c, \uc9c1\uc5c5\uc758 \uc790\uc720 \ubc0f \uad50\uc721\uc744 \ubc1b\uc744 \uad8c\ub9ac\ub97c \uce68\ud574\ud55c\ub2e4\uace0 \uc8fc\uc7a5\ud558\uba74\uc11c \ud5cc\ubc95\uc18c\uc6d0\uc2ec\ud310\uc744 \uccad\uad6c\ud558\uc600\ub2e4. \uccad\uad6c\uc778\uce21\uc740 \"\ubc95\ud559\uc804\ubb38\ub300\ud559\uc6d0 \uc815\uc6d0\uc774 \ucd1d 2,000\uba85\uc778\ub370 \uc774\ub300\uc5d0 \ud560\ub2f9\ub41c \uc815\uc6d0 100\uba85\uc744 \ube7c\uba74 \ub0a8\uc131\uc740 \uc0ac\uc2e4 1,900\uba85\uc758 \uc815\uc6d0\uc744 \ub450\uace0 \uacbd\uc7c1\ud558\ub294 \uac83\uc774\ubbc0\ub85c \ud3c9\ub4f1\uad8c\uacfc \uc9c1\uc5c5\uc758 \uc790\uc720 \ub4f1\uc744 \uce68\ud574\ubc1b\uc558\ub2e4\"\uace0 \uc8fc\uc7a5\ud558\uc600\uace0 \ubc18\uba74 \uc774\ud654\uc5ec\ub300\uce21\uc740 \"\uccad\uad6c\uc778\ub4e4\uc5d0\uac8c\ub294 \uc774\ub300 \ubfd0\ub9cc \uc544\ub2c8\ub77c \ub2e4\ub978 \ubc95\ud559\uc804\ubb38\ub300\ud559\uc6d0\uc5d0 \uc9c4\ud559\ud560 \uae30\ud68c\uac00 \ucda9\ubd84\ud788 \ubcf4\uc7a5\ub3fc \uc788\uc5b4 \uc774\ub300\uc758 \ubaa8\uc9d1\uc694\uac15\uc774 \uccad\uad6c\uc778\ub4e4\uc758 \uae30\ubcf8\uad8c\uc744 \uce68\ud574\ud558\uc9c0 \uc54a\ub294\ub2e4\"\uace0 \uc8fc\uc7a5\ud558\uc600\ub2e4.", "paragraph_answer": "\uccad\uad6c\uc778\uc740 \ub0a8\uc131 \ub85c\uc2a4\ucfe8 \uc9c0\uc6d0\uc790\ub4e4\ub85c \ud53c\uccad\uad6c\uc778 \uad50\uc721\ubd80\uc7a5\uad00\uc774 \ud53c\uccad\uad6c\uc778 \uc774\ud68c\ud559\ub2f9\uc5d0\uac8c \ubc95\ud559\uc804\ubb38\ub300\ud559\uc6d0(\ub85c\uc2a4\ucfe8) \uc124\uce58\uc778\uac00\ub97c \ud558\uba74\uc11c \uc81c\ucd9c\ud55c \uc785\ud559\uc804\ud615\uacc4\ud68d \uc911 \uc5ec\uc131\ub9cc\uc744 \uc785\ud559\uc790\uaca9\uc73c\ub85c \ud55c \ubd80\ubd84\uc744 \uc778\uc815\ud558\uace0 \uc774\ud654\ud559\ub2f9 \uc740 \uc785\ud559\ubaa8\uc9d1\uc744 \ud558\uba74\uc11c \uc5ec\uc131\ub9cc\uc744 \uc785\ud559\uc790\uaca9\uc694\uac74\uc73c\ub85c \ud558\uc600\ub2e4. \uc774\uc5d0 \ub0a8\uc131\uc778 \uccad\uad6c\uc778\ub4e4\uc758 \ud3c9\ub4f1\uad8c, \uc9c1\uc5c5\uc758 \uc790\uc720 \ubc0f \uad50\uc721\uc744 \ubc1b\uc744 \uad8c\ub9ac\ub97c \uce68\ud574\ud55c\ub2e4\uace0 \uc8fc\uc7a5\ud558\uba74\uc11c \ud5cc\ubc95\uc18c\uc6d0\uc2ec\ud310\uc744 \uccad\uad6c\ud558\uc600\ub2e4. \uccad\uad6c\uc778\uce21\uc740 \"\ubc95\ud559\uc804\ubb38\ub300\ud559\uc6d0 \uc815\uc6d0\uc774 \ucd1d 2,000\uba85\uc778\ub370 \uc774\ub300\uc5d0 \ud560\ub2f9\ub41c \uc815\uc6d0 100\uba85\uc744 \ube7c\uba74 \ub0a8\uc131\uc740 \uc0ac\uc2e4 1,900\uba85\uc758 \uc815\uc6d0\uc744 \ub450\uace0 \uacbd\uc7c1\ud558\ub294 \uac83\uc774\ubbc0\ub85c \ud3c9\ub4f1\uad8c\uacfc \uc9c1\uc5c5\uc758 \uc790\uc720 \ub4f1\uc744 \uce68\ud574\ubc1b\uc558\ub2e4\"\uace0 \uc8fc\uc7a5\ud558\uc600\uace0 \ubc18\uba74 \uc774\ud654\uc5ec\ub300\uce21\uc740 \"\uccad\uad6c\uc778\ub4e4\uc5d0\uac8c\ub294 \uc774\ub300 \ubfd0\ub9cc \uc544\ub2c8\ub77c \ub2e4\ub978 \ubc95\ud559\uc804\ubb38\ub300\ud559\uc6d0\uc5d0 \uc9c4\ud559\ud560 \uae30\ud68c\uac00 \ucda9\ubd84\ud788 \ubcf4\uc7a5\ub3fc \uc788\uc5b4 \uc774\ub300\uc758 \ubaa8\uc9d1\uc694\uac15\uc774 \uccad\uad6c\uc778\ub4e4\uc758 \uae30\ubcf8\uad8c\uc744 \uce68\ud574\ud558\uc9c0 \uc54a\ub294\ub2e4\"\uace0 \uc8fc\uc7a5\ud558\uc600\ub2e4.", "sentence_answer": "\uccad\uad6c\uc778\uc740 \ub0a8\uc131 \ub85c\uc2a4\ucfe8 \uc9c0\uc6d0\uc790\ub4e4\ub85c \ud53c\uccad\uad6c\uc778 \uad50\uc721\ubd80\uc7a5\uad00\uc774 \ud53c\uccad\uad6c\uc778 \uc774\ud68c\ud559\ub2f9\uc5d0\uac8c \ubc95\ud559\uc804\ubb38\ub300\ud559\uc6d0(\ub85c\uc2a4\ucfe8) \uc124\uce58\uc778\uac00\ub97c \ud558\uba74\uc11c \uc81c\ucd9c\ud55c \uc785\ud559\uc804\ud615\uacc4\ud68d \uc911 \uc5ec\uc131\ub9cc\uc744 \uc785\ud559\uc790\uaca9\uc73c\ub85c \ud55c \ubd80\ubd84\uc744 \uc778\uc815\ud558\uace0 \uc774\ud654\ud559\ub2f9 \uc740 \uc785\ud559\ubaa8\uc9d1\uc744 \ud558\uba74\uc11c \uc5ec\uc131\ub9cc\uc744 \uc785\ud559\uc790\uaca9\uc694\uac74\uc73c\ub85c \ud558\uc600\ub2e4.", "paragraph_id": "6602021-3-0", "paragraph_question": "question: \uc5ec\uc131\ub9cc \uc785\ud559\uc790\uaca9\uc694\uac74\uc73c\ub85c \ud55c \uacf3\uc740?, context: \uccad\uad6c\uc778\uc740 \ub0a8\uc131 \ub85c\uc2a4\ucfe8 \uc9c0\uc6d0\uc790\ub4e4\ub85c \ud53c\uccad\uad6c\uc778 \uad50\uc721\ubd80\uc7a5\uad00\uc774 \ud53c\uccad\uad6c\uc778 \uc774\ud68c\ud559\ub2f9\uc5d0\uac8c \ubc95\ud559\uc804\ubb38\ub300\ud559\uc6d0(\ub85c\uc2a4\ucfe8) \uc124\uce58\uc778\uac00\ub97c \ud558\uba74\uc11c \uc81c\ucd9c\ud55c \uc785\ud559\uc804\ud615\uacc4\ud68d \uc911 \uc5ec\uc131\ub9cc\uc744 \uc785\ud559\uc790\uaca9\uc73c\ub85c \ud55c \ubd80\ubd84\uc744 \uc778\uc815\ud558\uace0 \uc774\ud654\ud559\ub2f9\uc740 \uc785\ud559\ubaa8\uc9d1\uc744 \ud558\uba74\uc11c \uc5ec\uc131\ub9cc\uc744 \uc785\ud559\uc790\uaca9\uc694\uac74\uc73c\ub85c \ud558\uc600\ub2e4. \uc774\uc5d0 \ub0a8\uc131\uc778 \uccad\uad6c\uc778\ub4e4\uc758 \ud3c9\ub4f1\uad8c, \uc9c1\uc5c5\uc758 \uc790\uc720 \ubc0f \uad50\uc721\uc744 \ubc1b\uc744 \uad8c\ub9ac\ub97c \uce68\ud574\ud55c\ub2e4\uace0 \uc8fc\uc7a5\ud558\uba74\uc11c \ud5cc\ubc95\uc18c\uc6d0\uc2ec\ud310\uc744 \uccad\uad6c\ud558\uc600\ub2e4. \uccad\uad6c\uc778\uce21\uc740 \"\ubc95\ud559\uc804\ubb38\ub300\ud559\uc6d0 \uc815\uc6d0\uc774 \ucd1d 2,000\uba85\uc778\ub370 \uc774\ub300\uc5d0 \ud560\ub2f9\ub41c \uc815\uc6d0 100\uba85\uc744 \ube7c\uba74 \ub0a8\uc131\uc740 \uc0ac\uc2e4 1,900\uba85\uc758 \uc815\uc6d0\uc744 \ub450\uace0 \uacbd\uc7c1\ud558\ub294 \uac83\uc774\ubbc0\ub85c \ud3c9\ub4f1\uad8c\uacfc \uc9c1\uc5c5\uc758 \uc790\uc720 \ub4f1\uc744 \uce68\ud574\ubc1b\uc558\ub2e4\"\uace0 \uc8fc\uc7a5\ud558\uc600\uace0 \ubc18\uba74 \uc774\ud654\uc5ec\ub300\uce21\uc740 \"\uccad\uad6c\uc778\ub4e4\uc5d0\uac8c\ub294 \uc774\ub300 \ubfd0\ub9cc \uc544\ub2c8\ub77c \ub2e4\ub978 \ubc95\ud559\uc804\ubb38\ub300\ud559\uc6d0\uc5d0 \uc9c4\ud559\ud560 \uae30\ud68c\uac00 \ucda9\ubd84\ud788 \ubcf4\uc7a5\ub3fc \uc788\uc5b4 \uc774\ub300\uc758 \ubaa8\uc9d1\uc694\uac15\uc774 \uccad\uad6c\uc778\ub4e4\uc758 \uae30\ubcf8\uad8c\uc744 \uce68\ud574\ud558\uc9c0 \uc54a\ub294\ub2e4\"\uace0 \uc8fc\uc7a5\ud558\uc600\ub2e4."} {"question": "1981\ub144 \uc774\uc2dc\ubbf8\ub124 \uac00\uc988\ud788\ucf54\uac00 \uc18c\uc18d\ub41c \ud300\uc758 \uac10\ub3c5\uc73c\ub85c \ucde8\uc784\ud55c \uc0ac\ub78c\uc740?", "paragraph": "\ud504\ub85c 2\ub144\uc9f8\uc778 1980\ub144 2\uc6d4\uc5d0 \ud1f4\uc6d0\ud55c \ub4a4 \uc6d0\uc0c1\ubcf5\uadc0\ub418\uc9c0 \uc54a\ub294 \ubc18\ub2ec\ud310\uc744 \uc9c0\ud0f1\ud558\uae30 \uc704\ud574 \uc8fc\ubcc0 \uadfc\uc721\uc744 \uacc4\uc18d \ub2e8\ub828\ud588\ub2e4. \uc6e8\uc2a4\ud134 \ub9ac\uadf8\uc5d0\uc11c\ub294 \ubb34\ub98e\uc5d0 \ubd80\ub2f4\uc744 \uc8fc\uc9c0 \uc54a\ub3c4\ub85d \ub300\ud0c0\ub97c \uc911\uc2ec\uc73c\ub85c \uae30\uc6a9\ub418\uba74\uc11c 82\ud0c0\uc218 21\uc548\ud0c0\ub85c \ud0c0\uc728\uc740 2\ud560 5\ud47c 6\ub9ac\ub85c \ub0ae\uc544\uc84c\uc9c0\ub9cc \ud648\ub7f0\uc740 4\uac1c\ub85c \uc99d\uac00\ud574 \uc7a5\uac70\ub9ac\ud3ec\uc758 \ud3b8\ub9b0\uc744 \uc5ff\ubcf4\uac8c \ud588\ub2e4. \uc774\ub4ec\ud574 1981\ub144\uc5d0\ub294 \ud788\ub85c\uc2dc\ub9c8\uc640\uc758 \uc2dc\ubc94 \uacbd\uae30\uc5d0\uc11c 7\uc810 \ucc28\ub85c \ub4a4\uc9c4 9\ud68c\ub9d0\uc5d0 \ub300\ud0c0\ub85c\uc11c \ucd9c\uc804\ud574 \uc774 \ud0c0\uc11d\uc5d0\uc11c \ubcfc\ub137\uc744 \uc120\ud0dd\ud55c \uac83\uc744 \uae30\uc810\uc73c\ub85c \ud0c0\uc21c\uc774 \ud55c \ubc14\ud034 \ub3c4\ub294 \ub9f9\uacf5\uaca9\uc774 \uc2dc\uc791\ub410\uace0 9\ud68c\uc5d0 \ub2e4\uc2dc \ub3cc\uc544\uc628 \ud0c0\uc11d\uc5d0\uc11c \ub77c\uc774\ub108\uc131 \ub05d\ub0b4\uae30 3\uc810 \ud648\ub7f0\uc744 \ub54c\ub824\ub0c8\ub2e4. \uc774\uac83\uc744 \uacc4\uae30\ub85c \uadf8 \ud574\ubd80\ud130 \uac10\ub3c5\uc73c\ub85c \ucde8\uc784\ud55c \uc6b0\uc5d0\ub2e4 \ub3c4\uc2dc\ud558\ub8e8\ub85c\ubd80\ud130 \ud380\uce58\ub825\uc744 \ud3c9\uac00\ubc1b\uc544 \ub098\uce74\uc790\uc640 \uc2e0\uc9c0\ub098 \uc0ac\uc0ac\ubaa8\ud1a0 \uc2e0\uc9c0, \uac00\ud0c0\uc624\uce74 \uc2e0\ub178\uc2a4\ucf00, \uac00\uc640\ubb34\ub77c \uac90\uc774\uce58\ub85c\uc5d0 \uc774\uc5b4 \uc774\ub840\uc801\uc73c\ub85c 5\ubc88\uc9f8 \ud3ec\uc218\ub85c\uc11c \uac1c\ub9c9\uc804 1\uad70 \uc5d4\ud2b8\ub9ac\uc5d0 \uc9c4\uc785\ud588\ub2e4. \uac19\uc740 \ud574 36\uacbd\uae30\uc5d0 \ucd9c\uc804\ud574 \ud0c0\uc728 2\ud560 1\ud47c 4\ub9ac\ub97c \uae30\ub85d\ud588\uace0 \ud3ec\uc218\ub85c\uc11c\ub294 15\uacbd\uae30\uc5d0 \ucd9c\uc804\ud588\ub2e4.", "answer": "\uc6b0\uc5d0\ub2e4 \ub3c4\uc2dc\ud558\ub8e8", "sentence": "\uc774\uac83\uc744 \uacc4\uae30\ub85c \uadf8 \ud574\ubd80\ud130 \uac10\ub3c5\uc73c\ub85c \ucde8\uc784\ud55c \uc6b0\uc5d0\ub2e4 \ub3c4\uc2dc\ud558\ub8e8 \ub85c\ubd80\ud130 \ud380\uce58\ub825\uc744 \ud3c9\uac00\ubc1b\uc544 \ub098\uce74\uc790\uc640 \uc2e0\uc9c0\ub098 \uc0ac\uc0ac\ubaa8\ud1a0 \uc2e0\uc9c0, \uac00\ud0c0\uc624\uce74 \uc2e0\ub178\uc2a4\ucf00, \uac00\uc640\ubb34\ub77c \uac90\uc774\uce58\ub85c\uc5d0 \uc774\uc5b4 \uc774\ub840\uc801\uc73c\ub85c 5\ubc88\uc9f8 \ud3ec\uc218\ub85c\uc11c \uac1c\ub9c9\uc804 1\uad70 \uc5d4\ud2b8\ub9ac\uc5d0 \uc9c4\uc785\ud588\ub2e4.", "paragraph_sentence": "\ud504\ub85c 2\ub144\uc9f8\uc778 1980\ub144 2\uc6d4\uc5d0 \ud1f4\uc6d0\ud55c \ub4a4 \uc6d0\uc0c1\ubcf5\uadc0\ub418\uc9c0 \uc54a\ub294 \ubc18\ub2ec\ud310\uc744 \uc9c0\ud0f1\ud558\uae30 \uc704\ud574 \uc8fc\ubcc0 \uadfc\uc721\uc744 \uacc4\uc18d \ub2e8\ub828\ud588\ub2e4. \uc6e8\uc2a4\ud134 \ub9ac\uadf8\uc5d0\uc11c\ub294 \ubb34\ub98e\uc5d0 \ubd80\ub2f4\uc744 \uc8fc\uc9c0 \uc54a\ub3c4\ub85d \ub300\ud0c0\ub97c \uc911\uc2ec\uc73c\ub85c \uae30\uc6a9\ub418\uba74\uc11c 82\ud0c0\uc218 21\uc548\ud0c0\ub85c \ud0c0\uc728\uc740 2\ud560 5\ud47c 6\ub9ac\ub85c \ub0ae\uc544\uc84c\uc9c0\ub9cc \ud648\ub7f0\uc740 4\uac1c\ub85c \uc99d\uac00\ud574 \uc7a5\uac70\ub9ac\ud3ec\uc758 \ud3b8\ub9b0\uc744 \uc5ff\ubcf4\uac8c \ud588\ub2e4. \uc774\ub4ec\ud574 1981\ub144\uc5d0\ub294 \ud788\ub85c\uc2dc\ub9c8\uc640\uc758 \uc2dc\ubc94 \uacbd\uae30\uc5d0\uc11c 7\uc810 \ucc28\ub85c \ub4a4\uc9c4 9\ud68c\ub9d0\uc5d0 \ub300\ud0c0\ub85c\uc11c \ucd9c\uc804\ud574 \uc774 \ud0c0\uc11d\uc5d0\uc11c \ubcfc\ub137\uc744 \uc120\ud0dd\ud55c \uac83\uc744 \uae30\uc810\uc73c\ub85c \ud0c0\uc21c\uc774 \ud55c \ubc14\ud034 \ub3c4\ub294 \ub9f9\uacf5\uaca9\uc774 \uc2dc\uc791\ub410\uace0 9\ud68c\uc5d0 \ub2e4\uc2dc \ub3cc\uc544\uc628 \ud0c0\uc11d\uc5d0\uc11c \ub77c\uc774\ub108\uc131 \ub05d\ub0b4\uae30 3\uc810 \ud648\ub7f0\uc744 \ub54c\ub824\ub0c8\ub2e4. \uc774\uac83\uc744 \uacc4\uae30\ub85c \uadf8 \ud574\ubd80\ud130 \uac10\ub3c5\uc73c\ub85c \ucde8\uc784\ud55c \uc6b0\uc5d0\ub2e4 \ub3c4\uc2dc\ud558\ub8e8 \ub85c\ubd80\ud130 \ud380\uce58\ub825\uc744 \ud3c9\uac00\ubc1b\uc544 \ub098\uce74\uc790\uc640 \uc2e0\uc9c0\ub098 \uc0ac\uc0ac\ubaa8\ud1a0 \uc2e0\uc9c0, \uac00\ud0c0\uc624\uce74 \uc2e0\ub178\uc2a4\ucf00, \uac00\uc640\ubb34\ub77c \uac90\uc774\uce58\ub85c\uc5d0 \uc774\uc5b4 \uc774\ub840\uc801\uc73c\ub85c 5\ubc88\uc9f8 \ud3ec\uc218\ub85c\uc11c \uac1c\ub9c9\uc804 1\uad70 \uc5d4\ud2b8\ub9ac\uc5d0 \uc9c4\uc785\ud588\ub2e4. \uac19\uc740 \ud574 36\uacbd\uae30\uc5d0 \ucd9c\uc804\ud574 \ud0c0\uc728 2\ud560 1\ud47c 4\ub9ac\ub97c \uae30\ub85d\ud588\uace0 \ud3ec\uc218\ub85c\uc11c\ub294 15\uacbd\uae30\uc5d0 \ucd9c\uc804\ud588\ub2e4.", "paragraph_answer": "\ud504\ub85c 2\ub144\uc9f8\uc778 1980\ub144 2\uc6d4\uc5d0 \ud1f4\uc6d0\ud55c \ub4a4 \uc6d0\uc0c1\ubcf5\uadc0\ub418\uc9c0 \uc54a\ub294 \ubc18\ub2ec\ud310\uc744 \uc9c0\ud0f1\ud558\uae30 \uc704\ud574 \uc8fc\ubcc0 \uadfc\uc721\uc744 \uacc4\uc18d \ub2e8\ub828\ud588\ub2e4. \uc6e8\uc2a4\ud134 \ub9ac\uadf8\uc5d0\uc11c\ub294 \ubb34\ub98e\uc5d0 \ubd80\ub2f4\uc744 \uc8fc\uc9c0 \uc54a\ub3c4\ub85d \ub300\ud0c0\ub97c \uc911\uc2ec\uc73c\ub85c \uae30\uc6a9\ub418\uba74\uc11c 82\ud0c0\uc218 21\uc548\ud0c0\ub85c \ud0c0\uc728\uc740 2\ud560 5\ud47c 6\ub9ac\ub85c \ub0ae\uc544\uc84c\uc9c0\ub9cc \ud648\ub7f0\uc740 4\uac1c\ub85c \uc99d\uac00\ud574 \uc7a5\uac70\ub9ac\ud3ec\uc758 \ud3b8\ub9b0\uc744 \uc5ff\ubcf4\uac8c \ud588\ub2e4. \uc774\ub4ec\ud574 1981\ub144\uc5d0\ub294 \ud788\ub85c\uc2dc\ub9c8\uc640\uc758 \uc2dc\ubc94 \uacbd\uae30\uc5d0\uc11c 7\uc810 \ucc28\ub85c \ub4a4\uc9c4 9\ud68c\ub9d0\uc5d0 \ub300\ud0c0\ub85c\uc11c \ucd9c\uc804\ud574 \uc774 \ud0c0\uc11d\uc5d0\uc11c \ubcfc\ub137\uc744 \uc120\ud0dd\ud55c \uac83\uc744 \uae30\uc810\uc73c\ub85c \ud0c0\uc21c\uc774 \ud55c \ubc14\ud034 \ub3c4\ub294 \ub9f9\uacf5\uaca9\uc774 \uc2dc\uc791\ub410\uace0 9\ud68c\uc5d0 \ub2e4\uc2dc \ub3cc\uc544\uc628 \ud0c0\uc11d\uc5d0\uc11c \ub77c\uc774\ub108\uc131 \ub05d\ub0b4\uae30 3\uc810 \ud648\ub7f0\uc744 \ub54c\ub824\ub0c8\ub2e4. \uc774\uac83\uc744 \uacc4\uae30\ub85c \uadf8 \ud574\ubd80\ud130 \uac10\ub3c5\uc73c\ub85c \ucde8\uc784\ud55c \uc6b0\uc5d0\ub2e4 \ub3c4\uc2dc\ud558\ub8e8 \ub85c\ubd80\ud130 \ud380\uce58\ub825\uc744 \ud3c9\uac00\ubc1b\uc544 \ub098\uce74\uc790\uc640 \uc2e0\uc9c0\ub098 \uc0ac\uc0ac\ubaa8\ud1a0 \uc2e0\uc9c0, \uac00\ud0c0\uc624\uce74 \uc2e0\ub178\uc2a4\ucf00, \uac00\uc640\ubb34\ub77c \uac90\uc774\uce58\ub85c\uc5d0 \uc774\uc5b4 \uc774\ub840\uc801\uc73c\ub85c 5\ubc88\uc9f8 \ud3ec\uc218\ub85c\uc11c \uac1c\ub9c9\uc804 1\uad70 \uc5d4\ud2b8\ub9ac\uc5d0 \uc9c4\uc785\ud588\ub2e4. \uac19\uc740 \ud574 36\uacbd\uae30\uc5d0 \ucd9c\uc804\ud574 \ud0c0\uc728 2\ud560 1\ud47c 4\ub9ac\ub97c \uae30\ub85d\ud588\uace0 \ud3ec\uc218\ub85c\uc11c\ub294 15\uacbd\uae30\uc5d0 \ucd9c\uc804\ud588\ub2e4.", "sentence_answer": "\uc774\uac83\uc744 \uacc4\uae30\ub85c \uadf8 \ud574\ubd80\ud130 \uac10\ub3c5\uc73c\ub85c \ucde8\uc784\ud55c \uc6b0\uc5d0\ub2e4 \ub3c4\uc2dc\ud558\ub8e8 \ub85c\ubd80\ud130 \ud380\uce58\ub825\uc744 \ud3c9\uac00\ubc1b\uc544 \ub098\uce74\uc790\uc640 \uc2e0\uc9c0\ub098 \uc0ac\uc0ac\ubaa8\ud1a0 \uc2e0\uc9c0, \uac00\ud0c0\uc624\uce74 \uc2e0\ub178\uc2a4\ucf00, \uac00\uc640\ubb34\ub77c \uac90\uc774\uce58\ub85c\uc5d0 \uc774\uc5b4 \uc774\ub840\uc801\uc73c\ub85c 5\ubc88\uc9f8 \ud3ec\uc218\ub85c\uc11c \uac1c\ub9c9\uc804 1\uad70 \uc5d4\ud2b8\ub9ac\uc5d0 \uc9c4\uc785\ud588\ub2e4.", "paragraph_id": "6545783-1-1", "paragraph_question": "question: 1981\ub144 \uc774\uc2dc\ubbf8\ub124 \uac00\uc988\ud788\ucf54\uac00 \uc18c\uc18d\ub41c \ud300\uc758 \uac10\ub3c5\uc73c\ub85c \ucde8\uc784\ud55c \uc0ac\ub78c\uc740?, context: \ud504\ub85c 2\ub144\uc9f8\uc778 1980\ub144 2\uc6d4\uc5d0 \ud1f4\uc6d0\ud55c \ub4a4 \uc6d0\uc0c1\ubcf5\uadc0\ub418\uc9c0 \uc54a\ub294 \ubc18\ub2ec\ud310\uc744 \uc9c0\ud0f1\ud558\uae30 \uc704\ud574 \uc8fc\ubcc0 \uadfc\uc721\uc744 \uacc4\uc18d \ub2e8\ub828\ud588\ub2e4. \uc6e8\uc2a4\ud134 \ub9ac\uadf8\uc5d0\uc11c\ub294 \ubb34\ub98e\uc5d0 \ubd80\ub2f4\uc744 \uc8fc\uc9c0 \uc54a\ub3c4\ub85d \ub300\ud0c0\ub97c \uc911\uc2ec\uc73c\ub85c \uae30\uc6a9\ub418\uba74\uc11c 82\ud0c0\uc218 21\uc548\ud0c0\ub85c \ud0c0\uc728\uc740 2\ud560 5\ud47c 6\ub9ac\ub85c \ub0ae\uc544\uc84c\uc9c0\ub9cc \ud648\ub7f0\uc740 4\uac1c\ub85c \uc99d\uac00\ud574 \uc7a5\uac70\ub9ac\ud3ec\uc758 \ud3b8\ub9b0\uc744 \uc5ff\ubcf4\uac8c \ud588\ub2e4. \uc774\ub4ec\ud574 1981\ub144\uc5d0\ub294 \ud788\ub85c\uc2dc\ub9c8\uc640\uc758 \uc2dc\ubc94 \uacbd\uae30\uc5d0\uc11c 7\uc810 \ucc28\ub85c \ub4a4\uc9c4 9\ud68c\ub9d0\uc5d0 \ub300\ud0c0\ub85c\uc11c \ucd9c\uc804\ud574 \uc774 \ud0c0\uc11d\uc5d0\uc11c \ubcfc\ub137\uc744 \uc120\ud0dd\ud55c \uac83\uc744 \uae30\uc810\uc73c\ub85c \ud0c0\uc21c\uc774 \ud55c \ubc14\ud034 \ub3c4\ub294 \ub9f9\uacf5\uaca9\uc774 \uc2dc\uc791\ub410\uace0 9\ud68c\uc5d0 \ub2e4\uc2dc \ub3cc\uc544\uc628 \ud0c0\uc11d\uc5d0\uc11c \ub77c\uc774\ub108\uc131 \ub05d\ub0b4\uae30 3\uc810 \ud648\ub7f0\uc744 \ub54c\ub824\ub0c8\ub2e4. \uc774\uac83\uc744 \uacc4\uae30\ub85c \uadf8 \ud574\ubd80\ud130 \uac10\ub3c5\uc73c\ub85c \ucde8\uc784\ud55c \uc6b0\uc5d0\ub2e4 \ub3c4\uc2dc\ud558\ub8e8\ub85c\ubd80\ud130 \ud380\uce58\ub825\uc744 \ud3c9\uac00\ubc1b\uc544 \ub098\uce74\uc790\uc640 \uc2e0\uc9c0\ub098 \uc0ac\uc0ac\ubaa8\ud1a0 \uc2e0\uc9c0, \uac00\ud0c0\uc624\uce74 \uc2e0\ub178\uc2a4\ucf00, \uac00\uc640\ubb34\ub77c \uac90\uc774\uce58\ub85c\uc5d0 \uc774\uc5b4 \uc774\ub840\uc801\uc73c\ub85c 5\ubc88\uc9f8 \ud3ec\uc218\ub85c\uc11c \uac1c\ub9c9\uc804 1\uad70 \uc5d4\ud2b8\ub9ac\uc5d0 \uc9c4\uc785\ud588\ub2e4. \uac19\uc740 \ud574 36\uacbd\uae30\uc5d0 \ucd9c\uc804\ud574 \ud0c0\uc728 2\ud560 1\ud47c 4\ub9ac\ub97c \uae30\ub85d\ud588\uace0 \ud3ec\uc218\ub85c\uc11c\ub294 15\uacbd\uae30\uc5d0 \ucd9c\uc804\ud588\ub2e4."} {"question": "\ub192\uc740 \uccb4\uc628\uc744 \uc720\uc9c0\ud558\ub294 \ud2b9\uc815\ud55c \uc885\uc758 \uc5b4\ub958\ub294?", "paragraph": "\ub610\ud55c \ud2b9\uc815\ud55c \uc885\uc758 \uc5b4\ub958\ub294 \ub192\uc740 \uccb4\uc628\uc744 \uc720\uc9c0\ud55c\ub2e4. \uc628\ud608 \uacbd\uace8\uc5b4\ub958(\uc601\uc5b4:teleost, bony fish\ub77c\uace0\ub3c4 \ubd88\ub9bc)\ub4e4\uc740 \ubaa8\ub450 \uace0\ub4f1\uc5b4\uc544\ubaa9\uc5d0 \uc18d\ud558\uace0, \uc0c8\uce58\ub4e4\uacfc \ucc38\uce58\ub4e4, \ud55c \uc885\ub958\uc758 \"\uc6d0\uc2dc\" \uace0\ub4f1\uc5b4\ub97c \ud3ec\ud568\ud55c\ub2e4. \uc9e7\uc740 \uc9c0\ub290\ub7ec\ubbf8 \uccad\uc0c1\uc544\ub9ac, \uae34 \uc9c0\ub290\ub7ec\ubbf8 \uccad\uc0c1\uc544\ub9ac, \ubc31\uc0c1\uc544\ub9ac, \uc545\uc0c1\uc5b4, \uc5f0\uc5b4\uc0c1\uc5b4\ub4f1\uc758 \uc545\uc0c1\uc5b4\uacfc\uc758 \ubaa8\ub4e0 \uc0c1\uc5b4\ub4e4\ub3c4 \uc628\ud608\uc774\ub2e4. \ub610\ud55c \ud658\ub3c4\uc0c1\uc5b4\uacfc(\uc601\uc5b4:Alopiidae)\uc758 \uc885\ub4e4\ub3c4 \uc628\ud608\uc784\uc744 \ub098\ud0c0\ub0b4\ub294 \uc99d\uac70\uac00 \ubc1c\uacac\ub41c\ub2e4. \ub0b4\ubd80\uc5f4(\uc601\uc5b4:endothermy)\uc758 \uc628\ub3c4\ub294 \ub208\uacfc \ub1cc\ub9cc\uc744 \ub530\ub4ef\ud558\uac8c \ub9cc\ub4dc\ub294 \uc0c8\uce58\ub4e4\ub85c\ubd80\ud130 \uccb4\uc628\uc744 \uc8fc\ubcc0\uc758 \uc218\uc628\ubcf4\ub2e4 20\ub3c4 \uc815\ub3c4 \ub192\uac8c \uc720\uc9c0\ud558\ub294 \ucc38\uce58(\ucc38\ub2e4\ub791\uc5b4)\uc640 \uc545\uc0c1\uc5b4\uae4c\uc9c0 \uac01\uae30 \ub2e4\ub974\ub2e4. \"\uac70\ub300\uc628\ud608\ud56d\ubaa9\uc744 \ucc38\uc870\ud558\ub77c.\" \uc628\ud608\uc740 \ub300\uc0ac\uc801\uc73c\ub85c\ub294 \ub192\uc740 \ube44\uc6a9\uc774 \ub4e4\uc5b4\uac10\uc5d0\ub3c4, \ud5a5\uc0c1\ub41c \uadfc\uc721\uc758 \ud798, \uc911\uc559\uc2e0\uacbd\uacc4\uc758 \ube60\ub978 \uc18d\ub3c4, \uadf8\ub9ac\uace0 \uace0\ud6a8\uc728\uc758 \uc18c\ud654 \ub4f1\uc758 \uc774\uc810\uc744 \uc81c\uacf5\ud55c\ub2e4\uace0 \uc5ec\uaca8\uc9c4\ub2e4.", "answer": "\uc628\ud608 \uacbd\uace8\uc5b4\ub958", "sentence": "\uc628\ud608 \uacbd\uace8\uc5b4\ub958 (\uc601\uc5b4:teleost, bony fish\ub77c\uace0\ub3c4 \ubd88\ub9bc)\ub4e4\uc740 \ubaa8\ub450 \uace0\ub4f1\uc5b4\uc544\ubaa9\uc5d0 \uc18d\ud558\uace0, \uc0c8\uce58\ub4e4\uacfc \ucc38\uce58\ub4e4, \ud55c \uc885\ub958\uc758 \"\uc6d0\uc2dc\" \uace0\ub4f1\uc5b4\ub97c \ud3ec\ud568\ud55c\ub2e4.", "paragraph_sentence": "\ub610\ud55c \ud2b9\uc815\ud55c \uc885\uc758 \uc5b4\ub958\ub294 \ub192\uc740 \uccb4\uc628\uc744 \uc720\uc9c0\ud55c\ub2e4. \uc628\ud608 \uacbd\uace8\uc5b4\ub958 (\uc601\uc5b4:teleost, bony fish\ub77c\uace0\ub3c4 \ubd88\ub9bc)\ub4e4\uc740 \ubaa8\ub450 \uace0\ub4f1\uc5b4\uc544\ubaa9\uc5d0 \uc18d\ud558\uace0, \uc0c8\uce58\ub4e4\uacfc \ucc38\uce58\ub4e4, \ud55c \uc885\ub958\uc758 \"\uc6d0\uc2dc\" \uace0\ub4f1\uc5b4\ub97c \ud3ec\ud568\ud55c\ub2e4. \uc9e7\uc740 \uc9c0\ub290\ub7ec\ubbf8 \uccad\uc0c1\uc544\ub9ac, \uae34 \uc9c0\ub290\ub7ec\ubbf8 \uccad\uc0c1\uc544\ub9ac, \ubc31\uc0c1\uc544\ub9ac, \uc545\uc0c1\uc5b4, \uc5f0\uc5b4\uc0c1\uc5b4\ub4f1\uc758 \uc545\uc0c1\uc5b4\uacfc\uc758 \ubaa8\ub4e0 \uc0c1\uc5b4\ub4e4\ub3c4 \uc628\ud608\uc774\ub2e4. \ub610\ud55c \ud658\ub3c4\uc0c1\uc5b4\uacfc(\uc601\uc5b4:Alopiidae)\uc758 \uc885\ub4e4\ub3c4 \uc628\ud608\uc784\uc744 \ub098\ud0c0\ub0b4\ub294 \uc99d\uac70\uac00 \ubc1c\uacac\ub41c\ub2e4. \ub0b4\ubd80\uc5f4(\uc601\uc5b4:endothermy)\uc758 \uc628\ub3c4\ub294 \ub208\uacfc \ub1cc\ub9cc\uc744 \ub530\ub4ef\ud558\uac8c \ub9cc\ub4dc\ub294 \uc0c8\uce58\ub4e4\ub85c\ubd80\ud130 \uccb4\uc628\uc744 \uc8fc\ubcc0\uc758 \uc218\uc628\ubcf4\ub2e4 20\ub3c4 \uc815\ub3c4 \ub192\uac8c \uc720\uc9c0\ud558\ub294 \ucc38\uce58(\ucc38\ub2e4\ub791\uc5b4)\uc640 \uc545\uc0c1\uc5b4\uae4c\uc9c0 \uac01\uae30 \ub2e4\ub974\ub2e4. \"\uac70\ub300\uc628\ud608\ud56d\ubaa9\uc744 \ucc38\uc870\ud558\ub77c. \" \uc628\ud608\uc740 \ub300\uc0ac\uc801\uc73c\ub85c\ub294 \ub192\uc740 \ube44\uc6a9\uc774 \ub4e4\uc5b4\uac10\uc5d0\ub3c4, \ud5a5\uc0c1\ub41c \uadfc\uc721\uc758 \ud798, \uc911\uc559\uc2e0\uacbd\uacc4\uc758 \ube60\ub978 \uc18d\ub3c4, \uadf8\ub9ac\uace0 \uace0\ud6a8\uc728\uc758 \uc18c\ud654 \ub4f1\uc758 \uc774\uc810\uc744 \uc81c\uacf5\ud55c\ub2e4\uace0 \uc5ec\uaca8\uc9c4\ub2e4.", "paragraph_answer": "\ub610\ud55c \ud2b9\uc815\ud55c \uc885\uc758 \uc5b4\ub958\ub294 \ub192\uc740 \uccb4\uc628\uc744 \uc720\uc9c0\ud55c\ub2e4. \uc628\ud608 \uacbd\uace8\uc5b4\ub958 (\uc601\uc5b4:teleost, bony fish\ub77c\uace0\ub3c4 \ubd88\ub9bc)\ub4e4\uc740 \ubaa8\ub450 \uace0\ub4f1\uc5b4\uc544\ubaa9\uc5d0 \uc18d\ud558\uace0, \uc0c8\uce58\ub4e4\uacfc \ucc38\uce58\ub4e4, \ud55c \uc885\ub958\uc758 \"\uc6d0\uc2dc\" \uace0\ub4f1\uc5b4\ub97c \ud3ec\ud568\ud55c\ub2e4. \uc9e7\uc740 \uc9c0\ub290\ub7ec\ubbf8 \uccad\uc0c1\uc544\ub9ac, \uae34 \uc9c0\ub290\ub7ec\ubbf8 \uccad\uc0c1\uc544\ub9ac, \ubc31\uc0c1\uc544\ub9ac, \uc545\uc0c1\uc5b4, \uc5f0\uc5b4\uc0c1\uc5b4\ub4f1\uc758 \uc545\uc0c1\uc5b4\uacfc\uc758 \ubaa8\ub4e0 \uc0c1\uc5b4\ub4e4\ub3c4 \uc628\ud608\uc774\ub2e4. \ub610\ud55c \ud658\ub3c4\uc0c1\uc5b4\uacfc(\uc601\uc5b4:Alopiidae)\uc758 \uc885\ub4e4\ub3c4 \uc628\ud608\uc784\uc744 \ub098\ud0c0\ub0b4\ub294 \uc99d\uac70\uac00 \ubc1c\uacac\ub41c\ub2e4. \ub0b4\ubd80\uc5f4(\uc601\uc5b4:endothermy)\uc758 \uc628\ub3c4\ub294 \ub208\uacfc \ub1cc\ub9cc\uc744 \ub530\ub4ef\ud558\uac8c \ub9cc\ub4dc\ub294 \uc0c8\uce58\ub4e4\ub85c\ubd80\ud130 \uccb4\uc628\uc744 \uc8fc\ubcc0\uc758 \uc218\uc628\ubcf4\ub2e4 20\ub3c4 \uc815\ub3c4 \ub192\uac8c \uc720\uc9c0\ud558\ub294 \ucc38\uce58(\ucc38\ub2e4\ub791\uc5b4)\uc640 \uc545\uc0c1\uc5b4\uae4c\uc9c0 \uac01\uae30 \ub2e4\ub974\ub2e4. \"\uac70\ub300\uc628\ud608\ud56d\ubaa9\uc744 \ucc38\uc870\ud558\ub77c.\" \uc628\ud608\uc740 \ub300\uc0ac\uc801\uc73c\ub85c\ub294 \ub192\uc740 \ube44\uc6a9\uc774 \ub4e4\uc5b4\uac10\uc5d0\ub3c4, \ud5a5\uc0c1\ub41c \uadfc\uc721\uc758 \ud798, \uc911\uc559\uc2e0\uacbd\uacc4\uc758 \ube60\ub978 \uc18d\ub3c4, \uadf8\ub9ac\uace0 \uace0\ud6a8\uc728\uc758 \uc18c\ud654 \ub4f1\uc758 \uc774\uc810\uc744 \uc81c\uacf5\ud55c\ub2e4\uace0 \uc5ec\uaca8\uc9c4\ub2e4.", "sentence_answer": " \uc628\ud608 \uacbd\uace8\uc5b4\ub958 (\uc601\uc5b4:teleost, bony fish\ub77c\uace0\ub3c4 \ubd88\ub9bc)\ub4e4\uc740 \ubaa8\ub450 \uace0\ub4f1\uc5b4\uc544\ubaa9\uc5d0 \uc18d\ud558\uace0, \uc0c8\uce58\ub4e4\uacfc \ucc38\uce58\ub4e4, \ud55c \uc885\ub958\uc758 \"\uc6d0\uc2dc\" \uace0\ub4f1\uc5b4\ub97c \ud3ec\ud568\ud55c\ub2e4.", "paragraph_id": "6505119-7-0", "paragraph_question": "question: \ub192\uc740 \uccb4\uc628\uc744 \uc720\uc9c0\ud558\ub294 \ud2b9\uc815\ud55c \uc885\uc758 \uc5b4\ub958\ub294?, context: \ub610\ud55c \ud2b9\uc815\ud55c \uc885\uc758 \uc5b4\ub958\ub294 \ub192\uc740 \uccb4\uc628\uc744 \uc720\uc9c0\ud55c\ub2e4. \uc628\ud608 \uacbd\uace8\uc5b4\ub958(\uc601\uc5b4:teleost, bony fish\ub77c\uace0\ub3c4 \ubd88\ub9bc)\ub4e4\uc740 \ubaa8\ub450 \uace0\ub4f1\uc5b4\uc544\ubaa9\uc5d0 \uc18d\ud558\uace0, \uc0c8\uce58\ub4e4\uacfc \ucc38\uce58\ub4e4, \ud55c \uc885\ub958\uc758 \"\uc6d0\uc2dc\" \uace0\ub4f1\uc5b4\ub97c \ud3ec\ud568\ud55c\ub2e4. \uc9e7\uc740 \uc9c0\ub290\ub7ec\ubbf8 \uccad\uc0c1\uc544\ub9ac, \uae34 \uc9c0\ub290\ub7ec\ubbf8 \uccad\uc0c1\uc544\ub9ac, \ubc31\uc0c1\uc544\ub9ac, \uc545\uc0c1\uc5b4, \uc5f0\uc5b4\uc0c1\uc5b4\ub4f1\uc758 \uc545\uc0c1\uc5b4\uacfc\uc758 \ubaa8\ub4e0 \uc0c1\uc5b4\ub4e4\ub3c4 \uc628\ud608\uc774\ub2e4. \ub610\ud55c \ud658\ub3c4\uc0c1\uc5b4\uacfc(\uc601\uc5b4:Alopiidae)\uc758 \uc885\ub4e4\ub3c4 \uc628\ud608\uc784\uc744 \ub098\ud0c0\ub0b4\ub294 \uc99d\uac70\uac00 \ubc1c\uacac\ub41c\ub2e4. \ub0b4\ubd80\uc5f4(\uc601\uc5b4:endothermy)\uc758 \uc628\ub3c4\ub294 \ub208\uacfc \ub1cc\ub9cc\uc744 \ub530\ub4ef\ud558\uac8c \ub9cc\ub4dc\ub294 \uc0c8\uce58\ub4e4\ub85c\ubd80\ud130 \uccb4\uc628\uc744 \uc8fc\ubcc0\uc758 \uc218\uc628\ubcf4\ub2e4 20\ub3c4 \uc815\ub3c4 \ub192\uac8c \uc720\uc9c0\ud558\ub294 \ucc38\uce58(\ucc38\ub2e4\ub791\uc5b4)\uc640 \uc545\uc0c1\uc5b4\uae4c\uc9c0 \uac01\uae30 \ub2e4\ub974\ub2e4. \"\uac70\ub300\uc628\ud608\ud56d\ubaa9\uc744 \ucc38\uc870\ud558\ub77c.\" \uc628\ud608\uc740 \ub300\uc0ac\uc801\uc73c\ub85c\ub294 \ub192\uc740 \ube44\uc6a9\uc774 \ub4e4\uc5b4\uac10\uc5d0\ub3c4, \ud5a5\uc0c1\ub41c \uadfc\uc721\uc758 \ud798, \uc911\uc559\uc2e0\uacbd\uacc4\uc758 \ube60\ub978 \uc18d\ub3c4, \uadf8\ub9ac\uace0 \uace0\ud6a8\uc728\uc758 \uc18c\ud654 \ub4f1\uc758 \uc774\uc810\uc744 \uc81c\uacf5\ud55c\ub2e4\uace0 \uc5ec\uaca8\uc9c4\ub2e4."} {"question": "\ubcf8\uba85\uc740 \ubab0\ub790\ub358 \ucd5c\uc9c4\uc2e4\uc774 \uc54c\uace0 \uc788\uc5c8\ub358 \uc720\uc7ac\uc11d\uc758 \ubcc4\uba85\uc740?", "paragraph": "\ubc29\uc1a1\uc778 \uc720\uc7ac\uc11d\uc740 \ucd5c\uc9c4\uc2e4\uc758 \ucd94\ucc9c\uc73c\ub85c \ud1b1\uc2a4\ud0c0\uac00 \ub41c \ub300\ud45c\uc801\uc778 \uc608\uc774\ub2e4. 2000\ub144, \ucd5c\uc9c4\uc2e4\uc740 MBC\uc758 \ud55c \uace0\uc704\uae09 \uc5f0\ucd9c\uac00\uc5d0\uac8c \uc720\uc7ac\uc11d\uc744 MBC \u300a\ubaa9\ud45c \ub2ec\uc131 \ud1a0\uc694\uc77c - \uc2a4\ud0c0 \uc11c\ubc14\uc774\ubc8c \ub3d9\uac70\ub3d9\ub77d\u300b\uc758 \uba54\uc778 MC\ub85c \ucd94\ucc9c\ud558\uc600\ub2e4. \ub2f9\uc2dc \ud06c\uac8c \uc720\uba85\ud558\uc9c0 \uc54a\uc558\ub358 \uc720\uc7ac\uc11d\uc740 \ucd5c\uc9c4\uc2e4\uacfc\ub294 \uce5c\ubd84\uc774 \uc5c6\uc5c8\ub2e4. \ucd5c\uc9c4\uc2e4\uc740 \uc720\uc7ac\uc11d\uc758 \ubcc4\uba85\uc778 '\uba54\ub69c\uae30'\ub294 \uc54c\uc544\ub3c4 \uc2e4\uba85\uc740 \uc54c\uc9c0 \ubabb\ud588\uc9c0\ub9cc KBS \u300a\uc11c\uc138\uc6d0 \uc1fc\u300b\uc5d0 \ucd9c\uc5f0\ud588\ub358 \uc720\uc7ac\uc11d\uc744 \uae30\uc5b5\ud558\uace0 \uc788\uc5c8\uace0, \ucd5c\uc9c4\uc2e4\uc774 \uadf8 \uace0\uc704\uae09 \uc5f0\ucd9c\uac00\uc5d0\uac8c \"\uc0c8 \ud504\ub85c\uadf8\ub7a8\uc774 \uc788\uc73c\uba74 '\uba54\ub69c\uae30'\ub97c \uc4f0\ub77c\"\ub77c\uace0 \uac15\ub825\ud558\uac8c \ucd94\ucc9c\ud558\uba74\uc11c \u300a\uc2a4\ud0c0 \uc11c\ubc14\uc774\ubc8c \ub3d9\uac70\ub3d9\ub77d\u300b\uc758 \uba54\uc778 MC\ub85c \uc720\uc7ac\uc11d\uc774 \ubc1c\ud0c1\ub418\uc5c8\ub2e4. \uc720\uc7ac\uc11d\uc740 \u300a\uc2a4\ud0c0 \uc11c\ubc14\uc774\ubc8c \ub3d9\uace0\ub3d9\ub77d\u300b\uc758 \uba54\uc778 MC\ub85c \ud65c\ub3d9\ud558\uba74\uc11c \ub300\uc911\uc73c\ub85c\ubd80\ud130 \ub9ce\uc740 \uc8fc\ubaa9\uc744 \ubc1b\uc558\uace0 \ud1b1\uc2a4\ud0c0\ub85c \uc778\uc815\ubc1b\ub294 \uc804\uae30\ub97c \ub9c8\ub828\ud558\uc600\ub2e4. \uc720\uc7ac\uc11d\uc740 \uadf8 \ud6c4 \ud55c\ucc38\uc774 \uc9c0\ub098\uc11c\uc57c \ucd5c\uc9c4\uc2e4\uc774 \uc790\uc2e0\uc744 \ucd94\ucc9c\ud588\ub2e4\ub294 \uc0ac\uc2e4\uc744 \uc54c\uac8c \ub418\uc5c8\ub2e4.", "answer": "\uba54\ub69c\uae30", "sentence": "\ucd5c\uc9c4\uc2e4\uc740 \uc720\uc7ac\uc11d\uc758 \ubcc4\uba85\uc778 ' \uba54\ub69c\uae30 '\ub294 \uc54c\uc544\ub3c4 \uc2e4\uba85\uc740 \uc54c\uc9c0 \ubabb\ud588\uc9c0\ub9cc KBS \u300a\uc11c\uc138\uc6d0 \uc1fc\u300b\uc5d0 \ucd9c\uc5f0\ud588\ub358 \uc720\uc7ac\uc11d\uc744 \uae30\uc5b5\ud558\uace0 \uc788\uc5c8\uace0, \ucd5c\uc9c4\uc2e4\uc774 \uadf8 \uace0\uc704\uae09 \uc5f0\ucd9c\uac00\uc5d0\uac8c \"\uc0c8 \ud504\ub85c\uadf8\ub7a8\uc774 \uc788\uc73c\uba74 '\uba54\ub69c\uae30'\ub97c \uc4f0\ub77c\"\ub77c\uace0 \uac15\ub825\ud558\uac8c \ucd94\ucc9c\ud558\uba74\uc11c \u300a\uc2a4\ud0c0 \uc11c\ubc14\uc774\ubc8c \ub3d9\uac70\ub3d9\ub77d\u300b\uc758 \uba54\uc778 MC\ub85c \uc720\uc7ac\uc11d\uc774 \ubc1c\ud0c1\ub418\uc5c8\ub2e4.", "paragraph_sentence": "\ubc29\uc1a1\uc778 \uc720\uc7ac\uc11d\uc740 \ucd5c\uc9c4\uc2e4\uc758 \ucd94\ucc9c\uc73c\ub85c \ud1b1\uc2a4\ud0c0\uac00 \ub41c \ub300\ud45c\uc801\uc778 \uc608\uc774\ub2e4. 2000\ub144, \ucd5c\uc9c4\uc2e4\uc740 MBC\uc758 \ud55c \uace0\uc704\uae09 \uc5f0\ucd9c\uac00\uc5d0\uac8c \uc720\uc7ac\uc11d\uc744 MBC \u300a\ubaa9\ud45c \ub2ec\uc131 \ud1a0\uc694\uc77c - \uc2a4\ud0c0 \uc11c\ubc14\uc774\ubc8c \ub3d9\uac70\ub3d9\ub77d\u300b\uc758 \uba54\uc778 MC\ub85c \ucd94\ucc9c\ud558\uc600\ub2e4. \ub2f9\uc2dc \ud06c\uac8c \uc720\uba85\ud558\uc9c0 \uc54a\uc558\ub358 \uc720\uc7ac\uc11d\uc740 \ucd5c\uc9c4\uc2e4\uacfc\ub294 \uce5c\ubd84\uc774 \uc5c6\uc5c8\ub2e4. \ucd5c\uc9c4\uc2e4\uc740 \uc720\uc7ac\uc11d\uc758 \ubcc4\uba85\uc778 ' \uba54\ub69c\uae30 '\ub294 \uc54c\uc544\ub3c4 \uc2e4\uba85\uc740 \uc54c\uc9c0 \ubabb\ud588\uc9c0\ub9cc KBS \u300a\uc11c\uc138\uc6d0 \uc1fc\u300b\uc5d0 \ucd9c\uc5f0\ud588\ub358 \uc720\uc7ac\uc11d\uc744 \uae30\uc5b5\ud558\uace0 \uc788\uc5c8\uace0, \ucd5c\uc9c4\uc2e4\uc774 \uadf8 \uace0\uc704\uae09 \uc5f0\ucd9c\uac00\uc5d0\uac8c \"\uc0c8 \ud504\ub85c\uadf8\ub7a8\uc774 \uc788\uc73c\uba74 '\uba54\ub69c\uae30'\ub97c \uc4f0\ub77c\"\ub77c\uace0 \uac15\ub825\ud558\uac8c \ucd94\ucc9c\ud558\uba74\uc11c \u300a\uc2a4\ud0c0 \uc11c\ubc14\uc774\ubc8c \ub3d9\uac70\ub3d9\ub77d\u300b\uc758 \uba54\uc778 MC\ub85c \uc720\uc7ac\uc11d\uc774 \ubc1c\ud0c1\ub418\uc5c8\ub2e4. \uc720\uc7ac\uc11d\uc740 \u300a\uc2a4\ud0c0 \uc11c\ubc14\uc774\ubc8c \ub3d9\uace0\ub3d9\ub77d\u300b\uc758 \uba54\uc778 MC\ub85c \ud65c\ub3d9\ud558\uba74\uc11c \ub300\uc911\uc73c\ub85c\ubd80\ud130 \ub9ce\uc740 \uc8fc\ubaa9\uc744 \ubc1b\uc558\uace0 \ud1b1\uc2a4\ud0c0\ub85c \uc778\uc815\ubc1b\ub294 \uc804\uae30\ub97c \ub9c8\ub828\ud558\uc600\ub2e4. \uc720\uc7ac\uc11d\uc740 \uadf8 \ud6c4 \ud55c\ucc38\uc774 \uc9c0\ub098\uc11c\uc57c \ucd5c\uc9c4\uc2e4\uc774 \uc790\uc2e0\uc744 \ucd94\ucc9c\ud588\ub2e4\ub294 \uc0ac\uc2e4\uc744 \uc54c\uac8c \ub418\uc5c8\ub2e4.", "paragraph_answer": "\ubc29\uc1a1\uc778 \uc720\uc7ac\uc11d\uc740 \ucd5c\uc9c4\uc2e4\uc758 \ucd94\ucc9c\uc73c\ub85c \ud1b1\uc2a4\ud0c0\uac00 \ub41c \ub300\ud45c\uc801\uc778 \uc608\uc774\ub2e4. 2000\ub144, \ucd5c\uc9c4\uc2e4\uc740 MBC\uc758 \ud55c \uace0\uc704\uae09 \uc5f0\ucd9c\uac00\uc5d0\uac8c \uc720\uc7ac\uc11d\uc744 MBC \u300a\ubaa9\ud45c \ub2ec\uc131 \ud1a0\uc694\uc77c - \uc2a4\ud0c0 \uc11c\ubc14\uc774\ubc8c \ub3d9\uac70\ub3d9\ub77d\u300b\uc758 \uba54\uc778 MC\ub85c \ucd94\ucc9c\ud558\uc600\ub2e4. \ub2f9\uc2dc \ud06c\uac8c \uc720\uba85\ud558\uc9c0 \uc54a\uc558\ub358 \uc720\uc7ac\uc11d\uc740 \ucd5c\uc9c4\uc2e4\uacfc\ub294 \uce5c\ubd84\uc774 \uc5c6\uc5c8\ub2e4. \ucd5c\uc9c4\uc2e4\uc740 \uc720\uc7ac\uc11d\uc758 \ubcc4\uba85\uc778 ' \uba54\ub69c\uae30 '\ub294 \uc54c\uc544\ub3c4 \uc2e4\uba85\uc740 \uc54c\uc9c0 \ubabb\ud588\uc9c0\ub9cc KBS \u300a\uc11c\uc138\uc6d0 \uc1fc\u300b\uc5d0 \ucd9c\uc5f0\ud588\ub358 \uc720\uc7ac\uc11d\uc744 \uae30\uc5b5\ud558\uace0 \uc788\uc5c8\uace0, \ucd5c\uc9c4\uc2e4\uc774 \uadf8 \uace0\uc704\uae09 \uc5f0\ucd9c\uac00\uc5d0\uac8c \"\uc0c8 \ud504\ub85c\uadf8\ub7a8\uc774 \uc788\uc73c\uba74 '\uba54\ub69c\uae30'\ub97c \uc4f0\ub77c\"\ub77c\uace0 \uac15\ub825\ud558\uac8c \ucd94\ucc9c\ud558\uba74\uc11c \u300a\uc2a4\ud0c0 \uc11c\ubc14\uc774\ubc8c \ub3d9\uac70\ub3d9\ub77d\u300b\uc758 \uba54\uc778 MC\ub85c \uc720\uc7ac\uc11d\uc774 \ubc1c\ud0c1\ub418\uc5c8\ub2e4. \uc720\uc7ac\uc11d\uc740 \u300a\uc2a4\ud0c0 \uc11c\ubc14\uc774\ubc8c \ub3d9\uace0\ub3d9\ub77d\u300b\uc758 \uba54\uc778 MC\ub85c \ud65c\ub3d9\ud558\uba74\uc11c \ub300\uc911\uc73c\ub85c\ubd80\ud130 \ub9ce\uc740 \uc8fc\ubaa9\uc744 \ubc1b\uc558\uace0 \ud1b1\uc2a4\ud0c0\ub85c \uc778\uc815\ubc1b\ub294 \uc804\uae30\ub97c \ub9c8\ub828\ud558\uc600\ub2e4. \uc720\uc7ac\uc11d\uc740 \uadf8 \ud6c4 \ud55c\ucc38\uc774 \uc9c0\ub098\uc11c\uc57c \ucd5c\uc9c4\uc2e4\uc774 \uc790\uc2e0\uc744 \ucd94\ucc9c\ud588\ub2e4\ub294 \uc0ac\uc2e4\uc744 \uc54c\uac8c \ub418\uc5c8\ub2e4.", "sentence_answer": "\ucd5c\uc9c4\uc2e4\uc740 \uc720\uc7ac\uc11d\uc758 \ubcc4\uba85\uc778 ' \uba54\ub69c\uae30 '\ub294 \uc54c\uc544\ub3c4 \uc2e4\uba85\uc740 \uc54c\uc9c0 \ubabb\ud588\uc9c0\ub9cc KBS \u300a\uc11c\uc138\uc6d0 \uc1fc\u300b\uc5d0 \ucd9c\uc5f0\ud588\ub358 \uc720\uc7ac\uc11d\uc744 \uae30\uc5b5\ud558\uace0 \uc788\uc5c8\uace0, \ucd5c\uc9c4\uc2e4\uc774 \uadf8 \uace0\uc704\uae09 \uc5f0\ucd9c\uac00\uc5d0\uac8c \"\uc0c8 \ud504\ub85c\uadf8\ub7a8\uc774 \uc788\uc73c\uba74 '\uba54\ub69c\uae30'\ub97c \uc4f0\ub77c\"\ub77c\uace0 \uac15\ub825\ud558\uac8c \ucd94\ucc9c\ud558\uba74\uc11c \u300a\uc2a4\ud0c0 \uc11c\ubc14\uc774\ubc8c \ub3d9\uac70\ub3d9\ub77d\u300b\uc758 \uba54\uc778 MC\ub85c \uc720\uc7ac\uc11d\uc774 \ubc1c\ud0c1\ub418\uc5c8\ub2e4.", "paragraph_id": "6596968-5-1", "paragraph_question": "question: \ubcf8\uba85\uc740 \ubab0\ub790\ub358 \ucd5c\uc9c4\uc2e4\uc774 \uc54c\uace0 \uc788\uc5c8\ub358 \uc720\uc7ac\uc11d\uc758 \ubcc4\uba85\uc740?, context: \ubc29\uc1a1\uc778 \uc720\uc7ac\uc11d\uc740 \ucd5c\uc9c4\uc2e4\uc758 \ucd94\ucc9c\uc73c\ub85c \ud1b1\uc2a4\ud0c0\uac00 \ub41c \ub300\ud45c\uc801\uc778 \uc608\uc774\ub2e4. 2000\ub144, \ucd5c\uc9c4\uc2e4\uc740 MBC\uc758 \ud55c \uace0\uc704\uae09 \uc5f0\ucd9c\uac00\uc5d0\uac8c \uc720\uc7ac\uc11d\uc744 MBC \u300a\ubaa9\ud45c \ub2ec\uc131 \ud1a0\uc694\uc77c - \uc2a4\ud0c0 \uc11c\ubc14\uc774\ubc8c \ub3d9\uac70\ub3d9\ub77d\u300b\uc758 \uba54\uc778 MC\ub85c \ucd94\ucc9c\ud558\uc600\ub2e4. \ub2f9\uc2dc \ud06c\uac8c \uc720\uba85\ud558\uc9c0 \uc54a\uc558\ub358 \uc720\uc7ac\uc11d\uc740 \ucd5c\uc9c4\uc2e4\uacfc\ub294 \uce5c\ubd84\uc774 \uc5c6\uc5c8\ub2e4. \ucd5c\uc9c4\uc2e4\uc740 \uc720\uc7ac\uc11d\uc758 \ubcc4\uba85\uc778 '\uba54\ub69c\uae30'\ub294 \uc54c\uc544\ub3c4 \uc2e4\uba85\uc740 \uc54c\uc9c0 \ubabb\ud588\uc9c0\ub9cc KBS \u300a\uc11c\uc138\uc6d0 \uc1fc\u300b\uc5d0 \ucd9c\uc5f0\ud588\ub358 \uc720\uc7ac\uc11d\uc744 \uae30\uc5b5\ud558\uace0 \uc788\uc5c8\uace0, \ucd5c\uc9c4\uc2e4\uc774 \uadf8 \uace0\uc704\uae09 \uc5f0\ucd9c\uac00\uc5d0\uac8c \"\uc0c8 \ud504\ub85c\uadf8\ub7a8\uc774 \uc788\uc73c\uba74 '\uba54\ub69c\uae30'\ub97c \uc4f0\ub77c\"\ub77c\uace0 \uac15\ub825\ud558\uac8c \ucd94\ucc9c\ud558\uba74\uc11c \u300a\uc2a4\ud0c0 \uc11c\ubc14\uc774\ubc8c \ub3d9\uac70\ub3d9\ub77d\u300b\uc758 \uba54\uc778 MC\ub85c \uc720\uc7ac\uc11d\uc774 \ubc1c\ud0c1\ub418\uc5c8\ub2e4. \uc720\uc7ac\uc11d\uc740 \u300a\uc2a4\ud0c0 \uc11c\ubc14\uc774\ubc8c \ub3d9\uace0\ub3d9\ub77d\u300b\uc758 \uba54\uc778 MC\ub85c \ud65c\ub3d9\ud558\uba74\uc11c \ub300\uc911\uc73c\ub85c\ubd80\ud130 \ub9ce\uc740 \uc8fc\ubaa9\uc744 \ubc1b\uc558\uace0 \ud1b1\uc2a4\ud0c0\ub85c \uc778\uc815\ubc1b\ub294 \uc804\uae30\ub97c \ub9c8\ub828\ud558\uc600\ub2e4. \uc720\uc7ac\uc11d\uc740 \uadf8 \ud6c4 \ud55c\ucc38\uc774 \uc9c0\ub098\uc11c\uc57c \ucd5c\uc9c4\uc2e4\uc774 \uc790\uc2e0\uc744 \ucd94\ucc9c\ud588\ub2e4\ub294 \uc0ac\uc2e4\uc744 \uc54c\uac8c \ub418\uc5c8\ub2e4."} {"question": "2009\ub144 5\uc6d4, \ub9c8\ub204 \ucc28\uc624\uc758 \uace1 Mr.Bobby\ub97c \ucee4\ubc84\ud574 \uc18c\uac1c\ud55c B.O.B\uac00 \ubc1c\ud45c\ud55c \ud65c\ub3d9\uba85\uc740?", "paragraph": "2009\ub144 5\uc6d4 B.o.B\ub294 \ub9c8\ub204 \ucc28\uc624(Manu Chao)\uc758 \uace1 \"Mr. Bobby\"\ub97c \ucee4\ubc84\ud574\uc11c \ubc1c\ud45c\ud558\uba70 \ubc14\ube44 \ub808\uc774 \ud558\ub098\uc758 \uc774\ub984\ub9cc \uc0ac\uc6a9\ud558\uae38 \uc6d0\ud55c\ub2e4\uace0 \ubc1c\ud45c\ud588\ub2e4. \uadf8\ub9ac\uace0 \uadf8\ub294 \uc0c8\ub85c\uc6b4 \ubc29\ud5a5(new direction)\uc73c\ub85c \uc74c\uc545\uc744 \uc9c4\ud589\ud560 \uac70\ub77c\uace0 \ub9d0\ud588\ub2e4. : \"\uc774\uc81c \ub098\ub294 \ubc14\ube44 \ub808\uc774\ub85c \ud65c\ub3d9\ud560 \uac70\uc57c. \ubc14\ube44 \ub808\uc774\ub97c \ud655\uc2e4\ud558\uac8c \uc0c8\uaca8 \ub46c. \ub098\ub294 \uc815\ub9d0\ub85c \uc0c8\ub85c\uc6b4 \ubc29\ud5a5\uc73c\ub85c \ub098\uc544\uac08 \uac70\uc57c -\ub354 \uc790\uc720\ub86d\uac8c \ud2b9\uc815\ud55c \uc7a5\ub974\uc5d0 \ubd80\ud569\ud558\ub824 \ub178\ub825\ud558\uc9c0 \uc54a\uc744 \uac70\uc57c. \ub098\ub294 \uadf8\uac83\uc744 \ud574\uc654\ub358 \uac83 \uac19\uc9c0 \uc54a\uc544. \ub098\ub294 \uadf8\uac83\uc744 \uc77c\uce58 \uc2dc\ud0a4\ub824\uace0 \ub178\ub825\ud588\ub358 \uac83 \uac19\uc9c0 \uc54a\uc544. \ud558\uc9c0\ub9cc \ub098\ub294 \ub354 \ub9ce\uc740 \uc790\uc720\ub85c\uc6b4 \uc815\uc2e0\uc744 \ub290\uaef4. \ub098\ub294 \ub354 \ub9ce\uc740 \uc74c\uc545\uc801 \uc758\uc9c0\ub97c \uac00\uc84c\uc5b4. \uadf8\uac8c \ub098\uc5d0 \uad00\ud55c \ubaa8\ub4e0 \uac70\uc57c. \ub098\ub294 \uae30\ud0c0\uc774\uba70 \ud654\uc74c\uc774\uc790 \ud0a4\ubcf4\ub4dc\uc57c.\u201c 2009\ub144 6\uc6d4 22\uc77c B.o.B\ub294 \uadf8\uc758 \uc5ec\uc12f \ubc88\uc9f8 \ubbf9\uc2a4\ud14c\uc774\ud504 B.o.B vs. Bobby Ray\ub97c \ubc1c\ub9e4\ud588\ub2e4. T.I.'s T.I. vs. T.I.P. (2007)\ub97c \uc5f0\uc0c1\uc2dc\ud0a4\ub294 \uc774 \ubbf9\uc2a4\ud14c\uc774\ud504\ub294 \ud4e8\ub9ac(Fury)\uc640 B.o.B \uc911\uc2ec\uc73c\ub85c \uc81c\uc791\ub410\ub2e4. \uc774 \ubbf9\uc2a4\ud14c\uc774\ud504\uc5d0\ub294 \ub808\ub4dc \uc2a4\ud30c\uc774\ub2e4(Red Spyda)\uac00 \ud504\ub85c\ub4c0\uc11c\ud55c \"Fly Like Me\"\uc640 \ud2b8\ub77c\uc774\ube0c \ucf5c\ub4dc \ud018\uc2a4\ud2b8(A Tribe Called Quest)\uc758 1990\ub144\uc758 \ud788\ud2b8\uace1 \"Bonita Applebum\"\uc744 \ub9ac\ubbf9\uc2a4\ud55c \"Put Me On\"\uc774 \uc2e4\ub824 \uc788\ub2e4. \ub9ac\ubcf5\uc740 \ub9ac\ubcf5 \ud074\ub798\uc2dd \ub9ac\ubbf9\uc2a4 \uceec\ub809\uc158 \uae30\ub150\ud589\uc0ac\uc5d0\uc11c \"Put Me On\"\uc744 \uc0ac\uc6a9\ud588\ub2e4. \uc774 \ud2b8\ub799\uc740 \uc804\ubbf8 \ud48b \ub77d\ucee4(Foot Locker)\uc0c1\uc810\uc5d0\uc11c \ub9ac\ubcf5 \ud074\ub798\uc2dd \ub9ac\ubbf9\uc2a4 \uceec\ub809\uc158 \uc81c\ud488\uc744 \uad6c\uc785\ud55c \uace0\uac1d\ub4e4\ub9cc \ub3c5\uc810\uc801\uc73c\ub85c \ub2e4\uc6b4\ubc1b\uc744 \uc218 \uc788\uc5c8\ub2e4.", "answer": "\ubc14\ube44 \ub808\uc774", "sentence": "Bobby\"\ub97c \ucee4\ubc84\ud574\uc11c \ubc1c\ud45c\ud558\uba70 \ubc14\ube44 \ub808\uc774 \ud558\ub098\uc758 \uc774\ub984\ub9cc \uc0ac\uc6a9\ud558\uae38 \uc6d0\ud55c\ub2e4\uace0 \ubc1c\ud45c\ud588\ub2e4.", "paragraph_sentence": "2009\ub144 5\uc6d4 B.o. B\ub294 \ub9c8\ub204 \ucc28\uc624(Manu Chao)\uc758 \uace1 \"Mr. Bobby\"\ub97c \ucee4\ubc84\ud574\uc11c \ubc1c\ud45c\ud558\uba70 \ubc14\ube44 \ub808\uc774 \ud558\ub098\uc758 \uc774\ub984\ub9cc \uc0ac\uc6a9\ud558\uae38 \uc6d0\ud55c\ub2e4\uace0 \ubc1c\ud45c\ud588\ub2e4. \uadf8\ub9ac\uace0 \uadf8\ub294 \uc0c8\ub85c\uc6b4 \ubc29\ud5a5(new direction)\uc73c\ub85c \uc74c\uc545\uc744 \uc9c4\ud589\ud560 \uac70\ub77c\uace0 \ub9d0\ud588\ub2e4. : \"\uc774\uc81c \ub098\ub294 \ubc14\ube44 \ub808\uc774\ub85c \ud65c\ub3d9\ud560 \uac70\uc57c. \ubc14\ube44 \ub808\uc774\ub97c \ud655\uc2e4\ud558\uac8c \uc0c8\uaca8 \ub46c. \ub098\ub294 \uc815\ub9d0\ub85c \uc0c8\ub85c\uc6b4 \ubc29\ud5a5\uc73c\ub85c \ub098\uc544\uac08 \uac70\uc57c -\ub354 \uc790\uc720\ub86d\uac8c \ud2b9\uc815\ud55c \uc7a5\ub974\uc5d0 \ubd80\ud569\ud558\ub824 \ub178\ub825\ud558\uc9c0 \uc54a\uc744 \uac70\uc57c. \ub098\ub294 \uadf8\uac83\uc744 \ud574\uc654\ub358 \uac83 \uac19\uc9c0 \uc54a\uc544. \ub098\ub294 \uadf8\uac83\uc744 \uc77c\uce58 \uc2dc\ud0a4\ub824\uace0 \ub178\ub825\ud588\ub358 \uac83 \uac19\uc9c0 \uc54a\uc544. \ud558\uc9c0\ub9cc \ub098\ub294 \ub354 \ub9ce\uc740 \uc790\uc720\ub85c\uc6b4 \uc815\uc2e0\uc744 \ub290\uaef4. \ub098\ub294 \ub354 \ub9ce\uc740 \uc74c\uc545\uc801 \uc758\uc9c0\ub97c \uac00\uc84c\uc5b4. \uadf8\uac8c \ub098\uc5d0 \uad00\ud55c \ubaa8\ub4e0 \uac70\uc57c. \ub098\ub294 \uae30\ud0c0\uc774\uba70 \ud654\uc74c\uc774\uc790 \ud0a4\ubcf4\ub4dc\uc57c. \u201c 2009\ub144 6\uc6d4 22\uc77c B.o. B\ub294 \uadf8\uc758 \uc5ec\uc12f \ubc88\uc9f8 \ubbf9\uc2a4\ud14c\uc774\ud504 B.o. B vs. Bobby Ray\ub97c \ubc1c\ub9e4\ud588\ub2e4. T.I.'s T.I. vs. T.I.P. (2007)\ub97c \uc5f0\uc0c1\uc2dc\ud0a4\ub294 \uc774 \ubbf9\uc2a4\ud14c\uc774\ud504\ub294 \ud4e8\ub9ac(Fury)\uc640 B.o. B \uc911\uc2ec\uc73c\ub85c \uc81c\uc791\ub410\ub2e4. \uc774 \ubbf9\uc2a4\ud14c\uc774\ud504\uc5d0\ub294 \ub808\ub4dc \uc2a4\ud30c\uc774\ub2e4(Red Spyda)\uac00 \ud504\ub85c\ub4c0\uc11c\ud55c \"Fly Like Me\"\uc640 \ud2b8\ub77c\uc774\ube0c \ucf5c\ub4dc \ud018\uc2a4\ud2b8(A Tribe Called Quest)\uc758 1990\ub144\uc758 \ud788\ud2b8\uace1 \"Bonita Applebum\"\uc744 \ub9ac\ubbf9\uc2a4\ud55c \"Put Me On\"\uc774 \uc2e4\ub824 \uc788\ub2e4. \ub9ac\ubcf5\uc740 \ub9ac\ubcf5 \ud074\ub798\uc2dd \ub9ac\ubbf9\uc2a4 \uceec\ub809\uc158 \uae30\ub150\ud589\uc0ac\uc5d0\uc11c \"Put Me On\"\uc744 \uc0ac\uc6a9\ud588\ub2e4. \uc774 \ud2b8\ub799\uc740 \uc804\ubbf8 \ud48b \ub77d\ucee4(Foot Locker)\uc0c1\uc810\uc5d0\uc11c \ub9ac\ubcf5 \ud074\ub798\uc2dd \ub9ac\ubbf9\uc2a4 \uceec\ub809\uc158 \uc81c\ud488\uc744 \uad6c\uc785\ud55c \uace0\uac1d\ub4e4\ub9cc \ub3c5\uc810\uc801\uc73c\ub85c \ub2e4\uc6b4\ubc1b\uc744 \uc218 \uc788\uc5c8\ub2e4.", "paragraph_answer": "2009\ub144 5\uc6d4 B.o.B\ub294 \ub9c8\ub204 \ucc28\uc624(Manu Chao)\uc758 \uace1 \"Mr. Bobby\"\ub97c \ucee4\ubc84\ud574\uc11c \ubc1c\ud45c\ud558\uba70 \ubc14\ube44 \ub808\uc774 \ud558\ub098\uc758 \uc774\ub984\ub9cc \uc0ac\uc6a9\ud558\uae38 \uc6d0\ud55c\ub2e4\uace0 \ubc1c\ud45c\ud588\ub2e4. \uadf8\ub9ac\uace0 \uadf8\ub294 \uc0c8\ub85c\uc6b4 \ubc29\ud5a5(new direction)\uc73c\ub85c \uc74c\uc545\uc744 \uc9c4\ud589\ud560 \uac70\ub77c\uace0 \ub9d0\ud588\ub2e4. : \"\uc774\uc81c \ub098\ub294 \ubc14\ube44 \ub808\uc774\ub85c \ud65c\ub3d9\ud560 \uac70\uc57c. \ubc14\ube44 \ub808\uc774\ub97c \ud655\uc2e4\ud558\uac8c \uc0c8\uaca8 \ub46c. \ub098\ub294 \uc815\ub9d0\ub85c \uc0c8\ub85c\uc6b4 \ubc29\ud5a5\uc73c\ub85c \ub098\uc544\uac08 \uac70\uc57c -\ub354 \uc790\uc720\ub86d\uac8c \ud2b9\uc815\ud55c \uc7a5\ub974\uc5d0 \ubd80\ud569\ud558\ub824 \ub178\ub825\ud558\uc9c0 \uc54a\uc744 \uac70\uc57c. \ub098\ub294 \uadf8\uac83\uc744 \ud574\uc654\ub358 \uac83 \uac19\uc9c0 \uc54a\uc544. \ub098\ub294 \uadf8\uac83\uc744 \uc77c\uce58 \uc2dc\ud0a4\ub824\uace0 \ub178\ub825\ud588\ub358 \uac83 \uac19\uc9c0 \uc54a\uc544. \ud558\uc9c0\ub9cc \ub098\ub294 \ub354 \ub9ce\uc740 \uc790\uc720\ub85c\uc6b4 \uc815\uc2e0\uc744 \ub290\uaef4. \ub098\ub294 \ub354 \ub9ce\uc740 \uc74c\uc545\uc801 \uc758\uc9c0\ub97c \uac00\uc84c\uc5b4. \uadf8\uac8c \ub098\uc5d0 \uad00\ud55c \ubaa8\ub4e0 \uac70\uc57c. \ub098\ub294 \uae30\ud0c0\uc774\uba70 \ud654\uc74c\uc774\uc790 \ud0a4\ubcf4\ub4dc\uc57c.\u201c 2009\ub144 6\uc6d4 22\uc77c B.o.B\ub294 \uadf8\uc758 \uc5ec\uc12f \ubc88\uc9f8 \ubbf9\uc2a4\ud14c\uc774\ud504 B.o.B vs. Bobby Ray\ub97c \ubc1c\ub9e4\ud588\ub2e4. T.I.'s T.I. vs. T.I.P. (2007)\ub97c \uc5f0\uc0c1\uc2dc\ud0a4\ub294 \uc774 \ubbf9\uc2a4\ud14c\uc774\ud504\ub294 \ud4e8\ub9ac(Fury)\uc640 B.o.B \uc911\uc2ec\uc73c\ub85c \uc81c\uc791\ub410\ub2e4. \uc774 \ubbf9\uc2a4\ud14c\uc774\ud504\uc5d0\ub294 \ub808\ub4dc \uc2a4\ud30c\uc774\ub2e4(Red Spyda)\uac00 \ud504\ub85c\ub4c0\uc11c\ud55c \"Fly Like Me\"\uc640 \ud2b8\ub77c\uc774\ube0c \ucf5c\ub4dc \ud018\uc2a4\ud2b8(A Tribe Called Quest)\uc758 1990\ub144\uc758 \ud788\ud2b8\uace1 \"Bonita Applebum\"\uc744 \ub9ac\ubbf9\uc2a4\ud55c \"Put Me On\"\uc774 \uc2e4\ub824 \uc788\ub2e4. \ub9ac\ubcf5\uc740 \ub9ac\ubcf5 \ud074\ub798\uc2dd \ub9ac\ubbf9\uc2a4 \uceec\ub809\uc158 \uae30\ub150\ud589\uc0ac\uc5d0\uc11c \"Put Me On\"\uc744 \uc0ac\uc6a9\ud588\ub2e4. \uc774 \ud2b8\ub799\uc740 \uc804\ubbf8 \ud48b \ub77d\ucee4(Foot Locker)\uc0c1\uc810\uc5d0\uc11c \ub9ac\ubcf5 \ud074\ub798\uc2dd \ub9ac\ubbf9\uc2a4 \uceec\ub809\uc158 \uc81c\ud488\uc744 \uad6c\uc785\ud55c \uace0\uac1d\ub4e4\ub9cc \ub3c5\uc810\uc801\uc73c\ub85c \ub2e4\uc6b4\ubc1b\uc744 \uc218 \uc788\uc5c8\ub2e4.", "sentence_answer": "Bobby\"\ub97c \ucee4\ubc84\ud574\uc11c \ubc1c\ud45c\ud558\uba70 \ubc14\ube44 \ub808\uc774 \ud558\ub098\uc758 \uc774\ub984\ub9cc \uc0ac\uc6a9\ud558\uae38 \uc6d0\ud55c\ub2e4\uace0 \ubc1c\ud45c\ud588\ub2e4.", "paragraph_id": "6491044-5-0", "paragraph_question": "question: 2009\ub144 5\uc6d4, \ub9c8\ub204 \ucc28\uc624\uc758 \uace1 Mr.Bobby\ub97c \ucee4\ubc84\ud574 \uc18c\uac1c\ud55c B.O.B\uac00 \ubc1c\ud45c\ud55c \ud65c\ub3d9\uba85\uc740?, context: 2009\ub144 5\uc6d4 B.o.B\ub294 \ub9c8\ub204 \ucc28\uc624(Manu Chao)\uc758 \uace1 \"Mr. Bobby\"\ub97c \ucee4\ubc84\ud574\uc11c \ubc1c\ud45c\ud558\uba70 \ubc14\ube44 \ub808\uc774 \ud558\ub098\uc758 \uc774\ub984\ub9cc \uc0ac\uc6a9\ud558\uae38 \uc6d0\ud55c\ub2e4\uace0 \ubc1c\ud45c\ud588\ub2e4. \uadf8\ub9ac\uace0 \uadf8\ub294 \uc0c8\ub85c\uc6b4 \ubc29\ud5a5(new direction)\uc73c\ub85c \uc74c\uc545\uc744 \uc9c4\ud589\ud560 \uac70\ub77c\uace0 \ub9d0\ud588\ub2e4. : \"\uc774\uc81c \ub098\ub294 \ubc14\ube44 \ub808\uc774\ub85c \ud65c\ub3d9\ud560 \uac70\uc57c. \ubc14\ube44 \ub808\uc774\ub97c \ud655\uc2e4\ud558\uac8c \uc0c8\uaca8 \ub46c. \ub098\ub294 \uc815\ub9d0\ub85c \uc0c8\ub85c\uc6b4 \ubc29\ud5a5\uc73c\ub85c \ub098\uc544\uac08 \uac70\uc57c -\ub354 \uc790\uc720\ub86d\uac8c \ud2b9\uc815\ud55c \uc7a5\ub974\uc5d0 \ubd80\ud569\ud558\ub824 \ub178\ub825\ud558\uc9c0 \uc54a\uc744 \uac70\uc57c. \ub098\ub294 \uadf8\uac83\uc744 \ud574\uc654\ub358 \uac83 \uac19\uc9c0 \uc54a\uc544. \ub098\ub294 \uadf8\uac83\uc744 \uc77c\uce58 \uc2dc\ud0a4\ub824\uace0 \ub178\ub825\ud588\ub358 \uac83 \uac19\uc9c0 \uc54a\uc544. \ud558\uc9c0\ub9cc \ub098\ub294 \ub354 \ub9ce\uc740 \uc790\uc720\ub85c\uc6b4 \uc815\uc2e0\uc744 \ub290\uaef4. \ub098\ub294 \ub354 \ub9ce\uc740 \uc74c\uc545\uc801 \uc758\uc9c0\ub97c \uac00\uc84c\uc5b4. \uadf8\uac8c \ub098\uc5d0 \uad00\ud55c \ubaa8\ub4e0 \uac70\uc57c. \ub098\ub294 \uae30\ud0c0\uc774\uba70 \ud654\uc74c\uc774\uc790 \ud0a4\ubcf4\ub4dc\uc57c.\u201c 2009\ub144 6\uc6d4 22\uc77c B.o.B\ub294 \uadf8\uc758 \uc5ec\uc12f \ubc88\uc9f8 \ubbf9\uc2a4\ud14c\uc774\ud504 B.o.B vs. Bobby Ray\ub97c \ubc1c\ub9e4\ud588\ub2e4. T.I.'s T.I. vs. T.I.P. (2007)\ub97c \uc5f0\uc0c1\uc2dc\ud0a4\ub294 \uc774 \ubbf9\uc2a4\ud14c\uc774\ud504\ub294 \ud4e8\ub9ac(Fury)\uc640 B.o.B \uc911\uc2ec\uc73c\ub85c \uc81c\uc791\ub410\ub2e4. \uc774 \ubbf9\uc2a4\ud14c\uc774\ud504\uc5d0\ub294 \ub808\ub4dc \uc2a4\ud30c\uc774\ub2e4(Red Spyda)\uac00 \ud504\ub85c\ub4c0\uc11c\ud55c \"Fly Like Me\"\uc640 \ud2b8\ub77c\uc774\ube0c \ucf5c\ub4dc \ud018\uc2a4\ud2b8(A Tribe Called Quest)\uc758 1990\ub144\uc758 \ud788\ud2b8\uace1 \"Bonita Applebum\"\uc744 \ub9ac\ubbf9\uc2a4\ud55c \"Put Me On\"\uc774 \uc2e4\ub824 \uc788\ub2e4. \ub9ac\ubcf5\uc740 \ub9ac\ubcf5 \ud074\ub798\uc2dd \ub9ac\ubbf9\uc2a4 \uceec\ub809\uc158 \uae30\ub150\ud589\uc0ac\uc5d0\uc11c \"Put Me On\"\uc744 \uc0ac\uc6a9\ud588\ub2e4. \uc774 \ud2b8\ub799\uc740 \uc804\ubbf8 \ud48b \ub77d\ucee4(Foot Locker)\uc0c1\uc810\uc5d0\uc11c \ub9ac\ubcf5 \ud074\ub798\uc2dd \ub9ac\ubbf9\uc2a4 \uceec\ub809\uc158 \uc81c\ud488\uc744 \uad6c\uc785\ud55c \uace0\uac1d\ub4e4\ub9cc \ub3c5\uc810\uc801\uc73c\ub85c \ub2e4\uc6b4\ubc1b\uc744 \uc218 \uc788\uc5c8\ub2e4."} {"question": "\uce74\uc774\ud37c\uac00 \ubc1c\uc804\ub41c \ubb38\ud654\ub97c \ud1b5\ud574 \uc601\uad11\uc744 \ub3cc\ub824\uc57c \ud55c\ub2e4\uace0 \ud55c \ub300\uc0c1\uc740?", "paragraph": "\uc601\uad6d\uacfc \ubbf8\uad6d\uc774 \uc778\ub958\ud559\uc73c\ub85c \ubc1c\uc804\ud55c \ubc18\uba74 \ubb38\ud654\uc640 \uad00\ub828\ud558\uc5ec \ud65c\ubc1c\ud558\uac8c \uc5f0\uad6c\ub418\uace0 \ubc1c\ud45c\ub41c \ub098\ub77c\ub294 \ud654\ub780\uc774\ub2e4. \ubb38\ud654\uc758 \uc804\ub3c4\uc0ac\ub85c \ub300\ud45c\uc801 \ub300\ubd80\ub294 \ud654\ub780\uc758 \uc544\ube0c\ub77c\ud568 \uce74\uc774\ud37c(Abraham Kuyper, 1837-1920)\uc774\ub2e4. \uadf8\ub294 \ud558\ub098\ub2d8\uc758 \uc601\uc5ed\uc8fc\uad8c(sphere sovereignty) \uc0ac\uc0c1\uc744 \uac15\uc870\ud588\ub294\ub370, \uc601\uc5ed\uc8fc\uad8c\uc774\ub780 \ud558\ub098\ub2d8\uc774 \uc885\uad50\uc5d0\uc11c \ubfd0\ub9cc \uc544\ub2c8\ub77c \uc815\uce58, \uacbd\uc81c, \uad50\uc721, \ubb38\ud654 \ub4f1 \uc628 \uc6b0\uc8fc\uc758 \uc601\uc5ed\uc5d0\uc11c \ucc3d\uc870\uc8fc \ud558\ub098\ub2d8\uc73c\ub85c\uc11c \uc804\uc801\uc778 \uc8fc\uad8c\uc744 \uac00\uc9c0\uace0 \ud1b5\uce58\ud558\uc2e0\ub2e4\ub294 \uc0ac\uc0c1\uc774\ub2e4. \ubc15\ud0dc\ud604\uc5d0 \ub530\ub974\uba74 \uce74\uc774\ud37c\uc758 \uc601\uc5ed\uc8fc\uad8c \uac1c\ub150\uc5d0\uc11c \u2018\uc601\uc5ed\u2019\uc774\ub780 \uc77c\ucc28\uc801\uc73c\ub85c \ud558\ub098\ub2d8\uc758 \ub2e4\uc591\ud55c \ucc3d\uc870\uc138\uacc4\ub85c\uc11c \uad6d\uac00, \uad50\ud68c, \ubb38\ud654, \ud559\ubb38, \uc608\uc220 \ub4f1\uc758 \uc77c\uc0c1\uc801 \uc0ac\ud68c \uc0dd\ud65c\uc758 \ubc94\uc8fc\ub97c \uc758\ubbf8\ud558\uba70, \ub3d9\uc2dc\uc5d0 \u2018\uc601\uc5ed\u2019\uc740 \ub2e4\ub978 \uc138\uacc4\uad00\uc744 \uac00\uc9c4 \uc0ac\ub78c\ub4e4\uc744 \uc758\ubbf8\ud55c\ub2e4\uace0 \ud55c\ub2e4. \uadf8\ub7f0\ub370 \uc2e0\ud559\uc5d0\uc11c \uad6c\uc6d0\uc744 \uc5bb\ub294 \ud2b9\ubcc4\uc740\ucd1d\uc5d0 \ub300\ud55c \uac15\uc870\ub294 \ub9ce\uc9c0\ub9cc \uc77c\ubc18\uc740\ucd1d(common grace)\uc5d0 \ub300\ud558\uc5ec \uc18c\ud640\ud558\uac8c \ub2e4\ub8ec \uc810\uc744 \uc54c\uc558\ub358 \uce74\uc774\ud37c\ub294 \u300e\uc77c\ubc18 \uc740\ucd1d\u300f(De Gemeene Gratie)\uc774\ub77c\ub294 \uc81c\ubaa9\uc758 \ucc45\uc5d0\uc11c \uc778\ub958\uc758 \ubb38\ud654 \uc804\ubc18\uc5d0 \ub300\ud574 \uae4a\uace0\ub3c4 \ud3ed\ub113\uc740 \uc0ac\uc0c1\uc744 \uc804\uac1c\ud558\ub294\ub370, \ud558\ub098\ub2d8\uc758 \ub73b\uc740 \uc6b0\ub9ac\uac00 \uad6c\uc6d0\ubc1b\uace0 \ucc9c\uad6d\uc5d0 \uac00\ub294 \uac83\ub9cc\uc774 \uc544\ub2c8\ub77c, \uc628 \uc778\ub958\uc758 \ubb38\ud654\uac00 \uacc4\uc18d\uc801\uc73c\ub85c \ubc1c\uc804\ud558\uc5ec \uc774 \ubc1c\uc804\ub41c \ubb38\ud654\ub97c \ud1b5\ud558\uc5ec \ud558\ub098\ub2d8\uaed8 \uc601\uad11 \ub3cc\ub824\uc57c \ud55c\ub2e4\ub294 \uac83\uc774\ub2e4.", "answer": "\ud558\ub098\ub2d8", "sentence": "\uadf8\ub294 \ud558\ub098\ub2d8 \uc758 \uc601\uc5ed\uc8fc\uad8c(sphere sovereignty) \uc0ac\uc0c1\uc744 \uac15\uc870\ud588\ub294\ub370, \uc601\uc5ed\uc8fc\uad8c\uc774\ub780 \ud558\ub098\ub2d8\uc774 \uc885\uad50\uc5d0\uc11c \ubfd0\ub9cc \uc544\ub2c8\ub77c \uc815\uce58, \uacbd\uc81c, \uad50\uc721, \ubb38\ud654 \ub4f1 \uc628 \uc6b0\uc8fc\uc758 \uc601\uc5ed\uc5d0\uc11c \ucc3d\uc870\uc8fc \ud558\ub098\ub2d8\uc73c\ub85c\uc11c \uc804\uc801\uc778 \uc8fc\uad8c\uc744 \uac00\uc9c0\uace0 \ud1b5\uce58\ud558\uc2e0\ub2e4\ub294 \uc0ac\uc0c1\uc774\ub2e4.", "paragraph_sentence": "\uc601\uad6d\uacfc \ubbf8\uad6d\uc774 \uc778\ub958\ud559\uc73c\ub85c \ubc1c\uc804\ud55c \ubc18\uba74 \ubb38\ud654\uc640 \uad00\ub828\ud558\uc5ec \ud65c\ubc1c\ud558\uac8c \uc5f0\uad6c\ub418\uace0 \ubc1c\ud45c\ub41c \ub098\ub77c\ub294 \ud654\ub780\uc774\ub2e4. \ubb38\ud654\uc758 \uc804\ub3c4\uc0ac\ub85c \ub300\ud45c\uc801 \ub300\ubd80\ub294 \ud654\ub780\uc758 \uc544\ube0c\ub77c\ud568 \uce74\uc774\ud37c(Abraham Kuyper, 1837-1920)\uc774\ub2e4. \uadf8\ub294 \ud558\ub098\ub2d8 \uc758 \uc601\uc5ed\uc8fc\uad8c(sphere sovereignty) \uc0ac\uc0c1\uc744 \uac15\uc870\ud588\ub294\ub370, \uc601\uc5ed\uc8fc\uad8c\uc774\ub780 \ud558\ub098\ub2d8\uc774 \uc885\uad50\uc5d0\uc11c \ubfd0\ub9cc \uc544\ub2c8\ub77c \uc815\uce58, \uacbd\uc81c, \uad50\uc721, \ubb38\ud654 \ub4f1 \uc628 \uc6b0\uc8fc\uc758 \uc601\uc5ed\uc5d0\uc11c \ucc3d\uc870\uc8fc \ud558\ub098\ub2d8\uc73c\ub85c\uc11c \uc804\uc801\uc778 \uc8fc\uad8c\uc744 \uac00\uc9c0\uace0 \ud1b5\uce58\ud558\uc2e0\ub2e4\ub294 \uc0ac\uc0c1\uc774\ub2e4. \ubc15\ud0dc\ud604\uc5d0 \ub530\ub974\uba74 \uce74\uc774\ud37c\uc758 \uc601\uc5ed\uc8fc\uad8c \uac1c\ub150\uc5d0\uc11c \u2018\uc601\uc5ed\u2019\uc774\ub780 \uc77c\ucc28\uc801\uc73c\ub85c \ud558\ub098\ub2d8\uc758 \ub2e4\uc591\ud55c \ucc3d\uc870\uc138\uacc4\ub85c\uc11c \uad6d\uac00, \uad50\ud68c, \ubb38\ud654, \ud559\ubb38, \uc608\uc220 \ub4f1\uc758 \uc77c\uc0c1\uc801 \uc0ac\ud68c \uc0dd\ud65c\uc758 \ubc94\uc8fc\ub97c \uc758\ubbf8\ud558\uba70, \ub3d9\uc2dc\uc5d0 \u2018\uc601\uc5ed\u2019\uc740 \ub2e4\ub978 \uc138\uacc4\uad00\uc744 \uac00\uc9c4 \uc0ac\ub78c\ub4e4\uc744 \uc758\ubbf8\ud55c\ub2e4\uace0 \ud55c\ub2e4. \uadf8\ub7f0\ub370 \uc2e0\ud559\uc5d0\uc11c \uad6c\uc6d0\uc744 \uc5bb\ub294 \ud2b9\ubcc4\uc740\ucd1d\uc5d0 \ub300\ud55c \uac15\uc870\ub294 \ub9ce\uc9c0\ub9cc \uc77c\ubc18\uc740\ucd1d(common grace)\uc5d0 \ub300\ud558\uc5ec \uc18c\ud640\ud558\uac8c \ub2e4\ub8ec \uc810\uc744 \uc54c\uc558\ub358 \uce74\uc774\ud37c\ub294 \u300e\uc77c\ubc18 \uc740\ucd1d\u300f(De Gemeene Gratie)\uc774\ub77c\ub294 \uc81c\ubaa9\uc758 \ucc45\uc5d0\uc11c \uc778\ub958\uc758 \ubb38\ud654 \uc804\ubc18\uc5d0 \ub300\ud574 \uae4a\uace0\ub3c4 \ud3ed\ub113\uc740 \uc0ac\uc0c1\uc744 \uc804\uac1c\ud558\ub294\ub370, \ud558\ub098\ub2d8\uc758 \ub73b\uc740 \uc6b0\ub9ac\uac00 \uad6c\uc6d0\ubc1b\uace0 \ucc9c\uad6d\uc5d0 \uac00\ub294 \uac83\ub9cc\uc774 \uc544\ub2c8\ub77c, \uc628 \uc778\ub958\uc758 \ubb38\ud654\uac00 \uacc4\uc18d\uc801\uc73c\ub85c \ubc1c\uc804\ud558\uc5ec \uc774 \ubc1c\uc804\ub41c \ubb38\ud654\ub97c \ud1b5\ud558\uc5ec \ud558\ub098\ub2d8\uaed8 \uc601\uad11 \ub3cc\ub824\uc57c \ud55c\ub2e4\ub294 \uac83\uc774\ub2e4.", "paragraph_answer": "\uc601\uad6d\uacfc \ubbf8\uad6d\uc774 \uc778\ub958\ud559\uc73c\ub85c \ubc1c\uc804\ud55c \ubc18\uba74 \ubb38\ud654\uc640 \uad00\ub828\ud558\uc5ec \ud65c\ubc1c\ud558\uac8c \uc5f0\uad6c\ub418\uace0 \ubc1c\ud45c\ub41c \ub098\ub77c\ub294 \ud654\ub780\uc774\ub2e4. \ubb38\ud654\uc758 \uc804\ub3c4\uc0ac\ub85c \ub300\ud45c\uc801 \ub300\ubd80\ub294 \ud654\ub780\uc758 \uc544\ube0c\ub77c\ud568 \uce74\uc774\ud37c(Abraham Kuyper, 1837-1920)\uc774\ub2e4. \uadf8\ub294 \ud558\ub098\ub2d8 \uc758 \uc601\uc5ed\uc8fc\uad8c(sphere sovereignty) \uc0ac\uc0c1\uc744 \uac15\uc870\ud588\ub294\ub370, \uc601\uc5ed\uc8fc\uad8c\uc774\ub780 \ud558\ub098\ub2d8\uc774 \uc885\uad50\uc5d0\uc11c \ubfd0\ub9cc \uc544\ub2c8\ub77c \uc815\uce58, \uacbd\uc81c, \uad50\uc721, \ubb38\ud654 \ub4f1 \uc628 \uc6b0\uc8fc\uc758 \uc601\uc5ed\uc5d0\uc11c \ucc3d\uc870\uc8fc \ud558\ub098\ub2d8\uc73c\ub85c\uc11c \uc804\uc801\uc778 \uc8fc\uad8c\uc744 \uac00\uc9c0\uace0 \ud1b5\uce58\ud558\uc2e0\ub2e4\ub294 \uc0ac\uc0c1\uc774\ub2e4. \ubc15\ud0dc\ud604\uc5d0 \ub530\ub974\uba74 \uce74\uc774\ud37c\uc758 \uc601\uc5ed\uc8fc\uad8c \uac1c\ub150\uc5d0\uc11c \u2018\uc601\uc5ed\u2019\uc774\ub780 \uc77c\ucc28\uc801\uc73c\ub85c \ud558\ub098\ub2d8\uc758 \ub2e4\uc591\ud55c \ucc3d\uc870\uc138\uacc4\ub85c\uc11c \uad6d\uac00, \uad50\ud68c, \ubb38\ud654, \ud559\ubb38, \uc608\uc220 \ub4f1\uc758 \uc77c\uc0c1\uc801 \uc0ac\ud68c \uc0dd\ud65c\uc758 \ubc94\uc8fc\ub97c \uc758\ubbf8\ud558\uba70, \ub3d9\uc2dc\uc5d0 \u2018\uc601\uc5ed\u2019\uc740 \ub2e4\ub978 \uc138\uacc4\uad00\uc744 \uac00\uc9c4 \uc0ac\ub78c\ub4e4\uc744 \uc758\ubbf8\ud55c\ub2e4\uace0 \ud55c\ub2e4. \uadf8\ub7f0\ub370 \uc2e0\ud559\uc5d0\uc11c \uad6c\uc6d0\uc744 \uc5bb\ub294 \ud2b9\ubcc4\uc740\ucd1d\uc5d0 \ub300\ud55c \uac15\uc870\ub294 \ub9ce\uc9c0\ub9cc \uc77c\ubc18\uc740\ucd1d(common grace)\uc5d0 \ub300\ud558\uc5ec \uc18c\ud640\ud558\uac8c \ub2e4\ub8ec \uc810\uc744 \uc54c\uc558\ub358 \uce74\uc774\ud37c\ub294 \u300e\uc77c\ubc18 \uc740\ucd1d\u300f(De Gemeene Gratie)\uc774\ub77c\ub294 \uc81c\ubaa9\uc758 \ucc45\uc5d0\uc11c \uc778\ub958\uc758 \ubb38\ud654 \uc804\ubc18\uc5d0 \ub300\ud574 \uae4a\uace0\ub3c4 \ud3ed\ub113\uc740 \uc0ac\uc0c1\uc744 \uc804\uac1c\ud558\ub294\ub370, \ud558\ub098\ub2d8\uc758 \ub73b\uc740 \uc6b0\ub9ac\uac00 \uad6c\uc6d0\ubc1b\uace0 \ucc9c\uad6d\uc5d0 \uac00\ub294 \uac83\ub9cc\uc774 \uc544\ub2c8\ub77c, \uc628 \uc778\ub958\uc758 \ubb38\ud654\uac00 \uacc4\uc18d\uc801\uc73c\ub85c \ubc1c\uc804\ud558\uc5ec \uc774 \ubc1c\uc804\ub41c \ubb38\ud654\ub97c \ud1b5\ud558\uc5ec \ud558\ub098\ub2d8\uaed8 \uc601\uad11 \ub3cc\ub824\uc57c \ud55c\ub2e4\ub294 \uac83\uc774\ub2e4.", "sentence_answer": "\uadf8\ub294 \ud558\ub098\ub2d8 \uc758 \uc601\uc5ed\uc8fc\uad8c(sphere sovereignty) \uc0ac\uc0c1\uc744 \uac15\uc870\ud588\ub294\ub370, \uc601\uc5ed\uc8fc\uad8c\uc774\ub780 \ud558\ub098\ub2d8\uc774 \uc885\uad50\uc5d0\uc11c \ubfd0\ub9cc \uc544\ub2c8\ub77c \uc815\uce58, \uacbd\uc81c, \uad50\uc721, \ubb38\ud654 \ub4f1 \uc628 \uc6b0\uc8fc\uc758 \uc601\uc5ed\uc5d0\uc11c \ucc3d\uc870\uc8fc \ud558\ub098\ub2d8\uc73c\ub85c\uc11c \uc804\uc801\uc778 \uc8fc\uad8c\uc744 \uac00\uc9c0\uace0 \ud1b5\uce58\ud558\uc2e0\ub2e4\ub294 \uc0ac\uc0c1\uc774\ub2e4.", "paragraph_id": "6505013-0-2", "paragraph_question": "question: \uce74\uc774\ud37c\uac00 \ubc1c\uc804\ub41c \ubb38\ud654\ub97c \ud1b5\ud574 \uc601\uad11\uc744 \ub3cc\ub824\uc57c \ud55c\ub2e4\uace0 \ud55c \ub300\uc0c1\uc740?, context: \uc601\uad6d\uacfc \ubbf8\uad6d\uc774 \uc778\ub958\ud559\uc73c\ub85c \ubc1c\uc804\ud55c \ubc18\uba74 \ubb38\ud654\uc640 \uad00\ub828\ud558\uc5ec \ud65c\ubc1c\ud558\uac8c \uc5f0\uad6c\ub418\uace0 \ubc1c\ud45c\ub41c \ub098\ub77c\ub294 \ud654\ub780\uc774\ub2e4. \ubb38\ud654\uc758 \uc804\ub3c4\uc0ac\ub85c \ub300\ud45c\uc801 \ub300\ubd80\ub294 \ud654\ub780\uc758 \uc544\ube0c\ub77c\ud568 \uce74\uc774\ud37c(Abraham Kuyper, 1837-1920)\uc774\ub2e4. \uadf8\ub294 \ud558\ub098\ub2d8\uc758 \uc601\uc5ed\uc8fc\uad8c(sphere sovereignty) \uc0ac\uc0c1\uc744 \uac15\uc870\ud588\ub294\ub370, \uc601\uc5ed\uc8fc\uad8c\uc774\ub780 \ud558\ub098\ub2d8\uc774 \uc885\uad50\uc5d0\uc11c \ubfd0\ub9cc \uc544\ub2c8\ub77c \uc815\uce58, \uacbd\uc81c, \uad50\uc721, \ubb38\ud654 \ub4f1 \uc628 \uc6b0\uc8fc\uc758 \uc601\uc5ed\uc5d0\uc11c \ucc3d\uc870\uc8fc \ud558\ub098\ub2d8\uc73c\ub85c\uc11c \uc804\uc801\uc778 \uc8fc\uad8c\uc744 \uac00\uc9c0\uace0 \ud1b5\uce58\ud558\uc2e0\ub2e4\ub294 \uc0ac\uc0c1\uc774\ub2e4. \ubc15\ud0dc\ud604\uc5d0 \ub530\ub974\uba74 \uce74\uc774\ud37c\uc758 \uc601\uc5ed\uc8fc\uad8c \uac1c\ub150\uc5d0\uc11c \u2018\uc601\uc5ed\u2019\uc774\ub780 \uc77c\ucc28\uc801\uc73c\ub85c \ud558\ub098\ub2d8\uc758 \ub2e4\uc591\ud55c \ucc3d\uc870\uc138\uacc4\ub85c\uc11c \uad6d\uac00, \uad50\ud68c, \ubb38\ud654, \ud559\ubb38, \uc608\uc220 \ub4f1\uc758 \uc77c\uc0c1\uc801 \uc0ac\ud68c \uc0dd\ud65c\uc758 \ubc94\uc8fc\ub97c \uc758\ubbf8\ud558\uba70, \ub3d9\uc2dc\uc5d0 \u2018\uc601\uc5ed\u2019\uc740 \ub2e4\ub978 \uc138\uacc4\uad00\uc744 \uac00\uc9c4 \uc0ac\ub78c\ub4e4\uc744 \uc758\ubbf8\ud55c\ub2e4\uace0 \ud55c\ub2e4. \uadf8\ub7f0\ub370 \uc2e0\ud559\uc5d0\uc11c \uad6c\uc6d0\uc744 \uc5bb\ub294 \ud2b9\ubcc4\uc740\ucd1d\uc5d0 \ub300\ud55c \uac15\uc870\ub294 \ub9ce\uc9c0\ub9cc \uc77c\ubc18\uc740\ucd1d(common grace)\uc5d0 \ub300\ud558\uc5ec \uc18c\ud640\ud558\uac8c \ub2e4\ub8ec \uc810\uc744 \uc54c\uc558\ub358 \uce74\uc774\ud37c\ub294 \u300e\uc77c\ubc18 \uc740\ucd1d\u300f(De Gemeene Gratie)\uc774\ub77c\ub294 \uc81c\ubaa9\uc758 \ucc45\uc5d0\uc11c \uc778\ub958\uc758 \ubb38\ud654 \uc804\ubc18\uc5d0 \ub300\ud574 \uae4a\uace0\ub3c4 \ud3ed\ub113\uc740 \uc0ac\uc0c1\uc744 \uc804\uac1c\ud558\ub294\ub370, \ud558\ub098\ub2d8\uc758 \ub73b\uc740 \uc6b0\ub9ac\uac00 \uad6c\uc6d0\ubc1b\uace0 \ucc9c\uad6d\uc5d0 \uac00\ub294 \uac83\ub9cc\uc774 \uc544\ub2c8\ub77c, \uc628 \uc778\ub958\uc758 \ubb38\ud654\uac00 \uacc4\uc18d\uc801\uc73c\ub85c \ubc1c\uc804\ud558\uc5ec \uc774 \ubc1c\uc804\ub41c \ubb38\ud654\ub97c \ud1b5\ud558\uc5ec \ud558\ub098\ub2d8\uaed8 \uc601\uad11 \ub3cc\ub824\uc57c \ud55c\ub2e4\ub294 \uac83\uc774\ub2e4."} {"question": "2012\ub144 \ud558\uacc4 \uc62c\ub9bc\ud53d\uc758 \uac1c\ucd5c\uc9c0 \uc120\uc815\uacfc \uad00\ub828\ud55c \ub1cc\ubb3c \uc0ac\uac74\uc744 \ub2e4\ub8ec BBC\uc758 \ub2e4\ud050\uba58\ud130\ub9ac\ub294?", "paragraph": "BBC \ub2e4\ud050\uba58\ud130\ub9ac\uc778 '\ud30c\ub178\ub77c\ub9c8'\uc5d0\uc11c\ub294 '\ub9e4\uc218\ub41c \uc62c\ub9bc\ud53d'\uc774\ub780 \uc8fc\uc81c\ub85c 2004\ub144 8\uc6d4\uc5d0 \ubc29\uc1a1\uc744 \ub0b4\ubcf4\ub0b4\uae30\ub3c4 \ud588\ub2e4. \uc774\ub54c \uc774 \ud504\ub85c\uadf8\ub7a8\uc5d0\uc11c\ub294 2012\ub144 \ud558\uacc4 \uc62c\ub9bc\ud53d\uc758 \uac1c\ucd5c\uc9c0 \uc120\uc815\uacfc \uad00\ub828\ub41c \ub1cc\ubb3c\uc5d0 \ub300\ud574\uc11c \uc870\uc0ac\ud588\ub2e4. \uc774 \ub2e4\ud050\uba58\ud130\ub9ac\uc5d0\uc11c\ub294 \ud2b9\uc815 \ud6c4\ubcf4 \ub3c4\uc2dc\uac00 IOC \uc704\uc6d0\ub4e4\uc5d0\uac8c \ub1cc\ubb3c\uc218\uc218\ud558\ub294 \uac83\uc774 \uac00\ub2a5\ud588\ub2e4\uace0 \uc8fc\uc7a5\ud588\uc73c\uba70, \ud2b9\ud788 \ud30c\ub9ac \uc2dc\uc7a5\uc774\uc5c8\ub358 \ubca0\ub974\ud2b8\ub791 \ub4e4\ub77c\ub178\uc5d0(Bertrand Delano\u00eb)\ub294 \uc601\uad6d\uc758 \ucd1d\ub9ac\uc778 \ud1a0\ub2c8 \ube14\ub808\uc5b4\uc640 \ub7f0\ub358\uc62c\ub9bc\ud53d\uc720\uce58\uc704\uc6d0\ud68c\uac00 \uc785\ud6c4\ubcf4 \uaddc\uc815\uc744 \uc704\ubc18\ud588\ub2e4\uace0 \ube44\ub09c\ud588\ub2e4. \uadf8\ub294 \ub2f9\uc2dc \ud504\ub791\uc2a4 \ub300\ud1b5\ub839\uc774\uc5c8\ub358 \uc790\ud06c \uc2dc\ub77c\ud06c\ub97c \ubaa9\uaca9\uc790\ub85c \ub0b4\uc138\uc6e0\uc9c0\ub9cc \uc2dc\ub77c\ud06c \ub300\ud1b5\ub839\uc740 \uc774 \ubd84\uc7c1\uc5d0 \ud718\ub9d0\ub824\ub4dc\ub294 \uac83\uc744 \uc8fc\uc758\ud588\uc73c\uba70 \uc778\ud130\ubdf0\ub97c \uc0bc\uac14\ub2e4. \uacb0\uad6d \ubca0\ub974\ud2b8\ub791 \ub4e4\ub77c\ub178\uc5d0\uc758 \uc8fc\uc7a5\uc5d0 \ub300\ud55c \uc870\uc0ac\ub294 \uccb4\uacc4\uc801\uc73c\ub85c \uc774\ub8e8\uc5b4\uc9c0\uc9c0\ub294 \uc54a\uc558\ub2e4. 2006\ub144 \ub3d9\uacc4 \uc62c\ub9bc\ud53d\uc744 \uc720\uce58\ud588\ub358 \ud1a0\ub9ac\ub178\ub3c4 \uc774 \ub17c\uc7c1\uc5d0\uc11c \ube60\uc838\ub098\uac08 \uc218 \uc5c6\uc5c8\ub2e4. \uc774\ubc88\uc5d0\ub294 \uc2a4\uc704\uc2a4 \uad6d\uc801\uc758 IOC\uc704\uc6d0 \ub9c8\ud06c \ud638\ub4e4\ub7ec(Marc Hodler)\uac00 \uc774 \ub17c\uc7c1\uc758 \uc911\uc2ec\uc774 \ub418\uc5c8\ub294\ub370, \uc774 \uc704\uc6d0\uc740 \uc2a4\uc704\uc2a4 \uc2dc\uc628\uc758 \uacbd\uc7c1 \ub3c4\uc2dc\uc600\ub358 \ud1a0\ub9ac\ub178\uac00 IOC\uc704\uc6d0\ub4e4\uc5d0\uac8c \ub1cc\ubb3c\uc218\uc218\ub97c \ud588\ub2e4\uace0 \ub9d0\ud588\uace0, \uc774 \ubc1c\uc5b8\uc73c\ub85c \uad11\ubc94\uc704\ud55c \uc870\uc0ac\uac00 \uc774\ub8e8\uc5b4\uc84c\ub2e4. \uc774 \uc5b8\ud589\uc774 \ub9ce\uc740 IOC\uc704\uc6d0\ub4e4\uc774 \uc2dc\uc628\uc5d0 \ub300\ud574 \uc5b8\uc9e2\uac8c \uc0dd\uac01\ud558\uac8c \ub418\uace0 \ud1a0\ub9ac\ub178\uac00 \uac1c\ucd5c\uc9c0\ub85c \uc120\uc815\ub418\ub3c4\ub85d \ub3c4\uc640\uc8fc\ub294 \uc5ed\ud560\uc744 \ud588\uc744 \uac00\ub2a5\uc131\ub3c4 \uc81c\uae30\ub418\uace0 \uc788\ub2e4.", "answer": "\ud30c\ub178\ub77c\ub9c8", "sentence": "BBC \ub2e4\ud050\uba58\ud130\ub9ac\uc778 ' \ud30c\ub178\ub77c\ub9c8 '\uc5d0\uc11c\ub294 '\ub9e4\uc218\ub41c \uc62c\ub9bc\ud53d'\uc774\ub780 \uc8fc\uc81c\ub85c 2004\ub144 8\uc6d4\uc5d0 \ubc29\uc1a1\uc744 \ub0b4\ubcf4\ub0b4\uae30\ub3c4 \ud588\ub2e4.", "paragraph_sentence": " BBC \ub2e4\ud050\uba58\ud130\ub9ac\uc778 ' \ud30c\ub178\ub77c\ub9c8 '\uc5d0\uc11c\ub294 '\ub9e4\uc218\ub41c \uc62c\ub9bc\ud53d'\uc774\ub780 \uc8fc\uc81c\ub85c 2004\ub144 8\uc6d4\uc5d0 \ubc29\uc1a1\uc744 \ub0b4\ubcf4\ub0b4\uae30\ub3c4 \ud588\ub2e4. \uc774\ub54c \uc774 \ud504\ub85c\uadf8\ub7a8\uc5d0\uc11c\ub294 2012\ub144 \ud558\uacc4 \uc62c\ub9bc\ud53d\uc758 \uac1c\ucd5c\uc9c0 \uc120\uc815\uacfc \uad00\ub828\ub41c \ub1cc\ubb3c\uc5d0 \ub300\ud574\uc11c \uc870\uc0ac\ud588\ub2e4. \uc774 \ub2e4\ud050\uba58\ud130\ub9ac\uc5d0\uc11c\ub294 \ud2b9\uc815 \ud6c4\ubcf4 \ub3c4\uc2dc\uac00 IOC \uc704\uc6d0\ub4e4\uc5d0\uac8c \ub1cc\ubb3c\uc218\uc218\ud558\ub294 \uac83\uc774 \uac00\ub2a5\ud588\ub2e4\uace0 \uc8fc\uc7a5\ud588\uc73c\uba70, \ud2b9\ud788 \ud30c\ub9ac \uc2dc\uc7a5\uc774\uc5c8\ub358 \ubca0\ub974\ud2b8\ub791 \ub4e4\ub77c\ub178\uc5d0(Bertrand Delano\u00eb)\ub294 \uc601\uad6d\uc758 \ucd1d\ub9ac\uc778 \ud1a0\ub2c8 \ube14\ub808\uc5b4\uc640 \ub7f0\ub358\uc62c\ub9bc\ud53d\uc720\uce58\uc704\uc6d0\ud68c\uac00 \uc785\ud6c4\ubcf4 \uaddc\uc815\uc744 \uc704\ubc18\ud588\ub2e4\uace0 \ube44\ub09c\ud588\ub2e4. \uadf8\ub294 \ub2f9\uc2dc \ud504\ub791\uc2a4 \ub300\ud1b5\ub839\uc774\uc5c8\ub358 \uc790\ud06c \uc2dc\ub77c\ud06c\ub97c \ubaa9\uaca9\uc790\ub85c \ub0b4\uc138\uc6e0\uc9c0\ub9cc \uc2dc\ub77c\ud06c \ub300\ud1b5\ub839\uc740 \uc774 \ubd84\uc7c1\uc5d0 \ud718\ub9d0\ub824\ub4dc\ub294 \uac83\uc744 \uc8fc\uc758\ud588\uc73c\uba70 \uc778\ud130\ubdf0\ub97c \uc0bc\uac14\ub2e4. \uacb0\uad6d \ubca0\ub974\ud2b8\ub791 \ub4e4\ub77c\ub178\uc5d0\uc758 \uc8fc\uc7a5\uc5d0 \ub300\ud55c \uc870\uc0ac\ub294 \uccb4\uacc4\uc801\uc73c\ub85c \uc774\ub8e8\uc5b4\uc9c0\uc9c0\ub294 \uc54a\uc558\ub2e4. 2006\ub144 \ub3d9\uacc4 \uc62c\ub9bc\ud53d\uc744 \uc720\uce58\ud588\ub358 \ud1a0\ub9ac\ub178\ub3c4 \uc774 \ub17c\uc7c1\uc5d0\uc11c \ube60\uc838\ub098\uac08 \uc218 \uc5c6\uc5c8\ub2e4. \uc774\ubc88\uc5d0\ub294 \uc2a4\uc704\uc2a4 \uad6d\uc801\uc758 IOC\uc704\uc6d0 \ub9c8\ud06c \ud638\ub4e4\ub7ec(Marc Hodler)\uac00 \uc774 \ub17c\uc7c1\uc758 \uc911\uc2ec\uc774 \ub418\uc5c8\ub294\ub370, \uc774 \uc704\uc6d0\uc740 \uc2a4\uc704\uc2a4 \uc2dc\uc628\uc758 \uacbd\uc7c1 \ub3c4\uc2dc\uc600\ub358 \ud1a0\ub9ac\ub178\uac00 IOC\uc704\uc6d0\ub4e4\uc5d0\uac8c \ub1cc\ubb3c\uc218\uc218\ub97c \ud588\ub2e4\uace0 \ub9d0\ud588\uace0, \uc774 \ubc1c\uc5b8\uc73c\ub85c \uad11\ubc94\uc704\ud55c \uc870\uc0ac\uac00 \uc774\ub8e8\uc5b4\uc84c\ub2e4. \uc774 \uc5b8\ud589\uc774 \ub9ce\uc740 IOC\uc704\uc6d0\ub4e4\uc774 \uc2dc\uc628\uc5d0 \ub300\ud574 \uc5b8\uc9e2\uac8c \uc0dd\uac01\ud558\uac8c \ub418\uace0 \ud1a0\ub9ac\ub178\uac00 \uac1c\ucd5c\uc9c0\ub85c \uc120\uc815\ub418\ub3c4\ub85d \ub3c4\uc640\uc8fc\ub294 \uc5ed\ud560\uc744 \ud588\uc744 \uac00\ub2a5\uc131\ub3c4 \uc81c\uae30\ub418\uace0 \uc788\ub2e4.", "paragraph_answer": "BBC \ub2e4\ud050\uba58\ud130\ub9ac\uc778 ' \ud30c\ub178\ub77c\ub9c8 '\uc5d0\uc11c\ub294 '\ub9e4\uc218\ub41c \uc62c\ub9bc\ud53d'\uc774\ub780 \uc8fc\uc81c\ub85c 2004\ub144 8\uc6d4\uc5d0 \ubc29\uc1a1\uc744 \ub0b4\ubcf4\ub0b4\uae30\ub3c4 \ud588\ub2e4. \uc774\ub54c \uc774 \ud504\ub85c\uadf8\ub7a8\uc5d0\uc11c\ub294 2012\ub144 \ud558\uacc4 \uc62c\ub9bc\ud53d\uc758 \uac1c\ucd5c\uc9c0 \uc120\uc815\uacfc \uad00\ub828\ub41c \ub1cc\ubb3c\uc5d0 \ub300\ud574\uc11c \uc870\uc0ac\ud588\ub2e4. \uc774 \ub2e4\ud050\uba58\ud130\ub9ac\uc5d0\uc11c\ub294 \ud2b9\uc815 \ud6c4\ubcf4 \ub3c4\uc2dc\uac00 IOC \uc704\uc6d0\ub4e4\uc5d0\uac8c \ub1cc\ubb3c\uc218\uc218\ud558\ub294 \uac83\uc774 \uac00\ub2a5\ud588\ub2e4\uace0 \uc8fc\uc7a5\ud588\uc73c\uba70, \ud2b9\ud788 \ud30c\ub9ac \uc2dc\uc7a5\uc774\uc5c8\ub358 \ubca0\ub974\ud2b8\ub791 \ub4e4\ub77c\ub178\uc5d0(Bertrand Delano\u00eb)\ub294 \uc601\uad6d\uc758 \ucd1d\ub9ac\uc778 \ud1a0\ub2c8 \ube14\ub808\uc5b4\uc640 \ub7f0\ub358\uc62c\ub9bc\ud53d\uc720\uce58\uc704\uc6d0\ud68c\uac00 \uc785\ud6c4\ubcf4 \uaddc\uc815\uc744 \uc704\ubc18\ud588\ub2e4\uace0 \ube44\ub09c\ud588\ub2e4. \uadf8\ub294 \ub2f9\uc2dc \ud504\ub791\uc2a4 \ub300\ud1b5\ub839\uc774\uc5c8\ub358 \uc790\ud06c \uc2dc\ub77c\ud06c\ub97c \ubaa9\uaca9\uc790\ub85c \ub0b4\uc138\uc6e0\uc9c0\ub9cc \uc2dc\ub77c\ud06c \ub300\ud1b5\ub839\uc740 \uc774 \ubd84\uc7c1\uc5d0 \ud718\ub9d0\ub824\ub4dc\ub294 \uac83\uc744 \uc8fc\uc758\ud588\uc73c\uba70 \uc778\ud130\ubdf0\ub97c \uc0bc\uac14\ub2e4. \uacb0\uad6d \ubca0\ub974\ud2b8\ub791 \ub4e4\ub77c\ub178\uc5d0\uc758 \uc8fc\uc7a5\uc5d0 \ub300\ud55c \uc870\uc0ac\ub294 \uccb4\uacc4\uc801\uc73c\ub85c \uc774\ub8e8\uc5b4\uc9c0\uc9c0\ub294 \uc54a\uc558\ub2e4. 2006\ub144 \ub3d9\uacc4 \uc62c\ub9bc\ud53d\uc744 \uc720\uce58\ud588\ub358 \ud1a0\ub9ac\ub178\ub3c4 \uc774 \ub17c\uc7c1\uc5d0\uc11c \ube60\uc838\ub098\uac08 \uc218 \uc5c6\uc5c8\ub2e4. \uc774\ubc88\uc5d0\ub294 \uc2a4\uc704\uc2a4 \uad6d\uc801\uc758 IOC\uc704\uc6d0 \ub9c8\ud06c \ud638\ub4e4\ub7ec(Marc Hodler)\uac00 \uc774 \ub17c\uc7c1\uc758 \uc911\uc2ec\uc774 \ub418\uc5c8\ub294\ub370, \uc774 \uc704\uc6d0\uc740 \uc2a4\uc704\uc2a4 \uc2dc\uc628\uc758 \uacbd\uc7c1 \ub3c4\uc2dc\uc600\ub358 \ud1a0\ub9ac\ub178\uac00 IOC\uc704\uc6d0\ub4e4\uc5d0\uac8c \ub1cc\ubb3c\uc218\uc218\ub97c \ud588\ub2e4\uace0 \ub9d0\ud588\uace0, \uc774 \ubc1c\uc5b8\uc73c\ub85c \uad11\ubc94\uc704\ud55c \uc870\uc0ac\uac00 \uc774\ub8e8\uc5b4\uc84c\ub2e4. \uc774 \uc5b8\ud589\uc774 \ub9ce\uc740 IOC\uc704\uc6d0\ub4e4\uc774 \uc2dc\uc628\uc5d0 \ub300\ud574 \uc5b8\uc9e2\uac8c \uc0dd\uac01\ud558\uac8c \ub418\uace0 \ud1a0\ub9ac\ub178\uac00 \uac1c\ucd5c\uc9c0\ub85c \uc120\uc815\ub418\ub3c4\ub85d \ub3c4\uc640\uc8fc\ub294 \uc5ed\ud560\uc744 \ud588\uc744 \uac00\ub2a5\uc131\ub3c4 \uc81c\uae30\ub418\uace0 \uc788\ub2e4.", "sentence_answer": "BBC \ub2e4\ud050\uba58\ud130\ub9ac\uc778 ' \ud30c\ub178\ub77c\ub9c8 '\uc5d0\uc11c\ub294 '\ub9e4\uc218\ub41c \uc62c\ub9bc\ud53d'\uc774\ub780 \uc8fc\uc81c\ub85c 2004\ub144 8\uc6d4\uc5d0 \ubc29\uc1a1\uc744 \ub0b4\ubcf4\ub0b4\uae30\ub3c4 \ud588\ub2e4.", "paragraph_id": "6588341-12-0", "paragraph_question": "question: 2012\ub144 \ud558\uacc4 \uc62c\ub9bc\ud53d\uc758 \uac1c\ucd5c\uc9c0 \uc120\uc815\uacfc \uad00\ub828\ud55c \ub1cc\ubb3c \uc0ac\uac74\uc744 \ub2e4\ub8ec BBC\uc758 \ub2e4\ud050\uba58\ud130\ub9ac\ub294?, context: BBC \ub2e4\ud050\uba58\ud130\ub9ac\uc778 '\ud30c\ub178\ub77c\ub9c8'\uc5d0\uc11c\ub294 '\ub9e4\uc218\ub41c \uc62c\ub9bc\ud53d'\uc774\ub780 \uc8fc\uc81c\ub85c 2004\ub144 8\uc6d4\uc5d0 \ubc29\uc1a1\uc744 \ub0b4\ubcf4\ub0b4\uae30\ub3c4 \ud588\ub2e4. \uc774\ub54c \uc774 \ud504\ub85c\uadf8\ub7a8\uc5d0\uc11c\ub294 2012\ub144 \ud558\uacc4 \uc62c\ub9bc\ud53d\uc758 \uac1c\ucd5c\uc9c0 \uc120\uc815\uacfc \uad00\ub828\ub41c \ub1cc\ubb3c\uc5d0 \ub300\ud574\uc11c \uc870\uc0ac\ud588\ub2e4. \uc774 \ub2e4\ud050\uba58\ud130\ub9ac\uc5d0\uc11c\ub294 \ud2b9\uc815 \ud6c4\ubcf4 \ub3c4\uc2dc\uac00 IOC \uc704\uc6d0\ub4e4\uc5d0\uac8c \ub1cc\ubb3c\uc218\uc218\ud558\ub294 \uac83\uc774 \uac00\ub2a5\ud588\ub2e4\uace0 \uc8fc\uc7a5\ud588\uc73c\uba70, \ud2b9\ud788 \ud30c\ub9ac \uc2dc\uc7a5\uc774\uc5c8\ub358 \ubca0\ub974\ud2b8\ub791 \ub4e4\ub77c\ub178\uc5d0(Bertrand Delano\u00eb)\ub294 \uc601\uad6d\uc758 \ucd1d\ub9ac\uc778 \ud1a0\ub2c8 \ube14\ub808\uc5b4\uc640 \ub7f0\ub358\uc62c\ub9bc\ud53d\uc720\uce58\uc704\uc6d0\ud68c\uac00 \uc785\ud6c4\ubcf4 \uaddc\uc815\uc744 \uc704\ubc18\ud588\ub2e4\uace0 \ube44\ub09c\ud588\ub2e4. \uadf8\ub294 \ub2f9\uc2dc \ud504\ub791\uc2a4 \ub300\ud1b5\ub839\uc774\uc5c8\ub358 \uc790\ud06c \uc2dc\ub77c\ud06c\ub97c \ubaa9\uaca9\uc790\ub85c \ub0b4\uc138\uc6e0\uc9c0\ub9cc \uc2dc\ub77c\ud06c \ub300\ud1b5\ub839\uc740 \uc774 \ubd84\uc7c1\uc5d0 \ud718\ub9d0\ub824\ub4dc\ub294 \uac83\uc744 \uc8fc\uc758\ud588\uc73c\uba70 \uc778\ud130\ubdf0\ub97c \uc0bc\uac14\ub2e4. \uacb0\uad6d \ubca0\ub974\ud2b8\ub791 \ub4e4\ub77c\ub178\uc5d0\uc758 \uc8fc\uc7a5\uc5d0 \ub300\ud55c \uc870\uc0ac\ub294 \uccb4\uacc4\uc801\uc73c\ub85c \uc774\ub8e8\uc5b4\uc9c0\uc9c0\ub294 \uc54a\uc558\ub2e4. 2006\ub144 \ub3d9\uacc4 \uc62c\ub9bc\ud53d\uc744 \uc720\uce58\ud588\ub358 \ud1a0\ub9ac\ub178\ub3c4 \uc774 \ub17c\uc7c1\uc5d0\uc11c \ube60\uc838\ub098\uac08 \uc218 \uc5c6\uc5c8\ub2e4. \uc774\ubc88\uc5d0\ub294 \uc2a4\uc704\uc2a4 \uad6d\uc801\uc758 IOC\uc704\uc6d0 \ub9c8\ud06c \ud638\ub4e4\ub7ec(Marc Hodler)\uac00 \uc774 \ub17c\uc7c1\uc758 \uc911\uc2ec\uc774 \ub418\uc5c8\ub294\ub370, \uc774 \uc704\uc6d0\uc740 \uc2a4\uc704\uc2a4 \uc2dc\uc628\uc758 \uacbd\uc7c1 \ub3c4\uc2dc\uc600\ub358 \ud1a0\ub9ac\ub178\uac00 IOC\uc704\uc6d0\ub4e4\uc5d0\uac8c \ub1cc\ubb3c\uc218\uc218\ub97c \ud588\ub2e4\uace0 \ub9d0\ud588\uace0, \uc774 \ubc1c\uc5b8\uc73c\ub85c \uad11\ubc94\uc704\ud55c \uc870\uc0ac\uac00 \uc774\ub8e8\uc5b4\uc84c\ub2e4. \uc774 \uc5b8\ud589\uc774 \ub9ce\uc740 IOC\uc704\uc6d0\ub4e4\uc774 \uc2dc\uc628\uc5d0 \ub300\ud574 \uc5b8\uc9e2\uac8c \uc0dd\uac01\ud558\uac8c \ub418\uace0 \ud1a0\ub9ac\ub178\uac00 \uac1c\ucd5c\uc9c0\ub85c \uc120\uc815\ub418\ub3c4\ub85d \ub3c4\uc640\uc8fc\ub294 \uc5ed\ud560\uc744 \ud588\uc744 \uac00\ub2a5\uc131\ub3c4 \uc81c\uae30\ub418\uace0 \uc788\ub2e4."} {"question": "\ubbf8\uad6d\uc774 \ubb3c\uc790\uc640 \uc678\ud654\ubd80\uc871 \ubb38\uc81c\ub97c \ubb34\uc0c1\uc73c\ub85c \uc9c0\uc6d0\ud574 \uc900 \ub9c8\uc9c0\ub9c9 \ud574\ub294?", "paragraph": "\uc911\uad6d\uc774 \ub7ec\uc2dc\uc544\uc81c \uc0ac\ub4dc 1\ud3ec\ub300\ub97c 30\uc5b5 \ub2ec\ub7ec(\uc57d 3\uc8703\ucc9c\uc5b5\uc6d0)\uc5d0 \uc218\uc785\ud558\uae30\ub85c \ud55c \uac83\uc5d0 \ube44\ud558\uba74 \ubbf8\uad6d\uc81c \uc0ac\ub4dc 10\uc5b5 \ub2ec\ub7ec(\uc57d 1\uc8701\ucc9c\uc5b5\uc6d0)\ub294 \ub7ec\uc2dc\uc544\uc81c \uc0ac\ub4dc\uc758 3\ubd84\uc758 1\ubc16\uc5d0 \uc548\ub418\ub294 \uac00\uaca9\uc774\ub2e4. \ubbf8\uad6d\uc740 \ud55c\uad6d\uc774 \uc77c\ubcf8\uc73c\ub85c\ubd80\ud130 \ud574\ubc29\ud55c \uc9c1\ud6c4\ubd80\ud130 1970\ub144 5\uc6d4\uae4c\uc9c0 20\ub144 \ub3d9\uc548 \ubb3c\uc790\uc640 \uc678\ud654\ubd80\uc871 \ubb38\uc81c\ub97c \ubb34\uc0c1\uc73c\ub85c \uc9c0\uc6d0\ud574\uc8fc\uc5c8\uc73c\uba70, \ud2b9\ud788 1950\ub300\ub9d0\uae4c\uc9c0\ub294 \uc720\uc77c\ud55c \uc678\uc790\ub3c4\uc785 \ucc3d\uad6c\ub85c \ud55c\uad6d \uacbd\uc81c\ubd80\ud765\uc5d0 \ubbf8\uad6d\uc758 \ud55c\uad6d \uc6d0\uc870 \uc815\ucc45\uc774 \uac00\uc7a5 \ud070 \uae30\uc5ec\ub97c \ud574\uc8fc\uc5c8\ub2e4. \ubbf8\uad6d\uc758 \ud55c\uad6d \ubb34\uc0c1\uc6d0\uc870\ub294 \uc57d 44\uc5b5 \ub2ec\ub7ec, \uc720\uc0c1\uc6d0\uc870\ub294 \uc57d 4\uc5b5 \ub2ec\ub7ec\uc5d0 \ub2ec\ud558\uc5ec \ud55c\uad6d\uacbd\uc81c\uc758 \ud22c\uc790\uc790\uc6d0 \ub9c8\ub828, \uad6d\uc81c\uc218\uc9c0 \uc801\uc790\ubcf4\uc804 \ubc0f \uacbd\uc81c\uc131\uc7a5\uc5d0\uc11c \ub9e4\uc6b0 \uc911\uc694\ud55c \uc5ed\ud560\uc744 \ud588\uace0 \ubbf8\uad6d\uc758 \uc6d0\uc870\ub294 \uadf8 \ub2f9\uc2dc \uac00\ub09c\ud588\ub358 \ud55c\uad6d \uc815\ubd80 \uc608\uc0b0\uc5d0 \ud070 \ub3c4\uc6c0\uc774 \ub418\uc5c8\ub2e4. 1950~70\ub144 \ud55c\uad6d\uc758 \uacbd\uc81c\ub294 \ubbf8\uad6d \uc6d0\uc870 \uacbd\uc81c \uccb4\uc81c\ub85c, \ubbf8\uad6d\uc73c\ub85c\ubd80\ud130 \ub18d\uc0b0\ubb3c, \uc18c\ube44\uc7ac, \uac74\ucd95\uc790\uc7ac, \uc678\ud654 \ub4f1\uc758 \ubb34\uc0c1 \uc9c0\uc6d0\uc744 \ubc1b\uc73c\uba74\uc11c \uc2dd\ub7c9 \ubb38\uc81c\ub97c \ud574\uacb0\ud558\uc600\uace0 1961\ub144 \uae30\uc900\uc73c\ub85c \ud55c\uad6d\uc758 GDP(\uad6d\ub0b4\ucd1d\uc0dd\uc0b0)\uac00 23\uc5b5 \ub2ec\ub7ec \uc218\uc900\uc774\uc5c8\ub2e4\ub294 \uac83\uc744 \uac10\uc548\ud558\uba74 \ubbf8\uad6d\uc73c\ub85c\ubd80\ud130 \ubc1b\uc740 \ubb34\uc0c1\uc6d0\uc870 44\uc5b5 \ub2ec\ub7ec\ub294 \uadf8\ub2f9\uc2dc \uc6b0\ub9ac\ub098\ub77c GDP\uc758 2\ubc30\ub098 \ub41c\ub2e4.", "answer": "1970\ub144", "sentence": "\ubbf8\uad6d\uc740 \ud55c\uad6d\uc774 \uc77c\ubcf8\uc73c\ub85c\ubd80\ud130 \ud574\ubc29\ud55c \uc9c1\ud6c4\ubd80\ud130 1970\ub144 5\uc6d4\uae4c\uc9c0 20\ub144 \ub3d9\uc548 \ubb3c\uc790\uc640 \uc678\ud654\ubd80\uc871 \ubb38\uc81c\ub97c \ubb34\uc0c1\uc73c\ub85c \uc9c0\uc6d0\ud574\uc8fc\uc5c8\uc73c\uba70, \ud2b9\ud788 1950\ub300\ub9d0\uae4c\uc9c0\ub294 \uc720\uc77c\ud55c \uc678\uc790\ub3c4\uc785 \ucc3d\uad6c\ub85c \ud55c\uad6d \uacbd\uc81c\ubd80\ud765\uc5d0 \ubbf8\uad6d\uc758 \ud55c\uad6d \uc6d0\uc870 \uc815\ucc45\uc774 \uac00\uc7a5 \ud070 \uae30\uc5ec\ub97c \ud574\uc8fc\uc5c8\ub2e4.", "paragraph_sentence": "\uc911\uad6d\uc774 \ub7ec\uc2dc\uc544\uc81c \uc0ac\ub4dc 1\ud3ec\ub300\ub97c 30\uc5b5 \ub2ec\ub7ec(\uc57d 3\uc8703\ucc9c\uc5b5\uc6d0)\uc5d0 \uc218\uc785\ud558\uae30\ub85c \ud55c \uac83\uc5d0 \ube44\ud558\uba74 \ubbf8\uad6d\uc81c \uc0ac\ub4dc 10\uc5b5 \ub2ec\ub7ec(\uc57d 1\uc8701\ucc9c\uc5b5\uc6d0)\ub294 \ub7ec\uc2dc\uc544\uc81c \uc0ac\ub4dc\uc758 3\ubd84\uc758 1\ubc16\uc5d0 \uc548\ub418\ub294 \uac00\uaca9\uc774\ub2e4. \ubbf8\uad6d\uc740 \ud55c\uad6d\uc774 \uc77c\ubcf8\uc73c\ub85c\ubd80\ud130 \ud574\ubc29\ud55c \uc9c1\ud6c4\ubd80\ud130 1970\ub144 5\uc6d4\uae4c\uc9c0 20\ub144 \ub3d9\uc548 \ubb3c\uc790\uc640 \uc678\ud654\ubd80\uc871 \ubb38\uc81c\ub97c \ubb34\uc0c1\uc73c\ub85c \uc9c0\uc6d0\ud574\uc8fc\uc5c8\uc73c\uba70, \ud2b9\ud788 1950\ub300\ub9d0\uae4c\uc9c0\ub294 \uc720\uc77c\ud55c \uc678\uc790\ub3c4\uc785 \ucc3d\uad6c\ub85c \ud55c\uad6d \uacbd\uc81c\ubd80\ud765\uc5d0 \ubbf8\uad6d\uc758 \ud55c\uad6d \uc6d0\uc870 \uc815\ucc45\uc774 \uac00\uc7a5 \ud070 \uae30\uc5ec\ub97c \ud574\uc8fc\uc5c8\ub2e4. \ubbf8\uad6d\uc758 \ud55c\uad6d \ubb34\uc0c1\uc6d0\uc870\ub294 \uc57d 44\uc5b5 \ub2ec\ub7ec, \uc720\uc0c1\uc6d0\uc870\ub294 \uc57d 4\uc5b5 \ub2ec\ub7ec\uc5d0 \ub2ec\ud558\uc5ec \ud55c\uad6d\uacbd\uc81c\uc758 \ud22c\uc790\uc790\uc6d0 \ub9c8\ub828, \uad6d\uc81c\uc218\uc9c0 \uc801\uc790\ubcf4\uc804 \ubc0f \uacbd\uc81c\uc131\uc7a5\uc5d0\uc11c \ub9e4\uc6b0 \uc911\uc694\ud55c \uc5ed\ud560\uc744 \ud588\uace0 \ubbf8\uad6d\uc758 \uc6d0\uc870\ub294 \uadf8 \ub2f9\uc2dc \uac00\ub09c\ud588\ub358 \ud55c\uad6d \uc815\ubd80 \uc608\uc0b0\uc5d0 \ud070 \ub3c4\uc6c0\uc774 \ub418\uc5c8\ub2e4. 1950~70\ub144 \ud55c\uad6d\uc758 \uacbd\uc81c\ub294 \ubbf8\uad6d \uc6d0\uc870 \uacbd\uc81c \uccb4\uc81c\ub85c, \ubbf8\uad6d\uc73c\ub85c\ubd80\ud130 \ub18d\uc0b0\ubb3c, \uc18c\ube44\uc7ac, \uac74\ucd95\uc790\uc7ac, \uc678\ud654 \ub4f1\uc758 \ubb34\uc0c1 \uc9c0\uc6d0\uc744 \ubc1b\uc73c\uba74\uc11c \uc2dd\ub7c9 \ubb38\uc81c\ub97c \ud574\uacb0\ud558\uc600\uace0 1961\ub144 \uae30\uc900\uc73c\ub85c \ud55c\uad6d\uc758 GDP(\uad6d\ub0b4\ucd1d\uc0dd\uc0b0)\uac00 23\uc5b5 \ub2ec\ub7ec \uc218\uc900\uc774\uc5c8\ub2e4\ub294 \uac83\uc744 \uac10\uc548\ud558\uba74 \ubbf8\uad6d\uc73c\ub85c\ubd80\ud130 \ubc1b\uc740 \ubb34\uc0c1\uc6d0\uc870 44\uc5b5 \ub2ec\ub7ec\ub294 \uadf8\ub2f9\uc2dc \uc6b0\ub9ac\ub098\ub77c GDP\uc758 2\ubc30\ub098 \ub41c\ub2e4.", "paragraph_answer": "\uc911\uad6d\uc774 \ub7ec\uc2dc\uc544\uc81c \uc0ac\ub4dc 1\ud3ec\ub300\ub97c 30\uc5b5 \ub2ec\ub7ec(\uc57d 3\uc8703\ucc9c\uc5b5\uc6d0)\uc5d0 \uc218\uc785\ud558\uae30\ub85c \ud55c \uac83\uc5d0 \ube44\ud558\uba74 \ubbf8\uad6d\uc81c \uc0ac\ub4dc 10\uc5b5 \ub2ec\ub7ec(\uc57d 1\uc8701\ucc9c\uc5b5\uc6d0)\ub294 \ub7ec\uc2dc\uc544\uc81c \uc0ac\ub4dc\uc758 3\ubd84\uc758 1\ubc16\uc5d0 \uc548\ub418\ub294 \uac00\uaca9\uc774\ub2e4. \ubbf8\uad6d\uc740 \ud55c\uad6d\uc774 \uc77c\ubcf8\uc73c\ub85c\ubd80\ud130 \ud574\ubc29\ud55c \uc9c1\ud6c4\ubd80\ud130 1970\ub144 5\uc6d4\uae4c\uc9c0 20\ub144 \ub3d9\uc548 \ubb3c\uc790\uc640 \uc678\ud654\ubd80\uc871 \ubb38\uc81c\ub97c \ubb34\uc0c1\uc73c\ub85c \uc9c0\uc6d0\ud574\uc8fc\uc5c8\uc73c\uba70, \ud2b9\ud788 1950\ub300\ub9d0\uae4c\uc9c0\ub294 \uc720\uc77c\ud55c \uc678\uc790\ub3c4\uc785 \ucc3d\uad6c\ub85c \ud55c\uad6d \uacbd\uc81c\ubd80\ud765\uc5d0 \ubbf8\uad6d\uc758 \ud55c\uad6d \uc6d0\uc870 \uc815\ucc45\uc774 \uac00\uc7a5 \ud070 \uae30\uc5ec\ub97c \ud574\uc8fc\uc5c8\ub2e4. \ubbf8\uad6d\uc758 \ud55c\uad6d \ubb34\uc0c1\uc6d0\uc870\ub294 \uc57d 44\uc5b5 \ub2ec\ub7ec, \uc720\uc0c1\uc6d0\uc870\ub294 \uc57d 4\uc5b5 \ub2ec\ub7ec\uc5d0 \ub2ec\ud558\uc5ec \ud55c\uad6d\uacbd\uc81c\uc758 \ud22c\uc790\uc790\uc6d0 \ub9c8\ub828, \uad6d\uc81c\uc218\uc9c0 \uc801\uc790\ubcf4\uc804 \ubc0f \uacbd\uc81c\uc131\uc7a5\uc5d0\uc11c \ub9e4\uc6b0 \uc911\uc694\ud55c \uc5ed\ud560\uc744 \ud588\uace0 \ubbf8\uad6d\uc758 \uc6d0\uc870\ub294 \uadf8 \ub2f9\uc2dc \uac00\ub09c\ud588\ub358 \ud55c\uad6d \uc815\ubd80 \uc608\uc0b0\uc5d0 \ud070 \ub3c4\uc6c0\uc774 \ub418\uc5c8\ub2e4. 1950~70\ub144 \ud55c\uad6d\uc758 \uacbd\uc81c\ub294 \ubbf8\uad6d \uc6d0\uc870 \uacbd\uc81c \uccb4\uc81c\ub85c, \ubbf8\uad6d\uc73c\ub85c\ubd80\ud130 \ub18d\uc0b0\ubb3c, \uc18c\ube44\uc7ac, \uac74\ucd95\uc790\uc7ac, \uc678\ud654 \ub4f1\uc758 \ubb34\uc0c1 \uc9c0\uc6d0\uc744 \ubc1b\uc73c\uba74\uc11c \uc2dd\ub7c9 \ubb38\uc81c\ub97c \ud574\uacb0\ud558\uc600\uace0 1961\ub144 \uae30\uc900\uc73c\ub85c \ud55c\uad6d\uc758 GDP(\uad6d\ub0b4\ucd1d\uc0dd\uc0b0)\uac00 23\uc5b5 \ub2ec\ub7ec \uc218\uc900\uc774\uc5c8\ub2e4\ub294 \uac83\uc744 \uac10\uc548\ud558\uba74 \ubbf8\uad6d\uc73c\ub85c\ubd80\ud130 \ubc1b\uc740 \ubb34\uc0c1\uc6d0\uc870 44\uc5b5 \ub2ec\ub7ec\ub294 \uadf8\ub2f9\uc2dc \uc6b0\ub9ac\ub098\ub77c GDP\uc758 2\ubc30\ub098 \ub41c\ub2e4.", "sentence_answer": "\ubbf8\uad6d\uc740 \ud55c\uad6d\uc774 \uc77c\ubcf8\uc73c\ub85c\ubd80\ud130 \ud574\ubc29\ud55c \uc9c1\ud6c4\ubd80\ud130 1970\ub144 5\uc6d4\uae4c\uc9c0 20\ub144 \ub3d9\uc548 \ubb3c\uc790\uc640 \uc678\ud654\ubd80\uc871 \ubb38\uc81c\ub97c \ubb34\uc0c1\uc73c\ub85c \uc9c0\uc6d0\ud574\uc8fc\uc5c8\uc73c\uba70, \ud2b9\ud788 1950\ub300\ub9d0\uae4c\uc9c0\ub294 \uc720\uc77c\ud55c \uc678\uc790\ub3c4\uc785 \ucc3d\uad6c\ub85c \ud55c\uad6d \uacbd\uc81c\ubd80\ud765\uc5d0 \ubbf8\uad6d\uc758 \ud55c\uad6d \uc6d0\uc870 \uc815\ucc45\uc774 \uac00\uc7a5 \ud070 \uae30\uc5ec\ub97c \ud574\uc8fc\uc5c8\ub2e4.", "paragraph_id": "6512357-18-1", "paragraph_question": "question: \ubbf8\uad6d\uc774 \ubb3c\uc790\uc640 \uc678\ud654\ubd80\uc871 \ubb38\uc81c\ub97c \ubb34\uc0c1\uc73c\ub85c \uc9c0\uc6d0\ud574 \uc900 \ub9c8\uc9c0\ub9c9 \ud574\ub294?, context: \uc911\uad6d\uc774 \ub7ec\uc2dc\uc544\uc81c \uc0ac\ub4dc 1\ud3ec\ub300\ub97c 30\uc5b5 \ub2ec\ub7ec(\uc57d 3\uc8703\ucc9c\uc5b5\uc6d0)\uc5d0 \uc218\uc785\ud558\uae30\ub85c \ud55c \uac83\uc5d0 \ube44\ud558\uba74 \ubbf8\uad6d\uc81c \uc0ac\ub4dc 10\uc5b5 \ub2ec\ub7ec(\uc57d 1\uc8701\ucc9c\uc5b5\uc6d0)\ub294 \ub7ec\uc2dc\uc544\uc81c \uc0ac\ub4dc\uc758 3\ubd84\uc758 1\ubc16\uc5d0 \uc548\ub418\ub294 \uac00\uaca9\uc774\ub2e4. \ubbf8\uad6d\uc740 \ud55c\uad6d\uc774 \uc77c\ubcf8\uc73c\ub85c\ubd80\ud130 \ud574\ubc29\ud55c \uc9c1\ud6c4\ubd80\ud130 1970\ub144 5\uc6d4\uae4c\uc9c0 20\ub144 \ub3d9\uc548 \ubb3c\uc790\uc640 \uc678\ud654\ubd80\uc871 \ubb38\uc81c\ub97c \ubb34\uc0c1\uc73c\ub85c \uc9c0\uc6d0\ud574\uc8fc\uc5c8\uc73c\uba70, \ud2b9\ud788 1950\ub300\ub9d0\uae4c\uc9c0\ub294 \uc720\uc77c\ud55c \uc678\uc790\ub3c4\uc785 \ucc3d\uad6c\ub85c \ud55c\uad6d \uacbd\uc81c\ubd80\ud765\uc5d0 \ubbf8\uad6d\uc758 \ud55c\uad6d \uc6d0\uc870 \uc815\ucc45\uc774 \uac00\uc7a5 \ud070 \uae30\uc5ec\ub97c \ud574\uc8fc\uc5c8\ub2e4. \ubbf8\uad6d\uc758 \ud55c\uad6d \ubb34\uc0c1\uc6d0\uc870\ub294 \uc57d 44\uc5b5 \ub2ec\ub7ec, \uc720\uc0c1\uc6d0\uc870\ub294 \uc57d 4\uc5b5 \ub2ec\ub7ec\uc5d0 \ub2ec\ud558\uc5ec \ud55c\uad6d\uacbd\uc81c\uc758 \ud22c\uc790\uc790\uc6d0 \ub9c8\ub828, \uad6d\uc81c\uc218\uc9c0 \uc801\uc790\ubcf4\uc804 \ubc0f \uacbd\uc81c\uc131\uc7a5\uc5d0\uc11c \ub9e4\uc6b0 \uc911\uc694\ud55c \uc5ed\ud560\uc744 \ud588\uace0 \ubbf8\uad6d\uc758 \uc6d0\uc870\ub294 \uadf8 \ub2f9\uc2dc \uac00\ub09c\ud588\ub358 \ud55c\uad6d \uc815\ubd80 \uc608\uc0b0\uc5d0 \ud070 \ub3c4\uc6c0\uc774 \ub418\uc5c8\ub2e4. 1950~70\ub144 \ud55c\uad6d\uc758 \uacbd\uc81c\ub294 \ubbf8\uad6d \uc6d0\uc870 \uacbd\uc81c \uccb4\uc81c\ub85c, \ubbf8\uad6d\uc73c\ub85c\ubd80\ud130 \ub18d\uc0b0\ubb3c, \uc18c\ube44\uc7ac, \uac74\ucd95\uc790\uc7ac, \uc678\ud654 \ub4f1\uc758 \ubb34\uc0c1 \uc9c0\uc6d0\uc744 \ubc1b\uc73c\uba74\uc11c \uc2dd\ub7c9 \ubb38\uc81c\ub97c \ud574\uacb0\ud558\uc600\uace0 1961\ub144 \uae30\uc900\uc73c\ub85c \ud55c\uad6d\uc758 GDP(\uad6d\ub0b4\ucd1d\uc0dd\uc0b0)\uac00 23\uc5b5 \ub2ec\ub7ec \uc218\uc900\uc774\uc5c8\ub2e4\ub294 \uac83\uc744 \uac10\uc548\ud558\uba74 \ubbf8\uad6d\uc73c\ub85c\ubd80\ud130 \ubc1b\uc740 \ubb34\uc0c1\uc6d0\uc870 44\uc5b5 \ub2ec\ub7ec\ub294 \uadf8\ub2f9\uc2dc \uc6b0\ub9ac\ub098\ub77c GDP\uc758 2\ubc30\ub098 \ub41c\ub2e4."} {"question": "\uc77c\ubcf8\uc758 \uac15\uc218\uc758 \ud2b9\uc9d5 \uc911\uc5d0, \ub3d9\ud574\ub97c \uac74\ub108\uc628 \uc2b5\ud55c \ud55c\uae30\ub958\ub294 \uc77c\ubcf8\uc758 \uc5b4\ub5a4 \uc0b0\ub9e5\uc5d0\uc11c \uc0c1\uc2b9\ud558\ub294\uac00?", "paragraph": "\uac15\uc218 \uba74\uc5d0\uc11c \ubcfc \ub54c, \uac00\uc7a5 \uc911\uc694\ud55c \ud2b9\uc9d5\uc740 \uaca8\uc6b8 \uc2dc\ubca0\ub9ac\uc544\ub85c\ubd80\ud130 \ubd81\uc11c\uacc4\uc808\ud48d\uc774 \ubd88 \uc2dc\uae30\uc758 \uac15\uc218 \ubd84\ud3ec\uc774\ub2e4. \ub3d9\ud574\ub97c \uac74\ub108\uc628 \uc2b5\ud55c \ud55c\uae30\ub958\uac00 \uc77c\ubcf8\uc758 \ucc99\ub7c9\uc0b0\ub9e5(\u810a\u6881\u5c71\u8108, \ud788\ub2e4\u00b7\uc5d0\uce58\uace0\u00b7\uc624\uc6b0\uc0b0\ub9e5)\uc5d0\uc11c \uc0c1\uc2b9\ud558\uc5ec, \uc0b0\ub9e5\ubcf4\ub2e4 \uc11c\ucabd \ub3d9\ud574\ucabd\uc758 \uac01\uc9c0\uc5d0 \ub2e4\ub7c9\uc758 \uac15\uc124\uc744 \uac00\uc838\uc624\uace0 \uc788\ub2e4. \ud55c\ud3b8, \ud0dc\ud3c9\uc591\ucabd\uc5d0\uc11c\ub294 12\uc6d4 \ub9d0 2\uc6d4 \ucd08\uae30\uae4c\uc9c0\ub294 \uac70\uc758 \uac15\uc6b0\uc124\uc744 \ubcfc \uc218 \uc5c6\ub2e4. \ub610\ud55c \uac01\uc9c0\uc758 \uac15\uc218\ub7c9\uc740 \ud574\uc5d0 \ub530\ub77c \uadf8 \ubcc0\ub3d9\uc774 \uc2ec\ud558\uc5ec, \uc7a5\ub9c8\u00b7\ud0dc\ud48d\uc2dc\uc5d0 \ubcfc \uc218 \uc788\ub294 \uac83\ucc98\ub7fc \uc9d1\uc911\ud638\uc6b0\uac00 \ub418\ub294 \uc77c\uc774 \ub9ce\ub2e4. \ud558\ucc9c\uc774 \uae09\ub958\uc778\ub370\ub2e4\uac00, \ub2e8\uc2dc\uac04\uc5d0 \uc9d1\uc911\ud558\ub294 \uac15\uc218 \ub54c\ubb38\uc5d0 \uac15\uc218\uc758 80%\uac00 \ud558\ucc9c\uc744 \ud1b5\ud574\uc11c \uc9c1\uc811 \ubc14\ub2e4\ub85c \ud758\ub7ec\ub4e4\uc5b4\uac00\uace0, \ud638\uc6b0\uc758 \uacbd\uc6b0 \ud558\ucc9c\uc774 \ubc94\ub78c\ud558\uc5ec \ud558\ucc9c\uc758 \uc720\ub85c(\u6d41\u8def)\ub97c \ub530\ub77c \uc218\ud574\uac00 \uc0dd\uae30\ub294 \uc77c\uc774 \ub9ce\ub2e4.", "answer": "\ucc99\ub7c9\uc0b0\ub9e5", "sentence": "\ub3d9\ud574\ub97c \uac74\ub108\uc628 \uc2b5\ud55c \ud55c\uae30\ub958\uac00 \uc77c\ubcf8\uc758 \ucc99\ub7c9\uc0b0\ub9e5 (\u810a\u6881\u5c71\u8108, \ud788\ub2e4\u00b7\uc5d0\uce58\uace0\u00b7\uc624\uc6b0\uc0b0\ub9e5)\uc5d0\uc11c \uc0c1\uc2b9\ud558\uc5ec, \uc0b0\ub9e5\ubcf4\ub2e4 \uc11c\ucabd \ub3d9\ud574\ucabd\uc758 \uac01\uc9c0\uc5d0 \ub2e4\ub7c9\uc758 \uac15\uc124\uc744 \uac00\uc838\uc624\uace0 \uc788\ub2e4.", "paragraph_sentence": "\uac15\uc218 \uba74\uc5d0\uc11c \ubcfc \ub54c, \uac00\uc7a5 \uc911\uc694\ud55c \ud2b9\uc9d5\uc740 \uaca8\uc6b8 \uc2dc\ubca0\ub9ac\uc544\ub85c\ubd80\ud130 \ubd81\uc11c\uacc4\uc808\ud48d\uc774 \ubd88 \uc2dc\uae30\uc758 \uac15\uc218 \ubd84\ud3ec\uc774\ub2e4. \ub3d9\ud574\ub97c \uac74\ub108\uc628 \uc2b5\ud55c \ud55c\uae30\ub958\uac00 \uc77c\ubcf8\uc758 \ucc99\ub7c9\uc0b0\ub9e5 (\u810a\u6881\u5c71\u8108, \ud788\ub2e4\u00b7\uc5d0\uce58\uace0\u00b7\uc624\uc6b0\uc0b0\ub9e5)\uc5d0\uc11c \uc0c1\uc2b9\ud558\uc5ec, \uc0b0\ub9e5\ubcf4\ub2e4 \uc11c\ucabd \ub3d9\ud574\ucabd\uc758 \uac01\uc9c0\uc5d0 \ub2e4\ub7c9\uc758 \uac15\uc124\uc744 \uac00\uc838\uc624\uace0 \uc788\ub2e4. \ud55c\ud3b8, \ud0dc\ud3c9\uc591\ucabd\uc5d0\uc11c\ub294 12\uc6d4 \ub9d0 2\uc6d4 \ucd08\uae30\uae4c\uc9c0\ub294 \uac70\uc758 \uac15\uc6b0\uc124\uc744 \ubcfc \uc218 \uc5c6\ub2e4. \ub610\ud55c \uac01\uc9c0\uc758 \uac15\uc218\ub7c9\uc740 \ud574\uc5d0 \ub530\ub77c \uadf8 \ubcc0\ub3d9\uc774 \uc2ec\ud558\uc5ec, \uc7a5\ub9c8\u00b7\ud0dc\ud48d\uc2dc\uc5d0 \ubcfc \uc218 \uc788\ub294 \uac83\ucc98\ub7fc \uc9d1\uc911\ud638\uc6b0\uac00 \ub418\ub294 \uc77c\uc774 \ub9ce\ub2e4. \ud558\ucc9c\uc774 \uae09\ub958\uc778\ub370\ub2e4\uac00, \ub2e8\uc2dc\uac04\uc5d0 \uc9d1\uc911\ud558\ub294 \uac15\uc218 \ub54c\ubb38\uc5d0 \uac15\uc218\uc758 80%\uac00 \ud558\ucc9c\uc744 \ud1b5\ud574\uc11c \uc9c1\uc811 \ubc14\ub2e4\ub85c \ud758\ub7ec\ub4e4\uc5b4\uac00\uace0, \ud638\uc6b0\uc758 \uacbd\uc6b0 \ud558\ucc9c\uc774 \ubc94\ub78c\ud558\uc5ec \ud558\ucc9c\uc758 \uc720\ub85c(\u6d41\u8def)\ub97c \ub530\ub77c \uc218\ud574\uac00 \uc0dd\uae30\ub294 \uc77c\uc774 \ub9ce\ub2e4.", "paragraph_answer": "\uac15\uc218 \uba74\uc5d0\uc11c \ubcfc \ub54c, \uac00\uc7a5 \uc911\uc694\ud55c \ud2b9\uc9d5\uc740 \uaca8\uc6b8 \uc2dc\ubca0\ub9ac\uc544\ub85c\ubd80\ud130 \ubd81\uc11c\uacc4\uc808\ud48d\uc774 \ubd88 \uc2dc\uae30\uc758 \uac15\uc218 \ubd84\ud3ec\uc774\ub2e4. \ub3d9\ud574\ub97c \uac74\ub108\uc628 \uc2b5\ud55c \ud55c\uae30\ub958\uac00 \uc77c\ubcf8\uc758 \ucc99\ub7c9\uc0b0\ub9e5 (\u810a\u6881\u5c71\u8108, \ud788\ub2e4\u00b7\uc5d0\uce58\uace0\u00b7\uc624\uc6b0\uc0b0\ub9e5)\uc5d0\uc11c \uc0c1\uc2b9\ud558\uc5ec, \uc0b0\ub9e5\ubcf4\ub2e4 \uc11c\ucabd \ub3d9\ud574\ucabd\uc758 \uac01\uc9c0\uc5d0 \ub2e4\ub7c9\uc758 \uac15\uc124\uc744 \uac00\uc838\uc624\uace0 \uc788\ub2e4. \ud55c\ud3b8, \ud0dc\ud3c9\uc591\ucabd\uc5d0\uc11c\ub294 12\uc6d4 \ub9d0 2\uc6d4 \ucd08\uae30\uae4c\uc9c0\ub294 \uac70\uc758 \uac15\uc6b0\uc124\uc744 \ubcfc \uc218 \uc5c6\ub2e4. \ub610\ud55c \uac01\uc9c0\uc758 \uac15\uc218\ub7c9\uc740 \ud574\uc5d0 \ub530\ub77c \uadf8 \ubcc0\ub3d9\uc774 \uc2ec\ud558\uc5ec, \uc7a5\ub9c8\u00b7\ud0dc\ud48d\uc2dc\uc5d0 \ubcfc \uc218 \uc788\ub294 \uac83\ucc98\ub7fc \uc9d1\uc911\ud638\uc6b0\uac00 \ub418\ub294 \uc77c\uc774 \ub9ce\ub2e4. \ud558\ucc9c\uc774 \uae09\ub958\uc778\ub370\ub2e4\uac00, \ub2e8\uc2dc\uac04\uc5d0 \uc9d1\uc911\ud558\ub294 \uac15\uc218 \ub54c\ubb38\uc5d0 \uac15\uc218\uc758 80%\uac00 \ud558\ucc9c\uc744 \ud1b5\ud574\uc11c \uc9c1\uc811 \ubc14\ub2e4\ub85c \ud758\ub7ec\ub4e4\uc5b4\uac00\uace0, \ud638\uc6b0\uc758 \uacbd\uc6b0 \ud558\ucc9c\uc774 \ubc94\ub78c\ud558\uc5ec \ud558\ucc9c\uc758 \uc720\ub85c(\u6d41\u8def)\ub97c \ub530\ub77c \uc218\ud574\uac00 \uc0dd\uae30\ub294 \uc77c\uc774 \ub9ce\ub2e4.", "sentence_answer": "\ub3d9\ud574\ub97c \uac74\ub108\uc628 \uc2b5\ud55c \ud55c\uae30\ub958\uac00 \uc77c\ubcf8\uc758 \ucc99\ub7c9\uc0b0\ub9e5 (\u810a\u6881\u5c71\u8108, \ud788\ub2e4\u00b7\uc5d0\uce58\uace0\u00b7\uc624\uc6b0\uc0b0\ub9e5)\uc5d0\uc11c \uc0c1\uc2b9\ud558\uc5ec, \uc0b0\ub9e5\ubcf4\ub2e4 \uc11c\ucabd \ub3d9\ud574\ucabd\uc758 \uac01\uc9c0\uc5d0 \ub2e4\ub7c9\uc758 \uac15\uc124\uc744 \uac00\uc838\uc624\uace0 \uc788\ub2e4.", "paragraph_id": "6173085-13-0", "paragraph_question": "question: \uc77c\ubcf8\uc758 \uac15\uc218\uc758 \ud2b9\uc9d5 \uc911\uc5d0, \ub3d9\ud574\ub97c \uac74\ub108\uc628 \uc2b5\ud55c \ud55c\uae30\ub958\ub294 \uc77c\ubcf8\uc758 \uc5b4\ub5a4 \uc0b0\ub9e5\uc5d0\uc11c \uc0c1\uc2b9\ud558\ub294\uac00?, context: \uac15\uc218 \uba74\uc5d0\uc11c \ubcfc \ub54c, \uac00\uc7a5 \uc911\uc694\ud55c \ud2b9\uc9d5\uc740 \uaca8\uc6b8 \uc2dc\ubca0\ub9ac\uc544\ub85c\ubd80\ud130 \ubd81\uc11c\uacc4\uc808\ud48d\uc774 \ubd88 \uc2dc\uae30\uc758 \uac15\uc218 \ubd84\ud3ec\uc774\ub2e4. \ub3d9\ud574\ub97c \uac74\ub108\uc628 \uc2b5\ud55c \ud55c\uae30\ub958\uac00 \uc77c\ubcf8\uc758 \ucc99\ub7c9\uc0b0\ub9e5(\u810a\u6881\u5c71\u8108, \ud788\ub2e4\u00b7\uc5d0\uce58\uace0\u00b7\uc624\uc6b0\uc0b0\ub9e5)\uc5d0\uc11c \uc0c1\uc2b9\ud558\uc5ec, \uc0b0\ub9e5\ubcf4\ub2e4 \uc11c\ucabd \ub3d9\ud574\ucabd\uc758 \uac01\uc9c0\uc5d0 \ub2e4\ub7c9\uc758 \uac15\uc124\uc744 \uac00\uc838\uc624\uace0 \uc788\ub2e4. \ud55c\ud3b8, \ud0dc\ud3c9\uc591\ucabd\uc5d0\uc11c\ub294 12\uc6d4 \ub9d0 2\uc6d4 \ucd08\uae30\uae4c\uc9c0\ub294 \uac70\uc758 \uac15\uc6b0\uc124\uc744 \ubcfc \uc218 \uc5c6\ub2e4. \ub610\ud55c \uac01\uc9c0\uc758 \uac15\uc218\ub7c9\uc740 \ud574\uc5d0 \ub530\ub77c \uadf8 \ubcc0\ub3d9\uc774 \uc2ec\ud558\uc5ec, \uc7a5\ub9c8\u00b7\ud0dc\ud48d\uc2dc\uc5d0 \ubcfc \uc218 \uc788\ub294 \uac83\ucc98\ub7fc \uc9d1\uc911\ud638\uc6b0\uac00 \ub418\ub294 \uc77c\uc774 \ub9ce\ub2e4. \ud558\ucc9c\uc774 \uae09\ub958\uc778\ub370\ub2e4\uac00, \ub2e8\uc2dc\uac04\uc5d0 \uc9d1\uc911\ud558\ub294 \uac15\uc218 \ub54c\ubb38\uc5d0 \uac15\uc218\uc758 80%\uac00 \ud558\ucc9c\uc744 \ud1b5\ud574\uc11c \uc9c1\uc811 \ubc14\ub2e4\ub85c \ud758\ub7ec\ub4e4\uc5b4\uac00\uace0, \ud638\uc6b0\uc758 \uacbd\uc6b0 \ud558\ucc9c\uc774 \ubc94\ub78c\ud558\uc5ec \ud558\ucc9c\uc758 \uc720\ub85c(\u6d41\u8def)\ub97c \ub530\ub77c \uc218\ud574\uac00 \uc0dd\uae30\ub294 \uc77c\uc774 \ub9ce\ub2e4."} {"question": "\ubabd\uace8\uad70\uc774 \uc804\uba78\ud558\uace0, \ud0a4\ud2b8\ubd80\uce74\ub3c4 \uc804\uc0ac\ud558\uac8c \ub41c \uc804\ud22c \uc774\ub984\uc740?", "paragraph": "\ud6cc\ub77c\uad6c\ub294 \uc2dc\ub9ac\uc544\ub97c \ub5a0\ub098\uba70 \uc790\uc2e0\uc758 \uad70\ub300\ub97c \uc7a5\uad70 \ud0a4\ud2b8\ubd80\uce74\uc5d0\uac8c \ub9e1\uacbc\ub2e4. \ud0a4\ud2b8\ubd80\uce74\ub294 \ud6cc\ub77c\uad6c\uc758 \uad70\ub300\ub97c \uc774\uc5b4\ubc1b\uc544 \ub2e4\ub9c8\uc2a4\ucfe0\uc2a4\ub97c \uc810\ub839\ud558\uace0 \uc2dc\ub9ac\uc544 \uc9c0\uc5ed\uc758 \uc544\uc774\uc720\ube0c \uc220\ud0c4 \uc548\ub098\uc2dc\ub974 \uc720\uc218\ud504\ub97c \ud574\uce58\uc6b0\uace0 \uc2dc\ub9ac\uc544\ub97c \uc644\uc804\ud788 \uc7a5\uc545\ud588\ub2e4. \uc2dc\ub9ac\uc544\ub97c \uc644\uc804\ud788 \uc7a5\uc545\ud55c \ud0a4\ud2b8\ubd80\uce74\ub294 \uc774\uc9d1\ud2b8\uc758 \ub9d8\ub8e8\ud06c \uc655\uc870\uc5d0 \ud56d\ubcf5 \uad8c\uace0\ub97c \ubcf4\ub0c8\uc73c\ub098 \ub2f9\uc2dc \ub9d8\ub8e8\ud06c \uc655\uc870\uc758 \uc220\ud0c4\uc778 \ucfe0\ud22c\uc988\ub294 \uc774\ub97c \uac70\ubd80\ud558\uace0 \ubc14\uc774\ubc14\ub974\uc2a4\uc640 \uc5f0\ud569\ud558\uc5ec \ud0a4\ud2b8\ubd80\uce74\uc640 \ub9de\uc130\ub2e4. \ub9c8\uce68\ub0b4 \ubabd\uace8 \uad70\uacfc \ub9d8\ub8e8\ud06c \uc655\uc870 \uad70\uc740 \uc544\uc778\uc798\ub8e8\ud2b8\uc5d0\uc11c \ud06c\uac8c \uaca9\ub3cc\ud588\ub294\ub370 \uc774 \uc804\ud22c\uc5d0\uc11c \ubabd\uace8\uad70\uc740 \uc804\uba78\ud558\uc600\uc73c\uba70 \ucd1d\uc0ac\ub839\uad00\uc778 \ud0a4\ud2b8\ubd80\uce74\ub3c4 \uc804\uc0ac\ud558\uac8c \ub41c\ub2e4. (\uc544\uc778\uc798\ub8e8\ud2b8 \uc804\ud22c, 1260) \uc544\uc778\uc798\ub8e8\ud2b8 \uc804\ud22c\uc758 \ucc38\ud328 \uc774\ud6c4 \uc804\ud669\uc740 \uacb0\uc815\uc801\uc73c\ub85c \ubc14\ub00c\uc5b4 \ubabd\uace8\uad70\uc740 \uc774\uc2a4\ub77c\uc5d8\uc744 \ub0b4\uc8fc\uace0 \ud2f0\uadf8\ub9ac\uc2a4 \uac15 \uc774\ubd81\uc73c\ub85c \ubd81\uc0c1\ud558\uc600\uc73c\uba70 \uc544\uc778\uc798\ub8e8\ud2b8 \uc804\ud22c \uc2b9\ub9ac\uc758 \uc8fc\uc5ed\uc778 \ubc14\uc774\ubc14\ub974\uc2a4\ub294 \ubabd\uace8\uc5d0 \uc774\uc5b4 \uacbd\uc7c1\uc790\uc778 \ucfe0\ud22c\uc988\ub3c4 \uc81c\uac70\ud558\uace0\ub294 \ub9d8\ub8e8\ud06c \uc655\uc870\uc758 \uc720\uc77c\ud55c \uad8c\ub825\uc790\ub85c \uc9d1\uad8c\ud588\ub2e4. \uc774\ud6c4 \uc77c \ud55c\uad6d\uacfc \ub9d8\ub8e8\ud06c \uc655\uc870\uc758 \uad6d\uacbd\uc120\uc740 \uc2dc\ub9ac\uc544\uc758 \ud2f0\uadf8\ub9ac\uc2a4 \uac15\uc73c\ub85c \uacb0\uc815\ub418\uc5c8\uc73c\uba70 \uc57d 80\uc5ec \ub144 \ub3d9\uc548 \uadf8\ub300\ub85c \uc720\uc9c0\ub418\uc5c8\ub2e4.", "answer": "\uc544\uc778\uc798\ub8e8\ud2b8 \uc804\ud22c", "sentence": "( \uc544\uc778\uc798\ub8e8\ud2b8 \uc804\ud22c ,", "paragraph_sentence": "\ud6cc\ub77c\uad6c\ub294 \uc2dc\ub9ac\uc544\ub97c \ub5a0\ub098\uba70 \uc790\uc2e0\uc758 \uad70\ub300\ub97c \uc7a5\uad70 \ud0a4\ud2b8\ubd80\uce74\uc5d0\uac8c \ub9e1\uacbc\ub2e4. \ud0a4\ud2b8\ubd80\uce74\ub294 \ud6cc\ub77c\uad6c\uc758 \uad70\ub300\ub97c \uc774\uc5b4\ubc1b\uc544 \ub2e4\ub9c8\uc2a4\ucfe0\uc2a4\ub97c \uc810\ub839\ud558\uace0 \uc2dc\ub9ac\uc544 \uc9c0\uc5ed\uc758 \uc544\uc774\uc720\ube0c \uc220\ud0c4 \uc548\ub098\uc2dc\ub974 \uc720\uc218\ud504\ub97c \ud574\uce58\uc6b0\uace0 \uc2dc\ub9ac\uc544\ub97c \uc644\uc804\ud788 \uc7a5\uc545\ud588\ub2e4. \uc2dc\ub9ac\uc544\ub97c \uc644\uc804\ud788 \uc7a5\uc545\ud55c \ud0a4\ud2b8\ubd80\uce74\ub294 \uc774\uc9d1\ud2b8\uc758 \ub9d8\ub8e8\ud06c \uc655\uc870\uc5d0 \ud56d\ubcf5 \uad8c\uace0\ub97c \ubcf4\ub0c8\uc73c\ub098 \ub2f9\uc2dc \ub9d8\ub8e8\ud06c \uc655\uc870\uc758 \uc220\ud0c4\uc778 \ucfe0\ud22c\uc988\ub294 \uc774\ub97c \uac70\ubd80\ud558\uace0 \ubc14\uc774\ubc14\ub974\uc2a4\uc640 \uc5f0\ud569\ud558\uc5ec \ud0a4\ud2b8\ubd80\uce74\uc640 \ub9de\uc130\ub2e4. \ub9c8\uce68\ub0b4 \ubabd\uace8 \uad70\uacfc \ub9d8\ub8e8\ud06c \uc655\uc870 \uad70\uc740 \uc544\uc778\uc798\ub8e8\ud2b8\uc5d0\uc11c \ud06c\uac8c \uaca9\ub3cc\ud588\ub294\ub370 \uc774 \uc804\ud22c\uc5d0\uc11c \ubabd\uace8\uad70\uc740 \uc804\uba78\ud558\uc600\uc73c\uba70 \ucd1d\uc0ac\ub839\uad00\uc778 \ud0a4\ud2b8\ubd80\uce74\ub3c4 \uc804\uc0ac\ud558\uac8c \ub41c\ub2e4. ( \uc544\uc778\uc798\ub8e8\ud2b8 \uc804\ud22c , 1260) \uc544\uc778\uc798\ub8e8\ud2b8 \uc804\ud22c\uc758 \ucc38\ud328 \uc774\ud6c4 \uc804\ud669\uc740 \uacb0\uc815\uc801\uc73c\ub85c \ubc14\ub00c\uc5b4 \ubabd\uace8\uad70\uc740 \uc774\uc2a4\ub77c\uc5d8\uc744 \ub0b4\uc8fc\uace0 \ud2f0\uadf8\ub9ac\uc2a4 \uac15 \uc774\ubd81\uc73c\ub85c \ubd81\uc0c1\ud558\uc600\uc73c\uba70 \uc544\uc778\uc798\ub8e8\ud2b8 \uc804\ud22c \uc2b9\ub9ac\uc758 \uc8fc\uc5ed\uc778 \ubc14\uc774\ubc14\ub974\uc2a4\ub294 \ubabd\uace8\uc5d0 \uc774\uc5b4 \uacbd\uc7c1\uc790\uc778 \ucfe0\ud22c\uc988\ub3c4 \uc81c\uac70\ud558\uace0\ub294 \ub9d8\ub8e8\ud06c \uc655\uc870\uc758 \uc720\uc77c\ud55c \uad8c\ub825\uc790\ub85c \uc9d1\uad8c\ud588\ub2e4. \uc774\ud6c4 \uc77c \ud55c\uad6d\uacfc \ub9d8\ub8e8\ud06c \uc655\uc870\uc758 \uad6d\uacbd\uc120\uc740 \uc2dc\ub9ac\uc544\uc758 \ud2f0\uadf8\ub9ac\uc2a4 \uac15\uc73c\ub85c \uacb0\uc815\ub418\uc5c8\uc73c\uba70 \uc57d 80\uc5ec \ub144 \ub3d9\uc548 \uadf8\ub300\ub85c \uc720\uc9c0\ub418\uc5c8\ub2e4.", "paragraph_answer": "\ud6cc\ub77c\uad6c\ub294 \uc2dc\ub9ac\uc544\ub97c \ub5a0\ub098\uba70 \uc790\uc2e0\uc758 \uad70\ub300\ub97c \uc7a5\uad70 \ud0a4\ud2b8\ubd80\uce74\uc5d0\uac8c \ub9e1\uacbc\ub2e4. \ud0a4\ud2b8\ubd80\uce74\ub294 \ud6cc\ub77c\uad6c\uc758 \uad70\ub300\ub97c \uc774\uc5b4\ubc1b\uc544 \ub2e4\ub9c8\uc2a4\ucfe0\uc2a4\ub97c \uc810\ub839\ud558\uace0 \uc2dc\ub9ac\uc544 \uc9c0\uc5ed\uc758 \uc544\uc774\uc720\ube0c \uc220\ud0c4 \uc548\ub098\uc2dc\ub974 \uc720\uc218\ud504\ub97c \ud574\uce58\uc6b0\uace0 \uc2dc\ub9ac\uc544\ub97c \uc644\uc804\ud788 \uc7a5\uc545\ud588\ub2e4. \uc2dc\ub9ac\uc544\ub97c \uc644\uc804\ud788 \uc7a5\uc545\ud55c \ud0a4\ud2b8\ubd80\uce74\ub294 \uc774\uc9d1\ud2b8\uc758 \ub9d8\ub8e8\ud06c \uc655\uc870\uc5d0 \ud56d\ubcf5 \uad8c\uace0\ub97c \ubcf4\ub0c8\uc73c\ub098 \ub2f9\uc2dc \ub9d8\ub8e8\ud06c \uc655\uc870\uc758 \uc220\ud0c4\uc778 \ucfe0\ud22c\uc988\ub294 \uc774\ub97c \uac70\ubd80\ud558\uace0 \ubc14\uc774\ubc14\ub974\uc2a4\uc640 \uc5f0\ud569\ud558\uc5ec \ud0a4\ud2b8\ubd80\uce74\uc640 \ub9de\uc130\ub2e4. \ub9c8\uce68\ub0b4 \ubabd\uace8 \uad70\uacfc \ub9d8\ub8e8\ud06c \uc655\uc870 \uad70\uc740 \uc544\uc778\uc798\ub8e8\ud2b8\uc5d0\uc11c \ud06c\uac8c \uaca9\ub3cc\ud588\ub294\ub370 \uc774 \uc804\ud22c\uc5d0\uc11c \ubabd\uace8\uad70\uc740 \uc804\uba78\ud558\uc600\uc73c\uba70 \ucd1d\uc0ac\ub839\uad00\uc778 \ud0a4\ud2b8\ubd80\uce74\ub3c4 \uc804\uc0ac\ud558\uac8c \ub41c\ub2e4. ( \uc544\uc778\uc798\ub8e8\ud2b8 \uc804\ud22c , 1260) \uc544\uc778\uc798\ub8e8\ud2b8 \uc804\ud22c\uc758 \ucc38\ud328 \uc774\ud6c4 \uc804\ud669\uc740 \uacb0\uc815\uc801\uc73c\ub85c \ubc14\ub00c\uc5b4 \ubabd\uace8\uad70\uc740 \uc774\uc2a4\ub77c\uc5d8\uc744 \ub0b4\uc8fc\uace0 \ud2f0\uadf8\ub9ac\uc2a4 \uac15 \uc774\ubd81\uc73c\ub85c \ubd81\uc0c1\ud558\uc600\uc73c\uba70 \uc544\uc778\uc798\ub8e8\ud2b8 \uc804\ud22c \uc2b9\ub9ac\uc758 \uc8fc\uc5ed\uc778 \ubc14\uc774\ubc14\ub974\uc2a4\ub294 \ubabd\uace8\uc5d0 \uc774\uc5b4 \uacbd\uc7c1\uc790\uc778 \ucfe0\ud22c\uc988\ub3c4 \uc81c\uac70\ud558\uace0\ub294 \ub9d8\ub8e8\ud06c \uc655\uc870\uc758 \uc720\uc77c\ud55c \uad8c\ub825\uc790\ub85c \uc9d1\uad8c\ud588\ub2e4. \uc774\ud6c4 \uc77c \ud55c\uad6d\uacfc \ub9d8\ub8e8\ud06c \uc655\uc870\uc758 \uad6d\uacbd\uc120\uc740 \uc2dc\ub9ac\uc544\uc758 \ud2f0\uadf8\ub9ac\uc2a4 \uac15\uc73c\ub85c \uacb0\uc815\ub418\uc5c8\uc73c\uba70 \uc57d 80\uc5ec \ub144 \ub3d9\uc548 \uadf8\ub300\ub85c \uc720\uc9c0\ub418\uc5c8\ub2e4.", "sentence_answer": "( \uc544\uc778\uc798\ub8e8\ud2b8 \uc804\ud22c ,", "paragraph_id": "6548308-2-1", "paragraph_question": "question: \ubabd\uace8\uad70\uc774 \uc804\uba78\ud558\uace0, \ud0a4\ud2b8\ubd80\uce74\ub3c4 \uc804\uc0ac\ud558\uac8c \ub41c \uc804\ud22c \uc774\ub984\uc740?, context: \ud6cc\ub77c\uad6c\ub294 \uc2dc\ub9ac\uc544\ub97c \ub5a0\ub098\uba70 \uc790\uc2e0\uc758 \uad70\ub300\ub97c \uc7a5\uad70 \ud0a4\ud2b8\ubd80\uce74\uc5d0\uac8c \ub9e1\uacbc\ub2e4. \ud0a4\ud2b8\ubd80\uce74\ub294 \ud6cc\ub77c\uad6c\uc758 \uad70\ub300\ub97c \uc774\uc5b4\ubc1b\uc544 \ub2e4\ub9c8\uc2a4\ucfe0\uc2a4\ub97c \uc810\ub839\ud558\uace0 \uc2dc\ub9ac\uc544 \uc9c0\uc5ed\uc758 \uc544\uc774\uc720\ube0c \uc220\ud0c4 \uc548\ub098\uc2dc\ub974 \uc720\uc218\ud504\ub97c \ud574\uce58\uc6b0\uace0 \uc2dc\ub9ac\uc544\ub97c \uc644\uc804\ud788 \uc7a5\uc545\ud588\ub2e4. \uc2dc\ub9ac\uc544\ub97c \uc644\uc804\ud788 \uc7a5\uc545\ud55c \ud0a4\ud2b8\ubd80\uce74\ub294 \uc774\uc9d1\ud2b8\uc758 \ub9d8\ub8e8\ud06c \uc655\uc870\uc5d0 \ud56d\ubcf5 \uad8c\uace0\ub97c \ubcf4\ub0c8\uc73c\ub098 \ub2f9\uc2dc \ub9d8\ub8e8\ud06c \uc655\uc870\uc758 \uc220\ud0c4\uc778 \ucfe0\ud22c\uc988\ub294 \uc774\ub97c \uac70\ubd80\ud558\uace0 \ubc14\uc774\ubc14\ub974\uc2a4\uc640 \uc5f0\ud569\ud558\uc5ec \ud0a4\ud2b8\ubd80\uce74\uc640 \ub9de\uc130\ub2e4. \ub9c8\uce68\ub0b4 \ubabd\uace8 \uad70\uacfc \ub9d8\ub8e8\ud06c \uc655\uc870 \uad70\uc740 \uc544\uc778\uc798\ub8e8\ud2b8\uc5d0\uc11c \ud06c\uac8c \uaca9\ub3cc\ud588\ub294\ub370 \uc774 \uc804\ud22c\uc5d0\uc11c \ubabd\uace8\uad70\uc740 \uc804\uba78\ud558\uc600\uc73c\uba70 \ucd1d\uc0ac\ub839\uad00\uc778 \ud0a4\ud2b8\ubd80\uce74\ub3c4 \uc804\uc0ac\ud558\uac8c \ub41c\ub2e4. (\uc544\uc778\uc798\ub8e8\ud2b8 \uc804\ud22c, 1260) \uc544\uc778\uc798\ub8e8\ud2b8 \uc804\ud22c\uc758 \ucc38\ud328 \uc774\ud6c4 \uc804\ud669\uc740 \uacb0\uc815\uc801\uc73c\ub85c \ubc14\ub00c\uc5b4 \ubabd\uace8\uad70\uc740 \uc774\uc2a4\ub77c\uc5d8\uc744 \ub0b4\uc8fc\uace0 \ud2f0\uadf8\ub9ac\uc2a4 \uac15 \uc774\ubd81\uc73c\ub85c \ubd81\uc0c1\ud558\uc600\uc73c\uba70 \uc544\uc778\uc798\ub8e8\ud2b8 \uc804\ud22c \uc2b9\ub9ac\uc758 \uc8fc\uc5ed\uc778 \ubc14\uc774\ubc14\ub974\uc2a4\ub294 \ubabd\uace8\uc5d0 \uc774\uc5b4 \uacbd\uc7c1\uc790\uc778 \ucfe0\ud22c\uc988\ub3c4 \uc81c\uac70\ud558\uace0\ub294 \ub9d8\ub8e8\ud06c \uc655\uc870\uc758 \uc720\uc77c\ud55c \uad8c\ub825\uc790\ub85c \uc9d1\uad8c\ud588\ub2e4. \uc774\ud6c4 \uc77c \ud55c\uad6d\uacfc \ub9d8\ub8e8\ud06c \uc655\uc870\uc758 \uad6d\uacbd\uc120\uc740 \uc2dc\ub9ac\uc544\uc758 \ud2f0\uadf8\ub9ac\uc2a4 \uac15\uc73c\ub85c \uacb0\uc815\ub418\uc5c8\uc73c\uba70 \uc57d 80\uc5ec \ub144 \ub3d9\uc548 \uadf8\ub300\ub85c \uc720\uc9c0\ub418\uc5c8\ub2e4."} {"question": "\ub9e5\uadf8\ub9ac\uac70\uac00 \ucd9c\uc5f0\ud55c 1994\ub144 \uc601\ud654 \uc258\ub85c\uc6b0 \uadf8\ub808\uc774\ube0c\uc758 \uac10\ub3c5\uc740 \ub204\uad6c\uc778\uac00?", "paragraph": "\u300a\uc2a4\uce7c\ub9bf \uc564 \ube14\ub799\u300b\uc744 \ucd2c\uc601\ud558\uba74\uc11c \ub9e5\uadf8\ub9ac\uac70\ub294 \ub2f9\uc2dc\uc5d0 \uc0c8\ub0b4\uae30 \uac10\ub3c5\uc774\uc5c8\ub358 \ub300\ub2c8 \ubcf4\uc77c\uc744 \ub9cc\ub0ac\uace0, \uccab \uc601\ud654\uc5d0 \ucc38\uc5ec\ud560 \uae30\ud68c\ub97c \uc5bb\uc5c8\ub2e4. 1994\ub144, \ub300\ub2c8 \ubcf4\uc77c\uc774 \uac10\ub3c5\ud55c \uc2a4\ub9b4\ub7ec \uc601\ud654 \u300a\uc258\ub85c\uc6b0 \uadf8\ub808\uc774\ube0c\u300b\uc5d0 \ucd9c\uc5f0\ud558\uc600\ub2e4. \uc601\ud654\uc5d0\uc11c \ubcf4\uc5ec\uc900 \uc5f0\uae30\ub294 \ucc2c\uc0ac\ub97c \ubc1b\uc558\ub2e4. \ub9e5\uadf8\ub9ac\uac70\ub294 \uc5e0\ud30c\uc774\uc5b4\uc0c1\uc744 \uc218\uc0c1\ud558\uc600\uc73c\uba70, \uc601\ud654\ub294 \ud310\ud0c0\uc2a4\ud3ec\ub974\ud1a0 \uc601\ud654\uc81c\uc5d0\uc11c \ucd5c\uace0\uc758 \uc601\ud654\ub85c \uc120\uc815\ub418\uc5c8\ub2e4. 1996\ub144, \ub300\ub2c8 \ubcf4\uc77c\uc758 \u300a\ud2b8\ub808\uc778\uc2a4\ud3ec\ud305\u300b\uc5d0 \ub9c8\ud06c \ub80c\ud134 \uc5ed\ud560\ub85c \ucd9c\uc5f0\ud558\uc600\uc73c\uba70, \uc601\ud654\ub294 \uc5b4\ubc14\uc778 \uc6f0\uc2dc\uc758 \ub3d9\uba85\uc758 \uc18c\uc124\uc744 \ubc14\ud0d5\uc73c\ub85c \ud558\uace0 \uc788\ub2e4. \uc774 \uc601\ud654\uc758 \ucd9c\uc5f0\uc73c\ub85c \ub9e5\uadf8\ub9ac\uac70\ub294 BAFTA \uc2a4\ucf54\ud2c0\ub79c\ub4dc\uc0c1\uc5d0\uc11c \ucd5c\uc6b0\uc218 \ub0a8\uc790 \ubc30\uc6b0\uc0c1\uc744 \uc218\uc0c1\ud558\uc600\uace0, \uc138\uacc4\uc801\uc778 \uc2a4\ud0c0\ub85c \ubc1c\ub3cb\uc6c0\ud558\uac8c \ub418\ub294 \uacc4\uae30\uac00 \ub418\uc5c8\ub2e4. \uac19\uc740 \ud574, \ub9e5\uadf8\ub9ac\uac70\ub294 \u300a\ud544\ub85c\uc6b0 \ubd81\u300b\uc5d0 \uc5f0\uc778\uc744 \ucc3e\uc544 \ubc29\ud669\ud558\ub358 \uc911 \ubc88\uc5ed\uac00 \uc81c\ub86c\uc744 \uc5f0\uae30\ud558\uc600\ub2e4. \u300a\uc5e0\ub9c8\u300b\uc5d0\ub3c4 \ud504\ub7ad\ud06c \ucc98\uce60 \uc5ed\uc73c\ub85c \ub4f1\uc7a5\ud588\ub2e4.", "answer": "\ub300\ub2c8 \ubcf4\uc77c", "sentence": "\u300a\uc2a4\uce7c\ub9bf \uc564 \ube14\ub799\u300b\uc744 \ucd2c\uc601\ud558\uba74\uc11c \ub9e5\uadf8\ub9ac\uac70\ub294 \ub2f9\uc2dc\uc5d0 \uc0c8\ub0b4\uae30 \uac10\ub3c5\uc774\uc5c8\ub358 \ub300\ub2c8 \ubcf4\uc77c \uc744 \ub9cc\ub0ac\uace0, \uccab \uc601\ud654\uc5d0 \ucc38\uc5ec\ud560 \uae30\ud68c\ub97c \uc5bb\uc5c8\ub2e4.", "paragraph_sentence": " \u300a\uc2a4\uce7c\ub9bf \uc564 \ube14\ub799\u300b\uc744 \ucd2c\uc601\ud558\uba74\uc11c \ub9e5\uadf8\ub9ac\uac70\ub294 \ub2f9\uc2dc\uc5d0 \uc0c8\ub0b4\uae30 \uac10\ub3c5\uc774\uc5c8\ub358 \ub300\ub2c8 \ubcf4\uc77c \uc744 \ub9cc\ub0ac\uace0, \uccab \uc601\ud654\uc5d0 \ucc38\uc5ec\ud560 \uae30\ud68c\ub97c \uc5bb\uc5c8\ub2e4. 1994\ub144, \ub300\ub2c8 \ubcf4\uc77c\uc774 \uac10\ub3c5\ud55c \uc2a4\ub9b4\ub7ec \uc601\ud654 \u300a\uc258\ub85c\uc6b0 \uadf8\ub808\uc774\ube0c\u300b\uc5d0 \ucd9c\uc5f0\ud558\uc600\ub2e4. \uc601\ud654\uc5d0\uc11c \ubcf4\uc5ec\uc900 \uc5f0\uae30\ub294 \ucc2c\uc0ac\ub97c \ubc1b\uc558\ub2e4. \ub9e5\uadf8\ub9ac\uac70\ub294 \uc5e0\ud30c\uc774\uc5b4\uc0c1\uc744 \uc218\uc0c1\ud558\uc600\uc73c\uba70, \uc601\ud654\ub294 \ud310\ud0c0\uc2a4\ud3ec\ub974\ud1a0 \uc601\ud654\uc81c\uc5d0\uc11c \ucd5c\uace0\uc758 \uc601\ud654\ub85c \uc120\uc815\ub418\uc5c8\ub2e4. 1996\ub144, \ub300\ub2c8 \ubcf4\uc77c\uc758 \u300a\ud2b8\ub808\uc778\uc2a4\ud3ec\ud305\u300b\uc5d0 \ub9c8\ud06c \ub80c\ud134 \uc5ed\ud560\ub85c \ucd9c\uc5f0\ud558\uc600\uc73c\uba70, \uc601\ud654\ub294 \uc5b4\ubc14\uc778 \uc6f0\uc2dc\uc758 \ub3d9\uba85\uc758 \uc18c\uc124\uc744 \ubc14\ud0d5\uc73c\ub85c \ud558\uace0 \uc788\ub2e4. \uc774 \uc601\ud654\uc758 \ucd9c\uc5f0\uc73c\ub85c \ub9e5\uadf8\ub9ac\uac70\ub294 BAFTA \uc2a4\ucf54\ud2c0\ub79c\ub4dc\uc0c1\uc5d0\uc11c \ucd5c\uc6b0\uc218 \ub0a8\uc790 \ubc30\uc6b0\uc0c1\uc744 \uc218\uc0c1\ud558\uc600\uace0, \uc138\uacc4\uc801\uc778 \uc2a4\ud0c0\ub85c \ubc1c\ub3cb\uc6c0\ud558\uac8c \ub418\ub294 \uacc4\uae30\uac00 \ub418\uc5c8\ub2e4. \uac19\uc740 \ud574, \ub9e5\uadf8\ub9ac\uac70\ub294 \u300a\ud544\ub85c\uc6b0 \ubd81\u300b\uc5d0 \uc5f0\uc778\uc744 \ucc3e\uc544 \ubc29\ud669\ud558\ub358 \uc911 \ubc88\uc5ed\uac00 \uc81c\ub86c\uc744 \uc5f0\uae30\ud558\uc600\ub2e4. \u300a\uc5e0\ub9c8\u300b\uc5d0\ub3c4 \ud504\ub7ad\ud06c \ucc98\uce60 \uc5ed\uc73c\ub85c \ub4f1\uc7a5\ud588\ub2e4.", "paragraph_answer": "\u300a\uc2a4\uce7c\ub9bf \uc564 \ube14\ub799\u300b\uc744 \ucd2c\uc601\ud558\uba74\uc11c \ub9e5\uadf8\ub9ac\uac70\ub294 \ub2f9\uc2dc\uc5d0 \uc0c8\ub0b4\uae30 \uac10\ub3c5\uc774\uc5c8\ub358 \ub300\ub2c8 \ubcf4\uc77c \uc744 \ub9cc\ub0ac\uace0, \uccab \uc601\ud654\uc5d0 \ucc38\uc5ec\ud560 \uae30\ud68c\ub97c \uc5bb\uc5c8\ub2e4. 1994\ub144, \ub300\ub2c8 \ubcf4\uc77c\uc774 \uac10\ub3c5\ud55c \uc2a4\ub9b4\ub7ec \uc601\ud654 \u300a\uc258\ub85c\uc6b0 \uadf8\ub808\uc774\ube0c\u300b\uc5d0 \ucd9c\uc5f0\ud558\uc600\ub2e4. \uc601\ud654\uc5d0\uc11c \ubcf4\uc5ec\uc900 \uc5f0\uae30\ub294 \ucc2c\uc0ac\ub97c \ubc1b\uc558\ub2e4. \ub9e5\uadf8\ub9ac\uac70\ub294 \uc5e0\ud30c\uc774\uc5b4\uc0c1\uc744 \uc218\uc0c1\ud558\uc600\uc73c\uba70, \uc601\ud654\ub294 \ud310\ud0c0\uc2a4\ud3ec\ub974\ud1a0 \uc601\ud654\uc81c\uc5d0\uc11c \ucd5c\uace0\uc758 \uc601\ud654\ub85c \uc120\uc815\ub418\uc5c8\ub2e4. 1996\ub144, \ub300\ub2c8 \ubcf4\uc77c\uc758 \u300a\ud2b8\ub808\uc778\uc2a4\ud3ec\ud305\u300b\uc5d0 \ub9c8\ud06c \ub80c\ud134 \uc5ed\ud560\ub85c \ucd9c\uc5f0\ud558\uc600\uc73c\uba70, \uc601\ud654\ub294 \uc5b4\ubc14\uc778 \uc6f0\uc2dc\uc758 \ub3d9\uba85\uc758 \uc18c\uc124\uc744 \ubc14\ud0d5\uc73c\ub85c \ud558\uace0 \uc788\ub2e4. \uc774 \uc601\ud654\uc758 \ucd9c\uc5f0\uc73c\ub85c \ub9e5\uadf8\ub9ac\uac70\ub294 BAFTA \uc2a4\ucf54\ud2c0\ub79c\ub4dc\uc0c1\uc5d0\uc11c \ucd5c\uc6b0\uc218 \ub0a8\uc790 \ubc30\uc6b0\uc0c1\uc744 \uc218\uc0c1\ud558\uc600\uace0, \uc138\uacc4\uc801\uc778 \uc2a4\ud0c0\ub85c \ubc1c\ub3cb\uc6c0\ud558\uac8c \ub418\ub294 \uacc4\uae30\uac00 \ub418\uc5c8\ub2e4. \uac19\uc740 \ud574, \ub9e5\uadf8\ub9ac\uac70\ub294 \u300a\ud544\ub85c\uc6b0 \ubd81\u300b\uc5d0 \uc5f0\uc778\uc744 \ucc3e\uc544 \ubc29\ud669\ud558\ub358 \uc911 \ubc88\uc5ed\uac00 \uc81c\ub86c\uc744 \uc5f0\uae30\ud558\uc600\ub2e4. \u300a\uc5e0\ub9c8\u300b\uc5d0\ub3c4 \ud504\ub7ad\ud06c \ucc98\uce60 \uc5ed\uc73c\ub85c \ub4f1\uc7a5\ud588\ub2e4.", "sentence_answer": "\u300a\uc2a4\uce7c\ub9bf \uc564 \ube14\ub799\u300b\uc744 \ucd2c\uc601\ud558\uba74\uc11c \ub9e5\uadf8\ub9ac\uac70\ub294 \ub2f9\uc2dc\uc5d0 \uc0c8\ub0b4\uae30 \uac10\ub3c5\uc774\uc5c8\ub358 \ub300\ub2c8 \ubcf4\uc77c \uc744 \ub9cc\ub0ac\uace0, \uccab \uc601\ud654\uc5d0 \ucc38\uc5ec\ud560 \uae30\ud68c\ub97c \uc5bb\uc5c8\ub2e4.", "paragraph_id": "6545823-2-0", "paragraph_question": "question: \ub9e5\uadf8\ub9ac\uac70\uac00 \ucd9c\uc5f0\ud55c 1994\ub144 \uc601\ud654 \uc258\ub85c\uc6b0 \uadf8\ub808\uc774\ube0c\uc758 \uac10\ub3c5\uc740 \ub204\uad6c\uc778\uac00?, context: \u300a\uc2a4\uce7c\ub9bf \uc564 \ube14\ub799\u300b\uc744 \ucd2c\uc601\ud558\uba74\uc11c \ub9e5\uadf8\ub9ac\uac70\ub294 \ub2f9\uc2dc\uc5d0 \uc0c8\ub0b4\uae30 \uac10\ub3c5\uc774\uc5c8\ub358 \ub300\ub2c8 \ubcf4\uc77c\uc744 \ub9cc\ub0ac\uace0, \uccab \uc601\ud654\uc5d0 \ucc38\uc5ec\ud560 \uae30\ud68c\ub97c \uc5bb\uc5c8\ub2e4. 1994\ub144, \ub300\ub2c8 \ubcf4\uc77c\uc774 \uac10\ub3c5\ud55c \uc2a4\ub9b4\ub7ec \uc601\ud654 \u300a\uc258\ub85c\uc6b0 \uadf8\ub808\uc774\ube0c\u300b\uc5d0 \ucd9c\uc5f0\ud558\uc600\ub2e4. \uc601\ud654\uc5d0\uc11c \ubcf4\uc5ec\uc900 \uc5f0\uae30\ub294 \ucc2c\uc0ac\ub97c \ubc1b\uc558\ub2e4. \ub9e5\uadf8\ub9ac\uac70\ub294 \uc5e0\ud30c\uc774\uc5b4\uc0c1\uc744 \uc218\uc0c1\ud558\uc600\uc73c\uba70, \uc601\ud654\ub294 \ud310\ud0c0\uc2a4\ud3ec\ub974\ud1a0 \uc601\ud654\uc81c\uc5d0\uc11c \ucd5c\uace0\uc758 \uc601\ud654\ub85c \uc120\uc815\ub418\uc5c8\ub2e4. 1996\ub144, \ub300\ub2c8 \ubcf4\uc77c\uc758 \u300a\ud2b8\ub808\uc778\uc2a4\ud3ec\ud305\u300b\uc5d0 \ub9c8\ud06c \ub80c\ud134 \uc5ed\ud560\ub85c \ucd9c\uc5f0\ud558\uc600\uc73c\uba70, \uc601\ud654\ub294 \uc5b4\ubc14\uc778 \uc6f0\uc2dc\uc758 \ub3d9\uba85\uc758 \uc18c\uc124\uc744 \ubc14\ud0d5\uc73c\ub85c \ud558\uace0 \uc788\ub2e4. \uc774 \uc601\ud654\uc758 \ucd9c\uc5f0\uc73c\ub85c \ub9e5\uadf8\ub9ac\uac70\ub294 BAFTA \uc2a4\ucf54\ud2c0\ub79c\ub4dc\uc0c1\uc5d0\uc11c \ucd5c\uc6b0\uc218 \ub0a8\uc790 \ubc30\uc6b0\uc0c1\uc744 \uc218\uc0c1\ud558\uc600\uace0, \uc138\uacc4\uc801\uc778 \uc2a4\ud0c0\ub85c \ubc1c\ub3cb\uc6c0\ud558\uac8c \ub418\ub294 \uacc4\uae30\uac00 \ub418\uc5c8\ub2e4. \uac19\uc740 \ud574, \ub9e5\uadf8\ub9ac\uac70\ub294 \u300a\ud544\ub85c\uc6b0 \ubd81\u300b\uc5d0 \uc5f0\uc778\uc744 \ucc3e\uc544 \ubc29\ud669\ud558\ub358 \uc911 \ubc88\uc5ed\uac00 \uc81c\ub86c\uc744 \uc5f0\uae30\ud558\uc600\ub2e4. \u300a\uc5e0\ub9c8\u300b\uc5d0\ub3c4 \ud504\ub7ad\ud06c \ucc98\uce60 \uc5ed\uc73c\ub85c \ub4f1\uc7a5\ud588\ub2e4."} {"question": "\uc288\ud2b8\ub77c\uc6b0\uc2a4\uac00 1887\ub144 \uc774\ud6c4 \uc8fc\ub85c \uad00\uc2ec\uc744 \uac00\uc84c\ub358 \uc74c\uc545\uc758 \ubd84\uc57c\ub294?", "paragraph": "\uc288\ud2b8\ub77c\uc6b0\uc2a4\ub294 \ubc14\uc774\uc62c\ub9b0\uc744 \uc0c1\ub2f9\ud788 \uc798 \uc5f0\uc8fc\ud588\uc73c\uba70, \ud53c\uc544\ub178\uc758 \uc5f0\uc8fc \ud14c\ud06c\ub2c9 \ub610\ud55c \ub6f0\uc5b4\ub0ac\ub2e4. \uadf8\ub7fc\uc5d0\ub3c4 \ubd88\uad6c\ud558\uace0 \uc288\ud2b8\ub77c\uc6b0\uc2a4\ub294 \ubc14\uc774\uc62c\ub9b0 \uc18c\ub098\ud0c0\ub97c 1\uace1\ub9cc \ub0a8\uae30\uace0 \uc788\ub2e4. \uc774\uac83\uc740 1887\ub144 \uc5ec\ub984\uacfc \uac00\uc744\uc5d0 \ub300\ubd80\ubd84 \uc791\uace1\ub418\uc5b4, \ub2e4\uc74c\ud574 9\uc6d4\uc5d0 2\uc545\uc7a5\uc774 \uc644\uc131\ub418\uc5b4 \uc804\uccb4\uac00 \uc644\uc131\ub418\uc5c8\ub2e4. \uadf8\ub9ac\uace0 \uc774 \uc18c\ub098\ud0c0\ub294 \uc288\ud2b8\ub77c\uc6b0\uc2a4\uc758 \uc2e4\ub0b4\uc545\uace1 \ubd84\uc57c\uc5d0\uc11c\uc758 \ub9e4\ub4ed\uc774 \ub418\uc5c8\ub2e4. \uc989 \uc774 \uc774\ud6c4 \uc288\ud2b8\ub77c\uc6b0\uc2a4\ub294 \uace0\uc804\ud30c\uc801\uc778 \ud615\uc2dd\uc758 \uc808\ub300\uc74c\uc545\uc5d0\uc11c \ubc97\uc5b4\ub098 \uad50\ud5a5\uc2dc\uc640 \uc624\ud398\ub77c\uc758 \uc138\uacc4\uc5d0 \ubab0\uc785\ud574 \uac04\ub2e4. \uadf8\ub807\ub2e4\uace0\ub294 \ud558\ub098 \uc774 \ubc14\uc774\uc62c\ub9b0 \uc18c\ub098\ud0c0\ubcf4\ub2e4 \uc774\uc804\uc5d0 \uad50\ud5a5\uc2dc\uc758 \ud3ec\uc11d\uc774 \ub418\ub294 \uc774\ud0c8\ub9ac\uc544\uc5d0\uc11c\ub97c \uc774\ubbf8 \uc791\uace1\ud588\uace0, 1887\ub144\uc5d0\ub294 \uad50\ud5a5\uc2dc \u2018\ub9e5\ubca0\ub4dc\u2019\ub97c \uc4f0\uae30 \uc2dc\uc791\ud588\uc73c\uba70, 1888\ub144\uc5d0\ub294 \uad50\ud5a5\uc2dc \u2018\ub3c8 \ud6c4\uc548\u2019\uc758 \uc791\uace1\uc5d0 \ucc29\uc218\ud588\ub2e4. \uc288\ud2b8\ub77c\uc6b0\uc2a4\ub294 \uc7ac\ube68\ub9ac \uc0c8\ub85c\uc6b4 \ubc29\ud5a5\uc744 \uac1c\ucc99\ud558\uba70 \uc804\uc9c4\ud588\ub2e4. \ub530\ub9ac\uc11c \uc774 \uc18c\ub098\ud0c0\ub294 \uc288\ud2b8\ub77c\uc6b0\uc2a4\uc5d0\uac8c \uc788\uc5b4\uc11c \uacfc\ub3c4\uc801\uc778 \ucd5c\ud6c4\uc758 \uc791\ud488\uc774 \ub41c\ub2e4. \uadf8\ub7f0 \uae4c\ub2ed\uc5d0 \uace1\uc740 \uc804\ud1b5\uc801\uc778 3\uc545\uc7a5\uc81c\ub97c \ucde8\ud558\uace1 \uc788\uc9c0\ub9cc, \uc774\uc81c\uae4c\uc9c0\uc758 \uc288\ud2b8\ub77c\uc6b0\uc2a4\uc758 \uc2e4\ub0b4\uc545\uace1\uacfc \ud611\uc8fc\uace1\uacfc\ub294 \uc0c1\ub2f9\ud788 \ub2e4\ub978 \uc591\uc0c1\uc744 \ubcf4\uc778\ub2e4. \uc608\ub97c \ub4e4\uba74 \uc120\uc728\uc758 \uc6c0\uc9c1\uc784\uc740 \uc288\ud2b8\ub77c\uc6b0\uc2a4\uc758 \uac1c\uc131\uc744 \ud655\uc2e4\ud788 \ub098\ud0c0\ub0b4\uace0 \uc788\ub2e4. 1\uc545\uc7a5 \uc11c\ub450\uc758 \uc8fc\uc81c\uc5d0\uc11c \uc774\ubbf8 \uc288\ub9cc\uc801\uc774\ub098 \ube0c\ub78c\uc2a4\ud48d\uc73c\ub85c\ub294 \ud615\uc6a9\ud560 \uc218 \uc5c6\ub294 \uac83\uc73c\ub85c \ub418\uc5b4 \uc788\ub2e4. \uadf8\ub9ac\uace0 \uc774 \uc81c1\uc8fc\uc81c \ubd80\ubd84\ub9cc\uc73c\ub85c\ub3c4 \ub9d0\ud560 \uc218 \uc788\ub4ef\uc774 \ud654\uc131\uc740 \uc544\uc8fc \ub3c5\ucc3d\uc801\uc774\uba70, \ub300\uc704\ubc95\ub3c4 \ucc3d\uc758\uc5d0 \uac00\ub4dd \ucc28\uc11c \ud48d\ubd80\ud558\uac8c \uc4f0\uc774\uace0 \uc788\ub2e4. \ub3d9\uae30\uc758 \ucc98\uc640 \ubc1c\uc804\uacfc \uad00\ub828\uc5d0\ub3c4 \uad50\ubb18\ud568\uc774 \ub098\ud0c0\ub098\uace0 \uc788\uace0, \uac01 \uc545\uc7a5\uc740 \ub300\ube44\uc131\uc5d0 \ubc30\ub824\ud558\uba74\uc11c \uae34\ubc00\ud55c \uacb0\ub9d0\uc744 \ubcf4\uc778\ub2e4. \uac70\uae30\uc5d0\ub2e4\uac00 \ubc14\uc774\uc62c\ub9b0\ub3c4 \ud53c\uc544\ub178\ub3c4 \ud655\uc2e4\ud788 \uc288\ud2b8\ub77c\uc6b0\uc2a4\uac00 \uc88b\uc544\ud558\ub294 \uc0c1\ub2f9\ud788 \uace0\ub3c4\uc758 \uc5f0\uc8fc\uae30\uad50\ub97c \uc694\uad6c\ud55c\ub2e4. \uadf8 \ub54c\ubb38\uc5d0 \ub54c\ub85c\ub294 2\uc911\uc8fc \uc601\uc5ed\uc744 \ub118\ub294 \ud6a8\uacfc\uc758 \uc778\uc0c1\uc744 \uc900\ub2e4. \uc774\ucc98\ub7fc \uc774 \uace1\uc740 \uace0\uc804\ud30c\ub85c\ubd80\ud130\uc758 2\uc911\uc8fc \uc18c\ub098\ud0c0\uc758 \ub178\uc120\uc744 \uc288\ud2b8\ub77c\uc6b0\uc2a4 \ub098\ub984\ub300\ub85c \ud55c\uacc4\uc5d0\uae4c\uc9c0 \uc804\uc9c4\ud558\uac8c \ud55c \uac83\uc774 \ub77c\uace0 \ud560 \uc218 \uc788\ub2e4.", "answer": "\uad50\ud5a5\uc2dc\uc640 \uc624\ud398\ub77c", "sentence": "\uc989 \uc774 \uc774\ud6c4 \uc288\ud2b8\ub77c\uc6b0\uc2a4\ub294 \uace0\uc804\ud30c\uc801\uc778 \ud615\uc2dd\uc758 \uc808\ub300\uc74c\uc545\uc5d0\uc11c \ubc97\uc5b4\ub098 \uad50\ud5a5\uc2dc\uc640 \uc624\ud398\ub77c \uc758 \uc138\uacc4\uc5d0 \ubab0\uc785\ud574 \uac04\ub2e4.", "paragraph_sentence": "\uc288\ud2b8\ub77c\uc6b0\uc2a4\ub294 \ubc14\uc774\uc62c\ub9b0\uc744 \uc0c1\ub2f9\ud788 \uc798 \uc5f0\uc8fc\ud588\uc73c\uba70, \ud53c\uc544\ub178\uc758 \uc5f0\uc8fc \ud14c\ud06c\ub2c9 \ub610\ud55c \ub6f0\uc5b4\ub0ac\ub2e4. \uadf8\ub7fc\uc5d0\ub3c4 \ubd88\uad6c\ud558\uace0 \uc288\ud2b8\ub77c\uc6b0\uc2a4\ub294 \ubc14\uc774\uc62c\ub9b0 \uc18c\ub098\ud0c0\ub97c 1\uace1\ub9cc \ub0a8\uae30\uace0 \uc788\ub2e4. \uc774\uac83\uc740 1887\ub144 \uc5ec\ub984\uacfc \uac00\uc744\uc5d0 \ub300\ubd80\ubd84 \uc791\uace1\ub418\uc5b4, \ub2e4\uc74c\ud574 9\uc6d4\uc5d0 2\uc545\uc7a5\uc774 \uc644\uc131\ub418\uc5b4 \uc804\uccb4\uac00 \uc644\uc131\ub418\uc5c8\ub2e4. \uadf8\ub9ac\uace0 \uc774 \uc18c\ub098\ud0c0\ub294 \uc288\ud2b8\ub77c\uc6b0\uc2a4\uc758 \uc2e4\ub0b4\uc545\uace1 \ubd84\uc57c\uc5d0\uc11c\uc758 \ub9e4\ub4ed\uc774 \ub418\uc5c8\ub2e4. \uc989 \uc774 \uc774\ud6c4 \uc288\ud2b8\ub77c\uc6b0\uc2a4\ub294 \uace0\uc804\ud30c\uc801\uc778 \ud615\uc2dd\uc758 \uc808\ub300\uc74c\uc545\uc5d0\uc11c \ubc97\uc5b4\ub098 \uad50\ud5a5\uc2dc\uc640 \uc624\ud398\ub77c \uc758 \uc138\uacc4\uc5d0 \ubab0\uc785\ud574 \uac04\ub2e4. \uadf8\ub807\ub2e4\uace0\ub294 \ud558\ub098 \uc774 \ubc14\uc774\uc62c\ub9b0 \uc18c\ub098\ud0c0\ubcf4\ub2e4 \uc774\uc804\uc5d0 \uad50\ud5a5\uc2dc\uc758 \ud3ec\uc11d\uc774 \ub418\ub294 \uc774\ud0c8\ub9ac\uc544\uc5d0\uc11c\ub97c \uc774\ubbf8 \uc791\uace1\ud588\uace0, 1887\ub144\uc5d0\ub294 \uad50\ud5a5\uc2dc \u2018\ub9e5\ubca0\ub4dc\u2019\ub97c \uc4f0\uae30 \uc2dc\uc791\ud588\uc73c\uba70, 1888\ub144\uc5d0\ub294 \uad50\ud5a5\uc2dc \u2018\ub3c8 \ud6c4\uc548\u2019\uc758 \uc791\uace1\uc5d0 \ucc29\uc218\ud588\ub2e4. \uc288\ud2b8\ub77c\uc6b0\uc2a4\ub294 \uc7ac\ube68\ub9ac \uc0c8\ub85c\uc6b4 \ubc29\ud5a5\uc744 \uac1c\ucc99\ud558\uba70 \uc804\uc9c4\ud588\ub2e4. \ub530\ub9ac\uc11c \uc774 \uc18c\ub098\ud0c0\ub294 \uc288\ud2b8\ub77c\uc6b0\uc2a4\uc5d0\uac8c \uc788\uc5b4\uc11c \uacfc\ub3c4\uc801\uc778 \ucd5c\ud6c4\uc758 \uc791\ud488\uc774 \ub41c\ub2e4. \uadf8\ub7f0 \uae4c\ub2ed\uc5d0 \uace1\uc740 \uc804\ud1b5\uc801\uc778 3\uc545\uc7a5\uc81c\ub97c \ucde8\ud558\uace1 \uc788\uc9c0\ub9cc, \uc774\uc81c\uae4c\uc9c0\uc758 \uc288\ud2b8\ub77c\uc6b0\uc2a4\uc758 \uc2e4\ub0b4\uc545\uace1\uacfc \ud611\uc8fc\uace1\uacfc\ub294 \uc0c1\ub2f9\ud788 \ub2e4\ub978 \uc591\uc0c1\uc744 \ubcf4\uc778\ub2e4. \uc608\ub97c \ub4e4\uba74 \uc120\uc728\uc758 \uc6c0\uc9c1\uc784\uc740 \uc288\ud2b8\ub77c\uc6b0\uc2a4\uc758 \uac1c\uc131\uc744 \ud655\uc2e4\ud788 \ub098\ud0c0\ub0b4\uace0 \uc788\ub2e4. 1\uc545\uc7a5 \uc11c\ub450\uc758 \uc8fc\uc81c\uc5d0\uc11c \uc774\ubbf8 \uc288\ub9cc\uc801\uc774\ub098 \ube0c\ub78c\uc2a4\ud48d\uc73c\ub85c\ub294 \ud615\uc6a9\ud560 \uc218 \uc5c6\ub294 \uac83\uc73c\ub85c \ub418\uc5b4 \uc788\ub2e4. \uadf8\ub9ac\uace0 \uc774 \uc81c1\uc8fc\uc81c \ubd80\ubd84\ub9cc\uc73c\ub85c\ub3c4 \ub9d0\ud560 \uc218 \uc788\ub4ef\uc774 \ud654\uc131\uc740 \uc544\uc8fc \ub3c5\ucc3d\uc801\uc774\uba70, \ub300\uc704\ubc95\ub3c4 \ucc3d\uc758\uc5d0 \uac00\ub4dd \ucc28\uc11c \ud48d\ubd80\ud558\uac8c \uc4f0\uc774\uace0 \uc788\ub2e4. \ub3d9\uae30\uc758 \ucc98\uc640 \ubc1c\uc804\uacfc \uad00\ub828\uc5d0\ub3c4 \uad50\ubb18\ud568\uc774 \ub098\ud0c0\ub098\uace0 \uc788\uace0, \uac01 \uc545\uc7a5\uc740 \ub300\ube44\uc131\uc5d0 \ubc30\ub824\ud558\uba74\uc11c \uae34\ubc00\ud55c \uacb0\ub9d0\uc744 \ubcf4\uc778\ub2e4. \uac70\uae30\uc5d0\ub2e4\uac00 \ubc14\uc774\uc62c\ub9b0\ub3c4 \ud53c\uc544\ub178\ub3c4 \ud655\uc2e4\ud788 \uc288\ud2b8\ub77c\uc6b0\uc2a4\uac00 \uc88b\uc544\ud558\ub294 \uc0c1\ub2f9\ud788 \uace0\ub3c4\uc758 \uc5f0\uc8fc\uae30\uad50\ub97c \uc694\uad6c\ud55c\ub2e4. \uadf8 \ub54c\ubb38\uc5d0 \ub54c\ub85c\ub294 2\uc911\uc8fc \uc601\uc5ed\uc744 \ub118\ub294 \ud6a8\uacfc\uc758 \uc778\uc0c1\uc744 \uc900\ub2e4. \uc774\ucc98\ub7fc \uc774 \uace1\uc740 \uace0\uc804\ud30c\ub85c\ubd80\ud130\uc758 2\uc911\uc8fc \uc18c\ub098\ud0c0\uc758 \ub178\uc120\uc744 \uc288\ud2b8\ub77c\uc6b0\uc2a4 \ub098\ub984\ub300\ub85c \ud55c\uacc4\uc5d0\uae4c\uc9c0 \uc804\uc9c4\ud558\uac8c \ud55c \uac83\uc774 \ub77c\uace0 \ud560 \uc218 \uc788\ub2e4.", "paragraph_answer": "\uc288\ud2b8\ub77c\uc6b0\uc2a4\ub294 \ubc14\uc774\uc62c\ub9b0\uc744 \uc0c1\ub2f9\ud788 \uc798 \uc5f0\uc8fc\ud588\uc73c\uba70, \ud53c\uc544\ub178\uc758 \uc5f0\uc8fc \ud14c\ud06c\ub2c9 \ub610\ud55c \ub6f0\uc5b4\ub0ac\ub2e4. \uadf8\ub7fc\uc5d0\ub3c4 \ubd88\uad6c\ud558\uace0 \uc288\ud2b8\ub77c\uc6b0\uc2a4\ub294 \ubc14\uc774\uc62c\ub9b0 \uc18c\ub098\ud0c0\ub97c 1\uace1\ub9cc \ub0a8\uae30\uace0 \uc788\ub2e4. \uc774\uac83\uc740 1887\ub144 \uc5ec\ub984\uacfc \uac00\uc744\uc5d0 \ub300\ubd80\ubd84 \uc791\uace1\ub418\uc5b4, \ub2e4\uc74c\ud574 9\uc6d4\uc5d0 2\uc545\uc7a5\uc774 \uc644\uc131\ub418\uc5b4 \uc804\uccb4\uac00 \uc644\uc131\ub418\uc5c8\ub2e4. \uadf8\ub9ac\uace0 \uc774 \uc18c\ub098\ud0c0\ub294 \uc288\ud2b8\ub77c\uc6b0\uc2a4\uc758 \uc2e4\ub0b4\uc545\uace1 \ubd84\uc57c\uc5d0\uc11c\uc758 \ub9e4\ub4ed\uc774 \ub418\uc5c8\ub2e4. \uc989 \uc774 \uc774\ud6c4 \uc288\ud2b8\ub77c\uc6b0\uc2a4\ub294 \uace0\uc804\ud30c\uc801\uc778 \ud615\uc2dd\uc758 \uc808\ub300\uc74c\uc545\uc5d0\uc11c \ubc97\uc5b4\ub098 \uad50\ud5a5\uc2dc\uc640 \uc624\ud398\ub77c \uc758 \uc138\uacc4\uc5d0 \ubab0\uc785\ud574 \uac04\ub2e4. \uadf8\ub807\ub2e4\uace0\ub294 \ud558\ub098 \uc774 \ubc14\uc774\uc62c\ub9b0 \uc18c\ub098\ud0c0\ubcf4\ub2e4 \uc774\uc804\uc5d0 \uad50\ud5a5\uc2dc\uc758 \ud3ec\uc11d\uc774 \ub418\ub294 \uc774\ud0c8\ub9ac\uc544\uc5d0\uc11c\ub97c \uc774\ubbf8 \uc791\uace1\ud588\uace0, 1887\ub144\uc5d0\ub294 \uad50\ud5a5\uc2dc \u2018\ub9e5\ubca0\ub4dc\u2019\ub97c \uc4f0\uae30 \uc2dc\uc791\ud588\uc73c\uba70, 1888\ub144\uc5d0\ub294 \uad50\ud5a5\uc2dc \u2018\ub3c8 \ud6c4\uc548\u2019\uc758 \uc791\uace1\uc5d0 \ucc29\uc218\ud588\ub2e4. \uc288\ud2b8\ub77c\uc6b0\uc2a4\ub294 \uc7ac\ube68\ub9ac \uc0c8\ub85c\uc6b4 \ubc29\ud5a5\uc744 \uac1c\ucc99\ud558\uba70 \uc804\uc9c4\ud588\ub2e4. \ub530\ub9ac\uc11c \uc774 \uc18c\ub098\ud0c0\ub294 \uc288\ud2b8\ub77c\uc6b0\uc2a4\uc5d0\uac8c \uc788\uc5b4\uc11c \uacfc\ub3c4\uc801\uc778 \ucd5c\ud6c4\uc758 \uc791\ud488\uc774 \ub41c\ub2e4. \uadf8\ub7f0 \uae4c\ub2ed\uc5d0 \uace1\uc740 \uc804\ud1b5\uc801\uc778 3\uc545\uc7a5\uc81c\ub97c \ucde8\ud558\uace1 \uc788\uc9c0\ub9cc, \uc774\uc81c\uae4c\uc9c0\uc758 \uc288\ud2b8\ub77c\uc6b0\uc2a4\uc758 \uc2e4\ub0b4\uc545\uace1\uacfc \ud611\uc8fc\uace1\uacfc\ub294 \uc0c1\ub2f9\ud788 \ub2e4\ub978 \uc591\uc0c1\uc744 \ubcf4\uc778\ub2e4. \uc608\ub97c \ub4e4\uba74 \uc120\uc728\uc758 \uc6c0\uc9c1\uc784\uc740 \uc288\ud2b8\ub77c\uc6b0\uc2a4\uc758 \uac1c\uc131\uc744 \ud655\uc2e4\ud788 \ub098\ud0c0\ub0b4\uace0 \uc788\ub2e4. 1\uc545\uc7a5 \uc11c\ub450\uc758 \uc8fc\uc81c\uc5d0\uc11c \uc774\ubbf8 \uc288\ub9cc\uc801\uc774\ub098 \ube0c\ub78c\uc2a4\ud48d\uc73c\ub85c\ub294 \ud615\uc6a9\ud560 \uc218 \uc5c6\ub294 \uac83\uc73c\ub85c \ub418\uc5b4 \uc788\ub2e4. \uadf8\ub9ac\uace0 \uc774 \uc81c1\uc8fc\uc81c \ubd80\ubd84\ub9cc\uc73c\ub85c\ub3c4 \ub9d0\ud560 \uc218 \uc788\ub4ef\uc774 \ud654\uc131\uc740 \uc544\uc8fc \ub3c5\ucc3d\uc801\uc774\uba70, \ub300\uc704\ubc95\ub3c4 \ucc3d\uc758\uc5d0 \uac00\ub4dd \ucc28\uc11c \ud48d\ubd80\ud558\uac8c \uc4f0\uc774\uace0 \uc788\ub2e4. \ub3d9\uae30\uc758 \ucc98\uc640 \ubc1c\uc804\uacfc \uad00\ub828\uc5d0\ub3c4 \uad50\ubb18\ud568\uc774 \ub098\ud0c0\ub098\uace0 \uc788\uace0, \uac01 \uc545\uc7a5\uc740 \ub300\ube44\uc131\uc5d0 \ubc30\ub824\ud558\uba74\uc11c \uae34\ubc00\ud55c \uacb0\ub9d0\uc744 \ubcf4\uc778\ub2e4. \uac70\uae30\uc5d0\ub2e4\uac00 \ubc14\uc774\uc62c\ub9b0\ub3c4 \ud53c\uc544\ub178\ub3c4 \ud655\uc2e4\ud788 \uc288\ud2b8\ub77c\uc6b0\uc2a4\uac00 \uc88b\uc544\ud558\ub294 \uc0c1\ub2f9\ud788 \uace0\ub3c4\uc758 \uc5f0\uc8fc\uae30\uad50\ub97c \uc694\uad6c\ud55c\ub2e4. \uadf8 \ub54c\ubb38\uc5d0 \ub54c\ub85c\ub294 2\uc911\uc8fc \uc601\uc5ed\uc744 \ub118\ub294 \ud6a8\uacfc\uc758 \uc778\uc0c1\uc744 \uc900\ub2e4. \uc774\ucc98\ub7fc \uc774 \uace1\uc740 \uace0\uc804\ud30c\ub85c\ubd80\ud130\uc758 2\uc911\uc8fc \uc18c\ub098\ud0c0\uc758 \ub178\uc120\uc744 \uc288\ud2b8\ub77c\uc6b0\uc2a4 \ub098\ub984\ub300\ub85c \ud55c\uacc4\uc5d0\uae4c\uc9c0 \uc804\uc9c4\ud558\uac8c \ud55c \uac83\uc774 \ub77c\uace0 \ud560 \uc218 \uc788\ub2e4.", "sentence_answer": "\uc989 \uc774 \uc774\ud6c4 \uc288\ud2b8\ub77c\uc6b0\uc2a4\ub294 \uace0\uc804\ud30c\uc801\uc778 \ud615\uc2dd\uc758 \uc808\ub300\uc74c\uc545\uc5d0\uc11c \ubc97\uc5b4\ub098 \uad50\ud5a5\uc2dc\uc640 \uc624\ud398\ub77c \uc758 \uc138\uacc4\uc5d0 \ubab0\uc785\ud574 \uac04\ub2e4.", "paragraph_id": "6530058-0-1", "paragraph_question": "question: \uc288\ud2b8\ub77c\uc6b0\uc2a4\uac00 1887\ub144 \uc774\ud6c4 \uc8fc\ub85c \uad00\uc2ec\uc744 \uac00\uc84c\ub358 \uc74c\uc545\uc758 \ubd84\uc57c\ub294?, context: \uc288\ud2b8\ub77c\uc6b0\uc2a4\ub294 \ubc14\uc774\uc62c\ub9b0\uc744 \uc0c1\ub2f9\ud788 \uc798 \uc5f0\uc8fc\ud588\uc73c\uba70, \ud53c\uc544\ub178\uc758 \uc5f0\uc8fc \ud14c\ud06c\ub2c9 \ub610\ud55c \ub6f0\uc5b4\ub0ac\ub2e4. \uadf8\ub7fc\uc5d0\ub3c4 \ubd88\uad6c\ud558\uace0 \uc288\ud2b8\ub77c\uc6b0\uc2a4\ub294 \ubc14\uc774\uc62c\ub9b0 \uc18c\ub098\ud0c0\ub97c 1\uace1\ub9cc \ub0a8\uae30\uace0 \uc788\ub2e4. \uc774\uac83\uc740 1887\ub144 \uc5ec\ub984\uacfc \uac00\uc744\uc5d0 \ub300\ubd80\ubd84 \uc791\uace1\ub418\uc5b4, \ub2e4\uc74c\ud574 9\uc6d4\uc5d0 2\uc545\uc7a5\uc774 \uc644\uc131\ub418\uc5b4 \uc804\uccb4\uac00 \uc644\uc131\ub418\uc5c8\ub2e4. \uadf8\ub9ac\uace0 \uc774 \uc18c\ub098\ud0c0\ub294 \uc288\ud2b8\ub77c\uc6b0\uc2a4\uc758 \uc2e4\ub0b4\uc545\uace1 \ubd84\uc57c\uc5d0\uc11c\uc758 \ub9e4\ub4ed\uc774 \ub418\uc5c8\ub2e4. \uc989 \uc774 \uc774\ud6c4 \uc288\ud2b8\ub77c\uc6b0\uc2a4\ub294 \uace0\uc804\ud30c\uc801\uc778 \ud615\uc2dd\uc758 \uc808\ub300\uc74c\uc545\uc5d0\uc11c \ubc97\uc5b4\ub098 \uad50\ud5a5\uc2dc\uc640 \uc624\ud398\ub77c\uc758 \uc138\uacc4\uc5d0 \ubab0\uc785\ud574 \uac04\ub2e4. \uadf8\ub807\ub2e4\uace0\ub294 \ud558\ub098 \uc774 \ubc14\uc774\uc62c\ub9b0 \uc18c\ub098\ud0c0\ubcf4\ub2e4 \uc774\uc804\uc5d0 \uad50\ud5a5\uc2dc\uc758 \ud3ec\uc11d\uc774 \ub418\ub294 \uc774\ud0c8\ub9ac\uc544\uc5d0\uc11c\ub97c \uc774\ubbf8 \uc791\uace1\ud588\uace0, 1887\ub144\uc5d0\ub294 \uad50\ud5a5\uc2dc \u2018\ub9e5\ubca0\ub4dc\u2019\ub97c \uc4f0\uae30 \uc2dc\uc791\ud588\uc73c\uba70, 1888\ub144\uc5d0\ub294 \uad50\ud5a5\uc2dc \u2018\ub3c8 \ud6c4\uc548\u2019\uc758 \uc791\uace1\uc5d0 \ucc29\uc218\ud588\ub2e4. \uc288\ud2b8\ub77c\uc6b0\uc2a4\ub294 \uc7ac\ube68\ub9ac \uc0c8\ub85c\uc6b4 \ubc29\ud5a5\uc744 \uac1c\ucc99\ud558\uba70 \uc804\uc9c4\ud588\ub2e4. \ub530\ub9ac\uc11c \uc774 \uc18c\ub098\ud0c0\ub294 \uc288\ud2b8\ub77c\uc6b0\uc2a4\uc5d0\uac8c \uc788\uc5b4\uc11c \uacfc\ub3c4\uc801\uc778 \ucd5c\ud6c4\uc758 \uc791\ud488\uc774 \ub41c\ub2e4. \uadf8\ub7f0 \uae4c\ub2ed\uc5d0 \uace1\uc740 \uc804\ud1b5\uc801\uc778 3\uc545\uc7a5\uc81c\ub97c \ucde8\ud558\uace1 \uc788\uc9c0\ub9cc, \uc774\uc81c\uae4c\uc9c0\uc758 \uc288\ud2b8\ub77c\uc6b0\uc2a4\uc758 \uc2e4\ub0b4\uc545\uace1\uacfc \ud611\uc8fc\uace1\uacfc\ub294 \uc0c1\ub2f9\ud788 \ub2e4\ub978 \uc591\uc0c1\uc744 \ubcf4\uc778\ub2e4. \uc608\ub97c \ub4e4\uba74 \uc120\uc728\uc758 \uc6c0\uc9c1\uc784\uc740 \uc288\ud2b8\ub77c\uc6b0\uc2a4\uc758 \uac1c\uc131\uc744 \ud655\uc2e4\ud788 \ub098\ud0c0\ub0b4\uace0 \uc788\ub2e4. 1\uc545\uc7a5 \uc11c\ub450\uc758 \uc8fc\uc81c\uc5d0\uc11c \uc774\ubbf8 \uc288\ub9cc\uc801\uc774\ub098 \ube0c\ub78c\uc2a4\ud48d\uc73c\ub85c\ub294 \ud615\uc6a9\ud560 \uc218 \uc5c6\ub294 \uac83\uc73c\ub85c \ub418\uc5b4 \uc788\ub2e4. \uadf8\ub9ac\uace0 \uc774 \uc81c1\uc8fc\uc81c \ubd80\ubd84\ub9cc\uc73c\ub85c\ub3c4 \ub9d0\ud560 \uc218 \uc788\ub4ef\uc774 \ud654\uc131\uc740 \uc544\uc8fc \ub3c5\ucc3d\uc801\uc774\uba70, \ub300\uc704\ubc95\ub3c4 \ucc3d\uc758\uc5d0 \uac00\ub4dd \ucc28\uc11c \ud48d\ubd80\ud558\uac8c \uc4f0\uc774\uace0 \uc788\ub2e4. \ub3d9\uae30\uc758 \ucc98\uc640 \ubc1c\uc804\uacfc \uad00\ub828\uc5d0\ub3c4 \uad50\ubb18\ud568\uc774 \ub098\ud0c0\ub098\uace0 \uc788\uace0, \uac01 \uc545\uc7a5\uc740 \ub300\ube44\uc131\uc5d0 \ubc30\ub824\ud558\uba74\uc11c \uae34\ubc00\ud55c \uacb0\ub9d0\uc744 \ubcf4\uc778\ub2e4. \uac70\uae30\uc5d0\ub2e4\uac00 \ubc14\uc774\uc62c\ub9b0\ub3c4 \ud53c\uc544\ub178\ub3c4 \ud655\uc2e4\ud788 \uc288\ud2b8\ub77c\uc6b0\uc2a4\uac00 \uc88b\uc544\ud558\ub294 \uc0c1\ub2f9\ud788 \uace0\ub3c4\uc758 \uc5f0\uc8fc\uae30\uad50\ub97c \uc694\uad6c\ud55c\ub2e4. \uadf8 \ub54c\ubb38\uc5d0 \ub54c\ub85c\ub294 2\uc911\uc8fc \uc601\uc5ed\uc744 \ub118\ub294 \ud6a8\uacfc\uc758 \uc778\uc0c1\uc744 \uc900\ub2e4. \uc774\ucc98\ub7fc \uc774 \uace1\uc740 \uace0\uc804\ud30c\ub85c\ubd80\ud130\uc758 2\uc911\uc8fc \uc18c\ub098\ud0c0\uc758 \ub178\uc120\uc744 \uc288\ud2b8\ub77c\uc6b0\uc2a4 \ub098\ub984\ub300\ub85c \ud55c\uacc4\uc5d0\uae4c\uc9c0 \uc804\uc9c4\ud558\uac8c \ud55c \uac83\uc774 \ub77c\uace0 \ud560 \uc218 \uc788\ub2e4."} {"question": "\uc120\uc870\uc758 \uc728\uace1 \ubcf8\uc778\uacfc \uc131\ud63c\uc744 \uacac\uc8fc\uc5c8\ub54c\ub294 \uc5b4\ub5a4\uac00\ub77c\ub294 \ubb3c\uc74c\uc5d0 \uc728\uace1\uc740 \uc5b4\ub5a4 \uba74\uc5d0\uc11c \uc790\uc2e0\uc774 \uc6b0\uacc4\ubcf4\ub2e4 \ub354 \ub0ab\ub2e4\ub77c\ub294 \ud45c\ud604\uc744 \uc37c\ub294\uac00?", "paragraph": "\uc0ac\ub9bc\uc758 \ucc9c\uac70\ub85c \uce5c\uad6c \uc131\ud63c\uc774 \uc911\ub9dd\uc5d0 \uc624\ub974\uc790 \uc120\uc870\ub294 \uc728\uace1\uc744 \ubd88\ub7ec \"\uc0ac\ub9bc\uc5d0\uc11c\ub294 \uc774 \ub09c\uc138\ub97c \uce58\uc720\ud560 \uc218 \uc788\ub294 \uc778\ubb3c\ub85c \uc6b0\uacc4\ub97c \ucc9c\uac70\ud558\ub294\ub370 \uacbd\uc758 \uc0dd\uac01\uc740 \uc5b4\ub5a4\uac00?\" \ub77c\uace0 \uadf8\uc5d0\uac8c \uc131\ud63c\uc758 \uc0ac\ub78c\ub428\uc744 \ubb3c\uc5c8\ub2e4. \uc728\uace1\uacfc \uc6b0\uacc4\ub294 \"\uc0b4\uc544\ub3c4 \uac19\uc774 \uc0b4\uace0 \uc8fd\uc5b4\ub3c4 \uac19\uc774 \uc8fd\uc790\"\ub294 \ub3d9\uc2ec\uc77c\uccb4\uc758 \uad50\uc6b0\uad00\uacc4\ub97c \uc9c0\ub2cc \uc0ac\uc774\uc600\ub2e4. \ud558\uc9c0\ub9cc \uc120\uc870\uc758 \ubb3c\uc74c\uc5d0 \uc728\uace1\uc740 \ud55c\ub9c8\ub514\ub85c \"\uc6b0\uacc4\ub294 \uadf8\ub7ec\ud55c \uc704\uc778\uc740 \ubabb \ub418\uace0 \ud559\ubb38\uc5d0 \ud798\uc4f0\ub294 \ucc29\uc2e4\ud55c \uc120\ube44\uc785\ub2c8\ub2e4\" \ub77c\uace0 \ub2f5\ubcc0\ud588\ub2e4. \ub098\ub77c\uc758 \uc5b4\ub824\uc6c0\uc744 \uac74\uc9c8 \ub9cc\ud55c \uc778\ubb3c\uc774\ub77c\uace0 \uc0ac\ub9bc\uc5d0\uc11c\uae4c\uc9c0 \ub5a0\ubc1b\ub4dc\ub294 \uc808\uce5c\ud55c \uce5c\uad6c\ub97c \ucc29\uc2e4\ud55c \uc120\ube44\uc5d0 \ubd88\uacfc\ud558\ub2e4\uace0 \ud55c \uc774 \ub2f5\ubcc0 \uc5ed\uc2dc \uc77c\uc0c1\uc758 \uc0c1\uc2dd\uc73c\ub85c\ub294 \uc774\ud574\ud558\uae30 \uc5b4\ub824\uc6b4 \ub0c9\ud639\ud55c \ud3c9\uac00\ub2e4. \uc120\uc870\uac00 \uc774\uc5b4\uc11c \uacbd\uacfc \uc6b0\uacc4\ub97c \ube44\uad50\ud558\uba74 \uc5b4\ub5a4\uac00\ub77c\uace0 \ubb3b\uc790, \uc728\uace1\uc740 \"\uc7ac\uc8fc\ub294 \uc18c\uc2e0\uc774 \uc6b0\uacc4\ubcf4\ub2e4 \uc880 \ub098\uc73c\ub098 \uc218\uc2e0\uacfc \ud559\ubb38\uc758 \ud798\uc500\uc5d0 \uc788\uc5b4\uc11c\ub294 \uc6b0\uacc4\uc5d0 \ubbf8\uce58\uc9c0 \ubabb\ud569\ub2c8\ub2e4\" \ub77c\uace0 \ub2f5\ubcc0\ud588\ub2e4. \uc728\uace1\ub2e4\uc6b4 \uc815\uc9c1\ud55c \ub2f5\ubcc0\uc774\uc5c8\ub2e4.", "answer": "\uc7ac\uc8fc", "sentence": "\uc120\uc870\uac00 \uc774\uc5b4\uc11c \uacbd\uacfc \uc6b0\uacc4\ub97c \ube44\uad50\ud558\uba74 \uc5b4\ub5a4\uac00\ub77c\uace0 \ubb3b\uc790, \uc728\uace1\uc740 \" \uc7ac\uc8fc \ub294", "paragraph_sentence": "\uc0ac\ub9bc\uc758 \ucc9c\uac70\ub85c \uce5c\uad6c \uc131\ud63c\uc774 \uc911\ub9dd\uc5d0 \uc624\ub974\uc790 \uc120\uc870\ub294 \uc728\uace1\uc744 \ubd88\ub7ec \"\uc0ac\ub9bc\uc5d0\uc11c\ub294 \uc774 \ub09c\uc138\ub97c \uce58\uc720\ud560 \uc218 \uc788\ub294 \uc778\ubb3c\ub85c \uc6b0\uacc4\ub97c \ucc9c\uac70\ud558\ub294\ub370 \uacbd\uc758 \uc0dd\uac01\uc740 \uc5b4\ub5a4\uac00?\" \ub77c\uace0 \uadf8\uc5d0\uac8c \uc131\ud63c\uc758 \uc0ac\ub78c\ub428\uc744 \ubb3c\uc5c8\ub2e4. \uc728\uace1\uacfc \uc6b0\uacc4\ub294 \"\uc0b4\uc544\ub3c4 \uac19\uc774 \uc0b4\uace0 \uc8fd\uc5b4\ub3c4 \uac19\uc774 \uc8fd\uc790\"\ub294 \ub3d9\uc2ec\uc77c\uccb4\uc758 \uad50\uc6b0\uad00\uacc4\ub97c \uc9c0\ub2cc \uc0ac\uc774\uc600\ub2e4. \ud558\uc9c0\ub9cc \uc120\uc870\uc758 \ubb3c\uc74c\uc5d0 \uc728\uace1\uc740 \ud55c\ub9c8\ub514\ub85c \"\uc6b0\uacc4\ub294 \uadf8\ub7ec\ud55c \uc704\uc778\uc740 \ubabb \ub418\uace0 \ud559\ubb38\uc5d0 \ud798\uc4f0\ub294 \ucc29\uc2e4\ud55c \uc120\ube44\uc785\ub2c8\ub2e4\" \ub77c\uace0 \ub2f5\ubcc0\ud588\ub2e4. \ub098\ub77c\uc758 \uc5b4\ub824\uc6c0\uc744 \uac74\uc9c8 \ub9cc\ud55c \uc778\ubb3c\uc774\ub77c\uace0 \uc0ac\ub9bc\uc5d0\uc11c\uae4c\uc9c0 \ub5a0\ubc1b\ub4dc\ub294 \uc808\uce5c\ud55c \uce5c\uad6c\ub97c \ucc29\uc2e4\ud55c \uc120\ube44\uc5d0 \ubd88\uacfc\ud558\ub2e4\uace0 \ud55c \uc774 \ub2f5\ubcc0 \uc5ed\uc2dc \uc77c\uc0c1\uc758 \uc0c1\uc2dd\uc73c\ub85c\ub294 \uc774\ud574\ud558\uae30 \uc5b4\ub824\uc6b4 \ub0c9\ud639\ud55c \ud3c9\uac00\ub2e4. \uc120\uc870\uac00 \uc774\uc5b4\uc11c \uacbd\uacfc \uc6b0\uacc4\ub97c \ube44\uad50\ud558\uba74 \uc5b4\ub5a4\uac00\ub77c\uace0 \ubb3b\uc790, \uc728\uace1\uc740 \" \uc7ac\uc8fc \ub294 \uc18c\uc2e0\uc774 \uc6b0\uacc4\ubcf4\ub2e4 \uc880 \ub098\uc73c\ub098 \uc218\uc2e0\uacfc \ud559\ubb38\uc758 \ud798\uc500\uc5d0 \uc788\uc5b4\uc11c\ub294 \uc6b0\uacc4\uc5d0 \ubbf8\uce58\uc9c0 \ubabb\ud569\ub2c8\ub2e4\" \ub77c\uace0 \ub2f5\ubcc0\ud588\ub2e4. \uc728\uace1\ub2e4\uc6b4 \uc815\uc9c1\ud55c \ub2f5\ubcc0\uc774\uc5c8\ub2e4.", "paragraph_answer": "\uc0ac\ub9bc\uc758 \ucc9c\uac70\ub85c \uce5c\uad6c \uc131\ud63c\uc774 \uc911\ub9dd\uc5d0 \uc624\ub974\uc790 \uc120\uc870\ub294 \uc728\uace1\uc744 \ubd88\ub7ec \"\uc0ac\ub9bc\uc5d0\uc11c\ub294 \uc774 \ub09c\uc138\ub97c \uce58\uc720\ud560 \uc218 \uc788\ub294 \uc778\ubb3c\ub85c \uc6b0\uacc4\ub97c \ucc9c\uac70\ud558\ub294\ub370 \uacbd\uc758 \uc0dd\uac01\uc740 \uc5b4\ub5a4\uac00?\" \ub77c\uace0 \uadf8\uc5d0\uac8c \uc131\ud63c\uc758 \uc0ac\ub78c\ub428\uc744 \ubb3c\uc5c8\ub2e4. \uc728\uace1\uacfc \uc6b0\uacc4\ub294 \"\uc0b4\uc544\ub3c4 \uac19\uc774 \uc0b4\uace0 \uc8fd\uc5b4\ub3c4 \uac19\uc774 \uc8fd\uc790\"\ub294 \ub3d9\uc2ec\uc77c\uccb4\uc758 \uad50\uc6b0\uad00\uacc4\ub97c \uc9c0\ub2cc \uc0ac\uc774\uc600\ub2e4. \ud558\uc9c0\ub9cc \uc120\uc870\uc758 \ubb3c\uc74c\uc5d0 \uc728\uace1\uc740 \ud55c\ub9c8\ub514\ub85c \"\uc6b0\uacc4\ub294 \uadf8\ub7ec\ud55c \uc704\uc778\uc740 \ubabb \ub418\uace0 \ud559\ubb38\uc5d0 \ud798\uc4f0\ub294 \ucc29\uc2e4\ud55c \uc120\ube44\uc785\ub2c8\ub2e4\" \ub77c\uace0 \ub2f5\ubcc0\ud588\ub2e4. \ub098\ub77c\uc758 \uc5b4\ub824\uc6c0\uc744 \uac74\uc9c8 \ub9cc\ud55c \uc778\ubb3c\uc774\ub77c\uace0 \uc0ac\ub9bc\uc5d0\uc11c\uae4c\uc9c0 \ub5a0\ubc1b\ub4dc\ub294 \uc808\uce5c\ud55c \uce5c\uad6c\ub97c \ucc29\uc2e4\ud55c \uc120\ube44\uc5d0 \ubd88\uacfc\ud558\ub2e4\uace0 \ud55c \uc774 \ub2f5\ubcc0 \uc5ed\uc2dc \uc77c\uc0c1\uc758 \uc0c1\uc2dd\uc73c\ub85c\ub294 \uc774\ud574\ud558\uae30 \uc5b4\ub824\uc6b4 \ub0c9\ud639\ud55c \ud3c9\uac00\ub2e4. \uc120\uc870\uac00 \uc774\uc5b4\uc11c \uacbd\uacfc \uc6b0\uacc4\ub97c \ube44\uad50\ud558\uba74 \uc5b4\ub5a4\uac00\ub77c\uace0 \ubb3b\uc790, \uc728\uace1\uc740 \" \uc7ac\uc8fc \ub294 \uc18c\uc2e0\uc774 \uc6b0\uacc4\ubcf4\ub2e4 \uc880 \ub098\uc73c\ub098 \uc218\uc2e0\uacfc \ud559\ubb38\uc758 \ud798\uc500\uc5d0 \uc788\uc5b4\uc11c\ub294 \uc6b0\uacc4\uc5d0 \ubbf8\uce58\uc9c0 \ubabb\ud569\ub2c8\ub2e4\" \ub77c\uace0 \ub2f5\ubcc0\ud588\ub2e4. \uc728\uace1\ub2e4\uc6b4 \uc815\uc9c1\ud55c \ub2f5\ubcc0\uc774\uc5c8\ub2e4.", "sentence_answer": "\uc120\uc870\uac00 \uc774\uc5b4\uc11c \uacbd\uacfc \uc6b0\uacc4\ub97c \ube44\uad50\ud558\uba74 \uc5b4\ub5a4\uac00\ub77c\uace0 \ubb3b\uc790, \uc728\uace1\uc740 \" \uc7ac\uc8fc \ub294", "paragraph_id": "6545887-3-1", "paragraph_question": "question: \uc120\uc870\uc758 \uc728\uace1 \ubcf8\uc778\uacfc \uc131\ud63c\uc744 \uacac\uc8fc\uc5c8\ub54c\ub294 \uc5b4\ub5a4\uac00\ub77c\ub294 \ubb3c\uc74c\uc5d0 \uc728\uace1\uc740 \uc5b4\ub5a4 \uba74\uc5d0\uc11c \uc790\uc2e0\uc774 \uc6b0\uacc4\ubcf4\ub2e4 \ub354 \ub0ab\ub2e4\ub77c\ub294 \ud45c\ud604\uc744 \uc37c\ub294\uac00?, context: \uc0ac\ub9bc\uc758 \ucc9c\uac70\ub85c \uce5c\uad6c \uc131\ud63c\uc774 \uc911\ub9dd\uc5d0 \uc624\ub974\uc790 \uc120\uc870\ub294 \uc728\uace1\uc744 \ubd88\ub7ec \"\uc0ac\ub9bc\uc5d0\uc11c\ub294 \uc774 \ub09c\uc138\ub97c \uce58\uc720\ud560 \uc218 \uc788\ub294 \uc778\ubb3c\ub85c \uc6b0\uacc4\ub97c \ucc9c\uac70\ud558\ub294\ub370 \uacbd\uc758 \uc0dd\uac01\uc740 \uc5b4\ub5a4\uac00?\" \ub77c\uace0 \uadf8\uc5d0\uac8c \uc131\ud63c\uc758 \uc0ac\ub78c\ub428\uc744 \ubb3c\uc5c8\ub2e4. \uc728\uace1\uacfc \uc6b0\uacc4\ub294 \"\uc0b4\uc544\ub3c4 \uac19\uc774 \uc0b4\uace0 \uc8fd\uc5b4\ub3c4 \uac19\uc774 \uc8fd\uc790\"\ub294 \ub3d9\uc2ec\uc77c\uccb4\uc758 \uad50\uc6b0\uad00\uacc4\ub97c \uc9c0\ub2cc \uc0ac\uc774\uc600\ub2e4. \ud558\uc9c0\ub9cc \uc120\uc870\uc758 \ubb3c\uc74c\uc5d0 \uc728\uace1\uc740 \ud55c\ub9c8\ub514\ub85c \"\uc6b0\uacc4\ub294 \uadf8\ub7ec\ud55c \uc704\uc778\uc740 \ubabb \ub418\uace0 \ud559\ubb38\uc5d0 \ud798\uc4f0\ub294 \ucc29\uc2e4\ud55c \uc120\ube44\uc785\ub2c8\ub2e4\" \ub77c\uace0 \ub2f5\ubcc0\ud588\ub2e4. \ub098\ub77c\uc758 \uc5b4\ub824\uc6c0\uc744 \uac74\uc9c8 \ub9cc\ud55c \uc778\ubb3c\uc774\ub77c\uace0 \uc0ac\ub9bc\uc5d0\uc11c\uae4c\uc9c0 \ub5a0\ubc1b\ub4dc\ub294 \uc808\uce5c\ud55c \uce5c\uad6c\ub97c \ucc29\uc2e4\ud55c \uc120\ube44\uc5d0 \ubd88\uacfc\ud558\ub2e4\uace0 \ud55c \uc774 \ub2f5\ubcc0 \uc5ed\uc2dc \uc77c\uc0c1\uc758 \uc0c1\uc2dd\uc73c\ub85c\ub294 \uc774\ud574\ud558\uae30 \uc5b4\ub824\uc6b4 \ub0c9\ud639\ud55c \ud3c9\uac00\ub2e4. \uc120\uc870\uac00 \uc774\uc5b4\uc11c \uacbd\uacfc \uc6b0\uacc4\ub97c \ube44\uad50\ud558\uba74 \uc5b4\ub5a4\uac00\ub77c\uace0 \ubb3b\uc790, \uc728\uace1\uc740 \"\uc7ac\uc8fc\ub294 \uc18c\uc2e0\uc774 \uc6b0\uacc4\ubcf4\ub2e4 \uc880 \ub098\uc73c\ub098 \uc218\uc2e0\uacfc \ud559\ubb38\uc758 \ud798\uc500\uc5d0 \uc788\uc5b4\uc11c\ub294 \uc6b0\uacc4\uc5d0 \ubbf8\uce58\uc9c0 \ubabb\ud569\ub2c8\ub2e4\" \ub77c\uace0 \ub2f5\ubcc0\ud588\ub2e4. \uc728\uace1\ub2e4\uc6b4 \uc815\uc9c1\ud55c \ub2f5\ubcc0\uc774\uc5c8\ub2e4."} {"question": "\ubb34\ub77c\ud0c0\ub294 \ub204\uad6c\uc5d0\uac8c \ucee8\ub514\uc158 \uc870\uc808 \ubc29\ubc95\uc744 \ubc30\uc6b0\uac8c \ub418\ub294\uac00?", "paragraph": "\ud504\ub85c 1\ub144\uc9f8\uc778 1968\ub144\uc5d0\ub294 \ub2e8 \ud55c \ubc88\ub3c4 \uc2b9\ub9ac\uc5c6\uc774 1\ud328\ub9cc \uae30\ub85d\ud558\uc5ec \ubd80\uc9c4\uc5d0 \uc2dc\ub2ec\ub838\ub294\ub370 \ub2f9\uc2dc\uc5d0\ub294 \ub4dc\ub798\ud504\ud2b8 1\uc21c\uc704\ub85c \uacc4\uc57d\uae08\uc774 \ub0a8\uc544\ub3cc\uc544 \ud30c\uce5c\ucf54\ub098 \ub9c8\uc791\uacfc \uac19\uc740 \ub3c4\ubc15\uc5d0 \ube60\uc838\uc788\uc5c8\ub2e4. \uadf8\ub7ec\ub358 \uc5b4\ub290 \ub0a0, \ubc24\ub2a6\uac8c\uae4c\uc9c0 \ub9c8\uc791\uc744 \ub9c8\uce58\uace0 \ud640\ub85c \uc219\uc18c\uc5d0 \ub3cc\uc544\uc654\uc744 \ub54c \uc77c\uacfc\uc778 \ud2b8\ub808\uc774\ub2dd\uc744 \uc2dc\uc791\ud560\ub824\uace0 \ud558\uace0 \uc788\ub358 \uace0\uc57c\ub9c8 \ub9c8\uc0ac\uc544\ud0a4\uc640 \uc6b0\uc5f0\ud788 \ub9c8\uc8fc\uce58\uac8c \ub410\ub2e4. \uadf8 \ub54c \uc774\ubbf8 \ubca0\ud14c\ub791 \ud22c\uc218\uc600\ub358 \uace0\uc57c\ub9c8\uac00 \uc80a\uc740 \uc790\uc2e0\ubcf4\ub2e4\ub3c4 \ud6e8\uc52c \ub354 \ud798\ub4e0 \uc5f0\uc2b5\uc744 \uc790\uc2e0\uc5d0\uac8c \ubd80\uacfc\ud558\uace0 \uc788\ub2e4\ub294 \uac83\uc5d0 \ucda9\uaca9\uc744 \ubc1b\uc544 \uadf8\ub300\ub85c \ub3c4\ub9dd\uce58\ub4ef\uc774 \uc790\uc2e0\uc758 \ubc29\uc73c\ub85c \ub3cc\uc544\uac14\ub2e4. \uadf8 \ud6c4 \uc5f0\uc2b5 \uc911\uc5d0 \uace0\uc57c\ub9c8\uc5d0\uac8c \uc0ac\uacfc\ud558\ub7ec \uac14\uc744 \ub54c \u201c\uadf8\ub9cc\ud55c \uc7ac\ub2a5\uc744 \uac00\uc9c0\uace0 \uc788\uc73c\uba74\uc11c\ub3c4 \uadf8\uac78 \ub0ad\ube44\ud558\ub294\uac74 \uc544\uae5d\uc9c0 \uc54a\ub0d0\u201d\ub77c\uace0 \uc9c8\ucc45\uc744 \ubc1b\uc558\ub2e4. \uc57c\uad6c\uacc4\ub97c \ub300\ud45c\ud558\ub294 \uac04\ud310 \ud22c\uc218\uc5d0\uac8c\uc11c \uc9c8\ucc45\uc744 \ubc1b\uace0 \uadf8\uac83\uc5d0 \uac10\uba85\uc744 \ubc1b\uc740 \ubb34\ub77c\ud0c0\ub294 \uc774\ud6c4\uc5d0 \ub9c8\uce58 \ub534 \uc0ac\ub78c\uacfc \uac19\uc740 \uc5f0\uc2b5\uc5d0\ub9cc \ubab0\ub450\ud558\uac8c \ub410\ub2e4\uace0 \ud55c\ub2e4. \uc80a\uc740 \uc2dc\uc808 \ubb34\ub77c\ud0c0\uc758 \ub86f\ub370\uc5d0\ub294 \uace0\uc57c\ub9c8 \uc678\uc5d0\ub3c4 \ub098\ub9ac\ud0c0 \ud6c4\ubbf8\uc624, \uae30\ud0c0\ub8e8 \ub9c8\uc0ac\uc544\ud0a4, \uc0ac\uce74\uc774 \uac00\uc4f0\uc9c0, \uac00\ub124\ub2e4 \ub3c4\uba54\ud788\ub85c \ub4f1\uacfc \uac19\uc740 \uac04\ud310 \ud22c\uc218\ub4e4\uc774 \uc788\uc5b4\uc11c \ubb34\ub77c\ud0c0\ub294 \uadf8\ub4e4\ub85c\ubd80\ud130 \ud22c\uad6c \ubc29\ubc95\uc744 \ubc30\uc6cc \ub098\uac14\ub2e4. \ub610\ud55c 1973\ub144\ubd80\ud130 \uac10\ub3c5\uc73c\ub85c \ucde8\uc784\ud55c \uac00\ub124\ub2e4 \ub9c8\uc0ac\uc774\uce58\ub85c\ubd80\ud130 \ucee8\ub514\uc158 \uc870\uc808 \ubc29\ubc95\uc744 \ubc30\uc6e0\ub2e4. \ub610\ud55c \ud0c0\uaca9 \uc5f0\uc2b5\uc5d0 \uc788\uc5b4\uc11c\ub3c4 \uc5c4\ud55c \uc790\uc138\ub97c \ubcf4\uc774\ub358 \uc5d0\ub178\ubaa8\ud1a0 \uae30\ud558\uce58\uc5d0\uac8c\ub3c4 \ud504\ub85c \uc57c\uad6c \uc120\uc218\ub85c\uc11c \uc601\ud5a5\uc744 \ubc1b\uc558\ub2e4.", "answer": "\uac00\ub124\ub2e4 \ub9c8\uc0ac\uc774\uce58", "sentence": "\ub610\ud55c 1973\ub144\ubd80\ud130 \uac10\ub3c5\uc73c\ub85c \ucde8\uc784\ud55c \uac00\ub124\ub2e4 \ub9c8\uc0ac\uc774\uce58 \ub85c\ubd80\ud130 \ucee8\ub514\uc158 \uc870\uc808 \ubc29\ubc95\uc744 \ubc30\uc6e0\ub2e4.", "paragraph_sentence": "\ud504\ub85c 1\ub144\uc9f8\uc778 1968\ub144\uc5d0\ub294 \ub2e8 \ud55c \ubc88\ub3c4 \uc2b9\ub9ac\uc5c6\uc774 1\ud328\ub9cc \uae30\ub85d\ud558\uc5ec \ubd80\uc9c4\uc5d0 \uc2dc\ub2ec\ub838\ub294\ub370 \ub2f9\uc2dc\uc5d0\ub294 \ub4dc\ub798\ud504\ud2b8 1\uc21c\uc704\ub85c \uacc4\uc57d\uae08\uc774 \ub0a8\uc544\ub3cc\uc544 \ud30c\uce5c\ucf54\ub098 \ub9c8\uc791\uacfc \uac19\uc740 \ub3c4\ubc15\uc5d0 \ube60\uc838\uc788\uc5c8\ub2e4. \uadf8\ub7ec\ub358 \uc5b4\ub290 \ub0a0, \ubc24\ub2a6\uac8c\uae4c\uc9c0 \ub9c8\uc791\uc744 \ub9c8\uce58\uace0 \ud640\ub85c \uc219\uc18c\uc5d0 \ub3cc\uc544\uc654\uc744 \ub54c \uc77c\uacfc\uc778 \ud2b8\ub808\uc774\ub2dd\uc744 \uc2dc\uc791\ud560\ub824\uace0 \ud558\uace0 \uc788\ub358 \uace0\uc57c\ub9c8 \ub9c8\uc0ac\uc544\ud0a4\uc640 \uc6b0\uc5f0\ud788 \ub9c8\uc8fc\uce58\uac8c \ub410\ub2e4. \uadf8 \ub54c \uc774\ubbf8 \ubca0\ud14c\ub791 \ud22c\uc218\uc600\ub358 \uace0\uc57c\ub9c8\uac00 \uc80a\uc740 \uc790\uc2e0\ubcf4\ub2e4\ub3c4 \ud6e8\uc52c \ub354 \ud798\ub4e0 \uc5f0\uc2b5\uc744 \uc790\uc2e0\uc5d0\uac8c \ubd80\uacfc\ud558\uace0 \uc788\ub2e4\ub294 \uac83\uc5d0 \ucda9\uaca9\uc744 \ubc1b\uc544 \uadf8\ub300\ub85c \ub3c4\ub9dd\uce58\ub4ef\uc774 \uc790\uc2e0\uc758 \ubc29\uc73c\ub85c \ub3cc\uc544\uac14\ub2e4. \uadf8 \ud6c4 \uc5f0\uc2b5 \uc911\uc5d0 \uace0\uc57c\ub9c8\uc5d0\uac8c \uc0ac\uacfc\ud558\ub7ec \uac14\uc744 \ub54c \u201c\uadf8\ub9cc\ud55c \uc7ac\ub2a5\uc744 \uac00\uc9c0\uace0 \uc788\uc73c\uba74\uc11c\ub3c4 \uadf8\uac78 \ub0ad\ube44\ud558\ub294\uac74 \uc544\uae5d\uc9c0 \uc54a\ub0d0\u201d\ub77c\uace0 \uc9c8\ucc45\uc744 \ubc1b\uc558\ub2e4. \uc57c\uad6c\uacc4\ub97c \ub300\ud45c\ud558\ub294 \uac04\ud310 \ud22c\uc218\uc5d0\uac8c\uc11c \uc9c8\ucc45\uc744 \ubc1b\uace0 \uadf8\uac83\uc5d0 \uac10\uba85\uc744 \ubc1b\uc740 \ubb34\ub77c\ud0c0\ub294 \uc774\ud6c4\uc5d0 \ub9c8\uce58 \ub534 \uc0ac\ub78c\uacfc \uac19\uc740 \uc5f0\uc2b5\uc5d0\ub9cc \ubab0\ub450\ud558\uac8c \ub410\ub2e4\uace0 \ud55c\ub2e4. \uc80a\uc740 \uc2dc\uc808 \ubb34\ub77c\ud0c0\uc758 \ub86f\ub370\uc5d0\ub294 \uace0\uc57c\ub9c8 \uc678\uc5d0\ub3c4 \ub098\ub9ac\ud0c0 \ud6c4\ubbf8\uc624, \uae30\ud0c0\ub8e8 \ub9c8\uc0ac\uc544\ud0a4, \uc0ac\uce74\uc774 \uac00\uc4f0\uc9c0, \uac00\ub124\ub2e4 \ub3c4\uba54\ud788\ub85c \ub4f1\uacfc \uac19\uc740 \uac04\ud310 \ud22c\uc218\ub4e4\uc774 \uc788\uc5b4\uc11c \ubb34\ub77c\ud0c0\ub294 \uadf8\ub4e4\ub85c\ubd80\ud130 \ud22c\uad6c \ubc29\ubc95\uc744 \ubc30\uc6cc \ub098\uac14\ub2e4. \ub610\ud55c 1973\ub144\ubd80\ud130 \uac10\ub3c5\uc73c\ub85c \ucde8\uc784\ud55c \uac00\ub124\ub2e4 \ub9c8\uc0ac\uc774\uce58 \ub85c\ubd80\ud130 \ucee8\ub514\uc158 \uc870\uc808 \ubc29\ubc95\uc744 \ubc30\uc6e0\ub2e4. \ub610\ud55c \ud0c0\uaca9 \uc5f0\uc2b5\uc5d0 \uc788\uc5b4\uc11c\ub3c4 \uc5c4\ud55c \uc790\uc138\ub97c \ubcf4\uc774\ub358 \uc5d0\ub178\ubaa8\ud1a0 \uae30\ud558\uce58\uc5d0\uac8c\ub3c4 \ud504\ub85c \uc57c\uad6c \uc120\uc218\ub85c\uc11c \uc601\ud5a5\uc744 \ubc1b\uc558\ub2e4.", "paragraph_answer": "\ud504\ub85c 1\ub144\uc9f8\uc778 1968\ub144\uc5d0\ub294 \ub2e8 \ud55c \ubc88\ub3c4 \uc2b9\ub9ac\uc5c6\uc774 1\ud328\ub9cc \uae30\ub85d\ud558\uc5ec \ubd80\uc9c4\uc5d0 \uc2dc\ub2ec\ub838\ub294\ub370 \ub2f9\uc2dc\uc5d0\ub294 \ub4dc\ub798\ud504\ud2b8 1\uc21c\uc704\ub85c \uacc4\uc57d\uae08\uc774 \ub0a8\uc544\ub3cc\uc544 \ud30c\uce5c\ucf54\ub098 \ub9c8\uc791\uacfc \uac19\uc740 \ub3c4\ubc15\uc5d0 \ube60\uc838\uc788\uc5c8\ub2e4. \uadf8\ub7ec\ub358 \uc5b4\ub290 \ub0a0, \ubc24\ub2a6\uac8c\uae4c\uc9c0 \ub9c8\uc791\uc744 \ub9c8\uce58\uace0 \ud640\ub85c \uc219\uc18c\uc5d0 \ub3cc\uc544\uc654\uc744 \ub54c \uc77c\uacfc\uc778 \ud2b8\ub808\uc774\ub2dd\uc744 \uc2dc\uc791\ud560\ub824\uace0 \ud558\uace0 \uc788\ub358 \uace0\uc57c\ub9c8 \ub9c8\uc0ac\uc544\ud0a4\uc640 \uc6b0\uc5f0\ud788 \ub9c8\uc8fc\uce58\uac8c \ub410\ub2e4. \uadf8 \ub54c \uc774\ubbf8 \ubca0\ud14c\ub791 \ud22c\uc218\uc600\ub358 \uace0\uc57c\ub9c8\uac00 \uc80a\uc740 \uc790\uc2e0\ubcf4\ub2e4\ub3c4 \ud6e8\uc52c \ub354 \ud798\ub4e0 \uc5f0\uc2b5\uc744 \uc790\uc2e0\uc5d0\uac8c \ubd80\uacfc\ud558\uace0 \uc788\ub2e4\ub294 \uac83\uc5d0 \ucda9\uaca9\uc744 \ubc1b\uc544 \uadf8\ub300\ub85c \ub3c4\ub9dd\uce58\ub4ef\uc774 \uc790\uc2e0\uc758 \ubc29\uc73c\ub85c \ub3cc\uc544\uac14\ub2e4. \uadf8 \ud6c4 \uc5f0\uc2b5 \uc911\uc5d0 \uace0\uc57c\ub9c8\uc5d0\uac8c \uc0ac\uacfc\ud558\ub7ec \uac14\uc744 \ub54c \u201c\uadf8\ub9cc\ud55c \uc7ac\ub2a5\uc744 \uac00\uc9c0\uace0 \uc788\uc73c\uba74\uc11c\ub3c4 \uadf8\uac78 \ub0ad\ube44\ud558\ub294\uac74 \uc544\uae5d\uc9c0 \uc54a\ub0d0\u201d\ub77c\uace0 \uc9c8\ucc45\uc744 \ubc1b\uc558\ub2e4. \uc57c\uad6c\uacc4\ub97c \ub300\ud45c\ud558\ub294 \uac04\ud310 \ud22c\uc218\uc5d0\uac8c\uc11c \uc9c8\ucc45\uc744 \ubc1b\uace0 \uadf8\uac83\uc5d0 \uac10\uba85\uc744 \ubc1b\uc740 \ubb34\ub77c\ud0c0\ub294 \uc774\ud6c4\uc5d0 \ub9c8\uce58 \ub534 \uc0ac\ub78c\uacfc \uac19\uc740 \uc5f0\uc2b5\uc5d0\ub9cc \ubab0\ub450\ud558\uac8c \ub410\ub2e4\uace0 \ud55c\ub2e4. \uc80a\uc740 \uc2dc\uc808 \ubb34\ub77c\ud0c0\uc758 \ub86f\ub370\uc5d0\ub294 \uace0\uc57c\ub9c8 \uc678\uc5d0\ub3c4 \ub098\ub9ac\ud0c0 \ud6c4\ubbf8\uc624, \uae30\ud0c0\ub8e8 \ub9c8\uc0ac\uc544\ud0a4, \uc0ac\uce74\uc774 \uac00\uc4f0\uc9c0, \uac00\ub124\ub2e4 \ub3c4\uba54\ud788\ub85c \ub4f1\uacfc \uac19\uc740 \uac04\ud310 \ud22c\uc218\ub4e4\uc774 \uc788\uc5b4\uc11c \ubb34\ub77c\ud0c0\ub294 \uadf8\ub4e4\ub85c\ubd80\ud130 \ud22c\uad6c \ubc29\ubc95\uc744 \ubc30\uc6cc \ub098\uac14\ub2e4. \ub610\ud55c 1973\ub144\ubd80\ud130 \uac10\ub3c5\uc73c\ub85c \ucde8\uc784\ud55c \uac00\ub124\ub2e4 \ub9c8\uc0ac\uc774\uce58 \ub85c\ubd80\ud130 \ucee8\ub514\uc158 \uc870\uc808 \ubc29\ubc95\uc744 \ubc30\uc6e0\ub2e4. \ub610\ud55c \ud0c0\uaca9 \uc5f0\uc2b5\uc5d0 \uc788\uc5b4\uc11c\ub3c4 \uc5c4\ud55c \uc790\uc138\ub97c \ubcf4\uc774\ub358 \uc5d0\ub178\ubaa8\ud1a0 \uae30\ud558\uce58\uc5d0\uac8c\ub3c4 \ud504\ub85c \uc57c\uad6c \uc120\uc218\ub85c\uc11c \uc601\ud5a5\uc744 \ubc1b\uc558\ub2e4.", "sentence_answer": "\ub610\ud55c 1973\ub144\ubd80\ud130 \uac10\ub3c5\uc73c\ub85c \ucde8\uc784\ud55c \uac00\ub124\ub2e4 \ub9c8\uc0ac\uc774\uce58 \ub85c\ubd80\ud130 \ucee8\ub514\uc158 \uc870\uc808 \ubc29\ubc95\uc744 \ubc30\uc6e0\ub2e4.", "paragraph_id": "6529468-1-0", "paragraph_question": "question: \ubb34\ub77c\ud0c0\ub294 \ub204\uad6c\uc5d0\uac8c \ucee8\ub514\uc158 \uc870\uc808 \ubc29\ubc95\uc744 \ubc30\uc6b0\uac8c \ub418\ub294\uac00?, context: \ud504\ub85c 1\ub144\uc9f8\uc778 1968\ub144\uc5d0\ub294 \ub2e8 \ud55c \ubc88\ub3c4 \uc2b9\ub9ac\uc5c6\uc774 1\ud328\ub9cc \uae30\ub85d\ud558\uc5ec \ubd80\uc9c4\uc5d0 \uc2dc\ub2ec\ub838\ub294\ub370 \ub2f9\uc2dc\uc5d0\ub294 \ub4dc\ub798\ud504\ud2b8 1\uc21c\uc704\ub85c \uacc4\uc57d\uae08\uc774 \ub0a8\uc544\ub3cc\uc544 \ud30c\uce5c\ucf54\ub098 \ub9c8\uc791\uacfc \uac19\uc740 \ub3c4\ubc15\uc5d0 \ube60\uc838\uc788\uc5c8\ub2e4. \uadf8\ub7ec\ub358 \uc5b4\ub290 \ub0a0, \ubc24\ub2a6\uac8c\uae4c\uc9c0 \ub9c8\uc791\uc744 \ub9c8\uce58\uace0 \ud640\ub85c \uc219\uc18c\uc5d0 \ub3cc\uc544\uc654\uc744 \ub54c \uc77c\uacfc\uc778 \ud2b8\ub808\uc774\ub2dd\uc744 \uc2dc\uc791\ud560\ub824\uace0 \ud558\uace0 \uc788\ub358 \uace0\uc57c\ub9c8 \ub9c8\uc0ac\uc544\ud0a4\uc640 \uc6b0\uc5f0\ud788 \ub9c8\uc8fc\uce58\uac8c \ub410\ub2e4. \uadf8 \ub54c \uc774\ubbf8 \ubca0\ud14c\ub791 \ud22c\uc218\uc600\ub358 \uace0\uc57c\ub9c8\uac00 \uc80a\uc740 \uc790\uc2e0\ubcf4\ub2e4\ub3c4 \ud6e8\uc52c \ub354 \ud798\ub4e0 \uc5f0\uc2b5\uc744 \uc790\uc2e0\uc5d0\uac8c \ubd80\uacfc\ud558\uace0 \uc788\ub2e4\ub294 \uac83\uc5d0 \ucda9\uaca9\uc744 \ubc1b\uc544 \uadf8\ub300\ub85c \ub3c4\ub9dd\uce58\ub4ef\uc774 \uc790\uc2e0\uc758 \ubc29\uc73c\ub85c \ub3cc\uc544\uac14\ub2e4. \uadf8 \ud6c4 \uc5f0\uc2b5 \uc911\uc5d0 \uace0\uc57c\ub9c8\uc5d0\uac8c \uc0ac\uacfc\ud558\ub7ec \uac14\uc744 \ub54c \u201c\uadf8\ub9cc\ud55c \uc7ac\ub2a5\uc744 \uac00\uc9c0\uace0 \uc788\uc73c\uba74\uc11c\ub3c4 \uadf8\uac78 \ub0ad\ube44\ud558\ub294\uac74 \uc544\uae5d\uc9c0 \uc54a\ub0d0\u201d\ub77c\uace0 \uc9c8\ucc45\uc744 \ubc1b\uc558\ub2e4. \uc57c\uad6c\uacc4\ub97c \ub300\ud45c\ud558\ub294 \uac04\ud310 \ud22c\uc218\uc5d0\uac8c\uc11c \uc9c8\ucc45\uc744 \ubc1b\uace0 \uadf8\uac83\uc5d0 \uac10\uba85\uc744 \ubc1b\uc740 \ubb34\ub77c\ud0c0\ub294 \uc774\ud6c4\uc5d0 \ub9c8\uce58 \ub534 \uc0ac\ub78c\uacfc \uac19\uc740 \uc5f0\uc2b5\uc5d0\ub9cc \ubab0\ub450\ud558\uac8c \ub410\ub2e4\uace0 \ud55c\ub2e4. \uc80a\uc740 \uc2dc\uc808 \ubb34\ub77c\ud0c0\uc758 \ub86f\ub370\uc5d0\ub294 \uace0\uc57c\ub9c8 \uc678\uc5d0\ub3c4 \ub098\ub9ac\ud0c0 \ud6c4\ubbf8\uc624, \uae30\ud0c0\ub8e8 \ub9c8\uc0ac\uc544\ud0a4, \uc0ac\uce74\uc774 \uac00\uc4f0\uc9c0, \uac00\ub124\ub2e4 \ub3c4\uba54\ud788\ub85c \ub4f1\uacfc \uac19\uc740 \uac04\ud310 \ud22c\uc218\ub4e4\uc774 \uc788\uc5b4\uc11c \ubb34\ub77c\ud0c0\ub294 \uadf8\ub4e4\ub85c\ubd80\ud130 \ud22c\uad6c \ubc29\ubc95\uc744 \ubc30\uc6cc \ub098\uac14\ub2e4. \ub610\ud55c 1973\ub144\ubd80\ud130 \uac10\ub3c5\uc73c\ub85c \ucde8\uc784\ud55c \uac00\ub124\ub2e4 \ub9c8\uc0ac\uc774\uce58\ub85c\ubd80\ud130 \ucee8\ub514\uc158 \uc870\uc808 \ubc29\ubc95\uc744 \ubc30\uc6e0\ub2e4. \ub610\ud55c \ud0c0\uaca9 \uc5f0\uc2b5\uc5d0 \uc788\uc5b4\uc11c\ub3c4 \uc5c4\ud55c \uc790\uc138\ub97c \ubcf4\uc774\ub358 \uc5d0\ub178\ubaa8\ud1a0 \uae30\ud558\uce58\uc5d0\uac8c\ub3c4 \ud504\ub85c \uc57c\uad6c \uc120\uc218\ub85c\uc11c \uc601\ud5a5\uc744 \ubc1b\uc558\ub2e4."} {"question": "\uc2a4\ucf67 \ubc95\uc774 \ubb34\uae30\ud55c\uc73c\ub85c \uc5f0\uc7a5\ub41c \uc5f0\ub3c4\ub294?", "paragraph": "1884\ub144 \uac1c\uc815\ub41c \ubc95\ub960\uc740 \ud5c8\ub77d\ub41c \uc774\uc804 \uc774\ubbfc\uc790\ub4e4\uc774 \ub5a0\ub098\uace0, \ub3cc\uc544\uc624\ub294 \uaddc\uc815\uc744 \uac15\ud654\uc2dc\ucf30\uc73c\uba70, \uadf8 \ubc95\uc774 \uc6d0\ub798\uc758 \ub098\ub77c\uc640 \uc0c1\uad00\uc5c6\uc774 \uc911\uad6d \ubbfc\uc871\uc5d0\uac8c\ub9cc \uc801\uc6a9\ub41c\ub2e4\ub294 \uac83\uc744 \uba85\uc2dc\ud588\ub2e4. 1888\ub144\uc5d0\ub294 \u3008\uc2a4\ucf67 \ubc95\u3009\uc774 \uc2dc\ud589\ub418\uba74\uc11c \uc911\uad6d\uc778 \ubc30\ucc99\ubc95 \uc2dc\ud589 \uc774\ud6c4\uc5d0 \uadc0\uad6d\ud55c \uacbd\uc6b0 \uc7ac\uc785\uad6d\uc744 \uae08\uc9c0\uc2dc\ucf30\ub2e4. \u3008\uc2a4\ucf67 \ubc95\u3009\uc740 \ub300\ubc95\uc6d0\uc774 \"\uc678\uad6d\uc778 \ubc30\uc81c \uad8c\ud55c\uc740 \ud5cc\ubc95\uc5d0 \uc758\ud574 \uc704\uc784\ub41c \uc8fc\uad8c \ud589\uc0ac\uc758 \uc77c\ubd80\ub85c\uc11c \ubbf8\uad6d \uc815\ubd80\uc5d0 \uc18d\ud55c \ubd80\ub300 \uad8c\ub9ac\"\ub77c\uace0 \ubc1d\ud614\ub2e4. \uc774 \ubc95\uc740 1892\ub144 \u3008\uac8c\ub9ac \ubc95\u3009\uc5d0 \uc758\ud574 \ub2e4\uc2dc 10\ub144\ub3d9\uc548 \uac31\uc2e0\ub418\uc5c8\uc73c\uba70, 1902\ub144\uc5d0\ub294 \ubb34\uae30\ud55c\uc73c\ub85c \uc5f0\uc7a5\ub418\uc5c8\ub2e4. 1902\ub144 \uc601\uad6c\ud654\ud588\uc744 \ub54c, \ubaa8\ub4e0 \uc911\uad6d\uc778 \uac70\uc8fc\uc790\ub294 \ub4f1\ub85d\uc744 \ud574\uc57c \ud558\uace0, \uac70\uc8fc\uc99d\uba85\uc11c\ub97c \uc2b5\ub4dd\ud574\uc57c \ud560 \uac83\uc744 \uc694\uad6c\ud588\ub2e4. \uc99d\uba85\uc11c\uac00 \uc5c6\uc73c\uba74, \ub2f9\uc0ac\uc790\ub294 \ucd94\ubc29\uc5d0 \uc9c1\uba74\ud558\uc600\ub2e4.", "answer": "1902\ub144", "sentence": "\uc774 \ubc95\uc740 1892\ub144 \u3008\uac8c\ub9ac \ubc95\u3009\uc5d0 \uc758\ud574 \ub2e4\uc2dc 10\ub144\ub3d9\uc548 \uac31\uc2e0\ub418\uc5c8\uc73c\uba70, 1902\ub144 \uc5d0\ub294 \ubb34\uae30\ud55c\uc73c\ub85c \uc5f0\uc7a5\ub418\uc5c8\ub2e4.", "paragraph_sentence": "1884\ub144 \uac1c\uc815\ub41c \ubc95\ub960\uc740 \ud5c8\ub77d\ub41c \uc774\uc804 \uc774\ubbfc\uc790\ub4e4\uc774 \ub5a0\ub098\uace0, \ub3cc\uc544\uc624\ub294 \uaddc\uc815\uc744 \uac15\ud654\uc2dc\ucf30\uc73c\uba70, \uadf8 \ubc95\uc774 \uc6d0\ub798\uc758 \ub098\ub77c\uc640 \uc0c1\uad00\uc5c6\uc774 \uc911\uad6d \ubbfc\uc871\uc5d0\uac8c\ub9cc \uc801\uc6a9\ub41c\ub2e4\ub294 \uac83\uc744 \uba85\uc2dc\ud588\ub2e4. 1888\ub144\uc5d0\ub294 \u3008\uc2a4\ucf67 \ubc95\u3009\uc774 \uc2dc\ud589\ub418\uba74\uc11c \uc911\uad6d\uc778 \ubc30\ucc99\ubc95 \uc2dc\ud589 \uc774\ud6c4\uc5d0 \uadc0\uad6d\ud55c \uacbd\uc6b0 \uc7ac\uc785\uad6d\uc744 \uae08\uc9c0\uc2dc\ucf30\ub2e4. \u3008\uc2a4\ucf67 \ubc95\u3009\uc740 \ub300\ubc95\uc6d0\uc774 \"\uc678\uad6d\uc778 \ubc30\uc81c \uad8c\ud55c\uc740 \ud5cc\ubc95\uc5d0 \uc758\ud574 \uc704\uc784\ub41c \uc8fc\uad8c \ud589\uc0ac\uc758 \uc77c\ubd80\ub85c\uc11c \ubbf8\uad6d \uc815\ubd80\uc5d0 \uc18d\ud55c \ubd80\ub300 \uad8c\ub9ac\"\ub77c\uace0 \ubc1d\ud614\ub2e4. \uc774 \ubc95\uc740 1892\ub144 \u3008\uac8c\ub9ac \ubc95\u3009\uc5d0 \uc758\ud574 \ub2e4\uc2dc 10\ub144\ub3d9\uc548 \uac31\uc2e0\ub418\uc5c8\uc73c\uba70, 1902\ub144 \uc5d0\ub294 \ubb34\uae30\ud55c\uc73c\ub85c \uc5f0\uc7a5\ub418\uc5c8\ub2e4. 1902\ub144 \uc601\uad6c\ud654\ud588\uc744 \ub54c, \ubaa8\ub4e0 \uc911\uad6d\uc778 \uac70\uc8fc\uc790\ub294 \ub4f1\ub85d\uc744 \ud574\uc57c \ud558\uace0, \uac70\uc8fc\uc99d\uba85\uc11c\ub97c \uc2b5\ub4dd\ud574\uc57c \ud560 \uac83\uc744 \uc694\uad6c\ud588\ub2e4. \uc99d\uba85\uc11c\uac00 \uc5c6\uc73c\uba74, \ub2f9\uc0ac\uc790\ub294 \ucd94\ubc29\uc5d0 \uc9c1\uba74\ud558\uc600\ub2e4.", "paragraph_answer": "1884\ub144 \uac1c\uc815\ub41c \ubc95\ub960\uc740 \ud5c8\ub77d\ub41c \uc774\uc804 \uc774\ubbfc\uc790\ub4e4\uc774 \ub5a0\ub098\uace0, \ub3cc\uc544\uc624\ub294 \uaddc\uc815\uc744 \uac15\ud654\uc2dc\ucf30\uc73c\uba70, \uadf8 \ubc95\uc774 \uc6d0\ub798\uc758 \ub098\ub77c\uc640 \uc0c1\uad00\uc5c6\uc774 \uc911\uad6d \ubbfc\uc871\uc5d0\uac8c\ub9cc \uc801\uc6a9\ub41c\ub2e4\ub294 \uac83\uc744 \uba85\uc2dc\ud588\ub2e4. 1888\ub144\uc5d0\ub294 \u3008\uc2a4\ucf67 \ubc95\u3009\uc774 \uc2dc\ud589\ub418\uba74\uc11c \uc911\uad6d\uc778 \ubc30\ucc99\ubc95 \uc2dc\ud589 \uc774\ud6c4\uc5d0 \uadc0\uad6d\ud55c \uacbd\uc6b0 \uc7ac\uc785\uad6d\uc744 \uae08\uc9c0\uc2dc\ucf30\ub2e4. \u3008\uc2a4\ucf67 \ubc95\u3009\uc740 \ub300\ubc95\uc6d0\uc774 \"\uc678\uad6d\uc778 \ubc30\uc81c \uad8c\ud55c\uc740 \ud5cc\ubc95\uc5d0 \uc758\ud574 \uc704\uc784\ub41c \uc8fc\uad8c \ud589\uc0ac\uc758 \uc77c\ubd80\ub85c\uc11c \ubbf8\uad6d \uc815\ubd80\uc5d0 \uc18d\ud55c \ubd80\ub300 \uad8c\ub9ac\"\ub77c\uace0 \ubc1d\ud614\ub2e4. \uc774 \ubc95\uc740 1892\ub144 \u3008\uac8c\ub9ac \ubc95\u3009\uc5d0 \uc758\ud574 \ub2e4\uc2dc 10\ub144\ub3d9\uc548 \uac31\uc2e0\ub418\uc5c8\uc73c\uba70, 1902\ub144 \uc5d0\ub294 \ubb34\uae30\ud55c\uc73c\ub85c \uc5f0\uc7a5\ub418\uc5c8\ub2e4. 1902\ub144 \uc601\uad6c\ud654\ud588\uc744 \ub54c, \ubaa8\ub4e0 \uc911\uad6d\uc778 \uac70\uc8fc\uc790\ub294 \ub4f1\ub85d\uc744 \ud574\uc57c \ud558\uace0, \uac70\uc8fc\uc99d\uba85\uc11c\ub97c \uc2b5\ub4dd\ud574\uc57c \ud560 \uac83\uc744 \uc694\uad6c\ud588\ub2e4. \uc99d\uba85\uc11c\uac00 \uc5c6\uc73c\uba74, \ub2f9\uc0ac\uc790\ub294 \ucd94\ubc29\uc5d0 \uc9c1\uba74\ud558\uc600\ub2e4.", "sentence_answer": "\uc774 \ubc95\uc740 1892\ub144 \u3008\uac8c\ub9ac \ubc95\u3009\uc5d0 \uc758\ud574 \ub2e4\uc2dc 10\ub144\ub3d9\uc548 \uac31\uc2e0\ub418\uc5c8\uc73c\uba70, 1902\ub144 \uc5d0\ub294 \ubb34\uae30\ud55c\uc73c\ub85c \uc5f0\uc7a5\ub418\uc5c8\ub2e4.", "paragraph_id": "6596153-0-2", "paragraph_question": "question: \uc2a4\ucf67 \ubc95\uc774 \ubb34\uae30\ud55c\uc73c\ub85c \uc5f0\uc7a5\ub41c \uc5f0\ub3c4\ub294?, context: 1884\ub144 \uac1c\uc815\ub41c \ubc95\ub960\uc740 \ud5c8\ub77d\ub41c \uc774\uc804 \uc774\ubbfc\uc790\ub4e4\uc774 \ub5a0\ub098\uace0, \ub3cc\uc544\uc624\ub294 \uaddc\uc815\uc744 \uac15\ud654\uc2dc\ucf30\uc73c\uba70, \uadf8 \ubc95\uc774 \uc6d0\ub798\uc758 \ub098\ub77c\uc640 \uc0c1\uad00\uc5c6\uc774 \uc911\uad6d \ubbfc\uc871\uc5d0\uac8c\ub9cc \uc801\uc6a9\ub41c\ub2e4\ub294 \uac83\uc744 \uba85\uc2dc\ud588\ub2e4. 1888\ub144\uc5d0\ub294 \u3008\uc2a4\ucf67 \ubc95\u3009\uc774 \uc2dc\ud589\ub418\uba74\uc11c \uc911\uad6d\uc778 \ubc30\ucc99\ubc95 \uc2dc\ud589 \uc774\ud6c4\uc5d0 \uadc0\uad6d\ud55c \uacbd\uc6b0 \uc7ac\uc785\uad6d\uc744 \uae08\uc9c0\uc2dc\ucf30\ub2e4. \u3008\uc2a4\ucf67 \ubc95\u3009\uc740 \ub300\ubc95\uc6d0\uc774 \"\uc678\uad6d\uc778 \ubc30\uc81c \uad8c\ud55c\uc740 \ud5cc\ubc95\uc5d0 \uc758\ud574 \uc704\uc784\ub41c \uc8fc\uad8c \ud589\uc0ac\uc758 \uc77c\ubd80\ub85c\uc11c \ubbf8\uad6d \uc815\ubd80\uc5d0 \uc18d\ud55c \ubd80\ub300 \uad8c\ub9ac\"\ub77c\uace0 \ubc1d\ud614\ub2e4. \uc774 \ubc95\uc740 1892\ub144 \u3008\uac8c\ub9ac \ubc95\u3009\uc5d0 \uc758\ud574 \ub2e4\uc2dc 10\ub144\ub3d9\uc548 \uac31\uc2e0\ub418\uc5c8\uc73c\uba70, 1902\ub144\uc5d0\ub294 \ubb34\uae30\ud55c\uc73c\ub85c \uc5f0\uc7a5\ub418\uc5c8\ub2e4. 1902\ub144 \uc601\uad6c\ud654\ud588\uc744 \ub54c, \ubaa8\ub4e0 \uc911\uad6d\uc778 \uac70\uc8fc\uc790\ub294 \ub4f1\ub85d\uc744 \ud574\uc57c \ud558\uace0, \uac70\uc8fc\uc99d\uba85\uc11c\ub97c \uc2b5\ub4dd\ud574\uc57c \ud560 \uac83\uc744 \uc694\uad6c\ud588\ub2e4. \uc99d\uba85\uc11c\uac00 \uc5c6\uc73c\uba74, \ub2f9\uc0ac\uc790\ub294 \ucd94\ubc29\uc5d0 \uc9c1\uba74\ud558\uc600\ub2e4."} {"question": "1420\ud3b8 \ud56d\uacf5\uae30\uac00 \uace0\uc18d\uc73c\ub85c \ud65c\uc8fc\ub85c\ub97c \uc9c8\uc8fc\ud560 \ub54c \uae30\uc7a5\uc774 \uac1c\uc2dc\ud55c \uac83\uc740?", "paragraph": "23\uc2dc 50\ubd84 20\ucd08\uc5d0 \uc0ac\uace0\uae30\uac00 4R \ud65c\uc8fc\ub85c\uc5d0 \ub2ff\uc558\ub2e4. \ucc29\ub959\ubc14\ud034\uac00 \ub2ff\uc740\uc9c0 2\ucd08\ud6c4 \ubd80\uae30\uc7a5 \uc624\ub9ac\uac94\uc774 \"\uc6b0\ub9ac\ub294 \ubbf8\ub044\ub7ec\uc9c0\uace0 \uc788\uc5b4\uc694\"\ub77c\uace0 \uc0c1\uae30\ud588\ub2e4. \uc5b4\ub290 \ub204\uad6c\ub3c4 \uc2a4\ud3ec\uc77c\ub7ec\ub97c \uc62c\ub9ac\ub294\ub370 \uc2e4\ud328\ud588\uae30 \ub54c\ubb38\uc774\uc5c8\ub294\ub370 \ucc29\ub959\ud588\uc744\ub54c \uc2a4\ud3ec\uc77c\ub7ec\uac00 \uc790\ub3d9\uc73c\ub85c \uc804\uac1c\ub418\uc9c0 \uc54a\uc740\ub370\ub2e4 \ube44\ud589 \uc2b9\ubb34\uc6d0\ub4e4\ub3c4 \uc218\ub3d9\uc73c\ub85c \uc2a4\ud3ec\uc77c\ub7ec\ub97c \uc62c\ub9ac\ub294\ub370 \uc2e4\ud328\ud588\uae30 \ub54c\ubb38\uc774\uc5c8\ub2e4. \uc2a4\ud3ec\uc77c\ub7ec\uac00 \uacf5\uae30\uc800\ud56d\uc744 \ud615\uc131\ud568\uc73c\ub85c \uc778\ud574 \ud56d\uacf5\uae30\uc758 \uc911\ub7c9\uc740 \ucc29\ub959\ubc14\ud034\ub85c \uc9c0\uc9c0\ub418\ub294\ub370 \ub9cc\uc57d \uc2a4\ud3ec\uc77c\ub7ec\uac00 \uc804\uac1c\ub418\uc5c8\uc744 \uacbd\uc6b0\uc5d0\ub294 \uc0ac\uace0\uae30 \uc911\ub7c9\uc758 65%\ub294 \ucda9\ubd84\ud788 \ucc29\ub959\ubc14\ud034\ub9cc\uc73c\ub85c\ub3c4 \uc9c0\uc9c0\ub420 \uc218 \uc788\uc9c0\ub9cc \uc2a4\ud3ec\uc77c\ub7ec\uac00 \uc5c6\uc744 \uacbd\uc6b0 \uc774 \uc218\uce58\uc5d0\uc11c \uac70\uc758 \ube60\uc9c4 15%\ub85c \ub5a8\uc5b4\uc9c0\uac8c \ub41c\ub2e4. \ucc29\ub959\ubc14\ud034\uc758 \uac00\ubcbc\uc6b4 \uc911\ub7c9\uacfc \ud568\uaed8 \uc0ac\uace0\uae30\uc758 \ube0c\ub808\uc774\ud06c\ub294 \uac10\uc18d\ud558\ub294\ub370 \ud6a8\uacfc\uac00 \uc5c6\uc5c8\uc73c\uba70 \uacc4\uc18d \uace0\uc18d\uc73c\ub85c \ud65c\uc8fc\ub85c\ub97c \uc9c8\uc8fc\ud558\uc600\ub2e4. \ube0c\uc2dc\uba3c \uae30\uc7a5\uc774 \uc5d4\uc9c4 \uc5ed\ucd94\uc9c4\uc744 \uac1c\uc2dc\ud558\uba74\uc11c \ubc29\ud5a5 \uc81c\uc5b4\ub97c \uc0c1\uc2e4\ud558\uace0 \uc0ac\uace0\uae30\uc758 \ubc29\ud5a5\ud0a4\uc640 \uc218\uc9c1 \uc548\uc815\ud310\uc758 \ud6a8\uacfc\ub3c4 \uac70\uc758 \uc904\uc5b4\ub4e4\uc5c8\ub2e4.", "answer": "\uc5d4\uc9c4 \uc5ed\ucd94\uc9c4", "sentence": "\ube0c\uc2dc\uba3c \uae30\uc7a5\uc774 \uc5d4\uc9c4 \uc5ed\ucd94\uc9c4 \uc744 \uac1c\uc2dc\ud558\uba74\uc11c \ubc29\ud5a5 \uc81c\uc5b4\ub97c \uc0c1\uc2e4\ud558\uace0 \uc0ac\uace0\uae30\uc758 \ubc29\ud5a5\ud0a4\uc640 \uc218\uc9c1 \uc548\uc815\ud310\uc758 \ud6a8\uacfc\ub3c4 \uac70\uc758 \uc904\uc5b4\ub4e4\uc5c8\ub2e4.", "paragraph_sentence": "23\uc2dc 50\ubd84 20\ucd08\uc5d0 \uc0ac\uace0\uae30\uac00 4R \ud65c\uc8fc\ub85c\uc5d0 \ub2ff\uc558\ub2e4. \ucc29\ub959\ubc14\ud034\uac00 \ub2ff\uc740\uc9c0 2\ucd08\ud6c4 \ubd80\uae30\uc7a5 \uc624\ub9ac\uac94\uc774 \"\uc6b0\ub9ac\ub294 \ubbf8\ub044\ub7ec\uc9c0\uace0 \uc788\uc5b4\uc694\"\ub77c\uace0 \uc0c1\uae30\ud588\ub2e4. \uc5b4\ub290 \ub204\uad6c\ub3c4 \uc2a4\ud3ec\uc77c\ub7ec\ub97c \uc62c\ub9ac\ub294\ub370 \uc2e4\ud328\ud588\uae30 \ub54c\ubb38\uc774\uc5c8\ub294\ub370 \ucc29\ub959\ud588\uc744\ub54c \uc2a4\ud3ec\uc77c\ub7ec\uac00 \uc790\ub3d9\uc73c\ub85c \uc804\uac1c\ub418\uc9c0 \uc54a\uc740\ub370\ub2e4 \ube44\ud589 \uc2b9\ubb34\uc6d0\ub4e4\ub3c4 \uc218\ub3d9\uc73c\ub85c \uc2a4\ud3ec\uc77c\ub7ec\ub97c \uc62c\ub9ac\ub294\ub370 \uc2e4\ud328\ud588\uae30 \ub54c\ubb38\uc774\uc5c8\ub2e4. \uc2a4\ud3ec\uc77c\ub7ec\uac00 \uacf5\uae30\uc800\ud56d\uc744 \ud615\uc131\ud568\uc73c\ub85c \uc778\ud574 \ud56d\uacf5\uae30\uc758 \uc911\ub7c9\uc740 \ucc29\ub959\ubc14\ud034\ub85c \uc9c0\uc9c0\ub418\ub294\ub370 \ub9cc\uc57d \uc2a4\ud3ec\uc77c\ub7ec\uac00 \uc804\uac1c\ub418\uc5c8\uc744 \uacbd\uc6b0\uc5d0\ub294 \uc0ac\uace0\uae30 \uc911\ub7c9\uc758 65%\ub294 \ucda9\ubd84\ud788 \ucc29\ub959\ubc14\ud034\ub9cc\uc73c\ub85c\ub3c4 \uc9c0\uc9c0\ub420 \uc218 \uc788\uc9c0\ub9cc \uc2a4\ud3ec\uc77c\ub7ec\uac00 \uc5c6\uc744 \uacbd\uc6b0 \uc774 \uc218\uce58\uc5d0\uc11c \uac70\uc758 \ube60\uc9c4 15%\ub85c \ub5a8\uc5b4\uc9c0\uac8c \ub41c\ub2e4. \ucc29\ub959\ubc14\ud034\uc758 \uac00\ubcbc\uc6b4 \uc911\ub7c9\uacfc \ud568\uaed8 \uc0ac\uace0\uae30\uc758 \ube0c\ub808\uc774\ud06c\ub294 \uac10\uc18d\ud558\ub294\ub370 \ud6a8\uacfc\uac00 \uc5c6\uc5c8\uc73c\uba70 \uacc4\uc18d \uace0\uc18d\uc73c\ub85c \ud65c\uc8fc\ub85c\ub97c \uc9c8\uc8fc\ud558\uc600\ub2e4. \ube0c\uc2dc\uba3c \uae30\uc7a5\uc774 \uc5d4\uc9c4 \uc5ed\ucd94\uc9c4 \uc744 \uac1c\uc2dc\ud558\uba74\uc11c \ubc29\ud5a5 \uc81c\uc5b4\ub97c \uc0c1\uc2e4\ud558\uace0 \uc0ac\uace0\uae30\uc758 \ubc29\ud5a5\ud0a4\uc640 \uc218\uc9c1 \uc548\uc815\ud310\uc758 \ud6a8\uacfc\ub3c4 \uac70\uc758 \uc904\uc5b4\ub4e4\uc5c8\ub2e4. ", "paragraph_answer": "23\uc2dc 50\ubd84 20\ucd08\uc5d0 \uc0ac\uace0\uae30\uac00 4R \ud65c\uc8fc\ub85c\uc5d0 \ub2ff\uc558\ub2e4. \ucc29\ub959\ubc14\ud034\uac00 \ub2ff\uc740\uc9c0 2\ucd08\ud6c4 \ubd80\uae30\uc7a5 \uc624\ub9ac\uac94\uc774 \"\uc6b0\ub9ac\ub294 \ubbf8\ub044\ub7ec\uc9c0\uace0 \uc788\uc5b4\uc694\"\ub77c\uace0 \uc0c1\uae30\ud588\ub2e4. \uc5b4\ub290 \ub204\uad6c\ub3c4 \uc2a4\ud3ec\uc77c\ub7ec\ub97c \uc62c\ub9ac\ub294\ub370 \uc2e4\ud328\ud588\uae30 \ub54c\ubb38\uc774\uc5c8\ub294\ub370 \ucc29\ub959\ud588\uc744\ub54c \uc2a4\ud3ec\uc77c\ub7ec\uac00 \uc790\ub3d9\uc73c\ub85c \uc804\uac1c\ub418\uc9c0 \uc54a\uc740\ub370\ub2e4 \ube44\ud589 \uc2b9\ubb34\uc6d0\ub4e4\ub3c4 \uc218\ub3d9\uc73c\ub85c \uc2a4\ud3ec\uc77c\ub7ec\ub97c \uc62c\ub9ac\ub294\ub370 \uc2e4\ud328\ud588\uae30 \ub54c\ubb38\uc774\uc5c8\ub2e4. \uc2a4\ud3ec\uc77c\ub7ec\uac00 \uacf5\uae30\uc800\ud56d\uc744 \ud615\uc131\ud568\uc73c\ub85c \uc778\ud574 \ud56d\uacf5\uae30\uc758 \uc911\ub7c9\uc740 \ucc29\ub959\ubc14\ud034\ub85c \uc9c0\uc9c0\ub418\ub294\ub370 \ub9cc\uc57d \uc2a4\ud3ec\uc77c\ub7ec\uac00 \uc804\uac1c\ub418\uc5c8\uc744 \uacbd\uc6b0\uc5d0\ub294 \uc0ac\uace0\uae30 \uc911\ub7c9\uc758 65%\ub294 \ucda9\ubd84\ud788 \ucc29\ub959\ubc14\ud034\ub9cc\uc73c\ub85c\ub3c4 \uc9c0\uc9c0\ub420 \uc218 \uc788\uc9c0\ub9cc \uc2a4\ud3ec\uc77c\ub7ec\uac00 \uc5c6\uc744 \uacbd\uc6b0 \uc774 \uc218\uce58\uc5d0\uc11c \uac70\uc758 \ube60\uc9c4 15%\ub85c \ub5a8\uc5b4\uc9c0\uac8c \ub41c\ub2e4. \ucc29\ub959\ubc14\ud034\uc758 \uac00\ubcbc\uc6b4 \uc911\ub7c9\uacfc \ud568\uaed8 \uc0ac\uace0\uae30\uc758 \ube0c\ub808\uc774\ud06c\ub294 \uac10\uc18d\ud558\ub294\ub370 \ud6a8\uacfc\uac00 \uc5c6\uc5c8\uc73c\uba70 \uacc4\uc18d \uace0\uc18d\uc73c\ub85c \ud65c\uc8fc\ub85c\ub97c \uc9c8\uc8fc\ud558\uc600\ub2e4. \ube0c\uc2dc\uba3c \uae30\uc7a5\uc774 \uc5d4\uc9c4 \uc5ed\ucd94\uc9c4 \uc744 \uac1c\uc2dc\ud558\uba74\uc11c \ubc29\ud5a5 \uc81c\uc5b4\ub97c \uc0c1\uc2e4\ud558\uace0 \uc0ac\uace0\uae30\uc758 \ubc29\ud5a5\ud0a4\uc640 \uc218\uc9c1 \uc548\uc815\ud310\uc758 \ud6a8\uacfc\ub3c4 \uac70\uc758 \uc904\uc5b4\ub4e4\uc5c8\ub2e4.", "sentence_answer": "\ube0c\uc2dc\uba3c \uae30\uc7a5\uc774 \uc5d4\uc9c4 \uc5ed\ucd94\uc9c4 \uc744 \uac1c\uc2dc\ud558\uba74\uc11c \ubc29\ud5a5 \uc81c\uc5b4\ub97c \uc0c1\uc2e4\ud558\uace0 \uc0ac\uace0\uae30\uc758 \ubc29\ud5a5\ud0a4\uc640 \uc218\uc9c1 \uc548\uc815\ud310\uc758 \ud6a8\uacfc\ub3c4 \uac70\uc758 \uc904\uc5b4\ub4e4\uc5c8\ub2e4.", "paragraph_id": "6520451-3-2", "paragraph_question": "question: 1420\ud3b8 \ud56d\uacf5\uae30\uac00 \uace0\uc18d\uc73c\ub85c \ud65c\uc8fc\ub85c\ub97c \uc9c8\uc8fc\ud560 \ub54c \uae30\uc7a5\uc774 \uac1c\uc2dc\ud55c \uac83\uc740?, context: 23\uc2dc 50\ubd84 20\ucd08\uc5d0 \uc0ac\uace0\uae30\uac00 4R \ud65c\uc8fc\ub85c\uc5d0 \ub2ff\uc558\ub2e4. \ucc29\ub959\ubc14\ud034\uac00 \ub2ff\uc740\uc9c0 2\ucd08\ud6c4 \ubd80\uae30\uc7a5 \uc624\ub9ac\uac94\uc774 \"\uc6b0\ub9ac\ub294 \ubbf8\ub044\ub7ec\uc9c0\uace0 \uc788\uc5b4\uc694\"\ub77c\uace0 \uc0c1\uae30\ud588\ub2e4. \uc5b4\ub290 \ub204\uad6c\ub3c4 \uc2a4\ud3ec\uc77c\ub7ec\ub97c \uc62c\ub9ac\ub294\ub370 \uc2e4\ud328\ud588\uae30 \ub54c\ubb38\uc774\uc5c8\ub294\ub370 \ucc29\ub959\ud588\uc744\ub54c \uc2a4\ud3ec\uc77c\ub7ec\uac00 \uc790\ub3d9\uc73c\ub85c \uc804\uac1c\ub418\uc9c0 \uc54a\uc740\ub370\ub2e4 \ube44\ud589 \uc2b9\ubb34\uc6d0\ub4e4\ub3c4 \uc218\ub3d9\uc73c\ub85c \uc2a4\ud3ec\uc77c\ub7ec\ub97c \uc62c\ub9ac\ub294\ub370 \uc2e4\ud328\ud588\uae30 \ub54c\ubb38\uc774\uc5c8\ub2e4. \uc2a4\ud3ec\uc77c\ub7ec\uac00 \uacf5\uae30\uc800\ud56d\uc744 \ud615\uc131\ud568\uc73c\ub85c \uc778\ud574 \ud56d\uacf5\uae30\uc758 \uc911\ub7c9\uc740 \ucc29\ub959\ubc14\ud034\ub85c \uc9c0\uc9c0\ub418\ub294\ub370 \ub9cc\uc57d \uc2a4\ud3ec\uc77c\ub7ec\uac00 \uc804\uac1c\ub418\uc5c8\uc744 \uacbd\uc6b0\uc5d0\ub294 \uc0ac\uace0\uae30 \uc911\ub7c9\uc758 65%\ub294 \ucda9\ubd84\ud788 \ucc29\ub959\ubc14\ud034\ub9cc\uc73c\ub85c\ub3c4 \uc9c0\uc9c0\ub420 \uc218 \uc788\uc9c0\ub9cc \uc2a4\ud3ec\uc77c\ub7ec\uac00 \uc5c6\uc744 \uacbd\uc6b0 \uc774 \uc218\uce58\uc5d0\uc11c \uac70\uc758 \ube60\uc9c4 15%\ub85c \ub5a8\uc5b4\uc9c0\uac8c \ub41c\ub2e4. \ucc29\ub959\ubc14\ud034\uc758 \uac00\ubcbc\uc6b4 \uc911\ub7c9\uacfc \ud568\uaed8 \uc0ac\uace0\uae30\uc758 \ube0c\ub808\uc774\ud06c\ub294 \uac10\uc18d\ud558\ub294\ub370 \ud6a8\uacfc\uac00 \uc5c6\uc5c8\uc73c\uba70 \uacc4\uc18d \uace0\uc18d\uc73c\ub85c \ud65c\uc8fc\ub85c\ub97c \uc9c8\uc8fc\ud558\uc600\ub2e4. \ube0c\uc2dc\uba3c \uae30\uc7a5\uc774 \uc5d4\uc9c4 \uc5ed\ucd94\uc9c4\uc744 \uac1c\uc2dc\ud558\uba74\uc11c \ubc29\ud5a5 \uc81c\uc5b4\ub97c \uc0c1\uc2e4\ud558\uace0 \uc0ac\uace0\uae30\uc758 \ubc29\ud5a5\ud0a4\uc640 \uc218\uc9c1 \uc548\uc815\ud310\uc758 \ud6a8\uacfc\ub3c4 \uac70\uc758 \uc904\uc5b4\ub4e4\uc5c8\ub2e4."} {"question": "1748\ub144 \uc190\ubc14\uc774 \uc655\uad6d\uc758 \ud1b5\uce58\uc790 \uc54c\ud3f0\uc18c \uc0b4\ub808\ub9c8, \uc554\ud3ec\uc548\uacfc \uc544\ub9c8\ub204\ubc18\uc758 \uc655\ub4e4\uc774 \ubc18\ub780\uc744 \uc77c\uc73c\ud0a8 \ud6c4 \ub9dd\uba85\ud55c \uc9c0\uc5ed\uc740?", "paragraph": "\ud1a0\ud30c\uc988\uc778\ub4e4\uacfc \uc190\ubc14\uc774 \ubc84\uc0ac\ub974 \uc655\uad6d\uc758 \uc0ac\uc774\uc758 \uad00\uacc4\uc5d0\uc11c\ub294 \ud611\ub825\uacfc \ubc18\ubaa9\uc774 \uc790\uc8fc \ubc88\ubcf5 \ub418\uc5c8\ub2e4. \uadf8\ub9ac\uace0 \uc804\ud1b5\uc801\uc778 \ubc29\uc2dd\uc758 \uc2dd\ubbfc\uc9c0\ubc30\uac00 \uc218\ubc18\ub418\uc9c0 \ubabb\ud588\ub2e4. \uc190\ubc14\uc774 \ubc84\uc0ac\ub974 \uc655\uad6d\uc758 \uc601\uc8fc\ub4e4\uc740 \uc8fc\uc694 \uc218\uc785\uc6d0\uc778 \ubc31\ub2e8\uc720(\ubc31\ub2e8\ud5a5\uc5d0\uc11c \ucc44\ucde8\ud558\ub294 \uae30\ub984\uc73c\ub85c \ud5a5\uc218\uc758 \uc6d0\ub8cc)\uc744 \ud3ec\ub974\ud22c\uac08\uacfc \ub2e4\ub978 \ud574\uc678 \uc120\ubc15\ub4e4\uc774 \uc120\uc801\ud558\ub294 \ud574\uc548 \uc9c0\uc5ed\uc73c\ub85c \uc6b4\uc1a1\uc744 \ud588\uace0, \uc774\ub7ec\ud55c \ubc31\ub2e8\ud5a5\uc740 \ub9c8\uce74\uc624\uc640 \ub2f9\uc2dc \uc778\ub3c4\ub124\uc2dc\uc544\uc758 \ubc14\ud0c0\ube44\uc544 (Batavia, \uc790\uce74\ub974\ud0c0\uc758 \uc61b \uc774\ub984)\uc73c\ub85c \ud314\ub824 \ub098\uac14\ub2e4. \uadf8\ub7ec\ub294 \uc640\uc911\uc5d0, 1711\ub144~1713\ub144\uacfc 1722\ub144\uc5d0 \uc190\ubc14\uc774 \uc655\uad6d\uacfc \ud1a0\ud30c\uc988\uc778\ub4e4 \uc0ac\uc774\uc5d0 \uc2ec\uac01\ud55c \ubd84\uc7c1\uc774 \ubc1c\uc0dd\ud558\uc600\ub2e4. 1748\ub144 \uc190\ubc14\uc774 \uc655\uad6d\uc758 \ud1b5\uce58\uc790 \uc54c\ud3f0\uc18c \uc0b4\ub808\ub9c8 (Alfonso Salema), \uc554\ud3ec\uc548 (Afoan)\uacfc \uc544\ub9c8\ub204\ubc18 (Amanuban)\uc758 \uc655\ub4e4\uc740 \ubc18\ub780\uc744 \uc77c\uc73c\ud0a8\ud6c4, \ucfe0\ud321 \uc9c0\uc5ed\uc73c\ub85c \ub9dd\uba85\uc744 \ud55c\ub2e4. \uc54c\ud3f0\uc18c \uc0b4\ub808\ub9c8\ub294 \ub9ce\uc740 \uc218\uc758 \uc790\uae30\uc758 \ucd94\uc885 \uc138\ub825\ub4e4\uacfc \ud568\uaed8 \ucfe0\ud321 \uc9c0\uc5ed\uc73c\ub85c \uc774\ub3d9\uc744 \ud588\uace0, \uc774\ub294 \ucfe0\ud321 \uc9c0\uc5ed\uc5d0 \uc704\uce58\ud55c \ub124\ub35c\ub780\ub4dc \ub3d9\uc778\ub3c4 \ud68c\uc0ac\uc640 \ud1a0\ud30c\uc988 \ubd80\uc871\uc758 \uc9c0\ub3c4\uc790\uc778 \uac00\uc2a4\ud30c \ub2e4 \ucf54\uc2a4\ud0c0 (Gaspar da Costa)\uc640\uc758 \uadf9\ub3c4\uc758 \ubd84\uc7c1\uc758 \ubc1c\ub2e8\uc774 \ub418\uc5c8\ub2e4. 1749\ub144 11\uc6d4 \ud39c\ud478\uc774 \uc804\ud22c\uc5d0\uc11c \ud1a0\ud30c\uc988 \uad70\uc0ac\ub4e4\uc740 \ucfe0\ud321\uc758 \ub124\ub35c\ub780\ub4dc \ub3d9\uc778\ub3c4 \ud68c\uc0ac \uc5f0\ud569\uad70\ub4e4\uc5d0 \uc758\ud574\uc11c \ud328\ubc30\ub97c \ud558\uace0 \ub9cc\ub2e4. \uadf8\ub9ac\uace0 \uc11c\ud2f0\ubaa8\ub974 \uc9c0\uc5ed\uc758 \uc0c1\ub2f9\ubd80\ubd84\uc758 \uc601\ud1a0\uac00 \ub124\ub35c\ub780\ub4dc \ub3d9\uc778\ub3c4 \ud68c\uc0ac\uc758 \uc601\ud5a5\ub825 \uc544\ub798 \ub193\uc774\uac8c \ub41c\ub2e4.", "answer": "\ucfe0\ud321", "sentence": "1748\ub144 \uc190\ubc14\uc774 \uc655\uad6d\uc758 \ud1b5\uce58\uc790 \uc54c\ud3f0\uc18c \uc0b4\ub808\ub9c8 (Alfonso Salema), \uc554\ud3ec\uc548 (Afoan)\uacfc \uc544\ub9c8\ub204\ubc18 (Amanuban)\uc758 \uc655\ub4e4\uc740 \ubc18\ub780\uc744 \uc77c\uc73c\ud0a8\ud6c4, \ucfe0\ud321 \uc9c0\uc5ed\uc73c\ub85c \ub9dd\uba85\uc744 \ud55c\ub2e4.", "paragraph_sentence": "\ud1a0\ud30c\uc988\uc778\ub4e4\uacfc \uc190\ubc14\uc774 \ubc84\uc0ac\ub974 \uc655\uad6d\uc758 \uc0ac\uc774\uc758 \uad00\uacc4\uc5d0\uc11c\ub294 \ud611\ub825\uacfc \ubc18\ubaa9\uc774 \uc790\uc8fc \ubc88\ubcf5 \ub418\uc5c8\ub2e4. \uadf8\ub9ac\uace0 \uc804\ud1b5\uc801\uc778 \ubc29\uc2dd\uc758 \uc2dd\ubbfc\uc9c0\ubc30\uac00 \uc218\ubc18\ub418\uc9c0 \ubabb\ud588\ub2e4. \uc190\ubc14\uc774 \ubc84\uc0ac\ub974 \uc655\uad6d\uc758 \uc601\uc8fc\ub4e4\uc740 \uc8fc\uc694 \uc218\uc785\uc6d0\uc778 \ubc31\ub2e8\uc720(\ubc31\ub2e8\ud5a5\uc5d0\uc11c \ucc44\ucde8\ud558\ub294 \uae30\ub984\uc73c\ub85c \ud5a5\uc218\uc758 \uc6d0\ub8cc)\uc744 \ud3ec\ub974\ud22c\uac08\uacfc \ub2e4\ub978 \ud574\uc678 \uc120\ubc15\ub4e4\uc774 \uc120\uc801\ud558\ub294 \ud574\uc548 \uc9c0\uc5ed\uc73c\ub85c \uc6b4\uc1a1\uc744 \ud588\uace0, \uc774\ub7ec\ud55c \ubc31\ub2e8\ud5a5\uc740 \ub9c8\uce74\uc624\uc640 \ub2f9\uc2dc \uc778\ub3c4\ub124\uc2dc\uc544\uc758 \ubc14\ud0c0\ube44\uc544 (Batavia, \uc790\uce74\ub974\ud0c0\uc758 \uc61b \uc774\ub984)\uc73c\ub85c \ud314\ub824 \ub098\uac14\ub2e4. \uadf8\ub7ec\ub294 \uc640\uc911\uc5d0, 1711\ub144~1713\ub144\uacfc 1722\ub144\uc5d0 \uc190\ubc14\uc774 \uc655\uad6d\uacfc \ud1a0\ud30c\uc988\uc778\ub4e4 \uc0ac\uc774\uc5d0 \uc2ec\uac01\ud55c \ubd84\uc7c1\uc774 \ubc1c\uc0dd\ud558\uc600\ub2e4. 1748\ub144 \uc190\ubc14\uc774 \uc655\uad6d\uc758 \ud1b5\uce58\uc790 \uc54c\ud3f0\uc18c \uc0b4\ub808\ub9c8 (Alfonso Salema), \uc554\ud3ec\uc548 (Afoan)\uacfc \uc544\ub9c8\ub204\ubc18 (Amanuban)\uc758 \uc655\ub4e4\uc740 \ubc18\ub780\uc744 \uc77c\uc73c\ud0a8\ud6c4, \ucfe0\ud321 \uc9c0\uc5ed\uc73c\ub85c \ub9dd\uba85\uc744 \ud55c\ub2e4. \uc54c\ud3f0\uc18c \uc0b4\ub808\ub9c8\ub294 \ub9ce\uc740 \uc218\uc758 \uc790\uae30\uc758 \ucd94\uc885 \uc138\ub825\ub4e4\uacfc \ud568\uaed8 \ucfe0\ud321 \uc9c0\uc5ed\uc73c\ub85c \uc774\ub3d9\uc744 \ud588\uace0, \uc774\ub294 \ucfe0\ud321 \uc9c0\uc5ed\uc5d0 \uc704\uce58\ud55c \ub124\ub35c\ub780\ub4dc \ub3d9\uc778\ub3c4 \ud68c\uc0ac\uc640 \ud1a0\ud30c\uc988 \ubd80\uc871\uc758 \uc9c0\ub3c4\uc790\uc778 \uac00\uc2a4\ud30c \ub2e4 \ucf54\uc2a4\ud0c0 (Gaspar da Costa)\uc640\uc758 \uadf9\ub3c4\uc758 \ubd84\uc7c1\uc758 \ubc1c\ub2e8\uc774 \ub418\uc5c8\ub2e4. 1749\ub144 11\uc6d4 \ud39c\ud478\uc774 \uc804\ud22c\uc5d0\uc11c \ud1a0\ud30c\uc988 \uad70\uc0ac\ub4e4\uc740 \ucfe0\ud321\uc758 \ub124\ub35c\ub780\ub4dc \ub3d9\uc778\ub3c4 \ud68c\uc0ac \uc5f0\ud569\uad70\ub4e4\uc5d0 \uc758\ud574\uc11c \ud328\ubc30\ub97c \ud558\uace0 \ub9cc\ub2e4. \uadf8\ub9ac\uace0 \uc11c\ud2f0\ubaa8\ub974 \uc9c0\uc5ed\uc758 \uc0c1\ub2f9\ubd80\ubd84\uc758 \uc601\ud1a0\uac00 \ub124\ub35c\ub780\ub4dc \ub3d9\uc778\ub3c4 \ud68c\uc0ac\uc758 \uc601\ud5a5\ub825 \uc544\ub798 \ub193\uc774\uac8c \ub41c\ub2e4.", "paragraph_answer": "\ud1a0\ud30c\uc988\uc778\ub4e4\uacfc \uc190\ubc14\uc774 \ubc84\uc0ac\ub974 \uc655\uad6d\uc758 \uc0ac\uc774\uc758 \uad00\uacc4\uc5d0\uc11c\ub294 \ud611\ub825\uacfc \ubc18\ubaa9\uc774 \uc790\uc8fc \ubc88\ubcf5 \ub418\uc5c8\ub2e4. \uadf8\ub9ac\uace0 \uc804\ud1b5\uc801\uc778 \ubc29\uc2dd\uc758 \uc2dd\ubbfc\uc9c0\ubc30\uac00 \uc218\ubc18\ub418\uc9c0 \ubabb\ud588\ub2e4. \uc190\ubc14\uc774 \ubc84\uc0ac\ub974 \uc655\uad6d\uc758 \uc601\uc8fc\ub4e4\uc740 \uc8fc\uc694 \uc218\uc785\uc6d0\uc778 \ubc31\ub2e8\uc720(\ubc31\ub2e8\ud5a5\uc5d0\uc11c \ucc44\ucde8\ud558\ub294 \uae30\ub984\uc73c\ub85c \ud5a5\uc218\uc758 \uc6d0\ub8cc)\uc744 \ud3ec\ub974\ud22c\uac08\uacfc \ub2e4\ub978 \ud574\uc678 \uc120\ubc15\ub4e4\uc774 \uc120\uc801\ud558\ub294 \ud574\uc548 \uc9c0\uc5ed\uc73c\ub85c \uc6b4\uc1a1\uc744 \ud588\uace0, \uc774\ub7ec\ud55c \ubc31\ub2e8\ud5a5\uc740 \ub9c8\uce74\uc624\uc640 \ub2f9\uc2dc \uc778\ub3c4\ub124\uc2dc\uc544\uc758 \ubc14\ud0c0\ube44\uc544 (Batavia, \uc790\uce74\ub974\ud0c0\uc758 \uc61b \uc774\ub984)\uc73c\ub85c \ud314\ub824 \ub098\uac14\ub2e4. \uadf8\ub7ec\ub294 \uc640\uc911\uc5d0, 1711\ub144~1713\ub144\uacfc 1722\ub144\uc5d0 \uc190\ubc14\uc774 \uc655\uad6d\uacfc \ud1a0\ud30c\uc988\uc778\ub4e4 \uc0ac\uc774\uc5d0 \uc2ec\uac01\ud55c \ubd84\uc7c1\uc774 \ubc1c\uc0dd\ud558\uc600\ub2e4. 1748\ub144 \uc190\ubc14\uc774 \uc655\uad6d\uc758 \ud1b5\uce58\uc790 \uc54c\ud3f0\uc18c \uc0b4\ub808\ub9c8 (Alfonso Salema), \uc554\ud3ec\uc548 (Afoan)\uacfc \uc544\ub9c8\ub204\ubc18 (Amanuban)\uc758 \uc655\ub4e4\uc740 \ubc18\ub780\uc744 \uc77c\uc73c\ud0a8\ud6c4, \ucfe0\ud321 \uc9c0\uc5ed\uc73c\ub85c \ub9dd\uba85\uc744 \ud55c\ub2e4. \uc54c\ud3f0\uc18c \uc0b4\ub808\ub9c8\ub294 \ub9ce\uc740 \uc218\uc758 \uc790\uae30\uc758 \ucd94\uc885 \uc138\ub825\ub4e4\uacfc \ud568\uaed8 \ucfe0\ud321 \uc9c0\uc5ed\uc73c\ub85c \uc774\ub3d9\uc744 \ud588\uace0, \uc774\ub294 \ucfe0\ud321 \uc9c0\uc5ed\uc5d0 \uc704\uce58\ud55c \ub124\ub35c\ub780\ub4dc \ub3d9\uc778\ub3c4 \ud68c\uc0ac\uc640 \ud1a0\ud30c\uc988 \ubd80\uc871\uc758 \uc9c0\ub3c4\uc790\uc778 \uac00\uc2a4\ud30c \ub2e4 \ucf54\uc2a4\ud0c0 (Gaspar da Costa)\uc640\uc758 \uadf9\ub3c4\uc758 \ubd84\uc7c1\uc758 \ubc1c\ub2e8\uc774 \ub418\uc5c8\ub2e4. 1749\ub144 11\uc6d4 \ud39c\ud478\uc774 \uc804\ud22c\uc5d0\uc11c \ud1a0\ud30c\uc988 \uad70\uc0ac\ub4e4\uc740 \ucfe0\ud321\uc758 \ub124\ub35c\ub780\ub4dc \ub3d9\uc778\ub3c4 \ud68c\uc0ac \uc5f0\ud569\uad70\ub4e4\uc5d0 \uc758\ud574\uc11c \ud328\ubc30\ub97c \ud558\uace0 \ub9cc\ub2e4. \uadf8\ub9ac\uace0 \uc11c\ud2f0\ubaa8\ub974 \uc9c0\uc5ed\uc758 \uc0c1\ub2f9\ubd80\ubd84\uc758 \uc601\ud1a0\uac00 \ub124\ub35c\ub780\ub4dc \ub3d9\uc778\ub3c4 \ud68c\uc0ac\uc758 \uc601\ud5a5\ub825 \uc544\ub798 \ub193\uc774\uac8c \ub41c\ub2e4.", "sentence_answer": "1748\ub144 \uc190\ubc14\uc774 \uc655\uad6d\uc758 \ud1b5\uce58\uc790 \uc54c\ud3f0\uc18c \uc0b4\ub808\ub9c8 (Alfonso Salema), \uc554\ud3ec\uc548 (Afoan)\uacfc \uc544\ub9c8\ub204\ubc18 (Amanuban)\uc758 \uc655\ub4e4\uc740 \ubc18\ub780\uc744 \uc77c\uc73c\ud0a8\ud6c4, \ucfe0\ud321 \uc9c0\uc5ed\uc73c\ub85c \ub9dd\uba85\uc744 \ud55c\ub2e4.", "paragraph_id": "6446901-2-0", "paragraph_question": "question: 1748\ub144 \uc190\ubc14\uc774 \uc655\uad6d\uc758 \ud1b5\uce58\uc790 \uc54c\ud3f0\uc18c \uc0b4\ub808\ub9c8, \uc554\ud3ec\uc548\uacfc \uc544\ub9c8\ub204\ubc18\uc758 \uc655\ub4e4\uc774 \ubc18\ub780\uc744 \uc77c\uc73c\ud0a8 \ud6c4 \ub9dd\uba85\ud55c \uc9c0\uc5ed\uc740?, context: \ud1a0\ud30c\uc988\uc778\ub4e4\uacfc \uc190\ubc14\uc774 \ubc84\uc0ac\ub974 \uc655\uad6d\uc758 \uc0ac\uc774\uc758 \uad00\uacc4\uc5d0\uc11c\ub294 \ud611\ub825\uacfc \ubc18\ubaa9\uc774 \uc790\uc8fc \ubc88\ubcf5 \ub418\uc5c8\ub2e4. \uadf8\ub9ac\uace0 \uc804\ud1b5\uc801\uc778 \ubc29\uc2dd\uc758 \uc2dd\ubbfc\uc9c0\ubc30\uac00 \uc218\ubc18\ub418\uc9c0 \ubabb\ud588\ub2e4. \uc190\ubc14\uc774 \ubc84\uc0ac\ub974 \uc655\uad6d\uc758 \uc601\uc8fc\ub4e4\uc740 \uc8fc\uc694 \uc218\uc785\uc6d0\uc778 \ubc31\ub2e8\uc720(\ubc31\ub2e8\ud5a5\uc5d0\uc11c \ucc44\ucde8\ud558\ub294 \uae30\ub984\uc73c\ub85c \ud5a5\uc218\uc758 \uc6d0\ub8cc)\uc744 \ud3ec\ub974\ud22c\uac08\uacfc \ub2e4\ub978 \ud574\uc678 \uc120\ubc15\ub4e4\uc774 \uc120\uc801\ud558\ub294 \ud574\uc548 \uc9c0\uc5ed\uc73c\ub85c \uc6b4\uc1a1\uc744 \ud588\uace0, \uc774\ub7ec\ud55c \ubc31\ub2e8\ud5a5\uc740 \ub9c8\uce74\uc624\uc640 \ub2f9\uc2dc \uc778\ub3c4\ub124\uc2dc\uc544\uc758 \ubc14\ud0c0\ube44\uc544 (Batavia, \uc790\uce74\ub974\ud0c0\uc758 \uc61b \uc774\ub984)\uc73c\ub85c \ud314\ub824 \ub098\uac14\ub2e4. \uadf8\ub7ec\ub294 \uc640\uc911\uc5d0, 1711\ub144~1713\ub144\uacfc 1722\ub144\uc5d0 \uc190\ubc14\uc774 \uc655\uad6d\uacfc \ud1a0\ud30c\uc988\uc778\ub4e4 \uc0ac\uc774\uc5d0 \uc2ec\uac01\ud55c \ubd84\uc7c1\uc774 \ubc1c\uc0dd\ud558\uc600\ub2e4. 1748\ub144 \uc190\ubc14\uc774 \uc655\uad6d\uc758 \ud1b5\uce58\uc790 \uc54c\ud3f0\uc18c \uc0b4\ub808\ub9c8 (Alfonso Salema), \uc554\ud3ec\uc548 (Afoan)\uacfc \uc544\ub9c8\ub204\ubc18 (Amanuban)\uc758 \uc655\ub4e4\uc740 \ubc18\ub780\uc744 \uc77c\uc73c\ud0a8\ud6c4, \ucfe0\ud321 \uc9c0\uc5ed\uc73c\ub85c \ub9dd\uba85\uc744 \ud55c\ub2e4. \uc54c\ud3f0\uc18c \uc0b4\ub808\ub9c8\ub294 \ub9ce\uc740 \uc218\uc758 \uc790\uae30\uc758 \ucd94\uc885 \uc138\ub825\ub4e4\uacfc \ud568\uaed8 \ucfe0\ud321 \uc9c0\uc5ed\uc73c\ub85c \uc774\ub3d9\uc744 \ud588\uace0, \uc774\ub294 \ucfe0\ud321 \uc9c0\uc5ed\uc5d0 \uc704\uce58\ud55c \ub124\ub35c\ub780\ub4dc \ub3d9\uc778\ub3c4 \ud68c\uc0ac\uc640 \ud1a0\ud30c\uc988 \ubd80\uc871\uc758 \uc9c0\ub3c4\uc790\uc778 \uac00\uc2a4\ud30c \ub2e4 \ucf54\uc2a4\ud0c0 (Gaspar da Costa)\uc640\uc758 \uadf9\ub3c4\uc758 \ubd84\uc7c1\uc758 \ubc1c\ub2e8\uc774 \ub418\uc5c8\ub2e4. 1749\ub144 11\uc6d4 \ud39c\ud478\uc774 \uc804\ud22c\uc5d0\uc11c \ud1a0\ud30c\uc988 \uad70\uc0ac\ub4e4\uc740 \ucfe0\ud321\uc758 \ub124\ub35c\ub780\ub4dc \ub3d9\uc778\ub3c4 \ud68c\uc0ac \uc5f0\ud569\uad70\ub4e4\uc5d0 \uc758\ud574\uc11c \ud328\ubc30\ub97c \ud558\uace0 \ub9cc\ub2e4. \uadf8\ub9ac\uace0 \uc11c\ud2f0\ubaa8\ub974 \uc9c0\uc5ed\uc758 \uc0c1\ub2f9\ubd80\ubd84\uc758 \uc601\ud1a0\uac00 \ub124\ub35c\ub780\ub4dc \ub3d9\uc778\ub3c4 \ud68c\uc0ac\uc758 \uc601\ud5a5\ub825 \uc544\ub798 \ub193\uc774\uac8c \ub41c\ub2e4."} {"question": "\ubd80\uc0b0\uad11\uc5ed\uc2dc \uad50\ud1b5\uad6d\uc7a5 \uae40\ud638\uc601\uc5d0 \ub530\ub974\uba74, KTX\uc758 \uc644\uc804\ud55c \uac1c\ud1b5\uc5d0\ub3c4 \uc624\ud788\ub824 \uc774\uc6a9\uac1d\uc218\uac00 \ub298\uc5b4\ub098\uace0 \uc788\ub294 \uacf5\ud56d\uc740 \uc5b4\ub514\uc778\uac00?", "paragraph": "2008\ub144 9\uc6d4 \ub3d9\ub0a8\uad8c \uc2e0\uacf5\ud56d\uc744 30\ub300 \uad11\uc5ed \uc120\ub3c4 \ud504\ub85c\uc81d\ud2b8\ub85c \uc120\uc815\ud588\uace0 2009\ub144 4\uc6d4 \uad6d\ud1a0\uc5f0\uad6c\uc6d0 \ub3d9\ub0a8\uad8c \uc2e0\uacf5\ud56d \ucd5c\ucd08 35\uac1c \ud6c4\ubcf4\uc9c0\uc5d0\uc11c 5\uac1c \ud6c4\ubcf4\uc9c0\ub85c \uc555\ucd95\ubc1c\ud45c\ud588\ub2e4. \uc774\ub54c\ubd80\ud130 \ubc00\uc591, \uac00\ub355\ub3c4\uac00 \uc720\ub825\ud558\ub2e4\ub294 \uc18c\ubb38\uc774 \ub3cc\uae30 \uc2dc\uc791\ud588\ub2e4. 2009\ub144 12\uc6d4\uc5d0\ub294 \uad6d\ud1a0\uc5f0\uad6c\uc6d0\uc758 2\ucc28 \uc6a9\uc5ed\uacb0\uacfc\uac00 \ubc1c\ud45c\ub410\ub2e4. B/C(\ube44\uc6a9\ub300 \ud3b8\uc775:1.0\uc774\ud558\uba74 \uacbd\uc81c\uc131 \ubd80\uc871) \ubc00\uc591 0.73, \uac00\ub355\ub3c4 0.70\uc73c\ub85c \ub098\uc654\ub2e4. \uacbd\uc81c\uc131 \ubd84\uc11d\uacb0\uacfc \ubd80\uc801\uc808\ub85c \uacb0\ub860\uc774 \ub0ac\uc73c\ub098, \ub450 \ud6c4\ubcf4\uc9c0\ub97c \uacc4\uc18d \uac80\ud1a0\ud568\uacfc \ud568\uaed8 \uae40\ud574\uacf5\ud56d \ud655\uc7a5\uc548\ub3c4 \uac80\ud1a0\ub418\uc5c8\ub2e4. \uc774\ucc98\ub7fc \uc0ac\uc5c5 \ud0c0\ub2f9\uc131\uc5d0 \ub300\ud55c \uc9c0\uc801\uc744 \ud558\ub294 \uc804\ubb38\uac00\ub4e4\uc774 \ub9ce\uc558\ub2e4. \ud5c8\ud76c\uc601 \ud55c\uad6d\ud56d\uacf5\ub300\ud559\uad50 \uacbd\uc601\ud559\uacfc \uad50\uc218\ub294 \"\uc2e0\uacf5\ud56d\uc740 \ud0c0\ub2f9\uc131\uc774 \uc5c6\ub2e4\uace0 \ubcf8\ub2e4. \uc774 \uc0ac\uc5c5\uc740 \uc218\uc694\uac00 2025\ub144\ubd80\ud130 10\uc870\uc6d0 \uc774\uc0c1 \ud22c\uc790\ub418\ub294 \uc2e0\uacf5\ud56d\uc5d0 \ub300\ud55c \uc218\uc694\uac00 \uadf8\ub9cc\ud07c \uc77c\uc5b4\ub098\uaca0\ub290\ub0d0\"\ub294 \uac83\uc774\ub2e4. \uadf8\ub7ec\ub098 \uc774\ub7ec\ud55c \ud68c\uc758\ub860\uc5d0 \ub300\ud574 \uc9c0\uc790\uccb4\ub4e4\uc740 \uc989\uac01 \ubc18\ubc1c\ud588\ub2e4. \uc9c0\uc790\uccb4\ub4e4\uc758 \uc785\uc7a5\uc744 \ubaa8\ub974\ub294 \uc218\ub3c4\uad8c \ub17c\ub9ac\ub77c\ub294 \uac83\uc774\ub2e4. \uae40\ud638\uc601 \ubd80\uc0b0\uad11\uc5ed\uc2dc \uad50\ud1b5\uad6d\uc7a5\uc740 \"\uae40\ud574\uacf5\ud56d\uc774 KTX\uac00 \uc644\uc804 \uac1c\ud1b5\ub428\uc5d0\ub3c4 \ubd88\uad6c\ud558\uace0 \uc624\ud788\ub824 \uae40\ud574\uacf5\ud56d \uc774\uc6a9\uac1d\uc218\uac00 \ub298\uc5b4\ub098\uace0 \uc788\ub294 \uc0c1\ud669\uc744 \ubcf4\uba74\uc11c\ub3c4 \uc218\ub3c4\uad8c\uc5d0\uc11c\ub294 \ub3d9\ub0a8\uad8c \uc2e0\uacf5\ud56d\uc774 \ud544\uc694 \uc5c6\ub2e4\ub294 \uc8fc\uc7a5\uc744 \ud55c\ub2e4\ub294 \uac83\uc740 \uc9c0\uc790\uccb4 \uc785\uc7a5\uc744 \ubaa8\ub974\ub294 \uac83\"\uc774\ub77c\uace0 \uc9c0\uc801\ud588\ub2e4. \ubc15\uad11\uae38 \uc2e0\uad6d\uc81c\uacf5\ud56d \ubc00\uc591\uc720\uce58 \ucd94\uc9c4\ub2e8\uc7a5\ub3c4 \"\uc77c\ubd80 \uc815\uce58\uc778\ub4e4\uc774 \ud55c \ubc1c\uc5b8\uc740 \uc790\uc2e0\uc758 \uc815\uce58\uc801 \uc785\uc9c0\uac00 \uc57d\ud654\ub418\ub294 \uac83\uc744 \uc6b0\ub824\ud55c \ubc1c\uc5b8\uc73c\ub85c \uc7ac\uace0\ud560 \uac00\uce58\uac00 \uc5c6\ub294 \ubc1c\uc5b8\uc774\ub2e4\"\ub77c\uace0 \uc77c\ucd95\ud588\ub2e4.", "answer": "\uae40\ud574\uacf5\ud56d", "sentence": "\uacbd\uc81c\uc131 \ubd84\uc11d\uacb0\uacfc \ubd80\uc801\uc808\ub85c \uacb0\ub860\uc774 \ub0ac\uc73c\ub098, \ub450 \ud6c4\ubcf4\uc9c0\ub97c \uacc4\uc18d \uac80\ud1a0\ud568\uacfc \ud568\uaed8 \uae40\ud574\uacf5\ud56d \ud655\uc7a5\uc548\ub3c4 \uac80\ud1a0\ub418\uc5c8\ub2e4.", "paragraph_sentence": "2008\ub144 9\uc6d4 \ub3d9\ub0a8\uad8c \uc2e0\uacf5\ud56d\uc744 30\ub300 \uad11\uc5ed \uc120\ub3c4 \ud504\ub85c\uc81d\ud2b8\ub85c \uc120\uc815\ud588\uace0 2009\ub144 4\uc6d4 \uad6d\ud1a0\uc5f0\uad6c\uc6d0 \ub3d9\ub0a8\uad8c \uc2e0\uacf5\ud56d \ucd5c\ucd08 35\uac1c \ud6c4\ubcf4\uc9c0\uc5d0\uc11c 5\uac1c \ud6c4\ubcf4\uc9c0\ub85c \uc555\ucd95\ubc1c\ud45c\ud588\ub2e4. \uc774\ub54c\ubd80\ud130 \ubc00\uc591, \uac00\ub355\ub3c4\uac00 \uc720\ub825\ud558\ub2e4\ub294 \uc18c\ubb38\uc774 \ub3cc\uae30 \uc2dc\uc791\ud588\ub2e4. 2009\ub144 12\uc6d4\uc5d0\ub294 \uad6d\ud1a0\uc5f0\uad6c\uc6d0\uc758 2\ucc28 \uc6a9\uc5ed\uacb0\uacfc\uac00 \ubc1c\ud45c\ub410\ub2e4. B/C(\ube44\uc6a9\ub300 \ud3b8\uc775:1.0\uc774\ud558\uba74 \uacbd\uc81c\uc131 \ubd80\uc871) \ubc00\uc591 0.73, \uac00\ub355\ub3c4 0.70\uc73c\ub85c \ub098\uc654\ub2e4. \uacbd\uc81c\uc131 \ubd84\uc11d\uacb0\uacfc \ubd80\uc801\uc808\ub85c \uacb0\ub860\uc774 \ub0ac\uc73c\ub098, \ub450 \ud6c4\ubcf4\uc9c0\ub97c \uacc4\uc18d \uac80\ud1a0\ud568\uacfc \ud568\uaed8 \uae40\ud574\uacf5\ud56d \ud655\uc7a5\uc548\ub3c4 \uac80\ud1a0\ub418\uc5c8\ub2e4. \uc774\ucc98\ub7fc \uc0ac\uc5c5 \ud0c0\ub2f9\uc131\uc5d0 \ub300\ud55c \uc9c0\uc801\uc744 \ud558\ub294 \uc804\ubb38\uac00\ub4e4\uc774 \ub9ce\uc558\ub2e4. \ud5c8\ud76c\uc601 \ud55c\uad6d\ud56d\uacf5\ub300\ud559\uad50 \uacbd\uc601\ud559\uacfc \uad50\uc218\ub294 \"\uc2e0\uacf5\ud56d\uc740 \ud0c0\ub2f9\uc131\uc774 \uc5c6\ub2e4\uace0 \ubcf8\ub2e4. \uc774 \uc0ac\uc5c5\uc740 \uc218\uc694\uac00 2025\ub144\ubd80\ud130 10\uc870\uc6d0 \uc774\uc0c1 \ud22c\uc790\ub418\ub294 \uc2e0\uacf5\ud56d\uc5d0 \ub300\ud55c \uc218\uc694\uac00 \uadf8\ub9cc\ud07c \uc77c\uc5b4\ub098\uaca0\ub290\ub0d0\"\ub294 \uac83\uc774\ub2e4. \uadf8\ub7ec\ub098 \uc774\ub7ec\ud55c \ud68c\uc758\ub860\uc5d0 \ub300\ud574 \uc9c0\uc790\uccb4\ub4e4\uc740 \uc989\uac01 \ubc18\ubc1c\ud588\ub2e4. \uc9c0\uc790\uccb4\ub4e4\uc758 \uc785\uc7a5\uc744 \ubaa8\ub974\ub294 \uc218\ub3c4\uad8c \ub17c\ub9ac\ub77c\ub294 \uac83\uc774\ub2e4. \uae40\ud638\uc601 \ubd80\uc0b0\uad11\uc5ed\uc2dc \uad50\ud1b5\uad6d\uc7a5\uc740 \"\uae40\ud574\uacf5\ud56d\uc774 KTX\uac00 \uc644\uc804 \uac1c\ud1b5\ub428\uc5d0\ub3c4 \ubd88\uad6c\ud558\uace0 \uc624\ud788\ub824 \uae40\ud574\uacf5\ud56d \uc774\uc6a9\uac1d\uc218\uac00 \ub298\uc5b4\ub098\uace0 \uc788\ub294 \uc0c1\ud669\uc744 \ubcf4\uba74\uc11c\ub3c4 \uc218\ub3c4\uad8c\uc5d0\uc11c\ub294 \ub3d9\ub0a8\uad8c \uc2e0\uacf5\ud56d\uc774 \ud544\uc694 \uc5c6\ub2e4\ub294 \uc8fc\uc7a5\uc744 \ud55c\ub2e4\ub294 \uac83\uc740 \uc9c0\uc790\uccb4 \uc785\uc7a5\uc744 \ubaa8\ub974\ub294 \uac83\"\uc774\ub77c\uace0 \uc9c0\uc801\ud588\ub2e4. \ubc15\uad11\uae38 \uc2e0\uad6d\uc81c\uacf5\ud56d \ubc00\uc591\uc720\uce58 \ucd94\uc9c4\ub2e8\uc7a5\ub3c4 \"\uc77c\ubd80 \uc815\uce58\uc778\ub4e4\uc774 \ud55c \ubc1c\uc5b8\uc740 \uc790\uc2e0\uc758 \uc815\uce58\uc801 \uc785\uc9c0\uac00 \uc57d\ud654\ub418\ub294 \uac83\uc744 \uc6b0\ub824\ud55c \ubc1c\uc5b8\uc73c\ub85c \uc7ac\uace0\ud560 \uac00\uce58\uac00 \uc5c6\ub294 \ubc1c\uc5b8\uc774\ub2e4\"\ub77c\uace0 \uc77c\ucd95\ud588\ub2e4.", "paragraph_answer": "2008\ub144 9\uc6d4 \ub3d9\ub0a8\uad8c \uc2e0\uacf5\ud56d\uc744 30\ub300 \uad11\uc5ed \uc120\ub3c4 \ud504\ub85c\uc81d\ud2b8\ub85c \uc120\uc815\ud588\uace0 2009\ub144 4\uc6d4 \uad6d\ud1a0\uc5f0\uad6c\uc6d0 \ub3d9\ub0a8\uad8c \uc2e0\uacf5\ud56d \ucd5c\ucd08 35\uac1c \ud6c4\ubcf4\uc9c0\uc5d0\uc11c 5\uac1c \ud6c4\ubcf4\uc9c0\ub85c \uc555\ucd95\ubc1c\ud45c\ud588\ub2e4. \uc774\ub54c\ubd80\ud130 \ubc00\uc591, \uac00\ub355\ub3c4\uac00 \uc720\ub825\ud558\ub2e4\ub294 \uc18c\ubb38\uc774 \ub3cc\uae30 \uc2dc\uc791\ud588\ub2e4. 2009\ub144 12\uc6d4\uc5d0\ub294 \uad6d\ud1a0\uc5f0\uad6c\uc6d0\uc758 2\ucc28 \uc6a9\uc5ed\uacb0\uacfc\uac00 \ubc1c\ud45c\ub410\ub2e4. B/C(\ube44\uc6a9\ub300 \ud3b8\uc775:1.0\uc774\ud558\uba74 \uacbd\uc81c\uc131 \ubd80\uc871) \ubc00\uc591 0.73, \uac00\ub355\ub3c4 0.70\uc73c\ub85c \ub098\uc654\ub2e4. \uacbd\uc81c\uc131 \ubd84\uc11d\uacb0\uacfc \ubd80\uc801\uc808\ub85c \uacb0\ub860\uc774 \ub0ac\uc73c\ub098, \ub450 \ud6c4\ubcf4\uc9c0\ub97c \uacc4\uc18d \uac80\ud1a0\ud568\uacfc \ud568\uaed8 \uae40\ud574\uacf5\ud56d \ud655\uc7a5\uc548\ub3c4 \uac80\ud1a0\ub418\uc5c8\ub2e4. \uc774\ucc98\ub7fc \uc0ac\uc5c5 \ud0c0\ub2f9\uc131\uc5d0 \ub300\ud55c \uc9c0\uc801\uc744 \ud558\ub294 \uc804\ubb38\uac00\ub4e4\uc774 \ub9ce\uc558\ub2e4. \ud5c8\ud76c\uc601 \ud55c\uad6d\ud56d\uacf5\ub300\ud559\uad50 \uacbd\uc601\ud559\uacfc \uad50\uc218\ub294 \"\uc2e0\uacf5\ud56d\uc740 \ud0c0\ub2f9\uc131\uc774 \uc5c6\ub2e4\uace0 \ubcf8\ub2e4. \uc774 \uc0ac\uc5c5\uc740 \uc218\uc694\uac00 2025\ub144\ubd80\ud130 10\uc870\uc6d0 \uc774\uc0c1 \ud22c\uc790\ub418\ub294 \uc2e0\uacf5\ud56d\uc5d0 \ub300\ud55c \uc218\uc694\uac00 \uadf8\ub9cc\ud07c \uc77c\uc5b4\ub098\uaca0\ub290\ub0d0\"\ub294 \uac83\uc774\ub2e4. \uadf8\ub7ec\ub098 \uc774\ub7ec\ud55c \ud68c\uc758\ub860\uc5d0 \ub300\ud574 \uc9c0\uc790\uccb4\ub4e4\uc740 \uc989\uac01 \ubc18\ubc1c\ud588\ub2e4. \uc9c0\uc790\uccb4\ub4e4\uc758 \uc785\uc7a5\uc744 \ubaa8\ub974\ub294 \uc218\ub3c4\uad8c \ub17c\ub9ac\ub77c\ub294 \uac83\uc774\ub2e4. \uae40\ud638\uc601 \ubd80\uc0b0\uad11\uc5ed\uc2dc \uad50\ud1b5\uad6d\uc7a5\uc740 \"\uae40\ud574\uacf5\ud56d\uc774 KTX\uac00 \uc644\uc804 \uac1c\ud1b5\ub428\uc5d0\ub3c4 \ubd88\uad6c\ud558\uace0 \uc624\ud788\ub824 \uae40\ud574\uacf5\ud56d \uc774\uc6a9\uac1d\uc218\uac00 \ub298\uc5b4\ub098\uace0 \uc788\ub294 \uc0c1\ud669\uc744 \ubcf4\uba74\uc11c\ub3c4 \uc218\ub3c4\uad8c\uc5d0\uc11c\ub294 \ub3d9\ub0a8\uad8c \uc2e0\uacf5\ud56d\uc774 \ud544\uc694 \uc5c6\ub2e4\ub294 \uc8fc\uc7a5\uc744 \ud55c\ub2e4\ub294 \uac83\uc740 \uc9c0\uc790\uccb4 \uc785\uc7a5\uc744 \ubaa8\ub974\ub294 \uac83\"\uc774\ub77c\uace0 \uc9c0\uc801\ud588\ub2e4. \ubc15\uad11\uae38 \uc2e0\uad6d\uc81c\uacf5\ud56d \ubc00\uc591\uc720\uce58 \ucd94\uc9c4\ub2e8\uc7a5\ub3c4 \"\uc77c\ubd80 \uc815\uce58\uc778\ub4e4\uc774 \ud55c \ubc1c\uc5b8\uc740 \uc790\uc2e0\uc758 \uc815\uce58\uc801 \uc785\uc9c0\uac00 \uc57d\ud654\ub418\ub294 \uac83\uc744 \uc6b0\ub824\ud55c \ubc1c\uc5b8\uc73c\ub85c \uc7ac\uace0\ud560 \uac00\uce58\uac00 \uc5c6\ub294 \ubc1c\uc5b8\uc774\ub2e4\"\ub77c\uace0 \uc77c\ucd95\ud588\ub2e4.", "sentence_answer": "\uacbd\uc81c\uc131 \ubd84\uc11d\uacb0\uacfc \ubd80\uc801\uc808\ub85c \uacb0\ub860\uc774 \ub0ac\uc73c\ub098, \ub450 \ud6c4\ubcf4\uc9c0\ub97c \uacc4\uc18d \uac80\ud1a0\ud568\uacfc \ud568\uaed8 \uae40\ud574\uacf5\ud56d \ud655\uc7a5\uc548\ub3c4 \uac80\ud1a0\ub418\uc5c8\ub2e4.", "paragraph_id": "6524258-1-2", "paragraph_question": "question: \ubd80\uc0b0\uad11\uc5ed\uc2dc \uad50\ud1b5\uad6d\uc7a5 \uae40\ud638\uc601\uc5d0 \ub530\ub974\uba74, KTX\uc758 \uc644\uc804\ud55c \uac1c\ud1b5\uc5d0\ub3c4 \uc624\ud788\ub824 \uc774\uc6a9\uac1d\uc218\uac00 \ub298\uc5b4\ub098\uace0 \uc788\ub294 \uacf5\ud56d\uc740 \uc5b4\ub514\uc778\uac00?, context: 2008\ub144 9\uc6d4 \ub3d9\ub0a8\uad8c \uc2e0\uacf5\ud56d\uc744 30\ub300 \uad11\uc5ed \uc120\ub3c4 \ud504\ub85c\uc81d\ud2b8\ub85c \uc120\uc815\ud588\uace0 2009\ub144 4\uc6d4 \uad6d\ud1a0\uc5f0\uad6c\uc6d0 \ub3d9\ub0a8\uad8c \uc2e0\uacf5\ud56d \ucd5c\ucd08 35\uac1c \ud6c4\ubcf4\uc9c0\uc5d0\uc11c 5\uac1c \ud6c4\ubcf4\uc9c0\ub85c \uc555\ucd95\ubc1c\ud45c\ud588\ub2e4. \uc774\ub54c\ubd80\ud130 \ubc00\uc591, \uac00\ub355\ub3c4\uac00 \uc720\ub825\ud558\ub2e4\ub294 \uc18c\ubb38\uc774 \ub3cc\uae30 \uc2dc\uc791\ud588\ub2e4. 2009\ub144 12\uc6d4\uc5d0\ub294 \uad6d\ud1a0\uc5f0\uad6c\uc6d0\uc758 2\ucc28 \uc6a9\uc5ed\uacb0\uacfc\uac00 \ubc1c\ud45c\ub410\ub2e4. B/C(\ube44\uc6a9\ub300 \ud3b8\uc775:1.0\uc774\ud558\uba74 \uacbd\uc81c\uc131 \ubd80\uc871) \ubc00\uc591 0.73, \uac00\ub355\ub3c4 0.70\uc73c\ub85c \ub098\uc654\ub2e4. \uacbd\uc81c\uc131 \ubd84\uc11d\uacb0\uacfc \ubd80\uc801\uc808\ub85c \uacb0\ub860\uc774 \ub0ac\uc73c\ub098, \ub450 \ud6c4\ubcf4\uc9c0\ub97c \uacc4\uc18d \uac80\ud1a0\ud568\uacfc \ud568\uaed8 \uae40\ud574\uacf5\ud56d \ud655\uc7a5\uc548\ub3c4 \uac80\ud1a0\ub418\uc5c8\ub2e4. \uc774\ucc98\ub7fc \uc0ac\uc5c5 \ud0c0\ub2f9\uc131\uc5d0 \ub300\ud55c \uc9c0\uc801\uc744 \ud558\ub294 \uc804\ubb38\uac00\ub4e4\uc774 \ub9ce\uc558\ub2e4. \ud5c8\ud76c\uc601 \ud55c\uad6d\ud56d\uacf5\ub300\ud559\uad50 \uacbd\uc601\ud559\uacfc \uad50\uc218\ub294 \"\uc2e0\uacf5\ud56d\uc740 \ud0c0\ub2f9\uc131\uc774 \uc5c6\ub2e4\uace0 \ubcf8\ub2e4. \uc774 \uc0ac\uc5c5\uc740 \uc218\uc694\uac00 2025\ub144\ubd80\ud130 10\uc870\uc6d0 \uc774\uc0c1 \ud22c\uc790\ub418\ub294 \uc2e0\uacf5\ud56d\uc5d0 \ub300\ud55c \uc218\uc694\uac00 \uadf8\ub9cc\ud07c \uc77c\uc5b4\ub098\uaca0\ub290\ub0d0\"\ub294 \uac83\uc774\ub2e4. \uadf8\ub7ec\ub098 \uc774\ub7ec\ud55c \ud68c\uc758\ub860\uc5d0 \ub300\ud574 \uc9c0\uc790\uccb4\ub4e4\uc740 \uc989\uac01 \ubc18\ubc1c\ud588\ub2e4. \uc9c0\uc790\uccb4\ub4e4\uc758 \uc785\uc7a5\uc744 \ubaa8\ub974\ub294 \uc218\ub3c4\uad8c \ub17c\ub9ac\ub77c\ub294 \uac83\uc774\ub2e4. \uae40\ud638\uc601 \ubd80\uc0b0\uad11\uc5ed\uc2dc \uad50\ud1b5\uad6d\uc7a5\uc740 \"\uae40\ud574\uacf5\ud56d\uc774 KTX\uac00 \uc644\uc804 \uac1c\ud1b5\ub428\uc5d0\ub3c4 \ubd88\uad6c\ud558\uace0 \uc624\ud788\ub824 \uae40\ud574\uacf5\ud56d \uc774\uc6a9\uac1d\uc218\uac00 \ub298\uc5b4\ub098\uace0 \uc788\ub294 \uc0c1\ud669\uc744 \ubcf4\uba74\uc11c\ub3c4 \uc218\ub3c4\uad8c\uc5d0\uc11c\ub294 \ub3d9\ub0a8\uad8c \uc2e0\uacf5\ud56d\uc774 \ud544\uc694 \uc5c6\ub2e4\ub294 \uc8fc\uc7a5\uc744 \ud55c\ub2e4\ub294 \uac83\uc740 \uc9c0\uc790\uccb4 \uc785\uc7a5\uc744 \ubaa8\ub974\ub294 \uac83\"\uc774\ub77c\uace0 \uc9c0\uc801\ud588\ub2e4. \ubc15\uad11\uae38 \uc2e0\uad6d\uc81c\uacf5\ud56d \ubc00\uc591\uc720\uce58 \ucd94\uc9c4\ub2e8\uc7a5\ub3c4 \"\uc77c\ubd80 \uc815\uce58\uc778\ub4e4\uc774 \ud55c \ubc1c\uc5b8\uc740 \uc790\uc2e0\uc758 \uc815\uce58\uc801 \uc785\uc9c0\uac00 \uc57d\ud654\ub418\ub294 \uac83\uc744 \uc6b0\ub824\ud55c \ubc1c\uc5b8\uc73c\ub85c \uc7ac\uace0\ud560 \uac00\uce58\uac00 \uc5c6\ub294 \ubc1c\uc5b8\uc774\ub2e4\"\ub77c\uace0 \uc77c\ucd95\ud588\ub2e4."} {"question": "\ube0c\ub9ac\ud2b8\ub2c8 \uc2a4\ud53c\uc5b4\uc2a4\ub294 \ube4c\ubcf4\ub4dc\uac00 \ubc1c\ud45c\ud55c 2000\ub144\ub300 \uc544\ud2f0\uc2a4\ud2b8 \uc21c\uc704\uc5d0\uc11c \uba87 \uc704\uc5d0 \uc62c\ub790\ub098\uc694?", "paragraph": "\ube0c\ub9ac\ud2b8\ub2c8\ub294 \ub370\ubdd4\ud558\uc790\ub9c8\uc790 \ud31d \uc544\uc774\ucf58\uc73c\ub85c \uc790\ub9ac\uc7a1\uc558\uc73c\uba70, 1990\ub144\ub300 \ub9d0 \uc720\ud589\ud55c \ud2f4 \ud31d\uc758 \uc720\ud589\uc744 \uc120\ub3c4\ud55c \uc778\ubb3c\ub85c \uc778\uc815\ubc1b\uace0\uc788\ub2e4. \ub610 20\uc0b4\uc774 \ub418\uae30 \uc804 10\ub300 \uac00\uc218\ub85c\ub294 \uac00\uc7a5 \ub192\uc740 \uc74c\ubc18 \ud310\ub9e4\ub7c9\uc744 \uae30\ub85d\ud588\uace0, \uc774\ub7ec\ud55c \uc131\uacfc\ub85c \"\ud31d\uc758 \uacf5\uc8fc\"\uc640 \uac19\uc740 \uce6d\ud638\ub97c \uc5bb\uc5c8\ub2e4. \ube0c\ub9ac\ud2b8\ub2c8\uc758 \uc791\ud488\uacfc \uc131\uacfc\ub294 \uc218 \ub9ce\uc740 \uc2dc\uc0c1\uc2dd\uacfc \uae30\ub85d\uc744 \ud1b5\ud574 \uc778\uc815\ubc1b\uc558\ub294\ub370, \ub300\ud45c\uc801\uc73c\ub85c 1\ubc88\uc758 \uadf8\ub798\ubbf8 \uc0c1 \uc218\uc0c1, \uacf5\ub85c\uc0c1\uc744 \ud3ec\ud568\ud55c 6\ubc88\uc758 MTV \ube44\ub514\uc624 \ubba4\uc9c1 \uc5b4\uc6cc\ub4dc \uc218\uc0c1, 9\ubc88\uc758 \ube4c\ubcf4\ub4dc \ubba4\uc9c1 \uc5b4\uc6cc\ub4dc \uc218\uc0c1, \uadf8\ub9ac\uace0 \ud560\ub9ac\uc6b0\ub4dc \uba85\uc608\uc758 \uac70\ub9ac\uc5d0 \uc774\ub984\uc744 \uc62c\ub838\ub2e4. 2000\ub144\ub300 \ub300\ud45c\uc801\uc778 \ud31d \uc2a4\ud0c0 \uc911 \ud55c \uba85\uc73c\ub85c, \ube4c\ubcf4\ub4dc\uac00 \ubc1c\ud45c\ud55c 2000\ub144\ub300 \uc544\ud2f0\uc2a4\ud2b8 \uc21c\uc704\uc5d0\uc11c 8\uc704\uc5d0 \uc62c\ub790\ub2e4. \ubfd0\ub9cc \uc544\ub2c8\ub77c 2000\ub144\ub300 \uac00\uc7a5 \uc74c\ubc18\uc744 \ub9ce\uc774 \ud310 \uc5ec\uc790 \uac00\uc218\uc774\ub2e4. \ubbf8\uad6d \uc74c\ubc18 \uc0b0\uc5c5 \ud611\ud68c\uc5d0 \ub530\ub974\uba74 \ubbf8\uad6d\uc5d0\uc11c 3,400\ub9cc \uc7a5\uc758 \uc74c\ubc18 \ud310\ub9e4\ub7c9 \uc778\uc99d\uc744 \ubc1b\uc544 \uc804\uccb4 \uc5ec\uc790 \uac00\uc218 \uc911\uc5d0\uc11c \uc5ec\ub35f \ubc88\uc9f8\uc5d0 \ud574\ub2f9\ud55c\ub2e4. \ub610 \ub300\ud55c\ubbfc\uad6d\uc5d0\uc11c\ub3c4 130\ub9cc \uc7a5\uc758 \ud310\ub9e4\ub7c9\uc744 \uc62c\ub9ac\uace0\uc788\ub2e4. \uc138\uacc4\uc801\uc73c\ub85c\ub294 \ucd1d 1\uc5b5 \uc7a5\uc758 \uc74c\ubc18 \ud310\ub9e4\ub7c9\uc744 \uae30\ub85d\ud558\uba74\uc11c \uc5ed\uc0ac\uc0c1 \uac00\uc7a5 \ub9ce\uc740 \uc74c\ubc18\uc744 \ud310 \uac00\uc218 \uc911 \ud55c \uba85\uc774 \ub418\uc5c8\ub2e4. \u300a\ub864\ub9c1 \uc2a4\ud1a4\u300b\uc740 \uc5ed\uc0ac\uc0c1 \uc21c\uc2dd\uac04\uc5d0 \uc778\uae30\ub97c \uc5bb\uc740 \uc544\uc774\ub3cc 25\uc778 \uc911 \ud55c \uba85\uc73c\ub85c \uc120\uc815\ud558\uba70 \ube0c\ub9ac\ud2b8\ub2c8\uc758 \ube60\ub978 \uc131\uacf5\uc744 \uc778\uc815\ud588\ub2e4. VH1\uc740 2012\ub144 \uc120\uc815\ud55c \"\uac00\uc7a5 \uc704\ub300\ud55c \uc5ec\uc790 \uac00\uc218 100\uc778\" \uc21c\uc704 \uc911 11\uc704\uc5d0 \uc120\uc815\ud588\ub2e4. 2012\ub144\uc5d0\ub294 5,800\ub9cc \ub2ec\ub7ec\uc758 \uc218\uc775\uc744 \uae30\ub85d\ud574 2002\ub144 1\uc704\uc5d0 \uc624\ub978 \uc774\ud6c4 \ub2e4\uc2dc \u300a\ud3ec\ube0c\uc2a4\u300b\uc758 2012\ub144 \uac00\uc7a5 \ub3c8 \ub9ce\uc774 \ubc88 \uc5ec\uc131 \uac00\uc218\ub85c \uc120\uc815\ub418\uc5c8\ub2e4.", "answer": "8\uc704", "sentence": "2000\ub144\ub300 \ub300\ud45c\uc801\uc778 \ud31d \uc2a4\ud0c0 \uc911 \ud55c \uba85\uc73c\ub85c, \ube4c\ubcf4\ub4dc\uac00 \ubc1c\ud45c\ud55c 2000\ub144\ub300 \uc544\ud2f0\uc2a4\ud2b8 \uc21c\uc704\uc5d0\uc11c 8\uc704 \uc5d0 \uc62c\ub790\ub2e4.", "paragraph_sentence": "\ube0c\ub9ac\ud2b8\ub2c8\ub294 \ub370\ubdd4\ud558\uc790\ub9c8\uc790 \ud31d \uc544\uc774\ucf58\uc73c\ub85c \uc790\ub9ac\uc7a1\uc558\uc73c\uba70, 1990\ub144\ub300 \ub9d0 \uc720\ud589\ud55c \ud2f4 \ud31d\uc758 \uc720\ud589\uc744 \uc120\ub3c4\ud55c \uc778\ubb3c\ub85c \uc778\uc815\ubc1b\uace0\uc788\ub2e4. \ub610 20\uc0b4\uc774 \ub418\uae30 \uc804 10\ub300 \uac00\uc218\ub85c\ub294 \uac00\uc7a5 \ub192\uc740 \uc74c\ubc18 \ud310\ub9e4\ub7c9\uc744 \uae30\ub85d\ud588\uace0, \uc774\ub7ec\ud55c \uc131\uacfc\ub85c \"\ud31d\uc758 \uacf5\uc8fc\"\uc640 \uac19\uc740 \uce6d\ud638\ub97c \uc5bb\uc5c8\ub2e4. \ube0c\ub9ac\ud2b8\ub2c8\uc758 \uc791\ud488\uacfc \uc131\uacfc\ub294 \uc218 \ub9ce\uc740 \uc2dc\uc0c1\uc2dd\uacfc \uae30\ub85d\uc744 \ud1b5\ud574 \uc778\uc815\ubc1b\uc558\ub294\ub370, \ub300\ud45c\uc801\uc73c\ub85c 1\ubc88\uc758 \uadf8\ub798\ubbf8 \uc0c1 \uc218\uc0c1, \uacf5\ub85c\uc0c1\uc744 \ud3ec\ud568\ud55c 6\ubc88\uc758 MTV \ube44\ub514\uc624 \ubba4\uc9c1 \uc5b4\uc6cc\ub4dc \uc218\uc0c1, 9\ubc88\uc758 \ube4c\ubcf4\ub4dc \ubba4\uc9c1 \uc5b4\uc6cc\ub4dc \uc218\uc0c1, \uadf8\ub9ac\uace0 \ud560\ub9ac\uc6b0\ub4dc \uba85\uc608\uc758 \uac70\ub9ac\uc5d0 \uc774\ub984\uc744 \uc62c\ub838\ub2e4. 2000\ub144\ub300 \ub300\ud45c\uc801\uc778 \ud31d \uc2a4\ud0c0 \uc911 \ud55c \uba85\uc73c\ub85c, \ube4c\ubcf4\ub4dc\uac00 \ubc1c\ud45c\ud55c 2000\ub144\ub300 \uc544\ud2f0\uc2a4\ud2b8 \uc21c\uc704\uc5d0\uc11c 8\uc704 \uc5d0 \uc62c\ub790\ub2e4. \ubfd0\ub9cc \uc544\ub2c8\ub77c 2000\ub144\ub300 \uac00\uc7a5 \uc74c\ubc18\uc744 \ub9ce\uc774 \ud310 \uc5ec\uc790 \uac00\uc218\uc774\ub2e4. \ubbf8\uad6d \uc74c\ubc18 \uc0b0\uc5c5 \ud611\ud68c\uc5d0 \ub530\ub974\uba74 \ubbf8\uad6d\uc5d0\uc11c 3,400\ub9cc \uc7a5\uc758 \uc74c\ubc18 \ud310\ub9e4\ub7c9 \uc778\uc99d\uc744 \ubc1b\uc544 \uc804\uccb4 \uc5ec\uc790 \uac00\uc218 \uc911\uc5d0\uc11c \uc5ec\ub35f \ubc88\uc9f8\uc5d0 \ud574\ub2f9\ud55c\ub2e4. \ub610 \ub300\ud55c\ubbfc\uad6d\uc5d0\uc11c\ub3c4 130\ub9cc \uc7a5\uc758 \ud310\ub9e4\ub7c9\uc744 \uc62c\ub9ac\uace0\uc788\ub2e4. \uc138\uacc4\uc801\uc73c\ub85c\ub294 \ucd1d 1\uc5b5 \uc7a5\uc758 \uc74c\ubc18 \ud310\ub9e4\ub7c9\uc744 \uae30\ub85d\ud558\uba74\uc11c \uc5ed\uc0ac\uc0c1 \uac00\uc7a5 \ub9ce\uc740 \uc74c\ubc18\uc744 \ud310 \uac00\uc218 \uc911 \ud55c \uba85\uc774 \ub418\uc5c8\ub2e4. \u300a\ub864\ub9c1 \uc2a4\ud1a4\u300b\uc740 \uc5ed\uc0ac\uc0c1 \uc21c\uc2dd\uac04\uc5d0 \uc778\uae30\ub97c \uc5bb\uc740 \uc544\uc774\ub3cc 25\uc778 \uc911 \ud55c \uba85\uc73c\ub85c \uc120\uc815\ud558\uba70 \ube0c\ub9ac\ud2b8\ub2c8\uc758 \ube60\ub978 \uc131\uacf5\uc744 \uc778\uc815\ud588\ub2e4. VH1\uc740 2012\ub144 \uc120\uc815\ud55c \"\uac00\uc7a5 \uc704\ub300\ud55c \uc5ec\uc790 \uac00\uc218 100\uc778\" \uc21c\uc704 \uc911 11\uc704\uc5d0 \uc120\uc815\ud588\ub2e4. 2012\ub144\uc5d0\ub294 5,800\ub9cc \ub2ec\ub7ec\uc758 \uc218\uc775\uc744 \uae30\ub85d\ud574 2002\ub144 1\uc704\uc5d0 \uc624\ub978 \uc774\ud6c4 \ub2e4\uc2dc \u300a\ud3ec\ube0c\uc2a4\u300b\uc758 2012\ub144 \uac00\uc7a5 \ub3c8 \ub9ce\uc774 \ubc88 \uc5ec\uc131 \uac00\uc218\ub85c \uc120\uc815\ub418\uc5c8\ub2e4.", "paragraph_answer": "\ube0c\ub9ac\ud2b8\ub2c8\ub294 \ub370\ubdd4\ud558\uc790\ub9c8\uc790 \ud31d \uc544\uc774\ucf58\uc73c\ub85c \uc790\ub9ac\uc7a1\uc558\uc73c\uba70, 1990\ub144\ub300 \ub9d0 \uc720\ud589\ud55c \ud2f4 \ud31d\uc758 \uc720\ud589\uc744 \uc120\ub3c4\ud55c \uc778\ubb3c\ub85c \uc778\uc815\ubc1b\uace0\uc788\ub2e4. \ub610 20\uc0b4\uc774 \ub418\uae30 \uc804 10\ub300 \uac00\uc218\ub85c\ub294 \uac00\uc7a5 \ub192\uc740 \uc74c\ubc18 \ud310\ub9e4\ub7c9\uc744 \uae30\ub85d\ud588\uace0, \uc774\ub7ec\ud55c \uc131\uacfc\ub85c \"\ud31d\uc758 \uacf5\uc8fc\"\uc640 \uac19\uc740 \uce6d\ud638\ub97c \uc5bb\uc5c8\ub2e4. \ube0c\ub9ac\ud2b8\ub2c8\uc758 \uc791\ud488\uacfc \uc131\uacfc\ub294 \uc218 \ub9ce\uc740 \uc2dc\uc0c1\uc2dd\uacfc \uae30\ub85d\uc744 \ud1b5\ud574 \uc778\uc815\ubc1b\uc558\ub294\ub370, \ub300\ud45c\uc801\uc73c\ub85c 1\ubc88\uc758 \uadf8\ub798\ubbf8 \uc0c1 \uc218\uc0c1, \uacf5\ub85c\uc0c1\uc744 \ud3ec\ud568\ud55c 6\ubc88\uc758 MTV \ube44\ub514\uc624 \ubba4\uc9c1 \uc5b4\uc6cc\ub4dc \uc218\uc0c1, 9\ubc88\uc758 \ube4c\ubcf4\ub4dc \ubba4\uc9c1 \uc5b4\uc6cc\ub4dc \uc218\uc0c1, \uadf8\ub9ac\uace0 \ud560\ub9ac\uc6b0\ub4dc \uba85\uc608\uc758 \uac70\ub9ac\uc5d0 \uc774\ub984\uc744 \uc62c\ub838\ub2e4. 2000\ub144\ub300 \ub300\ud45c\uc801\uc778 \ud31d \uc2a4\ud0c0 \uc911 \ud55c \uba85\uc73c\ub85c, \ube4c\ubcf4\ub4dc\uac00 \ubc1c\ud45c\ud55c 2000\ub144\ub300 \uc544\ud2f0\uc2a4\ud2b8 \uc21c\uc704\uc5d0\uc11c 8\uc704 \uc5d0 \uc62c\ub790\ub2e4. \ubfd0\ub9cc \uc544\ub2c8\ub77c 2000\ub144\ub300 \uac00\uc7a5 \uc74c\ubc18\uc744 \ub9ce\uc774 \ud310 \uc5ec\uc790 \uac00\uc218\uc774\ub2e4. \ubbf8\uad6d \uc74c\ubc18 \uc0b0\uc5c5 \ud611\ud68c\uc5d0 \ub530\ub974\uba74 \ubbf8\uad6d\uc5d0\uc11c 3,400\ub9cc \uc7a5\uc758 \uc74c\ubc18 \ud310\ub9e4\ub7c9 \uc778\uc99d\uc744 \ubc1b\uc544 \uc804\uccb4 \uc5ec\uc790 \uac00\uc218 \uc911\uc5d0\uc11c \uc5ec\ub35f \ubc88\uc9f8\uc5d0 \ud574\ub2f9\ud55c\ub2e4. \ub610 \ub300\ud55c\ubbfc\uad6d\uc5d0\uc11c\ub3c4 130\ub9cc \uc7a5\uc758 \ud310\ub9e4\ub7c9\uc744 \uc62c\ub9ac\uace0\uc788\ub2e4. \uc138\uacc4\uc801\uc73c\ub85c\ub294 \ucd1d 1\uc5b5 \uc7a5\uc758 \uc74c\ubc18 \ud310\ub9e4\ub7c9\uc744 \uae30\ub85d\ud558\uba74\uc11c \uc5ed\uc0ac\uc0c1 \uac00\uc7a5 \ub9ce\uc740 \uc74c\ubc18\uc744 \ud310 \uac00\uc218 \uc911 \ud55c \uba85\uc774 \ub418\uc5c8\ub2e4. \u300a\ub864\ub9c1 \uc2a4\ud1a4\u300b\uc740 \uc5ed\uc0ac\uc0c1 \uc21c\uc2dd\uac04\uc5d0 \uc778\uae30\ub97c \uc5bb\uc740 \uc544\uc774\ub3cc 25\uc778 \uc911 \ud55c \uba85\uc73c\ub85c \uc120\uc815\ud558\uba70 \ube0c\ub9ac\ud2b8\ub2c8\uc758 \ube60\ub978 \uc131\uacf5\uc744 \uc778\uc815\ud588\ub2e4. VH1\uc740 2012\ub144 \uc120\uc815\ud55c \"\uac00\uc7a5 \uc704\ub300\ud55c \uc5ec\uc790 \uac00\uc218 100\uc778\" \uc21c\uc704 \uc911 11\uc704\uc5d0 \uc120\uc815\ud588\ub2e4. 2012\ub144\uc5d0\ub294 5,800\ub9cc \ub2ec\ub7ec\uc758 \uc218\uc775\uc744 \uae30\ub85d\ud574 2002\ub144 1\uc704\uc5d0 \uc624\ub978 \uc774\ud6c4 \ub2e4\uc2dc \u300a\ud3ec\ube0c\uc2a4\u300b\uc758 2012\ub144 \uac00\uc7a5 \ub3c8 \ub9ce\uc774 \ubc88 \uc5ec\uc131 \uac00\uc218\ub85c \uc120\uc815\ub418\uc5c8\ub2e4.", "sentence_answer": "2000\ub144\ub300 \ub300\ud45c\uc801\uc778 \ud31d \uc2a4\ud0c0 \uc911 \ud55c \uba85\uc73c\ub85c, \ube4c\ubcf4\ub4dc\uac00 \ubc1c\ud45c\ud55c 2000\ub144\ub300 \uc544\ud2f0\uc2a4\ud2b8 \uc21c\uc704\uc5d0\uc11c 8\uc704 \uc5d0 \uc62c\ub790\ub2e4.", "paragraph_id": "6509455-2-1", "paragraph_question": "question: \ube0c\ub9ac\ud2b8\ub2c8 \uc2a4\ud53c\uc5b4\uc2a4\ub294 \ube4c\ubcf4\ub4dc\uac00 \ubc1c\ud45c\ud55c 2000\ub144\ub300 \uc544\ud2f0\uc2a4\ud2b8 \uc21c\uc704\uc5d0\uc11c \uba87 \uc704\uc5d0 \uc62c\ub790\ub098\uc694?, context: \ube0c\ub9ac\ud2b8\ub2c8\ub294 \ub370\ubdd4\ud558\uc790\ub9c8\uc790 \ud31d \uc544\uc774\ucf58\uc73c\ub85c \uc790\ub9ac\uc7a1\uc558\uc73c\uba70, 1990\ub144\ub300 \ub9d0 \uc720\ud589\ud55c \ud2f4 \ud31d\uc758 \uc720\ud589\uc744 \uc120\ub3c4\ud55c \uc778\ubb3c\ub85c \uc778\uc815\ubc1b\uace0\uc788\ub2e4. \ub610 20\uc0b4\uc774 \ub418\uae30 \uc804 10\ub300 \uac00\uc218\ub85c\ub294 \uac00\uc7a5 \ub192\uc740 \uc74c\ubc18 \ud310\ub9e4\ub7c9\uc744 \uae30\ub85d\ud588\uace0, \uc774\ub7ec\ud55c \uc131\uacfc\ub85c \"\ud31d\uc758 \uacf5\uc8fc\"\uc640 \uac19\uc740 \uce6d\ud638\ub97c \uc5bb\uc5c8\ub2e4. \ube0c\ub9ac\ud2b8\ub2c8\uc758 \uc791\ud488\uacfc \uc131\uacfc\ub294 \uc218 \ub9ce\uc740 \uc2dc\uc0c1\uc2dd\uacfc \uae30\ub85d\uc744 \ud1b5\ud574 \uc778\uc815\ubc1b\uc558\ub294\ub370, \ub300\ud45c\uc801\uc73c\ub85c 1\ubc88\uc758 \uadf8\ub798\ubbf8 \uc0c1 \uc218\uc0c1, \uacf5\ub85c\uc0c1\uc744 \ud3ec\ud568\ud55c 6\ubc88\uc758 MTV \ube44\ub514\uc624 \ubba4\uc9c1 \uc5b4\uc6cc\ub4dc \uc218\uc0c1, 9\ubc88\uc758 \ube4c\ubcf4\ub4dc \ubba4\uc9c1 \uc5b4\uc6cc\ub4dc \uc218\uc0c1, \uadf8\ub9ac\uace0 \ud560\ub9ac\uc6b0\ub4dc \uba85\uc608\uc758 \uac70\ub9ac\uc5d0 \uc774\ub984\uc744 \uc62c\ub838\ub2e4. 2000\ub144\ub300 \ub300\ud45c\uc801\uc778 \ud31d \uc2a4\ud0c0 \uc911 \ud55c \uba85\uc73c\ub85c, \ube4c\ubcf4\ub4dc\uac00 \ubc1c\ud45c\ud55c 2000\ub144\ub300 \uc544\ud2f0\uc2a4\ud2b8 \uc21c\uc704\uc5d0\uc11c 8\uc704\uc5d0 \uc62c\ub790\ub2e4. \ubfd0\ub9cc \uc544\ub2c8\ub77c 2000\ub144\ub300 \uac00\uc7a5 \uc74c\ubc18\uc744 \ub9ce\uc774 \ud310 \uc5ec\uc790 \uac00\uc218\uc774\ub2e4. \ubbf8\uad6d \uc74c\ubc18 \uc0b0\uc5c5 \ud611\ud68c\uc5d0 \ub530\ub974\uba74 \ubbf8\uad6d\uc5d0\uc11c 3,400\ub9cc \uc7a5\uc758 \uc74c\ubc18 \ud310\ub9e4\ub7c9 \uc778\uc99d\uc744 \ubc1b\uc544 \uc804\uccb4 \uc5ec\uc790 \uac00\uc218 \uc911\uc5d0\uc11c \uc5ec\ub35f \ubc88\uc9f8\uc5d0 \ud574\ub2f9\ud55c\ub2e4. \ub610 \ub300\ud55c\ubbfc\uad6d\uc5d0\uc11c\ub3c4 130\ub9cc \uc7a5\uc758 \ud310\ub9e4\ub7c9\uc744 \uc62c\ub9ac\uace0\uc788\ub2e4. \uc138\uacc4\uc801\uc73c\ub85c\ub294 \ucd1d 1\uc5b5 \uc7a5\uc758 \uc74c\ubc18 \ud310\ub9e4\ub7c9\uc744 \uae30\ub85d\ud558\uba74\uc11c \uc5ed\uc0ac\uc0c1 \uac00\uc7a5 \ub9ce\uc740 \uc74c\ubc18\uc744 \ud310 \uac00\uc218 \uc911 \ud55c \uba85\uc774 \ub418\uc5c8\ub2e4. \u300a\ub864\ub9c1 \uc2a4\ud1a4\u300b\uc740 \uc5ed\uc0ac\uc0c1 \uc21c\uc2dd\uac04\uc5d0 \uc778\uae30\ub97c \uc5bb\uc740 \uc544\uc774\ub3cc 25\uc778 \uc911 \ud55c \uba85\uc73c\ub85c \uc120\uc815\ud558\uba70 \ube0c\ub9ac\ud2b8\ub2c8\uc758 \ube60\ub978 \uc131\uacf5\uc744 \uc778\uc815\ud588\ub2e4. VH1\uc740 2012\ub144 \uc120\uc815\ud55c \"\uac00\uc7a5 \uc704\ub300\ud55c \uc5ec\uc790 \uac00\uc218 100\uc778\" \uc21c\uc704 \uc911 11\uc704\uc5d0 \uc120\uc815\ud588\ub2e4. 2012\ub144\uc5d0\ub294 5,800\ub9cc \ub2ec\ub7ec\uc758 \uc218\uc775\uc744 \uae30\ub85d\ud574 2002\ub144 1\uc704\uc5d0 \uc624\ub978 \uc774\ud6c4 \ub2e4\uc2dc \u300a\ud3ec\ube0c\uc2a4\u300b\uc758 2012\ub144 \uac00\uc7a5 \ub3c8 \ub9ce\uc774 \ubc88 \uc5ec\uc131 \uac00\uc218\ub85c \uc120\uc815\ub418\uc5c8\ub2e4."} {"question": "\uaddc\uc288 \uc804\uae30 \uada4\ub3c4\uac00 \ud6c4\ucfe0\uc624\uce74 \uc2dc\uc5d0 \uac74\uc124\ubd80\ub97c \uc124\uce58\ud55c \uc2dc\uae30\ub294 \uc5b8\uc81c\uc778\uac00?", "paragraph": "\uaddc\uc288 \uc804\uae30 \uada4\ub3c4\uc5d0\uc11c\ub294 1911\ub144\ubd80\ud130 1914\ub144\uc758 \uc0ac\uc774\uc5d0 \ud6c4\uc758 \ub2c8\uc2dc\ud14c\uc4f0 \uae30\ud0c0\ud050\uc288 \uc120\uc5d0 \ud574\ub2f9\ud558\ub294 \ubaa8\uc9c0-\uc624\ub9ac\uc624\uac04\uc744 \uac1c\ud1b5\ud55c \ud6c4,1919\ub144 12\uc6d4 18\uc77c\uc5d0 \uc624\ub9ac\uc624-\ud6c4\ucfe0\uc624\uce74\uac04 47.27km\uc758 \uada4\ub3c4 \ubd80\uc124 \ud2b9\ud5c8\ub97c \ucde8\ub4dd\ud574 \u300c\uaddc\ud14c\uc4f0 \ud6c4\ucfe0\uc624\uce74 \uae09\ud589 \uc804\ucca0\u300d\uc774\ub77c\uace0 \uc774\ub984\uc774 \ubd99\uc5ec\uc84c\ub2e4. 1920\ub144\uc5d0\ub294 \ubaa8\uc9c0-\uad6c\ub9c8\ub370\uac04\uc758 \uada4\ub3c4 \ubd80\uc124 \ud2b9\ud5c8\ub97c \ucde8\ub4dd\ud558\uc600\ub2e4. 1928\ub144\uc5d0\ub294 \ud6c4\ucfe0\uc624\uce74 \uc2dc\uc5d0 \uac74\uc124\ubd80\ub97c \uc124\uce58\ud588\ub2e4. 1929\ub144\uc5d0\ub294 \uc624\ub9ac\uc624-\ud6c4\ucfe0\uc624\uce74\uac04\uc758 \uacc4\ud68d\uc744 \uad6c\ub9c8\ub370-\ud6c4\ucfe0\uc624\uce74\uac04, \uc2dc\ub85c\uc57c\ub9c8 \uace0\uac1c \uacbd\uc720\ub97c \uce74\ub77c\uc4f0 \uac00\ub3c4, \ud6c4\ub8e8\ubaac \uc655\ub798(\ud1b5\uce6d \u300c\uc601\uc8fc\ud1b5\u300d) \uacbd\uc720\ub85c \ubcc0\uacbd\ud574(\uc774\uac83\uc5d0 \uc758\ud574 \uacc4\ud68d \uac70\ub9ac\uac00 0.9 km \uc5f0\uc7a5),1931\ub144\uc5d0 \uacf5\uad6c\ub97c 3\uac1c\ub85c \ub098\ub204\uc5b4 \ubbf8\uc57c\uc9c0-\uc694\uc2dc\uc988\uce74\uac04\uc73c\ub85c\ubd80\ud130 \ucc29\uacf5\ud558\uc600\ub2e4. \uadf8\ub7ec\ub098, \uac19\uc740 1931\ub144\uc5d0\ub294 \uaddc\uc288 \uc804\uae30 \uada4\ub3c4 \uc0ac\uc7a5\uc5d0 \uc758\ud55c \ubd80\uc815 \uc5b4\uc74c \uc0ac\uac74\uc774 \ubc1c\uac01\ub418\uc5b4, \uc0ac\uac74\uc740 \ucc98\ub9ac\ub418\uc5c8\uc9c0\ub9cc, \uaddc\uc288 \uc804\uae30 \uada4\ub3c4\ub294 415\ub9cc\uc5d4\uc5d0 \uc774\ub974\ub294 \ucc44\ubb34\ub97c \ubd80\ub2f4\ud558\uac8c \ub418\uc5c8\ub2e4. \uc774 \ub54c\ubb38\uc5d0 \uc77c\ubcf8 \ud765\uc5c5 \uc740\ud589\uc73c\ub85c\ubd80\ud130 \ud2b9\ubcc4 \uc735\uc790\ub97c \uc811\uc218 \uc0c1\ud658\ud588\uc9c0\ub9cc,\uadf8 \ud2b9\ubcc4 \uc735\uc790\uc758 \uc870\uac74\uc73c\ub85c\uc11c \uacf5\uc0ac\uc758 \uc989\uc2dc \uc911\ub2e8\uc744 \ud53c\ud560 \uc218 \uc5c6\uac8c \ub418\uc5b4 1933\ub144 9\uc6d4 21\uc77c \ubaa8\uc9c0-\uad6c\ub9c8\ub370\uac04\uc758 \ud2b9\ud5c8\ub97c \ubc18\ub0a9\ud574, \uc624\ub9ac\uc624-\ud6c4\ucfe0\uc624\uce74\uac04\uc758 \uae30\uc5c5 \ud3d0\uc9c0\ub97c \uc2e4\uc2dc\ud588\ub2e4.", "answer": "1928\ub144", "sentence": "1928\ub144 \uc5d0\ub294 \ud6c4\ucfe0\uc624\uce74 \uc2dc\uc5d0 \uac74\uc124\ubd80\ub97c \uc124\uce58\ud588\ub2e4.", "paragraph_sentence": "\uaddc\uc288 \uc804\uae30 \uada4\ub3c4\uc5d0\uc11c\ub294 1911\ub144\ubd80\ud130 1914\ub144\uc758 \uc0ac\uc774\uc5d0 \ud6c4\uc758 \ub2c8\uc2dc\ud14c\uc4f0 \uae30\ud0c0\ud050\uc288 \uc120\uc5d0 \ud574\ub2f9\ud558\ub294 \ubaa8\uc9c0-\uc624\ub9ac\uc624\uac04\uc744 \uac1c\ud1b5\ud55c \ud6c4,1919\ub144 12\uc6d4 18\uc77c\uc5d0 \uc624\ub9ac\uc624-\ud6c4\ucfe0\uc624\uce74\uac04 47.27km\uc758 \uada4\ub3c4 \ubd80\uc124 \ud2b9\ud5c8\ub97c \ucde8\ub4dd\ud574 \u300c\uaddc\ud14c\uc4f0 \ud6c4\ucfe0\uc624\uce74 \uae09\ud589 \uc804\ucca0\u300d\uc774\ub77c\uace0 \uc774\ub984\uc774 \ubd99\uc5ec\uc84c\ub2e4. 1920\ub144\uc5d0\ub294 \ubaa8\uc9c0-\uad6c\ub9c8\ub370\uac04\uc758 \uada4\ub3c4 \ubd80\uc124 \ud2b9\ud5c8\ub97c \ucde8\ub4dd\ud558\uc600\ub2e4. 1928\ub144 \uc5d0\ub294 \ud6c4\ucfe0\uc624\uce74 \uc2dc\uc5d0 \uac74\uc124\ubd80\ub97c \uc124\uce58\ud588\ub2e4. 1929\ub144\uc5d0\ub294 \uc624\ub9ac\uc624-\ud6c4\ucfe0\uc624\uce74\uac04\uc758 \uacc4\ud68d\uc744 \uad6c\ub9c8\ub370-\ud6c4\ucfe0\uc624\uce74\uac04, \uc2dc\ub85c\uc57c\ub9c8 \uace0\uac1c \uacbd\uc720\ub97c \uce74\ub77c\uc4f0 \uac00\ub3c4, \ud6c4\ub8e8\ubaac \uc655\ub798(\ud1b5\uce6d \u300c\uc601\uc8fc\ud1b5\u300d) \uacbd\uc720\ub85c \ubcc0\uacbd\ud574(\uc774\uac83\uc5d0 \uc758\ud574 \uacc4\ud68d \uac70\ub9ac\uac00 0.9 km \uc5f0\uc7a5) , 1931\ub144\uc5d0 \uacf5\uad6c\ub97c 3\uac1c\ub85c \ub098\ub204\uc5b4 \ubbf8\uc57c\uc9c0-\uc694\uc2dc\uc988\uce74\uac04\uc73c\ub85c\ubd80\ud130 \ucc29\uacf5\ud558\uc600\ub2e4. \uadf8\ub7ec\ub098, \uac19\uc740 1931\ub144\uc5d0\ub294 \uaddc\uc288 \uc804\uae30 \uada4\ub3c4 \uc0ac\uc7a5\uc5d0 \uc758\ud55c \ubd80\uc815 \uc5b4\uc74c \uc0ac\uac74\uc774 \ubc1c\uac01\ub418\uc5b4, \uc0ac\uac74\uc740 \ucc98\ub9ac\ub418\uc5c8\uc9c0\ub9cc, \uaddc\uc288 \uc804\uae30 \uada4\ub3c4\ub294 415\ub9cc\uc5d4\uc5d0 \uc774\ub974\ub294 \ucc44\ubb34\ub97c \ubd80\ub2f4\ud558\uac8c \ub418\uc5c8\ub2e4. \uc774 \ub54c\ubb38\uc5d0 \uc77c\ubcf8 \ud765\uc5c5 \uc740\ud589\uc73c\ub85c\ubd80\ud130 \ud2b9\ubcc4 \uc735\uc790\ub97c \uc811\uc218 \uc0c1\ud658\ud588\uc9c0\ub9cc,\uadf8 \ud2b9\ubcc4 \uc735\uc790\uc758 \uc870\uac74\uc73c\ub85c\uc11c \uacf5\uc0ac\uc758 \uc989\uc2dc \uc911\ub2e8\uc744 \ud53c\ud560 \uc218 \uc5c6\uac8c \ub418\uc5b4 1933\ub144 9\uc6d4 21\uc77c \ubaa8\uc9c0-\uad6c\ub9c8\ub370\uac04\uc758 \ud2b9\ud5c8\ub97c \ubc18\ub0a9\ud574, \uc624\ub9ac\uc624-\ud6c4\ucfe0\uc624\uce74\uac04\uc758 \uae30\uc5c5 \ud3d0\uc9c0\ub97c \uc2e4\uc2dc\ud588\ub2e4.", "paragraph_answer": "\uaddc\uc288 \uc804\uae30 \uada4\ub3c4\uc5d0\uc11c\ub294 1911\ub144\ubd80\ud130 1914\ub144\uc758 \uc0ac\uc774\uc5d0 \ud6c4\uc758 \ub2c8\uc2dc\ud14c\uc4f0 \uae30\ud0c0\ud050\uc288 \uc120\uc5d0 \ud574\ub2f9\ud558\ub294 \ubaa8\uc9c0-\uc624\ub9ac\uc624\uac04\uc744 \uac1c\ud1b5\ud55c \ud6c4,1919\ub144 12\uc6d4 18\uc77c\uc5d0 \uc624\ub9ac\uc624-\ud6c4\ucfe0\uc624\uce74\uac04 47.27km\uc758 \uada4\ub3c4 \ubd80\uc124 \ud2b9\ud5c8\ub97c \ucde8\ub4dd\ud574 \u300c\uaddc\ud14c\uc4f0 \ud6c4\ucfe0\uc624\uce74 \uae09\ud589 \uc804\ucca0\u300d\uc774\ub77c\uace0 \uc774\ub984\uc774 \ubd99\uc5ec\uc84c\ub2e4. 1920\ub144\uc5d0\ub294 \ubaa8\uc9c0-\uad6c\ub9c8\ub370\uac04\uc758 \uada4\ub3c4 \ubd80\uc124 \ud2b9\ud5c8\ub97c \ucde8\ub4dd\ud558\uc600\ub2e4. 1928\ub144 \uc5d0\ub294 \ud6c4\ucfe0\uc624\uce74 \uc2dc\uc5d0 \uac74\uc124\ubd80\ub97c \uc124\uce58\ud588\ub2e4. 1929\ub144\uc5d0\ub294 \uc624\ub9ac\uc624-\ud6c4\ucfe0\uc624\uce74\uac04\uc758 \uacc4\ud68d\uc744 \uad6c\ub9c8\ub370-\ud6c4\ucfe0\uc624\uce74\uac04, \uc2dc\ub85c\uc57c\ub9c8 \uace0\uac1c \uacbd\uc720\ub97c \uce74\ub77c\uc4f0 \uac00\ub3c4, \ud6c4\ub8e8\ubaac \uc655\ub798(\ud1b5\uce6d \u300c\uc601\uc8fc\ud1b5\u300d) \uacbd\uc720\ub85c \ubcc0\uacbd\ud574(\uc774\uac83\uc5d0 \uc758\ud574 \uacc4\ud68d \uac70\ub9ac\uac00 0.9 km \uc5f0\uc7a5),1931\ub144\uc5d0 \uacf5\uad6c\ub97c 3\uac1c\ub85c \ub098\ub204\uc5b4 \ubbf8\uc57c\uc9c0-\uc694\uc2dc\uc988\uce74\uac04\uc73c\ub85c\ubd80\ud130 \ucc29\uacf5\ud558\uc600\ub2e4. \uadf8\ub7ec\ub098, \uac19\uc740 1931\ub144\uc5d0\ub294 \uaddc\uc288 \uc804\uae30 \uada4\ub3c4 \uc0ac\uc7a5\uc5d0 \uc758\ud55c \ubd80\uc815 \uc5b4\uc74c \uc0ac\uac74\uc774 \ubc1c\uac01\ub418\uc5b4, \uc0ac\uac74\uc740 \ucc98\ub9ac\ub418\uc5c8\uc9c0\ub9cc, \uaddc\uc288 \uc804\uae30 \uada4\ub3c4\ub294 415\ub9cc\uc5d4\uc5d0 \uc774\ub974\ub294 \ucc44\ubb34\ub97c \ubd80\ub2f4\ud558\uac8c \ub418\uc5c8\ub2e4. \uc774 \ub54c\ubb38\uc5d0 \uc77c\ubcf8 \ud765\uc5c5 \uc740\ud589\uc73c\ub85c\ubd80\ud130 \ud2b9\ubcc4 \uc735\uc790\ub97c \uc811\uc218 \uc0c1\ud658\ud588\uc9c0\ub9cc,\uadf8 \ud2b9\ubcc4 \uc735\uc790\uc758 \uc870\uac74\uc73c\ub85c\uc11c \uacf5\uc0ac\uc758 \uc989\uc2dc \uc911\ub2e8\uc744 \ud53c\ud560 \uc218 \uc5c6\uac8c \ub418\uc5b4 1933\ub144 9\uc6d4 21\uc77c \ubaa8\uc9c0-\uad6c\ub9c8\ub370\uac04\uc758 \ud2b9\ud5c8\ub97c \ubc18\ub0a9\ud574, \uc624\ub9ac\uc624-\ud6c4\ucfe0\uc624\uce74\uac04\uc758 \uae30\uc5c5 \ud3d0\uc9c0\ub97c \uc2e4\uc2dc\ud588\ub2e4.", "sentence_answer": " 1928\ub144 \uc5d0\ub294 \ud6c4\ucfe0\uc624\uce74 \uc2dc\uc5d0 \uac74\uc124\ubd80\ub97c \uc124\uce58\ud588\ub2e4.", "paragraph_id": "6557096-3-0", "paragraph_question": "question: \uaddc\uc288 \uc804\uae30 \uada4\ub3c4\uac00 \ud6c4\ucfe0\uc624\uce74 \uc2dc\uc5d0 \uac74\uc124\ubd80\ub97c \uc124\uce58\ud55c \uc2dc\uae30\ub294 \uc5b8\uc81c\uc778\uac00?, context: \uaddc\uc288 \uc804\uae30 \uada4\ub3c4\uc5d0\uc11c\ub294 1911\ub144\ubd80\ud130 1914\ub144\uc758 \uc0ac\uc774\uc5d0 \ud6c4\uc758 \ub2c8\uc2dc\ud14c\uc4f0 \uae30\ud0c0\ud050\uc288 \uc120\uc5d0 \ud574\ub2f9\ud558\ub294 \ubaa8\uc9c0-\uc624\ub9ac\uc624\uac04\uc744 \uac1c\ud1b5\ud55c \ud6c4,1919\ub144 12\uc6d4 18\uc77c\uc5d0 \uc624\ub9ac\uc624-\ud6c4\ucfe0\uc624\uce74\uac04 47.27km\uc758 \uada4\ub3c4 \ubd80\uc124 \ud2b9\ud5c8\ub97c \ucde8\ub4dd\ud574 \u300c\uaddc\ud14c\uc4f0 \ud6c4\ucfe0\uc624\uce74 \uae09\ud589 \uc804\ucca0\u300d\uc774\ub77c\uace0 \uc774\ub984\uc774 \ubd99\uc5ec\uc84c\ub2e4. 1920\ub144\uc5d0\ub294 \ubaa8\uc9c0-\uad6c\ub9c8\ub370\uac04\uc758 \uada4\ub3c4 \ubd80\uc124 \ud2b9\ud5c8\ub97c \ucde8\ub4dd\ud558\uc600\ub2e4. 1928\ub144\uc5d0\ub294 \ud6c4\ucfe0\uc624\uce74 \uc2dc\uc5d0 \uac74\uc124\ubd80\ub97c \uc124\uce58\ud588\ub2e4. 1929\ub144\uc5d0\ub294 \uc624\ub9ac\uc624-\ud6c4\ucfe0\uc624\uce74\uac04\uc758 \uacc4\ud68d\uc744 \uad6c\ub9c8\ub370-\ud6c4\ucfe0\uc624\uce74\uac04, \uc2dc\ub85c\uc57c\ub9c8 \uace0\uac1c \uacbd\uc720\ub97c \uce74\ub77c\uc4f0 \uac00\ub3c4, \ud6c4\ub8e8\ubaac \uc655\ub798(\ud1b5\uce6d \u300c\uc601\uc8fc\ud1b5\u300d) \uacbd\uc720\ub85c \ubcc0\uacbd\ud574(\uc774\uac83\uc5d0 \uc758\ud574 \uacc4\ud68d \uac70\ub9ac\uac00 0.9 km \uc5f0\uc7a5),1931\ub144\uc5d0 \uacf5\uad6c\ub97c 3\uac1c\ub85c \ub098\ub204\uc5b4 \ubbf8\uc57c\uc9c0-\uc694\uc2dc\uc988\uce74\uac04\uc73c\ub85c\ubd80\ud130 \ucc29\uacf5\ud558\uc600\ub2e4. \uadf8\ub7ec\ub098, \uac19\uc740 1931\ub144\uc5d0\ub294 \uaddc\uc288 \uc804\uae30 \uada4\ub3c4 \uc0ac\uc7a5\uc5d0 \uc758\ud55c \ubd80\uc815 \uc5b4\uc74c \uc0ac\uac74\uc774 \ubc1c\uac01\ub418\uc5b4, \uc0ac\uac74\uc740 \ucc98\ub9ac\ub418\uc5c8\uc9c0\ub9cc, \uaddc\uc288 \uc804\uae30 \uada4\ub3c4\ub294 415\ub9cc\uc5d4\uc5d0 \uc774\ub974\ub294 \ucc44\ubb34\ub97c \ubd80\ub2f4\ud558\uac8c \ub418\uc5c8\ub2e4. \uc774 \ub54c\ubb38\uc5d0 \uc77c\ubcf8 \ud765\uc5c5 \uc740\ud589\uc73c\ub85c\ubd80\ud130 \ud2b9\ubcc4 \uc735\uc790\ub97c \uc811\uc218 \uc0c1\ud658\ud588\uc9c0\ub9cc,\uadf8 \ud2b9\ubcc4 \uc735\uc790\uc758 \uc870\uac74\uc73c\ub85c\uc11c \uacf5\uc0ac\uc758 \uc989\uc2dc \uc911\ub2e8\uc744 \ud53c\ud560 \uc218 \uc5c6\uac8c \ub418\uc5b4 1933\ub144 9\uc6d4 21\uc77c \ubaa8\uc9c0-\uad6c\ub9c8\ub370\uac04\uc758 \ud2b9\ud5c8\ub97c \ubc18\ub0a9\ud574, \uc624\ub9ac\uc624-\ud6c4\ucfe0\uc624\uce74\uac04\uc758 \uae30\uc5c5 \ud3d0\uc9c0\ub97c \uc2e4\uc2dc\ud588\ub2e4."} {"question": "644\ub144 11\uc6d4 \uc815\uba85\uc9c4\uc758 \ub300\ub2f5\uc5d0 \ub9cc\uc871\ud55c \ud0dc\uc885\uc774 \uadf8\uc5d0\uac8c \ub0b4\ub9b0 \uc9c1\ucc45\uc740 \ubb34\uc5c7\uc778\uac00?", "paragraph": "\ub450\uac74\ub355 \ud718\ud558\uc5d0 \uc788\uc744 \ub54c\ubd80\ud130 \uc720\ub2a5\ud558\ub2e4\ub294 \ud3c9\uac00\ub97c \ubc1b\uc558\ub2e4. 644\ub144 11\uc6d4 \ud0dc\uc885\uc774 \uba85\uc8fc\uc790\uc0ac \uc815\uba85\uc9c4\uc774 \uc6a9\ubcd1\uc744 \uc798\ud55c\ub2e4\ub294 \ub9d0\uc744 \ub4e3\uace0 \uadf8\uc5d0\uac8c \uace0\uad6c\ub824 \uc815\ubc8c\uc5d0 \ub300\ud55c \ubc29\ub7b5\uc744 \ubb3c\uc5c8\ub2e4. \uc815\uba85\uc9c4\uc758 \ub2f5\ubcc0\uc774 \ub9e4\uc6b0 \uc7ac\ubbfc\ud588\uc73c\ubbc0\ub85c \uc7a5\uc218\ub098 \uc7ac\uc0c1\uc774 \ub420 \ub9cc\ud55c \uadf8\ub987\uc774\ub77c\uace0 \uaca9\ub824\ud558\uba70 \uace7 \uc77c\uc744 \ub9e1\uae30\uaca0\ub2e4\uace0 \ud558\uc600\ub2e4. \uadf8\ub7f0\ub370 \uc815\uba85\uc9c4\uc774 \uac10\uc0ac\uc758 \uc608\ub97c \ud45c\ud558\ub294 \uac83\uc744 \uae5c\ubc15\ud558\uc600\ub2e4. \ud0dc\uc885\uc774 \uc774\ub97c \uc5c4\ud558\uac8c \uafb8\uc9d6\uc790 \u201c\uc2e0\uc740 \uad11\uc57c\uc5d0\ub9cc \uc788\ub290\ub77c \ud669\uc81c\uc758 \uc9c8\ubb38\uc744 \ud55c \ubc88\ub3c4 \ubc1b\uc544\ubcf8 \uc801\uc774 \uc5c6\uc5c8\uc2b5\ub2c8\ub2e4. \uc624\uc9c1 \uadf8 \ub300\ub2f5\uc5d0\ub9cc \uc628 \uc2e0\uacbd\uc744 \uc9d1\uc911\ud558\ub2e4\uac00 \uadf8\ub9cc \uc608\ub97c \ud45c\ud558\ub294 \uac83\uc744 \uc78a\uace0 \ub9d0\uc558\uc2b5\ub2c8\ub2e4.\u201d\ub77c \ud0dc\uc5f0\ud558\uba74\uc11c\ub3c4 \uc7ac\uce58\uc788\uac8c \ub2f5\ud558\uc600\ub2e4. \ud0dc\uc885\uc774 \u201c\uc9d0\uc774 \ud0c0\uc778\uc744 \uacac\ucc45\ud560 \ub54c\ub9c8\ub2e4 20\uc5ec \ub144\uc744 \uacc1\uc5d0 \uc788\uc5b4\uc628 \ubc29\ud604\ub839\uc740 \ub298 \uc548\uc0c9\uc744 \uc8fc\uccb4\ud558\uc9c0 \ubabb\ud588\ub294\ub370, \uc815\uba85\uc9c4\uc740 \ud3c9\uc0dd \uc9d0\uc744 \ubcf4\uc9c0 \ubabb\ud588\ub294\ub370\ub3c4 \ubb34\uc11c\uc6cc\uc11c \ub5a0\ub294 \uae30\uc0c9\ub3c4 \uc5c6\uc774 \ub9d0\uc758 \uc870\ub9ac\ub97c \uc783\uc9c0 \uc54a\uc558\ub2e4. \ucc38\uc73c\ub85c \ub6f0\uc5b4\ub09c \uc790\ub85c\uad6c\ub098!\u201d\ub77c\uba70 \uac10\ud0c4\ud558\uc600\ub2e4. \uadf8\ub0a0\ub85c \uc6b0\ud6a8\uc704\uc7a5\uad70\uc5d0 \uc549\ud788\uace0 \ud3c9\uc591\ub3c4\ud589\uad70\ucd1d\uad00\uc744 \ub9e1\uacbc\ub2e4. \uc5ec\ub2f9\uc804\uc7c1 \ub54c\ub294 \ub204\ucc28 \uc801\uc740 \ubcd1\ub825\uc73c\ub85c\ub3c4 \ub9ce\uc740 \uc801\uc744 \uc0c1\ub300\ud588\uae30\uc5d0 \uba85\uc7a5\uc774\ub77c\ub294 \uc18c\ub9ac\uae4c\uc9c0 \ub4e4\uc5c8\ub2e4.", "answer": "\uc6b0\ud6a8\uc704\uc7a5\uad70\uc5d0 \uc549\ud788\uace0 \ud3c9\uc591\ub3c4\ud589\uad70\ucd1d\uad00\uc744 \ub9e1\uacbc\ub2e4.", "sentence": "\uadf8\ub0a0\ub85c \uc6b0\ud6a8\uc704\uc7a5\uad70\uc5d0 \uc549\ud788\uace0 \ud3c9\uc591\ub3c4\ud589\uad70\ucd1d\uad00\uc744 \ub9e1\uacbc\ub2e4. \uc5ec\ub2f9\uc804\uc7c1 \ub54c\ub294 \ub204\ucc28 \uc801\uc740 \ubcd1\ub825\uc73c\ub85c\ub3c4 \ub9ce\uc740 \uc801\uc744 \uc0c1\ub300\ud588\uae30\uc5d0 \uba85\uc7a5\uc774\ub77c\ub294 \uc18c\ub9ac\uae4c\uc9c0 \ub4e4\uc5c8\ub2e4.", "paragraph_sentence": "\ub450\uac74\ub355 \ud718\ud558\uc5d0 \uc788\uc744 \ub54c\ubd80\ud130 \uc720\ub2a5\ud558\ub2e4\ub294 \ud3c9\uac00\ub97c \ubc1b\uc558\ub2e4. 644\ub144 11\uc6d4 \ud0dc\uc885\uc774 \uba85\uc8fc\uc790\uc0ac \uc815\uba85\uc9c4\uc774 \uc6a9\ubcd1\uc744 \uc798\ud55c\ub2e4\ub294 \ub9d0\uc744 \ub4e3\uace0 \uadf8\uc5d0\uac8c \uace0\uad6c\ub824 \uc815\ubc8c\uc5d0 \ub300\ud55c \ubc29\ub7b5\uc744 \ubb3c\uc5c8\ub2e4. \uc815\uba85\uc9c4\uc758 \ub2f5\ubcc0\uc774 \ub9e4\uc6b0 \uc7ac\ubbfc\ud588\uc73c\ubbc0\ub85c \uc7a5\uc218\ub098 \uc7ac\uc0c1\uc774 \ub420 \ub9cc\ud55c \uadf8\ub987\uc774\ub77c\uace0 \uaca9\ub824\ud558\uba70 \uace7 \uc77c\uc744 \ub9e1\uae30\uaca0\ub2e4\uace0 \ud558\uc600\ub2e4. \uadf8\ub7f0\ub370 \uc815\uba85\uc9c4\uc774 \uac10\uc0ac\uc758 \uc608\ub97c \ud45c\ud558\ub294 \uac83\uc744 \uae5c\ubc15\ud558\uc600\ub2e4. \ud0dc\uc885\uc774 \uc774\ub97c \uc5c4\ud558\uac8c \uafb8\uc9d6\uc790 \u201c\uc2e0\uc740 \uad11\uc57c\uc5d0\ub9cc \uc788\ub290\ub77c \ud669\uc81c\uc758 \uc9c8\ubb38\uc744 \ud55c \ubc88\ub3c4 \ubc1b\uc544\ubcf8 \uc801\uc774 \uc5c6\uc5c8\uc2b5\ub2c8\ub2e4. \uc624\uc9c1 \uadf8 \ub300\ub2f5\uc5d0\ub9cc \uc628 \uc2e0\uacbd\uc744 \uc9d1\uc911\ud558\ub2e4\uac00 \uadf8\ub9cc \uc608\ub97c \ud45c\ud558\ub294 \uac83\uc744 \uc78a\uace0 \ub9d0\uc558\uc2b5\ub2c8\ub2e4. \u201d\ub77c \ud0dc\uc5f0\ud558\uba74\uc11c\ub3c4 \uc7ac\uce58\uc788\uac8c \ub2f5\ud558\uc600\ub2e4. \ud0dc\uc885\uc774 \u201c\uc9d0\uc774 \ud0c0\uc778\uc744 \uacac\ucc45\ud560 \ub54c\ub9c8\ub2e4 20\uc5ec \ub144\uc744 \uacc1\uc5d0 \uc788\uc5b4\uc628 \ubc29\ud604\ub839\uc740 \ub298 \uc548\uc0c9\uc744 \uc8fc\uccb4\ud558\uc9c0 \ubabb\ud588\ub294\ub370, \uc815\uba85\uc9c4\uc740 \ud3c9\uc0dd \uc9d0\uc744 \ubcf4\uc9c0 \ubabb\ud588\ub294\ub370\ub3c4 \ubb34\uc11c\uc6cc\uc11c \ub5a0\ub294 \uae30\uc0c9\ub3c4 \uc5c6\uc774 \ub9d0\uc758 \uc870\ub9ac\ub97c \uc783\uc9c0 \uc54a\uc558\ub2e4. \ucc38\uc73c\ub85c \ub6f0\uc5b4\ub09c \uc790\ub85c\uad6c\ub098!\u201d\ub77c\uba70 \uac10\ud0c4\ud558\uc600\ub2e4. \uadf8\ub0a0\ub85c \uc6b0\ud6a8\uc704\uc7a5\uad70\uc5d0 \uc549\ud788\uace0 \ud3c9\uc591\ub3c4\ud589\uad70\ucd1d\uad00\uc744 \ub9e1\uacbc\ub2e4. \uc5ec\ub2f9\uc804\uc7c1 \ub54c\ub294 \ub204\ucc28 \uc801\uc740 \ubcd1\ub825\uc73c\ub85c\ub3c4 \ub9ce\uc740 \uc801\uc744 \uc0c1\ub300\ud588\uae30\uc5d0 \uba85\uc7a5\uc774\ub77c\ub294 \uc18c\ub9ac\uae4c\uc9c0 \ub4e4\uc5c8\ub2e4. ", "paragraph_answer": "\ub450\uac74\ub355 \ud718\ud558\uc5d0 \uc788\uc744 \ub54c\ubd80\ud130 \uc720\ub2a5\ud558\ub2e4\ub294 \ud3c9\uac00\ub97c \ubc1b\uc558\ub2e4. 644\ub144 11\uc6d4 \ud0dc\uc885\uc774 \uba85\uc8fc\uc790\uc0ac \uc815\uba85\uc9c4\uc774 \uc6a9\ubcd1\uc744 \uc798\ud55c\ub2e4\ub294 \ub9d0\uc744 \ub4e3\uace0 \uadf8\uc5d0\uac8c \uace0\uad6c\ub824 \uc815\ubc8c\uc5d0 \ub300\ud55c \ubc29\ub7b5\uc744 \ubb3c\uc5c8\ub2e4. \uc815\uba85\uc9c4\uc758 \ub2f5\ubcc0\uc774 \ub9e4\uc6b0 \uc7ac\ubbfc\ud588\uc73c\ubbc0\ub85c \uc7a5\uc218\ub098 \uc7ac\uc0c1\uc774 \ub420 \ub9cc\ud55c \uadf8\ub987\uc774\ub77c\uace0 \uaca9\ub824\ud558\uba70 \uace7 \uc77c\uc744 \ub9e1\uae30\uaca0\ub2e4\uace0 \ud558\uc600\ub2e4. \uadf8\ub7f0\ub370 \uc815\uba85\uc9c4\uc774 \uac10\uc0ac\uc758 \uc608\ub97c \ud45c\ud558\ub294 \uac83\uc744 \uae5c\ubc15\ud558\uc600\ub2e4. \ud0dc\uc885\uc774 \uc774\ub97c \uc5c4\ud558\uac8c \uafb8\uc9d6\uc790 \u201c\uc2e0\uc740 \uad11\uc57c\uc5d0\ub9cc \uc788\ub290\ub77c \ud669\uc81c\uc758 \uc9c8\ubb38\uc744 \ud55c \ubc88\ub3c4 \ubc1b\uc544\ubcf8 \uc801\uc774 \uc5c6\uc5c8\uc2b5\ub2c8\ub2e4. \uc624\uc9c1 \uadf8 \ub300\ub2f5\uc5d0\ub9cc \uc628 \uc2e0\uacbd\uc744 \uc9d1\uc911\ud558\ub2e4\uac00 \uadf8\ub9cc \uc608\ub97c \ud45c\ud558\ub294 \uac83\uc744 \uc78a\uace0 \ub9d0\uc558\uc2b5\ub2c8\ub2e4.\u201d\ub77c \ud0dc\uc5f0\ud558\uba74\uc11c\ub3c4 \uc7ac\uce58\uc788\uac8c \ub2f5\ud558\uc600\ub2e4. \ud0dc\uc885\uc774 \u201c\uc9d0\uc774 \ud0c0\uc778\uc744 \uacac\ucc45\ud560 \ub54c\ub9c8\ub2e4 20\uc5ec \ub144\uc744 \uacc1\uc5d0 \uc788\uc5b4\uc628 \ubc29\ud604\ub839\uc740 \ub298 \uc548\uc0c9\uc744 \uc8fc\uccb4\ud558\uc9c0 \ubabb\ud588\ub294\ub370, \uc815\uba85\uc9c4\uc740 \ud3c9\uc0dd \uc9d0\uc744 \ubcf4\uc9c0 \ubabb\ud588\ub294\ub370\ub3c4 \ubb34\uc11c\uc6cc\uc11c \ub5a0\ub294 \uae30\uc0c9\ub3c4 \uc5c6\uc774 \ub9d0\uc758 \uc870\ub9ac\ub97c \uc783\uc9c0 \uc54a\uc558\ub2e4. \ucc38\uc73c\ub85c \ub6f0\uc5b4\ub09c \uc790\ub85c\uad6c\ub098!\u201d\ub77c\uba70 \uac10\ud0c4\ud558\uc600\ub2e4. \uadf8\ub0a0\ub85c \uc6b0\ud6a8\uc704\uc7a5\uad70\uc5d0 \uc549\ud788\uace0 \ud3c9\uc591\ub3c4\ud589\uad70\ucd1d\uad00\uc744 \ub9e1\uacbc\ub2e4. \uc5ec\ub2f9\uc804\uc7c1 \ub54c\ub294 \ub204\ucc28 \uc801\uc740 \ubcd1\ub825\uc73c\ub85c\ub3c4 \ub9ce\uc740 \uc801\uc744 \uc0c1\ub300\ud588\uae30\uc5d0 \uba85\uc7a5\uc774\ub77c\ub294 \uc18c\ub9ac\uae4c\uc9c0 \ub4e4\uc5c8\ub2e4.", "sentence_answer": "\uadf8\ub0a0\ub85c \uc6b0\ud6a8\uc704\uc7a5\uad70\uc5d0 \uc549\ud788\uace0 \ud3c9\uc591\ub3c4\ud589\uad70\ucd1d\uad00\uc744 \ub9e1\uacbc\ub2e4. \uc5ec\ub2f9\uc804\uc7c1 \ub54c\ub294 \ub204\ucc28 \uc801\uc740 \ubcd1\ub825\uc73c\ub85c\ub3c4 \ub9ce\uc740 \uc801\uc744 \uc0c1\ub300\ud588\uae30\uc5d0 \uba85\uc7a5\uc774\ub77c\ub294 \uc18c\ub9ac\uae4c\uc9c0 \ub4e4\uc5c8\ub2e4.", "paragraph_id": "5866697-2-0", "paragraph_question": "question: 644\ub144 11\uc6d4 \uc815\uba85\uc9c4\uc758 \ub300\ub2f5\uc5d0 \ub9cc\uc871\ud55c \ud0dc\uc885\uc774 \uadf8\uc5d0\uac8c \ub0b4\ub9b0 \uc9c1\ucc45\uc740 \ubb34\uc5c7\uc778\uac00?, context: \ub450\uac74\ub355 \ud718\ud558\uc5d0 \uc788\uc744 \ub54c\ubd80\ud130 \uc720\ub2a5\ud558\ub2e4\ub294 \ud3c9\uac00\ub97c \ubc1b\uc558\ub2e4. 644\ub144 11\uc6d4 \ud0dc\uc885\uc774 \uba85\uc8fc\uc790\uc0ac \uc815\uba85\uc9c4\uc774 \uc6a9\ubcd1\uc744 \uc798\ud55c\ub2e4\ub294 \ub9d0\uc744 \ub4e3\uace0 \uadf8\uc5d0\uac8c \uace0\uad6c\ub824 \uc815\ubc8c\uc5d0 \ub300\ud55c \ubc29\ub7b5\uc744 \ubb3c\uc5c8\ub2e4. \uc815\uba85\uc9c4\uc758 \ub2f5\ubcc0\uc774 \ub9e4\uc6b0 \uc7ac\ubbfc\ud588\uc73c\ubbc0\ub85c \uc7a5\uc218\ub098 \uc7ac\uc0c1\uc774 \ub420 \ub9cc\ud55c \uadf8\ub987\uc774\ub77c\uace0 \uaca9\ub824\ud558\uba70 \uace7 \uc77c\uc744 \ub9e1\uae30\uaca0\ub2e4\uace0 \ud558\uc600\ub2e4. \uadf8\ub7f0\ub370 \uc815\uba85\uc9c4\uc774 \uac10\uc0ac\uc758 \uc608\ub97c \ud45c\ud558\ub294 \uac83\uc744 \uae5c\ubc15\ud558\uc600\ub2e4. \ud0dc\uc885\uc774 \uc774\ub97c \uc5c4\ud558\uac8c \uafb8\uc9d6\uc790 \u201c\uc2e0\uc740 \uad11\uc57c\uc5d0\ub9cc \uc788\ub290\ub77c \ud669\uc81c\uc758 \uc9c8\ubb38\uc744 \ud55c \ubc88\ub3c4 \ubc1b\uc544\ubcf8 \uc801\uc774 \uc5c6\uc5c8\uc2b5\ub2c8\ub2e4. \uc624\uc9c1 \uadf8 \ub300\ub2f5\uc5d0\ub9cc \uc628 \uc2e0\uacbd\uc744 \uc9d1\uc911\ud558\ub2e4\uac00 \uadf8\ub9cc \uc608\ub97c \ud45c\ud558\ub294 \uac83\uc744 \uc78a\uace0 \ub9d0\uc558\uc2b5\ub2c8\ub2e4.\u201d\ub77c \ud0dc\uc5f0\ud558\uba74\uc11c\ub3c4 \uc7ac\uce58\uc788\uac8c \ub2f5\ud558\uc600\ub2e4. \ud0dc\uc885\uc774 \u201c\uc9d0\uc774 \ud0c0\uc778\uc744 \uacac\ucc45\ud560 \ub54c\ub9c8\ub2e4 20\uc5ec \ub144\uc744 \uacc1\uc5d0 \uc788\uc5b4\uc628 \ubc29\ud604\ub839\uc740 \ub298 \uc548\uc0c9\uc744 \uc8fc\uccb4\ud558\uc9c0 \ubabb\ud588\ub294\ub370, \uc815\uba85\uc9c4\uc740 \ud3c9\uc0dd \uc9d0\uc744 \ubcf4\uc9c0 \ubabb\ud588\ub294\ub370\ub3c4 \ubb34\uc11c\uc6cc\uc11c \ub5a0\ub294 \uae30\uc0c9\ub3c4 \uc5c6\uc774 \ub9d0\uc758 \uc870\ub9ac\ub97c \uc783\uc9c0 \uc54a\uc558\ub2e4. \ucc38\uc73c\ub85c \ub6f0\uc5b4\ub09c \uc790\ub85c\uad6c\ub098!\u201d\ub77c\uba70 \uac10\ud0c4\ud558\uc600\ub2e4. \uadf8\ub0a0\ub85c \uc6b0\ud6a8\uc704\uc7a5\uad70\uc5d0 \uc549\ud788\uace0 \ud3c9\uc591\ub3c4\ud589\uad70\ucd1d\uad00\uc744 \ub9e1\uacbc\ub2e4. \uc5ec\ub2f9\uc804\uc7c1 \ub54c\ub294 \ub204\ucc28 \uc801\uc740 \ubcd1\ub825\uc73c\ub85c\ub3c4 \ub9ce\uc740 \uc801\uc744 \uc0c1\ub300\ud588\uae30\uc5d0 \uba85\uc7a5\uc774\ub77c\ub294 \uc18c\ub9ac\uae4c\uc9c0 \ub4e4\uc5c8\ub2e4."} {"question": "\ud504\ub791\uc2a4\uc758 \ub8e8\uc774 \ud544\ub9ac\ud504 \uc655\uc740 \ubb34\uc2a8 \uc655\uc73c\ub85c\ub3c4 \ubd88\ub838\ub294\uac00?", "paragraph": "\uc774\ub7ec\ud55c \uad6d\ub0b4\uc815\uce58\uc801 \uae34\uc7a5\uc740 1847\ub144\ubd80\ud130 1848\ub144 \uc0ac\uc774\uc758 \uc804\ud658\uae30\uc5d0 \ubc1c\uc0dd\ud55c \uad6d\uc678\uc815\uce58\uc801 \ud604\uc0c1\uc73c\ub85c \ub354\uc6b1 \uc2ec\ud654\ub418\uc5c8\ub2e4. \uccab \uc815\uc810\uc740 \ud504\ub791\uc2a4\uc758 1848\ub144 2\uc6d4 \ud601\uba85\uc774\uc5c8\uc73c\uba70, \uc5ec\uae30\uc11c '\uc2dc\ubbfc\uc655' \ub8e8\uc774 \ud544\ub9ac\ud504\ub97c \ub04c\uc5b4\ub0b4\ub9ac\uace0 \uadf8 \uacb0\uacfc\ub85c \ud504\ub791\uc2a4 \uc81c2\uacf5\ud654\uad6d\uc774 \uc120\uc5b8\ub418\uc5c8\ub2e4. \ub9ce\uc740 \uc720\ub7fd \uad6d\uac00\ub4e4\uc5d0\uc11c \ubc18\ub3d9 \uc815\uce58\uc5d0 \ub300\ud55c \ud56d\uc7c1\uc774 \uc99d\uac00\ud588\uc73c\uba70 \uc774\ub294 \ud601\uba85\uc801 \ud63c\ub780\uc73c\ub85c \uc774\uc5b4\uc84c\ub2e4. \ub3c5\uc77c \uc5f0\ubc29\uc5d0\uc11c\ub3c4 1848\ub144 3\uc6d4 13\uc77c \ube48\uc758 \uc2dc\ubbfc \ubd09\uae30\ub97c \uc2dc\uc791\uc73c\ub85c \uc2dc\uc704\uac00 \ud655\uc0b0\ub418\uc5c8\ub2e4. \uc624\uc2a4\ud2b8\ub9ac\uc544 \uc81c\uad6d\uc758 \uc77c\ubd80\uc600\ub358 \ud5dd\uac00\ub9ac, \ubcf4\ud5e4\ubbf8\uc544, \ubd81\uc774\ud0c8\ub9ac\uc544 \ub4f1\uc9c0\uc5d0\uc11c\ub3c4 \uc720\ud608 \ubd09\uae30\uac00 \uc77c\uc5b4\ub0ac\uc73c\uba70, \uc774\ub4e4\uc740 \uc790\uce58\uad8c\uc744 \uc694\uad6c\ud558\uac70\ub098 \ub354 \ub098\uc544\uac00 \uace0\uc720\uc758 \ubbfc\uc871\uad6d\uac00 \ub3c5\ub9bd\uc5d0 \ub300\ud55c \uc694\uad6c\uae4c\uc9c0 \ud558\uc600\ub2e4. \ubc18\ub3d9 \uc815\uce58\uc758 \uc0c1\uc9d5\uc774\uc5c8\ub358 \uc624\uc2a4\ud2b8\ub9ac\uc544\uc758 \uc218\uc0c1 \uba54\ud14c\ub974\ub2c8\ud788\ub294 \ud601\uba85\uc774 \uc2ec\ud574\uc9c0\uc790 \uc2e4\uac01\ud558\uac8c \ub41c\ub2e4.", "answer": "\uc2dc\ubbfc\uc655", "sentence": "\uc5ec\uae30\uc11c ' \uc2dc\ubbfc\uc655 ' \ub8e8\uc774 \ud544\ub9ac\ud504\ub97c \ub04c\uc5b4\ub0b4\ub9ac\uace0 \uadf8 \uacb0\uacfc\ub85c \ud504\ub791\uc2a4 \uc81c2\uacf5\ud654\uad6d\uc774 \uc120\uc5b8\ub418\uc5c8\ub2e4.", "paragraph_sentence": "\uc774\ub7ec\ud55c \uad6d\ub0b4\uc815\uce58\uc801 \uae34\uc7a5\uc740 1847\ub144\ubd80\ud130 1848\ub144 \uc0ac\uc774\uc758 \uc804\ud658\uae30\uc5d0 \ubc1c\uc0dd\ud55c \uad6d\uc678\uc815\uce58\uc801 \ud604\uc0c1\uc73c\ub85c \ub354\uc6b1 \uc2ec\ud654\ub418\uc5c8\ub2e4. \uccab \uc815\uc810\uc740 \ud504\ub791\uc2a4\uc758 1848\ub144 2\uc6d4 \ud601\uba85\uc774\uc5c8\uc73c\uba70, \uc5ec\uae30\uc11c ' \uc2dc\ubbfc\uc655 ' \ub8e8\uc774 \ud544\ub9ac\ud504\ub97c \ub04c\uc5b4\ub0b4\ub9ac\uace0 \uadf8 \uacb0\uacfc\ub85c \ud504\ub791\uc2a4 \uc81c2\uacf5\ud654\uad6d\uc774 \uc120\uc5b8\ub418\uc5c8\ub2e4. \ub9ce\uc740 \uc720\ub7fd \uad6d\uac00\ub4e4\uc5d0\uc11c \ubc18\ub3d9 \uc815\uce58\uc5d0 \ub300\ud55c \ud56d\uc7c1\uc774 \uc99d\uac00\ud588\uc73c\uba70 \uc774\ub294 \ud601\uba85\uc801 \ud63c\ub780\uc73c\ub85c \uc774\uc5b4\uc84c\ub2e4. \ub3c5\uc77c \uc5f0\ubc29\uc5d0\uc11c\ub3c4 1848\ub144 3\uc6d4 13\uc77c \ube48\uc758 \uc2dc\ubbfc \ubd09\uae30\ub97c \uc2dc\uc791\uc73c\ub85c \uc2dc\uc704\uac00 \ud655\uc0b0\ub418\uc5c8\ub2e4. \uc624\uc2a4\ud2b8\ub9ac\uc544 \uc81c\uad6d\uc758 \uc77c\ubd80\uc600\ub358 \ud5dd\uac00\ub9ac, \ubcf4\ud5e4\ubbf8\uc544, \ubd81\uc774\ud0c8\ub9ac\uc544 \ub4f1\uc9c0\uc5d0\uc11c\ub3c4 \uc720\ud608 \ubd09\uae30\uac00 \uc77c\uc5b4\ub0ac\uc73c\uba70, \uc774\ub4e4\uc740 \uc790\uce58\uad8c\uc744 \uc694\uad6c\ud558\uac70\ub098 \ub354 \ub098\uc544\uac00 \uace0\uc720\uc758 \ubbfc\uc871\uad6d\uac00 \ub3c5\ub9bd\uc5d0 \ub300\ud55c \uc694\uad6c\uae4c\uc9c0 \ud558\uc600\ub2e4. \ubc18\ub3d9 \uc815\uce58\uc758 \uc0c1\uc9d5\uc774\uc5c8\ub358 \uc624\uc2a4\ud2b8\ub9ac\uc544\uc758 \uc218\uc0c1 \uba54\ud14c\ub974\ub2c8\ud788\ub294 \ud601\uba85\uc774 \uc2ec\ud574\uc9c0\uc790 \uc2e4\uac01\ud558\uac8c \ub41c\ub2e4.", "paragraph_answer": "\uc774\ub7ec\ud55c \uad6d\ub0b4\uc815\uce58\uc801 \uae34\uc7a5\uc740 1847\ub144\ubd80\ud130 1848\ub144 \uc0ac\uc774\uc758 \uc804\ud658\uae30\uc5d0 \ubc1c\uc0dd\ud55c \uad6d\uc678\uc815\uce58\uc801 \ud604\uc0c1\uc73c\ub85c \ub354\uc6b1 \uc2ec\ud654\ub418\uc5c8\ub2e4. \uccab \uc815\uc810\uc740 \ud504\ub791\uc2a4\uc758 1848\ub144 2\uc6d4 \ud601\uba85\uc774\uc5c8\uc73c\uba70, \uc5ec\uae30\uc11c ' \uc2dc\ubbfc\uc655 ' \ub8e8\uc774 \ud544\ub9ac\ud504\ub97c \ub04c\uc5b4\ub0b4\ub9ac\uace0 \uadf8 \uacb0\uacfc\ub85c \ud504\ub791\uc2a4 \uc81c2\uacf5\ud654\uad6d\uc774 \uc120\uc5b8\ub418\uc5c8\ub2e4. \ub9ce\uc740 \uc720\ub7fd \uad6d\uac00\ub4e4\uc5d0\uc11c \ubc18\ub3d9 \uc815\uce58\uc5d0 \ub300\ud55c \ud56d\uc7c1\uc774 \uc99d\uac00\ud588\uc73c\uba70 \uc774\ub294 \ud601\uba85\uc801 \ud63c\ub780\uc73c\ub85c \uc774\uc5b4\uc84c\ub2e4. \ub3c5\uc77c \uc5f0\ubc29\uc5d0\uc11c\ub3c4 1848\ub144 3\uc6d4 13\uc77c \ube48\uc758 \uc2dc\ubbfc \ubd09\uae30\ub97c \uc2dc\uc791\uc73c\ub85c \uc2dc\uc704\uac00 \ud655\uc0b0\ub418\uc5c8\ub2e4. \uc624\uc2a4\ud2b8\ub9ac\uc544 \uc81c\uad6d\uc758 \uc77c\ubd80\uc600\ub358 \ud5dd\uac00\ub9ac, \ubcf4\ud5e4\ubbf8\uc544, \ubd81\uc774\ud0c8\ub9ac\uc544 \ub4f1\uc9c0\uc5d0\uc11c\ub3c4 \uc720\ud608 \ubd09\uae30\uac00 \uc77c\uc5b4\ub0ac\uc73c\uba70, \uc774\ub4e4\uc740 \uc790\uce58\uad8c\uc744 \uc694\uad6c\ud558\uac70\ub098 \ub354 \ub098\uc544\uac00 \uace0\uc720\uc758 \ubbfc\uc871\uad6d\uac00 \ub3c5\ub9bd\uc5d0 \ub300\ud55c \uc694\uad6c\uae4c\uc9c0 \ud558\uc600\ub2e4. \ubc18\ub3d9 \uc815\uce58\uc758 \uc0c1\uc9d5\uc774\uc5c8\ub358 \uc624\uc2a4\ud2b8\ub9ac\uc544\uc758 \uc218\uc0c1 \uba54\ud14c\ub974\ub2c8\ud788\ub294 \ud601\uba85\uc774 \uc2ec\ud574\uc9c0\uc790 \uc2e4\uac01\ud558\uac8c \ub41c\ub2e4.", "sentence_answer": "\uc5ec\uae30\uc11c ' \uc2dc\ubbfc\uc655 ' \ub8e8\uc774 \ud544\ub9ac\ud504\ub97c \ub04c\uc5b4\ub0b4\ub9ac\uace0 \uadf8 \uacb0\uacfc\ub85c \ud504\ub791\uc2a4 \uc81c2\uacf5\ud654\uad6d\uc774 \uc120\uc5b8\ub418\uc5c8\ub2e4.", "paragraph_id": "6568666-5-1", "paragraph_question": "question: \ud504\ub791\uc2a4\uc758 \ub8e8\uc774 \ud544\ub9ac\ud504 \uc655\uc740 \ubb34\uc2a8 \uc655\uc73c\ub85c\ub3c4 \ubd88\ub838\ub294\uac00?, context: \uc774\ub7ec\ud55c \uad6d\ub0b4\uc815\uce58\uc801 \uae34\uc7a5\uc740 1847\ub144\ubd80\ud130 1848\ub144 \uc0ac\uc774\uc758 \uc804\ud658\uae30\uc5d0 \ubc1c\uc0dd\ud55c \uad6d\uc678\uc815\uce58\uc801 \ud604\uc0c1\uc73c\ub85c \ub354\uc6b1 \uc2ec\ud654\ub418\uc5c8\ub2e4. \uccab \uc815\uc810\uc740 \ud504\ub791\uc2a4\uc758 1848\ub144 2\uc6d4 \ud601\uba85\uc774\uc5c8\uc73c\uba70, \uc5ec\uae30\uc11c '\uc2dc\ubbfc\uc655' \ub8e8\uc774 \ud544\ub9ac\ud504\ub97c \ub04c\uc5b4\ub0b4\ub9ac\uace0 \uadf8 \uacb0\uacfc\ub85c \ud504\ub791\uc2a4 \uc81c2\uacf5\ud654\uad6d\uc774 \uc120\uc5b8\ub418\uc5c8\ub2e4. \ub9ce\uc740 \uc720\ub7fd \uad6d\uac00\ub4e4\uc5d0\uc11c \ubc18\ub3d9 \uc815\uce58\uc5d0 \ub300\ud55c \ud56d\uc7c1\uc774 \uc99d\uac00\ud588\uc73c\uba70 \uc774\ub294 \ud601\uba85\uc801 \ud63c\ub780\uc73c\ub85c \uc774\uc5b4\uc84c\ub2e4. \ub3c5\uc77c \uc5f0\ubc29\uc5d0\uc11c\ub3c4 1848\ub144 3\uc6d4 13\uc77c \ube48\uc758 \uc2dc\ubbfc \ubd09\uae30\ub97c \uc2dc\uc791\uc73c\ub85c \uc2dc\uc704\uac00 \ud655\uc0b0\ub418\uc5c8\ub2e4. \uc624\uc2a4\ud2b8\ub9ac\uc544 \uc81c\uad6d\uc758 \uc77c\ubd80\uc600\ub358 \ud5dd\uac00\ub9ac, \ubcf4\ud5e4\ubbf8\uc544, \ubd81\uc774\ud0c8\ub9ac\uc544 \ub4f1\uc9c0\uc5d0\uc11c\ub3c4 \uc720\ud608 \ubd09\uae30\uac00 \uc77c\uc5b4\ub0ac\uc73c\uba70, \uc774\ub4e4\uc740 \uc790\uce58\uad8c\uc744 \uc694\uad6c\ud558\uac70\ub098 \ub354 \ub098\uc544\uac00 \uace0\uc720\uc758 \ubbfc\uc871\uad6d\uac00 \ub3c5\ub9bd\uc5d0 \ub300\ud55c \uc694\uad6c\uae4c\uc9c0 \ud558\uc600\ub2e4. \ubc18\ub3d9 \uc815\uce58\uc758 \uc0c1\uc9d5\uc774\uc5c8\ub358 \uc624\uc2a4\ud2b8\ub9ac\uc544\uc758 \uc218\uc0c1 \uba54\ud14c\ub974\ub2c8\ud788\ub294 \ud601\uba85\uc774 \uc2ec\ud574\uc9c0\uc790 \uc2e4\uac01\ud558\uac8c \ub41c\ub2e4."} {"question": "2014\ub144 \ud55c\uad6d \ud53c\uaca8 \uc885\ud569 \uc120\uc218\uad8c \ub300\ud68c\uc5d0\uc11c \uae40\uc5f0\uc544\uac00 \uc5bb\uc740 \uc1fc\ud2b8\ud504\ub85c\uadf8\ub7a8 \uc810\uc218\ub294?", "paragraph": "\uae40\uc5f0\uc544\ub294 2014\ub144 1\uc6d4 3\uc77c\uc5d0\uc11c 5\uc77c\uae4c\uc9c0 \uac1c\ucd5c\ub418\ub294 \ud55c\uad6d \ud53c\uaca8 \uc885\ud569\uc120\uc218\uad8c \ub300\ud68c\uc5d0 \ucd9c\uc804\ud558\uc600\ub2e4. \uc5f0\uc2b5 \ub54c \ubd80\ud130 \ube48\ud2c8\uc5c6\ub294 \ubaa8\uc2b5\uc744 \ubcf4\uc5ec\uc900 \uae40\uc5f0\uc544\ub294 4\uc77c \uc5f4\ub9b0 \uc1fc\ud2b8 \ud504\ub85c\uadf8\ub7a8\uc5d0\uc11c \ubaa8\ub4e0 \uc218\ud589 \uae30\uc220\uc744 \uc644\ubcbd\ud558\uac8c \uc218\ud589\ud574\ub0b4\uba74\uc11c 80.60\uc810\uc744 \ud68d\ub4dd\ud558\uc600\ub2e4. \ube44\ub85d \ube44\uacf5\uc778 \uc810\uc218\uc774\uae34\ud558\uc9c0\ub9cc \ubc34\ucfe0\ubc84 \uc62c\ub9bc\ud53d \ub2f9\uc2dc \ubcf8\uc778\uc774 \uc218\ub9bd\ud55c \uc138\uacc4 \uc2e0\uae30\ub85d\uc778 78.50\uc810\ubcf4\ub2e4 2.1\uc810 \ub192\uc740, \uc5ec\uc790 \uc2f1\uae00 \uc0ac\uc0c1 \ucd5c\ucd08\uc758 80\uc810 \ub3cc\ud30c\uc774\ub2e4. 5\uc77c \uc5f4\ub9b0 \ud504\ub9ac \uc2a4\ucf00\uc774\ud305\uc5d0\uc120 \ub354\ube14 \uc545\uc140 \ub354\ube14 \ud1a0 \ub354\ube14 \ub8f9 \uc810\ud504\uc5d0\uc11c \uccab \uc810\ud504\uc778 \ub354\ube14 \uc545\uc140 \uc810\ud504\uc758 \ucc29\uc9c0\uac00 \ubd88\uc548\ud574 \uc138\ubc88\uc9f8 \uc810\ud504\uc778 \ub354\ube14 \ub8f9\uc744 \uc5f0\uacb0\ud558\uc9c0 \ubabb\ud558\uc600\uace0 \ub9c8\uc9c0\ub9c9 \uc810\ud504\uc778 \ub354\ube14 \uc545\uc140\uc744 \uc2f1\uae00 \uc545\uc140\ub85c \ucc98\ub9ac\ud558\ub294 \uc2e4\uc218\uac00 \uc788\uc5c8\uc9c0\ub9cc 147.26\uc810\uc744 \uae30\ub85d \ucd1d\uc810 227.86\uc810\uc744 \uae30\ub85d\ud588\ub2e4. \ubcf8\uc778\uc774 \uc218\ub9bd\ud55c \uc138\uacc4 \uc2e0\uae30\ub85d \ud504\ub9ac \uc2a4\ucf00\uc774\ud305 150.06\uc810\uacfc \ud569\uacc4 \uc810\uc218 228.56\uc810\uc5d0 \uac01\uac01 2.8\uc810, 0.7\uc810 \ubaa8\uc790\ub780 \uc810\uc218\uc774\ub2e4. 2\ub144 \uc5f0\uc18d \ud55c\uad6d \ud53c\uaca8 \uc885\ud569\uc120\uc218\uad8c \ub300\ud68c \uc6b0\uc2b9\uc744 \ucc28\uc9c0\ud558\uba70 2014\ub144 \ub3d9\uacc4 \uc62c\ub9bc\ud53d\uc744 \uc704\ud55c \ucd5c\uc885 \ub9ac\ud5c8\uc124\uc744 \uc131\uacf5\uc801\uc73c\ub85c \ub9c8\ucce4\ub2e4.", "answer": "80.60", "sentence": "\uc5f0\uc2b5 \ub54c \ubd80\ud130 \ube48\ud2c8\uc5c6\ub294 \ubaa8\uc2b5\uc744 \ubcf4\uc5ec\uc900 \uae40\uc5f0\uc544\ub294 4\uc77c \uc5f4\ub9b0 \uc1fc\ud2b8 \ud504\ub85c\uadf8\ub7a8\uc5d0\uc11c \ubaa8\ub4e0 \uc218\ud589 \uae30\uc220\uc744 \uc644\ubcbd\ud558\uac8c \uc218\ud589\ud574\ub0b4\uba74\uc11c 80.60 \uc810\uc744 \ud68d\ub4dd\ud558\uc600\ub2e4.", "paragraph_sentence": "\uae40\uc5f0\uc544\ub294 2014\ub144 1\uc6d4 3\uc77c\uc5d0\uc11c 5\uc77c\uae4c\uc9c0 \uac1c\ucd5c\ub418\ub294 \ud55c\uad6d \ud53c\uaca8 \uc885\ud569\uc120\uc218\uad8c \ub300\ud68c\uc5d0 \ucd9c\uc804\ud558\uc600\ub2e4. \uc5f0\uc2b5 \ub54c \ubd80\ud130 \ube48\ud2c8\uc5c6\ub294 \ubaa8\uc2b5\uc744 \ubcf4\uc5ec\uc900 \uae40\uc5f0\uc544\ub294 4\uc77c \uc5f4\ub9b0 \uc1fc\ud2b8 \ud504\ub85c\uadf8\ub7a8\uc5d0\uc11c \ubaa8\ub4e0 \uc218\ud589 \uae30\uc220\uc744 \uc644\ubcbd\ud558\uac8c \uc218\ud589\ud574\ub0b4\uba74\uc11c 80.60 \uc810\uc744 \ud68d\ub4dd\ud558\uc600\ub2e4. \ube44\ub85d \ube44\uacf5\uc778 \uc810\uc218\uc774\uae34\ud558\uc9c0\ub9cc \ubc34\ucfe0\ubc84 \uc62c\ub9bc\ud53d \ub2f9\uc2dc \ubcf8\uc778\uc774 \uc218\ub9bd\ud55c \uc138\uacc4 \uc2e0\uae30\ub85d\uc778 78.50\uc810\ubcf4\ub2e4 2.1\uc810 \ub192\uc740, \uc5ec\uc790 \uc2f1\uae00 \uc0ac\uc0c1 \ucd5c\ucd08\uc758 80\uc810 \ub3cc\ud30c\uc774\ub2e4. 5\uc77c \uc5f4\ub9b0 \ud504\ub9ac \uc2a4\ucf00\uc774\ud305\uc5d0\uc120 \ub354\ube14 \uc545\uc140 \ub354\ube14 \ud1a0 \ub354\ube14 \ub8f9 \uc810\ud504\uc5d0\uc11c \uccab \uc810\ud504\uc778 \ub354\ube14 \uc545\uc140 \uc810\ud504\uc758 \ucc29\uc9c0\uac00 \ubd88\uc548\ud574 \uc138\ubc88\uc9f8 \uc810\ud504\uc778 \ub354\ube14 \ub8f9\uc744 \uc5f0\uacb0\ud558\uc9c0 \ubabb\ud558\uc600\uace0 \ub9c8\uc9c0\ub9c9 \uc810\ud504\uc778 \ub354\ube14 \uc545\uc140\uc744 \uc2f1\uae00 \uc545\uc140\ub85c \ucc98\ub9ac\ud558\ub294 \uc2e4\uc218\uac00 \uc788\uc5c8\uc9c0\ub9cc 147.26\uc810\uc744 \uae30\ub85d \ucd1d\uc810 227.86\uc810\uc744 \uae30\ub85d\ud588\ub2e4. \ubcf8\uc778\uc774 \uc218\ub9bd\ud55c \uc138\uacc4 \uc2e0\uae30\ub85d \ud504\ub9ac \uc2a4\ucf00\uc774\ud305 150.06\uc810\uacfc \ud569\uacc4 \uc810\uc218 228.56\uc810\uc5d0 \uac01\uac01 2.8\uc810, 0.7\uc810 \ubaa8\uc790\ub780 \uc810\uc218\uc774\ub2e4. 2\ub144 \uc5f0\uc18d \ud55c\uad6d \ud53c\uaca8 \uc885\ud569\uc120\uc218\uad8c \ub300\ud68c \uc6b0\uc2b9\uc744 \ucc28\uc9c0\ud558\uba70 2014\ub144 \ub3d9\uacc4 \uc62c\ub9bc\ud53d\uc744 \uc704\ud55c \ucd5c\uc885 \ub9ac\ud5c8\uc124\uc744 \uc131\uacf5\uc801\uc73c\ub85c \ub9c8\ucce4\ub2e4.", "paragraph_answer": "\uae40\uc5f0\uc544\ub294 2014\ub144 1\uc6d4 3\uc77c\uc5d0\uc11c 5\uc77c\uae4c\uc9c0 \uac1c\ucd5c\ub418\ub294 \ud55c\uad6d \ud53c\uaca8 \uc885\ud569\uc120\uc218\uad8c \ub300\ud68c\uc5d0 \ucd9c\uc804\ud558\uc600\ub2e4. \uc5f0\uc2b5 \ub54c \ubd80\ud130 \ube48\ud2c8\uc5c6\ub294 \ubaa8\uc2b5\uc744 \ubcf4\uc5ec\uc900 \uae40\uc5f0\uc544\ub294 4\uc77c \uc5f4\ub9b0 \uc1fc\ud2b8 \ud504\ub85c\uadf8\ub7a8\uc5d0\uc11c \ubaa8\ub4e0 \uc218\ud589 \uae30\uc220\uc744 \uc644\ubcbd\ud558\uac8c \uc218\ud589\ud574\ub0b4\uba74\uc11c 80.60 \uc810\uc744 \ud68d\ub4dd\ud558\uc600\ub2e4. \ube44\ub85d \ube44\uacf5\uc778 \uc810\uc218\uc774\uae34\ud558\uc9c0\ub9cc \ubc34\ucfe0\ubc84 \uc62c\ub9bc\ud53d \ub2f9\uc2dc \ubcf8\uc778\uc774 \uc218\ub9bd\ud55c \uc138\uacc4 \uc2e0\uae30\ub85d\uc778 78.50\uc810\ubcf4\ub2e4 2.1\uc810 \ub192\uc740, \uc5ec\uc790 \uc2f1\uae00 \uc0ac\uc0c1 \ucd5c\ucd08\uc758 80\uc810 \ub3cc\ud30c\uc774\ub2e4. 5\uc77c \uc5f4\ub9b0 \ud504\ub9ac \uc2a4\ucf00\uc774\ud305\uc5d0\uc120 \ub354\ube14 \uc545\uc140 \ub354\ube14 \ud1a0 \ub354\ube14 \ub8f9 \uc810\ud504\uc5d0\uc11c \uccab \uc810\ud504\uc778 \ub354\ube14 \uc545\uc140 \uc810\ud504\uc758 \ucc29\uc9c0\uac00 \ubd88\uc548\ud574 \uc138\ubc88\uc9f8 \uc810\ud504\uc778 \ub354\ube14 \ub8f9\uc744 \uc5f0\uacb0\ud558\uc9c0 \ubabb\ud558\uc600\uace0 \ub9c8\uc9c0\ub9c9 \uc810\ud504\uc778 \ub354\ube14 \uc545\uc140\uc744 \uc2f1\uae00 \uc545\uc140\ub85c \ucc98\ub9ac\ud558\ub294 \uc2e4\uc218\uac00 \uc788\uc5c8\uc9c0\ub9cc 147.26\uc810\uc744 \uae30\ub85d \ucd1d\uc810 227.86\uc810\uc744 \uae30\ub85d\ud588\ub2e4. \ubcf8\uc778\uc774 \uc218\ub9bd\ud55c \uc138\uacc4 \uc2e0\uae30\ub85d \ud504\ub9ac \uc2a4\ucf00\uc774\ud305 150.06\uc810\uacfc \ud569\uacc4 \uc810\uc218 228.56\uc810\uc5d0 \uac01\uac01 2.8\uc810, 0.7\uc810 \ubaa8\uc790\ub780 \uc810\uc218\uc774\ub2e4. 2\ub144 \uc5f0\uc18d \ud55c\uad6d \ud53c\uaca8 \uc885\ud569\uc120\uc218\uad8c \ub300\ud68c \uc6b0\uc2b9\uc744 \ucc28\uc9c0\ud558\uba70 2014\ub144 \ub3d9\uacc4 \uc62c\ub9bc\ud53d\uc744 \uc704\ud55c \ucd5c\uc885 \ub9ac\ud5c8\uc124\uc744 \uc131\uacf5\uc801\uc73c\ub85c \ub9c8\ucce4\ub2e4.", "sentence_answer": "\uc5f0\uc2b5 \ub54c \ubd80\ud130 \ube48\ud2c8\uc5c6\ub294 \ubaa8\uc2b5\uc744 \ubcf4\uc5ec\uc900 \uae40\uc5f0\uc544\ub294 4\uc77c \uc5f4\ub9b0 \uc1fc\ud2b8 \ud504\ub85c\uadf8\ub7a8\uc5d0\uc11c \ubaa8\ub4e0 \uc218\ud589 \uae30\uc220\uc744 \uc644\ubcbd\ud558\uac8c \uc218\ud589\ud574\ub0b4\uba74\uc11c 80.60 \uc810\uc744 \ud68d\ub4dd\ud558\uc600\ub2e4.", "paragraph_id": "6482022-22-0", "paragraph_question": "question: 2014\ub144 \ud55c\uad6d \ud53c\uaca8 \uc885\ud569 \uc120\uc218\uad8c \ub300\ud68c\uc5d0\uc11c \uae40\uc5f0\uc544\uac00 \uc5bb\uc740 \uc1fc\ud2b8\ud504\ub85c\uadf8\ub7a8 \uc810\uc218\ub294?, context: \uae40\uc5f0\uc544\ub294 2014\ub144 1\uc6d4 3\uc77c\uc5d0\uc11c 5\uc77c\uae4c\uc9c0 \uac1c\ucd5c\ub418\ub294 \ud55c\uad6d \ud53c\uaca8 \uc885\ud569\uc120\uc218\uad8c \ub300\ud68c\uc5d0 \ucd9c\uc804\ud558\uc600\ub2e4. \uc5f0\uc2b5 \ub54c \ubd80\ud130 \ube48\ud2c8\uc5c6\ub294 \ubaa8\uc2b5\uc744 \ubcf4\uc5ec\uc900 \uae40\uc5f0\uc544\ub294 4\uc77c \uc5f4\ub9b0 \uc1fc\ud2b8 \ud504\ub85c\uadf8\ub7a8\uc5d0\uc11c \ubaa8\ub4e0 \uc218\ud589 \uae30\uc220\uc744 \uc644\ubcbd\ud558\uac8c \uc218\ud589\ud574\ub0b4\uba74\uc11c 80.60\uc810\uc744 \ud68d\ub4dd\ud558\uc600\ub2e4. \ube44\ub85d \ube44\uacf5\uc778 \uc810\uc218\uc774\uae34\ud558\uc9c0\ub9cc \ubc34\ucfe0\ubc84 \uc62c\ub9bc\ud53d \ub2f9\uc2dc \ubcf8\uc778\uc774 \uc218\ub9bd\ud55c \uc138\uacc4 \uc2e0\uae30\ub85d\uc778 78.50\uc810\ubcf4\ub2e4 2.1\uc810 \ub192\uc740, \uc5ec\uc790 \uc2f1\uae00 \uc0ac\uc0c1 \ucd5c\ucd08\uc758 80\uc810 \ub3cc\ud30c\uc774\ub2e4. 5\uc77c \uc5f4\ub9b0 \ud504\ub9ac \uc2a4\ucf00\uc774\ud305\uc5d0\uc120 \ub354\ube14 \uc545\uc140 \ub354\ube14 \ud1a0 \ub354\ube14 \ub8f9 \uc810\ud504\uc5d0\uc11c \uccab \uc810\ud504\uc778 \ub354\ube14 \uc545\uc140 \uc810\ud504\uc758 \ucc29\uc9c0\uac00 \ubd88\uc548\ud574 \uc138\ubc88\uc9f8 \uc810\ud504\uc778 \ub354\ube14 \ub8f9\uc744 \uc5f0\uacb0\ud558\uc9c0 \ubabb\ud558\uc600\uace0 \ub9c8\uc9c0\ub9c9 \uc810\ud504\uc778 \ub354\ube14 \uc545\uc140\uc744 \uc2f1\uae00 \uc545\uc140\ub85c \ucc98\ub9ac\ud558\ub294 \uc2e4\uc218\uac00 \uc788\uc5c8\uc9c0\ub9cc 147.26\uc810\uc744 \uae30\ub85d \ucd1d\uc810 227.86\uc810\uc744 \uae30\ub85d\ud588\ub2e4. \ubcf8\uc778\uc774 \uc218\ub9bd\ud55c \uc138\uacc4 \uc2e0\uae30\ub85d \ud504\ub9ac \uc2a4\ucf00\uc774\ud305 150.06\uc810\uacfc \ud569\uacc4 \uc810\uc218 228.56\uc810\uc5d0 \uac01\uac01 2.8\uc810, 0.7\uc810 \ubaa8\uc790\ub780 \uc810\uc218\uc774\ub2e4. 2\ub144 \uc5f0\uc18d \ud55c\uad6d \ud53c\uaca8 \uc885\ud569\uc120\uc218\uad8c \ub300\ud68c \uc6b0\uc2b9\uc744 \ucc28\uc9c0\ud558\uba70 2014\ub144 \ub3d9\uacc4 \uc62c\ub9bc\ud53d\uc744 \uc704\ud55c \ucd5c\uc885 \ub9ac\ud5c8\uc124\uc744 \uc131\uacf5\uc801\uc73c\ub85c \ub9c8\ucce4\ub2e4."} {"question": "\uc870\uc81c\ud504 \ube4c\ub810\ub818\uc740 \uc5b4\ub290\ub098\ub77c \ucd9c\uc2e0\uc758 \uc2e0\ubd80\uc600\ub098?", "paragraph": "1897\ub144 \uc544\ubc84\uc9c0\ub97c \ub530\ub77c \ucc9c\uc8fc\uad50\uc5d0 \uc785\uad50\ud558\uc5ec \ud504\ub791\uc2a4 \ucd9c\uc2e0\uc758 \ucc9c\uc8fc\uad50 \uc2e0\ubd80\uc600\ub358 \uc870\uc81c\ud504 \ube4c\ub810\ub818(Joseph Wilhelem, \ud55c\uad6d\uba85 \ud64d\uc11d\uad6c(\u6d2a\u932b\u4e5d))\uc73c\ub85c\ubd80\ud130 \ud1a0\ub9c8\uc2a4(\ub3c4\ub9c8, \ub2e4\ubb35(\u591a\u9ed8))\ub77c\ub294 \uc138\ub840\uba85\uc744 \ubc1b\uc558\ub2e4. \uc548\uc911\uadfc\uc774 \ud64d\uc11d\uad6c \uc2e0\ubd80\uc5d0\uac8c \ud504\ub791\uc2a4\uc5b4\ub97c \ubc30\uc6b0\uae30\ub3c4 \ud588\ub2e4\ub294 \uc124\uba85\ub3c4 \uc788\uc9c0\ub9cc, \uc548\uc911\uadfc\uc740 \ud64d\uc11d\uad6c \uc2e0\ubd80\uc640 \ud568\uaed8 \ubb88\ud154\uc8fc\uad50\uc640 \ub9cc\ub098 \uc774\uc57c\uae30\ub97c \ud55c \ud6c4\uc5d0 \ud504\ub791\uc2a4\uc5b4 \ubc30\uc6b0\uae30\ub97c \uadf8\ub9cc\ub450\uc5c8\ub2e4. \u300a\uc548\uc911\uadfc\u300b(\uc870\uc815\ub798 \uc9c0\uc74c, \ubb38\ud559\ub3d9\ub124)\uc5d0 \uc758\ud558\uba74, \ubb88\ud154 \uc8fc\uad50(\ud55c\uad6d\uc774\ub984 \ubbfc\uc8fc\uad50)\uac00 \uc870\uc120\uc0ac\ub78c\ub4e4\uc774 \ud559\ubb38\uc744 \ud558\ub294 \uac83\uc744 \ubc14\ub77c\uc9c0 \uc54a\uc558\uae30 \ub54c\ubb38\uc774\uc5c8\ub2e4. \ud559\ubb38\uc744 \ud558\uba74 \uba38\ub9ac\uac00 \uae68\uc77c \uac83\uc774\uace0, \uba38\ub9ac\uac00 \uae68\uc774\uba74 \ucc9c\uc8fc\uad50 \uad50\ub9ac\uc640 \uc2e0\uc559\uc5d0 \ub300\ud574 \ub098\ub984\ub300\ub85c\uc758 \ub17c\ub9ac\uc640 \ube44\ud310\uc73c\ub85c\uc368 \uc758\ubb38\uc744 \uac00\uc9c8 \uac83\uc774\uae30 \ub54c\ubb38\uc774\uc5c8\ub2e4. \uc774\ub97c \ubcf4\uba74\uc11c \uc2e4\ub9dd\ud55c \uc548\uc911\uadfc\uc740 \ud504\ub791\uc2a4\uc5b4 \ubc30\uc6b0\uae30\ub97c \uadf8\ub9cc\ub450\uc5c8\ub2e4. \uadf8 \ub4a4 \uc7a0\uc2dc \uad50\ud68c\uc758 \ucd1d\ub300(\u7e3d\u4ee3)\ub97c \ub9e1\uc558\uace0 \ub4a4\uc5d0 \ub9cc\uc778\uacc4(\u842c\u4eba\u5951\uff1a1,000\uba85 \uc774\uc0c1\uc758 \uacc4\uc6d0\uc744 \ubaa8\uc544 \ub3c8\uc744 \ucd9c\uc790\ud55c \ub4a4 \ucd94\ucca8\uc774\ub098 \uc785\ucc30\ub85c \ub3c8\uc744 \uc735\ud1b5\ud574 \uc8fc\ub294 \ubaa8\uc784)\uc758 \ucc44\ud45c\ud68c\uc0ac(\u5f69\u7968\u6703\u793e\uff1a\ub9cc\uc778\uacc4\uc758 \ub3c8\uc744 \uad00\ub9ac\ud558\uace0 \ucd94\ucca8\uc744 \ud558\ub294 \ud68c\uc0ac)\ub97c \uc124\ub9bd\ud558\uace0 \uc0ac\uc7a5\uc774 \ub418\uc5c8\ub2e4.", "answer": "\ud504\ub791\uc2a4", "sentence": "1897\ub144 \uc544\ubc84\uc9c0\ub97c \ub530\ub77c \ucc9c\uc8fc\uad50\uc5d0 \uc785\uad50\ud558\uc5ec \ud504\ub791\uc2a4 \ucd9c\uc2e0\uc758 \ucc9c\uc8fc\uad50 \uc2e0\ubd80\uc600\ub358 \uc870\uc81c\ud504 \ube4c\ub810\ub818(Joseph Wilhelem, \ud55c\uad6d\uba85 \ud64d\uc11d\uad6c(\u6d2a\u932b\u4e5d))\uc73c\ub85c\ubd80\ud130 \ud1a0\ub9c8\uc2a4(\ub3c4\ub9c8, \ub2e4\ubb35(\u591a\u9ed8))\ub77c\ub294 \uc138\ub840\uba85\uc744 \ubc1b\uc558\ub2e4.", "paragraph_sentence": " 1897\ub144 \uc544\ubc84\uc9c0\ub97c \ub530\ub77c \ucc9c\uc8fc\uad50\uc5d0 \uc785\uad50\ud558\uc5ec \ud504\ub791\uc2a4 \ucd9c\uc2e0\uc758 \ucc9c\uc8fc\uad50 \uc2e0\ubd80\uc600\ub358 \uc870\uc81c\ud504 \ube4c\ub810\ub818(Joseph Wilhelem, \ud55c\uad6d\uba85 \ud64d\uc11d\uad6c(\u6d2a\u932b\u4e5d))\uc73c\ub85c\ubd80\ud130 \ud1a0\ub9c8\uc2a4(\ub3c4\ub9c8, \ub2e4\ubb35(\u591a\u9ed8))\ub77c\ub294 \uc138\ub840\uba85\uc744 \ubc1b\uc558\ub2e4. \uc548\uc911\uadfc\uc774 \ud64d\uc11d\uad6c \uc2e0\ubd80\uc5d0\uac8c \ud504\ub791\uc2a4\uc5b4\ub97c \ubc30\uc6b0\uae30\ub3c4 \ud588\ub2e4\ub294 \uc124\uba85\ub3c4 \uc788\uc9c0\ub9cc, \uc548\uc911\uadfc\uc740 \ud64d\uc11d\uad6c \uc2e0\ubd80\uc640 \ud568\uaed8 \ubb88\ud154\uc8fc\uad50\uc640 \ub9cc\ub098 \uc774\uc57c\uae30\ub97c \ud55c \ud6c4\uc5d0 \ud504\ub791\uc2a4\uc5b4 \ubc30\uc6b0\uae30\ub97c \uadf8\ub9cc\ub450\uc5c8\ub2e4. \u300a\uc548\uc911\uadfc\u300b(\uc870\uc815\ub798 \uc9c0\uc74c, \ubb38\ud559\ub3d9\ub124)\uc5d0 \uc758\ud558\uba74, \ubb88\ud154 \uc8fc\uad50(\ud55c\uad6d\uc774\ub984 \ubbfc\uc8fc\uad50)\uac00 \uc870\uc120\uc0ac\ub78c\ub4e4\uc774 \ud559\ubb38\uc744 \ud558\ub294 \uac83\uc744 \ubc14\ub77c\uc9c0 \uc54a\uc558\uae30 \ub54c\ubb38\uc774\uc5c8\ub2e4. \ud559\ubb38\uc744 \ud558\uba74 \uba38\ub9ac\uac00 \uae68\uc77c \uac83\uc774\uace0, \uba38\ub9ac\uac00 \uae68\uc774\uba74 \ucc9c\uc8fc\uad50 \uad50\ub9ac\uc640 \uc2e0\uc559\uc5d0 \ub300\ud574 \ub098\ub984\ub300\ub85c\uc758 \ub17c\ub9ac\uc640 \ube44\ud310\uc73c\ub85c\uc368 \uc758\ubb38\uc744 \uac00\uc9c8 \uac83\uc774\uae30 \ub54c\ubb38\uc774\uc5c8\ub2e4. \uc774\ub97c \ubcf4\uba74\uc11c \uc2e4\ub9dd\ud55c \uc548\uc911\uadfc\uc740 \ud504\ub791\uc2a4\uc5b4 \ubc30\uc6b0\uae30\ub97c \uadf8\ub9cc\ub450\uc5c8\ub2e4. \uadf8 \ub4a4 \uc7a0\uc2dc \uad50\ud68c\uc758 \ucd1d\ub300(\u7e3d\u4ee3)\ub97c \ub9e1\uc558\uace0 \ub4a4\uc5d0 \ub9cc\uc778\uacc4(\u842c\u4eba\u5951\uff1a1,000\uba85 \uc774\uc0c1\uc758 \uacc4\uc6d0\uc744 \ubaa8\uc544 \ub3c8\uc744 \ucd9c\uc790\ud55c \ub4a4 \ucd94\ucca8\uc774\ub098 \uc785\ucc30\ub85c \ub3c8\uc744 \uc735\ud1b5\ud574 \uc8fc\ub294 \ubaa8\uc784)\uc758 \ucc44\ud45c\ud68c\uc0ac(\u5f69\u7968\u6703\u793e\uff1a\ub9cc\uc778\uacc4\uc758 \ub3c8\uc744 \uad00\ub9ac\ud558\uace0 \ucd94\ucca8\uc744 \ud558\ub294 \ud68c\uc0ac)\ub97c \uc124\ub9bd\ud558\uace0 \uc0ac\uc7a5\uc774 \ub418\uc5c8\ub2e4.", "paragraph_answer": "1897\ub144 \uc544\ubc84\uc9c0\ub97c \ub530\ub77c \ucc9c\uc8fc\uad50\uc5d0 \uc785\uad50\ud558\uc5ec \ud504\ub791\uc2a4 \ucd9c\uc2e0\uc758 \ucc9c\uc8fc\uad50 \uc2e0\ubd80\uc600\ub358 \uc870\uc81c\ud504 \ube4c\ub810\ub818(Joseph Wilhelem, \ud55c\uad6d\uba85 \ud64d\uc11d\uad6c(\u6d2a\u932b\u4e5d))\uc73c\ub85c\ubd80\ud130 \ud1a0\ub9c8\uc2a4(\ub3c4\ub9c8, \ub2e4\ubb35(\u591a\u9ed8))\ub77c\ub294 \uc138\ub840\uba85\uc744 \ubc1b\uc558\ub2e4. \uc548\uc911\uadfc\uc774 \ud64d\uc11d\uad6c \uc2e0\ubd80\uc5d0\uac8c \ud504\ub791\uc2a4\uc5b4\ub97c \ubc30\uc6b0\uae30\ub3c4 \ud588\ub2e4\ub294 \uc124\uba85\ub3c4 \uc788\uc9c0\ub9cc, \uc548\uc911\uadfc\uc740 \ud64d\uc11d\uad6c \uc2e0\ubd80\uc640 \ud568\uaed8 \ubb88\ud154\uc8fc\uad50\uc640 \ub9cc\ub098 \uc774\uc57c\uae30\ub97c \ud55c \ud6c4\uc5d0 \ud504\ub791\uc2a4\uc5b4 \ubc30\uc6b0\uae30\ub97c \uadf8\ub9cc\ub450\uc5c8\ub2e4. \u300a\uc548\uc911\uadfc\u300b(\uc870\uc815\ub798 \uc9c0\uc74c, \ubb38\ud559\ub3d9\ub124)\uc5d0 \uc758\ud558\uba74, \ubb88\ud154 \uc8fc\uad50(\ud55c\uad6d\uc774\ub984 \ubbfc\uc8fc\uad50)\uac00 \uc870\uc120\uc0ac\ub78c\ub4e4\uc774 \ud559\ubb38\uc744 \ud558\ub294 \uac83\uc744 \ubc14\ub77c\uc9c0 \uc54a\uc558\uae30 \ub54c\ubb38\uc774\uc5c8\ub2e4. \ud559\ubb38\uc744 \ud558\uba74 \uba38\ub9ac\uac00 \uae68\uc77c \uac83\uc774\uace0, \uba38\ub9ac\uac00 \uae68\uc774\uba74 \ucc9c\uc8fc\uad50 \uad50\ub9ac\uc640 \uc2e0\uc559\uc5d0 \ub300\ud574 \ub098\ub984\ub300\ub85c\uc758 \ub17c\ub9ac\uc640 \ube44\ud310\uc73c\ub85c\uc368 \uc758\ubb38\uc744 \uac00\uc9c8 \uac83\uc774\uae30 \ub54c\ubb38\uc774\uc5c8\ub2e4. \uc774\ub97c \ubcf4\uba74\uc11c \uc2e4\ub9dd\ud55c \uc548\uc911\uadfc\uc740 \ud504\ub791\uc2a4\uc5b4 \ubc30\uc6b0\uae30\ub97c \uadf8\ub9cc\ub450\uc5c8\ub2e4. \uadf8 \ub4a4 \uc7a0\uc2dc \uad50\ud68c\uc758 \ucd1d\ub300(\u7e3d\u4ee3)\ub97c \ub9e1\uc558\uace0 \ub4a4\uc5d0 \ub9cc\uc778\uacc4(\u842c\u4eba\u5951\uff1a1,000\uba85 \uc774\uc0c1\uc758 \uacc4\uc6d0\uc744 \ubaa8\uc544 \ub3c8\uc744 \ucd9c\uc790\ud55c \ub4a4 \ucd94\ucca8\uc774\ub098 \uc785\ucc30\ub85c \ub3c8\uc744 \uc735\ud1b5\ud574 \uc8fc\ub294 \ubaa8\uc784)\uc758 \ucc44\ud45c\ud68c\uc0ac(\u5f69\u7968\u6703\u793e\uff1a\ub9cc\uc778\uacc4\uc758 \ub3c8\uc744 \uad00\ub9ac\ud558\uace0 \ucd94\ucca8\uc744 \ud558\ub294 \ud68c\uc0ac)\ub97c \uc124\ub9bd\ud558\uace0 \uc0ac\uc7a5\uc774 \ub418\uc5c8\ub2e4.", "sentence_answer": "1897\ub144 \uc544\ubc84\uc9c0\ub97c \ub530\ub77c \ucc9c\uc8fc\uad50\uc5d0 \uc785\uad50\ud558\uc5ec \ud504\ub791\uc2a4 \ucd9c\uc2e0\uc758 \ucc9c\uc8fc\uad50 \uc2e0\ubd80\uc600\ub358 \uc870\uc81c\ud504 \ube4c\ub810\ub818(Joseph Wilhelem, \ud55c\uad6d\uba85 \ud64d\uc11d\uad6c(\u6d2a\u932b\u4e5d))\uc73c\ub85c\ubd80\ud130 \ud1a0\ub9c8\uc2a4(\ub3c4\ub9c8, \ub2e4\ubb35(\u591a\u9ed8))\ub77c\ub294 \uc138\ub840\uba85\uc744 \ubc1b\uc558\ub2e4.", "paragraph_id": "6585943-2-2", "paragraph_question": "question: \uc870\uc81c\ud504 \ube4c\ub810\ub818\uc740 \uc5b4\ub290\ub098\ub77c \ucd9c\uc2e0\uc758 \uc2e0\ubd80\uc600\ub098?, context: 1897\ub144 \uc544\ubc84\uc9c0\ub97c \ub530\ub77c \ucc9c\uc8fc\uad50\uc5d0 \uc785\uad50\ud558\uc5ec \ud504\ub791\uc2a4 \ucd9c\uc2e0\uc758 \ucc9c\uc8fc\uad50 \uc2e0\ubd80\uc600\ub358 \uc870\uc81c\ud504 \ube4c\ub810\ub818(Joseph Wilhelem, \ud55c\uad6d\uba85 \ud64d\uc11d\uad6c(\u6d2a\u932b\u4e5d))\uc73c\ub85c\ubd80\ud130 \ud1a0\ub9c8\uc2a4(\ub3c4\ub9c8, \ub2e4\ubb35(\u591a\u9ed8))\ub77c\ub294 \uc138\ub840\uba85\uc744 \ubc1b\uc558\ub2e4. \uc548\uc911\uadfc\uc774 \ud64d\uc11d\uad6c \uc2e0\ubd80\uc5d0\uac8c \ud504\ub791\uc2a4\uc5b4\ub97c \ubc30\uc6b0\uae30\ub3c4 \ud588\ub2e4\ub294 \uc124\uba85\ub3c4 \uc788\uc9c0\ub9cc, \uc548\uc911\uadfc\uc740 \ud64d\uc11d\uad6c \uc2e0\ubd80\uc640 \ud568\uaed8 \ubb88\ud154\uc8fc\uad50\uc640 \ub9cc\ub098 \uc774\uc57c\uae30\ub97c \ud55c \ud6c4\uc5d0 \ud504\ub791\uc2a4\uc5b4 \ubc30\uc6b0\uae30\ub97c \uadf8\ub9cc\ub450\uc5c8\ub2e4. \u300a\uc548\uc911\uadfc\u300b(\uc870\uc815\ub798 \uc9c0\uc74c, \ubb38\ud559\ub3d9\ub124)\uc5d0 \uc758\ud558\uba74, \ubb88\ud154 \uc8fc\uad50(\ud55c\uad6d\uc774\ub984 \ubbfc\uc8fc\uad50)\uac00 \uc870\uc120\uc0ac\ub78c\ub4e4\uc774 \ud559\ubb38\uc744 \ud558\ub294 \uac83\uc744 \ubc14\ub77c\uc9c0 \uc54a\uc558\uae30 \ub54c\ubb38\uc774\uc5c8\ub2e4. \ud559\ubb38\uc744 \ud558\uba74 \uba38\ub9ac\uac00 \uae68\uc77c \uac83\uc774\uace0, \uba38\ub9ac\uac00 \uae68\uc774\uba74 \ucc9c\uc8fc\uad50 \uad50\ub9ac\uc640 \uc2e0\uc559\uc5d0 \ub300\ud574 \ub098\ub984\ub300\ub85c\uc758 \ub17c\ub9ac\uc640 \ube44\ud310\uc73c\ub85c\uc368 \uc758\ubb38\uc744 \uac00\uc9c8 \uac83\uc774\uae30 \ub54c\ubb38\uc774\uc5c8\ub2e4. \uc774\ub97c \ubcf4\uba74\uc11c \uc2e4\ub9dd\ud55c \uc548\uc911\uadfc\uc740 \ud504\ub791\uc2a4\uc5b4 \ubc30\uc6b0\uae30\ub97c \uadf8\ub9cc\ub450\uc5c8\ub2e4. \uadf8 \ub4a4 \uc7a0\uc2dc \uad50\ud68c\uc758 \ucd1d\ub300(\u7e3d\u4ee3)\ub97c \ub9e1\uc558\uace0 \ub4a4\uc5d0 \ub9cc\uc778\uacc4(\u842c\u4eba\u5951\uff1a1,000\uba85 \uc774\uc0c1\uc758 \uacc4\uc6d0\uc744 \ubaa8\uc544 \ub3c8\uc744 \ucd9c\uc790\ud55c \ub4a4 \ucd94\ucca8\uc774\ub098 \uc785\ucc30\ub85c \ub3c8\uc744 \uc735\ud1b5\ud574 \uc8fc\ub294 \ubaa8\uc784)\uc758 \ucc44\ud45c\ud68c\uc0ac(\u5f69\u7968\u6703\u793e\uff1a\ub9cc\uc778\uacc4\uc758 \ub3c8\uc744 \uad00\ub9ac\ud558\uace0 \ucd94\ucca8\uc744 \ud558\ub294 \ud68c\uc0ac)\ub97c \uc124\ub9bd\ud558\uace0 \uc0ac\uc7a5\uc774 \ub418\uc5c8\ub2e4."} {"question": "\uadf8\uac00 \ub04a\uc784\uc5c6\uc774 \ubc29\ud574\ub97c \ubc1b\uac8c \ub418\ub294 \ub300\uc0c1\uc740 \ubb34\uc5c7\uc778\uac00?", "paragraph": "\uc8fc\uc778\uacf5 \ud0a4\ub9ac\uc774(Killy)\ub294 \ub05d\uc5c6\uc774 \ud3bc\uccd0\uc9c4 \uba87\ucc9c \uce35\uc774\ub098 \ub418\ub294 \uac70\ub300\ud55c \uad6c\uc870\ubb3c \uc18d\uc5d0\uc11c \uac10\uc5fc\ub418\uae30 \uc804\uc758 \ub124\ud2b8 \ub2e8\ub9d0 \uc720\uc804\uc790\ub97c \uac00\uc9c4 \uc778\uac04\uc744 \ucc3e\uc544 \uc5ec\ud589\ud55c\ub2e4. \uadf8\ub294 '\uc911\ub825\uc790 \ubc29\uc0ac\uc120 \uc0ac\ucd9c\uc7a5\uce58'(\uad8c\ucd1d \ubaa8\uc591\uc774\ub2e4)\ub77c\ub294 \uac15\ub825\ud55c \ubb34\uae30\ub97c \uac00\uc9c0\uace0 \uc788\uc73c\uba70, \ub4f1\ub85d\ucf54\ub4dc\uc640 \uac10\uc815\ucf54\ub4dc\uac00 \uc81c\uac70\ub41c \uc138\uc774\ud504 \uac00\ub4dc\uc774\ub2e4. \ub124\ud2b8\uc2a4\ud53c\uc5b4\uc5d0 \uc811\uc18d\ud560 \uc218 \uc788\ub294 \ub124\ud2b8 \ub2e8\ub9d0 \uc720\uc804\uc790\ub97c \uac00\uc9c4 \uc778\uac04\uc744 \ucc3e\uc544 \uc138\uacc4\ub97c '\uce74\uc624\uc2a4'\uc0c1\ud0dc\uc5d0\uc11c \uad6c\uc81c\ud558\ub294 \uac83\uc774 \uadf8\uc758 \uc784\ubb34\uc774\uba70 \uadf8 \uc678\uc758 \uacfc\uac70 \uae30\uc5b5\uc740 \ub9d0\uc18c\ub41c \uac83\uc73c\ub85c \ubcf4\uc778\ub2e4. \ub9cc\ud654\uc758 \ubc30\uacbd\uc774 \ub418\ub294 \uae30\uc800\ud604\uc2e4\uc758 \uba87 \ucc9c \uce35\uc774\ub098 \ub418\ub294 \uad6c\uc870\ubb3c\uc740 '\uac74\uc124\uc790'\ub4e4\uc5d0 \uc758\ud574 \uae30\ud558\uae09\uc218\uc801\uc73c\ub85c \uc99d\uc2dd\ub418\uc5b4 \ud604\uc7ac\ub294 \uadf8 \uaddc\ubaa8\uc870\ucc28 \uc54c \uc218 \uc5c6\ub294 \uc0c1\ud0dc\uc5d0 \uc788\uc73c\uba70 \uc774\uac83\uc744 \uba48\ucd9c \uc218 \uc788\ub294 \uac83\uc740 \uc624\uc9c1 \ub124\ud2b8 \ub2e8\ub9d0 \uc720\uc804\uc790\ub97c \uac00\uc9c4 \uc778\uac04\uc758 \uba85\ub839 \ubfd0\uc774\ub2e4. \ub610\ud55c \ub124\ud2b8 \ub2e8\ub9d0 \uc720\uc804\uc790\ub97c \uac00\uc9c0\uc9c0 \uc54a\uc740 \uc778\uac04\uc774 \ub124\ud2b8\uc2a4\ud53c\uc5b4\uc5d0 \uc811\uc18d\ud560 \uacbd\uc6b0 \ubd88\ubc95 \uac70\uc8fc\uc790\ub85c \uac04\uc8fc\ud558\uace0 \uc81c\uac70\ud558\ub294 \uac83\uc774 \uc784\ubb34\uc778 '\uc138\uc774\ud504\uac00\ub4dc'\uac00 '\uaddc\uc18c\uc0dd\ubb3c'\ub4e4\uc5d0 \uc758\ud574 \uac10\uc5fc\ub41c \uc778\uac04\ub4e4\uc744 \uc81c\uac70\ud558\ub294 \ud589\ub3d9\uc744 \ub9c9\ub294 \uac83\ub3c4 \ub124\ud2b8 \ub2e8\ub9d0 \uc720\uc804\uc790\ub97c \uac00\uc9c4 \uc778\uac04\ub9cc\uc774 \ud560 \uc218 \uc788\ub2e4. \uadf8\ub294 \uc138\uc0c1\uc758 '\uce74\uc624\uc2a4'(chaos) \uc0c1\ud0dc\ub97c \uc720\uc9c0\ud558\uae30 \uc704\ud574 \ub124\ud2b8 \ub2e8\ub9d0 \uc720\uc804\uc790\ub97c \uac00\uc9c4 \uc778\uac04\uc744 \uac10\uc5fc\uc2dc\ud0a4\uba70 \ud1b5\uce58\uad6d\uacfc \uc138\uc774\ud504 \uac00\ub4dc\uc640 \ub300\ub9bd\ud558\uace0 \uc788\ub294 '\uaddc\uc18c\uc0dd\ubb3c'\uc5d0\uac8c \ub04a\uc784\uc5c6\ub294 \ubc29\ud574\ub97c \ubc1b\uac8c \ub418\ub294\ub370, \uc774 \uc640\uc911\uc5d0 \uadf8\ub294 \uae30\uc800\ud604\uc2e4\uc758 \ud3d0\uae30\uacc4\uce35\uc5d0\uc11c '\uc0dd\uc804\uc0ac'\uc758 \uc8fc\uc784\uacfc\ud559\uc790\uc600\ub358 \uc2dc\ubcf4\ub97c \ub9cc\ub098\uac8c \ub418\uace0 \uadf8 \uc774\ud6c4 \uc2dc\ubcf4\ub294 \ud0a4\ub9ac\uc774\uc640 \ub3d9\ud589\ud558\uac8c \ub41c\ub2e4.", "answer": "\uaddc\uc18c\uc0dd\ubb3c", "sentence": "\ub610\ud55c \ub124\ud2b8 \ub2e8\ub9d0 \uc720\uc804\uc790\ub97c \uac00\uc9c0\uc9c0 \uc54a\uc740 \uc778\uac04\uc774 \ub124\ud2b8\uc2a4\ud53c\uc5b4\uc5d0 \uc811\uc18d\ud560 \uacbd\uc6b0 \ubd88\ubc95 \uac70\uc8fc\uc790\ub85c \uac04\uc8fc\ud558\uace0 \uc81c\uac70\ud558\ub294 \uac83\uc774 \uc784\ubb34\uc778 '\uc138\uc774\ud504\uac00\ub4dc'\uac00 ' \uaddc\uc18c\uc0dd\ubb3c '\ub4e4\uc5d0 \uc758\ud574 \uac10\uc5fc\ub41c \uc778\uac04\ub4e4\uc744 \uc81c\uac70\ud558\ub294 \ud589\ub3d9\uc744 \ub9c9\ub294 \uac83\ub3c4 \ub124\ud2b8 \ub2e8\ub9d0 \uc720\uc804\uc790\ub97c \uac00\uc9c4 \uc778\uac04\ub9cc\uc774 \ud560 \uc218 \uc788\ub2e4.", "paragraph_sentence": "\uc8fc\uc778\uacf5 \ud0a4\ub9ac\uc774(Killy)\ub294 \ub05d\uc5c6\uc774 \ud3bc\uccd0\uc9c4 \uba87\ucc9c \uce35\uc774\ub098 \ub418\ub294 \uac70\ub300\ud55c \uad6c\uc870\ubb3c \uc18d\uc5d0\uc11c \uac10\uc5fc\ub418\uae30 \uc804\uc758 \ub124\ud2b8 \ub2e8\ub9d0 \uc720\uc804\uc790\ub97c \uac00\uc9c4 \uc778\uac04\uc744 \ucc3e\uc544 \uc5ec\ud589\ud55c\ub2e4. \uadf8\ub294 '\uc911\ub825\uc790 \ubc29\uc0ac\uc120 \uc0ac\ucd9c\uc7a5\uce58'(\uad8c\ucd1d \ubaa8\uc591\uc774\ub2e4)\ub77c\ub294 \uac15\ub825\ud55c \ubb34\uae30\ub97c \uac00\uc9c0\uace0 \uc788\uc73c\uba70, \ub4f1\ub85d\ucf54\ub4dc\uc640 \uac10\uc815\ucf54\ub4dc\uac00 \uc81c\uac70\ub41c \uc138\uc774\ud504 \uac00\ub4dc\uc774\ub2e4. \ub124\ud2b8\uc2a4\ud53c\uc5b4\uc5d0 \uc811\uc18d\ud560 \uc218 \uc788\ub294 \ub124\ud2b8 \ub2e8\ub9d0 \uc720\uc804\uc790\ub97c \uac00\uc9c4 \uc778\uac04\uc744 \ucc3e\uc544 \uc138\uacc4\ub97c '\uce74\uc624\uc2a4'\uc0c1\ud0dc\uc5d0\uc11c \uad6c\uc81c\ud558\ub294 \uac83\uc774 \uadf8\uc758 \uc784\ubb34\uc774\uba70 \uadf8 \uc678\uc758 \uacfc\uac70 \uae30\uc5b5\uc740 \ub9d0\uc18c\ub41c \uac83\uc73c\ub85c \ubcf4\uc778\ub2e4. \ub9cc\ud654\uc758 \ubc30\uacbd\uc774 \ub418\ub294 \uae30\uc800\ud604\uc2e4\uc758 \uba87 \ucc9c \uce35\uc774\ub098 \ub418\ub294 \uad6c\uc870\ubb3c\uc740 '\uac74\uc124\uc790'\ub4e4\uc5d0 \uc758\ud574 \uae30\ud558\uae09\uc218\uc801\uc73c\ub85c \uc99d\uc2dd\ub418\uc5b4 \ud604\uc7ac\ub294 \uadf8 \uaddc\ubaa8\uc870\ucc28 \uc54c \uc218 \uc5c6\ub294 \uc0c1\ud0dc\uc5d0 \uc788\uc73c\uba70 \uc774\uac83\uc744 \uba48\ucd9c \uc218 \uc788\ub294 \uac83\uc740 \uc624\uc9c1 \ub124\ud2b8 \ub2e8\ub9d0 \uc720\uc804\uc790\ub97c \uac00\uc9c4 \uc778\uac04\uc758 \uba85\ub839 \ubfd0\uc774\ub2e4. \ub610\ud55c \ub124\ud2b8 \ub2e8\ub9d0 \uc720\uc804\uc790\ub97c \uac00\uc9c0\uc9c0 \uc54a\uc740 \uc778\uac04\uc774 \ub124\ud2b8\uc2a4\ud53c\uc5b4\uc5d0 \uc811\uc18d\ud560 \uacbd\uc6b0 \ubd88\ubc95 \uac70\uc8fc\uc790\ub85c \uac04\uc8fc\ud558\uace0 \uc81c\uac70\ud558\ub294 \uac83\uc774 \uc784\ubb34\uc778 '\uc138\uc774\ud504\uac00\ub4dc'\uac00 ' \uaddc\uc18c\uc0dd\ubb3c '\ub4e4\uc5d0 \uc758\ud574 \uac10\uc5fc\ub41c \uc778\uac04\ub4e4\uc744 \uc81c\uac70\ud558\ub294 \ud589\ub3d9\uc744 \ub9c9\ub294 \uac83\ub3c4 \ub124\ud2b8 \ub2e8\ub9d0 \uc720\uc804\uc790\ub97c \uac00\uc9c4 \uc778\uac04\ub9cc\uc774 \ud560 \uc218 \uc788\ub2e4. \uadf8\ub294 \uc138\uc0c1\uc758 '\uce74\uc624\uc2a4'(chaos) \uc0c1\ud0dc\ub97c \uc720\uc9c0\ud558\uae30 \uc704\ud574 \ub124\ud2b8 \ub2e8\ub9d0 \uc720\uc804\uc790\ub97c \uac00\uc9c4 \uc778\uac04\uc744 \uac10\uc5fc\uc2dc\ud0a4\uba70 \ud1b5\uce58\uad6d\uacfc \uc138\uc774\ud504 \uac00\ub4dc\uc640 \ub300\ub9bd\ud558\uace0 \uc788\ub294 '\uaddc\uc18c\uc0dd\ubb3c'\uc5d0\uac8c \ub04a\uc784\uc5c6\ub294 \ubc29\ud574\ub97c \ubc1b\uac8c \ub418\ub294\ub370, \uc774 \uc640\uc911\uc5d0 \uadf8\ub294 \uae30\uc800\ud604\uc2e4\uc758 \ud3d0\uae30\uacc4\uce35\uc5d0\uc11c '\uc0dd\uc804\uc0ac'\uc758 \uc8fc\uc784\uacfc\ud559\uc790\uc600\ub358 \uc2dc\ubcf4\ub97c \ub9cc\ub098\uac8c \ub418\uace0 \uadf8 \uc774\ud6c4 \uc2dc\ubcf4\ub294 \ud0a4\ub9ac\uc774\uc640 \ub3d9\ud589\ud558\uac8c \ub41c\ub2e4.", "paragraph_answer": "\uc8fc\uc778\uacf5 \ud0a4\ub9ac\uc774(Killy)\ub294 \ub05d\uc5c6\uc774 \ud3bc\uccd0\uc9c4 \uba87\ucc9c \uce35\uc774\ub098 \ub418\ub294 \uac70\ub300\ud55c \uad6c\uc870\ubb3c \uc18d\uc5d0\uc11c \uac10\uc5fc\ub418\uae30 \uc804\uc758 \ub124\ud2b8 \ub2e8\ub9d0 \uc720\uc804\uc790\ub97c \uac00\uc9c4 \uc778\uac04\uc744 \ucc3e\uc544 \uc5ec\ud589\ud55c\ub2e4. \uadf8\ub294 '\uc911\ub825\uc790 \ubc29\uc0ac\uc120 \uc0ac\ucd9c\uc7a5\uce58'(\uad8c\ucd1d \ubaa8\uc591\uc774\ub2e4)\ub77c\ub294 \uac15\ub825\ud55c \ubb34\uae30\ub97c \uac00\uc9c0\uace0 \uc788\uc73c\uba70, \ub4f1\ub85d\ucf54\ub4dc\uc640 \uac10\uc815\ucf54\ub4dc\uac00 \uc81c\uac70\ub41c \uc138\uc774\ud504 \uac00\ub4dc\uc774\ub2e4. \ub124\ud2b8\uc2a4\ud53c\uc5b4\uc5d0 \uc811\uc18d\ud560 \uc218 \uc788\ub294 \ub124\ud2b8 \ub2e8\ub9d0 \uc720\uc804\uc790\ub97c \uac00\uc9c4 \uc778\uac04\uc744 \ucc3e\uc544 \uc138\uacc4\ub97c '\uce74\uc624\uc2a4'\uc0c1\ud0dc\uc5d0\uc11c \uad6c\uc81c\ud558\ub294 \uac83\uc774 \uadf8\uc758 \uc784\ubb34\uc774\uba70 \uadf8 \uc678\uc758 \uacfc\uac70 \uae30\uc5b5\uc740 \ub9d0\uc18c\ub41c \uac83\uc73c\ub85c \ubcf4\uc778\ub2e4. \ub9cc\ud654\uc758 \ubc30\uacbd\uc774 \ub418\ub294 \uae30\uc800\ud604\uc2e4\uc758 \uba87 \ucc9c \uce35\uc774\ub098 \ub418\ub294 \uad6c\uc870\ubb3c\uc740 '\uac74\uc124\uc790'\ub4e4\uc5d0 \uc758\ud574 \uae30\ud558\uae09\uc218\uc801\uc73c\ub85c \uc99d\uc2dd\ub418\uc5b4 \ud604\uc7ac\ub294 \uadf8 \uaddc\ubaa8\uc870\ucc28 \uc54c \uc218 \uc5c6\ub294 \uc0c1\ud0dc\uc5d0 \uc788\uc73c\uba70 \uc774\uac83\uc744 \uba48\ucd9c \uc218 \uc788\ub294 \uac83\uc740 \uc624\uc9c1 \ub124\ud2b8 \ub2e8\ub9d0 \uc720\uc804\uc790\ub97c \uac00\uc9c4 \uc778\uac04\uc758 \uba85\ub839 \ubfd0\uc774\ub2e4. \ub610\ud55c \ub124\ud2b8 \ub2e8\ub9d0 \uc720\uc804\uc790\ub97c \uac00\uc9c0\uc9c0 \uc54a\uc740 \uc778\uac04\uc774 \ub124\ud2b8\uc2a4\ud53c\uc5b4\uc5d0 \uc811\uc18d\ud560 \uacbd\uc6b0 \ubd88\ubc95 \uac70\uc8fc\uc790\ub85c \uac04\uc8fc\ud558\uace0 \uc81c\uac70\ud558\ub294 \uac83\uc774 \uc784\ubb34\uc778 '\uc138\uc774\ud504\uac00\ub4dc'\uac00 ' \uaddc\uc18c\uc0dd\ubb3c '\ub4e4\uc5d0 \uc758\ud574 \uac10\uc5fc\ub41c \uc778\uac04\ub4e4\uc744 \uc81c\uac70\ud558\ub294 \ud589\ub3d9\uc744 \ub9c9\ub294 \uac83\ub3c4 \ub124\ud2b8 \ub2e8\ub9d0 \uc720\uc804\uc790\ub97c \uac00\uc9c4 \uc778\uac04\ub9cc\uc774 \ud560 \uc218 \uc788\ub2e4. \uadf8\ub294 \uc138\uc0c1\uc758 '\uce74\uc624\uc2a4'(chaos) \uc0c1\ud0dc\ub97c \uc720\uc9c0\ud558\uae30 \uc704\ud574 \ub124\ud2b8 \ub2e8\ub9d0 \uc720\uc804\uc790\ub97c \uac00\uc9c4 \uc778\uac04\uc744 \uac10\uc5fc\uc2dc\ud0a4\uba70 \ud1b5\uce58\uad6d\uacfc \uc138\uc774\ud504 \uac00\ub4dc\uc640 \ub300\ub9bd\ud558\uace0 \uc788\ub294 '\uaddc\uc18c\uc0dd\ubb3c'\uc5d0\uac8c \ub04a\uc784\uc5c6\ub294 \ubc29\ud574\ub97c \ubc1b\uac8c \ub418\ub294\ub370, \uc774 \uc640\uc911\uc5d0 \uadf8\ub294 \uae30\uc800\ud604\uc2e4\uc758 \ud3d0\uae30\uacc4\uce35\uc5d0\uc11c '\uc0dd\uc804\uc0ac'\uc758 \uc8fc\uc784\uacfc\ud559\uc790\uc600\ub358 \uc2dc\ubcf4\ub97c \ub9cc\ub098\uac8c \ub418\uace0 \uadf8 \uc774\ud6c4 \uc2dc\ubcf4\ub294 \ud0a4\ub9ac\uc774\uc640 \ub3d9\ud589\ud558\uac8c \ub41c\ub2e4.", "sentence_answer": "\ub610\ud55c \ub124\ud2b8 \ub2e8\ub9d0 \uc720\uc804\uc790\ub97c \uac00\uc9c0\uc9c0 \uc54a\uc740 \uc778\uac04\uc774 \ub124\ud2b8\uc2a4\ud53c\uc5b4\uc5d0 \uc811\uc18d\ud560 \uacbd\uc6b0 \ubd88\ubc95 \uac70\uc8fc\uc790\ub85c \uac04\uc8fc\ud558\uace0 \uc81c\uac70\ud558\ub294 \uac83\uc774 \uc784\ubb34\uc778 '\uc138\uc774\ud504\uac00\ub4dc'\uac00 ' \uaddc\uc18c\uc0dd\ubb3c '\ub4e4\uc5d0 \uc758\ud574 \uac10\uc5fc\ub41c \uc778\uac04\ub4e4\uc744 \uc81c\uac70\ud558\ub294 \ud589\ub3d9\uc744 \ub9c9\ub294 \uac83\ub3c4 \ub124\ud2b8 \ub2e8\ub9d0 \uc720\uc804\uc790\ub97c \uac00\uc9c4 \uc778\uac04\ub9cc\uc774 \ud560 \uc218 \uc788\ub2e4.", "paragraph_id": "6657167-0-2", "paragraph_question": "question: \uadf8\uac00 \ub04a\uc784\uc5c6\uc774 \ubc29\ud574\ub97c \ubc1b\uac8c \ub418\ub294 \ub300\uc0c1\uc740 \ubb34\uc5c7\uc778\uac00?, context: \uc8fc\uc778\uacf5 \ud0a4\ub9ac\uc774(Killy)\ub294 \ub05d\uc5c6\uc774 \ud3bc\uccd0\uc9c4 \uba87\ucc9c \uce35\uc774\ub098 \ub418\ub294 \uac70\ub300\ud55c \uad6c\uc870\ubb3c \uc18d\uc5d0\uc11c \uac10\uc5fc\ub418\uae30 \uc804\uc758 \ub124\ud2b8 \ub2e8\ub9d0 \uc720\uc804\uc790\ub97c \uac00\uc9c4 \uc778\uac04\uc744 \ucc3e\uc544 \uc5ec\ud589\ud55c\ub2e4. \uadf8\ub294 '\uc911\ub825\uc790 \ubc29\uc0ac\uc120 \uc0ac\ucd9c\uc7a5\uce58'(\uad8c\ucd1d \ubaa8\uc591\uc774\ub2e4)\ub77c\ub294 \uac15\ub825\ud55c \ubb34\uae30\ub97c \uac00\uc9c0\uace0 \uc788\uc73c\uba70, \ub4f1\ub85d\ucf54\ub4dc\uc640 \uac10\uc815\ucf54\ub4dc\uac00 \uc81c\uac70\ub41c \uc138\uc774\ud504 \uac00\ub4dc\uc774\ub2e4. \ub124\ud2b8\uc2a4\ud53c\uc5b4\uc5d0 \uc811\uc18d\ud560 \uc218 \uc788\ub294 \ub124\ud2b8 \ub2e8\ub9d0 \uc720\uc804\uc790\ub97c \uac00\uc9c4 \uc778\uac04\uc744 \ucc3e\uc544 \uc138\uacc4\ub97c '\uce74\uc624\uc2a4'\uc0c1\ud0dc\uc5d0\uc11c \uad6c\uc81c\ud558\ub294 \uac83\uc774 \uadf8\uc758 \uc784\ubb34\uc774\uba70 \uadf8 \uc678\uc758 \uacfc\uac70 \uae30\uc5b5\uc740 \ub9d0\uc18c\ub41c \uac83\uc73c\ub85c \ubcf4\uc778\ub2e4. \ub9cc\ud654\uc758 \ubc30\uacbd\uc774 \ub418\ub294 \uae30\uc800\ud604\uc2e4\uc758 \uba87 \ucc9c \uce35\uc774\ub098 \ub418\ub294 \uad6c\uc870\ubb3c\uc740 '\uac74\uc124\uc790'\ub4e4\uc5d0 \uc758\ud574 \uae30\ud558\uae09\uc218\uc801\uc73c\ub85c \uc99d\uc2dd\ub418\uc5b4 \ud604\uc7ac\ub294 \uadf8 \uaddc\ubaa8\uc870\ucc28 \uc54c \uc218 \uc5c6\ub294 \uc0c1\ud0dc\uc5d0 \uc788\uc73c\uba70 \uc774\uac83\uc744 \uba48\ucd9c \uc218 \uc788\ub294 \uac83\uc740 \uc624\uc9c1 \ub124\ud2b8 \ub2e8\ub9d0 \uc720\uc804\uc790\ub97c \uac00\uc9c4 \uc778\uac04\uc758 \uba85\ub839 \ubfd0\uc774\ub2e4. \ub610\ud55c \ub124\ud2b8 \ub2e8\ub9d0 \uc720\uc804\uc790\ub97c \uac00\uc9c0\uc9c0 \uc54a\uc740 \uc778\uac04\uc774 \ub124\ud2b8\uc2a4\ud53c\uc5b4\uc5d0 \uc811\uc18d\ud560 \uacbd\uc6b0 \ubd88\ubc95 \uac70\uc8fc\uc790\ub85c \uac04\uc8fc\ud558\uace0 \uc81c\uac70\ud558\ub294 \uac83\uc774 \uc784\ubb34\uc778 '\uc138\uc774\ud504\uac00\ub4dc'\uac00 '\uaddc\uc18c\uc0dd\ubb3c'\ub4e4\uc5d0 \uc758\ud574 \uac10\uc5fc\ub41c \uc778\uac04\ub4e4\uc744 \uc81c\uac70\ud558\ub294 \ud589\ub3d9\uc744 \ub9c9\ub294 \uac83\ub3c4 \ub124\ud2b8 \ub2e8\ub9d0 \uc720\uc804\uc790\ub97c \uac00\uc9c4 \uc778\uac04\ub9cc\uc774 \ud560 \uc218 \uc788\ub2e4. \uadf8\ub294 \uc138\uc0c1\uc758 '\uce74\uc624\uc2a4'(chaos) \uc0c1\ud0dc\ub97c \uc720\uc9c0\ud558\uae30 \uc704\ud574 \ub124\ud2b8 \ub2e8\ub9d0 \uc720\uc804\uc790\ub97c \uac00\uc9c4 \uc778\uac04\uc744 \uac10\uc5fc\uc2dc\ud0a4\uba70 \ud1b5\uce58\uad6d\uacfc \uc138\uc774\ud504 \uac00\ub4dc\uc640 \ub300\ub9bd\ud558\uace0 \uc788\ub294 '\uaddc\uc18c\uc0dd\ubb3c'\uc5d0\uac8c \ub04a\uc784\uc5c6\ub294 \ubc29\ud574\ub97c \ubc1b\uac8c \ub418\ub294\ub370, \uc774 \uc640\uc911\uc5d0 \uadf8\ub294 \uae30\uc800\ud604\uc2e4\uc758 \ud3d0\uae30\uacc4\uce35\uc5d0\uc11c '\uc0dd\uc804\uc0ac'\uc758 \uc8fc\uc784\uacfc\ud559\uc790\uc600\ub358 \uc2dc\ubcf4\ub97c \ub9cc\ub098\uac8c \ub418\uace0 \uadf8 \uc774\ud6c4 \uc2dc\ubcf4\ub294 \ud0a4\ub9ac\uc774\uc640 \ub3d9\ud589\ud558\uac8c \ub41c\ub2e4."} {"question": "\uc18c\ub828\uad70\uc740 \uc138\ubc14\uc2a4\ud1a0\ud3f4\uc744 \ubc29\uc5b4\ud558\uae30 \uc704\ud574 \uba87 \uac1c\uc758 \ubc29\uc5b4\uc120\uc744 \ub9cc\ub4e4\uc5b4 \ub450\uc5c8\ub294\uac00?", "paragraph": "\uc18c\ub828\uc5d0\uac8c\ub294 \ub2e4\ud589\uc2a4\ub7fd\uac8c\ub3c4, 1941\ub144 10\uc6d4 \ub9d0 \uc18c\uc7a5 \uc774\ubc18 \uc608\ud53c\ubaa8\ube44\uce58 \ud398\ud2b8\ub85c\ud504\uac00 \uc9c0\ud718\ud558\ub294 \ub3c5\ub9bd \ud574\uc548\uad70 32,000\uba85\uc774 \uc624\ub370\uc0ac\uc5d0\uc11c \uce58\uc5f4\ud55c \uc804\ud22c \uc774\ud6c4 \ubc14\ub2e4\ub97c \uc774\uc6a9\ud574 \uc138\ubc14\uc2a4\ud1a0\ud3f4\ub85c \ub3c4\ucc29\ud588\ub2e4. \uc18c\uc7a5 \uc774\ubc18 \uc608\ud53c\ubaa8\ube44\uce58 \ud398\ud2b8\ub85c\ud504\ub294 \ub0b4\ub959\uc5d0\uc11c \uc138\ubc14\uc2a4\ud1a0\ud3f4\ub85c \uc811\uadfc\ud558\ub294 \uacf3\uc744 \ubc29\uc5b4\ud558\uae30\ub85c \ud588\ub2e4. \uc18c\uc7a5 \uc774\ubc18 \uc608\ud53c\ubaa8\ube44\uce58 \ud398\ud2b8\ub85c\ud504\ub294 \ub0b4\ub959 \uc9c0\uc5ed\uc5d0 \uac00\uc7a5 \ubc14\uae65\ucabd\uc740 \ud56d\uad6c\ubd80\ud130 16km\uc5d0 \ub5a8\uc5b4\uc838 \uc788\ub294 3\uac1c \ubc29\uc5b4\uc120\uc744 \ub9cc\ub4e4\uc5b4 \ud56d\uad6c\ub97c \ud5a5\ud574 \uc804\uc9c4\ud558\ub294 \ucd94\ucd95\uad6d\uc744 \ubc29\uc5b4\ud558\ub824\uace0 \ud588\ub2e4. \uc18c\ub828 \uc81c51\uad70\uacfc \ud751\ud574 \ud568\ub300\ub97c \ud3ec\ud568\ud55c \uc18c\ub828\uad70\uc740 10\uc6d4 \ud06c\ub9bc\ubc18\ub3c4\uc5d0\uc11c \ud328\ubc30\ud558\uace0 12\uc6d4\uc5d0 \ud0c8\ucd9c\ud558\uba74\uc11c \uc138\ubc14\uc2a4\ud1a0\ud3f4\uc758 \uc8fc\uc694\ud55c \ubc29\uc5b4\uad70\uc73c\ub85c \uc18c\uc7a5 \uc774\ubc18 \uc608\ud53c\ubaa8\ube44\uce58\uac00 \uc9c0\ud718\ud558\ub294 \ub3c5\ub9bd \ud574\uc548\uad70\ub9cc \uc794\ub958\ud588\ub2e4. 9\uc6d4 26\uc77c\ubd80\ud130 11\uc6d4 16\uc77c \uc0ac\uc774 \ud06c\ub9bc\ubc18\ub3c4\uc758 \ub098\uba38\uc9c0 \uc9c0\uc5ed\uc744 \ub3c4\uc774\uce60\ub780\ud2b8\uad70\uc774 \uc810\ub839\ud558\uba74\uc11c \ub8e8\ub9c8\ub2c8\uc544 \uc81c3\uad70\uacfc \ub3c4\uc774\uce60\ub780\ud2b8 \uc81c11\uad70\uc740 \uc138\ubc14\uc2a4\ud1a0\ud3f4 \uacf5\uaca9\uc744 \uc900\ube44\ud588\ub2e4. \ub3c4\uc774\uce60\ub780\ud2b8 \uc81c11\uad70\uc740 \ucc98\uc74c\uc5d0\ub294 \ubcf4\ubcd1 7\uac1c \uc0ac\ub2e8\uc73c\ub85c \uc804\uccb4 \uc804\uc120 \uc911 \uac00\uc7a5 \uc57d\ud588\ub2e4. \ub8e8\ub9c8\ub2c8\uc544\uad70\uc740 \uc218\ub9cc \ub9ce\uc558\uc9c0 \uc81c\ub300\ub85c \ud6c8\ub828\ub418\uc9c0 \uc54a\uace0 \uc7a5\ube44\ub3c4 \ubd80\uc871\ud588\ub2e4. \ub0a0\uc528\uac00 \ub098\ube60\uc9c0\uace0 \uc9d1\uc911\ud638\uc6b0\ub85c \uc778\ud574 \ucd94\ucd95\uad6d\uc774 \ubd88\ub9ac\ud574\uc9c0\uba74\uc11c \uad70\uc0ac \uc99d\uac15\uc774 \uc9c0\uc5f0\ub41c 10\uc6d4 \uc911\uc21c, \ud751\ud574 \ud568\ub300 \uc0ac\ub839\uad00\uc778 \ubd80\uc81c\ub3c5 \ud544\ub9bd \uc624\ud06c\ud0f8\ube0c\ub974\uc2a4\ud0a4\ub294 \ub178\ubcf4\ub85c\uc2dc\uc2a4\ud06c\ub85c \ucd9c\ud56d\ud558\uc5ec \ubcd1\ub825\uacfc \ubb3c\uc790\ub97c \uc9c0\uc6d0\ubc1b\uc558\ub2e4. 12\uc6d4 17\uc77c\ubd80\ud130\ub294 \ub0a0\uc528\uac00 \ucd94\ucd95\uad6d\uc774 \uc791\uc804\uc744 \uc2dc\uc791\ud558\uae30 \uc88b\uc744 \uc815\ub3c4\ub85c \ub098\uc544\uc84c\ub2e4.", "answer": "3\uac1c", "sentence": "\uc18c\uc7a5 \uc774\ubc18 \uc608\ud53c\ubaa8\ube44\uce58 \ud398\ud2b8\ub85c\ud504\ub294 \ub0b4\ub959 \uc9c0\uc5ed\uc5d0 \uac00\uc7a5 \ubc14\uae65\ucabd\uc740 \ud56d\uad6c\ubd80\ud130 16km\uc5d0 \ub5a8\uc5b4\uc838 \uc788\ub294 3\uac1c \ubc29\uc5b4\uc120\uc744 \ub9cc\ub4e4\uc5b4 \ud56d\uad6c\ub97c \ud5a5\ud574 \uc804\uc9c4\ud558\ub294 \ucd94\ucd95\uad6d\uc744 \ubc29\uc5b4\ud558\ub824\uace0 \ud588\ub2e4.", "paragraph_sentence": "\uc18c\ub828\uc5d0\uac8c\ub294 \ub2e4\ud589\uc2a4\ub7fd\uac8c\ub3c4, 1941\ub144 10\uc6d4 \ub9d0 \uc18c\uc7a5 \uc774\ubc18 \uc608\ud53c\ubaa8\ube44\uce58 \ud398\ud2b8\ub85c\ud504\uac00 \uc9c0\ud718\ud558\ub294 \ub3c5\ub9bd \ud574\uc548\uad70 32,000\uba85\uc774 \uc624\ub370\uc0ac\uc5d0\uc11c \uce58\uc5f4\ud55c \uc804\ud22c \uc774\ud6c4 \ubc14\ub2e4\ub97c \uc774\uc6a9\ud574 \uc138\ubc14\uc2a4\ud1a0\ud3f4\ub85c \ub3c4\ucc29\ud588\ub2e4. \uc18c\uc7a5 \uc774\ubc18 \uc608\ud53c\ubaa8\ube44\uce58 \ud398\ud2b8\ub85c\ud504\ub294 \ub0b4\ub959\uc5d0\uc11c \uc138\ubc14\uc2a4\ud1a0\ud3f4\ub85c \uc811\uadfc\ud558\ub294 \uacf3\uc744 \ubc29\uc5b4\ud558\uae30\ub85c \ud588\ub2e4. \uc18c\uc7a5 \uc774\ubc18 \uc608\ud53c\ubaa8\ube44\uce58 \ud398\ud2b8\ub85c\ud504\ub294 \ub0b4\ub959 \uc9c0\uc5ed\uc5d0 \uac00\uc7a5 \ubc14\uae65\ucabd\uc740 \ud56d\uad6c\ubd80\ud130 16km\uc5d0 \ub5a8\uc5b4\uc838 \uc788\ub294 3\uac1c \ubc29\uc5b4\uc120\uc744 \ub9cc\ub4e4\uc5b4 \ud56d\uad6c\ub97c \ud5a5\ud574 \uc804\uc9c4\ud558\ub294 \ucd94\ucd95\uad6d\uc744 \ubc29\uc5b4\ud558\ub824\uace0 \ud588\ub2e4. \uc18c\ub828 \uc81c51\uad70\uacfc \ud751\ud574 \ud568\ub300\ub97c \ud3ec\ud568\ud55c \uc18c\ub828\uad70\uc740 10\uc6d4 \ud06c\ub9bc\ubc18\ub3c4\uc5d0\uc11c \ud328\ubc30\ud558\uace0 12\uc6d4\uc5d0 \ud0c8\ucd9c\ud558\uba74\uc11c \uc138\ubc14\uc2a4\ud1a0\ud3f4\uc758 \uc8fc\uc694\ud55c \ubc29\uc5b4\uad70\uc73c\ub85c \uc18c\uc7a5 \uc774\ubc18 \uc608\ud53c\ubaa8\ube44\uce58\uac00 \uc9c0\ud718\ud558\ub294 \ub3c5\ub9bd \ud574\uc548\uad70\ub9cc \uc794\ub958\ud588\ub2e4. 9\uc6d4 26\uc77c\ubd80\ud130 11\uc6d4 16\uc77c \uc0ac\uc774 \ud06c\ub9bc\ubc18\ub3c4\uc758 \ub098\uba38\uc9c0 \uc9c0\uc5ed\uc744 \ub3c4\uc774\uce60\ub780\ud2b8\uad70\uc774 \uc810\ub839\ud558\uba74\uc11c \ub8e8\ub9c8\ub2c8\uc544 \uc81c3\uad70\uacfc \ub3c4\uc774\uce60\ub780\ud2b8 \uc81c11\uad70\uc740 \uc138\ubc14\uc2a4\ud1a0\ud3f4 \uacf5\uaca9\uc744 \uc900\ube44\ud588\ub2e4. \ub3c4\uc774\uce60\ub780\ud2b8 \uc81c11\uad70\uc740 \ucc98\uc74c\uc5d0\ub294 \ubcf4\ubcd1 7\uac1c \uc0ac\ub2e8\uc73c\ub85c \uc804\uccb4 \uc804\uc120 \uc911 \uac00\uc7a5 \uc57d\ud588\ub2e4. \ub8e8\ub9c8\ub2c8\uc544\uad70\uc740 \uc218\ub9cc \ub9ce\uc558\uc9c0 \uc81c\ub300\ub85c \ud6c8\ub828\ub418\uc9c0 \uc54a\uace0 \uc7a5\ube44\ub3c4 \ubd80\uc871\ud588\ub2e4. \ub0a0\uc528\uac00 \ub098\ube60\uc9c0\uace0 \uc9d1\uc911\ud638\uc6b0\ub85c \uc778\ud574 \ucd94\ucd95\uad6d\uc774 \ubd88\ub9ac\ud574\uc9c0\uba74\uc11c \uad70\uc0ac \uc99d\uac15\uc774 \uc9c0\uc5f0\ub41c 10\uc6d4 \uc911\uc21c, \ud751\ud574 \ud568\ub300 \uc0ac\ub839\uad00\uc778 \ubd80\uc81c\ub3c5 \ud544\ub9bd \uc624\ud06c\ud0f8\ube0c\ub974\uc2a4\ud0a4\ub294 \ub178\ubcf4\ub85c\uc2dc\uc2a4\ud06c\ub85c \ucd9c\ud56d\ud558\uc5ec \ubcd1\ub825\uacfc \ubb3c\uc790\ub97c \uc9c0\uc6d0\ubc1b\uc558\ub2e4. 12\uc6d4 17\uc77c\ubd80\ud130\ub294 \ub0a0\uc528\uac00 \ucd94\ucd95\uad6d\uc774 \uc791\uc804\uc744 \uc2dc\uc791\ud558\uae30 \uc88b\uc744 \uc815\ub3c4\ub85c \ub098\uc544\uc84c\ub2e4.", "paragraph_answer": "\uc18c\ub828\uc5d0\uac8c\ub294 \ub2e4\ud589\uc2a4\ub7fd\uac8c\ub3c4, 1941\ub144 10\uc6d4 \ub9d0 \uc18c\uc7a5 \uc774\ubc18 \uc608\ud53c\ubaa8\ube44\uce58 \ud398\ud2b8\ub85c\ud504\uac00 \uc9c0\ud718\ud558\ub294 \ub3c5\ub9bd \ud574\uc548\uad70 32,000\uba85\uc774 \uc624\ub370\uc0ac\uc5d0\uc11c \uce58\uc5f4\ud55c \uc804\ud22c \uc774\ud6c4 \ubc14\ub2e4\ub97c \uc774\uc6a9\ud574 \uc138\ubc14\uc2a4\ud1a0\ud3f4\ub85c \ub3c4\ucc29\ud588\ub2e4. \uc18c\uc7a5 \uc774\ubc18 \uc608\ud53c\ubaa8\ube44\uce58 \ud398\ud2b8\ub85c\ud504\ub294 \ub0b4\ub959\uc5d0\uc11c \uc138\ubc14\uc2a4\ud1a0\ud3f4\ub85c \uc811\uadfc\ud558\ub294 \uacf3\uc744 \ubc29\uc5b4\ud558\uae30\ub85c \ud588\ub2e4. \uc18c\uc7a5 \uc774\ubc18 \uc608\ud53c\ubaa8\ube44\uce58 \ud398\ud2b8\ub85c\ud504\ub294 \ub0b4\ub959 \uc9c0\uc5ed\uc5d0 \uac00\uc7a5 \ubc14\uae65\ucabd\uc740 \ud56d\uad6c\ubd80\ud130 16km\uc5d0 \ub5a8\uc5b4\uc838 \uc788\ub294 3\uac1c \ubc29\uc5b4\uc120\uc744 \ub9cc\ub4e4\uc5b4 \ud56d\uad6c\ub97c \ud5a5\ud574 \uc804\uc9c4\ud558\ub294 \ucd94\ucd95\uad6d\uc744 \ubc29\uc5b4\ud558\ub824\uace0 \ud588\ub2e4. \uc18c\ub828 \uc81c51\uad70\uacfc \ud751\ud574 \ud568\ub300\ub97c \ud3ec\ud568\ud55c \uc18c\ub828\uad70\uc740 10\uc6d4 \ud06c\ub9bc\ubc18\ub3c4\uc5d0\uc11c \ud328\ubc30\ud558\uace0 12\uc6d4\uc5d0 \ud0c8\ucd9c\ud558\uba74\uc11c \uc138\ubc14\uc2a4\ud1a0\ud3f4\uc758 \uc8fc\uc694\ud55c \ubc29\uc5b4\uad70\uc73c\ub85c \uc18c\uc7a5 \uc774\ubc18 \uc608\ud53c\ubaa8\ube44\uce58\uac00 \uc9c0\ud718\ud558\ub294 \ub3c5\ub9bd \ud574\uc548\uad70\ub9cc \uc794\ub958\ud588\ub2e4. 9\uc6d4 26\uc77c\ubd80\ud130 11\uc6d4 16\uc77c \uc0ac\uc774 \ud06c\ub9bc\ubc18\ub3c4\uc758 \ub098\uba38\uc9c0 \uc9c0\uc5ed\uc744 \ub3c4\uc774\uce60\ub780\ud2b8\uad70\uc774 \uc810\ub839\ud558\uba74\uc11c \ub8e8\ub9c8\ub2c8\uc544 \uc81c3\uad70\uacfc \ub3c4\uc774\uce60\ub780\ud2b8 \uc81c11\uad70\uc740 \uc138\ubc14\uc2a4\ud1a0\ud3f4 \uacf5\uaca9\uc744 \uc900\ube44\ud588\ub2e4. \ub3c4\uc774\uce60\ub780\ud2b8 \uc81c11\uad70\uc740 \ucc98\uc74c\uc5d0\ub294 \ubcf4\ubcd1 7\uac1c \uc0ac\ub2e8\uc73c\ub85c \uc804\uccb4 \uc804\uc120 \uc911 \uac00\uc7a5 \uc57d\ud588\ub2e4. \ub8e8\ub9c8\ub2c8\uc544\uad70\uc740 \uc218\ub9cc \ub9ce\uc558\uc9c0 \uc81c\ub300\ub85c \ud6c8\ub828\ub418\uc9c0 \uc54a\uace0 \uc7a5\ube44\ub3c4 \ubd80\uc871\ud588\ub2e4. \ub0a0\uc528\uac00 \ub098\ube60\uc9c0\uace0 \uc9d1\uc911\ud638\uc6b0\ub85c \uc778\ud574 \ucd94\ucd95\uad6d\uc774 \ubd88\ub9ac\ud574\uc9c0\uba74\uc11c \uad70\uc0ac \uc99d\uac15\uc774 \uc9c0\uc5f0\ub41c 10\uc6d4 \uc911\uc21c, \ud751\ud574 \ud568\ub300 \uc0ac\ub839\uad00\uc778 \ubd80\uc81c\ub3c5 \ud544\ub9bd \uc624\ud06c\ud0f8\ube0c\ub974\uc2a4\ud0a4\ub294 \ub178\ubcf4\ub85c\uc2dc\uc2a4\ud06c\ub85c \ucd9c\ud56d\ud558\uc5ec \ubcd1\ub825\uacfc \ubb3c\uc790\ub97c \uc9c0\uc6d0\ubc1b\uc558\ub2e4. 12\uc6d4 17\uc77c\ubd80\ud130\ub294 \ub0a0\uc528\uac00 \ucd94\ucd95\uad6d\uc774 \uc791\uc804\uc744 \uc2dc\uc791\ud558\uae30 \uc88b\uc744 \uc815\ub3c4\ub85c \ub098\uc544\uc84c\ub2e4.", "sentence_answer": "\uc18c\uc7a5 \uc774\ubc18 \uc608\ud53c\ubaa8\ube44\uce58 \ud398\ud2b8\ub85c\ud504\ub294 \ub0b4\ub959 \uc9c0\uc5ed\uc5d0 \uac00\uc7a5 \ubc14\uae65\ucabd\uc740 \ud56d\uad6c\ubd80\ud130 16km\uc5d0 \ub5a8\uc5b4\uc838 \uc788\ub294 3\uac1c \ubc29\uc5b4\uc120\uc744 \ub9cc\ub4e4\uc5b4 \ud56d\uad6c\ub97c \ud5a5\ud574 \uc804\uc9c4\ud558\ub294 \ucd94\ucd95\uad6d\uc744 \ubc29\uc5b4\ud558\ub824\uace0 \ud588\ub2e4.", "paragraph_id": "6463472-3-0", "paragraph_question": "question: \uc18c\ub828\uad70\uc740 \uc138\ubc14\uc2a4\ud1a0\ud3f4\uc744 \ubc29\uc5b4\ud558\uae30 \uc704\ud574 \uba87 \uac1c\uc758 \ubc29\uc5b4\uc120\uc744 \ub9cc\ub4e4\uc5b4 \ub450\uc5c8\ub294\uac00?, context: \uc18c\ub828\uc5d0\uac8c\ub294 \ub2e4\ud589\uc2a4\ub7fd\uac8c\ub3c4, 1941\ub144 10\uc6d4 \ub9d0 \uc18c\uc7a5 \uc774\ubc18 \uc608\ud53c\ubaa8\ube44\uce58 \ud398\ud2b8\ub85c\ud504\uac00 \uc9c0\ud718\ud558\ub294 \ub3c5\ub9bd \ud574\uc548\uad70 32,000\uba85\uc774 \uc624\ub370\uc0ac\uc5d0\uc11c \uce58\uc5f4\ud55c \uc804\ud22c \uc774\ud6c4 \ubc14\ub2e4\ub97c \uc774\uc6a9\ud574 \uc138\ubc14\uc2a4\ud1a0\ud3f4\ub85c \ub3c4\ucc29\ud588\ub2e4. \uc18c\uc7a5 \uc774\ubc18 \uc608\ud53c\ubaa8\ube44\uce58 \ud398\ud2b8\ub85c\ud504\ub294 \ub0b4\ub959\uc5d0\uc11c \uc138\ubc14\uc2a4\ud1a0\ud3f4\ub85c \uc811\uadfc\ud558\ub294 \uacf3\uc744 \ubc29\uc5b4\ud558\uae30\ub85c \ud588\ub2e4. \uc18c\uc7a5 \uc774\ubc18 \uc608\ud53c\ubaa8\ube44\uce58 \ud398\ud2b8\ub85c\ud504\ub294 \ub0b4\ub959 \uc9c0\uc5ed\uc5d0 \uac00\uc7a5 \ubc14\uae65\ucabd\uc740 \ud56d\uad6c\ubd80\ud130 16km\uc5d0 \ub5a8\uc5b4\uc838 \uc788\ub294 3\uac1c \ubc29\uc5b4\uc120\uc744 \ub9cc\ub4e4\uc5b4 \ud56d\uad6c\ub97c \ud5a5\ud574 \uc804\uc9c4\ud558\ub294 \ucd94\ucd95\uad6d\uc744 \ubc29\uc5b4\ud558\ub824\uace0 \ud588\ub2e4. \uc18c\ub828 \uc81c51\uad70\uacfc \ud751\ud574 \ud568\ub300\ub97c \ud3ec\ud568\ud55c \uc18c\ub828\uad70\uc740 10\uc6d4 \ud06c\ub9bc\ubc18\ub3c4\uc5d0\uc11c \ud328\ubc30\ud558\uace0 12\uc6d4\uc5d0 \ud0c8\ucd9c\ud558\uba74\uc11c \uc138\ubc14\uc2a4\ud1a0\ud3f4\uc758 \uc8fc\uc694\ud55c \ubc29\uc5b4\uad70\uc73c\ub85c \uc18c\uc7a5 \uc774\ubc18 \uc608\ud53c\ubaa8\ube44\uce58\uac00 \uc9c0\ud718\ud558\ub294 \ub3c5\ub9bd \ud574\uc548\uad70\ub9cc \uc794\ub958\ud588\ub2e4. 9\uc6d4 26\uc77c\ubd80\ud130 11\uc6d4 16\uc77c \uc0ac\uc774 \ud06c\ub9bc\ubc18\ub3c4\uc758 \ub098\uba38\uc9c0 \uc9c0\uc5ed\uc744 \ub3c4\uc774\uce60\ub780\ud2b8\uad70\uc774 \uc810\ub839\ud558\uba74\uc11c \ub8e8\ub9c8\ub2c8\uc544 \uc81c3\uad70\uacfc \ub3c4\uc774\uce60\ub780\ud2b8 \uc81c11\uad70\uc740 \uc138\ubc14\uc2a4\ud1a0\ud3f4 \uacf5\uaca9\uc744 \uc900\ube44\ud588\ub2e4. \ub3c4\uc774\uce60\ub780\ud2b8 \uc81c11\uad70\uc740 \ucc98\uc74c\uc5d0\ub294 \ubcf4\ubcd1 7\uac1c \uc0ac\ub2e8\uc73c\ub85c \uc804\uccb4 \uc804\uc120 \uc911 \uac00\uc7a5 \uc57d\ud588\ub2e4. \ub8e8\ub9c8\ub2c8\uc544\uad70\uc740 \uc218\ub9cc \ub9ce\uc558\uc9c0 \uc81c\ub300\ub85c \ud6c8\ub828\ub418\uc9c0 \uc54a\uace0 \uc7a5\ube44\ub3c4 \ubd80\uc871\ud588\ub2e4. \ub0a0\uc528\uac00 \ub098\ube60\uc9c0\uace0 \uc9d1\uc911\ud638\uc6b0\ub85c \uc778\ud574 \ucd94\ucd95\uad6d\uc774 \ubd88\ub9ac\ud574\uc9c0\uba74\uc11c \uad70\uc0ac \uc99d\uac15\uc774 \uc9c0\uc5f0\ub41c 10\uc6d4 \uc911\uc21c, \ud751\ud574 \ud568\ub300 \uc0ac\ub839\uad00\uc778 \ubd80\uc81c\ub3c5 \ud544\ub9bd \uc624\ud06c\ud0f8\ube0c\ub974\uc2a4\ud0a4\ub294 \ub178\ubcf4\ub85c\uc2dc\uc2a4\ud06c\ub85c \ucd9c\ud56d\ud558\uc5ec \ubcd1\ub825\uacfc \ubb3c\uc790\ub97c \uc9c0\uc6d0\ubc1b\uc558\ub2e4. 12\uc6d4 17\uc77c\ubd80\ud130\ub294 \ub0a0\uc528\uac00 \ucd94\ucd95\uad6d\uc774 \uc791\uc804\uc744 \uc2dc\uc791\ud558\uae30 \uc88b\uc744 \uc815\ub3c4\ub85c \ub098\uc544\uc84c\ub2e4."} {"question": "\ub4dc\ub808\ud4cc\uc2a4 \uc0ac\uac74\uc5d0 \ub300\ud55c \uc548\ub0b4\uc11c\uc778 \uc99d\uac70\ub4e4\uc774\ub780 \ucc45\uc740 \ub204\uac00 \uc37c\ub294\uac00?", "paragraph": "\uc5d0\uc2a4\ud14c\ub77c\uc9c0\uc640 \ud568\uaed8 \ubb38\uc11c\ub97c \uc870\uc791\ud55c \uc559\ub9ac \uc911\ub839\uc774 \uccb4\ud3ec\ub418\uace0 \uac10\ubc29\uc5d0\uc11c \uc790\uc0b4\ud558\uba70 \ucc38\ubaa8\ubcf8\ubd80\uc758 \uc74c\ubaa8\uac00 \ub4dc\ub7ec\ub098\uc790 \uc88c\ud30c \uacf5\ud654\uc8fc\uc758\uc790\ub4e4\uc740 \uacbd\uc545\ud588\ub2e4. \uadf8\ub4e4\uc740 \uac00\ud1a8\ub9ad \uad50\ud68c\uc640 \uad70\ubd80\uac00 \uc548\ubcf4\ub97c \uad6c\uc2e4\ub85c \uc815\uce58\uc640 \uacf5\uacf5\uc0dd\ud65c\uc744 \uc5bc\ub9c8\ub098 \uce68\uc2dd\ud574 \ub4e4\uc5b4\uac14\ub294\uac00\ub97c \uc778\uc2dd\ud558\uae30 \uc2dc\uc791\ud588\ub2e4. \uc7a5 \uc870\ub808\uc2a4\ub294 \uc774 \ub54c \ub4dc\ub808\ud4cc\uc2a4 \uc0ac\uac74\uc744 \ud30c\ud5e4\uce58\ub294 \uc548\ub0b4\uc11c \u300a\uc99d\uac70\ub4e4\u300b\uc774\ub77c\ub294 \ucc45\uc744 \ud3b4\ub0c8\ub2e4. \uc774 \ub54c\ubd80\ud130 \ub4dc\ub808\ud4cc\uc2a4\uc758 \uc7ac\uc2ec\uc744 \uc694\uad6c\ud558\ub294 \ubaa9\uc18c\ub9ac\uac00 \ub192\uc544\uc84c\ub2e4. \ub85c\ub85c\ub974\uc9c0\ub294 \uac10\uc625\uc5d0\uc11c \uc790\uc0b4\ud55c \uc559\ub9ac \uc911\ub839\uc774 \uc790\uc0b4\ud558\uc9c0 \uc54a\uc558\ub2e4\ub294 \uae30\uc0ac\ub97c \uc2e4\uc5c8\ub2e4. \ub85c\ub85c\ub974\ub294 \"\uc608\uc218\ud68c\uc6d0\ub4e4\uc774 \uc559\ub9ac \uc911\ub839\uc758 \uc190\uc5d0 \uba74\ub3c4\ub0a0\uc744 \uc950\uc5b4\uc8fc\uba74\uc11c \ubd88\uba85\uc608 \ud1f4\uc5ed\uc73c\ub85c \ucd94\ubc29\ub418\uaca0\ub290\ub0d0 \uc544\ub2c8\uba74 \uadf8\uc758 \ubbf8\ub9dd\uc778\uc5d0\uac8c \uc5f0\uae08\uc744 \uc8fc\ub294 \uc870\uac74\uc73c\ub85c \uc790\uc0b4\ud558\uaca0\ub290\ub0d0 \ub458 \uc911 \ud558\ub098\ub97c \ud0dd\ud558\ub77c\uace0 \uac15\uc694\ud588\ub2e4\"\ub294 \uac83\uc774\uc5c8\ub2e4.", "answer": "\uc7a5 \uc870\ub808\uc2a4", "sentence": "\uc7a5 \uc870\ub808\uc2a4 \ub294 \uc774 \ub54c \ub4dc\ub808\ud4cc\uc2a4 \uc0ac\uac74\uc744 \ud30c\ud5e4\uce58\ub294 \uc548\ub0b4\uc11c \u300a\uc99d\uac70\ub4e4\u300b\uc774\ub77c\ub294 \ucc45\uc744 \ud3b4\ub0c8\ub2e4.", "paragraph_sentence": "\uc5d0\uc2a4\ud14c\ub77c\uc9c0\uc640 \ud568\uaed8 \ubb38\uc11c\ub97c \uc870\uc791\ud55c \uc559\ub9ac \uc911\ub839\uc774 \uccb4\ud3ec\ub418\uace0 \uac10\ubc29\uc5d0\uc11c \uc790\uc0b4\ud558\uba70 \ucc38\ubaa8\ubcf8\ubd80\uc758 \uc74c\ubaa8\uac00 \ub4dc\ub7ec\ub098\uc790 \uc88c\ud30c \uacf5\ud654\uc8fc\uc758\uc790\ub4e4\uc740 \uacbd\uc545\ud588\ub2e4. \uadf8\ub4e4\uc740 \uac00\ud1a8\ub9ad \uad50\ud68c\uc640 \uad70\ubd80\uac00 \uc548\ubcf4\ub97c \uad6c\uc2e4\ub85c \uc815\uce58\uc640 \uacf5\uacf5\uc0dd\ud65c\uc744 \uc5bc\ub9c8\ub098 \uce68\uc2dd\ud574 \ub4e4\uc5b4\uac14\ub294\uac00\ub97c \uc778\uc2dd\ud558\uae30 \uc2dc\uc791\ud588\ub2e4. \uc7a5 \uc870\ub808\uc2a4 \ub294 \uc774 \ub54c \ub4dc\ub808\ud4cc\uc2a4 \uc0ac\uac74\uc744 \ud30c\ud5e4\uce58\ub294 \uc548\ub0b4\uc11c \u300a\uc99d\uac70\ub4e4\u300b\uc774\ub77c\ub294 \ucc45\uc744 \ud3b4\ub0c8\ub2e4. \uc774 \ub54c\ubd80\ud130 \ub4dc\ub808\ud4cc\uc2a4\uc758 \uc7ac\uc2ec\uc744 \uc694\uad6c\ud558\ub294 \ubaa9\uc18c\ub9ac\uac00 \ub192\uc544\uc84c\ub2e4. \ub85c\ub85c\ub974\uc9c0\ub294 \uac10\uc625\uc5d0\uc11c \uc790\uc0b4\ud55c \uc559\ub9ac \uc911\ub839\uc774 \uc790\uc0b4\ud558\uc9c0 \uc54a\uc558\ub2e4\ub294 \uae30\uc0ac\ub97c \uc2e4\uc5c8\ub2e4. \ub85c\ub85c\ub974\ub294 \"\uc608\uc218\ud68c\uc6d0\ub4e4\uc774 \uc559\ub9ac \uc911\ub839\uc758 \uc190\uc5d0 \uba74\ub3c4\ub0a0\uc744 \uc950\uc5b4\uc8fc\uba74\uc11c \ubd88\uba85\uc608 \ud1f4\uc5ed\uc73c\ub85c \ucd94\ubc29\ub418\uaca0\ub290\ub0d0 \uc544\ub2c8\uba74 \uadf8\uc758 \ubbf8\ub9dd\uc778\uc5d0\uac8c \uc5f0\uae08\uc744 \uc8fc\ub294 \uc870\uac74\uc73c\ub85c \uc790\uc0b4\ud558\uaca0\ub290\ub0d0 \ub458 \uc911 \ud558\ub098\ub97c \ud0dd\ud558\ub77c\uace0 \uac15\uc694\ud588\ub2e4\"\ub294 \uac83\uc774\uc5c8\ub2e4.", "paragraph_answer": "\uc5d0\uc2a4\ud14c\ub77c\uc9c0\uc640 \ud568\uaed8 \ubb38\uc11c\ub97c \uc870\uc791\ud55c \uc559\ub9ac \uc911\ub839\uc774 \uccb4\ud3ec\ub418\uace0 \uac10\ubc29\uc5d0\uc11c \uc790\uc0b4\ud558\uba70 \ucc38\ubaa8\ubcf8\ubd80\uc758 \uc74c\ubaa8\uac00 \ub4dc\ub7ec\ub098\uc790 \uc88c\ud30c \uacf5\ud654\uc8fc\uc758\uc790\ub4e4\uc740 \uacbd\uc545\ud588\ub2e4. \uadf8\ub4e4\uc740 \uac00\ud1a8\ub9ad \uad50\ud68c\uc640 \uad70\ubd80\uac00 \uc548\ubcf4\ub97c \uad6c\uc2e4\ub85c \uc815\uce58\uc640 \uacf5\uacf5\uc0dd\ud65c\uc744 \uc5bc\ub9c8\ub098 \uce68\uc2dd\ud574 \ub4e4\uc5b4\uac14\ub294\uac00\ub97c \uc778\uc2dd\ud558\uae30 \uc2dc\uc791\ud588\ub2e4. \uc7a5 \uc870\ub808\uc2a4 \ub294 \uc774 \ub54c \ub4dc\ub808\ud4cc\uc2a4 \uc0ac\uac74\uc744 \ud30c\ud5e4\uce58\ub294 \uc548\ub0b4\uc11c \u300a\uc99d\uac70\ub4e4\u300b\uc774\ub77c\ub294 \ucc45\uc744 \ud3b4\ub0c8\ub2e4. \uc774 \ub54c\ubd80\ud130 \ub4dc\ub808\ud4cc\uc2a4\uc758 \uc7ac\uc2ec\uc744 \uc694\uad6c\ud558\ub294 \ubaa9\uc18c\ub9ac\uac00 \ub192\uc544\uc84c\ub2e4. \ub85c\ub85c\ub974\uc9c0\ub294 \uac10\uc625\uc5d0\uc11c \uc790\uc0b4\ud55c \uc559\ub9ac \uc911\ub839\uc774 \uc790\uc0b4\ud558\uc9c0 \uc54a\uc558\ub2e4\ub294 \uae30\uc0ac\ub97c \uc2e4\uc5c8\ub2e4. \ub85c\ub85c\ub974\ub294 \"\uc608\uc218\ud68c\uc6d0\ub4e4\uc774 \uc559\ub9ac \uc911\ub839\uc758 \uc190\uc5d0 \uba74\ub3c4\ub0a0\uc744 \uc950\uc5b4\uc8fc\uba74\uc11c \ubd88\uba85\uc608 \ud1f4\uc5ed\uc73c\ub85c \ucd94\ubc29\ub418\uaca0\ub290\ub0d0 \uc544\ub2c8\uba74 \uadf8\uc758 \ubbf8\ub9dd\uc778\uc5d0\uac8c \uc5f0\uae08\uc744 \uc8fc\ub294 \uc870\uac74\uc73c\ub85c \uc790\uc0b4\ud558\uaca0\ub290\ub0d0 \ub458 \uc911 \ud558\ub098\ub97c \ud0dd\ud558\ub77c\uace0 \uac15\uc694\ud588\ub2e4\"\ub294 \uac83\uc774\uc5c8\ub2e4.", "sentence_answer": " \uc7a5 \uc870\ub808\uc2a4 \ub294 \uc774 \ub54c \ub4dc\ub808\ud4cc\uc2a4 \uc0ac\uac74\uc744 \ud30c\ud5e4\uce58\ub294 \uc548\ub0b4\uc11c \u300a\uc99d\uac70\ub4e4\u300b\uc774\ub77c\ub294 \ucc45\uc744 \ud3b4\ub0c8\ub2e4.", "paragraph_id": "6579200-8-1", "paragraph_question": "question: \ub4dc\ub808\ud4cc\uc2a4 \uc0ac\uac74\uc5d0 \ub300\ud55c \uc548\ub0b4\uc11c\uc778 \uc99d\uac70\ub4e4\uc774\ub780 \ucc45\uc740 \ub204\uac00 \uc37c\ub294\uac00?, context: \uc5d0\uc2a4\ud14c\ub77c\uc9c0\uc640 \ud568\uaed8 \ubb38\uc11c\ub97c \uc870\uc791\ud55c \uc559\ub9ac \uc911\ub839\uc774 \uccb4\ud3ec\ub418\uace0 \uac10\ubc29\uc5d0\uc11c \uc790\uc0b4\ud558\uba70 \ucc38\ubaa8\ubcf8\ubd80\uc758 \uc74c\ubaa8\uac00 \ub4dc\ub7ec\ub098\uc790 \uc88c\ud30c \uacf5\ud654\uc8fc\uc758\uc790\ub4e4\uc740 \uacbd\uc545\ud588\ub2e4. \uadf8\ub4e4\uc740 \uac00\ud1a8\ub9ad \uad50\ud68c\uc640 \uad70\ubd80\uac00 \uc548\ubcf4\ub97c \uad6c\uc2e4\ub85c \uc815\uce58\uc640 \uacf5\uacf5\uc0dd\ud65c\uc744 \uc5bc\ub9c8\ub098 \uce68\uc2dd\ud574 \ub4e4\uc5b4\uac14\ub294\uac00\ub97c \uc778\uc2dd\ud558\uae30 \uc2dc\uc791\ud588\ub2e4. \uc7a5 \uc870\ub808\uc2a4\ub294 \uc774 \ub54c \ub4dc\ub808\ud4cc\uc2a4 \uc0ac\uac74\uc744 \ud30c\ud5e4\uce58\ub294 \uc548\ub0b4\uc11c \u300a\uc99d\uac70\ub4e4\u300b\uc774\ub77c\ub294 \ucc45\uc744 \ud3b4\ub0c8\ub2e4. \uc774 \ub54c\ubd80\ud130 \ub4dc\ub808\ud4cc\uc2a4\uc758 \uc7ac\uc2ec\uc744 \uc694\uad6c\ud558\ub294 \ubaa9\uc18c\ub9ac\uac00 \ub192\uc544\uc84c\ub2e4. \ub85c\ub85c\ub974\uc9c0\ub294 \uac10\uc625\uc5d0\uc11c \uc790\uc0b4\ud55c \uc559\ub9ac \uc911\ub839\uc774 \uc790\uc0b4\ud558\uc9c0 \uc54a\uc558\ub2e4\ub294 \uae30\uc0ac\ub97c \uc2e4\uc5c8\ub2e4. \ub85c\ub85c\ub974\ub294 \"\uc608\uc218\ud68c\uc6d0\ub4e4\uc774 \uc559\ub9ac \uc911\ub839\uc758 \uc190\uc5d0 \uba74\ub3c4\ub0a0\uc744 \uc950\uc5b4\uc8fc\uba74\uc11c \ubd88\uba85\uc608 \ud1f4\uc5ed\uc73c\ub85c \ucd94\ubc29\ub418\uaca0\ub290\ub0d0 \uc544\ub2c8\uba74 \uadf8\uc758 \ubbf8\ub9dd\uc778\uc5d0\uac8c \uc5f0\uae08\uc744 \uc8fc\ub294 \uc870\uac74\uc73c\ub85c \uc790\uc0b4\ud558\uaca0\ub290\ub0d0 \ub458 \uc911 \ud558\ub098\ub97c \ud0dd\ud558\ub77c\uace0 \uac15\uc694\ud588\ub2e4\"\ub294 \uac83\uc774\uc5c8\ub2e4."} {"question": "2013\ub144 8\uc6d4, \uc0c8\ub204\ub9ac\ub2f9 \uad00\uacc4\uc790\ub4e4\uc740 \uac74\uc124\ud604\uc7a5 \uc0ac\uace0\uc5d0 \uad00\ud55c \ucc45\uc784\uc744 \ubb3b\ub294 \ud56d\uc758\uc11c\ud55c\uc744 \uc81c\ucd9c\ud558\uae30 \uc704\ud574 \uc5b4\ub514\ub97c \ubc29\ubb38\ud558\uc600\ub294\uac00?", "paragraph": "\uc0c8\ub204\ub9ac\ub2f9 \uad00\uacc4\uc790\ub4e4\uc740 2013\ub144 8\uc6d4 2\uc77c\uc5d0 \uc11c\uc6b8\ud2b9\ubcc4\uc2dc\uccad\uc744 \ubc29\ubb38\ud558\uace0 \ubc15\uc6d0\uc21c \uc2dc\uc7a5\uc5d0\uac8c \uac74\uc124\ud604\uc7a5 \uc0ac\uace0\uc5d0 \uad00\ud55c \ucc45\uc784\uc744 \ubb3b\ub294 \ud56d\uc758 \uc11c\ud55c\uc744 \uc81c\ucd9c\ud558\ub824\uace0 \ud558\uc600\uc73c\ub098, \ubc15\uc6d0\uc21c\uacfc \uccad\uc6d0\uacbd\ucc30\uc758 \uac70\ubd80\ud589\uc704\ub85c \uc778\ud55c \ucda9\ub3cc\uc774 \uc788\uc5c8\ub2e4. \ud76c\uc0dd\uc790 \uc720\uac00\uc871\ub4e4\uc744 \ube44\ub86f\ud55c \uc0c8\ub204\ub9ac\ub2f9 \uad00\uacc4\uc790\ub4e4\uc740 \ubc15\uc6d0\uc21c\uc5d0\uac8c \ud56d\uc758 \uc11c\ud55c\uc744 \uc804\ub2ec\ud558\ub824\ub294 \ubaa9\uc801\uc774 \uc788\uc5c8\uc73c\ub098, \uc774\ub97c \uac70\ubd80\ud55c \uc11c\uc6b8\uc2dc\uccad \uce21\uc740 \uccad\uc6d0\uacbd\ucc30\uc744 \ub3d9\uc6d0\ud558\uc5ec \ubc29\ubb38\uc744 \ub9c9\uc558\uace0 \uc774 \uacfc\uc815\uc5d0\uc11c \ubab8\uc2f8\uc6c0\uc774 \ubc8c\uc5b4\uc838 \uccad\uc6d0\uacbd\ucc30\uc774 \ubd80\uc0c1\uc744 \uc785\uc5c8\ub2e4. \uc774\uc5d0 \ub300\ud574 \uc11c\uc6b8\uc2dc \uce21\uc740, \uc11c\uc6b8\uc2dc \uc9c1\uc6d0\uc774 \uc0c1\ud574\ub97c \uc785\uc740 \uac83\uc5d0 \ub300\ud574 \uc720\uac10\uc758 \ub73b\uc744 \ud45c\uba85\ud558\uc600\ub2e4. \ubc18\uba74 \uae40\uc131\ud0dc \uc758\uc6d0\uc2e4 \uce21\uc740, \uc11c\uc6b8\uc2dc\ubbfc\uc758 \uc548\uc804\uc744 \uc9c0\ud0a4\uc9c0 \ubabb\ud558\uace0 \uc720\uac00\uc871\uacfc \ubbfc\uc6d0\uc778\uc758 \ud56d\uc758 \ubc29\ubb38\uc744 \ud68c\ud53c\ud55c \ubc15\uc6d0\uc21c \uc2dc\uc7a5\uc774 \ucc45\uc784\uc744 \uc838\uc57c\ud55c\ub2e4\uace0 \uc8fc\uc7a5\ud558\uc600\ub2e4. \uc774\ub7ec\ud55c \uc77c\ub828\uc758 \uc0ac\uac74 \uc911 \uae30\ud0c0 \uc2e0\ubb38\uae30\uc0ac\ub098 \uc778\ud130\ub137 \ub4f1\uc9c0\uc5d0\uc11c \uc0c8\ub204\ub9ac\ub2f9 \uad00\uacc4\uc790\ub97c \uae40\uc131\ud0dc \uc758\uc6d0\uc2e4 \uad00\uacc4\uc790\ub85c \uc798\ubabb \ud45c\uae30\ud558\ub294 \uc624\ubcf4\uac00 \ubc1c\uc0dd\ud588\ub2e4.", "answer": "\uc11c\uc6b8\ud2b9\ubcc4\uc2dc\uccad", "sentence": "\uc0c8\ub204\ub9ac\ub2f9 \uad00\uacc4\uc790\ub4e4\uc740 2013\ub144 8\uc6d4 2\uc77c\uc5d0 \uc11c\uc6b8\ud2b9\ubcc4\uc2dc\uccad \uc744 \ubc29\ubb38\ud558\uace0 \ubc15\uc6d0\uc21c \uc2dc\uc7a5\uc5d0\uac8c \uac74\uc124\ud604\uc7a5 \uc0ac\uace0\uc5d0 \uad00\ud55c \ucc45\uc784\uc744 \ubb3b\ub294 \ud56d\uc758 \uc11c\ud55c\uc744 \uc81c\ucd9c\ud558\ub824\uace0 \ud558\uc600\uc73c\ub098, \ubc15\uc6d0\uc21c\uacfc \uccad\uc6d0\uacbd\ucc30\uc758 \uac70\ubd80\ud589\uc704\ub85c \uc778\ud55c \ucda9\ub3cc\uc774 \uc788\uc5c8\ub2e4.", "paragraph_sentence": " \uc0c8\ub204\ub9ac\ub2f9 \uad00\uacc4\uc790\ub4e4\uc740 2013\ub144 8\uc6d4 2\uc77c\uc5d0 \uc11c\uc6b8\ud2b9\ubcc4\uc2dc\uccad \uc744 \ubc29\ubb38\ud558\uace0 \ubc15\uc6d0\uc21c \uc2dc\uc7a5\uc5d0\uac8c \uac74\uc124\ud604\uc7a5 \uc0ac\uace0\uc5d0 \uad00\ud55c \ucc45\uc784\uc744 \ubb3b\ub294 \ud56d\uc758 \uc11c\ud55c\uc744 \uc81c\ucd9c\ud558\ub824\uace0 \ud558\uc600\uc73c\ub098, \ubc15\uc6d0\uc21c\uacfc \uccad\uc6d0\uacbd\ucc30\uc758 \uac70\ubd80\ud589\uc704\ub85c \uc778\ud55c \ucda9\ub3cc\uc774 \uc788\uc5c8\ub2e4. \ud76c\uc0dd\uc790 \uc720\uac00\uc871\ub4e4\uc744 \ube44\ub86f\ud55c \uc0c8\ub204\ub9ac\ub2f9 \uad00\uacc4\uc790\ub4e4\uc740 \ubc15\uc6d0\uc21c\uc5d0\uac8c \ud56d\uc758 \uc11c\ud55c\uc744 \uc804\ub2ec\ud558\ub824\ub294 \ubaa9\uc801\uc774 \uc788\uc5c8\uc73c\ub098, \uc774\ub97c \uac70\ubd80\ud55c \uc11c\uc6b8\uc2dc\uccad \uce21\uc740 \uccad\uc6d0\uacbd\ucc30\uc744 \ub3d9\uc6d0\ud558\uc5ec \ubc29\ubb38\uc744 \ub9c9\uc558\uace0 \uc774 \uacfc\uc815\uc5d0\uc11c \ubab8\uc2f8\uc6c0\uc774 \ubc8c\uc5b4\uc838 \uccad\uc6d0\uacbd\ucc30\uc774 \ubd80\uc0c1\uc744 \uc785\uc5c8\ub2e4. \uc774\uc5d0 \ub300\ud574 \uc11c\uc6b8\uc2dc \uce21\uc740, \uc11c\uc6b8\uc2dc \uc9c1\uc6d0\uc774 \uc0c1\ud574\ub97c \uc785\uc740 \uac83\uc5d0 \ub300\ud574 \uc720\uac10\uc758 \ub73b\uc744 \ud45c\uba85\ud558\uc600\ub2e4. \ubc18\uba74 \uae40\uc131\ud0dc \uc758\uc6d0\uc2e4 \uce21\uc740, \uc11c\uc6b8\uc2dc\ubbfc\uc758 \uc548\uc804\uc744 \uc9c0\ud0a4\uc9c0 \ubabb\ud558\uace0 \uc720\uac00\uc871\uacfc \ubbfc\uc6d0\uc778\uc758 \ud56d\uc758 \ubc29\ubb38\uc744 \ud68c\ud53c\ud55c \ubc15\uc6d0\uc21c \uc2dc\uc7a5\uc774 \ucc45\uc784\uc744 \uc838\uc57c\ud55c\ub2e4\uace0 \uc8fc\uc7a5\ud558\uc600\ub2e4. \uc774\ub7ec\ud55c \uc77c\ub828\uc758 \uc0ac\uac74 \uc911 \uae30\ud0c0 \uc2e0\ubb38\uae30\uc0ac\ub098 \uc778\ud130\ub137 \ub4f1\uc9c0\uc5d0\uc11c \uc0c8\ub204\ub9ac\ub2f9 \uad00\uacc4\uc790\ub97c \uae40\uc131\ud0dc \uc758\uc6d0\uc2e4 \uad00\uacc4\uc790\ub85c \uc798\ubabb \ud45c\uae30\ud558\ub294 \uc624\ubcf4\uac00 \ubc1c\uc0dd\ud588\ub2e4.", "paragraph_answer": "\uc0c8\ub204\ub9ac\ub2f9 \uad00\uacc4\uc790\ub4e4\uc740 2013\ub144 8\uc6d4 2\uc77c\uc5d0 \uc11c\uc6b8\ud2b9\ubcc4\uc2dc\uccad \uc744 \ubc29\ubb38\ud558\uace0 \ubc15\uc6d0\uc21c \uc2dc\uc7a5\uc5d0\uac8c \uac74\uc124\ud604\uc7a5 \uc0ac\uace0\uc5d0 \uad00\ud55c \ucc45\uc784\uc744 \ubb3b\ub294 \ud56d\uc758 \uc11c\ud55c\uc744 \uc81c\ucd9c\ud558\ub824\uace0 \ud558\uc600\uc73c\ub098, \ubc15\uc6d0\uc21c\uacfc \uccad\uc6d0\uacbd\ucc30\uc758 \uac70\ubd80\ud589\uc704\ub85c \uc778\ud55c \ucda9\ub3cc\uc774 \uc788\uc5c8\ub2e4. \ud76c\uc0dd\uc790 \uc720\uac00\uc871\ub4e4\uc744 \ube44\ub86f\ud55c \uc0c8\ub204\ub9ac\ub2f9 \uad00\uacc4\uc790\ub4e4\uc740 \ubc15\uc6d0\uc21c\uc5d0\uac8c \ud56d\uc758 \uc11c\ud55c\uc744 \uc804\ub2ec\ud558\ub824\ub294 \ubaa9\uc801\uc774 \uc788\uc5c8\uc73c\ub098, \uc774\ub97c \uac70\ubd80\ud55c \uc11c\uc6b8\uc2dc\uccad \uce21\uc740 \uccad\uc6d0\uacbd\ucc30\uc744 \ub3d9\uc6d0\ud558\uc5ec \ubc29\ubb38\uc744 \ub9c9\uc558\uace0 \uc774 \uacfc\uc815\uc5d0\uc11c \ubab8\uc2f8\uc6c0\uc774 \ubc8c\uc5b4\uc838 \uccad\uc6d0\uacbd\ucc30\uc774 \ubd80\uc0c1\uc744 \uc785\uc5c8\ub2e4. \uc774\uc5d0 \ub300\ud574 \uc11c\uc6b8\uc2dc \uce21\uc740, \uc11c\uc6b8\uc2dc \uc9c1\uc6d0\uc774 \uc0c1\ud574\ub97c \uc785\uc740 \uac83\uc5d0 \ub300\ud574 \uc720\uac10\uc758 \ub73b\uc744 \ud45c\uba85\ud558\uc600\ub2e4. \ubc18\uba74 \uae40\uc131\ud0dc \uc758\uc6d0\uc2e4 \uce21\uc740, \uc11c\uc6b8\uc2dc\ubbfc\uc758 \uc548\uc804\uc744 \uc9c0\ud0a4\uc9c0 \ubabb\ud558\uace0 \uc720\uac00\uc871\uacfc \ubbfc\uc6d0\uc778\uc758 \ud56d\uc758 \ubc29\ubb38\uc744 \ud68c\ud53c\ud55c \ubc15\uc6d0\uc21c \uc2dc\uc7a5\uc774 \ucc45\uc784\uc744 \uc838\uc57c\ud55c\ub2e4\uace0 \uc8fc\uc7a5\ud558\uc600\ub2e4. \uc774\ub7ec\ud55c \uc77c\ub828\uc758 \uc0ac\uac74 \uc911 \uae30\ud0c0 \uc2e0\ubb38\uae30\uc0ac\ub098 \uc778\ud130\ub137 \ub4f1\uc9c0\uc5d0\uc11c \uc0c8\ub204\ub9ac\ub2f9 \uad00\uacc4\uc790\ub97c \uae40\uc131\ud0dc \uc758\uc6d0\uc2e4 \uad00\uacc4\uc790\ub85c \uc798\ubabb \ud45c\uae30\ud558\ub294 \uc624\ubcf4\uac00 \ubc1c\uc0dd\ud588\ub2e4.", "sentence_answer": "\uc0c8\ub204\ub9ac\ub2f9 \uad00\uacc4\uc790\ub4e4\uc740 2013\ub144 8\uc6d4 2\uc77c\uc5d0 \uc11c\uc6b8\ud2b9\ubcc4\uc2dc\uccad \uc744 \ubc29\ubb38\ud558\uace0 \ubc15\uc6d0\uc21c \uc2dc\uc7a5\uc5d0\uac8c \uac74\uc124\ud604\uc7a5 \uc0ac\uace0\uc5d0 \uad00\ud55c \ucc45\uc784\uc744 \ubb3b\ub294 \ud56d\uc758 \uc11c\ud55c\uc744 \uc81c\ucd9c\ud558\ub824\uace0 \ud558\uc600\uc73c\ub098, \ubc15\uc6d0\uc21c\uacfc \uccad\uc6d0\uacbd\ucc30\uc758 \uac70\ubd80\ud589\uc704\ub85c \uc778\ud55c \ucda9\ub3cc\uc774 \uc788\uc5c8\ub2e4.", "paragraph_id": "6490557-1-0", "paragraph_question": "question: 2013\ub144 8\uc6d4, \uc0c8\ub204\ub9ac\ub2f9 \uad00\uacc4\uc790\ub4e4\uc740 \uac74\uc124\ud604\uc7a5 \uc0ac\uace0\uc5d0 \uad00\ud55c \ucc45\uc784\uc744 \ubb3b\ub294 \ud56d\uc758\uc11c\ud55c\uc744 \uc81c\ucd9c\ud558\uae30 \uc704\ud574 \uc5b4\ub514\ub97c \ubc29\ubb38\ud558\uc600\ub294\uac00?, context: \uc0c8\ub204\ub9ac\ub2f9 \uad00\uacc4\uc790\ub4e4\uc740 2013\ub144 8\uc6d4 2\uc77c\uc5d0 \uc11c\uc6b8\ud2b9\ubcc4\uc2dc\uccad\uc744 \ubc29\ubb38\ud558\uace0 \ubc15\uc6d0\uc21c \uc2dc\uc7a5\uc5d0\uac8c \uac74\uc124\ud604\uc7a5 \uc0ac\uace0\uc5d0 \uad00\ud55c \ucc45\uc784\uc744 \ubb3b\ub294 \ud56d\uc758 \uc11c\ud55c\uc744 \uc81c\ucd9c\ud558\ub824\uace0 \ud558\uc600\uc73c\ub098, \ubc15\uc6d0\uc21c\uacfc \uccad\uc6d0\uacbd\ucc30\uc758 \uac70\ubd80\ud589\uc704\ub85c \uc778\ud55c \ucda9\ub3cc\uc774 \uc788\uc5c8\ub2e4. \ud76c\uc0dd\uc790 \uc720\uac00\uc871\ub4e4\uc744 \ube44\ub86f\ud55c \uc0c8\ub204\ub9ac\ub2f9 \uad00\uacc4\uc790\ub4e4\uc740 \ubc15\uc6d0\uc21c\uc5d0\uac8c \ud56d\uc758 \uc11c\ud55c\uc744 \uc804\ub2ec\ud558\ub824\ub294 \ubaa9\uc801\uc774 \uc788\uc5c8\uc73c\ub098, \uc774\ub97c \uac70\ubd80\ud55c \uc11c\uc6b8\uc2dc\uccad \uce21\uc740 \uccad\uc6d0\uacbd\ucc30\uc744 \ub3d9\uc6d0\ud558\uc5ec \ubc29\ubb38\uc744 \ub9c9\uc558\uace0 \uc774 \uacfc\uc815\uc5d0\uc11c \ubab8\uc2f8\uc6c0\uc774 \ubc8c\uc5b4\uc838 \uccad\uc6d0\uacbd\ucc30\uc774 \ubd80\uc0c1\uc744 \uc785\uc5c8\ub2e4. \uc774\uc5d0 \ub300\ud574 \uc11c\uc6b8\uc2dc \uce21\uc740, \uc11c\uc6b8\uc2dc \uc9c1\uc6d0\uc774 \uc0c1\ud574\ub97c \uc785\uc740 \uac83\uc5d0 \ub300\ud574 \uc720\uac10\uc758 \ub73b\uc744 \ud45c\uba85\ud558\uc600\ub2e4. \ubc18\uba74 \uae40\uc131\ud0dc \uc758\uc6d0\uc2e4 \uce21\uc740, \uc11c\uc6b8\uc2dc\ubbfc\uc758 \uc548\uc804\uc744 \uc9c0\ud0a4\uc9c0 \ubabb\ud558\uace0 \uc720\uac00\uc871\uacfc \ubbfc\uc6d0\uc778\uc758 \ud56d\uc758 \ubc29\ubb38\uc744 \ud68c\ud53c\ud55c \ubc15\uc6d0\uc21c \uc2dc\uc7a5\uc774 \ucc45\uc784\uc744 \uc838\uc57c\ud55c\ub2e4\uace0 \uc8fc\uc7a5\ud558\uc600\ub2e4. \uc774\ub7ec\ud55c \uc77c\ub828\uc758 \uc0ac\uac74 \uc911 \uae30\ud0c0 \uc2e0\ubb38\uae30\uc0ac\ub098 \uc778\ud130\ub137 \ub4f1\uc9c0\uc5d0\uc11c \uc0c8\ub204\ub9ac\ub2f9 \uad00\uacc4\uc790\ub97c \uae40\uc131\ud0dc \uc758\uc6d0\uc2e4 \uad00\uacc4\uc790\ub85c \uc798\ubabb \ud45c\uae30\ud558\ub294 \uc624\ubcf4\uac00 \ubc1c\uc0dd\ud588\ub2e4."} {"question": "\ubc15\uadfc\ud61c\ub97c '\ubc1c\ub048\ud574'\ub77c\uace0 \ubd80\ub978 \uc870\uad6d \uad50\uc218\uac00 \ub2f9\uc2dc \uc18d\ud574\uc788\ub358 \ud559\uad50\ub294?", "paragraph": "2011\ub144 9\uc6d4 \uc815\uce58\uad8c\uc744 \uac15\ud0c0\ud55c '\uc548\ucca0\uc218 \uc5f4\ud48d'\uacfc \uad00\ub828\ud574 \uae30\uc790\ub4e4\uc774 \ubc15\uadfc\ud61c\uc5d0\uac8c '\uc548\ucca0\uc218\uc758 \uc9c0\uc9c0\uc728\uc774 \uc790\uc2e0\uc744 \ub118\uc5b4\uc130\ub294\ub370 \uc5b4\ub5bb\uac8c \uc0dd\uac01\ud558\ub290\ub0d0'\ub294 \ubc18\ubcf5\ub41c \uc9c8\ubb38\uc5d0 \ub300\ud574 \u201c\ubcd1 \uac78\ub9ac\uc168\uc5b4\uc694?\u201d\ub77c\ub294 \ubc18\uc751\uc744 \ubcf4\uc774\uba74\uc11c \ube44\ud310\uc744 \ubc1b\uae30\ub3c4 \ud588\ub2e4. \ubbfc\uc8fc\ub2f9\uc740 \uc0ac\uacfc\ub97c \uc694\uad6c\ud588\uace0 \uc774\ud6c4 \ubc15\uadfc\ud61c\ub294 \u201c\uc9c0\ub098\uac00\ub294 \uc2dd\uc73c\ub85c \ub18d\ub2f4\uc744 \ud588\ub294\ub370, \ud45c\ud604\uc774 \ubd80\uc801\uc808 \ud588\ub358 \uac83 \uac19\ub2e4\u201d\ub77c\uba70 \uc720\uac10\uc744 \ud45c\uba85\ud588\ub2e4. \ubc15\uadfc\ud61c\ub294 2004\ub144 MBC \ub77c\ub514\uc624 \ud504\ub85c\uadf8\ub7a8 '\uc190\uc11d\ud76c\uc758 \uc2dc\uc120\uc9d1\uc911'\uc5d0\uc11c\ub3c4 \uc9c4\ud589\uc790\uac00 \uacbd\uc81c \uc0b4\ub9ac\uae30\uc5d0 \ub300\ud558\uc5ec \uc9c8\ubb38\ud558\uc790 '\uc9c0\uae08 \uc800\ud558\uace0 \uc2f8\uc6c0\ud558\uc2dc\ub294 \uac70\uc608\uc694?'\ub77c\uace0 \uc5b8\uae09\ud55c \uac83\uc744 \ube44\ub86f\ud574 \uc62c\ud574 1\uc6d4 \ubcf5\uc9c0 \ubb38\uc81c\uc640 \uad00\ub828\ub41c \uae30\uc790\ub4e4\uc758 \uc9c8\ubb38\uc5d0 '\ud55c\uad6d\ub9d0 \ubaa8\ub974\uc138\uc694?'\ub77c\uace0 \ub300\ub2f5\ud55c \uac83 \ub4f1\uc774\ub2e4. \uc11c\uc6b8\ub300\ud559\uad50 \ubc95\ud559\uc804\ubb38\ub300\ud559\uc6d0 \uc870\uad6d \uad50\uc218\ub294 \"\ubc15\uadfc\ud61c \uc804 \ub300\ud45c\ub294 \ubd88\ud3b8\ud55c \uc9c8\ubb38\uacfc \ube44\ud310\uc744 \ucc38\uc9c0 \ubabb\ud55c\ub2e4. \uc601\uc560 \ubc15\uadfc\ud61c\ub2d8\uc774 \u2018\ud3c9\ubbfc\u2019\ub4e4\uacfc \uacb8\ud5c8\ud788 \uc18c\ud1b5\ud558\ub294 \ubc95\uc744 \ubc30\uc6b0\uae30 \uc804\uae4c\uc9c0 \uc0c8\ub85c\uc6b4 \ubcc4\ud638\ub97c \uc9c0\uc5b4 \uc62c\ub9ac\uace0\uc790 \ud55c\ub2e4. '\ubc1c\ub048\ud574'\"\uace0 \ub9d0\ud588\ub2e4.", "answer": "\uc11c\uc6b8\ub300\ud559\uad50", "sentence": "\uc11c\uc6b8\ub300\ud559\uad50 \ubc95\ud559\uc804\ubb38\ub300\ud559\uc6d0 \uc870\uad6d \uad50\uc218\ub294 \"\ubc15\uadfc\ud61c \uc804 \ub300\ud45c\ub294 \ubd88\ud3b8\ud55c \uc9c8\ubb38\uacfc \ube44\ud310\uc744 \ucc38\uc9c0 \ubabb\ud55c\ub2e4.", "paragraph_sentence": "2011\ub144 9\uc6d4 \uc815\uce58\uad8c\uc744 \uac15\ud0c0\ud55c '\uc548\ucca0\uc218 \uc5f4\ud48d'\uacfc \uad00\ub828\ud574 \uae30\uc790\ub4e4\uc774 \ubc15\uadfc\ud61c\uc5d0\uac8c '\uc548\ucca0\uc218\uc758 \uc9c0\uc9c0\uc728\uc774 \uc790\uc2e0\uc744 \ub118\uc5b4\uc130\ub294\ub370 \uc5b4\ub5bb\uac8c \uc0dd\uac01\ud558\ub290\ub0d0'\ub294 \ubc18\ubcf5\ub41c \uc9c8\ubb38\uc5d0 \ub300\ud574 \u201c\ubcd1 \uac78\ub9ac\uc168\uc5b4\uc694?\u201d\ub77c\ub294 \ubc18\uc751\uc744 \ubcf4\uc774\uba74\uc11c \ube44\ud310\uc744 \ubc1b\uae30\ub3c4 \ud588\ub2e4. \ubbfc\uc8fc\ub2f9\uc740 \uc0ac\uacfc\ub97c \uc694\uad6c\ud588\uace0 \uc774\ud6c4 \ubc15\uadfc\ud61c\ub294 \u201c\uc9c0\ub098\uac00\ub294 \uc2dd\uc73c\ub85c \ub18d\ub2f4\uc744 \ud588\ub294\ub370, \ud45c\ud604\uc774 \ubd80\uc801\uc808 \ud588\ub358 \uac83 \uac19\ub2e4\u201d\ub77c\uba70 \uc720\uac10\uc744 \ud45c\uba85\ud588\ub2e4. \ubc15\uadfc\ud61c\ub294 2004\ub144 MBC \ub77c\ub514\uc624 \ud504\ub85c\uadf8\ub7a8 '\uc190\uc11d\ud76c\uc758 \uc2dc\uc120\uc9d1\uc911'\uc5d0\uc11c\ub3c4 \uc9c4\ud589\uc790\uac00 \uacbd\uc81c \uc0b4\ub9ac\uae30\uc5d0 \ub300\ud558\uc5ec \uc9c8\ubb38\ud558\uc790 '\uc9c0\uae08 \uc800\ud558\uace0 \uc2f8\uc6c0\ud558\uc2dc\ub294 \uac70\uc608\uc694?'\ub77c\uace0 \uc5b8\uae09\ud55c \uac83\uc744 \ube44\ub86f\ud574 \uc62c\ud574 1\uc6d4 \ubcf5\uc9c0 \ubb38\uc81c\uc640 \uad00\ub828\ub41c \uae30\uc790\ub4e4\uc758 \uc9c8\ubb38\uc5d0 '\ud55c\uad6d\ub9d0 \ubaa8\ub974\uc138\uc694?'\ub77c\uace0 \ub300\ub2f5\ud55c \uac83 \ub4f1\uc774\ub2e4. \uc11c\uc6b8\ub300\ud559\uad50 \ubc95\ud559\uc804\ubb38\ub300\ud559\uc6d0 \uc870\uad6d \uad50\uc218\ub294 \"\ubc15\uadfc\ud61c \uc804 \ub300\ud45c\ub294 \ubd88\ud3b8\ud55c \uc9c8\ubb38\uacfc \ube44\ud310\uc744 \ucc38\uc9c0 \ubabb\ud55c\ub2e4. \uc601\uc560 \ubc15\uadfc\ud61c\ub2d8\uc774 \u2018\ud3c9\ubbfc\u2019\ub4e4\uacfc \uacb8\ud5c8\ud788 \uc18c\ud1b5\ud558\ub294 \ubc95\uc744 \ubc30\uc6b0\uae30 \uc804\uae4c\uc9c0 \uc0c8\ub85c\uc6b4 \ubcc4\ud638\ub97c \uc9c0\uc5b4 \uc62c\ub9ac\uace0\uc790 \ud55c\ub2e4. '\ubc1c\ub048\ud574'\"\uace0 \ub9d0\ud588\ub2e4.", "paragraph_answer": "2011\ub144 9\uc6d4 \uc815\uce58\uad8c\uc744 \uac15\ud0c0\ud55c '\uc548\ucca0\uc218 \uc5f4\ud48d'\uacfc \uad00\ub828\ud574 \uae30\uc790\ub4e4\uc774 \ubc15\uadfc\ud61c\uc5d0\uac8c '\uc548\ucca0\uc218\uc758 \uc9c0\uc9c0\uc728\uc774 \uc790\uc2e0\uc744 \ub118\uc5b4\uc130\ub294\ub370 \uc5b4\ub5bb\uac8c \uc0dd\uac01\ud558\ub290\ub0d0'\ub294 \ubc18\ubcf5\ub41c \uc9c8\ubb38\uc5d0 \ub300\ud574 \u201c\ubcd1 \uac78\ub9ac\uc168\uc5b4\uc694?\u201d\ub77c\ub294 \ubc18\uc751\uc744 \ubcf4\uc774\uba74\uc11c \ube44\ud310\uc744 \ubc1b\uae30\ub3c4 \ud588\ub2e4. \ubbfc\uc8fc\ub2f9\uc740 \uc0ac\uacfc\ub97c \uc694\uad6c\ud588\uace0 \uc774\ud6c4 \ubc15\uadfc\ud61c\ub294 \u201c\uc9c0\ub098\uac00\ub294 \uc2dd\uc73c\ub85c \ub18d\ub2f4\uc744 \ud588\ub294\ub370, \ud45c\ud604\uc774 \ubd80\uc801\uc808 \ud588\ub358 \uac83 \uac19\ub2e4\u201d\ub77c\uba70 \uc720\uac10\uc744 \ud45c\uba85\ud588\ub2e4. \ubc15\uadfc\ud61c\ub294 2004\ub144 MBC \ub77c\ub514\uc624 \ud504\ub85c\uadf8\ub7a8 '\uc190\uc11d\ud76c\uc758 \uc2dc\uc120\uc9d1\uc911'\uc5d0\uc11c\ub3c4 \uc9c4\ud589\uc790\uac00 \uacbd\uc81c \uc0b4\ub9ac\uae30\uc5d0 \ub300\ud558\uc5ec \uc9c8\ubb38\ud558\uc790 '\uc9c0\uae08 \uc800\ud558\uace0 \uc2f8\uc6c0\ud558\uc2dc\ub294 \uac70\uc608\uc694?'\ub77c\uace0 \uc5b8\uae09\ud55c \uac83\uc744 \ube44\ub86f\ud574 \uc62c\ud574 1\uc6d4 \ubcf5\uc9c0 \ubb38\uc81c\uc640 \uad00\ub828\ub41c \uae30\uc790\ub4e4\uc758 \uc9c8\ubb38\uc5d0 '\ud55c\uad6d\ub9d0 \ubaa8\ub974\uc138\uc694?'\ub77c\uace0 \ub300\ub2f5\ud55c \uac83 \ub4f1\uc774\ub2e4. \uc11c\uc6b8\ub300\ud559\uad50 \ubc95\ud559\uc804\ubb38\ub300\ud559\uc6d0 \uc870\uad6d \uad50\uc218\ub294 \"\ubc15\uadfc\ud61c \uc804 \ub300\ud45c\ub294 \ubd88\ud3b8\ud55c \uc9c8\ubb38\uacfc \ube44\ud310\uc744 \ucc38\uc9c0 \ubabb\ud55c\ub2e4. \uc601\uc560 \ubc15\uadfc\ud61c\ub2d8\uc774 \u2018\ud3c9\ubbfc\u2019\ub4e4\uacfc \uacb8\ud5c8\ud788 \uc18c\ud1b5\ud558\ub294 \ubc95\uc744 \ubc30\uc6b0\uae30 \uc804\uae4c\uc9c0 \uc0c8\ub85c\uc6b4 \ubcc4\ud638\ub97c \uc9c0\uc5b4 \uc62c\ub9ac\uace0\uc790 \ud55c\ub2e4. '\ubc1c\ub048\ud574'\"\uace0 \ub9d0\ud588\ub2e4.", "sentence_answer": " \uc11c\uc6b8\ub300\ud559\uad50 \ubc95\ud559\uc804\ubb38\ub300\ud559\uc6d0 \uc870\uad6d \uad50\uc218\ub294 \"\ubc15\uadfc\ud61c \uc804 \ub300\ud45c\ub294 \ubd88\ud3b8\ud55c \uc9c8\ubb38\uacfc \ube44\ud310\uc744 \ucc38\uc9c0 \ubabb\ud55c\ub2e4.", "paragraph_id": "6515822-7-2", "paragraph_question": "question: \ubc15\uadfc\ud61c\ub97c '\ubc1c\ub048\ud574'\ub77c\uace0 \ubd80\ub978 \uc870\uad6d \uad50\uc218\uac00 \ub2f9\uc2dc \uc18d\ud574\uc788\ub358 \ud559\uad50\ub294?, context: 2011\ub144 9\uc6d4 \uc815\uce58\uad8c\uc744 \uac15\ud0c0\ud55c '\uc548\ucca0\uc218 \uc5f4\ud48d'\uacfc \uad00\ub828\ud574 \uae30\uc790\ub4e4\uc774 \ubc15\uadfc\ud61c\uc5d0\uac8c '\uc548\ucca0\uc218\uc758 \uc9c0\uc9c0\uc728\uc774 \uc790\uc2e0\uc744 \ub118\uc5b4\uc130\ub294\ub370 \uc5b4\ub5bb\uac8c \uc0dd\uac01\ud558\ub290\ub0d0'\ub294 \ubc18\ubcf5\ub41c \uc9c8\ubb38\uc5d0 \ub300\ud574 \u201c\ubcd1 \uac78\ub9ac\uc168\uc5b4\uc694?\u201d\ub77c\ub294 \ubc18\uc751\uc744 \ubcf4\uc774\uba74\uc11c \ube44\ud310\uc744 \ubc1b\uae30\ub3c4 \ud588\ub2e4. \ubbfc\uc8fc\ub2f9\uc740 \uc0ac\uacfc\ub97c \uc694\uad6c\ud588\uace0 \uc774\ud6c4 \ubc15\uadfc\ud61c\ub294 \u201c\uc9c0\ub098\uac00\ub294 \uc2dd\uc73c\ub85c \ub18d\ub2f4\uc744 \ud588\ub294\ub370, \ud45c\ud604\uc774 \ubd80\uc801\uc808 \ud588\ub358 \uac83 \uac19\ub2e4\u201d\ub77c\uba70 \uc720\uac10\uc744 \ud45c\uba85\ud588\ub2e4. \ubc15\uadfc\ud61c\ub294 2004\ub144 MBC \ub77c\ub514\uc624 \ud504\ub85c\uadf8\ub7a8 '\uc190\uc11d\ud76c\uc758 \uc2dc\uc120\uc9d1\uc911'\uc5d0\uc11c\ub3c4 \uc9c4\ud589\uc790\uac00 \uacbd\uc81c \uc0b4\ub9ac\uae30\uc5d0 \ub300\ud558\uc5ec \uc9c8\ubb38\ud558\uc790 '\uc9c0\uae08 \uc800\ud558\uace0 \uc2f8\uc6c0\ud558\uc2dc\ub294 \uac70\uc608\uc694?'\ub77c\uace0 \uc5b8\uae09\ud55c \uac83\uc744 \ube44\ub86f\ud574 \uc62c\ud574 1\uc6d4 \ubcf5\uc9c0 \ubb38\uc81c\uc640 \uad00\ub828\ub41c \uae30\uc790\ub4e4\uc758 \uc9c8\ubb38\uc5d0 '\ud55c\uad6d\ub9d0 \ubaa8\ub974\uc138\uc694?'\ub77c\uace0 \ub300\ub2f5\ud55c \uac83 \ub4f1\uc774\ub2e4. \uc11c\uc6b8\ub300\ud559\uad50 \ubc95\ud559\uc804\ubb38\ub300\ud559\uc6d0 \uc870\uad6d \uad50\uc218\ub294 \"\ubc15\uadfc\ud61c \uc804 \ub300\ud45c\ub294 \ubd88\ud3b8\ud55c \uc9c8\ubb38\uacfc \ube44\ud310\uc744 \ucc38\uc9c0 \ubabb\ud55c\ub2e4. \uc601\uc560 \ubc15\uadfc\ud61c\ub2d8\uc774 \u2018\ud3c9\ubbfc\u2019\ub4e4\uacfc \uacb8\ud5c8\ud788 \uc18c\ud1b5\ud558\ub294 \ubc95\uc744 \ubc30\uc6b0\uae30 \uc804\uae4c\uc9c0 \uc0c8\ub85c\uc6b4 \ubcc4\ud638\ub97c \uc9c0\uc5b4 \uc62c\ub9ac\uace0\uc790 \ud55c\ub2e4. '\ubc1c\ub048\ud574'\"\uace0 \ub9d0\ud588\ub2e4."} {"question": "\ub2f9\uc2dc KBO\uc5d0 \uc758\ud574 \ucd5c\ub300 \uba87 \uba85\uae4c\uc9c0 \ud2b8\ub808\uc774\ub4dc\uac00 \ud5c8\uc6a9\ub41c\ub2e4\uace0 \uc54c\ub824\uc84c\ub294\uac00?", "paragraph": "\uac00\uc785\uae08 \ubb38\uc81c\ub294 \ub300\ub7b5\uc801\uc73c\ub85c \ucd1d 27\uc5b5\uc744 \ubc1b\uc740 \ub450\uc0b0\uacfc LG\uac00 7\uc5b5 \uc6d0\uc744 \uc57c\uad6c\ubc1c\uc804 \uae30\uae08\uc73c\ub85c KBO\uc5d0 \ub0a9\ubd80\ud558\uace0 KBO\ub294 SK\uc5d0\uac8c 20\uc5b5 \uc6d0\uc744 \ub3cc\ub824 \uc90c\uc73c\ub85c\uc368 \uac01 \ud300\ub2f9 20\uc5b5\uc529 \ubc1b\uac8c\ub054 \uad6c\ub450 \ud569\uc758\ub97c \ubcf4\uc558\ub2e4. KBO\ub294 12\uc6d4 30\uc77c\uc5d0 \uad00\ub828 \uc0ac\uac74\uc744 \ud569\uc758\ud558\uae30\ub85c \ud558\uc600\uace0, \ucd5c\ub300 3\uba85\uae4c\uc9c0\uc758 \ud2b8\ub808\uc774\ub4dc\ub9cc \ud5c8\uc6a9\ud55c\ub2e4\ub294 \ud2c0\uc774 \uc644\uc131\ub418\uc5c8\ub2e4. 2009\ub144 12\uc6d4 30\uc77c\uc5d0 \uc5f4\ub9b0 \uc774\uc0ac\ud68c\uc5d0\uc11c \uacb0\uad6d \ubbf8\ub0a9\uae08 \uc0ac\uac74\uc744 \ud574\uacb0\ud568\uc73c\ub85c\uc368 KBO\ub294 \ud788\uc5b4\ub85c\uc988\uc5d0\uac8c \ud2b8\ub808\uc774\ub4dc \uc2e0\uccad\uc11c \uc81c\ucd9c\uc744 \uc694\uad6c\ud558\uc600\ub2e4. \uc560\ucd08\uc758 \uad6c\ub450 \ud569\uc758\uc640\ub294 \ub2ec\ub9ac \uc0ac\uc2e4\uc0c1 \ub450\uc0b0 \ubca0\uc5b4\uc2a4\uc640 LG \ud2b8\uc708\uc2a4\ub294 22\uc5b5\uc529, SK \uc640\uc774\ubc88\uc2a4\ub294 16\uc5b5\uc529 \ubc1b\uc558\ub2e4. \uc774\uc5d0 \ud788\uc5b4\ub85c\uc988\ub294 \uc774\ud0dd\uadfc, \uc7a5\uc6d0\uc0bc, \uc774\ud604\uc2b9\uc758 \ud2b8\ub808\uc774\ub4dc \uc694\uccad\uc11c\ub97c \uc81c\ucd9c\ud558\uace0 KBO\uac00 \uadf8\ub0a0 \ubc14\ub85c \uc2b9\uc778\ud558\uc600\ub2e4.", "answer": "3\uba85", "sentence": "\ucd5c\ub300 3\uba85 \uae4c\uc9c0\uc758 \ud2b8\ub808\uc774\ub4dc\ub9cc \ud5c8\uc6a9\ud55c\ub2e4\ub294 \ud2c0\uc774 \uc644\uc131\ub418\uc5c8\ub2e4.", "paragraph_sentence": "\uac00\uc785\uae08 \ubb38\uc81c\ub294 \ub300\ub7b5\uc801\uc73c\ub85c \ucd1d 27\uc5b5\uc744 \ubc1b\uc740 \ub450\uc0b0\uacfc LG\uac00 7\uc5b5 \uc6d0\uc744 \uc57c\uad6c\ubc1c\uc804 \uae30\uae08\uc73c\ub85c KBO\uc5d0 \ub0a9\ubd80\ud558\uace0 KBO\ub294 SK\uc5d0\uac8c 20\uc5b5 \uc6d0\uc744 \ub3cc\ub824 \uc90c\uc73c\ub85c\uc368 \uac01 \ud300\ub2f9 20\uc5b5\uc529 \ubc1b\uac8c\ub054 \uad6c\ub450 \ud569\uc758\ub97c \ubcf4\uc558\ub2e4. KBO\ub294 12\uc6d4 30\uc77c\uc5d0 \uad00\ub828 \uc0ac\uac74\uc744 \ud569\uc758\ud558\uae30\ub85c \ud558\uc600\uace0, \ucd5c\ub300 3\uba85 \uae4c\uc9c0\uc758 \ud2b8\ub808\uc774\ub4dc\ub9cc \ud5c8\uc6a9\ud55c\ub2e4\ub294 \ud2c0\uc774 \uc644\uc131\ub418\uc5c8\ub2e4. 2009\ub144 12\uc6d4 30\uc77c\uc5d0 \uc5f4\ub9b0 \uc774\uc0ac\ud68c\uc5d0\uc11c \uacb0\uad6d \ubbf8\ub0a9\uae08 \uc0ac\uac74\uc744 \ud574\uacb0\ud568\uc73c\ub85c\uc368 KBO\ub294 \ud788\uc5b4\ub85c\uc988\uc5d0\uac8c \ud2b8\ub808\uc774\ub4dc \uc2e0\uccad\uc11c \uc81c\ucd9c\uc744 \uc694\uad6c\ud558\uc600\ub2e4. \uc560\ucd08\uc758 \uad6c\ub450 \ud569\uc758\uc640\ub294 \ub2ec\ub9ac \uc0ac\uc2e4\uc0c1 \ub450\uc0b0 \ubca0\uc5b4\uc2a4\uc640 LG \ud2b8\uc708\uc2a4\ub294 22\uc5b5\uc529, SK \uc640\uc774\ubc88\uc2a4\ub294 16\uc5b5\uc529 \ubc1b\uc558\ub2e4. \uc774\uc5d0 \ud788\uc5b4\ub85c\uc988\ub294 \uc774\ud0dd\uadfc, \uc7a5\uc6d0\uc0bc, \uc774\ud604\uc2b9\uc758 \ud2b8\ub808\uc774\ub4dc \uc694\uccad\uc11c\ub97c \uc81c\ucd9c\ud558\uace0 KBO\uac00 \uadf8\ub0a0 \ubc14\ub85c \uc2b9\uc778\ud558\uc600\ub2e4.", "paragraph_answer": "\uac00\uc785\uae08 \ubb38\uc81c\ub294 \ub300\ub7b5\uc801\uc73c\ub85c \ucd1d 27\uc5b5\uc744 \ubc1b\uc740 \ub450\uc0b0\uacfc LG\uac00 7\uc5b5 \uc6d0\uc744 \uc57c\uad6c\ubc1c\uc804 \uae30\uae08\uc73c\ub85c KBO\uc5d0 \ub0a9\ubd80\ud558\uace0 KBO\ub294 SK\uc5d0\uac8c 20\uc5b5 \uc6d0\uc744 \ub3cc\ub824 \uc90c\uc73c\ub85c\uc368 \uac01 \ud300\ub2f9 20\uc5b5\uc529 \ubc1b\uac8c\ub054 \uad6c\ub450 \ud569\uc758\ub97c \ubcf4\uc558\ub2e4. KBO\ub294 12\uc6d4 30\uc77c\uc5d0 \uad00\ub828 \uc0ac\uac74\uc744 \ud569\uc758\ud558\uae30\ub85c \ud558\uc600\uace0, \ucd5c\ub300 3\uba85 \uae4c\uc9c0\uc758 \ud2b8\ub808\uc774\ub4dc\ub9cc \ud5c8\uc6a9\ud55c\ub2e4\ub294 \ud2c0\uc774 \uc644\uc131\ub418\uc5c8\ub2e4. 2009\ub144 12\uc6d4 30\uc77c\uc5d0 \uc5f4\ub9b0 \uc774\uc0ac\ud68c\uc5d0\uc11c \uacb0\uad6d \ubbf8\ub0a9\uae08 \uc0ac\uac74\uc744 \ud574\uacb0\ud568\uc73c\ub85c\uc368 KBO\ub294 \ud788\uc5b4\ub85c\uc988\uc5d0\uac8c \ud2b8\ub808\uc774\ub4dc \uc2e0\uccad\uc11c \uc81c\ucd9c\uc744 \uc694\uad6c\ud558\uc600\ub2e4. \uc560\ucd08\uc758 \uad6c\ub450 \ud569\uc758\uc640\ub294 \ub2ec\ub9ac \uc0ac\uc2e4\uc0c1 \ub450\uc0b0 \ubca0\uc5b4\uc2a4\uc640 LG \ud2b8\uc708\uc2a4\ub294 22\uc5b5\uc529, SK \uc640\uc774\ubc88\uc2a4\ub294 16\uc5b5\uc529 \ubc1b\uc558\ub2e4. \uc774\uc5d0 \ud788\uc5b4\ub85c\uc988\ub294 \uc774\ud0dd\uadfc, \uc7a5\uc6d0\uc0bc, \uc774\ud604\uc2b9\uc758 \ud2b8\ub808\uc774\ub4dc \uc694\uccad\uc11c\ub97c \uc81c\ucd9c\ud558\uace0 KBO\uac00 \uadf8\ub0a0 \ubc14\ub85c \uc2b9\uc778\ud558\uc600\ub2e4.", "sentence_answer": "\ucd5c\ub300 3\uba85 \uae4c\uc9c0\uc758 \ud2b8\ub808\uc774\ub4dc\ub9cc \ud5c8\uc6a9\ud55c\ub2e4\ub294 \ud2c0\uc774 \uc644\uc131\ub418\uc5c8\ub2e4.", "paragraph_id": "6533929-7-1", "paragraph_question": "question: \ub2f9\uc2dc KBO\uc5d0 \uc758\ud574 \ucd5c\ub300 \uba87 \uba85\uae4c\uc9c0 \ud2b8\ub808\uc774\ub4dc\uac00 \ud5c8\uc6a9\ub41c\ub2e4\uace0 \uc54c\ub824\uc84c\ub294\uac00?, context: \uac00\uc785\uae08 \ubb38\uc81c\ub294 \ub300\ub7b5\uc801\uc73c\ub85c \ucd1d 27\uc5b5\uc744 \ubc1b\uc740 \ub450\uc0b0\uacfc LG\uac00 7\uc5b5 \uc6d0\uc744 \uc57c\uad6c\ubc1c\uc804 \uae30\uae08\uc73c\ub85c KBO\uc5d0 \ub0a9\ubd80\ud558\uace0 KBO\ub294 SK\uc5d0\uac8c 20\uc5b5 \uc6d0\uc744 \ub3cc\ub824 \uc90c\uc73c\ub85c\uc368 \uac01 \ud300\ub2f9 20\uc5b5\uc529 \ubc1b\uac8c\ub054 \uad6c\ub450 \ud569\uc758\ub97c \ubcf4\uc558\ub2e4. KBO\ub294 12\uc6d4 30\uc77c\uc5d0 \uad00\ub828 \uc0ac\uac74\uc744 \ud569\uc758\ud558\uae30\ub85c \ud558\uc600\uace0, \ucd5c\ub300 3\uba85\uae4c\uc9c0\uc758 \ud2b8\ub808\uc774\ub4dc\ub9cc \ud5c8\uc6a9\ud55c\ub2e4\ub294 \ud2c0\uc774 \uc644\uc131\ub418\uc5c8\ub2e4. 2009\ub144 12\uc6d4 30\uc77c\uc5d0 \uc5f4\ub9b0 \uc774\uc0ac\ud68c\uc5d0\uc11c \uacb0\uad6d \ubbf8\ub0a9\uae08 \uc0ac\uac74\uc744 \ud574\uacb0\ud568\uc73c\ub85c\uc368 KBO\ub294 \ud788\uc5b4\ub85c\uc988\uc5d0\uac8c \ud2b8\ub808\uc774\ub4dc \uc2e0\uccad\uc11c \uc81c\ucd9c\uc744 \uc694\uad6c\ud558\uc600\ub2e4. \uc560\ucd08\uc758 \uad6c\ub450 \ud569\uc758\uc640\ub294 \ub2ec\ub9ac \uc0ac\uc2e4\uc0c1 \ub450\uc0b0 \ubca0\uc5b4\uc2a4\uc640 LG \ud2b8\uc708\uc2a4\ub294 22\uc5b5\uc529, SK \uc640\uc774\ubc88\uc2a4\ub294 16\uc5b5\uc529 \ubc1b\uc558\ub2e4. \uc774\uc5d0 \ud788\uc5b4\ub85c\uc988\ub294 \uc774\ud0dd\uadfc, \uc7a5\uc6d0\uc0bc, \uc774\ud604\uc2b9\uc758 \ud2b8\ub808\uc774\ub4dc \uc694\uccad\uc11c\ub97c \uc81c\ucd9c\ud558\uace0 KBO\uac00 \uadf8\ub0a0 \ubc14\ub85c \uc2b9\uc778\ud558\uc600\ub2e4."} {"question": "\uc11c\uc608\uc758 \uadfc\ubcf8\uc815\uc2e0\uc740?", "paragraph": "\uc11c\uc608\uc758 \uc2ec\uc624\ud55c \ub73b\uc740 \uae30\ubc95\uc758 \uc5f0\uc2b5\ub9cc\uc73c\ub85c \ub418\ub294 \uac83\uc774 \uc544\ub2c8\uace0 \uae30\ubc95\uacfc \ubcd1\ud589\ud574\uc11c \uc815\uc2e0\uc218\ub828\uc744 \uc911\uc694\uc2dc\ud55c\ub2e4. \uc911\uad6d\uc5d0\uc11c \uc11c\ub294 \uc721\uc608(\u516d\u85dd), \uc989 \uc608(\u79ae)\u00b7\uc545(\u6a02)\u00b7\uc0ac(\u5c04)\u00b7\uc5b4(\u5fa1)\u00b7\uc11c(\u66f8)\u00b7\uc218(\u6578) \uc911\uc758 \uc77c\ubd80\ubd84\uc73c\ub85c \uc778\uac04\uc758 \uc120\ud589 \uc218\ub828\uc774\ub2e4. \uace0\uc0c1\ud55c \uc0ac\uad70\uc790 \uc218\uc5c5\uc758 \ub3c4(\u9053)\ub85c\uc11c \ub9ce\uc740 \uc704\uc778\ub4e4\uc774 \uc0dd\uc560\ub97c \ubc14\uccd0\uc11c \uc11c\ub85c \uc774\uc5b4\uc11c \ubc1c\uc804\uc2dc\ucf1c \uc654\ub2e4. \uc11c\uc608\ub294 \uc2e4\uc6a9\uc801\uc778 \uc694\uad6c\uc640 \uc11c\uc608\uc801 \ub9cc\uc871\uc5d0\ub9cc \uadf8\uce58\ub294 \uac83\uc774 \uc544\ub2c8\ub77c \ucc38\ub2e4\uc6b4 \uc778\uac04\uc758 \uc601\uc6d0\uc131\uc744 \ucd94\uad6c\ud558\ub294 \ub3c4(\u9053)\ub85c\uc11c \ucca0\ud559\uc801\uc73c\ub85c \uc885\uad50\uc801\uc73c\ub85c \uc5f4\ub82c\ud788 \uc694\uad6c\ub418\uc5b4 \uc788\ub2e4. \uadf8\ub798\uc11c \uc11c\uc608\uc758 \uadfc\ubcf8\uc815\uc2e0\uc774 \ub3d9\uc591 \uc77c\ubc18\uc758 \uc608\ub3c4(\u85dd\u9053)\uc5d0 \ud1b5\ud558\ub294 \uc790\uc5f0\uad00\uc744 \uae30\uc870\ub85c \ud55c \uac83\uc774\ub2e4. \uc11c\uc758 \ud45c\ud604 \ub0b4\uc6a9\uc740 \uadf8 \uc11c\uc758 \uc18c\uc7ac\u00b7\uc5b4\uad6c(\u8a9e\u53e5) \uc2dc\ubb38(\u8a69\u6587)\u00b7\ubb38\ud559\uc801\uc778 \uc694\uc18c\ub97c \uc758\ubbf8\ud558\uace0 \uc788\uc73c\ub098 \ub3c5\ub9bd\ub41c \uc2dc\uac01\uc131 \uc608\uc220\ub85c \ud5a5\uc0c1\ub418\ub294 \ud55c, \uc11c\uc758 \ub0b4\uc6a9\uc740 \uc18c\uc7ac\ub85c\uc11c \ubb38\uc790\ubcf4\ub2e4 \uadf8 \uc791\ud488\uc5d0 \ud3ec\ud568\ub41c \uc11c\ubbf8(\u66f8\u7f8e)\uc758 \ubb38\uc81c\uc810\uc774 \uc0dd\uae34\ub2e4. \uc989 \ud558\ub098\uc758 \uc644\uc131\ub41c \uc791\ud488\uc5d0\ub294 \uc804\uccb4\ub97c \uad6c\uc131\ud558\ub294 \ubb38\uc790\uc758 \ub300\uc18c(\ud06c\uace0 \uc791\uc74c)\u00b7\ud3ec\uce58(\u5e03\u7f6e, \ub113\uac8c \ub298\uc5b4\ub193\uc74c), \uba39\uc758 \uc724\uac08(\u6f64\u6e34, \uc724\ud0dd\ud558\uace0 \ub9c8\ub984), \ub099\uad00(\u843d\u6b3e)\uc758 \uc704\uce58 \uc989 \uc7a5\ubc95(\u7ae0\u6cd5, \u7d93\u71df\u4f4d\u7f6e, \ud654\uba74\uc744 \uc0b4\ub9ac\uae30 \uc704\ud55c \ubc30\uce58\ubc95)\uc774 \uc911\uc694\ud558\ub2e4.", "answer": "\uc790\uc5f0\uad00", "sentence": "\uadf8\ub798\uc11c \uc11c\uc608\uc758 \uadfc\ubcf8\uc815\uc2e0\uc774 \ub3d9\uc591 \uc77c\ubc18\uc758 \uc608\ub3c4(\u85dd\u9053)\uc5d0 \ud1b5\ud558\ub294 \uc790\uc5f0\uad00 \uc744 \uae30\uc870\ub85c \ud55c \uac83\uc774\ub2e4.", "paragraph_sentence": "\uc11c\uc608\uc758 \uc2ec\uc624\ud55c \ub73b\uc740 \uae30\ubc95\uc758 \uc5f0\uc2b5\ub9cc\uc73c\ub85c \ub418\ub294 \uac83\uc774 \uc544\ub2c8\uace0 \uae30\ubc95\uacfc \ubcd1\ud589\ud574\uc11c \uc815\uc2e0\uc218\ub828\uc744 \uc911\uc694\uc2dc\ud55c\ub2e4. \uc911\uad6d\uc5d0\uc11c \uc11c\ub294 \uc721\uc608(\u516d\u85dd), \uc989 \uc608(\u79ae)\u00b7\uc545(\u6a02)\u00b7\uc0ac(\u5c04)\u00b7\uc5b4(\u5fa1)\u00b7\uc11c(\u66f8)\u00b7\uc218(\u6578) \uc911\uc758 \uc77c\ubd80\ubd84\uc73c\ub85c \uc778\uac04\uc758 \uc120\ud589 \uc218\ub828\uc774\ub2e4. \uace0\uc0c1\ud55c \uc0ac\uad70\uc790 \uc218\uc5c5\uc758 \ub3c4(\u9053)\ub85c\uc11c \ub9ce\uc740 \uc704\uc778\ub4e4\uc774 \uc0dd\uc560\ub97c \ubc14\uccd0\uc11c \uc11c\ub85c \uc774\uc5b4\uc11c \ubc1c\uc804\uc2dc\ucf1c \uc654\ub2e4. \uc11c\uc608\ub294 \uc2e4\uc6a9\uc801\uc778 \uc694\uad6c\uc640 \uc11c\uc608\uc801 \ub9cc\uc871\uc5d0\ub9cc \uadf8\uce58\ub294 \uac83\uc774 \uc544\ub2c8\ub77c \ucc38\ub2e4\uc6b4 \uc778\uac04\uc758 \uc601\uc6d0\uc131\uc744 \ucd94\uad6c\ud558\ub294 \ub3c4(\u9053)\ub85c\uc11c \ucca0\ud559\uc801\uc73c\ub85c \uc885\uad50\uc801\uc73c\ub85c \uc5f4\ub82c\ud788 \uc694\uad6c\ub418\uc5b4 \uc788\ub2e4. \uadf8\ub798\uc11c \uc11c\uc608\uc758 \uadfc\ubcf8\uc815\uc2e0\uc774 \ub3d9\uc591 \uc77c\ubc18\uc758 \uc608\ub3c4(\u85dd\u9053)\uc5d0 \ud1b5\ud558\ub294 \uc790\uc5f0\uad00 \uc744 \uae30\uc870\ub85c \ud55c \uac83\uc774\ub2e4. \uc11c\uc758 \ud45c\ud604 \ub0b4\uc6a9\uc740 \uadf8 \uc11c\uc758 \uc18c\uc7ac\u00b7\uc5b4\uad6c(\u8a9e\u53e5) \uc2dc\ubb38(\u8a69\u6587)\u00b7\ubb38\ud559\uc801\uc778 \uc694\uc18c\ub97c \uc758\ubbf8\ud558\uace0 \uc788\uc73c\ub098 \ub3c5\ub9bd\ub41c \uc2dc\uac01\uc131 \uc608\uc220\ub85c \ud5a5\uc0c1\ub418\ub294 \ud55c, \uc11c\uc758 \ub0b4\uc6a9\uc740 \uc18c\uc7ac\ub85c\uc11c \ubb38\uc790\ubcf4\ub2e4 \uadf8 \uc791\ud488\uc5d0 \ud3ec\ud568\ub41c \uc11c\ubbf8(\u66f8\u7f8e)\uc758 \ubb38\uc81c\uc810\uc774 \uc0dd\uae34\ub2e4. \uc989 \ud558\ub098\uc758 \uc644\uc131\ub41c \uc791\ud488\uc5d0\ub294 \uc804\uccb4\ub97c \uad6c\uc131\ud558\ub294 \ubb38\uc790\uc758 \ub300\uc18c(\ud06c\uace0 \uc791\uc74c)\u00b7\ud3ec\uce58(\u5e03\u7f6e, \ub113\uac8c \ub298\uc5b4\ub193\uc74c), \uba39\uc758 \uc724\uac08(\u6f64\u6e34, \uc724\ud0dd\ud558\uace0 \ub9c8\ub984), \ub099\uad00(\u843d\u6b3e)\uc758 \uc704\uce58 \uc989 \uc7a5\ubc95(\u7ae0\u6cd5, \u7d93\u71df\u4f4d\u7f6e, \ud654\uba74\uc744 \uc0b4\ub9ac\uae30 \uc704\ud55c \ubc30\uce58\ubc95)\uc774 \uc911\uc694\ud558\ub2e4.", "paragraph_answer": "\uc11c\uc608\uc758 \uc2ec\uc624\ud55c \ub73b\uc740 \uae30\ubc95\uc758 \uc5f0\uc2b5\ub9cc\uc73c\ub85c \ub418\ub294 \uac83\uc774 \uc544\ub2c8\uace0 \uae30\ubc95\uacfc \ubcd1\ud589\ud574\uc11c \uc815\uc2e0\uc218\ub828\uc744 \uc911\uc694\uc2dc\ud55c\ub2e4. \uc911\uad6d\uc5d0\uc11c \uc11c\ub294 \uc721\uc608(\u516d\u85dd), \uc989 \uc608(\u79ae)\u00b7\uc545(\u6a02)\u00b7\uc0ac(\u5c04)\u00b7\uc5b4(\u5fa1)\u00b7\uc11c(\u66f8)\u00b7\uc218(\u6578) \uc911\uc758 \uc77c\ubd80\ubd84\uc73c\ub85c \uc778\uac04\uc758 \uc120\ud589 \uc218\ub828\uc774\ub2e4. \uace0\uc0c1\ud55c \uc0ac\uad70\uc790 \uc218\uc5c5\uc758 \ub3c4(\u9053)\ub85c\uc11c \ub9ce\uc740 \uc704\uc778\ub4e4\uc774 \uc0dd\uc560\ub97c \ubc14\uccd0\uc11c \uc11c\ub85c \uc774\uc5b4\uc11c \ubc1c\uc804\uc2dc\ucf1c \uc654\ub2e4. \uc11c\uc608\ub294 \uc2e4\uc6a9\uc801\uc778 \uc694\uad6c\uc640 \uc11c\uc608\uc801 \ub9cc\uc871\uc5d0\ub9cc \uadf8\uce58\ub294 \uac83\uc774 \uc544\ub2c8\ub77c \ucc38\ub2e4\uc6b4 \uc778\uac04\uc758 \uc601\uc6d0\uc131\uc744 \ucd94\uad6c\ud558\ub294 \ub3c4(\u9053)\ub85c\uc11c \ucca0\ud559\uc801\uc73c\ub85c \uc885\uad50\uc801\uc73c\ub85c \uc5f4\ub82c\ud788 \uc694\uad6c\ub418\uc5b4 \uc788\ub2e4. \uadf8\ub798\uc11c \uc11c\uc608\uc758 \uadfc\ubcf8\uc815\uc2e0\uc774 \ub3d9\uc591 \uc77c\ubc18\uc758 \uc608\ub3c4(\u85dd\u9053)\uc5d0 \ud1b5\ud558\ub294 \uc790\uc5f0\uad00 \uc744 \uae30\uc870\ub85c \ud55c \uac83\uc774\ub2e4. \uc11c\uc758 \ud45c\ud604 \ub0b4\uc6a9\uc740 \uadf8 \uc11c\uc758 \uc18c\uc7ac\u00b7\uc5b4\uad6c(\u8a9e\u53e5) \uc2dc\ubb38(\u8a69\u6587)\u00b7\ubb38\ud559\uc801\uc778 \uc694\uc18c\ub97c \uc758\ubbf8\ud558\uace0 \uc788\uc73c\ub098 \ub3c5\ub9bd\ub41c \uc2dc\uac01\uc131 \uc608\uc220\ub85c \ud5a5\uc0c1\ub418\ub294 \ud55c, \uc11c\uc758 \ub0b4\uc6a9\uc740 \uc18c\uc7ac\ub85c\uc11c \ubb38\uc790\ubcf4\ub2e4 \uadf8 \uc791\ud488\uc5d0 \ud3ec\ud568\ub41c \uc11c\ubbf8(\u66f8\u7f8e)\uc758 \ubb38\uc81c\uc810\uc774 \uc0dd\uae34\ub2e4. \uc989 \ud558\ub098\uc758 \uc644\uc131\ub41c \uc791\ud488\uc5d0\ub294 \uc804\uccb4\ub97c \uad6c\uc131\ud558\ub294 \ubb38\uc790\uc758 \ub300\uc18c(\ud06c\uace0 \uc791\uc74c)\u00b7\ud3ec\uce58(\u5e03\u7f6e, \ub113\uac8c \ub298\uc5b4\ub193\uc74c), \uba39\uc758 \uc724\uac08(\u6f64\u6e34, \uc724\ud0dd\ud558\uace0 \ub9c8\ub984), \ub099\uad00(\u843d\u6b3e)\uc758 \uc704\uce58 \uc989 \uc7a5\ubc95(\u7ae0\u6cd5, \u7d93\u71df\u4f4d\u7f6e, \ud654\uba74\uc744 \uc0b4\ub9ac\uae30 \uc704\ud55c \ubc30\uce58\ubc95)\uc774 \uc911\uc694\ud558\ub2e4.", "sentence_answer": "\uadf8\ub798\uc11c \uc11c\uc608\uc758 \uadfc\ubcf8\uc815\uc2e0\uc774 \ub3d9\uc591 \uc77c\ubc18\uc758 \uc608\ub3c4(\u85dd\u9053)\uc5d0 \ud1b5\ud558\ub294 \uc790\uc5f0\uad00 \uc744 \uae30\uc870\ub85c \ud55c \uac83\uc774\ub2e4.", "paragraph_id": "6463177-1-1", "paragraph_question": "question: \uc11c\uc608\uc758 \uadfc\ubcf8\uc815\uc2e0\uc740?, context: \uc11c\uc608\uc758 \uc2ec\uc624\ud55c \ub73b\uc740 \uae30\ubc95\uc758 \uc5f0\uc2b5\ub9cc\uc73c\ub85c \ub418\ub294 \uac83\uc774 \uc544\ub2c8\uace0 \uae30\ubc95\uacfc \ubcd1\ud589\ud574\uc11c \uc815\uc2e0\uc218\ub828\uc744 \uc911\uc694\uc2dc\ud55c\ub2e4. \uc911\uad6d\uc5d0\uc11c \uc11c\ub294 \uc721\uc608(\u516d\u85dd), \uc989 \uc608(\u79ae)\u00b7\uc545(\u6a02)\u00b7\uc0ac(\u5c04)\u00b7\uc5b4(\u5fa1)\u00b7\uc11c(\u66f8)\u00b7\uc218(\u6578) \uc911\uc758 \uc77c\ubd80\ubd84\uc73c\ub85c \uc778\uac04\uc758 \uc120\ud589 \uc218\ub828\uc774\ub2e4. \uace0\uc0c1\ud55c \uc0ac\uad70\uc790 \uc218\uc5c5\uc758 \ub3c4(\u9053)\ub85c\uc11c \ub9ce\uc740 \uc704\uc778\ub4e4\uc774 \uc0dd\uc560\ub97c \ubc14\uccd0\uc11c \uc11c\ub85c \uc774\uc5b4\uc11c \ubc1c\uc804\uc2dc\ucf1c \uc654\ub2e4. \uc11c\uc608\ub294 \uc2e4\uc6a9\uc801\uc778 \uc694\uad6c\uc640 \uc11c\uc608\uc801 \ub9cc\uc871\uc5d0\ub9cc \uadf8\uce58\ub294 \uac83\uc774 \uc544\ub2c8\ub77c \ucc38\ub2e4\uc6b4 \uc778\uac04\uc758 \uc601\uc6d0\uc131\uc744 \ucd94\uad6c\ud558\ub294 \ub3c4(\u9053)\ub85c\uc11c \ucca0\ud559\uc801\uc73c\ub85c \uc885\uad50\uc801\uc73c\ub85c \uc5f4\ub82c\ud788 \uc694\uad6c\ub418\uc5b4 \uc788\ub2e4. \uadf8\ub798\uc11c \uc11c\uc608\uc758 \uadfc\ubcf8\uc815\uc2e0\uc774 \ub3d9\uc591 \uc77c\ubc18\uc758 \uc608\ub3c4(\u85dd\u9053)\uc5d0 \ud1b5\ud558\ub294 \uc790\uc5f0\uad00\uc744 \uae30\uc870\ub85c \ud55c \uac83\uc774\ub2e4. \uc11c\uc758 \ud45c\ud604 \ub0b4\uc6a9\uc740 \uadf8 \uc11c\uc758 \uc18c\uc7ac\u00b7\uc5b4\uad6c(\u8a9e\u53e5) \uc2dc\ubb38(\u8a69\u6587)\u00b7\ubb38\ud559\uc801\uc778 \uc694\uc18c\ub97c \uc758\ubbf8\ud558\uace0 \uc788\uc73c\ub098 \ub3c5\ub9bd\ub41c \uc2dc\uac01\uc131 \uc608\uc220\ub85c \ud5a5\uc0c1\ub418\ub294 \ud55c, \uc11c\uc758 \ub0b4\uc6a9\uc740 \uc18c\uc7ac\ub85c\uc11c \ubb38\uc790\ubcf4\ub2e4 \uadf8 \uc791\ud488\uc5d0 \ud3ec\ud568\ub41c \uc11c\ubbf8(\u66f8\u7f8e)\uc758 \ubb38\uc81c\uc810\uc774 \uc0dd\uae34\ub2e4. \uc989 \ud558\ub098\uc758 \uc644\uc131\ub41c \uc791\ud488\uc5d0\ub294 \uc804\uccb4\ub97c \uad6c\uc131\ud558\ub294 \ubb38\uc790\uc758 \ub300\uc18c(\ud06c\uace0 \uc791\uc74c)\u00b7\ud3ec\uce58(\u5e03\u7f6e, \ub113\uac8c \ub298\uc5b4\ub193\uc74c), \uba39\uc758 \uc724\uac08(\u6f64\u6e34, \uc724\ud0dd\ud558\uace0 \ub9c8\ub984), \ub099\uad00(\u843d\u6b3e)\uc758 \uc704\uce58 \uc989 \uc7a5\ubc95(\u7ae0\u6cd5, \u7d93\u71df\u4f4d\u7f6e, \ud654\uba74\uc744 \uc0b4\ub9ac\uae30 \uc704\ud55c \ubc30\uce58\ubc95)\uc774 \uc911\uc694\ud558\ub2e4."} {"question": "\uc2e0\uc0ac\uc784\ub2f9\uc740 \uc774\uc774\uac00 \uba87 \uc0b4 \ub54c \uc8fd\uc5c8\ub294\uac00?", "paragraph": "\uc544\ud649 \ucc28\ub840\uc758 \uacfc\uac70\uc5d0 \uc7a5\uc6d0\uae09\uc81c\ud558\uc5ec \uad6c\ub3c4\uc7a5\uc6d0\uacf5(\u4e5d\u5ea6\u58ef\u5143\u516c)\uc774\ub77c\ub294 \ubcc4\uce6d\uc744 \uc5bb\uc5c8\ub2e4. 16\uc138 \ub54c \uc5b4\uba38\ub2c8 \uc2e0\uc0ac\uc784\ub2f9(\u7533\u5e2b\u4efb\u5802)\uc774 \uc8fd\uc790 3\ub144\uac04 \uc5ec\ubb18\uc0b4\uc774\ub97c \ud55c \ud6c4, \uc544\ubc84\uc9c0\uac00 \uacc4\ubaa8 \uad8c\uc528\ub97c \ub4e4\uc778 \ub4a4 \uae08\uac15\uc0b0\uc5d0 \ub4e4\uc5b4\uac00 \uc2b9\ub824\uac00 \ub418\uc5c8\ub294\ub370, \uc774 \ub54c\ubb38\uc5d0 \ud6d7\ub0a0 \uadf8\uac00 \uc8fd\uc740 \ud6c4\uc5d0\uae4c\uc9c0\ub3c4 '\uba38\ub9ac\ub97c \uae4e\uace0 \uc2b9\ub824\uac00 \ub418\ub824\ub2e4\uac00 \ud658\uc18d\ud55c \uc0ac\ub78c'\uc774\ub77c\uace0 \ub3d9\uc778\uacfc \ub0a8\uc778\uc774 \uacf5\uaca9\ud558\ub294 \ube4c\ubbf8\uac00 \ub418\uc5c8\ub2e4. \uc774\uc900\uacbd\uc774 \uc8fd\uae30 \uc9c1\uc804 \ubd95\ub2f9\uc758 \ud3d0\uc5d0 \uad00\ud55c \uc720\ucc28\ub97c \uc62c\ub9ac\uc790 '\uc8fd\uc74c\uc5d0 \uc774\ub974\ub7ec \ub9d0\uc774 \uc545\ud558\ub2e4'\uace0 \uacf5\uaca9\ud558\uc600\uc73c\uba70 \uc774\ud6c4 \uc774\uc900\uacbd\uc758 \ucc98\ubc8c\uae4c\uc9c0 \uac00\uae30\ub3c4 \ud588\ub2e4. \uadf8\ub7ec\ub098 \ud6c4\uc77c \ub2f9\uc7c1\uc774 \ud604\uc2e4\ud654\ud558\uc790 \uc2a4\uc2a4\ub85c \ud06c\uac8c \ub258\uc6b0\uce58\uace0 \ub3d9\uc778, \uc11c\uc778 \uc0ac\uc774\uc758 \ub2f9\uc7c1 \uc870\uc815\uc744 \ud3c9\uc0dd \uc815\uce58 \uc774\ub150\uc73c\ub85c \uc0bc\uc558\ub2e4. \uacf5\ub0a9(\u8ca2\u7d0d)\uc758 \ud3d0\ub2e8 \uc2dc\uc815\ucc45\uc778 \ub300\uacf5\uc218\ubbf8\ubc95(\u4ee3\u8ca2\u6536\u7c73\u6cd5) \uc2e4\uc2dc\ub97c \uc8fc\uc7a5\ud558\uace0, \ubcd1\uc870\ud310\uc11c\ub85c\uc11c \uc5ec\uc9c4\uc871 \uc774\ud0d5\uac1c\uc758 \uce68\uc785\uc744 \uaca9\ud1f4\ud55c \ud6c4, 10\ub9cc \uc591\ubcd1\uc124\uc744 \uc8fc\uc7a5\ud574 \uc784\uc9c4\uc65c\ub780\uc744 \uc608\uc5b8\ud588\ub2e4\ub294 \uba85\uc131\uc744 \uc5bb\uc5c8\ub2e4. \ubd95\ub2f9\uc744 \uc870\uc815\ud558\uc9c0 \ubabb\ud55c \ud55c\uc744 \ub0a8\uae34 \ucc44 \uc8fd\uc5c8\uc73c\uba70, \uc0ac\ud6c4 \uc758\uc815\ubd80\uc601\uc758\uc815\uc5d0 \ucd94\uc99d\ub418\uc5c8\ub2e4. \uc131\ud63c, \uc1a1\uc775\ud544, \uae40\uc7a5\uc0dd \ub4f1\uacfc \ud568\uaed8 \uae30\ud638 \uc9c0\uc5ed\uc774 \uae30\ubc18\uc778 \uc11c\uc778(\u897f\u4eba)\uc758 \uc885\uc8fc\ub85c \ucd94\uc559\ub41c\ub2e4. \uadf8\ub97c \ubb38\ubb18\uc5d0 \uc885\uc0ac\ud558\ub294 \ubb38\uc81c\ub97c \ub193\uace0 \uc778\uc870\ubc18\uc815 \uc774\ud6c4 50\ub144\uac04 \ub17c\uc7c1\uc758 \ub300\uc0c1\uc774 \ub418\ub2e4\uac00 \uc219\uc885 \ub54c \uacbd\uc2e0\ud658\uad6d\uc73c\ub85c \uc11c\uc778\ub4e4\uc774 \uc9d1\uad8c\ud55c \ud6c4 \ubb38\ubb18\uc5d0 \uc885\uc0ac\ub418\uc5c8\ub2e4.", "answer": "16\uc138", "sentence": "16\uc138 \ub54c \uc5b4\uba38\ub2c8 \uc2e0\uc0ac\uc784\ub2f9(\u7533\u5e2b\u4efb\u5802)\uc774 \uc8fd\uc790 3\ub144\uac04 \uc5ec\ubb18\uc0b4\uc774\ub97c \ud55c \ud6c4, \uc544\ubc84\uc9c0\uac00 \uacc4\ubaa8 \uad8c\uc528\ub97c \ub4e4\uc778 \ub4a4 \uae08\uac15\uc0b0\uc5d0 \ub4e4\uc5b4\uac00 \uc2b9\ub824\uac00 \ub418\uc5c8\ub294\ub370, \uc774 \ub54c\ubb38\uc5d0 \ud6d7\ub0a0 \uadf8\uac00 \uc8fd\uc740 \ud6c4\uc5d0\uae4c\uc9c0\ub3c4 '\uba38\ub9ac\ub97c \uae4e\uace0 \uc2b9\ub824\uac00 \ub418\ub824\ub2e4\uac00 \ud658\uc18d\ud55c \uc0ac\ub78c'\uc774\ub77c\uace0 \ub3d9\uc778\uacfc \ub0a8\uc778\uc774 \uacf5\uaca9\ud558\ub294 \ube4c\ubbf8\uac00 \ub418\uc5c8\ub2e4.", "paragraph_sentence": "\uc544\ud649 \ucc28\ub840\uc758 \uacfc\uac70\uc5d0 \uc7a5\uc6d0\uae09\uc81c\ud558\uc5ec \uad6c\ub3c4\uc7a5\uc6d0\uacf5(\u4e5d\u5ea6\u58ef\u5143\u516c)\uc774\ub77c\ub294 \ubcc4\uce6d\uc744 \uc5bb\uc5c8\ub2e4. 16\uc138 \ub54c \uc5b4\uba38\ub2c8 \uc2e0\uc0ac\uc784\ub2f9(\u7533\u5e2b\u4efb\u5802)\uc774 \uc8fd\uc790 3\ub144\uac04 \uc5ec\ubb18\uc0b4\uc774\ub97c \ud55c \ud6c4, \uc544\ubc84\uc9c0\uac00 \uacc4\ubaa8 \uad8c\uc528\ub97c \ub4e4\uc778 \ub4a4 \uae08\uac15\uc0b0\uc5d0 \ub4e4\uc5b4\uac00 \uc2b9\ub824\uac00 \ub418\uc5c8\ub294\ub370, \uc774 \ub54c\ubb38\uc5d0 \ud6d7\ub0a0 \uadf8\uac00 \uc8fd\uc740 \ud6c4\uc5d0\uae4c\uc9c0\ub3c4 '\uba38\ub9ac\ub97c \uae4e\uace0 \uc2b9\ub824\uac00 \ub418\ub824\ub2e4\uac00 \ud658\uc18d\ud55c \uc0ac\ub78c'\uc774\ub77c\uace0 \ub3d9\uc778\uacfc \ub0a8\uc778\uc774 \uacf5\uaca9\ud558\ub294 \ube4c\ubbf8\uac00 \ub418\uc5c8\ub2e4. \uc774\uc900\uacbd\uc774 \uc8fd\uae30 \uc9c1\uc804 \ubd95\ub2f9\uc758 \ud3d0\uc5d0 \uad00\ud55c \uc720\ucc28\ub97c \uc62c\ub9ac\uc790 '\uc8fd\uc74c\uc5d0 \uc774\ub974\ub7ec \ub9d0\uc774 \uc545\ud558\ub2e4'\uace0 \uacf5\uaca9\ud558\uc600\uc73c\uba70 \uc774\ud6c4 \uc774\uc900\uacbd\uc758 \ucc98\ubc8c\uae4c\uc9c0 \uac00\uae30\ub3c4 \ud588\ub2e4. \uadf8\ub7ec\ub098 \ud6c4\uc77c \ub2f9\uc7c1\uc774 \ud604\uc2e4\ud654\ud558\uc790 \uc2a4\uc2a4\ub85c \ud06c\uac8c \ub258\uc6b0\uce58\uace0 \ub3d9\uc778, \uc11c\uc778 \uc0ac\uc774\uc758 \ub2f9\uc7c1 \uc870\uc815\uc744 \ud3c9\uc0dd \uc815\uce58 \uc774\ub150\uc73c\ub85c \uc0bc\uc558\ub2e4. \uacf5\ub0a9(\u8ca2\u7d0d)\uc758 \ud3d0\ub2e8 \uc2dc\uc815\ucc45\uc778 \ub300\uacf5\uc218\ubbf8\ubc95(\u4ee3\u8ca2\u6536\u7c73\u6cd5) \uc2e4\uc2dc\ub97c \uc8fc\uc7a5\ud558\uace0, \ubcd1\uc870\ud310\uc11c\ub85c\uc11c \uc5ec\uc9c4\uc871 \uc774\ud0d5\uac1c\uc758 \uce68\uc785\uc744 \uaca9\ud1f4\ud55c \ud6c4, 10\ub9cc \uc591\ubcd1\uc124\uc744 \uc8fc\uc7a5\ud574 \uc784\uc9c4\uc65c\ub780\uc744 \uc608\uc5b8\ud588\ub2e4\ub294 \uba85\uc131\uc744 \uc5bb\uc5c8\ub2e4. \ubd95\ub2f9\uc744 \uc870\uc815\ud558\uc9c0 \ubabb\ud55c \ud55c\uc744 \ub0a8\uae34 \ucc44 \uc8fd\uc5c8\uc73c\uba70, \uc0ac\ud6c4 \uc758\uc815\ubd80\uc601\uc758\uc815\uc5d0 \ucd94\uc99d\ub418\uc5c8\ub2e4. \uc131\ud63c, \uc1a1\uc775\ud544, \uae40\uc7a5\uc0dd \ub4f1\uacfc \ud568\uaed8 \uae30\ud638 \uc9c0\uc5ed\uc774 \uae30\ubc18\uc778 \uc11c\uc778(\u897f\u4eba)\uc758 \uc885\uc8fc\ub85c \ucd94\uc559\ub41c\ub2e4. \uadf8\ub97c \ubb38\ubb18\uc5d0 \uc885\uc0ac\ud558\ub294 \ubb38\uc81c\ub97c \ub193\uace0 \uc778\uc870\ubc18\uc815 \uc774\ud6c4 50\ub144\uac04 \ub17c\uc7c1\uc758 \ub300\uc0c1\uc774 \ub418\ub2e4\uac00 \uc219\uc885 \ub54c \uacbd\uc2e0\ud658\uad6d\uc73c\ub85c \uc11c\uc778\ub4e4\uc774 \uc9d1\uad8c\ud55c \ud6c4 \ubb38\ubb18\uc5d0 \uc885\uc0ac\ub418\uc5c8\ub2e4.", "paragraph_answer": "\uc544\ud649 \ucc28\ub840\uc758 \uacfc\uac70\uc5d0 \uc7a5\uc6d0\uae09\uc81c\ud558\uc5ec \uad6c\ub3c4\uc7a5\uc6d0\uacf5(\u4e5d\u5ea6\u58ef\u5143\u516c)\uc774\ub77c\ub294 \ubcc4\uce6d\uc744 \uc5bb\uc5c8\ub2e4. 16\uc138 \ub54c \uc5b4\uba38\ub2c8 \uc2e0\uc0ac\uc784\ub2f9(\u7533\u5e2b\u4efb\u5802)\uc774 \uc8fd\uc790 3\ub144\uac04 \uc5ec\ubb18\uc0b4\uc774\ub97c \ud55c \ud6c4, \uc544\ubc84\uc9c0\uac00 \uacc4\ubaa8 \uad8c\uc528\ub97c \ub4e4\uc778 \ub4a4 \uae08\uac15\uc0b0\uc5d0 \ub4e4\uc5b4\uac00 \uc2b9\ub824\uac00 \ub418\uc5c8\ub294\ub370, \uc774 \ub54c\ubb38\uc5d0 \ud6d7\ub0a0 \uadf8\uac00 \uc8fd\uc740 \ud6c4\uc5d0\uae4c\uc9c0\ub3c4 '\uba38\ub9ac\ub97c \uae4e\uace0 \uc2b9\ub824\uac00 \ub418\ub824\ub2e4\uac00 \ud658\uc18d\ud55c \uc0ac\ub78c'\uc774\ub77c\uace0 \ub3d9\uc778\uacfc \ub0a8\uc778\uc774 \uacf5\uaca9\ud558\ub294 \ube4c\ubbf8\uac00 \ub418\uc5c8\ub2e4. \uc774\uc900\uacbd\uc774 \uc8fd\uae30 \uc9c1\uc804 \ubd95\ub2f9\uc758 \ud3d0\uc5d0 \uad00\ud55c \uc720\ucc28\ub97c \uc62c\ub9ac\uc790 '\uc8fd\uc74c\uc5d0 \uc774\ub974\ub7ec \ub9d0\uc774 \uc545\ud558\ub2e4'\uace0 \uacf5\uaca9\ud558\uc600\uc73c\uba70 \uc774\ud6c4 \uc774\uc900\uacbd\uc758 \ucc98\ubc8c\uae4c\uc9c0 \uac00\uae30\ub3c4 \ud588\ub2e4. \uadf8\ub7ec\ub098 \ud6c4\uc77c \ub2f9\uc7c1\uc774 \ud604\uc2e4\ud654\ud558\uc790 \uc2a4\uc2a4\ub85c \ud06c\uac8c \ub258\uc6b0\uce58\uace0 \ub3d9\uc778, \uc11c\uc778 \uc0ac\uc774\uc758 \ub2f9\uc7c1 \uc870\uc815\uc744 \ud3c9\uc0dd \uc815\uce58 \uc774\ub150\uc73c\ub85c \uc0bc\uc558\ub2e4. \uacf5\ub0a9(\u8ca2\u7d0d)\uc758 \ud3d0\ub2e8 \uc2dc\uc815\ucc45\uc778 \ub300\uacf5\uc218\ubbf8\ubc95(\u4ee3\u8ca2\u6536\u7c73\u6cd5) \uc2e4\uc2dc\ub97c \uc8fc\uc7a5\ud558\uace0, \ubcd1\uc870\ud310\uc11c\ub85c\uc11c \uc5ec\uc9c4\uc871 \uc774\ud0d5\uac1c\uc758 \uce68\uc785\uc744 \uaca9\ud1f4\ud55c \ud6c4, 10\ub9cc \uc591\ubcd1\uc124\uc744 \uc8fc\uc7a5\ud574 \uc784\uc9c4\uc65c\ub780\uc744 \uc608\uc5b8\ud588\ub2e4\ub294 \uba85\uc131\uc744 \uc5bb\uc5c8\ub2e4. \ubd95\ub2f9\uc744 \uc870\uc815\ud558\uc9c0 \ubabb\ud55c \ud55c\uc744 \ub0a8\uae34 \ucc44 \uc8fd\uc5c8\uc73c\uba70, \uc0ac\ud6c4 \uc758\uc815\ubd80\uc601\uc758\uc815\uc5d0 \ucd94\uc99d\ub418\uc5c8\ub2e4. \uc131\ud63c, \uc1a1\uc775\ud544, \uae40\uc7a5\uc0dd \ub4f1\uacfc \ud568\uaed8 \uae30\ud638 \uc9c0\uc5ed\uc774 \uae30\ubc18\uc778 \uc11c\uc778(\u897f\u4eba)\uc758 \uc885\uc8fc\ub85c \ucd94\uc559\ub41c\ub2e4. \uadf8\ub97c \ubb38\ubb18\uc5d0 \uc885\uc0ac\ud558\ub294 \ubb38\uc81c\ub97c \ub193\uace0 \uc778\uc870\ubc18\uc815 \uc774\ud6c4 50\ub144\uac04 \ub17c\uc7c1\uc758 \ub300\uc0c1\uc774 \ub418\ub2e4\uac00 \uc219\uc885 \ub54c \uacbd\uc2e0\ud658\uad6d\uc73c\ub85c \uc11c\uc778\ub4e4\uc774 \uc9d1\uad8c\ud55c \ud6c4 \ubb38\ubb18\uc5d0 \uc885\uc0ac\ub418\uc5c8\ub2e4.", "sentence_answer": " 16\uc138 \ub54c \uc5b4\uba38\ub2c8 \uc2e0\uc0ac\uc784\ub2f9(\u7533\u5e2b\u4efb\u5802)\uc774 \uc8fd\uc790 3\ub144\uac04 \uc5ec\ubb18\uc0b4\uc774\ub97c \ud55c \ud6c4, \uc544\ubc84\uc9c0\uac00 \uacc4\ubaa8 \uad8c\uc528\ub97c \ub4e4\uc778 \ub4a4 \uae08\uac15\uc0b0\uc5d0 \ub4e4\uc5b4\uac00 \uc2b9\ub824\uac00 \ub418\uc5c8\ub294\ub370, \uc774 \ub54c\ubb38\uc5d0 \ud6d7\ub0a0 \uadf8\uac00 \uc8fd\uc740 \ud6c4\uc5d0\uae4c\uc9c0\ub3c4 '\uba38\ub9ac\ub97c \uae4e\uace0 \uc2b9\ub824\uac00 \ub418\ub824\ub2e4\uac00 \ud658\uc18d\ud55c \uc0ac\ub78c'\uc774\ub77c\uace0 \ub3d9\uc778\uacfc \ub0a8\uc778\uc774 \uacf5\uaca9\ud558\ub294 \ube4c\ubbf8\uac00 \ub418\uc5c8\ub2e4.", "paragraph_id": "6545935-0-1", "paragraph_question": "question: \uc2e0\uc0ac\uc784\ub2f9\uc740 \uc774\uc774\uac00 \uba87 \uc0b4 \ub54c \uc8fd\uc5c8\ub294\uac00?, context: \uc544\ud649 \ucc28\ub840\uc758 \uacfc\uac70\uc5d0 \uc7a5\uc6d0\uae09\uc81c\ud558\uc5ec \uad6c\ub3c4\uc7a5\uc6d0\uacf5(\u4e5d\u5ea6\u58ef\u5143\u516c)\uc774\ub77c\ub294 \ubcc4\uce6d\uc744 \uc5bb\uc5c8\ub2e4. 16\uc138 \ub54c \uc5b4\uba38\ub2c8 \uc2e0\uc0ac\uc784\ub2f9(\u7533\u5e2b\u4efb\u5802)\uc774 \uc8fd\uc790 3\ub144\uac04 \uc5ec\ubb18\uc0b4\uc774\ub97c \ud55c \ud6c4, \uc544\ubc84\uc9c0\uac00 \uacc4\ubaa8 \uad8c\uc528\ub97c \ub4e4\uc778 \ub4a4 \uae08\uac15\uc0b0\uc5d0 \ub4e4\uc5b4\uac00 \uc2b9\ub824\uac00 \ub418\uc5c8\ub294\ub370, \uc774 \ub54c\ubb38\uc5d0 \ud6d7\ub0a0 \uadf8\uac00 \uc8fd\uc740 \ud6c4\uc5d0\uae4c\uc9c0\ub3c4 '\uba38\ub9ac\ub97c \uae4e\uace0 \uc2b9\ub824\uac00 \ub418\ub824\ub2e4\uac00 \ud658\uc18d\ud55c \uc0ac\ub78c'\uc774\ub77c\uace0 \ub3d9\uc778\uacfc \ub0a8\uc778\uc774 \uacf5\uaca9\ud558\ub294 \ube4c\ubbf8\uac00 \ub418\uc5c8\ub2e4. \uc774\uc900\uacbd\uc774 \uc8fd\uae30 \uc9c1\uc804 \ubd95\ub2f9\uc758 \ud3d0\uc5d0 \uad00\ud55c \uc720\ucc28\ub97c \uc62c\ub9ac\uc790 '\uc8fd\uc74c\uc5d0 \uc774\ub974\ub7ec \ub9d0\uc774 \uc545\ud558\ub2e4'\uace0 \uacf5\uaca9\ud558\uc600\uc73c\uba70 \uc774\ud6c4 \uc774\uc900\uacbd\uc758 \ucc98\ubc8c\uae4c\uc9c0 \uac00\uae30\ub3c4 \ud588\ub2e4. \uadf8\ub7ec\ub098 \ud6c4\uc77c \ub2f9\uc7c1\uc774 \ud604\uc2e4\ud654\ud558\uc790 \uc2a4\uc2a4\ub85c \ud06c\uac8c \ub258\uc6b0\uce58\uace0 \ub3d9\uc778, \uc11c\uc778 \uc0ac\uc774\uc758 \ub2f9\uc7c1 \uc870\uc815\uc744 \ud3c9\uc0dd \uc815\uce58 \uc774\ub150\uc73c\ub85c \uc0bc\uc558\ub2e4. \uacf5\ub0a9(\u8ca2\u7d0d)\uc758 \ud3d0\ub2e8 \uc2dc\uc815\ucc45\uc778 \ub300\uacf5\uc218\ubbf8\ubc95(\u4ee3\u8ca2\u6536\u7c73\u6cd5) \uc2e4\uc2dc\ub97c \uc8fc\uc7a5\ud558\uace0, \ubcd1\uc870\ud310\uc11c\ub85c\uc11c \uc5ec\uc9c4\uc871 \uc774\ud0d5\uac1c\uc758 \uce68\uc785\uc744 \uaca9\ud1f4\ud55c \ud6c4, 10\ub9cc \uc591\ubcd1\uc124\uc744 \uc8fc\uc7a5\ud574 \uc784\uc9c4\uc65c\ub780\uc744 \uc608\uc5b8\ud588\ub2e4\ub294 \uba85\uc131\uc744 \uc5bb\uc5c8\ub2e4. \ubd95\ub2f9\uc744 \uc870\uc815\ud558\uc9c0 \ubabb\ud55c \ud55c\uc744 \ub0a8\uae34 \ucc44 \uc8fd\uc5c8\uc73c\uba70, \uc0ac\ud6c4 \uc758\uc815\ubd80\uc601\uc758\uc815\uc5d0 \ucd94\uc99d\ub418\uc5c8\ub2e4. \uc131\ud63c, \uc1a1\uc775\ud544, \uae40\uc7a5\uc0dd \ub4f1\uacfc \ud568\uaed8 \uae30\ud638 \uc9c0\uc5ed\uc774 \uae30\ubc18\uc778 \uc11c\uc778(\u897f\u4eba)\uc758 \uc885\uc8fc\ub85c \ucd94\uc559\ub41c\ub2e4. \uadf8\ub97c \ubb38\ubb18\uc5d0 \uc885\uc0ac\ud558\ub294 \ubb38\uc81c\ub97c \ub193\uace0 \uc778\uc870\ubc18\uc815 \uc774\ud6c4 50\ub144\uac04 \ub17c\uc7c1\uc758 \ub300\uc0c1\uc774 \ub418\ub2e4\uac00 \uc219\uc885 \ub54c \uacbd\uc2e0\ud658\uad6d\uc73c\ub85c \uc11c\uc778\ub4e4\uc774 \uc9d1\uad8c\ud55c \ud6c4 \ubb38\ubb18\uc5d0 \uc885\uc0ac\ub418\uc5c8\ub2e4."} {"question": "\ub9c9\uc2dc\ubb34\uc2a4\ub294 \uc5b4\ub290 \uc885\uc73c\ub85c \ubd84\ub958\ub418\uc5c8\ub098?", "paragraph": "\uc54c\ub85c\uc0ac\uc6b0\ub8e8\uc2a4\uc18d\uc5d0 \uc5bc\ub9c8\ub098 \ub9ce\uc740 \uc885\uc774 \uc788\uc5c8\ub294\uc9c0\ub294 \uc54c \uc218 \uc5c6\ub2e4. 1988\ub144 \uc774\ub798 \uc77c\ubd80\ub9cc \uc9c0\uc18d\uc801\uc73c\ub85c \uc720\ud6a8\ud55c \uac83\uc73c\ub85c \uac04\uc8fc\ub418\uae34 \ud558\uc9c0\ub9cc, \uc7a0\uc7ac\uc801\uc73c\ub85c 7\uac1c \uc885(\uc54c\ub85c\uc0ac\uc6b0\ub8e8\uc2a4 \uc554\ud50c\ub809\uc218\uc2a4, \uc54c\ub85c\uc0ac\uc6b0\ub8e8\uc2a4 \uc544\ud2b8\ub85d\uc2a4, \uc54c\ub85c\uc0ac\uc6b0\ub8e8\uc2a4 \uc5d0\uc6b0\ub85c\ud30c\uc774\uc6b0\uc2a4, \ubaa8\uc2dd\uc885 \uc54c\ub85c\uc0ac\uc6b0\ub8e8\uc2a4 \ud504\ub77c\uae38\ub9ac\uc2a4, \uc544\uc9c1 \uacf5\uc2dd\uc801\uc73c\ub85c \uc124\uba85\ub418\uc9c0 \uc54a\uc740 \uc54c\ub85c\uc0ac\uc6b0\ub8e8\uc2a4 \uc9d0\ub9e4\ub4dc\uc2a4\ub2c8, \uc54c\ub85c\uc0ac\uc6b0\ub8e8\uc2a4 \ub9c9\uc2dc\ubb34\uc2a4, \uadf8\ub9ac\uace0 \uc54c\ub85c\uc0ac\uc6b0\ub8e8\uc2a4 \ud150\ub2e4\uad6c\ub80c\uc2dc\uc2a4)\uc774 \uc778\uc815\ubc1b\uace0 \uc788\ub2e4. \ub367\ubd99\uc5ec, \uc801\uc5b4\ub3c4 10\uac1c\uc758 \uc560\ub9e4\ud558\uac70\ub098 \ubbf8\uae30\uc220\ub41c \uc885\uc774 \uc54c\ub85c\uc0ac\uc6b0\ub8e8\uc2a4\uc18d\uc5d0 \uc18d\ud574 \uc788\ub2e4. \uac00\uc7a5 \ucd5c\uadfc\uc758 \ud14c\ud0c0\ub204\ub808\ud558\ubaa9\uc758 \uc218\uac01\ub958\ub4e4\uc5d0 \ub300\ud55c \uc7ac\uc870\uc0ac\uc5d0\uc11c\ub294, \uc624\uc9c1 \ud504\ub77c\uae38\ub9ac\uc2a4(\ubd84\ub958\uc0c1 \uc774\uba85\uc73c\ub85c\uc11c \uc554\ud50c\ub809\uc218\uc2a4\uc640 \uc544\ud2b8\ub85d\uc2a4\ub3c4 \ud3ec\ud568)\uc640 \uc9d0\ub9e4\ub4dc\uc2a4\ub2c8(\uc775\uba85\uc758 \uc885) \uadf8\ub9ac\uace0 \ud150\ub2e4\uad6c\ub80c\uc2dc\uc2a4\ub9cc\uc774 \uc7a0\uc7ac\uc801\uc73c\ub85c \uc720\ud6a8\ud55c \uc885\uc73c\ub85c \uc778\uc815\ub418\uc5c8\ub2e4. \uc5d0\uc6b0\ub85c\ud30c\uc774\uc6b0\uc2a4\ub294 \uc544\uc9c1 \uc81c\uc548\ub418\uc9c0\ub3c4 \uc54a\uc558\uc73c\uba70 \ub9c9\uc2dc\ubb34\uc2a4\ub294 \uc0ac\uc6b0\ub85c\ud30c\uac00\ub099\uc2a4\uc18d\uc73c\ub85c \ubd84\ub958\ub418\uc5c8\ub2e4.", "answer": "\uc0ac\uc6b0\ub85c\ud30c\uac00\ub099\uc2a4\uc18d", "sentence": "\uc5d0\uc6b0\ub85c\ud30c\uc774\uc6b0\uc2a4\ub294 \uc544\uc9c1 \uc81c\uc548\ub418\uc9c0\ub3c4 \uc54a\uc558\uc73c\uba70 \ub9c9\uc2dc\ubb34\uc2a4\ub294 \uc0ac\uc6b0\ub85c\ud30c\uac00\ub099\uc2a4\uc18d \uc73c\ub85c \ubd84\ub958\ub418\uc5c8\ub2e4.", "paragraph_sentence": "\uc54c\ub85c\uc0ac\uc6b0\ub8e8\uc2a4\uc18d\uc5d0 \uc5bc\ub9c8\ub098 \ub9ce\uc740 \uc885\uc774 \uc788\uc5c8\ub294\uc9c0\ub294 \uc54c \uc218 \uc5c6\ub2e4. 1988\ub144 \uc774\ub798 \uc77c\ubd80\ub9cc \uc9c0\uc18d\uc801\uc73c\ub85c \uc720\ud6a8\ud55c \uac83\uc73c\ub85c \uac04\uc8fc\ub418\uae34 \ud558\uc9c0\ub9cc, \uc7a0\uc7ac\uc801\uc73c\ub85c 7\uac1c \uc885(\uc54c\ub85c\uc0ac\uc6b0\ub8e8\uc2a4 \uc554\ud50c\ub809\uc218\uc2a4, \uc54c\ub85c\uc0ac\uc6b0\ub8e8\uc2a4 \uc544\ud2b8\ub85d\uc2a4, \uc54c\ub85c\uc0ac\uc6b0\ub8e8\uc2a4 \uc5d0\uc6b0\ub85c\ud30c\uc774\uc6b0\uc2a4, \ubaa8\uc2dd\uc885 \uc54c\ub85c\uc0ac\uc6b0\ub8e8\uc2a4 \ud504\ub77c\uae38\ub9ac\uc2a4, \uc544\uc9c1 \uacf5\uc2dd\uc801\uc73c\ub85c \uc124\uba85\ub418\uc9c0 \uc54a\uc740 \uc54c\ub85c\uc0ac\uc6b0\ub8e8\uc2a4 \uc9d0\ub9e4\ub4dc\uc2a4\ub2c8, \uc54c\ub85c\uc0ac\uc6b0\ub8e8\uc2a4 \ub9c9\uc2dc\ubb34\uc2a4, \uadf8\ub9ac\uace0 \uc54c\ub85c\uc0ac\uc6b0\ub8e8\uc2a4 \ud150\ub2e4\uad6c\ub80c\uc2dc\uc2a4)\uc774 \uc778\uc815\ubc1b\uace0 \uc788\ub2e4. \ub367\ubd99\uc5ec, \uc801\uc5b4\ub3c4 10\uac1c\uc758 \uc560\ub9e4\ud558\uac70\ub098 \ubbf8\uae30\uc220\ub41c \uc885\uc774 \uc54c\ub85c\uc0ac\uc6b0\ub8e8\uc2a4\uc18d\uc5d0 \uc18d\ud574 \uc788\ub2e4. \uac00\uc7a5 \ucd5c\uadfc\uc758 \ud14c\ud0c0\ub204\ub808\ud558\ubaa9\uc758 \uc218\uac01\ub958\ub4e4\uc5d0 \ub300\ud55c \uc7ac\uc870\uc0ac\uc5d0\uc11c\ub294, \uc624\uc9c1 \ud504\ub77c\uae38\ub9ac\uc2a4(\ubd84\ub958\uc0c1 \uc774\uba85\uc73c\ub85c\uc11c \uc554\ud50c\ub809\uc218\uc2a4\uc640 \uc544\ud2b8\ub85d\uc2a4\ub3c4 \ud3ec\ud568)\uc640 \uc9d0\ub9e4\ub4dc\uc2a4\ub2c8(\uc775\uba85\uc758 \uc885) \uadf8\ub9ac\uace0 \ud150\ub2e4\uad6c\ub80c\uc2dc\uc2a4\ub9cc\uc774 \uc7a0\uc7ac\uc801\uc73c\ub85c \uc720\ud6a8\ud55c \uc885\uc73c\ub85c \uc778\uc815\ub418\uc5c8\ub2e4. \uc5d0\uc6b0\ub85c\ud30c\uc774\uc6b0\uc2a4\ub294 \uc544\uc9c1 \uc81c\uc548\ub418\uc9c0\ub3c4 \uc54a\uc558\uc73c\uba70 \ub9c9\uc2dc\ubb34\uc2a4\ub294 \uc0ac\uc6b0\ub85c\ud30c\uac00\ub099\uc2a4\uc18d \uc73c\ub85c \ubd84\ub958\ub418\uc5c8\ub2e4. ", "paragraph_answer": "\uc54c\ub85c\uc0ac\uc6b0\ub8e8\uc2a4\uc18d\uc5d0 \uc5bc\ub9c8\ub098 \ub9ce\uc740 \uc885\uc774 \uc788\uc5c8\ub294\uc9c0\ub294 \uc54c \uc218 \uc5c6\ub2e4. 1988\ub144 \uc774\ub798 \uc77c\ubd80\ub9cc \uc9c0\uc18d\uc801\uc73c\ub85c \uc720\ud6a8\ud55c \uac83\uc73c\ub85c \uac04\uc8fc\ub418\uae34 \ud558\uc9c0\ub9cc, \uc7a0\uc7ac\uc801\uc73c\ub85c 7\uac1c \uc885(\uc54c\ub85c\uc0ac\uc6b0\ub8e8\uc2a4 \uc554\ud50c\ub809\uc218\uc2a4, \uc54c\ub85c\uc0ac\uc6b0\ub8e8\uc2a4 \uc544\ud2b8\ub85d\uc2a4, \uc54c\ub85c\uc0ac\uc6b0\ub8e8\uc2a4 \uc5d0\uc6b0\ub85c\ud30c\uc774\uc6b0\uc2a4, \ubaa8\uc2dd\uc885 \uc54c\ub85c\uc0ac\uc6b0\ub8e8\uc2a4 \ud504\ub77c\uae38\ub9ac\uc2a4, \uc544\uc9c1 \uacf5\uc2dd\uc801\uc73c\ub85c \uc124\uba85\ub418\uc9c0 \uc54a\uc740 \uc54c\ub85c\uc0ac\uc6b0\ub8e8\uc2a4 \uc9d0\ub9e4\ub4dc\uc2a4\ub2c8, \uc54c\ub85c\uc0ac\uc6b0\ub8e8\uc2a4 \ub9c9\uc2dc\ubb34\uc2a4, \uadf8\ub9ac\uace0 \uc54c\ub85c\uc0ac\uc6b0\ub8e8\uc2a4 \ud150\ub2e4\uad6c\ub80c\uc2dc\uc2a4)\uc774 \uc778\uc815\ubc1b\uace0 \uc788\ub2e4. \ub367\ubd99\uc5ec, \uc801\uc5b4\ub3c4 10\uac1c\uc758 \uc560\ub9e4\ud558\uac70\ub098 \ubbf8\uae30\uc220\ub41c \uc885\uc774 \uc54c\ub85c\uc0ac\uc6b0\ub8e8\uc2a4\uc18d\uc5d0 \uc18d\ud574 \uc788\ub2e4. \uac00\uc7a5 \ucd5c\uadfc\uc758 \ud14c\ud0c0\ub204\ub808\ud558\ubaa9\uc758 \uc218\uac01\ub958\ub4e4\uc5d0 \ub300\ud55c \uc7ac\uc870\uc0ac\uc5d0\uc11c\ub294, \uc624\uc9c1 \ud504\ub77c\uae38\ub9ac\uc2a4(\ubd84\ub958\uc0c1 \uc774\uba85\uc73c\ub85c\uc11c \uc554\ud50c\ub809\uc218\uc2a4\uc640 \uc544\ud2b8\ub85d\uc2a4\ub3c4 \ud3ec\ud568)\uc640 \uc9d0\ub9e4\ub4dc\uc2a4\ub2c8(\uc775\uba85\uc758 \uc885) \uadf8\ub9ac\uace0 \ud150\ub2e4\uad6c\ub80c\uc2dc\uc2a4\ub9cc\uc774 \uc7a0\uc7ac\uc801\uc73c\ub85c \uc720\ud6a8\ud55c \uc885\uc73c\ub85c \uc778\uc815\ub418\uc5c8\ub2e4. \uc5d0\uc6b0\ub85c\ud30c\uc774\uc6b0\uc2a4\ub294 \uc544\uc9c1 \uc81c\uc548\ub418\uc9c0\ub3c4 \uc54a\uc558\uc73c\uba70 \ub9c9\uc2dc\ubb34\uc2a4\ub294 \uc0ac\uc6b0\ub85c\ud30c\uac00\ub099\uc2a4\uc18d \uc73c\ub85c \ubd84\ub958\ub418\uc5c8\ub2e4.", "sentence_answer": "\uc5d0\uc6b0\ub85c\ud30c\uc774\uc6b0\uc2a4\ub294 \uc544\uc9c1 \uc81c\uc548\ub418\uc9c0\ub3c4 \uc54a\uc558\uc73c\uba70 \ub9c9\uc2dc\ubb34\uc2a4\ub294 \uc0ac\uc6b0\ub85c\ud30c\uac00\ub099\uc2a4\uc18d \uc73c\ub85c \ubd84\ub958\ub418\uc5c8\ub2e4.", "paragraph_id": "6465441-11-1", "paragraph_question": "question: \ub9c9\uc2dc\ubb34\uc2a4\ub294 \uc5b4\ub290 \uc885\uc73c\ub85c \ubd84\ub958\ub418\uc5c8\ub098?, context: \uc54c\ub85c\uc0ac\uc6b0\ub8e8\uc2a4\uc18d\uc5d0 \uc5bc\ub9c8\ub098 \ub9ce\uc740 \uc885\uc774 \uc788\uc5c8\ub294\uc9c0\ub294 \uc54c \uc218 \uc5c6\ub2e4. 1988\ub144 \uc774\ub798 \uc77c\ubd80\ub9cc \uc9c0\uc18d\uc801\uc73c\ub85c \uc720\ud6a8\ud55c \uac83\uc73c\ub85c \uac04\uc8fc\ub418\uae34 \ud558\uc9c0\ub9cc, \uc7a0\uc7ac\uc801\uc73c\ub85c 7\uac1c \uc885(\uc54c\ub85c\uc0ac\uc6b0\ub8e8\uc2a4 \uc554\ud50c\ub809\uc218\uc2a4, \uc54c\ub85c\uc0ac\uc6b0\ub8e8\uc2a4 \uc544\ud2b8\ub85d\uc2a4, \uc54c\ub85c\uc0ac\uc6b0\ub8e8\uc2a4 \uc5d0\uc6b0\ub85c\ud30c\uc774\uc6b0\uc2a4, \ubaa8\uc2dd\uc885 \uc54c\ub85c\uc0ac\uc6b0\ub8e8\uc2a4 \ud504\ub77c\uae38\ub9ac\uc2a4, \uc544\uc9c1 \uacf5\uc2dd\uc801\uc73c\ub85c \uc124\uba85\ub418\uc9c0 \uc54a\uc740 \uc54c\ub85c\uc0ac\uc6b0\ub8e8\uc2a4 \uc9d0\ub9e4\ub4dc\uc2a4\ub2c8, \uc54c\ub85c\uc0ac\uc6b0\ub8e8\uc2a4 \ub9c9\uc2dc\ubb34\uc2a4, \uadf8\ub9ac\uace0 \uc54c\ub85c\uc0ac\uc6b0\ub8e8\uc2a4 \ud150\ub2e4\uad6c\ub80c\uc2dc\uc2a4)\uc774 \uc778\uc815\ubc1b\uace0 \uc788\ub2e4. \ub367\ubd99\uc5ec, \uc801\uc5b4\ub3c4 10\uac1c\uc758 \uc560\ub9e4\ud558\uac70\ub098 \ubbf8\uae30\uc220\ub41c \uc885\uc774 \uc54c\ub85c\uc0ac\uc6b0\ub8e8\uc2a4\uc18d\uc5d0 \uc18d\ud574 \uc788\ub2e4. \uac00\uc7a5 \ucd5c\uadfc\uc758 \ud14c\ud0c0\ub204\ub808\ud558\ubaa9\uc758 \uc218\uac01\ub958\ub4e4\uc5d0 \ub300\ud55c \uc7ac\uc870\uc0ac\uc5d0\uc11c\ub294, \uc624\uc9c1 \ud504\ub77c\uae38\ub9ac\uc2a4(\ubd84\ub958\uc0c1 \uc774\uba85\uc73c\ub85c\uc11c \uc554\ud50c\ub809\uc218\uc2a4\uc640 \uc544\ud2b8\ub85d\uc2a4\ub3c4 \ud3ec\ud568)\uc640 \uc9d0\ub9e4\ub4dc\uc2a4\ub2c8(\uc775\uba85\uc758 \uc885) \uadf8\ub9ac\uace0 \ud150\ub2e4\uad6c\ub80c\uc2dc\uc2a4\ub9cc\uc774 \uc7a0\uc7ac\uc801\uc73c\ub85c \uc720\ud6a8\ud55c \uc885\uc73c\ub85c \uc778\uc815\ub418\uc5c8\ub2e4. \uc5d0\uc6b0\ub85c\ud30c\uc774\uc6b0\uc2a4\ub294 \uc544\uc9c1 \uc81c\uc548\ub418\uc9c0\ub3c4 \uc54a\uc558\uc73c\uba70 \ub9c9\uc2dc\ubb34\uc2a4\ub294 \uc0ac\uc6b0\ub85c\ud30c\uac00\ub099\uc2a4\uc18d\uc73c\ub85c \ubd84\ub958\ub418\uc5c8\ub2e4."} {"question": "1970\ub144 \ub300\uc5d0 \ub9c8\uc774\ub7f0 \ud06c\ub8e8\uac70\uc5d0 \uc758\ud574 \ub9cc\ub4e4\uc5b4\uc9c4 \ub2e8\uc5b4\ub85c \ud604\uc7ac\uc758 '\uac00\uc0c1\ud604\uc2e4' \uc758\ubbf8\uc640 \uac00\uae4c\uc6b4 \ub2e8\uc5b4\ub294?", "paragraph": "'\ubc84\ucd94\uc5bc\ub9ac\uc5bc\ub9ac\ud2f0(Virtual Reality; VR)'\ub77c\ub294 \ub2e8\uc5b4\uc758 \uae30\uc6d0\uc740 \ud504\ub791\uc2a4\uc758 \uadf9\uc791\uac00, \uc2dc\uc778, \ubc30\uc6b0\uc774\uc790 \uc5f0\ucd9c\uac00\uc778 \uc559\ud1a0\ub0c9 \uc544\ub974\ud1a0(Antonin Artaud)\uc758 \ucc45\uc5d0\uc11c \uadf8\uac00 \uadf9\uc7a5\uc744 \ubb18\uc0ac\ud558\ub294 \ub2e8\uc5b4\ub85c '\ubc84\ucd94\uc5bc \ub9ac\uc5bc\ub9ac\ud2f0'\ub97c \uc0ac\uc6a9\ud55c \uac83\uc774\ub2e4. \ud604\uc7ac\uc758 '\uac00\uc0c1\ud604\uc2e4' \uc758\ubbf8\uc640 \uac00\uae4c\uc6b4 '\uc778\uacf5 \ud604\uc2e4(artificial reality)'\uc774\ub77c\ub294 \ub2e8\uc5b4\ub294 1970\ub144 \ub300\uc5d0 \uccab \uc138\ub300 \uac00\uc0c1\ud604\uc2e4 \uc5f0\uad6c\uac00 \uc911 \ud558\ub098\uc778 \ub9c8\uc774\ub7f0 \ud06c\ub8e8\uac70(Myron Krueger)\uc5d0 \uc758\ud574 \ub9cc\ub4e4\uc5b4\uc84c\ub2e4. \uadf8 \ud6c4 1980\ub144\ub300 \ud6c4\ubc18\uc5d0 \ubbf8\uad6d\uc758 \ucef4\ud4e8\ud130 \uacfc\ud559\uc790\uc778 \uc7ac\ub7f0 \ub798\ub2c8\uc5b4(Jaron Lanier)\uc5d0 \uc758\ud574 \ud604\uc7ac\uc758 \uac00\uc0c1\ud604\uc2e4 \uac1c\ub150\uc744 \ub73b\ud558\ub294 \ub2e8\uc5b4\uc778 '\ubc84\ucd94\uc5bc \ub9ac\uc5bc\ub9ac\ud2f0'\uac00 \ub110\ub9ac \uc4f0\uc774\uac8c \ub418\uc5c8\ub2e4. \ud55c\uad6d\uc5b4\ub85c '\ubc84\ucd94\uc5bc \ub9ac\uc5bc\ub9ac\ud2f0'\ub294 '\uac00\uc0c1\ud604\uc2e4'\uc774\ub77c\uace0 \ubc88\uc5ed\ub418\ub294\ub370, \uc774\ub294 \uc77c\ubcf8\uc5b4 \ubc88\uc5ed\uc744 \uadf8\ub300\ub85c \ubcf8\ub72c \uac83\uc774\ub77c \uc54c\ub824\uc838 \uc788\ub2e4. \ucef4\ud4e8\ud130\uc758 \ucca8\ub2e8\uae30\uc220\uc744 \ub3d9\uc6d0\ud574 \uc778\uac04\uc758 \uc624\uac10\uc744 \ucc3d\uc870\ud558\ub294 \uac83\uc774 \ubc84\ucd94\uc5bc \ub9ac\uc5bc\ub9ac\ud2f0\uc758 \uac1c\ub150\uc778\ub370 \ub9c8\uce58 \ud604\uc2e4\uacfc \uc644\uc804\ud788 \ub300\uce6d\uc801 \uac1c\ub150\uc73c\ub85c \uc4f0\uc778 \ub4ef\ud55c '\uac00\uc0c1\ud604\uc2e4' \uc774\ub77c\ub294 \ubc88\uc5ed\uc774 \uc798\ubabb\ub418\uc5c8\ub2e4\ub294 \uc758\uacac\ub3c4 \uc788\ub2e4.", "answer": "\uc778\uacf5 \ud604\uc2e4(artificial reality)", "sentence": "\ud604\uc7ac\uc758 '\uac00\uc0c1\ud604\uc2e4' \uc758\ubbf8\uc640 \uac00\uae4c\uc6b4 ' \uc778\uacf5 \ud604\uc2e4(artificial reality) '\uc774\ub77c\ub294 \ub2e8\uc5b4\ub294 1970\ub144 \ub300\uc5d0 \uccab \uc138\ub300 \uac00\uc0c1\ud604\uc2e4 \uc5f0\uad6c\uac00 \uc911 \ud558\ub098\uc778 \ub9c8\uc774\ub7f0 \ud06c\ub8e8\uac70(Myron Krueger)\uc5d0 \uc758\ud574 \ub9cc\ub4e4\uc5b4\uc84c\ub2e4.", "paragraph_sentence": "'\ubc84\ucd94\uc5bc\ub9ac\uc5bc\ub9ac\ud2f0(Virtual Reality; VR)'\ub77c\ub294 \ub2e8\uc5b4\uc758 \uae30\uc6d0\uc740 \ud504\ub791\uc2a4\uc758 \uadf9\uc791\uac00, \uc2dc\uc778, \ubc30\uc6b0\uc774\uc790 \uc5f0\ucd9c\uac00\uc778 \uc559\ud1a0\ub0c9 \uc544\ub974\ud1a0(Antonin Artaud)\uc758 \ucc45\uc5d0\uc11c \uadf8\uac00 \uadf9\uc7a5\uc744 \ubb18\uc0ac\ud558\ub294 \ub2e8\uc5b4\ub85c '\ubc84\ucd94\uc5bc \ub9ac\uc5bc\ub9ac\ud2f0'\ub97c \uc0ac\uc6a9\ud55c \uac83\uc774\ub2e4. \ud604\uc7ac\uc758 '\uac00\uc0c1\ud604\uc2e4' \uc758\ubbf8\uc640 \uac00\uae4c\uc6b4 ' \uc778\uacf5 \ud604\uc2e4(artificial reality) '\uc774\ub77c\ub294 \ub2e8\uc5b4\ub294 1970\ub144 \ub300\uc5d0 \uccab \uc138\ub300 \uac00\uc0c1\ud604\uc2e4 \uc5f0\uad6c\uac00 \uc911 \ud558\ub098\uc778 \ub9c8\uc774\ub7f0 \ud06c\ub8e8\uac70(Myron Krueger)\uc5d0 \uc758\ud574 \ub9cc\ub4e4\uc5b4\uc84c\ub2e4. \uadf8 \ud6c4 1980\ub144\ub300 \ud6c4\ubc18\uc5d0 \ubbf8\uad6d\uc758 \ucef4\ud4e8\ud130 \uacfc\ud559\uc790\uc778 \uc7ac\ub7f0 \ub798\ub2c8\uc5b4(Jaron Lanier)\uc5d0 \uc758\ud574 \ud604\uc7ac\uc758 \uac00\uc0c1\ud604\uc2e4 \uac1c\ub150\uc744 \ub73b\ud558\ub294 \ub2e8\uc5b4\uc778 '\ubc84\ucd94\uc5bc \ub9ac\uc5bc\ub9ac\ud2f0'\uac00 \ub110\ub9ac \uc4f0\uc774\uac8c \ub418\uc5c8\ub2e4. \ud55c\uad6d\uc5b4\ub85c '\ubc84\ucd94\uc5bc \ub9ac\uc5bc\ub9ac\ud2f0'\ub294 '\uac00\uc0c1\ud604\uc2e4'\uc774\ub77c\uace0 \ubc88\uc5ed\ub418\ub294\ub370, \uc774\ub294 \uc77c\ubcf8\uc5b4 \ubc88\uc5ed\uc744 \uadf8\ub300\ub85c \ubcf8\ub72c \uac83\uc774\ub77c \uc54c\ub824\uc838 \uc788\ub2e4. \ucef4\ud4e8\ud130\uc758 \ucca8\ub2e8\uae30\uc220\uc744 \ub3d9\uc6d0\ud574 \uc778\uac04\uc758 \uc624\uac10\uc744 \ucc3d\uc870\ud558\ub294 \uac83\uc774 \ubc84\ucd94\uc5bc \ub9ac\uc5bc\ub9ac\ud2f0\uc758 \uac1c\ub150\uc778\ub370 \ub9c8\uce58 \ud604\uc2e4\uacfc \uc644\uc804\ud788 \ub300\uce6d\uc801 \uac1c\ub150\uc73c\ub85c \uc4f0\uc778 \ub4ef\ud55c '\uac00\uc0c1\ud604\uc2e4' \uc774\ub77c\ub294 \ubc88\uc5ed\uc774 \uc798\ubabb\ub418\uc5c8\ub2e4\ub294 \uc758\uacac\ub3c4 \uc788\ub2e4.", "paragraph_answer": "'\ubc84\ucd94\uc5bc\ub9ac\uc5bc\ub9ac\ud2f0(Virtual Reality; VR)'\ub77c\ub294 \ub2e8\uc5b4\uc758 \uae30\uc6d0\uc740 \ud504\ub791\uc2a4\uc758 \uadf9\uc791\uac00, \uc2dc\uc778, \ubc30\uc6b0\uc774\uc790 \uc5f0\ucd9c\uac00\uc778 \uc559\ud1a0\ub0c9 \uc544\ub974\ud1a0(Antonin Artaud)\uc758 \ucc45\uc5d0\uc11c \uadf8\uac00 \uadf9\uc7a5\uc744 \ubb18\uc0ac\ud558\ub294 \ub2e8\uc5b4\ub85c '\ubc84\ucd94\uc5bc \ub9ac\uc5bc\ub9ac\ud2f0'\ub97c \uc0ac\uc6a9\ud55c \uac83\uc774\ub2e4. \ud604\uc7ac\uc758 '\uac00\uc0c1\ud604\uc2e4' \uc758\ubbf8\uc640 \uac00\uae4c\uc6b4 ' \uc778\uacf5 \ud604\uc2e4(artificial reality) '\uc774\ub77c\ub294 \ub2e8\uc5b4\ub294 1970\ub144 \ub300\uc5d0 \uccab \uc138\ub300 \uac00\uc0c1\ud604\uc2e4 \uc5f0\uad6c\uac00 \uc911 \ud558\ub098\uc778 \ub9c8\uc774\ub7f0 \ud06c\ub8e8\uac70(Myron Krueger)\uc5d0 \uc758\ud574 \ub9cc\ub4e4\uc5b4\uc84c\ub2e4. \uadf8 \ud6c4 1980\ub144\ub300 \ud6c4\ubc18\uc5d0 \ubbf8\uad6d\uc758 \ucef4\ud4e8\ud130 \uacfc\ud559\uc790\uc778 \uc7ac\ub7f0 \ub798\ub2c8\uc5b4(Jaron Lanier)\uc5d0 \uc758\ud574 \ud604\uc7ac\uc758 \uac00\uc0c1\ud604\uc2e4 \uac1c\ub150\uc744 \ub73b\ud558\ub294 \ub2e8\uc5b4\uc778 '\ubc84\ucd94\uc5bc \ub9ac\uc5bc\ub9ac\ud2f0'\uac00 \ub110\ub9ac \uc4f0\uc774\uac8c \ub418\uc5c8\ub2e4. \ud55c\uad6d\uc5b4\ub85c '\ubc84\ucd94\uc5bc \ub9ac\uc5bc\ub9ac\ud2f0'\ub294 '\uac00\uc0c1\ud604\uc2e4'\uc774\ub77c\uace0 \ubc88\uc5ed\ub418\ub294\ub370, \uc774\ub294 \uc77c\ubcf8\uc5b4 \ubc88\uc5ed\uc744 \uadf8\ub300\ub85c \ubcf8\ub72c \uac83\uc774\ub77c \uc54c\ub824\uc838 \uc788\ub2e4. \ucef4\ud4e8\ud130\uc758 \ucca8\ub2e8\uae30\uc220\uc744 \ub3d9\uc6d0\ud574 \uc778\uac04\uc758 \uc624\uac10\uc744 \ucc3d\uc870\ud558\ub294 \uac83\uc774 \ubc84\ucd94\uc5bc \ub9ac\uc5bc\ub9ac\ud2f0\uc758 \uac1c\ub150\uc778\ub370 \ub9c8\uce58 \ud604\uc2e4\uacfc \uc644\uc804\ud788 \ub300\uce6d\uc801 \uac1c\ub150\uc73c\ub85c \uc4f0\uc778 \ub4ef\ud55c '\uac00\uc0c1\ud604\uc2e4' \uc774\ub77c\ub294 \ubc88\uc5ed\uc774 \uc798\ubabb\ub418\uc5c8\ub2e4\ub294 \uc758\uacac\ub3c4 \uc788\ub2e4.", "sentence_answer": "\ud604\uc7ac\uc758 '\uac00\uc0c1\ud604\uc2e4' \uc758\ubbf8\uc640 \uac00\uae4c\uc6b4 ' \uc778\uacf5 \ud604\uc2e4(artificial reality) '\uc774\ub77c\ub294 \ub2e8\uc5b4\ub294 1970\ub144 \ub300\uc5d0 \uccab \uc138\ub300 \uac00\uc0c1\ud604\uc2e4 \uc5f0\uad6c\uac00 \uc911 \ud558\ub098\uc778 \ub9c8\uc774\ub7f0 \ud06c\ub8e8\uac70(Myron Krueger)\uc5d0 \uc758\ud574 \ub9cc\ub4e4\uc5b4\uc84c\ub2e4.", "paragraph_id": "5782945-1-1", "paragraph_question": "question: 1970\ub144 \ub300\uc5d0 \ub9c8\uc774\ub7f0 \ud06c\ub8e8\uac70\uc5d0 \uc758\ud574 \ub9cc\ub4e4\uc5b4\uc9c4 \ub2e8\uc5b4\ub85c \ud604\uc7ac\uc758 '\uac00\uc0c1\ud604\uc2e4' \uc758\ubbf8\uc640 \uac00\uae4c\uc6b4 \ub2e8\uc5b4\ub294?, context: '\ubc84\ucd94\uc5bc\ub9ac\uc5bc\ub9ac\ud2f0(Virtual Reality; VR)'\ub77c\ub294 \ub2e8\uc5b4\uc758 \uae30\uc6d0\uc740 \ud504\ub791\uc2a4\uc758 \uadf9\uc791\uac00, \uc2dc\uc778, \ubc30\uc6b0\uc774\uc790 \uc5f0\ucd9c\uac00\uc778 \uc559\ud1a0\ub0c9 \uc544\ub974\ud1a0(Antonin Artaud)\uc758 \ucc45\uc5d0\uc11c \uadf8\uac00 \uadf9\uc7a5\uc744 \ubb18\uc0ac\ud558\ub294 \ub2e8\uc5b4\ub85c '\ubc84\ucd94\uc5bc \ub9ac\uc5bc\ub9ac\ud2f0'\ub97c \uc0ac\uc6a9\ud55c \uac83\uc774\ub2e4. \ud604\uc7ac\uc758 '\uac00\uc0c1\ud604\uc2e4' \uc758\ubbf8\uc640 \uac00\uae4c\uc6b4 '\uc778\uacf5 \ud604\uc2e4(artificial reality)'\uc774\ub77c\ub294 \ub2e8\uc5b4\ub294 1970\ub144 \ub300\uc5d0 \uccab \uc138\ub300 \uac00\uc0c1\ud604\uc2e4 \uc5f0\uad6c\uac00 \uc911 \ud558\ub098\uc778 \ub9c8\uc774\ub7f0 \ud06c\ub8e8\uac70(Myron Krueger)\uc5d0 \uc758\ud574 \ub9cc\ub4e4\uc5b4\uc84c\ub2e4. \uadf8 \ud6c4 1980\ub144\ub300 \ud6c4\ubc18\uc5d0 \ubbf8\uad6d\uc758 \ucef4\ud4e8\ud130 \uacfc\ud559\uc790\uc778 \uc7ac\ub7f0 \ub798\ub2c8\uc5b4(Jaron Lanier)\uc5d0 \uc758\ud574 \ud604\uc7ac\uc758 \uac00\uc0c1\ud604\uc2e4 \uac1c\ub150\uc744 \ub73b\ud558\ub294 \ub2e8\uc5b4\uc778 '\ubc84\ucd94\uc5bc \ub9ac\uc5bc\ub9ac\ud2f0'\uac00 \ub110\ub9ac \uc4f0\uc774\uac8c \ub418\uc5c8\ub2e4. \ud55c\uad6d\uc5b4\ub85c '\ubc84\ucd94\uc5bc \ub9ac\uc5bc\ub9ac\ud2f0'\ub294 '\uac00\uc0c1\ud604\uc2e4'\uc774\ub77c\uace0 \ubc88\uc5ed\ub418\ub294\ub370, \uc774\ub294 \uc77c\ubcf8\uc5b4 \ubc88\uc5ed\uc744 \uadf8\ub300\ub85c \ubcf8\ub72c \uac83\uc774\ub77c \uc54c\ub824\uc838 \uc788\ub2e4. \ucef4\ud4e8\ud130\uc758 \ucca8\ub2e8\uae30\uc220\uc744 \ub3d9\uc6d0\ud574 \uc778\uac04\uc758 \uc624\uac10\uc744 \ucc3d\uc870\ud558\ub294 \uac83\uc774 \ubc84\ucd94\uc5bc \ub9ac\uc5bc\ub9ac\ud2f0\uc758 \uac1c\ub150\uc778\ub370 \ub9c8\uce58 \ud604\uc2e4\uacfc \uc644\uc804\ud788 \ub300\uce6d\uc801 \uac1c\ub150\uc73c\ub85c \uc4f0\uc778 \ub4ef\ud55c '\uac00\uc0c1\ud604\uc2e4' \uc774\ub77c\ub294 \ubc88\uc5ed\uc774 \uc798\ubabb\ub418\uc5c8\ub2e4\ub294 \uc758\uacac\ub3c4 \uc788\ub2e4."} {"question": "\uccad\ub098\ub77c\uc5d0 \uc7a0\uc785\ud558\uc5ec \ud558\uc5bc\ube48 \uc5ed\uc5d0\uc11c \uc12d\uc815\uc655\uc758 \ub3d9\uc0dd\uc744 \uc554\uc0b4\ud558\ub824 \ud588\ub358 \uc0ac\ub78c\uc740?", "paragraph": "1908\ub144 10\uc6d4 \uc548\ud718 \uc131\uc5d0\uc11c \u2018\uc2e0\uad70 \uc0ac\uac74\u2019(\u65b0\u8ecd\u4e8b\u4ef6)\uc774 \ubc1c\uc0dd\ud588\ub294\ub370, \uc774\ub97c \u2018\uc548\ud718 \uc2e0\uad70 \ubd09\uae30\u2019\ub77c\uace0 \ubd80\ub978\ub2e4. \uc2e0\uad70\uc740 \uc704\uc548\uc2a4\uce74\uc774\uac00 \uc0c8\ub85c \ud3b8\uc131\ud55c \uc2e0\uc2dd \uc721\uad70\uc73c\ub85c \uc2e0\uad70\uc758 \uccad\ub144 \uc7a5\ubcd1 \uac00\uc6b4\ub370\uc5d0\ub294 \uc468\uc6d0\uc758 \ud601\uba85 \uc0ac\uc0c1\uc5d0 \ub3d9\uc870\ud558\ub294 \ubbfc\uc871\uc8fc\uc758\uc790\ub4e4\uc774 \ub9ce\uc558\ub2e4. \uc774\ub4e4\uc740 \uc545\uc655\ud68c\ub77c\ub294 \ube44\ubc00\ub2e8\uccb4\ub97c \ub9cc\ub4e4\uc5b4 \ud55c\uc871 \uad6d\uac00 \uac74\uc124\uc758 \uacb0\uc758\ub97c \ub2e4\uc84c\ub2e4. 1908\ub144 10\uc6d4 23\uc77c \uc545\uc655\ud68c\uc758 \uac04\ubd80\uc778 \ubc94\uc804\uac11\uc740 \uad11\uc11c\uc81c\uc640 \uc11c\ud0dc\ud6c4\uc758 \uc8fd\uc74c \uc18c\uc2dd\uc744 \ub4e3\uace0 10\uc6d4 26\uc77c\uc744 \uae30\ud558\uc5ec \ubd09\uae30\ud558\uae30\ub85c \ud558\uace0 \uac01 \uc9c4\uc601\uc758 \uac04\ubd80\uc640 \uc5f0\ub77d\ud558\uc5ec \uc6c5\uc131\uae30\ub97c \uc0ac\ub839\uad00\uc73c\ub85c \ucd94\ub300\ud558\uc600\ub2e4. \uc774\ub807\uac8c \ud574\uc11c \uc131 \ubc16\uc5d0 \uc788\ub358 \uc2e0\uad70\uc740 \uc77c\uc81c\ud788 \ubd09\uae30\ud558\uc600\uc73c\ub098 \ud0c4\uc57d\uc758 \ubd80\uc871\uacfc \uc131 \uc548\uc758 \uad70\ub300\uac00 \ub0b4\uc751\ud558\uc9c0 \uc54a\uace0 \ud574\uad70 \ub610\ud55c \ubd09\uae30\uc5d0 \ubc18\ub300\ud588\uae30 \ub54c\ubb38\uc5d0 \uc2e4\ud328\ub85c \ub3cc\uc544\uac14\ub2e4. \uc8fc\ub3d9\uc790\uc600\ub358 \ubc94\uc804\uac11\uc740 \uccb4\ud3ec, \ucc98\ud615\ub418\uace0 \uc6c5\uc131\uae30\ub294 \uc77c\ubcf8\uc73c\ub85c \uac74\ub108\uac00 \uadf8 \uacf3\uc5d0\uc11c \uc911\uad6d\ub3d9\ub9f9\ud68c\uc5d0 \uac00\uc785\ud558\uc600\ub2e4. \uadf8 \ud6c4 \uc6c5\uc131\uae30\ub294 \ub2e4\uc2dc \uccad\ub098\ub77c\uc5d0 \uc7a0\uc785\ud558\uc5ec \uc720\ub7fd\uc5d0\uc11c \uadc0\uad6d\ud558\ub294 \uc12d\uc815\uc655\uc758 \ub450 \ub3d9\uc0dd \uc7ac\ub3c4(\u8f09\u6fe4)\uc640 \uc7ac\uc21c(\u8f09\u6d35)\uc744 \ud558\uc5bc\ube48 \uc5ed\uc5d0\uc11c \uc554\uc0b4\ud558\ub824\ub2e4 \ubc00\uace0\ub85c \uccb4\ud3ec\ub418\uc5b4 \uc0ac\ud615\uc5d0 \ucc98\ud574\uc84c\ub2e4.", "answer": "\uc6c5\uc131\uae30", "sentence": "1908\ub144 10\uc6d4 23\uc77c \uc545\uc655\ud68c\uc758 \uac04\ubd80\uc778 \ubc94\uc804\uac11\uc740 \uad11\uc11c\uc81c\uc640 \uc11c\ud0dc\ud6c4\uc758 \uc8fd\uc74c \uc18c\uc2dd\uc744 \ub4e3\uace0 10\uc6d4 26\uc77c\uc744 \uae30\ud558\uc5ec \ubd09\uae30\ud558\uae30\ub85c \ud558\uace0 \uac01 \uc9c4\uc601\uc758 \uac04\ubd80\uc640 \uc5f0\ub77d\ud558\uc5ec \uc6c5\uc131\uae30 \ub97c \uc0ac\ub839\uad00\uc73c\ub85c \ucd94\ub300\ud558\uc600\ub2e4.", "paragraph_sentence": "1908\ub144 10\uc6d4 \uc548\ud718 \uc131\uc5d0\uc11c \u2018\uc2e0\uad70 \uc0ac\uac74\u2019(\u65b0\u8ecd\u4e8b\u4ef6)\uc774 \ubc1c\uc0dd\ud588\ub294\ub370, \uc774\ub97c \u2018\uc548\ud718 \uc2e0\uad70 \ubd09\uae30\u2019\ub77c\uace0 \ubd80\ub978\ub2e4. \uc2e0\uad70\uc740 \uc704\uc548\uc2a4\uce74\uc774\uac00 \uc0c8\ub85c \ud3b8\uc131\ud55c \uc2e0\uc2dd \uc721\uad70\uc73c\ub85c \uc2e0\uad70\uc758 \uccad\ub144 \uc7a5\ubcd1 \uac00\uc6b4\ub370\uc5d0\ub294 \uc468\uc6d0\uc758 \ud601\uba85 \uc0ac\uc0c1\uc5d0 \ub3d9\uc870\ud558\ub294 \ubbfc\uc871\uc8fc\uc758\uc790\ub4e4\uc774 \ub9ce\uc558\ub2e4. \uc774\ub4e4\uc740 \uc545\uc655\ud68c\ub77c\ub294 \ube44\ubc00\ub2e8\uccb4\ub97c \ub9cc\ub4e4\uc5b4 \ud55c\uc871 \uad6d\uac00 \uac74\uc124\uc758 \uacb0\uc758\ub97c \ub2e4\uc84c\ub2e4. 1908\ub144 10\uc6d4 23\uc77c \uc545\uc655\ud68c\uc758 \uac04\ubd80\uc778 \ubc94\uc804\uac11\uc740 \uad11\uc11c\uc81c\uc640 \uc11c\ud0dc\ud6c4\uc758 \uc8fd\uc74c \uc18c\uc2dd\uc744 \ub4e3\uace0 10\uc6d4 26\uc77c\uc744 \uae30\ud558\uc5ec \ubd09\uae30\ud558\uae30\ub85c \ud558\uace0 \uac01 \uc9c4\uc601\uc758 \uac04\ubd80\uc640 \uc5f0\ub77d\ud558\uc5ec \uc6c5\uc131\uae30 \ub97c \uc0ac\ub839\uad00\uc73c\ub85c \ucd94\ub300\ud558\uc600\ub2e4. \uc774\ub807\uac8c \ud574\uc11c \uc131 \ubc16\uc5d0 \uc788\ub358 \uc2e0\uad70\uc740 \uc77c\uc81c\ud788 \ubd09\uae30\ud558\uc600\uc73c\ub098 \ud0c4\uc57d\uc758 \ubd80\uc871\uacfc \uc131 \uc548\uc758 \uad70\ub300\uac00 \ub0b4\uc751\ud558\uc9c0 \uc54a\uace0 \ud574\uad70 \ub610\ud55c \ubd09\uae30\uc5d0 \ubc18\ub300\ud588\uae30 \ub54c\ubb38\uc5d0 \uc2e4\ud328\ub85c \ub3cc\uc544\uac14\ub2e4. \uc8fc\ub3d9\uc790\uc600\ub358 \ubc94\uc804\uac11\uc740 \uccb4\ud3ec, \ucc98\ud615\ub418\uace0 \uc6c5\uc131\uae30\ub294 \uc77c\ubcf8\uc73c\ub85c \uac74\ub108\uac00 \uadf8 \uacf3\uc5d0\uc11c \uc911\uad6d\ub3d9\ub9f9\ud68c\uc5d0 \uac00\uc785\ud558\uc600\ub2e4. \uadf8 \ud6c4 \uc6c5\uc131\uae30\ub294 \ub2e4\uc2dc \uccad\ub098\ub77c\uc5d0 \uc7a0\uc785\ud558\uc5ec \uc720\ub7fd\uc5d0\uc11c \uadc0\uad6d\ud558\ub294 \uc12d\uc815\uc655\uc758 \ub450 \ub3d9\uc0dd \uc7ac\ub3c4(\u8f09\u6fe4)\uc640 \uc7ac\uc21c(\u8f09\u6d35)\uc744 \ud558\uc5bc\ube48 \uc5ed\uc5d0\uc11c \uc554\uc0b4\ud558\ub824\ub2e4 \ubc00\uace0\ub85c \uccb4\ud3ec\ub418\uc5b4 \uc0ac\ud615\uc5d0 \ucc98\ud574\uc84c\ub2e4.", "paragraph_answer": "1908\ub144 10\uc6d4 \uc548\ud718 \uc131\uc5d0\uc11c \u2018\uc2e0\uad70 \uc0ac\uac74\u2019(\u65b0\u8ecd\u4e8b\u4ef6)\uc774 \ubc1c\uc0dd\ud588\ub294\ub370, \uc774\ub97c \u2018\uc548\ud718 \uc2e0\uad70 \ubd09\uae30\u2019\ub77c\uace0 \ubd80\ub978\ub2e4. \uc2e0\uad70\uc740 \uc704\uc548\uc2a4\uce74\uc774\uac00 \uc0c8\ub85c \ud3b8\uc131\ud55c \uc2e0\uc2dd \uc721\uad70\uc73c\ub85c \uc2e0\uad70\uc758 \uccad\ub144 \uc7a5\ubcd1 \uac00\uc6b4\ub370\uc5d0\ub294 \uc468\uc6d0\uc758 \ud601\uba85 \uc0ac\uc0c1\uc5d0 \ub3d9\uc870\ud558\ub294 \ubbfc\uc871\uc8fc\uc758\uc790\ub4e4\uc774 \ub9ce\uc558\ub2e4. \uc774\ub4e4\uc740 \uc545\uc655\ud68c\ub77c\ub294 \ube44\ubc00\ub2e8\uccb4\ub97c \ub9cc\ub4e4\uc5b4 \ud55c\uc871 \uad6d\uac00 \uac74\uc124\uc758 \uacb0\uc758\ub97c \ub2e4\uc84c\ub2e4. 1908\ub144 10\uc6d4 23\uc77c \uc545\uc655\ud68c\uc758 \uac04\ubd80\uc778 \ubc94\uc804\uac11\uc740 \uad11\uc11c\uc81c\uc640 \uc11c\ud0dc\ud6c4\uc758 \uc8fd\uc74c \uc18c\uc2dd\uc744 \ub4e3\uace0 10\uc6d4 26\uc77c\uc744 \uae30\ud558\uc5ec \ubd09\uae30\ud558\uae30\ub85c \ud558\uace0 \uac01 \uc9c4\uc601\uc758 \uac04\ubd80\uc640 \uc5f0\ub77d\ud558\uc5ec \uc6c5\uc131\uae30 \ub97c \uc0ac\ub839\uad00\uc73c\ub85c \ucd94\ub300\ud558\uc600\ub2e4. \uc774\ub807\uac8c \ud574\uc11c \uc131 \ubc16\uc5d0 \uc788\ub358 \uc2e0\uad70\uc740 \uc77c\uc81c\ud788 \ubd09\uae30\ud558\uc600\uc73c\ub098 \ud0c4\uc57d\uc758 \ubd80\uc871\uacfc \uc131 \uc548\uc758 \uad70\ub300\uac00 \ub0b4\uc751\ud558\uc9c0 \uc54a\uace0 \ud574\uad70 \ub610\ud55c \ubd09\uae30\uc5d0 \ubc18\ub300\ud588\uae30 \ub54c\ubb38\uc5d0 \uc2e4\ud328\ub85c \ub3cc\uc544\uac14\ub2e4. \uc8fc\ub3d9\uc790\uc600\ub358 \ubc94\uc804\uac11\uc740 \uccb4\ud3ec, \ucc98\ud615\ub418\uace0 \uc6c5\uc131\uae30\ub294 \uc77c\ubcf8\uc73c\ub85c \uac74\ub108\uac00 \uadf8 \uacf3\uc5d0\uc11c \uc911\uad6d\ub3d9\ub9f9\ud68c\uc5d0 \uac00\uc785\ud558\uc600\ub2e4. \uadf8 \ud6c4 \uc6c5\uc131\uae30\ub294 \ub2e4\uc2dc \uccad\ub098\ub77c\uc5d0 \uc7a0\uc785\ud558\uc5ec \uc720\ub7fd\uc5d0\uc11c \uadc0\uad6d\ud558\ub294 \uc12d\uc815\uc655\uc758 \ub450 \ub3d9\uc0dd \uc7ac\ub3c4(\u8f09\u6fe4)\uc640 \uc7ac\uc21c(\u8f09\u6d35)\uc744 \ud558\uc5bc\ube48 \uc5ed\uc5d0\uc11c \uc554\uc0b4\ud558\ub824\ub2e4 \ubc00\uace0\ub85c \uccb4\ud3ec\ub418\uc5b4 \uc0ac\ud615\uc5d0 \ucc98\ud574\uc84c\ub2e4.", "sentence_answer": "1908\ub144 10\uc6d4 23\uc77c \uc545\uc655\ud68c\uc758 \uac04\ubd80\uc778 \ubc94\uc804\uac11\uc740 \uad11\uc11c\uc81c\uc640 \uc11c\ud0dc\ud6c4\uc758 \uc8fd\uc74c \uc18c\uc2dd\uc744 \ub4e3\uace0 10\uc6d4 26\uc77c\uc744 \uae30\ud558\uc5ec \ubd09\uae30\ud558\uae30\ub85c \ud558\uace0 \uac01 \uc9c4\uc601\uc758 \uac04\ubd80\uc640 \uc5f0\ub77d\ud558\uc5ec \uc6c5\uc131\uae30 \ub97c \uc0ac\ub839\uad00\uc73c\ub85c \ucd94\ub300\ud558\uc600\ub2e4.", "paragraph_id": "6533378-13-2", "paragraph_question": "question: \uccad\ub098\ub77c\uc5d0 \uc7a0\uc785\ud558\uc5ec \ud558\uc5bc\ube48 \uc5ed\uc5d0\uc11c \uc12d\uc815\uc655\uc758 \ub3d9\uc0dd\uc744 \uc554\uc0b4\ud558\ub824 \ud588\ub358 \uc0ac\ub78c\uc740?, context: 1908\ub144 10\uc6d4 \uc548\ud718 \uc131\uc5d0\uc11c \u2018\uc2e0\uad70 \uc0ac\uac74\u2019(\u65b0\u8ecd\u4e8b\u4ef6)\uc774 \ubc1c\uc0dd\ud588\ub294\ub370, \uc774\ub97c \u2018\uc548\ud718 \uc2e0\uad70 \ubd09\uae30\u2019\ub77c\uace0 \ubd80\ub978\ub2e4. \uc2e0\uad70\uc740 \uc704\uc548\uc2a4\uce74\uc774\uac00 \uc0c8\ub85c \ud3b8\uc131\ud55c \uc2e0\uc2dd \uc721\uad70\uc73c\ub85c \uc2e0\uad70\uc758 \uccad\ub144 \uc7a5\ubcd1 \uac00\uc6b4\ub370\uc5d0\ub294 \uc468\uc6d0\uc758 \ud601\uba85 \uc0ac\uc0c1\uc5d0 \ub3d9\uc870\ud558\ub294 \ubbfc\uc871\uc8fc\uc758\uc790\ub4e4\uc774 \ub9ce\uc558\ub2e4. \uc774\ub4e4\uc740 \uc545\uc655\ud68c\ub77c\ub294 \ube44\ubc00\ub2e8\uccb4\ub97c \ub9cc\ub4e4\uc5b4 \ud55c\uc871 \uad6d\uac00 \uac74\uc124\uc758 \uacb0\uc758\ub97c \ub2e4\uc84c\ub2e4. 1908\ub144 10\uc6d4 23\uc77c \uc545\uc655\ud68c\uc758 \uac04\ubd80\uc778 \ubc94\uc804\uac11\uc740 \uad11\uc11c\uc81c\uc640 \uc11c\ud0dc\ud6c4\uc758 \uc8fd\uc74c \uc18c\uc2dd\uc744 \ub4e3\uace0 10\uc6d4 26\uc77c\uc744 \uae30\ud558\uc5ec \ubd09\uae30\ud558\uae30\ub85c \ud558\uace0 \uac01 \uc9c4\uc601\uc758 \uac04\ubd80\uc640 \uc5f0\ub77d\ud558\uc5ec \uc6c5\uc131\uae30\ub97c \uc0ac\ub839\uad00\uc73c\ub85c \ucd94\ub300\ud558\uc600\ub2e4. \uc774\ub807\uac8c \ud574\uc11c \uc131 \ubc16\uc5d0 \uc788\ub358 \uc2e0\uad70\uc740 \uc77c\uc81c\ud788 \ubd09\uae30\ud558\uc600\uc73c\ub098 \ud0c4\uc57d\uc758 \ubd80\uc871\uacfc \uc131 \uc548\uc758 \uad70\ub300\uac00 \ub0b4\uc751\ud558\uc9c0 \uc54a\uace0 \ud574\uad70 \ub610\ud55c \ubd09\uae30\uc5d0 \ubc18\ub300\ud588\uae30 \ub54c\ubb38\uc5d0 \uc2e4\ud328\ub85c \ub3cc\uc544\uac14\ub2e4. \uc8fc\ub3d9\uc790\uc600\ub358 \ubc94\uc804\uac11\uc740 \uccb4\ud3ec, \ucc98\ud615\ub418\uace0 \uc6c5\uc131\uae30\ub294 \uc77c\ubcf8\uc73c\ub85c \uac74\ub108\uac00 \uadf8 \uacf3\uc5d0\uc11c \uc911\uad6d\ub3d9\ub9f9\ud68c\uc5d0 \uac00\uc785\ud558\uc600\ub2e4. \uadf8 \ud6c4 \uc6c5\uc131\uae30\ub294 \ub2e4\uc2dc \uccad\ub098\ub77c\uc5d0 \uc7a0\uc785\ud558\uc5ec \uc720\ub7fd\uc5d0\uc11c \uadc0\uad6d\ud558\ub294 \uc12d\uc815\uc655\uc758 \ub450 \ub3d9\uc0dd \uc7ac\ub3c4(\u8f09\u6fe4)\uc640 \uc7ac\uc21c(\u8f09\u6d35)\uc744 \ud558\uc5bc\ube48 \uc5ed\uc5d0\uc11c \uc554\uc0b4\ud558\ub824\ub2e4 \ubc00\uace0\ub85c \uccb4\ud3ec\ub418\uc5b4 \uc0ac\ud615\uc5d0 \ucc98\ud574\uc84c\ub2e4."} {"question": "\uccad\uc774 \uacf5\uaca9\ud558\uc5ec \ube7c\uc557\uc740 \ub7ec\uc2dc\uc544\uc758 \uc694\uc0c8\ub85c \ub7ec\uc2dc\uc544\uad70\uc774 \ub2e4\uc2dc \ub418\ucc3e\uc73c\ub824\uace0 \ud55c \uacf3\uc740?", "paragraph": "\uc774\ub807\ub4ef \uac15\ud76c\uc81c \uce58\uc138 \uc774\uc804\ubd80\ud130 \ub7ec\uc2dc\uc544\ub294 \uad70\ub300\ub97c \ub3d9\uc6d0\ud558\uc5ec \ub9cc\uc8fc\ub85c \ub0b4\ub824\uc624\uace4 \ud588\ub2e4. \ub098\uc120\uc815\ubc8c \uc774\ud6c4 30\uc5ec \ub144 \ub4a4, \ub7ec\uc2dc\uc544\ub294 \uacc4\uc18d \uccad\ub098\ub77c\uc758 \ub3d9\ubd81\ucabd\uc744 \uce68\ub7b5\ud558\uc600\uace0 \uc77c\ubd80\ub294 \ubd81\ub9cc\uc8fc\uc5d0 \uc8fc\ub454\ud55c\ub2e4. \ub7ec\uc2dc\uc544\uc778\ub4e4\uc740 \uc2dc\ubca0\ub9ac\uc544\uc640 \ub9cc\uc8fc\uc758 \ud1a0\ucc29\ubbfc\uc5d0 \ub300\ud574 \uc57d\ud0c8\uacfc \ubd80\ub140\uc790 \uac15\uac04, \uc0b4\uc778\uc744 \uc77c\uc0bc\uace4 \ud558\uc600\ub294\ub370, \ub7ec\uc2dc\uc544 \uce21\uc5d0\uc11c\ub294 \uc774\ub97c \uc8fc\ubcc0 \uc57c\ub9cc\uc778\uacfc\uc758 \uc2f8\uc6c0\uc73c\ub85c \uc5ec\uacbc\uace0, \uccad\ub098\ub77c\uc758 \ubd81\ub9cc\uc8fc \uc8fc\ubbfc\ub4e4\uc740 \uc774\ub7ec\ud55c \ub7ec\uc2dc\uc544\uc778\ub4e4\uc744 \uc0dd\uc0ac\ub78c\uc744 \uc7a1\uc544\uba39\ub294 \ub098\ucc30\uc774\ub77c \ubd88\ub800\ub2e4. \uac15\ud76c\uc81c\ub294 \uc774\uc5d0 \ubd84\ub178\ud558\uc5ec \ub7ec\uc2dc\uc544\uc5d0 \uc0ac\uacfc\ub97c \uc694\uad6c\ud558\uc600\uace0 \ub7ec\uc2dc\uc544\ub294 \uadf8\uc5d0\uac8c \uc120\ubb3c\uc744 \ubcf4\ub0b4\uba70 \ud1b5\uc0c1 \uc790\uc720\uc758 \uc758\uc0ac\ub97c \ubc1d\ud614\ub2e4. \uadf8\ub7ec\ub098 \ub7ec\uc2dc\uc544\uac00 \uacc4\uc18d \uc1a1\ud654\uac15\uae4c\uc9c0 \uc138\ub825\uc744 \ub113\ud788\ub824 \ud558\uc790, \uac15\ud76c\uc81c\ub294 \uba3c\uc800 \uad6d\uacbd\uc5d0 \uc560\ud63c\uc131(\u611b\u743f\u57ce, \uc544\uc774\ud6c8 \uc131)\uc744 \uc313\uc558\uc73c\ub098 \uacc4\uc18d \ub7ec\uc2dc\uc544\uac00 \uc57c\uc695\uc744 \ub4dc\ub7ec\ub0b4\uc790 \uadf8\uc5d0 \uac15\uacbd\ucc45\uc744 \uc368\uc11c \ub7ec\uc2dc\uc544\uc758 \uad6d\uacbd \uc694\uc0c8\uc778 \uc544\ub974\ubc14\uc9c4\uc744 \uacf5\uaca9\ud558\uc600\ub2e4. \uc544\ub974\ubc14\uc9c4\uc744 \uc810\ub839\ud558\uc600\uc73c\ub098 \ub7ec\uc2dc\uc544\uad70\uc740 \ubb3c\ub7ec\ub098\uc9c0 \uc54a\uace0 \ube8f\uae34 \uc544\ub974\ubc14\uc9c4 \uc694\uc0c8\ub97c \uc218\ubcf5\ud558\ub824 \ud558\uc600\ub2e4.", "answer": "\uc544\ub974\ubc14\uc9c4 \uc694\uc0c8", "sentence": "\uc544\ub974\ubc14\uc9c4\uc744 \uc810\ub839\ud558\uc600\uc73c\ub098 \ub7ec\uc2dc\uc544\uad70\uc740 \ubb3c\ub7ec\ub098\uc9c0 \uc54a\uace0 \ube8f\uae34 \uc544\ub974\ubc14\uc9c4 \uc694\uc0c8 \ub97c \uc218\ubcf5\ud558\ub824 \ud558\uc600\ub2e4.", "paragraph_sentence": "\uc774\ub807\ub4ef \uac15\ud76c\uc81c \uce58\uc138 \uc774\uc804\ubd80\ud130 \ub7ec\uc2dc\uc544\ub294 \uad70\ub300\ub97c \ub3d9\uc6d0\ud558\uc5ec \ub9cc\uc8fc\ub85c \ub0b4\ub824\uc624\uace4 \ud588\ub2e4. \ub098\uc120\uc815\ubc8c \uc774\ud6c4 30\uc5ec \ub144 \ub4a4, \ub7ec\uc2dc\uc544\ub294 \uacc4\uc18d \uccad\ub098\ub77c\uc758 \ub3d9\ubd81\ucabd\uc744 \uce68\ub7b5\ud558\uc600\uace0 \uc77c\ubd80\ub294 \ubd81\ub9cc\uc8fc\uc5d0 \uc8fc\ub454\ud55c\ub2e4. \ub7ec\uc2dc\uc544\uc778\ub4e4\uc740 \uc2dc\ubca0\ub9ac\uc544\uc640 \ub9cc\uc8fc\uc758 \ud1a0\ucc29\ubbfc\uc5d0 \ub300\ud574 \uc57d\ud0c8\uacfc \ubd80\ub140\uc790 \uac15\uac04, \uc0b4\uc778\uc744 \uc77c\uc0bc\uace4 \ud558\uc600\ub294\ub370, \ub7ec\uc2dc\uc544 \uce21\uc5d0\uc11c\ub294 \uc774\ub97c \uc8fc\ubcc0 \uc57c\ub9cc\uc778\uacfc\uc758 \uc2f8\uc6c0\uc73c\ub85c \uc5ec\uacbc\uace0, \uccad\ub098\ub77c\uc758 \ubd81\ub9cc\uc8fc \uc8fc\ubbfc\ub4e4\uc740 \uc774\ub7ec\ud55c \ub7ec\uc2dc\uc544\uc778\ub4e4\uc744 \uc0dd\uc0ac\ub78c\uc744 \uc7a1\uc544\uba39\ub294 \ub098\ucc30\uc774\ub77c \ubd88\ub800\ub2e4. \uac15\ud76c\uc81c\ub294 \uc774\uc5d0 \ubd84\ub178\ud558\uc5ec \ub7ec\uc2dc\uc544\uc5d0 \uc0ac\uacfc\ub97c \uc694\uad6c\ud558\uc600\uace0 \ub7ec\uc2dc\uc544\ub294 \uadf8\uc5d0\uac8c \uc120\ubb3c\uc744 \ubcf4\ub0b4\uba70 \ud1b5\uc0c1 \uc790\uc720\uc758 \uc758\uc0ac\ub97c \ubc1d\ud614\ub2e4. \uadf8\ub7ec\ub098 \ub7ec\uc2dc\uc544\uac00 \uacc4\uc18d \uc1a1\ud654\uac15\uae4c\uc9c0 \uc138\ub825\uc744 \ub113\ud788\ub824 \ud558\uc790, \uac15\ud76c\uc81c\ub294 \uba3c\uc800 \uad6d\uacbd\uc5d0 \uc560\ud63c\uc131(\u611b\u743f\u57ce, \uc544\uc774\ud6c8 \uc131)\uc744 \uc313\uc558\uc73c\ub098 \uacc4\uc18d \ub7ec\uc2dc\uc544\uac00 \uc57c\uc695\uc744 \ub4dc\ub7ec\ub0b4\uc790 \uadf8\uc5d0 \uac15\uacbd\ucc45\uc744 \uc368\uc11c \ub7ec\uc2dc\uc544\uc758 \uad6d\uacbd \uc694\uc0c8\uc778 \uc544\ub974\ubc14\uc9c4\uc744 \uacf5\uaca9\ud558\uc600\ub2e4. \uc544\ub974\ubc14\uc9c4\uc744 \uc810\ub839\ud558\uc600\uc73c\ub098 \ub7ec\uc2dc\uc544\uad70\uc740 \ubb3c\ub7ec\ub098\uc9c0 \uc54a\uace0 \ube8f\uae34 \uc544\ub974\ubc14\uc9c4 \uc694\uc0c8 \ub97c \uc218\ubcf5\ud558\ub824 \ud558\uc600\ub2e4. ", "paragraph_answer": "\uc774\ub807\ub4ef \uac15\ud76c\uc81c \uce58\uc138 \uc774\uc804\ubd80\ud130 \ub7ec\uc2dc\uc544\ub294 \uad70\ub300\ub97c \ub3d9\uc6d0\ud558\uc5ec \ub9cc\uc8fc\ub85c \ub0b4\ub824\uc624\uace4 \ud588\ub2e4. \ub098\uc120\uc815\ubc8c \uc774\ud6c4 30\uc5ec \ub144 \ub4a4, \ub7ec\uc2dc\uc544\ub294 \uacc4\uc18d \uccad\ub098\ub77c\uc758 \ub3d9\ubd81\ucabd\uc744 \uce68\ub7b5\ud558\uc600\uace0 \uc77c\ubd80\ub294 \ubd81\ub9cc\uc8fc\uc5d0 \uc8fc\ub454\ud55c\ub2e4. \ub7ec\uc2dc\uc544\uc778\ub4e4\uc740 \uc2dc\ubca0\ub9ac\uc544\uc640 \ub9cc\uc8fc\uc758 \ud1a0\ucc29\ubbfc\uc5d0 \ub300\ud574 \uc57d\ud0c8\uacfc \ubd80\ub140\uc790 \uac15\uac04, \uc0b4\uc778\uc744 \uc77c\uc0bc\uace4 \ud558\uc600\ub294\ub370, \ub7ec\uc2dc\uc544 \uce21\uc5d0\uc11c\ub294 \uc774\ub97c \uc8fc\ubcc0 \uc57c\ub9cc\uc778\uacfc\uc758 \uc2f8\uc6c0\uc73c\ub85c \uc5ec\uacbc\uace0, \uccad\ub098\ub77c\uc758 \ubd81\ub9cc\uc8fc \uc8fc\ubbfc\ub4e4\uc740 \uc774\ub7ec\ud55c \ub7ec\uc2dc\uc544\uc778\ub4e4\uc744 \uc0dd\uc0ac\ub78c\uc744 \uc7a1\uc544\uba39\ub294 \ub098\ucc30\uc774\ub77c \ubd88\ub800\ub2e4. \uac15\ud76c\uc81c\ub294 \uc774\uc5d0 \ubd84\ub178\ud558\uc5ec \ub7ec\uc2dc\uc544\uc5d0 \uc0ac\uacfc\ub97c \uc694\uad6c\ud558\uc600\uace0 \ub7ec\uc2dc\uc544\ub294 \uadf8\uc5d0\uac8c \uc120\ubb3c\uc744 \ubcf4\ub0b4\uba70 \ud1b5\uc0c1 \uc790\uc720\uc758 \uc758\uc0ac\ub97c \ubc1d\ud614\ub2e4. \uadf8\ub7ec\ub098 \ub7ec\uc2dc\uc544\uac00 \uacc4\uc18d \uc1a1\ud654\uac15\uae4c\uc9c0 \uc138\ub825\uc744 \ub113\ud788\ub824 \ud558\uc790, \uac15\ud76c\uc81c\ub294 \uba3c\uc800 \uad6d\uacbd\uc5d0 \uc560\ud63c\uc131(\u611b\u743f\u57ce, \uc544\uc774\ud6c8 \uc131)\uc744 \uc313\uc558\uc73c\ub098 \uacc4\uc18d \ub7ec\uc2dc\uc544\uac00 \uc57c\uc695\uc744 \ub4dc\ub7ec\ub0b4\uc790 \uadf8\uc5d0 \uac15\uacbd\ucc45\uc744 \uc368\uc11c \ub7ec\uc2dc\uc544\uc758 \uad6d\uacbd \uc694\uc0c8\uc778 \uc544\ub974\ubc14\uc9c4\uc744 \uacf5\uaca9\ud558\uc600\ub2e4. \uc544\ub974\ubc14\uc9c4\uc744 \uc810\ub839\ud558\uc600\uc73c\ub098 \ub7ec\uc2dc\uc544\uad70\uc740 \ubb3c\ub7ec\ub098\uc9c0 \uc54a\uace0 \ube8f\uae34 \uc544\ub974\ubc14\uc9c4 \uc694\uc0c8 \ub97c \uc218\ubcf5\ud558\ub824 \ud558\uc600\ub2e4.", "sentence_answer": "\uc544\ub974\ubc14\uc9c4\uc744 \uc810\ub839\ud558\uc600\uc73c\ub098 \ub7ec\uc2dc\uc544\uad70\uc740 \ubb3c\ub7ec\ub098\uc9c0 \uc54a\uace0 \ube8f\uae34 \uc544\ub974\ubc14\uc9c4 \uc694\uc0c8 \ub97c \uc218\ubcf5\ud558\ub824 \ud558\uc600\ub2e4.", "paragraph_id": "6387920-19-1", "paragraph_question": "question: \uccad\uc774 \uacf5\uaca9\ud558\uc5ec \ube7c\uc557\uc740 \ub7ec\uc2dc\uc544\uc758 \uc694\uc0c8\ub85c \ub7ec\uc2dc\uc544\uad70\uc774 \ub2e4\uc2dc \ub418\ucc3e\uc73c\ub824\uace0 \ud55c \uacf3\uc740?, context: \uc774\ub807\ub4ef \uac15\ud76c\uc81c \uce58\uc138 \uc774\uc804\ubd80\ud130 \ub7ec\uc2dc\uc544\ub294 \uad70\ub300\ub97c \ub3d9\uc6d0\ud558\uc5ec \ub9cc\uc8fc\ub85c \ub0b4\ub824\uc624\uace4 \ud588\ub2e4. \ub098\uc120\uc815\ubc8c \uc774\ud6c4 30\uc5ec \ub144 \ub4a4, \ub7ec\uc2dc\uc544\ub294 \uacc4\uc18d \uccad\ub098\ub77c\uc758 \ub3d9\ubd81\ucabd\uc744 \uce68\ub7b5\ud558\uc600\uace0 \uc77c\ubd80\ub294 \ubd81\ub9cc\uc8fc\uc5d0 \uc8fc\ub454\ud55c\ub2e4. \ub7ec\uc2dc\uc544\uc778\ub4e4\uc740 \uc2dc\ubca0\ub9ac\uc544\uc640 \ub9cc\uc8fc\uc758 \ud1a0\ucc29\ubbfc\uc5d0 \ub300\ud574 \uc57d\ud0c8\uacfc \ubd80\ub140\uc790 \uac15\uac04, \uc0b4\uc778\uc744 \uc77c\uc0bc\uace4 \ud558\uc600\ub294\ub370, \ub7ec\uc2dc\uc544 \uce21\uc5d0\uc11c\ub294 \uc774\ub97c \uc8fc\ubcc0 \uc57c\ub9cc\uc778\uacfc\uc758 \uc2f8\uc6c0\uc73c\ub85c \uc5ec\uacbc\uace0, \uccad\ub098\ub77c\uc758 \ubd81\ub9cc\uc8fc \uc8fc\ubbfc\ub4e4\uc740 \uc774\ub7ec\ud55c \ub7ec\uc2dc\uc544\uc778\ub4e4\uc744 \uc0dd\uc0ac\ub78c\uc744 \uc7a1\uc544\uba39\ub294 \ub098\ucc30\uc774\ub77c \ubd88\ub800\ub2e4. \uac15\ud76c\uc81c\ub294 \uc774\uc5d0 \ubd84\ub178\ud558\uc5ec \ub7ec\uc2dc\uc544\uc5d0 \uc0ac\uacfc\ub97c \uc694\uad6c\ud558\uc600\uace0 \ub7ec\uc2dc\uc544\ub294 \uadf8\uc5d0\uac8c \uc120\ubb3c\uc744 \ubcf4\ub0b4\uba70 \ud1b5\uc0c1 \uc790\uc720\uc758 \uc758\uc0ac\ub97c \ubc1d\ud614\ub2e4. \uadf8\ub7ec\ub098 \ub7ec\uc2dc\uc544\uac00 \uacc4\uc18d \uc1a1\ud654\uac15\uae4c\uc9c0 \uc138\ub825\uc744 \ub113\ud788\ub824 \ud558\uc790, \uac15\ud76c\uc81c\ub294 \uba3c\uc800 \uad6d\uacbd\uc5d0 \uc560\ud63c\uc131(\u611b\u743f\u57ce, \uc544\uc774\ud6c8 \uc131)\uc744 \uc313\uc558\uc73c\ub098 \uacc4\uc18d \ub7ec\uc2dc\uc544\uac00 \uc57c\uc695\uc744 \ub4dc\ub7ec\ub0b4\uc790 \uadf8\uc5d0 \uac15\uacbd\ucc45\uc744 \uc368\uc11c \ub7ec\uc2dc\uc544\uc758 \uad6d\uacbd \uc694\uc0c8\uc778 \uc544\ub974\ubc14\uc9c4\uc744 \uacf5\uaca9\ud558\uc600\ub2e4. \uc544\ub974\ubc14\uc9c4\uc744 \uc810\ub839\ud558\uc600\uc73c\ub098 \ub7ec\uc2dc\uc544\uad70\uc740 \ubb3c\ub7ec\ub098\uc9c0 \uc54a\uace0 \ube8f\uae34 \uc544\ub974\ubc14\uc9c4 \uc694\uc0c8\ub97c \uc218\ubcf5\ud558\ub824 \ud558\uc600\ub2e4."} {"question": "\ub0a8\ub140\ub178\uc18c, \ube48\ubd80\uadc0\ucc9c \uac00\ub9ac\uc9c0 \uc54a\uace0 \uc11c\ub85c\ub97c \ud3c9\ub4f1\ud558\uac8c \ub300\uc6b0\ud558\uace0 \ud558\ub298\ucc98\ub7fc \uc12c\uae30\ub294 \ud0dc\ub3c4 \ubc0f \uc0ac\uc0c1\uc744 \ubb34\uc5c7\uc774\ub77c\uace0 \ud558\ub294\uac00?", "paragraph": "\ud55c\uad6d\uc5d0\uc11c\ub3c4 \uc591\ubc18\uacfc \uc0c1\ubbfc\u00b7\ub178\ube44\ub85c \uac08\ub824 \uc18c\uc218\uc758 \uc591\ubc18\uce35\uc774 \ubbfc\uc911\uc744 \uac00\ud639\ud558\uac8c \uc5b5\uc555,\ucc29\ucde8\ud558\ub358 \uc57c\ub9cc\uc2dc\ub300\ub97c \ub3d9\ud559\uc0ac\uc0c1\uc758 \ub300\ub450\ub85c \ud601\uba85\uc774 \ubc1c\ubc1c\ud558\uc5ec \uc138\uc0c1\uc774 \ub4a4\ubc14\ub00c\uae30 \uc2dc\uc791\ud588\ub2e4.\ub3d9\ud559\ub3c4\ub4e4\ub07c\ub9ac\ub294 \ub0a8\ub140\ub178\uc18c,\ube48\ubd80\uadc0\ucc9c \uac00\ub9ac\uc9c0 \uc54a\uace0 \uc11c\ub85c\ub97c \ud3c9\ub4f1\ud558\uac8c \ub300\uc6b0\ud558\uace0 \ud558\ub298\ucc98\ub7fc \uc12c\uacbc\ub2e4(\uc0ac\uc778\uc5ec\ucc9c,\uc778\ub0b4\ucc9c).\uadf8\ub7ec\ub098 \uc2dd\ubbfc\uae30\ub97c \uac70\uce58\uba74\uc11c \ud3c9\ub4f1\uc0ac\uc0c1\uc774 \ud76c\ubc15\ud574\uc84c\uace0 \uc624\ub79c \ub3c5\uc7ac\uc815\uce58\ub85c \uc644\uc804\ud788 \ud3c9\ub4f1\uc0ac\uc0c1\uc740 \uc0ac\ub77c\uc84c\ub2e4.\uadf8\ub9ac\uace0 5,18\uad11\uc8fc\ud56d\uc7c1\uacfc 87\ub144 6\uc6d4\ubbfc\uc911\ud56d\uc7c1\uc73c\ub85c \ubbfc\uc8fc\uc8fc\uc758\uac00 \ubd80\ud65c\ud568\uc73c\ub85c\uc368 \ud3c9\ub4f1\uc0ac\uc0c1\ub3c4 \uaca8\uc6b0 \ube5b\uc740 \ubcf4\uac8c \ub418\uc5c8\ub2e4.\uadf8\ub7ec\ub098 \uc774\ub7f0 \ub3c4\ub3c4\ud55c \ud3c9\ub4f1\uc758 \uc138\uacc4\uc0ac\uc640 \uc0dd\uc0dd\ud55c \ubbfc\uc871\uc801 \uccb4\ud5d8\uc774 \uc788\uc5c8\uc74c\uc5d0\ub3c4,\ud604\uc7ac \ud55c\uad6d\uc0ac\ud68c\ub294 \uc804\ud600 \ud3c9\ub4f1\ud558\uc9c0\uac00 \uc54a\ub2e4.\uacbd\uc81c,\uad70\uc0ac\uc801\uc73c\ub85c\ub294 \uc120\uc9c4\uc744 \ub118\uc5b4 \uc77c\ub958\uad6d\uac00\ub97c \ubd80\ub974\uc9d6\uc73c\uba74\uc11c\ub3c4 \uc778\uac04\uc758 \uc0c1\ub2f9\ud55c \uacbd\uc81c\uc801 \ud3c9\ub4f1\uc740 \uace0\uc0ac\ud558\uace0 \ud3c9\ub4f1\uc758 \ucd08\ubcf4\ub2e8\uacc4\uc778 \ubc95\uc801 \uc815\uce58\uad8c\ub825\uc801 \ud3c9\ub4f1\ub3c4 \uc815\ucc29\uc2dc\ud0a4\uc9c0 \ubabb\ud558\uace0 \uc788\ub2e4.\uc0ac\ud68c\ubcf5\uc9c0 \uc218\uc900,\uc0ac\ud68c\uad6c\uc131\uc6d0\uc758 \ub300\ub2e4\uc218\uc778 \ub178\ub3d9\uc790\uc758 \uc9c0\uc704\uc640 \uad8c\ud55c,\uc5ec\uc131\uc758 \uad8c\ub9ac \ub4f1\uc5d0 \uc788\uc5b4 \uc77c\ub958\ub294\ucee4\ub155 \uac70\uc758 \uc911\ud6c4\uc9c4\uad6d \uc218\uc900\uc744 \ub9f4\ub3cc\uace0 \uc788\ub294 \uac83\uc774\ub2e4.\uacbd\uc81c,\uad70\uc0ac \uc120\uc9c4\uad6d\ub2f5\uac8c \uc815\uce58\u00b7\uacbd\uc81c\uc801 \ud3c9\ub4f1\uc218\uc900\uacfc \uc0ac\ub78c\ub4e4 \ubaa8\ub450\uc758 \uad8c\uc775 \ubcf4\uc7a5\ub3c4 \uc120\uc9c4\uad6d\ub2e4\uc6cc\uc57c \uc815\uc0c1\uc774\uc9c0 \uc774\ub807\uac8c \ubd88\uade0\ud615\ud55c \uae30\ud615\uc801\uc778 \ub098\ub77c\uc5d0\uc11c \ubb34\uc2a8 \ud76c\ub9dd\uacfc \ubbff\uc74c\uc774 \uc788\uaca0\ub294\uac00!", "answer": "\uc0ac\uc778\uc5ec\ucc9c,\uc778\ub0b4\ucc9c", "sentence": "\ud55c\uad6d\uc5d0\uc11c\ub3c4 \uc591\ubc18\uacfc \uc0c1\ubbfc\u00b7\ub178\ube44\ub85c \uac08\ub824 \uc18c\uc218\uc758 \uc591\ubc18\uce35\uc774 \ubbfc\uc911\uc744 \uac00\ud639\ud558\uac8c \uc5b5\uc555,\ucc29\ucde8\ud558\ub358 \uc57c\ub9cc\uc2dc\ub300\ub97c \ub3d9\ud559\uc0ac\uc0c1\uc758 \ub300\ub450\ub85c \ud601\uba85\uc774 \ubc1c\ubc1c\ud558\uc5ec \uc138\uc0c1\uc774 \ub4a4\ubc14\ub00c\uae30 \uc2dc\uc791\ud588\ub2e4.\ub3d9\ud559\ub3c4\ub4e4\ub07c\ub9ac\ub294 \ub0a8\ub140\ub178\uc18c,\ube48\ubd80\uadc0\ucc9c \uac00\ub9ac\uc9c0 \uc54a\uace0 \uc11c\ub85c\ub97c \ud3c9\ub4f1\ud558\uac8c \ub300\uc6b0\ud558\uace0 \ud558\ub298\ucc98\ub7fc \uc12c\uacbc\ub2e4( \uc0ac\uc778\uc5ec\ucc9c,\uc778\ub0b4\ucc9c )", "paragraph_sentence": " \ud55c\uad6d\uc5d0\uc11c\ub3c4 \uc591\ubc18\uacfc \uc0c1\ubbfc\u00b7\ub178\ube44\ub85c \uac08\ub824 \uc18c\uc218\uc758 \uc591\ubc18\uce35\uc774 \ubbfc\uc911\uc744 \uac00\ud639\ud558\uac8c \uc5b5\uc555,\ucc29\ucde8\ud558\ub358 \uc57c\ub9cc\uc2dc\ub300\ub97c \ub3d9\ud559\uc0ac\uc0c1\uc758 \ub300\ub450\ub85c \ud601\uba85\uc774 \ubc1c\ubc1c\ud558\uc5ec \uc138\uc0c1\uc774 \ub4a4\ubc14\ub00c\uae30 \uc2dc\uc791\ud588\ub2e4.\ub3d9\ud559\ub3c4\ub4e4\ub07c\ub9ac\ub294 \ub0a8\ub140\ub178\uc18c,\ube48\ubd80\uadc0\ucc9c \uac00\ub9ac\uc9c0 \uc54a\uace0 \uc11c\ub85c\ub97c \ud3c9\ub4f1\ud558\uac8c \ub300\uc6b0\ud558\uace0 \ud558\ub298\ucc98\ub7fc \uc12c\uacbc\ub2e4( \uc0ac\uc778\uc5ec\ucc9c,\uc778\ub0b4\ucc9c ) .\uadf8\ub7ec\ub098 \uc2dd\ubbfc\uae30\ub97c \uac70\uce58\uba74\uc11c \ud3c9\ub4f1\uc0ac\uc0c1\uc774 \ud76c\ubc15\ud574\uc84c\uace0 \uc624\ub79c \ub3c5\uc7ac\uc815\uce58\ub85c \uc644\uc804\ud788 \ud3c9\ub4f1\uc0ac\uc0c1\uc740 \uc0ac\ub77c\uc84c\ub2e4.\uadf8\ub9ac\uace0 5,18\uad11\uc8fc\ud56d\uc7c1\uacfc 87\ub144 6\uc6d4\ubbfc\uc911\ud56d\uc7c1\uc73c\ub85c \ubbfc\uc8fc\uc8fc\uc758\uac00 \ubd80\ud65c\ud568\uc73c\ub85c\uc368 \ud3c9\ub4f1\uc0ac\uc0c1\ub3c4 \uaca8\uc6b0 \ube5b\uc740 \ubcf4\uac8c \ub418\uc5c8\ub2e4.\uadf8\ub7ec\ub098 \uc774\ub7f0 \ub3c4\ub3c4\ud55c \ud3c9\ub4f1\uc758 \uc138\uacc4\uc0ac\uc640 \uc0dd\uc0dd\ud55c \ubbfc\uc871\uc801 \uccb4\ud5d8\uc774 \uc788\uc5c8\uc74c\uc5d0\ub3c4,\ud604\uc7ac \ud55c\uad6d\uc0ac\ud68c\ub294 \uc804\ud600 \ud3c9\ub4f1\ud558\uc9c0\uac00 \uc54a\ub2e4. \uacbd\uc81c,\uad70\uc0ac\uc801\uc73c\ub85c\ub294 \uc120\uc9c4\uc744 \ub118\uc5b4 \uc77c\ub958\uad6d\uac00\ub97c \ubd80\ub974\uc9d6\uc73c\uba74\uc11c\ub3c4 \uc778\uac04\uc758 \uc0c1\ub2f9\ud55c \uacbd\uc81c\uc801 \ud3c9\ub4f1\uc740 \uace0\uc0ac\ud558\uace0 \ud3c9\ub4f1\uc758 \ucd08\ubcf4\ub2e8\uacc4\uc778 \ubc95\uc801 \uc815\uce58\uad8c\ub825\uc801 \ud3c9\ub4f1\ub3c4 \uc815\ucc29\uc2dc\ud0a4\uc9c0 \ubabb\ud558\uace0 \uc788\ub2e4.\uc0ac\ud68c\ubcf5\uc9c0 \uc218\uc900,\uc0ac\ud68c\uad6c\uc131\uc6d0\uc758 \ub300\ub2e4\uc218\uc778 \ub178\ub3d9\uc790\uc758 \uc9c0\uc704\uc640 \uad8c\ud55c,\uc5ec\uc131\uc758 \uad8c\ub9ac \ub4f1\uc5d0 \uc788\uc5b4 \uc77c\ub958\ub294\ucee4\ub155 \uac70\uc758 \uc911\ud6c4\uc9c4\uad6d \uc218\uc900\uc744 \ub9f4\ub3cc\uace0 \uc788\ub294 \uac83\uc774\ub2e4. \uacbd\uc81c,\uad70\uc0ac \uc120\uc9c4\uad6d\ub2f5\uac8c \uc815\uce58\u00b7\uacbd\uc81c\uc801 \ud3c9\ub4f1\uc218\uc900\uacfc \uc0ac\ub78c\ub4e4 \ubaa8\ub450\uc758 \uad8c\uc775 \ubcf4\uc7a5\ub3c4 \uc120\uc9c4\uad6d\ub2e4\uc6cc\uc57c \uc815\uc0c1\uc774\uc9c0 \uc774\ub807\uac8c \ubd88\uade0\ud615\ud55c \uae30\ud615\uc801\uc778 \ub098\ub77c\uc5d0\uc11c \ubb34\uc2a8 \ud76c\ub9dd\uacfc \ubbff\uc74c\uc774 \uc788\uaca0\ub294\uac00!", "paragraph_answer": "\ud55c\uad6d\uc5d0\uc11c\ub3c4 \uc591\ubc18\uacfc \uc0c1\ubbfc\u00b7\ub178\ube44\ub85c \uac08\ub824 \uc18c\uc218\uc758 \uc591\ubc18\uce35\uc774 \ubbfc\uc911\uc744 \uac00\ud639\ud558\uac8c \uc5b5\uc555,\ucc29\ucde8\ud558\ub358 \uc57c\ub9cc\uc2dc\ub300\ub97c \ub3d9\ud559\uc0ac\uc0c1\uc758 \ub300\ub450\ub85c \ud601\uba85\uc774 \ubc1c\ubc1c\ud558\uc5ec \uc138\uc0c1\uc774 \ub4a4\ubc14\ub00c\uae30 \uc2dc\uc791\ud588\ub2e4.\ub3d9\ud559\ub3c4\ub4e4\ub07c\ub9ac\ub294 \ub0a8\ub140\ub178\uc18c,\ube48\ubd80\uadc0\ucc9c \uac00\ub9ac\uc9c0 \uc54a\uace0 \uc11c\ub85c\ub97c \ud3c9\ub4f1\ud558\uac8c \ub300\uc6b0\ud558\uace0 \ud558\ub298\ucc98\ub7fc \uc12c\uacbc\ub2e4( \uc0ac\uc778\uc5ec\ucc9c,\uc778\ub0b4\ucc9c ).\uadf8\ub7ec\ub098 \uc2dd\ubbfc\uae30\ub97c \uac70\uce58\uba74\uc11c \ud3c9\ub4f1\uc0ac\uc0c1\uc774 \ud76c\ubc15\ud574\uc84c\uace0 \uc624\ub79c \ub3c5\uc7ac\uc815\uce58\ub85c \uc644\uc804\ud788 \ud3c9\ub4f1\uc0ac\uc0c1\uc740 \uc0ac\ub77c\uc84c\ub2e4.\uadf8\ub9ac\uace0 5,18\uad11\uc8fc\ud56d\uc7c1\uacfc 87\ub144 6\uc6d4\ubbfc\uc911\ud56d\uc7c1\uc73c\ub85c \ubbfc\uc8fc\uc8fc\uc758\uac00 \ubd80\ud65c\ud568\uc73c\ub85c\uc368 \ud3c9\ub4f1\uc0ac\uc0c1\ub3c4 \uaca8\uc6b0 \ube5b\uc740 \ubcf4\uac8c \ub418\uc5c8\ub2e4.\uadf8\ub7ec\ub098 \uc774\ub7f0 \ub3c4\ub3c4\ud55c \ud3c9\ub4f1\uc758 \uc138\uacc4\uc0ac\uc640 \uc0dd\uc0dd\ud55c \ubbfc\uc871\uc801 \uccb4\ud5d8\uc774 \uc788\uc5c8\uc74c\uc5d0\ub3c4,\ud604\uc7ac \ud55c\uad6d\uc0ac\ud68c\ub294 \uc804\ud600 \ud3c9\ub4f1\ud558\uc9c0\uac00 \uc54a\ub2e4.\uacbd\uc81c,\uad70\uc0ac\uc801\uc73c\ub85c\ub294 \uc120\uc9c4\uc744 \ub118\uc5b4 \uc77c\ub958\uad6d\uac00\ub97c \ubd80\ub974\uc9d6\uc73c\uba74\uc11c\ub3c4 \uc778\uac04\uc758 \uc0c1\ub2f9\ud55c \uacbd\uc81c\uc801 \ud3c9\ub4f1\uc740 \uace0\uc0ac\ud558\uace0 \ud3c9\ub4f1\uc758 \ucd08\ubcf4\ub2e8\uacc4\uc778 \ubc95\uc801 \uc815\uce58\uad8c\ub825\uc801 \ud3c9\ub4f1\ub3c4 \uc815\ucc29\uc2dc\ud0a4\uc9c0 \ubabb\ud558\uace0 \uc788\ub2e4.\uc0ac\ud68c\ubcf5\uc9c0 \uc218\uc900,\uc0ac\ud68c\uad6c\uc131\uc6d0\uc758 \ub300\ub2e4\uc218\uc778 \ub178\ub3d9\uc790\uc758 \uc9c0\uc704\uc640 \uad8c\ud55c,\uc5ec\uc131\uc758 \uad8c\ub9ac \ub4f1\uc5d0 \uc788\uc5b4 \uc77c\ub958\ub294\ucee4\ub155 \uac70\uc758 \uc911\ud6c4\uc9c4\uad6d \uc218\uc900\uc744 \ub9f4\ub3cc\uace0 \uc788\ub294 \uac83\uc774\ub2e4.\uacbd\uc81c,\uad70\uc0ac \uc120\uc9c4\uad6d\ub2f5\uac8c \uc815\uce58\u00b7\uacbd\uc81c\uc801 \ud3c9\ub4f1\uc218\uc900\uacfc \uc0ac\ub78c\ub4e4 \ubaa8\ub450\uc758 \uad8c\uc775 \ubcf4\uc7a5\ub3c4 \uc120\uc9c4\uad6d\ub2e4\uc6cc\uc57c \uc815\uc0c1\uc774\uc9c0 \uc774\ub807\uac8c \ubd88\uade0\ud615\ud55c \uae30\ud615\uc801\uc778 \ub098\ub77c\uc5d0\uc11c \ubb34\uc2a8 \ud76c\ub9dd\uacfc \ubbff\uc74c\uc774 \uc788\uaca0\ub294\uac00!", "sentence_answer": "\ud55c\uad6d\uc5d0\uc11c\ub3c4 \uc591\ubc18\uacfc \uc0c1\ubbfc\u00b7\ub178\ube44\ub85c \uac08\ub824 \uc18c\uc218\uc758 \uc591\ubc18\uce35\uc774 \ubbfc\uc911\uc744 \uac00\ud639\ud558\uac8c \uc5b5\uc555,\ucc29\ucde8\ud558\ub358 \uc57c\ub9cc\uc2dc\ub300\ub97c \ub3d9\ud559\uc0ac\uc0c1\uc758 \ub300\ub450\ub85c \ud601\uba85\uc774 \ubc1c\ubc1c\ud558\uc5ec \uc138\uc0c1\uc774 \ub4a4\ubc14\ub00c\uae30 \uc2dc\uc791\ud588\ub2e4.\ub3d9\ud559\ub3c4\ub4e4\ub07c\ub9ac\ub294 \ub0a8\ub140\ub178\uc18c,\ube48\ubd80\uadc0\ucc9c \uac00\ub9ac\uc9c0 \uc54a\uace0 \uc11c\ub85c\ub97c \ud3c9\ub4f1\ud558\uac8c \ub300\uc6b0\ud558\uace0 \ud558\ub298\ucc98\ub7fc \uc12c\uacbc\ub2e4( \uc0ac\uc778\uc5ec\ucc9c,\uc778\ub0b4\ucc9c )", "paragraph_id": "6511009-4-1", "paragraph_question": "question: \ub0a8\ub140\ub178\uc18c, \ube48\ubd80\uadc0\ucc9c \uac00\ub9ac\uc9c0 \uc54a\uace0 \uc11c\ub85c\ub97c \ud3c9\ub4f1\ud558\uac8c \ub300\uc6b0\ud558\uace0 \ud558\ub298\ucc98\ub7fc \uc12c\uae30\ub294 \ud0dc\ub3c4 \ubc0f \uc0ac\uc0c1\uc744 \ubb34\uc5c7\uc774\ub77c\uace0 \ud558\ub294\uac00?, context: \ud55c\uad6d\uc5d0\uc11c\ub3c4 \uc591\ubc18\uacfc \uc0c1\ubbfc\u00b7\ub178\ube44\ub85c \uac08\ub824 \uc18c\uc218\uc758 \uc591\ubc18\uce35\uc774 \ubbfc\uc911\uc744 \uac00\ud639\ud558\uac8c \uc5b5\uc555,\ucc29\ucde8\ud558\ub358 \uc57c\ub9cc\uc2dc\ub300\ub97c \ub3d9\ud559\uc0ac\uc0c1\uc758 \ub300\ub450\ub85c \ud601\uba85\uc774 \ubc1c\ubc1c\ud558\uc5ec \uc138\uc0c1\uc774 \ub4a4\ubc14\ub00c\uae30 \uc2dc\uc791\ud588\ub2e4.\ub3d9\ud559\ub3c4\ub4e4\ub07c\ub9ac\ub294 \ub0a8\ub140\ub178\uc18c,\ube48\ubd80\uadc0\ucc9c \uac00\ub9ac\uc9c0 \uc54a\uace0 \uc11c\ub85c\ub97c \ud3c9\ub4f1\ud558\uac8c \ub300\uc6b0\ud558\uace0 \ud558\ub298\ucc98\ub7fc \uc12c\uacbc\ub2e4(\uc0ac\uc778\uc5ec\ucc9c,\uc778\ub0b4\ucc9c).\uadf8\ub7ec\ub098 \uc2dd\ubbfc\uae30\ub97c \uac70\uce58\uba74\uc11c \ud3c9\ub4f1\uc0ac\uc0c1\uc774 \ud76c\ubc15\ud574\uc84c\uace0 \uc624\ub79c \ub3c5\uc7ac\uc815\uce58\ub85c \uc644\uc804\ud788 \ud3c9\ub4f1\uc0ac\uc0c1\uc740 \uc0ac\ub77c\uc84c\ub2e4.\uadf8\ub9ac\uace0 5,18\uad11\uc8fc\ud56d\uc7c1\uacfc 87\ub144 6\uc6d4\ubbfc\uc911\ud56d\uc7c1\uc73c\ub85c \ubbfc\uc8fc\uc8fc\uc758\uac00 \ubd80\ud65c\ud568\uc73c\ub85c\uc368 \ud3c9\ub4f1\uc0ac\uc0c1\ub3c4 \uaca8\uc6b0 \ube5b\uc740 \ubcf4\uac8c \ub418\uc5c8\ub2e4.\uadf8\ub7ec\ub098 \uc774\ub7f0 \ub3c4\ub3c4\ud55c \ud3c9\ub4f1\uc758 \uc138\uacc4\uc0ac\uc640 \uc0dd\uc0dd\ud55c \ubbfc\uc871\uc801 \uccb4\ud5d8\uc774 \uc788\uc5c8\uc74c\uc5d0\ub3c4,\ud604\uc7ac \ud55c\uad6d\uc0ac\ud68c\ub294 \uc804\ud600 \ud3c9\ub4f1\ud558\uc9c0\uac00 \uc54a\ub2e4.\uacbd\uc81c,\uad70\uc0ac\uc801\uc73c\ub85c\ub294 \uc120\uc9c4\uc744 \ub118\uc5b4 \uc77c\ub958\uad6d\uac00\ub97c \ubd80\ub974\uc9d6\uc73c\uba74\uc11c\ub3c4 \uc778\uac04\uc758 \uc0c1\ub2f9\ud55c \uacbd\uc81c\uc801 \ud3c9\ub4f1\uc740 \uace0\uc0ac\ud558\uace0 \ud3c9\ub4f1\uc758 \ucd08\ubcf4\ub2e8\uacc4\uc778 \ubc95\uc801 \uc815\uce58\uad8c\ub825\uc801 \ud3c9\ub4f1\ub3c4 \uc815\ucc29\uc2dc\ud0a4\uc9c0 \ubabb\ud558\uace0 \uc788\ub2e4.\uc0ac\ud68c\ubcf5\uc9c0 \uc218\uc900,\uc0ac\ud68c\uad6c\uc131\uc6d0\uc758 \ub300\ub2e4\uc218\uc778 \ub178\ub3d9\uc790\uc758 \uc9c0\uc704\uc640 \uad8c\ud55c,\uc5ec\uc131\uc758 \uad8c\ub9ac \ub4f1\uc5d0 \uc788\uc5b4 \uc77c\ub958\ub294\ucee4\ub155 \uac70\uc758 \uc911\ud6c4\uc9c4\uad6d \uc218\uc900\uc744 \ub9f4\ub3cc\uace0 \uc788\ub294 \uac83\uc774\ub2e4.\uacbd\uc81c,\uad70\uc0ac \uc120\uc9c4\uad6d\ub2f5\uac8c \uc815\uce58\u00b7\uacbd\uc81c\uc801 \ud3c9\ub4f1\uc218\uc900\uacfc \uc0ac\ub78c\ub4e4 \ubaa8\ub450\uc758 \uad8c\uc775 \ubcf4\uc7a5\ub3c4 \uc120\uc9c4\uad6d\ub2e4\uc6cc\uc57c \uc815\uc0c1\uc774\uc9c0 \uc774\ub807\uac8c \ubd88\uade0\ud615\ud55c \uae30\ud615\uc801\uc778 \ub098\ub77c\uc5d0\uc11c \ubb34\uc2a8 \ud76c\ub9dd\uacfc \ubbff\uc74c\uc774 \uc788\uaca0\ub294\uac00!"} {"question": "\uc2dc\ubcfc\ub4dc\uac00 \uba74\ub2f4 \ub0b4\uc6a9\uc744 \ubbf8\uad6d \uad6d\ubb34\ubd80\uc5d0 \ubcf4\uace0\ud55c \ub0a0\uc9dc\ub294?", "paragraph": "1948\ub144 8\uc6d4\uc5d0 \ub300\ud55c\ubbfc\uad6d \uc815\ubd80\uac00 \uc218\ub9bd\ub418\uc790 \uc774\uc740\uc740 \uc8fc\uc77c \ub300\ud55c\ubbfc\uad6d \ub300\ud45c\ubd80\ub97c \ud1b5\ud574\uc11c \uc5ec\ub7ec \ucc28\ub840 \uadc0\uad6d \uc758\uc0ac\ub97c \ud0c0\uc9c4\ud558\uc9c0\ub9cc \ub300\ud55c\ubbfc\uad6d \uc815\ubd80\ub294 \ub69c\ub837\ud55c \ubc18\uc751\uc744 \ubcf4\uc774\uc9c0 \uc54a\uc558\ub2e4. \uc624\ud788\ub824 \uc774\uc740\uc758 \uac1c\uc778 \uc7ac\uc0b0\uc778 \ub3c4\ucfc4 \uc800\ud0dd\uc744 \uad6d\uc720\uc7ac\uc0b0\uc774\ub2c8 \uc8fc\uc77c \ub300\ud45c\ubd80 \uac74\ubb3c\ub85c \uc0ac\uc6a9\ud558\uaca0\ub2e4\uba70 \ub0b4\uc5b4\ub193\uc73c\ub77c\ub294 \ud6c8\ub839\uc744 \ubcf4\ub0b4\uc654\ub2e4. \uac19\uc740 \ud574 \uc774\uc740\uc740 \uc5f0\ud569\uad70 \ucd5c\uace0\uc0ac\ub839\ubd80 \uc815\uce58 \uace0\ubb38 \uc70c\ub9ac\uc5c4 \uc2dc\ubcfc\ub4dc\uc640 \uba74\ub2f4\uc744 \ud558\uc5ec \uc790\uc2e0\uc758 \uad6d\uc801\uc5d0 \ub300\ud55c \uc720\uad8c \ud574\uc11d\uc744 \uc694\uccad\ud558\uc600\ub2e4. \uc774\uc740\uc740 \uc2dc\ubcfc\ub4dc\uc5d0\uac8c \ub300\ud55c\ubbfc\uad6d \ub0b4\uc758 \uc7ac\uc0b0\uc740 \ub300\ud55c\ubbfc\uad6d \uc815\ubd80\uc5d0 \uae30\uc99d\ud558\uace0, \uc77c\ubcf8 \ub0b4\uc758 \uc7ac\uc0b0\uc740 \uc0dd\uacc4\ub97c \uc704\ud574 \uc18c\uc720\uad8c\uc744 \ud589\uc0ac\ud558\uace0 \uc2f6\uc740\ub370 \ubc95\uc801\uc73c\ub85c \uac00\ub2a5\ud55c\uc9c0\ub97c \uc9c8\ubb38\ud558\uc600\uc73c\uba70, \ubbf8\uad6d\uc5d0 \uccb4\ub958\ud558\uac70\ub098 \uc601\uc8fc\uad8c\uc744 \uc5bb\uc5b4 \ucc28\ub0a8 \uc774\uad6c\ub97c \ubbf8\uad6d\uc5d0\uc11c \uad50\uc721\uc2dc\ud0a4\uace0 \uc2f6\ub2e4\ub294 \ub73b\uc744 \ubc1d\ud614\ub2e4. \ub610\ud55c, \uc774\uc740\uc740 \ub300\ud55c\ubbfc\uad6d\uc5d0\uc11c\uc758 \uc815\uce58\uc801 \ubaa9\uc801\uc744 \uc704\ud574 \uc774\uc6a9\ub418\uace0 \uc2f6\uc9c0 \uc54a\ub2e4\uace0 \ub9d0\ud558\uc600\ub2e4. \uc2dc\ubcfc\ub4dc\ub294 \uba74\ub2f4 \ub0b4\uc6a9\uc744 10\uc6d4 12\uc77c\uc5d0 \ubbf8\uad6d \uad6d\ubb34\ubd80\uc5d0 \ubcf4\uace0\ud558\uc600\ub2e4. \uc2dc\ubcfc\ub4dc\ub294 \uc601\uce5c\uc655\uc740 \uacb0\ud63c\uc744 \ud1b5\ud574 \uc77c\ubcf8 \ud669\uc2e4\uc758 \uc77c\uc6d0\uc774 \ub418\uc5c8\uc73c\ubbc0\ub85c \uc601\uce5c\uc655\uc740 \uc77c\ubcf8\uc778\uc774\ub77c\uace0 \ud574\uc11d\ud558\uc600\uc73c\ub098 \uc77c\ubcf8 \uc815\ubd80\ub85c\ubd80\ud130\ub3c4 \uc77c\ubcf8 \uad6d\uc801\uc744 \uc778\uc815\ubc1b\uc9c0 \ubabb\ud574 \uc0ac\uc2e4\uc0c1 \ubb34\uad6d\uc801\uc774 \ub418\uc5c8\ub2e4.", "answer": "10\uc6d4 12\uc77c", "sentence": "\uc2dc\ubcfc\ub4dc\ub294 \uba74\ub2f4 \ub0b4\uc6a9\uc744 10\uc6d4 12\uc77c \uc5d0 \ubbf8\uad6d \uad6d\ubb34\ubd80\uc5d0 \ubcf4\uace0\ud558\uc600\ub2e4.", "paragraph_sentence": "1948\ub144 8\uc6d4\uc5d0 \ub300\ud55c\ubbfc\uad6d \uc815\ubd80\uac00 \uc218\ub9bd\ub418\uc790 \uc774\uc740\uc740 \uc8fc\uc77c \ub300\ud55c\ubbfc\uad6d \ub300\ud45c\ubd80\ub97c \ud1b5\ud574\uc11c \uc5ec\ub7ec \ucc28\ub840 \uadc0\uad6d \uc758\uc0ac\ub97c \ud0c0\uc9c4\ud558\uc9c0\ub9cc \ub300\ud55c\ubbfc\uad6d \uc815\ubd80\ub294 \ub69c\ub837\ud55c \ubc18\uc751\uc744 \ubcf4\uc774\uc9c0 \uc54a\uc558\ub2e4. \uc624\ud788\ub824 \uc774\uc740\uc758 \uac1c\uc778 \uc7ac\uc0b0\uc778 \ub3c4\ucfc4 \uc800\ud0dd\uc744 \uad6d\uc720\uc7ac\uc0b0\uc774\ub2c8 \uc8fc\uc77c \ub300\ud45c\ubd80 \uac74\ubb3c\ub85c \uc0ac\uc6a9\ud558\uaca0\ub2e4\uba70 \ub0b4\uc5b4\ub193\uc73c\ub77c\ub294 \ud6c8\ub839\uc744 \ubcf4\ub0b4\uc654\ub2e4. \uac19\uc740 \ud574 \uc774\uc740\uc740 \uc5f0\ud569\uad70 \ucd5c\uace0\uc0ac\ub839\ubd80 \uc815\uce58 \uace0\ubb38 \uc70c\ub9ac\uc5c4 \uc2dc\ubcfc\ub4dc\uc640 \uba74\ub2f4\uc744 \ud558\uc5ec \uc790\uc2e0\uc758 \uad6d\uc801\uc5d0 \ub300\ud55c \uc720\uad8c \ud574\uc11d\uc744 \uc694\uccad\ud558\uc600\ub2e4. \uc774\uc740\uc740 \uc2dc\ubcfc\ub4dc\uc5d0\uac8c \ub300\ud55c\ubbfc\uad6d \ub0b4\uc758 \uc7ac\uc0b0\uc740 \ub300\ud55c\ubbfc\uad6d \uc815\ubd80\uc5d0 \uae30\uc99d\ud558\uace0, \uc77c\ubcf8 \ub0b4\uc758 \uc7ac\uc0b0\uc740 \uc0dd\uacc4\ub97c \uc704\ud574 \uc18c\uc720\uad8c\uc744 \ud589\uc0ac\ud558\uace0 \uc2f6\uc740\ub370 \ubc95\uc801\uc73c\ub85c \uac00\ub2a5\ud55c\uc9c0\ub97c \uc9c8\ubb38\ud558\uc600\uc73c\uba70, \ubbf8\uad6d\uc5d0 \uccb4\ub958\ud558\uac70\ub098 \uc601\uc8fc\uad8c\uc744 \uc5bb\uc5b4 \ucc28\ub0a8 \uc774\uad6c\ub97c \ubbf8\uad6d\uc5d0\uc11c \uad50\uc721\uc2dc\ud0a4\uace0 \uc2f6\ub2e4\ub294 \ub73b\uc744 \ubc1d\ud614\ub2e4. \ub610\ud55c, \uc774\uc740\uc740 \ub300\ud55c\ubbfc\uad6d\uc5d0\uc11c\uc758 \uc815\uce58\uc801 \ubaa9\uc801\uc744 \uc704\ud574 \uc774\uc6a9\ub418\uace0 \uc2f6\uc9c0 \uc54a\ub2e4\uace0 \ub9d0\ud558\uc600\ub2e4. \uc2dc\ubcfc\ub4dc\ub294 \uba74\ub2f4 \ub0b4\uc6a9\uc744 10\uc6d4 12\uc77c \uc5d0 \ubbf8\uad6d \uad6d\ubb34\ubd80\uc5d0 \ubcf4\uace0\ud558\uc600\ub2e4. \uc2dc\ubcfc\ub4dc\ub294 \uc601\uce5c\uc655\uc740 \uacb0\ud63c\uc744 \ud1b5\ud574 \uc77c\ubcf8 \ud669\uc2e4\uc758 \uc77c\uc6d0\uc774 \ub418\uc5c8\uc73c\ubbc0\ub85c \uc601\uce5c\uc655\uc740 \uc77c\ubcf8\uc778\uc774\ub77c\uace0 \ud574\uc11d\ud558\uc600\uc73c\ub098 \uc77c\ubcf8 \uc815\ubd80\ub85c\ubd80\ud130\ub3c4 \uc77c\ubcf8 \uad6d\uc801\uc744 \uc778\uc815\ubc1b\uc9c0 \ubabb\ud574 \uc0ac\uc2e4\uc0c1 \ubb34\uad6d\uc801\uc774 \ub418\uc5c8\ub2e4.", "paragraph_answer": "1948\ub144 8\uc6d4\uc5d0 \ub300\ud55c\ubbfc\uad6d \uc815\ubd80\uac00 \uc218\ub9bd\ub418\uc790 \uc774\uc740\uc740 \uc8fc\uc77c \ub300\ud55c\ubbfc\uad6d \ub300\ud45c\ubd80\ub97c \ud1b5\ud574\uc11c \uc5ec\ub7ec \ucc28\ub840 \uadc0\uad6d \uc758\uc0ac\ub97c \ud0c0\uc9c4\ud558\uc9c0\ub9cc \ub300\ud55c\ubbfc\uad6d \uc815\ubd80\ub294 \ub69c\ub837\ud55c \ubc18\uc751\uc744 \ubcf4\uc774\uc9c0 \uc54a\uc558\ub2e4. \uc624\ud788\ub824 \uc774\uc740\uc758 \uac1c\uc778 \uc7ac\uc0b0\uc778 \ub3c4\ucfc4 \uc800\ud0dd\uc744 \uad6d\uc720\uc7ac\uc0b0\uc774\ub2c8 \uc8fc\uc77c \ub300\ud45c\ubd80 \uac74\ubb3c\ub85c \uc0ac\uc6a9\ud558\uaca0\ub2e4\uba70 \ub0b4\uc5b4\ub193\uc73c\ub77c\ub294 \ud6c8\ub839\uc744 \ubcf4\ub0b4\uc654\ub2e4. \uac19\uc740 \ud574 \uc774\uc740\uc740 \uc5f0\ud569\uad70 \ucd5c\uace0\uc0ac\ub839\ubd80 \uc815\uce58 \uace0\ubb38 \uc70c\ub9ac\uc5c4 \uc2dc\ubcfc\ub4dc\uc640 \uba74\ub2f4\uc744 \ud558\uc5ec \uc790\uc2e0\uc758 \uad6d\uc801\uc5d0 \ub300\ud55c \uc720\uad8c \ud574\uc11d\uc744 \uc694\uccad\ud558\uc600\ub2e4. \uc774\uc740\uc740 \uc2dc\ubcfc\ub4dc\uc5d0\uac8c \ub300\ud55c\ubbfc\uad6d \ub0b4\uc758 \uc7ac\uc0b0\uc740 \ub300\ud55c\ubbfc\uad6d \uc815\ubd80\uc5d0 \uae30\uc99d\ud558\uace0, \uc77c\ubcf8 \ub0b4\uc758 \uc7ac\uc0b0\uc740 \uc0dd\uacc4\ub97c \uc704\ud574 \uc18c\uc720\uad8c\uc744 \ud589\uc0ac\ud558\uace0 \uc2f6\uc740\ub370 \ubc95\uc801\uc73c\ub85c \uac00\ub2a5\ud55c\uc9c0\ub97c \uc9c8\ubb38\ud558\uc600\uc73c\uba70, \ubbf8\uad6d\uc5d0 \uccb4\ub958\ud558\uac70\ub098 \uc601\uc8fc\uad8c\uc744 \uc5bb\uc5b4 \ucc28\ub0a8 \uc774\uad6c\ub97c \ubbf8\uad6d\uc5d0\uc11c \uad50\uc721\uc2dc\ud0a4\uace0 \uc2f6\ub2e4\ub294 \ub73b\uc744 \ubc1d\ud614\ub2e4. \ub610\ud55c, \uc774\uc740\uc740 \ub300\ud55c\ubbfc\uad6d\uc5d0\uc11c\uc758 \uc815\uce58\uc801 \ubaa9\uc801\uc744 \uc704\ud574 \uc774\uc6a9\ub418\uace0 \uc2f6\uc9c0 \uc54a\ub2e4\uace0 \ub9d0\ud558\uc600\ub2e4. \uc2dc\ubcfc\ub4dc\ub294 \uba74\ub2f4 \ub0b4\uc6a9\uc744 10\uc6d4 12\uc77c \uc5d0 \ubbf8\uad6d \uad6d\ubb34\ubd80\uc5d0 \ubcf4\uace0\ud558\uc600\ub2e4. \uc2dc\ubcfc\ub4dc\ub294 \uc601\uce5c\uc655\uc740 \uacb0\ud63c\uc744 \ud1b5\ud574 \uc77c\ubcf8 \ud669\uc2e4\uc758 \uc77c\uc6d0\uc774 \ub418\uc5c8\uc73c\ubbc0\ub85c \uc601\uce5c\uc655\uc740 \uc77c\ubcf8\uc778\uc774\ub77c\uace0 \ud574\uc11d\ud558\uc600\uc73c\ub098 \uc77c\ubcf8 \uc815\ubd80\ub85c\ubd80\ud130\ub3c4 \uc77c\ubcf8 \uad6d\uc801\uc744 \uc778\uc815\ubc1b\uc9c0 \ubabb\ud574 \uc0ac\uc2e4\uc0c1 \ubb34\uad6d\uc801\uc774 \ub418\uc5c8\ub2e4.", "sentence_answer": "\uc2dc\ubcfc\ub4dc\ub294 \uba74\ub2f4 \ub0b4\uc6a9\uc744 10\uc6d4 12\uc77c \uc5d0 \ubbf8\uad6d \uad6d\ubb34\ubd80\uc5d0 \ubcf4\uace0\ud558\uc600\ub2e4.", "paragraph_id": "5836625-12-2", "paragraph_question": "question: \uc2dc\ubcfc\ub4dc\uac00 \uba74\ub2f4 \ub0b4\uc6a9\uc744 \ubbf8\uad6d \uad6d\ubb34\ubd80\uc5d0 \ubcf4\uace0\ud55c \ub0a0\uc9dc\ub294?, context: 1948\ub144 8\uc6d4\uc5d0 \ub300\ud55c\ubbfc\uad6d \uc815\ubd80\uac00 \uc218\ub9bd\ub418\uc790 \uc774\uc740\uc740 \uc8fc\uc77c \ub300\ud55c\ubbfc\uad6d \ub300\ud45c\ubd80\ub97c \ud1b5\ud574\uc11c \uc5ec\ub7ec \ucc28\ub840 \uadc0\uad6d \uc758\uc0ac\ub97c \ud0c0\uc9c4\ud558\uc9c0\ub9cc \ub300\ud55c\ubbfc\uad6d \uc815\ubd80\ub294 \ub69c\ub837\ud55c \ubc18\uc751\uc744 \ubcf4\uc774\uc9c0 \uc54a\uc558\ub2e4. \uc624\ud788\ub824 \uc774\uc740\uc758 \uac1c\uc778 \uc7ac\uc0b0\uc778 \ub3c4\ucfc4 \uc800\ud0dd\uc744 \uad6d\uc720\uc7ac\uc0b0\uc774\ub2c8 \uc8fc\uc77c \ub300\ud45c\ubd80 \uac74\ubb3c\ub85c \uc0ac\uc6a9\ud558\uaca0\ub2e4\uba70 \ub0b4\uc5b4\ub193\uc73c\ub77c\ub294 \ud6c8\ub839\uc744 \ubcf4\ub0b4\uc654\ub2e4. \uac19\uc740 \ud574 \uc774\uc740\uc740 \uc5f0\ud569\uad70 \ucd5c\uace0\uc0ac\ub839\ubd80 \uc815\uce58 \uace0\ubb38 \uc70c\ub9ac\uc5c4 \uc2dc\ubcfc\ub4dc\uc640 \uba74\ub2f4\uc744 \ud558\uc5ec \uc790\uc2e0\uc758 \uad6d\uc801\uc5d0 \ub300\ud55c \uc720\uad8c \ud574\uc11d\uc744 \uc694\uccad\ud558\uc600\ub2e4. \uc774\uc740\uc740 \uc2dc\ubcfc\ub4dc\uc5d0\uac8c \ub300\ud55c\ubbfc\uad6d \ub0b4\uc758 \uc7ac\uc0b0\uc740 \ub300\ud55c\ubbfc\uad6d \uc815\ubd80\uc5d0 \uae30\uc99d\ud558\uace0, \uc77c\ubcf8 \ub0b4\uc758 \uc7ac\uc0b0\uc740 \uc0dd\uacc4\ub97c \uc704\ud574 \uc18c\uc720\uad8c\uc744 \ud589\uc0ac\ud558\uace0 \uc2f6\uc740\ub370 \ubc95\uc801\uc73c\ub85c \uac00\ub2a5\ud55c\uc9c0\ub97c \uc9c8\ubb38\ud558\uc600\uc73c\uba70, \ubbf8\uad6d\uc5d0 \uccb4\ub958\ud558\uac70\ub098 \uc601\uc8fc\uad8c\uc744 \uc5bb\uc5b4 \ucc28\ub0a8 \uc774\uad6c\ub97c \ubbf8\uad6d\uc5d0\uc11c \uad50\uc721\uc2dc\ud0a4\uace0 \uc2f6\ub2e4\ub294 \ub73b\uc744 \ubc1d\ud614\ub2e4. \ub610\ud55c, \uc774\uc740\uc740 \ub300\ud55c\ubbfc\uad6d\uc5d0\uc11c\uc758 \uc815\uce58\uc801 \ubaa9\uc801\uc744 \uc704\ud574 \uc774\uc6a9\ub418\uace0 \uc2f6\uc9c0 \uc54a\ub2e4\uace0 \ub9d0\ud558\uc600\ub2e4. \uc2dc\ubcfc\ub4dc\ub294 \uba74\ub2f4 \ub0b4\uc6a9\uc744 10\uc6d4 12\uc77c\uc5d0 \ubbf8\uad6d \uad6d\ubb34\ubd80\uc5d0 \ubcf4\uace0\ud558\uc600\ub2e4. \uc2dc\ubcfc\ub4dc\ub294 \uc601\uce5c\uc655\uc740 \uacb0\ud63c\uc744 \ud1b5\ud574 \uc77c\ubcf8 \ud669\uc2e4\uc758 \uc77c\uc6d0\uc774 \ub418\uc5c8\uc73c\ubbc0\ub85c \uc601\uce5c\uc655\uc740 \uc77c\ubcf8\uc778\uc774\ub77c\uace0 \ud574\uc11d\ud558\uc600\uc73c\ub098 \uc77c\ubcf8 \uc815\ubd80\ub85c\ubd80\ud130\ub3c4 \uc77c\ubcf8 \uad6d\uc801\uc744 \uc778\uc815\ubc1b\uc9c0 \ubabb\ud574 \uc0ac\uc2e4\uc0c1 \ubb34\uad6d\uc801\uc774 \ub418\uc5c8\ub2e4."} {"question": "\uc911\uad6d\uc758 \uc81c\ub3c4\uc778 \ubb18\ud638\ub97c \ub530\ub77c \uc4f0\ub294 \uac83\uc774 \uc633\uc9c0 \uc54a\uc73c\ub2c8 \uc2dc\ud638\ub9cc \uc4f0\uae30\ub97c \uba85\ud588\ub358 \uc784\uae08\uc740?", "paragraph": "\uc774 \uc774\ud6c4\ub85c, \uc911\uad6d\uacfc\uc758 \uc0ac\ub300 \uad00\uacc4\uac00 \ud655\ub9bd\ud654\ub418\uba70 \uc2a4\uc2a4\ub85c \uc81c\ud6c4\uad6d\uc744 \uc790\ucc98\ud55c \uc870\uc120\uc740 \uc548\uc73c\ub85c\ub294 \ubb18\ud638\ub97c \uc4f0\ub294 \ub300\uc2e0 \ubc16\uc73c\ub85c\ub294 \uc911\uad6d\uc73c\ub85c\ubd80\ud130 \uc2dc\ud638\ub97c \ubc1b\uc544\uc11c \uac19\uc774 \uc0ac\uc6a9\ud558\uc600\ub2e4. \uadf8\ub7ec\ub098 9\ub300 \uc784\uae08\uc778 \uc131\uc885 \ub300\uc5d0 \uc774\ub974\ub7ec \uc131\uc885\uc774 \u300a\ub3d9\uad6d\uc5ec\uc9c0\uc2b9\ub78c\u300b\uc744 \ud3b8\ucc2c\ud560 \ub54c \uc911\uad6d\uc758 \uc81c\ub3c4\ub97c \ub530\ub77c \ubb18\ud638\ub97c \uc4f0\ub294 \uac83\uc740 \uc633\uc9c0 \uc54a\ub2e4\uba70 \ubb18\ud638 \ub300\uc2e0 \uc2dc\ud638\ub85c\ub9cc \uc4f0\uc790 \uba85\ud558\uc600\uc73c\ub098, \uae40\uc885\uc9c1\uc744 \uc704\uc2dc\ud55c \uc0ac\ub9bc\ud30c\uac00 \uc655\ub989\uc758 \ube44\ubb38\uacfc \uc11c\uc801 \ub4f1\uc5d0 \uc774\ubbf8 \ubb18\ud638\ub85c \uc77c\uceeb\ub294 \ubd80\ubd84\uc774 \ub9ce\uc544 \uc77c\uc77c\uc774 \uace0\uce60 \uc218 \uc5c6\ub2e4 \uac04\ud558\uc790 \uc131\uc885\uc774 \uc774\ub97c \ubc1b\uc544\ub4e4\uc5ec \ub17c\uc758\ub97c \ucca0\ud68c\ud558\uae30\ub3c4 \ud588\ub2e4. \ub610\ud55c \uc815\uc720\uc7ac\ub780 \ub3c4\uc911\uc778 1598\ub144(\uc120\uc870 31\ub144), \uc870\uc120\uc5d0 \uc6d0\uad70\uc73c\ub85c \uc628 \uba85\ub098\ub77c\uc758 \ucc2c\ud68d\uc8fc\uc0ac \uc815\uc751\ud0dc\uac00 \ud669\uc81c\uc778 \ub9cc\ub825\uc81c\uc5d0\uac8c \ud45c\ubb38\uc744 \uc62c\ub824 \uc870\uc120\uc774 \uac10\ud788 \ub300\uad6d\uc758 \uace0\uc720\ud55c \uad00\uc2b5\uc778 \uce6d\uc870\uc874\uc0c1(\u7a31\u7956\u5c0a\u4e0a) \uc744 \ub530\ub77c\ud55c\ub2e4\uba70 \uc870\uc120 \uc815\ubd80\ub97c \ubb38\ucc45\ud560 \uac83\uc744 \uc8fc\uccad\ud558\uae30\ub3c4 \ud588\ub2e4.", "answer": "\uc131\uc885", "sentence": "\uadf8\ub7ec\ub098 9\ub300 \uc784\uae08\uc778 \uc131\uc885 \ub300\uc5d0 \uc774\ub974\ub7ec \uc131\uc885\uc774 \u300a\ub3d9\uad6d\uc5ec\uc9c0\uc2b9\ub78c\u300b\uc744 \ud3b8\ucc2c\ud560 \ub54c \uc911\uad6d\uc758 \uc81c\ub3c4\ub97c \ub530\ub77c \ubb18\ud638\ub97c \uc4f0\ub294 \uac83\uc740 \uc633\uc9c0 \uc54a\ub2e4\uba70 \ubb18\ud638 \ub300\uc2e0 \uc2dc\ud638\ub85c\ub9cc \uc4f0\uc790 \uba85\ud558\uc600\uc73c\ub098, \uae40\uc885\uc9c1\uc744 \uc704\uc2dc\ud55c \uc0ac\ub9bc\ud30c\uac00 \uc655\ub989\uc758 \ube44\ubb38\uacfc \uc11c\uc801 \ub4f1\uc5d0 \uc774\ubbf8 \ubb18\ud638\ub85c \uc77c\uceeb\ub294 \ubd80\ubd84\uc774 \ub9ce\uc544 \uc77c\uc77c\uc774 \uace0\uce60 \uc218 \uc5c6\ub2e4 \uac04\ud558\uc790 \uc131\uc885\uc774 \uc774\ub97c \ubc1b\uc544\ub4e4\uc5ec \ub17c\uc758\ub97c \ucca0\ud68c\ud558\uae30\ub3c4 \ud588\ub2e4.", "paragraph_sentence": "\uc774 \uc774\ud6c4\ub85c, \uc911\uad6d\uacfc\uc758 \uc0ac\ub300 \uad00\uacc4\uac00 \ud655\ub9bd\ud654\ub418\uba70 \uc2a4\uc2a4\ub85c \uc81c\ud6c4\uad6d\uc744 \uc790\ucc98\ud55c \uc870\uc120\uc740 \uc548\uc73c\ub85c\ub294 \ubb18\ud638\ub97c \uc4f0\ub294 \ub300\uc2e0 \ubc16\uc73c\ub85c\ub294 \uc911\uad6d\uc73c\ub85c\ubd80\ud130 \uc2dc\ud638\ub97c \ubc1b\uc544\uc11c \uac19\uc774 \uc0ac\uc6a9\ud558\uc600\ub2e4. \uadf8\ub7ec\ub098 9\ub300 \uc784\uae08\uc778 \uc131\uc885 \ub300\uc5d0 \uc774\ub974\ub7ec \uc131\uc885\uc774 \u300a\ub3d9\uad6d\uc5ec\uc9c0\uc2b9\ub78c\u300b\uc744 \ud3b8\ucc2c\ud560 \ub54c \uc911\uad6d\uc758 \uc81c\ub3c4\ub97c \ub530\ub77c \ubb18\ud638\ub97c \uc4f0\ub294 \uac83\uc740 \uc633\uc9c0 \uc54a\ub2e4\uba70 \ubb18\ud638 \ub300\uc2e0 \uc2dc\ud638\ub85c\ub9cc \uc4f0\uc790 \uba85\ud558\uc600\uc73c\ub098, \uae40\uc885\uc9c1\uc744 \uc704\uc2dc\ud55c \uc0ac\ub9bc\ud30c\uac00 \uc655\ub989\uc758 \ube44\ubb38\uacfc \uc11c\uc801 \ub4f1\uc5d0 \uc774\ubbf8 \ubb18\ud638\ub85c \uc77c\uceeb\ub294 \ubd80\ubd84\uc774 \ub9ce\uc544 \uc77c\uc77c\uc774 \uace0\uce60 \uc218 \uc5c6\ub2e4 \uac04\ud558\uc790 \uc131\uc885\uc774 \uc774\ub97c \ubc1b\uc544\ub4e4\uc5ec \ub17c\uc758\ub97c \ucca0\ud68c\ud558\uae30\ub3c4 \ud588\ub2e4. \ub610\ud55c \uc815\uc720\uc7ac\ub780 \ub3c4\uc911\uc778 1598\ub144(\uc120\uc870 31\ub144), \uc870\uc120\uc5d0 \uc6d0\uad70\uc73c\ub85c \uc628 \uba85\ub098\ub77c\uc758 \ucc2c\ud68d\uc8fc\uc0ac \uc815\uc751\ud0dc\uac00 \ud669\uc81c\uc778 \ub9cc\ub825\uc81c\uc5d0\uac8c \ud45c\ubb38\uc744 \uc62c\ub824 \uc870\uc120\uc774 \uac10\ud788 \ub300\uad6d\uc758 \uace0\uc720\ud55c \uad00\uc2b5\uc778 \uce6d\uc870\uc874\uc0c1(\u7a31\u7956\u5c0a\u4e0a) \uc744 \ub530\ub77c\ud55c\ub2e4\uba70 \uc870\uc120 \uc815\ubd80\ub97c \ubb38\ucc45\ud560 \uac83\uc744 \uc8fc\uccad\ud558\uae30\ub3c4 \ud588\ub2e4.", "paragraph_answer": "\uc774 \uc774\ud6c4\ub85c, \uc911\uad6d\uacfc\uc758 \uc0ac\ub300 \uad00\uacc4\uac00 \ud655\ub9bd\ud654\ub418\uba70 \uc2a4\uc2a4\ub85c \uc81c\ud6c4\uad6d\uc744 \uc790\ucc98\ud55c \uc870\uc120\uc740 \uc548\uc73c\ub85c\ub294 \ubb18\ud638\ub97c \uc4f0\ub294 \ub300\uc2e0 \ubc16\uc73c\ub85c\ub294 \uc911\uad6d\uc73c\ub85c\ubd80\ud130 \uc2dc\ud638\ub97c \ubc1b\uc544\uc11c \uac19\uc774 \uc0ac\uc6a9\ud558\uc600\ub2e4. \uadf8\ub7ec\ub098 9\ub300 \uc784\uae08\uc778 \uc131\uc885 \ub300\uc5d0 \uc774\ub974\ub7ec \uc131\uc885\uc774 \u300a\ub3d9\uad6d\uc5ec\uc9c0\uc2b9\ub78c\u300b\uc744 \ud3b8\ucc2c\ud560 \ub54c \uc911\uad6d\uc758 \uc81c\ub3c4\ub97c \ub530\ub77c \ubb18\ud638\ub97c \uc4f0\ub294 \uac83\uc740 \uc633\uc9c0 \uc54a\ub2e4\uba70 \ubb18\ud638 \ub300\uc2e0 \uc2dc\ud638\ub85c\ub9cc \uc4f0\uc790 \uba85\ud558\uc600\uc73c\ub098, \uae40\uc885\uc9c1\uc744 \uc704\uc2dc\ud55c \uc0ac\ub9bc\ud30c\uac00 \uc655\ub989\uc758 \ube44\ubb38\uacfc \uc11c\uc801 \ub4f1\uc5d0 \uc774\ubbf8 \ubb18\ud638\ub85c \uc77c\uceeb\ub294 \ubd80\ubd84\uc774 \ub9ce\uc544 \uc77c\uc77c\uc774 \uace0\uce60 \uc218 \uc5c6\ub2e4 \uac04\ud558\uc790 \uc131\uc885\uc774 \uc774\ub97c \ubc1b\uc544\ub4e4\uc5ec \ub17c\uc758\ub97c \ucca0\ud68c\ud558\uae30\ub3c4 \ud588\ub2e4. \ub610\ud55c \uc815\uc720\uc7ac\ub780 \ub3c4\uc911\uc778 1598\ub144(\uc120\uc870 31\ub144), \uc870\uc120\uc5d0 \uc6d0\uad70\uc73c\ub85c \uc628 \uba85\ub098\ub77c\uc758 \ucc2c\ud68d\uc8fc\uc0ac \uc815\uc751\ud0dc\uac00 \ud669\uc81c\uc778 \ub9cc\ub825\uc81c\uc5d0\uac8c \ud45c\ubb38\uc744 \uc62c\ub824 \uc870\uc120\uc774 \uac10\ud788 \ub300\uad6d\uc758 \uace0\uc720\ud55c \uad00\uc2b5\uc778 \uce6d\uc870\uc874\uc0c1(\u7a31\u7956\u5c0a\u4e0a) \uc744 \ub530\ub77c\ud55c\ub2e4\uba70 \uc870\uc120 \uc815\ubd80\ub97c \ubb38\ucc45\ud560 \uac83\uc744 \uc8fc\uccad\ud558\uae30\ub3c4 \ud588\ub2e4.", "sentence_answer": "\uadf8\ub7ec\ub098 9\ub300 \uc784\uae08\uc778 \uc131\uc885 \ub300\uc5d0 \uc774\ub974\ub7ec \uc131\uc885\uc774 \u300a\ub3d9\uad6d\uc5ec\uc9c0\uc2b9\ub78c\u300b\uc744 \ud3b8\ucc2c\ud560 \ub54c \uc911\uad6d\uc758 \uc81c\ub3c4\ub97c \ub530\ub77c \ubb18\ud638\ub97c \uc4f0\ub294 \uac83\uc740 \uc633\uc9c0 \uc54a\ub2e4\uba70 \ubb18\ud638 \ub300\uc2e0 \uc2dc\ud638\ub85c\ub9cc \uc4f0\uc790 \uba85\ud558\uc600\uc73c\ub098, \uae40\uc885\uc9c1\uc744 \uc704\uc2dc\ud55c \uc0ac\ub9bc\ud30c\uac00 \uc655\ub989\uc758 \ube44\ubb38\uacfc \uc11c\uc801 \ub4f1\uc5d0 \uc774\ubbf8 \ubb18\ud638\ub85c \uc77c\uceeb\ub294 \ubd80\ubd84\uc774 \ub9ce\uc544 \uc77c\uc77c\uc774 \uace0\uce60 \uc218 \uc5c6\ub2e4 \uac04\ud558\uc790 \uc131\uc885\uc774 \uc774\ub97c \ubc1b\uc544\ub4e4\uc5ec \ub17c\uc758\ub97c \ucca0\ud68c\ud558\uae30\ub3c4 \ud588\ub2e4.", "paragraph_id": "6504692-9-0", "paragraph_question": "question: \uc911\uad6d\uc758 \uc81c\ub3c4\uc778 \ubb18\ud638\ub97c \ub530\ub77c \uc4f0\ub294 \uac83\uc774 \uc633\uc9c0 \uc54a\uc73c\ub2c8 \uc2dc\ud638\ub9cc \uc4f0\uae30\ub97c \uba85\ud588\ub358 \uc784\uae08\uc740?, context: \uc774 \uc774\ud6c4\ub85c, \uc911\uad6d\uacfc\uc758 \uc0ac\ub300 \uad00\uacc4\uac00 \ud655\ub9bd\ud654\ub418\uba70 \uc2a4\uc2a4\ub85c \uc81c\ud6c4\uad6d\uc744 \uc790\ucc98\ud55c \uc870\uc120\uc740 \uc548\uc73c\ub85c\ub294 \ubb18\ud638\ub97c \uc4f0\ub294 \ub300\uc2e0 \ubc16\uc73c\ub85c\ub294 \uc911\uad6d\uc73c\ub85c\ubd80\ud130 \uc2dc\ud638\ub97c \ubc1b\uc544\uc11c \uac19\uc774 \uc0ac\uc6a9\ud558\uc600\ub2e4. \uadf8\ub7ec\ub098 9\ub300 \uc784\uae08\uc778 \uc131\uc885 \ub300\uc5d0 \uc774\ub974\ub7ec \uc131\uc885\uc774 \u300a\ub3d9\uad6d\uc5ec\uc9c0\uc2b9\ub78c\u300b\uc744 \ud3b8\ucc2c\ud560 \ub54c \uc911\uad6d\uc758 \uc81c\ub3c4\ub97c \ub530\ub77c \ubb18\ud638\ub97c \uc4f0\ub294 \uac83\uc740 \uc633\uc9c0 \uc54a\ub2e4\uba70 \ubb18\ud638 \ub300\uc2e0 \uc2dc\ud638\ub85c\ub9cc \uc4f0\uc790 \uba85\ud558\uc600\uc73c\ub098, \uae40\uc885\uc9c1\uc744 \uc704\uc2dc\ud55c \uc0ac\ub9bc\ud30c\uac00 \uc655\ub989\uc758 \ube44\ubb38\uacfc \uc11c\uc801 \ub4f1\uc5d0 \uc774\ubbf8 \ubb18\ud638\ub85c \uc77c\uceeb\ub294 \ubd80\ubd84\uc774 \ub9ce\uc544 \uc77c\uc77c\uc774 \uace0\uce60 \uc218 \uc5c6\ub2e4 \uac04\ud558\uc790 \uc131\uc885\uc774 \uc774\ub97c \ubc1b\uc544\ub4e4\uc5ec \ub17c\uc758\ub97c \ucca0\ud68c\ud558\uae30\ub3c4 \ud588\ub2e4. \ub610\ud55c \uc815\uc720\uc7ac\ub780 \ub3c4\uc911\uc778 1598\ub144(\uc120\uc870 31\ub144), \uc870\uc120\uc5d0 \uc6d0\uad70\uc73c\ub85c \uc628 \uba85\ub098\ub77c\uc758 \ucc2c\ud68d\uc8fc\uc0ac \uc815\uc751\ud0dc\uac00 \ud669\uc81c\uc778 \ub9cc\ub825\uc81c\uc5d0\uac8c \ud45c\ubb38\uc744 \uc62c\ub824 \uc870\uc120\uc774 \uac10\ud788 \ub300\uad6d\uc758 \uace0\uc720\ud55c \uad00\uc2b5\uc778 \uce6d\uc870\uc874\uc0c1(\u7a31\u7956\u5c0a\u4e0a) \uc744 \ub530\ub77c\ud55c\ub2e4\uba70 \uc870\uc120 \uc815\ubd80\ub97c \ubb38\ucc45\ud560 \uac83\uc744 \uc8fc\uccad\ud558\uae30\ub3c4 \ud588\ub2e4."} {"question": "\ucf70\ub9ac\uc988\ub9c8\uac00 \uc800\uc790\uc774\uba70 \ud6c4\uc5d0 \ub77c\ud2f4\uc5b4\ub85c \ubc88\uc5ed\ub41c \ucc45\uc740?", "paragraph": "\uc720\ud074\ub9ac\ub4dc \uae30\ud558\ud559\uc744 \ube44\ub86f\ud55c \uadf8\ub9ac\uc2a4 \uc218\ud559 \uc678\uc5d0\ub3c4 \uc778\ub3c4 \uc218\ud559\uc758 \uc601\ud5a5\uc744 \ubc1b\uc544 \ubc1c\ub2ec\ud588\ub2e4. \uadf8 \uc911\uc5d0\uc11c 10\uc9c4\ubc95\uacfc \uc601(0)\uc758 \uae30\ud638\ub97c \uac16\ub294 \uc778\ub3c4 \uc22b\uc790\uc758 \ucc44\uc6a9\uc740 \uc911\uc694\ud55c \uc758\ubbf8\ub97c \uac00\uc9c0\uba70, \uc544\ub77c\ube44\uc544 \uc22b\uc790\uc758 \ubc1c\ub2ec\uc744 \ucd09\uc9c4\uc2dc\ucf30\uc744 \ubfd0\ub9cc \uc544\ub2c8\ub77c \uc544\ub77c\ube44\uc544 \uc22b\uc790\ub85c\uc11c \uc720\ub7fd\uc5d0 \uc804\ud574\uc838 \uadfc\ub300 \uc218\ud559\uc758 \ubc1c\uc804\uc744 \ub3c4\uc654\ub2e4. \uc544\ub77c\ube44\uc544 \uc218\ud559\uc758 \uacf5\uc801\uc740 \ub300\uc218\ud559\uacfc \uc0bc\uac01\ubc95\uc5d0\uc11c \ud55c\uce35 \ub450\ub4dc\ub7ec\uc9c4\ub2e4. \ub300\uc218\ud559\uc5d0\uc11c\ub294 \ucf70\ub9ac\uc988\ubbf8\uac00 \uc720\uba85\ud558\uace0, \uadf8\uac00 \uc4f4 \u300a\ud788\uc0ac\ube14 \uc790\ube0c\ub974 \uc648\ubb34\uce74\ubc1c\ub77c\u300b\ub294 \ub77c\ud2f4\uc5b4\ub85c \ubc88\uc5ed\ub418\uc5b4 \uc911\uc138 \uc720\ub7fd\uc5d0 \ud06c\uac8c \uc601\ud5a5\uc744 \uc8fc\uc5c8\ub2e4. \uc568\uc9c0\ube0c\ub7ec(algebra, \ub300\uc218\ud559)\ub77c\ub294 \ub9d0\uc740 \uc774 \ucc45 \uc774\ub984\uc73c\ub85c\ubd80\ud130 \ub098\uc628 \uac83\uc774\ub2e4. \ucc9c\ubb38\ud559\uc790\ub85c\uc11c \uc54c\ub824\uc9c4 \ubc14\ud0c0\ub2c8\ub294 \uc0bc\uac01\ubc95\uc758 \uc0ac\uc778, \ucf54\uc0ac\uc778, \ud0c4\uc820\ud2b8\uc758 \uac1c\ub150\uc744 \ud655\ub9bd\ud558\uc5ec \uc774 \ubc29\uba74\uc5d0\uc11c\ub3c4 \ubd88\ud6c4\uc758 \uacf5\uc801\uc744 \ub0a8\uacbc\ub2e4.", "answer": "\ud788\uc0ac\ube14 \uc790\ube0c\ub974 \uc648\ubb34\uce74\ubc1c\ub77c", "sentence": "\ub300\uc218\ud559\uc5d0\uc11c\ub294 \ucf70\ub9ac\uc988\ubbf8\uac00 \uc720\uba85\ud558\uace0, \uadf8\uac00 \uc4f4 \u300a \ud788\uc0ac\ube14 \uc790\ube0c\ub974 \uc648\ubb34\uce74\ubc1c\ub77c \u300b\ub294 \ub77c\ud2f4\uc5b4\ub85c \ubc88\uc5ed\ub418\uc5b4 \uc911\uc138 \uc720\ub7fd\uc5d0 \ud06c\uac8c \uc601\ud5a5\uc744 \uc8fc\uc5c8\ub2e4.", "paragraph_sentence": "\uc720\ud074\ub9ac\ub4dc \uae30\ud558\ud559\uc744 \ube44\ub86f\ud55c \uadf8\ub9ac\uc2a4 \uc218\ud559 \uc678\uc5d0\ub3c4 \uc778\ub3c4 \uc218\ud559\uc758 \uc601\ud5a5\uc744 \ubc1b\uc544 \ubc1c\ub2ec\ud588\ub2e4. \uadf8 \uc911\uc5d0\uc11c 10\uc9c4\ubc95\uacfc \uc601(0)\uc758 \uae30\ud638\ub97c \uac16\ub294 \uc778\ub3c4 \uc22b\uc790\uc758 \ucc44\uc6a9\uc740 \uc911\uc694\ud55c \uc758\ubbf8\ub97c \uac00\uc9c0\uba70, \uc544\ub77c\ube44\uc544 \uc22b\uc790\uc758 \ubc1c\ub2ec\uc744 \ucd09\uc9c4\uc2dc\ucf30\uc744 \ubfd0\ub9cc \uc544\ub2c8\ub77c \uc544\ub77c\ube44\uc544 \uc22b\uc790\ub85c\uc11c \uc720\ub7fd\uc5d0 \uc804\ud574\uc838 \uadfc\ub300 \uc218\ud559\uc758 \ubc1c\uc804\uc744 \ub3c4\uc654\ub2e4. \uc544\ub77c\ube44\uc544 \uc218\ud559\uc758 \uacf5\uc801\uc740 \ub300\uc218\ud559\uacfc \uc0bc\uac01\ubc95\uc5d0\uc11c \ud55c\uce35 \ub450\ub4dc\ub7ec\uc9c4\ub2e4. \ub300\uc218\ud559\uc5d0\uc11c\ub294 \ucf70\ub9ac\uc988\ubbf8\uac00 \uc720\uba85\ud558\uace0, \uadf8\uac00 \uc4f4 \u300a \ud788\uc0ac\ube14 \uc790\ube0c\ub974 \uc648\ubb34\uce74\ubc1c\ub77c \u300b\ub294 \ub77c\ud2f4\uc5b4\ub85c \ubc88\uc5ed\ub418\uc5b4 \uc911\uc138 \uc720\ub7fd\uc5d0 \ud06c\uac8c \uc601\ud5a5\uc744 \uc8fc\uc5c8\ub2e4. \uc568\uc9c0\ube0c\ub7ec(algebra, \ub300\uc218\ud559)\ub77c\ub294 \ub9d0\uc740 \uc774 \ucc45 \uc774\ub984\uc73c\ub85c\ubd80\ud130 \ub098\uc628 \uac83\uc774\ub2e4. \ucc9c\ubb38\ud559\uc790\ub85c\uc11c \uc54c\ub824\uc9c4 \ubc14\ud0c0\ub2c8\ub294 \uc0bc\uac01\ubc95\uc758 \uc0ac\uc778, \ucf54\uc0ac\uc778, \ud0c4\uc820\ud2b8\uc758 \uac1c\ub150\uc744 \ud655\ub9bd\ud558\uc5ec \uc774 \ubc29\uba74\uc5d0\uc11c\ub3c4 \ubd88\ud6c4\uc758 \uacf5\uc801\uc744 \ub0a8\uacbc\ub2e4.", "paragraph_answer": "\uc720\ud074\ub9ac\ub4dc \uae30\ud558\ud559\uc744 \ube44\ub86f\ud55c \uadf8\ub9ac\uc2a4 \uc218\ud559 \uc678\uc5d0\ub3c4 \uc778\ub3c4 \uc218\ud559\uc758 \uc601\ud5a5\uc744 \ubc1b\uc544 \ubc1c\ub2ec\ud588\ub2e4. \uadf8 \uc911\uc5d0\uc11c 10\uc9c4\ubc95\uacfc \uc601(0)\uc758 \uae30\ud638\ub97c \uac16\ub294 \uc778\ub3c4 \uc22b\uc790\uc758 \ucc44\uc6a9\uc740 \uc911\uc694\ud55c \uc758\ubbf8\ub97c \uac00\uc9c0\uba70, \uc544\ub77c\ube44\uc544 \uc22b\uc790\uc758 \ubc1c\ub2ec\uc744 \ucd09\uc9c4\uc2dc\ucf30\uc744 \ubfd0\ub9cc \uc544\ub2c8\ub77c \uc544\ub77c\ube44\uc544 \uc22b\uc790\ub85c\uc11c \uc720\ub7fd\uc5d0 \uc804\ud574\uc838 \uadfc\ub300 \uc218\ud559\uc758 \ubc1c\uc804\uc744 \ub3c4\uc654\ub2e4. \uc544\ub77c\ube44\uc544 \uc218\ud559\uc758 \uacf5\uc801\uc740 \ub300\uc218\ud559\uacfc \uc0bc\uac01\ubc95\uc5d0\uc11c \ud55c\uce35 \ub450\ub4dc\ub7ec\uc9c4\ub2e4. \ub300\uc218\ud559\uc5d0\uc11c\ub294 \ucf70\ub9ac\uc988\ubbf8\uac00 \uc720\uba85\ud558\uace0, \uadf8\uac00 \uc4f4 \u300a \ud788\uc0ac\ube14 \uc790\ube0c\ub974 \uc648\ubb34\uce74\ubc1c\ub77c \u300b\ub294 \ub77c\ud2f4\uc5b4\ub85c \ubc88\uc5ed\ub418\uc5b4 \uc911\uc138 \uc720\ub7fd\uc5d0 \ud06c\uac8c \uc601\ud5a5\uc744 \uc8fc\uc5c8\ub2e4. \uc568\uc9c0\ube0c\ub7ec(algebra, \ub300\uc218\ud559)\ub77c\ub294 \ub9d0\uc740 \uc774 \ucc45 \uc774\ub984\uc73c\ub85c\ubd80\ud130 \ub098\uc628 \uac83\uc774\ub2e4. \ucc9c\ubb38\ud559\uc790\ub85c\uc11c \uc54c\ub824\uc9c4 \ubc14\ud0c0\ub2c8\ub294 \uc0bc\uac01\ubc95\uc758 \uc0ac\uc778, \ucf54\uc0ac\uc778, \ud0c4\uc820\ud2b8\uc758 \uac1c\ub150\uc744 \ud655\ub9bd\ud558\uc5ec \uc774 \ubc29\uba74\uc5d0\uc11c\ub3c4 \ubd88\ud6c4\uc758 \uacf5\uc801\uc744 \ub0a8\uacbc\ub2e4.", "sentence_answer": "\ub300\uc218\ud559\uc5d0\uc11c\ub294 \ucf70\ub9ac\uc988\ubbf8\uac00 \uc720\uba85\ud558\uace0, \uadf8\uac00 \uc4f4 \u300a \ud788\uc0ac\ube14 \uc790\ube0c\ub974 \uc648\ubb34\uce74\ubc1c\ub77c \u300b\ub294 \ub77c\ud2f4\uc5b4\ub85c \ubc88\uc5ed\ub418\uc5b4 \uc911\uc138 \uc720\ub7fd\uc5d0 \ud06c\uac8c \uc601\ud5a5\uc744 \uc8fc\uc5c8\ub2e4.", "paragraph_id": "6590047-2-0", "paragraph_question": "question: \ucf70\ub9ac\uc988\ub9c8\uac00 \uc800\uc790\uc774\uba70 \ud6c4\uc5d0 \ub77c\ud2f4\uc5b4\ub85c \ubc88\uc5ed\ub41c \ucc45\uc740?, context: \uc720\ud074\ub9ac\ub4dc \uae30\ud558\ud559\uc744 \ube44\ub86f\ud55c \uadf8\ub9ac\uc2a4 \uc218\ud559 \uc678\uc5d0\ub3c4 \uc778\ub3c4 \uc218\ud559\uc758 \uc601\ud5a5\uc744 \ubc1b\uc544 \ubc1c\ub2ec\ud588\ub2e4. \uadf8 \uc911\uc5d0\uc11c 10\uc9c4\ubc95\uacfc \uc601(0)\uc758 \uae30\ud638\ub97c \uac16\ub294 \uc778\ub3c4 \uc22b\uc790\uc758 \ucc44\uc6a9\uc740 \uc911\uc694\ud55c \uc758\ubbf8\ub97c \uac00\uc9c0\uba70, \uc544\ub77c\ube44\uc544 \uc22b\uc790\uc758 \ubc1c\ub2ec\uc744 \ucd09\uc9c4\uc2dc\ucf30\uc744 \ubfd0\ub9cc \uc544\ub2c8\ub77c \uc544\ub77c\ube44\uc544 \uc22b\uc790\ub85c\uc11c \uc720\ub7fd\uc5d0 \uc804\ud574\uc838 \uadfc\ub300 \uc218\ud559\uc758 \ubc1c\uc804\uc744 \ub3c4\uc654\ub2e4. \uc544\ub77c\ube44\uc544 \uc218\ud559\uc758 \uacf5\uc801\uc740 \ub300\uc218\ud559\uacfc \uc0bc\uac01\ubc95\uc5d0\uc11c \ud55c\uce35 \ub450\ub4dc\ub7ec\uc9c4\ub2e4. \ub300\uc218\ud559\uc5d0\uc11c\ub294 \ucf70\ub9ac\uc988\ubbf8\uac00 \uc720\uba85\ud558\uace0, \uadf8\uac00 \uc4f4 \u300a\ud788\uc0ac\ube14 \uc790\ube0c\ub974 \uc648\ubb34\uce74\ubc1c\ub77c\u300b\ub294 \ub77c\ud2f4\uc5b4\ub85c \ubc88\uc5ed\ub418\uc5b4 \uc911\uc138 \uc720\ub7fd\uc5d0 \ud06c\uac8c \uc601\ud5a5\uc744 \uc8fc\uc5c8\ub2e4. \uc568\uc9c0\ube0c\ub7ec(algebra, \ub300\uc218\ud559)\ub77c\ub294 \ub9d0\uc740 \uc774 \ucc45 \uc774\ub984\uc73c\ub85c\ubd80\ud130 \ub098\uc628 \uac83\uc774\ub2e4. \ucc9c\ubb38\ud559\uc790\ub85c\uc11c \uc54c\ub824\uc9c4 \ubc14\ud0c0\ub2c8\ub294 \uc0bc\uac01\ubc95\uc758 \uc0ac\uc778, \ucf54\uc0ac\uc778, \ud0c4\uc820\ud2b8\uc758 \uac1c\ub150\uc744 \ud655\ub9bd\ud558\uc5ec \uc774 \ubc29\uba74\uc5d0\uc11c\ub3c4 \ubd88\ud6c4\uc758 \uacf5\uc801\uc744 \ub0a8\uacbc\ub2e4."} {"question": "2018\ub144 \uae40\uae30\ucd98\uc5d0\uac8c \uc9d5\uc5ed 4\ub144\uc744 \uc120\uace0\ud55c \ud310\uc0ac\uc758 \uc774\ub984\uc740?", "paragraph": "2017\ub144 7\uc6d4 27\uc77c \uc11c\uc6b8\uc911\uc559\uc9c0\ubc95 \ud615\uc0ac\ud569\uc75830\ubd80(\ud669\ubcd1\ud604 \ubd80\uc7a5\ud310\uc0ac)\ub294 \ubb38\ud654,\uc608\uc220\uc778 \ube14\ub799\ub9ac\uc2a4\ud2b8\uc640 \uad00\ub828\ud558\uc5ec \uc9c1\uad8c\ub0a8\uc6a9 \uad8c\ub9ac\ud589\uc0ac \ubc29\ud574 \ub4f1\uc758 \ud610\uc758\ub85c \uad6c\uc18d\uae30\uc18c\ub41c \uae40\uae30\ucd98\uc5d0\uac8c \uaddc\uc81c\u00b7\uc870\uc138\u00b7\ubc18\ubd80\ud328\u00b7\ubb38\ud654\uc815\ucc45 \ub4f1\uc758 \ubd84\uc57c\uc5d0\uc11c \uc815\ubd80 \ub610\ub294 \uace0\uc704 \uacf5\ubb34\uc6d0\uc774 \"\uac04\uc12d\ud558\uc9c0 \uc54a\ub294\ub2e4\"\ub294 \uc6d0\uce59\uc778 \"\ud314 \uae38\uc774\ub9cc\ud07c \uac70\ub9ac\ub97c \ub454\ub2e4\"\ub97c \uc5b8\uae09\ud558\uba74\uc11c \uc9d5\uc5ed 3\ub144\uc758 \uc2e4\ud615\uc744 \uc120\uace0\ud588\ub2e4.(\uc11c\uc6b8\uc911\uc559\uc9c0\ubc29\ubc95\uc6d02017\uace0\ud569102) \uc774\uc5d0 \ubd88\ubcf5\ud55c \uae40\uae30\ucd98\uc740 \ud56d\uc18c\ub85c \uc5f4\ub9b0 \uc7ac\ud310\uc5d0\uc11c \uc11c\uc6b8\uace0\ub4f1\ubc95\uc6d0 \ud615\uc0ac3\ubd80(\uc870\uc601\ucca0 \ubd80\uc7a5\ud310\uc0ac)\ub294 2018\ub144 1\uc6d4 23\uc77c\uc5d0 \uc9c0\uc6d0 \ubc30\uc81c \ubfd0\ub9cc \uc544\ub2c8\ub77c 1\uc2ec\uc5d0\uc11c \ubb34\uc8c4 \ud310\uacb0\ub41c 1\uae09 \uacf5\ubb34\uc6d0 \uc0ac\uc9c1 \uac15\uc694 \ud610\uc758\uc5d0 \ub300\ud574\uc11c\ub3c4 \uc720\uc8c4\ub97c \uc778\uc815\ud558\uc5ec \uc9d5\uc5ed3\ub144\uc744 \uc120\uace0\ud55c \uc6d0\uc2ec\uc744 \ud30c\uae30\ud558\uba74\uc11c \uc9d5\uc5ed 4\ub144\uc744 \uc120\uace0\ud588\ub2e4.(\uc11c\uc6b8\uace0\ub4f1\ubc95\uc6d02017\ub1782425)", "answer": "\uc870\uc601\ucca0", "sentence": "2017\ub144 7\uc6d4 27\uc77c \uc11c\uc6b8\uc911\uc559\uc9c0\ubc95 \ud615\uc0ac\ud569\uc75830\ubd80(\ud669\ubcd1\ud604 \ubd80\uc7a5\ud310\uc0ac)\ub294 \ubb38\ud654,\uc608\uc220\uc778 \ube14\ub799\ub9ac\uc2a4\ud2b8\uc640 \uad00\ub828\ud558\uc5ec \uc9c1\uad8c\ub0a8\uc6a9 \uad8c\ub9ac\ud589\uc0ac \ubc29\ud574 \ub4f1\uc758 \ud610\uc758\ub85c \uad6c\uc18d\uae30\uc18c\ub41c \uae40\uae30\ucd98\uc5d0\uac8c \uaddc\uc81c\u00b7\uc870\uc138\u00b7\ubc18\ubd80\ud328\u00b7\ubb38\ud654\uc815\ucc45 \ub4f1\uc758 \ubd84\uc57c\uc5d0\uc11c \uc815\ubd80 \ub610\ub294 \uace0\uc704 \uacf5\ubb34\uc6d0\uc774 \"\uac04\uc12d\ud558\uc9c0 \uc54a\ub294\ub2e4\"\ub294 \uc6d0\uce59\uc778 \"\ud314 \uae38\uc774\ub9cc\ud07c \uac70\ub9ac\ub97c \ub454\ub2e4\"\ub97c \uc5b8\uae09\ud558\uba74\uc11c \uc9d5\uc5ed 3\ub144\uc758 \uc2e4\ud615\uc744 \uc120\uace0\ud588\ub2e4.(\uc11c\uc6b8\uc911\uc559\uc9c0\ubc29\ubc95\uc6d02017\uace0\ud569102) \uc774\uc5d0 \ubd88\ubcf5\ud55c \uae40\uae30\ucd98\uc740 \ud56d\uc18c\ub85c \uc5f4\ub9b0 \uc7ac\ud310\uc5d0\uc11c \uc11c\uc6b8\uace0\ub4f1\ubc95\uc6d0 \ud615\uc0ac3\ubd80( \uc870\uc601\ucca0 \ubd80\uc7a5\ud310\uc0ac)", "paragraph_sentence": " 2017\ub144 7\uc6d4 27\uc77c \uc11c\uc6b8\uc911\uc559\uc9c0\ubc95 \ud615\uc0ac\ud569\uc75830\ubd80(\ud669\ubcd1\ud604 \ubd80\uc7a5\ud310\uc0ac)\ub294 \ubb38\ud654,\uc608\uc220\uc778 \ube14\ub799\ub9ac\uc2a4\ud2b8\uc640 \uad00\ub828\ud558\uc5ec \uc9c1\uad8c\ub0a8\uc6a9 \uad8c\ub9ac\ud589\uc0ac \ubc29\ud574 \ub4f1\uc758 \ud610\uc758\ub85c \uad6c\uc18d\uae30\uc18c\ub41c \uae40\uae30\ucd98\uc5d0\uac8c \uaddc\uc81c\u00b7\uc870\uc138\u00b7\ubc18\ubd80\ud328\u00b7\ubb38\ud654\uc815\ucc45 \ub4f1\uc758 \ubd84\uc57c\uc5d0\uc11c \uc815\ubd80 \ub610\ub294 \uace0\uc704 \uacf5\ubb34\uc6d0\uc774 \"\uac04\uc12d\ud558\uc9c0 \uc54a\ub294\ub2e4\"\ub294 \uc6d0\uce59\uc778 \"\ud314 \uae38\uc774\ub9cc\ud07c \uac70\ub9ac\ub97c \ub454\ub2e4\"\ub97c \uc5b8\uae09\ud558\uba74\uc11c \uc9d5\uc5ed 3\ub144\uc758 \uc2e4\ud615\uc744 \uc120\uace0\ud588\ub2e4.(\uc11c\uc6b8\uc911\uc559\uc9c0\ubc29\ubc95\uc6d02017\uace0\ud569102) \uc774\uc5d0 \ubd88\ubcf5\ud55c \uae40\uae30\ucd98\uc740 \ud56d\uc18c\ub85c \uc5f4\ub9b0 \uc7ac\ud310\uc5d0\uc11c \uc11c\uc6b8\uace0\ub4f1\ubc95\uc6d0 \ud615\uc0ac3\ubd80( \uc870\uc601\ucca0 \ubd80\uc7a5\ud310\uc0ac) \ub294 2018\ub144 1\uc6d4 23\uc77c\uc5d0 \uc9c0\uc6d0 \ubc30\uc81c \ubfd0\ub9cc \uc544\ub2c8\ub77c 1\uc2ec\uc5d0\uc11c \ubb34\uc8c4 \ud310\uacb0\ub41c 1\uae09 \uacf5\ubb34\uc6d0 \uc0ac\uc9c1 \uac15\uc694 \ud610\uc758\uc5d0 \ub300\ud574\uc11c\ub3c4 \uc720\uc8c4\ub97c \uc778\uc815\ud558\uc5ec \uc9d5\uc5ed3\ub144\uc744 \uc120\uace0\ud55c \uc6d0\uc2ec\uc744 \ud30c\uae30\ud558\uba74\uc11c \uc9d5\uc5ed 4\ub144\uc744 \uc120\uace0\ud588\ub2e4.(\uc11c\uc6b8\uace0\ub4f1\ubc95\uc6d02017\ub1782425)", "paragraph_answer": "2017\ub144 7\uc6d4 27\uc77c \uc11c\uc6b8\uc911\uc559\uc9c0\ubc95 \ud615\uc0ac\ud569\uc75830\ubd80(\ud669\ubcd1\ud604 \ubd80\uc7a5\ud310\uc0ac)\ub294 \ubb38\ud654,\uc608\uc220\uc778 \ube14\ub799\ub9ac\uc2a4\ud2b8\uc640 \uad00\ub828\ud558\uc5ec \uc9c1\uad8c\ub0a8\uc6a9 \uad8c\ub9ac\ud589\uc0ac \ubc29\ud574 \ub4f1\uc758 \ud610\uc758\ub85c \uad6c\uc18d\uae30\uc18c\ub41c \uae40\uae30\ucd98\uc5d0\uac8c \uaddc\uc81c\u00b7\uc870\uc138\u00b7\ubc18\ubd80\ud328\u00b7\ubb38\ud654\uc815\ucc45 \ub4f1\uc758 \ubd84\uc57c\uc5d0\uc11c \uc815\ubd80 \ub610\ub294 \uace0\uc704 \uacf5\ubb34\uc6d0\uc774 \"\uac04\uc12d\ud558\uc9c0 \uc54a\ub294\ub2e4\"\ub294 \uc6d0\uce59\uc778 \"\ud314 \uae38\uc774\ub9cc\ud07c \uac70\ub9ac\ub97c \ub454\ub2e4\"\ub97c \uc5b8\uae09\ud558\uba74\uc11c \uc9d5\uc5ed 3\ub144\uc758 \uc2e4\ud615\uc744 \uc120\uace0\ud588\ub2e4.(\uc11c\uc6b8\uc911\uc559\uc9c0\ubc29\ubc95\uc6d02017\uace0\ud569102) \uc774\uc5d0 \ubd88\ubcf5\ud55c \uae40\uae30\ucd98\uc740 \ud56d\uc18c\ub85c \uc5f4\ub9b0 \uc7ac\ud310\uc5d0\uc11c \uc11c\uc6b8\uace0\ub4f1\ubc95\uc6d0 \ud615\uc0ac3\ubd80( \uc870\uc601\ucca0 \ubd80\uc7a5\ud310\uc0ac)\ub294 2018\ub144 1\uc6d4 23\uc77c\uc5d0 \uc9c0\uc6d0 \ubc30\uc81c \ubfd0\ub9cc \uc544\ub2c8\ub77c 1\uc2ec\uc5d0\uc11c \ubb34\uc8c4 \ud310\uacb0\ub41c 1\uae09 \uacf5\ubb34\uc6d0 \uc0ac\uc9c1 \uac15\uc694 \ud610\uc758\uc5d0 \ub300\ud574\uc11c\ub3c4 \uc720\uc8c4\ub97c \uc778\uc815\ud558\uc5ec \uc9d5\uc5ed3\ub144\uc744 \uc120\uace0\ud55c \uc6d0\uc2ec\uc744 \ud30c\uae30\ud558\uba74\uc11c \uc9d5\uc5ed 4\ub144\uc744 \uc120\uace0\ud588\ub2e4.(\uc11c\uc6b8\uace0\ub4f1\ubc95\uc6d02017\ub1782425)", "sentence_answer": "2017\ub144 7\uc6d4 27\uc77c \uc11c\uc6b8\uc911\uc559\uc9c0\ubc95 \ud615\uc0ac\ud569\uc75830\ubd80(\ud669\ubcd1\ud604 \ubd80\uc7a5\ud310\uc0ac)\ub294 \ubb38\ud654,\uc608\uc220\uc778 \ube14\ub799\ub9ac\uc2a4\ud2b8\uc640 \uad00\ub828\ud558\uc5ec \uc9c1\uad8c\ub0a8\uc6a9 \uad8c\ub9ac\ud589\uc0ac \ubc29\ud574 \ub4f1\uc758 \ud610\uc758\ub85c \uad6c\uc18d\uae30\uc18c\ub41c \uae40\uae30\ucd98\uc5d0\uac8c \uaddc\uc81c\u00b7\uc870\uc138\u00b7\ubc18\ubd80\ud328\u00b7\ubb38\ud654\uc815\ucc45 \ub4f1\uc758 \ubd84\uc57c\uc5d0\uc11c \uc815\ubd80 \ub610\ub294 \uace0\uc704 \uacf5\ubb34\uc6d0\uc774 \"\uac04\uc12d\ud558\uc9c0 \uc54a\ub294\ub2e4\"\ub294 \uc6d0\uce59\uc778 \"\ud314 \uae38\uc774\ub9cc\ud07c \uac70\ub9ac\ub97c \ub454\ub2e4\"\ub97c \uc5b8\uae09\ud558\uba74\uc11c \uc9d5\uc5ed 3\ub144\uc758 \uc2e4\ud615\uc744 \uc120\uace0\ud588\ub2e4.(\uc11c\uc6b8\uc911\uc559\uc9c0\ubc29\ubc95\uc6d02017\uace0\ud569102) \uc774\uc5d0 \ubd88\ubcf5\ud55c \uae40\uae30\ucd98\uc740 \ud56d\uc18c\ub85c \uc5f4\ub9b0 \uc7ac\ud310\uc5d0\uc11c \uc11c\uc6b8\uace0\ub4f1\ubc95\uc6d0 \ud615\uc0ac3\ubd80( \uc870\uc601\ucca0 \ubd80\uc7a5\ud310\uc0ac)", "paragraph_id": "6481327-1-1", "paragraph_question": "question: 2018\ub144 \uae40\uae30\ucd98\uc5d0\uac8c \uc9d5\uc5ed 4\ub144\uc744 \uc120\uace0\ud55c \ud310\uc0ac\uc758 \uc774\ub984\uc740?, context: 2017\ub144 7\uc6d4 27\uc77c \uc11c\uc6b8\uc911\uc559\uc9c0\ubc95 \ud615\uc0ac\ud569\uc75830\ubd80(\ud669\ubcd1\ud604 \ubd80\uc7a5\ud310\uc0ac)\ub294 \ubb38\ud654,\uc608\uc220\uc778 \ube14\ub799\ub9ac\uc2a4\ud2b8\uc640 \uad00\ub828\ud558\uc5ec \uc9c1\uad8c\ub0a8\uc6a9 \uad8c\ub9ac\ud589\uc0ac \ubc29\ud574 \ub4f1\uc758 \ud610\uc758\ub85c \uad6c\uc18d\uae30\uc18c\ub41c \uae40\uae30\ucd98\uc5d0\uac8c \uaddc\uc81c\u00b7\uc870\uc138\u00b7\ubc18\ubd80\ud328\u00b7\ubb38\ud654\uc815\ucc45 \ub4f1\uc758 \ubd84\uc57c\uc5d0\uc11c \uc815\ubd80 \ub610\ub294 \uace0\uc704 \uacf5\ubb34\uc6d0\uc774 \"\uac04\uc12d\ud558\uc9c0 \uc54a\ub294\ub2e4\"\ub294 \uc6d0\uce59\uc778 \"\ud314 \uae38\uc774\ub9cc\ud07c \uac70\ub9ac\ub97c \ub454\ub2e4\"\ub97c \uc5b8\uae09\ud558\uba74\uc11c \uc9d5\uc5ed 3\ub144\uc758 \uc2e4\ud615\uc744 \uc120\uace0\ud588\ub2e4.(\uc11c\uc6b8\uc911\uc559\uc9c0\ubc29\ubc95\uc6d02017\uace0\ud569102) \uc774\uc5d0 \ubd88\ubcf5\ud55c \uae40\uae30\ucd98\uc740 \ud56d\uc18c\ub85c \uc5f4\ub9b0 \uc7ac\ud310\uc5d0\uc11c \uc11c\uc6b8\uace0\ub4f1\ubc95\uc6d0 \ud615\uc0ac3\ubd80(\uc870\uc601\ucca0 \ubd80\uc7a5\ud310\uc0ac)\ub294 2018\ub144 1\uc6d4 23\uc77c\uc5d0 \uc9c0\uc6d0 \ubc30\uc81c \ubfd0\ub9cc \uc544\ub2c8\ub77c 1\uc2ec\uc5d0\uc11c \ubb34\uc8c4 \ud310\uacb0\ub41c 1\uae09 \uacf5\ubb34\uc6d0 \uc0ac\uc9c1 \uac15\uc694 \ud610\uc758\uc5d0 \ub300\ud574\uc11c\ub3c4 \uc720\uc8c4\ub97c \uc778\uc815\ud558\uc5ec \uc9d5\uc5ed3\ub144\uc744 \uc120\uace0\ud55c \uc6d0\uc2ec\uc744 \ud30c\uae30\ud558\uba74\uc11c \uc9d5\uc5ed 4\ub144\uc744 \uc120\uace0\ud588\ub2e4.(\uc11c\uc6b8\uace0\ub4f1\ubc95\uc6d02017\ub1782425)"} {"question": "\uc815\ubbfc\ucca0\uc740 2006\ub144 \uc5b4\ub5a4 \ud22c\uc218\ub85c\uc11c \uc7ac\uae30\uc5d0 \uc131\uacf5\ud588\ub294\uac00?", "paragraph": "\uce5c\uc815 \ud300\uc73c\ub85c \ub3cc\uc544\uc640 2002\ub144 \uc2dc\uc98c\ubd80\ud130 \ub2e4\uc2dc KBO \ub9ac\uadf8\uc5d0 \ucc38\uac00\ud558\uc9c0\ub9cc, \uc804\uacfc \uac19\uc9c0 \uc54a\uc740 \uad6c\uc704\ub85c \ub09c\uc870\ub97c \uacaa\uc5c8\ub2e4. \ud2b9\ud788 2004\ub144\uc5d0\ub294 \ub2e8 1\uc2b9\ub3c4 \uac70\ub450\uc9c0 \ubabb\ud558\uace0, \uadf8\ub3d9\uc548 \uad34\ub86d\ud600 \uc654\uc5c8\ub358 \ud314\uafc8\uce58 \ud1b5\uc99d\uc73c\ub85c \uc778\ud574 \uc218\uc220\uc744 \ubc1b\uae30\ub3c4 \ud558\uc600\ub2e4. \uc774\ud6c4 \uc7ac\uae30\ub97c \ub2e4\uc9d0\ud558\uba74\uc11c 2005\ub144\uc5d0 \uc7ac\uae30\uc5d0 \uac00\ub2a5\uc131\uc744 \ubcf4\uc600\uace0, 2006\ub144\ubd80\ud130 \uae30\uad50\ud30c \ud22c\uc218\ub85c \uc7ac\uae30\uc5d0 \uc131\uacf5\ud558\uc5ec \ud300\uc758 \uc120\ubc1c \uc8fc\ucd95\uc73c\ub85c \ud65c\uc57d\ud588\ub2e4. 2007\ub144 12\uc2b9 5\ud328 \ud3c9\uade0\uc790\ucc45\uc810 2.90\uc73c\ub85c \uc644\ubcbd\ud558\uac8c \uc7ac\uae30\ud588\ub2e4. \uadf8\ub7ec\ub098 2008\ub144 \ub178\uc1e0\ud654\ub85c \uc778\ud574 \ub2e4\uc2dc \ubd80\uc9c4\ud558\uc600\ub2e4. 2009\ub144, \ud55c\ud654 \uc774\uae00\uc2a4\uc5d0\uc11c \ud50c\ub808\uc789 \ucf54\uce58\ub85c \ud65c\ub3d9\ud558\ub2e4\uac00 \ud604\uc5ed \uc740\ud1f4\ub97c \uc120\uc5b8\ud588\uc73c\uba70, 9\uc6d4 12\uc77c \uc740\ud1f4\uc2dd\uacfc \uc601\uad6c \uacb0\ubc88\uc2dd\uc744 \uac70\ud589\ud558\uc600\ub2e4. \uc190\uac00\ub77d \uae38\uc774\uac00 \uc77c\ubc18\uc778\ubcf4\ub2e4 \ud55c \ub9c8\ub514 \uc815\ub3c4 \uae34 \ud2b9\uc9d5\uc774 \uc788\uc73c\uba70, \ud6e4\uce60\ud55c \ubbf8\ub0a8\ud615\uc73c\ub85c \ub9ce\uc740 \uc5ec\uc131\ud32c\ub4e4\uc744 \ubab0\uace0 \ub2e4\ub154\ub2e4.", "answer": "\uae30\uad50\ud30c \ud22c\uc218", "sentence": "\uc774\ud6c4 \uc7ac\uae30\ub97c \ub2e4\uc9d0\ud558\uba74\uc11c 2005\ub144\uc5d0 \uc7ac\uae30\uc5d0 \uac00\ub2a5\uc131\uc744 \ubcf4\uc600\uace0, 2006\ub144\ubd80\ud130 \uae30\uad50\ud30c \ud22c\uc218 \ub85c \uc7ac\uae30\uc5d0 \uc131\uacf5\ud558\uc5ec \ud300\uc758 \uc120\ubc1c \uc8fc\ucd95\uc73c\ub85c \ud65c\uc57d\ud588\ub2e4.", "paragraph_sentence": "\uce5c\uc815 \ud300\uc73c\ub85c \ub3cc\uc544\uc640 2002\ub144 \uc2dc\uc98c\ubd80\ud130 \ub2e4\uc2dc KBO \ub9ac\uadf8\uc5d0 \ucc38\uac00\ud558\uc9c0\ub9cc, \uc804\uacfc \uac19\uc9c0 \uc54a\uc740 \uad6c\uc704\ub85c \ub09c\uc870\ub97c \uacaa\uc5c8\ub2e4. \ud2b9\ud788 2004\ub144\uc5d0\ub294 \ub2e8 1\uc2b9\ub3c4 \uac70\ub450\uc9c0 \ubabb\ud558\uace0, \uadf8\ub3d9\uc548 \uad34\ub86d\ud600 \uc654\uc5c8\ub358 \ud314\uafc8\uce58 \ud1b5\uc99d\uc73c\ub85c \uc778\ud574 \uc218\uc220\uc744 \ubc1b\uae30\ub3c4 \ud558\uc600\ub2e4. \uc774\ud6c4 \uc7ac\uae30\ub97c \ub2e4\uc9d0\ud558\uba74\uc11c 2005\ub144\uc5d0 \uc7ac\uae30\uc5d0 \uac00\ub2a5\uc131\uc744 \ubcf4\uc600\uace0, 2006\ub144\ubd80\ud130 \uae30\uad50\ud30c \ud22c\uc218 \ub85c \uc7ac\uae30\uc5d0 \uc131\uacf5\ud558\uc5ec \ud300\uc758 \uc120\ubc1c \uc8fc\ucd95\uc73c\ub85c \ud65c\uc57d\ud588\ub2e4. 2007\ub144 12\uc2b9 5\ud328 \ud3c9\uade0\uc790\ucc45\uc810 2.90\uc73c\ub85c \uc644\ubcbd\ud558\uac8c \uc7ac\uae30\ud588\ub2e4. \uadf8\ub7ec\ub098 2008\ub144 \ub178\uc1e0\ud654\ub85c \uc778\ud574 \ub2e4\uc2dc \ubd80\uc9c4\ud558\uc600\ub2e4. 2009\ub144, \ud55c\ud654 \uc774\uae00\uc2a4\uc5d0\uc11c \ud50c\ub808\uc789 \ucf54\uce58\ub85c \ud65c\ub3d9\ud558\ub2e4\uac00 \ud604\uc5ed \uc740\ud1f4\ub97c \uc120\uc5b8\ud588\uc73c\uba70, 9\uc6d4 12\uc77c \uc740\ud1f4\uc2dd\uacfc \uc601\uad6c \uacb0\ubc88\uc2dd\uc744 \uac70\ud589\ud558\uc600\ub2e4. \uc190\uac00\ub77d \uae38\uc774\uac00 \uc77c\ubc18\uc778\ubcf4\ub2e4 \ud55c \ub9c8\ub514 \uc815\ub3c4 \uae34 \ud2b9\uc9d5\uc774 \uc788\uc73c\uba70, \ud6e4\uce60\ud55c \ubbf8\ub0a8\ud615\uc73c\ub85c \ub9ce\uc740 \uc5ec\uc131\ud32c\ub4e4\uc744 \ubab0\uace0 \ub2e4\ub154\ub2e4.", "paragraph_answer": "\uce5c\uc815 \ud300\uc73c\ub85c \ub3cc\uc544\uc640 2002\ub144 \uc2dc\uc98c\ubd80\ud130 \ub2e4\uc2dc KBO \ub9ac\uadf8\uc5d0 \ucc38\uac00\ud558\uc9c0\ub9cc, \uc804\uacfc \uac19\uc9c0 \uc54a\uc740 \uad6c\uc704\ub85c \ub09c\uc870\ub97c \uacaa\uc5c8\ub2e4. \ud2b9\ud788 2004\ub144\uc5d0\ub294 \ub2e8 1\uc2b9\ub3c4 \uac70\ub450\uc9c0 \ubabb\ud558\uace0, \uadf8\ub3d9\uc548 \uad34\ub86d\ud600 \uc654\uc5c8\ub358 \ud314\uafc8\uce58 \ud1b5\uc99d\uc73c\ub85c \uc778\ud574 \uc218\uc220\uc744 \ubc1b\uae30\ub3c4 \ud558\uc600\ub2e4. \uc774\ud6c4 \uc7ac\uae30\ub97c \ub2e4\uc9d0\ud558\uba74\uc11c 2005\ub144\uc5d0 \uc7ac\uae30\uc5d0 \uac00\ub2a5\uc131\uc744 \ubcf4\uc600\uace0, 2006\ub144\ubd80\ud130 \uae30\uad50\ud30c \ud22c\uc218 \ub85c \uc7ac\uae30\uc5d0 \uc131\uacf5\ud558\uc5ec \ud300\uc758 \uc120\ubc1c \uc8fc\ucd95\uc73c\ub85c \ud65c\uc57d\ud588\ub2e4. 2007\ub144 12\uc2b9 5\ud328 \ud3c9\uade0\uc790\ucc45\uc810 2.90\uc73c\ub85c \uc644\ubcbd\ud558\uac8c \uc7ac\uae30\ud588\ub2e4. \uadf8\ub7ec\ub098 2008\ub144 \ub178\uc1e0\ud654\ub85c \uc778\ud574 \ub2e4\uc2dc \ubd80\uc9c4\ud558\uc600\ub2e4. 2009\ub144, \ud55c\ud654 \uc774\uae00\uc2a4\uc5d0\uc11c \ud50c\ub808\uc789 \ucf54\uce58\ub85c \ud65c\ub3d9\ud558\ub2e4\uac00 \ud604\uc5ed \uc740\ud1f4\ub97c \uc120\uc5b8\ud588\uc73c\uba70, 9\uc6d4 12\uc77c \uc740\ud1f4\uc2dd\uacfc \uc601\uad6c \uacb0\ubc88\uc2dd\uc744 \uac70\ud589\ud558\uc600\ub2e4. \uc190\uac00\ub77d \uae38\uc774\uac00 \uc77c\ubc18\uc778\ubcf4\ub2e4 \ud55c \ub9c8\ub514 \uc815\ub3c4 \uae34 \ud2b9\uc9d5\uc774 \uc788\uc73c\uba70, \ud6e4\uce60\ud55c \ubbf8\ub0a8\ud615\uc73c\ub85c \ub9ce\uc740 \uc5ec\uc131\ud32c\ub4e4\uc744 \ubab0\uace0 \ub2e4\ub154\ub2e4.", "sentence_answer": "\uc774\ud6c4 \uc7ac\uae30\ub97c \ub2e4\uc9d0\ud558\uba74\uc11c 2005\ub144\uc5d0 \uc7ac\uae30\uc5d0 \uac00\ub2a5\uc131\uc744 \ubcf4\uc600\uace0, 2006\ub144\ubd80\ud130 \uae30\uad50\ud30c \ud22c\uc218 \ub85c \uc7ac\uae30\uc5d0 \uc131\uacf5\ud558\uc5ec \ud300\uc758 \uc120\ubc1c \uc8fc\ucd95\uc73c\ub85c \ud65c\uc57d\ud588\ub2e4.", "paragraph_id": "6550102-3-1", "paragraph_question": "question: \uc815\ubbfc\ucca0\uc740 2006\ub144 \uc5b4\ub5a4 \ud22c\uc218\ub85c\uc11c \uc7ac\uae30\uc5d0 \uc131\uacf5\ud588\ub294\uac00?, context: \uce5c\uc815 \ud300\uc73c\ub85c \ub3cc\uc544\uc640 2002\ub144 \uc2dc\uc98c\ubd80\ud130 \ub2e4\uc2dc KBO \ub9ac\uadf8\uc5d0 \ucc38\uac00\ud558\uc9c0\ub9cc, \uc804\uacfc \uac19\uc9c0 \uc54a\uc740 \uad6c\uc704\ub85c \ub09c\uc870\ub97c \uacaa\uc5c8\ub2e4. \ud2b9\ud788 2004\ub144\uc5d0\ub294 \ub2e8 1\uc2b9\ub3c4 \uac70\ub450\uc9c0 \ubabb\ud558\uace0, \uadf8\ub3d9\uc548 \uad34\ub86d\ud600 \uc654\uc5c8\ub358 \ud314\uafc8\uce58 \ud1b5\uc99d\uc73c\ub85c \uc778\ud574 \uc218\uc220\uc744 \ubc1b\uae30\ub3c4 \ud558\uc600\ub2e4. \uc774\ud6c4 \uc7ac\uae30\ub97c \ub2e4\uc9d0\ud558\uba74\uc11c 2005\ub144\uc5d0 \uc7ac\uae30\uc5d0 \uac00\ub2a5\uc131\uc744 \ubcf4\uc600\uace0, 2006\ub144\ubd80\ud130 \uae30\uad50\ud30c \ud22c\uc218\ub85c \uc7ac\uae30\uc5d0 \uc131\uacf5\ud558\uc5ec \ud300\uc758 \uc120\ubc1c \uc8fc\ucd95\uc73c\ub85c \ud65c\uc57d\ud588\ub2e4. 2007\ub144 12\uc2b9 5\ud328 \ud3c9\uade0\uc790\ucc45\uc810 2.90\uc73c\ub85c \uc644\ubcbd\ud558\uac8c \uc7ac\uae30\ud588\ub2e4. \uadf8\ub7ec\ub098 2008\ub144 \ub178\uc1e0\ud654\ub85c \uc778\ud574 \ub2e4\uc2dc \ubd80\uc9c4\ud558\uc600\ub2e4. 2009\ub144, \ud55c\ud654 \uc774\uae00\uc2a4\uc5d0\uc11c \ud50c\ub808\uc789 \ucf54\uce58\ub85c \ud65c\ub3d9\ud558\ub2e4\uac00 \ud604\uc5ed \uc740\ud1f4\ub97c \uc120\uc5b8\ud588\uc73c\uba70, 9\uc6d4 12\uc77c \uc740\ud1f4\uc2dd\uacfc \uc601\uad6c \uacb0\ubc88\uc2dd\uc744 \uac70\ud589\ud558\uc600\ub2e4. \uc190\uac00\ub77d \uae38\uc774\uac00 \uc77c\ubc18\uc778\ubcf4\ub2e4 \ud55c \ub9c8\ub514 \uc815\ub3c4 \uae34 \ud2b9\uc9d5\uc774 \uc788\uc73c\uba70, \ud6e4\uce60\ud55c \ubbf8\ub0a8\ud615\uc73c\ub85c \ub9ce\uc740 \uc5ec\uc131\ud32c\ub4e4\uc744 \ubab0\uace0 \ub2e4\ub154\ub2e4."} {"question": "\uc81c 12\ub300 \uad6d\ud68c\uc758\uc6d0\uc120\uac70\uac00 \uc5f4\ub9b0 \ub54c\ub294?", "paragraph": "1985\ub144 2\uc6d4 12\uc77c\uc5d0 \uc5f4\ub9b0 \uc81c12\ub300 \uad6d\ud68c\uc758\uc6d0\uc120\uac70\uc5d0\uc11c \uc0c8\ub85c \ucc3d\ub2e8\ub41c \uc2e0\ubbfc\ub2f9\uc774 \uc9c0\uc5ed\uad6c 50\uc11d\uacfc \uc804\uad6d\uad6c 17\uc11d\uc744 \ucc28\uc9c0\ud558\uc5ec \uae30\uc874\uc758 \ubbfc\ud55c\ub2f9\uacfc \uad6d\ubbfc\ub2f9\uc744 \ub204\ub974\uace0 \uc6d0\ub0b4 \uc81c 1\uc57c\ub2f9\uc774 \ub418\uc5c8\ub2e4. \uc5ec\ub2f9\uc778 \ubbfc\uc815\ub2f9\uc774 \uae30\ub300\ud55c \ub9cc\ud07c \uc758\uc11d\uc744 \ub298\ub9ac\uc9c0 \ubabb\ud558\uc790 \uc548\uae30\ubd80\uc7a5\uc778 \ub178\uc2e0\uc601\uc740 \uadf8\uc5d0 \ub300\ud55c \ucc45\uc784\uc73c\ub85c \uc804\ub450\ud658\uc5d0\uac8c \uc0ac\ud45c\ub97c \uc81c\ucd9c\ud558\uc600\ub2e4. \ud558\uc9c0\ub9cc \uc804\ub450\ud658\uc740 \uc0ac\ud45c\ub97c \uac70\ubd80\ud55c\ucc44 1\ub144\uac04\uc740 \uadf8\ub300\ub85c \uc720\uc784\uc2dc\ud0a4\uaca0\ub2e4\uba70 \uc548\uae30\ubd80 \uac04\ubd80\ud68c\uc758\ub97c \uc18c\uc9d1\ud558\uace0 \uc774 \uc0ac\uc2e4\uc744 \uc54c\ub9ac\ub77c\uace0 \ud55c\ub2e4. \uadf8\ub798\ub3c4 \ub178\uc2e0\uc601\uc774 \uc7ac\uc0bc \uc0ac\uc758\ub97c \ud45c\uba85\ud558\uc790 \uc804\ub450\ud658\uc740 \uadf8\uc758 \ub9d0\uc740 \ub4e3\uc9c0 \uc54a\uc73c\uba70 \"\ube68\ub9ac \ub0b4\ub824\uac00\ub77c\"\uace0 \uc9c0\uc2dc\ud558\uc600\uace0 \uc5b4\uca54 \uc218 \uc5c6\uc774 \uadf8\ub0a0 \ud68c\uc758\ub97c \uc5f4\uc5b4 \uc720\uc784\uc18c\uc2dd\uc744 \uc54c\ub838\ub2e4. \uadf8\ub7ec\ub098 \uc218\uc77c\uc774 \uc9c0\ub09c\ud6c4 \ub73b\ubc16\uc5d0\ub3c4 \ub178\uc2e0\uc601\uc740 \uad6d\ubb34\ucd1d\ub9ac\uc5d0 \uc784\uba85\ub41c\ub2e4. \ub178\uc2e0\uc601\uc740 \ud55c \uc2e0\ubb38\uc0ac\uc640\uc758 \uc778\ud130\ubdf0\uc5d0\uc11c \"\ub098\ub294 \ub204\uac00 \uc548\uae30\ubd80\uc7a5\uc778\uc9c0\ub97c \uad6d\ubbfc\ub4e4\uc774 \ubaa8\ub974\uba74 \ubaa8\ub97c \uc218\ub85d \uc0ac\ud68c\ub294 \uc548\uc815\ub418\uace0 \uad6d\uc815\uc740 \uc798\ub41c\ub2e4\uace0 \uc0dd\uac01\ud558\uc600\uc73c\uba70, \uc9c0\uae08\ub3c4 \uadf8 \uc0dd\uac01\uc5d0\ub294 \ubcc0\ud568\uc774 \uc5c6\ub2e4.\"\ub77c\ub294 \uc548\uae30\ubd80\uc7a5\uc73c\ub85c\uc11c\uc758 \uc2e0\ub150\uc744 \ub0b4\ube44\ucce4\ub2e4.", "answer": "1985\ub144 2\uc6d4 12\uc77c", "sentence": "1985\ub144 2\uc6d4 12\uc77c \uc5d0 \uc5f4\ub9b0 \uc81c12\ub300 \uad6d\ud68c\uc758\uc6d0\uc120\uac70\uc5d0\uc11c \uc0c8\ub85c \ucc3d\ub2e8\ub41c \uc2e0\ubbfc\ub2f9\uc774 \uc9c0\uc5ed\uad6c 50\uc11d\uacfc \uc804\uad6d\uad6c 17\uc11d\uc744 \ucc28\uc9c0\ud558\uc5ec \uae30\uc874\uc758 \ubbfc\ud55c\ub2f9\uacfc \uad6d\ubbfc\ub2f9\uc744 \ub204\ub974\uace0 \uc6d0\ub0b4 \uc81c 1\uc57c\ub2f9\uc774 \ub418\uc5c8\ub2e4.", "paragraph_sentence": " 1985\ub144 2\uc6d4 12\uc77c \uc5d0 \uc5f4\ub9b0 \uc81c12\ub300 \uad6d\ud68c\uc758\uc6d0\uc120\uac70\uc5d0\uc11c \uc0c8\ub85c \ucc3d\ub2e8\ub41c \uc2e0\ubbfc\ub2f9\uc774 \uc9c0\uc5ed\uad6c 50\uc11d\uacfc \uc804\uad6d\uad6c 17\uc11d\uc744 \ucc28\uc9c0\ud558\uc5ec \uae30\uc874\uc758 \ubbfc\ud55c\ub2f9\uacfc \uad6d\ubbfc\ub2f9\uc744 \ub204\ub974\uace0 \uc6d0\ub0b4 \uc81c 1\uc57c\ub2f9\uc774 \ub418\uc5c8\ub2e4. \uc5ec\ub2f9\uc778 \ubbfc\uc815\ub2f9\uc774 \uae30\ub300\ud55c \ub9cc\ud07c \uc758\uc11d\uc744 \ub298\ub9ac\uc9c0 \ubabb\ud558\uc790 \uc548\uae30\ubd80\uc7a5\uc778 \ub178\uc2e0\uc601\uc740 \uadf8\uc5d0 \ub300\ud55c \ucc45\uc784\uc73c\ub85c \uc804\ub450\ud658\uc5d0\uac8c \uc0ac\ud45c\ub97c \uc81c\ucd9c\ud558\uc600\ub2e4. \ud558\uc9c0\ub9cc \uc804\ub450\ud658\uc740 \uc0ac\ud45c\ub97c \uac70\ubd80\ud55c\ucc44 1\ub144\uac04\uc740 \uadf8\ub300\ub85c \uc720\uc784\uc2dc\ud0a4\uaca0\ub2e4\uba70 \uc548\uae30\ubd80 \uac04\ubd80\ud68c\uc758\ub97c \uc18c\uc9d1\ud558\uace0 \uc774 \uc0ac\uc2e4\uc744 \uc54c\ub9ac\ub77c\uace0 \ud55c\ub2e4. \uadf8\ub798\ub3c4 \ub178\uc2e0\uc601\uc774 \uc7ac\uc0bc \uc0ac\uc758\ub97c \ud45c\uba85\ud558\uc790 \uc804\ub450\ud658\uc740 \uadf8\uc758 \ub9d0\uc740 \ub4e3\uc9c0 \uc54a\uc73c\uba70 \"\ube68\ub9ac \ub0b4\ub824\uac00\ub77c\"\uace0 \uc9c0\uc2dc\ud558\uc600\uace0 \uc5b4\uca54 \uc218 \uc5c6\uc774 \uadf8\ub0a0 \ud68c\uc758\ub97c \uc5f4\uc5b4 \uc720\uc784\uc18c\uc2dd\uc744 \uc54c\ub838\ub2e4. \uadf8\ub7ec\ub098 \uc218\uc77c\uc774 \uc9c0\ub09c\ud6c4 \ub73b\ubc16\uc5d0\ub3c4 \ub178\uc2e0\uc601\uc740 \uad6d\ubb34\ucd1d\ub9ac\uc5d0 \uc784\uba85\ub41c\ub2e4. \ub178\uc2e0\uc601\uc740 \ud55c \uc2e0\ubb38\uc0ac\uc640\uc758 \uc778\ud130\ubdf0\uc5d0\uc11c \"\ub098\ub294 \ub204\uac00 \uc548\uae30\ubd80\uc7a5\uc778\uc9c0\ub97c \uad6d\ubbfc\ub4e4\uc774 \ubaa8\ub974\uba74 \ubaa8\ub97c \uc218\ub85d \uc0ac\ud68c\ub294 \uc548\uc815\ub418\uace0 \uad6d\uc815\uc740 \uc798\ub41c\ub2e4\uace0 \uc0dd\uac01\ud558\uc600\uc73c\uba70, \uc9c0\uae08\ub3c4 \uadf8 \uc0dd\uac01\uc5d0\ub294 \ubcc0\ud568\uc774 \uc5c6\ub2e4. \"\ub77c\ub294 \uc548\uae30\ubd80\uc7a5\uc73c\ub85c\uc11c\uc758 \uc2e0\ub150\uc744 \ub0b4\ube44\ucce4\ub2e4.", "paragraph_answer": " 1985\ub144 2\uc6d4 12\uc77c \uc5d0 \uc5f4\ub9b0 \uc81c12\ub300 \uad6d\ud68c\uc758\uc6d0\uc120\uac70\uc5d0\uc11c \uc0c8\ub85c \ucc3d\ub2e8\ub41c \uc2e0\ubbfc\ub2f9\uc774 \uc9c0\uc5ed\uad6c 50\uc11d\uacfc \uc804\uad6d\uad6c 17\uc11d\uc744 \ucc28\uc9c0\ud558\uc5ec \uae30\uc874\uc758 \ubbfc\ud55c\ub2f9\uacfc \uad6d\ubbfc\ub2f9\uc744 \ub204\ub974\uace0 \uc6d0\ub0b4 \uc81c 1\uc57c\ub2f9\uc774 \ub418\uc5c8\ub2e4. \uc5ec\ub2f9\uc778 \ubbfc\uc815\ub2f9\uc774 \uae30\ub300\ud55c \ub9cc\ud07c \uc758\uc11d\uc744 \ub298\ub9ac\uc9c0 \ubabb\ud558\uc790 \uc548\uae30\ubd80\uc7a5\uc778 \ub178\uc2e0\uc601\uc740 \uadf8\uc5d0 \ub300\ud55c \ucc45\uc784\uc73c\ub85c \uc804\ub450\ud658\uc5d0\uac8c \uc0ac\ud45c\ub97c \uc81c\ucd9c\ud558\uc600\ub2e4. \ud558\uc9c0\ub9cc \uc804\ub450\ud658\uc740 \uc0ac\ud45c\ub97c \uac70\ubd80\ud55c\ucc44 1\ub144\uac04\uc740 \uadf8\ub300\ub85c \uc720\uc784\uc2dc\ud0a4\uaca0\ub2e4\uba70 \uc548\uae30\ubd80 \uac04\ubd80\ud68c\uc758\ub97c \uc18c\uc9d1\ud558\uace0 \uc774 \uc0ac\uc2e4\uc744 \uc54c\ub9ac\ub77c\uace0 \ud55c\ub2e4. \uadf8\ub798\ub3c4 \ub178\uc2e0\uc601\uc774 \uc7ac\uc0bc \uc0ac\uc758\ub97c \ud45c\uba85\ud558\uc790 \uc804\ub450\ud658\uc740 \uadf8\uc758 \ub9d0\uc740 \ub4e3\uc9c0 \uc54a\uc73c\uba70 \"\ube68\ub9ac \ub0b4\ub824\uac00\ub77c\"\uace0 \uc9c0\uc2dc\ud558\uc600\uace0 \uc5b4\uca54 \uc218 \uc5c6\uc774 \uadf8\ub0a0 \ud68c\uc758\ub97c \uc5f4\uc5b4 \uc720\uc784\uc18c\uc2dd\uc744 \uc54c\ub838\ub2e4. \uadf8\ub7ec\ub098 \uc218\uc77c\uc774 \uc9c0\ub09c\ud6c4 \ub73b\ubc16\uc5d0\ub3c4 \ub178\uc2e0\uc601\uc740 \uad6d\ubb34\ucd1d\ub9ac\uc5d0 \uc784\uba85\ub41c\ub2e4. \ub178\uc2e0\uc601\uc740 \ud55c \uc2e0\ubb38\uc0ac\uc640\uc758 \uc778\ud130\ubdf0\uc5d0\uc11c \"\ub098\ub294 \ub204\uac00 \uc548\uae30\ubd80\uc7a5\uc778\uc9c0\ub97c \uad6d\ubbfc\ub4e4\uc774 \ubaa8\ub974\uba74 \ubaa8\ub97c \uc218\ub85d \uc0ac\ud68c\ub294 \uc548\uc815\ub418\uace0 \uad6d\uc815\uc740 \uc798\ub41c\ub2e4\uace0 \uc0dd\uac01\ud558\uc600\uc73c\uba70, \uc9c0\uae08\ub3c4 \uadf8 \uc0dd\uac01\uc5d0\ub294 \ubcc0\ud568\uc774 \uc5c6\ub2e4.\"\ub77c\ub294 \uc548\uae30\ubd80\uc7a5\uc73c\ub85c\uc11c\uc758 \uc2e0\ub150\uc744 \ub0b4\ube44\ucce4\ub2e4.", "sentence_answer": " 1985\ub144 2\uc6d4 12\uc77c \uc5d0 \uc5f4\ub9b0 \uc81c12\ub300 \uad6d\ud68c\uc758\uc6d0\uc120\uac70\uc5d0\uc11c \uc0c8\ub85c \ucc3d\ub2e8\ub41c \uc2e0\ubbfc\ub2f9\uc774 \uc9c0\uc5ed\uad6c 50\uc11d\uacfc \uc804\uad6d\uad6c 17\uc11d\uc744 \ucc28\uc9c0\ud558\uc5ec \uae30\uc874\uc758 \ubbfc\ud55c\ub2f9\uacfc \uad6d\ubbfc\ub2f9\uc744 \ub204\ub974\uace0 \uc6d0\ub0b4 \uc81c 1\uc57c\ub2f9\uc774 \ub418\uc5c8\ub2e4.", "paragraph_id": "6577408-4-0", "paragraph_question": "question: \uc81c 12\ub300 \uad6d\ud68c\uc758\uc6d0\uc120\uac70\uac00 \uc5f4\ub9b0 \ub54c\ub294?, context: 1985\ub144 2\uc6d4 12\uc77c\uc5d0 \uc5f4\ub9b0 \uc81c12\ub300 \uad6d\ud68c\uc758\uc6d0\uc120\uac70\uc5d0\uc11c \uc0c8\ub85c \ucc3d\ub2e8\ub41c \uc2e0\ubbfc\ub2f9\uc774 \uc9c0\uc5ed\uad6c 50\uc11d\uacfc \uc804\uad6d\uad6c 17\uc11d\uc744 \ucc28\uc9c0\ud558\uc5ec \uae30\uc874\uc758 \ubbfc\ud55c\ub2f9\uacfc \uad6d\ubbfc\ub2f9\uc744 \ub204\ub974\uace0 \uc6d0\ub0b4 \uc81c 1\uc57c\ub2f9\uc774 \ub418\uc5c8\ub2e4. \uc5ec\ub2f9\uc778 \ubbfc\uc815\ub2f9\uc774 \uae30\ub300\ud55c \ub9cc\ud07c \uc758\uc11d\uc744 \ub298\ub9ac\uc9c0 \ubabb\ud558\uc790 \uc548\uae30\ubd80\uc7a5\uc778 \ub178\uc2e0\uc601\uc740 \uadf8\uc5d0 \ub300\ud55c \ucc45\uc784\uc73c\ub85c \uc804\ub450\ud658\uc5d0\uac8c \uc0ac\ud45c\ub97c \uc81c\ucd9c\ud558\uc600\ub2e4. \ud558\uc9c0\ub9cc \uc804\ub450\ud658\uc740 \uc0ac\ud45c\ub97c \uac70\ubd80\ud55c\ucc44 1\ub144\uac04\uc740 \uadf8\ub300\ub85c \uc720\uc784\uc2dc\ud0a4\uaca0\ub2e4\uba70 \uc548\uae30\ubd80 \uac04\ubd80\ud68c\uc758\ub97c \uc18c\uc9d1\ud558\uace0 \uc774 \uc0ac\uc2e4\uc744 \uc54c\ub9ac\ub77c\uace0 \ud55c\ub2e4. \uadf8\ub798\ub3c4 \ub178\uc2e0\uc601\uc774 \uc7ac\uc0bc \uc0ac\uc758\ub97c \ud45c\uba85\ud558\uc790 \uc804\ub450\ud658\uc740 \uadf8\uc758 \ub9d0\uc740 \ub4e3\uc9c0 \uc54a\uc73c\uba70 \"\ube68\ub9ac \ub0b4\ub824\uac00\ub77c\"\uace0 \uc9c0\uc2dc\ud558\uc600\uace0 \uc5b4\uca54 \uc218 \uc5c6\uc774 \uadf8\ub0a0 \ud68c\uc758\ub97c \uc5f4\uc5b4 \uc720\uc784\uc18c\uc2dd\uc744 \uc54c\ub838\ub2e4. \uadf8\ub7ec\ub098 \uc218\uc77c\uc774 \uc9c0\ub09c\ud6c4 \ub73b\ubc16\uc5d0\ub3c4 \ub178\uc2e0\uc601\uc740 \uad6d\ubb34\ucd1d\ub9ac\uc5d0 \uc784\uba85\ub41c\ub2e4. \ub178\uc2e0\uc601\uc740 \ud55c \uc2e0\ubb38\uc0ac\uc640\uc758 \uc778\ud130\ubdf0\uc5d0\uc11c \"\ub098\ub294 \ub204\uac00 \uc548\uae30\ubd80\uc7a5\uc778\uc9c0\ub97c \uad6d\ubbfc\ub4e4\uc774 \ubaa8\ub974\uba74 \ubaa8\ub97c \uc218\ub85d \uc0ac\ud68c\ub294 \uc548\uc815\ub418\uace0 \uad6d\uc815\uc740 \uc798\ub41c\ub2e4\uace0 \uc0dd\uac01\ud558\uc600\uc73c\uba70, \uc9c0\uae08\ub3c4 \uadf8 \uc0dd\uac01\uc5d0\ub294 \ubcc0\ud568\uc774 \uc5c6\ub2e4.\"\ub77c\ub294 \uc548\uae30\ubd80\uc7a5\uc73c\ub85c\uc11c\uc758 \uc2e0\ub150\uc744 \ub0b4\ube44\ucce4\ub2e4."} {"question": "\uc5b8\ub9ac\uc5bc\uacfc \ub354\ubd88\uc5b4 <\uc2dc\uc2a4\ud15c \uc1fc\ud06c 2> \ub808\ubca8 \uc81c\uc791\uc790\ub4e4\uc5d0\uac8c \uc775\uc219\ud588\ub358 \uac8c\uc784\ud50c\ub808\uc774 \ud615\ud0dc\ub294?", "paragraph": "\uadf8\ub4e4\uc740 \ub808\ubca8 \ub514\uc790\uc778\uc5d0 \ub4e4\uc5b4\uac08 \ub178\ub825\uc744 \uc608\uc0c1\ud558\uc9c0 \ubabb \ud55c \uac83\uc744 \uac1c\ubc1c \uacfc\uc815\uc5d0\uc11c \uc798\ubabb\ub41c \uc810\uc73c\ub85c \ub4e4\uc5c8\ub2e4. \u300a\uc2dc\uc2a4\ud15c \uc1fc\ud06c 2\u300b\uc758 \ub9ce\uc740 \uac1d\uccb4\uc640 \uc2a4\ud06c\ub9bd\ud2b8, \ub9e4\uac1c \ubcc0\uc218\ub294 \ub808\ubca8 \ub514\uc790\uc774\ub108\uc5d0\uac8c \ub9ce\uc740 \uc791\uc5c5\ub7c9\uc744 \ubd80\uacfc\ud588\ub2e4. \uac8c\uc784 \uc0c1\uc5d0\uc11c \ub808\ubca8 \uac04 \uc774\ub3d9\uc774 \uc790\uc720\ub85c\uc6cc \ud55c \ub808\ubca8\ub9cc\uc744 \ub3c5\ub9bd\uc801\uc73c\ub85c \uc870\uc808\ud560 \uc218\ub3c4 \uc5c6\uc5c8\uace0, \uc0c1\ub300\uc801\uc73c\ub85c \uc791\uc740 \ubcc0\ud654\uc5d0\ub3c4 \ub9ce\uc740 \ub178\ub825\uc744 \uae30\uc6b8\uc5ec\uc57c \ud588\ub2e4. \uadf8\ub4e4\uc740 \ud558\ub098\uc758 \ub808\ubca8\uc744 \ub9cc\ub4dc\ub294 \uacfc\uc815\uc744 \ub2e8\uacc4\ubcc4\ub85c \ub098\ub204\uc5b4 \ub808\ubca8 \uc791\uc5c5\uc744 \uae30\ubbfc\ud558\uac8c \uc81c\uc5b4\ud558\ub824 \ud588\uc9c0\ub9cc, \ub808\ubca8 \uc81c\uc791 \ub2e8\uacc4\uac00 \uc644\uc804\ud788 \ubd84\ub9ac\ub420 \uc218 \uc788\ub294 \uac83\uc774 \uc544\ub2c8\uc5c8\uace0 \ub808\ubca8(\ud639\uc740 \uc791\uc5c5 \ub2e8\uacc4) \uac04\uc5d0 \uc778\ub825\uc744 \ubc30\uce58\ud558\ub294 \uac83\ub3c4 \uc26c\uc6b4 \uc77c\uc774 \uc544\ub2c8\uc5c8\ub2e4. \uac8c\ub2e4\uac00 \u300a\uc2dc\uc2a4\ud15c \uc1fc\ud06c 2\u300b\uac00 \uc758\ub3c4\ud558\ub294 \uac8c\uc784\ud50c\ub808\uc774 \ud615\ud0dc\uac00 \ub808\ubca8 \uc81c\uc791\uc790\ub4e4\uc5d0\uac8c \uc775\uc219\ud55c \uc5b8\ub9ac\uc5bc\uc774\ub098 \ud018\uc774\ud06c\uc640\ub294 \ub2ec\ub790\ub358 \uac83\ub3c4 \ubb38\uc81c\uc600\ub2e4.", "answer": "\ud018\uc774\ud06c", "sentence": "\uac8c\ub2e4\uac00 \u300a\uc2dc\uc2a4\ud15c \uc1fc\ud06c 2\u300b\uac00 \uc758\ub3c4\ud558\ub294 \uac8c\uc784\ud50c\ub808\uc774 \ud615\ud0dc\uac00 \ub808\ubca8 \uc81c\uc791\uc790\ub4e4\uc5d0\uac8c \uc775\uc219\ud55c \uc5b8\ub9ac\uc5bc\uc774\ub098 \ud018\uc774\ud06c \uc640\ub294 \ub2ec\ub790\ub358 \uac83\ub3c4 \ubb38\uc81c\uc600\ub2e4.", "paragraph_sentence": "\uadf8\ub4e4\uc740 \ub808\ubca8 \ub514\uc790\uc778\uc5d0 \ub4e4\uc5b4\uac08 \ub178\ub825\uc744 \uc608\uc0c1\ud558\uc9c0 \ubabb \ud55c \uac83\uc744 \uac1c\ubc1c \uacfc\uc815\uc5d0\uc11c \uc798\ubabb\ub41c \uc810\uc73c\ub85c \ub4e4\uc5c8\ub2e4. \u300a\uc2dc\uc2a4\ud15c \uc1fc\ud06c 2\u300b\uc758 \ub9ce\uc740 \uac1d\uccb4\uc640 \uc2a4\ud06c\ub9bd\ud2b8, \ub9e4\uac1c \ubcc0\uc218\ub294 \ub808\ubca8 \ub514\uc790\uc774\ub108\uc5d0\uac8c \ub9ce\uc740 \uc791\uc5c5\ub7c9\uc744 \ubd80\uacfc\ud588\ub2e4. \uac8c\uc784 \uc0c1\uc5d0\uc11c \ub808\ubca8 \uac04 \uc774\ub3d9\uc774 \uc790\uc720\ub85c\uc6cc \ud55c \ub808\ubca8\ub9cc\uc744 \ub3c5\ub9bd\uc801\uc73c\ub85c \uc870\uc808\ud560 \uc218\ub3c4 \uc5c6\uc5c8\uace0, \uc0c1\ub300\uc801\uc73c\ub85c \uc791\uc740 \ubcc0\ud654\uc5d0\ub3c4 \ub9ce\uc740 \ub178\ub825\uc744 \uae30\uc6b8\uc5ec\uc57c \ud588\ub2e4. \uadf8\ub4e4\uc740 \ud558\ub098\uc758 \ub808\ubca8\uc744 \ub9cc\ub4dc\ub294 \uacfc\uc815\uc744 \ub2e8\uacc4\ubcc4\ub85c \ub098\ub204\uc5b4 \ub808\ubca8 \uc791\uc5c5\uc744 \uae30\ubbfc\ud558\uac8c \uc81c\uc5b4\ud558\ub824 \ud588\uc9c0\ub9cc, \ub808\ubca8 \uc81c\uc791 \ub2e8\uacc4\uac00 \uc644\uc804\ud788 \ubd84\ub9ac\ub420 \uc218 \uc788\ub294 \uac83\uc774 \uc544\ub2c8\uc5c8\uace0 \ub808\ubca8(\ud639\uc740 \uc791\uc5c5 \ub2e8\uacc4) \uac04\uc5d0 \uc778\ub825\uc744 \ubc30\uce58\ud558\ub294 \uac83\ub3c4 \uc26c\uc6b4 \uc77c\uc774 \uc544\ub2c8\uc5c8\ub2e4. \uac8c\ub2e4\uac00 \u300a\uc2dc\uc2a4\ud15c \uc1fc\ud06c 2\u300b\uac00 \uc758\ub3c4\ud558\ub294 \uac8c\uc784\ud50c\ub808\uc774 \ud615\ud0dc\uac00 \ub808\ubca8 \uc81c\uc791\uc790\ub4e4\uc5d0\uac8c \uc775\uc219\ud55c \uc5b8\ub9ac\uc5bc\uc774\ub098 \ud018\uc774\ud06c \uc640\ub294 \ub2ec\ub790\ub358 \uac83\ub3c4 \ubb38\uc81c\uc600\ub2e4. ", "paragraph_answer": "\uadf8\ub4e4\uc740 \ub808\ubca8 \ub514\uc790\uc778\uc5d0 \ub4e4\uc5b4\uac08 \ub178\ub825\uc744 \uc608\uc0c1\ud558\uc9c0 \ubabb \ud55c \uac83\uc744 \uac1c\ubc1c \uacfc\uc815\uc5d0\uc11c \uc798\ubabb\ub41c \uc810\uc73c\ub85c \ub4e4\uc5c8\ub2e4. \u300a\uc2dc\uc2a4\ud15c \uc1fc\ud06c 2\u300b\uc758 \ub9ce\uc740 \uac1d\uccb4\uc640 \uc2a4\ud06c\ub9bd\ud2b8, \ub9e4\uac1c \ubcc0\uc218\ub294 \ub808\ubca8 \ub514\uc790\uc774\ub108\uc5d0\uac8c \ub9ce\uc740 \uc791\uc5c5\ub7c9\uc744 \ubd80\uacfc\ud588\ub2e4. \uac8c\uc784 \uc0c1\uc5d0\uc11c \ub808\ubca8 \uac04 \uc774\ub3d9\uc774 \uc790\uc720\ub85c\uc6cc \ud55c \ub808\ubca8\ub9cc\uc744 \ub3c5\ub9bd\uc801\uc73c\ub85c \uc870\uc808\ud560 \uc218\ub3c4 \uc5c6\uc5c8\uace0, \uc0c1\ub300\uc801\uc73c\ub85c \uc791\uc740 \ubcc0\ud654\uc5d0\ub3c4 \ub9ce\uc740 \ub178\ub825\uc744 \uae30\uc6b8\uc5ec\uc57c \ud588\ub2e4. \uadf8\ub4e4\uc740 \ud558\ub098\uc758 \ub808\ubca8\uc744 \ub9cc\ub4dc\ub294 \uacfc\uc815\uc744 \ub2e8\uacc4\ubcc4\ub85c \ub098\ub204\uc5b4 \ub808\ubca8 \uc791\uc5c5\uc744 \uae30\ubbfc\ud558\uac8c \uc81c\uc5b4\ud558\ub824 \ud588\uc9c0\ub9cc, \ub808\ubca8 \uc81c\uc791 \ub2e8\uacc4\uac00 \uc644\uc804\ud788 \ubd84\ub9ac\ub420 \uc218 \uc788\ub294 \uac83\uc774 \uc544\ub2c8\uc5c8\uace0 \ub808\ubca8(\ud639\uc740 \uc791\uc5c5 \ub2e8\uacc4) \uac04\uc5d0 \uc778\ub825\uc744 \ubc30\uce58\ud558\ub294 \uac83\ub3c4 \uc26c\uc6b4 \uc77c\uc774 \uc544\ub2c8\uc5c8\ub2e4. \uac8c\ub2e4\uac00 \u300a\uc2dc\uc2a4\ud15c \uc1fc\ud06c 2\u300b\uac00 \uc758\ub3c4\ud558\ub294 \uac8c\uc784\ud50c\ub808\uc774 \ud615\ud0dc\uac00 \ub808\ubca8 \uc81c\uc791\uc790\ub4e4\uc5d0\uac8c \uc775\uc219\ud55c \uc5b8\ub9ac\uc5bc\uc774\ub098 \ud018\uc774\ud06c \uc640\ub294 \ub2ec\ub790\ub358 \uac83\ub3c4 \ubb38\uc81c\uc600\ub2e4.", "sentence_answer": "\uac8c\ub2e4\uac00 \u300a\uc2dc\uc2a4\ud15c \uc1fc\ud06c 2\u300b\uac00 \uc758\ub3c4\ud558\ub294 \uac8c\uc784\ud50c\ub808\uc774 \ud615\ud0dc\uac00 \ub808\ubca8 \uc81c\uc791\uc790\ub4e4\uc5d0\uac8c \uc775\uc219\ud55c \uc5b8\ub9ac\uc5bc\uc774\ub098 \ud018\uc774\ud06c \uc640\ub294 \ub2ec\ub790\ub358 \uac83\ub3c4 \ubb38\uc81c\uc600\ub2e4.", "paragraph_id": "6519486-2-1", "paragraph_question": "question: \uc5b8\ub9ac\uc5bc\uacfc \ub354\ubd88\uc5b4 <\uc2dc\uc2a4\ud15c \uc1fc\ud06c 2> \ub808\ubca8 \uc81c\uc791\uc790\ub4e4\uc5d0\uac8c \uc775\uc219\ud588\ub358 \uac8c\uc784\ud50c\ub808\uc774 \ud615\ud0dc\ub294?, context: \uadf8\ub4e4\uc740 \ub808\ubca8 \ub514\uc790\uc778\uc5d0 \ub4e4\uc5b4\uac08 \ub178\ub825\uc744 \uc608\uc0c1\ud558\uc9c0 \ubabb \ud55c \uac83\uc744 \uac1c\ubc1c \uacfc\uc815\uc5d0\uc11c \uc798\ubabb\ub41c \uc810\uc73c\ub85c \ub4e4\uc5c8\ub2e4. \u300a\uc2dc\uc2a4\ud15c \uc1fc\ud06c 2\u300b\uc758 \ub9ce\uc740 \uac1d\uccb4\uc640 \uc2a4\ud06c\ub9bd\ud2b8, \ub9e4\uac1c \ubcc0\uc218\ub294 \ub808\ubca8 \ub514\uc790\uc774\ub108\uc5d0\uac8c \ub9ce\uc740 \uc791\uc5c5\ub7c9\uc744 \ubd80\uacfc\ud588\ub2e4. \uac8c\uc784 \uc0c1\uc5d0\uc11c \ub808\ubca8 \uac04 \uc774\ub3d9\uc774 \uc790\uc720\ub85c\uc6cc \ud55c \ub808\ubca8\ub9cc\uc744 \ub3c5\ub9bd\uc801\uc73c\ub85c \uc870\uc808\ud560 \uc218\ub3c4 \uc5c6\uc5c8\uace0, \uc0c1\ub300\uc801\uc73c\ub85c \uc791\uc740 \ubcc0\ud654\uc5d0\ub3c4 \ub9ce\uc740 \ub178\ub825\uc744 \uae30\uc6b8\uc5ec\uc57c \ud588\ub2e4. \uadf8\ub4e4\uc740 \ud558\ub098\uc758 \ub808\ubca8\uc744 \ub9cc\ub4dc\ub294 \uacfc\uc815\uc744 \ub2e8\uacc4\ubcc4\ub85c \ub098\ub204\uc5b4 \ub808\ubca8 \uc791\uc5c5\uc744 \uae30\ubbfc\ud558\uac8c \uc81c\uc5b4\ud558\ub824 \ud588\uc9c0\ub9cc, \ub808\ubca8 \uc81c\uc791 \ub2e8\uacc4\uac00 \uc644\uc804\ud788 \ubd84\ub9ac\ub420 \uc218 \uc788\ub294 \uac83\uc774 \uc544\ub2c8\uc5c8\uace0 \ub808\ubca8(\ud639\uc740 \uc791\uc5c5 \ub2e8\uacc4) \uac04\uc5d0 \uc778\ub825\uc744 \ubc30\uce58\ud558\ub294 \uac83\ub3c4 \uc26c\uc6b4 \uc77c\uc774 \uc544\ub2c8\uc5c8\ub2e4. \uac8c\ub2e4\uac00 \u300a\uc2dc\uc2a4\ud15c \uc1fc\ud06c 2\u300b\uac00 \uc758\ub3c4\ud558\ub294 \uac8c\uc784\ud50c\ub808\uc774 \ud615\ud0dc\uac00 \ub808\ubca8 \uc81c\uc791\uc790\ub4e4\uc5d0\uac8c \uc775\uc219\ud55c \uc5b8\ub9ac\uc5bc\uc774\ub098 \ud018\uc774\ud06c\uc640\ub294 \ub2ec\ub790\ub358 \uac83\ub3c4 \ubb38\uc81c\uc600\ub2e4."} {"question": "\ud0dc\uc591 \uc6d0\ubc18\uc744 \uc81c\uc791\ud55c \ub098\ub77c\ub294 \uc5b4\ub514\uc778\uac00?", "paragraph": "\uc774\uc9d1\ud2b8 \uc885\uad50\ub294 \uace0\ub300 \uc774\uc9d1\ud2b8 \uc0ac\ud68c\ub97c \uac00\uc7a5 \uc798 \ubcf4\uc5ec\uc8fc\ub294 \uae30\ub150\ubb3c\uc778 \uc218 \ub9ce\uc740 \uc2e0\uc804\ub4e4\uacfc \ubb34\ub364\ub4e4\uc744 \uc0dd\uc0b0\ud558\uc600\uc73c\uba70 \uc218 \ub9ce\uc740 \ubb38\ud654\ub4e4\uc5d0 \uac70\ub300\ud55c \uc601\ud5a5\uc744 \ub0a8\uacbc\ub2e4. \ud30c\ub77c\uc624 \uc2dc\ub300\uc758 \uc2a4\ud551\ud06c\uc2a4\uc640 \ud0dc\uc591 \uc6d0\ubc18\ub4f1\uc744 \ube44\ub86f\ud55c \ub2e4\uc591\ud55c \uc0c1\uc9d5\uacfc \uae30\ud638\ub4e4\uc740 \uc9c0\uc911\ud574\uc640 \uadfc\ub3d9\uc744 \ub118\uc5b4 \uad11\ubc94\uc704\ud558\uac8c \ud37c\uc838\ub098\uac14\ub2e4. \uadf8\ub9ac\uc2a4\uc778\ub4e4\uc758 \uc5d8\ub9ac\uc2dc\uc628\uc758 \ubc1c\uc0c1\uc740 \uc774\uc9d1\ud2b8\uc758 \uc0ac\ud6c4 \uc138\uacc4\uc5d0\uc11c \ube44\ub86f\ub41c \uac83\uc73c\ub85c \ucd94\uc815\ub418\uace0 \uc788\uc73c\uba70 \uc9c0\uadf8\ubb38\ud2b8 \ud504\ub85c\uc774\ud2b8\ub97c \uc2dc\uc791\uc73c\ub85c \ub9ce\uc740 \ud559\uc790\ub4e4\uc740 \uc720\ub300\uad50 \ub610\ud55c \uc774\uc9d1\ud2b8\uc758 \uc544\ud150\uc8fc\uc758\uc5d0\uc11c \uc2dc\uc791\ub418\uc5c8\uc744 \uac83\uc774\ub77c\uace0 \uc0dd\uac01\ud55c\ub2e4. \uace0\ub300 \ud6c4\uae30\uc5d0 \uc2dc\uc791\ub41c \uae30\ub3c5\uad50\uc758 \uc9c0\uc625\uc5d0 \ub300\ud55c \uac1c\ub150 \uc5ed\uc2dc \uc774\uc9d1\ud2b8 \uc2e0\ud654\uc5d0 \ub4f1\uc7a5\ud558\ub294 \ub450\uc544\ud2b8\uc640 \ub2ee\uc740 \uc810\uc774 \ub9ce\uc73c\uba70 \uc131\ubaa8\ub9c8\ub9ac\uc544 \ub610\ud55c \uc774\uc2dc\uc2a4\uc640 \ud638\ub8e8\uc2a4 \uc2e0\ud654\uc5d0\uc11c \ube44\ub86f\ub418\uc5c8\ub2e4. \uc774\uc9d1\ud2b8 \uc885\uad50\ub294 \uc601\uc9c0\uc8fc\uc758\uc758 \ud0c4\uc0dd\uacfc \uac1c\ub150\ud654\uc5d0 \uc601\ud5a5\uc744 \ub07c\ucce4\ub2e4. \uc608\ub97c \ub4e4\uc5b4 \uc2e0\ube44\uc8fc\uc758\ub294 \uc774\uc9d1\ud2b8\uc778\ub4e4\uc774 \ubbff\uc5c8\ub358 \ube44\ubc00\uc2a4\ub7ec\uc6b4 \ub9c8\ubc95 \uc9c0\uc2dd\uacfc \ud63c\ud569\ub41c \ud1a0\ud2b8 \uc2e0\uc559\uc758 \uc804\ud1b5\uc5d0\uc11c \ube44\ub86f\ub418\uc5c8\ub2e4.", "answer": "\uc774\uc9d1\ud2b8", "sentence": "\uc774\uc9d1\ud2b8 \uc885\uad50\ub294 \uace0\ub300 \uc774\uc9d1\ud2b8 \uc0ac\ud68c\ub97c \uac00\uc7a5 \uc798 \ubcf4\uc5ec\uc8fc\ub294 \uae30\ub150\ubb3c\uc778 \uc218 \ub9ce\uc740 \uc2e0\uc804\ub4e4\uacfc \ubb34\ub364\ub4e4\uc744 \uc0dd\uc0b0\ud558\uc600\uc73c\uba70 \uc218 \ub9ce\uc740 \ubb38\ud654\ub4e4\uc5d0 \uac70\ub300\ud55c \uc601\ud5a5\uc744 \ub0a8\uacbc\ub2e4.", "paragraph_sentence": " \uc774\uc9d1\ud2b8 \uc885\uad50\ub294 \uace0\ub300 \uc774\uc9d1\ud2b8 \uc0ac\ud68c\ub97c \uac00\uc7a5 \uc798 \ubcf4\uc5ec\uc8fc\ub294 \uae30\ub150\ubb3c\uc778 \uc218 \ub9ce\uc740 \uc2e0\uc804\ub4e4\uacfc \ubb34\ub364\ub4e4\uc744 \uc0dd\uc0b0\ud558\uc600\uc73c\uba70 \uc218 \ub9ce\uc740 \ubb38\ud654\ub4e4\uc5d0 \uac70\ub300\ud55c \uc601\ud5a5\uc744 \ub0a8\uacbc\ub2e4. \ud30c\ub77c\uc624 \uc2dc\ub300\uc758 \uc2a4\ud551\ud06c\uc2a4\uc640 \ud0dc\uc591 \uc6d0\ubc18\ub4f1\uc744 \ube44\ub86f\ud55c \ub2e4\uc591\ud55c \uc0c1\uc9d5\uacfc \uae30\ud638\ub4e4\uc740 \uc9c0\uc911\ud574\uc640 \uadfc\ub3d9\uc744 \ub118\uc5b4 \uad11\ubc94\uc704\ud558\uac8c \ud37c\uc838\ub098\uac14\ub2e4. \uadf8\ub9ac\uc2a4\uc778\ub4e4\uc758 \uc5d8\ub9ac\uc2dc\uc628\uc758 \ubc1c\uc0c1\uc740 \uc774\uc9d1\ud2b8\uc758 \uc0ac\ud6c4 \uc138\uacc4\uc5d0\uc11c \ube44\ub86f\ub41c \uac83\uc73c\ub85c \ucd94\uc815\ub418\uace0 \uc788\uc73c\uba70 \uc9c0\uadf8\ubb38\ud2b8 \ud504\ub85c\uc774\ud2b8\ub97c \uc2dc\uc791\uc73c\ub85c \ub9ce\uc740 \ud559\uc790\ub4e4\uc740 \uc720\ub300\uad50 \ub610\ud55c \uc774\uc9d1\ud2b8\uc758 \uc544\ud150\uc8fc\uc758\uc5d0\uc11c \uc2dc\uc791\ub418\uc5c8\uc744 \uac83\uc774\ub77c\uace0 \uc0dd\uac01\ud55c\ub2e4. \uace0\ub300 \ud6c4\uae30\uc5d0 \uc2dc\uc791\ub41c \uae30\ub3c5\uad50\uc758 \uc9c0\uc625\uc5d0 \ub300\ud55c \uac1c\ub150 \uc5ed\uc2dc \uc774\uc9d1\ud2b8 \uc2e0\ud654\uc5d0 \ub4f1\uc7a5\ud558\ub294 \ub450\uc544\ud2b8\uc640 \ub2ee\uc740 \uc810\uc774 \ub9ce\uc73c\uba70 \uc131\ubaa8\ub9c8\ub9ac\uc544 \ub610\ud55c \uc774\uc2dc\uc2a4\uc640 \ud638\ub8e8\uc2a4 \uc2e0\ud654\uc5d0\uc11c \ube44\ub86f\ub418\uc5c8\ub2e4. \uc774\uc9d1\ud2b8 \uc885\uad50\ub294 \uc601\uc9c0\uc8fc\uc758\uc758 \ud0c4\uc0dd\uacfc \uac1c\ub150\ud654\uc5d0 \uc601\ud5a5\uc744 \ub07c\ucce4\ub2e4. \uc608\ub97c \ub4e4\uc5b4 \uc2e0\ube44\uc8fc\uc758\ub294 \uc774\uc9d1\ud2b8\uc778\ub4e4\uc774 \ubbff\uc5c8\ub358 \ube44\ubc00\uc2a4\ub7ec\uc6b4 \ub9c8\ubc95 \uc9c0\uc2dd\uacfc \ud63c\ud569\ub41c \ud1a0\ud2b8 \uc2e0\uc559\uc758 \uc804\ud1b5\uc5d0\uc11c \ube44\ub86f\ub418\uc5c8\ub2e4.", "paragraph_answer": " \uc774\uc9d1\ud2b8 \uc885\uad50\ub294 \uace0\ub300 \uc774\uc9d1\ud2b8 \uc0ac\ud68c\ub97c \uac00\uc7a5 \uc798 \ubcf4\uc5ec\uc8fc\ub294 \uae30\ub150\ubb3c\uc778 \uc218 \ub9ce\uc740 \uc2e0\uc804\ub4e4\uacfc \ubb34\ub364\ub4e4\uc744 \uc0dd\uc0b0\ud558\uc600\uc73c\uba70 \uc218 \ub9ce\uc740 \ubb38\ud654\ub4e4\uc5d0 \uac70\ub300\ud55c \uc601\ud5a5\uc744 \ub0a8\uacbc\ub2e4. \ud30c\ub77c\uc624 \uc2dc\ub300\uc758 \uc2a4\ud551\ud06c\uc2a4\uc640 \ud0dc\uc591 \uc6d0\ubc18\ub4f1\uc744 \ube44\ub86f\ud55c \ub2e4\uc591\ud55c \uc0c1\uc9d5\uacfc \uae30\ud638\ub4e4\uc740 \uc9c0\uc911\ud574\uc640 \uadfc\ub3d9\uc744 \ub118\uc5b4 \uad11\ubc94\uc704\ud558\uac8c \ud37c\uc838\ub098\uac14\ub2e4. \uadf8\ub9ac\uc2a4\uc778\ub4e4\uc758 \uc5d8\ub9ac\uc2dc\uc628\uc758 \ubc1c\uc0c1\uc740 \uc774\uc9d1\ud2b8\uc758 \uc0ac\ud6c4 \uc138\uacc4\uc5d0\uc11c \ube44\ub86f\ub41c \uac83\uc73c\ub85c \ucd94\uc815\ub418\uace0 \uc788\uc73c\uba70 \uc9c0\uadf8\ubb38\ud2b8 \ud504\ub85c\uc774\ud2b8\ub97c \uc2dc\uc791\uc73c\ub85c \ub9ce\uc740 \ud559\uc790\ub4e4\uc740 \uc720\ub300\uad50 \ub610\ud55c \uc774\uc9d1\ud2b8\uc758 \uc544\ud150\uc8fc\uc758\uc5d0\uc11c \uc2dc\uc791\ub418\uc5c8\uc744 \uac83\uc774\ub77c\uace0 \uc0dd\uac01\ud55c\ub2e4. \uace0\ub300 \ud6c4\uae30\uc5d0 \uc2dc\uc791\ub41c \uae30\ub3c5\uad50\uc758 \uc9c0\uc625\uc5d0 \ub300\ud55c \uac1c\ub150 \uc5ed\uc2dc \uc774\uc9d1\ud2b8 \uc2e0\ud654\uc5d0 \ub4f1\uc7a5\ud558\ub294 \ub450\uc544\ud2b8\uc640 \ub2ee\uc740 \uc810\uc774 \ub9ce\uc73c\uba70 \uc131\ubaa8\ub9c8\ub9ac\uc544 \ub610\ud55c \uc774\uc2dc\uc2a4\uc640 \ud638\ub8e8\uc2a4 \uc2e0\ud654\uc5d0\uc11c \ube44\ub86f\ub418\uc5c8\ub2e4. \uc774\uc9d1\ud2b8 \uc885\uad50\ub294 \uc601\uc9c0\uc8fc\uc758\uc758 \ud0c4\uc0dd\uacfc \uac1c\ub150\ud654\uc5d0 \uc601\ud5a5\uc744 \ub07c\ucce4\ub2e4. \uc608\ub97c \ub4e4\uc5b4 \uc2e0\ube44\uc8fc\uc758\ub294 \uc774\uc9d1\ud2b8\uc778\ub4e4\uc774 \ubbff\uc5c8\ub358 \ube44\ubc00\uc2a4\ub7ec\uc6b4 \ub9c8\ubc95 \uc9c0\uc2dd\uacfc \ud63c\ud569\ub41c \ud1a0\ud2b8 \uc2e0\uc559\uc758 \uc804\ud1b5\uc5d0\uc11c \ube44\ub86f\ub418\uc5c8\ub2e4.", "sentence_answer": " \uc774\uc9d1\ud2b8 \uc885\uad50\ub294 \uace0\ub300 \uc774\uc9d1\ud2b8 \uc0ac\ud68c\ub97c \uac00\uc7a5 \uc798 \ubcf4\uc5ec\uc8fc\ub294 \uae30\ub150\ubb3c\uc778 \uc218 \ub9ce\uc740 \uc2e0\uc804\ub4e4\uacfc \ubb34\ub364\ub4e4\uc744 \uc0dd\uc0b0\ud558\uc600\uc73c\uba70 \uc218 \ub9ce\uc740 \ubb38\ud654\ub4e4\uc5d0 \uac70\ub300\ud55c \uc601\ud5a5\uc744 \ub0a8\uacbc\ub2e4.", "paragraph_id": "6471098-21-1", "paragraph_question": "question: \ud0dc\uc591 \uc6d0\ubc18\uc744 \uc81c\uc791\ud55c \ub098\ub77c\ub294 \uc5b4\ub514\uc778\uac00?, context: \uc774\uc9d1\ud2b8 \uc885\uad50\ub294 \uace0\ub300 \uc774\uc9d1\ud2b8 \uc0ac\ud68c\ub97c \uac00\uc7a5 \uc798 \ubcf4\uc5ec\uc8fc\ub294 \uae30\ub150\ubb3c\uc778 \uc218 \ub9ce\uc740 \uc2e0\uc804\ub4e4\uacfc \ubb34\ub364\ub4e4\uc744 \uc0dd\uc0b0\ud558\uc600\uc73c\uba70 \uc218 \ub9ce\uc740 \ubb38\ud654\ub4e4\uc5d0 \uac70\ub300\ud55c \uc601\ud5a5\uc744 \ub0a8\uacbc\ub2e4. \ud30c\ub77c\uc624 \uc2dc\ub300\uc758 \uc2a4\ud551\ud06c\uc2a4\uc640 \ud0dc\uc591 \uc6d0\ubc18\ub4f1\uc744 \ube44\ub86f\ud55c \ub2e4\uc591\ud55c \uc0c1\uc9d5\uacfc \uae30\ud638\ub4e4\uc740 \uc9c0\uc911\ud574\uc640 \uadfc\ub3d9\uc744 \ub118\uc5b4 \uad11\ubc94\uc704\ud558\uac8c \ud37c\uc838\ub098\uac14\ub2e4. \uadf8\ub9ac\uc2a4\uc778\ub4e4\uc758 \uc5d8\ub9ac\uc2dc\uc628\uc758 \ubc1c\uc0c1\uc740 \uc774\uc9d1\ud2b8\uc758 \uc0ac\ud6c4 \uc138\uacc4\uc5d0\uc11c \ube44\ub86f\ub41c \uac83\uc73c\ub85c \ucd94\uc815\ub418\uace0 \uc788\uc73c\uba70 \uc9c0\uadf8\ubb38\ud2b8 \ud504\ub85c\uc774\ud2b8\ub97c \uc2dc\uc791\uc73c\ub85c \ub9ce\uc740 \ud559\uc790\ub4e4\uc740 \uc720\ub300\uad50 \ub610\ud55c \uc774\uc9d1\ud2b8\uc758 \uc544\ud150\uc8fc\uc758\uc5d0\uc11c \uc2dc\uc791\ub418\uc5c8\uc744 \uac83\uc774\ub77c\uace0 \uc0dd\uac01\ud55c\ub2e4. \uace0\ub300 \ud6c4\uae30\uc5d0 \uc2dc\uc791\ub41c \uae30\ub3c5\uad50\uc758 \uc9c0\uc625\uc5d0 \ub300\ud55c \uac1c\ub150 \uc5ed\uc2dc \uc774\uc9d1\ud2b8 \uc2e0\ud654\uc5d0 \ub4f1\uc7a5\ud558\ub294 \ub450\uc544\ud2b8\uc640 \ub2ee\uc740 \uc810\uc774 \ub9ce\uc73c\uba70 \uc131\ubaa8\ub9c8\ub9ac\uc544 \ub610\ud55c \uc774\uc2dc\uc2a4\uc640 \ud638\ub8e8\uc2a4 \uc2e0\ud654\uc5d0\uc11c \ube44\ub86f\ub418\uc5c8\ub2e4. \uc774\uc9d1\ud2b8 \uc885\uad50\ub294 \uc601\uc9c0\uc8fc\uc758\uc758 \ud0c4\uc0dd\uacfc \uac1c\ub150\ud654\uc5d0 \uc601\ud5a5\uc744 \ub07c\ucce4\ub2e4. \uc608\ub97c \ub4e4\uc5b4 \uc2e0\ube44\uc8fc\uc758\ub294 \uc774\uc9d1\ud2b8\uc778\ub4e4\uc774 \ubbff\uc5c8\ub358 \ube44\ubc00\uc2a4\ub7ec\uc6b4 \ub9c8\ubc95 \uc9c0\uc2dd\uacfc \ud63c\ud569\ub41c \ud1a0\ud2b8 \uc2e0\uc559\uc758 \uc804\ud1b5\uc5d0\uc11c \ube44\ub86f\ub418\uc5c8\ub2e4."} {"question": "\ubc1c\ud640 \uc9c0\ubd95 \uaf2d\ub300\uae30\uc5d0\uc11c \uc5fc\uc18c \ud5e4\uc774\ub4dc\ub8ec\uc774 \uc78e\uc744 \uba39\ub294 \ub098\ubb34\ub294?", "paragraph": "\ubc1c\ud640(\uace0\ub300 \ub178\ub974\ub4dc\uc5b4: Valh\u01ebll\u2192\uc0b4\ud574\ub2f9\ud55c \uc790\uc758 \uc804\ub2f9, \ub3c5\uc77c\uc5b4: Walhall \ubc1c\ud560, \uc601\uc5b4: Valhalla \ubc1c\ud560\ub77c)\uc740 \ub178\ub974\ub4dc \uc2e0\ud654\uc5d0\uc11c \uc544\uc2a4\uac00\ub974\ub4dc\uc5d0 \uc704\uce58\ud55c \uac70\ub300\ud55c \uc800\ud0dd\uc73c\ub85c, \uc624\ub518\uc774 \ub2e4\uc2a4\ub9ac\uace0 \uc788\ub2e4. \uc804\uc7c1\ud130\uc5d0\uc11c \uc8fd\uc740 \uc790\ub4e4 \uc911 \uc808\ubc18\uc740 \ubc1c\ud0a4\ub9ac\uc758 \uc778\ub3c4\uc5d0 \ub530\ub77c \uc624\ub518\uc758 \ubc1c\ud640\ub85c \uac00\uace0, \ub2e4\ub978 \uc808\ubc18\uc740 \ud504\ub808\uc774\uc57c\uc758 \ud3f4\ud06c\ubc29\uc73c\ub85c \uac04\ub2e4. \ubc1c\ud640\uc5d0 \ubaa8\uc778 \ub9dd\uc790\ub4e4\uc744 \uc5d0\uc778\ud5e4\ub7b4\ub974\ub77c\uace0 \ud558\uba70, \uac8c\ub974\ub9cc \uc2e0\ud654\uc758 \uc231\ud55c \uc601\uc6c5\ub4e4\uacfc \uc655\ub4e4\uc774 \uc8fd\uc5b4\uc11c \uc5d0\uc778\ud5e4\ub7b4\ub974\uac00 \ub418\uc5c8\ub2e4. \uadf8\ub4e4\uc740 \ub77c\uadf8\ub098\ub85c\ud06c \ub54c \uc624\ub518\uc744 \ub3d5\uae30 \uc704\ud574 \uc900\ube44\ud558\uace0 \uc788\ub2e4. \uc800\ud0dd \uc55e\uc5d0\ub294 \ud669\uae08\ub098\ubb34 \uae00\ub77c\uc2dc\ub974\uac00 \uc11c \uc788\uace0, \uc800\ud0dd\uc758 \uc9c0\ubd95\uc740 \ud669\uae08 \ubc29\ud328\ub97c \ub36e\uc5b4\uc11c \uc62c\ub838\ub2e4. \uc218\uc0ac\uc2b4 \uc5d0\uc774\ud06c\uc26c\ub974\ub2c8\ub974, \uc5fc\uc18c \ud5e4\uc774\ub4dc\ub8ec\uc774 \ubc1c\ud640 \uc9c0\ubd95 \uaf2d\ub300\uae30\uc5d0\uc11c \ub808\ub77c\ub4dc \ub098\ubb34\uc758 \uc78e\uc744 \uba39\ub294\ub2e4.", "answer": "\ub808\ub77c\ub4dc \ub098\ubb34", "sentence": "\uc218\uc0ac\uc2b4 \uc5d0\uc774\ud06c\uc26c\ub974\ub2c8\ub974, \uc5fc\uc18c \ud5e4\uc774\ub4dc\ub8ec\uc774 \ubc1c\ud640 \uc9c0\ubd95 \uaf2d\ub300\uae30\uc5d0\uc11c \ub808\ub77c\ub4dc \ub098\ubb34 \uc758 \uc78e\uc744 \uba39\ub294\ub2e4.", "paragraph_sentence": "\ubc1c\ud640(\uace0\ub300 \ub178\ub974\ub4dc\uc5b4: Valh\u01ebll\u2192\uc0b4\ud574\ub2f9\ud55c \uc790\uc758 \uc804\ub2f9, \ub3c5\uc77c\uc5b4: Walhall \ubc1c\ud560, \uc601\uc5b4: Valhalla \ubc1c\ud560\ub77c)\uc740 \ub178\ub974\ub4dc \uc2e0\ud654\uc5d0\uc11c \uc544\uc2a4\uac00\ub974\ub4dc\uc5d0 \uc704\uce58\ud55c \uac70\ub300\ud55c \uc800\ud0dd\uc73c\ub85c, \uc624\ub518\uc774 \ub2e4\uc2a4\ub9ac\uace0 \uc788\ub2e4. \uc804\uc7c1\ud130\uc5d0\uc11c \uc8fd\uc740 \uc790\ub4e4 \uc911 \uc808\ubc18\uc740 \ubc1c\ud0a4\ub9ac\uc758 \uc778\ub3c4\uc5d0 \ub530\ub77c \uc624\ub518\uc758 \ubc1c\ud640\ub85c \uac00\uace0, \ub2e4\ub978 \uc808\ubc18\uc740 \ud504\ub808\uc774\uc57c\uc758 \ud3f4\ud06c\ubc29\uc73c\ub85c \uac04\ub2e4. \ubc1c\ud640\uc5d0 \ubaa8\uc778 \ub9dd\uc790\ub4e4\uc744 \uc5d0\uc778\ud5e4\ub7b4\ub974\ub77c\uace0 \ud558\uba70, \uac8c\ub974\ub9cc \uc2e0\ud654\uc758 \uc231\ud55c \uc601\uc6c5\ub4e4\uacfc \uc655\ub4e4\uc774 \uc8fd\uc5b4\uc11c \uc5d0\uc778\ud5e4\ub7b4\ub974\uac00 \ub418\uc5c8\ub2e4. \uadf8\ub4e4\uc740 \ub77c\uadf8\ub098\ub85c\ud06c \ub54c \uc624\ub518\uc744 \ub3d5\uae30 \uc704\ud574 \uc900\ube44\ud558\uace0 \uc788\ub2e4. \uc800\ud0dd \uc55e\uc5d0\ub294 \ud669\uae08\ub098\ubb34 \uae00\ub77c\uc2dc\ub974\uac00 \uc11c \uc788\uace0, \uc800\ud0dd\uc758 \uc9c0\ubd95\uc740 \ud669\uae08 \ubc29\ud328\ub97c \ub36e\uc5b4\uc11c \uc62c\ub838\ub2e4. \uc218\uc0ac\uc2b4 \uc5d0\uc774\ud06c\uc26c\ub974\ub2c8\ub974, \uc5fc\uc18c \ud5e4\uc774\ub4dc\ub8ec\uc774 \ubc1c\ud640 \uc9c0\ubd95 \uaf2d\ub300\uae30\uc5d0\uc11c \ub808\ub77c\ub4dc \ub098\ubb34 \uc758 \uc78e\uc744 \uba39\ub294\ub2e4. ", "paragraph_answer": "\ubc1c\ud640(\uace0\ub300 \ub178\ub974\ub4dc\uc5b4: Valh\u01ebll\u2192\uc0b4\ud574\ub2f9\ud55c \uc790\uc758 \uc804\ub2f9, \ub3c5\uc77c\uc5b4: Walhall \ubc1c\ud560, \uc601\uc5b4: Valhalla \ubc1c\ud560\ub77c)\uc740 \ub178\ub974\ub4dc \uc2e0\ud654\uc5d0\uc11c \uc544\uc2a4\uac00\ub974\ub4dc\uc5d0 \uc704\uce58\ud55c \uac70\ub300\ud55c \uc800\ud0dd\uc73c\ub85c, \uc624\ub518\uc774 \ub2e4\uc2a4\ub9ac\uace0 \uc788\ub2e4. \uc804\uc7c1\ud130\uc5d0\uc11c \uc8fd\uc740 \uc790\ub4e4 \uc911 \uc808\ubc18\uc740 \ubc1c\ud0a4\ub9ac\uc758 \uc778\ub3c4\uc5d0 \ub530\ub77c \uc624\ub518\uc758 \ubc1c\ud640\ub85c \uac00\uace0, \ub2e4\ub978 \uc808\ubc18\uc740 \ud504\ub808\uc774\uc57c\uc758 \ud3f4\ud06c\ubc29\uc73c\ub85c \uac04\ub2e4. \ubc1c\ud640\uc5d0 \ubaa8\uc778 \ub9dd\uc790\ub4e4\uc744 \uc5d0\uc778\ud5e4\ub7b4\ub974\ub77c\uace0 \ud558\uba70, \uac8c\ub974\ub9cc \uc2e0\ud654\uc758 \uc231\ud55c \uc601\uc6c5\ub4e4\uacfc \uc655\ub4e4\uc774 \uc8fd\uc5b4\uc11c \uc5d0\uc778\ud5e4\ub7b4\ub974\uac00 \ub418\uc5c8\ub2e4. \uadf8\ub4e4\uc740 \ub77c\uadf8\ub098\ub85c\ud06c \ub54c \uc624\ub518\uc744 \ub3d5\uae30 \uc704\ud574 \uc900\ube44\ud558\uace0 \uc788\ub2e4. \uc800\ud0dd \uc55e\uc5d0\ub294 \ud669\uae08\ub098\ubb34 \uae00\ub77c\uc2dc\ub974\uac00 \uc11c \uc788\uace0, \uc800\ud0dd\uc758 \uc9c0\ubd95\uc740 \ud669\uae08 \ubc29\ud328\ub97c \ub36e\uc5b4\uc11c \uc62c\ub838\ub2e4. \uc218\uc0ac\uc2b4 \uc5d0\uc774\ud06c\uc26c\ub974\ub2c8\ub974, \uc5fc\uc18c \ud5e4\uc774\ub4dc\ub8ec\uc774 \ubc1c\ud640 \uc9c0\ubd95 \uaf2d\ub300\uae30\uc5d0\uc11c \ub808\ub77c\ub4dc \ub098\ubb34 \uc758 \uc78e\uc744 \uba39\ub294\ub2e4.", "sentence_answer": "\uc218\uc0ac\uc2b4 \uc5d0\uc774\ud06c\uc26c\ub974\ub2c8\ub974, \uc5fc\uc18c \ud5e4\uc774\ub4dc\ub8ec\uc774 \ubc1c\ud640 \uc9c0\ubd95 \uaf2d\ub300\uae30\uc5d0\uc11c \ub808\ub77c\ub4dc \ub098\ubb34 \uc758 \uc78e\uc744 \uba39\ub294\ub2e4.", "paragraph_id": "6516398-0-2", "paragraph_question": "question: \ubc1c\ud640 \uc9c0\ubd95 \uaf2d\ub300\uae30\uc5d0\uc11c \uc5fc\uc18c \ud5e4\uc774\ub4dc\ub8ec\uc774 \uc78e\uc744 \uba39\ub294 \ub098\ubb34\ub294?, context: \ubc1c\ud640(\uace0\ub300 \ub178\ub974\ub4dc\uc5b4: Valh\u01ebll\u2192\uc0b4\ud574\ub2f9\ud55c \uc790\uc758 \uc804\ub2f9, \ub3c5\uc77c\uc5b4: Walhall \ubc1c\ud560, \uc601\uc5b4: Valhalla \ubc1c\ud560\ub77c)\uc740 \ub178\ub974\ub4dc \uc2e0\ud654\uc5d0\uc11c \uc544\uc2a4\uac00\ub974\ub4dc\uc5d0 \uc704\uce58\ud55c \uac70\ub300\ud55c \uc800\ud0dd\uc73c\ub85c, \uc624\ub518\uc774 \ub2e4\uc2a4\ub9ac\uace0 \uc788\ub2e4. \uc804\uc7c1\ud130\uc5d0\uc11c \uc8fd\uc740 \uc790\ub4e4 \uc911 \uc808\ubc18\uc740 \ubc1c\ud0a4\ub9ac\uc758 \uc778\ub3c4\uc5d0 \ub530\ub77c \uc624\ub518\uc758 \ubc1c\ud640\ub85c \uac00\uace0, \ub2e4\ub978 \uc808\ubc18\uc740 \ud504\ub808\uc774\uc57c\uc758 \ud3f4\ud06c\ubc29\uc73c\ub85c \uac04\ub2e4. \ubc1c\ud640\uc5d0 \ubaa8\uc778 \ub9dd\uc790\ub4e4\uc744 \uc5d0\uc778\ud5e4\ub7b4\ub974\ub77c\uace0 \ud558\uba70, \uac8c\ub974\ub9cc \uc2e0\ud654\uc758 \uc231\ud55c \uc601\uc6c5\ub4e4\uacfc \uc655\ub4e4\uc774 \uc8fd\uc5b4\uc11c \uc5d0\uc778\ud5e4\ub7b4\ub974\uac00 \ub418\uc5c8\ub2e4. \uadf8\ub4e4\uc740 \ub77c\uadf8\ub098\ub85c\ud06c \ub54c \uc624\ub518\uc744 \ub3d5\uae30 \uc704\ud574 \uc900\ube44\ud558\uace0 \uc788\ub2e4. \uc800\ud0dd \uc55e\uc5d0\ub294 \ud669\uae08\ub098\ubb34 \uae00\ub77c\uc2dc\ub974\uac00 \uc11c \uc788\uace0, \uc800\ud0dd\uc758 \uc9c0\ubd95\uc740 \ud669\uae08 \ubc29\ud328\ub97c \ub36e\uc5b4\uc11c \uc62c\ub838\ub2e4. \uc218\uc0ac\uc2b4 \uc5d0\uc774\ud06c\uc26c\ub974\ub2c8\ub974, \uc5fc\uc18c \ud5e4\uc774\ub4dc\ub8ec\uc774 \ubc1c\ud640 \uc9c0\ubd95 \uaf2d\ub300\uae30\uc5d0\uc11c \ub808\ub77c\ub4dc \ub098\ubb34\uc758 \uc78e\uc744 \uba39\ub294\ub2e4."} {"question": "\uccad\uad50\ub3c4\ub4e4\uc774 1621\ub144 \ucd94\uc218\ub97c \uac10\uc0ac\ud558\ub294 \ucd95\uc81c\ub97c \ubc8c\uc778 \uae30\uac04\uc740?", "paragraph": "\ucc98\uc74c \uccad\uad50\ub3c4\ub4e4\uc774 \ubbf8\uad6d\uc73c\ub85c \uc774\uc8fc\ud574\uc654\uc744 \ub54c \ud070 \uc218\ud655\uc744 \uac70\ub454 \uac83\uc5d0 \ub300\ud55c \uac10\uc0ac\uc5d0\uc11c \uc720\ub798\ud55c\ub2e4. \uc789\uae00\ub79c\ub4dc\uc758 \ub2f9\uc2dc \uc601\uad6d\uad6d\uad50\ud68c\ub97c \ubc18\ub300\ud558\ub294 \uc804\ud1b5\uc801 \ubcf5\uc74c\uc8fc\uc758\uc790\uc778 \uac1c\uc2e0\uad50 \uc2e0\uc790\ub4e4\uc744 \ub73b\ud558\ub294 \uccad\uad50\ub3c4 \ub4e4\uc740 \uc601\uad6d\uad6d\uad50\ud68c\uc640\uc758 \uac08\ub4f1\uc73c\ub85c \ubbf8\uad6d\uc758 \ud50c\ub9ac\uba38\uc2a4\ub85c \uc774\uc8fc\ud558\uc5ec \uc778\ub514\uc5b8\ub4e4\ub85c\ubd80\ud130 \ubc30\uc6b4 \uacbd\uc791\ubc95\uc73c\ub85c \ubd04\uc5d0 \uc625\uc218\uc218\ub97c \uc7ac\ubc30\ud558\uc5ec \uac00\uc744\uc5d0\ub294 \ud48d\ub144\uc744 \uac70\ub450\uc5c8\ub2e4. \uc774\uac83\uc744 \uacc4\uae30\ub85c \uadf8\ub4e4\uc740 \uc0c8\ub85c\uc6b4 \ud130\uc804\uc5d0 \uc815\ucc29\ud560 \uae30\ubc18\uc744 \uc7a1\uc744 \uc218 \uc788\uac8c \ub418\ub294\ub370, \ud070 \uc218\ud655\uc5d0 \ub300\ud55c \uae30\uc068\uacfc \uac10\uc0ac\ub97c \ud45c\ud558\uace0\uc790 1621\ub144\uc5d0 3\uc77c\ub3d9\uc548 \ucd94\uc218\ub97c \uac10\uc0ac\ud558\ub294 \ucd95\uc81c\ub97c \ubc8c\uc778\ub2e4. \uc774\ub54c \uccad\uad50\ub3c4\ub4e4\uc740 \uc790\uc2e0\ub4e4\uc5d0\uac8c \ub18d\uc0ac\ub97c \uac00\ub974\uccd0\uc8fc\uc5b4 \uad76\uc5b4\uc8fd\uc9c0 \uc54a\ub3c4\ub85d \ubc30\ub824\ud55c \uc778\ub514\uc5b8\ub4e4\uc744 \ucd08\ub300\ud558\uc5ec \ucd94\uc218\ud55c \uace1\uc2dd, \uacfc\uc77c\uacfc \uc57c\uc0dd \uce60\uba74\uc870\uc640 \uc0ac\uc2b4\uc744 \uc7a1\uc544 \ucd95\uc81c\ub97c \ud588\ub294\ub370 \uc774\uac83\uc774 \ubbf8\uad6d\uc5d0\uc11c\uc758 \ucd5c\ucd08\uc758 \ucd94\uc218\uac10\uc0ac\uc808\uc774\ub77c \uc5ec\uae34\ub2e4.", "answer": "3\uc77c", "sentence": "\uc774\uac83\uc744 \uacc4\uae30\ub85c \uadf8\ub4e4\uc740 \uc0c8\ub85c\uc6b4 \ud130\uc804\uc5d0 \uc815\ucc29\ud560 \uae30\ubc18\uc744 \uc7a1\uc744 \uc218 \uc788\uac8c \ub418\ub294\ub370, \ud070 \uc218\ud655\uc5d0 \ub300\ud55c \uae30\uc068\uacfc \uac10\uc0ac\ub97c \ud45c\ud558\uace0\uc790 1621\ub144\uc5d0 3\uc77c \ub3d9\uc548 \ucd94\uc218\ub97c \uac10\uc0ac\ud558\ub294 \ucd95\uc81c\ub97c \ubc8c\uc778\ub2e4.", "paragraph_sentence": "\ucc98\uc74c \uccad\uad50\ub3c4\ub4e4\uc774 \ubbf8\uad6d\uc73c\ub85c \uc774\uc8fc\ud574\uc654\uc744 \ub54c \ud070 \uc218\ud655\uc744 \uac70\ub454 \uac83\uc5d0 \ub300\ud55c \uac10\uc0ac\uc5d0\uc11c \uc720\ub798\ud55c\ub2e4. \uc789\uae00\ub79c\ub4dc\uc758 \ub2f9\uc2dc \uc601\uad6d\uad6d\uad50\ud68c\ub97c \ubc18\ub300\ud558\ub294 \uc804\ud1b5\uc801 \ubcf5\uc74c\uc8fc\uc758\uc790\uc778 \uac1c\uc2e0\uad50 \uc2e0\uc790\ub4e4\uc744 \ub73b\ud558\ub294 \uccad\uad50\ub3c4 \ub4e4\uc740 \uc601\uad6d\uad6d\uad50\ud68c\uc640\uc758 \uac08\ub4f1\uc73c\ub85c \ubbf8\uad6d\uc758 \ud50c\ub9ac\uba38\uc2a4\ub85c \uc774\uc8fc\ud558\uc5ec \uc778\ub514\uc5b8\ub4e4\ub85c\ubd80\ud130 \ubc30\uc6b4 \uacbd\uc791\ubc95\uc73c\ub85c \ubd04\uc5d0 \uc625\uc218\uc218\ub97c \uc7ac\ubc30\ud558\uc5ec \uac00\uc744\uc5d0\ub294 \ud48d\ub144\uc744 \uac70\ub450\uc5c8\ub2e4. \uc774\uac83\uc744 \uacc4\uae30\ub85c \uadf8\ub4e4\uc740 \uc0c8\ub85c\uc6b4 \ud130\uc804\uc5d0 \uc815\ucc29\ud560 \uae30\ubc18\uc744 \uc7a1\uc744 \uc218 \uc788\uac8c \ub418\ub294\ub370, \ud070 \uc218\ud655\uc5d0 \ub300\ud55c \uae30\uc068\uacfc \uac10\uc0ac\ub97c \ud45c\ud558\uace0\uc790 1621\ub144\uc5d0 3\uc77c \ub3d9\uc548 \ucd94\uc218\ub97c \uac10\uc0ac\ud558\ub294 \ucd95\uc81c\ub97c \ubc8c\uc778\ub2e4. \uc774\ub54c \uccad\uad50\ub3c4\ub4e4\uc740 \uc790\uc2e0\ub4e4\uc5d0\uac8c \ub18d\uc0ac\ub97c \uac00\ub974\uccd0\uc8fc\uc5b4 \uad76\uc5b4\uc8fd\uc9c0 \uc54a\ub3c4\ub85d \ubc30\ub824\ud55c \uc778\ub514\uc5b8\ub4e4\uc744 \ucd08\ub300\ud558\uc5ec \ucd94\uc218\ud55c \uace1\uc2dd, \uacfc\uc77c\uacfc \uc57c\uc0dd \uce60\uba74\uc870\uc640 \uc0ac\uc2b4\uc744 \uc7a1\uc544 \ucd95\uc81c\ub97c \ud588\ub294\ub370 \uc774\uac83\uc774 \ubbf8\uad6d\uc5d0\uc11c\uc758 \ucd5c\ucd08\uc758 \ucd94\uc218\uac10\uc0ac\uc808\uc774\ub77c \uc5ec\uae34\ub2e4.", "paragraph_answer": "\ucc98\uc74c \uccad\uad50\ub3c4\ub4e4\uc774 \ubbf8\uad6d\uc73c\ub85c \uc774\uc8fc\ud574\uc654\uc744 \ub54c \ud070 \uc218\ud655\uc744 \uac70\ub454 \uac83\uc5d0 \ub300\ud55c \uac10\uc0ac\uc5d0\uc11c \uc720\ub798\ud55c\ub2e4. \uc789\uae00\ub79c\ub4dc\uc758 \ub2f9\uc2dc \uc601\uad6d\uad6d\uad50\ud68c\ub97c \ubc18\ub300\ud558\ub294 \uc804\ud1b5\uc801 \ubcf5\uc74c\uc8fc\uc758\uc790\uc778 \uac1c\uc2e0\uad50 \uc2e0\uc790\ub4e4\uc744 \ub73b\ud558\ub294 \uccad\uad50\ub3c4 \ub4e4\uc740 \uc601\uad6d\uad6d\uad50\ud68c\uc640\uc758 \uac08\ub4f1\uc73c\ub85c \ubbf8\uad6d\uc758 \ud50c\ub9ac\uba38\uc2a4\ub85c \uc774\uc8fc\ud558\uc5ec \uc778\ub514\uc5b8\ub4e4\ub85c\ubd80\ud130 \ubc30\uc6b4 \uacbd\uc791\ubc95\uc73c\ub85c \ubd04\uc5d0 \uc625\uc218\uc218\ub97c \uc7ac\ubc30\ud558\uc5ec \uac00\uc744\uc5d0\ub294 \ud48d\ub144\uc744 \uac70\ub450\uc5c8\ub2e4. \uc774\uac83\uc744 \uacc4\uae30\ub85c \uadf8\ub4e4\uc740 \uc0c8\ub85c\uc6b4 \ud130\uc804\uc5d0 \uc815\ucc29\ud560 \uae30\ubc18\uc744 \uc7a1\uc744 \uc218 \uc788\uac8c \ub418\ub294\ub370, \ud070 \uc218\ud655\uc5d0 \ub300\ud55c \uae30\uc068\uacfc \uac10\uc0ac\ub97c \ud45c\ud558\uace0\uc790 1621\ub144\uc5d0 3\uc77c \ub3d9\uc548 \ucd94\uc218\ub97c \uac10\uc0ac\ud558\ub294 \ucd95\uc81c\ub97c \ubc8c\uc778\ub2e4. \uc774\ub54c \uccad\uad50\ub3c4\ub4e4\uc740 \uc790\uc2e0\ub4e4\uc5d0\uac8c \ub18d\uc0ac\ub97c \uac00\ub974\uccd0\uc8fc\uc5b4 \uad76\uc5b4\uc8fd\uc9c0 \uc54a\ub3c4\ub85d \ubc30\ub824\ud55c \uc778\ub514\uc5b8\ub4e4\uc744 \ucd08\ub300\ud558\uc5ec \ucd94\uc218\ud55c \uace1\uc2dd, \uacfc\uc77c\uacfc \uc57c\uc0dd \uce60\uba74\uc870\uc640 \uc0ac\uc2b4\uc744 \uc7a1\uc544 \ucd95\uc81c\ub97c \ud588\ub294\ub370 \uc774\uac83\uc774 \ubbf8\uad6d\uc5d0\uc11c\uc758 \ucd5c\ucd08\uc758 \ucd94\uc218\uac10\uc0ac\uc808\uc774\ub77c \uc5ec\uae34\ub2e4.", "sentence_answer": "\uc774\uac83\uc744 \uacc4\uae30\ub85c \uadf8\ub4e4\uc740 \uc0c8\ub85c\uc6b4 \ud130\uc804\uc5d0 \uc815\ucc29\ud560 \uae30\ubc18\uc744 \uc7a1\uc744 \uc218 \uc788\uac8c \ub418\ub294\ub370, \ud070 \uc218\ud655\uc5d0 \ub300\ud55c \uae30\uc068\uacfc \uac10\uc0ac\ub97c \ud45c\ud558\uace0\uc790 1621\ub144\uc5d0 3\uc77c \ub3d9\uc548 \ucd94\uc218\ub97c \uac10\uc0ac\ud558\ub294 \ucd95\uc81c\ub97c \ubc8c\uc778\ub2e4.", "paragraph_id": "6560529-3-1", "paragraph_question": "question: \uccad\uad50\ub3c4\ub4e4\uc774 1621\ub144 \ucd94\uc218\ub97c \uac10\uc0ac\ud558\ub294 \ucd95\uc81c\ub97c \ubc8c\uc778 \uae30\uac04\uc740?, context: \ucc98\uc74c \uccad\uad50\ub3c4\ub4e4\uc774 \ubbf8\uad6d\uc73c\ub85c \uc774\uc8fc\ud574\uc654\uc744 \ub54c \ud070 \uc218\ud655\uc744 \uac70\ub454 \uac83\uc5d0 \ub300\ud55c \uac10\uc0ac\uc5d0\uc11c \uc720\ub798\ud55c\ub2e4. \uc789\uae00\ub79c\ub4dc\uc758 \ub2f9\uc2dc \uc601\uad6d\uad6d\uad50\ud68c\ub97c \ubc18\ub300\ud558\ub294 \uc804\ud1b5\uc801 \ubcf5\uc74c\uc8fc\uc758\uc790\uc778 \uac1c\uc2e0\uad50 \uc2e0\uc790\ub4e4\uc744 \ub73b\ud558\ub294 \uccad\uad50\ub3c4 \ub4e4\uc740 \uc601\uad6d\uad6d\uad50\ud68c\uc640\uc758 \uac08\ub4f1\uc73c\ub85c \ubbf8\uad6d\uc758 \ud50c\ub9ac\uba38\uc2a4\ub85c \uc774\uc8fc\ud558\uc5ec \uc778\ub514\uc5b8\ub4e4\ub85c\ubd80\ud130 \ubc30\uc6b4 \uacbd\uc791\ubc95\uc73c\ub85c \ubd04\uc5d0 \uc625\uc218\uc218\ub97c \uc7ac\ubc30\ud558\uc5ec \uac00\uc744\uc5d0\ub294 \ud48d\ub144\uc744 \uac70\ub450\uc5c8\ub2e4. \uc774\uac83\uc744 \uacc4\uae30\ub85c \uadf8\ub4e4\uc740 \uc0c8\ub85c\uc6b4 \ud130\uc804\uc5d0 \uc815\ucc29\ud560 \uae30\ubc18\uc744 \uc7a1\uc744 \uc218 \uc788\uac8c \ub418\ub294\ub370, \ud070 \uc218\ud655\uc5d0 \ub300\ud55c \uae30\uc068\uacfc \uac10\uc0ac\ub97c \ud45c\ud558\uace0\uc790 1621\ub144\uc5d0 3\uc77c\ub3d9\uc548 \ucd94\uc218\ub97c \uac10\uc0ac\ud558\ub294 \ucd95\uc81c\ub97c \ubc8c\uc778\ub2e4. \uc774\ub54c \uccad\uad50\ub3c4\ub4e4\uc740 \uc790\uc2e0\ub4e4\uc5d0\uac8c \ub18d\uc0ac\ub97c \uac00\ub974\uccd0\uc8fc\uc5b4 \uad76\uc5b4\uc8fd\uc9c0 \uc54a\ub3c4\ub85d \ubc30\ub824\ud55c \uc778\ub514\uc5b8\ub4e4\uc744 \ucd08\ub300\ud558\uc5ec \ucd94\uc218\ud55c \uace1\uc2dd, \uacfc\uc77c\uacfc \uc57c\uc0dd \uce60\uba74\uc870\uc640 \uc0ac\uc2b4\uc744 \uc7a1\uc544 \ucd95\uc81c\ub97c \ud588\ub294\ub370 \uc774\uac83\uc774 \ubbf8\uad6d\uc5d0\uc11c\uc758 \ucd5c\ucd08\uc758 \ucd94\uc218\uac10\uc0ac\uc808\uc774\ub77c \uc5ec\uae34\ub2e4."} {"question": "\uc911\ud654\ubbfc\uad6d \ud559\uacc4\uc5d0\uc11c\ub294 \uc2e0\ud574\uba71\uc601\uc744 \ubb34\uc5c7\uc73c\ub85c \uaddc\uc815\ud558\uc600\ub294\uac00?", "paragraph": "\ubc18\uba74, \uc911\ud654\ubbfc\uad6d \ud559\uacc4\uc640 \uc911\uad6d \uad6d\ubbfc\ub2f9\uc5d0\uc11c\ub294 \uc911\uacf5\uc774 \uc2e0\ud574\ud601\uba85\uc744 '\ubd80\ub974\uc8fc\uc544 \ud601\uba85'\uc774\ub77c \ud3c4\ud558\ud558\uace0 \uc788\ub294 \uac83\uc5d0 \ub300\ud574 \ubc18\ub860\uc744 \ub0b4\uc138\uc6b0\uace0 \uc788\ub2e4. \uc989, \ud601\uba85\uc758 \uc911\uc2ec\uc774\uc5c8\ub358 \uc468\uc6d0\ub9cc \ud558\ub354\ub77c\ub3c4 \ubd80\ub974\uc8fc\uc544 \ucd9c\uc2e0\uc774 \uc544\ub2c8\ub77c\ub294 \uc810\uc744 \ub4e4\uace0 \uc788\ub2e4. \ud655\uc2e4\ud788 \uc468\uc6d0\uc740 \ubd80\ub974\uc8fc\uc544 \ucd9c\uc2e0\uc774 \uc544\ub2cc \ub18d\ubbfc \ucd9c\uc2e0\uc774\uba70 \uadf8\uac83\ub3c4 \uc911\ub18d\uc774\ud558\uc758 \uac00\uc815 \ucd9c\uc2e0\uc774\uc5c8\ub2e4. \uadf8\ub9ac\uace0 \ud601\uba85\uc5d0 \ucc38\uc5ec\ud55c \uacc4\uce35\uc740 \ud68c\ub2f9(\u6703\u9ee8)\uc744 \ube44\ub86f\ud558\uc5ec \uc2e0\uc9c0\uc2dd\uc778, \ud654\uad50, \uc2e0\uad70(\u65b0\u8ecd) \ub4f1 \uc911\uad6d\uc758 \ubaa8\ub4e0 \uacc4\uce35\uc774 \ucc38\uc5ec\ud588\uae30 \ub54c\ubb38\uc5d0, \uc911\ud654\ubbfc\uad6d \ud559\uacc4\uc5d0\uc11c\ub294 \uc2e0\ud574\ud601\uba85\uc744 \uc774\ub978 \ubc14 '\uc804\ubbfc\ud601\uba85'(\u5168\u6c11\u9769\u547d)'\uc73c\ub85c \uaddc\uc815\ud558\uc600\ub2e4. \uc774\ub807\ub4ef \uc911\ud654\ubbfc\uad6d \ud559\uacc4\uc5d0\uc11c\ub294 \uc911\uad6d \uacf5\uc0b0\ub2f9\uc5d0\uc11c \uc8fc\uc7a5\ud558\ub294 '\uad6c\ubbfc\uc8fc\uc8fc\uc758, \uc790\uc0b0\uacc4\uae09 \ubd80\ub974\uc8fc\uc544 \ud601\uba85'\uc774\ub780 \uc124\uc5d0 \ub300\ud55c \ubc18\ub860\uc73c\ub85c '\uc804\ubbfc\ud601\uba85'\uc744 \ub0b4\uc138\uc6b0\uace0 \uc788\ub2e4.", "answer": "\uc804\ubbfc\ud601\uba85", "sentence": "\uadf8\ub9ac\uace0 \ud601\uba85\uc5d0 \ucc38\uc5ec\ud55c \uacc4\uce35\uc740 \ud68c\ub2f9(\u6703\u9ee8)\uc744 \ube44\ub86f\ud558\uc5ec \uc2e0\uc9c0\uc2dd\uc778, \ud654\uad50, \uc2e0\uad70(\u65b0\u8ecd) \ub4f1 \uc911\uad6d\uc758 \ubaa8\ub4e0 \uacc4\uce35\uc774 \ucc38\uc5ec\ud588\uae30 \ub54c\ubb38\uc5d0, \uc911\ud654\ubbfc\uad6d \ud559\uacc4\uc5d0\uc11c\ub294 \uc2e0\ud574\ud601\uba85\uc744 \uc774\ub978 \ubc14 ' \uc804\ubbfc\ud601\uba85 '(\u5168\u6c11\u9769\u547d)", "paragraph_sentence": "\ubc18\uba74, \uc911\ud654\ubbfc\uad6d \ud559\uacc4\uc640 \uc911\uad6d \uad6d\ubbfc\ub2f9\uc5d0\uc11c\ub294 \uc911\uacf5\uc774 \uc2e0\ud574\ud601\uba85\uc744 '\ubd80\ub974\uc8fc\uc544 \ud601\uba85'\uc774\ub77c \ud3c4\ud558\ud558\uace0 \uc788\ub294 \uac83\uc5d0 \ub300\ud574 \ubc18\ub860\uc744 \ub0b4\uc138\uc6b0\uace0 \uc788\ub2e4. \uc989, \ud601\uba85\uc758 \uc911\uc2ec\uc774\uc5c8\ub358 \uc468\uc6d0\ub9cc \ud558\ub354\ub77c\ub3c4 \ubd80\ub974\uc8fc\uc544 \ucd9c\uc2e0\uc774 \uc544\ub2c8\ub77c\ub294 \uc810\uc744 \ub4e4\uace0 \uc788\ub2e4. \ud655\uc2e4\ud788 \uc468\uc6d0\uc740 \ubd80\ub974\uc8fc\uc544 \ucd9c\uc2e0\uc774 \uc544\ub2cc \ub18d\ubbfc \ucd9c\uc2e0\uc774\uba70 \uadf8\uac83\ub3c4 \uc911\ub18d\uc774\ud558\uc758 \uac00\uc815 \ucd9c\uc2e0\uc774\uc5c8\ub2e4. \uadf8\ub9ac\uace0 \ud601\uba85\uc5d0 \ucc38\uc5ec\ud55c \uacc4\uce35\uc740 \ud68c\ub2f9(\u6703\u9ee8)\uc744 \ube44\ub86f\ud558\uc5ec \uc2e0\uc9c0\uc2dd\uc778, \ud654\uad50, \uc2e0\uad70(\u65b0\u8ecd) \ub4f1 \uc911\uad6d\uc758 \ubaa8\ub4e0 \uacc4\uce35\uc774 \ucc38\uc5ec\ud588\uae30 \ub54c\ubb38\uc5d0, \uc911\ud654\ubbfc\uad6d \ud559\uacc4\uc5d0\uc11c\ub294 \uc2e0\ud574\ud601\uba85\uc744 \uc774\ub978 \ubc14 ' \uc804\ubbfc\ud601\uba85 '(\u5168\u6c11\u9769\u547d) ' \uc73c\ub85c \uaddc\uc815\ud558\uc600\ub2e4. \uc774\ub807\ub4ef \uc911\ud654\ubbfc\uad6d \ud559\uacc4\uc5d0\uc11c\ub294 \uc911\uad6d \uacf5\uc0b0\ub2f9\uc5d0\uc11c \uc8fc\uc7a5\ud558\ub294 '\uad6c\ubbfc\uc8fc\uc8fc\uc758, \uc790\uc0b0\uacc4\uae09 \ubd80\ub974\uc8fc\uc544 \ud601\uba85'\uc774\ub780 \uc124\uc5d0 \ub300\ud55c \ubc18\ub860\uc73c\ub85c '\uc804\ubbfc\ud601\uba85'\uc744 \ub0b4\uc138\uc6b0\uace0 \uc788\ub2e4.", "paragraph_answer": "\ubc18\uba74, \uc911\ud654\ubbfc\uad6d \ud559\uacc4\uc640 \uc911\uad6d \uad6d\ubbfc\ub2f9\uc5d0\uc11c\ub294 \uc911\uacf5\uc774 \uc2e0\ud574\ud601\uba85\uc744 '\ubd80\ub974\uc8fc\uc544 \ud601\uba85'\uc774\ub77c \ud3c4\ud558\ud558\uace0 \uc788\ub294 \uac83\uc5d0 \ub300\ud574 \ubc18\ub860\uc744 \ub0b4\uc138\uc6b0\uace0 \uc788\ub2e4. \uc989, \ud601\uba85\uc758 \uc911\uc2ec\uc774\uc5c8\ub358 \uc468\uc6d0\ub9cc \ud558\ub354\ub77c\ub3c4 \ubd80\ub974\uc8fc\uc544 \ucd9c\uc2e0\uc774 \uc544\ub2c8\ub77c\ub294 \uc810\uc744 \ub4e4\uace0 \uc788\ub2e4. \ud655\uc2e4\ud788 \uc468\uc6d0\uc740 \ubd80\ub974\uc8fc\uc544 \ucd9c\uc2e0\uc774 \uc544\ub2cc \ub18d\ubbfc \ucd9c\uc2e0\uc774\uba70 \uadf8\uac83\ub3c4 \uc911\ub18d\uc774\ud558\uc758 \uac00\uc815 \ucd9c\uc2e0\uc774\uc5c8\ub2e4. \uadf8\ub9ac\uace0 \ud601\uba85\uc5d0 \ucc38\uc5ec\ud55c \uacc4\uce35\uc740 \ud68c\ub2f9(\u6703\u9ee8)\uc744 \ube44\ub86f\ud558\uc5ec \uc2e0\uc9c0\uc2dd\uc778, \ud654\uad50, \uc2e0\uad70(\u65b0\u8ecd) \ub4f1 \uc911\uad6d\uc758 \ubaa8\ub4e0 \uacc4\uce35\uc774 \ucc38\uc5ec\ud588\uae30 \ub54c\ubb38\uc5d0, \uc911\ud654\ubbfc\uad6d \ud559\uacc4\uc5d0\uc11c\ub294 \uc2e0\ud574\ud601\uba85\uc744 \uc774\ub978 \ubc14 ' \uc804\ubbfc\ud601\uba85 '(\u5168\u6c11\u9769\u547d)'\uc73c\ub85c \uaddc\uc815\ud558\uc600\ub2e4. \uc774\ub807\ub4ef \uc911\ud654\ubbfc\uad6d \ud559\uacc4\uc5d0\uc11c\ub294 \uc911\uad6d \uacf5\uc0b0\ub2f9\uc5d0\uc11c \uc8fc\uc7a5\ud558\ub294 '\uad6c\ubbfc\uc8fc\uc8fc\uc758, \uc790\uc0b0\uacc4\uae09 \ubd80\ub974\uc8fc\uc544 \ud601\uba85'\uc774\ub780 \uc124\uc5d0 \ub300\ud55c \ubc18\ub860\uc73c\ub85c '\uc804\ubbfc\ud601\uba85'\uc744 \ub0b4\uc138\uc6b0\uace0 \uc788\ub2e4.", "sentence_answer": "\uadf8\ub9ac\uace0 \ud601\uba85\uc5d0 \ucc38\uc5ec\ud55c \uacc4\uce35\uc740 \ud68c\ub2f9(\u6703\u9ee8)\uc744 \ube44\ub86f\ud558\uc5ec \uc2e0\uc9c0\uc2dd\uc778, \ud654\uad50, \uc2e0\uad70(\u65b0\u8ecd) \ub4f1 \uc911\uad6d\uc758 \ubaa8\ub4e0 \uacc4\uce35\uc774 \ucc38\uc5ec\ud588\uae30 \ub54c\ubb38\uc5d0, \uc911\ud654\ubbfc\uad6d \ud559\uacc4\uc5d0\uc11c\ub294 \uc2e0\ud574\ud601\uba85\uc744 \uc774\ub978 \ubc14 ' \uc804\ubbfc\ud601\uba85 '(\u5168\u6c11\u9769\u547d)", "paragraph_id": "6457494-22-1", "paragraph_question": "question: \uc911\ud654\ubbfc\uad6d \ud559\uacc4\uc5d0\uc11c\ub294 \uc2e0\ud574\uba71\uc601\uc744 \ubb34\uc5c7\uc73c\ub85c \uaddc\uc815\ud558\uc600\ub294\uac00?, context: \ubc18\uba74, \uc911\ud654\ubbfc\uad6d \ud559\uacc4\uc640 \uc911\uad6d \uad6d\ubbfc\ub2f9\uc5d0\uc11c\ub294 \uc911\uacf5\uc774 \uc2e0\ud574\ud601\uba85\uc744 '\ubd80\ub974\uc8fc\uc544 \ud601\uba85'\uc774\ub77c \ud3c4\ud558\ud558\uace0 \uc788\ub294 \uac83\uc5d0 \ub300\ud574 \ubc18\ub860\uc744 \ub0b4\uc138\uc6b0\uace0 \uc788\ub2e4. \uc989, \ud601\uba85\uc758 \uc911\uc2ec\uc774\uc5c8\ub358 \uc468\uc6d0\ub9cc \ud558\ub354\ub77c\ub3c4 \ubd80\ub974\uc8fc\uc544 \ucd9c\uc2e0\uc774 \uc544\ub2c8\ub77c\ub294 \uc810\uc744 \ub4e4\uace0 \uc788\ub2e4. \ud655\uc2e4\ud788 \uc468\uc6d0\uc740 \ubd80\ub974\uc8fc\uc544 \ucd9c\uc2e0\uc774 \uc544\ub2cc \ub18d\ubbfc \ucd9c\uc2e0\uc774\uba70 \uadf8\uac83\ub3c4 \uc911\ub18d\uc774\ud558\uc758 \uac00\uc815 \ucd9c\uc2e0\uc774\uc5c8\ub2e4. \uadf8\ub9ac\uace0 \ud601\uba85\uc5d0 \ucc38\uc5ec\ud55c \uacc4\uce35\uc740 \ud68c\ub2f9(\u6703\u9ee8)\uc744 \ube44\ub86f\ud558\uc5ec \uc2e0\uc9c0\uc2dd\uc778, \ud654\uad50, \uc2e0\uad70(\u65b0\u8ecd) \ub4f1 \uc911\uad6d\uc758 \ubaa8\ub4e0 \uacc4\uce35\uc774 \ucc38\uc5ec\ud588\uae30 \ub54c\ubb38\uc5d0, \uc911\ud654\ubbfc\uad6d \ud559\uacc4\uc5d0\uc11c\ub294 \uc2e0\ud574\ud601\uba85\uc744 \uc774\ub978 \ubc14 '\uc804\ubbfc\ud601\uba85'(\u5168\u6c11\u9769\u547d)'\uc73c\ub85c \uaddc\uc815\ud558\uc600\ub2e4. \uc774\ub807\ub4ef \uc911\ud654\ubbfc\uad6d \ud559\uacc4\uc5d0\uc11c\ub294 \uc911\uad6d \uacf5\uc0b0\ub2f9\uc5d0\uc11c \uc8fc\uc7a5\ud558\ub294 '\uad6c\ubbfc\uc8fc\uc8fc\uc758, \uc790\uc0b0\uacc4\uae09 \ubd80\ub974\uc8fc\uc544 \ud601\uba85'\uc774\ub780 \uc124\uc5d0 \ub300\ud55c \ubc18\ub860\uc73c\ub85c '\uc804\ubbfc\ud601\uba85'\uc744 \ub0b4\uc138\uc6b0\uace0 \uc788\ub2e4."} {"question": "\ud074\ub9ac\ube14\ub79c\ub4dc \uc778\ub514\uc5b8\uc2a4\uac00 \uc544\uba54\ub9ac\uce78 \ub9ac\uadf8 \uc6b0\uc2b9\uc744 \ud55c \ub144\ub3c4\ub294?", "paragraph": "1920\ub144 \uc2dc\uc98c \ud074\ub9ac\ube14\ub79c\ub4dc \uc778\ub514\uc5b8\uc2a4\ub294 \uc544\uba54\ub9ac\uce78 \ub9ac\uadf8 \uc6b0\uc2b9\uc744 \ucc28\uc9c0\ud588\uace0, \ucf54\ubca8\ub808\uc2a4\ud0a4\ub3c4 \uc774\uc5d0 \ud55c\ubaab\uc744 \ud588\ub2e4. \uc778\ub514\uc5b8\uc2a4\ub294 \uc6d4\ub4dc \uc2dc\ub9ac\uc988\uc5d0 \uc9c4\ucd9c\ud574 \ube0c\ub8e8\ud074\ub9b0 \ub85c\ube48\uc2a4\uc640 \ub9de\ubd99\uc5c8\ub2e4. 1920\ub144 \uc6d4\ub4dc \uc2dc\ub9ac\uc988\uc758 \uc2a4\ud0c0\ub294 \uc138 \uacbd\uae30\uc5d0\uc11c \uc644\ud22c\uc2b9\uc744 \uac70\ub454 \ucf54\ubca8\ub808\uc2a4\ud0a4\uc600\ub2e4. 1\ucc28\uc804\uc5d0\uc11c \ub8e8\ube0c \ub9c8\ucffc\ub4dc\uc640 \uc120\ubc1c \ub9de\ub300\uacb0\uc744 \ubc8c\uc5ec 5\ud53c\uc548\ud0c0 1\uc2e4\uc810\ud558\uba70 \ud300\uc758 3-1 \uc2b9\ub9ac\ub97c \uc774\ub04c\uc5c8\ub2e4. \ub098\ud758 \ub4a4 \uce58\ub7ec\uc9c4 4\ucc28\uc804\uc5d0\uc11c\ub294 5\ud53c\uc548\ud0c0 1\uc2e4\uc810\ud588\uace0 \ud300\uc740 5-1\ub85c \uc2b9\ub9ac\ud588\ub2e4. \uc2dc\ub9ac\uc988 \ub9c8\uc9c0\ub9c9 \uacbd\uae30 7\ucc28\uc804\uc5d0\uc11c \ub2e4\uc2dc \uc120\ubc1c \ub4f1\ud310\ud574 \uc2a4\ud54f\ubcfc\ub7ec \ubc8c\ub9ac \uadf8\ub77c\uc784\uc2a4\uc640 \ub9de\ub300\uacb0\uc744 \ud3bc\ucce4\uace0, 5\ud53c\uc548\ud0c0 \uc644\ubd09\uc2b9\uc744 \uac70\ub450\uba74\uc11c \ud300\uc758 3-0 \uc2b9\ub9ac\ub97c \uc774\ub04c\uc5c8\ub2e4. \ud074\ub9ac\ube14\ub79c\ub4dc \uc778\ub514\uc5b8\uc2a4\ub294 \ud504\ub79c\ucc28\uc774\uc988 \uc5ed\uc0ac\uc0c1 \uccab \uc6d4\ub4dc \uc2dc\ub9ac\uc988 \uc6b0\uc2b9\uc744 \ucc28\uc9c0\ud588\ub2e4. \ucf54\ubca8\ub808\uc2a4\ud0a4\ub294 \uc774\ubc88 \uc2dc\ub9ac\uc988\uc5d0\uc11c 0.67\uc758 \ud3c9\uade0\uc790\ucc45\uc810\uc744 \uae30\ub85d\ud588\ub294\ub370, \uc774\ub294 \uc6d4\ub4dc \uc2dc\ub9ac\uc988 \uae30\ub85d\uc73c\ub85c \ub0a8\uc544\uc788\ub2e4.", "answer": "1920\ub144", "sentence": "1920\ub144 \uc2dc\uc98c \ud074\ub9ac\ube14\ub79c\ub4dc \uc778\ub514\uc5b8\uc2a4\ub294 \uc544\uba54\ub9ac\uce78 \ub9ac\uadf8 \uc6b0\uc2b9\uc744 \ucc28\uc9c0\ud588\uace0, \ucf54\ubca8\ub808\uc2a4\ud0a4\ub3c4 \uc774\uc5d0 \ud55c\ubaab\uc744 \ud588\ub2e4.", "paragraph_sentence": " 1920\ub144 \uc2dc\uc98c \ud074\ub9ac\ube14\ub79c\ub4dc \uc778\ub514\uc5b8\uc2a4\ub294 \uc544\uba54\ub9ac\uce78 \ub9ac\uadf8 \uc6b0\uc2b9\uc744 \ucc28\uc9c0\ud588\uace0, \ucf54\ubca8\ub808\uc2a4\ud0a4\ub3c4 \uc774\uc5d0 \ud55c\ubaab\uc744 \ud588\ub2e4. \uc778\ub514\uc5b8\uc2a4\ub294 \uc6d4\ub4dc \uc2dc\ub9ac\uc988\uc5d0 \uc9c4\ucd9c\ud574 \ube0c\ub8e8\ud074\ub9b0 \ub85c\ube48\uc2a4\uc640 \ub9de\ubd99\uc5c8\ub2e4. 1920\ub144 \uc6d4\ub4dc \uc2dc\ub9ac\uc988\uc758 \uc2a4\ud0c0\ub294 \uc138 \uacbd\uae30\uc5d0\uc11c \uc644\ud22c\uc2b9\uc744 \uac70\ub454 \ucf54\ubca8\ub808\uc2a4\ud0a4\uc600\ub2e4. 1\ucc28\uc804\uc5d0\uc11c \ub8e8\ube0c \ub9c8\ucffc\ub4dc\uc640 \uc120\ubc1c \ub9de\ub300\uacb0\uc744 \ubc8c\uc5ec 5\ud53c\uc548\ud0c0 1\uc2e4\uc810\ud558\uba70 \ud300\uc758 3-1 \uc2b9\ub9ac\ub97c \uc774\ub04c\uc5c8\ub2e4. \ub098\ud758 \ub4a4 \uce58\ub7ec\uc9c4 4\ucc28\uc804\uc5d0\uc11c\ub294 5\ud53c\uc548\ud0c0 1\uc2e4\uc810\ud588\uace0 \ud300\uc740 5-1\ub85c \uc2b9\ub9ac\ud588\ub2e4. \uc2dc\ub9ac\uc988 \ub9c8\uc9c0\ub9c9 \uacbd\uae30 7\ucc28\uc804\uc5d0\uc11c \ub2e4\uc2dc \uc120\ubc1c \ub4f1\ud310\ud574 \uc2a4\ud54f\ubcfc\ub7ec \ubc8c\ub9ac \uadf8\ub77c\uc784\uc2a4\uc640 \ub9de\ub300\uacb0\uc744 \ud3bc\ucce4\uace0, 5\ud53c\uc548\ud0c0 \uc644\ubd09\uc2b9\uc744 \uac70\ub450\uba74\uc11c \ud300\uc758 3-0 \uc2b9\ub9ac\ub97c \uc774\ub04c\uc5c8\ub2e4. \ud074\ub9ac\ube14\ub79c\ub4dc \uc778\ub514\uc5b8\uc2a4\ub294 \ud504\ub79c\ucc28\uc774\uc988 \uc5ed\uc0ac\uc0c1 \uccab \uc6d4\ub4dc \uc2dc\ub9ac\uc988 \uc6b0\uc2b9\uc744 \ucc28\uc9c0\ud588\ub2e4. \ucf54\ubca8\ub808\uc2a4\ud0a4\ub294 \uc774\ubc88 \uc2dc\ub9ac\uc988\uc5d0\uc11c 0.67\uc758 \ud3c9\uade0\uc790\ucc45\uc810\uc744 \uae30\ub85d\ud588\ub294\ub370, \uc774\ub294 \uc6d4\ub4dc \uc2dc\ub9ac\uc988 \uae30\ub85d\uc73c\ub85c \ub0a8\uc544\uc788\ub2e4.", "paragraph_answer": " 1920\ub144 \uc2dc\uc98c \ud074\ub9ac\ube14\ub79c\ub4dc \uc778\ub514\uc5b8\uc2a4\ub294 \uc544\uba54\ub9ac\uce78 \ub9ac\uadf8 \uc6b0\uc2b9\uc744 \ucc28\uc9c0\ud588\uace0, \ucf54\ubca8\ub808\uc2a4\ud0a4\ub3c4 \uc774\uc5d0 \ud55c\ubaab\uc744 \ud588\ub2e4. \uc778\ub514\uc5b8\uc2a4\ub294 \uc6d4\ub4dc \uc2dc\ub9ac\uc988\uc5d0 \uc9c4\ucd9c\ud574 \ube0c\ub8e8\ud074\ub9b0 \ub85c\ube48\uc2a4\uc640 \ub9de\ubd99\uc5c8\ub2e4. 1920\ub144 \uc6d4\ub4dc \uc2dc\ub9ac\uc988\uc758 \uc2a4\ud0c0\ub294 \uc138 \uacbd\uae30\uc5d0\uc11c \uc644\ud22c\uc2b9\uc744 \uac70\ub454 \ucf54\ubca8\ub808\uc2a4\ud0a4\uc600\ub2e4. 1\ucc28\uc804\uc5d0\uc11c \ub8e8\ube0c \ub9c8\ucffc\ub4dc\uc640 \uc120\ubc1c \ub9de\ub300\uacb0\uc744 \ubc8c\uc5ec 5\ud53c\uc548\ud0c0 1\uc2e4\uc810\ud558\uba70 \ud300\uc758 3-1 \uc2b9\ub9ac\ub97c \uc774\ub04c\uc5c8\ub2e4. \ub098\ud758 \ub4a4 \uce58\ub7ec\uc9c4 4\ucc28\uc804\uc5d0\uc11c\ub294 5\ud53c\uc548\ud0c0 1\uc2e4\uc810\ud588\uace0 \ud300\uc740 5-1\ub85c \uc2b9\ub9ac\ud588\ub2e4. \uc2dc\ub9ac\uc988 \ub9c8\uc9c0\ub9c9 \uacbd\uae30 7\ucc28\uc804\uc5d0\uc11c \ub2e4\uc2dc \uc120\ubc1c \ub4f1\ud310\ud574 \uc2a4\ud54f\ubcfc\ub7ec \ubc8c\ub9ac \uadf8\ub77c\uc784\uc2a4\uc640 \ub9de\ub300\uacb0\uc744 \ud3bc\ucce4\uace0, 5\ud53c\uc548\ud0c0 \uc644\ubd09\uc2b9\uc744 \uac70\ub450\uba74\uc11c \ud300\uc758 3-0 \uc2b9\ub9ac\ub97c \uc774\ub04c\uc5c8\ub2e4. \ud074\ub9ac\ube14\ub79c\ub4dc \uc778\ub514\uc5b8\uc2a4\ub294 \ud504\ub79c\ucc28\uc774\uc988 \uc5ed\uc0ac\uc0c1 \uccab \uc6d4\ub4dc \uc2dc\ub9ac\uc988 \uc6b0\uc2b9\uc744 \ucc28\uc9c0\ud588\ub2e4. \ucf54\ubca8\ub808\uc2a4\ud0a4\ub294 \uc774\ubc88 \uc2dc\ub9ac\uc988\uc5d0\uc11c 0.67\uc758 \ud3c9\uade0\uc790\ucc45\uc810\uc744 \uae30\ub85d\ud588\ub294\ub370, \uc774\ub294 \uc6d4\ub4dc \uc2dc\ub9ac\uc988 \uae30\ub85d\uc73c\ub85c \ub0a8\uc544\uc788\ub2e4.", "sentence_answer": " 1920\ub144 \uc2dc\uc98c \ud074\ub9ac\ube14\ub79c\ub4dc \uc778\ub514\uc5b8\uc2a4\ub294 \uc544\uba54\ub9ac\uce78 \ub9ac\uadf8 \uc6b0\uc2b9\uc744 \ucc28\uc9c0\ud588\uace0, \ucf54\ubca8\ub808\uc2a4\ud0a4\ub3c4 \uc774\uc5d0 \ud55c\ubaab\uc744 \ud588\ub2e4.", "paragraph_id": "6457302-7-1", "paragraph_question": "question: \ud074\ub9ac\ube14\ub79c\ub4dc \uc778\ub514\uc5b8\uc2a4\uac00 \uc544\uba54\ub9ac\uce78 \ub9ac\uadf8 \uc6b0\uc2b9\uc744 \ud55c \ub144\ub3c4\ub294?, context: 1920\ub144 \uc2dc\uc98c \ud074\ub9ac\ube14\ub79c\ub4dc \uc778\ub514\uc5b8\uc2a4\ub294 \uc544\uba54\ub9ac\uce78 \ub9ac\uadf8 \uc6b0\uc2b9\uc744 \ucc28\uc9c0\ud588\uace0, \ucf54\ubca8\ub808\uc2a4\ud0a4\ub3c4 \uc774\uc5d0 \ud55c\ubaab\uc744 \ud588\ub2e4. \uc778\ub514\uc5b8\uc2a4\ub294 \uc6d4\ub4dc \uc2dc\ub9ac\uc988\uc5d0 \uc9c4\ucd9c\ud574 \ube0c\ub8e8\ud074\ub9b0 \ub85c\ube48\uc2a4\uc640 \ub9de\ubd99\uc5c8\ub2e4. 1920\ub144 \uc6d4\ub4dc \uc2dc\ub9ac\uc988\uc758 \uc2a4\ud0c0\ub294 \uc138 \uacbd\uae30\uc5d0\uc11c \uc644\ud22c\uc2b9\uc744 \uac70\ub454 \ucf54\ubca8\ub808\uc2a4\ud0a4\uc600\ub2e4. 1\ucc28\uc804\uc5d0\uc11c \ub8e8\ube0c \ub9c8\ucffc\ub4dc\uc640 \uc120\ubc1c \ub9de\ub300\uacb0\uc744 \ubc8c\uc5ec 5\ud53c\uc548\ud0c0 1\uc2e4\uc810\ud558\uba70 \ud300\uc758 3-1 \uc2b9\ub9ac\ub97c \uc774\ub04c\uc5c8\ub2e4. \ub098\ud758 \ub4a4 \uce58\ub7ec\uc9c4 4\ucc28\uc804\uc5d0\uc11c\ub294 5\ud53c\uc548\ud0c0 1\uc2e4\uc810\ud588\uace0 \ud300\uc740 5-1\ub85c \uc2b9\ub9ac\ud588\ub2e4. \uc2dc\ub9ac\uc988 \ub9c8\uc9c0\ub9c9 \uacbd\uae30 7\ucc28\uc804\uc5d0\uc11c \ub2e4\uc2dc \uc120\ubc1c \ub4f1\ud310\ud574 \uc2a4\ud54f\ubcfc\ub7ec \ubc8c\ub9ac \uadf8\ub77c\uc784\uc2a4\uc640 \ub9de\ub300\uacb0\uc744 \ud3bc\ucce4\uace0, 5\ud53c\uc548\ud0c0 \uc644\ubd09\uc2b9\uc744 \uac70\ub450\uba74\uc11c \ud300\uc758 3-0 \uc2b9\ub9ac\ub97c \uc774\ub04c\uc5c8\ub2e4. \ud074\ub9ac\ube14\ub79c\ub4dc \uc778\ub514\uc5b8\uc2a4\ub294 \ud504\ub79c\ucc28\uc774\uc988 \uc5ed\uc0ac\uc0c1 \uccab \uc6d4\ub4dc \uc2dc\ub9ac\uc988 \uc6b0\uc2b9\uc744 \ucc28\uc9c0\ud588\ub2e4. \ucf54\ubca8\ub808\uc2a4\ud0a4\ub294 \uc774\ubc88 \uc2dc\ub9ac\uc988\uc5d0\uc11c 0.67\uc758 \ud3c9\uade0\uc790\ucc45\uc810\uc744 \uae30\ub85d\ud588\ub294\ub370, \uc774\ub294 \uc6d4\ub4dc \uc2dc\ub9ac\uc988 \uae30\ub85d\uc73c\ub85c \ub0a8\uc544\uc788\ub2e4."} {"question": "\uc815\ubd80\uc758 \uc138 \ubc88\uc9f8 \uac1c\ub150\uc740 \uad70\uc8fc\uac00 \ubb34\uc5c7\uc758 \uc81c\uc57d\uc744 \ubc1b\ub294\uac00?", "paragraph": "\uadf8\ub7ec\ub098 \uc815\ubd80\uc758 \uccab \ubc88\uc9f8 \uac1c\ub150\uc740 \uc624\ub298\ub0a0\uc5d0\ub3c4 \uc601\u00b7\ubbf8\uad6d(\u82f1\u00b7\u7f8e\u570b)\uc5d0\uc11c \ud754\ud788 \uc0ac\uc6a9\ub418\uace0 \uc788\ub294\ub370, \uc601\u00b7\ubbf8\uad6d\uc5d0\uc11c\ub294 \ubbfc\uc120 \ub300\ud45c(\u6c11\u9078\u4ee3\u8868)\ub85c\uc368 \uad6c\uc131\ub418\ub294 \uad6d\ud68c\ub97c \uae30\ubc18\uc73c\ub85c \ud558\uc5ec \uad6d\uac00\uc758 \ubaa8\ub4e0 \ud1b5\uce58 \uae30\uad6c\ub97c \uc124\uc815\ud55c\ub2e4\ub294 \ubfcc\ub9ac \uae4a\uc740 \uad00\ub150\uc774 \uc9c0\ubc30\ud558\uace0 \uc788\uae30 \ub54c\ubb38\uc774\ub2e4. \uc815\ubd80\uc758 \ub450 \ubc88\uc9f8 \uac1c\ub150\uc740 \uc804\uacfc\ub294 \ubc18\ub300\ub85c \uad8c\ub825 \ubd84\ub9bd\uc8fc\uc758\uc5d0 \uc758\uac70\ud55c \uc815\uce58 \uae30\uad6c\uac00 \uc124\uc815\ub41c \uc774\ud6c4\uc5d0\uc57c \ube44\ub85c\uc18c \ubc1c\uc0dd\ud55c \uac1c\ub150\uc778\ub370, \ud604\uc7ac \ub3c5\uc77c\uacfc \uc77c\ubcf8\uc5d0\uc11c \ud754\ud788 \uc0ac\uc6a9\ub418\uace0 \uc788\ub2e4. \uadf8 \uc774\uc720\ub294 \uc774 \ub450 \uad6d\uac00\uc5d0\uc11c\ub294 \uad6d\ubbfc \uc8fc\uad8c \uc0ac\uc0c1\uc758 \ubc1c\uc804\uc774 \ube44\uad50\uc801\uc73c\ub85c \uc9c0\uccb4\ub418\uc5c8\uace0, \ub530\ub77c\uc11c \ubbfc\uc120 \uc758\ud68c\ub97c \uae30\ubc18\uc73c\ub85c \ud558\uc5ec \uad6d\uac00\uc758 \ubaa8\ub4e0 \uc815\uce58 \uae30\uad6c\ub97c \uc124\uc815\ud55c\ub2e4\ub294 \uad00\ub150\uc774 \uc5c6\uc5c8\uace0, \uadf8 \ubc18\uba74\uc5d0 \uad8c\ub825 \ubd84\ub9bd\uc5d0 \uc911\uc810\uc744 \ub450\ub294 \uc785\ud5cc\uad70\uc8fc\uc81c \ud558\uc5d0\uc11c \uc624\ub7ab\ub3d9\uc548 \ud1b5\uce58\ub97c \ubc1b\uc544 \uc654\uae30 \ub54c\ubb38\uc774\ub2e4. \uc815\ubd80\uc758 \uc138 \ubc88\uc9f8 \uac1c\ub150\uc740 \uad70\uc8fc\uac00 \ubbfc\uc120 \uad6d\ud68c(\u6c11\u9078\u570b\u6703)\uc758 \uc81c\uc57d\uc744 \ubc1b\uc9c0\ub9cc, \uc544\uc9c1\ub3c4 \uac15\ub300\ud55c \uad8c\ud55c\uc744 \uac00\uc9c0\ub294 \uc774\ub978\ubc14 \uc785\ud5cc \uad70\uc8fc \uc815\uccb4\ub85c \uad70\uc8fc\uac00 \ub2e8\uc9c0 \uc0c1\uc9d5\uc801 \ub300\ud45c\uc801 \uad8c\ud55c\ubc16\uc5d0 \uac00\uc9c0\uc9c0 \ubabb\ud558\ub294 \uc774\ub978\ubc14 \uc758\ud68c\uc81c \uad70\uc8fc(parliam\u00adentarische Monarchie) \uc815\uccb4\ub85c \ubc1c\uc804\ud568\uc73c\ub85c\uc368 \ud655\ub9bd\ub41c \uac1c\ub150\uc774\ub2e4. \ub05d\uc73c\ub85c \uad6d\uac00\uc758 \uc911\uc559 \uc815\ubd80\uc640 \ub300\ub9bd\ub418\ub294 \uac1c\ub150\ub3c4 \uc788\uc9c0\ub9cc, \uc774\uac83\uc740 \ud5cc\ubc95\uc0c1 \uad6d\uac00 \uad8c\ub825\uc774 \uc0c1\ubc29(\u4e0a\u65b9\uff1a\u806f\u90a6)\uacfc \ud558\ubc29(\u4e0b\u65b9\uff1a\u652f\u5206\u570b) \uac04\uc5d0 \ubd84\ud560\ub418\uc5b4 \uc788\uc5b4\uc11c \uc0c1\ud638\uac04\uc5d0 \uce68\ud574\ud558\uc9c0 \ubabb\ud558\ub3c4\ub85d \ub418\uc5b4 \uc788\ub294 \uc5f0\ubc29\uc81c \uad6d\uac00\uc5d0\uc11c\ub294 \uc758\uc758\uac00 \uc788\uaca0\uc9c0\ub9cc, \uc6d0\uce59\uc801\uc73c\ub85c \uad6d\uac00 \uad8c\ub825\uc774 \uc911\uc559 \uc815\ubd80(\uad6d\uac00)\uc5d0 \uc9d1\uc911 \ud1b5\uc77c\ub418\uc5b4 \uc788\ub294 \ud1b5\uc77c\uc81c \uad6d\uac00\uc5d0\uc11c\ub294 \uadf8\ub2e4\uc9c0 \ud070 \uc758\uc758\ub97c \uac00\uc9c8 \uc218 \uc5c6\ub2e4.", "answer": "\ubbfc\uc120 \uad6d\ud68c", "sentence": "\uc815\ubd80\uc758 \uc138 \ubc88\uc9f8 \uac1c\ub150\uc740 \uad70\uc8fc\uac00 \ubbfc\uc120 \uad6d\ud68c (\u6c11\u9078\u570b\u6703)", "paragraph_sentence": "\uadf8\ub7ec\ub098 \uc815\ubd80\uc758 \uccab \ubc88\uc9f8 \uac1c\ub150\uc740 \uc624\ub298\ub0a0\uc5d0\ub3c4 \uc601\u00b7\ubbf8\uad6d(\u82f1\u00b7\u7f8e\u570b)\uc5d0\uc11c \ud754\ud788 \uc0ac\uc6a9\ub418\uace0 \uc788\ub294\ub370, \uc601\u00b7\ubbf8\uad6d\uc5d0\uc11c\ub294 \ubbfc\uc120 \ub300\ud45c(\u6c11\u9078\u4ee3\u8868)\ub85c\uc368 \uad6c\uc131\ub418\ub294 \uad6d\ud68c\ub97c \uae30\ubc18\uc73c\ub85c \ud558\uc5ec \uad6d\uac00\uc758 \ubaa8\ub4e0 \ud1b5\uce58 \uae30\uad6c\ub97c \uc124\uc815\ud55c\ub2e4\ub294 \ubfcc\ub9ac \uae4a\uc740 \uad00\ub150\uc774 \uc9c0\ubc30\ud558\uace0 \uc788\uae30 \ub54c\ubb38\uc774\ub2e4. \uc815\ubd80\uc758 \ub450 \ubc88\uc9f8 \uac1c\ub150\uc740 \uc804\uacfc\ub294 \ubc18\ub300\ub85c \uad8c\ub825 \ubd84\ub9bd\uc8fc\uc758\uc5d0 \uc758\uac70\ud55c \uc815\uce58 \uae30\uad6c\uac00 \uc124\uc815\ub41c \uc774\ud6c4\uc5d0\uc57c \ube44\ub85c\uc18c \ubc1c\uc0dd\ud55c \uac1c\ub150\uc778\ub370, \ud604\uc7ac \ub3c5\uc77c\uacfc \uc77c\ubcf8\uc5d0\uc11c \ud754\ud788 \uc0ac\uc6a9\ub418\uace0 \uc788\ub2e4. \uadf8 \uc774\uc720\ub294 \uc774 \ub450 \uad6d\uac00\uc5d0\uc11c\ub294 \uad6d\ubbfc \uc8fc\uad8c \uc0ac\uc0c1\uc758 \ubc1c\uc804\uc774 \ube44\uad50\uc801\uc73c\ub85c \uc9c0\uccb4\ub418\uc5c8\uace0, \ub530\ub77c\uc11c \ubbfc\uc120 \uc758\ud68c\ub97c \uae30\ubc18\uc73c\ub85c \ud558\uc5ec \uad6d\uac00\uc758 \ubaa8\ub4e0 \uc815\uce58 \uae30\uad6c\ub97c \uc124\uc815\ud55c\ub2e4\ub294 \uad00\ub150\uc774 \uc5c6\uc5c8\uace0, \uadf8 \ubc18\uba74\uc5d0 \uad8c\ub825 \ubd84\ub9bd\uc5d0 \uc911\uc810\uc744 \ub450\ub294 \uc785\ud5cc\uad70\uc8fc\uc81c \ud558\uc5d0\uc11c \uc624\ub7ab\ub3d9\uc548 \ud1b5\uce58\ub97c \ubc1b\uc544 \uc654\uae30 \ub54c\ubb38\uc774\ub2e4. \uc815\ubd80\uc758 \uc138 \ubc88\uc9f8 \uac1c\ub150\uc740 \uad70\uc8fc\uac00 \ubbfc\uc120 \uad6d\ud68c (\u6c11\u9078\u570b\u6703) \uc758 \uc81c\uc57d\uc744 \ubc1b\uc9c0\ub9cc, \uc544\uc9c1\ub3c4 \uac15\ub300\ud55c \uad8c\ud55c\uc744 \uac00\uc9c0\ub294 \uc774\ub978\ubc14 \uc785\ud5cc \uad70\uc8fc \uc815\uccb4\ub85c \uad70\uc8fc\uac00 \ub2e8\uc9c0 \uc0c1\uc9d5\uc801 \ub300\ud45c\uc801 \uad8c\ud55c\ubc16\uc5d0 \uac00\uc9c0\uc9c0 \ubabb\ud558\ub294 \uc774\ub978\ubc14 \uc758\ud68c\uc81c \uad70\uc8fc(parliam\u00adentarische Monarchie) \uc815\uccb4\ub85c \ubc1c\uc804\ud568\uc73c\ub85c\uc368 \ud655\ub9bd\ub41c \uac1c\ub150\uc774\ub2e4. \ub05d\uc73c\ub85c \uad6d\uac00\uc758 \uc911\uc559 \uc815\ubd80\uc640 \ub300\ub9bd\ub418\ub294 \uac1c\ub150\ub3c4 \uc788\uc9c0\ub9cc, \uc774\uac83\uc740 \ud5cc\ubc95\uc0c1 \uad6d\uac00 \uad8c\ub825\uc774 \uc0c1\ubc29(\u4e0a\u65b9\uff1a\u806f\u90a6)\uacfc \ud558\ubc29(\u4e0b\u65b9\uff1a\u652f\u5206\u570b) \uac04\uc5d0 \ubd84\ud560\ub418\uc5b4 \uc788\uc5b4\uc11c \uc0c1\ud638\uac04\uc5d0 \uce68\ud574\ud558\uc9c0 \ubabb\ud558\ub3c4\ub85d \ub418\uc5b4 \uc788\ub294 \uc5f0\ubc29\uc81c \uad6d\uac00\uc5d0\uc11c\ub294 \uc758\uc758\uac00 \uc788\uaca0\uc9c0\ub9cc, \uc6d0\uce59\uc801\uc73c\ub85c \uad6d\uac00 \uad8c\ub825\uc774 \uc911\uc559 \uc815\ubd80(\uad6d\uac00)\uc5d0 \uc9d1\uc911 \ud1b5\uc77c\ub418\uc5b4 \uc788\ub294 \ud1b5\uc77c\uc81c \uad6d\uac00\uc5d0\uc11c\ub294 \uadf8\ub2e4\uc9c0 \ud070 \uc758\uc758\ub97c \uac00\uc9c8 \uc218 \uc5c6\ub2e4.", "paragraph_answer": "\uadf8\ub7ec\ub098 \uc815\ubd80\uc758 \uccab \ubc88\uc9f8 \uac1c\ub150\uc740 \uc624\ub298\ub0a0\uc5d0\ub3c4 \uc601\u00b7\ubbf8\uad6d(\u82f1\u00b7\u7f8e\u570b)\uc5d0\uc11c \ud754\ud788 \uc0ac\uc6a9\ub418\uace0 \uc788\ub294\ub370, \uc601\u00b7\ubbf8\uad6d\uc5d0\uc11c\ub294 \ubbfc\uc120 \ub300\ud45c(\u6c11\u9078\u4ee3\u8868)\ub85c\uc368 \uad6c\uc131\ub418\ub294 \uad6d\ud68c\ub97c \uae30\ubc18\uc73c\ub85c \ud558\uc5ec \uad6d\uac00\uc758 \ubaa8\ub4e0 \ud1b5\uce58 \uae30\uad6c\ub97c \uc124\uc815\ud55c\ub2e4\ub294 \ubfcc\ub9ac \uae4a\uc740 \uad00\ub150\uc774 \uc9c0\ubc30\ud558\uace0 \uc788\uae30 \ub54c\ubb38\uc774\ub2e4. \uc815\ubd80\uc758 \ub450 \ubc88\uc9f8 \uac1c\ub150\uc740 \uc804\uacfc\ub294 \ubc18\ub300\ub85c \uad8c\ub825 \ubd84\ub9bd\uc8fc\uc758\uc5d0 \uc758\uac70\ud55c \uc815\uce58 \uae30\uad6c\uac00 \uc124\uc815\ub41c \uc774\ud6c4\uc5d0\uc57c \ube44\ub85c\uc18c \ubc1c\uc0dd\ud55c \uac1c\ub150\uc778\ub370, \ud604\uc7ac \ub3c5\uc77c\uacfc \uc77c\ubcf8\uc5d0\uc11c \ud754\ud788 \uc0ac\uc6a9\ub418\uace0 \uc788\ub2e4. \uadf8 \uc774\uc720\ub294 \uc774 \ub450 \uad6d\uac00\uc5d0\uc11c\ub294 \uad6d\ubbfc \uc8fc\uad8c \uc0ac\uc0c1\uc758 \ubc1c\uc804\uc774 \ube44\uad50\uc801\uc73c\ub85c \uc9c0\uccb4\ub418\uc5c8\uace0, \ub530\ub77c\uc11c \ubbfc\uc120 \uc758\ud68c\ub97c \uae30\ubc18\uc73c\ub85c \ud558\uc5ec \uad6d\uac00\uc758 \ubaa8\ub4e0 \uc815\uce58 \uae30\uad6c\ub97c \uc124\uc815\ud55c\ub2e4\ub294 \uad00\ub150\uc774 \uc5c6\uc5c8\uace0, \uadf8 \ubc18\uba74\uc5d0 \uad8c\ub825 \ubd84\ub9bd\uc5d0 \uc911\uc810\uc744 \ub450\ub294 \uc785\ud5cc\uad70\uc8fc\uc81c \ud558\uc5d0\uc11c \uc624\ub7ab\ub3d9\uc548 \ud1b5\uce58\ub97c \ubc1b\uc544 \uc654\uae30 \ub54c\ubb38\uc774\ub2e4. \uc815\ubd80\uc758 \uc138 \ubc88\uc9f8 \uac1c\ub150\uc740 \uad70\uc8fc\uac00 \ubbfc\uc120 \uad6d\ud68c (\u6c11\u9078\u570b\u6703)\uc758 \uc81c\uc57d\uc744 \ubc1b\uc9c0\ub9cc, \uc544\uc9c1\ub3c4 \uac15\ub300\ud55c \uad8c\ud55c\uc744 \uac00\uc9c0\ub294 \uc774\ub978\ubc14 \uc785\ud5cc \uad70\uc8fc \uc815\uccb4\ub85c \uad70\uc8fc\uac00 \ub2e8\uc9c0 \uc0c1\uc9d5\uc801 \ub300\ud45c\uc801 \uad8c\ud55c\ubc16\uc5d0 \uac00\uc9c0\uc9c0 \ubabb\ud558\ub294 \uc774\ub978\ubc14 \uc758\ud68c\uc81c \uad70\uc8fc(parliam\u00adentarische Monarchie) \uc815\uccb4\ub85c \ubc1c\uc804\ud568\uc73c\ub85c\uc368 \ud655\ub9bd\ub41c \uac1c\ub150\uc774\ub2e4. \ub05d\uc73c\ub85c \uad6d\uac00\uc758 \uc911\uc559 \uc815\ubd80\uc640 \ub300\ub9bd\ub418\ub294 \uac1c\ub150\ub3c4 \uc788\uc9c0\ub9cc, \uc774\uac83\uc740 \ud5cc\ubc95\uc0c1 \uad6d\uac00 \uad8c\ub825\uc774 \uc0c1\ubc29(\u4e0a\u65b9\uff1a\u806f\u90a6)\uacfc \ud558\ubc29(\u4e0b\u65b9\uff1a\u652f\u5206\u570b) \uac04\uc5d0 \ubd84\ud560\ub418\uc5b4 \uc788\uc5b4\uc11c \uc0c1\ud638\uac04\uc5d0 \uce68\ud574\ud558\uc9c0 \ubabb\ud558\ub3c4\ub85d \ub418\uc5b4 \uc788\ub294 \uc5f0\ubc29\uc81c \uad6d\uac00\uc5d0\uc11c\ub294 \uc758\uc758\uac00 \uc788\uaca0\uc9c0\ub9cc, \uc6d0\uce59\uc801\uc73c\ub85c \uad6d\uac00 \uad8c\ub825\uc774 \uc911\uc559 \uc815\ubd80(\uad6d\uac00)\uc5d0 \uc9d1\uc911 \ud1b5\uc77c\ub418\uc5b4 \uc788\ub294 \ud1b5\uc77c\uc81c \uad6d\uac00\uc5d0\uc11c\ub294 \uadf8\ub2e4\uc9c0 \ud070 \uc758\uc758\ub97c \uac00\uc9c8 \uc218 \uc5c6\ub2e4.", "sentence_answer": "\uc815\ubd80\uc758 \uc138 \ubc88\uc9f8 \uac1c\ub150\uc740 \uad70\uc8fc\uac00 \ubbfc\uc120 \uad6d\ud68c (\u6c11\u9078\u570b\u6703)", "paragraph_id": "6550170-1-2", "paragraph_question": "question: \uc815\ubd80\uc758 \uc138 \ubc88\uc9f8 \uac1c\ub150\uc740 \uad70\uc8fc\uac00 \ubb34\uc5c7\uc758 \uc81c\uc57d\uc744 \ubc1b\ub294\uac00?, context: \uadf8\ub7ec\ub098 \uc815\ubd80\uc758 \uccab \ubc88\uc9f8 \uac1c\ub150\uc740 \uc624\ub298\ub0a0\uc5d0\ub3c4 \uc601\u00b7\ubbf8\uad6d(\u82f1\u00b7\u7f8e\u570b)\uc5d0\uc11c \ud754\ud788 \uc0ac\uc6a9\ub418\uace0 \uc788\ub294\ub370, \uc601\u00b7\ubbf8\uad6d\uc5d0\uc11c\ub294 \ubbfc\uc120 \ub300\ud45c(\u6c11\u9078\u4ee3\u8868)\ub85c\uc368 \uad6c\uc131\ub418\ub294 \uad6d\ud68c\ub97c \uae30\ubc18\uc73c\ub85c \ud558\uc5ec \uad6d\uac00\uc758 \ubaa8\ub4e0 \ud1b5\uce58 \uae30\uad6c\ub97c \uc124\uc815\ud55c\ub2e4\ub294 \ubfcc\ub9ac \uae4a\uc740 \uad00\ub150\uc774 \uc9c0\ubc30\ud558\uace0 \uc788\uae30 \ub54c\ubb38\uc774\ub2e4. \uc815\ubd80\uc758 \ub450 \ubc88\uc9f8 \uac1c\ub150\uc740 \uc804\uacfc\ub294 \ubc18\ub300\ub85c \uad8c\ub825 \ubd84\ub9bd\uc8fc\uc758\uc5d0 \uc758\uac70\ud55c \uc815\uce58 \uae30\uad6c\uac00 \uc124\uc815\ub41c \uc774\ud6c4\uc5d0\uc57c \ube44\ub85c\uc18c \ubc1c\uc0dd\ud55c \uac1c\ub150\uc778\ub370, \ud604\uc7ac \ub3c5\uc77c\uacfc \uc77c\ubcf8\uc5d0\uc11c \ud754\ud788 \uc0ac\uc6a9\ub418\uace0 \uc788\ub2e4. \uadf8 \uc774\uc720\ub294 \uc774 \ub450 \uad6d\uac00\uc5d0\uc11c\ub294 \uad6d\ubbfc \uc8fc\uad8c \uc0ac\uc0c1\uc758 \ubc1c\uc804\uc774 \ube44\uad50\uc801\uc73c\ub85c \uc9c0\uccb4\ub418\uc5c8\uace0, \ub530\ub77c\uc11c \ubbfc\uc120 \uc758\ud68c\ub97c \uae30\ubc18\uc73c\ub85c \ud558\uc5ec \uad6d\uac00\uc758 \ubaa8\ub4e0 \uc815\uce58 \uae30\uad6c\ub97c \uc124\uc815\ud55c\ub2e4\ub294 \uad00\ub150\uc774 \uc5c6\uc5c8\uace0, \uadf8 \ubc18\uba74\uc5d0 \uad8c\ub825 \ubd84\ub9bd\uc5d0 \uc911\uc810\uc744 \ub450\ub294 \uc785\ud5cc\uad70\uc8fc\uc81c \ud558\uc5d0\uc11c \uc624\ub7ab\ub3d9\uc548 \ud1b5\uce58\ub97c \ubc1b\uc544 \uc654\uae30 \ub54c\ubb38\uc774\ub2e4. \uc815\ubd80\uc758 \uc138 \ubc88\uc9f8 \uac1c\ub150\uc740 \uad70\uc8fc\uac00 \ubbfc\uc120 \uad6d\ud68c(\u6c11\u9078\u570b\u6703)\uc758 \uc81c\uc57d\uc744 \ubc1b\uc9c0\ub9cc, \uc544\uc9c1\ub3c4 \uac15\ub300\ud55c \uad8c\ud55c\uc744 \uac00\uc9c0\ub294 \uc774\ub978\ubc14 \uc785\ud5cc \uad70\uc8fc \uc815\uccb4\ub85c \uad70\uc8fc\uac00 \ub2e8\uc9c0 \uc0c1\uc9d5\uc801 \ub300\ud45c\uc801 \uad8c\ud55c\ubc16\uc5d0 \uac00\uc9c0\uc9c0 \ubabb\ud558\ub294 \uc774\ub978\ubc14 \uc758\ud68c\uc81c \uad70\uc8fc(parliam\u00adentarische Monarchie) \uc815\uccb4\ub85c \ubc1c\uc804\ud568\uc73c\ub85c\uc368 \ud655\ub9bd\ub41c \uac1c\ub150\uc774\ub2e4. \ub05d\uc73c\ub85c \uad6d\uac00\uc758 \uc911\uc559 \uc815\ubd80\uc640 \ub300\ub9bd\ub418\ub294 \uac1c\ub150\ub3c4 \uc788\uc9c0\ub9cc, \uc774\uac83\uc740 \ud5cc\ubc95\uc0c1 \uad6d\uac00 \uad8c\ub825\uc774 \uc0c1\ubc29(\u4e0a\u65b9\uff1a\u806f\u90a6)\uacfc \ud558\ubc29(\u4e0b\u65b9\uff1a\u652f\u5206\u570b) \uac04\uc5d0 \ubd84\ud560\ub418\uc5b4 \uc788\uc5b4\uc11c \uc0c1\ud638\uac04\uc5d0 \uce68\ud574\ud558\uc9c0 \ubabb\ud558\ub3c4\ub85d \ub418\uc5b4 \uc788\ub294 \uc5f0\ubc29\uc81c \uad6d\uac00\uc5d0\uc11c\ub294 \uc758\uc758\uac00 \uc788\uaca0\uc9c0\ub9cc, \uc6d0\uce59\uc801\uc73c\ub85c \uad6d\uac00 \uad8c\ub825\uc774 \uc911\uc559 \uc815\ubd80(\uad6d\uac00)\uc5d0 \uc9d1\uc911 \ud1b5\uc77c\ub418\uc5b4 \uc788\ub294 \ud1b5\uc77c\uc81c \uad6d\uac00\uc5d0\uc11c\ub294 \uadf8\ub2e4\uc9c0 \ud070 \uc758\uc758\ub97c \uac00\uc9c8 \uc218 \uc5c6\ub2e4."} {"question": "\uc544\uba54\ub9ac\uce78 \ud56d\uacf5 1420\ud3b8\uc740 \ud56d\uacf5\uae30 \ub3c4\ucc29 \uc9c0\uc5f0\uc73c\ub85c \uc5b4\ub5a4 \ud56d\uacf5\uae30\ub85c \uad50\uccb4 \ub418\uc5c8\ub294\uac00?", "paragraph": "1420\ud3b8\uc740 \uc911\ubd80 \ud45c\uc900\uc2dc 20:28\ubd84\uc5d0 \ub308\ub7ec\uc2a4 \ud3ec\ud2b8\uc6cc\uc2a4\ub97c \ucd9c\ubc1c\ud558\uc5ec 21:41\ubd84\uc5d0 \ub9ac\ud2c0 \ub85d\uc5d0 \ub3c4\ucc29\ud558\ub294 \ube44\ud589\uc774\uc5c8\ub2e4. \ud558\uc9c0\ub9cc \ube44\ud589 \uc2b9\ubb34\uc6d0\ub4e4\uc740 1420\ud3b8\uc5d0 \ud0d1\uc2b9\ud558\uae30 \uc804 \uc870\uc5b8\uc744 \ubc1b\uc558\ub294\ub370 \ucd9c\ubc1c\uc774 \uc9c0\uc5f0\ub420 \uac83\uc774\uba70, \ubbf8\uad6d \uae30\uc0c1\uccad\uc5d0\uc11c \ubc1c\ud45c\ud55c \ud56d\uacf5 \uae30\uc0c1\uc5d0\uc11c\ub294 1420\ud3b8\uc758 \uacc4\ud68d\ub41c \ube44\ud589 \uacbd\ub85c\ub97c \ub530\ub77c \uaca9\uc2ec\ud55c \ub1cc\uc6b0\uac00 \ud45c\uc2dc\ub418\uace0 \uc788\ub2e4\ub294 \ub0b4\uc6a9\uc774\uc5c8\ub2e4. \uc545\ucc9c\ud6c4\ub85c \uc778\ud574 1420\ud3b8\uc73c\ub85c \uc774\ub959\ud560 \ud56d\uacf5\uae30\uc758 \ub308\ub7ec\uc2a4 \ud3ec\ud2b8\uc6cc\uc2a4 \ub3c4\ucc29\uc740 \uc9c0\uc5f0\ub418\uc5c8\uc73c\uba70 \ube44\ud589 \uc2b9\ubb34\uc6d0\ub4e4\uc740 \ud0d1\uc2b9 \uac8c\uc774\ud2b8 \uc55e\uc5d0\uc11c \uacc4\uc18d \uae30\ub2e4\ub824\uc57c \ud588\ub2e4. \ud56d\uacf5 \uaddc\uc815\uc5d0 \ub530\ub974\uba74 \uc870\uc885\uc0ac\uc758 \ucd5c\ub300 \uadfc\ubb34\uc2dc\uac04\uc744 14\uc2dc\uac04\uc774\uace0 1420\ud3b8\uc740 \uadf8\ub4e4\uc758 \uadfc\ubb34\uc77c \ub9c8\uc9c0\ub9c9 \ube44\ud589 \uc2a4\ucf00\uc904\uc774\uc5c8\ub2e4. \ubd80\uc870\uc885\uc0ac\ub294 \ub9cc\uc57d 1420\ud3b8\uc774 23:16\ubd84\uae4c\uc9c0 \ucd9c\ubc1c\ud558\uc9c0 \ubabb\ud560 \uacbd\uc6b0 \uadf8\ub4e4\uc758 \uaddc\uc815\ub41c \uc2dc\uac04\uc744 \ucd08\uacfc\ud560 \uac83\uc774\ub77c\uace0 \ud56d\uacf5\uc0ac\uc758 \ube44\ud589 \uc6b4\ud56d \uad00\ub9ac\uc790\uc5d0 \ud1b5\ubcf4\ud588\ub2e4. \uadf8\ub9ac\ud558\uc5ec \ud56d\uacf5\uae30\ub294 \ub2e4\ub978 MD-80\uc73c\ub85c \uad50\uccb4\ub418\uc5c8\uc73c\uba70 \uae30\uccb4 \ub4f1\ub85d \ubc88\ud638\ub294 N215AA, 22:40\ubd84\uc5d0 \ub308\ub7ec\uc2a4 \ud3ec\ud2b8\uc6cc\uc2a4 \ucd9c\ubc1c\uc744 \ud5c8\uac00\ud588\ub2e4.", "answer": "MD-80", "sentence": "\uadf8\ub9ac\ud558\uc5ec \ud56d\uacf5\uae30\ub294 \ub2e4\ub978 MD-80 \uc73c\ub85c \uad50\uccb4\ub418\uc5c8\uc73c\uba70 \uae30\uccb4 \ub4f1\ub85d \ubc88\ud638\ub294 N215AA, 22:40\ubd84\uc5d0 \ub308\ub7ec\uc2a4 \ud3ec\ud2b8\uc6cc\uc2a4 \ucd9c\ubc1c\uc744 \ud5c8\uac00\ud588\ub2e4.", "paragraph_sentence": "1420\ud3b8\uc740 \uc911\ubd80 \ud45c\uc900\uc2dc 20:28\ubd84\uc5d0 \ub308\ub7ec\uc2a4 \ud3ec\ud2b8\uc6cc\uc2a4\ub97c \ucd9c\ubc1c\ud558\uc5ec 21:41\ubd84\uc5d0 \ub9ac\ud2c0 \ub85d\uc5d0 \ub3c4\ucc29\ud558\ub294 \ube44\ud589\uc774\uc5c8\ub2e4. \ud558\uc9c0\ub9cc \ube44\ud589 \uc2b9\ubb34\uc6d0\ub4e4\uc740 1420\ud3b8\uc5d0 \ud0d1\uc2b9\ud558\uae30 \uc804 \uc870\uc5b8\uc744 \ubc1b\uc558\ub294\ub370 \ucd9c\ubc1c\uc774 \uc9c0\uc5f0\ub420 \uac83\uc774\uba70, \ubbf8\uad6d \uae30\uc0c1\uccad\uc5d0\uc11c \ubc1c\ud45c\ud55c \ud56d\uacf5 \uae30\uc0c1\uc5d0\uc11c\ub294 1420\ud3b8\uc758 \uacc4\ud68d\ub41c \ube44\ud589 \uacbd\ub85c\ub97c \ub530\ub77c \uaca9\uc2ec\ud55c \ub1cc\uc6b0\uac00 \ud45c\uc2dc\ub418\uace0 \uc788\ub2e4\ub294 \ub0b4\uc6a9\uc774\uc5c8\ub2e4. \uc545\ucc9c\ud6c4\ub85c \uc778\ud574 1420\ud3b8\uc73c\ub85c \uc774\ub959\ud560 \ud56d\uacf5\uae30\uc758 \ub308\ub7ec\uc2a4 \ud3ec\ud2b8\uc6cc\uc2a4 \ub3c4\ucc29\uc740 \uc9c0\uc5f0\ub418\uc5c8\uc73c\uba70 \ube44\ud589 \uc2b9\ubb34\uc6d0\ub4e4\uc740 \ud0d1\uc2b9 \uac8c\uc774\ud2b8 \uc55e\uc5d0\uc11c \uacc4\uc18d \uae30\ub2e4\ub824\uc57c \ud588\ub2e4. \ud56d\uacf5 \uaddc\uc815\uc5d0 \ub530\ub974\uba74 \uc870\uc885\uc0ac\uc758 \ucd5c\ub300 \uadfc\ubb34\uc2dc\uac04\uc744 14\uc2dc\uac04\uc774\uace0 1420\ud3b8\uc740 \uadf8\ub4e4\uc758 \uadfc\ubb34\uc77c \ub9c8\uc9c0\ub9c9 \ube44\ud589 \uc2a4\ucf00\uc904\uc774\uc5c8\ub2e4. \ubd80\uc870\uc885\uc0ac\ub294 \ub9cc\uc57d 1420\ud3b8\uc774 23:16\ubd84\uae4c\uc9c0 \ucd9c\ubc1c\ud558\uc9c0 \ubabb\ud560 \uacbd\uc6b0 \uadf8\ub4e4\uc758 \uaddc\uc815\ub41c \uc2dc\uac04\uc744 \ucd08\uacfc\ud560 \uac83\uc774\ub77c\uace0 \ud56d\uacf5\uc0ac\uc758 \ube44\ud589 \uc6b4\ud56d \uad00\ub9ac\uc790\uc5d0 \ud1b5\ubcf4\ud588\ub2e4. \uadf8\ub9ac\ud558\uc5ec \ud56d\uacf5\uae30\ub294 \ub2e4\ub978 MD-80 \uc73c\ub85c \uad50\uccb4\ub418\uc5c8\uc73c\uba70 \uae30\uccb4 \ub4f1\ub85d \ubc88\ud638\ub294 N215AA, 22:40\ubd84\uc5d0 \ub308\ub7ec\uc2a4 \ud3ec\ud2b8\uc6cc\uc2a4 \ucd9c\ubc1c\uc744 \ud5c8\uac00\ud588\ub2e4. ", "paragraph_answer": "1420\ud3b8\uc740 \uc911\ubd80 \ud45c\uc900\uc2dc 20:28\ubd84\uc5d0 \ub308\ub7ec\uc2a4 \ud3ec\ud2b8\uc6cc\uc2a4\ub97c \ucd9c\ubc1c\ud558\uc5ec 21:41\ubd84\uc5d0 \ub9ac\ud2c0 \ub85d\uc5d0 \ub3c4\ucc29\ud558\ub294 \ube44\ud589\uc774\uc5c8\ub2e4. \ud558\uc9c0\ub9cc \ube44\ud589 \uc2b9\ubb34\uc6d0\ub4e4\uc740 1420\ud3b8\uc5d0 \ud0d1\uc2b9\ud558\uae30 \uc804 \uc870\uc5b8\uc744 \ubc1b\uc558\ub294\ub370 \ucd9c\ubc1c\uc774 \uc9c0\uc5f0\ub420 \uac83\uc774\uba70, \ubbf8\uad6d \uae30\uc0c1\uccad\uc5d0\uc11c \ubc1c\ud45c\ud55c \ud56d\uacf5 \uae30\uc0c1\uc5d0\uc11c\ub294 1420\ud3b8\uc758 \uacc4\ud68d\ub41c \ube44\ud589 \uacbd\ub85c\ub97c \ub530\ub77c \uaca9\uc2ec\ud55c \ub1cc\uc6b0\uac00 \ud45c\uc2dc\ub418\uace0 \uc788\ub2e4\ub294 \ub0b4\uc6a9\uc774\uc5c8\ub2e4. \uc545\ucc9c\ud6c4\ub85c \uc778\ud574 1420\ud3b8\uc73c\ub85c \uc774\ub959\ud560 \ud56d\uacf5\uae30\uc758 \ub308\ub7ec\uc2a4 \ud3ec\ud2b8\uc6cc\uc2a4 \ub3c4\ucc29\uc740 \uc9c0\uc5f0\ub418\uc5c8\uc73c\uba70 \ube44\ud589 \uc2b9\ubb34\uc6d0\ub4e4\uc740 \ud0d1\uc2b9 \uac8c\uc774\ud2b8 \uc55e\uc5d0\uc11c \uacc4\uc18d \uae30\ub2e4\ub824\uc57c \ud588\ub2e4. \ud56d\uacf5 \uaddc\uc815\uc5d0 \ub530\ub974\uba74 \uc870\uc885\uc0ac\uc758 \ucd5c\ub300 \uadfc\ubb34\uc2dc\uac04\uc744 14\uc2dc\uac04\uc774\uace0 1420\ud3b8\uc740 \uadf8\ub4e4\uc758 \uadfc\ubb34\uc77c \ub9c8\uc9c0\ub9c9 \ube44\ud589 \uc2a4\ucf00\uc904\uc774\uc5c8\ub2e4. \ubd80\uc870\uc885\uc0ac\ub294 \ub9cc\uc57d 1420\ud3b8\uc774 23:16\ubd84\uae4c\uc9c0 \ucd9c\ubc1c\ud558\uc9c0 \ubabb\ud560 \uacbd\uc6b0 \uadf8\ub4e4\uc758 \uaddc\uc815\ub41c \uc2dc\uac04\uc744 \ucd08\uacfc\ud560 \uac83\uc774\ub77c\uace0 \ud56d\uacf5\uc0ac\uc758 \ube44\ud589 \uc6b4\ud56d \uad00\ub9ac\uc790\uc5d0 \ud1b5\ubcf4\ud588\ub2e4. \uadf8\ub9ac\ud558\uc5ec \ud56d\uacf5\uae30\ub294 \ub2e4\ub978 MD-80 \uc73c\ub85c \uad50\uccb4\ub418\uc5c8\uc73c\uba70 \uae30\uccb4 \ub4f1\ub85d \ubc88\ud638\ub294 N215AA, 22:40\ubd84\uc5d0 \ub308\ub7ec\uc2a4 \ud3ec\ud2b8\uc6cc\uc2a4 \ucd9c\ubc1c\uc744 \ud5c8\uac00\ud588\ub2e4.", "sentence_answer": "\uadf8\ub9ac\ud558\uc5ec \ud56d\uacf5\uae30\ub294 \ub2e4\ub978 MD-80 \uc73c\ub85c \uad50\uccb4\ub418\uc5c8\uc73c\uba70 \uae30\uccb4 \ub4f1\ub85d \ubc88\ud638\ub294 N215AA, 22:40\ubd84\uc5d0 \ub308\ub7ec\uc2a4 \ud3ec\ud2b8\uc6cc\uc2a4 \ucd9c\ubc1c\uc744 \ud5c8\uac00\ud588\ub2e4.", "paragraph_id": "6520451-0-2", "paragraph_question": "question: \uc544\uba54\ub9ac\uce78 \ud56d\uacf5 1420\ud3b8\uc740 \ud56d\uacf5\uae30 \ub3c4\ucc29 \uc9c0\uc5f0\uc73c\ub85c \uc5b4\ub5a4 \ud56d\uacf5\uae30\ub85c \uad50\uccb4 \ub418\uc5c8\ub294\uac00?, context: 1420\ud3b8\uc740 \uc911\ubd80 \ud45c\uc900\uc2dc 20:28\ubd84\uc5d0 \ub308\ub7ec\uc2a4 \ud3ec\ud2b8\uc6cc\uc2a4\ub97c \ucd9c\ubc1c\ud558\uc5ec 21:41\ubd84\uc5d0 \ub9ac\ud2c0 \ub85d\uc5d0 \ub3c4\ucc29\ud558\ub294 \ube44\ud589\uc774\uc5c8\ub2e4. \ud558\uc9c0\ub9cc \ube44\ud589 \uc2b9\ubb34\uc6d0\ub4e4\uc740 1420\ud3b8\uc5d0 \ud0d1\uc2b9\ud558\uae30 \uc804 \uc870\uc5b8\uc744 \ubc1b\uc558\ub294\ub370 \ucd9c\ubc1c\uc774 \uc9c0\uc5f0\ub420 \uac83\uc774\uba70, \ubbf8\uad6d \uae30\uc0c1\uccad\uc5d0\uc11c \ubc1c\ud45c\ud55c \ud56d\uacf5 \uae30\uc0c1\uc5d0\uc11c\ub294 1420\ud3b8\uc758 \uacc4\ud68d\ub41c \ube44\ud589 \uacbd\ub85c\ub97c \ub530\ub77c \uaca9\uc2ec\ud55c \ub1cc\uc6b0\uac00 \ud45c\uc2dc\ub418\uace0 \uc788\ub2e4\ub294 \ub0b4\uc6a9\uc774\uc5c8\ub2e4. \uc545\ucc9c\ud6c4\ub85c \uc778\ud574 1420\ud3b8\uc73c\ub85c \uc774\ub959\ud560 \ud56d\uacf5\uae30\uc758 \ub308\ub7ec\uc2a4 \ud3ec\ud2b8\uc6cc\uc2a4 \ub3c4\ucc29\uc740 \uc9c0\uc5f0\ub418\uc5c8\uc73c\uba70 \ube44\ud589 \uc2b9\ubb34\uc6d0\ub4e4\uc740 \ud0d1\uc2b9 \uac8c\uc774\ud2b8 \uc55e\uc5d0\uc11c \uacc4\uc18d \uae30\ub2e4\ub824\uc57c \ud588\ub2e4. \ud56d\uacf5 \uaddc\uc815\uc5d0 \ub530\ub974\uba74 \uc870\uc885\uc0ac\uc758 \ucd5c\ub300 \uadfc\ubb34\uc2dc\uac04\uc744 14\uc2dc\uac04\uc774\uace0 1420\ud3b8\uc740 \uadf8\ub4e4\uc758 \uadfc\ubb34\uc77c \ub9c8\uc9c0\ub9c9 \ube44\ud589 \uc2a4\ucf00\uc904\uc774\uc5c8\ub2e4. \ubd80\uc870\uc885\uc0ac\ub294 \ub9cc\uc57d 1420\ud3b8\uc774 23:16\ubd84\uae4c\uc9c0 \ucd9c\ubc1c\ud558\uc9c0 \ubabb\ud560 \uacbd\uc6b0 \uadf8\ub4e4\uc758 \uaddc\uc815\ub41c \uc2dc\uac04\uc744 \ucd08\uacfc\ud560 \uac83\uc774\ub77c\uace0 \ud56d\uacf5\uc0ac\uc758 \ube44\ud589 \uc6b4\ud56d \uad00\ub9ac\uc790\uc5d0 \ud1b5\ubcf4\ud588\ub2e4. \uadf8\ub9ac\ud558\uc5ec \ud56d\uacf5\uae30\ub294 \ub2e4\ub978 MD-80\uc73c\ub85c \uad50\uccb4\ub418\uc5c8\uc73c\uba70 \uae30\uccb4 \ub4f1\ub85d \ubc88\ud638\ub294 N215AA, 22:40\ubd84\uc5d0 \ub308\ub7ec\uc2a4 \ud3ec\ud2b8\uc6cc\uc2a4 \ucd9c\ubc1c\uc744 \ud5c8\uac00\ud588\ub2e4."} {"question": "\ub3d9\ub3c5\uacfc \uc11c\ub3c5\uc740 \ud504\ub791\ud06c\ud478\ub974\ud2b8 \uad6d\ubbfc\uc758\ud68c\uc758 \uc5b4\ub5a0\ud55c \uac83\uc744 \uc790\uad6d \ud2b9\uc720\uc758 \uc804\ud1b5\uc73c\ub85c \ubcf4\uc5ec\uc8fc\uace0\uc790 \ud558\uc600\ub294\uac00?", "paragraph": "\uad6d\ubbfc\uc758\ud68c\uc758 \uc5c5\uc801\uacfc 3\uc6d4 \ud601\uba85 \uc804\uccb4\ub294 \uc774\ud6c4 \uc218\uc2ed \ub144\uac04 \ub9e4\uc6b0 \ubd80\uc815\uc801\uc73c\ub85c \ud3c9\uac00\ub418\uc5c8\ub2e4. \ub8e8\ud2b8\ube44\ud788 \ud638\uc774\uc11c(Ludwig H\u00e4usser)\uc640 \uac19\uc740 \uae09\uc9c4\ubbfc\uc8fc \uc88c\ud30c\uc758 \uc0dd\uac01\uc740, \ubb34\ucc45\uc784\ud558\uace0 \uc21c\uc9c4\ud55c \ub9dd\uc0c1\uc73c\ub85c \ucde8\uae09\ub418\uc5c8\ub2e4. \ud558\uc9c0\ub9cc \ubd80\ub974\uc8fc\uc544 \uc790\uc720\uc8fc\uc758\uc790\ub4e4\ub3c4 \uc2e0\ub9dd\uc744 \uc783\uc5c8\uc73c\uba70, \ub300\ubd80\ubd84\uc740 \uc2e4\ub9dd\uc73c\ub85c \uc778\ud574\uc11c \uadf8\ub9ac\uace0 \ucd9c\uc2e0 \uc9c0\uc5ed \uad6d\ubbfc\ub4e4\uc5d0\uac8c \uc801\ub300\uc2dc \ub2f9\ud574 \uc815\uacc4\uc5d0\uc11c \ubb3c\ub7ec\ub0ac\ub2e4. \ubc14\uc11c\ub9cc \uc5ed\uc2dc 1855\ub144 \uc774\ub7ec\ud55c \uc774\uc720\ub85c \uc790\uc0b4\ud558\uace0 \ub9d0\uc558\ub2e4. \ud601\uba85\uc774 \uc2e4\ud328\ud55c \uc774\ud6c4 \ud601\uba85\uc740 \ub3c5\uc77c\uc0ac\uc758 \uc798\ubabb\ub41c \uae38\uc774\uc5c8\uc73c\uba70 \"\uaca9\ud1f4\ub418\uc5b4\uc57c \ud558\ub294 \uac83\"\ub294 \uc778\uc2dd\uc774 \uc9c0\ubc30\uc801\uc774\uc5c8\ub2e4. \uad6d\ubbfc\uc758\ud68c\uc758 \uc5c5\uc801\uc5d0 \ub300\ud55c \uae0d\uc815\uc801\uc778 \ud3c9\uac00\ub294 \uc2e4\uc9c8\uc801\uc73c\ub85c \ubc14\uc774\ub9c8\ub974 \uacf5\ud654\uad6d \ub54c\uc5d0\ub098 \uc774\ub8e8\uc5b4\uc84c\uc73c\uba70, \ud2b9\ud788 \uc81c2\ucc28 \uc138\uacc4 \ub300\uc804 \ud6c4 \ub3d9\ub3c5\uacfc \uc11c\ub3c5\uc5d0\uc11c \ud504\ub791\ud06c\ud478\ub974\ud2b8 \uad6d\ubbfc\uc758\ud68c\uc758 \ubbfc\uc8fc\uc801 \uc720\uc0b0\uc744 \uc11c\ub85c \uc790\uad6d \ud2b9\uc720\uc758 \uc804\ud1b5\uc73c\ub85c \ubcf4\uc5ec\uc8fc\ub824\uace0 \uc0c1\ud638 \uacbd\uc7c1\ud558\uba74\uc11c \uc778\uc815 \ubc1b\uac8c \ub418\uc5c8\ub2e4.", "answer": "\ubbfc\uc8fc\uc801 \uc720\uc0b0", "sentence": "\ud2b9\ud788 \uc81c2\ucc28 \uc138\uacc4 \ub300\uc804 \ud6c4 \ub3d9\ub3c5\uacfc \uc11c\ub3c5\uc5d0\uc11c \ud504\ub791\ud06c\ud478\ub974\ud2b8 \uad6d\ubbfc\uc758\ud68c\uc758 \ubbfc\uc8fc\uc801 \uc720\uc0b0 \uc744 \uc11c\ub85c \uc790\uad6d \ud2b9\uc720\uc758 \uc804\ud1b5\uc73c\ub85c \ubcf4\uc5ec\uc8fc\ub824\uace0 \uc0c1\ud638 \uacbd\uc7c1\ud558\uba74\uc11c \uc778\uc815 \ubc1b\uac8c \ub418\uc5c8\ub2e4.", "paragraph_sentence": "\uad6d\ubbfc\uc758\ud68c\uc758 \uc5c5\uc801\uacfc 3\uc6d4 \ud601\uba85 \uc804\uccb4\ub294 \uc774\ud6c4 \uc218\uc2ed \ub144\uac04 \ub9e4\uc6b0 \ubd80\uc815\uc801\uc73c\ub85c \ud3c9\uac00\ub418\uc5c8\ub2e4. \ub8e8\ud2b8\ube44\ud788 \ud638\uc774\uc11c(Ludwig H\u00e4usser)\uc640 \uac19\uc740 \uae09\uc9c4\ubbfc\uc8fc \uc88c\ud30c\uc758 \uc0dd\uac01\uc740, \ubb34\ucc45\uc784\ud558\uace0 \uc21c\uc9c4\ud55c \ub9dd\uc0c1\uc73c\ub85c \ucde8\uae09\ub418\uc5c8\ub2e4. \ud558\uc9c0\ub9cc \ubd80\ub974\uc8fc\uc544 \uc790\uc720\uc8fc\uc758\uc790\ub4e4\ub3c4 \uc2e0\ub9dd\uc744 \uc783\uc5c8\uc73c\uba70, \ub300\ubd80\ubd84\uc740 \uc2e4\ub9dd\uc73c\ub85c \uc778\ud574\uc11c \uadf8\ub9ac\uace0 \ucd9c\uc2e0 \uc9c0\uc5ed \uad6d\ubbfc\ub4e4\uc5d0\uac8c \uc801\ub300\uc2dc \ub2f9\ud574 \uc815\uacc4\uc5d0\uc11c \ubb3c\ub7ec\ub0ac\ub2e4. \ubc14\uc11c\ub9cc \uc5ed\uc2dc 1855\ub144 \uc774\ub7ec\ud55c \uc774\uc720\ub85c \uc790\uc0b4\ud558\uace0 \ub9d0\uc558\ub2e4. \ud601\uba85\uc774 \uc2e4\ud328\ud55c \uc774\ud6c4 \ud601\uba85\uc740 \ub3c5\uc77c\uc0ac\uc758 \uc798\ubabb\ub41c \uae38\uc774\uc5c8\uc73c\uba70 \"\uaca9\ud1f4\ub418\uc5b4\uc57c \ud558\ub294 \uac83\"\ub294 \uc778\uc2dd\uc774 \uc9c0\ubc30\uc801\uc774\uc5c8\ub2e4. \uad6d\ubbfc\uc758\ud68c\uc758 \uc5c5\uc801\uc5d0 \ub300\ud55c \uae0d\uc815\uc801\uc778 \ud3c9\uac00\ub294 \uc2e4\uc9c8\uc801\uc73c\ub85c \ubc14\uc774\ub9c8\ub974 \uacf5\ud654\uad6d \ub54c\uc5d0\ub098 \uc774\ub8e8\uc5b4\uc84c\uc73c\uba70, \ud2b9\ud788 \uc81c2\ucc28 \uc138\uacc4 \ub300\uc804 \ud6c4 \ub3d9\ub3c5\uacfc \uc11c\ub3c5\uc5d0\uc11c \ud504\ub791\ud06c\ud478\ub974\ud2b8 \uad6d\ubbfc\uc758\ud68c\uc758 \ubbfc\uc8fc\uc801 \uc720\uc0b0 \uc744 \uc11c\ub85c \uc790\uad6d \ud2b9\uc720\uc758 \uc804\ud1b5\uc73c\ub85c \ubcf4\uc5ec\uc8fc\ub824\uace0 \uc0c1\ud638 \uacbd\uc7c1\ud558\uba74\uc11c \uc778\uc815 \ubc1b\uac8c \ub418\uc5c8\ub2e4. ", "paragraph_answer": "\uad6d\ubbfc\uc758\ud68c\uc758 \uc5c5\uc801\uacfc 3\uc6d4 \ud601\uba85 \uc804\uccb4\ub294 \uc774\ud6c4 \uc218\uc2ed \ub144\uac04 \ub9e4\uc6b0 \ubd80\uc815\uc801\uc73c\ub85c \ud3c9\uac00\ub418\uc5c8\ub2e4. \ub8e8\ud2b8\ube44\ud788 \ud638\uc774\uc11c(Ludwig H\u00e4usser)\uc640 \uac19\uc740 \uae09\uc9c4\ubbfc\uc8fc \uc88c\ud30c\uc758 \uc0dd\uac01\uc740, \ubb34\ucc45\uc784\ud558\uace0 \uc21c\uc9c4\ud55c \ub9dd\uc0c1\uc73c\ub85c \ucde8\uae09\ub418\uc5c8\ub2e4. \ud558\uc9c0\ub9cc \ubd80\ub974\uc8fc\uc544 \uc790\uc720\uc8fc\uc758\uc790\ub4e4\ub3c4 \uc2e0\ub9dd\uc744 \uc783\uc5c8\uc73c\uba70, \ub300\ubd80\ubd84\uc740 \uc2e4\ub9dd\uc73c\ub85c \uc778\ud574\uc11c \uadf8\ub9ac\uace0 \ucd9c\uc2e0 \uc9c0\uc5ed \uad6d\ubbfc\ub4e4\uc5d0\uac8c \uc801\ub300\uc2dc \ub2f9\ud574 \uc815\uacc4\uc5d0\uc11c \ubb3c\ub7ec\ub0ac\ub2e4. \ubc14\uc11c\ub9cc \uc5ed\uc2dc 1855\ub144 \uc774\ub7ec\ud55c \uc774\uc720\ub85c \uc790\uc0b4\ud558\uace0 \ub9d0\uc558\ub2e4. \ud601\uba85\uc774 \uc2e4\ud328\ud55c \uc774\ud6c4 \ud601\uba85\uc740 \ub3c5\uc77c\uc0ac\uc758 \uc798\ubabb\ub41c \uae38\uc774\uc5c8\uc73c\uba70 \"\uaca9\ud1f4\ub418\uc5b4\uc57c \ud558\ub294 \uac83\"\ub294 \uc778\uc2dd\uc774 \uc9c0\ubc30\uc801\uc774\uc5c8\ub2e4. \uad6d\ubbfc\uc758\ud68c\uc758 \uc5c5\uc801\uc5d0 \ub300\ud55c \uae0d\uc815\uc801\uc778 \ud3c9\uac00\ub294 \uc2e4\uc9c8\uc801\uc73c\ub85c \ubc14\uc774\ub9c8\ub974 \uacf5\ud654\uad6d \ub54c\uc5d0\ub098 \uc774\ub8e8\uc5b4\uc84c\uc73c\uba70, \ud2b9\ud788 \uc81c2\ucc28 \uc138\uacc4 \ub300\uc804 \ud6c4 \ub3d9\ub3c5\uacfc \uc11c\ub3c5\uc5d0\uc11c \ud504\ub791\ud06c\ud478\ub974\ud2b8 \uad6d\ubbfc\uc758\ud68c\uc758 \ubbfc\uc8fc\uc801 \uc720\uc0b0 \uc744 \uc11c\ub85c \uc790\uad6d \ud2b9\uc720\uc758 \uc804\ud1b5\uc73c\ub85c \ubcf4\uc5ec\uc8fc\ub824\uace0 \uc0c1\ud638 \uacbd\uc7c1\ud558\uba74\uc11c \uc778\uc815 \ubc1b\uac8c \ub418\uc5c8\ub2e4.", "sentence_answer": "\ud2b9\ud788 \uc81c2\ucc28 \uc138\uacc4 \ub300\uc804 \ud6c4 \ub3d9\ub3c5\uacfc \uc11c\ub3c5\uc5d0\uc11c \ud504\ub791\ud06c\ud478\ub974\ud2b8 \uad6d\ubbfc\uc758\ud68c\uc758 \ubbfc\uc8fc\uc801 \uc720\uc0b0 \uc744 \uc11c\ub85c \uc790\uad6d \ud2b9\uc720\uc758 \uc804\ud1b5\uc73c\ub85c \ubcf4\uc5ec\uc8fc\ub824\uace0 \uc0c1\ud638 \uacbd\uc7c1\ud558\uba74\uc11c \uc778\uc815 \ubc1b\uac8c \ub418\uc5c8\ub2e4.", "paragraph_id": "6465718-32-2", "paragraph_question": "question: \ub3d9\ub3c5\uacfc \uc11c\ub3c5\uc740 \ud504\ub791\ud06c\ud478\ub974\ud2b8 \uad6d\ubbfc\uc758\ud68c\uc758 \uc5b4\ub5a0\ud55c \uac83\uc744 \uc790\uad6d \ud2b9\uc720\uc758 \uc804\ud1b5\uc73c\ub85c \ubcf4\uc5ec\uc8fc\uace0\uc790 \ud558\uc600\ub294\uac00?, context: \uad6d\ubbfc\uc758\ud68c\uc758 \uc5c5\uc801\uacfc 3\uc6d4 \ud601\uba85 \uc804\uccb4\ub294 \uc774\ud6c4 \uc218\uc2ed \ub144\uac04 \ub9e4\uc6b0 \ubd80\uc815\uc801\uc73c\ub85c \ud3c9\uac00\ub418\uc5c8\ub2e4. \ub8e8\ud2b8\ube44\ud788 \ud638\uc774\uc11c(Ludwig H\u00e4usser)\uc640 \uac19\uc740 \uae09\uc9c4\ubbfc\uc8fc \uc88c\ud30c\uc758 \uc0dd\uac01\uc740, \ubb34\ucc45\uc784\ud558\uace0 \uc21c\uc9c4\ud55c \ub9dd\uc0c1\uc73c\ub85c \ucde8\uae09\ub418\uc5c8\ub2e4. \ud558\uc9c0\ub9cc \ubd80\ub974\uc8fc\uc544 \uc790\uc720\uc8fc\uc758\uc790\ub4e4\ub3c4 \uc2e0\ub9dd\uc744 \uc783\uc5c8\uc73c\uba70, \ub300\ubd80\ubd84\uc740 \uc2e4\ub9dd\uc73c\ub85c \uc778\ud574\uc11c \uadf8\ub9ac\uace0 \ucd9c\uc2e0 \uc9c0\uc5ed \uad6d\ubbfc\ub4e4\uc5d0\uac8c \uc801\ub300\uc2dc \ub2f9\ud574 \uc815\uacc4\uc5d0\uc11c \ubb3c\ub7ec\ub0ac\ub2e4. \ubc14\uc11c\ub9cc \uc5ed\uc2dc 1855\ub144 \uc774\ub7ec\ud55c \uc774\uc720\ub85c \uc790\uc0b4\ud558\uace0 \ub9d0\uc558\ub2e4. \ud601\uba85\uc774 \uc2e4\ud328\ud55c \uc774\ud6c4 \ud601\uba85\uc740 \ub3c5\uc77c\uc0ac\uc758 \uc798\ubabb\ub41c \uae38\uc774\uc5c8\uc73c\uba70 \"\uaca9\ud1f4\ub418\uc5b4\uc57c \ud558\ub294 \uac83\"\ub294 \uc778\uc2dd\uc774 \uc9c0\ubc30\uc801\uc774\uc5c8\ub2e4. \uad6d\ubbfc\uc758\ud68c\uc758 \uc5c5\uc801\uc5d0 \ub300\ud55c \uae0d\uc815\uc801\uc778 \ud3c9\uac00\ub294 \uc2e4\uc9c8\uc801\uc73c\ub85c \ubc14\uc774\ub9c8\ub974 \uacf5\ud654\uad6d \ub54c\uc5d0\ub098 \uc774\ub8e8\uc5b4\uc84c\uc73c\uba70, \ud2b9\ud788 \uc81c2\ucc28 \uc138\uacc4 \ub300\uc804 \ud6c4 \ub3d9\ub3c5\uacfc \uc11c\ub3c5\uc5d0\uc11c \ud504\ub791\ud06c\ud478\ub974\ud2b8 \uad6d\ubbfc\uc758\ud68c\uc758 \ubbfc\uc8fc\uc801 \uc720\uc0b0\uc744 \uc11c\ub85c \uc790\uad6d \ud2b9\uc720\uc758 \uc804\ud1b5\uc73c\ub85c \ubcf4\uc5ec\uc8fc\ub824\uace0 \uc0c1\ud638 \uacbd\uc7c1\ud558\uba74\uc11c \uc778\uc815 \ubc1b\uac8c \ub418\uc5c8\ub2e4."} {"question": "\ub124\ud22c\uc6cc\ud06c 2011 \uacc4\ud68d\uc548\uc774 \ub098\uc628 \uac83\uc740 \uc5b8\uc81c\uc778\uac00?", "paragraph": "\ub2e4\uc6b4\ud0c0\uc6b4 \ub9b4\ub9ac\ud504 \uc120 (Downtown Relief Line, DRL)\uc740 \uc601-\uc720\ub2c8\ubc84\uc2dc\ud2f0 \uc120 \ub3c4\uc2ec \uad6c\uac04\uacfc \ube14\ub8e8\uc5b4-\ub304\ud3ec\uc2a4 \uc120\uc744 \uc787\ub294 \uc601 \uc9c0\ud558\ucca0\uc758 \ub118\uce58\ub294 \uc2b9\uac1d\uc744 \ubd84\uc0b0\ud558\uac8c \ub420 \uc77c\uc885\uc758 \ud63c\uc7a1 \uc644\ud654 (relief) \ub178\uc120\uc774\ub2e4. \uc774 \ub178\uc120\uc740 1985\ub144 \ub124\ud2b8\uc6cc\ud06c 2011 \uacc4\ud68d\uc548\uc73c\ub85c \uc2a4\ud30c\ub2e4\uc774\ub098 \uc560\ube44\ub274\uc5d0\uc11c \ub85c\uc800\uc2a4 \uc13c\ud130, \uc720\ub2c8\uc5b8 \uc5ed\uc744 \uac70\uccd0 \ud398\uc774\ud504 \uc5ed\uc744 \uc774\uc744 \uacc4\ud68d\uc774\uc600\ub2e4. 1985\ub144 \ub2f9\uc2dc, \uc601 \uc9c0\ud558\ucca0\uacfc \ube14\ub8e8\uc5b4-\ub304\ud3ec\uc2a4 \uc120\uc758 \ud658\uc2b9\uc5ed\uc778 \ube14\ub8e8\uc5b4-\uc601 \uc5ed\uc740 \uc124\uacc4 \ub2f9\uc2dc\uc758 \uc218\uc6a9\ub7c9\uc744 \ud55c\ucc38 \ucd08\uacfc\ud558\uc600\uc73c\uba70 \uc774 \uacc4\ud68d\uc5d0 \ub530\ub974\uba74 \uc170\ud37c\ub4dc \uc120\uc744 \uc601 \uc2a4\ud2b8\ub9ac\ud2b8\uc5d0\uc11c \ube45\ud1a0\ub9ac\uc544 \ud30c\ud06c \uc560\ube44\ub274\uae4c\uc9c0 \uba3c\uc800 \uc9d3\uace0 \uadf8 \ub2e4\uc74c\uc5d0 \uacf5\uc0ac\uac00 \uc9c4\ud589\ub420 \uc608\uc815\uc774\uc5c8\ub2e4. \uc774\ud6c4 1990\ub144\ub300 \ucd08\ubc18, \uc2b9\ud558\ucc28\ub7c9\uc774 \uc5b4\ub290\uc815\ub3c4 \uac10\uc18c\ud558\uace0 \ube14\ub8e8\uc5b4-\uc601 \uc5ed\uc758 \ud63c\uc7a1\ub3c4\uac00 \ub0ae\uc544\uc9c0\uba74\uc11c \uc774 \uacc4\ud68d\uc740 \uc804\uba74 \ubcf4\ub958\ub418\uc5c8\ub2e4.", "answer": "1985\ub144", "sentence": "\uc774 \ub178\uc120\uc740 1985\ub144 \ub124\ud2b8\uc6cc\ud06c 2011 \uacc4\ud68d\uc548\uc73c\ub85c \uc2a4\ud30c\ub2e4\uc774\ub098 \uc560\ube44\ub274\uc5d0\uc11c \ub85c\uc800\uc2a4 \uc13c\ud130, \uc720\ub2c8\uc5b8 \uc5ed\uc744 \uac70\uccd0 \ud398\uc774\ud504 \uc5ed\uc744 \uc774\uc744 \uacc4\ud68d\uc774\uc600\ub2e4.", "paragraph_sentence": "\ub2e4\uc6b4\ud0c0\uc6b4 \ub9b4\ub9ac\ud504 \uc120 (Downtown Relief Line, DRL)\uc740 \uc601-\uc720\ub2c8\ubc84\uc2dc\ud2f0 \uc120 \ub3c4\uc2ec \uad6c\uac04\uacfc \ube14\ub8e8\uc5b4-\ub304\ud3ec\uc2a4 \uc120\uc744 \uc787\ub294 \uc601 \uc9c0\ud558\ucca0\uc758 \ub118\uce58\ub294 \uc2b9\uac1d\uc744 \ubd84\uc0b0\ud558\uac8c \ub420 \uc77c\uc885\uc758 \ud63c\uc7a1 \uc644\ud654 (relief) \ub178\uc120\uc774\ub2e4. \uc774 \ub178\uc120\uc740 1985\ub144 \ub124\ud2b8\uc6cc\ud06c 2011 \uacc4\ud68d\uc548\uc73c\ub85c \uc2a4\ud30c\ub2e4\uc774\ub098 \uc560\ube44\ub274\uc5d0\uc11c \ub85c\uc800\uc2a4 \uc13c\ud130, \uc720\ub2c8\uc5b8 \uc5ed\uc744 \uac70\uccd0 \ud398\uc774\ud504 \uc5ed\uc744 \uc774\uc744 \uacc4\ud68d\uc774\uc600\ub2e4. 1985\ub144 \ub2f9\uc2dc, \uc601 \uc9c0\ud558\ucca0\uacfc \ube14\ub8e8\uc5b4-\ub304\ud3ec\uc2a4 \uc120\uc758 \ud658\uc2b9\uc5ed\uc778 \ube14\ub8e8\uc5b4-\uc601 \uc5ed\uc740 \uc124\uacc4 \ub2f9\uc2dc\uc758 \uc218\uc6a9\ub7c9\uc744 \ud55c\ucc38 \ucd08\uacfc\ud558\uc600\uc73c\uba70 \uc774 \uacc4\ud68d\uc5d0 \ub530\ub974\uba74 \uc170\ud37c\ub4dc \uc120\uc744 \uc601 \uc2a4\ud2b8\ub9ac\ud2b8\uc5d0\uc11c \ube45\ud1a0\ub9ac\uc544 \ud30c\ud06c \uc560\ube44\ub274\uae4c\uc9c0 \uba3c\uc800 \uc9d3\uace0 \uadf8 \ub2e4\uc74c\uc5d0 \uacf5\uc0ac\uac00 \uc9c4\ud589\ub420 \uc608\uc815\uc774\uc5c8\ub2e4. \uc774\ud6c4 1990\ub144\ub300 \ucd08\ubc18, \uc2b9\ud558\ucc28\ub7c9\uc774 \uc5b4\ub290\uc815\ub3c4 \uac10\uc18c\ud558\uace0 \ube14\ub8e8\uc5b4-\uc601 \uc5ed\uc758 \ud63c\uc7a1\ub3c4\uac00 \ub0ae\uc544\uc9c0\uba74\uc11c \uc774 \uacc4\ud68d\uc740 \uc804\uba74 \ubcf4\ub958\ub418\uc5c8\ub2e4.", "paragraph_answer": "\ub2e4\uc6b4\ud0c0\uc6b4 \ub9b4\ub9ac\ud504 \uc120 (Downtown Relief Line, DRL)\uc740 \uc601-\uc720\ub2c8\ubc84\uc2dc\ud2f0 \uc120 \ub3c4\uc2ec \uad6c\uac04\uacfc \ube14\ub8e8\uc5b4-\ub304\ud3ec\uc2a4 \uc120\uc744 \uc787\ub294 \uc601 \uc9c0\ud558\ucca0\uc758 \ub118\uce58\ub294 \uc2b9\uac1d\uc744 \ubd84\uc0b0\ud558\uac8c \ub420 \uc77c\uc885\uc758 \ud63c\uc7a1 \uc644\ud654 (relief) \ub178\uc120\uc774\ub2e4. \uc774 \ub178\uc120\uc740 1985\ub144 \ub124\ud2b8\uc6cc\ud06c 2011 \uacc4\ud68d\uc548\uc73c\ub85c \uc2a4\ud30c\ub2e4\uc774\ub098 \uc560\ube44\ub274\uc5d0\uc11c \ub85c\uc800\uc2a4 \uc13c\ud130, \uc720\ub2c8\uc5b8 \uc5ed\uc744 \uac70\uccd0 \ud398\uc774\ud504 \uc5ed\uc744 \uc774\uc744 \uacc4\ud68d\uc774\uc600\ub2e4. 1985\ub144 \ub2f9\uc2dc, \uc601 \uc9c0\ud558\ucca0\uacfc \ube14\ub8e8\uc5b4-\ub304\ud3ec\uc2a4 \uc120\uc758 \ud658\uc2b9\uc5ed\uc778 \ube14\ub8e8\uc5b4-\uc601 \uc5ed\uc740 \uc124\uacc4 \ub2f9\uc2dc\uc758 \uc218\uc6a9\ub7c9\uc744 \ud55c\ucc38 \ucd08\uacfc\ud558\uc600\uc73c\uba70 \uc774 \uacc4\ud68d\uc5d0 \ub530\ub974\uba74 \uc170\ud37c\ub4dc \uc120\uc744 \uc601 \uc2a4\ud2b8\ub9ac\ud2b8\uc5d0\uc11c \ube45\ud1a0\ub9ac\uc544 \ud30c\ud06c \uc560\ube44\ub274\uae4c\uc9c0 \uba3c\uc800 \uc9d3\uace0 \uadf8 \ub2e4\uc74c\uc5d0 \uacf5\uc0ac\uac00 \uc9c4\ud589\ub420 \uc608\uc815\uc774\uc5c8\ub2e4. \uc774\ud6c4 1990\ub144\ub300 \ucd08\ubc18, \uc2b9\ud558\ucc28\ub7c9\uc774 \uc5b4\ub290\uc815\ub3c4 \uac10\uc18c\ud558\uace0 \ube14\ub8e8\uc5b4-\uc601 \uc5ed\uc758 \ud63c\uc7a1\ub3c4\uac00 \ub0ae\uc544\uc9c0\uba74\uc11c \uc774 \uacc4\ud68d\uc740 \uc804\uba74 \ubcf4\ub958\ub418\uc5c8\ub2e4.", "sentence_answer": "\uc774 \ub178\uc120\uc740 1985\ub144 \ub124\ud2b8\uc6cc\ud06c 2011 \uacc4\ud68d\uc548\uc73c\ub85c \uc2a4\ud30c\ub2e4\uc774\ub098 \uc560\ube44\ub274\uc5d0\uc11c \ub85c\uc800\uc2a4 \uc13c\ud130, \uc720\ub2c8\uc5b8 \uc5ed\uc744 \uac70\uccd0 \ud398\uc774\ud504 \uc5ed\uc744 \uc774\uc744 \uacc4\ud68d\uc774\uc600\ub2e4.", "paragraph_id": "6566081-17-2", "paragraph_question": "question: \ub124\ud22c\uc6cc\ud06c 2011 \uacc4\ud68d\uc548\uc774 \ub098\uc628 \uac83\uc740 \uc5b8\uc81c\uc778\uac00?, context: \ub2e4\uc6b4\ud0c0\uc6b4 \ub9b4\ub9ac\ud504 \uc120 (Downtown Relief Line, DRL)\uc740 \uc601-\uc720\ub2c8\ubc84\uc2dc\ud2f0 \uc120 \ub3c4\uc2ec \uad6c\uac04\uacfc \ube14\ub8e8\uc5b4-\ub304\ud3ec\uc2a4 \uc120\uc744 \uc787\ub294 \uc601 \uc9c0\ud558\ucca0\uc758 \ub118\uce58\ub294 \uc2b9\uac1d\uc744 \ubd84\uc0b0\ud558\uac8c \ub420 \uc77c\uc885\uc758 \ud63c\uc7a1 \uc644\ud654 (relief) \ub178\uc120\uc774\ub2e4. \uc774 \ub178\uc120\uc740 1985\ub144 \ub124\ud2b8\uc6cc\ud06c 2011 \uacc4\ud68d\uc548\uc73c\ub85c \uc2a4\ud30c\ub2e4\uc774\ub098 \uc560\ube44\ub274\uc5d0\uc11c \ub85c\uc800\uc2a4 \uc13c\ud130, \uc720\ub2c8\uc5b8 \uc5ed\uc744 \uac70\uccd0 \ud398\uc774\ud504 \uc5ed\uc744 \uc774\uc744 \uacc4\ud68d\uc774\uc600\ub2e4. 1985\ub144 \ub2f9\uc2dc, \uc601 \uc9c0\ud558\ucca0\uacfc \ube14\ub8e8\uc5b4-\ub304\ud3ec\uc2a4 \uc120\uc758 \ud658\uc2b9\uc5ed\uc778 \ube14\ub8e8\uc5b4-\uc601 \uc5ed\uc740 \uc124\uacc4 \ub2f9\uc2dc\uc758 \uc218\uc6a9\ub7c9\uc744 \ud55c\ucc38 \ucd08\uacfc\ud558\uc600\uc73c\uba70 \uc774 \uacc4\ud68d\uc5d0 \ub530\ub974\uba74 \uc170\ud37c\ub4dc \uc120\uc744 \uc601 \uc2a4\ud2b8\ub9ac\ud2b8\uc5d0\uc11c \ube45\ud1a0\ub9ac\uc544 \ud30c\ud06c \uc560\ube44\ub274\uae4c\uc9c0 \uba3c\uc800 \uc9d3\uace0 \uadf8 \ub2e4\uc74c\uc5d0 \uacf5\uc0ac\uac00 \uc9c4\ud589\ub420 \uc608\uc815\uc774\uc5c8\ub2e4. \uc774\ud6c4 1990\ub144\ub300 \ucd08\ubc18, \uc2b9\ud558\ucc28\ub7c9\uc774 \uc5b4\ub290\uc815\ub3c4 \uac10\uc18c\ud558\uace0 \ube14\ub8e8\uc5b4-\uc601 \uc5ed\uc758 \ud63c\uc7a1\ub3c4\uac00 \ub0ae\uc544\uc9c0\uba74\uc11c \uc774 \uacc4\ud68d\uc740 \uc804\uba74 \ubcf4\ub958\ub418\uc5c8\ub2e4."} {"question": "\uc624\ub85c\ub77c\uc624\ubc8c\uc740 \ucee4\ud2bc\ud615\uc624\ub85c\ub77c\uc640 \ubb34\uc5c7\uc73c\ub85c \ub098\ub258\ub294\uac00?", "paragraph": "\uc624\ub85c\ub77c\uac00 \uac00\uc7a5 \uc790\uc8fc \ubcf4\uc774\ub294 \uacf3\uc740 \ub0a8\ubd81 \uc591\uadf9\uc9c0\ubc29\uc758 \uc9c0\uad6c\uc790\uae30\uc704\ub3c4 65\u223c70\ub3c4\uc758 \ubc94\uc704\ub85c\uc11c \uc774 \uc9c0\uc5ed\uc744 \uc624\ub85c\ub77c\ub300(auroral zone)\ub77c\uace0 \ud55c\ub2e4. \uc624\ub85c\ub77c\ub300\ubcf4\ub2e4 \uace0\uc704\ub3c4(\uadf9\uad00\uc9c0\uc5ed)\ub098 \uc800\uc704\ub3c4\uc5d0\uc11c\uc758 \ucd9c\ud604\ube48\ub3c4\ub294 \uac10\uc18c\ud55c\ub2e4. \ucd9c\ud604\ud558\ub294 \uc704\ub3c4\ub294 \uc9c0\ubc29\uc2dc(\u5730\u65b9\u6642)\uc5d0 \ub530\ub77c \ub2e4\ub974\uba70, \uc57c\uac04\uc5d0\ub294 65\u223c70\ub3c4\uc5d0 \ub9ce\uc73c\uba70, \uc8fc\uac04\uc5d0\ub294 75\u223c80\ub3c4\ub85c \uc704\ub3c4\uac00 \ub192\uc544\uc9c4\ub2e4. \uc774\ub807\uac8c \uc624\ub85c\ub77c\uac00 \ucd9c\ud604\ud558\ub294 \uc704\ub3c4\uac00 \uc9c0\ubc29\uc2dc\uc5d0 \ub530\ub77c \ubcc0\ud654\ub97c \ubcf4\uc774\uae30 \ub54c\ubb38\uc5d0 \uc624\ub85c\ub77c\ucd9c\ud604\ub300(\uc9c0\uad6c\ub97c \uadf9\uc9c0\uc758 \uc0c1\uacf5\uc5d0\uc11c \ub0b4\ub824\ub2e4 \ubcf4\uc558\uc744 \ub54c \ub3d9\uc2dc\uc5d0 \uc624\ub85c\ub77c\uac00 \ubcf4\uc774\ub294 \uc601\uc5ed)\ub97c \uc624\ub85c\ub77c\ub300\uc640 \uad6c\ubcc4\ud574\uc11c \uc624\ub85c\ub77c \uc624\ubc8c(aurora oval)\uc774\ub77c\uace0 \ud55c\ub2e4. \uc77c\ub828\uc758 \uc624\ub85c\ub77c\uc624\ubc8c\uc740 \ub300\uac1c 2\uc885\ub958\uc758 \uc624\ub85c\ub77c\ub85c \uad6c\uc131\ub418\ub294\ub370, \ub0ae\uc5d0\uc11c \uc800\ub141\uc744 \uac70\uccd0 \uc2ec\uc57c\uc5d0 \uc774\ub974\ub294 \uc2dc\uac04\uc5d0\ub294 \ucee4\ud2bc\ud615\uc624\ub85c\ub77c\uc774\uace0, \uadf8 \uc774\ud6c4 \uc544\uce68\uae4c\uc9c0\uc758 \ubc18(\u534a)\uc740 \uc8fc\ub85c \ub9e5\ub3d9\uc131\uc624\ub85c\ub77c\ub85c\uc11c \uc77c\ubc18\uc801\uc73c\ub85c \uc5f7\uc740 \ubc30\uacbd\uc73c\ub85c \ub3d9\ubc18\ud55c\ub2e4. \uacfc\uac70\uc5d0\ub294 \uad00\uce21\uc7a5\uce58\uc758 \uac10\ub3c4\ubd80\uc871\uc73c\ub85c \uc778\ud574\uc11c \ub9e5\ub3d9\uc131\uc624\ub85c\ub77c\ub97c \ucda9\ubd84\ud788 \uad00\uce21\ud560 \uc218 \uc5c6\uae30 \ub54c\ubb38\uc5d0 \uc774 \ubd80\ubd84\uc744 \ud76c\ubbf8\ud55c \ubd80\uc815\ud615\uc624\ub85c\ub77c\ub77c\uace0 \ud588\uc5c8\ub2e4.", "answer": "\ub9e5\ub3d9\uc131\uc624\ub85c\ub77c", "sentence": "\uadf8 \uc774\ud6c4 \uc544\uce68\uae4c\uc9c0\uc758 \ubc18(\u534a)\uc740 \uc8fc\ub85c \ub9e5\ub3d9\uc131\uc624\ub85c\ub77c \ub85c\uc11c \uc77c\ubc18\uc801\uc73c\ub85c \uc5f7\uc740 \ubc30\uacbd\uc73c\ub85c \ub3d9\ubc18\ud55c\ub2e4.", "paragraph_sentence": "\uc624\ub85c\ub77c\uac00 \uac00\uc7a5 \uc790\uc8fc \ubcf4\uc774\ub294 \uacf3\uc740 \ub0a8\ubd81 \uc591\uadf9\uc9c0\ubc29\uc758 \uc9c0\uad6c\uc790\uae30\uc704\ub3c4 65\u223c70\ub3c4\uc758 \ubc94\uc704\ub85c\uc11c \uc774 \uc9c0\uc5ed\uc744 \uc624\ub85c\ub77c\ub300(auroral zone)\ub77c\uace0 \ud55c\ub2e4. \uc624\ub85c\ub77c\ub300\ubcf4\ub2e4 \uace0\uc704\ub3c4(\uadf9\uad00\uc9c0\uc5ed)\ub098 \uc800\uc704\ub3c4\uc5d0\uc11c\uc758 \ucd9c\ud604\ube48\ub3c4\ub294 \uac10\uc18c\ud55c\ub2e4. \ucd9c\ud604\ud558\ub294 \uc704\ub3c4\ub294 \uc9c0\ubc29\uc2dc(\u5730\u65b9\u6642)\uc5d0 \ub530\ub77c \ub2e4\ub974\uba70, \uc57c\uac04\uc5d0\ub294 65\u223c70\ub3c4\uc5d0 \ub9ce\uc73c\uba70, \uc8fc\uac04\uc5d0\ub294 75\u223c80\ub3c4\ub85c \uc704\ub3c4\uac00 \ub192\uc544\uc9c4\ub2e4. \uc774\ub807\uac8c \uc624\ub85c\ub77c\uac00 \ucd9c\ud604\ud558\ub294 \uc704\ub3c4\uac00 \uc9c0\ubc29\uc2dc\uc5d0 \ub530\ub77c \ubcc0\ud654\ub97c \ubcf4\uc774\uae30 \ub54c\ubb38\uc5d0 \uc624\ub85c\ub77c\ucd9c\ud604\ub300(\uc9c0\uad6c\ub97c \uadf9\uc9c0\uc758 \uc0c1\uacf5\uc5d0\uc11c \ub0b4\ub824\ub2e4 \ubcf4\uc558\uc744 \ub54c \ub3d9\uc2dc\uc5d0 \uc624\ub85c\ub77c\uac00 \ubcf4\uc774\ub294 \uc601\uc5ed)\ub97c \uc624\ub85c\ub77c\ub300\uc640 \uad6c\ubcc4\ud574\uc11c \uc624\ub85c\ub77c \uc624\ubc8c(aurora oval)\uc774\ub77c\uace0 \ud55c\ub2e4. \uc77c\ub828\uc758 \uc624\ub85c\ub77c\uc624\ubc8c\uc740 \ub300\uac1c 2\uc885\ub958\uc758 \uc624\ub85c\ub77c\ub85c \uad6c\uc131\ub418\ub294\ub370, \ub0ae\uc5d0\uc11c \uc800\ub141\uc744 \uac70\uccd0 \uc2ec\uc57c\uc5d0 \uc774\ub974\ub294 \uc2dc\uac04\uc5d0\ub294 \ucee4\ud2bc\ud615\uc624\ub85c\ub77c\uc774\uace0, \uadf8 \uc774\ud6c4 \uc544\uce68\uae4c\uc9c0\uc758 \ubc18(\u534a)\uc740 \uc8fc\ub85c \ub9e5\ub3d9\uc131\uc624\ub85c\ub77c \ub85c\uc11c \uc77c\ubc18\uc801\uc73c\ub85c \uc5f7\uc740 \ubc30\uacbd\uc73c\ub85c \ub3d9\ubc18\ud55c\ub2e4. \uacfc\uac70\uc5d0\ub294 \uad00\uce21\uc7a5\uce58\uc758 \uac10\ub3c4\ubd80\uc871\uc73c\ub85c \uc778\ud574\uc11c \ub9e5\ub3d9\uc131\uc624\ub85c\ub77c\ub97c \ucda9\ubd84\ud788 \uad00\uce21\ud560 \uc218 \uc5c6\uae30 \ub54c\ubb38\uc5d0 \uc774 \ubd80\ubd84\uc744 \ud76c\ubbf8\ud55c \ubd80\uc815\ud615\uc624\ub85c\ub77c\ub77c\uace0 \ud588\uc5c8\ub2e4.", "paragraph_answer": "\uc624\ub85c\ub77c\uac00 \uac00\uc7a5 \uc790\uc8fc \ubcf4\uc774\ub294 \uacf3\uc740 \ub0a8\ubd81 \uc591\uadf9\uc9c0\ubc29\uc758 \uc9c0\uad6c\uc790\uae30\uc704\ub3c4 65\u223c70\ub3c4\uc758 \ubc94\uc704\ub85c\uc11c \uc774 \uc9c0\uc5ed\uc744 \uc624\ub85c\ub77c\ub300(auroral zone)\ub77c\uace0 \ud55c\ub2e4. \uc624\ub85c\ub77c\ub300\ubcf4\ub2e4 \uace0\uc704\ub3c4(\uadf9\uad00\uc9c0\uc5ed)\ub098 \uc800\uc704\ub3c4\uc5d0\uc11c\uc758 \ucd9c\ud604\ube48\ub3c4\ub294 \uac10\uc18c\ud55c\ub2e4. \ucd9c\ud604\ud558\ub294 \uc704\ub3c4\ub294 \uc9c0\ubc29\uc2dc(\u5730\u65b9\u6642)\uc5d0 \ub530\ub77c \ub2e4\ub974\uba70, \uc57c\uac04\uc5d0\ub294 65\u223c70\ub3c4\uc5d0 \ub9ce\uc73c\uba70, \uc8fc\uac04\uc5d0\ub294 75\u223c80\ub3c4\ub85c \uc704\ub3c4\uac00 \ub192\uc544\uc9c4\ub2e4. \uc774\ub807\uac8c \uc624\ub85c\ub77c\uac00 \ucd9c\ud604\ud558\ub294 \uc704\ub3c4\uac00 \uc9c0\ubc29\uc2dc\uc5d0 \ub530\ub77c \ubcc0\ud654\ub97c \ubcf4\uc774\uae30 \ub54c\ubb38\uc5d0 \uc624\ub85c\ub77c\ucd9c\ud604\ub300(\uc9c0\uad6c\ub97c \uadf9\uc9c0\uc758 \uc0c1\uacf5\uc5d0\uc11c \ub0b4\ub824\ub2e4 \ubcf4\uc558\uc744 \ub54c \ub3d9\uc2dc\uc5d0 \uc624\ub85c\ub77c\uac00 \ubcf4\uc774\ub294 \uc601\uc5ed)\ub97c \uc624\ub85c\ub77c\ub300\uc640 \uad6c\ubcc4\ud574\uc11c \uc624\ub85c\ub77c \uc624\ubc8c(aurora oval)\uc774\ub77c\uace0 \ud55c\ub2e4. \uc77c\ub828\uc758 \uc624\ub85c\ub77c\uc624\ubc8c\uc740 \ub300\uac1c 2\uc885\ub958\uc758 \uc624\ub85c\ub77c\ub85c \uad6c\uc131\ub418\ub294\ub370, \ub0ae\uc5d0\uc11c \uc800\ub141\uc744 \uac70\uccd0 \uc2ec\uc57c\uc5d0 \uc774\ub974\ub294 \uc2dc\uac04\uc5d0\ub294 \ucee4\ud2bc\ud615\uc624\ub85c\ub77c\uc774\uace0, \uadf8 \uc774\ud6c4 \uc544\uce68\uae4c\uc9c0\uc758 \ubc18(\u534a)\uc740 \uc8fc\ub85c \ub9e5\ub3d9\uc131\uc624\ub85c\ub77c \ub85c\uc11c \uc77c\ubc18\uc801\uc73c\ub85c \uc5f7\uc740 \ubc30\uacbd\uc73c\ub85c \ub3d9\ubc18\ud55c\ub2e4. \uacfc\uac70\uc5d0\ub294 \uad00\uce21\uc7a5\uce58\uc758 \uac10\ub3c4\ubd80\uc871\uc73c\ub85c \uc778\ud574\uc11c \ub9e5\ub3d9\uc131\uc624\ub85c\ub77c\ub97c \ucda9\ubd84\ud788 \uad00\uce21\ud560 \uc218 \uc5c6\uae30 \ub54c\ubb38\uc5d0 \uc774 \ubd80\ubd84\uc744 \ud76c\ubbf8\ud55c \ubd80\uc815\ud615\uc624\ub85c\ub77c\ub77c\uace0 \ud588\uc5c8\ub2e4.", "sentence_answer": "\uadf8 \uc774\ud6c4 \uc544\uce68\uae4c\uc9c0\uc758 \ubc18(\u534a)\uc740 \uc8fc\ub85c \ub9e5\ub3d9\uc131\uc624\ub85c\ub77c \ub85c\uc11c \uc77c\ubc18\uc801\uc73c\ub85c \uc5f7\uc740 \ubc30\uacbd\uc73c\ub85c \ub3d9\ubc18\ud55c\ub2e4.", "paragraph_id": "6541249-0-0", "paragraph_question": "question: \uc624\ub85c\ub77c\uc624\ubc8c\uc740 \ucee4\ud2bc\ud615\uc624\ub85c\ub77c\uc640 \ubb34\uc5c7\uc73c\ub85c \ub098\ub258\ub294\uac00?, context: \uc624\ub85c\ub77c\uac00 \uac00\uc7a5 \uc790\uc8fc \ubcf4\uc774\ub294 \uacf3\uc740 \ub0a8\ubd81 \uc591\uadf9\uc9c0\ubc29\uc758 \uc9c0\uad6c\uc790\uae30\uc704\ub3c4 65\u223c70\ub3c4\uc758 \ubc94\uc704\ub85c\uc11c \uc774 \uc9c0\uc5ed\uc744 \uc624\ub85c\ub77c\ub300(auroral zone)\ub77c\uace0 \ud55c\ub2e4. \uc624\ub85c\ub77c\ub300\ubcf4\ub2e4 \uace0\uc704\ub3c4(\uadf9\uad00\uc9c0\uc5ed)\ub098 \uc800\uc704\ub3c4\uc5d0\uc11c\uc758 \ucd9c\ud604\ube48\ub3c4\ub294 \uac10\uc18c\ud55c\ub2e4. \ucd9c\ud604\ud558\ub294 \uc704\ub3c4\ub294 \uc9c0\ubc29\uc2dc(\u5730\u65b9\u6642)\uc5d0 \ub530\ub77c \ub2e4\ub974\uba70, \uc57c\uac04\uc5d0\ub294 65\u223c70\ub3c4\uc5d0 \ub9ce\uc73c\uba70, \uc8fc\uac04\uc5d0\ub294 75\u223c80\ub3c4\ub85c \uc704\ub3c4\uac00 \ub192\uc544\uc9c4\ub2e4. \uc774\ub807\uac8c \uc624\ub85c\ub77c\uac00 \ucd9c\ud604\ud558\ub294 \uc704\ub3c4\uac00 \uc9c0\ubc29\uc2dc\uc5d0 \ub530\ub77c \ubcc0\ud654\ub97c \ubcf4\uc774\uae30 \ub54c\ubb38\uc5d0 \uc624\ub85c\ub77c\ucd9c\ud604\ub300(\uc9c0\uad6c\ub97c \uadf9\uc9c0\uc758 \uc0c1\uacf5\uc5d0\uc11c \ub0b4\ub824\ub2e4 \ubcf4\uc558\uc744 \ub54c \ub3d9\uc2dc\uc5d0 \uc624\ub85c\ub77c\uac00 \ubcf4\uc774\ub294 \uc601\uc5ed)\ub97c \uc624\ub85c\ub77c\ub300\uc640 \uad6c\ubcc4\ud574\uc11c \uc624\ub85c\ub77c \uc624\ubc8c(aurora oval)\uc774\ub77c\uace0 \ud55c\ub2e4. \uc77c\ub828\uc758 \uc624\ub85c\ub77c\uc624\ubc8c\uc740 \ub300\uac1c 2\uc885\ub958\uc758 \uc624\ub85c\ub77c\ub85c \uad6c\uc131\ub418\ub294\ub370, \ub0ae\uc5d0\uc11c \uc800\ub141\uc744 \uac70\uccd0 \uc2ec\uc57c\uc5d0 \uc774\ub974\ub294 \uc2dc\uac04\uc5d0\ub294 \ucee4\ud2bc\ud615\uc624\ub85c\ub77c\uc774\uace0, \uadf8 \uc774\ud6c4 \uc544\uce68\uae4c\uc9c0\uc758 \ubc18(\u534a)\uc740 \uc8fc\ub85c \ub9e5\ub3d9\uc131\uc624\ub85c\ub77c\ub85c\uc11c \uc77c\ubc18\uc801\uc73c\ub85c \uc5f7\uc740 \ubc30\uacbd\uc73c\ub85c \ub3d9\ubc18\ud55c\ub2e4. \uacfc\uac70\uc5d0\ub294 \uad00\uce21\uc7a5\uce58\uc758 \uac10\ub3c4\ubd80\uc871\uc73c\ub85c \uc778\ud574\uc11c \ub9e5\ub3d9\uc131\uc624\ub85c\ub77c\ub97c \ucda9\ubd84\ud788 \uad00\uce21\ud560 \uc218 \uc5c6\uae30 \ub54c\ubb38\uc5d0 \uc774 \ubd80\ubd84\uc744 \ud76c\ubbf8\ud55c \ubd80\uc815\ud615\uc624\ub85c\ub77c\ub77c\uace0 \ud588\uc5c8\ub2e4."} {"question": "\uae30\uc6d0\uc804 221\ub144 \ud544\ub85c\ud3ec\uc774\uba58\uc774 \uc6a9\ubcd1 \ub300\uc7a5\uc73c\ub85c \uc2f8\uc6b4 \uacf3\uc740 \uc5b4\ub514\uc778\uac00?", "paragraph": "\uae30\uc6d0\uc804 221\ub144, \ud544\ub85c\ud3ec\uc774\uba58\uc740 \uc804\uc7c1 \uc911\uc778 \ud06c\ub808\ud0c0\uc5d0\uc11c \uc6a9\ubcd1 \ub300\uc7a5\uc73c\ub85c \uc2f8\uc6e0\ub2e4. 10\ub144 \ud6c4 \uadc0\uad6d\uc5d0 \uc988\uc74c\ud558\uc5ec \uadf8\ub294 \uc544\uce74\uc774\uc544 \ub3d9\ub9f9\uc758 \uae30\ubcd1\ub300 \uc9c0\ud718\uad00\uc73c\ub85c \uc784\uba85\ub418\uc5c8\ub2e4. \ub2f9\uc2dc \uc544\uce74\uc774\uc544\uc758 \uae30\ubcd1\ub4e4\uc740 \uc57d\ud588\uae30 \ub54c\ubb38\uc5d0 \uadf8\ub294 \uae30\ubcd1\uc744 \uc6a9\ub9f9\ud558\uac8c \ub2e8\ub828\ud588\ub2e4. \uac19\uc740 \ud574\uc5d0 \uc81c1\ucc28 \ub9c8\ucf00\ub3c4\ub2c8\uc544 \uc804\uc7c1\uacfc \uc5f0\uad00\ub41c \uc804\ud22c\uc5d0\uc11c \ud544\ub85c\ud3ec\uc774\uba58\uc774 \uc774\ub044\ub294 \uc544\uce74\uc774\uc544 \uad70\uc740 \uc5d8\ub9ac\uc2a4\uc758 \uacbd\uacc4\uc5d0 \uc788\ub294 \ub77c\ub9ac\uc0ac \uac15\uc5d0\uc11c \uc544\uc774\ud1a8\ub9ac\uc544, \uc5d8\ub9ac\uc2a4 \uc5f0\ud569\uad70\uacfc \uc2f8\uc6cc \ubb3c\ub9ac\uce58\uace0, \uc801\uc7a5 \ub2e4\ubaa8\ud310\ud22c\uc2a4\uc640 \uc9c1\uc811 \ub9de\uc11c\uc11c \ubb3c\ub9ac\uce58\uace0 \uc2b9\ub9ac\ub97c \uac70\ub450\uc5c8\ub2e4. \uc774 \uc2b9\ub9ac\ub85c \ud544\ub85c\ud3ec\uc774\uba58\uc758 \uba85\uc131\uc740 \uadf8\ub9ac\uc2a4\uc5d0 \ub110\ub9ac \ud37c\uc84c\uc73c\uba70, \ub610\ud55c \uadf8\ub294 \uc544\uce74\uc774\uc544 \ub3d9\ub9f9\uc758 \ubcf4\ubcd1 \uc7a5\ube44\ub97c \uac1c\ud601\ud588\ub2e4. \uadf8\ub54c\uae4c\uc9c0 \uc544\uce74\uc774\uc544 \uad70\uc758 \uc7a5\ube44\ub294 \uc9e7\uc740 \ucc3d\uacfc \uac00\ubccd\uac8c \uc587\uc740 \ud0c0\uc6d0\ud615 \ubc29\ud328\uc600\uc73c\uba70, \uc774\uac83\uc740 \uc6d0\uac70\ub9ac \uc804\ud22c\uc5d0\uc11c\ub294 \uc720\ub9ac\ud558\uc9c0\ub9cc, \ubc31\ubcd1\uc804\uc5d0\uc11c\ub294 \ubd88\ub9ac\ud558\uc5ec \uc27d\uac8c \uc804\uc5f4\uc774 \ub3cc\ud30c\ub418\uace0 \uc788\uc5c8\ub2e4. \uadf8\ub294 \ud314\ub791\ud06c\uc2a4\uc758 \ubc00\uc9d1\ub300\ud615\uc744 \ud615\uc131\ud560 \ud615\ud3b8\uc774 \uc88b\uc740 \uc7a5\ucc3d\uacfc \ub465\uadfc\ubc29\ud328\ub85c \ubc14\uafb8\uc5c8\ub2e4.", "answer": "\ud06c\ub808\ud0c0", "sentence": "\uae30\uc6d0\uc804 221\ub144, \ud544\ub85c\ud3ec\uc774\uba58\uc740 \uc804\uc7c1 \uc911\uc778 \ud06c\ub808\ud0c0 \uc5d0\uc11c \uc6a9\ubcd1 \ub300\uc7a5\uc73c\ub85c \uc2f8\uc6e0\ub2e4.", "paragraph_sentence": " \uae30\uc6d0\uc804 221\ub144, \ud544\ub85c\ud3ec\uc774\uba58\uc740 \uc804\uc7c1 \uc911\uc778 \ud06c\ub808\ud0c0 \uc5d0\uc11c \uc6a9\ubcd1 \ub300\uc7a5\uc73c\ub85c \uc2f8\uc6e0\ub2e4. 10\ub144 \ud6c4 \uadc0\uad6d\uc5d0 \uc988\uc74c\ud558\uc5ec \uadf8\ub294 \uc544\uce74\uc774\uc544 \ub3d9\ub9f9\uc758 \uae30\ubcd1\ub300 \uc9c0\ud718\uad00\uc73c\ub85c \uc784\uba85\ub418\uc5c8\ub2e4. \ub2f9\uc2dc \uc544\uce74\uc774\uc544\uc758 \uae30\ubcd1\ub4e4\uc740 \uc57d\ud588\uae30 \ub54c\ubb38\uc5d0 \uadf8\ub294 \uae30\ubcd1\uc744 \uc6a9\ub9f9\ud558\uac8c \ub2e8\ub828\ud588\ub2e4. \uac19\uc740 \ud574\uc5d0 \uc81c1\ucc28 \ub9c8\ucf00\ub3c4\ub2c8\uc544 \uc804\uc7c1\uacfc \uc5f0\uad00\ub41c \uc804\ud22c\uc5d0\uc11c \ud544\ub85c\ud3ec\uc774\uba58\uc774 \uc774\ub044\ub294 \uc544\uce74\uc774\uc544 \uad70\uc740 \uc5d8\ub9ac\uc2a4\uc758 \uacbd\uacc4\uc5d0 \uc788\ub294 \ub77c\ub9ac\uc0ac \uac15\uc5d0\uc11c \uc544\uc774\ud1a8\ub9ac\uc544, \uc5d8\ub9ac\uc2a4 \uc5f0\ud569\uad70\uacfc \uc2f8\uc6cc \ubb3c\ub9ac\uce58\uace0, \uc801\uc7a5 \ub2e4\ubaa8\ud310\ud22c\uc2a4\uc640 \uc9c1\uc811 \ub9de\uc11c\uc11c \ubb3c\ub9ac\uce58\uace0 \uc2b9\ub9ac\ub97c \uac70\ub450\uc5c8\ub2e4. \uc774 \uc2b9\ub9ac\ub85c \ud544\ub85c\ud3ec\uc774\uba58\uc758 \uba85\uc131\uc740 \uadf8\ub9ac\uc2a4\uc5d0 \ub110\ub9ac \ud37c\uc84c\uc73c\uba70, \ub610\ud55c \uadf8\ub294 \uc544\uce74\uc774\uc544 \ub3d9\ub9f9\uc758 \ubcf4\ubcd1 \uc7a5\ube44\ub97c \uac1c\ud601\ud588\ub2e4. \uadf8\ub54c\uae4c\uc9c0 \uc544\uce74\uc774\uc544 \uad70\uc758 \uc7a5\ube44\ub294 \uc9e7\uc740 \ucc3d\uacfc \uac00\ubccd\uac8c \uc587\uc740 \ud0c0\uc6d0\ud615 \ubc29\ud328\uc600\uc73c\uba70, \uc774\uac83\uc740 \uc6d0\uac70\ub9ac \uc804\ud22c\uc5d0\uc11c\ub294 \uc720\ub9ac\ud558\uc9c0\ub9cc, \ubc31\ubcd1\uc804\uc5d0\uc11c\ub294 \ubd88\ub9ac\ud558\uc5ec \uc27d\uac8c \uc804\uc5f4\uc774 \ub3cc\ud30c\ub418\uace0 \uc788\uc5c8\ub2e4. \uadf8\ub294 \ud314\ub791\ud06c\uc2a4\uc758 \ubc00\uc9d1\ub300\ud615\uc744 \ud615\uc131\ud560 \ud615\ud3b8\uc774 \uc88b\uc740 \uc7a5\ucc3d\uacfc \ub465\uadfc\ubc29\ud328\ub85c \ubc14\uafb8\uc5c8\ub2e4.", "paragraph_answer": "\uae30\uc6d0\uc804 221\ub144, \ud544\ub85c\ud3ec\uc774\uba58\uc740 \uc804\uc7c1 \uc911\uc778 \ud06c\ub808\ud0c0 \uc5d0\uc11c \uc6a9\ubcd1 \ub300\uc7a5\uc73c\ub85c \uc2f8\uc6e0\ub2e4. 10\ub144 \ud6c4 \uadc0\uad6d\uc5d0 \uc988\uc74c\ud558\uc5ec \uadf8\ub294 \uc544\uce74\uc774\uc544 \ub3d9\ub9f9\uc758 \uae30\ubcd1\ub300 \uc9c0\ud718\uad00\uc73c\ub85c \uc784\uba85\ub418\uc5c8\ub2e4. \ub2f9\uc2dc \uc544\uce74\uc774\uc544\uc758 \uae30\ubcd1\ub4e4\uc740 \uc57d\ud588\uae30 \ub54c\ubb38\uc5d0 \uadf8\ub294 \uae30\ubcd1\uc744 \uc6a9\ub9f9\ud558\uac8c \ub2e8\ub828\ud588\ub2e4. \uac19\uc740 \ud574\uc5d0 \uc81c1\ucc28 \ub9c8\ucf00\ub3c4\ub2c8\uc544 \uc804\uc7c1\uacfc \uc5f0\uad00\ub41c \uc804\ud22c\uc5d0\uc11c \ud544\ub85c\ud3ec\uc774\uba58\uc774 \uc774\ub044\ub294 \uc544\uce74\uc774\uc544 \uad70\uc740 \uc5d8\ub9ac\uc2a4\uc758 \uacbd\uacc4\uc5d0 \uc788\ub294 \ub77c\ub9ac\uc0ac \uac15\uc5d0\uc11c \uc544\uc774\ud1a8\ub9ac\uc544, \uc5d8\ub9ac\uc2a4 \uc5f0\ud569\uad70\uacfc \uc2f8\uc6cc \ubb3c\ub9ac\uce58\uace0, \uc801\uc7a5 \ub2e4\ubaa8\ud310\ud22c\uc2a4\uc640 \uc9c1\uc811 \ub9de\uc11c\uc11c \ubb3c\ub9ac\uce58\uace0 \uc2b9\ub9ac\ub97c \uac70\ub450\uc5c8\ub2e4. \uc774 \uc2b9\ub9ac\ub85c \ud544\ub85c\ud3ec\uc774\uba58\uc758 \uba85\uc131\uc740 \uadf8\ub9ac\uc2a4\uc5d0 \ub110\ub9ac \ud37c\uc84c\uc73c\uba70, \ub610\ud55c \uadf8\ub294 \uc544\uce74\uc774\uc544 \ub3d9\ub9f9\uc758 \ubcf4\ubcd1 \uc7a5\ube44\ub97c \uac1c\ud601\ud588\ub2e4. \uadf8\ub54c\uae4c\uc9c0 \uc544\uce74\uc774\uc544 \uad70\uc758 \uc7a5\ube44\ub294 \uc9e7\uc740 \ucc3d\uacfc \uac00\ubccd\uac8c \uc587\uc740 \ud0c0\uc6d0\ud615 \ubc29\ud328\uc600\uc73c\uba70, \uc774\uac83\uc740 \uc6d0\uac70\ub9ac \uc804\ud22c\uc5d0\uc11c\ub294 \uc720\ub9ac\ud558\uc9c0\ub9cc, \ubc31\ubcd1\uc804\uc5d0\uc11c\ub294 \ubd88\ub9ac\ud558\uc5ec \uc27d\uac8c \uc804\uc5f4\uc774 \ub3cc\ud30c\ub418\uace0 \uc788\uc5c8\ub2e4. \uadf8\ub294 \ud314\ub791\ud06c\uc2a4\uc758 \ubc00\uc9d1\ub300\ud615\uc744 \ud615\uc131\ud560 \ud615\ud3b8\uc774 \uc88b\uc740 \uc7a5\ucc3d\uacfc \ub465\uadfc\ubc29\ud328\ub85c \ubc14\uafb8\uc5c8\ub2e4.", "sentence_answer": "\uae30\uc6d0\uc804 221\ub144, \ud544\ub85c\ud3ec\uc774\uba58\uc740 \uc804\uc7c1 \uc911\uc778 \ud06c\ub808\ud0c0 \uc5d0\uc11c \uc6a9\ubcd1 \ub300\uc7a5\uc73c\ub85c \uc2f8\uc6e0\ub2e4.", "paragraph_id": "6569451-3-0", "paragraph_question": "question: \uae30\uc6d0\uc804 221\ub144 \ud544\ub85c\ud3ec\uc774\uba58\uc774 \uc6a9\ubcd1 \ub300\uc7a5\uc73c\ub85c \uc2f8\uc6b4 \uacf3\uc740 \uc5b4\ub514\uc778\uac00?, context: \uae30\uc6d0\uc804 221\ub144, \ud544\ub85c\ud3ec\uc774\uba58\uc740 \uc804\uc7c1 \uc911\uc778 \ud06c\ub808\ud0c0\uc5d0\uc11c \uc6a9\ubcd1 \ub300\uc7a5\uc73c\ub85c \uc2f8\uc6e0\ub2e4. 10\ub144 \ud6c4 \uadc0\uad6d\uc5d0 \uc988\uc74c\ud558\uc5ec \uadf8\ub294 \uc544\uce74\uc774\uc544 \ub3d9\ub9f9\uc758 \uae30\ubcd1\ub300 \uc9c0\ud718\uad00\uc73c\ub85c \uc784\uba85\ub418\uc5c8\ub2e4. \ub2f9\uc2dc \uc544\uce74\uc774\uc544\uc758 \uae30\ubcd1\ub4e4\uc740 \uc57d\ud588\uae30 \ub54c\ubb38\uc5d0 \uadf8\ub294 \uae30\ubcd1\uc744 \uc6a9\ub9f9\ud558\uac8c \ub2e8\ub828\ud588\ub2e4. \uac19\uc740 \ud574\uc5d0 \uc81c1\ucc28 \ub9c8\ucf00\ub3c4\ub2c8\uc544 \uc804\uc7c1\uacfc \uc5f0\uad00\ub41c \uc804\ud22c\uc5d0\uc11c \ud544\ub85c\ud3ec\uc774\uba58\uc774 \uc774\ub044\ub294 \uc544\uce74\uc774\uc544 \uad70\uc740 \uc5d8\ub9ac\uc2a4\uc758 \uacbd\uacc4\uc5d0 \uc788\ub294 \ub77c\ub9ac\uc0ac \uac15\uc5d0\uc11c \uc544\uc774\ud1a8\ub9ac\uc544, \uc5d8\ub9ac\uc2a4 \uc5f0\ud569\uad70\uacfc \uc2f8\uc6cc \ubb3c\ub9ac\uce58\uace0, \uc801\uc7a5 \ub2e4\ubaa8\ud310\ud22c\uc2a4\uc640 \uc9c1\uc811 \ub9de\uc11c\uc11c \ubb3c\ub9ac\uce58\uace0 \uc2b9\ub9ac\ub97c \uac70\ub450\uc5c8\ub2e4. \uc774 \uc2b9\ub9ac\ub85c \ud544\ub85c\ud3ec\uc774\uba58\uc758 \uba85\uc131\uc740 \uadf8\ub9ac\uc2a4\uc5d0 \ub110\ub9ac \ud37c\uc84c\uc73c\uba70, \ub610\ud55c \uadf8\ub294 \uc544\uce74\uc774\uc544 \ub3d9\ub9f9\uc758 \ubcf4\ubcd1 \uc7a5\ube44\ub97c \uac1c\ud601\ud588\ub2e4. \uadf8\ub54c\uae4c\uc9c0 \uc544\uce74\uc774\uc544 \uad70\uc758 \uc7a5\ube44\ub294 \uc9e7\uc740 \ucc3d\uacfc \uac00\ubccd\uac8c \uc587\uc740 \ud0c0\uc6d0\ud615 \ubc29\ud328\uc600\uc73c\uba70, \uc774\uac83\uc740 \uc6d0\uac70\ub9ac \uc804\ud22c\uc5d0\uc11c\ub294 \uc720\ub9ac\ud558\uc9c0\ub9cc, \ubc31\ubcd1\uc804\uc5d0\uc11c\ub294 \ubd88\ub9ac\ud558\uc5ec \uc27d\uac8c \uc804\uc5f4\uc774 \ub3cc\ud30c\ub418\uace0 \uc788\uc5c8\ub2e4. \uadf8\ub294 \ud314\ub791\ud06c\uc2a4\uc758 \ubc00\uc9d1\ub300\ud615\uc744 \ud615\uc131\ud560 \ud615\ud3b8\uc774 \uc88b\uc740 \uc7a5\ucc3d\uacfc \ub465\uadfc\ubc29\ud328\ub85c \ubc14\uafb8\uc5c8\ub2e4."} {"question": "\uc2a4\ud14c\uace0\ucf00\ub77c\uc2a4\uc758 \ub2e4\ub9ac\ub294 \ud314\ubcf4\ub2e4 \uba87\ubc30 \ub354 \uae38\uc5c8\ub294\uac00?", "paragraph": "\uc2a4\ud14c\uace0\ucf00\ub77c\uc2a4\uc758 \ubab8\uae38\uc774\ub294 \uc57d 2 \ubbf8\ud130, \ubab8\ubb34\uac8c\ub294 10 \uc5d0\uc11c 40 \ud0ac\ub85c\uadf8\ub7a8 \uc815\ub3c4\uc600\ub358 \uac83\uc73c\ub85c \ubcf4\uc778\ub2e4. \uc0c1\ub300\uc801\uc73c\ub85c \ud070 \ud3b8\uc778 \ub1cc\ub294 7.6 \uc13c\ud2f0\ubbf8\ud130 \ub450\uaed8\uc758 \ub450 \ubd80\ubd84\uc73c\ub85c \ub098\ub25c \ubf08\ub85c \ub41c \ub3d4 \uc548\uc5d0 \ub4e4\uc5b4 \uc788\uc5c8\ub2e4. \ub3d4\uc758 \ud45c\uba74\uc740 \ub9e4\uc6b0 \ub9e4\ub048\ud588\uc73c\ub098 \ubd88\uaddc\uce59\uc801\uc73c\ub85c \ubc30\uc5f4\ub41c \uad6c\uba4d\uc774 \uc788\uc5b4\uc11c \ubf08 \ub0b4\ubd80\uc758 \ud1b5\ub85c\ub85c \uc5f0\uacb0\ub418\uc5c8\ub2e4. \uc2a4\ud14c\uace0\ucf00\ub77c\uc2a4\ub294 (\ucd5c\uc18c\ud55c S. validum \uc740) \ub450\uc815-\uc778\uc0c1\uace8(parietosquamosal)\uc5d0 \uc120\ubc18\ubaa8\uc591\uc73c\ub85c \ub69c\ub837\ud558\uac8c \ud280\uc5b4\ub098\uc628 \ubd80\ubd84\uc774 \uc788\uace0 \uc0c1\uce21\ub450\ucc3d (supratemporal fosse) \uc774 \uc5f4\ub824 \uc788\uc73c\uba70 \uc804\ub450-\ub450\uc815\uace8 (frontoparietal) \uc774 \ubbf8\uc57d\ud558\uac8c \ub3d4\uc744 \ud615\uc131\ud558\uace0 \uc791\uc740 \ub3cc\uae30\ub4e4\uc774 \ud6c4\uc548\uc640\uace8\uacfc \uc778\uc0c1\uace8\uc5d0 \ubab0\ub824\uc788\ub2e4. \uc2a4\ud14c\uace0\ucf00\ub77c\uc2a4\uc758 \ub208\uad6c\uba4d\uc740 \ub465\uae00\uace0 \uc55e\uc744 \ud5a5\ud574 \uc788\uc5b4 \uc2dc\ub825\uc774 \uc88b\uc558\uc73c\uba70 \uc544\ub9c8 \uc591\uc548\uc2dc\ub97c \uac00\uc9c0\uace0 \uc788\uc5c8\uc744 \uac83\uc774\ub2e4. \uc774\ube68\uc740 \uc791\uace0 \ud71c \ubaa8\uc591\uc73c\ub85c \uac00\uc7a5\uc790\ub9ac\ub294 \ud1b1\ub2c8\ud615\ud0dc\ub2e4. \uba38\ub9ac\ub294 \"S\" \uc790 \ud639\uc740 \"U\" \uc790 \ubaa8\uc591\uc758 \ubaa9\uc774 \uc9c0\ud0f1\ud574\uc900\ub2e4. \uc2a4\ud14c\uace0\ucf00\ub77c\uc2a4\uc758 \uace8\uaca9 \uc77c\ubd80\uac00 \ucc98\uc74c \ubc1c\uacac\ub418\uc5c8\uc744 \ub54c\ub294 \ub2e4\ub978 \uc870\ubc18\ub958 \uacf5\ub8e1\ub4e4\uc740 \ubcf4\ud1b5 \uac00\uc9c0\uace0 \uc788\uc9c0 \uc54a\uc740 \ubcf5\ubd80\uac08\ube44\ubf08\uac00 \uc788\ub294 \uac83\uc73c\ub85c \uc0dd\uac01\ub418\uc5c8\ub2e4. \ubcf5\ubd80\uac08\ube44\ubf08\ub77c\uace0 \uc0dd\uac01\ub418\uc5c8\ub358 \uac83\uc740 \ub098\uc911\uc5d0 \uace8\uc9c8\ud654\ub41c \ud798\uc904\uc774\uc5c8\ub358 \uac83\uc73c\ub85c \ubc1d\ud600\uc84c\ub2e4. \ub2e4\ub9ac\ub294 \ud314\ubcf4\ub2e4 \uc138 \ubc30 \uc774\uc0c1 \uae38\uc5c8\ub2e4.", "answer": "\uc138 \ubc30", "sentence": "\ub2e4\ub9ac\ub294 \ud314\ubcf4\ub2e4 \uc138 \ubc30 \uc774\uc0c1 \uae38\uc5c8\ub2e4.", "paragraph_sentence": "\uc2a4\ud14c\uace0\ucf00\ub77c\uc2a4\uc758 \ubab8\uae38\uc774\ub294 \uc57d 2 \ubbf8\ud130, \ubab8\ubb34\uac8c\ub294 10 \uc5d0\uc11c 40 \ud0ac\ub85c\uadf8\ub7a8 \uc815\ub3c4\uc600\ub358 \uac83\uc73c\ub85c \ubcf4\uc778\ub2e4. \uc0c1\ub300\uc801\uc73c\ub85c \ud070 \ud3b8\uc778 \ub1cc\ub294 7.6 \uc13c\ud2f0\ubbf8\ud130 \ub450\uaed8\uc758 \ub450 \ubd80\ubd84\uc73c\ub85c \ub098\ub25c \ubf08\ub85c \ub41c \ub3d4 \uc548\uc5d0 \ub4e4\uc5b4 \uc788\uc5c8\ub2e4. \ub3d4\uc758 \ud45c\uba74\uc740 \ub9e4\uc6b0 \ub9e4\ub048\ud588\uc73c\ub098 \ubd88\uaddc\uce59\uc801\uc73c\ub85c \ubc30\uc5f4\ub41c \uad6c\uba4d\uc774 \uc788\uc5b4\uc11c \ubf08 \ub0b4\ubd80\uc758 \ud1b5\ub85c\ub85c \uc5f0\uacb0\ub418\uc5c8\ub2e4. \uc2a4\ud14c\uace0\ucf00\ub77c\uc2a4\ub294 (\ucd5c\uc18c\ud55c S. validum \uc740) \ub450\uc815-\uc778\uc0c1\uace8(parietosquamosal)\uc5d0 \uc120\ubc18\ubaa8\uc591\uc73c\ub85c \ub69c\ub837\ud558\uac8c \ud280\uc5b4\ub098\uc628 \ubd80\ubd84\uc774 \uc788\uace0 \uc0c1\uce21\ub450\ucc3d (supratemporal fosse) \uc774 \uc5f4\ub824 \uc788\uc73c\uba70 \uc804\ub450-\ub450\uc815\uace8 (frontoparietal) \uc774 \ubbf8\uc57d\ud558\uac8c \ub3d4\uc744 \ud615\uc131\ud558\uace0 \uc791\uc740 \ub3cc\uae30\ub4e4\uc774 \ud6c4\uc548\uc640\uace8\uacfc \uc778\uc0c1\uace8\uc5d0 \ubab0\ub824\uc788\ub2e4. \uc2a4\ud14c\uace0\ucf00\ub77c\uc2a4\uc758 \ub208\uad6c\uba4d\uc740 \ub465\uae00\uace0 \uc55e\uc744 \ud5a5\ud574 \uc788\uc5b4 \uc2dc\ub825\uc774 \uc88b\uc558\uc73c\uba70 \uc544\ub9c8 \uc591\uc548\uc2dc\ub97c \uac00\uc9c0\uace0 \uc788\uc5c8\uc744 \uac83\uc774\ub2e4. \uc774\ube68\uc740 \uc791\uace0 \ud71c \ubaa8\uc591\uc73c\ub85c \uac00\uc7a5\uc790\ub9ac\ub294 \ud1b1\ub2c8\ud615\ud0dc\ub2e4. \uba38\ub9ac\ub294 \"S\" \uc790 \ud639\uc740 \"U\" \uc790 \ubaa8\uc591\uc758 \ubaa9\uc774 \uc9c0\ud0f1\ud574\uc900\ub2e4. \uc2a4\ud14c\uace0\ucf00\ub77c\uc2a4\uc758 \uace8\uaca9 \uc77c\ubd80\uac00 \ucc98\uc74c \ubc1c\uacac\ub418\uc5c8\uc744 \ub54c\ub294 \ub2e4\ub978 \uc870\ubc18\ub958 \uacf5\ub8e1\ub4e4\uc740 \ubcf4\ud1b5 \uac00\uc9c0\uace0 \uc788\uc9c0 \uc54a\uc740 \ubcf5\ubd80\uac08\ube44\ubf08\uac00 \uc788\ub294 \uac83\uc73c\ub85c \uc0dd\uac01\ub418\uc5c8\ub2e4. \ubcf5\ubd80\uac08\ube44\ubf08\ub77c\uace0 \uc0dd\uac01\ub418\uc5c8\ub358 \uac83\uc740 \ub098\uc911\uc5d0 \uace8\uc9c8\ud654\ub41c \ud798\uc904\uc774\uc5c8\ub358 \uac83\uc73c\ub85c \ubc1d\ud600\uc84c\ub2e4. \ub2e4\ub9ac\ub294 \ud314\ubcf4\ub2e4 \uc138 \ubc30 \uc774\uc0c1 \uae38\uc5c8\ub2e4. ", "paragraph_answer": "\uc2a4\ud14c\uace0\ucf00\ub77c\uc2a4\uc758 \ubab8\uae38\uc774\ub294 \uc57d 2 \ubbf8\ud130, \ubab8\ubb34\uac8c\ub294 10 \uc5d0\uc11c 40 \ud0ac\ub85c\uadf8\ub7a8 \uc815\ub3c4\uc600\ub358 \uac83\uc73c\ub85c \ubcf4\uc778\ub2e4. \uc0c1\ub300\uc801\uc73c\ub85c \ud070 \ud3b8\uc778 \ub1cc\ub294 7.6 \uc13c\ud2f0\ubbf8\ud130 \ub450\uaed8\uc758 \ub450 \ubd80\ubd84\uc73c\ub85c \ub098\ub25c \ubf08\ub85c \ub41c \ub3d4 \uc548\uc5d0 \ub4e4\uc5b4 \uc788\uc5c8\ub2e4. \ub3d4\uc758 \ud45c\uba74\uc740 \ub9e4\uc6b0 \ub9e4\ub048\ud588\uc73c\ub098 \ubd88\uaddc\uce59\uc801\uc73c\ub85c \ubc30\uc5f4\ub41c \uad6c\uba4d\uc774 \uc788\uc5b4\uc11c \ubf08 \ub0b4\ubd80\uc758 \ud1b5\ub85c\ub85c \uc5f0\uacb0\ub418\uc5c8\ub2e4. \uc2a4\ud14c\uace0\ucf00\ub77c\uc2a4\ub294 (\ucd5c\uc18c\ud55c S. validum \uc740) \ub450\uc815-\uc778\uc0c1\uace8(parietosquamosal)\uc5d0 \uc120\ubc18\ubaa8\uc591\uc73c\ub85c \ub69c\ub837\ud558\uac8c \ud280\uc5b4\ub098\uc628 \ubd80\ubd84\uc774 \uc788\uace0 \uc0c1\uce21\ub450\ucc3d (supratemporal fosse) \uc774 \uc5f4\ub824 \uc788\uc73c\uba70 \uc804\ub450-\ub450\uc815\uace8 (frontoparietal) \uc774 \ubbf8\uc57d\ud558\uac8c \ub3d4\uc744 \ud615\uc131\ud558\uace0 \uc791\uc740 \ub3cc\uae30\ub4e4\uc774 \ud6c4\uc548\uc640\uace8\uacfc \uc778\uc0c1\uace8\uc5d0 \ubab0\ub824\uc788\ub2e4. \uc2a4\ud14c\uace0\ucf00\ub77c\uc2a4\uc758 \ub208\uad6c\uba4d\uc740 \ub465\uae00\uace0 \uc55e\uc744 \ud5a5\ud574 \uc788\uc5b4 \uc2dc\ub825\uc774 \uc88b\uc558\uc73c\uba70 \uc544\ub9c8 \uc591\uc548\uc2dc\ub97c \uac00\uc9c0\uace0 \uc788\uc5c8\uc744 \uac83\uc774\ub2e4. \uc774\ube68\uc740 \uc791\uace0 \ud71c \ubaa8\uc591\uc73c\ub85c \uac00\uc7a5\uc790\ub9ac\ub294 \ud1b1\ub2c8\ud615\ud0dc\ub2e4. \uba38\ub9ac\ub294 \"S\" \uc790 \ud639\uc740 \"U\" \uc790 \ubaa8\uc591\uc758 \ubaa9\uc774 \uc9c0\ud0f1\ud574\uc900\ub2e4. \uc2a4\ud14c\uace0\ucf00\ub77c\uc2a4\uc758 \uace8\uaca9 \uc77c\ubd80\uac00 \ucc98\uc74c \ubc1c\uacac\ub418\uc5c8\uc744 \ub54c\ub294 \ub2e4\ub978 \uc870\ubc18\ub958 \uacf5\ub8e1\ub4e4\uc740 \ubcf4\ud1b5 \uac00\uc9c0\uace0 \uc788\uc9c0 \uc54a\uc740 \ubcf5\ubd80\uac08\ube44\ubf08\uac00 \uc788\ub294 \uac83\uc73c\ub85c \uc0dd\uac01\ub418\uc5c8\ub2e4. \ubcf5\ubd80\uac08\ube44\ubf08\ub77c\uace0 \uc0dd\uac01\ub418\uc5c8\ub358 \uac83\uc740 \ub098\uc911\uc5d0 \uace8\uc9c8\ud654\ub41c \ud798\uc904\uc774\uc5c8\ub358 \uac83\uc73c\ub85c \ubc1d\ud600\uc84c\ub2e4. \ub2e4\ub9ac\ub294 \ud314\ubcf4\ub2e4 \uc138 \ubc30 \uc774\uc0c1 \uae38\uc5c8\ub2e4.", "sentence_answer": "\ub2e4\ub9ac\ub294 \ud314\ubcf4\ub2e4 \uc138 \ubc30 \uc774\uc0c1 \uae38\uc5c8\ub2e4.", "paragraph_id": "6547153-0-0", "paragraph_question": "question: \uc2a4\ud14c\uace0\ucf00\ub77c\uc2a4\uc758 \ub2e4\ub9ac\ub294 \ud314\ubcf4\ub2e4 \uba87\ubc30 \ub354 \uae38\uc5c8\ub294\uac00?, context: \uc2a4\ud14c\uace0\ucf00\ub77c\uc2a4\uc758 \ubab8\uae38\uc774\ub294 \uc57d 2 \ubbf8\ud130, \ubab8\ubb34\uac8c\ub294 10 \uc5d0\uc11c 40 \ud0ac\ub85c\uadf8\ub7a8 \uc815\ub3c4\uc600\ub358 \uac83\uc73c\ub85c \ubcf4\uc778\ub2e4. \uc0c1\ub300\uc801\uc73c\ub85c \ud070 \ud3b8\uc778 \ub1cc\ub294 7.6 \uc13c\ud2f0\ubbf8\ud130 \ub450\uaed8\uc758 \ub450 \ubd80\ubd84\uc73c\ub85c \ub098\ub25c \ubf08\ub85c \ub41c \ub3d4 \uc548\uc5d0 \ub4e4\uc5b4 \uc788\uc5c8\ub2e4. \ub3d4\uc758 \ud45c\uba74\uc740 \ub9e4\uc6b0 \ub9e4\ub048\ud588\uc73c\ub098 \ubd88\uaddc\uce59\uc801\uc73c\ub85c \ubc30\uc5f4\ub41c \uad6c\uba4d\uc774 \uc788\uc5b4\uc11c \ubf08 \ub0b4\ubd80\uc758 \ud1b5\ub85c\ub85c \uc5f0\uacb0\ub418\uc5c8\ub2e4. \uc2a4\ud14c\uace0\ucf00\ub77c\uc2a4\ub294 (\ucd5c\uc18c\ud55c S. validum \uc740) \ub450\uc815-\uc778\uc0c1\uace8(parietosquamosal)\uc5d0 \uc120\ubc18\ubaa8\uc591\uc73c\ub85c \ub69c\ub837\ud558\uac8c \ud280\uc5b4\ub098\uc628 \ubd80\ubd84\uc774 \uc788\uace0 \uc0c1\uce21\ub450\ucc3d (supratemporal fosse) \uc774 \uc5f4\ub824 \uc788\uc73c\uba70 \uc804\ub450-\ub450\uc815\uace8 (frontoparietal) \uc774 \ubbf8\uc57d\ud558\uac8c \ub3d4\uc744 \ud615\uc131\ud558\uace0 \uc791\uc740 \ub3cc\uae30\ub4e4\uc774 \ud6c4\uc548\uc640\uace8\uacfc \uc778\uc0c1\uace8\uc5d0 \ubab0\ub824\uc788\ub2e4. \uc2a4\ud14c\uace0\ucf00\ub77c\uc2a4\uc758 \ub208\uad6c\uba4d\uc740 \ub465\uae00\uace0 \uc55e\uc744 \ud5a5\ud574 \uc788\uc5b4 \uc2dc\ub825\uc774 \uc88b\uc558\uc73c\uba70 \uc544\ub9c8 \uc591\uc548\uc2dc\ub97c \uac00\uc9c0\uace0 \uc788\uc5c8\uc744 \uac83\uc774\ub2e4. \uc774\ube68\uc740 \uc791\uace0 \ud71c \ubaa8\uc591\uc73c\ub85c \uac00\uc7a5\uc790\ub9ac\ub294 \ud1b1\ub2c8\ud615\ud0dc\ub2e4. \uba38\ub9ac\ub294 \"S\" \uc790 \ud639\uc740 \"U\" \uc790 \ubaa8\uc591\uc758 \ubaa9\uc774 \uc9c0\ud0f1\ud574\uc900\ub2e4. \uc2a4\ud14c\uace0\ucf00\ub77c\uc2a4\uc758 \uace8\uaca9 \uc77c\ubd80\uac00 \ucc98\uc74c \ubc1c\uacac\ub418\uc5c8\uc744 \ub54c\ub294 \ub2e4\ub978 \uc870\ubc18\ub958 \uacf5\ub8e1\ub4e4\uc740 \ubcf4\ud1b5 \uac00\uc9c0\uace0 \uc788\uc9c0 \uc54a\uc740 \ubcf5\ubd80\uac08\ube44\ubf08\uac00 \uc788\ub294 \uac83\uc73c\ub85c \uc0dd\uac01\ub418\uc5c8\ub2e4. \ubcf5\ubd80\uac08\ube44\ubf08\ub77c\uace0 \uc0dd\uac01\ub418\uc5c8\ub358 \uac83\uc740 \ub098\uc911\uc5d0 \uace8\uc9c8\ud654\ub41c \ud798\uc904\uc774\uc5c8\ub358 \uac83\uc73c\ub85c \ubc1d\ud600\uc84c\ub2e4. \ub2e4\ub9ac\ub294 \ud314\ubcf4\ub2e4 \uc138 \ubc30 \uc774\uc0c1 \uae38\uc5c8\ub2e4."} {"question": "\ube14\ub808\uc774\ud06c \ub77c\uc774\ube14\ub9ac\ub294 \uc5b4\ub290\ub098\ub77c \uc0ac\ub78c\uc778\uac00?", "paragraph": "\ube14\ub808\uc774\ud06c \ub77c\uc774\ube14\ub9ac(\uc601\uc5b4: Blake Lively, (\ubcf8\uba85: \ube14\ub808\uc774\ud06c \uc5d8\ub7f0\ub354 \ube0c\ub77c\uc6b4, Blake Ellender Brown), 1987\ub144 8\uc6d4 25\uc77c ~ )\ub294 \ubbf8\uad6d\uc758 \uc5ec\ubc30\uc6b0\uc774\ub2e4. \uadf8\ub140\ub294 2007\ub144\ubd80\ud130 2012\ub144\uae4c\uc9c0 The CW\uc5d0\uc11c \uc2dc\uc98c 6\uc5d0 \uac78\uccd0 \ubc29\uc1a1\ub41c \ud558\uc774\ud2f4 \ub85c\ub9e8\uc2a4 \ub4dc\ub77c\ub9c8 \u300a\uac00\uc2ed\uac78\u300b\uc5d0\uc11c \uc138\ub808\ub098 \ubc18 \ub354 \uc6b0\ub4dc\uc2a8 \uc5ed\ud560\uc744 \ub9e1\uc544 \uc8fc\ubaa9\uc744 \ubc1b\uae30 \uc2dc\uc791\ud588\ub2e4. \ub77c\uc774\ube14\ub9ac\ub294 \ub610\ud55c \u300a\uccad\ubc14\uc9c0 \ub3cc\ub824 \uc785\uae30\u300b(2005), \u300a\uc5b4\uc149\ud2f0\ub4dc\u300b(2006), \u300a\ud53c\ud30c \ub9ac\uc758 \ud2b9\ubcc4\ud55c \ub85c\ub9e8\uc2a4 2\u300b(2009) , \u300a\ud0c0\uc6b4\u300b(2010), \u300a\uadf8\ub9b0 \ub79c\ud134: \ubc18\uc9c0\uc758 \uc120\ud0dd\u300b(2011), \u300a\ud30c\uad34\uc790\ub4e4(Savages)\u300b(2012)\uc640 \u300a\uc544\ub378\ub77c\uc778 : \uba48\ucdb0\uc9c4 \uc2dc\uac04\u300b(2015)\uacfc \u300a\uc5b8\ub354 \uc6cc\ud130\u300b(2016) \ub4f1\uc758 \uc601\ud654\uc5d0 \ucd9c\uc5f0\ud558\uc600\ub2e4.", "answer": "\ubbf8\uad6d", "sentence": "\ube14\ub808\uc774\ud06c \ub77c\uc774\ube14\ub9ac(\uc601\uc5b4: Blake Lively, (\ubcf8\uba85: \ube14\ub808\uc774\ud06c \uc5d8\ub7f0\ub354 \ube0c\ub77c\uc6b4, Blake Ellender Brown), 1987\ub144 8\uc6d4 25\uc77c ~ )\ub294 \ubbf8\uad6d \uc758 \uc5ec\ubc30\uc6b0\uc774\ub2e4.", "paragraph_sentence": " \ube14\ub808\uc774\ud06c \ub77c\uc774\ube14\ub9ac(\uc601\uc5b4: Blake Lively, (\ubcf8\uba85: \ube14\ub808\uc774\ud06c \uc5d8\ub7f0\ub354 \ube0c\ub77c\uc6b4, Blake Ellender Brown), 1987\ub144 8\uc6d4 25\uc77c ~ )\ub294 \ubbf8\uad6d \uc758 \uc5ec\ubc30\uc6b0\uc774\ub2e4. \uadf8\ub140\ub294 2007\ub144\ubd80\ud130 2012\ub144\uae4c\uc9c0 The CW\uc5d0\uc11c \uc2dc\uc98c 6\uc5d0 \uac78\uccd0 \ubc29\uc1a1\ub41c \ud558\uc774\ud2f4 \ub85c\ub9e8\uc2a4 \ub4dc\ub77c\ub9c8 \u300a\uac00\uc2ed\uac78\u300b\uc5d0\uc11c \uc138\ub808\ub098 \ubc18 \ub354 \uc6b0\ub4dc\uc2a8 \uc5ed\ud560\uc744 \ub9e1\uc544 \uc8fc\ubaa9\uc744 \ubc1b\uae30 \uc2dc\uc791\ud588\ub2e4. \ub77c\uc774\ube14\ub9ac\ub294 \ub610\ud55c \u300a\uccad\ubc14\uc9c0 \ub3cc\ub824 \uc785\uae30\u300b(2005), \u300a\uc5b4\uc149\ud2f0\ub4dc\u300b(2006), \u300a\ud53c\ud30c \ub9ac\uc758 \ud2b9\ubcc4\ud55c \ub85c\ub9e8\uc2a4 2\u300b(2009) , \u300a\ud0c0\uc6b4\u300b(2010), \u300a\uadf8\ub9b0 \ub79c\ud134: \ubc18\uc9c0\uc758 \uc120\ud0dd\u300b(2011), \u300a\ud30c\uad34\uc790\ub4e4(Savages)\u300b(2012)\uc640 \u300a\uc544\ub378\ub77c\uc778 : \uba48\ucdb0\uc9c4 \uc2dc\uac04\u300b(2015)\uacfc \u300a\uc5b8\ub354 \uc6cc\ud130\u300b(2016) \ub4f1\uc758 \uc601\ud654\uc5d0 \ucd9c\uc5f0\ud558\uc600\ub2e4.", "paragraph_answer": "\ube14\ub808\uc774\ud06c \ub77c\uc774\ube14\ub9ac(\uc601\uc5b4: Blake Lively, (\ubcf8\uba85: \ube14\ub808\uc774\ud06c \uc5d8\ub7f0\ub354 \ube0c\ub77c\uc6b4, Blake Ellender Brown), 1987\ub144 8\uc6d4 25\uc77c ~ )\ub294 \ubbf8\uad6d \uc758 \uc5ec\ubc30\uc6b0\uc774\ub2e4. \uadf8\ub140\ub294 2007\ub144\ubd80\ud130 2012\ub144\uae4c\uc9c0 The CW\uc5d0\uc11c \uc2dc\uc98c 6\uc5d0 \uac78\uccd0 \ubc29\uc1a1\ub41c \ud558\uc774\ud2f4 \ub85c\ub9e8\uc2a4 \ub4dc\ub77c\ub9c8 \u300a\uac00\uc2ed\uac78\u300b\uc5d0\uc11c \uc138\ub808\ub098 \ubc18 \ub354 \uc6b0\ub4dc\uc2a8 \uc5ed\ud560\uc744 \ub9e1\uc544 \uc8fc\ubaa9\uc744 \ubc1b\uae30 \uc2dc\uc791\ud588\ub2e4. \ub77c\uc774\ube14\ub9ac\ub294 \ub610\ud55c \u300a\uccad\ubc14\uc9c0 \ub3cc\ub824 \uc785\uae30\u300b(2005), \u300a\uc5b4\uc149\ud2f0\ub4dc\u300b(2006), \u300a\ud53c\ud30c \ub9ac\uc758 \ud2b9\ubcc4\ud55c \ub85c\ub9e8\uc2a4 2\u300b(2009) , \u300a\ud0c0\uc6b4\u300b(2010), \u300a\uadf8\ub9b0 \ub79c\ud134: \ubc18\uc9c0\uc758 \uc120\ud0dd\u300b(2011), \u300a\ud30c\uad34\uc790\ub4e4(Savages)\u300b(2012)\uc640 \u300a\uc544\ub378\ub77c\uc778 : \uba48\ucdb0\uc9c4 \uc2dc\uac04\u300b(2015)\uacfc \u300a\uc5b8\ub354 \uc6cc\ud130\u300b(2016) \ub4f1\uc758 \uc601\ud654\uc5d0 \ucd9c\uc5f0\ud558\uc600\ub2e4.", "sentence_answer": "\ube14\ub808\uc774\ud06c \ub77c\uc774\ube14\ub9ac(\uc601\uc5b4: Blake Lively, (\ubcf8\uba85: \ube14\ub808\uc774\ud06c \uc5d8\ub7f0\ub354 \ube0c\ub77c\uc6b4, Blake Ellender Brown), 1987\ub144 8\uc6d4 25\uc77c ~ )\ub294 \ubbf8\uad6d \uc758 \uc5ec\ubc30\uc6b0\uc774\ub2e4.", "paragraph_id": "6123582-0-1", "paragraph_question": "question: \ube14\ub808\uc774\ud06c \ub77c\uc774\ube14\ub9ac\ub294 \uc5b4\ub290\ub098\ub77c \uc0ac\ub78c\uc778\uac00?, context: \ube14\ub808\uc774\ud06c \ub77c\uc774\ube14\ub9ac(\uc601\uc5b4: Blake Lively, (\ubcf8\uba85: \ube14\ub808\uc774\ud06c \uc5d8\ub7f0\ub354 \ube0c\ub77c\uc6b4, Blake Ellender Brown), 1987\ub144 8\uc6d4 25\uc77c ~ )\ub294 \ubbf8\uad6d\uc758 \uc5ec\ubc30\uc6b0\uc774\ub2e4. \uadf8\ub140\ub294 2007\ub144\ubd80\ud130 2012\ub144\uae4c\uc9c0 The CW\uc5d0\uc11c \uc2dc\uc98c 6\uc5d0 \uac78\uccd0 \ubc29\uc1a1\ub41c \ud558\uc774\ud2f4 \ub85c\ub9e8\uc2a4 \ub4dc\ub77c\ub9c8 \u300a\uac00\uc2ed\uac78\u300b\uc5d0\uc11c \uc138\ub808\ub098 \ubc18 \ub354 \uc6b0\ub4dc\uc2a8 \uc5ed\ud560\uc744 \ub9e1\uc544 \uc8fc\ubaa9\uc744 \ubc1b\uae30 \uc2dc\uc791\ud588\ub2e4. \ub77c\uc774\ube14\ub9ac\ub294 \ub610\ud55c \u300a\uccad\ubc14\uc9c0 \ub3cc\ub824 \uc785\uae30\u300b(2005), \u300a\uc5b4\uc149\ud2f0\ub4dc\u300b(2006), \u300a\ud53c\ud30c \ub9ac\uc758 \ud2b9\ubcc4\ud55c \ub85c\ub9e8\uc2a4 2\u300b(2009) , \u300a\ud0c0\uc6b4\u300b(2010), \u300a\uadf8\ub9b0 \ub79c\ud134: \ubc18\uc9c0\uc758 \uc120\ud0dd\u300b(2011), \u300a\ud30c\uad34\uc790\ub4e4(Savages)\u300b(2012)\uc640 \u300a\uc544\ub378\ub77c\uc778 : \uba48\ucdb0\uc9c4 \uc2dc\uac04\u300b(2015)\uacfc \u300a\uc5b8\ub354 \uc6cc\ud130\u300b(2016) \ub4f1\uc758 \uc601\ud654\uc5d0 \ucd9c\uc5f0\ud558\uc600\ub2e4."} {"question": "SM \uc5d4\ud130\ud14c\uc778\uba3c\ud2b8\uac00 \uc18c\ub140\uc2dc\ub300\uc758 The Boys \uc568\ubc94 \uc791\ud488\uc758 \ucee8\uc149\uc740 \ubb34\uc5c7\uc774\ub77c\uace0 \ubc1d\ud614\ub098?", "paragraph": "2011\ub144 8\uc6d4 12\uc77c \u300a\ub274\uc2a4\uc5d4\u300b\uc740 9\uc6d4\uaed8 \uc0c8 \uc568\ubc94\uc744 \ubc1c\ub9e4\ud55c\ub2e4\uace0 \uae30\uc0ac\ub97c \ubcf4\ub3c4\ud588\uace0, SM \uc5d4\ud130\ud14c\uc778\uba3c\ud2b8\uc640\uc758 \uc804\ud654\ud1b5\ud654\uc5d0\uc11c \"9, 10\uc6d4\uaed8\ub85c \uc0c8 \uc568\ubc94 \ubc1c\ud45c \uc2dc\uae30\ub97c \uc608\uc0c1\ud558\uace0 \uc788\ub2e4\"\uba70 \"\uc544\uc9c1 \uc815\ud655\ud55c \uc77c\uc790\uac00 \ub098\uc628 \uac83\uc740 \uc544\ub2c8\ub2e4\"\ub77c\uace0 \uc804\ud588\ub2e4. 2011\ub144 9\uc6d4 26\uc77c, SM \uc5d4\ud130\ud14c\uc778\uba3c\ud2b8\ub294 10\uc6d4 4\uc77c\uc5d0 \uc18c\ub140\uc2dc\ub300\uc758 \uc0c8 \uc815\uaddc \uc568\ubc94 \ud0c0\uc774\ud2c0 \uace1\uc744 \uc544\uc774\ud2a0\uc988\ub97c \ud1b5\ud574 \uc804 \uc138\uacc4\uc5d0 \ubc1c\ud45c\ud558\uace0 \ub2e4\uc74c\ub0a0 \uc568\ubc94\uc744 \uacf5\uac1c\ud55c\ub2e4\uace0 \ubc1d\ud614\ub2e4. \uc774\ubc88 \uc791\ud488\uc758 \ucee8\uc149\uc740 \"\uc6d4\ub4dc\uc640\uc774\ub4dc\"\uc778\ub370, SM\uc5d4\ud130\ud14c\uc778\uba3c\ud2b8\ub294 \uc18c\ub140\uc2dc\ub300\uac00 \"\uc138\uacc4\uac00 \uc8fc\ubaa9\ud558\ub294 \uac78\uadf8\ub8f9\uc73c\ub85c \uc131\uc7a5\ud55c \ub9cc\ud07c, \uc804 \uc138\uacc4\ub97c \ub300\uc0c1\uc73c\ub85c \ud558\ub294 \uc74c\ubc18\uc744 \ub0bc \ud544\uc694\uac00 \uc788\uc5c8\ub2e4\"\uace0 \ucde8\uc9c0\ub97c \uc124\uba85\ud588\ub2e4. \ub300\ud45c\uace1 \u300aThe Boys\u300b\ub294 \uc81c\uc791 \ub2e8\uacc4\ubd80\ud130 \"\uc138\uacc4 \uc5b4\ub514\uc5d0\uc11c \ub4e4\uc5b4\ub3c4 \uacf5\uac10\ud560 \uc218 \uc788\ub294\" \ud31d\uc1a1\uc73c\ub85c \uac00\ub2e5\uc774 \uc7a1\ud614\ub2e4\uace0 \ud55c\ub2e4. \uc774 \uace1\uc740 \ub9c8\uc774\ud074 \uc7ad\uc2a8, \ub808\uc774\ub514 \uac00\uac00, \ud478\uc2dc\ucea3 \ub3cc\uc2a4 \ub4f1\uc758 \uc74c\ubc18\uc744 \ud504\ub85c\ub4c0\uc2f1\ud588\ub358 \ud14c\ub514 \ub77c\uc77c\ub9ac\uac00 \uc791\uace1, \ud3b8\uace1\uc744 \ubaa8\ub450 \ub2f4\ub2f9\ud588\ub2e4. 10\uc6d4 19\uc77c \uc5f4\ub9b0 \uae30\uc790\uac04\ub2f4\ud68c\uc5d0\uc11c \uc18c\ub140\uc2dc\ub300\ub294 \uc6d4\ub4dc\uc640\uc774\ub4dc \uc568\ubc94\uc744 \ub0c8\ub2e4\uace0 \uc18c\ub140\uc2dc\ub300\ub77c\ub294 \ucee8\uc149\uc744 \ubc14\uafb8\uc9c0\ub294 \uc54a\ub294\ub2e4\uace0 \ub9d0\ud588\ub2e4. \ud0dc\uc5f0\uc740 \"\ub9ce\uc740 \ubd84\ub4e4\uc774 \uc720\ud29c\ube0c \uc601\uc0c1\uc744 \ubcf4\uace0 \uc800\ud76c\ub97c \ub530\ub77c\ud558\uc2dc\ub294 \uac78 \ubcf4\uba74\uc11c \uc6b0\ub9ac\uac00 \ubbf8\uad6d\uc774\ub098 \uc720\ub7fd \uc2a4\ud0c0\uc77c\uacfc \ub611\uac19\uc774 \ud574\uc11c \uc88b\uc544\ud558\ub294 \uac8c \uc544\ub2c8\ub77c \uc6b0\ub9ac\ub9cc\uc758 \uc2a4\ud0c0\uc77c\uc744 \uc88b\uc544\ud55c\ub2e4\ub294 \uac78 \ub290\uaf08\ub2e4. \uadf8\ub798\uc11c \uc774\ubc88\uc5d0\ub3c4 \uc6b0\ub9ac \uc2a4\ud0c0\uc77c\uc744 \uadf8\ub300\ub85c \ub2f4\uc558\ub2e4. \uac00\uc0ac\uc5d0 \uc601\uc5b4\uac00 \ub9ce\uc774 \ub4e4\uc5b4\uac14\ub2e4\uace0 \ud574\uc11c \ubbf8\uad6d\uc774\ub098 \uc720\ub7fd \uc2a4\ud0c0\uc77c\uc5d0 \ub9de\ucd94\uc9c4 \uc54a\uc558\ub2e4.\"\uace0 \uc5b8\uae09\ud588\ub2e4.", "answer": "\"\uc6d4\ub4dc\uc640\uc774\ub4dc", "sentence": "\uc774\ubc88 \uc791\ud488\uc758 \ucee8\uc149\uc740 \"\uc6d4\ub4dc\uc640\uc774\ub4dc \"\uc778\ub370, SM\uc5d4\ud130\ud14c\uc778\uba3c\ud2b8\ub294 \uc18c\ub140\uc2dc\ub300\uac00 \"\uc138\uacc4\uac00 \uc8fc\ubaa9\ud558\ub294 \uac78\uadf8\ub8f9\uc73c\ub85c \uc131\uc7a5\ud55c \ub9cc\ud07c, \uc804 \uc138\uacc4\ub97c \ub300\uc0c1\uc73c\ub85c \ud558\ub294 \uc74c\ubc18\uc744 \ub0bc \ud544\uc694\uac00 \uc788\uc5c8\ub2e4\"\uace0 \ucde8\uc9c0\ub97c \uc124\uba85\ud588\ub2e4.", "paragraph_sentence": "2011\ub144 8\uc6d4 12\uc77c \u300a\ub274\uc2a4\uc5d4\u300b\uc740 9\uc6d4\uaed8 \uc0c8 \uc568\ubc94\uc744 \ubc1c\ub9e4\ud55c\ub2e4\uace0 \uae30\uc0ac\ub97c \ubcf4\ub3c4\ud588\uace0, SM \uc5d4\ud130\ud14c\uc778\uba3c\ud2b8\uc640\uc758 \uc804\ud654\ud1b5\ud654\uc5d0\uc11c \"9, 10\uc6d4\uaed8\ub85c \uc0c8 \uc568\ubc94 \ubc1c\ud45c \uc2dc\uae30\ub97c \uc608\uc0c1\ud558\uace0 \uc788\ub2e4\"\uba70 \"\uc544\uc9c1 \uc815\ud655\ud55c \uc77c\uc790\uac00 \ub098\uc628 \uac83\uc740 \uc544\ub2c8\ub2e4\"\ub77c\uace0 \uc804\ud588\ub2e4. 2011\ub144 9\uc6d4 26\uc77c, SM \uc5d4\ud130\ud14c\uc778\uba3c\ud2b8\ub294 10\uc6d4 4\uc77c\uc5d0 \uc18c\ub140\uc2dc\ub300\uc758 \uc0c8 \uc815\uaddc \uc568\ubc94 \ud0c0\uc774\ud2c0 \uace1\uc744 \uc544\uc774\ud2a0\uc988\ub97c \ud1b5\ud574 \uc804 \uc138\uacc4\uc5d0 \ubc1c\ud45c\ud558\uace0 \ub2e4\uc74c\ub0a0 \uc568\ubc94\uc744 \uacf5\uac1c\ud55c\ub2e4\uace0 \ubc1d\ud614\ub2e4. \uc774\ubc88 \uc791\ud488\uc758 \ucee8\uc149\uc740 \"\uc6d4\ub4dc\uc640\uc774\ub4dc \"\uc778\ub370, SM\uc5d4\ud130\ud14c\uc778\uba3c\ud2b8\ub294 \uc18c\ub140\uc2dc\ub300\uac00 \"\uc138\uacc4\uac00 \uc8fc\ubaa9\ud558\ub294 \uac78\uadf8\ub8f9\uc73c\ub85c \uc131\uc7a5\ud55c \ub9cc\ud07c, \uc804 \uc138\uacc4\ub97c \ub300\uc0c1\uc73c\ub85c \ud558\ub294 \uc74c\ubc18\uc744 \ub0bc \ud544\uc694\uac00 \uc788\uc5c8\ub2e4\"\uace0 \ucde8\uc9c0\ub97c \uc124\uba85\ud588\ub2e4. \ub300\ud45c\uace1 \u300aThe Boys\u300b\ub294 \uc81c\uc791 \ub2e8\uacc4\ubd80\ud130 \"\uc138\uacc4 \uc5b4\ub514\uc5d0\uc11c \ub4e4\uc5b4\ub3c4 \uacf5\uac10\ud560 \uc218 \uc788\ub294\" \ud31d\uc1a1\uc73c\ub85c \uac00\ub2e5\uc774 \uc7a1\ud614\ub2e4\uace0 \ud55c\ub2e4. \uc774 \uace1\uc740 \ub9c8\uc774\ud074 \uc7ad\uc2a8, \ub808\uc774\ub514 \uac00\uac00, \ud478\uc2dc\ucea3 \ub3cc\uc2a4 \ub4f1\uc758 \uc74c\ubc18\uc744 \ud504\ub85c\ub4c0\uc2f1\ud588\ub358 \ud14c\ub514 \ub77c\uc77c\ub9ac\uac00 \uc791\uace1, \ud3b8\uace1\uc744 \ubaa8\ub450 \ub2f4\ub2f9\ud588\ub2e4. 10\uc6d4 19\uc77c \uc5f4\ub9b0 \uae30\uc790\uac04\ub2f4\ud68c\uc5d0\uc11c \uc18c\ub140\uc2dc\ub300\ub294 \uc6d4\ub4dc\uc640\uc774\ub4dc \uc568\ubc94\uc744 \ub0c8\ub2e4\uace0 \uc18c\ub140\uc2dc\ub300\ub77c\ub294 \ucee8\uc149\uc744 \ubc14\uafb8\uc9c0\ub294 \uc54a\ub294\ub2e4\uace0 \ub9d0\ud588\ub2e4. \ud0dc\uc5f0\uc740 \"\ub9ce\uc740 \ubd84\ub4e4\uc774 \uc720\ud29c\ube0c \uc601\uc0c1\uc744 \ubcf4\uace0 \uc800\ud76c\ub97c \ub530\ub77c\ud558\uc2dc\ub294 \uac78 \ubcf4\uba74\uc11c \uc6b0\ub9ac\uac00 \ubbf8\uad6d\uc774\ub098 \uc720\ub7fd \uc2a4\ud0c0\uc77c\uacfc \ub611\uac19\uc774 \ud574\uc11c \uc88b\uc544\ud558\ub294 \uac8c \uc544\ub2c8\ub77c \uc6b0\ub9ac\ub9cc\uc758 \uc2a4\ud0c0\uc77c\uc744 \uc88b\uc544\ud55c\ub2e4\ub294 \uac78 \ub290\uaf08\ub2e4. \uadf8\ub798\uc11c \uc774\ubc88\uc5d0\ub3c4 \uc6b0\ub9ac \uc2a4\ud0c0\uc77c\uc744 \uadf8\ub300\ub85c \ub2f4\uc558\ub2e4. \uac00\uc0ac\uc5d0 \uc601\uc5b4\uac00 \ub9ce\uc774 \ub4e4\uc5b4\uac14\ub2e4\uace0 \ud574\uc11c \ubbf8\uad6d\uc774\ub098 \uc720\ub7fd \uc2a4\ud0c0\uc77c\uc5d0 \ub9de\ucd94\uc9c4 \uc54a\uc558\ub2e4. \"\uace0 \uc5b8\uae09\ud588\ub2e4.", "paragraph_answer": "2011\ub144 8\uc6d4 12\uc77c \u300a\ub274\uc2a4\uc5d4\u300b\uc740 9\uc6d4\uaed8 \uc0c8 \uc568\ubc94\uc744 \ubc1c\ub9e4\ud55c\ub2e4\uace0 \uae30\uc0ac\ub97c \ubcf4\ub3c4\ud588\uace0, SM \uc5d4\ud130\ud14c\uc778\uba3c\ud2b8\uc640\uc758 \uc804\ud654\ud1b5\ud654\uc5d0\uc11c \"9, 10\uc6d4\uaed8\ub85c \uc0c8 \uc568\ubc94 \ubc1c\ud45c \uc2dc\uae30\ub97c \uc608\uc0c1\ud558\uace0 \uc788\ub2e4\"\uba70 \"\uc544\uc9c1 \uc815\ud655\ud55c \uc77c\uc790\uac00 \ub098\uc628 \uac83\uc740 \uc544\ub2c8\ub2e4\"\ub77c\uace0 \uc804\ud588\ub2e4. 2011\ub144 9\uc6d4 26\uc77c, SM \uc5d4\ud130\ud14c\uc778\uba3c\ud2b8\ub294 10\uc6d4 4\uc77c\uc5d0 \uc18c\ub140\uc2dc\ub300\uc758 \uc0c8 \uc815\uaddc \uc568\ubc94 \ud0c0\uc774\ud2c0 \uace1\uc744 \uc544\uc774\ud2a0\uc988\ub97c \ud1b5\ud574 \uc804 \uc138\uacc4\uc5d0 \ubc1c\ud45c\ud558\uace0 \ub2e4\uc74c\ub0a0 \uc568\ubc94\uc744 \uacf5\uac1c\ud55c\ub2e4\uace0 \ubc1d\ud614\ub2e4. \uc774\ubc88 \uc791\ud488\uc758 \ucee8\uc149\uc740 \"\uc6d4\ub4dc\uc640\uc774\ub4dc \"\uc778\ub370, SM\uc5d4\ud130\ud14c\uc778\uba3c\ud2b8\ub294 \uc18c\ub140\uc2dc\ub300\uac00 \"\uc138\uacc4\uac00 \uc8fc\ubaa9\ud558\ub294 \uac78\uadf8\ub8f9\uc73c\ub85c \uc131\uc7a5\ud55c \ub9cc\ud07c, \uc804 \uc138\uacc4\ub97c \ub300\uc0c1\uc73c\ub85c \ud558\ub294 \uc74c\ubc18\uc744 \ub0bc \ud544\uc694\uac00 \uc788\uc5c8\ub2e4\"\uace0 \ucde8\uc9c0\ub97c \uc124\uba85\ud588\ub2e4. \ub300\ud45c\uace1 \u300aThe Boys\u300b\ub294 \uc81c\uc791 \ub2e8\uacc4\ubd80\ud130 \"\uc138\uacc4 \uc5b4\ub514\uc5d0\uc11c \ub4e4\uc5b4\ub3c4 \uacf5\uac10\ud560 \uc218 \uc788\ub294\" \ud31d\uc1a1\uc73c\ub85c \uac00\ub2e5\uc774 \uc7a1\ud614\ub2e4\uace0 \ud55c\ub2e4. \uc774 \uace1\uc740 \ub9c8\uc774\ud074 \uc7ad\uc2a8, \ub808\uc774\ub514 \uac00\uac00, \ud478\uc2dc\ucea3 \ub3cc\uc2a4 \ub4f1\uc758 \uc74c\ubc18\uc744 \ud504\ub85c\ub4c0\uc2f1\ud588\ub358 \ud14c\ub514 \ub77c\uc77c\ub9ac\uac00 \uc791\uace1, \ud3b8\uace1\uc744 \ubaa8\ub450 \ub2f4\ub2f9\ud588\ub2e4. 10\uc6d4 19\uc77c \uc5f4\ub9b0 \uae30\uc790\uac04\ub2f4\ud68c\uc5d0\uc11c \uc18c\ub140\uc2dc\ub300\ub294 \uc6d4\ub4dc\uc640\uc774\ub4dc \uc568\ubc94\uc744 \ub0c8\ub2e4\uace0 \uc18c\ub140\uc2dc\ub300\ub77c\ub294 \ucee8\uc149\uc744 \ubc14\uafb8\uc9c0\ub294 \uc54a\ub294\ub2e4\uace0 \ub9d0\ud588\ub2e4. \ud0dc\uc5f0\uc740 \"\ub9ce\uc740 \ubd84\ub4e4\uc774 \uc720\ud29c\ube0c \uc601\uc0c1\uc744 \ubcf4\uace0 \uc800\ud76c\ub97c \ub530\ub77c\ud558\uc2dc\ub294 \uac78 \ubcf4\uba74\uc11c \uc6b0\ub9ac\uac00 \ubbf8\uad6d\uc774\ub098 \uc720\ub7fd \uc2a4\ud0c0\uc77c\uacfc \ub611\uac19\uc774 \ud574\uc11c \uc88b\uc544\ud558\ub294 \uac8c \uc544\ub2c8\ub77c \uc6b0\ub9ac\ub9cc\uc758 \uc2a4\ud0c0\uc77c\uc744 \uc88b\uc544\ud55c\ub2e4\ub294 \uac78 \ub290\uaf08\ub2e4. \uadf8\ub798\uc11c \uc774\ubc88\uc5d0\ub3c4 \uc6b0\ub9ac \uc2a4\ud0c0\uc77c\uc744 \uadf8\ub300\ub85c \ub2f4\uc558\ub2e4. \uac00\uc0ac\uc5d0 \uc601\uc5b4\uac00 \ub9ce\uc774 \ub4e4\uc5b4\uac14\ub2e4\uace0 \ud574\uc11c \ubbf8\uad6d\uc774\ub098 \uc720\ub7fd \uc2a4\ud0c0\uc77c\uc5d0 \ub9de\ucd94\uc9c4 \uc54a\uc558\ub2e4.\"\uace0 \uc5b8\uae09\ud588\ub2e4.", "sentence_answer": "\uc774\ubc88 \uc791\ud488\uc758 \ucee8\uc149\uc740 \"\uc6d4\ub4dc\uc640\uc774\ub4dc \"\uc778\ub370, SM\uc5d4\ud130\ud14c\uc778\uba3c\ud2b8\ub294 \uc18c\ub140\uc2dc\ub300\uac00 \"\uc138\uacc4\uac00 \uc8fc\ubaa9\ud558\ub294 \uac78\uadf8\ub8f9\uc73c\ub85c \uc131\uc7a5\ud55c \ub9cc\ud07c, \uc804 \uc138\uacc4\ub97c \ub300\uc0c1\uc73c\ub85c \ud558\ub294 \uc74c\ubc18\uc744 \ub0bc \ud544\uc694\uac00 \uc788\uc5c8\ub2e4\"\uace0 \ucde8\uc9c0\ub97c \uc124\uba85\ud588\ub2e4.", "paragraph_id": "6472456-1-0", "paragraph_question": "question: SM \uc5d4\ud130\ud14c\uc778\uba3c\ud2b8\uac00 \uc18c\ub140\uc2dc\ub300\uc758 The Boys \uc568\ubc94 \uc791\ud488\uc758 \ucee8\uc149\uc740 \ubb34\uc5c7\uc774\ub77c\uace0 \ubc1d\ud614\ub098?, context: 2011\ub144 8\uc6d4 12\uc77c \u300a\ub274\uc2a4\uc5d4\u300b\uc740 9\uc6d4\uaed8 \uc0c8 \uc568\ubc94\uc744 \ubc1c\ub9e4\ud55c\ub2e4\uace0 \uae30\uc0ac\ub97c \ubcf4\ub3c4\ud588\uace0, SM \uc5d4\ud130\ud14c\uc778\uba3c\ud2b8\uc640\uc758 \uc804\ud654\ud1b5\ud654\uc5d0\uc11c \"9, 10\uc6d4\uaed8\ub85c \uc0c8 \uc568\ubc94 \ubc1c\ud45c \uc2dc\uae30\ub97c \uc608\uc0c1\ud558\uace0 \uc788\ub2e4\"\uba70 \"\uc544\uc9c1 \uc815\ud655\ud55c \uc77c\uc790\uac00 \ub098\uc628 \uac83\uc740 \uc544\ub2c8\ub2e4\"\ub77c\uace0 \uc804\ud588\ub2e4. 2011\ub144 9\uc6d4 26\uc77c, SM \uc5d4\ud130\ud14c\uc778\uba3c\ud2b8\ub294 10\uc6d4 4\uc77c\uc5d0 \uc18c\ub140\uc2dc\ub300\uc758 \uc0c8 \uc815\uaddc \uc568\ubc94 \ud0c0\uc774\ud2c0 \uace1\uc744 \uc544\uc774\ud2a0\uc988\ub97c \ud1b5\ud574 \uc804 \uc138\uacc4\uc5d0 \ubc1c\ud45c\ud558\uace0 \ub2e4\uc74c\ub0a0 \uc568\ubc94\uc744 \uacf5\uac1c\ud55c\ub2e4\uace0 \ubc1d\ud614\ub2e4. \uc774\ubc88 \uc791\ud488\uc758 \ucee8\uc149\uc740 \"\uc6d4\ub4dc\uc640\uc774\ub4dc\"\uc778\ub370, SM\uc5d4\ud130\ud14c\uc778\uba3c\ud2b8\ub294 \uc18c\ub140\uc2dc\ub300\uac00 \"\uc138\uacc4\uac00 \uc8fc\ubaa9\ud558\ub294 \uac78\uadf8\ub8f9\uc73c\ub85c \uc131\uc7a5\ud55c \ub9cc\ud07c, \uc804 \uc138\uacc4\ub97c \ub300\uc0c1\uc73c\ub85c \ud558\ub294 \uc74c\ubc18\uc744 \ub0bc \ud544\uc694\uac00 \uc788\uc5c8\ub2e4\"\uace0 \ucde8\uc9c0\ub97c \uc124\uba85\ud588\ub2e4. \ub300\ud45c\uace1 \u300aThe Boys\u300b\ub294 \uc81c\uc791 \ub2e8\uacc4\ubd80\ud130 \"\uc138\uacc4 \uc5b4\ub514\uc5d0\uc11c \ub4e4\uc5b4\ub3c4 \uacf5\uac10\ud560 \uc218 \uc788\ub294\" \ud31d\uc1a1\uc73c\ub85c \uac00\ub2e5\uc774 \uc7a1\ud614\ub2e4\uace0 \ud55c\ub2e4. \uc774 \uace1\uc740 \ub9c8\uc774\ud074 \uc7ad\uc2a8, \ub808\uc774\ub514 \uac00\uac00, \ud478\uc2dc\ucea3 \ub3cc\uc2a4 \ub4f1\uc758 \uc74c\ubc18\uc744 \ud504\ub85c\ub4c0\uc2f1\ud588\ub358 \ud14c\ub514 \ub77c\uc77c\ub9ac\uac00 \uc791\uace1, \ud3b8\uace1\uc744 \ubaa8\ub450 \ub2f4\ub2f9\ud588\ub2e4. 10\uc6d4 19\uc77c \uc5f4\ub9b0 \uae30\uc790\uac04\ub2f4\ud68c\uc5d0\uc11c \uc18c\ub140\uc2dc\ub300\ub294 \uc6d4\ub4dc\uc640\uc774\ub4dc \uc568\ubc94\uc744 \ub0c8\ub2e4\uace0 \uc18c\ub140\uc2dc\ub300\ub77c\ub294 \ucee8\uc149\uc744 \ubc14\uafb8\uc9c0\ub294 \uc54a\ub294\ub2e4\uace0 \ub9d0\ud588\ub2e4. \ud0dc\uc5f0\uc740 \"\ub9ce\uc740 \ubd84\ub4e4\uc774 \uc720\ud29c\ube0c \uc601\uc0c1\uc744 \ubcf4\uace0 \uc800\ud76c\ub97c \ub530\ub77c\ud558\uc2dc\ub294 \uac78 \ubcf4\uba74\uc11c \uc6b0\ub9ac\uac00 \ubbf8\uad6d\uc774\ub098 \uc720\ub7fd \uc2a4\ud0c0\uc77c\uacfc \ub611\uac19\uc774 \ud574\uc11c \uc88b\uc544\ud558\ub294 \uac8c \uc544\ub2c8\ub77c \uc6b0\ub9ac\ub9cc\uc758 \uc2a4\ud0c0\uc77c\uc744 \uc88b\uc544\ud55c\ub2e4\ub294 \uac78 \ub290\uaf08\ub2e4. \uadf8\ub798\uc11c \uc774\ubc88\uc5d0\ub3c4 \uc6b0\ub9ac \uc2a4\ud0c0\uc77c\uc744 \uadf8\ub300\ub85c \ub2f4\uc558\ub2e4. \uac00\uc0ac\uc5d0 \uc601\uc5b4\uac00 \ub9ce\uc774 \ub4e4\uc5b4\uac14\ub2e4\uace0 \ud574\uc11c \ubbf8\uad6d\uc774\ub098 \uc720\ub7fd \uc2a4\ud0c0\uc77c\uc5d0 \ub9de\ucd94\uc9c4 \uc54a\uc558\ub2e4.\"\uace0 \uc5b8\uae09\ud588\ub2e4."} {"question": "2\uac1c\uc758 \uc5d0\ud53c\uc18c\ub4dc\uc5d0\uc11c 1\uac1c\uc758 \uc5d0\ud53c\uc18c\ub4dc \ubc29\uc601\uc73c\ub85c \ubc14\ub010 \uc2dc\uc810\uc758 \uc5d0\ud53c\uc18c\ub4dc\ub294?", "paragraph": "\u3008Business Men\u3009\uacfc \u3008Evicted!\u3009\ub294 \uce74\ud230 \ub124\ud2b8\uc6cc\ud06c\uc5d0\uc11c \uc815\uc2dd \ubc29\uc601\uc77c\uc778 3\uc6d4 11\uc77c\uacfc 18\uc77c \uc774\uc804\uc5d0 \uac01\uac01 \ud2b9\ubcc4 \ubc29\uc601\ub418\uc5c8\ub2e4. \uc2dc\ub9ac\uc988 \uc804\uccb4\uc758 \ucd5c\ucd08 \uc815\uc2dd \ubc29\uc601\uc740 2010\ub144 4\uc6d4 5\uc77c\uc5d0 \uc774\ub8e8\uc5b4\uc84c\ub2e4. \uccab \uc5d0\ud53c\uc18c\ub4dc\uc778 \u3008Slumber Party Panic\u3009\uacfc \u3008Trouble in Lumpy Space\u3009\uc740 \uac19\uc740 \ub0a0 \ud568\uaed8 \ubc29\uc601\ub418\uc5c8\uace0, 250\ub9cc \uba85\uc758 \uc2dc\uccad\uc790 \uc218\ub97c \uae30\ub85d\ud588\ub2e4. \uce74\ud230 \ub124\ud2b8\uc6cc\ud06c\uc5d0 \uc758\ud574 \ubc1c\ud45c\ub41c \uc790\ub8cc\ub97c \ubcf4\uba74 \ub3d9\uc77c\ud55c \uc2dc\uac04\ub300\uc758 \uc791\ub144 \ubc29\uc601 \ud504\ub85c\uadf8\ub7a8\ubcf4\ub2e4 \uc138\uc790\ub9bf \uc218 \uac00\ub7c9\uc758 \ud37c\uc13c\ud2b8 \uc99d\uac00\ub97c \ubcf4\uc600\ub2e4. \uadf8 \uc608\uc2dc\ub85c 2~11\uc138\uc758 \uc5b4\ub9b0\uc774 166\ub9cc 1\ucc9c \uba85\uc774 \uc2dc\uccad\ud558\uc5ec \uc791\ub144\uc5d0 \ube44\ud574 110%\uc758 \uc0c1\uc2b9\uc744 \uae30\ub85d\ud588\ub2e4. 9~14\uc138\uc758 \uc5b4\ub9b0\uc774 \uc870\uc0ac\uc5d0\uc11c\ub294 837,000\uba85\uc774 \uc2dc\uccad\ud588\uc73c\uba70, 239%\uc758 \uc0c1\uc2b9\uc744 \uae30\ub85d\ud588\ub2e4. \uc774\ub7f0 \uacb0\uacfc\ub85c \ub450 \uc5d0\ud53c\uc18c\ub4dc\ub294 \uccab \ubc88\uc9f8 \uc2dc\uc98c \uc911 \ucd5c\uace0\uc758 \uc2dc\uccad\uc790 \uc218\ub97c \uae30\ub85d\ud588\ub2e4. \uc774 \uc2dc\uc98c\uc740 \u3008Gut Grinder\u3009\uac00 177\ub9cc \uba85\uc758 \uc2dc\uccad\uc790 \uc218\ub97c \uae30\ub85d\ud558\uba70 \ub05d\ub0ac\ub2e4. \ucd08\uae30\uc5d0\ub294 \uccab 10\uac1c\uc758 \uc5d0\ud53c\uc18c\ub4dc\uac00 30\ubd84 \ubd84\ub7c9\uc758 \ubc29\uc1a1\uc73c\ub85c \ud568\uaed8 \ubc29\uc601\ub418\uc5c8\ub2e4. \uc774\ub294 \uac01 11\ubd84 \uc9dc\ub9ac\uc758 \uc5d0\ud53c\uc18c\ub4dc \ub450 \ud3b8\uc774 \ud558\ub098\ub85c \ubb36\uc5ec\uc11c \ubc29\uc601\ub418\uc5c8\ub2e4\ub294 \uac83\uc774\ub2e4. \uadf8\ub7ec\ub098 \uc5f4\ud55c \ubc88\uc9f8 \uc5d0\ud53c\uc18c\ub4dc\uc778 \u3008Wizard\u3009\ubd80\ud130\ub294 2\uac1c\uc758 \uc5d0\ud53c\uc18c\ub4dc\uac00 \uc544\ub2cc 1\uac1c\uc758 \uc5d0\ud53c\uc18c\ub4dc\ub9cc \ubc29\uc601\ud558\uae30\ub85c \ubc14\ub00c\uc5c8\ub2e4.", "answer": "Wizard", "sentence": "\uadf8\ub7ec\ub098 \uc5f4\ud55c \ubc88\uc9f8 \uc5d0\ud53c\uc18c\ub4dc\uc778 \u3008 Wizard \u3009\ubd80\ud130\ub294 2\uac1c\uc758 \uc5d0\ud53c\uc18c\ub4dc\uac00 \uc544\ub2cc 1\uac1c\uc758 \uc5d0\ud53c\uc18c\ub4dc\ub9cc \ubc29\uc601\ud558\uae30\ub85c \ubc14\ub00c\uc5c8\ub2e4.", "paragraph_sentence": "\u3008Business Men\u3009\uacfc \u3008Evicted!\u3009\ub294 \uce74\ud230 \ub124\ud2b8\uc6cc\ud06c\uc5d0\uc11c \uc815\uc2dd \ubc29\uc601\uc77c\uc778 3\uc6d4 11\uc77c\uacfc 18\uc77c \uc774\uc804\uc5d0 \uac01\uac01 \ud2b9\ubcc4 \ubc29\uc601\ub418\uc5c8\ub2e4. \uc2dc\ub9ac\uc988 \uc804\uccb4\uc758 \ucd5c\ucd08 \uc815\uc2dd \ubc29\uc601\uc740 2010\ub144 4\uc6d4 5\uc77c\uc5d0 \uc774\ub8e8\uc5b4\uc84c\ub2e4. \uccab \uc5d0\ud53c\uc18c\ub4dc\uc778 \u3008Slumber Party Panic\u3009\uacfc \u3008Trouble in Lumpy Space\u3009\uc740 \uac19\uc740 \ub0a0 \ud568\uaed8 \ubc29\uc601\ub418\uc5c8\uace0, 250\ub9cc \uba85\uc758 \uc2dc\uccad\uc790 \uc218\ub97c \uae30\ub85d\ud588\ub2e4. \uce74\ud230 \ub124\ud2b8\uc6cc\ud06c\uc5d0 \uc758\ud574 \ubc1c\ud45c\ub41c \uc790\ub8cc\ub97c \ubcf4\uba74 \ub3d9\uc77c\ud55c \uc2dc\uac04\ub300\uc758 \uc791\ub144 \ubc29\uc601 \ud504\ub85c\uadf8\ub7a8\ubcf4\ub2e4 \uc138\uc790\ub9bf \uc218 \uac00\ub7c9\uc758 \ud37c\uc13c\ud2b8 \uc99d\uac00\ub97c \ubcf4\uc600\ub2e4. \uadf8 \uc608\uc2dc\ub85c 2~11\uc138\uc758 \uc5b4\ub9b0\uc774 166\ub9cc 1\ucc9c \uba85\uc774 \uc2dc\uccad\ud558\uc5ec \uc791\ub144\uc5d0 \ube44\ud574 110%\uc758 \uc0c1\uc2b9\uc744 \uae30\ub85d\ud588\ub2e4. 9~14\uc138\uc758 \uc5b4\ub9b0\uc774 \uc870\uc0ac\uc5d0\uc11c\ub294 837,000\uba85\uc774 \uc2dc\uccad\ud588\uc73c\uba70, 239%\uc758 \uc0c1\uc2b9\uc744 \uae30\ub85d\ud588\ub2e4. \uc774\ub7f0 \uacb0\uacfc\ub85c \ub450 \uc5d0\ud53c\uc18c\ub4dc\ub294 \uccab \ubc88\uc9f8 \uc2dc\uc98c \uc911 \ucd5c\uace0\uc758 \uc2dc\uccad\uc790 \uc218\ub97c \uae30\ub85d\ud588\ub2e4. \uc774 \uc2dc\uc98c\uc740 \u3008Gut Grinder\u3009\uac00 177\ub9cc \uba85\uc758 \uc2dc\uccad\uc790 \uc218\ub97c \uae30\ub85d\ud558\uba70 \ub05d\ub0ac\ub2e4. \ucd08\uae30\uc5d0\ub294 \uccab 10\uac1c\uc758 \uc5d0\ud53c\uc18c\ub4dc\uac00 30\ubd84 \ubd84\ub7c9\uc758 \ubc29\uc1a1\uc73c\ub85c \ud568\uaed8 \ubc29\uc601\ub418\uc5c8\ub2e4. \uc774\ub294 \uac01 11\ubd84 \uc9dc\ub9ac\uc758 \uc5d0\ud53c\uc18c\ub4dc \ub450 \ud3b8\uc774 \ud558\ub098\ub85c \ubb36\uc5ec\uc11c \ubc29\uc601\ub418\uc5c8\ub2e4\ub294 \uac83\uc774\ub2e4. \uadf8\ub7ec\ub098 \uc5f4\ud55c \ubc88\uc9f8 \uc5d0\ud53c\uc18c\ub4dc\uc778 \u3008 Wizard \u3009\ubd80\ud130\ub294 2\uac1c\uc758 \uc5d0\ud53c\uc18c\ub4dc\uac00 \uc544\ub2cc 1\uac1c\uc758 \uc5d0\ud53c\uc18c\ub4dc\ub9cc \ubc29\uc601\ud558\uae30\ub85c \ubc14\ub00c\uc5c8\ub2e4. ", "paragraph_answer": "\u3008Business Men\u3009\uacfc \u3008Evicted!\u3009\ub294 \uce74\ud230 \ub124\ud2b8\uc6cc\ud06c\uc5d0\uc11c \uc815\uc2dd \ubc29\uc601\uc77c\uc778 3\uc6d4 11\uc77c\uacfc 18\uc77c \uc774\uc804\uc5d0 \uac01\uac01 \ud2b9\ubcc4 \ubc29\uc601\ub418\uc5c8\ub2e4. \uc2dc\ub9ac\uc988 \uc804\uccb4\uc758 \ucd5c\ucd08 \uc815\uc2dd \ubc29\uc601\uc740 2010\ub144 4\uc6d4 5\uc77c\uc5d0 \uc774\ub8e8\uc5b4\uc84c\ub2e4. \uccab \uc5d0\ud53c\uc18c\ub4dc\uc778 \u3008Slumber Party Panic\u3009\uacfc \u3008Trouble in Lumpy Space\u3009\uc740 \uac19\uc740 \ub0a0 \ud568\uaed8 \ubc29\uc601\ub418\uc5c8\uace0, 250\ub9cc \uba85\uc758 \uc2dc\uccad\uc790 \uc218\ub97c \uae30\ub85d\ud588\ub2e4. \uce74\ud230 \ub124\ud2b8\uc6cc\ud06c\uc5d0 \uc758\ud574 \ubc1c\ud45c\ub41c \uc790\ub8cc\ub97c \ubcf4\uba74 \ub3d9\uc77c\ud55c \uc2dc\uac04\ub300\uc758 \uc791\ub144 \ubc29\uc601 \ud504\ub85c\uadf8\ub7a8\ubcf4\ub2e4 \uc138\uc790\ub9bf \uc218 \uac00\ub7c9\uc758 \ud37c\uc13c\ud2b8 \uc99d\uac00\ub97c \ubcf4\uc600\ub2e4. \uadf8 \uc608\uc2dc\ub85c 2~11\uc138\uc758 \uc5b4\ub9b0\uc774 166\ub9cc 1\ucc9c \uba85\uc774 \uc2dc\uccad\ud558\uc5ec \uc791\ub144\uc5d0 \ube44\ud574 110%\uc758 \uc0c1\uc2b9\uc744 \uae30\ub85d\ud588\ub2e4. 9~14\uc138\uc758 \uc5b4\ub9b0\uc774 \uc870\uc0ac\uc5d0\uc11c\ub294 837,000\uba85\uc774 \uc2dc\uccad\ud588\uc73c\uba70, 239%\uc758 \uc0c1\uc2b9\uc744 \uae30\ub85d\ud588\ub2e4. \uc774\ub7f0 \uacb0\uacfc\ub85c \ub450 \uc5d0\ud53c\uc18c\ub4dc\ub294 \uccab \ubc88\uc9f8 \uc2dc\uc98c \uc911 \ucd5c\uace0\uc758 \uc2dc\uccad\uc790 \uc218\ub97c \uae30\ub85d\ud588\ub2e4. \uc774 \uc2dc\uc98c\uc740 \u3008Gut Grinder\u3009\uac00 177\ub9cc \uba85\uc758 \uc2dc\uccad\uc790 \uc218\ub97c \uae30\ub85d\ud558\uba70 \ub05d\ub0ac\ub2e4. \ucd08\uae30\uc5d0\ub294 \uccab 10\uac1c\uc758 \uc5d0\ud53c\uc18c\ub4dc\uac00 30\ubd84 \ubd84\ub7c9\uc758 \ubc29\uc1a1\uc73c\ub85c \ud568\uaed8 \ubc29\uc601\ub418\uc5c8\ub2e4. \uc774\ub294 \uac01 11\ubd84 \uc9dc\ub9ac\uc758 \uc5d0\ud53c\uc18c\ub4dc \ub450 \ud3b8\uc774 \ud558\ub098\ub85c \ubb36\uc5ec\uc11c \ubc29\uc601\ub418\uc5c8\ub2e4\ub294 \uac83\uc774\ub2e4. \uadf8\ub7ec\ub098 \uc5f4\ud55c \ubc88\uc9f8 \uc5d0\ud53c\uc18c\ub4dc\uc778 \u3008 Wizard \u3009\ubd80\ud130\ub294 2\uac1c\uc758 \uc5d0\ud53c\uc18c\ub4dc\uac00 \uc544\ub2cc 1\uac1c\uc758 \uc5d0\ud53c\uc18c\ub4dc\ub9cc \ubc29\uc601\ud558\uae30\ub85c \ubc14\ub00c\uc5c8\ub2e4.", "sentence_answer": "\uadf8\ub7ec\ub098 \uc5f4\ud55c \ubc88\uc9f8 \uc5d0\ud53c\uc18c\ub4dc\uc778 \u3008 Wizard \u3009\ubd80\ud130\ub294 2\uac1c\uc758 \uc5d0\ud53c\uc18c\ub4dc\uac00 \uc544\ub2cc 1\uac1c\uc758 \uc5d0\ud53c\uc18c\ub4dc\ub9cc \ubc29\uc601\ud558\uae30\ub85c \ubc14\ub00c\uc5c8\ub2e4.", "paragraph_id": "5931268-6-1", "paragraph_question": "question: 2\uac1c\uc758 \uc5d0\ud53c\uc18c\ub4dc\uc5d0\uc11c 1\uac1c\uc758 \uc5d0\ud53c\uc18c\ub4dc \ubc29\uc601\uc73c\ub85c \ubc14\ub010 \uc2dc\uc810\uc758 \uc5d0\ud53c\uc18c\ub4dc\ub294?, context: \u3008Business Men\u3009\uacfc \u3008Evicted!\u3009\ub294 \uce74\ud230 \ub124\ud2b8\uc6cc\ud06c\uc5d0\uc11c \uc815\uc2dd \ubc29\uc601\uc77c\uc778 3\uc6d4 11\uc77c\uacfc 18\uc77c \uc774\uc804\uc5d0 \uac01\uac01 \ud2b9\ubcc4 \ubc29\uc601\ub418\uc5c8\ub2e4. \uc2dc\ub9ac\uc988 \uc804\uccb4\uc758 \ucd5c\ucd08 \uc815\uc2dd \ubc29\uc601\uc740 2010\ub144 4\uc6d4 5\uc77c\uc5d0 \uc774\ub8e8\uc5b4\uc84c\ub2e4. \uccab \uc5d0\ud53c\uc18c\ub4dc\uc778 \u3008Slumber Party Panic\u3009\uacfc \u3008Trouble in Lumpy Space\u3009\uc740 \uac19\uc740 \ub0a0 \ud568\uaed8 \ubc29\uc601\ub418\uc5c8\uace0, 250\ub9cc \uba85\uc758 \uc2dc\uccad\uc790 \uc218\ub97c \uae30\ub85d\ud588\ub2e4. \uce74\ud230 \ub124\ud2b8\uc6cc\ud06c\uc5d0 \uc758\ud574 \ubc1c\ud45c\ub41c \uc790\ub8cc\ub97c \ubcf4\uba74 \ub3d9\uc77c\ud55c \uc2dc\uac04\ub300\uc758 \uc791\ub144 \ubc29\uc601 \ud504\ub85c\uadf8\ub7a8\ubcf4\ub2e4 \uc138\uc790\ub9bf \uc218 \uac00\ub7c9\uc758 \ud37c\uc13c\ud2b8 \uc99d\uac00\ub97c \ubcf4\uc600\ub2e4. \uadf8 \uc608\uc2dc\ub85c 2~11\uc138\uc758 \uc5b4\ub9b0\uc774 166\ub9cc 1\ucc9c \uba85\uc774 \uc2dc\uccad\ud558\uc5ec \uc791\ub144\uc5d0 \ube44\ud574 110%\uc758 \uc0c1\uc2b9\uc744 \uae30\ub85d\ud588\ub2e4. 9~14\uc138\uc758 \uc5b4\ub9b0\uc774 \uc870\uc0ac\uc5d0\uc11c\ub294 837,000\uba85\uc774 \uc2dc\uccad\ud588\uc73c\uba70, 239%\uc758 \uc0c1\uc2b9\uc744 \uae30\ub85d\ud588\ub2e4. \uc774\ub7f0 \uacb0\uacfc\ub85c \ub450 \uc5d0\ud53c\uc18c\ub4dc\ub294 \uccab \ubc88\uc9f8 \uc2dc\uc98c \uc911 \ucd5c\uace0\uc758 \uc2dc\uccad\uc790 \uc218\ub97c \uae30\ub85d\ud588\ub2e4. \uc774 \uc2dc\uc98c\uc740 \u3008Gut Grinder\u3009\uac00 177\ub9cc \uba85\uc758 \uc2dc\uccad\uc790 \uc218\ub97c \uae30\ub85d\ud558\uba70 \ub05d\ub0ac\ub2e4. \ucd08\uae30\uc5d0\ub294 \uccab 10\uac1c\uc758 \uc5d0\ud53c\uc18c\ub4dc\uac00 30\ubd84 \ubd84\ub7c9\uc758 \ubc29\uc1a1\uc73c\ub85c \ud568\uaed8 \ubc29\uc601\ub418\uc5c8\ub2e4. \uc774\ub294 \uac01 11\ubd84 \uc9dc\ub9ac\uc758 \uc5d0\ud53c\uc18c\ub4dc \ub450 \ud3b8\uc774 \ud558\ub098\ub85c \ubb36\uc5ec\uc11c \ubc29\uc601\ub418\uc5c8\ub2e4\ub294 \uac83\uc774\ub2e4. \uadf8\ub7ec\ub098 \uc5f4\ud55c \ubc88\uc9f8 \uc5d0\ud53c\uc18c\ub4dc\uc778 \u3008Wizard\u3009\ubd80\ud130\ub294 2\uac1c\uc758 \uc5d0\ud53c\uc18c\ub4dc\uac00 \uc544\ub2cc 1\uac1c\uc758 \uc5d0\ud53c\uc18c\ub4dc\ub9cc \ubc29\uc601\ud558\uae30\ub85c \ubc14\ub00c\uc5c8\ub2e4."} {"question": "\ud3a0\ub808\uc6b0\uc2a4 \uc655\uc758 \uad81\uc804\uc5d0 \ub3c4\ucc29\ud558\uae30 \uc804 \uc774\uc544\uc190\uc774 \uac15\uc5d0\uc11c \uc783\uc5b4\ubc84\ub9b0 \uac83\uc740?", "paragraph": "\uc544\ud3f4\ub85c\ub2c8\uc624\uc2a4 \ub85c\ub514\uc624\uc2a4(\uc11c\uc0ac \uc2dc\uc778, \ucca0\ud559\uc790, \uc54c\ub809\uc0b0\ub4dc\ub9ac\uc544 \ub3c4\uc11c\uad00 \uac10\ub3c5)\uc758 \u300a\uc544\ub974\uace0\ub098\uc6b0\ud2f0\uce74\u300b\ub294 \uc720\uc77c\ud558\uac8c \ub0a8\uc544\uc788\ub294 \ud5ec\ub808\ub2c8\uc998 \uc11c\uc0ac\uc2dc\ub85c\uc368, \uc774\uc544\uc190\uacfc \uc544\ub974\uace0\ub098\uc6b0\ud0c0\uc774\uac00 \ud669\uae08 \uc591\ubaa8\ub97c \ucc3e\uae30 \uc704\ud574 \uc2e0\ud654 \uc0c1\uc758 \uc9c0\uc5ed\uc778 \ucf5c\ud0a4\uc2a4\ub85c \ud56d\ud574\ub97c \ub5a0\ub098\ub294 \uc2e0\ud654\ub97c \ub2e4\ub8e8\uace0 \uc788\ub2e4. \u300a\uc544\ub974\uace0\ub098\uc6b0\ud2f0\uce74\u300b\uc5d0\uc11c \ud55c \ucabd\uc5d0\ub9cc \uc0cc\ub2ec\uc744 \uc2e0\uc740 \ub0a8\uc790\uac00 \uc790\uc2e0\uc758 \uc790\ub9ac\ub97c \uc704\ud611\ud560 \ub124\uba54\uc2dc\uc2a4\ub77c\ub294 \uc2e0\ud0c1\uc744 \ubc1b\uc740 \ud3a0\ub9ac\uc544\uc2a4 \uc655\uc740 \uc774\uc544\uc190\uc5d0\uac8c \uac15\uc81c\ub85c \uc784\ubb34\ub97c \ubd80\uc5ec\ud55c\ub2e4. \uc774\uc544\uc190\uc740 \uac15\uc5d0\uc11c \uc0cc\ub2ec\uc744 \uc783\uc5b4\ubc84\ub9b0 \ucc44\ub85c \ud3a0\ub9ac\uc544\uc2a4\uc758 \uad81\uc804\uc5d0 \ub3c4\ucc29\ud558\uace0, \uc774\ub54c\ubd80\ud130 \uc11c\uc0ac\uc2dc\uac00 \ubcf8\uaca9\uc801\uc73c\ub85c \uc2dc\uc791\ub41c\ub2e4. \ud5e4\ub77c\ud074\ub808\uc2a4\ubfd0\ub9cc \uc544\ub2c8\ub77c \uac70\uc758 \ubaa8\ub4e0 \ucc28\uc138\ub300 \uc601\uc6c5\ub4e4\uc774 \uc774\uc544\uc190\uacfc \ud568\uaed8 \uc544\ub974\uace0 \ud638\ub97c \ud0c0\uace0 \ud669\uae08 \uc591\ubaa8\ub97c \ucc3e\uc73c\ub7ec\uac00\ub294 \ubaa8\ud5d8\uc5d0 \uac00\ub2f4\ud558\uc600\ub2e4. \uc774 \uc138\ub300\uc5d0\ub294 \ud06c\ub808\ud0c0\uc5d0\uc11c \ubbf8\ub178\ud0c0\uc6b0\ub85c\uc2a4\ub97c \ubb34\ucc0c\ub978 \ud14c\uc138\uc6b0\uc2a4, \uc5ec\uac78 \uc544\ud0c8\ub780\ud14c, \ud55c\ub54c \u300a\uc77c\ub9ac\uc544\uc2a4\u300b\uc640 \u300a\uc624\ub514\uc138\uc774\uc544\u300b\uc640 \uacbd\uc7c1\ud558\ub358 \uc11c\uc0ac\uc2dc\uc758 \uc8fc\uc778\uacf5 \uba5c\ub808\uc544\uadf8\ub85c\uc2a4\ub3c4 \uc788\uc5c8\ub2e4. \ud540\ub2e4\ub85c\uc2a4, \uc544\ud3f4\ub85c\ub2c8\uc6b0\uc2a4, \uc544\ud3f4\ub85c\ub3c4\ub85c\uc2a4\ub294 \uc544\ub974\uace0\ub098\uc6b0\ud0c0\uc774\uc758 \uc804\uccb4 \ubaa9\ub85d\uc744 \ud30c\uc545\ud558\uace0\uc790 \ud558\uc600\ub2e4.", "answer": "\uc0cc\ub2ec", "sentence": "\u300a\uc544\ub974\uace0\ub098\uc6b0\ud2f0\uce74\u300b\uc5d0\uc11c \ud55c \ucabd\uc5d0\ub9cc \uc0cc\ub2ec \uc744 \uc2e0\uc740 \ub0a8\uc790\uac00 \uc790\uc2e0\uc758 \uc790\ub9ac\ub97c \uc704\ud611\ud560 \ub124\uba54\uc2dc\uc2a4\ub77c\ub294 \uc2e0\ud0c1\uc744 \ubc1b\uc740 \ud3a0\ub9ac\uc544\uc2a4 \uc655\uc740 \uc774\uc544\uc190\uc5d0\uac8c \uac15\uc81c\ub85c \uc784\ubb34\ub97c \ubd80\uc5ec\ud55c\ub2e4.", "paragraph_sentence": "\uc544\ud3f4\ub85c\ub2c8\uc624\uc2a4 \ub85c\ub514\uc624\uc2a4(\uc11c\uc0ac \uc2dc\uc778, \ucca0\ud559\uc790, \uc54c\ub809\uc0b0\ub4dc\ub9ac\uc544 \ub3c4\uc11c\uad00 \uac10\ub3c5)\uc758 \u300a\uc544\ub974\uace0\ub098\uc6b0\ud2f0\uce74\u300b\ub294 \uc720\uc77c\ud558\uac8c \ub0a8\uc544\uc788\ub294 \ud5ec\ub808\ub2c8\uc998 \uc11c\uc0ac\uc2dc\ub85c\uc368, \uc774\uc544\uc190\uacfc \uc544\ub974\uace0\ub098\uc6b0\ud0c0\uc774\uac00 \ud669\uae08 \uc591\ubaa8\ub97c \ucc3e\uae30 \uc704\ud574 \uc2e0\ud654 \uc0c1\uc758 \uc9c0\uc5ed\uc778 \ucf5c\ud0a4\uc2a4\ub85c \ud56d\ud574\ub97c \ub5a0\ub098\ub294 \uc2e0\ud654\ub97c \ub2e4\ub8e8\uace0 \uc788\ub2e4. \u300a\uc544\ub974\uace0\ub098\uc6b0\ud2f0\uce74\u300b\uc5d0\uc11c \ud55c \ucabd\uc5d0\ub9cc \uc0cc\ub2ec \uc744 \uc2e0\uc740 \ub0a8\uc790\uac00 \uc790\uc2e0\uc758 \uc790\ub9ac\ub97c \uc704\ud611\ud560 \ub124\uba54\uc2dc\uc2a4\ub77c\ub294 \uc2e0\ud0c1\uc744 \ubc1b\uc740 \ud3a0\ub9ac\uc544\uc2a4 \uc655\uc740 \uc774\uc544\uc190\uc5d0\uac8c \uac15\uc81c\ub85c \uc784\ubb34\ub97c \ubd80\uc5ec\ud55c\ub2e4. \uc774\uc544\uc190\uc740 \uac15\uc5d0\uc11c \uc0cc\ub2ec\uc744 \uc783\uc5b4\ubc84\ub9b0 \ucc44\ub85c \ud3a0\ub9ac\uc544\uc2a4\uc758 \uad81\uc804\uc5d0 \ub3c4\ucc29\ud558\uace0, \uc774\ub54c\ubd80\ud130 \uc11c\uc0ac\uc2dc\uac00 \ubcf8\uaca9\uc801\uc73c\ub85c \uc2dc\uc791\ub41c\ub2e4. \ud5e4\ub77c\ud074\ub808\uc2a4\ubfd0\ub9cc \uc544\ub2c8\ub77c \uac70\uc758 \ubaa8\ub4e0 \ucc28\uc138\ub300 \uc601\uc6c5\ub4e4\uc774 \uc774\uc544\uc190\uacfc \ud568\uaed8 \uc544\ub974\uace0 \ud638\ub97c \ud0c0\uace0 \ud669\uae08 \uc591\ubaa8\ub97c \ucc3e\uc73c\ub7ec\uac00\ub294 \ubaa8\ud5d8\uc5d0 \uac00\ub2f4\ud558\uc600\ub2e4. \uc774 \uc138\ub300\uc5d0\ub294 \ud06c\ub808\ud0c0\uc5d0\uc11c \ubbf8\ub178\ud0c0\uc6b0\ub85c\uc2a4\ub97c \ubb34\ucc0c\ub978 \ud14c\uc138\uc6b0\uc2a4, \uc5ec\uac78 \uc544\ud0c8\ub780\ud14c, \ud55c\ub54c \u300a\uc77c\ub9ac\uc544\uc2a4\u300b\uc640 \u300a\uc624\ub514\uc138\uc774\uc544\u300b\uc640 \uacbd\uc7c1\ud558\ub358 \uc11c\uc0ac\uc2dc\uc758 \uc8fc\uc778\uacf5 \uba5c\ub808\uc544\uadf8\ub85c\uc2a4\ub3c4 \uc788\uc5c8\ub2e4. \ud540\ub2e4\ub85c\uc2a4, \uc544\ud3f4\ub85c\ub2c8\uc6b0\uc2a4, \uc544\ud3f4\ub85c\ub3c4\ub85c\uc2a4\ub294 \uc544\ub974\uace0\ub098\uc6b0\ud0c0\uc774\uc758 \uc804\uccb4 \ubaa9\ub85d\uc744 \ud30c\uc545\ud558\uace0\uc790 \ud558\uc600\ub2e4.", "paragraph_answer": "\uc544\ud3f4\ub85c\ub2c8\uc624\uc2a4 \ub85c\ub514\uc624\uc2a4(\uc11c\uc0ac \uc2dc\uc778, \ucca0\ud559\uc790, \uc54c\ub809\uc0b0\ub4dc\ub9ac\uc544 \ub3c4\uc11c\uad00 \uac10\ub3c5)\uc758 \u300a\uc544\ub974\uace0\ub098\uc6b0\ud2f0\uce74\u300b\ub294 \uc720\uc77c\ud558\uac8c \ub0a8\uc544\uc788\ub294 \ud5ec\ub808\ub2c8\uc998 \uc11c\uc0ac\uc2dc\ub85c\uc368, \uc774\uc544\uc190\uacfc \uc544\ub974\uace0\ub098\uc6b0\ud0c0\uc774\uac00 \ud669\uae08 \uc591\ubaa8\ub97c \ucc3e\uae30 \uc704\ud574 \uc2e0\ud654 \uc0c1\uc758 \uc9c0\uc5ed\uc778 \ucf5c\ud0a4\uc2a4\ub85c \ud56d\ud574\ub97c \ub5a0\ub098\ub294 \uc2e0\ud654\ub97c \ub2e4\ub8e8\uace0 \uc788\ub2e4. \u300a\uc544\ub974\uace0\ub098\uc6b0\ud2f0\uce74\u300b\uc5d0\uc11c \ud55c \ucabd\uc5d0\ub9cc \uc0cc\ub2ec \uc744 \uc2e0\uc740 \ub0a8\uc790\uac00 \uc790\uc2e0\uc758 \uc790\ub9ac\ub97c \uc704\ud611\ud560 \ub124\uba54\uc2dc\uc2a4\ub77c\ub294 \uc2e0\ud0c1\uc744 \ubc1b\uc740 \ud3a0\ub9ac\uc544\uc2a4 \uc655\uc740 \uc774\uc544\uc190\uc5d0\uac8c \uac15\uc81c\ub85c \uc784\ubb34\ub97c \ubd80\uc5ec\ud55c\ub2e4. \uc774\uc544\uc190\uc740 \uac15\uc5d0\uc11c \uc0cc\ub2ec\uc744 \uc783\uc5b4\ubc84\ub9b0 \ucc44\ub85c \ud3a0\ub9ac\uc544\uc2a4\uc758 \uad81\uc804\uc5d0 \ub3c4\ucc29\ud558\uace0, \uc774\ub54c\ubd80\ud130 \uc11c\uc0ac\uc2dc\uac00 \ubcf8\uaca9\uc801\uc73c\ub85c \uc2dc\uc791\ub41c\ub2e4. \ud5e4\ub77c\ud074\ub808\uc2a4\ubfd0\ub9cc \uc544\ub2c8\ub77c \uac70\uc758 \ubaa8\ub4e0 \ucc28\uc138\ub300 \uc601\uc6c5\ub4e4\uc774 \uc774\uc544\uc190\uacfc \ud568\uaed8 \uc544\ub974\uace0 \ud638\ub97c \ud0c0\uace0 \ud669\uae08 \uc591\ubaa8\ub97c \ucc3e\uc73c\ub7ec\uac00\ub294 \ubaa8\ud5d8\uc5d0 \uac00\ub2f4\ud558\uc600\ub2e4. \uc774 \uc138\ub300\uc5d0\ub294 \ud06c\ub808\ud0c0\uc5d0\uc11c \ubbf8\ub178\ud0c0\uc6b0\ub85c\uc2a4\ub97c \ubb34\ucc0c\ub978 \ud14c\uc138\uc6b0\uc2a4, \uc5ec\uac78 \uc544\ud0c8\ub780\ud14c, \ud55c\ub54c \u300a\uc77c\ub9ac\uc544\uc2a4\u300b\uc640 \u300a\uc624\ub514\uc138\uc774\uc544\u300b\uc640 \uacbd\uc7c1\ud558\ub358 \uc11c\uc0ac\uc2dc\uc758 \uc8fc\uc778\uacf5 \uba5c\ub808\uc544\uadf8\ub85c\uc2a4\ub3c4 \uc788\uc5c8\ub2e4. \ud540\ub2e4\ub85c\uc2a4, \uc544\ud3f4\ub85c\ub2c8\uc6b0\uc2a4, \uc544\ud3f4\ub85c\ub3c4\ub85c\uc2a4\ub294 \uc544\ub974\uace0\ub098\uc6b0\ud0c0\uc774\uc758 \uc804\uccb4 \ubaa9\ub85d\uc744 \ud30c\uc545\ud558\uace0\uc790 \ud558\uc600\ub2e4.", "sentence_answer": "\u300a\uc544\ub974\uace0\ub098\uc6b0\ud2f0\uce74\u300b\uc5d0\uc11c \ud55c \ucabd\uc5d0\ub9cc \uc0cc\ub2ec \uc744 \uc2e0\uc740 \ub0a8\uc790\uac00 \uc790\uc2e0\uc758 \uc790\ub9ac\ub97c \uc704\ud611\ud560 \ub124\uba54\uc2dc\uc2a4\ub77c\ub294 \uc2e0\ud0c1\uc744 \ubc1b\uc740 \ud3a0\ub9ac\uc544\uc2a4 \uc655\uc740 \uc774\uc544\uc190\uc5d0\uac8c \uac15\uc81c\ub85c \uc784\ubb34\ub97c \ubd80\uc5ec\ud55c\ub2e4.", "paragraph_id": "6527377-17-1", "paragraph_question": "question: \ud3a0\ub808\uc6b0\uc2a4 \uc655\uc758 \uad81\uc804\uc5d0 \ub3c4\ucc29\ud558\uae30 \uc804 \uc774\uc544\uc190\uc774 \uac15\uc5d0\uc11c \uc783\uc5b4\ubc84\ub9b0 \uac83\uc740?, context: \uc544\ud3f4\ub85c\ub2c8\uc624\uc2a4 \ub85c\ub514\uc624\uc2a4(\uc11c\uc0ac \uc2dc\uc778, \ucca0\ud559\uc790, \uc54c\ub809\uc0b0\ub4dc\ub9ac\uc544 \ub3c4\uc11c\uad00 \uac10\ub3c5)\uc758 \u300a\uc544\ub974\uace0\ub098\uc6b0\ud2f0\uce74\u300b\ub294 \uc720\uc77c\ud558\uac8c \ub0a8\uc544\uc788\ub294 \ud5ec\ub808\ub2c8\uc998 \uc11c\uc0ac\uc2dc\ub85c\uc368, \uc774\uc544\uc190\uacfc \uc544\ub974\uace0\ub098\uc6b0\ud0c0\uc774\uac00 \ud669\uae08 \uc591\ubaa8\ub97c \ucc3e\uae30 \uc704\ud574 \uc2e0\ud654 \uc0c1\uc758 \uc9c0\uc5ed\uc778 \ucf5c\ud0a4\uc2a4\ub85c \ud56d\ud574\ub97c \ub5a0\ub098\ub294 \uc2e0\ud654\ub97c \ub2e4\ub8e8\uace0 \uc788\ub2e4. \u300a\uc544\ub974\uace0\ub098\uc6b0\ud2f0\uce74\u300b\uc5d0\uc11c \ud55c \ucabd\uc5d0\ub9cc \uc0cc\ub2ec\uc744 \uc2e0\uc740 \ub0a8\uc790\uac00 \uc790\uc2e0\uc758 \uc790\ub9ac\ub97c \uc704\ud611\ud560 \ub124\uba54\uc2dc\uc2a4\ub77c\ub294 \uc2e0\ud0c1\uc744 \ubc1b\uc740 \ud3a0\ub9ac\uc544\uc2a4 \uc655\uc740 \uc774\uc544\uc190\uc5d0\uac8c \uac15\uc81c\ub85c \uc784\ubb34\ub97c \ubd80\uc5ec\ud55c\ub2e4. \uc774\uc544\uc190\uc740 \uac15\uc5d0\uc11c \uc0cc\ub2ec\uc744 \uc783\uc5b4\ubc84\ub9b0 \ucc44\ub85c \ud3a0\ub9ac\uc544\uc2a4\uc758 \uad81\uc804\uc5d0 \ub3c4\ucc29\ud558\uace0, \uc774\ub54c\ubd80\ud130 \uc11c\uc0ac\uc2dc\uac00 \ubcf8\uaca9\uc801\uc73c\ub85c \uc2dc\uc791\ub41c\ub2e4. \ud5e4\ub77c\ud074\ub808\uc2a4\ubfd0\ub9cc \uc544\ub2c8\ub77c \uac70\uc758 \ubaa8\ub4e0 \ucc28\uc138\ub300 \uc601\uc6c5\ub4e4\uc774 \uc774\uc544\uc190\uacfc \ud568\uaed8 \uc544\ub974\uace0 \ud638\ub97c \ud0c0\uace0 \ud669\uae08 \uc591\ubaa8\ub97c \ucc3e\uc73c\ub7ec\uac00\ub294 \ubaa8\ud5d8\uc5d0 \uac00\ub2f4\ud558\uc600\ub2e4. \uc774 \uc138\ub300\uc5d0\ub294 \ud06c\ub808\ud0c0\uc5d0\uc11c \ubbf8\ub178\ud0c0\uc6b0\ub85c\uc2a4\ub97c \ubb34\ucc0c\ub978 \ud14c\uc138\uc6b0\uc2a4, \uc5ec\uac78 \uc544\ud0c8\ub780\ud14c, \ud55c\ub54c \u300a\uc77c\ub9ac\uc544\uc2a4\u300b\uc640 \u300a\uc624\ub514\uc138\uc774\uc544\u300b\uc640 \uacbd\uc7c1\ud558\ub358 \uc11c\uc0ac\uc2dc\uc758 \uc8fc\uc778\uacf5 \uba5c\ub808\uc544\uadf8\ub85c\uc2a4\ub3c4 \uc788\uc5c8\ub2e4. \ud540\ub2e4\ub85c\uc2a4, \uc544\ud3f4\ub85c\ub2c8\uc6b0\uc2a4, \uc544\ud3f4\ub85c\ub3c4\ub85c\uc2a4\ub294 \uc544\ub974\uace0\ub098\uc6b0\ud0c0\uc774\uc758 \uc804\uccb4 \ubaa9\ub85d\uc744 \ud30c\uc545\ud558\uace0\uc790 \ud558\uc600\ub2e4."} {"question": "\ub77c \ub77c\ub79c\ub4dc\uc758 \uac10\ub3c5\uc740?", "paragraph": "\uc2a4\ud1a4\uc740 \uace0\uc2ac\ub9c1\uacfc 3\ubc88\uc9f8 \ud568\uaed8\ud558\uace0 \ub370\uc774\ubbf8\uc5b8 \uc154\uc824\uc774 \uac10\ub3c5\ud55c \ubba4\uc9c0\uceec \ucf54\ubbf8\ub514 \ub4dc\ub77c\ub9c8 \uc601\ud654 \u300a\ub77c \ub77c\ub79c\ub4dc\u300b\uc5d0\uc11c \ub85c\uc2a4 \uc564\uc824\ub808\uc2a4\uc5d0 \uc0ac\ub294 \uc57c\uc2ec\ucc2c \uc5ec\ubc30\uc6b0 \ubbf8\uc544 \ub3cc\ub780 \uc5ed\ud560\uc744 \uc5f0\uae30\ud588\ub2e4. \uc2a4\ud1a4\uc740 \uadf8\ub140\uc758 \uc131\uaca9\uc5d0 \ub300\ud55c \uba87 \uac00\uc9c0 \uc2e4\uc0dd\ud65c \uacbd\ud5d8\uc744 \ube4c\ub838\uace0, \uc900\ube44 \uacfc\uc815\uc5d0\uc11c \u300a\uc170\ub974\ubd80\ub974\uc758 \uc6b0\uc0b0\u300b\uacfc \ud504\ub808\ub4dc \uc544\uc2a4\ud14c\uc5b4\uc640 \uc9c4\uc800 \ub85c\uc800\uc2a4\uc758 \uc601\ud654\ub97c \ubcf4\uc558\ub2e4. \uadf8\ub140\ub294 \ubba4\uc9c0\uceec\uc758 \uc0ac\uc6b4\ub4dc\ud2b8\ub799\uc744 \uc704\ud574 6\uace1 \"Another Day of Sun\", \"City of Stars\", \"Someone in the Crowd\", \"A Lovely Night\", \"Audition (The Fools Who Dream)\"\uc640 \"City of Stars (Humming)\"\uc744 \ub179\uc74c\ud588\ub2e4. \uc601\ud654\ub294 2016\ub144 \ubca0\ub124\uce58\uc544 \uad6d\uc81c \uc601\ud654\uc81c\uc5d0\uc11c \uac1c\ub9c9\uc791\uc73c\ub85c \uc120\uc815 \ub418\uc5b4 \ub9e4\uc6b0 \uae0d\uc815\uc801\uc778 \ud3c9\uac00\ub97c \ubc1b\uc558\uc73c\uba70, \uc2a4\ud1a4\uc740 \ubcfc\ud53c\ucef5 \uc5ec\uc6b0\uc8fc\uc5f0\uc0c1\uc744 \uc218\uc0c1\ud588\ub2e4. \uadf8\ub140\ub294 \ub610\ud55c \ubbf8\uad6d \uc544\uce74\ub370\ubbf8\uc0c1, \uc601\uad6d \uc544\uce74\ub370\ubbf8\uc0c1(BAFTA), \uace8\ub4e0 \uae00\ub85c\ube0c\uc0c1\uc640 \ubbf8\uad6d \ubc30\uc6b0 \uc870\ud569\uc0c1(SAG) \uc5ec\uc6b0\uc8fc\uc5f0\uc0c1\uc744 \uc218\uc0c1\ud558\uc600\ub2e4. \uc601\ud654\ub294 \ubc15\uc2a4\uc624\ud53c\uc2a4\uc5d0\uc11c\ub3c4 3,000\ub9cc \ub2ec\ub7ec\uc758 \uc81c\uc791\ube44\ub97c \ub4e4\uc5ec 4\uc5b5 4,000\ub9cc \ub2ec\ub7ec\ub77c\ub294 \ub192\uc740 \ud765\ud589 \uc218\uc775\uc744 \uac70\ub480\ub2e4.", "answer": "\ub370\uc774\ubbf8\uc5b8 \uc154\uc824", "sentence": "\uc2a4\ud1a4\uc740 \uace0\uc2ac\ub9c1\uacfc 3\ubc88\uc9f8 \ud568\uaed8\ud558\uace0 \ub370\uc774\ubbf8\uc5b8 \uc154\uc824 \uc774 \uac10\ub3c5\ud55c \ubba4\uc9c0\uceec \ucf54\ubbf8\ub514 \ub4dc\ub77c\ub9c8 \uc601\ud654 \u300a\ub77c \ub77c\ub79c\ub4dc\u300b\uc5d0\uc11c \ub85c\uc2a4 \uc564\uc824\ub808\uc2a4\uc5d0 \uc0ac\ub294 \uc57c\uc2ec\ucc2c \uc5ec\ubc30\uc6b0 \ubbf8\uc544 \ub3cc\ub780 \uc5ed\ud560\uc744 \uc5f0\uae30\ud588\ub2e4.", "paragraph_sentence": " \uc2a4\ud1a4\uc740 \uace0\uc2ac\ub9c1\uacfc 3\ubc88\uc9f8 \ud568\uaed8\ud558\uace0 \ub370\uc774\ubbf8\uc5b8 \uc154\uc824 \uc774 \uac10\ub3c5\ud55c \ubba4\uc9c0\uceec \ucf54\ubbf8\ub514 \ub4dc\ub77c\ub9c8 \uc601\ud654 \u300a\ub77c \ub77c\ub79c\ub4dc\u300b\uc5d0\uc11c \ub85c\uc2a4 \uc564\uc824\ub808\uc2a4\uc5d0 \uc0ac\ub294 \uc57c\uc2ec\ucc2c \uc5ec\ubc30\uc6b0 \ubbf8\uc544 \ub3cc\ub780 \uc5ed\ud560\uc744 \uc5f0\uae30\ud588\ub2e4. \uc2a4\ud1a4\uc740 \uadf8\ub140\uc758 \uc131\uaca9\uc5d0 \ub300\ud55c \uba87 \uac00\uc9c0 \uc2e4\uc0dd\ud65c \uacbd\ud5d8\uc744 \ube4c\ub838\uace0, \uc900\ube44 \uacfc\uc815\uc5d0\uc11c \u300a\uc170\ub974\ubd80\ub974\uc758 \uc6b0\uc0b0\u300b\uacfc \ud504\ub808\ub4dc \uc544\uc2a4\ud14c\uc5b4\uc640 \uc9c4\uc800 \ub85c\uc800\uc2a4\uc758 \uc601\ud654\ub97c \ubcf4\uc558\ub2e4. \uadf8\ub140\ub294 \ubba4\uc9c0\uceec\uc758 \uc0ac\uc6b4\ub4dc\ud2b8\ub799\uc744 \uc704\ud574 6\uace1 \"Another Day of Sun\", \"City of Stars\", \"Someone in the Crowd\", \"A Lovely Night\", \"Audition (The Fools Who Dream)\"\uc640 \"City of Stars (Humming)\"\uc744 \ub179\uc74c\ud588\ub2e4. \uc601\ud654\ub294 2016\ub144 \ubca0\ub124\uce58\uc544 \uad6d\uc81c \uc601\ud654\uc81c\uc5d0\uc11c \uac1c\ub9c9\uc791\uc73c\ub85c \uc120\uc815 \ub418\uc5b4 \ub9e4\uc6b0 \uae0d\uc815\uc801\uc778 \ud3c9\uac00\ub97c \ubc1b\uc558\uc73c\uba70, \uc2a4\ud1a4\uc740 \ubcfc\ud53c\ucef5 \uc5ec\uc6b0\uc8fc\uc5f0\uc0c1\uc744 \uc218\uc0c1\ud588\ub2e4. \uadf8\ub140\ub294 \ub610\ud55c \ubbf8\uad6d \uc544\uce74\ub370\ubbf8\uc0c1, \uc601\uad6d \uc544\uce74\ub370\ubbf8\uc0c1(BAFTA), \uace8\ub4e0 \uae00\ub85c\ube0c\uc0c1\uc640 \ubbf8\uad6d \ubc30\uc6b0 \uc870\ud569\uc0c1(SAG) \uc5ec\uc6b0\uc8fc\uc5f0\uc0c1\uc744 \uc218\uc0c1\ud558\uc600\ub2e4. \uc601\ud654\ub294 \ubc15\uc2a4\uc624\ud53c\uc2a4\uc5d0\uc11c\ub3c4 3,000\ub9cc \ub2ec\ub7ec\uc758 \uc81c\uc791\ube44\ub97c \ub4e4\uc5ec 4\uc5b5 4,000\ub9cc \ub2ec\ub7ec\ub77c\ub294 \ub192\uc740 \ud765\ud589 \uc218\uc775\uc744 \uac70\ub480\ub2e4.", "paragraph_answer": "\uc2a4\ud1a4\uc740 \uace0\uc2ac\ub9c1\uacfc 3\ubc88\uc9f8 \ud568\uaed8\ud558\uace0 \ub370\uc774\ubbf8\uc5b8 \uc154\uc824 \uc774 \uac10\ub3c5\ud55c \ubba4\uc9c0\uceec \ucf54\ubbf8\ub514 \ub4dc\ub77c\ub9c8 \uc601\ud654 \u300a\ub77c \ub77c\ub79c\ub4dc\u300b\uc5d0\uc11c \ub85c\uc2a4 \uc564\uc824\ub808\uc2a4\uc5d0 \uc0ac\ub294 \uc57c\uc2ec\ucc2c \uc5ec\ubc30\uc6b0 \ubbf8\uc544 \ub3cc\ub780 \uc5ed\ud560\uc744 \uc5f0\uae30\ud588\ub2e4. \uc2a4\ud1a4\uc740 \uadf8\ub140\uc758 \uc131\uaca9\uc5d0 \ub300\ud55c \uba87 \uac00\uc9c0 \uc2e4\uc0dd\ud65c \uacbd\ud5d8\uc744 \ube4c\ub838\uace0, \uc900\ube44 \uacfc\uc815\uc5d0\uc11c \u300a\uc170\ub974\ubd80\ub974\uc758 \uc6b0\uc0b0\u300b\uacfc \ud504\ub808\ub4dc \uc544\uc2a4\ud14c\uc5b4\uc640 \uc9c4\uc800 \ub85c\uc800\uc2a4\uc758 \uc601\ud654\ub97c \ubcf4\uc558\ub2e4. \uadf8\ub140\ub294 \ubba4\uc9c0\uceec\uc758 \uc0ac\uc6b4\ub4dc\ud2b8\ub799\uc744 \uc704\ud574 6\uace1 \"Another Day of Sun\", \"City of Stars\", \"Someone in the Crowd\", \"A Lovely Night\", \"Audition (The Fools Who Dream)\"\uc640 \"City of Stars (Humming)\"\uc744 \ub179\uc74c\ud588\ub2e4. \uc601\ud654\ub294 2016\ub144 \ubca0\ub124\uce58\uc544 \uad6d\uc81c \uc601\ud654\uc81c\uc5d0\uc11c \uac1c\ub9c9\uc791\uc73c\ub85c \uc120\uc815 \ub418\uc5b4 \ub9e4\uc6b0 \uae0d\uc815\uc801\uc778 \ud3c9\uac00\ub97c \ubc1b\uc558\uc73c\uba70, \uc2a4\ud1a4\uc740 \ubcfc\ud53c\ucef5 \uc5ec\uc6b0\uc8fc\uc5f0\uc0c1\uc744 \uc218\uc0c1\ud588\ub2e4. \uadf8\ub140\ub294 \ub610\ud55c \ubbf8\uad6d \uc544\uce74\ub370\ubbf8\uc0c1, \uc601\uad6d \uc544\uce74\ub370\ubbf8\uc0c1(BAFTA), \uace8\ub4e0 \uae00\ub85c\ube0c\uc0c1\uc640 \ubbf8\uad6d \ubc30\uc6b0 \uc870\ud569\uc0c1(SAG) \uc5ec\uc6b0\uc8fc\uc5f0\uc0c1\uc744 \uc218\uc0c1\ud558\uc600\ub2e4. \uc601\ud654\ub294 \ubc15\uc2a4\uc624\ud53c\uc2a4\uc5d0\uc11c\ub3c4 3,000\ub9cc \ub2ec\ub7ec\uc758 \uc81c\uc791\ube44\ub97c \ub4e4\uc5ec 4\uc5b5 4,000\ub9cc \ub2ec\ub7ec\ub77c\ub294 \ub192\uc740 \ud765\ud589 \uc218\uc775\uc744 \uac70\ub480\ub2e4.", "sentence_answer": "\uc2a4\ud1a4\uc740 \uace0\uc2ac\ub9c1\uacfc 3\ubc88\uc9f8 \ud568\uaed8\ud558\uace0 \ub370\uc774\ubbf8\uc5b8 \uc154\uc824 \uc774 \uac10\ub3c5\ud55c \ubba4\uc9c0\uceec \ucf54\ubbf8\ub514 \ub4dc\ub77c\ub9c8 \uc601\ud654 \u300a\ub77c \ub77c\ub79c\ub4dc\u300b\uc5d0\uc11c \ub85c\uc2a4 \uc564\uc824\ub808\uc2a4\uc5d0 \uc0ac\ub294 \uc57c\uc2ec\ucc2c \uc5ec\ubc30\uc6b0 \ubbf8\uc544 \ub3cc\ub780 \uc5ed\ud560\uc744 \uc5f0\uae30\ud588\ub2e4.", "paragraph_id": "6523707-9-2", "paragraph_question": "question: \ub77c \ub77c\ub79c\ub4dc\uc758 \uac10\ub3c5\uc740?, context: \uc2a4\ud1a4\uc740 \uace0\uc2ac\ub9c1\uacfc 3\ubc88\uc9f8 \ud568\uaed8\ud558\uace0 \ub370\uc774\ubbf8\uc5b8 \uc154\uc824\uc774 \uac10\ub3c5\ud55c \ubba4\uc9c0\uceec \ucf54\ubbf8\ub514 \ub4dc\ub77c\ub9c8 \uc601\ud654 \u300a\ub77c \ub77c\ub79c\ub4dc\u300b\uc5d0\uc11c \ub85c\uc2a4 \uc564\uc824\ub808\uc2a4\uc5d0 \uc0ac\ub294 \uc57c\uc2ec\ucc2c \uc5ec\ubc30\uc6b0 \ubbf8\uc544 \ub3cc\ub780 \uc5ed\ud560\uc744 \uc5f0\uae30\ud588\ub2e4. \uc2a4\ud1a4\uc740 \uadf8\ub140\uc758 \uc131\uaca9\uc5d0 \ub300\ud55c \uba87 \uac00\uc9c0 \uc2e4\uc0dd\ud65c \uacbd\ud5d8\uc744 \ube4c\ub838\uace0, \uc900\ube44 \uacfc\uc815\uc5d0\uc11c \u300a\uc170\ub974\ubd80\ub974\uc758 \uc6b0\uc0b0\u300b\uacfc \ud504\ub808\ub4dc \uc544\uc2a4\ud14c\uc5b4\uc640 \uc9c4\uc800 \ub85c\uc800\uc2a4\uc758 \uc601\ud654\ub97c \ubcf4\uc558\ub2e4. \uadf8\ub140\ub294 \ubba4\uc9c0\uceec\uc758 \uc0ac\uc6b4\ub4dc\ud2b8\ub799\uc744 \uc704\ud574 6\uace1 \"Another Day of Sun\", \"City of Stars\", \"Someone in the Crowd\", \"A Lovely Night\", \"Audition (The Fools Who Dream)\"\uc640 \"City of Stars (Humming)\"\uc744 \ub179\uc74c\ud588\ub2e4. \uc601\ud654\ub294 2016\ub144 \ubca0\ub124\uce58\uc544 \uad6d\uc81c \uc601\ud654\uc81c\uc5d0\uc11c \uac1c\ub9c9\uc791\uc73c\ub85c \uc120\uc815 \ub418\uc5b4 \ub9e4\uc6b0 \uae0d\uc815\uc801\uc778 \ud3c9\uac00\ub97c \ubc1b\uc558\uc73c\uba70, \uc2a4\ud1a4\uc740 \ubcfc\ud53c\ucef5 \uc5ec\uc6b0\uc8fc\uc5f0\uc0c1\uc744 \uc218\uc0c1\ud588\ub2e4. \uadf8\ub140\ub294 \ub610\ud55c \ubbf8\uad6d \uc544\uce74\ub370\ubbf8\uc0c1, \uc601\uad6d \uc544\uce74\ub370\ubbf8\uc0c1(BAFTA), \uace8\ub4e0 \uae00\ub85c\ube0c\uc0c1\uc640 \ubbf8\uad6d \ubc30\uc6b0 \uc870\ud569\uc0c1(SAG) \uc5ec\uc6b0\uc8fc\uc5f0\uc0c1\uc744 \uc218\uc0c1\ud558\uc600\ub2e4. \uc601\ud654\ub294 \ubc15\uc2a4\uc624\ud53c\uc2a4\uc5d0\uc11c\ub3c4 3,000\ub9cc \ub2ec\ub7ec\uc758 \uc81c\uc791\ube44\ub97c \ub4e4\uc5ec 4\uc5b5 4,000\ub9cc \ub2ec\ub7ec\ub77c\ub294 \ub192\uc740 \ud765\ud589 \uc218\uc775\uc744 \uac70\ub480\ub2e4."} {"question": "17\ud3b8\uc131\uc744 \uacf5\uc81c\ud558\uba74 211\uacc4\uc758 \uc789\uc5ec\ubd84\uc740 \uc5bc\ub9c8\uc778\uac00?", "paragraph": "\uadf8\ub9b0\ucc28 \uc5f0\uacb0\uc758 \ud3b8\uc131\uc740 10\ub7c9\u00d717\ud3b8\uc131(170\ub7c9)\uc774\uba70 3000\ubc88\ub300 34\uac1c\uac00 \uadf8\ub9b0\ucc28 \uc870\uc785\ud3b8\uc131\uc5d0 \uc0ac\uc6a9\ub418\uc5b4 \ub354 \ucd94\uac00\ud558\uc5ec \ubd80\uc18d\ud3b8\uc131 \ubd84\uc758 17\ud3b8\uc131\uc744 \uacf5\uc81c\ud558\uba74 211\uacc4\uc758 \uc789\uc5ec\ubd84\uc774 \uc815\ud655\ud788 \uc774\ubc88 \uc99d\ube44\ubd84\uc758 E231\uacc4\uc640 \uac19\uc740 \uc218\uc778 110\ub7c9(5\ub7c9\u00d722\ud3b8\uc131)\uc774 \ub41c\ub2e4. \uc774\ub7ec\ud55c \ud3b8\uc131 \uac00\uc6b4\ub370 70\ub7c9(5\ub7c9\u00d714\ud3b8\uc131)\uc740 \ud574\uc548\uac00\ub97c \uc8fc\ud589\ud558\uc5ec \uc5fc\ud574\uc5d0 \uc758\ud55c \ubd80\uc2dd\ub4f1\uc774 \uc9c4\ud589\ub418\uace0 \uc788\ub294 \ubcf4\uc18c \uc9c0\uad6c \uac01 \ub178\uc120\uc758 113\uacc4 \uad50\uccb4\uc6a9\uc73c\ub85c \ub9c8\ucfe0\ud558\ub9ac \ucc28\ub7c9\uc13c\ud130\uc5d0 \uc804\uc6a9\ud558\uac8c \ub418\uc5c8\ub2e4. \ub098\uba38\uc9c0 40\ub7c9\uc740 \uc6b0\uc4f0\ub178\ubbf8\uc57c \uc120\u00b7\ub2e4\uce74\uc0ac\ud0a4 \uc120\uc758 \uc218\uc1a1\ub825 \uc99d\uac15\ubd84\uc5d0 \ud22c\uc785\ud560 \uc218 \uc788\uac8c \ub418\uc5c8\ub2e4. 2006\ub144 7\uc6d4 \ub2e4\uc774\uc5b4 \uac1c\uc815\uc5d0\uc11c\ub294 \uc6b0\uc5d0\ub178\uad6c\uc758 15\ub7c9 \uc6b4\uc6a9\uc774 \uc99d\uac00\ud588\uae30 \ub54c\ubb38\uc5d0 \uc6b0\uc4f0\ub178\ubbf8\uc57c \uc5ed - \ucfe0\ub85c\uc774\uc18c \uc5ed \uad6c\uac04(\uc77c\ubd80 \uace0\uac00\ub124\uc774\uc5ed - \uad6c\ub85c\uc774\uc18c\uc5ed \uad6c\uac04)\uc5d0\uc11c \uc6b4\uc6a9\ub418\uace0 \uc788\ub358 E231\uacc4 5\ub7c9 \uc6b4\uc6a9\uc774 211\uacc4 5\ub7c9 \uc6b4\uc6a9(\uc77c\ubd80 5\ub7c9+5\ub7c9 \uc6b4\uc6a9\ub3c4 \uc788\ub2e4)\uc73c\ub85c \ubcc0\uacbd\ub418\uc5c8\ub2e4.", "answer": "110\ub7c9", "sentence": "\uadf8\ub9b0\ucc28 \uc5f0\uacb0\uc758 \ud3b8\uc131\uc740 10\ub7c9\u00d717\ud3b8\uc131(170\ub7c9)\uc774\uba70 3000\ubc88\ub300 34\uac1c\uac00 \uadf8\ub9b0\ucc28 \uc870\uc785\ud3b8\uc131\uc5d0 \uc0ac\uc6a9\ub418\uc5b4 \ub354 \ucd94\uac00\ud558\uc5ec \ubd80\uc18d\ud3b8\uc131 \ubd84\uc758 17\ud3b8\uc131\uc744 \uacf5\uc81c\ud558\uba74 211\uacc4\uc758 \uc789\uc5ec\ubd84\uc774 \uc815\ud655\ud788 \uc774\ubc88 \uc99d\ube44\ubd84\uc758 E231\uacc4\uc640 \uac19\uc740 \uc218\uc778 110\ub7c9 (5\ub7c9\u00d722\ud3b8\uc131)", "paragraph_sentence": " \uadf8\ub9b0\ucc28 \uc5f0\uacb0\uc758 \ud3b8\uc131\uc740 10\ub7c9\u00d717\ud3b8\uc131(170\ub7c9)\uc774\uba70 3000\ubc88\ub300 34\uac1c\uac00 \uadf8\ub9b0\ucc28 \uc870\uc785\ud3b8\uc131\uc5d0 \uc0ac\uc6a9\ub418\uc5b4 \ub354 \ucd94\uac00\ud558\uc5ec \ubd80\uc18d\ud3b8\uc131 \ubd84\uc758 17\ud3b8\uc131\uc744 \uacf5\uc81c\ud558\uba74 211\uacc4\uc758 \uc789\uc5ec\ubd84\uc774 \uc815\ud655\ud788 \uc774\ubc88 \uc99d\ube44\ubd84\uc758 E231\uacc4\uc640 \uac19\uc740 \uc218\uc778 110\ub7c9 (5\ub7c9\u00d722\ud3b8\uc131) \uc774 \ub41c\ub2e4. \uc774\ub7ec\ud55c \ud3b8\uc131 \uac00\uc6b4\ub370 70\ub7c9(5\ub7c9\u00d714\ud3b8\uc131)\uc740 \ud574\uc548\uac00\ub97c \uc8fc\ud589\ud558\uc5ec \uc5fc\ud574\uc5d0 \uc758\ud55c \ubd80\uc2dd\ub4f1\uc774 \uc9c4\ud589\ub418\uace0 \uc788\ub294 \ubcf4\uc18c \uc9c0\uad6c \uac01 \ub178\uc120\uc758 113\uacc4 \uad50\uccb4\uc6a9\uc73c\ub85c \ub9c8\ucfe0\ud558\ub9ac \ucc28\ub7c9\uc13c\ud130\uc5d0 \uc804\uc6a9\ud558\uac8c \ub418\uc5c8\ub2e4. \ub098\uba38\uc9c0 40\ub7c9\uc740 \uc6b0\uc4f0\ub178\ubbf8\uc57c \uc120\u00b7\ub2e4\uce74\uc0ac\ud0a4 \uc120\uc758 \uc218\uc1a1\ub825 \uc99d\uac15\ubd84\uc5d0 \ud22c\uc785\ud560 \uc218 \uc788\uac8c \ub418\uc5c8\ub2e4. 2006\ub144 7\uc6d4 \ub2e4\uc774\uc5b4 \uac1c\uc815\uc5d0\uc11c\ub294 \uc6b0\uc5d0\ub178\uad6c\uc758 15\ub7c9 \uc6b4\uc6a9\uc774 \uc99d\uac00\ud588\uae30 \ub54c\ubb38\uc5d0 \uc6b0\uc4f0\ub178\ubbf8\uc57c \uc5ed - \ucfe0\ub85c\uc774\uc18c \uc5ed \uad6c\uac04(\uc77c\ubd80 \uace0\uac00\ub124\uc774\uc5ed - \uad6c\ub85c\uc774\uc18c\uc5ed \uad6c\uac04)\uc5d0\uc11c \uc6b4\uc6a9\ub418\uace0 \uc788\ub358 E231\uacc4 5\ub7c9 \uc6b4\uc6a9\uc774 211\uacc4 5\ub7c9 \uc6b4\uc6a9(\uc77c\ubd80 5\ub7c9+5\ub7c9 \uc6b4\uc6a9\ub3c4 \uc788\ub2e4)\uc73c\ub85c \ubcc0\uacbd\ub418\uc5c8\ub2e4.", "paragraph_answer": "\uadf8\ub9b0\ucc28 \uc5f0\uacb0\uc758 \ud3b8\uc131\uc740 10\ub7c9\u00d717\ud3b8\uc131(170\ub7c9)\uc774\uba70 3000\ubc88\ub300 34\uac1c\uac00 \uadf8\ub9b0\ucc28 \uc870\uc785\ud3b8\uc131\uc5d0 \uc0ac\uc6a9\ub418\uc5b4 \ub354 \ucd94\uac00\ud558\uc5ec \ubd80\uc18d\ud3b8\uc131 \ubd84\uc758 17\ud3b8\uc131\uc744 \uacf5\uc81c\ud558\uba74 211\uacc4\uc758 \uc789\uc5ec\ubd84\uc774 \uc815\ud655\ud788 \uc774\ubc88 \uc99d\ube44\ubd84\uc758 E231\uacc4\uc640 \uac19\uc740 \uc218\uc778 110\ub7c9 (5\ub7c9\u00d722\ud3b8\uc131)\uc774 \ub41c\ub2e4. \uc774\ub7ec\ud55c \ud3b8\uc131 \uac00\uc6b4\ub370 70\ub7c9(5\ub7c9\u00d714\ud3b8\uc131)\uc740 \ud574\uc548\uac00\ub97c \uc8fc\ud589\ud558\uc5ec \uc5fc\ud574\uc5d0 \uc758\ud55c \ubd80\uc2dd\ub4f1\uc774 \uc9c4\ud589\ub418\uace0 \uc788\ub294 \ubcf4\uc18c \uc9c0\uad6c \uac01 \ub178\uc120\uc758 113\uacc4 \uad50\uccb4\uc6a9\uc73c\ub85c \ub9c8\ucfe0\ud558\ub9ac \ucc28\ub7c9\uc13c\ud130\uc5d0 \uc804\uc6a9\ud558\uac8c \ub418\uc5c8\ub2e4. \ub098\uba38\uc9c0 40\ub7c9\uc740 \uc6b0\uc4f0\ub178\ubbf8\uc57c \uc120\u00b7\ub2e4\uce74\uc0ac\ud0a4 \uc120\uc758 \uc218\uc1a1\ub825 \uc99d\uac15\ubd84\uc5d0 \ud22c\uc785\ud560 \uc218 \uc788\uac8c \ub418\uc5c8\ub2e4. 2006\ub144 7\uc6d4 \ub2e4\uc774\uc5b4 \uac1c\uc815\uc5d0\uc11c\ub294 \uc6b0\uc5d0\ub178\uad6c\uc758 15\ub7c9 \uc6b4\uc6a9\uc774 \uc99d\uac00\ud588\uae30 \ub54c\ubb38\uc5d0 \uc6b0\uc4f0\ub178\ubbf8\uc57c \uc5ed - \ucfe0\ub85c\uc774\uc18c \uc5ed \uad6c\uac04(\uc77c\ubd80 \uace0\uac00\ub124\uc774\uc5ed - \uad6c\ub85c\uc774\uc18c\uc5ed \uad6c\uac04)\uc5d0\uc11c \uc6b4\uc6a9\ub418\uace0 \uc788\ub358 E231\uacc4 5\ub7c9 \uc6b4\uc6a9\uc774 211\uacc4 5\ub7c9 \uc6b4\uc6a9(\uc77c\ubd80 5\ub7c9+5\ub7c9 \uc6b4\uc6a9\ub3c4 \uc788\ub2e4)\uc73c\ub85c \ubcc0\uacbd\ub418\uc5c8\ub2e4.", "sentence_answer": "\uadf8\ub9b0\ucc28 \uc5f0\uacb0\uc758 \ud3b8\uc131\uc740 10\ub7c9\u00d717\ud3b8\uc131(170\ub7c9)\uc774\uba70 3000\ubc88\ub300 34\uac1c\uac00 \uadf8\ub9b0\ucc28 \uc870\uc785\ud3b8\uc131\uc5d0 \uc0ac\uc6a9\ub418\uc5b4 \ub354 \ucd94\uac00\ud558\uc5ec \ubd80\uc18d\ud3b8\uc131 \ubd84\uc758 17\ud3b8\uc131\uc744 \uacf5\uc81c\ud558\uba74 211\uacc4\uc758 \uc789\uc5ec\ubd84\uc774 \uc815\ud655\ud788 \uc774\ubc88 \uc99d\ube44\ubd84\uc758 E231\uacc4\uc640 \uac19\uc740 \uc218\uc778 110\ub7c9 (5\ub7c9\u00d722\ud3b8\uc131)", "paragraph_id": "6548475-7-1", "paragraph_question": "question: 17\ud3b8\uc131\uc744 \uacf5\uc81c\ud558\uba74 211\uacc4\uc758 \uc789\uc5ec\ubd84\uc740 \uc5bc\ub9c8\uc778\uac00?, context: \uadf8\ub9b0\ucc28 \uc5f0\uacb0\uc758 \ud3b8\uc131\uc740 10\ub7c9\u00d717\ud3b8\uc131(170\ub7c9)\uc774\uba70 3000\ubc88\ub300 34\uac1c\uac00 \uadf8\ub9b0\ucc28 \uc870\uc785\ud3b8\uc131\uc5d0 \uc0ac\uc6a9\ub418\uc5b4 \ub354 \ucd94\uac00\ud558\uc5ec \ubd80\uc18d\ud3b8\uc131 \ubd84\uc758 17\ud3b8\uc131\uc744 \uacf5\uc81c\ud558\uba74 211\uacc4\uc758 \uc789\uc5ec\ubd84\uc774 \uc815\ud655\ud788 \uc774\ubc88 \uc99d\ube44\ubd84\uc758 E231\uacc4\uc640 \uac19\uc740 \uc218\uc778 110\ub7c9(5\ub7c9\u00d722\ud3b8\uc131)\uc774 \ub41c\ub2e4. \uc774\ub7ec\ud55c \ud3b8\uc131 \uac00\uc6b4\ub370 70\ub7c9(5\ub7c9\u00d714\ud3b8\uc131)\uc740 \ud574\uc548\uac00\ub97c \uc8fc\ud589\ud558\uc5ec \uc5fc\ud574\uc5d0 \uc758\ud55c \ubd80\uc2dd\ub4f1\uc774 \uc9c4\ud589\ub418\uace0 \uc788\ub294 \ubcf4\uc18c \uc9c0\uad6c \uac01 \ub178\uc120\uc758 113\uacc4 \uad50\uccb4\uc6a9\uc73c\ub85c \ub9c8\ucfe0\ud558\ub9ac \ucc28\ub7c9\uc13c\ud130\uc5d0 \uc804\uc6a9\ud558\uac8c \ub418\uc5c8\ub2e4. \ub098\uba38\uc9c0 40\ub7c9\uc740 \uc6b0\uc4f0\ub178\ubbf8\uc57c \uc120\u00b7\ub2e4\uce74\uc0ac\ud0a4 \uc120\uc758 \uc218\uc1a1\ub825 \uc99d\uac15\ubd84\uc5d0 \ud22c\uc785\ud560 \uc218 \uc788\uac8c \ub418\uc5c8\ub2e4. 2006\ub144 7\uc6d4 \ub2e4\uc774\uc5b4 \uac1c\uc815\uc5d0\uc11c\ub294 \uc6b0\uc5d0\ub178\uad6c\uc758 15\ub7c9 \uc6b4\uc6a9\uc774 \uc99d\uac00\ud588\uae30 \ub54c\ubb38\uc5d0 \uc6b0\uc4f0\ub178\ubbf8\uc57c \uc5ed - \ucfe0\ub85c\uc774\uc18c \uc5ed \uad6c\uac04(\uc77c\ubd80 \uace0\uac00\ub124\uc774\uc5ed - \uad6c\ub85c\uc774\uc18c\uc5ed \uad6c\uac04)\uc5d0\uc11c \uc6b4\uc6a9\ub418\uace0 \uc788\ub358 E231\uacc4 5\ub7c9 \uc6b4\uc6a9\uc774 211\uacc4 5\ub7c9 \uc6b4\uc6a9(\uc77c\ubd80 5\ub7c9+5\ub7c9 \uc6b4\uc6a9\ub3c4 \uc788\ub2e4)\uc73c\ub85c \ubcc0\uacbd\ub418\uc5c8\ub2e4."} {"question": "1930\ub144\ub300 \ud3f4\ub780\ub4dc\uc758 \uccad\ub144\uacc4\uac00 \uad6d\ubbfc\ub2f9\uc744 \ud0c8\ub2f9\ud558\uc5ec, \ucc3d\ub2f9\ud55c\uac83\uc740?", "paragraph": "1930\ub144\ub300 \uc580 \ubaa8\uc2a4\ub3c4\ub85c\ud504\uc640 \uadf8\ub97c \ub530\ub974\ub294 \uccad\ub144\uacc4\uac00 \uad6d\ubbfc\ub2f9\uc744 \ud0c8\ub2f9\ud558\uc5ec \uad6d\ubbfc\uae09\uc9c4\uae30\uc9c0\ub97c \ucc3d\ub2f9\ud558\uc600\ub2e4. \uadf8\ub7ec\ub098 \uad6d\ubbfc\uae09\uc9c4\uae30\uc9c0\ub294 \ud3f4\ub780\ub4dc \uc815\ubd80\uc5d0 \uc758\ud574 \ud574\uc0b0\ub418\uc5c8\uace0, \uc77c\ubd80 \uc138\ub825\ub4e4\uc740 \ud314\ub791\uac00\uc640 ABC\ub85c \ud65c\ub3d9\ud558\uac8c \ub41c\ub2e4. \uadf8\ub7ec\ub098 \uc81c2\ucc28 \uc138\uacc4 \ub300\uc804\uc774 \uc77c\uc5b4\ub098\uc790, \uc5ec\ub7ec \uce5c\ud3f4\ub780\ub4dc \ub808\uc9c0\uc2a4\ud0d5\uc2a4 \uae30\uad6c\uc5d0 \uc18c\uc18d\ub418\uc5c8\uace0, \ub3c5\uc77c\uad70\uacfc \uc2f8\uc6b0\uac8c \ub41c\ub2e4. \ud55c\ud3b8 \ubaa8\uc2a4\ub3c4\ub85c\ud504\ub294 \uad6d\ubbfc\ub2f9\uc5d0 \ubcf5\ub2f9\ud558\uc600\uace0, \uad6d\ubbfc\uad70\uc0ac\uae30\uad6c \ucc3d\uc124\uc744 \uc9c0\uc6d0\ud558\ub2e4 1943\ub144 \uc544\uc6b0\uc288\ube44\uce20 \uc218\uc6a9\uc18c\uc5d0 \ud559\uc0b4\ub85c \uc0ac\ub9dd\ud558\uac8c \ub418\uc5c8\ub2e4. \uadf8\ub7ec\ub098 \uc81c2\ucc28 \uc138\uacc4 \ub300\uc804 \uc885\uc804 \ud6c4 \uad6d\ubbfc\uae09\uc9c4\uae30\uc9c0\ub294 \ud798\uc744 \uc783\uc5c8\uace0, 1947\ub144 \ud3f4\ub780\ub4dc\ub294 \uad6d\ubbfc\ub2f9\uc744 \uac15\uc81c\ub85c \uc815\uce58 \ud65c\ub3d9 \uae08\uc9c0, \ud574\uc0b0\uc744 \ud558\uac8c \ub41c\ub2e4. 1989\ub144 \ud3f4\ub780\ub4dc \uc778\ubbfc\uacf5\ud654\uad6d\uc774 \ubd95\uad34\ub418\uc5c8\uace0, \uc774\ud6c4 \uc218\ub9ce\uc740 \uc6b0\uc775 \ub2e8\uccb4\ub4e4\uc774 \ub9cc\ub4e4\uc5b4\uc9c0\uac8c \ub41c\ub2e4. \uc774\ub97c \uacc4\uae30\ub85c 1993\ub144 \uad6d\ubbfc\uae09\uc9c4\uae30\uc9c0\uac00 \ubd80\ud65c\ud558\uc600\uace0, \uc815\uce58\uacc4\uc5d0\ub3c4 \ub6f0\uc5b4\ub4e4\uace0 \uc788\ub2e4. 2015\ub144\uc5d0\ub294 \ud30c\ubca8 \ucfe0\ud0a4\uc988\uac00 \ucfe0\ud0a4\uc98815\ub97c \ucc3d\ub2f9\ud568\uc5d0 \ub530\ub77c, \ucfe0\ud0a4\uc98815\ub294 2015\ub144 \uc758\ud68c \uc120\uac70\uc5d0\uc11c 29\uc11d\uc744 \ud76d\ub4dd\ud558\uc600\ub2e4. \ud604\uc7ac\uc758 \uadf9\uc6b0 \uc815\ub2f9\uc740 \uad6d\ubbfc\uc6b4\ub3d9, \ucfe0\ud0a4\uc98815, \ud3f4\ub780\ub4dc \uac00\uc871\uc5f0\ud569 \ub4f1\uc774 \uc788\ub2e4.", "answer": "\uad6d\ubbfc\uae09\uc9c4\uae30\uc9c0", "sentence": "1930\ub144\ub300 \uc580 \ubaa8\uc2a4\ub3c4\ub85c\ud504\uc640 \uadf8\ub97c \ub530\ub974\ub294 \uccad\ub144\uacc4\uac00 \uad6d\ubbfc\ub2f9\uc744 \ud0c8\ub2f9\ud558\uc5ec \uad6d\ubbfc\uae09\uc9c4\uae30\uc9c0 \ub97c", "paragraph_sentence": " 1930\ub144\ub300 \uc580 \ubaa8\uc2a4\ub3c4\ub85c\ud504\uc640 \uadf8\ub97c \ub530\ub974\ub294 \uccad\ub144\uacc4\uac00 \uad6d\ubbfc\ub2f9\uc744 \ud0c8\ub2f9\ud558\uc5ec \uad6d\ubbfc\uae09\uc9c4\uae30\uc9c0 \ub97c \ucc3d\ub2f9\ud558\uc600\ub2e4. \uadf8\ub7ec\ub098 \uad6d\ubbfc\uae09\uc9c4\uae30\uc9c0\ub294 \ud3f4\ub780\ub4dc \uc815\ubd80\uc5d0 \uc758\ud574 \ud574\uc0b0\ub418\uc5c8\uace0, \uc77c\ubd80 \uc138\ub825\ub4e4\uc740 \ud314\ub791\uac00\uc640 ABC\ub85c \ud65c\ub3d9\ud558\uac8c \ub41c\ub2e4. \uadf8\ub7ec\ub098 \uc81c2\ucc28 \uc138\uacc4 \ub300\uc804\uc774 \uc77c\uc5b4\ub098\uc790, \uc5ec\ub7ec \uce5c\ud3f4\ub780\ub4dc \ub808\uc9c0\uc2a4\ud0d5\uc2a4 \uae30\uad6c\uc5d0 \uc18c\uc18d\ub418\uc5c8\uace0, \ub3c5\uc77c\uad70\uacfc \uc2f8\uc6b0\uac8c \ub41c\ub2e4. \ud55c\ud3b8 \ubaa8\uc2a4\ub3c4\ub85c\ud504\ub294 \uad6d\ubbfc\ub2f9\uc5d0 \ubcf5\ub2f9\ud558\uc600\uace0, \uad6d\ubbfc\uad70\uc0ac\uae30\uad6c \ucc3d\uc124\uc744 \uc9c0\uc6d0\ud558\ub2e4 1943\ub144 \uc544\uc6b0\uc288\ube44\uce20 \uc218\uc6a9\uc18c\uc5d0 \ud559\uc0b4\ub85c \uc0ac\ub9dd\ud558\uac8c \ub418\uc5c8\ub2e4. \uadf8\ub7ec\ub098 \uc81c2\ucc28 \uc138\uacc4 \ub300\uc804 \uc885\uc804 \ud6c4 \uad6d\ubbfc\uae09\uc9c4\uae30\uc9c0\ub294 \ud798\uc744 \uc783\uc5c8\uace0, 1947\ub144 \ud3f4\ub780\ub4dc\ub294 \uad6d\ubbfc\ub2f9\uc744 \uac15\uc81c\ub85c \uc815\uce58 \ud65c\ub3d9 \uae08\uc9c0, \ud574\uc0b0\uc744 \ud558\uac8c \ub41c\ub2e4. 1989\ub144 \ud3f4\ub780\ub4dc \uc778\ubbfc\uacf5\ud654\uad6d\uc774 \ubd95\uad34\ub418\uc5c8\uace0, \uc774\ud6c4 \uc218\ub9ce\uc740 \uc6b0\uc775 \ub2e8\uccb4\ub4e4\uc774 \ub9cc\ub4e4\uc5b4\uc9c0\uac8c \ub41c\ub2e4. \uc774\ub97c \uacc4\uae30\ub85c 1993\ub144 \uad6d\ubbfc\uae09\uc9c4\uae30\uc9c0\uac00 \ubd80\ud65c\ud558\uc600\uace0, \uc815\uce58\uacc4\uc5d0\ub3c4 \ub6f0\uc5b4\ub4e4\uace0 \uc788\ub2e4. 2015\ub144\uc5d0\ub294 \ud30c\ubca8 \ucfe0\ud0a4\uc988\uac00 \ucfe0\ud0a4\uc98815\ub97c \ucc3d\ub2f9\ud568\uc5d0 \ub530\ub77c, \ucfe0\ud0a4\uc98815\ub294 2015\ub144 \uc758\ud68c \uc120\uac70\uc5d0\uc11c 29\uc11d\uc744 \ud76d\ub4dd\ud558\uc600\ub2e4. \ud604\uc7ac\uc758 \uadf9\uc6b0 \uc815\ub2f9\uc740 \uad6d\ubbfc\uc6b4\ub3d9, \ucfe0\ud0a4\uc98815, \ud3f4\ub780\ub4dc \uac00\uc871\uc5f0\ud569 \ub4f1\uc774 \uc788\ub2e4.", "paragraph_answer": "1930\ub144\ub300 \uc580 \ubaa8\uc2a4\ub3c4\ub85c\ud504\uc640 \uadf8\ub97c \ub530\ub974\ub294 \uccad\ub144\uacc4\uac00 \uad6d\ubbfc\ub2f9\uc744 \ud0c8\ub2f9\ud558\uc5ec \uad6d\ubbfc\uae09\uc9c4\uae30\uc9c0 \ub97c \ucc3d\ub2f9\ud558\uc600\ub2e4. \uadf8\ub7ec\ub098 \uad6d\ubbfc\uae09\uc9c4\uae30\uc9c0\ub294 \ud3f4\ub780\ub4dc \uc815\ubd80\uc5d0 \uc758\ud574 \ud574\uc0b0\ub418\uc5c8\uace0, \uc77c\ubd80 \uc138\ub825\ub4e4\uc740 \ud314\ub791\uac00\uc640 ABC\ub85c \ud65c\ub3d9\ud558\uac8c \ub41c\ub2e4. \uadf8\ub7ec\ub098 \uc81c2\ucc28 \uc138\uacc4 \ub300\uc804\uc774 \uc77c\uc5b4\ub098\uc790, \uc5ec\ub7ec \uce5c\ud3f4\ub780\ub4dc \ub808\uc9c0\uc2a4\ud0d5\uc2a4 \uae30\uad6c\uc5d0 \uc18c\uc18d\ub418\uc5c8\uace0, \ub3c5\uc77c\uad70\uacfc \uc2f8\uc6b0\uac8c \ub41c\ub2e4. \ud55c\ud3b8 \ubaa8\uc2a4\ub3c4\ub85c\ud504\ub294 \uad6d\ubbfc\ub2f9\uc5d0 \ubcf5\ub2f9\ud558\uc600\uace0, \uad6d\ubbfc\uad70\uc0ac\uae30\uad6c \ucc3d\uc124\uc744 \uc9c0\uc6d0\ud558\ub2e4 1943\ub144 \uc544\uc6b0\uc288\ube44\uce20 \uc218\uc6a9\uc18c\uc5d0 \ud559\uc0b4\ub85c \uc0ac\ub9dd\ud558\uac8c \ub418\uc5c8\ub2e4. \uadf8\ub7ec\ub098 \uc81c2\ucc28 \uc138\uacc4 \ub300\uc804 \uc885\uc804 \ud6c4 \uad6d\ubbfc\uae09\uc9c4\uae30\uc9c0\ub294 \ud798\uc744 \uc783\uc5c8\uace0, 1947\ub144 \ud3f4\ub780\ub4dc\ub294 \uad6d\ubbfc\ub2f9\uc744 \uac15\uc81c\ub85c \uc815\uce58 \ud65c\ub3d9 \uae08\uc9c0, \ud574\uc0b0\uc744 \ud558\uac8c \ub41c\ub2e4. 1989\ub144 \ud3f4\ub780\ub4dc \uc778\ubbfc\uacf5\ud654\uad6d\uc774 \ubd95\uad34\ub418\uc5c8\uace0, \uc774\ud6c4 \uc218\ub9ce\uc740 \uc6b0\uc775 \ub2e8\uccb4\ub4e4\uc774 \ub9cc\ub4e4\uc5b4\uc9c0\uac8c \ub41c\ub2e4. \uc774\ub97c \uacc4\uae30\ub85c 1993\ub144 \uad6d\ubbfc\uae09\uc9c4\uae30\uc9c0\uac00 \ubd80\ud65c\ud558\uc600\uace0, \uc815\uce58\uacc4\uc5d0\ub3c4 \ub6f0\uc5b4\ub4e4\uace0 \uc788\ub2e4. 2015\ub144\uc5d0\ub294 \ud30c\ubca8 \ucfe0\ud0a4\uc988\uac00 \ucfe0\ud0a4\uc98815\ub97c \ucc3d\ub2f9\ud568\uc5d0 \ub530\ub77c, \ucfe0\ud0a4\uc98815\ub294 2015\ub144 \uc758\ud68c \uc120\uac70\uc5d0\uc11c 29\uc11d\uc744 \ud76d\ub4dd\ud558\uc600\ub2e4. \ud604\uc7ac\uc758 \uadf9\uc6b0 \uc815\ub2f9\uc740 \uad6d\ubbfc\uc6b4\ub3d9, \ucfe0\ud0a4\uc98815, \ud3f4\ub780\ub4dc \uac00\uc871\uc5f0\ud569 \ub4f1\uc774 \uc788\ub2e4.", "sentence_answer": "1930\ub144\ub300 \uc580 \ubaa8\uc2a4\ub3c4\ub85c\ud504\uc640 \uadf8\ub97c \ub530\ub974\ub294 \uccad\ub144\uacc4\uac00 \uad6d\ubbfc\ub2f9\uc744 \ud0c8\ub2f9\ud558\uc5ec \uad6d\ubbfc\uae09\uc9c4\uae30\uc9c0 \ub97c", "paragraph_id": "6478086-6-0", "paragraph_question": "question: 1930\ub144\ub300 \ud3f4\ub780\ub4dc\uc758 \uccad\ub144\uacc4\uac00 \uad6d\ubbfc\ub2f9\uc744 \ud0c8\ub2f9\ud558\uc5ec, \ucc3d\ub2f9\ud55c\uac83\uc740?, context: 1930\ub144\ub300 \uc580 \ubaa8\uc2a4\ub3c4\ub85c\ud504\uc640 \uadf8\ub97c \ub530\ub974\ub294 \uccad\ub144\uacc4\uac00 \uad6d\ubbfc\ub2f9\uc744 \ud0c8\ub2f9\ud558\uc5ec \uad6d\ubbfc\uae09\uc9c4\uae30\uc9c0\ub97c \ucc3d\ub2f9\ud558\uc600\ub2e4. \uadf8\ub7ec\ub098 \uad6d\ubbfc\uae09\uc9c4\uae30\uc9c0\ub294 \ud3f4\ub780\ub4dc \uc815\ubd80\uc5d0 \uc758\ud574 \ud574\uc0b0\ub418\uc5c8\uace0, \uc77c\ubd80 \uc138\ub825\ub4e4\uc740 \ud314\ub791\uac00\uc640 ABC\ub85c \ud65c\ub3d9\ud558\uac8c \ub41c\ub2e4. \uadf8\ub7ec\ub098 \uc81c2\ucc28 \uc138\uacc4 \ub300\uc804\uc774 \uc77c\uc5b4\ub098\uc790, \uc5ec\ub7ec \uce5c\ud3f4\ub780\ub4dc \ub808\uc9c0\uc2a4\ud0d5\uc2a4 \uae30\uad6c\uc5d0 \uc18c\uc18d\ub418\uc5c8\uace0, \ub3c5\uc77c\uad70\uacfc \uc2f8\uc6b0\uac8c \ub41c\ub2e4. \ud55c\ud3b8 \ubaa8\uc2a4\ub3c4\ub85c\ud504\ub294 \uad6d\ubbfc\ub2f9\uc5d0 \ubcf5\ub2f9\ud558\uc600\uace0, \uad6d\ubbfc\uad70\uc0ac\uae30\uad6c \ucc3d\uc124\uc744 \uc9c0\uc6d0\ud558\ub2e4 1943\ub144 \uc544\uc6b0\uc288\ube44\uce20 \uc218\uc6a9\uc18c\uc5d0 \ud559\uc0b4\ub85c \uc0ac\ub9dd\ud558\uac8c \ub418\uc5c8\ub2e4. \uadf8\ub7ec\ub098 \uc81c2\ucc28 \uc138\uacc4 \ub300\uc804 \uc885\uc804 \ud6c4 \uad6d\ubbfc\uae09\uc9c4\uae30\uc9c0\ub294 \ud798\uc744 \uc783\uc5c8\uace0, 1947\ub144 \ud3f4\ub780\ub4dc\ub294 \uad6d\ubbfc\ub2f9\uc744 \uac15\uc81c\ub85c \uc815\uce58 \ud65c\ub3d9 \uae08\uc9c0, \ud574\uc0b0\uc744 \ud558\uac8c \ub41c\ub2e4. 1989\ub144 \ud3f4\ub780\ub4dc \uc778\ubbfc\uacf5\ud654\uad6d\uc774 \ubd95\uad34\ub418\uc5c8\uace0, \uc774\ud6c4 \uc218\ub9ce\uc740 \uc6b0\uc775 \ub2e8\uccb4\ub4e4\uc774 \ub9cc\ub4e4\uc5b4\uc9c0\uac8c \ub41c\ub2e4. \uc774\ub97c \uacc4\uae30\ub85c 1993\ub144 \uad6d\ubbfc\uae09\uc9c4\uae30\uc9c0\uac00 \ubd80\ud65c\ud558\uc600\uace0, \uc815\uce58\uacc4\uc5d0\ub3c4 \ub6f0\uc5b4\ub4e4\uace0 \uc788\ub2e4. 2015\ub144\uc5d0\ub294 \ud30c\ubca8 \ucfe0\ud0a4\uc988\uac00 \ucfe0\ud0a4\uc98815\ub97c \ucc3d\ub2f9\ud568\uc5d0 \ub530\ub77c, \ucfe0\ud0a4\uc98815\ub294 2015\ub144 \uc758\ud68c \uc120\uac70\uc5d0\uc11c 29\uc11d\uc744 \ud76d\ub4dd\ud558\uc600\ub2e4. \ud604\uc7ac\uc758 \uadf9\uc6b0 \uc815\ub2f9\uc740 \uad6d\ubbfc\uc6b4\ub3d9, \ucfe0\ud0a4\uc98815, \ud3f4\ub780\ub4dc \uac00\uc871\uc5f0\ud569 \ub4f1\uc774 \uc788\ub2e4."} {"question": "\uac15\ud76c\uc81c\uac00 \ud669\uc704\ub97c \ub118\uae34 \uc778\ubb3c\uc758 \uc774\ub984\uc740 \ubb34\uc5c7\uc778\uac00?", "paragraph": "\ub2f9\uc2dc \ud6c4\uacc4\uc790\ub85c 4\ud669\uc790 \uc639\uce5c\uc655 \uc724\uc9c4\uacfc 14\ud669\uc790 \uc21c\uad70\uc655 \uc724\uc81c \uc911 \uace0\uc2ec\ud558\ub358 \uac15\ud76c\uc81c\ub294 \uc131\uaca9\uc774 \uce58\ubc00\ud558\uace0 \uc2e0\uc911\ud558\uc5ec \ud669\uc704\ub97c \ubb3c\ub824\ubc1b\uc744 \uc218 \uc788\ub294 \uc7ac\ubaa9\uc774\ub77c \uc5ec\uae30\uace0 \uc724\uc9c4\uc5d0\uac8c \ub118\uae34 \uac83\uc774\ub2e4. \uadf8\ub7ec\ub098 \uac15\ud76c\uc81c\uc758 \uc8fd\uc74c\uacfc \ud6c4\uacc4\uc790\ub85c \uc724\uc9c4\uc744 \uc9c0\ubaa9\ud55c \uac83\uc5d0 \ub300\ud574 \ub9ce\uc740 \uac00\uc124\ub4e4\uc774 \uc788\ub2e4. \uc5ed\uc2dc \uc544\ubc84\uc9c0\ub97c \uc8fd\uc774\uace0 \ud669\uc81c\uc5d0 \uc62c\ub790\ub2e4\ub294 \uc218 \uc591\uc81c\ub294 \ubd80\ud669\uc778 \uc218 \ubb38\uc81c\ub97c \uc2dc\ud574\ud558\uc600\ub2e4\ub294 \uc99d\uac70\uac00 \uc815\uc0ac\uc778 \u300a\uc218\uc11c\u300b\uc640 \u300a\uc790\uce58\ud1b5\uac10\u300b\uc5d0 \uc0c1\uc138\ud788 \ub098\uc640 \uc788\uc5b4\uc11c \uc591\uc81c\uac00 \ubd80\ud669\uc744 \uc8fd\uc774\uace0 \ud669\uc704\uc5d0 \uc62c\ub790\ub2e4\ub294 \uac83\uc774 \uc815\uc0ac\uc640 \uc57c\uc0ac \ubaa8\ub450 \uc77c\uce58\ud558\ub098, \uc57c\uc0ac\uc5d0\uc11c \uc8fc\uc7a5\ud558\ub294 \uc724\uc9c4\uc758 \uac15\ud76c\uc81c \uc2dc\ud574\ub294 \uccad\ub098\ub77c \uc815\uc0ac\uc778 \u300a\uccad\uc0ac\uace0\u300b \uadf8 \uc5b4\ub514\uc5d0\ub3c4 \ucc3e\uc544\ubcfc \uc218\uac00 \uc5c6\ub2e4. \ud2b9\ud788 \uc57c\uc0ac\uc5d0\uc11c\ub294 \uadf8 \ub2f9\uc2dc \uac15\ud76c\uc81c\uc758 \uce68\uc804\uc744 \ub9c8\uc74c\ub300\ub85c \ucd9c\uc785\ud560 \uc218 \uc788\ub358 \uc735\uacfc\ub2e4\ub294 \uc724\uc9c4\uc744 \ud669\uc704\uc5d0 \uc62c\ub9ac\uae30 \uc704\ud574 \uac15\ud76c\uc81c\uc758 \uc720\uc870\ub97c \ubcc0\uc870\ud558\uace0 \uac15\ud76c\uc81c\ub97c \uc2dc\ud574\ud588\ub2e4\ub294 \uac00\uc124\uc774 \uc788\uc73c\ub098, \uc99d\uac70\uac00 \ubd88\ucda9\ubd84\ud558\ub2e4. \ub610\ub294 \uc724\uc9c4\uc774 \uc9c1\uc811 \uac15\ud76c\uc81c\ub97c \uc8fd\uc600\ub2e4\uace0 \ud558\ub098 \uc774 \uc5ed\uc2dc \uc99d\uac70\uac00 \ubd88\ucda9\ubd84\ud558\ub2e4.", "answer": "\uc724\uc9c4", "sentence": "\ub2f9\uc2dc \ud6c4\uacc4\uc790\ub85c 4\ud669\uc790 \uc639\uce5c\uc655 \uc724\uc9c4 \uacfc 14\ud669\uc790 \uc21c\uad70\uc655 \uc724\uc81c \uc911 \uace0\uc2ec\ud558\ub358 \uac15\ud76c\uc81c\ub294 \uc131\uaca9\uc774 \uce58\ubc00\ud558\uace0 \uc2e0\uc911\ud558\uc5ec \ud669\uc704\ub97c \ubb3c\ub824\ubc1b\uc744 \uc218 \uc788\ub294 \uc7ac\ubaa9\uc774\ub77c \uc5ec\uae30\uace0 \uc724\uc9c4\uc5d0\uac8c \ub118\uae34 \uac83\uc774\ub2e4.", "paragraph_sentence": " \ub2f9\uc2dc \ud6c4\uacc4\uc790\ub85c 4\ud669\uc790 \uc639\uce5c\uc655 \uc724\uc9c4 \uacfc 14\ud669\uc790 \uc21c\uad70\uc655 \uc724\uc81c \uc911 \uace0\uc2ec\ud558\ub358 \uac15\ud76c\uc81c\ub294 \uc131\uaca9\uc774 \uce58\ubc00\ud558\uace0 \uc2e0\uc911\ud558\uc5ec \ud669\uc704\ub97c \ubb3c\ub824\ubc1b\uc744 \uc218 \uc788\ub294 \uc7ac\ubaa9\uc774\ub77c \uc5ec\uae30\uace0 \uc724\uc9c4\uc5d0\uac8c \ub118\uae34 \uac83\uc774\ub2e4. \uadf8\ub7ec\ub098 \uac15\ud76c\uc81c\uc758 \uc8fd\uc74c\uacfc \ud6c4\uacc4\uc790\ub85c \uc724\uc9c4\uc744 \uc9c0\ubaa9\ud55c \uac83\uc5d0 \ub300\ud574 \ub9ce\uc740 \uac00\uc124\ub4e4\uc774 \uc788\ub2e4. \uc5ed\uc2dc \uc544\ubc84\uc9c0\ub97c \uc8fd\uc774\uace0 \ud669\uc81c\uc5d0 \uc62c\ub790\ub2e4\ub294 \uc218 \uc591\uc81c\ub294 \ubd80\ud669\uc778 \uc218 \ubb38\uc81c\ub97c \uc2dc\ud574\ud558\uc600\ub2e4\ub294 \uc99d\uac70\uac00 \uc815\uc0ac\uc778 \u300a\uc218\uc11c\u300b\uc640 \u300a\uc790\uce58\ud1b5\uac10\u300b\uc5d0 \uc0c1\uc138\ud788 \ub098\uc640 \uc788\uc5b4\uc11c \uc591\uc81c\uac00 \ubd80\ud669\uc744 \uc8fd\uc774\uace0 \ud669\uc704\uc5d0 \uc62c\ub790\ub2e4\ub294 \uac83\uc774 \uc815\uc0ac\uc640 \uc57c\uc0ac \ubaa8\ub450 \uc77c\uce58\ud558\ub098, \uc57c\uc0ac\uc5d0\uc11c \uc8fc\uc7a5\ud558\ub294 \uc724\uc9c4\uc758 \uac15\ud76c\uc81c \uc2dc\ud574\ub294 \uccad\ub098\ub77c \uc815\uc0ac\uc778 \u300a\uccad\uc0ac\uace0\u300b \uadf8 \uc5b4\ub514\uc5d0\ub3c4 \ucc3e\uc544\ubcfc \uc218\uac00 \uc5c6\ub2e4. \ud2b9\ud788 \uc57c\uc0ac\uc5d0\uc11c\ub294 \uadf8 \ub2f9\uc2dc \uac15\ud76c\uc81c\uc758 \uce68\uc804\uc744 \ub9c8\uc74c\ub300\ub85c \ucd9c\uc785\ud560 \uc218 \uc788\ub358 \uc735\uacfc\ub2e4\ub294 \uc724\uc9c4\uc744 \ud669\uc704\uc5d0 \uc62c\ub9ac\uae30 \uc704\ud574 \uac15\ud76c\uc81c\uc758 \uc720\uc870\ub97c \ubcc0\uc870\ud558\uace0 \uac15\ud76c\uc81c\ub97c \uc2dc\ud574\ud588\ub2e4\ub294 \uac00\uc124\uc774 \uc788\uc73c\ub098, \uc99d\uac70\uac00 \ubd88\ucda9\ubd84\ud558\ub2e4. \ub610\ub294 \uc724\uc9c4\uc774 \uc9c1\uc811 \uac15\ud76c\uc81c\ub97c \uc8fd\uc600\ub2e4\uace0 \ud558\ub098 \uc774 \uc5ed\uc2dc \uc99d\uac70\uac00 \ubd88\ucda9\ubd84\ud558\ub2e4.", "paragraph_answer": "\ub2f9\uc2dc \ud6c4\uacc4\uc790\ub85c 4\ud669\uc790 \uc639\uce5c\uc655 \uc724\uc9c4 \uacfc 14\ud669\uc790 \uc21c\uad70\uc655 \uc724\uc81c \uc911 \uace0\uc2ec\ud558\ub358 \uac15\ud76c\uc81c\ub294 \uc131\uaca9\uc774 \uce58\ubc00\ud558\uace0 \uc2e0\uc911\ud558\uc5ec \ud669\uc704\ub97c \ubb3c\ub824\ubc1b\uc744 \uc218 \uc788\ub294 \uc7ac\ubaa9\uc774\ub77c \uc5ec\uae30\uace0 \uc724\uc9c4\uc5d0\uac8c \ub118\uae34 \uac83\uc774\ub2e4. \uadf8\ub7ec\ub098 \uac15\ud76c\uc81c\uc758 \uc8fd\uc74c\uacfc \ud6c4\uacc4\uc790\ub85c \uc724\uc9c4\uc744 \uc9c0\ubaa9\ud55c \uac83\uc5d0 \ub300\ud574 \ub9ce\uc740 \uac00\uc124\ub4e4\uc774 \uc788\ub2e4. \uc5ed\uc2dc \uc544\ubc84\uc9c0\ub97c \uc8fd\uc774\uace0 \ud669\uc81c\uc5d0 \uc62c\ub790\ub2e4\ub294 \uc218 \uc591\uc81c\ub294 \ubd80\ud669\uc778 \uc218 \ubb38\uc81c\ub97c \uc2dc\ud574\ud558\uc600\ub2e4\ub294 \uc99d\uac70\uac00 \uc815\uc0ac\uc778 \u300a\uc218\uc11c\u300b\uc640 \u300a\uc790\uce58\ud1b5\uac10\u300b\uc5d0 \uc0c1\uc138\ud788 \ub098\uc640 \uc788\uc5b4\uc11c \uc591\uc81c\uac00 \ubd80\ud669\uc744 \uc8fd\uc774\uace0 \ud669\uc704\uc5d0 \uc62c\ub790\ub2e4\ub294 \uac83\uc774 \uc815\uc0ac\uc640 \uc57c\uc0ac \ubaa8\ub450 \uc77c\uce58\ud558\ub098, \uc57c\uc0ac\uc5d0\uc11c \uc8fc\uc7a5\ud558\ub294 \uc724\uc9c4\uc758 \uac15\ud76c\uc81c \uc2dc\ud574\ub294 \uccad\ub098\ub77c \uc815\uc0ac\uc778 \u300a\uccad\uc0ac\uace0\u300b \uadf8 \uc5b4\ub514\uc5d0\ub3c4 \ucc3e\uc544\ubcfc \uc218\uac00 \uc5c6\ub2e4. \ud2b9\ud788 \uc57c\uc0ac\uc5d0\uc11c\ub294 \uadf8 \ub2f9\uc2dc \uac15\ud76c\uc81c\uc758 \uce68\uc804\uc744 \ub9c8\uc74c\ub300\ub85c \ucd9c\uc785\ud560 \uc218 \uc788\ub358 \uc735\uacfc\ub2e4\ub294 \uc724\uc9c4\uc744 \ud669\uc704\uc5d0 \uc62c\ub9ac\uae30 \uc704\ud574 \uac15\ud76c\uc81c\uc758 \uc720\uc870\ub97c \ubcc0\uc870\ud558\uace0 \uac15\ud76c\uc81c\ub97c \uc2dc\ud574\ud588\ub2e4\ub294 \uac00\uc124\uc774 \uc788\uc73c\ub098, \uc99d\uac70\uac00 \ubd88\ucda9\ubd84\ud558\ub2e4. \ub610\ub294 \uc724\uc9c4\uc774 \uc9c1\uc811 \uac15\ud76c\uc81c\ub97c \uc8fd\uc600\ub2e4\uace0 \ud558\ub098 \uc774 \uc5ed\uc2dc \uc99d\uac70\uac00 \ubd88\ucda9\ubd84\ud558\ub2e4.", "sentence_answer": "\ub2f9\uc2dc \ud6c4\uacc4\uc790\ub85c 4\ud669\uc790 \uc639\uce5c\uc655 \uc724\uc9c4 \uacfc 14\ud669\uc790 \uc21c\uad70\uc655 \uc724\uc81c \uc911 \uace0\uc2ec\ud558\ub358 \uac15\ud76c\uc81c\ub294 \uc131\uaca9\uc774 \uce58\ubc00\ud558\uace0 \uc2e0\uc911\ud558\uc5ec \ud669\uc704\ub97c \ubb3c\ub824\ubc1b\uc744 \uc218 \uc788\ub294 \uc7ac\ubaa9\uc774\ub77c \uc5ec\uae30\uace0 \uc724\uc9c4\uc5d0\uac8c \ub118\uae34 \uac83\uc774\ub2e4.", "paragraph_id": "6541455-11-0", "paragraph_question": "question: \uac15\ud76c\uc81c\uac00 \ud669\uc704\ub97c \ub118\uae34 \uc778\ubb3c\uc758 \uc774\ub984\uc740 \ubb34\uc5c7\uc778\uac00?, context: \ub2f9\uc2dc \ud6c4\uacc4\uc790\ub85c 4\ud669\uc790 \uc639\uce5c\uc655 \uc724\uc9c4\uacfc 14\ud669\uc790 \uc21c\uad70\uc655 \uc724\uc81c \uc911 \uace0\uc2ec\ud558\ub358 \uac15\ud76c\uc81c\ub294 \uc131\uaca9\uc774 \uce58\ubc00\ud558\uace0 \uc2e0\uc911\ud558\uc5ec \ud669\uc704\ub97c \ubb3c\ub824\ubc1b\uc744 \uc218 \uc788\ub294 \uc7ac\ubaa9\uc774\ub77c \uc5ec\uae30\uace0 \uc724\uc9c4\uc5d0\uac8c \ub118\uae34 \uac83\uc774\ub2e4. \uadf8\ub7ec\ub098 \uac15\ud76c\uc81c\uc758 \uc8fd\uc74c\uacfc \ud6c4\uacc4\uc790\ub85c \uc724\uc9c4\uc744 \uc9c0\ubaa9\ud55c \uac83\uc5d0 \ub300\ud574 \ub9ce\uc740 \uac00\uc124\ub4e4\uc774 \uc788\ub2e4. \uc5ed\uc2dc \uc544\ubc84\uc9c0\ub97c \uc8fd\uc774\uace0 \ud669\uc81c\uc5d0 \uc62c\ub790\ub2e4\ub294 \uc218 \uc591\uc81c\ub294 \ubd80\ud669\uc778 \uc218 \ubb38\uc81c\ub97c \uc2dc\ud574\ud558\uc600\ub2e4\ub294 \uc99d\uac70\uac00 \uc815\uc0ac\uc778 \u300a\uc218\uc11c\u300b\uc640 \u300a\uc790\uce58\ud1b5\uac10\u300b\uc5d0 \uc0c1\uc138\ud788 \ub098\uc640 \uc788\uc5b4\uc11c \uc591\uc81c\uac00 \ubd80\ud669\uc744 \uc8fd\uc774\uace0 \ud669\uc704\uc5d0 \uc62c\ub790\ub2e4\ub294 \uac83\uc774 \uc815\uc0ac\uc640 \uc57c\uc0ac \ubaa8\ub450 \uc77c\uce58\ud558\ub098, \uc57c\uc0ac\uc5d0\uc11c \uc8fc\uc7a5\ud558\ub294 \uc724\uc9c4\uc758 \uac15\ud76c\uc81c \uc2dc\ud574\ub294 \uccad\ub098\ub77c \uc815\uc0ac\uc778 \u300a\uccad\uc0ac\uace0\u300b \uadf8 \uc5b4\ub514\uc5d0\ub3c4 \ucc3e\uc544\ubcfc \uc218\uac00 \uc5c6\ub2e4. \ud2b9\ud788 \uc57c\uc0ac\uc5d0\uc11c\ub294 \uadf8 \ub2f9\uc2dc \uac15\ud76c\uc81c\uc758 \uce68\uc804\uc744 \ub9c8\uc74c\ub300\ub85c \ucd9c\uc785\ud560 \uc218 \uc788\ub358 \uc735\uacfc\ub2e4\ub294 \uc724\uc9c4\uc744 \ud669\uc704\uc5d0 \uc62c\ub9ac\uae30 \uc704\ud574 \uac15\ud76c\uc81c\uc758 \uc720\uc870\ub97c \ubcc0\uc870\ud558\uace0 \uac15\ud76c\uc81c\ub97c \uc2dc\ud574\ud588\ub2e4\ub294 \uac00\uc124\uc774 \uc788\uc73c\ub098, \uc99d\uac70\uac00 \ubd88\ucda9\ubd84\ud558\ub2e4. \ub610\ub294 \uc724\uc9c4\uc774 \uc9c1\uc811 \uac15\ud76c\uc81c\ub97c \uc8fd\uc600\ub2e4\uace0 \ud558\ub098 \uc774 \uc5ed\uc2dc \uc99d\uac70\uac00 \ubd88\ucda9\ubd84\ud558\ub2e4."} {"question": "\ud3ed\uc2a4\ub294 \uc624\ub978\uc190 \uc5b4\ub290 \uc190\uac00\ub77d\uc5d0 \ub2e8\uc9c0\uc99d\uc744 \uc553\uace0\uc788\ub098?", "paragraph": "\ud3ed\uc2a4\ub294 \ub610\ud55c \uc624\ub978\uc190 \uc5c4\uc9c0 \uc190\uac00\ub77d\uc774 \ub2e8\uc9c0\uc99d\uc744 \uc553\uace0 \uc788\ub2e4\uace0\ud55c\ub2e4. 2010\ub144\uc5d0\ub294 \uac15\ubc15 \uc7a5\uc560\ub97c \uc553\uace0\uc788\ub2e4 \ubc1d\ud788\uba70 \"\ub2e4\ub978 \uc0ac\ub78c\uc774 \uc4f0\ub358 \ud654\uc7a5\uc2e4\uc774\ub098 \uc695\uc2e4\uc744 \ubcf4\uba74 \ubc15\ud14c\ub9ac\uc544\uac00 \uc0ac\ubc29\uc5d0 \ub4dd\uc2e4\uac70\ub9ac\ub294 \uac83 \uac19\ub2e4.\"\ub77c\uace0 \ub9d0\ud588\uace0 \uc774\uc5b4 \"\uc2dd\ub2f9 \ud3ec\ud06c \ub610\ud55c \ub2e4\ub978 \uc0ac\ub78c\ub4e4\uc774 \uc0ac\uc6a9\ud588\uc744 \ud150\ub370 \ub0b4 \uc785\uc18d\uc73c\ub85c \ubc15\ud14c\ub9ac\uc544\uac00 \ub4e4\uc5b4\uac04\ub2e4\ub294 \uc0dd\uac01\uc744 \ud558\uba74 \ub054\ucc0d\ud558\ub2e4.\"\ub77c\uba70 \uc2ec\uacbd\uc744 \ud1a0\ud588\ub2e4. \ub610\ud55c \uc601\uad6d\uc758 \uc7a1\uc9c0 \u300a\uc6d0\ub354\ub79c\ub4dc\u300b\uc640\uc758 \uc778\ud130\ubdf0\uc5d0\uc11c \"\ub098\ub294 \uc815\uc2e0 \uc9c8\ud658\uc774 \uc788\ub2e4. \uacbd\uacc4\uc131 \uc778\uaca9\uc7a5\uc560\ub098 \uacbd\ubbf8\ud55c \uc815\uc2e0 \ubd84\uc5f4\uc99d \ubc1c\uc791 \uc99d\uc138\uac00 \uc788\ub2e4\ub294 \uc0dd\uac01\uacfc \ub04a\uc784\uc5c6\uc774 \uc2f8\uc6b0\uace0 \uc788\ub2e4\"\ub77c\uace0 \ubc1d\ud614\uace0, \uc774\uc5b4 \ub9e4\ub9b4\ub9b0 \uba3c\ub85c\ub97c \ub9e4\uc6b0 \uc88b\uc544\ud55c\ub2e4\uace0 \ub9d0\ud588\ub2e4. 2009\ub144\uc5d0\ub294 NBC \u300a\uc9c0\ubbf8 \ud330\ub7f0 \uc1fc\u300b\uc5d0 \ucd9c\uc5f0\ud574 \"\ub300\ubcf8\uc774\ub4e0 \uc2e0\ubb38\uc774\ub4e0 \uac74\uc870\ud55c \uc885\uc774\ub97c \ub9cc\uc9c0\ub294 \uac83\uc744 \ucc38\uc9c0 \ubabb\ud558\uaca0\uc5b4\uc694. \uadf8\ub798\uc11c \ud56d\uc0c1 \uc190\uc5d0 \uce68\uc744 \ubc1c\ub77c\uc57c \ud574\uc694\"\ub77c\uace0 \ubc1d\ud614\uace0 \uc774\uc5b4 \"\ucc45\uc744 \uc77d\uace0 \uc2f6\uc73c\uba74 \uc606\uc5d0 \ubb3c \ud55c \ucef5\uc744 \ub5a0\ub2e4 \ub193\uace0 \uc190\uac00\ub77d\uc744 \ubb3c\uc5d0 \uacc4\uc18d \ub2f4\uac00 \uc900\ub2e4\"\uace0 \ub9d0\ud588\ub2e4. \ub610\ud55c \"\uc800\ub294 \uc9d1\uc5d0 \ubd88\uc744 \ud56d\uc0c1 \ucf1c\ub450\ub294\ub370, \ubd88\uc774 \uaebc\uc9c0\uba74 \uc11c\ub458\ub7ec \uc2a4\uc704\uce58\uac00 \uc788\ub294 \uacf3\uc73c\ub85c \ub2ec\ub824\uac04\ub2e4. \uc548 \ubcf4\uc774\uba74 \ubb34\uc12d\ub2e4\"\uace0 \ub9d0\ud558\uba70 \uc720\ub839\uc744 \ubbff\uc5b4 \uc5b4\ub450\uc6b4 \uacf3\uc744 \uc2eb\uc5b4\ud55c\ub2e4.", "answer": "\uc5c4\uc9c0 \uc190\uac00\ub77d", "sentence": "\ud3ed\uc2a4\ub294 \ub610\ud55c \uc624\ub978\uc190 \uc5c4\uc9c0 \uc190\uac00\ub77d \uc774 \ub2e8\uc9c0\uc99d\uc744 \uc553\uace0 \uc788\ub2e4\uace0\ud55c\ub2e4.", "paragraph_sentence": " \ud3ed\uc2a4\ub294 \ub610\ud55c \uc624\ub978\uc190 \uc5c4\uc9c0 \uc190\uac00\ub77d \uc774 \ub2e8\uc9c0\uc99d\uc744 \uc553\uace0 \uc788\ub2e4\uace0\ud55c\ub2e4. 2010\ub144\uc5d0\ub294 \uac15\ubc15 \uc7a5\uc560\ub97c \uc553\uace0\uc788\ub2e4 \ubc1d\ud788\uba70 \"\ub2e4\ub978 \uc0ac\ub78c\uc774 \uc4f0\ub358 \ud654\uc7a5\uc2e4\uc774\ub098 \uc695\uc2e4\uc744 \ubcf4\uba74 \ubc15\ud14c\ub9ac\uc544\uac00 \uc0ac\ubc29\uc5d0 \ub4dd\uc2e4\uac70\ub9ac\ub294 \uac83 \uac19\ub2e4. \"\ub77c\uace0 \ub9d0\ud588\uace0 \uc774\uc5b4 \"\uc2dd\ub2f9 \ud3ec\ud06c \ub610\ud55c \ub2e4\ub978 \uc0ac\ub78c\ub4e4\uc774 \uc0ac\uc6a9\ud588\uc744 \ud150\ub370 \ub0b4 \uc785\uc18d\uc73c\ub85c \ubc15\ud14c\ub9ac\uc544\uac00 \ub4e4\uc5b4\uac04\ub2e4\ub294 \uc0dd\uac01\uc744 \ud558\uba74 \ub054\ucc0d\ud558\ub2e4. \"\ub77c\uba70 \uc2ec\uacbd\uc744 \ud1a0\ud588\ub2e4. \ub610\ud55c \uc601\uad6d\uc758 \uc7a1\uc9c0 \u300a\uc6d0\ub354\ub79c\ub4dc\u300b\uc640\uc758 \uc778\ud130\ubdf0\uc5d0\uc11c \"\ub098\ub294 \uc815\uc2e0 \uc9c8\ud658\uc774 \uc788\ub2e4. \uacbd\uacc4\uc131 \uc778\uaca9\uc7a5\uc560\ub098 \uacbd\ubbf8\ud55c \uc815\uc2e0 \ubd84\uc5f4\uc99d \ubc1c\uc791 \uc99d\uc138\uac00 \uc788\ub2e4\ub294 \uc0dd\uac01\uacfc \ub04a\uc784\uc5c6\uc774 \uc2f8\uc6b0\uace0 \uc788\ub2e4\"\ub77c\uace0 \ubc1d\ud614\uace0, \uc774\uc5b4 \ub9e4\ub9b4\ub9b0 \uba3c\ub85c\ub97c \ub9e4\uc6b0 \uc88b\uc544\ud55c\ub2e4\uace0 \ub9d0\ud588\ub2e4. 2009\ub144\uc5d0\ub294 NBC \u300a\uc9c0\ubbf8 \ud330\ub7f0 \uc1fc\u300b\uc5d0 \ucd9c\uc5f0\ud574 \"\ub300\ubcf8\uc774\ub4e0 \uc2e0\ubb38\uc774\ub4e0 \uac74\uc870\ud55c \uc885\uc774\ub97c \ub9cc\uc9c0\ub294 \uac83\uc744 \ucc38\uc9c0 \ubabb\ud558\uaca0\uc5b4\uc694. \uadf8\ub798\uc11c \ud56d\uc0c1 \uc190\uc5d0 \uce68\uc744 \ubc1c\ub77c\uc57c \ud574\uc694\"\ub77c\uace0 \ubc1d\ud614\uace0 \uc774\uc5b4 \"\ucc45\uc744 \uc77d\uace0 \uc2f6\uc73c\uba74 \uc606\uc5d0 \ubb3c \ud55c \ucef5\uc744 \ub5a0\ub2e4 \ub193\uace0 \uc190\uac00\ub77d\uc744 \ubb3c\uc5d0 \uacc4\uc18d \ub2f4\uac00 \uc900\ub2e4\"\uace0 \ub9d0\ud588\ub2e4. \ub610\ud55c \"\uc800\ub294 \uc9d1\uc5d0 \ubd88\uc744 \ud56d\uc0c1 \ucf1c\ub450\ub294\ub370, \ubd88\uc774 \uaebc\uc9c0\uba74 \uc11c\ub458\ub7ec \uc2a4\uc704\uce58\uac00 \uc788\ub294 \uacf3\uc73c\ub85c \ub2ec\ub824\uac04\ub2e4. \uc548 \ubcf4\uc774\uba74 \ubb34\uc12d\ub2e4\"\uace0 \ub9d0\ud558\uba70 \uc720\ub839\uc744 \ubbff\uc5b4 \uc5b4\ub450\uc6b4 \uacf3\uc744 \uc2eb\uc5b4\ud55c\ub2e4.", "paragraph_answer": "\ud3ed\uc2a4\ub294 \ub610\ud55c \uc624\ub978\uc190 \uc5c4\uc9c0 \uc190\uac00\ub77d \uc774 \ub2e8\uc9c0\uc99d\uc744 \uc553\uace0 \uc788\ub2e4\uace0\ud55c\ub2e4. 2010\ub144\uc5d0\ub294 \uac15\ubc15 \uc7a5\uc560\ub97c \uc553\uace0\uc788\ub2e4 \ubc1d\ud788\uba70 \"\ub2e4\ub978 \uc0ac\ub78c\uc774 \uc4f0\ub358 \ud654\uc7a5\uc2e4\uc774\ub098 \uc695\uc2e4\uc744 \ubcf4\uba74 \ubc15\ud14c\ub9ac\uc544\uac00 \uc0ac\ubc29\uc5d0 \ub4dd\uc2e4\uac70\ub9ac\ub294 \uac83 \uac19\ub2e4.\"\ub77c\uace0 \ub9d0\ud588\uace0 \uc774\uc5b4 \"\uc2dd\ub2f9 \ud3ec\ud06c \ub610\ud55c \ub2e4\ub978 \uc0ac\ub78c\ub4e4\uc774 \uc0ac\uc6a9\ud588\uc744 \ud150\ub370 \ub0b4 \uc785\uc18d\uc73c\ub85c \ubc15\ud14c\ub9ac\uc544\uac00 \ub4e4\uc5b4\uac04\ub2e4\ub294 \uc0dd\uac01\uc744 \ud558\uba74 \ub054\ucc0d\ud558\ub2e4.\"\ub77c\uba70 \uc2ec\uacbd\uc744 \ud1a0\ud588\ub2e4. \ub610\ud55c \uc601\uad6d\uc758 \uc7a1\uc9c0 \u300a\uc6d0\ub354\ub79c\ub4dc\u300b\uc640\uc758 \uc778\ud130\ubdf0\uc5d0\uc11c \"\ub098\ub294 \uc815\uc2e0 \uc9c8\ud658\uc774 \uc788\ub2e4. \uacbd\uacc4\uc131 \uc778\uaca9\uc7a5\uc560\ub098 \uacbd\ubbf8\ud55c \uc815\uc2e0 \ubd84\uc5f4\uc99d \ubc1c\uc791 \uc99d\uc138\uac00 \uc788\ub2e4\ub294 \uc0dd\uac01\uacfc \ub04a\uc784\uc5c6\uc774 \uc2f8\uc6b0\uace0 \uc788\ub2e4\"\ub77c\uace0 \ubc1d\ud614\uace0, \uc774\uc5b4 \ub9e4\ub9b4\ub9b0 \uba3c\ub85c\ub97c \ub9e4\uc6b0 \uc88b\uc544\ud55c\ub2e4\uace0 \ub9d0\ud588\ub2e4. 2009\ub144\uc5d0\ub294 NBC \u300a\uc9c0\ubbf8 \ud330\ub7f0 \uc1fc\u300b\uc5d0 \ucd9c\uc5f0\ud574 \"\ub300\ubcf8\uc774\ub4e0 \uc2e0\ubb38\uc774\ub4e0 \uac74\uc870\ud55c \uc885\uc774\ub97c \ub9cc\uc9c0\ub294 \uac83\uc744 \ucc38\uc9c0 \ubabb\ud558\uaca0\uc5b4\uc694. \uadf8\ub798\uc11c \ud56d\uc0c1 \uc190\uc5d0 \uce68\uc744 \ubc1c\ub77c\uc57c \ud574\uc694\"\ub77c\uace0 \ubc1d\ud614\uace0 \uc774\uc5b4 \"\ucc45\uc744 \uc77d\uace0 \uc2f6\uc73c\uba74 \uc606\uc5d0 \ubb3c \ud55c \ucef5\uc744 \ub5a0\ub2e4 \ub193\uace0 \uc190\uac00\ub77d\uc744 \ubb3c\uc5d0 \uacc4\uc18d \ub2f4\uac00 \uc900\ub2e4\"\uace0 \ub9d0\ud588\ub2e4. \ub610\ud55c \"\uc800\ub294 \uc9d1\uc5d0 \ubd88\uc744 \ud56d\uc0c1 \ucf1c\ub450\ub294\ub370, \ubd88\uc774 \uaebc\uc9c0\uba74 \uc11c\ub458\ub7ec \uc2a4\uc704\uce58\uac00 \uc788\ub294 \uacf3\uc73c\ub85c \ub2ec\ub824\uac04\ub2e4. \uc548 \ubcf4\uc774\uba74 \ubb34\uc12d\ub2e4\"\uace0 \ub9d0\ud558\uba70 \uc720\ub839\uc744 \ubbff\uc5b4 \uc5b4\ub450\uc6b4 \uacf3\uc744 \uc2eb\uc5b4\ud55c\ub2e4.", "sentence_answer": "\ud3ed\uc2a4\ub294 \ub610\ud55c \uc624\ub978\uc190 \uc5c4\uc9c0 \uc190\uac00\ub77d \uc774 \ub2e8\uc9c0\uc99d\uc744 \uc553\uace0 \uc788\ub2e4\uace0\ud55c\ub2e4.", "paragraph_id": "6580501-7-0", "paragraph_question": "question: \ud3ed\uc2a4\ub294 \uc624\ub978\uc190 \uc5b4\ub290 \uc190\uac00\ub77d\uc5d0 \ub2e8\uc9c0\uc99d\uc744 \uc553\uace0\uc788\ub098?, context: \ud3ed\uc2a4\ub294 \ub610\ud55c \uc624\ub978\uc190 \uc5c4\uc9c0 \uc190\uac00\ub77d\uc774 \ub2e8\uc9c0\uc99d\uc744 \uc553\uace0 \uc788\ub2e4\uace0\ud55c\ub2e4. 2010\ub144\uc5d0\ub294 \uac15\ubc15 \uc7a5\uc560\ub97c \uc553\uace0\uc788\ub2e4 \ubc1d\ud788\uba70 \"\ub2e4\ub978 \uc0ac\ub78c\uc774 \uc4f0\ub358 \ud654\uc7a5\uc2e4\uc774\ub098 \uc695\uc2e4\uc744 \ubcf4\uba74 \ubc15\ud14c\ub9ac\uc544\uac00 \uc0ac\ubc29\uc5d0 \ub4dd\uc2e4\uac70\ub9ac\ub294 \uac83 \uac19\ub2e4.\"\ub77c\uace0 \ub9d0\ud588\uace0 \uc774\uc5b4 \"\uc2dd\ub2f9 \ud3ec\ud06c \ub610\ud55c \ub2e4\ub978 \uc0ac\ub78c\ub4e4\uc774 \uc0ac\uc6a9\ud588\uc744 \ud150\ub370 \ub0b4 \uc785\uc18d\uc73c\ub85c \ubc15\ud14c\ub9ac\uc544\uac00 \ub4e4\uc5b4\uac04\ub2e4\ub294 \uc0dd\uac01\uc744 \ud558\uba74 \ub054\ucc0d\ud558\ub2e4.\"\ub77c\uba70 \uc2ec\uacbd\uc744 \ud1a0\ud588\ub2e4. \ub610\ud55c \uc601\uad6d\uc758 \uc7a1\uc9c0 \u300a\uc6d0\ub354\ub79c\ub4dc\u300b\uc640\uc758 \uc778\ud130\ubdf0\uc5d0\uc11c \"\ub098\ub294 \uc815\uc2e0 \uc9c8\ud658\uc774 \uc788\ub2e4. \uacbd\uacc4\uc131 \uc778\uaca9\uc7a5\uc560\ub098 \uacbd\ubbf8\ud55c \uc815\uc2e0 \ubd84\uc5f4\uc99d \ubc1c\uc791 \uc99d\uc138\uac00 \uc788\ub2e4\ub294 \uc0dd\uac01\uacfc \ub04a\uc784\uc5c6\uc774 \uc2f8\uc6b0\uace0 \uc788\ub2e4\"\ub77c\uace0 \ubc1d\ud614\uace0, \uc774\uc5b4 \ub9e4\ub9b4\ub9b0 \uba3c\ub85c\ub97c \ub9e4\uc6b0 \uc88b\uc544\ud55c\ub2e4\uace0 \ub9d0\ud588\ub2e4. 2009\ub144\uc5d0\ub294 NBC \u300a\uc9c0\ubbf8 \ud330\ub7f0 \uc1fc\u300b\uc5d0 \ucd9c\uc5f0\ud574 \"\ub300\ubcf8\uc774\ub4e0 \uc2e0\ubb38\uc774\ub4e0 \uac74\uc870\ud55c \uc885\uc774\ub97c \ub9cc\uc9c0\ub294 \uac83\uc744 \ucc38\uc9c0 \ubabb\ud558\uaca0\uc5b4\uc694. \uadf8\ub798\uc11c \ud56d\uc0c1 \uc190\uc5d0 \uce68\uc744 \ubc1c\ub77c\uc57c \ud574\uc694\"\ub77c\uace0 \ubc1d\ud614\uace0 \uc774\uc5b4 \"\ucc45\uc744 \uc77d\uace0 \uc2f6\uc73c\uba74 \uc606\uc5d0 \ubb3c \ud55c \ucef5\uc744 \ub5a0\ub2e4 \ub193\uace0 \uc190\uac00\ub77d\uc744 \ubb3c\uc5d0 \uacc4\uc18d \ub2f4\uac00 \uc900\ub2e4\"\uace0 \ub9d0\ud588\ub2e4. \ub610\ud55c \"\uc800\ub294 \uc9d1\uc5d0 \ubd88\uc744 \ud56d\uc0c1 \ucf1c\ub450\ub294\ub370, \ubd88\uc774 \uaebc\uc9c0\uba74 \uc11c\ub458\ub7ec \uc2a4\uc704\uce58\uac00 \uc788\ub294 \uacf3\uc73c\ub85c \ub2ec\ub824\uac04\ub2e4. \uc548 \ubcf4\uc774\uba74 \ubb34\uc12d\ub2e4\"\uace0 \ub9d0\ud558\uba70 \uc720\ub839\uc744 \ubbff\uc5b4 \uc5b4\ub450\uc6b4 \uacf3\uc744 \uc2eb\uc5b4\ud55c\ub2e4."} {"question": "1986\ub144 \uc601\ud654 \uacf5\ud3ec\uc758 \uc2e4\ub85c\ud3f0\uc73c\ub85c \ub300\ubdd4\ud55c \ubc30\uc6b0\ub294?", "paragraph": "\ud3f4 \uc70c\ub9ac\uc5c4 \uc6cc\ucee4 4\uc138(Paul William Walker \u2163, 1973\ub144 9\uc6d4 12\uc77c ~ 2013\ub144 11\uc6d4 30\uc77c)\ub294 \ubbf8\uad6d\uc758 \ubc30\uc6b0\uc774\ub2e4. \ubbf8\uad6d \ub4dc\ub77c\ub9c8\uc5d0 \ub2e8\uc5ed\uc73c\ub85c \ucd9c\uc5f0\ud558\uba74\uc11c \uc5f0\uae30\ub97c \uc2dc\uc791\ud558\uc600\ub2e4. 1986\ub144 \u300a\uacf5\ud3ec\uc758 \uc2e4\ub85c\ud3f0\u300b\uc73c\ub85c \uc601\ud654 \ub370\ubdd4\ud558\uc5ec \u300a\uadf8\ub4e4\ub9cc\uc758 \uacc4\uc808\u300b (1999), \u300a\uc26c\uc988 \uc62c \ub313\u300b (1999), \u300a\uc2a4\uceec\uc2a4\u300b (2000) \ub4f1\uc5d0 \ucd9c\uc5f0\ud558\uba70 \uacbd\ub825\uc744 \uc313\uc558\ub2e4. 2001\ub144 \uac70\ub9ac \uc790\ub3d9\ucc28 \uacbd\uc8fc\ub97c \uc18c\uc7ac\ub85c \ud55c \uc561\uc158 \uc601\ud654 \u300a\ubd84\ub178\uc758 \uc9c8\uc8fc\u300b\uc5d0\uc11c \ube0c\ub77c\uc774\uc5b8 \uc624\ucf54\ub108 \uc5ed\uc744 \ub9e1\uc544 \uad6d\uc81c\uc801\uc778 \uba85\uc131\uc744 \uc5bb\uc5c8\ub2e4. \uc774\ud6c4 3\ud3b8 \u300a\ud328\uc2a4\ud2b8 & \ud4e8\ub9ac\uc5b4\uc2a4: \ub3c4\ucfc4 \ub4dc\ub9ac\ud504\ud2b8\u300b\ub97c \uc81c\uc678\ud55c 7\ud3b8\uae4c\uc9c0 \ubaa8\ub4e0 \u300a\ubd84\ub178\uc758 \uc9c8\uc8fc \uc2dc\ub9ac\uc988\u300b\uc5d0 \ucd9c\uc5f0\ud558\uc600\ub2e4. \uc774 \uc678\uc5d0\ub3c4 \u300a\uce94\ub514 \ucf00\uc778\u300b (2001), \u300a\ud0c0\uc784\ub77c\uc778\u300b (2003), \u300a\ube14\ub8e8 \uc2a4\ud1b0\u300b (2005), \u300a\uc5d0\uc774\ud2b8 \ube4c\ub85c\uc6b0\u300b (2006), \u300a\ub7ec\ub2dd \uc2a4\ucf00\uc5b4\ub4dc\u300b (2006), \u300a\ubc14\ube44 Z\uc758 \uc0b6\uacfc \uc8fd\uc74c\u300b (2007), \u300a\ub77c\uc790\ub8e8\uc2a4 \ud504\ub85c\uc81d\ud2b8\u300b (2008), \u300a\ube44\ud788\ud074 19\u300b (2013), \u300a\ud3f0 \uc0f5 \ud074\ub85c\ub2c8\ud074\uc2a4\u300b (2013) \ub4f1\uc5d0 \ucd9c\uc5f0\ud558\uc600\ub2e4.", "answer": "\ud3f4 \uc70c\ub9ac\uc5c4 \uc6cc\ucee4", "sentence": "\ud3f4 \uc70c\ub9ac\uc5c4 \uc6cc\ucee4 4\uc138(Paul William Walker \u2163, 1973\ub144 9\uc6d4 12\uc77c ~ 2013\ub144 11\uc6d4 30\uc77c)\ub294 \ubbf8\uad6d\uc758 \ubc30\uc6b0\uc774\ub2e4.", "paragraph_sentence": " \ud3f4 \uc70c\ub9ac\uc5c4 \uc6cc\ucee4 4\uc138(Paul William Walker \u2163, 1973\ub144 9\uc6d4 12\uc77c ~ 2013\ub144 11\uc6d4 30\uc77c)\ub294 \ubbf8\uad6d\uc758 \ubc30\uc6b0\uc774\ub2e4. \ubbf8\uad6d \ub4dc\ub77c\ub9c8\uc5d0 \ub2e8\uc5ed\uc73c\ub85c \ucd9c\uc5f0\ud558\uba74\uc11c \uc5f0\uae30\ub97c \uc2dc\uc791\ud558\uc600\ub2e4. 1986\ub144 \u300a\uacf5\ud3ec\uc758 \uc2e4\ub85c\ud3f0\u300b\uc73c\ub85c \uc601\ud654 \ub370\ubdd4\ud558\uc5ec \u300a\uadf8\ub4e4\ub9cc\uc758 \uacc4\uc808\u300b (1999), \u300a\uc26c\uc988 \uc62c \ub313\u300b (1999), \u300a\uc2a4\uceec\uc2a4\u300b (2000) \ub4f1\uc5d0 \ucd9c\uc5f0\ud558\uba70 \uacbd\ub825\uc744 \uc313\uc558\ub2e4. 2001\ub144 \uac70\ub9ac \uc790\ub3d9\ucc28 \uacbd\uc8fc\ub97c \uc18c\uc7ac\ub85c \ud55c \uc561\uc158 \uc601\ud654 \u300a\ubd84\ub178\uc758 \uc9c8\uc8fc\u300b\uc5d0\uc11c \ube0c\ub77c\uc774\uc5b8 \uc624\ucf54\ub108 \uc5ed\uc744 \ub9e1\uc544 \uad6d\uc81c\uc801\uc778 \uba85\uc131\uc744 \uc5bb\uc5c8\ub2e4. \uc774\ud6c4 3\ud3b8 \u300a\ud328\uc2a4\ud2b8 & \ud4e8\ub9ac\uc5b4\uc2a4: \ub3c4\ucfc4 \ub4dc\ub9ac\ud504\ud2b8\u300b\ub97c \uc81c\uc678\ud55c 7\ud3b8\uae4c\uc9c0 \ubaa8\ub4e0 \u300a\ubd84\ub178\uc758 \uc9c8\uc8fc \uc2dc\ub9ac\uc988\u300b\uc5d0 \ucd9c\uc5f0\ud558\uc600\ub2e4. \uc774 \uc678\uc5d0\ub3c4 \u300a\uce94\ub514 \ucf00\uc778\u300b (2001), \u300a\ud0c0\uc784\ub77c\uc778\u300b (2003), \u300a\ube14\ub8e8 \uc2a4\ud1b0\u300b (2005), \u300a\uc5d0\uc774\ud2b8 \ube4c\ub85c\uc6b0\u300b (2006), \u300a\ub7ec\ub2dd \uc2a4\ucf00\uc5b4\ub4dc\u300b (2006), \u300a\ubc14\ube44 Z\uc758 \uc0b6\uacfc \uc8fd\uc74c\u300b (2007), \u300a\ub77c\uc790\ub8e8\uc2a4 \ud504\ub85c\uc81d\ud2b8\u300b (2008), \u300a\ube44\ud788\ud074 19\u300b (2013), \u300a\ud3f0 \uc0f5 \ud074\ub85c\ub2c8\ud074\uc2a4\u300b (2013) \ub4f1\uc5d0 \ucd9c\uc5f0\ud558\uc600\ub2e4.", "paragraph_answer": " \ud3f4 \uc70c\ub9ac\uc5c4 \uc6cc\ucee4 4\uc138(Paul William Walker \u2163, 1973\ub144 9\uc6d4 12\uc77c ~ 2013\ub144 11\uc6d4 30\uc77c)\ub294 \ubbf8\uad6d\uc758 \ubc30\uc6b0\uc774\ub2e4. \ubbf8\uad6d \ub4dc\ub77c\ub9c8\uc5d0 \ub2e8\uc5ed\uc73c\ub85c \ucd9c\uc5f0\ud558\uba74\uc11c \uc5f0\uae30\ub97c \uc2dc\uc791\ud558\uc600\ub2e4. 1986\ub144 \u300a\uacf5\ud3ec\uc758 \uc2e4\ub85c\ud3f0\u300b\uc73c\ub85c \uc601\ud654 \ub370\ubdd4\ud558\uc5ec \u300a\uadf8\ub4e4\ub9cc\uc758 \uacc4\uc808\u300b (1999), \u300a\uc26c\uc988 \uc62c \ub313\u300b (1999), \u300a\uc2a4\uceec\uc2a4\u300b (2000) \ub4f1\uc5d0 \ucd9c\uc5f0\ud558\uba70 \uacbd\ub825\uc744 \uc313\uc558\ub2e4. 2001\ub144 \uac70\ub9ac \uc790\ub3d9\ucc28 \uacbd\uc8fc\ub97c \uc18c\uc7ac\ub85c \ud55c \uc561\uc158 \uc601\ud654 \u300a\ubd84\ub178\uc758 \uc9c8\uc8fc\u300b\uc5d0\uc11c \ube0c\ub77c\uc774\uc5b8 \uc624\ucf54\ub108 \uc5ed\uc744 \ub9e1\uc544 \uad6d\uc81c\uc801\uc778 \uba85\uc131\uc744 \uc5bb\uc5c8\ub2e4. \uc774\ud6c4 3\ud3b8 \u300a\ud328\uc2a4\ud2b8 & \ud4e8\ub9ac\uc5b4\uc2a4: \ub3c4\ucfc4 \ub4dc\ub9ac\ud504\ud2b8\u300b\ub97c \uc81c\uc678\ud55c 7\ud3b8\uae4c\uc9c0 \ubaa8\ub4e0 \u300a\ubd84\ub178\uc758 \uc9c8\uc8fc \uc2dc\ub9ac\uc988\u300b\uc5d0 \ucd9c\uc5f0\ud558\uc600\ub2e4. \uc774 \uc678\uc5d0\ub3c4 \u300a\uce94\ub514 \ucf00\uc778\u300b (2001), \u300a\ud0c0\uc784\ub77c\uc778\u300b (2003), \u300a\ube14\ub8e8 \uc2a4\ud1b0\u300b (2005), \u300a\uc5d0\uc774\ud2b8 \ube4c\ub85c\uc6b0\u300b (2006), \u300a\ub7ec\ub2dd \uc2a4\ucf00\uc5b4\ub4dc\u300b (2006), \u300a\ubc14\ube44 Z\uc758 \uc0b6\uacfc \uc8fd\uc74c\u300b (2007), \u300a\ub77c\uc790\ub8e8\uc2a4 \ud504\ub85c\uc81d\ud2b8\u300b (2008), \u300a\ube44\ud788\ud074 19\u300b (2013), \u300a\ud3f0 \uc0f5 \ud074\ub85c\ub2c8\ud074\uc2a4\u300b (2013) \ub4f1\uc5d0 \ucd9c\uc5f0\ud558\uc600\ub2e4.", "sentence_answer": " \ud3f4 \uc70c\ub9ac\uc5c4 \uc6cc\ucee4 4\uc138(Paul William Walker \u2163, 1973\ub144 9\uc6d4 12\uc77c ~ 2013\ub144 11\uc6d4 30\uc77c)\ub294 \ubbf8\uad6d\uc758 \ubc30\uc6b0\uc774\ub2e4.", "paragraph_id": "6568103-0-0", "paragraph_question": "question: 1986\ub144 \uc601\ud654 \uacf5\ud3ec\uc758 \uc2e4\ub85c\ud3f0\uc73c\ub85c \ub300\ubdd4\ud55c \ubc30\uc6b0\ub294?, context: \ud3f4 \uc70c\ub9ac\uc5c4 \uc6cc\ucee4 4\uc138(Paul William Walker \u2163, 1973\ub144 9\uc6d4 12\uc77c ~ 2013\ub144 11\uc6d4 30\uc77c)\ub294 \ubbf8\uad6d\uc758 \ubc30\uc6b0\uc774\ub2e4. \ubbf8\uad6d \ub4dc\ub77c\ub9c8\uc5d0 \ub2e8\uc5ed\uc73c\ub85c \ucd9c\uc5f0\ud558\uba74\uc11c \uc5f0\uae30\ub97c \uc2dc\uc791\ud558\uc600\ub2e4. 1986\ub144 \u300a\uacf5\ud3ec\uc758 \uc2e4\ub85c\ud3f0\u300b\uc73c\ub85c \uc601\ud654 \ub370\ubdd4\ud558\uc5ec \u300a\uadf8\ub4e4\ub9cc\uc758 \uacc4\uc808\u300b (1999), \u300a\uc26c\uc988 \uc62c \ub313\u300b (1999), \u300a\uc2a4\uceec\uc2a4\u300b (2000) \ub4f1\uc5d0 \ucd9c\uc5f0\ud558\uba70 \uacbd\ub825\uc744 \uc313\uc558\ub2e4. 2001\ub144 \uac70\ub9ac \uc790\ub3d9\ucc28 \uacbd\uc8fc\ub97c \uc18c\uc7ac\ub85c \ud55c \uc561\uc158 \uc601\ud654 \u300a\ubd84\ub178\uc758 \uc9c8\uc8fc\u300b\uc5d0\uc11c \ube0c\ub77c\uc774\uc5b8 \uc624\ucf54\ub108 \uc5ed\uc744 \ub9e1\uc544 \uad6d\uc81c\uc801\uc778 \uba85\uc131\uc744 \uc5bb\uc5c8\ub2e4. \uc774\ud6c4 3\ud3b8 \u300a\ud328\uc2a4\ud2b8 & \ud4e8\ub9ac\uc5b4\uc2a4: \ub3c4\ucfc4 \ub4dc\ub9ac\ud504\ud2b8\u300b\ub97c \uc81c\uc678\ud55c 7\ud3b8\uae4c\uc9c0 \ubaa8\ub4e0 \u300a\ubd84\ub178\uc758 \uc9c8\uc8fc \uc2dc\ub9ac\uc988\u300b\uc5d0 \ucd9c\uc5f0\ud558\uc600\ub2e4. \uc774 \uc678\uc5d0\ub3c4 \u300a\uce94\ub514 \ucf00\uc778\u300b (2001), \u300a\ud0c0\uc784\ub77c\uc778\u300b (2003), \u300a\ube14\ub8e8 \uc2a4\ud1b0\u300b (2005), \u300a\uc5d0\uc774\ud2b8 \ube4c\ub85c\uc6b0\u300b (2006), \u300a\ub7ec\ub2dd \uc2a4\ucf00\uc5b4\ub4dc\u300b (2006), \u300a\ubc14\ube44 Z\uc758 \uc0b6\uacfc \uc8fd\uc74c\u300b (2007), \u300a\ub77c\uc790\ub8e8\uc2a4 \ud504\ub85c\uc81d\ud2b8\u300b (2008), \u300a\ube44\ud788\ud074 19\u300b (2013), \u300a\ud3f0 \uc0f5 \ud074\ub85c\ub2c8\ud074\uc2a4\u300b (2013) \ub4f1\uc5d0 \ucd9c\uc5f0\ud558\uc600\ub2e4."} {"question": "\uae30\ub2c8\uc5d0 \ud37c\uc9c4 \uc5d0\ubcfc\ub77c \ubc14\uc774\ub7ec\uc2a4\uac00 \"\uc790\uc774\ub974 \uc5d0\ubcfc\ub77c \ubc14\uc774\ub7ec\uc2a4\"\uc758 \ubcc0\ud615\uc774\ub77c\ub294 \uac83\uc744 \ud655\uc778\ud55c \ud504\ub791\uc2a4 \ub9ac\uc639\uc758 \uc5f0\uad6c\uc18c \uc774\ub984\uc740?", "paragraph": "2014\ub144 2\uc6d4, \uae30\ub2c8 \uc9c0\uc5ed\uc5d0\uc11c \uc5d0\ubcfc\ub77c \ubc14\uc774\ub7ec\uc2a4\uc5d0 \uac10\uc5fc\ub41c \uc0ac\ub78c\uc774 \ucc98\uc74c \ud655\uc778\ub418\uc5c8\ub2e4. 2014\ub144 4\uc6d4 2\uc77c \uae30\uc900\uc73c\ub85c, \uc5d0\ubcfc\ub77c \ucd9c\ud608\uc5f4 \uac10\uc5fc \uc758\uc2ec\uc790\uc218 \ubc0f \ud655\uc778\uc790\uc218\ub294 \ucd1d 151\uba85\uc73c\ub85c, \uadf8 \uc911 \uc0ac\ub9dd\uc790\uac00 97\uba85\uc73c\ub85c \uce58\uba85\ub960\uc774 64.2%\uc774\ub2e4. \ucc98\uc74c\uc73c\ub85c \uc5d0\ubcfc\ub77c \ucd9c\ud608\uc5f4\uc774 \uc758\uc2ec\ub418\ub294 \ubcf4\uace0\uac00 \ub4e4\uc5b4\uc628 \uacf3\uc740 \ucf54\ub098\ud06c\ub9ac(4\uac74), \uad6c\uc5d0\ucf00\ub450\uad6c(4\uac74), \ub9c8\uc13c\ud0c0(1\uac74), \ub2e4\ubcfc\ub77c(1\uac74)\uc774\ub2e4. 2014\ub144 3\uc6d4 25\uc77c \uae30\uc900, \uae30\ub2c8 \ubcf4\uac74\ubd80\ub294 \ub0a8\ub3d9\ubd80\uc758 \uad6c\uc5d0\ucf00\ub450\uad6c, \ub9c8\uc13c\ud0c0, \uc740\uc81c\ub808\ucf54\ub808, \ud0a4\uc2dc\ub450\uad6c\ub4f1 4\uac1c \uc9c0\uc5ed\uc5d0\uc11c \uc5d0\ubcfc\ub77c \ucd9c\ud608\uc5f4 \uac10\uc5fc\uc774 \ubcf4\uace0\ub418\uc5c8\ub2e4\uace0 \ubc1d\ud614\ub2e4. 2014\ub144 3\uc6d4 26\uc77c, \uae30\ub2c8\uc5d0 \ud37c\uc9c0\ub294 \uc5d0\ubcfc\ub77c \ubc14\uc774\ub7ec\uc2a4\uac00 \ud504\ub791\uc2a4 \ub9ac\uc639\uc758 \ud30c\uc2a4\ud1f4\ub974 \uc5f0\uad6c\uc18c\uc5d0\uc11c \"\uc790\uc774\ub974 \uc5d0\ubcfc\ub77c \ubc14\uc774\ub7ec\uc2a4\"\uc758 \ubcc0\ud615\ub41c \uc885\uc784\uc744 \ud655\uc778\ud588\ub2e4. 2014\ub144 3\uc6d4 28\uc77c\uc5d0\ub294 \ubcd1\uc6d0\uc5d0 \uc785\uc6d0\ud55c \ud658\uc790 1\uba85\uc774 \uc5d0\ubcfc\ub77c \ubc14\uc774\ub7ec\uc2a4\ub85c \uc758\uc2ec\ub41c\ub2e4\ub294 \ubcf4\uace0\uac00 \ub4e4\uc5b4\uc654\ub2e4. 3\uc6d4 31\uc77c, \ubbf8\uad6d \uc9c8\ubcd1\ud1b5\uc81c\uc608\ubc29\uc13c\ud130\ub294 \"MOH \uae30\ub2c8 \uc9c0\ubd80\uc640 WHO\uac00 \uc8fc\ub3c4\ud558\ub294 \uc5d0\ubcfc\ub77c \ubc1c\ubc1c \uad6d\uc81c \ub300\uc751\ud300\" 5\uba85\uc744 \ubcf4\ub0c8\ub2e4. 4\uc6d4 4\uc77c, \uae30\ub2c8\uc5d0\uc11c \uad6d\uacbd \uc5c6\ub294 \uc758\uc0ac\ud68c\uac00 \uc6b4\uc601\ud558\ub294 \uc5d0\ubcfc\ub77c \ubc14\uc774\ub7ec\uc2a4 \uce58\ub8cc \uc13c\ud130\uac00 \uc8fc\ubbfc\ub4e4\uc758 \uacf5\uaca9\uc73c\ub85c \uc778\ud574 \ud3d0\uc1c4\ub418\uc5c8\ub2e4.", "answer": "\ud30c\uc2a4\ud1f4\ub974 \uc5f0\uad6c\uc18c", "sentence": "2014\ub144 3\uc6d4 26\uc77c, \uae30\ub2c8\uc5d0 \ud37c\uc9c0\ub294 \uc5d0\ubcfc\ub77c \ubc14\uc774\ub7ec\uc2a4\uac00 \ud504\ub791\uc2a4 \ub9ac\uc639\uc758 \ud30c\uc2a4\ud1f4\ub974 \uc5f0\uad6c\uc18c \uc5d0\uc11c \"\uc790\uc774\ub974 \uc5d0\ubcfc\ub77c \ubc14\uc774\ub7ec\uc2a4\"\uc758 \ubcc0\ud615\ub41c \uc885\uc784\uc744 \ud655\uc778\ud588\ub2e4.", "paragraph_sentence": "2014\ub144 2\uc6d4, \uae30\ub2c8 \uc9c0\uc5ed\uc5d0\uc11c \uc5d0\ubcfc\ub77c \ubc14\uc774\ub7ec\uc2a4\uc5d0 \uac10\uc5fc\ub41c \uc0ac\ub78c\uc774 \ucc98\uc74c \ud655\uc778\ub418\uc5c8\ub2e4. 2014\ub144 4\uc6d4 2\uc77c \uae30\uc900\uc73c\ub85c, \uc5d0\ubcfc\ub77c \ucd9c\ud608\uc5f4 \uac10\uc5fc \uc758\uc2ec\uc790\uc218 \ubc0f \ud655\uc778\uc790\uc218\ub294 \ucd1d 151\uba85\uc73c\ub85c, \uadf8 \uc911 \uc0ac\ub9dd\uc790\uac00 97\uba85\uc73c\ub85c \uce58\uba85\ub960\uc774 64.2%\uc774\ub2e4. \ucc98\uc74c\uc73c\ub85c \uc5d0\ubcfc\ub77c \ucd9c\ud608\uc5f4\uc774 \uc758\uc2ec\ub418\ub294 \ubcf4\uace0\uac00 \ub4e4\uc5b4\uc628 \uacf3\uc740 \ucf54\ub098\ud06c\ub9ac(4\uac74), \uad6c\uc5d0\ucf00\ub450\uad6c(4\uac74), \ub9c8\uc13c\ud0c0(1\uac74), \ub2e4\ubcfc\ub77c(1\uac74)\uc774\ub2e4. 2014\ub144 3\uc6d4 25\uc77c \uae30\uc900, \uae30\ub2c8 \ubcf4\uac74\ubd80\ub294 \ub0a8\ub3d9\ubd80\uc758 \uad6c\uc5d0\ucf00\ub450\uad6c, \ub9c8\uc13c\ud0c0, \uc740\uc81c\ub808\ucf54\ub808, \ud0a4\uc2dc\ub450\uad6c\ub4f1 4\uac1c \uc9c0\uc5ed\uc5d0\uc11c \uc5d0\ubcfc\ub77c \ucd9c\ud608\uc5f4 \uac10\uc5fc\uc774 \ubcf4\uace0\ub418\uc5c8\ub2e4\uace0 \ubc1d\ud614\ub2e4. 2014\ub144 3\uc6d4 26\uc77c, \uae30\ub2c8\uc5d0 \ud37c\uc9c0\ub294 \uc5d0\ubcfc\ub77c \ubc14\uc774\ub7ec\uc2a4\uac00 \ud504\ub791\uc2a4 \ub9ac\uc639\uc758 \ud30c\uc2a4\ud1f4\ub974 \uc5f0\uad6c\uc18c \uc5d0\uc11c \"\uc790\uc774\ub974 \uc5d0\ubcfc\ub77c \ubc14\uc774\ub7ec\uc2a4\"\uc758 \ubcc0\ud615\ub41c \uc885\uc784\uc744 \ud655\uc778\ud588\ub2e4. 2014\ub144 3\uc6d4 28\uc77c\uc5d0\ub294 \ubcd1\uc6d0\uc5d0 \uc785\uc6d0\ud55c \ud658\uc790 1\uba85\uc774 \uc5d0\ubcfc\ub77c \ubc14\uc774\ub7ec\uc2a4\ub85c \uc758\uc2ec\ub41c\ub2e4\ub294 \ubcf4\uace0\uac00 \ub4e4\uc5b4\uc654\ub2e4. 3\uc6d4 31\uc77c, \ubbf8\uad6d \uc9c8\ubcd1\ud1b5\uc81c\uc608\ubc29\uc13c\ud130\ub294 \"MOH \uae30\ub2c8 \uc9c0\ubd80\uc640 WHO\uac00 \uc8fc\ub3c4\ud558\ub294 \uc5d0\ubcfc\ub77c \ubc1c\ubc1c \uad6d\uc81c \ub300\uc751\ud300\" 5\uba85\uc744 \ubcf4\ub0c8\ub2e4. 4\uc6d4 4\uc77c, \uae30\ub2c8\uc5d0\uc11c \uad6d\uacbd \uc5c6\ub294 \uc758\uc0ac\ud68c\uac00 \uc6b4\uc601\ud558\ub294 \uc5d0\ubcfc\ub77c \ubc14\uc774\ub7ec\uc2a4 \uce58\ub8cc \uc13c\ud130\uac00 \uc8fc\ubbfc\ub4e4\uc758 \uacf5\uaca9\uc73c\ub85c \uc778\ud574 \ud3d0\uc1c4\ub418\uc5c8\ub2e4.", "paragraph_answer": "2014\ub144 2\uc6d4, \uae30\ub2c8 \uc9c0\uc5ed\uc5d0\uc11c \uc5d0\ubcfc\ub77c \ubc14\uc774\ub7ec\uc2a4\uc5d0 \uac10\uc5fc\ub41c \uc0ac\ub78c\uc774 \ucc98\uc74c \ud655\uc778\ub418\uc5c8\ub2e4. 2014\ub144 4\uc6d4 2\uc77c \uae30\uc900\uc73c\ub85c, \uc5d0\ubcfc\ub77c \ucd9c\ud608\uc5f4 \uac10\uc5fc \uc758\uc2ec\uc790\uc218 \ubc0f \ud655\uc778\uc790\uc218\ub294 \ucd1d 151\uba85\uc73c\ub85c, \uadf8 \uc911 \uc0ac\ub9dd\uc790\uac00 97\uba85\uc73c\ub85c \uce58\uba85\ub960\uc774 64.2%\uc774\ub2e4. \ucc98\uc74c\uc73c\ub85c \uc5d0\ubcfc\ub77c \ucd9c\ud608\uc5f4\uc774 \uc758\uc2ec\ub418\ub294 \ubcf4\uace0\uac00 \ub4e4\uc5b4\uc628 \uacf3\uc740 \ucf54\ub098\ud06c\ub9ac(4\uac74), \uad6c\uc5d0\ucf00\ub450\uad6c(4\uac74), \ub9c8\uc13c\ud0c0(1\uac74), \ub2e4\ubcfc\ub77c(1\uac74)\uc774\ub2e4. 2014\ub144 3\uc6d4 25\uc77c \uae30\uc900, \uae30\ub2c8 \ubcf4\uac74\ubd80\ub294 \ub0a8\ub3d9\ubd80\uc758 \uad6c\uc5d0\ucf00\ub450\uad6c, \ub9c8\uc13c\ud0c0, \uc740\uc81c\ub808\ucf54\ub808, \ud0a4\uc2dc\ub450\uad6c\ub4f1 4\uac1c \uc9c0\uc5ed\uc5d0\uc11c \uc5d0\ubcfc\ub77c \ucd9c\ud608\uc5f4 \uac10\uc5fc\uc774 \ubcf4\uace0\ub418\uc5c8\ub2e4\uace0 \ubc1d\ud614\ub2e4. 2014\ub144 3\uc6d4 26\uc77c, \uae30\ub2c8\uc5d0 \ud37c\uc9c0\ub294 \uc5d0\ubcfc\ub77c \ubc14\uc774\ub7ec\uc2a4\uac00 \ud504\ub791\uc2a4 \ub9ac\uc639\uc758 \ud30c\uc2a4\ud1f4\ub974 \uc5f0\uad6c\uc18c \uc5d0\uc11c \"\uc790\uc774\ub974 \uc5d0\ubcfc\ub77c \ubc14\uc774\ub7ec\uc2a4\"\uc758 \ubcc0\ud615\ub41c \uc885\uc784\uc744 \ud655\uc778\ud588\ub2e4. 2014\ub144 3\uc6d4 28\uc77c\uc5d0\ub294 \ubcd1\uc6d0\uc5d0 \uc785\uc6d0\ud55c \ud658\uc790 1\uba85\uc774 \uc5d0\ubcfc\ub77c \ubc14\uc774\ub7ec\uc2a4\ub85c \uc758\uc2ec\ub41c\ub2e4\ub294 \ubcf4\uace0\uac00 \ub4e4\uc5b4\uc654\ub2e4. 3\uc6d4 31\uc77c, \ubbf8\uad6d \uc9c8\ubcd1\ud1b5\uc81c\uc608\ubc29\uc13c\ud130\ub294 \"MOH \uae30\ub2c8 \uc9c0\ubd80\uc640 WHO\uac00 \uc8fc\ub3c4\ud558\ub294 \uc5d0\ubcfc\ub77c \ubc1c\ubc1c \uad6d\uc81c \ub300\uc751\ud300\" 5\uba85\uc744 \ubcf4\ub0c8\ub2e4. 4\uc6d4 4\uc77c, \uae30\ub2c8\uc5d0\uc11c \uad6d\uacbd \uc5c6\ub294 \uc758\uc0ac\ud68c\uac00 \uc6b4\uc601\ud558\ub294 \uc5d0\ubcfc\ub77c \ubc14\uc774\ub7ec\uc2a4 \uce58\ub8cc \uc13c\ud130\uac00 \uc8fc\ubbfc\ub4e4\uc758 \uacf5\uaca9\uc73c\ub85c \uc778\ud574 \ud3d0\uc1c4\ub418\uc5c8\ub2e4.", "sentence_answer": "2014\ub144 3\uc6d4 26\uc77c, \uae30\ub2c8\uc5d0 \ud37c\uc9c0\ub294 \uc5d0\ubcfc\ub77c \ubc14\uc774\ub7ec\uc2a4\uac00 \ud504\ub791\uc2a4 \ub9ac\uc639\uc758 \ud30c\uc2a4\ud1f4\ub974 \uc5f0\uad6c\uc18c \uc5d0\uc11c \"\uc790\uc774\ub974 \uc5d0\ubcfc\ub77c \ubc14\uc774\ub7ec\uc2a4\"\uc758 \ubcc0\ud615\ub41c \uc885\uc784\uc744 \ud655\uc778\ud588\ub2e4.", "paragraph_id": "6437831-1-2", "paragraph_question": "question: \uae30\ub2c8\uc5d0 \ud37c\uc9c4 \uc5d0\ubcfc\ub77c \ubc14\uc774\ub7ec\uc2a4\uac00 \"\uc790\uc774\ub974 \uc5d0\ubcfc\ub77c \ubc14\uc774\ub7ec\uc2a4\"\uc758 \ubcc0\ud615\uc774\ub77c\ub294 \uac83\uc744 \ud655\uc778\ud55c \ud504\ub791\uc2a4 \ub9ac\uc639\uc758 \uc5f0\uad6c\uc18c \uc774\ub984\uc740?, context: 2014\ub144 2\uc6d4, \uae30\ub2c8 \uc9c0\uc5ed\uc5d0\uc11c \uc5d0\ubcfc\ub77c \ubc14\uc774\ub7ec\uc2a4\uc5d0 \uac10\uc5fc\ub41c \uc0ac\ub78c\uc774 \ucc98\uc74c \ud655\uc778\ub418\uc5c8\ub2e4. 2014\ub144 4\uc6d4 2\uc77c \uae30\uc900\uc73c\ub85c, \uc5d0\ubcfc\ub77c \ucd9c\ud608\uc5f4 \uac10\uc5fc \uc758\uc2ec\uc790\uc218 \ubc0f \ud655\uc778\uc790\uc218\ub294 \ucd1d 151\uba85\uc73c\ub85c, \uadf8 \uc911 \uc0ac\ub9dd\uc790\uac00 97\uba85\uc73c\ub85c \uce58\uba85\ub960\uc774 64.2%\uc774\ub2e4. \ucc98\uc74c\uc73c\ub85c \uc5d0\ubcfc\ub77c \ucd9c\ud608\uc5f4\uc774 \uc758\uc2ec\ub418\ub294 \ubcf4\uace0\uac00 \ub4e4\uc5b4\uc628 \uacf3\uc740 \ucf54\ub098\ud06c\ub9ac(4\uac74), \uad6c\uc5d0\ucf00\ub450\uad6c(4\uac74), \ub9c8\uc13c\ud0c0(1\uac74), \ub2e4\ubcfc\ub77c(1\uac74)\uc774\ub2e4. 2014\ub144 3\uc6d4 25\uc77c \uae30\uc900, \uae30\ub2c8 \ubcf4\uac74\ubd80\ub294 \ub0a8\ub3d9\ubd80\uc758 \uad6c\uc5d0\ucf00\ub450\uad6c, \ub9c8\uc13c\ud0c0, \uc740\uc81c\ub808\ucf54\ub808, \ud0a4\uc2dc\ub450\uad6c\ub4f1 4\uac1c \uc9c0\uc5ed\uc5d0\uc11c \uc5d0\ubcfc\ub77c \ucd9c\ud608\uc5f4 \uac10\uc5fc\uc774 \ubcf4\uace0\ub418\uc5c8\ub2e4\uace0 \ubc1d\ud614\ub2e4. 2014\ub144 3\uc6d4 26\uc77c, \uae30\ub2c8\uc5d0 \ud37c\uc9c0\ub294 \uc5d0\ubcfc\ub77c \ubc14\uc774\ub7ec\uc2a4\uac00 \ud504\ub791\uc2a4 \ub9ac\uc639\uc758 \ud30c\uc2a4\ud1f4\ub974 \uc5f0\uad6c\uc18c\uc5d0\uc11c \"\uc790\uc774\ub974 \uc5d0\ubcfc\ub77c \ubc14\uc774\ub7ec\uc2a4\"\uc758 \ubcc0\ud615\ub41c \uc885\uc784\uc744 \ud655\uc778\ud588\ub2e4. 2014\ub144 3\uc6d4 28\uc77c\uc5d0\ub294 \ubcd1\uc6d0\uc5d0 \uc785\uc6d0\ud55c \ud658\uc790 1\uba85\uc774 \uc5d0\ubcfc\ub77c \ubc14\uc774\ub7ec\uc2a4\ub85c \uc758\uc2ec\ub41c\ub2e4\ub294 \ubcf4\uace0\uac00 \ub4e4\uc5b4\uc654\ub2e4. 3\uc6d4 31\uc77c, \ubbf8\uad6d \uc9c8\ubcd1\ud1b5\uc81c\uc608\ubc29\uc13c\ud130\ub294 \"MOH \uae30\ub2c8 \uc9c0\ubd80\uc640 WHO\uac00 \uc8fc\ub3c4\ud558\ub294 \uc5d0\ubcfc\ub77c \ubc1c\ubc1c \uad6d\uc81c \ub300\uc751\ud300\" 5\uba85\uc744 \ubcf4\ub0c8\ub2e4. 4\uc6d4 4\uc77c, \uae30\ub2c8\uc5d0\uc11c \uad6d\uacbd \uc5c6\ub294 \uc758\uc0ac\ud68c\uac00 \uc6b4\uc601\ud558\ub294 \uc5d0\ubcfc\ub77c \ubc14\uc774\ub7ec\uc2a4 \uce58\ub8cc \uc13c\ud130\uac00 \uc8fc\ubbfc\ub4e4\uc758 \uacf5\uaca9\uc73c\ub85c \uc778\ud574 \ud3d0\uc1c4\ub418\uc5c8\ub2e4."} {"question": "\ud584\ub9bf \uc774\uc57c\uae30\uc758 \uc6d0\ud615\uc744 \ud615\uc131\ud558\ub294 \uac83\uc73c\ub85c \uc5b4\ub5a4 \uc791\ud488\uc744 \uaf3d\uc744 \uc218 \uc788\ub294\uac00?", "paragraph": "\ud584\ub9bf\uacfc \uac19\uc740 \"\ubc14\ubcf4 \uc601\uc6c5\"\uc758 \uc6d0\ud615\uc740 \uc6d0\uc2dc \uc778\ub3c4 \uc720\ub7fd\uc5b4\uc871\uae4c\uc9c0 \uc18c\uae09\ub418\uba70 \uc774\ud0c8\ub9ac\uc544, \uc5d0\uc2a4\ud30c\ub0d0, \uc2a4\uce78\ub2e4\ub098\ube44\uc544 \ub4f1\uc9c0\uc758 \uc5ec\ub7ec \uc2e0\ud654\uc640 \uc774\uc57c\uae30\uc5d0\uc11c \uad11\ubc94\uc704\ud558\uac8c \ub2e4\ub8e8\uc5b4\uc9c0\uace0 \uc788\ub2e4. \ud584\ub9bf \uc774\uc57c\uae30\uc758 \uc6d0\ud615\uc744 \ud615\uc131\ud558\ub294 \uac83\uc73c\ub85c\ub294 \uc791\uac00\ubbf8\uc0c1\uc758 \uc2a4\uce78\ub2e4\ub098\ube44\uc544 \uc11c\uc0ac\uc2dc\uc778 \ud750\ub8b8\ud504\uc2a4 \uc0ac\uac00 \ud06c\ub77c\uce74(Hr\u00f3lfs saga kraka, \ud750\ub8b8\ud504\uc2a4 \ud06c\ub77c\uce74 \uc655\uc758 \uc774\uc57c\uae30)\ub97c \uaf3d\uc744 \uc218 \uc788\ub2e4. \uc774 \uc774\uc57c\uae30\uc5d0\ub294 \uc0b4\ud574\ub2f9\ud55c \uc655\uacfc \uadf8\uc758 \ub450 \uc544\ub4e4\uc774 \ub4f1\uc7a5\ud558\uba70 \ub9ce\uc740 \ubd80\ubd84\uc5d0\uc11c \ud584\ub9bf\uacfc \ube44\uc2b7\ud55c \uc791\ud488 \uad6c\uc131\uc744 \ubcf4\uc778\ub2e4. \ud584\ub9bf\uc758 \ub610 \ub2e4\ub978 \uc6d0\uc804\uc73c\ub85c\ub294 \ub8e8\uc2dc\uc6b0\uc2a4 \uc720\ub2c8\uc6b0\uc2a4 \ube0c\ub8e8\ud22c\uc2a4(Lucius Junius Brutus)\uc758 \uc804\uc124\uc774 \uc788\ub2e4. \uadf8\uc758 \uc774\ub984\uc5d0\uc11c \ub8e8\uc2dc\uc6b0\uc2a4\ub294 \"\ube5b\"\uc744 \ube0c\ub8e8\ud22c\uc2a4\ub294 \"\ubc14\ubcf4\"\ub97c \ub73b\ud55c\ub2e4. \uadf8\ub294 \uc790\uc2e0\uc758 \uc544\ubc84\uc9c0\uc640 \ud615\uc81c\ub97c \uc0b4\ud574\ud55c \ud0c0\ub974\ud034\ub2c8\uc6b0\uc2a4\uc5d0\uac8c \ubcf5\uc218\ud558\uae30 \uc704\ud574 \uc2a4\uc2a4\ub85c \ubbf8\uce5c\ucc99\ud558\uc600\ub2e4. 17\uc138\uae30\uc758 \ub178\ub974\ub9cc \ud559\uc790 \ud1a0\ub974\ud398\uc6b0\uc2a4\ub294 \uc170\uc775\uc2a4\ud53c\uc5b4\uc758 \ud584\ub9bf\uacfc \uc544\uc774\uc2ac\ub79c\ub4dc\uc758 \uc554\ub85c\ub514 \uc774\uc57c\uae30 \ubc0f \uc5d0\uc2a4\ud30c\ub0d0\uc758 \uc554\ubc1c\ub808\uc2a4 \uc655\uc790 \uc774\uc57c\uae30\ub97c \ube44\uad50\ud558\uc600\ub2e4. \uc774\ub4e4 \uc774\uc57c\uae30 \ubaa8\ub450 \uc5b4\uba38\ub2c8\uc758 \uce68\uc2e4\uc5d0 \uc228\uc5b4\ub4e0 \uc7ac\uc0c1\uc744 \uc655\uc73c\ub85c \ucc29\uac01\ud558\uc5ec \uc0b4\ud574\ud558\ub294 \ubd80\ubd84\uc774 \ub4f1\uc7a5\ud55c\ub2e4.", "answer": "\ud750\ub8b8\ud504\uc2a4 \uc0ac\uac00 \ud06c\ub77c\uce74", "sentence": "\ud584\ub9bf \uc774\uc57c\uae30\uc758 \uc6d0\ud615\uc744 \ud615\uc131\ud558\ub294 \uac83\uc73c\ub85c\ub294 \uc791\uac00\ubbf8\uc0c1\uc758 \uc2a4\uce78\ub2e4\ub098\ube44\uc544 \uc11c\uc0ac\uc2dc\uc778 \ud750\ub8b8\ud504\uc2a4 \uc0ac\uac00 \ud06c\ub77c\uce74 (Hr\u00f3lfs saga kraka, \ud750\ub8b8\ud504\uc2a4 \ud06c\ub77c\uce74 \uc655\uc758 \uc774\uc57c\uae30)\ub97c \uaf3d\uc744 \uc218 \uc788\ub2e4.", "paragraph_sentence": "\ud584\ub9bf\uacfc \uac19\uc740 \"\ubc14\ubcf4 \uc601\uc6c5\"\uc758 \uc6d0\ud615\uc740 \uc6d0\uc2dc \uc778\ub3c4 \uc720\ub7fd\uc5b4\uc871\uae4c\uc9c0 \uc18c\uae09\ub418\uba70 \uc774\ud0c8\ub9ac\uc544, \uc5d0\uc2a4\ud30c\ub0d0, \uc2a4\uce78\ub2e4\ub098\ube44\uc544 \ub4f1\uc9c0\uc758 \uc5ec\ub7ec \uc2e0\ud654\uc640 \uc774\uc57c\uae30\uc5d0\uc11c \uad11\ubc94\uc704\ud558\uac8c \ub2e4\ub8e8\uc5b4\uc9c0\uace0 \uc788\ub2e4. \ud584\ub9bf \uc774\uc57c\uae30\uc758 \uc6d0\ud615\uc744 \ud615\uc131\ud558\ub294 \uac83\uc73c\ub85c\ub294 \uc791\uac00\ubbf8\uc0c1\uc758 \uc2a4\uce78\ub2e4\ub098\ube44\uc544 \uc11c\uc0ac\uc2dc\uc778 \ud750\ub8b8\ud504\uc2a4 \uc0ac\uac00 \ud06c\ub77c\uce74 (Hr\u00f3lfs saga kraka, \ud750\ub8b8\ud504\uc2a4 \ud06c\ub77c\uce74 \uc655\uc758 \uc774\uc57c\uae30)\ub97c \uaf3d\uc744 \uc218 \uc788\ub2e4. \uc774 \uc774\uc57c\uae30\uc5d0\ub294 \uc0b4\ud574\ub2f9\ud55c \uc655\uacfc \uadf8\uc758 \ub450 \uc544\ub4e4\uc774 \ub4f1\uc7a5\ud558\uba70 \ub9ce\uc740 \ubd80\ubd84\uc5d0\uc11c \ud584\ub9bf\uacfc \ube44\uc2b7\ud55c \uc791\ud488 \uad6c\uc131\uc744 \ubcf4\uc778\ub2e4. \ud584\ub9bf\uc758 \ub610 \ub2e4\ub978 \uc6d0\uc804\uc73c\ub85c\ub294 \ub8e8\uc2dc\uc6b0\uc2a4 \uc720\ub2c8\uc6b0\uc2a4 \ube0c\ub8e8\ud22c\uc2a4(Lucius Junius Brutus)\uc758 \uc804\uc124\uc774 \uc788\ub2e4. \uadf8\uc758 \uc774\ub984\uc5d0\uc11c \ub8e8\uc2dc\uc6b0\uc2a4\ub294 \"\ube5b\"\uc744 \ube0c\ub8e8\ud22c\uc2a4\ub294 \"\ubc14\ubcf4\"\ub97c \ub73b\ud55c\ub2e4. \uadf8\ub294 \uc790\uc2e0\uc758 \uc544\ubc84\uc9c0\uc640 \ud615\uc81c\ub97c \uc0b4\ud574\ud55c \ud0c0\ub974\ud034\ub2c8\uc6b0\uc2a4\uc5d0\uac8c \ubcf5\uc218\ud558\uae30 \uc704\ud574 \uc2a4\uc2a4\ub85c \ubbf8\uce5c\ucc99\ud558\uc600\ub2e4. 17\uc138\uae30\uc758 \ub178\ub974\ub9cc \ud559\uc790 \ud1a0\ub974\ud398\uc6b0\uc2a4\ub294 \uc170\uc775\uc2a4\ud53c\uc5b4\uc758 \ud584\ub9bf\uacfc \uc544\uc774\uc2ac\ub79c\ub4dc\uc758 \uc554\ub85c\ub514 \uc774\uc57c\uae30 \ubc0f \uc5d0\uc2a4\ud30c\ub0d0\uc758 \uc554\ubc1c\ub808\uc2a4 \uc655\uc790 \uc774\uc57c\uae30\ub97c \ube44\uad50\ud558\uc600\ub2e4. \uc774\ub4e4 \uc774\uc57c\uae30 \ubaa8\ub450 \uc5b4\uba38\ub2c8\uc758 \uce68\uc2e4\uc5d0 \uc228\uc5b4\ub4e0 \uc7ac\uc0c1\uc744 \uc655\uc73c\ub85c \ucc29\uac01\ud558\uc5ec \uc0b4\ud574\ud558\ub294 \ubd80\ubd84\uc774 \ub4f1\uc7a5\ud55c\ub2e4.", "paragraph_answer": "\ud584\ub9bf\uacfc \uac19\uc740 \"\ubc14\ubcf4 \uc601\uc6c5\"\uc758 \uc6d0\ud615\uc740 \uc6d0\uc2dc \uc778\ub3c4 \uc720\ub7fd\uc5b4\uc871\uae4c\uc9c0 \uc18c\uae09\ub418\uba70 \uc774\ud0c8\ub9ac\uc544, \uc5d0\uc2a4\ud30c\ub0d0, \uc2a4\uce78\ub2e4\ub098\ube44\uc544 \ub4f1\uc9c0\uc758 \uc5ec\ub7ec \uc2e0\ud654\uc640 \uc774\uc57c\uae30\uc5d0\uc11c \uad11\ubc94\uc704\ud558\uac8c \ub2e4\ub8e8\uc5b4\uc9c0\uace0 \uc788\ub2e4. \ud584\ub9bf \uc774\uc57c\uae30\uc758 \uc6d0\ud615\uc744 \ud615\uc131\ud558\ub294 \uac83\uc73c\ub85c\ub294 \uc791\uac00\ubbf8\uc0c1\uc758 \uc2a4\uce78\ub2e4\ub098\ube44\uc544 \uc11c\uc0ac\uc2dc\uc778 \ud750\ub8b8\ud504\uc2a4 \uc0ac\uac00 \ud06c\ub77c\uce74 (Hr\u00f3lfs saga kraka, \ud750\ub8b8\ud504\uc2a4 \ud06c\ub77c\uce74 \uc655\uc758 \uc774\uc57c\uae30)\ub97c \uaf3d\uc744 \uc218 \uc788\ub2e4. \uc774 \uc774\uc57c\uae30\uc5d0\ub294 \uc0b4\ud574\ub2f9\ud55c \uc655\uacfc \uadf8\uc758 \ub450 \uc544\ub4e4\uc774 \ub4f1\uc7a5\ud558\uba70 \ub9ce\uc740 \ubd80\ubd84\uc5d0\uc11c \ud584\ub9bf\uacfc \ube44\uc2b7\ud55c \uc791\ud488 \uad6c\uc131\uc744 \ubcf4\uc778\ub2e4. \ud584\ub9bf\uc758 \ub610 \ub2e4\ub978 \uc6d0\uc804\uc73c\ub85c\ub294 \ub8e8\uc2dc\uc6b0\uc2a4 \uc720\ub2c8\uc6b0\uc2a4 \ube0c\ub8e8\ud22c\uc2a4(Lucius Junius Brutus)\uc758 \uc804\uc124\uc774 \uc788\ub2e4. \uadf8\uc758 \uc774\ub984\uc5d0\uc11c \ub8e8\uc2dc\uc6b0\uc2a4\ub294 \"\ube5b\"\uc744 \ube0c\ub8e8\ud22c\uc2a4\ub294 \"\ubc14\ubcf4\"\ub97c \ub73b\ud55c\ub2e4. \uadf8\ub294 \uc790\uc2e0\uc758 \uc544\ubc84\uc9c0\uc640 \ud615\uc81c\ub97c \uc0b4\ud574\ud55c \ud0c0\ub974\ud034\ub2c8\uc6b0\uc2a4\uc5d0\uac8c \ubcf5\uc218\ud558\uae30 \uc704\ud574 \uc2a4\uc2a4\ub85c \ubbf8\uce5c\ucc99\ud558\uc600\ub2e4. 17\uc138\uae30\uc758 \ub178\ub974\ub9cc \ud559\uc790 \ud1a0\ub974\ud398\uc6b0\uc2a4\ub294 \uc170\uc775\uc2a4\ud53c\uc5b4\uc758 \ud584\ub9bf\uacfc \uc544\uc774\uc2ac\ub79c\ub4dc\uc758 \uc554\ub85c\ub514 \uc774\uc57c\uae30 \ubc0f \uc5d0\uc2a4\ud30c\ub0d0\uc758 \uc554\ubc1c\ub808\uc2a4 \uc655\uc790 \uc774\uc57c\uae30\ub97c \ube44\uad50\ud558\uc600\ub2e4. \uc774\ub4e4 \uc774\uc57c\uae30 \ubaa8\ub450 \uc5b4\uba38\ub2c8\uc758 \uce68\uc2e4\uc5d0 \uc228\uc5b4\ub4e0 \uc7ac\uc0c1\uc744 \uc655\uc73c\ub85c \ucc29\uac01\ud558\uc5ec \uc0b4\ud574\ud558\ub294 \ubd80\ubd84\uc774 \ub4f1\uc7a5\ud55c\ub2e4.", "sentence_answer": "\ud584\ub9bf \uc774\uc57c\uae30\uc758 \uc6d0\ud615\uc744 \ud615\uc131\ud558\ub294 \uac83\uc73c\ub85c\ub294 \uc791\uac00\ubbf8\uc0c1\uc758 \uc2a4\uce78\ub2e4\ub098\ube44\uc544 \uc11c\uc0ac\uc2dc\uc778 \ud750\ub8b8\ud504\uc2a4 \uc0ac\uac00 \ud06c\ub77c\uce74 (Hr\u00f3lfs saga kraka, \ud750\ub8b8\ud504\uc2a4 \ud06c\ub77c\uce74 \uc655\uc758 \uc774\uc57c\uae30)\ub97c \uaf3d\uc744 \uc218 \uc788\ub2e4.", "paragraph_id": "6570844-3-0", "paragraph_question": "question: \ud584\ub9bf \uc774\uc57c\uae30\uc758 \uc6d0\ud615\uc744 \ud615\uc131\ud558\ub294 \uac83\uc73c\ub85c \uc5b4\ub5a4 \uc791\ud488\uc744 \uaf3d\uc744 \uc218 \uc788\ub294\uac00?, context: \ud584\ub9bf\uacfc \uac19\uc740 \"\ubc14\ubcf4 \uc601\uc6c5\"\uc758 \uc6d0\ud615\uc740 \uc6d0\uc2dc \uc778\ub3c4 \uc720\ub7fd\uc5b4\uc871\uae4c\uc9c0 \uc18c\uae09\ub418\uba70 \uc774\ud0c8\ub9ac\uc544, \uc5d0\uc2a4\ud30c\ub0d0, \uc2a4\uce78\ub2e4\ub098\ube44\uc544 \ub4f1\uc9c0\uc758 \uc5ec\ub7ec \uc2e0\ud654\uc640 \uc774\uc57c\uae30\uc5d0\uc11c \uad11\ubc94\uc704\ud558\uac8c \ub2e4\ub8e8\uc5b4\uc9c0\uace0 \uc788\ub2e4. \ud584\ub9bf \uc774\uc57c\uae30\uc758 \uc6d0\ud615\uc744 \ud615\uc131\ud558\ub294 \uac83\uc73c\ub85c\ub294 \uc791\uac00\ubbf8\uc0c1\uc758 \uc2a4\uce78\ub2e4\ub098\ube44\uc544 \uc11c\uc0ac\uc2dc\uc778 \ud750\ub8b8\ud504\uc2a4 \uc0ac\uac00 \ud06c\ub77c\uce74(Hr\u00f3lfs saga kraka, \ud750\ub8b8\ud504\uc2a4 \ud06c\ub77c\uce74 \uc655\uc758 \uc774\uc57c\uae30)\ub97c \uaf3d\uc744 \uc218 \uc788\ub2e4. \uc774 \uc774\uc57c\uae30\uc5d0\ub294 \uc0b4\ud574\ub2f9\ud55c \uc655\uacfc \uadf8\uc758 \ub450 \uc544\ub4e4\uc774 \ub4f1\uc7a5\ud558\uba70 \ub9ce\uc740 \ubd80\ubd84\uc5d0\uc11c \ud584\ub9bf\uacfc \ube44\uc2b7\ud55c \uc791\ud488 \uad6c\uc131\uc744 \ubcf4\uc778\ub2e4. \ud584\ub9bf\uc758 \ub610 \ub2e4\ub978 \uc6d0\uc804\uc73c\ub85c\ub294 \ub8e8\uc2dc\uc6b0\uc2a4 \uc720\ub2c8\uc6b0\uc2a4 \ube0c\ub8e8\ud22c\uc2a4(Lucius Junius Brutus)\uc758 \uc804\uc124\uc774 \uc788\ub2e4. \uadf8\uc758 \uc774\ub984\uc5d0\uc11c \ub8e8\uc2dc\uc6b0\uc2a4\ub294 \"\ube5b\"\uc744 \ube0c\ub8e8\ud22c\uc2a4\ub294 \"\ubc14\ubcf4\"\ub97c \ub73b\ud55c\ub2e4. \uadf8\ub294 \uc790\uc2e0\uc758 \uc544\ubc84\uc9c0\uc640 \ud615\uc81c\ub97c \uc0b4\ud574\ud55c \ud0c0\ub974\ud034\ub2c8\uc6b0\uc2a4\uc5d0\uac8c \ubcf5\uc218\ud558\uae30 \uc704\ud574 \uc2a4\uc2a4\ub85c \ubbf8\uce5c\ucc99\ud558\uc600\ub2e4. 17\uc138\uae30\uc758 \ub178\ub974\ub9cc \ud559\uc790 \ud1a0\ub974\ud398\uc6b0\uc2a4\ub294 \uc170\uc775\uc2a4\ud53c\uc5b4\uc758 \ud584\ub9bf\uacfc \uc544\uc774\uc2ac\ub79c\ub4dc\uc758 \uc554\ub85c\ub514 \uc774\uc57c\uae30 \ubc0f \uc5d0\uc2a4\ud30c\ub0d0\uc758 \uc554\ubc1c\ub808\uc2a4 \uc655\uc790 \uc774\uc57c\uae30\ub97c \ube44\uad50\ud558\uc600\ub2e4. \uc774\ub4e4 \uc774\uc57c\uae30 \ubaa8\ub450 \uc5b4\uba38\ub2c8\uc758 \uce68\uc2e4\uc5d0 \uc228\uc5b4\ub4e0 \uc7ac\uc0c1\uc744 \uc655\uc73c\ub85c \ucc29\uac01\ud558\uc5ec \uc0b4\ud574\ud558\ub294 \ubd80\ubd84\uc774 \ub4f1\uc7a5\ud55c\ub2e4."} {"question": "View\uc758 \uc7a5\ub974\ub294?", "paragraph": "\uc0e4\uc774\ub2c8\ub294 5\uc6d4 18\uc77c, 1\ub144 7\uac1c\uc6d4\uc5ec\uc758 \uacf5\ubc31\uc744 \uae68\uace0 \ub300\ud55c\ubbfc\uad6d \ud65c\ub3d9\uc73c\ub85c \ucef4\ubc31\ud558\uc600\ub2e4. \uc815\uaddc 4\uc9d1 \uc74c\ubc18 \u300aOdd\u300b\ub294 \uc774\uc804 \uc0e4\uc774\ub2c8\uc758 \ud589\ubcf4\uc5d0 \ube44\ucd94\uc5b4 \ubcf4\uba74 \uc774\ub840\uc801\uc73c\ub85c \uc790\uc815\uc5d0 \uc74c\uc6d0\uc774 \uacf5\uac1c\ub418\uc5c8\ub2e4. \ub3d9\uc2dc\uc5d0 \uc0e4\uc774\ub2c8\ub294 \uba64\ubc84 \uc885\ud604\uc774 \uc9c4\ud589\ud558\ub294 MBC \ub77c\ub514\uc624 \ud478\ub978\ubc24 \uc885\ud604\uc785\ub2c8\ub2e4\uc5d0 \uc2a4\ud398\uc15c \uac8c\uc2a4\ud2b8\ub85c \ucd08\ub300\ub418\uc5c8\ub2e4. \uc74c\uc6d0 \uc804\uace1\uc774 \uac01\uc885 \uc74c\uc6d0\ucc28\ud2b8\ub97c \ud729\uc4f8\uc5c8\uace0, \uc74c\uc545 \ubc29\uc1a1\uc5d0\uc11c\ub3c4 1\uc704\ub97c \ud729\uc4f8\uac8c \ub418\uc5c8\ub2e4. \ud2b9\ud788 \ud0c0\uc774\ud2c0\uace1\uc778 \u3008View\u3009\ub294 \uc774\uc804\uae4c\uc9c0 \ud55c\uad6d\uc5d0\uc11c\ub294 \uc0dd\uc18c\ud588\ub358 \ub525 \ud558\uc6b0\uc2a4 \uc7a5\ub974\ub85c, \uc601\uad6d\uc758 \uc791\uace1\ud300 LDN Noise\uac00 \uc791\uace1\ud558\uc600\uc73c\uba70, \uba64\ubc84 \uc885\ud604\uc774 \ub0b4\ubd80 \uc791\uc0ac \ucf58\ud14c\uc2a4\ud2b8\uc5d0\uc11c 40:1\uc758 \uacbd\uc7c1\ub960\uc744 \ub6ab\uace0 \uc120\uc815\ub41c \uac00\uc0ac\ub97c \uc37c\ub2e4. \uc885\ud604\uc740 \uc774 \uc678\uc5d0\ub3c4 \uc568\ubc94\uc758 \uccab \uace1\uc774\uc790 \uc568\ubc94\uc744 \ub300\ud45c\ud558\ub294 \uace1\uc778 \u3008Odd Eye\u3009\ub97c \uc791\uc0ac\u00b7\uc791\uace1\ud568\uc73c\ub85c\uc11c, \uc544\ud2f0\uc2a4\ud2b8\ub85c\uc11c\uc758 \ubaa8\uc2b5\uc744 \uc720\uac10 \uc5c6\uc774 \ubcf4\uc5ec\uc8fc\uc5c8\ub2e4. \ucd1d 11\uace1\uc774 \uc218\ub85d\ub418\uc5c8\uc73c\uba70, \uc74c\uc545 \ubc29\uc1a1\uc5d0\uc11c\ub294 \u3008View\u3009, \u3008Odd Eye\u3009, \u3008Love Sick\u3009, \u3008\uc7ac\uc5f0 (An Encore)\u3009 \ub4f1\uc758 \ubb34\ub300\uac00 \uacf5\uac1c\ub418\uc5c8\ub2e4. \ub610\ud55c \ud0c0\uc774\ud2c0\uace1 \u3008View\u3009\uc758 \ubba4\uc9c1\ube44\ub514\uc624\ub294 2015\ub144 5\uc6d4 \uc804\uc138\uacc4\uc5d0\uc11c \uac00\uc7a5 \ub9ce\uc774 \ubcf8 \ub3d9\uc601\uc0c1 1\uc704\uc5d0 \uc62c\ub790\ub2e4. \uc774\ud6c4 8\uc6d4 3\uc77c \ub9ac\ud328\ud0a4\uc9c0 \uc568\ubc94 \u300aMarried To The Music\u300b\ub97c \ubc1c\ub9e4\ud558\uba70 \ud6c4\uc18d\uace1 \ud65c\ub3d9\uc744 \uc2dc\uc791\ud558\uc600\ub2e4. \ud0c0\uc774\ud2c0 \uace1 \u3008Married To The Music\u3009\uc744 \ube44\ub86f\ud558\uc5ec \ucd1d 4\uace1\uc774 \ub354 \uc218\ub85d\ub418\uc5c8\uc73c\uba70, \ud2b9\ud788 \u3008Married To The Music\u3009\uc758 \ubba4\uc9c1\ube44\ub514\uc624\ub294 B\uae09 \ud638\ub7ec \ucee8\uc149\uc73c\ub85c \uc8fc\ubaa9\uc744 \ubc1b\uc558\ub2e4.", "answer": "\ub525 \ud558\uc6b0\uc2a4", "sentence": "\ud2b9\ud788 \ud0c0\uc774\ud2c0\uace1\uc778 \u3008View\u3009\ub294 \uc774\uc804\uae4c\uc9c0 \ud55c\uad6d\uc5d0\uc11c\ub294 \uc0dd\uc18c\ud588\ub358 \ub525 \ud558\uc6b0\uc2a4 \uc7a5\ub974\ub85c, \uc601\uad6d\uc758 \uc791\uace1\ud300 LDN Noise\uac00 \uc791\uace1\ud558\uc600\uc73c\uba70, \uba64\ubc84 \uc885\ud604\uc774 \ub0b4\ubd80 \uc791\uc0ac \ucf58\ud14c\uc2a4\ud2b8\uc5d0\uc11c 40:1\uc758 \uacbd\uc7c1\ub960\uc744 \ub6ab\uace0 \uc120\uc815\ub41c \uac00\uc0ac\ub97c \uc37c\ub2e4.", "paragraph_sentence": "\uc0e4\uc774\ub2c8\ub294 5\uc6d4 18\uc77c, 1\ub144 7\uac1c\uc6d4\uc5ec\uc758 \uacf5\ubc31\uc744 \uae68\uace0 \ub300\ud55c\ubbfc\uad6d \ud65c\ub3d9\uc73c\ub85c \ucef4\ubc31\ud558\uc600\ub2e4. \uc815\uaddc 4\uc9d1 \uc74c\ubc18 \u300aOdd\u300b\ub294 \uc774\uc804 \uc0e4\uc774\ub2c8\uc758 \ud589\ubcf4\uc5d0 \ube44\ucd94\uc5b4 \ubcf4\uba74 \uc774\ub840\uc801\uc73c\ub85c \uc790\uc815\uc5d0 \uc74c\uc6d0\uc774 \uacf5\uac1c\ub418\uc5c8\ub2e4. \ub3d9\uc2dc\uc5d0 \uc0e4\uc774\ub2c8\ub294 \uba64\ubc84 \uc885\ud604\uc774 \uc9c4\ud589\ud558\ub294 MBC \ub77c\ub514\uc624 \ud478\ub978\ubc24 \uc885\ud604\uc785\ub2c8\ub2e4\uc5d0 \uc2a4\ud398\uc15c \uac8c\uc2a4\ud2b8\ub85c \ucd08\ub300\ub418\uc5c8\ub2e4. \uc74c\uc6d0 \uc804\uace1\uc774 \uac01\uc885 \uc74c\uc6d0\ucc28\ud2b8\ub97c \ud729\uc4f8\uc5c8\uace0, \uc74c\uc545 \ubc29\uc1a1\uc5d0\uc11c\ub3c4 1\uc704\ub97c \ud729\uc4f8\uac8c \ub418\uc5c8\ub2e4. \ud2b9\ud788 \ud0c0\uc774\ud2c0\uace1\uc778 \u3008View\u3009\ub294 \uc774\uc804\uae4c\uc9c0 \ud55c\uad6d\uc5d0\uc11c\ub294 \uc0dd\uc18c\ud588\ub358 \ub525 \ud558\uc6b0\uc2a4 \uc7a5\ub974\ub85c, \uc601\uad6d\uc758 \uc791\uace1\ud300 LDN Noise\uac00 \uc791\uace1\ud558\uc600\uc73c\uba70, \uba64\ubc84 \uc885\ud604\uc774 \ub0b4\ubd80 \uc791\uc0ac \ucf58\ud14c\uc2a4\ud2b8\uc5d0\uc11c 40:1\uc758 \uacbd\uc7c1\ub960\uc744 \ub6ab\uace0 \uc120\uc815\ub41c \uac00\uc0ac\ub97c \uc37c\ub2e4. \uc885\ud604\uc740 \uc774 \uc678\uc5d0\ub3c4 \uc568\ubc94\uc758 \uccab \uace1\uc774\uc790 \uc568\ubc94\uc744 \ub300\ud45c\ud558\ub294 \uace1\uc778 \u3008Odd Eye\u3009\ub97c \uc791\uc0ac\u00b7\uc791\uace1\ud568\uc73c\ub85c\uc11c, \uc544\ud2f0\uc2a4\ud2b8\ub85c\uc11c\uc758 \ubaa8\uc2b5\uc744 \uc720\uac10 \uc5c6\uc774 \ubcf4\uc5ec\uc8fc\uc5c8\ub2e4. \ucd1d 11\uace1\uc774 \uc218\ub85d\ub418\uc5c8\uc73c\uba70, \uc74c\uc545 \ubc29\uc1a1\uc5d0\uc11c\ub294 \u3008View\u3009, \u3008Odd Eye\u3009, \u3008Love Sick\u3009, \u3008\uc7ac\uc5f0 (An Encore)\u3009 \ub4f1\uc758 \ubb34\ub300\uac00 \uacf5\uac1c\ub418\uc5c8\ub2e4. \ub610\ud55c \ud0c0\uc774\ud2c0\uace1 \u3008View\u3009\uc758 \ubba4\uc9c1\ube44\ub514\uc624\ub294 2015\ub144 5\uc6d4 \uc804\uc138\uacc4\uc5d0\uc11c \uac00\uc7a5 \ub9ce\uc774 \ubcf8 \ub3d9\uc601\uc0c1 1\uc704\uc5d0 \uc62c\ub790\ub2e4. \uc774\ud6c4 8\uc6d4 3\uc77c \ub9ac\ud328\ud0a4\uc9c0 \uc568\ubc94 \u300aMarried To The Music\u300b\ub97c \ubc1c\ub9e4\ud558\uba70 \ud6c4\uc18d\uace1 \ud65c\ub3d9\uc744 \uc2dc\uc791\ud558\uc600\ub2e4. \ud0c0\uc774\ud2c0 \uace1 \u3008Married To The Music\u3009\uc744 \ube44\ub86f\ud558\uc5ec \ucd1d 4\uace1\uc774 \ub354 \uc218\ub85d\ub418\uc5c8\uc73c\uba70, \ud2b9\ud788 \u3008Married To The Music\u3009\uc758 \ubba4\uc9c1\ube44\ub514\uc624\ub294 B\uae09 \ud638\ub7ec \ucee8\uc149\uc73c\ub85c \uc8fc\ubaa9\uc744 \ubc1b\uc558\ub2e4.", "paragraph_answer": "\uc0e4\uc774\ub2c8\ub294 5\uc6d4 18\uc77c, 1\ub144 7\uac1c\uc6d4\uc5ec\uc758 \uacf5\ubc31\uc744 \uae68\uace0 \ub300\ud55c\ubbfc\uad6d \ud65c\ub3d9\uc73c\ub85c \ucef4\ubc31\ud558\uc600\ub2e4. \uc815\uaddc 4\uc9d1 \uc74c\ubc18 \u300aOdd\u300b\ub294 \uc774\uc804 \uc0e4\uc774\ub2c8\uc758 \ud589\ubcf4\uc5d0 \ube44\ucd94\uc5b4 \ubcf4\uba74 \uc774\ub840\uc801\uc73c\ub85c \uc790\uc815\uc5d0 \uc74c\uc6d0\uc774 \uacf5\uac1c\ub418\uc5c8\ub2e4. \ub3d9\uc2dc\uc5d0 \uc0e4\uc774\ub2c8\ub294 \uba64\ubc84 \uc885\ud604\uc774 \uc9c4\ud589\ud558\ub294 MBC \ub77c\ub514\uc624 \ud478\ub978\ubc24 \uc885\ud604\uc785\ub2c8\ub2e4\uc5d0 \uc2a4\ud398\uc15c \uac8c\uc2a4\ud2b8\ub85c \ucd08\ub300\ub418\uc5c8\ub2e4. \uc74c\uc6d0 \uc804\uace1\uc774 \uac01\uc885 \uc74c\uc6d0\ucc28\ud2b8\ub97c \ud729\uc4f8\uc5c8\uace0, \uc74c\uc545 \ubc29\uc1a1\uc5d0\uc11c\ub3c4 1\uc704\ub97c \ud729\uc4f8\uac8c \ub418\uc5c8\ub2e4. \ud2b9\ud788 \ud0c0\uc774\ud2c0\uace1\uc778 \u3008View\u3009\ub294 \uc774\uc804\uae4c\uc9c0 \ud55c\uad6d\uc5d0\uc11c\ub294 \uc0dd\uc18c\ud588\ub358 \ub525 \ud558\uc6b0\uc2a4 \uc7a5\ub974\ub85c, \uc601\uad6d\uc758 \uc791\uace1\ud300 LDN Noise\uac00 \uc791\uace1\ud558\uc600\uc73c\uba70, \uba64\ubc84 \uc885\ud604\uc774 \ub0b4\ubd80 \uc791\uc0ac \ucf58\ud14c\uc2a4\ud2b8\uc5d0\uc11c 40:1\uc758 \uacbd\uc7c1\ub960\uc744 \ub6ab\uace0 \uc120\uc815\ub41c \uac00\uc0ac\ub97c \uc37c\ub2e4. \uc885\ud604\uc740 \uc774 \uc678\uc5d0\ub3c4 \uc568\ubc94\uc758 \uccab \uace1\uc774\uc790 \uc568\ubc94\uc744 \ub300\ud45c\ud558\ub294 \uace1\uc778 \u3008Odd Eye\u3009\ub97c \uc791\uc0ac\u00b7\uc791\uace1\ud568\uc73c\ub85c\uc11c, \uc544\ud2f0\uc2a4\ud2b8\ub85c\uc11c\uc758 \ubaa8\uc2b5\uc744 \uc720\uac10 \uc5c6\uc774 \ubcf4\uc5ec\uc8fc\uc5c8\ub2e4. \ucd1d 11\uace1\uc774 \uc218\ub85d\ub418\uc5c8\uc73c\uba70, \uc74c\uc545 \ubc29\uc1a1\uc5d0\uc11c\ub294 \u3008View\u3009, \u3008Odd Eye\u3009, \u3008Love Sick\u3009, \u3008\uc7ac\uc5f0 (An Encore)\u3009 \ub4f1\uc758 \ubb34\ub300\uac00 \uacf5\uac1c\ub418\uc5c8\ub2e4. \ub610\ud55c \ud0c0\uc774\ud2c0\uace1 \u3008View\u3009\uc758 \ubba4\uc9c1\ube44\ub514\uc624\ub294 2015\ub144 5\uc6d4 \uc804\uc138\uacc4\uc5d0\uc11c \uac00\uc7a5 \ub9ce\uc774 \ubcf8 \ub3d9\uc601\uc0c1 1\uc704\uc5d0 \uc62c\ub790\ub2e4. \uc774\ud6c4 8\uc6d4 3\uc77c \ub9ac\ud328\ud0a4\uc9c0 \uc568\ubc94 \u300aMarried To The Music\u300b\ub97c \ubc1c\ub9e4\ud558\uba70 \ud6c4\uc18d\uace1 \ud65c\ub3d9\uc744 \uc2dc\uc791\ud558\uc600\ub2e4. \ud0c0\uc774\ud2c0 \uace1 \u3008Married To The Music\u3009\uc744 \ube44\ub86f\ud558\uc5ec \ucd1d 4\uace1\uc774 \ub354 \uc218\ub85d\ub418\uc5c8\uc73c\uba70, \ud2b9\ud788 \u3008Married To The Music\u3009\uc758 \ubba4\uc9c1\ube44\ub514\uc624\ub294 B\uae09 \ud638\ub7ec \ucee8\uc149\uc73c\ub85c \uc8fc\ubaa9\uc744 \ubc1b\uc558\ub2e4.", "sentence_answer": "\ud2b9\ud788 \ud0c0\uc774\ud2c0\uace1\uc778 \u3008View\u3009\ub294 \uc774\uc804\uae4c\uc9c0 \ud55c\uad6d\uc5d0\uc11c\ub294 \uc0dd\uc18c\ud588\ub358 \ub525 \ud558\uc6b0\uc2a4 \uc7a5\ub974\ub85c, \uc601\uad6d\uc758 \uc791\uace1\ud300 LDN Noise\uac00 \uc791\uace1\ud558\uc600\uc73c\uba70, \uba64\ubc84 \uc885\ud604\uc774 \ub0b4\ubd80 \uc791\uc0ac \ucf58\ud14c\uc2a4\ud2b8\uc5d0\uc11c 40:1\uc758 \uacbd\uc7c1\ub960\uc744 \ub6ab\uace0 \uc120\uc815\ub41c \uac00\uc0ac\ub97c \uc37c\ub2e4.", "paragraph_id": "6511459-6-1", "paragraph_question": "question: View\uc758 \uc7a5\ub974\ub294?, context: \uc0e4\uc774\ub2c8\ub294 5\uc6d4 18\uc77c, 1\ub144 7\uac1c\uc6d4\uc5ec\uc758 \uacf5\ubc31\uc744 \uae68\uace0 \ub300\ud55c\ubbfc\uad6d \ud65c\ub3d9\uc73c\ub85c \ucef4\ubc31\ud558\uc600\ub2e4. \uc815\uaddc 4\uc9d1 \uc74c\ubc18 \u300aOdd\u300b\ub294 \uc774\uc804 \uc0e4\uc774\ub2c8\uc758 \ud589\ubcf4\uc5d0 \ube44\ucd94\uc5b4 \ubcf4\uba74 \uc774\ub840\uc801\uc73c\ub85c \uc790\uc815\uc5d0 \uc74c\uc6d0\uc774 \uacf5\uac1c\ub418\uc5c8\ub2e4. \ub3d9\uc2dc\uc5d0 \uc0e4\uc774\ub2c8\ub294 \uba64\ubc84 \uc885\ud604\uc774 \uc9c4\ud589\ud558\ub294 MBC \ub77c\ub514\uc624 \ud478\ub978\ubc24 \uc885\ud604\uc785\ub2c8\ub2e4\uc5d0 \uc2a4\ud398\uc15c \uac8c\uc2a4\ud2b8\ub85c \ucd08\ub300\ub418\uc5c8\ub2e4. \uc74c\uc6d0 \uc804\uace1\uc774 \uac01\uc885 \uc74c\uc6d0\ucc28\ud2b8\ub97c \ud729\uc4f8\uc5c8\uace0, \uc74c\uc545 \ubc29\uc1a1\uc5d0\uc11c\ub3c4 1\uc704\ub97c \ud729\uc4f8\uac8c \ub418\uc5c8\ub2e4. \ud2b9\ud788 \ud0c0\uc774\ud2c0\uace1\uc778 \u3008View\u3009\ub294 \uc774\uc804\uae4c\uc9c0 \ud55c\uad6d\uc5d0\uc11c\ub294 \uc0dd\uc18c\ud588\ub358 \ub525 \ud558\uc6b0\uc2a4 \uc7a5\ub974\ub85c, \uc601\uad6d\uc758 \uc791\uace1\ud300 LDN Noise\uac00 \uc791\uace1\ud558\uc600\uc73c\uba70, \uba64\ubc84 \uc885\ud604\uc774 \ub0b4\ubd80 \uc791\uc0ac \ucf58\ud14c\uc2a4\ud2b8\uc5d0\uc11c 40:1\uc758 \uacbd\uc7c1\ub960\uc744 \ub6ab\uace0 \uc120\uc815\ub41c \uac00\uc0ac\ub97c \uc37c\ub2e4. \uc885\ud604\uc740 \uc774 \uc678\uc5d0\ub3c4 \uc568\ubc94\uc758 \uccab \uace1\uc774\uc790 \uc568\ubc94\uc744 \ub300\ud45c\ud558\ub294 \uace1\uc778 \u3008Odd Eye\u3009\ub97c \uc791\uc0ac\u00b7\uc791\uace1\ud568\uc73c\ub85c\uc11c, \uc544\ud2f0\uc2a4\ud2b8\ub85c\uc11c\uc758 \ubaa8\uc2b5\uc744 \uc720\uac10 \uc5c6\uc774 \ubcf4\uc5ec\uc8fc\uc5c8\ub2e4. \ucd1d 11\uace1\uc774 \uc218\ub85d\ub418\uc5c8\uc73c\uba70, \uc74c\uc545 \ubc29\uc1a1\uc5d0\uc11c\ub294 \u3008View\u3009, \u3008Odd Eye\u3009, \u3008Love Sick\u3009, \u3008\uc7ac\uc5f0 (An Encore)\u3009 \ub4f1\uc758 \ubb34\ub300\uac00 \uacf5\uac1c\ub418\uc5c8\ub2e4. \ub610\ud55c \ud0c0\uc774\ud2c0\uace1 \u3008View\u3009\uc758 \ubba4\uc9c1\ube44\ub514\uc624\ub294 2015\ub144 5\uc6d4 \uc804\uc138\uacc4\uc5d0\uc11c \uac00\uc7a5 \ub9ce\uc774 \ubcf8 \ub3d9\uc601\uc0c1 1\uc704\uc5d0 \uc62c\ub790\ub2e4. \uc774\ud6c4 8\uc6d4 3\uc77c \ub9ac\ud328\ud0a4\uc9c0 \uc568\ubc94 \u300aMarried To The Music\u300b\ub97c \ubc1c\ub9e4\ud558\uba70 \ud6c4\uc18d\uace1 \ud65c\ub3d9\uc744 \uc2dc\uc791\ud558\uc600\ub2e4. \ud0c0\uc774\ud2c0 \uace1 \u3008Married To The Music\u3009\uc744 \ube44\ub86f\ud558\uc5ec \ucd1d 4\uace1\uc774 \ub354 \uc218\ub85d\ub418\uc5c8\uc73c\uba70, \ud2b9\ud788 \u3008Married To The Music\u3009\uc758 \ubba4\uc9c1\ube44\ub514\uc624\ub294 B\uae09 \ud638\ub7ec \ucee8\uc149\uc73c\ub85c \uc8fc\ubaa9\uc744 \ubc1b\uc558\ub2e4."} {"question": "2\ub9c8\ub9ac\uac00 \ub3d9\uc2dc\uc5d0 \ub098\uc624\ub294 \uc2a4\ud14c\uc774\uc9c0\ub294?", "paragraph": "\ubb3c\ub860 \uc774\uac83\ub3c4 EX \ubaa8\ub4dc \ubc84\uc804\uc774 \ub530\ub85c \uc788\ub2e4. \ub2e4\ub9cc \uccb4\ub825 \uc0ac\uc591\uc774 EX \ubaa8\ub4dc\ucc98\ub7fc \uc808\ubc18 \uc218\uc900\uc774 \uc544\ub2cc \uba54\uc778\ubaa8\ub4dc \ub9cc\ud07c\uc774\uba70, \ud68c\ubcf5\uc218\ub2e8\ub3c4 3\uac1c\ubfd0\uc774\uc9c0\ub9cc \ub9e5\uc2ec\ud1a0\ub9c8\ud1a0\uac00 \uc8fc\uc5b4\uc9c4\ub2e4. \ub300\uc2e0 \ud734\uc2dd\uacf5\uac04\uc5d0\uc11c \uc8fc\uc5b4\uc9c0\ub294 \uce74\ud53c \uc5d0\uc13c\uc2a4\uc911 \ud558\ub098\uac00 \ubc18\ub4dc\uc2dc \uc2ac\ub9bd\uc778\uac74 \uadf8\ub300\ub85c\ub2e4. \uc2a4\ud14c\uc774\uc9c0\uc218\ub294 HR-D3(\uba54\ud0c8 \uc81c\ub808\ub110 2\ucc28\uc804\uacfc 3\ucc28\uc804)\uc2a4\ud14c\uc774\uc9c0\uc640 \uac24\ub7ed\ud2f1 \ub098\uc774\ud2b8(\ub9e4\uc6b0 \ud76c\uadc0)\uc2a4\ud14c\uc774\uc9c0\uac00 \ucd94\uac00\ub418\uc5b4 \ucd1d 15\uc2a4\ud14c\uc774\uc9c0. 10\ubc88\uc9f8 \uae4c\uc9c0\ub294 \ub79c\ub364\uc73c\ub85c \ub4f1\uc7a5\ud558\uace0 \uadf8 \ub4a4 5\uc2a4\ud14c\uc774\uc9c0\ub294 \ub85c\uc544 - HR-D3 - \uac24\ub7ed\ud2f1 \ub098\uc774\ud2b8 - \ub9c8\ubc84\ub85c\uc544 - \ub9c8\ubc84\ub85c\uc544 \uc18c\uc6b8\ub85c \uace0\uc815\ub418\uba70, \uc2a4\ud14c\uc774\uc9c0\ub85c \uc785\uc7a5\ud558\ub294 \ubb38\uc774 \uc774\uacf5\uac04 \ud3ec\ud138\ub85c \ubc14\ub010\ub2e4. \uc989 \ud1a0\ub9c8\ud1a0 \uc0c1\uc790\ub97c \uc4f8 \uc218 \uc5c6\ub2e4. \uadf8 \uc678\uc758 \ucc28\uc774\uc810\uc740 \uc911\ubcf4\uc2a4 \ub7ec\uc2dc \uc2a4\ud14c\uc774\uc9c0\uc5d0\uc11c \ub9c8\uc9c0\ub9c9 2\ub9c8\ub9ac\uac00 \ub3d9\uc2dc\uc5d0 \ub098\uc628\ub2e4. \uc288\ud37c \ub2a5\ub825\uc740 \ub098\uc624\uc9c0 \uc54a\ub294\ub2e4. \ub05d", "answer": "\uc911\ubcf4\uc2a4 \ub7ec\uc2dc \uc2a4\ud14c\uc774\uc9c0", "sentence": "\uadf8 \uc678\uc758 \ucc28\uc774\uc810\uc740 \uc911\ubcf4\uc2a4 \ub7ec\uc2dc \uc2a4\ud14c\uc774\uc9c0 \uc5d0\uc11c \ub9c8\uc9c0\ub9c9 2\ub9c8\ub9ac\uac00 \ub3d9\uc2dc\uc5d0 \ub098\uc628\ub2e4.", "paragraph_sentence": "\ubb3c\ub860 \uc774\uac83\ub3c4 EX \ubaa8\ub4dc \ubc84\uc804\uc774 \ub530\ub85c \uc788\ub2e4. \ub2e4\ub9cc \uccb4\ub825 \uc0ac\uc591\uc774 EX \ubaa8\ub4dc\ucc98\ub7fc \uc808\ubc18 \uc218\uc900\uc774 \uc544\ub2cc \uba54\uc778\ubaa8\ub4dc \ub9cc\ud07c\uc774\uba70, \ud68c\ubcf5\uc218\ub2e8\ub3c4 3\uac1c\ubfd0\uc774\uc9c0\ub9cc \ub9e5\uc2ec\ud1a0\ub9c8\ud1a0\uac00 \uc8fc\uc5b4\uc9c4\ub2e4. \ub300\uc2e0 \ud734\uc2dd\uacf5\uac04\uc5d0\uc11c \uc8fc\uc5b4\uc9c0\ub294 \uce74\ud53c \uc5d0\uc13c\uc2a4\uc911 \ud558\ub098\uac00 \ubc18\ub4dc\uc2dc \uc2ac\ub9bd\uc778\uac74 \uadf8\ub300\ub85c\ub2e4. \uc2a4\ud14c\uc774\uc9c0\uc218\ub294 HR-D3(\uba54\ud0c8 \uc81c\ub808\ub110 2\ucc28\uc804\uacfc 3\ucc28\uc804)\uc2a4\ud14c\uc774\uc9c0\uc640 \uac24\ub7ed\ud2f1 \ub098\uc774\ud2b8(\ub9e4\uc6b0 \ud76c\uadc0)\uc2a4\ud14c\uc774\uc9c0\uac00 \ucd94\uac00\ub418\uc5b4 \ucd1d 15\uc2a4\ud14c\uc774\uc9c0. 10\ubc88\uc9f8 \uae4c\uc9c0\ub294 \ub79c\ub364\uc73c\ub85c \ub4f1\uc7a5\ud558\uace0 \uadf8 \ub4a4 5\uc2a4\ud14c\uc774\uc9c0\ub294 \ub85c\uc544 - HR-D3 - \uac24\ub7ed\ud2f1 \ub098\uc774\ud2b8 - \ub9c8\ubc84\ub85c\uc544 - \ub9c8\ubc84\ub85c\uc544 \uc18c\uc6b8\ub85c \uace0\uc815\ub418\uba70, \uc2a4\ud14c\uc774\uc9c0\ub85c \uc785\uc7a5\ud558\ub294 \ubb38\uc774 \uc774\uacf5\uac04 \ud3ec\ud138\ub85c \ubc14\ub010\ub2e4. \uc989 \ud1a0\ub9c8\ud1a0 \uc0c1\uc790\ub97c \uc4f8 \uc218 \uc5c6\ub2e4. \uadf8 \uc678\uc758 \ucc28\uc774\uc810\uc740 \uc911\ubcf4\uc2a4 \ub7ec\uc2dc \uc2a4\ud14c\uc774\uc9c0 \uc5d0\uc11c \ub9c8\uc9c0\ub9c9 2\ub9c8\ub9ac\uac00 \ub3d9\uc2dc\uc5d0 \ub098\uc628\ub2e4. \uc288\ud37c \ub2a5\ub825\uc740 \ub098\uc624\uc9c0 \uc54a\ub294\ub2e4. \ub05d", "paragraph_answer": "\ubb3c\ub860 \uc774\uac83\ub3c4 EX \ubaa8\ub4dc \ubc84\uc804\uc774 \ub530\ub85c \uc788\ub2e4. \ub2e4\ub9cc \uccb4\ub825 \uc0ac\uc591\uc774 EX \ubaa8\ub4dc\ucc98\ub7fc \uc808\ubc18 \uc218\uc900\uc774 \uc544\ub2cc \uba54\uc778\ubaa8\ub4dc \ub9cc\ud07c\uc774\uba70, \ud68c\ubcf5\uc218\ub2e8\ub3c4 3\uac1c\ubfd0\uc774\uc9c0\ub9cc \ub9e5\uc2ec\ud1a0\ub9c8\ud1a0\uac00 \uc8fc\uc5b4\uc9c4\ub2e4. \ub300\uc2e0 \ud734\uc2dd\uacf5\uac04\uc5d0\uc11c \uc8fc\uc5b4\uc9c0\ub294 \uce74\ud53c \uc5d0\uc13c\uc2a4\uc911 \ud558\ub098\uac00 \ubc18\ub4dc\uc2dc \uc2ac\ub9bd\uc778\uac74 \uadf8\ub300\ub85c\ub2e4. \uc2a4\ud14c\uc774\uc9c0\uc218\ub294 HR-D3(\uba54\ud0c8 \uc81c\ub808\ub110 2\ucc28\uc804\uacfc 3\ucc28\uc804)\uc2a4\ud14c\uc774\uc9c0\uc640 \uac24\ub7ed\ud2f1 \ub098\uc774\ud2b8(\ub9e4\uc6b0 \ud76c\uadc0)\uc2a4\ud14c\uc774\uc9c0\uac00 \ucd94\uac00\ub418\uc5b4 \ucd1d 15\uc2a4\ud14c\uc774\uc9c0. 10\ubc88\uc9f8 \uae4c\uc9c0\ub294 \ub79c\ub364\uc73c\ub85c \ub4f1\uc7a5\ud558\uace0 \uadf8 \ub4a4 5\uc2a4\ud14c\uc774\uc9c0\ub294 \ub85c\uc544 - HR-D3 - \uac24\ub7ed\ud2f1 \ub098\uc774\ud2b8 - \ub9c8\ubc84\ub85c\uc544 - \ub9c8\ubc84\ub85c\uc544 \uc18c\uc6b8\ub85c \uace0\uc815\ub418\uba70, \uc2a4\ud14c\uc774\uc9c0\ub85c \uc785\uc7a5\ud558\ub294 \ubb38\uc774 \uc774\uacf5\uac04 \ud3ec\ud138\ub85c \ubc14\ub010\ub2e4. \uc989 \ud1a0\ub9c8\ud1a0 \uc0c1\uc790\ub97c \uc4f8 \uc218 \uc5c6\ub2e4. \uadf8 \uc678\uc758 \ucc28\uc774\uc810\uc740 \uc911\ubcf4\uc2a4 \ub7ec\uc2dc \uc2a4\ud14c\uc774\uc9c0 \uc5d0\uc11c \ub9c8\uc9c0\ub9c9 2\ub9c8\ub9ac\uac00 \ub3d9\uc2dc\uc5d0 \ub098\uc628\ub2e4. \uc288\ud37c \ub2a5\ub825\uc740 \ub098\uc624\uc9c0 \uc54a\ub294\ub2e4. \ub05d", "sentence_answer": "\uadf8 \uc678\uc758 \ucc28\uc774\uc810\uc740 \uc911\ubcf4\uc2a4 \ub7ec\uc2dc \uc2a4\ud14c\uc774\uc9c0 \uc5d0\uc11c \ub9c8\uc9c0\ub9c9 2\ub9c8\ub9ac\uac00 \ub3d9\uc2dc\uc5d0 \ub098\uc628\ub2e4.", "paragraph_id": "6112589-4-0", "paragraph_question": "question: 2\ub9c8\ub9ac\uac00 \ub3d9\uc2dc\uc5d0 \ub098\uc624\ub294 \uc2a4\ud14c\uc774\uc9c0\ub294?, context: \ubb3c\ub860 \uc774\uac83\ub3c4 EX \ubaa8\ub4dc \ubc84\uc804\uc774 \ub530\ub85c \uc788\ub2e4. \ub2e4\ub9cc \uccb4\ub825 \uc0ac\uc591\uc774 EX \ubaa8\ub4dc\ucc98\ub7fc \uc808\ubc18 \uc218\uc900\uc774 \uc544\ub2cc \uba54\uc778\ubaa8\ub4dc \ub9cc\ud07c\uc774\uba70, \ud68c\ubcf5\uc218\ub2e8\ub3c4 3\uac1c\ubfd0\uc774\uc9c0\ub9cc \ub9e5\uc2ec\ud1a0\ub9c8\ud1a0\uac00 \uc8fc\uc5b4\uc9c4\ub2e4. \ub300\uc2e0 \ud734\uc2dd\uacf5\uac04\uc5d0\uc11c \uc8fc\uc5b4\uc9c0\ub294 \uce74\ud53c \uc5d0\uc13c\uc2a4\uc911 \ud558\ub098\uac00 \ubc18\ub4dc\uc2dc \uc2ac\ub9bd\uc778\uac74 \uadf8\ub300\ub85c\ub2e4. \uc2a4\ud14c\uc774\uc9c0\uc218\ub294 HR-D3(\uba54\ud0c8 \uc81c\ub808\ub110 2\ucc28\uc804\uacfc 3\ucc28\uc804)\uc2a4\ud14c\uc774\uc9c0\uc640 \uac24\ub7ed\ud2f1 \ub098\uc774\ud2b8(\ub9e4\uc6b0 \ud76c\uadc0)\uc2a4\ud14c\uc774\uc9c0\uac00 \ucd94\uac00\ub418\uc5b4 \ucd1d 15\uc2a4\ud14c\uc774\uc9c0. 10\ubc88\uc9f8 \uae4c\uc9c0\ub294 \ub79c\ub364\uc73c\ub85c \ub4f1\uc7a5\ud558\uace0 \uadf8 \ub4a4 5\uc2a4\ud14c\uc774\uc9c0\ub294 \ub85c\uc544 - HR-D3 - \uac24\ub7ed\ud2f1 \ub098\uc774\ud2b8 - \ub9c8\ubc84\ub85c\uc544 - \ub9c8\ubc84\ub85c\uc544 \uc18c\uc6b8\ub85c \uace0\uc815\ub418\uba70, \uc2a4\ud14c\uc774\uc9c0\ub85c \uc785\uc7a5\ud558\ub294 \ubb38\uc774 \uc774\uacf5\uac04 \ud3ec\ud138\ub85c \ubc14\ub010\ub2e4. \uc989 \ud1a0\ub9c8\ud1a0 \uc0c1\uc790\ub97c \uc4f8 \uc218 \uc5c6\ub2e4. \uadf8 \uc678\uc758 \ucc28\uc774\uc810\uc740 \uc911\ubcf4\uc2a4 \ub7ec\uc2dc \uc2a4\ud14c\uc774\uc9c0\uc5d0\uc11c \ub9c8\uc9c0\ub9c9 2\ub9c8\ub9ac\uac00 \ub3d9\uc2dc\uc5d0 \ub098\uc628\ub2e4. \uc288\ud37c \ub2a5\ub825\uc740 \ub098\uc624\uc9c0 \uc54a\ub294\ub2e4. \ub05d"} {"question": "\uc724\uc9c0\ub294 \uac15\ud76c\uc81c\uc758 \ub300\ud3b8\ucc2c\uc0ac\uc5c5\uc744 \ucd1d\uad04\ud558\uae30\ub9cc \ud55c \ubfd0 \ubb34\uc5c7\uc5d0\ub294 \uc2e0\uacbd\uc744 \uc4f0\uc9c0 \uc54a\uc558\ub098\uc694?", "paragraph": "\uadf8\ub7ec\ub098 \ud669\uc7a5\uc790 \uc724\uc2dc\ub294 \ubd80\ud669\uc5d0\uac8c \ud3d0\ud0dc\uc790 \uc724\uc789\uc744 \uc790\uae30 \uc190\uc73c\ub85c \uc8fd\uc774\uaca0\ub2e4\ub294 \ub9d0\uc744 \ud558\uc5ec \ubd80\ud669\uc758 \ubd84\ub178\ub97c \uc0c0\uace0, \uc774\ub85c \uc778\ud558\uc5ec \uc724\uc2dc\uc758 \uc791\uc704\ub294 \ubc15\ud0c8\ub418\uace0 \uc724\uc2dc\ub294 \uc720\ud3d0\ub418\uc5c8\ub2e4. 3\ub0a8\uc778 \uc724\uc9c0\ub294 \uc724\uc2dc\uc640 \uc724\uc789\uc774 \uc720\ud3d0\ub41c \uc774\ud6c4\ub85c \uc7a5\uc790 \ud589\uc138\ub97c \ud558\uc600\uc73c\ub098 \uac15\ud76c\uc81c\uc758 \ub300\ud3b8\ucc2c\uc0ac\uc5c5\uc744 \ucd1d\uad04\ud558\uae30\ub9cc \ud560 \ubfd0, \uc815\uce58\uc5d0\ub294 \uc2e0\uacbd\uc744 \uc4f0\uc9c0 \uc54a\uc558\ub2e4. \uc724\uc0ac\ub294 \uc724\uc9c4\uacfc \ud568\uaed8 \ud669\uc790\ub4e4 \uc911 \uac00\uc7a5 \uc601\ud2b9\ud558\uc600\uc73c\ub098 \uc724\uc789\uc774 \ud3d0\uc704\ub2f9\ud55c \uc774\ud6c4 \ub178\uace8\uc801\uc73c\ub85c \ud0dc\uc790\uc9c1\uc740 \ubb3c\ub860 \ud669\uc704\ub97c \ub178\ub838\uace0 \uc774 \uc18c\ubb38\uc774 \ub4e4\ub9ac\uc790, \uac15\ud76c\uc81c\ub294 \uc724\uc0ac\ub97c \ub9e4\uc6b0 \uc2eb\uc5b4\ud558\uace0 \uc724\uc0ac\uc758 \ubb38\uc548 \ubc1b\uae30\ub97c \uac70\ubd80\ud558\uc600\ub2e4. \uac15\ud76c\uc81c\uac00 \uc724\uc0ac\ub97c \ubd88\ub7ec\ub4e4\uc5ec \uc870\uc6a9\ud788 \uc0b4\ub77c\uace0 \ub9d0\uc744 \ud558\uc600\uc74c\uc5d0\ub3c4 \uc724\uc0ac\ub294 \uc790\uc2e0\uc758 \uc57c\uc2ec\uc744 \ub178\uace8\uc801\uc73c\ub85c \ub4dc\ub7ec\ub0c8\uace0 \uc774\uc5d0 \ub300\ud55c \uc870\uce58\ub85c \uac15\ud76c\uc81c\ub294 \uc724\uc0ac\ub97c \ud55c \ub2ec\uac04 \ub0c9\uad81\uc5d0 \uc720\ud3d0\uc2dc\ud0a4\uae30\ub3c4 \ud558\uc600\ub2e4. 14\ud669\uc790 \uc724\uc81c\ub294 \uc724\uc9c4\uc758 \ub3d9\ubcf5\uc544\uc6b0\ub85c \uac15\ud76c\uc81c\uc640 \uac19\uc774 \ub9ce\uc740 \uc6d0\uc815\uc744 \ub2e4\ub140\uc654\uace0, \ubd81\ucabd\uc758 \uad6d\uacbd\uc744 \uc9c0\ud0a4\ub294 \ubb34\uc6d0\ub300\uc7a5\uad70(\u64ab\u9060\u5927\u5c07\u8ecd)\uc744 \uad00\uc9c1\uc73c\ub85c \ubc1b\uc558\ub2e4. \ubb34\uc6d0\ub300\uc7a5\uad70\uc9c1\uc740 \ud754\ud788 \ud669\uc704\ub97c \uc774\uc5b4\ubc1b\uc744 \ud6c4\uacc4\uc790\ub098 \ub2e4\ub984\uc5c6\ub294 \uc790\ub9ac\uc5c8\uae30\uc5d0 \uc77c\ubd80 \ud559\uc790\ub4e4\uc740 \uac15\ud76c\uc81c\uac00 \ud669\uc704\ub85c \uc549\ud790 \uc724\uc81c\uc5d0\uac8c \ubbf8\ub9ac \ub300\uc7a5\uad70\uc758 \uc9c1\ud568\uc744 \ub0b4\ub824\uc900 \uac83\uc73c\ub85c \ud574\uc11d\ud55c\ub2e4. \uadf8\ub7ec\ub098 \uac15\ud76c\uc81c\ub294 \uad70\uc0ac\uc801\uc778 \uc77c\uc740 \ubaa8\ub450 \uc724\uc81c\uc640 \uc758\ub17c\ud558\uc600\uc73c\ub098 \uc815\uce58\uc640 \ub0b4\uc815\uc5d0 \uad00\ud55c \uc77c\uc740 \uc774\ubbf8 \uc624\ub7ab\ub3d9\uc548 \uc870\uc815\uc5d0\uc11c \ud65c\uc57d\ud558\ub358 \uc724\uc9c4\uc5d0\uac8c \ubb3c\uc5b4\ubcf4\uace0 \ucd98\uacc4\uc640 \ucd94\uacc4\uc5d0 \uace1\ubd80\uc5d0\uc11c \uc5f4\ub9ac\ub294 \uacf5\uc790\uc758 \uc81c\uc0ac\ub97c \uc724\uc9c4\uc5d0\uac8c \ub9e1\uacbc\ub2e4.", "answer": "\uc815\uce58", "sentence": "3\ub0a8\uc778 \uc724\uc9c0\ub294 \uc724\uc2dc\uc640 \uc724\uc789\uc774 \uc720\ud3d0\ub41c \uc774\ud6c4\ub85c \uc7a5\uc790 \ud589\uc138\ub97c \ud558\uc600\uc73c\ub098 \uac15\ud76c\uc81c\uc758 \ub300\ud3b8\ucc2c\uc0ac\uc5c5\uc744 \ucd1d\uad04\ud558\uae30\ub9cc \ud560 \ubfd0, \uc815\uce58 \uc5d0\ub294 \uc2e0\uacbd\uc744 \uc4f0\uc9c0 \uc54a\uc558\ub2e4.", "paragraph_sentence": "\uadf8\ub7ec\ub098 \ud669\uc7a5\uc790 \uc724\uc2dc\ub294 \ubd80\ud669\uc5d0\uac8c \ud3d0\ud0dc\uc790 \uc724\uc789\uc744 \uc790\uae30 \uc190\uc73c\ub85c \uc8fd\uc774\uaca0\ub2e4\ub294 \ub9d0\uc744 \ud558\uc5ec \ubd80\ud669\uc758 \ubd84\ub178\ub97c \uc0c0\uace0, \uc774\ub85c \uc778\ud558\uc5ec \uc724\uc2dc\uc758 \uc791\uc704\ub294 \ubc15\ud0c8\ub418\uace0 \uc724\uc2dc\ub294 \uc720\ud3d0\ub418\uc5c8\ub2e4. 3\ub0a8\uc778 \uc724\uc9c0\ub294 \uc724\uc2dc\uc640 \uc724\uc789\uc774 \uc720\ud3d0\ub41c \uc774\ud6c4\ub85c \uc7a5\uc790 \ud589\uc138\ub97c \ud558\uc600\uc73c\ub098 \uac15\ud76c\uc81c\uc758 \ub300\ud3b8\ucc2c\uc0ac\uc5c5\uc744 \ucd1d\uad04\ud558\uae30\ub9cc \ud560 \ubfd0, \uc815\uce58 \uc5d0\ub294 \uc2e0\uacbd\uc744 \uc4f0\uc9c0 \uc54a\uc558\ub2e4. \uc724\uc0ac\ub294 \uc724\uc9c4\uacfc \ud568\uaed8 \ud669\uc790\ub4e4 \uc911 \uac00\uc7a5 \uc601\ud2b9\ud558\uc600\uc73c\ub098 \uc724\uc789\uc774 \ud3d0\uc704\ub2f9\ud55c \uc774\ud6c4 \ub178\uace8\uc801\uc73c\ub85c \ud0dc\uc790\uc9c1\uc740 \ubb3c\ub860 \ud669\uc704\ub97c \ub178\ub838\uace0 \uc774 \uc18c\ubb38\uc774 \ub4e4\ub9ac\uc790, \uac15\ud76c\uc81c\ub294 \uc724\uc0ac\ub97c \ub9e4\uc6b0 \uc2eb\uc5b4\ud558\uace0 \uc724\uc0ac\uc758 \ubb38\uc548 \ubc1b\uae30\ub97c \uac70\ubd80\ud558\uc600\ub2e4. \uac15\ud76c\uc81c\uac00 \uc724\uc0ac\ub97c \ubd88\ub7ec\ub4e4\uc5ec \uc870\uc6a9\ud788 \uc0b4\ub77c\uace0 \ub9d0\uc744 \ud558\uc600\uc74c\uc5d0\ub3c4 \uc724\uc0ac\ub294 \uc790\uc2e0\uc758 \uc57c\uc2ec\uc744 \ub178\uace8\uc801\uc73c\ub85c \ub4dc\ub7ec\ub0c8\uace0 \uc774\uc5d0 \ub300\ud55c \uc870\uce58\ub85c \uac15\ud76c\uc81c\ub294 \uc724\uc0ac\ub97c \ud55c \ub2ec\uac04 \ub0c9\uad81\uc5d0 \uc720\ud3d0\uc2dc\ud0a4\uae30\ub3c4 \ud558\uc600\ub2e4. 14\ud669\uc790 \uc724\uc81c\ub294 \uc724\uc9c4\uc758 \ub3d9\ubcf5\uc544\uc6b0\ub85c \uac15\ud76c\uc81c\uc640 \uac19\uc774 \ub9ce\uc740 \uc6d0\uc815\uc744 \ub2e4\ub140\uc654\uace0, \ubd81\ucabd\uc758 \uad6d\uacbd\uc744 \uc9c0\ud0a4\ub294 \ubb34\uc6d0\ub300\uc7a5\uad70(\u64ab\u9060\u5927\u5c07\u8ecd)\uc744 \uad00\uc9c1\uc73c\ub85c \ubc1b\uc558\ub2e4. \ubb34\uc6d0\ub300\uc7a5\uad70\uc9c1\uc740 \ud754\ud788 \ud669\uc704\ub97c \uc774\uc5b4\ubc1b\uc744 \ud6c4\uacc4\uc790\ub098 \ub2e4\ub984\uc5c6\ub294 \uc790\ub9ac\uc5c8\uae30\uc5d0 \uc77c\ubd80 \ud559\uc790\ub4e4\uc740 \uac15\ud76c\uc81c\uac00 \ud669\uc704\ub85c \uc549\ud790 \uc724\uc81c\uc5d0\uac8c \ubbf8\ub9ac \ub300\uc7a5\uad70\uc758 \uc9c1\ud568\uc744 \ub0b4\ub824\uc900 \uac83\uc73c\ub85c \ud574\uc11d\ud55c\ub2e4. \uadf8\ub7ec\ub098 \uac15\ud76c\uc81c\ub294 \uad70\uc0ac\uc801\uc778 \uc77c\uc740 \ubaa8\ub450 \uc724\uc81c\uc640 \uc758\ub17c\ud558\uc600\uc73c\ub098 \uc815\uce58\uc640 \ub0b4\uc815\uc5d0 \uad00\ud55c \uc77c\uc740 \uc774\ubbf8 \uc624\ub7ab\ub3d9\uc548 \uc870\uc815\uc5d0\uc11c \ud65c\uc57d\ud558\ub358 \uc724\uc9c4\uc5d0\uac8c \ubb3c\uc5b4\ubcf4\uace0 \ucd98\uacc4\uc640 \ucd94\uacc4\uc5d0 \uace1\ubd80\uc5d0\uc11c \uc5f4\ub9ac\ub294 \uacf5\uc790\uc758 \uc81c\uc0ac\ub97c \uc724\uc9c4\uc5d0\uac8c \ub9e1\uacbc\ub2e4.", "paragraph_answer": "\uadf8\ub7ec\ub098 \ud669\uc7a5\uc790 \uc724\uc2dc\ub294 \ubd80\ud669\uc5d0\uac8c \ud3d0\ud0dc\uc790 \uc724\uc789\uc744 \uc790\uae30 \uc190\uc73c\ub85c \uc8fd\uc774\uaca0\ub2e4\ub294 \ub9d0\uc744 \ud558\uc5ec \ubd80\ud669\uc758 \ubd84\ub178\ub97c \uc0c0\uace0, \uc774\ub85c \uc778\ud558\uc5ec \uc724\uc2dc\uc758 \uc791\uc704\ub294 \ubc15\ud0c8\ub418\uace0 \uc724\uc2dc\ub294 \uc720\ud3d0\ub418\uc5c8\ub2e4. 3\ub0a8\uc778 \uc724\uc9c0\ub294 \uc724\uc2dc\uc640 \uc724\uc789\uc774 \uc720\ud3d0\ub41c \uc774\ud6c4\ub85c \uc7a5\uc790 \ud589\uc138\ub97c \ud558\uc600\uc73c\ub098 \uac15\ud76c\uc81c\uc758 \ub300\ud3b8\ucc2c\uc0ac\uc5c5\uc744 \ucd1d\uad04\ud558\uae30\ub9cc \ud560 \ubfd0, \uc815\uce58 \uc5d0\ub294 \uc2e0\uacbd\uc744 \uc4f0\uc9c0 \uc54a\uc558\ub2e4. \uc724\uc0ac\ub294 \uc724\uc9c4\uacfc \ud568\uaed8 \ud669\uc790\ub4e4 \uc911 \uac00\uc7a5 \uc601\ud2b9\ud558\uc600\uc73c\ub098 \uc724\uc789\uc774 \ud3d0\uc704\ub2f9\ud55c \uc774\ud6c4 \ub178\uace8\uc801\uc73c\ub85c \ud0dc\uc790\uc9c1\uc740 \ubb3c\ub860 \ud669\uc704\ub97c \ub178\ub838\uace0 \uc774 \uc18c\ubb38\uc774 \ub4e4\ub9ac\uc790, \uac15\ud76c\uc81c\ub294 \uc724\uc0ac\ub97c \ub9e4\uc6b0 \uc2eb\uc5b4\ud558\uace0 \uc724\uc0ac\uc758 \ubb38\uc548 \ubc1b\uae30\ub97c \uac70\ubd80\ud558\uc600\ub2e4. \uac15\ud76c\uc81c\uac00 \uc724\uc0ac\ub97c \ubd88\ub7ec\ub4e4\uc5ec \uc870\uc6a9\ud788 \uc0b4\ub77c\uace0 \ub9d0\uc744 \ud558\uc600\uc74c\uc5d0\ub3c4 \uc724\uc0ac\ub294 \uc790\uc2e0\uc758 \uc57c\uc2ec\uc744 \ub178\uace8\uc801\uc73c\ub85c \ub4dc\ub7ec\ub0c8\uace0 \uc774\uc5d0 \ub300\ud55c \uc870\uce58\ub85c \uac15\ud76c\uc81c\ub294 \uc724\uc0ac\ub97c \ud55c \ub2ec\uac04 \ub0c9\uad81\uc5d0 \uc720\ud3d0\uc2dc\ud0a4\uae30\ub3c4 \ud558\uc600\ub2e4. 14\ud669\uc790 \uc724\uc81c\ub294 \uc724\uc9c4\uc758 \ub3d9\ubcf5\uc544\uc6b0\ub85c \uac15\ud76c\uc81c\uc640 \uac19\uc774 \ub9ce\uc740 \uc6d0\uc815\uc744 \ub2e4\ub140\uc654\uace0, \ubd81\ucabd\uc758 \uad6d\uacbd\uc744 \uc9c0\ud0a4\ub294 \ubb34\uc6d0\ub300\uc7a5\uad70(\u64ab\u9060\u5927\u5c07\u8ecd)\uc744 \uad00\uc9c1\uc73c\ub85c \ubc1b\uc558\ub2e4. \ubb34\uc6d0\ub300\uc7a5\uad70\uc9c1\uc740 \ud754\ud788 \ud669\uc704\ub97c \uc774\uc5b4\ubc1b\uc744 \ud6c4\uacc4\uc790\ub098 \ub2e4\ub984\uc5c6\ub294 \uc790\ub9ac\uc5c8\uae30\uc5d0 \uc77c\ubd80 \ud559\uc790\ub4e4\uc740 \uac15\ud76c\uc81c\uac00 \ud669\uc704\ub85c \uc549\ud790 \uc724\uc81c\uc5d0\uac8c \ubbf8\ub9ac \ub300\uc7a5\uad70\uc758 \uc9c1\ud568\uc744 \ub0b4\ub824\uc900 \uac83\uc73c\ub85c \ud574\uc11d\ud55c\ub2e4. \uadf8\ub7ec\ub098 \uac15\ud76c\uc81c\ub294 \uad70\uc0ac\uc801\uc778 \uc77c\uc740 \ubaa8\ub450 \uc724\uc81c\uc640 \uc758\ub17c\ud558\uc600\uc73c\ub098 \uc815\uce58\uc640 \ub0b4\uc815\uc5d0 \uad00\ud55c \uc77c\uc740 \uc774\ubbf8 \uc624\ub7ab\ub3d9\uc548 \uc870\uc815\uc5d0\uc11c \ud65c\uc57d\ud558\ub358 \uc724\uc9c4\uc5d0\uac8c \ubb3c\uc5b4\ubcf4\uace0 \ucd98\uacc4\uc640 \ucd94\uacc4\uc5d0 \uace1\ubd80\uc5d0\uc11c \uc5f4\ub9ac\ub294 \uacf5\uc790\uc758 \uc81c\uc0ac\ub97c \uc724\uc9c4\uc5d0\uac8c \ub9e1\uacbc\ub2e4.", "sentence_answer": "3\ub0a8\uc778 \uc724\uc9c0\ub294 \uc724\uc2dc\uc640 \uc724\uc789\uc774 \uc720\ud3d0\ub41c \uc774\ud6c4\ub85c \uc7a5\uc790 \ud589\uc138\ub97c \ud558\uc600\uc73c\ub098 \uac15\ud76c\uc81c\uc758 \ub300\ud3b8\ucc2c\uc0ac\uc5c5\uc744 \ucd1d\uad04\ud558\uae30\ub9cc \ud560 \ubfd0, \uc815\uce58 \uc5d0\ub294 \uc2e0\uacbd\uc744 \uc4f0\uc9c0 \uc54a\uc558\ub2e4.", "paragraph_id": "6525595-9-1", "paragraph_question": "question: \uc724\uc9c0\ub294 \uac15\ud76c\uc81c\uc758 \ub300\ud3b8\ucc2c\uc0ac\uc5c5\uc744 \ucd1d\uad04\ud558\uae30\ub9cc \ud55c \ubfd0 \ubb34\uc5c7\uc5d0\ub294 \uc2e0\uacbd\uc744 \uc4f0\uc9c0 \uc54a\uc558\ub098\uc694?, context: \uadf8\ub7ec\ub098 \ud669\uc7a5\uc790 \uc724\uc2dc\ub294 \ubd80\ud669\uc5d0\uac8c \ud3d0\ud0dc\uc790 \uc724\uc789\uc744 \uc790\uae30 \uc190\uc73c\ub85c \uc8fd\uc774\uaca0\ub2e4\ub294 \ub9d0\uc744 \ud558\uc5ec \ubd80\ud669\uc758 \ubd84\ub178\ub97c \uc0c0\uace0, \uc774\ub85c \uc778\ud558\uc5ec \uc724\uc2dc\uc758 \uc791\uc704\ub294 \ubc15\ud0c8\ub418\uace0 \uc724\uc2dc\ub294 \uc720\ud3d0\ub418\uc5c8\ub2e4. 3\ub0a8\uc778 \uc724\uc9c0\ub294 \uc724\uc2dc\uc640 \uc724\uc789\uc774 \uc720\ud3d0\ub41c \uc774\ud6c4\ub85c \uc7a5\uc790 \ud589\uc138\ub97c \ud558\uc600\uc73c\ub098 \uac15\ud76c\uc81c\uc758 \ub300\ud3b8\ucc2c\uc0ac\uc5c5\uc744 \ucd1d\uad04\ud558\uae30\ub9cc \ud560 \ubfd0, \uc815\uce58\uc5d0\ub294 \uc2e0\uacbd\uc744 \uc4f0\uc9c0 \uc54a\uc558\ub2e4. \uc724\uc0ac\ub294 \uc724\uc9c4\uacfc \ud568\uaed8 \ud669\uc790\ub4e4 \uc911 \uac00\uc7a5 \uc601\ud2b9\ud558\uc600\uc73c\ub098 \uc724\uc789\uc774 \ud3d0\uc704\ub2f9\ud55c \uc774\ud6c4 \ub178\uace8\uc801\uc73c\ub85c \ud0dc\uc790\uc9c1\uc740 \ubb3c\ub860 \ud669\uc704\ub97c \ub178\ub838\uace0 \uc774 \uc18c\ubb38\uc774 \ub4e4\ub9ac\uc790, \uac15\ud76c\uc81c\ub294 \uc724\uc0ac\ub97c \ub9e4\uc6b0 \uc2eb\uc5b4\ud558\uace0 \uc724\uc0ac\uc758 \ubb38\uc548 \ubc1b\uae30\ub97c \uac70\ubd80\ud558\uc600\ub2e4. \uac15\ud76c\uc81c\uac00 \uc724\uc0ac\ub97c \ubd88\ub7ec\ub4e4\uc5ec \uc870\uc6a9\ud788 \uc0b4\ub77c\uace0 \ub9d0\uc744 \ud558\uc600\uc74c\uc5d0\ub3c4 \uc724\uc0ac\ub294 \uc790\uc2e0\uc758 \uc57c\uc2ec\uc744 \ub178\uace8\uc801\uc73c\ub85c \ub4dc\ub7ec\ub0c8\uace0 \uc774\uc5d0 \ub300\ud55c \uc870\uce58\ub85c \uac15\ud76c\uc81c\ub294 \uc724\uc0ac\ub97c \ud55c \ub2ec\uac04 \ub0c9\uad81\uc5d0 \uc720\ud3d0\uc2dc\ud0a4\uae30\ub3c4 \ud558\uc600\ub2e4. 14\ud669\uc790 \uc724\uc81c\ub294 \uc724\uc9c4\uc758 \ub3d9\ubcf5\uc544\uc6b0\ub85c \uac15\ud76c\uc81c\uc640 \uac19\uc774 \ub9ce\uc740 \uc6d0\uc815\uc744 \ub2e4\ub140\uc654\uace0, \ubd81\ucabd\uc758 \uad6d\uacbd\uc744 \uc9c0\ud0a4\ub294 \ubb34\uc6d0\ub300\uc7a5\uad70(\u64ab\u9060\u5927\u5c07\u8ecd)\uc744 \uad00\uc9c1\uc73c\ub85c \ubc1b\uc558\ub2e4. \ubb34\uc6d0\ub300\uc7a5\uad70\uc9c1\uc740 \ud754\ud788 \ud669\uc704\ub97c \uc774\uc5b4\ubc1b\uc744 \ud6c4\uacc4\uc790\ub098 \ub2e4\ub984\uc5c6\ub294 \uc790\ub9ac\uc5c8\uae30\uc5d0 \uc77c\ubd80 \ud559\uc790\ub4e4\uc740 \uac15\ud76c\uc81c\uac00 \ud669\uc704\ub85c \uc549\ud790 \uc724\uc81c\uc5d0\uac8c \ubbf8\ub9ac \ub300\uc7a5\uad70\uc758 \uc9c1\ud568\uc744 \ub0b4\ub824\uc900 \uac83\uc73c\ub85c \ud574\uc11d\ud55c\ub2e4. \uadf8\ub7ec\ub098 \uac15\ud76c\uc81c\ub294 \uad70\uc0ac\uc801\uc778 \uc77c\uc740 \ubaa8\ub450 \uc724\uc81c\uc640 \uc758\ub17c\ud558\uc600\uc73c\ub098 \uc815\uce58\uc640 \ub0b4\uc815\uc5d0 \uad00\ud55c \uc77c\uc740 \uc774\ubbf8 \uc624\ub7ab\ub3d9\uc548 \uc870\uc815\uc5d0\uc11c \ud65c\uc57d\ud558\ub358 \uc724\uc9c4\uc5d0\uac8c \ubb3c\uc5b4\ubcf4\uace0 \ucd98\uacc4\uc640 \ucd94\uacc4\uc5d0 \uace1\ubd80\uc5d0\uc11c \uc5f4\ub9ac\ub294 \uacf5\uc790\uc758 \uc81c\uc0ac\ub97c \uc724\uc9c4\uc5d0\uac8c \ub9e1\uacbc\ub2e4."} {"question": "\ubc00\uc591 \uc5ec\uc911\uc0dd \uc9d1\ub2e8 \uc131\ud3ed\ud589 \uc0ac\uac74 \ud6c4 \uc77c\ubd80 \uac00\ud574\uc790\uc640 \uac00\ud574\uc790\uc758 \uc5ec\uc790\uce5c\uad6c\ub4e4\uc5d0 \uc758\ud574 \uc720\ud3ec\ub41c \uac83\uc740 \ubb34\uc5c7\uc778\uac00?", "paragraph": "\ubd84\ub178\ud55c \ub124\ud2f0\uc98c\ub4e4\uc5d0 \uc758\ud574 \uac00\ud574\uc790\ub4e4\uc758 \uc778\uad8c\uc774 \uce68\ud574\ub418\ub294 \ubb38\uc81c\uac00 \ub098\ud0c0\ub0ac\ub2e4. \uc9d1\ub2e8 \ub2f4\ubc30 \ud761\uc5f0, \ud68c\uc2dd \uc7a5\uba74 \ub4f1\uc758 \uc0ac\uc9c4\uacfc \ud568\uaed8 \uc218\uc2ed \uba85\uc758 \uc2e4\uba85\uacfc \ucd9c\uc2e0\ud559\uad50, \ud734\ub300\uc804\ud654 \ubc88\ud638 \ub4f1\uc774 \uac8c\uc2dc\ub3fc \uc788\ub2e4. \ub610 \uac00\ub2f4\ud559\uc0dd\uc758 \uc5ec\uc790\uce5c\uad6c\ub85c \ud53c\ud574\ud559\uc0dd\ub4e4\uc744 \ube44\ud558\ud558\ub294 \ub0b4\uc6a9\uc758 \uae00\uc744 \uc62c\ub838\ub2e4\uba70 \uc774\ub984\uacfc \ud648\ud398\uc774\uc9c0 \uc8fc\uc18c \ub4f1\ub3c4 \ud568\uaed8 \uac8c\uc7ac\ub41c \ub0b4\uc6a9\uc774 \ud3ec\ud138\uc0ac\uc774\ud2b8\uc758 \ucee4\ubba4\ub2c8\ud2f0, \ubbf8\ub2c8\ud648\ud53c, \ube14\ub85c\uadf8 \ub4f1\uc744 \ud1b5\ud574 \uae09\uaca9\ud788 \ud655\uc0b0\ub410\ub2e4. \uc77c\ubd80 \uc778\ud130\ub137 \ub300\uc911\ub4e4\uc740 \u2018\ub9ce\uc740 \uc0ac\ub78c\uc5d0\uac8c \uc54c\ub824\uc11c \uc774\ub4e4\uc744 \uc751\uc9d5\ud558\uc790\u2019\ub294 \ub3c5\ub824\uae4c\uc9c0 \ud558\uace0 \uc788\ub294 \uc2e4\uc815\uc774\ub2e4. \uc774\ubbf8 \uc77c\ubd80 \uac00\ud574\uc790\uc640 \uac00\ud574\uc790\uc758 \uc5ec\uc790\uce5c\uad6c\ub4e4\uc5d0 \uc758\ud574 \ub3d9\uc601\uc0c1\uc774 \uc720\ud3ec\ub418\uc5c8\uace0, \ub3d9\uc601\uc0c1 \uc911\uc5d0\ub294 \ud53c\ud574\uc790, \uac00\ud574\uc790, \uac00\ud574\uc790\uc758 \uc5ec\uc790\uce5c\uad6c \ub4f1\uc758 \uc5bc\uad74\uacfc \uc2e0\uccb4\uac00 \uc120\uba85\ud558\uac8c \ub098\uc640 \uc788\uc5c8\uc73c\ubbc0\ub85c \ub3d9\uc601\uc0c1\ub4e4\uc744 \uadfc\uac70\ub85c \uc0ac\uc9c4 \ub4f1 \uc2e0\uc0c1\uc744 \uc27d\uac8c \uce90\ub0b4\uac8c \ub418\uc5c8\ub2e4. \uadf8\ub7ec\ub098 \uc5ec\uae30\uc5d0 \uac00\ud574\uc790\uc640\ub294 \uad00\ub828\uc5c6\ub294 \ub0a8\ud559\uc0dd\ub4e4\uc758 \uc2e0\uc0c1\uae4c\uc9c0 \uc62c\ub824\uc9c0\uba74\uc11c \uac00\ud574\uc790\ub85c \ubab0\ub824 \uc778\ud130\ub137\uc5d0 \uc62c\ub77c\uac00\uac8c \ub410\uace0, \uac00\ud574\uc790\uac00 \uc544\ub2cc \ub0a8\ud559\uc0dd\ub4e4\uc740 \uc5b5\uc6b8\ud568\uc744 \ud638\uc18c\ud588\ub2e4.", "answer": "\ub3d9\uc601\uc0c1", "sentence": "\uc774\ubbf8 \uc77c\ubd80 \uac00\ud574\uc790\uc640 \uac00\ud574\uc790\uc758 \uc5ec\uc790\uce5c\uad6c\ub4e4\uc5d0 \uc758\ud574 \ub3d9\uc601\uc0c1 \uc774 \uc720\ud3ec\ub418\uc5c8\uace0, \ub3d9\uc601\uc0c1 \uc911\uc5d0\ub294 \ud53c\ud574\uc790, \uac00\ud574\uc790, \uac00\ud574\uc790\uc758 \uc5ec\uc790\uce5c\uad6c \ub4f1\uc758 \uc5bc\uad74\uacfc \uc2e0\uccb4\uac00 \uc120\uba85\ud558\uac8c \ub098\uc640 \uc788\uc5c8\uc73c\ubbc0\ub85c \ub3d9\uc601\uc0c1\ub4e4\uc744 \uadfc\uac70\ub85c \uc0ac\uc9c4 \ub4f1 \uc2e0\uc0c1\uc744 \uc27d\uac8c \uce90\ub0b4\uac8c \ub418\uc5c8\ub2e4.", "paragraph_sentence": "\ubd84\ub178\ud55c \ub124\ud2f0\uc98c\ub4e4\uc5d0 \uc758\ud574 \uac00\ud574\uc790\ub4e4\uc758 \uc778\uad8c\uc774 \uce68\ud574\ub418\ub294 \ubb38\uc81c\uac00 \ub098\ud0c0\ub0ac\ub2e4. \uc9d1\ub2e8 \ub2f4\ubc30 \ud761\uc5f0, \ud68c\uc2dd \uc7a5\uba74 \ub4f1\uc758 \uc0ac\uc9c4\uacfc \ud568\uaed8 \uc218\uc2ed \uba85\uc758 \uc2e4\uba85\uacfc \ucd9c\uc2e0\ud559\uad50, \ud734\ub300\uc804\ud654 \ubc88\ud638 \ub4f1\uc774 \uac8c\uc2dc\ub3fc \uc788\ub2e4. \ub610 \uac00\ub2f4\ud559\uc0dd\uc758 \uc5ec\uc790\uce5c\uad6c\ub85c \ud53c\ud574\ud559\uc0dd\ub4e4\uc744 \ube44\ud558\ud558\ub294 \ub0b4\uc6a9\uc758 \uae00\uc744 \uc62c\ub838\ub2e4\uba70 \uc774\ub984\uacfc \ud648\ud398\uc774\uc9c0 \uc8fc\uc18c \ub4f1\ub3c4 \ud568\uaed8 \uac8c\uc7ac\ub41c \ub0b4\uc6a9\uc774 \ud3ec\ud138\uc0ac\uc774\ud2b8\uc758 \ucee4\ubba4\ub2c8\ud2f0, \ubbf8\ub2c8\ud648\ud53c, \ube14\ub85c\uadf8 \ub4f1\uc744 \ud1b5\ud574 \uae09\uaca9\ud788 \ud655\uc0b0\ub410\ub2e4. \uc77c\ubd80 \uc778\ud130\ub137 \ub300\uc911\ub4e4\uc740 \u2018\ub9ce\uc740 \uc0ac\ub78c\uc5d0\uac8c \uc54c\ub824\uc11c \uc774\ub4e4\uc744 \uc751\uc9d5\ud558\uc790\u2019\ub294 \ub3c5\ub824\uae4c\uc9c0 \ud558\uace0 \uc788\ub294 \uc2e4\uc815\uc774\ub2e4. \uc774\ubbf8 \uc77c\ubd80 \uac00\ud574\uc790\uc640 \uac00\ud574\uc790\uc758 \uc5ec\uc790\uce5c\uad6c\ub4e4\uc5d0 \uc758\ud574 \ub3d9\uc601\uc0c1 \uc774 \uc720\ud3ec\ub418\uc5c8\uace0, \ub3d9\uc601\uc0c1 \uc911\uc5d0\ub294 \ud53c\ud574\uc790, \uac00\ud574\uc790, \uac00\ud574\uc790\uc758 \uc5ec\uc790\uce5c\uad6c \ub4f1\uc758 \uc5bc\uad74\uacfc \uc2e0\uccb4\uac00 \uc120\uba85\ud558\uac8c \ub098\uc640 \uc788\uc5c8\uc73c\ubbc0\ub85c \ub3d9\uc601\uc0c1\ub4e4\uc744 \uadfc\uac70\ub85c \uc0ac\uc9c4 \ub4f1 \uc2e0\uc0c1\uc744 \uc27d\uac8c \uce90\ub0b4\uac8c \ub418\uc5c8\ub2e4. \uadf8\ub7ec\ub098 \uc5ec\uae30\uc5d0 \uac00\ud574\uc790\uc640\ub294 \uad00\ub828\uc5c6\ub294 \ub0a8\ud559\uc0dd\ub4e4\uc758 \uc2e0\uc0c1\uae4c\uc9c0 \uc62c\ub824\uc9c0\uba74\uc11c \uac00\ud574\uc790\ub85c \ubab0\ub824 \uc778\ud130\ub137\uc5d0 \uc62c\ub77c\uac00\uac8c \ub410\uace0, \uac00\ud574\uc790\uac00 \uc544\ub2cc \ub0a8\ud559\uc0dd\ub4e4\uc740 \uc5b5\uc6b8\ud568\uc744 \ud638\uc18c\ud588\ub2e4.", "paragraph_answer": "\ubd84\ub178\ud55c \ub124\ud2f0\uc98c\ub4e4\uc5d0 \uc758\ud574 \uac00\ud574\uc790\ub4e4\uc758 \uc778\uad8c\uc774 \uce68\ud574\ub418\ub294 \ubb38\uc81c\uac00 \ub098\ud0c0\ub0ac\ub2e4. \uc9d1\ub2e8 \ub2f4\ubc30 \ud761\uc5f0, \ud68c\uc2dd \uc7a5\uba74 \ub4f1\uc758 \uc0ac\uc9c4\uacfc \ud568\uaed8 \uc218\uc2ed \uba85\uc758 \uc2e4\uba85\uacfc \ucd9c\uc2e0\ud559\uad50, \ud734\ub300\uc804\ud654 \ubc88\ud638 \ub4f1\uc774 \uac8c\uc2dc\ub3fc \uc788\ub2e4. \ub610 \uac00\ub2f4\ud559\uc0dd\uc758 \uc5ec\uc790\uce5c\uad6c\ub85c \ud53c\ud574\ud559\uc0dd\ub4e4\uc744 \ube44\ud558\ud558\ub294 \ub0b4\uc6a9\uc758 \uae00\uc744 \uc62c\ub838\ub2e4\uba70 \uc774\ub984\uacfc \ud648\ud398\uc774\uc9c0 \uc8fc\uc18c \ub4f1\ub3c4 \ud568\uaed8 \uac8c\uc7ac\ub41c \ub0b4\uc6a9\uc774 \ud3ec\ud138\uc0ac\uc774\ud2b8\uc758 \ucee4\ubba4\ub2c8\ud2f0, \ubbf8\ub2c8\ud648\ud53c, \ube14\ub85c\uadf8 \ub4f1\uc744 \ud1b5\ud574 \uae09\uaca9\ud788 \ud655\uc0b0\ub410\ub2e4. \uc77c\ubd80 \uc778\ud130\ub137 \ub300\uc911\ub4e4\uc740 \u2018\ub9ce\uc740 \uc0ac\ub78c\uc5d0\uac8c \uc54c\ub824\uc11c \uc774\ub4e4\uc744 \uc751\uc9d5\ud558\uc790\u2019\ub294 \ub3c5\ub824\uae4c\uc9c0 \ud558\uace0 \uc788\ub294 \uc2e4\uc815\uc774\ub2e4. \uc774\ubbf8 \uc77c\ubd80 \uac00\ud574\uc790\uc640 \uac00\ud574\uc790\uc758 \uc5ec\uc790\uce5c\uad6c\ub4e4\uc5d0 \uc758\ud574 \ub3d9\uc601\uc0c1 \uc774 \uc720\ud3ec\ub418\uc5c8\uace0, \ub3d9\uc601\uc0c1 \uc911\uc5d0\ub294 \ud53c\ud574\uc790, \uac00\ud574\uc790, \uac00\ud574\uc790\uc758 \uc5ec\uc790\uce5c\uad6c \ub4f1\uc758 \uc5bc\uad74\uacfc \uc2e0\uccb4\uac00 \uc120\uba85\ud558\uac8c \ub098\uc640 \uc788\uc5c8\uc73c\ubbc0\ub85c \ub3d9\uc601\uc0c1\ub4e4\uc744 \uadfc\uac70\ub85c \uc0ac\uc9c4 \ub4f1 \uc2e0\uc0c1\uc744 \uc27d\uac8c \uce90\ub0b4\uac8c \ub418\uc5c8\ub2e4. \uadf8\ub7ec\ub098 \uc5ec\uae30\uc5d0 \uac00\ud574\uc790\uc640\ub294 \uad00\ub828\uc5c6\ub294 \ub0a8\ud559\uc0dd\ub4e4\uc758 \uc2e0\uc0c1\uae4c\uc9c0 \uc62c\ub824\uc9c0\uba74\uc11c \uac00\ud574\uc790\ub85c \ubab0\ub824 \uc778\ud130\ub137\uc5d0 \uc62c\ub77c\uac00\uac8c \ub410\uace0, \uac00\ud574\uc790\uac00 \uc544\ub2cc \ub0a8\ud559\uc0dd\ub4e4\uc740 \uc5b5\uc6b8\ud568\uc744 \ud638\uc18c\ud588\ub2e4.", "sentence_answer": "\uc774\ubbf8 \uc77c\ubd80 \uac00\ud574\uc790\uc640 \uac00\ud574\uc790\uc758 \uc5ec\uc790\uce5c\uad6c\ub4e4\uc5d0 \uc758\ud574 \ub3d9\uc601\uc0c1 \uc774 \uc720\ud3ec\ub418\uc5c8\uace0, \ub3d9\uc601\uc0c1 \uc911\uc5d0\ub294 \ud53c\ud574\uc790, \uac00\ud574\uc790, \uac00\ud574\uc790\uc758 \uc5ec\uc790\uce5c\uad6c \ub4f1\uc758 \uc5bc\uad74\uacfc \uc2e0\uccb4\uac00 \uc120\uba85\ud558\uac8c \ub098\uc640 \uc788\uc5c8\uc73c\ubbc0\ub85c \ub3d9\uc601\uc0c1\ub4e4\uc744 \uadfc\uac70\ub85c \uc0ac\uc9c4 \ub4f1 \uc2e0\uc0c1\uc744 \uc27d\uac8c \uce90\ub0b4\uac8c \ub418\uc5c8\ub2e4.", "paragraph_id": "6515514-11-1", "paragraph_question": "question: \ubc00\uc591 \uc5ec\uc911\uc0dd \uc9d1\ub2e8 \uc131\ud3ed\ud589 \uc0ac\uac74 \ud6c4 \uc77c\ubd80 \uac00\ud574\uc790\uc640 \uac00\ud574\uc790\uc758 \uc5ec\uc790\uce5c\uad6c\ub4e4\uc5d0 \uc758\ud574 \uc720\ud3ec\ub41c \uac83\uc740 \ubb34\uc5c7\uc778\uac00?, context: \ubd84\ub178\ud55c \ub124\ud2f0\uc98c\ub4e4\uc5d0 \uc758\ud574 \uac00\ud574\uc790\ub4e4\uc758 \uc778\uad8c\uc774 \uce68\ud574\ub418\ub294 \ubb38\uc81c\uac00 \ub098\ud0c0\ub0ac\ub2e4. \uc9d1\ub2e8 \ub2f4\ubc30 \ud761\uc5f0, \ud68c\uc2dd \uc7a5\uba74 \ub4f1\uc758 \uc0ac\uc9c4\uacfc \ud568\uaed8 \uc218\uc2ed \uba85\uc758 \uc2e4\uba85\uacfc \ucd9c\uc2e0\ud559\uad50, \ud734\ub300\uc804\ud654 \ubc88\ud638 \ub4f1\uc774 \uac8c\uc2dc\ub3fc \uc788\ub2e4. \ub610 \uac00\ub2f4\ud559\uc0dd\uc758 \uc5ec\uc790\uce5c\uad6c\ub85c \ud53c\ud574\ud559\uc0dd\ub4e4\uc744 \ube44\ud558\ud558\ub294 \ub0b4\uc6a9\uc758 \uae00\uc744 \uc62c\ub838\ub2e4\uba70 \uc774\ub984\uacfc \ud648\ud398\uc774\uc9c0 \uc8fc\uc18c \ub4f1\ub3c4 \ud568\uaed8 \uac8c\uc7ac\ub41c \ub0b4\uc6a9\uc774 \ud3ec\ud138\uc0ac\uc774\ud2b8\uc758 \ucee4\ubba4\ub2c8\ud2f0, \ubbf8\ub2c8\ud648\ud53c, \ube14\ub85c\uadf8 \ub4f1\uc744 \ud1b5\ud574 \uae09\uaca9\ud788 \ud655\uc0b0\ub410\ub2e4. \uc77c\ubd80 \uc778\ud130\ub137 \ub300\uc911\ub4e4\uc740 \u2018\ub9ce\uc740 \uc0ac\ub78c\uc5d0\uac8c \uc54c\ub824\uc11c \uc774\ub4e4\uc744 \uc751\uc9d5\ud558\uc790\u2019\ub294 \ub3c5\ub824\uae4c\uc9c0 \ud558\uace0 \uc788\ub294 \uc2e4\uc815\uc774\ub2e4. \uc774\ubbf8 \uc77c\ubd80 \uac00\ud574\uc790\uc640 \uac00\ud574\uc790\uc758 \uc5ec\uc790\uce5c\uad6c\ub4e4\uc5d0 \uc758\ud574 \ub3d9\uc601\uc0c1\uc774 \uc720\ud3ec\ub418\uc5c8\uace0, \ub3d9\uc601\uc0c1 \uc911\uc5d0\ub294 \ud53c\ud574\uc790, \uac00\ud574\uc790, \uac00\ud574\uc790\uc758 \uc5ec\uc790\uce5c\uad6c \ub4f1\uc758 \uc5bc\uad74\uacfc \uc2e0\uccb4\uac00 \uc120\uba85\ud558\uac8c \ub098\uc640 \uc788\uc5c8\uc73c\ubbc0\ub85c \ub3d9\uc601\uc0c1\ub4e4\uc744 \uadfc\uac70\ub85c \uc0ac\uc9c4 \ub4f1 \uc2e0\uc0c1\uc744 \uc27d\uac8c \uce90\ub0b4\uac8c \ub418\uc5c8\ub2e4. \uadf8\ub7ec\ub098 \uc5ec\uae30\uc5d0 \uac00\ud574\uc790\uc640\ub294 \uad00\ub828\uc5c6\ub294 \ub0a8\ud559\uc0dd\ub4e4\uc758 \uc2e0\uc0c1\uae4c\uc9c0 \uc62c\ub824\uc9c0\uba74\uc11c \uac00\ud574\uc790\ub85c \ubab0\ub824 \uc778\ud130\ub137\uc5d0 \uc62c\ub77c\uac00\uac8c \ub410\uace0, \uac00\ud574\uc790\uac00 \uc544\ub2cc \ub0a8\ud559\uc0dd\ub4e4\uc740 \uc5b5\uc6b8\ud568\uc744 \ud638\uc18c\ud588\ub2e4."} {"question": "\uc774\uc775\uc758 \uc5ed\uc0ac\uad00\uc744 \uc774\uc5b4 \ub3d9\uc0ac\uac15\ubaa9\uc774\ub77c\ub294 \uc791\ud488\uc744 \ud0c4\uc0dd\uc2dc\ud0a8 \uc778\ubb3c\uc740 \ub204\uad6c\uc778\uac00?", "paragraph": "\uc5ed\uc0ac\uc5d0 \ub300\ud574\uc11c\ub3c4 \uc774\uc775\uc740 \uc5ed\uc0ac\uc11c \uc800\uc220\uc774 \ub9e4\uc6b0 \uc5b4\ub824\uc6b4 \uc791\uc5c5\uacfc \ub3d9\uc2dc\uc5d0 \uc790\ub8cc\uc5d0 \uc5c4\ubc00\ud55c \uace0\uc99d\uc744 \ud589\ud560 \uac83\uc744 \uac15\uc870\ud55c\ub2e4. \ub3c5\ud2b9\ud55c \uacac\ud574\ub97c \ub9ce\uc774 \ub0b4\ub193\uc558\ub294\ub370, \ub2e8\uad70\uc870\uc120\uc758 \uad6d\ud638\ub97c '\ub2e8(\u6a80)', \uae30\uc790\uc870\uc120\uc758 \uad6d\ud638\ub97c '\uae30(\u7b95)'\ub85c \ubcf4\uace0, \uc0bc\ud55c(\u4e09\u97d3)\uc758 \uc6d0\uc8fc\ubbfc\uc744 \uc911\uad6d \uc804\uad6d\uc2dc\ub300 \ud55c(\u97d3)\uc758 \uc774\uc8fc\ubbfc\uc73c\ub85c \ud574\uc11d\ud55c \uac83\uc774 \uadf8\uac83\uc774\ub2e4. \uc774\ubc16\uc5d0 \uc870\uc120\uc758 \uc5ed\uc0ac\uac00 \uc911\uad6d\uc758 \uc694\uc21c\uc2dc\ub300\uc640 \ube44\uae38 \ub9cc\ud07c \uc624\ub798\ub418\uc5c8\uace0, \uc21c\uc774 \ub3d9\uc774\uc871\uc784\uc744 \uac15\uc870\ud558\uba74\uc11c\ub3c4 \uc911\uad6d\uc758 \uad50\ud654\uac00 \uc6b0\ub9ac \ub098\ub77c\uc5d0 \ubbf8\uce5c \uc810\uc744 \uac04\uacfc\ud558\uc9c0 \uc54a\uc558\ub294\ub370, \uc774\ub294 \uacb0\ub860\uc758 \ud0c0\ub2f9\uc131 \uc5ec\ubd80\ub97c \ub5a0\ub098\uc11c 18\uc138\uae30 \ucd08\u00b7\uc911\uc5fd\uc758 \uc5ed\uc0ac\ud559 \uc218\uc900\uc73c\ub85c \ubbf8\ub8e8\uc5b4 \ubcfc \ub54c \uac00\uc7a5 \uc138\ub828\ub41c \ubb38\ud5cc\uace0\uc99d\ud559\uc801 \ubc29\ubc95\ub860, \ub0b4\uc9c0\ub294 \ud55c\u00b7\uc911 \uc591\uad6d\uc758 \ubb38\ud654 \uad50\ub958\ub97c \ubcf4\ub2e4 \ud3ed\ub113\uac8c \uc774\ud574\ud55c \ud1a0\ub300 \uc704\uc5d0\uc11c \ub3c4\ucd9c\ub41c \uacac\ud574\ub4e4\uc774\ub77c\ub294 \uc810\uc5d0\uc11c \uadf8 \uc120\uc9c4\uc131\uc774 \uc778\uc815\ub41c\ub2e4. \uc774\ub7ec\ud55c \uadf8\uc758 \uc5ed\uc0ac\uad00\uc740 \uc81c\uc790 \uc548\uc815\ubcf5\uc5d0\uac8c \uc774\uc5b4\uc838 \u300a\ub3d9\uc0ac\uac15\ubaa9\u300b\uc774 \ud0c4\uc0dd\ud558\ub294 \ud55c \ubc30\uacbd\uc774 \ub418\uc5c8\ub2e4.", "answer": "\uc548\uc815\ubcf5", "sentence": "\uc774\ub7ec\ud55c \uadf8\uc758 \uc5ed\uc0ac\uad00\uc740 \uc81c\uc790 \uc548\uc815\ubcf5 \uc5d0\uac8c \uc774\uc5b4\uc838 \u300a\ub3d9\uc0ac\uac15\ubaa9\u300b\uc774 \ud0c4\uc0dd\ud558\ub294 \ud55c \ubc30\uacbd\uc774 \ub418\uc5c8\ub2e4.", "paragraph_sentence": "\uc5ed\uc0ac\uc5d0 \ub300\ud574\uc11c\ub3c4 \uc774\uc775\uc740 \uc5ed\uc0ac\uc11c \uc800\uc220\uc774 \ub9e4\uc6b0 \uc5b4\ub824\uc6b4 \uc791\uc5c5\uacfc \ub3d9\uc2dc\uc5d0 \uc790\ub8cc\uc5d0 \uc5c4\ubc00\ud55c \uace0\uc99d\uc744 \ud589\ud560 \uac83\uc744 \uac15\uc870\ud55c\ub2e4. \ub3c5\ud2b9\ud55c \uacac\ud574\ub97c \ub9ce\uc774 \ub0b4\ub193\uc558\ub294\ub370, \ub2e8\uad70\uc870\uc120\uc758 \uad6d\ud638\ub97c '\ub2e8(\u6a80)', \uae30\uc790\uc870\uc120\uc758 \uad6d\ud638\ub97c '\uae30(\u7b95)'\ub85c \ubcf4\uace0, \uc0bc\ud55c(\u4e09\u97d3)\uc758 \uc6d0\uc8fc\ubbfc\uc744 \uc911\uad6d \uc804\uad6d\uc2dc\ub300 \ud55c(\u97d3)\uc758 \uc774\uc8fc\ubbfc\uc73c\ub85c \ud574\uc11d\ud55c \uac83\uc774 \uadf8\uac83\uc774\ub2e4. \uc774\ubc16\uc5d0 \uc870\uc120\uc758 \uc5ed\uc0ac\uac00 \uc911\uad6d\uc758 \uc694\uc21c\uc2dc\ub300\uc640 \ube44\uae38 \ub9cc\ud07c \uc624\ub798\ub418\uc5c8\uace0, \uc21c\uc774 \ub3d9\uc774\uc871\uc784\uc744 \uac15\uc870\ud558\uba74\uc11c\ub3c4 \uc911\uad6d\uc758 \uad50\ud654\uac00 \uc6b0\ub9ac \ub098\ub77c\uc5d0 \ubbf8\uce5c \uc810\uc744 \uac04\uacfc\ud558\uc9c0 \uc54a\uc558\ub294\ub370, \uc774\ub294 \uacb0\ub860\uc758 \ud0c0\ub2f9\uc131 \uc5ec\ubd80\ub97c \ub5a0\ub098\uc11c 18\uc138\uae30 \ucd08\u00b7\uc911\uc5fd\uc758 \uc5ed\uc0ac\ud559 \uc218\uc900\uc73c\ub85c \ubbf8\ub8e8\uc5b4 \ubcfc \ub54c \uac00\uc7a5 \uc138\ub828\ub41c \ubb38\ud5cc\uace0\uc99d\ud559\uc801 \ubc29\ubc95\ub860, \ub0b4\uc9c0\ub294 \ud55c\u00b7\uc911 \uc591\uad6d\uc758 \ubb38\ud654 \uad50\ub958\ub97c \ubcf4\ub2e4 \ud3ed\ub113\uac8c \uc774\ud574\ud55c \ud1a0\ub300 \uc704\uc5d0\uc11c \ub3c4\ucd9c\ub41c \uacac\ud574\ub4e4\uc774\ub77c\ub294 \uc810\uc5d0\uc11c \uadf8 \uc120\uc9c4\uc131\uc774 \uc778\uc815\ub41c\ub2e4. \uc774\ub7ec\ud55c \uadf8\uc758 \uc5ed\uc0ac\uad00\uc740 \uc81c\uc790 \uc548\uc815\ubcf5 \uc5d0\uac8c \uc774\uc5b4\uc838 \u300a\ub3d9\uc0ac\uac15\ubaa9\u300b\uc774 \ud0c4\uc0dd\ud558\ub294 \ud55c \ubc30\uacbd\uc774 \ub418\uc5c8\ub2e4. ", "paragraph_answer": "\uc5ed\uc0ac\uc5d0 \ub300\ud574\uc11c\ub3c4 \uc774\uc775\uc740 \uc5ed\uc0ac\uc11c \uc800\uc220\uc774 \ub9e4\uc6b0 \uc5b4\ub824\uc6b4 \uc791\uc5c5\uacfc \ub3d9\uc2dc\uc5d0 \uc790\ub8cc\uc5d0 \uc5c4\ubc00\ud55c \uace0\uc99d\uc744 \ud589\ud560 \uac83\uc744 \uac15\uc870\ud55c\ub2e4. \ub3c5\ud2b9\ud55c \uacac\ud574\ub97c \ub9ce\uc774 \ub0b4\ub193\uc558\ub294\ub370, \ub2e8\uad70\uc870\uc120\uc758 \uad6d\ud638\ub97c '\ub2e8(\u6a80)', \uae30\uc790\uc870\uc120\uc758 \uad6d\ud638\ub97c '\uae30(\u7b95)'\ub85c \ubcf4\uace0, \uc0bc\ud55c(\u4e09\u97d3)\uc758 \uc6d0\uc8fc\ubbfc\uc744 \uc911\uad6d \uc804\uad6d\uc2dc\ub300 \ud55c(\u97d3)\uc758 \uc774\uc8fc\ubbfc\uc73c\ub85c \ud574\uc11d\ud55c \uac83\uc774 \uadf8\uac83\uc774\ub2e4. \uc774\ubc16\uc5d0 \uc870\uc120\uc758 \uc5ed\uc0ac\uac00 \uc911\uad6d\uc758 \uc694\uc21c\uc2dc\ub300\uc640 \ube44\uae38 \ub9cc\ud07c \uc624\ub798\ub418\uc5c8\uace0, \uc21c\uc774 \ub3d9\uc774\uc871\uc784\uc744 \uac15\uc870\ud558\uba74\uc11c\ub3c4 \uc911\uad6d\uc758 \uad50\ud654\uac00 \uc6b0\ub9ac \ub098\ub77c\uc5d0 \ubbf8\uce5c \uc810\uc744 \uac04\uacfc\ud558\uc9c0 \uc54a\uc558\ub294\ub370, \uc774\ub294 \uacb0\ub860\uc758 \ud0c0\ub2f9\uc131 \uc5ec\ubd80\ub97c \ub5a0\ub098\uc11c 18\uc138\uae30 \ucd08\u00b7\uc911\uc5fd\uc758 \uc5ed\uc0ac\ud559 \uc218\uc900\uc73c\ub85c \ubbf8\ub8e8\uc5b4 \ubcfc \ub54c \uac00\uc7a5 \uc138\ub828\ub41c \ubb38\ud5cc\uace0\uc99d\ud559\uc801 \ubc29\ubc95\ub860, \ub0b4\uc9c0\ub294 \ud55c\u00b7\uc911 \uc591\uad6d\uc758 \ubb38\ud654 \uad50\ub958\ub97c \ubcf4\ub2e4 \ud3ed\ub113\uac8c \uc774\ud574\ud55c \ud1a0\ub300 \uc704\uc5d0\uc11c \ub3c4\ucd9c\ub41c \uacac\ud574\ub4e4\uc774\ub77c\ub294 \uc810\uc5d0\uc11c \uadf8 \uc120\uc9c4\uc131\uc774 \uc778\uc815\ub41c\ub2e4. \uc774\ub7ec\ud55c \uadf8\uc758 \uc5ed\uc0ac\uad00\uc740 \uc81c\uc790 \uc548\uc815\ubcf5 \uc5d0\uac8c \uc774\uc5b4\uc838 \u300a\ub3d9\uc0ac\uac15\ubaa9\u300b\uc774 \ud0c4\uc0dd\ud558\ub294 \ud55c \ubc30\uacbd\uc774 \ub418\uc5c8\ub2e4.", "sentence_answer": "\uc774\ub7ec\ud55c \uadf8\uc758 \uc5ed\uc0ac\uad00\uc740 \uc81c\uc790 \uc548\uc815\ubcf5 \uc5d0\uac8c \uc774\uc5b4\uc838 \u300a\ub3d9\uc0ac\uac15\ubaa9\u300b\uc774 \ud0c4\uc0dd\ud558\ub294 \ud55c \ubc30\uacbd\uc774 \ub418\uc5c8\ub2e4.", "paragraph_id": "6465191-1-1", "paragraph_question": "question: \uc774\uc775\uc758 \uc5ed\uc0ac\uad00\uc744 \uc774\uc5b4 \ub3d9\uc0ac\uac15\ubaa9\uc774\ub77c\ub294 \uc791\ud488\uc744 \ud0c4\uc0dd\uc2dc\ud0a8 \uc778\ubb3c\uc740 \ub204\uad6c\uc778\uac00?, context: \uc5ed\uc0ac\uc5d0 \ub300\ud574\uc11c\ub3c4 \uc774\uc775\uc740 \uc5ed\uc0ac\uc11c \uc800\uc220\uc774 \ub9e4\uc6b0 \uc5b4\ub824\uc6b4 \uc791\uc5c5\uacfc \ub3d9\uc2dc\uc5d0 \uc790\ub8cc\uc5d0 \uc5c4\ubc00\ud55c \uace0\uc99d\uc744 \ud589\ud560 \uac83\uc744 \uac15\uc870\ud55c\ub2e4. \ub3c5\ud2b9\ud55c \uacac\ud574\ub97c \ub9ce\uc774 \ub0b4\ub193\uc558\ub294\ub370, \ub2e8\uad70\uc870\uc120\uc758 \uad6d\ud638\ub97c '\ub2e8(\u6a80)', \uae30\uc790\uc870\uc120\uc758 \uad6d\ud638\ub97c '\uae30(\u7b95)'\ub85c \ubcf4\uace0, \uc0bc\ud55c(\u4e09\u97d3)\uc758 \uc6d0\uc8fc\ubbfc\uc744 \uc911\uad6d \uc804\uad6d\uc2dc\ub300 \ud55c(\u97d3)\uc758 \uc774\uc8fc\ubbfc\uc73c\ub85c \ud574\uc11d\ud55c \uac83\uc774 \uadf8\uac83\uc774\ub2e4. \uc774\ubc16\uc5d0 \uc870\uc120\uc758 \uc5ed\uc0ac\uac00 \uc911\uad6d\uc758 \uc694\uc21c\uc2dc\ub300\uc640 \ube44\uae38 \ub9cc\ud07c \uc624\ub798\ub418\uc5c8\uace0, \uc21c\uc774 \ub3d9\uc774\uc871\uc784\uc744 \uac15\uc870\ud558\uba74\uc11c\ub3c4 \uc911\uad6d\uc758 \uad50\ud654\uac00 \uc6b0\ub9ac \ub098\ub77c\uc5d0 \ubbf8\uce5c \uc810\uc744 \uac04\uacfc\ud558\uc9c0 \uc54a\uc558\ub294\ub370, \uc774\ub294 \uacb0\ub860\uc758 \ud0c0\ub2f9\uc131 \uc5ec\ubd80\ub97c \ub5a0\ub098\uc11c 18\uc138\uae30 \ucd08\u00b7\uc911\uc5fd\uc758 \uc5ed\uc0ac\ud559 \uc218\uc900\uc73c\ub85c \ubbf8\ub8e8\uc5b4 \ubcfc \ub54c \uac00\uc7a5 \uc138\ub828\ub41c \ubb38\ud5cc\uace0\uc99d\ud559\uc801 \ubc29\ubc95\ub860, \ub0b4\uc9c0\ub294 \ud55c\u00b7\uc911 \uc591\uad6d\uc758 \ubb38\ud654 \uad50\ub958\ub97c \ubcf4\ub2e4 \ud3ed\ub113\uac8c \uc774\ud574\ud55c \ud1a0\ub300 \uc704\uc5d0\uc11c \ub3c4\ucd9c\ub41c \uacac\ud574\ub4e4\uc774\ub77c\ub294 \uc810\uc5d0\uc11c \uadf8 \uc120\uc9c4\uc131\uc774 \uc778\uc815\ub41c\ub2e4. \uc774\ub7ec\ud55c \uadf8\uc758 \uc5ed\uc0ac\uad00\uc740 \uc81c\uc790 \uc548\uc815\ubcf5\uc5d0\uac8c \uc774\uc5b4\uc838 \u300a\ub3d9\uc0ac\uac15\ubaa9\u300b\uc774 \ud0c4\uc0dd\ud558\ub294 \ud55c \ubc30\uacbd\uc774 \ub418\uc5c8\ub2e4."} {"question": "\uadf8\ub9ac\uc2a4 \ud310\ud14c\uc628\uc758 \uc601\uad11\uc744 \uae30\ub9ac\uae30 \uc704\ud55c \ud638\uba54\ub85c\uc2a4 \ucc2c\uac00\ub294 \ucd1d \uba87\ud3b8\uc758 \ub178\ub798\ub85c \uad6c\uc131\ub418\uc5b4 \uc788\ub294\uac00?", "paragraph": "\uace0\uc804 \uc2dc\ub300 \uc2e0\ud654\uc5d0\uc11c\ub294 \ud2f0\ud0c4\ub4e4\uc758 \ud328\ubc30 \uc774\ud6c4, \uc2e0\ub4e4\uc758 \uc0c8\ub85c\uc6b4 \ud310\ud14c\uc628\uc774 \uc138\uc6cc\uc84c\ub2e4\uace0 \uc124\uba85\ud55c\ub2e4. \uc8fc\uc694\ud55c \uadf8\ub9ac\uc2a4 \uc2e0\ub4e4 \uc911\uc5d0\uc11c \uc62c\ub9bc\ud53c\uc548\uc740 \uc62c\ub9bc\ud3ec\uc2a4 \uc0b0 \uc815\uc0c1\uc5d0\uc11c \uc81c\uc6b0\uc2a4\uc758 \ud1b5\uce58 \uc544\ub798 \uc0b4\uc544\uac00\ub294 \uc2e0\ub4e4\uc744 \ub9d0\ud55c\ub2e4. \uc774\ub4e4\uc758 \uc778\uc6d0\uc774 \uc5f4\ub450 \uba85\uc73c\ub85c \uc81c\ud55c\ub41c \uac83\uc740 \ube44\uad50\uc801 \ucd5c\uadfc\uc5d0 \ub3c4\uc785\ub41c \uac1c\ub150\uc73c\ub85c \ubcf4\uc778\ub2e4. \uc62c\ub9bc\ud53c\uc548 \uc774\uc678\uc5d0\ub3c4 \uadf8\ub9ac\uc2a4\uc778\ub4e4\uc740 \uc5fc\uc18c \uc2e0 \ud310, \uac15\uc758 \uc815\ub839 \ub2d8\ud504, \uc0d8\uc5d0 \uc0ac\ub294 \ub098\uc774\uc544\ub4dc, \ub098\ubb34\uc758 \uc815\ub839 \ub4dc\ub77c\uc774\uc5b4\ub4dc, \ubc14\ub2e4\uc5d0 \uc0ac\ub294 \ub124\ub808\uc774\ub4dc, \uac15\uc758 \uc2e0, \uc0ac\ud2f0\ub85c\uc2a4\ub97c \ube44\ub86f\ud55c \uadf8 \uc9c0\uc5ed\uc758 \ub2e4\uc591\ud55c \uc2e0\ub4e4\uc744 \uc22d\ubc30\ud558\uc600\ub2e4. \uc5ec\uae30\uc5d0\ub294 \uc5d0\ub9ac\ub2c8\uc5d0\uc2a4(\ub610\ub294 \ud478\ub9ac\uc544\uc774)\ucc98\ub7fc \ud608\uc5f0 \uad00\uacc4\uc5d0\uac8c \ubc94\uc8c4\ub97c \uc800\uc9c0\ub978 \uc8c4\uc778\uc744 \ub4a4\ucad3\ub294 \uc800\uc2b9\uc758 \uc554\ud751 \uc138\ub825\ub3c4 \uc788\uc5c8\ub2e4. \uc2dc\uc778\ub4e4\uc740 \uadf8\ub9ac\uc2a4 \ud310\ud14c\uc628\uc758 \uc601\uad11\uc744 \uae30\ub9ac\uace0\uc790 \ud638\uba54\ub85c\uc2a4 \ucc2c\uac00\ub97c \uc9c0\uc5c8\ub2e4.(33\ud3b8\uc758 \ub178\ub798). \uadf8\ub808\uace0\ub9ac \ub098\uc9c0\ub294 \ud638\uba54\ub85c\uc2a4 \ucc2c\uac00\ub97c \"\uac01 \ub178\ub798\ub9c8\ub2e4 \uc2e0\uc5d0 \ub300\ud55c \uae30\uc6d0\uc744 \ub178\ub798\ud558\ub294(\u300a\uc2e0\ud1b5\uae30\u300b\uc5d0 \ube44\ud574) \uac04\uacb0\ud55c \uc11c\uac00\"\ub85c \uac04\uc8fc\ud558\uc600\ub2e4.", "answer": "33\ud3b8", "sentence": "( 33\ud3b8 \uc758 \ub178\ub798)", "paragraph_sentence": "\uace0\uc804 \uc2dc\ub300 \uc2e0\ud654\uc5d0\uc11c\ub294 \ud2f0\ud0c4\ub4e4\uc758 \ud328\ubc30 \uc774\ud6c4, \uc2e0\ub4e4\uc758 \uc0c8\ub85c\uc6b4 \ud310\ud14c\uc628\uc774 \uc138\uc6cc\uc84c\ub2e4\uace0 \uc124\uba85\ud55c\ub2e4. \uc8fc\uc694\ud55c \uadf8\ub9ac\uc2a4 \uc2e0\ub4e4 \uc911\uc5d0\uc11c \uc62c\ub9bc\ud53c\uc548\uc740 \uc62c\ub9bc\ud3ec\uc2a4 \uc0b0 \uc815\uc0c1\uc5d0\uc11c \uc81c\uc6b0\uc2a4\uc758 \ud1b5\uce58 \uc544\ub798 \uc0b4\uc544\uac00\ub294 \uc2e0\ub4e4\uc744 \ub9d0\ud55c\ub2e4. \uc774\ub4e4\uc758 \uc778\uc6d0\uc774 \uc5f4\ub450 \uba85\uc73c\ub85c \uc81c\ud55c\ub41c \uac83\uc740 \ube44\uad50\uc801 \ucd5c\uadfc\uc5d0 \ub3c4\uc785\ub41c \uac1c\ub150\uc73c\ub85c \ubcf4\uc778\ub2e4. \uc62c\ub9bc\ud53c\uc548 \uc774\uc678\uc5d0\ub3c4 \uadf8\ub9ac\uc2a4\uc778\ub4e4\uc740 \uc5fc\uc18c \uc2e0 \ud310, \uac15\uc758 \uc815\ub839 \ub2d8\ud504, \uc0d8\uc5d0 \uc0ac\ub294 \ub098\uc774\uc544\ub4dc, \ub098\ubb34\uc758 \uc815\ub839 \ub4dc\ub77c\uc774\uc5b4\ub4dc, \ubc14\ub2e4\uc5d0 \uc0ac\ub294 \ub124\ub808\uc774\ub4dc, \uac15\uc758 \uc2e0, \uc0ac\ud2f0\ub85c\uc2a4\ub97c \ube44\ub86f\ud55c \uadf8 \uc9c0\uc5ed\uc758 \ub2e4\uc591\ud55c \uc2e0\ub4e4\uc744 \uc22d\ubc30\ud558\uc600\ub2e4. \uc5ec\uae30\uc5d0\ub294 \uc5d0\ub9ac\ub2c8\uc5d0\uc2a4(\ub610\ub294 \ud478\ub9ac\uc544\uc774)\ucc98\ub7fc \ud608\uc5f0 \uad00\uacc4\uc5d0\uac8c \ubc94\uc8c4\ub97c \uc800\uc9c0\ub978 \uc8c4\uc778\uc744 \ub4a4\ucad3\ub294 \uc800\uc2b9\uc758 \uc554\ud751 \uc138\ub825\ub3c4 \uc788\uc5c8\ub2e4. \uc2dc\uc778\ub4e4\uc740 \uadf8\ub9ac\uc2a4 \ud310\ud14c\uc628\uc758 \uc601\uad11\uc744 \uae30\ub9ac\uace0\uc790 \ud638\uba54\ub85c\uc2a4 \ucc2c\uac00\ub97c \uc9c0\uc5c8\ub2e4. ( 33\ud3b8 \uc758 \ub178\ub798) . \uadf8\ub808\uace0\ub9ac \ub098\uc9c0\ub294 \ud638\uba54\ub85c\uc2a4 \ucc2c\uac00\ub97c \"\uac01 \ub178\ub798\ub9c8\ub2e4 \uc2e0\uc5d0 \ub300\ud55c \uae30\uc6d0\uc744 \ub178\ub798\ud558\ub294(\u300a\uc2e0\ud1b5\uae30\u300b\uc5d0 \ube44\ud574) \uac04\uacb0\ud55c \uc11c\uac00\"\ub85c \uac04\uc8fc\ud558\uc600\ub2e4.", "paragraph_answer": "\uace0\uc804 \uc2dc\ub300 \uc2e0\ud654\uc5d0\uc11c\ub294 \ud2f0\ud0c4\ub4e4\uc758 \ud328\ubc30 \uc774\ud6c4, \uc2e0\ub4e4\uc758 \uc0c8\ub85c\uc6b4 \ud310\ud14c\uc628\uc774 \uc138\uc6cc\uc84c\ub2e4\uace0 \uc124\uba85\ud55c\ub2e4. \uc8fc\uc694\ud55c \uadf8\ub9ac\uc2a4 \uc2e0\ub4e4 \uc911\uc5d0\uc11c \uc62c\ub9bc\ud53c\uc548\uc740 \uc62c\ub9bc\ud3ec\uc2a4 \uc0b0 \uc815\uc0c1\uc5d0\uc11c \uc81c\uc6b0\uc2a4\uc758 \ud1b5\uce58 \uc544\ub798 \uc0b4\uc544\uac00\ub294 \uc2e0\ub4e4\uc744 \ub9d0\ud55c\ub2e4. \uc774\ub4e4\uc758 \uc778\uc6d0\uc774 \uc5f4\ub450 \uba85\uc73c\ub85c \uc81c\ud55c\ub41c \uac83\uc740 \ube44\uad50\uc801 \ucd5c\uadfc\uc5d0 \ub3c4\uc785\ub41c \uac1c\ub150\uc73c\ub85c \ubcf4\uc778\ub2e4. \uc62c\ub9bc\ud53c\uc548 \uc774\uc678\uc5d0\ub3c4 \uadf8\ub9ac\uc2a4\uc778\ub4e4\uc740 \uc5fc\uc18c \uc2e0 \ud310, \uac15\uc758 \uc815\ub839 \ub2d8\ud504, \uc0d8\uc5d0 \uc0ac\ub294 \ub098\uc774\uc544\ub4dc, \ub098\ubb34\uc758 \uc815\ub839 \ub4dc\ub77c\uc774\uc5b4\ub4dc, \ubc14\ub2e4\uc5d0 \uc0ac\ub294 \ub124\ub808\uc774\ub4dc, \uac15\uc758 \uc2e0, \uc0ac\ud2f0\ub85c\uc2a4\ub97c \ube44\ub86f\ud55c \uadf8 \uc9c0\uc5ed\uc758 \ub2e4\uc591\ud55c \uc2e0\ub4e4\uc744 \uc22d\ubc30\ud558\uc600\ub2e4. \uc5ec\uae30\uc5d0\ub294 \uc5d0\ub9ac\ub2c8\uc5d0\uc2a4(\ub610\ub294 \ud478\ub9ac\uc544\uc774)\ucc98\ub7fc \ud608\uc5f0 \uad00\uacc4\uc5d0\uac8c \ubc94\uc8c4\ub97c \uc800\uc9c0\ub978 \uc8c4\uc778\uc744 \ub4a4\ucad3\ub294 \uc800\uc2b9\uc758 \uc554\ud751 \uc138\ub825\ub3c4 \uc788\uc5c8\ub2e4. \uc2dc\uc778\ub4e4\uc740 \uadf8\ub9ac\uc2a4 \ud310\ud14c\uc628\uc758 \uc601\uad11\uc744 \uae30\ub9ac\uace0\uc790 \ud638\uba54\ub85c\uc2a4 \ucc2c\uac00\ub97c \uc9c0\uc5c8\ub2e4.( 33\ud3b8 \uc758 \ub178\ub798). \uadf8\ub808\uace0\ub9ac \ub098\uc9c0\ub294 \ud638\uba54\ub85c\uc2a4 \ucc2c\uac00\ub97c \"\uac01 \ub178\ub798\ub9c8\ub2e4 \uc2e0\uc5d0 \ub300\ud55c \uae30\uc6d0\uc744 \ub178\ub798\ud558\ub294(\u300a\uc2e0\ud1b5\uae30\u300b\uc5d0 \ube44\ud574) \uac04\uacb0\ud55c \uc11c\uac00\"\ub85c \uac04\uc8fc\ud558\uc600\ub2e4.", "sentence_answer": "( 33\ud3b8 \uc758 \ub178\ub798)", "paragraph_id": "6527377-11-1", "paragraph_question": "question: \uadf8\ub9ac\uc2a4 \ud310\ud14c\uc628\uc758 \uc601\uad11\uc744 \uae30\ub9ac\uae30 \uc704\ud55c \ud638\uba54\ub85c\uc2a4 \ucc2c\uac00\ub294 \ucd1d \uba87\ud3b8\uc758 \ub178\ub798\ub85c \uad6c\uc131\ub418\uc5b4 \uc788\ub294\uac00?, context: \uace0\uc804 \uc2dc\ub300 \uc2e0\ud654\uc5d0\uc11c\ub294 \ud2f0\ud0c4\ub4e4\uc758 \ud328\ubc30 \uc774\ud6c4, \uc2e0\ub4e4\uc758 \uc0c8\ub85c\uc6b4 \ud310\ud14c\uc628\uc774 \uc138\uc6cc\uc84c\ub2e4\uace0 \uc124\uba85\ud55c\ub2e4. \uc8fc\uc694\ud55c \uadf8\ub9ac\uc2a4 \uc2e0\ub4e4 \uc911\uc5d0\uc11c \uc62c\ub9bc\ud53c\uc548\uc740 \uc62c\ub9bc\ud3ec\uc2a4 \uc0b0 \uc815\uc0c1\uc5d0\uc11c \uc81c\uc6b0\uc2a4\uc758 \ud1b5\uce58 \uc544\ub798 \uc0b4\uc544\uac00\ub294 \uc2e0\ub4e4\uc744 \ub9d0\ud55c\ub2e4. \uc774\ub4e4\uc758 \uc778\uc6d0\uc774 \uc5f4\ub450 \uba85\uc73c\ub85c \uc81c\ud55c\ub41c \uac83\uc740 \ube44\uad50\uc801 \ucd5c\uadfc\uc5d0 \ub3c4\uc785\ub41c \uac1c\ub150\uc73c\ub85c \ubcf4\uc778\ub2e4. \uc62c\ub9bc\ud53c\uc548 \uc774\uc678\uc5d0\ub3c4 \uadf8\ub9ac\uc2a4\uc778\ub4e4\uc740 \uc5fc\uc18c \uc2e0 \ud310, \uac15\uc758 \uc815\ub839 \ub2d8\ud504, \uc0d8\uc5d0 \uc0ac\ub294 \ub098\uc774\uc544\ub4dc, \ub098\ubb34\uc758 \uc815\ub839 \ub4dc\ub77c\uc774\uc5b4\ub4dc, \ubc14\ub2e4\uc5d0 \uc0ac\ub294 \ub124\ub808\uc774\ub4dc, \uac15\uc758 \uc2e0, \uc0ac\ud2f0\ub85c\uc2a4\ub97c \ube44\ub86f\ud55c \uadf8 \uc9c0\uc5ed\uc758 \ub2e4\uc591\ud55c \uc2e0\ub4e4\uc744 \uc22d\ubc30\ud558\uc600\ub2e4. \uc5ec\uae30\uc5d0\ub294 \uc5d0\ub9ac\ub2c8\uc5d0\uc2a4(\ub610\ub294 \ud478\ub9ac\uc544\uc774)\ucc98\ub7fc \ud608\uc5f0 \uad00\uacc4\uc5d0\uac8c \ubc94\uc8c4\ub97c \uc800\uc9c0\ub978 \uc8c4\uc778\uc744 \ub4a4\ucad3\ub294 \uc800\uc2b9\uc758 \uc554\ud751 \uc138\ub825\ub3c4 \uc788\uc5c8\ub2e4. \uc2dc\uc778\ub4e4\uc740 \uadf8\ub9ac\uc2a4 \ud310\ud14c\uc628\uc758 \uc601\uad11\uc744 \uae30\ub9ac\uace0\uc790 \ud638\uba54\ub85c\uc2a4 \ucc2c\uac00\ub97c \uc9c0\uc5c8\ub2e4.(33\ud3b8\uc758 \ub178\ub798). \uadf8\ub808\uace0\ub9ac \ub098\uc9c0\ub294 \ud638\uba54\ub85c\uc2a4 \ucc2c\uac00\ub97c \"\uac01 \ub178\ub798\ub9c8\ub2e4 \uc2e0\uc5d0 \ub300\ud55c \uae30\uc6d0\uc744 \ub178\ub798\ud558\ub294(\u300a\uc2e0\ud1b5\uae30\u300b\uc5d0 \ube44\ud574) \uac04\uacb0\ud55c \uc11c\uac00\"\ub85c \uac04\uc8fc\ud558\uc600\ub2e4."} {"question": "\uad11\uace0\ub85c\ubd80\ud130 \uc720\ub3c4\ub41c \ud589\uc704\ub97c \ud558\ub294 \ubc29\ubb38\uc790\uc758 \ube44\uc728\uc744 \uce21\uc815\ud558\ub294 \uac83\uc740?", "paragraph": "\ub9c8\ucf00\ud305 \uc804\ubb38\uac00\ub4e4\uc740 \uc6f9 \ubc29\ubb38\uc790\ub4e4\uc758 \ubc18\uc751\uc744 \uc54c\uace0 \uc2f6\uc5b4\ud558\uc9c0\ub9cc, \ud2b9\uc815 \uc6f9\uc0ac\uc774\ud2b8\uc5d0 \ub300\ud55c \uc0ac\uc6a9\uc790\ub4e4\uc758 \uac10\uc815\uc801 \ubc18\uc751 \ubc0f \uc6f9\uc0ac\uc774\ud2b8\ub97c \ud1b5\ud55c \uae30\uc5c5\uc758 \ube0c\ub79c\ub4dc \uc774\ubbf8\uc9c0 \uc0c1\uc2b9 \ud6a8\uacfc\ub97c \uc644\uc804\ud788 \uc815\ub7c9\ud654 \uc2dc\ud0a4\ub294 \uac83\uc740 \ud604\uc7ac\uc758 \uae30\uc220\ub85c\uc11c\ub294 \uac70\uc758 \ubd88\uac00\ub2a5\ud558\ub2e4\uace0 \ubcfc \uc218 \uc788\ub2e4. \uc774\uc5d0 \ubc18\ud574 \ubcf4\ub2e4 \uc27d\uac8c \uc5bb\uc5b4\ub0bc \uc218 \uc788\ub294 \uc815\ubcf4\uac00 \ubc14\ub85c \ud074\ub9ad\ub960\uc774\ub2e4. \ud074\ub9ad\ub960\uc740 \uad11\uace0\uc5d0 \uc758\ud574 \uc758\ub3c4\ub41c \ud398\uc774\uc9c0\ub85c \uc5f0\uacb0\ub41c \ubc29\ubb38\uac1d \uc911 \uc5b4\ub5a0\ud55c \ud488\ubaa9\uc744 \uad6c\uc785\ud558\uac70\ub098 \uc81c\ud488 \ub610\ub294 \uc11c\ube44\uc2a4\uc5d0 \ub300\ud55c \uc815\ubcf4\ub97c \uc5bb\ub294 \ub4f1, \uad11\uace0\ub85c\ubd80\ud130 \uc720\ub3c4\ub41c \ud589\uc704\ub97c \uc2dc\ud589\ud55c \ubc29\ubb38\uc790\uc758 \ube44\uc728\uc744 \uce21\uc815\ud55c\ub2e4. \ub2e4\uc591\ud55c \ud615\ud0dc\uc758 \uc0c1\ud638 \uc791\uc6a9\uc774 \uac00\ub2a5\ud558\uc9c0\ub9cc \uc5ec\uae30\uc11c \uad11\uace0(\ud639\uc740 \ub9c1\ud06c)\uc5d0 \ub300\ud574 \"\ub9c8\uc6b0\uc2a4 \ud074\ub9ad\"\uc774\ub77c\ub294 \ud45c\ud604\uc744 \uc0ac\uc6a9\ud55c \uac83\uc740 \uc774 \ud45c\ud604\uc774 \uc77c\ubc18\uc801\uc73c\ub85c \ub110\ub9ac \uc4f0\uc774\ub294 \uc6a9\uc5b4\uc774\uae30 \ub54c\ubb38\uc774\ub2e4.", "answer": "\ud074\ub9ad\ub960", "sentence": "\ubc14\ub85c \ud074\ub9ad\ub960 \uc774\ub2e4.", "paragraph_sentence": "\ub9c8\ucf00\ud305 \uc804\ubb38\uac00\ub4e4\uc740 \uc6f9 \ubc29\ubb38\uc790\ub4e4\uc758 \ubc18\uc751\uc744 \uc54c\uace0 \uc2f6\uc5b4\ud558\uc9c0\ub9cc, \ud2b9\uc815 \uc6f9\uc0ac\uc774\ud2b8\uc5d0 \ub300\ud55c \uc0ac\uc6a9\uc790\ub4e4\uc758 \uac10\uc815\uc801 \ubc18\uc751 \ubc0f \uc6f9\uc0ac\uc774\ud2b8\ub97c \ud1b5\ud55c \uae30\uc5c5\uc758 \ube0c\ub79c\ub4dc \uc774\ubbf8\uc9c0 \uc0c1\uc2b9 \ud6a8\uacfc\ub97c \uc644\uc804\ud788 \uc815\ub7c9\ud654 \uc2dc\ud0a4\ub294 \uac83\uc740 \ud604\uc7ac\uc758 \uae30\uc220\ub85c\uc11c\ub294 \uac70\uc758 \ubd88\uac00\ub2a5\ud558\ub2e4\uace0 \ubcfc \uc218 \uc788\ub2e4. \uc774\uc5d0 \ubc18\ud574 \ubcf4\ub2e4 \uc27d\uac8c \uc5bb\uc5b4\ub0bc \uc218 \uc788\ub294 \uc815\ubcf4\uac00 \ubc14\ub85c \ud074\ub9ad\ub960 \uc774\ub2e4. \ud074\ub9ad\ub960\uc740 \uad11\uace0\uc5d0 \uc758\ud574 \uc758\ub3c4\ub41c \ud398\uc774\uc9c0\ub85c \uc5f0\uacb0\ub41c \ubc29\ubb38\uac1d \uc911 \uc5b4\ub5a0\ud55c \ud488\ubaa9\uc744 \uad6c\uc785\ud558\uac70\ub098 \uc81c\ud488 \ub610\ub294 \uc11c\ube44\uc2a4\uc5d0 \ub300\ud55c \uc815\ubcf4\ub97c \uc5bb\ub294 \ub4f1, \uad11\uace0\ub85c\ubd80\ud130 \uc720\ub3c4\ub41c \ud589\uc704\ub97c \uc2dc\ud589\ud55c \ubc29\ubb38\uc790\uc758 \ube44\uc728\uc744 \uce21\uc815\ud55c\ub2e4. \ub2e4\uc591\ud55c \ud615\ud0dc\uc758 \uc0c1\ud638 \uc791\uc6a9\uc774 \uac00\ub2a5\ud558\uc9c0\ub9cc \uc5ec\uae30\uc11c \uad11\uace0(\ud639\uc740 \ub9c1\ud06c)\uc5d0 \ub300\ud574 \"\ub9c8\uc6b0\uc2a4 \ud074\ub9ad\"\uc774\ub77c\ub294 \ud45c\ud604\uc744 \uc0ac\uc6a9\ud55c \uac83\uc740 \uc774 \ud45c\ud604\uc774 \uc77c\ubc18\uc801\uc73c\ub85c \ub110\ub9ac \uc4f0\uc774\ub294 \uc6a9\uc5b4\uc774\uae30 \ub54c\ubb38\uc774\ub2e4.", "paragraph_answer": "\ub9c8\ucf00\ud305 \uc804\ubb38\uac00\ub4e4\uc740 \uc6f9 \ubc29\ubb38\uc790\ub4e4\uc758 \ubc18\uc751\uc744 \uc54c\uace0 \uc2f6\uc5b4\ud558\uc9c0\ub9cc, \ud2b9\uc815 \uc6f9\uc0ac\uc774\ud2b8\uc5d0 \ub300\ud55c \uc0ac\uc6a9\uc790\ub4e4\uc758 \uac10\uc815\uc801 \ubc18\uc751 \ubc0f \uc6f9\uc0ac\uc774\ud2b8\ub97c \ud1b5\ud55c \uae30\uc5c5\uc758 \ube0c\ub79c\ub4dc \uc774\ubbf8\uc9c0 \uc0c1\uc2b9 \ud6a8\uacfc\ub97c \uc644\uc804\ud788 \uc815\ub7c9\ud654 \uc2dc\ud0a4\ub294 \uac83\uc740 \ud604\uc7ac\uc758 \uae30\uc220\ub85c\uc11c\ub294 \uac70\uc758 \ubd88\uac00\ub2a5\ud558\ub2e4\uace0 \ubcfc \uc218 \uc788\ub2e4. \uc774\uc5d0 \ubc18\ud574 \ubcf4\ub2e4 \uc27d\uac8c \uc5bb\uc5b4\ub0bc \uc218 \uc788\ub294 \uc815\ubcf4\uac00 \ubc14\ub85c \ud074\ub9ad\ub960 \uc774\ub2e4. \ud074\ub9ad\ub960\uc740 \uad11\uace0\uc5d0 \uc758\ud574 \uc758\ub3c4\ub41c \ud398\uc774\uc9c0\ub85c \uc5f0\uacb0\ub41c \ubc29\ubb38\uac1d \uc911 \uc5b4\ub5a0\ud55c \ud488\ubaa9\uc744 \uad6c\uc785\ud558\uac70\ub098 \uc81c\ud488 \ub610\ub294 \uc11c\ube44\uc2a4\uc5d0 \ub300\ud55c \uc815\ubcf4\ub97c \uc5bb\ub294 \ub4f1, \uad11\uace0\ub85c\ubd80\ud130 \uc720\ub3c4\ub41c \ud589\uc704\ub97c \uc2dc\ud589\ud55c \ubc29\ubb38\uc790\uc758 \ube44\uc728\uc744 \uce21\uc815\ud55c\ub2e4. \ub2e4\uc591\ud55c \ud615\ud0dc\uc758 \uc0c1\ud638 \uc791\uc6a9\uc774 \uac00\ub2a5\ud558\uc9c0\ub9cc \uc5ec\uae30\uc11c \uad11\uace0(\ud639\uc740 \ub9c1\ud06c)\uc5d0 \ub300\ud574 \"\ub9c8\uc6b0\uc2a4 \ud074\ub9ad\"\uc774\ub77c\ub294 \ud45c\ud604\uc744 \uc0ac\uc6a9\ud55c \uac83\uc740 \uc774 \ud45c\ud604\uc774 \uc77c\ubc18\uc801\uc73c\ub85c \ub110\ub9ac \uc4f0\uc774\ub294 \uc6a9\uc5b4\uc774\uae30 \ub54c\ubb38\uc774\ub2e4.", "sentence_answer": "\ubc14\ub85c \ud074\ub9ad\ub960 \uc774\ub2e4.", "paragraph_id": "6564785-0-0", "paragraph_question": "question: \uad11\uace0\ub85c\ubd80\ud130 \uc720\ub3c4\ub41c \ud589\uc704\ub97c \ud558\ub294 \ubc29\ubb38\uc790\uc758 \ube44\uc728\uc744 \uce21\uc815\ud558\ub294 \uac83\uc740?, context: \ub9c8\ucf00\ud305 \uc804\ubb38\uac00\ub4e4\uc740 \uc6f9 \ubc29\ubb38\uc790\ub4e4\uc758 \ubc18\uc751\uc744 \uc54c\uace0 \uc2f6\uc5b4\ud558\uc9c0\ub9cc, \ud2b9\uc815 \uc6f9\uc0ac\uc774\ud2b8\uc5d0 \ub300\ud55c \uc0ac\uc6a9\uc790\ub4e4\uc758 \uac10\uc815\uc801 \ubc18\uc751 \ubc0f \uc6f9\uc0ac\uc774\ud2b8\ub97c \ud1b5\ud55c \uae30\uc5c5\uc758 \ube0c\ub79c\ub4dc \uc774\ubbf8\uc9c0 \uc0c1\uc2b9 \ud6a8\uacfc\ub97c \uc644\uc804\ud788 \uc815\ub7c9\ud654 \uc2dc\ud0a4\ub294 \uac83\uc740 \ud604\uc7ac\uc758 \uae30\uc220\ub85c\uc11c\ub294 \uac70\uc758 \ubd88\uac00\ub2a5\ud558\ub2e4\uace0 \ubcfc \uc218 \uc788\ub2e4. \uc774\uc5d0 \ubc18\ud574 \ubcf4\ub2e4 \uc27d\uac8c \uc5bb\uc5b4\ub0bc \uc218 \uc788\ub294 \uc815\ubcf4\uac00 \ubc14\ub85c \ud074\ub9ad\ub960\uc774\ub2e4. \ud074\ub9ad\ub960\uc740 \uad11\uace0\uc5d0 \uc758\ud574 \uc758\ub3c4\ub41c \ud398\uc774\uc9c0\ub85c \uc5f0\uacb0\ub41c \ubc29\ubb38\uac1d \uc911 \uc5b4\ub5a0\ud55c \ud488\ubaa9\uc744 \uad6c\uc785\ud558\uac70\ub098 \uc81c\ud488 \ub610\ub294 \uc11c\ube44\uc2a4\uc5d0 \ub300\ud55c \uc815\ubcf4\ub97c \uc5bb\ub294 \ub4f1, \uad11\uace0\ub85c\ubd80\ud130 \uc720\ub3c4\ub41c \ud589\uc704\ub97c \uc2dc\ud589\ud55c \ubc29\ubb38\uc790\uc758 \ube44\uc728\uc744 \uce21\uc815\ud55c\ub2e4. \ub2e4\uc591\ud55c \ud615\ud0dc\uc758 \uc0c1\ud638 \uc791\uc6a9\uc774 \uac00\ub2a5\ud558\uc9c0\ub9cc \uc5ec\uae30\uc11c \uad11\uace0(\ud639\uc740 \ub9c1\ud06c)\uc5d0 \ub300\ud574 \"\ub9c8\uc6b0\uc2a4 \ud074\ub9ad\"\uc774\ub77c\ub294 \ud45c\ud604\uc744 \uc0ac\uc6a9\ud55c \uac83\uc740 \uc774 \ud45c\ud604\uc774 \uc77c\ubc18\uc801\uc73c\ub85c \ub110\ub9ac \uc4f0\uc774\ub294 \uc6a9\uc5b4\uc774\uae30 \ub54c\ubb38\uc774\ub2e4."} {"question": "\ud504\ub791\uc2a4\uc640 \uadf8\ub808\uc774\ud2b8\ube0c\ub9ac\ud2bc \uc655\uad6d\uc774 \uc870\uc57d\uc758 \ud56d\ubaa9\uc744 \ub17c\uc758\ud558\uae30 \uc2dc\uc791\ud55c \uac83\uc740 \uc5b8\uc81c\uc778\uac00?", "paragraph": "\ud504\ub791\uc2a4\uc640 \uadf8\ub808\uc774\ud2b8\ube0c\ub9ac\ud2bc \uc655\uad6d\uc740 \ub7f0\ub358\uc5d0\uc11c \uc870\uc57d\uc758 \ucd08\uc548\uc774 \uc774\ubbf8 \uc0ac\uc778\ub41c, 1711\ub144 10\uc6d4\uc5d0 \uc870\uc57d\uc758 \ud56d\ubaa9\uc744 \ub17c\uc758\ud558\ub294 \ub370 \uc774\ub974\ub800\ub2e4. \uc774 \uc870\uc57d\uc5d0\uc11c \uc6b0\uc120 \uc2a4\ud398\uc778\uc774 \uc18c\uc720\ud558\uace0 \uc788\ub294 \uc720\ub7fd\uc9c0\uc5ed\uc758 \uc601\ud1a0\ubd84\ud560\uc744 \uc778\uc815\ud558\uace0 \ubc1b\uc544\ub4e4\uc778\ub2e4\ub294 \uc591\ud574\ub97c \uae30\ucd08\ub85c \ud558\uace0 \uc788\ub2e4. \uc774\uc5d0 \ub4a4\uc774\uc5b4, \ube0c\ub9ac\uc2a4\ud1a8\uc758 \uc8fc\uad50 \uc874 \ub85c\ube48\uc2a8\uacfc \uc2a4\ud2b8\ub798\ud3ec\ub4dc \uacbd \ud1a0\ub9c8\uc2a4 \uc6ec\ud2b8\uc6cc\uc2a4\uac00 \ub300\ud45c\ub85c \ucc38\uc5ec\ud55c \ud68c\ub2f4\uc774 1712\ub144 1\uc6d4 29\uc77c \uc704\ud2b8\ub808\ud750\ud2b8\uc5d0\uc11c \uc5f4\ub838\ub2e4. \ub124\ub35c\ub780\ub4dc\ub294 \ub0b4\ud0a4\uc9c0 \uc54a\uc558\uc9c0\ub9cc, \uc870\uc57d\uc758 \ucd08\uc548\uc744 \ubc1b\uc544\ub4e4\uc774\uace0 \ub300\ud45c\ub97c \ud30c\uacac\ud558\ub294 \uac83\uc744 \ubc1b\uc544\ub4e4\uc600\ub2e4. \uadf8\ub7ec\ub098 \ud669\uc81c\ub294 \uadf8\uac00 \uc774 \ucd08\uc548\uc774 \uad6c\uc18d\ub825\uc774 \uc5c6\ub2e4\ub294 \uac83\uc744 \ud655\uc2e0\ud558\uae30 \uc804\uae4c\uc9c0\ub294 \ucd08\uc548\uc744 \ubc1b\uc544\ub4e4\uc774\uace0 \ub300\ud45c\ub97c \ud30c\uacac\ud558\ub294 \uac83\uc744 \uac70\ubd80\ud558\uc600\ub2e4. \uc774\ub7ec\ud55c \ud655\uc2e0\uc744 \ubc1b\uace0 2\uc6d4\uc774 \ub418\uc5b4\uc11c\uc57c \uc81c\uad6d\uc758 \ub300\ud45c\uac00 \ud68c\ub2f4\uc7a5\uc5d0 \ub098\ud0c0\ub0ac\ub2e4. \ud3a0\ub9ac\ud398 5\uc138\uac00 \uc544\uc9c1 \uc2a4\uc2a4\ub85c \uc2a4\ud398\uc778\uc758 \uc655\uc774\ub77c\ub294 \uac83\uc744 \uc790\uac01\ud558\uc9c0 \ubabb\ud588\uae30 \ub54c\ubb38\uc5d0, \uc2a4\ud398\uc778\uc740 \ucc98\uc74c\uc5d0\ub294 \uc804\uad8c\ub300\uc0ac\ub97c \ud30c\uacac\ud558\uc9c0 \uc54a\uc558\uc73c\ub098, \uc0ac\ubcf4\uc774 \uacf5\uad6d\uacfc \ud3ec\ub974\ud22c\uac08\uc740 \uc804\uad8c\ub300\uc0ac\ub97c \ubcf4\ub0c8\ub2e4.", "answer": "1711\ub144 10\uc6d4", "sentence": "\ud504\ub791\uc2a4\uc640 \uadf8\ub808\uc774\ud2b8\ube0c\ub9ac\ud2bc \uc655\uad6d\uc740 \ub7f0\ub358\uc5d0\uc11c \uc870\uc57d\uc758 \ucd08\uc548\uc774 \uc774\ubbf8 \uc0ac\uc778\ub41c, 1711\ub144 10\uc6d4 \uc5d0 \uc870\uc57d\uc758 \ud56d\ubaa9\uc744 \ub17c\uc758\ud558\ub294 \ub370 \uc774\ub974\ub800\ub2e4.", "paragraph_sentence": " \ud504\ub791\uc2a4\uc640 \uadf8\ub808\uc774\ud2b8\ube0c\ub9ac\ud2bc \uc655\uad6d\uc740 \ub7f0\ub358\uc5d0\uc11c \uc870\uc57d\uc758 \ucd08\uc548\uc774 \uc774\ubbf8 \uc0ac\uc778\ub41c, 1711\ub144 10\uc6d4 \uc5d0 \uc870\uc57d\uc758 \ud56d\ubaa9\uc744 \ub17c\uc758\ud558\ub294 \ub370 \uc774\ub974\ub800\ub2e4. \uc774 \uc870\uc57d\uc5d0\uc11c \uc6b0\uc120 \uc2a4\ud398\uc778\uc774 \uc18c\uc720\ud558\uace0 \uc788\ub294 \uc720\ub7fd\uc9c0\uc5ed\uc758 \uc601\ud1a0\ubd84\ud560\uc744 \uc778\uc815\ud558\uace0 \ubc1b\uc544\ub4e4\uc778\ub2e4\ub294 \uc591\ud574\ub97c \uae30\ucd08\ub85c \ud558\uace0 \uc788\ub2e4. \uc774\uc5d0 \ub4a4\uc774\uc5b4, \ube0c\ub9ac\uc2a4\ud1a8\uc758 \uc8fc\uad50 \uc874 \ub85c\ube48\uc2a8\uacfc \uc2a4\ud2b8\ub798\ud3ec\ub4dc \uacbd \ud1a0\ub9c8\uc2a4 \uc6ec\ud2b8\uc6cc\uc2a4\uac00 \ub300\ud45c\ub85c \ucc38\uc5ec\ud55c \ud68c\ub2f4\uc774 1712\ub144 1\uc6d4 29\uc77c \uc704\ud2b8\ub808\ud750\ud2b8\uc5d0\uc11c \uc5f4\ub838\ub2e4. \ub124\ub35c\ub780\ub4dc\ub294 \ub0b4\ud0a4\uc9c0 \uc54a\uc558\uc9c0\ub9cc, \uc870\uc57d\uc758 \ucd08\uc548\uc744 \ubc1b\uc544\ub4e4\uc774\uace0 \ub300\ud45c\ub97c \ud30c\uacac\ud558\ub294 \uac83\uc744 \ubc1b\uc544\ub4e4\uc600\ub2e4. \uadf8\ub7ec\ub098 \ud669\uc81c\ub294 \uadf8\uac00 \uc774 \ucd08\uc548\uc774 \uad6c\uc18d\ub825\uc774 \uc5c6\ub2e4\ub294 \uac83\uc744 \ud655\uc2e0\ud558\uae30 \uc804\uae4c\uc9c0\ub294 \ucd08\uc548\uc744 \ubc1b\uc544\ub4e4\uc774\uace0 \ub300\ud45c\ub97c \ud30c\uacac\ud558\ub294 \uac83\uc744 \uac70\ubd80\ud558\uc600\ub2e4. \uc774\ub7ec\ud55c \ud655\uc2e0\uc744 \ubc1b\uace0 2\uc6d4\uc774 \ub418\uc5b4\uc11c\uc57c \uc81c\uad6d\uc758 \ub300\ud45c\uac00 \ud68c\ub2f4\uc7a5\uc5d0 \ub098\ud0c0\ub0ac\ub2e4. \ud3a0\ub9ac\ud398 5\uc138\uac00 \uc544\uc9c1 \uc2a4\uc2a4\ub85c \uc2a4\ud398\uc778\uc758 \uc655\uc774\ub77c\ub294 \uac83\uc744 \uc790\uac01\ud558\uc9c0 \ubabb\ud588\uae30 \ub54c\ubb38\uc5d0, \uc2a4\ud398\uc778\uc740 \ucc98\uc74c\uc5d0\ub294 \uc804\uad8c\ub300\uc0ac\ub97c \ud30c\uacac\ud558\uc9c0 \uc54a\uc558\uc73c\ub098, \uc0ac\ubcf4\uc774 \uacf5\uad6d\uacfc \ud3ec\ub974\ud22c\uac08\uc740 \uc804\uad8c\ub300\uc0ac\ub97c \ubcf4\ub0c8\ub2e4.", "paragraph_answer": "\ud504\ub791\uc2a4\uc640 \uadf8\ub808\uc774\ud2b8\ube0c\ub9ac\ud2bc \uc655\uad6d\uc740 \ub7f0\ub358\uc5d0\uc11c \uc870\uc57d\uc758 \ucd08\uc548\uc774 \uc774\ubbf8 \uc0ac\uc778\ub41c, 1711\ub144 10\uc6d4 \uc5d0 \uc870\uc57d\uc758 \ud56d\ubaa9\uc744 \ub17c\uc758\ud558\ub294 \ub370 \uc774\ub974\ub800\ub2e4. \uc774 \uc870\uc57d\uc5d0\uc11c \uc6b0\uc120 \uc2a4\ud398\uc778\uc774 \uc18c\uc720\ud558\uace0 \uc788\ub294 \uc720\ub7fd\uc9c0\uc5ed\uc758 \uc601\ud1a0\ubd84\ud560\uc744 \uc778\uc815\ud558\uace0 \ubc1b\uc544\ub4e4\uc778\ub2e4\ub294 \uc591\ud574\ub97c \uae30\ucd08\ub85c \ud558\uace0 \uc788\ub2e4. \uc774\uc5d0 \ub4a4\uc774\uc5b4, \ube0c\ub9ac\uc2a4\ud1a8\uc758 \uc8fc\uad50 \uc874 \ub85c\ube48\uc2a8\uacfc \uc2a4\ud2b8\ub798\ud3ec\ub4dc \uacbd \ud1a0\ub9c8\uc2a4 \uc6ec\ud2b8\uc6cc\uc2a4\uac00 \ub300\ud45c\ub85c \ucc38\uc5ec\ud55c \ud68c\ub2f4\uc774 1712\ub144 1\uc6d4 29\uc77c \uc704\ud2b8\ub808\ud750\ud2b8\uc5d0\uc11c \uc5f4\ub838\ub2e4. \ub124\ub35c\ub780\ub4dc\ub294 \ub0b4\ud0a4\uc9c0 \uc54a\uc558\uc9c0\ub9cc, \uc870\uc57d\uc758 \ucd08\uc548\uc744 \ubc1b\uc544\ub4e4\uc774\uace0 \ub300\ud45c\ub97c \ud30c\uacac\ud558\ub294 \uac83\uc744 \ubc1b\uc544\ub4e4\uc600\ub2e4. \uadf8\ub7ec\ub098 \ud669\uc81c\ub294 \uadf8\uac00 \uc774 \ucd08\uc548\uc774 \uad6c\uc18d\ub825\uc774 \uc5c6\ub2e4\ub294 \uac83\uc744 \ud655\uc2e0\ud558\uae30 \uc804\uae4c\uc9c0\ub294 \ucd08\uc548\uc744 \ubc1b\uc544\ub4e4\uc774\uace0 \ub300\ud45c\ub97c \ud30c\uacac\ud558\ub294 \uac83\uc744 \uac70\ubd80\ud558\uc600\ub2e4. \uc774\ub7ec\ud55c \ud655\uc2e0\uc744 \ubc1b\uace0 2\uc6d4\uc774 \ub418\uc5b4\uc11c\uc57c \uc81c\uad6d\uc758 \ub300\ud45c\uac00 \ud68c\ub2f4\uc7a5\uc5d0 \ub098\ud0c0\ub0ac\ub2e4. \ud3a0\ub9ac\ud398 5\uc138\uac00 \uc544\uc9c1 \uc2a4\uc2a4\ub85c \uc2a4\ud398\uc778\uc758 \uc655\uc774\ub77c\ub294 \uac83\uc744 \uc790\uac01\ud558\uc9c0 \ubabb\ud588\uae30 \ub54c\ubb38\uc5d0, \uc2a4\ud398\uc778\uc740 \ucc98\uc74c\uc5d0\ub294 \uc804\uad8c\ub300\uc0ac\ub97c \ud30c\uacac\ud558\uc9c0 \uc54a\uc558\uc73c\ub098, \uc0ac\ubcf4\uc774 \uacf5\uad6d\uacfc \ud3ec\ub974\ud22c\uac08\uc740 \uc804\uad8c\ub300\uc0ac\ub97c \ubcf4\ub0c8\ub2e4.", "sentence_answer": "\ud504\ub791\uc2a4\uc640 \uadf8\ub808\uc774\ud2b8\ube0c\ub9ac\ud2bc \uc655\uad6d\uc740 \ub7f0\ub358\uc5d0\uc11c \uc870\uc57d\uc758 \ucd08\uc548\uc774 \uc774\ubbf8 \uc0ac\uc778\ub41c, 1711\ub144 10\uc6d4 \uc5d0 \uc870\uc57d\uc758 \ud56d\ubaa9\uc744 \ub17c\uc758\ud558\ub294 \ub370 \uc774\ub974\ub800\ub2e4.", "paragraph_id": "6657456-0-1", "paragraph_question": "question: \ud504\ub791\uc2a4\uc640 \uadf8\ub808\uc774\ud2b8\ube0c\ub9ac\ud2bc \uc655\uad6d\uc774 \uc870\uc57d\uc758 \ud56d\ubaa9\uc744 \ub17c\uc758\ud558\uae30 \uc2dc\uc791\ud55c \uac83\uc740 \uc5b8\uc81c\uc778\uac00?, context: \ud504\ub791\uc2a4\uc640 \uadf8\ub808\uc774\ud2b8\ube0c\ub9ac\ud2bc \uc655\uad6d\uc740 \ub7f0\ub358\uc5d0\uc11c \uc870\uc57d\uc758 \ucd08\uc548\uc774 \uc774\ubbf8 \uc0ac\uc778\ub41c, 1711\ub144 10\uc6d4\uc5d0 \uc870\uc57d\uc758 \ud56d\ubaa9\uc744 \ub17c\uc758\ud558\ub294 \ub370 \uc774\ub974\ub800\ub2e4. \uc774 \uc870\uc57d\uc5d0\uc11c \uc6b0\uc120 \uc2a4\ud398\uc778\uc774 \uc18c\uc720\ud558\uace0 \uc788\ub294 \uc720\ub7fd\uc9c0\uc5ed\uc758 \uc601\ud1a0\ubd84\ud560\uc744 \uc778\uc815\ud558\uace0 \ubc1b\uc544\ub4e4\uc778\ub2e4\ub294 \uc591\ud574\ub97c \uae30\ucd08\ub85c \ud558\uace0 \uc788\ub2e4. \uc774\uc5d0 \ub4a4\uc774\uc5b4, \ube0c\ub9ac\uc2a4\ud1a8\uc758 \uc8fc\uad50 \uc874 \ub85c\ube48\uc2a8\uacfc \uc2a4\ud2b8\ub798\ud3ec\ub4dc \uacbd \ud1a0\ub9c8\uc2a4 \uc6ec\ud2b8\uc6cc\uc2a4\uac00 \ub300\ud45c\ub85c \ucc38\uc5ec\ud55c \ud68c\ub2f4\uc774 1712\ub144 1\uc6d4 29\uc77c \uc704\ud2b8\ub808\ud750\ud2b8\uc5d0\uc11c \uc5f4\ub838\ub2e4. \ub124\ub35c\ub780\ub4dc\ub294 \ub0b4\ud0a4\uc9c0 \uc54a\uc558\uc9c0\ub9cc, \uc870\uc57d\uc758 \ucd08\uc548\uc744 \ubc1b\uc544\ub4e4\uc774\uace0 \ub300\ud45c\ub97c \ud30c\uacac\ud558\ub294 \uac83\uc744 \ubc1b\uc544\ub4e4\uc600\ub2e4. \uadf8\ub7ec\ub098 \ud669\uc81c\ub294 \uadf8\uac00 \uc774 \ucd08\uc548\uc774 \uad6c\uc18d\ub825\uc774 \uc5c6\ub2e4\ub294 \uac83\uc744 \ud655\uc2e0\ud558\uae30 \uc804\uae4c\uc9c0\ub294 \ucd08\uc548\uc744 \ubc1b\uc544\ub4e4\uc774\uace0 \ub300\ud45c\ub97c \ud30c\uacac\ud558\ub294 \uac83\uc744 \uac70\ubd80\ud558\uc600\ub2e4. \uc774\ub7ec\ud55c \ud655\uc2e0\uc744 \ubc1b\uace0 2\uc6d4\uc774 \ub418\uc5b4\uc11c\uc57c \uc81c\uad6d\uc758 \ub300\ud45c\uac00 \ud68c\ub2f4\uc7a5\uc5d0 \ub098\ud0c0\ub0ac\ub2e4. \ud3a0\ub9ac\ud398 5\uc138\uac00 \uc544\uc9c1 \uc2a4\uc2a4\ub85c \uc2a4\ud398\uc778\uc758 \uc655\uc774\ub77c\ub294 \uac83\uc744 \uc790\uac01\ud558\uc9c0 \ubabb\ud588\uae30 \ub54c\ubb38\uc5d0, \uc2a4\ud398\uc778\uc740 \ucc98\uc74c\uc5d0\ub294 \uc804\uad8c\ub300\uc0ac\ub97c \ud30c\uacac\ud558\uc9c0 \uc54a\uc558\uc73c\ub098, \uc0ac\ubcf4\uc774 \uacf5\uad6d\uacfc \ud3ec\ub974\ud22c\uac08\uc740 \uc804\uad8c\ub300\uc0ac\ub97c \ubcf4\ub0c8\ub2e4."} {"question": "1988\ub144 \uc774\ud6c4 \uc0ac\uac74\uc744 \ubbfc\uc8fc\ud654 \uc6b4\ub3d9\uc73c\ub85c \uaddc\uc815\ud558\uc5ec '5.18 \uad11\uc8fc \ubbfc\uc8fc\ud654 \uc6b4\ub3d9'\uc774\ub77c\ub294 \uba85\uce6d\uc774 \ub098\uc624\uac8c \ud55c \uc815\ubd80\uc758 \uc704\uc6d0\ud68c\ub294 \ubb34\uc5c7\uc778\uac00\uc694?", "paragraph": "5\u00b718 \uad11\uc8fc \ubbfc\uc8fc\ud654 \uc6b4\ub3d9\uc740 5\u00b718, \uad11\uc8fc\ud56d\uc7c1(\u5149\u5dde\u6297\u722d), \uad11\uc8fc\ud559\uc0b4(\u5149\u5dde\u8650\u6bba), \uad11\uc8fc\uc0ac\ud0dc(\u5149\u5dde\u4e8b\u614b), \uad11\uc8fc\ubbfc\uc911\ubd09\uae30(\u5149\u5dde\u6c11\u8846\u8702\u8d77), \uad11\uc8fc\uc2dc\ubbfc\ud56d\uc7c1(\u5149\u5dde\u5e02\u6c11\u6297\u722d) \ub4f1 \ub2e4\uc591\ud55c \uc774\ub984\uc73c\ub85c \ubd88\ub9ac\uace0 \uc788\ub2e4. '5\u00b718 \uad11\uc8fc \ubbfc\uc8fc\ud654 \uc6b4\ub3d9'\uc5d0 \ub300\ud55c \uba85\uce6d\uc740 \uc0ac\uac74\uc774 \ud55c\ucc3d \uc9c4\ud589 \uc911\uc774\ub358 1980\ub144 5\uc6d4 21\uc77c \uacc4\uc5c4\uc0ac\ub839\uad00 \uc774\ud76c\uc131\uc774 \"\uad11\uc8fc\uc5d0\uc11c \uc18c\uc694\uc0ac\ud0dc\uac00 \uc77c\uc5b4\ub098\uace0 \uc788\ub2e4\"\ub77c\uace0 \uad70\ubd80 \ubc1c\ud45c\uc5d0\uc11c \uc5b8\uae09\ud55c \uac83\uc774 \ucc98\uc74c\uc73c\ub85c, \uc774\ud6c4 \uc2e0\uad70\ubd80\uc640 \uad00\ubcc0 \uc5b8\ub860 \ub4f1\uc5d0 \uc758\ud574 '\uad11\uc8fc\uc18c\uc694\uc0ac\ud0dc' \ub610\ub294 '\uad11\uc8fc\uc0ac\ud0dc' \ub4f1\uc73c\ub85c \ubcf4\ub3c4\ub418\uba74\uc11c \uc77c\ubc18\ud654\ub418\uc5c8\ub2e4. \uc774\ub294 \uad11\uc8fc \uc790\uc720 \ubbfc\uc8fc\ud654 \ud56d\uc7c1\uc744 \"\ubd88\uc21c\ubd84\uc790\ub4e4\uc774 \uccb4\uc81c \uc804\ubcf5\uc744 \uae30\ub3c4\ud55c \uc0ac\ud0dc\"\ub85c \uc65c\uace1\ud55c \uc2e0\uad70\ubd80\uc758 \uc8fc\uc7a5\uc5d0 \uadfc\uac70\ud55c \ud638\uce6d\uc73c\ub85c \uc81c5\uacf5\ud654\uad6d \uae30\uac04 \ub0b4\ub0b4 \uc0ac\uc6a9\ub410\uc73c\uba70, \ud604\uc7ac\ub294 \ub2f9\uc2dc \ud638\uce6d\uc5d0 \uc775\uc219\ud55c \ub178\ub144\uce35\uc774\ub098 \uc2e0\uad70\ubd80\ub97c \uc9c0\uc9c0\ud558\ub294 \uc77c\ubd80 \uc6b0\ud30c \uc778\uc0ac\ub4e4\uc774 \uc0ac\uc6a9\ud558\uace0 \uc788\ub2e4. \ud604\uc7ac\uc758 \uba85\uce6d\uc778 '5\u00b718 \uad11\uc8fc \ubbfc\uc8fc\ud654 \uc6b4\ub3d9'\uc740 1988\ub144 \uc774\ud6c4 \uc815\ubd80 \uc0b0\ud558 \ubbfc\uc8fc\ud654\ud569\ucd94\uc9c4\uc704\uc6d0\ud68c\uac00 \uc0ac\uac74\uc744 \ubbfc\uc8fc\ud654 \uc6b4\ub3d9\uc73c\ub85c \uaddc\uc815\ud558\uba74\uc11c \ub098\uc654\uace0, \uc774\ud6c4 \ubb38\ubbfc\uc815\ubd80, \uad6d\ubbfc\uc758 \uc815\ubd80\uc758 \uacf5\uc2dd \uc5b8\uae09\uc5d0\uc11c\ub3c4 \uc774 \uba85\uce6d\uc774 \uc0ac\uc6a9\ub428\uc73c\ub85c\uc368 \uacf5\uc2dd \uba85\uce6d\ud654\ub418\uc5c8\ub2e4. \ud55c\ud3b8, \uc2e0\uad70\ubd80\uc758 \uad70\uc0ac\ub3c5\uc7ac\uc640 \ud3ed\ub825\uc5d0 \ub9de\uc120 \ubbfc\uc911\ud56d\uc7c1\uc744 \uad11\uc8fc \ubbfc\uc911\ub4e4\uc774 \uc8fc\ub3c4\ud588\ub2e4\ub294 \uc0ac\uc2e4\uc744 \uac15\uc870\ud55c '\uad11\uc8fc\ubbfc\uc911\ud56d\uc7c1' \ub610\ub294 '\uad11\uc8fc\ud56d\uc7c1'\uc774\ub780 \uba85\uce6d\ub3c4 \uc9c0\uc5ed \uc0ac\ud68c\uc640 5\uc6d4 \ub2e8\uccb4 \ub4f1\uc744 \uc911\uc2ec\uc73c\ub85c 1980\ub144\ub300\ubd80\ud130 \uc0ac\uc6a9\ub410\ub2e4. \uc0ac\uac74\uc774 \uc77c\uc5b4\ub09c \ub0a0\uc9dc\ub97c \ub534 '5\u00b718'\ub3c4 \ub110\ub9ac \ud1b5\uc6a9\ub418\ub294 \uba85\uce6d\uc774\ub2e4.", "answer": "\ubbfc\uc8fc\ud654\ud569\ucd94\uc9c4\uc704\uc6d0\ud68c", "sentence": "\ud604\uc7ac\uc758 \uba85\uce6d\uc778 '5\u00b718 \uad11\uc8fc \ubbfc\uc8fc\ud654 \uc6b4\ub3d9'\uc740 1988\ub144 \uc774\ud6c4 \uc815\ubd80 \uc0b0\ud558 \ubbfc\uc8fc\ud654\ud569\ucd94\uc9c4\uc704\uc6d0\ud68c \uac00 \uc0ac\uac74\uc744 \ubbfc\uc8fc\ud654 \uc6b4\ub3d9\uc73c\ub85c \uaddc\uc815\ud558\uba74\uc11c \ub098\uc654\uace0, \uc774\ud6c4 \ubb38\ubbfc\uc815\ubd80, \uad6d\ubbfc\uc758 \uc815\ubd80\uc758 \uacf5\uc2dd \uc5b8\uae09\uc5d0\uc11c\ub3c4 \uc774 \uba85\uce6d\uc774 \uc0ac\uc6a9\ub428\uc73c\ub85c\uc368 \uacf5\uc2dd \uba85\uce6d\ud654\ub418\uc5c8\ub2e4.", "paragraph_sentence": "5\u00b718 \uad11\uc8fc \ubbfc\uc8fc\ud654 \uc6b4\ub3d9\uc740 5\u00b718, \uad11\uc8fc\ud56d\uc7c1(\u5149\u5dde\u6297\u722d), \uad11\uc8fc\ud559\uc0b4(\u5149\u5dde\u8650\u6bba), \uad11\uc8fc\uc0ac\ud0dc(\u5149\u5dde\u4e8b\u614b), \uad11\uc8fc\ubbfc\uc911\ubd09\uae30(\u5149\u5dde\u6c11\u8846\u8702\u8d77), \uad11\uc8fc\uc2dc\ubbfc\ud56d\uc7c1(\u5149\u5dde\u5e02\u6c11\u6297\u722d) \ub4f1 \ub2e4\uc591\ud55c \uc774\ub984\uc73c\ub85c \ubd88\ub9ac\uace0 \uc788\ub2e4. '5\u00b718 \uad11\uc8fc \ubbfc\uc8fc\ud654 \uc6b4\ub3d9'\uc5d0 \ub300\ud55c \uba85\uce6d\uc740 \uc0ac\uac74\uc774 \ud55c\ucc3d \uc9c4\ud589 \uc911\uc774\ub358 1980\ub144 5\uc6d4 21\uc77c \uacc4\uc5c4\uc0ac\ub839\uad00 \uc774\ud76c\uc131\uc774 \"\uad11\uc8fc\uc5d0\uc11c \uc18c\uc694\uc0ac\ud0dc\uac00 \uc77c\uc5b4\ub098\uace0 \uc788\ub2e4\"\ub77c\uace0 \uad70\ubd80 \ubc1c\ud45c\uc5d0\uc11c \uc5b8\uae09\ud55c \uac83\uc774 \ucc98\uc74c\uc73c\ub85c, \uc774\ud6c4 \uc2e0\uad70\ubd80\uc640 \uad00\ubcc0 \uc5b8\ub860 \ub4f1\uc5d0 \uc758\ud574 '\uad11\uc8fc\uc18c\uc694\uc0ac\ud0dc' \ub610\ub294 '\uad11\uc8fc\uc0ac\ud0dc' \ub4f1\uc73c\ub85c \ubcf4\ub3c4\ub418\uba74\uc11c \uc77c\ubc18\ud654\ub418\uc5c8\ub2e4. \uc774\ub294 \uad11\uc8fc \uc790\uc720 \ubbfc\uc8fc\ud654 \ud56d\uc7c1\uc744 \"\ubd88\uc21c\ubd84\uc790\ub4e4\uc774 \uccb4\uc81c \uc804\ubcf5\uc744 \uae30\ub3c4\ud55c \uc0ac\ud0dc\"\ub85c \uc65c\uace1\ud55c \uc2e0\uad70\ubd80\uc758 \uc8fc\uc7a5\uc5d0 \uadfc\uac70\ud55c \ud638\uce6d\uc73c\ub85c \uc81c5\uacf5\ud654\uad6d \uae30\uac04 \ub0b4\ub0b4 \uc0ac\uc6a9\ub410\uc73c\uba70, \ud604\uc7ac\ub294 \ub2f9\uc2dc \ud638\uce6d\uc5d0 \uc775\uc219\ud55c \ub178\ub144\uce35\uc774\ub098 \uc2e0\uad70\ubd80\ub97c \uc9c0\uc9c0\ud558\ub294 \uc77c\ubd80 \uc6b0\ud30c \uc778\uc0ac\ub4e4\uc774 \uc0ac\uc6a9\ud558\uace0 \uc788\ub2e4. \ud604\uc7ac\uc758 \uba85\uce6d\uc778 '5\u00b718 \uad11\uc8fc \ubbfc\uc8fc\ud654 \uc6b4\ub3d9'\uc740 1988\ub144 \uc774\ud6c4 \uc815\ubd80 \uc0b0\ud558 \ubbfc\uc8fc\ud654\ud569\ucd94\uc9c4\uc704\uc6d0\ud68c \uac00 \uc0ac\uac74\uc744 \ubbfc\uc8fc\ud654 \uc6b4\ub3d9\uc73c\ub85c \uaddc\uc815\ud558\uba74\uc11c \ub098\uc654\uace0, \uc774\ud6c4 \ubb38\ubbfc\uc815\ubd80, \uad6d\ubbfc\uc758 \uc815\ubd80\uc758 \uacf5\uc2dd \uc5b8\uae09\uc5d0\uc11c\ub3c4 \uc774 \uba85\uce6d\uc774 \uc0ac\uc6a9\ub428\uc73c\ub85c\uc368 \uacf5\uc2dd \uba85\uce6d\ud654\ub418\uc5c8\ub2e4. \ud55c\ud3b8, \uc2e0\uad70\ubd80\uc758 \uad70\uc0ac\ub3c5\uc7ac\uc640 \ud3ed\ub825\uc5d0 \ub9de\uc120 \ubbfc\uc911\ud56d\uc7c1\uc744 \uad11\uc8fc \ubbfc\uc911\ub4e4\uc774 \uc8fc\ub3c4\ud588\ub2e4\ub294 \uc0ac\uc2e4\uc744 \uac15\uc870\ud55c '\uad11\uc8fc\ubbfc\uc911\ud56d\uc7c1' \ub610\ub294 '\uad11\uc8fc\ud56d\uc7c1'\uc774\ub780 \uba85\uce6d\ub3c4 \uc9c0\uc5ed \uc0ac\ud68c\uc640 5\uc6d4 \ub2e8\uccb4 \ub4f1\uc744 \uc911\uc2ec\uc73c\ub85c 1980\ub144\ub300\ubd80\ud130 \uc0ac\uc6a9\ub410\ub2e4. \uc0ac\uac74\uc774 \uc77c\uc5b4\ub09c \ub0a0\uc9dc\ub97c \ub534 '5\u00b718'\ub3c4 \ub110\ub9ac \ud1b5\uc6a9\ub418\ub294 \uba85\uce6d\uc774\ub2e4.", "paragraph_answer": "5\u00b718 \uad11\uc8fc \ubbfc\uc8fc\ud654 \uc6b4\ub3d9\uc740 5\u00b718, \uad11\uc8fc\ud56d\uc7c1(\u5149\u5dde\u6297\u722d), \uad11\uc8fc\ud559\uc0b4(\u5149\u5dde\u8650\u6bba), \uad11\uc8fc\uc0ac\ud0dc(\u5149\u5dde\u4e8b\u614b), \uad11\uc8fc\ubbfc\uc911\ubd09\uae30(\u5149\u5dde\u6c11\u8846\u8702\u8d77), \uad11\uc8fc\uc2dc\ubbfc\ud56d\uc7c1(\u5149\u5dde\u5e02\u6c11\u6297\u722d) \ub4f1 \ub2e4\uc591\ud55c \uc774\ub984\uc73c\ub85c \ubd88\ub9ac\uace0 \uc788\ub2e4. '5\u00b718 \uad11\uc8fc \ubbfc\uc8fc\ud654 \uc6b4\ub3d9'\uc5d0 \ub300\ud55c \uba85\uce6d\uc740 \uc0ac\uac74\uc774 \ud55c\ucc3d \uc9c4\ud589 \uc911\uc774\ub358 1980\ub144 5\uc6d4 21\uc77c \uacc4\uc5c4\uc0ac\ub839\uad00 \uc774\ud76c\uc131\uc774 \"\uad11\uc8fc\uc5d0\uc11c \uc18c\uc694\uc0ac\ud0dc\uac00 \uc77c\uc5b4\ub098\uace0 \uc788\ub2e4\"\ub77c\uace0 \uad70\ubd80 \ubc1c\ud45c\uc5d0\uc11c \uc5b8\uae09\ud55c \uac83\uc774 \ucc98\uc74c\uc73c\ub85c, \uc774\ud6c4 \uc2e0\uad70\ubd80\uc640 \uad00\ubcc0 \uc5b8\ub860 \ub4f1\uc5d0 \uc758\ud574 '\uad11\uc8fc\uc18c\uc694\uc0ac\ud0dc' \ub610\ub294 '\uad11\uc8fc\uc0ac\ud0dc' \ub4f1\uc73c\ub85c \ubcf4\ub3c4\ub418\uba74\uc11c \uc77c\ubc18\ud654\ub418\uc5c8\ub2e4. \uc774\ub294 \uad11\uc8fc \uc790\uc720 \ubbfc\uc8fc\ud654 \ud56d\uc7c1\uc744 \"\ubd88\uc21c\ubd84\uc790\ub4e4\uc774 \uccb4\uc81c \uc804\ubcf5\uc744 \uae30\ub3c4\ud55c \uc0ac\ud0dc\"\ub85c \uc65c\uace1\ud55c \uc2e0\uad70\ubd80\uc758 \uc8fc\uc7a5\uc5d0 \uadfc\uac70\ud55c \ud638\uce6d\uc73c\ub85c \uc81c5\uacf5\ud654\uad6d \uae30\uac04 \ub0b4\ub0b4 \uc0ac\uc6a9\ub410\uc73c\uba70, \ud604\uc7ac\ub294 \ub2f9\uc2dc \ud638\uce6d\uc5d0 \uc775\uc219\ud55c \ub178\ub144\uce35\uc774\ub098 \uc2e0\uad70\ubd80\ub97c \uc9c0\uc9c0\ud558\ub294 \uc77c\ubd80 \uc6b0\ud30c \uc778\uc0ac\ub4e4\uc774 \uc0ac\uc6a9\ud558\uace0 \uc788\ub2e4. \ud604\uc7ac\uc758 \uba85\uce6d\uc778 '5\u00b718 \uad11\uc8fc \ubbfc\uc8fc\ud654 \uc6b4\ub3d9'\uc740 1988\ub144 \uc774\ud6c4 \uc815\ubd80 \uc0b0\ud558 \ubbfc\uc8fc\ud654\ud569\ucd94\uc9c4\uc704\uc6d0\ud68c \uac00 \uc0ac\uac74\uc744 \ubbfc\uc8fc\ud654 \uc6b4\ub3d9\uc73c\ub85c \uaddc\uc815\ud558\uba74\uc11c \ub098\uc654\uace0, \uc774\ud6c4 \ubb38\ubbfc\uc815\ubd80, \uad6d\ubbfc\uc758 \uc815\ubd80\uc758 \uacf5\uc2dd \uc5b8\uae09\uc5d0\uc11c\ub3c4 \uc774 \uba85\uce6d\uc774 \uc0ac\uc6a9\ub428\uc73c\ub85c\uc368 \uacf5\uc2dd \uba85\uce6d\ud654\ub418\uc5c8\ub2e4. \ud55c\ud3b8, \uc2e0\uad70\ubd80\uc758 \uad70\uc0ac\ub3c5\uc7ac\uc640 \ud3ed\ub825\uc5d0 \ub9de\uc120 \ubbfc\uc911\ud56d\uc7c1\uc744 \uad11\uc8fc \ubbfc\uc911\ub4e4\uc774 \uc8fc\ub3c4\ud588\ub2e4\ub294 \uc0ac\uc2e4\uc744 \uac15\uc870\ud55c '\uad11\uc8fc\ubbfc\uc911\ud56d\uc7c1' \ub610\ub294 '\uad11\uc8fc\ud56d\uc7c1'\uc774\ub780 \uba85\uce6d\ub3c4 \uc9c0\uc5ed \uc0ac\ud68c\uc640 5\uc6d4 \ub2e8\uccb4 \ub4f1\uc744 \uc911\uc2ec\uc73c\ub85c 1980\ub144\ub300\ubd80\ud130 \uc0ac\uc6a9\ub410\ub2e4. \uc0ac\uac74\uc774 \uc77c\uc5b4\ub09c \ub0a0\uc9dc\ub97c \ub534 '5\u00b718'\ub3c4 \ub110\ub9ac \ud1b5\uc6a9\ub418\ub294 \uba85\uce6d\uc774\ub2e4.", "sentence_answer": "\ud604\uc7ac\uc758 \uba85\uce6d\uc778 '5\u00b718 \uad11\uc8fc \ubbfc\uc8fc\ud654 \uc6b4\ub3d9'\uc740 1988\ub144 \uc774\ud6c4 \uc815\ubd80 \uc0b0\ud558 \ubbfc\uc8fc\ud654\ud569\ucd94\uc9c4\uc704\uc6d0\ud68c \uac00 \uc0ac\uac74\uc744 \ubbfc\uc8fc\ud654 \uc6b4\ub3d9\uc73c\ub85c \uaddc\uc815\ud558\uba74\uc11c \ub098\uc654\uace0, \uc774\ud6c4 \ubb38\ubbfc\uc815\ubd80, \uad6d\ubbfc\uc758 \uc815\ubd80\uc758 \uacf5\uc2dd \uc5b8\uae09\uc5d0\uc11c\ub3c4 \uc774 \uba85\uce6d\uc774 \uc0ac\uc6a9\ub428\uc73c\ub85c\uc368 \uacf5\uc2dd \uba85\uce6d\ud654\ub418\uc5c8\ub2e4.", "paragraph_id": "5801882-4-1", "paragraph_question": "question: 1988\ub144 \uc774\ud6c4 \uc0ac\uac74\uc744 \ubbfc\uc8fc\ud654 \uc6b4\ub3d9\uc73c\ub85c \uaddc\uc815\ud558\uc5ec '5.18 \uad11\uc8fc \ubbfc\uc8fc\ud654 \uc6b4\ub3d9'\uc774\ub77c\ub294 \uba85\uce6d\uc774 \ub098\uc624\uac8c \ud55c \uc815\ubd80\uc758 \uc704\uc6d0\ud68c\ub294 \ubb34\uc5c7\uc778\uac00\uc694?, context: 5\u00b718 \uad11\uc8fc \ubbfc\uc8fc\ud654 \uc6b4\ub3d9\uc740 5\u00b718, \uad11\uc8fc\ud56d\uc7c1(\u5149\u5dde\u6297\u722d), \uad11\uc8fc\ud559\uc0b4(\u5149\u5dde\u8650\u6bba), \uad11\uc8fc\uc0ac\ud0dc(\u5149\u5dde\u4e8b\u614b), \uad11\uc8fc\ubbfc\uc911\ubd09\uae30(\u5149\u5dde\u6c11\u8846\u8702\u8d77), \uad11\uc8fc\uc2dc\ubbfc\ud56d\uc7c1(\u5149\u5dde\u5e02\u6c11\u6297\u722d) \ub4f1 \ub2e4\uc591\ud55c \uc774\ub984\uc73c\ub85c \ubd88\ub9ac\uace0 \uc788\ub2e4. '5\u00b718 \uad11\uc8fc \ubbfc\uc8fc\ud654 \uc6b4\ub3d9'\uc5d0 \ub300\ud55c \uba85\uce6d\uc740 \uc0ac\uac74\uc774 \ud55c\ucc3d \uc9c4\ud589 \uc911\uc774\ub358 1980\ub144 5\uc6d4 21\uc77c \uacc4\uc5c4\uc0ac\ub839\uad00 \uc774\ud76c\uc131\uc774 \"\uad11\uc8fc\uc5d0\uc11c \uc18c\uc694\uc0ac\ud0dc\uac00 \uc77c\uc5b4\ub098\uace0 \uc788\ub2e4\"\ub77c\uace0 \uad70\ubd80 \ubc1c\ud45c\uc5d0\uc11c \uc5b8\uae09\ud55c \uac83\uc774 \ucc98\uc74c\uc73c\ub85c, \uc774\ud6c4 \uc2e0\uad70\ubd80\uc640 \uad00\ubcc0 \uc5b8\ub860 \ub4f1\uc5d0 \uc758\ud574 '\uad11\uc8fc\uc18c\uc694\uc0ac\ud0dc' \ub610\ub294 '\uad11\uc8fc\uc0ac\ud0dc' \ub4f1\uc73c\ub85c \ubcf4\ub3c4\ub418\uba74\uc11c \uc77c\ubc18\ud654\ub418\uc5c8\ub2e4. \uc774\ub294 \uad11\uc8fc \uc790\uc720 \ubbfc\uc8fc\ud654 \ud56d\uc7c1\uc744 \"\ubd88\uc21c\ubd84\uc790\ub4e4\uc774 \uccb4\uc81c \uc804\ubcf5\uc744 \uae30\ub3c4\ud55c \uc0ac\ud0dc\"\ub85c \uc65c\uace1\ud55c \uc2e0\uad70\ubd80\uc758 \uc8fc\uc7a5\uc5d0 \uadfc\uac70\ud55c \ud638\uce6d\uc73c\ub85c \uc81c5\uacf5\ud654\uad6d \uae30\uac04 \ub0b4\ub0b4 \uc0ac\uc6a9\ub410\uc73c\uba70, \ud604\uc7ac\ub294 \ub2f9\uc2dc \ud638\uce6d\uc5d0 \uc775\uc219\ud55c \ub178\ub144\uce35\uc774\ub098 \uc2e0\uad70\ubd80\ub97c \uc9c0\uc9c0\ud558\ub294 \uc77c\ubd80 \uc6b0\ud30c \uc778\uc0ac\ub4e4\uc774 \uc0ac\uc6a9\ud558\uace0 \uc788\ub2e4. \ud604\uc7ac\uc758 \uba85\uce6d\uc778 '5\u00b718 \uad11\uc8fc \ubbfc\uc8fc\ud654 \uc6b4\ub3d9'\uc740 1988\ub144 \uc774\ud6c4 \uc815\ubd80 \uc0b0\ud558 \ubbfc\uc8fc\ud654\ud569\ucd94\uc9c4\uc704\uc6d0\ud68c\uac00 \uc0ac\uac74\uc744 \ubbfc\uc8fc\ud654 \uc6b4\ub3d9\uc73c\ub85c \uaddc\uc815\ud558\uba74\uc11c \ub098\uc654\uace0, \uc774\ud6c4 \ubb38\ubbfc\uc815\ubd80, \uad6d\ubbfc\uc758 \uc815\ubd80\uc758 \uacf5\uc2dd \uc5b8\uae09\uc5d0\uc11c\ub3c4 \uc774 \uba85\uce6d\uc774 \uc0ac\uc6a9\ub428\uc73c\ub85c\uc368 \uacf5\uc2dd \uba85\uce6d\ud654\ub418\uc5c8\ub2e4. \ud55c\ud3b8, \uc2e0\uad70\ubd80\uc758 \uad70\uc0ac\ub3c5\uc7ac\uc640 \ud3ed\ub825\uc5d0 \ub9de\uc120 \ubbfc\uc911\ud56d\uc7c1\uc744 \uad11\uc8fc \ubbfc\uc911\ub4e4\uc774 \uc8fc\ub3c4\ud588\ub2e4\ub294 \uc0ac\uc2e4\uc744 \uac15\uc870\ud55c '\uad11\uc8fc\ubbfc\uc911\ud56d\uc7c1' \ub610\ub294 '\uad11\uc8fc\ud56d\uc7c1'\uc774\ub780 \uba85\uce6d\ub3c4 \uc9c0\uc5ed \uc0ac\ud68c\uc640 5\uc6d4 \ub2e8\uccb4 \ub4f1\uc744 \uc911\uc2ec\uc73c\ub85c 1980\ub144\ub300\ubd80\ud130 \uc0ac\uc6a9\ub410\ub2e4. \uc0ac\uac74\uc774 \uc77c\uc5b4\ub09c \ub0a0\uc9dc\ub97c \ub534 '5\u00b718'\ub3c4 \ub110\ub9ac \ud1b5\uc6a9\ub418\ub294 \uba85\uce6d\uc774\ub2e4."} {"question": "9\ub144\uc5d0 \ucf00\ub8e8\uc2a4\ud0a4 \uc871 \ucd9c\uc2e0\uc758 \uc544\ub974\ubbf8\ub2c8\uc6b0\uc2a4\uac00 \uc774\ub044\ub294 \uac8c\ub974\ub9cc \uc871\uc5d0\uac8c \uc804\uba78\ub2f9\ud55c \uac8c\ub974\ub9c8\ub2c8\uc544\uc758 \ucd1d\ub3c5\uc740?", "paragraph": "\ud30c\ub974\ud2f0\uc544\uac00 \uc5b8\uc81c\ub098 \uc704\ud611\uc801\uc778 \uc0c1\ub300\uc774\uae30\ub294 \ud588\uc9c0\ub9cc, \uc2e4\uc81c \uc804\uc7c1\uc740 \uac8c\ub974\ub9cc \uc871\uc744 \uc0c1\ub300\ub85c \ub300\ubd80\ubd84 \ub77c\uc778 \uac15, \ub3c4\ub098\uc6b0 \uac15 \uadfc\uad50\uc5d0\uc11c \ubc8c\uc5b4\uc84c\ub2e4. \uc548\ud1a0\ub2c8\uc6b0\uc2a4\uc640 \ucd5c\uc885 \uc804\ud22c\ub97c \ubc8c\uc774\uae30 \uc804\uc5d0 \ub2ec\ub9c8\ud2f0\uc544\uc758 \ubd80\uc871\ub4e4\uacfc \ubc8c\uc600\ub358 \uc804\uc7c1 \uc774\ud6c4 \ub85c\ub9c8\uad70\uc740 \ucc29\uc2e4\ud788 \ub3c4\ub098\uc6b0 \uac15\uc744 \ud5a5\ud574 \uc9c4\uaca9\ud558\uae30 \uc2dc\uc791\ud588\ub2e4. \ud558\uc9c0\ub9cc \uc5ec\ub7ec \ucc28\ub840 \uc804\uc7c1\uc5d0\uc11c \uc2b9\ub9ac\ud588\uc74c\uc5d0\ub3c4 \ubd88\uad6c\ud558\uace0 \uac8c\ub974\ub9c8\ub2c8\uc544 \uc9c0\uc5ed\uc740 \ub85c\ub9c8\ud654\ud558\ub294 \ub370 \uc2e4\ud328\ud558\uc600\ub2e4. 9\ub144\uc5d0 \ud1a0\uc774\ud1a0\ubd80\ub974\ud06c \uc232\uc5d0\uc11c \ub2f9\ud55c \ucc38\ud328\uac00 \uc774\ub97c \uc0c1\uc9d5\uc801\uc73c\ub85c \ubcf4\uc5ec\uc8fc\ub294 \uc0ac\uac74\uc774\ub2e4. \uac8c\ub974\ub9c8\ub2c8\uc544\uc758 \ucd1d\ub3c5\uc778 \ubc14\ub8e8\uc2a4\uac00 \uc774\ub044\ub294 3\uac1c \uad70\ub2e8\uc774 \ucf00\ub8e8\uc2a4\ud0a4 \uc871 \ucd9c\uc2e0\uc758 \uc544\ub974\ubbf8\ub2c8\uc6b0\uc2a4\uac00 \uc774\ub044\ub294 \uac8c\ub974\ub9cc \uc871\uc5d0\uac8c \uc804\uba78\ud55c\ub2e4. \uc544\uc6b0\uad6c\uc2a4\ud22c\uc2a4\ub294 \uc0ac\ud0dc\ub97c \uc218\uc2b5\ud558\ub824 \ud588\uace0, \ud2f0\ubca0\ub9ac\uc6b0\uc2a4\ub294 \uc774\ud6c4 \uc5ec\ub7ec \ucc28\ub840 \ub77c\uc778\ub780\ud2b8\ub85c \uc9c4\uaca9\ud558\uc5ec \uc2b9\ub9ac\ub97c \uac70\ub450\uc5c8\ub2e4. \ud558\uc9c0\ub9cc \uc544\uc6b0\uad6c\uc2a4\ud22c\uc2a4\uc758 \uc8fd\uc740 \ud6c4\uc5d0 \uacc4\uc2b9\uc790\uc778 \ud2f0\ubca0\ub9ac\uc6b0\uc2a4\ub294 \uac8c\ub974\ub9c8\ub2c8\uc544\uc5d0\uc11c \ucca0\uc218\ud558\uae30\ub85c \uacb0\uc815\ud558\uc600\uace0, \uc774\ud6c4 \ub85c\ub9c8\uad70\uc740 \ub77c\uc778 \uac15\uacfc \ub3c4\ub098\uc6b0 \uac15\uc744 \uc8fc \ubc29\uc5b4\uc120\uc73c\ub85c \uc0bc\uc544 \ubc29\uc704 \uccb4\uacc4\ub97c \uad6c\ucd95\ud55c\ub2e4.", "answer": "\ubc14\ub8e8\uc2a4", "sentence": "\uac8c\ub974\ub9c8\ub2c8\uc544\uc758 \ucd1d\ub3c5\uc778 \ubc14\ub8e8\uc2a4 \uac00 \uc774\ub044\ub294 3\uac1c \uad70\ub2e8\uc774 \ucf00\ub8e8\uc2a4\ud0a4 \uc871 \ucd9c\uc2e0\uc758 \uc544\ub974\ubbf8\ub2c8\uc6b0\uc2a4\uac00 \uc774\ub044\ub294 \uac8c\ub974\ub9cc \uc871\uc5d0\uac8c \uc804\uba78\ud55c\ub2e4.", "paragraph_sentence": "\ud30c\ub974\ud2f0\uc544\uac00 \uc5b8\uc81c\ub098 \uc704\ud611\uc801\uc778 \uc0c1\ub300\uc774\uae30\ub294 \ud588\uc9c0\ub9cc, \uc2e4\uc81c \uc804\uc7c1\uc740 \uac8c\ub974\ub9cc \uc871\uc744 \uc0c1\ub300\ub85c \ub300\ubd80\ubd84 \ub77c\uc778 \uac15, \ub3c4\ub098\uc6b0 \uac15 \uadfc\uad50\uc5d0\uc11c \ubc8c\uc5b4\uc84c\ub2e4. \uc548\ud1a0\ub2c8\uc6b0\uc2a4\uc640 \ucd5c\uc885 \uc804\ud22c\ub97c \ubc8c\uc774\uae30 \uc804\uc5d0 \ub2ec\ub9c8\ud2f0\uc544\uc758 \ubd80\uc871\ub4e4\uacfc \ubc8c\uc600\ub358 \uc804\uc7c1 \uc774\ud6c4 \ub85c\ub9c8\uad70\uc740 \ucc29\uc2e4\ud788 \ub3c4\ub098\uc6b0 \uac15\uc744 \ud5a5\ud574 \uc9c4\uaca9\ud558\uae30 \uc2dc\uc791\ud588\ub2e4. \ud558\uc9c0\ub9cc \uc5ec\ub7ec \ucc28\ub840 \uc804\uc7c1\uc5d0\uc11c \uc2b9\ub9ac\ud588\uc74c\uc5d0\ub3c4 \ubd88\uad6c\ud558\uace0 \uac8c\ub974\ub9c8\ub2c8\uc544 \uc9c0\uc5ed\uc740 \ub85c\ub9c8\ud654\ud558\ub294 \ub370 \uc2e4\ud328\ud558\uc600\ub2e4. 9\ub144\uc5d0 \ud1a0\uc774\ud1a0\ubd80\ub974\ud06c \uc232\uc5d0\uc11c \ub2f9\ud55c \ucc38\ud328\uac00 \uc774\ub97c \uc0c1\uc9d5\uc801\uc73c\ub85c \ubcf4\uc5ec\uc8fc\ub294 \uc0ac\uac74\uc774\ub2e4. \uac8c\ub974\ub9c8\ub2c8\uc544\uc758 \ucd1d\ub3c5\uc778 \ubc14\ub8e8\uc2a4 \uac00 \uc774\ub044\ub294 3\uac1c \uad70\ub2e8\uc774 \ucf00\ub8e8\uc2a4\ud0a4 \uc871 \ucd9c\uc2e0\uc758 \uc544\ub974\ubbf8\ub2c8\uc6b0\uc2a4\uac00 \uc774\ub044\ub294 \uac8c\ub974\ub9cc \uc871\uc5d0\uac8c \uc804\uba78\ud55c\ub2e4. \uc544\uc6b0\uad6c\uc2a4\ud22c\uc2a4\ub294 \uc0ac\ud0dc\ub97c \uc218\uc2b5\ud558\ub824 \ud588\uace0, \ud2f0\ubca0\ub9ac\uc6b0\uc2a4\ub294 \uc774\ud6c4 \uc5ec\ub7ec \ucc28\ub840 \ub77c\uc778\ub780\ud2b8\ub85c \uc9c4\uaca9\ud558\uc5ec \uc2b9\ub9ac\ub97c \uac70\ub450\uc5c8\ub2e4. \ud558\uc9c0\ub9cc \uc544\uc6b0\uad6c\uc2a4\ud22c\uc2a4\uc758 \uc8fd\uc740 \ud6c4\uc5d0 \uacc4\uc2b9\uc790\uc778 \ud2f0\ubca0\ub9ac\uc6b0\uc2a4\ub294 \uac8c\ub974\ub9c8\ub2c8\uc544\uc5d0\uc11c \ucca0\uc218\ud558\uae30\ub85c \uacb0\uc815\ud558\uc600\uace0, \uc774\ud6c4 \ub85c\ub9c8\uad70\uc740 \ub77c\uc778 \uac15\uacfc \ub3c4\ub098\uc6b0 \uac15\uc744 \uc8fc \ubc29\uc5b4\uc120\uc73c\ub85c \uc0bc\uc544 \ubc29\uc704 \uccb4\uacc4\ub97c \uad6c\ucd95\ud55c\ub2e4.", "paragraph_answer": "\ud30c\ub974\ud2f0\uc544\uac00 \uc5b8\uc81c\ub098 \uc704\ud611\uc801\uc778 \uc0c1\ub300\uc774\uae30\ub294 \ud588\uc9c0\ub9cc, \uc2e4\uc81c \uc804\uc7c1\uc740 \uac8c\ub974\ub9cc \uc871\uc744 \uc0c1\ub300\ub85c \ub300\ubd80\ubd84 \ub77c\uc778 \uac15, \ub3c4\ub098\uc6b0 \uac15 \uadfc\uad50\uc5d0\uc11c \ubc8c\uc5b4\uc84c\ub2e4. \uc548\ud1a0\ub2c8\uc6b0\uc2a4\uc640 \ucd5c\uc885 \uc804\ud22c\ub97c \ubc8c\uc774\uae30 \uc804\uc5d0 \ub2ec\ub9c8\ud2f0\uc544\uc758 \ubd80\uc871\ub4e4\uacfc \ubc8c\uc600\ub358 \uc804\uc7c1 \uc774\ud6c4 \ub85c\ub9c8\uad70\uc740 \ucc29\uc2e4\ud788 \ub3c4\ub098\uc6b0 \uac15\uc744 \ud5a5\ud574 \uc9c4\uaca9\ud558\uae30 \uc2dc\uc791\ud588\ub2e4. \ud558\uc9c0\ub9cc \uc5ec\ub7ec \ucc28\ub840 \uc804\uc7c1\uc5d0\uc11c \uc2b9\ub9ac\ud588\uc74c\uc5d0\ub3c4 \ubd88\uad6c\ud558\uace0 \uac8c\ub974\ub9c8\ub2c8\uc544 \uc9c0\uc5ed\uc740 \ub85c\ub9c8\ud654\ud558\ub294 \ub370 \uc2e4\ud328\ud558\uc600\ub2e4. 9\ub144\uc5d0 \ud1a0\uc774\ud1a0\ubd80\ub974\ud06c \uc232\uc5d0\uc11c \ub2f9\ud55c \ucc38\ud328\uac00 \uc774\ub97c \uc0c1\uc9d5\uc801\uc73c\ub85c \ubcf4\uc5ec\uc8fc\ub294 \uc0ac\uac74\uc774\ub2e4. \uac8c\ub974\ub9c8\ub2c8\uc544\uc758 \ucd1d\ub3c5\uc778 \ubc14\ub8e8\uc2a4 \uac00 \uc774\ub044\ub294 3\uac1c \uad70\ub2e8\uc774 \ucf00\ub8e8\uc2a4\ud0a4 \uc871 \ucd9c\uc2e0\uc758 \uc544\ub974\ubbf8\ub2c8\uc6b0\uc2a4\uac00 \uc774\ub044\ub294 \uac8c\ub974\ub9cc \uc871\uc5d0\uac8c \uc804\uba78\ud55c\ub2e4. \uc544\uc6b0\uad6c\uc2a4\ud22c\uc2a4\ub294 \uc0ac\ud0dc\ub97c \uc218\uc2b5\ud558\ub824 \ud588\uace0, \ud2f0\ubca0\ub9ac\uc6b0\uc2a4\ub294 \uc774\ud6c4 \uc5ec\ub7ec \ucc28\ub840 \ub77c\uc778\ub780\ud2b8\ub85c \uc9c4\uaca9\ud558\uc5ec \uc2b9\ub9ac\ub97c \uac70\ub450\uc5c8\ub2e4. \ud558\uc9c0\ub9cc \uc544\uc6b0\uad6c\uc2a4\ud22c\uc2a4\uc758 \uc8fd\uc740 \ud6c4\uc5d0 \uacc4\uc2b9\uc790\uc778 \ud2f0\ubca0\ub9ac\uc6b0\uc2a4\ub294 \uac8c\ub974\ub9c8\ub2c8\uc544\uc5d0\uc11c \ucca0\uc218\ud558\uae30\ub85c \uacb0\uc815\ud558\uc600\uace0, \uc774\ud6c4 \ub85c\ub9c8\uad70\uc740 \ub77c\uc778 \uac15\uacfc \ub3c4\ub098\uc6b0 \uac15\uc744 \uc8fc \ubc29\uc5b4\uc120\uc73c\ub85c \uc0bc\uc544 \ubc29\uc704 \uccb4\uacc4\ub97c \uad6c\ucd95\ud55c\ub2e4.", "sentence_answer": "\uac8c\ub974\ub9c8\ub2c8\uc544\uc758 \ucd1d\ub3c5\uc778 \ubc14\ub8e8\uc2a4 \uac00 \uc774\ub044\ub294 3\uac1c \uad70\ub2e8\uc774 \ucf00\ub8e8\uc2a4\ud0a4 \uc871 \ucd9c\uc2e0\uc758 \uc544\ub974\ubbf8\ub2c8\uc6b0\uc2a4\uac00 \uc774\ub044\ub294 \uac8c\ub974\ub9cc \uc871\uc5d0\uac8c \uc804\uba78\ud55c\ub2e4.", "paragraph_id": "6585254-28-1", "paragraph_question": "question: 9\ub144\uc5d0 \ucf00\ub8e8\uc2a4\ud0a4 \uc871 \ucd9c\uc2e0\uc758 \uc544\ub974\ubbf8\ub2c8\uc6b0\uc2a4\uac00 \uc774\ub044\ub294 \uac8c\ub974\ub9cc \uc871\uc5d0\uac8c \uc804\uba78\ub2f9\ud55c \uac8c\ub974\ub9c8\ub2c8\uc544\uc758 \ucd1d\ub3c5\uc740?, context: \ud30c\ub974\ud2f0\uc544\uac00 \uc5b8\uc81c\ub098 \uc704\ud611\uc801\uc778 \uc0c1\ub300\uc774\uae30\ub294 \ud588\uc9c0\ub9cc, \uc2e4\uc81c \uc804\uc7c1\uc740 \uac8c\ub974\ub9cc \uc871\uc744 \uc0c1\ub300\ub85c \ub300\ubd80\ubd84 \ub77c\uc778 \uac15, \ub3c4\ub098\uc6b0 \uac15 \uadfc\uad50\uc5d0\uc11c \ubc8c\uc5b4\uc84c\ub2e4. \uc548\ud1a0\ub2c8\uc6b0\uc2a4\uc640 \ucd5c\uc885 \uc804\ud22c\ub97c \ubc8c\uc774\uae30 \uc804\uc5d0 \ub2ec\ub9c8\ud2f0\uc544\uc758 \ubd80\uc871\ub4e4\uacfc \ubc8c\uc600\ub358 \uc804\uc7c1 \uc774\ud6c4 \ub85c\ub9c8\uad70\uc740 \ucc29\uc2e4\ud788 \ub3c4\ub098\uc6b0 \uac15\uc744 \ud5a5\ud574 \uc9c4\uaca9\ud558\uae30 \uc2dc\uc791\ud588\ub2e4. \ud558\uc9c0\ub9cc \uc5ec\ub7ec \ucc28\ub840 \uc804\uc7c1\uc5d0\uc11c \uc2b9\ub9ac\ud588\uc74c\uc5d0\ub3c4 \ubd88\uad6c\ud558\uace0 \uac8c\ub974\ub9c8\ub2c8\uc544 \uc9c0\uc5ed\uc740 \ub85c\ub9c8\ud654\ud558\ub294 \ub370 \uc2e4\ud328\ud558\uc600\ub2e4. 9\ub144\uc5d0 \ud1a0\uc774\ud1a0\ubd80\ub974\ud06c \uc232\uc5d0\uc11c \ub2f9\ud55c \ucc38\ud328\uac00 \uc774\ub97c \uc0c1\uc9d5\uc801\uc73c\ub85c \ubcf4\uc5ec\uc8fc\ub294 \uc0ac\uac74\uc774\ub2e4. \uac8c\ub974\ub9c8\ub2c8\uc544\uc758 \ucd1d\ub3c5\uc778 \ubc14\ub8e8\uc2a4\uac00 \uc774\ub044\ub294 3\uac1c \uad70\ub2e8\uc774 \ucf00\ub8e8\uc2a4\ud0a4 \uc871 \ucd9c\uc2e0\uc758 \uc544\ub974\ubbf8\ub2c8\uc6b0\uc2a4\uac00 \uc774\ub044\ub294 \uac8c\ub974\ub9cc \uc871\uc5d0\uac8c \uc804\uba78\ud55c\ub2e4. \uc544\uc6b0\uad6c\uc2a4\ud22c\uc2a4\ub294 \uc0ac\ud0dc\ub97c \uc218\uc2b5\ud558\ub824 \ud588\uace0, \ud2f0\ubca0\ub9ac\uc6b0\uc2a4\ub294 \uc774\ud6c4 \uc5ec\ub7ec \ucc28\ub840 \ub77c\uc778\ub780\ud2b8\ub85c \uc9c4\uaca9\ud558\uc5ec \uc2b9\ub9ac\ub97c \uac70\ub450\uc5c8\ub2e4. \ud558\uc9c0\ub9cc \uc544\uc6b0\uad6c\uc2a4\ud22c\uc2a4\uc758 \uc8fd\uc740 \ud6c4\uc5d0 \uacc4\uc2b9\uc790\uc778 \ud2f0\ubca0\ub9ac\uc6b0\uc2a4\ub294 \uac8c\ub974\ub9c8\ub2c8\uc544\uc5d0\uc11c \ucca0\uc218\ud558\uae30\ub85c \uacb0\uc815\ud558\uc600\uace0, \uc774\ud6c4 \ub85c\ub9c8\uad70\uc740 \ub77c\uc778 \uac15\uacfc \ub3c4\ub098\uc6b0 \uac15\uc744 \uc8fc \ubc29\uc5b4\uc120\uc73c\ub85c \uc0bc\uc544 \ubc29\uc704 \uccb4\uacc4\ub97c \uad6c\ucd95\ud55c\ub2e4."} {"question": "\uc81c1\uc2ec\uc758 \ud310\uacb0\uc5d0 \ubd88\ubcf5\ud558\uc5ec \ud56d\uc18c\ub97c \uc2e0\uccad\ud560 \uc218 \uc788\ub294 \uc18c\uc1a1\uc0c1\uc758 \uad8c\ub9ac\ub97c \ubb34\uc5c7\uc774\ub77c \ud558\ub294\uac00?", "paragraph": "\ud56d\uc18c\uad8c(\u6297\u8a34\u6b0a)\uc740 \ub2f9\uc0ac\uc790\uac00 \uc81c1\uc2ec\uc758 \ud310\uacb0\uc5d0 \ubd88\ubcf5\uc774\ub77c\uace0 \ud558\uc5ec \ud56d\uc18c\ub97c \uc2e0\uccad\ud560 \uc218 \uc788\ub294 \uc18c\uc1a1\uc0c1\uc758 \uad8c\ub9ac\uc774\ub2e4. \ud56d\uc18c\uad8c\uc740 \uc81c1\uc2ec \ud310\uacb0\uc5d0 \uc758\ud558\uc5ec \ud328\uc18c\ub418\uc5b4 \ubd88\ubcf5\uc758 \uc774\uc775\uc744 \uac00\uc9c4 \uc6d0\uace0 \ub610\ub294 \ud53c\uace0\uc5d0 \uc0dd\uae34\ub2e4. \ubd88\ubcf5\uc758 \uc774\uc775\uc740 \uc6d0\uace0\uc758 \uccad\uad6c\ub97c \uae30\uc900\uc73c\ub85c \ud558\uc5ec \ud615\uc2dd\uc801\uc73c\ub85c \uacb0\uc815\ub418\uc5b4 \uccad\uad6c\uc758 \uc804\ubd80 \ub610\ub294 \uc77c\ubd80\uac00 \ubd80\uc815(\uc804\ubd80 \ub610\ub294 \uc77c\ubd80\ud328\uc18c)\ub418\uba74 \uc6d0\uace0\ub294 \ubd88\ubcf5\uc758 \uc774\uc775\uc774 \uc788\ub294 \uac83\uc73c\ub85c \ub41c\ub2e4. \ud53c\uace0\uc758 \ubd88\ubcf5\uc758 \uc774\uc775\uc740 \uc6d0\uace0\uc5d0\uac8c \uc5ed\uc73c\ub85c \ub300\uc751\ud574\uc11c \uc0dd\uae34\ub2e4. \uc989 \uc6d0\uace0\uac00 \uc804\ubd80\ud328\uc18c\ub418\ub294 \uacbd\uc6b0 \ud53c\uace0\ub294 \uc804\ubd80\uc2b9\uc18c\uac00 \ub418\uc5b4 \ubd88\ubcf5\uc758 \uc774\uc775\uc740 \uc5c6\uc73c\ub098, \uc6d0\uace0\uac00 \uc77c\ubd80\uc2b9\uc18c\u00b7\uc77c\ubd80\ud328\uc18c\ud558\ub294 \uacbd\uc6b0\ub294 \ud53c\uace0\ub3c4 \uc77c\ubd80\ud328\uc18c\u00b7\uc77c\ubd80\uc2b9\uc18c\uac00 \ub418\uc5b4 \uc591\uc790 \ubaa8\ub450 \ubcfc\ubcf5\uc774 \uc788\ub2e4. \uc774\uc640 \uac19\uc774 \uc6d0\uace0\uac00 \uc804\ubd80\uc2b9\uc18c\ub97c \ud558\uac8c \ub418\uba74 \ud56d\uc18c\uc758 \uc774\uc775\uc774 \uc5c6\ub294 \uac83\uc774 \uc6d0\uce59\uc774\ub098 \uc608\uc678\ub85c\uc11c \uc81c1\uc2ec\uc5d0\uc11c \uc804\ubd80\uc2b9\uc18c\ud558\ub354\ub77c\ub3c4 \ubd88\ubcf5\uc774 \uc788\ub294 \uac83\uc73c\ub85c \ud558\uc5ec \ud56d\uc18c\uc758 \uc774\uc775\uc744 \uc778\uc815\ud558\uc9c0 \uc54a\uc73c\uba74 \uc544\ub2c8 \ub418\ub294 \uacbd\uc6b0\uac00 \uc788\ub2e4. \uc608\ub97c \ub4e4\uba74 \uac15\uc81c\uc9d1\ud589\uc744 \ubc1b\ub294 \ucc44\ubb34\uc790\uac00 \ucc44\ubb34\ubcc0\uc81c\uc758 \uc720\uc608\ub97c \uc774\uc720\ub85c \uccad\uad6c\uc774\uc758\uc758 \uc18c\ub97c \uc81c\uae30\ud558\uace0, \uc804\ubd80\uc2b9\uc18c\ud55c \uacbd\uc6b0 \uadf8 \ud310\uacb0\uc758 \uae30\ud310\ub825\uc758 \ud45c\uc900\uc2dc \uc804\uc758 \ubcc0\uc81c\uc5d0 \uc758\ud55c \ucc44\ubb34\uc18c\uba78\uc744 \uc774\uc720\ub85c \ud558\ub294 \ubcc4\ub3c4\uc758 \uccad\uad6c\uc774\uc758\uc758 \uc18c\ub97c \uc81c\uae30\ud568\uc744 \uae08\ud558\uace0 \uc788\uc73c\ubbc0\ub85c, \uc720\uc608\ub97c \uc774\uc720\ub85c \ud558\ub294 \uc774\uc758\uc18c\uc1a1\uc5d0\uc11c \uc804\ubd80\uc2b9\uc18c\ub97c \ud558\ub354\ub77c\ub3c4 \ud56d\uc18c\ub97c \ud558\ub294 \uac83\uc744 \uc778\uc815\ud558\uba70 \ud56d\uc18c\uc2ec\uc5d0\uc11c \uc18c\uc758 \ubcc0\uacbd\uc5d0 \uc758\ud558\uc5ec \ucc44\ubb34\uc18c\uba78\uc744 \uc8fc\uc7a5\ud558\uc9c0 \uc54a\uc73c\uba74 \uc544\ub2c8 \ub41c\ub2e4. \uc6d0\uace0\uac00 \uc608\ube44\uc801 \uccad\uad6c\ub97c \ud558\uc5ec \uc774\uac83\uc774 \uc778\uc815\ub418\uc5b4 \uc2b9\uc18c\ud558\ub354\ub77c\ub3c4 \uc8fc\uc704\uccad\uad6c(\u4e3b\u4f4d\u8acb\u6c42)\uc5d0\uc11c \ud328\uc18c\ud558\uace0 \uc788\uc73c\ubbc0\ub85c \ubd88\ubcf5\uc758 \uc774\uc775\uc774 \uc788\ub2e4. \ud53c\uace0\uac00 \uccad\uad6c\uae30\uac01\uc744 \uc2e0\uccad\ud558\uc600\ub294\ub370 \uc18c\uc758 \ubd80\uc801\ubc95 \uac01\ud558 \ud310\uacb0\uc774 \ub0b4\ub824\uc9c4 \uacbd\uc6b0\ub294 \uc77c\ubd80\uc2b9\uc18c\u00b7\uc77c\ubd80\ud328\uc18c\uc758 \uad00\uacc4\uc5d0 \uc900\ud558\uc5ec \ubd88\ubcf5\uc758 \uc774\uc775\uc774 \uc788\uc73c\ub098 \ubc18\ub300\uc758 \uacbd\uc6b0\ub294 \uc804\ubd80\uc2b9\uc18c\uac00 \ub418\uc5b4 \ud56d\uc18c\uc758 \uc774\uc775\uc740 \uc5c6\ub2e4. \uc804\ubd80\uc2b9\uc18c\uc758 \uacbd\uc6b0\ub294 \uc6d0\uace0\uac00 \uc18c\ub97c \ubcc0\uacbd\ud558\uae30 \uc704\ud558\uc5ec, \ub610\ub294 \ud53c\uace0\uac00 \ubc18\uc18c\ub97c \uc81c\uae30\ud558\uae30 \uc704\ud558\uc5ec \ud56d\uc18c\ud560 \uc218\uac00 \uc5c6\ub2e4. \ud56d\uc18c\uad8c\uc740 \ub2f9\uc0ac\uc790\uc758 \uc18c\uc1a1\uc0c1\uc758 \uad8c\ub9ac\uc774\uba70, \uad8c\ub9ac\uc790\uac00 \uc774\uac83\uc744 \ud3ec\uae30\ud560 \uc218\ub3c4 \uc788\uace0, \ub2f9\ucd08\ubd80\ud130 \ub2f9\uc0ac\uc790 \uc0c1\ud638\uac04\uc758 \ud569\uc758\uc5d0 \uc758\ud558\uc5ec \ud56d\uc18c\ub97c \ud558\uc9c0 \uc54a\uc744 \uac83\uc744 \uc57d\uc18d\ud560 \uc218\ub3c4 \uc788\ub2e4. \uc774\uac83\uc744 '\ubd88\ud56d\uc18c(\u4e0d\u6297\u8a34)\uc758 \ud569\uc758'\ub77c \ud55c\ub2e4. \uc774\uc640 \uac19\uc740 \ud569\uc758\uac00 \uc0c1\ud638\uac04\uc758 \uc790\uc720\uc758\uc0ac\uc5d0 \uae30\ud574\uc11c \uc774\ub8e8\uc5b4\uc9c0\uba74 \ud56d\uc18c\uad8c\uc740 \ub2f9\uc0ac\uc790 \uc30d\ubc29\uc5d0 \ubc1c\uc0dd\ud558\uc9c0 \uc54a\ub294\ub2e4. \ub2e4\ub9cc \uc774\ub7ec\ud55c \ud569\uc758\uac00 \uc774\ub8e8\uc5b4\uc9c0\ub294 \uac83\uc740 \ud1b5\uc0c1\uc758 \ubbfc\uc0ac\uc0ac\uac74\uc5d0 \uc788\uc5b4\uc11c\ubfd0\uc774\uba70, \uc9c1\uad8c\ud0d0\uc9c0\uc8fc\uc758\uac00 \uac00\ubbf8\ub418\ub294 \uc778\uc0ac\uc18c\uc1a1\uc5d0\uc11c\ub294 \ud560 \uc218\uac00 \uc5c6\ub2e4.", "answer": "\ud56d\uc18c\uad8c", "sentence": "\ud56d\uc18c\uad8c (\u6297\u8a34\u6b0a)\uc740 \ub2f9\uc0ac\uc790\uac00 \uc81c1\uc2ec\uc758 \ud310\uacb0\uc5d0 \ubd88\ubcf5\uc774\ub77c\uace0 \ud558\uc5ec \ud56d\uc18c\ub97c \uc2e0\uccad\ud560 \uc218 \uc788\ub294 \uc18c\uc1a1\uc0c1\uc758 \uad8c\ub9ac\uc774\ub2e4.", "paragraph_sentence": " \ud56d\uc18c\uad8c (\u6297\u8a34\u6b0a)\uc740 \ub2f9\uc0ac\uc790\uac00 \uc81c1\uc2ec\uc758 \ud310\uacb0\uc5d0 \ubd88\ubcf5\uc774\ub77c\uace0 \ud558\uc5ec \ud56d\uc18c\ub97c \uc2e0\uccad\ud560 \uc218 \uc788\ub294 \uc18c\uc1a1\uc0c1\uc758 \uad8c\ub9ac\uc774\ub2e4. \ud56d\uc18c\uad8c\uc740 \uc81c1\uc2ec \ud310\uacb0\uc5d0 \uc758\ud558\uc5ec \ud328\uc18c\ub418\uc5b4 \ubd88\ubcf5\uc758 \uc774\uc775\uc744 \uac00\uc9c4 \uc6d0\uace0 \ub610\ub294 \ud53c\uace0\uc5d0 \uc0dd\uae34\ub2e4. \ubd88\ubcf5\uc758 \uc774\uc775\uc740 \uc6d0\uace0\uc758 \uccad\uad6c\ub97c \uae30\uc900\uc73c\ub85c \ud558\uc5ec \ud615\uc2dd\uc801\uc73c\ub85c \uacb0\uc815\ub418\uc5b4 \uccad\uad6c\uc758 \uc804\ubd80 \ub610\ub294 \uc77c\ubd80\uac00 \ubd80\uc815(\uc804\ubd80 \ub610\ub294 \uc77c\ubd80\ud328\uc18c)\ub418\uba74 \uc6d0\uace0\ub294 \ubd88\ubcf5\uc758 \uc774\uc775\uc774 \uc788\ub294 \uac83\uc73c\ub85c \ub41c\ub2e4. \ud53c\uace0\uc758 \ubd88\ubcf5\uc758 \uc774\uc775\uc740 \uc6d0\uace0\uc5d0\uac8c \uc5ed\uc73c\ub85c \ub300\uc751\ud574\uc11c \uc0dd\uae34\ub2e4. \uc989 \uc6d0\uace0\uac00 \uc804\ubd80\ud328\uc18c\ub418\ub294 \uacbd\uc6b0 \ud53c\uace0\ub294 \uc804\ubd80\uc2b9\uc18c\uac00 \ub418\uc5b4 \ubd88\ubcf5\uc758 \uc774\uc775\uc740 \uc5c6\uc73c\ub098, \uc6d0\uace0\uac00 \uc77c\ubd80\uc2b9\uc18c\u00b7\uc77c\ubd80\ud328\uc18c\ud558\ub294 \uacbd\uc6b0\ub294 \ud53c\uace0\ub3c4 \uc77c\ubd80\ud328\uc18c\u00b7\uc77c\ubd80\uc2b9\uc18c\uac00 \ub418\uc5b4 \uc591\uc790 \ubaa8\ub450 \ubcfc\ubcf5\uc774 \uc788\ub2e4. \uc774\uc640 \uac19\uc774 \uc6d0\uace0\uac00 \uc804\ubd80\uc2b9\uc18c\ub97c \ud558\uac8c \ub418\uba74 \ud56d\uc18c\uc758 \uc774\uc775\uc774 \uc5c6\ub294 \uac83\uc774 \uc6d0\uce59\uc774\ub098 \uc608\uc678\ub85c\uc11c \uc81c1\uc2ec\uc5d0\uc11c \uc804\ubd80\uc2b9\uc18c\ud558\ub354\ub77c\ub3c4 \ubd88\ubcf5\uc774 \uc788\ub294 \uac83\uc73c\ub85c \ud558\uc5ec \ud56d\uc18c\uc758 \uc774\uc775\uc744 \uc778\uc815\ud558\uc9c0 \uc54a\uc73c\uba74 \uc544\ub2c8 \ub418\ub294 \uacbd\uc6b0\uac00 \uc788\ub2e4. \uc608\ub97c \ub4e4\uba74 \uac15\uc81c\uc9d1\ud589\uc744 \ubc1b\ub294 \ucc44\ubb34\uc790\uac00 \ucc44\ubb34\ubcc0\uc81c\uc758 \uc720\uc608\ub97c \uc774\uc720\ub85c \uccad\uad6c\uc774\uc758\uc758 \uc18c\ub97c \uc81c\uae30\ud558\uace0, \uc804\ubd80\uc2b9\uc18c\ud55c \uacbd\uc6b0 \uadf8 \ud310\uacb0\uc758 \uae30\ud310\ub825\uc758 \ud45c\uc900\uc2dc \uc804\uc758 \ubcc0\uc81c\uc5d0 \uc758\ud55c \ucc44\ubb34\uc18c\uba78\uc744 \uc774\uc720\ub85c \ud558\ub294 \ubcc4\ub3c4\uc758 \uccad\uad6c\uc774\uc758\uc758 \uc18c\ub97c \uc81c\uae30\ud568\uc744 \uae08\ud558\uace0 \uc788\uc73c\ubbc0\ub85c, \uc720\uc608\ub97c \uc774\uc720\ub85c \ud558\ub294 \uc774\uc758\uc18c\uc1a1\uc5d0\uc11c \uc804\ubd80\uc2b9\uc18c\ub97c \ud558\ub354\ub77c\ub3c4 \ud56d\uc18c\ub97c \ud558\ub294 \uac83\uc744 \uc778\uc815\ud558\uba70 \ud56d\uc18c\uc2ec\uc5d0\uc11c \uc18c\uc758 \ubcc0\uacbd\uc5d0 \uc758\ud558\uc5ec \ucc44\ubb34\uc18c\uba78\uc744 \uc8fc\uc7a5\ud558\uc9c0 \uc54a\uc73c\uba74 \uc544\ub2c8 \ub41c\ub2e4. \uc6d0\uace0\uac00 \uc608\ube44\uc801 \uccad\uad6c\ub97c \ud558\uc5ec \uc774\uac83\uc774 \uc778\uc815\ub418\uc5b4 \uc2b9\uc18c\ud558\ub354\ub77c\ub3c4 \uc8fc\uc704\uccad\uad6c(\u4e3b\u4f4d\u8acb\u6c42)\uc5d0\uc11c \ud328\uc18c\ud558\uace0 \uc788\uc73c\ubbc0\ub85c \ubd88\ubcf5\uc758 \uc774\uc775\uc774 \uc788\ub2e4. \ud53c\uace0\uac00 \uccad\uad6c\uae30\uac01\uc744 \uc2e0\uccad\ud558\uc600\ub294\ub370 \uc18c\uc758 \ubd80\uc801\ubc95 \uac01\ud558 \ud310\uacb0\uc774 \ub0b4\ub824\uc9c4 \uacbd\uc6b0\ub294 \uc77c\ubd80\uc2b9\uc18c\u00b7\uc77c\ubd80\ud328\uc18c\uc758 \uad00\uacc4\uc5d0 \uc900\ud558\uc5ec \ubd88\ubcf5\uc758 \uc774\uc775\uc774 \uc788\uc73c\ub098 \ubc18\ub300\uc758 \uacbd\uc6b0\ub294 \uc804\ubd80\uc2b9\uc18c\uac00 \ub418\uc5b4 \ud56d\uc18c\uc758 \uc774\uc775\uc740 \uc5c6\ub2e4. \uc804\ubd80\uc2b9\uc18c\uc758 \uacbd\uc6b0\ub294 \uc6d0\uace0\uac00 \uc18c\ub97c \ubcc0\uacbd\ud558\uae30 \uc704\ud558\uc5ec, \ub610\ub294 \ud53c\uace0\uac00 \ubc18\uc18c\ub97c \uc81c\uae30\ud558\uae30 \uc704\ud558\uc5ec \ud56d\uc18c\ud560 \uc218\uac00 \uc5c6\ub2e4. \ud56d\uc18c\uad8c\uc740 \ub2f9\uc0ac\uc790\uc758 \uc18c\uc1a1\uc0c1\uc758 \uad8c\ub9ac\uc774\uba70, \uad8c\ub9ac\uc790\uac00 \uc774\uac83\uc744 \ud3ec\uae30\ud560 \uc218\ub3c4 \uc788\uace0, \ub2f9\ucd08\ubd80\ud130 \ub2f9\uc0ac\uc790 \uc0c1\ud638\uac04\uc758 \ud569\uc758\uc5d0 \uc758\ud558\uc5ec \ud56d\uc18c\ub97c \ud558\uc9c0 \uc54a\uc744 \uac83\uc744 \uc57d\uc18d\ud560 \uc218\ub3c4 \uc788\ub2e4. \uc774\uac83\uc744 '\ubd88\ud56d\uc18c(\u4e0d\u6297\u8a34)\uc758 \ud569\uc758'\ub77c \ud55c\ub2e4. \uc774\uc640 \uac19\uc740 \ud569\uc758\uac00 \uc0c1\ud638\uac04\uc758 \uc790\uc720\uc758\uc0ac\uc5d0 \uae30\ud574\uc11c \uc774\ub8e8\uc5b4\uc9c0\uba74 \ud56d\uc18c\uad8c\uc740 \ub2f9\uc0ac\uc790 \uc30d\ubc29\uc5d0 \ubc1c\uc0dd\ud558\uc9c0 \uc54a\ub294\ub2e4. \ub2e4\ub9cc \uc774\ub7ec\ud55c \ud569\uc758\uac00 \uc774\ub8e8\uc5b4\uc9c0\ub294 \uac83\uc740 \ud1b5\uc0c1\uc758 \ubbfc\uc0ac\uc0ac\uac74\uc5d0 \uc788\uc5b4\uc11c\ubfd0\uc774\uba70, \uc9c1\uad8c\ud0d0\uc9c0\uc8fc\uc758\uac00 \uac00\ubbf8\ub418\ub294 \uc778\uc0ac\uc18c\uc1a1\uc5d0\uc11c\ub294 \ud560 \uc218\uac00 \uc5c6\ub2e4.", "paragraph_answer": " \ud56d\uc18c\uad8c (\u6297\u8a34\u6b0a)\uc740 \ub2f9\uc0ac\uc790\uac00 \uc81c1\uc2ec\uc758 \ud310\uacb0\uc5d0 \ubd88\ubcf5\uc774\ub77c\uace0 \ud558\uc5ec \ud56d\uc18c\ub97c \uc2e0\uccad\ud560 \uc218 \uc788\ub294 \uc18c\uc1a1\uc0c1\uc758 \uad8c\ub9ac\uc774\ub2e4. \ud56d\uc18c\uad8c\uc740 \uc81c1\uc2ec \ud310\uacb0\uc5d0 \uc758\ud558\uc5ec \ud328\uc18c\ub418\uc5b4 \ubd88\ubcf5\uc758 \uc774\uc775\uc744 \uac00\uc9c4 \uc6d0\uace0 \ub610\ub294 \ud53c\uace0\uc5d0 \uc0dd\uae34\ub2e4. \ubd88\ubcf5\uc758 \uc774\uc775\uc740 \uc6d0\uace0\uc758 \uccad\uad6c\ub97c \uae30\uc900\uc73c\ub85c \ud558\uc5ec \ud615\uc2dd\uc801\uc73c\ub85c \uacb0\uc815\ub418\uc5b4 \uccad\uad6c\uc758 \uc804\ubd80 \ub610\ub294 \uc77c\ubd80\uac00 \ubd80\uc815(\uc804\ubd80 \ub610\ub294 \uc77c\ubd80\ud328\uc18c)\ub418\uba74 \uc6d0\uace0\ub294 \ubd88\ubcf5\uc758 \uc774\uc775\uc774 \uc788\ub294 \uac83\uc73c\ub85c \ub41c\ub2e4. \ud53c\uace0\uc758 \ubd88\ubcf5\uc758 \uc774\uc775\uc740 \uc6d0\uace0\uc5d0\uac8c \uc5ed\uc73c\ub85c \ub300\uc751\ud574\uc11c \uc0dd\uae34\ub2e4. \uc989 \uc6d0\uace0\uac00 \uc804\ubd80\ud328\uc18c\ub418\ub294 \uacbd\uc6b0 \ud53c\uace0\ub294 \uc804\ubd80\uc2b9\uc18c\uac00 \ub418\uc5b4 \ubd88\ubcf5\uc758 \uc774\uc775\uc740 \uc5c6\uc73c\ub098, \uc6d0\uace0\uac00 \uc77c\ubd80\uc2b9\uc18c\u00b7\uc77c\ubd80\ud328\uc18c\ud558\ub294 \uacbd\uc6b0\ub294 \ud53c\uace0\ub3c4 \uc77c\ubd80\ud328\uc18c\u00b7\uc77c\ubd80\uc2b9\uc18c\uac00 \ub418\uc5b4 \uc591\uc790 \ubaa8\ub450 \ubcfc\ubcf5\uc774 \uc788\ub2e4. \uc774\uc640 \uac19\uc774 \uc6d0\uace0\uac00 \uc804\ubd80\uc2b9\uc18c\ub97c \ud558\uac8c \ub418\uba74 \ud56d\uc18c\uc758 \uc774\uc775\uc774 \uc5c6\ub294 \uac83\uc774 \uc6d0\uce59\uc774\ub098 \uc608\uc678\ub85c\uc11c \uc81c1\uc2ec\uc5d0\uc11c \uc804\ubd80\uc2b9\uc18c\ud558\ub354\ub77c\ub3c4 \ubd88\ubcf5\uc774 \uc788\ub294 \uac83\uc73c\ub85c \ud558\uc5ec \ud56d\uc18c\uc758 \uc774\uc775\uc744 \uc778\uc815\ud558\uc9c0 \uc54a\uc73c\uba74 \uc544\ub2c8 \ub418\ub294 \uacbd\uc6b0\uac00 \uc788\ub2e4. \uc608\ub97c \ub4e4\uba74 \uac15\uc81c\uc9d1\ud589\uc744 \ubc1b\ub294 \ucc44\ubb34\uc790\uac00 \ucc44\ubb34\ubcc0\uc81c\uc758 \uc720\uc608\ub97c \uc774\uc720\ub85c \uccad\uad6c\uc774\uc758\uc758 \uc18c\ub97c \uc81c\uae30\ud558\uace0, \uc804\ubd80\uc2b9\uc18c\ud55c \uacbd\uc6b0 \uadf8 \ud310\uacb0\uc758 \uae30\ud310\ub825\uc758 \ud45c\uc900\uc2dc \uc804\uc758 \ubcc0\uc81c\uc5d0 \uc758\ud55c \ucc44\ubb34\uc18c\uba78\uc744 \uc774\uc720\ub85c \ud558\ub294 \ubcc4\ub3c4\uc758 \uccad\uad6c\uc774\uc758\uc758 \uc18c\ub97c \uc81c\uae30\ud568\uc744 \uae08\ud558\uace0 \uc788\uc73c\ubbc0\ub85c, \uc720\uc608\ub97c \uc774\uc720\ub85c \ud558\ub294 \uc774\uc758\uc18c\uc1a1\uc5d0\uc11c \uc804\ubd80\uc2b9\uc18c\ub97c \ud558\ub354\ub77c\ub3c4 \ud56d\uc18c\ub97c \ud558\ub294 \uac83\uc744 \uc778\uc815\ud558\uba70 \ud56d\uc18c\uc2ec\uc5d0\uc11c \uc18c\uc758 \ubcc0\uacbd\uc5d0 \uc758\ud558\uc5ec \ucc44\ubb34\uc18c\uba78\uc744 \uc8fc\uc7a5\ud558\uc9c0 \uc54a\uc73c\uba74 \uc544\ub2c8 \ub41c\ub2e4. \uc6d0\uace0\uac00 \uc608\ube44\uc801 \uccad\uad6c\ub97c \ud558\uc5ec \uc774\uac83\uc774 \uc778\uc815\ub418\uc5b4 \uc2b9\uc18c\ud558\ub354\ub77c\ub3c4 \uc8fc\uc704\uccad\uad6c(\u4e3b\u4f4d\u8acb\u6c42)\uc5d0\uc11c \ud328\uc18c\ud558\uace0 \uc788\uc73c\ubbc0\ub85c \ubd88\ubcf5\uc758 \uc774\uc775\uc774 \uc788\ub2e4. \ud53c\uace0\uac00 \uccad\uad6c\uae30\uac01\uc744 \uc2e0\uccad\ud558\uc600\ub294\ub370 \uc18c\uc758 \ubd80\uc801\ubc95 \uac01\ud558 \ud310\uacb0\uc774 \ub0b4\ub824\uc9c4 \uacbd\uc6b0\ub294 \uc77c\ubd80\uc2b9\uc18c\u00b7\uc77c\ubd80\ud328\uc18c\uc758 \uad00\uacc4\uc5d0 \uc900\ud558\uc5ec \ubd88\ubcf5\uc758 \uc774\uc775\uc774 \uc788\uc73c\ub098 \ubc18\ub300\uc758 \uacbd\uc6b0\ub294 \uc804\ubd80\uc2b9\uc18c\uac00 \ub418\uc5b4 \ud56d\uc18c\uc758 \uc774\uc775\uc740 \uc5c6\ub2e4. \uc804\ubd80\uc2b9\uc18c\uc758 \uacbd\uc6b0\ub294 \uc6d0\uace0\uac00 \uc18c\ub97c \ubcc0\uacbd\ud558\uae30 \uc704\ud558\uc5ec, \ub610\ub294 \ud53c\uace0\uac00 \ubc18\uc18c\ub97c \uc81c\uae30\ud558\uae30 \uc704\ud558\uc5ec \ud56d\uc18c\ud560 \uc218\uac00 \uc5c6\ub2e4. \ud56d\uc18c\uad8c\uc740 \ub2f9\uc0ac\uc790\uc758 \uc18c\uc1a1\uc0c1\uc758 \uad8c\ub9ac\uc774\uba70, \uad8c\ub9ac\uc790\uac00 \uc774\uac83\uc744 \ud3ec\uae30\ud560 \uc218\ub3c4 \uc788\uace0, \ub2f9\ucd08\ubd80\ud130 \ub2f9\uc0ac\uc790 \uc0c1\ud638\uac04\uc758 \ud569\uc758\uc5d0 \uc758\ud558\uc5ec \ud56d\uc18c\ub97c \ud558\uc9c0 \uc54a\uc744 \uac83\uc744 \uc57d\uc18d\ud560 \uc218\ub3c4 \uc788\ub2e4. \uc774\uac83\uc744 '\ubd88\ud56d\uc18c(\u4e0d\u6297\u8a34)\uc758 \ud569\uc758'\ub77c \ud55c\ub2e4. \uc774\uc640 \uac19\uc740 \ud569\uc758\uac00 \uc0c1\ud638\uac04\uc758 \uc790\uc720\uc758\uc0ac\uc5d0 \uae30\ud574\uc11c \uc774\ub8e8\uc5b4\uc9c0\uba74 \ud56d\uc18c\uad8c\uc740 \ub2f9\uc0ac\uc790 \uc30d\ubc29\uc5d0 \ubc1c\uc0dd\ud558\uc9c0 \uc54a\ub294\ub2e4. \ub2e4\ub9cc \uc774\ub7ec\ud55c \ud569\uc758\uac00 \uc774\ub8e8\uc5b4\uc9c0\ub294 \uac83\uc740 \ud1b5\uc0c1\uc758 \ubbfc\uc0ac\uc0ac\uac74\uc5d0 \uc788\uc5b4\uc11c\ubfd0\uc774\uba70, \uc9c1\uad8c\ud0d0\uc9c0\uc8fc\uc758\uac00 \uac00\ubbf8\ub418\ub294 \uc778\uc0ac\uc18c\uc1a1\uc5d0\uc11c\ub294 \ud560 \uc218\uac00 \uc5c6\ub2e4.", "sentence_answer": " \ud56d\uc18c\uad8c (\u6297\u8a34\u6b0a)\uc740 \ub2f9\uc0ac\uc790\uac00 \uc81c1\uc2ec\uc758 \ud310\uacb0\uc5d0 \ubd88\ubcf5\uc774\ub77c\uace0 \ud558\uc5ec \ud56d\uc18c\ub97c \uc2e0\uccad\ud560 \uc218 \uc788\ub294 \uc18c\uc1a1\uc0c1\uc758 \uad8c\ub9ac\uc774\ub2e4.", "paragraph_id": "6570281-1-0", "paragraph_question": "question: \uc81c1\uc2ec\uc758 \ud310\uacb0\uc5d0 \ubd88\ubcf5\ud558\uc5ec \ud56d\uc18c\ub97c \uc2e0\uccad\ud560 \uc218 \uc788\ub294 \uc18c\uc1a1\uc0c1\uc758 \uad8c\ub9ac\ub97c \ubb34\uc5c7\uc774\ub77c \ud558\ub294\uac00?, context: \ud56d\uc18c\uad8c(\u6297\u8a34\u6b0a)\uc740 \ub2f9\uc0ac\uc790\uac00 \uc81c1\uc2ec\uc758 \ud310\uacb0\uc5d0 \ubd88\ubcf5\uc774\ub77c\uace0 \ud558\uc5ec \ud56d\uc18c\ub97c \uc2e0\uccad\ud560 \uc218 \uc788\ub294 \uc18c\uc1a1\uc0c1\uc758 \uad8c\ub9ac\uc774\ub2e4. \ud56d\uc18c\uad8c\uc740 \uc81c1\uc2ec \ud310\uacb0\uc5d0 \uc758\ud558\uc5ec \ud328\uc18c\ub418\uc5b4 \ubd88\ubcf5\uc758 \uc774\uc775\uc744 \uac00\uc9c4 \uc6d0\uace0 \ub610\ub294 \ud53c\uace0\uc5d0 \uc0dd\uae34\ub2e4. \ubd88\ubcf5\uc758 \uc774\uc775\uc740 \uc6d0\uace0\uc758 \uccad\uad6c\ub97c \uae30\uc900\uc73c\ub85c \ud558\uc5ec \ud615\uc2dd\uc801\uc73c\ub85c \uacb0\uc815\ub418\uc5b4 \uccad\uad6c\uc758 \uc804\ubd80 \ub610\ub294 \uc77c\ubd80\uac00 \ubd80\uc815(\uc804\ubd80 \ub610\ub294 \uc77c\ubd80\ud328\uc18c)\ub418\uba74 \uc6d0\uace0\ub294 \ubd88\ubcf5\uc758 \uc774\uc775\uc774 \uc788\ub294 \uac83\uc73c\ub85c \ub41c\ub2e4. \ud53c\uace0\uc758 \ubd88\ubcf5\uc758 \uc774\uc775\uc740 \uc6d0\uace0\uc5d0\uac8c \uc5ed\uc73c\ub85c \ub300\uc751\ud574\uc11c \uc0dd\uae34\ub2e4. \uc989 \uc6d0\uace0\uac00 \uc804\ubd80\ud328\uc18c\ub418\ub294 \uacbd\uc6b0 \ud53c\uace0\ub294 \uc804\ubd80\uc2b9\uc18c\uac00 \ub418\uc5b4 \ubd88\ubcf5\uc758 \uc774\uc775\uc740 \uc5c6\uc73c\ub098, \uc6d0\uace0\uac00 \uc77c\ubd80\uc2b9\uc18c\u00b7\uc77c\ubd80\ud328\uc18c\ud558\ub294 \uacbd\uc6b0\ub294 \ud53c\uace0\ub3c4 \uc77c\ubd80\ud328\uc18c\u00b7\uc77c\ubd80\uc2b9\uc18c\uac00 \ub418\uc5b4 \uc591\uc790 \ubaa8\ub450 \ubcfc\ubcf5\uc774 \uc788\ub2e4. \uc774\uc640 \uac19\uc774 \uc6d0\uace0\uac00 \uc804\ubd80\uc2b9\uc18c\ub97c \ud558\uac8c \ub418\uba74 \ud56d\uc18c\uc758 \uc774\uc775\uc774 \uc5c6\ub294 \uac83\uc774 \uc6d0\uce59\uc774\ub098 \uc608\uc678\ub85c\uc11c \uc81c1\uc2ec\uc5d0\uc11c \uc804\ubd80\uc2b9\uc18c\ud558\ub354\ub77c\ub3c4 \ubd88\ubcf5\uc774 \uc788\ub294 \uac83\uc73c\ub85c \ud558\uc5ec \ud56d\uc18c\uc758 \uc774\uc775\uc744 \uc778\uc815\ud558\uc9c0 \uc54a\uc73c\uba74 \uc544\ub2c8 \ub418\ub294 \uacbd\uc6b0\uac00 \uc788\ub2e4. \uc608\ub97c \ub4e4\uba74 \uac15\uc81c\uc9d1\ud589\uc744 \ubc1b\ub294 \ucc44\ubb34\uc790\uac00 \ucc44\ubb34\ubcc0\uc81c\uc758 \uc720\uc608\ub97c \uc774\uc720\ub85c \uccad\uad6c\uc774\uc758\uc758 \uc18c\ub97c \uc81c\uae30\ud558\uace0, \uc804\ubd80\uc2b9\uc18c\ud55c \uacbd\uc6b0 \uadf8 \ud310\uacb0\uc758 \uae30\ud310\ub825\uc758 \ud45c\uc900\uc2dc \uc804\uc758 \ubcc0\uc81c\uc5d0 \uc758\ud55c \ucc44\ubb34\uc18c\uba78\uc744 \uc774\uc720\ub85c \ud558\ub294 \ubcc4\ub3c4\uc758 \uccad\uad6c\uc774\uc758\uc758 \uc18c\ub97c \uc81c\uae30\ud568\uc744 \uae08\ud558\uace0 \uc788\uc73c\ubbc0\ub85c, \uc720\uc608\ub97c \uc774\uc720\ub85c \ud558\ub294 \uc774\uc758\uc18c\uc1a1\uc5d0\uc11c \uc804\ubd80\uc2b9\uc18c\ub97c \ud558\ub354\ub77c\ub3c4 \ud56d\uc18c\ub97c \ud558\ub294 \uac83\uc744 \uc778\uc815\ud558\uba70 \ud56d\uc18c\uc2ec\uc5d0\uc11c \uc18c\uc758 \ubcc0\uacbd\uc5d0 \uc758\ud558\uc5ec \ucc44\ubb34\uc18c\uba78\uc744 \uc8fc\uc7a5\ud558\uc9c0 \uc54a\uc73c\uba74 \uc544\ub2c8 \ub41c\ub2e4. \uc6d0\uace0\uac00 \uc608\ube44\uc801 \uccad\uad6c\ub97c \ud558\uc5ec \uc774\uac83\uc774 \uc778\uc815\ub418\uc5b4 \uc2b9\uc18c\ud558\ub354\ub77c\ub3c4 \uc8fc\uc704\uccad\uad6c(\u4e3b\u4f4d\u8acb\u6c42)\uc5d0\uc11c \ud328\uc18c\ud558\uace0 \uc788\uc73c\ubbc0\ub85c \ubd88\ubcf5\uc758 \uc774\uc775\uc774 \uc788\ub2e4. \ud53c\uace0\uac00 \uccad\uad6c\uae30\uac01\uc744 \uc2e0\uccad\ud558\uc600\ub294\ub370 \uc18c\uc758 \ubd80\uc801\ubc95 \uac01\ud558 \ud310\uacb0\uc774 \ub0b4\ub824\uc9c4 \uacbd\uc6b0\ub294 \uc77c\ubd80\uc2b9\uc18c\u00b7\uc77c\ubd80\ud328\uc18c\uc758 \uad00\uacc4\uc5d0 \uc900\ud558\uc5ec \ubd88\ubcf5\uc758 \uc774\uc775\uc774 \uc788\uc73c\ub098 \ubc18\ub300\uc758 \uacbd\uc6b0\ub294 \uc804\ubd80\uc2b9\uc18c\uac00 \ub418\uc5b4 \ud56d\uc18c\uc758 \uc774\uc775\uc740 \uc5c6\ub2e4. \uc804\ubd80\uc2b9\uc18c\uc758 \uacbd\uc6b0\ub294 \uc6d0\uace0\uac00 \uc18c\ub97c \ubcc0\uacbd\ud558\uae30 \uc704\ud558\uc5ec, \ub610\ub294 \ud53c\uace0\uac00 \ubc18\uc18c\ub97c \uc81c\uae30\ud558\uae30 \uc704\ud558\uc5ec \ud56d\uc18c\ud560 \uc218\uac00 \uc5c6\ub2e4. \ud56d\uc18c\uad8c\uc740 \ub2f9\uc0ac\uc790\uc758 \uc18c\uc1a1\uc0c1\uc758 \uad8c\ub9ac\uc774\uba70, \uad8c\ub9ac\uc790\uac00 \uc774\uac83\uc744 \ud3ec\uae30\ud560 \uc218\ub3c4 \uc788\uace0, \ub2f9\ucd08\ubd80\ud130 \ub2f9\uc0ac\uc790 \uc0c1\ud638\uac04\uc758 \ud569\uc758\uc5d0 \uc758\ud558\uc5ec \ud56d\uc18c\ub97c \ud558\uc9c0 \uc54a\uc744 \uac83\uc744 \uc57d\uc18d\ud560 \uc218\ub3c4 \uc788\ub2e4. \uc774\uac83\uc744 '\ubd88\ud56d\uc18c(\u4e0d\u6297\u8a34)\uc758 \ud569\uc758'\ub77c \ud55c\ub2e4. \uc774\uc640 \uac19\uc740 \ud569\uc758\uac00 \uc0c1\ud638\uac04\uc758 \uc790\uc720\uc758\uc0ac\uc5d0 \uae30\ud574\uc11c \uc774\ub8e8\uc5b4\uc9c0\uba74 \ud56d\uc18c\uad8c\uc740 \ub2f9\uc0ac\uc790 \uc30d\ubc29\uc5d0 \ubc1c\uc0dd\ud558\uc9c0 \uc54a\ub294\ub2e4. \ub2e4\ub9cc \uc774\ub7ec\ud55c \ud569\uc758\uac00 \uc774\ub8e8\uc5b4\uc9c0\ub294 \uac83\uc740 \ud1b5\uc0c1\uc758 \ubbfc\uc0ac\uc0ac\uac74\uc5d0 \uc788\uc5b4\uc11c\ubfd0\uc774\uba70, \uc9c1\uad8c\ud0d0\uc9c0\uc8fc\uc758\uac00 \uac00\ubbf8\ub418\ub294 \uc778\uc0ac\uc18c\uc1a1\uc5d0\uc11c\ub294 \ud560 \uc218\uac00 \uc5c6\ub2e4."} {"question": "\ud55c\uc2b9\ud638\ub294 \ud310\uc18c\ub9ac \uc218\uc5c5\uc5d0\uc11c \ubc15\uc885\uc6d0\uc5d0\uac8c \ubb34\uc5c7\uc744 \ub9ce\uc774 \ubc30\uc6e0\ub294\uac00?", "paragraph": "\ubd80\uce5c \ud55c\uc131\ud0dc\ub294 \uc911\uc559\uc5d0\uc11c \ud06c\uac8c \ud65c\uc57d\ud558\uc9c0\ub294 \uc54a\uc558\uc73c\ub098, \ud310\uc18c\ub9ac \uba85\ucc3d\uc73c\ub85c \uc774\ub984\uc774 \ub192\uc558\uace0, \ub3d9\uc2dc\ub300\uc5d0 \ud65c\ub3d9\ud558\uc600\ub358 \uc7a5\ud310\uac1c(\u5f35\u5224\u4ecb, 1985\ub144~1937\ub144), \uae40\uc815\ubb38(\u91d1\u6b63\u6587, 1987\ub144~1935\ub144), \ubc15\ub3d9\uc2e4(\u6734\u6771\u5be6, 1897\ub144~1968\ub144) \ub4f1\uacfc \ud638\ud615\ud638\uc81c \ud558\uba74\uc11c \uc790\uc8fc \uad50\ub958\ud558\uc600\ub2e4. \ud55c\uc2b9\ud638\ub294 \uc774\ub7ec\ud55c \ubd84\uc704\uae30 \uc18d\uc5d0\uc11c \ud310\uc18c\ub9ac\ub97c \uc811\ud558\uae30 \uc2dc\uc791\ud55c \uac83\uc73c\ub85c \ubcf4\uc774\uba70, \uc774 \uc2dc\uae30\ub97c \uc804\ud6c4\ud558\uc5ec \ubc15\uc885\uc6d0(1894\ub144~1962\ub144), \uc131\uc6d0\ubaa9((\u6210\u5143\u7766, 1912\ub144~1969\ub144), \uc7a5\ud310\uac1c\uc5d0\uac8c \ud310\uc18c\ub9ac \uc218\uc5c5\uc744 \ubc1b\uc558\ub2e4. \uc774 \uac00\uc6b4\ub370 \ubc15\uc885\uc6d0\uacfc \uc131\uc6d0\ubaa9\uc740 \ubaa8\ub450 \uae40\ucc44\ub9cc\uc81c \uc18c\ub9ac\ub97c \ud588\uace0, \ud55c\uc2b9\ud638\ub294 \uc774 \uac00\uc6b4\ub370 \ubc15\uc885\uc6d0\uc758 \ubaa9\uae30\uad50\ub97c \ub9ce\uc774 \ubc30\uc6e0\ub2e4\uace0 \ud55c\ub2e4. \ub2e4\ub9cc \uc7a5\ud310\uac1c\ub9cc\uc740 \uc1a1\ub9cc\uac11\uc758 \uc81c\uc790\uc600\uace0 \ubcf8\ubc14\ud0d5\uc774 \ub3d9\ud3b8\uc81c \uc1a1\ud310 \uc18c\ub9ac\ub97c \ud558\uc600\uc73c\ubbc0\ub85c \uadf8 \uacc4\ud1b5\uc774 \ub2ec\ub790\ub294\ub370, \uc7a5\ud310\uac1c\uc5d0\uac8c\ub294 \uc81c\ube44\ub178\uc815\uae30 \uc815\ub3c4\ub97c \ubc30\uc6e0\ub2e4\uace0 \ud55c\ub2e4.", "answer": "\ubaa9\uae30\uad50", "sentence": "\uc774 \uac00\uc6b4\ub370 \ubc15\uc885\uc6d0\uc758 \ubaa9\uae30\uad50 \ub97c \ub9ce\uc774 \ubc30\uc6e0\ub2e4\uace0 \ud55c\ub2e4.", "paragraph_sentence": "\ubd80\uce5c \ud55c\uc131\ud0dc\ub294 \uc911\uc559\uc5d0\uc11c \ud06c\uac8c \ud65c\uc57d\ud558\uc9c0\ub294 \uc54a\uc558\uc73c\ub098, \ud310\uc18c\ub9ac \uba85\ucc3d\uc73c\ub85c \uc774\ub984\uc774 \ub192\uc558\uace0, \ub3d9\uc2dc\ub300\uc5d0 \ud65c\ub3d9\ud558\uc600\ub358 \uc7a5\ud310\uac1c(\u5f35\u5224\u4ecb, 1985\ub144~1937\ub144), \uae40\uc815\ubb38(\u91d1\u6b63\u6587, 1987\ub144~1935\ub144), \ubc15\ub3d9\uc2e4(\u6734\u6771\u5be6, 1897\ub144~1968\ub144) \ub4f1\uacfc \ud638\ud615\ud638\uc81c \ud558\uba74\uc11c \uc790\uc8fc \uad50\ub958\ud558\uc600\ub2e4. \ud55c\uc2b9\ud638\ub294 \uc774\ub7ec\ud55c \ubd84\uc704\uae30 \uc18d\uc5d0\uc11c \ud310\uc18c\ub9ac\ub97c \uc811\ud558\uae30 \uc2dc\uc791\ud55c \uac83\uc73c\ub85c \ubcf4\uc774\uba70, \uc774 \uc2dc\uae30\ub97c \uc804\ud6c4\ud558\uc5ec \ubc15\uc885\uc6d0(1894\ub144~1962\ub144), \uc131\uc6d0\ubaa9((\u6210\u5143\u7766, 1912\ub144~1969\ub144), \uc7a5\ud310\uac1c\uc5d0\uac8c \ud310\uc18c\ub9ac \uc218\uc5c5\uc744 \ubc1b\uc558\ub2e4. \uc774 \uac00\uc6b4\ub370 \ubc15\uc885\uc6d0\uacfc \uc131\uc6d0\ubaa9\uc740 \ubaa8\ub450 \uae40\ucc44\ub9cc\uc81c \uc18c\ub9ac\ub97c \ud588\uace0, \ud55c\uc2b9\ud638\ub294 \uc774 \uac00\uc6b4\ub370 \ubc15\uc885\uc6d0\uc758 \ubaa9\uae30\uad50 \ub97c \ub9ce\uc774 \ubc30\uc6e0\ub2e4\uace0 \ud55c\ub2e4. \ub2e4\ub9cc \uc7a5\ud310\uac1c\ub9cc\uc740 \uc1a1\ub9cc\uac11\uc758 \uc81c\uc790\uc600\uace0 \ubcf8\ubc14\ud0d5\uc774 \ub3d9\ud3b8\uc81c \uc1a1\ud310 \uc18c\ub9ac\ub97c \ud558\uc600\uc73c\ubbc0\ub85c \uadf8 \uacc4\ud1b5\uc774 \ub2ec\ub790\ub294\ub370, \uc7a5\ud310\uac1c\uc5d0\uac8c\ub294 \uc81c\ube44\ub178\uc815\uae30 \uc815\ub3c4\ub97c \ubc30\uc6e0\ub2e4\uace0 \ud55c\ub2e4.", "paragraph_answer": "\ubd80\uce5c \ud55c\uc131\ud0dc\ub294 \uc911\uc559\uc5d0\uc11c \ud06c\uac8c \ud65c\uc57d\ud558\uc9c0\ub294 \uc54a\uc558\uc73c\ub098, \ud310\uc18c\ub9ac \uba85\ucc3d\uc73c\ub85c \uc774\ub984\uc774 \ub192\uc558\uace0, \ub3d9\uc2dc\ub300\uc5d0 \ud65c\ub3d9\ud558\uc600\ub358 \uc7a5\ud310\uac1c(\u5f35\u5224\u4ecb, 1985\ub144~1937\ub144), \uae40\uc815\ubb38(\u91d1\u6b63\u6587, 1987\ub144~1935\ub144), \ubc15\ub3d9\uc2e4(\u6734\u6771\u5be6, 1897\ub144~1968\ub144) \ub4f1\uacfc \ud638\ud615\ud638\uc81c \ud558\uba74\uc11c \uc790\uc8fc \uad50\ub958\ud558\uc600\ub2e4. \ud55c\uc2b9\ud638\ub294 \uc774\ub7ec\ud55c \ubd84\uc704\uae30 \uc18d\uc5d0\uc11c \ud310\uc18c\ub9ac\ub97c \uc811\ud558\uae30 \uc2dc\uc791\ud55c \uac83\uc73c\ub85c \ubcf4\uc774\uba70, \uc774 \uc2dc\uae30\ub97c \uc804\ud6c4\ud558\uc5ec \ubc15\uc885\uc6d0(1894\ub144~1962\ub144), \uc131\uc6d0\ubaa9((\u6210\u5143\u7766, 1912\ub144~1969\ub144), \uc7a5\ud310\uac1c\uc5d0\uac8c \ud310\uc18c\ub9ac \uc218\uc5c5\uc744 \ubc1b\uc558\ub2e4. \uc774 \uac00\uc6b4\ub370 \ubc15\uc885\uc6d0\uacfc \uc131\uc6d0\ubaa9\uc740 \ubaa8\ub450 \uae40\ucc44\ub9cc\uc81c \uc18c\ub9ac\ub97c \ud588\uace0, \ud55c\uc2b9\ud638\ub294 \uc774 \uac00\uc6b4\ub370 \ubc15\uc885\uc6d0\uc758 \ubaa9\uae30\uad50 \ub97c \ub9ce\uc774 \ubc30\uc6e0\ub2e4\uace0 \ud55c\ub2e4. \ub2e4\ub9cc \uc7a5\ud310\uac1c\ub9cc\uc740 \uc1a1\ub9cc\uac11\uc758 \uc81c\uc790\uc600\uace0 \ubcf8\ubc14\ud0d5\uc774 \ub3d9\ud3b8\uc81c \uc1a1\ud310 \uc18c\ub9ac\ub97c \ud558\uc600\uc73c\ubbc0\ub85c \uadf8 \uacc4\ud1b5\uc774 \ub2ec\ub790\ub294\ub370, \uc7a5\ud310\uac1c\uc5d0\uac8c\ub294 \uc81c\ube44\ub178\uc815\uae30 \uc815\ub3c4\ub97c \ubc30\uc6e0\ub2e4\uace0 \ud55c\ub2e4.", "sentence_answer": "\uc774 \uac00\uc6b4\ub370 \ubc15\uc885\uc6d0\uc758 \ubaa9\uae30\uad50 \ub97c \ub9ce\uc774 \ubc30\uc6e0\ub2e4\uace0 \ud55c\ub2e4.", "paragraph_id": "6570115-1-0", "paragraph_question": "question: \ud55c\uc2b9\ud638\ub294 \ud310\uc18c\ub9ac \uc218\uc5c5\uc5d0\uc11c \ubc15\uc885\uc6d0\uc5d0\uac8c \ubb34\uc5c7\uc744 \ub9ce\uc774 \ubc30\uc6e0\ub294\uac00?, context: \ubd80\uce5c \ud55c\uc131\ud0dc\ub294 \uc911\uc559\uc5d0\uc11c \ud06c\uac8c \ud65c\uc57d\ud558\uc9c0\ub294 \uc54a\uc558\uc73c\ub098, \ud310\uc18c\ub9ac \uba85\ucc3d\uc73c\ub85c \uc774\ub984\uc774 \ub192\uc558\uace0, \ub3d9\uc2dc\ub300\uc5d0 \ud65c\ub3d9\ud558\uc600\ub358 \uc7a5\ud310\uac1c(\u5f35\u5224\u4ecb, 1985\ub144~1937\ub144), \uae40\uc815\ubb38(\u91d1\u6b63\u6587, 1987\ub144~1935\ub144), \ubc15\ub3d9\uc2e4(\u6734\u6771\u5be6, 1897\ub144~1968\ub144) \ub4f1\uacfc \ud638\ud615\ud638\uc81c \ud558\uba74\uc11c \uc790\uc8fc \uad50\ub958\ud558\uc600\ub2e4. \ud55c\uc2b9\ud638\ub294 \uc774\ub7ec\ud55c \ubd84\uc704\uae30 \uc18d\uc5d0\uc11c \ud310\uc18c\ub9ac\ub97c \uc811\ud558\uae30 \uc2dc\uc791\ud55c \uac83\uc73c\ub85c \ubcf4\uc774\uba70, \uc774 \uc2dc\uae30\ub97c \uc804\ud6c4\ud558\uc5ec \ubc15\uc885\uc6d0(1894\ub144~1962\ub144), \uc131\uc6d0\ubaa9((\u6210\u5143\u7766, 1912\ub144~1969\ub144), \uc7a5\ud310\uac1c\uc5d0\uac8c \ud310\uc18c\ub9ac \uc218\uc5c5\uc744 \ubc1b\uc558\ub2e4. \uc774 \uac00\uc6b4\ub370 \ubc15\uc885\uc6d0\uacfc \uc131\uc6d0\ubaa9\uc740 \ubaa8\ub450 \uae40\ucc44\ub9cc\uc81c \uc18c\ub9ac\ub97c \ud588\uace0, \ud55c\uc2b9\ud638\ub294 \uc774 \uac00\uc6b4\ub370 \ubc15\uc885\uc6d0\uc758 \ubaa9\uae30\uad50\ub97c \ub9ce\uc774 \ubc30\uc6e0\ub2e4\uace0 \ud55c\ub2e4. \ub2e4\ub9cc \uc7a5\ud310\uac1c\ub9cc\uc740 \uc1a1\ub9cc\uac11\uc758 \uc81c\uc790\uc600\uace0 \ubcf8\ubc14\ud0d5\uc774 \ub3d9\ud3b8\uc81c \uc1a1\ud310 \uc18c\ub9ac\ub97c \ud558\uc600\uc73c\ubbc0\ub85c \uadf8 \uacc4\ud1b5\uc774 \ub2ec\ub790\ub294\ub370, \uc7a5\ud310\uac1c\uc5d0\uac8c\ub294 \uc81c\ube44\ub178\uc815\uae30 \uc815\ub3c4\ub97c \ubc30\uc6e0\ub2e4\uace0 \ud55c\ub2e4."} {"question": "1657\ub144\uc5d0\uc11c 1658\ub144 \uc0ac\uc774\uc5d0 \ubc1c\uc0dd\ud55c \uc5f0\uc774\uc740 \uc804\uc7c1 \ud328\ubc30\ub85c \uc778\ud574 \uc190\ubc14\uc774 \uc655\uad6d\uc740 \uba87 \uac1c\ub85c \ubd84\ub9ac\ub418\uc5c8\ub294\uac00?", "paragraph": "\uc190\ubc14\uc774 \ubc84\uc0ac\ub974\uc758 \uac74\uad6d \uc5ed\uc0ac\ub294 \ud2f0\ubaa8\ub974 \uc12c\uc758 \uc9c0\ubc30\uad8c \ud0c8\ud658\uc744 \uc704\ud55c \ub124\ub35c\ub780\ub4dc\uc640 \ud3ec\ub974\ud22c\uac08 \uc591\uad6d\uc758 \uc624\ub79c \uac08\ub4f1\uacfc \ube48\ubc88\ud55c \ucda9\ub3cc\uc5d0 \uae4a\uc740 \uad00\ub828\uc774 \uc788\ub2e4. \uc11c\ud2f0\ubaa8\ub974\uc758 \uc544\ud1a0\ub2c8 (Atoni)\uc871\ub4e4 \uc0ac\uc774\uc5d0\uc11c \uac00\uc7a5 \uc601\ud5a5\ub825\uc774\uc788\ub358 \ud1b5\uc77c \uc190\ubc14\uc774 \uc655\uad6d\uc740 1649\uc5d0\uc11c 1655\ub144\uae4c\uc9c0 \ud3ec\ub974\ud22c\uac08 \uc815\ubd80\uc640 \ub3d9\ub9f9 \uad00\uacc4\ub97c \uc720\uc9c0 \ud588\ub2e4. \uadf8\ub7ec\ub098 \uc5bc\ub9c8 \uc9c0\ub098\uc9c0 \uc54a\uc544 \ud3ec\ub974\ud22c\uac08\uacfc\uc758 \uae30\uc874\uc758 \ub3d9\ub9f9\uc744 \ud30c\uae30\ud558\uace0, \ub2e4\uc2dc 1655\ub144\uc5d0\uc11c 1658\ub144\uae4c\uc9c0 \ub124\ub35c\ub780\ub4dc \ub3d9\uc778\ub3c4 \ud68c\uc0ac\uc640 \ub3d9\ub9f9\uc744 \ub9fa\uc5c8\ub2e4. 1657\ub144\uc5d0\uc11c 1658\ub10c \uc0ac\uc774\uc5d0 \ubc1c\uc0dd\ud55c \ud3ec\ub974\ud22c\uac08 \ud63c\ud608\uacc4 \ubbfc\uc871\uc778 \ud1a0\ud30c\uc988\uc871\uacfc\uc758 \uc5f0\uc774\uc740 \uc804\uc7c1\uc5d0\uc11c\uc758 \ud328\ubc30\ub85c \uc778\ud558\uc5ec, \uc190\ubc14\uc774 \uc655\uad6d\uc740 \ub450 \uac1c\uc758 \uc655\uad6d\uc73c\ub85c \ubd84\ub9ac\uac00 \ub41c\ub2e4. \uccab \ubc88\uc9f8\ub85c \ubd84\ub9ac\ub41c \uc655\uad6d\uc778 \uc190\ubc14\uc774 \ub044\ucc14 (Sonbai Kecil)\uc740 \ub124\ub35c\ub780\ub4dc\uac00 \uc694\uc0c8\ub97c \uc138\uc6b0\uace0 \uc790\uad6d\uc758 \uc601\ud1a0\ub85c \ud3b8\uc785 \uc2dc\ucf1c \uc9c0\ubc30\ud558\ub358 \ucfe0\ud321 \uc9c0\uc5ed\uc73c\ub85c \uc774\uc8fc\ud55c \uc9d1\ub2e8\ub4e4\uc5d0 \uc758\ud574\uc11c \uc0c8\ub85c\uc6b4 \uc655\uad6d\uc744 \uc138\uc6e0\ub2e4. \uc11c\ud2f0\ubaa8\ub974\uc758 \ub0b4\ub959\uc5d0\uc11c \ud3ec\ub974\ud22c\uac08\uc758 \uac10\uc2dc\uc640 \uc9c0\ubc30\ub97c \ubc1b\ub358 \ub098\uba38\uc9c0 \uc138\ub825\ub4e4\uc740 \uc190\ubc14\uc774 \ubc84\uc0ac\ub974 (Sonbai Besar)\ub85c \ubd88\ub9ac\uba70 \uc790\uccb4\uc801\uc778 \uc655\uad6d\uc744 \uc720\uc9c0\ud588\ub2e4.", "answer": "\ub450 \uac1c", "sentence": "\uc190\ubc14\uc774 \uc655\uad6d\uc740 \ub450 \uac1c \uc758 \uc655\uad6d\uc73c\ub85c \ubd84\ub9ac\uac00 \ub41c\ub2e4.", "paragraph_sentence": "\uc190\ubc14\uc774 \ubc84\uc0ac\ub974\uc758 \uac74\uad6d \uc5ed\uc0ac\ub294 \ud2f0\ubaa8\ub974 \uc12c\uc758 \uc9c0\ubc30\uad8c \ud0c8\ud658\uc744 \uc704\ud55c \ub124\ub35c\ub780\ub4dc\uc640 \ud3ec\ub974\ud22c\uac08 \uc591\uad6d\uc758 \uc624\ub79c \uac08\ub4f1\uacfc \ube48\ubc88\ud55c \ucda9\ub3cc\uc5d0 \uae4a\uc740 \uad00\ub828\uc774 \uc788\ub2e4. \uc11c\ud2f0\ubaa8\ub974\uc758 \uc544\ud1a0\ub2c8 (Atoni)\uc871\ub4e4 \uc0ac\uc774\uc5d0\uc11c \uac00\uc7a5 \uc601\ud5a5\ub825\uc774\uc788\ub358 \ud1b5\uc77c \uc190\ubc14\uc774 \uc655\uad6d\uc740 1649\uc5d0\uc11c 1655\ub144\uae4c\uc9c0 \ud3ec\ub974\ud22c\uac08 \uc815\ubd80\uc640 \ub3d9\ub9f9 \uad00\uacc4\ub97c \uc720\uc9c0 \ud588\ub2e4. \uadf8\ub7ec\ub098 \uc5bc\ub9c8 \uc9c0\ub098\uc9c0 \uc54a\uc544 \ud3ec\ub974\ud22c\uac08\uacfc\uc758 \uae30\uc874\uc758 \ub3d9\ub9f9\uc744 \ud30c\uae30\ud558\uace0, \ub2e4\uc2dc 1655\ub144\uc5d0\uc11c 1658\ub144\uae4c\uc9c0 \ub124\ub35c\ub780\ub4dc \ub3d9\uc778\ub3c4 \ud68c\uc0ac\uc640 \ub3d9\ub9f9\uc744 \ub9fa\uc5c8\ub2e4. 1657\ub144\uc5d0\uc11c 1658\ub10c \uc0ac\uc774\uc5d0 \ubc1c\uc0dd\ud55c \ud3ec\ub974\ud22c\uac08 \ud63c\ud608\uacc4 \ubbfc\uc871\uc778 \ud1a0\ud30c\uc988\uc871\uacfc\uc758 \uc5f0\uc774\uc740 \uc804\uc7c1\uc5d0\uc11c\uc758 \ud328\ubc30\ub85c \uc778\ud558\uc5ec, \uc190\ubc14\uc774 \uc655\uad6d\uc740 \ub450 \uac1c \uc758 \uc655\uad6d\uc73c\ub85c \ubd84\ub9ac\uac00 \ub41c\ub2e4. \uccab \ubc88\uc9f8\ub85c \ubd84\ub9ac\ub41c \uc655\uad6d\uc778 \uc190\ubc14\uc774 \ub044\ucc14 (Sonbai Kecil)\uc740 \ub124\ub35c\ub780\ub4dc\uac00 \uc694\uc0c8\ub97c \uc138\uc6b0\uace0 \uc790\uad6d\uc758 \uc601\ud1a0\ub85c \ud3b8\uc785 \uc2dc\ucf1c \uc9c0\ubc30\ud558\ub358 \ucfe0\ud321 \uc9c0\uc5ed\uc73c\ub85c \uc774\uc8fc\ud55c \uc9d1\ub2e8\ub4e4\uc5d0 \uc758\ud574\uc11c \uc0c8\ub85c\uc6b4 \uc655\uad6d\uc744 \uc138\uc6e0\ub2e4. \uc11c\ud2f0\ubaa8\ub974\uc758 \ub0b4\ub959\uc5d0\uc11c \ud3ec\ub974\ud22c\uac08\uc758 \uac10\uc2dc\uc640 \uc9c0\ubc30\ub97c \ubc1b\ub358 \ub098\uba38\uc9c0 \uc138\ub825\ub4e4\uc740 \uc190\ubc14\uc774 \ubc84\uc0ac\ub974 (Sonbai Besar)\ub85c \ubd88\ub9ac\uba70 \uc790\uccb4\uc801\uc778 \uc655\uad6d\uc744 \uc720\uc9c0\ud588\ub2e4.", "paragraph_answer": "\uc190\ubc14\uc774 \ubc84\uc0ac\ub974\uc758 \uac74\uad6d \uc5ed\uc0ac\ub294 \ud2f0\ubaa8\ub974 \uc12c\uc758 \uc9c0\ubc30\uad8c \ud0c8\ud658\uc744 \uc704\ud55c \ub124\ub35c\ub780\ub4dc\uc640 \ud3ec\ub974\ud22c\uac08 \uc591\uad6d\uc758 \uc624\ub79c \uac08\ub4f1\uacfc \ube48\ubc88\ud55c \ucda9\ub3cc\uc5d0 \uae4a\uc740 \uad00\ub828\uc774 \uc788\ub2e4. \uc11c\ud2f0\ubaa8\ub974\uc758 \uc544\ud1a0\ub2c8 (Atoni)\uc871\ub4e4 \uc0ac\uc774\uc5d0\uc11c \uac00\uc7a5 \uc601\ud5a5\ub825\uc774\uc788\ub358 \ud1b5\uc77c \uc190\ubc14\uc774 \uc655\uad6d\uc740 1649\uc5d0\uc11c 1655\ub144\uae4c\uc9c0 \ud3ec\ub974\ud22c\uac08 \uc815\ubd80\uc640 \ub3d9\ub9f9 \uad00\uacc4\ub97c \uc720\uc9c0 \ud588\ub2e4. \uadf8\ub7ec\ub098 \uc5bc\ub9c8 \uc9c0\ub098\uc9c0 \uc54a\uc544 \ud3ec\ub974\ud22c\uac08\uacfc\uc758 \uae30\uc874\uc758 \ub3d9\ub9f9\uc744 \ud30c\uae30\ud558\uace0, \ub2e4\uc2dc 1655\ub144\uc5d0\uc11c 1658\ub144\uae4c\uc9c0 \ub124\ub35c\ub780\ub4dc \ub3d9\uc778\ub3c4 \ud68c\uc0ac\uc640 \ub3d9\ub9f9\uc744 \ub9fa\uc5c8\ub2e4. 1657\ub144\uc5d0\uc11c 1658\ub10c \uc0ac\uc774\uc5d0 \ubc1c\uc0dd\ud55c \ud3ec\ub974\ud22c\uac08 \ud63c\ud608\uacc4 \ubbfc\uc871\uc778 \ud1a0\ud30c\uc988\uc871\uacfc\uc758 \uc5f0\uc774\uc740 \uc804\uc7c1\uc5d0\uc11c\uc758 \ud328\ubc30\ub85c \uc778\ud558\uc5ec, \uc190\ubc14\uc774 \uc655\uad6d\uc740 \ub450 \uac1c \uc758 \uc655\uad6d\uc73c\ub85c \ubd84\ub9ac\uac00 \ub41c\ub2e4. \uccab \ubc88\uc9f8\ub85c \ubd84\ub9ac\ub41c \uc655\uad6d\uc778 \uc190\ubc14\uc774 \ub044\ucc14 (Sonbai Kecil)\uc740 \ub124\ub35c\ub780\ub4dc\uac00 \uc694\uc0c8\ub97c \uc138\uc6b0\uace0 \uc790\uad6d\uc758 \uc601\ud1a0\ub85c \ud3b8\uc785 \uc2dc\ucf1c \uc9c0\ubc30\ud558\ub358 \ucfe0\ud321 \uc9c0\uc5ed\uc73c\ub85c \uc774\uc8fc\ud55c \uc9d1\ub2e8\ub4e4\uc5d0 \uc758\ud574\uc11c \uc0c8\ub85c\uc6b4 \uc655\uad6d\uc744 \uc138\uc6e0\ub2e4. \uc11c\ud2f0\ubaa8\ub974\uc758 \ub0b4\ub959\uc5d0\uc11c \ud3ec\ub974\ud22c\uac08\uc758 \uac10\uc2dc\uc640 \uc9c0\ubc30\ub97c \ubc1b\ub358 \ub098\uba38\uc9c0 \uc138\ub825\ub4e4\uc740 \uc190\ubc14\uc774 \ubc84\uc0ac\ub974 (Sonbai Besar)\ub85c \ubd88\ub9ac\uba70 \uc790\uccb4\uc801\uc778 \uc655\uad6d\uc744 \uc720\uc9c0\ud588\ub2e4.", "sentence_answer": "\uc190\ubc14\uc774 \uc655\uad6d\uc740 \ub450 \uac1c \uc758 \uc655\uad6d\uc73c\ub85c \ubd84\ub9ac\uac00 \ub41c\ub2e4.", "paragraph_id": "6584161-0-1", "paragraph_question": "question: 1657\ub144\uc5d0\uc11c 1658\ub144 \uc0ac\uc774\uc5d0 \ubc1c\uc0dd\ud55c \uc5f0\uc774\uc740 \uc804\uc7c1 \ud328\ubc30\ub85c \uc778\ud574 \uc190\ubc14\uc774 \uc655\uad6d\uc740 \uba87 \uac1c\ub85c \ubd84\ub9ac\ub418\uc5c8\ub294\uac00?, context: \uc190\ubc14\uc774 \ubc84\uc0ac\ub974\uc758 \uac74\uad6d \uc5ed\uc0ac\ub294 \ud2f0\ubaa8\ub974 \uc12c\uc758 \uc9c0\ubc30\uad8c \ud0c8\ud658\uc744 \uc704\ud55c \ub124\ub35c\ub780\ub4dc\uc640 \ud3ec\ub974\ud22c\uac08 \uc591\uad6d\uc758 \uc624\ub79c \uac08\ub4f1\uacfc \ube48\ubc88\ud55c \ucda9\ub3cc\uc5d0 \uae4a\uc740 \uad00\ub828\uc774 \uc788\ub2e4. \uc11c\ud2f0\ubaa8\ub974\uc758 \uc544\ud1a0\ub2c8 (Atoni)\uc871\ub4e4 \uc0ac\uc774\uc5d0\uc11c \uac00\uc7a5 \uc601\ud5a5\ub825\uc774\uc788\ub358 \ud1b5\uc77c \uc190\ubc14\uc774 \uc655\uad6d\uc740 1649\uc5d0\uc11c 1655\ub144\uae4c\uc9c0 \ud3ec\ub974\ud22c\uac08 \uc815\ubd80\uc640 \ub3d9\ub9f9 \uad00\uacc4\ub97c \uc720\uc9c0 \ud588\ub2e4. \uadf8\ub7ec\ub098 \uc5bc\ub9c8 \uc9c0\ub098\uc9c0 \uc54a\uc544 \ud3ec\ub974\ud22c\uac08\uacfc\uc758 \uae30\uc874\uc758 \ub3d9\ub9f9\uc744 \ud30c\uae30\ud558\uace0, \ub2e4\uc2dc 1655\ub144\uc5d0\uc11c 1658\ub144\uae4c\uc9c0 \ub124\ub35c\ub780\ub4dc \ub3d9\uc778\ub3c4 \ud68c\uc0ac\uc640 \ub3d9\ub9f9\uc744 \ub9fa\uc5c8\ub2e4. 1657\ub144\uc5d0\uc11c 1658\ub10c \uc0ac\uc774\uc5d0 \ubc1c\uc0dd\ud55c \ud3ec\ub974\ud22c\uac08 \ud63c\ud608\uacc4 \ubbfc\uc871\uc778 \ud1a0\ud30c\uc988\uc871\uacfc\uc758 \uc5f0\uc774\uc740 \uc804\uc7c1\uc5d0\uc11c\uc758 \ud328\ubc30\ub85c \uc778\ud558\uc5ec, \uc190\ubc14\uc774 \uc655\uad6d\uc740 \ub450 \uac1c\uc758 \uc655\uad6d\uc73c\ub85c \ubd84\ub9ac\uac00 \ub41c\ub2e4. \uccab \ubc88\uc9f8\ub85c \ubd84\ub9ac\ub41c \uc655\uad6d\uc778 \uc190\ubc14\uc774 \ub044\ucc14 (Sonbai Kecil)\uc740 \ub124\ub35c\ub780\ub4dc\uac00 \uc694\uc0c8\ub97c \uc138\uc6b0\uace0 \uc790\uad6d\uc758 \uc601\ud1a0\ub85c \ud3b8\uc785 \uc2dc\ucf1c \uc9c0\ubc30\ud558\ub358 \ucfe0\ud321 \uc9c0\uc5ed\uc73c\ub85c \uc774\uc8fc\ud55c \uc9d1\ub2e8\ub4e4\uc5d0 \uc758\ud574\uc11c \uc0c8\ub85c\uc6b4 \uc655\uad6d\uc744 \uc138\uc6e0\ub2e4. \uc11c\ud2f0\ubaa8\ub974\uc758 \ub0b4\ub959\uc5d0\uc11c \ud3ec\ub974\ud22c\uac08\uc758 \uac10\uc2dc\uc640 \uc9c0\ubc30\ub97c \ubc1b\ub358 \ub098\uba38\uc9c0 \uc138\ub825\ub4e4\uc740 \uc190\ubc14\uc774 \ubc84\uc0ac\ub974 (Sonbai Besar)\ub85c \ubd88\ub9ac\uba70 \uc790\uccb4\uc801\uc778 \uc655\uad6d\uc744 \uc720\uc9c0\ud588\ub2e4."} {"question": "1930\ub144\ub300 \uc720\ub300\uc778 \ub09c\ubbfc\uc744 \ub9cc\uc8fc\uad6d\uc73c\ub85c \uc774\uc8fc\uc2dc\ud0a4\ub824\ub358 \uacc4\ud68d\uc744 \ucd94\uc9c4\ud55c \uc815\ubd80\ub294?", "paragraph": "1930\ub144\ub300 \uc77c\ubcf8 \uc815\ubd80\ub294 \uc720\ub300\uc778 \ub09c\ubbfc\uc744 \ub9cc\uc8fc\uad6d\uc73c\ub85c \uc774\uc8fc\uc2dc\ud0a4\ub824\ub358 \uacc4\ud68d\uc744 \ucd94\uc9c4\ud55c\ub2e4(\ubcf5\uc5b4 \uacc4\ud68d, \uc77c\ubcf8\uc5b4: \u6cb3\u8c5a\u8a08\u753b). 1934\ub144\uc5d0 \uc544\uc720\uce74\uc640 \uc694\uc2dc\uc2a4\ucf00\uac00 \ucc98\uc74c \uc81c\uc548\ud55c \uacc4\ud68d\uc5d0\uc11c \uc2dc\uc791\ub418\uc5b4 1938\ub144\uc758 5\ubd80\uc7a5\uad00 \ud68c\uc758\uc5d0\uc11c \uc815\ubd80\uc758 \ubc29\uce68\uc73c\ub85c \uc815\ud574\uc84c\ub2e4. \uc721\uad70\ub300\uc88c \uc57c\uc2a4\uc5d0 \ub178\ub9ac\ud788\ub85c, \ud574\uad70\ub300\uc88c \uc774\ub204\uc988\uce74 \uace0\ub808\uc2dc\uac8c \ub4f1\uc774 \uc2e4\ubb34\ub97c \uc8fc\ub3c4\ud558\uc600\ub2e4. \ubc15\ud574\ub97c \ud53c\ud574 \uc720\ub7fd\uc5d0\uc11c \uc774\ud0c8\ud55c \uc720\ub300\uc778\ub4e4\uc744 \ub9cc\uc8fc\uad6d\uc5d0 \uc815\ucc29\uc2dc\ucf1c \uc790\uce58\uad6c\ub97c \uac74\uc124\ud558\ub824\ub358 \uacc4\ud68d\uc774\uc5c8\uc73c\ub098, \uc720\ub300\uc778 \uc801\ub300\uc815\ucc45\uc744 \ucd94\uc9c4\ud574\uac00\ub358 \ub098\uce58 \ub3c5\uc77c\uacfc\uc758 \uc6b0\ud638\uad00\uacc4 \uc190\uc0c1\uc744 \uc6b0\ub824\ud558\uc5ec \uacc4\ud68d\uc774 \uc810\ucc28 \ud750\uc9c0\ubd80\uc9c0\ub418\uc5c8\uc73c\uba70, \uc0bc\uad6d \uad70\uc0ac \ub3d9\ub9f9\uc758 \uacb0\uc131, \ub098\uce58 \ub3c5\uc77c\uacfc \uc77c\ubcf8 \uc81c\uad6d\uc774 \uc5f0\ud569\uad6d\uacfc\uc758 \uc804\uc7c1\uc744 \uac1c\uc2dc\ud558\uba74\uc11c \uc2e4\ud604 \uac00\ub2a5\uc131\uc774 \uc0ac\ub77c\uc838 \uacb0\uad6d \uacc4\ud68d\uc740 \ubb34\uc0b0\ub418\uc5c8\ub2e4.", "answer": "\uc77c\ubcf8 \uc815\ubd80", "sentence": "1930\ub144\ub300 \uc77c\ubcf8 \uc815\ubd80 \ub294 \uc720\ub300\uc778 \ub09c\ubbfc\uc744 \ub9cc\uc8fc\uad6d\uc73c\ub85c \uc774\uc8fc\uc2dc\ud0a4\ub824\ub358 \uacc4\ud68d\uc744 \ucd94\uc9c4\ud55c\ub2e4(\ubcf5\uc5b4 \uacc4\ud68d, \uc77c\ubcf8\uc5b4: \u6cb3\u8c5a\u8a08\u753b).", "paragraph_sentence": " 1930\ub144\ub300 \uc77c\ubcf8 \uc815\ubd80 \ub294 \uc720\ub300\uc778 \ub09c\ubbfc\uc744 \ub9cc\uc8fc\uad6d\uc73c\ub85c \uc774\uc8fc\uc2dc\ud0a4\ub824\ub358 \uacc4\ud68d\uc744 \ucd94\uc9c4\ud55c\ub2e4(\ubcf5\uc5b4 \uacc4\ud68d, \uc77c\ubcf8\uc5b4: \u6cb3\u8c5a\u8a08\u753b). 1934\ub144\uc5d0 \uc544\uc720\uce74\uc640 \uc694\uc2dc\uc2a4\ucf00\uac00 \ucc98\uc74c \uc81c\uc548\ud55c \uacc4\ud68d\uc5d0\uc11c \uc2dc\uc791\ub418\uc5b4 1938\ub144\uc758 5\ubd80\uc7a5\uad00 \ud68c\uc758\uc5d0\uc11c \uc815\ubd80\uc758 \ubc29\uce68\uc73c\ub85c \uc815\ud574\uc84c\ub2e4. \uc721\uad70\ub300\uc88c \uc57c\uc2a4\uc5d0 \ub178\ub9ac\ud788\ub85c, \ud574\uad70\ub300\uc88c \uc774\ub204\uc988\uce74 \uace0\ub808\uc2dc\uac8c \ub4f1\uc774 \uc2e4\ubb34\ub97c \uc8fc\ub3c4\ud558\uc600\ub2e4. \ubc15\ud574\ub97c \ud53c\ud574 \uc720\ub7fd\uc5d0\uc11c \uc774\ud0c8\ud55c \uc720\ub300\uc778\ub4e4\uc744 \ub9cc\uc8fc\uad6d\uc5d0 \uc815\ucc29\uc2dc\ucf1c \uc790\uce58\uad6c\ub97c \uac74\uc124\ud558\ub824\ub358 \uacc4\ud68d\uc774\uc5c8\uc73c\ub098, \uc720\ub300\uc778 \uc801\ub300\uc815\ucc45\uc744 \ucd94\uc9c4\ud574\uac00\ub358 \ub098\uce58 \ub3c5\uc77c\uacfc\uc758 \uc6b0\ud638\uad00\uacc4 \uc190\uc0c1\uc744 \uc6b0\ub824\ud558\uc5ec \uacc4\ud68d\uc774 \uc810\ucc28 \ud750\uc9c0\ubd80\uc9c0\ub418\uc5c8\uc73c\uba70, \uc0bc\uad6d \uad70\uc0ac \ub3d9\ub9f9\uc758 \uacb0\uc131, \ub098\uce58 \ub3c5\uc77c\uacfc \uc77c\ubcf8 \uc81c\uad6d\uc774 \uc5f0\ud569\uad6d\uacfc\uc758 \uc804\uc7c1\uc744 \uac1c\uc2dc\ud558\uba74\uc11c \uc2e4\ud604 \uac00\ub2a5\uc131\uc774 \uc0ac\ub77c\uc838 \uacb0\uad6d \uacc4\ud68d\uc740 \ubb34\uc0b0\ub418\uc5c8\ub2e4.", "paragraph_answer": "1930\ub144\ub300 \uc77c\ubcf8 \uc815\ubd80 \ub294 \uc720\ub300\uc778 \ub09c\ubbfc\uc744 \ub9cc\uc8fc\uad6d\uc73c\ub85c \uc774\uc8fc\uc2dc\ud0a4\ub824\ub358 \uacc4\ud68d\uc744 \ucd94\uc9c4\ud55c\ub2e4(\ubcf5\uc5b4 \uacc4\ud68d, \uc77c\ubcf8\uc5b4: \u6cb3\u8c5a\u8a08\u753b). 1934\ub144\uc5d0 \uc544\uc720\uce74\uc640 \uc694\uc2dc\uc2a4\ucf00\uac00 \ucc98\uc74c \uc81c\uc548\ud55c \uacc4\ud68d\uc5d0\uc11c \uc2dc\uc791\ub418\uc5b4 1938\ub144\uc758 5\ubd80\uc7a5\uad00 \ud68c\uc758\uc5d0\uc11c \uc815\ubd80\uc758 \ubc29\uce68\uc73c\ub85c \uc815\ud574\uc84c\ub2e4. \uc721\uad70\ub300\uc88c \uc57c\uc2a4\uc5d0 \ub178\ub9ac\ud788\ub85c, \ud574\uad70\ub300\uc88c \uc774\ub204\uc988\uce74 \uace0\ub808\uc2dc\uac8c \ub4f1\uc774 \uc2e4\ubb34\ub97c \uc8fc\ub3c4\ud558\uc600\ub2e4. \ubc15\ud574\ub97c \ud53c\ud574 \uc720\ub7fd\uc5d0\uc11c \uc774\ud0c8\ud55c \uc720\ub300\uc778\ub4e4\uc744 \ub9cc\uc8fc\uad6d\uc5d0 \uc815\ucc29\uc2dc\ucf1c \uc790\uce58\uad6c\ub97c \uac74\uc124\ud558\ub824\ub358 \uacc4\ud68d\uc774\uc5c8\uc73c\ub098, \uc720\ub300\uc778 \uc801\ub300\uc815\ucc45\uc744 \ucd94\uc9c4\ud574\uac00\ub358 \ub098\uce58 \ub3c5\uc77c\uacfc\uc758 \uc6b0\ud638\uad00\uacc4 \uc190\uc0c1\uc744 \uc6b0\ub824\ud558\uc5ec \uacc4\ud68d\uc774 \uc810\ucc28 \ud750\uc9c0\ubd80\uc9c0\ub418\uc5c8\uc73c\uba70, \uc0bc\uad6d \uad70\uc0ac \ub3d9\ub9f9\uc758 \uacb0\uc131, \ub098\uce58 \ub3c5\uc77c\uacfc \uc77c\ubcf8 \uc81c\uad6d\uc774 \uc5f0\ud569\uad6d\uacfc\uc758 \uc804\uc7c1\uc744 \uac1c\uc2dc\ud558\uba74\uc11c \uc2e4\ud604 \uac00\ub2a5\uc131\uc774 \uc0ac\ub77c\uc838 \uacb0\uad6d \uacc4\ud68d\uc740 \ubb34\uc0b0\ub418\uc5c8\ub2e4.", "sentence_answer": "1930\ub144\ub300 \uc77c\ubcf8 \uc815\ubd80 \ub294 \uc720\ub300\uc778 \ub09c\ubbfc\uc744 \ub9cc\uc8fc\uad6d\uc73c\ub85c \uc774\uc8fc\uc2dc\ud0a4\ub824\ub358 \uacc4\ud68d\uc744 \ucd94\uc9c4\ud55c\ub2e4(\ubcf5\uc5b4 \uacc4\ud68d, \uc77c\ubcf8\uc5b4: \u6cb3\u8c5a\u8a08\u753b).", "paragraph_id": "6529282-6-0", "paragraph_question": "question: 1930\ub144\ub300 \uc720\ub300\uc778 \ub09c\ubbfc\uc744 \ub9cc\uc8fc\uad6d\uc73c\ub85c \uc774\uc8fc\uc2dc\ud0a4\ub824\ub358 \uacc4\ud68d\uc744 \ucd94\uc9c4\ud55c \uc815\ubd80\ub294?, context: 1930\ub144\ub300 \uc77c\ubcf8 \uc815\ubd80\ub294 \uc720\ub300\uc778 \ub09c\ubbfc\uc744 \ub9cc\uc8fc\uad6d\uc73c\ub85c \uc774\uc8fc\uc2dc\ud0a4\ub824\ub358 \uacc4\ud68d\uc744 \ucd94\uc9c4\ud55c\ub2e4(\ubcf5\uc5b4 \uacc4\ud68d, \uc77c\ubcf8\uc5b4: \u6cb3\u8c5a\u8a08\u753b). 1934\ub144\uc5d0 \uc544\uc720\uce74\uc640 \uc694\uc2dc\uc2a4\ucf00\uac00 \ucc98\uc74c \uc81c\uc548\ud55c \uacc4\ud68d\uc5d0\uc11c \uc2dc\uc791\ub418\uc5b4 1938\ub144\uc758 5\ubd80\uc7a5\uad00 \ud68c\uc758\uc5d0\uc11c \uc815\ubd80\uc758 \ubc29\uce68\uc73c\ub85c \uc815\ud574\uc84c\ub2e4. \uc721\uad70\ub300\uc88c \uc57c\uc2a4\uc5d0 \ub178\ub9ac\ud788\ub85c, \ud574\uad70\ub300\uc88c \uc774\ub204\uc988\uce74 \uace0\ub808\uc2dc\uac8c \ub4f1\uc774 \uc2e4\ubb34\ub97c \uc8fc\ub3c4\ud558\uc600\ub2e4. \ubc15\ud574\ub97c \ud53c\ud574 \uc720\ub7fd\uc5d0\uc11c \uc774\ud0c8\ud55c \uc720\ub300\uc778\ub4e4\uc744 \ub9cc\uc8fc\uad6d\uc5d0 \uc815\ucc29\uc2dc\ucf1c \uc790\uce58\uad6c\ub97c \uac74\uc124\ud558\ub824\ub358 \uacc4\ud68d\uc774\uc5c8\uc73c\ub098, \uc720\ub300\uc778 \uc801\ub300\uc815\ucc45\uc744 \ucd94\uc9c4\ud574\uac00\ub358 \ub098\uce58 \ub3c5\uc77c\uacfc\uc758 \uc6b0\ud638\uad00\uacc4 \uc190\uc0c1\uc744 \uc6b0\ub824\ud558\uc5ec \uacc4\ud68d\uc774 \uc810\ucc28 \ud750\uc9c0\ubd80\uc9c0\ub418\uc5c8\uc73c\uba70, \uc0bc\uad6d \uad70\uc0ac \ub3d9\ub9f9\uc758 \uacb0\uc131, \ub098\uce58 \ub3c5\uc77c\uacfc \uc77c\ubcf8 \uc81c\uad6d\uc774 \uc5f0\ud569\uad6d\uacfc\uc758 \uc804\uc7c1\uc744 \uac1c\uc2dc\ud558\uba74\uc11c \uc2e4\ud604 \uac00\ub2a5\uc131\uc774 \uc0ac\ub77c\uc838 \uacb0\uad6d \uacc4\ud68d\uc740 \ubb34\uc0b0\ub418\uc5c8\ub2e4."} {"question": "\ubbf8\uad6d\ubcf4\ub2e4 \uba3c\uc800 \ud575\ubb34\uae30 \uac1c\ubc1c\uc744 \uc2dc\uc791\ud558\uc600\uc9c0\ub9cc \uc778\ub825\uacfc \uc790\uc6d0\uc744 \ubaa8\uc744 \uc218 \uc5c6\uc5c8\uae30 \ub54c\ubb38\uc5d0 \uacb0\uad6d \ub9e8\ud574\ud2bc \uacc4\ud68d\uc758 \uc77c\ubd80\ub85c \ud3ec\ud568\ub418\uac8c \ub41c \ub098\ub77c\ub294?", "paragraph": "\ubbf8\uad6d\uacfc \uc601\uad6d\uc740 \ud575 \uc815\ubcf4\ub97c \uc11c\ub85c \uad50\ud658\ud558\uace0 \uc788\uc5c8\uc9c0\ub9cc, \ucc98\uc74c\uc5d0\ub294 \uadf8\ub9ac \ub9ce\uc9c0 \uc54a\uc558\ub2e4. 1941\ub144 \uc601\uad6d\uc740 \ubd80\uc2dc\uc640 \ucf54\ub10c\ud2b8\uac00 \ud29c\ube0c \uc568\ub85c\uc774\uc2a4 \uacc4\ud68d\uc758 \ud611\ub825\uc744 \uac15\ud654\ud558\uc790\uace0 \ud558\uc600\uc744 \ub54c, \uc774\ub97c \uac70\ubd80\ud558\ub824 \ud558\uc600\ub2e4. \uadf8\ub7ec\ub098, \uc601\uad6d\uce21\uc774 \ubcf4\ub2e4 \uba3c\uc800 \ud575\ubb34\uae30 \uac1c\ubc1c\uc744 \uc2dc\uc791\ud558\uc600\ub294\ub370\ub3c4 \ubbf8\uad6d\uc5d0 \ube44\ud574 \uc778\ub825\uacfc \uc790\uc6d0\uc744 \ubaa8\uc744 \uc218 \uc5c6\uc5c8\uae30 \ub54c\ubb38\uc5d0, \uacb0\uad6d \ud29c\ube0c \uc54c\ub85c\uc774\uc988\ub294 \ub9e8\ud574\ud2bc \uacc4\ud68d\uc758 \uc77c\ubd80\ub85c \ud3ec\ud568\ub418\uac8c \ub418\uc5c8\ub2e4. 1942\ub144 7\uc6d4 30\uc77c \uc874 \uc564\ub354\uc2a8 \uc6e8\ubc84\ub9ac \uc790\uc791\uc740 \ucc98\uce60\uc5d0\uac8c \"\uc6b0\ub9b0 \uc0ac\uc2e4\uc744 \uc9c1\uc2dc\ud574\uc57c \ud569\ub2c8\ub2e4. \u2026\u2026 \uc6b0\ub9ac\uac00 \ud574 \ub193\uc740 \uc120\uad6c\uc801\uc778 \uc5c5\uc801\uc758 \uacb0\uacfc\ubb3c\uc740 \uc810\uc810 \uc904\uc5b4\ub4e4\uace0 \uc788\uc2b5\ub2c8\ub2e4. \uc6b0\ub9ac\uac00 \uc774\uac83\uc744 \ube68\ub9ac \uc0ac\uc6a9\ud558\uc5ec \uc11c\ub85c\uc758 \uacf5\ub85c\ub97c \ud569\uccd0\uc57c \ud569\ub2c8\ub2e4. \uc870\ub9cc\uac04 \uc6b0\ub9ac\ub294 \uc544\ubb34\uac83\ub3c4 \uac16\uc9c0 \ubabb\ud558\uac8c \ub420 \uac83\uc785\ub2c8\ub2e4.\"\ub77c\uace0 \uc870\uc5b8\ud558\uc600\ub2e4. \uc774 \ub54c, \uacc4\ud68d\uc5d0\uc11c \uc601\uad6d\uc774 \ucc28\uc9c0\ud558\ub294 \uc9c0\uc704\ub294 \uc774\ubbf8 \uc57d\ud654\ub418\uace0 \uc788\uc5c8\uace0, \uc8fc\ub3c4\uad8c\uc740 \ubbf8\uad6d\uc73c\ub85c \uc62e\uaca8\uac00\uace0 \uc788\uc5c8\ub2e4.", "answer": "\uc601\uad6d", "sentence": "\ubbf8\uad6d\uacfc \uc601\uad6d \uc740 \ud575 \uc815\ubcf4\ub97c \uc11c\ub85c \uad50\ud658\ud558\uace0 \uc788\uc5c8\uc9c0\ub9cc, \ucc98\uc74c\uc5d0\ub294 \uadf8\ub9ac \ub9ce\uc9c0 \uc54a\uc558\ub2e4.", "paragraph_sentence": " \ubbf8\uad6d\uacfc \uc601\uad6d \uc740 \ud575 \uc815\ubcf4\ub97c \uc11c\ub85c \uad50\ud658\ud558\uace0 \uc788\uc5c8\uc9c0\ub9cc, \ucc98\uc74c\uc5d0\ub294 \uadf8\ub9ac \ub9ce\uc9c0 \uc54a\uc558\ub2e4. 1941\ub144 \uc601\uad6d\uc740 \ubd80\uc2dc\uc640 \ucf54\ub10c\ud2b8\uac00 \ud29c\ube0c \uc568\ub85c\uc774\uc2a4 \uacc4\ud68d\uc758 \ud611\ub825\uc744 \uac15\ud654\ud558\uc790\uace0 \ud558\uc600\uc744 \ub54c, \uc774\ub97c \uac70\ubd80\ud558\ub824 \ud558\uc600\ub2e4. \uadf8\ub7ec\ub098, \uc601\uad6d\uce21\uc774 \ubcf4\ub2e4 \uba3c\uc800 \ud575\ubb34\uae30 \uac1c\ubc1c\uc744 \uc2dc\uc791\ud558\uc600\ub294\ub370\ub3c4 \ubbf8\uad6d\uc5d0 \ube44\ud574 \uc778\ub825\uacfc \uc790\uc6d0\uc744 \ubaa8\uc744 \uc218 \uc5c6\uc5c8\uae30 \ub54c\ubb38\uc5d0, \uacb0\uad6d \ud29c\ube0c \uc54c\ub85c\uc774\uc988\ub294 \ub9e8\ud574\ud2bc \uacc4\ud68d\uc758 \uc77c\ubd80\ub85c \ud3ec\ud568\ub418\uac8c \ub418\uc5c8\ub2e4. 1942\ub144 7\uc6d4 30\uc77c \uc874 \uc564\ub354\uc2a8 \uc6e8\ubc84\ub9ac \uc790\uc791\uc740 \ucc98\uce60\uc5d0\uac8c \"\uc6b0\ub9b0 \uc0ac\uc2e4\uc744 \uc9c1\uc2dc\ud574\uc57c \ud569\ub2c8\ub2e4. \u2026\u2026 \uc6b0\ub9ac\uac00 \ud574 \ub193\uc740 \uc120\uad6c\uc801\uc778 \uc5c5\uc801\uc758 \uacb0\uacfc\ubb3c\uc740 \uc810\uc810 \uc904\uc5b4\ub4e4\uace0 \uc788\uc2b5\ub2c8\ub2e4. \uc6b0\ub9ac\uac00 \uc774\uac83\uc744 \ube68\ub9ac \uc0ac\uc6a9\ud558\uc5ec \uc11c\ub85c\uc758 \uacf5\ub85c\ub97c \ud569\uccd0\uc57c \ud569\ub2c8\ub2e4. \uc870\ub9cc\uac04 \uc6b0\ub9ac\ub294 \uc544\ubb34\uac83\ub3c4 \uac16\uc9c0 \ubabb\ud558\uac8c \ub420 \uac83\uc785\ub2c8\ub2e4. \"\ub77c\uace0 \uc870\uc5b8\ud558\uc600\ub2e4. \uc774 \ub54c, \uacc4\ud68d\uc5d0\uc11c \uc601\uad6d\uc774 \ucc28\uc9c0\ud558\ub294 \uc9c0\uc704\ub294 \uc774\ubbf8 \uc57d\ud654\ub418\uace0 \uc788\uc5c8\uace0, \uc8fc\ub3c4\uad8c\uc740 \ubbf8\uad6d\uc73c\ub85c \uc62e\uaca8\uac00\uace0 \uc788\uc5c8\ub2e4.", "paragraph_answer": "\ubbf8\uad6d\uacfc \uc601\uad6d \uc740 \ud575 \uc815\ubcf4\ub97c \uc11c\ub85c \uad50\ud658\ud558\uace0 \uc788\uc5c8\uc9c0\ub9cc, \ucc98\uc74c\uc5d0\ub294 \uadf8\ub9ac \ub9ce\uc9c0 \uc54a\uc558\ub2e4. 1941\ub144 \uc601\uad6d\uc740 \ubd80\uc2dc\uc640 \ucf54\ub10c\ud2b8\uac00 \ud29c\ube0c \uc568\ub85c\uc774\uc2a4 \uacc4\ud68d\uc758 \ud611\ub825\uc744 \uac15\ud654\ud558\uc790\uace0 \ud558\uc600\uc744 \ub54c, \uc774\ub97c \uac70\ubd80\ud558\ub824 \ud558\uc600\ub2e4. \uadf8\ub7ec\ub098, \uc601\uad6d\uce21\uc774 \ubcf4\ub2e4 \uba3c\uc800 \ud575\ubb34\uae30 \uac1c\ubc1c\uc744 \uc2dc\uc791\ud558\uc600\ub294\ub370\ub3c4 \ubbf8\uad6d\uc5d0 \ube44\ud574 \uc778\ub825\uacfc \uc790\uc6d0\uc744 \ubaa8\uc744 \uc218 \uc5c6\uc5c8\uae30 \ub54c\ubb38\uc5d0, \uacb0\uad6d \ud29c\ube0c \uc54c\ub85c\uc774\uc988\ub294 \ub9e8\ud574\ud2bc \uacc4\ud68d\uc758 \uc77c\ubd80\ub85c \ud3ec\ud568\ub418\uac8c \ub418\uc5c8\ub2e4. 1942\ub144 7\uc6d4 30\uc77c \uc874 \uc564\ub354\uc2a8 \uc6e8\ubc84\ub9ac \uc790\uc791\uc740 \ucc98\uce60\uc5d0\uac8c \"\uc6b0\ub9b0 \uc0ac\uc2e4\uc744 \uc9c1\uc2dc\ud574\uc57c \ud569\ub2c8\ub2e4. \u2026\u2026 \uc6b0\ub9ac\uac00 \ud574 \ub193\uc740 \uc120\uad6c\uc801\uc778 \uc5c5\uc801\uc758 \uacb0\uacfc\ubb3c\uc740 \uc810\uc810 \uc904\uc5b4\ub4e4\uace0 \uc788\uc2b5\ub2c8\ub2e4. \uc6b0\ub9ac\uac00 \uc774\uac83\uc744 \ube68\ub9ac \uc0ac\uc6a9\ud558\uc5ec \uc11c\ub85c\uc758 \uacf5\ub85c\ub97c \ud569\uccd0\uc57c \ud569\ub2c8\ub2e4. \uc870\ub9cc\uac04 \uc6b0\ub9ac\ub294 \uc544\ubb34\uac83\ub3c4 \uac16\uc9c0 \ubabb\ud558\uac8c \ub420 \uac83\uc785\ub2c8\ub2e4.\"\ub77c\uace0 \uc870\uc5b8\ud558\uc600\ub2e4. \uc774 \ub54c, \uacc4\ud68d\uc5d0\uc11c \uc601\uad6d\uc774 \ucc28\uc9c0\ud558\ub294 \uc9c0\uc704\ub294 \uc774\ubbf8 \uc57d\ud654\ub418\uace0 \uc788\uc5c8\uace0, \uc8fc\ub3c4\uad8c\uc740 \ubbf8\uad6d\uc73c\ub85c \uc62e\uaca8\uac00\uace0 \uc788\uc5c8\ub2e4.", "sentence_answer": "\ubbf8\uad6d\uacfc \uc601\uad6d \uc740 \ud575 \uc815\ubcf4\ub97c \uc11c\ub85c \uad50\ud658\ud558\uace0 \uc788\uc5c8\uc9c0\ub9cc, \ucc98\uc74c\uc5d0\ub294 \uadf8\ub9ac \ub9ce\uc9c0 \uc54a\uc558\ub2e4.", "paragraph_id": "6075564-9-1", "paragraph_question": "question: \ubbf8\uad6d\ubcf4\ub2e4 \uba3c\uc800 \ud575\ubb34\uae30 \uac1c\ubc1c\uc744 \uc2dc\uc791\ud558\uc600\uc9c0\ub9cc \uc778\ub825\uacfc \uc790\uc6d0\uc744 \ubaa8\uc744 \uc218 \uc5c6\uc5c8\uae30 \ub54c\ubb38\uc5d0 \uacb0\uad6d \ub9e8\ud574\ud2bc \uacc4\ud68d\uc758 \uc77c\ubd80\ub85c \ud3ec\ud568\ub418\uac8c \ub41c \ub098\ub77c\ub294?, context: \ubbf8\uad6d\uacfc \uc601\uad6d\uc740 \ud575 \uc815\ubcf4\ub97c \uc11c\ub85c \uad50\ud658\ud558\uace0 \uc788\uc5c8\uc9c0\ub9cc, \ucc98\uc74c\uc5d0\ub294 \uadf8\ub9ac \ub9ce\uc9c0 \uc54a\uc558\ub2e4. 1941\ub144 \uc601\uad6d\uc740 \ubd80\uc2dc\uc640 \ucf54\ub10c\ud2b8\uac00 \ud29c\ube0c \uc568\ub85c\uc774\uc2a4 \uacc4\ud68d\uc758 \ud611\ub825\uc744 \uac15\ud654\ud558\uc790\uace0 \ud558\uc600\uc744 \ub54c, \uc774\ub97c \uac70\ubd80\ud558\ub824 \ud558\uc600\ub2e4. \uadf8\ub7ec\ub098, \uc601\uad6d\uce21\uc774 \ubcf4\ub2e4 \uba3c\uc800 \ud575\ubb34\uae30 \uac1c\ubc1c\uc744 \uc2dc\uc791\ud558\uc600\ub294\ub370\ub3c4 \ubbf8\uad6d\uc5d0 \ube44\ud574 \uc778\ub825\uacfc \uc790\uc6d0\uc744 \ubaa8\uc744 \uc218 \uc5c6\uc5c8\uae30 \ub54c\ubb38\uc5d0, \uacb0\uad6d \ud29c\ube0c \uc54c\ub85c\uc774\uc988\ub294 \ub9e8\ud574\ud2bc \uacc4\ud68d\uc758 \uc77c\ubd80\ub85c \ud3ec\ud568\ub418\uac8c \ub418\uc5c8\ub2e4. 1942\ub144 7\uc6d4 30\uc77c \uc874 \uc564\ub354\uc2a8 \uc6e8\ubc84\ub9ac \uc790\uc791\uc740 \ucc98\uce60\uc5d0\uac8c \"\uc6b0\ub9b0 \uc0ac\uc2e4\uc744 \uc9c1\uc2dc\ud574\uc57c \ud569\ub2c8\ub2e4. \u2026\u2026 \uc6b0\ub9ac\uac00 \ud574 \ub193\uc740 \uc120\uad6c\uc801\uc778 \uc5c5\uc801\uc758 \uacb0\uacfc\ubb3c\uc740 \uc810\uc810 \uc904\uc5b4\ub4e4\uace0 \uc788\uc2b5\ub2c8\ub2e4. \uc6b0\ub9ac\uac00 \uc774\uac83\uc744 \ube68\ub9ac \uc0ac\uc6a9\ud558\uc5ec \uc11c\ub85c\uc758 \uacf5\ub85c\ub97c \ud569\uccd0\uc57c \ud569\ub2c8\ub2e4. \uc870\ub9cc\uac04 \uc6b0\ub9ac\ub294 \uc544\ubb34\uac83\ub3c4 \uac16\uc9c0 \ubabb\ud558\uac8c \ub420 \uac83\uc785\ub2c8\ub2e4.\"\ub77c\uace0 \uc870\uc5b8\ud558\uc600\ub2e4. \uc774 \ub54c, \uacc4\ud68d\uc5d0\uc11c \uc601\uad6d\uc774 \ucc28\uc9c0\ud558\ub294 \uc9c0\uc704\ub294 \uc774\ubbf8 \uc57d\ud654\ub418\uace0 \uc788\uc5c8\uace0, \uc8fc\ub3c4\uad8c\uc740 \ubbf8\uad6d\uc73c\ub85c \uc62e\uaca8\uac00\uace0 \uc788\uc5c8\ub2e4."} {"question": "HP \ud68c\ubcf5\uc81c\ubcf4\ub2e4 MP \ud68c\ubcf5\uc81c\ub97c \uc8fc\ub85c \uc0ac\uc6a9\ud558\ub294 \ud074\ub798\uc2a4\ub294?", "paragraph": "\uc790\uc2e0\uc758 \uc815\uc2e0\ub825\uc744 \uacf5\uaca9\uc801\uc778 \ub9c8\ubc95\uc73c\ub85c \ubc14\uafb8\ub294 \uc2e0\ube44\ud55c \ub2a5\ub825\uc744 \uac00\uc9c4 \ud074\ub798\uc2a4\uc774\ub2e4. \uac00\uc7a5 \ub192\uc740 \uc218\uc900\uc758 \uc804\ub7b5\uacfc \ucee8\ud2b8\ub864\uc744 \ud544\uc694\ub85c \ud558\uae30 \ub54c\ubb38\uc5d0 \uac00\uc7a5 \ub2e4\ub8e8\uae30 \uc5b4\ub835\uae30\ub294 \ud558\uc9c0\ub9cc, \ube44\uad50\uc801 \ucd08\ubc18\ubd80\ud130 \uc0ac\uc6a9\ud560 \uc218 \uc788\ub294 \ubc94\uc704 \uacf5\uaca9 \ub9c8\ubc95\uc744 \ub2a5\uc219\ud558\uac8c\ub9cc \uc0ac\uc6a9\ud55c\ub2e4\uba74 \uac00\uc7a5 \ube60\ub974\uac8c \uc131\uc7a5\ud560 \uc218 \uc788\ub294 \uce90\ub9ad\ud130\uc774\uae30\ub3c4 \ud558\ub2e4. \ub9c8\ub3c4\uc0ac\ub294 \uae30\ubcf8\uc801\uc73c\ub85c \uc790\uc2e0\uc758 \uc815\uc2e0\ub825\uc744 \ubb34\uae30\ub85c \uc0bc\uae30 \ub54c\ubb38\uc5d0, \ubc29\uc5b4\ub825\uc774\ub098 \uacf5\uaca9\ub825\ubcf4\ub2e4\ub294 \uc790\uc2e0\uc758 \uc815\uc2e0\ub825\uc744 \ub192\uc5ec\uc8fc\ub294 \uc7a5\ube44\ub97c \uc990\uaca8 \ucc29\uc6a9\ud55c\ub2e4. \ub610\ud55c \uc6d0\uac70\ub9ac \uacf5\uaca9(\ub9c8\ubc95)\uc744 \uc8fc\ub85c \uc0ac\uc6a9\ud558\uae30 \ub54c\ubb38\uc5d0, \uc804\uc0ac\uc640 \uac19\uc740 \uadfc\uc811 \uc804\ud22c \ud074\ub798\uc2a4\uc640\ub294 \ub2ec\ub9ac HP \ud68c\ubcf5\uc81c\ubcf4\ub2e4\ub294 MP \ud68c\ubcf5\uc81c(\ucf00\ubaa8\ub9c8\uc77c \ud3ec\uc158)\ub97c \uc8fc\ub85c \uc0ac\uc6a9\ud558\uac8c \ub41c\ub2e4. \ub610\ud55c \uc804\ud22c \uc2dc \uc774\ub3d9\uc744 \ub9ce\uc774 \ud558\uac8c \ub418\ubbc0\ub85c \uc774\ub3d9 \uc18d\ub3c4\ub97c \uc62c\ub824\uc8fc\ub294 \uc57d\ubb3c(\ud654\uc774\ud2b8 \ud035 \ud3ec\uc158)\uc640 \uc2a4\ud0dc\ubbf8\ub108 \ud68c\ubcf5\uc81c\ub97c \ucc59\uae30\ub3c4\ub85d \ud55c\ub2e4. \ub9c8\ub3c4\uc0ac\ub294 \ub2e4\ub978 \ud074\ub798\uc2a4\uc5d0 \ube44\ud558\uc5ec \uc808\ub300\uc801\uc73c\ub85c \ubc29\uc5b4\ub825\uacfc \ubb3c\ub9ac \uacf5\uaca9\ub825\uc774 \ub0ae\uae30 \ub54c\ubb38\uc5d0, \ubaac\uc2a4\ud130\uc758 \uc811\uadfc\uc744 \ud5c8\uc6a9\ud560 \uacbd\uc6b0 \uc21c\uc2dd\uac04\uc5d0 \ucc2c \ubc14\ub2e5\uc5d0 \ub4dc\ub7ec\ub215\uac8c \ub420 \uac83\uc774\ub2e4. \uadf8\ub807\uae30 \ub54c\ubb38\uc5d0 \ub9c8\ub3c4\uc0ac\ub4e4\uc740 \uc804\ud22c\uc2dc \uae30\ubcf8\uc801\uc73c\ub85c \ub2e4\uc74c\uacfc \uac19\uc740 \uc804\uc220\uc744 \uc0ac\uc6a9\ud558\uac8c \ub41c\ub2e4.", "answer": "\ub9c8\ub3c4\uc0ac", "sentence": "\ub9c8\ub3c4\uc0ac \ub294 \uae30\ubcf8\uc801\uc73c\ub85c \uc790\uc2e0\uc758 \uc815\uc2e0\ub825\uc744 \ubb34\uae30\ub85c \uc0bc\uae30 \ub54c\ubb38\uc5d0, \ubc29\uc5b4\ub825\uc774\ub098 \uacf5\uaca9\ub825\ubcf4\ub2e4\ub294 \uc790\uc2e0\uc758 \uc815\uc2e0\ub825\uc744 \ub192\uc5ec\uc8fc\ub294 \uc7a5\ube44\ub97c \uc990\uaca8 \ucc29\uc6a9\ud55c\ub2e4.", "paragraph_sentence": "\uc790\uc2e0\uc758 \uc815\uc2e0\ub825\uc744 \uacf5\uaca9\uc801\uc778 \ub9c8\ubc95\uc73c\ub85c \ubc14\uafb8\ub294 \uc2e0\ube44\ud55c \ub2a5\ub825\uc744 \uac00\uc9c4 \ud074\ub798\uc2a4\uc774\ub2e4. \uac00\uc7a5 \ub192\uc740 \uc218\uc900\uc758 \uc804\ub7b5\uacfc \ucee8\ud2b8\ub864\uc744 \ud544\uc694\ub85c \ud558\uae30 \ub54c\ubb38\uc5d0 \uac00\uc7a5 \ub2e4\ub8e8\uae30 \uc5b4\ub835\uae30\ub294 \ud558\uc9c0\ub9cc, \ube44\uad50\uc801 \ucd08\ubc18\ubd80\ud130 \uc0ac\uc6a9\ud560 \uc218 \uc788\ub294 \ubc94\uc704 \uacf5\uaca9 \ub9c8\ubc95\uc744 \ub2a5\uc219\ud558\uac8c\ub9cc \uc0ac\uc6a9\ud55c\ub2e4\uba74 \uac00\uc7a5 \ube60\ub974\uac8c \uc131\uc7a5\ud560 \uc218 \uc788\ub294 \uce90\ub9ad\ud130\uc774\uae30\ub3c4 \ud558\ub2e4. \ub9c8\ub3c4\uc0ac \ub294 \uae30\ubcf8\uc801\uc73c\ub85c \uc790\uc2e0\uc758 \uc815\uc2e0\ub825\uc744 \ubb34\uae30\ub85c \uc0bc\uae30 \ub54c\ubb38\uc5d0, \ubc29\uc5b4\ub825\uc774\ub098 \uacf5\uaca9\ub825\ubcf4\ub2e4\ub294 \uc790\uc2e0\uc758 \uc815\uc2e0\ub825\uc744 \ub192\uc5ec\uc8fc\ub294 \uc7a5\ube44\ub97c \uc990\uaca8 \ucc29\uc6a9\ud55c\ub2e4. \ub610\ud55c \uc6d0\uac70\ub9ac \uacf5\uaca9(\ub9c8\ubc95)\uc744 \uc8fc\ub85c \uc0ac\uc6a9\ud558\uae30 \ub54c\ubb38\uc5d0, \uc804\uc0ac\uc640 \uac19\uc740 \uadfc\uc811 \uc804\ud22c \ud074\ub798\uc2a4\uc640\ub294 \ub2ec\ub9ac HP \ud68c\ubcf5\uc81c\ubcf4\ub2e4\ub294 MP \ud68c\ubcf5\uc81c(\ucf00\ubaa8\ub9c8\uc77c \ud3ec\uc158)\ub97c \uc8fc\ub85c \uc0ac\uc6a9\ud558\uac8c \ub41c\ub2e4. \ub610\ud55c \uc804\ud22c \uc2dc \uc774\ub3d9\uc744 \ub9ce\uc774 \ud558\uac8c \ub418\ubbc0\ub85c \uc774\ub3d9 \uc18d\ub3c4\ub97c \uc62c\ub824\uc8fc\ub294 \uc57d\ubb3c(\ud654\uc774\ud2b8 \ud035 \ud3ec\uc158)\uc640 \uc2a4\ud0dc\ubbf8\ub108 \ud68c\ubcf5\uc81c\ub97c \ucc59\uae30\ub3c4\ub85d \ud55c\ub2e4. \ub9c8\ub3c4\uc0ac\ub294 \ub2e4\ub978 \ud074\ub798\uc2a4\uc5d0 \ube44\ud558\uc5ec \uc808\ub300\uc801\uc73c\ub85c \ubc29\uc5b4\ub825\uacfc \ubb3c\ub9ac \uacf5\uaca9\ub825\uc774 \ub0ae\uae30 \ub54c\ubb38\uc5d0, \ubaac\uc2a4\ud130\uc758 \uc811\uadfc\uc744 \ud5c8\uc6a9\ud560 \uacbd\uc6b0 \uc21c\uc2dd\uac04\uc5d0 \ucc2c \ubc14\ub2e5\uc5d0 \ub4dc\ub7ec\ub215\uac8c \ub420 \uac83\uc774\ub2e4. \uadf8\ub807\uae30 \ub54c\ubb38\uc5d0 \ub9c8\ub3c4\uc0ac\ub4e4\uc740 \uc804\ud22c\uc2dc \uae30\ubcf8\uc801\uc73c\ub85c \ub2e4\uc74c\uacfc \uac19\uc740 \uc804\uc220\uc744 \uc0ac\uc6a9\ud558\uac8c \ub41c\ub2e4.", "paragraph_answer": "\uc790\uc2e0\uc758 \uc815\uc2e0\ub825\uc744 \uacf5\uaca9\uc801\uc778 \ub9c8\ubc95\uc73c\ub85c \ubc14\uafb8\ub294 \uc2e0\ube44\ud55c \ub2a5\ub825\uc744 \uac00\uc9c4 \ud074\ub798\uc2a4\uc774\ub2e4. \uac00\uc7a5 \ub192\uc740 \uc218\uc900\uc758 \uc804\ub7b5\uacfc \ucee8\ud2b8\ub864\uc744 \ud544\uc694\ub85c \ud558\uae30 \ub54c\ubb38\uc5d0 \uac00\uc7a5 \ub2e4\ub8e8\uae30 \uc5b4\ub835\uae30\ub294 \ud558\uc9c0\ub9cc, \ube44\uad50\uc801 \ucd08\ubc18\ubd80\ud130 \uc0ac\uc6a9\ud560 \uc218 \uc788\ub294 \ubc94\uc704 \uacf5\uaca9 \ub9c8\ubc95\uc744 \ub2a5\uc219\ud558\uac8c\ub9cc \uc0ac\uc6a9\ud55c\ub2e4\uba74 \uac00\uc7a5 \ube60\ub974\uac8c \uc131\uc7a5\ud560 \uc218 \uc788\ub294 \uce90\ub9ad\ud130\uc774\uae30\ub3c4 \ud558\ub2e4. \ub9c8\ub3c4\uc0ac \ub294 \uae30\ubcf8\uc801\uc73c\ub85c \uc790\uc2e0\uc758 \uc815\uc2e0\ub825\uc744 \ubb34\uae30\ub85c \uc0bc\uae30 \ub54c\ubb38\uc5d0, \ubc29\uc5b4\ub825\uc774\ub098 \uacf5\uaca9\ub825\ubcf4\ub2e4\ub294 \uc790\uc2e0\uc758 \uc815\uc2e0\ub825\uc744 \ub192\uc5ec\uc8fc\ub294 \uc7a5\ube44\ub97c \uc990\uaca8 \ucc29\uc6a9\ud55c\ub2e4. \ub610\ud55c \uc6d0\uac70\ub9ac \uacf5\uaca9(\ub9c8\ubc95)\uc744 \uc8fc\ub85c \uc0ac\uc6a9\ud558\uae30 \ub54c\ubb38\uc5d0, \uc804\uc0ac\uc640 \uac19\uc740 \uadfc\uc811 \uc804\ud22c \ud074\ub798\uc2a4\uc640\ub294 \ub2ec\ub9ac HP \ud68c\ubcf5\uc81c\ubcf4\ub2e4\ub294 MP \ud68c\ubcf5\uc81c(\ucf00\ubaa8\ub9c8\uc77c \ud3ec\uc158)\ub97c \uc8fc\ub85c \uc0ac\uc6a9\ud558\uac8c \ub41c\ub2e4. \ub610\ud55c \uc804\ud22c \uc2dc \uc774\ub3d9\uc744 \ub9ce\uc774 \ud558\uac8c \ub418\ubbc0\ub85c \uc774\ub3d9 \uc18d\ub3c4\ub97c \uc62c\ub824\uc8fc\ub294 \uc57d\ubb3c(\ud654\uc774\ud2b8 \ud035 \ud3ec\uc158)\uc640 \uc2a4\ud0dc\ubbf8\ub108 \ud68c\ubcf5\uc81c\ub97c \ucc59\uae30\ub3c4\ub85d \ud55c\ub2e4. \ub9c8\ub3c4\uc0ac\ub294 \ub2e4\ub978 \ud074\ub798\uc2a4\uc5d0 \ube44\ud558\uc5ec \uc808\ub300\uc801\uc73c\ub85c \ubc29\uc5b4\ub825\uacfc \ubb3c\ub9ac \uacf5\uaca9\ub825\uc774 \ub0ae\uae30 \ub54c\ubb38\uc5d0, \ubaac\uc2a4\ud130\uc758 \uc811\uadfc\uc744 \ud5c8\uc6a9\ud560 \uacbd\uc6b0 \uc21c\uc2dd\uac04\uc5d0 \ucc2c \ubc14\ub2e5\uc5d0 \ub4dc\ub7ec\ub215\uac8c \ub420 \uac83\uc774\ub2e4. \uadf8\ub807\uae30 \ub54c\ubb38\uc5d0 \ub9c8\ub3c4\uc0ac\ub4e4\uc740 \uc804\ud22c\uc2dc \uae30\ubcf8\uc801\uc73c\ub85c \ub2e4\uc74c\uacfc \uac19\uc740 \uc804\uc220\uc744 \uc0ac\uc6a9\ud558\uac8c \ub41c\ub2e4.", "sentence_answer": " \ub9c8\ub3c4\uc0ac \ub294 \uae30\ubcf8\uc801\uc73c\ub85c \uc790\uc2e0\uc758 \uc815\uc2e0\ub825\uc744 \ubb34\uae30\ub85c \uc0bc\uae30 \ub54c\ubb38\uc5d0, \ubc29\uc5b4\ub825\uc774\ub098 \uacf5\uaca9\ub825\ubcf4\ub2e4\ub294 \uc790\uc2e0\uc758 \uc815\uc2e0\ub825\uc744 \ub192\uc5ec\uc8fc\ub294 \uc7a5\ube44\ub97c \uc990\uaca8 \ucc29\uc6a9\ud55c\ub2e4.", "paragraph_id": "6520313-1-1", "paragraph_question": "question: HP \ud68c\ubcf5\uc81c\ubcf4\ub2e4 MP \ud68c\ubcf5\uc81c\ub97c \uc8fc\ub85c \uc0ac\uc6a9\ud558\ub294 \ud074\ub798\uc2a4\ub294?, context: \uc790\uc2e0\uc758 \uc815\uc2e0\ub825\uc744 \uacf5\uaca9\uc801\uc778 \ub9c8\ubc95\uc73c\ub85c \ubc14\uafb8\ub294 \uc2e0\ube44\ud55c \ub2a5\ub825\uc744 \uac00\uc9c4 \ud074\ub798\uc2a4\uc774\ub2e4. \uac00\uc7a5 \ub192\uc740 \uc218\uc900\uc758 \uc804\ub7b5\uacfc \ucee8\ud2b8\ub864\uc744 \ud544\uc694\ub85c \ud558\uae30 \ub54c\ubb38\uc5d0 \uac00\uc7a5 \ub2e4\ub8e8\uae30 \uc5b4\ub835\uae30\ub294 \ud558\uc9c0\ub9cc, \ube44\uad50\uc801 \ucd08\ubc18\ubd80\ud130 \uc0ac\uc6a9\ud560 \uc218 \uc788\ub294 \ubc94\uc704 \uacf5\uaca9 \ub9c8\ubc95\uc744 \ub2a5\uc219\ud558\uac8c\ub9cc \uc0ac\uc6a9\ud55c\ub2e4\uba74 \uac00\uc7a5 \ube60\ub974\uac8c \uc131\uc7a5\ud560 \uc218 \uc788\ub294 \uce90\ub9ad\ud130\uc774\uae30\ub3c4 \ud558\ub2e4. \ub9c8\ub3c4\uc0ac\ub294 \uae30\ubcf8\uc801\uc73c\ub85c \uc790\uc2e0\uc758 \uc815\uc2e0\ub825\uc744 \ubb34\uae30\ub85c \uc0bc\uae30 \ub54c\ubb38\uc5d0, \ubc29\uc5b4\ub825\uc774\ub098 \uacf5\uaca9\ub825\ubcf4\ub2e4\ub294 \uc790\uc2e0\uc758 \uc815\uc2e0\ub825\uc744 \ub192\uc5ec\uc8fc\ub294 \uc7a5\ube44\ub97c \uc990\uaca8 \ucc29\uc6a9\ud55c\ub2e4. \ub610\ud55c \uc6d0\uac70\ub9ac \uacf5\uaca9(\ub9c8\ubc95)\uc744 \uc8fc\ub85c \uc0ac\uc6a9\ud558\uae30 \ub54c\ubb38\uc5d0, \uc804\uc0ac\uc640 \uac19\uc740 \uadfc\uc811 \uc804\ud22c \ud074\ub798\uc2a4\uc640\ub294 \ub2ec\ub9ac HP \ud68c\ubcf5\uc81c\ubcf4\ub2e4\ub294 MP \ud68c\ubcf5\uc81c(\ucf00\ubaa8\ub9c8\uc77c \ud3ec\uc158)\ub97c \uc8fc\ub85c \uc0ac\uc6a9\ud558\uac8c \ub41c\ub2e4. \ub610\ud55c \uc804\ud22c \uc2dc \uc774\ub3d9\uc744 \ub9ce\uc774 \ud558\uac8c \ub418\ubbc0\ub85c \uc774\ub3d9 \uc18d\ub3c4\ub97c \uc62c\ub824\uc8fc\ub294 \uc57d\ubb3c(\ud654\uc774\ud2b8 \ud035 \ud3ec\uc158)\uc640 \uc2a4\ud0dc\ubbf8\ub108 \ud68c\ubcf5\uc81c\ub97c \ucc59\uae30\ub3c4\ub85d \ud55c\ub2e4. \ub9c8\ub3c4\uc0ac\ub294 \ub2e4\ub978 \ud074\ub798\uc2a4\uc5d0 \ube44\ud558\uc5ec \uc808\ub300\uc801\uc73c\ub85c \ubc29\uc5b4\ub825\uacfc \ubb3c\ub9ac \uacf5\uaca9\ub825\uc774 \ub0ae\uae30 \ub54c\ubb38\uc5d0, \ubaac\uc2a4\ud130\uc758 \uc811\uadfc\uc744 \ud5c8\uc6a9\ud560 \uacbd\uc6b0 \uc21c\uc2dd\uac04\uc5d0 \ucc2c \ubc14\ub2e5\uc5d0 \ub4dc\ub7ec\ub215\uac8c \ub420 \uac83\uc774\ub2e4. \uadf8\ub807\uae30 \ub54c\ubb38\uc5d0 \ub9c8\ub3c4\uc0ac\ub4e4\uc740 \uc804\ud22c\uc2dc \uae30\ubcf8\uc801\uc73c\ub85c \ub2e4\uc74c\uacfc \uac19\uc740 \uc804\uc220\uc744 \uc0ac\uc6a9\ud558\uac8c \ub41c\ub2e4."} {"question": "\ub378\ub77c\uc6e8\uc5b4\uc758 \uc608\uc5b8\uc790\ub85c \ubd88\ub9b0 \uc0ac\ub78c\uc740?", "paragraph": "\uc804\uc7c1\uc744 \ubc1c\ubc1c\ud558\uac8c \ud55c \ub610 \ub2e4\ub978 \uc694\uc778\uc73c\ub85c\ub294 1760\ub144\ub300 \ucd08\uae30 \uc778\ub514\uc5b8 \uc601\ud1a0\ub97c \ud729\uc4f8\uc5c8\ub358 \uc2e0\uc559\uc801\uc778 \uac01\uc131\uc774\uc5c8\ub2e4. \uc774 \uc6b4\ub3d9\uc740 \uc601\uad6d\uc5d0 \ub300\ud55c \ubd88\ub9cc\uacfc \ud568\uaed8 \uc2dd\ub7c9 \ubd80\uc871\uacfc \uc804\uc5fc\ubcd1\uc73c\ub85c\ub3c4 \uac00\uc18d\ud654\ub418\uc5c8\ub2e4. \uc774 \ud604\uc0c1\uc5d0\uc11c \uac00\uc7a5 \uc601\ud5a5\ub825\uc774 \uc788\uc5c8\ub358 \uc0ac\ub78c\uc740 \u201c\ub378\ub77c\uc6e8\uc5b4\uc758 \uc608\uc5b8\uc790\u201d\ub85c \uc54c\ub824\uc9c4 \ub124\uc624\ub9b0\uc774\uc5c8\ub2e4. \ub124\uc624\ub9b0\uc740 \uc778\ub514\uc5b8\ub4e4\uc5d0\uac8c \uad50\uc5ed \uc0c1\ud488\uc774\ub098 \uc54c\ucf54\uc62c \ubc0f \ubc31\uc778\uc758 \ubb34\uae30\ub97c \uba40\ub9ac\ud558\ub3c4\ub85d \uad8c\ud588\ub2e4. \ub124\uc624\ub9b0\uc740 \uc804\ud1b5\uc801\uc778 \uc778\ub514\uc5b8 \uc2e0\uc559\uc5d0 \uae30\ub3c5\uad50\uc801 \uc694\uc18c\ub97c \uc735\ud569\uc2dc\ucf1c, \uccad\uc911\uc744 \ud5a5\ud574 \ubc31\uc778\uc758 \uc545\uc2b5\uc73c\ub85c \ubb3c\ub4dc\ub294 \uc778\ub514\uc5b8\uc744 \ubcf4\uace0 \uc704\ub300\ud55c \uc601\ud63c\uc774 \ubd88\ucf8c\ud574 \ud558\uba70, \uc601\uad6d\uc774 \uc778\ub514\uc5b8\uc758 \uc874\uc7ac \uc790\uccb4\uc5d0 \uc704\ud611\uc774 \ub420 \uac83\uc774\ub77c\ub294 \ub9d0\uc744 \ub4e4\ub824 \uc92c\ub2e4. \ub124\uc624\ub9b0\uc740 \u201c\ub9cc\uc57d \ub108\ud76c\ub4e4\uc774 \uadf8 \uc548\uc5d0 \uc601\uad6d\uc778\uc744 \ub123\uc73c\uba74, \ub108\ud76c\ub4e4\uc740 \uc8fd\uc740 \uc790\uac00 \ub41c\ub2e4. \uc9c8\ubcd1, \ucc9c\uc5f0\ub450 \ubc0f \ub3c5(\uc54c\ucf54\uc62c)\uc774 \ub108\ud76c\ub4e4 \ubaa8\ub450\ub97c \ud30c\uad34\ud560 \uac83\u201d\uc774\ub77c\uace0 \ub9d0\ud588\ub2e4. \uc774\ub7ec\ud55c \uc8fc\uc7a5\uc740 \ud1b5\uc81c\ud560 \uc218 \uc5c6\ub294 \ud798\uc5d0 \uc758\ud574 \ubcc0\ud654\ud558\ub294 \uc138\uacc4 \uc18d\uc5d0 \uc788\ub358 \uc0ac\ub78c\ub4e4\uc5d0\uac8c\ub294 \uac15\ub825\ud55c \uba54\uc2dc\uc9c0\uac00 \ub418\uc5c8\ub2e4.", "answer": "\ub124\uc624\ub9b0", "sentence": "\uc774 \ud604\uc0c1\uc5d0\uc11c \uac00\uc7a5 \uc601\ud5a5\ub825\uc774 \uc788\uc5c8\ub358 \uc0ac\ub78c\uc740 \u201c\ub378\ub77c\uc6e8\uc5b4\uc758 \uc608\uc5b8\uc790\u201d\ub85c \uc54c\ub824\uc9c4 \ub124\uc624\ub9b0 \uc774\uc5c8\ub2e4.", "paragraph_sentence": "\uc804\uc7c1\uc744 \ubc1c\ubc1c\ud558\uac8c \ud55c \ub610 \ub2e4\ub978 \uc694\uc778\uc73c\ub85c\ub294 1760\ub144\ub300 \ucd08\uae30 \uc778\ub514\uc5b8 \uc601\ud1a0\ub97c \ud729\uc4f8\uc5c8\ub358 \uc2e0\uc559\uc801\uc778 \uac01\uc131\uc774\uc5c8\ub2e4. \uc774 \uc6b4\ub3d9\uc740 \uc601\uad6d\uc5d0 \ub300\ud55c \ubd88\ub9cc\uacfc \ud568\uaed8 \uc2dd\ub7c9 \ubd80\uc871\uacfc \uc804\uc5fc\ubcd1\uc73c\ub85c\ub3c4 \uac00\uc18d\ud654\ub418\uc5c8\ub2e4. \uc774 \ud604\uc0c1\uc5d0\uc11c \uac00\uc7a5 \uc601\ud5a5\ub825\uc774 \uc788\uc5c8\ub358 \uc0ac\ub78c\uc740 \u201c\ub378\ub77c\uc6e8\uc5b4\uc758 \uc608\uc5b8\uc790\u201d\ub85c \uc54c\ub824\uc9c4 \ub124\uc624\ub9b0 \uc774\uc5c8\ub2e4. \ub124\uc624\ub9b0\uc740 \uc778\ub514\uc5b8\ub4e4\uc5d0\uac8c \uad50\uc5ed \uc0c1\ud488\uc774\ub098 \uc54c\ucf54\uc62c \ubc0f \ubc31\uc778\uc758 \ubb34\uae30\ub97c \uba40\ub9ac\ud558\ub3c4\ub85d \uad8c\ud588\ub2e4. \ub124\uc624\ub9b0\uc740 \uc804\ud1b5\uc801\uc778 \uc778\ub514\uc5b8 \uc2e0\uc559\uc5d0 \uae30\ub3c5\uad50\uc801 \uc694\uc18c\ub97c \uc735\ud569\uc2dc\ucf1c, \uccad\uc911\uc744 \ud5a5\ud574 \ubc31\uc778\uc758 \uc545\uc2b5\uc73c\ub85c \ubb3c\ub4dc\ub294 \uc778\ub514\uc5b8\uc744 \ubcf4\uace0 \uc704\ub300\ud55c \uc601\ud63c\uc774 \ubd88\ucf8c\ud574 \ud558\uba70, \uc601\uad6d\uc774 \uc778\ub514\uc5b8\uc758 \uc874\uc7ac \uc790\uccb4\uc5d0 \uc704\ud611\uc774 \ub420 \uac83\uc774\ub77c\ub294 \ub9d0\uc744 \ub4e4\ub824 \uc92c\ub2e4. \ub124\uc624\ub9b0\uc740 \u201c\ub9cc\uc57d \ub108\ud76c\ub4e4\uc774 \uadf8 \uc548\uc5d0 \uc601\uad6d\uc778\uc744 \ub123\uc73c\uba74, \ub108\ud76c\ub4e4\uc740 \uc8fd\uc740 \uc790\uac00 \ub41c\ub2e4. \uc9c8\ubcd1, \ucc9c\uc5f0\ub450 \ubc0f \ub3c5(\uc54c\ucf54\uc62c)\uc774 \ub108\ud76c\ub4e4 \ubaa8\ub450\ub97c \ud30c\uad34\ud560 \uac83\u201d\uc774\ub77c\uace0 \ub9d0\ud588\ub2e4. \uc774\ub7ec\ud55c \uc8fc\uc7a5\uc740 \ud1b5\uc81c\ud560 \uc218 \uc5c6\ub294 \ud798\uc5d0 \uc758\ud574 \ubcc0\ud654\ud558\ub294 \uc138\uacc4 \uc18d\uc5d0 \uc788\ub358 \uc0ac\ub78c\ub4e4\uc5d0\uac8c\ub294 \uac15\ub825\ud55c \uba54\uc2dc\uc9c0\uac00 \ub418\uc5c8\ub2e4.", "paragraph_answer": "\uc804\uc7c1\uc744 \ubc1c\ubc1c\ud558\uac8c \ud55c \ub610 \ub2e4\ub978 \uc694\uc778\uc73c\ub85c\ub294 1760\ub144\ub300 \ucd08\uae30 \uc778\ub514\uc5b8 \uc601\ud1a0\ub97c \ud729\uc4f8\uc5c8\ub358 \uc2e0\uc559\uc801\uc778 \uac01\uc131\uc774\uc5c8\ub2e4. \uc774 \uc6b4\ub3d9\uc740 \uc601\uad6d\uc5d0 \ub300\ud55c \ubd88\ub9cc\uacfc \ud568\uaed8 \uc2dd\ub7c9 \ubd80\uc871\uacfc \uc804\uc5fc\ubcd1\uc73c\ub85c\ub3c4 \uac00\uc18d\ud654\ub418\uc5c8\ub2e4. \uc774 \ud604\uc0c1\uc5d0\uc11c \uac00\uc7a5 \uc601\ud5a5\ub825\uc774 \uc788\uc5c8\ub358 \uc0ac\ub78c\uc740 \u201c\ub378\ub77c\uc6e8\uc5b4\uc758 \uc608\uc5b8\uc790\u201d\ub85c \uc54c\ub824\uc9c4 \ub124\uc624\ub9b0 \uc774\uc5c8\ub2e4. \ub124\uc624\ub9b0\uc740 \uc778\ub514\uc5b8\ub4e4\uc5d0\uac8c \uad50\uc5ed \uc0c1\ud488\uc774\ub098 \uc54c\ucf54\uc62c \ubc0f \ubc31\uc778\uc758 \ubb34\uae30\ub97c \uba40\ub9ac\ud558\ub3c4\ub85d \uad8c\ud588\ub2e4. \ub124\uc624\ub9b0\uc740 \uc804\ud1b5\uc801\uc778 \uc778\ub514\uc5b8 \uc2e0\uc559\uc5d0 \uae30\ub3c5\uad50\uc801 \uc694\uc18c\ub97c \uc735\ud569\uc2dc\ucf1c, \uccad\uc911\uc744 \ud5a5\ud574 \ubc31\uc778\uc758 \uc545\uc2b5\uc73c\ub85c \ubb3c\ub4dc\ub294 \uc778\ub514\uc5b8\uc744 \ubcf4\uace0 \uc704\ub300\ud55c \uc601\ud63c\uc774 \ubd88\ucf8c\ud574 \ud558\uba70, \uc601\uad6d\uc774 \uc778\ub514\uc5b8\uc758 \uc874\uc7ac \uc790\uccb4\uc5d0 \uc704\ud611\uc774 \ub420 \uac83\uc774\ub77c\ub294 \ub9d0\uc744 \ub4e4\ub824 \uc92c\ub2e4. \ub124\uc624\ub9b0\uc740 \u201c\ub9cc\uc57d \ub108\ud76c\ub4e4\uc774 \uadf8 \uc548\uc5d0 \uc601\uad6d\uc778\uc744 \ub123\uc73c\uba74, \ub108\ud76c\ub4e4\uc740 \uc8fd\uc740 \uc790\uac00 \ub41c\ub2e4. \uc9c8\ubcd1, \ucc9c\uc5f0\ub450 \ubc0f \ub3c5(\uc54c\ucf54\uc62c)\uc774 \ub108\ud76c\ub4e4 \ubaa8\ub450\ub97c \ud30c\uad34\ud560 \uac83\u201d\uc774\ub77c\uace0 \ub9d0\ud588\ub2e4. \uc774\ub7ec\ud55c \uc8fc\uc7a5\uc740 \ud1b5\uc81c\ud560 \uc218 \uc5c6\ub294 \ud798\uc5d0 \uc758\ud574 \ubcc0\ud654\ud558\ub294 \uc138\uacc4 \uc18d\uc5d0 \uc788\ub358 \uc0ac\ub78c\ub4e4\uc5d0\uac8c\ub294 \uac15\ub825\ud55c \uba54\uc2dc\uc9c0\uac00 \ub418\uc5c8\ub2e4.", "sentence_answer": "\uc774 \ud604\uc0c1\uc5d0\uc11c \uac00\uc7a5 \uc601\ud5a5\ub825\uc774 \uc788\uc5c8\ub358 \uc0ac\ub78c\uc740 \u201c\ub378\ub77c\uc6e8\uc5b4\uc758 \uc608\uc5b8\uc790\u201d\ub85c \uc54c\ub824\uc9c4 \ub124\uc624\ub9b0 \uc774\uc5c8\ub2e4.", "paragraph_id": "6568011-7-0", "paragraph_question": "question: \ub378\ub77c\uc6e8\uc5b4\uc758 \uc608\uc5b8\uc790\ub85c \ubd88\ub9b0 \uc0ac\ub78c\uc740?, context: \uc804\uc7c1\uc744 \ubc1c\ubc1c\ud558\uac8c \ud55c \ub610 \ub2e4\ub978 \uc694\uc778\uc73c\ub85c\ub294 1760\ub144\ub300 \ucd08\uae30 \uc778\ub514\uc5b8 \uc601\ud1a0\ub97c \ud729\uc4f8\uc5c8\ub358 \uc2e0\uc559\uc801\uc778 \uac01\uc131\uc774\uc5c8\ub2e4. \uc774 \uc6b4\ub3d9\uc740 \uc601\uad6d\uc5d0 \ub300\ud55c \ubd88\ub9cc\uacfc \ud568\uaed8 \uc2dd\ub7c9 \ubd80\uc871\uacfc \uc804\uc5fc\ubcd1\uc73c\ub85c\ub3c4 \uac00\uc18d\ud654\ub418\uc5c8\ub2e4. \uc774 \ud604\uc0c1\uc5d0\uc11c \uac00\uc7a5 \uc601\ud5a5\ub825\uc774 \uc788\uc5c8\ub358 \uc0ac\ub78c\uc740 \u201c\ub378\ub77c\uc6e8\uc5b4\uc758 \uc608\uc5b8\uc790\u201d\ub85c \uc54c\ub824\uc9c4 \ub124\uc624\ub9b0\uc774\uc5c8\ub2e4. \ub124\uc624\ub9b0\uc740 \uc778\ub514\uc5b8\ub4e4\uc5d0\uac8c \uad50\uc5ed \uc0c1\ud488\uc774\ub098 \uc54c\ucf54\uc62c \ubc0f \ubc31\uc778\uc758 \ubb34\uae30\ub97c \uba40\ub9ac\ud558\ub3c4\ub85d \uad8c\ud588\ub2e4. \ub124\uc624\ub9b0\uc740 \uc804\ud1b5\uc801\uc778 \uc778\ub514\uc5b8 \uc2e0\uc559\uc5d0 \uae30\ub3c5\uad50\uc801 \uc694\uc18c\ub97c \uc735\ud569\uc2dc\ucf1c, \uccad\uc911\uc744 \ud5a5\ud574 \ubc31\uc778\uc758 \uc545\uc2b5\uc73c\ub85c \ubb3c\ub4dc\ub294 \uc778\ub514\uc5b8\uc744 \ubcf4\uace0 \uc704\ub300\ud55c \uc601\ud63c\uc774 \ubd88\ucf8c\ud574 \ud558\uba70, \uc601\uad6d\uc774 \uc778\ub514\uc5b8\uc758 \uc874\uc7ac \uc790\uccb4\uc5d0 \uc704\ud611\uc774 \ub420 \uac83\uc774\ub77c\ub294 \ub9d0\uc744 \ub4e4\ub824 \uc92c\ub2e4. \ub124\uc624\ub9b0\uc740 \u201c\ub9cc\uc57d \ub108\ud76c\ub4e4\uc774 \uadf8 \uc548\uc5d0 \uc601\uad6d\uc778\uc744 \ub123\uc73c\uba74, \ub108\ud76c\ub4e4\uc740 \uc8fd\uc740 \uc790\uac00 \ub41c\ub2e4. \uc9c8\ubcd1, \ucc9c\uc5f0\ub450 \ubc0f \ub3c5(\uc54c\ucf54\uc62c)\uc774 \ub108\ud76c\ub4e4 \ubaa8\ub450\ub97c \ud30c\uad34\ud560 \uac83\u201d\uc774\ub77c\uace0 \ub9d0\ud588\ub2e4. \uc774\ub7ec\ud55c \uc8fc\uc7a5\uc740 \ud1b5\uc81c\ud560 \uc218 \uc5c6\ub294 \ud798\uc5d0 \uc758\ud574 \ubcc0\ud654\ud558\ub294 \uc138\uacc4 \uc18d\uc5d0 \uc788\ub358 \uc0ac\ub78c\ub4e4\uc5d0\uac8c\ub294 \uac15\ub825\ud55c \uba54\uc2dc\uc9c0\uac00 \ub418\uc5c8\ub2e4."} {"question": "\uace0\ub9ac 1\ud638\uae30\uc5d0 \ub300\ud55c \uc218\uba85 \uad00\ub9ac 1\ub2e8\uacc4 \uc5f0\uad6c\uac00 \uc2dc\uc791\ub41c \ud574\ub294?", "paragraph": "\uace0\ub9ac1\ud638\uae30\ub294 2007\ub144 6\uc6d4 9\uc77c 30\ub144\uc778 \uc124\uacc4\uc218\uba85\uc744 \ub2e4\ud558\uc5ec \uac00\ub3d9\uc774 \uc911\ub2e8\ub418\uc5c8\uc73c\ub098 IAEA \uac80\uc99d\uacfc \uc9c0\uc5ed\uc0ac\ud68c \ud569\uc758 \ub4f1\uc744 \uac70\uccd0 \uc0c1\uc5c5\uc6b4\uc804\uc744 10\ub144 \uc5f0\uc7a5\ud558\uae30\ub85c \ud558\uc600\ub2e4. \uace0\ub9ac 1\ud638\uae30 \uacc4\uc18d\uc6b4\uc804 \ud0c0\ub2f9\uc131\uc740 1993\ub144 11\uc6d4\ubd80\ud130 1996\ub144 11\uc6d4\uae4c\uc9c0 \uc218\ud589\ub41c \uc6d0\uc804 \uacc4\uc18d\uc6b4\uc804\uc744 \uc704\ud55c \uc218\uba85\uad00\ub9ac 1\ub2e8\uacc4 \uc5f0\uad6c\ub97c \ud1a0\ub300\ub85c \uae30\uc220\uc801\u00b7\uacbd\uc81c\uc801\u00b7\uaddc\uc81c\uc801 \uad00\uc810\uc5d0\uc11c \ud3c9\uac00\ub418\uc5c8\ub2e4. \uc5f0\uad6c\uacb0\uacfc \uc801\uc808\ud55c \uacbd\ub144\uc5f4\ud654 \uad00\ub9ac\ub97c \ud1b5\ud55c \uc548\uc804\uc131\uc744 \ud655\ubcf4\ud558\uc600\uc73c\uba70, \uc124\uacc4\uc218\uba85 \uc774\ud6c4 20\ub144 \uc774\uc0c1\uc758 \uacc4\uc18d\uc6b4\uc804\uc740 \uae30\uc220\uc801\uc73c\ub85c \uac00\ub2a5\ud558\uba70 \uacbd\uc81c\uc801 \uc774\uc775\ub3c4 \ucda9\ubd84\ud55c \uac83\uc73c\ub85c \ub098\ud0c0\ub0ac\ub2e4. 1998\ub144 7\uc6d4\ubd80\ud130 2001\ub144 6\uc6d4\uae4c\uc9c0 \uc218\ud589\ub41c \uc218\uba85\uad00\ub9ac 2\ub2e8\uacc4 \uc5f0\uad6c\uc5d0\uc11c\ub294 \uacbd\ub144\uc5f4\ud654\ub41c \uacc4\ud1b5\u00b7\uad6c\uc870\ubb3c\u00b7\uae30\uae30\uc758 \uc218\uba85\ud3c9\uac00, \uc9c4\ub2e8 \ubc0f \uac10\uc2dc\ub97c \uc704\ud55c \uae30\uc220\uc744 \uac1c\ubc1c\ud558\uc600\ub2e4. \uacbd\ub144\uc5f4\ud654 \uad00\ub9ac \ud504\ub85c\uadf8\ub7a8\uc774 \ud6a8\uacfc\uc801\uc73c\ub85c \uc774\ud589\ub420 \ub54c \uacc4\uc18d\uc6b4\uc804 \uae30\uac04 \ub3d9\uc548\uc5d0\ub3c4 \ubc1c\uc804\uc18c\uc758 \uacc4\ud1b5\u00b7\uad6c\uc870\ubb3c\u00b7\uae30\uae30\uc758 \uace0\uc720\uae30\ub2a5\uacfc \uc548\uc804\uc131\uc774 \uc720\uc9c0\ub428\uc744 \ud655\uc778\ud558\uc600\ub2e4. \uc8fc\uae30\uc801\uc548\uc804\uc131\ud3c9\uac00\ub294 2000\ub144 5\uc6d4\ubd80\ud130 2002\ub144 11\uc6d4\uae4c\uc9c0 \uc218\ud589\ub418\uc5c8\uc73c\uba70 \uc2ec\uc0ac \uacb0\uacfc, \uace0\ub9ac1\ud638\uae30\ub294 \ub2f9\uc2dc \uc801\uc6a9\ub418\ub294 \uc720\ud6a8\ud55c \uae30\uc220\uae30\uc900\uc5d0 \uc801\ud569\ud558\uac8c \uc6b4\uc601\ub418\ub294 \uac83\uc73c\ub85c \uacc4\uc18d\uc6b4\uc804 \uac00\ub2a5\uc131\uc744 \ud655\ubcf4\ud558\uc600\ub2e4.", "answer": "1993\ub144", "sentence": "\uace0\ub9ac 1\ud638\uae30 \uacc4\uc18d\uc6b4\uc804 \ud0c0\ub2f9\uc131\uc740 1993\ub144 11\uc6d4\ubd80\ud130 1996\ub144 11\uc6d4\uae4c\uc9c0 \uc218\ud589\ub41c \uc6d0\uc804 \uacc4\uc18d\uc6b4\uc804\uc744 \uc704\ud55c \uc218\uba85\uad00\ub9ac 1\ub2e8\uacc4 \uc5f0\uad6c\ub97c \ud1a0\ub300\ub85c \uae30\uc220\uc801\u00b7\uacbd\uc81c\uc801\u00b7\uaddc\uc81c\uc801 \uad00\uc810\uc5d0\uc11c \ud3c9\uac00\ub418\uc5c8\ub2e4.", "paragraph_sentence": "\uace0\ub9ac1\ud638\uae30\ub294 2007\ub144 6\uc6d4 9\uc77c 30\ub144\uc778 \uc124\uacc4\uc218\uba85\uc744 \ub2e4\ud558\uc5ec \uac00\ub3d9\uc774 \uc911\ub2e8\ub418\uc5c8\uc73c\ub098 IAEA \uac80\uc99d\uacfc \uc9c0\uc5ed\uc0ac\ud68c \ud569\uc758 \ub4f1\uc744 \uac70\uccd0 \uc0c1\uc5c5\uc6b4\uc804\uc744 10\ub144 \uc5f0\uc7a5\ud558\uae30\ub85c \ud558\uc600\ub2e4. \uace0\ub9ac 1\ud638\uae30 \uacc4\uc18d\uc6b4\uc804 \ud0c0\ub2f9\uc131\uc740 1993\ub144 11\uc6d4\ubd80\ud130 1996\ub144 11\uc6d4\uae4c\uc9c0 \uc218\ud589\ub41c \uc6d0\uc804 \uacc4\uc18d\uc6b4\uc804\uc744 \uc704\ud55c \uc218\uba85\uad00\ub9ac 1\ub2e8\uacc4 \uc5f0\uad6c\ub97c \ud1a0\ub300\ub85c \uae30\uc220\uc801\u00b7\uacbd\uc81c\uc801\u00b7\uaddc\uc81c\uc801 \uad00\uc810\uc5d0\uc11c \ud3c9\uac00\ub418\uc5c8\ub2e4. \uc5f0\uad6c\uacb0\uacfc \uc801\uc808\ud55c \uacbd\ub144\uc5f4\ud654 \uad00\ub9ac\ub97c \ud1b5\ud55c \uc548\uc804\uc131\uc744 \ud655\ubcf4\ud558\uc600\uc73c\uba70, \uc124\uacc4\uc218\uba85 \uc774\ud6c4 20\ub144 \uc774\uc0c1\uc758 \uacc4\uc18d\uc6b4\uc804\uc740 \uae30\uc220\uc801\uc73c\ub85c \uac00\ub2a5\ud558\uba70 \uacbd\uc81c\uc801 \uc774\uc775\ub3c4 \ucda9\ubd84\ud55c \uac83\uc73c\ub85c \ub098\ud0c0\ub0ac\ub2e4. 1998\ub144 7\uc6d4\ubd80\ud130 2001\ub144 6\uc6d4\uae4c\uc9c0 \uc218\ud589\ub41c \uc218\uba85\uad00\ub9ac 2\ub2e8\uacc4 \uc5f0\uad6c\uc5d0\uc11c\ub294 \uacbd\ub144\uc5f4\ud654\ub41c \uacc4\ud1b5\u00b7\uad6c\uc870\ubb3c\u00b7\uae30\uae30\uc758 \uc218\uba85\ud3c9\uac00, \uc9c4\ub2e8 \ubc0f \uac10\uc2dc\ub97c \uc704\ud55c \uae30\uc220\uc744 \uac1c\ubc1c\ud558\uc600\ub2e4. \uacbd\ub144\uc5f4\ud654 \uad00\ub9ac \ud504\ub85c\uadf8\ub7a8\uc774 \ud6a8\uacfc\uc801\uc73c\ub85c \uc774\ud589\ub420 \ub54c \uacc4\uc18d\uc6b4\uc804 \uae30\uac04 \ub3d9\uc548\uc5d0\ub3c4 \ubc1c\uc804\uc18c\uc758 \uacc4\ud1b5\u00b7\uad6c\uc870\ubb3c\u00b7\uae30\uae30\uc758 \uace0\uc720\uae30\ub2a5\uacfc \uc548\uc804\uc131\uc774 \uc720\uc9c0\ub428\uc744 \ud655\uc778\ud558\uc600\ub2e4. \uc8fc\uae30\uc801\uc548\uc804\uc131\ud3c9\uac00\ub294 2000\ub144 5\uc6d4\ubd80\ud130 2002\ub144 11\uc6d4\uae4c\uc9c0 \uc218\ud589\ub418\uc5c8\uc73c\uba70 \uc2ec\uc0ac \uacb0\uacfc, \uace0\ub9ac1\ud638\uae30\ub294 \ub2f9\uc2dc \uc801\uc6a9\ub418\ub294 \uc720\ud6a8\ud55c \uae30\uc220\uae30\uc900\uc5d0 \uc801\ud569\ud558\uac8c \uc6b4\uc601\ub418\ub294 \uac83\uc73c\ub85c \uacc4\uc18d\uc6b4\uc804 \uac00\ub2a5\uc131\uc744 \ud655\ubcf4\ud558\uc600\ub2e4.", "paragraph_answer": "\uace0\ub9ac1\ud638\uae30\ub294 2007\ub144 6\uc6d4 9\uc77c 30\ub144\uc778 \uc124\uacc4\uc218\uba85\uc744 \ub2e4\ud558\uc5ec \uac00\ub3d9\uc774 \uc911\ub2e8\ub418\uc5c8\uc73c\ub098 IAEA \uac80\uc99d\uacfc \uc9c0\uc5ed\uc0ac\ud68c \ud569\uc758 \ub4f1\uc744 \uac70\uccd0 \uc0c1\uc5c5\uc6b4\uc804\uc744 10\ub144 \uc5f0\uc7a5\ud558\uae30\ub85c \ud558\uc600\ub2e4. \uace0\ub9ac 1\ud638\uae30 \uacc4\uc18d\uc6b4\uc804 \ud0c0\ub2f9\uc131\uc740 1993\ub144 11\uc6d4\ubd80\ud130 1996\ub144 11\uc6d4\uae4c\uc9c0 \uc218\ud589\ub41c \uc6d0\uc804 \uacc4\uc18d\uc6b4\uc804\uc744 \uc704\ud55c \uc218\uba85\uad00\ub9ac 1\ub2e8\uacc4 \uc5f0\uad6c\ub97c \ud1a0\ub300\ub85c \uae30\uc220\uc801\u00b7\uacbd\uc81c\uc801\u00b7\uaddc\uc81c\uc801 \uad00\uc810\uc5d0\uc11c \ud3c9\uac00\ub418\uc5c8\ub2e4. \uc5f0\uad6c\uacb0\uacfc \uc801\uc808\ud55c \uacbd\ub144\uc5f4\ud654 \uad00\ub9ac\ub97c \ud1b5\ud55c \uc548\uc804\uc131\uc744 \ud655\ubcf4\ud558\uc600\uc73c\uba70, \uc124\uacc4\uc218\uba85 \uc774\ud6c4 20\ub144 \uc774\uc0c1\uc758 \uacc4\uc18d\uc6b4\uc804\uc740 \uae30\uc220\uc801\uc73c\ub85c \uac00\ub2a5\ud558\uba70 \uacbd\uc81c\uc801 \uc774\uc775\ub3c4 \ucda9\ubd84\ud55c \uac83\uc73c\ub85c \ub098\ud0c0\ub0ac\ub2e4. 1998\ub144 7\uc6d4\ubd80\ud130 2001\ub144 6\uc6d4\uae4c\uc9c0 \uc218\ud589\ub41c \uc218\uba85\uad00\ub9ac 2\ub2e8\uacc4 \uc5f0\uad6c\uc5d0\uc11c\ub294 \uacbd\ub144\uc5f4\ud654\ub41c \uacc4\ud1b5\u00b7\uad6c\uc870\ubb3c\u00b7\uae30\uae30\uc758 \uc218\uba85\ud3c9\uac00, \uc9c4\ub2e8 \ubc0f \uac10\uc2dc\ub97c \uc704\ud55c \uae30\uc220\uc744 \uac1c\ubc1c\ud558\uc600\ub2e4. \uacbd\ub144\uc5f4\ud654 \uad00\ub9ac \ud504\ub85c\uadf8\ub7a8\uc774 \ud6a8\uacfc\uc801\uc73c\ub85c \uc774\ud589\ub420 \ub54c \uacc4\uc18d\uc6b4\uc804 \uae30\uac04 \ub3d9\uc548\uc5d0\ub3c4 \ubc1c\uc804\uc18c\uc758 \uacc4\ud1b5\u00b7\uad6c\uc870\ubb3c\u00b7\uae30\uae30\uc758 \uace0\uc720\uae30\ub2a5\uacfc \uc548\uc804\uc131\uc774 \uc720\uc9c0\ub428\uc744 \ud655\uc778\ud558\uc600\ub2e4. \uc8fc\uae30\uc801\uc548\uc804\uc131\ud3c9\uac00\ub294 2000\ub144 5\uc6d4\ubd80\ud130 2002\ub144 11\uc6d4\uae4c\uc9c0 \uc218\ud589\ub418\uc5c8\uc73c\uba70 \uc2ec\uc0ac \uacb0\uacfc, \uace0\ub9ac1\ud638\uae30\ub294 \ub2f9\uc2dc \uc801\uc6a9\ub418\ub294 \uc720\ud6a8\ud55c \uae30\uc220\uae30\uc900\uc5d0 \uc801\ud569\ud558\uac8c \uc6b4\uc601\ub418\ub294 \uac83\uc73c\ub85c \uacc4\uc18d\uc6b4\uc804 \uac00\ub2a5\uc131\uc744 \ud655\ubcf4\ud558\uc600\ub2e4.", "sentence_answer": "\uace0\ub9ac 1\ud638\uae30 \uacc4\uc18d\uc6b4\uc804 \ud0c0\ub2f9\uc131\uc740 1993\ub144 11\uc6d4\ubd80\ud130 1996\ub144 11\uc6d4\uae4c\uc9c0 \uc218\ud589\ub41c \uc6d0\uc804 \uacc4\uc18d\uc6b4\uc804\uc744 \uc704\ud55c \uc218\uba85\uad00\ub9ac 1\ub2e8\uacc4 \uc5f0\uad6c\ub97c \ud1a0\ub300\ub85c \uae30\uc220\uc801\u00b7\uacbd\uc81c\uc801\u00b7\uaddc\uc81c\uc801 \uad00\uc810\uc5d0\uc11c \ud3c9\uac00\ub418\uc5c8\ub2e4.", "paragraph_id": "6471365-1-2", "paragraph_question": "question: \uace0\ub9ac 1\ud638\uae30\uc5d0 \ub300\ud55c \uc218\uba85 \uad00\ub9ac 1\ub2e8\uacc4 \uc5f0\uad6c\uac00 \uc2dc\uc791\ub41c \ud574\ub294?, context: \uace0\ub9ac1\ud638\uae30\ub294 2007\ub144 6\uc6d4 9\uc77c 30\ub144\uc778 \uc124\uacc4\uc218\uba85\uc744 \ub2e4\ud558\uc5ec \uac00\ub3d9\uc774 \uc911\ub2e8\ub418\uc5c8\uc73c\ub098 IAEA \uac80\uc99d\uacfc \uc9c0\uc5ed\uc0ac\ud68c \ud569\uc758 \ub4f1\uc744 \uac70\uccd0 \uc0c1\uc5c5\uc6b4\uc804\uc744 10\ub144 \uc5f0\uc7a5\ud558\uae30\ub85c \ud558\uc600\ub2e4. \uace0\ub9ac 1\ud638\uae30 \uacc4\uc18d\uc6b4\uc804 \ud0c0\ub2f9\uc131\uc740 1993\ub144 11\uc6d4\ubd80\ud130 1996\ub144 11\uc6d4\uae4c\uc9c0 \uc218\ud589\ub41c \uc6d0\uc804 \uacc4\uc18d\uc6b4\uc804\uc744 \uc704\ud55c \uc218\uba85\uad00\ub9ac 1\ub2e8\uacc4 \uc5f0\uad6c\ub97c \ud1a0\ub300\ub85c \uae30\uc220\uc801\u00b7\uacbd\uc81c\uc801\u00b7\uaddc\uc81c\uc801 \uad00\uc810\uc5d0\uc11c \ud3c9\uac00\ub418\uc5c8\ub2e4. \uc5f0\uad6c\uacb0\uacfc \uc801\uc808\ud55c \uacbd\ub144\uc5f4\ud654 \uad00\ub9ac\ub97c \ud1b5\ud55c \uc548\uc804\uc131\uc744 \ud655\ubcf4\ud558\uc600\uc73c\uba70, \uc124\uacc4\uc218\uba85 \uc774\ud6c4 20\ub144 \uc774\uc0c1\uc758 \uacc4\uc18d\uc6b4\uc804\uc740 \uae30\uc220\uc801\uc73c\ub85c \uac00\ub2a5\ud558\uba70 \uacbd\uc81c\uc801 \uc774\uc775\ub3c4 \ucda9\ubd84\ud55c \uac83\uc73c\ub85c \ub098\ud0c0\ub0ac\ub2e4. 1998\ub144 7\uc6d4\ubd80\ud130 2001\ub144 6\uc6d4\uae4c\uc9c0 \uc218\ud589\ub41c \uc218\uba85\uad00\ub9ac 2\ub2e8\uacc4 \uc5f0\uad6c\uc5d0\uc11c\ub294 \uacbd\ub144\uc5f4\ud654\ub41c \uacc4\ud1b5\u00b7\uad6c\uc870\ubb3c\u00b7\uae30\uae30\uc758 \uc218\uba85\ud3c9\uac00, \uc9c4\ub2e8 \ubc0f \uac10\uc2dc\ub97c \uc704\ud55c \uae30\uc220\uc744 \uac1c\ubc1c\ud558\uc600\ub2e4. \uacbd\ub144\uc5f4\ud654 \uad00\ub9ac \ud504\ub85c\uadf8\ub7a8\uc774 \ud6a8\uacfc\uc801\uc73c\ub85c \uc774\ud589\ub420 \ub54c \uacc4\uc18d\uc6b4\uc804 \uae30\uac04 \ub3d9\uc548\uc5d0\ub3c4 \ubc1c\uc804\uc18c\uc758 \uacc4\ud1b5\u00b7\uad6c\uc870\ubb3c\u00b7\uae30\uae30\uc758 \uace0\uc720\uae30\ub2a5\uacfc \uc548\uc804\uc131\uc774 \uc720\uc9c0\ub428\uc744 \ud655\uc778\ud558\uc600\ub2e4. \uc8fc\uae30\uc801\uc548\uc804\uc131\ud3c9\uac00\ub294 2000\ub144 5\uc6d4\ubd80\ud130 2002\ub144 11\uc6d4\uae4c\uc9c0 \uc218\ud589\ub418\uc5c8\uc73c\uba70 \uc2ec\uc0ac \uacb0\uacfc, \uace0\ub9ac1\ud638\uae30\ub294 \ub2f9\uc2dc \uc801\uc6a9\ub418\ub294 \uc720\ud6a8\ud55c \uae30\uc220\uae30\uc900\uc5d0 \uc801\ud569\ud558\uac8c \uc6b4\uc601\ub418\ub294 \uac83\uc73c\ub85c \uacc4\uc18d\uc6b4\uc804 \uac00\ub2a5\uc131\uc744 \ud655\ubcf4\ud558\uc600\ub2e4."} {"question": "\uacbd\ucc30\uc774 \uc57d\ubb3c\uc911\ub3c5\uc774 \uc758\uc2ec\ub41c\ub2e4\uba70 \uc2a4\ud53c\uc5b4\uc2a4\ub97c \uc785\uc6d0\uc2dc\ud0a8 \ubcd1\uc6d0\uc740?", "paragraph": "2008\ub144 1\uc6d4 \uc2a4\ud53c\uc5b4\uc2a4\ub294 \ud328\ub354\ub77c\uc778\uc758 \ub300\ud45c\uc790\uc5d0\uac8c \uc591\uc721\uad8c\uc744 \ud3ec\uae30\ud558\uc9c0 \uc54a\ub294\ub2e4\uace0 \uc804\ud588\ub2e4. \uc2a4\ud53c\uc5b4\uc2a4 \uc9d1\uc5d0 \ubc29\ubb38\ud55c \uacbd\ucc30\uc740 \uc57d\ubb3c \uc911\ub3c5\uc774 \uc758\uc2ec\ub41c\ub2e4\uba70 \uc138\ub2e4\uc2a4 \uc2dc\ub098\uc774 \uc758\ub8cc\uc13c\ud130\uc5d0 \uc785\uc6d0\uc2dc\ucf30\ub2e4. \ub2e4\uc74c \ub0a0 \uc2a4\ud53c\uc5b4\uc2a4\uc758 \ubc29\ubb38\uad8c\uc740 \ubc95\uc6d0\uc758 \uacb0\uc815\uc5d0 \ub530\ub77c \uc911\ub2e8\ub418\uc5c8\uace0, \ud328\ub354\ub77c\uc778\ub9cc\uc774 \uc544\uc774\ub4e4\uc758 \ubb3c\ub9ac\uc801 \ubc0f \ubc95\uc801 \uc591\uc721\uad8c\uc744 \uac00\uc84c\ub2e4. \uc2a4\ud53c\uc5b4\uc2a4\ub294 \ub85c\ub110\ub4dc \ub808\uac74 UCLA \ubbf8\ub514\uceec \uc13c\ud130 \uc815\uc2e0\ubcd1\ub3d9\uc5d0\uc11c \uce58\ub8cc\uc5d0 \uc801\uadf9\uc801\uc73c\ub85c \uc784\ud588\ub2e4. \ubc95\uc6d0\uc740 \uc544\ubc84\uc9c0 \uc81c\uc784\uc2a4 \uc2a4\ud53c\uc5b4\uc2a4\uc640 \ubcc0\ud638\uc0ac \uc564\ub4dc\ub958 \uc6d4\ub9bf\uc744 \ucee8\uc11c\ubca0\uc774\ud130\uc27d\uc5d0 \uc784\uc2dc\ub85c \uad8c\ud55c\uc744 \uc92c\ub2e4. \uc774\ud6c4 \uc2a4\ud53c\uc5b4\uc2a4\uc758 \uc7ac\uc0b0\uc744 \uc644\uc804\ud788 \ud1b5\uc81c\ud558\uac8c \ub41c\ub2e4. \uc2a4\ud53c\uc5b4\uc2a4\ub294 5\uc77c \ud6c4 \ud480\ub824\ub0ac\ub2e4. \ud55c\ub2ec \ub4a4, \ub4dc\ub77c\ub9c8 \u300a\ub0b4\uac00 \uadf8\ub140\ub97c \ub9cc\ub0ac\uc744 \ub54c\u300b \uc5d0\ud53c\uc18c\ub4dc \"Ten Sessions\"\ud3b8\uc5d0\uc11c \uc811\uc218\uc6d0 \uc560\ube44 \uc5ed\uc73c\ub85c \ucd9c\uc5f0\ud588\ub2e4. \ub2f9\uc2dc \ucd5c\uace0 \uc2dc\uccad\ub960\uc744 \uae30\ub85d\ud588\uc744 \ubfd0\ub9cc \uc544\ub2c8\ub77c \uc5f0\uae30\uc5d0\uc11c\ub3c4 \uc88b\uc740 \ud3c9\uc744 \ub4e4\uc5c8\ub2e4. 2008\ub144 7\uc6d4 \ud328\ub354\ub77c\uc778\uacfc \ubcc0\ud638\uc0ac\uc758 \ub3d9\uc758\ud558\uc5d0 \ubc29\ubb38\uad8c\uc744 \ub418\ucc3e\uc558\ub2e4. 2008\ub144 9\uc6d4 7\uc77c \uc5f4\ub9b0 MTV \ube44\ub514\uc624 \ubba4\uc9c1 \uc5b4\uc6cc\ub4dc\uc5d0\uc11c \uc2a4\ud53c\uc5b4\uc2a4\ub294 \"Piece of Me\"\ub85c \ucd5c\uc6b0\uc218 \uc5ec\uc790 \ube44\ub514\uc624, \ucd5c\uc6b0\uc218 \ud31d \ube44\ub514\uc624 \uadf8\ub9ac\uace0 \uc62c\ud574\uc758 \ube44\ub514\uc624\uc0c1\uc744 \uc218\uc0c1\ud588\ub2e4. \uc774\ud6c4 \uc2a4\ud53c\uc5b4\uc2a4\uac00 \uc74c\uc545\uacc4\uc5d0 \ucef4\ubc31\ud55c \uac83\uc744 \uae30\ub150\uc73c\ub85c \uc5f0\ub300\uc21c\uc73c\ub85c \uc815\ub9ac\ud55c \ub2e4\ud050\uba58\ud130\ub9ac \u300aBritney: For the Record\u300b\uac00 \ubc29\uc601\ub418\uc5c8\ub2e4. \ud544 \uadf8\ub9ac\ud540\uc774 \uac10\ub3c5\uc744 \ub9e1\uc558\uc73c\uba70, 2008\ub144 3\ubd84\uae30\uc5d0 \ub274\uc695\uacfc \ube44\ubc8c\ub9ac\ud790\uc2a4\uc5d0\uc11c \ucd2c\uc601\ud588\ub2e4. \ub2e4\ud050\uba58\ud130\ub9ac\ub294 \uccab\ub0a0 \ubc24 MTV\uc5d0\uc11c \ub450 \ucc28\ub840 \ubc29\uc601\ub418\uc5c8\uace0, 560\ub9cc \uba85\uc758 \uc2dc\uccad\uc790 \uc218\ub97c \uae30\ub85d\ud588\ub2e4. \uc774\ub294 \ubc29\uc1a1 \uc5ed\uc0ac\uc0c1 \uc77c\uc694\uc77c \ubc24 \uc2dc\uac04\ub300\uc5d0 \uac00\uc7a5 \ub192\uc740 \uc2dc\uccad\ub960\uc774\ub2e4.", "answer": "\uc138\ub2e4\uc2a4 \uc2dc\ub098\uc774 \uc758\ub8cc\uc13c\ud130", "sentence": "\uc2a4\ud53c\uc5b4\uc2a4 \uc9d1\uc5d0 \ubc29\ubb38\ud55c \uacbd\ucc30\uc740 \uc57d\ubb3c \uc911\ub3c5\uc774 \uc758\uc2ec\ub41c\ub2e4\uba70 \uc138\ub2e4\uc2a4 \uc2dc\ub098\uc774 \uc758\ub8cc\uc13c\ud130 \uc5d0 \uc785\uc6d0\uc2dc\ucf30\ub2e4.", "paragraph_sentence": "2008\ub144 1\uc6d4 \uc2a4\ud53c\uc5b4\uc2a4\ub294 \ud328\ub354\ub77c\uc778\uc758 \ub300\ud45c\uc790\uc5d0\uac8c \uc591\uc721\uad8c\uc744 \ud3ec\uae30\ud558\uc9c0 \uc54a\ub294\ub2e4\uace0 \uc804\ud588\ub2e4. \uc2a4\ud53c\uc5b4\uc2a4 \uc9d1\uc5d0 \ubc29\ubb38\ud55c \uacbd\ucc30\uc740 \uc57d\ubb3c \uc911\ub3c5\uc774 \uc758\uc2ec\ub41c\ub2e4\uba70 \uc138\ub2e4\uc2a4 \uc2dc\ub098\uc774 \uc758\ub8cc\uc13c\ud130 \uc5d0 \uc785\uc6d0\uc2dc\ucf30\ub2e4. \ub2e4\uc74c \ub0a0 \uc2a4\ud53c\uc5b4\uc2a4\uc758 \ubc29\ubb38\uad8c\uc740 \ubc95\uc6d0\uc758 \uacb0\uc815\uc5d0 \ub530\ub77c \uc911\ub2e8\ub418\uc5c8\uace0, \ud328\ub354\ub77c\uc778\ub9cc\uc774 \uc544\uc774\ub4e4\uc758 \ubb3c\ub9ac\uc801 \ubc0f \ubc95\uc801 \uc591\uc721\uad8c\uc744 \uac00\uc84c\ub2e4. \uc2a4\ud53c\uc5b4\uc2a4\ub294 \ub85c\ub110\ub4dc \ub808\uac74 UCLA \ubbf8\ub514\uceec \uc13c\ud130 \uc815\uc2e0\ubcd1\ub3d9\uc5d0\uc11c \uce58\ub8cc\uc5d0 \uc801\uadf9\uc801\uc73c\ub85c \uc784\ud588\ub2e4. \ubc95\uc6d0\uc740 \uc544\ubc84\uc9c0 \uc81c\uc784\uc2a4 \uc2a4\ud53c\uc5b4\uc2a4\uc640 \ubcc0\ud638\uc0ac \uc564\ub4dc\ub958 \uc6d4\ub9bf\uc744 \ucee8\uc11c\ubca0\uc774\ud130\uc27d\uc5d0 \uc784\uc2dc\ub85c \uad8c\ud55c\uc744 \uc92c\ub2e4. \uc774\ud6c4 \uc2a4\ud53c\uc5b4\uc2a4\uc758 \uc7ac\uc0b0\uc744 \uc644\uc804\ud788 \ud1b5\uc81c\ud558\uac8c \ub41c\ub2e4. \uc2a4\ud53c\uc5b4\uc2a4\ub294 5\uc77c \ud6c4 \ud480\ub824\ub0ac\ub2e4. \ud55c\ub2ec \ub4a4, \ub4dc\ub77c\ub9c8 \u300a\ub0b4\uac00 \uadf8\ub140\ub97c \ub9cc\ub0ac\uc744 \ub54c\u300b \uc5d0\ud53c\uc18c\ub4dc \"Ten Sessions\"\ud3b8\uc5d0\uc11c \uc811\uc218\uc6d0 \uc560\ube44 \uc5ed\uc73c\ub85c \ucd9c\uc5f0\ud588\ub2e4. \ub2f9\uc2dc \ucd5c\uace0 \uc2dc\uccad\ub960\uc744 \uae30\ub85d\ud588\uc744 \ubfd0\ub9cc \uc544\ub2c8\ub77c \uc5f0\uae30\uc5d0\uc11c\ub3c4 \uc88b\uc740 \ud3c9\uc744 \ub4e4\uc5c8\ub2e4. 2008\ub144 7\uc6d4 \ud328\ub354\ub77c\uc778\uacfc \ubcc0\ud638\uc0ac\uc758 \ub3d9\uc758\ud558\uc5d0 \ubc29\ubb38\uad8c\uc744 \ub418\ucc3e\uc558\ub2e4. 2008\ub144 9\uc6d4 7\uc77c \uc5f4\ub9b0 MTV \ube44\ub514\uc624 \ubba4\uc9c1 \uc5b4\uc6cc\ub4dc\uc5d0\uc11c \uc2a4\ud53c\uc5b4\uc2a4\ub294 \"Piece of Me\"\ub85c \ucd5c\uc6b0\uc218 \uc5ec\uc790 \ube44\ub514\uc624, \ucd5c\uc6b0\uc218 \ud31d \ube44\ub514\uc624 \uadf8\ub9ac\uace0 \uc62c\ud574\uc758 \ube44\ub514\uc624\uc0c1\uc744 \uc218\uc0c1\ud588\ub2e4. \uc774\ud6c4 \uc2a4\ud53c\uc5b4\uc2a4\uac00 \uc74c\uc545\uacc4\uc5d0 \ucef4\ubc31\ud55c \uac83\uc744 \uae30\ub150\uc73c\ub85c \uc5f0\ub300\uc21c\uc73c\ub85c \uc815\ub9ac\ud55c \ub2e4\ud050\uba58\ud130\ub9ac \u300aBritney: For the Record\u300b\uac00 \ubc29\uc601\ub418\uc5c8\ub2e4. \ud544 \uadf8\ub9ac\ud540\uc774 \uac10\ub3c5\uc744 \ub9e1\uc558\uc73c\uba70, 2008\ub144 3\ubd84\uae30\uc5d0 \ub274\uc695\uacfc \ube44\ubc8c\ub9ac\ud790\uc2a4\uc5d0\uc11c \ucd2c\uc601\ud588\ub2e4. \ub2e4\ud050\uba58\ud130\ub9ac\ub294 \uccab\ub0a0 \ubc24 MTV\uc5d0\uc11c \ub450 \ucc28\ub840 \ubc29\uc601\ub418\uc5c8\uace0, 560\ub9cc \uba85\uc758 \uc2dc\uccad\uc790 \uc218\ub97c \uae30\ub85d\ud588\ub2e4. \uc774\ub294 \ubc29\uc1a1 \uc5ed\uc0ac\uc0c1 \uc77c\uc694\uc77c \ubc24 \uc2dc\uac04\ub300\uc5d0 \uac00\uc7a5 \ub192\uc740 \uc2dc\uccad\ub960\uc774\ub2e4.", "paragraph_answer": "2008\ub144 1\uc6d4 \uc2a4\ud53c\uc5b4\uc2a4\ub294 \ud328\ub354\ub77c\uc778\uc758 \ub300\ud45c\uc790\uc5d0\uac8c \uc591\uc721\uad8c\uc744 \ud3ec\uae30\ud558\uc9c0 \uc54a\ub294\ub2e4\uace0 \uc804\ud588\ub2e4. \uc2a4\ud53c\uc5b4\uc2a4 \uc9d1\uc5d0 \ubc29\ubb38\ud55c \uacbd\ucc30\uc740 \uc57d\ubb3c \uc911\ub3c5\uc774 \uc758\uc2ec\ub41c\ub2e4\uba70 \uc138\ub2e4\uc2a4 \uc2dc\ub098\uc774 \uc758\ub8cc\uc13c\ud130 \uc5d0 \uc785\uc6d0\uc2dc\ucf30\ub2e4. \ub2e4\uc74c \ub0a0 \uc2a4\ud53c\uc5b4\uc2a4\uc758 \ubc29\ubb38\uad8c\uc740 \ubc95\uc6d0\uc758 \uacb0\uc815\uc5d0 \ub530\ub77c \uc911\ub2e8\ub418\uc5c8\uace0, \ud328\ub354\ub77c\uc778\ub9cc\uc774 \uc544\uc774\ub4e4\uc758 \ubb3c\ub9ac\uc801 \ubc0f \ubc95\uc801 \uc591\uc721\uad8c\uc744 \uac00\uc84c\ub2e4. \uc2a4\ud53c\uc5b4\uc2a4\ub294 \ub85c\ub110\ub4dc \ub808\uac74 UCLA \ubbf8\ub514\uceec \uc13c\ud130 \uc815\uc2e0\ubcd1\ub3d9\uc5d0\uc11c \uce58\ub8cc\uc5d0 \uc801\uadf9\uc801\uc73c\ub85c \uc784\ud588\ub2e4. \ubc95\uc6d0\uc740 \uc544\ubc84\uc9c0 \uc81c\uc784\uc2a4 \uc2a4\ud53c\uc5b4\uc2a4\uc640 \ubcc0\ud638\uc0ac \uc564\ub4dc\ub958 \uc6d4\ub9bf\uc744 \ucee8\uc11c\ubca0\uc774\ud130\uc27d\uc5d0 \uc784\uc2dc\ub85c \uad8c\ud55c\uc744 \uc92c\ub2e4. \uc774\ud6c4 \uc2a4\ud53c\uc5b4\uc2a4\uc758 \uc7ac\uc0b0\uc744 \uc644\uc804\ud788 \ud1b5\uc81c\ud558\uac8c \ub41c\ub2e4. \uc2a4\ud53c\uc5b4\uc2a4\ub294 5\uc77c \ud6c4 \ud480\ub824\ub0ac\ub2e4. \ud55c\ub2ec \ub4a4, \ub4dc\ub77c\ub9c8 \u300a\ub0b4\uac00 \uadf8\ub140\ub97c \ub9cc\ub0ac\uc744 \ub54c\u300b \uc5d0\ud53c\uc18c\ub4dc \"Ten Sessions\"\ud3b8\uc5d0\uc11c \uc811\uc218\uc6d0 \uc560\ube44 \uc5ed\uc73c\ub85c \ucd9c\uc5f0\ud588\ub2e4. \ub2f9\uc2dc \ucd5c\uace0 \uc2dc\uccad\ub960\uc744 \uae30\ub85d\ud588\uc744 \ubfd0\ub9cc \uc544\ub2c8\ub77c \uc5f0\uae30\uc5d0\uc11c\ub3c4 \uc88b\uc740 \ud3c9\uc744 \ub4e4\uc5c8\ub2e4. 2008\ub144 7\uc6d4 \ud328\ub354\ub77c\uc778\uacfc \ubcc0\ud638\uc0ac\uc758 \ub3d9\uc758\ud558\uc5d0 \ubc29\ubb38\uad8c\uc744 \ub418\ucc3e\uc558\ub2e4. 2008\ub144 9\uc6d4 7\uc77c \uc5f4\ub9b0 MTV \ube44\ub514\uc624 \ubba4\uc9c1 \uc5b4\uc6cc\ub4dc\uc5d0\uc11c \uc2a4\ud53c\uc5b4\uc2a4\ub294 \"Piece of Me\"\ub85c \ucd5c\uc6b0\uc218 \uc5ec\uc790 \ube44\ub514\uc624, \ucd5c\uc6b0\uc218 \ud31d \ube44\ub514\uc624 \uadf8\ub9ac\uace0 \uc62c\ud574\uc758 \ube44\ub514\uc624\uc0c1\uc744 \uc218\uc0c1\ud588\ub2e4. \uc774\ud6c4 \uc2a4\ud53c\uc5b4\uc2a4\uac00 \uc74c\uc545\uacc4\uc5d0 \ucef4\ubc31\ud55c \uac83\uc744 \uae30\ub150\uc73c\ub85c \uc5f0\ub300\uc21c\uc73c\ub85c \uc815\ub9ac\ud55c \ub2e4\ud050\uba58\ud130\ub9ac \u300aBritney: For the Record\u300b\uac00 \ubc29\uc601\ub418\uc5c8\ub2e4. \ud544 \uadf8\ub9ac\ud540\uc774 \uac10\ub3c5\uc744 \ub9e1\uc558\uc73c\uba70, 2008\ub144 3\ubd84\uae30\uc5d0 \ub274\uc695\uacfc \ube44\ubc8c\ub9ac\ud790\uc2a4\uc5d0\uc11c \ucd2c\uc601\ud588\ub2e4. \ub2e4\ud050\uba58\ud130\ub9ac\ub294 \uccab\ub0a0 \ubc24 MTV\uc5d0\uc11c \ub450 \ucc28\ub840 \ubc29\uc601\ub418\uc5c8\uace0, 560\ub9cc \uba85\uc758 \uc2dc\uccad\uc790 \uc218\ub97c \uae30\ub85d\ud588\ub2e4. \uc774\ub294 \ubc29\uc1a1 \uc5ed\uc0ac\uc0c1 \uc77c\uc694\uc77c \ubc24 \uc2dc\uac04\ub300\uc5d0 \uac00\uc7a5 \ub192\uc740 \uc2dc\uccad\ub960\uc774\ub2e4.", "sentence_answer": "\uc2a4\ud53c\uc5b4\uc2a4 \uc9d1\uc5d0 \ubc29\ubb38\ud55c \uacbd\ucc30\uc740 \uc57d\ubb3c \uc911\ub3c5\uc774 \uc758\uc2ec\ub41c\ub2e4\uba70 \uc138\ub2e4\uc2a4 \uc2dc\ub098\uc774 \uc758\ub8cc\uc13c\ud130 \uc5d0 \uc785\uc6d0\uc2dc\ucf30\ub2e4.", "paragraph_id": "6509466-14-0", "paragraph_question": "question: \uacbd\ucc30\uc774 \uc57d\ubb3c\uc911\ub3c5\uc774 \uc758\uc2ec\ub41c\ub2e4\uba70 \uc2a4\ud53c\uc5b4\uc2a4\ub97c \uc785\uc6d0\uc2dc\ud0a8 \ubcd1\uc6d0\uc740?, context: 2008\ub144 1\uc6d4 \uc2a4\ud53c\uc5b4\uc2a4\ub294 \ud328\ub354\ub77c\uc778\uc758 \ub300\ud45c\uc790\uc5d0\uac8c \uc591\uc721\uad8c\uc744 \ud3ec\uae30\ud558\uc9c0 \uc54a\ub294\ub2e4\uace0 \uc804\ud588\ub2e4. \uc2a4\ud53c\uc5b4\uc2a4 \uc9d1\uc5d0 \ubc29\ubb38\ud55c \uacbd\ucc30\uc740 \uc57d\ubb3c \uc911\ub3c5\uc774 \uc758\uc2ec\ub41c\ub2e4\uba70 \uc138\ub2e4\uc2a4 \uc2dc\ub098\uc774 \uc758\ub8cc\uc13c\ud130\uc5d0 \uc785\uc6d0\uc2dc\ucf30\ub2e4. \ub2e4\uc74c \ub0a0 \uc2a4\ud53c\uc5b4\uc2a4\uc758 \ubc29\ubb38\uad8c\uc740 \ubc95\uc6d0\uc758 \uacb0\uc815\uc5d0 \ub530\ub77c \uc911\ub2e8\ub418\uc5c8\uace0, \ud328\ub354\ub77c\uc778\ub9cc\uc774 \uc544\uc774\ub4e4\uc758 \ubb3c\ub9ac\uc801 \ubc0f \ubc95\uc801 \uc591\uc721\uad8c\uc744 \uac00\uc84c\ub2e4. \uc2a4\ud53c\uc5b4\uc2a4\ub294 \ub85c\ub110\ub4dc \ub808\uac74 UCLA \ubbf8\ub514\uceec \uc13c\ud130 \uc815\uc2e0\ubcd1\ub3d9\uc5d0\uc11c \uce58\ub8cc\uc5d0 \uc801\uadf9\uc801\uc73c\ub85c \uc784\ud588\ub2e4. \ubc95\uc6d0\uc740 \uc544\ubc84\uc9c0 \uc81c\uc784\uc2a4 \uc2a4\ud53c\uc5b4\uc2a4\uc640 \ubcc0\ud638\uc0ac \uc564\ub4dc\ub958 \uc6d4\ub9bf\uc744 \ucee8\uc11c\ubca0\uc774\ud130\uc27d\uc5d0 \uc784\uc2dc\ub85c \uad8c\ud55c\uc744 \uc92c\ub2e4. \uc774\ud6c4 \uc2a4\ud53c\uc5b4\uc2a4\uc758 \uc7ac\uc0b0\uc744 \uc644\uc804\ud788 \ud1b5\uc81c\ud558\uac8c \ub41c\ub2e4. \uc2a4\ud53c\uc5b4\uc2a4\ub294 5\uc77c \ud6c4 \ud480\ub824\ub0ac\ub2e4. \ud55c\ub2ec \ub4a4, \ub4dc\ub77c\ub9c8 \u300a\ub0b4\uac00 \uadf8\ub140\ub97c \ub9cc\ub0ac\uc744 \ub54c\u300b \uc5d0\ud53c\uc18c\ub4dc \"Ten Sessions\"\ud3b8\uc5d0\uc11c \uc811\uc218\uc6d0 \uc560\ube44 \uc5ed\uc73c\ub85c \ucd9c\uc5f0\ud588\ub2e4. \ub2f9\uc2dc \ucd5c\uace0 \uc2dc\uccad\ub960\uc744 \uae30\ub85d\ud588\uc744 \ubfd0\ub9cc \uc544\ub2c8\ub77c \uc5f0\uae30\uc5d0\uc11c\ub3c4 \uc88b\uc740 \ud3c9\uc744 \ub4e4\uc5c8\ub2e4. 2008\ub144 7\uc6d4 \ud328\ub354\ub77c\uc778\uacfc \ubcc0\ud638\uc0ac\uc758 \ub3d9\uc758\ud558\uc5d0 \ubc29\ubb38\uad8c\uc744 \ub418\ucc3e\uc558\ub2e4. 2008\ub144 9\uc6d4 7\uc77c \uc5f4\ub9b0 MTV \ube44\ub514\uc624 \ubba4\uc9c1 \uc5b4\uc6cc\ub4dc\uc5d0\uc11c \uc2a4\ud53c\uc5b4\uc2a4\ub294 \"Piece of Me\"\ub85c \ucd5c\uc6b0\uc218 \uc5ec\uc790 \ube44\ub514\uc624, \ucd5c\uc6b0\uc218 \ud31d \ube44\ub514\uc624 \uadf8\ub9ac\uace0 \uc62c\ud574\uc758 \ube44\ub514\uc624\uc0c1\uc744 \uc218\uc0c1\ud588\ub2e4. \uc774\ud6c4 \uc2a4\ud53c\uc5b4\uc2a4\uac00 \uc74c\uc545\uacc4\uc5d0 \ucef4\ubc31\ud55c \uac83\uc744 \uae30\ub150\uc73c\ub85c \uc5f0\ub300\uc21c\uc73c\ub85c \uc815\ub9ac\ud55c \ub2e4\ud050\uba58\ud130\ub9ac \u300aBritney: For the Record\u300b\uac00 \ubc29\uc601\ub418\uc5c8\ub2e4. \ud544 \uadf8\ub9ac\ud540\uc774 \uac10\ub3c5\uc744 \ub9e1\uc558\uc73c\uba70, 2008\ub144 3\ubd84\uae30\uc5d0 \ub274\uc695\uacfc \ube44\ubc8c\ub9ac\ud790\uc2a4\uc5d0\uc11c \ucd2c\uc601\ud588\ub2e4. \ub2e4\ud050\uba58\ud130\ub9ac\ub294 \uccab\ub0a0 \ubc24 MTV\uc5d0\uc11c \ub450 \ucc28\ub840 \ubc29\uc601\ub418\uc5c8\uace0, 560\ub9cc \uba85\uc758 \uc2dc\uccad\uc790 \uc218\ub97c \uae30\ub85d\ud588\ub2e4. \uc774\ub294 \ubc29\uc1a1 \uc5ed\uc0ac\uc0c1 \uc77c\uc694\uc77c \ubc24 \uc2dc\uac04\ub300\uc5d0 \uac00\uc7a5 \ub192\uc740 \uc2dc\uccad\ub960\uc774\ub2e4."} {"question": "\ub7ec\uc2dc\uc544\uac00 CWC\uc5d0 \uac00\uc785\ud55c \uc5f0\ub3c4\ub294?", "paragraph": "\ub7ec\uc2dc\uc544\ub294 1997\ub144 CWC\uc5d0 \uac00\uc785\ud588\ub2e4. \uadf8\ub7f0\ub370 2002\ub144 \uccb4\uccb8 \ubc18\uad70\uc774 \ub7ec\uc2dc\uc544\uc758 \ubba4\uc9c0\uceec \uadf9\uc7a5\uc778 \ubaa8\uc2a4\ud06c\ubc14 \ubb38\ud654\ud68c\uad00(\ub3d4\ucfe8\ud2b8\ub974\uc774)\uc5d0 \uce68\uc785, \uac74\ubb3c\uc744 \uc810\ub839\ud558\uc5ec 850\uba85\uc758 \ub7ec\uc2dc\uc544\uc778\uc744 \uc778\uc9c8\ub85c \uc7a1\uc740 \uc0ac\uac74\uc774 \uc788\uc5c8\ub2e4. \uc778\uc9c8\uadf9 3\uc77c\uc9f8, \ub7ec\uc2dc\uc544 \ubcf4\uc548\uad70\uc740 \uac00\uc2a4\ub97c \uc0ac\uc6a9\ud558\uc5ec \uc778\uc9c8 \uc0ac\uac74\uc744 \uc9c4\uc555\ud558\uc600\uc73c\uba70, \uccb4\uccb8 \ubc18\uad70 50\uba85 \uc804\uc6d0\uacfc 120\uba85\uc758 \uc778\uc9c8\uc774 \uac00\uc2a4\ub85c \uc0ac\ub9dd\ud588\ub2e4\uace0 \uacf5\uc2dd \ubc1c\ud45c\ub418\uc5c8\ub2e4. \uc774\uc5d0 \ub300\ud574 \ubbf8\uad6d \uc815\ubd80\ub97c \ube44\ub86f\ud558\uc5ec \uc138\uacc4\uc758 \uc815\ubd80\uc640 \uc5b8\ub860\uc740 CWC\uc5d0\uc11c \uae08\uc9c0\ud558\ub294 \uc0ac\ub9b0 \uac00\uc2a4 \ub4f1 \ub3c5\uac00\uc2a4\ub97c \uc0ac\uc6a9\ud588\ub2e4\uace0 \uc758\uc2ec\ud558\uc600\ub2e4. \uadf8\ub7ec\ub098 2002\ub144 10\uc6d4 30\uc77c, \uc720\ub9ac \uc170\ube0c\uccb8\ucf54 \ub7ec\uc2dc\uc544 \ubcf4\uac74\uc7a5\uad00\uc740 \uc758\ub8cc\uc6a9 \ub9c8\ucde8\uac00\uc2a4\uc778 \ud39c\ud0c0\ub2d0\uc744 \uc0ac\uc6a9\ud588\ub2e4\uace0 \ubc1d\ud614\ub2e4. \ud39c\ud0c0\ub2d0\uc740 \ubaa8\ub974\ud540\ubcf4\ub2e4 \ud6a8\uacfc\uac00 \uc218\ubc31 \ubc30 \uc774\uc0c1 \uac15\ub825\ud55c \ubb3c\uc9c8\ub85c \uc218\uc220 \ub9c8\ucde8\uc81c\ub098 \uc554\ud658\uc790\ub97c \uc704\ud55c \uc9c4\ud1b5\uc81c\ub85c \uc4f0\uc774\uba70 \ub9c8\uc57d \ub300\uc6a9\uc73c\ub85c \uc0ac\uc6a9\ub418\uae30\ub3c4 \ud558\ub294\ub370, CWC\uc5d0\uc11c\ub294 \ud3ed\ub3d9\uc9c4\uc555\uc6a9\uc73c\ub85c \uc0ac\uc6a9\uc744 \ud5c8\uc6a9\ud558\uace0 \uc788\ub294 \ubb3c\uc9c8\ub85c \uc54c\ub824\uc84c\ub2e4. \uc774\uc5d0 \ubbf8\uad6d \ub4f1\uc758 \ubc18\uc751\uc740, \ub7ec\uc2dc\uc544\uac00 CWC\uc758 \ub9f9\uc810\uc744 \uc774\uc6a9\ud574 \uad50\ubb18\ud558\uac8c \ubc95\ub9dd\uc744 \ud68c\ud53c\ud558\uace0 \uc788\ub2e4\uba74\uc11c \ube44\ub09c\ud558\uc600\ub2e4.", "answer": "1997\ub144", "sentence": "\ub7ec\uc2dc\uc544\ub294 1997\ub144 CWC\uc5d0 \uac00\uc785\ud588\ub2e4.", "paragraph_sentence": " \ub7ec\uc2dc\uc544\ub294 1997\ub144 CWC\uc5d0 \uac00\uc785\ud588\ub2e4. \uadf8\ub7f0\ub370 2002\ub144 \uccb4\uccb8 \ubc18\uad70\uc774 \ub7ec\uc2dc\uc544\uc758 \ubba4\uc9c0\uceec \uadf9\uc7a5\uc778 \ubaa8\uc2a4\ud06c\ubc14 \ubb38\ud654\ud68c\uad00(\ub3d4\ucfe8\ud2b8\ub974\uc774)\uc5d0 \uce68\uc785, \uac74\ubb3c\uc744 \uc810\ub839\ud558\uc5ec 850\uba85\uc758 \ub7ec\uc2dc\uc544\uc778\uc744 \uc778\uc9c8\ub85c \uc7a1\uc740 \uc0ac\uac74\uc774 \uc788\uc5c8\ub2e4. \uc778\uc9c8\uadf9 3\uc77c\uc9f8, \ub7ec\uc2dc\uc544 \ubcf4\uc548\uad70\uc740 \uac00\uc2a4\ub97c \uc0ac\uc6a9\ud558\uc5ec \uc778\uc9c8 \uc0ac\uac74\uc744 \uc9c4\uc555\ud558\uc600\uc73c\uba70, \uccb4\uccb8 \ubc18\uad70 50\uba85 \uc804\uc6d0\uacfc 120\uba85\uc758 \uc778\uc9c8\uc774 \uac00\uc2a4\ub85c \uc0ac\ub9dd\ud588\ub2e4\uace0 \uacf5\uc2dd \ubc1c\ud45c\ub418\uc5c8\ub2e4. \uc774\uc5d0 \ub300\ud574 \ubbf8\uad6d \uc815\ubd80\ub97c \ube44\ub86f\ud558\uc5ec \uc138\uacc4\uc758 \uc815\ubd80\uc640 \uc5b8\ub860\uc740 CWC\uc5d0\uc11c \uae08\uc9c0\ud558\ub294 \uc0ac\ub9b0 \uac00\uc2a4 \ub4f1 \ub3c5\uac00\uc2a4\ub97c \uc0ac\uc6a9\ud588\ub2e4\uace0 \uc758\uc2ec\ud558\uc600\ub2e4. \uadf8\ub7ec\ub098 2002\ub144 10\uc6d4 30\uc77c, \uc720\ub9ac \uc170\ube0c\uccb8\ucf54 \ub7ec\uc2dc\uc544 \ubcf4\uac74\uc7a5\uad00\uc740 \uc758\ub8cc\uc6a9 \ub9c8\ucde8\uac00\uc2a4\uc778 \ud39c\ud0c0\ub2d0\uc744 \uc0ac\uc6a9\ud588\ub2e4\uace0 \ubc1d\ud614\ub2e4. \ud39c\ud0c0\ub2d0\uc740 \ubaa8\ub974\ud540\ubcf4\ub2e4 \ud6a8\uacfc\uac00 \uc218\ubc31 \ubc30 \uc774\uc0c1 \uac15\ub825\ud55c \ubb3c\uc9c8\ub85c \uc218\uc220 \ub9c8\ucde8\uc81c\ub098 \uc554\ud658\uc790\ub97c \uc704\ud55c \uc9c4\ud1b5\uc81c\ub85c \uc4f0\uc774\uba70 \ub9c8\uc57d \ub300\uc6a9\uc73c\ub85c \uc0ac\uc6a9\ub418\uae30\ub3c4 \ud558\ub294\ub370, CWC\uc5d0\uc11c\ub294 \ud3ed\ub3d9\uc9c4\uc555\uc6a9\uc73c\ub85c \uc0ac\uc6a9\uc744 \ud5c8\uc6a9\ud558\uace0 \uc788\ub294 \ubb3c\uc9c8\ub85c \uc54c\ub824\uc84c\ub2e4. \uc774\uc5d0 \ubbf8\uad6d \ub4f1\uc758 \ubc18\uc751\uc740, \ub7ec\uc2dc\uc544\uac00 CWC\uc758 \ub9f9\uc810\uc744 \uc774\uc6a9\ud574 \uad50\ubb18\ud558\uac8c \ubc95\ub9dd\uc744 \ud68c\ud53c\ud558\uace0 \uc788\ub2e4\uba74\uc11c \ube44\ub09c\ud558\uc600\ub2e4.", "paragraph_answer": "\ub7ec\uc2dc\uc544\ub294 1997\ub144 CWC\uc5d0 \uac00\uc785\ud588\ub2e4. \uadf8\ub7f0\ub370 2002\ub144 \uccb4\uccb8 \ubc18\uad70\uc774 \ub7ec\uc2dc\uc544\uc758 \ubba4\uc9c0\uceec \uadf9\uc7a5\uc778 \ubaa8\uc2a4\ud06c\ubc14 \ubb38\ud654\ud68c\uad00(\ub3d4\ucfe8\ud2b8\ub974\uc774)\uc5d0 \uce68\uc785, \uac74\ubb3c\uc744 \uc810\ub839\ud558\uc5ec 850\uba85\uc758 \ub7ec\uc2dc\uc544\uc778\uc744 \uc778\uc9c8\ub85c \uc7a1\uc740 \uc0ac\uac74\uc774 \uc788\uc5c8\ub2e4. \uc778\uc9c8\uadf9 3\uc77c\uc9f8, \ub7ec\uc2dc\uc544 \ubcf4\uc548\uad70\uc740 \uac00\uc2a4\ub97c \uc0ac\uc6a9\ud558\uc5ec \uc778\uc9c8 \uc0ac\uac74\uc744 \uc9c4\uc555\ud558\uc600\uc73c\uba70, \uccb4\uccb8 \ubc18\uad70 50\uba85 \uc804\uc6d0\uacfc 120\uba85\uc758 \uc778\uc9c8\uc774 \uac00\uc2a4\ub85c \uc0ac\ub9dd\ud588\ub2e4\uace0 \uacf5\uc2dd \ubc1c\ud45c\ub418\uc5c8\ub2e4. \uc774\uc5d0 \ub300\ud574 \ubbf8\uad6d \uc815\ubd80\ub97c \ube44\ub86f\ud558\uc5ec \uc138\uacc4\uc758 \uc815\ubd80\uc640 \uc5b8\ub860\uc740 CWC\uc5d0\uc11c \uae08\uc9c0\ud558\ub294 \uc0ac\ub9b0 \uac00\uc2a4 \ub4f1 \ub3c5\uac00\uc2a4\ub97c \uc0ac\uc6a9\ud588\ub2e4\uace0 \uc758\uc2ec\ud558\uc600\ub2e4. \uadf8\ub7ec\ub098 2002\ub144 10\uc6d4 30\uc77c, \uc720\ub9ac \uc170\ube0c\uccb8\ucf54 \ub7ec\uc2dc\uc544 \ubcf4\uac74\uc7a5\uad00\uc740 \uc758\ub8cc\uc6a9 \ub9c8\ucde8\uac00\uc2a4\uc778 \ud39c\ud0c0\ub2d0\uc744 \uc0ac\uc6a9\ud588\ub2e4\uace0 \ubc1d\ud614\ub2e4. \ud39c\ud0c0\ub2d0\uc740 \ubaa8\ub974\ud540\ubcf4\ub2e4 \ud6a8\uacfc\uac00 \uc218\ubc31 \ubc30 \uc774\uc0c1 \uac15\ub825\ud55c \ubb3c\uc9c8\ub85c \uc218\uc220 \ub9c8\ucde8\uc81c\ub098 \uc554\ud658\uc790\ub97c \uc704\ud55c \uc9c4\ud1b5\uc81c\ub85c \uc4f0\uc774\uba70 \ub9c8\uc57d \ub300\uc6a9\uc73c\ub85c \uc0ac\uc6a9\ub418\uae30\ub3c4 \ud558\ub294\ub370, CWC\uc5d0\uc11c\ub294 \ud3ed\ub3d9\uc9c4\uc555\uc6a9\uc73c\ub85c \uc0ac\uc6a9\uc744 \ud5c8\uc6a9\ud558\uace0 \uc788\ub294 \ubb3c\uc9c8\ub85c \uc54c\ub824\uc84c\ub2e4. \uc774\uc5d0 \ubbf8\uad6d \ub4f1\uc758 \ubc18\uc751\uc740, \ub7ec\uc2dc\uc544\uac00 CWC\uc758 \ub9f9\uc810\uc744 \uc774\uc6a9\ud574 \uad50\ubb18\ud558\uac8c \ubc95\ub9dd\uc744 \ud68c\ud53c\ud558\uace0 \uc788\ub2e4\uba74\uc11c \ube44\ub09c\ud558\uc600\ub2e4.", "sentence_answer": "\ub7ec\uc2dc\uc544\ub294 1997\ub144 CWC\uc5d0 \uac00\uc785\ud588\ub2e4.", "paragraph_id": "6572355-0-0", "paragraph_question": "question: \ub7ec\uc2dc\uc544\uac00 CWC\uc5d0 \uac00\uc785\ud55c \uc5f0\ub3c4\ub294?, context: \ub7ec\uc2dc\uc544\ub294 1997\ub144 CWC\uc5d0 \uac00\uc785\ud588\ub2e4. \uadf8\ub7f0\ub370 2002\ub144 \uccb4\uccb8 \ubc18\uad70\uc774 \ub7ec\uc2dc\uc544\uc758 \ubba4\uc9c0\uceec \uadf9\uc7a5\uc778 \ubaa8\uc2a4\ud06c\ubc14 \ubb38\ud654\ud68c\uad00(\ub3d4\ucfe8\ud2b8\ub974\uc774)\uc5d0 \uce68\uc785, \uac74\ubb3c\uc744 \uc810\ub839\ud558\uc5ec 850\uba85\uc758 \ub7ec\uc2dc\uc544\uc778\uc744 \uc778\uc9c8\ub85c \uc7a1\uc740 \uc0ac\uac74\uc774 \uc788\uc5c8\ub2e4. \uc778\uc9c8\uadf9 3\uc77c\uc9f8, \ub7ec\uc2dc\uc544 \ubcf4\uc548\uad70\uc740 \uac00\uc2a4\ub97c \uc0ac\uc6a9\ud558\uc5ec \uc778\uc9c8 \uc0ac\uac74\uc744 \uc9c4\uc555\ud558\uc600\uc73c\uba70, \uccb4\uccb8 \ubc18\uad70 50\uba85 \uc804\uc6d0\uacfc 120\uba85\uc758 \uc778\uc9c8\uc774 \uac00\uc2a4\ub85c \uc0ac\ub9dd\ud588\ub2e4\uace0 \uacf5\uc2dd \ubc1c\ud45c\ub418\uc5c8\ub2e4. \uc774\uc5d0 \ub300\ud574 \ubbf8\uad6d \uc815\ubd80\ub97c \ube44\ub86f\ud558\uc5ec \uc138\uacc4\uc758 \uc815\ubd80\uc640 \uc5b8\ub860\uc740 CWC\uc5d0\uc11c \uae08\uc9c0\ud558\ub294 \uc0ac\ub9b0 \uac00\uc2a4 \ub4f1 \ub3c5\uac00\uc2a4\ub97c \uc0ac\uc6a9\ud588\ub2e4\uace0 \uc758\uc2ec\ud558\uc600\ub2e4. \uadf8\ub7ec\ub098 2002\ub144 10\uc6d4 30\uc77c, \uc720\ub9ac \uc170\ube0c\uccb8\ucf54 \ub7ec\uc2dc\uc544 \ubcf4\uac74\uc7a5\uad00\uc740 \uc758\ub8cc\uc6a9 \ub9c8\ucde8\uac00\uc2a4\uc778 \ud39c\ud0c0\ub2d0\uc744 \uc0ac\uc6a9\ud588\ub2e4\uace0 \ubc1d\ud614\ub2e4. \ud39c\ud0c0\ub2d0\uc740 \ubaa8\ub974\ud540\ubcf4\ub2e4 \ud6a8\uacfc\uac00 \uc218\ubc31 \ubc30 \uc774\uc0c1 \uac15\ub825\ud55c \ubb3c\uc9c8\ub85c \uc218\uc220 \ub9c8\ucde8\uc81c\ub098 \uc554\ud658\uc790\ub97c \uc704\ud55c \uc9c4\ud1b5\uc81c\ub85c \uc4f0\uc774\uba70 \ub9c8\uc57d \ub300\uc6a9\uc73c\ub85c \uc0ac\uc6a9\ub418\uae30\ub3c4 \ud558\ub294\ub370, CWC\uc5d0\uc11c\ub294 \ud3ed\ub3d9\uc9c4\uc555\uc6a9\uc73c\ub85c \uc0ac\uc6a9\uc744 \ud5c8\uc6a9\ud558\uace0 \uc788\ub294 \ubb3c\uc9c8\ub85c \uc54c\ub824\uc84c\ub2e4. \uc774\uc5d0 \ubbf8\uad6d \ub4f1\uc758 \ubc18\uc751\uc740, \ub7ec\uc2dc\uc544\uac00 CWC\uc758 \ub9f9\uc810\uc744 \uc774\uc6a9\ud574 \uad50\ubb18\ud558\uac8c \ubc95\ub9dd\uc744 \ud68c\ud53c\ud558\uace0 \uc788\ub2e4\uba74\uc11c \ube44\ub09c\ud558\uc600\ub2e4."} {"question": "\ucf54\ub974\ub9c9 \ub9e5 \uc5d0\uc5b4\ud2b8\uac00 \ub291\ub300\ub85c \ubb18\uc0ac\ub41c \uac83\uc740 \ubb34\uc5c7\uc778\uac00?", "paragraph": "\ubd81\uc720\ub7fd \uc2e0\ud654\uc640 \uc77c\ubcf8 \uc2e0\ud654\uc5d0\uc11c \ub291\ub300\ub294 \uc2e0\uc73c\ub85c \uac00\uae5d\uac8c \ubb18\uc0ac\ub418\uc5c8\ub2e4. \uc77c\ubcf8\uc5d0\uc11c\ub294 \ub18d\ubd80\uac00 \uc2e0\uc0ac\uc5d0\uc11c \ub291\ub300\ub97c \uc22d\ubc30\ud558\uace0 \uad74 \uadfc\ucc98\uc5d0\uc11c \uba39\uc774\ub97c \uc8fc\uba70, \uc57c\uc0dd \ub9f7\ub3fc\uc9c0\uc640 \uc0ac\uc2b4\uc73c\ub85c\ubd80\ud130 \uc791\ubb3c\uc744 \ubcf4\ud638\ud558\uae30 \uc704\ud574\uc11c \ub291\ub300\uc5d0\uac8c \uac04\uccad\ud55c\ub2e4\ub294 \uc774\uc57c\uae30\uac00 \uc788\uace0, \ubd81\uc720\ub7fd \uc2e0\ud654\uc5d0\uc11c \ub098\uc624\ub294 \ud39c\ub9ac\ub974 \ub291\ub300\ub294 \ub85c\ud0a4\uc758 \uc544\ub4e4\ub85c \ubb18\uc0ac\ub418\uc5c8\ub2e4. \ub2e4\ub978 \ubb38\ud654\uc5d0\uc11c, \uc544\uc77c\ub79c\ub4dc \uc2e0\ud654\uc5d0\uc11c\ub294 \ucf54\ub974\ub9c9 \ub9e5 \uc5d0\uc5b4\ud2b8\uac00 \ub291\ub300\ub85c \ubb18\uc0ac\ub418\uba70, \ub85c\ub9c8 \uc2e0\ud654\uc5d0\uc11c\ub294 \uc554\ub291\ub300\uac00 \ub85c\ubb3c\ub8e8\uc2a4\uc640 \ub808\ubb34\uc2a4\ub97c \uae30\ub974\uba70 \ub85c\ub9c8\ub97c \uac74\uad6d\ud588\ub2e4\ub294 \ub4f1 \uc5ec\ub7ec \uae30\ucd08 \uc2e0\ud654\uc5d0\uc11c \ub291\ub300\ub97c \uc911\uc2ec \uc778\ubb3c\ub85c \ub450\uace0 \uc788\ub2e4. \ud280\ub974\ud06c \uc2e0\ud654,, \ubabd\uace8 \uc2e0\ud654, \uc544\uc774\ub204 \uc2e0\ud654\uc5d0\uc11c\ub294 \ub291\ub300\uac00 \uc790\uae30 \ubbfc\uc871\uc758 \uc870\uc0c1\uc73c\ub85c \ub098\uc628 \ubc18\uba74, \ub370\ub098\uc774\ub098 \ubbfc\uc871\uc740 \ub291\ub300\uac00 \ud55c \ub0a8\uc131\uc758 \ud615\ud0dc\uc774\uba70 \uc790\uc2e0\uc758 \ud615\uc81c\uc778 \uac83\uc73c\ub85c \ubbff\ub294\ub2e4. \ub291\ub300\ub294 \uace0\ub300 \uadf8\ub9ac\uc2a4\uc640 \uace0\ub300 \ub85c\ub9c8 \uc2e0\ud654\uc5d0\uc11c \uc544\ud3f4\ub860 \uc2e0\uacfc \ub291\ub300\uac00 \ud0dc\uc591\uc5d0 \uad00\ub828\ub418\uc5b4 \uc788\ub2e4\uace0 \uc124\uba85\ud558\uba70 \ubd81\uc720\ub7fd \uc2e0\ud654\uc5d0\uc11c\ub294 \ud0dc\uc591\uc758 \uc2e0\uc778 \uc2a4\ucf5c\uc774 \ub291\ub300\uc640 \uad00\ub828\ub418\ub294 \ub4f1 \uc77c\ubd80 \uc720\ub77c\uc2dc\uc544\uc758 \uc2e0\ud654\uc5d0\uc11c \ub291\ub300\uc640 \ud0dc\uc591\uc744 \uad00\ub828\uc2dc\ucf30\ub2e4. \ud30c\uc6e8\ub2c8 \ubbfc\uc871\uc758 \ucc3d\uc870 \uc124\ud654\uc5d0 \ub530\ub974\uba74 \ub291\ub300\ub294 \uc8fd\uc74c\uc744 \ucc98\uc74c\uc73c\ub85c \uacaa\uc740 \ub3d9\ubb3c\uc774\ub2e4. \ubd81\uc720\ub7fd \uc2e0\ud654\uc5d0\uc11c \ubcfc\ubc14 \ub9c8\ub140\uac00 \ud788\ub85c\ud0a8 \uac70\uc778\uacfc \ud790\ub2e4 \uac70\uc778\uc744 \ub291\ub300\ub85c \ub2e4\uc2a4\ub9ac\ub294 \uac83\uc73c\ub85c \ubb18\uc0ac\ud558\uace0, \ub098\ubc14\ud638 \uc871\uc740 \ub291\ub300\ub97c \ub9c8\ub140\ub85c \uc0dd\uac01\ud558\uace0 \ub450\ub824\uc6cc\ud558\ub294 \ub4f1 \uac00\ub054 \ubd81\uc720\ub7fd\uacfc \uc544\uba54\ub9ac\uce74\uc758 \uc77c\ubd80 \uc6d0\uc8fc\ubbfc \uc2e0\ud654\uc5d0\uc11c \ub291\ub300\ub294 \ubaa8\ub450 \ub9c8\ubc95\uacfc \uad00\ub828\ub41c \uac83\uc73c\ub85c \ubb18\uc0ac\ub41c\ub2e4. \ub9c8\ucc2c\uac00\uc9c0\ub85c, \uce60\ucf54\ud2f4 \uc871\uc740 \ub291\ub300\uac00 \uc815\uc2e0 \uc9c8\ud658\uacfc \uc0ac\ub9dd\uc744 \uc77c\uc73c\ud0ac \uc218 \uc788\ub2e4\uace0 \uc0dd\uac01\ud588\ub2e4.", "answer": "\uc544\uc77c\ub79c\ub4dc \uc2e0\ud654", "sentence": "\ub2e4\ub978 \ubb38\ud654\uc5d0\uc11c, \uc544\uc77c\ub79c\ub4dc \uc2e0\ud654 \uc5d0\uc11c\ub294 \ucf54\ub974\ub9c9 \ub9e5 \uc5d0\uc5b4\ud2b8\uac00 \ub291\ub300\ub85c \ubb18\uc0ac\ub418\uba70, \ub85c\ub9c8 \uc2e0\ud654\uc5d0\uc11c\ub294 \uc554\ub291\ub300\uac00 \ub85c\ubb3c\ub8e8\uc2a4\uc640 \ub808\ubb34\uc2a4\ub97c \uae30\ub974\uba70 \ub85c\ub9c8\ub97c \uac74\uad6d\ud588\ub2e4\ub294 \ub4f1 \uc5ec\ub7ec \uae30\ucd08 \uc2e0\ud654\uc5d0\uc11c \ub291\ub300\ub97c \uc911\uc2ec \uc778\ubb3c\ub85c \ub450\uace0 \uc788\ub2e4.", "paragraph_sentence": "\ubd81\uc720\ub7fd \uc2e0\ud654\uc640 \uc77c\ubcf8 \uc2e0\ud654\uc5d0\uc11c \ub291\ub300\ub294 \uc2e0\uc73c\ub85c \uac00\uae5d\uac8c \ubb18\uc0ac\ub418\uc5c8\ub2e4. \uc77c\ubcf8\uc5d0\uc11c\ub294 \ub18d\ubd80\uac00 \uc2e0\uc0ac\uc5d0\uc11c \ub291\ub300\ub97c \uc22d\ubc30\ud558\uace0 \uad74 \uadfc\ucc98\uc5d0\uc11c \uba39\uc774\ub97c \uc8fc\uba70, \uc57c\uc0dd \ub9f7\ub3fc\uc9c0\uc640 \uc0ac\uc2b4\uc73c\ub85c\ubd80\ud130 \uc791\ubb3c\uc744 \ubcf4\ud638\ud558\uae30 \uc704\ud574\uc11c \ub291\ub300\uc5d0\uac8c \uac04\uccad\ud55c\ub2e4\ub294 \uc774\uc57c\uae30\uac00 \uc788\uace0, \ubd81\uc720\ub7fd \uc2e0\ud654\uc5d0\uc11c \ub098\uc624\ub294 \ud39c\ub9ac\ub974 \ub291\ub300\ub294 \ub85c\ud0a4\uc758 \uc544\ub4e4\ub85c \ubb18\uc0ac\ub418\uc5c8\ub2e4. \ub2e4\ub978 \ubb38\ud654\uc5d0\uc11c, \uc544\uc77c\ub79c\ub4dc \uc2e0\ud654 \uc5d0\uc11c\ub294 \ucf54\ub974\ub9c9 \ub9e5 \uc5d0\uc5b4\ud2b8\uac00 \ub291\ub300\ub85c \ubb18\uc0ac\ub418\uba70, \ub85c\ub9c8 \uc2e0\ud654\uc5d0\uc11c\ub294 \uc554\ub291\ub300\uac00 \ub85c\ubb3c\ub8e8\uc2a4\uc640 \ub808\ubb34\uc2a4\ub97c \uae30\ub974\uba70 \ub85c\ub9c8\ub97c \uac74\uad6d\ud588\ub2e4\ub294 \ub4f1 \uc5ec\ub7ec \uae30\ucd08 \uc2e0\ud654\uc5d0\uc11c \ub291\ub300\ub97c \uc911\uc2ec \uc778\ubb3c\ub85c \ub450\uace0 \uc788\ub2e4. \ud280\ub974\ud06c \uc2e0\ud654,, \ubabd\uace8 \uc2e0\ud654, \uc544\uc774\ub204 \uc2e0\ud654\uc5d0\uc11c\ub294 \ub291\ub300\uac00 \uc790\uae30 \ubbfc\uc871\uc758 \uc870\uc0c1\uc73c\ub85c \ub098\uc628 \ubc18\uba74, \ub370\ub098\uc774\ub098 \ubbfc\uc871\uc740 \ub291\ub300\uac00 \ud55c \ub0a8\uc131\uc758 \ud615\ud0dc\uc774\uba70 \uc790\uc2e0\uc758 \ud615\uc81c\uc778 \uac83\uc73c\ub85c \ubbff\ub294\ub2e4. \ub291\ub300\ub294 \uace0\ub300 \uadf8\ub9ac\uc2a4\uc640 \uace0\ub300 \ub85c\ub9c8 \uc2e0\ud654\uc5d0\uc11c \uc544\ud3f4\ub860 \uc2e0\uacfc \ub291\ub300\uac00 \ud0dc\uc591\uc5d0 \uad00\ub828\ub418\uc5b4 \uc788\ub2e4\uace0 \uc124\uba85\ud558\uba70 \ubd81\uc720\ub7fd \uc2e0\ud654\uc5d0\uc11c\ub294 \ud0dc\uc591\uc758 \uc2e0\uc778 \uc2a4\ucf5c\uc774 \ub291\ub300\uc640 \uad00\ub828\ub418\ub294 \ub4f1 \uc77c\ubd80 \uc720\ub77c\uc2dc\uc544\uc758 \uc2e0\ud654\uc5d0\uc11c \ub291\ub300\uc640 \ud0dc\uc591\uc744 \uad00\ub828\uc2dc\ucf30\ub2e4. \ud30c\uc6e8\ub2c8 \ubbfc\uc871\uc758 \ucc3d\uc870 \uc124\ud654\uc5d0 \ub530\ub974\uba74 \ub291\ub300\ub294 \uc8fd\uc74c\uc744 \ucc98\uc74c\uc73c\ub85c \uacaa\uc740 \ub3d9\ubb3c\uc774\ub2e4. \ubd81\uc720\ub7fd \uc2e0\ud654\uc5d0\uc11c \ubcfc\ubc14 \ub9c8\ub140\uac00 \ud788\ub85c\ud0a8 \uac70\uc778\uacfc \ud790\ub2e4 \uac70\uc778\uc744 \ub291\ub300\ub85c \ub2e4\uc2a4\ub9ac\ub294 \uac83\uc73c\ub85c \ubb18\uc0ac\ud558\uace0, \ub098\ubc14\ud638 \uc871\uc740 \ub291\ub300\ub97c \ub9c8\ub140\ub85c \uc0dd\uac01\ud558\uace0 \ub450\ub824\uc6cc\ud558\ub294 \ub4f1 \uac00\ub054 \ubd81\uc720\ub7fd\uacfc \uc544\uba54\ub9ac\uce74\uc758 \uc77c\ubd80 \uc6d0\uc8fc\ubbfc \uc2e0\ud654\uc5d0\uc11c \ub291\ub300\ub294 \ubaa8\ub450 \ub9c8\ubc95\uacfc \uad00\ub828\ub41c \uac83\uc73c\ub85c \ubb18\uc0ac\ub41c\ub2e4. \ub9c8\ucc2c\uac00\uc9c0\ub85c, \uce60\ucf54\ud2f4 \uc871\uc740 \ub291\ub300\uac00 \uc815\uc2e0 \uc9c8\ud658\uacfc \uc0ac\ub9dd\uc744 \uc77c\uc73c\ud0ac \uc218 \uc788\ub2e4\uace0 \uc0dd\uac01\ud588\ub2e4.", "paragraph_answer": "\ubd81\uc720\ub7fd \uc2e0\ud654\uc640 \uc77c\ubcf8 \uc2e0\ud654\uc5d0\uc11c \ub291\ub300\ub294 \uc2e0\uc73c\ub85c \uac00\uae5d\uac8c \ubb18\uc0ac\ub418\uc5c8\ub2e4. \uc77c\ubcf8\uc5d0\uc11c\ub294 \ub18d\ubd80\uac00 \uc2e0\uc0ac\uc5d0\uc11c \ub291\ub300\ub97c \uc22d\ubc30\ud558\uace0 \uad74 \uadfc\ucc98\uc5d0\uc11c \uba39\uc774\ub97c \uc8fc\uba70, \uc57c\uc0dd \ub9f7\ub3fc\uc9c0\uc640 \uc0ac\uc2b4\uc73c\ub85c\ubd80\ud130 \uc791\ubb3c\uc744 \ubcf4\ud638\ud558\uae30 \uc704\ud574\uc11c \ub291\ub300\uc5d0\uac8c \uac04\uccad\ud55c\ub2e4\ub294 \uc774\uc57c\uae30\uac00 \uc788\uace0, \ubd81\uc720\ub7fd \uc2e0\ud654\uc5d0\uc11c \ub098\uc624\ub294 \ud39c\ub9ac\ub974 \ub291\ub300\ub294 \ub85c\ud0a4\uc758 \uc544\ub4e4\ub85c \ubb18\uc0ac\ub418\uc5c8\ub2e4. \ub2e4\ub978 \ubb38\ud654\uc5d0\uc11c, \uc544\uc77c\ub79c\ub4dc \uc2e0\ud654 \uc5d0\uc11c\ub294 \ucf54\ub974\ub9c9 \ub9e5 \uc5d0\uc5b4\ud2b8\uac00 \ub291\ub300\ub85c \ubb18\uc0ac\ub418\uba70, \ub85c\ub9c8 \uc2e0\ud654\uc5d0\uc11c\ub294 \uc554\ub291\ub300\uac00 \ub85c\ubb3c\ub8e8\uc2a4\uc640 \ub808\ubb34\uc2a4\ub97c \uae30\ub974\uba70 \ub85c\ub9c8\ub97c \uac74\uad6d\ud588\ub2e4\ub294 \ub4f1 \uc5ec\ub7ec \uae30\ucd08 \uc2e0\ud654\uc5d0\uc11c \ub291\ub300\ub97c \uc911\uc2ec \uc778\ubb3c\ub85c \ub450\uace0 \uc788\ub2e4. \ud280\ub974\ud06c \uc2e0\ud654,, \ubabd\uace8 \uc2e0\ud654, \uc544\uc774\ub204 \uc2e0\ud654\uc5d0\uc11c\ub294 \ub291\ub300\uac00 \uc790\uae30 \ubbfc\uc871\uc758 \uc870\uc0c1\uc73c\ub85c \ub098\uc628 \ubc18\uba74, \ub370\ub098\uc774\ub098 \ubbfc\uc871\uc740 \ub291\ub300\uac00 \ud55c \ub0a8\uc131\uc758 \ud615\ud0dc\uc774\uba70 \uc790\uc2e0\uc758 \ud615\uc81c\uc778 \uac83\uc73c\ub85c \ubbff\ub294\ub2e4. \ub291\ub300\ub294 \uace0\ub300 \uadf8\ub9ac\uc2a4\uc640 \uace0\ub300 \ub85c\ub9c8 \uc2e0\ud654\uc5d0\uc11c \uc544\ud3f4\ub860 \uc2e0\uacfc \ub291\ub300\uac00 \ud0dc\uc591\uc5d0 \uad00\ub828\ub418\uc5b4 \uc788\ub2e4\uace0 \uc124\uba85\ud558\uba70 \ubd81\uc720\ub7fd \uc2e0\ud654\uc5d0\uc11c\ub294 \ud0dc\uc591\uc758 \uc2e0\uc778 \uc2a4\ucf5c\uc774 \ub291\ub300\uc640 \uad00\ub828\ub418\ub294 \ub4f1 \uc77c\ubd80 \uc720\ub77c\uc2dc\uc544\uc758 \uc2e0\ud654\uc5d0\uc11c \ub291\ub300\uc640 \ud0dc\uc591\uc744 \uad00\ub828\uc2dc\ucf30\ub2e4. \ud30c\uc6e8\ub2c8 \ubbfc\uc871\uc758 \ucc3d\uc870 \uc124\ud654\uc5d0 \ub530\ub974\uba74 \ub291\ub300\ub294 \uc8fd\uc74c\uc744 \ucc98\uc74c\uc73c\ub85c \uacaa\uc740 \ub3d9\ubb3c\uc774\ub2e4. \ubd81\uc720\ub7fd \uc2e0\ud654\uc5d0\uc11c \ubcfc\ubc14 \ub9c8\ub140\uac00 \ud788\ub85c\ud0a8 \uac70\uc778\uacfc \ud790\ub2e4 \uac70\uc778\uc744 \ub291\ub300\ub85c \ub2e4\uc2a4\ub9ac\ub294 \uac83\uc73c\ub85c \ubb18\uc0ac\ud558\uace0, \ub098\ubc14\ud638 \uc871\uc740 \ub291\ub300\ub97c \ub9c8\ub140\ub85c \uc0dd\uac01\ud558\uace0 \ub450\ub824\uc6cc\ud558\ub294 \ub4f1 \uac00\ub054 \ubd81\uc720\ub7fd\uacfc \uc544\uba54\ub9ac\uce74\uc758 \uc77c\ubd80 \uc6d0\uc8fc\ubbfc \uc2e0\ud654\uc5d0\uc11c \ub291\ub300\ub294 \ubaa8\ub450 \ub9c8\ubc95\uacfc \uad00\ub828\ub41c \uac83\uc73c\ub85c \ubb18\uc0ac\ub41c\ub2e4. \ub9c8\ucc2c\uac00\uc9c0\ub85c, \uce60\ucf54\ud2f4 \uc871\uc740 \ub291\ub300\uac00 \uc815\uc2e0 \uc9c8\ud658\uacfc \uc0ac\ub9dd\uc744 \uc77c\uc73c\ud0ac \uc218 \uc788\ub2e4\uace0 \uc0dd\uac01\ud588\ub2e4.", "sentence_answer": "\ub2e4\ub978 \ubb38\ud654\uc5d0\uc11c, \uc544\uc77c\ub79c\ub4dc \uc2e0\ud654 \uc5d0\uc11c\ub294 \ucf54\ub974\ub9c9 \ub9e5 \uc5d0\uc5b4\ud2b8\uac00 \ub291\ub300\ub85c \ubb18\uc0ac\ub418\uba70, \ub85c\ub9c8 \uc2e0\ud654\uc5d0\uc11c\ub294 \uc554\ub291\ub300\uac00 \ub85c\ubb3c\ub8e8\uc2a4\uc640 \ub808\ubb34\uc2a4\ub97c \uae30\ub974\uba70 \ub85c\ub9c8\ub97c \uac74\uad6d\ud588\ub2e4\ub294 \ub4f1 \uc5ec\ub7ec \uae30\ucd08 \uc2e0\ud654\uc5d0\uc11c \ub291\ub300\ub97c \uc911\uc2ec \uc778\ubb3c\ub85c \ub450\uace0 \uc788\ub2e4.", "paragraph_id": "6572454-44-1", "paragraph_question": "question: \ucf54\ub974\ub9c9 \ub9e5 \uc5d0\uc5b4\ud2b8\uac00 \ub291\ub300\ub85c \ubb18\uc0ac\ub41c \uac83\uc740 \ubb34\uc5c7\uc778\uac00?, context: \ubd81\uc720\ub7fd \uc2e0\ud654\uc640 \uc77c\ubcf8 \uc2e0\ud654\uc5d0\uc11c \ub291\ub300\ub294 \uc2e0\uc73c\ub85c \uac00\uae5d\uac8c \ubb18\uc0ac\ub418\uc5c8\ub2e4. \uc77c\ubcf8\uc5d0\uc11c\ub294 \ub18d\ubd80\uac00 \uc2e0\uc0ac\uc5d0\uc11c \ub291\ub300\ub97c \uc22d\ubc30\ud558\uace0 \uad74 \uadfc\ucc98\uc5d0\uc11c \uba39\uc774\ub97c \uc8fc\uba70, \uc57c\uc0dd \ub9f7\ub3fc\uc9c0\uc640 \uc0ac\uc2b4\uc73c\ub85c\ubd80\ud130 \uc791\ubb3c\uc744 \ubcf4\ud638\ud558\uae30 \uc704\ud574\uc11c \ub291\ub300\uc5d0\uac8c \uac04\uccad\ud55c\ub2e4\ub294 \uc774\uc57c\uae30\uac00 \uc788\uace0, \ubd81\uc720\ub7fd \uc2e0\ud654\uc5d0\uc11c \ub098\uc624\ub294 \ud39c\ub9ac\ub974 \ub291\ub300\ub294 \ub85c\ud0a4\uc758 \uc544\ub4e4\ub85c \ubb18\uc0ac\ub418\uc5c8\ub2e4. \ub2e4\ub978 \ubb38\ud654\uc5d0\uc11c, \uc544\uc77c\ub79c\ub4dc \uc2e0\ud654\uc5d0\uc11c\ub294 \ucf54\ub974\ub9c9 \ub9e5 \uc5d0\uc5b4\ud2b8\uac00 \ub291\ub300\ub85c \ubb18\uc0ac\ub418\uba70, \ub85c\ub9c8 \uc2e0\ud654\uc5d0\uc11c\ub294 \uc554\ub291\ub300\uac00 \ub85c\ubb3c\ub8e8\uc2a4\uc640 \ub808\ubb34\uc2a4\ub97c \uae30\ub974\uba70 \ub85c\ub9c8\ub97c \uac74\uad6d\ud588\ub2e4\ub294 \ub4f1 \uc5ec\ub7ec \uae30\ucd08 \uc2e0\ud654\uc5d0\uc11c \ub291\ub300\ub97c \uc911\uc2ec \uc778\ubb3c\ub85c \ub450\uace0 \uc788\ub2e4. \ud280\ub974\ud06c \uc2e0\ud654,, \ubabd\uace8 \uc2e0\ud654, \uc544\uc774\ub204 \uc2e0\ud654\uc5d0\uc11c\ub294 \ub291\ub300\uac00 \uc790\uae30 \ubbfc\uc871\uc758 \uc870\uc0c1\uc73c\ub85c \ub098\uc628 \ubc18\uba74, \ub370\ub098\uc774\ub098 \ubbfc\uc871\uc740 \ub291\ub300\uac00 \ud55c \ub0a8\uc131\uc758 \ud615\ud0dc\uc774\uba70 \uc790\uc2e0\uc758 \ud615\uc81c\uc778 \uac83\uc73c\ub85c \ubbff\ub294\ub2e4. \ub291\ub300\ub294 \uace0\ub300 \uadf8\ub9ac\uc2a4\uc640 \uace0\ub300 \ub85c\ub9c8 \uc2e0\ud654\uc5d0\uc11c \uc544\ud3f4\ub860 \uc2e0\uacfc \ub291\ub300\uac00 \ud0dc\uc591\uc5d0 \uad00\ub828\ub418\uc5b4 \uc788\ub2e4\uace0 \uc124\uba85\ud558\uba70 \ubd81\uc720\ub7fd \uc2e0\ud654\uc5d0\uc11c\ub294 \ud0dc\uc591\uc758 \uc2e0\uc778 \uc2a4\ucf5c\uc774 \ub291\ub300\uc640 \uad00\ub828\ub418\ub294 \ub4f1 \uc77c\ubd80 \uc720\ub77c\uc2dc\uc544\uc758 \uc2e0\ud654\uc5d0\uc11c \ub291\ub300\uc640 \ud0dc\uc591\uc744 \uad00\ub828\uc2dc\ucf30\ub2e4. \ud30c\uc6e8\ub2c8 \ubbfc\uc871\uc758 \ucc3d\uc870 \uc124\ud654\uc5d0 \ub530\ub974\uba74 \ub291\ub300\ub294 \uc8fd\uc74c\uc744 \ucc98\uc74c\uc73c\ub85c \uacaa\uc740 \ub3d9\ubb3c\uc774\ub2e4. \ubd81\uc720\ub7fd \uc2e0\ud654\uc5d0\uc11c \ubcfc\ubc14 \ub9c8\ub140\uac00 \ud788\ub85c\ud0a8 \uac70\uc778\uacfc \ud790\ub2e4 \uac70\uc778\uc744 \ub291\ub300\ub85c \ub2e4\uc2a4\ub9ac\ub294 \uac83\uc73c\ub85c \ubb18\uc0ac\ud558\uace0, \ub098\ubc14\ud638 \uc871\uc740 \ub291\ub300\ub97c \ub9c8\ub140\ub85c \uc0dd\uac01\ud558\uace0 \ub450\ub824\uc6cc\ud558\ub294 \ub4f1 \uac00\ub054 \ubd81\uc720\ub7fd\uacfc \uc544\uba54\ub9ac\uce74\uc758 \uc77c\ubd80 \uc6d0\uc8fc\ubbfc \uc2e0\ud654\uc5d0\uc11c \ub291\ub300\ub294 \ubaa8\ub450 \ub9c8\ubc95\uacfc \uad00\ub828\ub41c \uac83\uc73c\ub85c \ubb18\uc0ac\ub41c\ub2e4. \ub9c8\ucc2c\uac00\uc9c0\ub85c, \uce60\ucf54\ud2f4 \uc871\uc740 \ub291\ub300\uac00 \uc815\uc2e0 \uc9c8\ud658\uacfc \uc0ac\ub9dd\uc744 \uc77c\uc73c\ud0ac \uc218 \uc788\ub2e4\uace0 \uc0dd\uac01\ud588\ub2e4."} {"question": "\uc778\ucc9c\uad11\uc5ed\uc2dc \uc911\uad6c\uc5d0 \uc704\uce58\ud55c \ucc28\uc774\ub098\ud0c0\uc6b4\uc5d0\ub294 \ud604\uc7ac \uba87 \uba85\uc758 \ud654\uad50\ub9cc\uc774 \ub0a8\uc544\uc788\ub294\uac00?", "paragraph": "\uc2e4\uc81c\ub85c \ud654\ud3d0\uac1c\ud601 \uc774\ud6c4 \ud654\uad50\ub4e4\uc758 \uc790\ubcf8\ub825\uc740 \uc0c1\ub2f9\ud55c \ud0c0\uaca9\uc744 \uc785\uc5c8\uc73c\uba70 \uc0c1\ub2f9\uc218\uc758 \ud654\uad50\ub4e4\uc740 \ud55c\uad6d\uc744 \ub5a0\ub0ac\uace0 \uc790\uc5f0\ud788 \uc678\uc2dd\uc5c5\uc5d0 \uc9c4\ucd9c\ud558\ub294 \ud654\uad50\uac00 \ub298\uc5b4\ub0ac\uc73c\ub098 \ub300\ud1b5\ub839 \ucde8\uc784 \ub4a4\uc778 1976\ub144\uc5d0\ub294 \ud654\uad50\uc5d0 \ub300\ud55c \uad50\uc721\uad8c\uacfc \uc7ac\uc0b0\uad8c\uc744 \ubc15\ud0c8\ud558\uc5ec \ud55c\uad6d \ub0b4\uc5d0\uc11c\uc758 \uc678\uad6d\uc778\uacfc \uc678\uad6d \uc790\ubcf8\uc758 \uacbd\uc81c \uc7a5\uc545\ub825\uc744 \uc5b5\uc81c\ud558\uae30\ub3c4 \ud558\uc600\ub2e4. \uc778\ucc9c\uad11\uc5ed\uc2dc \uc911\uad6c\uc5d0 \uc704\uce58\ud55c 1\ub9cc 8000\ud3c9\uc758 \ucc28\uc774\ub098\ud0c0\uc6b4\uc5d0\ub294 \ud55c\ub54c \ud654\uad50 5,000\uc5ec \uba85\uc774 \uac70\uc8fc\ud588\uc73c\ub098 \ubc15\uc815\ud76c \uc815\uad8c \uc774\ud6c4 \ud654\uad50\uc758 \uc7ac\uc0b0\uad8c \ud589\uc0ac\ub97c \uc81c\ud55c\ud558\ub294 \uc815\ucc45\uc5d0 \ubd88\ub9cc\uc744 \ud488\uace0 \ubbf8\uad6d, \ub3d9\ub0a8\uc544 \ub4f1\uc73c\ub85c \ub5a0\ub098 \ud604\uc7ac\ub294 500\uc5ec \uba85\ub9cc\uc774 \ub0a8\uc544\uc788\ub2e4. \uadf8\ub7ec\ub098, \ub2f9\uc2dc \ud55c\uad6d\uc758 \uae08\uc735 \uc0c1\uc5c5\uc801 \uacbd\uc81c \uad6c\uc870\ub97c \ubbf8\ucc98 \ud30c\uc545\ud558\uc9c0 \ubabb\ud55c, \ud1b5\ud654 \uac1c\ud601 \uc815\ucc45\uc740 \uc608\uae08 \ub3d9\uacb0 \uc870\uce58\ub97c \ubd88\ub7ec\uc654\uace0, \uc804\uccb4 \uacf5\uc7a5\uc758 45%\ub294 \uac00\ub3d9\uc744 \uc911\uc9c0\ud558\ub294 \ub4f1, \uc0ac\ud0dc\uac00 \uc545\ud654\ub418\uc5c8\ub2e4.", "answer": "500\uc5ec \uba85", "sentence": "\uc778\ucc9c\uad11\uc5ed\uc2dc \uc911\uad6c\uc5d0 \uc704\uce58\ud55c 1\ub9cc 8000\ud3c9\uc758 \ucc28\uc774\ub098\ud0c0\uc6b4\uc5d0\ub294 \ud55c\ub54c \ud654\uad50 5,000\uc5ec \uba85\uc774 \uac70\uc8fc\ud588\uc73c\ub098 \ubc15\uc815\ud76c \uc815\uad8c \uc774\ud6c4 \ud654\uad50\uc758 \uc7ac\uc0b0\uad8c \ud589\uc0ac\ub97c \uc81c\ud55c\ud558\ub294 \uc815\ucc45\uc5d0 \ubd88\ub9cc\uc744 \ud488\uace0 \ubbf8\uad6d, \ub3d9\ub0a8\uc544 \ub4f1\uc73c\ub85c \ub5a0\ub098 \ud604\uc7ac\ub294 500\uc5ec \uba85 \ub9cc\uc774 \ub0a8\uc544\uc788\ub2e4.", "paragraph_sentence": "\uc2e4\uc81c\ub85c \ud654\ud3d0\uac1c\ud601 \uc774\ud6c4 \ud654\uad50\ub4e4\uc758 \uc790\ubcf8\ub825\uc740 \uc0c1\ub2f9\ud55c \ud0c0\uaca9\uc744 \uc785\uc5c8\uc73c\uba70 \uc0c1\ub2f9\uc218\uc758 \ud654\uad50\ub4e4\uc740 \ud55c\uad6d\uc744 \ub5a0\ub0ac\uace0 \uc790\uc5f0\ud788 \uc678\uc2dd\uc5c5\uc5d0 \uc9c4\ucd9c\ud558\ub294 \ud654\uad50\uac00 \ub298\uc5b4\ub0ac\uc73c\ub098 \ub300\ud1b5\ub839 \ucde8\uc784 \ub4a4\uc778 1976\ub144\uc5d0\ub294 \ud654\uad50\uc5d0 \ub300\ud55c \uad50\uc721\uad8c\uacfc \uc7ac\uc0b0\uad8c\uc744 \ubc15\ud0c8\ud558\uc5ec \ud55c\uad6d \ub0b4\uc5d0\uc11c\uc758 \uc678\uad6d\uc778\uacfc \uc678\uad6d \uc790\ubcf8\uc758 \uacbd\uc81c \uc7a5\uc545\ub825\uc744 \uc5b5\uc81c\ud558\uae30\ub3c4 \ud558\uc600\ub2e4. \uc778\ucc9c\uad11\uc5ed\uc2dc \uc911\uad6c\uc5d0 \uc704\uce58\ud55c 1\ub9cc 8000\ud3c9\uc758 \ucc28\uc774\ub098\ud0c0\uc6b4\uc5d0\ub294 \ud55c\ub54c \ud654\uad50 5,000\uc5ec \uba85\uc774 \uac70\uc8fc\ud588\uc73c\ub098 \ubc15\uc815\ud76c \uc815\uad8c \uc774\ud6c4 \ud654\uad50\uc758 \uc7ac\uc0b0\uad8c \ud589\uc0ac\ub97c \uc81c\ud55c\ud558\ub294 \uc815\ucc45\uc5d0 \ubd88\ub9cc\uc744 \ud488\uace0 \ubbf8\uad6d, \ub3d9\ub0a8\uc544 \ub4f1\uc73c\ub85c \ub5a0\ub098 \ud604\uc7ac\ub294 500\uc5ec \uba85 \ub9cc\uc774 \ub0a8\uc544\uc788\ub2e4. \uadf8\ub7ec\ub098, \ub2f9\uc2dc \ud55c\uad6d\uc758 \uae08\uc735 \uc0c1\uc5c5\uc801 \uacbd\uc81c \uad6c\uc870\ub97c \ubbf8\ucc98 \ud30c\uc545\ud558\uc9c0 \ubabb\ud55c, \ud1b5\ud654 \uac1c\ud601 \uc815\ucc45\uc740 \uc608\uae08 \ub3d9\uacb0 \uc870\uce58\ub97c \ubd88\ub7ec\uc654\uace0, \uc804\uccb4 \uacf5\uc7a5\uc758 45%\ub294 \uac00\ub3d9\uc744 \uc911\uc9c0\ud558\ub294 \ub4f1, \uc0ac\ud0dc\uac00 \uc545\ud654\ub418\uc5c8\ub2e4.", "paragraph_answer": "\uc2e4\uc81c\ub85c \ud654\ud3d0\uac1c\ud601 \uc774\ud6c4 \ud654\uad50\ub4e4\uc758 \uc790\ubcf8\ub825\uc740 \uc0c1\ub2f9\ud55c \ud0c0\uaca9\uc744 \uc785\uc5c8\uc73c\uba70 \uc0c1\ub2f9\uc218\uc758 \ud654\uad50\ub4e4\uc740 \ud55c\uad6d\uc744 \ub5a0\ub0ac\uace0 \uc790\uc5f0\ud788 \uc678\uc2dd\uc5c5\uc5d0 \uc9c4\ucd9c\ud558\ub294 \ud654\uad50\uac00 \ub298\uc5b4\ub0ac\uc73c\ub098 \ub300\ud1b5\ub839 \ucde8\uc784 \ub4a4\uc778 1976\ub144\uc5d0\ub294 \ud654\uad50\uc5d0 \ub300\ud55c \uad50\uc721\uad8c\uacfc \uc7ac\uc0b0\uad8c\uc744 \ubc15\ud0c8\ud558\uc5ec \ud55c\uad6d \ub0b4\uc5d0\uc11c\uc758 \uc678\uad6d\uc778\uacfc \uc678\uad6d \uc790\ubcf8\uc758 \uacbd\uc81c \uc7a5\uc545\ub825\uc744 \uc5b5\uc81c\ud558\uae30\ub3c4 \ud558\uc600\ub2e4. \uc778\ucc9c\uad11\uc5ed\uc2dc \uc911\uad6c\uc5d0 \uc704\uce58\ud55c 1\ub9cc 8000\ud3c9\uc758 \ucc28\uc774\ub098\ud0c0\uc6b4\uc5d0\ub294 \ud55c\ub54c \ud654\uad50 5,000\uc5ec \uba85\uc774 \uac70\uc8fc\ud588\uc73c\ub098 \ubc15\uc815\ud76c \uc815\uad8c \uc774\ud6c4 \ud654\uad50\uc758 \uc7ac\uc0b0\uad8c \ud589\uc0ac\ub97c \uc81c\ud55c\ud558\ub294 \uc815\ucc45\uc5d0 \ubd88\ub9cc\uc744 \ud488\uace0 \ubbf8\uad6d, \ub3d9\ub0a8\uc544 \ub4f1\uc73c\ub85c \ub5a0\ub098 \ud604\uc7ac\ub294 500\uc5ec \uba85 \ub9cc\uc774 \ub0a8\uc544\uc788\ub2e4. \uadf8\ub7ec\ub098, \ub2f9\uc2dc \ud55c\uad6d\uc758 \uae08\uc735 \uc0c1\uc5c5\uc801 \uacbd\uc81c \uad6c\uc870\ub97c \ubbf8\ucc98 \ud30c\uc545\ud558\uc9c0 \ubabb\ud55c, \ud1b5\ud654 \uac1c\ud601 \uc815\ucc45\uc740 \uc608\uae08 \ub3d9\uacb0 \uc870\uce58\ub97c \ubd88\ub7ec\uc654\uace0, \uc804\uccb4 \uacf5\uc7a5\uc758 45%\ub294 \uac00\ub3d9\uc744 \uc911\uc9c0\ud558\ub294 \ub4f1, \uc0ac\ud0dc\uac00 \uc545\ud654\ub418\uc5c8\ub2e4.", "sentence_answer": "\uc778\ucc9c\uad11\uc5ed\uc2dc \uc911\uad6c\uc5d0 \uc704\uce58\ud55c 1\ub9cc 8000\ud3c9\uc758 \ucc28\uc774\ub098\ud0c0\uc6b4\uc5d0\ub294 \ud55c\ub54c \ud654\uad50 5,000\uc5ec \uba85\uc774 \uac70\uc8fc\ud588\uc73c\ub098 \ubc15\uc815\ud76c \uc815\uad8c \uc774\ud6c4 \ud654\uad50\uc758 \uc7ac\uc0b0\uad8c \ud589\uc0ac\ub97c \uc81c\ud55c\ud558\ub294 \uc815\ucc45\uc5d0 \ubd88\ub9cc\uc744 \ud488\uace0 \ubbf8\uad6d, \ub3d9\ub0a8\uc544 \ub4f1\uc73c\ub85c \ub5a0\ub098 \ud604\uc7ac\ub294 500\uc5ec \uba85 \ub9cc\uc774 \ub0a8\uc544\uc788\ub2e4.", "paragraph_id": "6516370-27-0", "paragraph_question": "question: \uc778\ucc9c\uad11\uc5ed\uc2dc \uc911\uad6c\uc5d0 \uc704\uce58\ud55c \ucc28\uc774\ub098\ud0c0\uc6b4\uc5d0\ub294 \ud604\uc7ac \uba87 \uba85\uc758 \ud654\uad50\ub9cc\uc774 \ub0a8\uc544\uc788\ub294\uac00?, context: \uc2e4\uc81c\ub85c \ud654\ud3d0\uac1c\ud601 \uc774\ud6c4 \ud654\uad50\ub4e4\uc758 \uc790\ubcf8\ub825\uc740 \uc0c1\ub2f9\ud55c \ud0c0\uaca9\uc744 \uc785\uc5c8\uc73c\uba70 \uc0c1\ub2f9\uc218\uc758 \ud654\uad50\ub4e4\uc740 \ud55c\uad6d\uc744 \ub5a0\ub0ac\uace0 \uc790\uc5f0\ud788 \uc678\uc2dd\uc5c5\uc5d0 \uc9c4\ucd9c\ud558\ub294 \ud654\uad50\uac00 \ub298\uc5b4\ub0ac\uc73c\ub098 \ub300\ud1b5\ub839 \ucde8\uc784 \ub4a4\uc778 1976\ub144\uc5d0\ub294 \ud654\uad50\uc5d0 \ub300\ud55c \uad50\uc721\uad8c\uacfc \uc7ac\uc0b0\uad8c\uc744 \ubc15\ud0c8\ud558\uc5ec \ud55c\uad6d \ub0b4\uc5d0\uc11c\uc758 \uc678\uad6d\uc778\uacfc \uc678\uad6d \uc790\ubcf8\uc758 \uacbd\uc81c \uc7a5\uc545\ub825\uc744 \uc5b5\uc81c\ud558\uae30\ub3c4 \ud558\uc600\ub2e4. \uc778\ucc9c\uad11\uc5ed\uc2dc \uc911\uad6c\uc5d0 \uc704\uce58\ud55c 1\ub9cc 8000\ud3c9\uc758 \ucc28\uc774\ub098\ud0c0\uc6b4\uc5d0\ub294 \ud55c\ub54c \ud654\uad50 5,000\uc5ec \uba85\uc774 \uac70\uc8fc\ud588\uc73c\ub098 \ubc15\uc815\ud76c \uc815\uad8c \uc774\ud6c4 \ud654\uad50\uc758 \uc7ac\uc0b0\uad8c \ud589\uc0ac\ub97c \uc81c\ud55c\ud558\ub294 \uc815\ucc45\uc5d0 \ubd88\ub9cc\uc744 \ud488\uace0 \ubbf8\uad6d, \ub3d9\ub0a8\uc544 \ub4f1\uc73c\ub85c \ub5a0\ub098 \ud604\uc7ac\ub294 500\uc5ec \uba85\ub9cc\uc774 \ub0a8\uc544\uc788\ub2e4. \uadf8\ub7ec\ub098, \ub2f9\uc2dc \ud55c\uad6d\uc758 \uae08\uc735 \uc0c1\uc5c5\uc801 \uacbd\uc81c \uad6c\uc870\ub97c \ubbf8\ucc98 \ud30c\uc545\ud558\uc9c0 \ubabb\ud55c, \ud1b5\ud654 \uac1c\ud601 \uc815\ucc45\uc740 \uc608\uae08 \ub3d9\uacb0 \uc870\uce58\ub97c \ubd88\ub7ec\uc654\uace0, \uc804\uccb4 \uacf5\uc7a5\uc758 45%\ub294 \uac00\ub3d9\uc744 \uc911\uc9c0\ud558\ub294 \ub4f1, \uc0ac\ud0dc\uac00 \uc545\ud654\ub418\uc5c8\ub2e4."} {"question": "single ladies\uc5d0\uc11c \ubcf4\uc5ec\uc900 \ube44\uc698\uc138\uc758 \ub610 \ub2e4\ub978 \uc790\uc544 \uc0ac\uc0e4 \ud53c\uc5b4\uc2a4\ub294 \uc5b4\ub5a4 \ubaa8\uc2b5\uc744 \ud558\uace0 \uc788\ub294\uac00?", "paragraph": "\u3008Single Ladies\u3009\uc5d0\uc11c \ube44\uc698\uc138\ub294 \uacf5\uaca9\uc801\uc774\uace0 \uad00\ub2a5\uc801\uc778 \ubaa8\uc2b5\uc758 \ub610 \ub2e4\ub978 \uc790\uc544 \uc0ac\uc0e4 \ud53c\uc5b4\uc2a4\ub97c \ubcf4\uc5ec\uc92c\ub2e4. \u300a\ub514\uc9c0\ud138 \uc2a4\ud30c\uc774\u300b\uc758 \ub2c9 \ub808\ube48\uc740 \ube44\uc698\uc138\uc758 \ubaa9\uc18c\ub9ac\uc5d0\ub3c4 \ubc18\ud56d\uc801\uc778 \ud0dc\ub3c4\uc758 \ubaa8\uc2b5 \ub4f1\uc744 \ub4dc\ub7ec\ub0b4\uace0 \uc788\ub2e4\uace0 \ud3c9\uac00\ud588\ub2e4. \ub808\ube48\uc758 \ub9d0\ucc98\ub7fc \ub9ce\uc740 \ud3c9\ub860\uac00\ub4e4\uc774 \"\uc720\ucf8c\ud558\uba74\uc11c\ub3c4 \uac74\ubc29\uc9c4\" \uc18c\ub9ac\uac00 \ub4e4\ub9b0\ub2e4\uace0 \uc37c\ub2e4. \uac00\uc0ac\ub294 \ud55c \uc5ec\uc131\uc758 \uc774\ubcc4 \ud6c4 \ucc98\uc9c0\ub97c \ubc18\uc601\ud558\uace0 \uc788\ub2e4. \ud48d\ubd80\ud55c \uae30\uacc4\uc74c\uc744 \ub3d9\ubc18\ud55c \ub178\ub798\uc758 \uc2dc\uc791 \ubd80\ubd84\uc740 \ucf5c \uc564\ub4dc \ub9ac\uc2a4\ud3f0\uc2a4\uac00 \uc0ac\uc6a9\ub418\uc5c8\ub294\ub370, \ube44\uc698\uc138\uac00 \"All the single ladies\"\ub77c\uace0 \ubd80\ub974\uba74 \ubc31\uadf8\ub77c\uc6b4\ub4dc \ubcf4\uceec\uc774 \ub2e4\uc74c \ud30c\ud2b8\ub97c \uc774\uc5b4 \ubd80\ub978\ub2e4. 1\uc808\uc5d0\uc11c \ube44\uc698\uc138\ub294 \"\uc88b\uc558\ub358 3\ub144 \ub3d9\uc548 \ub208\ubb3c\uc774 \uba48\ucd94\uc9c0 \uc54a\uc558\uc9c0(cried tears for three good years)\"\ub77c\ub294 \uac00\uc0ac\uc640 \ud568\uaed8 \ubd88\uc30d\ud55c \ucc98\uc9c0\ub97c \ub098\ud0c0\ub0b4\uba70 \ub05d\ub0b8\ub2e4. \uc774\uc5b4 \ubc14\ub78c\ub465\uc774\ub97c \uc62c\ubc14\ub974\uac8c \uace0\uccd0\uc8fc\uace0, \uc790\uc2e0\uc740 \ud589\ubcf5\uc744 \ub290\ub07c\uace0, \uc0ac\ub791\ud558\ub294 \ub204\uad70\uac00\uc640 \ud568\uaed8 \uc788\ub294 \uac83\uc5d0 \uc774\uc804\ubcf4\ub2e4 \ub354 \uc2dc\uac04\uc744 \uc3df\ub294\ub2e4. \ud074\ub7fd\uc5d0\uc11c \ube44\uc698\uc138\ub294 \ub0a8\uc790\uce5c\uad6c\uc640 \uae68\uc9c0\uace0 \uadf8\uacf3\uc5d0\uc11c \uc0c8\ub85c\uc6b4 \uc0ac\ub791\ud558\ub294 \uc0ac\ub78c\uc744 \ub9cc\ub09c\ub2e4. \uadf8\ub7ec\ub098 \uc774\uc804 \ub0a8\uc790\uce5c\uad6c\ub294 \ud5e4\uc5b4\uc9c4 \ud6c4\uc5d0\ub3c4 \uacc4\uc18d \uc9c0\ucf1c\ubcf4\uace0 \uc788\uace0, \ub178\ub798\uc5d0\uc11c \ub0a8\uc790\uce5c\uad6c\uc5d0\uac8c \ub3cc\uc544\uac00\uc9c0 \uc54a\ub294\ub2e4\ub294 \uac83\uc744 \ubcf4\uc5ec\uc900\ub2e4. \ube44\uc698\uc138\ub294 \ub9c8\uc774\ub108\ud55c \ucf54\ub4dc \ub97c \uc0ac\uc6a9\ud55c \"If you like it then you should have put a ring on it ... Oh oh oh\"\uc640 \uac19\uc740 \uba87\uba87 \ud6c4\ud06c\ub97c \uc0ac\uc6a9\ud55c \ucf54\ub7ec\uc2a4\ub97c \ud1b5\ud574 \ub178\ub798\uc5d0 \ub098\ud0c0\ub0b4\uace0 \uc788\ub2e4.", "answer": "\uacf5\uaca9\uc801\uc774\uace0 \uad00\ub2a5\uc801\uc778 \ubaa8\uc2b5", "sentence": "\u3008Single Ladies\u3009\uc5d0\uc11c \ube44\uc698\uc138\ub294 \uacf5\uaca9\uc801\uc774\uace0 \uad00\ub2a5\uc801\uc778 \ubaa8\uc2b5 \uc758 \ub610 \ub2e4\ub978 \uc790\uc544 \uc0ac\uc0e4 \ud53c\uc5b4\uc2a4\ub97c \ubcf4\uc5ec\uc92c\ub2e4.", "paragraph_sentence": " \u3008Single Ladies\u3009\uc5d0\uc11c \ube44\uc698\uc138\ub294 \uacf5\uaca9\uc801\uc774\uace0 \uad00\ub2a5\uc801\uc778 \ubaa8\uc2b5 \uc758 \ub610 \ub2e4\ub978 \uc790\uc544 \uc0ac\uc0e4 \ud53c\uc5b4\uc2a4\ub97c \ubcf4\uc5ec\uc92c\ub2e4. \u300a\ub514\uc9c0\ud138 \uc2a4\ud30c\uc774\u300b\uc758 \ub2c9 \ub808\ube48\uc740 \ube44\uc698\uc138\uc758 \ubaa9\uc18c\ub9ac\uc5d0\ub3c4 \ubc18\ud56d\uc801\uc778 \ud0dc\ub3c4\uc758 \ubaa8\uc2b5 \ub4f1\uc744 \ub4dc\ub7ec\ub0b4\uace0 \uc788\ub2e4\uace0 \ud3c9\uac00\ud588\ub2e4. \ub808\ube48\uc758 \ub9d0\ucc98\ub7fc \ub9ce\uc740 \ud3c9\ub860\uac00\ub4e4\uc774 \"\uc720\ucf8c\ud558\uba74\uc11c\ub3c4 \uac74\ubc29\uc9c4\" \uc18c\ub9ac\uac00 \ub4e4\ub9b0\ub2e4\uace0 \uc37c\ub2e4. \uac00\uc0ac\ub294 \ud55c \uc5ec\uc131\uc758 \uc774\ubcc4 \ud6c4 \ucc98\uc9c0\ub97c \ubc18\uc601\ud558\uace0 \uc788\ub2e4. \ud48d\ubd80\ud55c \uae30\uacc4\uc74c\uc744 \ub3d9\ubc18\ud55c \ub178\ub798\uc758 \uc2dc\uc791 \ubd80\ubd84\uc740 \ucf5c \uc564\ub4dc \ub9ac\uc2a4\ud3f0\uc2a4\uac00 \uc0ac\uc6a9\ub418\uc5c8\ub294\ub370, \ube44\uc698\uc138\uac00 \"All the single ladies\"\ub77c\uace0 \ubd80\ub974\uba74 \ubc31\uadf8\ub77c\uc6b4\ub4dc \ubcf4\uceec\uc774 \ub2e4\uc74c \ud30c\ud2b8\ub97c \uc774\uc5b4 \ubd80\ub978\ub2e4. 1\uc808\uc5d0\uc11c \ube44\uc698\uc138\ub294 \"\uc88b\uc558\ub358 3\ub144 \ub3d9\uc548 \ub208\ubb3c\uc774 \uba48\ucd94\uc9c0 \uc54a\uc558\uc9c0(cried tears for three good years)\"\ub77c\ub294 \uac00\uc0ac\uc640 \ud568\uaed8 \ubd88\uc30d\ud55c \ucc98\uc9c0\ub97c \ub098\ud0c0\ub0b4\uba70 \ub05d\ub0b8\ub2e4. \uc774\uc5b4 \ubc14\ub78c\ub465\uc774\ub97c \uc62c\ubc14\ub974\uac8c \uace0\uccd0\uc8fc\uace0, \uc790\uc2e0\uc740 \ud589\ubcf5\uc744 \ub290\ub07c\uace0, \uc0ac\ub791\ud558\ub294 \ub204\uad70\uac00\uc640 \ud568\uaed8 \uc788\ub294 \uac83\uc5d0 \uc774\uc804\ubcf4\ub2e4 \ub354 \uc2dc\uac04\uc744 \uc3df\ub294\ub2e4. \ud074\ub7fd\uc5d0\uc11c \ube44\uc698\uc138\ub294 \ub0a8\uc790\uce5c\uad6c\uc640 \uae68\uc9c0\uace0 \uadf8\uacf3\uc5d0\uc11c \uc0c8\ub85c\uc6b4 \uc0ac\ub791\ud558\ub294 \uc0ac\ub78c\uc744 \ub9cc\ub09c\ub2e4. \uadf8\ub7ec\ub098 \uc774\uc804 \ub0a8\uc790\uce5c\uad6c\ub294 \ud5e4\uc5b4\uc9c4 \ud6c4\uc5d0\ub3c4 \uacc4\uc18d \uc9c0\ucf1c\ubcf4\uace0 \uc788\uace0, \ub178\ub798\uc5d0\uc11c \ub0a8\uc790\uce5c\uad6c\uc5d0\uac8c \ub3cc\uc544\uac00\uc9c0 \uc54a\ub294\ub2e4\ub294 \uac83\uc744 \ubcf4\uc5ec\uc900\ub2e4. \ube44\uc698\uc138\ub294 \ub9c8\uc774\ub108\ud55c \ucf54\ub4dc \ub97c \uc0ac\uc6a9\ud55c \"If you like it then you should have put a ring on it ... Oh oh oh\"\uc640 \uac19\uc740 \uba87\uba87 \ud6c4\ud06c\ub97c \uc0ac\uc6a9\ud55c \ucf54\ub7ec\uc2a4\ub97c \ud1b5\ud574 \ub178\ub798\uc5d0 \ub098\ud0c0\ub0b4\uace0 \uc788\ub2e4.", "paragraph_answer": "\u3008Single Ladies\u3009\uc5d0\uc11c \ube44\uc698\uc138\ub294 \uacf5\uaca9\uc801\uc774\uace0 \uad00\ub2a5\uc801\uc778 \ubaa8\uc2b5 \uc758 \ub610 \ub2e4\ub978 \uc790\uc544 \uc0ac\uc0e4 \ud53c\uc5b4\uc2a4\ub97c \ubcf4\uc5ec\uc92c\ub2e4. \u300a\ub514\uc9c0\ud138 \uc2a4\ud30c\uc774\u300b\uc758 \ub2c9 \ub808\ube48\uc740 \ube44\uc698\uc138\uc758 \ubaa9\uc18c\ub9ac\uc5d0\ub3c4 \ubc18\ud56d\uc801\uc778 \ud0dc\ub3c4\uc758 \ubaa8\uc2b5 \ub4f1\uc744 \ub4dc\ub7ec\ub0b4\uace0 \uc788\ub2e4\uace0 \ud3c9\uac00\ud588\ub2e4. \ub808\ube48\uc758 \ub9d0\ucc98\ub7fc \ub9ce\uc740 \ud3c9\ub860\uac00\ub4e4\uc774 \"\uc720\ucf8c\ud558\uba74\uc11c\ub3c4 \uac74\ubc29\uc9c4\" \uc18c\ub9ac\uac00 \ub4e4\ub9b0\ub2e4\uace0 \uc37c\ub2e4. \uac00\uc0ac\ub294 \ud55c \uc5ec\uc131\uc758 \uc774\ubcc4 \ud6c4 \ucc98\uc9c0\ub97c \ubc18\uc601\ud558\uace0 \uc788\ub2e4. \ud48d\ubd80\ud55c \uae30\uacc4\uc74c\uc744 \ub3d9\ubc18\ud55c \ub178\ub798\uc758 \uc2dc\uc791 \ubd80\ubd84\uc740 \ucf5c \uc564\ub4dc \ub9ac\uc2a4\ud3f0\uc2a4\uac00 \uc0ac\uc6a9\ub418\uc5c8\ub294\ub370, \ube44\uc698\uc138\uac00 \"All the single ladies\"\ub77c\uace0 \ubd80\ub974\uba74 \ubc31\uadf8\ub77c\uc6b4\ub4dc \ubcf4\uceec\uc774 \ub2e4\uc74c \ud30c\ud2b8\ub97c \uc774\uc5b4 \ubd80\ub978\ub2e4. 1\uc808\uc5d0\uc11c \ube44\uc698\uc138\ub294 \"\uc88b\uc558\ub358 3\ub144 \ub3d9\uc548 \ub208\ubb3c\uc774 \uba48\ucd94\uc9c0 \uc54a\uc558\uc9c0(cried tears for three good years)\"\ub77c\ub294 \uac00\uc0ac\uc640 \ud568\uaed8 \ubd88\uc30d\ud55c \ucc98\uc9c0\ub97c \ub098\ud0c0\ub0b4\uba70 \ub05d\ub0b8\ub2e4. \uc774\uc5b4 \ubc14\ub78c\ub465\uc774\ub97c \uc62c\ubc14\ub974\uac8c \uace0\uccd0\uc8fc\uace0, \uc790\uc2e0\uc740 \ud589\ubcf5\uc744 \ub290\ub07c\uace0, \uc0ac\ub791\ud558\ub294 \ub204\uad70\uac00\uc640 \ud568\uaed8 \uc788\ub294 \uac83\uc5d0 \uc774\uc804\ubcf4\ub2e4 \ub354 \uc2dc\uac04\uc744 \uc3df\ub294\ub2e4. \ud074\ub7fd\uc5d0\uc11c \ube44\uc698\uc138\ub294 \ub0a8\uc790\uce5c\uad6c\uc640 \uae68\uc9c0\uace0 \uadf8\uacf3\uc5d0\uc11c \uc0c8\ub85c\uc6b4 \uc0ac\ub791\ud558\ub294 \uc0ac\ub78c\uc744 \ub9cc\ub09c\ub2e4. \uadf8\ub7ec\ub098 \uc774\uc804 \ub0a8\uc790\uce5c\uad6c\ub294 \ud5e4\uc5b4\uc9c4 \ud6c4\uc5d0\ub3c4 \uacc4\uc18d \uc9c0\ucf1c\ubcf4\uace0 \uc788\uace0, \ub178\ub798\uc5d0\uc11c \ub0a8\uc790\uce5c\uad6c\uc5d0\uac8c \ub3cc\uc544\uac00\uc9c0 \uc54a\ub294\ub2e4\ub294 \uac83\uc744 \ubcf4\uc5ec\uc900\ub2e4. \ube44\uc698\uc138\ub294 \ub9c8\uc774\ub108\ud55c \ucf54\ub4dc \ub97c \uc0ac\uc6a9\ud55c \"If you like it then you should have put a ring on it ... Oh oh oh\"\uc640 \uac19\uc740 \uba87\uba87 \ud6c4\ud06c\ub97c \uc0ac\uc6a9\ud55c \ucf54\ub7ec\uc2a4\ub97c \ud1b5\ud574 \ub178\ub798\uc5d0 \ub098\ud0c0\ub0b4\uace0 \uc788\ub2e4.", "sentence_answer": "\u3008Single Ladies\u3009\uc5d0\uc11c \ube44\uc698\uc138\ub294 \uacf5\uaca9\uc801\uc774\uace0 \uad00\ub2a5\uc801\uc778 \ubaa8\uc2b5 \uc758 \ub610 \ub2e4\ub978 \uc790\uc544 \uc0ac\uc0e4 \ud53c\uc5b4\uc2a4\ub97c \ubcf4\uc5ec\uc92c\ub2e4.", "paragraph_id": "6488992-6-0", "paragraph_question": "question: single ladies\uc5d0\uc11c \ubcf4\uc5ec\uc900 \ube44\uc698\uc138\uc758 \ub610 \ub2e4\ub978 \uc790\uc544 \uc0ac\uc0e4 \ud53c\uc5b4\uc2a4\ub294 \uc5b4\ub5a4 \ubaa8\uc2b5\uc744 \ud558\uace0 \uc788\ub294\uac00?, context: \u3008Single Ladies\u3009\uc5d0\uc11c \ube44\uc698\uc138\ub294 \uacf5\uaca9\uc801\uc774\uace0 \uad00\ub2a5\uc801\uc778 \ubaa8\uc2b5\uc758 \ub610 \ub2e4\ub978 \uc790\uc544 \uc0ac\uc0e4 \ud53c\uc5b4\uc2a4\ub97c \ubcf4\uc5ec\uc92c\ub2e4. \u300a\ub514\uc9c0\ud138 \uc2a4\ud30c\uc774\u300b\uc758 \ub2c9 \ub808\ube48\uc740 \ube44\uc698\uc138\uc758 \ubaa9\uc18c\ub9ac\uc5d0\ub3c4 \ubc18\ud56d\uc801\uc778 \ud0dc\ub3c4\uc758 \ubaa8\uc2b5 \ub4f1\uc744 \ub4dc\ub7ec\ub0b4\uace0 \uc788\ub2e4\uace0 \ud3c9\uac00\ud588\ub2e4. \ub808\ube48\uc758 \ub9d0\ucc98\ub7fc \ub9ce\uc740 \ud3c9\ub860\uac00\ub4e4\uc774 \"\uc720\ucf8c\ud558\uba74\uc11c\ub3c4 \uac74\ubc29\uc9c4\" \uc18c\ub9ac\uac00 \ub4e4\ub9b0\ub2e4\uace0 \uc37c\ub2e4. \uac00\uc0ac\ub294 \ud55c \uc5ec\uc131\uc758 \uc774\ubcc4 \ud6c4 \ucc98\uc9c0\ub97c \ubc18\uc601\ud558\uace0 \uc788\ub2e4. \ud48d\ubd80\ud55c \uae30\uacc4\uc74c\uc744 \ub3d9\ubc18\ud55c \ub178\ub798\uc758 \uc2dc\uc791 \ubd80\ubd84\uc740 \ucf5c \uc564\ub4dc \ub9ac\uc2a4\ud3f0\uc2a4\uac00 \uc0ac\uc6a9\ub418\uc5c8\ub294\ub370, \ube44\uc698\uc138\uac00 \"All the single ladies\"\ub77c\uace0 \ubd80\ub974\uba74 \ubc31\uadf8\ub77c\uc6b4\ub4dc \ubcf4\uceec\uc774 \ub2e4\uc74c \ud30c\ud2b8\ub97c \uc774\uc5b4 \ubd80\ub978\ub2e4. 1\uc808\uc5d0\uc11c \ube44\uc698\uc138\ub294 \"\uc88b\uc558\ub358 3\ub144 \ub3d9\uc548 \ub208\ubb3c\uc774 \uba48\ucd94\uc9c0 \uc54a\uc558\uc9c0(cried tears for three good years)\"\ub77c\ub294 \uac00\uc0ac\uc640 \ud568\uaed8 \ubd88\uc30d\ud55c \ucc98\uc9c0\ub97c \ub098\ud0c0\ub0b4\uba70 \ub05d\ub0b8\ub2e4. \uc774\uc5b4 \ubc14\ub78c\ub465\uc774\ub97c \uc62c\ubc14\ub974\uac8c \uace0\uccd0\uc8fc\uace0, \uc790\uc2e0\uc740 \ud589\ubcf5\uc744 \ub290\ub07c\uace0, \uc0ac\ub791\ud558\ub294 \ub204\uad70\uac00\uc640 \ud568\uaed8 \uc788\ub294 \uac83\uc5d0 \uc774\uc804\ubcf4\ub2e4 \ub354 \uc2dc\uac04\uc744 \uc3df\ub294\ub2e4. \ud074\ub7fd\uc5d0\uc11c \ube44\uc698\uc138\ub294 \ub0a8\uc790\uce5c\uad6c\uc640 \uae68\uc9c0\uace0 \uadf8\uacf3\uc5d0\uc11c \uc0c8\ub85c\uc6b4 \uc0ac\ub791\ud558\ub294 \uc0ac\ub78c\uc744 \ub9cc\ub09c\ub2e4. \uadf8\ub7ec\ub098 \uc774\uc804 \ub0a8\uc790\uce5c\uad6c\ub294 \ud5e4\uc5b4\uc9c4 \ud6c4\uc5d0\ub3c4 \uacc4\uc18d \uc9c0\ucf1c\ubcf4\uace0 \uc788\uace0, \ub178\ub798\uc5d0\uc11c \ub0a8\uc790\uce5c\uad6c\uc5d0\uac8c \ub3cc\uc544\uac00\uc9c0 \uc54a\ub294\ub2e4\ub294 \uac83\uc744 \ubcf4\uc5ec\uc900\ub2e4. \ube44\uc698\uc138\ub294 \ub9c8\uc774\ub108\ud55c \ucf54\ub4dc \ub97c \uc0ac\uc6a9\ud55c \"If you like it then you should have put a ring on it ... Oh oh oh\"\uc640 \uac19\uc740 \uba87\uba87 \ud6c4\ud06c\ub97c \uc0ac\uc6a9\ud55c \ucf54\ub7ec\uc2a4\ub97c \ud1b5\ud574 \ub178\ub798\uc5d0 \ub098\ud0c0\ub0b4\uace0 \uc788\ub2e4."} {"question": "\uc704\ub354\uc2a4\ud47c\uc774 2004\ub144 \ucd9c\uc5f0\ud55c \uc601\ud654\uc758 \uc81c\ubaa9\uc740 \ubb34\uc5c7\uc778\uac00?", "paragraph": "2004\ub144\uc5d0\ub294 19\uc138\uae30\uc758 \uace0\uc804\uc8fc\uc758 \uc18c\uc124\uc744 \uac01\uc0c9\ud55c \uc791\ud488\uc778 \uc601\ud654 \u300a\ubca0\ub2c8\ud2f0 \ud398\uc5b4\u300b\uc5d0 \ucd9c\uc5f0\ud558\uac8c \ub410\ub2e4. \uc704\ub354\uc2a4\ud47c\uc774 \uc5f0\uae30\ud55c \uce90\ub9ad\ud130\ub294 \ud559\ub300\ubc1b\ub358 \uc5b4\ub9b0 \uc2dc\uc808\uc744 \uac70\uccd0 \ubd80\uc640 \uba85\uc608\ub97c \ucc3e\uae30 \uc704\ud574 \uc57c\ub9dd\uc744 \ud488\uac8c \ub418\ub294 \ubca0\ud0a4 \uc0e4\ud504\uc600\ub2e4. \ucd2c\uc601 \ub2f9\uc2dc \uc784\uc2e0 \uc911\uc774\uc5c8\uc73c\ubbc0\ub85c \uc601\ud654 \ub0b4\ub0b4 \uc784\uc2e0\ub41c \uc0c1\ud0dc\uc784\uc744 \ub178\ucd9c\ud558\uc9c0 \uc54a\uae30 \uc704\ud574 \uc758\uc0c1 \uc900\ube44\uc5d0 \uad49\uc7a5\ud55c \uc8fc\uc758\ub97c \uae30\uc6b8\uc600\ub2e4\uace0 \ud55c\ub2e4. \ud558\uc9c0\ub9cc \uc790\uc2e0\uc740 \uc784\uc2e0\uc774\ub77c\ub294 \uc5b4\ub824\uc6c0\uc774 \uacb0\ucf54 \uc81c\uc57d\uc774 \ub418\uc9c0 \uc54a\uc558\uc73c\uba70 \uc0e4\ud504\uc758 \uc5ed\ud560\uc744 \ud558\uba74\uc11c \uc0dd\uba85\uc774\ub77c\ub294 \uac83, \uc0dd\ub3d9\uac10\uc774\ub77c\ub294 \uc544\ub984\ub2e4\uc6c0\uc744 \ub290\ub084 \uc218 \uc788\uc5b4\uc11c \ud589\ubcf5\ud588\ub2e4\uace0 \ubc1d\ud614\ub2e4. \u300a\ubca0\ub2c8\ud2f0 \ud398\uc5b4\u300b\uc5d0 \ub300\ud55c \ubc18\uc751\uc740 \uc0c1\ub2f9\ud788 \uae0d\uc815\uc801\uc774\uc5c8\uc73c\uba70 \ubaa8\ub4e0 \ubc30\uc5ed\uc774 \uc5ed\ud560\uc5d0 \ub9de\uac8c \ubc30\uc815\ub41c \ub370\uc5d0 \ub300\ud574 \ud070 \uc810\uc218\ub97c \ubc1b\uc558\ub2e4. \uc0ac\uc2e4 \uadc0\uc5fd\uace0 \ud1a1\ud1a1 \ud280\ub294 \uc5ed\ud560\uc744 2001\ub144 \uc601\ud654 \u300a\uae08\ubc1c\uc774 \ub108\ubb34\ud574\u300b\uc5d0\uc11c \uc120\ubcf4\uc600\ub358 \uc704\ub354\uc2a4\ud47c\uc758 \uc785\uc7a5\uc5d0\uc11c\ub294 \uc57c\ub9dd\uc744 \uc704\ud574 \uc5f0\uc778\uc744 \ubc30\uc2e0\ud558\uace0 \ub2e4\uc2dc \uc0ac\ub791\uc744 \uc7c1\ucde8\ud558\ub294 \uc545\uc5ed\uc744 \ub9e1\uae30\uc5d0\ub294 \ubd80\ub2f4\uc774 \ucef8\ub2e4. 19\uc138\uae30 \uc2e0\ubd84 \uc0c1\uc2b9\uc758 \uafc8\uc744 \uac00\uc2b4\uc5d0 \uac10\ucd98 \ud5c8\uc601\uc2ec \ub9ce\uc740 \uc5ec\uc778\uc744 \uc5b4\ub5a4 \uc2dd\uc73c\ub85c \ud45c\ud604\ud560\uc9c0\uc5d0 \ub300\ud574\uc11c\ub294 \uc6b0\ub824\ub3c4 \uc788\uc5c8\ub2e4. \uadf8\ub7ec\ub098 \ubbf8\uad6d \uac01\uc9c0\uc758 \uc5b8\ub860\uc5d0\uc11c\ub294 \ub9ac\uc988\uac00 \ub9e1\uc740 \ubca0\ud0a4 \uc0e4\ud504\uc5d0 \ub300\ud574 \u201c\uc774 \uc5ed\ud560\uc744 \ub9e1\uae30 \uc704\ud574 \ub9ac\uc988 \uc704\ub354\uc2a4\ud47c\uc774 \ud0dc\uc5b4\ub0ac\ub2e4.\u201d \ub610\ub294 \u201c\uc704\ud5d8\ud55c \uc120\ud0dd\uc774\uc5c8\uc744 \ud150\ub370 \uc5ed\ud560 \uc790\uccb4\uc758 \uac15\uc810\uc744 100% \ubc1c\ud718\ud588\ub2e4.\u201d \ub4f1\uc758 \ud6c4\ud55c \ud3c9\uac00\ub97c \ub0b4\ub838\ub2e4. 2004\ub144 \ud6c4\ubc18\uae30\uc5d0 \ub4e4\uba74\uc11c \uc704\ub354\uc2a4\ud47c\uc740 \ub9c8\ud06c \ub7ec\ud384\ub85c\uc640 \ud568\uaed8 \ub85c\ub9e8\uc2a4 \ucf54\ubbf8\ub514 \uc601\ud654\uc600\ub358 \u300a\uc800\uc2a4\ud2b8 \ub77c\uc774\ud06c \ud5e4\ube10\u300b\uc744 \ucd2c\uc601\ud558\uac8c \ub41c\ub2e4. \uadf8\ub140\uac00 \ub9e1\uc558\ub358 \uc5ed\ud560\uc778 \uc5d8\ub9ac\uc790\ubca0\uc2a4 \ub9c8\uc2a4\ud130\uc2a8\uc740 \uc720\ub2a5\ud55c \uc758\uc0ac\uc774\ub098 \uad50\ud1b5\uc0ac\uace0\ub85c \uc758\uc2dd \ubd88\uba85 \uc0c1\ud0dc\ub2e4. \uadf8\uc911 \uadf8\ub140\uc758 \uc601\ud63c\uc774 \uc61b\ub0a0\uc5d0 \uc0b4\ub358 \uc544\ud30c\ud2b8\ub85c \uac00 \uc9c4\uc815\ud55c \uc0ac\ub791\uc744 \ucc3e\uac8c \ub418\ub294 \uacfc\uc815\uc774 \uadf8\ub824\uc838 \uc788\ub2e4.", "answer": "\ubca0\ub2c8\ud2f0 \ud398\uc5b4", "sentence": "2004\ub144\uc5d0\ub294 19\uc138\uae30\uc758 \uace0\uc804\uc8fc\uc758 \uc18c\uc124\uc744 \uac01\uc0c9\ud55c \uc791\ud488\uc778 \uc601\ud654 \u300a \ubca0\ub2c8\ud2f0 \ud398\uc5b4 \u300b\uc5d0 \ucd9c\uc5f0\ud558\uac8c \ub410\ub2e4.", "paragraph_sentence": " 2004\ub144\uc5d0\ub294 19\uc138\uae30\uc758 \uace0\uc804\uc8fc\uc758 \uc18c\uc124\uc744 \uac01\uc0c9\ud55c \uc791\ud488\uc778 \uc601\ud654 \u300a \ubca0\ub2c8\ud2f0 \ud398\uc5b4 \u300b\uc5d0 \ucd9c\uc5f0\ud558\uac8c \ub410\ub2e4. \uc704\ub354\uc2a4\ud47c\uc774 \uc5f0\uae30\ud55c \uce90\ub9ad\ud130\ub294 \ud559\ub300\ubc1b\ub358 \uc5b4\ub9b0 \uc2dc\uc808\uc744 \uac70\uccd0 \ubd80\uc640 \uba85\uc608\ub97c \ucc3e\uae30 \uc704\ud574 \uc57c\ub9dd\uc744 \ud488\uac8c \ub418\ub294 \ubca0\ud0a4 \uc0e4\ud504\uc600\ub2e4. \ucd2c\uc601 \ub2f9\uc2dc \uc784\uc2e0 \uc911\uc774\uc5c8\uc73c\ubbc0\ub85c \uc601\ud654 \ub0b4\ub0b4 \uc784\uc2e0\ub41c \uc0c1\ud0dc\uc784\uc744 \ub178\ucd9c\ud558\uc9c0 \uc54a\uae30 \uc704\ud574 \uc758\uc0c1 \uc900\ube44\uc5d0 \uad49\uc7a5\ud55c \uc8fc\uc758\ub97c \uae30\uc6b8\uc600\ub2e4\uace0 \ud55c\ub2e4. \ud558\uc9c0\ub9cc \uc790\uc2e0\uc740 \uc784\uc2e0\uc774\ub77c\ub294 \uc5b4\ub824\uc6c0\uc774 \uacb0\ucf54 \uc81c\uc57d\uc774 \ub418\uc9c0 \uc54a\uc558\uc73c\uba70 \uc0e4\ud504\uc758 \uc5ed\ud560\uc744 \ud558\uba74\uc11c \uc0dd\uba85\uc774\ub77c\ub294 \uac83, \uc0dd\ub3d9\uac10\uc774\ub77c\ub294 \uc544\ub984\ub2e4\uc6c0\uc744 \ub290\ub084 \uc218 \uc788\uc5b4\uc11c \ud589\ubcf5\ud588\ub2e4\uace0 \ubc1d\ud614\ub2e4. \u300a\ubca0\ub2c8\ud2f0 \ud398\uc5b4\u300b\uc5d0 \ub300\ud55c \ubc18\uc751\uc740 \uc0c1\ub2f9\ud788 \uae0d\uc815\uc801\uc774\uc5c8\uc73c\uba70 \ubaa8\ub4e0 \ubc30\uc5ed\uc774 \uc5ed\ud560\uc5d0 \ub9de\uac8c \ubc30\uc815\ub41c \ub370\uc5d0 \ub300\ud574 \ud070 \uc810\uc218\ub97c \ubc1b\uc558\ub2e4. \uc0ac\uc2e4 \uadc0\uc5fd\uace0 \ud1a1\ud1a1 \ud280\ub294 \uc5ed\ud560\uc744 2001\ub144 \uc601\ud654 \u300a\uae08\ubc1c\uc774 \ub108\ubb34\ud574\u300b\uc5d0\uc11c \uc120\ubcf4\uc600\ub358 \uc704\ub354\uc2a4\ud47c\uc758 \uc785\uc7a5\uc5d0\uc11c\ub294 \uc57c\ub9dd\uc744 \uc704\ud574 \uc5f0\uc778\uc744 \ubc30\uc2e0\ud558\uace0 \ub2e4\uc2dc \uc0ac\ub791\uc744 \uc7c1\ucde8\ud558\ub294 \uc545\uc5ed\uc744 \ub9e1\uae30\uc5d0\ub294 \ubd80\ub2f4\uc774 \ucef8\ub2e4. 19\uc138\uae30 \uc2e0\ubd84 \uc0c1\uc2b9\uc758 \uafc8\uc744 \uac00\uc2b4\uc5d0 \uac10\ucd98 \ud5c8\uc601\uc2ec \ub9ce\uc740 \uc5ec\uc778\uc744 \uc5b4\ub5a4 \uc2dd\uc73c\ub85c \ud45c\ud604\ud560\uc9c0\uc5d0 \ub300\ud574\uc11c\ub294 \uc6b0\ub824\ub3c4 \uc788\uc5c8\ub2e4. \uadf8\ub7ec\ub098 \ubbf8\uad6d \uac01\uc9c0\uc758 \uc5b8\ub860\uc5d0\uc11c\ub294 \ub9ac\uc988\uac00 \ub9e1\uc740 \ubca0\ud0a4 \uc0e4\ud504\uc5d0 \ub300\ud574 \u201c\uc774 \uc5ed\ud560\uc744 \ub9e1\uae30 \uc704\ud574 \ub9ac\uc988 \uc704\ub354\uc2a4\ud47c\uc774 \ud0dc\uc5b4\ub0ac\ub2e4. \u201d \ub610\ub294 \u201c\uc704\ud5d8\ud55c \uc120\ud0dd\uc774\uc5c8\uc744 \ud150\ub370 \uc5ed\ud560 \uc790\uccb4\uc758 \uac15\uc810\uc744 100% \ubc1c\ud718\ud588\ub2e4. \u201d \ub4f1\uc758 \ud6c4\ud55c \ud3c9\uac00\ub97c \ub0b4\ub838\ub2e4. 2004\ub144 \ud6c4\ubc18\uae30\uc5d0 \ub4e4\uba74\uc11c \uc704\ub354\uc2a4\ud47c\uc740 \ub9c8\ud06c \ub7ec\ud384\ub85c\uc640 \ud568\uaed8 \ub85c\ub9e8\uc2a4 \ucf54\ubbf8\ub514 \uc601\ud654\uc600\ub358 \u300a\uc800\uc2a4\ud2b8 \ub77c\uc774\ud06c \ud5e4\ube10\u300b\uc744 \ucd2c\uc601\ud558\uac8c \ub41c\ub2e4. \uadf8\ub140\uac00 \ub9e1\uc558\ub358 \uc5ed\ud560\uc778 \uc5d8\ub9ac\uc790\ubca0\uc2a4 \ub9c8\uc2a4\ud130\uc2a8\uc740 \uc720\ub2a5\ud55c \uc758\uc0ac\uc774\ub098 \uad50\ud1b5\uc0ac\uace0\ub85c \uc758\uc2dd \ubd88\uba85 \uc0c1\ud0dc\ub2e4. \uadf8\uc911 \uadf8\ub140\uc758 \uc601\ud63c\uc774 \uc61b\ub0a0\uc5d0 \uc0b4\ub358 \uc544\ud30c\ud2b8\ub85c \uac00 \uc9c4\uc815\ud55c \uc0ac\ub791\uc744 \ucc3e\uac8c \ub418\ub294 \uacfc\uc815\uc774 \uadf8\ub824\uc838 \uc788\ub2e4.", "paragraph_answer": "2004\ub144\uc5d0\ub294 19\uc138\uae30\uc758 \uace0\uc804\uc8fc\uc758 \uc18c\uc124\uc744 \uac01\uc0c9\ud55c \uc791\ud488\uc778 \uc601\ud654 \u300a \ubca0\ub2c8\ud2f0 \ud398\uc5b4 \u300b\uc5d0 \ucd9c\uc5f0\ud558\uac8c \ub410\ub2e4. \uc704\ub354\uc2a4\ud47c\uc774 \uc5f0\uae30\ud55c \uce90\ub9ad\ud130\ub294 \ud559\ub300\ubc1b\ub358 \uc5b4\ub9b0 \uc2dc\uc808\uc744 \uac70\uccd0 \ubd80\uc640 \uba85\uc608\ub97c \ucc3e\uae30 \uc704\ud574 \uc57c\ub9dd\uc744 \ud488\uac8c \ub418\ub294 \ubca0\ud0a4 \uc0e4\ud504\uc600\ub2e4. \ucd2c\uc601 \ub2f9\uc2dc \uc784\uc2e0 \uc911\uc774\uc5c8\uc73c\ubbc0\ub85c \uc601\ud654 \ub0b4\ub0b4 \uc784\uc2e0\ub41c \uc0c1\ud0dc\uc784\uc744 \ub178\ucd9c\ud558\uc9c0 \uc54a\uae30 \uc704\ud574 \uc758\uc0c1 \uc900\ube44\uc5d0 \uad49\uc7a5\ud55c \uc8fc\uc758\ub97c \uae30\uc6b8\uc600\ub2e4\uace0 \ud55c\ub2e4. \ud558\uc9c0\ub9cc \uc790\uc2e0\uc740 \uc784\uc2e0\uc774\ub77c\ub294 \uc5b4\ub824\uc6c0\uc774 \uacb0\ucf54 \uc81c\uc57d\uc774 \ub418\uc9c0 \uc54a\uc558\uc73c\uba70 \uc0e4\ud504\uc758 \uc5ed\ud560\uc744 \ud558\uba74\uc11c \uc0dd\uba85\uc774\ub77c\ub294 \uac83, \uc0dd\ub3d9\uac10\uc774\ub77c\ub294 \uc544\ub984\ub2e4\uc6c0\uc744 \ub290\ub084 \uc218 \uc788\uc5b4\uc11c \ud589\ubcf5\ud588\ub2e4\uace0 \ubc1d\ud614\ub2e4. \u300a\ubca0\ub2c8\ud2f0 \ud398\uc5b4\u300b\uc5d0 \ub300\ud55c \ubc18\uc751\uc740 \uc0c1\ub2f9\ud788 \uae0d\uc815\uc801\uc774\uc5c8\uc73c\uba70 \ubaa8\ub4e0 \ubc30\uc5ed\uc774 \uc5ed\ud560\uc5d0 \ub9de\uac8c \ubc30\uc815\ub41c \ub370\uc5d0 \ub300\ud574 \ud070 \uc810\uc218\ub97c \ubc1b\uc558\ub2e4. \uc0ac\uc2e4 \uadc0\uc5fd\uace0 \ud1a1\ud1a1 \ud280\ub294 \uc5ed\ud560\uc744 2001\ub144 \uc601\ud654 \u300a\uae08\ubc1c\uc774 \ub108\ubb34\ud574\u300b\uc5d0\uc11c \uc120\ubcf4\uc600\ub358 \uc704\ub354\uc2a4\ud47c\uc758 \uc785\uc7a5\uc5d0\uc11c\ub294 \uc57c\ub9dd\uc744 \uc704\ud574 \uc5f0\uc778\uc744 \ubc30\uc2e0\ud558\uace0 \ub2e4\uc2dc \uc0ac\ub791\uc744 \uc7c1\ucde8\ud558\ub294 \uc545\uc5ed\uc744 \ub9e1\uae30\uc5d0\ub294 \ubd80\ub2f4\uc774 \ucef8\ub2e4. 19\uc138\uae30 \uc2e0\ubd84 \uc0c1\uc2b9\uc758 \uafc8\uc744 \uac00\uc2b4\uc5d0 \uac10\ucd98 \ud5c8\uc601\uc2ec \ub9ce\uc740 \uc5ec\uc778\uc744 \uc5b4\ub5a4 \uc2dd\uc73c\ub85c \ud45c\ud604\ud560\uc9c0\uc5d0 \ub300\ud574\uc11c\ub294 \uc6b0\ub824\ub3c4 \uc788\uc5c8\ub2e4. \uadf8\ub7ec\ub098 \ubbf8\uad6d \uac01\uc9c0\uc758 \uc5b8\ub860\uc5d0\uc11c\ub294 \ub9ac\uc988\uac00 \ub9e1\uc740 \ubca0\ud0a4 \uc0e4\ud504\uc5d0 \ub300\ud574 \u201c\uc774 \uc5ed\ud560\uc744 \ub9e1\uae30 \uc704\ud574 \ub9ac\uc988 \uc704\ub354\uc2a4\ud47c\uc774 \ud0dc\uc5b4\ub0ac\ub2e4.\u201d \ub610\ub294 \u201c\uc704\ud5d8\ud55c \uc120\ud0dd\uc774\uc5c8\uc744 \ud150\ub370 \uc5ed\ud560 \uc790\uccb4\uc758 \uac15\uc810\uc744 100% \ubc1c\ud718\ud588\ub2e4.\u201d \ub4f1\uc758 \ud6c4\ud55c \ud3c9\uac00\ub97c \ub0b4\ub838\ub2e4. 2004\ub144 \ud6c4\ubc18\uae30\uc5d0 \ub4e4\uba74\uc11c \uc704\ub354\uc2a4\ud47c\uc740 \ub9c8\ud06c \ub7ec\ud384\ub85c\uc640 \ud568\uaed8 \ub85c\ub9e8\uc2a4 \ucf54\ubbf8\ub514 \uc601\ud654\uc600\ub358 \u300a\uc800\uc2a4\ud2b8 \ub77c\uc774\ud06c \ud5e4\ube10\u300b\uc744 \ucd2c\uc601\ud558\uac8c \ub41c\ub2e4. \uadf8\ub140\uac00 \ub9e1\uc558\ub358 \uc5ed\ud560\uc778 \uc5d8\ub9ac\uc790\ubca0\uc2a4 \ub9c8\uc2a4\ud130\uc2a8\uc740 \uc720\ub2a5\ud55c \uc758\uc0ac\uc774\ub098 \uad50\ud1b5\uc0ac\uace0\ub85c \uc758\uc2dd \ubd88\uba85 \uc0c1\ud0dc\ub2e4. \uadf8\uc911 \uadf8\ub140\uc758 \uc601\ud63c\uc774 \uc61b\ub0a0\uc5d0 \uc0b4\ub358 \uc544\ud30c\ud2b8\ub85c \uac00 \uc9c4\uc815\ud55c \uc0ac\ub791\uc744 \ucc3e\uac8c \ub418\ub294 \uacfc\uc815\uc774 \uadf8\ub824\uc838 \uc788\ub2e4.", "sentence_answer": "2004\ub144\uc5d0\ub294 19\uc138\uae30\uc758 \uace0\uc804\uc8fc\uc758 \uc18c\uc124\uc744 \uac01\uc0c9\ud55c \uc791\ud488\uc778 \uc601\ud654 \u300a \ubca0\ub2c8\ud2f0 \ud398\uc5b4 \u300b\uc5d0 \ucd9c\uc5f0\ud558\uac8c \ub410\ub2e4.", "paragraph_id": "6513888-6-0", "paragraph_question": "question: \uc704\ub354\uc2a4\ud47c\uc774 2004\ub144 \ucd9c\uc5f0\ud55c \uc601\ud654\uc758 \uc81c\ubaa9\uc740 \ubb34\uc5c7\uc778\uac00?, context: 2004\ub144\uc5d0\ub294 19\uc138\uae30\uc758 \uace0\uc804\uc8fc\uc758 \uc18c\uc124\uc744 \uac01\uc0c9\ud55c \uc791\ud488\uc778 \uc601\ud654 \u300a\ubca0\ub2c8\ud2f0 \ud398\uc5b4\u300b\uc5d0 \ucd9c\uc5f0\ud558\uac8c \ub410\ub2e4. \uc704\ub354\uc2a4\ud47c\uc774 \uc5f0\uae30\ud55c \uce90\ub9ad\ud130\ub294 \ud559\ub300\ubc1b\ub358 \uc5b4\ub9b0 \uc2dc\uc808\uc744 \uac70\uccd0 \ubd80\uc640 \uba85\uc608\ub97c \ucc3e\uae30 \uc704\ud574 \uc57c\ub9dd\uc744 \ud488\uac8c \ub418\ub294 \ubca0\ud0a4 \uc0e4\ud504\uc600\ub2e4. \ucd2c\uc601 \ub2f9\uc2dc \uc784\uc2e0 \uc911\uc774\uc5c8\uc73c\ubbc0\ub85c \uc601\ud654 \ub0b4\ub0b4 \uc784\uc2e0\ub41c \uc0c1\ud0dc\uc784\uc744 \ub178\ucd9c\ud558\uc9c0 \uc54a\uae30 \uc704\ud574 \uc758\uc0c1 \uc900\ube44\uc5d0 \uad49\uc7a5\ud55c \uc8fc\uc758\ub97c \uae30\uc6b8\uc600\ub2e4\uace0 \ud55c\ub2e4. \ud558\uc9c0\ub9cc \uc790\uc2e0\uc740 \uc784\uc2e0\uc774\ub77c\ub294 \uc5b4\ub824\uc6c0\uc774 \uacb0\ucf54 \uc81c\uc57d\uc774 \ub418\uc9c0 \uc54a\uc558\uc73c\uba70 \uc0e4\ud504\uc758 \uc5ed\ud560\uc744 \ud558\uba74\uc11c \uc0dd\uba85\uc774\ub77c\ub294 \uac83, \uc0dd\ub3d9\uac10\uc774\ub77c\ub294 \uc544\ub984\ub2e4\uc6c0\uc744 \ub290\ub084 \uc218 \uc788\uc5b4\uc11c \ud589\ubcf5\ud588\ub2e4\uace0 \ubc1d\ud614\ub2e4. \u300a\ubca0\ub2c8\ud2f0 \ud398\uc5b4\u300b\uc5d0 \ub300\ud55c \ubc18\uc751\uc740 \uc0c1\ub2f9\ud788 \uae0d\uc815\uc801\uc774\uc5c8\uc73c\uba70 \ubaa8\ub4e0 \ubc30\uc5ed\uc774 \uc5ed\ud560\uc5d0 \ub9de\uac8c \ubc30\uc815\ub41c \ub370\uc5d0 \ub300\ud574 \ud070 \uc810\uc218\ub97c \ubc1b\uc558\ub2e4. \uc0ac\uc2e4 \uadc0\uc5fd\uace0 \ud1a1\ud1a1 \ud280\ub294 \uc5ed\ud560\uc744 2001\ub144 \uc601\ud654 \u300a\uae08\ubc1c\uc774 \ub108\ubb34\ud574\u300b\uc5d0\uc11c \uc120\ubcf4\uc600\ub358 \uc704\ub354\uc2a4\ud47c\uc758 \uc785\uc7a5\uc5d0\uc11c\ub294 \uc57c\ub9dd\uc744 \uc704\ud574 \uc5f0\uc778\uc744 \ubc30\uc2e0\ud558\uace0 \ub2e4\uc2dc \uc0ac\ub791\uc744 \uc7c1\ucde8\ud558\ub294 \uc545\uc5ed\uc744 \ub9e1\uae30\uc5d0\ub294 \ubd80\ub2f4\uc774 \ucef8\ub2e4. 19\uc138\uae30 \uc2e0\ubd84 \uc0c1\uc2b9\uc758 \uafc8\uc744 \uac00\uc2b4\uc5d0 \uac10\ucd98 \ud5c8\uc601\uc2ec \ub9ce\uc740 \uc5ec\uc778\uc744 \uc5b4\ub5a4 \uc2dd\uc73c\ub85c \ud45c\ud604\ud560\uc9c0\uc5d0 \ub300\ud574\uc11c\ub294 \uc6b0\ub824\ub3c4 \uc788\uc5c8\ub2e4. \uadf8\ub7ec\ub098 \ubbf8\uad6d \uac01\uc9c0\uc758 \uc5b8\ub860\uc5d0\uc11c\ub294 \ub9ac\uc988\uac00 \ub9e1\uc740 \ubca0\ud0a4 \uc0e4\ud504\uc5d0 \ub300\ud574 \u201c\uc774 \uc5ed\ud560\uc744 \ub9e1\uae30 \uc704\ud574 \ub9ac\uc988 \uc704\ub354\uc2a4\ud47c\uc774 \ud0dc\uc5b4\ub0ac\ub2e4.\u201d \ub610\ub294 \u201c\uc704\ud5d8\ud55c \uc120\ud0dd\uc774\uc5c8\uc744 \ud150\ub370 \uc5ed\ud560 \uc790\uccb4\uc758 \uac15\uc810\uc744 100% \ubc1c\ud718\ud588\ub2e4.\u201d \ub4f1\uc758 \ud6c4\ud55c \ud3c9\uac00\ub97c \ub0b4\ub838\ub2e4. 2004\ub144 \ud6c4\ubc18\uae30\uc5d0 \ub4e4\uba74\uc11c \uc704\ub354\uc2a4\ud47c\uc740 \ub9c8\ud06c \ub7ec\ud384\ub85c\uc640 \ud568\uaed8 \ub85c\ub9e8\uc2a4 \ucf54\ubbf8\ub514 \uc601\ud654\uc600\ub358 \u300a\uc800\uc2a4\ud2b8 \ub77c\uc774\ud06c \ud5e4\ube10\u300b\uc744 \ucd2c\uc601\ud558\uac8c \ub41c\ub2e4. \uadf8\ub140\uac00 \ub9e1\uc558\ub358 \uc5ed\ud560\uc778 \uc5d8\ub9ac\uc790\ubca0\uc2a4 \ub9c8\uc2a4\ud130\uc2a8\uc740 \uc720\ub2a5\ud55c \uc758\uc0ac\uc774\ub098 \uad50\ud1b5\uc0ac\uace0\ub85c \uc758\uc2dd \ubd88\uba85 \uc0c1\ud0dc\ub2e4. \uadf8\uc911 \uadf8\ub140\uc758 \uc601\ud63c\uc774 \uc61b\ub0a0\uc5d0 \uc0b4\ub358 \uc544\ud30c\ud2b8\ub85c \uac00 \uc9c4\uc815\ud55c \uc0ac\ub791\uc744 \ucc3e\uac8c \ub418\ub294 \uacfc\uc815\uc774 \uadf8\ub824\uc838 \uc788\ub2e4."} {"question": "3000\ubbf8\ud130 \uacc4\uc8fc\uc5d0\uc11c \ud55c\uad6d\uc744 \uc774\uae34 \ud300\uc740?", "paragraph": "\uae40\uc544\ub791\uc740 2014-2015 \uc2dc\uc98c \uad6d\uac00\ub300\ud45c \uc120\ubc1c\uc804\uc5d0\uc11c \uc5b4\uae68 \ubd80\uc0c1\uc744 \uc785\uc5c8\ub294\ub370 \uc774\uac83\uc740 \uc218\uc220\uc744 \ud544\uc694\ub85c \ud558\ub294 \ud68c\uc804\uadfc\uac1c(Rotator cuff) \uc190\uc0c1\uc774\uc5c8\uc73c\uba70 2015\ub144 \uad6d\uac00\ub300\ud45c \uc120\ubc1c\uc804\uc774 \ub05d\ub09c \ud6c4 \uc218\uc220\uc744 \ud558\uc600\ub2e4. 2014-2015 \uc2dc\uc98c\uc5d0\uc11c \uae40\uc544\ub791\uc740 5\ucc28\ub840 ISU \uc1fc\ud2b8\ud2b8\ub799 \uc6d4\ub4dc\ucef5\uc5d0 \ucd9c\uc804\ud558\uc5ec \uae08\uba54\ub2ec 3\uac1c \uc740\uba54\ub2ec 4\uac1c \ub3d9\uba54\ub2ec 1\uac1c\ub97c \ud68d\ub4dd\ud558\uc600\uc73c\uba70, 2015\ub144 2\uc6d4 \uc2a4\ud398\uc778 \uadf8\ub77c\ub098\ub2e4\uc5d0\uc11c \uc5f4\ub9b0 \ub3d9\uacc4 \uc720\ub2c8\ubc84\uc2dc\uc544\ub4dc \uc1fc\ud2b8\ud2b8\ub799 1000\ubbf8\ud130\uc640 1500\ubbf8\ud130\uc5d0\uc11c \uc6b0\uc2b9\ud558\uc5ec \uac1c\uc778\uc804 \uae08\uba54\ub2ec 2\uac1c\ub97c \ubaa9\uc5d0 \uac78\uc5c8\ub2e4. \ub2f9\uc2dc \uae40\uc544\ub791\uc740 \"\uae08\uba54\ub2ec 2\uac1c\ub97c \ucc28\uc9c0\ud55c \uac83\uc740 \uae30\uc058\uc9c0\ub9cc 3000\ubbf8\ud130 \uacc4\uc8fc\uc5d0\uc11c \uc911\uad6d\uc5d0\uac8c \uc11d\ud328\ud55c \uac83\uc774 \uc544\uc27d\ub2e4. \ub0b4\uac00 3\uad00\uc655\uc744 \ubabb\ud574\uc11c \uc544\uc26c\uc6b4 \uac83\uc774 \uc544\ub2c8\ub77c, \ud300 \uc804\uccb4\uac00 \ubaa8\ub450 \uae08\uba54\ub2ec\uc744 \ud568\uaed8 \ud560 \uc218 \uc788\ub294 \uae30\ud68c\ub97c \ub193\ucce4\uae30 \ub54c\ubb38\"\uc774\ub77c\uace0 \uae30\uc068\ubcf4\ub2e4\ub294 \uc544\uc26c\uc6c0\uc744 \ud45c\ud604\ud558\uc600\ub2e4.", "answer": "\uc911\uad6d", "sentence": "\ub2f9\uc2dc \uae40\uc544\ub791\uc740 \"\uae08\uba54\ub2ec 2\uac1c\ub97c \ucc28\uc9c0\ud55c \uac83\uc740 \uae30\uc058\uc9c0\ub9cc 3000\ubbf8\ud130 \uacc4\uc8fc\uc5d0\uc11c \uc911\uad6d \uc5d0\uac8c \uc11d\ud328\ud55c \uac83\uc774 \uc544\uc27d\ub2e4.", "paragraph_sentence": "\uae40\uc544\ub791\uc740 2014-2015 \uc2dc\uc98c \uad6d\uac00\ub300\ud45c \uc120\ubc1c\uc804\uc5d0\uc11c \uc5b4\uae68 \ubd80\uc0c1\uc744 \uc785\uc5c8\ub294\ub370 \uc774\uac83\uc740 \uc218\uc220\uc744 \ud544\uc694\ub85c \ud558\ub294 \ud68c\uc804\uadfc\uac1c(Rotator cuff) \uc190\uc0c1\uc774\uc5c8\uc73c\uba70 2015\ub144 \uad6d\uac00\ub300\ud45c \uc120\ubc1c\uc804\uc774 \ub05d\ub09c \ud6c4 \uc218\uc220\uc744 \ud558\uc600\ub2e4. 2014-2015 \uc2dc\uc98c\uc5d0\uc11c \uae40\uc544\ub791\uc740 5\ucc28\ub840 ISU \uc1fc\ud2b8\ud2b8\ub799 \uc6d4\ub4dc\ucef5\uc5d0 \ucd9c\uc804\ud558\uc5ec \uae08\uba54\ub2ec 3\uac1c \uc740\uba54\ub2ec 4\uac1c \ub3d9\uba54\ub2ec 1\uac1c\ub97c \ud68d\ub4dd\ud558\uc600\uc73c\uba70, 2015\ub144 2\uc6d4 \uc2a4\ud398\uc778 \uadf8\ub77c\ub098\ub2e4\uc5d0\uc11c \uc5f4\ub9b0 \ub3d9\uacc4 \uc720\ub2c8\ubc84\uc2dc\uc544\ub4dc \uc1fc\ud2b8\ud2b8\ub799 1000\ubbf8\ud130\uc640 1500\ubbf8\ud130\uc5d0\uc11c \uc6b0\uc2b9\ud558\uc5ec \uac1c\uc778\uc804 \uae08\uba54\ub2ec 2\uac1c\ub97c \ubaa9\uc5d0 \uac78\uc5c8\ub2e4. \ub2f9\uc2dc \uae40\uc544\ub791\uc740 \"\uae08\uba54\ub2ec 2\uac1c\ub97c \ucc28\uc9c0\ud55c \uac83\uc740 \uae30\uc058\uc9c0\ub9cc 3000\ubbf8\ud130 \uacc4\uc8fc\uc5d0\uc11c \uc911\uad6d \uc5d0\uac8c \uc11d\ud328\ud55c \uac83\uc774 \uc544\uc27d\ub2e4. \ub0b4\uac00 3\uad00\uc655\uc744 \ubabb\ud574\uc11c \uc544\uc26c\uc6b4 \uac83\uc774 \uc544\ub2c8\ub77c, \ud300 \uc804\uccb4\uac00 \ubaa8\ub450 \uae08\uba54\ub2ec\uc744 \ud568\uaed8 \ud560 \uc218 \uc788\ub294 \uae30\ud68c\ub97c \ub193\ucce4\uae30 \ub54c\ubb38\"\uc774\ub77c\uace0 \uae30\uc068\ubcf4\ub2e4\ub294 \uc544\uc26c\uc6c0\uc744 \ud45c\ud604\ud558\uc600\ub2e4.", "paragraph_answer": "\uae40\uc544\ub791\uc740 2014-2015 \uc2dc\uc98c \uad6d\uac00\ub300\ud45c \uc120\ubc1c\uc804\uc5d0\uc11c \uc5b4\uae68 \ubd80\uc0c1\uc744 \uc785\uc5c8\ub294\ub370 \uc774\uac83\uc740 \uc218\uc220\uc744 \ud544\uc694\ub85c \ud558\ub294 \ud68c\uc804\uadfc\uac1c(Rotator cuff) \uc190\uc0c1\uc774\uc5c8\uc73c\uba70 2015\ub144 \uad6d\uac00\ub300\ud45c \uc120\ubc1c\uc804\uc774 \ub05d\ub09c \ud6c4 \uc218\uc220\uc744 \ud558\uc600\ub2e4. 2014-2015 \uc2dc\uc98c\uc5d0\uc11c \uae40\uc544\ub791\uc740 5\ucc28\ub840 ISU \uc1fc\ud2b8\ud2b8\ub799 \uc6d4\ub4dc\ucef5\uc5d0 \ucd9c\uc804\ud558\uc5ec \uae08\uba54\ub2ec 3\uac1c \uc740\uba54\ub2ec 4\uac1c \ub3d9\uba54\ub2ec 1\uac1c\ub97c \ud68d\ub4dd\ud558\uc600\uc73c\uba70, 2015\ub144 2\uc6d4 \uc2a4\ud398\uc778 \uadf8\ub77c\ub098\ub2e4\uc5d0\uc11c \uc5f4\ub9b0 \ub3d9\uacc4 \uc720\ub2c8\ubc84\uc2dc\uc544\ub4dc \uc1fc\ud2b8\ud2b8\ub799 1000\ubbf8\ud130\uc640 1500\ubbf8\ud130\uc5d0\uc11c \uc6b0\uc2b9\ud558\uc5ec \uac1c\uc778\uc804 \uae08\uba54\ub2ec 2\uac1c\ub97c \ubaa9\uc5d0 \uac78\uc5c8\ub2e4. \ub2f9\uc2dc \uae40\uc544\ub791\uc740 \"\uae08\uba54\ub2ec 2\uac1c\ub97c \ucc28\uc9c0\ud55c \uac83\uc740 \uae30\uc058\uc9c0\ub9cc 3000\ubbf8\ud130 \uacc4\uc8fc\uc5d0\uc11c \uc911\uad6d \uc5d0\uac8c \uc11d\ud328\ud55c \uac83\uc774 \uc544\uc27d\ub2e4. \ub0b4\uac00 3\uad00\uc655\uc744 \ubabb\ud574\uc11c \uc544\uc26c\uc6b4 \uac83\uc774 \uc544\ub2c8\ub77c, \ud300 \uc804\uccb4\uac00 \ubaa8\ub450 \uae08\uba54\ub2ec\uc744 \ud568\uaed8 \ud560 \uc218 \uc788\ub294 \uae30\ud68c\ub97c \ub193\ucce4\uae30 \ub54c\ubb38\"\uc774\ub77c\uace0 \uae30\uc068\ubcf4\ub2e4\ub294 \uc544\uc26c\uc6c0\uc744 \ud45c\ud604\ud558\uc600\ub2e4.", "sentence_answer": "\ub2f9\uc2dc \uae40\uc544\ub791\uc740 \"\uae08\uba54\ub2ec 2\uac1c\ub97c \ucc28\uc9c0\ud55c \uac83\uc740 \uae30\uc058\uc9c0\ub9cc 3000\ubbf8\ud130 \uacc4\uc8fc\uc5d0\uc11c \uc911\uad6d \uc5d0\uac8c \uc11d\ud328\ud55c \uac83\uc774 \uc544\uc27d\ub2e4.", "paragraph_id": "6481874-2-2", "paragraph_question": "question: 3000\ubbf8\ud130 \uacc4\uc8fc\uc5d0\uc11c \ud55c\uad6d\uc744 \uc774\uae34 \ud300\uc740?, context: \uae40\uc544\ub791\uc740 2014-2015 \uc2dc\uc98c \uad6d\uac00\ub300\ud45c \uc120\ubc1c\uc804\uc5d0\uc11c \uc5b4\uae68 \ubd80\uc0c1\uc744 \uc785\uc5c8\ub294\ub370 \uc774\uac83\uc740 \uc218\uc220\uc744 \ud544\uc694\ub85c \ud558\ub294 \ud68c\uc804\uadfc\uac1c(Rotator cuff) \uc190\uc0c1\uc774\uc5c8\uc73c\uba70 2015\ub144 \uad6d\uac00\ub300\ud45c \uc120\ubc1c\uc804\uc774 \ub05d\ub09c \ud6c4 \uc218\uc220\uc744 \ud558\uc600\ub2e4. 2014-2015 \uc2dc\uc98c\uc5d0\uc11c \uae40\uc544\ub791\uc740 5\ucc28\ub840 ISU \uc1fc\ud2b8\ud2b8\ub799 \uc6d4\ub4dc\ucef5\uc5d0 \ucd9c\uc804\ud558\uc5ec \uae08\uba54\ub2ec 3\uac1c \uc740\uba54\ub2ec 4\uac1c \ub3d9\uba54\ub2ec 1\uac1c\ub97c \ud68d\ub4dd\ud558\uc600\uc73c\uba70, 2015\ub144 2\uc6d4 \uc2a4\ud398\uc778 \uadf8\ub77c\ub098\ub2e4\uc5d0\uc11c \uc5f4\ub9b0 \ub3d9\uacc4 \uc720\ub2c8\ubc84\uc2dc\uc544\ub4dc \uc1fc\ud2b8\ud2b8\ub799 1000\ubbf8\ud130\uc640 1500\ubbf8\ud130\uc5d0\uc11c \uc6b0\uc2b9\ud558\uc5ec \uac1c\uc778\uc804 \uae08\uba54\ub2ec 2\uac1c\ub97c \ubaa9\uc5d0 \uac78\uc5c8\ub2e4. \ub2f9\uc2dc \uae40\uc544\ub791\uc740 \"\uae08\uba54\ub2ec 2\uac1c\ub97c \ucc28\uc9c0\ud55c \uac83\uc740 \uae30\uc058\uc9c0\ub9cc 3000\ubbf8\ud130 \uacc4\uc8fc\uc5d0\uc11c \uc911\uad6d\uc5d0\uac8c \uc11d\ud328\ud55c \uac83\uc774 \uc544\uc27d\ub2e4. \ub0b4\uac00 3\uad00\uc655\uc744 \ubabb\ud574\uc11c \uc544\uc26c\uc6b4 \uac83\uc774 \uc544\ub2c8\ub77c, \ud300 \uc804\uccb4\uac00 \ubaa8\ub450 \uae08\uba54\ub2ec\uc744 \ud568\uaed8 \ud560 \uc218 \uc788\ub294 \uae30\ud68c\ub97c \ub193\ucce4\uae30 \ub54c\ubb38\"\uc774\ub77c\uace0 \uae30\uc068\ubcf4\ub2e4\ub294 \uc544\uc26c\uc6c0\uc744 \ud45c\ud604\ud558\uc600\ub2e4."} {"question": "\uc911\ud559\uad50\uc5d0\uc11c \uc2dc\ud5d8\uc744 \ud3d0\uc9c0\ud55c \ub54c\ub294?", "paragraph": "\ub300\ud559\uad50 \uc785\uc2dc \ubaa9\uc801\uc73c\ub85c \uace0\ub4f1\ud559\uad50\uc5d0\uc11c \uac15\uc81c\ub85c \uc2dc\ud589\ub418\ub358 \uc57c\uac04 \uc790\uc728\ud559\uc2b5\uacfc \uc6d4\ub9d0\uace0\uc0ac, \ud559\ub825\uace0\uc0ac, \ubaa8\uc758\uace0\uc0ac \ub4f1\uc744 \uc804\uba74 \ud3d0\uc9c0\ud558\uc600\uace0, \uc911\ud559\uad50\uc5d0\uc11c\ub3c4 1995\ub144 \ud3d0\uc9c0 \uc774\ud6c4\uc5d0\ub3c4 \ubd80\ubd84 \uc794\uc874\ud558\ub358 \uc5f0\ud569\uace0\uc0ac\uc640 \ud559\ub825\uace0\uc0ac, \ubaa8\uc758\uace0\uc0ac \ub4f1\uc744 \ud3d0\uc9c0\ud558\uc600\ub2e4. \ub610\ud55c \uac01\uae09 \ucd08\ub4f1\ud559\uad50, \uc911, \uace0\ub4f1\ud559\uad50\uc5d0 \ub300\ud55c \ubd88\uc2dc \uac10\uc0ac\ub97c \uc2dc\ud589\ud558\uc5ec \ucd0c\uc9c0\uad50\uc0ac\ub4e4\uc744 \ud574\uc784, \ud30c\uba74\ud558\uace0 \uad50\uc0ac\ub4e4\uc758 \uc131\uacfc\uc81c\ub97c \ub3c4\uc785\ud558\ub294 \ub4f1\uc758 \uc815\ucc45\uc740 \uad50\uc721\ubd80 \uc7a5\uad00 \uc7ac\uc9c1 \ub2f9\uc2dc \ud559\uc0dd\uacfc \ud559\ubd80\ubaa8 \ub4f1 \uad50\uc721 \uc218\uc694\uc790\uc5d0 \uae30\ubc18\ud55c \uc815\ucc45\uc774\ub77c\uba70 \uc77c\ubd80\ub294 \ud070 \ubc18\ud5a5\uc744 \ubd88\ub7ec \uc77c\uc73c\ucf30\ub2e4\uace0 \ud3c9\uac00\ud55c\ub2e4. \ud55c\ud3b8\uc73c\ub85c \uadf8\uc758 \uac1c\ud601\uc548\uc5d0 \ubc18\ubc1c\ud558\ub294 \uc77c\uc120 \uc911\ud559\uad50, \uace0\ub4f1\ud559\uad50 \uad50\uc0ac\ub4e4\uc758 \ubc18\ubc1c\uacfc \uc231\ud55c \ube44\ub09c \uacf5\uc138 \ub4f1\uc774 \uc5ed\uc2dc \ub9cc\ub9cc\uce58 \uc54a\uac8c \uc81c\uae30\ub418\uc5c8\uc73c\ub098 \uac15\ud589\ud558\uc600\ub2e4. \uad50\uc6d0 \uc815\ub144\uc744 \ub9cc65\uc138\uc5d0\uc11c \ub9cc62\uc138\ub85c \ub2e8\ucd95\ud558\ub294 \ub4f1\uc758 \uad50\uc6d0\uac1c\ud601\uc744 \ucd94\uc9c4\ud558\uace0 \uc644\ub8cc\ud558\uc600\ub2e4.", "answer": "1995\ub144", "sentence": "\uc911\ud559\uad50\uc5d0\uc11c\ub3c4 1995\ub144 \ud3d0\uc9c0 \uc774\ud6c4\uc5d0\ub3c4 \ubd80\ubd84 \uc794\uc874\ud558\ub358 \uc5f0\ud569\uace0\uc0ac\uc640 \ud559\ub825\uace0\uc0ac, \ubaa8\uc758\uace0\uc0ac \ub4f1\uc744 \ud3d0\uc9c0\ud558\uc600\ub2e4.", "paragraph_sentence": "\ub300\ud559\uad50 \uc785\uc2dc \ubaa9\uc801\uc73c\ub85c \uace0\ub4f1\ud559\uad50\uc5d0\uc11c \uac15\uc81c\ub85c \uc2dc\ud589\ub418\ub358 \uc57c\uac04 \uc790\uc728\ud559\uc2b5\uacfc \uc6d4\ub9d0\uace0\uc0ac, \ud559\ub825\uace0\uc0ac, \ubaa8\uc758\uace0\uc0ac \ub4f1\uc744 \uc804\uba74 \ud3d0\uc9c0\ud558\uc600\uace0, \uc911\ud559\uad50\uc5d0\uc11c\ub3c4 1995\ub144 \ud3d0\uc9c0 \uc774\ud6c4\uc5d0\ub3c4 \ubd80\ubd84 \uc794\uc874\ud558\ub358 \uc5f0\ud569\uace0\uc0ac\uc640 \ud559\ub825\uace0\uc0ac, \ubaa8\uc758\uace0\uc0ac \ub4f1\uc744 \ud3d0\uc9c0\ud558\uc600\ub2e4. \ub610\ud55c \uac01\uae09 \ucd08\ub4f1\ud559\uad50, \uc911, \uace0\ub4f1\ud559\uad50\uc5d0 \ub300\ud55c \ubd88\uc2dc \uac10\uc0ac\ub97c \uc2dc\ud589\ud558\uc5ec \ucd0c\uc9c0\uad50\uc0ac\ub4e4\uc744 \ud574\uc784, \ud30c\uba74\ud558\uace0 \uad50\uc0ac\ub4e4\uc758 \uc131\uacfc\uc81c\ub97c \ub3c4\uc785\ud558\ub294 \ub4f1\uc758 \uc815\ucc45\uc740 \uad50\uc721\ubd80 \uc7a5\uad00 \uc7ac\uc9c1 \ub2f9\uc2dc \ud559\uc0dd\uacfc \ud559\ubd80\ubaa8 \ub4f1 \uad50\uc721 \uc218\uc694\uc790\uc5d0 \uae30\ubc18\ud55c \uc815\ucc45\uc774\ub77c\uba70 \uc77c\ubd80\ub294 \ud070 \ubc18\ud5a5\uc744 \ubd88\ub7ec \uc77c\uc73c\ucf30\ub2e4\uace0 \ud3c9\uac00\ud55c\ub2e4. \ud55c\ud3b8\uc73c\ub85c \uadf8\uc758 \uac1c\ud601\uc548\uc5d0 \ubc18\ubc1c\ud558\ub294 \uc77c\uc120 \uc911\ud559\uad50, \uace0\ub4f1\ud559\uad50 \uad50\uc0ac\ub4e4\uc758 \ubc18\ubc1c\uacfc \uc231\ud55c \ube44\ub09c \uacf5\uc138 \ub4f1\uc774 \uc5ed\uc2dc \ub9cc\ub9cc\uce58 \uc54a\uac8c \uc81c\uae30\ub418\uc5c8\uc73c\ub098 \uac15\ud589\ud558\uc600\ub2e4. \uad50\uc6d0 \uc815\ub144\uc744 \ub9cc65\uc138\uc5d0\uc11c \ub9cc62\uc138\ub85c \ub2e8\ucd95\ud558\ub294 \ub4f1\uc758 \uad50\uc6d0\uac1c\ud601\uc744 \ucd94\uc9c4\ud558\uace0 \uc644\ub8cc\ud558\uc600\ub2e4.", "paragraph_answer": "\ub300\ud559\uad50 \uc785\uc2dc \ubaa9\uc801\uc73c\ub85c \uace0\ub4f1\ud559\uad50\uc5d0\uc11c \uac15\uc81c\ub85c \uc2dc\ud589\ub418\ub358 \uc57c\uac04 \uc790\uc728\ud559\uc2b5\uacfc \uc6d4\ub9d0\uace0\uc0ac, \ud559\ub825\uace0\uc0ac, \ubaa8\uc758\uace0\uc0ac \ub4f1\uc744 \uc804\uba74 \ud3d0\uc9c0\ud558\uc600\uace0, \uc911\ud559\uad50\uc5d0\uc11c\ub3c4 1995\ub144 \ud3d0\uc9c0 \uc774\ud6c4\uc5d0\ub3c4 \ubd80\ubd84 \uc794\uc874\ud558\ub358 \uc5f0\ud569\uace0\uc0ac\uc640 \ud559\ub825\uace0\uc0ac, \ubaa8\uc758\uace0\uc0ac \ub4f1\uc744 \ud3d0\uc9c0\ud558\uc600\ub2e4. \ub610\ud55c \uac01\uae09 \ucd08\ub4f1\ud559\uad50, \uc911, \uace0\ub4f1\ud559\uad50\uc5d0 \ub300\ud55c \ubd88\uc2dc \uac10\uc0ac\ub97c \uc2dc\ud589\ud558\uc5ec \ucd0c\uc9c0\uad50\uc0ac\ub4e4\uc744 \ud574\uc784, \ud30c\uba74\ud558\uace0 \uad50\uc0ac\ub4e4\uc758 \uc131\uacfc\uc81c\ub97c \ub3c4\uc785\ud558\ub294 \ub4f1\uc758 \uc815\ucc45\uc740 \uad50\uc721\ubd80 \uc7a5\uad00 \uc7ac\uc9c1 \ub2f9\uc2dc \ud559\uc0dd\uacfc \ud559\ubd80\ubaa8 \ub4f1 \uad50\uc721 \uc218\uc694\uc790\uc5d0 \uae30\ubc18\ud55c \uc815\ucc45\uc774\ub77c\uba70 \uc77c\ubd80\ub294 \ud070 \ubc18\ud5a5\uc744 \ubd88\ub7ec \uc77c\uc73c\ucf30\ub2e4\uace0 \ud3c9\uac00\ud55c\ub2e4. \ud55c\ud3b8\uc73c\ub85c \uadf8\uc758 \uac1c\ud601\uc548\uc5d0 \ubc18\ubc1c\ud558\ub294 \uc77c\uc120 \uc911\ud559\uad50, \uace0\ub4f1\ud559\uad50 \uad50\uc0ac\ub4e4\uc758 \ubc18\ubc1c\uacfc \uc231\ud55c \ube44\ub09c \uacf5\uc138 \ub4f1\uc774 \uc5ed\uc2dc \ub9cc\ub9cc\uce58 \uc54a\uac8c \uc81c\uae30\ub418\uc5c8\uc73c\ub098 \uac15\ud589\ud558\uc600\ub2e4. \uad50\uc6d0 \uc815\ub144\uc744 \ub9cc65\uc138\uc5d0\uc11c \ub9cc62\uc138\ub85c \ub2e8\ucd95\ud558\ub294 \ub4f1\uc758 \uad50\uc6d0\uac1c\ud601\uc744 \ucd94\uc9c4\ud558\uace0 \uc644\ub8cc\ud558\uc600\ub2e4.", "sentence_answer": "\uc911\ud559\uad50\uc5d0\uc11c\ub3c4 1995\ub144 \ud3d0\uc9c0 \uc774\ud6c4\uc5d0\ub3c4 \ubd80\ubd84 \uc794\uc874\ud558\ub358 \uc5f0\ud569\uace0\uc0ac\uc640 \ud559\ub825\uace0\uc0ac, \ubaa8\uc758\uace0\uc0ac \ub4f1\uc744 \ud3d0\uc9c0\ud558\uc600\ub2e4.", "paragraph_id": "5849814-1-0", "paragraph_question": "question: \uc911\ud559\uad50\uc5d0\uc11c \uc2dc\ud5d8\uc744 \ud3d0\uc9c0\ud55c \ub54c\ub294?, context: \ub300\ud559\uad50 \uc785\uc2dc \ubaa9\uc801\uc73c\ub85c \uace0\ub4f1\ud559\uad50\uc5d0\uc11c \uac15\uc81c\ub85c \uc2dc\ud589\ub418\ub358 \uc57c\uac04 \uc790\uc728\ud559\uc2b5\uacfc \uc6d4\ub9d0\uace0\uc0ac, \ud559\ub825\uace0\uc0ac, \ubaa8\uc758\uace0\uc0ac \ub4f1\uc744 \uc804\uba74 \ud3d0\uc9c0\ud558\uc600\uace0, \uc911\ud559\uad50\uc5d0\uc11c\ub3c4 1995\ub144 \ud3d0\uc9c0 \uc774\ud6c4\uc5d0\ub3c4 \ubd80\ubd84 \uc794\uc874\ud558\ub358 \uc5f0\ud569\uace0\uc0ac\uc640 \ud559\ub825\uace0\uc0ac, \ubaa8\uc758\uace0\uc0ac \ub4f1\uc744 \ud3d0\uc9c0\ud558\uc600\ub2e4. \ub610\ud55c \uac01\uae09 \ucd08\ub4f1\ud559\uad50, \uc911, \uace0\ub4f1\ud559\uad50\uc5d0 \ub300\ud55c \ubd88\uc2dc \uac10\uc0ac\ub97c \uc2dc\ud589\ud558\uc5ec \ucd0c\uc9c0\uad50\uc0ac\ub4e4\uc744 \ud574\uc784, \ud30c\uba74\ud558\uace0 \uad50\uc0ac\ub4e4\uc758 \uc131\uacfc\uc81c\ub97c \ub3c4\uc785\ud558\ub294 \ub4f1\uc758 \uc815\ucc45\uc740 \uad50\uc721\ubd80 \uc7a5\uad00 \uc7ac\uc9c1 \ub2f9\uc2dc \ud559\uc0dd\uacfc \ud559\ubd80\ubaa8 \ub4f1 \uad50\uc721 \uc218\uc694\uc790\uc5d0 \uae30\ubc18\ud55c \uc815\ucc45\uc774\ub77c\uba70 \uc77c\ubd80\ub294 \ud070 \ubc18\ud5a5\uc744 \ubd88\ub7ec \uc77c\uc73c\ucf30\ub2e4\uace0 \ud3c9\uac00\ud55c\ub2e4. \ud55c\ud3b8\uc73c\ub85c \uadf8\uc758 \uac1c\ud601\uc548\uc5d0 \ubc18\ubc1c\ud558\ub294 \uc77c\uc120 \uc911\ud559\uad50, \uace0\ub4f1\ud559\uad50 \uad50\uc0ac\ub4e4\uc758 \ubc18\ubc1c\uacfc \uc231\ud55c \ube44\ub09c \uacf5\uc138 \ub4f1\uc774 \uc5ed\uc2dc \ub9cc\ub9cc\uce58 \uc54a\uac8c \uc81c\uae30\ub418\uc5c8\uc73c\ub098 \uac15\ud589\ud558\uc600\ub2e4. \uad50\uc6d0 \uc815\ub144\uc744 \ub9cc65\uc138\uc5d0\uc11c \ub9cc62\uc138\ub85c \ub2e8\ucd95\ud558\ub294 \ub4f1\uc758 \uad50\uc6d0\uac1c\ud601\uc744 \ucd94\uc9c4\ud558\uace0 \uc644\ub8cc\ud558\uc600\ub2e4."} {"question": "\ud5e8\ub9ac \ud78c\ub9ac\uc13c\uc740 \uba87 \ub144\ub3c4\uc5d0 \uac8c\uc624\ub974\ud06c \uad3c\ub7ec\uac00 \uc5f0\uc8fc\ub97c \ud76c\ub9dd\ud588\uc744 \ub54c\ub3c4 \ucd5c\uc885\ud310\uc744 \ub9cc\ub4e4\uc9c0 \uc54a\uc558\ub098\uc694?", "paragraph": "\ub9d0\ub7ec\ub85c\uc120 \uc218\ub3c4 \uc5c6\ub294 \uac1c\uc815\uc744 \uac70\uce58\uba70 \ub098\uc544\uc9c0\ub294 \uc624\ucf00\uc2a4\ud2b8\ub808\uc774\uc158\uc5d0 \ub9e4\uc6b0 \ubfcc\ub4ef\ud588\uaca0\uc9c0\ub9cc, \uadf8\uac78 \ucd9c\ud310\ud574\uc57c \ud558\ub294 \ucd9c\ud310\uc5c5\uc790\uc758 \uc785\uc7a5\uc740 \uace8\uce58\uac00 \uc544\ud50c\uc218\ubc16\uc5d0 \uc5c6\uc5c8\ub2e4. \uc624\ub298\ub0a0\uc774\uc57c \ucef4\ud4e8\ud130 \uc785\ub825\uc73c\ub85c \uc778\uc1c4\uac00 \uc27d\uac8c \uac00\ub2a5\ud558\ub2e4\uc9c0\ub9cc \uadf8 \ub2f9\uc2dc\ub294 \uae08\uc18d\ud310\ud615\uc744 \ub9cc\ub4e4\uc5b4 \ucc0d\uc5b4\ub0b4\ub294 \ubc29\uc2dd\uc774\uc5c8\uc73c\ub2c8 \uacc4\uc18d \ubc14\ub00c\ub294 \uc545\ubcf4\ub300\ub85c \uae08\uc18d\ud310\ud615\uc744 \ub9cc\ub4e4\uc5b4\ub0b4\ub294\uac83\ub3c4 \uace0\uc5ed\uc774\uc5c8\uc744\uac8c \ubd84\uba85\ud558\ub2e4. \uadf8\ub9cc\ud07c \ub9ce\uc740 \ub3c8\uc774 \ub4e4\uc5b4\uac14\uc744\uac83\uc740 \ub9d0\uc548\ud574\ub3c4 \ubed4\ud55c \uc77c\uc774\uc5c8\uc744 \uac83\uc774\uace0. \uacb0\uad6d \uacc4\uc18d\ub418\ub294 \uc218\uc815\uc5d0 \uc9dc\uc99d\ub0a0\ub300\ub85c \uc9dc\uc99d\uc774 \ub0ac\ub358 \uc774 \ubb38\uc81c\uc758 \uc8fc\uc778\uacf5, C.F. \ud398\ud130\uc2a4 \ucd9c\ud310\uc0ac\uc758 \uc18c\uc720\uc790\uc600\ub358 \ud5e8\ub9ac \ud78c\ub9ac\uc13c\uc740 \uacc4\uc18d \uc218\uc815\ub418\ub294\ub300\ub85c \uae08\uc18d\ud310\ud615\uc744 \ub9cc\ub4dc\ub294\ub370 \ub108\ubb34 \ub9ce\uc740 \ub3c8\uc744 \uc37c\ub2e4\uace0 \uc0dd\uac01\ud574 1913\ub144\uc5d0 \uac8c\uc624\ub974\ud06c \uad3c\ub7ec\uac00 \uc5f0\uc8fc\ub97c \ud76c\ub9dd\ud588\uc744 \ub54c\ub3c4 \ucd5c\uc885\ud310\uc744 \ub9cc\ub4e4\uc9c0 \uc54a\uc558\ub2e4. \uac8c\ub2e4\uac00 \uc9dc\uc99d\ub098\uac8c \ub9cc\ub4e0 \ub9d0\ub7ec\uc5d0\uac8c \ubcf5\uc218\ud560 \uc0dd\uac01\uc774\uc5c8\ub358 \uac83\uc778\uc9c0\ub294 \ubaa8\ub974\uaca0\uc9c0\ub9cc, \uc778\uc1c4\ud310\uae4c\uc9c0 \uc5c6\uc560\uaca0\ub2e4\uace0 \uc120\uc5b8\ud574\ubc84\ub838\ub2e4.", "answer": "1913\ub144", "sentence": "\uacb0\uad6d \uacc4\uc18d\ub418\ub294 \uc218\uc815\uc5d0 \uc9dc\uc99d\ub0a0\ub300\ub85c \uc9dc\uc99d\uc774 \ub0ac\ub358 \uc774 \ubb38\uc81c\uc758 \uc8fc\uc778\uacf5, C.F. \ud398\ud130\uc2a4 \ucd9c\ud310\uc0ac\uc758 \uc18c\uc720\uc790\uc600\ub358 \ud5e8\ub9ac \ud78c\ub9ac\uc13c\uc740 \uacc4\uc18d \uc218\uc815\ub418\ub294\ub300\ub85c \uae08\uc18d\ud310\ud615\uc744 \ub9cc\ub4dc\ub294\ub370 \ub108\ubb34 \ub9ce\uc740 \ub3c8\uc744 \uc37c\ub2e4\uace0 \uc0dd\uac01\ud574 1913\ub144 \uc5d0 \uac8c\uc624\ub974\ud06c \uad3c\ub7ec\uac00 \uc5f0\uc8fc\ub97c \ud76c\ub9dd\ud588\uc744 \ub54c\ub3c4 \ucd5c\uc885\ud310\uc744 \ub9cc\ub4e4\uc9c0 \uc54a\uc558\ub2e4.", "paragraph_sentence": "\ub9d0\ub7ec\ub85c\uc120 \uc218\ub3c4 \uc5c6\ub294 \uac1c\uc815\uc744 \uac70\uce58\uba70 \ub098\uc544\uc9c0\ub294 \uc624\ucf00\uc2a4\ud2b8\ub808\uc774\uc158\uc5d0 \ub9e4\uc6b0 \ubfcc\ub4ef\ud588\uaca0\uc9c0\ub9cc, \uadf8\uac78 \ucd9c\ud310\ud574\uc57c \ud558\ub294 \ucd9c\ud310\uc5c5\uc790\uc758 \uc785\uc7a5\uc740 \uace8\uce58\uac00 \uc544\ud50c\uc218\ubc16\uc5d0 \uc5c6\uc5c8\ub2e4. \uc624\ub298\ub0a0\uc774\uc57c \ucef4\ud4e8\ud130 \uc785\ub825\uc73c\ub85c \uc778\uc1c4\uac00 \uc27d\uac8c \uac00\ub2a5\ud558\ub2e4\uc9c0\ub9cc \uadf8 \ub2f9\uc2dc\ub294 \uae08\uc18d\ud310\ud615\uc744 \ub9cc\ub4e4\uc5b4 \ucc0d\uc5b4\ub0b4\ub294 \ubc29\uc2dd\uc774\uc5c8\uc73c\ub2c8 \uacc4\uc18d \ubc14\ub00c\ub294 \uc545\ubcf4\ub300\ub85c \uae08\uc18d\ud310\ud615\uc744 \ub9cc\ub4e4\uc5b4\ub0b4\ub294\uac83\ub3c4 \uace0\uc5ed\uc774\uc5c8\uc744\uac8c \ubd84\uba85\ud558\ub2e4. \uadf8\ub9cc\ud07c \ub9ce\uc740 \ub3c8\uc774 \ub4e4\uc5b4\uac14\uc744\uac83\uc740 \ub9d0\uc548\ud574\ub3c4 \ubed4\ud55c \uc77c\uc774\uc5c8\uc744 \uac83\uc774\uace0. \uacb0\uad6d \uacc4\uc18d\ub418\ub294 \uc218\uc815\uc5d0 \uc9dc\uc99d\ub0a0\ub300\ub85c \uc9dc\uc99d\uc774 \ub0ac\ub358 \uc774 \ubb38\uc81c\uc758 \uc8fc\uc778\uacf5, C.F. \ud398\ud130\uc2a4 \ucd9c\ud310\uc0ac\uc758 \uc18c\uc720\uc790\uc600\ub358 \ud5e8\ub9ac \ud78c\ub9ac\uc13c\uc740 \uacc4\uc18d \uc218\uc815\ub418\ub294\ub300\ub85c \uae08\uc18d\ud310\ud615\uc744 \ub9cc\ub4dc\ub294\ub370 \ub108\ubb34 \ub9ce\uc740 \ub3c8\uc744 \uc37c\ub2e4\uace0 \uc0dd\uac01\ud574 1913\ub144 \uc5d0 \uac8c\uc624\ub974\ud06c \uad3c\ub7ec\uac00 \uc5f0\uc8fc\ub97c \ud76c\ub9dd\ud588\uc744 \ub54c\ub3c4 \ucd5c\uc885\ud310\uc744 \ub9cc\ub4e4\uc9c0 \uc54a\uc558\ub2e4. \uac8c\ub2e4\uac00 \uc9dc\uc99d\ub098\uac8c \ub9cc\ub4e0 \ub9d0\ub7ec\uc5d0\uac8c \ubcf5\uc218\ud560 \uc0dd\uac01\uc774\uc5c8\ub358 \uac83\uc778\uc9c0\ub294 \ubaa8\ub974\uaca0\uc9c0\ub9cc, \uc778\uc1c4\ud310\uae4c\uc9c0 \uc5c6\uc560\uaca0\ub2e4\uace0 \uc120\uc5b8\ud574\ubc84\ub838\ub2e4.", "paragraph_answer": "\ub9d0\ub7ec\ub85c\uc120 \uc218\ub3c4 \uc5c6\ub294 \uac1c\uc815\uc744 \uac70\uce58\uba70 \ub098\uc544\uc9c0\ub294 \uc624\ucf00\uc2a4\ud2b8\ub808\uc774\uc158\uc5d0 \ub9e4\uc6b0 \ubfcc\ub4ef\ud588\uaca0\uc9c0\ub9cc, \uadf8\uac78 \ucd9c\ud310\ud574\uc57c \ud558\ub294 \ucd9c\ud310\uc5c5\uc790\uc758 \uc785\uc7a5\uc740 \uace8\uce58\uac00 \uc544\ud50c\uc218\ubc16\uc5d0 \uc5c6\uc5c8\ub2e4. \uc624\ub298\ub0a0\uc774\uc57c \ucef4\ud4e8\ud130 \uc785\ub825\uc73c\ub85c \uc778\uc1c4\uac00 \uc27d\uac8c \uac00\ub2a5\ud558\ub2e4\uc9c0\ub9cc \uadf8 \ub2f9\uc2dc\ub294 \uae08\uc18d\ud310\ud615\uc744 \ub9cc\ub4e4\uc5b4 \ucc0d\uc5b4\ub0b4\ub294 \ubc29\uc2dd\uc774\uc5c8\uc73c\ub2c8 \uacc4\uc18d \ubc14\ub00c\ub294 \uc545\ubcf4\ub300\ub85c \uae08\uc18d\ud310\ud615\uc744 \ub9cc\ub4e4\uc5b4\ub0b4\ub294\uac83\ub3c4 \uace0\uc5ed\uc774\uc5c8\uc744\uac8c \ubd84\uba85\ud558\ub2e4. \uadf8\ub9cc\ud07c \ub9ce\uc740 \ub3c8\uc774 \ub4e4\uc5b4\uac14\uc744\uac83\uc740 \ub9d0\uc548\ud574\ub3c4 \ubed4\ud55c \uc77c\uc774\uc5c8\uc744 \uac83\uc774\uace0. \uacb0\uad6d \uacc4\uc18d\ub418\ub294 \uc218\uc815\uc5d0 \uc9dc\uc99d\ub0a0\ub300\ub85c \uc9dc\uc99d\uc774 \ub0ac\ub358 \uc774 \ubb38\uc81c\uc758 \uc8fc\uc778\uacf5, C.F. \ud398\ud130\uc2a4 \ucd9c\ud310\uc0ac\uc758 \uc18c\uc720\uc790\uc600\ub358 \ud5e8\ub9ac \ud78c\ub9ac\uc13c\uc740 \uacc4\uc18d \uc218\uc815\ub418\ub294\ub300\ub85c \uae08\uc18d\ud310\ud615\uc744 \ub9cc\ub4dc\ub294\ub370 \ub108\ubb34 \ub9ce\uc740 \ub3c8\uc744 \uc37c\ub2e4\uace0 \uc0dd\uac01\ud574 1913\ub144 \uc5d0 \uac8c\uc624\ub974\ud06c \uad3c\ub7ec\uac00 \uc5f0\uc8fc\ub97c \ud76c\ub9dd\ud588\uc744 \ub54c\ub3c4 \ucd5c\uc885\ud310\uc744 \ub9cc\ub4e4\uc9c0 \uc54a\uc558\ub2e4. \uac8c\ub2e4\uac00 \uc9dc\uc99d\ub098\uac8c \ub9cc\ub4e0 \ub9d0\ub7ec\uc5d0\uac8c \ubcf5\uc218\ud560 \uc0dd\uac01\uc774\uc5c8\ub358 \uac83\uc778\uc9c0\ub294 \ubaa8\ub974\uaca0\uc9c0\ub9cc, \uc778\uc1c4\ud310\uae4c\uc9c0 \uc5c6\uc560\uaca0\ub2e4\uace0 \uc120\uc5b8\ud574\ubc84\ub838\ub2e4.", "sentence_answer": "\uacb0\uad6d \uacc4\uc18d\ub418\ub294 \uc218\uc815\uc5d0 \uc9dc\uc99d\ub0a0\ub300\ub85c \uc9dc\uc99d\uc774 \ub0ac\ub358 \uc774 \ubb38\uc81c\uc758 \uc8fc\uc778\uacf5, C.F. \ud398\ud130\uc2a4 \ucd9c\ud310\uc0ac\uc758 \uc18c\uc720\uc790\uc600\ub358 \ud5e8\ub9ac \ud78c\ub9ac\uc13c\uc740 \uacc4\uc18d \uc218\uc815\ub418\ub294\ub300\ub85c \uae08\uc18d\ud310\ud615\uc744 \ub9cc\ub4dc\ub294\ub370 \ub108\ubb34 \ub9ce\uc740 \ub3c8\uc744 \uc37c\ub2e4\uace0 \uc0dd\uac01\ud574 1913\ub144 \uc5d0 \uac8c\uc624\ub974\ud06c \uad3c\ub7ec\uac00 \uc5f0\uc8fc\ub97c \ud76c\ub9dd\ud588\uc744 \ub54c\ub3c4 \ucd5c\uc885\ud310\uc744 \ub9cc\ub4e4\uc9c0 \uc54a\uc558\ub2e4.", "paragraph_id": "6472086-2-1", "paragraph_question": "question: \ud5e8\ub9ac \ud78c\ub9ac\uc13c\uc740 \uba87 \ub144\ub3c4\uc5d0 \uac8c\uc624\ub974\ud06c \uad3c\ub7ec\uac00 \uc5f0\uc8fc\ub97c \ud76c\ub9dd\ud588\uc744 \ub54c\ub3c4 \ucd5c\uc885\ud310\uc744 \ub9cc\ub4e4\uc9c0 \uc54a\uc558\ub098\uc694?, context: \ub9d0\ub7ec\ub85c\uc120 \uc218\ub3c4 \uc5c6\ub294 \uac1c\uc815\uc744 \uac70\uce58\uba70 \ub098\uc544\uc9c0\ub294 \uc624\ucf00\uc2a4\ud2b8\ub808\uc774\uc158\uc5d0 \ub9e4\uc6b0 \ubfcc\ub4ef\ud588\uaca0\uc9c0\ub9cc, \uadf8\uac78 \ucd9c\ud310\ud574\uc57c \ud558\ub294 \ucd9c\ud310\uc5c5\uc790\uc758 \uc785\uc7a5\uc740 \uace8\uce58\uac00 \uc544\ud50c\uc218\ubc16\uc5d0 \uc5c6\uc5c8\ub2e4. \uc624\ub298\ub0a0\uc774\uc57c \ucef4\ud4e8\ud130 \uc785\ub825\uc73c\ub85c \uc778\uc1c4\uac00 \uc27d\uac8c \uac00\ub2a5\ud558\ub2e4\uc9c0\ub9cc \uadf8 \ub2f9\uc2dc\ub294 \uae08\uc18d\ud310\ud615\uc744 \ub9cc\ub4e4\uc5b4 \ucc0d\uc5b4\ub0b4\ub294 \ubc29\uc2dd\uc774\uc5c8\uc73c\ub2c8 \uacc4\uc18d \ubc14\ub00c\ub294 \uc545\ubcf4\ub300\ub85c \uae08\uc18d\ud310\ud615\uc744 \ub9cc\ub4e4\uc5b4\ub0b4\ub294\uac83\ub3c4 \uace0\uc5ed\uc774\uc5c8\uc744\uac8c \ubd84\uba85\ud558\ub2e4. \uadf8\ub9cc\ud07c \ub9ce\uc740 \ub3c8\uc774 \ub4e4\uc5b4\uac14\uc744\uac83\uc740 \ub9d0\uc548\ud574\ub3c4 \ubed4\ud55c \uc77c\uc774\uc5c8\uc744 \uac83\uc774\uace0. \uacb0\uad6d \uacc4\uc18d\ub418\ub294 \uc218\uc815\uc5d0 \uc9dc\uc99d\ub0a0\ub300\ub85c \uc9dc\uc99d\uc774 \ub0ac\ub358 \uc774 \ubb38\uc81c\uc758 \uc8fc\uc778\uacf5, C.F. \ud398\ud130\uc2a4 \ucd9c\ud310\uc0ac\uc758 \uc18c\uc720\uc790\uc600\ub358 \ud5e8\ub9ac \ud78c\ub9ac\uc13c\uc740 \uacc4\uc18d \uc218\uc815\ub418\ub294\ub300\ub85c \uae08\uc18d\ud310\ud615\uc744 \ub9cc\ub4dc\ub294\ub370 \ub108\ubb34 \ub9ce\uc740 \ub3c8\uc744 \uc37c\ub2e4\uace0 \uc0dd\uac01\ud574 1913\ub144\uc5d0 \uac8c\uc624\ub974\ud06c \uad3c\ub7ec\uac00 \uc5f0\uc8fc\ub97c \ud76c\ub9dd\ud588\uc744 \ub54c\ub3c4 \ucd5c\uc885\ud310\uc744 \ub9cc\ub4e4\uc9c0 \uc54a\uc558\ub2e4. \uac8c\ub2e4\uac00 \uc9dc\uc99d\ub098\uac8c \ub9cc\ub4e0 \ub9d0\ub7ec\uc5d0\uac8c \ubcf5\uc218\ud560 \uc0dd\uac01\uc774\uc5c8\ub358 \uac83\uc778\uc9c0\ub294 \ubaa8\ub974\uaca0\uc9c0\ub9cc, \uc778\uc1c4\ud310\uae4c\uc9c0 \uc5c6\uc560\uaca0\ub2e4\uace0 \uc120\uc5b8\ud574\ubc84\ub838\ub2e4."} {"question": "\uc774\ud718\uc18c\uac00 \ubc15\uc0ac\ud559\uc704\ub97c \ubc1b\uc740 \ub098\uc774\ub294?", "paragraph": "\ud074\ub77c\uc778\uc740 \ub2f9\uc2dc \uc11c\ub978 \uc138 \uc0b4\uc758 \uc80a\uc740 \uad50\uc218\uc600\ub294\ub370, \uc774\ud718\uc18c\ub294 \ud074\ub77c\uc778\uacfc \ud568\uaed8 \uacf5\ub3d9 \uc5f0\uad6c\ub97c \uc218\ud589\ud558\uba74\uc11c \uc790\uc2e0\uc758 \ubc15\uc0ac \ud559\uc704 \ub17c\ubb38\uc5d0 \uc870\uae08\uc529 \uc811\uadfc\ud574 \ub098\uac14\ub2e4. \uadf8\ub7ec\ub2e4 1960\ub144 11\uc6d4\uc5d0 \u3008K \uc911\uac04\uc790\uc640 \ud575\uc790 \uc0b0\ub780 \ud604\uc0c1\uc758 \uc774\uc911 \ubd84\uc0b0 \uad00\uacc4\u3009(Study of K Scattering in the Double Dispersion Representation)\uc73c\ub85c \ubb3c\ub9ac\ud559 \ubc15\uc0ac(Doctor of Philosophy) \ud559\uc704\ub97c \ubc1b\uc558\ub294\ub370 \uc774 \ub54c \ub098\uc774\ub294 \ubd88\uacfc 25\uc138\uc600\ub2e4. \uadf8\uac00 \ubc15\uc0ac \ud559\uc704\uc99d\uc744 \uacf5\uc2dd\uc801\uc73c\ub85c \ubc1b\uc740 \uac83\uc740 1961\ub144 2\uc6d4 4\uc77c\uc774\uc5c8\ub2e4. \ubc15\uc0ac \ub17c\ubb38 \ub514\ud39c\uc2a4\uac00 \ub05d\ub09c 11\uc6d4\ubd80\ud130 1961\ub144 8\uc6d4\uae4c\uc9c0, \uc774\ud718\uc18c\ub294 \ud39c\uc2e4\ubca0\uc774\ub2c8\uc544 \ub300\ud559\uad50\uc758 \ubc15\uc0ac\ud6c4 \uc5f0\uad6c\uc6d0 \ubc0f \uc804\uc784 \uac15\uc0ac\ub85c \uc784\uc6a9\ub410\ub2e4.", "answer": "25\uc138", "sentence": "\uc774 \ub54c \ub098\uc774\ub294 \ubd88\uacfc 25\uc138 \uc600\ub2e4.", "paragraph_sentence": "\ud074\ub77c\uc778\uc740 \ub2f9\uc2dc \uc11c\ub978 \uc138 \uc0b4\uc758 \uc80a\uc740 \uad50\uc218\uc600\ub294\ub370, \uc774\ud718\uc18c\ub294 \ud074\ub77c\uc778\uacfc \ud568\uaed8 \uacf5\ub3d9 \uc5f0\uad6c\ub97c \uc218\ud589\ud558\uba74\uc11c \uc790\uc2e0\uc758 \ubc15\uc0ac \ud559\uc704 \ub17c\ubb38\uc5d0 \uc870\uae08\uc529 \uc811\uadfc\ud574 \ub098\uac14\ub2e4. \uadf8\ub7ec\ub2e4 1960\ub144 11\uc6d4\uc5d0 \u3008K \uc911\uac04\uc790\uc640 \ud575\uc790 \uc0b0\ub780 \ud604\uc0c1\uc758 \uc774\uc911 \ubd84\uc0b0 \uad00\uacc4\u3009(Study of K Scattering in the Double Dispersion Representation)\uc73c\ub85c \ubb3c\ub9ac\ud559 \ubc15\uc0ac(Doctor of Philosophy) \ud559\uc704\ub97c \ubc1b\uc558\ub294\ub370 \uc774 \ub54c \ub098\uc774\ub294 \ubd88\uacfc 25\uc138 \uc600\ub2e4. \uadf8\uac00 \ubc15\uc0ac \ud559\uc704\uc99d\uc744 \uacf5\uc2dd\uc801\uc73c\ub85c \ubc1b\uc740 \uac83\uc740 1961\ub144 2\uc6d4 4\uc77c\uc774\uc5c8\ub2e4. \ubc15\uc0ac \ub17c\ubb38 \ub514\ud39c\uc2a4\uac00 \ub05d\ub09c 11\uc6d4\ubd80\ud130 1961\ub144 8\uc6d4\uae4c\uc9c0, \uc774\ud718\uc18c\ub294 \ud39c\uc2e4\ubca0\uc774\ub2c8\uc544 \ub300\ud559\uad50\uc758 \ubc15\uc0ac\ud6c4 \uc5f0\uad6c\uc6d0 \ubc0f \uc804\uc784 \uac15\uc0ac\ub85c \uc784\uc6a9\ub410\ub2e4.", "paragraph_answer": "\ud074\ub77c\uc778\uc740 \ub2f9\uc2dc \uc11c\ub978 \uc138 \uc0b4\uc758 \uc80a\uc740 \uad50\uc218\uc600\ub294\ub370, \uc774\ud718\uc18c\ub294 \ud074\ub77c\uc778\uacfc \ud568\uaed8 \uacf5\ub3d9 \uc5f0\uad6c\ub97c \uc218\ud589\ud558\uba74\uc11c \uc790\uc2e0\uc758 \ubc15\uc0ac \ud559\uc704 \ub17c\ubb38\uc5d0 \uc870\uae08\uc529 \uc811\uadfc\ud574 \ub098\uac14\ub2e4. \uadf8\ub7ec\ub2e4 1960\ub144 11\uc6d4\uc5d0 \u3008K \uc911\uac04\uc790\uc640 \ud575\uc790 \uc0b0\ub780 \ud604\uc0c1\uc758 \uc774\uc911 \ubd84\uc0b0 \uad00\uacc4\u3009(Study of K Scattering in the Double Dispersion Representation)\uc73c\ub85c \ubb3c\ub9ac\ud559 \ubc15\uc0ac(Doctor of Philosophy) \ud559\uc704\ub97c \ubc1b\uc558\ub294\ub370 \uc774 \ub54c \ub098\uc774\ub294 \ubd88\uacfc 25\uc138 \uc600\ub2e4. \uadf8\uac00 \ubc15\uc0ac \ud559\uc704\uc99d\uc744 \uacf5\uc2dd\uc801\uc73c\ub85c \ubc1b\uc740 \uac83\uc740 1961\ub144 2\uc6d4 4\uc77c\uc774\uc5c8\ub2e4. \ubc15\uc0ac \ub17c\ubb38 \ub514\ud39c\uc2a4\uac00 \ub05d\ub09c 11\uc6d4\ubd80\ud130 1961\ub144 8\uc6d4\uae4c\uc9c0, \uc774\ud718\uc18c\ub294 \ud39c\uc2e4\ubca0\uc774\ub2c8\uc544 \ub300\ud559\uad50\uc758 \ubc15\uc0ac\ud6c4 \uc5f0\uad6c\uc6d0 \ubc0f \uc804\uc784 \uac15\uc0ac\ub85c \uc784\uc6a9\ub410\ub2e4.", "sentence_answer": "\uc774 \ub54c \ub098\uc774\ub294 \ubd88\uacfc 25\uc138 \uc600\ub2e4.", "paragraph_id": "6329268-5-0", "paragraph_question": "question: \uc774\ud718\uc18c\uac00 \ubc15\uc0ac\ud559\uc704\ub97c \ubc1b\uc740 \ub098\uc774\ub294?, context: \ud074\ub77c\uc778\uc740 \ub2f9\uc2dc \uc11c\ub978 \uc138 \uc0b4\uc758 \uc80a\uc740 \uad50\uc218\uc600\ub294\ub370, \uc774\ud718\uc18c\ub294 \ud074\ub77c\uc778\uacfc \ud568\uaed8 \uacf5\ub3d9 \uc5f0\uad6c\ub97c \uc218\ud589\ud558\uba74\uc11c \uc790\uc2e0\uc758 \ubc15\uc0ac \ud559\uc704 \ub17c\ubb38\uc5d0 \uc870\uae08\uc529 \uc811\uadfc\ud574 \ub098\uac14\ub2e4. \uadf8\ub7ec\ub2e4 1960\ub144 11\uc6d4\uc5d0 \u3008K \uc911\uac04\uc790\uc640 \ud575\uc790 \uc0b0\ub780 \ud604\uc0c1\uc758 \uc774\uc911 \ubd84\uc0b0 \uad00\uacc4\u3009(Study of K Scattering in the Double Dispersion Representation)\uc73c\ub85c \ubb3c\ub9ac\ud559 \ubc15\uc0ac(Doctor of Philosophy) \ud559\uc704\ub97c \ubc1b\uc558\ub294\ub370 \uc774 \ub54c \ub098\uc774\ub294 \ubd88\uacfc 25\uc138\uc600\ub2e4. \uadf8\uac00 \ubc15\uc0ac \ud559\uc704\uc99d\uc744 \uacf5\uc2dd\uc801\uc73c\ub85c \ubc1b\uc740 \uac83\uc740 1961\ub144 2\uc6d4 4\uc77c\uc774\uc5c8\ub2e4. \ubc15\uc0ac \ub17c\ubb38 \ub514\ud39c\uc2a4\uac00 \ub05d\ub09c 11\uc6d4\ubd80\ud130 1961\ub144 8\uc6d4\uae4c\uc9c0, \uc774\ud718\uc18c\ub294 \ud39c\uc2e4\ubca0\uc774\ub2c8\uc544 \ub300\ud559\uad50\uc758 \ubc15\uc0ac\ud6c4 \uc5f0\uad6c\uc6d0 \ubc0f \uc804\uc784 \uac15\uc0ac\ub85c \uc784\uc6a9\ub410\ub2e4."} {"question": "2010\ub144 1\uc6d4\uc5d0 \ubc1c\ub9e4\ub418\uc5c8\uace0 \uc77c\ubcf8 \uc624\ub9ac\ucf58 \uc704\ud074\ub9ac \ucc28\ud2b8 2\uc704\ub97c \uae30\ub85d\ud558\uc600\ub358 \uc568\ubc94\uc740 \ubb34\uc5c7\uc778\uac00?", "paragraph": "2010\ub144 1\uc6d4 20\uc77c\uc5d0\ub294 46\ubc88\uc9f8 \u300c\uc2f1\uae00 Can We Go Back\u300d\uc744 \ubc1c\ub9e4\ud558\uc600\ub2e4. \uccab\uc8fc 31,078\uc7a5\uc744 \ub9e4\uc0c1\ud574 \uc624\ub9ac\ucf58 weekly \ucc28\ud2b8 2\uc704\ub97c \ub7ad\ud06c\ud558\uc600\uc73c\uba70 2\uc6d4 03\uc77c \ub9b4\ub9ac\uc2a4 \ub41c 8 th\uc568\ubc94 \u300cBEST ~third universe ~ & 8th AL \"UNIVERSE\"\u300d\uac00 \uccab\uc8fc 22.1 \ub9cc\uc7a5\uc744 \ud310\ub9e4\ud558\uace0 \uc624\ub9ac\ucf58 weekly \ub7ad\ud0b9 1\uc8fc\uac04 1\uc704\ub97c \ud68d\ub4dd\ud588\ub2e4. \uadf8\ub9ac\uace0 7\uc6d4\uc5d0\ub294 \uc5ec\ub984 \uc2f1\uae00 \u300cGossip Candy\u300d\uac00 \ubc1c\ud45c\ub418\uc5c8\ub2e4.\uace7 \uc774\uc5b4 \uaca8\uc6b8 \ubc1c\ub77c\ub4dc \uc2f1\uae00 \u300c\u597d\u304d\u3067\u3001\u597d\u304d\u3067\u3001\u597d\u304d\u3067\u3002 / \u3042\u306a\u305f\u3060\u3051\u304c\u300d\uac00 \uccab\uc8fc 5.5\ub9cc\uc7a5\uc744 \ub9e4\uc0c1\ud588\uc9c0\ub9cc \uac19\uc740\ub0a0 \ub3d9\uc2dc \ubc1c\ub9e4\ub41c \ud558\ub9c8\uc0ac\ud0a4 \uc544\uc720\ubbf8\ub97c \ub4a4 \uc774\uc5b4 \uc624\ub9ac\ucf58 weekly \ub7ad\ud0b9 2\uc704\ub97c \ud68d\ub4dd\ud588\ub2e4 . MP3 \ub2e4\uc6b4\ub85c\ub4dc 100\ub9cc\uac74\uc744 \ub3cc\ud30c\ud558\uba70 \ud788\ud2b8\ub97c \uae30\ub85d\ud55c\ub2e4.", "answer": "Can We Go Back", "sentence": "2010\ub144 1\uc6d4 20\uc77c\uc5d0\ub294 46\ubc88\uc9f8 \u300c\uc2f1\uae00 Can We Go Back \u300d\uc744 \ubc1c\ub9e4\ud558\uc600\ub2e4.", "paragraph_sentence": " 2010\ub144 1\uc6d4 20\uc77c\uc5d0\ub294 46\ubc88\uc9f8 \u300c\uc2f1\uae00 Can We Go Back \u300d\uc744 \ubc1c\ub9e4\ud558\uc600\ub2e4. \uccab\uc8fc 31,078\uc7a5\uc744 \ub9e4\uc0c1\ud574 \uc624\ub9ac\ucf58 weekly \ucc28\ud2b8 2\uc704\ub97c \ub7ad\ud06c\ud558\uc600\uc73c\uba70 2\uc6d4 03\uc77c \ub9b4\ub9ac\uc2a4 \ub41c 8 th\uc568\ubc94 \u300cBEST ~third universe ~ & 8th AL \"UNIVERSE\"\u300d\uac00 \uccab\uc8fc 22.1 \ub9cc\uc7a5\uc744 \ud310\ub9e4\ud558\uace0 \uc624\ub9ac\ucf58 weekly \ub7ad\ud0b9 1\uc8fc\uac04 1\uc704\ub97c \ud68d\ub4dd\ud588\ub2e4. \uadf8\ub9ac\uace0 7\uc6d4\uc5d0\ub294 \uc5ec\ub984 \uc2f1\uae00 \u300cGossip Candy\u300d\uac00 \ubc1c\ud45c\ub418\uc5c8\ub2e4. \uace7 \uc774\uc5b4 \uaca8\uc6b8 \ubc1c\ub77c\ub4dc \uc2f1\uae00 \u300c\u597d\u304d\u3067\u3001\u597d\u304d\u3067\u3001\u597d\u304d\u3067\u3002 / \u3042\u306a\u305f\u3060\u3051\u304c\u300d\uac00 \uccab\uc8fc 5.5\ub9cc\uc7a5\uc744 \ub9e4\uc0c1\ud588\uc9c0\ub9cc \uac19\uc740\ub0a0 \ub3d9\uc2dc \ubc1c\ub9e4\ub41c \ud558\ub9c8\uc0ac\ud0a4 \uc544\uc720\ubbf8\ub97c \ub4a4 \uc774\uc5b4 \uc624\ub9ac\ucf58 weekly \ub7ad\ud0b9 2\uc704\ub97c \ud68d\ub4dd\ud588\ub2e4 . MP3 \ub2e4\uc6b4\ub85c\ub4dc 100\ub9cc\uac74\uc744 \ub3cc\ud30c\ud558\uba70 \ud788\ud2b8\ub97c \uae30\ub85d\ud55c\ub2e4.", "paragraph_answer": "2010\ub144 1\uc6d4 20\uc77c\uc5d0\ub294 46\ubc88\uc9f8 \u300c\uc2f1\uae00 Can We Go Back \u300d\uc744 \ubc1c\ub9e4\ud558\uc600\ub2e4. \uccab\uc8fc 31,078\uc7a5\uc744 \ub9e4\uc0c1\ud574 \uc624\ub9ac\ucf58 weekly \ucc28\ud2b8 2\uc704\ub97c \ub7ad\ud06c\ud558\uc600\uc73c\uba70 2\uc6d4 03\uc77c \ub9b4\ub9ac\uc2a4 \ub41c 8 th\uc568\ubc94 \u300cBEST ~third universe ~ & 8th AL \"UNIVERSE\"\u300d\uac00 \uccab\uc8fc 22.1 \ub9cc\uc7a5\uc744 \ud310\ub9e4\ud558\uace0 \uc624\ub9ac\ucf58 weekly \ub7ad\ud0b9 1\uc8fc\uac04 1\uc704\ub97c \ud68d\ub4dd\ud588\ub2e4. \uadf8\ub9ac\uace0 7\uc6d4\uc5d0\ub294 \uc5ec\ub984 \uc2f1\uae00 \u300cGossip Candy\u300d\uac00 \ubc1c\ud45c\ub418\uc5c8\ub2e4.\uace7 \uc774\uc5b4 \uaca8\uc6b8 \ubc1c\ub77c\ub4dc \uc2f1\uae00 \u300c\u597d\u304d\u3067\u3001\u597d\u304d\u3067\u3001\u597d\u304d\u3067\u3002 / \u3042\u306a\u305f\u3060\u3051\u304c\u300d\uac00 \uccab\uc8fc 5.5\ub9cc\uc7a5\uc744 \ub9e4\uc0c1\ud588\uc9c0\ub9cc \uac19\uc740\ub0a0 \ub3d9\uc2dc \ubc1c\ub9e4\ub41c \ud558\ub9c8\uc0ac\ud0a4 \uc544\uc720\ubbf8\ub97c \ub4a4 \uc774\uc5b4 \uc624\ub9ac\ucf58 weekly \ub7ad\ud0b9 2\uc704\ub97c \ud68d\ub4dd\ud588\ub2e4 . MP3 \ub2e4\uc6b4\ub85c\ub4dc 100\ub9cc\uac74\uc744 \ub3cc\ud30c\ud558\uba70 \ud788\ud2b8\ub97c \uae30\ub85d\ud55c\ub2e4.", "sentence_answer": "2010\ub144 1\uc6d4 20\uc77c\uc5d0\ub294 46\ubc88\uc9f8 \u300c\uc2f1\uae00 Can We Go Back \u300d\uc744 \ubc1c\ub9e4\ud558\uc600\ub2e4.", "paragraph_id": "6562271-3-0", "paragraph_question": "question: 2010\ub144 1\uc6d4\uc5d0 \ubc1c\ub9e4\ub418\uc5c8\uace0 \uc77c\ubcf8 \uc624\ub9ac\ucf58 \uc704\ud074\ub9ac \ucc28\ud2b8 2\uc704\ub97c \uae30\ub85d\ud558\uc600\ub358 \uc568\ubc94\uc740 \ubb34\uc5c7\uc778\uac00?, context: 2010\ub144 1\uc6d4 20\uc77c\uc5d0\ub294 46\ubc88\uc9f8 \u300c\uc2f1\uae00 Can We Go Back\u300d\uc744 \ubc1c\ub9e4\ud558\uc600\ub2e4. \uccab\uc8fc 31,078\uc7a5\uc744 \ub9e4\uc0c1\ud574 \uc624\ub9ac\ucf58 weekly \ucc28\ud2b8 2\uc704\ub97c \ub7ad\ud06c\ud558\uc600\uc73c\uba70 2\uc6d4 03\uc77c \ub9b4\ub9ac\uc2a4 \ub41c 8 th\uc568\ubc94 \u300cBEST ~third universe ~ & 8th AL \"UNIVERSE\"\u300d\uac00 \uccab\uc8fc 22.1 \ub9cc\uc7a5\uc744 \ud310\ub9e4\ud558\uace0 \uc624\ub9ac\ucf58 weekly \ub7ad\ud0b9 1\uc8fc\uac04 1\uc704\ub97c \ud68d\ub4dd\ud588\ub2e4. \uadf8\ub9ac\uace0 7\uc6d4\uc5d0\ub294 \uc5ec\ub984 \uc2f1\uae00 \u300cGossip Candy\u300d\uac00 \ubc1c\ud45c\ub418\uc5c8\ub2e4.\uace7 \uc774\uc5b4 \uaca8\uc6b8 \ubc1c\ub77c\ub4dc \uc2f1\uae00 \u300c\u597d\u304d\u3067\u3001\u597d\u304d\u3067\u3001\u597d\u304d\u3067\u3002 / \u3042\u306a\u305f\u3060\u3051\u304c\u300d\uac00 \uccab\uc8fc 5.5\ub9cc\uc7a5\uc744 \ub9e4\uc0c1\ud588\uc9c0\ub9cc \uac19\uc740\ub0a0 \ub3d9\uc2dc \ubc1c\ub9e4\ub41c \ud558\ub9c8\uc0ac\ud0a4 \uc544\uc720\ubbf8\ub97c \ub4a4 \uc774\uc5b4 \uc624\ub9ac\ucf58 weekly \ub7ad\ud0b9 2\uc704\ub97c \ud68d\ub4dd\ud588\ub2e4 . MP3 \ub2e4\uc6b4\ub85c\ub4dc 100\ub9cc\uac74\uc744 \ub3cc\ud30c\ud558\uba70 \ud788\ud2b8\ub97c \uae30\ub85d\ud55c\ub2e4."} {"question": "2015\ub144 \ucd5c\ucd08\ub85c \uc911\ub825\ud30c \uad00\uce21\uc5d0 \uc131\uacf5\ud588\ub2e4\uace0 \ubc1c\ud45c\ud55c \uc5f0\uad6c\uc9c4\uc740?", "paragraph": "LIGO \ud569\ub3d9\uc5f0\uad6c\uc9c4\uc740 2015\ub144 9\uc6d4 14\uc77c \uc9c0\uad6c\uc5d0\uc11c 13\uc5b5 \uad11\ub144 \ub5a8\uc5b4\uc9c4 \uc9c8\ub7c9 \uc57d 30 M\u2609\uc758 \ube14\ub799\ud640 \ub450 \uac1c\uac00 \uc11c\ub85c \ucda9\ub3cc\ud558\uc5ec \uc9c8\ub7c9 60 M\u2609\uc758 \ub2e8\uc77c \ube14\ub799\ud640\ub85c \uc735\ud569\ub420 \ub54c \ubc1c\uc0dd\ud55c \uc911\ub825\ud30c\ub97c \uac80\ucd9c\ud568\uc73c\ub85c\uc368 \ucd5c\ucd08\ub85c \uc911\ub825\ud30c \uad00\uce21\uc5d0 \uc131\uacf5\ud588\ub2e4\uace0 2016\ub144 2\uc6d4 11\uc77c \ubc1c\ud45c\ud588\ub2e4. \uc5f0\uad6c\uc6d0\ub4e4 \uc911 \ud55c \uba85 \uc774\uc0c1\uc774 \uc774 \uc0ac\uac74\uc744 \ube14\ub799\ud640\uc744 \uc9c1\uc811\uc801\uc73c\ub85c \uac10\uc9c0\ud55c \ucd5c\ucd08\uc758 \uc0ac\ub840\ub77c\uace0 \uaddc\uc815\ud588\ub2e4. LIGO\uc758 \uc911\ub825\ud30c \uac80\ucd9c \uc131\uacf5\uc740 \ucd5c\ucd08\uc758 \uc911\ub825\ud30c \uad00\uce21\uc77c \ubfd0 \uc544\ub2c8\ub77c \ube14\ub799\ud640 \uc30d\uc131\uacc4\uac00 \uc11c\ub85c \uc735\ud569\ud558\ub294 \uac83\uc774 \ucd5c\ucd08\ub85c \uad00\uce21\ub41c \uc0ac\ub840\uc774\uae30\ub3c4 \ud558\ub2e4. \uc735\ud569\ub418\uae30 \uc9c1\uc804 \ub450 \ucc9c\uccb4 \uc0ac\uc774\uc758 \uac70\ub9ac\ub294 \ubd88\uacfc 350 \ud0ac\ub85c\ubbf8\ud130\uc600\uc73c\uba70, \ub450 \ucc9c\uccb4\uac00 \ubaa8\ub450 \ube14\ub799\ud640\uc774\ub77c\ub294 \uac83 \uc678\uc5d0\ub294 \uc9c8\ub7c9\uc774 30 M\u2609\uc778 \ub450 \uac1c\uc758 \ucc9c\uccb4\uac00 \uc774\ub807\uac8c \uac00\uae4c\uc774 \uc811\uadfc\ud560 \uc218 \uc788\uc5c8\uc74c\uc5d0 \ub300\ud55c \ud0c0\ub2f9\ud55c \uc124\uba85\uc774 \uc5c6\ub2e4. \uac80\ucd9c\ub41c \uc911\ub825\ud30c\uc758 \ud2b9\uc9d5\uc740 \uc735\ud569\ub41c \uc774\ud6c4\uc5d0\ub294 \ube14\ub799\ud640\uc774 \uc548\uc815\ud654\ub418\uba74\uc11c \uc911\ub825\ud30c\uac00 \uae09\uc18d\ud788 \uac10\uc1e0\ud558\ub294 \ub4f1, \ub450 \ube14\ub799\ud640 \uc0ac\uc774\uc758 \uc735\ud569\uc73c\ub85c \uc778\ud574 \ubc1c\uc0dd\ud560 \uc911\ub825\ud30c\uc758 \uc774\ub860\uc801 \uc608\uc0c1\uacfc \uc815\ud655\ud788 \uc77c\uce58\ud588\ub2e4. \uc774 \uac80\ucd9c\uc740 \ube14\ub799\ud640\uc758 \uc874\uc7ac\uac00 \uc608\uc0c1\ub418\ub294 \uac15\ub825\ud55c \uc911\ub825\uc7a5 \ud658\uacbd\uc5d0\uc11c \uc77c\ubc18\uc0c1\ub300\ub860\uc774 \uac80\uc99d\ub41c \ud604\uc7ac\uae4c\uc9c0 \uac00\uc7a5 \uac15\ub825\ud55c \uc0ac\ub840\ub85c\uc11c, \uc77c\ubc18\uc0c1\ub300\ub860\uc758 \uc608\uce21\uacfc \ubc30\uce58\ub418\ub294 \uc99d\uac70\ub294 \uc804\ud600 \ubc1c\uacac\ub418\uc9c0 \uc54a\uc558\ub2e4. \ub610\ud55c \uc9c8\ub7c9 25 M\u2609 \uc774\uc0c1\uc758 \ud56d\uc131\uc9c8\ub7c9 \ube14\ub799\ud640\uc774 \uc790\uc5f0\uacc4\uc5d0\uc11c \ud615\uc131\ub420 \uc218 \uc788\ub2e4\ub294 \uccab \uc99d\uac70\uc774\uae30\ub3c4 \ud558\ub2e4.", "answer": "LIGO", "sentence": "LIGO \ud569\ub3d9\uc5f0\uad6c\uc9c4\uc740 2015\ub144 9\uc6d4 14\uc77c \uc9c0\uad6c\uc5d0\uc11c 13\uc5b5 \uad11\ub144 \ub5a8\uc5b4\uc9c4 \uc9c8\ub7c9 \uc57d 30 M\u2609\uc758 \ube14\ub799\ud640 \ub450 \uac1c\uac00 \uc11c\ub85c \ucda9\ub3cc\ud558\uc5ec \uc9c8\ub7c9 60 M\u2609\uc758 \ub2e8\uc77c \ube14\ub799\ud640\ub85c \uc735\ud569\ub420 \ub54c \ubc1c\uc0dd\ud55c \uc911\ub825\ud30c\ub97c \uac80\ucd9c\ud568\uc73c\ub85c\uc368 \ucd5c\ucd08\ub85c \uc911\ub825\ud30c \uad00\uce21\uc5d0 \uc131\uacf5\ud588\ub2e4\uace0 2016\ub144 2\uc6d4 11\uc77c \ubc1c\ud45c\ud588\ub2e4.", "paragraph_sentence": " LIGO \ud569\ub3d9\uc5f0\uad6c\uc9c4\uc740 2015\ub144 9\uc6d4 14\uc77c \uc9c0\uad6c\uc5d0\uc11c 13\uc5b5 \uad11\ub144 \ub5a8\uc5b4\uc9c4 \uc9c8\ub7c9 \uc57d 30 M\u2609\uc758 \ube14\ub799\ud640 \ub450 \uac1c\uac00 \uc11c\ub85c \ucda9\ub3cc\ud558\uc5ec \uc9c8\ub7c9 60 M\u2609\uc758 \ub2e8\uc77c \ube14\ub799\ud640\ub85c \uc735\ud569\ub420 \ub54c \ubc1c\uc0dd\ud55c \uc911\ub825\ud30c\ub97c \uac80\ucd9c\ud568\uc73c\ub85c\uc368 \ucd5c\ucd08\ub85c \uc911\ub825\ud30c \uad00\uce21\uc5d0 \uc131\uacf5\ud588\ub2e4\uace0 2016\ub144 2\uc6d4 11\uc77c \ubc1c\ud45c\ud588\ub2e4. \uc5f0\uad6c\uc6d0\ub4e4 \uc911 \ud55c \uba85 \uc774\uc0c1\uc774 \uc774 \uc0ac\uac74\uc744 \ube14\ub799\ud640\uc744 \uc9c1\uc811\uc801\uc73c\ub85c \uac10\uc9c0\ud55c \ucd5c\ucd08\uc758 \uc0ac\ub840\ub77c\uace0 \uaddc\uc815\ud588\ub2e4. LIGO\uc758 \uc911\ub825\ud30c \uac80\ucd9c \uc131\uacf5\uc740 \ucd5c\ucd08\uc758 \uc911\ub825\ud30c \uad00\uce21\uc77c \ubfd0 \uc544\ub2c8\ub77c \ube14\ub799\ud640 \uc30d\uc131\uacc4\uac00 \uc11c\ub85c \uc735\ud569\ud558\ub294 \uac83\uc774 \ucd5c\ucd08\ub85c \uad00\uce21\ub41c \uc0ac\ub840\uc774\uae30\ub3c4 \ud558\ub2e4. \uc735\ud569\ub418\uae30 \uc9c1\uc804 \ub450 \ucc9c\uccb4 \uc0ac\uc774\uc758 \uac70\ub9ac\ub294 \ubd88\uacfc 350 \ud0ac\ub85c\ubbf8\ud130\uc600\uc73c\uba70, \ub450 \ucc9c\uccb4\uac00 \ubaa8\ub450 \ube14\ub799\ud640\uc774\ub77c\ub294 \uac83 \uc678\uc5d0\ub294 \uc9c8\ub7c9\uc774 30 M\u2609\uc778 \ub450 \uac1c\uc758 \ucc9c\uccb4\uac00 \uc774\ub807\uac8c \uac00\uae4c\uc774 \uc811\uadfc\ud560 \uc218 \uc788\uc5c8\uc74c\uc5d0 \ub300\ud55c \ud0c0\ub2f9\ud55c \uc124\uba85\uc774 \uc5c6\ub2e4. \uac80\ucd9c\ub41c \uc911\ub825\ud30c\uc758 \ud2b9\uc9d5\uc740 \uc735\ud569\ub41c \uc774\ud6c4\uc5d0\ub294 \ube14\ub799\ud640\uc774 \uc548\uc815\ud654\ub418\uba74\uc11c \uc911\ub825\ud30c\uac00 \uae09\uc18d\ud788 \uac10\uc1e0\ud558\ub294 \ub4f1, \ub450 \ube14\ub799\ud640 \uc0ac\uc774\uc758 \uc735\ud569\uc73c\ub85c \uc778\ud574 \ubc1c\uc0dd\ud560 \uc911\ub825\ud30c\uc758 \uc774\ub860\uc801 \uc608\uc0c1\uacfc \uc815\ud655\ud788 \uc77c\uce58\ud588\ub2e4. \uc774 \uac80\ucd9c\uc740 \ube14\ub799\ud640\uc758 \uc874\uc7ac\uac00 \uc608\uc0c1\ub418\ub294 \uac15\ub825\ud55c \uc911\ub825\uc7a5 \ud658\uacbd\uc5d0\uc11c \uc77c\ubc18\uc0c1\ub300\ub860\uc774 \uac80\uc99d\ub41c \ud604\uc7ac\uae4c\uc9c0 \uac00\uc7a5 \uac15\ub825\ud55c \uc0ac\ub840\ub85c\uc11c, \uc77c\ubc18\uc0c1\ub300\ub860\uc758 \uc608\uce21\uacfc \ubc30\uce58\ub418\ub294 \uc99d\uac70\ub294 \uc804\ud600 \ubc1c\uacac\ub418\uc9c0 \uc54a\uc558\ub2e4. \ub610\ud55c \uc9c8\ub7c9 25 M\u2609 \uc774\uc0c1\uc758 \ud56d\uc131\uc9c8\ub7c9 \ube14\ub799\ud640\uc774 \uc790\uc5f0\uacc4\uc5d0\uc11c \ud615\uc131\ub420 \uc218 \uc788\ub2e4\ub294 \uccab \uc99d\uac70\uc774\uae30\ub3c4 \ud558\ub2e4.", "paragraph_answer": " LIGO \ud569\ub3d9\uc5f0\uad6c\uc9c4\uc740 2015\ub144 9\uc6d4 14\uc77c \uc9c0\uad6c\uc5d0\uc11c 13\uc5b5 \uad11\ub144 \ub5a8\uc5b4\uc9c4 \uc9c8\ub7c9 \uc57d 30 M\u2609\uc758 \ube14\ub799\ud640 \ub450 \uac1c\uac00 \uc11c\ub85c \ucda9\ub3cc\ud558\uc5ec \uc9c8\ub7c9 60 M\u2609\uc758 \ub2e8\uc77c \ube14\ub799\ud640\ub85c \uc735\ud569\ub420 \ub54c \ubc1c\uc0dd\ud55c \uc911\ub825\ud30c\ub97c \uac80\ucd9c\ud568\uc73c\ub85c\uc368 \ucd5c\ucd08\ub85c \uc911\ub825\ud30c \uad00\uce21\uc5d0 \uc131\uacf5\ud588\ub2e4\uace0 2016\ub144 2\uc6d4 11\uc77c \ubc1c\ud45c\ud588\ub2e4. \uc5f0\uad6c\uc6d0\ub4e4 \uc911 \ud55c \uba85 \uc774\uc0c1\uc774 \uc774 \uc0ac\uac74\uc744 \ube14\ub799\ud640\uc744 \uc9c1\uc811\uc801\uc73c\ub85c \uac10\uc9c0\ud55c \ucd5c\ucd08\uc758 \uc0ac\ub840\ub77c\uace0 \uaddc\uc815\ud588\ub2e4. LIGO\uc758 \uc911\ub825\ud30c \uac80\ucd9c \uc131\uacf5\uc740 \ucd5c\ucd08\uc758 \uc911\ub825\ud30c \uad00\uce21\uc77c \ubfd0 \uc544\ub2c8\ub77c \ube14\ub799\ud640 \uc30d\uc131\uacc4\uac00 \uc11c\ub85c \uc735\ud569\ud558\ub294 \uac83\uc774 \ucd5c\ucd08\ub85c \uad00\uce21\ub41c \uc0ac\ub840\uc774\uae30\ub3c4 \ud558\ub2e4. \uc735\ud569\ub418\uae30 \uc9c1\uc804 \ub450 \ucc9c\uccb4 \uc0ac\uc774\uc758 \uac70\ub9ac\ub294 \ubd88\uacfc 350 \ud0ac\ub85c\ubbf8\ud130\uc600\uc73c\uba70, \ub450 \ucc9c\uccb4\uac00 \ubaa8\ub450 \ube14\ub799\ud640\uc774\ub77c\ub294 \uac83 \uc678\uc5d0\ub294 \uc9c8\ub7c9\uc774 30 M\u2609\uc778 \ub450 \uac1c\uc758 \ucc9c\uccb4\uac00 \uc774\ub807\uac8c \uac00\uae4c\uc774 \uc811\uadfc\ud560 \uc218 \uc788\uc5c8\uc74c\uc5d0 \ub300\ud55c \ud0c0\ub2f9\ud55c \uc124\uba85\uc774 \uc5c6\ub2e4. \uac80\ucd9c\ub41c \uc911\ub825\ud30c\uc758 \ud2b9\uc9d5\uc740 \uc735\ud569\ub41c \uc774\ud6c4\uc5d0\ub294 \ube14\ub799\ud640\uc774 \uc548\uc815\ud654\ub418\uba74\uc11c \uc911\ub825\ud30c\uac00 \uae09\uc18d\ud788 \uac10\uc1e0\ud558\ub294 \ub4f1, \ub450 \ube14\ub799\ud640 \uc0ac\uc774\uc758 \uc735\ud569\uc73c\ub85c \uc778\ud574 \ubc1c\uc0dd\ud560 \uc911\ub825\ud30c\uc758 \uc774\ub860\uc801 \uc608\uc0c1\uacfc \uc815\ud655\ud788 \uc77c\uce58\ud588\ub2e4. \uc774 \uac80\ucd9c\uc740 \ube14\ub799\ud640\uc758 \uc874\uc7ac\uac00 \uc608\uc0c1\ub418\ub294 \uac15\ub825\ud55c \uc911\ub825\uc7a5 \ud658\uacbd\uc5d0\uc11c \uc77c\ubc18\uc0c1\ub300\ub860\uc774 \uac80\uc99d\ub41c \ud604\uc7ac\uae4c\uc9c0 \uac00\uc7a5 \uac15\ub825\ud55c \uc0ac\ub840\ub85c\uc11c, \uc77c\ubc18\uc0c1\ub300\ub860\uc758 \uc608\uce21\uacfc \ubc30\uce58\ub418\ub294 \uc99d\uac70\ub294 \uc804\ud600 \ubc1c\uacac\ub418\uc9c0 \uc54a\uc558\ub2e4. \ub610\ud55c \uc9c8\ub7c9 25 M\u2609 \uc774\uc0c1\uc758 \ud56d\uc131\uc9c8\ub7c9 \ube14\ub799\ud640\uc774 \uc790\uc5f0\uacc4\uc5d0\uc11c \ud615\uc131\ub420 \uc218 \uc788\ub2e4\ub294 \uccab \uc99d\uac70\uc774\uae30\ub3c4 \ud558\ub2e4.", "sentence_answer": " LIGO \ud569\ub3d9\uc5f0\uad6c\uc9c4\uc740 2015\ub144 9\uc6d4 14\uc77c \uc9c0\uad6c\uc5d0\uc11c 13\uc5b5 \uad11\ub144 \ub5a8\uc5b4\uc9c4 \uc9c8\ub7c9 \uc57d 30 M\u2609\uc758 \ube14\ub799\ud640 \ub450 \uac1c\uac00 \uc11c\ub85c \ucda9\ub3cc\ud558\uc5ec \uc9c8\ub7c9 60 M\u2609\uc758 \ub2e8\uc77c \ube14\ub799\ud640\ub85c \uc735\ud569\ub420 \ub54c \ubc1c\uc0dd\ud55c \uc911\ub825\ud30c\ub97c \uac80\ucd9c\ud568\uc73c\ub85c\uc368 \ucd5c\ucd08\ub85c \uc911\ub825\ud30c \uad00\uce21\uc5d0 \uc131\uacf5\ud588\ub2e4\uace0 2016\ub144 2\uc6d4 11\uc77c \ubc1c\ud45c\ud588\ub2e4.", "paragraph_id": "6530722-12-0", "paragraph_question": "question: 2015\ub144 \ucd5c\ucd08\ub85c \uc911\ub825\ud30c \uad00\uce21\uc5d0 \uc131\uacf5\ud588\ub2e4\uace0 \ubc1c\ud45c\ud55c \uc5f0\uad6c\uc9c4\uc740?, context: LIGO \ud569\ub3d9\uc5f0\uad6c\uc9c4\uc740 2015\ub144 9\uc6d4 14\uc77c \uc9c0\uad6c\uc5d0\uc11c 13\uc5b5 \uad11\ub144 \ub5a8\uc5b4\uc9c4 \uc9c8\ub7c9 \uc57d 30 M\u2609\uc758 \ube14\ub799\ud640 \ub450 \uac1c\uac00 \uc11c\ub85c \ucda9\ub3cc\ud558\uc5ec \uc9c8\ub7c9 60 M\u2609\uc758 \ub2e8\uc77c \ube14\ub799\ud640\ub85c \uc735\ud569\ub420 \ub54c \ubc1c\uc0dd\ud55c \uc911\ub825\ud30c\ub97c \uac80\ucd9c\ud568\uc73c\ub85c\uc368 \ucd5c\ucd08\ub85c \uc911\ub825\ud30c \uad00\uce21\uc5d0 \uc131\uacf5\ud588\ub2e4\uace0 2016\ub144 2\uc6d4 11\uc77c \ubc1c\ud45c\ud588\ub2e4. \uc5f0\uad6c\uc6d0\ub4e4 \uc911 \ud55c \uba85 \uc774\uc0c1\uc774 \uc774 \uc0ac\uac74\uc744 \ube14\ub799\ud640\uc744 \uc9c1\uc811\uc801\uc73c\ub85c \uac10\uc9c0\ud55c \ucd5c\ucd08\uc758 \uc0ac\ub840\ub77c\uace0 \uaddc\uc815\ud588\ub2e4. LIGO\uc758 \uc911\ub825\ud30c \uac80\ucd9c \uc131\uacf5\uc740 \ucd5c\ucd08\uc758 \uc911\ub825\ud30c \uad00\uce21\uc77c \ubfd0 \uc544\ub2c8\ub77c \ube14\ub799\ud640 \uc30d\uc131\uacc4\uac00 \uc11c\ub85c \uc735\ud569\ud558\ub294 \uac83\uc774 \ucd5c\ucd08\ub85c \uad00\uce21\ub41c \uc0ac\ub840\uc774\uae30\ub3c4 \ud558\ub2e4. \uc735\ud569\ub418\uae30 \uc9c1\uc804 \ub450 \ucc9c\uccb4 \uc0ac\uc774\uc758 \uac70\ub9ac\ub294 \ubd88\uacfc 350 \ud0ac\ub85c\ubbf8\ud130\uc600\uc73c\uba70, \ub450 \ucc9c\uccb4\uac00 \ubaa8\ub450 \ube14\ub799\ud640\uc774\ub77c\ub294 \uac83 \uc678\uc5d0\ub294 \uc9c8\ub7c9\uc774 30 M\u2609\uc778 \ub450 \uac1c\uc758 \ucc9c\uccb4\uac00 \uc774\ub807\uac8c \uac00\uae4c\uc774 \uc811\uadfc\ud560 \uc218 \uc788\uc5c8\uc74c\uc5d0 \ub300\ud55c \ud0c0\ub2f9\ud55c \uc124\uba85\uc774 \uc5c6\ub2e4. \uac80\ucd9c\ub41c \uc911\ub825\ud30c\uc758 \ud2b9\uc9d5\uc740 \uc735\ud569\ub41c \uc774\ud6c4\uc5d0\ub294 \ube14\ub799\ud640\uc774 \uc548\uc815\ud654\ub418\uba74\uc11c \uc911\ub825\ud30c\uac00 \uae09\uc18d\ud788 \uac10\uc1e0\ud558\ub294 \ub4f1, \ub450 \ube14\ub799\ud640 \uc0ac\uc774\uc758 \uc735\ud569\uc73c\ub85c \uc778\ud574 \ubc1c\uc0dd\ud560 \uc911\ub825\ud30c\uc758 \uc774\ub860\uc801 \uc608\uc0c1\uacfc \uc815\ud655\ud788 \uc77c\uce58\ud588\ub2e4. \uc774 \uac80\ucd9c\uc740 \ube14\ub799\ud640\uc758 \uc874\uc7ac\uac00 \uc608\uc0c1\ub418\ub294 \uac15\ub825\ud55c \uc911\ub825\uc7a5 \ud658\uacbd\uc5d0\uc11c \uc77c\ubc18\uc0c1\ub300\ub860\uc774 \uac80\uc99d\ub41c \ud604\uc7ac\uae4c\uc9c0 \uac00\uc7a5 \uac15\ub825\ud55c \uc0ac\ub840\ub85c\uc11c, \uc77c\ubc18\uc0c1\ub300\ub860\uc758 \uc608\uce21\uacfc \ubc30\uce58\ub418\ub294 \uc99d\uac70\ub294 \uc804\ud600 \ubc1c\uacac\ub418\uc9c0 \uc54a\uc558\ub2e4. \ub610\ud55c \uc9c8\ub7c9 25 M\u2609 \uc774\uc0c1\uc758 \ud56d\uc131\uc9c8\ub7c9 \ube14\ub799\ud640\uc774 \uc790\uc5f0\uacc4\uc5d0\uc11c \ud615\uc131\ub420 \uc218 \uc788\ub2e4\ub294 \uccab \uc99d\uac70\uc774\uae30\ub3c4 \ud558\ub2e4."} {"question": "\uccad\ub098\ub77c\uac00 \uc81c\uad6d\uc8fc\uc758 \uc5f4\uac15\uc5d0 \uac70\uc561\uc758 \ubc30\uc0c1\uae08\uc744 \uc9c0\uae09\ud558\uace0 \uc911\uad6d \ub0b4 \uad70\ub300 \uc8fc\ub454\uad8c\uc744 \uc778\uc815\ud55c\ub2e4\ub294 \ub0b4\uc6a9\uc744 \ub2f4\uc740 \uac15\ud654 \uc870\uc57d\uc740?", "paragraph": "1900\ub144 1\uc6d4 \uc11c\ud0dc\ud6c4\uac00 \ud669\uc81c\uc778 \uad11\uc11c\uc81c\ub97c \ud3d0\uc704\uc2dc\ud0a4\ub824\uace0 \ud588\uc73c\ub098 \uc5f4\uac15\uc774 \uc11c\ud0dc\ud6c4\uc758 \uc758\ub3c4\ub97c \uac04\ud30c\ud558\uace0 \uacf5\ub3d9\uc73c\ub85c \uc555\ub825\uc744 \uac00\ud574 \uadf8 \uc758\ub3c4\ub97c \uc88c\uc808\uc2dc\ucf30\ub2e4. \uc774 \ub54c\ubb38\uc5d0 \uccad\ub098\ub77c \uc815\ubd80\uc758 \uc218\uad6c\ud30c\ub294 \uc758\ud654\ub2e8\uc758 \ubc30\uc678 \uc6b4\ub3d9\uc744 \uace0\ubb34\ud574\uc11c \uc5f4\uac15\uc5d0 \uc555\ub825\uc744 \uac00\ud558\uace0\uc790 \ud588\ub2e4. 1900\ub144 6\uc6d4\uc5d0 \uc758\ud654\ub2e8\uc774 \ubca0\uc774\uc9d5\uc5d0 \uc788\ub294 \uc678\uad6d \uacf5\uad00\uc744 \ud3ec\uc704 \uacf5\uaca9\ud558\uc790 \uc11c\ud0dc\ud6c4\ub294 \uadf8\ub4e4\uc744 \uc758\ubbfc(\u7fa9\u6c11)\uc73c\ub85c \uaddc\uc815\ud558\uace0 \uc5f4\uac15\uc5d0 \uc120\uc804 \ud3ec\uace0\ud588\ub2e4. \uc774\uc5d0 \ub7ec\uc2dc\uc544, \uc77c\ubcf8, \ub3c5\uc77c, \uc601\uad6d, \ubbf8\uad6d, \uc774\ud0c8\ub9ac\uc544, \uc624\uc2a4\ud2b8\ub9ac\uc544, \ud504\ub791\uc2a4 8\uac1c\uad6d\uc774 \ud30c\ubcd1\ud574\uc11c \ubca0\uc774\uc9d5\uc744 \ube44\ub86f\ud574 \uc591\ucbd4 \uac15 \uc774\ubd81 \uc9c0\uc5ed\uc744 \ub300\ubd80\ubd84 \uc810\ub839\ud588\ub2e4. \uc5f4\uac15\uc740 \uc911\uad6d\uc744 \ubd84\ud560\ud558\uc9c0 \uc54a\ub294 \ub300\uc2e0 \ubcf4\uc874\ud558\uae30\ub85c \uacb0\uc815\ud558\ub294 \ud55c\ud3b8 \uccad\ub098\ub77c \uc655\uc870\uc640\uc758 \ud611\uc0c1\uc744 \uac70\uccd0 1901\ub144 9\uc6d4 7\uc77c\uc5d0 \uac15\ud654 \uc870\uc57d\uc778 \uc2e0\ucd95 \uc870\uc57d(\ubca0\uc774\uc9d5 \uc758\uc815\uc11c)\ub97c \uccb4\uacb0\ud588\ub2e4. \uadf8 \ub0b4\uc6a9\uc740 \uccad\ub098\ub77c\uac00 \uc81c\uad6d\uc8fc\uc758 \uc5f4\uac15\uc5d0 \uac70\uc561\uc758 \ubc30\uc0c1\uae08\uc744 \uc9c0\uae09\ud558\ub294 \ub3d9\uc2dc\uc5d0 \uc5f4\uac15\uc758 \uc911\uad6d \ub0b4 \uad70\ub300 \uc8fc\ub454\uad8c\uc744 \uc778\uc815\ud558\ub294 \uac83\uc774\uc5c8\ub2e4. \uc774 \uc0ac\uac74\uc73c\ub85c \uc778\ud574 \uc911\uad6d\uc758 \ubc18\uc2dd\ubbfc\uc9c0 \uc0c1\ud0dc\uac00 \ub354\uc6b1 \uc2ec\ud654\ub418\uc5c8\ub2e4.", "answer": "\ubca0\uc774\uc9d5 \uc758\uc815\uc11c", "sentence": "\uc5f4\uac15\uc740 \uc911\uad6d\uc744 \ubd84\ud560\ud558\uc9c0 \uc54a\ub294 \ub300\uc2e0 \ubcf4\uc874\ud558\uae30\ub85c \uacb0\uc815\ud558\ub294 \ud55c\ud3b8 \uccad\ub098\ub77c \uc655\uc870\uc640\uc758 \ud611\uc0c1\uc744 \uac70\uccd0 1901\ub144 9\uc6d4 7\uc77c\uc5d0 \uac15\ud654 \uc870\uc57d\uc778 \uc2e0\ucd95 \uc870\uc57d( \ubca0\uc774\uc9d5 \uc758\uc815\uc11c )", "paragraph_sentence": "1900\ub144 1\uc6d4 \uc11c\ud0dc\ud6c4\uac00 \ud669\uc81c\uc778 \uad11\uc11c\uc81c\ub97c \ud3d0\uc704\uc2dc\ud0a4\ub824\uace0 \ud588\uc73c\ub098 \uc5f4\uac15\uc774 \uc11c\ud0dc\ud6c4\uc758 \uc758\ub3c4\ub97c \uac04\ud30c\ud558\uace0 \uacf5\ub3d9\uc73c\ub85c \uc555\ub825\uc744 \uac00\ud574 \uadf8 \uc758\ub3c4\ub97c \uc88c\uc808\uc2dc\ucf30\ub2e4. \uc774 \ub54c\ubb38\uc5d0 \uccad\ub098\ub77c \uc815\ubd80\uc758 \uc218\uad6c\ud30c\ub294 \uc758\ud654\ub2e8\uc758 \ubc30\uc678 \uc6b4\ub3d9\uc744 \uace0\ubb34\ud574\uc11c \uc5f4\uac15\uc5d0 \uc555\ub825\uc744 \uac00\ud558\uace0\uc790 \ud588\ub2e4. 1900\ub144 6\uc6d4\uc5d0 \uc758\ud654\ub2e8\uc774 \ubca0\uc774\uc9d5\uc5d0 \uc788\ub294 \uc678\uad6d \uacf5\uad00\uc744 \ud3ec\uc704 \uacf5\uaca9\ud558\uc790 \uc11c\ud0dc\ud6c4\ub294 \uadf8\ub4e4\uc744 \uc758\ubbfc(\u7fa9\u6c11)\uc73c\ub85c \uaddc\uc815\ud558\uace0 \uc5f4\uac15\uc5d0 \uc120\uc804 \ud3ec\uace0\ud588\ub2e4. \uc774\uc5d0 \ub7ec\uc2dc\uc544, \uc77c\ubcf8, \ub3c5\uc77c, \uc601\uad6d, \ubbf8\uad6d, \uc774\ud0c8\ub9ac\uc544, \uc624\uc2a4\ud2b8\ub9ac\uc544, \ud504\ub791\uc2a4 8\uac1c\uad6d\uc774 \ud30c\ubcd1\ud574\uc11c \ubca0\uc774\uc9d5\uc744 \ube44\ub86f\ud574 \uc591\ucbd4 \uac15 \uc774\ubd81 \uc9c0\uc5ed\uc744 \ub300\ubd80\ubd84 \uc810\ub839\ud588\ub2e4. \uc5f4\uac15\uc740 \uc911\uad6d\uc744 \ubd84\ud560\ud558\uc9c0 \uc54a\ub294 \ub300\uc2e0 \ubcf4\uc874\ud558\uae30\ub85c \uacb0\uc815\ud558\ub294 \ud55c\ud3b8 \uccad\ub098\ub77c \uc655\uc870\uc640\uc758 \ud611\uc0c1\uc744 \uac70\uccd0 1901\ub144 9\uc6d4 7\uc77c\uc5d0 \uac15\ud654 \uc870\uc57d\uc778 \uc2e0\ucd95 \uc870\uc57d( \ubca0\uc774\uc9d5 \uc758\uc815\uc11c ) \ub97c \uccb4\uacb0\ud588\ub2e4. \uadf8 \ub0b4\uc6a9\uc740 \uccad\ub098\ub77c\uac00 \uc81c\uad6d\uc8fc\uc758 \uc5f4\uac15\uc5d0 \uac70\uc561\uc758 \ubc30\uc0c1\uae08\uc744 \uc9c0\uae09\ud558\ub294 \ub3d9\uc2dc\uc5d0 \uc5f4\uac15\uc758 \uc911\uad6d \ub0b4 \uad70\ub300 \uc8fc\ub454\uad8c\uc744 \uc778\uc815\ud558\ub294 \uac83\uc774\uc5c8\ub2e4. \uc774 \uc0ac\uac74\uc73c\ub85c \uc778\ud574 \uc911\uad6d\uc758 \ubc18\uc2dd\ubbfc\uc9c0 \uc0c1\ud0dc\uac00 \ub354\uc6b1 \uc2ec\ud654\ub418\uc5c8\ub2e4.", "paragraph_answer": "1900\ub144 1\uc6d4 \uc11c\ud0dc\ud6c4\uac00 \ud669\uc81c\uc778 \uad11\uc11c\uc81c\ub97c \ud3d0\uc704\uc2dc\ud0a4\ub824\uace0 \ud588\uc73c\ub098 \uc5f4\uac15\uc774 \uc11c\ud0dc\ud6c4\uc758 \uc758\ub3c4\ub97c \uac04\ud30c\ud558\uace0 \uacf5\ub3d9\uc73c\ub85c \uc555\ub825\uc744 \uac00\ud574 \uadf8 \uc758\ub3c4\ub97c \uc88c\uc808\uc2dc\ucf30\ub2e4. \uc774 \ub54c\ubb38\uc5d0 \uccad\ub098\ub77c \uc815\ubd80\uc758 \uc218\uad6c\ud30c\ub294 \uc758\ud654\ub2e8\uc758 \ubc30\uc678 \uc6b4\ub3d9\uc744 \uace0\ubb34\ud574\uc11c \uc5f4\uac15\uc5d0 \uc555\ub825\uc744 \uac00\ud558\uace0\uc790 \ud588\ub2e4. 1900\ub144 6\uc6d4\uc5d0 \uc758\ud654\ub2e8\uc774 \ubca0\uc774\uc9d5\uc5d0 \uc788\ub294 \uc678\uad6d \uacf5\uad00\uc744 \ud3ec\uc704 \uacf5\uaca9\ud558\uc790 \uc11c\ud0dc\ud6c4\ub294 \uadf8\ub4e4\uc744 \uc758\ubbfc(\u7fa9\u6c11)\uc73c\ub85c \uaddc\uc815\ud558\uace0 \uc5f4\uac15\uc5d0 \uc120\uc804 \ud3ec\uace0\ud588\ub2e4. \uc774\uc5d0 \ub7ec\uc2dc\uc544, \uc77c\ubcf8, \ub3c5\uc77c, \uc601\uad6d, \ubbf8\uad6d, \uc774\ud0c8\ub9ac\uc544, \uc624\uc2a4\ud2b8\ub9ac\uc544, \ud504\ub791\uc2a4 8\uac1c\uad6d\uc774 \ud30c\ubcd1\ud574\uc11c \ubca0\uc774\uc9d5\uc744 \ube44\ub86f\ud574 \uc591\ucbd4 \uac15 \uc774\ubd81 \uc9c0\uc5ed\uc744 \ub300\ubd80\ubd84 \uc810\ub839\ud588\ub2e4. \uc5f4\uac15\uc740 \uc911\uad6d\uc744 \ubd84\ud560\ud558\uc9c0 \uc54a\ub294 \ub300\uc2e0 \ubcf4\uc874\ud558\uae30\ub85c \uacb0\uc815\ud558\ub294 \ud55c\ud3b8 \uccad\ub098\ub77c \uc655\uc870\uc640\uc758 \ud611\uc0c1\uc744 \uac70\uccd0 1901\ub144 9\uc6d4 7\uc77c\uc5d0 \uac15\ud654 \uc870\uc57d\uc778 \uc2e0\ucd95 \uc870\uc57d( \ubca0\uc774\uc9d5 \uc758\uc815\uc11c )\ub97c \uccb4\uacb0\ud588\ub2e4. \uadf8 \ub0b4\uc6a9\uc740 \uccad\ub098\ub77c\uac00 \uc81c\uad6d\uc8fc\uc758 \uc5f4\uac15\uc5d0 \uac70\uc561\uc758 \ubc30\uc0c1\uae08\uc744 \uc9c0\uae09\ud558\ub294 \ub3d9\uc2dc\uc5d0 \uc5f4\uac15\uc758 \uc911\uad6d \ub0b4 \uad70\ub300 \uc8fc\ub454\uad8c\uc744 \uc778\uc815\ud558\ub294 \uac83\uc774\uc5c8\ub2e4. \uc774 \uc0ac\uac74\uc73c\ub85c \uc778\ud574 \uc911\uad6d\uc758 \ubc18\uc2dd\ubbfc\uc9c0 \uc0c1\ud0dc\uac00 \ub354\uc6b1 \uc2ec\ud654\ub418\uc5c8\ub2e4.", "sentence_answer": "\uc5f4\uac15\uc740 \uc911\uad6d\uc744 \ubd84\ud560\ud558\uc9c0 \uc54a\ub294 \ub300\uc2e0 \ubcf4\uc874\ud558\uae30\ub85c \uacb0\uc815\ud558\ub294 \ud55c\ud3b8 \uccad\ub098\ub77c \uc655\uc870\uc640\uc758 \ud611\uc0c1\uc744 \uac70\uccd0 1901\ub144 9\uc6d4 7\uc77c\uc5d0 \uac15\ud654 \uc870\uc57d\uc778 \uc2e0\ucd95 \uc870\uc57d( \ubca0\uc774\uc9d5 \uc758\uc815\uc11c )", "paragraph_id": "6544410-1-1", "paragraph_question": "question: \uccad\ub098\ub77c\uac00 \uc81c\uad6d\uc8fc\uc758 \uc5f4\uac15\uc5d0 \uac70\uc561\uc758 \ubc30\uc0c1\uae08\uc744 \uc9c0\uae09\ud558\uace0 \uc911\uad6d \ub0b4 \uad70\ub300 \uc8fc\ub454\uad8c\uc744 \uc778\uc815\ud55c\ub2e4\ub294 \ub0b4\uc6a9\uc744 \ub2f4\uc740 \uac15\ud654 \uc870\uc57d\uc740?, context: 1900\ub144 1\uc6d4 \uc11c\ud0dc\ud6c4\uac00 \ud669\uc81c\uc778 \uad11\uc11c\uc81c\ub97c \ud3d0\uc704\uc2dc\ud0a4\ub824\uace0 \ud588\uc73c\ub098 \uc5f4\uac15\uc774 \uc11c\ud0dc\ud6c4\uc758 \uc758\ub3c4\ub97c \uac04\ud30c\ud558\uace0 \uacf5\ub3d9\uc73c\ub85c \uc555\ub825\uc744 \uac00\ud574 \uadf8 \uc758\ub3c4\ub97c \uc88c\uc808\uc2dc\ucf30\ub2e4. \uc774 \ub54c\ubb38\uc5d0 \uccad\ub098\ub77c \uc815\ubd80\uc758 \uc218\uad6c\ud30c\ub294 \uc758\ud654\ub2e8\uc758 \ubc30\uc678 \uc6b4\ub3d9\uc744 \uace0\ubb34\ud574\uc11c \uc5f4\uac15\uc5d0 \uc555\ub825\uc744 \uac00\ud558\uace0\uc790 \ud588\ub2e4. 1900\ub144 6\uc6d4\uc5d0 \uc758\ud654\ub2e8\uc774 \ubca0\uc774\uc9d5\uc5d0 \uc788\ub294 \uc678\uad6d \uacf5\uad00\uc744 \ud3ec\uc704 \uacf5\uaca9\ud558\uc790 \uc11c\ud0dc\ud6c4\ub294 \uadf8\ub4e4\uc744 \uc758\ubbfc(\u7fa9\u6c11)\uc73c\ub85c \uaddc\uc815\ud558\uace0 \uc5f4\uac15\uc5d0 \uc120\uc804 \ud3ec\uace0\ud588\ub2e4. \uc774\uc5d0 \ub7ec\uc2dc\uc544, \uc77c\ubcf8, \ub3c5\uc77c, \uc601\uad6d, \ubbf8\uad6d, \uc774\ud0c8\ub9ac\uc544, \uc624\uc2a4\ud2b8\ub9ac\uc544, \ud504\ub791\uc2a4 8\uac1c\uad6d\uc774 \ud30c\ubcd1\ud574\uc11c \ubca0\uc774\uc9d5\uc744 \ube44\ub86f\ud574 \uc591\ucbd4 \uac15 \uc774\ubd81 \uc9c0\uc5ed\uc744 \ub300\ubd80\ubd84 \uc810\ub839\ud588\ub2e4. \uc5f4\uac15\uc740 \uc911\uad6d\uc744 \ubd84\ud560\ud558\uc9c0 \uc54a\ub294 \ub300\uc2e0 \ubcf4\uc874\ud558\uae30\ub85c \uacb0\uc815\ud558\ub294 \ud55c\ud3b8 \uccad\ub098\ub77c \uc655\uc870\uc640\uc758 \ud611\uc0c1\uc744 \uac70\uccd0 1901\ub144 9\uc6d4 7\uc77c\uc5d0 \uac15\ud654 \uc870\uc57d\uc778 \uc2e0\ucd95 \uc870\uc57d(\ubca0\uc774\uc9d5 \uc758\uc815\uc11c)\ub97c \uccb4\uacb0\ud588\ub2e4. \uadf8 \ub0b4\uc6a9\uc740 \uccad\ub098\ub77c\uac00 \uc81c\uad6d\uc8fc\uc758 \uc5f4\uac15\uc5d0 \uac70\uc561\uc758 \ubc30\uc0c1\uae08\uc744 \uc9c0\uae09\ud558\ub294 \ub3d9\uc2dc\uc5d0 \uc5f4\uac15\uc758 \uc911\uad6d \ub0b4 \uad70\ub300 \uc8fc\ub454\uad8c\uc744 \uc778\uc815\ud558\ub294 \uac83\uc774\uc5c8\ub2e4. \uc774 \uc0ac\uac74\uc73c\ub85c \uc778\ud574 \uc911\uad6d\uc758 \ubc18\uc2dd\ubbfc\uc9c0 \uc0c1\ud0dc\uac00 \ub354\uc6b1 \uc2ec\ud654\ub418\uc5c8\ub2e4."} {"question": "\ucd5c\uc9c4\uc2e4\uc740 \ud1a0\ub860\uc744 \ud1b5\ud574 \ubb34\uc5c7\uc744 \ub17c\ud30c\ud558\ub824\uace0 \ud588\ub098?", "paragraph": "2008\ub144 9\uc6d4 30\uc77c, \ucd5c\uc9c4\uc2e4\uc740 \uce5c\uad6c\uc778 \ud038\uc758 \uae40\uc7ac\uc6b0 \uae30\uc790\uc5d0\uac8c \"\uc0ac\ucc44\uc124 \uc720\ud3ec\uc790\uac00 \uc7a1\ud614\uc73c\ub2c8 \uc774\uc81c\ub294 \uc0ac\ub78c\ub4e4\uc774 \ub0b4 \uc9c4\uc2e4\uc744 \ubbff\uc5b4\uc904 \uc904 \uc54c\uc558\uc5b4. \uadf8\ub7f0\ub370 \ud1a0\ub860 \uac8c\uc2dc\ud310\uc5d0\uc11c \ub0b4 \uc774\uc57c\uae30\uc5d0 \uc11c\ub85c \ucc2c\ubc18\uc591\ub860\uc744 \ud3bc\uce58\ub294\ub370, \uc544\uc9c1\ub3c4 \ub0b4\uac00 \uc548\uc7ac\ud658\uc744 \uc8fd\uc74c\uc73c\ub85c \ubab0\uace0 \uac04 \uc545\ub355 \uc0ac\ucc44\uc5c5\uc790\ub85c \uadf8\ub300\ub85c \ubb35\uc778\ud558\ub294 \uc0ac\ub78c\ub4e4\uc774 \ub9ce\ub354\ub77c. \ub098 \ub108\ubb34 \ub5a8\ub824. \uadf8 \uac8c\uc2dc\ud310\uc5d0 \uae00\uc744 \ub0a8\uae30\uace0 \uc2f6\uc5b4. \ub098 \uc774\uc81c \ub354 \uc774\uc0c1 \ucc38\uc744 \uc218\uac00 \uc5c6\uc5b4. \uadf8\ub0e5 \uadf8 \uc0ac\ub78c\ub4e4\ud558\uace0 \ud53c \ud130\uc9c0\uac8c \uc2f8\uc6c0\uc744 \ud558\uace0 \uc2f6\uc5b4. \uc774\ub300\ub85c \uac00\ub2e4\uac04 \uc815\ub9d0 \uc8fd\uc744 \uac83 \uac19\uc544\"\ub77c\uace0 \ub9d0\ud588\ub2e4. \ucd5c\uc9c4\uc2e4\uc740 \ud1a0\ub860\uc744 \ud1b5\ud574 '\uc5ec\ub860\uc7ac\ud310'\uc744 \ub17c\ud30c\ud558\ub824\uace0 \ud588\ub358 \uac83\uc774\ub2e4. \uc774\ud6c4 \ucd5c\uc9c4\uc2e4\uc740 \uae40\uc7ac\uc6b0\uc5d0\uac8c \"\uc218\ub9ce\uc740 \uc545\ud50c\ub7ec\ub4e4\uacfc \uc2f8\uc6b0\uae30\uc5d0\ub294 \ub098 \uc790\uc2e0\uc774 \uadf8 \uc21c\uac04 \ub108\ubb34\ub3c4 \uc57d\ud55c \uc874\uc7ac\ub77c\ub294 \uac83\uc744 \ubc14\ub85c \ub290\uaf08\uace0 \uc624\ud574\uc758 \ubcbd\uc740 \ub108\ubb34\ub3c4 \ub192\uc544 \uc5b4\ub5bb\uac8c \uc190\uc744 \uc4f8 \uc218\uac00 \uc5c6\uc5b4 \ub2e8 \ud55c \uc904\uc758 \uae00\ub3c4 \uc62c\ub9b4 \uc218 \uc5c6\uc5c8\ub2e4\"\ub77c\uace0 \ub9d0\ud588\ub2e4. 2008\ub144 10\uc6d4 2\uc77c, \ucd5c\uc9c4\uc2e4\uc740 \ub9c8\ub140\uc0ac\ub0e5\uc2dd '\uc5ec\ub860\uc7ac\ud310'\uc758 \uace0\ud1b5\uc744 \ud1a0\ub85c\ud558\ub2e4\uac00 \uacb0\uad6d \uc2a4\uc2a4\ub85c \ubaa9\uc228\uc744 \ub04a\uc5c8\ub2e4.", "answer": "\uc5ec\ub860\uc7ac\ud310", "sentence": "\ucd5c\uc9c4\uc2e4\uc740 \ud1a0\ub860\uc744 \ud1b5\ud574 ' \uc5ec\ub860\uc7ac\ud310 '\uc744", "paragraph_sentence": "2008\ub144 9\uc6d4 30\uc77c, \ucd5c\uc9c4\uc2e4\uc740 \uce5c\uad6c\uc778 \ud038\uc758 \uae40\uc7ac\uc6b0 \uae30\uc790\uc5d0\uac8c \"\uc0ac\ucc44\uc124 \uc720\ud3ec\uc790\uac00 \uc7a1\ud614\uc73c\ub2c8 \uc774\uc81c\ub294 \uc0ac\ub78c\ub4e4\uc774 \ub0b4 \uc9c4\uc2e4\uc744 \ubbff\uc5b4\uc904 \uc904 \uc54c\uc558\uc5b4. \uadf8\ub7f0\ub370 \ud1a0\ub860 \uac8c\uc2dc\ud310\uc5d0\uc11c \ub0b4 \uc774\uc57c\uae30\uc5d0 \uc11c\ub85c \ucc2c\ubc18\uc591\ub860\uc744 \ud3bc\uce58\ub294\ub370, \uc544\uc9c1\ub3c4 \ub0b4\uac00 \uc548\uc7ac\ud658\uc744 \uc8fd\uc74c\uc73c\ub85c \ubab0\uace0 \uac04 \uc545\ub355 \uc0ac\ucc44\uc5c5\uc790\ub85c \uadf8\ub300\ub85c \ubb35\uc778\ud558\ub294 \uc0ac\ub78c\ub4e4\uc774 \ub9ce\ub354\ub77c. \ub098 \ub108\ubb34 \ub5a8\ub824. \uadf8 \uac8c\uc2dc\ud310\uc5d0 \uae00\uc744 \ub0a8\uae30\uace0 \uc2f6\uc5b4. \ub098 \uc774\uc81c \ub354 \uc774\uc0c1 \ucc38\uc744 \uc218\uac00 \uc5c6\uc5b4. \uadf8\ub0e5 \uadf8 \uc0ac\ub78c\ub4e4\ud558\uace0 \ud53c \ud130\uc9c0\uac8c \uc2f8\uc6c0\uc744 \ud558\uace0 \uc2f6\uc5b4. \uc774\ub300\ub85c \uac00\ub2e4\uac04 \uc815\ub9d0 \uc8fd\uc744 \uac83 \uac19\uc544\"\ub77c\uace0 \ub9d0\ud588\ub2e4. \ucd5c\uc9c4\uc2e4\uc740 \ud1a0\ub860\uc744 \ud1b5\ud574 ' \uc5ec\ub860\uc7ac\ud310 '\uc744 \ub17c\ud30c\ud558\ub824\uace0 \ud588\ub358 \uac83\uc774\ub2e4. \uc774\ud6c4 \ucd5c\uc9c4\uc2e4\uc740 \uae40\uc7ac\uc6b0\uc5d0\uac8c \"\uc218\ub9ce\uc740 \uc545\ud50c\ub7ec\ub4e4\uacfc \uc2f8\uc6b0\uae30\uc5d0\ub294 \ub098 \uc790\uc2e0\uc774 \uadf8 \uc21c\uac04 \ub108\ubb34\ub3c4 \uc57d\ud55c \uc874\uc7ac\ub77c\ub294 \uac83\uc744 \ubc14\ub85c \ub290\uaf08\uace0 \uc624\ud574\uc758 \ubcbd\uc740 \ub108\ubb34\ub3c4 \ub192\uc544 \uc5b4\ub5bb\uac8c \uc190\uc744 \uc4f8 \uc218\uac00 \uc5c6\uc5b4 \ub2e8 \ud55c \uc904\uc758 \uae00\ub3c4 \uc62c\ub9b4 \uc218 \uc5c6\uc5c8\ub2e4\"\ub77c\uace0 \ub9d0\ud588\ub2e4. 2008\ub144 10\uc6d4 2\uc77c, \ucd5c\uc9c4\uc2e4\uc740 \ub9c8\ub140\uc0ac\ub0e5\uc2dd '\uc5ec\ub860\uc7ac\ud310'\uc758 \uace0\ud1b5\uc744 \ud1a0\ub85c\ud558\ub2e4\uac00 \uacb0\uad6d \uc2a4\uc2a4\ub85c \ubaa9\uc228\uc744 \ub04a\uc5c8\ub2e4.", "paragraph_answer": "2008\ub144 9\uc6d4 30\uc77c, \ucd5c\uc9c4\uc2e4\uc740 \uce5c\uad6c\uc778 \ud038\uc758 \uae40\uc7ac\uc6b0 \uae30\uc790\uc5d0\uac8c \"\uc0ac\ucc44\uc124 \uc720\ud3ec\uc790\uac00 \uc7a1\ud614\uc73c\ub2c8 \uc774\uc81c\ub294 \uc0ac\ub78c\ub4e4\uc774 \ub0b4 \uc9c4\uc2e4\uc744 \ubbff\uc5b4\uc904 \uc904 \uc54c\uc558\uc5b4. \uadf8\ub7f0\ub370 \ud1a0\ub860 \uac8c\uc2dc\ud310\uc5d0\uc11c \ub0b4 \uc774\uc57c\uae30\uc5d0 \uc11c\ub85c \ucc2c\ubc18\uc591\ub860\uc744 \ud3bc\uce58\ub294\ub370, \uc544\uc9c1\ub3c4 \ub0b4\uac00 \uc548\uc7ac\ud658\uc744 \uc8fd\uc74c\uc73c\ub85c \ubab0\uace0 \uac04 \uc545\ub355 \uc0ac\ucc44\uc5c5\uc790\ub85c \uadf8\ub300\ub85c \ubb35\uc778\ud558\ub294 \uc0ac\ub78c\ub4e4\uc774 \ub9ce\ub354\ub77c. \ub098 \ub108\ubb34 \ub5a8\ub824. \uadf8 \uac8c\uc2dc\ud310\uc5d0 \uae00\uc744 \ub0a8\uae30\uace0 \uc2f6\uc5b4. \ub098 \uc774\uc81c \ub354 \uc774\uc0c1 \ucc38\uc744 \uc218\uac00 \uc5c6\uc5b4. \uadf8\ub0e5 \uadf8 \uc0ac\ub78c\ub4e4\ud558\uace0 \ud53c \ud130\uc9c0\uac8c \uc2f8\uc6c0\uc744 \ud558\uace0 \uc2f6\uc5b4. \uc774\ub300\ub85c \uac00\ub2e4\uac04 \uc815\ub9d0 \uc8fd\uc744 \uac83 \uac19\uc544\"\ub77c\uace0 \ub9d0\ud588\ub2e4. \ucd5c\uc9c4\uc2e4\uc740 \ud1a0\ub860\uc744 \ud1b5\ud574 ' \uc5ec\ub860\uc7ac\ud310 '\uc744 \ub17c\ud30c\ud558\ub824\uace0 \ud588\ub358 \uac83\uc774\ub2e4. \uc774\ud6c4 \ucd5c\uc9c4\uc2e4\uc740 \uae40\uc7ac\uc6b0\uc5d0\uac8c \"\uc218\ub9ce\uc740 \uc545\ud50c\ub7ec\ub4e4\uacfc \uc2f8\uc6b0\uae30\uc5d0\ub294 \ub098 \uc790\uc2e0\uc774 \uadf8 \uc21c\uac04 \ub108\ubb34\ub3c4 \uc57d\ud55c \uc874\uc7ac\ub77c\ub294 \uac83\uc744 \ubc14\ub85c \ub290\uaf08\uace0 \uc624\ud574\uc758 \ubcbd\uc740 \ub108\ubb34\ub3c4 \ub192\uc544 \uc5b4\ub5bb\uac8c \uc190\uc744 \uc4f8 \uc218\uac00 \uc5c6\uc5b4 \ub2e8 \ud55c \uc904\uc758 \uae00\ub3c4 \uc62c\ub9b4 \uc218 \uc5c6\uc5c8\ub2e4\"\ub77c\uace0 \ub9d0\ud588\ub2e4. 2008\ub144 10\uc6d4 2\uc77c, \ucd5c\uc9c4\uc2e4\uc740 \ub9c8\ub140\uc0ac\ub0e5\uc2dd '\uc5ec\ub860\uc7ac\ud310'\uc758 \uace0\ud1b5\uc744 \ud1a0\ub85c\ud558\ub2e4\uac00 \uacb0\uad6d \uc2a4\uc2a4\ub85c \ubaa9\uc228\uc744 \ub04a\uc5c8\ub2e4.", "sentence_answer": "\ucd5c\uc9c4\uc2e4\uc740 \ud1a0\ub860\uc744 \ud1b5\ud574 ' \uc5ec\ub860\uc7ac\ud310 '\uc744", "paragraph_id": "6344596-28-1", "paragraph_question": "question: \ucd5c\uc9c4\uc2e4\uc740 \ud1a0\ub860\uc744 \ud1b5\ud574 \ubb34\uc5c7\uc744 \ub17c\ud30c\ud558\ub824\uace0 \ud588\ub098?, context: 2008\ub144 9\uc6d4 30\uc77c, \ucd5c\uc9c4\uc2e4\uc740 \uce5c\uad6c\uc778 \ud038\uc758 \uae40\uc7ac\uc6b0 \uae30\uc790\uc5d0\uac8c \"\uc0ac\ucc44\uc124 \uc720\ud3ec\uc790\uac00 \uc7a1\ud614\uc73c\ub2c8 \uc774\uc81c\ub294 \uc0ac\ub78c\ub4e4\uc774 \ub0b4 \uc9c4\uc2e4\uc744 \ubbff\uc5b4\uc904 \uc904 \uc54c\uc558\uc5b4. \uadf8\ub7f0\ub370 \ud1a0\ub860 \uac8c\uc2dc\ud310\uc5d0\uc11c \ub0b4 \uc774\uc57c\uae30\uc5d0 \uc11c\ub85c \ucc2c\ubc18\uc591\ub860\uc744 \ud3bc\uce58\ub294\ub370, \uc544\uc9c1\ub3c4 \ub0b4\uac00 \uc548\uc7ac\ud658\uc744 \uc8fd\uc74c\uc73c\ub85c \ubab0\uace0 \uac04 \uc545\ub355 \uc0ac\ucc44\uc5c5\uc790\ub85c \uadf8\ub300\ub85c \ubb35\uc778\ud558\ub294 \uc0ac\ub78c\ub4e4\uc774 \ub9ce\ub354\ub77c. \ub098 \ub108\ubb34 \ub5a8\ub824. \uadf8 \uac8c\uc2dc\ud310\uc5d0 \uae00\uc744 \ub0a8\uae30\uace0 \uc2f6\uc5b4. \ub098 \uc774\uc81c \ub354 \uc774\uc0c1 \ucc38\uc744 \uc218\uac00 \uc5c6\uc5b4. \uadf8\ub0e5 \uadf8 \uc0ac\ub78c\ub4e4\ud558\uace0 \ud53c \ud130\uc9c0\uac8c \uc2f8\uc6c0\uc744 \ud558\uace0 \uc2f6\uc5b4. \uc774\ub300\ub85c \uac00\ub2e4\uac04 \uc815\ub9d0 \uc8fd\uc744 \uac83 \uac19\uc544\"\ub77c\uace0 \ub9d0\ud588\ub2e4. \ucd5c\uc9c4\uc2e4\uc740 \ud1a0\ub860\uc744 \ud1b5\ud574 '\uc5ec\ub860\uc7ac\ud310'\uc744 \ub17c\ud30c\ud558\ub824\uace0 \ud588\ub358 \uac83\uc774\ub2e4. \uc774\ud6c4 \ucd5c\uc9c4\uc2e4\uc740 \uae40\uc7ac\uc6b0\uc5d0\uac8c \"\uc218\ub9ce\uc740 \uc545\ud50c\ub7ec\ub4e4\uacfc \uc2f8\uc6b0\uae30\uc5d0\ub294 \ub098 \uc790\uc2e0\uc774 \uadf8 \uc21c\uac04 \ub108\ubb34\ub3c4 \uc57d\ud55c \uc874\uc7ac\ub77c\ub294 \uac83\uc744 \ubc14\ub85c \ub290\uaf08\uace0 \uc624\ud574\uc758 \ubcbd\uc740 \ub108\ubb34\ub3c4 \ub192\uc544 \uc5b4\ub5bb\uac8c \uc190\uc744 \uc4f8 \uc218\uac00 \uc5c6\uc5b4 \ub2e8 \ud55c \uc904\uc758 \uae00\ub3c4 \uc62c\ub9b4 \uc218 \uc5c6\uc5c8\ub2e4\"\ub77c\uace0 \ub9d0\ud588\ub2e4. 2008\ub144 10\uc6d4 2\uc77c, \ucd5c\uc9c4\uc2e4\uc740 \ub9c8\ub140\uc0ac\ub0e5\uc2dd '\uc5ec\ub860\uc7ac\ud310'\uc758 \uace0\ud1b5\uc744 \ud1a0\ub85c\ud558\ub2e4\uac00 \uacb0\uad6d \uc2a4\uc2a4\ub85c \ubaa9\uc228\uc744 \ub04a\uc5c8\ub2e4."} {"question": "\uc5ec\uc9c0\uc804\ub3c4\uac00 \uad6c\ub300\ub959\ub9cc\uc744 \uc138\uacc4\uc9c0\ub3c4\uc5d0 \ub2f4\uc740 \uc774\uc720\ub294?", "paragraph": "\uc5ec\uc9c0\uc804\ub3c4\ub294 \uc591\ubc18\uad6c\ub3c4\ub97c \uae30\ucd08\uc790\ub8cc\ub85c \ud65c\uc6a9\ud588\uc74c\uc5d0\ub3c4 \ubd88\uad6c\ud558\uace0 \uc2e0\ub300\ub959\uc774 \uadf8\ub824\uc9c4 \ubc18\uad6c\ub3c4\ub97c \uc81c\uc678\uc2dc\ud0a4\uace0 \uad6c\ub300\ub959\uc744 \uc911\uc2ec\uc73c\ub85c \uadf8\ub9b0 \uc810\uc774 \ud2b9\uc9d5\uc801\uc774\ub2e4. \uc774\ucc98\ub7fc \uad6c\ub300\ub959\uacfc \uc2e0\ub300\ub959\uc73c\ub85c \uc774\ub8e8\uc5b4\uc9c4 \uc591\ubc18\uad6c\ub3c4 \uac00\uc6b4\ub370 \uc624\uc138\uc544\ub2c8\uc544 \ub300\ub959\uc744 \ud3ec\ud568\ud55c \uad6c\ub300\ub959\ub9cc\uc744 \uc138\uacc4\uc9c0\ub3c4\uc5d0 \ud3ec\ud604\ud55c \uc774\uc720\ub294 \ubb34\uc5c7\uc77c\uae4c? \ub9c8\ud14c\uc624\ub9ac\uce58\uac00 \uc81c\uc791\ud588\ub358 \uc11c\uad6c\uc2dd \uc138\uacc4\uc9c0\ub3c4\ub294 \ub300\ubd80\ubd84 \ud0c0\uc6d0\ud615\uc73c\ub85c \uc81c\uc791\ub418\uc5b4 \uc774 \uc548\uc5d0 \uc2e0\ub300\ub959\uae4c\uc9c0 \ud3ec\uad04\ud558\uace0 \uc788\uc5c8\ub2e4. \uadf8\ub7ec\ub098 \ud398\ub974\ube44\uc2a4\ud2b8\uc758 \uace4\uc5ec\uc804\ub3c4\uc5d0\uc11c\ub294 \uc591\ubc18\uad6c\ub3c4\ub85c \uc81c\uc791\ub418\uc5c8\ub294\ub370, \uc9c0\uad6c\uad6c\uccb4\uc124\uc5d0 \ub300\ud55c \uc774\ud574\uac00 \uc5c6\uc73c\uba74 \uc9c0\ub3c4\ub97c \uc774\ud574\ud558\uae30\uac00 \uc5b4\ub824\uc6e0\ub358 \uac83\uc73c\ub85c \ubcf4\uc778\ub2e4. \ubb3c\ub860 \ub9c8\ud14c\uc624 \ub9ac\uce58\uc758 \uace4\uc5ec\ub9cc\uad6d\uc804\ub3c4\ub3c4 \uc9c0\uad6c\uad6c\uccb4\uc124\uc774 \ub4b7\ubc1b\uce68\uc774 \ub418\uc5b4\uc57c \uc81c\ub300\ub85c \uc774\ud574\ub418\ub294 \uce21\uba74\uc774 \uc788\uc5c8\uc9c0\ub9cc \uc591\ubc18\uad6c\ub3c4\ubcf4\ub2e4\ub294 \uadf8 \uc815\ub3c4\uac00 \ub35c\ud558\ub2e4\uace0 \ubcfc \uc218 \uc788\ub2e4. \uadf8\ub7ec\ud55c \uc0ac\uc2e4\ub4e4\uc744 \uace0\ub824\ud560 \ub54c \uc591\ubc18\uad6c\ub3c4 \uac00\uc6b4\ub370 \ub3d9\uc591\uc778\uc5d0\uac8c\ub294 \uc0dd\uc18c\ud55c \uc2e0\ub300\ub959\uc744 \uadf8\ub9b0 \uc9c0\ub3c4\ub294 \ud070 \uc758\ubbf8\ub97c \uc9c0\ub2c8\uc9c0 \ubabb\ud588\ub358 \uac83\uc73c\ub85c \ubcfc \uc218 \uc788\ub2e4. \ubbf8\uc9c0\uc758 \uc138\uacc4\uc778 \uc2e0\ub300\ub959\uc774 \ub300\ub959\uc758 \ub3d9\ub2e8\uc5d0 \uc704\uce58\ud55c \uc870\uc120\uc758 \uc9c0\uc2dd\uc778\uc5d0\uac8c \uc774\ud574\ub418\uae30\uc5d0\ub294 \ub9ce\uc740 \uc5b4\ub824\uc6c0\uc774 \uc788\uc5c8\ub358 \uac83\uc774\ub2e4. \uc2e0\ub300\ub959\uc774\ub77c\ub294 \uacf5\uac04\uc774 \ub2f9\uc2dc \uc870\uc120\uc9c0\uc2dd\uc778\uc5d0\uac8c \uc758\ubbf8\uc788\ub294 \uc7a5\uc18c\ub85c \uc774\ud574\ub418\uae30 \uc704\ud574\uc11c\ub294 \ucd5c\uc18c\ud55c\uc758 \uad00\uacc4\uac00 \uc5ed\uc0ac\uc801\uc73c\ub85c \uc131\ub9bd\ub418\uc5b4 \uc788\uc5b4\uc57c \ud588\ub2e4. \uc774\uac83\uc740 \uc2e4\uc9c0\uc758 \uacbd\ud5d8 \uc18d\uc5d0\uc11c \ud68d\ub4dd\ub418\ub4e0\uc9c0 \uc544\ub2c8\uba74 \uc0b0\ud574\uacbd\uc5d0 \ub098\uc624\ub294 \uc9c0\uba85\ucc98\ub7fc \uc0c1\uc0c1 \uc18d\uc5d0\uc11c \ud68d\ub4dd\ub420 \uc218\ub3c4 \uc788\ub2e4. \uadf8\ub7ec\ub098 \uc2e0\ub300\ub959\uc5d0 \ud45c\uae30\ub41c \uc9c0\uba85\ub4e4\uc740 \uc774\uc640\ub294 \uac70\ub9ac\uac00 \uba3c \uc0dd\uc18c\ud55c \uac83\ub4e4\uc774\ub2e4. \ub530\ub77c\uc11c \uc2e0\ub300\ub959\uc740 \uac1d\uad00\uc801\uc778 \uc2e4\uc7ac \uc5ec\ubd80\ub97c \ub5a0\ub098 \uc778\uc2dd\ub41c \uc138\uacc4\uc5d0\uc11c\ub294 \uc0ac\ub77c\uc84c\ub358 \uac83\uc774\ub2e4.", "answer": "\ub3d9\uc591\uc778\uc5d0\uac8c\ub294 \uc0dd\uc18c\ud55c \uc2e0\ub300\ub959", "sentence": "\uadf8\ub7ec\ud55c \uc0ac\uc2e4\ub4e4\uc744 \uace0\ub824\ud560 \ub54c \uc591\ubc18\uad6c\ub3c4 \uac00\uc6b4\ub370 \ub3d9\uc591\uc778\uc5d0\uac8c\ub294 \uc0dd\uc18c\ud55c \uc2e0\ub300\ub959 \uc744 \uadf8\ub9b0 \uc9c0\ub3c4\ub294 \ud070 \uc758\ubbf8\ub97c \uc9c0\ub2c8\uc9c0 \ubabb\ud588\ub358 \uac83\uc73c\ub85c \ubcfc \uc218 \uc788\ub2e4.", "paragraph_sentence": "\uc5ec\uc9c0\uc804\ub3c4\ub294 \uc591\ubc18\uad6c\ub3c4\ub97c \uae30\ucd08\uc790\ub8cc\ub85c \ud65c\uc6a9\ud588\uc74c\uc5d0\ub3c4 \ubd88\uad6c\ud558\uace0 \uc2e0\ub300\ub959\uc774 \uadf8\ub824\uc9c4 \ubc18\uad6c\ub3c4\ub97c \uc81c\uc678\uc2dc\ud0a4\uace0 \uad6c\ub300\ub959\uc744 \uc911\uc2ec\uc73c\ub85c \uadf8\ub9b0 \uc810\uc774 \ud2b9\uc9d5\uc801\uc774\ub2e4. \uc774\ucc98\ub7fc \uad6c\ub300\ub959\uacfc \uc2e0\ub300\ub959\uc73c\ub85c \uc774\ub8e8\uc5b4\uc9c4 \uc591\ubc18\uad6c\ub3c4 \uac00\uc6b4\ub370 \uc624\uc138\uc544\ub2c8\uc544 \ub300\ub959\uc744 \ud3ec\ud568\ud55c \uad6c\ub300\ub959\ub9cc\uc744 \uc138\uacc4\uc9c0\ub3c4\uc5d0 \ud3ec\ud604\ud55c \uc774\uc720\ub294 \ubb34\uc5c7\uc77c\uae4c? \ub9c8\ud14c\uc624\ub9ac\uce58\uac00 \uc81c\uc791\ud588\ub358 \uc11c\uad6c\uc2dd \uc138\uacc4\uc9c0\ub3c4\ub294 \ub300\ubd80\ubd84 \ud0c0\uc6d0\ud615\uc73c\ub85c \uc81c\uc791\ub418\uc5b4 \uc774 \uc548\uc5d0 \uc2e0\ub300\ub959\uae4c\uc9c0 \ud3ec\uad04\ud558\uace0 \uc788\uc5c8\ub2e4. \uadf8\ub7ec\ub098 \ud398\ub974\ube44\uc2a4\ud2b8\uc758 \uace4\uc5ec\uc804\ub3c4\uc5d0\uc11c\ub294 \uc591\ubc18\uad6c\ub3c4\ub85c \uc81c\uc791\ub418\uc5c8\ub294\ub370, \uc9c0\uad6c\uad6c\uccb4\uc124\uc5d0 \ub300\ud55c \uc774\ud574\uac00 \uc5c6\uc73c\uba74 \uc9c0\ub3c4\ub97c \uc774\ud574\ud558\uae30\uac00 \uc5b4\ub824\uc6e0\ub358 \uac83\uc73c\ub85c \ubcf4\uc778\ub2e4. \ubb3c\ub860 \ub9c8\ud14c\uc624 \ub9ac\uce58\uc758 \uace4\uc5ec\ub9cc\uad6d\uc804\ub3c4\ub3c4 \uc9c0\uad6c\uad6c\uccb4\uc124\uc774 \ub4b7\ubc1b\uce68\uc774 \ub418\uc5b4\uc57c \uc81c\ub300\ub85c \uc774\ud574\ub418\ub294 \uce21\uba74\uc774 \uc788\uc5c8\uc9c0\ub9cc \uc591\ubc18\uad6c\ub3c4\ubcf4\ub2e4\ub294 \uadf8 \uc815\ub3c4\uac00 \ub35c\ud558\ub2e4\uace0 \ubcfc \uc218 \uc788\ub2e4. \uadf8\ub7ec\ud55c \uc0ac\uc2e4\ub4e4\uc744 \uace0\ub824\ud560 \ub54c \uc591\ubc18\uad6c\ub3c4 \uac00\uc6b4\ub370 \ub3d9\uc591\uc778\uc5d0\uac8c\ub294 \uc0dd\uc18c\ud55c \uc2e0\ub300\ub959 \uc744 \uadf8\ub9b0 \uc9c0\ub3c4\ub294 \ud070 \uc758\ubbf8\ub97c \uc9c0\ub2c8\uc9c0 \ubabb\ud588\ub358 \uac83\uc73c\ub85c \ubcfc \uc218 \uc788\ub2e4. \ubbf8\uc9c0\uc758 \uc138\uacc4\uc778 \uc2e0\ub300\ub959\uc774 \ub300\ub959\uc758 \ub3d9\ub2e8\uc5d0 \uc704\uce58\ud55c \uc870\uc120\uc758 \uc9c0\uc2dd\uc778\uc5d0\uac8c \uc774\ud574\ub418\uae30\uc5d0\ub294 \ub9ce\uc740 \uc5b4\ub824\uc6c0\uc774 \uc788\uc5c8\ub358 \uac83\uc774\ub2e4. \uc2e0\ub300\ub959\uc774\ub77c\ub294 \uacf5\uac04\uc774 \ub2f9\uc2dc \uc870\uc120\uc9c0\uc2dd\uc778\uc5d0\uac8c \uc758\ubbf8\uc788\ub294 \uc7a5\uc18c\ub85c \uc774\ud574\ub418\uae30 \uc704\ud574\uc11c\ub294 \ucd5c\uc18c\ud55c\uc758 \uad00\uacc4\uac00 \uc5ed\uc0ac\uc801\uc73c\ub85c \uc131\ub9bd\ub418\uc5b4 \uc788\uc5b4\uc57c \ud588\ub2e4. \uc774\uac83\uc740 \uc2e4\uc9c0\uc758 \uacbd\ud5d8 \uc18d\uc5d0\uc11c \ud68d\ub4dd\ub418\ub4e0\uc9c0 \uc544\ub2c8\uba74 \uc0b0\ud574\uacbd\uc5d0 \ub098\uc624\ub294 \uc9c0\uba85\ucc98\ub7fc \uc0c1\uc0c1 \uc18d\uc5d0\uc11c \ud68d\ub4dd\ub420 \uc218\ub3c4 \uc788\ub2e4. \uadf8\ub7ec\ub098 \uc2e0\ub300\ub959\uc5d0 \ud45c\uae30\ub41c \uc9c0\uba85\ub4e4\uc740 \uc774\uc640\ub294 \uac70\ub9ac\uac00 \uba3c \uc0dd\uc18c\ud55c \uac83\ub4e4\uc774\ub2e4. \ub530\ub77c\uc11c \uc2e0\ub300\ub959\uc740 \uac1d\uad00\uc801\uc778 \uc2e4\uc7ac \uc5ec\ubd80\ub97c \ub5a0\ub098 \uc778\uc2dd\ub41c \uc138\uacc4\uc5d0\uc11c\ub294 \uc0ac\ub77c\uc84c\ub358 \uac83\uc774\ub2e4.", "paragraph_answer": "\uc5ec\uc9c0\uc804\ub3c4\ub294 \uc591\ubc18\uad6c\ub3c4\ub97c \uae30\ucd08\uc790\ub8cc\ub85c \ud65c\uc6a9\ud588\uc74c\uc5d0\ub3c4 \ubd88\uad6c\ud558\uace0 \uc2e0\ub300\ub959\uc774 \uadf8\ub824\uc9c4 \ubc18\uad6c\ub3c4\ub97c \uc81c\uc678\uc2dc\ud0a4\uace0 \uad6c\ub300\ub959\uc744 \uc911\uc2ec\uc73c\ub85c \uadf8\ub9b0 \uc810\uc774 \ud2b9\uc9d5\uc801\uc774\ub2e4. \uc774\ucc98\ub7fc \uad6c\ub300\ub959\uacfc \uc2e0\ub300\ub959\uc73c\ub85c \uc774\ub8e8\uc5b4\uc9c4 \uc591\ubc18\uad6c\ub3c4 \uac00\uc6b4\ub370 \uc624\uc138\uc544\ub2c8\uc544 \ub300\ub959\uc744 \ud3ec\ud568\ud55c \uad6c\ub300\ub959\ub9cc\uc744 \uc138\uacc4\uc9c0\ub3c4\uc5d0 \ud3ec\ud604\ud55c \uc774\uc720\ub294 \ubb34\uc5c7\uc77c\uae4c? \ub9c8\ud14c\uc624\ub9ac\uce58\uac00 \uc81c\uc791\ud588\ub358 \uc11c\uad6c\uc2dd \uc138\uacc4\uc9c0\ub3c4\ub294 \ub300\ubd80\ubd84 \ud0c0\uc6d0\ud615\uc73c\ub85c \uc81c\uc791\ub418\uc5b4 \uc774 \uc548\uc5d0 \uc2e0\ub300\ub959\uae4c\uc9c0 \ud3ec\uad04\ud558\uace0 \uc788\uc5c8\ub2e4. \uadf8\ub7ec\ub098 \ud398\ub974\ube44\uc2a4\ud2b8\uc758 \uace4\uc5ec\uc804\ub3c4\uc5d0\uc11c\ub294 \uc591\ubc18\uad6c\ub3c4\ub85c \uc81c\uc791\ub418\uc5c8\ub294\ub370, \uc9c0\uad6c\uad6c\uccb4\uc124\uc5d0 \ub300\ud55c \uc774\ud574\uac00 \uc5c6\uc73c\uba74 \uc9c0\ub3c4\ub97c \uc774\ud574\ud558\uae30\uac00 \uc5b4\ub824\uc6e0\ub358 \uac83\uc73c\ub85c \ubcf4\uc778\ub2e4. \ubb3c\ub860 \ub9c8\ud14c\uc624 \ub9ac\uce58\uc758 \uace4\uc5ec\ub9cc\uad6d\uc804\ub3c4\ub3c4 \uc9c0\uad6c\uad6c\uccb4\uc124\uc774 \ub4b7\ubc1b\uce68\uc774 \ub418\uc5b4\uc57c \uc81c\ub300\ub85c \uc774\ud574\ub418\ub294 \uce21\uba74\uc774 \uc788\uc5c8\uc9c0\ub9cc \uc591\ubc18\uad6c\ub3c4\ubcf4\ub2e4\ub294 \uadf8 \uc815\ub3c4\uac00 \ub35c\ud558\ub2e4\uace0 \ubcfc \uc218 \uc788\ub2e4. \uadf8\ub7ec\ud55c \uc0ac\uc2e4\ub4e4\uc744 \uace0\ub824\ud560 \ub54c \uc591\ubc18\uad6c\ub3c4 \uac00\uc6b4\ub370 \ub3d9\uc591\uc778\uc5d0\uac8c\ub294 \uc0dd\uc18c\ud55c \uc2e0\ub300\ub959 \uc744 \uadf8\ub9b0 \uc9c0\ub3c4\ub294 \ud070 \uc758\ubbf8\ub97c \uc9c0\ub2c8\uc9c0 \ubabb\ud588\ub358 \uac83\uc73c\ub85c \ubcfc \uc218 \uc788\ub2e4. \ubbf8\uc9c0\uc758 \uc138\uacc4\uc778 \uc2e0\ub300\ub959\uc774 \ub300\ub959\uc758 \ub3d9\ub2e8\uc5d0 \uc704\uce58\ud55c \uc870\uc120\uc758 \uc9c0\uc2dd\uc778\uc5d0\uac8c \uc774\ud574\ub418\uae30\uc5d0\ub294 \ub9ce\uc740 \uc5b4\ub824\uc6c0\uc774 \uc788\uc5c8\ub358 \uac83\uc774\ub2e4. \uc2e0\ub300\ub959\uc774\ub77c\ub294 \uacf5\uac04\uc774 \ub2f9\uc2dc \uc870\uc120\uc9c0\uc2dd\uc778\uc5d0\uac8c \uc758\ubbf8\uc788\ub294 \uc7a5\uc18c\ub85c \uc774\ud574\ub418\uae30 \uc704\ud574\uc11c\ub294 \ucd5c\uc18c\ud55c\uc758 \uad00\uacc4\uac00 \uc5ed\uc0ac\uc801\uc73c\ub85c \uc131\ub9bd\ub418\uc5b4 \uc788\uc5b4\uc57c \ud588\ub2e4. \uc774\uac83\uc740 \uc2e4\uc9c0\uc758 \uacbd\ud5d8 \uc18d\uc5d0\uc11c \ud68d\ub4dd\ub418\ub4e0\uc9c0 \uc544\ub2c8\uba74 \uc0b0\ud574\uacbd\uc5d0 \ub098\uc624\ub294 \uc9c0\uba85\ucc98\ub7fc \uc0c1\uc0c1 \uc18d\uc5d0\uc11c \ud68d\ub4dd\ub420 \uc218\ub3c4 \uc788\ub2e4. \uadf8\ub7ec\ub098 \uc2e0\ub300\ub959\uc5d0 \ud45c\uae30\ub41c \uc9c0\uba85\ub4e4\uc740 \uc774\uc640\ub294 \uac70\ub9ac\uac00 \uba3c \uc0dd\uc18c\ud55c \uac83\ub4e4\uc774\ub2e4. \ub530\ub77c\uc11c \uc2e0\ub300\ub959\uc740 \uac1d\uad00\uc801\uc778 \uc2e4\uc7ac \uc5ec\ubd80\ub97c \ub5a0\ub098 \uc778\uc2dd\ub41c \uc138\uacc4\uc5d0\uc11c\ub294 \uc0ac\ub77c\uc84c\ub358 \uac83\uc774\ub2e4.", "sentence_answer": "\uadf8\ub7ec\ud55c \uc0ac\uc2e4\ub4e4\uc744 \uace0\ub824\ud560 \ub54c \uc591\ubc18\uad6c\ub3c4 \uac00\uc6b4\ub370 \ub3d9\uc591\uc778\uc5d0\uac8c\ub294 \uc0dd\uc18c\ud55c \uc2e0\ub300\ub959 \uc744 \uadf8\ub9b0 \uc9c0\ub3c4\ub294 \ud070 \uc758\ubbf8\ub97c \uc9c0\ub2c8\uc9c0 \ubabb\ud588\ub358 \uac83\uc73c\ub85c \ubcfc \uc218 \uc788\ub2e4.", "paragraph_id": "6587352-2-0", "paragraph_question": "question: \uc5ec\uc9c0\uc804\ub3c4\uac00 \uad6c\ub300\ub959\ub9cc\uc744 \uc138\uacc4\uc9c0\ub3c4\uc5d0 \ub2f4\uc740 \uc774\uc720\ub294?, context: \uc5ec\uc9c0\uc804\ub3c4\ub294 \uc591\ubc18\uad6c\ub3c4\ub97c \uae30\ucd08\uc790\ub8cc\ub85c \ud65c\uc6a9\ud588\uc74c\uc5d0\ub3c4 \ubd88\uad6c\ud558\uace0 \uc2e0\ub300\ub959\uc774 \uadf8\ub824\uc9c4 \ubc18\uad6c\ub3c4\ub97c \uc81c\uc678\uc2dc\ud0a4\uace0 \uad6c\ub300\ub959\uc744 \uc911\uc2ec\uc73c\ub85c \uadf8\ub9b0 \uc810\uc774 \ud2b9\uc9d5\uc801\uc774\ub2e4. \uc774\ucc98\ub7fc \uad6c\ub300\ub959\uacfc \uc2e0\ub300\ub959\uc73c\ub85c \uc774\ub8e8\uc5b4\uc9c4 \uc591\ubc18\uad6c\ub3c4 \uac00\uc6b4\ub370 \uc624\uc138\uc544\ub2c8\uc544 \ub300\ub959\uc744 \ud3ec\ud568\ud55c \uad6c\ub300\ub959\ub9cc\uc744 \uc138\uacc4\uc9c0\ub3c4\uc5d0 \ud3ec\ud604\ud55c \uc774\uc720\ub294 \ubb34\uc5c7\uc77c\uae4c? \ub9c8\ud14c\uc624\ub9ac\uce58\uac00 \uc81c\uc791\ud588\ub358 \uc11c\uad6c\uc2dd \uc138\uacc4\uc9c0\ub3c4\ub294 \ub300\ubd80\ubd84 \ud0c0\uc6d0\ud615\uc73c\ub85c \uc81c\uc791\ub418\uc5b4 \uc774 \uc548\uc5d0 \uc2e0\ub300\ub959\uae4c\uc9c0 \ud3ec\uad04\ud558\uace0 \uc788\uc5c8\ub2e4. \uadf8\ub7ec\ub098 \ud398\ub974\ube44\uc2a4\ud2b8\uc758 \uace4\uc5ec\uc804\ub3c4\uc5d0\uc11c\ub294 \uc591\ubc18\uad6c\ub3c4\ub85c \uc81c\uc791\ub418\uc5c8\ub294\ub370, \uc9c0\uad6c\uad6c\uccb4\uc124\uc5d0 \ub300\ud55c \uc774\ud574\uac00 \uc5c6\uc73c\uba74 \uc9c0\ub3c4\ub97c \uc774\ud574\ud558\uae30\uac00 \uc5b4\ub824\uc6e0\ub358 \uac83\uc73c\ub85c \ubcf4\uc778\ub2e4. \ubb3c\ub860 \ub9c8\ud14c\uc624 \ub9ac\uce58\uc758 \uace4\uc5ec\ub9cc\uad6d\uc804\ub3c4\ub3c4 \uc9c0\uad6c\uad6c\uccb4\uc124\uc774 \ub4b7\ubc1b\uce68\uc774 \ub418\uc5b4\uc57c \uc81c\ub300\ub85c \uc774\ud574\ub418\ub294 \uce21\uba74\uc774 \uc788\uc5c8\uc9c0\ub9cc \uc591\ubc18\uad6c\ub3c4\ubcf4\ub2e4\ub294 \uadf8 \uc815\ub3c4\uac00 \ub35c\ud558\ub2e4\uace0 \ubcfc \uc218 \uc788\ub2e4. \uadf8\ub7ec\ud55c \uc0ac\uc2e4\ub4e4\uc744 \uace0\ub824\ud560 \ub54c \uc591\ubc18\uad6c\ub3c4 \uac00\uc6b4\ub370 \ub3d9\uc591\uc778\uc5d0\uac8c\ub294 \uc0dd\uc18c\ud55c \uc2e0\ub300\ub959\uc744 \uadf8\ub9b0 \uc9c0\ub3c4\ub294 \ud070 \uc758\ubbf8\ub97c \uc9c0\ub2c8\uc9c0 \ubabb\ud588\ub358 \uac83\uc73c\ub85c \ubcfc \uc218 \uc788\ub2e4. \ubbf8\uc9c0\uc758 \uc138\uacc4\uc778 \uc2e0\ub300\ub959\uc774 \ub300\ub959\uc758 \ub3d9\ub2e8\uc5d0 \uc704\uce58\ud55c \uc870\uc120\uc758 \uc9c0\uc2dd\uc778\uc5d0\uac8c \uc774\ud574\ub418\uae30\uc5d0\ub294 \ub9ce\uc740 \uc5b4\ub824\uc6c0\uc774 \uc788\uc5c8\ub358 \uac83\uc774\ub2e4. \uc2e0\ub300\ub959\uc774\ub77c\ub294 \uacf5\uac04\uc774 \ub2f9\uc2dc \uc870\uc120\uc9c0\uc2dd\uc778\uc5d0\uac8c \uc758\ubbf8\uc788\ub294 \uc7a5\uc18c\ub85c \uc774\ud574\ub418\uae30 \uc704\ud574\uc11c\ub294 \ucd5c\uc18c\ud55c\uc758 \uad00\uacc4\uac00 \uc5ed\uc0ac\uc801\uc73c\ub85c \uc131\ub9bd\ub418\uc5b4 \uc788\uc5b4\uc57c \ud588\ub2e4. \uc774\uac83\uc740 \uc2e4\uc9c0\uc758 \uacbd\ud5d8 \uc18d\uc5d0\uc11c \ud68d\ub4dd\ub418\ub4e0\uc9c0 \uc544\ub2c8\uba74 \uc0b0\ud574\uacbd\uc5d0 \ub098\uc624\ub294 \uc9c0\uba85\ucc98\ub7fc \uc0c1\uc0c1 \uc18d\uc5d0\uc11c \ud68d\ub4dd\ub420 \uc218\ub3c4 \uc788\ub2e4. \uadf8\ub7ec\ub098 \uc2e0\ub300\ub959\uc5d0 \ud45c\uae30\ub41c \uc9c0\uba85\ub4e4\uc740 \uc774\uc640\ub294 \uac70\ub9ac\uac00 \uba3c \uc0dd\uc18c\ud55c \uac83\ub4e4\uc774\ub2e4. \ub530\ub77c\uc11c \uc2e0\ub300\ub959\uc740 \uac1d\uad00\uc801\uc778 \uc2e4\uc7ac \uc5ec\ubd80\ub97c \ub5a0\ub098 \uc778\uc2dd\ub41c \uc138\uacc4\uc5d0\uc11c\ub294 \uc0ac\ub77c\uc84c\ub358 \uac83\uc774\ub2e4."} {"question": "\uc2dc\ud2f0\uac00 \uc878\uc5c5\ud55c \ucd08\ub4f1\ud559\uad50\ub294?", "paragraph": "\uc5b4\ub9b4 \uc801 \uc2dc\ud2f0\ub294 \uccb4\uc721\ud65c\ub3d9 \ubc0f \ud2b9\ubcc4\ud65c\ub3d9(co-curricular) \uac19\uc740 \ud559\uad50 \ud65c\ub3d9\uc744 \ud558\uace0 \uc788\uc5c8\ub2e4. \ubca0\ub809 \ud3f4\ub9ac\uc2a4\uc5d0 \uc704\uce58\ud55c \ud0c0\ub514\uce74 \ud37c\ub974\ucef5(Tadika Perkep)\uc5d0\uc11c \uccab \ud559\uad50 \ud65c\ub3d9\uc744 \ud588\ub294\ub370, \uc774 \ud559\uad50\ub294 \uc2dc\ud2f0\uac00 \ucc98\uc74c\uc73c\ub85c \ub178\ub798 \uc2e4\ub825\uc744 \uc120\ubcf4\uc600\ub358 \ud559\uad50\uc600\ub2e4(\ub2f9\uc2dc \uc2dc\ud2f0\ub294 \ub9d0\ub808\uc774 \uc804\ud1b5 \uac00\uc694\uc778 \uc744 \ubd88\ub800\ub2e4). \ud6d7\ub0a0 \uc2dc\ud2f0\ub294 \ud074\ub9ac\ud3ec\ub4dc \ucd08\ub4f1\ud559\uad50\uc5d0\uc11c \ucd08\ub4f1 \uad50\uc721\uc744, \ud30c\ud56d \uc8fc \ucfe0\uc54c\ub77c \ub9ac\ud53c\uc2a4\uc758 \ud074\ub9ac\ud3ec\ub4dc \uc911\ud559\uad50\uc5d0\uc11c \uc911\ub4f1 \uad50\uc721\uc744 \ubc1b\uc558\ub2e4. 12\uc0b4\uc774\ub358 1991\ub144 \ub3c5\ub9bd \uae30\ub150\uc744\uc744 \ub9de\uc544 \uc5f4\ub838\ub358 \uc560\uad6d\uac00\uc694 \ub300\ud68c\uc5d0\uc11c \ub97c \ubd80\ub974\uace0 \uc0c1\uc744 \ud0d4\ub2e4. \ud559\ucc3d \uc2dc\uc808\uc5d0\ub294 \uc6b4\ub3d9 \uc120\uc218\ub85c\ub3c4 \ud65c\ub3d9\ud588\uc5c8\ub294\ub370, \uadf8 \uc911\uc5d0\uc11c\ub3c4 \ub124\ud2b8\ubcfc(netball)\uc744 \ud588\ub2e4. \uc774\ub97c \uacc4\uae30\ub85c 2006\ub144 \ucfe0\uc548\ud0c4\uc5d0\uc11c \uc5f4\ub9b0 \ud53c\uc5d0\uc2a4\ud0c0 \uba54\ub514\uc544 \uc774\ub3cc\ub77c 2006(Fiesta Media Idola 2006)\uc5d0 \ucc38\uac00\ud558\uae30\ub3c4 \ud588\ub2e4.", "answer": "\ud074\ub9ac\ud3ec\ub4dc \ucd08\ub4f1\ud559\uad50", "sentence": "\ud6d7\ub0a0 \uc2dc\ud2f0\ub294 \ud074\ub9ac\ud3ec\ub4dc \ucd08\ub4f1\ud559\uad50 \uc5d0\uc11c \ucd08\ub4f1 \uad50\uc721\uc744, \ud30c\ud56d \uc8fc \ucfe0\uc54c\ub77c \ub9ac\ud53c\uc2a4\uc758 \ud074\ub9ac\ud3ec\ub4dc \uc911\ud559\uad50\uc5d0\uc11c \uc911\ub4f1 \uad50\uc721\uc744 \ubc1b\uc558\ub2e4.", "paragraph_sentence": "\uc5b4\ub9b4 \uc801 \uc2dc\ud2f0\ub294 \uccb4\uc721\ud65c\ub3d9 \ubc0f \ud2b9\ubcc4\ud65c\ub3d9(co-curricular) \uac19\uc740 \ud559\uad50 \ud65c\ub3d9\uc744 \ud558\uace0 \uc788\uc5c8\ub2e4. \ubca0\ub809 \ud3f4\ub9ac\uc2a4\uc5d0 \uc704\uce58\ud55c \ud0c0\ub514\uce74 \ud37c\ub974\ucef5(Tadika Perkep)\uc5d0\uc11c \uccab \ud559\uad50 \ud65c\ub3d9\uc744 \ud588\ub294\ub370, \uc774 \ud559\uad50\ub294 \uc2dc\ud2f0\uac00 \ucc98\uc74c\uc73c\ub85c \ub178\ub798 \uc2e4\ub825\uc744 \uc120\ubcf4\uc600\ub358 \ud559\uad50\uc600\ub2e4(\ub2f9\uc2dc \uc2dc\ud2f0\ub294 \ub9d0\ub808\uc774 \uc804\ud1b5 \uac00\uc694\uc778 \uc744 \ubd88\ub800\ub2e4). \ud6d7\ub0a0 \uc2dc\ud2f0\ub294 \ud074\ub9ac\ud3ec\ub4dc \ucd08\ub4f1\ud559\uad50 \uc5d0\uc11c \ucd08\ub4f1 \uad50\uc721\uc744, \ud30c\ud56d \uc8fc \ucfe0\uc54c\ub77c \ub9ac\ud53c\uc2a4\uc758 \ud074\ub9ac\ud3ec\ub4dc \uc911\ud559\uad50\uc5d0\uc11c \uc911\ub4f1 \uad50\uc721\uc744 \ubc1b\uc558\ub2e4. 12\uc0b4\uc774\ub358 1991\ub144 \ub3c5\ub9bd \uae30\ub150\uc744\uc744 \ub9de\uc544 \uc5f4\ub838\ub358 \uc560\uad6d\uac00\uc694 \ub300\ud68c\uc5d0\uc11c \ub97c \ubd80\ub974\uace0 \uc0c1\uc744 \ud0d4\ub2e4. \ud559\ucc3d \uc2dc\uc808\uc5d0\ub294 \uc6b4\ub3d9 \uc120\uc218\ub85c\ub3c4 \ud65c\ub3d9\ud588\uc5c8\ub294\ub370, \uadf8 \uc911\uc5d0\uc11c\ub3c4 \ub124\ud2b8\ubcfc(netball)\uc744 \ud588\ub2e4. \uc774\ub97c \uacc4\uae30\ub85c 2006\ub144 \ucfe0\uc548\ud0c4\uc5d0\uc11c \uc5f4\ub9b0 \ud53c\uc5d0\uc2a4\ud0c0 \uba54\ub514\uc544 \uc774\ub3cc\ub77c 2006(Fiesta Media Idola 2006)\uc5d0 \ucc38\uac00\ud558\uae30\ub3c4 \ud588\ub2e4.", "paragraph_answer": "\uc5b4\ub9b4 \uc801 \uc2dc\ud2f0\ub294 \uccb4\uc721\ud65c\ub3d9 \ubc0f \ud2b9\ubcc4\ud65c\ub3d9(co-curricular) \uac19\uc740 \ud559\uad50 \ud65c\ub3d9\uc744 \ud558\uace0 \uc788\uc5c8\ub2e4. \ubca0\ub809 \ud3f4\ub9ac\uc2a4\uc5d0 \uc704\uce58\ud55c \ud0c0\ub514\uce74 \ud37c\ub974\ucef5(Tadika Perkep)\uc5d0\uc11c \uccab \ud559\uad50 \ud65c\ub3d9\uc744 \ud588\ub294\ub370, \uc774 \ud559\uad50\ub294 \uc2dc\ud2f0\uac00 \ucc98\uc74c\uc73c\ub85c \ub178\ub798 \uc2e4\ub825\uc744 \uc120\ubcf4\uc600\ub358 \ud559\uad50\uc600\ub2e4(\ub2f9\uc2dc \uc2dc\ud2f0\ub294 \ub9d0\ub808\uc774 \uc804\ud1b5 \uac00\uc694\uc778 \uc744 \ubd88\ub800\ub2e4). \ud6d7\ub0a0 \uc2dc\ud2f0\ub294 \ud074\ub9ac\ud3ec\ub4dc \ucd08\ub4f1\ud559\uad50 \uc5d0\uc11c \ucd08\ub4f1 \uad50\uc721\uc744, \ud30c\ud56d \uc8fc \ucfe0\uc54c\ub77c \ub9ac\ud53c\uc2a4\uc758 \ud074\ub9ac\ud3ec\ub4dc \uc911\ud559\uad50\uc5d0\uc11c \uc911\ub4f1 \uad50\uc721\uc744 \ubc1b\uc558\ub2e4. 12\uc0b4\uc774\ub358 1991\ub144 \ub3c5\ub9bd \uae30\ub150\uc744\uc744 \ub9de\uc544 \uc5f4\ub838\ub358 \uc560\uad6d\uac00\uc694 \ub300\ud68c\uc5d0\uc11c \ub97c \ubd80\ub974\uace0 \uc0c1\uc744 \ud0d4\ub2e4. \ud559\ucc3d \uc2dc\uc808\uc5d0\ub294 \uc6b4\ub3d9 \uc120\uc218\ub85c\ub3c4 \ud65c\ub3d9\ud588\uc5c8\ub294\ub370, \uadf8 \uc911\uc5d0\uc11c\ub3c4 \ub124\ud2b8\ubcfc(netball)\uc744 \ud588\ub2e4. \uc774\ub97c \uacc4\uae30\ub85c 2006\ub144 \ucfe0\uc548\ud0c4\uc5d0\uc11c \uc5f4\ub9b0 \ud53c\uc5d0\uc2a4\ud0c0 \uba54\ub514\uc544 \uc774\ub3cc\ub77c 2006(Fiesta Media Idola 2006)\uc5d0 \ucc38\uac00\ud558\uae30\ub3c4 \ud588\ub2e4.", "sentence_answer": "\ud6d7\ub0a0 \uc2dc\ud2f0\ub294 \ud074\ub9ac\ud3ec\ub4dc \ucd08\ub4f1\ud559\uad50 \uc5d0\uc11c \ucd08\ub4f1 \uad50\uc721\uc744, \ud30c\ud56d \uc8fc \ucfe0\uc54c\ub77c \ub9ac\ud53c\uc2a4\uc758 \ud074\ub9ac\ud3ec\ub4dc \uc911\ud559\uad50\uc5d0\uc11c \uc911\ub4f1 \uad50\uc721\uc744 \ubc1b\uc558\ub2e4.", "paragraph_id": "6519565-0-1", "paragraph_question": "question: \uc2dc\ud2f0\uac00 \uc878\uc5c5\ud55c \ucd08\ub4f1\ud559\uad50\ub294?, context: \uc5b4\ub9b4 \uc801 \uc2dc\ud2f0\ub294 \uccb4\uc721\ud65c\ub3d9 \ubc0f \ud2b9\ubcc4\ud65c\ub3d9(co-curricular) \uac19\uc740 \ud559\uad50 \ud65c\ub3d9\uc744 \ud558\uace0 \uc788\uc5c8\ub2e4. \ubca0\ub809 \ud3f4\ub9ac\uc2a4\uc5d0 \uc704\uce58\ud55c \ud0c0\ub514\uce74 \ud37c\ub974\ucef5(Tadika Perkep)\uc5d0\uc11c \uccab \ud559\uad50 \ud65c\ub3d9\uc744 \ud588\ub294\ub370, \uc774 \ud559\uad50\ub294 \uc2dc\ud2f0\uac00 \ucc98\uc74c\uc73c\ub85c \ub178\ub798 \uc2e4\ub825\uc744 \uc120\ubcf4\uc600\ub358 \ud559\uad50\uc600\ub2e4(\ub2f9\uc2dc \uc2dc\ud2f0\ub294 \ub9d0\ub808\uc774 \uc804\ud1b5 \uac00\uc694\uc778 \uc744 \ubd88\ub800\ub2e4). \ud6d7\ub0a0 \uc2dc\ud2f0\ub294 \ud074\ub9ac\ud3ec\ub4dc \ucd08\ub4f1\ud559\uad50\uc5d0\uc11c \ucd08\ub4f1 \uad50\uc721\uc744, \ud30c\ud56d \uc8fc \ucfe0\uc54c\ub77c \ub9ac\ud53c\uc2a4\uc758 \ud074\ub9ac\ud3ec\ub4dc \uc911\ud559\uad50\uc5d0\uc11c \uc911\ub4f1 \uad50\uc721\uc744 \ubc1b\uc558\ub2e4. 12\uc0b4\uc774\ub358 1991\ub144 \ub3c5\ub9bd \uae30\ub150\uc744\uc744 \ub9de\uc544 \uc5f4\ub838\ub358 \uc560\uad6d\uac00\uc694 \ub300\ud68c\uc5d0\uc11c \ub97c \ubd80\ub974\uace0 \uc0c1\uc744 \ud0d4\ub2e4. \ud559\ucc3d \uc2dc\uc808\uc5d0\ub294 \uc6b4\ub3d9 \uc120\uc218\ub85c\ub3c4 \ud65c\ub3d9\ud588\uc5c8\ub294\ub370, \uadf8 \uc911\uc5d0\uc11c\ub3c4 \ub124\ud2b8\ubcfc(netball)\uc744 \ud588\ub2e4. \uc774\ub97c \uacc4\uae30\ub85c 2006\ub144 \ucfe0\uc548\ud0c4\uc5d0\uc11c \uc5f4\ub9b0 \ud53c\uc5d0\uc2a4\ud0c0 \uba54\ub514\uc544 \uc774\ub3cc\ub77c 2006(Fiesta Media Idola 2006)\uc5d0 \ucc38\uac00\ud558\uae30\ub3c4 \ud588\ub2e4."} {"question": "2M3T \ud3b8\uc131\uc774 \uc8fc\ud589 \ubd80\ubb38\uc5d0\uc11c 113\uacc4\uc640 115\uacc4\uc758 \ubb34\uc5c7\uacfc \uac19\uc740 \uc131\ub2a5\uc744 \uac00\uc9c0\uac8c \ub418\uc5c8\ub294\uac00?", "paragraph": "\uc9c1\ub958 \uadfc\uad50\ud615 \uc804\ub3d9\ucc28\ub294 1962\ub144\ubd80\ud130 20\ub144 \uc774\uc0c1\uc758 \uae34 \uc138\uc6d4\uc5d0 \uac78\uccd0 111\u00b7113\uacc4\uc640 115\uacc4\uac00 \uc81c\uc870\ub418\uc5b4 \uc654\ub2e4. \ubcf8\uacc4\uc5f4\uc740 \uc774\ub4e4\uc744 \ub300\uc2e0\ud55c \ud480 \ubaa8\ub378 \uccb4\uc778\uc9c0\uc774\uba70, \uacbd\ub7c9 \uc2a4\ud150\ub808\uc2a4\uc81c \ucc28\uccb4\uc640 \uac04\ud3b8\ud55c \uad6c\uc870\uc758 \ubcfc\uc2a4\ud130\ub9ac\uc2a4 \ub300\ucc28, \uc804\uae30\uc790 \ucd08\ud37c \uc81c\uc5b4\ubcf4\ub2e4 \uac04\ud3b8\ud558\uace0 \uc2fc \uac00\uaca9\uc73c\ub85c \ud68c\uc0dd \uc81c\ub3d9\uc774 \uc0ac\uc6a9 \uac00\ub2a5\ud55c \uc800\ud56d\uc81c\uc5b4\ub97c \uae30\ubcf8\uc73c\ub85c \ud55c \uacc4\uc790\ucca8\uac00 \uc5ec\uc790\uc81c\uc5b4, \uc751\ub2f5\uc131\uc774 \ub192\uc740 \uc804\uae30\uc9c0\ub839\uc2dd \uc81c\ub3d9\uc774\ub098 \uac04\uc774 \ubaa8\ub2c8\ud130 \uc7a5\uce58 \ub4f1 \uc5d0\ub108\uc9c0 \uc808\uc57d\uc774\ub098 \ubcf4\uc218\ube44 \uc800\uac10\uc744 \ubc18\uc601\ud55c \uc2e0\uae30\uc220\uc774 \uac01\uc9c0\uc5d0 \ucc44\uc6a9\ub418\uace0 \uc788\uc5c8\ub2e4. \uc774\ub4e4\uc740 \ud1b5\uadfc\ud615 \uc804\ub3d9\ucc28\uc778 205\uacc4\uc5d0\uc11c \uba3c\uc800 \ucc44\uc6a9\ub41c \uac83\uc774\uc9c0\ub9cc, \ubcf8\ub798\ub294 \uadfc\uad50\ud615 \uc804\ub3d9\ucc28\uc6a9\uc758 \uc2dc\uc2a4\ud15c\uc73c\ub85c \uac1c\ubc1c\ub418\uace0 \uc788\uc5c8\ub2e4. \uc720\ub2db\uc758 \uc5ed\ud589 \uc131\ub2a5 \ud5a5\uc0c1\uc5d0 \uc758\ud574 \uc804\ub3d9\ucc28\ube44\uc728\uc744 \ub0ae\ucd98 2M3T \ud3b8\uc131\uc73c\ub85c 25\u2030\uae4c\uc9c0\uc758 \ud1b5\uc0c1 \uc0ac\uc6a9\uc774 \uac00\ub2a5\ud55c \uc124\uacc4\ub85c \ud558\uc5ec \uc2e0\uc870 \ube44\uc6a9\uacfc \uc6b4\uc601 \ube44\uc6a9\uc758 \uac10\uc18c\ub97c \ub178\ub9b0 \uc124\uacc4\ub85c \ud558\uc600\ub2e4. \uc774\uac83\uc5d0 \uc758\ud574 2M3T \ud3b8\uc131\ub3c4 113\uacc4\u00b7115\uacc4\uc758 2M2T \ud3b8\uc131\uacfc \ub3d9\uc77c\ud55c \uc8fc\ud589 \uc131\ub2a5\uc744 \ud655\ubcf4\ud558\uac8c \ub418\uc5c8\ub2e4.", "answer": "2M2T \ud3b8\uc131", "sentence": "\uc774\uac83\uc5d0 \uc758\ud574 2M3T \ud3b8\uc131\ub3c4 113\uacc4\u00b7115\uacc4\uc758 2M2T \ud3b8\uc131 \uacfc \ub3d9\uc77c\ud55c \uc8fc\ud589 \uc131\ub2a5\uc744 \ud655\ubcf4\ud558\uac8c \ub418\uc5c8\ub2e4.", "paragraph_sentence": "\uc9c1\ub958 \uadfc\uad50\ud615 \uc804\ub3d9\ucc28\ub294 1962\ub144\ubd80\ud130 20\ub144 \uc774\uc0c1\uc758 \uae34 \uc138\uc6d4\uc5d0 \uac78\uccd0 111\u00b7113\uacc4\uc640 115\uacc4\uac00 \uc81c\uc870\ub418\uc5b4 \uc654\ub2e4. \ubcf8\uacc4\uc5f4\uc740 \uc774\ub4e4\uc744 \ub300\uc2e0\ud55c \ud480 \ubaa8\ub378 \uccb4\uc778\uc9c0\uc774\uba70, \uacbd\ub7c9 \uc2a4\ud150\ub808\uc2a4\uc81c \ucc28\uccb4\uc640 \uac04\ud3b8\ud55c \uad6c\uc870\uc758 \ubcfc\uc2a4\ud130\ub9ac\uc2a4 \ub300\ucc28, \uc804\uae30\uc790 \ucd08\ud37c \uc81c\uc5b4\ubcf4\ub2e4 \uac04\ud3b8\ud558\uace0 \uc2fc \uac00\uaca9\uc73c\ub85c \ud68c\uc0dd \uc81c\ub3d9\uc774 \uc0ac\uc6a9 \uac00\ub2a5\ud55c \uc800\ud56d\uc81c\uc5b4\ub97c \uae30\ubcf8\uc73c\ub85c \ud55c \uacc4\uc790\ucca8\uac00 \uc5ec\uc790\uc81c\uc5b4, \uc751\ub2f5\uc131\uc774 \ub192\uc740 \uc804\uae30\uc9c0\ub839\uc2dd \uc81c\ub3d9\uc774\ub098 \uac04\uc774 \ubaa8\ub2c8\ud130 \uc7a5\uce58 \ub4f1 \uc5d0\ub108\uc9c0 \uc808\uc57d\uc774\ub098 \ubcf4\uc218\ube44 \uc800\uac10\uc744 \ubc18\uc601\ud55c \uc2e0\uae30\uc220\uc774 \uac01\uc9c0\uc5d0 \ucc44\uc6a9\ub418\uace0 \uc788\uc5c8\ub2e4. \uc774\ub4e4\uc740 \ud1b5\uadfc\ud615 \uc804\ub3d9\ucc28\uc778 205\uacc4\uc5d0\uc11c \uba3c\uc800 \ucc44\uc6a9\ub41c \uac83\uc774\uc9c0\ub9cc, \ubcf8\ub798\ub294 \uadfc\uad50\ud615 \uc804\ub3d9\ucc28\uc6a9\uc758 \uc2dc\uc2a4\ud15c\uc73c\ub85c \uac1c\ubc1c\ub418\uace0 \uc788\uc5c8\ub2e4. \uc720\ub2db\uc758 \uc5ed\ud589 \uc131\ub2a5 \ud5a5\uc0c1\uc5d0 \uc758\ud574 \uc804\ub3d9\ucc28\ube44\uc728\uc744 \ub0ae\ucd98 2M3T \ud3b8\uc131\uc73c\ub85c 25\u2030\uae4c\uc9c0\uc758 \ud1b5\uc0c1 \uc0ac\uc6a9\uc774 \uac00\ub2a5\ud55c \uc124\uacc4\ub85c \ud558\uc5ec \uc2e0\uc870 \ube44\uc6a9\uacfc \uc6b4\uc601 \ube44\uc6a9\uc758 \uac10\uc18c\ub97c \ub178\ub9b0 \uc124\uacc4\ub85c \ud558\uc600\ub2e4. \uc774\uac83\uc5d0 \uc758\ud574 2M3T \ud3b8\uc131\ub3c4 113\uacc4\u00b7115\uacc4\uc758 2M2T \ud3b8\uc131 \uacfc \ub3d9\uc77c\ud55c \uc8fc\ud589 \uc131\ub2a5\uc744 \ud655\ubcf4\ud558\uac8c \ub418\uc5c8\ub2e4. ", "paragraph_answer": "\uc9c1\ub958 \uadfc\uad50\ud615 \uc804\ub3d9\ucc28\ub294 1962\ub144\ubd80\ud130 20\ub144 \uc774\uc0c1\uc758 \uae34 \uc138\uc6d4\uc5d0 \uac78\uccd0 111\u00b7113\uacc4\uc640 115\uacc4\uac00 \uc81c\uc870\ub418\uc5b4 \uc654\ub2e4. \ubcf8\uacc4\uc5f4\uc740 \uc774\ub4e4\uc744 \ub300\uc2e0\ud55c \ud480 \ubaa8\ub378 \uccb4\uc778\uc9c0\uc774\uba70, \uacbd\ub7c9 \uc2a4\ud150\ub808\uc2a4\uc81c \ucc28\uccb4\uc640 \uac04\ud3b8\ud55c \uad6c\uc870\uc758 \ubcfc\uc2a4\ud130\ub9ac\uc2a4 \ub300\ucc28, \uc804\uae30\uc790 \ucd08\ud37c \uc81c\uc5b4\ubcf4\ub2e4 \uac04\ud3b8\ud558\uace0 \uc2fc \uac00\uaca9\uc73c\ub85c \ud68c\uc0dd \uc81c\ub3d9\uc774 \uc0ac\uc6a9 \uac00\ub2a5\ud55c \uc800\ud56d\uc81c\uc5b4\ub97c \uae30\ubcf8\uc73c\ub85c \ud55c \uacc4\uc790\ucca8\uac00 \uc5ec\uc790\uc81c\uc5b4, \uc751\ub2f5\uc131\uc774 \ub192\uc740 \uc804\uae30\uc9c0\ub839\uc2dd \uc81c\ub3d9\uc774\ub098 \uac04\uc774 \ubaa8\ub2c8\ud130 \uc7a5\uce58 \ub4f1 \uc5d0\ub108\uc9c0 \uc808\uc57d\uc774\ub098 \ubcf4\uc218\ube44 \uc800\uac10\uc744 \ubc18\uc601\ud55c \uc2e0\uae30\uc220\uc774 \uac01\uc9c0\uc5d0 \ucc44\uc6a9\ub418\uace0 \uc788\uc5c8\ub2e4. \uc774\ub4e4\uc740 \ud1b5\uadfc\ud615 \uc804\ub3d9\ucc28\uc778 205\uacc4\uc5d0\uc11c \uba3c\uc800 \ucc44\uc6a9\ub41c \uac83\uc774\uc9c0\ub9cc, \ubcf8\ub798\ub294 \uadfc\uad50\ud615 \uc804\ub3d9\ucc28\uc6a9\uc758 \uc2dc\uc2a4\ud15c\uc73c\ub85c \uac1c\ubc1c\ub418\uace0 \uc788\uc5c8\ub2e4. \uc720\ub2db\uc758 \uc5ed\ud589 \uc131\ub2a5 \ud5a5\uc0c1\uc5d0 \uc758\ud574 \uc804\ub3d9\ucc28\ube44\uc728\uc744 \ub0ae\ucd98 2M3T \ud3b8\uc131\uc73c\ub85c 25\u2030\uae4c\uc9c0\uc758 \ud1b5\uc0c1 \uc0ac\uc6a9\uc774 \uac00\ub2a5\ud55c \uc124\uacc4\ub85c \ud558\uc5ec \uc2e0\uc870 \ube44\uc6a9\uacfc \uc6b4\uc601 \ube44\uc6a9\uc758 \uac10\uc18c\ub97c \ub178\ub9b0 \uc124\uacc4\ub85c \ud558\uc600\ub2e4. \uc774\uac83\uc5d0 \uc758\ud574 2M3T \ud3b8\uc131\ub3c4 113\uacc4\u00b7115\uacc4\uc758 2M2T \ud3b8\uc131 \uacfc \ub3d9\uc77c\ud55c \uc8fc\ud589 \uc131\ub2a5\uc744 \ud655\ubcf4\ud558\uac8c \ub418\uc5c8\ub2e4.", "sentence_answer": "\uc774\uac83\uc5d0 \uc758\ud574 2M3T \ud3b8\uc131\ub3c4 113\uacc4\u00b7115\uacc4\uc758 2M2T \ud3b8\uc131 \uacfc \ub3d9\uc77c\ud55c \uc8fc\ud589 \uc131\ub2a5\uc744 \ud655\ubcf4\ud558\uac8c \ub418\uc5c8\ub2e4.", "paragraph_id": "6389209-0-2", "paragraph_question": "question: 2M3T \ud3b8\uc131\uc774 \uc8fc\ud589 \ubd80\ubb38\uc5d0\uc11c 113\uacc4\uc640 115\uacc4\uc758 \ubb34\uc5c7\uacfc \uac19\uc740 \uc131\ub2a5\uc744 \uac00\uc9c0\uac8c \ub418\uc5c8\ub294\uac00?, context: \uc9c1\ub958 \uadfc\uad50\ud615 \uc804\ub3d9\ucc28\ub294 1962\ub144\ubd80\ud130 20\ub144 \uc774\uc0c1\uc758 \uae34 \uc138\uc6d4\uc5d0 \uac78\uccd0 111\u00b7113\uacc4\uc640 115\uacc4\uac00 \uc81c\uc870\ub418\uc5b4 \uc654\ub2e4. \ubcf8\uacc4\uc5f4\uc740 \uc774\ub4e4\uc744 \ub300\uc2e0\ud55c \ud480 \ubaa8\ub378 \uccb4\uc778\uc9c0\uc774\uba70, \uacbd\ub7c9 \uc2a4\ud150\ub808\uc2a4\uc81c \ucc28\uccb4\uc640 \uac04\ud3b8\ud55c \uad6c\uc870\uc758 \ubcfc\uc2a4\ud130\ub9ac\uc2a4 \ub300\ucc28, \uc804\uae30\uc790 \ucd08\ud37c \uc81c\uc5b4\ubcf4\ub2e4 \uac04\ud3b8\ud558\uace0 \uc2fc \uac00\uaca9\uc73c\ub85c \ud68c\uc0dd \uc81c\ub3d9\uc774 \uc0ac\uc6a9 \uac00\ub2a5\ud55c \uc800\ud56d\uc81c\uc5b4\ub97c \uae30\ubcf8\uc73c\ub85c \ud55c \uacc4\uc790\ucca8\uac00 \uc5ec\uc790\uc81c\uc5b4, \uc751\ub2f5\uc131\uc774 \ub192\uc740 \uc804\uae30\uc9c0\ub839\uc2dd \uc81c\ub3d9\uc774\ub098 \uac04\uc774 \ubaa8\ub2c8\ud130 \uc7a5\uce58 \ub4f1 \uc5d0\ub108\uc9c0 \uc808\uc57d\uc774\ub098 \ubcf4\uc218\ube44 \uc800\uac10\uc744 \ubc18\uc601\ud55c \uc2e0\uae30\uc220\uc774 \uac01\uc9c0\uc5d0 \ucc44\uc6a9\ub418\uace0 \uc788\uc5c8\ub2e4. \uc774\ub4e4\uc740 \ud1b5\uadfc\ud615 \uc804\ub3d9\ucc28\uc778 205\uacc4\uc5d0\uc11c \uba3c\uc800 \ucc44\uc6a9\ub41c \uac83\uc774\uc9c0\ub9cc, \ubcf8\ub798\ub294 \uadfc\uad50\ud615 \uc804\ub3d9\ucc28\uc6a9\uc758 \uc2dc\uc2a4\ud15c\uc73c\ub85c \uac1c\ubc1c\ub418\uace0 \uc788\uc5c8\ub2e4. \uc720\ub2db\uc758 \uc5ed\ud589 \uc131\ub2a5 \ud5a5\uc0c1\uc5d0 \uc758\ud574 \uc804\ub3d9\ucc28\ube44\uc728\uc744 \ub0ae\ucd98 2M3T \ud3b8\uc131\uc73c\ub85c 25\u2030\uae4c\uc9c0\uc758 \ud1b5\uc0c1 \uc0ac\uc6a9\uc774 \uac00\ub2a5\ud55c \uc124\uacc4\ub85c \ud558\uc5ec \uc2e0\uc870 \ube44\uc6a9\uacfc \uc6b4\uc601 \ube44\uc6a9\uc758 \uac10\uc18c\ub97c \ub178\ub9b0 \uc124\uacc4\ub85c \ud558\uc600\ub2e4. \uc774\uac83\uc5d0 \uc758\ud574 2M3T \ud3b8\uc131\ub3c4 113\uacc4\u00b7115\uacc4\uc758 2M2T \ud3b8\uc131\uacfc \ub3d9\uc77c\ud55c \uc8fc\ud589 \uc131\ub2a5\uc744 \ud655\ubcf4\ud558\uac8c \ub418\uc5c8\ub2e4."} {"question": "\ub2c8\ucf54\ud3f4\ub9ac\uc2a4 \uc804\ud22c\uc758 \uc2b9\ub9ac\ub85c \ubc14\uc608\uc9c0\ub4dc 1\uc138\uac00 \uce7c\ub9ac\ud30c\ub85c\ubd80\ud130 \ubc1b\uc740 \uce6d\ud638\ub294?", "paragraph": "\ub2c8\ucf54\ud3f4\ub9ac\uc2a4 \uc804\ud22c(\ubd88\uac00\ub9ac\uc544\uc5b4: \u0411\u0438\u0442\u043a\u0430 \u043f\u0440\u0438 \u041d\u0438\u043a\u043e\u043f\u043e\u043b, Bitka pri Nikopol;\ud130\ud0a4\uc5b4: Ni\u011fbolu Sava\u015f\u0131;\ud5dd\uac00\ub9ac\uc5b4: Nik\u00e1polyi Csata;\ub8e8\ub9c8\ub2c8\uc544\uc5b4: B\u0103t\u0103lia de la Nicopole)\ub294 1396\ub144 9\uc6d4 25\uc77c(\ud639\uc740 9\uc6d4 28\uc77c\uc774\ub780 \ub9d0\ub3c4 \uc788\ub2e4)\uc5d0 \ub3c4\ub098\uc6b0\uac15\ubcc0\uc758 \ub2c8\ucf54\ud3f4\ub9ac\uc2a4\uc5d0\uc11c \uc624\uc2a4\ub9cc \uc81c\uad6d\uc758 \ubc14\uc608\uc9c0\ub4dc 1\uc138(\uc81c\uc704:1389\ub144~1402\ub144)\uc640 \ud5dd\uac00\ub9ac \uc655 \uc9c0\uae30\uc2a4\ubb38\ud2b8\uac00 \uc774\ub044\ub294 \uc720\ub7fd\uc5f0\ud569(\ucc38\uac00\uc138\ub825:\ud5dd\uac00\ub9ac \uc655\uad6d, \uc2e0\uc131\ub85c\ub9c8\uc81c\uad6d, \ud504\ub791\uc2a4, \uc648\ub77c\ud0a4\uc544, \ud3f4\ub780\ub4dc, \uc789\uae00\ub79c\ub4dc, \uc2a4\ucf54\ud2c0\ub79c\ub4dc \uc655\uad6d, \uad6c \uc2a4\uc704\uc2a4 \uc5f0\ud569, \ud29c\ud2bc \uae30\uc0ac\ub2e8, \ubca0\ub124\uce58\uc544 \uacf5\ud654\uad6d, \uc81c\ub178\ubc14 \uacf5\ud654\uad6d, \uc131 \uc694\ud55c \uae30\uc0ac\ub2e8)\uc0ac\uc774\uc5d0\uc11c \uc77c\uc5b4\ub09c \uc804\ud22c\uc774\ub2e4. \ud754\ud788 \ub2c8\ucf54\ud3f4\ub9ac\uc2a4 \uc2ed\uc790\uad70\uc774\ub77c\uace0 \ubd88\ub9ac\uba70, \uc911\uc138 \ucd5c\ud6c4\uc758 \ub300\uaddc\ubaa8 \uc2ed\uc790\uad70\uc774\uc5c8\ub2e4. \uc804\ud22c \uacb0\uacfc\ub294 \uc624\uc2a4\ub9cc \uc81c\uad6d\uc758 \uacb0\uc815\uc801 \uc2b9\ub9ac\ub85c \ub05d\ub0ac\uace0, \ubc14\uc608\uc9c0\ub4dc 1\uc138\ub294 \uce74\uc774\ub85c\uc758 \ub9d8\ub8e8\ud06c \uc655\uc870 \ubcf4\ud638\ud558\uc5d0 \uc788\ub358 \uce7c\ub9ac\ud30c\ub85c\ubd80\ud130 \ub192\uc740 \ud3c9\uac00\ub97c \ubc1b\uc544 \uc220\ud0c4\uc758 \uce6d\ud638\ub97c \ud558\uc0ac\ubc1b\uc558\ub2e4.", "answer": "\uc220\ud0c4", "sentence": "\uc804\ud22c \uacb0\uacfc\ub294 \uc624\uc2a4\ub9cc \uc81c\uad6d\uc758 \uacb0\uc815\uc801 \uc2b9\ub9ac\ub85c \ub05d\ub0ac\uace0, \ubc14\uc608\uc9c0\ub4dc 1\uc138\ub294 \uce74\uc774\ub85c\uc758 \ub9d8\ub8e8\ud06c \uc655\uc870 \ubcf4\ud638\ud558\uc5d0 \uc788\ub358 \uce7c\ub9ac\ud30c\ub85c\ubd80\ud130 \ub192\uc740 \ud3c9\uac00\ub97c \ubc1b\uc544 \uc220\ud0c4 \uc758", "paragraph_sentence": "\ub2c8\ucf54\ud3f4\ub9ac\uc2a4 \uc804\ud22c(\ubd88\uac00\ub9ac\uc544\uc5b4: \u0411\u0438\u0442\u043a\u0430 \u043f\u0440\u0438 \u041d\u0438\u043a\u043e\u043f\u043e\u043b, Bitka pri Nikopol;\ud130\ud0a4\uc5b4: Ni\u011fbolu Sava\u015f\u0131;\ud5dd\uac00\ub9ac\uc5b4: Nik\u00e1polyi Csata;\ub8e8\ub9c8\ub2c8\uc544\uc5b4: B\u0103t\u0103lia de la Nicopole)\ub294 1396\ub144 9\uc6d4 25\uc77c(\ud639\uc740 9\uc6d4 28\uc77c\uc774\ub780 \ub9d0\ub3c4 \uc788\ub2e4)\uc5d0 \ub3c4\ub098\uc6b0\uac15\ubcc0\uc758 \ub2c8\ucf54\ud3f4\ub9ac\uc2a4\uc5d0\uc11c \uc624\uc2a4\ub9cc \uc81c\uad6d\uc758 \ubc14\uc608\uc9c0\ub4dc 1\uc138(\uc81c\uc704:1389\ub144~1402\ub144)\uc640 \ud5dd\uac00\ub9ac \uc655 \uc9c0\uae30\uc2a4\ubb38\ud2b8\uac00 \uc774\ub044\ub294 \uc720\ub7fd\uc5f0\ud569(\ucc38\uac00\uc138\ub825:\ud5dd\uac00\ub9ac \uc655\uad6d, \uc2e0\uc131\ub85c\ub9c8\uc81c\uad6d, \ud504\ub791\uc2a4, \uc648\ub77c\ud0a4\uc544, \ud3f4\ub780\ub4dc, \uc789\uae00\ub79c\ub4dc, \uc2a4\ucf54\ud2c0\ub79c\ub4dc \uc655\uad6d, \uad6c \uc2a4\uc704\uc2a4 \uc5f0\ud569, \ud29c\ud2bc \uae30\uc0ac\ub2e8, \ubca0\ub124\uce58\uc544 \uacf5\ud654\uad6d, \uc81c\ub178\ubc14 \uacf5\ud654\uad6d, \uc131 \uc694\ud55c \uae30\uc0ac\ub2e8)\uc0ac\uc774\uc5d0\uc11c \uc77c\uc5b4\ub09c \uc804\ud22c\uc774\ub2e4. \ud754\ud788 \ub2c8\ucf54\ud3f4\ub9ac\uc2a4 \uc2ed\uc790\uad70\uc774\ub77c\uace0 \ubd88\ub9ac\uba70, \uc911\uc138 \ucd5c\ud6c4\uc758 \ub300\uaddc\ubaa8 \uc2ed\uc790\uad70\uc774\uc5c8\ub2e4. \uc804\ud22c \uacb0\uacfc\ub294 \uc624\uc2a4\ub9cc \uc81c\uad6d\uc758 \uacb0\uc815\uc801 \uc2b9\ub9ac\ub85c \ub05d\ub0ac\uace0, \ubc14\uc608\uc9c0\ub4dc 1\uc138\ub294 \uce74\uc774\ub85c\uc758 \ub9d8\ub8e8\ud06c \uc655\uc870 \ubcf4\ud638\ud558\uc5d0 \uc788\ub358 \uce7c\ub9ac\ud30c\ub85c\ubd80\ud130 \ub192\uc740 \ud3c9\uac00\ub97c \ubc1b\uc544 \uc220\ud0c4 \uc758 \uce6d\ud638\ub97c \ud558\uc0ac\ubc1b\uc558\ub2e4.", "paragraph_answer": "\ub2c8\ucf54\ud3f4\ub9ac\uc2a4 \uc804\ud22c(\ubd88\uac00\ub9ac\uc544\uc5b4: \u0411\u0438\u0442\u043a\u0430 \u043f\u0440\u0438 \u041d\u0438\u043a\u043e\u043f\u043e\u043b, Bitka pri Nikopol;\ud130\ud0a4\uc5b4: Ni\u011fbolu Sava\u015f\u0131;\ud5dd\uac00\ub9ac\uc5b4: Nik\u00e1polyi Csata;\ub8e8\ub9c8\ub2c8\uc544\uc5b4: B\u0103t\u0103lia de la Nicopole)\ub294 1396\ub144 9\uc6d4 25\uc77c(\ud639\uc740 9\uc6d4 28\uc77c\uc774\ub780 \ub9d0\ub3c4 \uc788\ub2e4)\uc5d0 \ub3c4\ub098\uc6b0\uac15\ubcc0\uc758 \ub2c8\ucf54\ud3f4\ub9ac\uc2a4\uc5d0\uc11c \uc624\uc2a4\ub9cc \uc81c\uad6d\uc758 \ubc14\uc608\uc9c0\ub4dc 1\uc138(\uc81c\uc704:1389\ub144~1402\ub144)\uc640 \ud5dd\uac00\ub9ac \uc655 \uc9c0\uae30\uc2a4\ubb38\ud2b8\uac00 \uc774\ub044\ub294 \uc720\ub7fd\uc5f0\ud569(\ucc38\uac00\uc138\ub825:\ud5dd\uac00\ub9ac \uc655\uad6d, \uc2e0\uc131\ub85c\ub9c8\uc81c\uad6d, \ud504\ub791\uc2a4, \uc648\ub77c\ud0a4\uc544, \ud3f4\ub780\ub4dc, \uc789\uae00\ub79c\ub4dc, \uc2a4\ucf54\ud2c0\ub79c\ub4dc \uc655\uad6d, \uad6c \uc2a4\uc704\uc2a4 \uc5f0\ud569, \ud29c\ud2bc \uae30\uc0ac\ub2e8, \ubca0\ub124\uce58\uc544 \uacf5\ud654\uad6d, \uc81c\ub178\ubc14 \uacf5\ud654\uad6d, \uc131 \uc694\ud55c \uae30\uc0ac\ub2e8)\uc0ac\uc774\uc5d0\uc11c \uc77c\uc5b4\ub09c \uc804\ud22c\uc774\ub2e4. \ud754\ud788 \ub2c8\ucf54\ud3f4\ub9ac\uc2a4 \uc2ed\uc790\uad70\uc774\ub77c\uace0 \ubd88\ub9ac\uba70, \uc911\uc138 \ucd5c\ud6c4\uc758 \ub300\uaddc\ubaa8 \uc2ed\uc790\uad70\uc774\uc5c8\ub2e4. \uc804\ud22c \uacb0\uacfc\ub294 \uc624\uc2a4\ub9cc \uc81c\uad6d\uc758 \uacb0\uc815\uc801 \uc2b9\ub9ac\ub85c \ub05d\ub0ac\uace0, \ubc14\uc608\uc9c0\ub4dc 1\uc138\ub294 \uce74\uc774\ub85c\uc758 \ub9d8\ub8e8\ud06c \uc655\uc870 \ubcf4\ud638\ud558\uc5d0 \uc788\ub358 \uce7c\ub9ac\ud30c\ub85c\ubd80\ud130 \ub192\uc740 \ud3c9\uac00\ub97c \ubc1b\uc544 \uc220\ud0c4 \uc758 \uce6d\ud638\ub97c \ud558\uc0ac\ubc1b\uc558\ub2e4.", "sentence_answer": "\uc804\ud22c \uacb0\uacfc\ub294 \uc624\uc2a4\ub9cc \uc81c\uad6d\uc758 \uacb0\uc815\uc801 \uc2b9\ub9ac\ub85c \ub05d\ub0ac\uace0, \ubc14\uc608\uc9c0\ub4dc 1\uc138\ub294 \uce74\uc774\ub85c\uc758 \ub9d8\ub8e8\ud06c \uc655\uc870 \ubcf4\ud638\ud558\uc5d0 \uc788\ub358 \uce7c\ub9ac\ud30c\ub85c\ubd80\ud130 \ub192\uc740 \ud3c9\uac00\ub97c \ubc1b\uc544 \uc220\ud0c4 \uc758", "paragraph_id": "6484822-0-2", "paragraph_question": "question: \ub2c8\ucf54\ud3f4\ub9ac\uc2a4 \uc804\ud22c\uc758 \uc2b9\ub9ac\ub85c \ubc14\uc608\uc9c0\ub4dc 1\uc138\uac00 \uce7c\ub9ac\ud30c\ub85c\ubd80\ud130 \ubc1b\uc740 \uce6d\ud638\ub294?, context: \ub2c8\ucf54\ud3f4\ub9ac\uc2a4 \uc804\ud22c(\ubd88\uac00\ub9ac\uc544\uc5b4: \u0411\u0438\u0442\u043a\u0430 \u043f\u0440\u0438 \u041d\u0438\u043a\u043e\u043f\u043e\u043b, Bitka pri Nikopol;\ud130\ud0a4\uc5b4: Ni\u011fbolu Sava\u015f\u0131;\ud5dd\uac00\ub9ac\uc5b4: Nik\u00e1polyi Csata;\ub8e8\ub9c8\ub2c8\uc544\uc5b4: B\u0103t\u0103lia de la Nicopole)\ub294 1396\ub144 9\uc6d4 25\uc77c(\ud639\uc740 9\uc6d4 28\uc77c\uc774\ub780 \ub9d0\ub3c4 \uc788\ub2e4)\uc5d0 \ub3c4\ub098\uc6b0\uac15\ubcc0\uc758 \ub2c8\ucf54\ud3f4\ub9ac\uc2a4\uc5d0\uc11c \uc624\uc2a4\ub9cc \uc81c\uad6d\uc758 \ubc14\uc608\uc9c0\ub4dc 1\uc138(\uc81c\uc704:1389\ub144~1402\ub144)\uc640 \ud5dd\uac00\ub9ac \uc655 \uc9c0\uae30\uc2a4\ubb38\ud2b8\uac00 \uc774\ub044\ub294 \uc720\ub7fd\uc5f0\ud569(\ucc38\uac00\uc138\ub825:\ud5dd\uac00\ub9ac \uc655\uad6d, \uc2e0\uc131\ub85c\ub9c8\uc81c\uad6d, \ud504\ub791\uc2a4, \uc648\ub77c\ud0a4\uc544, \ud3f4\ub780\ub4dc, \uc789\uae00\ub79c\ub4dc, \uc2a4\ucf54\ud2c0\ub79c\ub4dc \uc655\uad6d, \uad6c \uc2a4\uc704\uc2a4 \uc5f0\ud569, \ud29c\ud2bc \uae30\uc0ac\ub2e8, \ubca0\ub124\uce58\uc544 \uacf5\ud654\uad6d, \uc81c\ub178\ubc14 \uacf5\ud654\uad6d, \uc131 \uc694\ud55c \uae30\uc0ac\ub2e8)\uc0ac\uc774\uc5d0\uc11c \uc77c\uc5b4\ub09c \uc804\ud22c\uc774\ub2e4. \ud754\ud788 \ub2c8\ucf54\ud3f4\ub9ac\uc2a4 \uc2ed\uc790\uad70\uc774\ub77c\uace0 \ubd88\ub9ac\uba70, \uc911\uc138 \ucd5c\ud6c4\uc758 \ub300\uaddc\ubaa8 \uc2ed\uc790\uad70\uc774\uc5c8\ub2e4. \uc804\ud22c \uacb0\uacfc\ub294 \uc624\uc2a4\ub9cc \uc81c\uad6d\uc758 \uacb0\uc815\uc801 \uc2b9\ub9ac\ub85c \ub05d\ub0ac\uace0, \ubc14\uc608\uc9c0\ub4dc 1\uc138\ub294 \uce74\uc774\ub85c\uc758 \ub9d8\ub8e8\ud06c \uc655\uc870 \ubcf4\ud638\ud558\uc5d0 \uc788\ub358 \uce7c\ub9ac\ud30c\ub85c\ubd80\ud130 \ub192\uc740 \ud3c9\uac00\ub97c \ubc1b\uc544 \uc220\ud0c4\uc758 \uce6d\ud638\ub97c \ud558\uc0ac\ubc1b\uc558\ub2e4."} {"question": "\uc720\uae38\uc900\uc740 \uc720\ud55c\uc900\uc758 \uc544\ub4e4 \uc720\ud68c\uc8fc\uc758 \uba87\ub300\uc190\uc778\uac00?", "paragraph": "1871\ub144 \ud64d\ubb38\uad00 \ub300\uc81c\ud559 \ubc15\uaddc\uc218\ub294 \ud5a5\uc2dc\uc5d0\uc11c \uc7a5\uc6d0\uc73c\ub85c \ubf51\ud78c \uc2dc\ub97c \ubcf4\uace0, \uadf8 \uc2dc\uc758 \uc8fc\uc778\uacf5\uc744 \ubd88\ub7ec\ub4e4\uc600\ub2e4. \uc2dc\ub97c \uc9c0\uc740 \uc774\ub294 16\uc138\uc758 \uc720\uae38\uc900\uc774\uc5c8\ub2e4. \ubc15\uaddc\uc218\ub294 \uc5f0\uc554 \ubc15\uc9c0\uc6d0\uc758 \uc544\ub4e4 \ubc15\uc885\ucc44\uc758 \uc544\ub4e4\uc774\uba70, \uc720\uae38\uc900\uc740 \uc720\ud55c\uc900\uc758 \uc544\ub4e4 \uc720\ud68c\uc8fc\uc758 4\ub300\uc190\uc774\uc5c8\ub2e4. \uc720\uae38\uc900\uc774 \ucc98\uc74c\uc73c\ub85c \ubc15\uaddc\uc218\ub97c \ub9cc\ub098\ub7ec \uac08 \ub54c \uc720\uae38\uc900\uc758 \uc544\ubc84\uc9c0 \uc720\uc9c4\uc218\ub294 '\uc6b0\ub9ac\uc9d1\uacfc \uc11c\ub85c \uc6d0\uc218\uac19\uc774 \uc9c0\ub0b4\uc654\ub294\ub370 \uc5b4\ub5bb\uac8c \uadf8 \uc790\ub97c \ucc3e\uc544\uac04\ub2e4\ub294 \ub9d0\uc774\ub0d0'\uba70 \uc644\uac15\ud788 \ubc18\ub300\ud588\ub2e4. \uadf8\ub7ec\ub098 \uc6b0\uc5ec\uace1\uc808 \ub05d\uc5d0 \uc720\uae38\uc900\uc744 \ub9cc\ub09c \ubc15\uaddc\uc218\ub294 '\ub108\ud76c \uc9d1\uacfc \uc6b0\ub9ac \uc9d1\uc774 \uc9c0\ub09c\ub0a0 \uc0ac\uc18c\ud55c \ubb38\uc81c\ub85c \ubd88\ud654\ud588\uc73c\ub098 \uc774\uc81c\ubd80\ud130 \uc61b\ub0a0\ucc98\ub7fc \ub2e4\uc2dc \ud654\ubaa9\ud558\uac8c \uc9c0\ub0bc \uc218 \uc788\ub2e4\uba74 \uc5b4\ub978\ub4e4\uc774 \ud480\uc9c0 \ubabb\ud558\uc168\ub358 \uac10\uc815\uc744 \uc6b0\ub9ac\uac00 \ud480\uc5b4\ub4dc\ub9ac\ub294 \uc148\uc774 \ub418\ub294\uac8c \uc544\ub2c8\uaca0\ub0d0'\uba70 \uac10\uac1c\ubb34\ub7c9\ud574\ud558\uc600\ub2e4. \ubc15\uaddc\uc218\ub294 \uc544\ubc84\uc9c0\uac00 \uadf8\ud1a0\ub85d \uac15\uc870\ud588\ub358 \ubc31\uc138\uc758 \uc6d0\uc218\uc5d0 \ub300\ud55c \uc0dd\uac01\uc740 \uc78a\uace0 \uba3c\uc800 \uc190\uc744 \ub0b4\ubc00\uc5c8\uace0, \uc624\ud788\ub824 \uc9d1\uc548\uac04\uc758 \ubd88\ud654\ub97c \uc78a\uc790\uba70 \uc720\uae38\uc900\uc758 \ub6f0\uc5b4\ub09c \uc7ac\uc8fc\ub97c \uac70\ub4ed \uce6d\ucc2c\ud558\uc600\ub2e4. \ub610\ud55c \ud798\uc368 \uacf5\ubd80\ud560 \uac83\uc744 \ub2f9\ubd80\ud558\uba70 \uc790\uc8fc \ucc3e\uc544\uc62c \uac83\uc744 \uad8c\uace0\ud588\ub2e4. \ubc15\uaddc\uc218\uc758 \uc778\ud488\uc5d0 \uac10\ubcf5\ud55c \uc720\uae38\uc900\uc740 \uc624\ud788\ub824 \ubc15\uaddc\uc218\ub97c \uc2a4\uc2b9\uc73c\ub85c \ubc1b\ub4e4\uace0 \uadf8\ub85c\ubd80\ud130 \ud559\ubb38\uc744 \uc0ac\uc0ac\ubc1b\uc558\ub2e4.", "answer": "4\ub300\uc190", "sentence": "\ubc15\uaddc\uc218\ub294 \uc5f0\uc554 \ubc15\uc9c0\uc6d0\uc758 \uc544\ub4e4 \ubc15\uc885\ucc44\uc758 \uc544\ub4e4\uc774\uba70, \uc720\uae38\uc900\uc740 \uc720\ud55c\uc900\uc758 \uc544\ub4e4 \uc720\ud68c\uc8fc\uc758 4\ub300\uc190 \uc774\uc5c8\ub2e4.", "paragraph_sentence": "1871\ub144 \ud64d\ubb38\uad00 \ub300\uc81c\ud559 \ubc15\uaddc\uc218\ub294 \ud5a5\uc2dc\uc5d0\uc11c \uc7a5\uc6d0\uc73c\ub85c \ubf51\ud78c \uc2dc\ub97c \ubcf4\uace0, \uadf8 \uc2dc\uc758 \uc8fc\uc778\uacf5\uc744 \ubd88\ub7ec\ub4e4\uc600\ub2e4. \uc2dc\ub97c \uc9c0\uc740 \uc774\ub294 16\uc138\uc758 \uc720\uae38\uc900\uc774\uc5c8\ub2e4. \ubc15\uaddc\uc218\ub294 \uc5f0\uc554 \ubc15\uc9c0\uc6d0\uc758 \uc544\ub4e4 \ubc15\uc885\ucc44\uc758 \uc544\ub4e4\uc774\uba70, \uc720\uae38\uc900\uc740 \uc720\ud55c\uc900\uc758 \uc544\ub4e4 \uc720\ud68c\uc8fc\uc758 4\ub300\uc190 \uc774\uc5c8\ub2e4. \uc720\uae38\uc900\uc774 \ucc98\uc74c\uc73c\ub85c \ubc15\uaddc\uc218\ub97c \ub9cc\ub098\ub7ec \uac08 \ub54c \uc720\uae38\uc900\uc758 \uc544\ubc84\uc9c0 \uc720\uc9c4\uc218\ub294 '\uc6b0\ub9ac\uc9d1\uacfc \uc11c\ub85c \uc6d0\uc218\uac19\uc774 \uc9c0\ub0b4\uc654\ub294\ub370 \uc5b4\ub5bb\uac8c \uadf8 \uc790\ub97c \ucc3e\uc544\uac04\ub2e4\ub294 \ub9d0\uc774\ub0d0'\uba70 \uc644\uac15\ud788 \ubc18\ub300\ud588\ub2e4. \uadf8\ub7ec\ub098 \uc6b0\uc5ec\uace1\uc808 \ub05d\uc5d0 \uc720\uae38\uc900\uc744 \ub9cc\ub09c \ubc15\uaddc\uc218\ub294 '\ub108\ud76c \uc9d1\uacfc \uc6b0\ub9ac \uc9d1\uc774 \uc9c0\ub09c\ub0a0 \uc0ac\uc18c\ud55c \ubb38\uc81c\ub85c \ubd88\ud654\ud588\uc73c\ub098 \uc774\uc81c\ubd80\ud130 \uc61b\ub0a0\ucc98\ub7fc \ub2e4\uc2dc \ud654\ubaa9\ud558\uac8c \uc9c0\ub0bc \uc218 \uc788\ub2e4\uba74 \uc5b4\ub978\ub4e4\uc774 \ud480\uc9c0 \ubabb\ud558\uc168\ub358 \uac10\uc815\uc744 \uc6b0\ub9ac\uac00 \ud480\uc5b4\ub4dc\ub9ac\ub294 \uc148\uc774 \ub418\ub294\uac8c \uc544\ub2c8\uaca0\ub0d0'\uba70 \uac10\uac1c\ubb34\ub7c9\ud574\ud558\uc600\ub2e4. \ubc15\uaddc\uc218\ub294 \uc544\ubc84\uc9c0\uac00 \uadf8\ud1a0\ub85d \uac15\uc870\ud588\ub358 \ubc31\uc138\uc758 \uc6d0\uc218\uc5d0 \ub300\ud55c \uc0dd\uac01\uc740 \uc78a\uace0 \uba3c\uc800 \uc190\uc744 \ub0b4\ubc00\uc5c8\uace0, \uc624\ud788\ub824 \uc9d1\uc548\uac04\uc758 \ubd88\ud654\ub97c \uc78a\uc790\uba70 \uc720\uae38\uc900\uc758 \ub6f0\uc5b4\ub09c \uc7ac\uc8fc\ub97c \uac70\ub4ed \uce6d\ucc2c\ud558\uc600\ub2e4. \ub610\ud55c \ud798\uc368 \uacf5\ubd80\ud560 \uac83\uc744 \ub2f9\ubd80\ud558\uba70 \uc790\uc8fc \ucc3e\uc544\uc62c \uac83\uc744 \uad8c\uace0\ud588\ub2e4. \ubc15\uaddc\uc218\uc758 \uc778\ud488\uc5d0 \uac10\ubcf5\ud55c \uc720\uae38\uc900\uc740 \uc624\ud788\ub824 \ubc15\uaddc\uc218\ub97c \uc2a4\uc2b9\uc73c\ub85c \ubc1b\ub4e4\uace0 \uadf8\ub85c\ubd80\ud130 \ud559\ubb38\uc744 \uc0ac\uc0ac\ubc1b\uc558\ub2e4.", "paragraph_answer": "1871\ub144 \ud64d\ubb38\uad00 \ub300\uc81c\ud559 \ubc15\uaddc\uc218\ub294 \ud5a5\uc2dc\uc5d0\uc11c \uc7a5\uc6d0\uc73c\ub85c \ubf51\ud78c \uc2dc\ub97c \ubcf4\uace0, \uadf8 \uc2dc\uc758 \uc8fc\uc778\uacf5\uc744 \ubd88\ub7ec\ub4e4\uc600\ub2e4. \uc2dc\ub97c \uc9c0\uc740 \uc774\ub294 16\uc138\uc758 \uc720\uae38\uc900\uc774\uc5c8\ub2e4. \ubc15\uaddc\uc218\ub294 \uc5f0\uc554 \ubc15\uc9c0\uc6d0\uc758 \uc544\ub4e4 \ubc15\uc885\ucc44\uc758 \uc544\ub4e4\uc774\uba70, \uc720\uae38\uc900\uc740 \uc720\ud55c\uc900\uc758 \uc544\ub4e4 \uc720\ud68c\uc8fc\uc758 4\ub300\uc190 \uc774\uc5c8\ub2e4. \uc720\uae38\uc900\uc774 \ucc98\uc74c\uc73c\ub85c \ubc15\uaddc\uc218\ub97c \ub9cc\ub098\ub7ec \uac08 \ub54c \uc720\uae38\uc900\uc758 \uc544\ubc84\uc9c0 \uc720\uc9c4\uc218\ub294 '\uc6b0\ub9ac\uc9d1\uacfc \uc11c\ub85c \uc6d0\uc218\uac19\uc774 \uc9c0\ub0b4\uc654\ub294\ub370 \uc5b4\ub5bb\uac8c \uadf8 \uc790\ub97c \ucc3e\uc544\uac04\ub2e4\ub294 \ub9d0\uc774\ub0d0'\uba70 \uc644\uac15\ud788 \ubc18\ub300\ud588\ub2e4. \uadf8\ub7ec\ub098 \uc6b0\uc5ec\uace1\uc808 \ub05d\uc5d0 \uc720\uae38\uc900\uc744 \ub9cc\ub09c \ubc15\uaddc\uc218\ub294 '\ub108\ud76c \uc9d1\uacfc \uc6b0\ub9ac \uc9d1\uc774 \uc9c0\ub09c\ub0a0 \uc0ac\uc18c\ud55c \ubb38\uc81c\ub85c \ubd88\ud654\ud588\uc73c\ub098 \uc774\uc81c\ubd80\ud130 \uc61b\ub0a0\ucc98\ub7fc \ub2e4\uc2dc \ud654\ubaa9\ud558\uac8c \uc9c0\ub0bc \uc218 \uc788\ub2e4\uba74 \uc5b4\ub978\ub4e4\uc774 \ud480\uc9c0 \ubabb\ud558\uc168\ub358 \uac10\uc815\uc744 \uc6b0\ub9ac\uac00 \ud480\uc5b4\ub4dc\ub9ac\ub294 \uc148\uc774 \ub418\ub294\uac8c \uc544\ub2c8\uaca0\ub0d0'\uba70 \uac10\uac1c\ubb34\ub7c9\ud574\ud558\uc600\ub2e4. \ubc15\uaddc\uc218\ub294 \uc544\ubc84\uc9c0\uac00 \uadf8\ud1a0\ub85d \uac15\uc870\ud588\ub358 \ubc31\uc138\uc758 \uc6d0\uc218\uc5d0 \ub300\ud55c \uc0dd\uac01\uc740 \uc78a\uace0 \uba3c\uc800 \uc190\uc744 \ub0b4\ubc00\uc5c8\uace0, \uc624\ud788\ub824 \uc9d1\uc548\uac04\uc758 \ubd88\ud654\ub97c \uc78a\uc790\uba70 \uc720\uae38\uc900\uc758 \ub6f0\uc5b4\ub09c \uc7ac\uc8fc\ub97c \uac70\ub4ed \uce6d\ucc2c\ud558\uc600\ub2e4. \ub610\ud55c \ud798\uc368 \uacf5\ubd80\ud560 \uac83\uc744 \ub2f9\ubd80\ud558\uba70 \uc790\uc8fc \ucc3e\uc544\uc62c \uac83\uc744 \uad8c\uace0\ud588\ub2e4. \ubc15\uaddc\uc218\uc758 \uc778\ud488\uc5d0 \uac10\ubcf5\ud55c \uc720\uae38\uc900\uc740 \uc624\ud788\ub824 \ubc15\uaddc\uc218\ub97c \uc2a4\uc2b9\uc73c\ub85c \ubc1b\ub4e4\uace0 \uadf8\ub85c\ubd80\ud130 \ud559\ubb38\uc744 \uc0ac\uc0ac\ubc1b\uc558\ub2e4.", "sentence_answer": "\ubc15\uaddc\uc218\ub294 \uc5f0\uc554 \ubc15\uc9c0\uc6d0\uc758 \uc544\ub4e4 \ubc15\uc885\ucc44\uc758 \uc544\ub4e4\uc774\uba70, \uc720\uae38\uc900\uc740 \uc720\ud55c\uc900\uc758 \uc544\ub4e4 \uc720\ud68c\uc8fc\uc758 4\ub300\uc190 \uc774\uc5c8\ub2e4.", "paragraph_id": "6542752-6-1", "paragraph_question": "question: \uc720\uae38\uc900\uc740 \uc720\ud55c\uc900\uc758 \uc544\ub4e4 \uc720\ud68c\uc8fc\uc758 \uba87\ub300\uc190\uc778\uac00?, context: 1871\ub144 \ud64d\ubb38\uad00 \ub300\uc81c\ud559 \ubc15\uaddc\uc218\ub294 \ud5a5\uc2dc\uc5d0\uc11c \uc7a5\uc6d0\uc73c\ub85c \ubf51\ud78c \uc2dc\ub97c \ubcf4\uace0, \uadf8 \uc2dc\uc758 \uc8fc\uc778\uacf5\uc744 \ubd88\ub7ec\ub4e4\uc600\ub2e4. \uc2dc\ub97c \uc9c0\uc740 \uc774\ub294 16\uc138\uc758 \uc720\uae38\uc900\uc774\uc5c8\ub2e4. \ubc15\uaddc\uc218\ub294 \uc5f0\uc554 \ubc15\uc9c0\uc6d0\uc758 \uc544\ub4e4 \ubc15\uc885\ucc44\uc758 \uc544\ub4e4\uc774\uba70, \uc720\uae38\uc900\uc740 \uc720\ud55c\uc900\uc758 \uc544\ub4e4 \uc720\ud68c\uc8fc\uc758 4\ub300\uc190\uc774\uc5c8\ub2e4. \uc720\uae38\uc900\uc774 \ucc98\uc74c\uc73c\ub85c \ubc15\uaddc\uc218\ub97c \ub9cc\ub098\ub7ec \uac08 \ub54c \uc720\uae38\uc900\uc758 \uc544\ubc84\uc9c0 \uc720\uc9c4\uc218\ub294 '\uc6b0\ub9ac\uc9d1\uacfc \uc11c\ub85c \uc6d0\uc218\uac19\uc774 \uc9c0\ub0b4\uc654\ub294\ub370 \uc5b4\ub5bb\uac8c \uadf8 \uc790\ub97c \ucc3e\uc544\uac04\ub2e4\ub294 \ub9d0\uc774\ub0d0'\uba70 \uc644\uac15\ud788 \ubc18\ub300\ud588\ub2e4. \uadf8\ub7ec\ub098 \uc6b0\uc5ec\uace1\uc808 \ub05d\uc5d0 \uc720\uae38\uc900\uc744 \ub9cc\ub09c \ubc15\uaddc\uc218\ub294 '\ub108\ud76c \uc9d1\uacfc \uc6b0\ub9ac \uc9d1\uc774 \uc9c0\ub09c\ub0a0 \uc0ac\uc18c\ud55c \ubb38\uc81c\ub85c \ubd88\ud654\ud588\uc73c\ub098 \uc774\uc81c\ubd80\ud130 \uc61b\ub0a0\ucc98\ub7fc \ub2e4\uc2dc \ud654\ubaa9\ud558\uac8c \uc9c0\ub0bc \uc218 \uc788\ub2e4\uba74 \uc5b4\ub978\ub4e4\uc774 \ud480\uc9c0 \ubabb\ud558\uc168\ub358 \uac10\uc815\uc744 \uc6b0\ub9ac\uac00 \ud480\uc5b4\ub4dc\ub9ac\ub294 \uc148\uc774 \ub418\ub294\uac8c \uc544\ub2c8\uaca0\ub0d0'\uba70 \uac10\uac1c\ubb34\ub7c9\ud574\ud558\uc600\ub2e4. \ubc15\uaddc\uc218\ub294 \uc544\ubc84\uc9c0\uac00 \uadf8\ud1a0\ub85d \uac15\uc870\ud588\ub358 \ubc31\uc138\uc758 \uc6d0\uc218\uc5d0 \ub300\ud55c \uc0dd\uac01\uc740 \uc78a\uace0 \uba3c\uc800 \uc190\uc744 \ub0b4\ubc00\uc5c8\uace0, \uc624\ud788\ub824 \uc9d1\uc548\uac04\uc758 \ubd88\ud654\ub97c \uc78a\uc790\uba70 \uc720\uae38\uc900\uc758 \ub6f0\uc5b4\ub09c \uc7ac\uc8fc\ub97c \uac70\ub4ed \uce6d\ucc2c\ud558\uc600\ub2e4. \ub610\ud55c \ud798\uc368 \uacf5\ubd80\ud560 \uac83\uc744 \ub2f9\ubd80\ud558\uba70 \uc790\uc8fc \ucc3e\uc544\uc62c \uac83\uc744 \uad8c\uace0\ud588\ub2e4. \ubc15\uaddc\uc218\uc758 \uc778\ud488\uc5d0 \uac10\ubcf5\ud55c \uc720\uae38\uc900\uc740 \uc624\ud788\ub824 \ubc15\uaddc\uc218\ub97c \uc2a4\uc2b9\uc73c\ub85c \ubc1b\ub4e4\uace0 \uadf8\ub85c\ubd80\ud130 \ud559\ubb38\uc744 \uc0ac\uc0ac\ubc1b\uc558\ub2e4."} {"question": "\uc774 \uc9d1\ub2e8\uc758 \uc0c1\uc2b9\uacfc \uc774\ub85c \uc778\ud55c \ubab0\ub77d\uc740 \uc774 \uc9d1\ub2e8\uc758 \uad6c\uc131\uc744 \uc7a5\uae30\uac04\uc5d0 \uac78\uccd0 \uc870\uae08\uc529 \ub2ec\ub77c\uc9c0\uac8c \ud588\ub294\ub370, \uc774 \uc9d1\ub2e8\uc740 \ubb34\uc5c7\uc778\uac00?", "paragraph": "\ubb38\ubc8c \ucd9c\uc2e0\uc740 \uc720\ub9ac\ud55c \uad50\uc721\uc0c1\uc758 \uc5ec\uac74\uc73c\ub85c \ub2e4\uc218\uc758 \uacfc\uac70 \ud569\uaca9\uc790\ub97c \ubc30\ucd9c\ud558\uc600\ub2e4. \ub610\ud55c \uc774\ub4e4\uc740 \uacfc\uac70\uc5d0 \ud569\uaca9\ud558\uc9c0 \ubabb\ud558\ub354\ub77c\ub3c4 \uace0\uad00\uc774\ub098 \uacf5\uc2e0\uc774\uc5c8\ub358 \uc120\ub300\uc758 \ub355\ud0dd\uc73c\ub85c \uad00\uc9c1\uc744 \ubc1b\uc744 \uc218 \uc788\ub294 \uc5ec\ub7ec \uac00\uc9c0 \uc74c\uc11c(\u852d\u654d)\uc758 \uae30\ud68c\uac00 \uc788\uc5b4, \uad00\ub9ac\ub85c\uc758 \uc9c4\ucd9c\uc774 \uc0ac\uc2e4\uc0c1 \ubcf4\uc7a5\ub418\uc5c8\ub2e4. \uacfc\uac70\ub4e0 \uc74c\uc11c\ub4e0 \uc77c\ub2e8 \ud558\uc704\uad00\ub9ac\ub85c \uc9c4\ucd9c\ud55c \ubb38\ubc8c \ucd9c\uc2e0\uc740 \uace0\ub824 \uc870\uc815\uc758 \ucd5c\uace0\uc704 \uad00\ub9ac\ub4e4\uc778 \ub0b4\uc678 \uce5c\uc871\ub4e4\uc758 \ube44\ud638\ub97c \ubc1b\uc73c\uba70 \uc27d\uac8c \uace0\uad00\uc73c\ub85c \uc2b9\uc9c4\ud560 \uc218 \uc788\uc5c8\ub2e4. \ud558\uc704\uc758 \uc9c0\ubc30\uce35 \ucd9c\uc2e0\uc740 \uacfc\uac70\uc5d0 \uc6b0\uc218\ud55c \uc131\uc801\uc73c\ub85c \ud569\uaca9\ud558\uace0 \uad00\ub9ac\ub85c\uc11c \uce58\uc801\uc744 \uc313\uc544\ub3c4 \ub0ae\uc740 \ud488\uad00\uc9c1\uc5d0\uc11c \uc815\uccb4\ub418\ub294 \uacbd\uc6b0\uac00 \uc788\uc5c8\uace0, \uc5b4\ub835\uac8c \uace0\uad00\uc774 \ub418\uc5b4\ub3c4 \ub2e4\uc74c \uc138\ub300\uc5d0 \uac70\ub4ed \uace0\uad00\uc744 \ubc30\ucd9c\ud558\uae30\ub294 \uadf9\ud788 \uc5b4\ub824\uc6e0\ub2e4. \uadf8\ub807\uc9c0\ub9cc \ud558\uc704\uc758 \uc9c0\ubc30\uce35\uc5d0\uc11c\ub3c4 \uc18c\uc218\ub098\ub9c8 \uc7ac\ucd94 \ub4f1\uc758 \uace0\uad00\uc774 \ubc30\ucd9c\ub418\uc5c8\uc73c\uba70, \uadf9\uc18c\uc218\ub294 \ubb38\ubc8c\uce35\uacfc \ud1b5\ud63c\ud558\uba70 \ubb38\ubc8c\uce35\uc73c\ub85c \uc0c1\uc2b9\ud574\uac00\ub294 \uacbd\uc6b0\ub3c4 \uc788\uc5c8\ub2e4. \uc774\ub7ec\ud55c \ubb38\ubc8c\uce35\uc73c\ub85c\uc758 \uc0c1\uc2b9\uacfc \uadf8\ub85c\ubd80\ud130\uc758 \ubab0\ub77d\uc740 \ubb38\ubc8c\uce35\uc758 \uad6c\uc131\uc744 \uc7a5\uae30\uac04\uc5d0 \uac78\uccd0 \uc870\uae08\uc529 \ub2ec\ub77c\uc9c0\uac8c \ud558\uc600\ub2e4.", "answer": "\ubb38\ubc8c\uce35", "sentence": "\uadf8\ub807\uc9c0\ub9cc \ud558\uc704\uc758 \uc9c0\ubc30\uce35\uc5d0\uc11c\ub3c4 \uc18c\uc218\ub098\ub9c8 \uc7ac\ucd94 \ub4f1\uc758 \uace0\uad00\uc774 \ubc30\ucd9c\ub418\uc5c8\uc73c\uba70, \uadf9\uc18c\uc218\ub294 \ubb38\ubc8c\uce35 \uacfc \ud1b5\ud63c\ud558\uba70 \ubb38\ubc8c\uce35\uc73c\ub85c \uc0c1\uc2b9\ud574\uac00\ub294 \uacbd\uc6b0\ub3c4 \uc788\uc5c8\ub2e4.", "paragraph_sentence": "\ubb38\ubc8c \ucd9c\uc2e0\uc740 \uc720\ub9ac\ud55c \uad50\uc721\uc0c1\uc758 \uc5ec\uac74\uc73c\ub85c \ub2e4\uc218\uc758 \uacfc\uac70 \ud569\uaca9\uc790\ub97c \ubc30\ucd9c\ud558\uc600\ub2e4. \ub610\ud55c \uc774\ub4e4\uc740 \uacfc\uac70\uc5d0 \ud569\uaca9\ud558\uc9c0 \ubabb\ud558\ub354\ub77c\ub3c4 \uace0\uad00\uc774\ub098 \uacf5\uc2e0\uc774\uc5c8\ub358 \uc120\ub300\uc758 \ub355\ud0dd\uc73c\ub85c \uad00\uc9c1\uc744 \ubc1b\uc744 \uc218 \uc788\ub294 \uc5ec\ub7ec \uac00\uc9c0 \uc74c\uc11c(\u852d\u654d)\uc758 \uae30\ud68c\uac00 \uc788\uc5b4, \uad00\ub9ac\ub85c\uc758 \uc9c4\ucd9c\uc774 \uc0ac\uc2e4\uc0c1 \ubcf4\uc7a5\ub418\uc5c8\ub2e4. \uacfc\uac70\ub4e0 \uc74c\uc11c\ub4e0 \uc77c\ub2e8 \ud558\uc704\uad00\ub9ac\ub85c \uc9c4\ucd9c\ud55c \ubb38\ubc8c \ucd9c\uc2e0\uc740 \uace0\ub824 \uc870\uc815\uc758 \ucd5c\uace0\uc704 \uad00\ub9ac\ub4e4\uc778 \ub0b4\uc678 \uce5c\uc871\ub4e4\uc758 \ube44\ud638\ub97c \ubc1b\uc73c\uba70 \uc27d\uac8c \uace0\uad00\uc73c\ub85c \uc2b9\uc9c4\ud560 \uc218 \uc788\uc5c8\ub2e4. \ud558\uc704\uc758 \uc9c0\ubc30\uce35 \ucd9c\uc2e0\uc740 \uacfc\uac70\uc5d0 \uc6b0\uc218\ud55c \uc131\uc801\uc73c\ub85c \ud569\uaca9\ud558\uace0 \uad00\ub9ac\ub85c\uc11c \uce58\uc801\uc744 \uc313\uc544\ub3c4 \ub0ae\uc740 \ud488\uad00\uc9c1\uc5d0\uc11c \uc815\uccb4\ub418\ub294 \uacbd\uc6b0\uac00 \uc788\uc5c8\uace0, \uc5b4\ub835\uac8c \uace0\uad00\uc774 \ub418\uc5b4\ub3c4 \ub2e4\uc74c \uc138\ub300\uc5d0 \uac70\ub4ed \uace0\uad00\uc744 \ubc30\ucd9c\ud558\uae30\ub294 \uadf9\ud788 \uc5b4\ub824\uc6e0\ub2e4. \uadf8\ub807\uc9c0\ub9cc \ud558\uc704\uc758 \uc9c0\ubc30\uce35\uc5d0\uc11c\ub3c4 \uc18c\uc218\ub098\ub9c8 \uc7ac\ucd94 \ub4f1\uc758 \uace0\uad00\uc774 \ubc30\ucd9c\ub418\uc5c8\uc73c\uba70, \uadf9\uc18c\uc218\ub294 \ubb38\ubc8c\uce35 \uacfc \ud1b5\ud63c\ud558\uba70 \ubb38\ubc8c\uce35\uc73c\ub85c \uc0c1\uc2b9\ud574\uac00\ub294 \uacbd\uc6b0\ub3c4 \uc788\uc5c8\ub2e4. \uc774\ub7ec\ud55c \ubb38\ubc8c\uce35\uc73c\ub85c\uc758 \uc0c1\uc2b9\uacfc \uadf8\ub85c\ubd80\ud130\uc758 \ubab0\ub77d\uc740 \ubb38\ubc8c\uce35\uc758 \uad6c\uc131\uc744 \uc7a5\uae30\uac04\uc5d0 \uac78\uccd0 \uc870\uae08\uc529 \ub2ec\ub77c\uc9c0\uac8c \ud558\uc600\ub2e4.", "paragraph_answer": "\ubb38\ubc8c \ucd9c\uc2e0\uc740 \uc720\ub9ac\ud55c \uad50\uc721\uc0c1\uc758 \uc5ec\uac74\uc73c\ub85c \ub2e4\uc218\uc758 \uacfc\uac70 \ud569\uaca9\uc790\ub97c \ubc30\ucd9c\ud558\uc600\ub2e4. \ub610\ud55c \uc774\ub4e4\uc740 \uacfc\uac70\uc5d0 \ud569\uaca9\ud558\uc9c0 \ubabb\ud558\ub354\ub77c\ub3c4 \uace0\uad00\uc774\ub098 \uacf5\uc2e0\uc774\uc5c8\ub358 \uc120\ub300\uc758 \ub355\ud0dd\uc73c\ub85c \uad00\uc9c1\uc744 \ubc1b\uc744 \uc218 \uc788\ub294 \uc5ec\ub7ec \uac00\uc9c0 \uc74c\uc11c(\u852d\u654d)\uc758 \uae30\ud68c\uac00 \uc788\uc5b4, \uad00\ub9ac\ub85c\uc758 \uc9c4\ucd9c\uc774 \uc0ac\uc2e4\uc0c1 \ubcf4\uc7a5\ub418\uc5c8\ub2e4. \uacfc\uac70\ub4e0 \uc74c\uc11c\ub4e0 \uc77c\ub2e8 \ud558\uc704\uad00\ub9ac\ub85c \uc9c4\ucd9c\ud55c \ubb38\ubc8c \ucd9c\uc2e0\uc740 \uace0\ub824 \uc870\uc815\uc758 \ucd5c\uace0\uc704 \uad00\ub9ac\ub4e4\uc778 \ub0b4\uc678 \uce5c\uc871\ub4e4\uc758 \ube44\ud638\ub97c \ubc1b\uc73c\uba70 \uc27d\uac8c \uace0\uad00\uc73c\ub85c \uc2b9\uc9c4\ud560 \uc218 \uc788\uc5c8\ub2e4. \ud558\uc704\uc758 \uc9c0\ubc30\uce35 \ucd9c\uc2e0\uc740 \uacfc\uac70\uc5d0 \uc6b0\uc218\ud55c \uc131\uc801\uc73c\ub85c \ud569\uaca9\ud558\uace0 \uad00\ub9ac\ub85c\uc11c \uce58\uc801\uc744 \uc313\uc544\ub3c4 \ub0ae\uc740 \ud488\uad00\uc9c1\uc5d0\uc11c \uc815\uccb4\ub418\ub294 \uacbd\uc6b0\uac00 \uc788\uc5c8\uace0, \uc5b4\ub835\uac8c \uace0\uad00\uc774 \ub418\uc5b4\ub3c4 \ub2e4\uc74c \uc138\ub300\uc5d0 \uac70\ub4ed \uace0\uad00\uc744 \ubc30\ucd9c\ud558\uae30\ub294 \uadf9\ud788 \uc5b4\ub824\uc6e0\ub2e4. \uadf8\ub807\uc9c0\ub9cc \ud558\uc704\uc758 \uc9c0\ubc30\uce35\uc5d0\uc11c\ub3c4 \uc18c\uc218\ub098\ub9c8 \uc7ac\ucd94 \ub4f1\uc758 \uace0\uad00\uc774 \ubc30\ucd9c\ub418\uc5c8\uc73c\uba70, \uadf9\uc18c\uc218\ub294 \ubb38\ubc8c\uce35 \uacfc \ud1b5\ud63c\ud558\uba70 \ubb38\ubc8c\uce35\uc73c\ub85c \uc0c1\uc2b9\ud574\uac00\ub294 \uacbd\uc6b0\ub3c4 \uc788\uc5c8\ub2e4. \uc774\ub7ec\ud55c \ubb38\ubc8c\uce35\uc73c\ub85c\uc758 \uc0c1\uc2b9\uacfc \uadf8\ub85c\ubd80\ud130\uc758 \ubab0\ub77d\uc740 \ubb38\ubc8c\uce35\uc758 \uad6c\uc131\uc744 \uc7a5\uae30\uac04\uc5d0 \uac78\uccd0 \uc870\uae08\uc529 \ub2ec\ub77c\uc9c0\uac8c \ud558\uc600\ub2e4.", "sentence_answer": "\uadf8\ub807\uc9c0\ub9cc \ud558\uc704\uc758 \uc9c0\ubc30\uce35\uc5d0\uc11c\ub3c4 \uc18c\uc218\ub098\ub9c8 \uc7ac\ucd94 \ub4f1\uc758 \uace0\uad00\uc774 \ubc30\ucd9c\ub418\uc5c8\uc73c\uba70, \uadf9\uc18c\uc218\ub294 \ubb38\ubc8c\uce35 \uacfc \ud1b5\ud63c\ud558\uba70 \ubb38\ubc8c\uce35\uc73c\ub85c \uc0c1\uc2b9\ud574\uac00\ub294 \uacbd\uc6b0\ub3c4 \uc788\uc5c8\ub2e4.", "paragraph_id": "6471358-0-1", "paragraph_question": "question: \uc774 \uc9d1\ub2e8\uc758 \uc0c1\uc2b9\uacfc \uc774\ub85c \uc778\ud55c \ubab0\ub77d\uc740 \uc774 \uc9d1\ub2e8\uc758 \uad6c\uc131\uc744 \uc7a5\uae30\uac04\uc5d0 \uac78\uccd0 \uc870\uae08\uc529 \ub2ec\ub77c\uc9c0\uac8c \ud588\ub294\ub370, \uc774 \uc9d1\ub2e8\uc740 \ubb34\uc5c7\uc778\uac00?, context: \ubb38\ubc8c \ucd9c\uc2e0\uc740 \uc720\ub9ac\ud55c \uad50\uc721\uc0c1\uc758 \uc5ec\uac74\uc73c\ub85c \ub2e4\uc218\uc758 \uacfc\uac70 \ud569\uaca9\uc790\ub97c \ubc30\ucd9c\ud558\uc600\ub2e4. \ub610\ud55c \uc774\ub4e4\uc740 \uacfc\uac70\uc5d0 \ud569\uaca9\ud558\uc9c0 \ubabb\ud558\ub354\ub77c\ub3c4 \uace0\uad00\uc774\ub098 \uacf5\uc2e0\uc774\uc5c8\ub358 \uc120\ub300\uc758 \ub355\ud0dd\uc73c\ub85c \uad00\uc9c1\uc744 \ubc1b\uc744 \uc218 \uc788\ub294 \uc5ec\ub7ec \uac00\uc9c0 \uc74c\uc11c(\u852d\u654d)\uc758 \uae30\ud68c\uac00 \uc788\uc5b4, \uad00\ub9ac\ub85c\uc758 \uc9c4\ucd9c\uc774 \uc0ac\uc2e4\uc0c1 \ubcf4\uc7a5\ub418\uc5c8\ub2e4. \uacfc\uac70\ub4e0 \uc74c\uc11c\ub4e0 \uc77c\ub2e8 \ud558\uc704\uad00\ub9ac\ub85c \uc9c4\ucd9c\ud55c \ubb38\ubc8c \ucd9c\uc2e0\uc740 \uace0\ub824 \uc870\uc815\uc758 \ucd5c\uace0\uc704 \uad00\ub9ac\ub4e4\uc778 \ub0b4\uc678 \uce5c\uc871\ub4e4\uc758 \ube44\ud638\ub97c \ubc1b\uc73c\uba70 \uc27d\uac8c \uace0\uad00\uc73c\ub85c \uc2b9\uc9c4\ud560 \uc218 \uc788\uc5c8\ub2e4. \ud558\uc704\uc758 \uc9c0\ubc30\uce35 \ucd9c\uc2e0\uc740 \uacfc\uac70\uc5d0 \uc6b0\uc218\ud55c \uc131\uc801\uc73c\ub85c \ud569\uaca9\ud558\uace0 \uad00\ub9ac\ub85c\uc11c \uce58\uc801\uc744 \uc313\uc544\ub3c4 \ub0ae\uc740 \ud488\uad00\uc9c1\uc5d0\uc11c \uc815\uccb4\ub418\ub294 \uacbd\uc6b0\uac00 \uc788\uc5c8\uace0, \uc5b4\ub835\uac8c \uace0\uad00\uc774 \ub418\uc5b4\ub3c4 \ub2e4\uc74c \uc138\ub300\uc5d0 \uac70\ub4ed \uace0\uad00\uc744 \ubc30\ucd9c\ud558\uae30\ub294 \uadf9\ud788 \uc5b4\ub824\uc6e0\ub2e4. \uadf8\ub807\uc9c0\ub9cc \ud558\uc704\uc758 \uc9c0\ubc30\uce35\uc5d0\uc11c\ub3c4 \uc18c\uc218\ub098\ub9c8 \uc7ac\ucd94 \ub4f1\uc758 \uace0\uad00\uc774 \ubc30\ucd9c\ub418\uc5c8\uc73c\uba70, \uadf9\uc18c\uc218\ub294 \ubb38\ubc8c\uce35\uacfc \ud1b5\ud63c\ud558\uba70 \ubb38\ubc8c\uce35\uc73c\ub85c \uc0c1\uc2b9\ud574\uac00\ub294 \uacbd\uc6b0\ub3c4 \uc788\uc5c8\ub2e4. \uc774\ub7ec\ud55c \ubb38\ubc8c\uce35\uc73c\ub85c\uc758 \uc0c1\uc2b9\uacfc \uadf8\ub85c\ubd80\ud130\uc758 \ubab0\ub77d\uc740 \ubb38\ubc8c\uce35\uc758 \uad6c\uc131\uc744 \uc7a5\uae30\uac04\uc5d0 \uac78\uccd0 \uc870\uae08\uc529 \ub2ec\ub77c\uc9c0\uac8c \ud558\uc600\ub2e4."} {"question": "\ud3c9\ucc3d \uc62c\ub9bc\ud53d\uc744 \ub9de\uc544 \uae40\uc5ec\uc815\uacfc \uae40\uc601\ub0a8\uc774 \ubc29\ub0a8\ud558\uace0 \ub2e4\uc74c \ub0a0 \ubc29\ubb38\ud55c \uacf3\uc740?", "paragraph": "\ud3c9\ucc3d\uc62c\ub9bc\ud53d\uc744 \ub9de\uc774\ud558\uc5ec 2\uc6d4 9\uc77c\uc5d0 \uae40\uc815\uc740 \uc704\uc6d0\uc7a5\uc758 \uc5ec\ub3d9\uc0dd\uc778 \uae40\uc5ec\uc815 \ub85c\ub3d9\ub2f9 \uc81c1\ubd80\ubd80\uc7a5\uc774 \ud2b9\uc0ac \uc790\uaca9\uc73c\ub85c 2\ubc15 3\uc77c \uac04 \ubc29\ub0a8\ud588\ub2e4. \uae40\uc601\ub0a8 \ucd5c\uace0\uc778\ubbfc\ud68c\uc758 \uc0c1\uc784\uc704\uc6d0\uc7a5, \ucd5c\ud718 \uad6d\uac00\uccb4\uc721\uc9c0\ub3c4\uc704\uc6d0\uc7a5, \ub9ac\uc120\uad8c \uc870\uad6d\ud3c9\ud654\ud1b5\uc77c\uc704\uc6d0\ud68c \uc704\uc6d0\uc7a5\ub3c4 \ub3d9\ud589\ud588\ub2e4. \ud55c\uad6d \uc804\uc7c1 \uc774\ud6c4 \uae40\uc77c\uc131 \uac00\ubb38 \ud608\ud1b5\uc758 \uccab \ubc29\ubb38\uc774\ub77c\ub294 \uc810\uc5d0\uc11c \ud070 \uc758\ubbf8\ub97c \uc0c0\uc9c0\ub9cc \ud55c\uad6d\ub2f9\uc744 \ube44\ub86f\ud55c \ubcf4\uc218 \uc138\ub825\uc740 \ube44\ud575\ud654\ub97c \uc804\uc81c\ub85c \ud55c \ub0a8\ubd81\ub300\ud654\ub97c \uc8fc\uc7a5\ud558\uba70 \uc704\uc7a5 \ud3c9\ud654 \uacf5\uc138\uc5d0 \ud718\ub458\ub9ac\uc9c0 \ub9d0 \uac83\uc744 \uac15\ub825\ud788 \uc694\uad6c\ud588\ub2e4. \ub2e4\uc74c \ub0a0\uc5d0\ub294 \uae40\uc5ec\uc815\uacfc \uae40\uc601\ub0a8\uc774 \uccad\uc640\ub300\ub97c \ubc29\ubb38\ud558\uc5ec \ubb38\uc7ac\uc778\uacfc \ud68c\ub3d9\ud558\uc600\ub2e4. \ubb38\uc7ac\uc778\uc740 \uac74\ubc30\uc0ac\uc5d0\uc11c \"\uc624\ub298 \uc774 \uc790\ub9ac\uc5d0 \uc804 \uc138\uacc4\uc758 \uc774\ubaa9\uc774 \uc9d1\uc911\ub418\uace0 \ub0a8\ubd81\uc5d0 \uac70\ub294 \uae30\ub300\uac00 \ud06c\ub2e4\"\uba70 \ub73b\uae4a\uc740 \uc790\ub9ac\uac00 \ub418\uae30\ub97c \ud76c\ub9dd\ud588\uc73c\uba70 \uae40\uc601\ub0a8\uc740 \"\uc6b0\ub9ac\ub97c \ub530\ub73b\ud558\uace0 \uce5c\uc808\ud558\uac8c \ud658\ub300\ud574 \uc918 \ub3d9\ud3ec\uc758 \uc815\uc744 \ub290\ub080\ub2e4\"\uba70 \ubd81\ub0a8\uad00\uacc4\uc758 \ud68d\uae30\uc801 \uc804\ud658\uc810\uc774 \ub418\uae30\ub97c \uae30\ub300\ud55c\ub2e4\uace0 \uac15\uc870\ud588\ub2e4. \uc774\uc5b4 11\uc77c\uc5d0\ub294 \uc774\ub099\uc5f0\uc774 \uc6cc\ucee4\ud790\ud638\ud154\uc5d0\uc11c \uae40\uc5ec\uc815\uc744 \ub9cc\ub0ac\ub2e4. \uc774\ub099\uc5f0\uc740 2018\ub144 \ub3d9\uacc4 \uc62c\ub9bc\ud53d \ub0a8\ubd81 \ub2e8\uc77c \uc120\uc218\ub2e8\uacfc \uae40\uc5ec\uc815\uc758 \ubc29\ub0a8\uc744 \uc5b8\uae09\ud558\uba70 \uc5bc\ub9c8 \uc804\uae4c\uc9c0 \uc0c1\uc0c1\uc870\ucc28 \uc5b4\ub824\uc6e0\ub358 \uc77c\ub4e4\uc774 \ud604\uc2e4\ub85c \uc774\ub8e8\uc5b4\uc84c\ub2e4\uba70 \"\ub0a8\uacfc \ubd81\ub3c4 \ubaa8\ub4e0 \ub09c\uad00\uc744 \uc774\uae30\uace0 \uacf5\ub3d9\ubc88\uc601\uacfc \ud3c9\ud654\ud1b5\uc77c\uc758 \ubaa9\ud45c\uc5d0 \uc774\ub974\uae30\ub97c \uc18c\ub9dd\ud55c\ub2e4\"\uace0 \uc804\ud588\ub2e4.", "answer": "\uccad\uc640\ub300", "sentence": "\ub2e4\uc74c \ub0a0\uc5d0\ub294 \uae40\uc5ec\uc815\uacfc \uae40\uc601\ub0a8\uc774 \uccad\uc640\ub300 \ub97c \ubc29\ubb38\ud558\uc5ec \ubb38\uc7ac\uc778\uacfc \ud68c\ub3d9\ud558\uc600\ub2e4.", "paragraph_sentence": "\ud3c9\ucc3d\uc62c\ub9bc\ud53d\uc744 \ub9de\uc774\ud558\uc5ec 2\uc6d4 9\uc77c\uc5d0 \uae40\uc815\uc740 \uc704\uc6d0\uc7a5\uc758 \uc5ec\ub3d9\uc0dd\uc778 \uae40\uc5ec\uc815 \ub85c\ub3d9\ub2f9 \uc81c1\ubd80\ubd80\uc7a5\uc774 \ud2b9\uc0ac \uc790\uaca9\uc73c\ub85c 2\ubc15 3\uc77c \uac04 \ubc29\ub0a8\ud588\ub2e4. \uae40\uc601\ub0a8 \ucd5c\uace0\uc778\ubbfc\ud68c\uc758 \uc0c1\uc784\uc704\uc6d0\uc7a5, \ucd5c\ud718 \uad6d\uac00\uccb4\uc721\uc9c0\ub3c4\uc704\uc6d0\uc7a5, \ub9ac\uc120\uad8c \uc870\uad6d\ud3c9\ud654\ud1b5\uc77c\uc704\uc6d0\ud68c \uc704\uc6d0\uc7a5\ub3c4 \ub3d9\ud589\ud588\ub2e4. \ud55c\uad6d \uc804\uc7c1 \uc774\ud6c4 \uae40\uc77c\uc131 \uac00\ubb38 \ud608\ud1b5\uc758 \uccab \ubc29\ubb38\uc774\ub77c\ub294 \uc810\uc5d0\uc11c \ud070 \uc758\ubbf8\ub97c \uc0c0\uc9c0\ub9cc \ud55c\uad6d\ub2f9\uc744 \ube44\ub86f\ud55c \ubcf4\uc218 \uc138\ub825\uc740 \ube44\ud575\ud654\ub97c \uc804\uc81c\ub85c \ud55c \ub0a8\ubd81\ub300\ud654\ub97c \uc8fc\uc7a5\ud558\uba70 \uc704\uc7a5 \ud3c9\ud654 \uacf5\uc138\uc5d0 \ud718\ub458\ub9ac\uc9c0 \ub9d0 \uac83\uc744 \uac15\ub825\ud788 \uc694\uad6c\ud588\ub2e4. \ub2e4\uc74c \ub0a0\uc5d0\ub294 \uae40\uc5ec\uc815\uacfc \uae40\uc601\ub0a8\uc774 \uccad\uc640\ub300 \ub97c \ubc29\ubb38\ud558\uc5ec \ubb38\uc7ac\uc778\uacfc \ud68c\ub3d9\ud558\uc600\ub2e4. \ubb38\uc7ac\uc778\uc740 \uac74\ubc30\uc0ac\uc5d0\uc11c \"\uc624\ub298 \uc774 \uc790\ub9ac\uc5d0 \uc804 \uc138\uacc4\uc758 \uc774\ubaa9\uc774 \uc9d1\uc911\ub418\uace0 \ub0a8\ubd81\uc5d0 \uac70\ub294 \uae30\ub300\uac00 \ud06c\ub2e4\"\uba70 \ub73b\uae4a\uc740 \uc790\ub9ac\uac00 \ub418\uae30\ub97c \ud76c\ub9dd\ud588\uc73c\uba70 \uae40\uc601\ub0a8\uc740 \"\uc6b0\ub9ac\ub97c \ub530\ub73b\ud558\uace0 \uce5c\uc808\ud558\uac8c \ud658\ub300\ud574 \uc918 \ub3d9\ud3ec\uc758 \uc815\uc744 \ub290\ub080\ub2e4\"\uba70 \ubd81\ub0a8\uad00\uacc4\uc758 \ud68d\uae30\uc801 \uc804\ud658\uc810\uc774 \ub418\uae30\ub97c \uae30\ub300\ud55c\ub2e4\uace0 \uac15\uc870\ud588\ub2e4. \uc774\uc5b4 11\uc77c\uc5d0\ub294 \uc774\ub099\uc5f0\uc774 \uc6cc\ucee4\ud790\ud638\ud154\uc5d0\uc11c \uae40\uc5ec\uc815\uc744 \ub9cc\ub0ac\ub2e4. \uc774\ub099\uc5f0\uc740 2018\ub144 \ub3d9\uacc4 \uc62c\ub9bc\ud53d \ub0a8\ubd81 \ub2e8\uc77c \uc120\uc218\ub2e8\uacfc \uae40\uc5ec\uc815\uc758 \ubc29\ub0a8\uc744 \uc5b8\uae09\ud558\uba70 \uc5bc\ub9c8 \uc804\uae4c\uc9c0 \uc0c1\uc0c1\uc870\ucc28 \uc5b4\ub824\uc6e0\ub358 \uc77c\ub4e4\uc774 \ud604\uc2e4\ub85c \uc774\ub8e8\uc5b4\uc84c\ub2e4\uba70 \"\ub0a8\uacfc \ubd81\ub3c4 \ubaa8\ub4e0 \ub09c\uad00\uc744 \uc774\uae30\uace0 \uacf5\ub3d9\ubc88\uc601\uacfc \ud3c9\ud654\ud1b5\uc77c\uc758 \ubaa9\ud45c\uc5d0 \uc774\ub974\uae30\ub97c \uc18c\ub9dd\ud55c\ub2e4\"\uace0 \uc804\ud588\ub2e4.", "paragraph_answer": "\ud3c9\ucc3d\uc62c\ub9bc\ud53d\uc744 \ub9de\uc774\ud558\uc5ec 2\uc6d4 9\uc77c\uc5d0 \uae40\uc815\uc740 \uc704\uc6d0\uc7a5\uc758 \uc5ec\ub3d9\uc0dd\uc778 \uae40\uc5ec\uc815 \ub85c\ub3d9\ub2f9 \uc81c1\ubd80\ubd80\uc7a5\uc774 \ud2b9\uc0ac \uc790\uaca9\uc73c\ub85c 2\ubc15 3\uc77c \uac04 \ubc29\ub0a8\ud588\ub2e4. \uae40\uc601\ub0a8 \ucd5c\uace0\uc778\ubbfc\ud68c\uc758 \uc0c1\uc784\uc704\uc6d0\uc7a5, \ucd5c\ud718 \uad6d\uac00\uccb4\uc721\uc9c0\ub3c4\uc704\uc6d0\uc7a5, \ub9ac\uc120\uad8c \uc870\uad6d\ud3c9\ud654\ud1b5\uc77c\uc704\uc6d0\ud68c \uc704\uc6d0\uc7a5\ub3c4 \ub3d9\ud589\ud588\ub2e4. \ud55c\uad6d \uc804\uc7c1 \uc774\ud6c4 \uae40\uc77c\uc131 \uac00\ubb38 \ud608\ud1b5\uc758 \uccab \ubc29\ubb38\uc774\ub77c\ub294 \uc810\uc5d0\uc11c \ud070 \uc758\ubbf8\ub97c \uc0c0\uc9c0\ub9cc \ud55c\uad6d\ub2f9\uc744 \ube44\ub86f\ud55c \ubcf4\uc218 \uc138\ub825\uc740 \ube44\ud575\ud654\ub97c \uc804\uc81c\ub85c \ud55c \ub0a8\ubd81\ub300\ud654\ub97c \uc8fc\uc7a5\ud558\uba70 \uc704\uc7a5 \ud3c9\ud654 \uacf5\uc138\uc5d0 \ud718\ub458\ub9ac\uc9c0 \ub9d0 \uac83\uc744 \uac15\ub825\ud788 \uc694\uad6c\ud588\ub2e4. \ub2e4\uc74c \ub0a0\uc5d0\ub294 \uae40\uc5ec\uc815\uacfc \uae40\uc601\ub0a8\uc774 \uccad\uc640\ub300 \ub97c \ubc29\ubb38\ud558\uc5ec \ubb38\uc7ac\uc778\uacfc \ud68c\ub3d9\ud558\uc600\ub2e4. \ubb38\uc7ac\uc778\uc740 \uac74\ubc30\uc0ac\uc5d0\uc11c \"\uc624\ub298 \uc774 \uc790\ub9ac\uc5d0 \uc804 \uc138\uacc4\uc758 \uc774\ubaa9\uc774 \uc9d1\uc911\ub418\uace0 \ub0a8\ubd81\uc5d0 \uac70\ub294 \uae30\ub300\uac00 \ud06c\ub2e4\"\uba70 \ub73b\uae4a\uc740 \uc790\ub9ac\uac00 \ub418\uae30\ub97c \ud76c\ub9dd\ud588\uc73c\uba70 \uae40\uc601\ub0a8\uc740 \"\uc6b0\ub9ac\ub97c \ub530\ub73b\ud558\uace0 \uce5c\uc808\ud558\uac8c \ud658\ub300\ud574 \uc918 \ub3d9\ud3ec\uc758 \uc815\uc744 \ub290\ub080\ub2e4\"\uba70 \ubd81\ub0a8\uad00\uacc4\uc758 \ud68d\uae30\uc801 \uc804\ud658\uc810\uc774 \ub418\uae30\ub97c \uae30\ub300\ud55c\ub2e4\uace0 \uac15\uc870\ud588\ub2e4. \uc774\uc5b4 11\uc77c\uc5d0\ub294 \uc774\ub099\uc5f0\uc774 \uc6cc\ucee4\ud790\ud638\ud154\uc5d0\uc11c \uae40\uc5ec\uc815\uc744 \ub9cc\ub0ac\ub2e4. \uc774\ub099\uc5f0\uc740 2018\ub144 \ub3d9\uacc4 \uc62c\ub9bc\ud53d \ub0a8\ubd81 \ub2e8\uc77c \uc120\uc218\ub2e8\uacfc \uae40\uc5ec\uc815\uc758 \ubc29\ub0a8\uc744 \uc5b8\uae09\ud558\uba70 \uc5bc\ub9c8 \uc804\uae4c\uc9c0 \uc0c1\uc0c1\uc870\ucc28 \uc5b4\ub824\uc6e0\ub358 \uc77c\ub4e4\uc774 \ud604\uc2e4\ub85c \uc774\ub8e8\uc5b4\uc84c\ub2e4\uba70 \"\ub0a8\uacfc \ubd81\ub3c4 \ubaa8\ub4e0 \ub09c\uad00\uc744 \uc774\uae30\uace0 \uacf5\ub3d9\ubc88\uc601\uacfc \ud3c9\ud654\ud1b5\uc77c\uc758 \ubaa9\ud45c\uc5d0 \uc774\ub974\uae30\ub97c \uc18c\ub9dd\ud55c\ub2e4\"\uace0 \uc804\ud588\ub2e4.", "sentence_answer": "\ub2e4\uc74c \ub0a0\uc5d0\ub294 \uae40\uc5ec\uc815\uacfc \uae40\uc601\ub0a8\uc774 \uccad\uc640\ub300 \ub97c \ubc29\ubb38\ud558\uc5ec \ubb38\uc7ac\uc778\uacfc \ud68c\ub3d9\ud558\uc600\ub2e4.", "paragraph_id": "6580792-34-1", "paragraph_question": "question: \ud3c9\ucc3d \uc62c\ub9bc\ud53d\uc744 \ub9de\uc544 \uae40\uc5ec\uc815\uacfc \uae40\uc601\ub0a8\uc774 \ubc29\ub0a8\ud558\uace0 \ub2e4\uc74c \ub0a0 \ubc29\ubb38\ud55c \uacf3\uc740?, context: \ud3c9\ucc3d\uc62c\ub9bc\ud53d\uc744 \ub9de\uc774\ud558\uc5ec 2\uc6d4 9\uc77c\uc5d0 \uae40\uc815\uc740 \uc704\uc6d0\uc7a5\uc758 \uc5ec\ub3d9\uc0dd\uc778 \uae40\uc5ec\uc815 \ub85c\ub3d9\ub2f9 \uc81c1\ubd80\ubd80\uc7a5\uc774 \ud2b9\uc0ac \uc790\uaca9\uc73c\ub85c 2\ubc15 3\uc77c \uac04 \ubc29\ub0a8\ud588\ub2e4. \uae40\uc601\ub0a8 \ucd5c\uace0\uc778\ubbfc\ud68c\uc758 \uc0c1\uc784\uc704\uc6d0\uc7a5, \ucd5c\ud718 \uad6d\uac00\uccb4\uc721\uc9c0\ub3c4\uc704\uc6d0\uc7a5, \ub9ac\uc120\uad8c \uc870\uad6d\ud3c9\ud654\ud1b5\uc77c\uc704\uc6d0\ud68c \uc704\uc6d0\uc7a5\ub3c4 \ub3d9\ud589\ud588\ub2e4. \ud55c\uad6d \uc804\uc7c1 \uc774\ud6c4 \uae40\uc77c\uc131 \uac00\ubb38 \ud608\ud1b5\uc758 \uccab \ubc29\ubb38\uc774\ub77c\ub294 \uc810\uc5d0\uc11c \ud070 \uc758\ubbf8\ub97c \uc0c0\uc9c0\ub9cc \ud55c\uad6d\ub2f9\uc744 \ube44\ub86f\ud55c \ubcf4\uc218 \uc138\ub825\uc740 \ube44\ud575\ud654\ub97c \uc804\uc81c\ub85c \ud55c \ub0a8\ubd81\ub300\ud654\ub97c \uc8fc\uc7a5\ud558\uba70 \uc704\uc7a5 \ud3c9\ud654 \uacf5\uc138\uc5d0 \ud718\ub458\ub9ac\uc9c0 \ub9d0 \uac83\uc744 \uac15\ub825\ud788 \uc694\uad6c\ud588\ub2e4. \ub2e4\uc74c \ub0a0\uc5d0\ub294 \uae40\uc5ec\uc815\uacfc \uae40\uc601\ub0a8\uc774 \uccad\uc640\ub300\ub97c \ubc29\ubb38\ud558\uc5ec \ubb38\uc7ac\uc778\uacfc \ud68c\ub3d9\ud558\uc600\ub2e4. \ubb38\uc7ac\uc778\uc740 \uac74\ubc30\uc0ac\uc5d0\uc11c \"\uc624\ub298 \uc774 \uc790\ub9ac\uc5d0 \uc804 \uc138\uacc4\uc758 \uc774\ubaa9\uc774 \uc9d1\uc911\ub418\uace0 \ub0a8\ubd81\uc5d0 \uac70\ub294 \uae30\ub300\uac00 \ud06c\ub2e4\"\uba70 \ub73b\uae4a\uc740 \uc790\ub9ac\uac00 \ub418\uae30\ub97c \ud76c\ub9dd\ud588\uc73c\uba70 \uae40\uc601\ub0a8\uc740 \"\uc6b0\ub9ac\ub97c \ub530\ub73b\ud558\uace0 \uce5c\uc808\ud558\uac8c \ud658\ub300\ud574 \uc918 \ub3d9\ud3ec\uc758 \uc815\uc744 \ub290\ub080\ub2e4\"\uba70 \ubd81\ub0a8\uad00\uacc4\uc758 \ud68d\uae30\uc801 \uc804\ud658\uc810\uc774 \ub418\uae30\ub97c \uae30\ub300\ud55c\ub2e4\uace0 \uac15\uc870\ud588\ub2e4. \uc774\uc5b4 11\uc77c\uc5d0\ub294 \uc774\ub099\uc5f0\uc774 \uc6cc\ucee4\ud790\ud638\ud154\uc5d0\uc11c \uae40\uc5ec\uc815\uc744 \ub9cc\ub0ac\ub2e4. \uc774\ub099\uc5f0\uc740 2018\ub144 \ub3d9\uacc4 \uc62c\ub9bc\ud53d \ub0a8\ubd81 \ub2e8\uc77c \uc120\uc218\ub2e8\uacfc \uae40\uc5ec\uc815\uc758 \ubc29\ub0a8\uc744 \uc5b8\uae09\ud558\uba70 \uc5bc\ub9c8 \uc804\uae4c\uc9c0 \uc0c1\uc0c1\uc870\ucc28 \uc5b4\ub824\uc6e0\ub358 \uc77c\ub4e4\uc774 \ud604\uc2e4\ub85c \uc774\ub8e8\uc5b4\uc84c\ub2e4\uba70 \"\ub0a8\uacfc \ubd81\ub3c4 \ubaa8\ub4e0 \ub09c\uad00\uc744 \uc774\uae30\uace0 \uacf5\ub3d9\ubc88\uc601\uacfc \ud3c9\ud654\ud1b5\uc77c\uc758 \ubaa9\ud45c\uc5d0 \uc774\ub974\uae30\ub97c \uc18c\ub9dd\ud55c\ub2e4\"\uace0 \uc804\ud588\ub2e4."} {"question": "\uc74c\uc545 \uadf8\ub8f9 \uc5ec\uc790\uce5c\uad6c\uc758 \uc5ec\uc12f \ubc88\uc9f8 \ubbf8\ub2c8 \uc568\ubc94\uc758 \uc81c\ubaa9\uc740?", "paragraph": "4\uc6d4 30\uc77c \uc5ec\uc12f \ubc88\uc9f8 \ubbf8\ub2c8 \uc568\ubc94 \u300aTime for the moon night\u300b\uc744 \ubc1c\ub9e4\ud558\uc600\ub2e4. \ud0c0\uc774\ud2c0 \uace1\uc740 \uc3d8\uc2a4\ubba4\uc9c1\uc758 \ud558\uc6b0\uc2a4 \ud504\ub85c\ub4c0\uc11c \ub178\uc8fc\ud658\uc774 \uc774\uc6d0\uc885\uacfc \uc791\uc5c5\ud55c \u201c\ubc24\u201d\uc73c\ub85c, \uc0ac\ub791\ud558\ub294 \uc774\ub97c \uc0dd\uac01\ud558\uba70 \uace0\ubbfc\uc5d0 \uc816\uc5b4\ub4dc\ub294 \ubc24\ubd80\ud130 \uc0c8\ubcbd\uae4c\uc9c0\uc758 \uc2dc\uac04\uc744 \uc11c\uc815\uc801\uc73c\ub85c \ud45c\ud604\ud55c \uace1\uc774\ub2e4. \u201c\ubc24\u201d\uc740 \uac00\uc628 \uc8fc\uac04 \ub2e4\uc6b4\ub85c\ub4dc \ucc28\ud2b8, \uc568\ubc94 \ucc28\ud2b8, \uc18c\uc15c \ucc28\ud2b8\uc5d0\uc11c 1\uc704\ub97c \uae30\ub85d\ud558\uc600\uc744 \ubfd0 \uc544\ub2c8\ub77c \ube4c\ubcf4\ub4dc \uc568\ubc94\ucc28\ud2b8 6\uc704\ub97c \ucc28\uc9c0\ud558\uc600\uc73c\uba70, \ub300\ud55c\ubbfc\uad6d\uc5d0\uc11c\ub294 \uba5c\ub860, \ubc85\uc2a4, \uc18c\ub9ac\ubc14\ub2e4, \uc9c0\ub2c8, \ub124\uc774\ubc84 \ub4f1 5\uac1c\uc758 \ucc28\ud2b8\uc5d0\uc11c \uc2e4\uc2dc\uac04 1\uc704\uc5d0 \uc62c\ub790\uace0, \ubba4\uc9c1\ube44\ub514\uc624\ub294 \uacf5\uac1c 1\uc8fc\uc77c \ub9cc\uc5d0 \uc870\ud68c\uc218 1300\ub9cc \ud68c\ub97c \ub2ec\uc131\ud558\uc600\ub2e4. 5\uc6d4 3\uc77c \uc5e0\uce74\uc6b4\ud2b8\ub2e4\uc6b4\uc5d0\uc11c \uccab \ucef4\ubc31 \ubb34\ub300\ub97c \uc120\ubcf4\uc778 \uc774\ud6c4, 5\uc6d4 8\uc77c \ub354 \uc1fc, 5\uc6d4 9\uc77c \uc1fc \ucc54\ud53c\uc5b8, 5\uc6d4 10\uc77c \uc5e0\uce74\uc6b4\ud2b8\ub2e4\uc6b4, 5\uc6d4 11\uc77c \ubba4\uc9c1\ubc45\ud06c, 5\uc6d4 12\uc77c \uc74c\uc545\uc911\uc2ec\uc5d0\uc11c 1\uc704\uc5d0 \uc62c\ub790\ub2e4.", "answer": "Time for the moon night", "sentence": "4\uc6d4 30\uc77c \uc5ec\uc12f \ubc88\uc9f8 \ubbf8\ub2c8 \uc568\ubc94 \u300a Time for the moon night \u300b\uc744 \ubc1c\ub9e4\ud558\uc600\ub2e4.", "paragraph_sentence": " 4\uc6d4 30\uc77c \uc5ec\uc12f \ubc88\uc9f8 \ubbf8\ub2c8 \uc568\ubc94 \u300a Time for the moon night \u300b\uc744 \ubc1c\ub9e4\ud558\uc600\ub2e4. \ud0c0\uc774\ud2c0 \uace1\uc740 \uc3d8\uc2a4\ubba4\uc9c1\uc758 \ud558\uc6b0\uc2a4 \ud504\ub85c\ub4c0\uc11c \ub178\uc8fc\ud658\uc774 \uc774\uc6d0\uc885\uacfc \uc791\uc5c5\ud55c \u201c\ubc24\u201d\uc73c\ub85c, \uc0ac\ub791\ud558\ub294 \uc774\ub97c \uc0dd\uac01\ud558\uba70 \uace0\ubbfc\uc5d0 \uc816\uc5b4\ub4dc\ub294 \ubc24\ubd80\ud130 \uc0c8\ubcbd\uae4c\uc9c0\uc758 \uc2dc\uac04\uc744 \uc11c\uc815\uc801\uc73c\ub85c \ud45c\ud604\ud55c \uace1\uc774\ub2e4. \u201c\ubc24\u201d\uc740 \uac00\uc628 \uc8fc\uac04 \ub2e4\uc6b4\ub85c\ub4dc \ucc28\ud2b8, \uc568\ubc94 \ucc28\ud2b8, \uc18c\uc15c \ucc28\ud2b8\uc5d0\uc11c 1\uc704\ub97c \uae30\ub85d\ud558\uc600\uc744 \ubfd0 \uc544\ub2c8\ub77c \ube4c\ubcf4\ub4dc \uc568\ubc94\ucc28\ud2b8 6\uc704\ub97c \ucc28\uc9c0\ud558\uc600\uc73c\uba70, \ub300\ud55c\ubbfc\uad6d\uc5d0\uc11c\ub294 \uba5c\ub860, \ubc85\uc2a4, \uc18c\ub9ac\ubc14\ub2e4, \uc9c0\ub2c8, \ub124\uc774\ubc84 \ub4f1 5\uac1c\uc758 \ucc28\ud2b8\uc5d0\uc11c \uc2e4\uc2dc\uac04 1\uc704\uc5d0 \uc62c\ub790\uace0, \ubba4\uc9c1\ube44\ub514\uc624\ub294 \uacf5\uac1c 1\uc8fc\uc77c \ub9cc\uc5d0 \uc870\ud68c\uc218 1300\ub9cc \ud68c\ub97c \ub2ec\uc131\ud558\uc600\ub2e4. 5\uc6d4 3\uc77c \uc5e0\uce74\uc6b4\ud2b8\ub2e4\uc6b4\uc5d0\uc11c \uccab \ucef4\ubc31 \ubb34\ub300\ub97c \uc120\ubcf4\uc778 \uc774\ud6c4, 5\uc6d4 8\uc77c \ub354 \uc1fc, 5\uc6d4 9\uc77c \uc1fc \ucc54\ud53c\uc5b8, 5\uc6d4 10\uc77c \uc5e0\uce74\uc6b4\ud2b8\ub2e4\uc6b4, 5\uc6d4 11\uc77c \ubba4\uc9c1\ubc45\ud06c, 5\uc6d4 12\uc77c \uc74c\uc545\uc911\uc2ec\uc5d0\uc11c 1\uc704\uc5d0 \uc62c\ub790\ub2e4.", "paragraph_answer": "4\uc6d4 30\uc77c \uc5ec\uc12f \ubc88\uc9f8 \ubbf8\ub2c8 \uc568\ubc94 \u300a Time for the moon night \u300b\uc744 \ubc1c\ub9e4\ud558\uc600\ub2e4. \ud0c0\uc774\ud2c0 \uace1\uc740 \uc3d8\uc2a4\ubba4\uc9c1\uc758 \ud558\uc6b0\uc2a4 \ud504\ub85c\ub4c0\uc11c \ub178\uc8fc\ud658\uc774 \uc774\uc6d0\uc885\uacfc \uc791\uc5c5\ud55c \u201c\ubc24\u201d\uc73c\ub85c, \uc0ac\ub791\ud558\ub294 \uc774\ub97c \uc0dd\uac01\ud558\uba70 \uace0\ubbfc\uc5d0 \uc816\uc5b4\ub4dc\ub294 \ubc24\ubd80\ud130 \uc0c8\ubcbd\uae4c\uc9c0\uc758 \uc2dc\uac04\uc744 \uc11c\uc815\uc801\uc73c\ub85c \ud45c\ud604\ud55c \uace1\uc774\ub2e4. \u201c\ubc24\u201d\uc740 \uac00\uc628 \uc8fc\uac04 \ub2e4\uc6b4\ub85c\ub4dc \ucc28\ud2b8, \uc568\ubc94 \ucc28\ud2b8, \uc18c\uc15c \ucc28\ud2b8\uc5d0\uc11c 1\uc704\ub97c \uae30\ub85d\ud558\uc600\uc744 \ubfd0 \uc544\ub2c8\ub77c \ube4c\ubcf4\ub4dc \uc568\ubc94\ucc28\ud2b8 6\uc704\ub97c \ucc28\uc9c0\ud558\uc600\uc73c\uba70, \ub300\ud55c\ubbfc\uad6d\uc5d0\uc11c\ub294 \uba5c\ub860, \ubc85\uc2a4, \uc18c\ub9ac\ubc14\ub2e4, \uc9c0\ub2c8, \ub124\uc774\ubc84 \ub4f1 5\uac1c\uc758 \ucc28\ud2b8\uc5d0\uc11c \uc2e4\uc2dc\uac04 1\uc704\uc5d0 \uc62c\ub790\uace0, \ubba4\uc9c1\ube44\ub514\uc624\ub294 \uacf5\uac1c 1\uc8fc\uc77c \ub9cc\uc5d0 \uc870\ud68c\uc218 1300\ub9cc \ud68c\ub97c \ub2ec\uc131\ud558\uc600\ub2e4. 5\uc6d4 3\uc77c \uc5e0\uce74\uc6b4\ud2b8\ub2e4\uc6b4\uc5d0\uc11c \uccab \ucef4\ubc31 \ubb34\ub300\ub97c \uc120\ubcf4\uc778 \uc774\ud6c4, 5\uc6d4 8\uc77c \ub354 \uc1fc, 5\uc6d4 9\uc77c \uc1fc \ucc54\ud53c\uc5b8, 5\uc6d4 10\uc77c \uc5e0\uce74\uc6b4\ud2b8\ub2e4\uc6b4, 5\uc6d4 11\uc77c \ubba4\uc9c1\ubc45\ud06c, 5\uc6d4 12\uc77c \uc74c\uc545\uc911\uc2ec\uc5d0\uc11c 1\uc704\uc5d0 \uc62c\ub790\ub2e4.", "sentence_answer": "4\uc6d4 30\uc77c \uc5ec\uc12f \ubc88\uc9f8 \ubbf8\ub2c8 \uc568\ubc94 \u300a Time for the moon night \u300b\uc744 \ubc1c\ub9e4\ud558\uc600\ub2e4.", "paragraph_id": "6657367-4-0", "paragraph_question": "question: \uc74c\uc545 \uadf8\ub8f9 \uc5ec\uc790\uce5c\uad6c\uc758 \uc5ec\uc12f \ubc88\uc9f8 \ubbf8\ub2c8 \uc568\ubc94\uc758 \uc81c\ubaa9\uc740?, context: 4\uc6d4 30\uc77c \uc5ec\uc12f \ubc88\uc9f8 \ubbf8\ub2c8 \uc568\ubc94 \u300aTime for the moon night\u300b\uc744 \ubc1c\ub9e4\ud558\uc600\ub2e4. \ud0c0\uc774\ud2c0 \uace1\uc740 \uc3d8\uc2a4\ubba4\uc9c1\uc758 \ud558\uc6b0\uc2a4 \ud504\ub85c\ub4c0\uc11c \ub178\uc8fc\ud658\uc774 \uc774\uc6d0\uc885\uacfc \uc791\uc5c5\ud55c \u201c\ubc24\u201d\uc73c\ub85c, \uc0ac\ub791\ud558\ub294 \uc774\ub97c \uc0dd\uac01\ud558\uba70 \uace0\ubbfc\uc5d0 \uc816\uc5b4\ub4dc\ub294 \ubc24\ubd80\ud130 \uc0c8\ubcbd\uae4c\uc9c0\uc758 \uc2dc\uac04\uc744 \uc11c\uc815\uc801\uc73c\ub85c \ud45c\ud604\ud55c \uace1\uc774\ub2e4. \u201c\ubc24\u201d\uc740 \uac00\uc628 \uc8fc\uac04 \ub2e4\uc6b4\ub85c\ub4dc \ucc28\ud2b8, \uc568\ubc94 \ucc28\ud2b8, \uc18c\uc15c \ucc28\ud2b8\uc5d0\uc11c 1\uc704\ub97c \uae30\ub85d\ud558\uc600\uc744 \ubfd0 \uc544\ub2c8\ub77c \ube4c\ubcf4\ub4dc \uc568\ubc94\ucc28\ud2b8 6\uc704\ub97c \ucc28\uc9c0\ud558\uc600\uc73c\uba70, \ub300\ud55c\ubbfc\uad6d\uc5d0\uc11c\ub294 \uba5c\ub860, \ubc85\uc2a4, \uc18c\ub9ac\ubc14\ub2e4, \uc9c0\ub2c8, \ub124\uc774\ubc84 \ub4f1 5\uac1c\uc758 \ucc28\ud2b8\uc5d0\uc11c \uc2e4\uc2dc\uac04 1\uc704\uc5d0 \uc62c\ub790\uace0, \ubba4\uc9c1\ube44\ub514\uc624\ub294 \uacf5\uac1c 1\uc8fc\uc77c \ub9cc\uc5d0 \uc870\ud68c\uc218 1300\ub9cc \ud68c\ub97c \ub2ec\uc131\ud558\uc600\ub2e4. 5\uc6d4 3\uc77c \uc5e0\uce74\uc6b4\ud2b8\ub2e4\uc6b4\uc5d0\uc11c \uccab \ucef4\ubc31 \ubb34\ub300\ub97c \uc120\ubcf4\uc778 \uc774\ud6c4, 5\uc6d4 8\uc77c \ub354 \uc1fc, 5\uc6d4 9\uc77c \uc1fc \ucc54\ud53c\uc5b8, 5\uc6d4 10\uc77c \uc5e0\uce74\uc6b4\ud2b8\ub2e4\uc6b4, 5\uc6d4 11\uc77c \ubba4\uc9c1\ubc45\ud06c, 5\uc6d4 12\uc77c \uc74c\uc545\uc911\uc2ec\uc5d0\uc11c 1\uc704\uc5d0 \uc62c\ub790\ub2e4."} {"question": "\ubd80\uc870\uc885\uc0ac\uc758 \ucd5c\ub300 \uadfc\ubb34\uc2dc\uac04\uc744 \uc9c0\ud0a4\uae30 \uc704\ud574 \ub300\uccb4 \ud22c\uc785\ub41c \ud56d\uacf5\uae30\uc758 \uae30\uccb4 \ub4f1\ub85d \ubc88\ud638\ub294 \ubb34\uc5c7\uc778\uac00?", "paragraph": "1420\ud3b8\uc740 \uc911\ubd80 \ud45c\uc900\uc2dc 20:28\ubd84\uc5d0 \ub308\ub7ec\uc2a4 \ud3ec\ud2b8\uc6cc\uc2a4\ub97c \ucd9c\ubc1c\ud558\uc5ec 21:41\ubd84\uc5d0 \ub9ac\ud2c0 \ub85d\uc5d0 \ub3c4\ucc29\ud558\ub294 \ube44\ud589\uc774\uc5c8\ub2e4. \ud558\uc9c0\ub9cc \ube44\ud589 \uc2b9\ubb34\uc6d0\ub4e4\uc740 1420\ud3b8\uc5d0 \ud0d1\uc2b9\ud558\uae30 \uc804 \uc870\uc5b8\uc744 \ubc1b\uc558\ub294\ub370 \ucd9c\ubc1c\uc774 \uc9c0\uc5f0\ub420 \uac83\uc774\uba70, \ubbf8\uad6d \uae30\uc0c1\uccad\uc5d0\uc11c \ubc1c\ud45c\ud55c \ud56d\uacf5 \uae30\uc0c1\uc5d0\uc11c\ub294 1420\ud3b8\uc758 \uacc4\ud68d\ub41c \ube44\ud589 \uacbd\ub85c\ub97c \ub530\ub77c \uaca9\uc2ec\ud55c \ub1cc\uc6b0\uac00 \ud45c\uc2dc\ub418\uace0 \uc788\ub2e4\ub294 \ub0b4\uc6a9\uc774\uc5c8\ub2e4. \uc545\ucc9c\ud6c4\ub85c \uc778\ud574 1420\ud3b8\uc73c\ub85c \uc774\ub959\ud560 \ud56d\uacf5\uae30\uc758 \ub308\ub7ec\uc2a4 \ud3ec\ud2b8\uc6cc\uc2a4 \ub3c4\ucc29\uc740 \uc9c0\uc5f0\ub418\uc5c8\uc73c\uba70 \ube44\ud589 \uc2b9\ubb34\uc6d0\ub4e4\uc740 \ud0d1\uc2b9 \uac8c\uc774\ud2b8 \uc55e\uc5d0\uc11c \uacc4\uc18d \uae30\ub2e4\ub824\uc57c \ud588\ub2e4. \ud56d\uacf5 \uaddc\uc815\uc5d0 \ub530\ub974\uba74 \uc870\uc885\uc0ac\uc758 \ucd5c\ub300 \uadfc\ubb34\uc2dc\uac04\uc744 14\uc2dc\uac04\uc774\uace0 1420\ud3b8\uc740 \uadf8\ub4e4\uc758 \uadfc\ubb34\uc77c \ub9c8\uc9c0\ub9c9 \ube44\ud589 \uc2a4\ucf00\uc904\uc774\uc5c8\ub2e4. \ubd80\uc870\uc885\uc0ac\ub294 \ub9cc\uc57d 1420\ud3b8\uc774 23:16\ubd84\uae4c\uc9c0 \ucd9c\ubc1c\ud558\uc9c0 \ubabb\ud560 \uacbd\uc6b0 \uadf8\ub4e4\uc758 \uaddc\uc815\ub41c \uc2dc\uac04\uc744 \ucd08\uacfc\ud560 \uac83\uc774\ub77c\uace0 \ud56d\uacf5\uc0ac\uc758 \ube44\ud589 \uc6b4\ud56d \uad00\ub9ac\uc790\uc5d0 \ud1b5\ubcf4\ud588\ub2e4. \uadf8\ub9ac\ud558\uc5ec \ud56d\uacf5\uae30\ub294 \ub2e4\ub978 MD-80\uc73c\ub85c \uad50\uccb4\ub418\uc5c8\uc73c\uba70 \uae30\uccb4 \ub4f1\ub85d \ubc88\ud638\ub294 N215AA, 22:40\ubd84\uc5d0 \ub308\ub7ec\uc2a4 \ud3ec\ud2b8\uc6cc\uc2a4 \ucd9c\ubc1c\uc744 \ud5c8\uac00\ud588\ub2e4.", "answer": "N215AA", "sentence": "\uae30\uccb4 \ub4f1\ub85d \ubc88\ud638\ub294 N215AA ,", "paragraph_sentence": "1420\ud3b8\uc740 \uc911\ubd80 \ud45c\uc900\uc2dc 20:28\ubd84\uc5d0 \ub308\ub7ec\uc2a4 \ud3ec\ud2b8\uc6cc\uc2a4\ub97c \ucd9c\ubc1c\ud558\uc5ec 21:41\ubd84\uc5d0 \ub9ac\ud2c0 \ub85d\uc5d0 \ub3c4\ucc29\ud558\ub294 \ube44\ud589\uc774\uc5c8\ub2e4. \ud558\uc9c0\ub9cc \ube44\ud589 \uc2b9\ubb34\uc6d0\ub4e4\uc740 1420\ud3b8\uc5d0 \ud0d1\uc2b9\ud558\uae30 \uc804 \uc870\uc5b8\uc744 \ubc1b\uc558\ub294\ub370 \ucd9c\ubc1c\uc774 \uc9c0\uc5f0\ub420 \uac83\uc774\uba70, \ubbf8\uad6d \uae30\uc0c1\uccad\uc5d0\uc11c \ubc1c\ud45c\ud55c \ud56d\uacf5 \uae30\uc0c1\uc5d0\uc11c\ub294 1420\ud3b8\uc758 \uacc4\ud68d\ub41c \ube44\ud589 \uacbd\ub85c\ub97c \ub530\ub77c \uaca9\uc2ec\ud55c \ub1cc\uc6b0\uac00 \ud45c\uc2dc\ub418\uace0 \uc788\ub2e4\ub294 \ub0b4\uc6a9\uc774\uc5c8\ub2e4. \uc545\ucc9c\ud6c4\ub85c \uc778\ud574 1420\ud3b8\uc73c\ub85c \uc774\ub959\ud560 \ud56d\uacf5\uae30\uc758 \ub308\ub7ec\uc2a4 \ud3ec\ud2b8\uc6cc\uc2a4 \ub3c4\ucc29\uc740 \uc9c0\uc5f0\ub418\uc5c8\uc73c\uba70 \ube44\ud589 \uc2b9\ubb34\uc6d0\ub4e4\uc740 \ud0d1\uc2b9 \uac8c\uc774\ud2b8 \uc55e\uc5d0\uc11c \uacc4\uc18d \uae30\ub2e4\ub824\uc57c \ud588\ub2e4. \ud56d\uacf5 \uaddc\uc815\uc5d0 \ub530\ub974\uba74 \uc870\uc885\uc0ac\uc758 \ucd5c\ub300 \uadfc\ubb34\uc2dc\uac04\uc744 14\uc2dc\uac04\uc774\uace0 1420\ud3b8\uc740 \uadf8\ub4e4\uc758 \uadfc\ubb34\uc77c \ub9c8\uc9c0\ub9c9 \ube44\ud589 \uc2a4\ucf00\uc904\uc774\uc5c8\ub2e4. \ubd80\uc870\uc885\uc0ac\ub294 \ub9cc\uc57d 1420\ud3b8\uc774 23:16\ubd84\uae4c\uc9c0 \ucd9c\ubc1c\ud558\uc9c0 \ubabb\ud560 \uacbd\uc6b0 \uadf8\ub4e4\uc758 \uaddc\uc815\ub41c \uc2dc\uac04\uc744 \ucd08\uacfc\ud560 \uac83\uc774\ub77c\uace0 \ud56d\uacf5\uc0ac\uc758 \ube44\ud589 \uc6b4\ud56d \uad00\ub9ac\uc790\uc5d0 \ud1b5\ubcf4\ud588\ub2e4. \uadf8\ub9ac\ud558\uc5ec \ud56d\uacf5\uae30\ub294 \ub2e4\ub978 MD-80\uc73c\ub85c \uad50\uccb4\ub418\uc5c8\uc73c\uba70 \uae30\uccb4 \ub4f1\ub85d \ubc88\ud638\ub294 N215AA , 22:40\ubd84\uc5d0 \ub308\ub7ec\uc2a4 \ud3ec\ud2b8\uc6cc\uc2a4 \ucd9c\ubc1c\uc744 \ud5c8\uac00\ud588\ub2e4.", "paragraph_answer": "1420\ud3b8\uc740 \uc911\ubd80 \ud45c\uc900\uc2dc 20:28\ubd84\uc5d0 \ub308\ub7ec\uc2a4 \ud3ec\ud2b8\uc6cc\uc2a4\ub97c \ucd9c\ubc1c\ud558\uc5ec 21:41\ubd84\uc5d0 \ub9ac\ud2c0 \ub85d\uc5d0 \ub3c4\ucc29\ud558\ub294 \ube44\ud589\uc774\uc5c8\ub2e4. \ud558\uc9c0\ub9cc \ube44\ud589 \uc2b9\ubb34\uc6d0\ub4e4\uc740 1420\ud3b8\uc5d0 \ud0d1\uc2b9\ud558\uae30 \uc804 \uc870\uc5b8\uc744 \ubc1b\uc558\ub294\ub370 \ucd9c\ubc1c\uc774 \uc9c0\uc5f0\ub420 \uac83\uc774\uba70, \ubbf8\uad6d \uae30\uc0c1\uccad\uc5d0\uc11c \ubc1c\ud45c\ud55c \ud56d\uacf5 \uae30\uc0c1\uc5d0\uc11c\ub294 1420\ud3b8\uc758 \uacc4\ud68d\ub41c \ube44\ud589 \uacbd\ub85c\ub97c \ub530\ub77c \uaca9\uc2ec\ud55c \ub1cc\uc6b0\uac00 \ud45c\uc2dc\ub418\uace0 \uc788\ub2e4\ub294 \ub0b4\uc6a9\uc774\uc5c8\ub2e4. \uc545\ucc9c\ud6c4\ub85c \uc778\ud574 1420\ud3b8\uc73c\ub85c \uc774\ub959\ud560 \ud56d\uacf5\uae30\uc758 \ub308\ub7ec\uc2a4 \ud3ec\ud2b8\uc6cc\uc2a4 \ub3c4\ucc29\uc740 \uc9c0\uc5f0\ub418\uc5c8\uc73c\uba70 \ube44\ud589 \uc2b9\ubb34\uc6d0\ub4e4\uc740 \ud0d1\uc2b9 \uac8c\uc774\ud2b8 \uc55e\uc5d0\uc11c \uacc4\uc18d \uae30\ub2e4\ub824\uc57c \ud588\ub2e4. \ud56d\uacf5 \uaddc\uc815\uc5d0 \ub530\ub974\uba74 \uc870\uc885\uc0ac\uc758 \ucd5c\ub300 \uadfc\ubb34\uc2dc\uac04\uc744 14\uc2dc\uac04\uc774\uace0 1420\ud3b8\uc740 \uadf8\ub4e4\uc758 \uadfc\ubb34\uc77c \ub9c8\uc9c0\ub9c9 \ube44\ud589 \uc2a4\ucf00\uc904\uc774\uc5c8\ub2e4. \ubd80\uc870\uc885\uc0ac\ub294 \ub9cc\uc57d 1420\ud3b8\uc774 23:16\ubd84\uae4c\uc9c0 \ucd9c\ubc1c\ud558\uc9c0 \ubabb\ud560 \uacbd\uc6b0 \uadf8\ub4e4\uc758 \uaddc\uc815\ub41c \uc2dc\uac04\uc744 \ucd08\uacfc\ud560 \uac83\uc774\ub77c\uace0 \ud56d\uacf5\uc0ac\uc758 \ube44\ud589 \uc6b4\ud56d \uad00\ub9ac\uc790\uc5d0 \ud1b5\ubcf4\ud588\ub2e4. \uadf8\ub9ac\ud558\uc5ec \ud56d\uacf5\uae30\ub294 \ub2e4\ub978 MD-80\uc73c\ub85c \uad50\uccb4\ub418\uc5c8\uc73c\uba70 \uae30\uccb4 \ub4f1\ub85d \ubc88\ud638\ub294 N215AA , 22:40\ubd84\uc5d0 \ub308\ub7ec\uc2a4 \ud3ec\ud2b8\uc6cc\uc2a4 \ucd9c\ubc1c\uc744 \ud5c8\uac00\ud588\ub2e4.", "sentence_answer": "\uae30\uccb4 \ub4f1\ub85d \ubc88\ud638\ub294 N215AA ,", "paragraph_id": "6464010-0-2", "paragraph_question": "question: \ubd80\uc870\uc885\uc0ac\uc758 \ucd5c\ub300 \uadfc\ubb34\uc2dc\uac04\uc744 \uc9c0\ud0a4\uae30 \uc704\ud574 \ub300\uccb4 \ud22c\uc785\ub41c \ud56d\uacf5\uae30\uc758 \uae30\uccb4 \ub4f1\ub85d \ubc88\ud638\ub294 \ubb34\uc5c7\uc778\uac00?, context: 1420\ud3b8\uc740 \uc911\ubd80 \ud45c\uc900\uc2dc 20:28\ubd84\uc5d0 \ub308\ub7ec\uc2a4 \ud3ec\ud2b8\uc6cc\uc2a4\ub97c \ucd9c\ubc1c\ud558\uc5ec 21:41\ubd84\uc5d0 \ub9ac\ud2c0 \ub85d\uc5d0 \ub3c4\ucc29\ud558\ub294 \ube44\ud589\uc774\uc5c8\ub2e4. \ud558\uc9c0\ub9cc \ube44\ud589 \uc2b9\ubb34\uc6d0\ub4e4\uc740 1420\ud3b8\uc5d0 \ud0d1\uc2b9\ud558\uae30 \uc804 \uc870\uc5b8\uc744 \ubc1b\uc558\ub294\ub370 \ucd9c\ubc1c\uc774 \uc9c0\uc5f0\ub420 \uac83\uc774\uba70, \ubbf8\uad6d \uae30\uc0c1\uccad\uc5d0\uc11c \ubc1c\ud45c\ud55c \ud56d\uacf5 \uae30\uc0c1\uc5d0\uc11c\ub294 1420\ud3b8\uc758 \uacc4\ud68d\ub41c \ube44\ud589 \uacbd\ub85c\ub97c \ub530\ub77c \uaca9\uc2ec\ud55c \ub1cc\uc6b0\uac00 \ud45c\uc2dc\ub418\uace0 \uc788\ub2e4\ub294 \ub0b4\uc6a9\uc774\uc5c8\ub2e4. \uc545\ucc9c\ud6c4\ub85c \uc778\ud574 1420\ud3b8\uc73c\ub85c \uc774\ub959\ud560 \ud56d\uacf5\uae30\uc758 \ub308\ub7ec\uc2a4 \ud3ec\ud2b8\uc6cc\uc2a4 \ub3c4\ucc29\uc740 \uc9c0\uc5f0\ub418\uc5c8\uc73c\uba70 \ube44\ud589 \uc2b9\ubb34\uc6d0\ub4e4\uc740 \ud0d1\uc2b9 \uac8c\uc774\ud2b8 \uc55e\uc5d0\uc11c \uacc4\uc18d \uae30\ub2e4\ub824\uc57c \ud588\ub2e4. \ud56d\uacf5 \uaddc\uc815\uc5d0 \ub530\ub974\uba74 \uc870\uc885\uc0ac\uc758 \ucd5c\ub300 \uadfc\ubb34\uc2dc\uac04\uc744 14\uc2dc\uac04\uc774\uace0 1420\ud3b8\uc740 \uadf8\ub4e4\uc758 \uadfc\ubb34\uc77c \ub9c8\uc9c0\ub9c9 \ube44\ud589 \uc2a4\ucf00\uc904\uc774\uc5c8\ub2e4. \ubd80\uc870\uc885\uc0ac\ub294 \ub9cc\uc57d 1420\ud3b8\uc774 23:16\ubd84\uae4c\uc9c0 \ucd9c\ubc1c\ud558\uc9c0 \ubabb\ud560 \uacbd\uc6b0 \uadf8\ub4e4\uc758 \uaddc\uc815\ub41c \uc2dc\uac04\uc744 \ucd08\uacfc\ud560 \uac83\uc774\ub77c\uace0 \ud56d\uacf5\uc0ac\uc758 \ube44\ud589 \uc6b4\ud56d \uad00\ub9ac\uc790\uc5d0 \ud1b5\ubcf4\ud588\ub2e4. \uadf8\ub9ac\ud558\uc5ec \ud56d\uacf5\uae30\ub294 \ub2e4\ub978 MD-80\uc73c\ub85c \uad50\uccb4\ub418\uc5c8\uc73c\uba70 \uae30\uccb4 \ub4f1\ub85d \ubc88\ud638\ub294 N215AA, 22:40\ubd84\uc5d0 \ub308\ub7ec\uc2a4 \ud3ec\ud2b8\uc6cc\uc2a4 \ucd9c\ubc1c\uc744 \ud5c8\uac00\ud588\ub2e4."} {"question": "\uc870\uc591\ud638 \ud55c\uc9c4\uadf8\ub8f9 \ud68c\uc7a5 \uc77c\uac00\uac00 \uc9c0\ubd84 25%\ub97c \ubcf4\uc720\ud55c \ud55c\uc9c4\uadf8\ub8f9 \uc9c0\uc8fc\ud68c\uc0ac\ub294?", "paragraph": "\ud55c\uc9c4\uce7c\uc740 \uc870\uc591\ud638 \ud55c\uc9c4\uadf8\ub8f9 \ud68c\uc7a5 \uc77c\uac00\uac00 \uc9c0\ubd84 25%\ub97c \ubcf4\uc720\ud55c \ud55c\uc9c4\uadf8\ub8f9 \uc9c0\uc8fc\ud68c\uc0ac\uc774\uba70, \ub300\ud55c\ud56d\uacf5\uc740 \ub9e4\ub144 \ub9e4\ucd9c\uc561\uc758 0.25%(2017\ub144 \uae30\uc900 300\uc5b5\uc6d0)\ub97c \ud55c\uc9c4\uce7c\uc5d0 '\ub300\ud55c\ud56d\uacf5 \uc0c1\ud45c\uad8c' \uc0ac\uc6a9\ub8cc\ub85c \ub0a9\ubd80\ud55c\ub2e4. \ud2b9\ud5c8\uccad\uc5d0 \ub530\ub974\uba74, \ub300\ud55c\ud56d\uacf5\uc740 2013\ub144 8\uc6d4 6\uc77c '\ub300\ud55c\ud56d\uacf5'\uacfc 'Korean Air' \uc774\ub984, \ud0dc\uadf9\ubb38\uc591\uc758 \ub85c\uace0 \ub4f1 \uc0c1\ud45c\uad8c \uc804\ubd80\uc640 \uc9c4\uc5d0\uc5b4\u00b7 \uce7c\ud638\ud154 \ub124\ud2b8\uc6cc\ud06c \ub4f1\uc758 \uacc4\uc5f4\uc0ac \uc8fc\uc2dd, \uc11c\uc6b8 \uc11c\uc18c\ubb38\uc0ac\uc625, \ud604\uae08 \ubc0f \uc608\uae08 1000\uc5b5\uc6d0\uc744 \ud55c\uc9c4\uce7c\uc5d0 \ub118\uaca8\uc92c\ub2e4. \ud55c\uc9c4\uce7c\uc740 \ub300\ud55c\ud56d\uacf5\uc73c\ub85c\ubd80\ud130 \uc0c1\ud45c\uad8c\uc744 \ub118\uaca8\ubc1b\uae30 \ub2f7\uc0c8 \uc804\uc778 2013\ub144 8\uc6d4 1\uc77c \ub300\ud55c\ud56d\uacf5\uc73c\ub85c\ubd80\ud130 \uacc4\uc5f4 \ubd84\ub9ac\ub41c, \ud55c\uc9c4\uadf8\ub8f9\uc758 \uc9c0\uc8fc \ud68c\uc0ac\ub2e4. \uc870\uc591\ud638 \ud68c\uc7a5\uacfc \uc7a5\ub140 \uc870\ud604\uc544, \uc7a5\ub0a8 \uc870\uc6d0\ud0dc, \ucc28\ub140 \uc870\ud604\ubbfc\uc740 \ub300\ud55c\ud56d\uacf5\uc758 \uc8fc\uc2dd\uc744 \ubaa8\ub450 \ub118\uae30\uace0 \ud55c\uc9c4\uce7c\uc758 \uc8fc\uc2dd\uc744 \ubc1b\uc558\ub2e4. \uc774\ud6c4 \uc870\uc591\ud638 \ud68c\uc7a5 \uc77c\uac00\ub294 2014\ub144 ~ 2017\ub144 \uae4c\uc9c0 \uc138 \ubc88\uc5d0 \uac78\uccd0 37\uc5b5\uc6d0\uc758 \ud604\uae08 \ubc30\ub2f9\uc744 \ubc1b\uc558\ub2e4. \uc624\ub108 \uc77c\uac00\uac00 \ud55c\uc9c4\uce7c\uc5d0\uc11c \uc218\uc2ed\uc5b5\uc6d0\ub300\uc758 \ubc30\ub2f9\uae08\uc744 \ubc1b\uc558\ub358 \uc774 \uc2dc\uae30, \ub300\ud55c\ud56d\uacf5\uc740 \uc800\uc720\uac00\u00b7\ud56d\uacf5\uc218\uc694 \uc99d\uac00\uac00 \uacb9\uce5c \ud638\ud669\uae30\uc600\uc74c\uc5d0\ub3c4 \ud55c\uc9c4\ud574\uc6b4 \ubd80\uc2e4\ucc98\ub9ac \ubb38\uc81c\ub97c \ub5a0\uc548\uc73c\uba70 \uace4\ub780\uc744 \uacaa\uac8c \ub418\uc5c8\ub2e4. \uc0ac\uc815\uc774 \uc5b4\ub824\uc6cc \uc218 \ub144\uac04 \uc8fc\uc8fc\ub4e4\uc5d0\uac8c \ubc30\ub2f9\uae08\uc744 \uc9c0\uae09\ud558\uc9c0 \ubabb\ud558\ub358 \ub300\ud55c\ud56d\uacf5\uc774 2014\ub144 ~ 2017\ub144\uae4c\uc9c0 \uc624\ub108 \uc77c\uac00\uc758 \uc9c0\ubd84\uc774 \ubab0\ub824\uc788\ub294 \ud55c\uc9c4\uce7c\uc5d0 \uc9c0\ubd88\ud55c '\ub300\ud55c\ud56d\uacf5 \uc0c1\ud45c\uad8c \uc0ac\uc6a9\ub8cc'\uc640 '\uac74\ubb3c \uc784\ub300\ub8cc'\ub294 1,229\uc5b5\uc6d0\uc73c\ub85c \ubc1d\ud600\uc84c\ub2e4.", "answer": "\ud55c\uc9c4\uce7c", "sentence": "\ud55c\uc9c4\uce7c \uc740 \uc870\uc591\ud638 \ud55c\uc9c4\uadf8\ub8f9 \ud68c\uc7a5 \uc77c\uac00\uac00 \uc9c0\ubd84 25%\ub97c \ubcf4\uc720\ud55c \ud55c\uc9c4\uadf8\ub8f9 \uc9c0\uc8fc\ud68c\uc0ac\uc774\uba70, \ub300\ud55c\ud56d\uacf5\uc740 \ub9e4\ub144 \ub9e4\ucd9c\uc561\uc758 0.25%(2017\ub144 \uae30\uc900 300\uc5b5\uc6d0)\ub97c \ud55c\uc9c4\uce7c\uc5d0 '\ub300\ud55c\ud56d\uacf5 \uc0c1\ud45c\uad8c' \uc0ac\uc6a9\ub8cc\ub85c \ub0a9\ubd80\ud55c\ub2e4.", "paragraph_sentence": " \ud55c\uc9c4\uce7c \uc740 \uc870\uc591\ud638 \ud55c\uc9c4\uadf8\ub8f9 \ud68c\uc7a5 \uc77c\uac00\uac00 \uc9c0\ubd84 25%\ub97c \ubcf4\uc720\ud55c \ud55c\uc9c4\uadf8\ub8f9 \uc9c0\uc8fc\ud68c\uc0ac\uc774\uba70, \ub300\ud55c\ud56d\uacf5\uc740 \ub9e4\ub144 \ub9e4\ucd9c\uc561\uc758 0.25%(2017\ub144 \uae30\uc900 300\uc5b5\uc6d0)\ub97c \ud55c\uc9c4\uce7c\uc5d0 '\ub300\ud55c\ud56d\uacf5 \uc0c1\ud45c\uad8c' \uc0ac\uc6a9\ub8cc\ub85c \ub0a9\ubd80\ud55c\ub2e4. \ud2b9\ud5c8\uccad\uc5d0 \ub530\ub974\uba74, \ub300\ud55c\ud56d\uacf5\uc740 2013\ub144 8\uc6d4 6\uc77c '\ub300\ud55c\ud56d\uacf5'\uacfc 'Korean Air' \uc774\ub984, \ud0dc\uadf9\ubb38\uc591\uc758 \ub85c\uace0 \ub4f1 \uc0c1\ud45c\uad8c \uc804\ubd80\uc640 \uc9c4\uc5d0\uc5b4\u00b7 \uce7c\ud638\ud154 \ub124\ud2b8\uc6cc\ud06c \ub4f1\uc758 \uacc4\uc5f4\uc0ac \uc8fc\uc2dd, \uc11c\uc6b8 \uc11c\uc18c\ubb38\uc0ac\uc625, \ud604\uae08 \ubc0f \uc608\uae08 1000\uc5b5\uc6d0\uc744 \ud55c\uc9c4\uce7c\uc5d0 \ub118\uaca8\uc92c\ub2e4. \ud55c\uc9c4\uce7c\uc740 \ub300\ud55c\ud56d\uacf5\uc73c\ub85c\ubd80\ud130 \uc0c1\ud45c\uad8c\uc744 \ub118\uaca8\ubc1b\uae30 \ub2f7\uc0c8 \uc804\uc778 2013\ub144 8\uc6d4 1\uc77c \ub300\ud55c\ud56d\uacf5\uc73c\ub85c\ubd80\ud130 \uacc4\uc5f4 \ubd84\ub9ac\ub41c, \ud55c\uc9c4\uadf8\ub8f9\uc758 \uc9c0\uc8fc \ud68c\uc0ac\ub2e4. \uc870\uc591\ud638 \ud68c\uc7a5\uacfc \uc7a5\ub140 \uc870\ud604\uc544, \uc7a5\ub0a8 \uc870\uc6d0\ud0dc, \ucc28\ub140 \uc870\ud604\ubbfc\uc740 \ub300\ud55c\ud56d\uacf5\uc758 \uc8fc\uc2dd\uc744 \ubaa8\ub450 \ub118\uae30\uace0 \ud55c\uc9c4\uce7c\uc758 \uc8fc\uc2dd\uc744 \ubc1b\uc558\ub2e4. \uc774\ud6c4 \uc870\uc591\ud638 \ud68c\uc7a5 \uc77c\uac00\ub294 2014\ub144 ~ 2017\ub144 \uae4c\uc9c0 \uc138 \ubc88\uc5d0 \uac78\uccd0 37\uc5b5\uc6d0\uc758 \ud604\uae08 \ubc30\ub2f9\uc744 \ubc1b\uc558\ub2e4. \uc624\ub108 \uc77c\uac00\uac00 \ud55c\uc9c4\uce7c\uc5d0\uc11c \uc218\uc2ed\uc5b5\uc6d0\ub300\uc758 \ubc30\ub2f9\uae08\uc744 \ubc1b\uc558\ub358 \uc774 \uc2dc\uae30, \ub300\ud55c\ud56d\uacf5\uc740 \uc800\uc720\uac00\u00b7\ud56d\uacf5\uc218\uc694 \uc99d\uac00\uac00 \uacb9\uce5c \ud638\ud669\uae30\uc600\uc74c\uc5d0\ub3c4 \ud55c\uc9c4\ud574\uc6b4 \ubd80\uc2e4\ucc98\ub9ac \ubb38\uc81c\ub97c \ub5a0\uc548\uc73c\uba70 \uace4\ub780\uc744 \uacaa\uac8c \ub418\uc5c8\ub2e4. \uc0ac\uc815\uc774 \uc5b4\ub824\uc6cc \uc218 \ub144\uac04 \uc8fc\uc8fc\ub4e4\uc5d0\uac8c \ubc30\ub2f9\uae08\uc744 \uc9c0\uae09\ud558\uc9c0 \ubabb\ud558\ub358 \ub300\ud55c\ud56d\uacf5\uc774 2014\ub144 ~ 2017\ub144\uae4c\uc9c0 \uc624\ub108 \uc77c\uac00\uc758 \uc9c0\ubd84\uc774 \ubab0\ub824\uc788\ub294 \ud55c\uc9c4\uce7c\uc5d0 \uc9c0\ubd88\ud55c '\ub300\ud55c\ud56d\uacf5 \uc0c1\ud45c\uad8c \uc0ac\uc6a9\ub8cc'\uc640 '\uac74\ubb3c \uc784\ub300\ub8cc'\ub294 1,229\uc5b5\uc6d0\uc73c\ub85c \ubc1d\ud600\uc84c\ub2e4.", "paragraph_answer": " \ud55c\uc9c4\uce7c \uc740 \uc870\uc591\ud638 \ud55c\uc9c4\uadf8\ub8f9 \ud68c\uc7a5 \uc77c\uac00\uac00 \uc9c0\ubd84 25%\ub97c \ubcf4\uc720\ud55c \ud55c\uc9c4\uadf8\ub8f9 \uc9c0\uc8fc\ud68c\uc0ac\uc774\uba70, \ub300\ud55c\ud56d\uacf5\uc740 \ub9e4\ub144 \ub9e4\ucd9c\uc561\uc758 0.25%(2017\ub144 \uae30\uc900 300\uc5b5\uc6d0)\ub97c \ud55c\uc9c4\uce7c\uc5d0 '\ub300\ud55c\ud56d\uacf5 \uc0c1\ud45c\uad8c' \uc0ac\uc6a9\ub8cc\ub85c \ub0a9\ubd80\ud55c\ub2e4. \ud2b9\ud5c8\uccad\uc5d0 \ub530\ub974\uba74, \ub300\ud55c\ud56d\uacf5\uc740 2013\ub144 8\uc6d4 6\uc77c '\ub300\ud55c\ud56d\uacf5'\uacfc 'Korean Air' \uc774\ub984, \ud0dc\uadf9\ubb38\uc591\uc758 \ub85c\uace0 \ub4f1 \uc0c1\ud45c\uad8c \uc804\ubd80\uc640 \uc9c4\uc5d0\uc5b4\u00b7 \uce7c\ud638\ud154 \ub124\ud2b8\uc6cc\ud06c \ub4f1\uc758 \uacc4\uc5f4\uc0ac \uc8fc\uc2dd, \uc11c\uc6b8 \uc11c\uc18c\ubb38\uc0ac\uc625, \ud604\uae08 \ubc0f \uc608\uae08 1000\uc5b5\uc6d0\uc744 \ud55c\uc9c4\uce7c\uc5d0 \ub118\uaca8\uc92c\ub2e4. \ud55c\uc9c4\uce7c\uc740 \ub300\ud55c\ud56d\uacf5\uc73c\ub85c\ubd80\ud130 \uc0c1\ud45c\uad8c\uc744 \ub118\uaca8\ubc1b\uae30 \ub2f7\uc0c8 \uc804\uc778 2013\ub144 8\uc6d4 1\uc77c \ub300\ud55c\ud56d\uacf5\uc73c\ub85c\ubd80\ud130 \uacc4\uc5f4 \ubd84\ub9ac\ub41c, \ud55c\uc9c4\uadf8\ub8f9\uc758 \uc9c0\uc8fc \ud68c\uc0ac\ub2e4. \uc870\uc591\ud638 \ud68c\uc7a5\uacfc \uc7a5\ub140 \uc870\ud604\uc544, \uc7a5\ub0a8 \uc870\uc6d0\ud0dc, \ucc28\ub140 \uc870\ud604\ubbfc\uc740 \ub300\ud55c\ud56d\uacf5\uc758 \uc8fc\uc2dd\uc744 \ubaa8\ub450 \ub118\uae30\uace0 \ud55c\uc9c4\uce7c\uc758 \uc8fc\uc2dd\uc744 \ubc1b\uc558\ub2e4. \uc774\ud6c4 \uc870\uc591\ud638 \ud68c\uc7a5 \uc77c\uac00\ub294 2014\ub144 ~ 2017\ub144 \uae4c\uc9c0 \uc138 \ubc88\uc5d0 \uac78\uccd0 37\uc5b5\uc6d0\uc758 \ud604\uae08 \ubc30\ub2f9\uc744 \ubc1b\uc558\ub2e4. \uc624\ub108 \uc77c\uac00\uac00 \ud55c\uc9c4\uce7c\uc5d0\uc11c \uc218\uc2ed\uc5b5\uc6d0\ub300\uc758 \ubc30\ub2f9\uae08\uc744 \ubc1b\uc558\ub358 \uc774 \uc2dc\uae30, \ub300\ud55c\ud56d\uacf5\uc740 \uc800\uc720\uac00\u00b7\ud56d\uacf5\uc218\uc694 \uc99d\uac00\uac00 \uacb9\uce5c \ud638\ud669\uae30\uc600\uc74c\uc5d0\ub3c4 \ud55c\uc9c4\ud574\uc6b4 \ubd80\uc2e4\ucc98\ub9ac \ubb38\uc81c\ub97c \ub5a0\uc548\uc73c\uba70 \uace4\ub780\uc744 \uacaa\uac8c \ub418\uc5c8\ub2e4. \uc0ac\uc815\uc774 \uc5b4\ub824\uc6cc \uc218 \ub144\uac04 \uc8fc\uc8fc\ub4e4\uc5d0\uac8c \ubc30\ub2f9\uae08\uc744 \uc9c0\uae09\ud558\uc9c0 \ubabb\ud558\ub358 \ub300\ud55c\ud56d\uacf5\uc774 2014\ub144 ~ 2017\ub144\uae4c\uc9c0 \uc624\ub108 \uc77c\uac00\uc758 \uc9c0\ubd84\uc774 \ubab0\ub824\uc788\ub294 \ud55c\uc9c4\uce7c\uc5d0 \uc9c0\ubd88\ud55c '\ub300\ud55c\ud56d\uacf5 \uc0c1\ud45c\uad8c \uc0ac\uc6a9\ub8cc'\uc640 '\uac74\ubb3c \uc784\ub300\ub8cc'\ub294 1,229\uc5b5\uc6d0\uc73c\ub85c \ubc1d\ud600\uc84c\ub2e4.", "sentence_answer": " \ud55c\uc9c4\uce7c \uc740 \uc870\uc591\ud638 \ud55c\uc9c4\uadf8\ub8f9 \ud68c\uc7a5 \uc77c\uac00\uac00 \uc9c0\ubd84 25%\ub97c \ubcf4\uc720\ud55c \ud55c\uc9c4\uadf8\ub8f9 \uc9c0\uc8fc\ud68c\uc0ac\uc774\uba70, \ub300\ud55c\ud56d\uacf5\uc740 \ub9e4\ub144 \ub9e4\ucd9c\uc561\uc758 0.25%(2017\ub144 \uae30\uc900 300\uc5b5\uc6d0)\ub97c \ud55c\uc9c4\uce7c\uc5d0 '\ub300\ud55c\ud56d\uacf5 \uc0c1\ud45c\uad8c' \uc0ac\uc6a9\ub8cc\ub85c \ub0a9\ubd80\ud55c\ub2e4.", "paragraph_id": "6553326-1-0", "paragraph_question": "question: \uc870\uc591\ud638 \ud55c\uc9c4\uadf8\ub8f9 \ud68c\uc7a5 \uc77c\uac00\uac00 \uc9c0\ubd84 25%\ub97c \ubcf4\uc720\ud55c \ud55c\uc9c4\uadf8\ub8f9 \uc9c0\uc8fc\ud68c\uc0ac\ub294?, context: \ud55c\uc9c4\uce7c\uc740 \uc870\uc591\ud638 \ud55c\uc9c4\uadf8\ub8f9 \ud68c\uc7a5 \uc77c\uac00\uac00 \uc9c0\ubd84 25%\ub97c \ubcf4\uc720\ud55c \ud55c\uc9c4\uadf8\ub8f9 \uc9c0\uc8fc\ud68c\uc0ac\uc774\uba70, \ub300\ud55c\ud56d\uacf5\uc740 \ub9e4\ub144 \ub9e4\ucd9c\uc561\uc758 0.25%(2017\ub144 \uae30\uc900 300\uc5b5\uc6d0)\ub97c \ud55c\uc9c4\uce7c\uc5d0 '\ub300\ud55c\ud56d\uacf5 \uc0c1\ud45c\uad8c' \uc0ac\uc6a9\ub8cc\ub85c \ub0a9\ubd80\ud55c\ub2e4. \ud2b9\ud5c8\uccad\uc5d0 \ub530\ub974\uba74, \ub300\ud55c\ud56d\uacf5\uc740 2013\ub144 8\uc6d4 6\uc77c '\ub300\ud55c\ud56d\uacf5'\uacfc 'Korean Air' \uc774\ub984, \ud0dc\uadf9\ubb38\uc591\uc758 \ub85c\uace0 \ub4f1 \uc0c1\ud45c\uad8c \uc804\ubd80\uc640 \uc9c4\uc5d0\uc5b4\u00b7 \uce7c\ud638\ud154 \ub124\ud2b8\uc6cc\ud06c \ub4f1\uc758 \uacc4\uc5f4\uc0ac \uc8fc\uc2dd, \uc11c\uc6b8 \uc11c\uc18c\ubb38\uc0ac\uc625, \ud604\uae08 \ubc0f \uc608\uae08 1000\uc5b5\uc6d0\uc744 \ud55c\uc9c4\uce7c\uc5d0 \ub118\uaca8\uc92c\ub2e4. \ud55c\uc9c4\uce7c\uc740 \ub300\ud55c\ud56d\uacf5\uc73c\ub85c\ubd80\ud130 \uc0c1\ud45c\uad8c\uc744 \ub118\uaca8\ubc1b\uae30 \ub2f7\uc0c8 \uc804\uc778 2013\ub144 8\uc6d4 1\uc77c \ub300\ud55c\ud56d\uacf5\uc73c\ub85c\ubd80\ud130 \uacc4\uc5f4 \ubd84\ub9ac\ub41c, \ud55c\uc9c4\uadf8\ub8f9\uc758 \uc9c0\uc8fc \ud68c\uc0ac\ub2e4. \uc870\uc591\ud638 \ud68c\uc7a5\uacfc \uc7a5\ub140 \uc870\ud604\uc544, \uc7a5\ub0a8 \uc870\uc6d0\ud0dc, \ucc28\ub140 \uc870\ud604\ubbfc\uc740 \ub300\ud55c\ud56d\uacf5\uc758 \uc8fc\uc2dd\uc744 \ubaa8\ub450 \ub118\uae30\uace0 \ud55c\uc9c4\uce7c\uc758 \uc8fc\uc2dd\uc744 \ubc1b\uc558\ub2e4. \uc774\ud6c4 \uc870\uc591\ud638 \ud68c\uc7a5 \uc77c\uac00\ub294 2014\ub144 ~ 2017\ub144 \uae4c\uc9c0 \uc138 \ubc88\uc5d0 \uac78\uccd0 37\uc5b5\uc6d0\uc758 \ud604\uae08 \ubc30\ub2f9\uc744 \ubc1b\uc558\ub2e4. \uc624\ub108 \uc77c\uac00\uac00 \ud55c\uc9c4\uce7c\uc5d0\uc11c \uc218\uc2ed\uc5b5\uc6d0\ub300\uc758 \ubc30\ub2f9\uae08\uc744 \ubc1b\uc558\ub358 \uc774 \uc2dc\uae30, \ub300\ud55c\ud56d\uacf5\uc740 \uc800\uc720\uac00\u00b7\ud56d\uacf5\uc218\uc694 \uc99d\uac00\uac00 \uacb9\uce5c \ud638\ud669\uae30\uc600\uc74c\uc5d0\ub3c4 \ud55c\uc9c4\ud574\uc6b4 \ubd80\uc2e4\ucc98\ub9ac \ubb38\uc81c\ub97c \ub5a0\uc548\uc73c\uba70 \uace4\ub780\uc744 \uacaa\uac8c \ub418\uc5c8\ub2e4. \uc0ac\uc815\uc774 \uc5b4\ub824\uc6cc \uc218 \ub144\uac04 \uc8fc\uc8fc\ub4e4\uc5d0\uac8c \ubc30\ub2f9\uae08\uc744 \uc9c0\uae09\ud558\uc9c0 \ubabb\ud558\ub358 \ub300\ud55c\ud56d\uacf5\uc774 2014\ub144 ~ 2017\ub144\uae4c\uc9c0 \uc624\ub108 \uc77c\uac00\uc758 \uc9c0\ubd84\uc774 \ubab0\ub824\uc788\ub294 \ud55c\uc9c4\uce7c\uc5d0 \uc9c0\ubd88\ud55c '\ub300\ud55c\ud56d\uacf5 \uc0c1\ud45c\uad8c \uc0ac\uc6a9\ub8cc'\uc640 '\uac74\ubb3c \uc784\ub300\ub8cc'\ub294 1,229\uc5b5\uc6d0\uc73c\ub85c \ubc1d\ud600\uc84c\ub2e4."} {"question": "\ud55c\uad6d\uad50\uc6d0\ub300\ud559\uad50 \ucd1d\ud559\uc0dd\ud68c\uac00 \ubc18\ubbfc\uc790\ub2f9 \ud22c\uc7c1\uc744 \uc8fc\ub3c4\ud588\ub358 \uc5f0\ub3c4\ub294?", "paragraph": "\ucd1d\ud559\uc0dd\ud68c\ub294 \ud559\uc0dd\ub4e4\uc758 \uc694\uad6c\ub97c \ub300\uc678\uc801\uc73c\ub85c \uad00\ucca0\ud558\uae30 \uc704\ud574 \ub3c5\ub9bd\uc801\uc73c\ub85c \ud65c\ub3d9\ud558\ub294 \ud559\uc0dd \uc790\uce58 \uae30\uad6c\uc774\ub2e4. 1985\ub144 4\uc6d4 2\uc77c\uc5d0 \uc81c1\ub300 \ucd1d\ud559\uc0dd\ud68c\uc7a5\ub2e8\uc744 \uc120\ucd9c\ud558\uace0 \uc9d1\ud589\ubd80\ub97c \uad6c\uc131\ud558\uba74\uc11c \ubc1c\uc871\ud558\uc600\uc73c\uba70 1988\ub144 \uad00\uc120\ucd1d\uc7a5\uc758 \ucde8\uc784 \uac70\ubd80\ud558\uace0 \ud559\uc6d0 \uc790\uc728\ud654\ub97c \uc694\uad6c\ud558\ub294 \ucde8\uc784\uc2dd \ub18d\uc131\uc744 \uc2dc\uc791\uc73c\ub85c \uad6d\uac00\uace0\uc2dc\uc81c \ub3c4\uc785 \ucca0\ud68c\ub97c \uc704\ud55c \ucd1d\uc7a5\uc2e4 \uc810\uac70 \ub18d\uc131\uacfc \uc218\uc5c5\uac70\ubd80 \ud22c\uc7c1, 1991\ub144 \ubc18\ubbfc\uc790\ub2f9 \ud22c\uc7c1 \ubc0f \ub3d9\ub9f9\ud734\uc5c5 \uacb0\uc758, 1999\ub144 \ud559\ubd80 \uc815\uc6d0 \ucd95\uc18c \ubc0f \ucd1d\u00b7\ud559\uc7a5 \uc120\ucd9c\uc758 \uac04\uc120\uc81c \ub3c4\uc785 \ub4f1 6\uac1c \uad6c\uc870\uc870\uc815\uc548\uc5d0 \ub300\ud55c \uc800\uc9c0 \ud22c\uc7c1, 2005\ub144 \uae30\uc131\ud68c\ube44 \uc778\uc0c1 \ubc18\ub300 \ucd1d\uad90\uae30 \ub4f1\uc744 \uc8fc\ub3c4\ud558\uba70 \ud559\ub0b4\u00b7\uc678\uc758 \ud06c\uace0 \uc791\uc740 \uc0ac\uc548\uc5d0 \ub300\ud574 \ud559\uc0dd\ub4e4\uc758 \ubaa9\uc18c\ub9ac\ub97c \ub300\ubcc0\ud558\ub294 \uc5ed\ud560\uc744 \ud574\uc624\uace0 \uc788\ub2e4. \ub610\ud55c \uc0c8\ub0b4\uae30 \ubbf8\ub9ac \ubc30\uc6c0\ud130, \ud574\uc624\ub984\uc2dd, \uccad\ub78c\ucd95\uc804(\ub300\ub3d9\uc81c), \uba85\uc0ac \ucd08\uccad \ud2b9\uac15, \uad50\uc721\ubd09\uc0ac \ud65c\ub3d9, \uadc0\ud5a5\ubc84\uc2a4 \uc6b4\ud589, \ud1f4\uc0ac\ud0dd\ubc30 \uc0ac\uc5c5 \ub4f1\uc744 \uae30\ud68d\ud558\uace0 \ucd94\uc9c4\ud558\ub294 \uc77c\ub3c4 \ub9e1\uace0 \uc788\ub2e4. \ub9e4\ub144 \uc120\uac70\ub97c \ud1b5\ud574 \ucd1d\ud559\uc0dd\ud68c\uc7a5\uacfc \ubd80\ucd1d\ud559\uc0dd\ud68c\uc7a5\uc744 \uc120\ucd9c\ud558\uace0 \uc788\uc73c\uba70 \ud559\uc0dd \ubcf5\uc9c0\ub97c \uad00\uc7a5\ud558\ub294 \ud559\uc0dd\ubcf5\uc9c0\uc704\uc6d0\ud68c\uc640 \uc790\uce58 \uc5b8\ub860\uc778 \uad50\uc9c0\ud3b8\uc9d1\uc704\uc6d0\ud68c\uac00 \ucd1d\ud559\uc0dd\ud68c\uc758 \ud558\uc704 \uae30\uad6c\ub85c \uc124\uce58\ub418\uc5b4 \uc788\ub2e4.", "answer": "1991\ub144", "sentence": "1988\ub144 \uad00\uc120\ucd1d\uc7a5\uc758 \ucde8\uc784 \uac70\ubd80\ud558\uace0 \ud559\uc6d0 \uc790\uc728\ud654\ub97c \uc694\uad6c\ud558\ub294 \ucde8\uc784\uc2dd \ub18d\uc131\uc744 \uc2dc\uc791\uc73c\ub85c \uad6d\uac00\uace0\uc2dc\uc81c \ub3c4\uc785 \ucca0\ud68c\ub97c \uc704\ud55c \ucd1d\uc7a5\uc2e4 \uc810\uac70 \ub18d\uc131\uacfc \uc218\uc5c5\uac70\ubd80 \ud22c\uc7c1, 1991\ub144 \ubc18\ubbfc\uc790\ub2f9 \ud22c\uc7c1 \ubc0f \ub3d9\ub9f9\ud734\uc5c5 \uacb0\uc758, 1999\ub144 \ud559\ubd80 \uc815\uc6d0 \ucd95\uc18c \ubc0f \ucd1d\u00b7\ud559\uc7a5 \uc120\ucd9c\uc758 \uac04\uc120\uc81c \ub3c4\uc785 \ub4f1 6\uac1c \uad6c\uc870\uc870\uc815\uc548\uc5d0 \ub300\ud55c \uc800\uc9c0 \ud22c\uc7c1, 2005\ub144 \uae30\uc131\ud68c\ube44 \uc778\uc0c1 \ubc18\ub300 \ucd1d\uad90\uae30 \ub4f1\uc744 \uc8fc\ub3c4\ud558\uba70 \ud559\ub0b4\u00b7\uc678\uc758 \ud06c\uace0 \uc791\uc740 \uc0ac\uc548\uc5d0 \ub300\ud574 \ud559\uc0dd\ub4e4\uc758 \ubaa9\uc18c\ub9ac\ub97c \ub300\ubcc0\ud558\ub294 \uc5ed\ud560\uc744 \ud574\uc624\uace0 \uc788\ub2e4.", "paragraph_sentence": "\ucd1d\ud559\uc0dd\ud68c\ub294 \ud559\uc0dd\ub4e4\uc758 \uc694\uad6c\ub97c \ub300\uc678\uc801\uc73c\ub85c \uad00\ucca0\ud558\uae30 \uc704\ud574 \ub3c5\ub9bd\uc801\uc73c\ub85c \ud65c\ub3d9\ud558\ub294 \ud559\uc0dd \uc790\uce58 \uae30\uad6c\uc774\ub2e4. 1985\ub144 4\uc6d4 2\uc77c\uc5d0 \uc81c1\ub300 \ucd1d\ud559\uc0dd\ud68c\uc7a5\ub2e8\uc744 \uc120\ucd9c\ud558\uace0 \uc9d1\ud589\ubd80\ub97c \uad6c\uc131\ud558\uba74\uc11c \ubc1c\uc871\ud558\uc600\uc73c\uba70 1988\ub144 \uad00\uc120\ucd1d\uc7a5\uc758 \ucde8\uc784 \uac70\ubd80\ud558\uace0 \ud559\uc6d0 \uc790\uc728\ud654\ub97c \uc694\uad6c\ud558\ub294 \ucde8\uc784\uc2dd \ub18d\uc131\uc744 \uc2dc\uc791\uc73c\ub85c \uad6d\uac00\uace0\uc2dc\uc81c \ub3c4\uc785 \ucca0\ud68c\ub97c \uc704\ud55c \ucd1d\uc7a5\uc2e4 \uc810\uac70 \ub18d\uc131\uacfc \uc218\uc5c5\uac70\ubd80 \ud22c\uc7c1, 1991\ub144 \ubc18\ubbfc\uc790\ub2f9 \ud22c\uc7c1 \ubc0f \ub3d9\ub9f9\ud734\uc5c5 \uacb0\uc758, 1999\ub144 \ud559\ubd80 \uc815\uc6d0 \ucd95\uc18c \ubc0f \ucd1d\u00b7\ud559\uc7a5 \uc120\ucd9c\uc758 \uac04\uc120\uc81c \ub3c4\uc785 \ub4f1 6\uac1c \uad6c\uc870\uc870\uc815\uc548\uc5d0 \ub300\ud55c \uc800\uc9c0 \ud22c\uc7c1, 2005\ub144 \uae30\uc131\ud68c\ube44 \uc778\uc0c1 \ubc18\ub300 \ucd1d\uad90\uae30 \ub4f1\uc744 \uc8fc\ub3c4\ud558\uba70 \ud559\ub0b4\u00b7\uc678\uc758 \ud06c\uace0 \uc791\uc740 \uc0ac\uc548\uc5d0 \ub300\ud574 \ud559\uc0dd\ub4e4\uc758 \ubaa9\uc18c\ub9ac\ub97c \ub300\ubcc0\ud558\ub294 \uc5ed\ud560\uc744 \ud574\uc624\uace0 \uc788\ub2e4. \ub610\ud55c \uc0c8\ub0b4\uae30 \ubbf8\ub9ac \ubc30\uc6c0\ud130, \ud574\uc624\ub984\uc2dd, \uccad\ub78c\ucd95\uc804(\ub300\ub3d9\uc81c), \uba85\uc0ac \ucd08\uccad \ud2b9\uac15, \uad50\uc721\ubd09\uc0ac \ud65c\ub3d9, \uadc0\ud5a5\ubc84\uc2a4 \uc6b4\ud589, \ud1f4\uc0ac\ud0dd\ubc30 \uc0ac\uc5c5 \ub4f1\uc744 \uae30\ud68d\ud558\uace0 \ucd94\uc9c4\ud558\ub294 \uc77c\ub3c4 \ub9e1\uace0 \uc788\ub2e4. \ub9e4\ub144 \uc120\uac70\ub97c \ud1b5\ud574 \ucd1d\ud559\uc0dd\ud68c\uc7a5\uacfc \ubd80\ucd1d\ud559\uc0dd\ud68c\uc7a5\uc744 \uc120\ucd9c\ud558\uace0 \uc788\uc73c\uba70 \ud559\uc0dd \ubcf5\uc9c0\ub97c \uad00\uc7a5\ud558\ub294 \ud559\uc0dd\ubcf5\uc9c0\uc704\uc6d0\ud68c\uc640 \uc790\uce58 \uc5b8\ub860\uc778 \uad50\uc9c0\ud3b8\uc9d1\uc704\uc6d0\ud68c\uac00 \ucd1d\ud559\uc0dd\ud68c\uc758 \ud558\uc704 \uae30\uad6c\ub85c \uc124\uce58\ub418\uc5b4 \uc788\ub2e4.", "paragraph_answer": "\ucd1d\ud559\uc0dd\ud68c\ub294 \ud559\uc0dd\ub4e4\uc758 \uc694\uad6c\ub97c \ub300\uc678\uc801\uc73c\ub85c \uad00\ucca0\ud558\uae30 \uc704\ud574 \ub3c5\ub9bd\uc801\uc73c\ub85c \ud65c\ub3d9\ud558\ub294 \ud559\uc0dd \uc790\uce58 \uae30\uad6c\uc774\ub2e4. 1985\ub144 4\uc6d4 2\uc77c\uc5d0 \uc81c1\ub300 \ucd1d\ud559\uc0dd\ud68c\uc7a5\ub2e8\uc744 \uc120\ucd9c\ud558\uace0 \uc9d1\ud589\ubd80\ub97c \uad6c\uc131\ud558\uba74\uc11c \ubc1c\uc871\ud558\uc600\uc73c\uba70 1988\ub144 \uad00\uc120\ucd1d\uc7a5\uc758 \ucde8\uc784 \uac70\ubd80\ud558\uace0 \ud559\uc6d0 \uc790\uc728\ud654\ub97c \uc694\uad6c\ud558\ub294 \ucde8\uc784\uc2dd \ub18d\uc131\uc744 \uc2dc\uc791\uc73c\ub85c \uad6d\uac00\uace0\uc2dc\uc81c \ub3c4\uc785 \ucca0\ud68c\ub97c \uc704\ud55c \ucd1d\uc7a5\uc2e4 \uc810\uac70 \ub18d\uc131\uacfc \uc218\uc5c5\uac70\ubd80 \ud22c\uc7c1, 1991\ub144 \ubc18\ubbfc\uc790\ub2f9 \ud22c\uc7c1 \ubc0f \ub3d9\ub9f9\ud734\uc5c5 \uacb0\uc758, 1999\ub144 \ud559\ubd80 \uc815\uc6d0 \ucd95\uc18c \ubc0f \ucd1d\u00b7\ud559\uc7a5 \uc120\ucd9c\uc758 \uac04\uc120\uc81c \ub3c4\uc785 \ub4f1 6\uac1c \uad6c\uc870\uc870\uc815\uc548\uc5d0 \ub300\ud55c \uc800\uc9c0 \ud22c\uc7c1, 2005\ub144 \uae30\uc131\ud68c\ube44 \uc778\uc0c1 \ubc18\ub300 \ucd1d\uad90\uae30 \ub4f1\uc744 \uc8fc\ub3c4\ud558\uba70 \ud559\ub0b4\u00b7\uc678\uc758 \ud06c\uace0 \uc791\uc740 \uc0ac\uc548\uc5d0 \ub300\ud574 \ud559\uc0dd\ub4e4\uc758 \ubaa9\uc18c\ub9ac\ub97c \ub300\ubcc0\ud558\ub294 \uc5ed\ud560\uc744 \ud574\uc624\uace0 \uc788\ub2e4. \ub610\ud55c \uc0c8\ub0b4\uae30 \ubbf8\ub9ac \ubc30\uc6c0\ud130, \ud574\uc624\ub984\uc2dd, \uccad\ub78c\ucd95\uc804(\ub300\ub3d9\uc81c), \uba85\uc0ac \ucd08\uccad \ud2b9\uac15, \uad50\uc721\ubd09\uc0ac \ud65c\ub3d9, \uadc0\ud5a5\ubc84\uc2a4 \uc6b4\ud589, \ud1f4\uc0ac\ud0dd\ubc30 \uc0ac\uc5c5 \ub4f1\uc744 \uae30\ud68d\ud558\uace0 \ucd94\uc9c4\ud558\ub294 \uc77c\ub3c4 \ub9e1\uace0 \uc788\ub2e4. \ub9e4\ub144 \uc120\uac70\ub97c \ud1b5\ud574 \ucd1d\ud559\uc0dd\ud68c\uc7a5\uacfc \ubd80\ucd1d\ud559\uc0dd\ud68c\uc7a5\uc744 \uc120\ucd9c\ud558\uace0 \uc788\uc73c\uba70 \ud559\uc0dd \ubcf5\uc9c0\ub97c \uad00\uc7a5\ud558\ub294 \ud559\uc0dd\ubcf5\uc9c0\uc704\uc6d0\ud68c\uc640 \uc790\uce58 \uc5b8\ub860\uc778 \uad50\uc9c0\ud3b8\uc9d1\uc704\uc6d0\ud68c\uac00 \ucd1d\ud559\uc0dd\ud68c\uc758 \ud558\uc704 \uae30\uad6c\ub85c \uc124\uce58\ub418\uc5b4 \uc788\ub2e4.", "sentence_answer": "1988\ub144 \uad00\uc120\ucd1d\uc7a5\uc758 \ucde8\uc784 \uac70\ubd80\ud558\uace0 \ud559\uc6d0 \uc790\uc728\ud654\ub97c \uc694\uad6c\ud558\ub294 \ucde8\uc784\uc2dd \ub18d\uc131\uc744 \uc2dc\uc791\uc73c\ub85c \uad6d\uac00\uace0\uc2dc\uc81c \ub3c4\uc785 \ucca0\ud68c\ub97c \uc704\ud55c \ucd1d\uc7a5\uc2e4 \uc810\uac70 \ub18d\uc131\uacfc \uc218\uc5c5\uac70\ubd80 \ud22c\uc7c1, 1991\ub144 \ubc18\ubbfc\uc790\ub2f9 \ud22c\uc7c1 \ubc0f \ub3d9\ub9f9\ud734\uc5c5 \uacb0\uc758, 1999\ub144 \ud559\ubd80 \uc815\uc6d0 \ucd95\uc18c \ubc0f \ucd1d\u00b7\ud559\uc7a5 \uc120\ucd9c\uc758 \uac04\uc120\uc81c \ub3c4\uc785 \ub4f1 6\uac1c \uad6c\uc870\uc870\uc815\uc548\uc5d0 \ub300\ud55c \uc800\uc9c0 \ud22c\uc7c1, 2005\ub144 \uae30\uc131\ud68c\ube44 \uc778\uc0c1 \ubc18\ub300 \ucd1d\uad90\uae30 \ub4f1\uc744 \uc8fc\ub3c4\ud558\uba70 \ud559\ub0b4\u00b7\uc678\uc758 \ud06c\uace0 \uc791\uc740 \uc0ac\uc548\uc5d0 \ub300\ud574 \ud559\uc0dd\ub4e4\uc758 \ubaa9\uc18c\ub9ac\ub97c \ub300\ubcc0\ud558\ub294 \uc5ed\ud560\uc744 \ud574\uc624\uace0 \uc788\ub2e4.", "paragraph_id": "6569874-11-1", "paragraph_question": "question: \ud55c\uad6d\uad50\uc6d0\ub300\ud559\uad50 \ucd1d\ud559\uc0dd\ud68c\uac00 \ubc18\ubbfc\uc790\ub2f9 \ud22c\uc7c1\uc744 \uc8fc\ub3c4\ud588\ub358 \uc5f0\ub3c4\ub294?, context: \ucd1d\ud559\uc0dd\ud68c\ub294 \ud559\uc0dd\ub4e4\uc758 \uc694\uad6c\ub97c \ub300\uc678\uc801\uc73c\ub85c \uad00\ucca0\ud558\uae30 \uc704\ud574 \ub3c5\ub9bd\uc801\uc73c\ub85c \ud65c\ub3d9\ud558\ub294 \ud559\uc0dd \uc790\uce58 \uae30\uad6c\uc774\ub2e4. 1985\ub144 4\uc6d4 2\uc77c\uc5d0 \uc81c1\ub300 \ucd1d\ud559\uc0dd\ud68c\uc7a5\ub2e8\uc744 \uc120\ucd9c\ud558\uace0 \uc9d1\ud589\ubd80\ub97c \uad6c\uc131\ud558\uba74\uc11c \ubc1c\uc871\ud558\uc600\uc73c\uba70 1988\ub144 \uad00\uc120\ucd1d\uc7a5\uc758 \ucde8\uc784 \uac70\ubd80\ud558\uace0 \ud559\uc6d0 \uc790\uc728\ud654\ub97c \uc694\uad6c\ud558\ub294 \ucde8\uc784\uc2dd \ub18d\uc131\uc744 \uc2dc\uc791\uc73c\ub85c \uad6d\uac00\uace0\uc2dc\uc81c \ub3c4\uc785 \ucca0\ud68c\ub97c \uc704\ud55c \ucd1d\uc7a5\uc2e4 \uc810\uac70 \ub18d\uc131\uacfc \uc218\uc5c5\uac70\ubd80 \ud22c\uc7c1, 1991\ub144 \ubc18\ubbfc\uc790\ub2f9 \ud22c\uc7c1 \ubc0f \ub3d9\ub9f9\ud734\uc5c5 \uacb0\uc758, 1999\ub144 \ud559\ubd80 \uc815\uc6d0 \ucd95\uc18c \ubc0f \ucd1d\u00b7\ud559\uc7a5 \uc120\ucd9c\uc758 \uac04\uc120\uc81c \ub3c4\uc785 \ub4f1 6\uac1c \uad6c\uc870\uc870\uc815\uc548\uc5d0 \ub300\ud55c \uc800\uc9c0 \ud22c\uc7c1, 2005\ub144 \uae30\uc131\ud68c\ube44 \uc778\uc0c1 \ubc18\ub300 \ucd1d\uad90\uae30 \ub4f1\uc744 \uc8fc\ub3c4\ud558\uba70 \ud559\ub0b4\u00b7\uc678\uc758 \ud06c\uace0 \uc791\uc740 \uc0ac\uc548\uc5d0 \ub300\ud574 \ud559\uc0dd\ub4e4\uc758 \ubaa9\uc18c\ub9ac\ub97c \ub300\ubcc0\ud558\ub294 \uc5ed\ud560\uc744 \ud574\uc624\uace0 \uc788\ub2e4. \ub610\ud55c \uc0c8\ub0b4\uae30 \ubbf8\ub9ac \ubc30\uc6c0\ud130, \ud574\uc624\ub984\uc2dd, \uccad\ub78c\ucd95\uc804(\ub300\ub3d9\uc81c), \uba85\uc0ac \ucd08\uccad \ud2b9\uac15, \uad50\uc721\ubd09\uc0ac \ud65c\ub3d9, \uadc0\ud5a5\ubc84\uc2a4 \uc6b4\ud589, \ud1f4\uc0ac\ud0dd\ubc30 \uc0ac\uc5c5 \ub4f1\uc744 \uae30\ud68d\ud558\uace0 \ucd94\uc9c4\ud558\ub294 \uc77c\ub3c4 \ub9e1\uace0 \uc788\ub2e4. \ub9e4\ub144 \uc120\uac70\ub97c \ud1b5\ud574 \ucd1d\ud559\uc0dd\ud68c\uc7a5\uacfc \ubd80\ucd1d\ud559\uc0dd\ud68c\uc7a5\uc744 \uc120\ucd9c\ud558\uace0 \uc788\uc73c\uba70 \ud559\uc0dd \ubcf5\uc9c0\ub97c \uad00\uc7a5\ud558\ub294 \ud559\uc0dd\ubcf5\uc9c0\uc704\uc6d0\ud68c\uc640 \uc790\uce58 \uc5b8\ub860\uc778 \uad50\uc9c0\ud3b8\uc9d1\uc704\uc6d0\ud68c\uac00 \ucd1d\ud559\uc0dd\ud68c\uc758 \ud558\uc704 \uae30\uad6c\ub85c \uc124\uce58\ub418\uc5b4 \uc788\ub2e4."} {"question": "\uc815\ub2f9\uc774 \uc0ac\ud68c\ub97c \ub300\ud45c\ud558\uc5ec \uad6d\uac00\ub97c \uc6c0\uc9c1\uc774\ub294 \uc6d0\ub3d9\ub825\uc774\ub77c\ub294 \uac83\uc740 \uba87\ubc88\uc9f8 \ud559\uc124\uc778\uac00?", "paragraph": "\uc815\ub2f9\uc758 \ubcf8\uc9c8\uc774 \ubb34\uc5c7\uc778\uac00\uc5d0 \ub300\ud574\uc11c\ub294 \ub450 \uac00\uc9c0 \ud559\uc124\uc774 \uc788\ub2e4. \uadf8 \ud558\ub098\ub294 \uc815\ub2f9\uc744 \ub2e4\uc591\ud55c \uc0ac\ud68c\uc801 \uc774\uc775\uc758 \uc815\uce58\uc801 \uc694\uad6c\ub97c \uc870\uc9c1\ud654\uc2dc\ucf1c \uc774\ub97c \uc815\uce58\uc5d0 \ubc18\uc601\uc2dc\ud0a4\ub294 \ubbfc\uc8fc\uc815\uce58\uc758 \ub3c4\uad6c\ub85c\uc11c \ubcf4\ub294 \uc124\uc774\uba70, \ub610 \ud558\ub098\ub294 \uc18c\uc218 \uc9c0\ubc30\uce35\uc774 \uc120\uac70\ub97c \ud1b5\ud574\uc11c \uc720\uad8c\uc790\ub97c \uc870\uc885\ud558\uc5ec \ubbfc\uc758\ub97c \uc870\uc791\ud568\uc73c\ub85c\uc368 \ud6a8\uacfc\uc801\uc73c\ub85c \uc9c0\ubc30\ub97c \ud569\ub9ac\ud654\ud558\uae30 \uc704\ud55c \uc9c0\ubc30\uce35\uc758 \ub4dd\ud45c\uc870\uc9c1\uc5d0 \ubd88\uacfc\ud558\ub2e4\uace0 \ubcf4\ub294 \uc124\uc774\ub2e4. \uc804\uc790\uc758 \uc124\uc5d0 \uc758\ud558\uba74 \uc790\ubcf8\uc8fc\uc758\uc758 \ubc1c\uc804\uc5d0 \ub530\ub77c \ub2e4\uc591\ud55c \uc0ac\ud68c\uc801 \uc774\uc775\uc774 \ubcf5\uc7a1\ud55c \uc0c1\ud638\uad00\uacc4\ub97c \uac00\uc9c0\uba74\uc11c \uc7a1\ub2e4\ud55c \uc815\uce58\uc801 \uc694\uad6c\ub97c \ub0b4\uc138\uc6b0\ub294\ub370, \uc774\ub7ec\ud55c \uc694\uad6c\ub294 \uadf8 \uc790\uccb4\ub85c\uc11c\ub294 \uc0ac\ud68c \uc804\uccb4 \uc911 \uc18c\uc218\uc790\uc758 \uc758\uc0ac\ub97c \ubc18\uc601\ud558\uba70, \uc9c1\uc811 \uc815\uce58\uc640\ub294 \uc5f0\uacb0\ub418\uc9c0 \uc54a\ub294\ub2e4. \uc774\ub7ec\ud55c \uc7a1\ub2e4\ud55c \uc694\uad6c\ub97c \uc9d1\uc57d\ud574\uc11c \uc815\uce58\uc801 \ud45c\ud604\uc73c\ub85c \ubc1c\uc804\uc2dc\ud0a4\ub294 \uc791\uc6a9\uc774 \uc815\ub2f9\uc758 \uc874\uc7ac\ub85c \uc778\ud574\uc11c \ube44\ub85c\uc18c \uac00\ub2a5\ud574\uc9c4\ub2e4. \ub2e4\uc2dc \ub9d0\ud574\uc11c \uc815\ub2f9\uc740 \uadf8 \uc785\ud6c4\ubcf4\uc790\uc640 \uc815\ucc45\uc744 \ud1b5\ud574\uc11c \uc0ac\ud68c\uc801 \ubaa8\ub4e0 \uc774\ud574\uc758 \uc18c\ub9ac\ub97c \uc815\uce58\uc5d0 \ubc18\uc601\uc2dc\ud0a4\ub294 \uc784\ubb34\ub97c \uc9c0\ub2c8\uace0 \uc788\uc73c\uba70, \ub530\ub77c\uc11c \uc0ac\ud68c\ub97c \ub300\ud45c\ud574\uc11c \uad6d\uac00\ub97c \uc6c0\uc9c1\uc774\ub294 \uc6d0\ub3d9\ub825\uc774\ub77c\ub294 \uac83\uc774 \uc81c1\uc758 \ud559\uc124\uc774\ub2e4. \uc5ec\uae30\uc5d0 \ub300\ud574 \uc81c2\uc758 \ud559\uc124\uc5d0 \uc758\ud558\uba74 \ub2e4\uc591\ud55c \uc0ac\ud68c\uc801 \uc774\ud574\ub294 \uc800\ub9c8\ub2e4\uc758 \uc815\uce58\uc801 \uc694\uad6c\ub97c \uac16\uace0 \uc788\uc73c\ub098 \uc785\ud6c4\ubcf4\uc790\uc640 \uc815\ucc45\uc744 \uacb0\uc815\ud558\ub294 \uac83\uc740 \uc0ac\ud68c\uc801 \uc774\ud574\uac00 \uc544\ub2c8\ub77c \uc815\ub2f9\uc774\uba70, \uc0ac\ud68c\uc801 \uc774\ud574\ub294 \uc815\ub2f9\uc774 \uacb0\uc815\ud55c \uac83\uc744 \ubc1b\uc544\ub4e4\uc774\ub294 \ub370 \uc9c0\ub098\uc9c0 \uc54a\ub294\ub2e4. \uc0ac\ud68c\uc801 \uc774\uc775\uacfc \uc815\ub2f9\uc758 \uad00\uacc4\ub97c \ubcf4\ub354\ub77c\ub3c4 \uc815\ub2f9\uc758 \uc6b0\uc704\uac00 \ud655\uc2e4\ud558\ub2e4. \ubb3c\ub860 \uc815\ub2f9\uc73c\ub85c\uc11c\ub3c4 \ub2e4\uc218\uc758 \uc9c0\uc9c0\ub97c \uc5bb\uc5b4\uc11c \uc815\uad8c\uc744 \uc7a5\uc545\ud558\uae30 \uc704\ud574\uc11c\ub294 \uac01\uc885 \uc0ac\ud68c\uc801 \uc774\ud574\uc640 \uc5f0\uacb0\uc744 \uac00\uc9c0\uace0, \uadf8\ub4e4\uc758 \uc694\uad6c\ub97c \uc785\ud6c4\ubcf4\uc790\ub098 \uc815\ucc45\uc5d0 \ubc18\uc601\uc2dc\ucf1c\uc57c \ud55c\ub2e4\ub294 \uac83\uc740 \uc0ac\uc2e4\uc774\ub2e4. \uc774\uc640 \ubc18\ub300\ub85c \uc0ac\ud68c\uc801 \uc774\ud574\uc758 \uce21\uc5d0\uc11c\ub3c4 \uc815\uce58\uc801 \ubc1c\uc5b8\uad8c\uc744 \uc5bb\uae30 \uc704\ud574\uc11c\ub294 \uc815\ub2f9\uacfc \uc190\uc744 \uc7a1\uc744 \uc218\ubc16\uc5d0 \uc5c6\ub2e4. \uadf8\ub7f0\ub370 \uc0ac\ud68c\uc801 \uc774\ud574\ub294 \ubb34\uc218\ud788 \uc788\ub294 \ub370 \ubc18\ud574 \uc815\ub2f9\uc740 \uace0\uc791 \uba87\uba87 \uac1c\uac00 \uc815\uce58\uc801 \ubc1c\uc5b8\uc744 \ud560 \uc218 \uc788\uc73c\ubbc0\ub85c \uadf8 \uad00\uacc4\ub294 \uc815\ub2f9\uc744 \uc815\uce58\uc801 \uc9c0\uc9c0\uc758 \uacb0\uc815\uc801 \ub9e4\uc218\uc2dc\uc7a5\uc73c\ub85c \ud588\uc73c\uba70 \uc815\ub2f9\uc758 \uc6b0\uc704\ub294 \uc6c0\uc9c1\uc77c \uc218 \uc5c6\ub294 \uac83\uc774\ub2e4. \uac8c\ub2e4\uac00 \uc815\ub2f9\uc774\ub780 \uc77c\ubc18 \uc720\uad8c\uc790\uac00 \uc790\uc720\ub85c\uc774 \ucc38\uac00\ud558\uc5ec \ub2f9\uc6d0\ub300\uc911\uc758 \ubbfc\uc8fc\uc801 \uc758\uc0ac\ub85c\uc368 \uc6b4\uc601\ud558\ub294 \uc870\uc9c1\uc774 \uc544\ub2c8\ub77c, \uc18c\uc218\uc758 \uc9c1\uc5c5\uc815\uce58\uac00\uc758 \uc9d1\ub2e8\uc774 \ub2f9\uc6b4\uc601\uc758 \uc2e4\uad8c\uc744 \uc7a5\uc545\ud558\uace0 \uc785\ud6c4\ubcf4\uc790\uc640 \uc815\ucc45\uc744 \uacb0\uc815\ud558\ub294 \ub2e8\uccb4\uc774\ub2e4. \uadf8\ub7ec\ubbc0\ub85c \uc815\ub2f9\uc744 \ud604\uc2e4\uc801 \uc785\uc7a5\uc5d0\uc11c \ubcf4\ub294 \ud55c \uc720\uad8c\uc790\ub97c \uc704\ub85c\ubd80\ud130 \uc870\uc885\ud574\uc11c \ubbfc\uc758\uc758 \ud5a5\ubc29\uc744 \uacb0\uc815\uc9d3\ub294 \ub4dd\ud45c\uacf5\uc791 \uae30\uad6c\ub77c\ub294 \uc124\uc774 \uc131\ub9bd\ub420 \uc218 \uc788\ub294 \uac83\uc774\ub2e4. \uadf8\ub7ec\ub098 \uc774 \uc124\uc740 \uc601\uad6d\uc774\ub098 \ubbf8\uad6d \uac19\uc740 \uc591\ub2f9\uc81c\ub97c \uc804\uc81c\ub85c \ud558\uba70, \uc2a4\uce78\ub514\ub098\ube44\uc544 3\uad6d\uacfc \uac19\uc740 \ub2e4\uc218\uc815\ub2f9\uc81c\ud558\uc5d0\uc11c\ub294 \ud574\ub2f9\ub418\uc9c0 \uc54a\ub294\ub2e4. \uc774\ub7ec\ud55c \uc81c\ub3c4\ud558\uc5d0\uc11c\ub294 \uac01 \uc815\ub2f9\uacfc \uc0ac\ud68c\uc801 \uc774\uc775\uc774 \uba85\ud655\ud55c \ud615\ud0dc\ub85c \uacb0\ubd80\ub418\uc5b4 \uc788\uc73c\ubbc0\ub85c \uc815\ub2f9\uc758 \uc0ac\ud68c\ub300\ud45c\uc801 \uc131\uaca9\uc740 \ubd80\uc778\ud560 \uc218 \uc5c6\ub2e4.", "answer": "\uc81c1\uc758 \ud559\uc124", "sentence": "\ub2e4\uc2dc \ub9d0\ud574\uc11c \uc815\ub2f9\uc740 \uadf8 \uc785\ud6c4\ubcf4\uc790\uc640 \uc815\ucc45\uc744 \ud1b5\ud574\uc11c \uc0ac\ud68c\uc801 \ubaa8\ub4e0 \uc774\ud574\uc758 \uc18c\ub9ac\ub97c \uc815\uce58\uc5d0 \ubc18\uc601\uc2dc\ud0a4\ub294 \uc784\ubb34\ub97c \uc9c0\ub2c8\uace0 \uc788\uc73c\uba70, \ub530\ub77c\uc11c \uc0ac\ud68c\ub97c \ub300\ud45c\ud574\uc11c \uad6d\uac00\ub97c \uc6c0\uc9c1\uc774\ub294 \uc6d0\ub3d9\ub825\uc774\ub77c\ub294 \uac83\uc774 \uc81c1\uc758 \ud559\uc124 \uc774\ub2e4.", "paragraph_sentence": "\uc815\ub2f9\uc758 \ubcf8\uc9c8\uc774 \ubb34\uc5c7\uc778\uac00\uc5d0 \ub300\ud574\uc11c\ub294 \ub450 \uac00\uc9c0 \ud559\uc124\uc774 \uc788\ub2e4. \uadf8 \ud558\ub098\ub294 \uc815\ub2f9\uc744 \ub2e4\uc591\ud55c \uc0ac\ud68c\uc801 \uc774\uc775\uc758 \uc815\uce58\uc801 \uc694\uad6c\ub97c \uc870\uc9c1\ud654\uc2dc\ucf1c \uc774\ub97c \uc815\uce58\uc5d0 \ubc18\uc601\uc2dc\ud0a4\ub294 \ubbfc\uc8fc\uc815\uce58\uc758 \ub3c4\uad6c\ub85c\uc11c \ubcf4\ub294 \uc124\uc774\uba70, \ub610 \ud558\ub098\ub294 \uc18c\uc218 \uc9c0\ubc30\uce35\uc774 \uc120\uac70\ub97c \ud1b5\ud574\uc11c \uc720\uad8c\uc790\ub97c \uc870\uc885\ud558\uc5ec \ubbfc\uc758\ub97c \uc870\uc791\ud568\uc73c\ub85c\uc368 \ud6a8\uacfc\uc801\uc73c\ub85c \uc9c0\ubc30\ub97c \ud569\ub9ac\ud654\ud558\uae30 \uc704\ud55c \uc9c0\ubc30\uce35\uc758 \ub4dd\ud45c\uc870\uc9c1\uc5d0 \ubd88\uacfc\ud558\ub2e4\uace0 \ubcf4\ub294 \uc124\uc774\ub2e4. \uc804\uc790\uc758 \uc124\uc5d0 \uc758\ud558\uba74 \uc790\ubcf8\uc8fc\uc758\uc758 \ubc1c\uc804\uc5d0 \ub530\ub77c \ub2e4\uc591\ud55c \uc0ac\ud68c\uc801 \uc774\uc775\uc774 \ubcf5\uc7a1\ud55c \uc0c1\ud638\uad00\uacc4\ub97c \uac00\uc9c0\uba74\uc11c \uc7a1\ub2e4\ud55c \uc815\uce58\uc801 \uc694\uad6c\ub97c \ub0b4\uc138\uc6b0\ub294\ub370, \uc774\ub7ec\ud55c \uc694\uad6c\ub294 \uadf8 \uc790\uccb4\ub85c\uc11c\ub294 \uc0ac\ud68c \uc804\uccb4 \uc911 \uc18c\uc218\uc790\uc758 \uc758\uc0ac\ub97c \ubc18\uc601\ud558\uba70, \uc9c1\uc811 \uc815\uce58\uc640\ub294 \uc5f0\uacb0\ub418\uc9c0 \uc54a\ub294\ub2e4. \uc774\ub7ec\ud55c \uc7a1\ub2e4\ud55c \uc694\uad6c\ub97c \uc9d1\uc57d\ud574\uc11c \uc815\uce58\uc801 \ud45c\ud604\uc73c\ub85c \ubc1c\uc804\uc2dc\ud0a4\ub294 \uc791\uc6a9\uc774 \uc815\ub2f9\uc758 \uc874\uc7ac\ub85c \uc778\ud574\uc11c \ube44\ub85c\uc18c \uac00\ub2a5\ud574\uc9c4\ub2e4. \ub2e4\uc2dc \ub9d0\ud574\uc11c \uc815\ub2f9\uc740 \uadf8 \uc785\ud6c4\ubcf4\uc790\uc640 \uc815\ucc45\uc744 \ud1b5\ud574\uc11c \uc0ac\ud68c\uc801 \ubaa8\ub4e0 \uc774\ud574\uc758 \uc18c\ub9ac\ub97c \uc815\uce58\uc5d0 \ubc18\uc601\uc2dc\ud0a4\ub294 \uc784\ubb34\ub97c \uc9c0\ub2c8\uace0 \uc788\uc73c\uba70, \ub530\ub77c\uc11c \uc0ac\ud68c\ub97c \ub300\ud45c\ud574\uc11c \uad6d\uac00\ub97c \uc6c0\uc9c1\uc774\ub294 \uc6d0\ub3d9\ub825\uc774\ub77c\ub294 \uac83\uc774 \uc81c1\uc758 \ud559\uc124 \uc774\ub2e4. \uc5ec\uae30\uc5d0 \ub300\ud574 \uc81c2\uc758 \ud559\uc124\uc5d0 \uc758\ud558\uba74 \ub2e4\uc591\ud55c \uc0ac\ud68c\uc801 \uc774\ud574\ub294 \uc800\ub9c8\ub2e4\uc758 \uc815\uce58\uc801 \uc694\uad6c\ub97c \uac16\uace0 \uc788\uc73c\ub098 \uc785\ud6c4\ubcf4\uc790\uc640 \uc815\ucc45\uc744 \uacb0\uc815\ud558\ub294 \uac83\uc740 \uc0ac\ud68c\uc801 \uc774\ud574\uac00 \uc544\ub2c8\ub77c \uc815\ub2f9\uc774\uba70, \uc0ac\ud68c\uc801 \uc774\ud574\ub294 \uc815\ub2f9\uc774 \uacb0\uc815\ud55c \uac83\uc744 \ubc1b\uc544\ub4e4\uc774\ub294 \ub370 \uc9c0\ub098\uc9c0 \uc54a\ub294\ub2e4. \uc0ac\ud68c\uc801 \uc774\uc775\uacfc \uc815\ub2f9\uc758 \uad00\uacc4\ub97c \ubcf4\ub354\ub77c\ub3c4 \uc815\ub2f9\uc758 \uc6b0\uc704\uac00 \ud655\uc2e4\ud558\ub2e4. \ubb3c\ub860 \uc815\ub2f9\uc73c\ub85c\uc11c\ub3c4 \ub2e4\uc218\uc758 \uc9c0\uc9c0\ub97c \uc5bb\uc5b4\uc11c \uc815\uad8c\uc744 \uc7a5\uc545\ud558\uae30 \uc704\ud574\uc11c\ub294 \uac01\uc885 \uc0ac\ud68c\uc801 \uc774\ud574\uc640 \uc5f0\uacb0\uc744 \uac00\uc9c0\uace0, \uadf8\ub4e4\uc758 \uc694\uad6c\ub97c \uc785\ud6c4\ubcf4\uc790\ub098 \uc815\ucc45\uc5d0 \ubc18\uc601\uc2dc\ucf1c\uc57c \ud55c\ub2e4\ub294 \uac83\uc740 \uc0ac\uc2e4\uc774\ub2e4. \uc774\uc640 \ubc18\ub300\ub85c \uc0ac\ud68c\uc801 \uc774\ud574\uc758 \uce21\uc5d0\uc11c\ub3c4 \uc815\uce58\uc801 \ubc1c\uc5b8\uad8c\uc744 \uc5bb\uae30 \uc704\ud574\uc11c\ub294 \uc815\ub2f9\uacfc \uc190\uc744 \uc7a1\uc744 \uc218\ubc16\uc5d0 \uc5c6\ub2e4. \uadf8\ub7f0\ub370 \uc0ac\ud68c\uc801 \uc774\ud574\ub294 \ubb34\uc218\ud788 \uc788\ub294 \ub370 \ubc18\ud574 \uc815\ub2f9\uc740 \uace0\uc791 \uba87\uba87 \uac1c\uac00 \uc815\uce58\uc801 \ubc1c\uc5b8\uc744 \ud560 \uc218 \uc788\uc73c\ubbc0\ub85c \uadf8 \uad00\uacc4\ub294 \uc815\ub2f9\uc744 \uc815\uce58\uc801 \uc9c0\uc9c0\uc758 \uacb0\uc815\uc801 \ub9e4\uc218\uc2dc\uc7a5\uc73c\ub85c \ud588\uc73c\uba70 \uc815\ub2f9\uc758 \uc6b0\uc704\ub294 \uc6c0\uc9c1\uc77c \uc218 \uc5c6\ub294 \uac83\uc774\ub2e4. \uac8c\ub2e4\uac00 \uc815\ub2f9\uc774\ub780 \uc77c\ubc18 \uc720\uad8c\uc790\uac00 \uc790\uc720\ub85c\uc774 \ucc38\uac00\ud558\uc5ec \ub2f9\uc6d0\ub300\uc911\uc758 \ubbfc\uc8fc\uc801 \uc758\uc0ac\ub85c\uc368 \uc6b4\uc601\ud558\ub294 \uc870\uc9c1\uc774 \uc544\ub2c8\ub77c, \uc18c\uc218\uc758 \uc9c1\uc5c5\uc815\uce58\uac00\uc758 \uc9d1\ub2e8\uc774 \ub2f9\uc6b4\uc601\uc758 \uc2e4\uad8c\uc744 \uc7a5\uc545\ud558\uace0 \uc785\ud6c4\ubcf4\uc790\uc640 \uc815\ucc45\uc744 \uacb0\uc815\ud558\ub294 \ub2e8\uccb4\uc774\ub2e4. \uadf8\ub7ec\ubbc0\ub85c \uc815\ub2f9\uc744 \ud604\uc2e4\uc801 \uc785\uc7a5\uc5d0\uc11c \ubcf4\ub294 \ud55c \uc720\uad8c\uc790\ub97c \uc704\ub85c\ubd80\ud130 \uc870\uc885\ud574\uc11c \ubbfc\uc758\uc758 \ud5a5\ubc29\uc744 \uacb0\uc815\uc9d3\ub294 \ub4dd\ud45c\uacf5\uc791 \uae30\uad6c\ub77c\ub294 \uc124\uc774 \uc131\ub9bd\ub420 \uc218 \uc788\ub294 \uac83\uc774\ub2e4. \uadf8\ub7ec\ub098 \uc774 \uc124\uc740 \uc601\uad6d\uc774\ub098 \ubbf8\uad6d \uac19\uc740 \uc591\ub2f9\uc81c\ub97c \uc804\uc81c\ub85c \ud558\uba70, \uc2a4\uce78\ub514\ub098\ube44\uc544 3\uad6d\uacfc \uac19\uc740 \ub2e4\uc218\uc815\ub2f9\uc81c\ud558\uc5d0\uc11c\ub294 \ud574\ub2f9\ub418\uc9c0 \uc54a\ub294\ub2e4. \uc774\ub7ec\ud55c \uc81c\ub3c4\ud558\uc5d0\uc11c\ub294 \uac01 \uc815\ub2f9\uacfc \uc0ac\ud68c\uc801 \uc774\uc775\uc774 \uba85\ud655\ud55c \ud615\ud0dc\ub85c \uacb0\ubd80\ub418\uc5b4 \uc788\uc73c\ubbc0\ub85c \uc815\ub2f9\uc758 \uc0ac\ud68c\ub300\ud45c\uc801 \uc131\uaca9\uc740 \ubd80\uc778\ud560 \uc218 \uc5c6\ub2e4.", "paragraph_answer": "\uc815\ub2f9\uc758 \ubcf8\uc9c8\uc774 \ubb34\uc5c7\uc778\uac00\uc5d0 \ub300\ud574\uc11c\ub294 \ub450 \uac00\uc9c0 \ud559\uc124\uc774 \uc788\ub2e4. \uadf8 \ud558\ub098\ub294 \uc815\ub2f9\uc744 \ub2e4\uc591\ud55c \uc0ac\ud68c\uc801 \uc774\uc775\uc758 \uc815\uce58\uc801 \uc694\uad6c\ub97c \uc870\uc9c1\ud654\uc2dc\ucf1c \uc774\ub97c \uc815\uce58\uc5d0 \ubc18\uc601\uc2dc\ud0a4\ub294 \ubbfc\uc8fc\uc815\uce58\uc758 \ub3c4\uad6c\ub85c\uc11c \ubcf4\ub294 \uc124\uc774\uba70, \ub610 \ud558\ub098\ub294 \uc18c\uc218 \uc9c0\ubc30\uce35\uc774 \uc120\uac70\ub97c \ud1b5\ud574\uc11c \uc720\uad8c\uc790\ub97c \uc870\uc885\ud558\uc5ec \ubbfc\uc758\ub97c \uc870\uc791\ud568\uc73c\ub85c\uc368 \ud6a8\uacfc\uc801\uc73c\ub85c \uc9c0\ubc30\ub97c \ud569\ub9ac\ud654\ud558\uae30 \uc704\ud55c \uc9c0\ubc30\uce35\uc758 \ub4dd\ud45c\uc870\uc9c1\uc5d0 \ubd88\uacfc\ud558\ub2e4\uace0 \ubcf4\ub294 \uc124\uc774\ub2e4. \uc804\uc790\uc758 \uc124\uc5d0 \uc758\ud558\uba74 \uc790\ubcf8\uc8fc\uc758\uc758 \ubc1c\uc804\uc5d0 \ub530\ub77c \ub2e4\uc591\ud55c \uc0ac\ud68c\uc801 \uc774\uc775\uc774 \ubcf5\uc7a1\ud55c \uc0c1\ud638\uad00\uacc4\ub97c \uac00\uc9c0\uba74\uc11c \uc7a1\ub2e4\ud55c \uc815\uce58\uc801 \uc694\uad6c\ub97c \ub0b4\uc138\uc6b0\ub294\ub370, \uc774\ub7ec\ud55c \uc694\uad6c\ub294 \uadf8 \uc790\uccb4\ub85c\uc11c\ub294 \uc0ac\ud68c \uc804\uccb4 \uc911 \uc18c\uc218\uc790\uc758 \uc758\uc0ac\ub97c \ubc18\uc601\ud558\uba70, \uc9c1\uc811 \uc815\uce58\uc640\ub294 \uc5f0\uacb0\ub418\uc9c0 \uc54a\ub294\ub2e4. \uc774\ub7ec\ud55c \uc7a1\ub2e4\ud55c \uc694\uad6c\ub97c \uc9d1\uc57d\ud574\uc11c \uc815\uce58\uc801 \ud45c\ud604\uc73c\ub85c \ubc1c\uc804\uc2dc\ud0a4\ub294 \uc791\uc6a9\uc774 \uc815\ub2f9\uc758 \uc874\uc7ac\ub85c \uc778\ud574\uc11c \ube44\ub85c\uc18c \uac00\ub2a5\ud574\uc9c4\ub2e4. \ub2e4\uc2dc \ub9d0\ud574\uc11c \uc815\ub2f9\uc740 \uadf8 \uc785\ud6c4\ubcf4\uc790\uc640 \uc815\ucc45\uc744 \ud1b5\ud574\uc11c \uc0ac\ud68c\uc801 \ubaa8\ub4e0 \uc774\ud574\uc758 \uc18c\ub9ac\ub97c \uc815\uce58\uc5d0 \ubc18\uc601\uc2dc\ud0a4\ub294 \uc784\ubb34\ub97c \uc9c0\ub2c8\uace0 \uc788\uc73c\uba70, \ub530\ub77c\uc11c \uc0ac\ud68c\ub97c \ub300\ud45c\ud574\uc11c \uad6d\uac00\ub97c \uc6c0\uc9c1\uc774\ub294 \uc6d0\ub3d9\ub825\uc774\ub77c\ub294 \uac83\uc774 \uc81c1\uc758 \ud559\uc124 \uc774\ub2e4. \uc5ec\uae30\uc5d0 \ub300\ud574 \uc81c2\uc758 \ud559\uc124\uc5d0 \uc758\ud558\uba74 \ub2e4\uc591\ud55c \uc0ac\ud68c\uc801 \uc774\ud574\ub294 \uc800\ub9c8\ub2e4\uc758 \uc815\uce58\uc801 \uc694\uad6c\ub97c \uac16\uace0 \uc788\uc73c\ub098 \uc785\ud6c4\ubcf4\uc790\uc640 \uc815\ucc45\uc744 \uacb0\uc815\ud558\ub294 \uac83\uc740 \uc0ac\ud68c\uc801 \uc774\ud574\uac00 \uc544\ub2c8\ub77c \uc815\ub2f9\uc774\uba70, \uc0ac\ud68c\uc801 \uc774\ud574\ub294 \uc815\ub2f9\uc774 \uacb0\uc815\ud55c \uac83\uc744 \ubc1b\uc544\ub4e4\uc774\ub294 \ub370 \uc9c0\ub098\uc9c0 \uc54a\ub294\ub2e4. \uc0ac\ud68c\uc801 \uc774\uc775\uacfc \uc815\ub2f9\uc758 \uad00\uacc4\ub97c \ubcf4\ub354\ub77c\ub3c4 \uc815\ub2f9\uc758 \uc6b0\uc704\uac00 \ud655\uc2e4\ud558\ub2e4. \ubb3c\ub860 \uc815\ub2f9\uc73c\ub85c\uc11c\ub3c4 \ub2e4\uc218\uc758 \uc9c0\uc9c0\ub97c \uc5bb\uc5b4\uc11c \uc815\uad8c\uc744 \uc7a5\uc545\ud558\uae30 \uc704\ud574\uc11c\ub294 \uac01\uc885 \uc0ac\ud68c\uc801 \uc774\ud574\uc640 \uc5f0\uacb0\uc744 \uac00\uc9c0\uace0, \uadf8\ub4e4\uc758 \uc694\uad6c\ub97c \uc785\ud6c4\ubcf4\uc790\ub098 \uc815\ucc45\uc5d0 \ubc18\uc601\uc2dc\ucf1c\uc57c \ud55c\ub2e4\ub294 \uac83\uc740 \uc0ac\uc2e4\uc774\ub2e4. \uc774\uc640 \ubc18\ub300\ub85c \uc0ac\ud68c\uc801 \uc774\ud574\uc758 \uce21\uc5d0\uc11c\ub3c4 \uc815\uce58\uc801 \ubc1c\uc5b8\uad8c\uc744 \uc5bb\uae30 \uc704\ud574\uc11c\ub294 \uc815\ub2f9\uacfc \uc190\uc744 \uc7a1\uc744 \uc218\ubc16\uc5d0 \uc5c6\ub2e4. \uadf8\ub7f0\ub370 \uc0ac\ud68c\uc801 \uc774\ud574\ub294 \ubb34\uc218\ud788 \uc788\ub294 \ub370 \ubc18\ud574 \uc815\ub2f9\uc740 \uace0\uc791 \uba87\uba87 \uac1c\uac00 \uc815\uce58\uc801 \ubc1c\uc5b8\uc744 \ud560 \uc218 \uc788\uc73c\ubbc0\ub85c \uadf8 \uad00\uacc4\ub294 \uc815\ub2f9\uc744 \uc815\uce58\uc801 \uc9c0\uc9c0\uc758 \uacb0\uc815\uc801 \ub9e4\uc218\uc2dc\uc7a5\uc73c\ub85c \ud588\uc73c\uba70 \uc815\ub2f9\uc758 \uc6b0\uc704\ub294 \uc6c0\uc9c1\uc77c \uc218 \uc5c6\ub294 \uac83\uc774\ub2e4. \uac8c\ub2e4\uac00 \uc815\ub2f9\uc774\ub780 \uc77c\ubc18 \uc720\uad8c\uc790\uac00 \uc790\uc720\ub85c\uc774 \ucc38\uac00\ud558\uc5ec \ub2f9\uc6d0\ub300\uc911\uc758 \ubbfc\uc8fc\uc801 \uc758\uc0ac\ub85c\uc368 \uc6b4\uc601\ud558\ub294 \uc870\uc9c1\uc774 \uc544\ub2c8\ub77c, \uc18c\uc218\uc758 \uc9c1\uc5c5\uc815\uce58\uac00\uc758 \uc9d1\ub2e8\uc774 \ub2f9\uc6b4\uc601\uc758 \uc2e4\uad8c\uc744 \uc7a5\uc545\ud558\uace0 \uc785\ud6c4\ubcf4\uc790\uc640 \uc815\ucc45\uc744 \uacb0\uc815\ud558\ub294 \ub2e8\uccb4\uc774\ub2e4. \uadf8\ub7ec\ubbc0\ub85c \uc815\ub2f9\uc744 \ud604\uc2e4\uc801 \uc785\uc7a5\uc5d0\uc11c \ubcf4\ub294 \ud55c \uc720\uad8c\uc790\ub97c \uc704\ub85c\ubd80\ud130 \uc870\uc885\ud574\uc11c \ubbfc\uc758\uc758 \ud5a5\ubc29\uc744 \uacb0\uc815\uc9d3\ub294 \ub4dd\ud45c\uacf5\uc791 \uae30\uad6c\ub77c\ub294 \uc124\uc774 \uc131\ub9bd\ub420 \uc218 \uc788\ub294 \uac83\uc774\ub2e4. \uadf8\ub7ec\ub098 \uc774 \uc124\uc740 \uc601\uad6d\uc774\ub098 \ubbf8\uad6d \uac19\uc740 \uc591\ub2f9\uc81c\ub97c \uc804\uc81c\ub85c \ud558\uba70, \uc2a4\uce78\ub514\ub098\ube44\uc544 3\uad6d\uacfc \uac19\uc740 \ub2e4\uc218\uc815\ub2f9\uc81c\ud558\uc5d0\uc11c\ub294 \ud574\ub2f9\ub418\uc9c0 \uc54a\ub294\ub2e4. \uc774\ub7ec\ud55c \uc81c\ub3c4\ud558\uc5d0\uc11c\ub294 \uac01 \uc815\ub2f9\uacfc \uc0ac\ud68c\uc801 \uc774\uc775\uc774 \uba85\ud655\ud55c \ud615\ud0dc\ub85c \uacb0\ubd80\ub418\uc5b4 \uc788\uc73c\ubbc0\ub85c \uc815\ub2f9\uc758 \uc0ac\ud68c\ub300\ud45c\uc801 \uc131\uaca9\uc740 \ubd80\uc778\ud560 \uc218 \uc5c6\ub2e4.", "sentence_answer": "\ub2e4\uc2dc \ub9d0\ud574\uc11c \uc815\ub2f9\uc740 \uadf8 \uc785\ud6c4\ubcf4\uc790\uc640 \uc815\ucc45\uc744 \ud1b5\ud574\uc11c \uc0ac\ud68c\uc801 \ubaa8\ub4e0 \uc774\ud574\uc758 \uc18c\ub9ac\ub97c \uc815\uce58\uc5d0 \ubc18\uc601\uc2dc\ud0a4\ub294 \uc784\ubb34\ub97c \uc9c0\ub2c8\uace0 \uc788\uc73c\uba70, \ub530\ub77c\uc11c \uc0ac\ud68c\ub97c \ub300\ud45c\ud574\uc11c \uad6d\uac00\ub97c \uc6c0\uc9c1\uc774\ub294 \uc6d0\ub3d9\ub825\uc774\ub77c\ub294 \uac83\uc774 \uc81c1\uc758 \ud559\uc124 \uc774\ub2e4.", "paragraph_id": "6334198-9-1", "paragraph_question": "question: \uc815\ub2f9\uc774 \uc0ac\ud68c\ub97c \ub300\ud45c\ud558\uc5ec \uad6d\uac00\ub97c \uc6c0\uc9c1\uc774\ub294 \uc6d0\ub3d9\ub825\uc774\ub77c\ub294 \uac83\uc740 \uba87\ubc88\uc9f8 \ud559\uc124\uc778\uac00?, context: \uc815\ub2f9\uc758 \ubcf8\uc9c8\uc774 \ubb34\uc5c7\uc778\uac00\uc5d0 \ub300\ud574\uc11c\ub294 \ub450 \uac00\uc9c0 \ud559\uc124\uc774 \uc788\ub2e4. \uadf8 \ud558\ub098\ub294 \uc815\ub2f9\uc744 \ub2e4\uc591\ud55c \uc0ac\ud68c\uc801 \uc774\uc775\uc758 \uc815\uce58\uc801 \uc694\uad6c\ub97c \uc870\uc9c1\ud654\uc2dc\ucf1c \uc774\ub97c \uc815\uce58\uc5d0 \ubc18\uc601\uc2dc\ud0a4\ub294 \ubbfc\uc8fc\uc815\uce58\uc758 \ub3c4\uad6c\ub85c\uc11c \ubcf4\ub294 \uc124\uc774\uba70, \ub610 \ud558\ub098\ub294 \uc18c\uc218 \uc9c0\ubc30\uce35\uc774 \uc120\uac70\ub97c \ud1b5\ud574\uc11c \uc720\uad8c\uc790\ub97c \uc870\uc885\ud558\uc5ec \ubbfc\uc758\ub97c \uc870\uc791\ud568\uc73c\ub85c\uc368 \ud6a8\uacfc\uc801\uc73c\ub85c \uc9c0\ubc30\ub97c \ud569\ub9ac\ud654\ud558\uae30 \uc704\ud55c \uc9c0\ubc30\uce35\uc758 \ub4dd\ud45c\uc870\uc9c1\uc5d0 \ubd88\uacfc\ud558\ub2e4\uace0 \ubcf4\ub294 \uc124\uc774\ub2e4. \uc804\uc790\uc758 \uc124\uc5d0 \uc758\ud558\uba74 \uc790\ubcf8\uc8fc\uc758\uc758 \ubc1c\uc804\uc5d0 \ub530\ub77c \ub2e4\uc591\ud55c \uc0ac\ud68c\uc801 \uc774\uc775\uc774 \ubcf5\uc7a1\ud55c \uc0c1\ud638\uad00\uacc4\ub97c \uac00\uc9c0\uba74\uc11c \uc7a1\ub2e4\ud55c \uc815\uce58\uc801 \uc694\uad6c\ub97c \ub0b4\uc138\uc6b0\ub294\ub370, \uc774\ub7ec\ud55c \uc694\uad6c\ub294 \uadf8 \uc790\uccb4\ub85c\uc11c\ub294 \uc0ac\ud68c \uc804\uccb4 \uc911 \uc18c\uc218\uc790\uc758 \uc758\uc0ac\ub97c \ubc18\uc601\ud558\uba70, \uc9c1\uc811 \uc815\uce58\uc640\ub294 \uc5f0\uacb0\ub418\uc9c0 \uc54a\ub294\ub2e4. \uc774\ub7ec\ud55c \uc7a1\ub2e4\ud55c \uc694\uad6c\ub97c \uc9d1\uc57d\ud574\uc11c \uc815\uce58\uc801 \ud45c\ud604\uc73c\ub85c \ubc1c\uc804\uc2dc\ud0a4\ub294 \uc791\uc6a9\uc774 \uc815\ub2f9\uc758 \uc874\uc7ac\ub85c \uc778\ud574\uc11c \ube44\ub85c\uc18c \uac00\ub2a5\ud574\uc9c4\ub2e4. \ub2e4\uc2dc \ub9d0\ud574\uc11c \uc815\ub2f9\uc740 \uadf8 \uc785\ud6c4\ubcf4\uc790\uc640 \uc815\ucc45\uc744 \ud1b5\ud574\uc11c \uc0ac\ud68c\uc801 \ubaa8\ub4e0 \uc774\ud574\uc758 \uc18c\ub9ac\ub97c \uc815\uce58\uc5d0 \ubc18\uc601\uc2dc\ud0a4\ub294 \uc784\ubb34\ub97c \uc9c0\ub2c8\uace0 \uc788\uc73c\uba70, \ub530\ub77c\uc11c \uc0ac\ud68c\ub97c \ub300\ud45c\ud574\uc11c \uad6d\uac00\ub97c \uc6c0\uc9c1\uc774\ub294 \uc6d0\ub3d9\ub825\uc774\ub77c\ub294 \uac83\uc774 \uc81c1\uc758 \ud559\uc124\uc774\ub2e4. \uc5ec\uae30\uc5d0 \ub300\ud574 \uc81c2\uc758 \ud559\uc124\uc5d0 \uc758\ud558\uba74 \ub2e4\uc591\ud55c \uc0ac\ud68c\uc801 \uc774\ud574\ub294 \uc800\ub9c8\ub2e4\uc758 \uc815\uce58\uc801 \uc694\uad6c\ub97c \uac16\uace0 \uc788\uc73c\ub098 \uc785\ud6c4\ubcf4\uc790\uc640 \uc815\ucc45\uc744 \uacb0\uc815\ud558\ub294 \uac83\uc740 \uc0ac\ud68c\uc801 \uc774\ud574\uac00 \uc544\ub2c8\ub77c \uc815\ub2f9\uc774\uba70, \uc0ac\ud68c\uc801 \uc774\ud574\ub294 \uc815\ub2f9\uc774 \uacb0\uc815\ud55c \uac83\uc744 \ubc1b\uc544\ub4e4\uc774\ub294 \ub370 \uc9c0\ub098\uc9c0 \uc54a\ub294\ub2e4. \uc0ac\ud68c\uc801 \uc774\uc775\uacfc \uc815\ub2f9\uc758 \uad00\uacc4\ub97c \ubcf4\ub354\ub77c\ub3c4 \uc815\ub2f9\uc758 \uc6b0\uc704\uac00 \ud655\uc2e4\ud558\ub2e4. \ubb3c\ub860 \uc815\ub2f9\uc73c\ub85c\uc11c\ub3c4 \ub2e4\uc218\uc758 \uc9c0\uc9c0\ub97c \uc5bb\uc5b4\uc11c \uc815\uad8c\uc744 \uc7a5\uc545\ud558\uae30 \uc704\ud574\uc11c\ub294 \uac01\uc885 \uc0ac\ud68c\uc801 \uc774\ud574\uc640 \uc5f0\uacb0\uc744 \uac00\uc9c0\uace0, \uadf8\ub4e4\uc758 \uc694\uad6c\ub97c \uc785\ud6c4\ubcf4\uc790\ub098 \uc815\ucc45\uc5d0 \ubc18\uc601\uc2dc\ucf1c\uc57c \ud55c\ub2e4\ub294 \uac83\uc740 \uc0ac\uc2e4\uc774\ub2e4. \uc774\uc640 \ubc18\ub300\ub85c \uc0ac\ud68c\uc801 \uc774\ud574\uc758 \uce21\uc5d0\uc11c\ub3c4 \uc815\uce58\uc801 \ubc1c\uc5b8\uad8c\uc744 \uc5bb\uae30 \uc704\ud574\uc11c\ub294 \uc815\ub2f9\uacfc \uc190\uc744 \uc7a1\uc744 \uc218\ubc16\uc5d0 \uc5c6\ub2e4. \uadf8\ub7f0\ub370 \uc0ac\ud68c\uc801 \uc774\ud574\ub294 \ubb34\uc218\ud788 \uc788\ub294 \ub370 \ubc18\ud574 \uc815\ub2f9\uc740 \uace0\uc791 \uba87\uba87 \uac1c\uac00 \uc815\uce58\uc801 \ubc1c\uc5b8\uc744 \ud560 \uc218 \uc788\uc73c\ubbc0\ub85c \uadf8 \uad00\uacc4\ub294 \uc815\ub2f9\uc744 \uc815\uce58\uc801 \uc9c0\uc9c0\uc758 \uacb0\uc815\uc801 \ub9e4\uc218\uc2dc\uc7a5\uc73c\ub85c \ud588\uc73c\uba70 \uc815\ub2f9\uc758 \uc6b0\uc704\ub294 \uc6c0\uc9c1\uc77c \uc218 \uc5c6\ub294 \uac83\uc774\ub2e4. \uac8c\ub2e4\uac00 \uc815\ub2f9\uc774\ub780 \uc77c\ubc18 \uc720\uad8c\uc790\uac00 \uc790\uc720\ub85c\uc774 \ucc38\uac00\ud558\uc5ec \ub2f9\uc6d0\ub300\uc911\uc758 \ubbfc\uc8fc\uc801 \uc758\uc0ac\ub85c\uc368 \uc6b4\uc601\ud558\ub294 \uc870\uc9c1\uc774 \uc544\ub2c8\ub77c, \uc18c\uc218\uc758 \uc9c1\uc5c5\uc815\uce58\uac00\uc758 \uc9d1\ub2e8\uc774 \ub2f9\uc6b4\uc601\uc758 \uc2e4\uad8c\uc744 \uc7a5\uc545\ud558\uace0 \uc785\ud6c4\ubcf4\uc790\uc640 \uc815\ucc45\uc744 \uacb0\uc815\ud558\ub294 \ub2e8\uccb4\uc774\ub2e4. \uadf8\ub7ec\ubbc0\ub85c \uc815\ub2f9\uc744 \ud604\uc2e4\uc801 \uc785\uc7a5\uc5d0\uc11c \ubcf4\ub294 \ud55c \uc720\uad8c\uc790\ub97c \uc704\ub85c\ubd80\ud130 \uc870\uc885\ud574\uc11c \ubbfc\uc758\uc758 \ud5a5\ubc29\uc744 \uacb0\uc815\uc9d3\ub294 \ub4dd\ud45c\uacf5\uc791 \uae30\uad6c\ub77c\ub294 \uc124\uc774 \uc131\ub9bd\ub420 \uc218 \uc788\ub294 \uac83\uc774\ub2e4. \uadf8\ub7ec\ub098 \uc774 \uc124\uc740 \uc601\uad6d\uc774\ub098 \ubbf8\uad6d \uac19\uc740 \uc591\ub2f9\uc81c\ub97c \uc804\uc81c\ub85c \ud558\uba70, \uc2a4\uce78\ub514\ub098\ube44\uc544 3\uad6d\uacfc \uac19\uc740 \ub2e4\uc218\uc815\ub2f9\uc81c\ud558\uc5d0\uc11c\ub294 \ud574\ub2f9\ub418\uc9c0 \uc54a\ub294\ub2e4. \uc774\ub7ec\ud55c \uc81c\ub3c4\ud558\uc5d0\uc11c\ub294 \uac01 \uc815\ub2f9\uacfc \uc0ac\ud68c\uc801 \uc774\uc775\uc774 \uba85\ud655\ud55c \ud615\ud0dc\ub85c \uacb0\ubd80\ub418\uc5b4 \uc788\uc73c\ubbc0\ub85c \uc815\ub2f9\uc758 \uc0ac\ud68c\ub300\ud45c\uc801 \uc131\uaca9\uc740 \ubd80\uc778\ud560 \uc218 \uc5c6\ub2e4."} {"question": "\ubb34\ud55c\ub3c4\uc804\uc774 \uc790\ub9c9\uc73c\ub85c \ud604 \uc815\ubd80\uc758 \ube44\ud310\uc744 \ud558\uace0 \uc788\ub2e4\uace0 \uc8fc\uc7a5\ud55c \ub2e8\uccb4\ub294 \uc5b4\ub514\uc778\uac00?", "paragraph": "2009\ub144 7\uc6d4 22\uc77c \ub274\ub77c\uc774\ud2b8 \uc804\uad6d\uc5f0\ud569 \uce21\uc5d0\uc11c '\ubb34\ud55c\ub3c4\uc804\uc774 \uc790\ub9c9\uc744 \uc774\uc6a9\ud574 \ud604 \uc815\ubd80\ub97c \ube44\ud310\ud558\uace0 \uc788\ub2e4'\uace0 \uc8fc\uc7a5\ud558\uc600\ub2e4. \uc774\ub4e4\uc740 \uc790\uc2e0\ub4e4\uc758 \uc6f9\uc0ac\uc774\ud2b8\uc758 'Mr.\ud76c\ub9dd\uc774\uc640 \ud568\uaed8\ud558\ub294 \uc218\uc694\uc5f0\uc7ac\ub9cc\ud654'\ub97c \ud1b5\ud574 \u201c\ud604 \uc815\ubd80\ub97c \ud5a5\ud55c MBC\uc758 \u2018\ubb34\ud55c\ub3c4\uc804\u2019\u201d\uc774\ub77c\ub294 \uc81c\ubaa9\uc758 \ub9cc\ud654\ub97c \uac8c\uc7ac\ud588\ub2e4.\u2018\uad11\uc6b0\ubcd1 \uc1a1\uc544\uc9c0\u2019, \u2018\uae4c\ubd88\uba74 \ub354 \uc138\uac8c\u2026\uc9c4\uc555\uc758 \ubc95\uce59\u2019, \u2018\ub1cc\uc6a9\ub7c9 1.9\uba54\uac00\u2019 \ub4f1\uc758 \uc790\ub9c9\ub4e4\uc774 \ud604 \uc815\ubd80\uc640 \ub300\ud1b5\ub839\uc744 \ube44\ud310\ud558\uace0 \uc788\uc73c\uba70, \ud2b9\ud788 \uc9c0\ub09c 2009\ub144 3\uc6d4 \ubb34\ud55c\ub3c4\uc804\uc5d0 \uc774\uba85\ubc15 \ub300\ud1b5\ub839\uc744 \ub2ee\uc740 \uc77c\ubc18\uc778 \ucd9c\uc5f0\uc790\uac00 \ub300\uc120 \uad11\uace0\ub97c \ud328\ub7ec\ub514\ud55c \ud654\uba74\uc744 \ub450\uace0 \u201c\ud604\uc9c1 \ub300\ud1b5\ub839\uc5d0 \ub300\ud55c \ud76c\ud654\ud654\ub9c8\uc800\ub3c4 \uc11c\uc2b4\uc9c0 \uc54a\ub294 \ubc29\uc1a1 \ub0b4\uc6a9\uc740 \uc774\ubbf8 \ub3c4\ub97c \ub118\uc5b4\uc130\ub2e4.\u201d\uace0 \ube44\ub09c\ud588\ub2e4. \uc774\ub7ec\ud55c \uc8fc\uc7a5\uc5d0 \ub300\ud574 \ub204\ub9ac\uafbc\ub4e4 \uc0ac\uc774\uc5d0\uc11c \ube44\ub09c \uc5ec\ub860\uc774 \uaca9\ud654\ub418\uc5c8\uc73c\uba70, \ud574\ub2f9 \ub2e8\uccb4\uc758 \uc6f9\uc0ac\uc774\ud2b8\ub294 \ub204\ub9ac\uafbc\ub4e4\uc758 \ud56d\uc758\ubc29\ubb38\uc73c\ub85c \ud55c\ub54c \uc811\uc18d\uc774 \ub2e4\uc6b4\ub418\uae30\ub3c4 \ud588\ub2e4.", "answer": "\ub274\ub77c\uc774\ud2b8 \uc804\uad6d\uc5f0\ud569", "sentence": "2009\ub144 7\uc6d4 22\uc77c \ub274\ub77c\uc774\ud2b8 \uc804\uad6d\uc5f0\ud569 \uce21\uc5d0\uc11c '\ubb34\ud55c\ub3c4\uc804\uc774 \uc790\ub9c9\uc744 \uc774\uc6a9\ud574 \ud604 \uc815\ubd80\ub97c \ube44\ud310\ud558\uace0 \uc788\ub2e4'\uace0 \uc8fc\uc7a5\ud558\uc600\ub2e4.", "paragraph_sentence": " 2009\ub144 7\uc6d4 22\uc77c \ub274\ub77c\uc774\ud2b8 \uc804\uad6d\uc5f0\ud569 \uce21\uc5d0\uc11c '\ubb34\ud55c\ub3c4\uc804\uc774 \uc790\ub9c9\uc744 \uc774\uc6a9\ud574 \ud604 \uc815\ubd80\ub97c \ube44\ud310\ud558\uace0 \uc788\ub2e4'\uace0 \uc8fc\uc7a5\ud558\uc600\ub2e4. \uc774\ub4e4\uc740 \uc790\uc2e0\ub4e4\uc758 \uc6f9\uc0ac\uc774\ud2b8\uc758 'Mr.\ud76c\ub9dd\uc774\uc640 \ud568\uaed8\ud558\ub294 \uc218\uc694\uc5f0\uc7ac\ub9cc\ud654'\ub97c \ud1b5\ud574 \u201c\ud604 \uc815\ubd80\ub97c \ud5a5\ud55c MBC\uc758 \u2018\ubb34\ud55c\ub3c4\uc804\u2019\u201d\uc774\ub77c\ub294 \uc81c\ubaa9\uc758 \ub9cc\ud654\ub97c \uac8c\uc7ac\ud588\ub2e4. \u2018\uad11\uc6b0\ubcd1 \uc1a1\uc544\uc9c0\u2019, \u2018\uae4c\ubd88\uba74 \ub354 \uc138\uac8c\u2026\uc9c4\uc555\uc758 \ubc95\uce59\u2019, \u2018\ub1cc\uc6a9\ub7c9 1.9\uba54\uac00\u2019 \ub4f1\uc758 \uc790\ub9c9\ub4e4\uc774 \ud604 \uc815\ubd80\uc640 \ub300\ud1b5\ub839\uc744 \ube44\ud310\ud558\uace0 \uc788\uc73c\uba70, \ud2b9\ud788 \uc9c0\ub09c 2009\ub144 3\uc6d4 \ubb34\ud55c\ub3c4\uc804\uc5d0 \uc774\uba85\ubc15 \ub300\ud1b5\ub839\uc744 \ub2ee\uc740 \uc77c\ubc18\uc778 \ucd9c\uc5f0\uc790\uac00 \ub300\uc120 \uad11\uace0\ub97c \ud328\ub7ec\ub514\ud55c \ud654\uba74\uc744 \ub450\uace0 \u201c\ud604\uc9c1 \ub300\ud1b5\ub839\uc5d0 \ub300\ud55c \ud76c\ud654\ud654\ub9c8\uc800\ub3c4 \uc11c\uc2b4\uc9c0 \uc54a\ub294 \ubc29\uc1a1 \ub0b4\uc6a9\uc740 \uc774\ubbf8 \ub3c4\ub97c \ub118\uc5b4\uc130\ub2e4. \u201d\uace0 \ube44\ub09c\ud588\ub2e4. \uc774\ub7ec\ud55c \uc8fc\uc7a5\uc5d0 \ub300\ud574 \ub204\ub9ac\uafbc\ub4e4 \uc0ac\uc774\uc5d0\uc11c \ube44\ub09c \uc5ec\ub860\uc774 \uaca9\ud654\ub418\uc5c8\uc73c\uba70, \ud574\ub2f9 \ub2e8\uccb4\uc758 \uc6f9\uc0ac\uc774\ud2b8\ub294 \ub204\ub9ac\uafbc\ub4e4\uc758 \ud56d\uc758\ubc29\ubb38\uc73c\ub85c \ud55c\ub54c \uc811\uc18d\uc774 \ub2e4\uc6b4\ub418\uae30\ub3c4 \ud588\ub2e4.", "paragraph_answer": "2009\ub144 7\uc6d4 22\uc77c \ub274\ub77c\uc774\ud2b8 \uc804\uad6d\uc5f0\ud569 \uce21\uc5d0\uc11c '\ubb34\ud55c\ub3c4\uc804\uc774 \uc790\ub9c9\uc744 \uc774\uc6a9\ud574 \ud604 \uc815\ubd80\ub97c \ube44\ud310\ud558\uace0 \uc788\ub2e4'\uace0 \uc8fc\uc7a5\ud558\uc600\ub2e4. \uc774\ub4e4\uc740 \uc790\uc2e0\ub4e4\uc758 \uc6f9\uc0ac\uc774\ud2b8\uc758 'Mr.\ud76c\ub9dd\uc774\uc640 \ud568\uaed8\ud558\ub294 \uc218\uc694\uc5f0\uc7ac\ub9cc\ud654'\ub97c \ud1b5\ud574 \u201c\ud604 \uc815\ubd80\ub97c \ud5a5\ud55c MBC\uc758 \u2018\ubb34\ud55c\ub3c4\uc804\u2019\u201d\uc774\ub77c\ub294 \uc81c\ubaa9\uc758 \ub9cc\ud654\ub97c \uac8c\uc7ac\ud588\ub2e4.\u2018\uad11\uc6b0\ubcd1 \uc1a1\uc544\uc9c0\u2019, \u2018\uae4c\ubd88\uba74 \ub354 \uc138\uac8c\u2026\uc9c4\uc555\uc758 \ubc95\uce59\u2019, \u2018\ub1cc\uc6a9\ub7c9 1.9\uba54\uac00\u2019 \ub4f1\uc758 \uc790\ub9c9\ub4e4\uc774 \ud604 \uc815\ubd80\uc640 \ub300\ud1b5\ub839\uc744 \ube44\ud310\ud558\uace0 \uc788\uc73c\uba70, \ud2b9\ud788 \uc9c0\ub09c 2009\ub144 3\uc6d4 \ubb34\ud55c\ub3c4\uc804\uc5d0 \uc774\uba85\ubc15 \ub300\ud1b5\ub839\uc744 \ub2ee\uc740 \uc77c\ubc18\uc778 \ucd9c\uc5f0\uc790\uac00 \ub300\uc120 \uad11\uace0\ub97c \ud328\ub7ec\ub514\ud55c \ud654\uba74\uc744 \ub450\uace0 \u201c\ud604\uc9c1 \ub300\ud1b5\ub839\uc5d0 \ub300\ud55c \ud76c\ud654\ud654\ub9c8\uc800\ub3c4 \uc11c\uc2b4\uc9c0 \uc54a\ub294 \ubc29\uc1a1 \ub0b4\uc6a9\uc740 \uc774\ubbf8 \ub3c4\ub97c \ub118\uc5b4\uc130\ub2e4.\u201d\uace0 \ube44\ub09c\ud588\ub2e4. \uc774\ub7ec\ud55c \uc8fc\uc7a5\uc5d0 \ub300\ud574 \ub204\ub9ac\uafbc\ub4e4 \uc0ac\uc774\uc5d0\uc11c \ube44\ub09c \uc5ec\ub860\uc774 \uaca9\ud654\ub418\uc5c8\uc73c\uba70, \ud574\ub2f9 \ub2e8\uccb4\uc758 \uc6f9\uc0ac\uc774\ud2b8\ub294 \ub204\ub9ac\uafbc\ub4e4\uc758 \ud56d\uc758\ubc29\ubb38\uc73c\ub85c \ud55c\ub54c \uc811\uc18d\uc774 \ub2e4\uc6b4\ub418\uae30\ub3c4 \ud588\ub2e4.", "sentence_answer": "2009\ub144 7\uc6d4 22\uc77c \ub274\ub77c\uc774\ud2b8 \uc804\uad6d\uc5f0\ud569 \uce21\uc5d0\uc11c '\ubb34\ud55c\ub3c4\uc804\uc774 \uc790\ub9c9\uc744 \uc774\uc6a9\ud574 \ud604 \uc815\ubd80\ub97c \ube44\ud310\ud558\uace0 \uc788\ub2e4'\uace0 \uc8fc\uc7a5\ud558\uc600\ub2e4.", "paragraph_id": "6411454-5-0", "paragraph_question": "question: \ubb34\ud55c\ub3c4\uc804\uc774 \uc790\ub9c9\uc73c\ub85c \ud604 \uc815\ubd80\uc758 \ube44\ud310\uc744 \ud558\uace0 \uc788\ub2e4\uace0 \uc8fc\uc7a5\ud55c \ub2e8\uccb4\ub294 \uc5b4\ub514\uc778\uac00?, context: 2009\ub144 7\uc6d4 22\uc77c \ub274\ub77c\uc774\ud2b8 \uc804\uad6d\uc5f0\ud569 \uce21\uc5d0\uc11c '\ubb34\ud55c\ub3c4\uc804\uc774 \uc790\ub9c9\uc744 \uc774\uc6a9\ud574 \ud604 \uc815\ubd80\ub97c \ube44\ud310\ud558\uace0 \uc788\ub2e4'\uace0 \uc8fc\uc7a5\ud558\uc600\ub2e4. \uc774\ub4e4\uc740 \uc790\uc2e0\ub4e4\uc758 \uc6f9\uc0ac\uc774\ud2b8\uc758 'Mr.\ud76c\ub9dd\uc774\uc640 \ud568\uaed8\ud558\ub294 \uc218\uc694\uc5f0\uc7ac\ub9cc\ud654'\ub97c \ud1b5\ud574 \u201c\ud604 \uc815\ubd80\ub97c \ud5a5\ud55c MBC\uc758 \u2018\ubb34\ud55c\ub3c4\uc804\u2019\u201d\uc774\ub77c\ub294 \uc81c\ubaa9\uc758 \ub9cc\ud654\ub97c \uac8c\uc7ac\ud588\ub2e4.\u2018\uad11\uc6b0\ubcd1 \uc1a1\uc544\uc9c0\u2019, \u2018\uae4c\ubd88\uba74 \ub354 \uc138\uac8c\u2026\uc9c4\uc555\uc758 \ubc95\uce59\u2019, \u2018\ub1cc\uc6a9\ub7c9 1.9\uba54\uac00\u2019 \ub4f1\uc758 \uc790\ub9c9\ub4e4\uc774 \ud604 \uc815\ubd80\uc640 \ub300\ud1b5\ub839\uc744 \ube44\ud310\ud558\uace0 \uc788\uc73c\uba70, \ud2b9\ud788 \uc9c0\ub09c 2009\ub144 3\uc6d4 \ubb34\ud55c\ub3c4\uc804\uc5d0 \uc774\uba85\ubc15 \ub300\ud1b5\ub839\uc744 \ub2ee\uc740 \uc77c\ubc18\uc778 \ucd9c\uc5f0\uc790\uac00 \ub300\uc120 \uad11\uace0\ub97c \ud328\ub7ec\ub514\ud55c \ud654\uba74\uc744 \ub450\uace0 \u201c\ud604\uc9c1 \ub300\ud1b5\ub839\uc5d0 \ub300\ud55c \ud76c\ud654\ud654\ub9c8\uc800\ub3c4 \uc11c\uc2b4\uc9c0 \uc54a\ub294 \ubc29\uc1a1 \ub0b4\uc6a9\uc740 \uc774\ubbf8 \ub3c4\ub97c \ub118\uc5b4\uc130\ub2e4.\u201d\uace0 \ube44\ub09c\ud588\ub2e4. \uc774\ub7ec\ud55c \uc8fc\uc7a5\uc5d0 \ub300\ud574 \ub204\ub9ac\uafbc\ub4e4 \uc0ac\uc774\uc5d0\uc11c \ube44\ub09c \uc5ec\ub860\uc774 \uaca9\ud654\ub418\uc5c8\uc73c\uba70, \ud574\ub2f9 \ub2e8\uccb4\uc758 \uc6f9\uc0ac\uc774\ud2b8\ub294 \ub204\ub9ac\uafbc\ub4e4\uc758 \ud56d\uc758\ubc29\ubb38\uc73c\ub85c \ud55c\ub54c \uc811\uc18d\uc774 \ub2e4\uc6b4\ub418\uae30\ub3c4 \ud588\ub2e4."} {"question": "2013-2014 \uad6d\uc81c\ube59\uc0c1\uacbd\uae30\uc5f0\ub9f9 \uc2a4\ud53c\ub4dc\uc2a4\ucf00\uc774\ud305\uc6d4\ub4dc\ucef5 1\ucc28\ub300\ud68c 3000m\ub294 \uc5b4\ub290\ub098\ub77c\uc5d0\uc11c \uc5f4\ub838\ub294\uac00?", "paragraph": "2013\ub144 11\uc6d4 8\uc77c \uce90\ub098\ub2e4 \uce98\uac70\ub9ac\uc5d0\uc11c \uc5f4\ub9b0 2013-2014 \uad6d\uc81c\ube59\uc0c1\uacbd\uae30\uc5f0\ub9f9 \uc2a4\ud53c\ub4dc\uc2a4\ucf00\uc774\ud305\uc6d4\ub4dc\ucef5 1\ucc28 \ub300\ud68c 3000m\uc5d0\uc11c \uae40\ubcf4\ub984\uc740 4\ubd8407\ucd0893\uc758 \uae30\ub85d\uc73c\ub85c 12\uc704\uc5d0 \uba38\ubb3c\ub800\ub2e4. 9\uc77c \uc5f4\ub9b0 1500m\uc5d0\uc11c\ub294 \uae40\ubcf4\ub984\uc774 1\ubd8456\ucd0894 \ub9cc\uc5d0 \uacb0\uc2b9\uc120\uc744 \ud1b5\uacfc, 20\uba85\uc758 \uc120\uc218 \uc911 \ucd5c\ud558\uc704\uc5d0 \uba38\ubb3c\ub800\ub2e4. 10\uc77c \uc5f4\ub9b0 \ud300\ucd94\uc6d4\uc5d0\uc11c\ub294 \uae40\ubcf4\ub984, \ub178\uc120\uc601, \uc591\uc2e0\uc601\uc774 3\ubd8400\ucd0832\ub85c 5\uc704\ub97c \ucc28\uc9c0\ud588\ub2e4. 2013\ub144 11\uc6d4 15\uc77c \ubbf8\uad6d \uc194\ud2b8\ub808\uc774\ud06c\uc2dc\ud2f0\uc5d0\uc11c \uc5f4\ub9b0 \uc6d4\ub4dc\ucef5 2\ucc28 \ub300\ud68c 3000m \ub514\ube44\uc804A\uc5d0\uc11c \uae40\ubcf4\ub984\uc740 4\ubd8404\ucd0862\ub85c 15\uc704\ub97c \ucc28\uc9c0\ud588\ub2e4. 16\uc77c \uc5f4\ub9b0 1500m \ub514\ube44\uc804B\uc5d0\uc11c\ub294 \uae40\ubcf4\ub984\uc774 1\ubd8456\ucd0812\ub85c \uac1c\uc778\ucd5c\uace0\uae30\ub85d\uc744 \uc138\uc6b0\uba70 8\uc704\ub97c \ucc28\uc9c0\ud588\ub2e4. 17\uc77c \uc5f4\ub9b0 \ud300\ucd94\uc6d4\uc5d0\uc11c\ub294 \uae40\ubcf4\ub984, \ub178\uc120\uc601, \ubc15\ub3c4\uc601\uc774 2\ubd8458\ucd0832\ub85c 7\uc704\ub97c \ucc28\uc9c0\ud588\ub2e4. 2013\ub144 11\uc6d4 29\uc77c \uce74\uc790\ud750\uc2a4\ud0c4 \uc544\uc2a4\ud0c0\ub098\uc5d0\uc11c \uc5f4\ub9b0 \uc6d4\ub4dc\ucef5 2\ucc28 \ub300\ud68c 5000m \ub514\ube44\uc804B\uc5d0\uc11c \uae40\ubcf4\ub984\uc740 7\ubd8409\ucd0899\ub85c 6\uc704\ub97c \ucc28\uc9c0\ud588\ub2e4. 30\uc77c \uc5f4\ub9b0 1500m \ub514\ube44\uc804B\uc5d0\uc11c\ub294 \uae40\ubcf4\ub984\uc774 1\ubd8458\ucd0812\ub85c 1\uc704\ub97c \ucc28\uc9c0\ud588\ub2e4. 2013\ub144 12\uc6d4 9\uc77c \ub3c5\uc77c \ubca0\ub97c\ub9b0\uc5d0\uc11c \uc5f4\ub9b0 \uc6d4\ub4dc\ucef5 4\ucc28 \ub300\ud68c \ud300\ucd94\uc6d4\uc5d0 \uae40\ubcf4\ub984\uc740 \ub178\uc120\uc601(24-\uac15\uc6d0\ub3c4\uccad), \uc591\uc2e0\uc601(23-\uc804\ub77c\ubd81\ub3c4\uccad)\uacfc \ud568\uaed8 \ucd9c\uc804\ud574 3\ubd8402\ucd0804\uc758 \uae30\ub85d\uc73c\ub85c \ub124\ub35c\ub780\ub4dc(2\ubd8458\ucd0819), \ud3f4\ub780\ub4dc(3\ubd841\ucd0818)\uc5d0 \uc774\uc5b4 3\uc704\uc5d0 \uc62c\ub790\ub2e4.", "answer": "\uce90\ub098\ub2e4", "sentence": "2013\ub144 11\uc6d4 8\uc77c \uce90\ub098\ub2e4 \uce98\uac70\ub9ac\uc5d0\uc11c \uc5f4\ub9b0 2013-2014 \uad6d\uc81c\ube59\uc0c1\uacbd\uae30\uc5f0\ub9f9 \uc2a4\ud53c\ub4dc\uc2a4\ucf00\uc774\ud305\uc6d4\ub4dc\ucef5 1\ucc28 \ub300\ud68c 3000m\uc5d0\uc11c \uae40\ubcf4\ub984\uc740 4\ubd8407\ucd0893\uc758 \uae30\ub85d\uc73c\ub85c 12\uc704\uc5d0 \uba38\ubb3c\ub800\ub2e4.", "paragraph_sentence": " 2013\ub144 11\uc6d4 8\uc77c \uce90\ub098\ub2e4 \uce98\uac70\ub9ac\uc5d0\uc11c \uc5f4\ub9b0 2013-2014 \uad6d\uc81c\ube59\uc0c1\uacbd\uae30\uc5f0\ub9f9 \uc2a4\ud53c\ub4dc\uc2a4\ucf00\uc774\ud305\uc6d4\ub4dc\ucef5 1\ucc28 \ub300\ud68c 3000m\uc5d0\uc11c \uae40\ubcf4\ub984\uc740 4\ubd8407\ucd0893\uc758 \uae30\ub85d\uc73c\ub85c 12\uc704\uc5d0 \uba38\ubb3c\ub800\ub2e4. 9\uc77c \uc5f4\ub9b0 1500m\uc5d0\uc11c\ub294 \uae40\ubcf4\ub984\uc774 1\ubd8456\ucd0894 \ub9cc\uc5d0 \uacb0\uc2b9\uc120\uc744 \ud1b5\uacfc, 20\uba85\uc758 \uc120\uc218 \uc911 \ucd5c\ud558\uc704\uc5d0 \uba38\ubb3c\ub800\ub2e4. 10\uc77c \uc5f4\ub9b0 \ud300\ucd94\uc6d4\uc5d0\uc11c\ub294 \uae40\ubcf4\ub984, \ub178\uc120\uc601, \uc591\uc2e0\uc601\uc774 3\ubd8400\ucd0832\ub85c 5\uc704\ub97c \ucc28\uc9c0\ud588\ub2e4. 2013\ub144 11\uc6d4 15\uc77c \ubbf8\uad6d \uc194\ud2b8\ub808\uc774\ud06c\uc2dc\ud2f0\uc5d0\uc11c \uc5f4\ub9b0 \uc6d4\ub4dc\ucef5 2\ucc28 \ub300\ud68c 3000m \ub514\ube44\uc804A\uc5d0\uc11c \uae40\ubcf4\ub984\uc740 4\ubd8404\ucd0862\ub85c 15\uc704\ub97c \ucc28\uc9c0\ud588\ub2e4. 16\uc77c \uc5f4\ub9b0 1500m \ub514\ube44\uc804B\uc5d0\uc11c\ub294 \uae40\ubcf4\ub984\uc774 1\ubd8456\ucd0812\ub85c \uac1c\uc778\ucd5c\uace0\uae30\ub85d\uc744 \uc138\uc6b0\uba70 8\uc704\ub97c \ucc28\uc9c0\ud588\ub2e4. 17\uc77c \uc5f4\ub9b0 \ud300\ucd94\uc6d4\uc5d0\uc11c\ub294 \uae40\ubcf4\ub984, \ub178\uc120\uc601, \ubc15\ub3c4\uc601\uc774 2\ubd8458\ucd0832\ub85c 7\uc704\ub97c \ucc28\uc9c0\ud588\ub2e4. 2013\ub144 11\uc6d4 29\uc77c \uce74\uc790\ud750\uc2a4\ud0c4 \uc544\uc2a4\ud0c0\ub098\uc5d0\uc11c \uc5f4\ub9b0 \uc6d4\ub4dc\ucef5 2\ucc28 \ub300\ud68c 5000m \ub514\ube44\uc804B\uc5d0\uc11c \uae40\ubcf4\ub984\uc740 7\ubd8409\ucd0899\ub85c 6\uc704\ub97c \ucc28\uc9c0\ud588\ub2e4. 30\uc77c \uc5f4\ub9b0 1500m \ub514\ube44\uc804B\uc5d0\uc11c\ub294 \uae40\ubcf4\ub984\uc774 1\ubd8458\ucd0812\ub85c 1\uc704\ub97c \ucc28\uc9c0\ud588\ub2e4. 2013\ub144 12\uc6d4 9\uc77c \ub3c5\uc77c \ubca0\ub97c\ub9b0\uc5d0\uc11c \uc5f4\ub9b0 \uc6d4\ub4dc\ucef5 4\ucc28 \ub300\ud68c \ud300\ucd94\uc6d4\uc5d0 \uae40\ubcf4\ub984\uc740 \ub178\uc120\uc601(24-\uac15\uc6d0\ub3c4\uccad), \uc591\uc2e0\uc601(23-\uc804\ub77c\ubd81\ub3c4\uccad)\uacfc \ud568\uaed8 \ucd9c\uc804\ud574 3\ubd8402\ucd0804\uc758 \uae30\ub85d\uc73c\ub85c \ub124\ub35c\ub780\ub4dc(2\ubd8458\ucd0819), \ud3f4\ub780\ub4dc(3\ubd841\ucd0818)\uc5d0 \uc774\uc5b4 3\uc704\uc5d0 \uc62c\ub790\ub2e4.", "paragraph_answer": "2013\ub144 11\uc6d4 8\uc77c \uce90\ub098\ub2e4 \uce98\uac70\ub9ac\uc5d0\uc11c \uc5f4\ub9b0 2013-2014 \uad6d\uc81c\ube59\uc0c1\uacbd\uae30\uc5f0\ub9f9 \uc2a4\ud53c\ub4dc\uc2a4\ucf00\uc774\ud305\uc6d4\ub4dc\ucef5 1\ucc28 \ub300\ud68c 3000m\uc5d0\uc11c \uae40\ubcf4\ub984\uc740 4\ubd8407\ucd0893\uc758 \uae30\ub85d\uc73c\ub85c 12\uc704\uc5d0 \uba38\ubb3c\ub800\ub2e4. 9\uc77c \uc5f4\ub9b0 1500m\uc5d0\uc11c\ub294 \uae40\ubcf4\ub984\uc774 1\ubd8456\ucd0894 \ub9cc\uc5d0 \uacb0\uc2b9\uc120\uc744 \ud1b5\uacfc, 20\uba85\uc758 \uc120\uc218 \uc911 \ucd5c\ud558\uc704\uc5d0 \uba38\ubb3c\ub800\ub2e4. 10\uc77c \uc5f4\ub9b0 \ud300\ucd94\uc6d4\uc5d0\uc11c\ub294 \uae40\ubcf4\ub984, \ub178\uc120\uc601, \uc591\uc2e0\uc601\uc774 3\ubd8400\ucd0832\ub85c 5\uc704\ub97c \ucc28\uc9c0\ud588\ub2e4. 2013\ub144 11\uc6d4 15\uc77c \ubbf8\uad6d \uc194\ud2b8\ub808\uc774\ud06c\uc2dc\ud2f0\uc5d0\uc11c \uc5f4\ub9b0 \uc6d4\ub4dc\ucef5 2\ucc28 \ub300\ud68c 3000m \ub514\ube44\uc804A\uc5d0\uc11c \uae40\ubcf4\ub984\uc740 4\ubd8404\ucd0862\ub85c 15\uc704\ub97c \ucc28\uc9c0\ud588\ub2e4. 16\uc77c \uc5f4\ub9b0 1500m \ub514\ube44\uc804B\uc5d0\uc11c\ub294 \uae40\ubcf4\ub984\uc774 1\ubd8456\ucd0812\ub85c \uac1c\uc778\ucd5c\uace0\uae30\ub85d\uc744 \uc138\uc6b0\uba70 8\uc704\ub97c \ucc28\uc9c0\ud588\ub2e4. 17\uc77c \uc5f4\ub9b0 \ud300\ucd94\uc6d4\uc5d0\uc11c\ub294 \uae40\ubcf4\ub984, \ub178\uc120\uc601, \ubc15\ub3c4\uc601\uc774 2\ubd8458\ucd0832\ub85c 7\uc704\ub97c \ucc28\uc9c0\ud588\ub2e4. 2013\ub144 11\uc6d4 29\uc77c \uce74\uc790\ud750\uc2a4\ud0c4 \uc544\uc2a4\ud0c0\ub098\uc5d0\uc11c \uc5f4\ub9b0 \uc6d4\ub4dc\ucef5 2\ucc28 \ub300\ud68c 5000m \ub514\ube44\uc804B\uc5d0\uc11c \uae40\ubcf4\ub984\uc740 7\ubd8409\ucd0899\ub85c 6\uc704\ub97c \ucc28\uc9c0\ud588\ub2e4. 30\uc77c \uc5f4\ub9b0 1500m \ub514\ube44\uc804B\uc5d0\uc11c\ub294 \uae40\ubcf4\ub984\uc774 1\ubd8458\ucd0812\ub85c 1\uc704\ub97c \ucc28\uc9c0\ud588\ub2e4. 2013\ub144 12\uc6d4 9\uc77c \ub3c5\uc77c \ubca0\ub97c\ub9b0\uc5d0\uc11c \uc5f4\ub9b0 \uc6d4\ub4dc\ucef5 4\ucc28 \ub300\ud68c \ud300\ucd94\uc6d4\uc5d0 \uae40\ubcf4\ub984\uc740 \ub178\uc120\uc601(24-\uac15\uc6d0\ub3c4\uccad), \uc591\uc2e0\uc601(23-\uc804\ub77c\ubd81\ub3c4\uccad)\uacfc \ud568\uaed8 \ucd9c\uc804\ud574 3\ubd8402\ucd0804\uc758 \uae30\ub85d\uc73c\ub85c \ub124\ub35c\ub780\ub4dc(2\ubd8458\ucd0819), \ud3f4\ub780\ub4dc(3\ubd841\ucd0818)\uc5d0 \uc774\uc5b4 3\uc704\uc5d0 \uc62c\ub790\ub2e4.", "sentence_answer": "2013\ub144 11\uc6d4 8\uc77c \uce90\ub098\ub2e4 \uce98\uac70\ub9ac\uc5d0\uc11c \uc5f4\ub9b0 2013-2014 \uad6d\uc81c\ube59\uc0c1\uacbd\uae30\uc5f0\ub9f9 \uc2a4\ud53c\ub4dc\uc2a4\ucf00\uc774\ud305\uc6d4\ub4dc\ucef5 1\ucc28 \ub300\ud68c 3000m\uc5d0\uc11c \uae40\ubcf4\ub984\uc740 4\ubd8407\ucd0893\uc758 \uae30\ub85d\uc73c\ub85c 12\uc704\uc5d0 \uba38\ubb3c\ub800\ub2e4.", "paragraph_id": "6488275-6-0", "paragraph_question": "question: 2013-2014 \uad6d\uc81c\ube59\uc0c1\uacbd\uae30\uc5f0\ub9f9 \uc2a4\ud53c\ub4dc\uc2a4\ucf00\uc774\ud305\uc6d4\ub4dc\ucef5 1\ucc28\ub300\ud68c 3000m\ub294 \uc5b4\ub290\ub098\ub77c\uc5d0\uc11c \uc5f4\ub838\ub294\uac00?, context: 2013\ub144 11\uc6d4 8\uc77c \uce90\ub098\ub2e4 \uce98\uac70\ub9ac\uc5d0\uc11c \uc5f4\ub9b0 2013-2014 \uad6d\uc81c\ube59\uc0c1\uacbd\uae30\uc5f0\ub9f9 \uc2a4\ud53c\ub4dc\uc2a4\ucf00\uc774\ud305\uc6d4\ub4dc\ucef5 1\ucc28 \ub300\ud68c 3000m\uc5d0\uc11c \uae40\ubcf4\ub984\uc740 4\ubd8407\ucd0893\uc758 \uae30\ub85d\uc73c\ub85c 12\uc704\uc5d0 \uba38\ubb3c\ub800\ub2e4. 9\uc77c \uc5f4\ub9b0 1500m\uc5d0\uc11c\ub294 \uae40\ubcf4\ub984\uc774 1\ubd8456\ucd0894 \ub9cc\uc5d0 \uacb0\uc2b9\uc120\uc744 \ud1b5\uacfc, 20\uba85\uc758 \uc120\uc218 \uc911 \ucd5c\ud558\uc704\uc5d0 \uba38\ubb3c\ub800\ub2e4. 10\uc77c \uc5f4\ub9b0 \ud300\ucd94\uc6d4\uc5d0\uc11c\ub294 \uae40\ubcf4\ub984, \ub178\uc120\uc601, \uc591\uc2e0\uc601\uc774 3\ubd8400\ucd0832\ub85c 5\uc704\ub97c \ucc28\uc9c0\ud588\ub2e4. 2013\ub144 11\uc6d4 15\uc77c \ubbf8\uad6d \uc194\ud2b8\ub808\uc774\ud06c\uc2dc\ud2f0\uc5d0\uc11c \uc5f4\ub9b0 \uc6d4\ub4dc\ucef5 2\ucc28 \ub300\ud68c 3000m \ub514\ube44\uc804A\uc5d0\uc11c \uae40\ubcf4\ub984\uc740 4\ubd8404\ucd0862\ub85c 15\uc704\ub97c \ucc28\uc9c0\ud588\ub2e4. 16\uc77c \uc5f4\ub9b0 1500m \ub514\ube44\uc804B\uc5d0\uc11c\ub294 \uae40\ubcf4\ub984\uc774 1\ubd8456\ucd0812\ub85c \uac1c\uc778\ucd5c\uace0\uae30\ub85d\uc744 \uc138\uc6b0\uba70 8\uc704\ub97c \ucc28\uc9c0\ud588\ub2e4. 17\uc77c \uc5f4\ub9b0 \ud300\ucd94\uc6d4\uc5d0\uc11c\ub294 \uae40\ubcf4\ub984, \ub178\uc120\uc601, \ubc15\ub3c4\uc601\uc774 2\ubd8458\ucd0832\ub85c 7\uc704\ub97c \ucc28\uc9c0\ud588\ub2e4. 2013\ub144 11\uc6d4 29\uc77c \uce74\uc790\ud750\uc2a4\ud0c4 \uc544\uc2a4\ud0c0\ub098\uc5d0\uc11c \uc5f4\ub9b0 \uc6d4\ub4dc\ucef5 2\ucc28 \ub300\ud68c 5000m \ub514\ube44\uc804B\uc5d0\uc11c \uae40\ubcf4\ub984\uc740 7\ubd8409\ucd0899\ub85c 6\uc704\ub97c \ucc28\uc9c0\ud588\ub2e4. 30\uc77c \uc5f4\ub9b0 1500m \ub514\ube44\uc804B\uc5d0\uc11c\ub294 \uae40\ubcf4\ub984\uc774 1\ubd8458\ucd0812\ub85c 1\uc704\ub97c \ucc28\uc9c0\ud588\ub2e4. 2013\ub144 12\uc6d4 9\uc77c \ub3c5\uc77c \ubca0\ub97c\ub9b0\uc5d0\uc11c \uc5f4\ub9b0 \uc6d4\ub4dc\ucef5 4\ucc28 \ub300\ud68c \ud300\ucd94\uc6d4\uc5d0 \uae40\ubcf4\ub984\uc740 \ub178\uc120\uc601(24-\uac15\uc6d0\ub3c4\uccad), \uc591\uc2e0\uc601(23-\uc804\ub77c\ubd81\ub3c4\uccad)\uacfc \ud568\uaed8 \ucd9c\uc804\ud574 3\ubd8402\ucd0804\uc758 \uae30\ub85d\uc73c\ub85c \ub124\ub35c\ub780\ub4dc(2\ubd8458\ucd0819), \ud3f4\ub780\ub4dc(3\ubd841\ucd0818)\uc5d0 \uc774\uc5b4 3\uc704\uc5d0 \uc62c\ub790\ub2e4."} {"question": "\uc624\ub798\ub41c \uc778\ub958\uc758 \uc720\uc801\ub4f1\uc744 \uc5f0\uad6c\ud558\ub294 \uc778\ub958\ud559\uc758 \ud558\uc704 \ud559\ubb38\uc744\ubb34\uc5c7\uc774\ub77c\uace0 \ud558\ub294\uac00?", "paragraph": "\uc778\ub958\ud559\uc740 \uc778\uac04\uc5d0 \uad00\ud55c \ubaa8\ub4e0 \uac83\uc744 \uc5f0\uad6c\ud558\ub294 \ud559\ubb38\uc774\ub2e4. \ub530\ub77c\uc11c \uc5f0\uad6c\uc758 \ub300\uc0c1\uacfc \ubc94\uc704\uac00 \ub9e4\uc6b0 \uad11\ubc94\uc704\ud558\ub2e4. 19\uc138\uae30 \uc774\ud6c4 \ud559\ubb38\uc73c\ub85c\uc11c \uccb4\uacc4\ud654\ub418\uc5c8\uc73c\uba70 \ubbf8\uad6d\uacfc \uc720\ub7fd\uc5d0\uc11c \uc11c\ub85c \ub2e4\ub978 \uad00\uc2ec\uc744 \uac16\uace0 \ubc1c\uc804\ub418\uc5b4 \uc654\ub2e4. \uc624\ub298\ub0a0 \uc778\ub958\ud559\uc740 \ubcf4\ub2e4 \uc804\ubb38\uc801\uc778 \uc5ec\ub7ec \ud558\uc704 \ud559\ubb38\uc73c\ub85c \ub098\ub258\uc5b4 \uc788\ub2e4. \ud558\uc704 \ud559\ubb38\uc73c\ub85c\ub294 \ud615\uc9c8\uc778\ub958\ud559, \uace0\uace0\ud559, \ubb38\ud654\uc778\ub958\ud559 \ub4f1\uc774 \uc788\ub2e4. \ud615\uc9c8\uc778\ub958\ud559\uc758 \uc5f0\uad6c\ubd84\uc57c\ub85c\ub294 \uc778\uac04\uc758 \uae30\uc6d0\uacfc \uc9c4\ud654\ub97c \ub2e4\ub8e8\ub294 \ubd84\uc57c\uc640 \ud604\ub300 \uc778\ub958\uc758 \ub2e4\uc591\uc131\uc744 \uc5f0\uad6c\ud558\ub294 \ubd84\uc57c\uac00 \uc788\ub2e4. \uace0\uace0\ud559\uc740 \uc120\uc0ac\uc2dc\ub300\uc640 \uac19\uc740 \uc624\ub798\ub41c \uc778\ub958\uc758 \uc720\uc801 \ub4f1\uc744 \uc5f0\uad6c\ud558\uc5ec \ub2f9\uc2dc\uc758 \ubb38\ud654 \ub4f1\uc744 \uaddc\uba85\ud558\ub294 \uac83\uc744 \uc8fc\uc694 \uc5f0\uad6c\ubd84\uc57c\ub85c \uc0bc\uace0\uc788\ub2e4. \ubb38\ud654\uc778\ub958\ud559\uc740 \uc624\ub298\ub0a0 \ub2e4\uc591\ud558\uac8c \uc874\uc7ac\ud558\ub294 \uc5ec\ub7ec \ubb38\ud654\ub4e4\uc5d0 \ub300\ud574 \ucd1d\uccb4\uc801\uc73c\ub85c \uc5f0\uad6c\ud558\ub294 \uac83\uc774\ub2e4. \uc5ec\uc804\ud788 \uc11d\uae30\ub97c \uc0ac\uc6a9\ud558\ub294 \ubb38\ud654\uc5d0\uc11c\ubd80\ud130 \ud604\ub300 \uc0b0\uc5c5\uc0ac\ud68c\uc5d0 \uc774\ub974\uae30\uae4c\uc9c0 \ub2e4\uc591\ud55c \ubb38\ud654\uc5d0 \ub300\ud574\uc11c \uac00\uc871, \uce5c\uc871\uad00\uacc4, \uacbd\uc81c\uc0dd\ud65c, \uc815\uce58\uc0dd\ud65c, \uc885\uad50\uc0dd\ud65c\uacfc \uac19\uc740 \uc5ec\ub7ec \ubb38\ud654\uc801 \uc0dd\ud65c\uc744 \uc5f0\uad6c\ud558\uc5ec \ud574\ub2f9 \uc0ac\ud68c\uc758 \ub0b4\uc801 \uaddc\uce59\uc744 \ubc1c\uacac\ud558\uace0\uc790 \ud55c\ub2e4.", "answer": "\uace0\uace0\ud559", "sentence": "\ud558\uc704 \ud559\ubb38\uc73c\ub85c\ub294 \ud615\uc9c8\uc778\ub958\ud559, \uace0\uace0\ud559 ,", "paragraph_sentence": "\uc778\ub958\ud559\uc740 \uc778\uac04\uc5d0 \uad00\ud55c \ubaa8\ub4e0 \uac83\uc744 \uc5f0\uad6c\ud558\ub294 \ud559\ubb38\uc774\ub2e4. \ub530\ub77c\uc11c \uc5f0\uad6c\uc758 \ub300\uc0c1\uacfc \ubc94\uc704\uac00 \ub9e4\uc6b0 \uad11\ubc94\uc704\ud558\ub2e4. 19\uc138\uae30 \uc774\ud6c4 \ud559\ubb38\uc73c\ub85c\uc11c \uccb4\uacc4\ud654\ub418\uc5c8\uc73c\uba70 \ubbf8\uad6d\uacfc \uc720\ub7fd\uc5d0\uc11c \uc11c\ub85c \ub2e4\ub978 \uad00\uc2ec\uc744 \uac16\uace0 \ubc1c\uc804\ub418\uc5b4 \uc654\ub2e4. \uc624\ub298\ub0a0 \uc778\ub958\ud559\uc740 \ubcf4\ub2e4 \uc804\ubb38\uc801\uc778 \uc5ec\ub7ec \ud558\uc704 \ud559\ubb38\uc73c\ub85c \ub098\ub258\uc5b4 \uc788\ub2e4. \ud558\uc704 \ud559\ubb38\uc73c\ub85c\ub294 \ud615\uc9c8\uc778\ub958\ud559, \uace0\uace0\ud559 , \ubb38\ud654\uc778\ub958\ud559 \ub4f1\uc774 \uc788\ub2e4. \ud615\uc9c8\uc778\ub958\ud559\uc758 \uc5f0\uad6c\ubd84\uc57c\ub85c\ub294 \uc778\uac04\uc758 \uae30\uc6d0\uacfc \uc9c4\ud654\ub97c \ub2e4\ub8e8\ub294 \ubd84\uc57c\uc640 \ud604\ub300 \uc778\ub958\uc758 \ub2e4\uc591\uc131\uc744 \uc5f0\uad6c\ud558\ub294 \ubd84\uc57c\uac00 \uc788\ub2e4. \uace0\uace0\ud559\uc740 \uc120\uc0ac\uc2dc\ub300\uc640 \uac19\uc740 \uc624\ub798\ub41c \uc778\ub958\uc758 \uc720\uc801 \ub4f1\uc744 \uc5f0\uad6c\ud558\uc5ec \ub2f9\uc2dc\uc758 \ubb38\ud654 \ub4f1\uc744 \uaddc\uba85\ud558\ub294 \uac83\uc744 \uc8fc\uc694 \uc5f0\uad6c\ubd84\uc57c\ub85c \uc0bc\uace0\uc788\ub2e4. \ubb38\ud654\uc778\ub958\ud559\uc740 \uc624\ub298\ub0a0 \ub2e4\uc591\ud558\uac8c \uc874\uc7ac\ud558\ub294 \uc5ec\ub7ec \ubb38\ud654\ub4e4\uc5d0 \ub300\ud574 \ucd1d\uccb4\uc801\uc73c\ub85c \uc5f0\uad6c\ud558\ub294 \uac83\uc774\ub2e4. \uc5ec\uc804\ud788 \uc11d\uae30\ub97c \uc0ac\uc6a9\ud558\ub294 \ubb38\ud654\uc5d0\uc11c\ubd80\ud130 \ud604\ub300 \uc0b0\uc5c5\uc0ac\ud68c\uc5d0 \uc774\ub974\uae30\uae4c\uc9c0 \ub2e4\uc591\ud55c \ubb38\ud654\uc5d0 \ub300\ud574\uc11c \uac00\uc871, \uce5c\uc871\uad00\uacc4, \uacbd\uc81c\uc0dd\ud65c, \uc815\uce58\uc0dd\ud65c, \uc885\uad50\uc0dd\ud65c\uacfc \uac19\uc740 \uc5ec\ub7ec \ubb38\ud654\uc801 \uc0dd\ud65c\uc744 \uc5f0\uad6c\ud558\uc5ec \ud574\ub2f9 \uc0ac\ud68c\uc758 \ub0b4\uc801 \uaddc\uce59\uc744 \ubc1c\uacac\ud558\uace0\uc790 \ud55c\ub2e4.", "paragraph_answer": "\uc778\ub958\ud559\uc740 \uc778\uac04\uc5d0 \uad00\ud55c \ubaa8\ub4e0 \uac83\uc744 \uc5f0\uad6c\ud558\ub294 \ud559\ubb38\uc774\ub2e4. \ub530\ub77c\uc11c \uc5f0\uad6c\uc758 \ub300\uc0c1\uacfc \ubc94\uc704\uac00 \ub9e4\uc6b0 \uad11\ubc94\uc704\ud558\ub2e4. 19\uc138\uae30 \uc774\ud6c4 \ud559\ubb38\uc73c\ub85c\uc11c \uccb4\uacc4\ud654\ub418\uc5c8\uc73c\uba70 \ubbf8\uad6d\uacfc \uc720\ub7fd\uc5d0\uc11c \uc11c\ub85c \ub2e4\ub978 \uad00\uc2ec\uc744 \uac16\uace0 \ubc1c\uc804\ub418\uc5b4 \uc654\ub2e4. \uc624\ub298\ub0a0 \uc778\ub958\ud559\uc740 \ubcf4\ub2e4 \uc804\ubb38\uc801\uc778 \uc5ec\ub7ec \ud558\uc704 \ud559\ubb38\uc73c\ub85c \ub098\ub258\uc5b4 \uc788\ub2e4. \ud558\uc704 \ud559\ubb38\uc73c\ub85c\ub294 \ud615\uc9c8\uc778\ub958\ud559, \uace0\uace0\ud559 , \ubb38\ud654\uc778\ub958\ud559 \ub4f1\uc774 \uc788\ub2e4. \ud615\uc9c8\uc778\ub958\ud559\uc758 \uc5f0\uad6c\ubd84\uc57c\ub85c\ub294 \uc778\uac04\uc758 \uae30\uc6d0\uacfc \uc9c4\ud654\ub97c \ub2e4\ub8e8\ub294 \ubd84\uc57c\uc640 \ud604\ub300 \uc778\ub958\uc758 \ub2e4\uc591\uc131\uc744 \uc5f0\uad6c\ud558\ub294 \ubd84\uc57c\uac00 \uc788\ub2e4. \uace0\uace0\ud559\uc740 \uc120\uc0ac\uc2dc\ub300\uc640 \uac19\uc740 \uc624\ub798\ub41c \uc778\ub958\uc758 \uc720\uc801 \ub4f1\uc744 \uc5f0\uad6c\ud558\uc5ec \ub2f9\uc2dc\uc758 \ubb38\ud654 \ub4f1\uc744 \uaddc\uba85\ud558\ub294 \uac83\uc744 \uc8fc\uc694 \uc5f0\uad6c\ubd84\uc57c\ub85c \uc0bc\uace0\uc788\ub2e4. \ubb38\ud654\uc778\ub958\ud559\uc740 \uc624\ub298\ub0a0 \ub2e4\uc591\ud558\uac8c \uc874\uc7ac\ud558\ub294 \uc5ec\ub7ec \ubb38\ud654\ub4e4\uc5d0 \ub300\ud574 \ucd1d\uccb4\uc801\uc73c\ub85c \uc5f0\uad6c\ud558\ub294 \uac83\uc774\ub2e4. \uc5ec\uc804\ud788 \uc11d\uae30\ub97c \uc0ac\uc6a9\ud558\ub294 \ubb38\ud654\uc5d0\uc11c\ubd80\ud130 \ud604\ub300 \uc0b0\uc5c5\uc0ac\ud68c\uc5d0 \uc774\ub974\uae30\uae4c\uc9c0 \ub2e4\uc591\ud55c \ubb38\ud654\uc5d0 \ub300\ud574\uc11c \uac00\uc871, \uce5c\uc871\uad00\uacc4, \uacbd\uc81c\uc0dd\ud65c, \uc815\uce58\uc0dd\ud65c, \uc885\uad50\uc0dd\ud65c\uacfc \uac19\uc740 \uc5ec\ub7ec \ubb38\ud654\uc801 \uc0dd\ud65c\uc744 \uc5f0\uad6c\ud558\uc5ec \ud574\ub2f9 \uc0ac\ud68c\uc758 \ub0b4\uc801 \uaddc\uce59\uc744 \ubc1c\uacac\ud558\uace0\uc790 \ud55c\ub2e4.", "sentence_answer": "\ud558\uc704 \ud559\ubb38\uc73c\ub85c\ub294 \ud615\uc9c8\uc778\ub958\ud559, \uace0\uace0\ud559 ,", "paragraph_id": "6517478-0-0", "paragraph_question": "question: \uc624\ub798\ub41c \uc778\ub958\uc758 \uc720\uc801\ub4f1\uc744 \uc5f0\uad6c\ud558\ub294 \uc778\ub958\ud559\uc758 \ud558\uc704 \ud559\ubb38\uc744\ubb34\uc5c7\uc774\ub77c\uace0 \ud558\ub294\uac00?, context: \uc778\ub958\ud559\uc740 \uc778\uac04\uc5d0 \uad00\ud55c \ubaa8\ub4e0 \uac83\uc744 \uc5f0\uad6c\ud558\ub294 \ud559\ubb38\uc774\ub2e4. \ub530\ub77c\uc11c \uc5f0\uad6c\uc758 \ub300\uc0c1\uacfc \ubc94\uc704\uac00 \ub9e4\uc6b0 \uad11\ubc94\uc704\ud558\ub2e4. 19\uc138\uae30 \uc774\ud6c4 \ud559\ubb38\uc73c\ub85c\uc11c \uccb4\uacc4\ud654\ub418\uc5c8\uc73c\uba70 \ubbf8\uad6d\uacfc \uc720\ub7fd\uc5d0\uc11c \uc11c\ub85c \ub2e4\ub978 \uad00\uc2ec\uc744 \uac16\uace0 \ubc1c\uc804\ub418\uc5b4 \uc654\ub2e4. \uc624\ub298\ub0a0 \uc778\ub958\ud559\uc740 \ubcf4\ub2e4 \uc804\ubb38\uc801\uc778 \uc5ec\ub7ec \ud558\uc704 \ud559\ubb38\uc73c\ub85c \ub098\ub258\uc5b4 \uc788\ub2e4. \ud558\uc704 \ud559\ubb38\uc73c\ub85c\ub294 \ud615\uc9c8\uc778\ub958\ud559, \uace0\uace0\ud559, \ubb38\ud654\uc778\ub958\ud559 \ub4f1\uc774 \uc788\ub2e4. \ud615\uc9c8\uc778\ub958\ud559\uc758 \uc5f0\uad6c\ubd84\uc57c\ub85c\ub294 \uc778\uac04\uc758 \uae30\uc6d0\uacfc \uc9c4\ud654\ub97c \ub2e4\ub8e8\ub294 \ubd84\uc57c\uc640 \ud604\ub300 \uc778\ub958\uc758 \ub2e4\uc591\uc131\uc744 \uc5f0\uad6c\ud558\ub294 \ubd84\uc57c\uac00 \uc788\ub2e4. \uace0\uace0\ud559\uc740 \uc120\uc0ac\uc2dc\ub300\uc640 \uac19\uc740 \uc624\ub798\ub41c \uc778\ub958\uc758 \uc720\uc801 \ub4f1\uc744 \uc5f0\uad6c\ud558\uc5ec \ub2f9\uc2dc\uc758 \ubb38\ud654 \ub4f1\uc744 \uaddc\uba85\ud558\ub294 \uac83\uc744 \uc8fc\uc694 \uc5f0\uad6c\ubd84\uc57c\ub85c \uc0bc\uace0\uc788\ub2e4. \ubb38\ud654\uc778\ub958\ud559\uc740 \uc624\ub298\ub0a0 \ub2e4\uc591\ud558\uac8c \uc874\uc7ac\ud558\ub294 \uc5ec\ub7ec \ubb38\ud654\ub4e4\uc5d0 \ub300\ud574 \ucd1d\uccb4\uc801\uc73c\ub85c \uc5f0\uad6c\ud558\ub294 \uac83\uc774\ub2e4. \uc5ec\uc804\ud788 \uc11d\uae30\ub97c \uc0ac\uc6a9\ud558\ub294 \ubb38\ud654\uc5d0\uc11c\ubd80\ud130 \ud604\ub300 \uc0b0\uc5c5\uc0ac\ud68c\uc5d0 \uc774\ub974\uae30\uae4c\uc9c0 \ub2e4\uc591\ud55c \ubb38\ud654\uc5d0 \ub300\ud574\uc11c \uac00\uc871, \uce5c\uc871\uad00\uacc4, \uacbd\uc81c\uc0dd\ud65c, \uc815\uce58\uc0dd\ud65c, \uc885\uad50\uc0dd\ud65c\uacfc \uac19\uc740 \uc5ec\ub7ec \ubb38\ud654\uc801 \uc0dd\ud65c\uc744 \uc5f0\uad6c\ud558\uc5ec \ud574\ub2f9 \uc0ac\ud68c\uc758 \ub0b4\uc801 \uaddc\uce59\uc744 \ubc1c\uacac\ud558\uace0\uc790 \ud55c\ub2e4."} {"question": "\uc774 \ud559\uad50\uc758 \ub450 \ucea0\ud37c\uc2a4 \uc911\uc5d0\uc11c \ub2e8\uacfc\ub300\ud559\uc774 \ub2e4\uc12f \uac1c\uac00 \uc788\ub294 \uacf3\uc740?", "paragraph": "2017\ub144 \uae30\uc900\uc73c\ub85c \uc11c\uc6b8\ucea0\ud37c\uc2a4\uc5d0\ub294 17\uac1c \ub2e8\uacfc\ub300\ud559\uc774, \uc138\uc885\ucea0\ud37c\uc2a4\uc5d0\ub294 5\uac1c \ub2e8\uacfc\ub300\ud559\uc774 \uc124\uce58\ub3fc \uc788\ub2e4. 2009\ud559\ub144\ub3c4\ubd80\ud130 \ubc95\ud559\uc804\ubb38\ub300\ud559\uc6d0(\ub85c\uc2a4\ucfe8)\uc774 \uac1c\uc6d0\ud568\uc5d0 \ub530\ub77c \ubc95\uacfc\ub300\ud559\uc5d0\uc11c\ub294 \uc2e0\uc785\uc0dd\uc744 \ubc1b\uc9c0 \uc54a\uace0 \uc788\uc73c\uba70, \ub300\uc2e0 \uc790\uc720\uc804\uacf5\ud559\ubd80\ub97c \uc2e0\uc124\ud558\uc5ec \uae30\uc874\uc758 \ubc95\ud559\uacfc \uc2e0\uc785\uc0dd \uc815\uc6d0\uc744 \ub300\uccb4\ud558\uace0 \uc788\ub2e4. \uc0ac\uc774\ubc84\uad6d\ubc29\ud559\uacfc\ub294 \uc815\ubcf4\ub300\ud559 \uc18c\uc18d\uc758 \ud559\uacfc\ub85c \uad6d\ubc29\ubd80\uc640\uc758 \ud611\uc57d\uc744 \ud1b5\ud574 2012\ub144\uc5d0 \ucc98\uc74c \uac1c\uc124\ub41c \ud559\uacfc\uc774\ub2e4. \ucef4\ud4e8\ud130\uad50\uc721\uacfc\uc758 \uacbd\uc6b0 2014\ud559\ub144\ub3c4\ub97c \uc2dc\uc791\uc73c\ub85c \ucef4\ud4e8\ud130\ud559\uacfc\ub85c \uac1c\ud3b8\ub418\uba74\uc11c \ub354\uc774\uc0c1 \uc0ac\ubc94\ub300\ud559\uc774 \uc544\ub2cc \uc815\ubcf4\ud1b5\uc2e0\ub300\ud559\uc758 \uc18c\uc18d\uc73c\ub85c \ubcc0\uacbd\ub410\ub2e4. \uc77c\ubd80 \ud559\uacfc\uc5d0\uc11c\ub294 \ud559\ubd80\uc81c\ub85c \ubaa8\uc9d1\uc744 \ud558\uba70 \ud559\ubd80\uc81c\ub85c \uc785\ud559\ud55c \ud559\uc0dd\uc758 \uacbd\uc6b0 2\ud559\ub144 \uc9c4\uae09\uc2dc \uc790\uc2e0\uc758 \uc804\uacf5\uc744 \ubc30\uc815\ubc1b\ub294\ub370, \uc804\uacf5\uc744 \ubc30\uc815\ud558\ub294 \uae30\uc900\uc740 \ub2e8\uacfc\ub300\ud559 \uc0ac\uc815\uc5d0 \ub530\ub77c \ub2e4\ub974\uac8c \uc124\uc815\ub3fc \uc788\ub2e4. \ud559\ubd80\uacfc\uc815\uc5d0\uc11c\ub294 2004\ub144 \uc785\ud559\uc0dd\ubd80\ud130 \uc2ec\ud654\uc804\uacf5, \uc774\uc911\uc804\uacf5, \uc735\ud569\uc804\uacf5, \ud559\uc0dd\uc124\uacc4\uc804\uacf5\uc73c\ub85c \uc138\ubd84\ud654\ud560 \uc218 \uc788\ub294 \uc81c2\uc804\uacf5\uc758 \uc774\uc218\uac00 \uc758\ubb34\ud654 \ub3fc \uc788\ub2e4.", "answer": "\uc138\uc885\ucea0\ud37c\uc2a4", "sentence": "2017\ub144 \uae30\uc900\uc73c\ub85c \uc11c\uc6b8\ucea0\ud37c\uc2a4\uc5d0\ub294 17\uac1c \ub2e8\uacfc\ub300\ud559\uc774, \uc138\uc885\ucea0\ud37c\uc2a4 \uc5d0\ub294 5\uac1c \ub2e8\uacfc\ub300\ud559\uc774 \uc124\uce58\ub3fc \uc788\ub2e4.", "paragraph_sentence": " 2017\ub144 \uae30\uc900\uc73c\ub85c \uc11c\uc6b8\ucea0\ud37c\uc2a4\uc5d0\ub294 17\uac1c \ub2e8\uacfc\ub300\ud559\uc774, \uc138\uc885\ucea0\ud37c\uc2a4 \uc5d0\ub294 5\uac1c \ub2e8\uacfc\ub300\ud559\uc774 \uc124\uce58\ub3fc \uc788\ub2e4. 2009\ud559\ub144\ub3c4\ubd80\ud130 \ubc95\ud559\uc804\ubb38\ub300\ud559\uc6d0(\ub85c\uc2a4\ucfe8)\uc774 \uac1c\uc6d0\ud568\uc5d0 \ub530\ub77c \ubc95\uacfc\ub300\ud559\uc5d0\uc11c\ub294 \uc2e0\uc785\uc0dd\uc744 \ubc1b\uc9c0 \uc54a\uace0 \uc788\uc73c\uba70, \ub300\uc2e0 \uc790\uc720\uc804\uacf5\ud559\ubd80\ub97c \uc2e0\uc124\ud558\uc5ec \uae30\uc874\uc758 \ubc95\ud559\uacfc \uc2e0\uc785\uc0dd \uc815\uc6d0\uc744 \ub300\uccb4\ud558\uace0 \uc788\ub2e4. \uc0ac\uc774\ubc84\uad6d\ubc29\ud559\uacfc\ub294 \uc815\ubcf4\ub300\ud559 \uc18c\uc18d\uc758 \ud559\uacfc\ub85c \uad6d\ubc29\ubd80\uc640\uc758 \ud611\uc57d\uc744 \ud1b5\ud574 2012\ub144\uc5d0 \ucc98\uc74c \uac1c\uc124\ub41c \ud559\uacfc\uc774\ub2e4. \ucef4\ud4e8\ud130\uad50\uc721\uacfc\uc758 \uacbd\uc6b0 2014\ud559\ub144\ub3c4\ub97c \uc2dc\uc791\uc73c\ub85c \ucef4\ud4e8\ud130\ud559\uacfc\ub85c \uac1c\ud3b8\ub418\uba74\uc11c \ub354\uc774\uc0c1 \uc0ac\ubc94\ub300\ud559\uc774 \uc544\ub2cc \uc815\ubcf4\ud1b5\uc2e0\ub300\ud559\uc758 \uc18c\uc18d\uc73c\ub85c \ubcc0\uacbd\ub410\ub2e4. \uc77c\ubd80 \ud559\uacfc\uc5d0\uc11c\ub294 \ud559\ubd80\uc81c\ub85c \ubaa8\uc9d1\uc744 \ud558\uba70 \ud559\ubd80\uc81c\ub85c \uc785\ud559\ud55c \ud559\uc0dd\uc758 \uacbd\uc6b0 2\ud559\ub144 \uc9c4\uae09\uc2dc \uc790\uc2e0\uc758 \uc804\uacf5\uc744 \ubc30\uc815\ubc1b\ub294\ub370, \uc804\uacf5\uc744 \ubc30\uc815\ud558\ub294 \uae30\uc900\uc740 \ub2e8\uacfc\ub300\ud559 \uc0ac\uc815\uc5d0 \ub530\ub77c \ub2e4\ub974\uac8c \uc124\uc815\ub3fc \uc788\ub2e4. \ud559\ubd80\uacfc\uc815\uc5d0\uc11c\ub294 2004\ub144 \uc785\ud559\uc0dd\ubd80\ud130 \uc2ec\ud654\uc804\uacf5, \uc774\uc911\uc804\uacf5, \uc735\ud569\uc804\uacf5, \ud559\uc0dd\uc124\uacc4\uc804\uacf5\uc73c\ub85c \uc138\ubd84\ud654\ud560 \uc218 \uc788\ub294 \uc81c2\uc804\uacf5\uc758 \uc774\uc218\uac00 \uc758\ubb34\ud654 \ub3fc \uc788\ub2e4.", "paragraph_answer": "2017\ub144 \uae30\uc900\uc73c\ub85c \uc11c\uc6b8\ucea0\ud37c\uc2a4\uc5d0\ub294 17\uac1c \ub2e8\uacfc\ub300\ud559\uc774, \uc138\uc885\ucea0\ud37c\uc2a4 \uc5d0\ub294 5\uac1c \ub2e8\uacfc\ub300\ud559\uc774 \uc124\uce58\ub3fc \uc788\ub2e4. 2009\ud559\ub144\ub3c4\ubd80\ud130 \ubc95\ud559\uc804\ubb38\ub300\ud559\uc6d0(\ub85c\uc2a4\ucfe8)\uc774 \uac1c\uc6d0\ud568\uc5d0 \ub530\ub77c \ubc95\uacfc\ub300\ud559\uc5d0\uc11c\ub294 \uc2e0\uc785\uc0dd\uc744 \ubc1b\uc9c0 \uc54a\uace0 \uc788\uc73c\uba70, \ub300\uc2e0 \uc790\uc720\uc804\uacf5\ud559\ubd80\ub97c \uc2e0\uc124\ud558\uc5ec \uae30\uc874\uc758 \ubc95\ud559\uacfc \uc2e0\uc785\uc0dd \uc815\uc6d0\uc744 \ub300\uccb4\ud558\uace0 \uc788\ub2e4. \uc0ac\uc774\ubc84\uad6d\ubc29\ud559\uacfc\ub294 \uc815\ubcf4\ub300\ud559 \uc18c\uc18d\uc758 \ud559\uacfc\ub85c \uad6d\ubc29\ubd80\uc640\uc758 \ud611\uc57d\uc744 \ud1b5\ud574 2012\ub144\uc5d0 \ucc98\uc74c \uac1c\uc124\ub41c \ud559\uacfc\uc774\ub2e4. \ucef4\ud4e8\ud130\uad50\uc721\uacfc\uc758 \uacbd\uc6b0 2014\ud559\ub144\ub3c4\ub97c \uc2dc\uc791\uc73c\ub85c \ucef4\ud4e8\ud130\ud559\uacfc\ub85c \uac1c\ud3b8\ub418\uba74\uc11c \ub354\uc774\uc0c1 \uc0ac\ubc94\ub300\ud559\uc774 \uc544\ub2cc \uc815\ubcf4\ud1b5\uc2e0\ub300\ud559\uc758 \uc18c\uc18d\uc73c\ub85c \ubcc0\uacbd\ub410\ub2e4. \uc77c\ubd80 \ud559\uacfc\uc5d0\uc11c\ub294 \ud559\ubd80\uc81c\ub85c \ubaa8\uc9d1\uc744 \ud558\uba70 \ud559\ubd80\uc81c\ub85c \uc785\ud559\ud55c \ud559\uc0dd\uc758 \uacbd\uc6b0 2\ud559\ub144 \uc9c4\uae09\uc2dc \uc790\uc2e0\uc758 \uc804\uacf5\uc744 \ubc30\uc815\ubc1b\ub294\ub370, \uc804\uacf5\uc744 \ubc30\uc815\ud558\ub294 \uae30\uc900\uc740 \ub2e8\uacfc\ub300\ud559 \uc0ac\uc815\uc5d0 \ub530\ub77c \ub2e4\ub974\uac8c \uc124\uc815\ub3fc \uc788\ub2e4. \ud559\ubd80\uacfc\uc815\uc5d0\uc11c\ub294 2004\ub144 \uc785\ud559\uc0dd\ubd80\ud130 \uc2ec\ud654\uc804\uacf5, \uc774\uc911\uc804\uacf5, \uc735\ud569\uc804\uacf5, \ud559\uc0dd\uc124\uacc4\uc804\uacf5\uc73c\ub85c \uc138\ubd84\ud654\ud560 \uc218 \uc788\ub294 \uc81c2\uc804\uacf5\uc758 \uc774\uc218\uac00 \uc758\ubb34\ud654 \ub3fc \uc788\ub2e4.", "sentence_answer": "2017\ub144 \uae30\uc900\uc73c\ub85c \uc11c\uc6b8\ucea0\ud37c\uc2a4\uc5d0\ub294 17\uac1c \ub2e8\uacfc\ub300\ud559\uc774, \uc138\uc885\ucea0\ud37c\uc2a4 \uc5d0\ub294 5\uac1c \ub2e8\uacfc\ub300\ud559\uc774 \uc124\uce58\ub3fc \uc788\ub2e4.", "paragraph_id": "5914551-3-0", "paragraph_question": "question: \uc774 \ud559\uad50\uc758 \ub450 \ucea0\ud37c\uc2a4 \uc911\uc5d0\uc11c \ub2e8\uacfc\ub300\ud559\uc774 \ub2e4\uc12f \uac1c\uac00 \uc788\ub294 \uacf3\uc740?, context: 2017\ub144 \uae30\uc900\uc73c\ub85c \uc11c\uc6b8\ucea0\ud37c\uc2a4\uc5d0\ub294 17\uac1c \ub2e8\uacfc\ub300\ud559\uc774, \uc138\uc885\ucea0\ud37c\uc2a4\uc5d0\ub294 5\uac1c \ub2e8\uacfc\ub300\ud559\uc774 \uc124\uce58\ub3fc \uc788\ub2e4. 2009\ud559\ub144\ub3c4\ubd80\ud130 \ubc95\ud559\uc804\ubb38\ub300\ud559\uc6d0(\ub85c\uc2a4\ucfe8)\uc774 \uac1c\uc6d0\ud568\uc5d0 \ub530\ub77c \ubc95\uacfc\ub300\ud559\uc5d0\uc11c\ub294 \uc2e0\uc785\uc0dd\uc744 \ubc1b\uc9c0 \uc54a\uace0 \uc788\uc73c\uba70, \ub300\uc2e0 \uc790\uc720\uc804\uacf5\ud559\ubd80\ub97c \uc2e0\uc124\ud558\uc5ec \uae30\uc874\uc758 \ubc95\ud559\uacfc \uc2e0\uc785\uc0dd \uc815\uc6d0\uc744 \ub300\uccb4\ud558\uace0 \uc788\ub2e4. \uc0ac\uc774\ubc84\uad6d\ubc29\ud559\uacfc\ub294 \uc815\ubcf4\ub300\ud559 \uc18c\uc18d\uc758 \ud559\uacfc\ub85c \uad6d\ubc29\ubd80\uc640\uc758 \ud611\uc57d\uc744 \ud1b5\ud574 2012\ub144\uc5d0 \ucc98\uc74c \uac1c\uc124\ub41c \ud559\uacfc\uc774\ub2e4. \ucef4\ud4e8\ud130\uad50\uc721\uacfc\uc758 \uacbd\uc6b0 2014\ud559\ub144\ub3c4\ub97c \uc2dc\uc791\uc73c\ub85c \ucef4\ud4e8\ud130\ud559\uacfc\ub85c \uac1c\ud3b8\ub418\uba74\uc11c \ub354\uc774\uc0c1 \uc0ac\ubc94\ub300\ud559\uc774 \uc544\ub2cc \uc815\ubcf4\ud1b5\uc2e0\ub300\ud559\uc758 \uc18c\uc18d\uc73c\ub85c \ubcc0\uacbd\ub410\ub2e4. \uc77c\ubd80 \ud559\uacfc\uc5d0\uc11c\ub294 \ud559\ubd80\uc81c\ub85c \ubaa8\uc9d1\uc744 \ud558\uba70 \ud559\ubd80\uc81c\ub85c \uc785\ud559\ud55c \ud559\uc0dd\uc758 \uacbd\uc6b0 2\ud559\ub144 \uc9c4\uae09\uc2dc \uc790\uc2e0\uc758 \uc804\uacf5\uc744 \ubc30\uc815\ubc1b\ub294\ub370, \uc804\uacf5\uc744 \ubc30\uc815\ud558\ub294 \uae30\uc900\uc740 \ub2e8\uacfc\ub300\ud559 \uc0ac\uc815\uc5d0 \ub530\ub77c \ub2e4\ub974\uac8c \uc124\uc815\ub3fc \uc788\ub2e4. \ud559\ubd80\uacfc\uc815\uc5d0\uc11c\ub294 2004\ub144 \uc785\ud559\uc0dd\ubd80\ud130 \uc2ec\ud654\uc804\uacf5, \uc774\uc911\uc804\uacf5, \uc735\ud569\uc804\uacf5, \ud559\uc0dd\uc124\uacc4\uc804\uacf5\uc73c\ub85c \uc138\ubd84\ud654\ud560 \uc218 \uc788\ub294 \uc81c2\uc804\uacf5\uc758 \uc774\uc218\uac00 \uc758\ubb34\ud654 \ub3fc \uc788\ub2e4."} {"question": "\ub300\uad6c\uce21\uc774 \uc9c0\uc801\ud55c \ubb38\uc81c\uac00 \uc788\ub294 \uc2e0\uacf5\ud56d \ud6c4\ubcf4\uc9c0\ub294?", "paragraph": "\ub300\uad6c\uce21\uc740 \uac00\ub355\ub3c4 \ud6c4\ubcf4\uc9c0\uc5d0 \uc5ec\ub7ec \ubb38\uc81c\uac00 \uc788\ub2e4\uace0 \uc9c0\uc801\ud55c\ub2e4. \uac00\ub355\ub3c4\ub294 \ubc00\uc591\uc5d0 \ube44\ud574 \uc811\uadfc\uc131\uc774 \ub0ae\ub2e4\ub294 \uac83\uc774\ub2e4. \ubc15\uad11\uae38 \uc2e0\uad6d\uc81c\uacf5\ud56d \ubc00\uc591\uc720\uce58 \ucd94\uc9c4\ub2e8\uc7a5\uc740 \"\uac00\ub355\ub3c4\uac00 \uc81c2\uad00\ubb38\uacf5\ud56d\uc774 \ub420 \uacbd\uc6b0\uc5d0 \ubb38\uc81c\uc810\uc740 \uc601\ub0a8\uc9c0\uc5ed, \ub098\uc544\uac00 \ub0a8\ubd80\uad8c \uc9c0\uc5ed\uc5d0\uc11c \uc811\uadfc\uc131\uc774 \ubb38\uc81c\uac00 \ub9ce\ub2e4\"\uace0 \uc9c0\uc801\ud588\ub2e4. \ud3ec\ud56d\uc774\ub098 \uad6c\ubbf8 \ub4f1\uc5d0\uc11c \uac08 \ub54c \uae38\uc774 \uc5c6\uae30 \ub54c\ubb38\uc5d0 \uc0c8\ub85c \ub193\uc544\uc57c \ud55c\ub2e4\ub294 \uac83\uc774\ub2e4. \uc774\ub7f0 \uc778\ud504\ub77c \uad6c\ucd95\ube44, \uc811\uadfc\ud560 \uc218 \uc788\ub294 \uad6c\ucd95\ube44\uac00 \ucc9c\ubb38\ud559\uc801\uc774\ub77c\uace0 \uc8fc\uc7a5\ud588\ub2e4. \uadf8\ub7ec\ub098 \ubd80\uc0b0\uc740 \uacf5\ud56d \uc774\uc6a9\uac1d\uc774 \ubd80\uc0b0\uc5d0 \uc9d1\uc911\ub418\uc5b4 \uc788\ub2e4\uace0 \ubc18\ubc15\ud55c\ub2e4. \ubd80\uc0b0\uc2dc\uac00 \uc81c\uacf5\ud55c \uc790\ub8cc\uc5d0 \uc758\ud558\uba74 \ubd80\uc0b0 \uc2dc\ubbfc\uc774 72%\uc600\ub2e4. \uc815\ud5cc\uc601 \ubd80\uc0b0\ub300\ud559\uad50 \ub3c4\uc2dc\uacf5\ud559\uacfc \uad50\uc218\ub294 \"\ubaa8\ub4e0 \uc774\uc6a9\uc790\uac00 \uc774\uc6a9\ud588\uc744 \ub54c \ucd5c\uc801\uc758 \ud5e4\ud0dd\uc744 \ubc1b\uc744 \uc218 \uc788\ub294 \uc785\uc9c0\uac00 \ub418\uc5b4\uc57c \ud558\uae30 \ub54c\ubb38\uc5d0 \ubd80\uc0b0\uacfc \uc8fc\uba74\uc5d0 \ub9ce\uc740 \uc778\uad6c\uac00 \uc788\uae30 \ub54c\ubb38\uc5d0 \ubd80\uc0b0 \uac00\ub355\ub3c4\uac00 \ubd88\ub9ac\ud55c \uc785\uc9c0\ub294 \uc544\ub2c8\ub2e4\"\ub77c\uace0 \uc8fc\uc7a5\ud588\ub2e4.", "answer": "\uac00\ub355\ub3c4", "sentence": "\ub300\uad6c\uce21\uc740 \uac00\ub355\ub3c4 \ud6c4\ubcf4\uc9c0\uc5d0 \uc5ec\ub7ec \ubb38\uc81c\uac00 \uc788\ub2e4\uace0 \uc9c0\uc801\ud55c\ub2e4.", "paragraph_sentence": " \ub300\uad6c\uce21\uc740 \uac00\ub355\ub3c4 \ud6c4\ubcf4\uc9c0\uc5d0 \uc5ec\ub7ec \ubb38\uc81c\uac00 \uc788\ub2e4\uace0 \uc9c0\uc801\ud55c\ub2e4. \uac00\ub355\ub3c4\ub294 \ubc00\uc591\uc5d0 \ube44\ud574 \uc811\uadfc\uc131\uc774 \ub0ae\ub2e4\ub294 \uac83\uc774\ub2e4. \ubc15\uad11\uae38 \uc2e0\uad6d\uc81c\uacf5\ud56d \ubc00\uc591\uc720\uce58 \ucd94\uc9c4\ub2e8\uc7a5\uc740 \"\uac00\ub355\ub3c4\uac00 \uc81c2\uad00\ubb38\uacf5\ud56d\uc774 \ub420 \uacbd\uc6b0\uc5d0 \ubb38\uc81c\uc810\uc740 \uc601\ub0a8\uc9c0\uc5ed, \ub098\uc544\uac00 \ub0a8\ubd80\uad8c \uc9c0\uc5ed\uc5d0\uc11c \uc811\uadfc\uc131\uc774 \ubb38\uc81c\uac00 \ub9ce\ub2e4\"\uace0 \uc9c0\uc801\ud588\ub2e4. \ud3ec\ud56d\uc774\ub098 \uad6c\ubbf8 \ub4f1\uc5d0\uc11c \uac08 \ub54c \uae38\uc774 \uc5c6\uae30 \ub54c\ubb38\uc5d0 \uc0c8\ub85c \ub193\uc544\uc57c \ud55c\ub2e4\ub294 \uac83\uc774\ub2e4. \uc774\ub7f0 \uc778\ud504\ub77c \uad6c\ucd95\ube44, \uc811\uadfc\ud560 \uc218 \uc788\ub294 \uad6c\ucd95\ube44\uac00 \ucc9c\ubb38\ud559\uc801\uc774\ub77c\uace0 \uc8fc\uc7a5\ud588\ub2e4. \uadf8\ub7ec\ub098 \ubd80\uc0b0\uc740 \uacf5\ud56d \uc774\uc6a9\uac1d\uc774 \ubd80\uc0b0\uc5d0 \uc9d1\uc911\ub418\uc5b4 \uc788\ub2e4\uace0 \ubc18\ubc15\ud55c\ub2e4. \ubd80\uc0b0\uc2dc\uac00 \uc81c\uacf5\ud55c \uc790\ub8cc\uc5d0 \uc758\ud558\uba74 \ubd80\uc0b0 \uc2dc\ubbfc\uc774 72%\uc600\ub2e4. \uc815\ud5cc\uc601 \ubd80\uc0b0\ub300\ud559\uad50 \ub3c4\uc2dc\uacf5\ud559\uacfc \uad50\uc218\ub294 \"\ubaa8\ub4e0 \uc774\uc6a9\uc790\uac00 \uc774\uc6a9\ud588\uc744 \ub54c \ucd5c\uc801\uc758 \ud5e4\ud0dd\uc744 \ubc1b\uc744 \uc218 \uc788\ub294 \uc785\uc9c0\uac00 \ub418\uc5b4\uc57c \ud558\uae30 \ub54c\ubb38\uc5d0 \ubd80\uc0b0\uacfc \uc8fc\uba74\uc5d0 \ub9ce\uc740 \uc778\uad6c\uac00 \uc788\uae30 \ub54c\ubb38\uc5d0 \ubd80\uc0b0 \uac00\ub355\ub3c4\uac00 \ubd88\ub9ac\ud55c \uc785\uc9c0\ub294 \uc544\ub2c8\ub2e4\"\ub77c\uace0 \uc8fc\uc7a5\ud588\ub2e4.", "paragraph_answer": "\ub300\uad6c\uce21\uc740 \uac00\ub355\ub3c4 \ud6c4\ubcf4\uc9c0\uc5d0 \uc5ec\ub7ec \ubb38\uc81c\uac00 \uc788\ub2e4\uace0 \uc9c0\uc801\ud55c\ub2e4. \uac00\ub355\ub3c4\ub294 \ubc00\uc591\uc5d0 \ube44\ud574 \uc811\uadfc\uc131\uc774 \ub0ae\ub2e4\ub294 \uac83\uc774\ub2e4. \ubc15\uad11\uae38 \uc2e0\uad6d\uc81c\uacf5\ud56d \ubc00\uc591\uc720\uce58 \ucd94\uc9c4\ub2e8\uc7a5\uc740 \"\uac00\ub355\ub3c4\uac00 \uc81c2\uad00\ubb38\uacf5\ud56d\uc774 \ub420 \uacbd\uc6b0\uc5d0 \ubb38\uc81c\uc810\uc740 \uc601\ub0a8\uc9c0\uc5ed, \ub098\uc544\uac00 \ub0a8\ubd80\uad8c \uc9c0\uc5ed\uc5d0\uc11c \uc811\uadfc\uc131\uc774 \ubb38\uc81c\uac00 \ub9ce\ub2e4\"\uace0 \uc9c0\uc801\ud588\ub2e4. \ud3ec\ud56d\uc774\ub098 \uad6c\ubbf8 \ub4f1\uc5d0\uc11c \uac08 \ub54c \uae38\uc774 \uc5c6\uae30 \ub54c\ubb38\uc5d0 \uc0c8\ub85c \ub193\uc544\uc57c \ud55c\ub2e4\ub294 \uac83\uc774\ub2e4. \uc774\ub7f0 \uc778\ud504\ub77c \uad6c\ucd95\ube44, \uc811\uadfc\ud560 \uc218 \uc788\ub294 \uad6c\ucd95\ube44\uac00 \ucc9c\ubb38\ud559\uc801\uc774\ub77c\uace0 \uc8fc\uc7a5\ud588\ub2e4. \uadf8\ub7ec\ub098 \ubd80\uc0b0\uc740 \uacf5\ud56d \uc774\uc6a9\uac1d\uc774 \ubd80\uc0b0\uc5d0 \uc9d1\uc911\ub418\uc5b4 \uc788\ub2e4\uace0 \ubc18\ubc15\ud55c\ub2e4. \ubd80\uc0b0\uc2dc\uac00 \uc81c\uacf5\ud55c \uc790\ub8cc\uc5d0 \uc758\ud558\uba74 \ubd80\uc0b0 \uc2dc\ubbfc\uc774 72%\uc600\ub2e4. \uc815\ud5cc\uc601 \ubd80\uc0b0\ub300\ud559\uad50 \ub3c4\uc2dc\uacf5\ud559\uacfc \uad50\uc218\ub294 \"\ubaa8\ub4e0 \uc774\uc6a9\uc790\uac00 \uc774\uc6a9\ud588\uc744 \ub54c \ucd5c\uc801\uc758 \ud5e4\ud0dd\uc744 \ubc1b\uc744 \uc218 \uc788\ub294 \uc785\uc9c0\uac00 \ub418\uc5b4\uc57c \ud558\uae30 \ub54c\ubb38\uc5d0 \ubd80\uc0b0\uacfc \uc8fc\uba74\uc5d0 \ub9ce\uc740 \uc778\uad6c\uac00 \uc788\uae30 \ub54c\ubb38\uc5d0 \ubd80\uc0b0 \uac00\ub355\ub3c4\uac00 \ubd88\ub9ac\ud55c \uc785\uc9c0\ub294 \uc544\ub2c8\ub2e4\"\ub77c\uace0 \uc8fc\uc7a5\ud588\ub2e4.", "sentence_answer": "\ub300\uad6c\uce21\uc740 \uac00\ub355\ub3c4 \ud6c4\ubcf4\uc9c0\uc5d0 \uc5ec\ub7ec \ubb38\uc81c\uac00 \uc788\ub2e4\uace0 \uc9c0\uc801\ud55c\ub2e4.", "paragraph_id": "6524243-0-0", "paragraph_question": "question: \ub300\uad6c\uce21\uc774 \uc9c0\uc801\ud55c \ubb38\uc81c\uac00 \uc788\ub294 \uc2e0\uacf5\ud56d \ud6c4\ubcf4\uc9c0\ub294?, context: \ub300\uad6c\uce21\uc740 \uac00\ub355\ub3c4 \ud6c4\ubcf4\uc9c0\uc5d0 \uc5ec\ub7ec \ubb38\uc81c\uac00 \uc788\ub2e4\uace0 \uc9c0\uc801\ud55c\ub2e4. \uac00\ub355\ub3c4\ub294 \ubc00\uc591\uc5d0 \ube44\ud574 \uc811\uadfc\uc131\uc774 \ub0ae\ub2e4\ub294 \uac83\uc774\ub2e4. \ubc15\uad11\uae38 \uc2e0\uad6d\uc81c\uacf5\ud56d \ubc00\uc591\uc720\uce58 \ucd94\uc9c4\ub2e8\uc7a5\uc740 \"\uac00\ub355\ub3c4\uac00 \uc81c2\uad00\ubb38\uacf5\ud56d\uc774 \ub420 \uacbd\uc6b0\uc5d0 \ubb38\uc81c\uc810\uc740 \uc601\ub0a8\uc9c0\uc5ed, \ub098\uc544\uac00 \ub0a8\ubd80\uad8c \uc9c0\uc5ed\uc5d0\uc11c \uc811\uadfc\uc131\uc774 \ubb38\uc81c\uac00 \ub9ce\ub2e4\"\uace0 \uc9c0\uc801\ud588\ub2e4. \ud3ec\ud56d\uc774\ub098 \uad6c\ubbf8 \ub4f1\uc5d0\uc11c \uac08 \ub54c \uae38\uc774 \uc5c6\uae30 \ub54c\ubb38\uc5d0 \uc0c8\ub85c \ub193\uc544\uc57c \ud55c\ub2e4\ub294 \uac83\uc774\ub2e4. \uc774\ub7f0 \uc778\ud504\ub77c \uad6c\ucd95\ube44, \uc811\uadfc\ud560 \uc218 \uc788\ub294 \uad6c\ucd95\ube44\uac00 \ucc9c\ubb38\ud559\uc801\uc774\ub77c\uace0 \uc8fc\uc7a5\ud588\ub2e4. \uadf8\ub7ec\ub098 \ubd80\uc0b0\uc740 \uacf5\ud56d \uc774\uc6a9\uac1d\uc774 \ubd80\uc0b0\uc5d0 \uc9d1\uc911\ub418\uc5b4 \uc788\ub2e4\uace0 \ubc18\ubc15\ud55c\ub2e4. \ubd80\uc0b0\uc2dc\uac00 \uc81c\uacf5\ud55c \uc790\ub8cc\uc5d0 \uc758\ud558\uba74 \ubd80\uc0b0 \uc2dc\ubbfc\uc774 72%\uc600\ub2e4. \uc815\ud5cc\uc601 \ubd80\uc0b0\ub300\ud559\uad50 \ub3c4\uc2dc\uacf5\ud559\uacfc \uad50\uc218\ub294 \"\ubaa8\ub4e0 \uc774\uc6a9\uc790\uac00 \uc774\uc6a9\ud588\uc744 \ub54c \ucd5c\uc801\uc758 \ud5e4\ud0dd\uc744 \ubc1b\uc744 \uc218 \uc788\ub294 \uc785\uc9c0\uac00 \ub418\uc5b4\uc57c \ud558\uae30 \ub54c\ubb38\uc5d0 \ubd80\uc0b0\uacfc \uc8fc\uba74\uc5d0 \ub9ce\uc740 \uc778\uad6c\uac00 \uc788\uae30 \ub54c\ubb38\uc5d0 \ubd80\uc0b0 \uac00\ub355\ub3c4\uac00 \ubd88\ub9ac\ud55c \uc785\uc9c0\ub294 \uc544\ub2c8\ub2e4\"\ub77c\uace0 \uc8fc\uc7a5\ud588\ub2e4."} {"question": "\ud398\ub354\ub7ec\uac00 \uc6dc\ube14\ub358 \uacb0\uc2b9\uc5d0\uc11c \uc774\uae34 \uc120\uc218\uc758 \uc774\ub984\uc740?", "paragraph": "\uc714\ube14\ub358 \uacb0\uc2b9\uc5d0\uc11c\ub294 \uc804\ub144\ub3c4 \uacb0\uc2b9\uc5d0\uc11c\ub3c4 \ub9cc\ub0ac\ub358 \uc564\ub514 \ub85c\ub515\uc744 \uc0c1\ub300\ub85c 3\uc138\ud2b8\ub97c \ub0b4\ub9ac \ub530\ub0b4\uba70 \uc2b9\ub9ac, 3\ub144 \uc5f0\uc18d \uc6b0\uc2b9\uc744 \uae30\ub85d\ud588\ub2e4. \ud398\ub354\ub7ec\ub294 \uc774\ud6c4 \uc2e0\uc2dc\ub0b4\ud2f0 \ub9c8\uc2a4\ud130\uc2a4 \uacb0\uc2b9\uc5d0\uc11c\ub3c4 \ub85c\ub515\uc744 \ub9cc\ub098 \ub610 \ud55c \ubc88 \uc2b9\ub9ac\ud558\uba74\uc11c \uc774 \ud574 4\ubc88\uc9f8 \ub9c8\uc2a4\ud130\uc2a4 \uc2dc\ub9ac\uc988 \uc6b0\uc2b9\uc744 \uae30\ub85d\ud588\ub2e4. \uc774\ub85c\uc368 \ud398\ub354\ub7ec\ub294 2005\ub144 \ubd81\ubbf8 \uc9c0\uc5ed\uc5d0\uc11c \uc5f4\ub9b0 \ub9c8\uc2a4\ud130\uc2a4 \uc2dc\ub9ac\uc988 \ub300\ud68c\uc5d0\uc11c \ubaa8\ub450 \uc6b0\uc2b9\ud588\uc73c\uba70, \ud55c \ud574\uc5d0 4\uac1c \ub9c8\uc2a4\ud130\uc2a4 \uc2dc\ub9ac\uc988 \ub300\ud68c\uc5d0\uc11c \uc6b0\uc2b9\ud55c \ucd5c\ucd08\uc758 \uc120\uc218\uac00 \ub418\uc5c8\ub2e4. US \uc624\ud508 \uacb0\uc2b9\uc5d0\uc11c\ub294 \uc548\ub4dc\ub808 \uc560\uac70\uc2dc\ub97c 4\uc138\ud2b8\ub9cc\uc5d0 \uaebe\uc73c\uba74\uc11c \uc6b0\uc2b9\ud588\ub294\ub370, \ud2b9\ud788 1\ud68c\uc804\uc5d0\uc11c \uc900\uacb0\uc2b9\uae4c\uc9c0\uc758 \ubaa8\ub4e0 \uacbd\uae30\uc5d0\uc11c \ub2e8 \ub450 \uc138\ud2b8\ub9cc\uc744 \uc783\uc5c8\uc744 \uc815\ub3c4\ub85c \ud0c1\uc6d4\ud55c \uacbd\uae30\ub825\uc744 \ubcf4\uc5ec\uc8fc\uc5c8\ub2e4. \uc774\ub85c\uc368 \ud398\ub354\ub7ec\ub294 \ud55c \ud574\uc5d0 \uc714\ube14\ub358\uacfc US \uc624\ud508\uc5d0\uc11c \ubaa8\ub450 \uc6b0\uc2b9\ud558\ub294 \uac83\uc744 2\ub144 \uc5f0\uc18d\uc73c\ub85c \ub2ec\uc131\ud55c \uc624\ud508 \uc2dc\ub300 \uc774\ub798 \ucd5c\ucd08\uc758 \ub0a8\uc790 \uc120\uc218\uac00 \ub418\uc5c8\ub2e4.", "answer": "\uc564\ub514 \ub85c\ub515", "sentence": "\uc714\ube14\ub358 \uacb0\uc2b9\uc5d0\uc11c\ub294 \uc804\ub144\ub3c4 \uacb0\uc2b9\uc5d0\uc11c\ub3c4 \ub9cc\ub0ac\ub358 \uc564\ub514 \ub85c\ub515 \uc744 \uc0c1\ub300\ub85c 3\uc138\ud2b8\ub97c \ub0b4\ub9ac \ub530\ub0b4\uba70 \uc2b9\ub9ac, 3\ub144 \uc5f0\uc18d \uc6b0\uc2b9\uc744 \uae30\ub85d\ud588\ub2e4.", "paragraph_sentence": " \uc714\ube14\ub358 \uacb0\uc2b9\uc5d0\uc11c\ub294 \uc804\ub144\ub3c4 \uacb0\uc2b9\uc5d0\uc11c\ub3c4 \ub9cc\ub0ac\ub358 \uc564\ub514 \ub85c\ub515 \uc744 \uc0c1\ub300\ub85c 3\uc138\ud2b8\ub97c \ub0b4\ub9ac \ub530\ub0b4\uba70 \uc2b9\ub9ac, 3\ub144 \uc5f0\uc18d \uc6b0\uc2b9\uc744 \uae30\ub85d\ud588\ub2e4. \ud398\ub354\ub7ec\ub294 \uc774\ud6c4 \uc2e0\uc2dc\ub0b4\ud2f0 \ub9c8\uc2a4\ud130\uc2a4 \uacb0\uc2b9\uc5d0\uc11c\ub3c4 \ub85c\ub515\uc744 \ub9cc\ub098 \ub610 \ud55c \ubc88 \uc2b9\ub9ac\ud558\uba74\uc11c \uc774 \ud574 4\ubc88\uc9f8 \ub9c8\uc2a4\ud130\uc2a4 \uc2dc\ub9ac\uc988 \uc6b0\uc2b9\uc744 \uae30\ub85d\ud588\ub2e4. \uc774\ub85c\uc368 \ud398\ub354\ub7ec\ub294 2005\ub144 \ubd81\ubbf8 \uc9c0\uc5ed\uc5d0\uc11c \uc5f4\ub9b0 \ub9c8\uc2a4\ud130\uc2a4 \uc2dc\ub9ac\uc988 \ub300\ud68c\uc5d0\uc11c \ubaa8\ub450 \uc6b0\uc2b9\ud588\uc73c\uba70, \ud55c \ud574\uc5d0 4\uac1c \ub9c8\uc2a4\ud130\uc2a4 \uc2dc\ub9ac\uc988 \ub300\ud68c\uc5d0\uc11c \uc6b0\uc2b9\ud55c \ucd5c\ucd08\uc758 \uc120\uc218\uac00 \ub418\uc5c8\ub2e4. US \uc624\ud508 \uacb0\uc2b9\uc5d0\uc11c\ub294 \uc548\ub4dc\ub808 \uc560\uac70\uc2dc\ub97c 4\uc138\ud2b8\ub9cc\uc5d0 \uaebe\uc73c\uba74\uc11c \uc6b0\uc2b9\ud588\ub294\ub370, \ud2b9\ud788 1\ud68c\uc804\uc5d0\uc11c \uc900\uacb0\uc2b9\uae4c\uc9c0\uc758 \ubaa8\ub4e0 \uacbd\uae30\uc5d0\uc11c \ub2e8 \ub450 \uc138\ud2b8\ub9cc\uc744 \uc783\uc5c8\uc744 \uc815\ub3c4\ub85c \ud0c1\uc6d4\ud55c \uacbd\uae30\ub825\uc744 \ubcf4\uc5ec\uc8fc\uc5c8\ub2e4. \uc774\ub85c\uc368 \ud398\ub354\ub7ec\ub294 \ud55c \ud574\uc5d0 \uc714\ube14\ub358\uacfc US \uc624\ud508\uc5d0\uc11c \ubaa8\ub450 \uc6b0\uc2b9\ud558\ub294 \uac83\uc744 2\ub144 \uc5f0\uc18d\uc73c\ub85c \ub2ec\uc131\ud55c \uc624\ud508 \uc2dc\ub300 \uc774\ub798 \ucd5c\ucd08\uc758 \ub0a8\uc790 \uc120\uc218\uac00 \ub418\uc5c8\ub2e4.", "paragraph_answer": "\uc714\ube14\ub358 \uacb0\uc2b9\uc5d0\uc11c\ub294 \uc804\ub144\ub3c4 \uacb0\uc2b9\uc5d0\uc11c\ub3c4 \ub9cc\ub0ac\ub358 \uc564\ub514 \ub85c\ub515 \uc744 \uc0c1\ub300\ub85c 3\uc138\ud2b8\ub97c \ub0b4\ub9ac \ub530\ub0b4\uba70 \uc2b9\ub9ac, 3\ub144 \uc5f0\uc18d \uc6b0\uc2b9\uc744 \uae30\ub85d\ud588\ub2e4. \ud398\ub354\ub7ec\ub294 \uc774\ud6c4 \uc2e0\uc2dc\ub0b4\ud2f0 \ub9c8\uc2a4\ud130\uc2a4 \uacb0\uc2b9\uc5d0\uc11c\ub3c4 \ub85c\ub515\uc744 \ub9cc\ub098 \ub610 \ud55c \ubc88 \uc2b9\ub9ac\ud558\uba74\uc11c \uc774 \ud574 4\ubc88\uc9f8 \ub9c8\uc2a4\ud130\uc2a4 \uc2dc\ub9ac\uc988 \uc6b0\uc2b9\uc744 \uae30\ub85d\ud588\ub2e4. \uc774\ub85c\uc368 \ud398\ub354\ub7ec\ub294 2005\ub144 \ubd81\ubbf8 \uc9c0\uc5ed\uc5d0\uc11c \uc5f4\ub9b0 \ub9c8\uc2a4\ud130\uc2a4 \uc2dc\ub9ac\uc988 \ub300\ud68c\uc5d0\uc11c \ubaa8\ub450 \uc6b0\uc2b9\ud588\uc73c\uba70, \ud55c \ud574\uc5d0 4\uac1c \ub9c8\uc2a4\ud130\uc2a4 \uc2dc\ub9ac\uc988 \ub300\ud68c\uc5d0\uc11c \uc6b0\uc2b9\ud55c \ucd5c\ucd08\uc758 \uc120\uc218\uac00 \ub418\uc5c8\ub2e4. US \uc624\ud508 \uacb0\uc2b9\uc5d0\uc11c\ub294 \uc548\ub4dc\ub808 \uc560\uac70\uc2dc\ub97c 4\uc138\ud2b8\ub9cc\uc5d0 \uaebe\uc73c\uba74\uc11c \uc6b0\uc2b9\ud588\ub294\ub370, \ud2b9\ud788 1\ud68c\uc804\uc5d0\uc11c \uc900\uacb0\uc2b9\uae4c\uc9c0\uc758 \ubaa8\ub4e0 \uacbd\uae30\uc5d0\uc11c \ub2e8 \ub450 \uc138\ud2b8\ub9cc\uc744 \uc783\uc5c8\uc744 \uc815\ub3c4\ub85c \ud0c1\uc6d4\ud55c \uacbd\uae30\ub825\uc744 \ubcf4\uc5ec\uc8fc\uc5c8\ub2e4. \uc774\ub85c\uc368 \ud398\ub354\ub7ec\ub294 \ud55c \ud574\uc5d0 \uc714\ube14\ub358\uacfc US \uc624\ud508\uc5d0\uc11c \ubaa8\ub450 \uc6b0\uc2b9\ud558\ub294 \uac83\uc744 2\ub144 \uc5f0\uc18d\uc73c\ub85c \ub2ec\uc131\ud55c \uc624\ud508 \uc2dc\ub300 \uc774\ub798 \ucd5c\ucd08\uc758 \ub0a8\uc790 \uc120\uc218\uac00 \ub418\uc5c8\ub2e4.", "sentence_answer": "\uc714\ube14\ub358 \uacb0\uc2b9\uc5d0\uc11c\ub294 \uc804\ub144\ub3c4 \uacb0\uc2b9\uc5d0\uc11c\ub3c4 \ub9cc\ub0ac\ub358 \uc564\ub514 \ub85c\ub515 \uc744 \uc0c1\ub300\ub85c 3\uc138\ud2b8\ub97c \ub0b4\ub9ac \ub530\ub0b4\uba70 \uc2b9\ub9ac, 3\ub144 \uc5f0\uc18d \uc6b0\uc2b9\uc744 \uae30\ub85d\ud588\ub2e4.", "paragraph_id": "6056919-7-0", "paragraph_question": "question: \ud398\ub354\ub7ec\uac00 \uc6dc\ube14\ub358 \uacb0\uc2b9\uc5d0\uc11c \uc774\uae34 \uc120\uc218\uc758 \uc774\ub984\uc740?, context: \uc714\ube14\ub358 \uacb0\uc2b9\uc5d0\uc11c\ub294 \uc804\ub144\ub3c4 \uacb0\uc2b9\uc5d0\uc11c\ub3c4 \ub9cc\ub0ac\ub358 \uc564\ub514 \ub85c\ub515\uc744 \uc0c1\ub300\ub85c 3\uc138\ud2b8\ub97c \ub0b4\ub9ac \ub530\ub0b4\uba70 \uc2b9\ub9ac, 3\ub144 \uc5f0\uc18d \uc6b0\uc2b9\uc744 \uae30\ub85d\ud588\ub2e4. \ud398\ub354\ub7ec\ub294 \uc774\ud6c4 \uc2e0\uc2dc\ub0b4\ud2f0 \ub9c8\uc2a4\ud130\uc2a4 \uacb0\uc2b9\uc5d0\uc11c\ub3c4 \ub85c\ub515\uc744 \ub9cc\ub098 \ub610 \ud55c \ubc88 \uc2b9\ub9ac\ud558\uba74\uc11c \uc774 \ud574 4\ubc88\uc9f8 \ub9c8\uc2a4\ud130\uc2a4 \uc2dc\ub9ac\uc988 \uc6b0\uc2b9\uc744 \uae30\ub85d\ud588\ub2e4. \uc774\ub85c\uc368 \ud398\ub354\ub7ec\ub294 2005\ub144 \ubd81\ubbf8 \uc9c0\uc5ed\uc5d0\uc11c \uc5f4\ub9b0 \ub9c8\uc2a4\ud130\uc2a4 \uc2dc\ub9ac\uc988 \ub300\ud68c\uc5d0\uc11c \ubaa8\ub450 \uc6b0\uc2b9\ud588\uc73c\uba70, \ud55c \ud574\uc5d0 4\uac1c \ub9c8\uc2a4\ud130\uc2a4 \uc2dc\ub9ac\uc988 \ub300\ud68c\uc5d0\uc11c \uc6b0\uc2b9\ud55c \ucd5c\ucd08\uc758 \uc120\uc218\uac00 \ub418\uc5c8\ub2e4. US \uc624\ud508 \uacb0\uc2b9\uc5d0\uc11c\ub294 \uc548\ub4dc\ub808 \uc560\uac70\uc2dc\ub97c 4\uc138\ud2b8\ub9cc\uc5d0 \uaebe\uc73c\uba74\uc11c \uc6b0\uc2b9\ud588\ub294\ub370, \ud2b9\ud788 1\ud68c\uc804\uc5d0\uc11c \uc900\uacb0\uc2b9\uae4c\uc9c0\uc758 \ubaa8\ub4e0 \uacbd\uae30\uc5d0\uc11c \ub2e8 \ub450 \uc138\ud2b8\ub9cc\uc744 \uc783\uc5c8\uc744 \uc815\ub3c4\ub85c \ud0c1\uc6d4\ud55c \uacbd\uae30\ub825\uc744 \ubcf4\uc5ec\uc8fc\uc5c8\ub2e4. \uc774\ub85c\uc368 \ud398\ub354\ub7ec\ub294 \ud55c \ud574\uc5d0 \uc714\ube14\ub358\uacfc US \uc624\ud508\uc5d0\uc11c \ubaa8\ub450 \uc6b0\uc2b9\ud558\ub294 \uac83\uc744 2\ub144 \uc5f0\uc18d\uc73c\ub85c \ub2ec\uc131\ud55c \uc624\ud508 \uc2dc\ub300 \uc774\ub798 \ucd5c\ucd08\uc758 \ub0a8\uc790 \uc120\uc218\uac00 \ub418\uc5c8\ub2e4."} {"question": "\ub514\uc624\ud074\ub808\ud2f0\uc544\ub204\uc2a4\uac00 293\ub144 \uc81c\uad6d\uc744 \ub3d9\ubd80\uc640 \uc11c\ubd80\ub85c \ub098\ub204\uc5b4 \uac01\uac01 \uc9c0\ubc30\ud558\ub3c4\ub85d \ub3c4\uc785\ud55c \uccb4\uc81c\ub294?", "paragraph": "193\ub144\uc5d0\uc11c 235\ub144 \uc0ac\uc774\ub294 \uc138\ubca0\ub8e8\uc2a4 \ud669\uc870\uc758 \uce58\uc81c(\ud754\ud788 \ub9d0\ud558\ub294 \uad70\uc778\ud669\uc81c\uc2dc\ub300)\ub85c \ud5ec\ub9ac\uc624\uac00\ubc1c\ub8e8\uc2a4\uac19\uc740 \ubb34\ub2a5\ud55c \uc9c0\ubc30\uc790\ub3c4 \ub098\ud0c0\ub0ac\ub2e4. \uac8c\ub2e4\uac00 \uad70\ub300\uac00 \uc81c\uc704 \uacc4\uc2b9\uc5d0 \uc601\ud5a5\ub825\uc744 \ub354\uc6b1 \ud589\uc0ac\ud558\uac8c \ub418\uba74\uc11c \uc624\ub79c \uae30\uac04 \u20183\uc138\uae30\uc758 \uc704\uae30\u2019\ub77c\ub294 \ud63c\ub780\uae30(49\ub144\uac04 20\uc5ec\uba85\uc758 \ud669\uc81c\uac00 \uad50\uccb4\ub428)\ub85c \uc811\uc5b4\ub4e4\uc5c8\ub2e4. \uc774 \uc704\uae30\ub294 \uc720\ub2a5\ud588\ub358 \ud669\uc81c \ub514\uc624\ud074\ub808\ud2f0\uc544\ub204\uc2a4\uac00 \uc77c\ub2e8\ub77d\uc9c0\uc5c8\uc73c\uba70, 293\ub144\uc5d0 \uc81c\uad6d\uc744 \ub3d9\ubd80\uc640 \uc11c\ubd80\ub85c \ub098\ub204\uc5b4 \uac01\uac01 \ub450 \uacf5\ub3d9 \ud669\uc81c\uc640 \ubd80\uc81c(\u526f\u5e1d)\uac00 \uc9c0\ubc30\ud558\ub294 \uc0ac\ub450 \uccb4\uc81c\ub97c \ub3c4\uc785\ud588\ub2e4. \uc5ec\ub7ec \uacf5\ub3d9 \ud669\uc81c\ub294 50\ub144 \uc774\uc0c1 \uc11c\ub85c \ub300\ub9bd\ud558\uba70 \uad8c\ub825 \ud22c\uc7c1\uc744 \ubc8c\uc600\ub2e4. 330\ub144 5\uc6d4 11\uc77c \ucf58\uc2a4\ud0c4\ud2f0\ub204\uc2a4 \ub300\uc81c\ub294 \ube44\uc794\ud2f0\uc628\uc744 \ucf58\uc2a4\ud0c4\ud2f0\ub178\ud3f4\ub9ac\uc2a4\ub85c \uba85\uba85(\ud604 \uc774\uc2a4\ud0c4\ubd88)\ud558\uace0 \ub85c\ub9c8 \uc81c\uad6d\uc758 \uc0c8 \uc218\ub3c4\ub85c \uc0bc\uc558\ub2e4. \uc81c\uad6d\uc740 \ud14c\uc624\ub3c4\uc2dc\uc6b0\uc2a4 1\uc138 \uc0ac\ud6c4\uc778 395\ub144\uc5d0 \ube44\uc794\ud2f0\uc6c0 \uc81c\uad6d\uacfc \uc11c\ub85c\ub9c8 \uc81c\uad6d\uc73c\ub85c \uc601\uad6c\ud788 \uac08\ub77c\uc84c\ub2e4.", "answer": "\uc0ac\ub450 \uccb4\uc81c", "sentence": "\uc774 \uc704\uae30\ub294 \uc720\ub2a5\ud588\ub358 \ud669\uc81c \ub514\uc624\ud074\ub808\ud2f0\uc544\ub204\uc2a4\uac00 \uc77c\ub2e8\ub77d\uc9c0\uc5c8\uc73c\uba70, 293\ub144\uc5d0 \uc81c\uad6d\uc744 \ub3d9\ubd80\uc640 \uc11c\ubd80\ub85c \ub098\ub204\uc5b4 \uac01\uac01 \ub450 \uacf5\ub3d9 \ud669\uc81c\uc640 \ubd80\uc81c(\u526f\u5e1d)\uac00 \uc9c0\ubc30\ud558\ub294 \uc0ac\ub450 \uccb4\uc81c \ub97c \ub3c4\uc785\ud588\ub2e4.", "paragraph_sentence": "193\ub144\uc5d0\uc11c 235\ub144 \uc0ac\uc774\ub294 \uc138\ubca0\ub8e8\uc2a4 \ud669\uc870\uc758 \uce58\uc81c(\ud754\ud788 \ub9d0\ud558\ub294 \uad70\uc778\ud669\uc81c\uc2dc\ub300)\ub85c \ud5ec\ub9ac\uc624\uac00\ubc1c\ub8e8\uc2a4\uac19\uc740 \ubb34\ub2a5\ud55c \uc9c0\ubc30\uc790\ub3c4 \ub098\ud0c0\ub0ac\ub2e4. \uac8c\ub2e4\uac00 \uad70\ub300\uac00 \uc81c\uc704 \uacc4\uc2b9\uc5d0 \uc601\ud5a5\ub825\uc744 \ub354\uc6b1 \ud589\uc0ac\ud558\uac8c \ub418\uba74\uc11c \uc624\ub79c \uae30\uac04 \u20183\uc138\uae30\uc758 \uc704\uae30\u2019\ub77c\ub294 \ud63c\ub780\uae30(49\ub144\uac04 20\uc5ec\uba85\uc758 \ud669\uc81c\uac00 \uad50\uccb4\ub428)\ub85c \uc811\uc5b4\ub4e4\uc5c8\ub2e4. \uc774 \uc704\uae30\ub294 \uc720\ub2a5\ud588\ub358 \ud669\uc81c \ub514\uc624\ud074\ub808\ud2f0\uc544\ub204\uc2a4\uac00 \uc77c\ub2e8\ub77d\uc9c0\uc5c8\uc73c\uba70, 293\ub144\uc5d0 \uc81c\uad6d\uc744 \ub3d9\ubd80\uc640 \uc11c\ubd80\ub85c \ub098\ub204\uc5b4 \uac01\uac01 \ub450 \uacf5\ub3d9 \ud669\uc81c\uc640 \ubd80\uc81c(\u526f\u5e1d)\uac00 \uc9c0\ubc30\ud558\ub294 \uc0ac\ub450 \uccb4\uc81c \ub97c \ub3c4\uc785\ud588\ub2e4. \uc5ec\ub7ec \uacf5\ub3d9 \ud669\uc81c\ub294 50\ub144 \uc774\uc0c1 \uc11c\ub85c \ub300\ub9bd\ud558\uba70 \uad8c\ub825 \ud22c\uc7c1\uc744 \ubc8c\uc600\ub2e4. 330\ub144 5\uc6d4 11\uc77c \ucf58\uc2a4\ud0c4\ud2f0\ub204\uc2a4 \ub300\uc81c\ub294 \ube44\uc794\ud2f0\uc628\uc744 \ucf58\uc2a4\ud0c4\ud2f0\ub178\ud3f4\ub9ac\uc2a4\ub85c \uba85\uba85(\ud604 \uc774\uc2a4\ud0c4\ubd88)\ud558\uace0 \ub85c\ub9c8 \uc81c\uad6d\uc758 \uc0c8 \uc218\ub3c4\ub85c \uc0bc\uc558\ub2e4. \uc81c\uad6d\uc740 \ud14c\uc624\ub3c4\uc2dc\uc6b0\uc2a4 1\uc138 \uc0ac\ud6c4\uc778 395\ub144\uc5d0 \ube44\uc794\ud2f0\uc6c0 \uc81c\uad6d\uacfc \uc11c\ub85c\ub9c8 \uc81c\uad6d\uc73c\ub85c \uc601\uad6c\ud788 \uac08\ub77c\uc84c\ub2e4.", "paragraph_answer": "193\ub144\uc5d0\uc11c 235\ub144 \uc0ac\uc774\ub294 \uc138\ubca0\ub8e8\uc2a4 \ud669\uc870\uc758 \uce58\uc81c(\ud754\ud788 \ub9d0\ud558\ub294 \uad70\uc778\ud669\uc81c\uc2dc\ub300)\ub85c \ud5ec\ub9ac\uc624\uac00\ubc1c\ub8e8\uc2a4\uac19\uc740 \ubb34\ub2a5\ud55c \uc9c0\ubc30\uc790\ub3c4 \ub098\ud0c0\ub0ac\ub2e4. \uac8c\ub2e4\uac00 \uad70\ub300\uac00 \uc81c\uc704 \uacc4\uc2b9\uc5d0 \uc601\ud5a5\ub825\uc744 \ub354\uc6b1 \ud589\uc0ac\ud558\uac8c \ub418\uba74\uc11c \uc624\ub79c \uae30\uac04 \u20183\uc138\uae30\uc758 \uc704\uae30\u2019\ub77c\ub294 \ud63c\ub780\uae30(49\ub144\uac04 20\uc5ec\uba85\uc758 \ud669\uc81c\uac00 \uad50\uccb4\ub428)\ub85c \uc811\uc5b4\ub4e4\uc5c8\ub2e4. \uc774 \uc704\uae30\ub294 \uc720\ub2a5\ud588\ub358 \ud669\uc81c \ub514\uc624\ud074\ub808\ud2f0\uc544\ub204\uc2a4\uac00 \uc77c\ub2e8\ub77d\uc9c0\uc5c8\uc73c\uba70, 293\ub144\uc5d0 \uc81c\uad6d\uc744 \ub3d9\ubd80\uc640 \uc11c\ubd80\ub85c \ub098\ub204\uc5b4 \uac01\uac01 \ub450 \uacf5\ub3d9 \ud669\uc81c\uc640 \ubd80\uc81c(\u526f\u5e1d)\uac00 \uc9c0\ubc30\ud558\ub294 \uc0ac\ub450 \uccb4\uc81c \ub97c \ub3c4\uc785\ud588\ub2e4. \uc5ec\ub7ec \uacf5\ub3d9 \ud669\uc81c\ub294 50\ub144 \uc774\uc0c1 \uc11c\ub85c \ub300\ub9bd\ud558\uba70 \uad8c\ub825 \ud22c\uc7c1\uc744 \ubc8c\uc600\ub2e4. 330\ub144 5\uc6d4 11\uc77c \ucf58\uc2a4\ud0c4\ud2f0\ub204\uc2a4 \ub300\uc81c\ub294 \ube44\uc794\ud2f0\uc628\uc744 \ucf58\uc2a4\ud0c4\ud2f0\ub178\ud3f4\ub9ac\uc2a4\ub85c \uba85\uba85(\ud604 \uc774\uc2a4\ud0c4\ubd88)\ud558\uace0 \ub85c\ub9c8 \uc81c\uad6d\uc758 \uc0c8 \uc218\ub3c4\ub85c \uc0bc\uc558\ub2e4. \uc81c\uad6d\uc740 \ud14c\uc624\ub3c4\uc2dc\uc6b0\uc2a4 1\uc138 \uc0ac\ud6c4\uc778 395\ub144\uc5d0 \ube44\uc794\ud2f0\uc6c0 \uc81c\uad6d\uacfc \uc11c\ub85c\ub9c8 \uc81c\uad6d\uc73c\ub85c \uc601\uad6c\ud788 \uac08\ub77c\uc84c\ub2e4.", "sentence_answer": "\uc774 \uc704\uae30\ub294 \uc720\ub2a5\ud588\ub358 \ud669\uc81c \ub514\uc624\ud074\ub808\ud2f0\uc544\ub204\uc2a4\uac00 \uc77c\ub2e8\ub77d\uc9c0\uc5c8\uc73c\uba70, 293\ub144\uc5d0 \uc81c\uad6d\uc744 \ub3d9\ubd80\uc640 \uc11c\ubd80\ub85c \ub098\ub204\uc5b4 \uac01\uac01 \ub450 \uacf5\ub3d9 \ud669\uc81c\uc640 \ubd80\uc81c(\u526f\u5e1d)\uac00 \uc9c0\ubc30\ud558\ub294 \uc0ac\ub450 \uccb4\uc81c \ub97c \ub3c4\uc785\ud588\ub2e4.", "paragraph_id": "6527248-6-2", "paragraph_question": "question: \ub514\uc624\ud074\ub808\ud2f0\uc544\ub204\uc2a4\uac00 293\ub144 \uc81c\uad6d\uc744 \ub3d9\ubd80\uc640 \uc11c\ubd80\ub85c \ub098\ub204\uc5b4 \uac01\uac01 \uc9c0\ubc30\ud558\ub3c4\ub85d \ub3c4\uc785\ud55c \uccb4\uc81c\ub294?, context: 193\ub144\uc5d0\uc11c 235\ub144 \uc0ac\uc774\ub294 \uc138\ubca0\ub8e8\uc2a4 \ud669\uc870\uc758 \uce58\uc81c(\ud754\ud788 \ub9d0\ud558\ub294 \uad70\uc778\ud669\uc81c\uc2dc\ub300)\ub85c \ud5ec\ub9ac\uc624\uac00\ubc1c\ub8e8\uc2a4\uac19\uc740 \ubb34\ub2a5\ud55c \uc9c0\ubc30\uc790\ub3c4 \ub098\ud0c0\ub0ac\ub2e4. \uac8c\ub2e4\uac00 \uad70\ub300\uac00 \uc81c\uc704 \uacc4\uc2b9\uc5d0 \uc601\ud5a5\ub825\uc744 \ub354\uc6b1 \ud589\uc0ac\ud558\uac8c \ub418\uba74\uc11c \uc624\ub79c \uae30\uac04 \u20183\uc138\uae30\uc758 \uc704\uae30\u2019\ub77c\ub294 \ud63c\ub780\uae30(49\ub144\uac04 20\uc5ec\uba85\uc758 \ud669\uc81c\uac00 \uad50\uccb4\ub428)\ub85c \uc811\uc5b4\ub4e4\uc5c8\ub2e4. \uc774 \uc704\uae30\ub294 \uc720\ub2a5\ud588\ub358 \ud669\uc81c \ub514\uc624\ud074\ub808\ud2f0\uc544\ub204\uc2a4\uac00 \uc77c\ub2e8\ub77d\uc9c0\uc5c8\uc73c\uba70, 293\ub144\uc5d0 \uc81c\uad6d\uc744 \ub3d9\ubd80\uc640 \uc11c\ubd80\ub85c \ub098\ub204\uc5b4 \uac01\uac01 \ub450 \uacf5\ub3d9 \ud669\uc81c\uc640 \ubd80\uc81c(\u526f\u5e1d)\uac00 \uc9c0\ubc30\ud558\ub294 \uc0ac\ub450 \uccb4\uc81c\ub97c \ub3c4\uc785\ud588\ub2e4. \uc5ec\ub7ec \uacf5\ub3d9 \ud669\uc81c\ub294 50\ub144 \uc774\uc0c1 \uc11c\ub85c \ub300\ub9bd\ud558\uba70 \uad8c\ub825 \ud22c\uc7c1\uc744 \ubc8c\uc600\ub2e4. 330\ub144 5\uc6d4 11\uc77c \ucf58\uc2a4\ud0c4\ud2f0\ub204\uc2a4 \ub300\uc81c\ub294 \ube44\uc794\ud2f0\uc628\uc744 \ucf58\uc2a4\ud0c4\ud2f0\ub178\ud3f4\ub9ac\uc2a4\ub85c \uba85\uba85(\ud604 \uc774\uc2a4\ud0c4\ubd88)\ud558\uace0 \ub85c\ub9c8 \uc81c\uad6d\uc758 \uc0c8 \uc218\ub3c4\ub85c \uc0bc\uc558\ub2e4. \uc81c\uad6d\uc740 \ud14c\uc624\ub3c4\uc2dc\uc6b0\uc2a4 1\uc138 \uc0ac\ud6c4\uc778 395\ub144\uc5d0 \ube44\uc794\ud2f0\uc6c0 \uc81c\uad6d\uacfc \uc11c\ub85c\ub9c8 \uc81c\uad6d\uc73c\ub85c \uc601\uad6c\ud788 \uac08\ub77c\uc84c\ub2e4."} {"question": "\uae40\uc625\uade0\uc758 \ubb18\uc640 \ube44\uc11d\uc740 \uc5b4\ub290\uacf3\uc5d0 \uc11c\uc788\ub098?", "paragraph": "1900\ub144 \uc544\uc624\uc57c\ub9c8\uc758 \uc678\uc778 \ubb18\uc9c0\uc5d0 \uc138\uc6b8 \uae40\uc625\uade0\uc758 \ubb18\ube44\ubb38\uc744 \uc774\uc900\uc6a9\uc5d0\uac8c\uc11c \ubc1b\uc544\uc11c \uc9c1\uc811 \ub3cc\uc5d0 \uc0c8\uacbc\ub2e4. \uc544\uc624\uc57c\ub9c8\uc758 \uc678\uc778 \ubb18\uc9c0\uc5d0 \uc11c \uc788\ub294 \uae40\uc625\uade0 \ubb18\uc640 \ube44\uc11d\uc5d0\ub294 \ubc15\uc601\ud6a8\uac00 \ube44\ubb38\uc744 \uc9d3\uace0 \uc774\uc900\uc6a9(\u674e\u57c8\u9394)\uc774 \uae00\uc528\ub97c \uc4f4 \uac83\uc73c\ub85c \ub418\uc5b4 \uc788\uc9c0\ub9cc, \uc2e4\uc0c1 \uadf8 \ube44\ubb38\uc740 \uc720\uae38\uc900\uc774 \uc4f4 \uac83\uc774\ub2e4. \ube44\ubb38\uc740 \ubc15\uc601\ud6a8\uac00 \uc9c0\uc5c8\uace0, \uc774\uc900\uc6a9\uc774 \uae00\uc528\ub97c \uc37c\uc9c0\ub9cc \uc774\uac83\uc744 \ub3cc\uc5d0 \uc9c1\uc811 \uc870\uac01\ud55c \uc774\ub294 \uc720\uae38\uc900\uc774\uc5c8\ub2e4. \ud6d7\ub0a0 \ub300\ud55c\ubbfc\uad6d\uc758 \uc5ed\uc0ac\uac00 \uacb8 \uc791\uac00 \uc2e0\ubd09\uc2b9\uc740 \uc774\ub97c \ub450\uace0 '\ucc38\uc73c\ub85c \uacf5\uad50\ub86d\uac8c\ub3c4 \uc774\ub54c \uc720\uae38\uc900\uc740 \uc870\uc120\uc5d0\uc11c\uc758 \ub610\ub2e4\ub978 \ucfe0\ub370\ud0c0\uc5d0 \uc5f0\ub8e8\ub418\uc5b4 \uc77c\ubcf8 \uc815\ubd80\ub85c\ubd80\ud130 \uc624\uc0ac\uac00\ud558\ub77c \uc12c\uc758 \ubaa8\ub3c4\uc5d0 \uc720\ubc30\ub418\uc5b4 \uc788\uc5c8\ub2e4. \uae40\uc625\uade0\uc774 \uc720\ud3d0\ub418\uc5c8\ub358 \ubc14\ub85c \uadf8 \uc808\ud574\uace0\ub3c4\uc5d0\uc11c \uae40\uc625\uade0\uc758 \ube44\ubb38\uc744 \uc368\uc57c \ud558\ub294 \uc720\uae38\uc900\uc758 \uc2ec\uc815\uc740 \ucc29\uc7a1\ud568\uc744 \ub118\uc5b4\uc11c \uc544\ud514\uc774\uc5c8\uc744 \uac83\uc774\ub2e4.'\ub77c\uace0 \ud3c9\ud558\uc600\ub2e4. \uae40\uc625\uade0\uc758 \ube44\ubb38\uc744 \ub3cc\uc5d0 \uc870\uac01\ud558\uba74\uc11c \uadf8\ub294 \ud1b5\uace1\ud558\uc600\ub2e4. \uc720\ubc30\uc0dd\ud65c \ub0b4\ub0b4 \uadf8\ub294 \uc870\uc120 \uc870\uc815\uacfc \ubbfc\uc911\uc5d0 \ub300\ud55c \ubd88\uc2e0\uacfc \ud610\uc624, \uc801\uac1c\uc2ec\uc744 \ud0a4\uc6e0\ub2e4. \ud2b9\ud788 \uadf8\ub294 \uac1c\ud601\uc790, \uc560\uad6d\uc790\ub97c \uc0ac\uc0b4\ud558\ub294 \uc790\ub4e4, \ud669\uc81c\uac00 \uadf8\ub807\ub2e4\uba74 \uadf8\ub7f0 \uc904 \uc544\ub294 \uc790\ub4e4\uc5d0\uac8c\ub294 \uc778\uac04\uc131 \uac1c\uc870\ub9cc\uc774 \uc815\ub2f5\uc774\ub77c\uba70 \uc870\uc120 \ubbfc\uc911\ub4e4\uc5d0 \ub300\ud55c \ubd88\uc2e0\uacfc \uacbd\uba78, \uc6d0\ud55c, \uba78\uc2dc\ub97c \ud55c\uce35 \uc99d\ud3ed\uc2dc\ucf30\ub2e4.", "answer": "\uc544\uc624\uc57c\ub9c8\uc758 \uc678\uc778 \ubb18\uc9c0", "sentence": "1900\ub144 \uc544\uc624\uc57c\ub9c8\uc758 \uc678\uc778 \ubb18\uc9c0 \uc5d0 \uc138\uc6b8 \uae40\uc625\uade0\uc758 \ubb18\ube44\ubb38\uc744 \uc774\uc900\uc6a9\uc5d0\uac8c\uc11c \ubc1b\uc544\uc11c \uc9c1\uc811 \ub3cc\uc5d0 \uc0c8\uacbc\ub2e4.", "paragraph_sentence": " 1900\ub144 \uc544\uc624\uc57c\ub9c8\uc758 \uc678\uc778 \ubb18\uc9c0 \uc5d0 \uc138\uc6b8 \uae40\uc625\uade0\uc758 \ubb18\ube44\ubb38\uc744 \uc774\uc900\uc6a9\uc5d0\uac8c\uc11c \ubc1b\uc544\uc11c \uc9c1\uc811 \ub3cc\uc5d0 \uc0c8\uacbc\ub2e4. \uc544\uc624\uc57c\ub9c8\uc758 \uc678\uc778 \ubb18\uc9c0\uc5d0 \uc11c \uc788\ub294 \uae40\uc625\uade0 \ubb18\uc640 \ube44\uc11d\uc5d0\ub294 \ubc15\uc601\ud6a8\uac00 \ube44\ubb38\uc744 \uc9d3\uace0 \uc774\uc900\uc6a9(\u674e\u57c8\u9394)\uc774 \uae00\uc528\ub97c \uc4f4 \uac83\uc73c\ub85c \ub418\uc5b4 \uc788\uc9c0\ub9cc, \uc2e4\uc0c1 \uadf8 \ube44\ubb38\uc740 \uc720\uae38\uc900\uc774 \uc4f4 \uac83\uc774\ub2e4. \ube44\ubb38\uc740 \ubc15\uc601\ud6a8\uac00 \uc9c0\uc5c8\uace0, \uc774\uc900\uc6a9\uc774 \uae00\uc528\ub97c \uc37c\uc9c0\ub9cc \uc774\uac83\uc744 \ub3cc\uc5d0 \uc9c1\uc811 \uc870\uac01\ud55c \uc774\ub294 \uc720\uae38\uc900\uc774\uc5c8\ub2e4. \ud6d7\ub0a0 \ub300\ud55c\ubbfc\uad6d\uc758 \uc5ed\uc0ac\uac00 \uacb8 \uc791\uac00 \uc2e0\ubd09\uc2b9\uc740 \uc774\ub97c \ub450\uace0 '\ucc38\uc73c\ub85c \uacf5\uad50\ub86d\uac8c\ub3c4 \uc774\ub54c \uc720\uae38\uc900\uc740 \uc870\uc120\uc5d0\uc11c\uc758 \ub610\ub2e4\ub978 \ucfe0\ub370\ud0c0\uc5d0 \uc5f0\ub8e8\ub418\uc5b4 \uc77c\ubcf8 \uc815\ubd80\ub85c\ubd80\ud130 \uc624\uc0ac\uac00\ud558\ub77c \uc12c\uc758 \ubaa8\ub3c4\uc5d0 \uc720\ubc30\ub418\uc5b4 \uc788\uc5c8\ub2e4. \uae40\uc625\uade0\uc774 \uc720\ud3d0\ub418\uc5c8\ub358 \ubc14\ub85c \uadf8 \uc808\ud574\uace0\ub3c4\uc5d0\uc11c \uae40\uc625\uade0\uc758 \ube44\ubb38\uc744 \uc368\uc57c \ud558\ub294 \uc720\uae38\uc900\uc758 \uc2ec\uc815\uc740 \ucc29\uc7a1\ud568\uc744 \ub118\uc5b4\uc11c \uc544\ud514\uc774\uc5c8\uc744 \uac83\uc774\ub2e4. '\ub77c\uace0 \ud3c9\ud558\uc600\ub2e4. \uae40\uc625\uade0\uc758 \ube44\ubb38\uc744 \ub3cc\uc5d0 \uc870\uac01\ud558\uba74\uc11c \uadf8\ub294 \ud1b5\uace1\ud558\uc600\ub2e4. \uc720\ubc30\uc0dd\ud65c \ub0b4\ub0b4 \uadf8\ub294 \uc870\uc120 \uc870\uc815\uacfc \ubbfc\uc911\uc5d0 \ub300\ud55c \ubd88\uc2e0\uacfc \ud610\uc624, \uc801\uac1c\uc2ec\uc744 \ud0a4\uc6e0\ub2e4. \ud2b9\ud788 \uadf8\ub294 \uac1c\ud601\uc790, \uc560\uad6d\uc790\ub97c \uc0ac\uc0b4\ud558\ub294 \uc790\ub4e4, \ud669\uc81c\uac00 \uadf8\ub807\ub2e4\uba74 \uadf8\ub7f0 \uc904 \uc544\ub294 \uc790\ub4e4\uc5d0\uac8c\ub294 \uc778\uac04\uc131 \uac1c\uc870\ub9cc\uc774 \uc815\ub2f5\uc774\ub77c\uba70 \uc870\uc120 \ubbfc\uc911\ub4e4\uc5d0 \ub300\ud55c \ubd88\uc2e0\uacfc \uacbd\uba78, \uc6d0\ud55c, \uba78\uc2dc\ub97c \ud55c\uce35 \uc99d\ud3ed\uc2dc\ucf30\ub2e4.", "paragraph_answer": "1900\ub144 \uc544\uc624\uc57c\ub9c8\uc758 \uc678\uc778 \ubb18\uc9c0 \uc5d0 \uc138\uc6b8 \uae40\uc625\uade0\uc758 \ubb18\ube44\ubb38\uc744 \uc774\uc900\uc6a9\uc5d0\uac8c\uc11c \ubc1b\uc544\uc11c \uc9c1\uc811 \ub3cc\uc5d0 \uc0c8\uacbc\ub2e4. \uc544\uc624\uc57c\ub9c8\uc758 \uc678\uc778 \ubb18\uc9c0\uc5d0 \uc11c \uc788\ub294 \uae40\uc625\uade0 \ubb18\uc640 \ube44\uc11d\uc5d0\ub294 \ubc15\uc601\ud6a8\uac00 \ube44\ubb38\uc744 \uc9d3\uace0 \uc774\uc900\uc6a9(\u674e\u57c8\u9394)\uc774 \uae00\uc528\ub97c \uc4f4 \uac83\uc73c\ub85c \ub418\uc5b4 \uc788\uc9c0\ub9cc, \uc2e4\uc0c1 \uadf8 \ube44\ubb38\uc740 \uc720\uae38\uc900\uc774 \uc4f4 \uac83\uc774\ub2e4. \ube44\ubb38\uc740 \ubc15\uc601\ud6a8\uac00 \uc9c0\uc5c8\uace0, \uc774\uc900\uc6a9\uc774 \uae00\uc528\ub97c \uc37c\uc9c0\ub9cc \uc774\uac83\uc744 \ub3cc\uc5d0 \uc9c1\uc811 \uc870\uac01\ud55c \uc774\ub294 \uc720\uae38\uc900\uc774\uc5c8\ub2e4. \ud6d7\ub0a0 \ub300\ud55c\ubbfc\uad6d\uc758 \uc5ed\uc0ac\uac00 \uacb8 \uc791\uac00 \uc2e0\ubd09\uc2b9\uc740 \uc774\ub97c \ub450\uace0 '\ucc38\uc73c\ub85c \uacf5\uad50\ub86d\uac8c\ub3c4 \uc774\ub54c \uc720\uae38\uc900\uc740 \uc870\uc120\uc5d0\uc11c\uc758 \ub610\ub2e4\ub978 \ucfe0\ub370\ud0c0\uc5d0 \uc5f0\ub8e8\ub418\uc5b4 \uc77c\ubcf8 \uc815\ubd80\ub85c\ubd80\ud130 \uc624\uc0ac\uac00\ud558\ub77c \uc12c\uc758 \ubaa8\ub3c4\uc5d0 \uc720\ubc30\ub418\uc5b4 \uc788\uc5c8\ub2e4. \uae40\uc625\uade0\uc774 \uc720\ud3d0\ub418\uc5c8\ub358 \ubc14\ub85c \uadf8 \uc808\ud574\uace0\ub3c4\uc5d0\uc11c \uae40\uc625\uade0\uc758 \ube44\ubb38\uc744 \uc368\uc57c \ud558\ub294 \uc720\uae38\uc900\uc758 \uc2ec\uc815\uc740 \ucc29\uc7a1\ud568\uc744 \ub118\uc5b4\uc11c \uc544\ud514\uc774\uc5c8\uc744 \uac83\uc774\ub2e4.'\ub77c\uace0 \ud3c9\ud558\uc600\ub2e4. \uae40\uc625\uade0\uc758 \ube44\ubb38\uc744 \ub3cc\uc5d0 \uc870\uac01\ud558\uba74\uc11c \uadf8\ub294 \ud1b5\uace1\ud558\uc600\ub2e4. \uc720\ubc30\uc0dd\ud65c \ub0b4\ub0b4 \uadf8\ub294 \uc870\uc120 \uc870\uc815\uacfc \ubbfc\uc911\uc5d0 \ub300\ud55c \ubd88\uc2e0\uacfc \ud610\uc624, \uc801\uac1c\uc2ec\uc744 \ud0a4\uc6e0\ub2e4. \ud2b9\ud788 \uadf8\ub294 \uac1c\ud601\uc790, \uc560\uad6d\uc790\ub97c \uc0ac\uc0b4\ud558\ub294 \uc790\ub4e4, \ud669\uc81c\uac00 \uadf8\ub807\ub2e4\uba74 \uadf8\ub7f0 \uc904 \uc544\ub294 \uc790\ub4e4\uc5d0\uac8c\ub294 \uc778\uac04\uc131 \uac1c\uc870\ub9cc\uc774 \uc815\ub2f5\uc774\ub77c\uba70 \uc870\uc120 \ubbfc\uc911\ub4e4\uc5d0 \ub300\ud55c \ubd88\uc2e0\uacfc \uacbd\uba78, \uc6d0\ud55c, \uba78\uc2dc\ub97c \ud55c\uce35 \uc99d\ud3ed\uc2dc\ucf30\ub2e4.", "sentence_answer": "1900\ub144 \uc544\uc624\uc57c\ub9c8\uc758 \uc678\uc778 \ubb18\uc9c0 \uc5d0 \uc138\uc6b8 \uae40\uc625\uade0\uc758 \ubb18\ube44\ubb38\uc744 \uc774\uc900\uc6a9\uc5d0\uac8c\uc11c \ubc1b\uc544\uc11c \uc9c1\uc811 \ub3cc\uc5d0 \uc0c8\uacbc\ub2e4.", "paragraph_id": "6542752-20-1", "paragraph_question": "question: \uae40\uc625\uade0\uc758 \ubb18\uc640 \ube44\uc11d\uc740 \uc5b4\ub290\uacf3\uc5d0 \uc11c\uc788\ub098?, context: 1900\ub144 \uc544\uc624\uc57c\ub9c8\uc758 \uc678\uc778 \ubb18\uc9c0\uc5d0 \uc138\uc6b8 \uae40\uc625\uade0\uc758 \ubb18\ube44\ubb38\uc744 \uc774\uc900\uc6a9\uc5d0\uac8c\uc11c \ubc1b\uc544\uc11c \uc9c1\uc811 \ub3cc\uc5d0 \uc0c8\uacbc\ub2e4. \uc544\uc624\uc57c\ub9c8\uc758 \uc678\uc778 \ubb18\uc9c0\uc5d0 \uc11c \uc788\ub294 \uae40\uc625\uade0 \ubb18\uc640 \ube44\uc11d\uc5d0\ub294 \ubc15\uc601\ud6a8\uac00 \ube44\ubb38\uc744 \uc9d3\uace0 \uc774\uc900\uc6a9(\u674e\u57c8\u9394)\uc774 \uae00\uc528\ub97c \uc4f4 \uac83\uc73c\ub85c \ub418\uc5b4 \uc788\uc9c0\ub9cc, \uc2e4\uc0c1 \uadf8 \ube44\ubb38\uc740 \uc720\uae38\uc900\uc774 \uc4f4 \uac83\uc774\ub2e4. \ube44\ubb38\uc740 \ubc15\uc601\ud6a8\uac00 \uc9c0\uc5c8\uace0, \uc774\uc900\uc6a9\uc774 \uae00\uc528\ub97c \uc37c\uc9c0\ub9cc \uc774\uac83\uc744 \ub3cc\uc5d0 \uc9c1\uc811 \uc870\uac01\ud55c \uc774\ub294 \uc720\uae38\uc900\uc774\uc5c8\ub2e4. \ud6d7\ub0a0 \ub300\ud55c\ubbfc\uad6d\uc758 \uc5ed\uc0ac\uac00 \uacb8 \uc791\uac00 \uc2e0\ubd09\uc2b9\uc740 \uc774\ub97c \ub450\uace0 '\ucc38\uc73c\ub85c \uacf5\uad50\ub86d\uac8c\ub3c4 \uc774\ub54c \uc720\uae38\uc900\uc740 \uc870\uc120\uc5d0\uc11c\uc758 \ub610\ub2e4\ub978 \ucfe0\ub370\ud0c0\uc5d0 \uc5f0\ub8e8\ub418\uc5b4 \uc77c\ubcf8 \uc815\ubd80\ub85c\ubd80\ud130 \uc624\uc0ac\uac00\ud558\ub77c \uc12c\uc758 \ubaa8\ub3c4\uc5d0 \uc720\ubc30\ub418\uc5b4 \uc788\uc5c8\ub2e4. \uae40\uc625\uade0\uc774 \uc720\ud3d0\ub418\uc5c8\ub358 \ubc14\ub85c \uadf8 \uc808\ud574\uace0\ub3c4\uc5d0\uc11c \uae40\uc625\uade0\uc758 \ube44\ubb38\uc744 \uc368\uc57c \ud558\ub294 \uc720\uae38\uc900\uc758 \uc2ec\uc815\uc740 \ucc29\uc7a1\ud568\uc744 \ub118\uc5b4\uc11c \uc544\ud514\uc774\uc5c8\uc744 \uac83\uc774\ub2e4.'\ub77c\uace0 \ud3c9\ud558\uc600\ub2e4. \uae40\uc625\uade0\uc758 \ube44\ubb38\uc744 \ub3cc\uc5d0 \uc870\uac01\ud558\uba74\uc11c \uadf8\ub294 \ud1b5\uace1\ud558\uc600\ub2e4. \uc720\ubc30\uc0dd\ud65c \ub0b4\ub0b4 \uadf8\ub294 \uc870\uc120 \uc870\uc815\uacfc \ubbfc\uc911\uc5d0 \ub300\ud55c \ubd88\uc2e0\uacfc \ud610\uc624, \uc801\uac1c\uc2ec\uc744 \ud0a4\uc6e0\ub2e4. \ud2b9\ud788 \uadf8\ub294 \uac1c\ud601\uc790, \uc560\uad6d\uc790\ub97c \uc0ac\uc0b4\ud558\ub294 \uc790\ub4e4, \ud669\uc81c\uac00 \uadf8\ub807\ub2e4\uba74 \uadf8\ub7f0 \uc904 \uc544\ub294 \uc790\ub4e4\uc5d0\uac8c\ub294 \uc778\uac04\uc131 \uac1c\uc870\ub9cc\uc774 \uc815\ub2f5\uc774\ub77c\uba70 \uc870\uc120 \ubbfc\uc911\ub4e4\uc5d0 \ub300\ud55c \ubd88\uc2e0\uacfc \uacbd\uba78, \uc6d0\ud55c, \uba78\uc2dc\ub97c \ud55c\uce35 \uc99d\ud3ed\uc2dc\ucf30\ub2e4."} {"question": "1927\ub144 \uc544\uba54\ub9ac\uce78 \ub9ac\uadf8\uc640 \uc6d4\ub4dc \uc2dc\ub9ac\uc988 \ubaa8\ub450 \uc6b0\uc2b9\ud55c \ud300\uc740?", "paragraph": "1927\ub144 \ub8e8 \uac8c\ub9ad\uc740 \ud0c0\uc790 \uc5ed\uc0ac\uc0c1 \uac00\uc7a5 \uc704\ub300\ud55c \uc2dc\uc98c \uc911\uc758 \ud558\ub098\ub97c \ubcf4\ub0b4\uac8c \ub418\uc5c8\ub2e4. \uadf8 \ud574 218\uac1c\uc758 \uc548\ud0c0\ub97c \uce58\uba74\uc11c 0.373\uc758 \ud0c0\uc728, 52\uac1c\uc758 2\ub8e8\ud0c0, 18\uac1c\uc758 3\ub8e8\ud0c0, 47\uac1c\uc758 \ud648\ub7f0\uacfc 175 \ud0c0\uc810\uc744 \uae30\ub85d\ud588\uace0, \uc7a5\ud0c0\uc728\uc740 \ubb34\ub824 0.765(7\ud560 6\ud47c 5\ub9ac)\uc5d0 \uc774\ub974\ub800\ub2e4. 117\uac1c\uc758 \uc7a5\ud0c0(Extra-base Hits: \uc548\ud0c0\uc911\uc5d0\uc11c \ub2e8\ud0c0\ub97c \uc81c\uc678\ud55c \uac83) \ub294 \ubca0\uc774\ube0c \ub8e8\uc2a4\uac00 1921\ub144 \uae30\ub85d\ud55c 119\uac1c\uc5d0 \uc774\uc740 \uc5ed\ub300 2\ubc88\uc9f8 \uae30\ub85d\uc774\uace0, 447 \ucd1d\ub8e8\ud0c0(Total Bases) \ub294 1921\ub144 \uae30\ub85d\ud55c \ubca0\uc774\ube0c \ub8e8\uc2a4\uc758 457\ub8e8\ud0c0\uc640 1922\ub144 \ub85c\uc800\uc2a4 \ud63c\uc988\ube44\uac00 \uae30\ub85d\ud55c 450\ub8e8\ud0c0\uc5d0 \uc774\uc740 \uc5ed\ub300 3\ubc88\uc9f8 \uae30\ub85d\uc774\ub2e4. 1927\ub144 \ub8e8 \uac8c\ub9ad\uc758 \uc5c4\uccad\ub09c \ud0c0\uaca9 \ud589\uc9c4\uc5d0 \ud798\uc785\uc5b4 \uc591\ud0a4\uc2a4\ub294 110\uc2b9 44\ud328\ub97c \uae30\ub85d\ud558\uba70, \uc544\uba54\ub9ac\uce78 \ub9ac\uadf8 \uc6b0\uc2b9\uc744 \ud558\uace0 \ud53c\uce20\ubc84\uadf8 \ud30c\uc774\ub9ac\uce20\uc640 \uaca8\ub8ec 1927\ub144 \uc6d4\ub4dc \uc2dc\ub9ac\uc988\uc5d0\uc11c\ub3c4 4\uc2b9 \ubb34\ud328\uc758 \uae30\ub85d\uc73c\ub85c \uc6b0\uc2b9\uc744 \ucc28\uc9c0\ud558\uc600\ub2e4. \uc544\uba54\ub9ac\uce78 \ub9ac\uadf8 \ucd5c\uc6b0\uc218 \uc120\uc218\uc5d0 \ubf51\ud614\uc9c0\ub9cc, \ub8e8 \uac8c\ub9ad\uc758 1927\ub144 \uc2dc\uc98c\uc5d0 \ub300\ud574 \ubca0\uc774\ube0c \ub8e8\uc2a4\uc758 60\ud648\ub7f0\uacfc \uadf8 \ub2f9\uc2dc \ub274\uc695 \uc591\ud0a4\uc2a4\uc758 \ud0c0\uc120\uc774 \ub9ac\uadf8 \uc804\uccb4\ub97c \uc555\ub3c4\ud560 \uc815\ub3c4\ub85c \uac15\ub825\ud588\ub358 (\ud6d7\ub0a0 \ub300\ubd80\ubd84\uc758 \uc120\uc218\uac00 \uba85\uc608\uc758 \uc804\ub2f9\uc5d0 \uc785\uc131\ud55c \uc120\uc218\ub85c \uc774\ub8e8\uc5b4\uc9c4) '\uc0b4\uc778 \ud0c0\uc120'(Murderer's Row)\uc778 \uc774\uc720\ub97c \ub4e4\uc5b4 \uadf8 \uac00\uce58\ub97c \uc808\ud558\ud558\ub294 \uc0ac\ub78c\ub4e4\ub3c4 \uc788\ub2e4.", "answer": "\ub274\uc695 \uc591\ud0a4\uc2a4", "sentence": "\uc544\uba54\ub9ac\uce78 \ub9ac\uadf8 \ucd5c\uc6b0\uc218 \uc120\uc218\uc5d0 \ubf51\ud614\uc9c0\ub9cc, \ub8e8 \uac8c\ub9ad\uc758 1927\ub144 \uc2dc\uc98c\uc5d0 \ub300\ud574 \ubca0\uc774\ube0c \ub8e8\uc2a4\uc758 60\ud648\ub7f0\uacfc \uadf8 \ub2f9\uc2dc \ub274\uc695 \uc591\ud0a4\uc2a4 \uc758 \ud0c0\uc120\uc774 \ub9ac\uadf8 \uc804\uccb4\ub97c \uc555\ub3c4\ud560 \uc815\ub3c4\ub85c \uac15\ub825\ud588\ub358 (\ud6d7\ub0a0 \ub300\ubd80\ubd84\uc758 \uc120\uc218\uac00 \uba85\uc608\uc758 \uc804\ub2f9\uc5d0 \uc785\uc131\ud55c \uc120\uc218\ub85c \uc774\ub8e8\uc5b4\uc9c4) '\uc0b4\uc778 \ud0c0\uc120'(Murderer's Row)\uc778 \uc774\uc720\ub97c \ub4e4\uc5b4 \uadf8 \uac00\uce58\ub97c \uc808\ud558\ud558\ub294 \uc0ac\ub78c\ub4e4\ub3c4 \uc788\ub2e4.", "paragraph_sentence": "1927\ub144 \ub8e8 \uac8c\ub9ad\uc740 \ud0c0\uc790 \uc5ed\uc0ac\uc0c1 \uac00\uc7a5 \uc704\ub300\ud55c \uc2dc\uc98c \uc911\uc758 \ud558\ub098\ub97c \ubcf4\ub0b4\uac8c \ub418\uc5c8\ub2e4. \uadf8 \ud574 218\uac1c\uc758 \uc548\ud0c0\ub97c \uce58\uba74\uc11c 0.373\uc758 \ud0c0\uc728, 52\uac1c\uc758 2\ub8e8\ud0c0, 18\uac1c\uc758 3\ub8e8\ud0c0, 47\uac1c\uc758 \ud648\ub7f0\uacfc 175 \ud0c0\uc810\uc744 \uae30\ub85d\ud588\uace0, \uc7a5\ud0c0\uc728\uc740 \ubb34\ub824 0.765(7\ud560 6\ud47c 5\ub9ac)\uc5d0 \uc774\ub974\ub800\ub2e4. 117\uac1c\uc758 \uc7a5\ud0c0(Extra-base Hits: \uc548\ud0c0\uc911\uc5d0\uc11c \ub2e8\ud0c0\ub97c \uc81c\uc678\ud55c \uac83) \ub294 \ubca0\uc774\ube0c \ub8e8\uc2a4\uac00 1921\ub144 \uae30\ub85d\ud55c 119\uac1c\uc5d0 \uc774\uc740 \uc5ed\ub300 2\ubc88\uc9f8 \uae30\ub85d\uc774\uace0, 447 \ucd1d\ub8e8\ud0c0(Total Bases) \ub294 1921\ub144 \uae30\ub85d\ud55c \ubca0\uc774\ube0c \ub8e8\uc2a4\uc758 457\ub8e8\ud0c0\uc640 1922\ub144 \ub85c\uc800\uc2a4 \ud63c\uc988\ube44\uac00 \uae30\ub85d\ud55c 450\ub8e8\ud0c0\uc5d0 \uc774\uc740 \uc5ed\ub300 3\ubc88\uc9f8 \uae30\ub85d\uc774\ub2e4. 1927\ub144 \ub8e8 \uac8c\ub9ad\uc758 \uc5c4\uccad\ub09c \ud0c0\uaca9 \ud589\uc9c4\uc5d0 \ud798\uc785\uc5b4 \uc591\ud0a4\uc2a4\ub294 110\uc2b9 44\ud328\ub97c \uae30\ub85d\ud558\uba70, \uc544\uba54\ub9ac\uce78 \ub9ac\uadf8 \uc6b0\uc2b9\uc744 \ud558\uace0 \ud53c\uce20\ubc84\uadf8 \ud30c\uc774\ub9ac\uce20\uc640 \uaca8\ub8ec 1927\ub144 \uc6d4\ub4dc \uc2dc\ub9ac\uc988\uc5d0\uc11c\ub3c4 4\uc2b9 \ubb34\ud328\uc758 \uae30\ub85d\uc73c\ub85c \uc6b0\uc2b9\uc744 \ucc28\uc9c0\ud558\uc600\ub2e4. \uc544\uba54\ub9ac\uce78 \ub9ac\uadf8 \ucd5c\uc6b0\uc218 \uc120\uc218\uc5d0 \ubf51\ud614\uc9c0\ub9cc, \ub8e8 \uac8c\ub9ad\uc758 1927\ub144 \uc2dc\uc98c\uc5d0 \ub300\ud574 \ubca0\uc774\ube0c \ub8e8\uc2a4\uc758 60\ud648\ub7f0\uacfc \uadf8 \ub2f9\uc2dc \ub274\uc695 \uc591\ud0a4\uc2a4 \uc758 \ud0c0\uc120\uc774 \ub9ac\uadf8 \uc804\uccb4\ub97c \uc555\ub3c4\ud560 \uc815\ub3c4\ub85c \uac15\ub825\ud588\ub358 (\ud6d7\ub0a0 \ub300\ubd80\ubd84\uc758 \uc120\uc218\uac00 \uba85\uc608\uc758 \uc804\ub2f9\uc5d0 \uc785\uc131\ud55c \uc120\uc218\ub85c \uc774\ub8e8\uc5b4\uc9c4) '\uc0b4\uc778 \ud0c0\uc120'(Murderer's Row)\uc778 \uc774\uc720\ub97c \ub4e4\uc5b4 \uadf8 \uac00\uce58\ub97c \uc808\ud558\ud558\ub294 \uc0ac\ub78c\ub4e4\ub3c4 \uc788\ub2e4. ", "paragraph_answer": "1927\ub144 \ub8e8 \uac8c\ub9ad\uc740 \ud0c0\uc790 \uc5ed\uc0ac\uc0c1 \uac00\uc7a5 \uc704\ub300\ud55c \uc2dc\uc98c \uc911\uc758 \ud558\ub098\ub97c \ubcf4\ub0b4\uac8c \ub418\uc5c8\ub2e4. \uadf8 \ud574 218\uac1c\uc758 \uc548\ud0c0\ub97c \uce58\uba74\uc11c 0.373\uc758 \ud0c0\uc728, 52\uac1c\uc758 2\ub8e8\ud0c0, 18\uac1c\uc758 3\ub8e8\ud0c0, 47\uac1c\uc758 \ud648\ub7f0\uacfc 175 \ud0c0\uc810\uc744 \uae30\ub85d\ud588\uace0, \uc7a5\ud0c0\uc728\uc740 \ubb34\ub824 0.765(7\ud560 6\ud47c 5\ub9ac)\uc5d0 \uc774\ub974\ub800\ub2e4. 117\uac1c\uc758 \uc7a5\ud0c0(Extra-base Hits: \uc548\ud0c0\uc911\uc5d0\uc11c \ub2e8\ud0c0\ub97c \uc81c\uc678\ud55c \uac83) \ub294 \ubca0\uc774\ube0c \ub8e8\uc2a4\uac00 1921\ub144 \uae30\ub85d\ud55c 119\uac1c\uc5d0 \uc774\uc740 \uc5ed\ub300 2\ubc88\uc9f8 \uae30\ub85d\uc774\uace0, 447 \ucd1d\ub8e8\ud0c0(Total Bases) \ub294 1921\ub144 \uae30\ub85d\ud55c \ubca0\uc774\ube0c \ub8e8\uc2a4\uc758 457\ub8e8\ud0c0\uc640 1922\ub144 \ub85c\uc800\uc2a4 \ud63c\uc988\ube44\uac00 \uae30\ub85d\ud55c 450\ub8e8\ud0c0\uc5d0 \uc774\uc740 \uc5ed\ub300 3\ubc88\uc9f8 \uae30\ub85d\uc774\ub2e4. 1927\ub144 \ub8e8 \uac8c\ub9ad\uc758 \uc5c4\uccad\ub09c \ud0c0\uaca9 \ud589\uc9c4\uc5d0 \ud798\uc785\uc5b4 \uc591\ud0a4\uc2a4\ub294 110\uc2b9 44\ud328\ub97c \uae30\ub85d\ud558\uba70, \uc544\uba54\ub9ac\uce78 \ub9ac\uadf8 \uc6b0\uc2b9\uc744 \ud558\uace0 \ud53c\uce20\ubc84\uadf8 \ud30c\uc774\ub9ac\uce20\uc640 \uaca8\ub8ec 1927\ub144 \uc6d4\ub4dc \uc2dc\ub9ac\uc988\uc5d0\uc11c\ub3c4 4\uc2b9 \ubb34\ud328\uc758 \uae30\ub85d\uc73c\ub85c \uc6b0\uc2b9\uc744 \ucc28\uc9c0\ud558\uc600\ub2e4. \uc544\uba54\ub9ac\uce78 \ub9ac\uadf8 \ucd5c\uc6b0\uc218 \uc120\uc218\uc5d0 \ubf51\ud614\uc9c0\ub9cc, \ub8e8 \uac8c\ub9ad\uc758 1927\ub144 \uc2dc\uc98c\uc5d0 \ub300\ud574 \ubca0\uc774\ube0c \ub8e8\uc2a4\uc758 60\ud648\ub7f0\uacfc \uadf8 \ub2f9\uc2dc \ub274\uc695 \uc591\ud0a4\uc2a4 \uc758 \ud0c0\uc120\uc774 \ub9ac\uadf8 \uc804\uccb4\ub97c \uc555\ub3c4\ud560 \uc815\ub3c4\ub85c \uac15\ub825\ud588\ub358 (\ud6d7\ub0a0 \ub300\ubd80\ubd84\uc758 \uc120\uc218\uac00 \uba85\uc608\uc758 \uc804\ub2f9\uc5d0 \uc785\uc131\ud55c \uc120\uc218\ub85c \uc774\ub8e8\uc5b4\uc9c4) '\uc0b4\uc778 \ud0c0\uc120'(Murderer's Row)\uc778 \uc774\uc720\ub97c \ub4e4\uc5b4 \uadf8 \uac00\uce58\ub97c \uc808\ud558\ud558\ub294 \uc0ac\ub78c\ub4e4\ub3c4 \uc788\ub2e4.", "sentence_answer": "\uc544\uba54\ub9ac\uce78 \ub9ac\uadf8 \ucd5c\uc6b0\uc218 \uc120\uc218\uc5d0 \ubf51\ud614\uc9c0\ub9cc, \ub8e8 \uac8c\ub9ad\uc758 1927\ub144 \uc2dc\uc98c\uc5d0 \ub300\ud574 \ubca0\uc774\ube0c \ub8e8\uc2a4\uc758 60\ud648\ub7f0\uacfc \uadf8 \ub2f9\uc2dc \ub274\uc695 \uc591\ud0a4\uc2a4 \uc758 \ud0c0\uc120\uc774 \ub9ac\uadf8 \uc804\uccb4\ub97c \uc555\ub3c4\ud560 \uc815\ub3c4\ub85c \uac15\ub825\ud588\ub358 (\ud6d7\ub0a0 \ub300\ubd80\ubd84\uc758 \uc120\uc218\uac00 \uba85\uc608\uc758 \uc804\ub2f9\uc5d0 \uc785\uc131\ud55c \uc120\uc218\ub85c \uc774\ub8e8\uc5b4\uc9c4) '\uc0b4\uc778 \ud0c0\uc120'(Murderer's Row)\uc778 \uc774\uc720\ub97c \ub4e4\uc5b4 \uadf8 \uac00\uce58\ub97c \uc808\ud558\ud558\ub294 \uc0ac\ub78c\ub4e4\ub3c4 \uc788\ub2e4.", "paragraph_id": "6513788-1-0", "paragraph_question": "question: 1927\ub144 \uc544\uba54\ub9ac\uce78 \ub9ac\uadf8\uc640 \uc6d4\ub4dc \uc2dc\ub9ac\uc988 \ubaa8\ub450 \uc6b0\uc2b9\ud55c \ud300\uc740?, context: 1927\ub144 \ub8e8 \uac8c\ub9ad\uc740 \ud0c0\uc790 \uc5ed\uc0ac\uc0c1 \uac00\uc7a5 \uc704\ub300\ud55c \uc2dc\uc98c \uc911\uc758 \ud558\ub098\ub97c \ubcf4\ub0b4\uac8c \ub418\uc5c8\ub2e4. \uadf8 \ud574 218\uac1c\uc758 \uc548\ud0c0\ub97c \uce58\uba74\uc11c 0.373\uc758 \ud0c0\uc728, 52\uac1c\uc758 2\ub8e8\ud0c0, 18\uac1c\uc758 3\ub8e8\ud0c0, 47\uac1c\uc758 \ud648\ub7f0\uacfc 175 \ud0c0\uc810\uc744 \uae30\ub85d\ud588\uace0, \uc7a5\ud0c0\uc728\uc740 \ubb34\ub824 0.765(7\ud560 6\ud47c 5\ub9ac)\uc5d0 \uc774\ub974\ub800\ub2e4. 117\uac1c\uc758 \uc7a5\ud0c0(Extra-base Hits: \uc548\ud0c0\uc911\uc5d0\uc11c \ub2e8\ud0c0\ub97c \uc81c\uc678\ud55c \uac83) \ub294 \ubca0\uc774\ube0c \ub8e8\uc2a4\uac00 1921\ub144 \uae30\ub85d\ud55c 119\uac1c\uc5d0 \uc774\uc740 \uc5ed\ub300 2\ubc88\uc9f8 \uae30\ub85d\uc774\uace0, 447 \ucd1d\ub8e8\ud0c0(Total Bases) \ub294 1921\ub144 \uae30\ub85d\ud55c \ubca0\uc774\ube0c \ub8e8\uc2a4\uc758 457\ub8e8\ud0c0\uc640 1922\ub144 \ub85c\uc800\uc2a4 \ud63c\uc988\ube44\uac00 \uae30\ub85d\ud55c 450\ub8e8\ud0c0\uc5d0 \uc774\uc740 \uc5ed\ub300 3\ubc88\uc9f8 \uae30\ub85d\uc774\ub2e4. 1927\ub144 \ub8e8 \uac8c\ub9ad\uc758 \uc5c4\uccad\ub09c \ud0c0\uaca9 \ud589\uc9c4\uc5d0 \ud798\uc785\uc5b4 \uc591\ud0a4\uc2a4\ub294 110\uc2b9 44\ud328\ub97c \uae30\ub85d\ud558\uba70, \uc544\uba54\ub9ac\uce78 \ub9ac\uadf8 \uc6b0\uc2b9\uc744 \ud558\uace0 \ud53c\uce20\ubc84\uadf8 \ud30c\uc774\ub9ac\uce20\uc640 \uaca8\ub8ec 1927\ub144 \uc6d4\ub4dc \uc2dc\ub9ac\uc988\uc5d0\uc11c\ub3c4 4\uc2b9 \ubb34\ud328\uc758 \uae30\ub85d\uc73c\ub85c \uc6b0\uc2b9\uc744 \ucc28\uc9c0\ud558\uc600\ub2e4. \uc544\uba54\ub9ac\uce78 \ub9ac\uadf8 \ucd5c\uc6b0\uc218 \uc120\uc218\uc5d0 \ubf51\ud614\uc9c0\ub9cc, \ub8e8 \uac8c\ub9ad\uc758 1927\ub144 \uc2dc\uc98c\uc5d0 \ub300\ud574 \ubca0\uc774\ube0c \ub8e8\uc2a4\uc758 60\ud648\ub7f0\uacfc \uadf8 \ub2f9\uc2dc \ub274\uc695 \uc591\ud0a4\uc2a4\uc758 \ud0c0\uc120\uc774 \ub9ac\uadf8 \uc804\uccb4\ub97c \uc555\ub3c4\ud560 \uc815\ub3c4\ub85c \uac15\ub825\ud588\ub358 (\ud6d7\ub0a0 \ub300\ubd80\ubd84\uc758 \uc120\uc218\uac00 \uba85\uc608\uc758 \uc804\ub2f9\uc5d0 \uc785\uc131\ud55c \uc120\uc218\ub85c \uc774\ub8e8\uc5b4\uc9c4) '\uc0b4\uc778 \ud0c0\uc120'(Murderer's Row)\uc778 \uc774\uc720\ub97c \ub4e4\uc5b4 \uadf8 \uac00\uce58\ub97c \uc808\ud558\ud558\ub294 \uc0ac\ub78c\ub4e4\ub3c4 \uc788\ub2e4."} {"question": "\uc624\uc2a4\ud2b8\ub808\uc77c\ub9ac\uc544\uccad\uac1c\uad6c\ub9ac\uac00 \ubc24\uc5d0 \uc870\uba85\uc73c\ub85c \ubaa8\uc5ec\ub4dc\ub294 \uc774\uc720\ub294 \ubb34\uc5c7\uc744 \uba39\uae30 \uc704\ud568\uc778\uac00?", "paragraph": "\uc624\uc2a4\ud2b8\ub808\uc77c\ub9ac\uc544\uccad\uac1c\uad6c\ub9ac\ub294 \uc778\uac04\uc744 \uc798 \uac81\ub0b4\uc9c0 \uc54a\uc73c\uba70, \uc778\uac04\uacfc \uac00\uae4c\uc6b4 \uacf3\uc5d0 \ub20c\ub7ec\uc549\uc544 \uc0ac\ub294 \uacbd\uc6b0\ub3c4 \ud754\ud558\ub2e4. \uac00\ub054\uc740 \uc544\uc608 \uc9d1 \uc548\uc5d0 \ub4e4\uc5b4\uc640\uc11c \uac1c\uc218\ub300\ub098 \ubcc0\uae30 \uc18d\uc5d0\uc11c \ubc1c\uacac\ub418\uae30\ub3c4 \ud55c\ub2e4. \ub610\ud55c \ubc24\uc774 \ub418\uba74 \ucc3d\ud2c0 \ubc16\uc5d0 \ubd99\uc5b4\uc11c \ubbfc\uac00\uc758 \ubd88\ube5b\uc5d0 \uc720\ud639\ub41c \ub0a0\ubc8c\ub808\ub4e4\uc744 \uc7a1\uc544\uba39\ub294\ub370, \uc544\ub9c8 \uac19\uc740 \uc774\uc720\ub85c \uc57c\uc678 \uc870\uba85\uc5d0\ub3c4 \ubaa8\uc5ec\ub4dc\ub294 \uac83\uc73c\ub85c \ubcf4\uc778\ub2e4. \uadf8 \uc678\uc5d0\ub3c4 \ubb3c\ud0f1\ud06c, \uc218\uc9c1 \ub099\uc218\ud1b5, \uc218\ud3c9 \ud648\ud1b5 \ub4f1 \ubc14\uae65\ubcf4\ub2e4 \uc11c\ub298\ud558\uace0 \uc2b5\ub3c4\uac00 \ub192\uc740 \uc778\uacf5 \uad6c\uc870\ubb3c \uc18d\uc744 \ucc28\uc9c0\ud558\uace0 \uc0ac\ub294 \uacbd\uc6b0\uac00 \uc788\ub2e4. \ud648\ud1b5\uc774\ub098 \ubb3c\ud0f1\ud06c \uc18d\uc5d0\uc11c \uc6b8\uba74 \uc6b8\uc74c\uc18c\ub9ac\uac00 \uba54\uc544\ub9ac\uccd0 \uc99d\ud3ed\ub418\uae30 \ub54c\ubb38\uc5d0 \ubc88\uc2dd\uae30\uc5d0 \uc774 \ud6a8\uacfc\ub97c \ub178\ub9ac\uace0 \uc5ec\uae30 \uae30\uc5b4\ub4e4\uc5b4\uc640 \uc0ac\ub294 \uac83\uc73c\ub85c \ubcf4\uc778\ub2e4. \uc624\uc2a4\ud2b8\ub808\uc77c\ub9ac\uc544\uccad\uac1c\uad6c\ub9ac\ub294 \uadc0\uc18c\uc131 \ub2a5\ub825\ub3c4 \uac00\uc9c0\uace0 \uc788\ub294 \uac83\uc73c\ub85c \ubcf4\uc778\ub2e4. \ud3ec\ud68d\ud55c \ub4a4 \uba40\ub9ac \ub5a8\uc5b4\uc9c4 \uacf3\uc5d0 \ub0b4\ubc84\ub824\ub3c4 \ucc98\uc74c \uc7a1\ud614\ub358 \uacf3\uc73c\ub85c \ub2e4\uc2dc \ub3cc\uc544\uac08 \uc218 \uc788\ub2e4.", "answer": "\ub0a0\ubc8c\ub808", "sentence": "\ub610\ud55c \ubc24\uc774 \ub418\uba74 \ucc3d\ud2c0 \ubc16\uc5d0 \ubd99\uc5b4\uc11c \ubbfc\uac00\uc758 \ubd88\ube5b\uc5d0 \uc720\ud639\ub41c \ub0a0\ubc8c\ub808 \ub4e4\uc744 \uc7a1\uc544\uba39\ub294\ub370, \uc544\ub9c8 \uac19\uc740 \uc774\uc720\ub85c \uc57c\uc678 \uc870\uba85\uc5d0\ub3c4 \ubaa8\uc5ec\ub4dc\ub294 \uac83\uc73c\ub85c \ubcf4\uc778\ub2e4.", "paragraph_sentence": "\uc624\uc2a4\ud2b8\ub808\uc77c\ub9ac\uc544\uccad\uac1c\uad6c\ub9ac\ub294 \uc778\uac04\uc744 \uc798 \uac81\ub0b4\uc9c0 \uc54a\uc73c\uba70, \uc778\uac04\uacfc \uac00\uae4c\uc6b4 \uacf3\uc5d0 \ub20c\ub7ec\uc549\uc544 \uc0ac\ub294 \uacbd\uc6b0\ub3c4 \ud754\ud558\ub2e4. \uac00\ub054\uc740 \uc544\uc608 \uc9d1 \uc548\uc5d0 \ub4e4\uc5b4\uc640\uc11c \uac1c\uc218\ub300\ub098 \ubcc0\uae30 \uc18d\uc5d0\uc11c \ubc1c\uacac\ub418\uae30\ub3c4 \ud55c\ub2e4. \ub610\ud55c \ubc24\uc774 \ub418\uba74 \ucc3d\ud2c0 \ubc16\uc5d0 \ubd99\uc5b4\uc11c \ubbfc\uac00\uc758 \ubd88\ube5b\uc5d0 \uc720\ud639\ub41c \ub0a0\ubc8c\ub808 \ub4e4\uc744 \uc7a1\uc544\uba39\ub294\ub370, \uc544\ub9c8 \uac19\uc740 \uc774\uc720\ub85c \uc57c\uc678 \uc870\uba85\uc5d0\ub3c4 \ubaa8\uc5ec\ub4dc\ub294 \uac83\uc73c\ub85c \ubcf4\uc778\ub2e4. \uadf8 \uc678\uc5d0\ub3c4 \ubb3c\ud0f1\ud06c, \uc218\uc9c1 \ub099\uc218\ud1b5, \uc218\ud3c9 \ud648\ud1b5 \ub4f1 \ubc14\uae65\ubcf4\ub2e4 \uc11c\ub298\ud558\uace0 \uc2b5\ub3c4\uac00 \ub192\uc740 \uc778\uacf5 \uad6c\uc870\ubb3c \uc18d\uc744 \ucc28\uc9c0\ud558\uace0 \uc0ac\ub294 \uacbd\uc6b0\uac00 \uc788\ub2e4. \ud648\ud1b5\uc774\ub098 \ubb3c\ud0f1\ud06c \uc18d\uc5d0\uc11c \uc6b8\uba74 \uc6b8\uc74c\uc18c\ub9ac\uac00 \uba54\uc544\ub9ac\uccd0 \uc99d\ud3ed\ub418\uae30 \ub54c\ubb38\uc5d0 \ubc88\uc2dd\uae30\uc5d0 \uc774 \ud6a8\uacfc\ub97c \ub178\ub9ac\uace0 \uc5ec\uae30 \uae30\uc5b4\ub4e4\uc5b4\uc640 \uc0ac\ub294 \uac83\uc73c\ub85c \ubcf4\uc778\ub2e4. \uc624\uc2a4\ud2b8\ub808\uc77c\ub9ac\uc544\uccad\uac1c\uad6c\ub9ac\ub294 \uadc0\uc18c\uc131 \ub2a5\ub825\ub3c4 \uac00\uc9c0\uace0 \uc788\ub294 \uac83\uc73c\ub85c \ubcf4\uc778\ub2e4. \ud3ec\ud68d\ud55c \ub4a4 \uba40\ub9ac \ub5a8\uc5b4\uc9c4 \uacf3\uc5d0 \ub0b4\ubc84\ub824\ub3c4 \ucc98\uc74c \uc7a1\ud614\ub358 \uacf3\uc73c\ub85c \ub2e4\uc2dc \ub3cc\uc544\uac08 \uc218 \uc788\ub2e4.", "paragraph_answer": "\uc624\uc2a4\ud2b8\ub808\uc77c\ub9ac\uc544\uccad\uac1c\uad6c\ub9ac\ub294 \uc778\uac04\uc744 \uc798 \uac81\ub0b4\uc9c0 \uc54a\uc73c\uba70, \uc778\uac04\uacfc \uac00\uae4c\uc6b4 \uacf3\uc5d0 \ub20c\ub7ec\uc549\uc544 \uc0ac\ub294 \uacbd\uc6b0\ub3c4 \ud754\ud558\ub2e4. \uac00\ub054\uc740 \uc544\uc608 \uc9d1 \uc548\uc5d0 \ub4e4\uc5b4\uc640\uc11c \uac1c\uc218\ub300\ub098 \ubcc0\uae30 \uc18d\uc5d0\uc11c \ubc1c\uacac\ub418\uae30\ub3c4 \ud55c\ub2e4. \ub610\ud55c \ubc24\uc774 \ub418\uba74 \ucc3d\ud2c0 \ubc16\uc5d0 \ubd99\uc5b4\uc11c \ubbfc\uac00\uc758 \ubd88\ube5b\uc5d0 \uc720\ud639\ub41c \ub0a0\ubc8c\ub808 \ub4e4\uc744 \uc7a1\uc544\uba39\ub294\ub370, \uc544\ub9c8 \uac19\uc740 \uc774\uc720\ub85c \uc57c\uc678 \uc870\uba85\uc5d0\ub3c4 \ubaa8\uc5ec\ub4dc\ub294 \uac83\uc73c\ub85c \ubcf4\uc778\ub2e4. \uadf8 \uc678\uc5d0\ub3c4 \ubb3c\ud0f1\ud06c, \uc218\uc9c1 \ub099\uc218\ud1b5, \uc218\ud3c9 \ud648\ud1b5 \ub4f1 \ubc14\uae65\ubcf4\ub2e4 \uc11c\ub298\ud558\uace0 \uc2b5\ub3c4\uac00 \ub192\uc740 \uc778\uacf5 \uad6c\uc870\ubb3c \uc18d\uc744 \ucc28\uc9c0\ud558\uace0 \uc0ac\ub294 \uacbd\uc6b0\uac00 \uc788\ub2e4. \ud648\ud1b5\uc774\ub098 \ubb3c\ud0f1\ud06c \uc18d\uc5d0\uc11c \uc6b8\uba74 \uc6b8\uc74c\uc18c\ub9ac\uac00 \uba54\uc544\ub9ac\uccd0 \uc99d\ud3ed\ub418\uae30 \ub54c\ubb38\uc5d0 \ubc88\uc2dd\uae30\uc5d0 \uc774 \ud6a8\uacfc\ub97c \ub178\ub9ac\uace0 \uc5ec\uae30 \uae30\uc5b4\ub4e4\uc5b4\uc640 \uc0ac\ub294 \uac83\uc73c\ub85c \ubcf4\uc778\ub2e4. \uc624\uc2a4\ud2b8\ub808\uc77c\ub9ac\uc544\uccad\uac1c\uad6c\ub9ac\ub294 \uadc0\uc18c\uc131 \ub2a5\ub825\ub3c4 \uac00\uc9c0\uace0 \uc788\ub294 \uac83\uc73c\ub85c \ubcf4\uc778\ub2e4. \ud3ec\ud68d\ud55c \ub4a4 \uba40\ub9ac \ub5a8\uc5b4\uc9c4 \uacf3\uc5d0 \ub0b4\ubc84\ub824\ub3c4 \ucc98\uc74c \uc7a1\ud614\ub358 \uacf3\uc73c\ub85c \ub2e4\uc2dc \ub3cc\uc544\uac08 \uc218 \uc788\ub2e4.", "sentence_answer": "\ub610\ud55c \ubc24\uc774 \ub418\uba74 \ucc3d\ud2c0 \ubc16\uc5d0 \ubd99\uc5b4\uc11c \ubbfc\uac00\uc758 \ubd88\ube5b\uc5d0 \uc720\ud639\ub41c \ub0a0\ubc8c\ub808 \ub4e4\uc744 \uc7a1\uc544\uba39\ub294\ub370, \uc544\ub9c8 \uac19\uc740 \uc774\uc720\ub85c \uc57c\uc678 \uc870\uba85\uc5d0\ub3c4 \ubaa8\uc5ec\ub4dc\ub294 \uac83\uc73c\ub85c \ubcf4\uc778\ub2e4.", "paragraph_id": "6587749-3-0", "paragraph_question": "question: \uc624\uc2a4\ud2b8\ub808\uc77c\ub9ac\uc544\uccad\uac1c\uad6c\ub9ac\uac00 \ubc24\uc5d0 \uc870\uba85\uc73c\ub85c \ubaa8\uc5ec\ub4dc\ub294 \uc774\uc720\ub294 \ubb34\uc5c7\uc744 \uba39\uae30 \uc704\ud568\uc778\uac00?, context: \uc624\uc2a4\ud2b8\ub808\uc77c\ub9ac\uc544\uccad\uac1c\uad6c\ub9ac\ub294 \uc778\uac04\uc744 \uc798 \uac81\ub0b4\uc9c0 \uc54a\uc73c\uba70, \uc778\uac04\uacfc \uac00\uae4c\uc6b4 \uacf3\uc5d0 \ub20c\ub7ec\uc549\uc544 \uc0ac\ub294 \uacbd\uc6b0\ub3c4 \ud754\ud558\ub2e4. \uac00\ub054\uc740 \uc544\uc608 \uc9d1 \uc548\uc5d0 \ub4e4\uc5b4\uc640\uc11c \uac1c\uc218\ub300\ub098 \ubcc0\uae30 \uc18d\uc5d0\uc11c \ubc1c\uacac\ub418\uae30\ub3c4 \ud55c\ub2e4. \ub610\ud55c \ubc24\uc774 \ub418\uba74 \ucc3d\ud2c0 \ubc16\uc5d0 \ubd99\uc5b4\uc11c \ubbfc\uac00\uc758 \ubd88\ube5b\uc5d0 \uc720\ud639\ub41c \ub0a0\ubc8c\ub808\ub4e4\uc744 \uc7a1\uc544\uba39\ub294\ub370, \uc544\ub9c8 \uac19\uc740 \uc774\uc720\ub85c \uc57c\uc678 \uc870\uba85\uc5d0\ub3c4 \ubaa8\uc5ec\ub4dc\ub294 \uac83\uc73c\ub85c \ubcf4\uc778\ub2e4. \uadf8 \uc678\uc5d0\ub3c4 \ubb3c\ud0f1\ud06c, \uc218\uc9c1 \ub099\uc218\ud1b5, \uc218\ud3c9 \ud648\ud1b5 \ub4f1 \ubc14\uae65\ubcf4\ub2e4 \uc11c\ub298\ud558\uace0 \uc2b5\ub3c4\uac00 \ub192\uc740 \uc778\uacf5 \uad6c\uc870\ubb3c \uc18d\uc744 \ucc28\uc9c0\ud558\uace0 \uc0ac\ub294 \uacbd\uc6b0\uac00 \uc788\ub2e4. \ud648\ud1b5\uc774\ub098 \ubb3c\ud0f1\ud06c \uc18d\uc5d0\uc11c \uc6b8\uba74 \uc6b8\uc74c\uc18c\ub9ac\uac00 \uba54\uc544\ub9ac\uccd0 \uc99d\ud3ed\ub418\uae30 \ub54c\ubb38\uc5d0 \ubc88\uc2dd\uae30\uc5d0 \uc774 \ud6a8\uacfc\ub97c \ub178\ub9ac\uace0 \uc5ec\uae30 \uae30\uc5b4\ub4e4\uc5b4\uc640 \uc0ac\ub294 \uac83\uc73c\ub85c \ubcf4\uc778\ub2e4. \uc624\uc2a4\ud2b8\ub808\uc77c\ub9ac\uc544\uccad\uac1c\uad6c\ub9ac\ub294 \uadc0\uc18c\uc131 \ub2a5\ub825\ub3c4 \uac00\uc9c0\uace0 \uc788\ub294 \uac83\uc73c\ub85c \ubcf4\uc778\ub2e4. \ud3ec\ud68d\ud55c \ub4a4 \uba40\ub9ac \ub5a8\uc5b4\uc9c4 \uacf3\uc5d0 \ub0b4\ubc84\ub824\ub3c4 \ucc98\uc74c \uc7a1\ud614\ub358 \uacf3\uc73c\ub85c \ub2e4\uc2dc \ub3cc\uc544\uac08 \uc218 \uc788\ub2e4."} {"question": "\ubca0\ud0c0 \ubd95\uad34\ub97c \ud558\uace0 \ub098\uc11c\ub3c4 \uc544\uc8fc \ub4dc\ubb38 \uacbd\uc6b0\ub85c \uc804\ud558\uac00 \ubcc0\ud558\uc9c0 \uc54a\uc744 \uc218 \uc788\ub294\ub370 \ubb34\uc5c7\uc774\ub77c \ud558\ub294\uac00?", "paragraph": "\uc18c\ub9bd\uc790\ub294 \ubca0\ud0c0 \ubd95\uad34\uc640 \ud568\uaed8 \uadf8 \uc804\ud558\ub97c \ubc14\uafb8\uac8c \ub418\ub294\ub370, \uc544\uc8fc \ub4dc\ubb38 \uacbd\uc6b0\uc774\uc9c0\ub9cc \ubca0\ud0c0 \ubd95\uad34\ub97c \ud558\uace0 \ub098\uc11c\ub3c4 \uc804\ud558\uac00 \ubcc0\ud558\uc9c0 \uc54a\uc744 \uc218 \uc788\ub2e4. \uc774\uac83\uc744 \uc911\uc131 \ubcf4\uc874\ub958\ub77c \ud55c\ub2e4. \ud558\uc9c0\ub9cc \uc5f0\uad6c \uacb0\uacfc \uae30\ubb18\ub3c4\ub97c \uac00\uc9c4 \uc785\uc790\uac00 \ubca0\ud0c0 \ubd95\uad34\ub97c \ud558\uba74 \uc5b8\uc81c\ub098 \uc911\uc131 \ubcf4\uc874\ub958\uac00 \uc5c6\ub2e4\ub294 \ud765\ubbf8\ub85c\uc6b4 \uacb0\ub860\uc774 \ub0b4\ub824\uc84c\ub2e4. \uc774\uac83\uc744 \uc124\uba85\ud558\uae30 \uc704\ud574 \uc178\ub358 \uae00\ub798\uc1fc, \uc774\uc624\uc544\ub2c8\uc2a4 \uc77c\ub9ac\uc624\ud480\ub85c\uc2a4, \ub8e8\ucc28\ub178 \ub9c8\uc774\uc544\ub2c8\ub294 1970\ub144\uc5d0, \ub9f5\uc2dc \ucffc\ud06c(\ud639\uc740 \ucc38\ucffc\ud06c)\ub77c\ub294 \ub610\ub2e4\ub978 \ucffc\ud06c\uc758 \uc874\uc7ac\ub97c \uac00\uc815\ud558\uc5ec \uc774\ub97c \uc124\uba85\ud558\uc600\ub2e4. \uc774\uc5d0 \uc774\ud718\uc18c\ub294 1974\ub144 \uc5ec\ub984\uc5d0 \uba54\ub9ac \uac8c\uc77c\ub7ec\ub4dc(Mary K. Gaillard), \uc870\ub108\uc120 \ub85c\uc988\ub108\uc640 \ud568\uaed8 \u3008\ucc38\ucffc\ud06c\ub97c \ucc3e\uc544\uc11c\u3009\ub77c\ub294 \ub17c\ubb38\uc5d0\uc11c \ucf00\uc774\uc628\uc758 \uc11e\uc784\uacfc \ubd95\uad34\uc5d0 \ud574\ub2f9\ud558\ub294 \uc591\uc744 \uacc4\uc0b0\ud558\uc5ec \ub9f5\uc2dc \ucffc\ud06c\uac00 \uc874\uc7ac\ud55c\ub2e4\uba74 \uadf8\uac00 \uac00\uc9c8 \uc218 \uc788\ub294 \uc9c8\ub7c9 \ubc94\uc704\ub97c \uc608\uce21\ud558\uc600\ub2e4.", "answer": "\uc911\uc131 \ubcf4\uc874\ub958", "sentence": "\uc774\uac83\uc744 \uc911\uc131 \ubcf4\uc874\ub958 \ub77c \ud55c\ub2e4.", "paragraph_sentence": "\uc18c\ub9bd\uc790\ub294 \ubca0\ud0c0 \ubd95\uad34\uc640 \ud568\uaed8 \uadf8 \uc804\ud558\ub97c \ubc14\uafb8\uac8c \ub418\ub294\ub370, \uc544\uc8fc \ub4dc\ubb38 \uacbd\uc6b0\uc774\uc9c0\ub9cc \ubca0\ud0c0 \ubd95\uad34\ub97c \ud558\uace0 \ub098\uc11c\ub3c4 \uc804\ud558\uac00 \ubcc0\ud558\uc9c0 \uc54a\uc744 \uc218 \uc788\ub2e4. \uc774\uac83\uc744 \uc911\uc131 \ubcf4\uc874\ub958 \ub77c \ud55c\ub2e4. \ud558\uc9c0\ub9cc \uc5f0\uad6c \uacb0\uacfc \uae30\ubb18\ub3c4\ub97c \uac00\uc9c4 \uc785\uc790\uac00 \ubca0\ud0c0 \ubd95\uad34\ub97c \ud558\uba74 \uc5b8\uc81c\ub098 \uc911\uc131 \ubcf4\uc874\ub958\uac00 \uc5c6\ub2e4\ub294 \ud765\ubbf8\ub85c\uc6b4 \uacb0\ub860\uc774 \ub0b4\ub824\uc84c\ub2e4. \uc774\uac83\uc744 \uc124\uba85\ud558\uae30 \uc704\ud574 \uc178\ub358 \uae00\ub798\uc1fc, \uc774\uc624\uc544\ub2c8\uc2a4 \uc77c\ub9ac\uc624\ud480\ub85c\uc2a4, \ub8e8\ucc28\ub178 \ub9c8\uc774\uc544\ub2c8\ub294 1970\ub144\uc5d0, \ub9f5\uc2dc \ucffc\ud06c(\ud639\uc740 \ucc38\ucffc\ud06c)\ub77c\ub294 \ub610\ub2e4\ub978 \ucffc\ud06c\uc758 \uc874\uc7ac\ub97c \uac00\uc815\ud558\uc5ec \uc774\ub97c \uc124\uba85\ud558\uc600\ub2e4. \uc774\uc5d0 \uc774\ud718\uc18c\ub294 1974\ub144 \uc5ec\ub984\uc5d0 \uba54\ub9ac \uac8c\uc77c\ub7ec\ub4dc(Mary K. Gaillard), \uc870\ub108\uc120 \ub85c\uc988\ub108\uc640 \ud568\uaed8 \u3008\ucc38\ucffc\ud06c\ub97c \ucc3e\uc544\uc11c\u3009\ub77c\ub294 \ub17c\ubb38\uc5d0\uc11c \ucf00\uc774\uc628\uc758 \uc11e\uc784\uacfc \ubd95\uad34\uc5d0 \ud574\ub2f9\ud558\ub294 \uc591\uc744 \uacc4\uc0b0\ud558\uc5ec \ub9f5\uc2dc \ucffc\ud06c\uac00 \uc874\uc7ac\ud55c\ub2e4\uba74 \uadf8\uac00 \uac00\uc9c8 \uc218 \uc788\ub294 \uc9c8\ub7c9 \ubc94\uc704\ub97c \uc608\uce21\ud558\uc600\ub2e4.", "paragraph_answer": "\uc18c\ub9bd\uc790\ub294 \ubca0\ud0c0 \ubd95\uad34\uc640 \ud568\uaed8 \uadf8 \uc804\ud558\ub97c \ubc14\uafb8\uac8c \ub418\ub294\ub370, \uc544\uc8fc \ub4dc\ubb38 \uacbd\uc6b0\uc774\uc9c0\ub9cc \ubca0\ud0c0 \ubd95\uad34\ub97c \ud558\uace0 \ub098\uc11c\ub3c4 \uc804\ud558\uac00 \ubcc0\ud558\uc9c0 \uc54a\uc744 \uc218 \uc788\ub2e4. \uc774\uac83\uc744 \uc911\uc131 \ubcf4\uc874\ub958 \ub77c \ud55c\ub2e4. \ud558\uc9c0\ub9cc \uc5f0\uad6c \uacb0\uacfc \uae30\ubb18\ub3c4\ub97c \uac00\uc9c4 \uc785\uc790\uac00 \ubca0\ud0c0 \ubd95\uad34\ub97c \ud558\uba74 \uc5b8\uc81c\ub098 \uc911\uc131 \ubcf4\uc874\ub958\uac00 \uc5c6\ub2e4\ub294 \ud765\ubbf8\ub85c\uc6b4 \uacb0\ub860\uc774 \ub0b4\ub824\uc84c\ub2e4. \uc774\uac83\uc744 \uc124\uba85\ud558\uae30 \uc704\ud574 \uc178\ub358 \uae00\ub798\uc1fc, \uc774\uc624\uc544\ub2c8\uc2a4 \uc77c\ub9ac\uc624\ud480\ub85c\uc2a4, \ub8e8\ucc28\ub178 \ub9c8\uc774\uc544\ub2c8\ub294 1970\ub144\uc5d0, \ub9f5\uc2dc \ucffc\ud06c(\ud639\uc740 \ucc38\ucffc\ud06c)\ub77c\ub294 \ub610\ub2e4\ub978 \ucffc\ud06c\uc758 \uc874\uc7ac\ub97c \uac00\uc815\ud558\uc5ec \uc774\ub97c \uc124\uba85\ud558\uc600\ub2e4. \uc774\uc5d0 \uc774\ud718\uc18c\ub294 1974\ub144 \uc5ec\ub984\uc5d0 \uba54\ub9ac \uac8c\uc77c\ub7ec\ub4dc(Mary K. Gaillard), \uc870\ub108\uc120 \ub85c\uc988\ub108\uc640 \ud568\uaed8 \u3008\ucc38\ucffc\ud06c\ub97c \ucc3e\uc544\uc11c\u3009\ub77c\ub294 \ub17c\ubb38\uc5d0\uc11c \ucf00\uc774\uc628\uc758 \uc11e\uc784\uacfc \ubd95\uad34\uc5d0 \ud574\ub2f9\ud558\ub294 \uc591\uc744 \uacc4\uc0b0\ud558\uc5ec \ub9f5\uc2dc \ucffc\ud06c\uac00 \uc874\uc7ac\ud55c\ub2e4\uba74 \uadf8\uac00 \uac00\uc9c8 \uc218 \uc788\ub294 \uc9c8\ub7c9 \ubc94\uc704\ub97c \uc608\uce21\ud558\uc600\ub2e4.", "sentence_answer": "\uc774\uac83\uc744 \uc911\uc131 \ubcf4\uc874\ub958 \ub77c \ud55c\ub2e4.", "paragraph_id": "6547992-18-1", "paragraph_question": "question: \ubca0\ud0c0 \ubd95\uad34\ub97c \ud558\uace0 \ub098\uc11c\ub3c4 \uc544\uc8fc \ub4dc\ubb38 \uacbd\uc6b0\ub85c \uc804\ud558\uac00 \ubcc0\ud558\uc9c0 \uc54a\uc744 \uc218 \uc788\ub294\ub370 \ubb34\uc5c7\uc774\ub77c \ud558\ub294\uac00?, context: \uc18c\ub9bd\uc790\ub294 \ubca0\ud0c0 \ubd95\uad34\uc640 \ud568\uaed8 \uadf8 \uc804\ud558\ub97c \ubc14\uafb8\uac8c \ub418\ub294\ub370, \uc544\uc8fc \ub4dc\ubb38 \uacbd\uc6b0\uc774\uc9c0\ub9cc \ubca0\ud0c0 \ubd95\uad34\ub97c \ud558\uace0 \ub098\uc11c\ub3c4 \uc804\ud558\uac00 \ubcc0\ud558\uc9c0 \uc54a\uc744 \uc218 \uc788\ub2e4. \uc774\uac83\uc744 \uc911\uc131 \ubcf4\uc874\ub958\ub77c \ud55c\ub2e4. \ud558\uc9c0\ub9cc \uc5f0\uad6c \uacb0\uacfc \uae30\ubb18\ub3c4\ub97c \uac00\uc9c4 \uc785\uc790\uac00 \ubca0\ud0c0 \ubd95\uad34\ub97c \ud558\uba74 \uc5b8\uc81c\ub098 \uc911\uc131 \ubcf4\uc874\ub958\uac00 \uc5c6\ub2e4\ub294 \ud765\ubbf8\ub85c\uc6b4 \uacb0\ub860\uc774 \ub0b4\ub824\uc84c\ub2e4. \uc774\uac83\uc744 \uc124\uba85\ud558\uae30 \uc704\ud574 \uc178\ub358 \uae00\ub798\uc1fc, \uc774\uc624\uc544\ub2c8\uc2a4 \uc77c\ub9ac\uc624\ud480\ub85c\uc2a4, \ub8e8\ucc28\ub178 \ub9c8\uc774\uc544\ub2c8\ub294 1970\ub144\uc5d0, \ub9f5\uc2dc \ucffc\ud06c(\ud639\uc740 \ucc38\ucffc\ud06c)\ub77c\ub294 \ub610\ub2e4\ub978 \ucffc\ud06c\uc758 \uc874\uc7ac\ub97c \uac00\uc815\ud558\uc5ec \uc774\ub97c \uc124\uba85\ud558\uc600\ub2e4. \uc774\uc5d0 \uc774\ud718\uc18c\ub294 1974\ub144 \uc5ec\ub984\uc5d0 \uba54\ub9ac \uac8c\uc77c\ub7ec\ub4dc(Mary K. Gaillard), \uc870\ub108\uc120 \ub85c\uc988\ub108\uc640 \ud568\uaed8 \u3008\ucc38\ucffc\ud06c\ub97c \ucc3e\uc544\uc11c\u3009\ub77c\ub294 \ub17c\ubb38\uc5d0\uc11c \ucf00\uc774\uc628\uc758 \uc11e\uc784\uacfc \ubd95\uad34\uc5d0 \ud574\ub2f9\ud558\ub294 \uc591\uc744 \uacc4\uc0b0\ud558\uc5ec \ub9f5\uc2dc \ucffc\ud06c\uac00 \uc874\uc7ac\ud55c\ub2e4\uba74 \uadf8\uac00 \uac00\uc9c8 \uc218 \uc788\ub294 \uc9c8\ub7c9 \ubc94\uc704\ub97c \uc608\uce21\ud558\uc600\ub2e4."} {"question": "\ubbf8\uad6d\uc5d0\uc11c \uc8fc\ub85c \uc911\ubd80 \ucabd \ub300\ud559\uc5d0 \uc9c4\ud559\ud558\uae30 \uc704\ud574\uc11c \uce58\ub7ec\uc57c \ud558\ub294 \uac83\uc740?", "paragraph": "\ubbf8\uad6d\uc758 \uad50\uc721\uc740 \ucd08\uae30 \uc2dd\ubbfc\uc9c0 \uc2dc\uc808\ubd80\ud130 \uc911\uc694\uc2dc\ub418\uc5b4\uc654\ub294\ub370, \uace0\ub4f1\uad50\uc721\uae30\uad00\uc758 \ubc1c\uc804\uc740 \uc804\uc7c1\uacfc \uacfc\ud559 \uc5f0\uad6c \ub4f1\uc5d0 \uc788\uc5b4 \ubbf8\uad6d\uc758 \uc5ed\uc0ac\uc640 \ud568\uaed8\ud574\uc654\ub2e4. \ucd08\uae30\uc5d0\uc11c\ubd80\ud130 \ud604\uc7ac\uae4c\uc9c0 \uad50\uc721\uc5d0 \uc788\uc5b4 \uc885\uad50\uc758 \uc601\ud5a5\uc740 \ub9e4\uc6b0 \ud06c\uba70, \uc5d8\ub9ac\ud2b8\ub4e4\uc758 \uad6d\uac00 \uacbd\uc601\uc774 \uc7a5\ub824\ub418\ub294 \uc0ac\ud68c\uc5ec\uc11c, \uc0ac\ud559\uc774 \ubc1c\ub2ec\ud588\ub2e4. \ud06c\uac8c \uc0ac\ub9bd\uacfc \uc8fc\ub9bd \ud639\uc740 \uad6d\uacf5\ub9bd \uad50\uc721\uae30\uad00\uc73c\ub85c \ub098\ub258\uba70, \ub300\ubd80\ubd84\uc758 \uc8fc\uc5d0\uc11c\ub294 6\uc138\uc5d0\uc11c 16\uc138\uae4c\uc9c0 \ubb34\uc0c1\u00b7\uc758\ubb34 \uad50\uc721\uc744 \uc2e4\uc2dc\ud55c\ub2e4. \ubbf8\uad6d \ud559\uc0dd\ub4e4\uc758 \uc808\ub300 \ub2e4\uc218\uac00 \uc911\ub4f1\uad50\uc721\uc744 \ub9c8\uce58\ub294 17, 18\uc138 (K-12 \ud559\uc81c \uc0c1 \uace0\ub4f1\ud559\uad50 \uc878\uc5c5\ubc18)\uae4c\uc9c0 \ud559\uad50\uc5d0 \ub2e4\ub2cc\ub2e4. \ubd80\uc790\ub4e4\uc740 \ub300\uccb4\ub85c \uc0ac\ub9bd \ud559\uad50\uc5d0 \ub2e4\ub2cc\ub2e4. \uc2e4\uc6a9\uc801\uc778 \uad50\uc721 \ucca0\ud559\uc740 \uad50\uc721\uc758 \ub9c8\uc9c0\ub9c9 \uae30\uac04\uc778 \ub300\ud559\uad50\uc640 \ub300\ud559\uc6d0\uc758 \uc6b0\uc218\uc131\uc5d0\uc11c \uc54c \uc218 \uc788\ub294\ub370, \ud2b9\ud788 \ub300\ud559\uad50\uc640 \ub300\ud559\uc6d0 \ub4f1 \uace0\ub4f1\uad50\uc721\uc740 \uadf8 \uba85\uc131\uacfc \ud559\uc5f4, \ud559\uc0dd \uc218\uc900, \uadf8\ub9ac\uace0 \uc5f0\uad6c \uc2e4\uc801\uc5d0\uc11c \uc138\uacc4 \uc5ec\ub290 \ub098\ub77c\uc758 \uace0\ub4f1\uad50\uc721\uae30\uad00\uc744 \uc555\ub3c4\ud55c\ub2e4. \ubbf8\uad6d\uc5d0\uc11c \ub300\ud559\uc5d0 \uc9c4\ud559\ud558\ub824\uba74 ACT(\uc8fc\ub85c \uc911\ubd80 \ucabd \ub300\ud559)\ub098 SAT(\uc8fc\ub85c \ub3d9\ubd80, \uc11c\ubd80 \ucabd \ub300\ud559)\ub97c \uce58\ub7ec\uc57c \ud55c\ub2e4. \ub2e4\ub978 \uc720\ub7fd\uc758 \uad6d\uac00\ub4e4\ucc98\ub7fc \ubbf8\uad6d\ub3c4 \uc911\ub4f1 \uad50\uc721 \ub2e8\uacc4\ubd80\ud130 \ud559\uc810\uc81c\ub97c \ucc44\ud0dd\ud55c\ub2e4. \uad50\uc721\uc5d0\uc11c\ub294 \uc601\uc5b4\ub97c \uc0ac\uc6a9\ud558\uace0, \uc678\uad6d\uc5b4\ub85c\ub294 \ub3c5\uc77c\uc5b4, \ud504\ub791\uc2a4\uc5b4, \uc2a4\ud398\uc778\uc5b4, \ub77c\ud2f4\uc5b4, \uadf8\ub9ac\uc2a4\uc5b4, \ud788\ube0c\ub9ac\uc5b4, \uc774\ud0c8\ub9ac\uc544\uc5b4, \uc911\uad6d\uc5b4, \uc77c\ubcf8\uc5b4, \ud55c\uad6d\uc5b4 \uc911 \ud558\ub098\ub97c \uc120\ud0dd\ud55c\ub2e4.", "answer": "ACT", "sentence": "\ubbf8\uad6d\uc5d0\uc11c \ub300\ud559\uc5d0 \uc9c4\ud559\ud558\ub824\uba74 ACT (\uc8fc\ub85c \uc911\ubd80 \ucabd \ub300\ud559)\ub098 SAT(\uc8fc\ub85c \ub3d9\ubd80, \uc11c\ubd80 \ucabd \ub300\ud559)\ub97c \uce58\ub7ec\uc57c \ud55c\ub2e4.", "paragraph_sentence": "\ubbf8\uad6d\uc758 \uad50\uc721\uc740 \ucd08\uae30 \uc2dd\ubbfc\uc9c0 \uc2dc\uc808\ubd80\ud130 \uc911\uc694\uc2dc\ub418\uc5b4\uc654\ub294\ub370, \uace0\ub4f1\uad50\uc721\uae30\uad00\uc758 \ubc1c\uc804\uc740 \uc804\uc7c1\uacfc \uacfc\ud559 \uc5f0\uad6c \ub4f1\uc5d0 \uc788\uc5b4 \ubbf8\uad6d\uc758 \uc5ed\uc0ac\uc640 \ud568\uaed8\ud574\uc654\ub2e4. \ucd08\uae30\uc5d0\uc11c\ubd80\ud130 \ud604\uc7ac\uae4c\uc9c0 \uad50\uc721\uc5d0 \uc788\uc5b4 \uc885\uad50\uc758 \uc601\ud5a5\uc740 \ub9e4\uc6b0 \ud06c\uba70, \uc5d8\ub9ac\ud2b8\ub4e4\uc758 \uad6d\uac00 \uacbd\uc601\uc774 \uc7a5\ub824\ub418\ub294 \uc0ac\ud68c\uc5ec\uc11c, \uc0ac\ud559\uc774 \ubc1c\ub2ec\ud588\ub2e4. \ud06c\uac8c \uc0ac\ub9bd\uacfc \uc8fc\ub9bd \ud639\uc740 \uad6d\uacf5\ub9bd \uad50\uc721\uae30\uad00\uc73c\ub85c \ub098\ub258\uba70, \ub300\ubd80\ubd84\uc758 \uc8fc\uc5d0\uc11c\ub294 6\uc138\uc5d0\uc11c 16\uc138\uae4c\uc9c0 \ubb34\uc0c1\u00b7\uc758\ubb34 \uad50\uc721\uc744 \uc2e4\uc2dc\ud55c\ub2e4. \ubbf8\uad6d \ud559\uc0dd\ub4e4\uc758 \uc808\ub300 \ub2e4\uc218\uac00 \uc911\ub4f1\uad50\uc721\uc744 \ub9c8\uce58\ub294 17, 18\uc138 (K-12 \ud559\uc81c \uc0c1 \uace0\ub4f1\ud559\uad50 \uc878\uc5c5\ubc18)\uae4c\uc9c0 \ud559\uad50\uc5d0 \ub2e4\ub2cc\ub2e4. \ubd80\uc790\ub4e4\uc740 \ub300\uccb4\ub85c \uc0ac\ub9bd \ud559\uad50\uc5d0 \ub2e4\ub2cc\ub2e4. \uc2e4\uc6a9\uc801\uc778 \uad50\uc721 \ucca0\ud559\uc740 \uad50\uc721\uc758 \ub9c8\uc9c0\ub9c9 \uae30\uac04\uc778 \ub300\ud559\uad50\uc640 \ub300\ud559\uc6d0\uc758 \uc6b0\uc218\uc131\uc5d0\uc11c \uc54c \uc218 \uc788\ub294\ub370, \ud2b9\ud788 \ub300\ud559\uad50\uc640 \ub300\ud559\uc6d0 \ub4f1 \uace0\ub4f1\uad50\uc721\uc740 \uadf8 \uba85\uc131\uacfc \ud559\uc5f4, \ud559\uc0dd \uc218\uc900, \uadf8\ub9ac\uace0 \uc5f0\uad6c \uc2e4\uc801\uc5d0\uc11c \uc138\uacc4 \uc5ec\ub290 \ub098\ub77c\uc758 \uace0\ub4f1\uad50\uc721\uae30\uad00\uc744 \uc555\ub3c4\ud55c\ub2e4. \ubbf8\uad6d\uc5d0\uc11c \ub300\ud559\uc5d0 \uc9c4\ud559\ud558\ub824\uba74 ACT (\uc8fc\ub85c \uc911\ubd80 \ucabd \ub300\ud559)\ub098 SAT(\uc8fc\ub85c \ub3d9\ubd80, \uc11c\ubd80 \ucabd \ub300\ud559)\ub97c \uce58\ub7ec\uc57c \ud55c\ub2e4. \ub2e4\ub978 \uc720\ub7fd\uc758 \uad6d\uac00\ub4e4\ucc98\ub7fc \ubbf8\uad6d\ub3c4 \uc911\ub4f1 \uad50\uc721 \ub2e8\uacc4\ubd80\ud130 \ud559\uc810\uc81c\ub97c \ucc44\ud0dd\ud55c\ub2e4. \uad50\uc721\uc5d0\uc11c\ub294 \uc601\uc5b4\ub97c \uc0ac\uc6a9\ud558\uace0, \uc678\uad6d\uc5b4\ub85c\ub294 \ub3c5\uc77c\uc5b4, \ud504\ub791\uc2a4\uc5b4, \uc2a4\ud398\uc778\uc5b4, \ub77c\ud2f4\uc5b4, \uadf8\ub9ac\uc2a4\uc5b4, \ud788\ube0c\ub9ac\uc5b4, \uc774\ud0c8\ub9ac\uc544\uc5b4, \uc911\uad6d\uc5b4, \uc77c\ubcf8\uc5b4, \ud55c\uad6d\uc5b4 \uc911 \ud558\ub098\ub97c \uc120\ud0dd\ud55c\ub2e4.", "paragraph_answer": "\ubbf8\uad6d\uc758 \uad50\uc721\uc740 \ucd08\uae30 \uc2dd\ubbfc\uc9c0 \uc2dc\uc808\ubd80\ud130 \uc911\uc694\uc2dc\ub418\uc5b4\uc654\ub294\ub370, \uace0\ub4f1\uad50\uc721\uae30\uad00\uc758 \ubc1c\uc804\uc740 \uc804\uc7c1\uacfc \uacfc\ud559 \uc5f0\uad6c \ub4f1\uc5d0 \uc788\uc5b4 \ubbf8\uad6d\uc758 \uc5ed\uc0ac\uc640 \ud568\uaed8\ud574\uc654\ub2e4. \ucd08\uae30\uc5d0\uc11c\ubd80\ud130 \ud604\uc7ac\uae4c\uc9c0 \uad50\uc721\uc5d0 \uc788\uc5b4 \uc885\uad50\uc758 \uc601\ud5a5\uc740 \ub9e4\uc6b0 \ud06c\uba70, \uc5d8\ub9ac\ud2b8\ub4e4\uc758 \uad6d\uac00 \uacbd\uc601\uc774 \uc7a5\ub824\ub418\ub294 \uc0ac\ud68c\uc5ec\uc11c, \uc0ac\ud559\uc774 \ubc1c\ub2ec\ud588\ub2e4. \ud06c\uac8c \uc0ac\ub9bd\uacfc \uc8fc\ub9bd \ud639\uc740 \uad6d\uacf5\ub9bd \uad50\uc721\uae30\uad00\uc73c\ub85c \ub098\ub258\uba70, \ub300\ubd80\ubd84\uc758 \uc8fc\uc5d0\uc11c\ub294 6\uc138\uc5d0\uc11c 16\uc138\uae4c\uc9c0 \ubb34\uc0c1\u00b7\uc758\ubb34 \uad50\uc721\uc744 \uc2e4\uc2dc\ud55c\ub2e4. \ubbf8\uad6d \ud559\uc0dd\ub4e4\uc758 \uc808\ub300 \ub2e4\uc218\uac00 \uc911\ub4f1\uad50\uc721\uc744 \ub9c8\uce58\ub294 17, 18\uc138 (K-12 \ud559\uc81c \uc0c1 \uace0\ub4f1\ud559\uad50 \uc878\uc5c5\ubc18)\uae4c\uc9c0 \ud559\uad50\uc5d0 \ub2e4\ub2cc\ub2e4. \ubd80\uc790\ub4e4\uc740 \ub300\uccb4\ub85c \uc0ac\ub9bd \ud559\uad50\uc5d0 \ub2e4\ub2cc\ub2e4. \uc2e4\uc6a9\uc801\uc778 \uad50\uc721 \ucca0\ud559\uc740 \uad50\uc721\uc758 \ub9c8\uc9c0\ub9c9 \uae30\uac04\uc778 \ub300\ud559\uad50\uc640 \ub300\ud559\uc6d0\uc758 \uc6b0\uc218\uc131\uc5d0\uc11c \uc54c \uc218 \uc788\ub294\ub370, \ud2b9\ud788 \ub300\ud559\uad50\uc640 \ub300\ud559\uc6d0 \ub4f1 \uace0\ub4f1\uad50\uc721\uc740 \uadf8 \uba85\uc131\uacfc \ud559\uc5f4, \ud559\uc0dd \uc218\uc900, \uadf8\ub9ac\uace0 \uc5f0\uad6c \uc2e4\uc801\uc5d0\uc11c \uc138\uacc4 \uc5ec\ub290 \ub098\ub77c\uc758 \uace0\ub4f1\uad50\uc721\uae30\uad00\uc744 \uc555\ub3c4\ud55c\ub2e4. \ubbf8\uad6d\uc5d0\uc11c \ub300\ud559\uc5d0 \uc9c4\ud559\ud558\ub824\uba74 ACT (\uc8fc\ub85c \uc911\ubd80 \ucabd \ub300\ud559)\ub098 SAT(\uc8fc\ub85c \ub3d9\ubd80, \uc11c\ubd80 \ucabd \ub300\ud559)\ub97c \uce58\ub7ec\uc57c \ud55c\ub2e4. \ub2e4\ub978 \uc720\ub7fd\uc758 \uad6d\uac00\ub4e4\ucc98\ub7fc \ubbf8\uad6d\ub3c4 \uc911\ub4f1 \uad50\uc721 \ub2e8\uacc4\ubd80\ud130 \ud559\uc810\uc81c\ub97c \ucc44\ud0dd\ud55c\ub2e4. \uad50\uc721\uc5d0\uc11c\ub294 \uc601\uc5b4\ub97c \uc0ac\uc6a9\ud558\uace0, \uc678\uad6d\uc5b4\ub85c\ub294 \ub3c5\uc77c\uc5b4, \ud504\ub791\uc2a4\uc5b4, \uc2a4\ud398\uc778\uc5b4, \ub77c\ud2f4\uc5b4, \uadf8\ub9ac\uc2a4\uc5b4, \ud788\ube0c\ub9ac\uc5b4, \uc774\ud0c8\ub9ac\uc544\uc5b4, \uc911\uad6d\uc5b4, \uc77c\ubcf8\uc5b4, \ud55c\uad6d\uc5b4 \uc911 \ud558\ub098\ub97c \uc120\ud0dd\ud55c\ub2e4.", "sentence_answer": "\ubbf8\uad6d\uc5d0\uc11c \ub300\ud559\uc5d0 \uc9c4\ud559\ud558\ub824\uba74 ACT (\uc8fc\ub85c \uc911\ubd80 \ucabd \ub300\ud559)\ub098 SAT(\uc8fc\ub85c \ub3d9\ubd80, \uc11c\ubd80 \ucabd \ub300\ud559)\ub97c \uce58\ub7ec\uc57c \ud55c\ub2e4.", "paragraph_id": "6514860-32-1", "paragraph_question": "question: \ubbf8\uad6d\uc5d0\uc11c \uc8fc\ub85c \uc911\ubd80 \ucabd \ub300\ud559\uc5d0 \uc9c4\ud559\ud558\uae30 \uc704\ud574\uc11c \uce58\ub7ec\uc57c \ud558\ub294 \uac83\uc740?, context: \ubbf8\uad6d\uc758 \uad50\uc721\uc740 \ucd08\uae30 \uc2dd\ubbfc\uc9c0 \uc2dc\uc808\ubd80\ud130 \uc911\uc694\uc2dc\ub418\uc5b4\uc654\ub294\ub370, \uace0\ub4f1\uad50\uc721\uae30\uad00\uc758 \ubc1c\uc804\uc740 \uc804\uc7c1\uacfc \uacfc\ud559 \uc5f0\uad6c \ub4f1\uc5d0 \uc788\uc5b4 \ubbf8\uad6d\uc758 \uc5ed\uc0ac\uc640 \ud568\uaed8\ud574\uc654\ub2e4. \ucd08\uae30\uc5d0\uc11c\ubd80\ud130 \ud604\uc7ac\uae4c\uc9c0 \uad50\uc721\uc5d0 \uc788\uc5b4 \uc885\uad50\uc758 \uc601\ud5a5\uc740 \ub9e4\uc6b0 \ud06c\uba70, \uc5d8\ub9ac\ud2b8\ub4e4\uc758 \uad6d\uac00 \uacbd\uc601\uc774 \uc7a5\ub824\ub418\ub294 \uc0ac\ud68c\uc5ec\uc11c, \uc0ac\ud559\uc774 \ubc1c\ub2ec\ud588\ub2e4. \ud06c\uac8c \uc0ac\ub9bd\uacfc \uc8fc\ub9bd \ud639\uc740 \uad6d\uacf5\ub9bd \uad50\uc721\uae30\uad00\uc73c\ub85c \ub098\ub258\uba70, \ub300\ubd80\ubd84\uc758 \uc8fc\uc5d0\uc11c\ub294 6\uc138\uc5d0\uc11c 16\uc138\uae4c\uc9c0 \ubb34\uc0c1\u00b7\uc758\ubb34 \uad50\uc721\uc744 \uc2e4\uc2dc\ud55c\ub2e4. \ubbf8\uad6d \ud559\uc0dd\ub4e4\uc758 \uc808\ub300 \ub2e4\uc218\uac00 \uc911\ub4f1\uad50\uc721\uc744 \ub9c8\uce58\ub294 17, 18\uc138 (K-12 \ud559\uc81c \uc0c1 \uace0\ub4f1\ud559\uad50 \uc878\uc5c5\ubc18)\uae4c\uc9c0 \ud559\uad50\uc5d0 \ub2e4\ub2cc\ub2e4. \ubd80\uc790\ub4e4\uc740 \ub300\uccb4\ub85c \uc0ac\ub9bd \ud559\uad50\uc5d0 \ub2e4\ub2cc\ub2e4. \uc2e4\uc6a9\uc801\uc778 \uad50\uc721 \ucca0\ud559\uc740 \uad50\uc721\uc758 \ub9c8\uc9c0\ub9c9 \uae30\uac04\uc778 \ub300\ud559\uad50\uc640 \ub300\ud559\uc6d0\uc758 \uc6b0\uc218\uc131\uc5d0\uc11c \uc54c \uc218 \uc788\ub294\ub370, \ud2b9\ud788 \ub300\ud559\uad50\uc640 \ub300\ud559\uc6d0 \ub4f1 \uace0\ub4f1\uad50\uc721\uc740 \uadf8 \uba85\uc131\uacfc \ud559\uc5f4, \ud559\uc0dd \uc218\uc900, \uadf8\ub9ac\uace0 \uc5f0\uad6c \uc2e4\uc801\uc5d0\uc11c \uc138\uacc4 \uc5ec\ub290 \ub098\ub77c\uc758 \uace0\ub4f1\uad50\uc721\uae30\uad00\uc744 \uc555\ub3c4\ud55c\ub2e4. \ubbf8\uad6d\uc5d0\uc11c \ub300\ud559\uc5d0 \uc9c4\ud559\ud558\ub824\uba74 ACT(\uc8fc\ub85c \uc911\ubd80 \ucabd \ub300\ud559)\ub098 SAT(\uc8fc\ub85c \ub3d9\ubd80, \uc11c\ubd80 \ucabd \ub300\ud559)\ub97c \uce58\ub7ec\uc57c \ud55c\ub2e4. \ub2e4\ub978 \uc720\ub7fd\uc758 \uad6d\uac00\ub4e4\ucc98\ub7fc \ubbf8\uad6d\ub3c4 \uc911\ub4f1 \uad50\uc721 \ub2e8\uacc4\ubd80\ud130 \ud559\uc810\uc81c\ub97c \ucc44\ud0dd\ud55c\ub2e4. \uad50\uc721\uc5d0\uc11c\ub294 \uc601\uc5b4\ub97c \uc0ac\uc6a9\ud558\uace0, \uc678\uad6d\uc5b4\ub85c\ub294 \ub3c5\uc77c\uc5b4, \ud504\ub791\uc2a4\uc5b4, \uc2a4\ud398\uc778\uc5b4, \ub77c\ud2f4\uc5b4, \uadf8\ub9ac\uc2a4\uc5b4, \ud788\ube0c\ub9ac\uc5b4, \uc774\ud0c8\ub9ac\uc544\uc5b4, \uc911\uad6d\uc5b4, \uc77c\ubcf8\uc5b4, \ud55c\uad6d\uc5b4 \uc911 \ud558\ub098\ub97c \uc120\ud0dd\ud55c\ub2e4."} {"question": "\ube0c\uc774 \ud3ec \ubca4\ub370\ud0c0\uc640 1984 \uc0ac\uc774\uc758 \uc5f0\uad00\uc131\uc744 \ubc1c\uacac\ud560 \uc218 \uc788\ub294 \ub3d9\ubb3c\uc740?", "paragraph": "\uc790\uc720\uc640 \uad8c\ub825 \uc0ac\uc774\uc758 \uc2f8\uc6c0\uc5d0 \ub300\ud55c \uc601\ud654\ub85c\uc11c \u300a\ube0c\uc774 \ud3ec \ubca4\ub370\ud0c0\u300b\ub294 \uc544\ub3cc\ud504 \ud788\ud2c0\ub7ec \uce58\ud558\uc758 \uc81c3\uc81c\uad6d\uacfc \uc870\uc9c0 \uc624\uc6f0\uc758 \u300a1984\ub144\u300b\ub4f1 \uace0\uc804\uc801\uc778 \uc804\uccb4\uc8fc\uc758 \uc544\uc774\ucf58\ub4e4\uc758 \uc774\ubbf8\uc9c0\ub97c \ub530\uc654\ub2e4. \uadf8 \uc608\ub85c, \uc11c\ud2c0\ub7ec \ub294 \uc8fc\ub85c \uac70\ub300\ud55c \ube44\ub514\uc624 \uc2a4\ud06c\ub9b0\uc5d0 \ub098\ud0c0\ub098\uba70, \uc9d1\uc9d1\ub9c8\ub2e4 \uc11c\ud2c0\ub7ec\uc758 \uc0ac\uc9c4\uc774 \uac78\ub824 \uc788\ub2e4. \uc774\uac83\uc740 \ube45 \ube0c\ub77c\ub354\ub97c \ub5a0\uc62c\ub9ac\uac8c \ud55c\ub2e4. \ub3c5\uc7ac \uc815\ubd80\uc758 \uc2ac\ub85c\uac74 '\ud1b5\ud569\uc744 \ud1b5\ud574 \ud798\uc744, \ubbff\uc74c\uc744 \ud1b5\ud574 \ud1b5\ud569\uc744' \ub3c4 \uc5b5\uc591\uc0c1 \u300a1984\ub144\u300b\uc5d0 \ub098\uc624\ub294 \uad6c\ud638 \"\uc804\uc7c1\uc740 \ud3c9\ud654. \uc790\uc720\ub294 \uad74\uc885. \ubb34\uc2dd\uc740 \ud798\" \uacfc \ube44\uc2b7\ud558\ub2e4. \ub450 \uc791\ud488\uc758 \uc5f0\uad00\uc131\uc740 \uc774\ube44\uac00 \uac10\uae08\ub418\uc5c8\uc744 \ub54c \uac10\ubc29\uc5d0\uc11c \uc950\ub97c \ubc1c\uacac\ud558\ub294 \uac83\uc5d0\uc11c\ub3c4 \ucc3e\uc744 \uc218 \uc788\ub2e4. \u300a1984\ub144\u300b\uc758 \uc8fc\uc778\uacf5 \uc708\uc2a4\ud134 \uc2a4\ubbf8\uc2a4\ub294 \uc950 \uacf5\ud3ec\uc99d\uc774 \uc788\ub2e4. \ub610\ud55c \uad8c\ub825\uc790\uac00 \ud3d0\uc1c4 \ud68c\ub85c \ud154\ub808\ube44\uc804 \uac19\uc740 \ub300\uc911 \uac10\uc2dc\ub97c \uc0ac\uc6a9\ud558\ub294 \uac83\ub3c4 \uac19\ub2e4. \ubc1c\ub808\ub9ac\ub294 \ub808\uc988\ube44\uc5b8 \uad6c\ub958 \uc2dc\uc124\ub85c \ubcf4\ub0b4\uc838 \uc0dd\uccb4 \uc2e4\ud5d8\uc744 \ub2f9\ud55c\ub2e4. \uc774\uac83\uc740 \ud640\ub85c\ucf54\uc2a4\ud2b8 \ub2f9\uc2dc \ub098\uce58\uc758 \ub3d9\uc131\uc560\uc790\uc5d0 \ub300\ud55c \ucc98\uc6b0 \ubc0f \uc77c\ubcf8\uc758 731 \ubd80\ub300\uc758 \ub9cc\ud589\uacfc \uc77c\ub9e5\uc0c1\ud1b5\ud55c\ub2e4. \uc544\ub2f4 \uc11c\ud2c0\ub7ec\ub77c\ub294 \uc774\ub984\uc740 \uc544\ub3cc\ud504 \ud788\ud2c0\ub7ec\uc5d0\uc11c \ub530\uc628 \uac83\uc774\ub2e4. \uc11c\ud2c0\ub7ec\uc758 \ud788\uc2a4\ud14c\ub9ad\ud55c \uc5f0\uc124\ub3c4 \ud788\ud2c0\ub7ec\uc758 \uc5f0\uc124 \uc2a4\ud0c0\uc77c\uacfc \ube44\uc2b7\ud558\ub2e4. \ub178\uc2a4\ud30c\uc774\uc5b4\ub294 \uc601\uad6d\uc758 \uc0c1\uc9d5\uc744 \uc131\uc870\uc9c0 \uc2ed\uc790\uac00\uc5d0\uc11c \ub85c\ub80c \uc2ed\uc790\uac00\ub85c \ubc14\uafb8\uc5b4 \ub193\uc558\ub2e4. \ub85c\ub808\uc778 \uc2ed\uc790\uac00\ub294 2\ucc28 \ub300\uc804 \uc911 \uc790\uc720 \ud504\ub791\uc2a4\uc758 \uc0c1\uc9d5\uc73c\ub85c, \ub098\uce58\uc758 \ud558\ucf04\ud06c\ub85c\uc774\uce20\uc5d0 \ub300\uc751\ud558\ub294, \ud504\ub791\uc2a4 \uc560\uad6d\uc2ec\uc758 \uc0c1\uc9d5\uc774\uc5c8\ub2e4. \ub9e4\uc2a4 \ubbf8\ub514\uc5b4\ub294 \uc815\ubd80\uc758 \ud504\ub85c\ud30c\uac04\ub2e4\uc758 \uc2dc\uc885 \ub178\ub987\uc744 \ud558\uace0 \uc788\ub2e4. \uc774\uac83\uc740 \uc804\uccb4\uc8fc\uc758 \uc815\uad8c\ud558\uba74 \ub5a0\uc62c\ub824\uc9c0\ub294 \ub300\uc911\uc801 \uc774\ubbf8\uc9c0\uc774\ub2e4.", "answer": "\uc950", "sentence": "\ub450 \uc791\ud488\uc758 \uc5f0\uad00\uc131\uc740 \uc774\ube44\uac00 \uac10\uae08\ub418\uc5c8\uc744 \ub54c \uac10\ubc29\uc5d0\uc11c \uc950 \ub97c \ubc1c\uacac\ud558\ub294 \uac83\uc5d0\uc11c\ub3c4 \ucc3e\uc744 \uc218 \uc788\ub2e4.", "paragraph_sentence": "\uc790\uc720\uc640 \uad8c\ub825 \uc0ac\uc774\uc758 \uc2f8\uc6c0\uc5d0 \ub300\ud55c \uc601\ud654\ub85c\uc11c \u300a\ube0c\uc774 \ud3ec \ubca4\ub370\ud0c0\u300b\ub294 \uc544\ub3cc\ud504 \ud788\ud2c0\ub7ec \uce58\ud558\uc758 \uc81c3\uc81c\uad6d\uacfc \uc870\uc9c0 \uc624\uc6f0\uc758 \u300a1984\ub144\u300b\ub4f1 \uace0\uc804\uc801\uc778 \uc804\uccb4\uc8fc\uc758 \uc544\uc774\ucf58\ub4e4\uc758 \uc774\ubbf8\uc9c0\ub97c \ub530\uc654\ub2e4. \uadf8 \uc608\ub85c, \uc11c\ud2c0\ub7ec \ub294 \uc8fc\ub85c \uac70\ub300\ud55c \ube44\ub514\uc624 \uc2a4\ud06c\ub9b0\uc5d0 \ub098\ud0c0\ub098\uba70, \uc9d1\uc9d1\ub9c8\ub2e4 \uc11c\ud2c0\ub7ec\uc758 \uc0ac\uc9c4\uc774 \uac78\ub824 \uc788\ub2e4. \uc774\uac83\uc740 \ube45 \ube0c\ub77c\ub354\ub97c \ub5a0\uc62c\ub9ac\uac8c \ud55c\ub2e4. \ub3c5\uc7ac \uc815\ubd80\uc758 \uc2ac\ub85c\uac74 '\ud1b5\ud569\uc744 \ud1b5\ud574 \ud798\uc744, \ubbff\uc74c\uc744 \ud1b5\ud574 \ud1b5\ud569\uc744' \ub3c4 \uc5b5\uc591\uc0c1 \u300a1984\ub144\u300b\uc5d0 \ub098\uc624\ub294 \uad6c\ud638 \"\uc804\uc7c1\uc740 \ud3c9\ud654. \uc790\uc720\ub294 \uad74\uc885. \ubb34\uc2dd\uc740 \ud798\" \uacfc \ube44\uc2b7\ud558\ub2e4. \ub450 \uc791\ud488\uc758 \uc5f0\uad00\uc131\uc740 \uc774\ube44\uac00 \uac10\uae08\ub418\uc5c8\uc744 \ub54c \uac10\ubc29\uc5d0\uc11c \uc950 \ub97c \ubc1c\uacac\ud558\ub294 \uac83\uc5d0\uc11c\ub3c4 \ucc3e\uc744 \uc218 \uc788\ub2e4. \u300a1984\ub144\u300b\uc758 \uc8fc\uc778\uacf5 \uc708\uc2a4\ud134 \uc2a4\ubbf8\uc2a4\ub294 \uc950 \uacf5\ud3ec\uc99d\uc774 \uc788\ub2e4. \ub610\ud55c \uad8c\ub825\uc790\uac00 \ud3d0\uc1c4 \ud68c\ub85c \ud154\ub808\ube44\uc804 \uac19\uc740 \ub300\uc911 \uac10\uc2dc\ub97c \uc0ac\uc6a9\ud558\ub294 \uac83\ub3c4 \uac19\ub2e4. \ubc1c\ub808\ub9ac\ub294 \ub808\uc988\ube44\uc5b8 \uad6c\ub958 \uc2dc\uc124\ub85c \ubcf4\ub0b4\uc838 \uc0dd\uccb4 \uc2e4\ud5d8\uc744 \ub2f9\ud55c\ub2e4. \uc774\uac83\uc740 \ud640\ub85c\ucf54\uc2a4\ud2b8 \ub2f9\uc2dc \ub098\uce58\uc758 \ub3d9\uc131\uc560\uc790\uc5d0 \ub300\ud55c \ucc98\uc6b0 \ubc0f \uc77c\ubcf8\uc758 731 \ubd80\ub300\uc758 \ub9cc\ud589\uacfc \uc77c\ub9e5\uc0c1\ud1b5\ud55c\ub2e4. \uc544\ub2f4 \uc11c\ud2c0\ub7ec\ub77c\ub294 \uc774\ub984\uc740 \uc544\ub3cc\ud504 \ud788\ud2c0\ub7ec\uc5d0\uc11c \ub530\uc628 \uac83\uc774\ub2e4. \uc11c\ud2c0\ub7ec\uc758 \ud788\uc2a4\ud14c\ub9ad\ud55c \uc5f0\uc124\ub3c4 \ud788\ud2c0\ub7ec\uc758 \uc5f0\uc124 \uc2a4\ud0c0\uc77c\uacfc \ube44\uc2b7\ud558\ub2e4. \ub178\uc2a4\ud30c\uc774\uc5b4\ub294 \uc601\uad6d\uc758 \uc0c1\uc9d5\uc744 \uc131\uc870\uc9c0 \uc2ed\uc790\uac00\uc5d0\uc11c \ub85c\ub80c \uc2ed\uc790\uac00\ub85c \ubc14\uafb8\uc5b4 \ub193\uc558\ub2e4. \ub85c\ub808\uc778 \uc2ed\uc790\uac00\ub294 2\ucc28 \ub300\uc804 \uc911 \uc790\uc720 \ud504\ub791\uc2a4\uc758 \uc0c1\uc9d5\uc73c\ub85c, \ub098\uce58\uc758 \ud558\ucf04\ud06c\ub85c\uc774\uce20\uc5d0 \ub300\uc751\ud558\ub294, \ud504\ub791\uc2a4 \uc560\uad6d\uc2ec\uc758 \uc0c1\uc9d5\uc774\uc5c8\ub2e4. \ub9e4\uc2a4 \ubbf8\ub514\uc5b4\ub294 \uc815\ubd80\uc758 \ud504\ub85c\ud30c\uac04\ub2e4\uc758 \uc2dc\uc885 \ub178\ub987\uc744 \ud558\uace0 \uc788\ub2e4. \uc774\uac83\uc740 \uc804\uccb4\uc8fc\uc758 \uc815\uad8c\ud558\uba74 \ub5a0\uc62c\ub824\uc9c0\ub294 \ub300\uc911\uc801 \uc774\ubbf8\uc9c0\uc774\ub2e4.", "paragraph_answer": "\uc790\uc720\uc640 \uad8c\ub825 \uc0ac\uc774\uc758 \uc2f8\uc6c0\uc5d0 \ub300\ud55c \uc601\ud654\ub85c\uc11c \u300a\ube0c\uc774 \ud3ec \ubca4\ub370\ud0c0\u300b\ub294 \uc544\ub3cc\ud504 \ud788\ud2c0\ub7ec \uce58\ud558\uc758 \uc81c3\uc81c\uad6d\uacfc \uc870\uc9c0 \uc624\uc6f0\uc758 \u300a1984\ub144\u300b\ub4f1 \uace0\uc804\uc801\uc778 \uc804\uccb4\uc8fc\uc758 \uc544\uc774\ucf58\ub4e4\uc758 \uc774\ubbf8\uc9c0\ub97c \ub530\uc654\ub2e4. \uadf8 \uc608\ub85c, \uc11c\ud2c0\ub7ec \ub294 \uc8fc\ub85c \uac70\ub300\ud55c \ube44\ub514\uc624 \uc2a4\ud06c\ub9b0\uc5d0 \ub098\ud0c0\ub098\uba70, \uc9d1\uc9d1\ub9c8\ub2e4 \uc11c\ud2c0\ub7ec\uc758 \uc0ac\uc9c4\uc774 \uac78\ub824 \uc788\ub2e4. \uc774\uac83\uc740 \ube45 \ube0c\ub77c\ub354\ub97c \ub5a0\uc62c\ub9ac\uac8c \ud55c\ub2e4. \ub3c5\uc7ac \uc815\ubd80\uc758 \uc2ac\ub85c\uac74 '\ud1b5\ud569\uc744 \ud1b5\ud574 \ud798\uc744, \ubbff\uc74c\uc744 \ud1b5\ud574 \ud1b5\ud569\uc744' \ub3c4 \uc5b5\uc591\uc0c1 \u300a1984\ub144\u300b\uc5d0 \ub098\uc624\ub294 \uad6c\ud638 \"\uc804\uc7c1\uc740 \ud3c9\ud654. \uc790\uc720\ub294 \uad74\uc885. \ubb34\uc2dd\uc740 \ud798\" \uacfc \ube44\uc2b7\ud558\ub2e4. \ub450 \uc791\ud488\uc758 \uc5f0\uad00\uc131\uc740 \uc774\ube44\uac00 \uac10\uae08\ub418\uc5c8\uc744 \ub54c \uac10\ubc29\uc5d0\uc11c \uc950 \ub97c \ubc1c\uacac\ud558\ub294 \uac83\uc5d0\uc11c\ub3c4 \ucc3e\uc744 \uc218 \uc788\ub2e4. \u300a1984\ub144\u300b\uc758 \uc8fc\uc778\uacf5 \uc708\uc2a4\ud134 \uc2a4\ubbf8\uc2a4\ub294 \uc950 \uacf5\ud3ec\uc99d\uc774 \uc788\ub2e4. \ub610\ud55c \uad8c\ub825\uc790\uac00 \ud3d0\uc1c4 \ud68c\ub85c \ud154\ub808\ube44\uc804 \uac19\uc740 \ub300\uc911 \uac10\uc2dc\ub97c \uc0ac\uc6a9\ud558\ub294 \uac83\ub3c4 \uac19\ub2e4. \ubc1c\ub808\ub9ac\ub294 \ub808\uc988\ube44\uc5b8 \uad6c\ub958 \uc2dc\uc124\ub85c \ubcf4\ub0b4\uc838 \uc0dd\uccb4 \uc2e4\ud5d8\uc744 \ub2f9\ud55c\ub2e4. \uc774\uac83\uc740 \ud640\ub85c\ucf54\uc2a4\ud2b8 \ub2f9\uc2dc \ub098\uce58\uc758 \ub3d9\uc131\uc560\uc790\uc5d0 \ub300\ud55c \ucc98\uc6b0 \ubc0f \uc77c\ubcf8\uc758 731 \ubd80\ub300\uc758 \ub9cc\ud589\uacfc \uc77c\ub9e5\uc0c1\ud1b5\ud55c\ub2e4. \uc544\ub2f4 \uc11c\ud2c0\ub7ec\ub77c\ub294 \uc774\ub984\uc740 \uc544\ub3cc\ud504 \ud788\ud2c0\ub7ec\uc5d0\uc11c \ub530\uc628 \uac83\uc774\ub2e4. \uc11c\ud2c0\ub7ec\uc758 \ud788\uc2a4\ud14c\ub9ad\ud55c \uc5f0\uc124\ub3c4 \ud788\ud2c0\ub7ec\uc758 \uc5f0\uc124 \uc2a4\ud0c0\uc77c\uacfc \ube44\uc2b7\ud558\ub2e4. \ub178\uc2a4\ud30c\uc774\uc5b4\ub294 \uc601\uad6d\uc758 \uc0c1\uc9d5\uc744 \uc131\uc870\uc9c0 \uc2ed\uc790\uac00\uc5d0\uc11c \ub85c\ub80c \uc2ed\uc790\uac00\ub85c \ubc14\uafb8\uc5b4 \ub193\uc558\ub2e4. \ub85c\ub808\uc778 \uc2ed\uc790\uac00\ub294 2\ucc28 \ub300\uc804 \uc911 \uc790\uc720 \ud504\ub791\uc2a4\uc758 \uc0c1\uc9d5\uc73c\ub85c, \ub098\uce58\uc758 \ud558\ucf04\ud06c\ub85c\uc774\uce20\uc5d0 \ub300\uc751\ud558\ub294, \ud504\ub791\uc2a4 \uc560\uad6d\uc2ec\uc758 \uc0c1\uc9d5\uc774\uc5c8\ub2e4. \ub9e4\uc2a4 \ubbf8\ub514\uc5b4\ub294 \uc815\ubd80\uc758 \ud504\ub85c\ud30c\uac04\ub2e4\uc758 \uc2dc\uc885 \ub178\ub987\uc744 \ud558\uace0 \uc788\ub2e4. \uc774\uac83\uc740 \uc804\uccb4\uc8fc\uc758 \uc815\uad8c\ud558\uba74 \ub5a0\uc62c\ub824\uc9c0\ub294 \ub300\uc911\uc801 \uc774\ubbf8\uc9c0\uc774\ub2e4.", "sentence_answer": "\ub450 \uc791\ud488\uc758 \uc5f0\uad00\uc131\uc740 \uc774\ube44\uac00 \uac10\uae08\ub418\uc5c8\uc744 \ub54c \uac10\ubc29\uc5d0\uc11c \uc950 \ub97c \ubc1c\uacac\ud558\ub294 \uac83\uc5d0\uc11c\ub3c4 \ucc3e\uc744 \uc218 \uc788\ub2e4.", "paragraph_id": "6583521-14-1", "paragraph_question": "question: \ube0c\uc774 \ud3ec \ubca4\ub370\ud0c0\uc640 1984 \uc0ac\uc774\uc758 \uc5f0\uad00\uc131\uc744 \ubc1c\uacac\ud560 \uc218 \uc788\ub294 \ub3d9\ubb3c\uc740?, context: \uc790\uc720\uc640 \uad8c\ub825 \uc0ac\uc774\uc758 \uc2f8\uc6c0\uc5d0 \ub300\ud55c \uc601\ud654\ub85c\uc11c \u300a\ube0c\uc774 \ud3ec \ubca4\ub370\ud0c0\u300b\ub294 \uc544\ub3cc\ud504 \ud788\ud2c0\ub7ec \uce58\ud558\uc758 \uc81c3\uc81c\uad6d\uacfc \uc870\uc9c0 \uc624\uc6f0\uc758 \u300a1984\ub144\u300b\ub4f1 \uace0\uc804\uc801\uc778 \uc804\uccb4\uc8fc\uc758 \uc544\uc774\ucf58\ub4e4\uc758 \uc774\ubbf8\uc9c0\ub97c \ub530\uc654\ub2e4. \uadf8 \uc608\ub85c, \uc11c\ud2c0\ub7ec \ub294 \uc8fc\ub85c \uac70\ub300\ud55c \ube44\ub514\uc624 \uc2a4\ud06c\ub9b0\uc5d0 \ub098\ud0c0\ub098\uba70, \uc9d1\uc9d1\ub9c8\ub2e4 \uc11c\ud2c0\ub7ec\uc758 \uc0ac\uc9c4\uc774 \uac78\ub824 \uc788\ub2e4. \uc774\uac83\uc740 \ube45 \ube0c\ub77c\ub354\ub97c \ub5a0\uc62c\ub9ac\uac8c \ud55c\ub2e4. \ub3c5\uc7ac \uc815\ubd80\uc758 \uc2ac\ub85c\uac74 '\ud1b5\ud569\uc744 \ud1b5\ud574 \ud798\uc744, \ubbff\uc74c\uc744 \ud1b5\ud574 \ud1b5\ud569\uc744' \ub3c4 \uc5b5\uc591\uc0c1 \u300a1984\ub144\u300b\uc5d0 \ub098\uc624\ub294 \uad6c\ud638 \"\uc804\uc7c1\uc740 \ud3c9\ud654. \uc790\uc720\ub294 \uad74\uc885. \ubb34\uc2dd\uc740 \ud798\" \uacfc \ube44\uc2b7\ud558\ub2e4. \ub450 \uc791\ud488\uc758 \uc5f0\uad00\uc131\uc740 \uc774\ube44\uac00 \uac10\uae08\ub418\uc5c8\uc744 \ub54c \uac10\ubc29\uc5d0\uc11c \uc950\ub97c \ubc1c\uacac\ud558\ub294 \uac83\uc5d0\uc11c\ub3c4 \ucc3e\uc744 \uc218 \uc788\ub2e4. \u300a1984\ub144\u300b\uc758 \uc8fc\uc778\uacf5 \uc708\uc2a4\ud134 \uc2a4\ubbf8\uc2a4\ub294 \uc950 \uacf5\ud3ec\uc99d\uc774 \uc788\ub2e4. \ub610\ud55c \uad8c\ub825\uc790\uac00 \ud3d0\uc1c4 \ud68c\ub85c \ud154\ub808\ube44\uc804 \uac19\uc740 \ub300\uc911 \uac10\uc2dc\ub97c \uc0ac\uc6a9\ud558\ub294 \uac83\ub3c4 \uac19\ub2e4. \ubc1c\ub808\ub9ac\ub294 \ub808\uc988\ube44\uc5b8 \uad6c\ub958 \uc2dc\uc124\ub85c \ubcf4\ub0b4\uc838 \uc0dd\uccb4 \uc2e4\ud5d8\uc744 \ub2f9\ud55c\ub2e4. \uc774\uac83\uc740 \ud640\ub85c\ucf54\uc2a4\ud2b8 \ub2f9\uc2dc \ub098\uce58\uc758 \ub3d9\uc131\uc560\uc790\uc5d0 \ub300\ud55c \ucc98\uc6b0 \ubc0f \uc77c\ubcf8\uc758 731 \ubd80\ub300\uc758 \ub9cc\ud589\uacfc \uc77c\ub9e5\uc0c1\ud1b5\ud55c\ub2e4. \uc544\ub2f4 \uc11c\ud2c0\ub7ec\ub77c\ub294 \uc774\ub984\uc740 \uc544\ub3cc\ud504 \ud788\ud2c0\ub7ec\uc5d0\uc11c \ub530\uc628 \uac83\uc774\ub2e4. \uc11c\ud2c0\ub7ec\uc758 \ud788\uc2a4\ud14c\ub9ad\ud55c \uc5f0\uc124\ub3c4 \ud788\ud2c0\ub7ec\uc758 \uc5f0\uc124 \uc2a4\ud0c0\uc77c\uacfc \ube44\uc2b7\ud558\ub2e4. \ub178\uc2a4\ud30c\uc774\uc5b4\ub294 \uc601\uad6d\uc758 \uc0c1\uc9d5\uc744 \uc131\uc870\uc9c0 \uc2ed\uc790\uac00\uc5d0\uc11c \ub85c\ub80c \uc2ed\uc790\uac00\ub85c \ubc14\uafb8\uc5b4 \ub193\uc558\ub2e4. \ub85c\ub808\uc778 \uc2ed\uc790\uac00\ub294 2\ucc28 \ub300\uc804 \uc911 \uc790\uc720 \ud504\ub791\uc2a4\uc758 \uc0c1\uc9d5\uc73c\ub85c, \ub098\uce58\uc758 \ud558\ucf04\ud06c\ub85c\uc774\uce20\uc5d0 \ub300\uc751\ud558\ub294, \ud504\ub791\uc2a4 \uc560\uad6d\uc2ec\uc758 \uc0c1\uc9d5\uc774\uc5c8\ub2e4. \ub9e4\uc2a4 \ubbf8\ub514\uc5b4\ub294 \uc815\ubd80\uc758 \ud504\ub85c\ud30c\uac04\ub2e4\uc758 \uc2dc\uc885 \ub178\ub987\uc744 \ud558\uace0 \uc788\ub2e4. \uc774\uac83\uc740 \uc804\uccb4\uc8fc\uc758 \uc815\uad8c\ud558\uba74 \ub5a0\uc62c\ub824\uc9c0\ub294 \ub300\uc911\uc801 \uc774\ubbf8\uc9c0\uc774\ub2e4."} {"question": "\ucca0\uc77c\uc804\uc7c1 \uc774\ud6c4 \uc758\ud654\uad8c\uc758 \uad6c\ud638\ub294?", "paragraph": "\uc758\ud654\ub2e8\uc758 \uad6c\ud638\ub294 \uc758\ud654\uad8c\uc73c\ub85c \uc788\uc744 \ub54c\uc5d0 '\ubc18\uccad\ubcf5\uba85'(\u53cd\u6df8\u5fa9\u660e)\uc5d0 \ub450\uc5c8\ub2e4. \uadf8\ub7ec\ub098 \uccad\uc77c\uc804\uc7c1 \uc774\ud6c4\uc5d0 \uc5f4\uac15\uc758 \uce68\ub7b5\uc774 \uac00\uc18d\ud654\ub418\uc790 '\ubd80\uccad\uba78\uc591'(\u6276\u6df8\u6ec5\u6d0b)\uc758 \uad6c\ud638\ub85c \ubc14\ub00c\uac8c \ub418\uc5c8\ub2e4. \uc774\ucc98\ub7fc \uad6c\ud638\uac00 \ubc14\ub00c\uac8c \ub41c \uac83\uc740 1898\ub144 7\uc6d4\uc5d0 \uc4f0\ucd28 \uc131\uc5d0\uc11c \uc77c\uc5b4\ub09c \ubc18\uae30\ub3c5\uad50 \uc6b4\ub3d9 \ub54c\uc758 \uaca9\ubb38\uc5d0 \ucc98\uc74c\uc73c\ub85c \ub098\ud0c0\ub0ac\ub294\ub370, '\uc21c\uccad\uba78\uc591'(\u9806\u6df8\u6ec5\u6d0b, \ubd80\uccad\uba78\uc591\uacfc \uac19\uc740 \uc758\ubbf8.)\uc774\ub780 \uad6c\ud638\uc600\ub2e4. \uadf8\ub9ac\uace0 \uc774 \ud574 \uac00\uc744\uc5d0 \uc0b0\ub465 \uc131\uc758 \uc720\uba85\ud55c \uc758\ud654\uad8c \uc218\ub839 \uc870\uc0bc\ub2e4(\u8d99\u4e09\u591a) \ub4f1\uc774 \uad00\ud604(\u51a0\u7e23)\uc5d0\uc11c 3000\uba85\uc744 \ubaa8\uc544 \uac01\uc9c0\uc758 \uad50\ud68c\ub97c \ubd88\ud0dc\uc6b0\uace0 \uc120\uad50\uc0ac\uc640 \uad50\ubbfc\uc744 \uc0b4\ud574\ud55c \uad00\ud604\uad50\uc548\ub54c \uc774\uc640 \ube44\uc2b7\ud55c '\uc870\uccad\uba78\uc591'(\u52a9\u6df8\u6ec5\u6d0b), \ud639\uc740 '\ubd80\uccad\uba78\uc591'(\u6276\u6df8\u6ec5\u6d0b)\uc758 \uae30\uce58\ub97c \ub0b4\uc138\uc6b0\uba74\uc11c \ubcf4\ud3b8\uc801\uc73c\ub85c \uc0ac\uc6a9\ub418\uc5c8\ub2e4. \uc804\uc790\ub294 \uc5f0\uc18d\uc131\uc774 \uc5c6\uc5c8\uc73c\ub098 \ud6c4\uc790\ub294 \uc5f0\uc18d\uc131\uc744 \uac16\uace0 \uc788\uc5c8\ub2e4. \uc774 \uad6c\ud638\ub97c \uc0ac\uc6a9\ud558\uba74\uc11c\ubd80\ud130 \uccad\ub098\ub77c\uc758 \uad00\ub9ac\ub294 \ubb3c\ub860 \uc0ac\ub300\ubd80, \uc7a5\uad70\ub4e4\ub85c\ubd80\ud130 \ub9ce\uc740 \uc9c0\uc9c0\uc640 \ub3d9\uc815\uc744 \ubc1b\uac8c\ub418\uc5c8\uc73c\uba70, \ub354 \ub098\uc544\uac00 \uad11\ubc94\uc704\ud55c \ub300\uc911\uc801 \uae30\ubc18\uc73c\ub85c \ubc18\uc81c\u00b7\uc560\uad6d \uc6b4\ub3d9\uc744 \uc804\uac1c\ud560 \uc218 \uc788\uc5c8\ub2e4.", "answer": "\ubd80\uccad\uba78\uc591", "sentence": "\uadf8\ub7ec\ub098 \uccad\uc77c\uc804\uc7c1 \uc774\ud6c4\uc5d0 \uc5f4\uac15\uc758 \uce68\ub7b5\uc774 \uac00\uc18d\ud654\ub418\uc790 ' \ubd80\uccad\uba78\uc591 '(\u6276\u6df8\u6ec5\u6d0b)\uc758 \uad6c\ud638\ub85c \ubc14\ub00c\uac8c \ub418\uc5c8\ub2e4.", "paragraph_sentence": "\uc758\ud654\ub2e8\uc758 \uad6c\ud638\ub294 \uc758\ud654\uad8c\uc73c\ub85c \uc788\uc744 \ub54c\uc5d0 '\ubc18\uccad\ubcf5\uba85'(\u53cd\u6df8\u5fa9\u660e)\uc5d0 \ub450\uc5c8\ub2e4. \uadf8\ub7ec\ub098 \uccad\uc77c\uc804\uc7c1 \uc774\ud6c4\uc5d0 \uc5f4\uac15\uc758 \uce68\ub7b5\uc774 \uac00\uc18d\ud654\ub418\uc790 ' \ubd80\uccad\uba78\uc591 '(\u6276\u6df8\u6ec5\u6d0b)\uc758 \uad6c\ud638\ub85c \ubc14\ub00c\uac8c \ub418\uc5c8\ub2e4. \uc774\ucc98\ub7fc \uad6c\ud638\uac00 \ubc14\ub00c\uac8c \ub41c \uac83\uc740 1898\ub144 7\uc6d4\uc5d0 \uc4f0\ucd28 \uc131\uc5d0\uc11c \uc77c\uc5b4\ub09c \ubc18\uae30\ub3c5\uad50 \uc6b4\ub3d9 \ub54c\uc758 \uaca9\ubb38\uc5d0 \ucc98\uc74c\uc73c\ub85c \ub098\ud0c0\ub0ac\ub294\ub370, '\uc21c\uccad\uba78\uc591'(\u9806\u6df8\u6ec5\u6d0b, \ubd80\uccad\uba78\uc591\uacfc \uac19\uc740 \uc758\ubbf8.)\uc774\ub780 \uad6c\ud638\uc600\ub2e4. \uadf8\ub9ac\uace0 \uc774 \ud574 \uac00\uc744\uc5d0 \uc0b0\ub465 \uc131\uc758 \uc720\uba85\ud55c \uc758\ud654\uad8c \uc218\ub839 \uc870\uc0bc\ub2e4(\u8d99\u4e09\u591a) \ub4f1\uc774 \uad00\ud604(\u51a0\u7e23)\uc5d0\uc11c 3000\uba85\uc744 \ubaa8\uc544 \uac01\uc9c0\uc758 \uad50\ud68c\ub97c \ubd88\ud0dc\uc6b0\uace0 \uc120\uad50\uc0ac\uc640 \uad50\ubbfc\uc744 \uc0b4\ud574\ud55c \uad00\ud604\uad50\uc548\ub54c \uc774\uc640 \ube44\uc2b7\ud55c '\uc870\uccad\uba78\uc591'(\u52a9\u6df8\u6ec5\u6d0b), \ud639\uc740 '\ubd80\uccad\uba78\uc591'(\u6276\u6df8\u6ec5\u6d0b)\uc758 \uae30\uce58\ub97c \ub0b4\uc138\uc6b0\uba74\uc11c \ubcf4\ud3b8\uc801\uc73c\ub85c \uc0ac\uc6a9\ub418\uc5c8\ub2e4. \uc804\uc790\ub294 \uc5f0\uc18d\uc131\uc774 \uc5c6\uc5c8\uc73c\ub098 \ud6c4\uc790\ub294 \uc5f0\uc18d\uc131\uc744 \uac16\uace0 \uc788\uc5c8\ub2e4. \uc774 \uad6c\ud638\ub97c \uc0ac\uc6a9\ud558\uba74\uc11c\ubd80\ud130 \uccad\ub098\ub77c\uc758 \uad00\ub9ac\ub294 \ubb3c\ub860 \uc0ac\ub300\ubd80, \uc7a5\uad70\ub4e4\ub85c\ubd80\ud130 \ub9ce\uc740 \uc9c0\uc9c0\uc640 \ub3d9\uc815\uc744 \ubc1b\uac8c\ub418\uc5c8\uc73c\uba70, \ub354 \ub098\uc544\uac00 \uad11\ubc94\uc704\ud55c \ub300\uc911\uc801 \uae30\ubc18\uc73c\ub85c \ubc18\uc81c\u00b7\uc560\uad6d \uc6b4\ub3d9\uc744 \uc804\uac1c\ud560 \uc218 \uc788\uc5c8\ub2e4.", "paragraph_answer": "\uc758\ud654\ub2e8\uc758 \uad6c\ud638\ub294 \uc758\ud654\uad8c\uc73c\ub85c \uc788\uc744 \ub54c\uc5d0 '\ubc18\uccad\ubcf5\uba85'(\u53cd\u6df8\u5fa9\u660e)\uc5d0 \ub450\uc5c8\ub2e4. \uadf8\ub7ec\ub098 \uccad\uc77c\uc804\uc7c1 \uc774\ud6c4\uc5d0 \uc5f4\uac15\uc758 \uce68\ub7b5\uc774 \uac00\uc18d\ud654\ub418\uc790 ' \ubd80\uccad\uba78\uc591 '(\u6276\u6df8\u6ec5\u6d0b)\uc758 \uad6c\ud638\ub85c \ubc14\ub00c\uac8c \ub418\uc5c8\ub2e4. \uc774\ucc98\ub7fc \uad6c\ud638\uac00 \ubc14\ub00c\uac8c \ub41c \uac83\uc740 1898\ub144 7\uc6d4\uc5d0 \uc4f0\ucd28 \uc131\uc5d0\uc11c \uc77c\uc5b4\ub09c \ubc18\uae30\ub3c5\uad50 \uc6b4\ub3d9 \ub54c\uc758 \uaca9\ubb38\uc5d0 \ucc98\uc74c\uc73c\ub85c \ub098\ud0c0\ub0ac\ub294\ub370, '\uc21c\uccad\uba78\uc591'(\u9806\u6df8\u6ec5\u6d0b, \ubd80\uccad\uba78\uc591\uacfc \uac19\uc740 \uc758\ubbf8.)\uc774\ub780 \uad6c\ud638\uc600\ub2e4. \uadf8\ub9ac\uace0 \uc774 \ud574 \uac00\uc744\uc5d0 \uc0b0\ub465 \uc131\uc758 \uc720\uba85\ud55c \uc758\ud654\uad8c \uc218\ub839 \uc870\uc0bc\ub2e4(\u8d99\u4e09\u591a) \ub4f1\uc774 \uad00\ud604(\u51a0\u7e23)\uc5d0\uc11c 3000\uba85\uc744 \ubaa8\uc544 \uac01\uc9c0\uc758 \uad50\ud68c\ub97c \ubd88\ud0dc\uc6b0\uace0 \uc120\uad50\uc0ac\uc640 \uad50\ubbfc\uc744 \uc0b4\ud574\ud55c \uad00\ud604\uad50\uc548\ub54c \uc774\uc640 \ube44\uc2b7\ud55c '\uc870\uccad\uba78\uc591'(\u52a9\u6df8\u6ec5\u6d0b), \ud639\uc740 '\ubd80\uccad\uba78\uc591'(\u6276\u6df8\u6ec5\u6d0b)\uc758 \uae30\uce58\ub97c \ub0b4\uc138\uc6b0\uba74\uc11c \ubcf4\ud3b8\uc801\uc73c\ub85c \uc0ac\uc6a9\ub418\uc5c8\ub2e4. \uc804\uc790\ub294 \uc5f0\uc18d\uc131\uc774 \uc5c6\uc5c8\uc73c\ub098 \ud6c4\uc790\ub294 \uc5f0\uc18d\uc131\uc744 \uac16\uace0 \uc788\uc5c8\ub2e4. \uc774 \uad6c\ud638\ub97c \uc0ac\uc6a9\ud558\uba74\uc11c\ubd80\ud130 \uccad\ub098\ub77c\uc758 \uad00\ub9ac\ub294 \ubb3c\ub860 \uc0ac\ub300\ubd80, \uc7a5\uad70\ub4e4\ub85c\ubd80\ud130 \ub9ce\uc740 \uc9c0\uc9c0\uc640 \ub3d9\uc815\uc744 \ubc1b\uac8c\ub418\uc5c8\uc73c\uba70, \ub354 \ub098\uc544\uac00 \uad11\ubc94\uc704\ud55c \ub300\uc911\uc801 \uae30\ubc18\uc73c\ub85c \ubc18\uc81c\u00b7\uc560\uad6d \uc6b4\ub3d9\uc744 \uc804\uac1c\ud560 \uc218 \uc788\uc5c8\ub2e4.", "sentence_answer": "\uadf8\ub7ec\ub098 \uccad\uc77c\uc804\uc7c1 \uc774\ud6c4\uc5d0 \uc5f4\uac15\uc758 \uce68\ub7b5\uc774 \uac00\uc18d\ud654\ub418\uc790 ' \ubd80\uccad\uba78\uc591 '(\u6276\u6df8\u6ec5\u6d0b)\uc758 \uad6c\ud638\ub85c \ubc14\ub00c\uac8c \ub418\uc5c8\ub2e4.", "paragraph_id": "6544419-4-1", "paragraph_question": "question: \ucca0\uc77c\uc804\uc7c1 \uc774\ud6c4 \uc758\ud654\uad8c\uc758 \uad6c\ud638\ub294?, context: \uc758\ud654\ub2e8\uc758 \uad6c\ud638\ub294 \uc758\ud654\uad8c\uc73c\ub85c \uc788\uc744 \ub54c\uc5d0 '\ubc18\uccad\ubcf5\uba85'(\u53cd\u6df8\u5fa9\u660e)\uc5d0 \ub450\uc5c8\ub2e4. \uadf8\ub7ec\ub098 \uccad\uc77c\uc804\uc7c1 \uc774\ud6c4\uc5d0 \uc5f4\uac15\uc758 \uce68\ub7b5\uc774 \uac00\uc18d\ud654\ub418\uc790 '\ubd80\uccad\uba78\uc591'(\u6276\u6df8\u6ec5\u6d0b)\uc758 \uad6c\ud638\ub85c \ubc14\ub00c\uac8c \ub418\uc5c8\ub2e4. \uc774\ucc98\ub7fc \uad6c\ud638\uac00 \ubc14\ub00c\uac8c \ub41c \uac83\uc740 1898\ub144 7\uc6d4\uc5d0 \uc4f0\ucd28 \uc131\uc5d0\uc11c \uc77c\uc5b4\ub09c \ubc18\uae30\ub3c5\uad50 \uc6b4\ub3d9 \ub54c\uc758 \uaca9\ubb38\uc5d0 \ucc98\uc74c\uc73c\ub85c \ub098\ud0c0\ub0ac\ub294\ub370, '\uc21c\uccad\uba78\uc591'(\u9806\u6df8\u6ec5\u6d0b, \ubd80\uccad\uba78\uc591\uacfc \uac19\uc740 \uc758\ubbf8.)\uc774\ub780 \uad6c\ud638\uc600\ub2e4. \uadf8\ub9ac\uace0 \uc774 \ud574 \uac00\uc744\uc5d0 \uc0b0\ub465 \uc131\uc758 \uc720\uba85\ud55c \uc758\ud654\uad8c \uc218\ub839 \uc870\uc0bc\ub2e4(\u8d99\u4e09\u591a) \ub4f1\uc774 \uad00\ud604(\u51a0\u7e23)\uc5d0\uc11c 3000\uba85\uc744 \ubaa8\uc544 \uac01\uc9c0\uc758 \uad50\ud68c\ub97c \ubd88\ud0dc\uc6b0\uace0 \uc120\uad50\uc0ac\uc640 \uad50\ubbfc\uc744 \uc0b4\ud574\ud55c \uad00\ud604\uad50\uc548\ub54c \uc774\uc640 \ube44\uc2b7\ud55c '\uc870\uccad\uba78\uc591'(\u52a9\u6df8\u6ec5\u6d0b), \ud639\uc740 '\ubd80\uccad\uba78\uc591'(\u6276\u6df8\u6ec5\u6d0b)\uc758 \uae30\uce58\ub97c \ub0b4\uc138\uc6b0\uba74\uc11c \ubcf4\ud3b8\uc801\uc73c\ub85c \uc0ac\uc6a9\ub418\uc5c8\ub2e4. \uc804\uc790\ub294 \uc5f0\uc18d\uc131\uc774 \uc5c6\uc5c8\uc73c\ub098 \ud6c4\uc790\ub294 \uc5f0\uc18d\uc131\uc744 \uac16\uace0 \uc788\uc5c8\ub2e4. \uc774 \uad6c\ud638\ub97c \uc0ac\uc6a9\ud558\uba74\uc11c\ubd80\ud130 \uccad\ub098\ub77c\uc758 \uad00\ub9ac\ub294 \ubb3c\ub860 \uc0ac\ub300\ubd80, \uc7a5\uad70\ub4e4\ub85c\ubd80\ud130 \ub9ce\uc740 \uc9c0\uc9c0\uc640 \ub3d9\uc815\uc744 \ubc1b\uac8c\ub418\uc5c8\uc73c\uba70, \ub354 \ub098\uc544\uac00 \uad11\ubc94\uc704\ud55c \ub300\uc911\uc801 \uae30\ubc18\uc73c\ub85c \ubc18\uc81c\u00b7\uc560\uad6d \uc6b4\ub3d9\uc744 \uc804\uac1c\ud560 \uc218 \uc788\uc5c8\ub2e4."} {"question": "\uc0ac\ud68c\ub294 \ud558\ub098\uc758 \uac70\ub300\ud55c \uc5b4\ub5a4\uac83\uacfc \uc720\uc0ac\ud55c\uac00?", "paragraph": "\ub3c4\ub355\uc801 \uc758\ubb34\uc758 \ubcf8\uc131\uacfc \ubc1c\uc0dd\uc801 \uc6d0\ucc9c\uc744 \ubd84\uc11d\ud558\uc5ec \ub2eb\ud78c\ub3c4\ub355\uacfc \uc5f4\ub9b0\ub3c4\ub355\uc758 \ubcf8\uc131\uc0c1 \ucc28\uc774\ub97c \uc9c0\uc801\ud558\uace0, \uc804\uc790\uc5d0 \uc758\uc874\ud558\ub294 \ub2eb\ud78c\uc0ac\ud68c\ub85c\ubd80\ud130 \ud6c4\uc790\uc5d0 \uae30\ucd08\ud558\ub294 \uc5f4\ub9b0\uc0ac\ud68c\ub85c\uc758 \uc774\ud589\uc5d0\uc11c \uc815\uc11c\uc801 \uac10\ub3d9\uc758 \uc5ed\ud560\uc774 \uc911\uc694\ud568\uc744 \uc5ed\uc124\ud55c\ub2e4. \uc0ac\ud68c\ub294 \ud558\ub098\uc758 \uac70\ub300\ud55c \uc720\uae30\uccb4\uc640 \uc720\uc0ac\ud558\uc5ec \uc0ac\ud68c\uc758 \uc0dd\uc874\uacfc \uc9c8\uc11c\ub97c \uc720\uc9c0\ud558\uae30 \uc704\ud574\uc11c\ub294 \uc131\uc6d0\ub4e4\uc774 \ubb34\uc758\uc2dd\uc801\uc73c\ub85c \uc218\ud589\ud558\ub294 \uc2b5\uad00\ub4e4\uc758 \uccb4\uacc4\ub97c \ud544\uc694\ub85c \ud55c\ub2e4. \uc778\uac04 \uc9c0\uc131\uc758 \ubc1c\uba85\ud488\uc73c\ub85c \ubcf4\uc774\ub294 \uad00\uc2b5, \ubc95, \uaddc\uc728, \ub3c4\ub355\uc801 \uc758\ubb34\ub4e4\uc758 \ubc1c\uc0dd\uc801 \uadfc\uc6d0\uc5d0\ub294 \uc9d1\ub2e8\uc758 \uc874\uc18d\uc744 \uc720\uc9c0\ud558\uae30 \uc704\ud55c \uc790\uc5f0\uc801 \ubcf8\ub2a5\uc758 \uc694\uad6c\uac00 \ub4e4\uc5b4 \uc788\ub2e4. \uc0ac\ud68c\uc801 \uacb0\uc18d\uc744 \uc9c0\ud5a5\ud558\ub294 \ubcf8\ub2a5\uc5d0 \ub530\ub77c \uc758\ubb34\ub97c \uac15\uc81c\ud558\ub294 \uc5b5\uc555\uc758 \ub3c4\ub355\uc740 \ub2eb\ud78c\uc0ac\ud68c\ub97c \uaca8\ub0e5\ud558\uba70 \uac00\uc871\uc560\uc640 \uc560\uad6d\uc2ec\uc73c\ub85c \ub618\ub618 \ubb49\uccd0 \ub2e4\ub978 \uc0ac\ud68c\uc5d0 \ub300\ud574 \ubc30\ud0c0\uc801\uc778 \uac70\ub9ac\ub97c \ucde8\ud55c\ub2e4. \uadf8\ub7ec\ub098 \uc5f4\ub9b0\uc0ac\ud68c\uc758 \uac00\ub2a5\uc131\uc774 \uc5c6\ub294 \uac83\uc740 \uc544\ub2c8\ub2e4. \uc0ac\ud68c\uc801 \ud615\ud0dc\ub85c \ub4dc\ub7ec\ub09c \uc0dd\uba85\uc758 \ud750\ub984\uc740 \uc790\uae30 \ubcf4\uc874\uc5d0\ub9cc \ubab0\ub450\ud558\ub294 \uac83\uc774 \uc544\ub2c8\ub77c \uc790\uae30 \ucc3d\uc870\uc758 \ub3c4\uc57d\ub3c4 \uac10\ud589\ud558\uae30 \ub54c\ubb38\uc774\ub2e4.", "answer": "\uc720\uae30\uccb4", "sentence": "\uc0ac\ud68c\ub294 \ud558\ub098\uc758 \uac70\ub300\ud55c \uc720\uae30\uccb4 \uc640 \uc720\uc0ac\ud558\uc5ec \uc0ac\ud68c\uc758 \uc0dd\uc874\uacfc \uc9c8\uc11c\ub97c \uc720\uc9c0\ud558\uae30 \uc704\ud574\uc11c\ub294 \uc131\uc6d0\ub4e4\uc774 \ubb34\uc758\uc2dd\uc801\uc73c\ub85c \uc218\ud589\ud558\ub294 \uc2b5\uad00\ub4e4\uc758 \uccb4\uacc4\ub97c \ud544\uc694\ub85c \ud55c\ub2e4.", "paragraph_sentence": "\ub3c4\ub355\uc801 \uc758\ubb34\uc758 \ubcf8\uc131\uacfc \ubc1c\uc0dd\uc801 \uc6d0\ucc9c\uc744 \ubd84\uc11d\ud558\uc5ec \ub2eb\ud78c\ub3c4\ub355\uacfc \uc5f4\ub9b0\ub3c4\ub355\uc758 \ubcf8\uc131\uc0c1 \ucc28\uc774\ub97c \uc9c0\uc801\ud558\uace0, \uc804\uc790\uc5d0 \uc758\uc874\ud558\ub294 \ub2eb\ud78c\uc0ac\ud68c\ub85c\ubd80\ud130 \ud6c4\uc790\uc5d0 \uae30\ucd08\ud558\ub294 \uc5f4\ub9b0\uc0ac\ud68c\ub85c\uc758 \uc774\ud589\uc5d0\uc11c \uc815\uc11c\uc801 \uac10\ub3d9\uc758 \uc5ed\ud560\uc774 \uc911\uc694\ud568\uc744 \uc5ed\uc124\ud55c\ub2e4. \uc0ac\ud68c\ub294 \ud558\ub098\uc758 \uac70\ub300\ud55c \uc720\uae30\uccb4 \uc640 \uc720\uc0ac\ud558\uc5ec \uc0ac\ud68c\uc758 \uc0dd\uc874\uacfc \uc9c8\uc11c\ub97c \uc720\uc9c0\ud558\uae30 \uc704\ud574\uc11c\ub294 \uc131\uc6d0\ub4e4\uc774 \ubb34\uc758\uc2dd\uc801\uc73c\ub85c \uc218\ud589\ud558\ub294 \uc2b5\uad00\ub4e4\uc758 \uccb4\uacc4\ub97c \ud544\uc694\ub85c \ud55c\ub2e4. \uc778\uac04 \uc9c0\uc131\uc758 \ubc1c\uba85\ud488\uc73c\ub85c \ubcf4\uc774\ub294 \uad00\uc2b5, \ubc95, \uaddc\uc728, \ub3c4\ub355\uc801 \uc758\ubb34\ub4e4\uc758 \ubc1c\uc0dd\uc801 \uadfc\uc6d0\uc5d0\ub294 \uc9d1\ub2e8\uc758 \uc874\uc18d\uc744 \uc720\uc9c0\ud558\uae30 \uc704\ud55c \uc790\uc5f0\uc801 \ubcf8\ub2a5\uc758 \uc694\uad6c\uac00 \ub4e4\uc5b4 \uc788\ub2e4. \uc0ac\ud68c\uc801 \uacb0\uc18d\uc744 \uc9c0\ud5a5\ud558\ub294 \ubcf8\ub2a5\uc5d0 \ub530\ub77c \uc758\ubb34\ub97c \uac15\uc81c\ud558\ub294 \uc5b5\uc555\uc758 \ub3c4\ub355\uc740 \ub2eb\ud78c\uc0ac\ud68c\ub97c \uaca8\ub0e5\ud558\uba70 \uac00\uc871\uc560\uc640 \uc560\uad6d\uc2ec\uc73c\ub85c \ub618\ub618 \ubb49\uccd0 \ub2e4\ub978 \uc0ac\ud68c\uc5d0 \ub300\ud574 \ubc30\ud0c0\uc801\uc778 \uac70\ub9ac\ub97c \ucde8\ud55c\ub2e4. \uadf8\ub7ec\ub098 \uc5f4\ub9b0\uc0ac\ud68c\uc758 \uac00\ub2a5\uc131\uc774 \uc5c6\ub294 \uac83\uc740 \uc544\ub2c8\ub2e4. \uc0ac\ud68c\uc801 \ud615\ud0dc\ub85c \ub4dc\ub7ec\ub09c \uc0dd\uba85\uc758 \ud750\ub984\uc740 \uc790\uae30 \ubcf4\uc874\uc5d0\ub9cc \ubab0\ub450\ud558\ub294 \uac83\uc774 \uc544\ub2c8\ub77c \uc790\uae30 \ucc3d\uc870\uc758 \ub3c4\uc57d\ub3c4 \uac10\ud589\ud558\uae30 \ub54c\ubb38\uc774\ub2e4.", "paragraph_answer": "\ub3c4\ub355\uc801 \uc758\ubb34\uc758 \ubcf8\uc131\uacfc \ubc1c\uc0dd\uc801 \uc6d0\ucc9c\uc744 \ubd84\uc11d\ud558\uc5ec \ub2eb\ud78c\ub3c4\ub355\uacfc \uc5f4\ub9b0\ub3c4\ub355\uc758 \ubcf8\uc131\uc0c1 \ucc28\uc774\ub97c \uc9c0\uc801\ud558\uace0, \uc804\uc790\uc5d0 \uc758\uc874\ud558\ub294 \ub2eb\ud78c\uc0ac\ud68c\ub85c\ubd80\ud130 \ud6c4\uc790\uc5d0 \uae30\ucd08\ud558\ub294 \uc5f4\ub9b0\uc0ac\ud68c\ub85c\uc758 \uc774\ud589\uc5d0\uc11c \uc815\uc11c\uc801 \uac10\ub3d9\uc758 \uc5ed\ud560\uc774 \uc911\uc694\ud568\uc744 \uc5ed\uc124\ud55c\ub2e4. \uc0ac\ud68c\ub294 \ud558\ub098\uc758 \uac70\ub300\ud55c \uc720\uae30\uccb4 \uc640 \uc720\uc0ac\ud558\uc5ec \uc0ac\ud68c\uc758 \uc0dd\uc874\uacfc \uc9c8\uc11c\ub97c \uc720\uc9c0\ud558\uae30 \uc704\ud574\uc11c\ub294 \uc131\uc6d0\ub4e4\uc774 \ubb34\uc758\uc2dd\uc801\uc73c\ub85c \uc218\ud589\ud558\ub294 \uc2b5\uad00\ub4e4\uc758 \uccb4\uacc4\ub97c \ud544\uc694\ub85c \ud55c\ub2e4. \uc778\uac04 \uc9c0\uc131\uc758 \ubc1c\uba85\ud488\uc73c\ub85c \ubcf4\uc774\ub294 \uad00\uc2b5, \ubc95, \uaddc\uc728, \ub3c4\ub355\uc801 \uc758\ubb34\ub4e4\uc758 \ubc1c\uc0dd\uc801 \uadfc\uc6d0\uc5d0\ub294 \uc9d1\ub2e8\uc758 \uc874\uc18d\uc744 \uc720\uc9c0\ud558\uae30 \uc704\ud55c \uc790\uc5f0\uc801 \ubcf8\ub2a5\uc758 \uc694\uad6c\uac00 \ub4e4\uc5b4 \uc788\ub2e4. \uc0ac\ud68c\uc801 \uacb0\uc18d\uc744 \uc9c0\ud5a5\ud558\ub294 \ubcf8\ub2a5\uc5d0 \ub530\ub77c \uc758\ubb34\ub97c \uac15\uc81c\ud558\ub294 \uc5b5\uc555\uc758 \ub3c4\ub355\uc740 \ub2eb\ud78c\uc0ac\ud68c\ub97c \uaca8\ub0e5\ud558\uba70 \uac00\uc871\uc560\uc640 \uc560\uad6d\uc2ec\uc73c\ub85c \ub618\ub618 \ubb49\uccd0 \ub2e4\ub978 \uc0ac\ud68c\uc5d0 \ub300\ud574 \ubc30\ud0c0\uc801\uc778 \uac70\ub9ac\ub97c \ucde8\ud55c\ub2e4. \uadf8\ub7ec\ub098 \uc5f4\ub9b0\uc0ac\ud68c\uc758 \uac00\ub2a5\uc131\uc774 \uc5c6\ub294 \uac83\uc740 \uc544\ub2c8\ub2e4. \uc0ac\ud68c\uc801 \ud615\ud0dc\ub85c \ub4dc\ub7ec\ub09c \uc0dd\uba85\uc758 \ud750\ub984\uc740 \uc790\uae30 \ubcf4\uc874\uc5d0\ub9cc \ubab0\ub450\ud558\ub294 \uac83\uc774 \uc544\ub2c8\ub77c \uc790\uae30 \ucc3d\uc870\uc758 \ub3c4\uc57d\ub3c4 \uac10\ud589\ud558\uae30 \ub54c\ubb38\uc774\ub2e4.", "sentence_answer": "\uc0ac\ud68c\ub294 \ud558\ub098\uc758 \uac70\ub300\ud55c \uc720\uae30\uccb4 \uc640 \uc720\uc0ac\ud558\uc5ec \uc0ac\ud68c\uc758 \uc0dd\uc874\uacfc \uc9c8\uc11c\ub97c \uc720\uc9c0\ud558\uae30 \uc704\ud574\uc11c\ub294 \uc131\uc6d0\ub4e4\uc774 \ubb34\uc758\uc2dd\uc801\uc73c\ub85c \uc218\ud589\ud558\ub294 \uc2b5\uad00\ub4e4\uc758 \uccb4\uacc4\ub97c \ud544\uc694\ub85c \ud55c\ub2e4.", "paragraph_id": "6404411-0-1", "paragraph_question": "question: \uc0ac\ud68c\ub294 \ud558\ub098\uc758 \uac70\ub300\ud55c \uc5b4\ub5a4\uac83\uacfc \uc720\uc0ac\ud55c\uac00?, context: \ub3c4\ub355\uc801 \uc758\ubb34\uc758 \ubcf8\uc131\uacfc \ubc1c\uc0dd\uc801 \uc6d0\ucc9c\uc744 \ubd84\uc11d\ud558\uc5ec \ub2eb\ud78c\ub3c4\ub355\uacfc \uc5f4\ub9b0\ub3c4\ub355\uc758 \ubcf8\uc131\uc0c1 \ucc28\uc774\ub97c \uc9c0\uc801\ud558\uace0, \uc804\uc790\uc5d0 \uc758\uc874\ud558\ub294 \ub2eb\ud78c\uc0ac\ud68c\ub85c\ubd80\ud130 \ud6c4\uc790\uc5d0 \uae30\ucd08\ud558\ub294 \uc5f4\ub9b0\uc0ac\ud68c\ub85c\uc758 \uc774\ud589\uc5d0\uc11c \uc815\uc11c\uc801 \uac10\ub3d9\uc758 \uc5ed\ud560\uc774 \uc911\uc694\ud568\uc744 \uc5ed\uc124\ud55c\ub2e4. \uc0ac\ud68c\ub294 \ud558\ub098\uc758 \uac70\ub300\ud55c \uc720\uae30\uccb4\uc640 \uc720\uc0ac\ud558\uc5ec \uc0ac\ud68c\uc758 \uc0dd\uc874\uacfc \uc9c8\uc11c\ub97c \uc720\uc9c0\ud558\uae30 \uc704\ud574\uc11c\ub294 \uc131\uc6d0\ub4e4\uc774 \ubb34\uc758\uc2dd\uc801\uc73c\ub85c \uc218\ud589\ud558\ub294 \uc2b5\uad00\ub4e4\uc758 \uccb4\uacc4\ub97c \ud544\uc694\ub85c \ud55c\ub2e4. \uc778\uac04 \uc9c0\uc131\uc758 \ubc1c\uba85\ud488\uc73c\ub85c \ubcf4\uc774\ub294 \uad00\uc2b5, \ubc95, \uaddc\uc728, \ub3c4\ub355\uc801 \uc758\ubb34\ub4e4\uc758 \ubc1c\uc0dd\uc801 \uadfc\uc6d0\uc5d0\ub294 \uc9d1\ub2e8\uc758 \uc874\uc18d\uc744 \uc720\uc9c0\ud558\uae30 \uc704\ud55c \uc790\uc5f0\uc801 \ubcf8\ub2a5\uc758 \uc694\uad6c\uac00 \ub4e4\uc5b4 \uc788\ub2e4. \uc0ac\ud68c\uc801 \uacb0\uc18d\uc744 \uc9c0\ud5a5\ud558\ub294 \ubcf8\ub2a5\uc5d0 \ub530\ub77c \uc758\ubb34\ub97c \uac15\uc81c\ud558\ub294 \uc5b5\uc555\uc758 \ub3c4\ub355\uc740 \ub2eb\ud78c\uc0ac\ud68c\ub97c \uaca8\ub0e5\ud558\uba70 \uac00\uc871\uc560\uc640 \uc560\uad6d\uc2ec\uc73c\ub85c \ub618\ub618 \ubb49\uccd0 \ub2e4\ub978 \uc0ac\ud68c\uc5d0 \ub300\ud574 \ubc30\ud0c0\uc801\uc778 \uac70\ub9ac\ub97c \ucde8\ud55c\ub2e4. \uadf8\ub7ec\ub098 \uc5f4\ub9b0\uc0ac\ud68c\uc758 \uac00\ub2a5\uc131\uc774 \uc5c6\ub294 \uac83\uc740 \uc544\ub2c8\ub2e4. \uc0ac\ud68c\uc801 \ud615\ud0dc\ub85c \ub4dc\ub7ec\ub09c \uc0dd\uba85\uc758 \ud750\ub984\uc740 \uc790\uae30 \ubcf4\uc874\uc5d0\ub9cc \ubab0\ub450\ud558\ub294 \uac83\uc774 \uc544\ub2c8\ub77c \uc790\uae30 \ucc3d\uc870\uc758 \ub3c4\uc57d\ub3c4 \uac10\ud589\ud558\uae30 \ub54c\ubb38\uc774\ub2e4."} {"question": "\ucd5c\uac15\ud76c\uac00 \ucd9c\uc5f0\ud558\uace0 2009\ub144\uc5d0 \uac1c\ubd09\ud55c \uc601\ud654\uc758 \uc774\ub984\uc740?", "paragraph": "2006\ub144 \uc601\ud654 \u300a\ub2ec\ucf64, \uc0b4\ubc8c\ud55c \uc5f0\uc778\u300b\uc758 \ud765\ud589\uc73c\ub85c \uc601\ud654\uacc4\uc758 \ube14\ub8e8\uce69\uc73c\ub85c \ub5a0\uc624\ub974\uba70 \uc544\uc5ed \ucd9c\uc2e0, \uccad\uc21c\uac00\ub828 \uc774\ubbf8\uc9c0\ub77c\ub294 \uaf2c\ub9ac\ud45c\ub97c \ub5bc\uace0 \uac1c\uc131\ud30c \ubc30\uc6b0\ub85c\uc11c \uc7ac\uc870\uba85\ubc1b\uae30 \uc2dc\uc791\ud588\ub2e4. 30\ub300 \uc9c1\uc7a5 \uc5ec\uc131\ub4e4\uc758 \uc77c\uacfc \uc0ac\ub791\uc744 \uadf8\ub9b0 \ub4dc\ub77c\ub9c8\u300a\ub2ec\ucf64\ud55c \ub098\uc758 \ub3c4\uc2dc\u300b\uc5d0\uc11c\ub294 \ud2b8\ub80c\ub514\ud558\uba74\uc11c\ub3c4 \uac1c\uc131 \uc788\ub294 \ubaa8\uc2b5\uc73c\ub85c \uc774\ub978\ubc14 \u2018\ucd5c\uac15\ud76c \uc2a4\ud0c0\uc77c\u2019\uc744 \uc120\ubcf4\uc774\uba70 \ud328\uc154\ub2c8\uc2a4\ud0c0\ub85c\ub3c4 \uc790\ub9ac\ub9e4\uae40\ud588\ub2e4. 2009\ub144 \uc601\ud654\u300a\uc560\uc790\u300b\ub3c4 \ud765\ud589\uc5d0 \uc131\uacf5\ud558\uba70 \ucda9\ubb34\ub85c \uc6d0\ud1b1 \ubc30\uc6b0\ub85c\uc11c\uc758 \uc785\uc9c0\ub97c \ub2e4\uc84c\ub2e4. 2011\ub144 KBS \uc0c8\ud574 \uac1c\ud3b8\uc73c\ub85c 2011\ub144 1\uc6d4 1\uc77c\ubd80\ud130 \u300a\ucd5c\uac15\ud76c\uc758 \ubcfc\ub968\uc744 \ub192\uc5ec\uc694\u300b\ub97c \ub2e4\uc2dc \uc2dc\uc791\ud588\uace0 \uac00\uc744 \uac1c\ud3b8\uc744 \ud1b5\ud574 2011\ub144 11\uc6d4 \ubd80\ud130\ub294 \u300a\ucd5c\uac15\ud76c\uc758 \uc57c\uac04\ube44\ud589\u300b\uc758 \uc9c4\ud589\uc744 \ub9e1\uc558\uc73c\ub098 2012\ub144 11\uc6d4\uc5d0 \ud558\ucc28\ud588\ub2e4.", "answer": "\uc560\uc790", "sentence": "2009\ub144 \uc601\ud654\u300a \uc560\uc790 \u300b\ub3c4 \ud765\ud589\uc5d0 \uc131\uacf5\ud558\uba70 \ucda9\ubb34\ub85c \uc6d0\ud1b1 \ubc30\uc6b0\ub85c\uc11c\uc758 \uc785\uc9c0\ub97c \ub2e4\uc84c\ub2e4.", "paragraph_sentence": "2006\ub144 \uc601\ud654 \u300a\ub2ec\ucf64, \uc0b4\ubc8c\ud55c \uc5f0\uc778\u300b\uc758 \ud765\ud589\uc73c\ub85c \uc601\ud654\uacc4\uc758 \ube14\ub8e8\uce69\uc73c\ub85c \ub5a0\uc624\ub974\uba70 \uc544\uc5ed \ucd9c\uc2e0, \uccad\uc21c\uac00\ub828 \uc774\ubbf8\uc9c0\ub77c\ub294 \uaf2c\ub9ac\ud45c\ub97c \ub5bc\uace0 \uac1c\uc131\ud30c \ubc30\uc6b0\ub85c\uc11c \uc7ac\uc870\uba85\ubc1b\uae30 \uc2dc\uc791\ud588\ub2e4. 30\ub300 \uc9c1\uc7a5 \uc5ec\uc131\ub4e4\uc758 \uc77c\uacfc \uc0ac\ub791\uc744 \uadf8\ub9b0 \ub4dc\ub77c\ub9c8\u300a\ub2ec\ucf64\ud55c \ub098\uc758 \ub3c4\uc2dc\u300b\uc5d0\uc11c\ub294 \ud2b8\ub80c\ub514\ud558\uba74\uc11c\ub3c4 \uac1c\uc131 \uc788\ub294 \ubaa8\uc2b5\uc73c\ub85c \uc774\ub978\ubc14 \u2018\ucd5c\uac15\ud76c \uc2a4\ud0c0\uc77c\u2019\uc744 \uc120\ubcf4\uc774\uba70 \ud328\uc154\ub2c8\uc2a4\ud0c0\ub85c\ub3c4 \uc790\ub9ac\ub9e4\uae40\ud588\ub2e4. 2009\ub144 \uc601\ud654\u300a \uc560\uc790 \u300b\ub3c4 \ud765\ud589\uc5d0 \uc131\uacf5\ud558\uba70 \ucda9\ubb34\ub85c \uc6d0\ud1b1 \ubc30\uc6b0\ub85c\uc11c\uc758 \uc785\uc9c0\ub97c \ub2e4\uc84c\ub2e4. 2011\ub144 KBS \uc0c8\ud574 \uac1c\ud3b8\uc73c\ub85c 2011\ub144 1\uc6d4 1\uc77c\ubd80\ud130 \u300a\ucd5c\uac15\ud76c\uc758 \ubcfc\ub968\uc744 \ub192\uc5ec\uc694\u300b\ub97c \ub2e4\uc2dc \uc2dc\uc791\ud588\uace0 \uac00\uc744 \uac1c\ud3b8\uc744 \ud1b5\ud574 2011\ub144 11\uc6d4 \ubd80\ud130\ub294 \u300a\ucd5c\uac15\ud76c\uc758 \uc57c\uac04\ube44\ud589\u300b\uc758 \uc9c4\ud589\uc744 \ub9e1\uc558\uc73c\ub098 2012\ub144 11\uc6d4\uc5d0 \ud558\ucc28\ud588\ub2e4.", "paragraph_answer": "2006\ub144 \uc601\ud654 \u300a\ub2ec\ucf64, \uc0b4\ubc8c\ud55c \uc5f0\uc778\u300b\uc758 \ud765\ud589\uc73c\ub85c \uc601\ud654\uacc4\uc758 \ube14\ub8e8\uce69\uc73c\ub85c \ub5a0\uc624\ub974\uba70 \uc544\uc5ed \ucd9c\uc2e0, \uccad\uc21c\uac00\ub828 \uc774\ubbf8\uc9c0\ub77c\ub294 \uaf2c\ub9ac\ud45c\ub97c \ub5bc\uace0 \uac1c\uc131\ud30c \ubc30\uc6b0\ub85c\uc11c \uc7ac\uc870\uba85\ubc1b\uae30 \uc2dc\uc791\ud588\ub2e4. 30\ub300 \uc9c1\uc7a5 \uc5ec\uc131\ub4e4\uc758 \uc77c\uacfc \uc0ac\ub791\uc744 \uadf8\ub9b0 \ub4dc\ub77c\ub9c8\u300a\ub2ec\ucf64\ud55c \ub098\uc758 \ub3c4\uc2dc\u300b\uc5d0\uc11c\ub294 \ud2b8\ub80c\ub514\ud558\uba74\uc11c\ub3c4 \uac1c\uc131 \uc788\ub294 \ubaa8\uc2b5\uc73c\ub85c \uc774\ub978\ubc14 \u2018\ucd5c\uac15\ud76c \uc2a4\ud0c0\uc77c\u2019\uc744 \uc120\ubcf4\uc774\uba70 \ud328\uc154\ub2c8\uc2a4\ud0c0\ub85c\ub3c4 \uc790\ub9ac\ub9e4\uae40\ud588\ub2e4. 2009\ub144 \uc601\ud654\u300a \uc560\uc790 \u300b\ub3c4 \ud765\ud589\uc5d0 \uc131\uacf5\ud558\uba70 \ucda9\ubb34\ub85c \uc6d0\ud1b1 \ubc30\uc6b0\ub85c\uc11c\uc758 \uc785\uc9c0\ub97c \ub2e4\uc84c\ub2e4. 2011\ub144 KBS \uc0c8\ud574 \uac1c\ud3b8\uc73c\ub85c 2011\ub144 1\uc6d4 1\uc77c\ubd80\ud130 \u300a\ucd5c\uac15\ud76c\uc758 \ubcfc\ub968\uc744 \ub192\uc5ec\uc694\u300b\ub97c \ub2e4\uc2dc \uc2dc\uc791\ud588\uace0 \uac00\uc744 \uac1c\ud3b8\uc744 \ud1b5\ud574 2011\ub144 11\uc6d4 \ubd80\ud130\ub294 \u300a\ucd5c\uac15\ud76c\uc758 \uc57c\uac04\ube44\ud589\u300b\uc758 \uc9c4\ud589\uc744 \ub9e1\uc558\uc73c\ub098 2012\ub144 11\uc6d4\uc5d0 \ud558\ucc28\ud588\ub2e4.", "sentence_answer": "2009\ub144 \uc601\ud654\u300a \uc560\uc790 \u300b\ub3c4 \ud765\ud589\uc5d0 \uc131\uacf5\ud558\uba70 \ucda9\ubb34\ub85c \uc6d0\ud1b1 \ubc30\uc6b0\ub85c\uc11c\uc758 \uc785\uc9c0\ub97c \ub2e4\uc84c\ub2e4.", "paragraph_id": "5899613-0-0", "paragraph_question": "question: \ucd5c\uac15\ud76c\uac00 \ucd9c\uc5f0\ud558\uace0 2009\ub144\uc5d0 \uac1c\ubd09\ud55c \uc601\ud654\uc758 \uc774\ub984\uc740?, context: 2006\ub144 \uc601\ud654 \u300a\ub2ec\ucf64, \uc0b4\ubc8c\ud55c \uc5f0\uc778\u300b\uc758 \ud765\ud589\uc73c\ub85c \uc601\ud654\uacc4\uc758 \ube14\ub8e8\uce69\uc73c\ub85c \ub5a0\uc624\ub974\uba70 \uc544\uc5ed \ucd9c\uc2e0, \uccad\uc21c\uac00\ub828 \uc774\ubbf8\uc9c0\ub77c\ub294 \uaf2c\ub9ac\ud45c\ub97c \ub5bc\uace0 \uac1c\uc131\ud30c \ubc30\uc6b0\ub85c\uc11c \uc7ac\uc870\uba85\ubc1b\uae30 \uc2dc\uc791\ud588\ub2e4. 30\ub300 \uc9c1\uc7a5 \uc5ec\uc131\ub4e4\uc758 \uc77c\uacfc \uc0ac\ub791\uc744 \uadf8\ub9b0 \ub4dc\ub77c\ub9c8\u300a\ub2ec\ucf64\ud55c \ub098\uc758 \ub3c4\uc2dc\u300b\uc5d0\uc11c\ub294 \ud2b8\ub80c\ub514\ud558\uba74\uc11c\ub3c4 \uac1c\uc131 \uc788\ub294 \ubaa8\uc2b5\uc73c\ub85c \uc774\ub978\ubc14 \u2018\ucd5c\uac15\ud76c \uc2a4\ud0c0\uc77c\u2019\uc744 \uc120\ubcf4\uc774\uba70 \ud328\uc154\ub2c8\uc2a4\ud0c0\ub85c\ub3c4 \uc790\ub9ac\ub9e4\uae40\ud588\ub2e4. 2009\ub144 \uc601\ud654\u300a\uc560\uc790\u300b\ub3c4 \ud765\ud589\uc5d0 \uc131\uacf5\ud558\uba70 \ucda9\ubb34\ub85c \uc6d0\ud1b1 \ubc30\uc6b0\ub85c\uc11c\uc758 \uc785\uc9c0\ub97c \ub2e4\uc84c\ub2e4. 2011\ub144 KBS \uc0c8\ud574 \uac1c\ud3b8\uc73c\ub85c 2011\ub144 1\uc6d4 1\uc77c\ubd80\ud130 \u300a\ucd5c\uac15\ud76c\uc758 \ubcfc\ub968\uc744 \ub192\uc5ec\uc694\u300b\ub97c \ub2e4\uc2dc \uc2dc\uc791\ud588\uace0 \uac00\uc744 \uac1c\ud3b8\uc744 \ud1b5\ud574 2011\ub144 11\uc6d4 \ubd80\ud130\ub294 \u300a\ucd5c\uac15\ud76c\uc758 \uc57c\uac04\ube44\ud589\u300b\uc758 \uc9c4\ud589\uc744 \ub9e1\uc558\uc73c\ub098 2012\ub144 11\uc6d4\uc5d0 \ud558\ucc28\ud588\ub2e4."} {"question": "2011\ub144 7\uc6d4, \ucf00\uc774\ud2f0 \ud398\ub9ac\uac00 \uc2a4\uba38\ud398\ud2b8 \uc131\uc6b0 \uc5ed\ud560\ub85c \uc601\ud654\uacc4\uc5d0 \ub370\ubdd4\ud560 \uc218 \uc788\uac8c \ub9cc\ub4e0 \uc601\ud654\uc758 \uc81c\ubaa9\uc740 \ubb34\uc5c7\uc785\ub2c8\uae4c?", "paragraph": "2011\ub144 7\uc6d4 \ud398\ub9ac\ub294 3D \ud328\ubc00\ub9ac \uc601\ud654 \u300a\uac1c\uad6c\uc7c1\uc774 \uc2a4\uba38\ud504\u300b\uc5d0\uc11c \uc2a4\uba38\ud398\ud2b8 \uc131\uc6b0 \uc5ed\ud560\ub85c \uc601\ud654\uacc4\uc5d0 \ub370\ubdd4\ud588\ub294\ub370, \uc601\ud654\ub294 \uc138\uacc4\uc801\uc73c\ub85c 557,771,535\ub9cc \ub2ec\ub7ec\uc758 \uc218\uc775\uc744 \ub0a8\uacbc\ub2e4. \ud398\ub9ac\ub294 2011\ub144 8\uc6d4 28\uc77c \uc5f4\ub9b0 MTV \ube44\ub514\uc624 \ubba4\uc9c1 \uc5b4\uc6cc\ub4dc\uc5d0 \ucc38\uc11d\ud574 \"Firework\"\uc73c\ub85c \uc62c\ud574\uc758 \ube44\ub514\uc624, \"E.T.\"\ub85c \ucd5c\uc6b0\uc218 \ucf5c\ub77c\ubcf4\ub808\uc774\uc158, \ucd5c\uc6b0\uc218 \uc2a4\ud3d0\uc15c \ud6a8\uacfc\uc758 \ucd1d \uc138 \ubd80\ubb38\uc5d0\uc11c \uc218\uc0c1\uc744 \ud588\ub2e4. 2011\ub144 12\uc6d4 \u300a\ud3ec\ube0c\uc2a4\u300b\ub294 \"2011\ub144 \uac00\uc7a5 \ub9ce\uc740 \uc218\uc775\uc744 \uc62c\ub9b0 \uc5ec\uc790 \uac00\uc218\" \uc21c\uc704\ub97c \ubc1c\ud45c\ud588\uace0, \ud398\ub9ac\ub294 4,400\ub9cc \ub2ec\ub7ec\ub97c \ubc8c\uc5b4\ub4e4\uc5ec \ub808\uc774\ub514 \uac00\uac00(9,000\ub9cc \ub2ec\ub7ec), \ud14c\uc77c\ub7ec \uc2a4\uc704\ud504\ud2b8(4,500\ub9cc \ub2ec\ub7ec)\uc5d0 \uc774\uc5b4 3\uc704\uc5d0 \uc62c\ub790\ub2e4. 2011\ub144 12\uc6d4 15\uc77c\uc5d0\ub294 \"\ud398\ub9ac\ub294 \uc62c \ud55c\ud574 \ub208\ubd80\uc2e0 \ud65c\uc57d\uc744 \ud3bc\uccd0 \uc74c\uc545\uc801 \uc131\uacf5\uc744 \uac70\ub480\ub2e4\"\uba70 MTV\uc5d0\uc11c \uc120\uc815\ud55c \uc62c\ud574\uc758 \uc544\ud2f0\uc2a4\ud2b8\ub85c \uc120\uc815\ub418\uc5c8\ub2e4. \ud398\ub9ac\ub294 2012\ub144 1\uc6d4 \ub9c8\uc9c0\ub9c9 \uc8fc\uc5d0 \ubbf8\uad6d\uc5d0\uc11c \ub124 \uace1\uc774\ub098 500\ub9cc \uac74\uc758 \ub514\uc9c0\ud138 \ud310\ub9e4\ub97c \uae30\ub85d\ud55c \uccab \uc544\ud2f0\uc2a4\ud2b8\uac00 \ub418\uc5c8\ub2e4(\"Hot N Cold\", \"California Gurls\", \"Firework\", \"E.T.\"). 2012\ub144 1\uc6d4 16\uc77c \ud398\ub9ac\ub294 \uc2ec\uc988 3\uc758 \ud655\uc7a5\ud329\uc778 \"\uc2ec\uc988 3: \ub450\uadfc\ub450\uadfc \uc1fc\ud0c0\uc784\"\uc758 \ud64d\ubcf4\ub97c \uc704\ud574 EA \uac8c\uc784\uc988\uc5d0 \ucc44\uc6a9\ub418\uc5c8\ub2e4. \uc774 \ud655\uc7a5\ud329\uc740 \ud398\ub9ac\uac00 \uc9c1\uc811 \ucc38\uc5ec\ud558\uae30\ub3c4 \ud588\uc73c\uba70, 3\uc6d4 6\uc77c \ucd9c\uc2dc\ub418\uc5c8\ub2e4. 2012\ub144 2\uc6d4 \ud398\ub9ac\ub294 \ub9ac\ud328\ud0a4\uc9c0 \uc568\ubc94\uc758 \uccab \uc2f1\uae00 \"Part of Me\"\ub97c \ubc1c\ub9e4\ud588\ub2e4. \"Part of Me\"\ub294 \ube4c\ubcf4\ub4dc \ud56b 100 1\uc704\ub85c \ub370\ubdd4\ud574 \ud398\ub9ac\uc758 7\ubc88\uc9f8 1\uc704 \uc2f1\uae00\uc774\uc790 20\ubc88\uc9f8 1\uc704 \ub370\ubdd4 \uc2f1\uae00\uc774 \ub418\uc5c8\ub2e4. \ub610\ud55c \uce5c\uad6c \ub9ac\ud55c\ub098\uc640 \ud568\uaed8 6\uace1\uc758 \ub178\ub798\ub97c 1\uc704\uc5d0 \uc62c\ub824 2010\ub144\ub300\uc5d0 \ub4e4\uc5b4 \ud604\uc7ac\uae4c\uc9c0 \uac00\uc7a5 \ub9ce\uc740 1\uc704 \uc2f1\uae00\uc744 \ubc30\ucd9c\ud55c \uc544\ud2f0\uc2a4\ud2b8\uc774\ub2e4. 2012\ub144 3\uc6d4 27\uc77c\uc5d0\ub294 \ub450 \ubc88\uc9f8 \uc568\ubc94\uc758 \ub9ac\ud328\uc9c0\ud0a4 \uc568\ubc94\uc778 Teenage Dream: The Complete Confection\uc774 \ubc1c\ub9e4\ub418\uc5c8\ub2e4. \ub9ac\ud328\ud0a4\uc9c0 \uc568\ubc94\uc758 \ub9ac\ub4dc \uc2f1\uae00 \"Part of Me\"\ub294 2012\ub144 2\uc6d4 \ubc1c\ub9e4\ub418\uc5c8\uace0, \ube4c\ubcf4\ub4dc \ud56b 100 1\uc704\uc5d0 \uc624\ub974\uba70 \ud398\ub9ac\uc758 \uc77c\uacf1 \ubc88\uc9f8 1\uc704 \uc2f1\uae00\uc774\uc790 \uc568\ubc94\uc758 \uc5ec\uc12f \ubc88\uc9f8 1\uc704 \uc2f1\uae00\uc774 \ub418\uc5c8\ub2e4. 2012\ub144 7\uc6d4 5\uc77c\uc5d0\ub294 \uc790\uc2e0\uc758 \uc804\uae30 \uc601\ud654\uc774\uc790 \ucf58\uc11c\ud2b8 \uc601\ud654 \u300a\ucf00\uc774\ud2f0 \ud398\ub9ac: \ud30c\ud2b8 \uc624\ube0c \ubbf8\u300b\uac00 \ud30c\ub77c\ub9c8\uc6b4\ud2b8 \ud53d\ucc98\uc2a4\ub97c \ud1b5\ud574 \uac1c\ubd09\ub418\uc5c8\ub2e4.", "answer": "\uac1c\uad6c\uc7c1\uc774 \uc2a4\uba38\ud504", "sentence": "2011\ub144 7\uc6d4 \ud398\ub9ac\ub294 3D \ud328\ubc00\ub9ac \uc601\ud654 \u300a \uac1c\uad6c\uc7c1\uc774 \uc2a4\uba38\ud504 \u300b\uc5d0\uc11c \uc2a4\uba38\ud398\ud2b8 \uc131\uc6b0 \uc5ed\ud560\ub85c \uc601\ud654\uacc4\uc5d0 \ub370\ubdd4\ud588\ub294\ub370, \uc601\ud654\ub294 \uc138\uacc4\uc801\uc73c\ub85c 557,771,535\ub9cc \ub2ec\ub7ec\uc758 \uc218\uc775\uc744 \ub0a8\uacbc\ub2e4.", "paragraph_sentence": " 2011\ub144 7\uc6d4 \ud398\ub9ac\ub294 3D \ud328\ubc00\ub9ac \uc601\ud654 \u300a \uac1c\uad6c\uc7c1\uc774 \uc2a4\uba38\ud504 \u300b\uc5d0\uc11c \uc2a4\uba38\ud398\ud2b8 \uc131\uc6b0 \uc5ed\ud560\ub85c \uc601\ud654\uacc4\uc5d0 \ub370\ubdd4\ud588\ub294\ub370, \uc601\ud654\ub294 \uc138\uacc4\uc801\uc73c\ub85c 557,771,535\ub9cc \ub2ec\ub7ec\uc758 \uc218\uc775\uc744 \ub0a8\uacbc\ub2e4. \ud398\ub9ac\ub294 2011\ub144 8\uc6d4 28\uc77c \uc5f4\ub9b0 MTV \ube44\ub514\uc624 \ubba4\uc9c1 \uc5b4\uc6cc\ub4dc\uc5d0 \ucc38\uc11d\ud574 \"Firework\"\uc73c\ub85c \uc62c\ud574\uc758 \ube44\ub514\uc624, \"E.T.\"\ub85c \ucd5c\uc6b0\uc218 \ucf5c\ub77c\ubcf4\ub808\uc774\uc158, \ucd5c\uc6b0\uc218 \uc2a4\ud3d0\uc15c \ud6a8\uacfc\uc758 \ucd1d \uc138 \ubd80\ubb38\uc5d0\uc11c \uc218\uc0c1\uc744 \ud588\ub2e4. 2011\ub144 12\uc6d4 \u300a\ud3ec\ube0c\uc2a4\u300b\ub294 \"2011\ub144 \uac00\uc7a5 \ub9ce\uc740 \uc218\uc775\uc744 \uc62c\ub9b0 \uc5ec\uc790 \uac00\uc218\" \uc21c\uc704\ub97c \ubc1c\ud45c\ud588\uace0, \ud398\ub9ac\ub294 4,400\ub9cc \ub2ec\ub7ec\ub97c \ubc8c\uc5b4\ub4e4\uc5ec \ub808\uc774\ub514 \uac00\uac00(9,000\ub9cc \ub2ec\ub7ec), \ud14c\uc77c\ub7ec \uc2a4\uc704\ud504\ud2b8(4,500\ub9cc \ub2ec\ub7ec)\uc5d0 \uc774\uc5b4 3\uc704\uc5d0 \uc62c\ub790\ub2e4. 2011\ub144 12\uc6d4 15\uc77c\uc5d0\ub294 \"\ud398\ub9ac\ub294 \uc62c \ud55c\ud574 \ub208\ubd80\uc2e0 \ud65c\uc57d\uc744 \ud3bc\uccd0 \uc74c\uc545\uc801 \uc131\uacf5\uc744 \uac70\ub480\ub2e4\"\uba70 MTV\uc5d0\uc11c \uc120\uc815\ud55c \uc62c\ud574\uc758 \uc544\ud2f0\uc2a4\ud2b8\ub85c \uc120\uc815\ub418\uc5c8\ub2e4. \ud398\ub9ac\ub294 2012\ub144 1\uc6d4 \ub9c8\uc9c0\ub9c9 \uc8fc\uc5d0 \ubbf8\uad6d\uc5d0\uc11c \ub124 \uace1\uc774\ub098 500\ub9cc \uac74\uc758 \ub514\uc9c0\ud138 \ud310\ub9e4\ub97c \uae30\ub85d\ud55c \uccab \uc544\ud2f0\uc2a4\ud2b8\uac00 \ub418\uc5c8\ub2e4(\"Hot N Cold\", \"California Gurls\", \"Firework\", \"E.T.\"). 2012\ub144 1\uc6d4 16\uc77c \ud398\ub9ac\ub294 \uc2ec\uc988 3\uc758 \ud655\uc7a5\ud329\uc778 \"\uc2ec\uc988 3: \ub450\uadfc\ub450\uadfc \uc1fc\ud0c0\uc784\"\uc758 \ud64d\ubcf4\ub97c \uc704\ud574 EA \uac8c\uc784\uc988\uc5d0 \ucc44\uc6a9\ub418\uc5c8\ub2e4. \uc774 \ud655\uc7a5\ud329\uc740 \ud398\ub9ac\uac00 \uc9c1\uc811 \ucc38\uc5ec\ud558\uae30\ub3c4 \ud588\uc73c\uba70, 3\uc6d4 6\uc77c \ucd9c\uc2dc\ub418\uc5c8\ub2e4. 2012\ub144 2\uc6d4 \ud398\ub9ac\ub294 \ub9ac\ud328\ud0a4\uc9c0 \uc568\ubc94\uc758 \uccab \uc2f1\uae00 \"Part of Me\"\ub97c \ubc1c\ub9e4\ud588\ub2e4. \"Part of Me\"\ub294 \ube4c\ubcf4\ub4dc \ud56b 100 1\uc704\ub85c \ub370\ubdd4\ud574 \ud398\ub9ac\uc758 7\ubc88\uc9f8 1\uc704 \uc2f1\uae00\uc774\uc790 20\ubc88\uc9f8 1\uc704 \ub370\ubdd4 \uc2f1\uae00\uc774 \ub418\uc5c8\ub2e4. \ub610\ud55c \uce5c\uad6c \ub9ac\ud55c\ub098\uc640 \ud568\uaed8 6\uace1\uc758 \ub178\ub798\ub97c 1\uc704\uc5d0 \uc62c\ub824 2010\ub144\ub300\uc5d0 \ub4e4\uc5b4 \ud604\uc7ac\uae4c\uc9c0 \uac00\uc7a5 \ub9ce\uc740 1\uc704 \uc2f1\uae00\uc744 \ubc30\ucd9c\ud55c \uc544\ud2f0\uc2a4\ud2b8\uc774\ub2e4. 2012\ub144 3\uc6d4 27\uc77c\uc5d0\ub294 \ub450 \ubc88\uc9f8 \uc568\ubc94\uc758 \ub9ac\ud328\uc9c0\ud0a4 \uc568\ubc94\uc778 Teenage Dream: The Complete Confection\uc774 \ubc1c\ub9e4\ub418\uc5c8\ub2e4. \ub9ac\ud328\ud0a4\uc9c0 \uc568\ubc94\uc758 \ub9ac\ub4dc \uc2f1\uae00 \"Part of Me\"\ub294 2012\ub144 2\uc6d4 \ubc1c\ub9e4\ub418\uc5c8\uace0, \ube4c\ubcf4\ub4dc \ud56b 100 1\uc704\uc5d0 \uc624\ub974\uba70 \ud398\ub9ac\uc758 \uc77c\uacf1 \ubc88\uc9f8 1\uc704 \uc2f1\uae00\uc774\uc790 \uc568\ubc94\uc758 \uc5ec\uc12f \ubc88\uc9f8 1\uc704 \uc2f1\uae00\uc774 \ub418\uc5c8\ub2e4. 2012\ub144 7\uc6d4 5\uc77c\uc5d0\ub294 \uc790\uc2e0\uc758 \uc804\uae30 \uc601\ud654\uc774\uc790 \ucf58\uc11c\ud2b8 \uc601\ud654 \u300a\ucf00\uc774\ud2f0 \ud398\ub9ac: \ud30c\ud2b8 \uc624\ube0c \ubbf8\u300b\uac00 \ud30c\ub77c\ub9c8\uc6b4\ud2b8 \ud53d\ucc98\uc2a4\ub97c \ud1b5\ud574 \uac1c\ubd09\ub418\uc5c8\ub2e4.", "paragraph_answer": "2011\ub144 7\uc6d4 \ud398\ub9ac\ub294 3D \ud328\ubc00\ub9ac \uc601\ud654 \u300a \uac1c\uad6c\uc7c1\uc774 \uc2a4\uba38\ud504 \u300b\uc5d0\uc11c \uc2a4\uba38\ud398\ud2b8 \uc131\uc6b0 \uc5ed\ud560\ub85c \uc601\ud654\uacc4\uc5d0 \ub370\ubdd4\ud588\ub294\ub370, \uc601\ud654\ub294 \uc138\uacc4\uc801\uc73c\ub85c 557,771,535\ub9cc \ub2ec\ub7ec\uc758 \uc218\uc775\uc744 \ub0a8\uacbc\ub2e4. \ud398\ub9ac\ub294 2011\ub144 8\uc6d4 28\uc77c \uc5f4\ub9b0 MTV \ube44\ub514\uc624 \ubba4\uc9c1 \uc5b4\uc6cc\ub4dc\uc5d0 \ucc38\uc11d\ud574 \"Firework\"\uc73c\ub85c \uc62c\ud574\uc758 \ube44\ub514\uc624, \"E.T.\"\ub85c \ucd5c\uc6b0\uc218 \ucf5c\ub77c\ubcf4\ub808\uc774\uc158, \ucd5c\uc6b0\uc218 \uc2a4\ud3d0\uc15c \ud6a8\uacfc\uc758 \ucd1d \uc138 \ubd80\ubb38\uc5d0\uc11c \uc218\uc0c1\uc744 \ud588\ub2e4. 2011\ub144 12\uc6d4 \u300a\ud3ec\ube0c\uc2a4\u300b\ub294 \"2011\ub144 \uac00\uc7a5 \ub9ce\uc740 \uc218\uc775\uc744 \uc62c\ub9b0 \uc5ec\uc790 \uac00\uc218\" \uc21c\uc704\ub97c \ubc1c\ud45c\ud588\uace0, \ud398\ub9ac\ub294 4,400\ub9cc \ub2ec\ub7ec\ub97c \ubc8c\uc5b4\ub4e4\uc5ec \ub808\uc774\ub514 \uac00\uac00(9,000\ub9cc \ub2ec\ub7ec), \ud14c\uc77c\ub7ec \uc2a4\uc704\ud504\ud2b8(4,500\ub9cc \ub2ec\ub7ec)\uc5d0 \uc774\uc5b4 3\uc704\uc5d0 \uc62c\ub790\ub2e4. 2011\ub144 12\uc6d4 15\uc77c\uc5d0\ub294 \"\ud398\ub9ac\ub294 \uc62c \ud55c\ud574 \ub208\ubd80\uc2e0 \ud65c\uc57d\uc744 \ud3bc\uccd0 \uc74c\uc545\uc801 \uc131\uacf5\uc744 \uac70\ub480\ub2e4\"\uba70 MTV\uc5d0\uc11c \uc120\uc815\ud55c \uc62c\ud574\uc758 \uc544\ud2f0\uc2a4\ud2b8\ub85c \uc120\uc815\ub418\uc5c8\ub2e4. \ud398\ub9ac\ub294 2012\ub144 1\uc6d4 \ub9c8\uc9c0\ub9c9 \uc8fc\uc5d0 \ubbf8\uad6d\uc5d0\uc11c \ub124 \uace1\uc774\ub098 500\ub9cc \uac74\uc758 \ub514\uc9c0\ud138 \ud310\ub9e4\ub97c \uae30\ub85d\ud55c \uccab \uc544\ud2f0\uc2a4\ud2b8\uac00 \ub418\uc5c8\ub2e4(\"Hot N Cold\", \"California Gurls\", \"Firework\", \"E.T.\"). 2012\ub144 1\uc6d4 16\uc77c \ud398\ub9ac\ub294 \uc2ec\uc988 3\uc758 \ud655\uc7a5\ud329\uc778 \"\uc2ec\uc988 3: \ub450\uadfc\ub450\uadfc \uc1fc\ud0c0\uc784\"\uc758 \ud64d\ubcf4\ub97c \uc704\ud574 EA \uac8c\uc784\uc988\uc5d0 \ucc44\uc6a9\ub418\uc5c8\ub2e4. \uc774 \ud655\uc7a5\ud329\uc740 \ud398\ub9ac\uac00 \uc9c1\uc811 \ucc38\uc5ec\ud558\uae30\ub3c4 \ud588\uc73c\uba70, 3\uc6d4 6\uc77c \ucd9c\uc2dc\ub418\uc5c8\ub2e4. 2012\ub144 2\uc6d4 \ud398\ub9ac\ub294 \ub9ac\ud328\ud0a4\uc9c0 \uc568\ubc94\uc758 \uccab \uc2f1\uae00 \"Part of Me\"\ub97c \ubc1c\ub9e4\ud588\ub2e4. \"Part of Me\"\ub294 \ube4c\ubcf4\ub4dc \ud56b 100 1\uc704\ub85c \ub370\ubdd4\ud574 \ud398\ub9ac\uc758 7\ubc88\uc9f8 1\uc704 \uc2f1\uae00\uc774\uc790 20\ubc88\uc9f8 1\uc704 \ub370\ubdd4 \uc2f1\uae00\uc774 \ub418\uc5c8\ub2e4. \ub610\ud55c \uce5c\uad6c \ub9ac\ud55c\ub098\uc640 \ud568\uaed8 6\uace1\uc758 \ub178\ub798\ub97c 1\uc704\uc5d0 \uc62c\ub824 2010\ub144\ub300\uc5d0 \ub4e4\uc5b4 \ud604\uc7ac\uae4c\uc9c0 \uac00\uc7a5 \ub9ce\uc740 1\uc704 \uc2f1\uae00\uc744 \ubc30\ucd9c\ud55c \uc544\ud2f0\uc2a4\ud2b8\uc774\ub2e4. 2012\ub144 3\uc6d4 27\uc77c\uc5d0\ub294 \ub450 \ubc88\uc9f8 \uc568\ubc94\uc758 \ub9ac\ud328\uc9c0\ud0a4 \uc568\ubc94\uc778 Teenage Dream: The Complete Confection\uc774 \ubc1c\ub9e4\ub418\uc5c8\ub2e4. \ub9ac\ud328\ud0a4\uc9c0 \uc568\ubc94\uc758 \ub9ac\ub4dc \uc2f1\uae00 \"Part of Me\"\ub294 2012\ub144 2\uc6d4 \ubc1c\ub9e4\ub418\uc5c8\uace0, \ube4c\ubcf4\ub4dc \ud56b 100 1\uc704\uc5d0 \uc624\ub974\uba70 \ud398\ub9ac\uc758 \uc77c\uacf1 \ubc88\uc9f8 1\uc704 \uc2f1\uae00\uc774\uc790 \uc568\ubc94\uc758 \uc5ec\uc12f \ubc88\uc9f8 1\uc704 \uc2f1\uae00\uc774 \ub418\uc5c8\ub2e4. 2012\ub144 7\uc6d4 5\uc77c\uc5d0\ub294 \uc790\uc2e0\uc758 \uc804\uae30 \uc601\ud654\uc774\uc790 \ucf58\uc11c\ud2b8 \uc601\ud654 \u300a\ucf00\uc774\ud2f0 \ud398\ub9ac: \ud30c\ud2b8 \uc624\ube0c \ubbf8\u300b\uac00 \ud30c\ub77c\ub9c8\uc6b4\ud2b8 \ud53d\ucc98\uc2a4\ub97c \ud1b5\ud574 \uac1c\ubd09\ub418\uc5c8\ub2e4.", "sentence_answer": "2011\ub144 7\uc6d4 \ud398\ub9ac\ub294 3D \ud328\ubc00\ub9ac \uc601\ud654 \u300a \uac1c\uad6c\uc7c1\uc774 \uc2a4\uba38\ud504 \u300b\uc5d0\uc11c \uc2a4\uba38\ud398\ud2b8 \uc131\uc6b0 \uc5ed\ud560\ub85c \uc601\ud654\uacc4\uc5d0 \ub370\ubdd4\ud588\ub294\ub370, \uc601\ud654\ub294 \uc138\uacc4\uc801\uc73c\ub85c 557,771,535\ub9cc \ub2ec\ub7ec\uc758 \uc218\uc775\uc744 \ub0a8\uacbc\ub2e4.", "paragraph_id": "6561799-13-0", "paragraph_question": "question: 2011\ub144 7\uc6d4, \ucf00\uc774\ud2f0 \ud398\ub9ac\uac00 \uc2a4\uba38\ud398\ud2b8 \uc131\uc6b0 \uc5ed\ud560\ub85c \uc601\ud654\uacc4\uc5d0 \ub370\ubdd4\ud560 \uc218 \uc788\uac8c \ub9cc\ub4e0 \uc601\ud654\uc758 \uc81c\ubaa9\uc740 \ubb34\uc5c7\uc785\ub2c8\uae4c?, context: 2011\ub144 7\uc6d4 \ud398\ub9ac\ub294 3D \ud328\ubc00\ub9ac \uc601\ud654 \u300a\uac1c\uad6c\uc7c1\uc774 \uc2a4\uba38\ud504\u300b\uc5d0\uc11c \uc2a4\uba38\ud398\ud2b8 \uc131\uc6b0 \uc5ed\ud560\ub85c \uc601\ud654\uacc4\uc5d0 \ub370\ubdd4\ud588\ub294\ub370, \uc601\ud654\ub294 \uc138\uacc4\uc801\uc73c\ub85c 557,771,535\ub9cc \ub2ec\ub7ec\uc758 \uc218\uc775\uc744 \ub0a8\uacbc\ub2e4. \ud398\ub9ac\ub294 2011\ub144 8\uc6d4 28\uc77c \uc5f4\ub9b0 MTV \ube44\ub514\uc624 \ubba4\uc9c1 \uc5b4\uc6cc\ub4dc\uc5d0 \ucc38\uc11d\ud574 \"Firework\"\uc73c\ub85c \uc62c\ud574\uc758 \ube44\ub514\uc624, \"E.T.\"\ub85c \ucd5c\uc6b0\uc218 \ucf5c\ub77c\ubcf4\ub808\uc774\uc158, \ucd5c\uc6b0\uc218 \uc2a4\ud3d0\uc15c \ud6a8\uacfc\uc758 \ucd1d \uc138 \ubd80\ubb38\uc5d0\uc11c \uc218\uc0c1\uc744 \ud588\ub2e4. 2011\ub144 12\uc6d4 \u300a\ud3ec\ube0c\uc2a4\u300b\ub294 \"2011\ub144 \uac00\uc7a5 \ub9ce\uc740 \uc218\uc775\uc744 \uc62c\ub9b0 \uc5ec\uc790 \uac00\uc218\" \uc21c\uc704\ub97c \ubc1c\ud45c\ud588\uace0, \ud398\ub9ac\ub294 4,400\ub9cc \ub2ec\ub7ec\ub97c \ubc8c\uc5b4\ub4e4\uc5ec \ub808\uc774\ub514 \uac00\uac00(9,000\ub9cc \ub2ec\ub7ec), \ud14c\uc77c\ub7ec \uc2a4\uc704\ud504\ud2b8(4,500\ub9cc \ub2ec\ub7ec)\uc5d0 \uc774\uc5b4 3\uc704\uc5d0 \uc62c\ub790\ub2e4. 2011\ub144 12\uc6d4 15\uc77c\uc5d0\ub294 \"\ud398\ub9ac\ub294 \uc62c \ud55c\ud574 \ub208\ubd80\uc2e0 \ud65c\uc57d\uc744 \ud3bc\uccd0 \uc74c\uc545\uc801 \uc131\uacf5\uc744 \uac70\ub480\ub2e4\"\uba70 MTV\uc5d0\uc11c \uc120\uc815\ud55c \uc62c\ud574\uc758 \uc544\ud2f0\uc2a4\ud2b8\ub85c \uc120\uc815\ub418\uc5c8\ub2e4. \ud398\ub9ac\ub294 2012\ub144 1\uc6d4 \ub9c8\uc9c0\ub9c9 \uc8fc\uc5d0 \ubbf8\uad6d\uc5d0\uc11c \ub124 \uace1\uc774\ub098 500\ub9cc \uac74\uc758 \ub514\uc9c0\ud138 \ud310\ub9e4\ub97c \uae30\ub85d\ud55c \uccab \uc544\ud2f0\uc2a4\ud2b8\uac00 \ub418\uc5c8\ub2e4(\"Hot N Cold\", \"California Gurls\", \"Firework\", \"E.T.\"). 2012\ub144 1\uc6d4 16\uc77c \ud398\ub9ac\ub294 \uc2ec\uc988 3\uc758 \ud655\uc7a5\ud329\uc778 \"\uc2ec\uc988 3: \ub450\uadfc\ub450\uadfc \uc1fc\ud0c0\uc784\"\uc758 \ud64d\ubcf4\ub97c \uc704\ud574 EA \uac8c\uc784\uc988\uc5d0 \ucc44\uc6a9\ub418\uc5c8\ub2e4. \uc774 \ud655\uc7a5\ud329\uc740 \ud398\ub9ac\uac00 \uc9c1\uc811 \ucc38\uc5ec\ud558\uae30\ub3c4 \ud588\uc73c\uba70, 3\uc6d4 6\uc77c \ucd9c\uc2dc\ub418\uc5c8\ub2e4. 2012\ub144 2\uc6d4 \ud398\ub9ac\ub294 \ub9ac\ud328\ud0a4\uc9c0 \uc568\ubc94\uc758 \uccab \uc2f1\uae00 \"Part of Me\"\ub97c \ubc1c\ub9e4\ud588\ub2e4. \"Part of Me\"\ub294 \ube4c\ubcf4\ub4dc \ud56b 100 1\uc704\ub85c \ub370\ubdd4\ud574 \ud398\ub9ac\uc758 7\ubc88\uc9f8 1\uc704 \uc2f1\uae00\uc774\uc790 20\ubc88\uc9f8 1\uc704 \ub370\ubdd4 \uc2f1\uae00\uc774 \ub418\uc5c8\ub2e4. \ub610\ud55c \uce5c\uad6c \ub9ac\ud55c\ub098\uc640 \ud568\uaed8 6\uace1\uc758 \ub178\ub798\ub97c 1\uc704\uc5d0 \uc62c\ub824 2010\ub144\ub300\uc5d0 \ub4e4\uc5b4 \ud604\uc7ac\uae4c\uc9c0 \uac00\uc7a5 \ub9ce\uc740 1\uc704 \uc2f1\uae00\uc744 \ubc30\ucd9c\ud55c \uc544\ud2f0\uc2a4\ud2b8\uc774\ub2e4. 2012\ub144 3\uc6d4 27\uc77c\uc5d0\ub294 \ub450 \ubc88\uc9f8 \uc568\ubc94\uc758 \ub9ac\ud328\uc9c0\ud0a4 \uc568\ubc94\uc778 Teenage Dream: The Complete Confection\uc774 \ubc1c\ub9e4\ub418\uc5c8\ub2e4. \ub9ac\ud328\ud0a4\uc9c0 \uc568\ubc94\uc758 \ub9ac\ub4dc \uc2f1\uae00 \"Part of Me\"\ub294 2012\ub144 2\uc6d4 \ubc1c\ub9e4\ub418\uc5c8\uace0, \ube4c\ubcf4\ub4dc \ud56b 100 1\uc704\uc5d0 \uc624\ub974\uba70 \ud398\ub9ac\uc758 \uc77c\uacf1 \ubc88\uc9f8 1\uc704 \uc2f1\uae00\uc774\uc790 \uc568\ubc94\uc758 \uc5ec\uc12f \ubc88\uc9f8 1\uc704 \uc2f1\uae00\uc774 \ub418\uc5c8\ub2e4. 2012\ub144 7\uc6d4 5\uc77c\uc5d0\ub294 \uc790\uc2e0\uc758 \uc804\uae30 \uc601\ud654\uc774\uc790 \ucf58\uc11c\ud2b8 \uc601\ud654 \u300a\ucf00\uc774\ud2f0 \ud398\ub9ac: \ud30c\ud2b8 \uc624\ube0c \ubbf8\u300b\uac00 \ud30c\ub77c\ub9c8\uc6b4\ud2b8 \ud53d\ucc98\uc2a4\ub97c \ud1b5\ud574 \uac1c\ubd09\ub418\uc5c8\ub2e4."} {"question": "\ub9e8\ud574\ud2bc \uacc4\ud68d \ub2f9\uc2dc \uc804\uc2dc \uad6d\ubb34 \ucc28\uad00\uc774\uc790 \ubbf8\uc721\uad70 \uacf5\ubcd1\uc0ac\ub839\ubd80\uc758 \uc9c0\ud718\uad8c\uc744 \uac00\uc9c0\uace0 \uc788\uc5c8\ub358 \uc0ac\ub78c\uc740?", "paragraph": "\uadf8\ub85c\ube0c\uc2a4\ub294 \ub9e8\ud574\ud2bc \uacc4\ud68d\uc758 \ucc45\uc784\uc790\uac00 \ub41c \uc9c1\ud6c4 \ud14c\ub124\uc2dc \uc8fc\ub85c \uac00\uc11c \uacf5\uc7a5 \uac74\uc124 \ud6c4\ubcf4\uc9c0\ub97c \uac80\ud1a0\ud558\uc600\uace0, \ub9c8\uc74c\uc5d0 \ub4e4\uc5b4 \ud558\uc600\ub2e4. 9\uc6d4 29\uc77c, \uc804\uc2dc \uad6d\ubb34 \ucc28\uad00\uc774\uc790 \ubbf8\uc721\uad70 \uacf5\ubcd1\uc0ac\ub839\ubd80\uc758 \uc9c0\ud718\uad8c\uc744 \uac16\uace0 \uc788\uc5c8\ub358 \ub85c\ubc84\ud2b8 \ud398\ud130\uc2a8\uc740 3\ubc3150\ub9cc \ub2ec\ub7ec\ub97c \ub4e4\uc5ec 23,000 \ud5e5\ud0c0\ub974\uc758 \ubd80\uc9c0\ub97c \ub9c8\ub828\ud558\ub294 \uc785\uc548\uc744 \uc2b9\uc778\ud558\uc600\ub2e4. \uc2e4\uc81c \uacf5\uc7a5\uc740 12,000 \ud5e5\ud0c0\ub974\uac00 \ub354 \ucd94\uac00\ub418\uc5c8\uace0, 10\uc6d4 7\uc77c 1\ucc9c\uc5ec \uc138\ub300\uc5d0 \ud574\ub2f9 \uc9c0\uc5ed\uc5d0\uc11c \uc774\uc8fc\ud558\ub77c\ub294 \uba85\ub839\uc774 \ubc1c\ud45c\ub418\uc5c8\ub2e4. \uc9c0\uc5ed \uc8fc\ubbfc\ub4e4\uc740 \uc774\ub7ec\ud55c \uba85\ub839\uc5d0 \ud56d\uc758\ud558\uc600\uc73c\ub098 1943\ub144 \uc758\ud68c\ub294 \ud574\ub2f9 \uc9c0\uc5ed \uc0ac\uc6a9\uc744 \uc758\uacb0\ud558\uc600\ub2e4. 11\uc6d4 \uc911\uc21c \ubbf8\uad6d \uc5f0\ubc29\ubcf4\uc548\uad00\uc740 \uc6a9\uc5ed\uc744 \uace0\uc6a9\ud558\uc5ec \ub18d\uc7a5\uc744 \ud3d0\uc1c4\ud558\uace0 \uc9c0\uc5ed\ubbfc\uc744 \ucd94\ubc29\ud558\uc600\ub2e4. \uc77c\ubd80 \uc138\ub300\uc5d0\ub294 2\uc8fc\uc548\uc5d0 \uc774\uc8fc\ud558\ub77c\ub294 \ud1b5\ubcf4\uac00 \uc804\ub2ec\ub418\uae30\ub3c4 \ud558\uc600\ub2e4. \uc624\ud06c\ub9ac\uc9c0\uc5d0 \uac70\uc8fc\ud558\ub358 \uc138\ub300\ub4e4 \uac00\uc6b4\ub370 \uc0c1\ub2f9 \uc218\ub294 \uc774\ubbf8 1920\ub144\ub300 \uadf8\ub798\uc774\ud2b8 \uc2a4\ubaa8\ud0a4 \ub9c8\uc6b4\ud2f4 \uad6d\ub9bd\uacf5\uc6d0\uc758 \uc124\uc815 \uacfc\uc815\uacfc 1930\ub144\ub300 \ub178\ub9ac\uc2a4 \ub310\uc758 \uac74\uc124 \uacfc\uc815\uc5d0\uc11c \uace0\ud5a5\uc744 \ub5a0\ub098\uc57c \ud588\ub358 \uc774\uc8fc\ubbfc\ub4e4\uc774\uc5c8\ub2e4. 1945\ub144 3\uc6d4\uae4c\uc9c0 \ud1a0\uc9c0\ubcf4\uc0c1\uae08\uc73c\ub85c 260\ub9cc \ub2ec\ub7ec\uac00 \uc9c0\uae09\ub418\uc5c8\uc73c\uba70, \uc774\uac83\uc740 1 \uc5d0\uc774\ucee4 \ub2f9 47 \ub2ec\ub7ec\uc600\ub2e4. \uc624\ud06c\ub9ac\uc9c0\uc758 \ubaa8\ub4e0 \uc9c0\uc5ed\uc5d0\uc11c \uad70\ub2f9\uad6d\uc758 \ud5c8\uac00\ub97c \ubc1b\uc9c0 \ubabb\ud55c \uc138\ub300\ub294 \uac70\uc8fc\ud560 \uc218 \uc5c6\ub2e4\ub294 \uacf5\uacf5 \uba85\ub839 2\ud638\uac00 \uacf5\ud45c\ub418\uc790, \ud14c\ub124\uc2dc \uc8fc\uc9c0\uc0ac \ud504\ub80c\ud2f0\uc2a4 \ucfe0\ud37c\ub294 \uc774\ub97c \ub9f9\ub82c\ud788 \ube44\ub09c\ud558\uc600\ub2e4.", "answer": "\ub85c\ubc84\ud2b8 \ud398\ud130\uc2a8", "sentence": "9\uc6d4 29\uc77c, \uc804\uc2dc \uad6d\ubb34 \ucc28\uad00\uc774\uc790 \ubbf8\uc721\uad70 \uacf5\ubcd1\uc0ac\ub839\ubd80\uc758 \uc9c0\ud718\uad8c\uc744 \uac16\uace0 \uc788\uc5c8\ub358 \ub85c\ubc84\ud2b8 \ud398\ud130\uc2a8 \uc740 3\ubc3150\ub9cc \ub2ec\ub7ec\ub97c \ub4e4\uc5ec 23,000 \ud5e5\ud0c0\ub974\uc758 \ubd80\uc9c0\ub97c \ub9c8\ub828\ud558\ub294 \uc785\uc548\uc744 \uc2b9\uc778\ud558\uc600\ub2e4.", "paragraph_sentence": "\uadf8\ub85c\ube0c\uc2a4\ub294 \ub9e8\ud574\ud2bc \uacc4\ud68d\uc758 \ucc45\uc784\uc790\uac00 \ub41c \uc9c1\ud6c4 \ud14c\ub124\uc2dc \uc8fc\ub85c \uac00\uc11c \uacf5\uc7a5 \uac74\uc124 \ud6c4\ubcf4\uc9c0\ub97c \uac80\ud1a0\ud558\uc600\uace0, \ub9c8\uc74c\uc5d0 \ub4e4\uc5b4 \ud558\uc600\ub2e4. 9\uc6d4 29\uc77c, \uc804\uc2dc \uad6d\ubb34 \ucc28\uad00\uc774\uc790 \ubbf8\uc721\uad70 \uacf5\ubcd1\uc0ac\ub839\ubd80\uc758 \uc9c0\ud718\uad8c\uc744 \uac16\uace0 \uc788\uc5c8\ub358 \ub85c\ubc84\ud2b8 \ud398\ud130\uc2a8 \uc740 3\ubc3150\ub9cc \ub2ec\ub7ec\ub97c \ub4e4\uc5ec 23,000 \ud5e5\ud0c0\ub974\uc758 \ubd80\uc9c0\ub97c \ub9c8\ub828\ud558\ub294 \uc785\uc548\uc744 \uc2b9\uc778\ud558\uc600\ub2e4. \uc2e4\uc81c \uacf5\uc7a5\uc740 12,000 \ud5e5\ud0c0\ub974\uac00 \ub354 \ucd94\uac00\ub418\uc5c8\uace0, 10\uc6d4 7\uc77c 1\ucc9c\uc5ec \uc138\ub300\uc5d0 \ud574\ub2f9 \uc9c0\uc5ed\uc5d0\uc11c \uc774\uc8fc\ud558\ub77c\ub294 \uba85\ub839\uc774 \ubc1c\ud45c\ub418\uc5c8\ub2e4. \uc9c0\uc5ed \uc8fc\ubbfc\ub4e4\uc740 \uc774\ub7ec\ud55c \uba85\ub839\uc5d0 \ud56d\uc758\ud558\uc600\uc73c\ub098 1943\ub144 \uc758\ud68c\ub294 \ud574\ub2f9 \uc9c0\uc5ed \uc0ac\uc6a9\uc744 \uc758\uacb0\ud558\uc600\ub2e4. 11\uc6d4 \uc911\uc21c \ubbf8\uad6d \uc5f0\ubc29\ubcf4\uc548\uad00\uc740 \uc6a9\uc5ed\uc744 \uace0\uc6a9\ud558\uc5ec \ub18d\uc7a5\uc744 \ud3d0\uc1c4\ud558\uace0 \uc9c0\uc5ed\ubbfc\uc744 \ucd94\ubc29\ud558\uc600\ub2e4. \uc77c\ubd80 \uc138\ub300\uc5d0\ub294 2\uc8fc\uc548\uc5d0 \uc774\uc8fc\ud558\ub77c\ub294 \ud1b5\ubcf4\uac00 \uc804\ub2ec\ub418\uae30\ub3c4 \ud558\uc600\ub2e4. \uc624\ud06c\ub9ac\uc9c0\uc5d0 \uac70\uc8fc\ud558\ub358 \uc138\ub300\ub4e4 \uac00\uc6b4\ub370 \uc0c1\ub2f9 \uc218\ub294 \uc774\ubbf8 1920\ub144\ub300 \uadf8\ub798\uc774\ud2b8 \uc2a4\ubaa8\ud0a4 \ub9c8\uc6b4\ud2f4 \uad6d\ub9bd\uacf5\uc6d0\uc758 \uc124\uc815 \uacfc\uc815\uacfc 1930\ub144\ub300 \ub178\ub9ac\uc2a4 \ub310\uc758 \uac74\uc124 \uacfc\uc815\uc5d0\uc11c \uace0\ud5a5\uc744 \ub5a0\ub098\uc57c \ud588\ub358 \uc774\uc8fc\ubbfc\ub4e4\uc774\uc5c8\ub2e4. 1945\ub144 3\uc6d4\uae4c\uc9c0 \ud1a0\uc9c0\ubcf4\uc0c1\uae08\uc73c\ub85c 260\ub9cc \ub2ec\ub7ec\uac00 \uc9c0\uae09\ub418\uc5c8\uc73c\uba70, \uc774\uac83\uc740 1 \uc5d0\uc774\ucee4 \ub2f9 47 \ub2ec\ub7ec\uc600\ub2e4. \uc624\ud06c\ub9ac\uc9c0\uc758 \ubaa8\ub4e0 \uc9c0\uc5ed\uc5d0\uc11c \uad70\ub2f9\uad6d\uc758 \ud5c8\uac00\ub97c \ubc1b\uc9c0 \ubabb\ud55c \uc138\ub300\ub294 \uac70\uc8fc\ud560 \uc218 \uc5c6\ub2e4\ub294 \uacf5\uacf5 \uba85\ub839 2\ud638\uac00 \uacf5\ud45c\ub418\uc790, \ud14c\ub124\uc2dc \uc8fc\uc9c0\uc0ac \ud504\ub80c\ud2f0\uc2a4 \ucfe0\ud37c\ub294 \uc774\ub97c \ub9f9\ub82c\ud788 \ube44\ub09c\ud558\uc600\ub2e4.", "paragraph_answer": "\uadf8\ub85c\ube0c\uc2a4\ub294 \ub9e8\ud574\ud2bc \uacc4\ud68d\uc758 \ucc45\uc784\uc790\uac00 \ub41c \uc9c1\ud6c4 \ud14c\ub124\uc2dc \uc8fc\ub85c \uac00\uc11c \uacf5\uc7a5 \uac74\uc124 \ud6c4\ubcf4\uc9c0\ub97c \uac80\ud1a0\ud558\uc600\uace0, \ub9c8\uc74c\uc5d0 \ub4e4\uc5b4 \ud558\uc600\ub2e4. 9\uc6d4 29\uc77c, \uc804\uc2dc \uad6d\ubb34 \ucc28\uad00\uc774\uc790 \ubbf8\uc721\uad70 \uacf5\ubcd1\uc0ac\ub839\ubd80\uc758 \uc9c0\ud718\uad8c\uc744 \uac16\uace0 \uc788\uc5c8\ub358 \ub85c\ubc84\ud2b8 \ud398\ud130\uc2a8 \uc740 3\ubc3150\ub9cc \ub2ec\ub7ec\ub97c \ub4e4\uc5ec 23,000 \ud5e5\ud0c0\ub974\uc758 \ubd80\uc9c0\ub97c \ub9c8\ub828\ud558\ub294 \uc785\uc548\uc744 \uc2b9\uc778\ud558\uc600\ub2e4. \uc2e4\uc81c \uacf5\uc7a5\uc740 12,000 \ud5e5\ud0c0\ub974\uac00 \ub354 \ucd94\uac00\ub418\uc5c8\uace0, 10\uc6d4 7\uc77c 1\ucc9c\uc5ec \uc138\ub300\uc5d0 \ud574\ub2f9 \uc9c0\uc5ed\uc5d0\uc11c \uc774\uc8fc\ud558\ub77c\ub294 \uba85\ub839\uc774 \ubc1c\ud45c\ub418\uc5c8\ub2e4. \uc9c0\uc5ed \uc8fc\ubbfc\ub4e4\uc740 \uc774\ub7ec\ud55c \uba85\ub839\uc5d0 \ud56d\uc758\ud558\uc600\uc73c\ub098 1943\ub144 \uc758\ud68c\ub294 \ud574\ub2f9 \uc9c0\uc5ed \uc0ac\uc6a9\uc744 \uc758\uacb0\ud558\uc600\ub2e4. 11\uc6d4 \uc911\uc21c \ubbf8\uad6d \uc5f0\ubc29\ubcf4\uc548\uad00\uc740 \uc6a9\uc5ed\uc744 \uace0\uc6a9\ud558\uc5ec \ub18d\uc7a5\uc744 \ud3d0\uc1c4\ud558\uace0 \uc9c0\uc5ed\ubbfc\uc744 \ucd94\ubc29\ud558\uc600\ub2e4. \uc77c\ubd80 \uc138\ub300\uc5d0\ub294 2\uc8fc\uc548\uc5d0 \uc774\uc8fc\ud558\ub77c\ub294 \ud1b5\ubcf4\uac00 \uc804\ub2ec\ub418\uae30\ub3c4 \ud558\uc600\ub2e4. \uc624\ud06c\ub9ac\uc9c0\uc5d0 \uac70\uc8fc\ud558\ub358 \uc138\ub300\ub4e4 \uac00\uc6b4\ub370 \uc0c1\ub2f9 \uc218\ub294 \uc774\ubbf8 1920\ub144\ub300 \uadf8\ub798\uc774\ud2b8 \uc2a4\ubaa8\ud0a4 \ub9c8\uc6b4\ud2f4 \uad6d\ub9bd\uacf5\uc6d0\uc758 \uc124\uc815 \uacfc\uc815\uacfc 1930\ub144\ub300 \ub178\ub9ac\uc2a4 \ub310\uc758 \uac74\uc124 \uacfc\uc815\uc5d0\uc11c \uace0\ud5a5\uc744 \ub5a0\ub098\uc57c \ud588\ub358 \uc774\uc8fc\ubbfc\ub4e4\uc774\uc5c8\ub2e4. 1945\ub144 3\uc6d4\uae4c\uc9c0 \ud1a0\uc9c0\ubcf4\uc0c1\uae08\uc73c\ub85c 260\ub9cc \ub2ec\ub7ec\uac00 \uc9c0\uae09\ub418\uc5c8\uc73c\uba70, \uc774\uac83\uc740 1 \uc5d0\uc774\ucee4 \ub2f9 47 \ub2ec\ub7ec\uc600\ub2e4. \uc624\ud06c\ub9ac\uc9c0\uc758 \ubaa8\ub4e0 \uc9c0\uc5ed\uc5d0\uc11c \uad70\ub2f9\uad6d\uc758 \ud5c8\uac00\ub97c \ubc1b\uc9c0 \ubabb\ud55c \uc138\ub300\ub294 \uac70\uc8fc\ud560 \uc218 \uc5c6\ub2e4\ub294 \uacf5\uacf5 \uba85\ub839 2\ud638\uac00 \uacf5\ud45c\ub418\uc790, \ud14c\ub124\uc2dc \uc8fc\uc9c0\uc0ac \ud504\ub80c\ud2f0\uc2a4 \ucfe0\ud37c\ub294 \uc774\ub97c \ub9f9\ub82c\ud788 \ube44\ub09c\ud558\uc600\ub2e4.", "sentence_answer": "9\uc6d4 29\uc77c, \uc804\uc2dc \uad6d\ubb34 \ucc28\uad00\uc774\uc790 \ubbf8\uc721\uad70 \uacf5\ubcd1\uc0ac\ub839\ubd80\uc758 \uc9c0\ud718\uad8c\uc744 \uac16\uace0 \uc788\uc5c8\ub358 \ub85c\ubc84\ud2b8 \ud398\ud130\uc2a8 \uc740 3\ubc3150\ub9cc \ub2ec\ub7ec\ub97c \ub4e4\uc5ec 23,000 \ud5e5\ud0c0\ub974\uc758 \ubd80\uc9c0\ub97c \ub9c8\ub828\ud558\ub294 \uc785\uc548\uc744 \uc2b9\uc778\ud558\uc600\ub2e4.", "paragraph_id": "6502200-10-1", "paragraph_question": "question: \ub9e8\ud574\ud2bc \uacc4\ud68d \ub2f9\uc2dc \uc804\uc2dc \uad6d\ubb34 \ucc28\uad00\uc774\uc790 \ubbf8\uc721\uad70 \uacf5\ubcd1\uc0ac\ub839\ubd80\uc758 \uc9c0\ud718\uad8c\uc744 \uac00\uc9c0\uace0 \uc788\uc5c8\ub358 \uc0ac\ub78c\uc740?, context: \uadf8\ub85c\ube0c\uc2a4\ub294 \ub9e8\ud574\ud2bc \uacc4\ud68d\uc758 \ucc45\uc784\uc790\uac00 \ub41c \uc9c1\ud6c4 \ud14c\ub124\uc2dc \uc8fc\ub85c \uac00\uc11c \uacf5\uc7a5 \uac74\uc124 \ud6c4\ubcf4\uc9c0\ub97c \uac80\ud1a0\ud558\uc600\uace0, \ub9c8\uc74c\uc5d0 \ub4e4\uc5b4 \ud558\uc600\ub2e4. 9\uc6d4 29\uc77c, \uc804\uc2dc \uad6d\ubb34 \ucc28\uad00\uc774\uc790 \ubbf8\uc721\uad70 \uacf5\ubcd1\uc0ac\ub839\ubd80\uc758 \uc9c0\ud718\uad8c\uc744 \uac16\uace0 \uc788\uc5c8\ub358 \ub85c\ubc84\ud2b8 \ud398\ud130\uc2a8\uc740 3\ubc3150\ub9cc \ub2ec\ub7ec\ub97c \ub4e4\uc5ec 23,000 \ud5e5\ud0c0\ub974\uc758 \ubd80\uc9c0\ub97c \ub9c8\ub828\ud558\ub294 \uc785\uc548\uc744 \uc2b9\uc778\ud558\uc600\ub2e4. \uc2e4\uc81c \uacf5\uc7a5\uc740 12,000 \ud5e5\ud0c0\ub974\uac00 \ub354 \ucd94\uac00\ub418\uc5c8\uace0, 10\uc6d4 7\uc77c 1\ucc9c\uc5ec \uc138\ub300\uc5d0 \ud574\ub2f9 \uc9c0\uc5ed\uc5d0\uc11c \uc774\uc8fc\ud558\ub77c\ub294 \uba85\ub839\uc774 \ubc1c\ud45c\ub418\uc5c8\ub2e4. \uc9c0\uc5ed \uc8fc\ubbfc\ub4e4\uc740 \uc774\ub7ec\ud55c \uba85\ub839\uc5d0 \ud56d\uc758\ud558\uc600\uc73c\ub098 1943\ub144 \uc758\ud68c\ub294 \ud574\ub2f9 \uc9c0\uc5ed \uc0ac\uc6a9\uc744 \uc758\uacb0\ud558\uc600\ub2e4. 11\uc6d4 \uc911\uc21c \ubbf8\uad6d \uc5f0\ubc29\ubcf4\uc548\uad00\uc740 \uc6a9\uc5ed\uc744 \uace0\uc6a9\ud558\uc5ec \ub18d\uc7a5\uc744 \ud3d0\uc1c4\ud558\uace0 \uc9c0\uc5ed\ubbfc\uc744 \ucd94\ubc29\ud558\uc600\ub2e4. \uc77c\ubd80 \uc138\ub300\uc5d0\ub294 2\uc8fc\uc548\uc5d0 \uc774\uc8fc\ud558\ub77c\ub294 \ud1b5\ubcf4\uac00 \uc804\ub2ec\ub418\uae30\ub3c4 \ud558\uc600\ub2e4. \uc624\ud06c\ub9ac\uc9c0\uc5d0 \uac70\uc8fc\ud558\ub358 \uc138\ub300\ub4e4 \uac00\uc6b4\ub370 \uc0c1\ub2f9 \uc218\ub294 \uc774\ubbf8 1920\ub144\ub300 \uadf8\ub798\uc774\ud2b8 \uc2a4\ubaa8\ud0a4 \ub9c8\uc6b4\ud2f4 \uad6d\ub9bd\uacf5\uc6d0\uc758 \uc124\uc815 \uacfc\uc815\uacfc 1930\ub144\ub300 \ub178\ub9ac\uc2a4 \ub310\uc758 \uac74\uc124 \uacfc\uc815\uc5d0\uc11c \uace0\ud5a5\uc744 \ub5a0\ub098\uc57c \ud588\ub358 \uc774\uc8fc\ubbfc\ub4e4\uc774\uc5c8\ub2e4. 1945\ub144 3\uc6d4\uae4c\uc9c0 \ud1a0\uc9c0\ubcf4\uc0c1\uae08\uc73c\ub85c 260\ub9cc \ub2ec\ub7ec\uac00 \uc9c0\uae09\ub418\uc5c8\uc73c\uba70, \uc774\uac83\uc740 1 \uc5d0\uc774\ucee4 \ub2f9 47 \ub2ec\ub7ec\uc600\ub2e4. \uc624\ud06c\ub9ac\uc9c0\uc758 \ubaa8\ub4e0 \uc9c0\uc5ed\uc5d0\uc11c \uad70\ub2f9\uad6d\uc758 \ud5c8\uac00\ub97c \ubc1b\uc9c0 \ubabb\ud55c \uc138\ub300\ub294 \uac70\uc8fc\ud560 \uc218 \uc5c6\ub2e4\ub294 \uacf5\uacf5 \uba85\ub839 2\ud638\uac00 \uacf5\ud45c\ub418\uc790, \ud14c\ub124\uc2dc \uc8fc\uc9c0\uc0ac \ud504\ub80c\ud2f0\uc2a4 \ucfe0\ud37c\ub294 \uc774\ub97c \ub9f9\ub82c\ud788 \ube44\ub09c\ud558\uc600\ub2e4."} {"question": "2009\ub144\uc5d0 \uc7ac\uc9d1\uacc4\ud55c \uad11\uc8fc \ubbfc\uc8fc\ud654 \uc6b4\ub3d9\uc758 \ud53c\ud574\uc790\ub294 \uba87 \uba85\uc778\uac00?", "paragraph": "5\u00b718 \uad11\uc8fc \ubbfc\uc8fc\ud654 \uc6b4\ub3d9\uc73c\ub85c \uc778\ud55c \uc0ac\ub9dd\uc790 \ubc0f \ud589\ubc29\ubd88\uba85\uc790\ub294 \uc57d 200\uc5ec \uba85\uc774\uace0 \ubd80\uc0c1\uc790 \ub4f1 \ud53c\ud574\uc790\ub294 \uc57d 4,300\uc5ec \uba85\uc774\ub2e4. \uad11\uc8fc\uad11\uc5ed\uc2dc\uac00 2009\ub144\uc5d0 5\u00b718 \uad11\uc8fc \ubbfc\uc8fc\ud654 \uc6b4\ub3d9 29\uc8fc\ub144\uc744 \ub9de\uc544 \ub2f9\uc2dc \ubaa9\uc228\uc744 \uc783\uac70\ub098 \ub2e4\uce5c \uc0ac\ub78c\uc744 \uc9d1\uacc4\ud55c \uacb0\uacfc, \uc0ac\ub9dd\uc790\uac00 163\uba85, \ud589\ubc29\ubd88\uba85\uc790\uac00 166\uba85, \ubd80\uc0c1 \ub4a4 \uc228\uc9c4 \uc0ac\ub78c\uc774 101\uba85, \ubd80\uc0c1\uc790\uac00 3,139\uba85, \uad6c\uc18d \ubc0f \uad6c\uae08 \ub4f1\uc758 \uae30\ud0c0 \ud53c\ud574\uc790 1,589\uba85, \uc544\uc9c1 \uc5f0\uace0\uac00 \ud655\uc778\ub418\uc9c0 \uc54a\uc544 \ubb18\ube44\uba85\ub3c4 \uc5c6\uc774 \ubb3b\ud600 \uc788\ub294 \ud76c\uc0dd\uc790 5\uba85 \ub4f1 \ucd1d 5,189\uba85\uc73c\ub85c \ud655\uc778\ub410\ub2e4. \uc774 \ud1b5\uacc4 \uc911 \uc0ac\ub9dd\uc790 163\uba85\uc740 \uc720\uc871\uc774 \ubcf4\uc0c1\uae08\uc744 \uc218\ub839\ud55c \uc0ac\ub9dd\uc790 \uc218\uc774\ub2e4. \ud655\uc2e4\ud558\uac8c \uc2e0\uc6d0\uc774 \ubc1d\ud600\uc84c\uc9c0\ub9cc, \ubcf4\uc0c1\uae08\uc744 \uc218\ub839\ubc1b\uc9c0 \uc54a\uc740 \uc0ac\ub78c\uc744 \ud3ec\ud568\ud558\uba74 \uc0ac\ub9dd\uc790\ub294 165\uba85 \uc774\uc0c1\uc73c\ub85c \ub298\uc5b4\ub09c\ub2e4. 1994\ub144 \uac80\ucc30\uc740 \uc0ac\uc0c1\uc790 \uc218\ub97c \ubc1c\ud45c\ud588\uc9c0\ub9cc, \ucd5c\ucd08 \ubc1c\ud3ec \uba85\ub839\uc790\uc640 \uc554\ub9e4\uc7a5 \uc7a5\uc18c\uc640 \uac19\uc740 \ud575\uc2ec \uc7c1\uc810\uc774 \ubc1d\ud600\uc9c0\uc9c0 \uc54a\uc73c\uba74\uc11c, 5\u00b718\uc774 \ubc1c\uc0dd\ud55c \uc9c0 \ud55c \uc138\ub300\uac00 \uc9c0\ub098\ub3c4\ub85d \uc774 \ubb38\uc81c\ub294 \ubbf8\uc644\uc758 \uacfc\uc81c\ub85c \ub0a8\uc544 \uc788\ub2e4. 5\u00b718 \ubbfc\uc8fc\ud654 \uc6b4\ub3d9 \uad00\ub828 \ubcf4\uc0c1\uc790 \ud1b5\uacc4\ub97c \ubcf4\uba74, \uc0ac\ub9dd\uc790 240\uba85, \ud589\ubc29\ubd88\uba85\uc790 409\uba85, \uc0c1\uc774 2,052\uba85 \ub4f1 \ucd1d 7,716\uba85\uc774 \ubcf4\uc0c1\uae08\uc744 \uc2e0\uccad\ud588\uc73c\uba70, \uc774 \uc911 \uc778\uc815\ub41c \ubcf4\uc0c1\uc790\ub294 \uc0ac\ub9dd\uc790 154\uba85, \ud589\ubc29\ubd88\uba85\uc790 70\uba85, \uc0c1\uc774 1,628\uba85 \ub4f1 \ucd1d 5,060\uba85\uc774\ub2e4. \ubcf4\uc0c1\uae08 \uc218\ub839\uc790 \ucd1d 5,060\uba85 \uc911 \uc911\ubcf5 \uc9c0\uae09\uc790 698\uba85\uc744 \uc81c\uc678\ud560 \uacbd\uc6b0, \ubcf4\uc0c1\uae08 \uc218\ub839\uc790\ub294 4,362\uba85\uc774\ub2e4.", "answer": "5,189\uba85", "sentence": "101\uba85, \ubd80\uc0c1\uc790\uac00 3,139\uba85, \uad6c\uc18d \ubc0f \uad6c\uae08 \ub4f1\uc758 \uae30\ud0c0 \ud53c\ud574\uc790 1,589\uba85, \uc544\uc9c1 \uc5f0\uace0\uac00 \ud655\uc778\ub418\uc9c0 \uc54a\uc544 \ubb18\ube44\uba85\ub3c4 \uc5c6\uc774 \ubb3b\ud600 \uc788\ub294 \ud76c\uc0dd\uc790 5\uba85 \ub4f1 \ucd1d 5,189\uba85 \uc73c\ub85c \ud655\uc778\ub410\ub2e4.", "paragraph_sentence": "5\u00b718 \uad11\uc8fc \ubbfc\uc8fc\ud654 \uc6b4\ub3d9\uc73c\ub85c \uc778\ud55c \uc0ac\ub9dd\uc790 \ubc0f \ud589\ubc29\ubd88\uba85\uc790\ub294 \uc57d 200\uc5ec \uba85\uc774\uace0 \ubd80\uc0c1\uc790 \ub4f1 \ud53c\ud574\uc790\ub294 \uc57d 4,300\uc5ec \uba85\uc774\ub2e4. \uad11\uc8fc\uad11\uc5ed\uc2dc\uac00 2009\ub144\uc5d0 5\u00b718 \uad11\uc8fc \ubbfc\uc8fc\ud654 \uc6b4\ub3d9 29\uc8fc\ub144\uc744 \ub9de\uc544 \ub2f9\uc2dc \ubaa9\uc228\uc744 \uc783\uac70\ub098 \ub2e4\uce5c \uc0ac\ub78c\uc744 \uc9d1\uacc4\ud55c \uacb0\uacfc, \uc0ac\ub9dd\uc790\uac00 163\uba85, \ud589\ubc29\ubd88\uba85\uc790\uac00 166\uba85, \ubd80\uc0c1 \ub4a4 \uc228\uc9c4 \uc0ac\ub78c\uc774 101\uba85, \ubd80\uc0c1\uc790\uac00 3,139\uba85, \uad6c\uc18d \ubc0f \uad6c\uae08 \ub4f1\uc758 \uae30\ud0c0 \ud53c\ud574\uc790 1,589\uba85, \uc544\uc9c1 \uc5f0\uace0\uac00 \ud655\uc778\ub418\uc9c0 \uc54a\uc544 \ubb18\ube44\uba85\ub3c4 \uc5c6\uc774 \ubb3b\ud600 \uc788\ub294 \ud76c\uc0dd\uc790 5\uba85 \ub4f1 \ucd1d 5,189\uba85 \uc73c\ub85c \ud655\uc778\ub410\ub2e4. \uc774 \ud1b5\uacc4 \uc911 \uc0ac\ub9dd\uc790 163\uba85\uc740 \uc720\uc871\uc774 \ubcf4\uc0c1\uae08\uc744 \uc218\ub839\ud55c \uc0ac\ub9dd\uc790 \uc218\uc774\ub2e4. \ud655\uc2e4\ud558\uac8c \uc2e0\uc6d0\uc774 \ubc1d\ud600\uc84c\uc9c0\ub9cc, \ubcf4\uc0c1\uae08\uc744 \uc218\ub839\ubc1b\uc9c0 \uc54a\uc740 \uc0ac\ub78c\uc744 \ud3ec\ud568\ud558\uba74 \uc0ac\ub9dd\uc790\ub294 165\uba85 \uc774\uc0c1\uc73c\ub85c \ub298\uc5b4\ub09c\ub2e4. 1994\ub144 \uac80\ucc30\uc740 \uc0ac\uc0c1\uc790 \uc218\ub97c \ubc1c\ud45c\ud588\uc9c0\ub9cc, \ucd5c\ucd08 \ubc1c\ud3ec \uba85\ub839\uc790\uc640 \uc554\ub9e4\uc7a5 \uc7a5\uc18c\uc640 \uac19\uc740 \ud575\uc2ec \uc7c1\uc810\uc774 \ubc1d\ud600\uc9c0\uc9c0 \uc54a\uc73c\uba74\uc11c, 5\u00b718\uc774 \ubc1c\uc0dd\ud55c \uc9c0 \ud55c \uc138\ub300\uac00 \uc9c0\ub098\ub3c4\ub85d \uc774 \ubb38\uc81c\ub294 \ubbf8\uc644\uc758 \uacfc\uc81c\ub85c \ub0a8\uc544 \uc788\ub2e4. 5\u00b718 \ubbfc\uc8fc\ud654 \uc6b4\ub3d9 \uad00\ub828 \ubcf4\uc0c1\uc790 \ud1b5\uacc4\ub97c \ubcf4\uba74, \uc0ac\ub9dd\uc790 240\uba85, \ud589\ubc29\ubd88\uba85\uc790 409\uba85, \uc0c1\uc774 2,052\uba85 \ub4f1 \ucd1d 7,716\uba85\uc774 \ubcf4\uc0c1\uae08\uc744 \uc2e0\uccad\ud588\uc73c\uba70, \uc774 \uc911 \uc778\uc815\ub41c \ubcf4\uc0c1\uc790\ub294 \uc0ac\ub9dd\uc790 154\uba85, \ud589\ubc29\ubd88\uba85\uc790 70\uba85, \uc0c1\uc774 1,628\uba85 \ub4f1 \ucd1d 5,060\uba85\uc774\ub2e4. \ubcf4\uc0c1\uae08 \uc218\ub839\uc790 \ucd1d 5,060\uba85 \uc911 \uc911\ubcf5 \uc9c0\uae09\uc790 698\uba85\uc744 \uc81c\uc678\ud560 \uacbd\uc6b0, \ubcf4\uc0c1\uae08 \uc218\ub839\uc790\ub294 4,362\uba85\uc774\ub2e4.", "paragraph_answer": "5\u00b718 \uad11\uc8fc \ubbfc\uc8fc\ud654 \uc6b4\ub3d9\uc73c\ub85c \uc778\ud55c \uc0ac\ub9dd\uc790 \ubc0f \ud589\ubc29\ubd88\uba85\uc790\ub294 \uc57d 200\uc5ec \uba85\uc774\uace0 \ubd80\uc0c1\uc790 \ub4f1 \ud53c\ud574\uc790\ub294 \uc57d 4,300\uc5ec \uba85\uc774\ub2e4. \uad11\uc8fc\uad11\uc5ed\uc2dc\uac00 2009\ub144\uc5d0 5\u00b718 \uad11\uc8fc \ubbfc\uc8fc\ud654 \uc6b4\ub3d9 29\uc8fc\ub144\uc744 \ub9de\uc544 \ub2f9\uc2dc \ubaa9\uc228\uc744 \uc783\uac70\ub098 \ub2e4\uce5c \uc0ac\ub78c\uc744 \uc9d1\uacc4\ud55c \uacb0\uacfc, \uc0ac\ub9dd\uc790\uac00 163\uba85, \ud589\ubc29\ubd88\uba85\uc790\uac00 166\uba85, \ubd80\uc0c1 \ub4a4 \uc228\uc9c4 \uc0ac\ub78c\uc774 101\uba85, \ubd80\uc0c1\uc790\uac00 3,139\uba85, \uad6c\uc18d \ubc0f \uad6c\uae08 \ub4f1\uc758 \uae30\ud0c0 \ud53c\ud574\uc790 1,589\uba85, \uc544\uc9c1 \uc5f0\uace0\uac00 \ud655\uc778\ub418\uc9c0 \uc54a\uc544 \ubb18\ube44\uba85\ub3c4 \uc5c6\uc774 \ubb3b\ud600 \uc788\ub294 \ud76c\uc0dd\uc790 5\uba85 \ub4f1 \ucd1d 5,189\uba85 \uc73c\ub85c \ud655\uc778\ub410\ub2e4. \uc774 \ud1b5\uacc4 \uc911 \uc0ac\ub9dd\uc790 163\uba85\uc740 \uc720\uc871\uc774 \ubcf4\uc0c1\uae08\uc744 \uc218\ub839\ud55c \uc0ac\ub9dd\uc790 \uc218\uc774\ub2e4. \ud655\uc2e4\ud558\uac8c \uc2e0\uc6d0\uc774 \ubc1d\ud600\uc84c\uc9c0\ub9cc, \ubcf4\uc0c1\uae08\uc744 \uc218\ub839\ubc1b\uc9c0 \uc54a\uc740 \uc0ac\ub78c\uc744 \ud3ec\ud568\ud558\uba74 \uc0ac\ub9dd\uc790\ub294 165\uba85 \uc774\uc0c1\uc73c\ub85c \ub298\uc5b4\ub09c\ub2e4. 1994\ub144 \uac80\ucc30\uc740 \uc0ac\uc0c1\uc790 \uc218\ub97c \ubc1c\ud45c\ud588\uc9c0\ub9cc, \ucd5c\ucd08 \ubc1c\ud3ec \uba85\ub839\uc790\uc640 \uc554\ub9e4\uc7a5 \uc7a5\uc18c\uc640 \uac19\uc740 \ud575\uc2ec \uc7c1\uc810\uc774 \ubc1d\ud600\uc9c0\uc9c0 \uc54a\uc73c\uba74\uc11c, 5\u00b718\uc774 \ubc1c\uc0dd\ud55c \uc9c0 \ud55c \uc138\ub300\uac00 \uc9c0\ub098\ub3c4\ub85d \uc774 \ubb38\uc81c\ub294 \ubbf8\uc644\uc758 \uacfc\uc81c\ub85c \ub0a8\uc544 \uc788\ub2e4. 5\u00b718 \ubbfc\uc8fc\ud654 \uc6b4\ub3d9 \uad00\ub828 \ubcf4\uc0c1\uc790 \ud1b5\uacc4\ub97c \ubcf4\uba74, \uc0ac\ub9dd\uc790 240\uba85, \ud589\ubc29\ubd88\uba85\uc790 409\uba85, \uc0c1\uc774 2,052\uba85 \ub4f1 \ucd1d 7,716\uba85\uc774 \ubcf4\uc0c1\uae08\uc744 \uc2e0\uccad\ud588\uc73c\uba70, \uc774 \uc911 \uc778\uc815\ub41c \ubcf4\uc0c1\uc790\ub294 \uc0ac\ub9dd\uc790 154\uba85, \ud589\ubc29\ubd88\uba85\uc790 70\uba85, \uc0c1\uc774 1,628\uba85 \ub4f1 \ucd1d 5,060\uba85\uc774\ub2e4. \ubcf4\uc0c1\uae08 \uc218\ub839\uc790 \ucd1d 5,060\uba85 \uc911 \uc911\ubcf5 \uc9c0\uae09\uc790 698\uba85\uc744 \uc81c\uc678\ud560 \uacbd\uc6b0, \ubcf4\uc0c1\uae08 \uc218\ub839\uc790\ub294 4,362\uba85\uc774\ub2e4.", "sentence_answer": "101\uba85, \ubd80\uc0c1\uc790\uac00 3,139\uba85, \uad6c\uc18d \ubc0f \uad6c\uae08 \ub4f1\uc758 \uae30\ud0c0 \ud53c\ud574\uc790 1,589\uba85, \uc544\uc9c1 \uc5f0\uace0\uac00 \ud655\uc778\ub418\uc9c0 \uc54a\uc544 \ubb18\ube44\uba85\ub3c4 \uc5c6\uc774 \ubb3b\ud600 \uc788\ub294 \ud76c\uc0dd\uc790 5\uba85 \ub4f1 \ucd1d 5,189\uba85 \uc73c\ub85c \ud655\uc778\ub410\ub2e4.", "paragraph_id": "6481190-16-2", "paragraph_question": "question: 2009\ub144\uc5d0 \uc7ac\uc9d1\uacc4\ud55c \uad11\uc8fc \ubbfc\uc8fc\ud654 \uc6b4\ub3d9\uc758 \ud53c\ud574\uc790\ub294 \uba87 \uba85\uc778\uac00?, context: 5\u00b718 \uad11\uc8fc \ubbfc\uc8fc\ud654 \uc6b4\ub3d9\uc73c\ub85c \uc778\ud55c \uc0ac\ub9dd\uc790 \ubc0f \ud589\ubc29\ubd88\uba85\uc790\ub294 \uc57d 200\uc5ec \uba85\uc774\uace0 \ubd80\uc0c1\uc790 \ub4f1 \ud53c\ud574\uc790\ub294 \uc57d 4,300\uc5ec \uba85\uc774\ub2e4. \uad11\uc8fc\uad11\uc5ed\uc2dc\uac00 2009\ub144\uc5d0 5\u00b718 \uad11\uc8fc \ubbfc\uc8fc\ud654 \uc6b4\ub3d9 29\uc8fc\ub144\uc744 \ub9de\uc544 \ub2f9\uc2dc \ubaa9\uc228\uc744 \uc783\uac70\ub098 \ub2e4\uce5c \uc0ac\ub78c\uc744 \uc9d1\uacc4\ud55c \uacb0\uacfc, \uc0ac\ub9dd\uc790\uac00 163\uba85, \ud589\ubc29\ubd88\uba85\uc790\uac00 166\uba85, \ubd80\uc0c1 \ub4a4 \uc228\uc9c4 \uc0ac\ub78c\uc774 101\uba85, \ubd80\uc0c1\uc790\uac00 3,139\uba85, \uad6c\uc18d \ubc0f \uad6c\uae08 \ub4f1\uc758 \uae30\ud0c0 \ud53c\ud574\uc790 1,589\uba85, \uc544\uc9c1 \uc5f0\uace0\uac00 \ud655\uc778\ub418\uc9c0 \uc54a\uc544 \ubb18\ube44\uba85\ub3c4 \uc5c6\uc774 \ubb3b\ud600 \uc788\ub294 \ud76c\uc0dd\uc790 5\uba85 \ub4f1 \ucd1d 5,189\uba85\uc73c\ub85c \ud655\uc778\ub410\ub2e4. \uc774 \ud1b5\uacc4 \uc911 \uc0ac\ub9dd\uc790 163\uba85\uc740 \uc720\uc871\uc774 \ubcf4\uc0c1\uae08\uc744 \uc218\ub839\ud55c \uc0ac\ub9dd\uc790 \uc218\uc774\ub2e4. \ud655\uc2e4\ud558\uac8c \uc2e0\uc6d0\uc774 \ubc1d\ud600\uc84c\uc9c0\ub9cc, \ubcf4\uc0c1\uae08\uc744 \uc218\ub839\ubc1b\uc9c0 \uc54a\uc740 \uc0ac\ub78c\uc744 \ud3ec\ud568\ud558\uba74 \uc0ac\ub9dd\uc790\ub294 165\uba85 \uc774\uc0c1\uc73c\ub85c \ub298\uc5b4\ub09c\ub2e4. 1994\ub144 \uac80\ucc30\uc740 \uc0ac\uc0c1\uc790 \uc218\ub97c \ubc1c\ud45c\ud588\uc9c0\ub9cc, \ucd5c\ucd08 \ubc1c\ud3ec \uba85\ub839\uc790\uc640 \uc554\ub9e4\uc7a5 \uc7a5\uc18c\uc640 \uac19\uc740 \ud575\uc2ec \uc7c1\uc810\uc774 \ubc1d\ud600\uc9c0\uc9c0 \uc54a\uc73c\uba74\uc11c, 5\u00b718\uc774 \ubc1c\uc0dd\ud55c \uc9c0 \ud55c \uc138\ub300\uac00 \uc9c0\ub098\ub3c4\ub85d \uc774 \ubb38\uc81c\ub294 \ubbf8\uc644\uc758 \uacfc\uc81c\ub85c \ub0a8\uc544 \uc788\ub2e4. 5\u00b718 \ubbfc\uc8fc\ud654 \uc6b4\ub3d9 \uad00\ub828 \ubcf4\uc0c1\uc790 \ud1b5\uacc4\ub97c \ubcf4\uba74, \uc0ac\ub9dd\uc790 240\uba85, \ud589\ubc29\ubd88\uba85\uc790 409\uba85, \uc0c1\uc774 2,052\uba85 \ub4f1 \ucd1d 7,716\uba85\uc774 \ubcf4\uc0c1\uae08\uc744 \uc2e0\uccad\ud588\uc73c\uba70, \uc774 \uc911 \uc778\uc815\ub41c \ubcf4\uc0c1\uc790\ub294 \uc0ac\ub9dd\uc790 154\uba85, \ud589\ubc29\ubd88\uba85\uc790 70\uba85, \uc0c1\uc774 1,628\uba85 \ub4f1 \ucd1d 5,060\uba85\uc774\ub2e4. \ubcf4\uc0c1\uae08 \uc218\ub839\uc790 \ucd1d 5,060\uba85 \uc911 \uc911\ubcf5 \uc9c0\uae09\uc790 698\uba85\uc744 \uc81c\uc678\ud560 \uacbd\uc6b0, \ubcf4\uc0c1\uae08 \uc218\ub839\uc790\ub294 4,362\uba85\uc774\ub2e4."} {"question": "\uc790\uae08\uc131 \uc11c\ubb38\uc758 \uba85\uce6d\uc740?", "paragraph": "\ud55c\ud3b8, \ubc31\ub828\uad50 \uc774\uc678\uc5d0\ub3c4 \ubc31\ub828\uad50\uc758 \uc77c\ud30c\uc600\ub358 \ucc9c\ub9ac\uad50(\u5929\u7406\u654e) \uc5ed\uc2dc \ubc31\uc131\ub4e4\uc5d0\uac8c \ub110\ub9ac \ud37c\uc838 \uc788\uc5c8\ub2e4. \ucc9c\ub9ac\uad50\ub3c4 \uc5ed\uc2dc \ubc31\ub828\uad50\ub3c4\ub4e4\uacfc \ube44\uc2b7\ud558\uac8c \uac01\uc9c0\uc5d0\uc11c \uc18c\uaddc\ubaa8\uc758 \uc138\ub825\uc744 \uaddc\ud569\ud558\uc5ec \uc870\uc815\uc5d0 \ud56d\uac70\ud558\ub824 \ud558\uc600\ub2e4. 1813\ub144(\uac00\uacbd 18\ub144), \ucc9c\ub9ac\uad50\uc758 \ub450\ub839 \uc784\uccad(\u6797\u6df8)\uacfc \uc774\ubb38\uc131(\u674e\u6587\u6210) \ub4f1\uc740 \ubc31\ub828\uad50\ub3c4\uc758 \ub09c\uacfc \ub2ec\ub9ac \uc774\ubc88\uc5d4 \ucc9c\ub9ac\uad50\ub3c4\ub97c \uc740\ubc00\ud788 \uad81\uad90\uc5d0 \ubcf4\ub0b4\uc5b4 \ud669\uad81 \ub0b4\uc758 \ud658\uad00\ub4e4\uae4c\uc9c0 \ud3ec\uc12d\ud558\uc600\ub2e4. \uc774 \ud574 7\uc6d4 18\uc77c, \uac00\uacbd\uc81c\ub294 \uac00\uc744 \uc0ac\ub0e5\uc744 \ud558\uae30 \uc704\ud574 \ud53c\uc11c\uc0b0\uc7a5\uc73c\ub85c \ub5a0\ub098\uc11c \uac00\uacbd\uc81c\uac00 \uc5c6\ub294 \uc0ac\uc774\uc5d0 \uc790\uae08\uc131\uc744 \ud0c8\ucde8\ud558\uae30\ub85c \uac70\uc0ac\ub97c \uacb0\ud589\ud558\uc600\ub2e4. 9\uc6d4 14\uc77c \ubc24, \ud658\uad00\ub4e4\uc740 \uc790\uae08\uc131\uc758 \ub3d9\ubb38\uc778 \ub3d9\ud654\ubb38(\u6771\u83ef\u9580), \uc11c\ubb38\uc778 \uc11c\ud654\ubb38(\u897f\u83ef\u9580)\uc744 \ubab0\ub798 \uc5f4\uc5b4 \uc57d 200\uba85\uc758 \ucc9c\ub9ac\uad50\ub3c4\ub4e4\uacfc \ub0b4\uc751\ud558\uc5ec \ud669\uad81\uc778 \uc790\uae08\uc131\uc5d0 \uc9c4\uc785\ud558\uc600\ub2e4. \uc790\uae08\uc131\uc73c\ub85c \ub4e4\uc5b4\uc628 \ucc9c\ub9ac\uad50\ub3c4\ub4e4\uc740 \uace7\ubc14\ub85c \ub0b4\uc804\uc778 \uc591\uc2ec\uc804\uc73c\ub85c \ud5a5\ud558\uc600\uc73c\ub098 \ub2f9\uc2dc \ud669\uad81\uc744 \uc9c0\ud0a4\ub358 \uc81c2\ud669\uc790 \uba74\ub155(\u7dbf\u5be7)\uc774 \uace7\ubc14\ub85c \uad81\uad90\uacfc \ud669\uc131\uc744 \uc9c0\ud0a4\ub294 \uae08\uad70\uacfc \uae08\uc758\uc704\ub97c \uc18c\uc9d1\ud55c \ud6c4 \uc9c1\uc811 \ucc9c\ub9ac\uad50\ub3c4 \ub450 \uba85\uc744 \ucd1d\uc73c\ub85c \uaca9\uc0b4\ud55c \ub4a4 \ucc9c\ub9ac\uad50\ub3c4\ub4e4\uc744 \uaca9\ud1f4\uc2dc\ucf30\ub2e4. \ub180\ub780 \uac00\uacbd\uc81c\ub294 \uadf8 \ub2e4\uc74c\ub0a0\uc5d0 \uace7\ubc14\ub85c \ud658\uad81\ud558\uc5ec \uc790\uae08\uc131\uc744 \uc9c0\ucf1c\ub0b8 \uba74\ub155\uc744 \uc9c0\uce5c\uc655(\u667a\u89aa\u738b)\uc5d0 \ucc45\ubd09\ud558\uc600\uace0 \uad70\uc0ac\ub4e4\uc744 \ub300\ub300\uc801\uc73c\ub85c \ud480\uc5b4 \uc784\uccad\uacfc \uadf8 \uc218\ud558\ub4e4\uc744 \uccb4\ud3ec\ud55c \ub4a4 \uace7\ubc14\ub85c \ub300\uc5ed\uc8c4\ub97c \ubb3c\uc5b4 \ub2a5\uc9c0\ud615\uacfc \ucc38\uc218\ud615\uc5d0 \ucc98\ud558\uc600\uace0 \uc774\ubb38\uc131\uc740 \uac70\uc0ac\uac00 \uc2e4\ud328\ud558\uc790 \ubc14\ub85c \uc790\uacb0\ud558\uc600\ub2e4. \uc774 \uc0ac\uac74\uc774 \ubc14\ub85c 1813\ub144, \uc989 \uacc4\uc720\ub144\uc5d0 \uc77c\uc5b4\ub0ac\ub2e4 \ud558\uc5ec \uacc4\uc720\uc9c0\ubcc0\uc73c\ub85c \ubd88\ub9b0\ub2e4. \uc774\ud6c4 \uac00\uacbd\uc81c\ub294 \uc790\uc2e0\uc758 \uc8c4\ub97c \uc4f4 \uc8c4\uae30\uc870(\u7f6a\u5df1\u8a54)\ub97c \uc368\uc11c \ucc9c\ud558\uc5d0 \uacf5\ud45c\ud558\uc600\uc73c\ub098 \ubcc4\ub2e4\ub978 \ubc18\ud5a5\uc740 \uc5c6\uc5c8\ub2e4.", "answer": "\uc11c\ud654\ubb38(\u897f\u83ef\u9580)", "sentence": "9\uc6d4 14\uc77c \ubc24, \ud658\uad00\ub4e4\uc740 \uc790\uae08\uc131\uc758 \ub3d9\ubb38\uc778 \ub3d9\ud654\ubb38(\u6771\u83ef\u9580), \uc11c\ubb38\uc778 \uc11c\ud654\ubb38(\u897f\u83ef\u9580) \uc744 \ubab0\ub798 \uc5f4\uc5b4 \uc57d 200\uba85\uc758 \ucc9c\ub9ac\uad50\ub3c4\ub4e4\uacfc \ub0b4\uc751\ud558\uc5ec \ud669\uad81\uc778 \uc790\uae08\uc131\uc5d0 \uc9c4\uc785\ud558\uc600\ub2e4.", "paragraph_sentence": "\ud55c\ud3b8, \ubc31\ub828\uad50 \uc774\uc678\uc5d0\ub3c4 \ubc31\ub828\uad50\uc758 \uc77c\ud30c\uc600\ub358 \ucc9c\ub9ac\uad50(\u5929\u7406\u654e) \uc5ed\uc2dc \ubc31\uc131\ub4e4\uc5d0\uac8c \ub110\ub9ac \ud37c\uc838 \uc788\uc5c8\ub2e4. \ucc9c\ub9ac\uad50\ub3c4 \uc5ed\uc2dc \ubc31\ub828\uad50\ub3c4\ub4e4\uacfc \ube44\uc2b7\ud558\uac8c \uac01\uc9c0\uc5d0\uc11c \uc18c\uaddc\ubaa8\uc758 \uc138\ub825\uc744 \uaddc\ud569\ud558\uc5ec \uc870\uc815\uc5d0 \ud56d\uac70\ud558\ub824 \ud558\uc600\ub2e4. 1813\ub144(\uac00\uacbd 18\ub144), \ucc9c\ub9ac\uad50\uc758 \ub450\ub839 \uc784\uccad(\u6797\u6df8)\uacfc \uc774\ubb38\uc131(\u674e\u6587\u6210) \ub4f1\uc740 \ubc31\ub828\uad50\ub3c4\uc758 \ub09c\uacfc \ub2ec\ub9ac \uc774\ubc88\uc5d4 \ucc9c\ub9ac\uad50\ub3c4\ub97c \uc740\ubc00\ud788 \uad81\uad90\uc5d0 \ubcf4\ub0b4\uc5b4 \ud669\uad81 \ub0b4\uc758 \ud658\uad00\ub4e4\uae4c\uc9c0 \ud3ec\uc12d\ud558\uc600\ub2e4. \uc774 \ud574 7\uc6d4 18\uc77c, \uac00\uacbd\uc81c\ub294 \uac00\uc744 \uc0ac\ub0e5\uc744 \ud558\uae30 \uc704\ud574 \ud53c\uc11c\uc0b0\uc7a5\uc73c\ub85c \ub5a0\ub098\uc11c \uac00\uacbd\uc81c\uac00 \uc5c6\ub294 \uc0ac\uc774\uc5d0 \uc790\uae08\uc131\uc744 \ud0c8\ucde8\ud558\uae30\ub85c \uac70\uc0ac\ub97c \uacb0\ud589\ud558\uc600\ub2e4. 9\uc6d4 14\uc77c \ubc24, \ud658\uad00\ub4e4\uc740 \uc790\uae08\uc131\uc758 \ub3d9\ubb38\uc778 \ub3d9\ud654\ubb38(\u6771\u83ef\u9580), \uc11c\ubb38\uc778 \uc11c\ud654\ubb38(\u897f\u83ef\u9580) \uc744 \ubab0\ub798 \uc5f4\uc5b4 \uc57d 200\uba85\uc758 \ucc9c\ub9ac\uad50\ub3c4\ub4e4\uacfc \ub0b4\uc751\ud558\uc5ec \ud669\uad81\uc778 \uc790\uae08\uc131\uc5d0 \uc9c4\uc785\ud558\uc600\ub2e4. \uc790\uae08\uc131\uc73c\ub85c \ub4e4\uc5b4\uc628 \ucc9c\ub9ac\uad50\ub3c4\ub4e4\uc740 \uace7\ubc14\ub85c \ub0b4\uc804\uc778 \uc591\uc2ec\uc804\uc73c\ub85c \ud5a5\ud558\uc600\uc73c\ub098 \ub2f9\uc2dc \ud669\uad81\uc744 \uc9c0\ud0a4\ub358 \uc81c2\ud669\uc790 \uba74\ub155(\u7dbf\u5be7)\uc774 \uace7\ubc14\ub85c \uad81\uad90\uacfc \ud669\uc131\uc744 \uc9c0\ud0a4\ub294 \uae08\uad70\uacfc \uae08\uc758\uc704\ub97c \uc18c\uc9d1\ud55c \ud6c4 \uc9c1\uc811 \ucc9c\ub9ac\uad50\ub3c4 \ub450 \uba85\uc744 \ucd1d\uc73c\ub85c \uaca9\uc0b4\ud55c \ub4a4 \ucc9c\ub9ac\uad50\ub3c4\ub4e4\uc744 \uaca9\ud1f4\uc2dc\ucf30\ub2e4. \ub180\ub780 \uac00\uacbd\uc81c\ub294 \uadf8 \ub2e4\uc74c\ub0a0\uc5d0 \uace7\ubc14\ub85c \ud658\uad81\ud558\uc5ec \uc790\uae08\uc131\uc744 \uc9c0\ucf1c\ub0b8 \uba74\ub155\uc744 \uc9c0\uce5c\uc655(\u667a\u89aa\u738b)\uc5d0 \ucc45\ubd09\ud558\uc600\uace0 \uad70\uc0ac\ub4e4\uc744 \ub300\ub300\uc801\uc73c\ub85c \ud480\uc5b4 \uc784\uccad\uacfc \uadf8 \uc218\ud558\ub4e4\uc744 \uccb4\ud3ec\ud55c \ub4a4 \uace7\ubc14\ub85c \ub300\uc5ed\uc8c4\ub97c \ubb3c\uc5b4 \ub2a5\uc9c0\ud615\uacfc \ucc38\uc218\ud615\uc5d0 \ucc98\ud558\uc600\uace0 \uc774\ubb38\uc131\uc740 \uac70\uc0ac\uac00 \uc2e4\ud328\ud558\uc790 \ubc14\ub85c \uc790\uacb0\ud558\uc600\ub2e4. \uc774 \uc0ac\uac74\uc774 \ubc14\ub85c 1813\ub144, \uc989 \uacc4\uc720\ub144\uc5d0 \uc77c\uc5b4\ub0ac\ub2e4 \ud558\uc5ec \uacc4\uc720\uc9c0\ubcc0\uc73c\ub85c \ubd88\ub9b0\ub2e4. \uc774\ud6c4 \uac00\uacbd\uc81c\ub294 \uc790\uc2e0\uc758 \uc8c4\ub97c \uc4f4 \uc8c4\uae30\uc870(\u7f6a\u5df1\u8a54)\ub97c \uc368\uc11c \ucc9c\ud558\uc5d0 \uacf5\ud45c\ud558\uc600\uc73c\ub098 \ubcc4\ub2e4\ub978 \ubc18\ud5a5\uc740 \uc5c6\uc5c8\ub2e4.", "paragraph_answer": "\ud55c\ud3b8, \ubc31\ub828\uad50 \uc774\uc678\uc5d0\ub3c4 \ubc31\ub828\uad50\uc758 \uc77c\ud30c\uc600\ub358 \ucc9c\ub9ac\uad50(\u5929\u7406\u654e) \uc5ed\uc2dc \ubc31\uc131\ub4e4\uc5d0\uac8c \ub110\ub9ac \ud37c\uc838 \uc788\uc5c8\ub2e4. \ucc9c\ub9ac\uad50\ub3c4 \uc5ed\uc2dc \ubc31\ub828\uad50\ub3c4\ub4e4\uacfc \ube44\uc2b7\ud558\uac8c \uac01\uc9c0\uc5d0\uc11c \uc18c\uaddc\ubaa8\uc758 \uc138\ub825\uc744 \uaddc\ud569\ud558\uc5ec \uc870\uc815\uc5d0 \ud56d\uac70\ud558\ub824 \ud558\uc600\ub2e4. 1813\ub144(\uac00\uacbd 18\ub144), \ucc9c\ub9ac\uad50\uc758 \ub450\ub839 \uc784\uccad(\u6797\u6df8)\uacfc \uc774\ubb38\uc131(\u674e\u6587\u6210) \ub4f1\uc740 \ubc31\ub828\uad50\ub3c4\uc758 \ub09c\uacfc \ub2ec\ub9ac \uc774\ubc88\uc5d4 \ucc9c\ub9ac\uad50\ub3c4\ub97c \uc740\ubc00\ud788 \uad81\uad90\uc5d0 \ubcf4\ub0b4\uc5b4 \ud669\uad81 \ub0b4\uc758 \ud658\uad00\ub4e4\uae4c\uc9c0 \ud3ec\uc12d\ud558\uc600\ub2e4. \uc774 \ud574 7\uc6d4 18\uc77c, \uac00\uacbd\uc81c\ub294 \uac00\uc744 \uc0ac\ub0e5\uc744 \ud558\uae30 \uc704\ud574 \ud53c\uc11c\uc0b0\uc7a5\uc73c\ub85c \ub5a0\ub098\uc11c \uac00\uacbd\uc81c\uac00 \uc5c6\ub294 \uc0ac\uc774\uc5d0 \uc790\uae08\uc131\uc744 \ud0c8\ucde8\ud558\uae30\ub85c \uac70\uc0ac\ub97c \uacb0\ud589\ud558\uc600\ub2e4. 9\uc6d4 14\uc77c \ubc24, \ud658\uad00\ub4e4\uc740 \uc790\uae08\uc131\uc758 \ub3d9\ubb38\uc778 \ub3d9\ud654\ubb38(\u6771\u83ef\u9580), \uc11c\ubb38\uc778 \uc11c\ud654\ubb38(\u897f\u83ef\u9580) \uc744 \ubab0\ub798 \uc5f4\uc5b4 \uc57d 200\uba85\uc758 \ucc9c\ub9ac\uad50\ub3c4\ub4e4\uacfc \ub0b4\uc751\ud558\uc5ec \ud669\uad81\uc778 \uc790\uae08\uc131\uc5d0 \uc9c4\uc785\ud558\uc600\ub2e4. \uc790\uae08\uc131\uc73c\ub85c \ub4e4\uc5b4\uc628 \ucc9c\ub9ac\uad50\ub3c4\ub4e4\uc740 \uace7\ubc14\ub85c \ub0b4\uc804\uc778 \uc591\uc2ec\uc804\uc73c\ub85c \ud5a5\ud558\uc600\uc73c\ub098 \ub2f9\uc2dc \ud669\uad81\uc744 \uc9c0\ud0a4\ub358 \uc81c2\ud669\uc790 \uba74\ub155(\u7dbf\u5be7)\uc774 \uace7\ubc14\ub85c \uad81\uad90\uacfc \ud669\uc131\uc744 \uc9c0\ud0a4\ub294 \uae08\uad70\uacfc \uae08\uc758\uc704\ub97c \uc18c\uc9d1\ud55c \ud6c4 \uc9c1\uc811 \ucc9c\ub9ac\uad50\ub3c4 \ub450 \uba85\uc744 \ucd1d\uc73c\ub85c \uaca9\uc0b4\ud55c \ub4a4 \ucc9c\ub9ac\uad50\ub3c4\ub4e4\uc744 \uaca9\ud1f4\uc2dc\ucf30\ub2e4. \ub180\ub780 \uac00\uacbd\uc81c\ub294 \uadf8 \ub2e4\uc74c\ub0a0\uc5d0 \uace7\ubc14\ub85c \ud658\uad81\ud558\uc5ec \uc790\uae08\uc131\uc744 \uc9c0\ucf1c\ub0b8 \uba74\ub155\uc744 \uc9c0\uce5c\uc655(\u667a\u89aa\u738b)\uc5d0 \ucc45\ubd09\ud558\uc600\uace0 \uad70\uc0ac\ub4e4\uc744 \ub300\ub300\uc801\uc73c\ub85c \ud480\uc5b4 \uc784\uccad\uacfc \uadf8 \uc218\ud558\ub4e4\uc744 \uccb4\ud3ec\ud55c \ub4a4 \uace7\ubc14\ub85c \ub300\uc5ed\uc8c4\ub97c \ubb3c\uc5b4 \ub2a5\uc9c0\ud615\uacfc \ucc38\uc218\ud615\uc5d0 \ucc98\ud558\uc600\uace0 \uc774\ubb38\uc131\uc740 \uac70\uc0ac\uac00 \uc2e4\ud328\ud558\uc790 \ubc14\ub85c \uc790\uacb0\ud558\uc600\ub2e4. \uc774 \uc0ac\uac74\uc774 \ubc14\ub85c 1813\ub144, \uc989 \uacc4\uc720\ub144\uc5d0 \uc77c\uc5b4\ub0ac\ub2e4 \ud558\uc5ec \uacc4\uc720\uc9c0\ubcc0\uc73c\ub85c \ubd88\ub9b0\ub2e4. \uc774\ud6c4 \uac00\uacbd\uc81c\ub294 \uc790\uc2e0\uc758 \uc8c4\ub97c \uc4f4 \uc8c4\uae30\uc870(\u7f6a\u5df1\u8a54)\ub97c \uc368\uc11c \ucc9c\ud558\uc5d0 \uacf5\ud45c\ud558\uc600\uc73c\ub098 \ubcc4\ub2e4\ub978 \ubc18\ud5a5\uc740 \uc5c6\uc5c8\ub2e4.", "sentence_answer": "9\uc6d4 14\uc77c \ubc24, \ud658\uad00\ub4e4\uc740 \uc790\uae08\uc131\uc758 \ub3d9\ubb38\uc778 \ub3d9\ud654\ubb38(\u6771\u83ef\u9580), \uc11c\ubb38\uc778 \uc11c\ud654\ubb38(\u897f\u83ef\u9580) \uc744 \ubab0\ub798 \uc5f4\uc5b4 \uc57d 200\uba85\uc758 \ucc9c\ub9ac\uad50\ub3c4\ub4e4\uacfc \ub0b4\uc751\ud558\uc5ec \ud669\uad81\uc778 \uc790\uae08\uc131\uc5d0 \uc9c4\uc785\ud558\uc600\ub2e4.", "paragraph_id": "6472899-8-0", "paragraph_question": "question: \uc790\uae08\uc131 \uc11c\ubb38\uc758 \uba85\uce6d\uc740?, context: \ud55c\ud3b8, \ubc31\ub828\uad50 \uc774\uc678\uc5d0\ub3c4 \ubc31\ub828\uad50\uc758 \uc77c\ud30c\uc600\ub358 \ucc9c\ub9ac\uad50(\u5929\u7406\u654e) \uc5ed\uc2dc \ubc31\uc131\ub4e4\uc5d0\uac8c \ub110\ub9ac \ud37c\uc838 \uc788\uc5c8\ub2e4. \ucc9c\ub9ac\uad50\ub3c4 \uc5ed\uc2dc \ubc31\ub828\uad50\ub3c4\ub4e4\uacfc \ube44\uc2b7\ud558\uac8c \uac01\uc9c0\uc5d0\uc11c \uc18c\uaddc\ubaa8\uc758 \uc138\ub825\uc744 \uaddc\ud569\ud558\uc5ec \uc870\uc815\uc5d0 \ud56d\uac70\ud558\ub824 \ud558\uc600\ub2e4. 1813\ub144(\uac00\uacbd 18\ub144), \ucc9c\ub9ac\uad50\uc758 \ub450\ub839 \uc784\uccad(\u6797\u6df8)\uacfc \uc774\ubb38\uc131(\u674e\u6587\u6210) \ub4f1\uc740 \ubc31\ub828\uad50\ub3c4\uc758 \ub09c\uacfc \ub2ec\ub9ac \uc774\ubc88\uc5d4 \ucc9c\ub9ac\uad50\ub3c4\ub97c \uc740\ubc00\ud788 \uad81\uad90\uc5d0 \ubcf4\ub0b4\uc5b4 \ud669\uad81 \ub0b4\uc758 \ud658\uad00\ub4e4\uae4c\uc9c0 \ud3ec\uc12d\ud558\uc600\ub2e4. \uc774 \ud574 7\uc6d4 18\uc77c, \uac00\uacbd\uc81c\ub294 \uac00\uc744 \uc0ac\ub0e5\uc744 \ud558\uae30 \uc704\ud574 \ud53c\uc11c\uc0b0\uc7a5\uc73c\ub85c \ub5a0\ub098\uc11c \uac00\uacbd\uc81c\uac00 \uc5c6\ub294 \uc0ac\uc774\uc5d0 \uc790\uae08\uc131\uc744 \ud0c8\ucde8\ud558\uae30\ub85c \uac70\uc0ac\ub97c \uacb0\ud589\ud558\uc600\ub2e4. 9\uc6d4 14\uc77c \ubc24, \ud658\uad00\ub4e4\uc740 \uc790\uae08\uc131\uc758 \ub3d9\ubb38\uc778 \ub3d9\ud654\ubb38(\u6771\u83ef\u9580), \uc11c\ubb38\uc778 \uc11c\ud654\ubb38(\u897f\u83ef\u9580)\uc744 \ubab0\ub798 \uc5f4\uc5b4 \uc57d 200\uba85\uc758 \ucc9c\ub9ac\uad50\ub3c4\ub4e4\uacfc \ub0b4\uc751\ud558\uc5ec \ud669\uad81\uc778 \uc790\uae08\uc131\uc5d0 \uc9c4\uc785\ud558\uc600\ub2e4. \uc790\uae08\uc131\uc73c\ub85c \ub4e4\uc5b4\uc628 \ucc9c\ub9ac\uad50\ub3c4\ub4e4\uc740 \uace7\ubc14\ub85c \ub0b4\uc804\uc778 \uc591\uc2ec\uc804\uc73c\ub85c \ud5a5\ud558\uc600\uc73c\ub098 \ub2f9\uc2dc \ud669\uad81\uc744 \uc9c0\ud0a4\ub358 \uc81c2\ud669\uc790 \uba74\ub155(\u7dbf\u5be7)\uc774 \uace7\ubc14\ub85c \uad81\uad90\uacfc \ud669\uc131\uc744 \uc9c0\ud0a4\ub294 \uae08\uad70\uacfc \uae08\uc758\uc704\ub97c \uc18c\uc9d1\ud55c \ud6c4 \uc9c1\uc811 \ucc9c\ub9ac\uad50\ub3c4 \ub450 \uba85\uc744 \ucd1d\uc73c\ub85c \uaca9\uc0b4\ud55c \ub4a4 \ucc9c\ub9ac\uad50\ub3c4\ub4e4\uc744 \uaca9\ud1f4\uc2dc\ucf30\ub2e4. \ub180\ub780 \uac00\uacbd\uc81c\ub294 \uadf8 \ub2e4\uc74c\ub0a0\uc5d0 \uace7\ubc14\ub85c \ud658\uad81\ud558\uc5ec \uc790\uae08\uc131\uc744 \uc9c0\ucf1c\ub0b8 \uba74\ub155\uc744 \uc9c0\uce5c\uc655(\u667a\u89aa\u738b)\uc5d0 \ucc45\ubd09\ud558\uc600\uace0 \uad70\uc0ac\ub4e4\uc744 \ub300\ub300\uc801\uc73c\ub85c \ud480\uc5b4 \uc784\uccad\uacfc \uadf8 \uc218\ud558\ub4e4\uc744 \uccb4\ud3ec\ud55c \ub4a4 \uace7\ubc14\ub85c \ub300\uc5ed\uc8c4\ub97c \ubb3c\uc5b4 \ub2a5\uc9c0\ud615\uacfc \ucc38\uc218\ud615\uc5d0 \ucc98\ud558\uc600\uace0 \uc774\ubb38\uc131\uc740 \uac70\uc0ac\uac00 \uc2e4\ud328\ud558\uc790 \ubc14\ub85c \uc790\uacb0\ud558\uc600\ub2e4. \uc774 \uc0ac\uac74\uc774 \ubc14\ub85c 1813\ub144, \uc989 \uacc4\uc720\ub144\uc5d0 \uc77c\uc5b4\ub0ac\ub2e4 \ud558\uc5ec \uacc4\uc720\uc9c0\ubcc0\uc73c\ub85c \ubd88\ub9b0\ub2e4. \uc774\ud6c4 \uac00\uacbd\uc81c\ub294 \uc790\uc2e0\uc758 \uc8c4\ub97c \uc4f4 \uc8c4\uae30\uc870(\u7f6a\u5df1\u8a54)\ub97c \uc368\uc11c \ucc9c\ud558\uc5d0 \uacf5\ud45c\ud558\uc600\uc73c\ub098 \ubcc4\ub2e4\ub978 \ubc18\ud5a5\uc740 \uc5c6\uc5c8\ub2e4."} {"question": "\uc548\ucca0\uc218\uac00 \uc2dc\uc7a5\uc9c1 \uc790\ub9ac\ub97c \uc591\ubcf4\ud55c \uc0ac\ub78c\uc740?", "paragraph": "\ubc15\uc6d0\uc21c\uc740 \ub2e8\uc77c\ud654\uc5d0 \ub300\ud574 \u201c\ub450 \uc0ac\ub78c \ubaa8\ub450 \uc2dc\uc7a5\uc9c1 \uc790\ub9ac\ub97c \uc6d0\ud55c \uac8c \uc544\ub2c8\ub2e4. \uc9c4\uc815 \uc0c8\ub85c\uc6b4 \uc138\uc0c1\uc744 \ub9cc\ub4dc\ub294 \ub370 \uad00\uc2ec\uc774 \uc788\uc5c8\uae30 \ub54c\ubb38\uc5d0 \uc774\ub807\uac8c \uc0c1\uc2dd\uc801\uc73c\ub85c \uc774\ud574 \uc548 \ub418\ub294 \uacb0\ub860\uc774 \ub098\uc628 \uac83\u201d\ub77c\uace0 \ub9d0\ud588\ub2e4. \ubc15\uc6d0\uc21c\uc740 \ub610 \uc548\ucca0\uc218\uc5d0 \ub300\ud574 \u201c\uc544\ubb34\ub9ac \uc2e0\ub8b0\uad00\uacc4\uac00 \uc788\ub2e4\ud574\ub3c4 \uc800\ubcf4\ub2e4 10\ubc30\ub098 \ub354 \ub418\ub294 \uc9c0\uc9c0\ub3c4\ub97c \uac16\uace0 \uc788\ub358 \ubd84\uc774 \uc815\ub9d0 \uc544\ubb34 \uc870\uac74 \uc5c6\uc774 \u2018\ub354 \uc798 \ud560 \uc218 \uc788\ub2e4\u2019\uace0 \ud558\ub294 (\ub0b4 \ub9d0) \ud55c\ub9c8\ub514\ub85c \uc591\ubcf4\ud55c\ub2e4\ub294 \uac8c \uc0ac\uc2e4 \ub610 \ubbff\uae30 \uc5b4\ub824\uc6b4 \uadf8\ub7f0 \uc77c\u201d\uc774\ub77c\uba70 \u201c\uc548 \uad50\uc218\uac00 \uac1c\uc778\uc758 \uc774\uc775\ubcf4\ub2e4 \uc0ac\ud68c\uc758 \uc5b4\ub5a4 \uacf5\uacf5\uc801\uc778 \uc774\uc775\uc744 \uc704\ud574\uc11c \ud574\uc654\ub358 \ubd84\uc774\uc5c8\uae30 \ub54c\ubb38\uc5d0 \uac00\ub2a5\ud588\ub358 \ud0dc\ub3c4\uc600\ub2e4\uace0 \ubcf8\ub2e4\u201d\ub77c\uace0 \ub9d0\ud588\ub2e4. \uc774\ud6c4 \ubc15\uc6d0\uc21c, \ud55c\uba85\uc219, \ubb38\uc7ac\uc778 \ub4f1\uc740 \u201c\uc120\uac70 \uc2b9\ub9ac\ub97c \uc704\ud574\uc11c\ub294 \ubc94\uc2dc\ubbfc \uc57c\uad8c \ub2e8\uc77c\ud6c4\ubcf4\ub97c \ud1b5\ud574 \ud55c\ub098\ub77c\ub2f9\uacfc 1:1 \uad6c\ub3c4\ub97c \ub9cc\ub4e4\uc5b4\uc57c \ud55c\ub2e4. (\uc11c\uc6b8\uc2dc\uc7a5 \ud6c4\ubcf4\ub85c \uac70\ub860\ub418\ub294)\ubc15\uc6d0\uc21c-\ud55c\uba85\uc219 \ub450 \uc0ac\ub78c\uc740 \ubc94\uc2dc\ubbfc \uc57c\uad8c \ub2e8\uc77c \ud6c4\ubcf4 \uc120\ucd9c\uc744 \uc704\ud574 \uc0c1\ud638 \ud611\ub825\ud558\uace0, \uc774\ud6c4\uc5d4 \uc120\uac70 \uc2b9\ub9ac\ub97c \uc704\ud574 \ubaa8\ub4e0 \ud798\uc744 \uae30\uc6b8\uc778\ub2e4\u201d\ub77c\uba70 \uacb0\uc758\ub97c \ub2e4\uc84c\ub2e4.", "answer": "\ubc15\uc6d0\uc21c", "sentence": "\ubc15\uc6d0\uc21c \uc740 \ub2e8\uc77c\ud654\uc5d0 \ub300\ud574 \u201c\ub450 \uc0ac\ub78c \ubaa8\ub450 \uc2dc\uc7a5\uc9c1 \uc790\ub9ac\ub97c \uc6d0\ud55c \uac8c \uc544\ub2c8\ub2e4.", "paragraph_sentence": " \ubc15\uc6d0\uc21c \uc740 \ub2e8\uc77c\ud654\uc5d0 \ub300\ud574 \u201c\ub450 \uc0ac\ub78c \ubaa8\ub450 \uc2dc\uc7a5\uc9c1 \uc790\ub9ac\ub97c \uc6d0\ud55c \uac8c \uc544\ub2c8\ub2e4. \uc9c4\uc815 \uc0c8\ub85c\uc6b4 \uc138\uc0c1\uc744 \ub9cc\ub4dc\ub294 \ub370 \uad00\uc2ec\uc774 \uc788\uc5c8\uae30 \ub54c\ubb38\uc5d0 \uc774\ub807\uac8c \uc0c1\uc2dd\uc801\uc73c\ub85c \uc774\ud574 \uc548 \ub418\ub294 \uacb0\ub860\uc774 \ub098\uc628 \uac83\u201d\ub77c\uace0 \ub9d0\ud588\ub2e4. \ubc15\uc6d0\uc21c\uc740 \ub610 \uc548\ucca0\uc218\uc5d0 \ub300\ud574 \u201c\uc544\ubb34\ub9ac \uc2e0\ub8b0\uad00\uacc4\uac00 \uc788\ub2e4\ud574\ub3c4 \uc800\ubcf4\ub2e4 10\ubc30\ub098 \ub354 \ub418\ub294 \uc9c0\uc9c0\ub3c4\ub97c \uac16\uace0 \uc788\ub358 \ubd84\uc774 \uc815\ub9d0 \uc544\ubb34 \uc870\uac74 \uc5c6\uc774 \u2018\ub354 \uc798 \ud560 \uc218 \uc788\ub2e4\u2019\uace0 \ud558\ub294 (\ub0b4 \ub9d0) \ud55c\ub9c8\ub514\ub85c \uc591\ubcf4\ud55c\ub2e4\ub294 \uac8c \uc0ac\uc2e4 \ub610 \ubbff\uae30 \uc5b4\ub824\uc6b4 \uadf8\ub7f0 \uc77c\u201d\uc774\ub77c\uba70 \u201c\uc548 \uad50\uc218\uac00 \uac1c\uc778\uc758 \uc774\uc775\ubcf4\ub2e4 \uc0ac\ud68c\uc758 \uc5b4\ub5a4 \uacf5\uacf5\uc801\uc778 \uc774\uc775\uc744 \uc704\ud574\uc11c \ud574\uc654\ub358 \ubd84\uc774\uc5c8\uae30 \ub54c\ubb38\uc5d0 \uac00\ub2a5\ud588\ub358 \ud0dc\ub3c4\uc600\ub2e4\uace0 \ubcf8\ub2e4\u201d\ub77c\uace0 \ub9d0\ud588\ub2e4. \uc774\ud6c4 \ubc15\uc6d0\uc21c, \ud55c\uba85\uc219, \ubb38\uc7ac\uc778 \ub4f1\uc740 \u201c\uc120\uac70 \uc2b9\ub9ac\ub97c \uc704\ud574\uc11c\ub294 \ubc94\uc2dc\ubbfc \uc57c\uad8c \ub2e8\uc77c\ud6c4\ubcf4\ub97c \ud1b5\ud574 \ud55c\ub098\ub77c\ub2f9\uacfc 1:1 \uad6c\ub3c4\ub97c \ub9cc\ub4e4\uc5b4\uc57c \ud55c\ub2e4. (\uc11c\uc6b8\uc2dc\uc7a5 \ud6c4\ubcf4\ub85c \uac70\ub860\ub418\ub294)\ubc15\uc6d0\uc21c-\ud55c\uba85\uc219 \ub450 \uc0ac\ub78c\uc740 \ubc94\uc2dc\ubbfc \uc57c\uad8c \ub2e8\uc77c \ud6c4\ubcf4 \uc120\ucd9c\uc744 \uc704\ud574 \uc0c1\ud638 \ud611\ub825\ud558\uace0, \uc774\ud6c4\uc5d4 \uc120\uac70 \uc2b9\ub9ac\ub97c \uc704\ud574 \ubaa8\ub4e0 \ud798\uc744 \uae30\uc6b8\uc778\ub2e4\u201d\ub77c\uba70 \uacb0\uc758\ub97c \ub2e4\uc84c\ub2e4.", "paragraph_answer": " \ubc15\uc6d0\uc21c \uc740 \ub2e8\uc77c\ud654\uc5d0 \ub300\ud574 \u201c\ub450 \uc0ac\ub78c \ubaa8\ub450 \uc2dc\uc7a5\uc9c1 \uc790\ub9ac\ub97c \uc6d0\ud55c \uac8c \uc544\ub2c8\ub2e4. \uc9c4\uc815 \uc0c8\ub85c\uc6b4 \uc138\uc0c1\uc744 \ub9cc\ub4dc\ub294 \ub370 \uad00\uc2ec\uc774 \uc788\uc5c8\uae30 \ub54c\ubb38\uc5d0 \uc774\ub807\uac8c \uc0c1\uc2dd\uc801\uc73c\ub85c \uc774\ud574 \uc548 \ub418\ub294 \uacb0\ub860\uc774 \ub098\uc628 \uac83\u201d\ub77c\uace0 \ub9d0\ud588\ub2e4. \ubc15\uc6d0\uc21c\uc740 \ub610 \uc548\ucca0\uc218\uc5d0 \ub300\ud574 \u201c\uc544\ubb34\ub9ac \uc2e0\ub8b0\uad00\uacc4\uac00 \uc788\ub2e4\ud574\ub3c4 \uc800\ubcf4\ub2e4 10\ubc30\ub098 \ub354 \ub418\ub294 \uc9c0\uc9c0\ub3c4\ub97c \uac16\uace0 \uc788\ub358 \ubd84\uc774 \uc815\ub9d0 \uc544\ubb34 \uc870\uac74 \uc5c6\uc774 \u2018\ub354 \uc798 \ud560 \uc218 \uc788\ub2e4\u2019\uace0 \ud558\ub294 (\ub0b4 \ub9d0) \ud55c\ub9c8\ub514\ub85c \uc591\ubcf4\ud55c\ub2e4\ub294 \uac8c \uc0ac\uc2e4 \ub610 \ubbff\uae30 \uc5b4\ub824\uc6b4 \uadf8\ub7f0 \uc77c\u201d\uc774\ub77c\uba70 \u201c\uc548 \uad50\uc218\uac00 \uac1c\uc778\uc758 \uc774\uc775\ubcf4\ub2e4 \uc0ac\ud68c\uc758 \uc5b4\ub5a4 \uacf5\uacf5\uc801\uc778 \uc774\uc775\uc744 \uc704\ud574\uc11c \ud574\uc654\ub358 \ubd84\uc774\uc5c8\uae30 \ub54c\ubb38\uc5d0 \uac00\ub2a5\ud588\ub358 \ud0dc\ub3c4\uc600\ub2e4\uace0 \ubcf8\ub2e4\u201d\ub77c\uace0 \ub9d0\ud588\ub2e4. \uc774\ud6c4 \ubc15\uc6d0\uc21c, \ud55c\uba85\uc219, \ubb38\uc7ac\uc778 \ub4f1\uc740 \u201c\uc120\uac70 \uc2b9\ub9ac\ub97c \uc704\ud574\uc11c\ub294 \ubc94\uc2dc\ubbfc \uc57c\uad8c \ub2e8\uc77c\ud6c4\ubcf4\ub97c \ud1b5\ud574 \ud55c\ub098\ub77c\ub2f9\uacfc 1:1 \uad6c\ub3c4\ub97c \ub9cc\ub4e4\uc5b4\uc57c \ud55c\ub2e4. (\uc11c\uc6b8\uc2dc\uc7a5 \ud6c4\ubcf4\ub85c \uac70\ub860\ub418\ub294)\ubc15\uc6d0\uc21c-\ud55c\uba85\uc219 \ub450 \uc0ac\ub78c\uc740 \ubc94\uc2dc\ubbfc \uc57c\uad8c \ub2e8\uc77c \ud6c4\ubcf4 \uc120\ucd9c\uc744 \uc704\ud574 \uc0c1\ud638 \ud611\ub825\ud558\uace0, \uc774\ud6c4\uc5d4 \uc120\uac70 \uc2b9\ub9ac\ub97c \uc704\ud574 \ubaa8\ub4e0 \ud798\uc744 \uae30\uc6b8\uc778\ub2e4\u201d\ub77c\uba70 \uacb0\uc758\ub97c \ub2e4\uc84c\ub2e4.", "sentence_answer": " \ubc15\uc6d0\uc21c \uc740 \ub2e8\uc77c\ud654\uc5d0 \ub300\ud574 \u201c\ub450 \uc0ac\ub78c \ubaa8\ub450 \uc2dc\uc7a5\uc9c1 \uc790\ub9ac\ub97c \uc6d0\ud55c \uac8c \uc544\ub2c8\ub2e4.", "paragraph_id": "6547507-10-0", "paragraph_question": "question: \uc548\ucca0\uc218\uac00 \uc2dc\uc7a5\uc9c1 \uc790\ub9ac\ub97c \uc591\ubcf4\ud55c \uc0ac\ub78c\uc740?, context: \ubc15\uc6d0\uc21c\uc740 \ub2e8\uc77c\ud654\uc5d0 \ub300\ud574 \u201c\ub450 \uc0ac\ub78c \ubaa8\ub450 \uc2dc\uc7a5\uc9c1 \uc790\ub9ac\ub97c \uc6d0\ud55c \uac8c \uc544\ub2c8\ub2e4. \uc9c4\uc815 \uc0c8\ub85c\uc6b4 \uc138\uc0c1\uc744 \ub9cc\ub4dc\ub294 \ub370 \uad00\uc2ec\uc774 \uc788\uc5c8\uae30 \ub54c\ubb38\uc5d0 \uc774\ub807\uac8c \uc0c1\uc2dd\uc801\uc73c\ub85c \uc774\ud574 \uc548 \ub418\ub294 \uacb0\ub860\uc774 \ub098\uc628 \uac83\u201d\ub77c\uace0 \ub9d0\ud588\ub2e4. \ubc15\uc6d0\uc21c\uc740 \ub610 \uc548\ucca0\uc218\uc5d0 \ub300\ud574 \u201c\uc544\ubb34\ub9ac \uc2e0\ub8b0\uad00\uacc4\uac00 \uc788\ub2e4\ud574\ub3c4 \uc800\ubcf4\ub2e4 10\ubc30\ub098 \ub354 \ub418\ub294 \uc9c0\uc9c0\ub3c4\ub97c \uac16\uace0 \uc788\ub358 \ubd84\uc774 \uc815\ub9d0 \uc544\ubb34 \uc870\uac74 \uc5c6\uc774 \u2018\ub354 \uc798 \ud560 \uc218 \uc788\ub2e4\u2019\uace0 \ud558\ub294 (\ub0b4 \ub9d0) \ud55c\ub9c8\ub514\ub85c \uc591\ubcf4\ud55c\ub2e4\ub294 \uac8c \uc0ac\uc2e4 \ub610 \ubbff\uae30 \uc5b4\ub824\uc6b4 \uadf8\ub7f0 \uc77c\u201d\uc774\ub77c\uba70 \u201c\uc548 \uad50\uc218\uac00 \uac1c\uc778\uc758 \uc774\uc775\ubcf4\ub2e4 \uc0ac\ud68c\uc758 \uc5b4\ub5a4 \uacf5\uacf5\uc801\uc778 \uc774\uc775\uc744 \uc704\ud574\uc11c \ud574\uc654\ub358 \ubd84\uc774\uc5c8\uae30 \ub54c\ubb38\uc5d0 \uac00\ub2a5\ud588\ub358 \ud0dc\ub3c4\uc600\ub2e4\uace0 \ubcf8\ub2e4\u201d\ub77c\uace0 \ub9d0\ud588\ub2e4. \uc774\ud6c4 \ubc15\uc6d0\uc21c, \ud55c\uba85\uc219, \ubb38\uc7ac\uc778 \ub4f1\uc740 \u201c\uc120\uac70 \uc2b9\ub9ac\ub97c \uc704\ud574\uc11c\ub294 \ubc94\uc2dc\ubbfc \uc57c\uad8c \ub2e8\uc77c\ud6c4\ubcf4\ub97c \ud1b5\ud574 \ud55c\ub098\ub77c\ub2f9\uacfc 1:1 \uad6c\ub3c4\ub97c \ub9cc\ub4e4\uc5b4\uc57c \ud55c\ub2e4. (\uc11c\uc6b8\uc2dc\uc7a5 \ud6c4\ubcf4\ub85c \uac70\ub860\ub418\ub294)\ubc15\uc6d0\uc21c-\ud55c\uba85\uc219 \ub450 \uc0ac\ub78c\uc740 \ubc94\uc2dc\ubbfc \uc57c\uad8c \ub2e8\uc77c \ud6c4\ubcf4 \uc120\ucd9c\uc744 \uc704\ud574 \uc0c1\ud638 \ud611\ub825\ud558\uace0, \uc774\ud6c4\uc5d4 \uc120\uac70 \uc2b9\ub9ac\ub97c \uc704\ud574 \ubaa8\ub4e0 \ud798\uc744 \uae30\uc6b8\uc778\ub2e4\u201d\ub77c\uba70 \uacb0\uc758\ub97c \ub2e4\uc84c\ub2e4."} {"question": "\ubc15\ubcf4\uac80\uc774 2014\ub144 \ub4dc\ub77c\ub9c8 \ucc38 \uc88b\uc740 \uc2dc\uc808\uc5d0\uc11c \ub204\uad6c\uc758 \uc544\uc5ed\uc73c\ub85c \uce90\uc2a4\ud305 \ub418\uc5c8\ub294\uac00?", "paragraph": "2014\ub144 2\uc6d4 \ub4dc\ub77c\ub9c8 \u300a\ucc38 \uc88b\uc740 \uc2dc\uc808\u300b\uc5d0\uc11c \uac15\ub3d9\uc11d(\uc774\uc11c\uc9c4 \ubd84)\uc758 \uace0\ub4f1\ud559\uad50 \uc2dc\uc808 \uc544\uc5ed\uc5d0 \uce90\uc2a4\ud305 \ub418\uc5c8\ub2e4. \uc774 \uc791\ud488\uc5d0\uc11c\ub294 \ud604\uc7ac \uc18d\uc5d0\uc11c \uacfc\uac70 \ud68c\uc0c1 \uc7a5\uba74\uc73c\ub85c \ub4f1\uc7a5\ud558\uba70 \uadf8\ub294 \uc2e4\uac10\ub098\ub294 \uacbd\uc0c1\ub3c4 \uc0ac\ud22c\ub9ac \uc5f0\uae30\ub85c \ud638\ud3c9\uc744 \ubc1b\uc558\ub2e4. \ubc15\ubcf4\uac80\uc740 \ub2f9\uc2dc \uac19\uc740 \uc18c\uc18d\uc0ac \uc120\ubc30\uc778 \uc2e0\uc2b9\ud658\uc5d0\uac8c \uc0ac\ud22c\ub9ac \uc9c0\ub3c4\ub97c \ubc1b\uc558\ub2e4\uace0 \ud55c\ub2e4. \uadf8\ub294 \uac19\uc740 \ud574 5\uc6d4 \uce78 \uc601\ud654\uc81c \uac10\ub3c5\uc8fc\uac04 \ubd80\ubb38\uc5d0 \ucd08\uccad \ub41c \uc601\ud654 \u300a\ub05d\uae4c\uc9c0 \uac04\ub2e4\u300b, 1700\ub9cc \uad00\uac1d\uc744 \ubaa8\uc740 \uc601\ud654 \u300a\uba85\ub7c9\u300b\uc5d0\uc11c \uc774\uc21c\uc2e0 \uc7a5\uad70(\ucd5c\ubbfc\uc2dd \ubd84)\uc5d0\uac8c \ud1a0\ub780\uc744 \uac74\ub124\ub294 \uc18c\ub144 \uc5ed\uacfc \ub4dc\ub77c\ub9c8\u300a\ub0b4\uc77c\ub3c4 \uce78\ud0c0\ube4c\ub808\u300b \ub4f1\uc5d0\uc11c \uc791\uc9c0\ub9cc \ub3cb\ubcf4\uc774\ub294 \uce90\ub9ad\ud130\ub97c \uc5f0\uae30\ud558\uba74\uc11c \ucc28\uadfc\ucc28\uadfc \ud544\ubaa8\uadf8\ub798\ud53c\ub97c \uc313\uc558\ub2e4. \ud2b9\ud788 \uadf8\ub294 \ub4dc\ub77c\ub9c8 \u300a\ub0b4\uc77c\ub3c4 \uce78\ud0c0\ube4c\ub808\u300b\uc5d0\uc11c\ub294 \uacf1\uac8c \uc790\ub780 \ub3c4\ub828\ub2d8 \uac19\uc740 \uc678\ubaa8\ub97c \uc18c\uc720\ud55c \ucc9c\uc7ac \uccbc\ub9ac\uc2a4\ud2b8 \uc774\uc724\ud6c4 \uc5ed\ud560\uc744 \uc5f0\uae30\ud588\ub294\ub370, \ub0a8\ubab0\ub798 \uc190\uac00\ub77d \ud1b5\uc99d\uc744 \uc553\uc73c\uba70 \uc790\uc2e0\uacfc \uc2f8\uc6cc\uc57c \ud558\ub294 \uc678\ub85c\uc6b4 \uc874\uc7ac\ub85c \uccbc\ub9ac\uc2a4\ud2b8\ub85c\uc11c \uace0\ub3c5\ud568\uacfc \uc124\ub0b4\uc77c(\uc2ec\uc740\uacbd \ubd84)\uc744 \ud5a5\ud55c \uc9dd\uc0ac\ub791\ub0a8\uc758 \uc560\uc794\ud55c \ubaa8\uc2b5\uc744 \ub3d9\uc2dc\uc5d0 \uadf8\ub824\ub0b4\uba70 \uc720\ub9dd\uc8fc\ub85c \uc8fc\ubaa9 \ubc1b\uae30 \uc2dc\uc791\ud588\uace0, KBS \uc5f0\uae30\ub300\uc0c1\uacfc \uc5d0\uc774\ud310 \uc2a4\ud0c0 \uc5b4\uc6cc\uc988 \uc2e0\uc778\uc0c1 \ud6c4\ubcf4\uc5d0 \uc62c\ub790\ub2e4. \uc601\ud654 \u300a\ub05d\uae4c\uc9c0 \uac04\ub2e4\u300b\uc758 \uae40\uc131\ud6c8 \uac10\ub3c5\uc740 \uc774\ud6c4 \ud55c \uc5b8\ub860 \uc778\ud130\ubdf0\uc5d0\uc11c \uadf8\uc5d0 \ub300\ud574 \ub2e4\uc74c\uacfc \uac19\uc774 \uc5b8\uae09\ud588\ub2e4.", "answer": "\uc774\uc11c\uc9c4", "sentence": "2014\ub144 2\uc6d4 \ub4dc\ub77c\ub9c8 \u300a\ucc38 \uc88b\uc740 \uc2dc\uc808\u300b\uc5d0\uc11c \uac15\ub3d9\uc11d( \uc774\uc11c\uc9c4 \ubd84)\uc758 \uace0\ub4f1\ud559\uad50 \uc2dc\uc808 \uc544\uc5ed\uc5d0 \uce90\uc2a4\ud305 \ub418\uc5c8\ub2e4.", "paragraph_sentence": " 2014\ub144 2\uc6d4 \ub4dc\ub77c\ub9c8 \u300a\ucc38 \uc88b\uc740 \uc2dc\uc808\u300b\uc5d0\uc11c \uac15\ub3d9\uc11d( \uc774\uc11c\uc9c4 \ubd84)\uc758 \uace0\ub4f1\ud559\uad50 \uc2dc\uc808 \uc544\uc5ed\uc5d0 \uce90\uc2a4\ud305 \ub418\uc5c8\ub2e4. \uc774 \uc791\ud488\uc5d0\uc11c\ub294 \ud604\uc7ac \uc18d\uc5d0\uc11c \uacfc\uac70 \ud68c\uc0c1 \uc7a5\uba74\uc73c\ub85c \ub4f1\uc7a5\ud558\uba70 \uadf8\ub294 \uc2e4\uac10\ub098\ub294 \uacbd\uc0c1\ub3c4 \uc0ac\ud22c\ub9ac \uc5f0\uae30\ub85c \ud638\ud3c9\uc744 \ubc1b\uc558\ub2e4. \ubc15\ubcf4\uac80\uc740 \ub2f9\uc2dc \uac19\uc740 \uc18c\uc18d\uc0ac \uc120\ubc30\uc778 \uc2e0\uc2b9\ud658\uc5d0\uac8c \uc0ac\ud22c\ub9ac \uc9c0\ub3c4\ub97c \ubc1b\uc558\ub2e4\uace0 \ud55c\ub2e4. \uadf8\ub294 \uac19\uc740 \ud574 5\uc6d4 \uce78 \uc601\ud654\uc81c \uac10\ub3c5\uc8fc\uac04 \ubd80\ubb38\uc5d0 \ucd08\uccad \ub41c \uc601\ud654 \u300a\ub05d\uae4c\uc9c0 \uac04\ub2e4\u300b, 1700\ub9cc \uad00\uac1d\uc744 \ubaa8\uc740 \uc601\ud654 \u300a\uba85\ub7c9\u300b\uc5d0\uc11c \uc774\uc21c\uc2e0 \uc7a5\uad70(\ucd5c\ubbfc\uc2dd \ubd84)\uc5d0\uac8c \ud1a0\ub780\uc744 \uac74\ub124\ub294 \uc18c\ub144 \uc5ed\uacfc \ub4dc\ub77c\ub9c8\u300a\ub0b4\uc77c\ub3c4 \uce78\ud0c0\ube4c\ub808\u300b \ub4f1\uc5d0\uc11c \uc791\uc9c0\ub9cc \ub3cb\ubcf4\uc774\ub294 \uce90\ub9ad\ud130\ub97c \uc5f0\uae30\ud558\uba74\uc11c \ucc28\uadfc\ucc28\uadfc \ud544\ubaa8\uadf8\ub798\ud53c\ub97c \uc313\uc558\ub2e4. \ud2b9\ud788 \uadf8\ub294 \ub4dc\ub77c\ub9c8 \u300a\ub0b4\uc77c\ub3c4 \uce78\ud0c0\ube4c\ub808\u300b\uc5d0\uc11c\ub294 \uacf1\uac8c \uc790\ub780 \ub3c4\ub828\ub2d8 \uac19\uc740 \uc678\ubaa8\ub97c \uc18c\uc720\ud55c \ucc9c\uc7ac \uccbc\ub9ac\uc2a4\ud2b8 \uc774\uc724\ud6c4 \uc5ed\ud560\uc744 \uc5f0\uae30\ud588\ub294\ub370, \ub0a8\ubab0\ub798 \uc190\uac00\ub77d \ud1b5\uc99d\uc744 \uc553\uc73c\uba70 \uc790\uc2e0\uacfc \uc2f8\uc6cc\uc57c \ud558\ub294 \uc678\ub85c\uc6b4 \uc874\uc7ac\ub85c \uccbc\ub9ac\uc2a4\ud2b8\ub85c\uc11c \uace0\ub3c5\ud568\uacfc \uc124\ub0b4\uc77c(\uc2ec\uc740\uacbd \ubd84)\uc744 \ud5a5\ud55c \uc9dd\uc0ac\ub791\ub0a8\uc758 \uc560\uc794\ud55c \ubaa8\uc2b5\uc744 \ub3d9\uc2dc\uc5d0 \uadf8\ub824\ub0b4\uba70 \uc720\ub9dd\uc8fc\ub85c \uc8fc\ubaa9 \ubc1b\uae30 \uc2dc\uc791\ud588\uace0, KBS \uc5f0\uae30\ub300\uc0c1\uacfc \uc5d0\uc774\ud310 \uc2a4\ud0c0 \uc5b4\uc6cc\uc988 \uc2e0\uc778\uc0c1 \ud6c4\ubcf4\uc5d0 \uc62c\ub790\ub2e4. \uc601\ud654 \u300a\ub05d\uae4c\uc9c0 \uac04\ub2e4\u300b\uc758 \uae40\uc131\ud6c8 \uac10\ub3c5\uc740 \uc774\ud6c4 \ud55c \uc5b8\ub860 \uc778\ud130\ubdf0\uc5d0\uc11c \uadf8\uc5d0 \ub300\ud574 \ub2e4\uc74c\uacfc \uac19\uc774 \uc5b8\uae09\ud588\ub2e4.", "paragraph_answer": "2014\ub144 2\uc6d4 \ub4dc\ub77c\ub9c8 \u300a\ucc38 \uc88b\uc740 \uc2dc\uc808\u300b\uc5d0\uc11c \uac15\ub3d9\uc11d( \uc774\uc11c\uc9c4 \ubd84)\uc758 \uace0\ub4f1\ud559\uad50 \uc2dc\uc808 \uc544\uc5ed\uc5d0 \uce90\uc2a4\ud305 \ub418\uc5c8\ub2e4. \uc774 \uc791\ud488\uc5d0\uc11c\ub294 \ud604\uc7ac \uc18d\uc5d0\uc11c \uacfc\uac70 \ud68c\uc0c1 \uc7a5\uba74\uc73c\ub85c \ub4f1\uc7a5\ud558\uba70 \uadf8\ub294 \uc2e4\uac10\ub098\ub294 \uacbd\uc0c1\ub3c4 \uc0ac\ud22c\ub9ac \uc5f0\uae30\ub85c \ud638\ud3c9\uc744 \ubc1b\uc558\ub2e4. \ubc15\ubcf4\uac80\uc740 \ub2f9\uc2dc \uac19\uc740 \uc18c\uc18d\uc0ac \uc120\ubc30\uc778 \uc2e0\uc2b9\ud658\uc5d0\uac8c \uc0ac\ud22c\ub9ac \uc9c0\ub3c4\ub97c \ubc1b\uc558\ub2e4\uace0 \ud55c\ub2e4. \uadf8\ub294 \uac19\uc740 \ud574 5\uc6d4 \uce78 \uc601\ud654\uc81c \uac10\ub3c5\uc8fc\uac04 \ubd80\ubb38\uc5d0 \ucd08\uccad \ub41c \uc601\ud654 \u300a\ub05d\uae4c\uc9c0 \uac04\ub2e4\u300b, 1700\ub9cc \uad00\uac1d\uc744 \ubaa8\uc740 \uc601\ud654 \u300a\uba85\ub7c9\u300b\uc5d0\uc11c \uc774\uc21c\uc2e0 \uc7a5\uad70(\ucd5c\ubbfc\uc2dd \ubd84)\uc5d0\uac8c \ud1a0\ub780\uc744 \uac74\ub124\ub294 \uc18c\ub144 \uc5ed\uacfc \ub4dc\ub77c\ub9c8\u300a\ub0b4\uc77c\ub3c4 \uce78\ud0c0\ube4c\ub808\u300b \ub4f1\uc5d0\uc11c \uc791\uc9c0\ub9cc \ub3cb\ubcf4\uc774\ub294 \uce90\ub9ad\ud130\ub97c \uc5f0\uae30\ud558\uba74\uc11c \ucc28\uadfc\ucc28\uadfc \ud544\ubaa8\uadf8\ub798\ud53c\ub97c \uc313\uc558\ub2e4. \ud2b9\ud788 \uadf8\ub294 \ub4dc\ub77c\ub9c8 \u300a\ub0b4\uc77c\ub3c4 \uce78\ud0c0\ube4c\ub808\u300b\uc5d0\uc11c\ub294 \uacf1\uac8c \uc790\ub780 \ub3c4\ub828\ub2d8 \uac19\uc740 \uc678\ubaa8\ub97c \uc18c\uc720\ud55c \ucc9c\uc7ac \uccbc\ub9ac\uc2a4\ud2b8 \uc774\uc724\ud6c4 \uc5ed\ud560\uc744 \uc5f0\uae30\ud588\ub294\ub370, \ub0a8\ubab0\ub798 \uc190\uac00\ub77d \ud1b5\uc99d\uc744 \uc553\uc73c\uba70 \uc790\uc2e0\uacfc \uc2f8\uc6cc\uc57c \ud558\ub294 \uc678\ub85c\uc6b4 \uc874\uc7ac\ub85c \uccbc\ub9ac\uc2a4\ud2b8\ub85c\uc11c \uace0\ub3c5\ud568\uacfc \uc124\ub0b4\uc77c(\uc2ec\uc740\uacbd \ubd84)\uc744 \ud5a5\ud55c \uc9dd\uc0ac\ub791\ub0a8\uc758 \uc560\uc794\ud55c \ubaa8\uc2b5\uc744 \ub3d9\uc2dc\uc5d0 \uadf8\ub824\ub0b4\uba70 \uc720\ub9dd\uc8fc\ub85c \uc8fc\ubaa9 \ubc1b\uae30 \uc2dc\uc791\ud588\uace0, KBS \uc5f0\uae30\ub300\uc0c1\uacfc \uc5d0\uc774\ud310 \uc2a4\ud0c0 \uc5b4\uc6cc\uc988 \uc2e0\uc778\uc0c1 \ud6c4\ubcf4\uc5d0 \uc62c\ub790\ub2e4. \uc601\ud654 \u300a\ub05d\uae4c\uc9c0 \uac04\ub2e4\u300b\uc758 \uae40\uc131\ud6c8 \uac10\ub3c5\uc740 \uc774\ud6c4 \ud55c \uc5b8\ub860 \uc778\ud130\ubdf0\uc5d0\uc11c \uadf8\uc5d0 \ub300\ud574 \ub2e4\uc74c\uacfc \uac19\uc774 \uc5b8\uae09\ud588\ub2e4.", "sentence_answer": "2014\ub144 2\uc6d4 \ub4dc\ub77c\ub9c8 \u300a\ucc38 \uc88b\uc740 \uc2dc\uc808\u300b\uc5d0\uc11c \uac15\ub3d9\uc11d( \uc774\uc11c\uc9c4 \ubd84)\uc758 \uace0\ub4f1\ud559\uad50 \uc2dc\uc808 \uc544\uc5ed\uc5d0 \uce90\uc2a4\ud305 \ub418\uc5c8\ub2e4.", "paragraph_id": "6530163-2-0", "paragraph_question": "question: \ubc15\ubcf4\uac80\uc774 2014\ub144 \ub4dc\ub77c\ub9c8 \ucc38 \uc88b\uc740 \uc2dc\uc808\uc5d0\uc11c \ub204\uad6c\uc758 \uc544\uc5ed\uc73c\ub85c \uce90\uc2a4\ud305 \ub418\uc5c8\ub294\uac00?, context: 2014\ub144 2\uc6d4 \ub4dc\ub77c\ub9c8 \u300a\ucc38 \uc88b\uc740 \uc2dc\uc808\u300b\uc5d0\uc11c \uac15\ub3d9\uc11d(\uc774\uc11c\uc9c4 \ubd84)\uc758 \uace0\ub4f1\ud559\uad50 \uc2dc\uc808 \uc544\uc5ed\uc5d0 \uce90\uc2a4\ud305 \ub418\uc5c8\ub2e4. \uc774 \uc791\ud488\uc5d0\uc11c\ub294 \ud604\uc7ac \uc18d\uc5d0\uc11c \uacfc\uac70 \ud68c\uc0c1 \uc7a5\uba74\uc73c\ub85c \ub4f1\uc7a5\ud558\uba70 \uadf8\ub294 \uc2e4\uac10\ub098\ub294 \uacbd\uc0c1\ub3c4 \uc0ac\ud22c\ub9ac \uc5f0\uae30\ub85c \ud638\ud3c9\uc744 \ubc1b\uc558\ub2e4. \ubc15\ubcf4\uac80\uc740 \ub2f9\uc2dc \uac19\uc740 \uc18c\uc18d\uc0ac \uc120\ubc30\uc778 \uc2e0\uc2b9\ud658\uc5d0\uac8c \uc0ac\ud22c\ub9ac \uc9c0\ub3c4\ub97c \ubc1b\uc558\ub2e4\uace0 \ud55c\ub2e4. \uadf8\ub294 \uac19\uc740 \ud574 5\uc6d4 \uce78 \uc601\ud654\uc81c \uac10\ub3c5\uc8fc\uac04 \ubd80\ubb38\uc5d0 \ucd08\uccad \ub41c \uc601\ud654 \u300a\ub05d\uae4c\uc9c0 \uac04\ub2e4\u300b, 1700\ub9cc \uad00\uac1d\uc744 \ubaa8\uc740 \uc601\ud654 \u300a\uba85\ub7c9\u300b\uc5d0\uc11c \uc774\uc21c\uc2e0 \uc7a5\uad70(\ucd5c\ubbfc\uc2dd \ubd84)\uc5d0\uac8c \ud1a0\ub780\uc744 \uac74\ub124\ub294 \uc18c\ub144 \uc5ed\uacfc \ub4dc\ub77c\ub9c8\u300a\ub0b4\uc77c\ub3c4 \uce78\ud0c0\ube4c\ub808\u300b \ub4f1\uc5d0\uc11c \uc791\uc9c0\ub9cc \ub3cb\ubcf4\uc774\ub294 \uce90\ub9ad\ud130\ub97c \uc5f0\uae30\ud558\uba74\uc11c \ucc28\uadfc\ucc28\uadfc \ud544\ubaa8\uadf8\ub798\ud53c\ub97c \uc313\uc558\ub2e4. \ud2b9\ud788 \uadf8\ub294 \ub4dc\ub77c\ub9c8 \u300a\ub0b4\uc77c\ub3c4 \uce78\ud0c0\ube4c\ub808\u300b\uc5d0\uc11c\ub294 \uacf1\uac8c \uc790\ub780 \ub3c4\ub828\ub2d8 \uac19\uc740 \uc678\ubaa8\ub97c \uc18c\uc720\ud55c \ucc9c\uc7ac \uccbc\ub9ac\uc2a4\ud2b8 \uc774\uc724\ud6c4 \uc5ed\ud560\uc744 \uc5f0\uae30\ud588\ub294\ub370, \ub0a8\ubab0\ub798 \uc190\uac00\ub77d \ud1b5\uc99d\uc744 \uc553\uc73c\uba70 \uc790\uc2e0\uacfc \uc2f8\uc6cc\uc57c \ud558\ub294 \uc678\ub85c\uc6b4 \uc874\uc7ac\ub85c \uccbc\ub9ac\uc2a4\ud2b8\ub85c\uc11c \uace0\ub3c5\ud568\uacfc \uc124\ub0b4\uc77c(\uc2ec\uc740\uacbd \ubd84)\uc744 \ud5a5\ud55c \uc9dd\uc0ac\ub791\ub0a8\uc758 \uc560\uc794\ud55c \ubaa8\uc2b5\uc744 \ub3d9\uc2dc\uc5d0 \uadf8\ub824\ub0b4\uba70 \uc720\ub9dd\uc8fc\ub85c \uc8fc\ubaa9 \ubc1b\uae30 \uc2dc\uc791\ud588\uace0, KBS \uc5f0\uae30\ub300\uc0c1\uacfc \uc5d0\uc774\ud310 \uc2a4\ud0c0 \uc5b4\uc6cc\uc988 \uc2e0\uc778\uc0c1 \ud6c4\ubcf4\uc5d0 \uc62c\ub790\ub2e4. \uc601\ud654 \u300a\ub05d\uae4c\uc9c0 \uac04\ub2e4\u300b\uc758 \uae40\uc131\ud6c8 \uac10\ub3c5\uc740 \uc774\ud6c4 \ud55c \uc5b8\ub860 \uc778\ud130\ubdf0\uc5d0\uc11c \uadf8\uc5d0 \ub300\ud574 \ub2e4\uc74c\uacfc \uac19\uc774 \uc5b8\uae09\ud588\ub2e4."} {"question": "\uc2a4\ud0c0\ud0a4\uac00 \uace8\ubb34\ub97c \ucc29\uc6a9\ud558\uace0 \ubb38\uc9c0\ub978 \uac83\uc73c\ub85c \uc6d0\uc2dc\uc801\uc774\uace0 \uac15\ub82c\ud55c \ub9ac\ub4ec\uc744 \ub0c8\ub358 \uac83\uc740?", "paragraph": "\ud2b8\ub798\ud3ec\ub4dc\uac00 \uc2a4\ud0c0\ud0a4\uc758 \uc2a4\ud0a4\ud50c \ud765\ubbf8\ub97c \uc7a1\uc544\ub048\ub300 \uc774\uc5b4 \uadf8 \ub450 \uba85\uc740 \uc81c\uc870\uacf5\uc7a5\uc758 \uc9c0\ud558\uc800\uc7a5\uc2e4\uc5d0\uc11c \uc810\uc2ec\uc2dc\uac04\uc774 \ub418\uba74 \uace1\uc744 \uc5f0\uc2b5\ud558\uae30 \uc2dc\uc791\ud588\ub2e4. \ud2b8\ub798\ud3ec\ub4dc\uac00 \ud68c\uc0c1\ud558\uae30\ub97c \"\uc800\ub294 \uae30\ud0c0\ub97c \uc5f0\uc8fc\ud588\uace0, [\ub9ac\uce58]\ub294 \ub2e8\uc21c\ud788 \uc0c1\uc790\ub85c \uc18c\uc74c\uc744 \ub0c8\uc8e0 ... \uac00\ub054\uc529 \uadf8 \ub140\uc11d\uc740 \uacfc\uc790\uc0c1\uc790\ub97c \uc5f4\uc1e0\ubb49\uce58\ub85c \ub0b4\ub824\uc9c0\uac70\ub098 \uc758\uc790 \ub4f1\ubc1b\uc774\ub97c \ub54c\ub9ac\uae30\ub3c4 \ud588\uc5b4\uc694.\" \uadf8\ub4e4\uc740 \uc2a4\ud0c0\ud0a4\uc758 \uc774\uc6c3\uc774\uc790 \uc9c1\uc7a5\ub3d9\ub8cc\uc778 \uae30\ud0c0\ub9ac\uc2a4\ud2b8 \uc5d0\ub514 \ub9c8\uc77c\uc2a4\uc640 \ud569\ub958\ud574 \uc5d0\ub514 \ub9c8\uc77c\uc2a4 \ubc34\ub4dc\ub97c \uacb0\uc131\ud588\ub2e4. \uc774\ud6c4 \ub9ac\ubc84\ud480 \uba85\uc18c\uc758 \uc774\ub984\uc744 \ub530 \uc5d0\ub514 \ud074\ub808\uc774\ud2bc \uc564 \ub354 \ud074\ub808\uc774\ud2bc \uc2a4\ud018\uc5b4\ub85c \uac1c\uba85\ud588\ub2e4. \ubc34\ub4dc\ub294 \u3008Rock Island Line\u3009, \u3008Walking Cane\u3009 \ub4f1 \uc720\uba85 \uc2a4\ud0a4\ud50c \uace1\uc744 \uc5f0\uc8fc\ud588\uace0, \uc2a4\ud0c0\ud0a4\ub294 \uace8\ubb34\ub97c \ub07c\uace0 \ube68\ub798\ud310\uc744 \ubb38\uc9c8\ub7ec \uc6d0\uc2dc\uc801\uc774\uba74\uc11c \uac15\ub82c\ud55c \ub9ac\ub4ec\uc744 \ub9cc\ub4e4\uc5c8\ub2e4. \uba87 \ub144 \uc804\ubd80\ud130 \uac00\uc871\ub4e4\uacfc \ucda4\uc744 \uc990\uacbc\ub358 \uc2a4\ud0c0\ud0a4\ub294 \ud2b8\ub798\ud3ec\ub4dc\uc640 \uc7a0\uc2dc \ub450 \ud559\uad50\uc5d0\uc11c \uc5f4\ub9ac\ub294 \ucda4 \uac15\uc2b5\uc5d0\ub3c4 \ucc38\uc5ec\ud588\ub2e4. \ube44\ub85d \uac15\uc2b5\uc740 \ub2e8\uba85\ud588\uc73c\ub098 \uadf8\ub4e4\uc5d0\uac8c \ucda4\uc758 \uc9c0\uce68\uc11c\ub97c \uc81c\uacf5\ud588\uace0, \ubc24 \uc678\ucd9c\uc5d0\uc11c \uadf8\ub4e4\uc740 \ub2a5\uc219\ud55c \ucda4 \uc2e4\ub825\uc744 \ubf50\ub0bc \uc218 \uc788\uc5c8\ub2e4.", "answer": "\ube68\ub798\ud310", "sentence": "\ubc34\ub4dc\ub294 \u3008Rock Island Line\u3009, \u3008Walking Cane\u3009 \ub4f1 \uc720\uba85 \uc2a4\ud0a4\ud50c \uace1\uc744 \uc5f0\uc8fc\ud588\uace0, \uc2a4\ud0c0\ud0a4\ub294 \uace8\ubb34\ub97c \ub07c\uace0 \ube68\ub798\ud310 \uc744 \ubb38\uc9c8\ub7ec \uc6d0\uc2dc\uc801\uc774\uba74\uc11c \uac15\ub82c\ud55c \ub9ac\ub4ec\uc744 \ub9cc\ub4e4\uc5c8\ub2e4.", "paragraph_sentence": "\ud2b8\ub798\ud3ec\ub4dc\uac00 \uc2a4\ud0c0\ud0a4\uc758 \uc2a4\ud0a4\ud50c \ud765\ubbf8\ub97c \uc7a1\uc544\ub048\ub300 \uc774\uc5b4 \uadf8 \ub450 \uba85\uc740 \uc81c\uc870\uacf5\uc7a5\uc758 \uc9c0\ud558\uc800\uc7a5\uc2e4\uc5d0\uc11c \uc810\uc2ec\uc2dc\uac04\uc774 \ub418\uba74 \uace1\uc744 \uc5f0\uc2b5\ud558\uae30 \uc2dc\uc791\ud588\ub2e4. \ud2b8\ub798\ud3ec\ub4dc\uac00 \ud68c\uc0c1\ud558\uae30\ub97c \"\uc800\ub294 \uae30\ud0c0\ub97c \uc5f0\uc8fc\ud588\uace0, [\ub9ac\uce58]\ub294 \ub2e8\uc21c\ud788 \uc0c1\uc790\ub85c \uc18c\uc74c\uc744 \ub0c8\uc8e0 ... \uac00\ub054\uc529 \uadf8 \ub140\uc11d\uc740 \uacfc\uc790\uc0c1\uc790\ub97c \uc5f4\uc1e0\ubb49\uce58\ub85c \ub0b4\ub824\uc9c0\uac70\ub098 \uc758\uc790 \ub4f1\ubc1b\uc774\ub97c \ub54c\ub9ac\uae30\ub3c4 \ud588\uc5b4\uc694.\" \uadf8\ub4e4\uc740 \uc2a4\ud0c0\ud0a4\uc758 \uc774\uc6c3\uc774\uc790 \uc9c1\uc7a5\ub3d9\ub8cc\uc778 \uae30\ud0c0\ub9ac\uc2a4\ud2b8 \uc5d0\ub514 \ub9c8\uc77c\uc2a4\uc640 \ud569\ub958\ud574 \uc5d0\ub514 \ub9c8\uc77c\uc2a4 \ubc34\ub4dc\ub97c \uacb0\uc131\ud588\ub2e4. \uc774\ud6c4 \ub9ac\ubc84\ud480 \uba85\uc18c\uc758 \uc774\ub984\uc744 \ub530 \uc5d0\ub514 \ud074\ub808\uc774\ud2bc \uc564 \ub354 \ud074\ub808\uc774\ud2bc \uc2a4\ud018\uc5b4\ub85c \uac1c\uba85\ud588\ub2e4. \ubc34\ub4dc\ub294 \u3008Rock Island Line\u3009, \u3008Walking Cane\u3009 \ub4f1 \uc720\uba85 \uc2a4\ud0a4\ud50c \uace1\uc744 \uc5f0\uc8fc\ud588\uace0, \uc2a4\ud0c0\ud0a4\ub294 \uace8\ubb34\ub97c \ub07c\uace0 \ube68\ub798\ud310 \uc744 \ubb38\uc9c8\ub7ec \uc6d0\uc2dc\uc801\uc774\uba74\uc11c \uac15\ub82c\ud55c \ub9ac\ub4ec\uc744 \ub9cc\ub4e4\uc5c8\ub2e4. \uba87 \ub144 \uc804\ubd80\ud130 \uac00\uc871\ub4e4\uacfc \ucda4\uc744 \uc990\uacbc\ub358 \uc2a4\ud0c0\ud0a4\ub294 \ud2b8\ub798\ud3ec\ub4dc\uc640 \uc7a0\uc2dc \ub450 \ud559\uad50\uc5d0\uc11c \uc5f4\ub9ac\ub294 \ucda4 \uac15\uc2b5\uc5d0\ub3c4 \ucc38\uc5ec\ud588\ub2e4. \ube44\ub85d \uac15\uc2b5\uc740 \ub2e8\uba85\ud588\uc73c\ub098 \uadf8\ub4e4\uc5d0\uac8c \ucda4\uc758 \uc9c0\uce68\uc11c\ub97c \uc81c\uacf5\ud588\uace0, \ubc24 \uc678\ucd9c\uc5d0\uc11c \uadf8\ub4e4\uc740 \ub2a5\uc219\ud55c \ucda4 \uc2e4\ub825\uc744 \ubf50\ub0bc \uc218 \uc788\uc5c8\ub2e4.", "paragraph_answer": "\ud2b8\ub798\ud3ec\ub4dc\uac00 \uc2a4\ud0c0\ud0a4\uc758 \uc2a4\ud0a4\ud50c \ud765\ubbf8\ub97c \uc7a1\uc544\ub048\ub300 \uc774\uc5b4 \uadf8 \ub450 \uba85\uc740 \uc81c\uc870\uacf5\uc7a5\uc758 \uc9c0\ud558\uc800\uc7a5\uc2e4\uc5d0\uc11c \uc810\uc2ec\uc2dc\uac04\uc774 \ub418\uba74 \uace1\uc744 \uc5f0\uc2b5\ud558\uae30 \uc2dc\uc791\ud588\ub2e4. \ud2b8\ub798\ud3ec\ub4dc\uac00 \ud68c\uc0c1\ud558\uae30\ub97c \"\uc800\ub294 \uae30\ud0c0\ub97c \uc5f0\uc8fc\ud588\uace0, [\ub9ac\uce58]\ub294 \ub2e8\uc21c\ud788 \uc0c1\uc790\ub85c \uc18c\uc74c\uc744 \ub0c8\uc8e0 ... \uac00\ub054\uc529 \uadf8 \ub140\uc11d\uc740 \uacfc\uc790\uc0c1\uc790\ub97c \uc5f4\uc1e0\ubb49\uce58\ub85c \ub0b4\ub824\uc9c0\uac70\ub098 \uc758\uc790 \ub4f1\ubc1b\uc774\ub97c \ub54c\ub9ac\uae30\ub3c4 \ud588\uc5b4\uc694.\" \uadf8\ub4e4\uc740 \uc2a4\ud0c0\ud0a4\uc758 \uc774\uc6c3\uc774\uc790 \uc9c1\uc7a5\ub3d9\ub8cc\uc778 \uae30\ud0c0\ub9ac\uc2a4\ud2b8 \uc5d0\ub514 \ub9c8\uc77c\uc2a4\uc640 \ud569\ub958\ud574 \uc5d0\ub514 \ub9c8\uc77c\uc2a4 \ubc34\ub4dc\ub97c \uacb0\uc131\ud588\ub2e4. \uc774\ud6c4 \ub9ac\ubc84\ud480 \uba85\uc18c\uc758 \uc774\ub984\uc744 \ub530 \uc5d0\ub514 \ud074\ub808\uc774\ud2bc \uc564 \ub354 \ud074\ub808\uc774\ud2bc \uc2a4\ud018\uc5b4\ub85c \uac1c\uba85\ud588\ub2e4. \ubc34\ub4dc\ub294 \u3008Rock Island Line\u3009, \u3008Walking Cane\u3009 \ub4f1 \uc720\uba85 \uc2a4\ud0a4\ud50c \uace1\uc744 \uc5f0\uc8fc\ud588\uace0, \uc2a4\ud0c0\ud0a4\ub294 \uace8\ubb34\ub97c \ub07c\uace0 \ube68\ub798\ud310 \uc744 \ubb38\uc9c8\ub7ec \uc6d0\uc2dc\uc801\uc774\uba74\uc11c \uac15\ub82c\ud55c \ub9ac\ub4ec\uc744 \ub9cc\ub4e4\uc5c8\ub2e4. \uba87 \ub144 \uc804\ubd80\ud130 \uac00\uc871\ub4e4\uacfc \ucda4\uc744 \uc990\uacbc\ub358 \uc2a4\ud0c0\ud0a4\ub294 \ud2b8\ub798\ud3ec\ub4dc\uc640 \uc7a0\uc2dc \ub450 \ud559\uad50\uc5d0\uc11c \uc5f4\ub9ac\ub294 \ucda4 \uac15\uc2b5\uc5d0\ub3c4 \ucc38\uc5ec\ud588\ub2e4. \ube44\ub85d \uac15\uc2b5\uc740 \ub2e8\uba85\ud588\uc73c\ub098 \uadf8\ub4e4\uc5d0\uac8c \ucda4\uc758 \uc9c0\uce68\uc11c\ub97c \uc81c\uacf5\ud588\uace0, \ubc24 \uc678\ucd9c\uc5d0\uc11c \uadf8\ub4e4\uc740 \ub2a5\uc219\ud55c \ucda4 \uc2e4\ub825\uc744 \ubf50\ub0bc \uc218 \uc788\uc5c8\ub2e4.", "sentence_answer": "\ubc34\ub4dc\ub294 \u3008Rock Island Line\u3009, \u3008Walking Cane\u3009 \ub4f1 \uc720\uba85 \uc2a4\ud0a4\ud50c \uace1\uc744 \uc5f0\uc8fc\ud588\uace0, \uc2a4\ud0c0\ud0a4\ub294 \uace8\ubb34\ub97c \ub07c\uace0 \ube68\ub798\ud310 \uc744 \ubb38\uc9c8\ub7ec \uc6d0\uc2dc\uc801\uc774\uba74\uc11c \uac15\ub82c\ud55c \ub9ac\ub4ec\uc744 \ub9cc\ub4e4\uc5c8\ub2e4.", "paragraph_id": "6314792-7-1", "paragraph_question": "question: \uc2a4\ud0c0\ud0a4\uac00 \uace8\ubb34\ub97c \ucc29\uc6a9\ud558\uace0 \ubb38\uc9c0\ub978 \uac83\uc73c\ub85c \uc6d0\uc2dc\uc801\uc774\uace0 \uac15\ub82c\ud55c \ub9ac\ub4ec\uc744 \ub0c8\ub358 \uac83\uc740?, context: \ud2b8\ub798\ud3ec\ub4dc\uac00 \uc2a4\ud0c0\ud0a4\uc758 \uc2a4\ud0a4\ud50c \ud765\ubbf8\ub97c \uc7a1\uc544\ub048\ub300 \uc774\uc5b4 \uadf8 \ub450 \uba85\uc740 \uc81c\uc870\uacf5\uc7a5\uc758 \uc9c0\ud558\uc800\uc7a5\uc2e4\uc5d0\uc11c \uc810\uc2ec\uc2dc\uac04\uc774 \ub418\uba74 \uace1\uc744 \uc5f0\uc2b5\ud558\uae30 \uc2dc\uc791\ud588\ub2e4. \ud2b8\ub798\ud3ec\ub4dc\uac00 \ud68c\uc0c1\ud558\uae30\ub97c \"\uc800\ub294 \uae30\ud0c0\ub97c \uc5f0\uc8fc\ud588\uace0, [\ub9ac\uce58]\ub294 \ub2e8\uc21c\ud788 \uc0c1\uc790\ub85c \uc18c\uc74c\uc744 \ub0c8\uc8e0 ... \uac00\ub054\uc529 \uadf8 \ub140\uc11d\uc740 \uacfc\uc790\uc0c1\uc790\ub97c \uc5f4\uc1e0\ubb49\uce58\ub85c \ub0b4\ub824\uc9c0\uac70\ub098 \uc758\uc790 \ub4f1\ubc1b\uc774\ub97c \ub54c\ub9ac\uae30\ub3c4 \ud588\uc5b4\uc694.\" \uadf8\ub4e4\uc740 \uc2a4\ud0c0\ud0a4\uc758 \uc774\uc6c3\uc774\uc790 \uc9c1\uc7a5\ub3d9\ub8cc\uc778 \uae30\ud0c0\ub9ac\uc2a4\ud2b8 \uc5d0\ub514 \ub9c8\uc77c\uc2a4\uc640 \ud569\ub958\ud574 \uc5d0\ub514 \ub9c8\uc77c\uc2a4 \ubc34\ub4dc\ub97c \uacb0\uc131\ud588\ub2e4. \uc774\ud6c4 \ub9ac\ubc84\ud480 \uba85\uc18c\uc758 \uc774\ub984\uc744 \ub530 \uc5d0\ub514 \ud074\ub808\uc774\ud2bc \uc564 \ub354 \ud074\ub808\uc774\ud2bc \uc2a4\ud018\uc5b4\ub85c \uac1c\uba85\ud588\ub2e4. \ubc34\ub4dc\ub294 \u3008Rock Island Line\u3009, \u3008Walking Cane\u3009 \ub4f1 \uc720\uba85 \uc2a4\ud0a4\ud50c \uace1\uc744 \uc5f0\uc8fc\ud588\uace0, \uc2a4\ud0c0\ud0a4\ub294 \uace8\ubb34\ub97c \ub07c\uace0 \ube68\ub798\ud310\uc744 \ubb38\uc9c8\ub7ec \uc6d0\uc2dc\uc801\uc774\uba74\uc11c \uac15\ub82c\ud55c \ub9ac\ub4ec\uc744 \ub9cc\ub4e4\uc5c8\ub2e4. \uba87 \ub144 \uc804\ubd80\ud130 \uac00\uc871\ub4e4\uacfc \ucda4\uc744 \uc990\uacbc\ub358 \uc2a4\ud0c0\ud0a4\ub294 \ud2b8\ub798\ud3ec\ub4dc\uc640 \uc7a0\uc2dc \ub450 \ud559\uad50\uc5d0\uc11c \uc5f4\ub9ac\ub294 \ucda4 \uac15\uc2b5\uc5d0\ub3c4 \ucc38\uc5ec\ud588\ub2e4. \ube44\ub85d \uac15\uc2b5\uc740 \ub2e8\uba85\ud588\uc73c\ub098 \uadf8\ub4e4\uc5d0\uac8c \ucda4\uc758 \uc9c0\uce68\uc11c\ub97c \uc81c\uacf5\ud588\uace0, \ubc24 \uc678\ucd9c\uc5d0\uc11c \uadf8\ub4e4\uc740 \ub2a5\uc219\ud55c \ucda4 \uc2e4\ub825\uc744 \ubf50\ub0bc \uc218 \uc788\uc5c8\ub2e4."} {"question": "\ub7ec\uc2dc\uc544, \ud504\ub85c\uc774\uc13c, \uc624\uc2a4\ud2b8\ub9ac\uc544 \uac04\uc758 \ud3f4\ub780\ub4dc \ubd84\ud560 \uc870\uc57d\uc774 \uccb4\uacb0\ub41c \uc5f0\ub3c4\ub294?", "paragraph": "\uc624\uc2a4\ud2b8\ub9ac\uc544\uac00 1769-1770\ub144\uc5d0 \ub3c5\uc77c\uacc4 \uc8fc\ubbfc\ub4e4\uc774 \uc0b4\uace0\uc788\ub294 \uc2a4\ud53c\uc2dc(Spisz)\uc9c0\uc5ed\uacfc \ub178\ube44 \ud0c0\ub974\ud06c(Nowy Targ), \ub178\ube44 \uc1a1\uce58, \uc2a4\ud0c0\ub9ac \uc1a1\uce58 \ub4f1 \ud3f4\ub780\ub4dc \ub0a8\ubd80\uc9c0\uc5ed\uc744 \ubcd1\ud569\ud588\ub294\ub370\ub3c4 \uc774\uc5d0 \ub300\ud574\uc11c \ud3f4\ub780\ub4dc\ub294 \uc5b4\ub5a4 \ubb3c\ub9ac\uc801 \ub300\uc751\ub3c4 \ud558\uc9c0 \ubabb\ud588\ub2e4. \uc774\ub807\uac8c \uc601\ud1a0\ub97c \uc810\ub839 \ub2f9\ud574\ub3c4 \ubb34\ubc29\ube44\uc0c1\ud0dc\uc778 \ud3f4\ub780\ub4dc\ub97c \ub193\uace0 \ub7ec\uc2dc\uc544, \ud504\ub85c\uc774\uc13c, \uc624\uc2a4\ud2b8\ub9ac\uc544\ub294 1771\ub144 \ucd08\ubd80\ud130 \ubcf8\uaca9\uc801\uc73c\ub85c \ud3f4\ub780\ub4dc \ubd84\ud560\uc744 \ub17c\uc758\ud558\uae30 \uc2dc\uc791\ud588\ub2e4. 1772\ub144 8\uc6d4 5\uc77c \ub7ec\uc2dc\uc544, \ud504\ub85c\uc774\uc13c, \uc624\uc2a4\ud2b8\ub9ac\uc544\ub294 \ud3f4\ub780\ub4dc \ubd84\ud560 \uc870\uc57d(1\ucc28 \ubd84\ud560)\uc744 \uccb4\uacb0\ud558\uace0 \uace7 \uc774\uc5b4\uc11c \uac01\uac01 \ud3f4\ub780\ub4dc\ub97c \uc810\ub839\ud588\ub2e4. \uc774 \uc870\uc57d\uc740 1773\ub144 8\uc6d4 \uc0c8\ub85c \uc120\ucd9c\ub41c \uc758\ud68c\uc758 \ub300\ud45c\ub2e8\uc5d0 \uc758\ud574\uc11c \ub3d9\uc758\ub97c \ubc1b\uc558\ub294\ub370 \uc774\ub54c\ub294 \ub9cc\uc7a5\uc77c\uce58\uc81c\uac00 \uc801\uc6a9\ub418\uc9c0 \uc54a\uace0 \ub2e4\uc218\uacb0\ub85c \uacb0\uc815\ub418\uc5c8\ub2e4. \ud3f4\ub780\ub4dc \ubd84\ud560\uc740 \uac15\ub300\uad6d\ub4e4\uc774 \uc57d\uc18c\uad6d\uc744 \uc790\uae30\ub4e4\uc758 \uc774\ud574\uad00\uacc4\uc5d0 \ub530\ub77c \uc790\uc758\uc801\uc73c\ub85c \ub098\ub204\uc5b4 \uac00\uc9c0\ub294 \uace0\uc804\uc801\uc778 \uc601\ud1a0\ud33d\ucc3d \uc218\ubc95\uc758 \uc804\ub840\uac00 \ub418\uc5c8\ub2e4.", "answer": "1772\ub144", "sentence": "1772\ub144 8\uc6d4 5\uc77c \ub7ec\uc2dc\uc544, \ud504\ub85c\uc774\uc13c, \uc624\uc2a4\ud2b8\ub9ac\uc544\ub294 \ud3f4\ub780\ub4dc \ubd84\ud560 \uc870\uc57d(1\ucc28 \ubd84\ud560)\uc744 \uccb4\uacb0\ud558\uace0 \uace7 \uc774\uc5b4\uc11c \uac01\uac01 \ud3f4\ub780\ub4dc\ub97c \uc810\ub839\ud588\ub2e4.", "paragraph_sentence": "\uc624\uc2a4\ud2b8\ub9ac\uc544\uac00 1769-1770\ub144\uc5d0 \ub3c5\uc77c\uacc4 \uc8fc\ubbfc\ub4e4\uc774 \uc0b4\uace0\uc788\ub294 \uc2a4\ud53c\uc2dc(Spisz)\uc9c0\uc5ed\uacfc \ub178\ube44 \ud0c0\ub974\ud06c(Nowy Targ), \ub178\ube44 \uc1a1\uce58, \uc2a4\ud0c0\ub9ac \uc1a1\uce58 \ub4f1 \ud3f4\ub780\ub4dc \ub0a8\ubd80\uc9c0\uc5ed\uc744 \ubcd1\ud569\ud588\ub294\ub370\ub3c4 \uc774\uc5d0 \ub300\ud574\uc11c \ud3f4\ub780\ub4dc\ub294 \uc5b4\ub5a4 \ubb3c\ub9ac\uc801 \ub300\uc751\ub3c4 \ud558\uc9c0 \ubabb\ud588\ub2e4. \uc774\ub807\uac8c \uc601\ud1a0\ub97c \uc810\ub839 \ub2f9\ud574\ub3c4 \ubb34\ubc29\ube44\uc0c1\ud0dc\uc778 \ud3f4\ub780\ub4dc\ub97c \ub193\uace0 \ub7ec\uc2dc\uc544, \ud504\ub85c\uc774\uc13c, \uc624\uc2a4\ud2b8\ub9ac\uc544\ub294 1771\ub144 \ucd08\ubd80\ud130 \ubcf8\uaca9\uc801\uc73c\ub85c \ud3f4\ub780\ub4dc \ubd84\ud560\uc744 \ub17c\uc758\ud558\uae30 \uc2dc\uc791\ud588\ub2e4. 1772\ub144 8\uc6d4 5\uc77c \ub7ec\uc2dc\uc544, \ud504\ub85c\uc774\uc13c, \uc624\uc2a4\ud2b8\ub9ac\uc544\ub294 \ud3f4\ub780\ub4dc \ubd84\ud560 \uc870\uc57d(1\ucc28 \ubd84\ud560)\uc744 \uccb4\uacb0\ud558\uace0 \uace7 \uc774\uc5b4\uc11c \uac01\uac01 \ud3f4\ub780\ub4dc\ub97c \uc810\ub839\ud588\ub2e4. \uc774 \uc870\uc57d\uc740 1773\ub144 8\uc6d4 \uc0c8\ub85c \uc120\ucd9c\ub41c \uc758\ud68c\uc758 \ub300\ud45c\ub2e8\uc5d0 \uc758\ud574\uc11c \ub3d9\uc758\ub97c \ubc1b\uc558\ub294\ub370 \uc774\ub54c\ub294 \ub9cc\uc7a5\uc77c\uce58\uc81c\uac00 \uc801\uc6a9\ub418\uc9c0 \uc54a\uace0 \ub2e4\uc218\uacb0\ub85c \uacb0\uc815\ub418\uc5c8\ub2e4. \ud3f4\ub780\ub4dc \ubd84\ud560\uc740 \uac15\ub300\uad6d\ub4e4\uc774 \uc57d\uc18c\uad6d\uc744 \uc790\uae30\ub4e4\uc758 \uc774\ud574\uad00\uacc4\uc5d0 \ub530\ub77c \uc790\uc758\uc801\uc73c\ub85c \ub098\ub204\uc5b4 \uac00\uc9c0\ub294 \uace0\uc804\uc801\uc778 \uc601\ud1a0\ud33d\ucc3d \uc218\ubc95\uc758 \uc804\ub840\uac00 \ub418\uc5c8\ub2e4.", "paragraph_answer": "\uc624\uc2a4\ud2b8\ub9ac\uc544\uac00 1769-1770\ub144\uc5d0 \ub3c5\uc77c\uacc4 \uc8fc\ubbfc\ub4e4\uc774 \uc0b4\uace0\uc788\ub294 \uc2a4\ud53c\uc2dc(Spisz)\uc9c0\uc5ed\uacfc \ub178\ube44 \ud0c0\ub974\ud06c(Nowy Targ), \ub178\ube44 \uc1a1\uce58, \uc2a4\ud0c0\ub9ac \uc1a1\uce58 \ub4f1 \ud3f4\ub780\ub4dc \ub0a8\ubd80\uc9c0\uc5ed\uc744 \ubcd1\ud569\ud588\ub294\ub370\ub3c4 \uc774\uc5d0 \ub300\ud574\uc11c \ud3f4\ub780\ub4dc\ub294 \uc5b4\ub5a4 \ubb3c\ub9ac\uc801 \ub300\uc751\ub3c4 \ud558\uc9c0 \ubabb\ud588\ub2e4. \uc774\ub807\uac8c \uc601\ud1a0\ub97c \uc810\ub839 \ub2f9\ud574\ub3c4 \ubb34\ubc29\ube44\uc0c1\ud0dc\uc778 \ud3f4\ub780\ub4dc\ub97c \ub193\uace0 \ub7ec\uc2dc\uc544, \ud504\ub85c\uc774\uc13c, \uc624\uc2a4\ud2b8\ub9ac\uc544\ub294 1771\ub144 \ucd08\ubd80\ud130 \ubcf8\uaca9\uc801\uc73c\ub85c \ud3f4\ub780\ub4dc \ubd84\ud560\uc744 \ub17c\uc758\ud558\uae30 \uc2dc\uc791\ud588\ub2e4. 1772\ub144 8\uc6d4 5\uc77c \ub7ec\uc2dc\uc544, \ud504\ub85c\uc774\uc13c, \uc624\uc2a4\ud2b8\ub9ac\uc544\ub294 \ud3f4\ub780\ub4dc \ubd84\ud560 \uc870\uc57d(1\ucc28 \ubd84\ud560)\uc744 \uccb4\uacb0\ud558\uace0 \uace7 \uc774\uc5b4\uc11c \uac01\uac01 \ud3f4\ub780\ub4dc\ub97c \uc810\ub839\ud588\ub2e4. \uc774 \uc870\uc57d\uc740 1773\ub144 8\uc6d4 \uc0c8\ub85c \uc120\ucd9c\ub41c \uc758\ud68c\uc758 \ub300\ud45c\ub2e8\uc5d0 \uc758\ud574\uc11c \ub3d9\uc758\ub97c \ubc1b\uc558\ub294\ub370 \uc774\ub54c\ub294 \ub9cc\uc7a5\uc77c\uce58\uc81c\uac00 \uc801\uc6a9\ub418\uc9c0 \uc54a\uace0 \ub2e4\uc218\uacb0\ub85c \uacb0\uc815\ub418\uc5c8\ub2e4. \ud3f4\ub780\ub4dc \ubd84\ud560\uc740 \uac15\ub300\uad6d\ub4e4\uc774 \uc57d\uc18c\uad6d\uc744 \uc790\uae30\ub4e4\uc758 \uc774\ud574\uad00\uacc4\uc5d0 \ub530\ub77c \uc790\uc758\uc801\uc73c\ub85c \ub098\ub204\uc5b4 \uac00\uc9c0\ub294 \uace0\uc804\uc801\uc778 \uc601\ud1a0\ud33d\ucc3d \uc218\ubc95\uc758 \uc804\ub840\uac00 \ub418\uc5c8\ub2e4.", "sentence_answer": " 1772\ub144 8\uc6d4 5\uc77c \ub7ec\uc2dc\uc544, \ud504\ub85c\uc774\uc13c, \uc624\uc2a4\ud2b8\ub9ac\uc544\ub294 \ud3f4\ub780\ub4dc \ubd84\ud560 \uc870\uc57d(1\ucc28 \ubd84\ud560)\uc744 \uccb4\uacb0\ud558\uace0 \uace7 \uc774\uc5b4\uc11c \uac01\uac01 \ud3f4\ub780\ub4dc\ub97c \uc810\ub839\ud588\ub2e4.", "paragraph_id": "6600375-11-1", "paragraph_question": "question: \ub7ec\uc2dc\uc544, \ud504\ub85c\uc774\uc13c, \uc624\uc2a4\ud2b8\ub9ac\uc544 \uac04\uc758 \ud3f4\ub780\ub4dc \ubd84\ud560 \uc870\uc57d\uc774 \uccb4\uacb0\ub41c \uc5f0\ub3c4\ub294?, context: \uc624\uc2a4\ud2b8\ub9ac\uc544\uac00 1769-1770\ub144\uc5d0 \ub3c5\uc77c\uacc4 \uc8fc\ubbfc\ub4e4\uc774 \uc0b4\uace0\uc788\ub294 \uc2a4\ud53c\uc2dc(Spisz)\uc9c0\uc5ed\uacfc \ub178\ube44 \ud0c0\ub974\ud06c(Nowy Targ), \ub178\ube44 \uc1a1\uce58, \uc2a4\ud0c0\ub9ac \uc1a1\uce58 \ub4f1 \ud3f4\ub780\ub4dc \ub0a8\ubd80\uc9c0\uc5ed\uc744 \ubcd1\ud569\ud588\ub294\ub370\ub3c4 \uc774\uc5d0 \ub300\ud574\uc11c \ud3f4\ub780\ub4dc\ub294 \uc5b4\ub5a4 \ubb3c\ub9ac\uc801 \ub300\uc751\ub3c4 \ud558\uc9c0 \ubabb\ud588\ub2e4. \uc774\ub807\uac8c \uc601\ud1a0\ub97c \uc810\ub839 \ub2f9\ud574\ub3c4 \ubb34\ubc29\ube44\uc0c1\ud0dc\uc778 \ud3f4\ub780\ub4dc\ub97c \ub193\uace0 \ub7ec\uc2dc\uc544, \ud504\ub85c\uc774\uc13c, \uc624\uc2a4\ud2b8\ub9ac\uc544\ub294 1771\ub144 \ucd08\ubd80\ud130 \ubcf8\uaca9\uc801\uc73c\ub85c \ud3f4\ub780\ub4dc \ubd84\ud560\uc744 \ub17c\uc758\ud558\uae30 \uc2dc\uc791\ud588\ub2e4. 1772\ub144 8\uc6d4 5\uc77c \ub7ec\uc2dc\uc544, \ud504\ub85c\uc774\uc13c, \uc624\uc2a4\ud2b8\ub9ac\uc544\ub294 \ud3f4\ub780\ub4dc \ubd84\ud560 \uc870\uc57d(1\ucc28 \ubd84\ud560)\uc744 \uccb4\uacb0\ud558\uace0 \uace7 \uc774\uc5b4\uc11c \uac01\uac01 \ud3f4\ub780\ub4dc\ub97c \uc810\ub839\ud588\ub2e4. \uc774 \uc870\uc57d\uc740 1773\ub144 8\uc6d4 \uc0c8\ub85c \uc120\ucd9c\ub41c \uc758\ud68c\uc758 \ub300\ud45c\ub2e8\uc5d0 \uc758\ud574\uc11c \ub3d9\uc758\ub97c \ubc1b\uc558\ub294\ub370 \uc774\ub54c\ub294 \ub9cc\uc7a5\uc77c\uce58\uc81c\uac00 \uc801\uc6a9\ub418\uc9c0 \uc54a\uace0 \ub2e4\uc218\uacb0\ub85c \uacb0\uc815\ub418\uc5c8\ub2e4. \ud3f4\ub780\ub4dc \ubd84\ud560\uc740 \uac15\ub300\uad6d\ub4e4\uc774 \uc57d\uc18c\uad6d\uc744 \uc790\uae30\ub4e4\uc758 \uc774\ud574\uad00\uacc4\uc5d0 \ub530\ub77c \uc790\uc758\uc801\uc73c\ub85c \ub098\ub204\uc5b4 \uac00\uc9c0\ub294 \uace0\uc804\uc801\uc778 \uc601\ud1a0\ud33d\ucc3d \uc218\ubc95\uc758 \uc804\ub840\uac00 \ub418\uc5c8\ub2e4."} {"question": "2013\ub144\uc5d0 \uacf5\uac1c\ud55c \uc5e0\ub9c8 \uc2a4\ud1a4\uc774 \ubaa9\uc18c\ub9ac \uc5f0\uae30\ub97c \ub9e1\uc740 \uc560\ub2c8\uba54\uc774\uc158\uc758 \uc81c\ubaa9\uc740?", "paragraph": "\uc2a4\ud1a4\uc740 2013\ub144\ubd80\ud130 \ub4dc\ub9bc\uc6cd\uc2a4 \uc560\ub2c8\uba54\uc774\uc158 \uc81c\uc791\uc758 \ucef4\ud4e8\ud130 \uc560\ub2c8\uba54\uc774\uc158 \uc601\ud654 \u300a\ud06c\ub8e8\uc988 \ud328\ubc00\ub9ac\u300b\uc5d0\uc11c \ub77c\uc774\uc5b8 \ub808\uc774\ub180\uc988\uac00 \uc5f0\uae30\ud55c \uce90\ub9ad\ud130\uc758 \uc560\uc778 \ubaa9\uc18c\ub9ac \uc5ed\ud560\uc744 \ub9e1\uae30 \uc2dc\uc791\ud588\uc73c\uba70, \uc601\ud654\ub294 2013\ub144 3\uc6d4 22\uc77c\uc5d0 \uacf5\uac1c\ub418\uc5b4 \ubbf8\uad6d \uc544\uce74\ub370\ubbf8\uc0c1 \ucd5c\uc6b0\uc218 \uc7a5\ud3b8 \uc560\ub2c8\uba54\uc774\uc158 \uc601\ud654 \ud6c4\ubcf4\uc5d0 \uc62c\ub790\ub2e4. \uadf8 \ud6c4 16\uac1c\uc758 \ub2e8\ud3b8\uc73c\ub85c \uad6c\uc131\ub41c \u300a\ubb34\ube44 43\u300b\uc5d0 \ucd9c\uc5f0\ud55c \uadf8\ub140\ub294 \uadf8\uc911\uc5d0\uc11c\ub3c4 \uadf8\ub9ac\ud540 \ub358\uc774 \uac10\ub3c5\ud55c \ub2e8\ud3b8 \ucf54\ubbf8\ub514 \uc601\ud654 \u300a\ubca0\ub85c\ub2c8\uce74\u300b\uc5d0\uc11c \uc8fc\uc5f0\uc744 \ub9e1\uc544 \ud0a4\uc5b4\ub7f0 \uceec\ud0a8\uacfc \ud568\uaed8 \ucd9c\uc5f0\ud558\uc600\ub2e4. \u300a\uc880\ube44\ub79c\ub4dc\u300b\uc758 \uac10\ub3c5\uc778 \ub8e8\ubca4 \ud50c\ub77c\uc774\uc154\uc640\ub294 2013\ub144 \uacf5\uac1c \ub41c \u300a\uac31\uc2a4\ud130 \uc2a4\ucffc\ub4dc\u300b\uc5d0\uc11c \ub2e4\uc2dc \ud611\ub825\ud588\ub2e4. \ub77c\uc774\uc5b8 \uace0\uc2ac\ub9c1\uacfc\ub294 \ub450\ubc88\uc9f8\ub85c \uac19\uc774 \uc5f0\uae30\ud55c \uc791\ud488\uc774\uba70, \uc200 \ud39c, \uc870\uc2dc \ube0c\ub864\ub9b0 \ub4f1\uc774 \ucd9c\uc5f0\ud558\uc600\ub2e4. \uc2a4\ud1a4\uc774 \uc5f0\uae30\ud55c \uadf8\ub808\uc774\uc2a4 \ud328\ub7ec\ub370\uc774\ub294 \uace0\uc2ac\ub9c1\uc774 \uc5f0\uae30\ud55c \uc81c\ub9ac \uc6b0\ud130\uc2a4 \uacbd\uc0ac\uc640 \ud39c\uc774 \uc5f0\uae30\ud55c \ubbf8\ud0a4 \ucf54\ud5e8 \uc0ac\uc774\uc5d0\uc11c \ud754\ub4e4\ub9b0\ub2e4\ub294 \uc5ed\ud560\uc774\ub2e4.", "answer": "\ud06c\ub8e8\uc988 \ud328\ubc00\ub9ac", "sentence": "\uc2a4\ud1a4\uc740 2013\ub144\ubd80\ud130 \ub4dc\ub9bc\uc6cd\uc2a4 \uc560\ub2c8\uba54\uc774\uc158 \uc81c\uc791\uc758 \ucef4\ud4e8\ud130 \uc560\ub2c8\uba54\uc774\uc158 \uc601\ud654 \u300a \ud06c\ub8e8\uc988 \ud328\ubc00\ub9ac \u300b\uc5d0\uc11c \ub77c\uc774\uc5b8 \ub808\uc774\ub180\uc988\uac00 \uc5f0\uae30\ud55c \uce90\ub9ad\ud130\uc758 \uc560\uc778 \ubaa9\uc18c\ub9ac \uc5ed\ud560\uc744 \ub9e1\uae30 \uc2dc\uc791\ud588\uc73c\uba70, \uc601\ud654\ub294 2013\ub144 3\uc6d4 22\uc77c\uc5d0 \uacf5\uac1c\ub418\uc5b4 \ubbf8\uad6d \uc544\uce74\ub370\ubbf8\uc0c1 \ucd5c\uc6b0\uc218 \uc7a5\ud3b8 \uc560\ub2c8\uba54\uc774\uc158 \uc601\ud654 \ud6c4\ubcf4\uc5d0 \uc62c\ub790\ub2e4.", "paragraph_sentence": " \uc2a4\ud1a4\uc740 2013\ub144\ubd80\ud130 \ub4dc\ub9bc\uc6cd\uc2a4 \uc560\ub2c8\uba54\uc774\uc158 \uc81c\uc791\uc758 \ucef4\ud4e8\ud130 \uc560\ub2c8\uba54\uc774\uc158 \uc601\ud654 \u300a \ud06c\ub8e8\uc988 \ud328\ubc00\ub9ac \u300b\uc5d0\uc11c \ub77c\uc774\uc5b8 \ub808\uc774\ub180\uc988\uac00 \uc5f0\uae30\ud55c \uce90\ub9ad\ud130\uc758 \uc560\uc778 \ubaa9\uc18c\ub9ac \uc5ed\ud560\uc744 \ub9e1\uae30 \uc2dc\uc791\ud588\uc73c\uba70, \uc601\ud654\ub294 2013\ub144 3\uc6d4 22\uc77c\uc5d0 \uacf5\uac1c\ub418\uc5b4 \ubbf8\uad6d \uc544\uce74\ub370\ubbf8\uc0c1 \ucd5c\uc6b0\uc218 \uc7a5\ud3b8 \uc560\ub2c8\uba54\uc774\uc158 \uc601\ud654 \ud6c4\ubcf4\uc5d0 \uc62c\ub790\ub2e4. \uadf8 \ud6c4 16\uac1c\uc758 \ub2e8\ud3b8\uc73c\ub85c \uad6c\uc131\ub41c \u300a\ubb34\ube44 43\u300b\uc5d0 \ucd9c\uc5f0\ud55c \uadf8\ub140\ub294 \uadf8\uc911\uc5d0\uc11c\ub3c4 \uadf8\ub9ac\ud540 \ub358\uc774 \uac10\ub3c5\ud55c \ub2e8\ud3b8 \ucf54\ubbf8\ub514 \uc601\ud654 \u300a\ubca0\ub85c\ub2c8\uce74\u300b\uc5d0\uc11c \uc8fc\uc5f0\uc744 \ub9e1\uc544 \ud0a4\uc5b4\ub7f0 \uceec\ud0a8\uacfc \ud568\uaed8 \ucd9c\uc5f0\ud558\uc600\ub2e4. \u300a\uc880\ube44\ub79c\ub4dc\u300b\uc758 \uac10\ub3c5\uc778 \ub8e8\ubca4 \ud50c\ub77c\uc774\uc154\uc640\ub294 2013\ub144 \uacf5\uac1c \ub41c \u300a\uac31\uc2a4\ud130 \uc2a4\ucffc\ub4dc\u300b\uc5d0\uc11c \ub2e4\uc2dc \ud611\ub825\ud588\ub2e4. \ub77c\uc774\uc5b8 \uace0\uc2ac\ub9c1\uacfc\ub294 \ub450\ubc88\uc9f8\ub85c \uac19\uc774 \uc5f0\uae30\ud55c \uc791\ud488\uc774\uba70, \uc200 \ud39c, \uc870\uc2dc \ube0c\ub864\ub9b0 \ub4f1\uc774 \ucd9c\uc5f0\ud558\uc600\ub2e4. \uc2a4\ud1a4\uc774 \uc5f0\uae30\ud55c \uadf8\ub808\uc774\uc2a4 \ud328\ub7ec\ub370\uc774\ub294 \uace0\uc2ac\ub9c1\uc774 \uc5f0\uae30\ud55c \uc81c\ub9ac \uc6b0\ud130\uc2a4 \uacbd\uc0ac\uc640 \ud39c\uc774 \uc5f0\uae30\ud55c \ubbf8\ud0a4 \ucf54\ud5e8 \uc0ac\uc774\uc5d0\uc11c \ud754\ub4e4\ub9b0\ub2e4\ub294 \uc5ed\ud560\uc774\ub2e4.", "paragraph_answer": "\uc2a4\ud1a4\uc740 2013\ub144\ubd80\ud130 \ub4dc\ub9bc\uc6cd\uc2a4 \uc560\ub2c8\uba54\uc774\uc158 \uc81c\uc791\uc758 \ucef4\ud4e8\ud130 \uc560\ub2c8\uba54\uc774\uc158 \uc601\ud654 \u300a \ud06c\ub8e8\uc988 \ud328\ubc00\ub9ac \u300b\uc5d0\uc11c \ub77c\uc774\uc5b8 \ub808\uc774\ub180\uc988\uac00 \uc5f0\uae30\ud55c \uce90\ub9ad\ud130\uc758 \uc560\uc778 \ubaa9\uc18c\ub9ac \uc5ed\ud560\uc744 \ub9e1\uae30 \uc2dc\uc791\ud588\uc73c\uba70, \uc601\ud654\ub294 2013\ub144 3\uc6d4 22\uc77c\uc5d0 \uacf5\uac1c\ub418\uc5b4 \ubbf8\uad6d \uc544\uce74\ub370\ubbf8\uc0c1 \ucd5c\uc6b0\uc218 \uc7a5\ud3b8 \uc560\ub2c8\uba54\uc774\uc158 \uc601\ud654 \ud6c4\ubcf4\uc5d0 \uc62c\ub790\ub2e4. \uadf8 \ud6c4 16\uac1c\uc758 \ub2e8\ud3b8\uc73c\ub85c \uad6c\uc131\ub41c \u300a\ubb34\ube44 43\u300b\uc5d0 \ucd9c\uc5f0\ud55c \uadf8\ub140\ub294 \uadf8\uc911\uc5d0\uc11c\ub3c4 \uadf8\ub9ac\ud540 \ub358\uc774 \uac10\ub3c5\ud55c \ub2e8\ud3b8 \ucf54\ubbf8\ub514 \uc601\ud654 \u300a\ubca0\ub85c\ub2c8\uce74\u300b\uc5d0\uc11c \uc8fc\uc5f0\uc744 \ub9e1\uc544 \ud0a4\uc5b4\ub7f0 \uceec\ud0a8\uacfc \ud568\uaed8 \ucd9c\uc5f0\ud558\uc600\ub2e4. \u300a\uc880\ube44\ub79c\ub4dc\u300b\uc758 \uac10\ub3c5\uc778 \ub8e8\ubca4 \ud50c\ub77c\uc774\uc154\uc640\ub294 2013\ub144 \uacf5\uac1c \ub41c \u300a\uac31\uc2a4\ud130 \uc2a4\ucffc\ub4dc\u300b\uc5d0\uc11c \ub2e4\uc2dc \ud611\ub825\ud588\ub2e4. \ub77c\uc774\uc5b8 \uace0\uc2ac\ub9c1\uacfc\ub294 \ub450\ubc88\uc9f8\ub85c \uac19\uc774 \uc5f0\uae30\ud55c \uc791\ud488\uc774\uba70, \uc200 \ud39c, \uc870\uc2dc \ube0c\ub864\ub9b0 \ub4f1\uc774 \ucd9c\uc5f0\ud558\uc600\ub2e4. \uc2a4\ud1a4\uc774 \uc5f0\uae30\ud55c \uadf8\ub808\uc774\uc2a4 \ud328\ub7ec\ub370\uc774\ub294 \uace0\uc2ac\ub9c1\uc774 \uc5f0\uae30\ud55c \uc81c\ub9ac \uc6b0\ud130\uc2a4 \uacbd\uc0ac\uc640 \ud39c\uc774 \uc5f0\uae30\ud55c \ubbf8\ud0a4 \ucf54\ud5e8 \uc0ac\uc774\uc5d0\uc11c \ud754\ub4e4\ub9b0\ub2e4\ub294 \uc5ed\ud560\uc774\ub2e4.", "sentence_answer": "\uc2a4\ud1a4\uc740 2013\ub144\ubd80\ud130 \ub4dc\ub9bc\uc6cd\uc2a4 \uc560\ub2c8\uba54\uc774\uc158 \uc81c\uc791\uc758 \ucef4\ud4e8\ud130 \uc560\ub2c8\uba54\uc774\uc158 \uc601\ud654 \u300a \ud06c\ub8e8\uc988 \ud328\ubc00\ub9ac \u300b\uc5d0\uc11c \ub77c\uc774\uc5b8 \ub808\uc774\ub180\uc988\uac00 \uc5f0\uae30\ud55c \uce90\ub9ad\ud130\uc758 \uc560\uc778 \ubaa9\uc18c\ub9ac \uc5ed\ud560\uc744 \ub9e1\uae30 \uc2dc\uc791\ud588\uc73c\uba70, \uc601\ud654\ub294 2013\ub144 3\uc6d4 22\uc77c\uc5d0 \uacf5\uac1c\ub418\uc5b4 \ubbf8\uad6d \uc544\uce74\ub370\ubbf8\uc0c1 \ucd5c\uc6b0\uc218 \uc7a5\ud3b8 \uc560\ub2c8\uba54\uc774\uc158 \uc601\ud654 \ud6c4\ubcf4\uc5d0 \uc62c\ub790\ub2e4.", "paragraph_id": "6458263-7-1", "paragraph_question": "question: 2013\ub144\uc5d0 \uacf5\uac1c\ud55c \uc5e0\ub9c8 \uc2a4\ud1a4\uc774 \ubaa9\uc18c\ub9ac \uc5f0\uae30\ub97c \ub9e1\uc740 \uc560\ub2c8\uba54\uc774\uc158\uc758 \uc81c\ubaa9\uc740?, context: \uc2a4\ud1a4\uc740 2013\ub144\ubd80\ud130 \ub4dc\ub9bc\uc6cd\uc2a4 \uc560\ub2c8\uba54\uc774\uc158 \uc81c\uc791\uc758 \ucef4\ud4e8\ud130 \uc560\ub2c8\uba54\uc774\uc158 \uc601\ud654 \u300a\ud06c\ub8e8\uc988 \ud328\ubc00\ub9ac\u300b\uc5d0\uc11c \ub77c\uc774\uc5b8 \ub808\uc774\ub180\uc988\uac00 \uc5f0\uae30\ud55c \uce90\ub9ad\ud130\uc758 \uc560\uc778 \ubaa9\uc18c\ub9ac \uc5ed\ud560\uc744 \ub9e1\uae30 \uc2dc\uc791\ud588\uc73c\uba70, \uc601\ud654\ub294 2013\ub144 3\uc6d4 22\uc77c\uc5d0 \uacf5\uac1c\ub418\uc5b4 \ubbf8\uad6d \uc544\uce74\ub370\ubbf8\uc0c1 \ucd5c\uc6b0\uc218 \uc7a5\ud3b8 \uc560\ub2c8\uba54\uc774\uc158 \uc601\ud654 \ud6c4\ubcf4\uc5d0 \uc62c\ub790\ub2e4. \uadf8 \ud6c4 16\uac1c\uc758 \ub2e8\ud3b8\uc73c\ub85c \uad6c\uc131\ub41c \u300a\ubb34\ube44 43\u300b\uc5d0 \ucd9c\uc5f0\ud55c \uadf8\ub140\ub294 \uadf8\uc911\uc5d0\uc11c\ub3c4 \uadf8\ub9ac\ud540 \ub358\uc774 \uac10\ub3c5\ud55c \ub2e8\ud3b8 \ucf54\ubbf8\ub514 \uc601\ud654 \u300a\ubca0\ub85c\ub2c8\uce74\u300b\uc5d0\uc11c \uc8fc\uc5f0\uc744 \ub9e1\uc544 \ud0a4\uc5b4\ub7f0 \uceec\ud0a8\uacfc \ud568\uaed8 \ucd9c\uc5f0\ud558\uc600\ub2e4. \u300a\uc880\ube44\ub79c\ub4dc\u300b\uc758 \uac10\ub3c5\uc778 \ub8e8\ubca4 \ud50c\ub77c\uc774\uc154\uc640\ub294 2013\ub144 \uacf5\uac1c \ub41c \u300a\uac31\uc2a4\ud130 \uc2a4\ucffc\ub4dc\u300b\uc5d0\uc11c \ub2e4\uc2dc \ud611\ub825\ud588\ub2e4. \ub77c\uc774\uc5b8 \uace0\uc2ac\ub9c1\uacfc\ub294 \ub450\ubc88\uc9f8\ub85c \uac19\uc774 \uc5f0\uae30\ud55c \uc791\ud488\uc774\uba70, \uc200 \ud39c, \uc870\uc2dc \ube0c\ub864\ub9b0 \ub4f1\uc774 \ucd9c\uc5f0\ud558\uc600\ub2e4. \uc2a4\ud1a4\uc774 \uc5f0\uae30\ud55c \uadf8\ub808\uc774\uc2a4 \ud328\ub7ec\ub370\uc774\ub294 \uace0\uc2ac\ub9c1\uc774 \uc5f0\uae30\ud55c \uc81c\ub9ac \uc6b0\ud130\uc2a4 \uacbd\uc0ac\uc640 \ud39c\uc774 \uc5f0\uae30\ud55c \ubbf8\ud0a4 \ucf54\ud5e8 \uc0ac\uc774\uc5d0\uc11c \ud754\ub4e4\ub9b0\ub2e4\ub294 \uc5ed\ud560\uc774\ub2e4."} {"question": "6\uc6d4 16\uc77c\uc5d0 \uad6c\uc131\ub41c \uacbd\ucc30\uac1c\ud601\uc704\uc6d0\ud68c\uc758 \uc704\uc6d0\uc7a5\uc740 \ub204\uad6c\uc778\uac00?", "paragraph": "6\uc6d4 16\uc77c\uc5d0\ub294 \uacbd\ucc30\uac1c\ud601\uc704\uc6d0\ud68c\ub97c \uad6c\uc131\ud558\uc600\ub294\ub370 \uacbd\ucc30\uc758 \ubcc0\ud654 \ubc29\uc548\uc744 \ub9c8\ub828\ud558\ub294 \uc790\ubb38\uae30\uad6c\ub85c \ud65c\ub3d9\ud558\uac8c \ub41c\ub2e4. \ubc15\uacbd\uc11c \uc704\uc6d0\uc7a5\uc744 \ube44\ub86f\ud558\uc5ec 18\uba85\uc758 \uc704\uc6d0\uc73c\ub85c \uad6c\uc131\ud558\uc600\ub294\ub370 \uc778\uad8c\ubcf4\ud638, \uc790\uce58\uacbd\ucc30, \uc218\uc0ac\uac1c\ud601 \ub4f1 3\uac1c \ubd84\uacfc\ub85c \ub098\ub220\uc11c \ud65c\ub3d9\ud55c\ub2e4. \uc704\uc6d0\ud68c\ub294 7\uc6d4 19\uc77c \uacbd\ucc30 \uc9c1\ubb34\uc9d1\ud589 \uacfc\uc815\uc5d0\uc11c \ubc1c\uc0dd\ud55c \uc8fc\uc694 \uc778\uad8c\uce68\ud574 \uc0ac\uac74\uc5d0 \ub300\ud55c \uc9c4\uc0c1\uc870\uc0ac\ub97c \uc2e4\uc2dc\ud560 \ubbfc\uac04 \uc8fc\ub3c4\uc758 \uc9c4\uc0c1\uc870\uc0ac\uc704\uc6d0\ud68c\ub97c \uc124\uce58\ud558\ub294 \uac83\uc744 \uace8\uc790\ub85c \ud55c \uccab \ubc88\uc9f8 \uad8c\uace0\uc548\uc744, \uc774\uc5b4\uc11c 31\uc77c\uc5d0\ub294 2016\ub144\ubd80\ud130 23\ucc28\ub840 \uc9c4\ud589\ub410\ub358 \ucd1b\ubd88\uc9d1\ud68c\uc758 \uc804 \uacfc\uc815\uc744 \ubc31\uc11c\ub85c \ubc1c\uac04\ud558\uace0 \uc218\uc0ac\uc758 \uacf5\uc815\uc131\uc744 \ud655\ubcf4\ud558\uae30 \uc704\ud55c \ubc29\uc548 \ub4f1 \ub450 \ubc88\uc9f8 \uad8c\uace0\uc548\uc744 \uc81c\uc2dc\ud588\ub2e4. 9\uc6d4 29\uc77c\uc5d0\ub294 \uc720\uce58\uc778 \uc778\uad8c\uc744 \ubcf4\ud638\ud558\uace0 \uc720\uce58\uc778 \ubcf4\ud638\uad00 \uadfc\ubb34\ud658\uacbd\uc744 \uac1c\uc120\ud560 \uac83\uc744 \uacbd\ucc30\uccad\uc5d0 \uad8c\uace0\ud588\uc73c\uba70, 10\uc6d4 19\uc77c\uc5d0\ub294 \ucd1d\uacbd \uc774\uc0c1\uc774 \uae30\uad00\uc7a5\uc744 \ub9e1\ub294 \uc18c\uc18d\uae30\uad00\uc5d0 \uacbd\uac10 \uc774\ud558 \uacbd\ucc30\uad00\uc73c\ub85c \uad6c\uc131\ub41c \uc9c1\uc7a5\ud611\uc758\ud68c\ub97c \uc124\uce58\ud574\uc57c \ud55c\ub2e4\ub294 \uc758\uacac\uc744 \ub0c8\ub2e4. \uc774\ub294 \ub2e4\ub978 \uc9c1\uc885\uc5d0 \ube44\ud574 \ub9e4\uc6b0 \uc5f4\uc545\ud55c \uadfc\ubb34 \ud658\uacbd\uc5d0 \ucc98\ud574 \uc788\ub294 \uacbd\ucc30\uc5d0 \ucd5c\uc18c\ud55c\uc758 \uc758\uc0ac\uc18c\ud1b5 \uae30\uad6c\ub97c \ub458 \ud544\uc694\uac00 \uc788\uae30 \ub54c\ubb38\uc774\ub77c\uace0 \ubc1d\ud614\ub2e4. \uacbd\ucc30\uc5d0 \ub300\ud55c \uc678\ud48d\uc744 \ucc28\ub2e8\ud558\uae30 \uc704\ud55c \ubaa9\uc801\uc73c\ub85c \uacbd\ucc30\uc704\uc6d0\ud68c\ub97c \ucd1d\ub9ac \uc18c\uc18d \uc911\uc559\ud589\uc815\uae30\uad00\uc73c\ub85c \ub450\uace0, \uacbd\ucc30\uccad\uc7a5 \uc784\uba85\uc81c\uccad\uad8c\uacfc \ucd1d\uacbd \uc774\uc0c1 \uc2b9\uc9c4\uc778\uc0ac\uc5d0 \ub300\ud55c \uc2ec\uc758\u00b7\uc758\uacb0\uad8c\uc744 \ud589\uc0ac\ud1a0\ub85d \ud558\ub294 \ubc29\uc548\ub3c4 \uad8c\uace0\ud588\ub2e4. \uc11c\ubcf4\ud559 \uc704\uc6d0\uc740 \"\uc774\ubc88 \uad8c\uace0\ub294 \uacbd\ucc30\uc704\uc6d0\ud68c\uc5d0 \uacbd\ucc30\uc758 \ud1b5\uc81c\ub97c \ub9e1\uae30\ub294 \uac83\uc774\uae30 \ub54c\ubb38\uc5d0 \uacbd\ucc30\uc758 \uc5ed\ub7c9\uc744 \ub118\uc5b4\uc11c\ub294 \uccad\uc640\ub300\uc758 \uacb0\ub2e8\uc774 \ud544\uc694\ud55c \uc0ac\uc548\"\uc774\ub77c\uba70 \"\ub300\ud1b5\ub839\uc774 \uc790\uae30 \uc190\uc544\uadc0\uc5d0 \uc788\ub294 \uad8c\ub825\uae30\uad00\uc744 \ub193\uc544 \uc8fc\ub294 \uac83\"\uc774\ub77c\uace0 \ud588\ub2e4. \uc218\uc0ac\uad8c \ub3c5\ub9bd\uacfc \uad00\ub828\ud574\uc11c \uc77c\uc885\uc758 \ubd80\uc791\uc6a9 \uc9c0\uc6b0\uae30\ub85c\uc368 \uc678\ubd80 \ubbfc\uac04\uc778\uc774 \uae30\uad00\uc7a5\uc744 \ub9e1\ub294 \uad6d\uac00\uc218\uc0ac\ubcf8\ubd80\ub97c \ub450\uc5b4 \ub3c5\ub9bd\uc801\uc73c\ub85c \uc218\uc0ac\ub97c \uc9c4\ud589\ud560 \uc218 \uc788\ub294 \ubc29\uc548\uc744 \ubc1c\ud45c\ud588\ub2e4. \uacbd\ucc30\uccad\uc7a5\uc774\ub098 \uc77c\uc120 \uacbd\ucc30\uc11c\uc7a5\uc740 \uc77c\ubc18\uc9c1 \uc9c0\ud718\ub9cc \ud558\uace0 \uad6c\uccb4\uc801 \uc9c0\ud718\ub294 \uc218\uc0ac\ubcf8\ubd80\uc7a5\uc774 \ub2f4\ub2f9\ud558\uba70 \uacbd\ucc30\uccad\uc5d0\ub294 \uc9c1\uc811 \uc218\uc0ac\ubd80\uc11c\ub97c \ud3d0\uc9c0\ud574 \uc218\uc0ac \uad8c\ub825\uc744 \ubd84\uc0b0\uc2dc\ud0a4\uaca0\ub2e4\ub294 \uc758\ub3c4\ub2e4. \uc704\uc6d0\ud68c \uad00\uacc4\uc790\ub294 \"\uad6d\uac00\uc218\uc0ac\ubcf8\ubd80\uac00 \ucd5c\ub300\ud55c \ub3c5\ub9bd\uc131\uc744 \ud655\ubcf4\ud558\ub294 \ub370 \uc911\uc810\uc744 \ub480\ub2e4\"\uace0 \ud588\ub294\ub370 \uc774\ubc88 \uad8c\uace0\uc548\uc740 \uac80\uacbd \uc218\uc0ac\uad8c \uc870\uc815\uc744 \uc704\ud55c \uacbd\ucc30 \uce21\uc758 \ubc11\uadf8\ub9bc\uc774 \uc644\uc131\ub410\ub2e4\ub294 \ud3c9\uac00\ub3c4 \ub098\uc628\ub2e4. 12\uc6d4 7\uc77c\uc5d0\ub294 \uacbd\ucc30\uc774 \uc218\uc0ac\ub97c \uc804\ub2f4\ud558\uace0 \uac80\ucc30\uc740 \uacbd\ucc30\uc774 \uc1a1\uce58\ud55c \uc218\uc0ac \uacb0\uacfc\ub97c \ubc14\ud0d5\uc73c\ub85c \uacf5\uc18c\uad8c\uc744 \ud589\uc0ac\ud558\ub3c4\ub85d \ud558\ub294 \uc218\uc0ac\uad6c\uc870\uac1c\ud601 \ubc29\uc548\uc744 \ubc1c\ud45c\ud588\ub2e4. \uac80\ucc30\uc758 \uc218\uc0ac\uc9c0\ud718\uad8c\uacfc \uc9c1\uc811\uc218\uc0ac\uad8c\uc740 \ud3d0\uc9c0\ud558\uace0 \uac80\uc0ac\ub9cc\uc774 \uc601\uc7a5\uc744 \uccad\uad6c\ud560 \uc218 \uc788\ub3c4\ub85d \ud55c \ud5cc\ubc95 \uc870\ud56d\ub3c4 \uac1c\ud5cc \uacfc\uc815\uc5d0\uc11c \uc0ad\uc81c\ud560 \uac83\ub3c4 \uc81c\uc548\ud588\ub2e4. \ub610\ud55c \uc9c0\uae08\uae4c\uc9c0 \ub098\uc654\ub358 \uc704\uc6d0\ud68c\uc758 \uad8c\uace0\uc548\uc744 \uacbd\ucc30\uccad\uc740 \ub300\ubd80\ubd84 \ubc1b\uc544\ub4e4\uc600\ub2e4. \ud558\uc9c0\ub9cc \uc704\uc6d0\ud68c\uac00 \ubbfc\uc8fc\uc0ac\ud68c\ub97c \uc704\ud55c \ubcc0\ud638\uc0ac \ubaa8\uc784, \ucc38\uc5ec\uc5f0\ub300\ub098 \ubbfc\uc8fc\ub2f9, \ub178\ubb34\ud604 \uc815\ubd80 \ucd9c\uc2e0 \ub4f1 \uc704\uc6d0 19\uba85 \uc911 15\uba85\uc774 \uc88c\ud30c \uc9c4\uc601\uc774\ub77c\uba70 \uade0\ud615\uc774 \ub9de\uc9c0 \uc54a\ub2e4\ub294 \uc9c0\uc801\ub3c4 \uc788\ub2e4. \ub610\ud55c \uad8c\uace0\uc548\uc774 \uc5b4\ub5a4 \ub17c\uc758\ub97c \uac70\uccd0 \ub9c8\ub828\ub410\ub294\uc9c0 \ub179\ucde8\ub85d\ub3c4 \uacf5\uac1c\ub418\uc9c0 \uc54a\ub294 \ube44\ud310\ub3c4 \uc788\ub2e4.", "answer": "\ubc15\uacbd\uc11c", "sentence": "\ubc15\uacbd\uc11c \uc704\uc6d0\uc7a5\uc744 \ube44\ub86f\ud558\uc5ec 18\uba85\uc758 \uc704\uc6d0\uc73c\ub85c \uad6c\uc131\ud558\uc600\ub294\ub370 \uc778\uad8c\ubcf4\ud638, \uc790\uce58\uacbd\ucc30, \uc218\uc0ac\uac1c\ud601 \ub4f1 3\uac1c \ubd84\uacfc\ub85c \ub098\ub220\uc11c \ud65c\ub3d9\ud55c\ub2e4.", "paragraph_sentence": "6\uc6d4 16\uc77c\uc5d0\ub294 \uacbd\ucc30\uac1c\ud601\uc704\uc6d0\ud68c\ub97c \uad6c\uc131\ud558\uc600\ub294\ub370 \uacbd\ucc30\uc758 \ubcc0\ud654 \ubc29\uc548\uc744 \ub9c8\ub828\ud558\ub294 \uc790\ubb38\uae30\uad6c\ub85c \ud65c\ub3d9\ud558\uac8c \ub41c\ub2e4. \ubc15\uacbd\uc11c \uc704\uc6d0\uc7a5\uc744 \ube44\ub86f\ud558\uc5ec 18\uba85\uc758 \uc704\uc6d0\uc73c\ub85c \uad6c\uc131\ud558\uc600\ub294\ub370 \uc778\uad8c\ubcf4\ud638, \uc790\uce58\uacbd\ucc30, \uc218\uc0ac\uac1c\ud601 \ub4f1 3\uac1c \ubd84\uacfc\ub85c \ub098\ub220\uc11c \ud65c\ub3d9\ud55c\ub2e4. \uc704\uc6d0\ud68c\ub294 7\uc6d4 19\uc77c \uacbd\ucc30 \uc9c1\ubb34\uc9d1\ud589 \uacfc\uc815\uc5d0\uc11c \ubc1c\uc0dd\ud55c \uc8fc\uc694 \uc778\uad8c\uce68\ud574 \uc0ac\uac74\uc5d0 \ub300\ud55c \uc9c4\uc0c1\uc870\uc0ac\ub97c \uc2e4\uc2dc\ud560 \ubbfc\uac04 \uc8fc\ub3c4\uc758 \uc9c4\uc0c1\uc870\uc0ac\uc704\uc6d0\ud68c\ub97c \uc124\uce58\ud558\ub294 \uac83\uc744 \uace8\uc790\ub85c \ud55c \uccab \ubc88\uc9f8 \uad8c\uace0\uc548\uc744, \uc774\uc5b4\uc11c 31\uc77c\uc5d0\ub294 2016\ub144\ubd80\ud130 23\ucc28\ub840 \uc9c4\ud589\ub410\ub358 \ucd1b\ubd88\uc9d1\ud68c\uc758 \uc804 \uacfc\uc815\uc744 \ubc31\uc11c\ub85c \ubc1c\uac04\ud558\uace0 \uc218\uc0ac\uc758 \uacf5\uc815\uc131\uc744 \ud655\ubcf4\ud558\uae30 \uc704\ud55c \ubc29\uc548 \ub4f1 \ub450 \ubc88\uc9f8 \uad8c\uace0\uc548\uc744 \uc81c\uc2dc\ud588\ub2e4. 9\uc6d4 29\uc77c\uc5d0\ub294 \uc720\uce58\uc778 \uc778\uad8c\uc744 \ubcf4\ud638\ud558\uace0 \uc720\uce58\uc778 \ubcf4\ud638\uad00 \uadfc\ubb34\ud658\uacbd\uc744 \uac1c\uc120\ud560 \uac83\uc744 \uacbd\ucc30\uccad\uc5d0 \uad8c\uace0\ud588\uc73c\uba70, 10\uc6d4 19\uc77c\uc5d0\ub294 \ucd1d\uacbd \uc774\uc0c1\uc774 \uae30\uad00\uc7a5\uc744 \ub9e1\ub294 \uc18c\uc18d\uae30\uad00\uc5d0 \uacbd\uac10 \uc774\ud558 \uacbd\ucc30\uad00\uc73c\ub85c \uad6c\uc131\ub41c \uc9c1\uc7a5\ud611\uc758\ud68c\ub97c \uc124\uce58\ud574\uc57c \ud55c\ub2e4\ub294 \uc758\uacac\uc744 \ub0c8\ub2e4. \uc774\ub294 \ub2e4\ub978 \uc9c1\uc885\uc5d0 \ube44\ud574 \ub9e4\uc6b0 \uc5f4\uc545\ud55c \uadfc\ubb34 \ud658\uacbd\uc5d0 \ucc98\ud574 \uc788\ub294 \uacbd\ucc30\uc5d0 \ucd5c\uc18c\ud55c\uc758 \uc758\uc0ac\uc18c\ud1b5 \uae30\uad6c\ub97c \ub458 \ud544\uc694\uac00 \uc788\uae30 \ub54c\ubb38\uc774\ub77c\uace0 \ubc1d\ud614\ub2e4. \uacbd\ucc30\uc5d0 \ub300\ud55c \uc678\ud48d\uc744 \ucc28\ub2e8\ud558\uae30 \uc704\ud55c \ubaa9\uc801\uc73c\ub85c \uacbd\ucc30\uc704\uc6d0\ud68c\ub97c \ucd1d\ub9ac \uc18c\uc18d \uc911\uc559\ud589\uc815\uae30\uad00\uc73c\ub85c \ub450\uace0, \uacbd\ucc30\uccad\uc7a5 \uc784\uba85\uc81c\uccad\uad8c\uacfc \ucd1d\uacbd \uc774\uc0c1 \uc2b9\uc9c4\uc778\uc0ac\uc5d0 \ub300\ud55c \uc2ec\uc758\u00b7\uc758\uacb0\uad8c\uc744 \ud589\uc0ac\ud1a0\ub85d \ud558\ub294 \ubc29\uc548\ub3c4 \uad8c\uace0\ud588\ub2e4. \uc11c\ubcf4\ud559 \uc704\uc6d0\uc740 \"\uc774\ubc88 \uad8c\uace0\ub294 \uacbd\ucc30\uc704\uc6d0\ud68c\uc5d0 \uacbd\ucc30\uc758 \ud1b5\uc81c\ub97c \ub9e1\uae30\ub294 \uac83\uc774\uae30 \ub54c\ubb38\uc5d0 \uacbd\ucc30\uc758 \uc5ed\ub7c9\uc744 \ub118\uc5b4\uc11c\ub294 \uccad\uc640\ub300\uc758 \uacb0\ub2e8\uc774 \ud544\uc694\ud55c \uc0ac\uc548\"\uc774\ub77c\uba70 \"\ub300\ud1b5\ub839\uc774 \uc790\uae30 \uc190\uc544\uadc0\uc5d0 \uc788\ub294 \uad8c\ub825\uae30\uad00\uc744 \ub193\uc544 \uc8fc\ub294 \uac83\"\uc774\ub77c\uace0 \ud588\ub2e4. \uc218\uc0ac\uad8c \ub3c5\ub9bd\uacfc \uad00\ub828\ud574\uc11c \uc77c\uc885\uc758 \ubd80\uc791\uc6a9 \uc9c0\uc6b0\uae30\ub85c\uc368 \uc678\ubd80 \ubbfc\uac04\uc778\uc774 \uae30\uad00\uc7a5\uc744 \ub9e1\ub294 \uad6d\uac00\uc218\uc0ac\ubcf8\ubd80\ub97c \ub450\uc5b4 \ub3c5\ub9bd\uc801\uc73c\ub85c \uc218\uc0ac\ub97c \uc9c4\ud589\ud560 \uc218 \uc788\ub294 \ubc29\uc548\uc744 \ubc1c\ud45c\ud588\ub2e4. \uacbd\ucc30\uccad\uc7a5\uc774\ub098 \uc77c\uc120 \uacbd\ucc30\uc11c\uc7a5\uc740 \uc77c\ubc18\uc9c1 \uc9c0\ud718\ub9cc \ud558\uace0 \uad6c\uccb4\uc801 \uc9c0\ud718\ub294 \uc218\uc0ac\ubcf8\ubd80\uc7a5\uc774 \ub2f4\ub2f9\ud558\uba70 \uacbd\ucc30\uccad\uc5d0\ub294 \uc9c1\uc811 \uc218\uc0ac\ubd80\uc11c\ub97c \ud3d0\uc9c0\ud574 \uc218\uc0ac \uad8c\ub825\uc744 \ubd84\uc0b0\uc2dc\ud0a4\uaca0\ub2e4\ub294 \uc758\ub3c4\ub2e4. \uc704\uc6d0\ud68c \uad00\uacc4\uc790\ub294 \"\uad6d\uac00\uc218\uc0ac\ubcf8\ubd80\uac00 \ucd5c\ub300\ud55c \ub3c5\ub9bd\uc131\uc744 \ud655\ubcf4\ud558\ub294 \ub370 \uc911\uc810\uc744 \ub480\ub2e4\"\uace0 \ud588\ub294\ub370 \uc774\ubc88 \uad8c\uace0\uc548\uc740 \uac80\uacbd \uc218\uc0ac\uad8c \uc870\uc815\uc744 \uc704\ud55c \uacbd\ucc30 \uce21\uc758 \ubc11\uadf8\ub9bc\uc774 \uc644\uc131\ub410\ub2e4\ub294 \ud3c9\uac00\ub3c4 \ub098\uc628\ub2e4. 12\uc6d4 7\uc77c\uc5d0\ub294 \uacbd\ucc30\uc774 \uc218\uc0ac\ub97c \uc804\ub2f4\ud558\uace0 \uac80\ucc30\uc740 \uacbd\ucc30\uc774 \uc1a1\uce58\ud55c \uc218\uc0ac \uacb0\uacfc\ub97c \ubc14\ud0d5\uc73c\ub85c \uacf5\uc18c\uad8c\uc744 \ud589\uc0ac\ud558\ub3c4\ub85d \ud558\ub294 \uc218\uc0ac\uad6c\uc870\uac1c\ud601 \ubc29\uc548\uc744 \ubc1c\ud45c\ud588\ub2e4. \uac80\ucc30\uc758 \uc218\uc0ac\uc9c0\ud718\uad8c\uacfc \uc9c1\uc811\uc218\uc0ac\uad8c\uc740 \ud3d0\uc9c0\ud558\uace0 \uac80\uc0ac\ub9cc\uc774 \uc601\uc7a5\uc744 \uccad\uad6c\ud560 \uc218 \uc788\ub3c4\ub85d \ud55c \ud5cc\ubc95 \uc870\ud56d\ub3c4 \uac1c\ud5cc \uacfc\uc815\uc5d0\uc11c \uc0ad\uc81c\ud560 \uac83\ub3c4 \uc81c\uc548\ud588\ub2e4. \ub610\ud55c \uc9c0\uae08\uae4c\uc9c0 \ub098\uc654\ub358 \uc704\uc6d0\ud68c\uc758 \uad8c\uace0\uc548\uc744 \uacbd\ucc30\uccad\uc740 \ub300\ubd80\ubd84 \ubc1b\uc544\ub4e4\uc600\ub2e4. \ud558\uc9c0\ub9cc \uc704\uc6d0\ud68c\uac00 \ubbfc\uc8fc\uc0ac\ud68c\ub97c \uc704\ud55c \ubcc0\ud638\uc0ac \ubaa8\uc784, \ucc38\uc5ec\uc5f0\ub300\ub098 \ubbfc\uc8fc\ub2f9, \ub178\ubb34\ud604 \uc815\ubd80 \ucd9c\uc2e0 \ub4f1 \uc704\uc6d0 19\uba85 \uc911 15\uba85\uc774 \uc88c\ud30c \uc9c4\uc601\uc774\ub77c\uba70 \uade0\ud615\uc774 \ub9de\uc9c0 \uc54a\ub2e4\ub294 \uc9c0\uc801\ub3c4 \uc788\ub2e4. \ub610\ud55c \uad8c\uace0\uc548\uc774 \uc5b4\ub5a4 \ub17c\uc758\ub97c \uac70\uccd0 \ub9c8\ub828\ub410\ub294\uc9c0 \ub179\ucde8\ub85d\ub3c4 \uacf5\uac1c\ub418\uc9c0 \uc54a\ub294 \ube44\ud310\ub3c4 \uc788\ub2e4.", "paragraph_answer": "6\uc6d4 16\uc77c\uc5d0\ub294 \uacbd\ucc30\uac1c\ud601\uc704\uc6d0\ud68c\ub97c \uad6c\uc131\ud558\uc600\ub294\ub370 \uacbd\ucc30\uc758 \ubcc0\ud654 \ubc29\uc548\uc744 \ub9c8\ub828\ud558\ub294 \uc790\ubb38\uae30\uad6c\ub85c \ud65c\ub3d9\ud558\uac8c \ub41c\ub2e4. \ubc15\uacbd\uc11c \uc704\uc6d0\uc7a5\uc744 \ube44\ub86f\ud558\uc5ec 18\uba85\uc758 \uc704\uc6d0\uc73c\ub85c \uad6c\uc131\ud558\uc600\ub294\ub370 \uc778\uad8c\ubcf4\ud638, \uc790\uce58\uacbd\ucc30, \uc218\uc0ac\uac1c\ud601 \ub4f1 3\uac1c \ubd84\uacfc\ub85c \ub098\ub220\uc11c \ud65c\ub3d9\ud55c\ub2e4. \uc704\uc6d0\ud68c\ub294 7\uc6d4 19\uc77c \uacbd\ucc30 \uc9c1\ubb34\uc9d1\ud589 \uacfc\uc815\uc5d0\uc11c \ubc1c\uc0dd\ud55c \uc8fc\uc694 \uc778\uad8c\uce68\ud574 \uc0ac\uac74\uc5d0 \ub300\ud55c \uc9c4\uc0c1\uc870\uc0ac\ub97c \uc2e4\uc2dc\ud560 \ubbfc\uac04 \uc8fc\ub3c4\uc758 \uc9c4\uc0c1\uc870\uc0ac\uc704\uc6d0\ud68c\ub97c \uc124\uce58\ud558\ub294 \uac83\uc744 \uace8\uc790\ub85c \ud55c \uccab \ubc88\uc9f8 \uad8c\uace0\uc548\uc744, \uc774\uc5b4\uc11c 31\uc77c\uc5d0\ub294 2016\ub144\ubd80\ud130 23\ucc28\ub840 \uc9c4\ud589\ub410\ub358 \ucd1b\ubd88\uc9d1\ud68c\uc758 \uc804 \uacfc\uc815\uc744 \ubc31\uc11c\ub85c \ubc1c\uac04\ud558\uace0 \uc218\uc0ac\uc758 \uacf5\uc815\uc131\uc744 \ud655\ubcf4\ud558\uae30 \uc704\ud55c \ubc29\uc548 \ub4f1 \ub450 \ubc88\uc9f8 \uad8c\uace0\uc548\uc744 \uc81c\uc2dc\ud588\ub2e4. 9\uc6d4 29\uc77c\uc5d0\ub294 \uc720\uce58\uc778 \uc778\uad8c\uc744 \ubcf4\ud638\ud558\uace0 \uc720\uce58\uc778 \ubcf4\ud638\uad00 \uadfc\ubb34\ud658\uacbd\uc744 \uac1c\uc120\ud560 \uac83\uc744 \uacbd\ucc30\uccad\uc5d0 \uad8c\uace0\ud588\uc73c\uba70, 10\uc6d4 19\uc77c\uc5d0\ub294 \ucd1d\uacbd \uc774\uc0c1\uc774 \uae30\uad00\uc7a5\uc744 \ub9e1\ub294 \uc18c\uc18d\uae30\uad00\uc5d0 \uacbd\uac10 \uc774\ud558 \uacbd\ucc30\uad00\uc73c\ub85c \uad6c\uc131\ub41c \uc9c1\uc7a5\ud611\uc758\ud68c\ub97c \uc124\uce58\ud574\uc57c \ud55c\ub2e4\ub294 \uc758\uacac\uc744 \ub0c8\ub2e4. \uc774\ub294 \ub2e4\ub978 \uc9c1\uc885\uc5d0 \ube44\ud574 \ub9e4\uc6b0 \uc5f4\uc545\ud55c \uadfc\ubb34 \ud658\uacbd\uc5d0 \ucc98\ud574 \uc788\ub294 \uacbd\ucc30\uc5d0 \ucd5c\uc18c\ud55c\uc758 \uc758\uc0ac\uc18c\ud1b5 \uae30\uad6c\ub97c \ub458 \ud544\uc694\uac00 \uc788\uae30 \ub54c\ubb38\uc774\ub77c\uace0 \ubc1d\ud614\ub2e4. \uacbd\ucc30\uc5d0 \ub300\ud55c \uc678\ud48d\uc744 \ucc28\ub2e8\ud558\uae30 \uc704\ud55c \ubaa9\uc801\uc73c\ub85c \uacbd\ucc30\uc704\uc6d0\ud68c\ub97c \ucd1d\ub9ac \uc18c\uc18d \uc911\uc559\ud589\uc815\uae30\uad00\uc73c\ub85c \ub450\uace0, \uacbd\ucc30\uccad\uc7a5 \uc784\uba85\uc81c\uccad\uad8c\uacfc \ucd1d\uacbd \uc774\uc0c1 \uc2b9\uc9c4\uc778\uc0ac\uc5d0 \ub300\ud55c \uc2ec\uc758\u00b7\uc758\uacb0\uad8c\uc744 \ud589\uc0ac\ud1a0\ub85d \ud558\ub294 \ubc29\uc548\ub3c4 \uad8c\uace0\ud588\ub2e4. \uc11c\ubcf4\ud559 \uc704\uc6d0\uc740 \"\uc774\ubc88 \uad8c\uace0\ub294 \uacbd\ucc30\uc704\uc6d0\ud68c\uc5d0 \uacbd\ucc30\uc758 \ud1b5\uc81c\ub97c \ub9e1\uae30\ub294 \uac83\uc774\uae30 \ub54c\ubb38\uc5d0 \uacbd\ucc30\uc758 \uc5ed\ub7c9\uc744 \ub118\uc5b4\uc11c\ub294 \uccad\uc640\ub300\uc758 \uacb0\ub2e8\uc774 \ud544\uc694\ud55c \uc0ac\uc548\"\uc774\ub77c\uba70 \"\ub300\ud1b5\ub839\uc774 \uc790\uae30 \uc190\uc544\uadc0\uc5d0 \uc788\ub294 \uad8c\ub825\uae30\uad00\uc744 \ub193\uc544 \uc8fc\ub294 \uac83\"\uc774\ub77c\uace0 \ud588\ub2e4. \uc218\uc0ac\uad8c \ub3c5\ub9bd\uacfc \uad00\ub828\ud574\uc11c \uc77c\uc885\uc758 \ubd80\uc791\uc6a9 \uc9c0\uc6b0\uae30\ub85c\uc368 \uc678\ubd80 \ubbfc\uac04\uc778\uc774 \uae30\uad00\uc7a5\uc744 \ub9e1\ub294 \uad6d\uac00\uc218\uc0ac\ubcf8\ubd80\ub97c \ub450\uc5b4 \ub3c5\ub9bd\uc801\uc73c\ub85c \uc218\uc0ac\ub97c \uc9c4\ud589\ud560 \uc218 \uc788\ub294 \ubc29\uc548\uc744 \ubc1c\ud45c\ud588\ub2e4. \uacbd\ucc30\uccad\uc7a5\uc774\ub098 \uc77c\uc120 \uacbd\ucc30\uc11c\uc7a5\uc740 \uc77c\ubc18\uc9c1 \uc9c0\ud718\ub9cc \ud558\uace0 \uad6c\uccb4\uc801 \uc9c0\ud718\ub294 \uc218\uc0ac\ubcf8\ubd80\uc7a5\uc774 \ub2f4\ub2f9\ud558\uba70 \uacbd\ucc30\uccad\uc5d0\ub294 \uc9c1\uc811 \uc218\uc0ac\ubd80\uc11c\ub97c \ud3d0\uc9c0\ud574 \uc218\uc0ac \uad8c\ub825\uc744 \ubd84\uc0b0\uc2dc\ud0a4\uaca0\ub2e4\ub294 \uc758\ub3c4\ub2e4. \uc704\uc6d0\ud68c \uad00\uacc4\uc790\ub294 \"\uad6d\uac00\uc218\uc0ac\ubcf8\ubd80\uac00 \ucd5c\ub300\ud55c \ub3c5\ub9bd\uc131\uc744 \ud655\ubcf4\ud558\ub294 \ub370 \uc911\uc810\uc744 \ub480\ub2e4\"\uace0 \ud588\ub294\ub370 \uc774\ubc88 \uad8c\uace0\uc548\uc740 \uac80\uacbd \uc218\uc0ac\uad8c \uc870\uc815\uc744 \uc704\ud55c \uacbd\ucc30 \uce21\uc758 \ubc11\uadf8\ub9bc\uc774 \uc644\uc131\ub410\ub2e4\ub294 \ud3c9\uac00\ub3c4 \ub098\uc628\ub2e4. 12\uc6d4 7\uc77c\uc5d0\ub294 \uacbd\ucc30\uc774 \uc218\uc0ac\ub97c \uc804\ub2f4\ud558\uace0 \uac80\ucc30\uc740 \uacbd\ucc30\uc774 \uc1a1\uce58\ud55c \uc218\uc0ac \uacb0\uacfc\ub97c \ubc14\ud0d5\uc73c\ub85c \uacf5\uc18c\uad8c\uc744 \ud589\uc0ac\ud558\ub3c4\ub85d \ud558\ub294 \uc218\uc0ac\uad6c\uc870\uac1c\ud601 \ubc29\uc548\uc744 \ubc1c\ud45c\ud588\ub2e4. \uac80\ucc30\uc758 \uc218\uc0ac\uc9c0\ud718\uad8c\uacfc \uc9c1\uc811\uc218\uc0ac\uad8c\uc740 \ud3d0\uc9c0\ud558\uace0 \uac80\uc0ac\ub9cc\uc774 \uc601\uc7a5\uc744 \uccad\uad6c\ud560 \uc218 \uc788\ub3c4\ub85d \ud55c \ud5cc\ubc95 \uc870\ud56d\ub3c4 \uac1c\ud5cc \uacfc\uc815\uc5d0\uc11c \uc0ad\uc81c\ud560 \uac83\ub3c4 \uc81c\uc548\ud588\ub2e4. \ub610\ud55c \uc9c0\uae08\uae4c\uc9c0 \ub098\uc654\ub358 \uc704\uc6d0\ud68c\uc758 \uad8c\uace0\uc548\uc744 \uacbd\ucc30\uccad\uc740 \ub300\ubd80\ubd84 \ubc1b\uc544\ub4e4\uc600\ub2e4. \ud558\uc9c0\ub9cc \uc704\uc6d0\ud68c\uac00 \ubbfc\uc8fc\uc0ac\ud68c\ub97c \uc704\ud55c \ubcc0\ud638\uc0ac \ubaa8\uc784, \ucc38\uc5ec\uc5f0\ub300\ub098 \ubbfc\uc8fc\ub2f9, \ub178\ubb34\ud604 \uc815\ubd80 \ucd9c\uc2e0 \ub4f1 \uc704\uc6d0 19\uba85 \uc911 15\uba85\uc774 \uc88c\ud30c \uc9c4\uc601\uc774\ub77c\uba70 \uade0\ud615\uc774 \ub9de\uc9c0 \uc54a\ub2e4\ub294 \uc9c0\uc801\ub3c4 \uc788\ub2e4. \ub610\ud55c \uad8c\uace0\uc548\uc774 \uc5b4\ub5a4 \ub17c\uc758\ub97c \uac70\uccd0 \ub9c8\ub828\ub410\ub294\uc9c0 \ub179\ucde8\ub85d\ub3c4 \uacf5\uac1c\ub418\uc9c0 \uc54a\ub294 \ube44\ud310\ub3c4 \uc788\ub2e4.", "sentence_answer": " \ubc15\uacbd\uc11c \uc704\uc6d0\uc7a5\uc744 \ube44\ub86f\ud558\uc5ec 18\uba85\uc758 \uc704\uc6d0\uc73c\ub85c \uad6c\uc131\ud558\uc600\ub294\ub370 \uc778\uad8c\ubcf4\ud638, \uc790\uce58\uacbd\ucc30, \uc218\uc0ac\uac1c\ud601 \ub4f1 3\uac1c \ubd84\uacfc\ub85c \ub098\ub220\uc11c \ud65c\ub3d9\ud55c\ub2e4.", "paragraph_id": "6580768-69-0", "paragraph_question": "question: 6\uc6d4 16\uc77c\uc5d0 \uad6c\uc131\ub41c \uacbd\ucc30\uac1c\ud601\uc704\uc6d0\ud68c\uc758 \uc704\uc6d0\uc7a5\uc740 \ub204\uad6c\uc778\uac00?, context: 6\uc6d4 16\uc77c\uc5d0\ub294 \uacbd\ucc30\uac1c\ud601\uc704\uc6d0\ud68c\ub97c \uad6c\uc131\ud558\uc600\ub294\ub370 \uacbd\ucc30\uc758 \ubcc0\ud654 \ubc29\uc548\uc744 \ub9c8\ub828\ud558\ub294 \uc790\ubb38\uae30\uad6c\ub85c \ud65c\ub3d9\ud558\uac8c \ub41c\ub2e4. \ubc15\uacbd\uc11c \uc704\uc6d0\uc7a5\uc744 \ube44\ub86f\ud558\uc5ec 18\uba85\uc758 \uc704\uc6d0\uc73c\ub85c \uad6c\uc131\ud558\uc600\ub294\ub370 \uc778\uad8c\ubcf4\ud638, \uc790\uce58\uacbd\ucc30, \uc218\uc0ac\uac1c\ud601 \ub4f1 3\uac1c \ubd84\uacfc\ub85c \ub098\ub220\uc11c \ud65c\ub3d9\ud55c\ub2e4. \uc704\uc6d0\ud68c\ub294 7\uc6d4 19\uc77c \uacbd\ucc30 \uc9c1\ubb34\uc9d1\ud589 \uacfc\uc815\uc5d0\uc11c \ubc1c\uc0dd\ud55c \uc8fc\uc694 \uc778\uad8c\uce68\ud574 \uc0ac\uac74\uc5d0 \ub300\ud55c \uc9c4\uc0c1\uc870\uc0ac\ub97c \uc2e4\uc2dc\ud560 \ubbfc\uac04 \uc8fc\ub3c4\uc758 \uc9c4\uc0c1\uc870\uc0ac\uc704\uc6d0\ud68c\ub97c \uc124\uce58\ud558\ub294 \uac83\uc744 \uace8\uc790\ub85c \ud55c \uccab \ubc88\uc9f8 \uad8c\uace0\uc548\uc744, \uc774\uc5b4\uc11c 31\uc77c\uc5d0\ub294 2016\ub144\ubd80\ud130 23\ucc28\ub840 \uc9c4\ud589\ub410\ub358 \ucd1b\ubd88\uc9d1\ud68c\uc758 \uc804 \uacfc\uc815\uc744 \ubc31\uc11c\ub85c \ubc1c\uac04\ud558\uace0 \uc218\uc0ac\uc758 \uacf5\uc815\uc131\uc744 \ud655\ubcf4\ud558\uae30 \uc704\ud55c \ubc29\uc548 \ub4f1 \ub450 \ubc88\uc9f8 \uad8c\uace0\uc548\uc744 \uc81c\uc2dc\ud588\ub2e4. 9\uc6d4 29\uc77c\uc5d0\ub294 \uc720\uce58\uc778 \uc778\uad8c\uc744 \ubcf4\ud638\ud558\uace0 \uc720\uce58\uc778 \ubcf4\ud638\uad00 \uadfc\ubb34\ud658\uacbd\uc744 \uac1c\uc120\ud560 \uac83\uc744 \uacbd\ucc30\uccad\uc5d0 \uad8c\uace0\ud588\uc73c\uba70, 10\uc6d4 19\uc77c\uc5d0\ub294 \ucd1d\uacbd \uc774\uc0c1\uc774 \uae30\uad00\uc7a5\uc744 \ub9e1\ub294 \uc18c\uc18d\uae30\uad00\uc5d0 \uacbd\uac10 \uc774\ud558 \uacbd\ucc30\uad00\uc73c\ub85c \uad6c\uc131\ub41c \uc9c1\uc7a5\ud611\uc758\ud68c\ub97c \uc124\uce58\ud574\uc57c \ud55c\ub2e4\ub294 \uc758\uacac\uc744 \ub0c8\ub2e4. \uc774\ub294 \ub2e4\ub978 \uc9c1\uc885\uc5d0 \ube44\ud574 \ub9e4\uc6b0 \uc5f4\uc545\ud55c \uadfc\ubb34 \ud658\uacbd\uc5d0 \ucc98\ud574 \uc788\ub294 \uacbd\ucc30\uc5d0 \ucd5c\uc18c\ud55c\uc758 \uc758\uc0ac\uc18c\ud1b5 \uae30\uad6c\ub97c \ub458 \ud544\uc694\uac00 \uc788\uae30 \ub54c\ubb38\uc774\ub77c\uace0 \ubc1d\ud614\ub2e4. \uacbd\ucc30\uc5d0 \ub300\ud55c \uc678\ud48d\uc744 \ucc28\ub2e8\ud558\uae30 \uc704\ud55c \ubaa9\uc801\uc73c\ub85c \uacbd\ucc30\uc704\uc6d0\ud68c\ub97c \ucd1d\ub9ac \uc18c\uc18d \uc911\uc559\ud589\uc815\uae30\uad00\uc73c\ub85c \ub450\uace0, \uacbd\ucc30\uccad\uc7a5 \uc784\uba85\uc81c\uccad\uad8c\uacfc \ucd1d\uacbd \uc774\uc0c1 \uc2b9\uc9c4\uc778\uc0ac\uc5d0 \ub300\ud55c \uc2ec\uc758\u00b7\uc758\uacb0\uad8c\uc744 \ud589\uc0ac\ud1a0\ub85d \ud558\ub294 \ubc29\uc548\ub3c4 \uad8c\uace0\ud588\ub2e4. \uc11c\ubcf4\ud559 \uc704\uc6d0\uc740 \"\uc774\ubc88 \uad8c\uace0\ub294 \uacbd\ucc30\uc704\uc6d0\ud68c\uc5d0 \uacbd\ucc30\uc758 \ud1b5\uc81c\ub97c \ub9e1\uae30\ub294 \uac83\uc774\uae30 \ub54c\ubb38\uc5d0 \uacbd\ucc30\uc758 \uc5ed\ub7c9\uc744 \ub118\uc5b4\uc11c\ub294 \uccad\uc640\ub300\uc758 \uacb0\ub2e8\uc774 \ud544\uc694\ud55c \uc0ac\uc548\"\uc774\ub77c\uba70 \"\ub300\ud1b5\ub839\uc774 \uc790\uae30 \uc190\uc544\uadc0\uc5d0 \uc788\ub294 \uad8c\ub825\uae30\uad00\uc744 \ub193\uc544 \uc8fc\ub294 \uac83\"\uc774\ub77c\uace0 \ud588\ub2e4. \uc218\uc0ac\uad8c \ub3c5\ub9bd\uacfc \uad00\ub828\ud574\uc11c \uc77c\uc885\uc758 \ubd80\uc791\uc6a9 \uc9c0\uc6b0\uae30\ub85c\uc368 \uc678\ubd80 \ubbfc\uac04\uc778\uc774 \uae30\uad00\uc7a5\uc744 \ub9e1\ub294 \uad6d\uac00\uc218\uc0ac\ubcf8\ubd80\ub97c \ub450\uc5b4 \ub3c5\ub9bd\uc801\uc73c\ub85c \uc218\uc0ac\ub97c \uc9c4\ud589\ud560 \uc218 \uc788\ub294 \ubc29\uc548\uc744 \ubc1c\ud45c\ud588\ub2e4. \uacbd\ucc30\uccad\uc7a5\uc774\ub098 \uc77c\uc120 \uacbd\ucc30\uc11c\uc7a5\uc740 \uc77c\ubc18\uc9c1 \uc9c0\ud718\ub9cc \ud558\uace0 \uad6c\uccb4\uc801 \uc9c0\ud718\ub294 \uc218\uc0ac\ubcf8\ubd80\uc7a5\uc774 \ub2f4\ub2f9\ud558\uba70 \uacbd\ucc30\uccad\uc5d0\ub294 \uc9c1\uc811 \uc218\uc0ac\ubd80\uc11c\ub97c \ud3d0\uc9c0\ud574 \uc218\uc0ac \uad8c\ub825\uc744 \ubd84\uc0b0\uc2dc\ud0a4\uaca0\ub2e4\ub294 \uc758\ub3c4\ub2e4. \uc704\uc6d0\ud68c \uad00\uacc4\uc790\ub294 \"\uad6d\uac00\uc218\uc0ac\ubcf8\ubd80\uac00 \ucd5c\ub300\ud55c \ub3c5\ub9bd\uc131\uc744 \ud655\ubcf4\ud558\ub294 \ub370 \uc911\uc810\uc744 \ub480\ub2e4\"\uace0 \ud588\ub294\ub370 \uc774\ubc88 \uad8c\uace0\uc548\uc740 \uac80\uacbd \uc218\uc0ac\uad8c \uc870\uc815\uc744 \uc704\ud55c \uacbd\ucc30 \uce21\uc758 \ubc11\uadf8\ub9bc\uc774 \uc644\uc131\ub410\ub2e4\ub294 \ud3c9\uac00\ub3c4 \ub098\uc628\ub2e4. 12\uc6d4 7\uc77c\uc5d0\ub294 \uacbd\ucc30\uc774 \uc218\uc0ac\ub97c \uc804\ub2f4\ud558\uace0 \uac80\ucc30\uc740 \uacbd\ucc30\uc774 \uc1a1\uce58\ud55c \uc218\uc0ac \uacb0\uacfc\ub97c \ubc14\ud0d5\uc73c\ub85c \uacf5\uc18c\uad8c\uc744 \ud589\uc0ac\ud558\ub3c4\ub85d \ud558\ub294 \uc218\uc0ac\uad6c\uc870\uac1c\ud601 \ubc29\uc548\uc744 \ubc1c\ud45c\ud588\ub2e4. \uac80\ucc30\uc758 \uc218\uc0ac\uc9c0\ud718\uad8c\uacfc \uc9c1\uc811\uc218\uc0ac\uad8c\uc740 \ud3d0\uc9c0\ud558\uace0 \uac80\uc0ac\ub9cc\uc774 \uc601\uc7a5\uc744 \uccad\uad6c\ud560 \uc218 \uc788\ub3c4\ub85d \ud55c \ud5cc\ubc95 \uc870\ud56d\ub3c4 \uac1c\ud5cc \uacfc\uc815\uc5d0\uc11c \uc0ad\uc81c\ud560 \uac83\ub3c4 \uc81c\uc548\ud588\ub2e4. \ub610\ud55c \uc9c0\uae08\uae4c\uc9c0 \ub098\uc654\ub358 \uc704\uc6d0\ud68c\uc758 \uad8c\uace0\uc548\uc744 \uacbd\ucc30\uccad\uc740 \ub300\ubd80\ubd84 \ubc1b\uc544\ub4e4\uc600\ub2e4. \ud558\uc9c0\ub9cc \uc704\uc6d0\ud68c\uac00 \ubbfc\uc8fc\uc0ac\ud68c\ub97c \uc704\ud55c \ubcc0\ud638\uc0ac \ubaa8\uc784, \ucc38\uc5ec\uc5f0\ub300\ub098 \ubbfc\uc8fc\ub2f9, \ub178\ubb34\ud604 \uc815\ubd80 \ucd9c\uc2e0 \ub4f1 \uc704\uc6d0 19\uba85 \uc911 15\uba85\uc774 \uc88c\ud30c \uc9c4\uc601\uc774\ub77c\uba70 \uade0\ud615\uc774 \ub9de\uc9c0 \uc54a\ub2e4\ub294 \uc9c0\uc801\ub3c4 \uc788\ub2e4. \ub610\ud55c \uad8c\uace0\uc548\uc774 \uc5b4\ub5a4 \ub17c\uc758\ub97c \uac70\uccd0 \ub9c8\ub828\ub410\ub294\uc9c0 \ub179\ucde8\ub85d\ub3c4 \uacf5\uac1c\ub418\uc9c0 \uc54a\ub294 \ube44\ud310\ub3c4 \uc788\ub2e4."} {"question": "2013\ub144\uc5d0 \ubc1c\ub9e4\ud55c [ I Like 2 Party]\uc568\ubc94\uc744 \ub0b8 \uac00\uc218\ub294 \ub204\uad6c\uc778\uac00?", "paragraph": "\ubc15\uc7ac\ubc94(Jay Park)\uc740 2013\ub144 7\uc6d4 10\uc77c \ubc1c\ub9e4\ud55c \uc2f1\uae00 [I Like 2 Party] \uc568\ubc94\uacfc 2013\ub144 9\uc6d4 6\uc77c \ub3c4\ub07c\uc640 \ud568\uaed8 \ubc1c\ud45c\ud55c \ubbf9\uc2a4\ud14c\uc78e [TRILL]\uc758 \ucee4\ubc84\uc0ac\uc9c4\uc5d0 'AOMG'\ub77c\uace0 \uae00\uc790\ub97c \uc801\uc5b4 \uacf5\uac1c\ud558\uba70 \ub9ce\uc740 \ud32c\ub4e4 \uc0ac\uc774\uc5d0\uc11c \uad81\uae08\uc99d\uc744 \uc790\uc544\ub0c8\ub2e4. \uadf8 \uc774\ud6c4, 2013\ub144 10\uc6d4 10\uc77c \ud074\ub7fd The A\uc5d0\uc11c 'AOMG \ub7f0\uce6d\ud30c\ud2f0'\ub97c \uc5f4\uc5b4 \uc774 \uacf3\uc5d0\uc11c \ud799\ud569\ub808\uc774\ube14 'AOMG'\uc758 \uc124\ub9bd\uc744 \ubc1c\ud45c\ud558\uc600\ub2e4. \ud30c\ud2f0\uc5d0\ub294 \uc800\uc2a4\ud2f4 \ube44\ubc84, \uc9c0\ub4dc\ub798\uace4, 2NE1 \ub4f1\uc774 \ucc38\uc11d\ud588\uc73c\uba70, Dok2, \ube48\uc9c0\ub178, \uc790\uc774\uc5b8\ud2f0, \ud06c\ub7ec\uc26c \ub4f1\uc758 \ucd95\ud558\uacf5\uc5f0\uc774 \uc788\uc5c8\ub2e4. \ucc3d\ub9bd\uba64\ubc84\ub294 \ub300\ud45c \ubc15\uc7ac\ubc94(Jay Park), \uc2f1\uc5b4\uc1a1\ub77c\uc774\ud130 \uadf8\ub808\uc774(GRAY), \ud504\ub85c\ub4c0\uc11c \ucc28 \ucc28 \ub9d0\ub860(Cha Cha Malone)\uc774\ub2e4. \uadf8\ub9ac\uace0 \uacf5\uc2dd\uc801\uc73c\ub85c \ubc1c\ud45c\ub41c \uba64\ubc84\ub294 \uc544\ub2c8\uc9c0\ub9cc \ubc15\uc7ac\ubc94\uc758 B-boy \ud06c\ub8e8 Art of Movement\uc640 \ubba4\uc9c1\ube44\ub514\uc624 \uac10\ub3c5 \uc9c0\ub204\uc57c(Jinooya) \ub610\ud55c \ud568\uaed8 \ud558\uace0 \uc788\ub2e4.", "answer": "\ubc15\uc7ac\ubc94", "sentence": "\ubc15\uc7ac\ubc94 (Jay Park)\uc740 2013\ub144 7\uc6d4 10\uc77c \ubc1c\ub9e4\ud55c \uc2f1\uae00 [I Like 2 Party] \uc568\ubc94\uacfc 2013\ub144 9\uc6d4 6\uc77c \ub3c4\ub07c\uc640 \ud568\uaed8 \ubc1c\ud45c\ud55c \ubbf9\uc2a4\ud14c\uc78e [TRILL]\uc758 \ucee4\ubc84\uc0ac\uc9c4\uc5d0 'AOMG'\ub77c\uace0 \uae00\uc790\ub97c \uc801\uc5b4 \uacf5\uac1c\ud558\uba70 \ub9ce\uc740 \ud32c\ub4e4 \uc0ac\uc774\uc5d0\uc11c \uad81\uae08\uc99d\uc744 \uc790\uc544\ub0c8\ub2e4.", "paragraph_sentence": " \ubc15\uc7ac\ubc94 (Jay Park)\uc740 2013\ub144 7\uc6d4 10\uc77c \ubc1c\ub9e4\ud55c \uc2f1\uae00 [I Like 2 Party] \uc568\ubc94\uacfc 2013\ub144 9\uc6d4 6\uc77c \ub3c4\ub07c\uc640 \ud568\uaed8 \ubc1c\ud45c\ud55c \ubbf9\uc2a4\ud14c\uc78e [TRILL]\uc758 \ucee4\ubc84\uc0ac\uc9c4\uc5d0 'AOMG'\ub77c\uace0 \uae00\uc790\ub97c \uc801\uc5b4 \uacf5\uac1c\ud558\uba70 \ub9ce\uc740 \ud32c\ub4e4 \uc0ac\uc774\uc5d0\uc11c \uad81\uae08\uc99d\uc744 \uc790\uc544\ub0c8\ub2e4. \uadf8 \uc774\ud6c4, 2013\ub144 10\uc6d4 10\uc77c \ud074\ub7fd The A\uc5d0\uc11c 'AOMG \ub7f0\uce6d\ud30c\ud2f0'\ub97c \uc5f4\uc5b4 \uc774 \uacf3\uc5d0\uc11c \ud799\ud569\ub808\uc774\ube14 'AOMG'\uc758 \uc124\ub9bd\uc744 \ubc1c\ud45c\ud558\uc600\ub2e4. \ud30c\ud2f0\uc5d0\ub294 \uc800\uc2a4\ud2f4 \ube44\ubc84, \uc9c0\ub4dc\ub798\uace4, 2NE1 \ub4f1\uc774 \ucc38\uc11d\ud588\uc73c\uba70, Dok2, \ube48\uc9c0\ub178, \uc790\uc774\uc5b8\ud2f0, \ud06c\ub7ec\uc26c \ub4f1\uc758 \ucd95\ud558\uacf5\uc5f0\uc774 \uc788\uc5c8\ub2e4. \ucc3d\ub9bd\uba64\ubc84\ub294 \ub300\ud45c \ubc15\uc7ac\ubc94(Jay Park), \uc2f1\uc5b4\uc1a1\ub77c\uc774\ud130 \uadf8\ub808\uc774(GRAY), \ud504\ub85c\ub4c0\uc11c \ucc28 \ucc28 \ub9d0\ub860(Cha Cha Malone)\uc774\ub2e4. \uadf8\ub9ac\uace0 \uacf5\uc2dd\uc801\uc73c\ub85c \ubc1c\ud45c\ub41c \uba64\ubc84\ub294 \uc544\ub2c8\uc9c0\ub9cc \ubc15\uc7ac\ubc94\uc758 B-boy \ud06c\ub8e8 Art of Movement\uc640 \ubba4\uc9c1\ube44\ub514\uc624 \uac10\ub3c5 \uc9c0\ub204\uc57c(Jinooya) \ub610\ud55c \ud568\uaed8 \ud558\uace0 \uc788\ub2e4.", "paragraph_answer": " \ubc15\uc7ac\ubc94 (Jay Park)\uc740 2013\ub144 7\uc6d4 10\uc77c \ubc1c\ub9e4\ud55c \uc2f1\uae00 [I Like 2 Party] \uc568\ubc94\uacfc 2013\ub144 9\uc6d4 6\uc77c \ub3c4\ub07c\uc640 \ud568\uaed8 \ubc1c\ud45c\ud55c \ubbf9\uc2a4\ud14c\uc78e [TRILL]\uc758 \ucee4\ubc84\uc0ac\uc9c4\uc5d0 'AOMG'\ub77c\uace0 \uae00\uc790\ub97c \uc801\uc5b4 \uacf5\uac1c\ud558\uba70 \ub9ce\uc740 \ud32c\ub4e4 \uc0ac\uc774\uc5d0\uc11c \uad81\uae08\uc99d\uc744 \uc790\uc544\ub0c8\ub2e4. \uadf8 \uc774\ud6c4, 2013\ub144 10\uc6d4 10\uc77c \ud074\ub7fd The A\uc5d0\uc11c 'AOMG \ub7f0\uce6d\ud30c\ud2f0'\ub97c \uc5f4\uc5b4 \uc774 \uacf3\uc5d0\uc11c \ud799\ud569\ub808\uc774\ube14 'AOMG'\uc758 \uc124\ub9bd\uc744 \ubc1c\ud45c\ud558\uc600\ub2e4. \ud30c\ud2f0\uc5d0\ub294 \uc800\uc2a4\ud2f4 \ube44\ubc84, \uc9c0\ub4dc\ub798\uace4, 2NE1 \ub4f1\uc774 \ucc38\uc11d\ud588\uc73c\uba70, Dok2, \ube48\uc9c0\ub178, \uc790\uc774\uc5b8\ud2f0, \ud06c\ub7ec\uc26c \ub4f1\uc758 \ucd95\ud558\uacf5\uc5f0\uc774 \uc788\uc5c8\ub2e4. \ucc3d\ub9bd\uba64\ubc84\ub294 \ub300\ud45c \ubc15\uc7ac\ubc94(Jay Park), \uc2f1\uc5b4\uc1a1\ub77c\uc774\ud130 \uadf8\ub808\uc774(GRAY), \ud504\ub85c\ub4c0\uc11c \ucc28 \ucc28 \ub9d0\ub860(Cha Cha Malone)\uc774\ub2e4. \uadf8\ub9ac\uace0 \uacf5\uc2dd\uc801\uc73c\ub85c \ubc1c\ud45c\ub41c \uba64\ubc84\ub294 \uc544\ub2c8\uc9c0\ub9cc \ubc15\uc7ac\ubc94\uc758 B-boy \ud06c\ub8e8 Art of Movement\uc640 \ubba4\uc9c1\ube44\ub514\uc624 \uac10\ub3c5 \uc9c0\ub204\uc57c(Jinooya) \ub610\ud55c \ud568\uaed8 \ud558\uace0 \uc788\ub2e4.", "sentence_answer": " \ubc15\uc7ac\ubc94 (Jay Park)\uc740 2013\ub144 7\uc6d4 10\uc77c \ubc1c\ub9e4\ud55c \uc2f1\uae00 [I Like 2 Party] \uc568\ubc94\uacfc 2013\ub144 9\uc6d4 6\uc77c \ub3c4\ub07c\uc640 \ud568\uaed8 \ubc1c\ud45c\ud55c \ubbf9\uc2a4\ud14c\uc78e [TRILL]\uc758 \ucee4\ubc84\uc0ac\uc9c4\uc5d0 'AOMG'\ub77c\uace0 \uae00\uc790\ub97c \uc801\uc5b4 \uacf5\uac1c\ud558\uba70 \ub9ce\uc740 \ud32c\ub4e4 \uc0ac\uc774\uc5d0\uc11c \uad81\uae08\uc99d\uc744 \uc790\uc544\ub0c8\ub2e4.", "paragraph_id": "6458108-0-0", "paragraph_question": "question: 2013\ub144\uc5d0 \ubc1c\ub9e4\ud55c [ I Like 2 Party]\uc568\ubc94\uc744 \ub0b8 \uac00\uc218\ub294 \ub204\uad6c\uc778\uac00?, context: \ubc15\uc7ac\ubc94(Jay Park)\uc740 2013\ub144 7\uc6d4 10\uc77c \ubc1c\ub9e4\ud55c \uc2f1\uae00 [I Like 2 Party] \uc568\ubc94\uacfc 2013\ub144 9\uc6d4 6\uc77c \ub3c4\ub07c\uc640 \ud568\uaed8 \ubc1c\ud45c\ud55c \ubbf9\uc2a4\ud14c\uc78e [TRILL]\uc758 \ucee4\ubc84\uc0ac\uc9c4\uc5d0 'AOMG'\ub77c\uace0 \uae00\uc790\ub97c \uc801\uc5b4 \uacf5\uac1c\ud558\uba70 \ub9ce\uc740 \ud32c\ub4e4 \uc0ac\uc774\uc5d0\uc11c \uad81\uae08\uc99d\uc744 \uc790\uc544\ub0c8\ub2e4. \uadf8 \uc774\ud6c4, 2013\ub144 10\uc6d4 10\uc77c \ud074\ub7fd The A\uc5d0\uc11c 'AOMG \ub7f0\uce6d\ud30c\ud2f0'\ub97c \uc5f4\uc5b4 \uc774 \uacf3\uc5d0\uc11c \ud799\ud569\ub808\uc774\ube14 'AOMG'\uc758 \uc124\ub9bd\uc744 \ubc1c\ud45c\ud558\uc600\ub2e4. \ud30c\ud2f0\uc5d0\ub294 \uc800\uc2a4\ud2f4 \ube44\ubc84, \uc9c0\ub4dc\ub798\uace4, 2NE1 \ub4f1\uc774 \ucc38\uc11d\ud588\uc73c\uba70, Dok2, \ube48\uc9c0\ub178, \uc790\uc774\uc5b8\ud2f0, \ud06c\ub7ec\uc26c \ub4f1\uc758 \ucd95\ud558\uacf5\uc5f0\uc774 \uc788\uc5c8\ub2e4. \ucc3d\ub9bd\uba64\ubc84\ub294 \ub300\ud45c \ubc15\uc7ac\ubc94(Jay Park), \uc2f1\uc5b4\uc1a1\ub77c\uc774\ud130 \uadf8\ub808\uc774(GRAY), \ud504\ub85c\ub4c0\uc11c \ucc28 \ucc28 \ub9d0\ub860(Cha Cha Malone)\uc774\ub2e4. \uadf8\ub9ac\uace0 \uacf5\uc2dd\uc801\uc73c\ub85c \ubc1c\ud45c\ub41c \uba64\ubc84\ub294 \uc544\ub2c8\uc9c0\ub9cc \ubc15\uc7ac\ubc94\uc758 B-boy \ud06c\ub8e8 Art of Movement\uc640 \ubba4\uc9c1\ube44\ub514\uc624 \uac10\ub3c5 \uc9c0\ub204\uc57c(Jinooya) \ub610\ud55c \ud568\uaed8 \ud558\uace0 \uc788\ub2e4."} {"question": "10\uc6d4 25\uc77c\uc5d0 \ubc15\ub300\ud1b5\ub839\uc774 \ub300\uad6d\ubbfc\uc0ac\uacfc\ub97c \uc9c4\ud589\ud55c \uacf3\uc740?", "paragraph": "\uacb0\uad6d \ub2e4\uc74c \ub0a0\uc778 10\uc6d4 25\uc77c \uc624\ud6c4 4\uc2dc, \ubc15\uadfc\ud61c \ub300\ud1b5\ub839\uc740 \uccad\uc640\ub300 \ucd98\ucd94\uad00\uc5d0\uc11c \ub300\uad6d\ubbfc\uc0ac\uacfc\ub97c \uc9c4\ud589\ud574 \"\ucd5c\uc21c\uc2e4 \uc528\ub294 \uacfc\uac70 \uc5b4\ub824\uc6b8 \ub54c \ub3c4\uc640\uc900 \uc778\uc5f0\"\uc73c\ub85c\uc11c \"\uc77c\ubd80 \uc5f0\uc124\ubb38 \ub4f1\uc740 \ucd5c\uc21c\uc2e4 \uc528\uc5d0\uac8c \ub3c4\uc6c0\ubc1b\uc558\ub2e4\"\uba70 \uc2dc\uc778\ud558\uc600\ub2e4. \uadf8\ub7ec\ub098 1\ucc28 \ub300\uad6d\ubbfc\uc0ac\uacfc\ub294 \uc9c0\uae08\uae4c\uc9c0\uc758 \uc0ac\ud0dc\ub97c '\uac1c\uc778\uc801 \uc778\uc5f0'\uc73c\ub85c\ub9cc \uc124\uba85\ud558\ub824\ub294 \ud0dc\ub3c4\ub85c\uc11c \ub300\ud1b5\ub839\uc774 \uc815\ub9d0\ub85c \uad6d\uc815\uc6b4\uc601 \ucc45\uc784\uc790\ub85c\uc11c \ubb34\uac70\uc6b4 \ucc45\uc784\uc744 \uc778\uc2dd\ud558\uc600\ub2e4\uace0\ub294 \ubcfc \uc218 \uc5c6\uc5c8\uae30\uc5d0, \uc624\ud788\ub824 \ubc15\uadfc\ud61c-\ucd5c\uc21c\uc2e4 \uad00\uacc4\uc5d0 \ub300\ud55c \uc758\ud639\uc744 \uc99d\ud3ed\uc2dc\ud0a4\ub294 \uacb0\uacfc\ub9cc \ub0b3\uc558\ub2e4. \ub300\uad6d\ubbfc \uc0ac\uacfc \uc9c1\ud6c4\uc778 4\uc2dc 35\ubd84\uacbd \ub124\uc774\ubc84\uc640 \ub2e4\uc74c\uc758 \uc778\uae30\uac80\uc0c9\uc5b4\uc5d0\ub294 \"\ud0c4\ud575\"\uc774\ub098 \"\ubc15\uadfc\ud61c \ud0c4\ud575\", \"\ud558\uc57c\" \ub4f1\uc758 \ud0a4\uc6cc\ub4dc\uac00 \uc0c1\uc704\uad8c\uc5d0 \uc62c\ub790\ub2e4. \uc5ec\ub860\ub3c4 \uae09\uaca9\ud788 \uc545\ud654\ud574 \ub9ac\uc5bc\ubbf8\ud130\uc758 \ubc15\uadfc\ud61c \ub300\ud1b5\ub839 \uad6d\uc815 \uc9c0\uc9c0\ub3c4 \uc870\uc0ac\uc5d0\uc11c\ub294 25\uc77c\uc5d0\ub294 22.7%, 26\uc77c\uc5d0\ub294 17.5%\ub85c, \ubd88\uacfc \uc774\ud2c0 \ub9cc\uc5d0 10%\ub300\ub85c \uc8fc\uc800\uc549\uc740 \uac83\uc73c\ub85c \uc870\uc0ac\ub418\uc5c8\ub2e4.", "answer": "\uccad\uc640\ub300 \ucd98\ucd94\uad00", "sentence": "\uacb0\uad6d \ub2e4\uc74c \ub0a0\uc778 10\uc6d4 25\uc77c \uc624\ud6c4 4\uc2dc, \ubc15\uadfc\ud61c \ub300\ud1b5\ub839\uc740 \uccad\uc640\ub300 \ucd98\ucd94\uad00 \uc5d0\uc11c \ub300\uad6d\ubbfc\uc0ac\uacfc\ub97c \uc9c4\ud589\ud574 \"\ucd5c\uc21c\uc2e4 \uc528\ub294 \uacfc\uac70 \uc5b4\ub824\uc6b8 \ub54c \ub3c4\uc640\uc900 \uc778\uc5f0\"\uc73c\ub85c\uc11c \"\uc77c\ubd80 \uc5f0\uc124\ubb38 \ub4f1\uc740 \ucd5c\uc21c\uc2e4 \uc528\uc5d0\uac8c \ub3c4\uc6c0\ubc1b\uc558\ub2e4\"\uba70 \uc2dc\uc778\ud558\uc600\ub2e4.", "paragraph_sentence": " \uacb0\uad6d \ub2e4\uc74c \ub0a0\uc778 10\uc6d4 25\uc77c \uc624\ud6c4 4\uc2dc, \ubc15\uadfc\ud61c \ub300\ud1b5\ub839\uc740 \uccad\uc640\ub300 \ucd98\ucd94\uad00 \uc5d0\uc11c \ub300\uad6d\ubbfc\uc0ac\uacfc\ub97c \uc9c4\ud589\ud574 \"\ucd5c\uc21c\uc2e4 \uc528\ub294 \uacfc\uac70 \uc5b4\ub824\uc6b8 \ub54c \ub3c4\uc640\uc900 \uc778\uc5f0\"\uc73c\ub85c\uc11c \"\uc77c\ubd80 \uc5f0\uc124\ubb38 \ub4f1\uc740 \ucd5c\uc21c\uc2e4 \uc528\uc5d0\uac8c \ub3c4\uc6c0\ubc1b\uc558\ub2e4\"\uba70 \uc2dc\uc778\ud558\uc600\ub2e4. \uadf8\ub7ec\ub098 1\ucc28 \ub300\uad6d\ubbfc\uc0ac\uacfc\ub294 \uc9c0\uae08\uae4c\uc9c0\uc758 \uc0ac\ud0dc\ub97c '\uac1c\uc778\uc801 \uc778\uc5f0'\uc73c\ub85c\ub9cc \uc124\uba85\ud558\ub824\ub294 \ud0dc\ub3c4\ub85c\uc11c \ub300\ud1b5\ub839\uc774 \uc815\ub9d0\ub85c \uad6d\uc815\uc6b4\uc601 \ucc45\uc784\uc790\ub85c\uc11c \ubb34\uac70\uc6b4 \ucc45\uc784\uc744 \uc778\uc2dd\ud558\uc600\ub2e4\uace0\ub294 \ubcfc \uc218 \uc5c6\uc5c8\uae30\uc5d0, \uc624\ud788\ub824 \ubc15\uadfc\ud61c-\ucd5c\uc21c\uc2e4 \uad00\uacc4\uc5d0 \ub300\ud55c \uc758\ud639\uc744 \uc99d\ud3ed\uc2dc\ud0a4\ub294 \uacb0\uacfc\ub9cc \ub0b3\uc558\ub2e4. \ub300\uad6d\ubbfc \uc0ac\uacfc \uc9c1\ud6c4\uc778 4\uc2dc 35\ubd84\uacbd \ub124\uc774\ubc84\uc640 \ub2e4\uc74c\uc758 \uc778\uae30\uac80\uc0c9\uc5b4\uc5d0\ub294 \"\ud0c4\ud575\"\uc774\ub098 \"\ubc15\uadfc\ud61c \ud0c4\ud575\", \"\ud558\uc57c\" \ub4f1\uc758 \ud0a4\uc6cc\ub4dc\uac00 \uc0c1\uc704\uad8c\uc5d0 \uc62c\ub790\ub2e4. \uc5ec\ub860\ub3c4 \uae09\uaca9\ud788 \uc545\ud654\ud574 \ub9ac\uc5bc\ubbf8\ud130\uc758 \ubc15\uadfc\ud61c \ub300\ud1b5\ub839 \uad6d\uc815 \uc9c0\uc9c0\ub3c4 \uc870\uc0ac\uc5d0\uc11c\ub294 25\uc77c\uc5d0\ub294 22.7%, 26\uc77c\uc5d0\ub294 17.5%\ub85c, \ubd88\uacfc \uc774\ud2c0 \ub9cc\uc5d0 10%\ub300\ub85c \uc8fc\uc800\uc549\uc740 \uac83\uc73c\ub85c \uc870\uc0ac\ub418\uc5c8\ub2e4.", "paragraph_answer": "\uacb0\uad6d \ub2e4\uc74c \ub0a0\uc778 10\uc6d4 25\uc77c \uc624\ud6c4 4\uc2dc, \ubc15\uadfc\ud61c \ub300\ud1b5\ub839\uc740 \uccad\uc640\ub300 \ucd98\ucd94\uad00 \uc5d0\uc11c \ub300\uad6d\ubbfc\uc0ac\uacfc\ub97c \uc9c4\ud589\ud574 \"\ucd5c\uc21c\uc2e4 \uc528\ub294 \uacfc\uac70 \uc5b4\ub824\uc6b8 \ub54c \ub3c4\uc640\uc900 \uc778\uc5f0\"\uc73c\ub85c\uc11c \"\uc77c\ubd80 \uc5f0\uc124\ubb38 \ub4f1\uc740 \ucd5c\uc21c\uc2e4 \uc528\uc5d0\uac8c \ub3c4\uc6c0\ubc1b\uc558\ub2e4\"\uba70 \uc2dc\uc778\ud558\uc600\ub2e4. \uadf8\ub7ec\ub098 1\ucc28 \ub300\uad6d\ubbfc\uc0ac\uacfc\ub294 \uc9c0\uae08\uae4c\uc9c0\uc758 \uc0ac\ud0dc\ub97c '\uac1c\uc778\uc801 \uc778\uc5f0'\uc73c\ub85c\ub9cc \uc124\uba85\ud558\ub824\ub294 \ud0dc\ub3c4\ub85c\uc11c \ub300\ud1b5\ub839\uc774 \uc815\ub9d0\ub85c \uad6d\uc815\uc6b4\uc601 \ucc45\uc784\uc790\ub85c\uc11c \ubb34\uac70\uc6b4 \ucc45\uc784\uc744 \uc778\uc2dd\ud558\uc600\ub2e4\uace0\ub294 \ubcfc \uc218 \uc5c6\uc5c8\uae30\uc5d0, \uc624\ud788\ub824 \ubc15\uadfc\ud61c-\ucd5c\uc21c\uc2e4 \uad00\uacc4\uc5d0 \ub300\ud55c \uc758\ud639\uc744 \uc99d\ud3ed\uc2dc\ud0a4\ub294 \uacb0\uacfc\ub9cc \ub0b3\uc558\ub2e4. \ub300\uad6d\ubbfc \uc0ac\uacfc \uc9c1\ud6c4\uc778 4\uc2dc 35\ubd84\uacbd \ub124\uc774\ubc84\uc640 \ub2e4\uc74c\uc758 \uc778\uae30\uac80\uc0c9\uc5b4\uc5d0\ub294 \"\ud0c4\ud575\"\uc774\ub098 \"\ubc15\uadfc\ud61c \ud0c4\ud575\", \"\ud558\uc57c\" \ub4f1\uc758 \ud0a4\uc6cc\ub4dc\uac00 \uc0c1\uc704\uad8c\uc5d0 \uc62c\ub790\ub2e4. \uc5ec\ub860\ub3c4 \uae09\uaca9\ud788 \uc545\ud654\ud574 \ub9ac\uc5bc\ubbf8\ud130\uc758 \ubc15\uadfc\ud61c \ub300\ud1b5\ub839 \uad6d\uc815 \uc9c0\uc9c0\ub3c4 \uc870\uc0ac\uc5d0\uc11c\ub294 25\uc77c\uc5d0\ub294 22.7%, 26\uc77c\uc5d0\ub294 17.5%\ub85c, \ubd88\uacfc \uc774\ud2c0 \ub9cc\uc5d0 10%\ub300\ub85c \uc8fc\uc800\uc549\uc740 \uac83\uc73c\ub85c \uc870\uc0ac\ub418\uc5c8\ub2e4.", "sentence_answer": "\uacb0\uad6d \ub2e4\uc74c \ub0a0\uc778 10\uc6d4 25\uc77c \uc624\ud6c4 4\uc2dc, \ubc15\uadfc\ud61c \ub300\ud1b5\ub839\uc740 \uccad\uc640\ub300 \ucd98\ucd94\uad00 \uc5d0\uc11c \ub300\uad6d\ubbfc\uc0ac\uacfc\ub97c \uc9c4\ud589\ud574 \"\ucd5c\uc21c\uc2e4 \uc528\ub294 \uacfc\uac70 \uc5b4\ub824\uc6b8 \ub54c \ub3c4\uc640\uc900 \uc778\uc5f0\"\uc73c\ub85c\uc11c \"\uc77c\ubd80 \uc5f0\uc124\ubb38 \ub4f1\uc740 \ucd5c\uc21c\uc2e4 \uc528\uc5d0\uac8c \ub3c4\uc6c0\ubc1b\uc558\ub2e4\"\uba70 \uc2dc\uc778\ud558\uc600\ub2e4.", "paragraph_id": "6534298-1-0", "paragraph_question": "question: 10\uc6d4 25\uc77c\uc5d0 \ubc15\ub300\ud1b5\ub839\uc774 \ub300\uad6d\ubbfc\uc0ac\uacfc\ub97c \uc9c4\ud589\ud55c \uacf3\uc740?, context: \uacb0\uad6d \ub2e4\uc74c \ub0a0\uc778 10\uc6d4 25\uc77c \uc624\ud6c4 4\uc2dc, \ubc15\uadfc\ud61c \ub300\ud1b5\ub839\uc740 \uccad\uc640\ub300 \ucd98\ucd94\uad00\uc5d0\uc11c \ub300\uad6d\ubbfc\uc0ac\uacfc\ub97c \uc9c4\ud589\ud574 \"\ucd5c\uc21c\uc2e4 \uc528\ub294 \uacfc\uac70 \uc5b4\ub824\uc6b8 \ub54c \ub3c4\uc640\uc900 \uc778\uc5f0\"\uc73c\ub85c\uc11c \"\uc77c\ubd80 \uc5f0\uc124\ubb38 \ub4f1\uc740 \ucd5c\uc21c\uc2e4 \uc528\uc5d0\uac8c \ub3c4\uc6c0\ubc1b\uc558\ub2e4\"\uba70 \uc2dc\uc778\ud558\uc600\ub2e4. \uadf8\ub7ec\ub098 1\ucc28 \ub300\uad6d\ubbfc\uc0ac\uacfc\ub294 \uc9c0\uae08\uae4c\uc9c0\uc758 \uc0ac\ud0dc\ub97c '\uac1c\uc778\uc801 \uc778\uc5f0'\uc73c\ub85c\ub9cc \uc124\uba85\ud558\ub824\ub294 \ud0dc\ub3c4\ub85c\uc11c \ub300\ud1b5\ub839\uc774 \uc815\ub9d0\ub85c \uad6d\uc815\uc6b4\uc601 \ucc45\uc784\uc790\ub85c\uc11c \ubb34\uac70\uc6b4 \ucc45\uc784\uc744 \uc778\uc2dd\ud558\uc600\ub2e4\uace0\ub294 \ubcfc \uc218 \uc5c6\uc5c8\uae30\uc5d0, \uc624\ud788\ub824 \ubc15\uadfc\ud61c-\ucd5c\uc21c\uc2e4 \uad00\uacc4\uc5d0 \ub300\ud55c \uc758\ud639\uc744 \uc99d\ud3ed\uc2dc\ud0a4\ub294 \uacb0\uacfc\ub9cc \ub0b3\uc558\ub2e4. \ub300\uad6d\ubbfc \uc0ac\uacfc \uc9c1\ud6c4\uc778 4\uc2dc 35\ubd84\uacbd \ub124\uc774\ubc84\uc640 \ub2e4\uc74c\uc758 \uc778\uae30\uac80\uc0c9\uc5b4\uc5d0\ub294 \"\ud0c4\ud575\"\uc774\ub098 \"\ubc15\uadfc\ud61c \ud0c4\ud575\", \"\ud558\uc57c\" \ub4f1\uc758 \ud0a4\uc6cc\ub4dc\uac00 \uc0c1\uc704\uad8c\uc5d0 \uc62c\ub790\ub2e4. \uc5ec\ub860\ub3c4 \uae09\uaca9\ud788 \uc545\ud654\ud574 \ub9ac\uc5bc\ubbf8\ud130\uc758 \ubc15\uadfc\ud61c \ub300\ud1b5\ub839 \uad6d\uc815 \uc9c0\uc9c0\ub3c4 \uc870\uc0ac\uc5d0\uc11c\ub294 25\uc77c\uc5d0\ub294 22.7%, 26\uc77c\uc5d0\ub294 17.5%\ub85c, \ubd88\uacfc \uc774\ud2c0 \ub9cc\uc5d0 10%\ub300\ub85c \uc8fc\uc800\uc549\uc740 \uac83\uc73c\ub85c \uc870\uc0ac\ub418\uc5c8\ub2e4."} {"question": "\uc624\ubc84\uc6cc\uce58\uc5d0\uc11c \uc0dd\uba85\ub825\uc774 \ub0ae\uc9c0\ub9cc \ube60\ub978 \uc18d\ub3c4\uc640 \uacf5\uaca9\ub825\uc744 \uac00\uc9c4 \uc601\uc6c5\uc740?", "paragraph": "\u300a\uc624\ubc84\uc6cc\uce58\u300b\ub294 \uac01\uac01 \uc5ec\uc12f \uba85\uc758 \ud50c\ub808\uc774\uc5b4\ub85c \uad6c\uc131\ub41c \ub450 \ud300 \uac04\uc758 \ubd84\ub300 \ub2e8\uc704 \uc804\ud22c\ub97c \uae30\ubcf8\uc73c\ub85c \ud55c\ub2e4. \ud50c\ub808\uc774\uc5b4\ub294 \uc800\ub9c8\ub2e4\uc758 \ub3c5\ud2b9\ud55c \ub2a5\ub825\uacfc \uc5ed\ud560\uc744 \uac00\uc9c4 \uc601\uc6c5 \uce90\ub9ad\ud130 \uc911 \ud55c \uba85\uc744 \uc120\ud0dd\ud55c\ub2e4. \uce90\ub9ad\ud130\uc758 \uc5ed\ud560\uc740 \ucd1d \ub124 \uac00\uc9c0\ub2e4. \uacf5\uaca9 \uc601\uc6c5\ub4e4\uc740 \ubcf4\ud1b5 \ube60\ub978 \uc18d\ub3c4\uc640 \uacf5\uaca9\uc744 \uac00\uc84c\uc73c\ub098 \uc0dd\uba85\ub825\uc774 \ub0ae\uc73c\uba70, \uc218\ube44 \uc601\uc6c5\ub4e4\uc740 \uc870\uc784\ubaa9\uc5d0\uc11c \uc801\uc758 \ub3cc\uaca9\uc744 \ub9c9\uace0 \uac70\uc810\uc218\ube44\uc5d0 \ud6a8\uacfc\uc801\uc774\uba70, \ub3cc\uaca9 \uc601\uc6c5\uc740 \uc801\uc758 \uacf5\uaca9\uc744 \uacac\ub38c\ub0b4\ub294 \ubc29\uc5b4\uad6c\uc640 \uc0dd\uba85\ub825\uc744 \uac16\ucd94\uace0 \uc544\uad70\uc5d0\uac8c \ud5a5\ud558\ub294 \uacf5\uaca9\uc744 \uc790\uc2e0\uc5d0\uac8c \uc9d1\uc911\uc2dc\ucf1c \uc544\uad70\uc744 \ubcf4\ud638\ud558\uace0 \ud300\uc758 \uacf5\uaca9\uc744 \uc8fc\ub3c4\ud558\uba70, \uc9c0\uc6d0 \uc601\uc6c5\uc740 \uc544\uad70\uacfc \uc801\uad70 \uac01\uac01\uc5d0 \ub300\ud55c \uac15\ud654\uc640 \uc57d\ud654(\uce58\ub8cc\ub098 \uc18d\ub3c4 \ubcc0\ud654 \ub4f1)\ub97c \ubd80\uc5ec\ud558\uace0 \uc0dd\uc874\uc744\ub3d5\ub294\ub2e4(\uc21c\uac04\uc774\ub3d9\uae30, \ubcf4\ud638\ub9c9). \uacbd\uae30\uc5d0 \ub4e4\uc5b4\uac00\uae30 \uc804 \uc601\uc6c5 \uc120\ud0dd \ud654\uba74\uc5d0\uc11c, \ud300\uc758 \ud50c\ub808\uc774\uc5b4\ub294 \uc218\ube44 \uc601\uc6c5\uc774 \ubd80\uc871\ud560 \ub54c\ucc98\ub7fc \ud300\uc758 \ubc38\ub7f0\uc2a4\uac00 \ub9de\uc9c0 \uc54a\uc744 \uacbd\uc6b0 \uc870\uc5b8\uc744 \ubc1b\uc73c\uba70, \ud50c\ub808\uc774\uc5b4\ub294 \uacbd\uae30 \uc804\uc5d0 \ub2e4\ub978 \uc601\uc6c5\uc73c\ub85c \ubcc0\uacbd\ud558\uc5ec \ubc38\ub7f0\uc2a4\ub97c \ub9de\ucd98 \ub4a4 \ud300\uc744 \uc2dc\uc791\ud560 \uc218 \uc788\ub2e4. \uacbd\uae30 \ub3c4\uc911\uc5d0 \ud50c\ub808\uc774\uc5b4\ub294 \uce90\ub9ad\ud130\uac00 \uc8fd\uac70\ub098 \ubcf8\uc9c4\uc73c\ub85c \ub3cc\uc544\uac00\uba74 \ub2e4\ub978 \uc601\uc6c5\uc73c\ub85c \ubcc0\uacbd\ud560 \uc218 \uc788\uc73c\uba70, \uac8c\uc784\uc758 \uc804\uccb4\uc801\uc778 \uc124\uacc4\uc5d0\uc11c \uc774\ub97c \uad8c\uc7a5\ud558\uace0 \uc788\ub2e4.", "answer": "\uacf5\uaca9 \uc601\uc6c5", "sentence": "\uacf5\uaca9 \uc601\uc6c5 \ub4e4\uc740 \ubcf4\ud1b5 \ube60\ub978 \uc18d\ub3c4\uc640 \uacf5\uaca9\uc744 \uac00\uc84c\uc73c\ub098 \uc0dd\uba85\ub825\uc774 \ub0ae\uc73c\uba70, \uc218\ube44 \uc601\uc6c5\ub4e4\uc740 \uc870\uc784\ubaa9\uc5d0\uc11c \uc801\uc758 \ub3cc\uaca9\uc744 \ub9c9\uace0 \uac70\uc810\uc218\ube44\uc5d0 \ud6a8\uacfc\uc801\uc774\uba70, \ub3cc\uaca9 \uc601\uc6c5\uc740 \uc801\uc758 \uacf5\uaca9\uc744 \uacac\ub38c\ub0b4\ub294 \ubc29\uc5b4\uad6c\uc640 \uc0dd\uba85\ub825\uc744 \uac16\ucd94\uace0 \uc544\uad70\uc5d0\uac8c \ud5a5\ud558\ub294 \uacf5\uaca9\uc744 \uc790\uc2e0\uc5d0\uac8c \uc9d1\uc911\uc2dc\ucf1c \uc544\uad70\uc744 \ubcf4\ud638\ud558\uace0 \ud300\uc758 \uacf5\uaca9\uc744 \uc8fc\ub3c4\ud558\uba70, \uc9c0\uc6d0 \uc601\uc6c5\uc740 \uc544\uad70\uacfc \uc801\uad70 \uac01\uac01\uc5d0 \ub300\ud55c \uac15\ud654\uc640 \uc57d\ud654(\uce58\ub8cc\ub098 \uc18d\ub3c4 \ubcc0\ud654 \ub4f1)\ub97c \ubd80\uc5ec\ud558\uace0 \uc0dd\uc874\uc744\ub3d5\ub294\ub2e4(\uc21c\uac04\uc774\ub3d9\uae30, \ubcf4\ud638\ub9c9).", "paragraph_sentence": "\u300a\uc624\ubc84\uc6cc\uce58\u300b\ub294 \uac01\uac01 \uc5ec\uc12f \uba85\uc758 \ud50c\ub808\uc774\uc5b4\ub85c \uad6c\uc131\ub41c \ub450 \ud300 \uac04\uc758 \ubd84\ub300 \ub2e8\uc704 \uc804\ud22c\ub97c \uae30\ubcf8\uc73c\ub85c \ud55c\ub2e4. \ud50c\ub808\uc774\uc5b4\ub294 \uc800\ub9c8\ub2e4\uc758 \ub3c5\ud2b9\ud55c \ub2a5\ub825\uacfc \uc5ed\ud560\uc744 \uac00\uc9c4 \uc601\uc6c5 \uce90\ub9ad\ud130 \uc911 \ud55c \uba85\uc744 \uc120\ud0dd\ud55c\ub2e4. \uce90\ub9ad\ud130\uc758 \uc5ed\ud560\uc740 \ucd1d \ub124 \uac00\uc9c0\ub2e4. \uacf5\uaca9 \uc601\uc6c5 \ub4e4\uc740 \ubcf4\ud1b5 \ube60\ub978 \uc18d\ub3c4\uc640 \uacf5\uaca9\uc744 \uac00\uc84c\uc73c\ub098 \uc0dd\uba85\ub825\uc774 \ub0ae\uc73c\uba70, \uc218\ube44 \uc601\uc6c5\ub4e4\uc740 \uc870\uc784\ubaa9\uc5d0\uc11c \uc801\uc758 \ub3cc\uaca9\uc744 \ub9c9\uace0 \uac70\uc810\uc218\ube44\uc5d0 \ud6a8\uacfc\uc801\uc774\uba70, \ub3cc\uaca9 \uc601\uc6c5\uc740 \uc801\uc758 \uacf5\uaca9\uc744 \uacac\ub38c\ub0b4\ub294 \ubc29\uc5b4\uad6c\uc640 \uc0dd\uba85\ub825\uc744 \uac16\ucd94\uace0 \uc544\uad70\uc5d0\uac8c \ud5a5\ud558\ub294 \uacf5\uaca9\uc744 \uc790\uc2e0\uc5d0\uac8c \uc9d1\uc911\uc2dc\ucf1c \uc544\uad70\uc744 \ubcf4\ud638\ud558\uace0 \ud300\uc758 \uacf5\uaca9\uc744 \uc8fc\ub3c4\ud558\uba70, \uc9c0\uc6d0 \uc601\uc6c5\uc740 \uc544\uad70\uacfc \uc801\uad70 \uac01\uac01\uc5d0 \ub300\ud55c \uac15\ud654\uc640 \uc57d\ud654(\uce58\ub8cc\ub098 \uc18d\ub3c4 \ubcc0\ud654 \ub4f1)\ub97c \ubd80\uc5ec\ud558\uace0 \uc0dd\uc874\uc744\ub3d5\ub294\ub2e4(\uc21c\uac04\uc774\ub3d9\uae30, \ubcf4\ud638\ub9c9). \uacbd\uae30\uc5d0 \ub4e4\uc5b4\uac00\uae30 \uc804 \uc601\uc6c5 \uc120\ud0dd \ud654\uba74\uc5d0\uc11c, \ud300\uc758 \ud50c\ub808\uc774\uc5b4\ub294 \uc218\ube44 \uc601\uc6c5\uc774 \ubd80\uc871\ud560 \ub54c\ucc98\ub7fc \ud300\uc758 \ubc38\ub7f0\uc2a4\uac00 \ub9de\uc9c0 \uc54a\uc744 \uacbd\uc6b0 \uc870\uc5b8\uc744 \ubc1b\uc73c\uba70, \ud50c\ub808\uc774\uc5b4\ub294 \uacbd\uae30 \uc804\uc5d0 \ub2e4\ub978 \uc601\uc6c5\uc73c\ub85c \ubcc0\uacbd\ud558\uc5ec \ubc38\ub7f0\uc2a4\ub97c \ub9de\ucd98 \ub4a4 \ud300\uc744 \uc2dc\uc791\ud560 \uc218 \uc788\ub2e4. \uacbd\uae30 \ub3c4\uc911\uc5d0 \ud50c\ub808\uc774\uc5b4\ub294 \uce90\ub9ad\ud130\uac00 \uc8fd\uac70\ub098 \ubcf8\uc9c4\uc73c\ub85c \ub3cc\uc544\uac00\uba74 \ub2e4\ub978 \uc601\uc6c5\uc73c\ub85c \ubcc0\uacbd\ud560 \uc218 \uc788\uc73c\uba70, \uac8c\uc784\uc758 \uc804\uccb4\uc801\uc778 \uc124\uacc4\uc5d0\uc11c \uc774\ub97c \uad8c\uc7a5\ud558\uace0 \uc788\ub2e4.", "paragraph_answer": "\u300a\uc624\ubc84\uc6cc\uce58\u300b\ub294 \uac01\uac01 \uc5ec\uc12f \uba85\uc758 \ud50c\ub808\uc774\uc5b4\ub85c \uad6c\uc131\ub41c \ub450 \ud300 \uac04\uc758 \ubd84\ub300 \ub2e8\uc704 \uc804\ud22c\ub97c \uae30\ubcf8\uc73c\ub85c \ud55c\ub2e4. \ud50c\ub808\uc774\uc5b4\ub294 \uc800\ub9c8\ub2e4\uc758 \ub3c5\ud2b9\ud55c \ub2a5\ub825\uacfc \uc5ed\ud560\uc744 \uac00\uc9c4 \uc601\uc6c5 \uce90\ub9ad\ud130 \uc911 \ud55c \uba85\uc744 \uc120\ud0dd\ud55c\ub2e4. \uce90\ub9ad\ud130\uc758 \uc5ed\ud560\uc740 \ucd1d \ub124 \uac00\uc9c0\ub2e4. \uacf5\uaca9 \uc601\uc6c5 \ub4e4\uc740 \ubcf4\ud1b5 \ube60\ub978 \uc18d\ub3c4\uc640 \uacf5\uaca9\uc744 \uac00\uc84c\uc73c\ub098 \uc0dd\uba85\ub825\uc774 \ub0ae\uc73c\uba70, \uc218\ube44 \uc601\uc6c5\ub4e4\uc740 \uc870\uc784\ubaa9\uc5d0\uc11c \uc801\uc758 \ub3cc\uaca9\uc744 \ub9c9\uace0 \uac70\uc810\uc218\ube44\uc5d0 \ud6a8\uacfc\uc801\uc774\uba70, \ub3cc\uaca9 \uc601\uc6c5\uc740 \uc801\uc758 \uacf5\uaca9\uc744 \uacac\ub38c\ub0b4\ub294 \ubc29\uc5b4\uad6c\uc640 \uc0dd\uba85\ub825\uc744 \uac16\ucd94\uace0 \uc544\uad70\uc5d0\uac8c \ud5a5\ud558\ub294 \uacf5\uaca9\uc744 \uc790\uc2e0\uc5d0\uac8c \uc9d1\uc911\uc2dc\ucf1c \uc544\uad70\uc744 \ubcf4\ud638\ud558\uace0 \ud300\uc758 \uacf5\uaca9\uc744 \uc8fc\ub3c4\ud558\uba70, \uc9c0\uc6d0 \uc601\uc6c5\uc740 \uc544\uad70\uacfc \uc801\uad70 \uac01\uac01\uc5d0 \ub300\ud55c \uac15\ud654\uc640 \uc57d\ud654(\uce58\ub8cc\ub098 \uc18d\ub3c4 \ubcc0\ud654 \ub4f1)\ub97c \ubd80\uc5ec\ud558\uace0 \uc0dd\uc874\uc744\ub3d5\ub294\ub2e4(\uc21c\uac04\uc774\ub3d9\uae30, \ubcf4\ud638\ub9c9). \uacbd\uae30\uc5d0 \ub4e4\uc5b4\uac00\uae30 \uc804 \uc601\uc6c5 \uc120\ud0dd \ud654\uba74\uc5d0\uc11c, \ud300\uc758 \ud50c\ub808\uc774\uc5b4\ub294 \uc218\ube44 \uc601\uc6c5\uc774 \ubd80\uc871\ud560 \ub54c\ucc98\ub7fc \ud300\uc758 \ubc38\ub7f0\uc2a4\uac00 \ub9de\uc9c0 \uc54a\uc744 \uacbd\uc6b0 \uc870\uc5b8\uc744 \ubc1b\uc73c\uba70, \ud50c\ub808\uc774\uc5b4\ub294 \uacbd\uae30 \uc804\uc5d0 \ub2e4\ub978 \uc601\uc6c5\uc73c\ub85c \ubcc0\uacbd\ud558\uc5ec \ubc38\ub7f0\uc2a4\ub97c \ub9de\ucd98 \ub4a4 \ud300\uc744 \uc2dc\uc791\ud560 \uc218 \uc788\ub2e4. \uacbd\uae30 \ub3c4\uc911\uc5d0 \ud50c\ub808\uc774\uc5b4\ub294 \uce90\ub9ad\ud130\uac00 \uc8fd\uac70\ub098 \ubcf8\uc9c4\uc73c\ub85c \ub3cc\uc544\uac00\uba74 \ub2e4\ub978 \uc601\uc6c5\uc73c\ub85c \ubcc0\uacbd\ud560 \uc218 \uc788\uc73c\uba70, \uac8c\uc784\uc758 \uc804\uccb4\uc801\uc778 \uc124\uacc4\uc5d0\uc11c \uc774\ub97c \uad8c\uc7a5\ud558\uace0 \uc788\ub2e4.", "sentence_answer": " \uacf5\uaca9 \uc601\uc6c5 \ub4e4\uc740 \ubcf4\ud1b5 \ube60\ub978 \uc18d\ub3c4\uc640 \uacf5\uaca9\uc744 \uac00\uc84c\uc73c\ub098 \uc0dd\uba85\ub825\uc774 \ub0ae\uc73c\uba70, \uc218\ube44 \uc601\uc6c5\ub4e4\uc740 \uc870\uc784\ubaa9\uc5d0\uc11c \uc801\uc758 \ub3cc\uaca9\uc744 \ub9c9\uace0 \uac70\uc810\uc218\ube44\uc5d0 \ud6a8\uacfc\uc801\uc774\uba70, \ub3cc\uaca9 \uc601\uc6c5\uc740 \uc801\uc758 \uacf5\uaca9\uc744 \uacac\ub38c\ub0b4\ub294 \ubc29\uc5b4\uad6c\uc640 \uc0dd\uba85\ub825\uc744 \uac16\ucd94\uace0 \uc544\uad70\uc5d0\uac8c \ud5a5\ud558\ub294 \uacf5\uaca9\uc744 \uc790\uc2e0\uc5d0\uac8c \uc9d1\uc911\uc2dc\ucf1c \uc544\uad70\uc744 \ubcf4\ud638\ud558\uace0 \ud300\uc758 \uacf5\uaca9\uc744 \uc8fc\ub3c4\ud558\uba70, \uc9c0\uc6d0 \uc601\uc6c5\uc740 \uc544\uad70\uacfc \uc801\uad70 \uac01\uac01\uc5d0 \ub300\ud55c \uac15\ud654\uc640 \uc57d\ud654(\uce58\ub8cc\ub098 \uc18d\ub3c4 \ubcc0\ud654 \ub4f1)\ub97c \ubd80\uc5ec\ud558\uace0 \uc0dd\uc874\uc744\ub3d5\ub294\ub2e4(\uc21c\uac04\uc774\ub3d9\uae30, \ubcf4\ud638\ub9c9).", "paragraph_id": "6524701-2-1", "paragraph_question": "question: \uc624\ubc84\uc6cc\uce58\uc5d0\uc11c \uc0dd\uba85\ub825\uc774 \ub0ae\uc9c0\ub9cc \ube60\ub978 \uc18d\ub3c4\uc640 \uacf5\uaca9\ub825\uc744 \uac00\uc9c4 \uc601\uc6c5\uc740?, context: \u300a\uc624\ubc84\uc6cc\uce58\u300b\ub294 \uac01\uac01 \uc5ec\uc12f \uba85\uc758 \ud50c\ub808\uc774\uc5b4\ub85c \uad6c\uc131\ub41c \ub450 \ud300 \uac04\uc758 \ubd84\ub300 \ub2e8\uc704 \uc804\ud22c\ub97c \uae30\ubcf8\uc73c\ub85c \ud55c\ub2e4. \ud50c\ub808\uc774\uc5b4\ub294 \uc800\ub9c8\ub2e4\uc758 \ub3c5\ud2b9\ud55c \ub2a5\ub825\uacfc \uc5ed\ud560\uc744 \uac00\uc9c4 \uc601\uc6c5 \uce90\ub9ad\ud130 \uc911 \ud55c \uba85\uc744 \uc120\ud0dd\ud55c\ub2e4. \uce90\ub9ad\ud130\uc758 \uc5ed\ud560\uc740 \ucd1d \ub124 \uac00\uc9c0\ub2e4. \uacf5\uaca9 \uc601\uc6c5\ub4e4\uc740 \ubcf4\ud1b5 \ube60\ub978 \uc18d\ub3c4\uc640 \uacf5\uaca9\uc744 \uac00\uc84c\uc73c\ub098 \uc0dd\uba85\ub825\uc774 \ub0ae\uc73c\uba70, \uc218\ube44 \uc601\uc6c5\ub4e4\uc740 \uc870\uc784\ubaa9\uc5d0\uc11c \uc801\uc758 \ub3cc\uaca9\uc744 \ub9c9\uace0 \uac70\uc810\uc218\ube44\uc5d0 \ud6a8\uacfc\uc801\uc774\uba70, \ub3cc\uaca9 \uc601\uc6c5\uc740 \uc801\uc758 \uacf5\uaca9\uc744 \uacac\ub38c\ub0b4\ub294 \ubc29\uc5b4\uad6c\uc640 \uc0dd\uba85\ub825\uc744 \uac16\ucd94\uace0 \uc544\uad70\uc5d0\uac8c \ud5a5\ud558\ub294 \uacf5\uaca9\uc744 \uc790\uc2e0\uc5d0\uac8c \uc9d1\uc911\uc2dc\ucf1c \uc544\uad70\uc744 \ubcf4\ud638\ud558\uace0 \ud300\uc758 \uacf5\uaca9\uc744 \uc8fc\ub3c4\ud558\uba70, \uc9c0\uc6d0 \uc601\uc6c5\uc740 \uc544\uad70\uacfc \uc801\uad70 \uac01\uac01\uc5d0 \ub300\ud55c \uac15\ud654\uc640 \uc57d\ud654(\uce58\ub8cc\ub098 \uc18d\ub3c4 \ubcc0\ud654 \ub4f1)\ub97c \ubd80\uc5ec\ud558\uace0 \uc0dd\uc874\uc744\ub3d5\ub294\ub2e4(\uc21c\uac04\uc774\ub3d9\uae30, \ubcf4\ud638\ub9c9). \uacbd\uae30\uc5d0 \ub4e4\uc5b4\uac00\uae30 \uc804 \uc601\uc6c5 \uc120\ud0dd \ud654\uba74\uc5d0\uc11c, \ud300\uc758 \ud50c\ub808\uc774\uc5b4\ub294 \uc218\ube44 \uc601\uc6c5\uc774 \ubd80\uc871\ud560 \ub54c\ucc98\ub7fc \ud300\uc758 \ubc38\ub7f0\uc2a4\uac00 \ub9de\uc9c0 \uc54a\uc744 \uacbd\uc6b0 \uc870\uc5b8\uc744 \ubc1b\uc73c\uba70, \ud50c\ub808\uc774\uc5b4\ub294 \uacbd\uae30 \uc804\uc5d0 \ub2e4\ub978 \uc601\uc6c5\uc73c\ub85c \ubcc0\uacbd\ud558\uc5ec \ubc38\ub7f0\uc2a4\ub97c \ub9de\ucd98 \ub4a4 \ud300\uc744 \uc2dc\uc791\ud560 \uc218 \uc788\ub2e4. \uacbd\uae30 \ub3c4\uc911\uc5d0 \ud50c\ub808\uc774\uc5b4\ub294 \uce90\ub9ad\ud130\uac00 \uc8fd\uac70\ub098 \ubcf8\uc9c4\uc73c\ub85c \ub3cc\uc544\uac00\uba74 \ub2e4\ub978 \uc601\uc6c5\uc73c\ub85c \ubcc0\uacbd\ud560 \uc218 \uc788\uc73c\uba70, \uac8c\uc784\uc758 \uc804\uccb4\uc801\uc778 \uc124\uacc4\uc5d0\uc11c \uc774\ub97c \uad8c\uc7a5\ud558\uace0 \uc788\ub2e4."} {"question": "\ub2f9\uc0ac\uc790 \uc77c\ubc29\uc774 \uadf8 \uc0c1\ub300\ubc29\uc758 \uc2b9\ub099\uc5d0 \uc758\ud558\uc5ec \ubc95\uc801\uc73c\ub85c \uad6c\uc18d\ub825\uc744 \uac16\ub294 \ub9e4\ub9e4\uacc4\uc57d\uc5d0 \ub300\ud574 \ud655\uc815\uc801\uc778 \uc758\uc0ac\ud45c\uc2dc\ub97c \ud558\ub294 \uac83\uc740?", "paragraph": "\uccad\uc57d(\u8acb\u7d04)\uc740 \ub2f9\uc0ac\uc790 \uc77c\ubc29\uc774 \uadf8 \uc0c1\ub300\ubc29\uc758 \uc2b9\ub099\uc5d0 \uc758\ud558\uc5ec \ubc95\uc801\uc73c\ub85c \uad6c\uc18d\ub825\uc744 \uac16\ub294 \uc77c\uc815\ud55c \ub0b4\uc6a9\uc758 \ub9e4\ub9e4\uacc4\uc57d\uc744 \uc131\ub9bd\uc2dc\ud0ac \uac83\uc744 \uae30\ub300\ud558\uace0 \ud589\ud558\ub294 \ud655\uc815\uc801\uc778 \uc758\uc0ac\ud45c\uc2dc\ub97c \ub9d0\ud55c\ub2e4. \uc774\ub7ec\ud55c \uccad\uc57d\uc5d0 \ub300\ud574 \uc2b9\ub099\uc758 \uc758\uc0ac\ud45c\uc2dc\uac00 \ud569\uce58\ub418\uba74 \uacc4\uc57d\uc774 \uc131\ub9bd\ub41c\ub2e4. \uccad\uc57d\uc5d0\ub294 \uacc4\uc57d\uc758 \ub0b4\uc6a9\uc744 \uacb0\uc815\uc9c0\uc744 \uc218 \uc788\uc744 \ub9cc\ud55c \uc0ac\ud56d\uc744 \ud3ec\ud568\ud574\uc57c \ud558\ub294\ub370, \uadf8 \uc0ac\ud56d\uc774 \uc608\uc57d\u00b7\uad00\uc2b5 \ub4f1\uc5d0\uc11c \uba85\ubc31\ud560 \uacbd\uc6b0\uc5d0\ub294 \uccad\uc57d \uc18d\uc5d0 \ud3ec\ud568\uc2dc\ud0ac \ud544\uc694\uac00 \uc5c6\ub2e4. \uccad\uc57d\uc758 \uc0c1\ub300\ubc29\uc740 \uc790\ub3d9\ud310\ub9e4\uae30\uc5d0 \uc758\ud55c \ud310\ub9e4\uc640 \uac19\uc774 \ubd88\ud2b9\uc815 \ub2e4\uc218\uc778(\u4e0d\u7279\u5b9a\u591a\u6578\u4eba)\uc758 \uacbd\uc6b0\ub3c4 \uc788\ub2e4. \ub300\ud55c\ubbfc\uad6d \ubbfc\ubc95\uc5d0 \uc758\ud558\uba74 \uacc4\uc57d\uc758 \uccad\uc57d\uc740 \ud568\ubd80\ub85c \uc774\ub97c \ucca0\ud68c\ud560 \uc218 \uc5c6\uc73c\ub098(\uc81c527\uc870), \uad6d\uc81c \ubb3c\ud488 \ub9e4\ub9e4 \uacc4\uc57d\uc5d0 \uad00\ud55c \uad6d\uc81c \uc5f0\ud569 \ud611\uc57d\uc5d0\ub294 \u201c\uccad\uc57d\uc774 \ud53c\uccad\uc57d\uc790\uc5d0\uac8c \ub3c4\ub2ec\ud558\uae30 \uc804\uc5d0 \ub610\ub294 \uadf8\uc640 \ub3d9\uc2dc\uc5d0 \ucca0\ud68c\uc758 \uc758\uc0ac\ud45c\uc2dc\uac00 \ub3c4\ub2ec\ud560 \uacbd\uc6b0\uc5d0\ub294 \uc774\ub97c \ucca0\ud68c\ud560 \uc218 \uc788\ub2e4\u201d\uace0 \uaddc\uc815\ud558\uace0 \uc788\ub2e4.(\uc81c15\uc870 2\ud56d).", "answer": "\uccad\uc57d", "sentence": "\uccad\uc57d (\u8acb\u7d04)", "paragraph_sentence": " \uccad\uc57d (\u8acb\u7d04) \uc740 \ub2f9\uc0ac\uc790 \uc77c\ubc29\uc774 \uadf8 \uc0c1\ub300\ubc29\uc758 \uc2b9\ub099\uc5d0 \uc758\ud558\uc5ec \ubc95\uc801\uc73c\ub85c \uad6c\uc18d\ub825\uc744 \uac16\ub294 \uc77c\uc815\ud55c \ub0b4\uc6a9\uc758 \ub9e4\ub9e4\uacc4\uc57d\uc744 \uc131\ub9bd\uc2dc\ud0ac \uac83\uc744 \uae30\ub300\ud558\uace0 \ud589\ud558\ub294 \ud655\uc815\uc801\uc778 \uc758\uc0ac\ud45c\uc2dc\ub97c \ub9d0\ud55c\ub2e4. \uc774\ub7ec\ud55c \uccad\uc57d\uc5d0 \ub300\ud574 \uc2b9\ub099\uc758 \uc758\uc0ac\ud45c\uc2dc\uac00 \ud569\uce58\ub418\uba74 \uacc4\uc57d\uc774 \uc131\ub9bd\ub41c\ub2e4. \uccad\uc57d\uc5d0\ub294 \uacc4\uc57d\uc758 \ub0b4\uc6a9\uc744 \uacb0\uc815\uc9c0\uc744 \uc218 \uc788\uc744 \ub9cc\ud55c \uc0ac\ud56d\uc744 \ud3ec\ud568\ud574\uc57c \ud558\ub294\ub370, \uadf8 \uc0ac\ud56d\uc774 \uc608\uc57d\u00b7\uad00\uc2b5 \ub4f1\uc5d0\uc11c \uba85\ubc31\ud560 \uacbd\uc6b0\uc5d0\ub294 \uccad\uc57d \uc18d\uc5d0 \ud3ec\ud568\uc2dc\ud0ac \ud544\uc694\uac00 \uc5c6\ub2e4. \uccad\uc57d\uc758 \uc0c1\ub300\ubc29\uc740 \uc790\ub3d9\ud310\ub9e4\uae30\uc5d0 \uc758\ud55c \ud310\ub9e4\uc640 \uac19\uc774 \ubd88\ud2b9\uc815 \ub2e4\uc218\uc778(\u4e0d\u7279\u5b9a\u591a\u6578\u4eba)\uc758 \uacbd\uc6b0\ub3c4 \uc788\ub2e4. \ub300\ud55c\ubbfc\uad6d \ubbfc\ubc95\uc5d0 \uc758\ud558\uba74 \uacc4\uc57d\uc758 \uccad\uc57d\uc740 \ud568\ubd80\ub85c \uc774\ub97c \ucca0\ud68c\ud560 \uc218 \uc5c6\uc73c\ub098(\uc81c527\uc870), \uad6d\uc81c \ubb3c\ud488 \ub9e4\ub9e4 \uacc4\uc57d\uc5d0 \uad00\ud55c \uad6d\uc81c \uc5f0\ud569 \ud611\uc57d\uc5d0\ub294 \u201c\uccad\uc57d\uc774 \ud53c\uccad\uc57d\uc790\uc5d0\uac8c \ub3c4\ub2ec\ud558\uae30 \uc804\uc5d0 \ub610\ub294 \uadf8\uc640 \ub3d9\uc2dc\uc5d0 \ucca0\ud68c\uc758 \uc758\uc0ac\ud45c\uc2dc\uac00 \ub3c4\ub2ec\ud560 \uacbd\uc6b0\uc5d0\ub294 \uc774\ub97c \ucca0\ud68c\ud560 \uc218 \uc788\ub2e4\u201d\uace0 \uaddc\uc815\ud558\uace0 \uc788\ub2e4.(\uc81c15\uc870 2\ud56d).", "paragraph_answer": " \uccad\uc57d (\u8acb\u7d04)\uc740 \ub2f9\uc0ac\uc790 \uc77c\ubc29\uc774 \uadf8 \uc0c1\ub300\ubc29\uc758 \uc2b9\ub099\uc5d0 \uc758\ud558\uc5ec \ubc95\uc801\uc73c\ub85c \uad6c\uc18d\ub825\uc744 \uac16\ub294 \uc77c\uc815\ud55c \ub0b4\uc6a9\uc758 \ub9e4\ub9e4\uacc4\uc57d\uc744 \uc131\ub9bd\uc2dc\ud0ac \uac83\uc744 \uae30\ub300\ud558\uace0 \ud589\ud558\ub294 \ud655\uc815\uc801\uc778 \uc758\uc0ac\ud45c\uc2dc\ub97c \ub9d0\ud55c\ub2e4. \uc774\ub7ec\ud55c \uccad\uc57d\uc5d0 \ub300\ud574 \uc2b9\ub099\uc758 \uc758\uc0ac\ud45c\uc2dc\uac00 \ud569\uce58\ub418\uba74 \uacc4\uc57d\uc774 \uc131\ub9bd\ub41c\ub2e4. \uccad\uc57d\uc5d0\ub294 \uacc4\uc57d\uc758 \ub0b4\uc6a9\uc744 \uacb0\uc815\uc9c0\uc744 \uc218 \uc788\uc744 \ub9cc\ud55c \uc0ac\ud56d\uc744 \ud3ec\ud568\ud574\uc57c \ud558\ub294\ub370, \uadf8 \uc0ac\ud56d\uc774 \uc608\uc57d\u00b7\uad00\uc2b5 \ub4f1\uc5d0\uc11c \uba85\ubc31\ud560 \uacbd\uc6b0\uc5d0\ub294 \uccad\uc57d \uc18d\uc5d0 \ud3ec\ud568\uc2dc\ud0ac \ud544\uc694\uac00 \uc5c6\ub2e4. \uccad\uc57d\uc758 \uc0c1\ub300\ubc29\uc740 \uc790\ub3d9\ud310\ub9e4\uae30\uc5d0 \uc758\ud55c \ud310\ub9e4\uc640 \uac19\uc774 \ubd88\ud2b9\uc815 \ub2e4\uc218\uc778(\u4e0d\u7279\u5b9a\u591a\u6578\u4eba)\uc758 \uacbd\uc6b0\ub3c4 \uc788\ub2e4. \ub300\ud55c\ubbfc\uad6d \ubbfc\ubc95\uc5d0 \uc758\ud558\uba74 \uacc4\uc57d\uc758 \uccad\uc57d\uc740 \ud568\ubd80\ub85c \uc774\ub97c \ucca0\ud68c\ud560 \uc218 \uc5c6\uc73c\ub098(\uc81c527\uc870), \uad6d\uc81c \ubb3c\ud488 \ub9e4\ub9e4 \uacc4\uc57d\uc5d0 \uad00\ud55c \uad6d\uc81c \uc5f0\ud569 \ud611\uc57d\uc5d0\ub294 \u201c\uccad\uc57d\uc774 \ud53c\uccad\uc57d\uc790\uc5d0\uac8c \ub3c4\ub2ec\ud558\uae30 \uc804\uc5d0 \ub610\ub294 \uadf8\uc640 \ub3d9\uc2dc\uc5d0 \ucca0\ud68c\uc758 \uc758\uc0ac\ud45c\uc2dc\uac00 \ub3c4\ub2ec\ud560 \uacbd\uc6b0\uc5d0\ub294 \uc774\ub97c \ucca0\ud68c\ud560 \uc218 \uc788\ub2e4\u201d\uace0 \uaddc\uc815\ud558\uace0 \uc788\ub2e4.(\uc81c15\uc870 2\ud56d).", "sentence_answer": " \uccad\uc57d (\u8acb\u7d04)", "paragraph_id": "6527244-6-0", "paragraph_question": "question: \ub2f9\uc0ac\uc790 \uc77c\ubc29\uc774 \uadf8 \uc0c1\ub300\ubc29\uc758 \uc2b9\ub099\uc5d0 \uc758\ud558\uc5ec \ubc95\uc801\uc73c\ub85c \uad6c\uc18d\ub825\uc744 \uac16\ub294 \ub9e4\ub9e4\uacc4\uc57d\uc5d0 \ub300\ud574 \ud655\uc815\uc801\uc778 \uc758\uc0ac\ud45c\uc2dc\ub97c \ud558\ub294 \uac83\uc740?, context: \uccad\uc57d(\u8acb\u7d04)\uc740 \ub2f9\uc0ac\uc790 \uc77c\ubc29\uc774 \uadf8 \uc0c1\ub300\ubc29\uc758 \uc2b9\ub099\uc5d0 \uc758\ud558\uc5ec \ubc95\uc801\uc73c\ub85c \uad6c\uc18d\ub825\uc744 \uac16\ub294 \uc77c\uc815\ud55c \ub0b4\uc6a9\uc758 \ub9e4\ub9e4\uacc4\uc57d\uc744 \uc131\ub9bd\uc2dc\ud0ac \uac83\uc744 \uae30\ub300\ud558\uace0 \ud589\ud558\ub294 \ud655\uc815\uc801\uc778 \uc758\uc0ac\ud45c\uc2dc\ub97c \ub9d0\ud55c\ub2e4. \uc774\ub7ec\ud55c \uccad\uc57d\uc5d0 \ub300\ud574 \uc2b9\ub099\uc758 \uc758\uc0ac\ud45c\uc2dc\uac00 \ud569\uce58\ub418\uba74 \uacc4\uc57d\uc774 \uc131\ub9bd\ub41c\ub2e4. \uccad\uc57d\uc5d0\ub294 \uacc4\uc57d\uc758 \ub0b4\uc6a9\uc744 \uacb0\uc815\uc9c0\uc744 \uc218 \uc788\uc744 \ub9cc\ud55c \uc0ac\ud56d\uc744 \ud3ec\ud568\ud574\uc57c \ud558\ub294\ub370, \uadf8 \uc0ac\ud56d\uc774 \uc608\uc57d\u00b7\uad00\uc2b5 \ub4f1\uc5d0\uc11c \uba85\ubc31\ud560 \uacbd\uc6b0\uc5d0\ub294 \uccad\uc57d \uc18d\uc5d0 \ud3ec\ud568\uc2dc\ud0ac \ud544\uc694\uac00 \uc5c6\ub2e4. \uccad\uc57d\uc758 \uc0c1\ub300\ubc29\uc740 \uc790\ub3d9\ud310\ub9e4\uae30\uc5d0 \uc758\ud55c \ud310\ub9e4\uc640 \uac19\uc774 \ubd88\ud2b9\uc815 \ub2e4\uc218\uc778(\u4e0d\u7279\u5b9a\u591a\u6578\u4eba)\uc758 \uacbd\uc6b0\ub3c4 \uc788\ub2e4. \ub300\ud55c\ubbfc\uad6d \ubbfc\ubc95\uc5d0 \uc758\ud558\uba74 \uacc4\uc57d\uc758 \uccad\uc57d\uc740 \ud568\ubd80\ub85c \uc774\ub97c \ucca0\ud68c\ud560 \uc218 \uc5c6\uc73c\ub098(\uc81c527\uc870), \uad6d\uc81c \ubb3c\ud488 \ub9e4\ub9e4 \uacc4\uc57d\uc5d0 \uad00\ud55c \uad6d\uc81c \uc5f0\ud569 \ud611\uc57d\uc5d0\ub294 \u201c\uccad\uc57d\uc774 \ud53c\uccad\uc57d\uc790\uc5d0\uac8c \ub3c4\ub2ec\ud558\uae30 \uc804\uc5d0 \ub610\ub294 \uadf8\uc640 \ub3d9\uc2dc\uc5d0 \ucca0\ud68c\uc758 \uc758\uc0ac\ud45c\uc2dc\uac00 \ub3c4\ub2ec\ud560 \uacbd\uc6b0\uc5d0\ub294 \uc774\ub97c \ucca0\ud68c\ud560 \uc218 \uc788\ub2e4\u201d\uace0 \uaddc\uc815\ud558\uace0 \uc788\ub2e4.(\uc81c15\uc870 2\ud56d)."} {"question": "\ubc11\ubc14\ub2e5\uc5d0\uc11c \ub77c\ub294 \uace1\uc5d0\uc11c \ub9c8\uc9c0\ub9c9 \uc544\uae30 \ubaa9\uc18c\ub9ac\uc758 \uc8fc\uc778\uacf5\uc740?", "paragraph": "\uc74c\ubc18 \ubc1c\ub9e4\uc5d0 \uc55e\uc11c \uc2f1\uae00\ub85c \uc120\uacf5\uac1c\ub418\uae30\ub3c4 \ud588\ub358 \"Airbag\"\uc740 \uc6b4\uc804\uc5d0 \uc11c\ud230 \ud0c0\ube14\ub85c\uac00 \uc0c8\ubcbd\uc5d0 \ud0dd\uc2dc\ub97c \uc790\uc8fc \ud0c0\uac8c \ub418\uba74\uc11c \uadf8\ub85c \ubd80\ud130 \uace1\uc5d0 \ub300\ud55c \uc601\uac10\uc744 \uc5bb\uc5c8\ub2e4. \ud0c0\ube14\ub85c \uc790\uc2e0\uc758 \uc774\uc57c\uae30\ub294 \uc544\ub2c8\uc9c0\ub9cc, '\ud0dd\uc2dc\ub97c \ud0c0\uace0 \uc9d1\uc73c\ub85c \ud5a5\ud558\ub294 \uc5b4\ub290 \ud55c \uc678\ub85c\uc6b4 \uc0ac\ub78c\uc5d0 \ub300\ud55c \uc774\uc57c\uae30' \ub610\ub294 '\ubaa8\ub4e0 \uc678\ub85c\uc6b4 \uc0ac\ub78c\ub4e4\uc758 \uc774\uc57c\uae30'\ub77c\uace0 \ubc1d\ud614\ub2e4. \uc6d0\uace1\uc758 \uac00\uc774\ub4dc \ubcf4\uceec\ub85c \u3008\ubc11\ubc14\ub2e5\uc5d0\uc11c\u3009\uc5d0 \ucc38\uc5ec\ud55c \ubc94\ud0a4\uac00 \ub179\uc74c\ud588\uc5c8\ub2e4. \u3008\ubc00\ubb3c\u3009\uc740 \ud2b9\ubcc4\ud788 \ud559\uc0dd\uc774\ub098 \uc131\uc778\uc774 \ub418\uc5b4\uac00\ub294 \uc0ac\ub78c\uc774 \uacf5\uac10\ud560 \ub9cc\ud55c \uace1\uc73c\ub85c, \ud0c0\ube14\ub85c\uac00 \uc5c9\ub6b1\ud558\uac8c\ub3c4 \"\uc6b0\uc5f0\ud788 \ubc25\uc744 \uba39\ub2e4\uac00 \uba39\ub2e4 \ub0a8\uc740 \uc0dd\uc120\uad6c\uc774\ub97c \ubcf4\uace0 \uc0dd\uac01 \ub098 \uc790\uc2e0\uacfc \ub2ee\uc558\ub2e4\uace0 \uc0dd\uac01\"\ud574 \uadf8 \ub0a0 \uc791\uc0ac\ud558\uc5ec \ub9cc\ub4e4\uc5b4\uc9c0\uac8c \ub418\uc5c8\ub2e4. \u3008\ubc11\ubc14\ub2e5\uc5d0\uc11c\u3009\ub294 \ud0c0\ube14\ub85c\uac00 \uac00\uc7a5 \uc4f0\uae30 \uc5b4\ub824\uc6e0\ub358 \uace1\uc774\uc790 \ub179\uc74c\ud558\uae30 \uc5b4\ub824\uc6e0\ub358 \uace1\uc774\ub2e4. 1\uc808\uc5d0\uc11c\ub294 \uc544\ub0b4\uc5d0\uac8c, 2\uc808\uc5d0\ub294 \uc790\uc2e0\uc758 \ub538\uc5d0\uac8c \ud558\ub294 \uc774\uc57c\uae30\uc774\ub2e4. \ub9c8\uc9c0\ub9c9 \uc544\uae30 \ubaa9\uc18c\ub9ac\ub294 \uc790\uc2e0\uc758 \ub538\uc774 \ucc98\uc74c \u2018\uc544\ube60\u2019\ub77c\uace0 \ud588\uc744 \ub54c\uc758 \ubaa9\uc18c\ub9ac\ub2e4.", "answer": "\ub538", "sentence": "1\uc808\uc5d0\uc11c\ub294 \uc544\ub0b4\uc5d0\uac8c, 2\uc808\uc5d0\ub294 \uc790\uc2e0\uc758 \ub538 \uc5d0\uac8c \ud558\ub294 \uc774\uc57c\uae30\uc774\ub2e4.", "paragraph_sentence": "\uc74c\ubc18 \ubc1c\ub9e4\uc5d0 \uc55e\uc11c \uc2f1\uae00\ub85c \uc120\uacf5\uac1c\ub418\uae30\ub3c4 \ud588\ub358 \"Airbag\"\uc740 \uc6b4\uc804\uc5d0 \uc11c\ud230 \ud0c0\ube14\ub85c\uac00 \uc0c8\ubcbd\uc5d0 \ud0dd\uc2dc\ub97c \uc790\uc8fc \ud0c0\uac8c \ub418\uba74\uc11c \uadf8\ub85c \ubd80\ud130 \uace1\uc5d0 \ub300\ud55c \uc601\uac10\uc744 \uc5bb\uc5c8\ub2e4. \ud0c0\ube14\ub85c \uc790\uc2e0\uc758 \uc774\uc57c\uae30\ub294 \uc544\ub2c8\uc9c0\ub9cc, '\ud0dd\uc2dc\ub97c \ud0c0\uace0 \uc9d1\uc73c\ub85c \ud5a5\ud558\ub294 \uc5b4\ub290 \ud55c \uc678\ub85c\uc6b4 \uc0ac\ub78c\uc5d0 \ub300\ud55c \uc774\uc57c\uae30' \ub610\ub294 '\ubaa8\ub4e0 \uc678\ub85c\uc6b4 \uc0ac\ub78c\ub4e4\uc758 \uc774\uc57c\uae30'\ub77c\uace0 \ubc1d\ud614\ub2e4. \uc6d0\uace1\uc758 \uac00\uc774\ub4dc \ubcf4\uceec\ub85c \u3008\ubc11\ubc14\ub2e5\uc5d0\uc11c\u3009\uc5d0 \ucc38\uc5ec\ud55c \ubc94\ud0a4\uac00 \ub179\uc74c\ud588\uc5c8\ub2e4. \u3008\ubc00\ubb3c\u3009\uc740 \ud2b9\ubcc4\ud788 \ud559\uc0dd\uc774\ub098 \uc131\uc778\uc774 \ub418\uc5b4\uac00\ub294 \uc0ac\ub78c\uc774 \uacf5\uac10\ud560 \ub9cc\ud55c \uace1\uc73c\ub85c, \ud0c0\ube14\ub85c\uac00 \uc5c9\ub6b1\ud558\uac8c\ub3c4 \"\uc6b0\uc5f0\ud788 \ubc25\uc744 \uba39\ub2e4\uac00 \uba39\ub2e4 \ub0a8\uc740 \uc0dd\uc120\uad6c\uc774\ub97c \ubcf4\uace0 \uc0dd\uac01 \ub098 \uc790\uc2e0\uacfc \ub2ee\uc558\ub2e4\uace0 \uc0dd\uac01\"\ud574 \uadf8 \ub0a0 \uc791\uc0ac\ud558\uc5ec \ub9cc\ub4e4\uc5b4\uc9c0\uac8c \ub418\uc5c8\ub2e4. \u3008\ubc11\ubc14\ub2e5\uc5d0\uc11c\u3009\ub294 \ud0c0\ube14\ub85c\uac00 \uac00\uc7a5 \uc4f0\uae30 \uc5b4\ub824\uc6e0\ub358 \uace1\uc774\uc790 \ub179\uc74c\ud558\uae30 \uc5b4\ub824\uc6e0\ub358 \uace1\uc774\ub2e4. 1\uc808\uc5d0\uc11c\ub294 \uc544\ub0b4\uc5d0\uac8c, 2\uc808\uc5d0\ub294 \uc790\uc2e0\uc758 \ub538 \uc5d0\uac8c \ud558\ub294 \uc774\uc57c\uae30\uc774\ub2e4. \ub9c8\uc9c0\ub9c9 \uc544\uae30 \ubaa9\uc18c\ub9ac\ub294 \uc790\uc2e0\uc758 \ub538\uc774 \ucc98\uc74c \u2018\uc544\ube60\u2019\ub77c\uace0 \ud588\uc744 \ub54c\uc758 \ubaa9\uc18c\ub9ac\ub2e4.", "paragraph_answer": "\uc74c\ubc18 \ubc1c\ub9e4\uc5d0 \uc55e\uc11c \uc2f1\uae00\ub85c \uc120\uacf5\uac1c\ub418\uae30\ub3c4 \ud588\ub358 \"Airbag\"\uc740 \uc6b4\uc804\uc5d0 \uc11c\ud230 \ud0c0\ube14\ub85c\uac00 \uc0c8\ubcbd\uc5d0 \ud0dd\uc2dc\ub97c \uc790\uc8fc \ud0c0\uac8c \ub418\uba74\uc11c \uadf8\ub85c \ubd80\ud130 \uace1\uc5d0 \ub300\ud55c \uc601\uac10\uc744 \uc5bb\uc5c8\ub2e4. \ud0c0\ube14\ub85c \uc790\uc2e0\uc758 \uc774\uc57c\uae30\ub294 \uc544\ub2c8\uc9c0\ub9cc, '\ud0dd\uc2dc\ub97c \ud0c0\uace0 \uc9d1\uc73c\ub85c \ud5a5\ud558\ub294 \uc5b4\ub290 \ud55c \uc678\ub85c\uc6b4 \uc0ac\ub78c\uc5d0 \ub300\ud55c \uc774\uc57c\uae30' \ub610\ub294 '\ubaa8\ub4e0 \uc678\ub85c\uc6b4 \uc0ac\ub78c\ub4e4\uc758 \uc774\uc57c\uae30'\ub77c\uace0 \ubc1d\ud614\ub2e4. \uc6d0\uace1\uc758 \uac00\uc774\ub4dc \ubcf4\uceec\ub85c \u3008\ubc11\ubc14\ub2e5\uc5d0\uc11c\u3009\uc5d0 \ucc38\uc5ec\ud55c \ubc94\ud0a4\uac00 \ub179\uc74c\ud588\uc5c8\ub2e4. \u3008\ubc00\ubb3c\u3009\uc740 \ud2b9\ubcc4\ud788 \ud559\uc0dd\uc774\ub098 \uc131\uc778\uc774 \ub418\uc5b4\uac00\ub294 \uc0ac\ub78c\uc774 \uacf5\uac10\ud560 \ub9cc\ud55c \uace1\uc73c\ub85c, \ud0c0\ube14\ub85c\uac00 \uc5c9\ub6b1\ud558\uac8c\ub3c4 \"\uc6b0\uc5f0\ud788 \ubc25\uc744 \uba39\ub2e4\uac00 \uba39\ub2e4 \ub0a8\uc740 \uc0dd\uc120\uad6c\uc774\ub97c \ubcf4\uace0 \uc0dd\uac01 \ub098 \uc790\uc2e0\uacfc \ub2ee\uc558\ub2e4\uace0 \uc0dd\uac01\"\ud574 \uadf8 \ub0a0 \uc791\uc0ac\ud558\uc5ec \ub9cc\ub4e4\uc5b4\uc9c0\uac8c \ub418\uc5c8\ub2e4. \u3008\ubc11\ubc14\ub2e5\uc5d0\uc11c\u3009\ub294 \ud0c0\ube14\ub85c\uac00 \uac00\uc7a5 \uc4f0\uae30 \uc5b4\ub824\uc6e0\ub358 \uace1\uc774\uc790 \ub179\uc74c\ud558\uae30 \uc5b4\ub824\uc6e0\ub358 \uace1\uc774\ub2e4. 1\uc808\uc5d0\uc11c\ub294 \uc544\ub0b4\uc5d0\uac8c, 2\uc808\uc5d0\ub294 \uc790\uc2e0\uc758 \ub538 \uc5d0\uac8c \ud558\ub294 \uc774\uc57c\uae30\uc774\ub2e4. \ub9c8\uc9c0\ub9c9 \uc544\uae30 \ubaa9\uc18c\ub9ac\ub294 \uc790\uc2e0\uc758 \ub538\uc774 \ucc98\uc74c \u2018\uc544\ube60\u2019\ub77c\uace0 \ud588\uc744 \ub54c\uc758 \ubaa9\uc18c\ub9ac\ub2e4.", "sentence_answer": "1\uc808\uc5d0\uc11c\ub294 \uc544\ub0b4\uc5d0\uac8c, 2\uc808\uc5d0\ub294 \uc790\uc2e0\uc758 \ub538 \uc5d0\uac8c \ud558\ub294 \uc774\uc57c\uae30\uc774\ub2e4.", "paragraph_id": "6524108-2-2", "paragraph_question": "question: \ubc11\ubc14\ub2e5\uc5d0\uc11c \ub77c\ub294 \uace1\uc5d0\uc11c \ub9c8\uc9c0\ub9c9 \uc544\uae30 \ubaa9\uc18c\ub9ac\uc758 \uc8fc\uc778\uacf5\uc740?, context: \uc74c\ubc18 \ubc1c\ub9e4\uc5d0 \uc55e\uc11c \uc2f1\uae00\ub85c \uc120\uacf5\uac1c\ub418\uae30\ub3c4 \ud588\ub358 \"Airbag\"\uc740 \uc6b4\uc804\uc5d0 \uc11c\ud230 \ud0c0\ube14\ub85c\uac00 \uc0c8\ubcbd\uc5d0 \ud0dd\uc2dc\ub97c \uc790\uc8fc \ud0c0\uac8c \ub418\uba74\uc11c \uadf8\ub85c \ubd80\ud130 \uace1\uc5d0 \ub300\ud55c \uc601\uac10\uc744 \uc5bb\uc5c8\ub2e4. \ud0c0\ube14\ub85c \uc790\uc2e0\uc758 \uc774\uc57c\uae30\ub294 \uc544\ub2c8\uc9c0\ub9cc, '\ud0dd\uc2dc\ub97c \ud0c0\uace0 \uc9d1\uc73c\ub85c \ud5a5\ud558\ub294 \uc5b4\ub290 \ud55c \uc678\ub85c\uc6b4 \uc0ac\ub78c\uc5d0 \ub300\ud55c \uc774\uc57c\uae30' \ub610\ub294 '\ubaa8\ub4e0 \uc678\ub85c\uc6b4 \uc0ac\ub78c\ub4e4\uc758 \uc774\uc57c\uae30'\ub77c\uace0 \ubc1d\ud614\ub2e4. \uc6d0\uace1\uc758 \uac00\uc774\ub4dc \ubcf4\uceec\ub85c \u3008\ubc11\ubc14\ub2e5\uc5d0\uc11c\u3009\uc5d0 \ucc38\uc5ec\ud55c \ubc94\ud0a4\uac00 \ub179\uc74c\ud588\uc5c8\ub2e4. \u3008\ubc00\ubb3c\u3009\uc740 \ud2b9\ubcc4\ud788 \ud559\uc0dd\uc774\ub098 \uc131\uc778\uc774 \ub418\uc5b4\uac00\ub294 \uc0ac\ub78c\uc774 \uacf5\uac10\ud560 \ub9cc\ud55c \uace1\uc73c\ub85c, \ud0c0\ube14\ub85c\uac00 \uc5c9\ub6b1\ud558\uac8c\ub3c4 \"\uc6b0\uc5f0\ud788 \ubc25\uc744 \uba39\ub2e4\uac00 \uba39\ub2e4 \ub0a8\uc740 \uc0dd\uc120\uad6c\uc774\ub97c \ubcf4\uace0 \uc0dd\uac01 \ub098 \uc790\uc2e0\uacfc \ub2ee\uc558\ub2e4\uace0 \uc0dd\uac01\"\ud574 \uadf8 \ub0a0 \uc791\uc0ac\ud558\uc5ec \ub9cc\ub4e4\uc5b4\uc9c0\uac8c \ub418\uc5c8\ub2e4. \u3008\ubc11\ubc14\ub2e5\uc5d0\uc11c\u3009\ub294 \ud0c0\ube14\ub85c\uac00 \uac00\uc7a5 \uc4f0\uae30 \uc5b4\ub824\uc6e0\ub358 \uace1\uc774\uc790 \ub179\uc74c\ud558\uae30 \uc5b4\ub824\uc6e0\ub358 \uace1\uc774\ub2e4. 1\uc808\uc5d0\uc11c\ub294 \uc544\ub0b4\uc5d0\uac8c, 2\uc808\uc5d0\ub294 \uc790\uc2e0\uc758 \ub538\uc5d0\uac8c \ud558\ub294 \uc774\uc57c\uae30\uc774\ub2e4. \ub9c8\uc9c0\ub9c9 \uc544\uae30 \ubaa9\uc18c\ub9ac\ub294 \uc790\uc2e0\uc758 \ub538\uc774 \ucc98\uc74c \u2018\uc544\ube60\u2019\ub77c\uace0 \ud588\uc744 \ub54c\uc758 \ubaa9\uc18c\ub9ac\ub2e4."} {"question": "\ud1a0\ub07c\uc694\ub9ac\uc640 \ub354\ubd88\uc5b4 \uc0dd\uc120\uc694\ub9ac\ub85c\ub3c4 \uc720\uba85\ud55c \uc12c\uc740?", "paragraph": "\ub610 \ub2e4\ub978 \uce84\ud30c\ub2c8\uc544 \uc74c\uc2dd\uc740 \ub9c8\uc694\ub124\uc988\uc640 \uac10\uc790\ub97c \ub123\uace0 \uc0c8\uc6b0\ub97c \uace0\uba85\uc73c\ub85c \uc5b9\uace0 \uc57c\ucc44\uc5d0 \uc2dd\ucd08\ub97c \ub07c\uc5b9\uc740 \uc18c\uc704 \ub7ec\uc2dc\uc544\uc2dd \uc0d0\ub7ec\ub4dc(\ud504\ub791\uc2a4\uc5d0\uc11c \ube44\uc2b7\ud55c \uc694\ub9ac\ub97c \uae30\ucd08\ub85c\ud55c\ub2e4)\ub77c\uace0 \ubd88\ub9b0\ub2e4. \ub7ec\uc2dc\uc544\uc5d0\uc11c\ub294 \uc774 \uac19\uc740 \uc74c\uc2dd\uc744 \uc62c\ub9ac\ube44\uc5d0 \uc0d0\ub7ec\ub4dc\ub77c\uace0 \ubd80\ub974\uace0, \ub3c5\uc77c\uc5d0\uc11c\ub294 \uc774\ud0c8\ub9ac\uc544\uc2dd \uc0d0\ub7ec\ub4dc\ub77c\uace0 \ud55c\ub2e4. \ud504\ub791\uc2a4\uc5d0\uc11c \uc804\ub798\ub41c \ub2e4\ub978 \uc694\ub9ac\ub294 \uc0b6\uc740 \uac10\uc790\ub97c \uc624\ube10\uc5d0\uc11c \ucf00\uc774\ud06c \ud615\uc2dd\uc73c\ub85c \uad6c\uc6b4 \"\uac00\ud1a0(gatt\u00f2)\" \ub610\ub294 \"\uac00\ud14c\uc544\uc6b0 \ub514 \ud30c\ud0c0\ud14c(gateau di patate)\"\ub77c\uace0 \ud558\ub294 \uc74c\uc2dd\uc774\ub2e4. \ub7ec\uc2dc\uc544\uc2dd \uc0d0\ub7ec\ub4dc \ucc98\ub7fc, \uce84\ud30c\ub2c8\uc544 \uc9c0\uc5ed\uc5d0\ub294 \ud574\uc0b0\ubb3c\uc744 \uae30\ubcf8\uc73c\ub85c \ud558\ub294 \uc694\ub9ac\ub4e4\uc758 \uace0\ud5a5\uc774\uae30\ub3c4 \ud558\uc5ec \"\uc778\uc0b4\ub77c\ud0c0 \ub514 \ub9c8\ub808(insalata di mare / \ud574\uc0b0\ubb3c \uc0d0\ub7ec\ub4dc)\", \"\ucd94\ud30c \ub514 \ud3f4\ud3ec(zuppa di polpo / \ubb38\uc5b4 \uc218\ud504)\"\uc640 \"\ucd94\ud30c \ub514 \ucf54\uccb4(zuppa di cozze / \ud64d\ud569 \uc218\ud504\") \uac19\uc740 \uc74c\uc2dd\ub4e4\uc774 \uc778\uae30\uac00 \uc788\ub2e4. \ub2e4\ub978 \uc9c0\uc5ed\uc758 \ud574\uc0b0\ubb3c \uc74c\uc2dd\uc5d0\ub294 \"\ud504\ub9ac\ud154\ub808 \ub514 \ub9c8\ub808(frittelle di mare / \ud574\ucd08\ub958 \ud280\uae40\"), \uc2dd\uc6a9\ud560 \uc218\uc788\ub294 \ud3ec\uc138\uc774\ub3c4\ub2c8\uc544 \ud574\uc870\ub85c \ub9cc\ub4e0 \"\ud2b8\ub9b4\ub9ac\uc5d0 \uc54c \uce74\ub974\ud1a0\ucd08(triglie al cartoccio / red mullet in the bag\")\uc640 \"\uc54c\ub9ac\uce58 \ub9c8\ub9ac\ub098\ud14c(alici marinate / \uc2e0\uc120\ud55c \uba78\uce58\uc640 \uc62c\ub9ac\ube0c \uae30\ub984\uc744 \uc774\uc6a9\ud55c \uc694\ub9ac\")\uac00 \uc788\ub2e4. \uc774\uc2a4\ud0a4\uc544 \uc12c\uc740 \ud1a0\ub07c\uc694\ub9ac\uc5d0 \uc0dd\uc120\uc694\ub9ac\ub85c\ub3c4 \uc720\uba85\ud558\ub2e4. \uce84\ud30c\ub2c8\uc544\ub294 \uc18c\ub80c\ud1a0 \ub808\ubaac\uc758 \uace0\ud5a5\uc774\uae30\ub3c4 \ud558\ub2e4. \uc9c0\uc5ed\uc5d0\uc11c\ub294 \ud504\ub9ac\uc544\ub9ac\uc5d8\ub9ac(friarielli)\ub77c\uace0\ub3c4 \uc54c\ub824\uc9c4 \ub77c\ud53c\ub2c8(\ube0c\ub85c\ucf5c\ub9ac\ub77c\ube0c)\ub294 \uc9c0\uc5ed \uc74c\uc2dd\uc744 \ud558\ub294\ub370 \uc790\uc8fc \uc0ac\uc6a9\ub41c\ub2e4. \uce84\ud30c\ub2c8\uc544\ub294 \ub9ce\uc740 \uacac\uacfc\ub958\ub97c \uc0dd\uc0b0\ud558\uba70 \ud2b9\ud788 \uc0b4\ub808\ub974\ub178\uc640 \ubca0\ub124\ubca4\ud1a0 \uc9c0\uc5ed\uc5d0\uc11c \ub9ce\uc774 \uc0dd\uc0b0\ud55c\ub2e4.", "answer": "\uc774\uc2a4\ud0a4\uc544 \uc12c", "sentence": "\uc774\uc2a4\ud0a4\uc544 \uc12c \uc740 \ud1a0\ub07c\uc694\ub9ac\uc5d0 \uc0dd\uc120\uc694\ub9ac\ub85c\ub3c4 \uc720\uba85\ud558\ub2e4.", "paragraph_sentence": "\ub610 \ub2e4\ub978 \uce84\ud30c\ub2c8\uc544 \uc74c\uc2dd\uc740 \ub9c8\uc694\ub124\uc988\uc640 \uac10\uc790\ub97c \ub123\uace0 \uc0c8\uc6b0\ub97c \uace0\uba85\uc73c\ub85c \uc5b9\uace0 \uc57c\ucc44\uc5d0 \uc2dd\ucd08\ub97c \ub07c\uc5b9\uc740 \uc18c\uc704 \ub7ec\uc2dc\uc544\uc2dd \uc0d0\ub7ec\ub4dc(\ud504\ub791\uc2a4\uc5d0\uc11c \ube44\uc2b7\ud55c \uc694\ub9ac\ub97c \uae30\ucd08\ub85c\ud55c\ub2e4)\ub77c\uace0 \ubd88\ub9b0\ub2e4. \ub7ec\uc2dc\uc544\uc5d0\uc11c\ub294 \uc774 \uac19\uc740 \uc74c\uc2dd\uc744 \uc62c\ub9ac\ube44\uc5d0 \uc0d0\ub7ec\ub4dc\ub77c\uace0 \ubd80\ub974\uace0, \ub3c5\uc77c\uc5d0\uc11c\ub294 \uc774\ud0c8\ub9ac\uc544\uc2dd \uc0d0\ub7ec\ub4dc\ub77c\uace0 \ud55c\ub2e4. \ud504\ub791\uc2a4\uc5d0\uc11c \uc804\ub798\ub41c \ub2e4\ub978 \uc694\ub9ac\ub294 \uc0b6\uc740 \uac10\uc790\ub97c \uc624\ube10\uc5d0\uc11c \ucf00\uc774\ud06c \ud615\uc2dd\uc73c\ub85c \uad6c\uc6b4 \"\uac00\ud1a0(gatt\u00f2)\" \ub610\ub294 \"\uac00\ud14c\uc544\uc6b0 \ub514 \ud30c\ud0c0\ud14c(gateau di patate)\"\ub77c\uace0 \ud558\ub294 \uc74c\uc2dd\uc774\ub2e4. \ub7ec\uc2dc\uc544\uc2dd \uc0d0\ub7ec\ub4dc \ucc98\ub7fc, \uce84\ud30c\ub2c8\uc544 \uc9c0\uc5ed\uc5d0\ub294 \ud574\uc0b0\ubb3c\uc744 \uae30\ubcf8\uc73c\ub85c \ud558\ub294 \uc694\ub9ac\ub4e4\uc758 \uace0\ud5a5\uc774\uae30\ub3c4 \ud558\uc5ec \"\uc778\uc0b4\ub77c\ud0c0 \ub514 \ub9c8\ub808(insalata di mare / \ud574\uc0b0\ubb3c \uc0d0\ub7ec\ub4dc) \", \"\ucd94\ud30c \ub514 \ud3f4\ud3ec(zuppa di polpo / \ubb38\uc5b4 \uc218\ud504)\"\uc640 \"\ucd94\ud30c \ub514 \ucf54\uccb4(zuppa di cozze / \ud64d\ud569 \uc218\ud504\") \uac19\uc740 \uc74c\uc2dd\ub4e4\uc774 \uc778\uae30\uac00 \uc788\ub2e4. \ub2e4\ub978 \uc9c0\uc5ed\uc758 \ud574\uc0b0\ubb3c \uc74c\uc2dd\uc5d0\ub294 \"\ud504\ub9ac\ud154\ub808 \ub514 \ub9c8\ub808(frittelle di mare / \ud574\ucd08\ub958 \ud280\uae40\"), \uc2dd\uc6a9\ud560 \uc218\uc788\ub294 \ud3ec\uc138\uc774\ub3c4\ub2c8\uc544 \ud574\uc870\ub85c \ub9cc\ub4e0 \"\ud2b8\ub9b4\ub9ac\uc5d0 \uc54c \uce74\ub974\ud1a0\ucd08(triglie al cartoccio / red mullet in the bag\")\uc640 \"\uc54c\ub9ac\uce58 \ub9c8\ub9ac\ub098\ud14c(alici marinate / \uc2e0\uc120\ud55c \uba78\uce58\uc640 \uc62c\ub9ac\ube0c \uae30\ub984\uc744 \uc774\uc6a9\ud55c \uc694\ub9ac\")\uac00 \uc788\ub2e4. \uc774\uc2a4\ud0a4\uc544 \uc12c \uc740 \ud1a0\ub07c\uc694\ub9ac\uc5d0 \uc0dd\uc120\uc694\ub9ac\ub85c\ub3c4 \uc720\uba85\ud558\ub2e4. \uce84\ud30c\ub2c8\uc544\ub294 \uc18c\ub80c\ud1a0 \ub808\ubaac\uc758 \uace0\ud5a5\uc774\uae30\ub3c4 \ud558\ub2e4. \uc9c0\uc5ed\uc5d0\uc11c\ub294 \ud504\ub9ac\uc544\ub9ac\uc5d8\ub9ac(friarielli)\ub77c\uace0\ub3c4 \uc54c\ub824\uc9c4 \ub77c\ud53c\ub2c8(\ube0c\ub85c\ucf5c\ub9ac\ub77c\ube0c)\ub294 \uc9c0\uc5ed \uc74c\uc2dd\uc744 \ud558\ub294\ub370 \uc790\uc8fc \uc0ac\uc6a9\ub41c\ub2e4. \uce84\ud30c\ub2c8\uc544\ub294 \ub9ce\uc740 \uacac\uacfc\ub958\ub97c \uc0dd\uc0b0\ud558\uba70 \ud2b9\ud788 \uc0b4\ub808\ub974\ub178\uc640 \ubca0\ub124\ubca4\ud1a0 \uc9c0\uc5ed\uc5d0\uc11c \ub9ce\uc774 \uc0dd\uc0b0\ud55c\ub2e4.", "paragraph_answer": "\ub610 \ub2e4\ub978 \uce84\ud30c\ub2c8\uc544 \uc74c\uc2dd\uc740 \ub9c8\uc694\ub124\uc988\uc640 \uac10\uc790\ub97c \ub123\uace0 \uc0c8\uc6b0\ub97c \uace0\uba85\uc73c\ub85c \uc5b9\uace0 \uc57c\ucc44\uc5d0 \uc2dd\ucd08\ub97c \ub07c\uc5b9\uc740 \uc18c\uc704 \ub7ec\uc2dc\uc544\uc2dd \uc0d0\ub7ec\ub4dc(\ud504\ub791\uc2a4\uc5d0\uc11c \ube44\uc2b7\ud55c \uc694\ub9ac\ub97c \uae30\ucd08\ub85c\ud55c\ub2e4)\ub77c\uace0 \ubd88\ub9b0\ub2e4. \ub7ec\uc2dc\uc544\uc5d0\uc11c\ub294 \uc774 \uac19\uc740 \uc74c\uc2dd\uc744 \uc62c\ub9ac\ube44\uc5d0 \uc0d0\ub7ec\ub4dc\ub77c\uace0 \ubd80\ub974\uace0, \ub3c5\uc77c\uc5d0\uc11c\ub294 \uc774\ud0c8\ub9ac\uc544\uc2dd \uc0d0\ub7ec\ub4dc\ub77c\uace0 \ud55c\ub2e4. \ud504\ub791\uc2a4\uc5d0\uc11c \uc804\ub798\ub41c \ub2e4\ub978 \uc694\ub9ac\ub294 \uc0b6\uc740 \uac10\uc790\ub97c \uc624\ube10\uc5d0\uc11c \ucf00\uc774\ud06c \ud615\uc2dd\uc73c\ub85c \uad6c\uc6b4 \"\uac00\ud1a0(gatt\u00f2)\" \ub610\ub294 \"\uac00\ud14c\uc544\uc6b0 \ub514 \ud30c\ud0c0\ud14c(gateau di patate)\"\ub77c\uace0 \ud558\ub294 \uc74c\uc2dd\uc774\ub2e4. \ub7ec\uc2dc\uc544\uc2dd \uc0d0\ub7ec\ub4dc \ucc98\ub7fc, \uce84\ud30c\ub2c8\uc544 \uc9c0\uc5ed\uc5d0\ub294 \ud574\uc0b0\ubb3c\uc744 \uae30\ubcf8\uc73c\ub85c \ud558\ub294 \uc694\ub9ac\ub4e4\uc758 \uace0\ud5a5\uc774\uae30\ub3c4 \ud558\uc5ec \"\uc778\uc0b4\ub77c\ud0c0 \ub514 \ub9c8\ub808(insalata di mare / \ud574\uc0b0\ubb3c \uc0d0\ub7ec\ub4dc)\", \"\ucd94\ud30c \ub514 \ud3f4\ud3ec(zuppa di polpo / \ubb38\uc5b4 \uc218\ud504)\"\uc640 \"\ucd94\ud30c \ub514 \ucf54\uccb4(zuppa di cozze / \ud64d\ud569 \uc218\ud504\") \uac19\uc740 \uc74c\uc2dd\ub4e4\uc774 \uc778\uae30\uac00 \uc788\ub2e4. \ub2e4\ub978 \uc9c0\uc5ed\uc758 \ud574\uc0b0\ubb3c \uc74c\uc2dd\uc5d0\ub294 \"\ud504\ub9ac\ud154\ub808 \ub514 \ub9c8\ub808(frittelle di mare / \ud574\ucd08\ub958 \ud280\uae40\"), \uc2dd\uc6a9\ud560 \uc218\uc788\ub294 \ud3ec\uc138\uc774\ub3c4\ub2c8\uc544 \ud574\uc870\ub85c \ub9cc\ub4e0 \"\ud2b8\ub9b4\ub9ac\uc5d0 \uc54c \uce74\ub974\ud1a0\ucd08(triglie al cartoccio / red mullet in the bag\")\uc640 \"\uc54c\ub9ac\uce58 \ub9c8\ub9ac\ub098\ud14c(alici marinate / \uc2e0\uc120\ud55c \uba78\uce58\uc640 \uc62c\ub9ac\ube0c \uae30\ub984\uc744 \uc774\uc6a9\ud55c \uc694\ub9ac\")\uac00 \uc788\ub2e4. \uc774\uc2a4\ud0a4\uc544 \uc12c \uc740 \ud1a0\ub07c\uc694\ub9ac\uc5d0 \uc0dd\uc120\uc694\ub9ac\ub85c\ub3c4 \uc720\uba85\ud558\ub2e4. \uce84\ud30c\ub2c8\uc544\ub294 \uc18c\ub80c\ud1a0 \ub808\ubaac\uc758 \uace0\ud5a5\uc774\uae30\ub3c4 \ud558\ub2e4. \uc9c0\uc5ed\uc5d0\uc11c\ub294 \ud504\ub9ac\uc544\ub9ac\uc5d8\ub9ac(friarielli)\ub77c\uace0\ub3c4 \uc54c\ub824\uc9c4 \ub77c\ud53c\ub2c8(\ube0c\ub85c\ucf5c\ub9ac\ub77c\ube0c)\ub294 \uc9c0\uc5ed \uc74c\uc2dd\uc744 \ud558\ub294\ub370 \uc790\uc8fc \uc0ac\uc6a9\ub41c\ub2e4. \uce84\ud30c\ub2c8\uc544\ub294 \ub9ce\uc740 \uacac\uacfc\ub958\ub97c \uc0dd\uc0b0\ud558\uba70 \ud2b9\ud788 \uc0b4\ub808\ub974\ub178\uc640 \ubca0\ub124\ubca4\ud1a0 \uc9c0\uc5ed\uc5d0\uc11c \ub9ce\uc774 \uc0dd\uc0b0\ud55c\ub2e4.", "sentence_answer": " \uc774\uc2a4\ud0a4\uc544 \uc12c \uc740 \ud1a0\ub07c\uc694\ub9ac\uc5d0 \uc0dd\uc120\uc694\ub9ac\ub85c\ub3c4 \uc720\uba85\ud558\ub2e4.", "paragraph_id": "6561315-9-1", "paragraph_question": "question: \ud1a0\ub07c\uc694\ub9ac\uc640 \ub354\ubd88\uc5b4 \uc0dd\uc120\uc694\ub9ac\ub85c\ub3c4 \uc720\uba85\ud55c \uc12c\uc740?, context: \ub610 \ub2e4\ub978 \uce84\ud30c\ub2c8\uc544 \uc74c\uc2dd\uc740 \ub9c8\uc694\ub124\uc988\uc640 \uac10\uc790\ub97c \ub123\uace0 \uc0c8\uc6b0\ub97c \uace0\uba85\uc73c\ub85c \uc5b9\uace0 \uc57c\ucc44\uc5d0 \uc2dd\ucd08\ub97c \ub07c\uc5b9\uc740 \uc18c\uc704 \ub7ec\uc2dc\uc544\uc2dd \uc0d0\ub7ec\ub4dc(\ud504\ub791\uc2a4\uc5d0\uc11c \ube44\uc2b7\ud55c \uc694\ub9ac\ub97c \uae30\ucd08\ub85c\ud55c\ub2e4)\ub77c\uace0 \ubd88\ub9b0\ub2e4. \ub7ec\uc2dc\uc544\uc5d0\uc11c\ub294 \uc774 \uac19\uc740 \uc74c\uc2dd\uc744 \uc62c\ub9ac\ube44\uc5d0 \uc0d0\ub7ec\ub4dc\ub77c\uace0 \ubd80\ub974\uace0, \ub3c5\uc77c\uc5d0\uc11c\ub294 \uc774\ud0c8\ub9ac\uc544\uc2dd \uc0d0\ub7ec\ub4dc\ub77c\uace0 \ud55c\ub2e4. \ud504\ub791\uc2a4\uc5d0\uc11c \uc804\ub798\ub41c \ub2e4\ub978 \uc694\ub9ac\ub294 \uc0b6\uc740 \uac10\uc790\ub97c \uc624\ube10\uc5d0\uc11c \ucf00\uc774\ud06c \ud615\uc2dd\uc73c\ub85c \uad6c\uc6b4 \"\uac00\ud1a0(gatt\u00f2)\" \ub610\ub294 \"\uac00\ud14c\uc544\uc6b0 \ub514 \ud30c\ud0c0\ud14c(gateau di patate)\"\ub77c\uace0 \ud558\ub294 \uc74c\uc2dd\uc774\ub2e4. \ub7ec\uc2dc\uc544\uc2dd \uc0d0\ub7ec\ub4dc \ucc98\ub7fc, \uce84\ud30c\ub2c8\uc544 \uc9c0\uc5ed\uc5d0\ub294 \ud574\uc0b0\ubb3c\uc744 \uae30\ubcf8\uc73c\ub85c \ud558\ub294 \uc694\ub9ac\ub4e4\uc758 \uace0\ud5a5\uc774\uae30\ub3c4 \ud558\uc5ec \"\uc778\uc0b4\ub77c\ud0c0 \ub514 \ub9c8\ub808(insalata di mare / \ud574\uc0b0\ubb3c \uc0d0\ub7ec\ub4dc)\", \"\ucd94\ud30c \ub514 \ud3f4\ud3ec(zuppa di polpo / \ubb38\uc5b4 \uc218\ud504)\"\uc640 \"\ucd94\ud30c \ub514 \ucf54\uccb4(zuppa di cozze / \ud64d\ud569 \uc218\ud504\") \uac19\uc740 \uc74c\uc2dd\ub4e4\uc774 \uc778\uae30\uac00 \uc788\ub2e4. \ub2e4\ub978 \uc9c0\uc5ed\uc758 \ud574\uc0b0\ubb3c \uc74c\uc2dd\uc5d0\ub294 \"\ud504\ub9ac\ud154\ub808 \ub514 \ub9c8\ub808(frittelle di mare / \ud574\ucd08\ub958 \ud280\uae40\"), \uc2dd\uc6a9\ud560 \uc218\uc788\ub294 \ud3ec\uc138\uc774\ub3c4\ub2c8\uc544 \ud574\uc870\ub85c \ub9cc\ub4e0 \"\ud2b8\ub9b4\ub9ac\uc5d0 \uc54c \uce74\ub974\ud1a0\ucd08(triglie al cartoccio / red mullet in the bag\")\uc640 \"\uc54c\ub9ac\uce58 \ub9c8\ub9ac\ub098\ud14c(alici marinate / \uc2e0\uc120\ud55c \uba78\uce58\uc640 \uc62c\ub9ac\ube0c \uae30\ub984\uc744 \uc774\uc6a9\ud55c \uc694\ub9ac\")\uac00 \uc788\ub2e4. \uc774\uc2a4\ud0a4\uc544 \uc12c\uc740 \ud1a0\ub07c\uc694\ub9ac\uc5d0 \uc0dd\uc120\uc694\ub9ac\ub85c\ub3c4 \uc720\uba85\ud558\ub2e4. \uce84\ud30c\ub2c8\uc544\ub294 \uc18c\ub80c\ud1a0 \ub808\ubaac\uc758 \uace0\ud5a5\uc774\uae30\ub3c4 \ud558\ub2e4. \uc9c0\uc5ed\uc5d0\uc11c\ub294 \ud504\ub9ac\uc544\ub9ac\uc5d8\ub9ac(friarielli)\ub77c\uace0\ub3c4 \uc54c\ub824\uc9c4 \ub77c\ud53c\ub2c8(\ube0c\ub85c\ucf5c\ub9ac\ub77c\ube0c)\ub294 \uc9c0\uc5ed \uc74c\uc2dd\uc744 \ud558\ub294\ub370 \uc790\uc8fc \uc0ac\uc6a9\ub41c\ub2e4. \uce84\ud30c\ub2c8\uc544\ub294 \ub9ce\uc740 \uacac\uacfc\ub958\ub97c \uc0dd\uc0b0\ud558\uba70 \ud2b9\ud788 \uc0b4\ub808\ub974\ub178\uc640 \ubca0\ub124\ubca4\ud1a0 \uc9c0\uc5ed\uc5d0\uc11c \ub9ce\uc774 \uc0dd\uc0b0\ud55c\ub2e4."} {"question": "\uc591\uc6d0\uc81c \uc758\ud68c\ub85c \uc0c1\uc6d0\uacfc \ud558\uc6d0\uc73c\ub85c \uc774\ub8e8\uc5b4\uc9c4\uac83\uc740?", "paragraph": "\uc544\uba54\ub9ac\uce74 \ud569\uc911\uad6d\uc774\ub780 \ub098\ub77c \uc774\ub984\uc774 \ub098\ud0c0\ub0b4\uace0 \uc788\ub294 \ubc14\uc640 \uac19\uc774, \uc774 \ub098\ub77c\ub294 \uc8fc\uad8c\uc744 \uac16\ub294 50\uac1c\uc758 \uc8fc\uc640 \uceec\ub7fc\ube44\uc544 \ud2b9\ubcc4\uad6c \ubc0f \ud574\uc678\uc758 \uc18d\ub839\u00b7\uc900\uc601\ud1a0\u00b7\uc2e0\ud0c1\ud1b5\uce58\ub839\uc73c\ub85c \ud615\uc131\ub418\ub294 \uc5f0\ubc29 \uacf5\ud654\uad6d\uc774\ub2e4. \ubbf8\uad6d \ucd5c\uc0c1\uc704 \ubc95\uc804\uc778 \ud5cc\ubc95\uc5d0 \uaddc\uc815\ub41c \uacac\uc81c\uc640 \uade0\ud615 \uccb4\uc81c\uc5d0 \ub530\ub77c \ucca0\uc800\ud55c \uc0bc\uad8c\ubd84\ub9bd \uccb4\uc81c\ub97c \ucde8\ud558\uba70, \uc5f0\ubc29 \ub2f9\uad6d\uc740 \uc785\ubc95\u00b7\uc0ac\ubc95\u00b7\ud589\uc815\uc758 \uc138 \ubd84\uc57c\uc5d0\uc11c\ub294 \ubaa8\ub4e0 \uc8fc \ub610\ub294 2\uc8fc \uc774\uc0c1\uc5d0 \uac78\uce58\ub294 \uac83\ub9cc\uc744 \uad8c\ud55c\uc758 \ub300\uc0c1\uc73c\ub85c \ud558\uace0, \uc8fc\uc5d0 \uace0\uc720\ud55c \uac83\uc740 \ubaa8\ub450 \ub3c5\uc790\uc801\uc778 \ud5cc\ubc95\uacfc \uc0bc\uad8c\uc744 \uac00\uc9c0\ub294 \uac01\uc8fc\uc5d0 \uc704\uc784\ub41c\ub2e4. \ubbf8\uad6d \uc5f0\ubc29 \uc815\ubd80\ub294 \uc0bc\ubd80\ub85c \uad6c\uc131\ub418\uc5b4 \uc788\ub2e4. \uc785\ubc95\ubd80\ub294 \uc591\uc6d0\uc81c \uc758\ud68c\ub85c, \uc0c1\uc6d0\uacfc \ud558\uc6d0\uc73c\ub85c \uc774\ub8e8\uc5b4\uc838 \uc788\uc73c\uba70, \uc5f0\ubc29\ubc95 \uc81c\uc815, \uc804\uc7c1 \uc120\ud3ec, \uc870\uc57d \uc2b9\uc778, \uc608\uc0b0 \uc758\uacb0, \ud0c4\ud575 \uc18c\ucd94 \ub4f1\uc758 \uae30\ub2a5\uc744 \ub9e1\ub294\ub2e4. \ub300\ud1b5\ub839\uc740 \uad6d\uac00 \uc6d0\uc218\uc774\uc790 \uad70\ub300 \ucd5c\uace0 \ud1b5\uc218\uad8c\uc790, \uc870\uc57d \uccb4\uacb0\uad8c\uc790\uc774\uba70, \ud589\uc815 \uc218\ubc18\uc73c\ub85c\uc11c \uc785\ubc95 \uc81c\uc548, \ub300\uc678 \uc815\ucc45 \uc785\uc548 \ub4f1\uc758 \uad8c\ud55c\uc774 \uc788\ub2e4. \ub610 \ubc95\uc548\uc774 \uacf5\ud3ec\ub418\uae30 \uc804\uc5d0 \uac70\ubd80\uad8c\uc744 \ud589\uc0ac\ud560 \uc218 \uc788\uace0, \uac01\ub8cc(\uc0c1\uc6d0\uc758 \uc2b9\uc778\uc744 \ubc1b\uc544\uc57c \ud568)\ub97c \ube44\ub86f\ud558\uc5ec \uc5f0\ubc29\ubc95\uc744 \uc9d1\ud589\ud560 \ud589\uc815 \uacf5\ubb34\uc6d0\uc744 \uc784\uba85\ud55c\ub2e4. \uc5f0\ubc29 \ub300\ubc95\uc6d0\uacfc \ud558\uc704 \uc5f0\ubc29 \ubc95\uc6d0\uc740 \uc0c1\uc6d0\uc758 \uc2b9\uc778\uc744 \uac70\uccd0 \ub300\ud1b5\ub839\uc774 \ubc95\uad00\uc744 \uc784\uba85\ud558\uba70, \ubc95\uc744 \ud574\uc11d\ud55c\ub2e4. \uc5f0\ubc29 \ub300\ubc95\uc6d0\uc740 \uc7ac\uc2ec\uc758\uad8c\uc744 \uc774\uc6a9\ud558\uc5ec \ud5cc\ubc95\uc5d0 \uc704\ubc30\ub418\ub294 \uc785\ubc95\ubd80\u00b7\uc0ac\ubc95\ubd80\u00b7\ud589\uc815\ubd80\uc758 \ubc95\uc548\uc744 \ubb34\ud6a8\ud654\uc2dc\ud0ac \uc218 \uc788\ub2e4. \ud558\uae09 \ubc95\uc6d0\uc758 \ud310\uacb0\uc744 \ub2e4\ub8e8\ub294 \ucd5c\uc885 \uc0c1\uc18c\uc2ec \ubc95\uc6d0\uc774\uae30\ub3c4 \ud558\ub2e4.", "answer": "\uc785\ubc95\ubd80", "sentence": "\uc785\ubc95\ubd80 \ub294 \uc591\uc6d0\uc81c \uc758\ud68c\ub85c, \uc0c1\uc6d0\uacfc \ud558\uc6d0\uc73c\ub85c \uc774\ub8e8\uc5b4\uc838 \uc788\uc73c\uba70, \uc5f0\ubc29\ubc95 \uc81c\uc815, \uc804\uc7c1 \uc120\ud3ec, \uc870\uc57d \uc2b9\uc778, \uc608\uc0b0 \uc758\uacb0, \ud0c4\ud575 \uc18c\ucd94 \ub4f1\uc758 \uae30\ub2a5\uc744 \ub9e1\ub294\ub2e4.", "paragraph_sentence": "\uc544\uba54\ub9ac\uce74 \ud569\uc911\uad6d\uc774\ub780 \ub098\ub77c \uc774\ub984\uc774 \ub098\ud0c0\ub0b4\uace0 \uc788\ub294 \ubc14\uc640 \uac19\uc774, \uc774 \ub098\ub77c\ub294 \uc8fc\uad8c\uc744 \uac16\ub294 50\uac1c\uc758 \uc8fc\uc640 \uceec\ub7fc\ube44\uc544 \ud2b9\ubcc4\uad6c \ubc0f \ud574\uc678\uc758 \uc18d\ub839\u00b7\uc900\uc601\ud1a0\u00b7\uc2e0\ud0c1\ud1b5\uce58\ub839\uc73c\ub85c \ud615\uc131\ub418\ub294 \uc5f0\ubc29 \uacf5\ud654\uad6d\uc774\ub2e4. \ubbf8\uad6d \ucd5c\uc0c1\uc704 \ubc95\uc804\uc778 \ud5cc\ubc95\uc5d0 \uaddc\uc815\ub41c \uacac\uc81c\uc640 \uade0\ud615 \uccb4\uc81c\uc5d0 \ub530\ub77c \ucca0\uc800\ud55c \uc0bc\uad8c\ubd84\ub9bd \uccb4\uc81c\ub97c \ucde8\ud558\uba70, \uc5f0\ubc29 \ub2f9\uad6d\uc740 \uc785\ubc95\u00b7\uc0ac\ubc95\u00b7\ud589\uc815\uc758 \uc138 \ubd84\uc57c\uc5d0\uc11c\ub294 \ubaa8\ub4e0 \uc8fc \ub610\ub294 2\uc8fc \uc774\uc0c1\uc5d0 \uac78\uce58\ub294 \uac83\ub9cc\uc744 \uad8c\ud55c\uc758 \ub300\uc0c1\uc73c\ub85c \ud558\uace0, \uc8fc\uc5d0 \uace0\uc720\ud55c \uac83\uc740 \ubaa8\ub450 \ub3c5\uc790\uc801\uc778 \ud5cc\ubc95\uacfc \uc0bc\uad8c\uc744 \uac00\uc9c0\ub294 \uac01\uc8fc\uc5d0 \uc704\uc784\ub41c\ub2e4. \ubbf8\uad6d \uc5f0\ubc29 \uc815\ubd80\ub294 \uc0bc\ubd80\ub85c \uad6c\uc131\ub418\uc5b4 \uc788\ub2e4. \uc785\ubc95\ubd80 \ub294 \uc591\uc6d0\uc81c \uc758\ud68c\ub85c, \uc0c1\uc6d0\uacfc \ud558\uc6d0\uc73c\ub85c \uc774\ub8e8\uc5b4\uc838 \uc788\uc73c\uba70, \uc5f0\ubc29\ubc95 \uc81c\uc815, \uc804\uc7c1 \uc120\ud3ec, \uc870\uc57d \uc2b9\uc778, \uc608\uc0b0 \uc758\uacb0, \ud0c4\ud575 \uc18c\ucd94 \ub4f1\uc758 \uae30\ub2a5\uc744 \ub9e1\ub294\ub2e4. \ub300\ud1b5\ub839\uc740 \uad6d\uac00 \uc6d0\uc218\uc774\uc790 \uad70\ub300 \ucd5c\uace0 \ud1b5\uc218\uad8c\uc790, \uc870\uc57d \uccb4\uacb0\uad8c\uc790\uc774\uba70, \ud589\uc815 \uc218\ubc18\uc73c\ub85c\uc11c \uc785\ubc95 \uc81c\uc548, \ub300\uc678 \uc815\ucc45 \uc785\uc548 \ub4f1\uc758 \uad8c\ud55c\uc774 \uc788\ub2e4. \ub610 \ubc95\uc548\uc774 \uacf5\ud3ec\ub418\uae30 \uc804\uc5d0 \uac70\ubd80\uad8c\uc744 \ud589\uc0ac\ud560 \uc218 \uc788\uace0, \uac01\ub8cc(\uc0c1\uc6d0\uc758 \uc2b9\uc778\uc744 \ubc1b\uc544\uc57c \ud568)\ub97c \ube44\ub86f\ud558\uc5ec \uc5f0\ubc29\ubc95\uc744 \uc9d1\ud589\ud560 \ud589\uc815 \uacf5\ubb34\uc6d0\uc744 \uc784\uba85\ud55c\ub2e4. \uc5f0\ubc29 \ub300\ubc95\uc6d0\uacfc \ud558\uc704 \uc5f0\ubc29 \ubc95\uc6d0\uc740 \uc0c1\uc6d0\uc758 \uc2b9\uc778\uc744 \uac70\uccd0 \ub300\ud1b5\ub839\uc774 \ubc95\uad00\uc744 \uc784\uba85\ud558\uba70, \ubc95\uc744 \ud574\uc11d\ud55c\ub2e4. \uc5f0\ubc29 \ub300\ubc95\uc6d0\uc740 \uc7ac\uc2ec\uc758\uad8c\uc744 \uc774\uc6a9\ud558\uc5ec \ud5cc\ubc95\uc5d0 \uc704\ubc30\ub418\ub294 \uc785\ubc95\ubd80\u00b7\uc0ac\ubc95\ubd80\u00b7\ud589\uc815\ubd80\uc758 \ubc95\uc548\uc744 \ubb34\ud6a8\ud654\uc2dc\ud0ac \uc218 \uc788\ub2e4. \ud558\uae09 \ubc95\uc6d0\uc758 \ud310\uacb0\uc744 \ub2e4\ub8e8\ub294 \ucd5c\uc885 \uc0c1\uc18c\uc2ec \ubc95\uc6d0\uc774\uae30\ub3c4 \ud558\ub2e4.", "paragraph_answer": "\uc544\uba54\ub9ac\uce74 \ud569\uc911\uad6d\uc774\ub780 \ub098\ub77c \uc774\ub984\uc774 \ub098\ud0c0\ub0b4\uace0 \uc788\ub294 \ubc14\uc640 \uac19\uc774, \uc774 \ub098\ub77c\ub294 \uc8fc\uad8c\uc744 \uac16\ub294 50\uac1c\uc758 \uc8fc\uc640 \uceec\ub7fc\ube44\uc544 \ud2b9\ubcc4\uad6c \ubc0f \ud574\uc678\uc758 \uc18d\ub839\u00b7\uc900\uc601\ud1a0\u00b7\uc2e0\ud0c1\ud1b5\uce58\ub839\uc73c\ub85c \ud615\uc131\ub418\ub294 \uc5f0\ubc29 \uacf5\ud654\uad6d\uc774\ub2e4. \ubbf8\uad6d \ucd5c\uc0c1\uc704 \ubc95\uc804\uc778 \ud5cc\ubc95\uc5d0 \uaddc\uc815\ub41c \uacac\uc81c\uc640 \uade0\ud615 \uccb4\uc81c\uc5d0 \ub530\ub77c \ucca0\uc800\ud55c \uc0bc\uad8c\ubd84\ub9bd \uccb4\uc81c\ub97c \ucde8\ud558\uba70, \uc5f0\ubc29 \ub2f9\uad6d\uc740 \uc785\ubc95\u00b7\uc0ac\ubc95\u00b7\ud589\uc815\uc758 \uc138 \ubd84\uc57c\uc5d0\uc11c\ub294 \ubaa8\ub4e0 \uc8fc \ub610\ub294 2\uc8fc \uc774\uc0c1\uc5d0 \uac78\uce58\ub294 \uac83\ub9cc\uc744 \uad8c\ud55c\uc758 \ub300\uc0c1\uc73c\ub85c \ud558\uace0, \uc8fc\uc5d0 \uace0\uc720\ud55c \uac83\uc740 \ubaa8\ub450 \ub3c5\uc790\uc801\uc778 \ud5cc\ubc95\uacfc \uc0bc\uad8c\uc744 \uac00\uc9c0\ub294 \uac01\uc8fc\uc5d0 \uc704\uc784\ub41c\ub2e4. \ubbf8\uad6d \uc5f0\ubc29 \uc815\ubd80\ub294 \uc0bc\ubd80\ub85c \uad6c\uc131\ub418\uc5b4 \uc788\ub2e4. \uc785\ubc95\ubd80 \ub294 \uc591\uc6d0\uc81c \uc758\ud68c\ub85c, \uc0c1\uc6d0\uacfc \ud558\uc6d0\uc73c\ub85c \uc774\ub8e8\uc5b4\uc838 \uc788\uc73c\uba70, \uc5f0\ubc29\ubc95 \uc81c\uc815, \uc804\uc7c1 \uc120\ud3ec, \uc870\uc57d \uc2b9\uc778, \uc608\uc0b0 \uc758\uacb0, \ud0c4\ud575 \uc18c\ucd94 \ub4f1\uc758 \uae30\ub2a5\uc744 \ub9e1\ub294\ub2e4. \ub300\ud1b5\ub839\uc740 \uad6d\uac00 \uc6d0\uc218\uc774\uc790 \uad70\ub300 \ucd5c\uace0 \ud1b5\uc218\uad8c\uc790, \uc870\uc57d \uccb4\uacb0\uad8c\uc790\uc774\uba70, \ud589\uc815 \uc218\ubc18\uc73c\ub85c\uc11c \uc785\ubc95 \uc81c\uc548, \ub300\uc678 \uc815\ucc45 \uc785\uc548 \ub4f1\uc758 \uad8c\ud55c\uc774 \uc788\ub2e4. \ub610 \ubc95\uc548\uc774 \uacf5\ud3ec\ub418\uae30 \uc804\uc5d0 \uac70\ubd80\uad8c\uc744 \ud589\uc0ac\ud560 \uc218 \uc788\uace0, \uac01\ub8cc(\uc0c1\uc6d0\uc758 \uc2b9\uc778\uc744 \ubc1b\uc544\uc57c \ud568)\ub97c \ube44\ub86f\ud558\uc5ec \uc5f0\ubc29\ubc95\uc744 \uc9d1\ud589\ud560 \ud589\uc815 \uacf5\ubb34\uc6d0\uc744 \uc784\uba85\ud55c\ub2e4. \uc5f0\ubc29 \ub300\ubc95\uc6d0\uacfc \ud558\uc704 \uc5f0\ubc29 \ubc95\uc6d0\uc740 \uc0c1\uc6d0\uc758 \uc2b9\uc778\uc744 \uac70\uccd0 \ub300\ud1b5\ub839\uc774 \ubc95\uad00\uc744 \uc784\uba85\ud558\uba70, \ubc95\uc744 \ud574\uc11d\ud55c\ub2e4. \uc5f0\ubc29 \ub300\ubc95\uc6d0\uc740 \uc7ac\uc2ec\uc758\uad8c\uc744 \uc774\uc6a9\ud558\uc5ec \ud5cc\ubc95\uc5d0 \uc704\ubc30\ub418\ub294 \uc785\ubc95\ubd80\u00b7\uc0ac\ubc95\ubd80\u00b7\ud589\uc815\ubd80\uc758 \ubc95\uc548\uc744 \ubb34\ud6a8\ud654\uc2dc\ud0ac \uc218 \uc788\ub2e4. \ud558\uae09 \ubc95\uc6d0\uc758 \ud310\uacb0\uc744 \ub2e4\ub8e8\ub294 \ucd5c\uc885 \uc0c1\uc18c\uc2ec \ubc95\uc6d0\uc774\uae30\ub3c4 \ud558\ub2e4.", "sentence_answer": " \uc785\ubc95\ubd80 \ub294 \uc591\uc6d0\uc81c \uc758\ud68c\ub85c, \uc0c1\uc6d0\uacfc \ud558\uc6d0\uc73c\ub85c \uc774\ub8e8\uc5b4\uc838 \uc788\uc73c\uba70, \uc5f0\ubc29\ubc95 \uc81c\uc815, \uc804\uc7c1 \uc120\ud3ec, \uc870\uc57d \uc2b9\uc778, \uc608\uc0b0 \uc758\uacb0, \ud0c4\ud575 \uc18c\ucd94 \ub4f1\uc758 \uae30\ub2a5\uc744 \ub9e1\ub294\ub2e4.", "paragraph_id": "6534652-7-0", "paragraph_question": "question: \uc591\uc6d0\uc81c \uc758\ud68c\ub85c \uc0c1\uc6d0\uacfc \ud558\uc6d0\uc73c\ub85c \uc774\ub8e8\uc5b4\uc9c4\uac83\uc740?, context: \uc544\uba54\ub9ac\uce74 \ud569\uc911\uad6d\uc774\ub780 \ub098\ub77c \uc774\ub984\uc774 \ub098\ud0c0\ub0b4\uace0 \uc788\ub294 \ubc14\uc640 \uac19\uc774, \uc774 \ub098\ub77c\ub294 \uc8fc\uad8c\uc744 \uac16\ub294 50\uac1c\uc758 \uc8fc\uc640 \uceec\ub7fc\ube44\uc544 \ud2b9\ubcc4\uad6c \ubc0f \ud574\uc678\uc758 \uc18d\ub839\u00b7\uc900\uc601\ud1a0\u00b7\uc2e0\ud0c1\ud1b5\uce58\ub839\uc73c\ub85c \ud615\uc131\ub418\ub294 \uc5f0\ubc29 \uacf5\ud654\uad6d\uc774\ub2e4. \ubbf8\uad6d \ucd5c\uc0c1\uc704 \ubc95\uc804\uc778 \ud5cc\ubc95\uc5d0 \uaddc\uc815\ub41c \uacac\uc81c\uc640 \uade0\ud615 \uccb4\uc81c\uc5d0 \ub530\ub77c \ucca0\uc800\ud55c \uc0bc\uad8c\ubd84\ub9bd \uccb4\uc81c\ub97c \ucde8\ud558\uba70, \uc5f0\ubc29 \ub2f9\uad6d\uc740 \uc785\ubc95\u00b7\uc0ac\ubc95\u00b7\ud589\uc815\uc758 \uc138 \ubd84\uc57c\uc5d0\uc11c\ub294 \ubaa8\ub4e0 \uc8fc \ub610\ub294 2\uc8fc \uc774\uc0c1\uc5d0 \uac78\uce58\ub294 \uac83\ub9cc\uc744 \uad8c\ud55c\uc758 \ub300\uc0c1\uc73c\ub85c \ud558\uace0, \uc8fc\uc5d0 \uace0\uc720\ud55c \uac83\uc740 \ubaa8\ub450 \ub3c5\uc790\uc801\uc778 \ud5cc\ubc95\uacfc \uc0bc\uad8c\uc744 \uac00\uc9c0\ub294 \uac01\uc8fc\uc5d0 \uc704\uc784\ub41c\ub2e4. \ubbf8\uad6d \uc5f0\ubc29 \uc815\ubd80\ub294 \uc0bc\ubd80\ub85c \uad6c\uc131\ub418\uc5b4 \uc788\ub2e4. \uc785\ubc95\ubd80\ub294 \uc591\uc6d0\uc81c \uc758\ud68c\ub85c, \uc0c1\uc6d0\uacfc \ud558\uc6d0\uc73c\ub85c \uc774\ub8e8\uc5b4\uc838 \uc788\uc73c\uba70, \uc5f0\ubc29\ubc95 \uc81c\uc815, \uc804\uc7c1 \uc120\ud3ec, \uc870\uc57d \uc2b9\uc778, \uc608\uc0b0 \uc758\uacb0, \ud0c4\ud575 \uc18c\ucd94 \ub4f1\uc758 \uae30\ub2a5\uc744 \ub9e1\ub294\ub2e4. \ub300\ud1b5\ub839\uc740 \uad6d\uac00 \uc6d0\uc218\uc774\uc790 \uad70\ub300 \ucd5c\uace0 \ud1b5\uc218\uad8c\uc790, \uc870\uc57d \uccb4\uacb0\uad8c\uc790\uc774\uba70, \ud589\uc815 \uc218\ubc18\uc73c\ub85c\uc11c \uc785\ubc95 \uc81c\uc548, \ub300\uc678 \uc815\ucc45 \uc785\uc548 \ub4f1\uc758 \uad8c\ud55c\uc774 \uc788\ub2e4. \ub610 \ubc95\uc548\uc774 \uacf5\ud3ec\ub418\uae30 \uc804\uc5d0 \uac70\ubd80\uad8c\uc744 \ud589\uc0ac\ud560 \uc218 \uc788\uace0, \uac01\ub8cc(\uc0c1\uc6d0\uc758 \uc2b9\uc778\uc744 \ubc1b\uc544\uc57c \ud568)\ub97c \ube44\ub86f\ud558\uc5ec \uc5f0\ubc29\ubc95\uc744 \uc9d1\ud589\ud560 \ud589\uc815 \uacf5\ubb34\uc6d0\uc744 \uc784\uba85\ud55c\ub2e4. \uc5f0\ubc29 \ub300\ubc95\uc6d0\uacfc \ud558\uc704 \uc5f0\ubc29 \ubc95\uc6d0\uc740 \uc0c1\uc6d0\uc758 \uc2b9\uc778\uc744 \uac70\uccd0 \ub300\ud1b5\ub839\uc774 \ubc95\uad00\uc744 \uc784\uba85\ud558\uba70, \ubc95\uc744 \ud574\uc11d\ud55c\ub2e4. \uc5f0\ubc29 \ub300\ubc95\uc6d0\uc740 \uc7ac\uc2ec\uc758\uad8c\uc744 \uc774\uc6a9\ud558\uc5ec \ud5cc\ubc95\uc5d0 \uc704\ubc30\ub418\ub294 \uc785\ubc95\ubd80\u00b7\uc0ac\ubc95\ubd80\u00b7\ud589\uc815\ubd80\uc758 \ubc95\uc548\uc744 \ubb34\ud6a8\ud654\uc2dc\ud0ac \uc218 \uc788\ub2e4. \ud558\uae09 \ubc95\uc6d0\uc758 \ud310\uacb0\uc744 \ub2e4\ub8e8\ub294 \ucd5c\uc885 \uc0c1\uc18c\uc2ec \ubc95\uc6d0\uc774\uae30\ub3c4 \ud558\ub2e4."} {"question": "\uadf8\ub9ac\ub9c8\uc758 \uac00\uc7a5 \ubb34\uc11c\uc6b4 \ucc9c\uc801\uc740 \ubb34\uc5c7\uc778\uac00?", "paragraph": "\uadf8\ub9ac\ub9c8(house centipede)\ub294 \uadf8\ub9ac\ub9c8\uacfc(Scutigeridae, \ud45c\uc900\uc5b4: \uadf8\ub9ac\ub9db\uacfc)\uc5d0 \uc18d\ud558\ub294 \uc808\uc9c0\ub3d9\ubb3c\uc758 \ucd1d\uce6d\uc774\ub2e4. \uc9c0\ub124\ub098 \ub178\ub798\uae30\uc640 \uac19\uc740 \ubc8c\ub808\uc640 \uc0dd\uae40\uc0c8\uac00 \uc720\uc0ac\ud558\uba70, \uc800\uc791\ud560 \uc218 \uc788\ub294 \ud131\uc774 \uc788\uc73c\ub098 \ub300\uccb4\ub85c \uc0ac\ub78c\uc744 \ubb3c\uc9c0\ub294 \uc54a\ub294\ub2e4. \uc0dd\uae40\uc0c8\uc640 \uc6c0\uc9c1\uc774\ub294 \ud3fc\uc774 \uc0ac\ub78c\ub4e4\uc758 \ud610\uc624\uac10\uc744 \uc720\ubc1c\ud558\ub098, \uc2e4\uc9c8\uc801\uc778 \ud574\ub97c \ub07c\uce58\uc9c0\ub294 \uc54a\uace0 \uc624\ud788\ub824 \ud574\ucda9\uc744 \uc7a1\uc544\uba39\ub294 \uc775\ucda9\uc774\ub2e4. \uae30\uc628\uc774 \ub0b4\ub824\uac00\uba74 \uc8fc\ud0dd \ub0b4\ubd80\uc5d0\ub3c4 \uce68\uc785\ud558\ub294\ub370 \ucd94\uc6b4 \uc9d1\ubcf4\ub2e4 \ub530\ub73b\ud55c \uc9d1\uc5d0\uc11c \ub9ce\uc774 \ucc3e\uc544\ubcfc \uc218 \uc788\ub2e4 \ud558\uc5ec \ud55c\uad6d\uc5d0\uc11c\ub294 \ub3c8\ubc8c\ub808\ub77c\uace0 \ubd80\ub974\uae30\ub3c4 \ud55c\ub2e4. \ud55c\uad6d\uc5d0 \uc11c\uc2dd\ud558\ub294 \uc885\ub958\ub85c\ub294 \uc9d1\uadf8\ub9ac\ub9c8, \ub531\uc815\uadf8\ub9ac\ub9c8 \ub4f1\uc774 \uc788\ub2e4. \uadf8\ub9ac\ub9c8\ub294 \ub2e4\ub978 \uace4\ucda9\uacfc \uadf8 \ud5c8\ubb3c, \uc54c\uc744 \uc8fc\uc2dd\uc73c\ub85c \ud558\uba70, \uac00\uc815\uc5d0\uc11c \ubc14\ud034\ubc8c\ub808\uc640 \uadf8 \uc54c\uc744 \uba39\uae30\ub3c4 \ud558\ub098 \uc8fc\ub85c \uc8fc\ud0dd \ubc16\uc5d0\uc11c \uc11c\uc2dd\ud558\ub294 \ub3d9\ubb3c\uc774\ub2e4. \ucc9c\uc801\uc73c\ub85c\ub294 \ud3ec\uc720\ub958\ub098 \ub3c4\ub9c8\ubc40, \ubc40, \uc0c8, \ub450\uaebc\ube44, \ud0c0\ub780\ud234\ub77c \ub4f1\uc774 \uc788\uc73c\uba70, \uc774 \uc911 \uac00\uc7a5 \ubb34\uc11c\uc6b4 \ucc9c\uc801\uc740 \uc790\uc2e0\uc744 \ub9c8\ucde8\uc2dc\ucf1c \uc560\ubc8c\ub808\uc758 \uba39\uc774\ub85c \uc0bc\ub294 \ub300\ubaa8\ubc8c\uc774 \uc788\ub2e4.", "answer": "\ub300\ubaa8\ubc8c", "sentence": "\ucc9c\uc801\uc73c\ub85c\ub294 \ud3ec\uc720\ub958\ub098 \ub3c4\ub9c8\ubc40, \ubc40, \uc0c8, \ub450\uaebc\ube44, \ud0c0\ub780\ud234\ub77c \ub4f1\uc774 \uc788\uc73c\uba70, \uc774 \uc911 \uac00\uc7a5 \ubb34\uc11c\uc6b4 \ucc9c\uc801\uc740 \uc790\uc2e0\uc744 \ub9c8\ucde8\uc2dc\ucf1c \uc560\ubc8c\ub808\uc758 \uba39\uc774\ub85c \uc0bc\ub294 \ub300\ubaa8\ubc8c \uc774 \uc788\ub2e4.", "paragraph_sentence": "\uadf8\ub9ac\ub9c8(house centipede)\ub294 \uadf8\ub9ac\ub9c8\uacfc(Scutigeridae, \ud45c\uc900\uc5b4: \uadf8\ub9ac\ub9db\uacfc)\uc5d0 \uc18d\ud558\ub294 \uc808\uc9c0\ub3d9\ubb3c\uc758 \ucd1d\uce6d\uc774\ub2e4. \uc9c0\ub124\ub098 \ub178\ub798\uae30\uc640 \uac19\uc740 \ubc8c\ub808\uc640 \uc0dd\uae40\uc0c8\uac00 \uc720\uc0ac\ud558\uba70, \uc800\uc791\ud560 \uc218 \uc788\ub294 \ud131\uc774 \uc788\uc73c\ub098 \ub300\uccb4\ub85c \uc0ac\ub78c\uc744 \ubb3c\uc9c0\ub294 \uc54a\ub294\ub2e4. \uc0dd\uae40\uc0c8\uc640 \uc6c0\uc9c1\uc774\ub294 \ud3fc\uc774 \uc0ac\ub78c\ub4e4\uc758 \ud610\uc624\uac10\uc744 \uc720\ubc1c\ud558\ub098, \uc2e4\uc9c8\uc801\uc778 \ud574\ub97c \ub07c\uce58\uc9c0\ub294 \uc54a\uace0 \uc624\ud788\ub824 \ud574\ucda9\uc744 \uc7a1\uc544\uba39\ub294 \uc775\ucda9\uc774\ub2e4. \uae30\uc628\uc774 \ub0b4\ub824\uac00\uba74 \uc8fc\ud0dd \ub0b4\ubd80\uc5d0\ub3c4 \uce68\uc785\ud558\ub294\ub370 \ucd94\uc6b4 \uc9d1\ubcf4\ub2e4 \ub530\ub73b\ud55c \uc9d1\uc5d0\uc11c \ub9ce\uc774 \ucc3e\uc544\ubcfc \uc218 \uc788\ub2e4 \ud558\uc5ec \ud55c\uad6d\uc5d0\uc11c\ub294 \ub3c8\ubc8c\ub808\ub77c\uace0 \ubd80\ub974\uae30\ub3c4 \ud55c\ub2e4. \ud55c\uad6d\uc5d0 \uc11c\uc2dd\ud558\ub294 \uc885\ub958\ub85c\ub294 \uc9d1\uadf8\ub9ac\ub9c8, \ub531\uc815\uadf8\ub9ac\ub9c8 \ub4f1\uc774 \uc788\ub2e4. \uadf8\ub9ac\ub9c8\ub294 \ub2e4\ub978 \uace4\ucda9\uacfc \uadf8 \ud5c8\ubb3c, \uc54c\uc744 \uc8fc\uc2dd\uc73c\ub85c \ud558\uba70, \uac00\uc815\uc5d0\uc11c \ubc14\ud034\ubc8c\ub808\uc640 \uadf8 \uc54c\uc744 \uba39\uae30\ub3c4 \ud558\ub098 \uc8fc\ub85c \uc8fc\ud0dd \ubc16\uc5d0\uc11c \uc11c\uc2dd\ud558\ub294 \ub3d9\ubb3c\uc774\ub2e4. \ucc9c\uc801\uc73c\ub85c\ub294 \ud3ec\uc720\ub958\ub098 \ub3c4\ub9c8\ubc40, \ubc40, \uc0c8, \ub450\uaebc\ube44, \ud0c0\ub780\ud234\ub77c \ub4f1\uc774 \uc788\uc73c\uba70, \uc774 \uc911 \uac00\uc7a5 \ubb34\uc11c\uc6b4 \ucc9c\uc801\uc740 \uc790\uc2e0\uc744 \ub9c8\ucde8\uc2dc\ucf1c \uc560\ubc8c\ub808\uc758 \uba39\uc774\ub85c \uc0bc\ub294 \ub300\ubaa8\ubc8c \uc774 \uc788\ub2e4. ", "paragraph_answer": "\uadf8\ub9ac\ub9c8(house centipede)\ub294 \uadf8\ub9ac\ub9c8\uacfc(Scutigeridae, \ud45c\uc900\uc5b4: \uadf8\ub9ac\ub9db\uacfc)\uc5d0 \uc18d\ud558\ub294 \uc808\uc9c0\ub3d9\ubb3c\uc758 \ucd1d\uce6d\uc774\ub2e4. \uc9c0\ub124\ub098 \ub178\ub798\uae30\uc640 \uac19\uc740 \ubc8c\ub808\uc640 \uc0dd\uae40\uc0c8\uac00 \uc720\uc0ac\ud558\uba70, \uc800\uc791\ud560 \uc218 \uc788\ub294 \ud131\uc774 \uc788\uc73c\ub098 \ub300\uccb4\ub85c \uc0ac\ub78c\uc744 \ubb3c\uc9c0\ub294 \uc54a\ub294\ub2e4. \uc0dd\uae40\uc0c8\uc640 \uc6c0\uc9c1\uc774\ub294 \ud3fc\uc774 \uc0ac\ub78c\ub4e4\uc758 \ud610\uc624\uac10\uc744 \uc720\ubc1c\ud558\ub098, \uc2e4\uc9c8\uc801\uc778 \ud574\ub97c \ub07c\uce58\uc9c0\ub294 \uc54a\uace0 \uc624\ud788\ub824 \ud574\ucda9\uc744 \uc7a1\uc544\uba39\ub294 \uc775\ucda9\uc774\ub2e4. \uae30\uc628\uc774 \ub0b4\ub824\uac00\uba74 \uc8fc\ud0dd \ub0b4\ubd80\uc5d0\ub3c4 \uce68\uc785\ud558\ub294\ub370 \ucd94\uc6b4 \uc9d1\ubcf4\ub2e4 \ub530\ub73b\ud55c \uc9d1\uc5d0\uc11c \ub9ce\uc774 \ucc3e\uc544\ubcfc \uc218 \uc788\ub2e4 \ud558\uc5ec \ud55c\uad6d\uc5d0\uc11c\ub294 \ub3c8\ubc8c\ub808\ub77c\uace0 \ubd80\ub974\uae30\ub3c4 \ud55c\ub2e4. \ud55c\uad6d\uc5d0 \uc11c\uc2dd\ud558\ub294 \uc885\ub958\ub85c\ub294 \uc9d1\uadf8\ub9ac\ub9c8, \ub531\uc815\uadf8\ub9ac\ub9c8 \ub4f1\uc774 \uc788\ub2e4. \uadf8\ub9ac\ub9c8\ub294 \ub2e4\ub978 \uace4\ucda9\uacfc \uadf8 \ud5c8\ubb3c, \uc54c\uc744 \uc8fc\uc2dd\uc73c\ub85c \ud558\uba70, \uac00\uc815\uc5d0\uc11c \ubc14\ud034\ubc8c\ub808\uc640 \uadf8 \uc54c\uc744 \uba39\uae30\ub3c4 \ud558\ub098 \uc8fc\ub85c \uc8fc\ud0dd \ubc16\uc5d0\uc11c \uc11c\uc2dd\ud558\ub294 \ub3d9\ubb3c\uc774\ub2e4. \ucc9c\uc801\uc73c\ub85c\ub294 \ud3ec\uc720\ub958\ub098 \ub3c4\ub9c8\ubc40, \ubc40, \uc0c8, \ub450\uaebc\ube44, \ud0c0\ub780\ud234\ub77c \ub4f1\uc774 \uc788\uc73c\uba70, \uc774 \uc911 \uac00\uc7a5 \ubb34\uc11c\uc6b4 \ucc9c\uc801\uc740 \uc790\uc2e0\uc744 \ub9c8\ucde8\uc2dc\ucf1c \uc560\ubc8c\ub808\uc758 \uba39\uc774\ub85c \uc0bc\ub294 \ub300\ubaa8\ubc8c \uc774 \uc788\ub2e4.", "sentence_answer": "\ucc9c\uc801\uc73c\ub85c\ub294 \ud3ec\uc720\ub958\ub098 \ub3c4\ub9c8\ubc40, \ubc40, \uc0c8, \ub450\uaebc\ube44, \ud0c0\ub780\ud234\ub77c \ub4f1\uc774 \uc788\uc73c\uba70, \uc774 \uc911 \uac00\uc7a5 \ubb34\uc11c\uc6b4 \ucc9c\uc801\uc740 \uc790\uc2e0\uc744 \ub9c8\ucde8\uc2dc\ucf1c \uc560\ubc8c\ub808\uc758 \uba39\uc774\ub85c \uc0bc\ub294 \ub300\ubaa8\ubc8c \uc774 \uc788\ub2e4.", "paragraph_id": "6477946-0-2", "paragraph_question": "question: \uadf8\ub9ac\ub9c8\uc758 \uac00\uc7a5 \ubb34\uc11c\uc6b4 \ucc9c\uc801\uc740 \ubb34\uc5c7\uc778\uac00?, context: \uadf8\ub9ac\ub9c8(house centipede)\ub294 \uadf8\ub9ac\ub9c8\uacfc(Scutigeridae, \ud45c\uc900\uc5b4: \uadf8\ub9ac\ub9db\uacfc)\uc5d0 \uc18d\ud558\ub294 \uc808\uc9c0\ub3d9\ubb3c\uc758 \ucd1d\uce6d\uc774\ub2e4. \uc9c0\ub124\ub098 \ub178\ub798\uae30\uc640 \uac19\uc740 \ubc8c\ub808\uc640 \uc0dd\uae40\uc0c8\uac00 \uc720\uc0ac\ud558\uba70, \uc800\uc791\ud560 \uc218 \uc788\ub294 \ud131\uc774 \uc788\uc73c\ub098 \ub300\uccb4\ub85c \uc0ac\ub78c\uc744 \ubb3c\uc9c0\ub294 \uc54a\ub294\ub2e4. \uc0dd\uae40\uc0c8\uc640 \uc6c0\uc9c1\uc774\ub294 \ud3fc\uc774 \uc0ac\ub78c\ub4e4\uc758 \ud610\uc624\uac10\uc744 \uc720\ubc1c\ud558\ub098, \uc2e4\uc9c8\uc801\uc778 \ud574\ub97c \ub07c\uce58\uc9c0\ub294 \uc54a\uace0 \uc624\ud788\ub824 \ud574\ucda9\uc744 \uc7a1\uc544\uba39\ub294 \uc775\ucda9\uc774\ub2e4. \uae30\uc628\uc774 \ub0b4\ub824\uac00\uba74 \uc8fc\ud0dd \ub0b4\ubd80\uc5d0\ub3c4 \uce68\uc785\ud558\ub294\ub370 \ucd94\uc6b4 \uc9d1\ubcf4\ub2e4 \ub530\ub73b\ud55c \uc9d1\uc5d0\uc11c \ub9ce\uc774 \ucc3e\uc544\ubcfc \uc218 \uc788\ub2e4 \ud558\uc5ec \ud55c\uad6d\uc5d0\uc11c\ub294 \ub3c8\ubc8c\ub808\ub77c\uace0 \ubd80\ub974\uae30\ub3c4 \ud55c\ub2e4. \ud55c\uad6d\uc5d0 \uc11c\uc2dd\ud558\ub294 \uc885\ub958\ub85c\ub294 \uc9d1\uadf8\ub9ac\ub9c8, \ub531\uc815\uadf8\ub9ac\ub9c8 \ub4f1\uc774 \uc788\ub2e4. \uadf8\ub9ac\ub9c8\ub294 \ub2e4\ub978 \uace4\ucda9\uacfc \uadf8 \ud5c8\ubb3c, \uc54c\uc744 \uc8fc\uc2dd\uc73c\ub85c \ud558\uba70, \uac00\uc815\uc5d0\uc11c \ubc14\ud034\ubc8c\ub808\uc640 \uadf8 \uc54c\uc744 \uba39\uae30\ub3c4 \ud558\ub098 \uc8fc\ub85c \uc8fc\ud0dd \ubc16\uc5d0\uc11c \uc11c\uc2dd\ud558\ub294 \ub3d9\ubb3c\uc774\ub2e4. \ucc9c\uc801\uc73c\ub85c\ub294 \ud3ec\uc720\ub958\ub098 \ub3c4\ub9c8\ubc40, \ubc40, \uc0c8, \ub450\uaebc\ube44, \ud0c0\ub780\ud234\ub77c \ub4f1\uc774 \uc788\uc73c\uba70, \uc774 \uc911 \uac00\uc7a5 \ubb34\uc11c\uc6b4 \ucc9c\uc801\uc740 \uc790\uc2e0\uc744 \ub9c8\ucde8\uc2dc\ucf1c \uc560\ubc8c\ub808\uc758 \uba39\uc774\ub85c \uc0bc\ub294 \ub300\ubaa8\ubc8c\uc774 \uc788\ub2e4."} {"question": "\uc624\uc2dc\ud0c0\uc758 \uc5b4\uba38\ub2c8\ub294 \ubb34\uc5c7\uc5d0 \uc911\ub3c5 \ub418\uc5b4 \uc788\uc5c8\ub294\uac00?", "paragraph": "\uc624\uc2dc\ud0c0\uc758 \uc5b4\uba38\ub2c8\ub294 \ud544\ub85c\ud3f0 \uc911\ub3c5\uc744 \uc553\uace0 \uc788\uc5c8\uc73c\uba70 \uad6c\ub2e8\uc5d0\uc11c \uac00\ubd88\ud55c \uae08\uc561\uc774 \uc0c1\ub2f9 \ubd80\ubd84 \uc5b4\uba38\ub2c8\uc758 \uc57d\uac12\uc73c\ub85c \uc4f0\uc778 \uac83\uc73c\ub85c \uc54c\ub824\uc838 \uc788\ub2e4. \uad6c\ub2e8\uacfc \uae08\uc804 \uba74\uc5d0\uc11c \ubd88\ud654\uac00 \uc0dd\uaca8 1952\ub144\uc5d0\ub294 \uc8fc\ubcc0\uc744 \ub9d0\ub824\ub4e4\uac8c \ud55c \ud1f4\ub2e8 \uc18c\ub3d9(\uc774\ub978\ubc14 '\uc624\uc2dc\ud0c0 \uc18c\ub3d9')\uc73c\ub85c \uc2dc\uc98c \uc2dc\uc791 \uc774\ud6c4 \ub2c8\uc2dc\ud14c\uc4f0 \ub77c\uc774\uc628\uc2a4\ub85c \ud2b8\ub808\uc774\ub4dc \uc774\uc801\ud588\ub2e4. \uc774\uc801 \uc774\ud6c4 \uace0\ub77c\ucfe0\uc5d4 \uad6c\uc7a5\uc758 \ub3c4\ud050 \ud32c\ub4e4\ub85c\ubd80\ud130 \uc870\ub871\uc744 \ubc1b\uc740 \uc624\uc2dc\ud0c0\ub294 \ucd9c\ub8e8\ud55c 1\ub8e8\uc5d0\uc11c \uad00\uc911\uc11d\uc744 \ud5a5\ud574 \uace0\uac1c\ub97c \uafb8\ubc85 \uc219\uc600\uace0 \uc774\uc5d0 \ub3c4\ud050 \ud32c\ub4e4\uc740 \uce68\ubb35\ud560 \uc218 \ubc16\uc5d0 \uc5c6\uc5c8\ub2e4. \ub610\ud55c \ud5e4\uc774\uc640\ub2e4\uc774 \uc0ac\uac74 \ub2f9\uc2dc \ud3ed\ud589\uc744 \ub2f9\ud574 \ud53c\ud22c\uc131\uc774\uac00 \ub418\uc5c8\uc74c\uc5d0\ub3c4 \uad00\uc911\ub4e4\uc744 \uc81c\uc9c0\ud558\ub824 \ud588\ub358 \uc810\uc744 \uc778\uc815\ubc1b\uc544 \ub178\uad6c\uce58 \ub9c8\uc0ac\uc544\ud0a4\uc640 \ud568\uaed8 \uc5f0\ub9f9 \ud45c\ucc3d\uc744 \uc218\uc0c1\ud588\ub2e4. \ub367\ubd99\uc5ec \uc624\uc2dc\ud0c0\uc758 \ud2b8\ub808\uc774\ub4dc \uc0c1\ub300\uc600\ub358 \ud6c4\uce74\ubbf8 \uc57c\uc2a4\ud788\ub85c\ub294 25\uac1c\uc758 \ud648\ub7f0\uc744 \uae30\ub85d\ud558\uba70 \ud648\ub7f0\uc655\uc744 \ucc28\uc9c0\ud574 \ud648\ub7f0\uc655\uc774 \ud504\ub85c \uc57c\uad6c \uc0ac\uc0c1 \uc720\uc77c\ud558\uac8c \ub450 \ud300\uc5d0 \uc7ac\uc801\ud55c \ud648\ub7f0\uc655\uc774 \ub418\uc5c8\ub2e4.", "answer": "\ud544\ub85c\ud3f0", "sentence": "\uc624\uc2dc\ud0c0\uc758 \uc5b4\uba38\ub2c8\ub294 \ud544\ub85c\ud3f0 \uc911\ub3c5\uc744 \uc553\uace0 \uc788\uc5c8\uc73c\uba70 \uad6c\ub2e8\uc5d0\uc11c \uac00\ubd88\ud55c \uae08\uc561\uc774 \uc0c1\ub2f9 \ubd80\ubd84 \uc5b4\uba38\ub2c8\uc758 \uc57d\uac12\uc73c\ub85c \uc4f0\uc778 \uac83\uc73c\ub85c \uc54c\ub824\uc838 \uc788\ub2e4.", "paragraph_sentence": " \uc624\uc2dc\ud0c0\uc758 \uc5b4\uba38\ub2c8\ub294 \ud544\ub85c\ud3f0 \uc911\ub3c5\uc744 \uc553\uace0 \uc788\uc5c8\uc73c\uba70 \uad6c\ub2e8\uc5d0\uc11c \uac00\ubd88\ud55c \uae08\uc561\uc774 \uc0c1\ub2f9 \ubd80\ubd84 \uc5b4\uba38\ub2c8\uc758 \uc57d\uac12\uc73c\ub85c \uc4f0\uc778 \uac83\uc73c\ub85c \uc54c\ub824\uc838 \uc788\ub2e4. \uad6c\ub2e8\uacfc \uae08\uc804 \uba74\uc5d0\uc11c \ubd88\ud654\uac00 \uc0dd\uaca8 1952\ub144\uc5d0\ub294 \uc8fc\ubcc0\uc744 \ub9d0\ub824\ub4e4\uac8c \ud55c \ud1f4\ub2e8 \uc18c\ub3d9(\uc774\ub978\ubc14 '\uc624\uc2dc\ud0c0 \uc18c\ub3d9')\uc73c\ub85c \uc2dc\uc98c \uc2dc\uc791 \uc774\ud6c4 \ub2c8\uc2dc\ud14c\uc4f0 \ub77c\uc774\uc628\uc2a4\ub85c \ud2b8\ub808\uc774\ub4dc \uc774\uc801\ud588\ub2e4. \uc774\uc801 \uc774\ud6c4 \uace0\ub77c\ucfe0\uc5d4 \uad6c\uc7a5\uc758 \ub3c4\ud050 \ud32c\ub4e4\ub85c\ubd80\ud130 \uc870\ub871\uc744 \ubc1b\uc740 \uc624\uc2dc\ud0c0\ub294 \ucd9c\ub8e8\ud55c 1\ub8e8\uc5d0\uc11c \uad00\uc911\uc11d\uc744 \ud5a5\ud574 \uace0\uac1c\ub97c \uafb8\ubc85 \uc219\uc600\uace0 \uc774\uc5d0 \ub3c4\ud050 \ud32c\ub4e4\uc740 \uce68\ubb35\ud560 \uc218 \ubc16\uc5d0 \uc5c6\uc5c8\ub2e4. \ub610\ud55c \ud5e4\uc774\uc640\ub2e4\uc774 \uc0ac\uac74 \ub2f9\uc2dc \ud3ed\ud589\uc744 \ub2f9\ud574 \ud53c\ud22c\uc131\uc774\uac00 \ub418\uc5c8\uc74c\uc5d0\ub3c4 \uad00\uc911\ub4e4\uc744 \uc81c\uc9c0\ud558\ub824 \ud588\ub358 \uc810\uc744 \uc778\uc815\ubc1b\uc544 \ub178\uad6c\uce58 \ub9c8\uc0ac\uc544\ud0a4\uc640 \ud568\uaed8 \uc5f0\ub9f9 \ud45c\ucc3d\uc744 \uc218\uc0c1\ud588\ub2e4. \ub367\ubd99\uc5ec \uc624\uc2dc\ud0c0\uc758 \ud2b8\ub808\uc774\ub4dc \uc0c1\ub300\uc600\ub358 \ud6c4\uce74\ubbf8 \uc57c\uc2a4\ud788\ub85c\ub294 25\uac1c\uc758 \ud648\ub7f0\uc744 \uae30\ub85d\ud558\uba70 \ud648\ub7f0\uc655\uc744 \ucc28\uc9c0\ud574 \ud648\ub7f0\uc655\uc774 \ud504\ub85c \uc57c\uad6c \uc0ac\uc0c1 \uc720\uc77c\ud558\uac8c \ub450 \ud300\uc5d0 \uc7ac\uc801\ud55c \ud648\ub7f0\uc655\uc774 \ub418\uc5c8\ub2e4.", "paragraph_answer": "\uc624\uc2dc\ud0c0\uc758 \uc5b4\uba38\ub2c8\ub294 \ud544\ub85c\ud3f0 \uc911\ub3c5\uc744 \uc553\uace0 \uc788\uc5c8\uc73c\uba70 \uad6c\ub2e8\uc5d0\uc11c \uac00\ubd88\ud55c \uae08\uc561\uc774 \uc0c1\ub2f9 \ubd80\ubd84 \uc5b4\uba38\ub2c8\uc758 \uc57d\uac12\uc73c\ub85c \uc4f0\uc778 \uac83\uc73c\ub85c \uc54c\ub824\uc838 \uc788\ub2e4. \uad6c\ub2e8\uacfc \uae08\uc804 \uba74\uc5d0\uc11c \ubd88\ud654\uac00 \uc0dd\uaca8 1952\ub144\uc5d0\ub294 \uc8fc\ubcc0\uc744 \ub9d0\ub824\ub4e4\uac8c \ud55c \ud1f4\ub2e8 \uc18c\ub3d9(\uc774\ub978\ubc14 '\uc624\uc2dc\ud0c0 \uc18c\ub3d9')\uc73c\ub85c \uc2dc\uc98c \uc2dc\uc791 \uc774\ud6c4 \ub2c8\uc2dc\ud14c\uc4f0 \ub77c\uc774\uc628\uc2a4\ub85c \ud2b8\ub808\uc774\ub4dc \uc774\uc801\ud588\ub2e4. \uc774\uc801 \uc774\ud6c4 \uace0\ub77c\ucfe0\uc5d4 \uad6c\uc7a5\uc758 \ub3c4\ud050 \ud32c\ub4e4\ub85c\ubd80\ud130 \uc870\ub871\uc744 \ubc1b\uc740 \uc624\uc2dc\ud0c0\ub294 \ucd9c\ub8e8\ud55c 1\ub8e8\uc5d0\uc11c \uad00\uc911\uc11d\uc744 \ud5a5\ud574 \uace0\uac1c\ub97c \uafb8\ubc85 \uc219\uc600\uace0 \uc774\uc5d0 \ub3c4\ud050 \ud32c\ub4e4\uc740 \uce68\ubb35\ud560 \uc218 \ubc16\uc5d0 \uc5c6\uc5c8\ub2e4. \ub610\ud55c \ud5e4\uc774\uc640\ub2e4\uc774 \uc0ac\uac74 \ub2f9\uc2dc \ud3ed\ud589\uc744 \ub2f9\ud574 \ud53c\ud22c\uc131\uc774\uac00 \ub418\uc5c8\uc74c\uc5d0\ub3c4 \uad00\uc911\ub4e4\uc744 \uc81c\uc9c0\ud558\ub824 \ud588\ub358 \uc810\uc744 \uc778\uc815\ubc1b\uc544 \ub178\uad6c\uce58 \ub9c8\uc0ac\uc544\ud0a4\uc640 \ud568\uaed8 \uc5f0\ub9f9 \ud45c\ucc3d\uc744 \uc218\uc0c1\ud588\ub2e4. \ub367\ubd99\uc5ec \uc624\uc2dc\ud0c0\uc758 \ud2b8\ub808\uc774\ub4dc \uc0c1\ub300\uc600\ub358 \ud6c4\uce74\ubbf8 \uc57c\uc2a4\ud788\ub85c\ub294 25\uac1c\uc758 \ud648\ub7f0\uc744 \uae30\ub85d\ud558\uba70 \ud648\ub7f0\uc655\uc744 \ucc28\uc9c0\ud574 \ud648\ub7f0\uc655\uc774 \ud504\ub85c \uc57c\uad6c \uc0ac\uc0c1 \uc720\uc77c\ud558\uac8c \ub450 \ud300\uc5d0 \uc7ac\uc801\ud55c \ud648\ub7f0\uc655\uc774 \ub418\uc5c8\ub2e4.", "sentence_answer": "\uc624\uc2dc\ud0c0\uc758 \uc5b4\uba38\ub2c8\ub294 \ud544\ub85c\ud3f0 \uc911\ub3c5\uc744 \uc553\uace0 \uc788\uc5c8\uc73c\uba70 \uad6c\ub2e8\uc5d0\uc11c \uac00\ubd88\ud55c \uae08\uc561\uc774 \uc0c1\ub2f9 \ubd80\ubd84 \uc5b4\uba38\ub2c8\uc758 \uc57d\uac12\uc73c\ub85c \uc4f0\uc778 \uac83\uc73c\ub85c \uc54c\ub824\uc838 \uc788\ub2e4.", "paragraph_id": "6605248-1-0", "paragraph_question": "question: \uc624\uc2dc\ud0c0\uc758 \uc5b4\uba38\ub2c8\ub294 \ubb34\uc5c7\uc5d0 \uc911\ub3c5 \ub418\uc5b4 \uc788\uc5c8\ub294\uac00?, context: \uc624\uc2dc\ud0c0\uc758 \uc5b4\uba38\ub2c8\ub294 \ud544\ub85c\ud3f0 \uc911\ub3c5\uc744 \uc553\uace0 \uc788\uc5c8\uc73c\uba70 \uad6c\ub2e8\uc5d0\uc11c \uac00\ubd88\ud55c \uae08\uc561\uc774 \uc0c1\ub2f9 \ubd80\ubd84 \uc5b4\uba38\ub2c8\uc758 \uc57d\uac12\uc73c\ub85c \uc4f0\uc778 \uac83\uc73c\ub85c \uc54c\ub824\uc838 \uc788\ub2e4. \uad6c\ub2e8\uacfc \uae08\uc804 \uba74\uc5d0\uc11c \ubd88\ud654\uac00 \uc0dd\uaca8 1952\ub144\uc5d0\ub294 \uc8fc\ubcc0\uc744 \ub9d0\ub824\ub4e4\uac8c \ud55c \ud1f4\ub2e8 \uc18c\ub3d9(\uc774\ub978\ubc14 '\uc624\uc2dc\ud0c0 \uc18c\ub3d9')\uc73c\ub85c \uc2dc\uc98c \uc2dc\uc791 \uc774\ud6c4 \ub2c8\uc2dc\ud14c\uc4f0 \ub77c\uc774\uc628\uc2a4\ub85c \ud2b8\ub808\uc774\ub4dc \uc774\uc801\ud588\ub2e4. \uc774\uc801 \uc774\ud6c4 \uace0\ub77c\ucfe0\uc5d4 \uad6c\uc7a5\uc758 \ub3c4\ud050 \ud32c\ub4e4\ub85c\ubd80\ud130 \uc870\ub871\uc744 \ubc1b\uc740 \uc624\uc2dc\ud0c0\ub294 \ucd9c\ub8e8\ud55c 1\ub8e8\uc5d0\uc11c \uad00\uc911\uc11d\uc744 \ud5a5\ud574 \uace0\uac1c\ub97c \uafb8\ubc85 \uc219\uc600\uace0 \uc774\uc5d0 \ub3c4\ud050 \ud32c\ub4e4\uc740 \uce68\ubb35\ud560 \uc218 \ubc16\uc5d0 \uc5c6\uc5c8\ub2e4. \ub610\ud55c \ud5e4\uc774\uc640\ub2e4\uc774 \uc0ac\uac74 \ub2f9\uc2dc \ud3ed\ud589\uc744 \ub2f9\ud574 \ud53c\ud22c\uc131\uc774\uac00 \ub418\uc5c8\uc74c\uc5d0\ub3c4 \uad00\uc911\ub4e4\uc744 \uc81c\uc9c0\ud558\ub824 \ud588\ub358 \uc810\uc744 \uc778\uc815\ubc1b\uc544 \ub178\uad6c\uce58 \ub9c8\uc0ac\uc544\ud0a4\uc640 \ud568\uaed8 \uc5f0\ub9f9 \ud45c\ucc3d\uc744 \uc218\uc0c1\ud588\ub2e4. \ub367\ubd99\uc5ec \uc624\uc2dc\ud0c0\uc758 \ud2b8\ub808\uc774\ub4dc \uc0c1\ub300\uc600\ub358 \ud6c4\uce74\ubbf8 \uc57c\uc2a4\ud788\ub85c\ub294 25\uac1c\uc758 \ud648\ub7f0\uc744 \uae30\ub85d\ud558\uba70 \ud648\ub7f0\uc655\uc744 \ucc28\uc9c0\ud574 \ud648\ub7f0\uc655\uc774 \ud504\ub85c \uc57c\uad6c \uc0ac\uc0c1 \uc720\uc77c\ud558\uac8c \ub450 \ud300\uc5d0 \uc7ac\uc801\ud55c \ud648\ub7f0\uc655\uc774 \ub418\uc5c8\ub2e4."} {"question": "\uc54c\ub809\uc0b0\ub370\ub974 7\uc138 \ub54c\uc778 1664\ub144 \uad50\ud669\uc774 \ud504\ub791\uc2a4\uc5d0\uac8c \ub9fa\uc5b4\uc57c\ud588\ub358 \uad74\uc695\uc801\uc778 \uc870\uc57d\uc740?", "paragraph": "\uc54c\ub809\uc0b0\ub370\ub974 7\uc138\ub294 \ud504\ub791\uc2a4 \uc655 \ub8e8\uc774 14\uc138\uc758 \uc870\uc5b8\uc790\uc778 \uc958 \ub9c8\uc790\ub7ad \ucd94\uae30\uacbd\uacfc\uc758 \uc9c0\uc18d\uc801\uc73c\ub85c \uc54c\ub825\uc744 \ubc8c\uc600\ub2e4. \ub9c8\uc790\ub7ad\uc740 \ubca0\uc2a4\ud2b8\ud314\ub80c \uc870\uc57d\uc744 \uccb4\uacb0\ud558\ub294 \ud611\uc0c1 \ub3d9\uc548 \uad50\ud669\uccad\uc758 \ub73b\uc744 \uac70\uc2a4\ub974\uace0 \uac08\ub9ac\uc544 \uad50\ud68c\uc758 \ud2b9\uad8c\uc744 \uc8fc\uc7a5\ud588\ub2e4. \ucf58\ud074\ub77c\ubca0 \uc911\uc5d0 \uadf8\ub294 \ud0a4\uc9c0\uac00 \uc0c8 \uad50\ud669\uc73c\ub85c \uc120\ucd9c\ub418\ub294 \uac83\uc5d0 \ubc18\ub300\ud588\uc9c0\ub9cc, \uacb0\uad6d \ub9c9\ud310\uc5d0 \uc5b4\uca54 \uc218 \uc5c6\uc774 \uadf8\ub97c \ud0c0\ud611\uc548\uc73c\ub85c \ubc1b\uc544\ub4e4\uc77c \uc218 \ubc16\uc5d0 \uc5c6\uc5c8\ub2e4. \ud558\uc9c0\ub9cc \ub9c8\uc790\ub7ad\uc740 \uc790\uc2e0\uc774 \uc0b4\uc544\uc788\ub294 \ub3d9\uc548 \ub8e8\uc774 14\uc138\uac00 \uc54c\ub809\uc0b0\ub370\ub974 7\uc138\uc5d0 \ub300\ud55c \uc21c\uba85\uc758 \uc758\ubbf8\ub85c \ud1b5\uc0c1\uc801\uc73c\ub85c \ud558\ub294 \ub85c\ub9c8\uc5d0 \uc678\uad50 \ub300\uc0ac\ub97c \ud30c\uacac\ud558\ub294 \uac83\uc744 \uac00\ub85c\ub9c9\uc558\uc73c\uba70, \uadf8\ub3d9\uc548 \uad50\ud669\ub839\uacfc\uc758 \uc678\uad50\uc801 \uc5c5\ubb34\ub294 \uc77c\ubc18\uc801\uc73c\ub85c \uad50\ud669\uacfc \uc0ac\uc774\uac00 \uc88b\uc9c0 \uc54a\uc740 \ucd94\uae30\uacbd\ub4e4\uc774 \uc218\ud589\ud588\ub2e4. 1662\ub144 \uc5ed\uc2dc \uad50\ud669\uacfc \uc801\ub300\uc801\uc778 \uad00\uacc4\uc600\ub358 \ub4c0\ud06c \ub4dc \ud06c\ub974\ud0a4\uac00 \ub300\uc0ac\ub85c \uc784\uba85\ub410\ub2e4. \uadf8\ub294 \uc678\uad50 \ub300\uc0ac\uc5d0\uac8c \uc804\ud1b5\uc801\uc73c\ub85c \uc8fc\uc5b4\uc838 \uc654\ub358 \ub9dd\uba85 \uad8c\ud55c\uc744 \ub0a8\uc6a9\ud558\uc5ec \uad50\ud669\ub839\uacfc \ud504\ub791\uc2a4 \uc0ac\uc774\uc5d0 \uc815\uce58\uc801 \uac08\ub4f1\uc744 \ucd09\ubc1c\uc2dc\ucf30\ub2e4. \uc774\ub294 \uacb0\uad6d \uc54c\ub809\uc0b0\ub370\ub974 7\uc138 \ub54c \uc77c\uc2dc\uc801\uc73c\ub85c \uc544\ube44\ub1fd\uc774 \ud504\ub791\uc2a4\ub85c \uadc0\uc18d\ub418\uace0 1664\ub144 \uad74\uc695\uc801\uc778 \ub0b4\uc6a9\uc758 \ud53c\uc0ac \uc870\uc57d\uc744 \ub9fa\ub294 \uacb0\uacfc\ub97c \ub0b3\uc558\ub2e4.", "answer": "\ud53c\uc0ac \uc870\uc57d", "sentence": "\uc774\ub294 \uacb0\uad6d \uc54c\ub809\uc0b0\ub370\ub974 7\uc138 \ub54c \uc77c\uc2dc\uc801\uc73c\ub85c \uc544\ube44\ub1fd\uc774 \ud504\ub791\uc2a4\ub85c \uadc0\uc18d\ub418\uace0 1664\ub144 \uad74\uc695\uc801\uc778 \ub0b4\uc6a9\uc758 \ud53c\uc0ac \uc870\uc57d \uc744 \ub9fa\ub294 \uacb0\uacfc\ub97c \ub0b3\uc558\ub2e4.", "paragraph_sentence": "\uc54c\ub809\uc0b0\ub370\ub974 7\uc138\ub294 \ud504\ub791\uc2a4 \uc655 \ub8e8\uc774 14\uc138\uc758 \uc870\uc5b8\uc790\uc778 \uc958 \ub9c8\uc790\ub7ad \ucd94\uae30\uacbd\uacfc\uc758 \uc9c0\uc18d\uc801\uc73c\ub85c \uc54c\ub825\uc744 \ubc8c\uc600\ub2e4. \ub9c8\uc790\ub7ad\uc740 \ubca0\uc2a4\ud2b8\ud314\ub80c \uc870\uc57d\uc744 \uccb4\uacb0\ud558\ub294 \ud611\uc0c1 \ub3d9\uc548 \uad50\ud669\uccad\uc758 \ub73b\uc744 \uac70\uc2a4\ub974\uace0 \uac08\ub9ac\uc544 \uad50\ud68c\uc758 \ud2b9\uad8c\uc744 \uc8fc\uc7a5\ud588\ub2e4. \ucf58\ud074\ub77c\ubca0 \uc911\uc5d0 \uadf8\ub294 \ud0a4\uc9c0\uac00 \uc0c8 \uad50\ud669\uc73c\ub85c \uc120\ucd9c\ub418\ub294 \uac83\uc5d0 \ubc18\ub300\ud588\uc9c0\ub9cc, \uacb0\uad6d \ub9c9\ud310\uc5d0 \uc5b4\uca54 \uc218 \uc5c6\uc774 \uadf8\ub97c \ud0c0\ud611\uc548\uc73c\ub85c \ubc1b\uc544\ub4e4\uc77c \uc218 \ubc16\uc5d0 \uc5c6\uc5c8\ub2e4. \ud558\uc9c0\ub9cc \ub9c8\uc790\ub7ad\uc740 \uc790\uc2e0\uc774 \uc0b4\uc544\uc788\ub294 \ub3d9\uc548 \ub8e8\uc774 14\uc138\uac00 \uc54c\ub809\uc0b0\ub370\ub974 7\uc138\uc5d0 \ub300\ud55c \uc21c\uba85\uc758 \uc758\ubbf8\ub85c \ud1b5\uc0c1\uc801\uc73c\ub85c \ud558\ub294 \ub85c\ub9c8\uc5d0 \uc678\uad50 \ub300\uc0ac\ub97c \ud30c\uacac\ud558\ub294 \uac83\uc744 \uac00\ub85c\ub9c9\uc558\uc73c\uba70, \uadf8\ub3d9\uc548 \uad50\ud669\ub839\uacfc\uc758 \uc678\uad50\uc801 \uc5c5\ubb34\ub294 \uc77c\ubc18\uc801\uc73c\ub85c \uad50\ud669\uacfc \uc0ac\uc774\uac00 \uc88b\uc9c0 \uc54a\uc740 \ucd94\uae30\uacbd\ub4e4\uc774 \uc218\ud589\ud588\ub2e4. 1662\ub144 \uc5ed\uc2dc \uad50\ud669\uacfc \uc801\ub300\uc801\uc778 \uad00\uacc4\uc600\ub358 \ub4c0\ud06c \ub4dc \ud06c\ub974\ud0a4\uac00 \ub300\uc0ac\ub85c \uc784\uba85\ub410\ub2e4. \uadf8\ub294 \uc678\uad50 \ub300\uc0ac\uc5d0\uac8c \uc804\ud1b5\uc801\uc73c\ub85c \uc8fc\uc5b4\uc838 \uc654\ub358 \ub9dd\uba85 \uad8c\ud55c\uc744 \ub0a8\uc6a9\ud558\uc5ec \uad50\ud669\ub839\uacfc \ud504\ub791\uc2a4 \uc0ac\uc774\uc5d0 \uc815\uce58\uc801 \uac08\ub4f1\uc744 \ucd09\ubc1c\uc2dc\ucf30\ub2e4. \uc774\ub294 \uacb0\uad6d \uc54c\ub809\uc0b0\ub370\ub974 7\uc138 \ub54c \uc77c\uc2dc\uc801\uc73c\ub85c \uc544\ube44\ub1fd\uc774 \ud504\ub791\uc2a4\ub85c \uadc0\uc18d\ub418\uace0 1664\ub144 \uad74\uc695\uc801\uc778 \ub0b4\uc6a9\uc758 \ud53c\uc0ac \uc870\uc57d \uc744 \ub9fa\ub294 \uacb0\uacfc\ub97c \ub0b3\uc558\ub2e4. ", "paragraph_answer": "\uc54c\ub809\uc0b0\ub370\ub974 7\uc138\ub294 \ud504\ub791\uc2a4 \uc655 \ub8e8\uc774 14\uc138\uc758 \uc870\uc5b8\uc790\uc778 \uc958 \ub9c8\uc790\ub7ad \ucd94\uae30\uacbd\uacfc\uc758 \uc9c0\uc18d\uc801\uc73c\ub85c \uc54c\ub825\uc744 \ubc8c\uc600\ub2e4. \ub9c8\uc790\ub7ad\uc740 \ubca0\uc2a4\ud2b8\ud314\ub80c \uc870\uc57d\uc744 \uccb4\uacb0\ud558\ub294 \ud611\uc0c1 \ub3d9\uc548 \uad50\ud669\uccad\uc758 \ub73b\uc744 \uac70\uc2a4\ub974\uace0 \uac08\ub9ac\uc544 \uad50\ud68c\uc758 \ud2b9\uad8c\uc744 \uc8fc\uc7a5\ud588\ub2e4. \ucf58\ud074\ub77c\ubca0 \uc911\uc5d0 \uadf8\ub294 \ud0a4\uc9c0\uac00 \uc0c8 \uad50\ud669\uc73c\ub85c \uc120\ucd9c\ub418\ub294 \uac83\uc5d0 \ubc18\ub300\ud588\uc9c0\ub9cc, \uacb0\uad6d \ub9c9\ud310\uc5d0 \uc5b4\uca54 \uc218 \uc5c6\uc774 \uadf8\ub97c \ud0c0\ud611\uc548\uc73c\ub85c \ubc1b\uc544\ub4e4\uc77c \uc218 \ubc16\uc5d0 \uc5c6\uc5c8\ub2e4. \ud558\uc9c0\ub9cc \ub9c8\uc790\ub7ad\uc740 \uc790\uc2e0\uc774 \uc0b4\uc544\uc788\ub294 \ub3d9\uc548 \ub8e8\uc774 14\uc138\uac00 \uc54c\ub809\uc0b0\ub370\ub974 7\uc138\uc5d0 \ub300\ud55c \uc21c\uba85\uc758 \uc758\ubbf8\ub85c \ud1b5\uc0c1\uc801\uc73c\ub85c \ud558\ub294 \ub85c\ub9c8\uc5d0 \uc678\uad50 \ub300\uc0ac\ub97c \ud30c\uacac\ud558\ub294 \uac83\uc744 \uac00\ub85c\ub9c9\uc558\uc73c\uba70, \uadf8\ub3d9\uc548 \uad50\ud669\ub839\uacfc\uc758 \uc678\uad50\uc801 \uc5c5\ubb34\ub294 \uc77c\ubc18\uc801\uc73c\ub85c \uad50\ud669\uacfc \uc0ac\uc774\uac00 \uc88b\uc9c0 \uc54a\uc740 \ucd94\uae30\uacbd\ub4e4\uc774 \uc218\ud589\ud588\ub2e4. 1662\ub144 \uc5ed\uc2dc \uad50\ud669\uacfc \uc801\ub300\uc801\uc778 \uad00\uacc4\uc600\ub358 \ub4c0\ud06c \ub4dc \ud06c\ub974\ud0a4\uac00 \ub300\uc0ac\ub85c \uc784\uba85\ub410\ub2e4. \uadf8\ub294 \uc678\uad50 \ub300\uc0ac\uc5d0\uac8c \uc804\ud1b5\uc801\uc73c\ub85c \uc8fc\uc5b4\uc838 \uc654\ub358 \ub9dd\uba85 \uad8c\ud55c\uc744 \ub0a8\uc6a9\ud558\uc5ec \uad50\ud669\ub839\uacfc \ud504\ub791\uc2a4 \uc0ac\uc774\uc5d0 \uc815\uce58\uc801 \uac08\ub4f1\uc744 \ucd09\ubc1c\uc2dc\ucf30\ub2e4. \uc774\ub294 \uacb0\uad6d \uc54c\ub809\uc0b0\ub370\ub974 7\uc138 \ub54c \uc77c\uc2dc\uc801\uc73c\ub85c \uc544\ube44\ub1fd\uc774 \ud504\ub791\uc2a4\ub85c \uadc0\uc18d\ub418\uace0 1664\ub144 \uad74\uc695\uc801\uc778 \ub0b4\uc6a9\uc758 \ud53c\uc0ac \uc870\uc57d \uc744 \ub9fa\ub294 \uacb0\uacfc\ub97c \ub0b3\uc558\ub2e4.", "sentence_answer": "\uc774\ub294 \uacb0\uad6d \uc54c\ub809\uc0b0\ub370\ub974 7\uc138 \ub54c \uc77c\uc2dc\uc801\uc73c\ub85c \uc544\ube44\ub1fd\uc774 \ud504\ub791\uc2a4\ub85c \uadc0\uc18d\ub418\uace0 1664\ub144 \uad74\uc695\uc801\uc778 \ub0b4\uc6a9\uc758 \ud53c\uc0ac \uc870\uc57d \uc744 \ub9fa\ub294 \uacb0\uacfc\ub97c \ub0b3\uc558\ub2e4.", "paragraph_id": "6476359-4-1", "paragraph_question": "question: \uc54c\ub809\uc0b0\ub370\ub974 7\uc138 \ub54c\uc778 1664\ub144 \uad50\ud669\uc774 \ud504\ub791\uc2a4\uc5d0\uac8c \ub9fa\uc5b4\uc57c\ud588\ub358 \uad74\uc695\uc801\uc778 \uc870\uc57d\uc740?, context: \uc54c\ub809\uc0b0\ub370\ub974 7\uc138\ub294 \ud504\ub791\uc2a4 \uc655 \ub8e8\uc774 14\uc138\uc758 \uc870\uc5b8\uc790\uc778 \uc958 \ub9c8\uc790\ub7ad \ucd94\uae30\uacbd\uacfc\uc758 \uc9c0\uc18d\uc801\uc73c\ub85c \uc54c\ub825\uc744 \ubc8c\uc600\ub2e4. \ub9c8\uc790\ub7ad\uc740 \ubca0\uc2a4\ud2b8\ud314\ub80c \uc870\uc57d\uc744 \uccb4\uacb0\ud558\ub294 \ud611\uc0c1 \ub3d9\uc548 \uad50\ud669\uccad\uc758 \ub73b\uc744 \uac70\uc2a4\ub974\uace0 \uac08\ub9ac\uc544 \uad50\ud68c\uc758 \ud2b9\uad8c\uc744 \uc8fc\uc7a5\ud588\ub2e4. \ucf58\ud074\ub77c\ubca0 \uc911\uc5d0 \uadf8\ub294 \ud0a4\uc9c0\uac00 \uc0c8 \uad50\ud669\uc73c\ub85c \uc120\ucd9c\ub418\ub294 \uac83\uc5d0 \ubc18\ub300\ud588\uc9c0\ub9cc, \uacb0\uad6d \ub9c9\ud310\uc5d0 \uc5b4\uca54 \uc218 \uc5c6\uc774 \uadf8\ub97c \ud0c0\ud611\uc548\uc73c\ub85c \ubc1b\uc544\ub4e4\uc77c \uc218 \ubc16\uc5d0 \uc5c6\uc5c8\ub2e4. \ud558\uc9c0\ub9cc \ub9c8\uc790\ub7ad\uc740 \uc790\uc2e0\uc774 \uc0b4\uc544\uc788\ub294 \ub3d9\uc548 \ub8e8\uc774 14\uc138\uac00 \uc54c\ub809\uc0b0\ub370\ub974 7\uc138\uc5d0 \ub300\ud55c \uc21c\uba85\uc758 \uc758\ubbf8\ub85c \ud1b5\uc0c1\uc801\uc73c\ub85c \ud558\ub294 \ub85c\ub9c8\uc5d0 \uc678\uad50 \ub300\uc0ac\ub97c \ud30c\uacac\ud558\ub294 \uac83\uc744 \uac00\ub85c\ub9c9\uc558\uc73c\uba70, \uadf8\ub3d9\uc548 \uad50\ud669\ub839\uacfc\uc758 \uc678\uad50\uc801 \uc5c5\ubb34\ub294 \uc77c\ubc18\uc801\uc73c\ub85c \uad50\ud669\uacfc \uc0ac\uc774\uac00 \uc88b\uc9c0 \uc54a\uc740 \ucd94\uae30\uacbd\ub4e4\uc774 \uc218\ud589\ud588\ub2e4. 1662\ub144 \uc5ed\uc2dc \uad50\ud669\uacfc \uc801\ub300\uc801\uc778 \uad00\uacc4\uc600\ub358 \ub4c0\ud06c \ub4dc \ud06c\ub974\ud0a4\uac00 \ub300\uc0ac\ub85c \uc784\uba85\ub410\ub2e4. \uadf8\ub294 \uc678\uad50 \ub300\uc0ac\uc5d0\uac8c \uc804\ud1b5\uc801\uc73c\ub85c \uc8fc\uc5b4\uc838 \uc654\ub358 \ub9dd\uba85 \uad8c\ud55c\uc744 \ub0a8\uc6a9\ud558\uc5ec \uad50\ud669\ub839\uacfc \ud504\ub791\uc2a4 \uc0ac\uc774\uc5d0 \uc815\uce58\uc801 \uac08\ub4f1\uc744 \ucd09\ubc1c\uc2dc\ucf30\ub2e4. \uc774\ub294 \uacb0\uad6d \uc54c\ub809\uc0b0\ub370\ub974 7\uc138 \ub54c \uc77c\uc2dc\uc801\uc73c\ub85c \uc544\ube44\ub1fd\uc774 \ud504\ub791\uc2a4\ub85c \uadc0\uc18d\ub418\uace0 1664\ub144 \uad74\uc695\uc801\uc778 \ub0b4\uc6a9\uc758 \ud53c\uc0ac \uc870\uc57d\uc744 \ub9fa\ub294 \uacb0\uacfc\ub97c \ub0b3\uc558\ub2e4."} {"question": "\uad50\uc721\ucd1d\uac10\ubd80\ub85c \uc804\uc784\ub41c \ub54c\ub294?", "paragraph": "\ucc38\ubaa8\ubcf8\ubd80 \ubd80\uc6d0 \uacb8 \uc870\uc120\uad70\uc0ac\ub839\ubd80 \uc18c\uc18d\uc73c\ub85c \uc788\ub2e4\uac00 1928\ub144 8\uc6d4 11\uc77c\uc5d0 \uc18c\uc88c\ub85c \uc9c4\uae09\ud558\uc5ec \uadfc\uc704 \ubcf4\ubcd1 \uc81c2\uc5f0\ub300 \ub300\ub300\uc7a5\uc73c\ub85c \uc784\uba85\ub418\uc5c8\ub2e4. 1929\ub144 8\uc6d4\uc5d0 \uc81c1\uc0ac\ub2e8 \uad00\ud560 \ubcf4\ubcd1 \uc81c1\uc5f0\ub300 \uc18c\uc18d\uc73c\ub85c \uc804\ubcf4\ub418\uc5c8\uc73c\uba70, 1930\ub144 12\uc6d4 22\uc77c\uc5d0 \uad50\uc721\ucd1d\uac10\ubd80\ub85c \uc804\uc784\ub418\uc5c8\ub2e4. 1932\ub144 6\uc6d4 24\uc77c\uc5d0 \uc911\uc88c\ub85c \uc9c4\uae09\ud558\uc600\uace0, 1933\ub144 4\uc6d4\uc5d0 \uad50\uc721\ucd1d\uac10\ubd80 \uacfc\uc6d0\uc73c\ub85c \ubcf4\uc784\ub418\uc5c8\uc73c\uba70, 1935\ub144 7\uc6d4 22\uc77c\uc5d0 \ub300\uc88c\ub85c \uc9c4\uae09\ub418\uba74\uc11c \uc6b0\uce20\ub178\ubbf8\uc57c(\u5b87\u90fd\u5bae) \ubcf4\ubcd1 \uc81c59\uc5f0\ub300 \uc5f0\ub300\uc7a5\uc5d0 \uc784\uba85\ub418\uc5c8\ub2e4. 1936\ub144\uc5d0 \uc721\uad70\uc758 \ud669\ub3c4\ud30c \uccad\ub144 \uc7a5\uad50\ub4e4\uc774 \ubc18\ub780\uc744 \uc77c\uc73c\ud0a8 2\u00b726 \uc0ac\uac74\uc774 \ubc1c\uc0dd\ud558\uc790 \uc774\uc740\uc740 \uc9c4\uc555 \uba85\ub839\uc744 \ubc1b\uc544 \uc81c59\uc5f0\ub300 \ubcd1\ub825\uc744 \uc774\ub04c\uace0 \ub3c4\ucfc4\ub85c \uac14\ub2e4\uac00 \ubc18\ub780 \ubd80\ub300\uac00 \ud574\uc0b0\ud558\uba74\uc11c \uc6d0\ub300\ub85c \ubcf5\uadc0\ud558\uc600\ub2e4. 1937\ub144 3\uc6d4\uc5d0 \uc721\uad70\uc0ac\uad00\ud559\uad50 \uad50\uad00\uc73c\ub85c \uc784\uba85\ub418\uc5c8\uace0, \uac19\uc740 \ud574 8\uc6d4\uc5d0 \uc721\uad70\uc0ac\uad00\ud559\uad50 \uc608\uacfc \uad50\uc218\ubd80\uc7a5\uc5d0 \uc784\uba85\ub418\uc5c8\ub2e4. 1938\ub144 2\uc6d4\uc5d0 \ubd81\uc9c0\ub098\ubc29\uba74\uad70\uc0ac\ub839\ubd80 \uc18c\uc18d\uc73c\ub85c \uc804\uc784\ub418\uc5c8\uace0, \uac19\uc740 \ud574 7\uc6d4 15\uc77c\uc5d0 \uc721\uad70 \uc18c\uc7a5\uc73c\ub85c \uc9c4\uae09\ud558\uc600\ub2e4. 1939\ub144 8\uc6d4\uc5d0 \uadfc\uc704 \ubcf4\ubcd1 \uc81c2\uc5ec\ub2e8\uc7a5\uc5d0 \uc784\uba85\ub418\uc5c8\ub2e4\uac00 1940\ub144 8\uc6d4\uc5d0 \uc624\uc0ac\uce74 \uc81c4\uc0ac\ub2e8\uc7a5\uc5d0 \uc784\uba85\ub418\uc5c8\uc73c\uba70, \uac19\uc740 \ud574 12\uc6d4 2\uc77c\uc5d0 \uc721\uad70 \uc911\uc7a5\uc73c\ub85c \uc9c4\uae09\ub418\uc5c8\ub2e4. 1941\ub144 7\uc6d4\uc5d0 \uc81c51\uc0ac\ub2e8\uc7a5\uc5d0 \uc784\uba85\ub418\uc5c8\uace0, \uac19\uc740 \ud574 11\uc6d4 6\uc77c\uc5d0 \uad50\uc721\ucd1d\uac10\ubd80 \uc18c\uc18d\uc73c\ub85c \uc804\uc784\ub418\uc5c8\ub2e4. 1942\ub144 8\uc6d4 1\uc77c\uc5d0 \uc81c1\ud56d\uacf5\uad70\uc0ac\ub839\ubd80\uc5d0 \uc18c\uc18d\ub418\uc5b4 1943\ub144 7\uc6d4 20\uc77c\uc5d0 \uc81c1\ud56d\uacf5\uad70\uc0ac\ub839\uad00\uc73c\ub85c \uc784\uba85\ub418\uc5c8\ub2e4\uac00 1945\ub144 4\uc6d4 1\uc77c\uc5d0 \uad70\uc0ac\ucc38\uc758\uad00\uc73c\ub85c \uc804\uc784\ub418\uc5c8\ub2e4.", "answer": "1930\ub144 12\uc6d4 22\uc77c", "sentence": "1929\ub144 8\uc6d4\uc5d0 \uc81c1\uc0ac\ub2e8 \uad00\ud560 \ubcf4\ubcd1 \uc81c1\uc5f0\ub300 \uc18c\uc18d\uc73c\ub85c \uc804\ubcf4\ub418\uc5c8\uc73c\uba70, 1930\ub144 12\uc6d4 22\uc77c \uc5d0 \uad50\uc721\ucd1d\uac10\ubd80\ub85c \uc804\uc784\ub418\uc5c8\ub2e4.", "paragraph_sentence": "\ucc38\ubaa8\ubcf8\ubd80 \ubd80\uc6d0 \uacb8 \uc870\uc120\uad70\uc0ac\ub839\ubd80 \uc18c\uc18d\uc73c\ub85c \uc788\ub2e4\uac00 1928\ub144 8\uc6d4 11\uc77c\uc5d0 \uc18c\uc88c\ub85c \uc9c4\uae09\ud558\uc5ec \uadfc\uc704 \ubcf4\ubcd1 \uc81c2\uc5f0\ub300 \ub300\ub300\uc7a5\uc73c\ub85c \uc784\uba85\ub418\uc5c8\ub2e4. 1929\ub144 8\uc6d4\uc5d0 \uc81c1\uc0ac\ub2e8 \uad00\ud560 \ubcf4\ubcd1 \uc81c1\uc5f0\ub300 \uc18c\uc18d\uc73c\ub85c \uc804\ubcf4\ub418\uc5c8\uc73c\uba70, 1930\ub144 12\uc6d4 22\uc77c \uc5d0 \uad50\uc721\ucd1d\uac10\ubd80\ub85c \uc804\uc784\ub418\uc5c8\ub2e4. 1932\ub144 6\uc6d4 24\uc77c\uc5d0 \uc911\uc88c\ub85c \uc9c4\uae09\ud558\uc600\uace0, 1933\ub144 4\uc6d4\uc5d0 \uad50\uc721\ucd1d\uac10\ubd80 \uacfc\uc6d0\uc73c\ub85c \ubcf4\uc784\ub418\uc5c8\uc73c\uba70, 1935\ub144 7\uc6d4 22\uc77c\uc5d0 \ub300\uc88c\ub85c \uc9c4\uae09\ub418\uba74\uc11c \uc6b0\uce20\ub178\ubbf8\uc57c(\u5b87\u90fd\u5bae) \ubcf4\ubcd1 \uc81c59\uc5f0\ub300 \uc5f0\ub300\uc7a5\uc5d0 \uc784\uba85\ub418\uc5c8\ub2e4. 1936\ub144\uc5d0 \uc721\uad70\uc758 \ud669\ub3c4\ud30c \uccad\ub144 \uc7a5\uad50\ub4e4\uc774 \ubc18\ub780\uc744 \uc77c\uc73c\ud0a8 2\u00b726 \uc0ac\uac74\uc774 \ubc1c\uc0dd\ud558\uc790 \uc774\uc740\uc740 \uc9c4\uc555 \uba85\ub839\uc744 \ubc1b\uc544 \uc81c59\uc5f0\ub300 \ubcd1\ub825\uc744 \uc774\ub04c\uace0 \ub3c4\ucfc4\ub85c \uac14\ub2e4\uac00 \ubc18\ub780 \ubd80\ub300\uac00 \ud574\uc0b0\ud558\uba74\uc11c \uc6d0\ub300\ub85c \ubcf5\uadc0\ud558\uc600\ub2e4. 1937\ub144 3\uc6d4\uc5d0 \uc721\uad70\uc0ac\uad00\ud559\uad50 \uad50\uad00\uc73c\ub85c \uc784\uba85\ub418\uc5c8\uace0, \uac19\uc740 \ud574 8\uc6d4\uc5d0 \uc721\uad70\uc0ac\uad00\ud559\uad50 \uc608\uacfc \uad50\uc218\ubd80\uc7a5\uc5d0 \uc784\uba85\ub418\uc5c8\ub2e4. 1938\ub144 2\uc6d4\uc5d0 \ubd81\uc9c0\ub098\ubc29\uba74\uad70\uc0ac\ub839\ubd80 \uc18c\uc18d\uc73c\ub85c \uc804\uc784\ub418\uc5c8\uace0, \uac19\uc740 \ud574 7\uc6d4 15\uc77c\uc5d0 \uc721\uad70 \uc18c\uc7a5\uc73c\ub85c \uc9c4\uae09\ud558\uc600\ub2e4. 1939\ub144 8\uc6d4\uc5d0 \uadfc\uc704 \ubcf4\ubcd1 \uc81c2\uc5ec\ub2e8\uc7a5\uc5d0 \uc784\uba85\ub418\uc5c8\ub2e4\uac00 1940\ub144 8\uc6d4\uc5d0 \uc624\uc0ac\uce74 \uc81c4\uc0ac\ub2e8\uc7a5\uc5d0 \uc784\uba85\ub418\uc5c8\uc73c\uba70, \uac19\uc740 \ud574 12\uc6d4 2\uc77c\uc5d0 \uc721\uad70 \uc911\uc7a5\uc73c\ub85c \uc9c4\uae09\ub418\uc5c8\ub2e4. 1941\ub144 7\uc6d4\uc5d0 \uc81c51\uc0ac\ub2e8\uc7a5\uc5d0 \uc784\uba85\ub418\uc5c8\uace0, \uac19\uc740 \ud574 11\uc6d4 6\uc77c\uc5d0 \uad50\uc721\ucd1d\uac10\ubd80 \uc18c\uc18d\uc73c\ub85c \uc804\uc784\ub418\uc5c8\ub2e4. 1942\ub144 8\uc6d4 1\uc77c\uc5d0 \uc81c1\ud56d\uacf5\uad70\uc0ac\ub839\ubd80\uc5d0 \uc18c\uc18d\ub418\uc5b4 1943\ub144 7\uc6d4 20\uc77c\uc5d0 \uc81c1\ud56d\uacf5\uad70\uc0ac\ub839\uad00\uc73c\ub85c \uc784\uba85\ub418\uc5c8\ub2e4\uac00 1945\ub144 4\uc6d4 1\uc77c\uc5d0 \uad70\uc0ac\ucc38\uc758\uad00\uc73c\ub85c \uc804\uc784\ub418\uc5c8\ub2e4.", "paragraph_answer": "\ucc38\ubaa8\ubcf8\ubd80 \ubd80\uc6d0 \uacb8 \uc870\uc120\uad70\uc0ac\ub839\ubd80 \uc18c\uc18d\uc73c\ub85c \uc788\ub2e4\uac00 1928\ub144 8\uc6d4 11\uc77c\uc5d0 \uc18c\uc88c\ub85c \uc9c4\uae09\ud558\uc5ec \uadfc\uc704 \ubcf4\ubcd1 \uc81c2\uc5f0\ub300 \ub300\ub300\uc7a5\uc73c\ub85c \uc784\uba85\ub418\uc5c8\ub2e4. 1929\ub144 8\uc6d4\uc5d0 \uc81c1\uc0ac\ub2e8 \uad00\ud560 \ubcf4\ubcd1 \uc81c1\uc5f0\ub300 \uc18c\uc18d\uc73c\ub85c \uc804\ubcf4\ub418\uc5c8\uc73c\uba70, 1930\ub144 12\uc6d4 22\uc77c \uc5d0 \uad50\uc721\ucd1d\uac10\ubd80\ub85c \uc804\uc784\ub418\uc5c8\ub2e4. 1932\ub144 6\uc6d4 24\uc77c\uc5d0 \uc911\uc88c\ub85c \uc9c4\uae09\ud558\uc600\uace0, 1933\ub144 4\uc6d4\uc5d0 \uad50\uc721\ucd1d\uac10\ubd80 \uacfc\uc6d0\uc73c\ub85c \ubcf4\uc784\ub418\uc5c8\uc73c\uba70, 1935\ub144 7\uc6d4 22\uc77c\uc5d0 \ub300\uc88c\ub85c \uc9c4\uae09\ub418\uba74\uc11c \uc6b0\uce20\ub178\ubbf8\uc57c(\u5b87\u90fd\u5bae) \ubcf4\ubcd1 \uc81c59\uc5f0\ub300 \uc5f0\ub300\uc7a5\uc5d0 \uc784\uba85\ub418\uc5c8\ub2e4. 1936\ub144\uc5d0 \uc721\uad70\uc758 \ud669\ub3c4\ud30c \uccad\ub144 \uc7a5\uad50\ub4e4\uc774 \ubc18\ub780\uc744 \uc77c\uc73c\ud0a8 2\u00b726 \uc0ac\uac74\uc774 \ubc1c\uc0dd\ud558\uc790 \uc774\uc740\uc740 \uc9c4\uc555 \uba85\ub839\uc744 \ubc1b\uc544 \uc81c59\uc5f0\ub300 \ubcd1\ub825\uc744 \uc774\ub04c\uace0 \ub3c4\ucfc4\ub85c \uac14\ub2e4\uac00 \ubc18\ub780 \ubd80\ub300\uac00 \ud574\uc0b0\ud558\uba74\uc11c \uc6d0\ub300\ub85c \ubcf5\uadc0\ud558\uc600\ub2e4. 1937\ub144 3\uc6d4\uc5d0 \uc721\uad70\uc0ac\uad00\ud559\uad50 \uad50\uad00\uc73c\ub85c \uc784\uba85\ub418\uc5c8\uace0, \uac19\uc740 \ud574 8\uc6d4\uc5d0 \uc721\uad70\uc0ac\uad00\ud559\uad50 \uc608\uacfc \uad50\uc218\ubd80\uc7a5\uc5d0 \uc784\uba85\ub418\uc5c8\ub2e4. 1938\ub144 2\uc6d4\uc5d0 \ubd81\uc9c0\ub098\ubc29\uba74\uad70\uc0ac\ub839\ubd80 \uc18c\uc18d\uc73c\ub85c \uc804\uc784\ub418\uc5c8\uace0, \uac19\uc740 \ud574 7\uc6d4 15\uc77c\uc5d0 \uc721\uad70 \uc18c\uc7a5\uc73c\ub85c \uc9c4\uae09\ud558\uc600\ub2e4. 1939\ub144 8\uc6d4\uc5d0 \uadfc\uc704 \ubcf4\ubcd1 \uc81c2\uc5ec\ub2e8\uc7a5\uc5d0 \uc784\uba85\ub418\uc5c8\ub2e4\uac00 1940\ub144 8\uc6d4\uc5d0 \uc624\uc0ac\uce74 \uc81c4\uc0ac\ub2e8\uc7a5\uc5d0 \uc784\uba85\ub418\uc5c8\uc73c\uba70, \uac19\uc740 \ud574 12\uc6d4 2\uc77c\uc5d0 \uc721\uad70 \uc911\uc7a5\uc73c\ub85c \uc9c4\uae09\ub418\uc5c8\ub2e4. 1941\ub144 7\uc6d4\uc5d0 \uc81c51\uc0ac\ub2e8\uc7a5\uc5d0 \uc784\uba85\ub418\uc5c8\uace0, \uac19\uc740 \ud574 11\uc6d4 6\uc77c\uc5d0 \uad50\uc721\ucd1d\uac10\ubd80 \uc18c\uc18d\uc73c\ub85c \uc804\uc784\ub418\uc5c8\ub2e4. 1942\ub144 8\uc6d4 1\uc77c\uc5d0 \uc81c1\ud56d\uacf5\uad70\uc0ac\ub839\ubd80\uc5d0 \uc18c\uc18d\ub418\uc5b4 1943\ub144 7\uc6d4 20\uc77c\uc5d0 \uc81c1\ud56d\uacf5\uad70\uc0ac\ub839\uad00\uc73c\ub85c \uc784\uba85\ub418\uc5c8\ub2e4\uac00 1945\ub144 4\uc6d4 1\uc77c\uc5d0 \uad70\uc0ac\ucc38\uc758\uad00\uc73c\ub85c \uc804\uc784\ub418\uc5c8\ub2e4.", "sentence_answer": "1929\ub144 8\uc6d4\uc5d0 \uc81c1\uc0ac\ub2e8 \uad00\ud560 \ubcf4\ubcd1 \uc81c1\uc5f0\ub300 \uc18c\uc18d\uc73c\ub85c \uc804\ubcf4\ub418\uc5c8\uc73c\uba70, 1930\ub144 12\uc6d4 22\uc77c \uc5d0 \uad50\uc721\ucd1d\uac10\ubd80\ub85c \uc804\uc784\ub418\uc5c8\ub2e4.", "paragraph_id": "6589848-9-1", "paragraph_question": "question: \uad50\uc721\ucd1d\uac10\ubd80\ub85c \uc804\uc784\ub41c \ub54c\ub294?, context: \ucc38\ubaa8\ubcf8\ubd80 \ubd80\uc6d0 \uacb8 \uc870\uc120\uad70\uc0ac\ub839\ubd80 \uc18c\uc18d\uc73c\ub85c \uc788\ub2e4\uac00 1928\ub144 8\uc6d4 11\uc77c\uc5d0 \uc18c\uc88c\ub85c \uc9c4\uae09\ud558\uc5ec \uadfc\uc704 \ubcf4\ubcd1 \uc81c2\uc5f0\ub300 \ub300\ub300\uc7a5\uc73c\ub85c \uc784\uba85\ub418\uc5c8\ub2e4. 1929\ub144 8\uc6d4\uc5d0 \uc81c1\uc0ac\ub2e8 \uad00\ud560 \ubcf4\ubcd1 \uc81c1\uc5f0\ub300 \uc18c\uc18d\uc73c\ub85c \uc804\ubcf4\ub418\uc5c8\uc73c\uba70, 1930\ub144 12\uc6d4 22\uc77c\uc5d0 \uad50\uc721\ucd1d\uac10\ubd80\ub85c \uc804\uc784\ub418\uc5c8\ub2e4. 1932\ub144 6\uc6d4 24\uc77c\uc5d0 \uc911\uc88c\ub85c \uc9c4\uae09\ud558\uc600\uace0, 1933\ub144 4\uc6d4\uc5d0 \uad50\uc721\ucd1d\uac10\ubd80 \uacfc\uc6d0\uc73c\ub85c \ubcf4\uc784\ub418\uc5c8\uc73c\uba70, 1935\ub144 7\uc6d4 22\uc77c\uc5d0 \ub300\uc88c\ub85c \uc9c4\uae09\ub418\uba74\uc11c \uc6b0\uce20\ub178\ubbf8\uc57c(\u5b87\u90fd\u5bae) \ubcf4\ubcd1 \uc81c59\uc5f0\ub300 \uc5f0\ub300\uc7a5\uc5d0 \uc784\uba85\ub418\uc5c8\ub2e4. 1936\ub144\uc5d0 \uc721\uad70\uc758 \ud669\ub3c4\ud30c \uccad\ub144 \uc7a5\uad50\ub4e4\uc774 \ubc18\ub780\uc744 \uc77c\uc73c\ud0a8 2\u00b726 \uc0ac\uac74\uc774 \ubc1c\uc0dd\ud558\uc790 \uc774\uc740\uc740 \uc9c4\uc555 \uba85\ub839\uc744 \ubc1b\uc544 \uc81c59\uc5f0\ub300 \ubcd1\ub825\uc744 \uc774\ub04c\uace0 \ub3c4\ucfc4\ub85c \uac14\ub2e4\uac00 \ubc18\ub780 \ubd80\ub300\uac00 \ud574\uc0b0\ud558\uba74\uc11c \uc6d0\ub300\ub85c \ubcf5\uadc0\ud558\uc600\ub2e4. 1937\ub144 3\uc6d4\uc5d0 \uc721\uad70\uc0ac\uad00\ud559\uad50 \uad50\uad00\uc73c\ub85c \uc784\uba85\ub418\uc5c8\uace0, \uac19\uc740 \ud574 8\uc6d4\uc5d0 \uc721\uad70\uc0ac\uad00\ud559\uad50 \uc608\uacfc \uad50\uc218\ubd80\uc7a5\uc5d0 \uc784\uba85\ub418\uc5c8\ub2e4. 1938\ub144 2\uc6d4\uc5d0 \ubd81\uc9c0\ub098\ubc29\uba74\uad70\uc0ac\ub839\ubd80 \uc18c\uc18d\uc73c\ub85c \uc804\uc784\ub418\uc5c8\uace0, \uac19\uc740 \ud574 7\uc6d4 15\uc77c\uc5d0 \uc721\uad70 \uc18c\uc7a5\uc73c\ub85c \uc9c4\uae09\ud558\uc600\ub2e4. 1939\ub144 8\uc6d4\uc5d0 \uadfc\uc704 \ubcf4\ubcd1 \uc81c2\uc5ec\ub2e8\uc7a5\uc5d0 \uc784\uba85\ub418\uc5c8\ub2e4\uac00 1940\ub144 8\uc6d4\uc5d0 \uc624\uc0ac\uce74 \uc81c4\uc0ac\ub2e8\uc7a5\uc5d0 \uc784\uba85\ub418\uc5c8\uc73c\uba70, \uac19\uc740 \ud574 12\uc6d4 2\uc77c\uc5d0 \uc721\uad70 \uc911\uc7a5\uc73c\ub85c \uc9c4\uae09\ub418\uc5c8\ub2e4. 1941\ub144 7\uc6d4\uc5d0 \uc81c51\uc0ac\ub2e8\uc7a5\uc5d0 \uc784\uba85\ub418\uc5c8\uace0, \uac19\uc740 \ud574 11\uc6d4 6\uc77c\uc5d0 \uad50\uc721\ucd1d\uac10\ubd80 \uc18c\uc18d\uc73c\ub85c \uc804\uc784\ub418\uc5c8\ub2e4. 1942\ub144 8\uc6d4 1\uc77c\uc5d0 \uc81c1\ud56d\uacf5\uad70\uc0ac\ub839\ubd80\uc5d0 \uc18c\uc18d\ub418\uc5b4 1943\ub144 7\uc6d4 20\uc77c\uc5d0 \uc81c1\ud56d\uacf5\uad70\uc0ac\ub839\uad00\uc73c\ub85c \uc784\uba85\ub418\uc5c8\ub2e4\uac00 1945\ub144 4\uc6d4 1\uc77c\uc5d0 \uad70\uc0ac\ucc38\uc758\uad00\uc73c\ub85c \uc804\uc784\ub418\uc5c8\ub2e4."} {"question": "\uae40\ub300\uc911\uc758 \ucc3d\uc528\uac1c\uba85 \uc774\ub984\uc740?", "paragraph": "\ud558\uc9c0\ub9cc \uae40\ub300\uc911\uc740 \uacb0\uad6d \ub300\ud559 \uc9c4\ud559\uc744 \ud3ec\uae30\ud558\uc600\ub2e4. 4\ud559\ub144\uc774 \ub418\uba74\uc11c \ub3c5\uc11c\ub85c \uc778\ud574 \ub113\uc5b4\uc9c4 \uacac\ubb38 \ud0d3\uc5d0 \ubc18\uc77c \uc0ac\uc0c1\uc774 \uc0dd\uaca8\ub098 \uc77c\ubcf8\uc778 \ud559\uc0dd\ub4e4\uacfc \uac08\ub4f1\uc744 \ube5a\uc73c\uba74\uc11c \uc804 \uacfc\ubaa9\uc5d0 \uac78\uccd0 \uc131\uc801\uc774 \ub5a8\uc5b4\uc9c0\uae30\ub3c4 \ud588\uc73c\uba70, \ubc18\uc77c \uc791\ubb38\uc774 \ubb38\uc81c\uac00 \ub410\ub2e4. \uc77c\ubcf8\uc758 \ub300\ud559 \uc9c4\ud559\uc744 \ud76c\ub9dd\ud588\uc9c0\ub9cc \ubbf8\uad6d \ud574\uad70\uc758 \ud574\uc0c1 \ubd09\uc1c4 \uc815\ucc45\uc73c\ub85c \uc778\ud574 \uc5ec\ud589 \ud5c8\uac00\uac00 \uc5b4\ub824\uc6e0\uace0 \uc774\uc5d0 \ud559\ube44\uac00 \ubb34\ub8cc\uc778 \ub9cc\uc8fc\uad6d\uc758 \uac74\uad6d\ub300\ud559\uc5d0 \uc9c4\ud559\ud558\ub294 \uac83\ub3c4 \uace0\ub824\ud574 \ubd24\uc9c0\ub9cc \uc9c4\ud559\ud574\ubd10\uc57c \uc77c\ubcf8\uad70\uc5d0 \uc9d5\uc9d1\uc774 \ub420 \uac83\uc774 \ubed4\ud558\ub2e4\ub294 2\uac00\uc9c0 \uc774\uc720\uc5d0\uc11c\uc600\ub2e4. 15\uc138\uac00 \ub418\ub358 1938\ub144\uc5d0 \ub3c4\uc694\ud0c0 \ub2e4\uc774\ucd94(\uc77c\ubcf8\uc5b4: \u8c4a\u7530\u5927\u4e2d)\ub85c \ucc3d\uc528\uac1c\uba85\uc744 \ud558\uc600\ub2e4. \ub300\ud1b5\ub839\uc774 \ub41c \ub4a4\uc5d0\ub3c4 \uc77c\ubcf8\uc5d0 \uc62c \ub54c\ub9c8\ub2e4 \uc0c1\uc5c5\ud559\uad50 \uc2dc\uc808 \uc740\uc0ac\uc5d0\uac8c \uc774 \uc774\ub984\uc744 \uc0ac\uc6a9\ud588\ub2e4\uace0 \uc544\uc0ac\ud788 \uc2e0\ubb38\uc740 \ubcf4\ub3c4\ud588\ub2e4.", "answer": "\ub3c4\uc694\ud0c0 \ub2e4\uc774\ucd94", "sentence": "15\uc138\uac00 \ub418\ub358 1938\ub144\uc5d0 \ub3c4\uc694\ud0c0 \ub2e4\uc774\ucd94 (\uc77c\ubcf8\uc5b4: \u8c4a\u7530\u5927\u4e2d)\ub85c \ucc3d\uc528\uac1c\uba85\uc744 \ud558\uc600\ub2e4.", "paragraph_sentence": "\ud558\uc9c0\ub9cc \uae40\ub300\uc911\uc740 \uacb0\uad6d \ub300\ud559 \uc9c4\ud559\uc744 \ud3ec\uae30\ud558\uc600\ub2e4. 4\ud559\ub144\uc774 \ub418\uba74\uc11c \ub3c5\uc11c\ub85c \uc778\ud574 \ub113\uc5b4\uc9c4 \uacac\ubb38 \ud0d3\uc5d0 \ubc18\uc77c \uc0ac\uc0c1\uc774 \uc0dd\uaca8\ub098 \uc77c\ubcf8\uc778 \ud559\uc0dd\ub4e4\uacfc \uac08\ub4f1\uc744 \ube5a\uc73c\uba74\uc11c \uc804 \uacfc\ubaa9\uc5d0 \uac78\uccd0 \uc131\uc801\uc774 \ub5a8\uc5b4\uc9c0\uae30\ub3c4 \ud588\uc73c\uba70, \ubc18\uc77c \uc791\ubb38\uc774 \ubb38\uc81c\uac00 \ub410\ub2e4. \uc77c\ubcf8\uc758 \ub300\ud559 \uc9c4\ud559\uc744 \ud76c\ub9dd\ud588\uc9c0\ub9cc \ubbf8\uad6d \ud574\uad70\uc758 \ud574\uc0c1 \ubd09\uc1c4 \uc815\ucc45\uc73c\ub85c \uc778\ud574 \uc5ec\ud589 \ud5c8\uac00\uac00 \uc5b4\ub824\uc6e0\uace0 \uc774\uc5d0 \ud559\ube44\uac00 \ubb34\ub8cc\uc778 \ub9cc\uc8fc\uad6d\uc758 \uac74\uad6d\ub300\ud559\uc5d0 \uc9c4\ud559\ud558\ub294 \uac83\ub3c4 \uace0\ub824\ud574 \ubd24\uc9c0\ub9cc \uc9c4\ud559\ud574\ubd10\uc57c \uc77c\ubcf8\uad70\uc5d0 \uc9d5\uc9d1\uc774 \ub420 \uac83\uc774 \ubed4\ud558\ub2e4\ub294 2\uac00\uc9c0 \uc774\uc720\uc5d0\uc11c\uc600\ub2e4. 15\uc138\uac00 \ub418\ub358 1938\ub144\uc5d0 \ub3c4\uc694\ud0c0 \ub2e4\uc774\ucd94 (\uc77c\ubcf8\uc5b4: \u8c4a\u7530\u5927\u4e2d)\ub85c \ucc3d\uc528\uac1c\uba85\uc744 \ud558\uc600\ub2e4. \ub300\ud1b5\ub839\uc774 \ub41c \ub4a4\uc5d0\ub3c4 \uc77c\ubcf8\uc5d0 \uc62c \ub54c\ub9c8\ub2e4 \uc0c1\uc5c5\ud559\uad50 \uc2dc\uc808 \uc740\uc0ac\uc5d0\uac8c \uc774 \uc774\ub984\uc744 \uc0ac\uc6a9\ud588\ub2e4\uace0 \uc544\uc0ac\ud788 \uc2e0\ubb38\uc740 \ubcf4\ub3c4\ud588\ub2e4.", "paragraph_answer": "\ud558\uc9c0\ub9cc \uae40\ub300\uc911\uc740 \uacb0\uad6d \ub300\ud559 \uc9c4\ud559\uc744 \ud3ec\uae30\ud558\uc600\ub2e4. 4\ud559\ub144\uc774 \ub418\uba74\uc11c \ub3c5\uc11c\ub85c \uc778\ud574 \ub113\uc5b4\uc9c4 \uacac\ubb38 \ud0d3\uc5d0 \ubc18\uc77c \uc0ac\uc0c1\uc774 \uc0dd\uaca8\ub098 \uc77c\ubcf8\uc778 \ud559\uc0dd\ub4e4\uacfc \uac08\ub4f1\uc744 \ube5a\uc73c\uba74\uc11c \uc804 \uacfc\ubaa9\uc5d0 \uac78\uccd0 \uc131\uc801\uc774 \ub5a8\uc5b4\uc9c0\uae30\ub3c4 \ud588\uc73c\uba70, \ubc18\uc77c \uc791\ubb38\uc774 \ubb38\uc81c\uac00 \ub410\ub2e4. \uc77c\ubcf8\uc758 \ub300\ud559 \uc9c4\ud559\uc744 \ud76c\ub9dd\ud588\uc9c0\ub9cc \ubbf8\uad6d \ud574\uad70\uc758 \ud574\uc0c1 \ubd09\uc1c4 \uc815\ucc45\uc73c\ub85c \uc778\ud574 \uc5ec\ud589 \ud5c8\uac00\uac00 \uc5b4\ub824\uc6e0\uace0 \uc774\uc5d0 \ud559\ube44\uac00 \ubb34\ub8cc\uc778 \ub9cc\uc8fc\uad6d\uc758 \uac74\uad6d\ub300\ud559\uc5d0 \uc9c4\ud559\ud558\ub294 \uac83\ub3c4 \uace0\ub824\ud574 \ubd24\uc9c0\ub9cc \uc9c4\ud559\ud574\ubd10\uc57c \uc77c\ubcf8\uad70\uc5d0 \uc9d5\uc9d1\uc774 \ub420 \uac83\uc774 \ubed4\ud558\ub2e4\ub294 2\uac00\uc9c0 \uc774\uc720\uc5d0\uc11c\uc600\ub2e4. 15\uc138\uac00 \ub418\ub358 1938\ub144\uc5d0 \ub3c4\uc694\ud0c0 \ub2e4\uc774\ucd94 (\uc77c\ubcf8\uc5b4: \u8c4a\u7530\u5927\u4e2d)\ub85c \ucc3d\uc528\uac1c\uba85\uc744 \ud558\uc600\ub2e4. \ub300\ud1b5\ub839\uc774 \ub41c \ub4a4\uc5d0\ub3c4 \uc77c\ubcf8\uc5d0 \uc62c \ub54c\ub9c8\ub2e4 \uc0c1\uc5c5\ud559\uad50 \uc2dc\uc808 \uc740\uc0ac\uc5d0\uac8c \uc774 \uc774\ub984\uc744 \uc0ac\uc6a9\ud588\ub2e4\uace0 \uc544\uc0ac\ud788 \uc2e0\ubb38\uc740 \ubcf4\ub3c4\ud588\ub2e4.", "sentence_answer": "15\uc138\uac00 \ub418\ub358 1938\ub144\uc5d0 \ub3c4\uc694\ud0c0 \ub2e4\uc774\ucd94 (\uc77c\ubcf8\uc5b4: \u8c4a\u7530\u5927\u4e2d)\ub85c \ucc3d\uc528\uac1c\uba85\uc744 \ud558\uc600\ub2e4.", "paragraph_id": "6488203-3-1", "paragraph_question": "question: \uae40\ub300\uc911\uc758 \ucc3d\uc528\uac1c\uba85 \uc774\ub984\uc740?, context: \ud558\uc9c0\ub9cc \uae40\ub300\uc911\uc740 \uacb0\uad6d \ub300\ud559 \uc9c4\ud559\uc744 \ud3ec\uae30\ud558\uc600\ub2e4. 4\ud559\ub144\uc774 \ub418\uba74\uc11c \ub3c5\uc11c\ub85c \uc778\ud574 \ub113\uc5b4\uc9c4 \uacac\ubb38 \ud0d3\uc5d0 \ubc18\uc77c \uc0ac\uc0c1\uc774 \uc0dd\uaca8\ub098 \uc77c\ubcf8\uc778 \ud559\uc0dd\ub4e4\uacfc \uac08\ub4f1\uc744 \ube5a\uc73c\uba74\uc11c \uc804 \uacfc\ubaa9\uc5d0 \uac78\uccd0 \uc131\uc801\uc774 \ub5a8\uc5b4\uc9c0\uae30\ub3c4 \ud588\uc73c\uba70, \ubc18\uc77c \uc791\ubb38\uc774 \ubb38\uc81c\uac00 \ub410\ub2e4. \uc77c\ubcf8\uc758 \ub300\ud559 \uc9c4\ud559\uc744 \ud76c\ub9dd\ud588\uc9c0\ub9cc \ubbf8\uad6d \ud574\uad70\uc758 \ud574\uc0c1 \ubd09\uc1c4 \uc815\ucc45\uc73c\ub85c \uc778\ud574 \uc5ec\ud589 \ud5c8\uac00\uac00 \uc5b4\ub824\uc6e0\uace0 \uc774\uc5d0 \ud559\ube44\uac00 \ubb34\ub8cc\uc778 \ub9cc\uc8fc\uad6d\uc758 \uac74\uad6d\ub300\ud559\uc5d0 \uc9c4\ud559\ud558\ub294 \uac83\ub3c4 \uace0\ub824\ud574 \ubd24\uc9c0\ub9cc \uc9c4\ud559\ud574\ubd10\uc57c \uc77c\ubcf8\uad70\uc5d0 \uc9d5\uc9d1\uc774 \ub420 \uac83\uc774 \ubed4\ud558\ub2e4\ub294 2\uac00\uc9c0 \uc774\uc720\uc5d0\uc11c\uc600\ub2e4. 15\uc138\uac00 \ub418\ub358 1938\ub144\uc5d0 \ub3c4\uc694\ud0c0 \ub2e4\uc774\ucd94(\uc77c\ubcf8\uc5b4: \u8c4a\u7530\u5927\u4e2d)\ub85c \ucc3d\uc528\uac1c\uba85\uc744 \ud558\uc600\ub2e4. \ub300\ud1b5\ub839\uc774 \ub41c \ub4a4\uc5d0\ub3c4 \uc77c\ubcf8\uc5d0 \uc62c \ub54c\ub9c8\ub2e4 \uc0c1\uc5c5\ud559\uad50 \uc2dc\uc808 \uc740\uc0ac\uc5d0\uac8c \uc774 \uc774\ub984\uc744 \uc0ac\uc6a9\ud588\ub2e4\uace0 \uc544\uc0ac\ud788 \uc2e0\ubb38\uc740 \ubcf4\ub3c4\ud588\ub2e4."} {"question": "\uad6c\uc131\uc6d0\uc758 \ud611\ub825\uc744 \ub3c4\ubaa8\ud558\uace0 \uc6b0\ud638\uad00\uacc4\ub97c \uc99d\uc9c4\ud558\uace0\uc790 \ud558\ub294 \uc695\uad6c\ub294?", "paragraph": "\ubbf8\uad6d\uc758 \ud0e4\ucef7 \ud30c\uc2a8\uc2a4\ub294 \uc0ac\ud68c\uac00 \uc9c8\uc11c\ub97c \uc720\uc9c0\ud558\uae30 \uc704\ud558\uc5ec \uac16\uace0 \uc788\uc5b4\uc57c \ud560 \ud2b9\uc9d5\uc744 \ud615\ud0dc\uc720\uc9c0\u00b7 \ub2e8\uc704\ud1b5\ud569\u00b7 \ubaa9\ud45c\ub2ec\uc131\u00b7 \uc801\uc751\uc758 4\uac00\uc9c0\ub85c \ub098\ub234\ub2e4. \ud615\ud0dc\uc720\uc9c0\ub294 \uc0ac\ud68c\uac00 \ubb38\ud654\uc640 \uad00\uc2b5\uc744 \ubcf4\uc874\ud568\uc73c\ub85c\uc368 \uc790\uc2e0\uc758 \uc815\uccb4\uc131\uc744 \uc720\uc9c0\ud558\ub824\ub294 \uc695\uad6c\ub85c, \uc0dd\uba85\uccb4\uac00 \uc790\uc2e0\uc758 \uc678\ud615\uacfc \ub0b4\ubd80 \uae30\uad00\uc758 \uae30\ub2a5\uc744 \uc720\uc9c0\ud558\ub294 \uac83\uacfc \ube44\uc2b7\ud558\ub2e4. \uc774 \uc695\uad6c\ub97c \ucda9\uc871\ud558\uae30 \uc704\ud574 \ud559\uad50\ub098 \uc885\uad50\u00b7 \ubc95 \ub4f1\uc774 \uc874\uc7ac\ud55c\ub2e4. \ub2e8\uc704\ud1b5\ud569\uc740 \uc0ac\ud68c\uac00 \uad6c\uc131\uc6d0\uc744 \uc720\uae30\uc801\uc73c\ub85c \uacb0\ud569\ud568\uc73c\ub85c\uc368 \uc640\ud574\ub97c \ub9c9\uace0\uc790\ud558\ub294 \uc695\uad6c\ub85c, \uce5c\uc871\uad00\uacc4\ub098 \uc798 \uc9dc\uc5ec\uc9c4 \uc870\uc9c1\uc774 \uc774\ub7ec\ud55c \uc695\uad6c\ub97c \ucda9\uc871\ud55c\ub2e4. \uc0dd\ubb3c\uccb4\uac00 \ub0b4\ubd80 \uae30\uad00\uc744 \uc720\uae30\uc801\uc73c\ub85c \uc5f0\uacb0\ud558\uc5ec \ud56d\uc0c1\uc131\uc744 \uc720\uc9c0\ud558\ub294 \uac83\uacfc \ube44\uc2b7\ud55c \uc148\uc774\ub2e4. \ud55c\ud3b8, \ubaa9\ud45c\ub2ec\uc131\uc740 \uc0ac\ud68c\uac00 \uacf5\uacf5\uc758 \ubaa9\ud45c\ub97c \uc124\uc815\ud568\uc73c\ub85c\uc368 \uad6c\uc131\uc6d0\uc758 \ud611\ub825\uc744 \ub3c4\ubaa8\ud558\uace0 \uc6b0\ud638\uad00\uacc4\ub97c \uc99d\uc9c4\ud558\uace0\uc790 \ud558\ub294 \uc695\uad6c\ub97c \ub9d0\ud55c\ub2e4. \uc815\uce58\ud65c\ub3d9\uc774 \uc874\uc7ac\ud558\ub294 \uc774\uc720\uac00 \ubc14\ub85c \uc774 \ub54c\ubb38\uc774\ub2e4. \ub9c8\uc9c0\ub9c9\uc73c\ub85c \uc801\uc751\uc740 \uc0ac\ud68c\uac00 \uc678\ubd80\uc758 \ucda9\uaca9\uc5d0\uc73c\ub85c\ubd80\ud130 \uc790\uc2e0\uc744 \ubcf4\ud638\ud558\ub824\ub294 \uc695\uad6c\ub85c, \uc0dd\uba85\uccb4\uac00 \uc678\ubd80 \ubcc0\ud654\uc5d0 \uc801\uc751\ud558\uace0 \uc9c4\ud654\ud558\ub294 \uac83\uacfc \ube44\uc2b7\ud558\ub2e4. \uae30\ub2a5\uc8fc\uc758\ub294 20\uc138\uae30 \uc911\ubc18\uae4c\uc9c0 \uc0ac\ud68c\ud559\uc744 \uc9c0\ubc30\ud558\ub294 \uc0ac\uc0c1\uc774\uc5c8\uc73c\uba70 \uba38\ud134\uc774 \uacfc\ud559\uc0ac\ud68c\ud559\uc744 \uc815\ub9bd\ud558\ub294\ub370 \ud070 \uc601\ud5a5\uc744 \uc8fc\uc5c8\ub2e4.", "answer": "\ubaa9\ud45c\ub2ec\uc131", "sentence": "\ubbf8\uad6d\uc758 \ud0e4\ucef7 \ud30c\uc2a8\uc2a4\ub294 \uc0ac\ud68c\uac00 \uc9c8\uc11c\ub97c \uc720\uc9c0\ud558\uae30 \uc704\ud558\uc5ec \uac16\uace0 \uc788\uc5b4\uc57c \ud560 \ud2b9\uc9d5\uc744 \ud615\ud0dc\uc720\uc9c0\u00b7 \ub2e8\uc704\ud1b5\ud569\u00b7 \ubaa9\ud45c\ub2ec\uc131 \u00b7 \uc801\uc751\uc758 4\uac00\uc9c0\ub85c \ub098\ub234\ub2e4.", "paragraph_sentence": " \ubbf8\uad6d\uc758 \ud0e4\ucef7 \ud30c\uc2a8\uc2a4\ub294 \uc0ac\ud68c\uac00 \uc9c8\uc11c\ub97c \uc720\uc9c0\ud558\uae30 \uc704\ud558\uc5ec \uac16\uace0 \uc788\uc5b4\uc57c \ud560 \ud2b9\uc9d5\uc744 \ud615\ud0dc\uc720\uc9c0\u00b7 \ub2e8\uc704\ud1b5\ud569\u00b7 \ubaa9\ud45c\ub2ec\uc131 \u00b7 \uc801\uc751\uc758 4\uac00\uc9c0\ub85c \ub098\ub234\ub2e4. \ud615\ud0dc\uc720\uc9c0\ub294 \uc0ac\ud68c\uac00 \ubb38\ud654\uc640 \uad00\uc2b5\uc744 \ubcf4\uc874\ud568\uc73c\ub85c\uc368 \uc790\uc2e0\uc758 \uc815\uccb4\uc131\uc744 \uc720\uc9c0\ud558\ub824\ub294 \uc695\uad6c\ub85c, \uc0dd\uba85\uccb4\uac00 \uc790\uc2e0\uc758 \uc678\ud615\uacfc \ub0b4\ubd80 \uae30\uad00\uc758 \uae30\ub2a5\uc744 \uc720\uc9c0\ud558\ub294 \uac83\uacfc \ube44\uc2b7\ud558\ub2e4. \uc774 \uc695\uad6c\ub97c \ucda9\uc871\ud558\uae30 \uc704\ud574 \ud559\uad50\ub098 \uc885\uad50\u00b7 \ubc95 \ub4f1\uc774 \uc874\uc7ac\ud55c\ub2e4. \ub2e8\uc704\ud1b5\ud569\uc740 \uc0ac\ud68c\uac00 \uad6c\uc131\uc6d0\uc744 \uc720\uae30\uc801\uc73c\ub85c \uacb0\ud569\ud568\uc73c\ub85c\uc368 \uc640\ud574\ub97c \ub9c9\uace0\uc790\ud558\ub294 \uc695\uad6c\ub85c, \uce5c\uc871\uad00\uacc4\ub098 \uc798 \uc9dc\uc5ec\uc9c4 \uc870\uc9c1\uc774 \uc774\ub7ec\ud55c \uc695\uad6c\ub97c \ucda9\uc871\ud55c\ub2e4. \uc0dd\ubb3c\uccb4\uac00 \ub0b4\ubd80 \uae30\uad00\uc744 \uc720\uae30\uc801\uc73c\ub85c \uc5f0\uacb0\ud558\uc5ec \ud56d\uc0c1\uc131\uc744 \uc720\uc9c0\ud558\ub294 \uac83\uacfc \ube44\uc2b7\ud55c \uc148\uc774\ub2e4. \ud55c\ud3b8, \ubaa9\ud45c\ub2ec\uc131\uc740 \uc0ac\ud68c\uac00 \uacf5\uacf5\uc758 \ubaa9\ud45c\ub97c \uc124\uc815\ud568\uc73c\ub85c\uc368 \uad6c\uc131\uc6d0\uc758 \ud611\ub825\uc744 \ub3c4\ubaa8\ud558\uace0 \uc6b0\ud638\uad00\uacc4\ub97c \uc99d\uc9c4\ud558\uace0\uc790 \ud558\ub294 \uc695\uad6c\ub97c \ub9d0\ud55c\ub2e4. \uc815\uce58\ud65c\ub3d9\uc774 \uc874\uc7ac\ud558\ub294 \uc774\uc720\uac00 \ubc14\ub85c \uc774 \ub54c\ubb38\uc774\ub2e4. \ub9c8\uc9c0\ub9c9\uc73c\ub85c \uc801\uc751\uc740 \uc0ac\ud68c\uac00 \uc678\ubd80\uc758 \ucda9\uaca9\uc5d0\uc73c\ub85c\ubd80\ud130 \uc790\uc2e0\uc744 \ubcf4\ud638\ud558\ub824\ub294 \uc695\uad6c\ub85c, \uc0dd\uba85\uccb4\uac00 \uc678\ubd80 \ubcc0\ud654\uc5d0 \uc801\uc751\ud558\uace0 \uc9c4\ud654\ud558\ub294 \uac83\uacfc \ube44\uc2b7\ud558\ub2e4. \uae30\ub2a5\uc8fc\uc758\ub294 20\uc138\uae30 \uc911\ubc18\uae4c\uc9c0 \uc0ac\ud68c\ud559\uc744 \uc9c0\ubc30\ud558\ub294 \uc0ac\uc0c1\uc774\uc5c8\uc73c\uba70 \uba38\ud134\uc774 \uacfc\ud559\uc0ac\ud68c\ud559\uc744 \uc815\ub9bd\ud558\ub294\ub370 \ud070 \uc601\ud5a5\uc744 \uc8fc\uc5c8\ub2e4.", "paragraph_answer": "\ubbf8\uad6d\uc758 \ud0e4\ucef7 \ud30c\uc2a8\uc2a4\ub294 \uc0ac\ud68c\uac00 \uc9c8\uc11c\ub97c \uc720\uc9c0\ud558\uae30 \uc704\ud558\uc5ec \uac16\uace0 \uc788\uc5b4\uc57c \ud560 \ud2b9\uc9d5\uc744 \ud615\ud0dc\uc720\uc9c0\u00b7 \ub2e8\uc704\ud1b5\ud569\u00b7 \ubaa9\ud45c\ub2ec\uc131 \u00b7 \uc801\uc751\uc758 4\uac00\uc9c0\ub85c \ub098\ub234\ub2e4. \ud615\ud0dc\uc720\uc9c0\ub294 \uc0ac\ud68c\uac00 \ubb38\ud654\uc640 \uad00\uc2b5\uc744 \ubcf4\uc874\ud568\uc73c\ub85c\uc368 \uc790\uc2e0\uc758 \uc815\uccb4\uc131\uc744 \uc720\uc9c0\ud558\ub824\ub294 \uc695\uad6c\ub85c, \uc0dd\uba85\uccb4\uac00 \uc790\uc2e0\uc758 \uc678\ud615\uacfc \ub0b4\ubd80 \uae30\uad00\uc758 \uae30\ub2a5\uc744 \uc720\uc9c0\ud558\ub294 \uac83\uacfc \ube44\uc2b7\ud558\ub2e4. \uc774 \uc695\uad6c\ub97c \ucda9\uc871\ud558\uae30 \uc704\ud574 \ud559\uad50\ub098 \uc885\uad50\u00b7 \ubc95 \ub4f1\uc774 \uc874\uc7ac\ud55c\ub2e4. \ub2e8\uc704\ud1b5\ud569\uc740 \uc0ac\ud68c\uac00 \uad6c\uc131\uc6d0\uc744 \uc720\uae30\uc801\uc73c\ub85c \uacb0\ud569\ud568\uc73c\ub85c\uc368 \uc640\ud574\ub97c \ub9c9\uace0\uc790\ud558\ub294 \uc695\uad6c\ub85c, \uce5c\uc871\uad00\uacc4\ub098 \uc798 \uc9dc\uc5ec\uc9c4 \uc870\uc9c1\uc774 \uc774\ub7ec\ud55c \uc695\uad6c\ub97c \ucda9\uc871\ud55c\ub2e4. \uc0dd\ubb3c\uccb4\uac00 \ub0b4\ubd80 \uae30\uad00\uc744 \uc720\uae30\uc801\uc73c\ub85c \uc5f0\uacb0\ud558\uc5ec \ud56d\uc0c1\uc131\uc744 \uc720\uc9c0\ud558\ub294 \uac83\uacfc \ube44\uc2b7\ud55c \uc148\uc774\ub2e4. \ud55c\ud3b8, \ubaa9\ud45c\ub2ec\uc131\uc740 \uc0ac\ud68c\uac00 \uacf5\uacf5\uc758 \ubaa9\ud45c\ub97c \uc124\uc815\ud568\uc73c\ub85c\uc368 \uad6c\uc131\uc6d0\uc758 \ud611\ub825\uc744 \ub3c4\ubaa8\ud558\uace0 \uc6b0\ud638\uad00\uacc4\ub97c \uc99d\uc9c4\ud558\uace0\uc790 \ud558\ub294 \uc695\uad6c\ub97c \ub9d0\ud55c\ub2e4. \uc815\uce58\ud65c\ub3d9\uc774 \uc874\uc7ac\ud558\ub294 \uc774\uc720\uac00 \ubc14\ub85c \uc774 \ub54c\ubb38\uc774\ub2e4. \ub9c8\uc9c0\ub9c9\uc73c\ub85c \uc801\uc751\uc740 \uc0ac\ud68c\uac00 \uc678\ubd80\uc758 \ucda9\uaca9\uc5d0\uc73c\ub85c\ubd80\ud130 \uc790\uc2e0\uc744 \ubcf4\ud638\ud558\ub824\ub294 \uc695\uad6c\ub85c, \uc0dd\uba85\uccb4\uac00 \uc678\ubd80 \ubcc0\ud654\uc5d0 \uc801\uc751\ud558\uace0 \uc9c4\ud654\ud558\ub294 \uac83\uacfc \ube44\uc2b7\ud558\ub2e4. \uae30\ub2a5\uc8fc\uc758\ub294 20\uc138\uae30 \uc911\ubc18\uae4c\uc9c0 \uc0ac\ud68c\ud559\uc744 \uc9c0\ubc30\ud558\ub294 \uc0ac\uc0c1\uc774\uc5c8\uc73c\uba70 \uba38\ud134\uc774 \uacfc\ud559\uc0ac\ud68c\ud559\uc744 \uc815\ub9bd\ud558\ub294\ub370 \ud070 \uc601\ud5a5\uc744 \uc8fc\uc5c8\ub2e4.", "sentence_answer": "\ubbf8\uad6d\uc758 \ud0e4\ucef7 \ud30c\uc2a8\uc2a4\ub294 \uc0ac\ud68c\uac00 \uc9c8\uc11c\ub97c \uc720\uc9c0\ud558\uae30 \uc704\ud558\uc5ec \uac16\uace0 \uc788\uc5b4\uc57c \ud560 \ud2b9\uc9d5\uc744 \ud615\ud0dc\uc720\uc9c0\u00b7 \ub2e8\uc704\ud1b5\ud569\u00b7 \ubaa9\ud45c\ub2ec\uc131 \u00b7 \uc801\uc751\uc758 4\uac00\uc9c0\ub85c \ub098\ub234\ub2e4.", "paragraph_id": "5959284-2-1", "paragraph_question": "question: \uad6c\uc131\uc6d0\uc758 \ud611\ub825\uc744 \ub3c4\ubaa8\ud558\uace0 \uc6b0\ud638\uad00\uacc4\ub97c \uc99d\uc9c4\ud558\uace0\uc790 \ud558\ub294 \uc695\uad6c\ub294?, context: \ubbf8\uad6d\uc758 \ud0e4\ucef7 \ud30c\uc2a8\uc2a4\ub294 \uc0ac\ud68c\uac00 \uc9c8\uc11c\ub97c \uc720\uc9c0\ud558\uae30 \uc704\ud558\uc5ec \uac16\uace0 \uc788\uc5b4\uc57c \ud560 \ud2b9\uc9d5\uc744 \ud615\ud0dc\uc720\uc9c0\u00b7 \ub2e8\uc704\ud1b5\ud569\u00b7 \ubaa9\ud45c\ub2ec\uc131\u00b7 \uc801\uc751\uc758 4\uac00\uc9c0\ub85c \ub098\ub234\ub2e4. \ud615\ud0dc\uc720\uc9c0\ub294 \uc0ac\ud68c\uac00 \ubb38\ud654\uc640 \uad00\uc2b5\uc744 \ubcf4\uc874\ud568\uc73c\ub85c\uc368 \uc790\uc2e0\uc758 \uc815\uccb4\uc131\uc744 \uc720\uc9c0\ud558\ub824\ub294 \uc695\uad6c\ub85c, \uc0dd\uba85\uccb4\uac00 \uc790\uc2e0\uc758 \uc678\ud615\uacfc \ub0b4\ubd80 \uae30\uad00\uc758 \uae30\ub2a5\uc744 \uc720\uc9c0\ud558\ub294 \uac83\uacfc \ube44\uc2b7\ud558\ub2e4. \uc774 \uc695\uad6c\ub97c \ucda9\uc871\ud558\uae30 \uc704\ud574 \ud559\uad50\ub098 \uc885\uad50\u00b7 \ubc95 \ub4f1\uc774 \uc874\uc7ac\ud55c\ub2e4. \ub2e8\uc704\ud1b5\ud569\uc740 \uc0ac\ud68c\uac00 \uad6c\uc131\uc6d0\uc744 \uc720\uae30\uc801\uc73c\ub85c \uacb0\ud569\ud568\uc73c\ub85c\uc368 \uc640\ud574\ub97c \ub9c9\uace0\uc790\ud558\ub294 \uc695\uad6c\ub85c, \uce5c\uc871\uad00\uacc4\ub098 \uc798 \uc9dc\uc5ec\uc9c4 \uc870\uc9c1\uc774 \uc774\ub7ec\ud55c \uc695\uad6c\ub97c \ucda9\uc871\ud55c\ub2e4. \uc0dd\ubb3c\uccb4\uac00 \ub0b4\ubd80 \uae30\uad00\uc744 \uc720\uae30\uc801\uc73c\ub85c \uc5f0\uacb0\ud558\uc5ec \ud56d\uc0c1\uc131\uc744 \uc720\uc9c0\ud558\ub294 \uac83\uacfc \ube44\uc2b7\ud55c \uc148\uc774\ub2e4. \ud55c\ud3b8, \ubaa9\ud45c\ub2ec\uc131\uc740 \uc0ac\ud68c\uac00 \uacf5\uacf5\uc758 \ubaa9\ud45c\ub97c \uc124\uc815\ud568\uc73c\ub85c\uc368 \uad6c\uc131\uc6d0\uc758 \ud611\ub825\uc744 \ub3c4\ubaa8\ud558\uace0 \uc6b0\ud638\uad00\uacc4\ub97c \uc99d\uc9c4\ud558\uace0\uc790 \ud558\ub294 \uc695\uad6c\ub97c \ub9d0\ud55c\ub2e4. \uc815\uce58\ud65c\ub3d9\uc774 \uc874\uc7ac\ud558\ub294 \uc774\uc720\uac00 \ubc14\ub85c \uc774 \ub54c\ubb38\uc774\ub2e4. \ub9c8\uc9c0\ub9c9\uc73c\ub85c \uc801\uc751\uc740 \uc0ac\ud68c\uac00 \uc678\ubd80\uc758 \ucda9\uaca9\uc5d0\uc73c\ub85c\ubd80\ud130 \uc790\uc2e0\uc744 \ubcf4\ud638\ud558\ub824\ub294 \uc695\uad6c\ub85c, \uc0dd\uba85\uccb4\uac00 \uc678\ubd80 \ubcc0\ud654\uc5d0 \uc801\uc751\ud558\uace0 \uc9c4\ud654\ud558\ub294 \uac83\uacfc \ube44\uc2b7\ud558\ub2e4. \uae30\ub2a5\uc8fc\uc758\ub294 20\uc138\uae30 \uc911\ubc18\uae4c\uc9c0 \uc0ac\ud68c\ud559\uc744 \uc9c0\ubc30\ud558\ub294 \uc0ac\uc0c1\uc774\uc5c8\uc73c\uba70 \uba38\ud134\uc774 \uacfc\ud559\uc0ac\ud68c\ud559\uc744 \uc815\ub9bd\ud558\ub294\ub370 \ud070 \uc601\ud5a5\uc744 \uc8fc\uc5c8\ub2e4."} {"question": "\uac04\ub514\ub97c \uc790\ubcf8\uac00\ub4e4\uc744 \uc704\ud574 \uc774\ubc14\uc9c0\ud558\ub294 \uc9c0\uc2dd\uc778\uc73c\ub85c \ubcf8 \uc0ac\ub78c\ub4e4\uc740 \ub204\uad6c\uc778\uac00?", "paragraph": "\uac04\ub514\ub294 \uac70\ub8e9\ud558\uace0 \ud65c\ubc1c\ud55c \uc0ac\ud68c\uc6b4\ub3d9\uc744 \ud588\uc9c0\ub9cc, \ub178\ub3d9\uc790\ub4e4\uc5d0\uac8c \uac04\ub514\ub294 \uc790\ubcf8\uac00\ub4e4\uc744 \uc704\ud574 \ubd09\uc0ac\ud558\ub294 \uc9c0\uc2dd\uc778\uc5d0 \ubd88\uacfc\ud55c \uce21\uba74\ub3c4 \uc788\ub2e4. \uadf8\ub294 1917\ub144 \ud30c\uc5c5\uad8c\uc774 \uc5c6\ub294 \ub178\ub3d9\uc870\ud569\uacb0\uc131 \uc9c0\uc6d0\uc744 \uc790\ubcf8\uac00\ub4e4\uc5d0\uac8c \uc81c\uc758\ud574\uc11c \uad6c\uc790\ub77c\ud2b8 \uc8fc\uc758 \ub178\ub3d9\uc6b4\ub3d9\uc744 \uce68\uccb4\uc2dc\ucf30\ub294\ub370, \uc774\ub294 \ud65c\ubc1c\ud55c \ub178\ub3d9\uc6b4\ub3d9\uc73c\ub85c \ub178\ub3d9\uc778\uad8c\uc744 \uc7c1\ucde8\ud574 \uac04 \ubd04\ubca0\uc774\uc758 \ub178\ub3d9\uc790\ub4e4\uacfc\ub294 \ub300\uc870\ub418\ub294 \ubaa8\uc2b5\uc774\uc5c8\ub2e4. \ub610\ud55c 1935\ub144 \uac00\uc871\uc784\uae08\uc81c\ub3c4(\uac00\uc871 \uc911 \ud55c \uba85\uc774 \uc2e4\uc9c1\uc2dc \ub2e4\ub978 \uac00\uc871\uc758 \uc784\uae08\uc744 \uc778\uc0c1\ud558\ub294 \uc81c\ub3c4)\ub97c \ubc1b\uc544\ub4e4\uc5ec, \ud798\uc5c6\ub294 \uc5ec\uc131\uacfc \ub178\uc778\ub178\ub3d9\uc790\ub4e4\uc774 \ud574\uace0\ub2f9\ud558\uac8c \ud558\uc600\ub2e4. \uc81c1\ucc28 \uc138\uacc4 \ub300\uc804 \ub54c\uc5d0\ub294 \uc804\uc7c1\uc5d0 \ud611\ub825\ud558\uba74 \uc778\ub3c4\ub97c \ub3c5\ub9bd\uc2dc\ucf1c\uc8fc\uaca0\ub2e4\ub294 \uc601\uad6d\uc758 \ud68c\uc720\uc5d0 \ud604\ud639\ub418\uc5b4 \uc778\ub3c4\uc758 \uc804\uc7c1\uac00\ub2f4\uc5d0 \uad00\uc5ec\ud588\ub294\ub370, \uc774\ub294 \uadf8\uc758 \ube44\ud3ed\ub825\uc8fc\uc758\uc5d0 \uc5b4\uae0b\ub098\ub294 \uc810\uc774 \uc788\ub2e4. \ucc38\uace0\ub85c \ube44\ud3ed\ub825\uc8fc\uc758\ub294 \ub300\uad6d\uc778 \uc778\ub3c4 \uc2e4\uc815\uc5d0\ub294 \ubd80\ud569\ud560 \uc218\uc788\uc9c0\ub9cc \uc57d\uc18c\uad6d\uc758 \ub3c5\ub9bd\uc6b4\ub3d9\uc73c\ub85c\ub294 \uc801\ud569\ud558\uc9c0 \uc54a\uc744 \uc218\uc788\uc5b4 \uc2e4\uc81c\ub85c \ub9ce\uc740 \uc2dd\ubbfc\uc9c0 \ub098\ub77c\ub4e4\uc774 \uc774\ub97c \ucc44\ud0dd\ud558\uc9c0 \uc54a\uc558\ub2e4.", "answer": "\ub178\ub3d9\uc790\ub4e4", "sentence": "\uac04\ub514\ub294 \uac70\ub8e9\ud558\uace0 \ud65c\ubc1c\ud55c \uc0ac\ud68c\uc6b4\ub3d9\uc744 \ud588\uc9c0\ub9cc, \ub178\ub3d9\uc790\ub4e4 \uc5d0\uac8c \uac04\ub514\ub294 \uc790\ubcf8\uac00\ub4e4\uc744 \uc704\ud574 \ubd09\uc0ac\ud558\ub294 \uc9c0\uc2dd\uc778\uc5d0 \ubd88\uacfc\ud55c \uce21\uba74\ub3c4 \uc788\ub2e4.", "paragraph_sentence": " \uac04\ub514\ub294 \uac70\ub8e9\ud558\uace0 \ud65c\ubc1c\ud55c \uc0ac\ud68c\uc6b4\ub3d9\uc744 \ud588\uc9c0\ub9cc, \ub178\ub3d9\uc790\ub4e4 \uc5d0\uac8c \uac04\ub514\ub294 \uc790\ubcf8\uac00\ub4e4\uc744 \uc704\ud574 \ubd09\uc0ac\ud558\ub294 \uc9c0\uc2dd\uc778\uc5d0 \ubd88\uacfc\ud55c \uce21\uba74\ub3c4 \uc788\ub2e4. \uadf8\ub294 1917\ub144 \ud30c\uc5c5\uad8c\uc774 \uc5c6\ub294 \ub178\ub3d9\uc870\ud569\uacb0\uc131 \uc9c0\uc6d0\uc744 \uc790\ubcf8\uac00\ub4e4\uc5d0\uac8c \uc81c\uc758\ud574\uc11c \uad6c\uc790\ub77c\ud2b8 \uc8fc\uc758 \ub178\ub3d9\uc6b4\ub3d9\uc744 \uce68\uccb4\uc2dc\ucf30\ub294\ub370, \uc774\ub294 \ud65c\ubc1c\ud55c \ub178\ub3d9\uc6b4\ub3d9\uc73c\ub85c \ub178\ub3d9\uc778\uad8c\uc744 \uc7c1\ucde8\ud574 \uac04 \ubd04\ubca0\uc774\uc758 \ub178\ub3d9\uc790\ub4e4\uacfc\ub294 \ub300\uc870\ub418\ub294 \ubaa8\uc2b5\uc774\uc5c8\ub2e4. \ub610\ud55c 1935\ub144 \uac00\uc871\uc784\uae08\uc81c\ub3c4(\uac00\uc871 \uc911 \ud55c \uba85\uc774 \uc2e4\uc9c1\uc2dc \ub2e4\ub978 \uac00\uc871\uc758 \uc784\uae08\uc744 \uc778\uc0c1\ud558\ub294 \uc81c\ub3c4)\ub97c \ubc1b\uc544\ub4e4\uc5ec, \ud798\uc5c6\ub294 \uc5ec\uc131\uacfc \ub178\uc778\ub178\ub3d9\uc790\ub4e4\uc774 \ud574\uace0\ub2f9\ud558\uac8c \ud558\uc600\ub2e4. \uc81c1\ucc28 \uc138\uacc4 \ub300\uc804 \ub54c\uc5d0\ub294 \uc804\uc7c1\uc5d0 \ud611\ub825\ud558\uba74 \uc778\ub3c4\ub97c \ub3c5\ub9bd\uc2dc\ucf1c\uc8fc\uaca0\ub2e4\ub294 \uc601\uad6d\uc758 \ud68c\uc720\uc5d0 \ud604\ud639\ub418\uc5b4 \uc778\ub3c4\uc758 \uc804\uc7c1\uac00\ub2f4\uc5d0 \uad00\uc5ec\ud588\ub294\ub370, \uc774\ub294 \uadf8\uc758 \ube44\ud3ed\ub825\uc8fc\uc758\uc5d0 \uc5b4\uae0b\ub098\ub294 \uc810\uc774 \uc788\ub2e4. \ucc38\uace0\ub85c \ube44\ud3ed\ub825\uc8fc\uc758\ub294 \ub300\uad6d\uc778 \uc778\ub3c4 \uc2e4\uc815\uc5d0\ub294 \ubd80\ud569\ud560 \uc218\uc788\uc9c0\ub9cc \uc57d\uc18c\uad6d\uc758 \ub3c5\ub9bd\uc6b4\ub3d9\uc73c\ub85c\ub294 \uc801\ud569\ud558\uc9c0 \uc54a\uc744 \uc218\uc788\uc5b4 \uc2e4\uc81c\ub85c \ub9ce\uc740 \uc2dd\ubbfc\uc9c0 \ub098\ub77c\ub4e4\uc774 \uc774\ub97c \ucc44\ud0dd\ud558\uc9c0 \uc54a\uc558\ub2e4.", "paragraph_answer": "\uac04\ub514\ub294 \uac70\ub8e9\ud558\uace0 \ud65c\ubc1c\ud55c \uc0ac\ud68c\uc6b4\ub3d9\uc744 \ud588\uc9c0\ub9cc, \ub178\ub3d9\uc790\ub4e4 \uc5d0\uac8c \uac04\ub514\ub294 \uc790\ubcf8\uac00\ub4e4\uc744 \uc704\ud574 \ubd09\uc0ac\ud558\ub294 \uc9c0\uc2dd\uc778\uc5d0 \ubd88\uacfc\ud55c \uce21\uba74\ub3c4 \uc788\ub2e4. \uadf8\ub294 1917\ub144 \ud30c\uc5c5\uad8c\uc774 \uc5c6\ub294 \ub178\ub3d9\uc870\ud569\uacb0\uc131 \uc9c0\uc6d0\uc744 \uc790\ubcf8\uac00\ub4e4\uc5d0\uac8c \uc81c\uc758\ud574\uc11c \uad6c\uc790\ub77c\ud2b8 \uc8fc\uc758 \ub178\ub3d9\uc6b4\ub3d9\uc744 \uce68\uccb4\uc2dc\ucf30\ub294\ub370, \uc774\ub294 \ud65c\ubc1c\ud55c \ub178\ub3d9\uc6b4\ub3d9\uc73c\ub85c \ub178\ub3d9\uc778\uad8c\uc744 \uc7c1\ucde8\ud574 \uac04 \ubd04\ubca0\uc774\uc758 \ub178\ub3d9\uc790\ub4e4\uacfc\ub294 \ub300\uc870\ub418\ub294 \ubaa8\uc2b5\uc774\uc5c8\ub2e4. \ub610\ud55c 1935\ub144 \uac00\uc871\uc784\uae08\uc81c\ub3c4(\uac00\uc871 \uc911 \ud55c \uba85\uc774 \uc2e4\uc9c1\uc2dc \ub2e4\ub978 \uac00\uc871\uc758 \uc784\uae08\uc744 \uc778\uc0c1\ud558\ub294 \uc81c\ub3c4)\ub97c \ubc1b\uc544\ub4e4\uc5ec, \ud798\uc5c6\ub294 \uc5ec\uc131\uacfc \ub178\uc778\ub178\ub3d9\uc790\ub4e4\uc774 \ud574\uace0\ub2f9\ud558\uac8c \ud558\uc600\ub2e4. \uc81c1\ucc28 \uc138\uacc4 \ub300\uc804 \ub54c\uc5d0\ub294 \uc804\uc7c1\uc5d0 \ud611\ub825\ud558\uba74 \uc778\ub3c4\ub97c \ub3c5\ub9bd\uc2dc\ucf1c\uc8fc\uaca0\ub2e4\ub294 \uc601\uad6d\uc758 \ud68c\uc720\uc5d0 \ud604\ud639\ub418\uc5b4 \uc778\ub3c4\uc758 \uc804\uc7c1\uac00\ub2f4\uc5d0 \uad00\uc5ec\ud588\ub294\ub370, \uc774\ub294 \uadf8\uc758 \ube44\ud3ed\ub825\uc8fc\uc758\uc5d0 \uc5b4\uae0b\ub098\ub294 \uc810\uc774 \uc788\ub2e4. \ucc38\uace0\ub85c \ube44\ud3ed\ub825\uc8fc\uc758\ub294 \ub300\uad6d\uc778 \uc778\ub3c4 \uc2e4\uc815\uc5d0\ub294 \ubd80\ud569\ud560 \uc218\uc788\uc9c0\ub9cc \uc57d\uc18c\uad6d\uc758 \ub3c5\ub9bd\uc6b4\ub3d9\uc73c\ub85c\ub294 \uc801\ud569\ud558\uc9c0 \uc54a\uc744 \uc218\uc788\uc5b4 \uc2e4\uc81c\ub85c \ub9ce\uc740 \uc2dd\ubbfc\uc9c0 \ub098\ub77c\ub4e4\uc774 \uc774\ub97c \ucc44\ud0dd\ud558\uc9c0 \uc54a\uc558\ub2e4.", "sentence_answer": "\uac04\ub514\ub294 \uac70\ub8e9\ud558\uace0 \ud65c\ubc1c\ud55c \uc0ac\ud68c\uc6b4\ub3d9\uc744 \ud588\uc9c0\ub9cc, \ub178\ub3d9\uc790\ub4e4 \uc5d0\uac8c \uac04\ub514\ub294 \uc790\ubcf8\uac00\ub4e4\uc744 \uc704\ud574 \ubd09\uc0ac\ud558\ub294 \uc9c0\uc2dd\uc778\uc5d0 \ubd88\uacfc\ud55c \uce21\uba74\ub3c4 \uc788\ub2e4.", "paragraph_id": "6514246-4-1", "paragraph_question": "question: \uac04\ub514\ub97c \uc790\ubcf8\uac00\ub4e4\uc744 \uc704\ud574 \uc774\ubc14\uc9c0\ud558\ub294 \uc9c0\uc2dd\uc778\uc73c\ub85c \ubcf8 \uc0ac\ub78c\ub4e4\uc740 \ub204\uad6c\uc778\uac00?, context: \uac04\ub514\ub294 \uac70\ub8e9\ud558\uace0 \ud65c\ubc1c\ud55c \uc0ac\ud68c\uc6b4\ub3d9\uc744 \ud588\uc9c0\ub9cc, \ub178\ub3d9\uc790\ub4e4\uc5d0\uac8c \uac04\ub514\ub294 \uc790\ubcf8\uac00\ub4e4\uc744 \uc704\ud574 \ubd09\uc0ac\ud558\ub294 \uc9c0\uc2dd\uc778\uc5d0 \ubd88\uacfc\ud55c \uce21\uba74\ub3c4 \uc788\ub2e4. \uadf8\ub294 1917\ub144 \ud30c\uc5c5\uad8c\uc774 \uc5c6\ub294 \ub178\ub3d9\uc870\ud569\uacb0\uc131 \uc9c0\uc6d0\uc744 \uc790\ubcf8\uac00\ub4e4\uc5d0\uac8c \uc81c\uc758\ud574\uc11c \uad6c\uc790\ub77c\ud2b8 \uc8fc\uc758 \ub178\ub3d9\uc6b4\ub3d9\uc744 \uce68\uccb4\uc2dc\ucf30\ub294\ub370, \uc774\ub294 \ud65c\ubc1c\ud55c \ub178\ub3d9\uc6b4\ub3d9\uc73c\ub85c \ub178\ub3d9\uc778\uad8c\uc744 \uc7c1\ucde8\ud574 \uac04 \ubd04\ubca0\uc774\uc758 \ub178\ub3d9\uc790\ub4e4\uacfc\ub294 \ub300\uc870\ub418\ub294 \ubaa8\uc2b5\uc774\uc5c8\ub2e4. \ub610\ud55c 1935\ub144 \uac00\uc871\uc784\uae08\uc81c\ub3c4(\uac00\uc871 \uc911 \ud55c \uba85\uc774 \uc2e4\uc9c1\uc2dc \ub2e4\ub978 \uac00\uc871\uc758 \uc784\uae08\uc744 \uc778\uc0c1\ud558\ub294 \uc81c\ub3c4)\ub97c \ubc1b\uc544\ub4e4\uc5ec, \ud798\uc5c6\ub294 \uc5ec\uc131\uacfc \ub178\uc778\ub178\ub3d9\uc790\ub4e4\uc774 \ud574\uace0\ub2f9\ud558\uac8c \ud558\uc600\ub2e4. \uc81c1\ucc28 \uc138\uacc4 \ub300\uc804 \ub54c\uc5d0\ub294 \uc804\uc7c1\uc5d0 \ud611\ub825\ud558\uba74 \uc778\ub3c4\ub97c \ub3c5\ub9bd\uc2dc\ucf1c\uc8fc\uaca0\ub2e4\ub294 \uc601\uad6d\uc758 \ud68c\uc720\uc5d0 \ud604\ud639\ub418\uc5b4 \uc778\ub3c4\uc758 \uc804\uc7c1\uac00\ub2f4\uc5d0 \uad00\uc5ec\ud588\ub294\ub370, \uc774\ub294 \uadf8\uc758 \ube44\ud3ed\ub825\uc8fc\uc758\uc5d0 \uc5b4\uae0b\ub098\ub294 \uc810\uc774 \uc788\ub2e4. \ucc38\uace0\ub85c \ube44\ud3ed\ub825\uc8fc\uc758\ub294 \ub300\uad6d\uc778 \uc778\ub3c4 \uc2e4\uc815\uc5d0\ub294 \ubd80\ud569\ud560 \uc218\uc788\uc9c0\ub9cc \uc57d\uc18c\uad6d\uc758 \ub3c5\ub9bd\uc6b4\ub3d9\uc73c\ub85c\ub294 \uc801\ud569\ud558\uc9c0 \uc54a\uc744 \uc218\uc788\uc5b4 \uc2e4\uc81c\ub85c \ub9ce\uc740 \uc2dd\ubbfc\uc9c0 \ub098\ub77c\ub4e4\uc774 \uc774\ub97c \ucc44\ud0dd\ud558\uc9c0 \uc54a\uc558\ub2e4."} {"question": "\"Airbag\" \ubcf4\uceec \ud53c\uccd0\ub9c1\uc740 \ub204\uac00 \ucc38\uc5ec\ud588\ub098\uc694?", "paragraph": "\"Airbag\"\uc740 \ud0c0\ube14\ub85c\uac00 \uc9c1\uc811 \uc791\uace1\uacfc \uc791\uc0ac, \ud3b8\uace1\uc744 \ub9e1\uc558\ub2e4. \ubcf4\uceec \ud53c\uccd0\ub9c1\uc73c\ub85c\ub294 \ud3c9\uc18c \ud0c0\ube14\ub85c\uac00 \"\uc874\uacbd\ud558\ub294 \ubaa9\uc18c\ub9ac\"\ub97c \uac00\uc9c4 \ube0c\ub77c\uc6b4 \uc544\uc774\ub4dc \uc18c\uc6b8\uc758 \uba64\ubc84 \ub098\uc5bc\uc774 \ucc38\uc5ec\ud588\ub2e4. \uc6b4\uc804\uc5d0 \uc11c\ud234\ub7ec \uc0c8\ubcbd\uc5d0 \uc678\ucd9c\ud560 \ub54c \ud0dd\uc2dc\ub97c \ud0c0\uace0 \ub2e4\ub2cc \ud0c0\ube14\ub85c\ub294 \"\ud0dd\uc2dc \uc548\uc5d0\uc11c \uac00\uc7a5 \ub9ce\uc740 \uc0dd\uac01\uc744 \ud558\uac8c \ub418\uace0, \uac00\uc7a5 \ub9ce\uc740 \uac00\uc0ac\ub97c\" \uc4f0\uac8c \ub418\uc5c8\ub2e4. \ud0c0\ube14\ub85c\ub294 \ud0dd\uc2dc \uc548\uc5d0\uc11c \ucc3d\ubb38 \ubc16 \ud48d\uacbd\uc744 \ubc14\ub77c\ubcf4\uace0 \ub77c\ub514\uc624\uc5d0\uc11c \ub098\uc624\ub294 \uc598\uae30\ub4e4\uc744 \ub4e4\uc73c\uba70 \ud55c \uacf5\uac04\uc5d0\uc11c \uc138\uc0c1 \uacf3\uacf3\uc5d0 \uc788\ub294 \uc0ac\ub78c\ub4e4\uc758 \uc678\ub85c\uc6c0\uc744 \ub290\ub07c\uac8c \ub418\uc5b4 \uace1\uc5d0 \ub300\ud55c \uc601\uac10\uc744 \uc5bb\uac8c \ub418\uc5c8\ub2e4. \ud0c0\ube14\ub85c\uac00 \ud0dd\uc2dc \uc548\uc5d0\uc11c \ub290\ub080 \"\uc138\uc0c1\uc758 \ubaa8\ub4e0 \uc678\ub85c\uc6cc \ud558\ub294 \uc0ac\ub78c\ub4e4\uc758 \uc598\uae30\"\ub77c\uace0 \uc0dd\uac01\uc744 \ud558\uace0 \uc4f0\uac8c \ub41c \uace1\uc774\ub2e4. \"Airbag\"\uc774\ub77c\ub294 \uc81c\ubaa9\uacfc \uc8fc\uc81c\uc5d0 \ub300\ud574\uc11c\ub294 \ucc28 \uc548\uc5d0 \uc0ac\uace0\ub97c \ub300\ube44\ud55c \uc5d0\uc5b4\ubc31\uc774 \uc788\ub4ef\uc774, '\uc0ac\ub78c\uc774 \uc2ac\ud514\uc774\ub098 \uc5ed\uacbd\uc5d0 \ubd80\ub52a\ud788\ub294 \uc21c\uac04\uc5d0\ub3c4 \ub9d0\uc5c6\uc774 \uac10\uc2f8\uc8fc\ub294 \uc874\uc7ac\uac00 \uc788\ub2e4\uba74 \uc5bc\ub9c8\ub098 \uc88b\uc744\uae4c'\ub77c\ub294 \uc0dd\uac01\uc5d0\uc11c \ube44\ub86f\ub418\uc5c8\ub2e4. \ud0c0\ube14\ub85c\ub294 \ud558\uc9c0\ub9cc \"\uadf8\ub7ec\ud55c \uc874\uc7ac\ub97c \ucc3e\uc9c0 \ubabb\ud574 \ubc29\ud669\ud558\ub294 \uc678\ub85c\uc6b4 \uc0ac\ub78c\ub4e4\uc744 \uc704\ub85c\"\ud558\uae30 \uc704\ud574 \uc774 \uace1\uc744 \uc791\uc5c5\ud558\uc600\ub2e4.", "answer": "\ub098\uc5bc", "sentence": "\ubcf4\uceec \ud53c\uccd0\ub9c1\uc73c\ub85c\ub294 \ud3c9\uc18c \ud0c0\ube14\ub85c\uac00 \"\uc874\uacbd\ud558\ub294 \ubaa9\uc18c\ub9ac\"\ub97c \uac00\uc9c4 \ube0c\ub77c\uc6b4 \uc544\uc774\ub4dc \uc18c\uc6b8\uc758 \uba64\ubc84 \ub098\uc5bc \uc774 \ucc38\uc5ec\ud588\ub2e4.", "paragraph_sentence": "\"Airbag\"\uc740 \ud0c0\ube14\ub85c\uac00 \uc9c1\uc811 \uc791\uace1\uacfc \uc791\uc0ac, \ud3b8\uace1\uc744 \ub9e1\uc558\ub2e4. \ubcf4\uceec \ud53c\uccd0\ub9c1\uc73c\ub85c\ub294 \ud3c9\uc18c \ud0c0\ube14\ub85c\uac00 \"\uc874\uacbd\ud558\ub294 \ubaa9\uc18c\ub9ac\"\ub97c \uac00\uc9c4 \ube0c\ub77c\uc6b4 \uc544\uc774\ub4dc \uc18c\uc6b8\uc758 \uba64\ubc84 \ub098\uc5bc \uc774 \ucc38\uc5ec\ud588\ub2e4. \uc6b4\uc804\uc5d0 \uc11c\ud234\ub7ec \uc0c8\ubcbd\uc5d0 \uc678\ucd9c\ud560 \ub54c \ud0dd\uc2dc\ub97c \ud0c0\uace0 \ub2e4\ub2cc \ud0c0\ube14\ub85c\ub294 \"\ud0dd\uc2dc \uc548\uc5d0\uc11c \uac00\uc7a5 \ub9ce\uc740 \uc0dd\uac01\uc744 \ud558\uac8c \ub418\uace0, \uac00\uc7a5 \ub9ce\uc740 \uac00\uc0ac\ub97c\" \uc4f0\uac8c \ub418\uc5c8\ub2e4. \ud0c0\ube14\ub85c\ub294 \ud0dd\uc2dc \uc548\uc5d0\uc11c \ucc3d\ubb38 \ubc16 \ud48d\uacbd\uc744 \ubc14\ub77c\ubcf4\uace0 \ub77c\ub514\uc624\uc5d0\uc11c \ub098\uc624\ub294 \uc598\uae30\ub4e4\uc744 \ub4e4\uc73c\uba70 \ud55c \uacf5\uac04\uc5d0\uc11c \uc138\uc0c1 \uacf3\uacf3\uc5d0 \uc788\ub294 \uc0ac\ub78c\ub4e4\uc758 \uc678\ub85c\uc6c0\uc744 \ub290\ub07c\uac8c \ub418\uc5b4 \uace1\uc5d0 \ub300\ud55c \uc601\uac10\uc744 \uc5bb\uac8c \ub418\uc5c8\ub2e4. \ud0c0\ube14\ub85c\uac00 \ud0dd\uc2dc \uc548\uc5d0\uc11c \ub290\ub080 \"\uc138\uc0c1\uc758 \ubaa8\ub4e0 \uc678\ub85c\uc6cc \ud558\ub294 \uc0ac\ub78c\ub4e4\uc758 \uc598\uae30\"\ub77c\uace0 \uc0dd\uac01\uc744 \ud558\uace0 \uc4f0\uac8c \ub41c \uace1\uc774\ub2e4. \"Airbag\"\uc774\ub77c\ub294 \uc81c\ubaa9\uacfc \uc8fc\uc81c\uc5d0 \ub300\ud574\uc11c\ub294 \ucc28 \uc548\uc5d0 \uc0ac\uace0\ub97c \ub300\ube44\ud55c \uc5d0\uc5b4\ubc31\uc774 \uc788\ub4ef\uc774, '\uc0ac\ub78c\uc774 \uc2ac\ud514\uc774\ub098 \uc5ed\uacbd\uc5d0 \ubd80\ub52a\ud788\ub294 \uc21c\uac04\uc5d0\ub3c4 \ub9d0\uc5c6\uc774 \uac10\uc2f8\uc8fc\ub294 \uc874\uc7ac\uac00 \uc788\ub2e4\uba74 \uc5bc\ub9c8\ub098 \uc88b\uc744\uae4c'\ub77c\ub294 \uc0dd\uac01\uc5d0\uc11c \ube44\ub86f\ub418\uc5c8\ub2e4. \ud0c0\ube14\ub85c\ub294 \ud558\uc9c0\ub9cc \"\uadf8\ub7ec\ud55c \uc874\uc7ac\ub97c \ucc3e\uc9c0 \ubabb\ud574 \ubc29\ud669\ud558\ub294 \uc678\ub85c\uc6b4 \uc0ac\ub78c\ub4e4\uc744 \uc704\ub85c\"\ud558\uae30 \uc704\ud574 \uc774 \uace1\uc744 \uc791\uc5c5\ud558\uc600\ub2e4.", "paragraph_answer": "\"Airbag\"\uc740 \ud0c0\ube14\ub85c\uac00 \uc9c1\uc811 \uc791\uace1\uacfc \uc791\uc0ac, \ud3b8\uace1\uc744 \ub9e1\uc558\ub2e4. \ubcf4\uceec \ud53c\uccd0\ub9c1\uc73c\ub85c\ub294 \ud3c9\uc18c \ud0c0\ube14\ub85c\uac00 \"\uc874\uacbd\ud558\ub294 \ubaa9\uc18c\ub9ac\"\ub97c \uac00\uc9c4 \ube0c\ub77c\uc6b4 \uc544\uc774\ub4dc \uc18c\uc6b8\uc758 \uba64\ubc84 \ub098\uc5bc \uc774 \ucc38\uc5ec\ud588\ub2e4. \uc6b4\uc804\uc5d0 \uc11c\ud234\ub7ec \uc0c8\ubcbd\uc5d0 \uc678\ucd9c\ud560 \ub54c \ud0dd\uc2dc\ub97c \ud0c0\uace0 \ub2e4\ub2cc \ud0c0\ube14\ub85c\ub294 \"\ud0dd\uc2dc \uc548\uc5d0\uc11c \uac00\uc7a5 \ub9ce\uc740 \uc0dd\uac01\uc744 \ud558\uac8c \ub418\uace0, \uac00\uc7a5 \ub9ce\uc740 \uac00\uc0ac\ub97c\" \uc4f0\uac8c \ub418\uc5c8\ub2e4. \ud0c0\ube14\ub85c\ub294 \ud0dd\uc2dc \uc548\uc5d0\uc11c \ucc3d\ubb38 \ubc16 \ud48d\uacbd\uc744 \ubc14\ub77c\ubcf4\uace0 \ub77c\ub514\uc624\uc5d0\uc11c \ub098\uc624\ub294 \uc598\uae30\ub4e4\uc744 \ub4e4\uc73c\uba70 \ud55c \uacf5\uac04\uc5d0\uc11c \uc138\uc0c1 \uacf3\uacf3\uc5d0 \uc788\ub294 \uc0ac\ub78c\ub4e4\uc758 \uc678\ub85c\uc6c0\uc744 \ub290\ub07c\uac8c \ub418\uc5b4 \uace1\uc5d0 \ub300\ud55c \uc601\uac10\uc744 \uc5bb\uac8c \ub418\uc5c8\ub2e4. \ud0c0\ube14\ub85c\uac00 \ud0dd\uc2dc \uc548\uc5d0\uc11c \ub290\ub080 \"\uc138\uc0c1\uc758 \ubaa8\ub4e0 \uc678\ub85c\uc6cc \ud558\ub294 \uc0ac\ub78c\ub4e4\uc758 \uc598\uae30\"\ub77c\uace0 \uc0dd\uac01\uc744 \ud558\uace0 \uc4f0\uac8c \ub41c \uace1\uc774\ub2e4. \"Airbag\"\uc774\ub77c\ub294 \uc81c\ubaa9\uacfc \uc8fc\uc81c\uc5d0 \ub300\ud574\uc11c\ub294 \ucc28 \uc548\uc5d0 \uc0ac\uace0\ub97c \ub300\ube44\ud55c \uc5d0\uc5b4\ubc31\uc774 \uc788\ub4ef\uc774, '\uc0ac\ub78c\uc774 \uc2ac\ud514\uc774\ub098 \uc5ed\uacbd\uc5d0 \ubd80\ub52a\ud788\ub294 \uc21c\uac04\uc5d0\ub3c4 \ub9d0\uc5c6\uc774 \uac10\uc2f8\uc8fc\ub294 \uc874\uc7ac\uac00 \uc788\ub2e4\uba74 \uc5bc\ub9c8\ub098 \uc88b\uc744\uae4c'\ub77c\ub294 \uc0dd\uac01\uc5d0\uc11c \ube44\ub86f\ub418\uc5c8\ub2e4. \ud0c0\ube14\ub85c\ub294 \ud558\uc9c0\ub9cc \"\uadf8\ub7ec\ud55c \uc874\uc7ac\ub97c \ucc3e\uc9c0 \ubabb\ud574 \ubc29\ud669\ud558\ub294 \uc678\ub85c\uc6b4 \uc0ac\ub78c\ub4e4\uc744 \uc704\ub85c\"\ud558\uae30 \uc704\ud574 \uc774 \uace1\uc744 \uc791\uc5c5\ud558\uc600\ub2e4.", "sentence_answer": "\ubcf4\uceec \ud53c\uccd0\ub9c1\uc73c\ub85c\ub294 \ud3c9\uc18c \ud0c0\ube14\ub85c\uac00 \"\uc874\uacbd\ud558\ub294 \ubaa9\uc18c\ub9ac\"\ub97c \uac00\uc9c4 \ube0c\ub77c\uc6b4 \uc544\uc774\ub4dc \uc18c\uc6b8\uc758 \uba64\ubc84 \ub098\uc5bc \uc774 \ucc38\uc5ec\ud588\ub2e4.", "paragraph_id": "5780446-0-1", "paragraph_question": "question: \"Airbag\" \ubcf4\uceec \ud53c\uccd0\ub9c1\uc740 \ub204\uac00 \ucc38\uc5ec\ud588\ub098\uc694?, context: \"Airbag\"\uc740 \ud0c0\ube14\ub85c\uac00 \uc9c1\uc811 \uc791\uace1\uacfc \uc791\uc0ac, \ud3b8\uace1\uc744 \ub9e1\uc558\ub2e4. \ubcf4\uceec \ud53c\uccd0\ub9c1\uc73c\ub85c\ub294 \ud3c9\uc18c \ud0c0\ube14\ub85c\uac00 \"\uc874\uacbd\ud558\ub294 \ubaa9\uc18c\ub9ac\"\ub97c \uac00\uc9c4 \ube0c\ub77c\uc6b4 \uc544\uc774\ub4dc \uc18c\uc6b8\uc758 \uba64\ubc84 \ub098\uc5bc\uc774 \ucc38\uc5ec\ud588\ub2e4. \uc6b4\uc804\uc5d0 \uc11c\ud234\ub7ec \uc0c8\ubcbd\uc5d0 \uc678\ucd9c\ud560 \ub54c \ud0dd\uc2dc\ub97c \ud0c0\uace0 \ub2e4\ub2cc \ud0c0\ube14\ub85c\ub294 \"\ud0dd\uc2dc \uc548\uc5d0\uc11c \uac00\uc7a5 \ub9ce\uc740 \uc0dd\uac01\uc744 \ud558\uac8c \ub418\uace0, \uac00\uc7a5 \ub9ce\uc740 \uac00\uc0ac\ub97c\" \uc4f0\uac8c \ub418\uc5c8\ub2e4. \ud0c0\ube14\ub85c\ub294 \ud0dd\uc2dc \uc548\uc5d0\uc11c \ucc3d\ubb38 \ubc16 \ud48d\uacbd\uc744 \ubc14\ub77c\ubcf4\uace0 \ub77c\ub514\uc624\uc5d0\uc11c \ub098\uc624\ub294 \uc598\uae30\ub4e4\uc744 \ub4e4\uc73c\uba70 \ud55c \uacf5\uac04\uc5d0\uc11c \uc138\uc0c1 \uacf3\uacf3\uc5d0 \uc788\ub294 \uc0ac\ub78c\ub4e4\uc758 \uc678\ub85c\uc6c0\uc744 \ub290\ub07c\uac8c \ub418\uc5b4 \uace1\uc5d0 \ub300\ud55c \uc601\uac10\uc744 \uc5bb\uac8c \ub418\uc5c8\ub2e4. \ud0c0\ube14\ub85c\uac00 \ud0dd\uc2dc \uc548\uc5d0\uc11c \ub290\ub080 \"\uc138\uc0c1\uc758 \ubaa8\ub4e0 \uc678\ub85c\uc6cc \ud558\ub294 \uc0ac\ub78c\ub4e4\uc758 \uc598\uae30\"\ub77c\uace0 \uc0dd\uac01\uc744 \ud558\uace0 \uc4f0\uac8c \ub41c \uace1\uc774\ub2e4. \"Airbag\"\uc774\ub77c\ub294 \uc81c\ubaa9\uacfc \uc8fc\uc81c\uc5d0 \ub300\ud574\uc11c\ub294 \ucc28 \uc548\uc5d0 \uc0ac\uace0\ub97c \ub300\ube44\ud55c \uc5d0\uc5b4\ubc31\uc774 \uc788\ub4ef\uc774, '\uc0ac\ub78c\uc774 \uc2ac\ud514\uc774\ub098 \uc5ed\uacbd\uc5d0 \ubd80\ub52a\ud788\ub294 \uc21c\uac04\uc5d0\ub3c4 \ub9d0\uc5c6\uc774 \uac10\uc2f8\uc8fc\ub294 \uc874\uc7ac\uac00 \uc788\ub2e4\uba74 \uc5bc\ub9c8\ub098 \uc88b\uc744\uae4c'\ub77c\ub294 \uc0dd\uac01\uc5d0\uc11c \ube44\ub86f\ub418\uc5c8\ub2e4. \ud0c0\ube14\ub85c\ub294 \ud558\uc9c0\ub9cc \"\uadf8\ub7ec\ud55c \uc874\uc7ac\ub97c \ucc3e\uc9c0 \ubabb\ud574 \ubc29\ud669\ud558\ub294 \uc678\ub85c\uc6b4 \uc0ac\ub78c\ub4e4\uc744 \uc704\ub85c\"\ud558\uae30 \uc704\ud574 \uc774 \uace1\uc744 \uc791\uc5c5\ud558\uc600\ub2e4."} {"question": "\uac00\uc2b5\uae30 \uc0b4\uade0\uc81c\ub294 \uc5b4\ub5a4\uac83\uc73c\ub85c \ubd84\ub958\ub418\uc5b4 \ud488\uc9c8\uacbd\uc601 \ubc0f \uacf5\uc0b0\ud488\uc548\uc804\uad00\ub9ac\ubc95\uc5d0 \ub530\ub978 \uc548\uc804\uae30\uc900\uc774 \uc801\uc6a9\ub418\uc5c8\ub098?", "paragraph": "\uac00\uc2b5\uae30 \uc0b4\uade0\uc81c\uc758 \uc0b4\uade0\uc81c \uc131\ubd84\uc740 \uc8fc\ub85c \ud3f4\ub9ac\ud5e5\uc0ac\uba54\ud2f8\ub80c \uad6c\uc544\ub2c8\ub518(polyhexamethylene guanidine; PHMG)\uacfc \uc5fc\ud654 \uc62c\ub9ac\uace0-(\ud639\uc740 2-)\uc5d0\ud1a1\uc2dc\uc5d0\ud2f8 \uad6c\uc544\ub2c8\ub518(Oligo(2-)ethoxy ethoxyethyl guanidine chloride; PGH)\uc774\uace0, \uba54\ud2f8\ud074\ub85c\ub85c\uc774\uc18c\uce58\uc544\uc878\ub9ac\ub17c(Methylchloroisothiazolinone; MCI; MCIT)\uc744 \uc774\uc6a9\ud558\ub294 \uacbd\uc6b0\ub3c4 \uc788\ub2e4. \uc774\ub4e4 \ubb3c\uc9c8\uc740 \ud53c\ubd80\ub3c5\uc131\uc774 \ub2e4\ub978 \uc0b4\uade0\uc81c\uc5d0 \ube44\ud574 5~10\ubd84\uc758 1 \uc815\ub3c4\uc5d0 \ubd88\uacfc\ud574\uc11c \uac00\uc2b5\uae30 \uc0b4\uade0\uc81c \ubfd0 \uc544\ub2c8\ub77c, \uc0f4\ud478, \ubb3c\ud2f0\uc288 \ub4f1 \uc5ec\ub7ec \uac00\uc9c0 \uc81c\ud488\uc5d0 \uc774\uc6a9\ub41c\ub2e4. \ud558\uc9c0\ub9cc \uc774\ub4e4 \uc131\ubd84\uc774 \ud638\ud761\uae30\ub85c \ud761\uc785\ub420 \ub54c \ubc1c\uc0dd\ud558\ub294 \ub3c5\uc131\uc5d0 \ub300\ud574\uc11c\ub294 \uc5f0\uad6c\uac00 \ub418\uc9c0 \uc54a\uc558\uae30 \ub54c\ubb38\uc5d0, \ud53c\ud574\uc790\uac00 \ubc1c\uc0dd\ud560 \ub54c\uae4c\uc9c0 \uc544\ubb34\ub7f0 \uc81c\uc7ac\uac00 \uc774\ub8e8\uc5b4\uc9c0\uc9c0 \uc54a\uc558\ub2e4. \ud2b9\ud788, \uac00\uc2b5\uae30 \uc0b4\uade0\uc81c\ub294 \uacf5\uc0b0\ud488\uc73c\ub85c \ubd84\ub958\ub418\uc5c8\uae30 \ub54c\ubb38\uc5d0 \uc2dd\ud488\uc704\uc0dd\ubc95\uc774\ub098 \uc57d\uc0ac\ubc95\uc774 \uc544\ub2cc \u201c\ud488\uc9c8\uacbd\uc601 \ubc0f \uacf5\uc0b0\ud488\uc548\uc804\uad00\ub9ac\ubc95\u201d\uc5d0 \ub530\ub978 \uc77c\ubc18\uc801\uc778 \uc548\uc804\uae30\uc900\ub9cc\uc774 \uc801\uc6a9\ub418\uc5b4 \ud53c\ud574\ub97c \uc608\ubc29\ud558\uc9c0 \ubabb\ud588\ub2e4.", "answer": "\uacf5\uc0b0\ud488", "sentence": "\ud2b9\ud788, \uac00\uc2b5\uae30 \uc0b4\uade0\uc81c\ub294 \uacf5\uc0b0\ud488 \uc73c\ub85c \ubd84\ub958\ub418\uc5c8\uae30 \ub54c\ubb38\uc5d0 \uc2dd\ud488\uc704\uc0dd\ubc95\uc774\ub098 \uc57d\uc0ac\ubc95\uc774 \uc544\ub2cc \u201c\ud488\uc9c8\uacbd\uc601 \ubc0f \uacf5\uc0b0\ud488\uc548\uc804\uad00\ub9ac\ubc95\u201d\uc5d0 \ub530\ub978 \uc77c\ubc18\uc801\uc778 \uc548\uc804\uae30\uc900\ub9cc\uc774 \uc801\uc6a9\ub418\uc5b4 \ud53c\ud574\ub97c \uc608\ubc29\ud558\uc9c0 \ubabb\ud588\ub2e4.", "paragraph_sentence": "\uac00\uc2b5\uae30 \uc0b4\uade0\uc81c\uc758 \uc0b4\uade0\uc81c \uc131\ubd84\uc740 \uc8fc\ub85c \ud3f4\ub9ac\ud5e5\uc0ac\uba54\ud2f8\ub80c \uad6c\uc544\ub2c8\ub518(polyhexamethylene guanidine; PHMG)\uacfc \uc5fc\ud654 \uc62c\ub9ac\uace0-(\ud639\uc740 2-)\uc5d0\ud1a1\uc2dc\uc5d0\ud2f8 \uad6c\uc544\ub2c8\ub518(Oligo(2-)ethoxy ethoxyethyl guanidine chloride; PGH)\uc774\uace0, \uba54\ud2f8\ud074\ub85c\ub85c\uc774\uc18c\uce58\uc544\uc878\ub9ac\ub17c(Methylchloroisothiazolinone; MCI; MCIT)\uc744 \uc774\uc6a9\ud558\ub294 \uacbd\uc6b0\ub3c4 \uc788\ub2e4. \uc774\ub4e4 \ubb3c\uc9c8\uc740 \ud53c\ubd80\ub3c5\uc131\uc774 \ub2e4\ub978 \uc0b4\uade0\uc81c\uc5d0 \ube44\ud574 5~10\ubd84\uc758 1 \uc815\ub3c4\uc5d0 \ubd88\uacfc\ud574\uc11c \uac00\uc2b5\uae30 \uc0b4\uade0\uc81c \ubfd0 \uc544\ub2c8\ub77c, \uc0f4\ud478, \ubb3c\ud2f0\uc288 \ub4f1 \uc5ec\ub7ec \uac00\uc9c0 \uc81c\ud488\uc5d0 \uc774\uc6a9\ub41c\ub2e4. \ud558\uc9c0\ub9cc \uc774\ub4e4 \uc131\ubd84\uc774 \ud638\ud761\uae30\ub85c \ud761\uc785\ub420 \ub54c \ubc1c\uc0dd\ud558\ub294 \ub3c5\uc131\uc5d0 \ub300\ud574\uc11c\ub294 \uc5f0\uad6c\uac00 \ub418\uc9c0 \uc54a\uc558\uae30 \ub54c\ubb38\uc5d0, \ud53c\ud574\uc790\uac00 \ubc1c\uc0dd\ud560 \ub54c\uae4c\uc9c0 \uc544\ubb34\ub7f0 \uc81c\uc7ac\uac00 \uc774\ub8e8\uc5b4\uc9c0\uc9c0 \uc54a\uc558\ub2e4. \ud2b9\ud788, \uac00\uc2b5\uae30 \uc0b4\uade0\uc81c\ub294 \uacf5\uc0b0\ud488 \uc73c\ub85c \ubd84\ub958\ub418\uc5c8\uae30 \ub54c\ubb38\uc5d0 \uc2dd\ud488\uc704\uc0dd\ubc95\uc774\ub098 \uc57d\uc0ac\ubc95\uc774 \uc544\ub2cc \u201c\ud488\uc9c8\uacbd\uc601 \ubc0f \uacf5\uc0b0\ud488\uc548\uc804\uad00\ub9ac\ubc95\u201d\uc5d0 \ub530\ub978 \uc77c\ubc18\uc801\uc778 \uc548\uc804\uae30\uc900\ub9cc\uc774 \uc801\uc6a9\ub418\uc5b4 \ud53c\ud574\ub97c \uc608\ubc29\ud558\uc9c0 \ubabb\ud588\ub2e4. ", "paragraph_answer": "\uac00\uc2b5\uae30 \uc0b4\uade0\uc81c\uc758 \uc0b4\uade0\uc81c \uc131\ubd84\uc740 \uc8fc\ub85c \ud3f4\ub9ac\ud5e5\uc0ac\uba54\ud2f8\ub80c \uad6c\uc544\ub2c8\ub518(polyhexamethylene guanidine; PHMG)\uacfc \uc5fc\ud654 \uc62c\ub9ac\uace0-(\ud639\uc740 2-)\uc5d0\ud1a1\uc2dc\uc5d0\ud2f8 \uad6c\uc544\ub2c8\ub518(Oligo(2-)ethoxy ethoxyethyl guanidine chloride; PGH)\uc774\uace0, \uba54\ud2f8\ud074\ub85c\ub85c\uc774\uc18c\uce58\uc544\uc878\ub9ac\ub17c(Methylchloroisothiazolinone; MCI; MCIT)\uc744 \uc774\uc6a9\ud558\ub294 \uacbd\uc6b0\ub3c4 \uc788\ub2e4. \uc774\ub4e4 \ubb3c\uc9c8\uc740 \ud53c\ubd80\ub3c5\uc131\uc774 \ub2e4\ub978 \uc0b4\uade0\uc81c\uc5d0 \ube44\ud574 5~10\ubd84\uc758 1 \uc815\ub3c4\uc5d0 \ubd88\uacfc\ud574\uc11c \uac00\uc2b5\uae30 \uc0b4\uade0\uc81c \ubfd0 \uc544\ub2c8\ub77c, \uc0f4\ud478, \ubb3c\ud2f0\uc288 \ub4f1 \uc5ec\ub7ec \uac00\uc9c0 \uc81c\ud488\uc5d0 \uc774\uc6a9\ub41c\ub2e4. \ud558\uc9c0\ub9cc \uc774\ub4e4 \uc131\ubd84\uc774 \ud638\ud761\uae30\ub85c \ud761\uc785\ub420 \ub54c \ubc1c\uc0dd\ud558\ub294 \ub3c5\uc131\uc5d0 \ub300\ud574\uc11c\ub294 \uc5f0\uad6c\uac00 \ub418\uc9c0 \uc54a\uc558\uae30 \ub54c\ubb38\uc5d0, \ud53c\ud574\uc790\uac00 \ubc1c\uc0dd\ud560 \ub54c\uae4c\uc9c0 \uc544\ubb34\ub7f0 \uc81c\uc7ac\uac00 \uc774\ub8e8\uc5b4\uc9c0\uc9c0 \uc54a\uc558\ub2e4. \ud2b9\ud788, \uac00\uc2b5\uae30 \uc0b4\uade0\uc81c\ub294 \uacf5\uc0b0\ud488 \uc73c\ub85c \ubd84\ub958\ub418\uc5c8\uae30 \ub54c\ubb38\uc5d0 \uc2dd\ud488\uc704\uc0dd\ubc95\uc774\ub098 \uc57d\uc0ac\ubc95\uc774 \uc544\ub2cc \u201c\ud488\uc9c8\uacbd\uc601 \ubc0f \uacf5\uc0b0\ud488\uc548\uc804\uad00\ub9ac\ubc95\u201d\uc5d0 \ub530\ub978 \uc77c\ubc18\uc801\uc778 \uc548\uc804\uae30\uc900\ub9cc\uc774 \uc801\uc6a9\ub418\uc5b4 \ud53c\ud574\ub97c \uc608\ubc29\ud558\uc9c0 \ubabb\ud588\ub2e4.", "sentence_answer": "\ud2b9\ud788, \uac00\uc2b5\uae30 \uc0b4\uade0\uc81c\ub294 \uacf5\uc0b0\ud488 \uc73c\ub85c \ubd84\ub958\ub418\uc5c8\uae30 \ub54c\ubb38\uc5d0 \uc2dd\ud488\uc704\uc0dd\ubc95\uc774\ub098 \uc57d\uc0ac\ubc95\uc774 \uc544\ub2cc \u201c\ud488\uc9c8\uacbd\uc601 \ubc0f \uacf5\uc0b0\ud488\uc548\uc804\uad00\ub9ac\ubc95\u201d\uc5d0 \ub530\ub978 \uc77c\ubc18\uc801\uc778 \uc548\uc804\uae30\uc900\ub9cc\uc774 \uc801\uc6a9\ub418\uc5b4 \ud53c\ud574\ub97c \uc608\ubc29\ud558\uc9c0 \ubabb\ud588\ub2e4.", "paragraph_id": "6527118-0-1", "paragraph_question": "question: \uac00\uc2b5\uae30 \uc0b4\uade0\uc81c\ub294 \uc5b4\ub5a4\uac83\uc73c\ub85c \ubd84\ub958\ub418\uc5b4 \ud488\uc9c8\uacbd\uc601 \ubc0f \uacf5\uc0b0\ud488\uc548\uc804\uad00\ub9ac\ubc95\uc5d0 \ub530\ub978 \uc548\uc804\uae30\uc900\uc774 \uc801\uc6a9\ub418\uc5c8\ub098?, context: \uac00\uc2b5\uae30 \uc0b4\uade0\uc81c\uc758 \uc0b4\uade0\uc81c \uc131\ubd84\uc740 \uc8fc\ub85c \ud3f4\ub9ac\ud5e5\uc0ac\uba54\ud2f8\ub80c \uad6c\uc544\ub2c8\ub518(polyhexamethylene guanidine; PHMG)\uacfc \uc5fc\ud654 \uc62c\ub9ac\uace0-(\ud639\uc740 2-)\uc5d0\ud1a1\uc2dc\uc5d0\ud2f8 \uad6c\uc544\ub2c8\ub518(Oligo(2-)ethoxy ethoxyethyl guanidine chloride; PGH)\uc774\uace0, \uba54\ud2f8\ud074\ub85c\ub85c\uc774\uc18c\uce58\uc544\uc878\ub9ac\ub17c(Methylchloroisothiazolinone; MCI; MCIT)\uc744 \uc774\uc6a9\ud558\ub294 \uacbd\uc6b0\ub3c4 \uc788\ub2e4. \uc774\ub4e4 \ubb3c\uc9c8\uc740 \ud53c\ubd80\ub3c5\uc131\uc774 \ub2e4\ub978 \uc0b4\uade0\uc81c\uc5d0 \ube44\ud574 5~10\ubd84\uc758 1 \uc815\ub3c4\uc5d0 \ubd88\uacfc\ud574\uc11c \uac00\uc2b5\uae30 \uc0b4\uade0\uc81c \ubfd0 \uc544\ub2c8\ub77c, \uc0f4\ud478, \ubb3c\ud2f0\uc288 \ub4f1 \uc5ec\ub7ec \uac00\uc9c0 \uc81c\ud488\uc5d0 \uc774\uc6a9\ub41c\ub2e4. \ud558\uc9c0\ub9cc \uc774\ub4e4 \uc131\ubd84\uc774 \ud638\ud761\uae30\ub85c \ud761\uc785\ub420 \ub54c \ubc1c\uc0dd\ud558\ub294 \ub3c5\uc131\uc5d0 \ub300\ud574\uc11c\ub294 \uc5f0\uad6c\uac00 \ub418\uc9c0 \uc54a\uc558\uae30 \ub54c\ubb38\uc5d0, \ud53c\ud574\uc790\uac00 \ubc1c\uc0dd\ud560 \ub54c\uae4c\uc9c0 \uc544\ubb34\ub7f0 \uc81c\uc7ac\uac00 \uc774\ub8e8\uc5b4\uc9c0\uc9c0 \uc54a\uc558\ub2e4. \ud2b9\ud788, \uac00\uc2b5\uae30 \uc0b4\uade0\uc81c\ub294 \uacf5\uc0b0\ud488\uc73c\ub85c \ubd84\ub958\ub418\uc5c8\uae30 \ub54c\ubb38\uc5d0 \uc2dd\ud488\uc704\uc0dd\ubc95\uc774\ub098 \uc57d\uc0ac\ubc95\uc774 \uc544\ub2cc \u201c\ud488\uc9c8\uacbd\uc601 \ubc0f \uacf5\uc0b0\ud488\uc548\uc804\uad00\ub9ac\ubc95\u201d\uc5d0 \ub530\ub978 \uc77c\ubc18\uc801\uc778 \uc548\uc804\uae30\uc900\ub9cc\uc774 \uc801\uc6a9\ub418\uc5b4 \ud53c\ud574\ub97c \uc608\ubc29\ud558\uc9c0 \ubabb\ud588\ub2e4."} {"question": "2008\ub144 10\uc6d4 \ud604\uc7ac, \uc720\ub8cc\ud68c\uc6d0\uc758 \uaddc\ubaa8\ub294 2001\ub144 6\uc6d4 \uc11c\ube44\uc2a4\ub97c \uc2dc\uc791\ud588\uc744 \ub54c\uc5d0 \ube44\ud574 \uba87 \ubc30 \uc774\uc0c1 \uc99d\uac00\ud558\uc600\ub294\uac00?", "paragraph": "\uc544\ud2b8\uc564\uc2a4\ud130\ub514\ub294 \ub300\ud55c\ubbfc\uad6d\uc758 \uae30\uc5c5\uc73c\ub85c, \uc778\ubb38\ud559\uacfc \ubb38\ud654\uc608\uc220\uc758 \uc800\ubcc0 \ud655\ub300\ub97c \uc704\ud574 2000\ub144\uc5d0 \uc124\ub9bd\ub418\uc5c8\ub2e4. \ub300\ud55c\ubbfc\uad6d \ucd5c\ucd08\uc758 \uc778\ubb38\ud559 \ud3ec\ud138 \uc0ac\uc774\ud2b8 '\uc544\ud2b8\uc564\uc2a4\ud130\ub514'\ub97c 2001\ub144\uc5d0 \uac1c\uc124\ud558\uc5ec \uc6b4\uc601\ud558\uc600\ub2e4. '\uc778\ubb38\ud559\uc758 \uc704\uae30'\ub77c\ub294 \ub9d0\uc774 \uc778\uad6c\uc5d0 \ud68c\uc790\ub418\ub294 \uc640\uc911\uc5d0 \uc628\ub77c\uc778 \uac15\uc88c\uac00 \uc778\ubb38\ud559 \ub300\uc911\ud654\uc758 \ucc3d\uad6c\ub85c \uc790\ub9ac\ub97c \uc7a1\uc544\uac00\uace0, \uc778\ud130\ub137\ub9cc \uc5f0\uacb0\ud558\uba74 \uc544\ubb34 \ub54c\uace0 \uac15\uc758\ub97c \ub4e4\uc744 \uc218 \uc788\ub294 \ub370\ub2e4 \uc624\ud504\ub77c\uc778 \uac15\uc88c\ubcf4\ub2e4 \uc218\uac15\ub8cc\ub3c4 \uc2f8\ub2e4\ub294 \uc7a5\uc810 \ub4f1\uc5d0 \ud798\uc785\uc5b4 \uae09\uc131\uc7a5\ud558\uc600\uace0 \uc774\uc5d0 \uc544\ud2b8\uc564\uc2a4\ud130\ub514 \uac15\uc758\ub3c4 \ub9ce\uc740 \uc0ac\ub791\uc744 \ubc1b\uc558\ub2e4. 2008\ub144 10\uc6d4 \ud604\uc7ac, \uc720\ub8cc\ud68c\uc6d0 \uaddc\ubaa8\ub294 2001\ub144 6\uc6d4 \uc11c\ube44\uc2a4\ub97c \ucc98\uc74c \uc2dc\uc791\ud588\uc744 \ub54c 300\uc5ec \uba85\uc5d0\uc11c 100\ubc30 \uc774\uc0c1 \uc99d\uac00\ud558\uc600\uace0 30\uac1c\ub85c \uc2dc\uc791\ud588\ub358 \uac15\uc88c\ub294 170\uc5ec \uac1c\ub85c \ub298\uc5c8\uace0 \uc5f0\uac04 1000\ub9cc \uc6d0\ub3c4 \ub418\uc9c0 \uc54a\uc558\ub358 \ub9e4\ucd9c \uaddc\ubaa8\ub294 \uc5f0\uac04 6\uc5b5 \uc6d0 \uc218\uc900\uc73c\ub85c \ub6f0\uc5b4\uc62c\ub790\ub2e4.", "answer": "100\ubc30", "sentence": "2008\ub144 10\uc6d4 \ud604\uc7ac, \uc720\ub8cc\ud68c\uc6d0 \uaddc\ubaa8\ub294 2001\ub144 6\uc6d4 \uc11c\ube44\uc2a4\ub97c \ucc98\uc74c \uc2dc\uc791\ud588\uc744 \ub54c 300\uc5ec \uba85\uc5d0\uc11c 100\ubc30 \uc774\uc0c1 \uc99d\uac00\ud558\uc600\uace0 30\uac1c\ub85c \uc2dc\uc791\ud588\ub358 \uac15\uc88c\ub294 170\uc5ec \uac1c\ub85c \ub298\uc5c8\uace0 \uc5f0\uac04 1000\ub9cc \uc6d0\ub3c4 \ub418\uc9c0 \uc54a\uc558\ub358 \ub9e4\ucd9c \uaddc\ubaa8\ub294 \uc5f0\uac04 6\uc5b5 \uc6d0 \uc218\uc900\uc73c\ub85c \ub6f0\uc5b4\uc62c\ub790\ub2e4.", "paragraph_sentence": "\uc544\ud2b8\uc564\uc2a4\ud130\ub514\ub294 \ub300\ud55c\ubbfc\uad6d\uc758 \uae30\uc5c5\uc73c\ub85c, \uc778\ubb38\ud559\uacfc \ubb38\ud654\uc608\uc220\uc758 \uc800\ubcc0 \ud655\ub300\ub97c \uc704\ud574 2000\ub144\uc5d0 \uc124\ub9bd\ub418\uc5c8\ub2e4. \ub300\ud55c\ubbfc\uad6d \ucd5c\ucd08\uc758 \uc778\ubb38\ud559 \ud3ec\ud138 \uc0ac\uc774\ud2b8 '\uc544\ud2b8\uc564\uc2a4\ud130\ub514'\ub97c 2001\ub144\uc5d0 \uac1c\uc124\ud558\uc5ec \uc6b4\uc601\ud558\uc600\ub2e4. '\uc778\ubb38\ud559\uc758 \uc704\uae30'\ub77c\ub294 \ub9d0\uc774 \uc778\uad6c\uc5d0 \ud68c\uc790\ub418\ub294 \uc640\uc911\uc5d0 \uc628\ub77c\uc778 \uac15\uc88c\uac00 \uc778\ubb38\ud559 \ub300\uc911\ud654\uc758 \ucc3d\uad6c\ub85c \uc790\ub9ac\ub97c \uc7a1\uc544\uac00\uace0, \uc778\ud130\ub137\ub9cc \uc5f0\uacb0\ud558\uba74 \uc544\ubb34 \ub54c\uace0 \uac15\uc758\ub97c \ub4e4\uc744 \uc218 \uc788\ub294 \ub370\ub2e4 \uc624\ud504\ub77c\uc778 \uac15\uc88c\ubcf4\ub2e4 \uc218\uac15\ub8cc\ub3c4 \uc2f8\ub2e4\ub294 \uc7a5\uc810 \ub4f1\uc5d0 \ud798\uc785\uc5b4 \uae09\uc131\uc7a5\ud558\uc600\uace0 \uc774\uc5d0 \uc544\ud2b8\uc564\uc2a4\ud130\ub514 \uac15\uc758\ub3c4 \ub9ce\uc740 \uc0ac\ub791\uc744 \ubc1b\uc558\ub2e4. 2008\ub144 10\uc6d4 \ud604\uc7ac, \uc720\ub8cc\ud68c\uc6d0 \uaddc\ubaa8\ub294 2001\ub144 6\uc6d4 \uc11c\ube44\uc2a4\ub97c \ucc98\uc74c \uc2dc\uc791\ud588\uc744 \ub54c 300\uc5ec \uba85\uc5d0\uc11c 100\ubc30 \uc774\uc0c1 \uc99d\uac00\ud558\uc600\uace0 30\uac1c\ub85c \uc2dc\uc791\ud588\ub358 \uac15\uc88c\ub294 170\uc5ec \uac1c\ub85c \ub298\uc5c8\uace0 \uc5f0\uac04 1000\ub9cc \uc6d0\ub3c4 \ub418\uc9c0 \uc54a\uc558\ub358 \ub9e4\ucd9c \uaddc\ubaa8\ub294 \uc5f0\uac04 6\uc5b5 \uc6d0 \uc218\uc900\uc73c\ub85c \ub6f0\uc5b4\uc62c\ub790\ub2e4. ", "paragraph_answer": "\uc544\ud2b8\uc564\uc2a4\ud130\ub514\ub294 \ub300\ud55c\ubbfc\uad6d\uc758 \uae30\uc5c5\uc73c\ub85c, \uc778\ubb38\ud559\uacfc \ubb38\ud654\uc608\uc220\uc758 \uc800\ubcc0 \ud655\ub300\ub97c \uc704\ud574 2000\ub144\uc5d0 \uc124\ub9bd\ub418\uc5c8\ub2e4. \ub300\ud55c\ubbfc\uad6d \ucd5c\ucd08\uc758 \uc778\ubb38\ud559 \ud3ec\ud138 \uc0ac\uc774\ud2b8 '\uc544\ud2b8\uc564\uc2a4\ud130\ub514'\ub97c 2001\ub144\uc5d0 \uac1c\uc124\ud558\uc5ec \uc6b4\uc601\ud558\uc600\ub2e4. '\uc778\ubb38\ud559\uc758 \uc704\uae30'\ub77c\ub294 \ub9d0\uc774 \uc778\uad6c\uc5d0 \ud68c\uc790\ub418\ub294 \uc640\uc911\uc5d0 \uc628\ub77c\uc778 \uac15\uc88c\uac00 \uc778\ubb38\ud559 \ub300\uc911\ud654\uc758 \ucc3d\uad6c\ub85c \uc790\ub9ac\ub97c \uc7a1\uc544\uac00\uace0, \uc778\ud130\ub137\ub9cc \uc5f0\uacb0\ud558\uba74 \uc544\ubb34 \ub54c\uace0 \uac15\uc758\ub97c \ub4e4\uc744 \uc218 \uc788\ub294 \ub370\ub2e4 \uc624\ud504\ub77c\uc778 \uac15\uc88c\ubcf4\ub2e4 \uc218\uac15\ub8cc\ub3c4 \uc2f8\ub2e4\ub294 \uc7a5\uc810 \ub4f1\uc5d0 \ud798\uc785\uc5b4 \uae09\uc131\uc7a5\ud558\uc600\uace0 \uc774\uc5d0 \uc544\ud2b8\uc564\uc2a4\ud130\ub514 \uac15\uc758\ub3c4 \ub9ce\uc740 \uc0ac\ub791\uc744 \ubc1b\uc558\ub2e4. 2008\ub144 10\uc6d4 \ud604\uc7ac, \uc720\ub8cc\ud68c\uc6d0 \uaddc\ubaa8\ub294 2001\ub144 6\uc6d4 \uc11c\ube44\uc2a4\ub97c \ucc98\uc74c \uc2dc\uc791\ud588\uc744 \ub54c 300\uc5ec \uba85\uc5d0\uc11c 100\ubc30 \uc774\uc0c1 \uc99d\uac00\ud558\uc600\uace0 30\uac1c\ub85c \uc2dc\uc791\ud588\ub358 \uac15\uc88c\ub294 170\uc5ec \uac1c\ub85c \ub298\uc5c8\uace0 \uc5f0\uac04 1000\ub9cc \uc6d0\ub3c4 \ub418\uc9c0 \uc54a\uc558\ub358 \ub9e4\ucd9c \uaddc\ubaa8\ub294 \uc5f0\uac04 6\uc5b5 \uc6d0 \uc218\uc900\uc73c\ub85c \ub6f0\uc5b4\uc62c\ub790\ub2e4.", "sentence_answer": "2008\ub144 10\uc6d4 \ud604\uc7ac, \uc720\ub8cc\ud68c\uc6d0 \uaddc\ubaa8\ub294 2001\ub144 6\uc6d4 \uc11c\ube44\uc2a4\ub97c \ucc98\uc74c \uc2dc\uc791\ud588\uc744 \ub54c 300\uc5ec \uba85\uc5d0\uc11c 100\ubc30 \uc774\uc0c1 \uc99d\uac00\ud558\uc600\uace0 30\uac1c\ub85c \uc2dc\uc791\ud588\ub358 \uac15\uc88c\ub294 170\uc5ec \uac1c\ub85c \ub298\uc5c8\uace0 \uc5f0\uac04 1000\ub9cc \uc6d0\ub3c4 \ub418\uc9c0 \uc54a\uc558\ub358 \ub9e4\ucd9c \uaddc\ubaa8\ub294 \uc5f0\uac04 6\uc5b5 \uc6d0 \uc218\uc900\uc73c\ub85c \ub6f0\uc5b4\uc62c\ub790\ub2e4.", "paragraph_id": "6457685-0-1", "paragraph_question": "question: 2008\ub144 10\uc6d4 \ud604\uc7ac, \uc720\ub8cc\ud68c\uc6d0\uc758 \uaddc\ubaa8\ub294 2001\ub144 6\uc6d4 \uc11c\ube44\uc2a4\ub97c \uc2dc\uc791\ud588\uc744 \ub54c\uc5d0 \ube44\ud574 \uba87 \ubc30 \uc774\uc0c1 \uc99d\uac00\ud558\uc600\ub294\uac00?, context: \uc544\ud2b8\uc564\uc2a4\ud130\ub514\ub294 \ub300\ud55c\ubbfc\uad6d\uc758 \uae30\uc5c5\uc73c\ub85c, \uc778\ubb38\ud559\uacfc \ubb38\ud654\uc608\uc220\uc758 \uc800\ubcc0 \ud655\ub300\ub97c \uc704\ud574 2000\ub144\uc5d0 \uc124\ub9bd\ub418\uc5c8\ub2e4. \ub300\ud55c\ubbfc\uad6d \ucd5c\ucd08\uc758 \uc778\ubb38\ud559 \ud3ec\ud138 \uc0ac\uc774\ud2b8 '\uc544\ud2b8\uc564\uc2a4\ud130\ub514'\ub97c 2001\ub144\uc5d0 \uac1c\uc124\ud558\uc5ec \uc6b4\uc601\ud558\uc600\ub2e4. '\uc778\ubb38\ud559\uc758 \uc704\uae30'\ub77c\ub294 \ub9d0\uc774 \uc778\uad6c\uc5d0 \ud68c\uc790\ub418\ub294 \uc640\uc911\uc5d0 \uc628\ub77c\uc778 \uac15\uc88c\uac00 \uc778\ubb38\ud559 \ub300\uc911\ud654\uc758 \ucc3d\uad6c\ub85c \uc790\ub9ac\ub97c \uc7a1\uc544\uac00\uace0, \uc778\ud130\ub137\ub9cc \uc5f0\uacb0\ud558\uba74 \uc544\ubb34 \ub54c\uace0 \uac15\uc758\ub97c \ub4e4\uc744 \uc218 \uc788\ub294 \ub370\ub2e4 \uc624\ud504\ub77c\uc778 \uac15\uc88c\ubcf4\ub2e4 \uc218\uac15\ub8cc\ub3c4 \uc2f8\ub2e4\ub294 \uc7a5\uc810 \ub4f1\uc5d0 \ud798\uc785\uc5b4 \uae09\uc131\uc7a5\ud558\uc600\uace0 \uc774\uc5d0 \uc544\ud2b8\uc564\uc2a4\ud130\ub514 \uac15\uc758\ub3c4 \ub9ce\uc740 \uc0ac\ub791\uc744 \ubc1b\uc558\ub2e4. 2008\ub144 10\uc6d4 \ud604\uc7ac, \uc720\ub8cc\ud68c\uc6d0 \uaddc\ubaa8\ub294 2001\ub144 6\uc6d4 \uc11c\ube44\uc2a4\ub97c \ucc98\uc74c \uc2dc\uc791\ud588\uc744 \ub54c 300\uc5ec \uba85\uc5d0\uc11c 100\ubc30 \uc774\uc0c1 \uc99d\uac00\ud558\uc600\uace0 30\uac1c\ub85c \uc2dc\uc791\ud588\ub358 \uac15\uc88c\ub294 170\uc5ec \uac1c\ub85c \ub298\uc5c8\uace0 \uc5f0\uac04 1000\ub9cc \uc6d0\ub3c4 \ub418\uc9c0 \uc54a\uc558\ub358 \ub9e4\ucd9c \uaddc\ubaa8\ub294 \uc5f0\uac04 6\uc5b5 \uc6d0 \uc218\uc900\uc73c\ub85c \ub6f0\uc5b4\uc62c\ub790\ub2e4."} {"question": "\uc774\ub9c8\ub098\uce74 \uc2e0\uc9c0\ub294 1996\ub144 \uc62c\uc2a4\ud0c0\uc804 \ucd9c\uc804\uc744 \ud3ec\uae30\ud55c \uac83\uc740 \ubb34\uc5c7\uc774 \ub9d0\uc18c\ub418\uc5c8\uae30 \ub54c\ubb38\uc778\uac00?", "paragraph": "1996\ub144, 4\ub144 \uc5f0\uc18d\uc73c\ub85c \uac1c\ub9c9\uc804 \uc120\ubc1c \ud22c\uc218\ub97c \ub9e1\uc544 6\uc6d4 27\uc77c \ub098\uace0\uc57c \uad6c\uc7a5\uc5d0\uc11c \uc5f4\ub9b0 \ud788\ub85c\uc2dc\ub9c8\uc804\uc5d0\uc11c \uad6c\ub2e8 \uc5ed\uc0ac\uc0c1 \uac00\uc7a5 \ube60\ub978 186\uacbd\uae30\uc5d0\uc11c \ud1b5\uc0b0 1000\ud0c8\uc0bc\uc9c4\uc744 \ub2ec\uc131\ud558\ub294 \ub4f1\uc758 \ud65c\uc57d\uc73c\ub85c 7\uc6d4\uae4c\uc9c0 9\uc2b9\uc744 \uc62c\ub9ac\uace0 \uc788\ub2e4. \uadf8\ub7ec\ub098 7\uc6d4 16\uc77c \ub3c4\ucfc4 \ub3d4\uc5d0\uc11c \uc5f4\ub9b0 \uc694\ubbf8\uc6b0\ub9ac\uc804\uc5d0\uc11c \ud504\ub85c \ub370\ubdd4 \ud6c4 \uac00\uc7a5 \uc9e7\uc740 1/3\uc774\ub2dd\uc5d0 \uac15\ud310\ub3fc \uc67c\ucabd \uc5b4\uae68 \ud1b5\uc99d\uc744 \ud638\uc18c\ud588\ub2e4. \uc774\ud2bf\ub0a0\uc758 \uc815\ubc00 \uac80\uc0ac\uc5d0\uc11c \uc67c\ucabd \uc5b4\uae68\uad00\uc808 \uc8fc\uc704 \uc5fc\uc99d \ub54c\ubb38\uc5d0 2\uc8fc \uac04\uc758 \uc548\uc815\uc774 \ud544\uc694\ud558\ub2e4\ub294 \uc9c4\ub2e8\uc744 \ubc1b\uace0 1\uad70 \ub4f1\ub85d\uc774 \ub9d0\uc18c\ub3fc \uc9c1\ud6c4\uc5d0 \uc5f4\ub9ac\ub294 \uc62c\uc2a4\ud0c0\uc804 \ucd9c\uc804\uc744 \ud3ec\uae30\ud558\uace0 \uc774\ub9c8\ub098\uce74\ub97c \ub300\uc2e0\ud574\uc11c \uc694\uc2dc\uc774 \ub9c8\uc0ac\ud1a0\uac00 \ucd9c\uc804\ud588\ub2e4. \uc774 \ubd80\uc0c1\uc758 \ubcf4\ub984 \uc804\uc5d0\ub3c4 \ud5c8\ub9ac \ud1b5\uc99d\uc73c\ub85c \ub4f1\ud310\uc744 \ud53c\ud588\uc9c0\ub9cc \uc6b0\uc2b9 \uacbd\uc7c1\uc744 \ud558\uace0 \uc788\ub294 \ud300\uc758 \uc0ac\uc815\ub3c4 \uc788\uc5b4 1\uc8fc\uc77c \ud6c4\uc5d0\ub294 \ud22c\uad6c \uc5f0\uc2b5\uc744 \uc7ac\uac1c\ud588\ub2e4. 7\uc6d4 29\uc77c\uc5d0\ub294 1\uad70\uc5d0 \ubcf5\uadc0\ud588\uace0 8\uc6d4 4\uc77c \ub098\uace0\uc57c \uad6c\uc7a5\uc5d0\uc11c \uc5f4\ub9b0 \uc694\ubbf8\uc6b0\ub9ac\uc804\uc5d0\uc11c\ub294 1\uc2e4\uc810 \uc644\ud22c\ub85c \uc2b9\ub9ac \ud22c\uc218\uac00 \ub410\ub2e4. \uadf8 \ud6c4\uc5d0\ub3c4 \uc67c\ucabd \uc5b4\uae68\uc758 \ud1b5\uc99d\uc774 \ub0a8\uc740 \ucc44\ub85c \ub4f1\ud310\ud574\uc11c 4\uc2b9\uc744 \uc62c\ub9ac\ub294 \ub4f1 4\ub144 \uc5f0\uc18d \ub450 \uc790\ub9bf\uc218 \uc2b9\ub9ac\uac00 \ub418\ub294 14\uc2b9\uc744 \uae30\ub85d\ud588\ub2e4.", "answer": "1\uad70 \ub4f1\ub85d", "sentence": "\uc774\ud2bf\ub0a0\uc758 \uc815\ubc00 \uac80\uc0ac\uc5d0\uc11c \uc67c\ucabd \uc5b4\uae68\uad00\uc808 \uc8fc\uc704 \uc5fc\uc99d \ub54c\ubb38\uc5d0 2\uc8fc \uac04\uc758 \uc548\uc815\uc774 \ud544\uc694\ud558\ub2e4\ub294 \uc9c4\ub2e8\uc744 \ubc1b\uace0 1\uad70 \ub4f1\ub85d \uc774 \ub9d0\uc18c\ub3fc \uc9c1\ud6c4\uc5d0 \uc5f4\ub9ac\ub294 \uc62c\uc2a4\ud0c0\uc804 \ucd9c\uc804\uc744 \ud3ec\uae30\ud558\uace0 \uc774\ub9c8\ub098\uce74\ub97c \ub300\uc2e0\ud574\uc11c \uc694\uc2dc\uc774 \ub9c8\uc0ac\ud1a0\uac00 \ucd9c\uc804\ud588\ub2e4.", "paragraph_sentence": "1996\ub144, 4\ub144 \uc5f0\uc18d\uc73c\ub85c \uac1c\ub9c9\uc804 \uc120\ubc1c \ud22c\uc218\ub97c \ub9e1\uc544 6\uc6d4 27\uc77c \ub098\uace0\uc57c \uad6c\uc7a5\uc5d0\uc11c \uc5f4\ub9b0 \ud788\ub85c\uc2dc\ub9c8\uc804\uc5d0\uc11c \uad6c\ub2e8 \uc5ed\uc0ac\uc0c1 \uac00\uc7a5 \ube60\ub978 186\uacbd\uae30\uc5d0\uc11c \ud1b5\uc0b0 1000\ud0c8\uc0bc\uc9c4\uc744 \ub2ec\uc131\ud558\ub294 \ub4f1\uc758 \ud65c\uc57d\uc73c\ub85c 7\uc6d4\uae4c\uc9c0 9\uc2b9\uc744 \uc62c\ub9ac\uace0 \uc788\ub2e4. \uadf8\ub7ec\ub098 7\uc6d4 16\uc77c \ub3c4\ucfc4 \ub3d4\uc5d0\uc11c \uc5f4\ub9b0 \uc694\ubbf8\uc6b0\ub9ac\uc804\uc5d0\uc11c \ud504\ub85c \ub370\ubdd4 \ud6c4 \uac00\uc7a5 \uc9e7\uc740 1/3\uc774\ub2dd\uc5d0 \uac15\ud310\ub3fc \uc67c\ucabd \uc5b4\uae68 \ud1b5\uc99d\uc744 \ud638\uc18c\ud588\ub2e4. \uc774\ud2bf\ub0a0\uc758 \uc815\ubc00 \uac80\uc0ac\uc5d0\uc11c \uc67c\ucabd \uc5b4\uae68\uad00\uc808 \uc8fc\uc704 \uc5fc\uc99d \ub54c\ubb38\uc5d0 2\uc8fc \uac04\uc758 \uc548\uc815\uc774 \ud544\uc694\ud558\ub2e4\ub294 \uc9c4\ub2e8\uc744 \ubc1b\uace0 1\uad70 \ub4f1\ub85d \uc774 \ub9d0\uc18c\ub3fc \uc9c1\ud6c4\uc5d0 \uc5f4\ub9ac\ub294 \uc62c\uc2a4\ud0c0\uc804 \ucd9c\uc804\uc744 \ud3ec\uae30\ud558\uace0 \uc774\ub9c8\ub098\uce74\ub97c \ub300\uc2e0\ud574\uc11c \uc694\uc2dc\uc774 \ub9c8\uc0ac\ud1a0\uac00 \ucd9c\uc804\ud588\ub2e4. \uc774 \ubd80\uc0c1\uc758 \ubcf4\ub984 \uc804\uc5d0\ub3c4 \ud5c8\ub9ac \ud1b5\uc99d\uc73c\ub85c \ub4f1\ud310\uc744 \ud53c\ud588\uc9c0\ub9cc \uc6b0\uc2b9 \uacbd\uc7c1\uc744 \ud558\uace0 \uc788\ub294 \ud300\uc758 \uc0ac\uc815\ub3c4 \uc788\uc5b4 1\uc8fc\uc77c \ud6c4\uc5d0\ub294 \ud22c\uad6c \uc5f0\uc2b5\uc744 \uc7ac\uac1c\ud588\ub2e4. 7\uc6d4 29\uc77c\uc5d0\ub294 1\uad70\uc5d0 \ubcf5\uadc0\ud588\uace0 8\uc6d4 4\uc77c \ub098\uace0\uc57c \uad6c\uc7a5\uc5d0\uc11c \uc5f4\ub9b0 \uc694\ubbf8\uc6b0\ub9ac\uc804\uc5d0\uc11c\ub294 1\uc2e4\uc810 \uc644\ud22c\ub85c \uc2b9\ub9ac \ud22c\uc218\uac00 \ub410\ub2e4. \uadf8 \ud6c4\uc5d0\ub3c4 \uc67c\ucabd \uc5b4\uae68\uc758 \ud1b5\uc99d\uc774 \ub0a8\uc740 \ucc44\ub85c \ub4f1\ud310\ud574\uc11c 4\uc2b9\uc744 \uc62c\ub9ac\ub294 \ub4f1 4\ub144 \uc5f0\uc18d \ub450 \uc790\ub9bf\uc218 \uc2b9\ub9ac\uac00 \ub418\ub294 14\uc2b9\uc744 \uae30\ub85d\ud588\ub2e4.", "paragraph_answer": "1996\ub144, 4\ub144 \uc5f0\uc18d\uc73c\ub85c \uac1c\ub9c9\uc804 \uc120\ubc1c \ud22c\uc218\ub97c \ub9e1\uc544 6\uc6d4 27\uc77c \ub098\uace0\uc57c \uad6c\uc7a5\uc5d0\uc11c \uc5f4\ub9b0 \ud788\ub85c\uc2dc\ub9c8\uc804\uc5d0\uc11c \uad6c\ub2e8 \uc5ed\uc0ac\uc0c1 \uac00\uc7a5 \ube60\ub978 186\uacbd\uae30\uc5d0\uc11c \ud1b5\uc0b0 1000\ud0c8\uc0bc\uc9c4\uc744 \ub2ec\uc131\ud558\ub294 \ub4f1\uc758 \ud65c\uc57d\uc73c\ub85c 7\uc6d4\uae4c\uc9c0 9\uc2b9\uc744 \uc62c\ub9ac\uace0 \uc788\ub2e4. \uadf8\ub7ec\ub098 7\uc6d4 16\uc77c \ub3c4\ucfc4 \ub3d4\uc5d0\uc11c \uc5f4\ub9b0 \uc694\ubbf8\uc6b0\ub9ac\uc804\uc5d0\uc11c \ud504\ub85c \ub370\ubdd4 \ud6c4 \uac00\uc7a5 \uc9e7\uc740 1/3\uc774\ub2dd\uc5d0 \uac15\ud310\ub3fc \uc67c\ucabd \uc5b4\uae68 \ud1b5\uc99d\uc744 \ud638\uc18c\ud588\ub2e4. \uc774\ud2bf\ub0a0\uc758 \uc815\ubc00 \uac80\uc0ac\uc5d0\uc11c \uc67c\ucabd \uc5b4\uae68\uad00\uc808 \uc8fc\uc704 \uc5fc\uc99d \ub54c\ubb38\uc5d0 2\uc8fc \uac04\uc758 \uc548\uc815\uc774 \ud544\uc694\ud558\ub2e4\ub294 \uc9c4\ub2e8\uc744 \ubc1b\uace0 1\uad70 \ub4f1\ub85d \uc774 \ub9d0\uc18c\ub3fc \uc9c1\ud6c4\uc5d0 \uc5f4\ub9ac\ub294 \uc62c\uc2a4\ud0c0\uc804 \ucd9c\uc804\uc744 \ud3ec\uae30\ud558\uace0 \uc774\ub9c8\ub098\uce74\ub97c \ub300\uc2e0\ud574\uc11c \uc694\uc2dc\uc774 \ub9c8\uc0ac\ud1a0\uac00 \ucd9c\uc804\ud588\ub2e4. \uc774 \ubd80\uc0c1\uc758 \ubcf4\ub984 \uc804\uc5d0\ub3c4 \ud5c8\ub9ac \ud1b5\uc99d\uc73c\ub85c \ub4f1\ud310\uc744 \ud53c\ud588\uc9c0\ub9cc \uc6b0\uc2b9 \uacbd\uc7c1\uc744 \ud558\uace0 \uc788\ub294 \ud300\uc758 \uc0ac\uc815\ub3c4 \uc788\uc5b4 1\uc8fc\uc77c \ud6c4\uc5d0\ub294 \ud22c\uad6c \uc5f0\uc2b5\uc744 \uc7ac\uac1c\ud588\ub2e4. 7\uc6d4 29\uc77c\uc5d0\ub294 1\uad70\uc5d0 \ubcf5\uadc0\ud588\uace0 8\uc6d4 4\uc77c \ub098\uace0\uc57c \uad6c\uc7a5\uc5d0\uc11c \uc5f4\ub9b0 \uc694\ubbf8\uc6b0\ub9ac\uc804\uc5d0\uc11c\ub294 1\uc2e4\uc810 \uc644\ud22c\ub85c \uc2b9\ub9ac \ud22c\uc218\uac00 \ub410\ub2e4. \uadf8 \ud6c4\uc5d0\ub3c4 \uc67c\ucabd \uc5b4\uae68\uc758 \ud1b5\uc99d\uc774 \ub0a8\uc740 \ucc44\ub85c \ub4f1\ud310\ud574\uc11c 4\uc2b9\uc744 \uc62c\ub9ac\ub294 \ub4f1 4\ub144 \uc5f0\uc18d \ub450 \uc790\ub9bf\uc218 \uc2b9\ub9ac\uac00 \ub418\ub294 14\uc2b9\uc744 \uae30\ub85d\ud588\ub2e4.", "sentence_answer": "\uc774\ud2bf\ub0a0\uc758 \uc815\ubc00 \uac80\uc0ac\uc5d0\uc11c \uc67c\ucabd \uc5b4\uae68\uad00\uc808 \uc8fc\uc704 \uc5fc\uc99d \ub54c\ubb38\uc5d0 2\uc8fc \uac04\uc758 \uc548\uc815\uc774 \ud544\uc694\ud558\ub2e4\ub294 \uc9c4\ub2e8\uc744 \ubc1b\uace0 1\uad70 \ub4f1\ub85d \uc774 \ub9d0\uc18c\ub3fc \uc9c1\ud6c4\uc5d0 \uc5f4\ub9ac\ub294 \uc62c\uc2a4\ud0c0\uc804 \ucd9c\uc804\uc744 \ud3ec\uae30\ud558\uace0 \uc774\ub9c8\ub098\uce74\ub97c \ub300\uc2e0\ud574\uc11c \uc694\uc2dc\uc774 \ub9c8\uc0ac\ud1a0\uac00 \ucd9c\uc804\ud588\ub2e4.", "paragraph_id": "6545003-7-1", "paragraph_question": "question: \uc774\ub9c8\ub098\uce74 \uc2e0\uc9c0\ub294 1996\ub144 \uc62c\uc2a4\ud0c0\uc804 \ucd9c\uc804\uc744 \ud3ec\uae30\ud55c \uac83\uc740 \ubb34\uc5c7\uc774 \ub9d0\uc18c\ub418\uc5c8\uae30 \ub54c\ubb38\uc778\uac00?, context: 1996\ub144, 4\ub144 \uc5f0\uc18d\uc73c\ub85c \uac1c\ub9c9\uc804 \uc120\ubc1c \ud22c\uc218\ub97c \ub9e1\uc544 6\uc6d4 27\uc77c \ub098\uace0\uc57c \uad6c\uc7a5\uc5d0\uc11c \uc5f4\ub9b0 \ud788\ub85c\uc2dc\ub9c8\uc804\uc5d0\uc11c \uad6c\ub2e8 \uc5ed\uc0ac\uc0c1 \uac00\uc7a5 \ube60\ub978 186\uacbd\uae30\uc5d0\uc11c \ud1b5\uc0b0 1000\ud0c8\uc0bc\uc9c4\uc744 \ub2ec\uc131\ud558\ub294 \ub4f1\uc758 \ud65c\uc57d\uc73c\ub85c 7\uc6d4\uae4c\uc9c0 9\uc2b9\uc744 \uc62c\ub9ac\uace0 \uc788\ub2e4. \uadf8\ub7ec\ub098 7\uc6d4 16\uc77c \ub3c4\ucfc4 \ub3d4\uc5d0\uc11c \uc5f4\ub9b0 \uc694\ubbf8\uc6b0\ub9ac\uc804\uc5d0\uc11c \ud504\ub85c \ub370\ubdd4 \ud6c4 \uac00\uc7a5 \uc9e7\uc740 1/3\uc774\ub2dd\uc5d0 \uac15\ud310\ub3fc \uc67c\ucabd \uc5b4\uae68 \ud1b5\uc99d\uc744 \ud638\uc18c\ud588\ub2e4. \uc774\ud2bf\ub0a0\uc758 \uc815\ubc00 \uac80\uc0ac\uc5d0\uc11c \uc67c\ucabd \uc5b4\uae68\uad00\uc808 \uc8fc\uc704 \uc5fc\uc99d \ub54c\ubb38\uc5d0 2\uc8fc \uac04\uc758 \uc548\uc815\uc774 \ud544\uc694\ud558\ub2e4\ub294 \uc9c4\ub2e8\uc744 \ubc1b\uace0 1\uad70 \ub4f1\ub85d\uc774 \ub9d0\uc18c\ub3fc \uc9c1\ud6c4\uc5d0 \uc5f4\ub9ac\ub294 \uc62c\uc2a4\ud0c0\uc804 \ucd9c\uc804\uc744 \ud3ec\uae30\ud558\uace0 \uc774\ub9c8\ub098\uce74\ub97c \ub300\uc2e0\ud574\uc11c \uc694\uc2dc\uc774 \ub9c8\uc0ac\ud1a0\uac00 \ucd9c\uc804\ud588\ub2e4. \uc774 \ubd80\uc0c1\uc758 \ubcf4\ub984 \uc804\uc5d0\ub3c4 \ud5c8\ub9ac \ud1b5\uc99d\uc73c\ub85c \ub4f1\ud310\uc744 \ud53c\ud588\uc9c0\ub9cc \uc6b0\uc2b9 \uacbd\uc7c1\uc744 \ud558\uace0 \uc788\ub294 \ud300\uc758 \uc0ac\uc815\ub3c4 \uc788\uc5b4 1\uc8fc\uc77c \ud6c4\uc5d0\ub294 \ud22c\uad6c \uc5f0\uc2b5\uc744 \uc7ac\uac1c\ud588\ub2e4. 7\uc6d4 29\uc77c\uc5d0\ub294 1\uad70\uc5d0 \ubcf5\uadc0\ud588\uace0 8\uc6d4 4\uc77c \ub098\uace0\uc57c \uad6c\uc7a5\uc5d0\uc11c \uc5f4\ub9b0 \uc694\ubbf8\uc6b0\ub9ac\uc804\uc5d0\uc11c\ub294 1\uc2e4\uc810 \uc644\ud22c\ub85c \uc2b9\ub9ac \ud22c\uc218\uac00 \ub410\ub2e4. \uadf8 \ud6c4\uc5d0\ub3c4 \uc67c\ucabd \uc5b4\uae68\uc758 \ud1b5\uc99d\uc774 \ub0a8\uc740 \ucc44\ub85c \ub4f1\ud310\ud574\uc11c 4\uc2b9\uc744 \uc62c\ub9ac\ub294 \ub4f1 4\ub144 \uc5f0\uc18d \ub450 \uc790\ub9bf\uc218 \uc2b9\ub9ac\uac00 \ub418\ub294 14\uc2b9\uc744 \uae30\ub85d\ud588\ub2e4."} {"question": "\uc548\ucca0\uc218\uc758 \ud559\ucc3d\uc2dc\uc808 \ucde8\ubbf8\ub294?", "paragraph": "\uc548\ucca0\uc218\ub294 1962\ub144 2\uc6d4 26\uc77c\uc5d0 \uacbd\uc0c1\ub0a8\ub3c4 \ubc00\uc591\uc5d0\uc11c 2\ub0a8 1\ub140 \uc911 \uc7a5\ub0a8\uc73c\ub85c \ud0dc\uc5b4\ub0ac\ub2e4. \uc774\ud6c4 \ubd80\uc0b0\uc5d0\uc11c \ud559\ucc3d \uc2dc\uc808\uc744 \ubcf4\ub0c8\ub2e4. \ud559\ucc3d \uc2dc\uc808\uc5d0 60\uba85 \uc911 30\ub4f1\uc744 \ud560 \uc815\ub3c4\ub85c \ud3c9\ubc94\ud588\uc73c\uba70 \ud2b9\ubcc4\ud788 \uc798 \ud558\ub294 \uac83\uc774 \uc788\ub294 \ud559\uc0dd\uc740 \uc544\ub2c8\uc5c8\ub2e4. \ud558\uc9c0\ub9cc \ub3c5\uc11c\ub97c \ub9e4\uc6b0 \uc88b\uc544\ud588\ub2e4.\ubd80\uc0b0\ub3d9\uc131\uad6d\ubbfc\ud559\uad50 \uc7ac\ud559 \uc2dc\uc808 \ud559\uad50 \ub3c4\uc11c\uad00\uc758 \ucc45\uc744 \ub9e4\uc77c \uba87 \uad8c\uc529 \uc77d\uc5b4 \uacb0\uad6d \ub3c4\uc11c\uad00\uc5d0 \uc788\ub294 \ucc45\uc740 \uac70\uc758 \ub2e4 \uc77d\uac8c \ub410\ub2e4. \ub3c4\uc11c\uad00 \uc0ac\uc11c\ub294 \ub9e4\uc77c \uba87 \uad8c\uc529 \ub300\ucd9c\uacfc \ubc18\ub0a9\uc744 \ud558\ub294 \uc548\ucca0\uc218\uac00 \uc7a5\ub09c\uce5c\ub2e4\uace0 \uc0dd\uac01\ud574\uc11c \ucc45 \ub300\ucd9c\uc744 \uac70\ubd80\ud560 \uc815\ub3c4\uc600\ub2e4. \uc548\ucca0\uc218\ub294 \"\ub2f9\uc2dc \ucc45\uc758 \ud398\uc774\uc9c0\uc218, \ubc1c\ud589 \ub144\uc6d4\uc77c, \uc800\uc790\uae4c\uc9c0 \ubaa8\ub450 \ub2e4 \uc77d\uace0, \ubc14\ub2e5\uc5d0 \uc885\uc774\uac00 \ub5a8\uc5b4\uc838 \uc788\uc73c\uba74 \uadf8\uac83\ub9c8\uc800\ub3c4 \uc77d\uc5b4\uc57c \uc9c1\uc131\uc774 \ud480\ub9ac\ub294 \ud65c\uc790 \uc911\ub3c5\uc99d\uc774\uc5c8\ub358 \uac83 \uac19\ub2e4.\"\ub77c\uace0 \ud68c\uace0\ud588\ub2e4.\uadf8\ub7ec\ub098 \uc218\uc5c5 \uad50\uacfc\uc11c\ub294 \ubcc4\ub85c \uc88b\uc544\ud558\uc9c0 \uc54a\uc558\uace0 \uacfc\ud559\ucc45\uc774\ub098 \uc18c\uc124\ucc45\uc744 \uc88b\uc544\ud574 \uc8fc\ub85c \uc77d\uc5c8\ub294\ub370 \ucc45\uc744 \uc88b\uc544\ud588\ub358 \ud0d3\uc778\uc9c0 \uc0ac\ucd98\uae30\ub3c4 \uc5c6\uc5c8\ub2e4\uace0 \ud55c\ub2e4.", "answer": "\ub3c5\uc11c", "sentence": "\ud558\uc9c0\ub9cc \ub3c5\uc11c \ub97c \ub9e4\uc6b0 \uc88b\uc544\ud588\ub2e4.\ubd80\uc0b0\ub3d9\uc131\uad6d\ubbfc\ud559\uad50 \uc7ac\ud559 \uc2dc\uc808 \ud559\uad50 \ub3c4\uc11c\uad00\uc758 \ucc45\uc744 \ub9e4\uc77c \uba87 \uad8c\uc529 \uc77d\uc5b4 \uacb0\uad6d \ub3c4\uc11c\uad00\uc5d0 \uc788\ub294 \ucc45\uc740 \uac70\uc758 \ub2e4 \uc77d\uac8c \ub410\ub2e4.", "paragraph_sentence": "\uc548\ucca0\uc218\ub294 1962\ub144 2\uc6d4 26\uc77c\uc5d0 \uacbd\uc0c1\ub0a8\ub3c4 \ubc00\uc591\uc5d0\uc11c 2\ub0a8 1\ub140 \uc911 \uc7a5\ub0a8\uc73c\ub85c \ud0dc\uc5b4\ub0ac\ub2e4. \uc774\ud6c4 \ubd80\uc0b0\uc5d0\uc11c \ud559\ucc3d \uc2dc\uc808\uc744 \ubcf4\ub0c8\ub2e4. \ud559\ucc3d \uc2dc\uc808\uc5d0 60\uba85 \uc911 30\ub4f1\uc744 \ud560 \uc815\ub3c4\ub85c \ud3c9\ubc94\ud588\uc73c\uba70 \ud2b9\ubcc4\ud788 \uc798 \ud558\ub294 \uac83\uc774 \uc788\ub294 \ud559\uc0dd\uc740 \uc544\ub2c8\uc5c8\ub2e4. \ud558\uc9c0\ub9cc \ub3c5\uc11c \ub97c \ub9e4\uc6b0 \uc88b\uc544\ud588\ub2e4.\ubd80\uc0b0\ub3d9\uc131\uad6d\ubbfc\ud559\uad50 \uc7ac\ud559 \uc2dc\uc808 \ud559\uad50 \ub3c4\uc11c\uad00\uc758 \ucc45\uc744 \ub9e4\uc77c \uba87 \uad8c\uc529 \uc77d\uc5b4 \uacb0\uad6d \ub3c4\uc11c\uad00\uc5d0 \uc788\ub294 \ucc45\uc740 \uac70\uc758 \ub2e4 \uc77d\uac8c \ub410\ub2e4. \ub3c4\uc11c\uad00 \uc0ac\uc11c\ub294 \ub9e4\uc77c \uba87 \uad8c\uc529 \ub300\ucd9c\uacfc \ubc18\ub0a9\uc744 \ud558\ub294 \uc548\ucca0\uc218\uac00 \uc7a5\ub09c\uce5c\ub2e4\uace0 \uc0dd\uac01\ud574\uc11c \ucc45 \ub300\ucd9c\uc744 \uac70\ubd80\ud560 \uc815\ub3c4\uc600\ub2e4. \uc548\ucca0\uc218\ub294 \"\ub2f9\uc2dc \ucc45\uc758 \ud398\uc774\uc9c0\uc218, \ubc1c\ud589 \ub144\uc6d4\uc77c, \uc800\uc790\uae4c\uc9c0 \ubaa8\ub450 \ub2e4 \uc77d\uace0, \ubc14\ub2e5\uc5d0 \uc885\uc774\uac00 \ub5a8\uc5b4\uc838 \uc788\uc73c\uba74 \uadf8\uac83\ub9c8\uc800\ub3c4 \uc77d\uc5b4\uc57c \uc9c1\uc131\uc774 \ud480\ub9ac\ub294 \ud65c\uc790 \uc911\ub3c5\uc99d\uc774\uc5c8\ub358 \uac83 \uac19\ub2e4. \"\ub77c\uace0 \ud68c\uace0\ud588\ub2e4.\uadf8\ub7ec\ub098 \uc218\uc5c5 \uad50\uacfc\uc11c\ub294 \ubcc4\ub85c \uc88b\uc544\ud558\uc9c0 \uc54a\uc558\uace0 \uacfc\ud559\ucc45\uc774\ub098 \uc18c\uc124\ucc45\uc744 \uc88b\uc544\ud574 \uc8fc\ub85c \uc77d\uc5c8\ub294\ub370 \ucc45\uc744 \uc88b\uc544\ud588\ub358 \ud0d3\uc778\uc9c0 \uc0ac\ucd98\uae30\ub3c4 \uc5c6\uc5c8\ub2e4\uace0 \ud55c\ub2e4.", "paragraph_answer": "\uc548\ucca0\uc218\ub294 1962\ub144 2\uc6d4 26\uc77c\uc5d0 \uacbd\uc0c1\ub0a8\ub3c4 \ubc00\uc591\uc5d0\uc11c 2\ub0a8 1\ub140 \uc911 \uc7a5\ub0a8\uc73c\ub85c \ud0dc\uc5b4\ub0ac\ub2e4. \uc774\ud6c4 \ubd80\uc0b0\uc5d0\uc11c \ud559\ucc3d \uc2dc\uc808\uc744 \ubcf4\ub0c8\ub2e4. \ud559\ucc3d \uc2dc\uc808\uc5d0 60\uba85 \uc911 30\ub4f1\uc744 \ud560 \uc815\ub3c4\ub85c \ud3c9\ubc94\ud588\uc73c\uba70 \ud2b9\ubcc4\ud788 \uc798 \ud558\ub294 \uac83\uc774 \uc788\ub294 \ud559\uc0dd\uc740 \uc544\ub2c8\uc5c8\ub2e4. \ud558\uc9c0\ub9cc \ub3c5\uc11c \ub97c \ub9e4\uc6b0 \uc88b\uc544\ud588\ub2e4.\ubd80\uc0b0\ub3d9\uc131\uad6d\ubbfc\ud559\uad50 \uc7ac\ud559 \uc2dc\uc808 \ud559\uad50 \ub3c4\uc11c\uad00\uc758 \ucc45\uc744 \ub9e4\uc77c \uba87 \uad8c\uc529 \uc77d\uc5b4 \uacb0\uad6d \ub3c4\uc11c\uad00\uc5d0 \uc788\ub294 \ucc45\uc740 \uac70\uc758 \ub2e4 \uc77d\uac8c \ub410\ub2e4. \ub3c4\uc11c\uad00 \uc0ac\uc11c\ub294 \ub9e4\uc77c \uba87 \uad8c\uc529 \ub300\ucd9c\uacfc \ubc18\ub0a9\uc744 \ud558\ub294 \uc548\ucca0\uc218\uac00 \uc7a5\ub09c\uce5c\ub2e4\uace0 \uc0dd\uac01\ud574\uc11c \ucc45 \ub300\ucd9c\uc744 \uac70\ubd80\ud560 \uc815\ub3c4\uc600\ub2e4. \uc548\ucca0\uc218\ub294 \"\ub2f9\uc2dc \ucc45\uc758 \ud398\uc774\uc9c0\uc218, \ubc1c\ud589 \ub144\uc6d4\uc77c, \uc800\uc790\uae4c\uc9c0 \ubaa8\ub450 \ub2e4 \uc77d\uace0, \ubc14\ub2e5\uc5d0 \uc885\uc774\uac00 \ub5a8\uc5b4\uc838 \uc788\uc73c\uba74 \uadf8\uac83\ub9c8\uc800\ub3c4 \uc77d\uc5b4\uc57c \uc9c1\uc131\uc774 \ud480\ub9ac\ub294 \ud65c\uc790 \uc911\ub3c5\uc99d\uc774\uc5c8\ub358 \uac83 \uac19\ub2e4.\"\ub77c\uace0 \ud68c\uace0\ud588\ub2e4.\uadf8\ub7ec\ub098 \uc218\uc5c5 \uad50\uacfc\uc11c\ub294 \ubcc4\ub85c \uc88b\uc544\ud558\uc9c0 \uc54a\uc558\uace0 \uacfc\ud559\ucc45\uc774\ub098 \uc18c\uc124\ucc45\uc744 \uc88b\uc544\ud574 \uc8fc\ub85c \uc77d\uc5c8\ub294\ub370 \ucc45\uc744 \uc88b\uc544\ud588\ub358 \ud0d3\uc778\uc9c0 \uc0ac\ucd98\uae30\ub3c4 \uc5c6\uc5c8\ub2e4\uace0 \ud55c\ub2e4.", "sentence_answer": "\ud558\uc9c0\ub9cc \ub3c5\uc11c \ub97c \ub9e4\uc6b0 \uc88b\uc544\ud588\ub2e4.\ubd80\uc0b0\ub3d9\uc131\uad6d\ubbfc\ud559\uad50 \uc7ac\ud559 \uc2dc\uc808 \ud559\uad50 \ub3c4\uc11c\uad00\uc758 \ucc45\uc744 \ub9e4\uc77c \uba87 \uad8c\uc529 \uc77d\uc5b4 \uacb0\uad6d \ub3c4\uc11c\uad00\uc5d0 \uc788\ub294 \ucc45\uc740 \uac70\uc758 \ub2e4 \uc77d\uac8c \ub410\ub2e4.", "paragraph_id": "6547507-0-1", "paragraph_question": "question: \uc548\ucca0\uc218\uc758 \ud559\ucc3d\uc2dc\uc808 \ucde8\ubbf8\ub294?, context: \uc548\ucca0\uc218\ub294 1962\ub144 2\uc6d4 26\uc77c\uc5d0 \uacbd\uc0c1\ub0a8\ub3c4 \ubc00\uc591\uc5d0\uc11c 2\ub0a8 1\ub140 \uc911 \uc7a5\ub0a8\uc73c\ub85c \ud0dc\uc5b4\ub0ac\ub2e4. \uc774\ud6c4 \ubd80\uc0b0\uc5d0\uc11c \ud559\ucc3d \uc2dc\uc808\uc744 \ubcf4\ub0c8\ub2e4. \ud559\ucc3d \uc2dc\uc808\uc5d0 60\uba85 \uc911 30\ub4f1\uc744 \ud560 \uc815\ub3c4\ub85c \ud3c9\ubc94\ud588\uc73c\uba70 \ud2b9\ubcc4\ud788 \uc798 \ud558\ub294 \uac83\uc774 \uc788\ub294 \ud559\uc0dd\uc740 \uc544\ub2c8\uc5c8\ub2e4. \ud558\uc9c0\ub9cc \ub3c5\uc11c\ub97c \ub9e4\uc6b0 \uc88b\uc544\ud588\ub2e4.\ubd80\uc0b0\ub3d9\uc131\uad6d\ubbfc\ud559\uad50 \uc7ac\ud559 \uc2dc\uc808 \ud559\uad50 \ub3c4\uc11c\uad00\uc758 \ucc45\uc744 \ub9e4\uc77c \uba87 \uad8c\uc529 \uc77d\uc5b4 \uacb0\uad6d \ub3c4\uc11c\uad00\uc5d0 \uc788\ub294 \ucc45\uc740 \uac70\uc758 \ub2e4 \uc77d\uac8c \ub410\ub2e4. \ub3c4\uc11c\uad00 \uc0ac\uc11c\ub294 \ub9e4\uc77c \uba87 \uad8c\uc529 \ub300\ucd9c\uacfc \ubc18\ub0a9\uc744 \ud558\ub294 \uc548\ucca0\uc218\uac00 \uc7a5\ub09c\uce5c\ub2e4\uace0 \uc0dd\uac01\ud574\uc11c \ucc45 \ub300\ucd9c\uc744 \uac70\ubd80\ud560 \uc815\ub3c4\uc600\ub2e4. \uc548\ucca0\uc218\ub294 \"\ub2f9\uc2dc \ucc45\uc758 \ud398\uc774\uc9c0\uc218, \ubc1c\ud589 \ub144\uc6d4\uc77c, \uc800\uc790\uae4c\uc9c0 \ubaa8\ub450 \ub2e4 \uc77d\uace0, \ubc14\ub2e5\uc5d0 \uc885\uc774\uac00 \ub5a8\uc5b4\uc838 \uc788\uc73c\uba74 \uadf8\uac83\ub9c8\uc800\ub3c4 \uc77d\uc5b4\uc57c \uc9c1\uc131\uc774 \ud480\ub9ac\ub294 \ud65c\uc790 \uc911\ub3c5\uc99d\uc774\uc5c8\ub358 \uac83 \uac19\ub2e4.\"\ub77c\uace0 \ud68c\uace0\ud588\ub2e4.\uadf8\ub7ec\ub098 \uc218\uc5c5 \uad50\uacfc\uc11c\ub294 \ubcc4\ub85c \uc88b\uc544\ud558\uc9c0 \uc54a\uc558\uace0 \uacfc\ud559\ucc45\uc774\ub098 \uc18c\uc124\ucc45\uc744 \uc88b\uc544\ud574 \uc8fc\ub85c \uc77d\uc5c8\ub294\ub370 \ucc45\uc744 \uc88b\uc544\ud588\ub358 \ud0d3\uc778\uc9c0 \uc0ac\ucd98\uae30\ub3c4 \uc5c6\uc5c8\ub2e4\uace0 \ud55c\ub2e4."} {"question": "\uc2a4\ud0e0\ub9ac \ud050\ube0c\ub9ad \uac10\ub3c5\uc758 \uc0ac\ub9dd \uc6d0\uc778\uc740?", "paragraph": "\uc2a4\ud0e0\ub9ac \ud050\ube0c\ub9ad \uac10\ub3c5\uc740 \uc544\uc774\uc988 \uc640\uc774\ub4dc \uc167\uc758 \ucd5c\uc885 \ucd2c\uc601\uc774 \ub05d\ub098\uace0 \ucd5c\uc885 \ud3b8\uc9d1 \uae30\uac04\uc778 1999\ub144 3\uc6d4 7\uc77c\uc5d0 \uc2ec\uc7a5\ub9c8\ube44\ub85c \uc0ac\ub9dd\ud558\uc600\uae30 \ub54c\ubb38\uc5d0 \ud050\ube0c\ub9ad\uc740 \uacf5\uc2dd\uc801\uc778 \uc790\ub9ac\uc5d0\uc11c \uc544\uc774\uc988 \uc640\uc774\ub4dc \uc167\uc5d0 \ub300\ud55c \uacac\ud574\ub97c \ud45c\ucd9c\ud55c \uc801\uc774 \uc5c6\ub2e4. \ud558\uc9c0\ub9cc \uc8fc\uc704\uc758 \uc9c0\uc778\ub4e4\uc744 \ud1b5\ud574 \ub4e4\uc740 \ud050\ube0c\ub9ad \uc790\uc2e0\uc758 \ud3c9\uac00\ub294 \ub9e4\uc6b0 \uc5c7\uac08\ub9ac\uace0 \uc788\ub2e4. \uc544\uc774\uc988 \uc640\uc774\ub4dc \uc167\uc758 \uc218\uc11d \uc81c\uc791\uc790\uc778 \uc7ad \ud560\ub7f0(Jack Harlan)\uc740 \ud050\ube0c\ub9ad\uc774 \uc774 \uc601\ud654\uc5d0 \ub300\ud574 \ub9e4\uc6b0 \u201c\ud589\ubcf5\ud574\u201d \ud588\uc73c\uba70, \uc790\uc2e0\uc774 \uc774 \uc601\ud654\uac00 \uadf8\uc758 \u201c\uc601\ud654\ub77c\ub294 \uc608\uc220\uc5d0 \ub300\ud55c \uac00\uc7a5 \ud070 \uae30\uc5ec\u201d\ub77c\uace0 \ud3c9\uac00\ud588\ub2e4\uace0 \ud55c\ub2e4. \ub610\ud55c \uc601\ud654\uc5d0 \ubc30\uc6b0\ub85c \ucc38\uc5ec\ud558\uace0 \ub610\ud55c \ud050\ube0c\ub9ad\uc758 \uc808\uce5c\ud55c \uce5c\uad6c\uc778 \ud1a0\ub4dc \ud544\ub4dc\ub294 \ud050\ube0c\ub9ad\uc774 \ub9c8\uc9c0\ub9c9 \uc21c\uac04\uae4c\uc9c0 \uc774 \uc601\ud654 \ub54c\ubb38\uc5d0 \u201c\uc804\uc728\uc774 \ud758\ub800\ub2e4\u201d\ub77c\uace0 \ud588\uc73c\uba70 \uc601\ud654 \ud6c4\ubc18\ubd80 \uc791\uc5c5\uc744 \ud558\uba74\uc11c \ub9e4\uc6b0 \ud589\ubcf5\ud574 \ud588\ub2e4\uace0 \uc804\ud588\ub2e4. \ud558\uc9c0\ub9cc \ud050\ube0c\ub9ad \uac10\ub3c5\uc758 \ub2e4\ub978 \uc601\ud654\uc778 \ud480 \uba54\ud0c8 \uc790\ucf13\uc5d0 \ubc30\uc6b0\ub85c \ucc38\uc5ec\ud588\ub358 R. \ub9ac \uc5b4\ubbf8(R. Lee. Ermey)\uc5d0 \ub530\ub974\uba74 \ud050\ube0c\ub9ad\uc740 \uc8fd\uae30 2\uc8fc\uc804\uc5d0 \uc5bc\ubbf8\uc5d0\uac8c \uc804\ud654\ub97c \uac78\uc5b4 \uc774 \uc601\ud654\ub294 \ub625\ub369\uc5b4\ub9ac(Piece of Shit)\uc774\ub77c\uace0 \ud45c\ud604\ud558\uba70 \uac15\ud55c \ubd88\ub9cc\uc871\uc744 \ud45c\ucd9c\ud588\ub2e4\uace0 \ud55c\ub2e4.", "answer": "\uc2ec\uc7a5\ub9c8\ube44", "sentence": "\uc2a4\ud0e0\ub9ac \ud050\ube0c\ub9ad \uac10\ub3c5\uc740 \uc544\uc774\uc988 \uc640\uc774\ub4dc \uc167\uc758 \ucd5c\uc885 \ucd2c\uc601\uc774 \ub05d\ub098\uace0 \ucd5c\uc885 \ud3b8\uc9d1 \uae30\uac04\uc778 1999\ub144 3\uc6d4 7\uc77c\uc5d0 \uc2ec\uc7a5\ub9c8\ube44 \ub85c \uc0ac\ub9dd\ud558\uc600\uae30 \ub54c\ubb38\uc5d0 \ud050\ube0c\ub9ad\uc740 \uacf5\uc2dd\uc801\uc778 \uc790\ub9ac\uc5d0\uc11c \uc544\uc774\uc988 \uc640\uc774\ub4dc \uc167\uc5d0 \ub300\ud55c \uacac\ud574\ub97c \ud45c\ucd9c\ud55c \uc801\uc774 \uc5c6\ub2e4.", "paragraph_sentence": " \uc2a4\ud0e0\ub9ac \ud050\ube0c\ub9ad \uac10\ub3c5\uc740 \uc544\uc774\uc988 \uc640\uc774\ub4dc \uc167\uc758 \ucd5c\uc885 \ucd2c\uc601\uc774 \ub05d\ub098\uace0 \ucd5c\uc885 \ud3b8\uc9d1 \uae30\uac04\uc778 1999\ub144 3\uc6d4 7\uc77c\uc5d0 \uc2ec\uc7a5\ub9c8\ube44 \ub85c \uc0ac\ub9dd\ud558\uc600\uae30 \ub54c\ubb38\uc5d0 \ud050\ube0c\ub9ad\uc740 \uacf5\uc2dd\uc801\uc778 \uc790\ub9ac\uc5d0\uc11c \uc544\uc774\uc988 \uc640\uc774\ub4dc \uc167\uc5d0 \ub300\ud55c \uacac\ud574\ub97c \ud45c\ucd9c\ud55c \uc801\uc774 \uc5c6\ub2e4. \ud558\uc9c0\ub9cc \uc8fc\uc704\uc758 \uc9c0\uc778\ub4e4\uc744 \ud1b5\ud574 \ub4e4\uc740 \ud050\ube0c\ub9ad \uc790\uc2e0\uc758 \ud3c9\uac00\ub294 \ub9e4\uc6b0 \uc5c7\uac08\ub9ac\uace0 \uc788\ub2e4. \uc544\uc774\uc988 \uc640\uc774\ub4dc \uc167\uc758 \uc218\uc11d \uc81c\uc791\uc790\uc778 \uc7ad \ud560\ub7f0(Jack Harlan)\uc740 \ud050\ube0c\ub9ad\uc774 \uc774 \uc601\ud654\uc5d0 \ub300\ud574 \ub9e4\uc6b0 \u201c\ud589\ubcf5\ud574\u201d \ud588\uc73c\uba70, \uc790\uc2e0\uc774 \uc774 \uc601\ud654\uac00 \uadf8\uc758 \u201c\uc601\ud654\ub77c\ub294 \uc608\uc220\uc5d0 \ub300\ud55c \uac00\uc7a5 \ud070 \uae30\uc5ec\u201d\ub77c\uace0 \ud3c9\uac00\ud588\ub2e4\uace0 \ud55c\ub2e4. \ub610\ud55c \uc601\ud654\uc5d0 \ubc30\uc6b0\ub85c \ucc38\uc5ec\ud558\uace0 \ub610\ud55c \ud050\ube0c\ub9ad\uc758 \uc808\uce5c\ud55c \uce5c\uad6c\uc778 \ud1a0\ub4dc \ud544\ub4dc\ub294 \ud050\ube0c\ub9ad\uc774 \ub9c8\uc9c0\ub9c9 \uc21c\uac04\uae4c\uc9c0 \uc774 \uc601\ud654 \ub54c\ubb38\uc5d0 \u201c\uc804\uc728\uc774 \ud758\ub800\ub2e4\u201d\ub77c\uace0 \ud588\uc73c\uba70 \uc601\ud654 \ud6c4\ubc18\ubd80 \uc791\uc5c5\uc744 \ud558\uba74\uc11c \ub9e4\uc6b0 \ud589\ubcf5\ud574 \ud588\ub2e4\uace0 \uc804\ud588\ub2e4. \ud558\uc9c0\ub9cc \ud050\ube0c\ub9ad \uac10\ub3c5\uc758 \ub2e4\ub978 \uc601\ud654\uc778 \ud480 \uba54\ud0c8 \uc790\ucf13\uc5d0 \ubc30\uc6b0\ub85c \ucc38\uc5ec\ud588\ub358 R. \ub9ac \uc5b4\ubbf8(R. Lee. Ermey)\uc5d0 \ub530\ub974\uba74 \ud050\ube0c\ub9ad\uc740 \uc8fd\uae30 2\uc8fc\uc804\uc5d0 \uc5bc\ubbf8\uc5d0\uac8c \uc804\ud654\ub97c \uac78\uc5b4 \uc774 \uc601\ud654\ub294 \ub625\ub369\uc5b4\ub9ac(Piece of Shit)\uc774\ub77c\uace0 \ud45c\ud604\ud558\uba70 \uac15\ud55c \ubd88\ub9cc\uc871\uc744 \ud45c\ucd9c\ud588\ub2e4\uace0 \ud55c\ub2e4.", "paragraph_answer": "\uc2a4\ud0e0\ub9ac \ud050\ube0c\ub9ad \uac10\ub3c5\uc740 \uc544\uc774\uc988 \uc640\uc774\ub4dc \uc167\uc758 \ucd5c\uc885 \ucd2c\uc601\uc774 \ub05d\ub098\uace0 \ucd5c\uc885 \ud3b8\uc9d1 \uae30\uac04\uc778 1999\ub144 3\uc6d4 7\uc77c\uc5d0 \uc2ec\uc7a5\ub9c8\ube44 \ub85c \uc0ac\ub9dd\ud558\uc600\uae30 \ub54c\ubb38\uc5d0 \ud050\ube0c\ub9ad\uc740 \uacf5\uc2dd\uc801\uc778 \uc790\ub9ac\uc5d0\uc11c \uc544\uc774\uc988 \uc640\uc774\ub4dc \uc167\uc5d0 \ub300\ud55c \uacac\ud574\ub97c \ud45c\ucd9c\ud55c \uc801\uc774 \uc5c6\ub2e4. \ud558\uc9c0\ub9cc \uc8fc\uc704\uc758 \uc9c0\uc778\ub4e4\uc744 \ud1b5\ud574 \ub4e4\uc740 \ud050\ube0c\ub9ad \uc790\uc2e0\uc758 \ud3c9\uac00\ub294 \ub9e4\uc6b0 \uc5c7\uac08\ub9ac\uace0 \uc788\ub2e4. \uc544\uc774\uc988 \uc640\uc774\ub4dc \uc167\uc758 \uc218\uc11d \uc81c\uc791\uc790\uc778 \uc7ad \ud560\ub7f0(Jack Harlan)\uc740 \ud050\ube0c\ub9ad\uc774 \uc774 \uc601\ud654\uc5d0 \ub300\ud574 \ub9e4\uc6b0 \u201c\ud589\ubcf5\ud574\u201d \ud588\uc73c\uba70, \uc790\uc2e0\uc774 \uc774 \uc601\ud654\uac00 \uadf8\uc758 \u201c\uc601\ud654\ub77c\ub294 \uc608\uc220\uc5d0 \ub300\ud55c \uac00\uc7a5 \ud070 \uae30\uc5ec\u201d\ub77c\uace0 \ud3c9\uac00\ud588\ub2e4\uace0 \ud55c\ub2e4. \ub610\ud55c \uc601\ud654\uc5d0 \ubc30\uc6b0\ub85c \ucc38\uc5ec\ud558\uace0 \ub610\ud55c \ud050\ube0c\ub9ad\uc758 \uc808\uce5c\ud55c \uce5c\uad6c\uc778 \ud1a0\ub4dc \ud544\ub4dc\ub294 \ud050\ube0c\ub9ad\uc774 \ub9c8\uc9c0\ub9c9 \uc21c\uac04\uae4c\uc9c0 \uc774 \uc601\ud654 \ub54c\ubb38\uc5d0 \u201c\uc804\uc728\uc774 \ud758\ub800\ub2e4\u201d\ub77c\uace0 \ud588\uc73c\uba70 \uc601\ud654 \ud6c4\ubc18\ubd80 \uc791\uc5c5\uc744 \ud558\uba74\uc11c \ub9e4\uc6b0 \ud589\ubcf5\ud574 \ud588\ub2e4\uace0 \uc804\ud588\ub2e4. \ud558\uc9c0\ub9cc \ud050\ube0c\ub9ad \uac10\ub3c5\uc758 \ub2e4\ub978 \uc601\ud654\uc778 \ud480 \uba54\ud0c8 \uc790\ucf13\uc5d0 \ubc30\uc6b0\ub85c \ucc38\uc5ec\ud588\ub358 R. \ub9ac \uc5b4\ubbf8(R. Lee. Ermey)\uc5d0 \ub530\ub974\uba74 \ud050\ube0c\ub9ad\uc740 \uc8fd\uae30 2\uc8fc\uc804\uc5d0 \uc5bc\ubbf8\uc5d0\uac8c \uc804\ud654\ub97c \uac78\uc5b4 \uc774 \uc601\ud654\ub294 \ub625\ub369\uc5b4\ub9ac(Piece of Shit)\uc774\ub77c\uace0 \ud45c\ud604\ud558\uba70 \uac15\ud55c \ubd88\ub9cc\uc871\uc744 \ud45c\ucd9c\ud588\ub2e4\uace0 \ud55c\ub2e4.", "sentence_answer": "\uc2a4\ud0e0\ub9ac \ud050\ube0c\ub9ad \uac10\ub3c5\uc740 \uc544\uc774\uc988 \uc640\uc774\ub4dc \uc167\uc758 \ucd5c\uc885 \ucd2c\uc601\uc774 \ub05d\ub098\uace0 \ucd5c\uc885 \ud3b8\uc9d1 \uae30\uac04\uc778 1999\ub144 3\uc6d4 7\uc77c\uc5d0 \uc2ec\uc7a5\ub9c8\ube44 \ub85c \uc0ac\ub9dd\ud558\uc600\uae30 \ub54c\ubb38\uc5d0 \ud050\ube0c\ub9ad\uc740 \uacf5\uc2dd\uc801\uc778 \uc790\ub9ac\uc5d0\uc11c \uc544\uc774\uc988 \uc640\uc774\ub4dc \uc167\uc5d0 \ub300\ud55c \uacac\ud574\ub97c \ud45c\ucd9c\ud55c \uc801\uc774 \uc5c6\ub2e4.", "paragraph_id": "6538620-6-0", "paragraph_question": "question: \uc2a4\ud0e0\ub9ac \ud050\ube0c\ub9ad \uac10\ub3c5\uc758 \uc0ac\ub9dd \uc6d0\uc778\uc740?, context: \uc2a4\ud0e0\ub9ac \ud050\ube0c\ub9ad \uac10\ub3c5\uc740 \uc544\uc774\uc988 \uc640\uc774\ub4dc \uc167\uc758 \ucd5c\uc885 \ucd2c\uc601\uc774 \ub05d\ub098\uace0 \ucd5c\uc885 \ud3b8\uc9d1 \uae30\uac04\uc778 1999\ub144 3\uc6d4 7\uc77c\uc5d0 \uc2ec\uc7a5\ub9c8\ube44\ub85c \uc0ac\ub9dd\ud558\uc600\uae30 \ub54c\ubb38\uc5d0 \ud050\ube0c\ub9ad\uc740 \uacf5\uc2dd\uc801\uc778 \uc790\ub9ac\uc5d0\uc11c \uc544\uc774\uc988 \uc640\uc774\ub4dc \uc167\uc5d0 \ub300\ud55c \uacac\ud574\ub97c \ud45c\ucd9c\ud55c \uc801\uc774 \uc5c6\ub2e4. \ud558\uc9c0\ub9cc \uc8fc\uc704\uc758 \uc9c0\uc778\ub4e4\uc744 \ud1b5\ud574 \ub4e4\uc740 \ud050\ube0c\ub9ad \uc790\uc2e0\uc758 \ud3c9\uac00\ub294 \ub9e4\uc6b0 \uc5c7\uac08\ub9ac\uace0 \uc788\ub2e4. \uc544\uc774\uc988 \uc640\uc774\ub4dc \uc167\uc758 \uc218\uc11d \uc81c\uc791\uc790\uc778 \uc7ad \ud560\ub7f0(Jack Harlan)\uc740 \ud050\ube0c\ub9ad\uc774 \uc774 \uc601\ud654\uc5d0 \ub300\ud574 \ub9e4\uc6b0 \u201c\ud589\ubcf5\ud574\u201d \ud588\uc73c\uba70, \uc790\uc2e0\uc774 \uc774 \uc601\ud654\uac00 \uadf8\uc758 \u201c\uc601\ud654\ub77c\ub294 \uc608\uc220\uc5d0 \ub300\ud55c \uac00\uc7a5 \ud070 \uae30\uc5ec\u201d\ub77c\uace0 \ud3c9\uac00\ud588\ub2e4\uace0 \ud55c\ub2e4. \ub610\ud55c \uc601\ud654\uc5d0 \ubc30\uc6b0\ub85c \ucc38\uc5ec\ud558\uace0 \ub610\ud55c \ud050\ube0c\ub9ad\uc758 \uc808\uce5c\ud55c \uce5c\uad6c\uc778 \ud1a0\ub4dc \ud544\ub4dc\ub294 \ud050\ube0c\ub9ad\uc774 \ub9c8\uc9c0\ub9c9 \uc21c\uac04\uae4c\uc9c0 \uc774 \uc601\ud654 \ub54c\ubb38\uc5d0 \u201c\uc804\uc728\uc774 \ud758\ub800\ub2e4\u201d\ub77c\uace0 \ud588\uc73c\uba70 \uc601\ud654 \ud6c4\ubc18\ubd80 \uc791\uc5c5\uc744 \ud558\uba74\uc11c \ub9e4\uc6b0 \ud589\ubcf5\ud574 \ud588\ub2e4\uace0 \uc804\ud588\ub2e4. \ud558\uc9c0\ub9cc \ud050\ube0c\ub9ad \uac10\ub3c5\uc758 \ub2e4\ub978 \uc601\ud654\uc778 \ud480 \uba54\ud0c8 \uc790\ucf13\uc5d0 \ubc30\uc6b0\ub85c \ucc38\uc5ec\ud588\ub358 R. \ub9ac \uc5b4\ubbf8(R. Lee. Ermey)\uc5d0 \ub530\ub974\uba74 \ud050\ube0c\ub9ad\uc740 \uc8fd\uae30 2\uc8fc\uc804\uc5d0 \uc5bc\ubbf8\uc5d0\uac8c \uc804\ud654\ub97c \uac78\uc5b4 \uc774 \uc601\ud654\ub294 \ub625\ub369\uc5b4\ub9ac(Piece of Shit)\uc774\ub77c\uace0 \ud45c\ud604\ud558\uba70 \uac15\ud55c \ubd88\ub9cc\uc871\uc744 \ud45c\ucd9c\ud588\ub2e4\uace0 \ud55c\ub2e4."} {"question": "\ud3ec\uc2a4\ud2b8 \uc2dc\uc98c\uc5d0\uc11c \uce5c 4\uac1c\uc758 \uc548\ud0c0\ub4e4 \uc911\uc5d0\uc11c \ud558\ub098\ub3c4 \uc5c6\uc5c8\ub358 \uac83\uc740?", "paragraph": "1938\ub144 \uc2dc\uc98c \uc911\ubc18\uc774 \ub418\uc790 \ub8e8 \uac8c\ub9ad\uc774 \uc57c\uad6c \uc120\uc218\ub85c\uc11c \ubcf4\uc5ec\uc8fc\ub358 \uc131\uacfc\ub4e4\uc774 \uc810\ucc28 \uc904\uc5b4\ub4e4\uae30 \uc2dc\uc791\ud588\ub2e4. \uc2dc\uc98c\uc774 \ub05d\ub098\uace0 \u201c\uc2dc\uc98c \uc911\ubc18\uc5d0 \uc774\ubbf8 \uc9c0\uccd0\ubc84\ub838\uc5b4. \uc774\uc720\ub294 \ubaa8\ub974\uaca0\uc9c0\ub9cc \ub2e4\uc2e0 \uadf8\ub7f4 \uc218\ub294 \uc5c6\uc9c0.\u201d \ub77c\ub294 \ub9d0\uc744 \ub0a8\uacbc\ub2e4. \ud480\ud0c0\uc784 14\ubc88\uc9f8 \uc2dc\uc98c\uc774\uc5c8\ub358 1938\ub144 \uc2dc\uc98c \uae30\ub85d\uc740 \ud0c0\uc728\uc774 13\ub144 \ub9cc\uc5d0 2\ud560\ub300(0.295)\ub85c \ub5a8\uc5b4\uc9c0\uae34 \ud588\uc5b4\ub3c4 170\uac1c\uc758 \uc548\ud0c0\ub97c \uccd0\uc11c 114\ud0c0\uc810, 0.523\uc758 \uc7a5\ud0c0\uc728\uc744 \uae30\ub85d\ud588\uace0, 689 \ud0c0\uc11d\uc5d0 \ub098\uc640 \uc0bc\uc9c4\uc740 \ub2e8\uc9c0 75\uac1c, \ud648\ub7f0\uc740 29\uac1c\ub97c \ucce4\ub2e4. \ud558\uc9c0\ub9cc \uc774\uc804 \uc2dc\uc98c\uc774\uc5c8\ub358 1937\ub144\uc758 \ud0c0\uc728 0.351, \uc7a5\ud0c0\uc728 0.643\uacfc \ube44\uad50\ud574 \ubcf4\uba74 \uadf8 \ud558\ub77d\ud3ed\uc740 \uc81c\ubc95 \ud070 \uac83\uc774\uc5c8\uace0, 1938\ub144 \ud3ec\uc2a4\ud2b8\uc2dc\uc98c\uc5d0\uc11c\ub3c4 4\uac1c\uc758 \uc548\ud0c0\ub97c \uccd0\uc11c 0.286\uc758 \ud0c0\uc728\uc744 \uae30\ub85d\ud588\ub294\ub370 \uadf8 4\uac1c\uc758 \uc548\ud0c0\ub4e4 \uc911\uc5d0\uc11c \uc7a5\ud0c0\ub294 \ud558\ub098\ub3c4 \uc5c6\uc5c8\ub2e4.", "answer": "\uc7a5\ud0c0", "sentence": "\ud480\ud0c0\uc784 14\ubc88\uc9f8 \uc2dc\uc98c\uc774\uc5c8\ub358 1938\ub144 \uc2dc\uc98c \uae30\ub85d\uc740 \ud0c0\uc728\uc774 13\ub144 \ub9cc\uc5d0 2\ud560\ub300(0.295)\ub85c \ub5a8\uc5b4\uc9c0\uae34 \ud588\uc5b4\ub3c4 170\uac1c\uc758 \uc548\ud0c0\ub97c \uccd0\uc11c 114\ud0c0\uc810, 0.523\uc758 \uc7a5\ud0c0 \uc728\uc744 \uae30\ub85d\ud588\uace0, 689 \ud0c0\uc11d\uc5d0 \ub098\uc640 \uc0bc\uc9c4\uc740 \ub2e8\uc9c0 75\uac1c, \ud648\ub7f0\uc740 29\uac1c\ub97c \ucce4\ub2e4.", "paragraph_sentence": "1938\ub144 \uc2dc\uc98c \uc911\ubc18\uc774 \ub418\uc790 \ub8e8 \uac8c\ub9ad\uc774 \uc57c\uad6c \uc120\uc218\ub85c\uc11c \ubcf4\uc5ec\uc8fc\ub358 \uc131\uacfc\ub4e4\uc774 \uc810\ucc28 \uc904\uc5b4\ub4e4\uae30 \uc2dc\uc791\ud588\ub2e4. \uc2dc\uc98c\uc774 \ub05d\ub098\uace0 \u201c\uc2dc\uc98c \uc911\ubc18\uc5d0 \uc774\ubbf8 \uc9c0\uccd0\ubc84\ub838\uc5b4. \uc774\uc720\ub294 \ubaa8\ub974\uaca0\uc9c0\ub9cc \ub2e4\uc2e0 \uadf8\ub7f4 \uc218\ub294 \uc5c6\uc9c0. \u201d \ub77c\ub294 \ub9d0\uc744 \ub0a8\uacbc\ub2e4. \ud480\ud0c0\uc784 14\ubc88\uc9f8 \uc2dc\uc98c\uc774\uc5c8\ub358 1938\ub144 \uc2dc\uc98c \uae30\ub85d\uc740 \ud0c0\uc728\uc774 13\ub144 \ub9cc\uc5d0 2\ud560\ub300(0.295)\ub85c \ub5a8\uc5b4\uc9c0\uae34 \ud588\uc5b4\ub3c4 170\uac1c\uc758 \uc548\ud0c0\ub97c \uccd0\uc11c 114\ud0c0\uc810, 0.523\uc758 \uc7a5\ud0c0 \uc728\uc744 \uae30\ub85d\ud588\uace0, 689 \ud0c0\uc11d\uc5d0 \ub098\uc640 \uc0bc\uc9c4\uc740 \ub2e8\uc9c0 75\uac1c, \ud648\ub7f0\uc740 29\uac1c\ub97c \ucce4\ub2e4. \ud558\uc9c0\ub9cc \uc774\uc804 \uc2dc\uc98c\uc774\uc5c8\ub358 1937\ub144\uc758 \ud0c0\uc728 0.351, \uc7a5\ud0c0\uc728 0.643\uacfc \ube44\uad50\ud574 \ubcf4\uba74 \uadf8 \ud558\ub77d\ud3ed\uc740 \uc81c\ubc95 \ud070 \uac83\uc774\uc5c8\uace0, 1938\ub144 \ud3ec\uc2a4\ud2b8\uc2dc\uc98c\uc5d0\uc11c\ub3c4 4\uac1c\uc758 \uc548\ud0c0\ub97c \uccd0\uc11c 0.286\uc758 \ud0c0\uc728\uc744 \uae30\ub85d\ud588\ub294\ub370 \uadf8 4\uac1c\uc758 \uc548\ud0c0\ub4e4 \uc911\uc5d0\uc11c \uc7a5\ud0c0\ub294 \ud558\ub098\ub3c4 \uc5c6\uc5c8\ub2e4.", "paragraph_answer": "1938\ub144 \uc2dc\uc98c \uc911\ubc18\uc774 \ub418\uc790 \ub8e8 \uac8c\ub9ad\uc774 \uc57c\uad6c \uc120\uc218\ub85c\uc11c \ubcf4\uc5ec\uc8fc\ub358 \uc131\uacfc\ub4e4\uc774 \uc810\ucc28 \uc904\uc5b4\ub4e4\uae30 \uc2dc\uc791\ud588\ub2e4. \uc2dc\uc98c\uc774 \ub05d\ub098\uace0 \u201c\uc2dc\uc98c \uc911\ubc18\uc5d0 \uc774\ubbf8 \uc9c0\uccd0\ubc84\ub838\uc5b4. \uc774\uc720\ub294 \ubaa8\ub974\uaca0\uc9c0\ub9cc \ub2e4\uc2e0 \uadf8\ub7f4 \uc218\ub294 \uc5c6\uc9c0.\u201d \ub77c\ub294 \ub9d0\uc744 \ub0a8\uacbc\ub2e4. \ud480\ud0c0\uc784 14\ubc88\uc9f8 \uc2dc\uc98c\uc774\uc5c8\ub358 1938\ub144 \uc2dc\uc98c \uae30\ub85d\uc740 \ud0c0\uc728\uc774 13\ub144 \ub9cc\uc5d0 2\ud560\ub300(0.295)\ub85c \ub5a8\uc5b4\uc9c0\uae34 \ud588\uc5b4\ub3c4 170\uac1c\uc758 \uc548\ud0c0\ub97c \uccd0\uc11c 114\ud0c0\uc810, 0.523\uc758 \uc7a5\ud0c0 \uc728\uc744 \uae30\ub85d\ud588\uace0, 689 \ud0c0\uc11d\uc5d0 \ub098\uc640 \uc0bc\uc9c4\uc740 \ub2e8\uc9c0 75\uac1c, \ud648\ub7f0\uc740 29\uac1c\ub97c \ucce4\ub2e4. \ud558\uc9c0\ub9cc \uc774\uc804 \uc2dc\uc98c\uc774\uc5c8\ub358 1937\ub144\uc758 \ud0c0\uc728 0.351, \uc7a5\ud0c0\uc728 0.643\uacfc \ube44\uad50\ud574 \ubcf4\uba74 \uadf8 \ud558\ub77d\ud3ed\uc740 \uc81c\ubc95 \ud070 \uac83\uc774\uc5c8\uace0, 1938\ub144 \ud3ec\uc2a4\ud2b8\uc2dc\uc98c\uc5d0\uc11c\ub3c4 4\uac1c\uc758 \uc548\ud0c0\ub97c \uccd0\uc11c 0.286\uc758 \ud0c0\uc728\uc744 \uae30\ub85d\ud588\ub294\ub370 \uadf8 4\uac1c\uc758 \uc548\ud0c0\ub4e4 \uc911\uc5d0\uc11c \uc7a5\ud0c0\ub294 \ud558\ub098\ub3c4 \uc5c6\uc5c8\ub2e4.", "sentence_answer": "\ud480\ud0c0\uc784 14\ubc88\uc9f8 \uc2dc\uc98c\uc774\uc5c8\ub358 1938\ub144 \uc2dc\uc98c \uae30\ub85d\uc740 \ud0c0\uc728\uc774 13\ub144 \ub9cc\uc5d0 2\ud560\ub300(0.295)\ub85c \ub5a8\uc5b4\uc9c0\uae34 \ud588\uc5b4\ub3c4 170\uac1c\uc758 \uc548\ud0c0\ub97c \uccd0\uc11c 114\ud0c0\uc810, 0.523\uc758 \uc7a5\ud0c0 \uc728\uc744 \uae30\ub85d\ud588\uace0, 689 \ud0c0\uc11d\uc5d0 \ub098\uc640 \uc0bc\uc9c4\uc740 \ub2e8\uc9c0 75\uac1c, \ud648\ub7f0\uc740 29\uac1c\ub97c \ucce4\ub2e4.", "paragraph_id": "6602417-6-1", "paragraph_question": "question: \ud3ec\uc2a4\ud2b8 \uc2dc\uc98c\uc5d0\uc11c \uce5c 4\uac1c\uc758 \uc548\ud0c0\ub4e4 \uc911\uc5d0\uc11c \ud558\ub098\ub3c4 \uc5c6\uc5c8\ub358 \uac83\uc740?, context: 1938\ub144 \uc2dc\uc98c \uc911\ubc18\uc774 \ub418\uc790 \ub8e8 \uac8c\ub9ad\uc774 \uc57c\uad6c \uc120\uc218\ub85c\uc11c \ubcf4\uc5ec\uc8fc\ub358 \uc131\uacfc\ub4e4\uc774 \uc810\ucc28 \uc904\uc5b4\ub4e4\uae30 \uc2dc\uc791\ud588\ub2e4. \uc2dc\uc98c\uc774 \ub05d\ub098\uace0 \u201c\uc2dc\uc98c \uc911\ubc18\uc5d0 \uc774\ubbf8 \uc9c0\uccd0\ubc84\ub838\uc5b4. \uc774\uc720\ub294 \ubaa8\ub974\uaca0\uc9c0\ub9cc \ub2e4\uc2e0 \uadf8\ub7f4 \uc218\ub294 \uc5c6\uc9c0.\u201d \ub77c\ub294 \ub9d0\uc744 \ub0a8\uacbc\ub2e4. \ud480\ud0c0\uc784 14\ubc88\uc9f8 \uc2dc\uc98c\uc774\uc5c8\ub358 1938\ub144 \uc2dc\uc98c \uae30\ub85d\uc740 \ud0c0\uc728\uc774 13\ub144 \ub9cc\uc5d0 2\ud560\ub300(0.295)\ub85c \ub5a8\uc5b4\uc9c0\uae34 \ud588\uc5b4\ub3c4 170\uac1c\uc758 \uc548\ud0c0\ub97c \uccd0\uc11c 114\ud0c0\uc810, 0.523\uc758 \uc7a5\ud0c0\uc728\uc744 \uae30\ub85d\ud588\uace0, 689 \ud0c0\uc11d\uc5d0 \ub098\uc640 \uc0bc\uc9c4\uc740 \ub2e8\uc9c0 75\uac1c, \ud648\ub7f0\uc740 29\uac1c\ub97c \ucce4\ub2e4. \ud558\uc9c0\ub9cc \uc774\uc804 \uc2dc\uc98c\uc774\uc5c8\ub358 1937\ub144\uc758 \ud0c0\uc728 0.351, \uc7a5\ud0c0\uc728 0.643\uacfc \ube44\uad50\ud574 \ubcf4\uba74 \uadf8 \ud558\ub77d\ud3ed\uc740 \uc81c\ubc95 \ud070 \uac83\uc774\uc5c8\uace0, 1938\ub144 \ud3ec\uc2a4\ud2b8\uc2dc\uc98c\uc5d0\uc11c\ub3c4 4\uac1c\uc758 \uc548\ud0c0\ub97c \uccd0\uc11c 0.286\uc758 \ud0c0\uc728\uc744 \uae30\ub85d\ud588\ub294\ub370 \uadf8 4\uac1c\uc758 \uc548\ud0c0\ub4e4 \uc911\uc5d0\uc11c \uc7a5\ud0c0\ub294 \ud558\ub098\ub3c4 \uc5c6\uc5c8\ub2e4."} {"question": "\uc0dd\uba85\uccb4\ub3c4 \ubb3c\uacfc \ud568\uaed8 \uc9c0\uad6c\uc5d0 \ub3c4\ucc29\ud588\uc744\uaebc\ub77c\uace0 \ubcf4\uace0 \uc788\ub294 \uc124\uc740?", "paragraph": "\ub0b4\ud589\uc131\uc5d0 \uac70\ub300 \ucda9\ub3cc\uc774 \uc77c\uc5b4\ub098\ub358 \uc2dc\uae30, \ub0b4\ud589\uc131\uc73c\ub85c \ud769\uc5b4\uc838 \ub0a0\uc544 \ub4e4\uc5b4\uc628 \uc18c\ud589\uc131 \ubb3c\uc9c8\ub85c\ubd80\ud130 \uc9c0\uad6c\uc5d0 \ubb3c(~6\u00d710^\ud0ac\ub85c\uadf8\ub7a8)\uc774 \uacf5\uae09\ub418\uc5c8\uc744 \uac83\uc73c\ub85c \ubcf4\uc778\ub2e4. \ubb3c\uc740 \ud718\ubc1c\uc131\uc774 \uac15\ud574\uc11c \uc9c0\uad6c\uac00 \uc0dd\uaca8\ub09c \uc7a5\uc18c\uc5d0\uc11c\ub294 \uc561\uccb4 \uc0c1\ud0dc\ub85c \uc874\uc7ac\ud560 \uc218 \uc5c6\uc5c8\ub2e4. \ub530\ub77c\uc11c \ubb3c\uc740 \ub354 \ucc28\uac00\uc6b4 \ubc14\uae65\ucabd \uada4\ub3c4\uc5d0\uc11c \uc628 \ucc9c\uccb4\ub97c \ud1b5\ud574 \uacf5\uae09\ub418\uc5c8\uc744 \uac83\uc774\ub2e4. \ubb3c\uc740 \uc544\ub9c8\ub3c4 \ubaa9\uc131 \ub54c\ubb38\uc5d0 \ud769\uc5b4\uc9c4 \uc18c\ud589\uc131\ub300\uc5d0 \uc788\ub358 \uc791\uc740 \ubbf8\ud589\uc131 \ub610\ub294 \uc6d0\uc2dc \ud589\uc131\uc774 \uc9c0\uad6c\ub85c \uc6b4\ubc18\ud558\uc600\uc744 \uac83\uc774\ub2e4. 2006\ub144 \ubc1c\uacac\ub41c \uc8fc\ub760 \ud61c\uc131 \uc9d1\ub2e8\uc740 \uc9c0\uad6c\uc5d0 \ubb3c\uc744 \uacf5\uae09\ud588\ub358 \uc720\ub825\ud55c \uc6d0\ucc9c\uc73c\ub85c \uc5ec\uaca8\uc9c4\ub2e4. \ubc18\ub300\ub85c \uce74\uc774\ud37c \ub300 \ud639\uc740 \ub354 \uba3c \uacf3\uc5d0\uc11c \uc628 \ud61c\uc131\ub4e4\uc774 \uc9c0\uad6c\uc5d0 \uac00\uc838\uc628 \ubb3c\uc758 \uc591\uc740 \ud604\uc7ac \uc9c0\uad6c \uc0c1 \ubb3c\uc758 \uc591\uc758 6\ud37c\uc13c\ud2b8\ub97c \ub118\uc9c0 \uc54a\ub294\ub2e4. \ud3ed\ub113\uac8c \uc778\uc815\ub418\uc9c0\ub294 \ubabb\ud558\uace0 \uc788\uc73c\ub098, \ubc30\uc885\ubc1c\ub2ec\uc124\uc5d0 \ub530\ub974\uba74, \uc0dd\uba85\uccb4\ub3c4 \uc774\ub7f0 \uc2dd\uc73c\ub85c \ubb3c\uacfc \ud568\uaed8 \uc9c0\uad6c\uc5d0 \ub3c4\ucc29\ud588\uc744 \uac83\uc73c\ub85c \ubcf4\uace0 \uc788\ub2e4.", "answer": "\ubc30\uc885\ubc1c\ub2ec\uc124", "sentence": "\ud3ed\ub113\uac8c \uc778\uc815\ub418\uc9c0\ub294 \ubabb\ud558\uace0 \uc788\uc73c\ub098, \ubc30\uc885\ubc1c\ub2ec\uc124 \uc5d0", "paragraph_sentence": "\ub0b4\ud589\uc131\uc5d0 \uac70\ub300 \ucda9\ub3cc\uc774 \uc77c\uc5b4\ub098\ub358 \uc2dc\uae30, \ub0b4\ud589\uc131\uc73c\ub85c \ud769\uc5b4\uc838 \ub0a0\uc544 \ub4e4\uc5b4\uc628 \uc18c\ud589\uc131 \ubb3c\uc9c8\ub85c\ubd80\ud130 \uc9c0\uad6c\uc5d0 \ubb3c(~6\u00d710^\ud0ac\ub85c\uadf8\ub7a8)\uc774 \uacf5\uae09\ub418\uc5c8\uc744 \uac83\uc73c\ub85c \ubcf4\uc778\ub2e4. \ubb3c\uc740 \ud718\ubc1c\uc131\uc774 \uac15\ud574\uc11c \uc9c0\uad6c\uac00 \uc0dd\uaca8\ub09c \uc7a5\uc18c\uc5d0\uc11c\ub294 \uc561\uccb4 \uc0c1\ud0dc\ub85c \uc874\uc7ac\ud560 \uc218 \uc5c6\uc5c8\ub2e4. \ub530\ub77c\uc11c \ubb3c\uc740 \ub354 \ucc28\uac00\uc6b4 \ubc14\uae65\ucabd \uada4\ub3c4\uc5d0\uc11c \uc628 \ucc9c\uccb4\ub97c \ud1b5\ud574 \uacf5\uae09\ub418\uc5c8\uc744 \uac83\uc774\ub2e4. \ubb3c\uc740 \uc544\ub9c8\ub3c4 \ubaa9\uc131 \ub54c\ubb38\uc5d0 \ud769\uc5b4\uc9c4 \uc18c\ud589\uc131\ub300\uc5d0 \uc788\ub358 \uc791\uc740 \ubbf8\ud589\uc131 \ub610\ub294 \uc6d0\uc2dc \ud589\uc131\uc774 \uc9c0\uad6c\ub85c \uc6b4\ubc18\ud558\uc600\uc744 \uac83\uc774\ub2e4. 2006\ub144 \ubc1c\uacac\ub41c \uc8fc\ub760 \ud61c\uc131 \uc9d1\ub2e8\uc740 \uc9c0\uad6c\uc5d0 \ubb3c\uc744 \uacf5\uae09\ud588\ub358 \uc720\ub825\ud55c \uc6d0\ucc9c\uc73c\ub85c \uc5ec\uaca8\uc9c4\ub2e4. \ubc18\ub300\ub85c \uce74\uc774\ud37c \ub300 \ud639\uc740 \ub354 \uba3c \uacf3\uc5d0\uc11c \uc628 \ud61c\uc131\ub4e4\uc774 \uc9c0\uad6c\uc5d0 \uac00\uc838\uc628 \ubb3c\uc758 \uc591\uc740 \ud604\uc7ac \uc9c0\uad6c \uc0c1 \ubb3c\uc758 \uc591\uc758 6\ud37c\uc13c\ud2b8\ub97c \ub118\uc9c0 \uc54a\ub294\ub2e4. \ud3ed\ub113\uac8c \uc778\uc815\ub418\uc9c0\ub294 \ubabb\ud558\uace0 \uc788\uc73c\ub098, \ubc30\uc885\ubc1c\ub2ec\uc124 \uc5d0 \ub530\ub974\uba74, \uc0dd\uba85\uccb4\ub3c4 \uc774\ub7f0 \uc2dd\uc73c\ub85c \ubb3c\uacfc \ud568\uaed8 \uc9c0\uad6c\uc5d0 \ub3c4\ucc29\ud588\uc744 \uac83\uc73c\ub85c \ubcf4\uace0 \uc788\ub2e4.", "paragraph_answer": "\ub0b4\ud589\uc131\uc5d0 \uac70\ub300 \ucda9\ub3cc\uc774 \uc77c\uc5b4\ub098\ub358 \uc2dc\uae30, \ub0b4\ud589\uc131\uc73c\ub85c \ud769\uc5b4\uc838 \ub0a0\uc544 \ub4e4\uc5b4\uc628 \uc18c\ud589\uc131 \ubb3c\uc9c8\ub85c\ubd80\ud130 \uc9c0\uad6c\uc5d0 \ubb3c(~6\u00d710^\ud0ac\ub85c\uadf8\ub7a8)\uc774 \uacf5\uae09\ub418\uc5c8\uc744 \uac83\uc73c\ub85c \ubcf4\uc778\ub2e4. \ubb3c\uc740 \ud718\ubc1c\uc131\uc774 \uac15\ud574\uc11c \uc9c0\uad6c\uac00 \uc0dd\uaca8\ub09c \uc7a5\uc18c\uc5d0\uc11c\ub294 \uc561\uccb4 \uc0c1\ud0dc\ub85c \uc874\uc7ac\ud560 \uc218 \uc5c6\uc5c8\ub2e4. \ub530\ub77c\uc11c \ubb3c\uc740 \ub354 \ucc28\uac00\uc6b4 \ubc14\uae65\ucabd \uada4\ub3c4\uc5d0\uc11c \uc628 \ucc9c\uccb4\ub97c \ud1b5\ud574 \uacf5\uae09\ub418\uc5c8\uc744 \uac83\uc774\ub2e4. \ubb3c\uc740 \uc544\ub9c8\ub3c4 \ubaa9\uc131 \ub54c\ubb38\uc5d0 \ud769\uc5b4\uc9c4 \uc18c\ud589\uc131\ub300\uc5d0 \uc788\ub358 \uc791\uc740 \ubbf8\ud589\uc131 \ub610\ub294 \uc6d0\uc2dc \ud589\uc131\uc774 \uc9c0\uad6c\ub85c \uc6b4\ubc18\ud558\uc600\uc744 \uac83\uc774\ub2e4. 2006\ub144 \ubc1c\uacac\ub41c \uc8fc\ub760 \ud61c\uc131 \uc9d1\ub2e8\uc740 \uc9c0\uad6c\uc5d0 \ubb3c\uc744 \uacf5\uae09\ud588\ub358 \uc720\ub825\ud55c \uc6d0\ucc9c\uc73c\ub85c \uc5ec\uaca8\uc9c4\ub2e4. \ubc18\ub300\ub85c \uce74\uc774\ud37c \ub300 \ud639\uc740 \ub354 \uba3c \uacf3\uc5d0\uc11c \uc628 \ud61c\uc131\ub4e4\uc774 \uc9c0\uad6c\uc5d0 \uac00\uc838\uc628 \ubb3c\uc758 \uc591\uc740 \ud604\uc7ac \uc9c0\uad6c \uc0c1 \ubb3c\uc758 \uc591\uc758 6\ud37c\uc13c\ud2b8\ub97c \ub118\uc9c0 \uc54a\ub294\ub2e4. \ud3ed\ub113\uac8c \uc778\uc815\ub418\uc9c0\ub294 \ubabb\ud558\uace0 \uc788\uc73c\ub098, \ubc30\uc885\ubc1c\ub2ec\uc124 \uc5d0 \ub530\ub974\uba74, \uc0dd\uba85\uccb4\ub3c4 \uc774\ub7f0 \uc2dd\uc73c\ub85c \ubb3c\uacfc \ud568\uaed8 \uc9c0\uad6c\uc5d0 \ub3c4\ucc29\ud588\uc744 \uac83\uc73c\ub85c \ubcf4\uace0 \uc788\ub2e4.", "sentence_answer": "\ud3ed\ub113\uac8c \uc778\uc815\ub418\uc9c0\ub294 \ubabb\ud558\uace0 \uc788\uc73c\ub098, \ubc30\uc885\ubc1c\ub2ec\uc124 \uc5d0", "paragraph_id": "5911343-10-1", "paragraph_question": "question: \uc0dd\uba85\uccb4\ub3c4 \ubb3c\uacfc \ud568\uaed8 \uc9c0\uad6c\uc5d0 \ub3c4\ucc29\ud588\uc744\uaebc\ub77c\uace0 \ubcf4\uace0 \uc788\ub294 \uc124\uc740?, context: \ub0b4\ud589\uc131\uc5d0 \uac70\ub300 \ucda9\ub3cc\uc774 \uc77c\uc5b4\ub098\ub358 \uc2dc\uae30, \ub0b4\ud589\uc131\uc73c\ub85c \ud769\uc5b4\uc838 \ub0a0\uc544 \ub4e4\uc5b4\uc628 \uc18c\ud589\uc131 \ubb3c\uc9c8\ub85c\ubd80\ud130 \uc9c0\uad6c\uc5d0 \ubb3c(~6\u00d710^\ud0ac\ub85c\uadf8\ub7a8)\uc774 \uacf5\uae09\ub418\uc5c8\uc744 \uac83\uc73c\ub85c \ubcf4\uc778\ub2e4. \ubb3c\uc740 \ud718\ubc1c\uc131\uc774 \uac15\ud574\uc11c \uc9c0\uad6c\uac00 \uc0dd\uaca8\ub09c \uc7a5\uc18c\uc5d0\uc11c\ub294 \uc561\uccb4 \uc0c1\ud0dc\ub85c \uc874\uc7ac\ud560 \uc218 \uc5c6\uc5c8\ub2e4. \ub530\ub77c\uc11c \ubb3c\uc740 \ub354 \ucc28\uac00\uc6b4 \ubc14\uae65\ucabd \uada4\ub3c4\uc5d0\uc11c \uc628 \ucc9c\uccb4\ub97c \ud1b5\ud574 \uacf5\uae09\ub418\uc5c8\uc744 \uac83\uc774\ub2e4. \ubb3c\uc740 \uc544\ub9c8\ub3c4 \ubaa9\uc131 \ub54c\ubb38\uc5d0 \ud769\uc5b4\uc9c4 \uc18c\ud589\uc131\ub300\uc5d0 \uc788\ub358 \uc791\uc740 \ubbf8\ud589\uc131 \ub610\ub294 \uc6d0\uc2dc \ud589\uc131\uc774 \uc9c0\uad6c\ub85c \uc6b4\ubc18\ud558\uc600\uc744 \uac83\uc774\ub2e4. 2006\ub144 \ubc1c\uacac\ub41c \uc8fc\ub760 \ud61c\uc131 \uc9d1\ub2e8\uc740 \uc9c0\uad6c\uc5d0 \ubb3c\uc744 \uacf5\uae09\ud588\ub358 \uc720\ub825\ud55c \uc6d0\ucc9c\uc73c\ub85c \uc5ec\uaca8\uc9c4\ub2e4. \ubc18\ub300\ub85c \uce74\uc774\ud37c \ub300 \ud639\uc740 \ub354 \uba3c \uacf3\uc5d0\uc11c \uc628 \ud61c\uc131\ub4e4\uc774 \uc9c0\uad6c\uc5d0 \uac00\uc838\uc628 \ubb3c\uc758 \uc591\uc740 \ud604\uc7ac \uc9c0\uad6c \uc0c1 \ubb3c\uc758 \uc591\uc758 6\ud37c\uc13c\ud2b8\ub97c \ub118\uc9c0 \uc54a\ub294\ub2e4. \ud3ed\ub113\uac8c \uc778\uc815\ub418\uc9c0\ub294 \ubabb\ud558\uace0 \uc788\uc73c\ub098, \ubc30\uc885\ubc1c\ub2ec\uc124\uc5d0 \ub530\ub974\uba74, \uc0dd\uba85\uccb4\ub3c4 \uc774\ub7f0 \uc2dd\uc73c\ub85c \ubb3c\uacfc \ud568\uaed8 \uc9c0\uad6c\uc5d0 \ub3c4\ucc29\ud588\uc744 \uac83\uc73c\ub85c \ubcf4\uace0 \uc788\ub2e4."} {"question": "\uccb4\ub85c\ud0a4\uc871 \ubc18\ub780 \uc2e4\ud328\uc758 \uc774\uc720\ub294?", "paragraph": "\uc560\uba38\uc2a4\ud2b8\ub294 \ub610\ud55c \uc778\ub514\uc5b8\ub4e4\uc5d0\uac8c \ud310\ub9e4\ud558\ub294 \ud0c4\uc57d\uc774\ub098 \ud654\uc57d\uc758 \uc591\uc744 \uc81c\ud55c\ud558\uae30 \uc2dc\uc791\ud588\ub2e4. \ud504\ub791\uc2a4\ub294 \ud56d\uc0c1 \uc774\ub7ec\ud55c \ubb3c\uc790\ub97c \uc0ac\uc6a9\ud560 \uc218 \uc788\uac8c \ud588\uc9c0\ub9cc, \uc560\uba38\uc2a4\ud2b8\ub294 \ub3d9\ub9f9\uc790\uc600\ub358 \uccb4\ub85c\ud0a4\uc871\uc758 1761\ub144\uc758 \ubc18\ub780 \uc774\ud6c4 \uc778\ub514\uc5b8\ub4e4\uc744 \ubbff\uc9c0 \uc54a\uc558\ub2e4. \uccb4\ub85c\ud0a4\uc871\uc758 \ubc18\ub780\uc740 \ud654\uc57d \ubd80\uc871\uc73c\ub85c \uc2e4\ud328\ud588\uae30 \ub54c\ubb38\uc5d0, \uc560\uba38\uc2a4\ud2b8\ub294 \ud654\uc57d\uc744 \ubd84\ubc30\ud558\ub294 \uc591\uc744 \uc5b5\uc81c\ud558\uba74 \ubbf8\ub798\uc758 \ub3c4\uc804\uc744 \ub9c9\uc744 \uc218 \uc788\uc744 \uac83\uc73c\ub85c \ubd24\ub2e4. \uc778\ub514\uc5b8 \ub0a8\uc790\ub4e4\uc740 \uac00\uc871\uc744 \uc704\ud574 \uc2dd\ub7c9\uc744 \ud655\ubcf4\ud574\uc57c \ud588\uc73c\uba70, \ubaa8\ud53c \ubb34\uc5ed\uc744 \uc704\ud574 \ud0c4\uc57d\uc740 \ud544\uc218\ud488\uc774\uc5c8\uace0, \uc774\ub7ec\ud55c \uc81c\ud55c \uc815\ucc45\uc740 \uadf8\ub4e4\uc5d0\uac8c \uacf5\ubd84\uacfc \uace4\uad81\uc744 \uac00\uc838\uc654\ub2e4. \uadf8\ub4e4\uc740 \uc601\uad6d\uad70\uc774 \uc778\ub514\uc5b8\ub4e4\uc5d0\uac8c \uc804\uc7c1\uc744 \uc900\ube44\ud558\uc9c0 \ubabb\ud558\ub3c4\ub85d \ubb34\uc7a5\uc744 \ud574\uc81c\uc2dc\ud0a4\ub294 \uac83\uc774\ub77c\uace0 \ubbff\uae30 \uc2dc\uc791\ud588\ub2e4. \uc778\ub514\uc5b8 \ub2f4\ub2f9 \ubd80\uc11c\uc758 \uac10\ucc30\uad00\uc774\uc5c8\ub358 \uc70c\ub9ac\uc5c4 \uc874\uc2a8 \uacbd\uc740 \uc560\uba38\uc2a4\ud2b8\uc5d0\uac8c \uc120\ubb3c\uc774\ub098 \ud654\uc57d\uc744 \uc81c\ud55c\ud558\ub294 \uc815\ucc45\uc758 \uc704\ud5d8\uc131\uc5d0 \ub300\ud574 \uacbd\uace0\ub97c \ud588\uc9c0\ub9cc, \ud5db\uc218\uace0\ub85c \ub05d\ub0ac\ub2e4.", "answer": "\ud654\uc57d \ubd80\uc871", "sentence": "\uccb4\ub85c\ud0a4\uc871\uc758 \ubc18\ub780\uc740 \ud654\uc57d \ubd80\uc871 \uc73c\ub85c \uc2e4\ud328\ud588\uae30 \ub54c\ubb38\uc5d0, \uc560\uba38\uc2a4\ud2b8\ub294 \ud654\uc57d\uc744 \ubd84\ubc30\ud558\ub294 \uc591\uc744 \uc5b5\uc81c\ud558\uba74 \ubbf8\ub798\uc758 \ub3c4\uc804\uc744 \ub9c9\uc744 \uc218 \uc788\uc744 \uac83\uc73c\ub85c \ubd24\ub2e4.", "paragraph_sentence": "\uc560\uba38\uc2a4\ud2b8\ub294 \ub610\ud55c \uc778\ub514\uc5b8\ub4e4\uc5d0\uac8c \ud310\ub9e4\ud558\ub294 \ud0c4\uc57d\uc774\ub098 \ud654\uc57d\uc758 \uc591\uc744 \uc81c\ud55c\ud558\uae30 \uc2dc\uc791\ud588\ub2e4. \ud504\ub791\uc2a4\ub294 \ud56d\uc0c1 \uc774\ub7ec\ud55c \ubb3c\uc790\ub97c \uc0ac\uc6a9\ud560 \uc218 \uc788\uac8c \ud588\uc9c0\ub9cc, \uc560\uba38\uc2a4\ud2b8\ub294 \ub3d9\ub9f9\uc790\uc600\ub358 \uccb4\ub85c\ud0a4\uc871\uc758 1761\ub144\uc758 \ubc18\ub780 \uc774\ud6c4 \uc778\ub514\uc5b8\ub4e4\uc744 \ubbff\uc9c0 \uc54a\uc558\ub2e4. \uccb4\ub85c\ud0a4\uc871\uc758 \ubc18\ub780\uc740 \ud654\uc57d \ubd80\uc871 \uc73c\ub85c \uc2e4\ud328\ud588\uae30 \ub54c\ubb38\uc5d0, \uc560\uba38\uc2a4\ud2b8\ub294 \ud654\uc57d\uc744 \ubd84\ubc30\ud558\ub294 \uc591\uc744 \uc5b5\uc81c\ud558\uba74 \ubbf8\ub798\uc758 \ub3c4\uc804\uc744 \ub9c9\uc744 \uc218 \uc788\uc744 \uac83\uc73c\ub85c \ubd24\ub2e4. \uc778\ub514\uc5b8 \ub0a8\uc790\ub4e4\uc740 \uac00\uc871\uc744 \uc704\ud574 \uc2dd\ub7c9\uc744 \ud655\ubcf4\ud574\uc57c \ud588\uc73c\uba70, \ubaa8\ud53c \ubb34\uc5ed\uc744 \uc704\ud574 \ud0c4\uc57d\uc740 \ud544\uc218\ud488\uc774\uc5c8\uace0, \uc774\ub7ec\ud55c \uc81c\ud55c \uc815\ucc45\uc740 \uadf8\ub4e4\uc5d0\uac8c \uacf5\ubd84\uacfc \uace4\uad81\uc744 \uac00\uc838\uc654\ub2e4. \uadf8\ub4e4\uc740 \uc601\uad6d\uad70\uc774 \uc778\ub514\uc5b8\ub4e4\uc5d0\uac8c \uc804\uc7c1\uc744 \uc900\ube44\ud558\uc9c0 \ubabb\ud558\ub3c4\ub85d \ubb34\uc7a5\uc744 \ud574\uc81c\uc2dc\ud0a4\ub294 \uac83\uc774\ub77c\uace0 \ubbff\uae30 \uc2dc\uc791\ud588\ub2e4. \uc778\ub514\uc5b8 \ub2f4\ub2f9 \ubd80\uc11c\uc758 \uac10\ucc30\uad00\uc774\uc5c8\ub358 \uc70c\ub9ac\uc5c4 \uc874\uc2a8 \uacbd\uc740 \uc560\uba38\uc2a4\ud2b8\uc5d0\uac8c \uc120\ubb3c\uc774\ub098 \ud654\uc57d\uc744 \uc81c\ud55c\ud558\ub294 \uc815\ucc45\uc758 \uc704\ud5d8\uc131\uc5d0 \ub300\ud574 \uacbd\uace0\ub97c \ud588\uc9c0\ub9cc, \ud5db\uc218\uace0\ub85c \ub05d\ub0ac\ub2e4.", "paragraph_answer": "\uc560\uba38\uc2a4\ud2b8\ub294 \ub610\ud55c \uc778\ub514\uc5b8\ub4e4\uc5d0\uac8c \ud310\ub9e4\ud558\ub294 \ud0c4\uc57d\uc774\ub098 \ud654\uc57d\uc758 \uc591\uc744 \uc81c\ud55c\ud558\uae30 \uc2dc\uc791\ud588\ub2e4. \ud504\ub791\uc2a4\ub294 \ud56d\uc0c1 \uc774\ub7ec\ud55c \ubb3c\uc790\ub97c \uc0ac\uc6a9\ud560 \uc218 \uc788\uac8c \ud588\uc9c0\ub9cc, \uc560\uba38\uc2a4\ud2b8\ub294 \ub3d9\ub9f9\uc790\uc600\ub358 \uccb4\ub85c\ud0a4\uc871\uc758 1761\ub144\uc758 \ubc18\ub780 \uc774\ud6c4 \uc778\ub514\uc5b8\ub4e4\uc744 \ubbff\uc9c0 \uc54a\uc558\ub2e4. \uccb4\ub85c\ud0a4\uc871\uc758 \ubc18\ub780\uc740 \ud654\uc57d \ubd80\uc871 \uc73c\ub85c \uc2e4\ud328\ud588\uae30 \ub54c\ubb38\uc5d0, \uc560\uba38\uc2a4\ud2b8\ub294 \ud654\uc57d\uc744 \ubd84\ubc30\ud558\ub294 \uc591\uc744 \uc5b5\uc81c\ud558\uba74 \ubbf8\ub798\uc758 \ub3c4\uc804\uc744 \ub9c9\uc744 \uc218 \uc788\uc744 \uac83\uc73c\ub85c \ubd24\ub2e4. \uc778\ub514\uc5b8 \ub0a8\uc790\ub4e4\uc740 \uac00\uc871\uc744 \uc704\ud574 \uc2dd\ub7c9\uc744 \ud655\ubcf4\ud574\uc57c \ud588\uc73c\uba70, \ubaa8\ud53c \ubb34\uc5ed\uc744 \uc704\ud574 \ud0c4\uc57d\uc740 \ud544\uc218\ud488\uc774\uc5c8\uace0, \uc774\ub7ec\ud55c \uc81c\ud55c \uc815\ucc45\uc740 \uadf8\ub4e4\uc5d0\uac8c \uacf5\ubd84\uacfc \uace4\uad81\uc744 \uac00\uc838\uc654\ub2e4. \uadf8\ub4e4\uc740 \uc601\uad6d\uad70\uc774 \uc778\ub514\uc5b8\ub4e4\uc5d0\uac8c \uc804\uc7c1\uc744 \uc900\ube44\ud558\uc9c0 \ubabb\ud558\ub3c4\ub85d \ubb34\uc7a5\uc744 \ud574\uc81c\uc2dc\ud0a4\ub294 \uac83\uc774\ub77c\uace0 \ubbff\uae30 \uc2dc\uc791\ud588\ub2e4. \uc778\ub514\uc5b8 \ub2f4\ub2f9 \ubd80\uc11c\uc758 \uac10\ucc30\uad00\uc774\uc5c8\ub358 \uc70c\ub9ac\uc5c4 \uc874\uc2a8 \uacbd\uc740 \uc560\uba38\uc2a4\ud2b8\uc5d0\uac8c \uc120\ubb3c\uc774\ub098 \ud654\uc57d\uc744 \uc81c\ud55c\ud558\ub294 \uc815\ucc45\uc758 \uc704\ud5d8\uc131\uc5d0 \ub300\ud574 \uacbd\uace0\ub97c \ud588\uc9c0\ub9cc, \ud5db\uc218\uace0\ub85c \ub05d\ub0ac\ub2e4.", "sentence_answer": "\uccb4\ub85c\ud0a4\uc871\uc758 \ubc18\ub780\uc740 \ud654\uc57d \ubd80\uc871 \uc73c\ub85c \uc2e4\ud328\ud588\uae30 \ub54c\ubb38\uc5d0, \uc560\uba38\uc2a4\ud2b8\ub294 \ud654\uc57d\uc744 \ubd84\ubc30\ud558\ub294 \uc591\uc744 \uc5b5\uc81c\ud558\uba74 \ubbf8\ub798\uc758 \ub3c4\uc804\uc744 \ub9c9\uc744 \uc218 \uc788\uc744 \uac83\uc73c\ub85c \ubd24\ub2e4.", "paragraph_id": "6567978-5-1", "paragraph_question": "question: \uccb4\ub85c\ud0a4\uc871 \ubc18\ub780 \uc2e4\ud328\uc758 \uc774\uc720\ub294?, context: \uc560\uba38\uc2a4\ud2b8\ub294 \ub610\ud55c \uc778\ub514\uc5b8\ub4e4\uc5d0\uac8c \ud310\ub9e4\ud558\ub294 \ud0c4\uc57d\uc774\ub098 \ud654\uc57d\uc758 \uc591\uc744 \uc81c\ud55c\ud558\uae30 \uc2dc\uc791\ud588\ub2e4. \ud504\ub791\uc2a4\ub294 \ud56d\uc0c1 \uc774\ub7ec\ud55c \ubb3c\uc790\ub97c \uc0ac\uc6a9\ud560 \uc218 \uc788\uac8c \ud588\uc9c0\ub9cc, \uc560\uba38\uc2a4\ud2b8\ub294 \ub3d9\ub9f9\uc790\uc600\ub358 \uccb4\ub85c\ud0a4\uc871\uc758 1761\ub144\uc758 \ubc18\ub780 \uc774\ud6c4 \uc778\ub514\uc5b8\ub4e4\uc744 \ubbff\uc9c0 \uc54a\uc558\ub2e4. \uccb4\ub85c\ud0a4\uc871\uc758 \ubc18\ub780\uc740 \ud654\uc57d \ubd80\uc871\uc73c\ub85c \uc2e4\ud328\ud588\uae30 \ub54c\ubb38\uc5d0, \uc560\uba38\uc2a4\ud2b8\ub294 \ud654\uc57d\uc744 \ubd84\ubc30\ud558\ub294 \uc591\uc744 \uc5b5\uc81c\ud558\uba74 \ubbf8\ub798\uc758 \ub3c4\uc804\uc744 \ub9c9\uc744 \uc218 \uc788\uc744 \uac83\uc73c\ub85c \ubd24\ub2e4. \uc778\ub514\uc5b8 \ub0a8\uc790\ub4e4\uc740 \uac00\uc871\uc744 \uc704\ud574 \uc2dd\ub7c9\uc744 \ud655\ubcf4\ud574\uc57c \ud588\uc73c\uba70, \ubaa8\ud53c \ubb34\uc5ed\uc744 \uc704\ud574 \ud0c4\uc57d\uc740 \ud544\uc218\ud488\uc774\uc5c8\uace0, \uc774\ub7ec\ud55c \uc81c\ud55c \uc815\ucc45\uc740 \uadf8\ub4e4\uc5d0\uac8c \uacf5\ubd84\uacfc \uace4\uad81\uc744 \uac00\uc838\uc654\ub2e4. \uadf8\ub4e4\uc740 \uc601\uad6d\uad70\uc774 \uc778\ub514\uc5b8\ub4e4\uc5d0\uac8c \uc804\uc7c1\uc744 \uc900\ube44\ud558\uc9c0 \ubabb\ud558\ub3c4\ub85d \ubb34\uc7a5\uc744 \ud574\uc81c\uc2dc\ud0a4\ub294 \uac83\uc774\ub77c\uace0 \ubbff\uae30 \uc2dc\uc791\ud588\ub2e4. \uc778\ub514\uc5b8 \ub2f4\ub2f9 \ubd80\uc11c\uc758 \uac10\ucc30\uad00\uc774\uc5c8\ub358 \uc70c\ub9ac\uc5c4 \uc874\uc2a8 \uacbd\uc740 \uc560\uba38\uc2a4\ud2b8\uc5d0\uac8c \uc120\ubb3c\uc774\ub098 \ud654\uc57d\uc744 \uc81c\ud55c\ud558\ub294 \uc815\ucc45\uc758 \uc704\ud5d8\uc131\uc5d0 \ub300\ud574 \uacbd\uace0\ub97c \ud588\uc9c0\ub9cc, \ud5db\uc218\uace0\ub85c \ub05d\ub0ac\ub2e4."} {"question": "\ubbf8\uad6d \uba55\uc2dc\ucf54 \uc804\uc7c1\uc774 \ub05d\ub09c \uac83\uc740 \uc5b8\uc81c\uc778\uac00?", "paragraph": "\uc774 \uae38\uc744 1849\ub144 \uc774\uc804\uc5d0 \uc774\uc6a9\ud55c \uc0ac\ub78c\ub4e4\uc740 \uc57d 2,700\uba85\uc774\uc5c8\ub2e4. \uc774\ub7ec\ud55c \uac1c\ucc99\uc790\ub4e4\uc740 1846\ub144\uacfc 1847\ub144\uc5d0 \uc874 C. \ud504\ub9ac\ubaac\ud2b8\uac00 \uce98\ub9ac\ud3ec\ub2c8\uc544 \ub300\ub300\uc758 \uc77c\uc6d0\uc73c\ub85c\uc11c \uce98\ub9ac\ud3ec\ub2c8\uc544\ub97c \ubbf8\uad6d\uc758 \uc601\ud1a0\ub85c \ub3c4\ubaa8\ud560 \ub54c \ucd94\uc9c4\uc790\uac00 \ub418\uc5c8\ub2e4. 1845\ub144\uacbd, \uc774 \uc9c0\uc5ed\uc5d0\ub294 \u2018\uce74\ub974\ud3ec\ub974\ub2c8\uc624\u2019\ub77c\uace0 \ubd88\ub9ac\ub294 \uc6d0\uc8fc\ubbfc\uc774 \uc788\uc5c8\uc73c\uba70, \uc57d 1,500\uba85\uc758 \uc131\uc778 \ub0a8\uc131\uacfc 6,500\uba85\uc758 \uc5ec\uc131, \uc5b4\ub9b0\uc774\ub85c \uad6c\uc131\ub41c \uc9d1\ub2e8\uc774 \ub85c\uc2a4\uc564\uc824\ub808\uc2a4 \uc8fc\ubcc0 \ud604\uc7ac \ub0a8\ubd80 \uce98\ub9ac\ud3ec\ub2c8\uc544 \uc9c0\uc5ed\uc5d0 \ub300\ubd80\ubd84 \uac70\uc8fc\ud558\uc600\ub2e4. \uc774\ubbfc\uc790 \ub300\ubd80\ubd84(\uac70\uc758 \ubaa8\ub450\uac00 \uc131\uc778 \ub0a8\uc131)\uc740 \uce98\ub9ac\ud3ec\ub2c8\uc544\uc758 \ubd81\ucabd\uc5d0 \uc808\ubc18\uc774 \uc0b4\uc558\ub2e4. 1845\ub144\uc5d0 \ubbf8\uad6d\uc774 \uba55\uc2dc\ucf54\ub85c\ubd80\ud130 \uce98\ub9ac\ud3ec\ub2c8\uc544\uc758 \uc9c0\ubc30\uad8c\uc744 \ube7c\uc557\uace0, 1847\ub144 1\uc6d4 14\uc77c\uc5d0 \u3008\uce74\ud6c4\uc5d4\uac00 \uc870\uc57d\u3009\uc774 \uccb4\uacb0\ub418\uc5c8\uc744 \ub54c, \uce98\ub9ac\ud3ec\ub2c8\uc544\uc758 \uc18c\uaddc\ubaa8 \ubb34\uc7a5 \uc800\ud56d\ub3c4 \ub05d\uc774 \ub0ac\ub2e4. \ubbf8\uad6d \uba55\uc2dc\ucf54 \uc804\uc7c1\uc744 \ub05d\ub0b8 1848\ub144 2\uc6d4 \u3008\uacfc\ub2ec\ub8e8\ud398 \uc774\ub2ec\uace0 \uc870\uc57d\u3009\uc5d0 \uc758\ud574 \uce98\ub9ac\ud3ec\ub2c8\uc544\ub294 \uba55\uc2dc\ucf54\uc5d0\uc11c (\uc720\uc0c1\uc73c\ub85c) \ubbf8\uad6d \uc601\ud1a0\ub85c \ubc14\ub00c\uc5c8\ub2e4. 1848\ub144 1\uc6d4\uc5d0 \uae08\uc774 \ubc1c\uacac\ub41c \ud6c4 \uce98\ub9ac\ud3ec\ub2c8\uc544 \uace8\ub4dc\ub7ec\uc2dc\uc5d0 \uad00\ud55c \uc18c\ubb38\uc774 \ube60\ub974\uac8c \ud37c\uc838 \ub098\uac14\ub2e4. 1848\ub144 \ud6c4\ubc18\uc5d0\ub294 25\ub9cc\uba85\uc774 \ub118\ub294 \uc0ac\uc5c5\uac00, \ub18d\ubd80, \uac1c\ucc99\uc790 \ubc0f \uad11\ubd80\ub4e4\uc774 \uce98\ub9ac\ud3ec\ub2c8\uc544 \uac00\ub3c4\ub97c \ub118\uc5b4 \uce98\ub9ac\ud3ec\ub2c8\uc544\uc5d0 \ub3c4\ucc29\ud588\ub2e4. \ud1b5\ud589\uc790\ub4e4\uc774 \ub9e4\uc6b0 \ub9ce\uc558\uae30 \ub54c\ubb38\uc5d0, 2\ub144\ub3c4 \ubabb\ub418\uc5b4 \ud30c\ub098\ub9c8 \uc9c0\ud611\uc744 \uac70\uccd0 \ub610\ub294 \ucf00\uc774\ud504 \ud63c\uc744 \ub3cc\uc544 \ubc14\ub2f7\uae38 \ubc29\ubb38\uc790\ub4e4\uae4c\uc9c0 \ud569\uccd0 \uc5c4\uccad\ub09c \uc778\uad6c\uac00 \ubd88\uc5b4\ub098\uc790, 1850\ub144 \uce98\ub9ac\ud3ec\ub2c8\uc544\ub294 \ubbf8\uad6d\uc758 31\ubc88\uc9f8 \uc8fc\ub85c \uc2b9\uaca9\ud558\uc600\ub2e4.", "answer": "1848\ub144 2\uc6d4", "sentence": "\ubbf8\uad6d \uba55\uc2dc\ucf54 \uc804\uc7c1\uc744 \ub05d\ub0b8 1848\ub144 2\uc6d4 \u3008\uacfc\ub2ec\ub8e8\ud398 \uc774\ub2ec\uace0 \uc870\uc57d\u3009\uc5d0 \uc758\ud574 \uce98\ub9ac\ud3ec\ub2c8\uc544\ub294 \uba55\uc2dc\ucf54\uc5d0\uc11c (\uc720\uc0c1\uc73c\ub85c) \ubbf8\uad6d \uc601\ud1a0\ub85c \ubc14\ub00c\uc5c8\ub2e4.", "paragraph_sentence": "\uc774 \uae38\uc744 1849\ub144 \uc774\uc804\uc5d0 \uc774\uc6a9\ud55c \uc0ac\ub78c\ub4e4\uc740 \uc57d 2,700\uba85\uc774\uc5c8\ub2e4. \uc774\ub7ec\ud55c \uac1c\ucc99\uc790\ub4e4\uc740 1846\ub144\uacfc 1847\ub144\uc5d0 \uc874 C. \ud504\ub9ac\ubaac\ud2b8\uac00 \uce98\ub9ac\ud3ec\ub2c8\uc544 \ub300\ub300\uc758 \uc77c\uc6d0\uc73c\ub85c\uc11c \uce98\ub9ac\ud3ec\ub2c8\uc544\ub97c \ubbf8\uad6d\uc758 \uc601\ud1a0\ub85c \ub3c4\ubaa8\ud560 \ub54c \ucd94\uc9c4\uc790\uac00 \ub418\uc5c8\ub2e4. 1845\ub144\uacbd, \uc774 \uc9c0\uc5ed\uc5d0\ub294 \u2018\uce74\ub974\ud3ec\ub974\ub2c8\uc624\u2019\ub77c\uace0 \ubd88\ub9ac\ub294 \uc6d0\uc8fc\ubbfc\uc774 \uc788\uc5c8\uc73c\uba70, \uc57d 1,500\uba85\uc758 \uc131\uc778 \ub0a8\uc131\uacfc 6,500\uba85\uc758 \uc5ec\uc131, \uc5b4\ub9b0\uc774\ub85c \uad6c\uc131\ub41c \uc9d1\ub2e8\uc774 \ub85c\uc2a4\uc564\uc824\ub808\uc2a4 \uc8fc\ubcc0 \ud604\uc7ac \ub0a8\ubd80 \uce98\ub9ac\ud3ec\ub2c8\uc544 \uc9c0\uc5ed\uc5d0 \ub300\ubd80\ubd84 \uac70\uc8fc\ud558\uc600\ub2e4. \uc774\ubbfc\uc790 \ub300\ubd80\ubd84(\uac70\uc758 \ubaa8\ub450\uac00 \uc131\uc778 \ub0a8\uc131)\uc740 \uce98\ub9ac\ud3ec\ub2c8\uc544\uc758 \ubd81\ucabd\uc5d0 \uc808\ubc18\uc774 \uc0b4\uc558\ub2e4. 1845\ub144\uc5d0 \ubbf8\uad6d\uc774 \uba55\uc2dc\ucf54\ub85c\ubd80\ud130 \uce98\ub9ac\ud3ec\ub2c8\uc544\uc758 \uc9c0\ubc30\uad8c\uc744 \ube7c\uc557\uace0, 1847\ub144 1\uc6d4 14\uc77c\uc5d0 \u3008\uce74\ud6c4\uc5d4\uac00 \uc870\uc57d\u3009\uc774 \uccb4\uacb0\ub418\uc5c8\uc744 \ub54c, \uce98\ub9ac\ud3ec\ub2c8\uc544\uc758 \uc18c\uaddc\ubaa8 \ubb34\uc7a5 \uc800\ud56d\ub3c4 \ub05d\uc774 \ub0ac\ub2e4. \ubbf8\uad6d \uba55\uc2dc\ucf54 \uc804\uc7c1\uc744 \ub05d\ub0b8 1848\ub144 2\uc6d4 \u3008\uacfc\ub2ec\ub8e8\ud398 \uc774\ub2ec\uace0 \uc870\uc57d\u3009\uc5d0 \uc758\ud574 \uce98\ub9ac\ud3ec\ub2c8\uc544\ub294 \uba55\uc2dc\ucf54\uc5d0\uc11c (\uc720\uc0c1\uc73c\ub85c) \ubbf8\uad6d \uc601\ud1a0\ub85c \ubc14\ub00c\uc5c8\ub2e4. 1848\ub144 1\uc6d4\uc5d0 \uae08\uc774 \ubc1c\uacac\ub41c \ud6c4 \uce98\ub9ac\ud3ec\ub2c8\uc544 \uace8\ub4dc\ub7ec\uc2dc\uc5d0 \uad00\ud55c \uc18c\ubb38\uc774 \ube60\ub974\uac8c \ud37c\uc838 \ub098\uac14\ub2e4. 1848\ub144 \ud6c4\ubc18\uc5d0\ub294 25\ub9cc\uba85\uc774 \ub118\ub294 \uc0ac\uc5c5\uac00, \ub18d\ubd80, \uac1c\ucc99\uc790 \ubc0f \uad11\ubd80\ub4e4\uc774 \uce98\ub9ac\ud3ec\ub2c8\uc544 \uac00\ub3c4\ub97c \ub118\uc5b4 \uce98\ub9ac\ud3ec\ub2c8\uc544\uc5d0 \ub3c4\ucc29\ud588\ub2e4. \ud1b5\ud589\uc790\ub4e4\uc774 \ub9e4\uc6b0 \ub9ce\uc558\uae30 \ub54c\ubb38\uc5d0, 2\ub144\ub3c4 \ubabb\ub418\uc5b4 \ud30c\ub098\ub9c8 \uc9c0\ud611\uc744 \uac70\uccd0 \ub610\ub294 \ucf00\uc774\ud504 \ud63c\uc744 \ub3cc\uc544 \ubc14\ub2f7\uae38 \ubc29\ubb38\uc790\ub4e4\uae4c\uc9c0 \ud569\uccd0 \uc5c4\uccad\ub09c \uc778\uad6c\uac00 \ubd88\uc5b4\ub098\uc790, 1850\ub144 \uce98\ub9ac\ud3ec\ub2c8\uc544\ub294 \ubbf8\uad6d\uc758 31\ubc88\uc9f8 \uc8fc\ub85c \uc2b9\uaca9\ud558\uc600\ub2e4.", "paragraph_answer": "\uc774 \uae38\uc744 1849\ub144 \uc774\uc804\uc5d0 \uc774\uc6a9\ud55c \uc0ac\ub78c\ub4e4\uc740 \uc57d 2,700\uba85\uc774\uc5c8\ub2e4. \uc774\ub7ec\ud55c \uac1c\ucc99\uc790\ub4e4\uc740 1846\ub144\uacfc 1847\ub144\uc5d0 \uc874 C. \ud504\ub9ac\ubaac\ud2b8\uac00 \uce98\ub9ac\ud3ec\ub2c8\uc544 \ub300\ub300\uc758 \uc77c\uc6d0\uc73c\ub85c\uc11c \uce98\ub9ac\ud3ec\ub2c8\uc544\ub97c \ubbf8\uad6d\uc758 \uc601\ud1a0\ub85c \ub3c4\ubaa8\ud560 \ub54c \ucd94\uc9c4\uc790\uac00 \ub418\uc5c8\ub2e4. 1845\ub144\uacbd, \uc774 \uc9c0\uc5ed\uc5d0\ub294 \u2018\uce74\ub974\ud3ec\ub974\ub2c8\uc624\u2019\ub77c\uace0 \ubd88\ub9ac\ub294 \uc6d0\uc8fc\ubbfc\uc774 \uc788\uc5c8\uc73c\uba70, \uc57d 1,500\uba85\uc758 \uc131\uc778 \ub0a8\uc131\uacfc 6,500\uba85\uc758 \uc5ec\uc131, \uc5b4\ub9b0\uc774\ub85c \uad6c\uc131\ub41c \uc9d1\ub2e8\uc774 \ub85c\uc2a4\uc564\uc824\ub808\uc2a4 \uc8fc\ubcc0 \ud604\uc7ac \ub0a8\ubd80 \uce98\ub9ac\ud3ec\ub2c8\uc544 \uc9c0\uc5ed\uc5d0 \ub300\ubd80\ubd84 \uac70\uc8fc\ud558\uc600\ub2e4. \uc774\ubbfc\uc790 \ub300\ubd80\ubd84(\uac70\uc758 \ubaa8\ub450\uac00 \uc131\uc778 \ub0a8\uc131)\uc740 \uce98\ub9ac\ud3ec\ub2c8\uc544\uc758 \ubd81\ucabd\uc5d0 \uc808\ubc18\uc774 \uc0b4\uc558\ub2e4. 1845\ub144\uc5d0 \ubbf8\uad6d\uc774 \uba55\uc2dc\ucf54\ub85c\ubd80\ud130 \uce98\ub9ac\ud3ec\ub2c8\uc544\uc758 \uc9c0\ubc30\uad8c\uc744 \ube7c\uc557\uace0, 1847\ub144 1\uc6d4 14\uc77c\uc5d0 \u3008\uce74\ud6c4\uc5d4\uac00 \uc870\uc57d\u3009\uc774 \uccb4\uacb0\ub418\uc5c8\uc744 \ub54c, \uce98\ub9ac\ud3ec\ub2c8\uc544\uc758 \uc18c\uaddc\ubaa8 \ubb34\uc7a5 \uc800\ud56d\ub3c4 \ub05d\uc774 \ub0ac\ub2e4. \ubbf8\uad6d \uba55\uc2dc\ucf54 \uc804\uc7c1\uc744 \ub05d\ub0b8 1848\ub144 2\uc6d4 \u3008\uacfc\ub2ec\ub8e8\ud398 \uc774\ub2ec\uace0 \uc870\uc57d\u3009\uc5d0 \uc758\ud574 \uce98\ub9ac\ud3ec\ub2c8\uc544\ub294 \uba55\uc2dc\ucf54\uc5d0\uc11c (\uc720\uc0c1\uc73c\ub85c) \ubbf8\uad6d \uc601\ud1a0\ub85c \ubc14\ub00c\uc5c8\ub2e4. 1848\ub144 1\uc6d4\uc5d0 \uae08\uc774 \ubc1c\uacac\ub41c \ud6c4 \uce98\ub9ac\ud3ec\ub2c8\uc544 \uace8\ub4dc\ub7ec\uc2dc\uc5d0 \uad00\ud55c \uc18c\ubb38\uc774 \ube60\ub974\uac8c \ud37c\uc838 \ub098\uac14\ub2e4. 1848\ub144 \ud6c4\ubc18\uc5d0\ub294 25\ub9cc\uba85\uc774 \ub118\ub294 \uc0ac\uc5c5\uac00, \ub18d\ubd80, \uac1c\ucc99\uc790 \ubc0f \uad11\ubd80\ub4e4\uc774 \uce98\ub9ac\ud3ec\ub2c8\uc544 \uac00\ub3c4\ub97c \ub118\uc5b4 \uce98\ub9ac\ud3ec\ub2c8\uc544\uc5d0 \ub3c4\ucc29\ud588\ub2e4. \ud1b5\ud589\uc790\ub4e4\uc774 \ub9e4\uc6b0 \ub9ce\uc558\uae30 \ub54c\ubb38\uc5d0, 2\ub144\ub3c4 \ubabb\ub418\uc5b4 \ud30c\ub098\ub9c8 \uc9c0\ud611\uc744 \uac70\uccd0 \ub610\ub294 \ucf00\uc774\ud504 \ud63c\uc744 \ub3cc\uc544 \ubc14\ub2f7\uae38 \ubc29\ubb38\uc790\ub4e4\uae4c\uc9c0 \ud569\uccd0 \uc5c4\uccad\ub09c \uc778\uad6c\uac00 \ubd88\uc5b4\ub098\uc790, 1850\ub144 \uce98\ub9ac\ud3ec\ub2c8\uc544\ub294 \ubbf8\uad6d\uc758 31\ubc88\uc9f8 \uc8fc\ub85c \uc2b9\uaca9\ud558\uc600\ub2e4.", "sentence_answer": "\ubbf8\uad6d \uba55\uc2dc\ucf54 \uc804\uc7c1\uc744 \ub05d\ub0b8 1848\ub144 2\uc6d4 \u3008\uacfc\ub2ec\ub8e8\ud398 \uc774\ub2ec\uace0 \uc870\uc57d\u3009\uc5d0 \uc758\ud574 \uce98\ub9ac\ud3ec\ub2c8\uc544\ub294 \uba55\uc2dc\ucf54\uc5d0\uc11c (\uc720\uc0c1\uc73c\ub85c) \ubbf8\uad6d \uc601\ud1a0\ub85c \ubc14\ub00c\uc5c8\ub2e4.", "paragraph_id": "6561614-1-0", "paragraph_question": "question: \ubbf8\uad6d \uba55\uc2dc\ucf54 \uc804\uc7c1\uc774 \ub05d\ub09c \uac83\uc740 \uc5b8\uc81c\uc778\uac00?, context: \uc774 \uae38\uc744 1849\ub144 \uc774\uc804\uc5d0 \uc774\uc6a9\ud55c \uc0ac\ub78c\ub4e4\uc740 \uc57d 2,700\uba85\uc774\uc5c8\ub2e4. \uc774\ub7ec\ud55c \uac1c\ucc99\uc790\ub4e4\uc740 1846\ub144\uacfc 1847\ub144\uc5d0 \uc874 C. \ud504\ub9ac\ubaac\ud2b8\uac00 \uce98\ub9ac\ud3ec\ub2c8\uc544 \ub300\ub300\uc758 \uc77c\uc6d0\uc73c\ub85c\uc11c \uce98\ub9ac\ud3ec\ub2c8\uc544\ub97c \ubbf8\uad6d\uc758 \uc601\ud1a0\ub85c \ub3c4\ubaa8\ud560 \ub54c \ucd94\uc9c4\uc790\uac00 \ub418\uc5c8\ub2e4. 1845\ub144\uacbd, \uc774 \uc9c0\uc5ed\uc5d0\ub294 \u2018\uce74\ub974\ud3ec\ub974\ub2c8\uc624\u2019\ub77c\uace0 \ubd88\ub9ac\ub294 \uc6d0\uc8fc\ubbfc\uc774 \uc788\uc5c8\uc73c\uba70, \uc57d 1,500\uba85\uc758 \uc131\uc778 \ub0a8\uc131\uacfc 6,500\uba85\uc758 \uc5ec\uc131, \uc5b4\ub9b0\uc774\ub85c \uad6c\uc131\ub41c \uc9d1\ub2e8\uc774 \ub85c\uc2a4\uc564\uc824\ub808\uc2a4 \uc8fc\ubcc0 \ud604\uc7ac \ub0a8\ubd80 \uce98\ub9ac\ud3ec\ub2c8\uc544 \uc9c0\uc5ed\uc5d0 \ub300\ubd80\ubd84 \uac70\uc8fc\ud558\uc600\ub2e4. \uc774\ubbfc\uc790 \ub300\ubd80\ubd84(\uac70\uc758 \ubaa8\ub450\uac00 \uc131\uc778 \ub0a8\uc131)\uc740 \uce98\ub9ac\ud3ec\ub2c8\uc544\uc758 \ubd81\ucabd\uc5d0 \uc808\ubc18\uc774 \uc0b4\uc558\ub2e4. 1845\ub144\uc5d0 \ubbf8\uad6d\uc774 \uba55\uc2dc\ucf54\ub85c\ubd80\ud130 \uce98\ub9ac\ud3ec\ub2c8\uc544\uc758 \uc9c0\ubc30\uad8c\uc744 \ube7c\uc557\uace0, 1847\ub144 1\uc6d4 14\uc77c\uc5d0 \u3008\uce74\ud6c4\uc5d4\uac00 \uc870\uc57d\u3009\uc774 \uccb4\uacb0\ub418\uc5c8\uc744 \ub54c, \uce98\ub9ac\ud3ec\ub2c8\uc544\uc758 \uc18c\uaddc\ubaa8 \ubb34\uc7a5 \uc800\ud56d\ub3c4 \ub05d\uc774 \ub0ac\ub2e4. \ubbf8\uad6d \uba55\uc2dc\ucf54 \uc804\uc7c1\uc744 \ub05d\ub0b8 1848\ub144 2\uc6d4 \u3008\uacfc\ub2ec\ub8e8\ud398 \uc774\ub2ec\uace0 \uc870\uc57d\u3009\uc5d0 \uc758\ud574 \uce98\ub9ac\ud3ec\ub2c8\uc544\ub294 \uba55\uc2dc\ucf54\uc5d0\uc11c (\uc720\uc0c1\uc73c\ub85c) \ubbf8\uad6d \uc601\ud1a0\ub85c \ubc14\ub00c\uc5c8\ub2e4. 1848\ub144 1\uc6d4\uc5d0 \uae08\uc774 \ubc1c\uacac\ub41c \ud6c4 \uce98\ub9ac\ud3ec\ub2c8\uc544 \uace8\ub4dc\ub7ec\uc2dc\uc5d0 \uad00\ud55c \uc18c\ubb38\uc774 \ube60\ub974\uac8c \ud37c\uc838 \ub098\uac14\ub2e4. 1848\ub144 \ud6c4\ubc18\uc5d0\ub294 25\ub9cc\uba85\uc774 \ub118\ub294 \uc0ac\uc5c5\uac00, \ub18d\ubd80, \uac1c\ucc99\uc790 \ubc0f \uad11\ubd80\ub4e4\uc774 \uce98\ub9ac\ud3ec\ub2c8\uc544 \uac00\ub3c4\ub97c \ub118\uc5b4 \uce98\ub9ac\ud3ec\ub2c8\uc544\uc5d0 \ub3c4\ucc29\ud588\ub2e4. \ud1b5\ud589\uc790\ub4e4\uc774 \ub9e4\uc6b0 \ub9ce\uc558\uae30 \ub54c\ubb38\uc5d0, 2\ub144\ub3c4 \ubabb\ub418\uc5b4 \ud30c\ub098\ub9c8 \uc9c0\ud611\uc744 \uac70\uccd0 \ub610\ub294 \ucf00\uc774\ud504 \ud63c\uc744 \ub3cc\uc544 \ubc14\ub2f7\uae38 \ubc29\ubb38\uc790\ub4e4\uae4c\uc9c0 \ud569\uccd0 \uc5c4\uccad\ub09c \uc778\uad6c\uac00 \ubd88\uc5b4\ub098\uc790, 1850\ub144 \uce98\ub9ac\ud3ec\ub2c8\uc544\ub294 \ubbf8\uad6d\uc758 31\ubc88\uc9f8 \uc8fc\ub85c \uc2b9\uaca9\ud558\uc600\ub2e4."} {"question": "\uc0ac\uc6b0\uc2a4\ub9bc\uc758 \uadf8\ub79c\ub4dc \uce90\ub2c8\uc5b8 \ube4c\ub9ac\uc9c0\ub85c \ud5a5\ud558\ub294 65\ub9c8\uc77c \uc6b4\ud589\uac70\ub9ac\uc758 \uc655\ubcf5\uc5f4\ucc28\uac00 \ucd9c\ubc1c\ud558\ub294 \uc5ed\uc740?", "paragraph": "\uadf8\ub79c\ub4dc \uce90\ub2c8\uc5b8 \uad6d\ub9bd\uacf5\uc6d0\uc73c\ub85c \uac00\ub294 \uae38\uc740 \uc0ac\uc6b0\uc2a4\ub9bc(South Rim)\uc758 \uacbd\uc6b0\ub294 I-40\uace0\uc18d \ub3c4\ub85c\ub85c \uc70c\ub9ac\uc5c4\uc2a4(Williams)\uae4c\uc9c0 \uc640\uc11c 64\ubc88 \ub3c4\ub85c\ub85c 100km \uac70\ub9ac\ub97c \uc62c\ub77c\uac00\ub4e0\uc9c0 \ud50c\ub798\uadf8\uc2a4\ud0dc\ud504(Flagstaff)\uc5d0 \uc640\uc11c 180\ubc88 \ub3c4\ub85c\ub85c 130km \uac70\ub9ac\ub97c \uc62c\ub77c\uac00\uc11c \ud22c\uc0ac\uc580(Tusayan) \ub9c8\uc744\uc744 \uac70\uccd0 \ub0a8\ucabd \uc785\uad6c\ub85c \uacf5\uc6d0\uc5d0 \ub4e4\uc5b4\uac00\ub294\uac8c \ub300\ubd80\ubd84\uc758 \uc5ec\ud589\uac1d\uc774 \ud0dd\ud558\ub294 \uae38\uc774\ub2e4. \ud50c\ub798\uadf8\uc2a4\ud0dc\ud504\uc5d0\uc11c 89\ubc88\uacfc 64\ubc88\uc744 \ud1b5\ud574\uc11c \ub3d9\ucabd \uc785\uad6c\ub85c \ub4e4\uc5b4\uac00\ub294 \ubc29\ubc95\ub3c4 \uc788\ub2e4. \ud53c\ub2c9\uc2a4(Phoenix)\uc640 \ud50c\ub798\uadf8\uc2a4\ud0dc\ud504 \uc0ac\uc774 \uac70\ub9ac\ub294 136\ub9c8\uc77c(217km), \ub77c\uc2a4\ubca0\uc774\uac70\uc2a4(Las Vegas)\uc640 \uc70c\ub9ac\uc5c4\uc2a4 \uc0ac\uc774 \uac70\ub9ac\ub294 221\ub9c8\uc77c(353km)\uc774\ub2e4. \ub178\uc2a4\ub9bc\uc73c\ub85c \uac00\ub294 \uae38\uc740 \uc560\ub9ac\uc870\ub098 \ubd81\ubd80\uc5d0 \uc788\ub294 \ud398\uc774\uc9c0(Page)\ub97c \uac70\uce58\ub4e0\uac00 \uc720\ud0c0\uc8fc\uc5d0 \uc788\ub294 \uce74\ub098\ube0c(Kanab)\ub97c \uac70\uccd0 \uc81c\uc774\ucf54\ube0c\ub808\uc774\ud06c(Jacob Lake)\ub97c \uacbd\uc720\ud574\uc11c \ub4e4\uc5b4\uac04\ub2e4. \uc70c\ub9ac\uc5c4\uc2a4\uc640 \uc0ac\uc6b0\uc2a4\ub9bc\uc5d0 \uc788\ub294 \uadf8\ub79c\ub4dc \uce90\ub2c8\uc5b8 \ube4c\ub9ac\uc9c0 \uc0ac\uc774 65\ub9c8\uc77c \uac70\ub9ac\ub97c \ud558\ub8e8\uc5d0 \ud55c\ubc88\uc529 \uc655\ubcf5\ud558\ub294 \uae30\ucc28\ub97c \uc774\uc6a9\ud558\ub294 \uc5ec\ud589\uac1d\ub3c4 \ub9ce\ub2e4. \uc61b\ub0a0\uc758 \ubd84\uc704\uae30\ub97c \ub290\uaef4 \ubcfc\uc218 \uc788\ub294 \uc5ec\ud589 \ubc29\ubc95\uc774\ub2e4. \uacf5\uc6d0 \uc548\uc5d0 \uc788\ub294 \ud638\ud154(Railway Hotel)\uacfc \uae30\ucc28\ud45c\ub97c \ud328\ud0a4\uc9c0\ub85c \uc608\uc57d\ud560 \uc218\ub3c4 \uc788\ub2e4.", "answer": "\uc70c\ub9ac\uc5c4\uc2a4", "sentence": "\uadf8\ub79c\ub4dc \uce90\ub2c8\uc5b8 \uad6d\ub9bd\uacf5\uc6d0\uc73c\ub85c \uac00\ub294 \uae38\uc740 \uc0ac\uc6b0\uc2a4\ub9bc(South Rim)\uc758 \uacbd\uc6b0\ub294 I-40\uace0\uc18d \ub3c4\ub85c\ub85c \uc70c\ub9ac\uc5c4\uc2a4 (Williams)", "paragraph_sentence": " \uadf8\ub79c\ub4dc \uce90\ub2c8\uc5b8 \uad6d\ub9bd\uacf5\uc6d0\uc73c\ub85c \uac00\ub294 \uae38\uc740 \uc0ac\uc6b0\uc2a4\ub9bc(South Rim)\uc758 \uacbd\uc6b0\ub294 I-40\uace0\uc18d \ub3c4\ub85c\ub85c \uc70c\ub9ac\uc5c4\uc2a4 (Williams) \uae4c\uc9c0 \uc640\uc11c 64\ubc88 \ub3c4\ub85c\ub85c 100km \uac70\ub9ac\ub97c \uc62c\ub77c\uac00\ub4e0\uc9c0 \ud50c\ub798\uadf8\uc2a4\ud0dc\ud504(Flagstaff)\uc5d0 \uc640\uc11c 180\ubc88 \ub3c4\ub85c\ub85c 130km \uac70\ub9ac\ub97c \uc62c\ub77c\uac00\uc11c \ud22c\uc0ac\uc580(Tusayan) \ub9c8\uc744\uc744 \uac70\uccd0 \ub0a8\ucabd \uc785\uad6c\ub85c \uacf5\uc6d0\uc5d0 \ub4e4\uc5b4\uac00\ub294\uac8c \ub300\ubd80\ubd84\uc758 \uc5ec\ud589\uac1d\uc774 \ud0dd\ud558\ub294 \uae38\uc774\ub2e4. \ud50c\ub798\uadf8\uc2a4\ud0dc\ud504\uc5d0\uc11c 89\ubc88\uacfc 64\ubc88\uc744 \ud1b5\ud574\uc11c \ub3d9\ucabd \uc785\uad6c\ub85c \ub4e4\uc5b4\uac00\ub294 \ubc29\ubc95\ub3c4 \uc788\ub2e4. \ud53c\ub2c9\uc2a4(Phoenix)\uc640 \ud50c\ub798\uadf8\uc2a4\ud0dc\ud504 \uc0ac\uc774 \uac70\ub9ac\ub294 136\ub9c8\uc77c(217km), \ub77c\uc2a4\ubca0\uc774\uac70\uc2a4(Las Vegas)\uc640 \uc70c\ub9ac\uc5c4\uc2a4 \uc0ac\uc774 \uac70\ub9ac\ub294 221\ub9c8\uc77c(353km)\uc774\ub2e4. \ub178\uc2a4\ub9bc\uc73c\ub85c \uac00\ub294 \uae38\uc740 \uc560\ub9ac\uc870\ub098 \ubd81\ubd80\uc5d0 \uc788\ub294 \ud398\uc774\uc9c0(Page)\ub97c \uac70\uce58\ub4e0\uac00 \uc720\ud0c0\uc8fc\uc5d0 \uc788\ub294 \uce74\ub098\ube0c(Kanab)\ub97c \uac70\uccd0 \uc81c\uc774\ucf54\ube0c\ub808\uc774\ud06c(Jacob Lake)\ub97c \uacbd\uc720\ud574\uc11c \ub4e4\uc5b4\uac04\ub2e4. \uc70c\ub9ac\uc5c4\uc2a4\uc640 \uc0ac\uc6b0\uc2a4\ub9bc\uc5d0 \uc788\ub294 \uadf8\ub79c\ub4dc \uce90\ub2c8\uc5b8 \ube4c\ub9ac\uc9c0 \uc0ac\uc774 65\ub9c8\uc77c \uac70\ub9ac\ub97c \ud558\ub8e8\uc5d0 \ud55c\ubc88\uc529 \uc655\ubcf5\ud558\ub294 \uae30\ucc28\ub97c \uc774\uc6a9\ud558\ub294 \uc5ec\ud589\uac1d\ub3c4 \ub9ce\ub2e4. \uc61b\ub0a0\uc758 \ubd84\uc704\uae30\ub97c \ub290\uaef4 \ubcfc\uc218 \uc788\ub294 \uc5ec\ud589 \ubc29\ubc95\uc774\ub2e4. \uacf5\uc6d0 \uc548\uc5d0 \uc788\ub294 \ud638\ud154(Railway Hotel)\uacfc \uae30\ucc28\ud45c\ub97c \ud328\ud0a4\uc9c0\ub85c \uc608\uc57d\ud560 \uc218\ub3c4 \uc788\ub2e4.", "paragraph_answer": "\uadf8\ub79c\ub4dc \uce90\ub2c8\uc5b8 \uad6d\ub9bd\uacf5\uc6d0\uc73c\ub85c \uac00\ub294 \uae38\uc740 \uc0ac\uc6b0\uc2a4\ub9bc(South Rim)\uc758 \uacbd\uc6b0\ub294 I-40\uace0\uc18d \ub3c4\ub85c\ub85c \uc70c\ub9ac\uc5c4\uc2a4 (Williams)\uae4c\uc9c0 \uc640\uc11c 64\ubc88 \ub3c4\ub85c\ub85c 100km \uac70\ub9ac\ub97c \uc62c\ub77c\uac00\ub4e0\uc9c0 \ud50c\ub798\uadf8\uc2a4\ud0dc\ud504(Flagstaff)\uc5d0 \uc640\uc11c 180\ubc88 \ub3c4\ub85c\ub85c 130km \uac70\ub9ac\ub97c \uc62c\ub77c\uac00\uc11c \ud22c\uc0ac\uc580(Tusayan) \ub9c8\uc744\uc744 \uac70\uccd0 \ub0a8\ucabd \uc785\uad6c\ub85c \uacf5\uc6d0\uc5d0 \ub4e4\uc5b4\uac00\ub294\uac8c \ub300\ubd80\ubd84\uc758 \uc5ec\ud589\uac1d\uc774 \ud0dd\ud558\ub294 \uae38\uc774\ub2e4. \ud50c\ub798\uadf8\uc2a4\ud0dc\ud504\uc5d0\uc11c 89\ubc88\uacfc 64\ubc88\uc744 \ud1b5\ud574\uc11c \ub3d9\ucabd \uc785\uad6c\ub85c \ub4e4\uc5b4\uac00\ub294 \ubc29\ubc95\ub3c4 \uc788\ub2e4. \ud53c\ub2c9\uc2a4(Phoenix)\uc640 \ud50c\ub798\uadf8\uc2a4\ud0dc\ud504 \uc0ac\uc774 \uac70\ub9ac\ub294 136\ub9c8\uc77c(217km), \ub77c\uc2a4\ubca0\uc774\uac70\uc2a4(Las Vegas)\uc640 \uc70c\ub9ac\uc5c4\uc2a4 \uc0ac\uc774 \uac70\ub9ac\ub294 221\ub9c8\uc77c(353km)\uc774\ub2e4. \ub178\uc2a4\ub9bc\uc73c\ub85c \uac00\ub294 \uae38\uc740 \uc560\ub9ac\uc870\ub098 \ubd81\ubd80\uc5d0 \uc788\ub294 \ud398\uc774\uc9c0(Page)\ub97c \uac70\uce58\ub4e0\uac00 \uc720\ud0c0\uc8fc\uc5d0 \uc788\ub294 \uce74\ub098\ube0c(Kanab)\ub97c \uac70\uccd0 \uc81c\uc774\ucf54\ube0c\ub808\uc774\ud06c(Jacob Lake)\ub97c \uacbd\uc720\ud574\uc11c \ub4e4\uc5b4\uac04\ub2e4. \uc70c\ub9ac\uc5c4\uc2a4\uc640 \uc0ac\uc6b0\uc2a4\ub9bc\uc5d0 \uc788\ub294 \uadf8\ub79c\ub4dc \uce90\ub2c8\uc5b8 \ube4c\ub9ac\uc9c0 \uc0ac\uc774 65\ub9c8\uc77c \uac70\ub9ac\ub97c \ud558\ub8e8\uc5d0 \ud55c\ubc88\uc529 \uc655\ubcf5\ud558\ub294 \uae30\ucc28\ub97c \uc774\uc6a9\ud558\ub294 \uc5ec\ud589\uac1d\ub3c4 \ub9ce\ub2e4. \uc61b\ub0a0\uc758 \ubd84\uc704\uae30\ub97c \ub290\uaef4 \ubcfc\uc218 \uc788\ub294 \uc5ec\ud589 \ubc29\ubc95\uc774\ub2e4. \uacf5\uc6d0 \uc548\uc5d0 \uc788\ub294 \ud638\ud154(Railway Hotel)\uacfc \uae30\ucc28\ud45c\ub97c \ud328\ud0a4\uc9c0\ub85c \uc608\uc57d\ud560 \uc218\ub3c4 \uc788\ub2e4.", "sentence_answer": "\uadf8\ub79c\ub4dc \uce90\ub2c8\uc5b8 \uad6d\ub9bd\uacf5\uc6d0\uc73c\ub85c \uac00\ub294 \uae38\uc740 \uc0ac\uc6b0\uc2a4\ub9bc(South Rim)\uc758 \uacbd\uc6b0\ub294 I-40\uace0\uc18d \ub3c4\ub85c\ub85c \uc70c\ub9ac\uc5c4\uc2a4 (Williams)", "paragraph_id": "6490009-2-0", "paragraph_question": "question: \uc0ac\uc6b0\uc2a4\ub9bc\uc758 \uadf8\ub79c\ub4dc \uce90\ub2c8\uc5b8 \ube4c\ub9ac\uc9c0\ub85c \ud5a5\ud558\ub294 65\ub9c8\uc77c \uc6b4\ud589\uac70\ub9ac\uc758 \uc655\ubcf5\uc5f4\ucc28\uac00 \ucd9c\ubc1c\ud558\ub294 \uc5ed\uc740?, context: \uadf8\ub79c\ub4dc \uce90\ub2c8\uc5b8 \uad6d\ub9bd\uacf5\uc6d0\uc73c\ub85c \uac00\ub294 \uae38\uc740 \uc0ac\uc6b0\uc2a4\ub9bc(South Rim)\uc758 \uacbd\uc6b0\ub294 I-40\uace0\uc18d \ub3c4\ub85c\ub85c \uc70c\ub9ac\uc5c4\uc2a4(Williams)\uae4c\uc9c0 \uc640\uc11c 64\ubc88 \ub3c4\ub85c\ub85c 100km \uac70\ub9ac\ub97c \uc62c\ub77c\uac00\ub4e0\uc9c0 \ud50c\ub798\uadf8\uc2a4\ud0dc\ud504(Flagstaff)\uc5d0 \uc640\uc11c 180\ubc88 \ub3c4\ub85c\ub85c 130km \uac70\ub9ac\ub97c \uc62c\ub77c\uac00\uc11c \ud22c\uc0ac\uc580(Tusayan) \ub9c8\uc744\uc744 \uac70\uccd0 \ub0a8\ucabd \uc785\uad6c\ub85c \uacf5\uc6d0\uc5d0 \ub4e4\uc5b4\uac00\ub294\uac8c \ub300\ubd80\ubd84\uc758 \uc5ec\ud589\uac1d\uc774 \ud0dd\ud558\ub294 \uae38\uc774\ub2e4. \ud50c\ub798\uadf8\uc2a4\ud0dc\ud504\uc5d0\uc11c 89\ubc88\uacfc 64\ubc88\uc744 \ud1b5\ud574\uc11c \ub3d9\ucabd \uc785\uad6c\ub85c \ub4e4\uc5b4\uac00\ub294 \ubc29\ubc95\ub3c4 \uc788\ub2e4. \ud53c\ub2c9\uc2a4(Phoenix)\uc640 \ud50c\ub798\uadf8\uc2a4\ud0dc\ud504 \uc0ac\uc774 \uac70\ub9ac\ub294 136\ub9c8\uc77c(217km), \ub77c\uc2a4\ubca0\uc774\uac70\uc2a4(Las Vegas)\uc640 \uc70c\ub9ac\uc5c4\uc2a4 \uc0ac\uc774 \uac70\ub9ac\ub294 221\ub9c8\uc77c(353km)\uc774\ub2e4. \ub178\uc2a4\ub9bc\uc73c\ub85c \uac00\ub294 \uae38\uc740 \uc560\ub9ac\uc870\ub098 \ubd81\ubd80\uc5d0 \uc788\ub294 \ud398\uc774\uc9c0(Page)\ub97c \uac70\uce58\ub4e0\uac00 \uc720\ud0c0\uc8fc\uc5d0 \uc788\ub294 \uce74\ub098\ube0c(Kanab)\ub97c \uac70\uccd0 \uc81c\uc774\ucf54\ube0c\ub808\uc774\ud06c(Jacob Lake)\ub97c \uacbd\uc720\ud574\uc11c \ub4e4\uc5b4\uac04\ub2e4. \uc70c\ub9ac\uc5c4\uc2a4\uc640 \uc0ac\uc6b0\uc2a4\ub9bc\uc5d0 \uc788\ub294 \uadf8\ub79c\ub4dc \uce90\ub2c8\uc5b8 \ube4c\ub9ac\uc9c0 \uc0ac\uc774 65\ub9c8\uc77c \uac70\ub9ac\ub97c \ud558\ub8e8\uc5d0 \ud55c\ubc88\uc529 \uc655\ubcf5\ud558\ub294 \uae30\ucc28\ub97c \uc774\uc6a9\ud558\ub294 \uc5ec\ud589\uac1d\ub3c4 \ub9ce\ub2e4. \uc61b\ub0a0\uc758 \ubd84\uc704\uae30\ub97c \ub290\uaef4 \ubcfc\uc218 \uc788\ub294 \uc5ec\ud589 \ubc29\ubc95\uc774\ub2e4. \uacf5\uc6d0 \uc548\uc5d0 \uc788\ub294 \ud638\ud154(Railway Hotel)\uacfc \uae30\ucc28\ud45c\ub97c \ud328\ud0a4\uc9c0\ub85c \uc608\uc57d\ud560 \uc218\ub3c4 \uc788\ub2e4."} {"question": "\uc6d0\ud130\ub864\ub9ac\ub294 \ub204\uad6c\uc5d0\uac8c \ud3ec\ud0c4\uc758 \uac1c\uc218\ub97c \uc54c\uc544\ub0b4\ub77c\uace0 \ud558\uc600\ub294\uac00?", "paragraph": "1590\ub144\ub300 \ub9d0\uacbd\uc5d0 \uc601\uad6d\uc758 \uc6d4\ud130 \ub864\ub9ac \uacbd\uc740 \ubd80\ud558 \ud1a0\uba38\uc2a4 \ud574\ub9ac\uc5c7\uc5d0\uac8c \ud3ec\ud0c4 \ubb34\ub354\uae30\uac00 \uc313\uc778 \ubaa8\uc591\uc744 \ubcf4\uace0 \ud3ec\ud0c4\uc758 \uac1c\uc218\ub97c \uc54c\uc544\ub0bc \uc218 \uc788\ub294 \uacf5\uc2dd\uc744 \ub9cc\ub4e4\ub77c\uace0 \uc694\uad6c\ud588\ub2e4. \ud574\ub9ac\uc5c7\uc740 \uac70\uae30\uc11c \ub354 \ub098\uc544\uac00 \ud3ec\ud0c4\uc744 \ucd5c\ub300\ud55c \ud6a8\uc728\uc801\uc73c\ub85c \uc313\uc744 \uc218 \uc788\ub294 \ubc29\ubc95\uc744 \ucc3e\uc73c\ub824\uace0 \ud588\ub2e4. \ud574\ub9ac\uc5c7\uc740 \uc774 \ubb38\uc81c\ub97c \uac00\uc9c0\uace0 \uace0\uc2ec\ud558\ub2e4\uac00 \ub2f9\uc2dc \ucd5c\uace0\uc758 \uc218\ud559\uc790\ub85c \uba85\uc131\uc744 \ub0a0\ub9ac\ub358 \ucf00\ud50c\ub7ec\uc5d0\uac8c \ud3b8\uc9c0\ub97c \ubcf4\ub0c8\ub2e4. \ud574\ub9ac\uc5c7\uc758 \ud3b8\uc9c0\ub97c \ubc1b\uc740 \ucf00\ud50c\ub7ec\ub294 \uac00\uc7a5 \ubc00\ub3c4\uac00 \ub192\uac8c \uad6c\ub97c \uc313\ub294 \ubc29\ubc95\uc740 \uc2dc\uc7a5 \uc0c1\uc778\ub4e4\uc774 \uacfc\uc77c\uc744 \uc313\uc744 \ub54c\ucc98\ub7fc \uc721\ubc29 \ubc00\uc9d1 \uc313\uae30\ub97c \ud558\ub294 \uac83\uc774\ub77c\uace0 \uacb0\ub860\uc744 \ub0b4\ub838\ub2e4. 1611\ub144 \uc0c8\ud574 \uc120\ubb3c\ub85c \ucf00\ud50c\ub7ec\ub294 \uce5c\uad6c\uc774\uc790 \uc624\ub79c \ud6c4\uc6d0\uc790 \ub9c8\ud0dc\uc6b0\uc2a4 \ubc14\ucee4 \ud3f0 \ubc14\ucf04\ud3a0\uc2a4 \ub0a8\uc791\uc744 \uc704\ud574 \u300a\uc721\uac01\ud615 \ub208\uc1a1\uc774\uc5d0 \uad00\ud558\uc5ec\u300b(Strena Seu de Nive Sexangula)\ub77c\ub294 \uc81c\ubaa9\uc758 \uc9e7\uc740 \uc18c\ub17c\ubb38\uc744 \uc9c0\uc5c8\ub2e4. \uc774 \uc18c\ub17c\ubb38\uc5d0\uc11c \ucf00\ud50c\ub7ec\ub294 \ub208\uc1a1\uc774\uc758 \uc721\ubc29\uc815\uacc4 \uad6c\uc870\ub97c \uc5f0\uad6c\ud558\uace0, \uadf8\uac83\uc744 \uc6d0\uc790\ub860\uc758 \ubb3c\ub9ac\uc801 \uadfc\uac70\ub85c \ud574\uc11d\ud588\uc73c\uba70, \uad6c\ub97c \uac00\uc7a5 \ucd18\ucd18\ud558\uac8c \ucc44\uc6b8 \uc218 \uc788\ub294 \ubc29\ubc95\uc5d0 \ub300\ud574 \uae30\uc220\ud588\ub2e4. \ub9c8\uc9c0\ub9c9 \uac83\uc774 \ud6c4\uc5d0 \ucf00\ud50c\ub7ec\uc758 \ucd94\uce21\uc774\ub77c\uace0 \uc54c\ub824\uc9c4 \uac83\uc73c\ub85c, \ud398\ub974\ub9c8\uc758 \ub9c8\uc9c0\ub9c9 \uc815\ub9ac\uc640 \ud568\uaed8 \uc218\ubc31\ub144 \ub3d9\uc548 \uc218\ud559\uc801 \ub09c\uc81c\ub85c \ub0a8\uc558\ub2e4. \u300a\uc721\uac01\ud615 \ub208\uc1a1\uc774\uc5d0 \uad00\ud558\uc5ec\u300b\ub294 \ucf00\ud50c\ub7ec\uc758 \ub2e4\ub978 \ubc29\ub300\ud55c \uc800\uc791\ub4e4\uc5d0 \ube44\ud574 \uacfc\uc18c\ud3c9\uac00\ub418\ub294 \uacbd\ud5a5\uc774 \uc788\uc9c0\ub9cc, \uc774 \ucc45\uc740 \uacb0\uc815\uccb4\ub098 \uc2dd\ubb3c\uc758 \ud615\ud0dc\ub97c \uacfc\ud559\uc801\uc73c\ub85c \uc124\uba85\ud558\ub824\uace0 \ud55c \ucd5c\ucd08\uc758 \uc2dc\ub3c4\uc600\uae30\uc5d0 \ucda9\ubd84\ud55c \uc911\uc694\uc131\uc744 \uc9c0\ub2c8\uace0 \uc788\ub2e4.", "answer": "\ud1a0\uba38\uc2a4 \ud574\ub9ac\uc5c7", "sentence": "1590\ub144\ub300 \ub9d0\uacbd\uc5d0 \uc601\uad6d\uc758 \uc6d4\ud130 \ub864\ub9ac \uacbd\uc740 \ubd80\ud558 \ud1a0\uba38\uc2a4 \ud574\ub9ac\uc5c7 \uc5d0\uac8c \ud3ec\ud0c4 \ubb34\ub354\uae30\uac00 \uc313\uc778 \ubaa8\uc591\uc744 \ubcf4\uace0 \ud3ec\ud0c4\uc758 \uac1c\uc218\ub97c \uc54c\uc544\ub0bc \uc218 \uc788\ub294 \uacf5\uc2dd\uc744 \ub9cc\ub4e4\ub77c\uace0 \uc694\uad6c\ud588\ub2e4.", "paragraph_sentence": " 1590\ub144\ub300 \ub9d0\uacbd\uc5d0 \uc601\uad6d\uc758 \uc6d4\ud130 \ub864\ub9ac \uacbd\uc740 \ubd80\ud558 \ud1a0\uba38\uc2a4 \ud574\ub9ac\uc5c7 \uc5d0\uac8c \ud3ec\ud0c4 \ubb34\ub354\uae30\uac00 \uc313\uc778 \ubaa8\uc591\uc744 \ubcf4\uace0 \ud3ec\ud0c4\uc758 \uac1c\uc218\ub97c \uc54c\uc544\ub0bc \uc218 \uc788\ub294 \uacf5\uc2dd\uc744 \ub9cc\ub4e4\ub77c\uace0 \uc694\uad6c\ud588\ub2e4. \ud574\ub9ac\uc5c7\uc740 \uac70\uae30\uc11c \ub354 \ub098\uc544\uac00 \ud3ec\ud0c4\uc744 \ucd5c\ub300\ud55c \ud6a8\uc728\uc801\uc73c\ub85c \uc313\uc744 \uc218 \uc788\ub294 \ubc29\ubc95\uc744 \ucc3e\uc73c\ub824\uace0 \ud588\ub2e4. \ud574\ub9ac\uc5c7\uc740 \uc774 \ubb38\uc81c\ub97c \uac00\uc9c0\uace0 \uace0\uc2ec\ud558\ub2e4\uac00 \ub2f9\uc2dc \ucd5c\uace0\uc758 \uc218\ud559\uc790\ub85c \uba85\uc131\uc744 \ub0a0\ub9ac\ub358 \ucf00\ud50c\ub7ec\uc5d0\uac8c \ud3b8\uc9c0\ub97c \ubcf4\ub0c8\ub2e4. \ud574\ub9ac\uc5c7\uc758 \ud3b8\uc9c0\ub97c \ubc1b\uc740 \ucf00\ud50c\ub7ec\ub294 \uac00\uc7a5 \ubc00\ub3c4\uac00 \ub192\uac8c \uad6c\ub97c \uc313\ub294 \ubc29\ubc95\uc740 \uc2dc\uc7a5 \uc0c1\uc778\ub4e4\uc774 \uacfc\uc77c\uc744 \uc313\uc744 \ub54c\ucc98\ub7fc \uc721\ubc29 \ubc00\uc9d1 \uc313\uae30\ub97c \ud558\ub294 \uac83\uc774\ub77c\uace0 \uacb0\ub860\uc744 \ub0b4\ub838\ub2e4. 1611\ub144 \uc0c8\ud574 \uc120\ubb3c\ub85c \ucf00\ud50c\ub7ec\ub294 \uce5c\uad6c\uc774\uc790 \uc624\ub79c \ud6c4\uc6d0\uc790 \ub9c8\ud0dc\uc6b0\uc2a4 \ubc14\ucee4 \ud3f0 \ubc14\ucf04\ud3a0\uc2a4 \ub0a8\uc791\uc744 \uc704\ud574 \u300a\uc721\uac01\ud615 \ub208\uc1a1\uc774\uc5d0 \uad00\ud558\uc5ec\u300b(Strena Seu de Nive Sexangula)\ub77c\ub294 \uc81c\ubaa9\uc758 \uc9e7\uc740 \uc18c\ub17c\ubb38\uc744 \uc9c0\uc5c8\ub2e4. \uc774 \uc18c\ub17c\ubb38\uc5d0\uc11c \ucf00\ud50c\ub7ec\ub294 \ub208\uc1a1\uc774\uc758 \uc721\ubc29\uc815\uacc4 \uad6c\uc870\ub97c \uc5f0\uad6c\ud558\uace0, \uadf8\uac83\uc744 \uc6d0\uc790\ub860\uc758 \ubb3c\ub9ac\uc801 \uadfc\uac70\ub85c \ud574\uc11d\ud588\uc73c\uba70, \uad6c\ub97c \uac00\uc7a5 \ucd18\ucd18\ud558\uac8c \ucc44\uc6b8 \uc218 \uc788\ub294 \ubc29\ubc95\uc5d0 \ub300\ud574 \uae30\uc220\ud588\ub2e4. \ub9c8\uc9c0\ub9c9 \uac83\uc774 \ud6c4\uc5d0 \ucf00\ud50c\ub7ec\uc758 \ucd94\uce21\uc774\ub77c\uace0 \uc54c\ub824\uc9c4 \uac83\uc73c\ub85c, \ud398\ub974\ub9c8\uc758 \ub9c8\uc9c0\ub9c9 \uc815\ub9ac\uc640 \ud568\uaed8 \uc218\ubc31\ub144 \ub3d9\uc548 \uc218\ud559\uc801 \ub09c\uc81c\ub85c \ub0a8\uc558\ub2e4. \u300a\uc721\uac01\ud615 \ub208\uc1a1\uc774\uc5d0 \uad00\ud558\uc5ec\u300b\ub294 \ucf00\ud50c\ub7ec\uc758 \ub2e4\ub978 \ubc29\ub300\ud55c \uc800\uc791\ub4e4\uc5d0 \ube44\ud574 \uacfc\uc18c\ud3c9\uac00\ub418\ub294 \uacbd\ud5a5\uc774 \uc788\uc9c0\ub9cc, \uc774 \ucc45\uc740 \uacb0\uc815\uccb4\ub098 \uc2dd\ubb3c\uc758 \ud615\ud0dc\ub97c \uacfc\ud559\uc801\uc73c\ub85c \uc124\uba85\ud558\ub824\uace0 \ud55c \ucd5c\ucd08\uc758 \uc2dc\ub3c4\uc600\uae30\uc5d0 \ucda9\ubd84\ud55c \uc911\uc694\uc131\uc744 \uc9c0\ub2c8\uace0 \uc788\ub2e4.", "paragraph_answer": "1590\ub144\ub300 \ub9d0\uacbd\uc5d0 \uc601\uad6d\uc758 \uc6d4\ud130 \ub864\ub9ac \uacbd\uc740 \ubd80\ud558 \ud1a0\uba38\uc2a4 \ud574\ub9ac\uc5c7 \uc5d0\uac8c \ud3ec\ud0c4 \ubb34\ub354\uae30\uac00 \uc313\uc778 \ubaa8\uc591\uc744 \ubcf4\uace0 \ud3ec\ud0c4\uc758 \uac1c\uc218\ub97c \uc54c\uc544\ub0bc \uc218 \uc788\ub294 \uacf5\uc2dd\uc744 \ub9cc\ub4e4\ub77c\uace0 \uc694\uad6c\ud588\ub2e4. \ud574\ub9ac\uc5c7\uc740 \uac70\uae30\uc11c \ub354 \ub098\uc544\uac00 \ud3ec\ud0c4\uc744 \ucd5c\ub300\ud55c \ud6a8\uc728\uc801\uc73c\ub85c \uc313\uc744 \uc218 \uc788\ub294 \ubc29\ubc95\uc744 \ucc3e\uc73c\ub824\uace0 \ud588\ub2e4. \ud574\ub9ac\uc5c7\uc740 \uc774 \ubb38\uc81c\ub97c \uac00\uc9c0\uace0 \uace0\uc2ec\ud558\ub2e4\uac00 \ub2f9\uc2dc \ucd5c\uace0\uc758 \uc218\ud559\uc790\ub85c \uba85\uc131\uc744 \ub0a0\ub9ac\ub358 \ucf00\ud50c\ub7ec\uc5d0\uac8c \ud3b8\uc9c0\ub97c \ubcf4\ub0c8\ub2e4. \ud574\ub9ac\uc5c7\uc758 \ud3b8\uc9c0\ub97c \ubc1b\uc740 \ucf00\ud50c\ub7ec\ub294 \uac00\uc7a5 \ubc00\ub3c4\uac00 \ub192\uac8c \uad6c\ub97c \uc313\ub294 \ubc29\ubc95\uc740 \uc2dc\uc7a5 \uc0c1\uc778\ub4e4\uc774 \uacfc\uc77c\uc744 \uc313\uc744 \ub54c\ucc98\ub7fc \uc721\ubc29 \ubc00\uc9d1 \uc313\uae30\ub97c \ud558\ub294 \uac83\uc774\ub77c\uace0 \uacb0\ub860\uc744 \ub0b4\ub838\ub2e4. 1611\ub144 \uc0c8\ud574 \uc120\ubb3c\ub85c \ucf00\ud50c\ub7ec\ub294 \uce5c\uad6c\uc774\uc790 \uc624\ub79c \ud6c4\uc6d0\uc790 \ub9c8\ud0dc\uc6b0\uc2a4 \ubc14\ucee4 \ud3f0 \ubc14\ucf04\ud3a0\uc2a4 \ub0a8\uc791\uc744 \uc704\ud574 \u300a\uc721\uac01\ud615 \ub208\uc1a1\uc774\uc5d0 \uad00\ud558\uc5ec\u300b(Strena Seu de Nive Sexangula)\ub77c\ub294 \uc81c\ubaa9\uc758 \uc9e7\uc740 \uc18c\ub17c\ubb38\uc744 \uc9c0\uc5c8\ub2e4. \uc774 \uc18c\ub17c\ubb38\uc5d0\uc11c \ucf00\ud50c\ub7ec\ub294 \ub208\uc1a1\uc774\uc758 \uc721\ubc29\uc815\uacc4 \uad6c\uc870\ub97c \uc5f0\uad6c\ud558\uace0, \uadf8\uac83\uc744 \uc6d0\uc790\ub860\uc758 \ubb3c\ub9ac\uc801 \uadfc\uac70\ub85c \ud574\uc11d\ud588\uc73c\uba70, \uad6c\ub97c \uac00\uc7a5 \ucd18\ucd18\ud558\uac8c \ucc44\uc6b8 \uc218 \uc788\ub294 \ubc29\ubc95\uc5d0 \ub300\ud574 \uae30\uc220\ud588\ub2e4. \ub9c8\uc9c0\ub9c9 \uac83\uc774 \ud6c4\uc5d0 \ucf00\ud50c\ub7ec\uc758 \ucd94\uce21\uc774\ub77c\uace0 \uc54c\ub824\uc9c4 \uac83\uc73c\ub85c, \ud398\ub974\ub9c8\uc758 \ub9c8\uc9c0\ub9c9 \uc815\ub9ac\uc640 \ud568\uaed8 \uc218\ubc31\ub144 \ub3d9\uc548 \uc218\ud559\uc801 \ub09c\uc81c\ub85c \ub0a8\uc558\ub2e4. \u300a\uc721\uac01\ud615 \ub208\uc1a1\uc774\uc5d0 \uad00\ud558\uc5ec\u300b\ub294 \ucf00\ud50c\ub7ec\uc758 \ub2e4\ub978 \ubc29\ub300\ud55c \uc800\uc791\ub4e4\uc5d0 \ube44\ud574 \uacfc\uc18c\ud3c9\uac00\ub418\ub294 \uacbd\ud5a5\uc774 \uc788\uc9c0\ub9cc, \uc774 \ucc45\uc740 \uacb0\uc815\uccb4\ub098 \uc2dd\ubb3c\uc758 \ud615\ud0dc\ub97c \uacfc\ud559\uc801\uc73c\ub85c \uc124\uba85\ud558\ub824\uace0 \ud55c \ucd5c\ucd08\uc758 \uc2dc\ub3c4\uc600\uae30\uc5d0 \ucda9\ubd84\ud55c \uc911\uc694\uc131\uc744 \uc9c0\ub2c8\uace0 \uc788\ub2e4.", "sentence_answer": "1590\ub144\ub300 \ub9d0\uacbd\uc5d0 \uc601\uad6d\uc758 \uc6d4\ud130 \ub864\ub9ac \uacbd\uc740 \ubd80\ud558 \ud1a0\uba38\uc2a4 \ud574\ub9ac\uc5c7 \uc5d0\uac8c \ud3ec\ud0c4 \ubb34\ub354\uae30\uac00 \uc313\uc778 \ubaa8\uc591\uc744 \ubcf4\uace0 \ud3ec\ud0c4\uc758 \uac1c\uc218\ub97c \uc54c\uc544\ub0bc \uc218 \uc788\ub294 \uacf5\uc2dd\uc744 \ub9cc\ub4e4\ub77c\uace0 \uc694\uad6c\ud588\ub2e4.", "paragraph_id": "6384301-22-0", "paragraph_question": "question: \uc6d0\ud130\ub864\ub9ac\ub294 \ub204\uad6c\uc5d0\uac8c \ud3ec\ud0c4\uc758 \uac1c\uc218\ub97c \uc54c\uc544\ub0b4\ub77c\uace0 \ud558\uc600\ub294\uac00?, context: 1590\ub144\ub300 \ub9d0\uacbd\uc5d0 \uc601\uad6d\uc758 \uc6d4\ud130 \ub864\ub9ac \uacbd\uc740 \ubd80\ud558 \ud1a0\uba38\uc2a4 \ud574\ub9ac\uc5c7\uc5d0\uac8c \ud3ec\ud0c4 \ubb34\ub354\uae30\uac00 \uc313\uc778 \ubaa8\uc591\uc744 \ubcf4\uace0 \ud3ec\ud0c4\uc758 \uac1c\uc218\ub97c \uc54c\uc544\ub0bc \uc218 \uc788\ub294 \uacf5\uc2dd\uc744 \ub9cc\ub4e4\ub77c\uace0 \uc694\uad6c\ud588\ub2e4. \ud574\ub9ac\uc5c7\uc740 \uac70\uae30\uc11c \ub354 \ub098\uc544\uac00 \ud3ec\ud0c4\uc744 \ucd5c\ub300\ud55c \ud6a8\uc728\uc801\uc73c\ub85c \uc313\uc744 \uc218 \uc788\ub294 \ubc29\ubc95\uc744 \ucc3e\uc73c\ub824\uace0 \ud588\ub2e4. \ud574\ub9ac\uc5c7\uc740 \uc774 \ubb38\uc81c\ub97c \uac00\uc9c0\uace0 \uace0\uc2ec\ud558\ub2e4\uac00 \ub2f9\uc2dc \ucd5c\uace0\uc758 \uc218\ud559\uc790\ub85c \uba85\uc131\uc744 \ub0a0\ub9ac\ub358 \ucf00\ud50c\ub7ec\uc5d0\uac8c \ud3b8\uc9c0\ub97c \ubcf4\ub0c8\ub2e4. \ud574\ub9ac\uc5c7\uc758 \ud3b8\uc9c0\ub97c \ubc1b\uc740 \ucf00\ud50c\ub7ec\ub294 \uac00\uc7a5 \ubc00\ub3c4\uac00 \ub192\uac8c \uad6c\ub97c \uc313\ub294 \ubc29\ubc95\uc740 \uc2dc\uc7a5 \uc0c1\uc778\ub4e4\uc774 \uacfc\uc77c\uc744 \uc313\uc744 \ub54c\ucc98\ub7fc \uc721\ubc29 \ubc00\uc9d1 \uc313\uae30\ub97c \ud558\ub294 \uac83\uc774\ub77c\uace0 \uacb0\ub860\uc744 \ub0b4\ub838\ub2e4. 1611\ub144 \uc0c8\ud574 \uc120\ubb3c\ub85c \ucf00\ud50c\ub7ec\ub294 \uce5c\uad6c\uc774\uc790 \uc624\ub79c \ud6c4\uc6d0\uc790 \ub9c8\ud0dc\uc6b0\uc2a4 \ubc14\ucee4 \ud3f0 \ubc14\ucf04\ud3a0\uc2a4 \ub0a8\uc791\uc744 \uc704\ud574 \u300a\uc721\uac01\ud615 \ub208\uc1a1\uc774\uc5d0 \uad00\ud558\uc5ec\u300b(Strena Seu de Nive Sexangula)\ub77c\ub294 \uc81c\ubaa9\uc758 \uc9e7\uc740 \uc18c\ub17c\ubb38\uc744 \uc9c0\uc5c8\ub2e4. \uc774 \uc18c\ub17c\ubb38\uc5d0\uc11c \ucf00\ud50c\ub7ec\ub294 \ub208\uc1a1\uc774\uc758 \uc721\ubc29\uc815\uacc4 \uad6c\uc870\ub97c \uc5f0\uad6c\ud558\uace0, \uadf8\uac83\uc744 \uc6d0\uc790\ub860\uc758 \ubb3c\ub9ac\uc801 \uadfc\uac70\ub85c \ud574\uc11d\ud588\uc73c\uba70, \uad6c\ub97c \uac00\uc7a5 \ucd18\ucd18\ud558\uac8c \ucc44\uc6b8 \uc218 \uc788\ub294 \ubc29\ubc95\uc5d0 \ub300\ud574 \uae30\uc220\ud588\ub2e4. \ub9c8\uc9c0\ub9c9 \uac83\uc774 \ud6c4\uc5d0 \ucf00\ud50c\ub7ec\uc758 \ucd94\uce21\uc774\ub77c\uace0 \uc54c\ub824\uc9c4 \uac83\uc73c\ub85c, \ud398\ub974\ub9c8\uc758 \ub9c8\uc9c0\ub9c9 \uc815\ub9ac\uc640 \ud568\uaed8 \uc218\ubc31\ub144 \ub3d9\uc548 \uc218\ud559\uc801 \ub09c\uc81c\ub85c \ub0a8\uc558\ub2e4. \u300a\uc721\uac01\ud615 \ub208\uc1a1\uc774\uc5d0 \uad00\ud558\uc5ec\u300b\ub294 \ucf00\ud50c\ub7ec\uc758 \ub2e4\ub978 \ubc29\ub300\ud55c \uc800\uc791\ub4e4\uc5d0 \ube44\ud574 \uacfc\uc18c\ud3c9\uac00\ub418\ub294 \uacbd\ud5a5\uc774 \uc788\uc9c0\ub9cc, \uc774 \ucc45\uc740 \uacb0\uc815\uccb4\ub098 \uc2dd\ubb3c\uc758 \ud615\ud0dc\ub97c \uacfc\ud559\uc801\uc73c\ub85c \uc124\uba85\ud558\ub824\uace0 \ud55c \ucd5c\ucd08\uc758 \uc2dc\ub3c4\uc600\uae30\uc5d0 \ucda9\ubd84\ud55c \uc911\uc694\uc131\uc744 \uc9c0\ub2c8\uace0 \uc788\ub2e4."} {"question": "\ubcf5\ub155\ub2f9 \uc544\uae30\uc528\ub77c\uace0 \ubd88\ub838\ub358 \uc870\uc120\uc758 \uace0\uc885\ud669\uc81c\uc758 \uace0\uba85\ub538\uc740 \ub204\uad6c\uc778\uac00?", "paragraph": "\ub355\ud61c\uc639\uc8fc(\u5fb7\u60e0\u7fc1\u4e3b, 1912\ub144 5\uc6d4 25\uc77c ~ 1989\ub144 4\uc6d4 21\uc77c)\ub294 \uc870\uc120\uc758 \uc81c26\ub300 \uc655\uc774\uc790 \ub300\ud55c\uc81c\uad6d\uc758 \ucd08\ub300 \ud669\uc81c\uc600\ub358 \uace0\uc885\uacfc \uadc0\uc778 \uc591\uc528 \uc0ac\uc774\uc5d0\uc11c \ud0dc\uc5b4\ub09c \uace0\uba85\ub538\uc774\ub2e4. \ud669\ub140\ub85c\uc11c \ub355\ud61c\ub77c\ub294 \ud638\ub97c \ud558\uc0ac\ubc1b\uae30 \uc804\uae4c\uc9c0 \u2018\ubcf5\ub155\ub2f9 \uc544\uae30\uc528\u2019\ub85c \ubd88\ub838\uace0, 1962\ub144 \u2018\uc774\ub355\ud61c\u2019(\u674e\u5fb7\u60e0)\ub85c \ub300\ud55c\ubbfc\uad6d\uc758 \uad6d\uc801\uc744 \ucde8\ub4dd\ud558\uc600\ub2e4. \uc77c\uc81c \uac15\uc810\uae30 \uacbd\uae30\ub3c4 \uacbd\uc131\ubd80 \ub355\uc218\uad81\uc5d0\uc11c \ud0dc\uc5b4\ub098 \uacbd\uc131\uc77c\ucd9c\uacf5\ub9bd\uc2ec\uc0c1\uc18c\ud559\uad50 \uc7ac\ud559 \uc911\uc5d0 \uc77c\ubcf8\uc758 \uac15\uc81c\uc801\uc778 \uc694\uad6c\uc5d0 \ub530\ub77c \uc720\ud559\uc744 \uba85\ubd84\uc73c\ub85c \ub3c4\ucfc4\ub85c \ubcf4\ub0b4\uc838 \uc77c\ubcf8 \ud669\uc871\ub4e4\uc774 \uacf5\ubd80\ud558\ub294 \ud559\uad50\uc778 \uc5ec\uc790 \uac00\ucfe0\uc288\uc778\uc5d0\uc11c \uc218\ud559\ud558\uc600\ub2e4. 1931\ub144 \uc61b \uc4f0\uc2dc\ub9c8 \ubc88\uc8fc \uac00\ubb38\uc758 \ub2f9\uc8fc\uc774\uc790 \ubc31\uc791 \uc18c \ub2e4\ucf00\uc720\ud0a4\uc640 \uc815\ub7b5 \uacb0\ud63c\uc744 \ud558\uc5ec 1932\ub144 \ub538 \uc18c \ub9c8\uc0ac\uc5d0\ub97c \ub0b3\uc558\ub2e4. \uadf8\ub7ec\ub098 \uc774\uc988\uc74c \uc870\uc6b8\uc99d, \uc6b0\uc6b8\uc7a5\uc560, \ubc18\ubcf5\uc131 \uc6b0\uc6b8 \uc7a5\uc560\uc640 \ub354\ubd88\uc5b4 \uc815\uc2e0\uc7a5\uc560\uc778 \uc870\ud604\ubcd1(\uc815\uc2e0\ubd84\uc5f4\uc99d) \uc99d\uc138\ub97c \ucc98\uc74c \ubcf4\uc600\uc73c\uba70, \uacb0\ud63c \uc774\ud6c4 \ubcd1\uc138\uac00 \uc545\ud654\ub418\uc5c8\ub2e4. 1946\ub144\ubd80\ud130 \ub9c8\uc4f0\uc790\uc640 \ub3c4\ub9bd \uc815\uc2e0\ubcd1\uc6d0\uc5d0 \uc785\uc6d0\ud558\uc600\uace0, 1955\ub144 \uc774\ud63c\ud558\uc600\ub2e4.", "answer": "\ub355\ud61c\uc639\uc8fc", "sentence": "\ub355\ud61c\uc639\uc8fc (\u5fb7\u60e0\u7fc1\u4e3b, 1912\ub144 5\uc6d4 25\uc77c ~ 1989\ub144 4\uc6d4 21\uc77c)\ub294 \uc870\uc120\uc758 \uc81c26\ub300 \uc655\uc774\uc790 \ub300\ud55c\uc81c\uad6d\uc758 \ucd08\ub300 \ud669\uc81c\uc600\ub358 \uace0\uc885\uacfc \uadc0\uc778 \uc591\uc528 \uc0ac\uc774\uc5d0\uc11c \ud0dc\uc5b4\ub09c \uace0\uba85\ub538\uc774\ub2e4.", "paragraph_sentence": " \ub355\ud61c\uc639\uc8fc (\u5fb7\u60e0\u7fc1\u4e3b, 1912\ub144 5\uc6d4 25\uc77c ~ 1989\ub144 4\uc6d4 21\uc77c)\ub294 \uc870\uc120\uc758 \uc81c26\ub300 \uc655\uc774\uc790 \ub300\ud55c\uc81c\uad6d\uc758 \ucd08\ub300 \ud669\uc81c\uc600\ub358 \uace0\uc885\uacfc \uadc0\uc778 \uc591\uc528 \uc0ac\uc774\uc5d0\uc11c \ud0dc\uc5b4\ub09c \uace0\uba85\ub538\uc774\ub2e4. \ud669\ub140\ub85c\uc11c \ub355\ud61c\ub77c\ub294 \ud638\ub97c \ud558\uc0ac\ubc1b\uae30 \uc804\uae4c\uc9c0 \u2018\ubcf5\ub155\ub2f9 \uc544\uae30\uc528\u2019\ub85c \ubd88\ub838\uace0, 1962\ub144 \u2018\uc774\ub355\ud61c\u2019(\u674e\u5fb7\u60e0)\ub85c \ub300\ud55c\ubbfc\uad6d\uc758 \uad6d\uc801\uc744 \ucde8\ub4dd\ud558\uc600\ub2e4. \uc77c\uc81c \uac15\uc810\uae30 \uacbd\uae30\ub3c4 \uacbd\uc131\ubd80 \ub355\uc218\uad81\uc5d0\uc11c \ud0dc\uc5b4\ub098 \uacbd\uc131\uc77c\ucd9c\uacf5\ub9bd\uc2ec\uc0c1\uc18c\ud559\uad50 \uc7ac\ud559 \uc911\uc5d0 \uc77c\ubcf8\uc758 \uac15\uc81c\uc801\uc778 \uc694\uad6c\uc5d0 \ub530\ub77c \uc720\ud559\uc744 \uba85\ubd84\uc73c\ub85c \ub3c4\ucfc4\ub85c \ubcf4\ub0b4\uc838 \uc77c\ubcf8 \ud669\uc871\ub4e4\uc774 \uacf5\ubd80\ud558\ub294 \ud559\uad50\uc778 \uc5ec\uc790 \uac00\ucfe0\uc288\uc778\uc5d0\uc11c \uc218\ud559\ud558\uc600\ub2e4. 1931\ub144 \uc61b \uc4f0\uc2dc\ub9c8 \ubc88\uc8fc \uac00\ubb38\uc758 \ub2f9\uc8fc\uc774\uc790 \ubc31\uc791 \uc18c \ub2e4\ucf00\uc720\ud0a4\uc640 \uc815\ub7b5 \uacb0\ud63c\uc744 \ud558\uc5ec 1932\ub144 \ub538 \uc18c \ub9c8\uc0ac\uc5d0\ub97c \ub0b3\uc558\ub2e4. \uadf8\ub7ec\ub098 \uc774\uc988\uc74c \uc870\uc6b8\uc99d, \uc6b0\uc6b8\uc7a5\uc560, \ubc18\ubcf5\uc131 \uc6b0\uc6b8 \uc7a5\uc560\uc640 \ub354\ubd88\uc5b4 \uc815\uc2e0\uc7a5\uc560\uc778 \uc870\ud604\ubcd1(\uc815\uc2e0\ubd84\uc5f4\uc99d) \uc99d\uc138\ub97c \ucc98\uc74c \ubcf4\uc600\uc73c\uba70, \uacb0\ud63c \uc774\ud6c4 \ubcd1\uc138\uac00 \uc545\ud654\ub418\uc5c8\ub2e4. 1946\ub144\ubd80\ud130 \ub9c8\uc4f0\uc790\uc640 \ub3c4\ub9bd \uc815\uc2e0\ubcd1\uc6d0\uc5d0 \uc785\uc6d0\ud558\uc600\uace0, 1955\ub144 \uc774\ud63c\ud558\uc600\ub2e4.", "paragraph_answer": " \ub355\ud61c\uc639\uc8fc (\u5fb7\u60e0\u7fc1\u4e3b, 1912\ub144 5\uc6d4 25\uc77c ~ 1989\ub144 4\uc6d4 21\uc77c)\ub294 \uc870\uc120\uc758 \uc81c26\ub300 \uc655\uc774\uc790 \ub300\ud55c\uc81c\uad6d\uc758 \ucd08\ub300 \ud669\uc81c\uc600\ub358 \uace0\uc885\uacfc \uadc0\uc778 \uc591\uc528 \uc0ac\uc774\uc5d0\uc11c \ud0dc\uc5b4\ub09c \uace0\uba85\ub538\uc774\ub2e4. \ud669\ub140\ub85c\uc11c \ub355\ud61c\ub77c\ub294 \ud638\ub97c \ud558\uc0ac\ubc1b\uae30 \uc804\uae4c\uc9c0 \u2018\ubcf5\ub155\ub2f9 \uc544\uae30\uc528\u2019\ub85c \ubd88\ub838\uace0, 1962\ub144 \u2018\uc774\ub355\ud61c\u2019(\u674e\u5fb7\u60e0)\ub85c \ub300\ud55c\ubbfc\uad6d\uc758 \uad6d\uc801\uc744 \ucde8\ub4dd\ud558\uc600\ub2e4. \uc77c\uc81c \uac15\uc810\uae30 \uacbd\uae30\ub3c4 \uacbd\uc131\ubd80 \ub355\uc218\uad81\uc5d0\uc11c \ud0dc\uc5b4\ub098 \uacbd\uc131\uc77c\ucd9c\uacf5\ub9bd\uc2ec\uc0c1\uc18c\ud559\uad50 \uc7ac\ud559 \uc911\uc5d0 \uc77c\ubcf8\uc758 \uac15\uc81c\uc801\uc778 \uc694\uad6c\uc5d0 \ub530\ub77c \uc720\ud559\uc744 \uba85\ubd84\uc73c\ub85c \ub3c4\ucfc4\ub85c \ubcf4\ub0b4\uc838 \uc77c\ubcf8 \ud669\uc871\ub4e4\uc774 \uacf5\ubd80\ud558\ub294 \ud559\uad50\uc778 \uc5ec\uc790 \uac00\ucfe0\uc288\uc778\uc5d0\uc11c \uc218\ud559\ud558\uc600\ub2e4. 1931\ub144 \uc61b \uc4f0\uc2dc\ub9c8 \ubc88\uc8fc \uac00\ubb38\uc758 \ub2f9\uc8fc\uc774\uc790 \ubc31\uc791 \uc18c \ub2e4\ucf00\uc720\ud0a4\uc640 \uc815\ub7b5 \uacb0\ud63c\uc744 \ud558\uc5ec 1932\ub144 \ub538 \uc18c \ub9c8\uc0ac\uc5d0\ub97c \ub0b3\uc558\ub2e4. \uadf8\ub7ec\ub098 \uc774\uc988\uc74c \uc870\uc6b8\uc99d, \uc6b0\uc6b8\uc7a5\uc560, \ubc18\ubcf5\uc131 \uc6b0\uc6b8 \uc7a5\uc560\uc640 \ub354\ubd88\uc5b4 \uc815\uc2e0\uc7a5\uc560\uc778 \uc870\ud604\ubcd1(\uc815\uc2e0\ubd84\uc5f4\uc99d) \uc99d\uc138\ub97c \ucc98\uc74c \ubcf4\uc600\uc73c\uba70, \uacb0\ud63c \uc774\ud6c4 \ubcd1\uc138\uac00 \uc545\ud654\ub418\uc5c8\ub2e4. 1946\ub144\ubd80\ud130 \ub9c8\uc4f0\uc790\uc640 \ub3c4\ub9bd \uc815\uc2e0\ubcd1\uc6d0\uc5d0 \uc785\uc6d0\ud558\uc600\uace0, 1955\ub144 \uc774\ud63c\ud558\uc600\ub2e4.", "sentence_answer": " \ub355\ud61c\uc639\uc8fc (\u5fb7\u60e0\u7fc1\u4e3b, 1912\ub144 5\uc6d4 25\uc77c ~ 1989\ub144 4\uc6d4 21\uc77c)\ub294 \uc870\uc120\uc758 \uc81c26\ub300 \uc655\uc774\uc790 \ub300\ud55c\uc81c\uad6d\uc758 \ucd08\ub300 \ud669\uc81c\uc600\ub358 \uace0\uc885\uacfc \uadc0\uc778 \uc591\uc528 \uc0ac\uc774\uc5d0\uc11c \ud0dc\uc5b4\ub09c \uace0\uba85\ub538\uc774\ub2e4.", "paragraph_id": "6292399-0-0", "paragraph_question": "question: \ubcf5\ub155\ub2f9 \uc544\uae30\uc528\ub77c\uace0 \ubd88\ub838\ub358 \uc870\uc120\uc758 \uace0\uc885\ud669\uc81c\uc758 \uace0\uba85\ub538\uc740 \ub204\uad6c\uc778\uac00?, context: \ub355\ud61c\uc639\uc8fc(\u5fb7\u60e0\u7fc1\u4e3b, 1912\ub144 5\uc6d4 25\uc77c ~ 1989\ub144 4\uc6d4 21\uc77c)\ub294 \uc870\uc120\uc758 \uc81c26\ub300 \uc655\uc774\uc790 \ub300\ud55c\uc81c\uad6d\uc758 \ucd08\ub300 \ud669\uc81c\uc600\ub358 \uace0\uc885\uacfc \uadc0\uc778 \uc591\uc528 \uc0ac\uc774\uc5d0\uc11c \ud0dc\uc5b4\ub09c \uace0\uba85\ub538\uc774\ub2e4. \ud669\ub140\ub85c\uc11c \ub355\ud61c\ub77c\ub294 \ud638\ub97c \ud558\uc0ac\ubc1b\uae30 \uc804\uae4c\uc9c0 \u2018\ubcf5\ub155\ub2f9 \uc544\uae30\uc528\u2019\ub85c \ubd88\ub838\uace0, 1962\ub144 \u2018\uc774\ub355\ud61c\u2019(\u674e\u5fb7\u60e0)\ub85c \ub300\ud55c\ubbfc\uad6d\uc758 \uad6d\uc801\uc744 \ucde8\ub4dd\ud558\uc600\ub2e4. \uc77c\uc81c \uac15\uc810\uae30 \uacbd\uae30\ub3c4 \uacbd\uc131\ubd80 \ub355\uc218\uad81\uc5d0\uc11c \ud0dc\uc5b4\ub098 \uacbd\uc131\uc77c\ucd9c\uacf5\ub9bd\uc2ec\uc0c1\uc18c\ud559\uad50 \uc7ac\ud559 \uc911\uc5d0 \uc77c\ubcf8\uc758 \uac15\uc81c\uc801\uc778 \uc694\uad6c\uc5d0 \ub530\ub77c \uc720\ud559\uc744 \uba85\ubd84\uc73c\ub85c \ub3c4\ucfc4\ub85c \ubcf4\ub0b4\uc838 \uc77c\ubcf8 \ud669\uc871\ub4e4\uc774 \uacf5\ubd80\ud558\ub294 \ud559\uad50\uc778 \uc5ec\uc790 \uac00\ucfe0\uc288\uc778\uc5d0\uc11c \uc218\ud559\ud558\uc600\ub2e4. 1931\ub144 \uc61b \uc4f0\uc2dc\ub9c8 \ubc88\uc8fc \uac00\ubb38\uc758 \ub2f9\uc8fc\uc774\uc790 \ubc31\uc791 \uc18c \ub2e4\ucf00\uc720\ud0a4\uc640 \uc815\ub7b5 \uacb0\ud63c\uc744 \ud558\uc5ec 1932\ub144 \ub538 \uc18c \ub9c8\uc0ac\uc5d0\ub97c \ub0b3\uc558\ub2e4. \uadf8\ub7ec\ub098 \uc774\uc988\uc74c \uc870\uc6b8\uc99d, \uc6b0\uc6b8\uc7a5\uc560, \ubc18\ubcf5\uc131 \uc6b0\uc6b8 \uc7a5\uc560\uc640 \ub354\ubd88\uc5b4 \uc815\uc2e0\uc7a5\uc560\uc778 \uc870\ud604\ubcd1(\uc815\uc2e0\ubd84\uc5f4\uc99d) \uc99d\uc138\ub97c \ucc98\uc74c \ubcf4\uc600\uc73c\uba70, \uacb0\ud63c \uc774\ud6c4 \ubcd1\uc138\uac00 \uc545\ud654\ub418\uc5c8\ub2e4. 1946\ub144\ubd80\ud130 \ub9c8\uc4f0\uc790\uc640 \ub3c4\ub9bd \uc815\uc2e0\ubcd1\uc6d0\uc5d0 \uc785\uc6d0\ud558\uc600\uace0, 1955\ub144 \uc774\ud63c\ud558\uc600\ub2e4."} {"question": "UEFA \uc720\ub85c 1976\uc5d0 \ucd9c\uc804\ud574 \ud398\ub110\ud2f0 \ud0a5\uc744 \ub123\uc740 \ud30c\ub128\uce74\ub294 \uba87 \ubc88\uc9f8 \uc120\uc218\ub85c \ub098\uc130\ub098?", "paragraph": "\ud30c\ub128\uce74\ub294 UEFA \uc720\ub85c 1976\uc5d0 \ucd9c\uc804\ud558\uc5ec \ub9ce\uc740 \uba85\uc131\uc744 \uc5bb\uc5c8\ub2e4. \uccb4\ucf54\uc2ac\ub85c\ubc14\ud0a4\uc544\ub294 \uacb0\uc2b9\uc804\uc5d0 \ub3c4\ub2ec\ud588\uace0, \uc11c\ub3c5\uacfc \uacb0\uc2b9\uc804\uc5d0\uc11c \ub9cc\ub0ac\ub2e4. \uacbd\uae30\ub294 \uc5f0\uc7a5\uc804\uae4c\uc9c0 \uac14\uc73c\ub098 2-2\ub85c \uc2b9\ubd80\ub97c \uac00\ub9ac\uc9c0 \ubabb\ud558\uace0 UEFA \uc720\ub7fd \ucd95\uad6c \uc120\uc218\uad8c \ub300\ud68c \uacb0\uc2b9\uc804 \uc5ed\uc0ac\uc0c1 \ucc98\uc74c\uc73c\ub85c \uc2b9\ubd80\ucc28\uae30\uc5d0 \ub3cc\uc785\ud588\ub2e4. \uc591 \ud300 \ubaa8\ub450 \uc138 \ubc88\uc9f8 \uc120\uc218\uae4c\uc9c0\ub294 \uc131\uacf5\ud588\uc73c\ub098, \uc11c\ub3c5\uc758 \ub124 \ubc88\uc9f8 \uc120\uc218 \uc6b8\ub9ac \ud68c\ub124\uc2a4\uac00 \uacf5\uc744 \uace8\ub300 \uc704\ub85c \ucc28\uc11c \uc2e4\ucd95\ud588\ub2e4. 4-3\uc758 \uc0c1\ud669\uc5d0\uc11c, \ud30c\ub128\uce74\ub294 \uccb4\ucf54\uc2ac\ub85c\ubc14\ud0a4\uc544\uc758 \ub2e4\uc12f \ubc88\uc9f8 \uc120\uc218\ub85c \ub098\uc130\ub2e4. \uc11c\ub3c5 \uace8\ud0a4\ud37c \uc81c\ud504 \ub9c8\uc774\uc5b4\ub294 \uc67c\ucabd\uc73c\ub85c \ubab8\uc744 \ub0a0\ub838\uc9c0\ub9cc, \ud30c\ub128\uce74\ub294 \uacf5\uc744 \uac00\ubccd\uac8c \ucc28\uc11c \uc815\uba74\uc73c\ub85c \ub0a0\ub824 \ubcf4\ub0c8\uace0 \uacf5\uc740 \uadf8\ub300\ub85c \ub4e4\uc5b4\uac14\ub2e4. \uc774\ud6c4 \uc774\ub7ec\ud55c \ud615\uc2dd\uc758 \ud398\ub110\ud2f0 \ud0a5\uc740 \ud30c\ub128\uce74\uc758 \uc774\ub984\uc744 \ub530\uc11c '\ud30c\ub128\uce74 \ud0a5'\uc774\ub77c\uace0 \uba85\uba85\ub418\uc5c8\ub2e4.", "answer": "\ub2e4\uc12f \ubc88\uc9f8", "sentence": "4-3\uc758 \uc0c1\ud669\uc5d0\uc11c, \ud30c\ub128\uce74\ub294 \uccb4\ucf54\uc2ac\ub85c\ubc14\ud0a4\uc544\uc758 \ub2e4\uc12f \ubc88\uc9f8 \uc120\uc218\ub85c \ub098\uc130\ub2e4.", "paragraph_sentence": "\ud30c\ub128\uce74\ub294 UEFA \uc720\ub85c 1976\uc5d0 \ucd9c\uc804\ud558\uc5ec \ub9ce\uc740 \uba85\uc131\uc744 \uc5bb\uc5c8\ub2e4. \uccb4\ucf54\uc2ac\ub85c\ubc14\ud0a4\uc544\ub294 \uacb0\uc2b9\uc804\uc5d0 \ub3c4\ub2ec\ud588\uace0, \uc11c\ub3c5\uacfc \uacb0\uc2b9\uc804\uc5d0\uc11c \ub9cc\ub0ac\ub2e4. \uacbd\uae30\ub294 \uc5f0\uc7a5\uc804\uae4c\uc9c0 \uac14\uc73c\ub098 2-2\ub85c \uc2b9\ubd80\ub97c \uac00\ub9ac\uc9c0 \ubabb\ud558\uace0 UEFA \uc720\ub7fd \ucd95\uad6c \uc120\uc218\uad8c \ub300\ud68c \uacb0\uc2b9\uc804 \uc5ed\uc0ac\uc0c1 \ucc98\uc74c\uc73c\ub85c \uc2b9\ubd80\ucc28\uae30\uc5d0 \ub3cc\uc785\ud588\ub2e4. \uc591 \ud300 \ubaa8\ub450 \uc138 \ubc88\uc9f8 \uc120\uc218\uae4c\uc9c0\ub294 \uc131\uacf5\ud588\uc73c\ub098, \uc11c\ub3c5\uc758 \ub124 \ubc88\uc9f8 \uc120\uc218 \uc6b8\ub9ac \ud68c\ub124\uc2a4\uac00 \uacf5\uc744 \uace8\ub300 \uc704\ub85c \ucc28\uc11c \uc2e4\ucd95\ud588\ub2e4. 4-3\uc758 \uc0c1\ud669\uc5d0\uc11c, \ud30c\ub128\uce74\ub294 \uccb4\ucf54\uc2ac\ub85c\ubc14\ud0a4\uc544\uc758 \ub2e4\uc12f \ubc88\uc9f8 \uc120\uc218\ub85c \ub098\uc130\ub2e4. \uc11c\ub3c5 \uace8\ud0a4\ud37c \uc81c\ud504 \ub9c8\uc774\uc5b4\ub294 \uc67c\ucabd\uc73c\ub85c \ubab8\uc744 \ub0a0\ub838\uc9c0\ub9cc, \ud30c\ub128\uce74\ub294 \uacf5\uc744 \uac00\ubccd\uac8c \ucc28\uc11c \uc815\uba74\uc73c\ub85c \ub0a0\ub824 \ubcf4\ub0c8\uace0 \uacf5\uc740 \uadf8\ub300\ub85c \ub4e4\uc5b4\uac14\ub2e4. \uc774\ud6c4 \uc774\ub7ec\ud55c \ud615\uc2dd\uc758 \ud398\ub110\ud2f0 \ud0a5\uc740 \ud30c\ub128\uce74\uc758 \uc774\ub984\uc744 \ub530\uc11c '\ud30c\ub128\uce74 \ud0a5'\uc774\ub77c\uace0 \uba85\uba85\ub418\uc5c8\ub2e4.", "paragraph_answer": "\ud30c\ub128\uce74\ub294 UEFA \uc720\ub85c 1976\uc5d0 \ucd9c\uc804\ud558\uc5ec \ub9ce\uc740 \uba85\uc131\uc744 \uc5bb\uc5c8\ub2e4. \uccb4\ucf54\uc2ac\ub85c\ubc14\ud0a4\uc544\ub294 \uacb0\uc2b9\uc804\uc5d0 \ub3c4\ub2ec\ud588\uace0, \uc11c\ub3c5\uacfc \uacb0\uc2b9\uc804\uc5d0\uc11c \ub9cc\ub0ac\ub2e4. \uacbd\uae30\ub294 \uc5f0\uc7a5\uc804\uae4c\uc9c0 \uac14\uc73c\ub098 2-2\ub85c \uc2b9\ubd80\ub97c \uac00\ub9ac\uc9c0 \ubabb\ud558\uace0 UEFA \uc720\ub7fd \ucd95\uad6c \uc120\uc218\uad8c \ub300\ud68c \uacb0\uc2b9\uc804 \uc5ed\uc0ac\uc0c1 \ucc98\uc74c\uc73c\ub85c \uc2b9\ubd80\ucc28\uae30\uc5d0 \ub3cc\uc785\ud588\ub2e4. \uc591 \ud300 \ubaa8\ub450 \uc138 \ubc88\uc9f8 \uc120\uc218\uae4c\uc9c0\ub294 \uc131\uacf5\ud588\uc73c\ub098, \uc11c\ub3c5\uc758 \ub124 \ubc88\uc9f8 \uc120\uc218 \uc6b8\ub9ac \ud68c\ub124\uc2a4\uac00 \uacf5\uc744 \uace8\ub300 \uc704\ub85c \ucc28\uc11c \uc2e4\ucd95\ud588\ub2e4. 4-3\uc758 \uc0c1\ud669\uc5d0\uc11c, \ud30c\ub128\uce74\ub294 \uccb4\ucf54\uc2ac\ub85c\ubc14\ud0a4\uc544\uc758 \ub2e4\uc12f \ubc88\uc9f8 \uc120\uc218\ub85c \ub098\uc130\ub2e4. \uc11c\ub3c5 \uace8\ud0a4\ud37c \uc81c\ud504 \ub9c8\uc774\uc5b4\ub294 \uc67c\ucabd\uc73c\ub85c \ubab8\uc744 \ub0a0\ub838\uc9c0\ub9cc, \ud30c\ub128\uce74\ub294 \uacf5\uc744 \uac00\ubccd\uac8c \ucc28\uc11c \uc815\uba74\uc73c\ub85c \ub0a0\ub824 \ubcf4\ub0c8\uace0 \uacf5\uc740 \uadf8\ub300\ub85c \ub4e4\uc5b4\uac14\ub2e4. \uc774\ud6c4 \uc774\ub7ec\ud55c \ud615\uc2dd\uc758 \ud398\ub110\ud2f0 \ud0a5\uc740 \ud30c\ub128\uce74\uc758 \uc774\ub984\uc744 \ub530\uc11c '\ud30c\ub128\uce74 \ud0a5'\uc774\ub77c\uace0 \uba85\uba85\ub418\uc5c8\ub2e4.", "sentence_answer": "4-3\uc758 \uc0c1\ud669\uc5d0\uc11c, \ud30c\ub128\uce74\ub294 \uccb4\ucf54\uc2ac\ub85c\ubc14\ud0a4\uc544\uc758 \ub2e4\uc12f \ubc88\uc9f8 \uc120\uc218\ub85c \ub098\uc130\ub2e4.", "paragraph_id": "6457763-0-2", "paragraph_question": "question: UEFA \uc720\ub85c 1976\uc5d0 \ucd9c\uc804\ud574 \ud398\ub110\ud2f0 \ud0a5\uc744 \ub123\uc740 \ud30c\ub128\uce74\ub294 \uba87 \ubc88\uc9f8 \uc120\uc218\ub85c \ub098\uc130\ub098?, context: \ud30c\ub128\uce74\ub294 UEFA \uc720\ub85c 1976\uc5d0 \ucd9c\uc804\ud558\uc5ec \ub9ce\uc740 \uba85\uc131\uc744 \uc5bb\uc5c8\ub2e4. \uccb4\ucf54\uc2ac\ub85c\ubc14\ud0a4\uc544\ub294 \uacb0\uc2b9\uc804\uc5d0 \ub3c4\ub2ec\ud588\uace0, \uc11c\ub3c5\uacfc \uacb0\uc2b9\uc804\uc5d0\uc11c \ub9cc\ub0ac\ub2e4. \uacbd\uae30\ub294 \uc5f0\uc7a5\uc804\uae4c\uc9c0 \uac14\uc73c\ub098 2-2\ub85c \uc2b9\ubd80\ub97c \uac00\ub9ac\uc9c0 \ubabb\ud558\uace0 UEFA \uc720\ub7fd \ucd95\uad6c \uc120\uc218\uad8c \ub300\ud68c \uacb0\uc2b9\uc804 \uc5ed\uc0ac\uc0c1 \ucc98\uc74c\uc73c\ub85c \uc2b9\ubd80\ucc28\uae30\uc5d0 \ub3cc\uc785\ud588\ub2e4. \uc591 \ud300 \ubaa8\ub450 \uc138 \ubc88\uc9f8 \uc120\uc218\uae4c\uc9c0\ub294 \uc131\uacf5\ud588\uc73c\ub098, \uc11c\ub3c5\uc758 \ub124 \ubc88\uc9f8 \uc120\uc218 \uc6b8\ub9ac \ud68c\ub124\uc2a4\uac00 \uacf5\uc744 \uace8\ub300 \uc704\ub85c \ucc28\uc11c \uc2e4\ucd95\ud588\ub2e4. 4-3\uc758 \uc0c1\ud669\uc5d0\uc11c, \ud30c\ub128\uce74\ub294 \uccb4\ucf54\uc2ac\ub85c\ubc14\ud0a4\uc544\uc758 \ub2e4\uc12f \ubc88\uc9f8 \uc120\uc218\ub85c \ub098\uc130\ub2e4. \uc11c\ub3c5 \uace8\ud0a4\ud37c \uc81c\ud504 \ub9c8\uc774\uc5b4\ub294 \uc67c\ucabd\uc73c\ub85c \ubab8\uc744 \ub0a0\ub838\uc9c0\ub9cc, \ud30c\ub128\uce74\ub294 \uacf5\uc744 \uac00\ubccd\uac8c \ucc28\uc11c \uc815\uba74\uc73c\ub85c \ub0a0\ub824 \ubcf4\ub0c8\uace0 \uacf5\uc740 \uadf8\ub300\ub85c \ub4e4\uc5b4\uac14\ub2e4. \uc774\ud6c4 \uc774\ub7ec\ud55c \ud615\uc2dd\uc758 \ud398\ub110\ud2f0 \ud0a5\uc740 \ud30c\ub128\uce74\uc758 \uc774\ub984\uc744 \ub530\uc11c '\ud30c\ub128\uce74 \ud0a5'\uc774\ub77c\uace0 \uba85\uba85\ub418\uc5c8\ub2e4."} {"question": "\uc138\uc885\ub300\uc655 \uc774\uc804 1\uacb0\ub2f9 \uc218\uc870\ub7c9\uc740?", "paragraph": "\ud604\uc9c1,\ud1f4\uc9c1\uad00\ub8cc\uc5d0\uac8c \uc218\uc870\uad8c\uc744 \uc900 \uac83\uc73c\ub85c \uad00\ub8cc \ubcf8\uc778 \uc989, \uc77c\ub300(\u4e00\u4ee3)\uc5d0 \ud55c\ud558\uc5ec \uc218\uc870\uad8c\uc744 \uc778\uc815\ud574 \uc900 \uc81c\ub3c4\uc774\ub2e4. \uad00\ub8cc\uac00 \uc0ac\ub9dd\ud558\uba74 \ubc18\ub0a9\ud558\ub294 \uac83\uc774 \uc6d0\uce59\uc774\ub098, \uc720\uc871\uc758 \uc0dd\uacc4 \uc720\uc9c0\ub77c\ub294 \uba85\ubaa9\uc73c\ub85c \ud73c\uc591\uc804\uacfc \uc218\uc2e0\uc804\uc744 \ud1b5\ud558\uc5ec \uadf8 \ud1a0\uc9c0\ub97c \uc77c\ubd80\ub77c\ub3c4 \ubb3c\ub824\ubc1b\uc744 \uc218 \uc788\uc5c8\ub2e4. \uad00\ub9ac\ub294 \uacfc\uc804\uc5d0\uc11c \ub098\uc624\ub294 \uc18c\ucd9c\uc758 1\ud560(10\ubd84\uc758 1)\uc744 \uc870\uc138\ub85c \ubc1b\uc558\ub2e4. \uacfc\uc804\ubc95 \ub2f9\uc2dc\uc758 1\uacb0\uc758 \uc218\ud655\ub7c9\uc740 300\ub450(\ub9d0)\ub85c 30\ub450\uae4c\uc9c0 \uc218\uc870\uad8c\uc73c\ub85c \uac70\ub450\uc5b4 \ub4e4\uc774\ub294 \uac83\uc774 \uac00\ub2a5\ud558\uc600\uc73c\uba70, \uc774\ub294 \uc138\uc885\ub300\uc655\ub54c\uc758 \uacf5\ubc95\uc73c\ub85c 1\uacb0\ub2f9 \uc0dd\uc0b0\ub7c9\uc744 400\ub450(\uae30\uc874 300\ub450)\ub85c \ubcc0\uacbd\ud558\uace0 \uc218\uc870\ub7c9\uc744 1/20(\uae30\uc874 1/10)\ub85c \ubcc0\uacbd\ud558\uc600\uc744 \ub54c\uae4c\uc9c0 \uc720\uc9c0\ub418\uc5c8\ub2e4. \uc774\ub97c \ubcc0\uacbd\ud55c \uc774\uc720\ub294 \ubc31\uc131\uc758 \uc804\uc138\ubd80\ub2f4\uc744 \uacbd\uac10\uc2dc\ud0a4\uae30 \uc704\ud574\uc11c\uc600\ub294\ub370, \uc774\ub85c \uc778\ud558\uc5ec 1\uacb0\ub2f9 \uc218\uc870\ub7c9\uc774 30\ub450\uc5d0\uc11c 20\ub450\ub85c \uacbd\uac10\ud558\uc600\ub2e4.", "answer": "30\ub450", "sentence": "\uacfc\uc804\ubc95 \ub2f9\uc2dc\uc758 1\uacb0\uc758 \uc218\ud655\ub7c9\uc740 300\ub450(\ub9d0)\ub85c 30\ub450 \uae4c\uc9c0 \uc218\uc870\uad8c\uc73c\ub85c \uac70\ub450\uc5b4 \ub4e4\uc774\ub294 \uac83\uc774 \uac00\ub2a5\ud558\uc600\uc73c\uba70, \uc774\ub294 \uc138\uc885\ub300\uc655\ub54c\uc758 \uacf5\ubc95\uc73c\ub85c 1\uacb0\ub2f9 \uc0dd\uc0b0\ub7c9\uc744 400\ub450(\uae30\uc874 300\ub450)\ub85c \ubcc0\uacbd\ud558\uace0 \uc218\uc870\ub7c9\uc744 1/20(\uae30\uc874 1/10)\ub85c \ubcc0\uacbd\ud558\uc600\uc744 \ub54c\uae4c\uc9c0 \uc720\uc9c0\ub418\uc5c8\ub2e4.", "paragraph_sentence": "\ud604\uc9c1,\ud1f4\uc9c1\uad00\ub8cc\uc5d0\uac8c \uc218\uc870\uad8c\uc744 \uc900 \uac83\uc73c\ub85c \uad00\ub8cc \ubcf8\uc778 \uc989, \uc77c\ub300(\u4e00\u4ee3)\uc5d0 \ud55c\ud558\uc5ec \uc218\uc870\uad8c\uc744 \uc778\uc815\ud574 \uc900 \uc81c\ub3c4\uc774\ub2e4. \uad00\ub8cc\uac00 \uc0ac\ub9dd\ud558\uba74 \ubc18\ub0a9\ud558\ub294 \uac83\uc774 \uc6d0\uce59\uc774\ub098, \uc720\uc871\uc758 \uc0dd\uacc4 \uc720\uc9c0\ub77c\ub294 \uba85\ubaa9\uc73c\ub85c \ud73c\uc591\uc804\uacfc \uc218\uc2e0\uc804\uc744 \ud1b5\ud558\uc5ec \uadf8 \ud1a0\uc9c0\ub97c \uc77c\ubd80\ub77c\ub3c4 \ubb3c\ub824\ubc1b\uc744 \uc218 \uc788\uc5c8\ub2e4. \uad00\ub9ac\ub294 \uacfc\uc804\uc5d0\uc11c \ub098\uc624\ub294 \uc18c\ucd9c\uc758 1\ud560(10\ubd84\uc758 1)\uc744 \uc870\uc138\ub85c \ubc1b\uc558\ub2e4. \uacfc\uc804\ubc95 \ub2f9\uc2dc\uc758 1\uacb0\uc758 \uc218\ud655\ub7c9\uc740 300\ub450(\ub9d0)\ub85c 30\ub450 \uae4c\uc9c0 \uc218\uc870\uad8c\uc73c\ub85c \uac70\ub450\uc5b4 \ub4e4\uc774\ub294 \uac83\uc774 \uac00\ub2a5\ud558\uc600\uc73c\uba70, \uc774\ub294 \uc138\uc885\ub300\uc655\ub54c\uc758 \uacf5\ubc95\uc73c\ub85c 1\uacb0\ub2f9 \uc0dd\uc0b0\ub7c9\uc744 400\ub450(\uae30\uc874 300\ub450)\ub85c \ubcc0\uacbd\ud558\uace0 \uc218\uc870\ub7c9\uc744 1/20(\uae30\uc874 1/10)\ub85c \ubcc0\uacbd\ud558\uc600\uc744 \ub54c\uae4c\uc9c0 \uc720\uc9c0\ub418\uc5c8\ub2e4. \uc774\ub97c \ubcc0\uacbd\ud55c \uc774\uc720\ub294 \ubc31\uc131\uc758 \uc804\uc138\ubd80\ub2f4\uc744 \uacbd\uac10\uc2dc\ud0a4\uae30 \uc704\ud574\uc11c\uc600\ub294\ub370, \uc774\ub85c \uc778\ud558\uc5ec 1\uacb0\ub2f9 \uc218\uc870\ub7c9\uc774 30\ub450\uc5d0\uc11c 20\ub450\ub85c \uacbd\uac10\ud558\uc600\ub2e4.", "paragraph_answer": "\ud604\uc9c1,\ud1f4\uc9c1\uad00\ub8cc\uc5d0\uac8c \uc218\uc870\uad8c\uc744 \uc900 \uac83\uc73c\ub85c \uad00\ub8cc \ubcf8\uc778 \uc989, \uc77c\ub300(\u4e00\u4ee3)\uc5d0 \ud55c\ud558\uc5ec \uc218\uc870\uad8c\uc744 \uc778\uc815\ud574 \uc900 \uc81c\ub3c4\uc774\ub2e4. \uad00\ub8cc\uac00 \uc0ac\ub9dd\ud558\uba74 \ubc18\ub0a9\ud558\ub294 \uac83\uc774 \uc6d0\uce59\uc774\ub098, \uc720\uc871\uc758 \uc0dd\uacc4 \uc720\uc9c0\ub77c\ub294 \uba85\ubaa9\uc73c\ub85c \ud73c\uc591\uc804\uacfc \uc218\uc2e0\uc804\uc744 \ud1b5\ud558\uc5ec \uadf8 \ud1a0\uc9c0\ub97c \uc77c\ubd80\ub77c\ub3c4 \ubb3c\ub824\ubc1b\uc744 \uc218 \uc788\uc5c8\ub2e4. \uad00\ub9ac\ub294 \uacfc\uc804\uc5d0\uc11c \ub098\uc624\ub294 \uc18c\ucd9c\uc758 1\ud560(10\ubd84\uc758 1)\uc744 \uc870\uc138\ub85c \ubc1b\uc558\ub2e4. \uacfc\uc804\ubc95 \ub2f9\uc2dc\uc758 1\uacb0\uc758 \uc218\ud655\ub7c9\uc740 300\ub450(\ub9d0)\ub85c 30\ub450 \uae4c\uc9c0 \uc218\uc870\uad8c\uc73c\ub85c \uac70\ub450\uc5b4 \ub4e4\uc774\ub294 \uac83\uc774 \uac00\ub2a5\ud558\uc600\uc73c\uba70, \uc774\ub294 \uc138\uc885\ub300\uc655\ub54c\uc758 \uacf5\ubc95\uc73c\ub85c 1\uacb0\ub2f9 \uc0dd\uc0b0\ub7c9\uc744 400\ub450(\uae30\uc874 300\ub450)\ub85c \ubcc0\uacbd\ud558\uace0 \uc218\uc870\ub7c9\uc744 1/20(\uae30\uc874 1/10)\ub85c \ubcc0\uacbd\ud558\uc600\uc744 \ub54c\uae4c\uc9c0 \uc720\uc9c0\ub418\uc5c8\ub2e4. \uc774\ub97c \ubcc0\uacbd\ud55c \uc774\uc720\ub294 \ubc31\uc131\uc758 \uc804\uc138\ubd80\ub2f4\uc744 \uacbd\uac10\uc2dc\ud0a4\uae30 \uc704\ud574\uc11c\uc600\ub294\ub370, \uc774\ub85c \uc778\ud558\uc5ec 1\uacb0\ub2f9 \uc218\uc870\ub7c9\uc774 30\ub450\uc5d0\uc11c 20\ub450\ub85c \uacbd\uac10\ud558\uc600\ub2e4.", "sentence_answer": "\uacfc\uc804\ubc95 \ub2f9\uc2dc\uc758 1\uacb0\uc758 \uc218\ud655\ub7c9\uc740 300\ub450(\ub9d0)\ub85c 30\ub450 \uae4c\uc9c0 \uc218\uc870\uad8c\uc73c\ub85c \uac70\ub450\uc5b4 \ub4e4\uc774\ub294 \uac83\uc774 \uac00\ub2a5\ud558\uc600\uc73c\uba70, \uc774\ub294 \uc138\uc885\ub300\uc655\ub54c\uc758 \uacf5\ubc95\uc73c\ub85c 1\uacb0\ub2f9 \uc0dd\uc0b0\ub7c9\uc744 400\ub450(\uae30\uc874 300\ub450)\ub85c \ubcc0\uacbd\ud558\uace0 \uc218\uc870\ub7c9\uc744 1/20(\uae30\uc874 1/10)\ub85c \ubcc0\uacbd\ud558\uc600\uc744 \ub54c\uae4c\uc9c0 \uc720\uc9c0\ub418\uc5c8\ub2e4.", "paragraph_id": "6657794-0-2", "paragraph_question": "question: \uc138\uc885\ub300\uc655 \uc774\uc804 1\uacb0\ub2f9 \uc218\uc870\ub7c9\uc740?, context: \ud604\uc9c1,\ud1f4\uc9c1\uad00\ub8cc\uc5d0\uac8c \uc218\uc870\uad8c\uc744 \uc900 \uac83\uc73c\ub85c \uad00\ub8cc \ubcf8\uc778 \uc989, \uc77c\ub300(\u4e00\u4ee3)\uc5d0 \ud55c\ud558\uc5ec \uc218\uc870\uad8c\uc744 \uc778\uc815\ud574 \uc900 \uc81c\ub3c4\uc774\ub2e4. \uad00\ub8cc\uac00 \uc0ac\ub9dd\ud558\uba74 \ubc18\ub0a9\ud558\ub294 \uac83\uc774 \uc6d0\uce59\uc774\ub098, \uc720\uc871\uc758 \uc0dd\uacc4 \uc720\uc9c0\ub77c\ub294 \uba85\ubaa9\uc73c\ub85c \ud73c\uc591\uc804\uacfc \uc218\uc2e0\uc804\uc744 \ud1b5\ud558\uc5ec \uadf8 \ud1a0\uc9c0\ub97c \uc77c\ubd80\ub77c\ub3c4 \ubb3c\ub824\ubc1b\uc744 \uc218 \uc788\uc5c8\ub2e4. \uad00\ub9ac\ub294 \uacfc\uc804\uc5d0\uc11c \ub098\uc624\ub294 \uc18c\ucd9c\uc758 1\ud560(10\ubd84\uc758 1)\uc744 \uc870\uc138\ub85c \ubc1b\uc558\ub2e4. \uacfc\uc804\ubc95 \ub2f9\uc2dc\uc758 1\uacb0\uc758 \uc218\ud655\ub7c9\uc740 300\ub450(\ub9d0)\ub85c 30\ub450\uae4c\uc9c0 \uc218\uc870\uad8c\uc73c\ub85c \uac70\ub450\uc5b4 \ub4e4\uc774\ub294 \uac83\uc774 \uac00\ub2a5\ud558\uc600\uc73c\uba70, \uc774\ub294 \uc138\uc885\ub300\uc655\ub54c\uc758 \uacf5\ubc95\uc73c\ub85c 1\uacb0\ub2f9 \uc0dd\uc0b0\ub7c9\uc744 400\ub450(\uae30\uc874 300\ub450)\ub85c \ubcc0\uacbd\ud558\uace0 \uc218\uc870\ub7c9\uc744 1/20(\uae30\uc874 1/10)\ub85c \ubcc0\uacbd\ud558\uc600\uc744 \ub54c\uae4c\uc9c0 \uc720\uc9c0\ub418\uc5c8\ub2e4. \uc774\ub97c \ubcc0\uacbd\ud55c \uc774\uc720\ub294 \ubc31\uc131\uc758 \uc804\uc138\ubd80\ub2f4\uc744 \uacbd\uac10\uc2dc\ud0a4\uae30 \uc704\ud574\uc11c\uc600\ub294\ub370, \uc774\ub85c \uc778\ud558\uc5ec 1\uacb0\ub2f9 \uc218\uc870\ub7c9\uc774 30\ub450\uc5d0\uc11c 20\ub450\ub85c \uacbd\uac10\ud558\uc600\ub2e4."} {"question": "\uc11c\uc778\uc601\uc774 \uc568\ubc94 \uc218\ub85d\uace1 '\ub108\ub97c \uc6d0\ud574'\ub97c \ucc98\uc74c\uc73c\ub85c \ubc29\uc1a1\ud558\uac8c\ub41c \ud504\ub85c\uadf8\ub7a8 \uc774\ub984\uc740?", "paragraph": "\uc11c\uc778\uc601\uc758 \uc194\ub85c \ub370\ubdd4\ub294 2005\ub144\ubd80\ud130 \uacc4\uc18d \uc5b8\uae09 \ub3fc \uc654\ub294\ub370, \ub2f9\uc2dc \uc194\ub85c \ub370\ubdd4\ub97c \uc704\ud574 \ud799\ud569\ub304\uc2a4, \ubcf4\uceec \ud2b8\ub808\uc774\ub2dd, \uccb4\ud615 \uad50\uc815\uc744 \ubc1b\ub294 \uc911\uc774\ub77c\uace0 \ub9d0\ud588\ub2e4. 2007\ub144 3\uc6d4 5\uc77c \ub9c8\uce68\ub0b4 \uc11c\uc778\uc601\uc758 \uccab \uc815\uaddc \uc568\ubc94\uc774\uc790 \uc194\ub85c \uc568\ubc94 \u300aElly Is so HOT\u300b\uc774 \ubc1c\ub9e4\ub418\uc5c8\ub2e4. \uc568\ubc94 \uacf5\uac1c\uc5d0 \uc55e\uc11c 2007\ub144 1\uc6d4 \uc194\ub85c \uc568\ubc94\uc758 \ub179\uc74c\uc774 \ub9c8\ubb34\ub9ac\ub418\uc5c8\ub2e4\uace0 \uc54c\ub838\uace0, 1\uc6d4 25\uc77c '\ub9ac\uc5bc \uc624\ub514\uc158 \uc624\ud508 \ucf58\uc11c\ud2b8'\uc5d0\uc11c \uc218\ub85d\uace1\uc778 \u3008\uac00\ub974\uccd0\uc918\uc694\u3009\ub97c \uc120\ubcf4\uc600\ub2e4. 2\uc6d4 23\uc77c\uc5d0\ub294 \uc1fc\ucf00\uc774\uc2a4\uac00 \uc5f4\ub838\uc73c\uba70, 25\uc77c\uc5d0\ub294 \u300a\uc778\uae30\uac00\uc694\u300b\ub97c \ud1b5\ud574 \ucc98\uc74c\uc73c\ub85c \ubc29\uc1a1\uc5d0\uc11c \uc120\ubcf4\uc600\ub2e4. \ud0c0\uc774\ud2c0\uace1\uc740 \u3008\ub108\ub97c \uc6d0\ud574\u3009\ub85c, \ucee8\uc149\uc774 \uad49\uc7a5\ud788 \ud30c\uaca9\uc801\uc774\uc600\ub294\ub370, \"\uce58\uace8 \ud328\uc158\"\uacfc \uba54\uc774\ud06c\uc5c5 \uc2a4\ud0c0\uc77c \ub4f1 \ub300\uc911\ub4e4\uc5d0\uac8c \uc2e0\uc120\ud55c \ucda9\uaca9\uc744 \uc8fc\uc5c8\ub2e4. \uadf8\ub7ec\ub098, \ud30c\uaca9\uc801\uc778 \ub178\ucd9c\ub85c \ub9ce\uc740 \ube44\ub09c\uc744 \ubc1b\uc558\uace0, \uc9c0\uc0c1\ud30c \uc74c\uc545 \ud504\ub85c\uadf8\ub7a8\uc5d0\uc11c\ub294 \"\uce58\uace8 \ud328\uc158\"\uc774 \ubc29\uc1a1 \ubd88\uac00 \ud310\uc815\uc744 \ubc1b\uc544 \ub178\ucd9c\uc774 \uc801\uc740 \uc758\uc0c1\uc73c\ub85c \ubb34\ub300\uc5d0 \uc62c\ub790\ub2e4. \ub178\ucd9c\uc774 \uc801\uc5b4\uc9c4 \uc758\uc0c1\uc758 \uc774\ub984\uc740 \"\uc2dc\uc2a4\ub8e8\ub8e9\"\uc774\ub77c\uace0 \ubd88\ub9ac\uba70 \uc5ed\uc2dc \ud654\uc81c\ub97c \ubaa8\uc558\ub2e4. \uc11c\uc778\uc601\uc740 \uc758\uc0c1 \uc2ec\uc758\uc5d0 \ub300\ud574 \u300a\uc870\uc774\ub274\uc2a424\u300b\uc640\uc758 \uc778\ud130\ubdf0\uc5d0\uc11c \"\uc6b0\ub9ac\ub098\ub77c\uac00 \uc720\ub3c5 \ub178\ucd9c\uc5d0 \ubbfc\uac10\ud55c \uac83 \uac19\ub2e4. \uc6b0\ub9ac \ubb38\ud654\uac00 \ubcf4\uc218\uc801\uc774\uae30 \ub54c\ubb38\uc778\uac70 \uac19\ub2e4.\"\ub77c\uba70 \uc544\uc26c\uc6c0\uc744 \ud45c\ud588\ub2e4. \uc774\ud6c4 \ud558\uc6b0\uc2a4 \ud48d\uc758 \u3008\ub108\ub97c \uc6d0\ud574\u3009 \ub9ac\ubbf9\uc2a4 \ubc84\uc804\uc73c\ub85c \ud65c\ub3d9\ud588\ub2e4. \u3008\ub108\ub97c \uc6d0\ud574 remix ver.\u3009\ub294 2\uc8fc \uc5f0\uc18d \ubc29\uc1a1 \ud69f\uc218 \uc885\ud569 \ucc28\ud2b8 1\uc704\ub97c \ucc28\uc9c0\ud588\ub2e4. \ud0c0\uc774\ud2c0\uace1 \ud65c\ub3d9\uc774 \ub05d\ub098\uace0 \ub098\uc11c \ub4dc\ub77c\ub9c8 \u300a\ud788\ud2b8\u300b\uc5d0 \uc0bd\uc785\uace1\uc73c\ub85c \uc4f0\uc778 \u3008HIT\u3009\uc640 \ubc1c\ub77c\ub4dc \u3008\uac00\ub974\uccd0\uc918\uc694\u3009 \ub450 \uace1 \uc911 \ud558\ub098\ub85c \ud6c4\uc18d\uace1 \ud65c\ub3d9\uc744 \uc2dc\uc791\ud558\ub824 \ud588\uace0, \uc11c\uc778\uc601\uc740 \uc74c\uc545\uc131\uc744 \ub300\uc911\uc5d0\uac8c \ub354 \uc120\ubcf4\uc774\uace0 \uc2f6\uc5b4\ud574 \u3008\uac00\ub974\uccd0\uc918\uc694\u3009\ub85c \ud65c\ub3d9\uc744 \uc2dc\uc791\ud588\ub2e4. 4\uc6d4 28\uc77c \u300a\uc74c\uc545\uc911\uc2ec\u300b\uc5d0\uc11c \ud6c4\uc18d\uace1 \ubb34\ub300\ub97c \ucc98\uc74c\uc73c\ub85c \uc120\ubcf4\uc600\ub2e4.", "answer": "\uc778\uae30\uac00\uc694", "sentence": "2\uc6d4 23\uc77c\uc5d0\ub294 \uc1fc\ucf00\uc774\uc2a4\uac00 \uc5f4\ub838\uc73c\uba70, 25\uc77c\uc5d0\ub294 \u300a \uc778\uae30\uac00\uc694 \u300b\ub97c \ud1b5\ud574 \ucc98\uc74c\uc73c\ub85c \ubc29\uc1a1\uc5d0\uc11c \uc120\ubcf4\uc600\ub2e4.", "paragraph_sentence": "\uc11c\uc778\uc601\uc758 \uc194\ub85c \ub370\ubdd4\ub294 2005\ub144\ubd80\ud130 \uacc4\uc18d \uc5b8\uae09 \ub3fc \uc654\ub294\ub370, \ub2f9\uc2dc \uc194\ub85c \ub370\ubdd4\ub97c \uc704\ud574 \ud799\ud569\ub304\uc2a4, \ubcf4\uceec \ud2b8\ub808\uc774\ub2dd, \uccb4\ud615 \uad50\uc815\uc744 \ubc1b\ub294 \uc911\uc774\ub77c\uace0 \ub9d0\ud588\ub2e4. 2007\ub144 3\uc6d4 5\uc77c \ub9c8\uce68\ub0b4 \uc11c\uc778\uc601\uc758 \uccab \uc815\uaddc \uc568\ubc94\uc774\uc790 \uc194\ub85c \uc568\ubc94 \u300aElly Is so HOT\u300b\uc774 \ubc1c\ub9e4\ub418\uc5c8\ub2e4. \uc568\ubc94 \uacf5\uac1c\uc5d0 \uc55e\uc11c 2007\ub144 1\uc6d4 \uc194\ub85c \uc568\ubc94\uc758 \ub179\uc74c\uc774 \ub9c8\ubb34\ub9ac\ub418\uc5c8\ub2e4\uace0 \uc54c\ub838\uace0, 1\uc6d4 25\uc77c '\ub9ac\uc5bc \uc624\ub514\uc158 \uc624\ud508 \ucf58\uc11c\ud2b8'\uc5d0\uc11c \uc218\ub85d\uace1\uc778 \u3008\uac00\ub974\uccd0\uc918\uc694\u3009\ub97c \uc120\ubcf4\uc600\ub2e4. 2\uc6d4 23\uc77c\uc5d0\ub294 \uc1fc\ucf00\uc774\uc2a4\uac00 \uc5f4\ub838\uc73c\uba70, 25\uc77c\uc5d0\ub294 \u300a \uc778\uae30\uac00\uc694 \u300b\ub97c \ud1b5\ud574 \ucc98\uc74c\uc73c\ub85c \ubc29\uc1a1\uc5d0\uc11c \uc120\ubcf4\uc600\ub2e4. \ud0c0\uc774\ud2c0\uace1\uc740 \u3008\ub108\ub97c \uc6d0\ud574\u3009\ub85c, \ucee8\uc149\uc774 \uad49\uc7a5\ud788 \ud30c\uaca9\uc801\uc774\uc600\ub294\ub370, \"\uce58\uace8 \ud328\uc158\"\uacfc \uba54\uc774\ud06c\uc5c5 \uc2a4\ud0c0\uc77c \ub4f1 \ub300\uc911\ub4e4\uc5d0\uac8c \uc2e0\uc120\ud55c \ucda9\uaca9\uc744 \uc8fc\uc5c8\ub2e4. \uadf8\ub7ec\ub098, \ud30c\uaca9\uc801\uc778 \ub178\ucd9c\ub85c \ub9ce\uc740 \ube44\ub09c\uc744 \ubc1b\uc558\uace0, \uc9c0\uc0c1\ud30c \uc74c\uc545 \ud504\ub85c\uadf8\ub7a8\uc5d0\uc11c\ub294 \"\uce58\uace8 \ud328\uc158\"\uc774 \ubc29\uc1a1 \ubd88\uac00 \ud310\uc815\uc744 \ubc1b\uc544 \ub178\ucd9c\uc774 \uc801\uc740 \uc758\uc0c1\uc73c\ub85c \ubb34\ub300\uc5d0 \uc62c\ub790\ub2e4. \ub178\ucd9c\uc774 \uc801\uc5b4\uc9c4 \uc758\uc0c1\uc758 \uc774\ub984\uc740 \"\uc2dc\uc2a4\ub8e8\ub8e9\"\uc774\ub77c\uace0 \ubd88\ub9ac\uba70 \uc5ed\uc2dc \ud654\uc81c\ub97c \ubaa8\uc558\ub2e4. \uc11c\uc778\uc601\uc740 \uc758\uc0c1 \uc2ec\uc758\uc5d0 \ub300\ud574 \u300a\uc870\uc774\ub274\uc2a424\u300b\uc640\uc758 \uc778\ud130\ubdf0\uc5d0\uc11c \"\uc6b0\ub9ac\ub098\ub77c\uac00 \uc720\ub3c5 \ub178\ucd9c\uc5d0 \ubbfc\uac10\ud55c \uac83 \uac19\ub2e4. \uc6b0\ub9ac \ubb38\ud654\uac00 \ubcf4\uc218\uc801\uc774\uae30 \ub54c\ubb38\uc778\uac70 \uac19\ub2e4. \"\ub77c\uba70 \uc544\uc26c\uc6c0\uc744 \ud45c\ud588\ub2e4. \uc774\ud6c4 \ud558\uc6b0\uc2a4 \ud48d\uc758 \u3008\ub108\ub97c \uc6d0\ud574\u3009 \ub9ac\ubbf9\uc2a4 \ubc84\uc804\uc73c\ub85c \ud65c\ub3d9\ud588\ub2e4. \u3008\ub108\ub97c \uc6d0\ud574 remix ver.\u3009\ub294 2\uc8fc \uc5f0\uc18d \ubc29\uc1a1 \ud69f\uc218 \uc885\ud569 \ucc28\ud2b8 1\uc704\ub97c \ucc28\uc9c0\ud588\ub2e4. \ud0c0\uc774\ud2c0\uace1 \ud65c\ub3d9\uc774 \ub05d\ub098\uace0 \ub098\uc11c \ub4dc\ub77c\ub9c8 \u300a\ud788\ud2b8\u300b\uc5d0 \uc0bd\uc785\uace1\uc73c\ub85c \uc4f0\uc778 \u3008HIT\u3009\uc640 \ubc1c\ub77c\ub4dc \u3008\uac00\ub974\uccd0\uc918\uc694\u3009 \ub450 \uace1 \uc911 \ud558\ub098\ub85c \ud6c4\uc18d\uace1 \ud65c\ub3d9\uc744 \uc2dc\uc791\ud558\ub824 \ud588\uace0, \uc11c\uc778\uc601\uc740 \uc74c\uc545\uc131\uc744 \ub300\uc911\uc5d0\uac8c \ub354 \uc120\ubcf4\uc774\uace0 \uc2f6\uc5b4\ud574 \u3008\uac00\ub974\uccd0\uc918\uc694\u3009\ub85c \ud65c\ub3d9\uc744 \uc2dc\uc791\ud588\ub2e4. 4\uc6d4 28\uc77c \u300a\uc74c\uc545\uc911\uc2ec\u300b\uc5d0\uc11c \ud6c4\uc18d\uace1 \ubb34\ub300\ub97c \ucc98\uc74c\uc73c\ub85c \uc120\ubcf4\uc600\ub2e4.", "paragraph_answer": "\uc11c\uc778\uc601\uc758 \uc194\ub85c \ub370\ubdd4\ub294 2005\ub144\ubd80\ud130 \uacc4\uc18d \uc5b8\uae09 \ub3fc \uc654\ub294\ub370, \ub2f9\uc2dc \uc194\ub85c \ub370\ubdd4\ub97c \uc704\ud574 \ud799\ud569\ub304\uc2a4, \ubcf4\uceec \ud2b8\ub808\uc774\ub2dd, \uccb4\ud615 \uad50\uc815\uc744 \ubc1b\ub294 \uc911\uc774\ub77c\uace0 \ub9d0\ud588\ub2e4. 2007\ub144 3\uc6d4 5\uc77c \ub9c8\uce68\ub0b4 \uc11c\uc778\uc601\uc758 \uccab \uc815\uaddc \uc568\ubc94\uc774\uc790 \uc194\ub85c \uc568\ubc94 \u300aElly Is so HOT\u300b\uc774 \ubc1c\ub9e4\ub418\uc5c8\ub2e4. \uc568\ubc94 \uacf5\uac1c\uc5d0 \uc55e\uc11c 2007\ub144 1\uc6d4 \uc194\ub85c \uc568\ubc94\uc758 \ub179\uc74c\uc774 \ub9c8\ubb34\ub9ac\ub418\uc5c8\ub2e4\uace0 \uc54c\ub838\uace0, 1\uc6d4 25\uc77c '\ub9ac\uc5bc \uc624\ub514\uc158 \uc624\ud508 \ucf58\uc11c\ud2b8'\uc5d0\uc11c \uc218\ub85d\uace1\uc778 \u3008\uac00\ub974\uccd0\uc918\uc694\u3009\ub97c \uc120\ubcf4\uc600\ub2e4. 2\uc6d4 23\uc77c\uc5d0\ub294 \uc1fc\ucf00\uc774\uc2a4\uac00 \uc5f4\ub838\uc73c\uba70, 25\uc77c\uc5d0\ub294 \u300a \uc778\uae30\uac00\uc694 \u300b\ub97c \ud1b5\ud574 \ucc98\uc74c\uc73c\ub85c \ubc29\uc1a1\uc5d0\uc11c \uc120\ubcf4\uc600\ub2e4. \ud0c0\uc774\ud2c0\uace1\uc740 \u3008\ub108\ub97c \uc6d0\ud574\u3009\ub85c, \ucee8\uc149\uc774 \uad49\uc7a5\ud788 \ud30c\uaca9\uc801\uc774\uc600\ub294\ub370, \"\uce58\uace8 \ud328\uc158\"\uacfc \uba54\uc774\ud06c\uc5c5 \uc2a4\ud0c0\uc77c \ub4f1 \ub300\uc911\ub4e4\uc5d0\uac8c \uc2e0\uc120\ud55c \ucda9\uaca9\uc744 \uc8fc\uc5c8\ub2e4. \uadf8\ub7ec\ub098, \ud30c\uaca9\uc801\uc778 \ub178\ucd9c\ub85c \ub9ce\uc740 \ube44\ub09c\uc744 \ubc1b\uc558\uace0, \uc9c0\uc0c1\ud30c \uc74c\uc545 \ud504\ub85c\uadf8\ub7a8\uc5d0\uc11c\ub294 \"\uce58\uace8 \ud328\uc158\"\uc774 \ubc29\uc1a1 \ubd88\uac00 \ud310\uc815\uc744 \ubc1b\uc544 \ub178\ucd9c\uc774 \uc801\uc740 \uc758\uc0c1\uc73c\ub85c \ubb34\ub300\uc5d0 \uc62c\ub790\ub2e4. \ub178\ucd9c\uc774 \uc801\uc5b4\uc9c4 \uc758\uc0c1\uc758 \uc774\ub984\uc740 \"\uc2dc\uc2a4\ub8e8\ub8e9\"\uc774\ub77c\uace0 \ubd88\ub9ac\uba70 \uc5ed\uc2dc \ud654\uc81c\ub97c \ubaa8\uc558\ub2e4. \uc11c\uc778\uc601\uc740 \uc758\uc0c1 \uc2ec\uc758\uc5d0 \ub300\ud574 \u300a\uc870\uc774\ub274\uc2a424\u300b\uc640\uc758 \uc778\ud130\ubdf0\uc5d0\uc11c \"\uc6b0\ub9ac\ub098\ub77c\uac00 \uc720\ub3c5 \ub178\ucd9c\uc5d0 \ubbfc\uac10\ud55c \uac83 \uac19\ub2e4. \uc6b0\ub9ac \ubb38\ud654\uac00 \ubcf4\uc218\uc801\uc774\uae30 \ub54c\ubb38\uc778\uac70 \uac19\ub2e4.\"\ub77c\uba70 \uc544\uc26c\uc6c0\uc744 \ud45c\ud588\ub2e4. \uc774\ud6c4 \ud558\uc6b0\uc2a4 \ud48d\uc758 \u3008\ub108\ub97c \uc6d0\ud574\u3009 \ub9ac\ubbf9\uc2a4 \ubc84\uc804\uc73c\ub85c \ud65c\ub3d9\ud588\ub2e4. \u3008\ub108\ub97c \uc6d0\ud574 remix ver.\u3009\ub294 2\uc8fc \uc5f0\uc18d \ubc29\uc1a1 \ud69f\uc218 \uc885\ud569 \ucc28\ud2b8 1\uc704\ub97c \ucc28\uc9c0\ud588\ub2e4. \ud0c0\uc774\ud2c0\uace1 \ud65c\ub3d9\uc774 \ub05d\ub098\uace0 \ub098\uc11c \ub4dc\ub77c\ub9c8 \u300a\ud788\ud2b8\u300b\uc5d0 \uc0bd\uc785\uace1\uc73c\ub85c \uc4f0\uc778 \u3008HIT\u3009\uc640 \ubc1c\ub77c\ub4dc \u3008\uac00\ub974\uccd0\uc918\uc694\u3009 \ub450 \uace1 \uc911 \ud558\ub098\ub85c \ud6c4\uc18d\uace1 \ud65c\ub3d9\uc744 \uc2dc\uc791\ud558\ub824 \ud588\uace0, \uc11c\uc778\uc601\uc740 \uc74c\uc545\uc131\uc744 \ub300\uc911\uc5d0\uac8c \ub354 \uc120\ubcf4\uc774\uace0 \uc2f6\uc5b4\ud574 \u3008\uac00\ub974\uccd0\uc918\uc694\u3009\ub85c \ud65c\ub3d9\uc744 \uc2dc\uc791\ud588\ub2e4. 4\uc6d4 28\uc77c \u300a\uc74c\uc545\uc911\uc2ec\u300b\uc5d0\uc11c \ud6c4\uc18d\uace1 \ubb34\ub300\ub97c \ucc98\uc74c\uc73c\ub85c \uc120\ubcf4\uc600\ub2e4.", "sentence_answer": "2\uc6d4 23\uc77c\uc5d0\ub294 \uc1fc\ucf00\uc774\uc2a4\uac00 \uc5f4\ub838\uc73c\uba70, 25\uc77c\uc5d0\ub294 \u300a \uc778\uae30\uac00\uc694 \u300b\ub97c \ud1b5\ud574 \ucc98\uc74c\uc73c\ub85c \ubc29\uc1a1\uc5d0\uc11c \uc120\ubcf4\uc600\ub2e4.", "paragraph_id": "6463309-3-1", "paragraph_question": "question: \uc11c\uc778\uc601\uc774 \uc568\ubc94 \uc218\ub85d\uace1 '\ub108\ub97c \uc6d0\ud574'\ub97c \ucc98\uc74c\uc73c\ub85c \ubc29\uc1a1\ud558\uac8c\ub41c \ud504\ub85c\uadf8\ub7a8 \uc774\ub984\uc740?, context: \uc11c\uc778\uc601\uc758 \uc194\ub85c \ub370\ubdd4\ub294 2005\ub144\ubd80\ud130 \uacc4\uc18d \uc5b8\uae09 \ub3fc \uc654\ub294\ub370, \ub2f9\uc2dc \uc194\ub85c \ub370\ubdd4\ub97c \uc704\ud574 \ud799\ud569\ub304\uc2a4, \ubcf4\uceec \ud2b8\ub808\uc774\ub2dd, \uccb4\ud615 \uad50\uc815\uc744 \ubc1b\ub294 \uc911\uc774\ub77c\uace0 \ub9d0\ud588\ub2e4. 2007\ub144 3\uc6d4 5\uc77c \ub9c8\uce68\ub0b4 \uc11c\uc778\uc601\uc758 \uccab \uc815\uaddc \uc568\ubc94\uc774\uc790 \uc194\ub85c \uc568\ubc94 \u300aElly Is so HOT\u300b\uc774 \ubc1c\ub9e4\ub418\uc5c8\ub2e4. \uc568\ubc94 \uacf5\uac1c\uc5d0 \uc55e\uc11c 2007\ub144 1\uc6d4 \uc194\ub85c \uc568\ubc94\uc758 \ub179\uc74c\uc774 \ub9c8\ubb34\ub9ac\ub418\uc5c8\ub2e4\uace0 \uc54c\ub838\uace0, 1\uc6d4 25\uc77c '\ub9ac\uc5bc \uc624\ub514\uc158 \uc624\ud508 \ucf58\uc11c\ud2b8'\uc5d0\uc11c \uc218\ub85d\uace1\uc778 \u3008\uac00\ub974\uccd0\uc918\uc694\u3009\ub97c \uc120\ubcf4\uc600\ub2e4. 2\uc6d4 23\uc77c\uc5d0\ub294 \uc1fc\ucf00\uc774\uc2a4\uac00 \uc5f4\ub838\uc73c\uba70, 25\uc77c\uc5d0\ub294 \u300a\uc778\uae30\uac00\uc694\u300b\ub97c \ud1b5\ud574 \ucc98\uc74c\uc73c\ub85c \ubc29\uc1a1\uc5d0\uc11c \uc120\ubcf4\uc600\ub2e4. \ud0c0\uc774\ud2c0\uace1\uc740 \u3008\ub108\ub97c \uc6d0\ud574\u3009\ub85c, \ucee8\uc149\uc774 \uad49\uc7a5\ud788 \ud30c\uaca9\uc801\uc774\uc600\ub294\ub370, \"\uce58\uace8 \ud328\uc158\"\uacfc \uba54\uc774\ud06c\uc5c5 \uc2a4\ud0c0\uc77c \ub4f1 \ub300\uc911\ub4e4\uc5d0\uac8c \uc2e0\uc120\ud55c \ucda9\uaca9\uc744 \uc8fc\uc5c8\ub2e4. \uadf8\ub7ec\ub098, \ud30c\uaca9\uc801\uc778 \ub178\ucd9c\ub85c \ub9ce\uc740 \ube44\ub09c\uc744 \ubc1b\uc558\uace0, \uc9c0\uc0c1\ud30c \uc74c\uc545 \ud504\ub85c\uadf8\ub7a8\uc5d0\uc11c\ub294 \"\uce58\uace8 \ud328\uc158\"\uc774 \ubc29\uc1a1 \ubd88\uac00 \ud310\uc815\uc744 \ubc1b\uc544 \ub178\ucd9c\uc774 \uc801\uc740 \uc758\uc0c1\uc73c\ub85c \ubb34\ub300\uc5d0 \uc62c\ub790\ub2e4. \ub178\ucd9c\uc774 \uc801\uc5b4\uc9c4 \uc758\uc0c1\uc758 \uc774\ub984\uc740 \"\uc2dc\uc2a4\ub8e8\ub8e9\"\uc774\ub77c\uace0 \ubd88\ub9ac\uba70 \uc5ed\uc2dc \ud654\uc81c\ub97c \ubaa8\uc558\ub2e4. \uc11c\uc778\uc601\uc740 \uc758\uc0c1 \uc2ec\uc758\uc5d0 \ub300\ud574 \u300a\uc870\uc774\ub274\uc2a424\u300b\uc640\uc758 \uc778\ud130\ubdf0\uc5d0\uc11c \"\uc6b0\ub9ac\ub098\ub77c\uac00 \uc720\ub3c5 \ub178\ucd9c\uc5d0 \ubbfc\uac10\ud55c \uac83 \uac19\ub2e4. \uc6b0\ub9ac \ubb38\ud654\uac00 \ubcf4\uc218\uc801\uc774\uae30 \ub54c\ubb38\uc778\uac70 \uac19\ub2e4.\"\ub77c\uba70 \uc544\uc26c\uc6c0\uc744 \ud45c\ud588\ub2e4. \uc774\ud6c4 \ud558\uc6b0\uc2a4 \ud48d\uc758 \u3008\ub108\ub97c \uc6d0\ud574\u3009 \ub9ac\ubbf9\uc2a4 \ubc84\uc804\uc73c\ub85c \ud65c\ub3d9\ud588\ub2e4. \u3008\ub108\ub97c \uc6d0\ud574 remix ver.\u3009\ub294 2\uc8fc \uc5f0\uc18d \ubc29\uc1a1 \ud69f\uc218 \uc885\ud569 \ucc28\ud2b8 1\uc704\ub97c \ucc28\uc9c0\ud588\ub2e4. \ud0c0\uc774\ud2c0\uace1 \ud65c\ub3d9\uc774 \ub05d\ub098\uace0 \ub098\uc11c \ub4dc\ub77c\ub9c8 \u300a\ud788\ud2b8\u300b\uc5d0 \uc0bd\uc785\uace1\uc73c\ub85c \uc4f0\uc778 \u3008HIT\u3009\uc640 \ubc1c\ub77c\ub4dc \u3008\uac00\ub974\uccd0\uc918\uc694\u3009 \ub450 \uace1 \uc911 \ud558\ub098\ub85c \ud6c4\uc18d\uace1 \ud65c\ub3d9\uc744 \uc2dc\uc791\ud558\ub824 \ud588\uace0, \uc11c\uc778\uc601\uc740 \uc74c\uc545\uc131\uc744 \ub300\uc911\uc5d0\uac8c \ub354 \uc120\ubcf4\uc774\uace0 \uc2f6\uc5b4\ud574 \u3008\uac00\ub974\uccd0\uc918\uc694\u3009\ub85c \ud65c\ub3d9\uc744 \uc2dc\uc791\ud588\ub2e4. 4\uc6d4 28\uc77c \u300a\uc74c\uc545\uc911\uc2ec\u300b\uc5d0\uc11c \ud6c4\uc18d\uace1 \ubb34\ub300\ub97c \ucc98\uc74c\uc73c\ub85c \uc120\ubcf4\uc600\ub2e4."} {"question": "\ud30c\ubca8 1\uc138\ub294 \uc218\ubcf4\ub85c\ud504\uac00 \ub204\uad6c\uc758 \uc9c0\ud718\ub97c \ubc1b\uac8c \ud558\uc600\ub294\uac00?", "paragraph": "\uc77c\ubc18\uc801\uc73c\ub85c \uc704\ub300\ud55c \uc7a5\uad70\ub4e4\uc740 \uc804\ud22c\uc5d0\uc11c\ub294 \uc9c0\uc9c0 \uc54a\ub354\ub77c\ub3c4 \uc815\uce58\uafbc\ub4e4\uc758 \uc74c\ubaa8\ub294 \uc81c\ub300\ub85c \uc0c1\ub300\ud558\uc9c0 \ubabb\ud558\ub294 \ubc95\uc774\ub2e4. \uc624\uc2a4\ud2b8\ub9ac\uc544\uc640 \uc601\uad6d\uc740 \uc774\ud0c8\ub9ac\uc544\uc5d0\uc11c \ub7ec\uc2dc\uc544\uc758 \uc131\uacf5\uc744 \ubcf4\uace0 \ub450\ub824\uc6b4 \ub9c8\uc74c\uc744 \ud488\uc5c8\uace0, \uacb0\uad6d \uc218\ubcf4\ub85c\ud504\uc758 \ud504\ub791\uc2a4 \ubcf8\ud1a0 \uc9c4\uaca9 \uacc4\ud68d\uc744 \uc88c\uc808\uc2dc\ucf30\ub2e4. \ub300\uc2e0 \uc800\uc9c0\ub300 \uc9c0\ubc29\uc758 \uc804\uc5ed\uc5d0 \uc9d1\uc911\ud558\uac8c \ud558\uc600\ub2e4. \uc218\ubcf4\ub85c\ud504\uc758 \ubc18\ub300\uc5d0\ub3c4 \ubd88\uad6c\ud558\uace0 \ub7ec\uc2dc\uc544\uc758 \ud669\uc81c \ud30c\ubca8 1\uc138\ub294 \uc218\ubcf4\ub85c\ud504\uc5d0\uac8c \uc2a4\uc704\uc2a4\ub85c \ubcd1\ub825\uc744 \uc774\ub3d9\uc2dc\ucf1c \ubb34\ub2a5\ud55c \uc54c\ub809\uc0b0\ub4dc\ub974 \ucf54\ub974\uc0ac\ucf54\ud504(Alexander Korsakov)\uc758 \uc9c0\ud718\ub97c \ubc1b\uac8c \ud558\uc600\ub2e4. \uadf8\ub7ec\ub098 \ucf54\ub974\uc0ac\ucf54\ud504\ub294 \uc559\ub4dc\ub808 \ub9c8\uc138\ub098(Andre Massena)\uc5d0\uac8c \uc81c2\ucc28 \ucde8\ub9ac\ud788 \uc804\ud22c(Second Battle of Zurich)\uc5d0\uc11c \ub300\ud328\ub97c \ub2f9\ud588\ub2e4. \uc774 \uc2b9\uc138\ub97c \ubab0\uc544 \ub9c8\uc138\ub098\ub294 \ud718\ud558\uc5d0 \uc788\ub294 80,000\uba85\uc758 \ubcd1\uc0ac\ub97c \uc774\ub04c\uace0 \uc218\ubcf4\ub85c\ud504 \ud718\ud558\uc758 \ub7ec\uc2dc\uc544\uad70 \uc794\uc5ec\ubcd1\uc778 18,000\uba85\uc758 \uc815\uaddc\ubcd1\uacfc 5,000\uba85\uc758 \ucf54\uc0ac\ud06c(Cossacks) \ubd80\ub300\ub97c \uaca9\uba78\ud558\uae30 \uc704\ud574 \uc9c4\uad70\ud558\uc600\ub2e4. \uc218\ubcf4\ub85c\ud504\ub294 \ud1f4\uac01\ud558\uac70\ub098 \uc7a5\ub82c\ud558\uac8c \uc804\uba78\ud558\uac70\ub098 \ud558\ub294 \uc120\ud0dd\ubc16\uc5d0 \uc5c6\uc5c8\ub2e4.", "answer": "\uc54c\ub809\uc0b0\ub4dc\ub974 \ucf54\ub974\uc0ac\ucf54\ud504", "sentence": "\uc218\ubcf4\ub85c\ud504\uc758 \ubc18\ub300\uc5d0\ub3c4 \ubd88\uad6c\ud558\uace0 \ub7ec\uc2dc\uc544\uc758 \ud669\uc81c \ud30c\ubca8 1\uc138\ub294 \uc218\ubcf4\ub85c\ud504\uc5d0\uac8c \uc2a4\uc704\uc2a4\ub85c \ubcd1\ub825\uc744 \uc774\ub3d9\uc2dc\ucf1c \ubb34\ub2a5\ud55c \uc54c\ub809\uc0b0\ub4dc\ub974 \ucf54\ub974\uc0ac\ucf54\ud504 (Alexander Korsakov)", "paragraph_sentence": "\uc77c\ubc18\uc801\uc73c\ub85c \uc704\ub300\ud55c \uc7a5\uad70\ub4e4\uc740 \uc804\ud22c\uc5d0\uc11c\ub294 \uc9c0\uc9c0 \uc54a\ub354\ub77c\ub3c4 \uc815\uce58\uafbc\ub4e4\uc758 \uc74c\ubaa8\ub294 \uc81c\ub300\ub85c \uc0c1\ub300\ud558\uc9c0 \ubabb\ud558\ub294 \ubc95\uc774\ub2e4. \uc624\uc2a4\ud2b8\ub9ac\uc544\uc640 \uc601\uad6d\uc740 \uc774\ud0c8\ub9ac\uc544\uc5d0\uc11c \ub7ec\uc2dc\uc544\uc758 \uc131\uacf5\uc744 \ubcf4\uace0 \ub450\ub824\uc6b4 \ub9c8\uc74c\uc744 \ud488\uc5c8\uace0, \uacb0\uad6d \uc218\ubcf4\ub85c\ud504\uc758 \ud504\ub791\uc2a4 \ubcf8\ud1a0 \uc9c4\uaca9 \uacc4\ud68d\uc744 \uc88c\uc808\uc2dc\ucf30\ub2e4. \ub300\uc2e0 \uc800\uc9c0\ub300 \uc9c0\ubc29\uc758 \uc804\uc5ed\uc5d0 \uc9d1\uc911\ud558\uac8c \ud558\uc600\ub2e4. \uc218\ubcf4\ub85c\ud504\uc758 \ubc18\ub300\uc5d0\ub3c4 \ubd88\uad6c\ud558\uace0 \ub7ec\uc2dc\uc544\uc758 \ud669\uc81c \ud30c\ubca8 1\uc138\ub294 \uc218\ubcf4\ub85c\ud504\uc5d0\uac8c \uc2a4\uc704\uc2a4\ub85c \ubcd1\ub825\uc744 \uc774\ub3d9\uc2dc\ucf1c \ubb34\ub2a5\ud55c \uc54c\ub809\uc0b0\ub4dc\ub974 \ucf54\ub974\uc0ac\ucf54\ud504 (Alexander Korsakov) \uc758 \uc9c0\ud718\ub97c \ubc1b\uac8c \ud558\uc600\ub2e4. \uadf8\ub7ec\ub098 \ucf54\ub974\uc0ac\ucf54\ud504\ub294 \uc559\ub4dc\ub808 \ub9c8\uc138\ub098(Andre Massena)\uc5d0\uac8c \uc81c2\ucc28 \ucde8\ub9ac\ud788 \uc804\ud22c(Second Battle of Zurich)\uc5d0\uc11c \ub300\ud328\ub97c \ub2f9\ud588\ub2e4. \uc774 \uc2b9\uc138\ub97c \ubab0\uc544 \ub9c8\uc138\ub098\ub294 \ud718\ud558\uc5d0 \uc788\ub294 80,000\uba85\uc758 \ubcd1\uc0ac\ub97c \uc774\ub04c\uace0 \uc218\ubcf4\ub85c\ud504 \ud718\ud558\uc758 \ub7ec\uc2dc\uc544\uad70 \uc794\uc5ec\ubcd1\uc778 18,000\uba85\uc758 \uc815\uaddc\ubcd1\uacfc 5,000\uba85\uc758 \ucf54\uc0ac\ud06c(Cossacks) \ubd80\ub300\ub97c \uaca9\uba78\ud558\uae30 \uc704\ud574 \uc9c4\uad70\ud558\uc600\ub2e4. \uc218\ubcf4\ub85c\ud504\ub294 \ud1f4\uac01\ud558\uac70\ub098 \uc7a5\ub82c\ud558\uac8c \uc804\uba78\ud558\uac70\ub098 \ud558\ub294 \uc120\ud0dd\ubc16\uc5d0 \uc5c6\uc5c8\ub2e4.", "paragraph_answer": "\uc77c\ubc18\uc801\uc73c\ub85c \uc704\ub300\ud55c \uc7a5\uad70\ub4e4\uc740 \uc804\ud22c\uc5d0\uc11c\ub294 \uc9c0\uc9c0 \uc54a\ub354\ub77c\ub3c4 \uc815\uce58\uafbc\ub4e4\uc758 \uc74c\ubaa8\ub294 \uc81c\ub300\ub85c \uc0c1\ub300\ud558\uc9c0 \ubabb\ud558\ub294 \ubc95\uc774\ub2e4. \uc624\uc2a4\ud2b8\ub9ac\uc544\uc640 \uc601\uad6d\uc740 \uc774\ud0c8\ub9ac\uc544\uc5d0\uc11c \ub7ec\uc2dc\uc544\uc758 \uc131\uacf5\uc744 \ubcf4\uace0 \ub450\ub824\uc6b4 \ub9c8\uc74c\uc744 \ud488\uc5c8\uace0, \uacb0\uad6d \uc218\ubcf4\ub85c\ud504\uc758 \ud504\ub791\uc2a4 \ubcf8\ud1a0 \uc9c4\uaca9 \uacc4\ud68d\uc744 \uc88c\uc808\uc2dc\ucf30\ub2e4. \ub300\uc2e0 \uc800\uc9c0\ub300 \uc9c0\ubc29\uc758 \uc804\uc5ed\uc5d0 \uc9d1\uc911\ud558\uac8c \ud558\uc600\ub2e4. \uc218\ubcf4\ub85c\ud504\uc758 \ubc18\ub300\uc5d0\ub3c4 \ubd88\uad6c\ud558\uace0 \ub7ec\uc2dc\uc544\uc758 \ud669\uc81c \ud30c\ubca8 1\uc138\ub294 \uc218\ubcf4\ub85c\ud504\uc5d0\uac8c \uc2a4\uc704\uc2a4\ub85c \ubcd1\ub825\uc744 \uc774\ub3d9\uc2dc\ucf1c \ubb34\ub2a5\ud55c \uc54c\ub809\uc0b0\ub4dc\ub974 \ucf54\ub974\uc0ac\ucf54\ud504 (Alexander Korsakov)\uc758 \uc9c0\ud718\ub97c \ubc1b\uac8c \ud558\uc600\ub2e4. \uadf8\ub7ec\ub098 \ucf54\ub974\uc0ac\ucf54\ud504\ub294 \uc559\ub4dc\ub808 \ub9c8\uc138\ub098(Andre Massena)\uc5d0\uac8c \uc81c2\ucc28 \ucde8\ub9ac\ud788 \uc804\ud22c(Second Battle of Zurich)\uc5d0\uc11c \ub300\ud328\ub97c \ub2f9\ud588\ub2e4. \uc774 \uc2b9\uc138\ub97c \ubab0\uc544 \ub9c8\uc138\ub098\ub294 \ud718\ud558\uc5d0 \uc788\ub294 80,000\uba85\uc758 \ubcd1\uc0ac\ub97c \uc774\ub04c\uace0 \uc218\ubcf4\ub85c\ud504 \ud718\ud558\uc758 \ub7ec\uc2dc\uc544\uad70 \uc794\uc5ec\ubcd1\uc778 18,000\uba85\uc758 \uc815\uaddc\ubcd1\uacfc 5,000\uba85\uc758 \ucf54\uc0ac\ud06c(Cossacks) \ubd80\ub300\ub97c \uaca9\uba78\ud558\uae30 \uc704\ud574 \uc9c4\uad70\ud558\uc600\ub2e4. \uc218\ubcf4\ub85c\ud504\ub294 \ud1f4\uac01\ud558\uac70\ub098 \uc7a5\ub82c\ud558\uac8c \uc804\uba78\ud558\uac70\ub098 \ud558\ub294 \uc120\ud0dd\ubc16\uc5d0 \uc5c6\uc5c8\ub2e4.", "sentence_answer": "\uc218\ubcf4\ub85c\ud504\uc758 \ubc18\ub300\uc5d0\ub3c4 \ubd88\uad6c\ud558\uace0 \ub7ec\uc2dc\uc544\uc758 \ud669\uc81c \ud30c\ubca8 1\uc138\ub294 \uc218\ubcf4\ub85c\ud504\uc5d0\uac8c \uc2a4\uc704\uc2a4\ub85c \ubcd1\ub825\uc744 \uc774\ub3d9\uc2dc\ucf1c \ubb34\ub2a5\ud55c \uc54c\ub809\uc0b0\ub4dc\ub974 \ucf54\ub974\uc0ac\ucf54\ud504 (Alexander Korsakov)", "paragraph_id": "6518720-1-1", "paragraph_question": "question: \ud30c\ubca8 1\uc138\ub294 \uc218\ubcf4\ub85c\ud504\uac00 \ub204\uad6c\uc758 \uc9c0\ud718\ub97c \ubc1b\uac8c \ud558\uc600\ub294\uac00?, context: \uc77c\ubc18\uc801\uc73c\ub85c \uc704\ub300\ud55c \uc7a5\uad70\ub4e4\uc740 \uc804\ud22c\uc5d0\uc11c\ub294 \uc9c0\uc9c0 \uc54a\ub354\ub77c\ub3c4 \uc815\uce58\uafbc\ub4e4\uc758 \uc74c\ubaa8\ub294 \uc81c\ub300\ub85c \uc0c1\ub300\ud558\uc9c0 \ubabb\ud558\ub294 \ubc95\uc774\ub2e4. \uc624\uc2a4\ud2b8\ub9ac\uc544\uc640 \uc601\uad6d\uc740 \uc774\ud0c8\ub9ac\uc544\uc5d0\uc11c \ub7ec\uc2dc\uc544\uc758 \uc131\uacf5\uc744 \ubcf4\uace0 \ub450\ub824\uc6b4 \ub9c8\uc74c\uc744 \ud488\uc5c8\uace0, \uacb0\uad6d \uc218\ubcf4\ub85c\ud504\uc758 \ud504\ub791\uc2a4 \ubcf8\ud1a0 \uc9c4\uaca9 \uacc4\ud68d\uc744 \uc88c\uc808\uc2dc\ucf30\ub2e4. \ub300\uc2e0 \uc800\uc9c0\ub300 \uc9c0\ubc29\uc758 \uc804\uc5ed\uc5d0 \uc9d1\uc911\ud558\uac8c \ud558\uc600\ub2e4. \uc218\ubcf4\ub85c\ud504\uc758 \ubc18\ub300\uc5d0\ub3c4 \ubd88\uad6c\ud558\uace0 \ub7ec\uc2dc\uc544\uc758 \ud669\uc81c \ud30c\ubca8 1\uc138\ub294 \uc218\ubcf4\ub85c\ud504\uc5d0\uac8c \uc2a4\uc704\uc2a4\ub85c \ubcd1\ub825\uc744 \uc774\ub3d9\uc2dc\ucf1c \ubb34\ub2a5\ud55c \uc54c\ub809\uc0b0\ub4dc\ub974 \ucf54\ub974\uc0ac\ucf54\ud504(Alexander Korsakov)\uc758 \uc9c0\ud718\ub97c \ubc1b\uac8c \ud558\uc600\ub2e4. \uadf8\ub7ec\ub098 \ucf54\ub974\uc0ac\ucf54\ud504\ub294 \uc559\ub4dc\ub808 \ub9c8\uc138\ub098(Andre Massena)\uc5d0\uac8c \uc81c2\ucc28 \ucde8\ub9ac\ud788 \uc804\ud22c(Second Battle of Zurich)\uc5d0\uc11c \ub300\ud328\ub97c \ub2f9\ud588\ub2e4. \uc774 \uc2b9\uc138\ub97c \ubab0\uc544 \ub9c8\uc138\ub098\ub294 \ud718\ud558\uc5d0 \uc788\ub294 80,000\uba85\uc758 \ubcd1\uc0ac\ub97c \uc774\ub04c\uace0 \uc218\ubcf4\ub85c\ud504 \ud718\ud558\uc758 \ub7ec\uc2dc\uc544\uad70 \uc794\uc5ec\ubcd1\uc778 18,000\uba85\uc758 \uc815\uaddc\ubcd1\uacfc 5,000\uba85\uc758 \ucf54\uc0ac\ud06c(Cossacks) \ubd80\ub300\ub97c \uaca9\uba78\ud558\uae30 \uc704\ud574 \uc9c4\uad70\ud558\uc600\ub2e4. \uc218\ubcf4\ub85c\ud504\ub294 \ud1f4\uac01\ud558\uac70\ub098 \uc7a5\ub82c\ud558\uac8c \uc804\uba78\ud558\uac70\ub098 \ud558\ub294 \uc120\ud0dd\ubc16\uc5d0 \uc5c6\uc5c8\ub2e4."} {"question": "\ub2e8\uc21c \uc678\uad50\ubb38\uc81c \ubfd0\ub9cc \uc544\ub2c8\ub77c \ud1b5\uc0c1, \ubb34\uae30\uad6c\uc785 \ub4f1 \uc11c\uc591\uacfc \uad00\ub828\ub41c \uc5c5\ubb34\ub97c \ucd1d\uad04\ud55c \uae30\uad00\uc740?", "paragraph": "1861\ub144 \uccad\uc870\ub294 \uad6c\ubbf8\uc5f4\uac15\uacfc\uc758 \uc678\uad50\ubb38\uc81c\ub97c \uc804\ub2f4\ud560 \ucd1d\ub9ac\uc544\ubb38\uc744 \uc124\uce58\ud558\uc600\ub2e4. \ucd1d\ub9ac\uc544\ubb38\uc740 \ub2e8\uc21c\ud55c \uc678\uad50\ubb38\uc81c\ub9cc\uc744 \ucde8\uae09\ud558\uc9c0 \uc54a\uace0 \ud1b5\uc0c1(\u901a\u5546)\uc744 \ud3ec\ud568\ud558\uc5ec \ubb34\uae30\uc758 \uad6c\uc785\uae4c\uc9c0 \uc11c\uc591\uacfc \uad00\ub828\ub41c \uc5c5\ubb34\ub97c \ucd1d\uad04\ud558\ub294 \uae30\uad00\uc774\ub2e4. \uad70\uae30\ucc98\uc640\uc758 \uad00\uacc4\ub294 \uba85\ubb38\uaddc\uc815\uc740 \uc5c6\uc73c\ub098 \uad70\uae30\ub300\uc2e0\uc774 \ucd1d\ub9ac\uc544\ubb38\uc758 \uc7a5\uad00\uc778 \ucd1d\uc11c\ub300\uc2e0\uc744 \uacb8\ud558\uc5ec \uad70\uae30\ucc98\uc758 \ubd84\uad6d(\u5206\u570b)\uacfc \uac19\uc740 \uc131\uaca9\uc744 \ub760\uace0 \uc788\uc5b4\uc11c \uc2e0\uc815\uad8c\uc758 \uad8c\ub825\ud575\uc2ec\uc5d0 \ud574\ub2f9\ud558\uc600\ub2e4. \uc720\ub7fd \uc5f4\uac15\uc740 \uc774 \uac19\uc740 \ud0c0\ud611\uc801\uc778 \uc11c\ud0dc\ud6c4\uc640 \uacf5\uce5c\uc655 \uc815\uad8c\uc5d0 \ub300\ud558\uc5ec \ud611\uc870\uc815\ucc45\uc744 \ucde8\ud558\uace0 \uccad\uc870\uc640 \ud0dc\ud3c9\ucc9c\uad6d\uad70\uc5d0 \ub300\ud574 \ucde8\ud558\ub358 \uc911\ub9bd\uc801 \uc790\uc138\ub97c \ubc84\ub9ac\uace0 \uccad\uc5d0 \ub300\ud55c \uc801\uadf9\uc801\uc778 \uc6d0\uc870\ub97c \uc81c\uacf5\ud558\uba74\uc11c \uadfc\ub300\uc801 \uc0b0\uc5c5\uae30\uc220\uacfc \uc790\ubcf8\ud611\ub825\uc744 \uc81c\uc548\ud558\uc600\ub2e4. \uc774\uc5d0 \ub530\ub77c \ub300\uc678\uad00\uacc4\ub97c \uc218\uc2b5\ud55c \uccad\uc870\uc758 \uad00\ub8cc\ub4e4\uc740 \uc11c\uad6c\uc758 \uadfc\ub300\uc801 \ubb34\uae30\ub97c \uc774\uc6a9\ud558\uc5ec \ud0dc\ud3c9\ucc9c\uad6d\uad70\uc744 \uc9c4\uc555\ud560 \ubaa9\uc801\uc73c\ub85c \uc601\uad6d\uc73c\ub85c\ubd80\ud130 \uad70\ud568\uc758 \ub3c4\uc785\uacfc \uc601\uad6d\uc2dd \uad70\uc0ac\ud6c8\ub828\uc744 \uacc4\ud68d\ud558\uc600\uace0, \uad70\uae30\ub300\uc2e0 \ubb38\uc0c1\uc740 \ub7ec\uc2dc\uc544\ub85c\ubd80\ud130 \uc81c\uacf5\ubc1b\uc740 \uadfc\ub300\uc801 \ubb34\uae30\ub85c \ubb34\uc7a5\ud55c '\uc2e0\uae30\uc601'(\u795e\u6a5f\u71df)\uc744 \ucc3d\uc124\ud558\uc600\ub2e4.", "answer": "\ucd1d\ub9ac\uc544\ubb38", "sentence": "1861\ub144 \uccad\uc870\ub294 \uad6c\ubbf8\uc5f4\uac15\uacfc\uc758 \uc678\uad50\ubb38\uc81c\ub97c \uc804\ub2f4\ud560 \ucd1d\ub9ac\uc544\ubb38 \uc744 \uc124\uce58\ud558\uc600\ub2e4.", "paragraph_sentence": " 1861\ub144 \uccad\uc870\ub294 \uad6c\ubbf8\uc5f4\uac15\uacfc\uc758 \uc678\uad50\ubb38\uc81c\ub97c \uc804\ub2f4\ud560 \ucd1d\ub9ac\uc544\ubb38 \uc744 \uc124\uce58\ud558\uc600\ub2e4. \ucd1d\ub9ac\uc544\ubb38\uc740 \ub2e8\uc21c\ud55c \uc678\uad50\ubb38\uc81c\ub9cc\uc744 \ucde8\uae09\ud558\uc9c0 \uc54a\uace0 \ud1b5\uc0c1(\u901a\u5546)\uc744 \ud3ec\ud568\ud558\uc5ec \ubb34\uae30\uc758 \uad6c\uc785\uae4c\uc9c0 \uc11c\uc591\uacfc \uad00\ub828\ub41c \uc5c5\ubb34\ub97c \ucd1d\uad04\ud558\ub294 \uae30\uad00\uc774\ub2e4. \uad70\uae30\ucc98\uc640\uc758 \uad00\uacc4\ub294 \uba85\ubb38\uaddc\uc815\uc740 \uc5c6\uc73c\ub098 \uad70\uae30\ub300\uc2e0\uc774 \ucd1d\ub9ac\uc544\ubb38\uc758 \uc7a5\uad00\uc778 \ucd1d\uc11c\ub300\uc2e0\uc744 \uacb8\ud558\uc5ec \uad70\uae30\ucc98\uc758 \ubd84\uad6d(\u5206\u570b)\uacfc \uac19\uc740 \uc131\uaca9\uc744 \ub760\uace0 \uc788\uc5b4\uc11c \uc2e0\uc815\uad8c\uc758 \uad8c\ub825\ud575\uc2ec\uc5d0 \ud574\ub2f9\ud558\uc600\ub2e4. \uc720\ub7fd \uc5f4\uac15\uc740 \uc774 \uac19\uc740 \ud0c0\ud611\uc801\uc778 \uc11c\ud0dc\ud6c4\uc640 \uacf5\uce5c\uc655 \uc815\uad8c\uc5d0 \ub300\ud558\uc5ec \ud611\uc870\uc815\ucc45\uc744 \ucde8\ud558\uace0 \uccad\uc870\uc640 \ud0dc\ud3c9\ucc9c\uad6d\uad70\uc5d0 \ub300\ud574 \ucde8\ud558\ub358 \uc911\ub9bd\uc801 \uc790\uc138\ub97c \ubc84\ub9ac\uace0 \uccad\uc5d0 \ub300\ud55c \uc801\uadf9\uc801\uc778 \uc6d0\uc870\ub97c \uc81c\uacf5\ud558\uba74\uc11c \uadfc\ub300\uc801 \uc0b0\uc5c5\uae30\uc220\uacfc \uc790\ubcf8\ud611\ub825\uc744 \uc81c\uc548\ud558\uc600\ub2e4. \uc774\uc5d0 \ub530\ub77c \ub300\uc678\uad00\uacc4\ub97c \uc218\uc2b5\ud55c \uccad\uc870\uc758 \uad00\ub8cc\ub4e4\uc740 \uc11c\uad6c\uc758 \uadfc\ub300\uc801 \ubb34\uae30\ub97c \uc774\uc6a9\ud558\uc5ec \ud0dc\ud3c9\ucc9c\uad6d\uad70\uc744 \uc9c4\uc555\ud560 \ubaa9\uc801\uc73c\ub85c \uc601\uad6d\uc73c\ub85c\ubd80\ud130 \uad70\ud568\uc758 \ub3c4\uc785\uacfc \uc601\uad6d\uc2dd \uad70\uc0ac\ud6c8\ub828\uc744 \uacc4\ud68d\ud558\uc600\uace0, \uad70\uae30\ub300\uc2e0 \ubb38\uc0c1\uc740 \ub7ec\uc2dc\uc544\ub85c\ubd80\ud130 \uc81c\uacf5\ubc1b\uc740 \uadfc\ub300\uc801 \ubb34\uae30\ub85c \ubb34\uc7a5\ud55c '\uc2e0\uae30\uc601'(\u795e\u6a5f\u71df)\uc744 \ucc3d\uc124\ud558\uc600\ub2e4.", "paragraph_answer": "1861\ub144 \uccad\uc870\ub294 \uad6c\ubbf8\uc5f4\uac15\uacfc\uc758 \uc678\uad50\ubb38\uc81c\ub97c \uc804\ub2f4\ud560 \ucd1d\ub9ac\uc544\ubb38 \uc744 \uc124\uce58\ud558\uc600\ub2e4. \ucd1d\ub9ac\uc544\ubb38\uc740 \ub2e8\uc21c\ud55c \uc678\uad50\ubb38\uc81c\ub9cc\uc744 \ucde8\uae09\ud558\uc9c0 \uc54a\uace0 \ud1b5\uc0c1(\u901a\u5546)\uc744 \ud3ec\ud568\ud558\uc5ec \ubb34\uae30\uc758 \uad6c\uc785\uae4c\uc9c0 \uc11c\uc591\uacfc \uad00\ub828\ub41c \uc5c5\ubb34\ub97c \ucd1d\uad04\ud558\ub294 \uae30\uad00\uc774\ub2e4. \uad70\uae30\ucc98\uc640\uc758 \uad00\uacc4\ub294 \uba85\ubb38\uaddc\uc815\uc740 \uc5c6\uc73c\ub098 \uad70\uae30\ub300\uc2e0\uc774 \ucd1d\ub9ac\uc544\ubb38\uc758 \uc7a5\uad00\uc778 \ucd1d\uc11c\ub300\uc2e0\uc744 \uacb8\ud558\uc5ec \uad70\uae30\ucc98\uc758 \ubd84\uad6d(\u5206\u570b)\uacfc \uac19\uc740 \uc131\uaca9\uc744 \ub760\uace0 \uc788\uc5b4\uc11c \uc2e0\uc815\uad8c\uc758 \uad8c\ub825\ud575\uc2ec\uc5d0 \ud574\ub2f9\ud558\uc600\ub2e4. \uc720\ub7fd \uc5f4\uac15\uc740 \uc774 \uac19\uc740 \ud0c0\ud611\uc801\uc778 \uc11c\ud0dc\ud6c4\uc640 \uacf5\uce5c\uc655 \uc815\uad8c\uc5d0 \ub300\ud558\uc5ec \ud611\uc870\uc815\ucc45\uc744 \ucde8\ud558\uace0 \uccad\uc870\uc640 \ud0dc\ud3c9\ucc9c\uad6d\uad70\uc5d0 \ub300\ud574 \ucde8\ud558\ub358 \uc911\ub9bd\uc801 \uc790\uc138\ub97c \ubc84\ub9ac\uace0 \uccad\uc5d0 \ub300\ud55c \uc801\uadf9\uc801\uc778 \uc6d0\uc870\ub97c \uc81c\uacf5\ud558\uba74\uc11c \uadfc\ub300\uc801 \uc0b0\uc5c5\uae30\uc220\uacfc \uc790\ubcf8\ud611\ub825\uc744 \uc81c\uc548\ud558\uc600\ub2e4. \uc774\uc5d0 \ub530\ub77c \ub300\uc678\uad00\uacc4\ub97c \uc218\uc2b5\ud55c \uccad\uc870\uc758 \uad00\ub8cc\ub4e4\uc740 \uc11c\uad6c\uc758 \uadfc\ub300\uc801 \ubb34\uae30\ub97c \uc774\uc6a9\ud558\uc5ec \ud0dc\ud3c9\ucc9c\uad6d\uad70\uc744 \uc9c4\uc555\ud560 \ubaa9\uc801\uc73c\ub85c \uc601\uad6d\uc73c\ub85c\ubd80\ud130 \uad70\ud568\uc758 \ub3c4\uc785\uacfc \uc601\uad6d\uc2dd \uad70\uc0ac\ud6c8\ub828\uc744 \uacc4\ud68d\ud558\uc600\uace0, \uad70\uae30\ub300\uc2e0 \ubb38\uc0c1\uc740 \ub7ec\uc2dc\uc544\ub85c\ubd80\ud130 \uc81c\uacf5\ubc1b\uc740 \uadfc\ub300\uc801 \ubb34\uae30\ub85c \ubb34\uc7a5\ud55c '\uc2e0\uae30\uc601'(\u795e\u6a5f\u71df)\uc744 \ucc3d\uc124\ud558\uc600\ub2e4.", "sentence_answer": "1861\ub144 \uccad\uc870\ub294 \uad6c\ubbf8\uc5f4\uac15\uacfc\uc758 \uc678\uad50\ubb38\uc81c\ub97c \uc804\ub2f4\ud560 \ucd1d\ub9ac\uc544\ubb38 \uc744 \uc124\uce58\ud558\uc600\ub2e4.", "paragraph_id": "6522804-0-1", "paragraph_question": "question: \ub2e8\uc21c \uc678\uad50\ubb38\uc81c \ubfd0\ub9cc \uc544\ub2c8\ub77c \ud1b5\uc0c1, \ubb34\uae30\uad6c\uc785 \ub4f1 \uc11c\uc591\uacfc \uad00\ub828\ub41c \uc5c5\ubb34\ub97c \ucd1d\uad04\ud55c \uae30\uad00\uc740?, context: 1861\ub144 \uccad\uc870\ub294 \uad6c\ubbf8\uc5f4\uac15\uacfc\uc758 \uc678\uad50\ubb38\uc81c\ub97c \uc804\ub2f4\ud560 \ucd1d\ub9ac\uc544\ubb38\uc744 \uc124\uce58\ud558\uc600\ub2e4. \ucd1d\ub9ac\uc544\ubb38\uc740 \ub2e8\uc21c\ud55c \uc678\uad50\ubb38\uc81c\ub9cc\uc744 \ucde8\uae09\ud558\uc9c0 \uc54a\uace0 \ud1b5\uc0c1(\u901a\u5546)\uc744 \ud3ec\ud568\ud558\uc5ec \ubb34\uae30\uc758 \uad6c\uc785\uae4c\uc9c0 \uc11c\uc591\uacfc \uad00\ub828\ub41c \uc5c5\ubb34\ub97c \ucd1d\uad04\ud558\ub294 \uae30\uad00\uc774\ub2e4. \uad70\uae30\ucc98\uc640\uc758 \uad00\uacc4\ub294 \uba85\ubb38\uaddc\uc815\uc740 \uc5c6\uc73c\ub098 \uad70\uae30\ub300\uc2e0\uc774 \ucd1d\ub9ac\uc544\ubb38\uc758 \uc7a5\uad00\uc778 \ucd1d\uc11c\ub300\uc2e0\uc744 \uacb8\ud558\uc5ec \uad70\uae30\ucc98\uc758 \ubd84\uad6d(\u5206\u570b)\uacfc \uac19\uc740 \uc131\uaca9\uc744 \ub760\uace0 \uc788\uc5b4\uc11c \uc2e0\uc815\uad8c\uc758 \uad8c\ub825\ud575\uc2ec\uc5d0 \ud574\ub2f9\ud558\uc600\ub2e4. \uc720\ub7fd \uc5f4\uac15\uc740 \uc774 \uac19\uc740 \ud0c0\ud611\uc801\uc778 \uc11c\ud0dc\ud6c4\uc640 \uacf5\uce5c\uc655 \uc815\uad8c\uc5d0 \ub300\ud558\uc5ec \ud611\uc870\uc815\ucc45\uc744 \ucde8\ud558\uace0 \uccad\uc870\uc640 \ud0dc\ud3c9\ucc9c\uad6d\uad70\uc5d0 \ub300\ud574 \ucde8\ud558\ub358 \uc911\ub9bd\uc801 \uc790\uc138\ub97c \ubc84\ub9ac\uace0 \uccad\uc5d0 \ub300\ud55c \uc801\uadf9\uc801\uc778 \uc6d0\uc870\ub97c \uc81c\uacf5\ud558\uba74\uc11c \uadfc\ub300\uc801 \uc0b0\uc5c5\uae30\uc220\uacfc \uc790\ubcf8\ud611\ub825\uc744 \uc81c\uc548\ud558\uc600\ub2e4. \uc774\uc5d0 \ub530\ub77c \ub300\uc678\uad00\uacc4\ub97c \uc218\uc2b5\ud55c \uccad\uc870\uc758 \uad00\ub8cc\ub4e4\uc740 \uc11c\uad6c\uc758 \uadfc\ub300\uc801 \ubb34\uae30\ub97c \uc774\uc6a9\ud558\uc5ec \ud0dc\ud3c9\ucc9c\uad6d\uad70\uc744 \uc9c4\uc555\ud560 \ubaa9\uc801\uc73c\ub85c \uc601\uad6d\uc73c\ub85c\ubd80\ud130 \uad70\ud568\uc758 \ub3c4\uc785\uacfc \uc601\uad6d\uc2dd \uad70\uc0ac\ud6c8\ub828\uc744 \uacc4\ud68d\ud558\uc600\uace0, \uad70\uae30\ub300\uc2e0 \ubb38\uc0c1\uc740 \ub7ec\uc2dc\uc544\ub85c\ubd80\ud130 \uc81c\uacf5\ubc1b\uc740 \uadfc\ub300\uc801 \ubb34\uae30\ub85c \ubb34\uc7a5\ud55c '\uc2e0\uae30\uc601'(\u795e\u6a5f\u71df)\uc744 \ucc3d\uc124\ud558\uc600\ub2e4."} {"question": "\uc870\uc9c0\uc544\uc758 \uc218\ub3c4\ub294?", "paragraph": "\uc870\uc9c0\uc544(\uc870\uc9c0\uc544\uc5b4: \u10e1\u10d0\u10e5\u10d0\u10e0\u10d7\u10d5\u10d4\u10da\u10dd \uc0ac\uce74\ub974\ud2b8\ubca8\ub85c [s\u0251k\u02b0\u0251rt\u02b0v\u025bl\u0254] (\ub3c4\uc6c0\ub9d0\u00b7\uc815\ubcf4), \uc601\uc5b4: Georgia [\u02c8d\u0292\u0254rd\u0292\u0259] (\ub3c4\uc6c0\ub9d0\u00b7\uc815\ubcf4), \ubb38\ud654\uc5b4: \uadf8\ub8e8\uc9c0\uc57c)\ub294 \ub3d9\uc720\ub7fd\uc758 \uad6d\uac00\uc774\ub2e4. \uba74\uc801\uc740 7\ub9cc 7000 km, \uc778\uad6c\ub294 463\ub9cc 841 \uba85(2008\ub144 \ucd94\uacc4)\uc774\ub2e4. \ubd81\ucabd\uc740 \ub7ec\uc2dc\uc544, \ub0a8\ucabd\uc740 \ud130\ud0a4\uc640 \uc544\uc81c\ub974\ubc14\uc774\uc794, \ub0a8\ub3d9\ucabd\uc740 \uc544\ub974\uba54\ub2c8\uc544\uc640 \uad6d\uacbd\uc744 \uc811\ud558\uace0 \uc788\ub2e4. \uc218\ub3c4\ub294 \ud2b8\ube4c\ub9ac\uc2dc\uc774\ub2e4. 1936\ub144 \uc18c\ube44\uc5d0\ud2b8 \uc5f0\ubc29\uc744 \uad6c\uc131\ud558\ub358 \uacf5\ud654\uad6d\uc758 \ud558\ub098\uc778 \uadf8\ub8e8\uc9c0\uc57c \uc18c\ube44\uc5d0\ud2b8 \uc0ac\ud68c\uc8fc\uc758 \uacf5\ud654\uad6d\uc744 \uc774\ub8e8\uace0 \uc788\ub2e4\uac00, 1991\ub144 4\uc6d4 9\uc77c \ub3c5\ub9bd\ud558\uc600\ub2e4. \ud604\uc7ac \uc870\uc9c0\uc544\uc758 \uc601\uc5ed \uc548\uc5d0\ub294 \uce5c\ub7ec \uc131\ud5a5\uc73c\ub85c \uc774 \ub098\ub77c\uc5d0\uc11c \ub3c5\ub9bd\ud558\ub824\uace0 \ud558\ub294 \uc555\ud558\uc2a4\uc640 \ub0a8\uc624\uc138\ud2f0\uc57c\uac00 \ud3ec\ud568\ub418\uc5b4 \uc788\uc73c\uba70, \uc774 \uc9c0\uc5ed\ub4e4\uc740 \ub7ec\uc2dc\uc544\ub85c\ubd80\ud130 \ub3c5\ub9bd\uc744 \uc2b9\uc778\ubc1b\uc558\uc744 \ubfd0 \ub300\ub2e4\uc218 \uad6d\uac00\ub4e4\ub85c\ubd80\ud130 \ub3c5\ub9bd \uad6d\uac00\ub85c \uc778\uc815\ubc1b\uc9c0 \ubabb\ud558\uace0 \uc788\ub2e4. \uc18c\ube44\uc5d0\ud2b8 \uc5f0\ubc29\uc758 \uacf5\uc0b0\ub2f9 \uc11c\uae30\uc7a5\uc73c\ub85c 1952\ub144\uae4c\uc9c0 30\ub144\uac04 \uad6d\uac00 \uc6d0\uc218\uc600\ub358 \uc2a4\ud0c8\ub9b0(\ubcf8\uba85: \uc774\uc624\uc138\ube0c \uc8fc\uac00\uc288\ube4c\ub9ac)\uc774 \uc870\uc9c0\uc544 \ucd9c\uc2e0\uc774\ub2e4.", "answer": "\ud2b8\ube4c\ub9ac\uc2dc", "sentence": "\uc218\ub3c4\ub294 \ud2b8\ube4c\ub9ac\uc2dc \uc774\ub2e4.", "paragraph_sentence": "\uc870\uc9c0\uc544(\uc870\uc9c0\uc544\uc5b4: \u10e1\u10d0\u10e5\u10d0\u10e0\u10d7\u10d5\u10d4\u10da\u10dd \uc0ac\uce74\ub974\ud2b8\ubca8\ub85c [s\u0251k\u02b0\u0251rt\u02b0v\u025bl\u0254] (\ub3c4\uc6c0\ub9d0\u00b7\uc815\ubcf4), \uc601\uc5b4: Georgia [\u02c8d\u0292\u0254rd\u0292\u0259] (\ub3c4\uc6c0\ub9d0\u00b7\uc815\ubcf4), \ubb38\ud654\uc5b4: \uadf8\ub8e8\uc9c0\uc57c)\ub294 \ub3d9\uc720\ub7fd\uc758 \uad6d\uac00\uc774\ub2e4. \uba74\uc801\uc740 7\ub9cc 7000 km, \uc778\uad6c\ub294 463\ub9cc 841 \uba85(2008\ub144 \ucd94\uacc4)\uc774\ub2e4. \ubd81\ucabd\uc740 \ub7ec\uc2dc\uc544, \ub0a8\ucabd\uc740 \ud130\ud0a4\uc640 \uc544\uc81c\ub974\ubc14\uc774\uc794, \ub0a8\ub3d9\ucabd\uc740 \uc544\ub974\uba54\ub2c8\uc544\uc640 \uad6d\uacbd\uc744 \uc811\ud558\uace0 \uc788\ub2e4. \uc218\ub3c4\ub294 \ud2b8\ube4c\ub9ac\uc2dc \uc774\ub2e4. 1936\ub144 \uc18c\ube44\uc5d0\ud2b8 \uc5f0\ubc29\uc744 \uad6c\uc131\ud558\ub358 \uacf5\ud654\uad6d\uc758 \ud558\ub098\uc778 \uadf8\ub8e8\uc9c0\uc57c \uc18c\ube44\uc5d0\ud2b8 \uc0ac\ud68c\uc8fc\uc758 \uacf5\ud654\uad6d\uc744 \uc774\ub8e8\uace0 \uc788\ub2e4\uac00, 1991\ub144 4\uc6d4 9\uc77c \ub3c5\ub9bd\ud558\uc600\ub2e4. \ud604\uc7ac \uc870\uc9c0\uc544\uc758 \uc601\uc5ed \uc548\uc5d0\ub294 \uce5c\ub7ec \uc131\ud5a5\uc73c\ub85c \uc774 \ub098\ub77c\uc5d0\uc11c \ub3c5\ub9bd\ud558\ub824\uace0 \ud558\ub294 \uc555\ud558\uc2a4\uc640 \ub0a8\uc624\uc138\ud2f0\uc57c\uac00 \ud3ec\ud568\ub418\uc5b4 \uc788\uc73c\uba70, \uc774 \uc9c0\uc5ed\ub4e4\uc740 \ub7ec\uc2dc\uc544\ub85c\ubd80\ud130 \ub3c5\ub9bd\uc744 \uc2b9\uc778\ubc1b\uc558\uc744 \ubfd0 \ub300\ub2e4\uc218 \uad6d\uac00\ub4e4\ub85c\ubd80\ud130 \ub3c5\ub9bd \uad6d\uac00\ub85c \uc778\uc815\ubc1b\uc9c0 \ubabb\ud558\uace0 \uc788\ub2e4. \uc18c\ube44\uc5d0\ud2b8 \uc5f0\ubc29\uc758 \uacf5\uc0b0\ub2f9 \uc11c\uae30\uc7a5\uc73c\ub85c 1952\ub144\uae4c\uc9c0 30\ub144\uac04 \uad6d\uac00 \uc6d0\uc218\uc600\ub358 \uc2a4\ud0c8\ub9b0(\ubcf8\uba85: \uc774\uc624\uc138\ube0c \uc8fc\uac00\uc288\ube4c\ub9ac)\uc774 \uc870\uc9c0\uc544 \ucd9c\uc2e0\uc774\ub2e4.", "paragraph_answer": "\uc870\uc9c0\uc544(\uc870\uc9c0\uc544\uc5b4: \u10e1\u10d0\u10e5\u10d0\u10e0\u10d7\u10d5\u10d4\u10da\u10dd \uc0ac\uce74\ub974\ud2b8\ubca8\ub85c [s\u0251k\u02b0\u0251rt\u02b0v\u025bl\u0254] (\ub3c4\uc6c0\ub9d0\u00b7\uc815\ubcf4), \uc601\uc5b4: Georgia [\u02c8d\u0292\u0254rd\u0292\u0259] (\ub3c4\uc6c0\ub9d0\u00b7\uc815\ubcf4), \ubb38\ud654\uc5b4: \uadf8\ub8e8\uc9c0\uc57c)\ub294 \ub3d9\uc720\ub7fd\uc758 \uad6d\uac00\uc774\ub2e4. \uba74\uc801\uc740 7\ub9cc 7000 km, \uc778\uad6c\ub294 463\ub9cc 841 \uba85(2008\ub144 \ucd94\uacc4)\uc774\ub2e4. \ubd81\ucabd\uc740 \ub7ec\uc2dc\uc544, \ub0a8\ucabd\uc740 \ud130\ud0a4\uc640 \uc544\uc81c\ub974\ubc14\uc774\uc794, \ub0a8\ub3d9\ucabd\uc740 \uc544\ub974\uba54\ub2c8\uc544\uc640 \uad6d\uacbd\uc744 \uc811\ud558\uace0 \uc788\ub2e4. \uc218\ub3c4\ub294 \ud2b8\ube4c\ub9ac\uc2dc \uc774\ub2e4. 1936\ub144 \uc18c\ube44\uc5d0\ud2b8 \uc5f0\ubc29\uc744 \uad6c\uc131\ud558\ub358 \uacf5\ud654\uad6d\uc758 \ud558\ub098\uc778 \uadf8\ub8e8\uc9c0\uc57c \uc18c\ube44\uc5d0\ud2b8 \uc0ac\ud68c\uc8fc\uc758 \uacf5\ud654\uad6d\uc744 \uc774\ub8e8\uace0 \uc788\ub2e4\uac00, 1991\ub144 4\uc6d4 9\uc77c \ub3c5\ub9bd\ud558\uc600\ub2e4. \ud604\uc7ac \uc870\uc9c0\uc544\uc758 \uc601\uc5ed \uc548\uc5d0\ub294 \uce5c\ub7ec \uc131\ud5a5\uc73c\ub85c \uc774 \ub098\ub77c\uc5d0\uc11c \ub3c5\ub9bd\ud558\ub824\uace0 \ud558\ub294 \uc555\ud558\uc2a4\uc640 \ub0a8\uc624\uc138\ud2f0\uc57c\uac00 \ud3ec\ud568\ub418\uc5b4 \uc788\uc73c\uba70, \uc774 \uc9c0\uc5ed\ub4e4\uc740 \ub7ec\uc2dc\uc544\ub85c\ubd80\ud130 \ub3c5\ub9bd\uc744 \uc2b9\uc778\ubc1b\uc558\uc744 \ubfd0 \ub300\ub2e4\uc218 \uad6d\uac00\ub4e4\ub85c\ubd80\ud130 \ub3c5\ub9bd \uad6d\uac00\ub85c \uc778\uc815\ubc1b\uc9c0 \ubabb\ud558\uace0 \uc788\ub2e4. \uc18c\ube44\uc5d0\ud2b8 \uc5f0\ubc29\uc758 \uacf5\uc0b0\ub2f9 \uc11c\uae30\uc7a5\uc73c\ub85c 1952\ub144\uae4c\uc9c0 30\ub144\uac04 \uad6d\uac00 \uc6d0\uc218\uc600\ub358 \uc2a4\ud0c8\ub9b0(\ubcf8\uba85: \uc774\uc624\uc138\ube0c \uc8fc\uac00\uc288\ube4c\ub9ac)\uc774 \uc870\uc9c0\uc544 \ucd9c\uc2e0\uc774\ub2e4.", "sentence_answer": "\uc218\ub3c4\ub294 \ud2b8\ube4c\ub9ac\uc2dc \uc774\ub2e4.", "paragraph_id": "5885698-0-1", "paragraph_question": "question: \uc870\uc9c0\uc544\uc758 \uc218\ub3c4\ub294?, context: \uc870\uc9c0\uc544(\uc870\uc9c0\uc544\uc5b4: \u10e1\u10d0\u10e5\u10d0\u10e0\u10d7\u10d5\u10d4\u10da\u10dd \uc0ac\uce74\ub974\ud2b8\ubca8\ub85c [s\u0251k\u02b0\u0251rt\u02b0v\u025bl\u0254] (\ub3c4\uc6c0\ub9d0\u00b7\uc815\ubcf4), \uc601\uc5b4: Georgia [\u02c8d\u0292\u0254rd\u0292\u0259] (\ub3c4\uc6c0\ub9d0\u00b7\uc815\ubcf4), \ubb38\ud654\uc5b4: \uadf8\ub8e8\uc9c0\uc57c)\ub294 \ub3d9\uc720\ub7fd\uc758 \uad6d\uac00\uc774\ub2e4. \uba74\uc801\uc740 7\ub9cc 7000 km, \uc778\uad6c\ub294 463\ub9cc 841 \uba85(2008\ub144 \ucd94\uacc4)\uc774\ub2e4. \ubd81\ucabd\uc740 \ub7ec\uc2dc\uc544, \ub0a8\ucabd\uc740 \ud130\ud0a4\uc640 \uc544\uc81c\ub974\ubc14\uc774\uc794, \ub0a8\ub3d9\ucabd\uc740 \uc544\ub974\uba54\ub2c8\uc544\uc640 \uad6d\uacbd\uc744 \uc811\ud558\uace0 \uc788\ub2e4. \uc218\ub3c4\ub294 \ud2b8\ube4c\ub9ac\uc2dc\uc774\ub2e4. 1936\ub144 \uc18c\ube44\uc5d0\ud2b8 \uc5f0\ubc29\uc744 \uad6c\uc131\ud558\ub358 \uacf5\ud654\uad6d\uc758 \ud558\ub098\uc778 \uadf8\ub8e8\uc9c0\uc57c \uc18c\ube44\uc5d0\ud2b8 \uc0ac\ud68c\uc8fc\uc758 \uacf5\ud654\uad6d\uc744 \uc774\ub8e8\uace0 \uc788\ub2e4\uac00, 1991\ub144 4\uc6d4 9\uc77c \ub3c5\ub9bd\ud558\uc600\ub2e4. \ud604\uc7ac \uc870\uc9c0\uc544\uc758 \uc601\uc5ed \uc548\uc5d0\ub294 \uce5c\ub7ec \uc131\ud5a5\uc73c\ub85c \uc774 \ub098\ub77c\uc5d0\uc11c \ub3c5\ub9bd\ud558\ub824\uace0 \ud558\ub294 \uc555\ud558\uc2a4\uc640 \ub0a8\uc624\uc138\ud2f0\uc57c\uac00 \ud3ec\ud568\ub418\uc5b4 \uc788\uc73c\uba70, \uc774 \uc9c0\uc5ed\ub4e4\uc740 \ub7ec\uc2dc\uc544\ub85c\ubd80\ud130 \ub3c5\ub9bd\uc744 \uc2b9\uc778\ubc1b\uc558\uc744 \ubfd0 \ub300\ub2e4\uc218 \uad6d\uac00\ub4e4\ub85c\ubd80\ud130 \ub3c5\ub9bd \uad6d\uac00\ub85c \uc778\uc815\ubc1b\uc9c0 \ubabb\ud558\uace0 \uc788\ub2e4. \uc18c\ube44\uc5d0\ud2b8 \uc5f0\ubc29\uc758 \uacf5\uc0b0\ub2f9 \uc11c\uae30\uc7a5\uc73c\ub85c 1952\ub144\uae4c\uc9c0 30\ub144\uac04 \uad6d\uac00 \uc6d0\uc218\uc600\ub358 \uc2a4\ud0c8\ub9b0(\ubcf8\uba85: \uc774\uc624\uc138\ube0c \uc8fc\uac00\uc288\ube4c\ub9ac)\uc774 \uc870\uc9c0\uc544 \ucd9c\uc2e0\uc774\ub2e4."} {"question": "\ubca0\ud2b8\ub0a8\uc5d0\uc11c \ubb18\ud638 \uc81c\ub3c4\uac00 \ubcf8\uaca9\uc801\uc73c\ub85c \uc815\ucc29\ub41c \uc655\uc870\ub294?", "paragraph": "\ubca0\ud2b8\ub0a8\ub3c4 \uc678\uc655\ub0b4\uc81c\ub97c \uc9c0\ud5a5\ud558\uc5ec \uc548\uc73c\ub85c\ub294 \ud669\uc81c\ub97c \uc790\uce6d\ud558\uc600\uace0, \uc7ac\uc704\ud55c \uad70\uc8fc\ub4e4\uc5d0\uac8c \ubb18\ud638\ub97c \ubd99\uc600\ub2e4. \ubca0\ud2b8\ub0a8\uc5d0\uc11c \uac00\uc7a5 \uba3c\uc800 \ubb18\ud638\ub97c \uc4f0\uae30 \uc2dc\uc791\ud55c \uc655\uc870\ub294 \uc804 \ub808 \uc655\uc870\uc600\uc73c\ub098, \uc2dc\ud638\ub97c \uc62c\ub9ac\uc9c0 \uc54a\uace0 \ubb18\uc81c \ub610\ud55c \uc815\ub9bd\ub418\uc9c0 \uc54a\uc544, \uc81c\ub300\ub85c \uc808\ucc28\ub97c \ubc1f\uc544 \uc62c\ub824\uc9c4 \ubb18\ud638\uac00 \uc544\ub2c8\uc5c8\ub2e4. \uc774\ud6c4 \ubcf8\uaca9\uc801\uc73c\ub85c \ubb18\ud638 \uc81c\ub3c4\uac00 \uc815\ucc29\ub41c \uc655\uc870\ub294 12\uc138\uae30 \uc911\uc5fd\uc5d0 \uc138\uc6cc\uc9c4 \ub9ac \uc655\uc870\ub85c, \uc911\uad6d\uc758 \ub0a8\uc1a1\uc5d0 \uc758\ud574 \uc774\uc804\uc758 \uad50\uc9c0\uad70\uc655(\u4ea4\u8dbe\u90e1\u738b)\uc5d0\uc11c \uc548\ub0a8\uad6d\uc655(\u5b89\u5357\u570b\u738b)\uc73c\ub85c \uaca9\uc0c1\ub418\uc5b4 \ucc45\ubd09\ub418\uba70, \uc678\uc655\ub0b4\uc81c \uccb4\uc81c\ub97c \ud655\ub9bd\ud568\uacfc \ub3d9\uc2dc\uc5d0 \uad70\uc8fc\ub4e4\uc5d0\uac8c \ubb18\ud638\ub97c \ubd99\uc600\ub2e4. \uc911\uac04\uc5d0 \uba85\ub098\ub77c\uac00 \ubca0\ud2b8\ub0a8\uc744 \uce68\ub7b5\ud558\uc5ec \uc548\ub0a8\uad6d \ub300\uc2e0 \uad50\uc9c0\uad6d\uc73c\ub85c \uadf8 \uaca9\uc744 \ub0ae\ucd94\uace0, \uc5ed\uc2dc \ubb18\ud638\uc640 \uc2dc\ud638\ub97c \uc4f0\ub294 \ub4f1\uc758 \ud48d\uc2b5\uc744 \uae08\ud558\uc600\uc73c\ub098, 20\ub144\ub9cc\uc5d0 \ub808 \uc655\uc870\uac00 \ub4e4\uc5b4\uc11c\uba70 \ub2e4\uc2dc \ubb18\uc81c\ub97c \ubd80\ud65c\uc2dc\ucf30\ub2e4. \ubca0\ud2b8\ub0a8\uc778\ub4e4\uc740 \ub9c8\uc9c0\ub9c9 \uc655\uc870\uc778 \uc751\uc6b0\uc60c \uc655\uc870\uae4c\uc9c0 \uc81c\ub3c4\ub97c \uc720\uc9c0\ud558\uc5ec \uad70\uc8fc\ub4e4\uc5d0\uac8c \ubaa8\ub450 \ubb18\ud638\uc640 \uc2dc\ud638\ub97c \uc62c\ub838\ub2e4.", "answer": "\ub9ac \uc655\uc870", "sentence": "\uc774\ud6c4 \ubcf8\uaca9\uc801\uc73c\ub85c \ubb18\ud638 \uc81c\ub3c4\uac00 \uc815\ucc29\ub41c \uc655\uc870\ub294 12\uc138\uae30 \uc911\uc5fd\uc5d0 \uc138\uc6cc\uc9c4 \ub9ac \uc655\uc870 \ub85c, \uc911\uad6d\uc758 \ub0a8\uc1a1\uc5d0 \uc758\ud574 \uc774\uc804\uc758 \uad50\uc9c0\uad70\uc655(\u4ea4\u8dbe\u90e1\u738b)\uc5d0\uc11c \uc548\ub0a8\uad6d\uc655(\u5b89\u5357\u570b\u738b)\uc73c\ub85c \uaca9\uc0c1\ub418\uc5b4 \ucc45\ubd09\ub418\uba70, \uc678\uc655\ub0b4\uc81c \uccb4\uc81c\ub97c \ud655\ub9bd\ud568\uacfc \ub3d9\uc2dc\uc5d0 \uad70\uc8fc\ub4e4\uc5d0\uac8c \ubb18\ud638\ub97c \ubd99\uc600\ub2e4.", "paragraph_sentence": "\ubca0\ud2b8\ub0a8\ub3c4 \uc678\uc655\ub0b4\uc81c\ub97c \uc9c0\ud5a5\ud558\uc5ec \uc548\uc73c\ub85c\ub294 \ud669\uc81c\ub97c \uc790\uce6d\ud558\uc600\uace0, \uc7ac\uc704\ud55c \uad70\uc8fc\ub4e4\uc5d0\uac8c \ubb18\ud638\ub97c \ubd99\uc600\ub2e4. \ubca0\ud2b8\ub0a8\uc5d0\uc11c \uac00\uc7a5 \uba3c\uc800 \ubb18\ud638\ub97c \uc4f0\uae30 \uc2dc\uc791\ud55c \uc655\uc870\ub294 \uc804 \ub808 \uc655\uc870\uc600\uc73c\ub098, \uc2dc\ud638\ub97c \uc62c\ub9ac\uc9c0 \uc54a\uace0 \ubb18\uc81c \ub610\ud55c \uc815\ub9bd\ub418\uc9c0 \uc54a\uc544, \uc81c\ub300\ub85c \uc808\ucc28\ub97c \ubc1f\uc544 \uc62c\ub824\uc9c4 \ubb18\ud638\uac00 \uc544\ub2c8\uc5c8\ub2e4. \uc774\ud6c4 \ubcf8\uaca9\uc801\uc73c\ub85c \ubb18\ud638 \uc81c\ub3c4\uac00 \uc815\ucc29\ub41c \uc655\uc870\ub294 12\uc138\uae30 \uc911\uc5fd\uc5d0 \uc138\uc6cc\uc9c4 \ub9ac \uc655\uc870 \ub85c, \uc911\uad6d\uc758 \ub0a8\uc1a1\uc5d0 \uc758\ud574 \uc774\uc804\uc758 \uad50\uc9c0\uad70\uc655(\u4ea4\u8dbe\u90e1\u738b)\uc5d0\uc11c \uc548\ub0a8\uad6d\uc655(\u5b89\u5357\u570b\u738b)\uc73c\ub85c \uaca9\uc0c1\ub418\uc5b4 \ucc45\ubd09\ub418\uba70, \uc678\uc655\ub0b4\uc81c \uccb4\uc81c\ub97c \ud655\ub9bd\ud568\uacfc \ub3d9\uc2dc\uc5d0 \uad70\uc8fc\ub4e4\uc5d0\uac8c \ubb18\ud638\ub97c \ubd99\uc600\ub2e4. \uc911\uac04\uc5d0 \uba85\ub098\ub77c\uac00 \ubca0\ud2b8\ub0a8\uc744 \uce68\ub7b5\ud558\uc5ec \uc548\ub0a8\uad6d \ub300\uc2e0 \uad50\uc9c0\uad6d\uc73c\ub85c \uadf8 \uaca9\uc744 \ub0ae\ucd94\uace0, \uc5ed\uc2dc \ubb18\ud638\uc640 \uc2dc\ud638\ub97c \uc4f0\ub294 \ub4f1\uc758 \ud48d\uc2b5\uc744 \uae08\ud558\uc600\uc73c\ub098, 20\ub144\ub9cc\uc5d0 \ub808 \uc655\uc870\uac00 \ub4e4\uc5b4\uc11c\uba70 \ub2e4\uc2dc \ubb18\uc81c\ub97c \ubd80\ud65c\uc2dc\ucf30\ub2e4. \ubca0\ud2b8\ub0a8\uc778\ub4e4\uc740 \ub9c8\uc9c0\ub9c9 \uc655\uc870\uc778 \uc751\uc6b0\uc60c \uc655\uc870\uae4c\uc9c0 \uc81c\ub3c4\ub97c \uc720\uc9c0\ud558\uc5ec \uad70\uc8fc\ub4e4\uc5d0\uac8c \ubaa8\ub450 \ubb18\ud638\uc640 \uc2dc\ud638\ub97c \uc62c\ub838\ub2e4.", "paragraph_answer": "\ubca0\ud2b8\ub0a8\ub3c4 \uc678\uc655\ub0b4\uc81c\ub97c \uc9c0\ud5a5\ud558\uc5ec \uc548\uc73c\ub85c\ub294 \ud669\uc81c\ub97c \uc790\uce6d\ud558\uc600\uace0, \uc7ac\uc704\ud55c \uad70\uc8fc\ub4e4\uc5d0\uac8c \ubb18\ud638\ub97c \ubd99\uc600\ub2e4. \ubca0\ud2b8\ub0a8\uc5d0\uc11c \uac00\uc7a5 \uba3c\uc800 \ubb18\ud638\ub97c \uc4f0\uae30 \uc2dc\uc791\ud55c \uc655\uc870\ub294 \uc804 \ub808 \uc655\uc870\uc600\uc73c\ub098, \uc2dc\ud638\ub97c \uc62c\ub9ac\uc9c0 \uc54a\uace0 \ubb18\uc81c \ub610\ud55c \uc815\ub9bd\ub418\uc9c0 \uc54a\uc544, \uc81c\ub300\ub85c \uc808\ucc28\ub97c \ubc1f\uc544 \uc62c\ub824\uc9c4 \ubb18\ud638\uac00 \uc544\ub2c8\uc5c8\ub2e4. \uc774\ud6c4 \ubcf8\uaca9\uc801\uc73c\ub85c \ubb18\ud638 \uc81c\ub3c4\uac00 \uc815\ucc29\ub41c \uc655\uc870\ub294 12\uc138\uae30 \uc911\uc5fd\uc5d0 \uc138\uc6cc\uc9c4 \ub9ac \uc655\uc870 \ub85c, \uc911\uad6d\uc758 \ub0a8\uc1a1\uc5d0 \uc758\ud574 \uc774\uc804\uc758 \uad50\uc9c0\uad70\uc655(\u4ea4\u8dbe\u90e1\u738b)\uc5d0\uc11c \uc548\ub0a8\uad6d\uc655(\u5b89\u5357\u570b\u738b)\uc73c\ub85c \uaca9\uc0c1\ub418\uc5b4 \ucc45\ubd09\ub418\uba70, \uc678\uc655\ub0b4\uc81c \uccb4\uc81c\ub97c \ud655\ub9bd\ud568\uacfc \ub3d9\uc2dc\uc5d0 \uad70\uc8fc\ub4e4\uc5d0\uac8c \ubb18\ud638\ub97c \ubd99\uc600\ub2e4. \uc911\uac04\uc5d0 \uba85\ub098\ub77c\uac00 \ubca0\ud2b8\ub0a8\uc744 \uce68\ub7b5\ud558\uc5ec \uc548\ub0a8\uad6d \ub300\uc2e0 \uad50\uc9c0\uad6d\uc73c\ub85c \uadf8 \uaca9\uc744 \ub0ae\ucd94\uace0, \uc5ed\uc2dc \ubb18\ud638\uc640 \uc2dc\ud638\ub97c \uc4f0\ub294 \ub4f1\uc758 \ud48d\uc2b5\uc744 \uae08\ud558\uc600\uc73c\ub098, 20\ub144\ub9cc\uc5d0 \ub808 \uc655\uc870\uac00 \ub4e4\uc5b4\uc11c\uba70 \ub2e4\uc2dc \ubb18\uc81c\ub97c \ubd80\ud65c\uc2dc\ucf30\ub2e4. \ubca0\ud2b8\ub0a8\uc778\ub4e4\uc740 \ub9c8\uc9c0\ub9c9 \uc655\uc870\uc778 \uc751\uc6b0\uc60c \uc655\uc870\uae4c\uc9c0 \uc81c\ub3c4\ub97c \uc720\uc9c0\ud558\uc5ec \uad70\uc8fc\ub4e4\uc5d0\uac8c \ubaa8\ub450 \ubb18\ud638\uc640 \uc2dc\ud638\ub97c \uc62c\ub838\ub2e4.", "sentence_answer": "\uc774\ud6c4 \ubcf8\uaca9\uc801\uc73c\ub85c \ubb18\ud638 \uc81c\ub3c4\uac00 \uc815\ucc29\ub41c \uc655\uc870\ub294 12\uc138\uae30 \uc911\uc5fd\uc5d0 \uc138\uc6cc\uc9c4 \ub9ac \uc655\uc870 \ub85c, \uc911\uad6d\uc758 \ub0a8\uc1a1\uc5d0 \uc758\ud574 \uc774\uc804\uc758 \uad50\uc9c0\uad70\uc655(\u4ea4\u8dbe\u90e1\u738b)\uc5d0\uc11c \uc548\ub0a8\uad6d\uc655(\u5b89\u5357\u570b\u738b)\uc73c\ub85c \uaca9\uc0c1\ub418\uc5b4 \ucc45\ubd09\ub418\uba70, \uc678\uc655\ub0b4\uc81c \uccb4\uc81c\ub97c \ud655\ub9bd\ud568\uacfc \ub3d9\uc2dc\uc5d0 \uad70\uc8fc\ub4e4\uc5d0\uac8c \ubb18\ud638\ub97c \ubd99\uc600\ub2e4.", "paragraph_id": "6657037-13-2", "paragraph_question": "question: \ubca0\ud2b8\ub0a8\uc5d0\uc11c \ubb18\ud638 \uc81c\ub3c4\uac00 \ubcf8\uaca9\uc801\uc73c\ub85c \uc815\ucc29\ub41c \uc655\uc870\ub294?, context: \ubca0\ud2b8\ub0a8\ub3c4 \uc678\uc655\ub0b4\uc81c\ub97c \uc9c0\ud5a5\ud558\uc5ec \uc548\uc73c\ub85c\ub294 \ud669\uc81c\ub97c \uc790\uce6d\ud558\uc600\uace0, \uc7ac\uc704\ud55c \uad70\uc8fc\ub4e4\uc5d0\uac8c \ubb18\ud638\ub97c \ubd99\uc600\ub2e4. \ubca0\ud2b8\ub0a8\uc5d0\uc11c \uac00\uc7a5 \uba3c\uc800 \ubb18\ud638\ub97c \uc4f0\uae30 \uc2dc\uc791\ud55c \uc655\uc870\ub294 \uc804 \ub808 \uc655\uc870\uc600\uc73c\ub098, \uc2dc\ud638\ub97c \uc62c\ub9ac\uc9c0 \uc54a\uace0 \ubb18\uc81c \ub610\ud55c \uc815\ub9bd\ub418\uc9c0 \uc54a\uc544, \uc81c\ub300\ub85c \uc808\ucc28\ub97c \ubc1f\uc544 \uc62c\ub824\uc9c4 \ubb18\ud638\uac00 \uc544\ub2c8\uc5c8\ub2e4. \uc774\ud6c4 \ubcf8\uaca9\uc801\uc73c\ub85c \ubb18\ud638 \uc81c\ub3c4\uac00 \uc815\ucc29\ub41c \uc655\uc870\ub294 12\uc138\uae30 \uc911\uc5fd\uc5d0 \uc138\uc6cc\uc9c4 \ub9ac \uc655\uc870\ub85c, \uc911\uad6d\uc758 \ub0a8\uc1a1\uc5d0 \uc758\ud574 \uc774\uc804\uc758 \uad50\uc9c0\uad70\uc655(\u4ea4\u8dbe\u90e1\u738b)\uc5d0\uc11c \uc548\ub0a8\uad6d\uc655(\u5b89\u5357\u570b\u738b)\uc73c\ub85c \uaca9\uc0c1\ub418\uc5b4 \ucc45\ubd09\ub418\uba70, \uc678\uc655\ub0b4\uc81c \uccb4\uc81c\ub97c \ud655\ub9bd\ud568\uacfc \ub3d9\uc2dc\uc5d0 \uad70\uc8fc\ub4e4\uc5d0\uac8c \ubb18\ud638\ub97c \ubd99\uc600\ub2e4. \uc911\uac04\uc5d0 \uba85\ub098\ub77c\uac00 \ubca0\ud2b8\ub0a8\uc744 \uce68\ub7b5\ud558\uc5ec \uc548\ub0a8\uad6d \ub300\uc2e0 \uad50\uc9c0\uad6d\uc73c\ub85c \uadf8 \uaca9\uc744 \ub0ae\ucd94\uace0, \uc5ed\uc2dc \ubb18\ud638\uc640 \uc2dc\ud638\ub97c \uc4f0\ub294 \ub4f1\uc758 \ud48d\uc2b5\uc744 \uae08\ud558\uc600\uc73c\ub098, 20\ub144\ub9cc\uc5d0 \ub808 \uc655\uc870\uac00 \ub4e4\uc5b4\uc11c\uba70 \ub2e4\uc2dc \ubb18\uc81c\ub97c \ubd80\ud65c\uc2dc\ucf30\ub2e4. \ubca0\ud2b8\ub0a8\uc778\ub4e4\uc740 \ub9c8\uc9c0\ub9c9 \uc655\uc870\uc778 \uc751\uc6b0\uc60c \uc655\uc870\uae4c\uc9c0 \uc81c\ub3c4\ub97c \uc720\uc9c0\ud558\uc5ec \uad70\uc8fc\ub4e4\uc5d0\uac8c \ubaa8\ub450 \ubb18\ud638\uc640 \uc2dc\ud638\ub97c \uc62c\ub838\ub2e4."} {"question": "1983\ub144 \ucd98\ucc9c\uad50\ub3c4\uc18c \ubcf4\ud638\uac10\ud638\uc18c\uac00 \uc774\uc804 \ub41c \uc9c0\uc5ed\uc740?", "paragraph": "1981\ub144 \ubcf4\ud638\u00b7\uac10\ud638\ubc95\uc744 \uc81c\uc815\ud588\ub2e4. \uc0c1\uc2b5\ubc94\uc740 \ud615\uae30 \uc885\ub8cc \ud6c4\uc5d0\ub3c4 \uc989\uac01\uc801\uc778 \uc0ac\ud68c \ubcf5\uadc0\ub97c \ub9c9\uc544\uc57c \ud55c\ub2e4\ub294 \ub17c\ub9ac\ub85c \ubcf4\ud638\uac10\ud638\uc81c\ub3c4\ub97c \uc2dc\ud589, \uc2dc\ud589\ud558\uace0 \uac19\uc740 \ud574 \uac15\uc6d0\ub3c4 \ucd98\ucc9c\uc2dc \ucd98\ucc9c\uad50\ub3c4\uc18c\uc5d0 \ubcf4\ud638\uac10\ud638\uc2dc\uc124\uc744 \uc124\uce58\ud588\ub2e4. \ucd98\ucc9c\uad50\ub3c4\uc18c \ubcf4\ud638\uac10\ud638\uc18c\ub294 \ub4a4\uc5d0 1983\ub144 2\uc6d4 12\uc77c \uacbd\uc0c1\ubd81\ub3c4 \uccad\uc1a1\uad70\uc73c\ub85c \uc774\uc804\ud588\ub2e4. 1982\ub144 10\uc6d4 26\uc77c \uac15\uc6d0\ub3c4\uc758 \uc774\uc2b9\ubcf5\uae30\ub150\uad00 \uc900\uacf5\uc2dd\uc5d0 \ucc38\uc11d\ud588\ub2e4. \uc774\ub0a0 \uc804\ub450\ud658\uc740 \"\ucd1d\uce7c \uc55e\uc5d0\uc11c \uc5b4\ub9b0 \ud559\uc0dd\uc774 \uc790\uae30 \uc2ec\uc815\uc744 \uc774\uc57c\uae30\ud588\ub2e4\ub294 \uac83\uc740 \uc5bc\ub9c8\ub098 \uc6a9\uae30 \uc788\ub294 \uc77c\uc774\ub0d0\"\uace0 \uc774\uc2b9\ubcf5\uc758 \uc6a9\uae30\ub97c \uce6d\uc1a1\ud558\uba70, \"\uc774\uc2b9\ubcf5 \uae30\ub150\uad00\uc758 \uc900\uacf5\uc744 \uacc4\uae30\ub85c \uc5b4\ub9b0\uc774\ub4e4 \ubfd0\ub9cc\uc774 \uc544\ub2c8\ub77c \uc804 \uad6d\ubbfc\uc5d0\uac8c\uae4c\uc9c0 \uc774\uc2b9\ubcf5 \uad70\uc758 \ubc18\uacf5\uc815\uc2e0\uc774 \ud30c\uae09\ub418\uc5b4 \uc6b0\ub9ac 2\uc138\ub4e4\uc5d0\uac8c \uad6d\ubbfc\uc815\uc2e0\uad50\uc721\uacfc \ubc18\uacf5\uad50\uc721\uc758 \uc77c\ub300 \uc804\uae30\uac00 \ub418\uc5b4\uc57c \ud560 \uac83\"\uc774\ub77c\uace0 \ud558\uc600\ub2e4. \uc0bc\uccad\uad50\uc721\ub300\ub97c \ud1b5\ud574 \uce5c\uc870\uc120\ubbfc\uc8fc\uc8fc\uc758\uc778\ubbfc\uacf5\ud654\uad6d\uacc4 \uc778\uc0ac, \ubc18\uccb4\uc81c \uc778\uc0ac, 5.18 \ubbfc\uc8fc\ud654\uc6b4\ub3d9 \ucc38\uac00\uc790, \ubc94\uc8c4\uc790, \uae61\ud328, \uac74\ub2ec, \ubb34\uc9c1\uc790, \ub178\uc219\uc790 \ub4f1\uc744 \uc7a1\uc544\ub4e4\uc5ec \ud2b9\uc218\ud6c8\ub828\uc744 \uc2dc\ud0a4\uae30\ub3c4 \ud588\ub2e4. \uc0bc\uccad\uad50\uc721\ub300\uc5d0 \uc5f0\ud589\ub41c \ub300\ub2e4\uc218\uac00 \uc804\uacfc\uac00 \uc5c6\uac70\ub098 \ucd08\ubc94\uc778\ub370\ub3c4 \ubd88\uad6c\ud558\uace0, \uc0bc\uccad\uad50\uc721\ub300 \ud2b9\uc218\ud6c8\ub828 \uacfc\uc815\uc5d0\uc11c \ub2e4\uc218\uc758 \uc0ac\ub9dd\uc790\uac00 \ubc1c\uc0dd\ud574 \uc778\uad8c\ud0c4\uc555 \ub17c\ub780\uc744 \ubd88\ub7ec\uc624\uae30\ub3c4 \ud588\ub2e4. \ub179\ud654\uc0ac\uc5c5\uc744 \ucd94\uc9c4\ud558\uba74\uc11c \uc6b4\ub3d9\uad8c \uc131\ud5a5\uc758 \ub300\ud559\uc0dd\ub4e4\uc758 \uc0ac\uc0c1\uac1c\uc870\ub97c \uc704\ud55c \ubaa9\uc801\uc73c\ub85c \ud65c\uc6a9\ud588\ub2e4.", "answer": "\uacbd\uc0c1\ubd81\ub3c4 \uccad\uc1a1\uad70", "sentence": "\ucd98\ucc9c\uad50\ub3c4\uc18c \ubcf4\ud638\uac10\ud638\uc18c\ub294 \ub4a4\uc5d0 1983\ub144 2\uc6d4 12\uc77c \uacbd\uc0c1\ubd81\ub3c4 \uccad\uc1a1\uad70 \uc73c\ub85c \uc774\uc804\ud588\ub2e4.", "paragraph_sentence": "1981\ub144 \ubcf4\ud638\u00b7\uac10\ud638\ubc95\uc744 \uc81c\uc815\ud588\ub2e4. \uc0c1\uc2b5\ubc94\uc740 \ud615\uae30 \uc885\ub8cc \ud6c4\uc5d0\ub3c4 \uc989\uac01\uc801\uc778 \uc0ac\ud68c \ubcf5\uadc0\ub97c \ub9c9\uc544\uc57c \ud55c\ub2e4\ub294 \ub17c\ub9ac\ub85c \ubcf4\ud638\uac10\ud638\uc81c\ub3c4\ub97c \uc2dc\ud589, \uc2dc\ud589\ud558\uace0 \uac19\uc740 \ud574 \uac15\uc6d0\ub3c4 \ucd98\ucc9c\uc2dc \ucd98\ucc9c\uad50\ub3c4\uc18c\uc5d0 \ubcf4\ud638\uac10\ud638\uc2dc\uc124\uc744 \uc124\uce58\ud588\ub2e4. \ucd98\ucc9c\uad50\ub3c4\uc18c \ubcf4\ud638\uac10\ud638\uc18c\ub294 \ub4a4\uc5d0 1983\ub144 2\uc6d4 12\uc77c \uacbd\uc0c1\ubd81\ub3c4 \uccad\uc1a1\uad70 \uc73c\ub85c \uc774\uc804\ud588\ub2e4. 1982\ub144 10\uc6d4 26\uc77c \uac15\uc6d0\ub3c4\uc758 \uc774\uc2b9\ubcf5\uae30\ub150\uad00 \uc900\uacf5\uc2dd\uc5d0 \ucc38\uc11d\ud588\ub2e4. \uc774\ub0a0 \uc804\ub450\ud658\uc740 \"\ucd1d\uce7c \uc55e\uc5d0\uc11c \uc5b4\ub9b0 \ud559\uc0dd\uc774 \uc790\uae30 \uc2ec\uc815\uc744 \uc774\uc57c\uae30\ud588\ub2e4\ub294 \uac83\uc740 \uc5bc\ub9c8\ub098 \uc6a9\uae30 \uc788\ub294 \uc77c\uc774\ub0d0\"\uace0 \uc774\uc2b9\ubcf5\uc758 \uc6a9\uae30\ub97c \uce6d\uc1a1\ud558\uba70, \"\uc774\uc2b9\ubcf5 \uae30\ub150\uad00\uc758 \uc900\uacf5\uc744 \uacc4\uae30\ub85c \uc5b4\ub9b0\uc774\ub4e4 \ubfd0\ub9cc\uc774 \uc544\ub2c8\ub77c \uc804 \uad6d\ubbfc\uc5d0\uac8c\uae4c\uc9c0 \uc774\uc2b9\ubcf5 \uad70\uc758 \ubc18\uacf5\uc815\uc2e0\uc774 \ud30c\uae09\ub418\uc5b4 \uc6b0\ub9ac 2\uc138\ub4e4\uc5d0\uac8c \uad6d\ubbfc\uc815\uc2e0\uad50\uc721\uacfc \ubc18\uacf5\uad50\uc721\uc758 \uc77c\ub300 \uc804\uae30\uac00 \ub418\uc5b4\uc57c \ud560 \uac83\"\uc774\ub77c\uace0 \ud558\uc600\ub2e4. \uc0bc\uccad\uad50\uc721\ub300\ub97c \ud1b5\ud574 \uce5c\uc870\uc120\ubbfc\uc8fc\uc8fc\uc758\uc778\ubbfc\uacf5\ud654\uad6d\uacc4 \uc778\uc0ac, \ubc18\uccb4\uc81c \uc778\uc0ac, 5.18 \ubbfc\uc8fc\ud654\uc6b4\ub3d9 \ucc38\uac00\uc790, \ubc94\uc8c4\uc790, \uae61\ud328, \uac74\ub2ec, \ubb34\uc9c1\uc790, \ub178\uc219\uc790 \ub4f1\uc744 \uc7a1\uc544\ub4e4\uc5ec \ud2b9\uc218\ud6c8\ub828\uc744 \uc2dc\ud0a4\uae30\ub3c4 \ud588\ub2e4. \uc0bc\uccad\uad50\uc721\ub300\uc5d0 \uc5f0\ud589\ub41c \ub300\ub2e4\uc218\uac00 \uc804\uacfc\uac00 \uc5c6\uac70\ub098 \ucd08\ubc94\uc778\ub370\ub3c4 \ubd88\uad6c\ud558\uace0, \uc0bc\uccad\uad50\uc721\ub300 \ud2b9\uc218\ud6c8\ub828 \uacfc\uc815\uc5d0\uc11c \ub2e4\uc218\uc758 \uc0ac\ub9dd\uc790\uac00 \ubc1c\uc0dd\ud574 \uc778\uad8c\ud0c4\uc555 \ub17c\ub780\uc744 \ubd88\ub7ec\uc624\uae30\ub3c4 \ud588\ub2e4. \ub179\ud654\uc0ac\uc5c5\uc744 \ucd94\uc9c4\ud558\uba74\uc11c \uc6b4\ub3d9\uad8c \uc131\ud5a5\uc758 \ub300\ud559\uc0dd\ub4e4\uc758 \uc0ac\uc0c1\uac1c\uc870\ub97c \uc704\ud55c \ubaa9\uc801\uc73c\ub85c \ud65c\uc6a9\ud588\ub2e4.", "paragraph_answer": "1981\ub144 \ubcf4\ud638\u00b7\uac10\ud638\ubc95\uc744 \uc81c\uc815\ud588\ub2e4. \uc0c1\uc2b5\ubc94\uc740 \ud615\uae30 \uc885\ub8cc \ud6c4\uc5d0\ub3c4 \uc989\uac01\uc801\uc778 \uc0ac\ud68c \ubcf5\uadc0\ub97c \ub9c9\uc544\uc57c \ud55c\ub2e4\ub294 \ub17c\ub9ac\ub85c \ubcf4\ud638\uac10\ud638\uc81c\ub3c4\ub97c \uc2dc\ud589, \uc2dc\ud589\ud558\uace0 \uac19\uc740 \ud574 \uac15\uc6d0\ub3c4 \ucd98\ucc9c\uc2dc \ucd98\ucc9c\uad50\ub3c4\uc18c\uc5d0 \ubcf4\ud638\uac10\ud638\uc2dc\uc124\uc744 \uc124\uce58\ud588\ub2e4. \ucd98\ucc9c\uad50\ub3c4\uc18c \ubcf4\ud638\uac10\ud638\uc18c\ub294 \ub4a4\uc5d0 1983\ub144 2\uc6d4 12\uc77c \uacbd\uc0c1\ubd81\ub3c4 \uccad\uc1a1\uad70 \uc73c\ub85c \uc774\uc804\ud588\ub2e4. 1982\ub144 10\uc6d4 26\uc77c \uac15\uc6d0\ub3c4\uc758 \uc774\uc2b9\ubcf5\uae30\ub150\uad00 \uc900\uacf5\uc2dd\uc5d0 \ucc38\uc11d\ud588\ub2e4. \uc774\ub0a0 \uc804\ub450\ud658\uc740 \"\ucd1d\uce7c \uc55e\uc5d0\uc11c \uc5b4\ub9b0 \ud559\uc0dd\uc774 \uc790\uae30 \uc2ec\uc815\uc744 \uc774\uc57c\uae30\ud588\ub2e4\ub294 \uac83\uc740 \uc5bc\ub9c8\ub098 \uc6a9\uae30 \uc788\ub294 \uc77c\uc774\ub0d0\"\uace0 \uc774\uc2b9\ubcf5\uc758 \uc6a9\uae30\ub97c \uce6d\uc1a1\ud558\uba70, \"\uc774\uc2b9\ubcf5 \uae30\ub150\uad00\uc758 \uc900\uacf5\uc744 \uacc4\uae30\ub85c \uc5b4\ub9b0\uc774\ub4e4 \ubfd0\ub9cc\uc774 \uc544\ub2c8\ub77c \uc804 \uad6d\ubbfc\uc5d0\uac8c\uae4c\uc9c0 \uc774\uc2b9\ubcf5 \uad70\uc758 \ubc18\uacf5\uc815\uc2e0\uc774 \ud30c\uae09\ub418\uc5b4 \uc6b0\ub9ac 2\uc138\ub4e4\uc5d0\uac8c \uad6d\ubbfc\uc815\uc2e0\uad50\uc721\uacfc \ubc18\uacf5\uad50\uc721\uc758 \uc77c\ub300 \uc804\uae30\uac00 \ub418\uc5b4\uc57c \ud560 \uac83\"\uc774\ub77c\uace0 \ud558\uc600\ub2e4. \uc0bc\uccad\uad50\uc721\ub300\ub97c \ud1b5\ud574 \uce5c\uc870\uc120\ubbfc\uc8fc\uc8fc\uc758\uc778\ubbfc\uacf5\ud654\uad6d\uacc4 \uc778\uc0ac, \ubc18\uccb4\uc81c \uc778\uc0ac, 5.18 \ubbfc\uc8fc\ud654\uc6b4\ub3d9 \ucc38\uac00\uc790, \ubc94\uc8c4\uc790, \uae61\ud328, \uac74\ub2ec, \ubb34\uc9c1\uc790, \ub178\uc219\uc790 \ub4f1\uc744 \uc7a1\uc544\ub4e4\uc5ec \ud2b9\uc218\ud6c8\ub828\uc744 \uc2dc\ud0a4\uae30\ub3c4 \ud588\ub2e4. \uc0bc\uccad\uad50\uc721\ub300\uc5d0 \uc5f0\ud589\ub41c \ub300\ub2e4\uc218\uac00 \uc804\uacfc\uac00 \uc5c6\uac70\ub098 \ucd08\ubc94\uc778\ub370\ub3c4 \ubd88\uad6c\ud558\uace0, \uc0bc\uccad\uad50\uc721\ub300 \ud2b9\uc218\ud6c8\ub828 \uacfc\uc815\uc5d0\uc11c \ub2e4\uc218\uc758 \uc0ac\ub9dd\uc790\uac00 \ubc1c\uc0dd\ud574 \uc778\uad8c\ud0c4\uc555 \ub17c\ub780\uc744 \ubd88\ub7ec\uc624\uae30\ub3c4 \ud588\ub2e4. \ub179\ud654\uc0ac\uc5c5\uc744 \ucd94\uc9c4\ud558\uba74\uc11c \uc6b4\ub3d9\uad8c \uc131\ud5a5\uc758 \ub300\ud559\uc0dd\ub4e4\uc758 \uc0ac\uc0c1\uac1c\uc870\ub97c \uc704\ud55c \ubaa9\uc801\uc73c\ub85c \ud65c\uc6a9\ud588\ub2e4.", "sentence_answer": "\ucd98\ucc9c\uad50\ub3c4\uc18c \ubcf4\ud638\uac10\ud638\uc18c\ub294 \ub4a4\uc5d0 1983\ub144 2\uc6d4 12\uc77c \uacbd\uc0c1\ubd81\ub3c4 \uccad\uc1a1\uad70 \uc73c\ub85c \uc774\uc804\ud588\ub2e4.", "paragraph_id": "6390080-18-1", "paragraph_question": "question: 1983\ub144 \ucd98\ucc9c\uad50\ub3c4\uc18c \ubcf4\ud638\uac10\ud638\uc18c\uac00 \uc774\uc804 \ub41c \uc9c0\uc5ed\uc740?, context: 1981\ub144 \ubcf4\ud638\u00b7\uac10\ud638\ubc95\uc744 \uc81c\uc815\ud588\ub2e4. \uc0c1\uc2b5\ubc94\uc740 \ud615\uae30 \uc885\ub8cc \ud6c4\uc5d0\ub3c4 \uc989\uac01\uc801\uc778 \uc0ac\ud68c \ubcf5\uadc0\ub97c \ub9c9\uc544\uc57c \ud55c\ub2e4\ub294 \ub17c\ub9ac\ub85c \ubcf4\ud638\uac10\ud638\uc81c\ub3c4\ub97c \uc2dc\ud589, \uc2dc\ud589\ud558\uace0 \uac19\uc740 \ud574 \uac15\uc6d0\ub3c4 \ucd98\ucc9c\uc2dc \ucd98\ucc9c\uad50\ub3c4\uc18c\uc5d0 \ubcf4\ud638\uac10\ud638\uc2dc\uc124\uc744 \uc124\uce58\ud588\ub2e4. \ucd98\ucc9c\uad50\ub3c4\uc18c \ubcf4\ud638\uac10\ud638\uc18c\ub294 \ub4a4\uc5d0 1983\ub144 2\uc6d4 12\uc77c \uacbd\uc0c1\ubd81\ub3c4 \uccad\uc1a1\uad70\uc73c\ub85c \uc774\uc804\ud588\ub2e4. 1982\ub144 10\uc6d4 26\uc77c \uac15\uc6d0\ub3c4\uc758 \uc774\uc2b9\ubcf5\uae30\ub150\uad00 \uc900\uacf5\uc2dd\uc5d0 \ucc38\uc11d\ud588\ub2e4. \uc774\ub0a0 \uc804\ub450\ud658\uc740 \"\ucd1d\uce7c \uc55e\uc5d0\uc11c \uc5b4\ub9b0 \ud559\uc0dd\uc774 \uc790\uae30 \uc2ec\uc815\uc744 \uc774\uc57c\uae30\ud588\ub2e4\ub294 \uac83\uc740 \uc5bc\ub9c8\ub098 \uc6a9\uae30 \uc788\ub294 \uc77c\uc774\ub0d0\"\uace0 \uc774\uc2b9\ubcf5\uc758 \uc6a9\uae30\ub97c \uce6d\uc1a1\ud558\uba70, \"\uc774\uc2b9\ubcf5 \uae30\ub150\uad00\uc758 \uc900\uacf5\uc744 \uacc4\uae30\ub85c \uc5b4\ub9b0\uc774\ub4e4 \ubfd0\ub9cc\uc774 \uc544\ub2c8\ub77c \uc804 \uad6d\ubbfc\uc5d0\uac8c\uae4c\uc9c0 \uc774\uc2b9\ubcf5 \uad70\uc758 \ubc18\uacf5\uc815\uc2e0\uc774 \ud30c\uae09\ub418\uc5b4 \uc6b0\ub9ac 2\uc138\ub4e4\uc5d0\uac8c \uad6d\ubbfc\uc815\uc2e0\uad50\uc721\uacfc \ubc18\uacf5\uad50\uc721\uc758 \uc77c\ub300 \uc804\uae30\uac00 \ub418\uc5b4\uc57c \ud560 \uac83\"\uc774\ub77c\uace0 \ud558\uc600\ub2e4. \uc0bc\uccad\uad50\uc721\ub300\ub97c \ud1b5\ud574 \uce5c\uc870\uc120\ubbfc\uc8fc\uc8fc\uc758\uc778\ubbfc\uacf5\ud654\uad6d\uacc4 \uc778\uc0ac, \ubc18\uccb4\uc81c \uc778\uc0ac, 5.18 \ubbfc\uc8fc\ud654\uc6b4\ub3d9 \ucc38\uac00\uc790, \ubc94\uc8c4\uc790, \uae61\ud328, \uac74\ub2ec, \ubb34\uc9c1\uc790, \ub178\uc219\uc790 \ub4f1\uc744 \uc7a1\uc544\ub4e4\uc5ec \ud2b9\uc218\ud6c8\ub828\uc744 \uc2dc\ud0a4\uae30\ub3c4 \ud588\ub2e4. \uc0bc\uccad\uad50\uc721\ub300\uc5d0 \uc5f0\ud589\ub41c \ub300\ub2e4\uc218\uac00 \uc804\uacfc\uac00 \uc5c6\uac70\ub098 \ucd08\ubc94\uc778\ub370\ub3c4 \ubd88\uad6c\ud558\uace0, \uc0bc\uccad\uad50\uc721\ub300 \ud2b9\uc218\ud6c8\ub828 \uacfc\uc815\uc5d0\uc11c \ub2e4\uc218\uc758 \uc0ac\ub9dd\uc790\uac00 \ubc1c\uc0dd\ud574 \uc778\uad8c\ud0c4\uc555 \ub17c\ub780\uc744 \ubd88\ub7ec\uc624\uae30\ub3c4 \ud588\ub2e4. \ub179\ud654\uc0ac\uc5c5\uc744 \ucd94\uc9c4\ud558\uba74\uc11c \uc6b4\ub3d9\uad8c \uc131\ud5a5\uc758 \ub300\ud559\uc0dd\ub4e4\uc758 \uc0ac\uc0c1\uac1c\uc870\ub97c \uc704\ud55c \ubaa9\uc801\uc73c\ub85c \ud65c\uc6a9\ud588\ub2e4."} {"question": "1970\ub144 \ucf54\ub974\uc2dc\uce74\uc5d0\uc11c \uc774\ud718\uc18c\uc758 \uac15\uc758\ub97c \ub4e3\uace0 \uc544\uc774\ub514\uc5b4\ub97c \uc5bb\uc5b4 \ub178\ubca8 \ubb3c\ub9ac\ud559\uc0c1\uc744 \uc218\uc0c1\ud55c \uc778\ubb3c\uc740?", "paragraph": "1969\ub144\uc5d0\ub294 \uc790\ubc1c \ub300\uce6d \uae68\uc9d0\uc744 \ub17c\ud560 \ub54c \uc7a5\ub09c\uac10 \ubaa8\ud615\uc73c\ub85c\uc11c \ub300\ud45c\uc801\uc73c\ub85c \uc560\uc6a9\ub418\uace0 \uc788\ub294 \uc2dc\uadf8\ub9c8 \ubaa8\ud615\uc758 \uc7ac\uaddc\uaca9\ud654\uc5d0 \uc131\uacf5\ud558\uc600\ub2e4. \uc774\ub7f0 \uac00\uc6b4\ub370 \ub2f9\uc2dc \ub124\ub35c\ub780\ub4dc\uc758 \ub300\ud559\uc6d0\uc0dd\uc774\ub358 \ud5e4\ub77c\ub974\ub4a4\uc2a4 \uc5c7\ud638\ud504\ud2b8\ub294 \ud789\uc2a4 \uba54\ucee4\ub2c8\uc998\uc744 \uc591-\ubc00\uc2a4 \uc774\ub860\uc5d0 \uc801\uc6a9\ud558\uc5ec \ube44\uac00\ud658 \uac8c\uc774\uc9c0 \uc774\ub860\uc758 \uad6d\uc18c \ub300\uce6d\uc131\uc774 \uc790\ubc1c\uc801\uc73c\ub85c \uae68\uc9c0\ub294 \ubaa8\ud615\uc744 \uc5f0\uad6c\ud558\uace0 \uc788\uc5c8\ub2e4. \uadf8\ub294 1970\ub144 \ud504\ub791\uc2a4\ub839 \ucf54\ub974\uc2dc\uce74\uc758 \uce74\ub974\uc81c\uc2a4 \uc5ec\ub984\ud559\uad50\uc5d0\uc11c \uc774\ud718\uc18c\uc758 \uac15\uc758\ub97c \ub4e4\uc5c8\ub294\ub370 \uc774\ub54c \uadf8\ub294 \uadf8\uc758 \ud559\uc704 \ub17c\ubb38 \uc8fc\uc81c\uc600\ub358 \uc790\ubc1c\uc801\uc73c\ub85c \ub300\uce6d\uc131\uc774 \ubd80\uc11c\uc9c4 \ube44\uac00\ud658 \uac8c\uc774\uc9c0 \uc774\ub860\uc758 \uc7ac\uaddc\uaca9\ud654\uc5d0 \ub300\ud574 \uacb0\uc815\uc801\uc778 \uc544\uc774\ub514\uc5b4\ub97c \uc5bb\uc5c8\uc73c\uba70 \ub9c8\uce68\ub0b4 \uc774\uc5d0 \uc131\uacf5\ud558\uac8c \ub41c\ub2e4. \uc5c7\ud638\ud504\ud2b8\ub294 \uc774 \uc5c5\uc801\uc73c\ub85c 1999\ub144\uc5d0 \ub2f9\uc2dc \uc790\uc2e0\uc758 \uc9c0\ub3c4\uad50\uc218\uc600\ub358 \ud3a0\ud2b8\ub9cc\uacfc \ud568\uaed8 \ub178\ubca8 \ubb3c\ub9ac\ud559\uc0c1\uc744 \uc218\uc0c1\ud558\uc600\ub2e4.", "answer": "\ud5e4\ub77c\ub974\ub4a4\uc2a4 \uc5c7\ud638\ud504\ud2b8", "sentence": "\uc774\ub7f0 \uac00\uc6b4\ub370 \ub2f9\uc2dc \ub124\ub35c\ub780\ub4dc\uc758 \ub300\ud559\uc6d0\uc0dd\uc774\ub358 \ud5e4\ub77c\ub974\ub4a4\uc2a4 \uc5c7\ud638\ud504\ud2b8 \ub294 \ud789\uc2a4 \uba54\ucee4\ub2c8\uc998\uc744 \uc591-\ubc00\uc2a4 \uc774\ub860\uc5d0 \uc801\uc6a9\ud558\uc5ec \ube44\uac00\ud658 \uac8c\uc774\uc9c0 \uc774\ub860\uc758 \uad6d\uc18c \ub300\uce6d\uc131\uc774 \uc790\ubc1c\uc801\uc73c\ub85c \uae68\uc9c0\ub294 \ubaa8\ud615\uc744 \uc5f0\uad6c\ud558\uace0 \uc788\uc5c8\ub2e4.", "paragraph_sentence": "1969\ub144\uc5d0\ub294 \uc790\ubc1c \ub300\uce6d \uae68\uc9d0\uc744 \ub17c\ud560 \ub54c \uc7a5\ub09c\uac10 \ubaa8\ud615\uc73c\ub85c\uc11c \ub300\ud45c\uc801\uc73c\ub85c \uc560\uc6a9\ub418\uace0 \uc788\ub294 \uc2dc\uadf8\ub9c8 \ubaa8\ud615\uc758 \uc7ac\uaddc\uaca9\ud654\uc5d0 \uc131\uacf5\ud558\uc600\ub2e4. \uc774\ub7f0 \uac00\uc6b4\ub370 \ub2f9\uc2dc \ub124\ub35c\ub780\ub4dc\uc758 \ub300\ud559\uc6d0\uc0dd\uc774\ub358 \ud5e4\ub77c\ub974\ub4a4\uc2a4 \uc5c7\ud638\ud504\ud2b8 \ub294 \ud789\uc2a4 \uba54\ucee4\ub2c8\uc998\uc744 \uc591-\ubc00\uc2a4 \uc774\ub860\uc5d0 \uc801\uc6a9\ud558\uc5ec \ube44\uac00\ud658 \uac8c\uc774\uc9c0 \uc774\ub860\uc758 \uad6d\uc18c \ub300\uce6d\uc131\uc774 \uc790\ubc1c\uc801\uc73c\ub85c \uae68\uc9c0\ub294 \ubaa8\ud615\uc744 \uc5f0\uad6c\ud558\uace0 \uc788\uc5c8\ub2e4. \uadf8\ub294 1970\ub144 \ud504\ub791\uc2a4\ub839 \ucf54\ub974\uc2dc\uce74\uc758 \uce74\ub974\uc81c\uc2a4 \uc5ec\ub984\ud559\uad50\uc5d0\uc11c \uc774\ud718\uc18c\uc758 \uac15\uc758\ub97c \ub4e4\uc5c8\ub294\ub370 \uc774\ub54c \uadf8\ub294 \uadf8\uc758 \ud559\uc704 \ub17c\ubb38 \uc8fc\uc81c\uc600\ub358 \uc790\ubc1c\uc801\uc73c\ub85c \ub300\uce6d\uc131\uc774 \ubd80\uc11c\uc9c4 \ube44\uac00\ud658 \uac8c\uc774\uc9c0 \uc774\ub860\uc758 \uc7ac\uaddc\uaca9\ud654\uc5d0 \ub300\ud574 \uacb0\uc815\uc801\uc778 \uc544\uc774\ub514\uc5b4\ub97c \uc5bb\uc5c8\uc73c\uba70 \ub9c8\uce68\ub0b4 \uc774\uc5d0 \uc131\uacf5\ud558\uac8c \ub41c\ub2e4. \uc5c7\ud638\ud504\ud2b8\ub294 \uc774 \uc5c5\uc801\uc73c\ub85c 1999\ub144\uc5d0 \ub2f9\uc2dc \uc790\uc2e0\uc758 \uc9c0\ub3c4\uad50\uc218\uc600\ub358 \ud3a0\ud2b8\ub9cc\uacfc \ud568\uaed8 \ub178\ubca8 \ubb3c\ub9ac\ud559\uc0c1\uc744 \uc218\uc0c1\ud558\uc600\ub2e4.", "paragraph_answer": "1969\ub144\uc5d0\ub294 \uc790\ubc1c \ub300\uce6d \uae68\uc9d0\uc744 \ub17c\ud560 \ub54c \uc7a5\ub09c\uac10 \ubaa8\ud615\uc73c\ub85c\uc11c \ub300\ud45c\uc801\uc73c\ub85c \uc560\uc6a9\ub418\uace0 \uc788\ub294 \uc2dc\uadf8\ub9c8 \ubaa8\ud615\uc758 \uc7ac\uaddc\uaca9\ud654\uc5d0 \uc131\uacf5\ud558\uc600\ub2e4. \uc774\ub7f0 \uac00\uc6b4\ub370 \ub2f9\uc2dc \ub124\ub35c\ub780\ub4dc\uc758 \ub300\ud559\uc6d0\uc0dd\uc774\ub358 \ud5e4\ub77c\ub974\ub4a4\uc2a4 \uc5c7\ud638\ud504\ud2b8 \ub294 \ud789\uc2a4 \uba54\ucee4\ub2c8\uc998\uc744 \uc591-\ubc00\uc2a4 \uc774\ub860\uc5d0 \uc801\uc6a9\ud558\uc5ec \ube44\uac00\ud658 \uac8c\uc774\uc9c0 \uc774\ub860\uc758 \uad6d\uc18c \ub300\uce6d\uc131\uc774 \uc790\ubc1c\uc801\uc73c\ub85c \uae68\uc9c0\ub294 \ubaa8\ud615\uc744 \uc5f0\uad6c\ud558\uace0 \uc788\uc5c8\ub2e4. \uadf8\ub294 1970\ub144 \ud504\ub791\uc2a4\ub839 \ucf54\ub974\uc2dc\uce74\uc758 \uce74\ub974\uc81c\uc2a4 \uc5ec\ub984\ud559\uad50\uc5d0\uc11c \uc774\ud718\uc18c\uc758 \uac15\uc758\ub97c \ub4e4\uc5c8\ub294\ub370 \uc774\ub54c \uadf8\ub294 \uadf8\uc758 \ud559\uc704 \ub17c\ubb38 \uc8fc\uc81c\uc600\ub358 \uc790\ubc1c\uc801\uc73c\ub85c \ub300\uce6d\uc131\uc774 \ubd80\uc11c\uc9c4 \ube44\uac00\ud658 \uac8c\uc774\uc9c0 \uc774\ub860\uc758 \uc7ac\uaddc\uaca9\ud654\uc5d0 \ub300\ud574 \uacb0\uc815\uc801\uc778 \uc544\uc774\ub514\uc5b4\ub97c \uc5bb\uc5c8\uc73c\uba70 \ub9c8\uce68\ub0b4 \uc774\uc5d0 \uc131\uacf5\ud558\uac8c \ub41c\ub2e4. \uc5c7\ud638\ud504\ud2b8\ub294 \uc774 \uc5c5\uc801\uc73c\ub85c 1999\ub144\uc5d0 \ub2f9\uc2dc \uc790\uc2e0\uc758 \uc9c0\ub3c4\uad50\uc218\uc600\ub358 \ud3a0\ud2b8\ub9cc\uacfc \ud568\uaed8 \ub178\ubca8 \ubb3c\ub9ac\ud559\uc0c1\uc744 \uc218\uc0c1\ud558\uc600\ub2e4.", "sentence_answer": "\uc774\ub7f0 \uac00\uc6b4\ub370 \ub2f9\uc2dc \ub124\ub35c\ub780\ub4dc\uc758 \ub300\ud559\uc6d0\uc0dd\uc774\ub358 \ud5e4\ub77c\ub974\ub4a4\uc2a4 \uc5c7\ud638\ud504\ud2b8 \ub294 \ud789\uc2a4 \uba54\ucee4\ub2c8\uc998\uc744 \uc591-\ubc00\uc2a4 \uc774\ub860\uc5d0 \uc801\uc6a9\ud558\uc5ec \ube44\uac00\ud658 \uac8c\uc774\uc9c0 \uc774\ub860\uc758 \uad6d\uc18c \ub300\uce6d\uc131\uc774 \uc790\ubc1c\uc801\uc73c\ub85c \uae68\uc9c0\ub294 \ubaa8\ud615\uc744 \uc5f0\uad6c\ud558\uace0 \uc788\uc5c8\ub2e4.", "paragraph_id": "6329268-17-1", "paragraph_question": "question: 1970\ub144 \ucf54\ub974\uc2dc\uce74\uc5d0\uc11c \uc774\ud718\uc18c\uc758 \uac15\uc758\ub97c \ub4e3\uace0 \uc544\uc774\ub514\uc5b4\ub97c \uc5bb\uc5b4 \ub178\ubca8 \ubb3c\ub9ac\ud559\uc0c1\uc744 \uc218\uc0c1\ud55c \uc778\ubb3c\uc740?, context: 1969\ub144\uc5d0\ub294 \uc790\ubc1c \ub300\uce6d \uae68\uc9d0\uc744 \ub17c\ud560 \ub54c \uc7a5\ub09c\uac10 \ubaa8\ud615\uc73c\ub85c\uc11c \ub300\ud45c\uc801\uc73c\ub85c \uc560\uc6a9\ub418\uace0 \uc788\ub294 \uc2dc\uadf8\ub9c8 \ubaa8\ud615\uc758 \uc7ac\uaddc\uaca9\ud654\uc5d0 \uc131\uacf5\ud558\uc600\ub2e4. \uc774\ub7f0 \uac00\uc6b4\ub370 \ub2f9\uc2dc \ub124\ub35c\ub780\ub4dc\uc758 \ub300\ud559\uc6d0\uc0dd\uc774\ub358 \ud5e4\ub77c\ub974\ub4a4\uc2a4 \uc5c7\ud638\ud504\ud2b8\ub294 \ud789\uc2a4 \uba54\ucee4\ub2c8\uc998\uc744 \uc591-\ubc00\uc2a4 \uc774\ub860\uc5d0 \uc801\uc6a9\ud558\uc5ec \ube44\uac00\ud658 \uac8c\uc774\uc9c0 \uc774\ub860\uc758 \uad6d\uc18c \ub300\uce6d\uc131\uc774 \uc790\ubc1c\uc801\uc73c\ub85c \uae68\uc9c0\ub294 \ubaa8\ud615\uc744 \uc5f0\uad6c\ud558\uace0 \uc788\uc5c8\ub2e4. \uadf8\ub294 1970\ub144 \ud504\ub791\uc2a4\ub839 \ucf54\ub974\uc2dc\uce74\uc758 \uce74\ub974\uc81c\uc2a4 \uc5ec\ub984\ud559\uad50\uc5d0\uc11c \uc774\ud718\uc18c\uc758 \uac15\uc758\ub97c \ub4e4\uc5c8\ub294\ub370 \uc774\ub54c \uadf8\ub294 \uadf8\uc758 \ud559\uc704 \ub17c\ubb38 \uc8fc\uc81c\uc600\ub358 \uc790\ubc1c\uc801\uc73c\ub85c \ub300\uce6d\uc131\uc774 \ubd80\uc11c\uc9c4 \ube44\uac00\ud658 \uac8c\uc774\uc9c0 \uc774\ub860\uc758 \uc7ac\uaddc\uaca9\ud654\uc5d0 \ub300\ud574 \uacb0\uc815\uc801\uc778 \uc544\uc774\ub514\uc5b4\ub97c \uc5bb\uc5c8\uc73c\uba70 \ub9c8\uce68\ub0b4 \uc774\uc5d0 \uc131\uacf5\ud558\uac8c \ub41c\ub2e4. \uc5c7\ud638\ud504\ud2b8\ub294 \uc774 \uc5c5\uc801\uc73c\ub85c 1999\ub144\uc5d0 \ub2f9\uc2dc \uc790\uc2e0\uc758 \uc9c0\ub3c4\uad50\uc218\uc600\ub358 \ud3a0\ud2b8\ub9cc\uacfc \ud568\uaed8 \ub178\ubca8 \ubb3c\ub9ac\ud559\uc0c1\uc744 \uc218\uc0c1\ud558\uc600\ub2e4."} {"question": "\uc218\ub2e8\uc758 \uc8fc\uc694 \uacbd\uc791\ubb3c\uc740 \ubb34\uc5c7\uc778\uac00?", "paragraph": "1996-2005\ub144 \uae30\uac04 \ub3d9\uc548 \uc5f0\ud3c9\uade0 6.3%\uc758 \uacbd\uc81c \uc131\uc7a5\uc744 \uae30\ub85d\ud558\uc600\ub2e4. \ucd1d\ub178\ub3d9\uc778\uad6c\uc758 80%\uac00 \ub18d\uc5c5 \ub610\ub294 \ubaa9\ucd95\uc5c5\uc5d0 \uc885\uc0ac\ud558\uace0 \uc788\ub2e4. \uacbd\uc9c0 \uba74\uc801\uc740 800\ub9cc\u33ca\ub85c \uc8fc\ub85c \ub098\uc77c\uac15 \uc720\uc5ed\uc5d0 \ubd84\ud3ec\ud55c\ub2e4. \uc8fc\uc694 \ub18d\uc0b0\ubb3c\uc740 \uba74\ud654\uc778\ub370, \uc8fc\ub85c \uc601\uad6d\uc778\uc774 \uac74\uc124\ud55c \uc13c\ub108\ub310\uc5d0 \uc758\ud574 \uad00\uac1c(\u704c\u6f11)\ub41c \ucf00\ub514\ub77c \uc9c0\uad6c\uc5d0\uc11c \uc7ac\ubc30\ub41c\ub2e4. \uba74\ud654 \uc774\uc678\uc758 \uacbd\uc791\ubb3c\ub85c\ub294 \uc544\ub77c\ube44\uc544\uace0\ubb34, \uae68, \ub099\ud654\uc0dd, \uc0ac\ud0d5\uc218\uc218, \uc870, \ubc00 \ub4f1\uc774 \uc788\ub2e4. \uacf5\uc5c5\uc5d0 \uc788\uc5b4\uc11c\ub294 \ub3c5\ub9bd \ud6c4 \uc12c\uc720, \uc2dd\ud488, \uc81c\ub2f9, \uc81c\uc9c0, \uc815\uc720, \uc2dc\uba58\ud2b8 \ub4f1\uc758 \uacf5\uc7a5\uc774 \uac74\uc124\ub418\uc5c8\ub2e4. \ud604\uc7ac \uc218\ub2e8\uc5d0\uc11c \uacf5\uc5c5\ud654\uc758 \uc7a5\uc560\uac00 \ub418\uace0 \uc788\ub294 \uac83\uc740 \uc790\ubcf8\uc758 \ubd80\uc871, \uad50\ud1b5\uc2dc\uc124\uc758 \ubbf8\ube44, \uae30\uc220\uc758 \ud6c4\uc9c4\uc131 \ub4f1\uc774\ub2e4. \uc9c0\ud558\uc790\uc6d0\uc73c\ub85c\ub294 \ucca0\uad11\uc11d\u00b7\ub9dd\uac04\u00b7\ub3d9 \ub4f1\uc774 \uc788\ub2e4. 2002\ub144 \uc218\ucd9c 17\uc5b5 1,770\ub9cc \ub2ec\ub7ec, \uc218\uc785 19\uc5b5 1,480\ub9cc \ub2ec\ub7ec\ub97c \uae30\ub85d\ud588\ub2e4. \uc8fc\uc694 \uc218\ucd9c\ud488\uc740 \uba74\ud654\uc640 \ub099\ud654\uc0dd, \uc544\ub77c\ube44\uc544\uace0\ubb34 \ub4f1\uc774\ub2e4. \ub300\ud3ed\uc801\uc778 \ubb34\uc5ed\uc801\uc790\ub294 \uc678\uad6d\uc758 \uac01\uc885 \uc6d0\uc870\uc640 \uc544\ub78d\uc0b0\uc720\uad6d\uc5d0 \ucde8\uc5c5\ud55c \ub178\ub3d9\uc790\uc758 \ubcf8\uad6d \uc1a1\uae08\uc73c\ub85c \uba54\uc6b4\ub2e4.", "answer": "\uba74\ud654", "sentence": "\uc8fc\uc694 \ub18d\uc0b0\ubb3c\uc740 \uba74\ud654 \uc778\ub370, \uc8fc\ub85c \uc601\uad6d\uc778\uc774 \uac74\uc124\ud55c \uc13c\ub108\ub310\uc5d0 \uc758\ud574 \uad00\uac1c(\u704c\u6f11)\ub41c \ucf00\ub514\ub77c \uc9c0\uad6c\uc5d0\uc11c \uc7ac\ubc30\ub41c\ub2e4.", "paragraph_sentence": "1996-2005\ub144 \uae30\uac04 \ub3d9\uc548 \uc5f0\ud3c9\uade0 6.3%\uc758 \uacbd\uc81c \uc131\uc7a5\uc744 \uae30\ub85d\ud558\uc600\ub2e4. \ucd1d\ub178\ub3d9\uc778\uad6c\uc758 80%\uac00 \ub18d\uc5c5 \ub610\ub294 \ubaa9\ucd95\uc5c5\uc5d0 \uc885\uc0ac\ud558\uace0 \uc788\ub2e4. \uacbd\uc9c0 \uba74\uc801\uc740 800\ub9cc\u33ca\ub85c \uc8fc\ub85c \ub098\uc77c\uac15 \uc720\uc5ed\uc5d0 \ubd84\ud3ec\ud55c\ub2e4. \uc8fc\uc694 \ub18d\uc0b0\ubb3c\uc740 \uba74\ud654 \uc778\ub370, \uc8fc\ub85c \uc601\uad6d\uc778\uc774 \uac74\uc124\ud55c \uc13c\ub108\ub310\uc5d0 \uc758\ud574 \uad00\uac1c(\u704c\u6f11)\ub41c \ucf00\ub514\ub77c \uc9c0\uad6c\uc5d0\uc11c \uc7ac\ubc30\ub41c\ub2e4. \uba74\ud654 \uc774\uc678\uc758 \uacbd\uc791\ubb3c\ub85c\ub294 \uc544\ub77c\ube44\uc544\uace0\ubb34, \uae68, \ub099\ud654\uc0dd, \uc0ac\ud0d5\uc218\uc218, \uc870, \ubc00 \ub4f1\uc774 \uc788\ub2e4. \uacf5\uc5c5\uc5d0 \uc788\uc5b4\uc11c\ub294 \ub3c5\ub9bd \ud6c4 \uc12c\uc720, \uc2dd\ud488, \uc81c\ub2f9, \uc81c\uc9c0, \uc815\uc720, \uc2dc\uba58\ud2b8 \ub4f1\uc758 \uacf5\uc7a5\uc774 \uac74\uc124\ub418\uc5c8\ub2e4. \ud604\uc7ac \uc218\ub2e8\uc5d0\uc11c \uacf5\uc5c5\ud654\uc758 \uc7a5\uc560\uac00 \ub418\uace0 \uc788\ub294 \uac83\uc740 \uc790\ubcf8\uc758 \ubd80\uc871, \uad50\ud1b5\uc2dc\uc124\uc758 \ubbf8\ube44, \uae30\uc220\uc758 \ud6c4\uc9c4\uc131 \ub4f1\uc774\ub2e4. \uc9c0\ud558\uc790\uc6d0\uc73c\ub85c\ub294 \ucca0\uad11\uc11d\u00b7\ub9dd\uac04\u00b7\ub3d9 \ub4f1\uc774 \uc788\ub2e4. 2002\ub144 \uc218\ucd9c 17\uc5b5 1,770\ub9cc \ub2ec\ub7ec, \uc218\uc785 19\uc5b5 1,480\ub9cc \ub2ec\ub7ec\ub97c \uae30\ub85d\ud588\ub2e4. \uc8fc\uc694 \uc218\ucd9c\ud488\uc740 \uba74\ud654\uc640 \ub099\ud654\uc0dd, \uc544\ub77c\ube44\uc544\uace0\ubb34 \ub4f1\uc774\ub2e4. \ub300\ud3ed\uc801\uc778 \ubb34\uc5ed\uc801\uc790\ub294 \uc678\uad6d\uc758 \uac01\uc885 \uc6d0\uc870\uc640 \uc544\ub78d\uc0b0\uc720\uad6d\uc5d0 \ucde8\uc5c5\ud55c \ub178\ub3d9\uc790\uc758 \ubcf8\uad6d \uc1a1\uae08\uc73c\ub85c \uba54\uc6b4\ub2e4.", "paragraph_answer": "1996-2005\ub144 \uae30\uac04 \ub3d9\uc548 \uc5f0\ud3c9\uade0 6.3%\uc758 \uacbd\uc81c \uc131\uc7a5\uc744 \uae30\ub85d\ud558\uc600\ub2e4. \ucd1d\ub178\ub3d9\uc778\uad6c\uc758 80%\uac00 \ub18d\uc5c5 \ub610\ub294 \ubaa9\ucd95\uc5c5\uc5d0 \uc885\uc0ac\ud558\uace0 \uc788\ub2e4. \uacbd\uc9c0 \uba74\uc801\uc740 800\ub9cc\u33ca\ub85c \uc8fc\ub85c \ub098\uc77c\uac15 \uc720\uc5ed\uc5d0 \ubd84\ud3ec\ud55c\ub2e4. \uc8fc\uc694 \ub18d\uc0b0\ubb3c\uc740 \uba74\ud654 \uc778\ub370, \uc8fc\ub85c \uc601\uad6d\uc778\uc774 \uac74\uc124\ud55c \uc13c\ub108\ub310\uc5d0 \uc758\ud574 \uad00\uac1c(\u704c\u6f11)\ub41c \ucf00\ub514\ub77c \uc9c0\uad6c\uc5d0\uc11c \uc7ac\ubc30\ub41c\ub2e4. \uba74\ud654 \uc774\uc678\uc758 \uacbd\uc791\ubb3c\ub85c\ub294 \uc544\ub77c\ube44\uc544\uace0\ubb34, \uae68, \ub099\ud654\uc0dd, \uc0ac\ud0d5\uc218\uc218, \uc870, \ubc00 \ub4f1\uc774 \uc788\ub2e4. \uacf5\uc5c5\uc5d0 \uc788\uc5b4\uc11c\ub294 \ub3c5\ub9bd \ud6c4 \uc12c\uc720, \uc2dd\ud488, \uc81c\ub2f9, \uc81c\uc9c0, \uc815\uc720, \uc2dc\uba58\ud2b8 \ub4f1\uc758 \uacf5\uc7a5\uc774 \uac74\uc124\ub418\uc5c8\ub2e4. \ud604\uc7ac \uc218\ub2e8\uc5d0\uc11c \uacf5\uc5c5\ud654\uc758 \uc7a5\uc560\uac00 \ub418\uace0 \uc788\ub294 \uac83\uc740 \uc790\ubcf8\uc758 \ubd80\uc871, \uad50\ud1b5\uc2dc\uc124\uc758 \ubbf8\ube44, \uae30\uc220\uc758 \ud6c4\uc9c4\uc131 \ub4f1\uc774\ub2e4. \uc9c0\ud558\uc790\uc6d0\uc73c\ub85c\ub294 \ucca0\uad11\uc11d\u00b7\ub9dd\uac04\u00b7\ub3d9 \ub4f1\uc774 \uc788\ub2e4. 2002\ub144 \uc218\ucd9c 17\uc5b5 1,770\ub9cc \ub2ec\ub7ec, \uc218\uc785 19\uc5b5 1,480\ub9cc \ub2ec\ub7ec\ub97c \uae30\ub85d\ud588\ub2e4. \uc8fc\uc694 \uc218\ucd9c\ud488\uc740 \uba74\ud654\uc640 \ub099\ud654\uc0dd, \uc544\ub77c\ube44\uc544\uace0\ubb34 \ub4f1\uc774\ub2e4. \ub300\ud3ed\uc801\uc778 \ubb34\uc5ed\uc801\uc790\ub294 \uc678\uad6d\uc758 \uac01\uc885 \uc6d0\uc870\uc640 \uc544\ub78d\uc0b0\uc720\uad6d\uc5d0 \ucde8\uc5c5\ud55c \ub178\ub3d9\uc790\uc758 \ubcf8\uad6d \uc1a1\uae08\uc73c\ub85c \uba54\uc6b4\ub2e4.", "sentence_answer": "\uc8fc\uc694 \ub18d\uc0b0\ubb3c\uc740 \uba74\ud654 \uc778\ub370, \uc8fc\ub85c \uc601\uad6d\uc778\uc774 \uac74\uc124\ud55c \uc13c\ub108\ub310\uc5d0 \uc758\ud574 \uad00\uac1c(\u704c\u6f11)\ub41c \ucf00\ub514\ub77c \uc9c0\uad6c\uc5d0\uc11c \uc7ac\ubc30\ub41c\ub2e4.", "paragraph_id": "6657217-1-1", "paragraph_question": "question: \uc218\ub2e8\uc758 \uc8fc\uc694 \uacbd\uc791\ubb3c\uc740 \ubb34\uc5c7\uc778\uac00?, context: 1996-2005\ub144 \uae30\uac04 \ub3d9\uc548 \uc5f0\ud3c9\uade0 6.3%\uc758 \uacbd\uc81c \uc131\uc7a5\uc744 \uae30\ub85d\ud558\uc600\ub2e4. \ucd1d\ub178\ub3d9\uc778\uad6c\uc758 80%\uac00 \ub18d\uc5c5 \ub610\ub294 \ubaa9\ucd95\uc5c5\uc5d0 \uc885\uc0ac\ud558\uace0 \uc788\ub2e4. \uacbd\uc9c0 \uba74\uc801\uc740 800\ub9cc\u33ca\ub85c \uc8fc\ub85c \ub098\uc77c\uac15 \uc720\uc5ed\uc5d0 \ubd84\ud3ec\ud55c\ub2e4. \uc8fc\uc694 \ub18d\uc0b0\ubb3c\uc740 \uba74\ud654\uc778\ub370, \uc8fc\ub85c \uc601\uad6d\uc778\uc774 \uac74\uc124\ud55c \uc13c\ub108\ub310\uc5d0 \uc758\ud574 \uad00\uac1c(\u704c\u6f11)\ub41c \ucf00\ub514\ub77c \uc9c0\uad6c\uc5d0\uc11c \uc7ac\ubc30\ub41c\ub2e4. \uba74\ud654 \uc774\uc678\uc758 \uacbd\uc791\ubb3c\ub85c\ub294 \uc544\ub77c\ube44\uc544\uace0\ubb34, \uae68, \ub099\ud654\uc0dd, \uc0ac\ud0d5\uc218\uc218, \uc870, \ubc00 \ub4f1\uc774 \uc788\ub2e4. \uacf5\uc5c5\uc5d0 \uc788\uc5b4\uc11c\ub294 \ub3c5\ub9bd \ud6c4 \uc12c\uc720, \uc2dd\ud488, \uc81c\ub2f9, \uc81c\uc9c0, \uc815\uc720, \uc2dc\uba58\ud2b8 \ub4f1\uc758 \uacf5\uc7a5\uc774 \uac74\uc124\ub418\uc5c8\ub2e4. \ud604\uc7ac \uc218\ub2e8\uc5d0\uc11c \uacf5\uc5c5\ud654\uc758 \uc7a5\uc560\uac00 \ub418\uace0 \uc788\ub294 \uac83\uc740 \uc790\ubcf8\uc758 \ubd80\uc871, \uad50\ud1b5\uc2dc\uc124\uc758 \ubbf8\ube44, \uae30\uc220\uc758 \ud6c4\uc9c4\uc131 \ub4f1\uc774\ub2e4. \uc9c0\ud558\uc790\uc6d0\uc73c\ub85c\ub294 \ucca0\uad11\uc11d\u00b7\ub9dd\uac04\u00b7\ub3d9 \ub4f1\uc774 \uc788\ub2e4. 2002\ub144 \uc218\ucd9c 17\uc5b5 1,770\ub9cc \ub2ec\ub7ec, \uc218\uc785 19\uc5b5 1,480\ub9cc \ub2ec\ub7ec\ub97c \uae30\ub85d\ud588\ub2e4. \uc8fc\uc694 \uc218\ucd9c\ud488\uc740 \uba74\ud654\uc640 \ub099\ud654\uc0dd, \uc544\ub77c\ube44\uc544\uace0\ubb34 \ub4f1\uc774\ub2e4. \ub300\ud3ed\uc801\uc778 \ubb34\uc5ed\uc801\uc790\ub294 \uc678\uad6d\uc758 \uac01\uc885 \uc6d0\uc870\uc640 \uc544\ub78d\uc0b0\uc720\uad6d\uc5d0 \ucde8\uc5c5\ud55c \ub178\ub3d9\uc790\uc758 \ubcf8\uad6d \uc1a1\uae08\uc73c\ub85c \uba54\uc6b4\ub2e4."} {"question": "T-2 \ud6c4\uacc4\uae30\ub85c\uc11c \uad50\uc721\ube44\ud589\ub300 21\uae30\ub294 \uc0dd\ub3c4 \uad50\uc721\ubcf4\ub2e4 \uc5b4\ub5a4 \uc758\ubbf8\ub97c \ub744\ub294\uac00?", "paragraph": "\ucd94\uac00\ub85c 2002\ub144 8\uc6d4 \uc870\ub2ec \uae30\uc218 \uac10\uc218\uac00 \uc774\ub8e8\uc5b4\uc838 \ube44\ud589\uad50\ub3c4\ub300 \ubc30\uce58 \uc608\uc815\uc778 8\uae30\uac00 \uc804\uae30 \uc18c\uac10, \ud604\uc7a5 \uc18c\ubaa8 \uc608\ube44\uae30\uac00 15\uae30\ub85c \uc18c\uac10\ub418\uc5b4 \ud569\uacc4 32\uae30\ub97c \uc904\uc778 98\uae30\uac00 \uc870\ub2ec\ub418\uc5c8\ub2e4. \uc18c\uac10 \ub300\uc0c1\uc740 \ud604\uc7ac F-15DJ\ub97c \uc0ac\uc6a9\ud558\uace0 \uc788\ub294 \ubd80\ub300\ub85c \uc774\ud6c4\ub3c4 \uc7a5\uae30\uac04 F-15DJ \uc0ac\uc6a9\uc774 \uac00\ub2a5\ud55c \ube44\ud589\uad50\ub3c4\ub300\uc774\ub2e4. \ucd94\uac00\ub85c \uc18c\ubaa8 \ubc0f \uc815\ube44 \ub85c\ud14c\uc774\uc158\uc744 \uc704\ud55c \ud604\uc7ac \uc18c\ubaa8 \uc608\uc815\uae30\ub97c 15\uae30\ub85c \uc555\ucd95\ud558\ub294 \ubc29\ubc95\uc73c\ub85c \uc870\ub2ec\uc218\ub97c \uc18c\uac10\ud588\ub2e4. \ube44\ud589\ub300 \ubc30\uce58 \uc815\uc218 81\uae30\uc5d0 \ube44\ud574 15\uae30\ub294 \ub9e4\uc6b0 \uc801\uc740 \uc22b\uc790\uc774\ub2e4(F-4, F-1 \ub4f1\uc740 \uae30\uccb4 \uc815\uc218 \uc57d 30 \ud37c\uc13c\ud2b8\ub97c \uc608\ube44\uae30\ub85c \uc870\ub2ec\ud588\ub2e4). T-2 \ud6c4\uacc4\uae30\ub85c\uc368 \uad50\uc721\ube44\ud589\ub300 21\uae30\ub294 \ub108\ubb34 \uc801\uc740 \uc22b\uc790\uba70(T-2\ub294 96\uae30 \uc870\ub2ec), \uc0dd\ub3c4 \uad50\uc721\ubcf4\ub2e4\ub294 \uae30\uc885 \uc804\ud658 \ud6c8\ub828\uc6a9\uc774\ub77c\ub294 \uc758\ubbf8\uac00 \ud06c\ub2e4.", "answer": "\uae30\uc885 \uc804\ud658 \ud6c8\ub828\uc6a9", "sentence": "T-2 \ud6c4\uacc4\uae30\ub85c\uc368 \uad50\uc721\ube44\ud589\ub300 21\uae30\ub294 \ub108\ubb34 \uc801\uc740 \uc22b\uc790\uba70(T-2\ub294 96\uae30 \uc870\ub2ec), \uc0dd\ub3c4 \uad50\uc721\ubcf4\ub2e4\ub294 \uae30\uc885 \uc804\ud658 \ud6c8\ub828\uc6a9 \uc774\ub77c\ub294 \uc758\ubbf8\uac00 \ud06c\ub2e4.", "paragraph_sentence": "\ucd94\uac00\ub85c 2002\ub144 8\uc6d4 \uc870\ub2ec \uae30\uc218 \uac10\uc218\uac00 \uc774\ub8e8\uc5b4\uc838 \ube44\ud589\uad50\ub3c4\ub300 \ubc30\uce58 \uc608\uc815\uc778 8\uae30\uac00 \uc804\uae30 \uc18c\uac10, \ud604\uc7a5 \uc18c\ubaa8 \uc608\ube44\uae30\uac00 15\uae30\ub85c \uc18c\uac10\ub418\uc5b4 \ud569\uacc4 32\uae30\ub97c \uc904\uc778 98\uae30\uac00 \uc870\ub2ec\ub418\uc5c8\ub2e4. \uc18c\uac10 \ub300\uc0c1\uc740 \ud604\uc7ac F-15DJ\ub97c \uc0ac\uc6a9\ud558\uace0 \uc788\ub294 \ubd80\ub300\ub85c \uc774\ud6c4\ub3c4 \uc7a5\uae30\uac04 F-15DJ \uc0ac\uc6a9\uc774 \uac00\ub2a5\ud55c \ube44\ud589\uad50\ub3c4\ub300\uc774\ub2e4. \ucd94\uac00\ub85c \uc18c\ubaa8 \ubc0f \uc815\ube44 \ub85c\ud14c\uc774\uc158\uc744 \uc704\ud55c \ud604\uc7ac \uc18c\ubaa8 \uc608\uc815\uae30\ub97c 15\uae30\ub85c \uc555\ucd95\ud558\ub294 \ubc29\ubc95\uc73c\ub85c \uc870\ub2ec\uc218\ub97c \uc18c\uac10\ud588\ub2e4. \ube44\ud589\ub300 \ubc30\uce58 \uc815\uc218 81\uae30\uc5d0 \ube44\ud574 15\uae30\ub294 \ub9e4\uc6b0 \uc801\uc740 \uc22b\uc790\uc774\ub2e4(F-4, F-1 \ub4f1\uc740 \uae30\uccb4 \uc815\uc218 \uc57d 30 \ud37c\uc13c\ud2b8\ub97c \uc608\ube44\uae30\ub85c \uc870\ub2ec\ud588\ub2e4). T-2 \ud6c4\uacc4\uae30\ub85c\uc368 \uad50\uc721\ube44\ud589\ub300 21\uae30\ub294 \ub108\ubb34 \uc801\uc740 \uc22b\uc790\uba70(T-2\ub294 96\uae30 \uc870\ub2ec), \uc0dd\ub3c4 \uad50\uc721\ubcf4\ub2e4\ub294 \uae30\uc885 \uc804\ud658 \ud6c8\ub828\uc6a9 \uc774\ub77c\ub294 \uc758\ubbf8\uac00 \ud06c\ub2e4. ", "paragraph_answer": "\ucd94\uac00\ub85c 2002\ub144 8\uc6d4 \uc870\ub2ec \uae30\uc218 \uac10\uc218\uac00 \uc774\ub8e8\uc5b4\uc838 \ube44\ud589\uad50\ub3c4\ub300 \ubc30\uce58 \uc608\uc815\uc778 8\uae30\uac00 \uc804\uae30 \uc18c\uac10, \ud604\uc7a5 \uc18c\ubaa8 \uc608\ube44\uae30\uac00 15\uae30\ub85c \uc18c\uac10\ub418\uc5b4 \ud569\uacc4 32\uae30\ub97c \uc904\uc778 98\uae30\uac00 \uc870\ub2ec\ub418\uc5c8\ub2e4. \uc18c\uac10 \ub300\uc0c1\uc740 \ud604\uc7ac F-15DJ\ub97c \uc0ac\uc6a9\ud558\uace0 \uc788\ub294 \ubd80\ub300\ub85c \uc774\ud6c4\ub3c4 \uc7a5\uae30\uac04 F-15DJ \uc0ac\uc6a9\uc774 \uac00\ub2a5\ud55c \ube44\ud589\uad50\ub3c4\ub300\uc774\ub2e4. \ucd94\uac00\ub85c \uc18c\ubaa8 \ubc0f \uc815\ube44 \ub85c\ud14c\uc774\uc158\uc744 \uc704\ud55c \ud604\uc7ac \uc18c\ubaa8 \uc608\uc815\uae30\ub97c 15\uae30\ub85c \uc555\ucd95\ud558\ub294 \ubc29\ubc95\uc73c\ub85c \uc870\ub2ec\uc218\ub97c \uc18c\uac10\ud588\ub2e4. \ube44\ud589\ub300 \ubc30\uce58 \uc815\uc218 81\uae30\uc5d0 \ube44\ud574 15\uae30\ub294 \ub9e4\uc6b0 \uc801\uc740 \uc22b\uc790\uc774\ub2e4(F-4, F-1 \ub4f1\uc740 \uae30\uccb4 \uc815\uc218 \uc57d 30 \ud37c\uc13c\ud2b8\ub97c \uc608\ube44\uae30\ub85c \uc870\ub2ec\ud588\ub2e4). T-2 \ud6c4\uacc4\uae30\ub85c\uc368 \uad50\uc721\ube44\ud589\ub300 21\uae30\ub294 \ub108\ubb34 \uc801\uc740 \uc22b\uc790\uba70(T-2\ub294 96\uae30 \uc870\ub2ec), \uc0dd\ub3c4 \uad50\uc721\ubcf4\ub2e4\ub294 \uae30\uc885 \uc804\ud658 \ud6c8\ub828\uc6a9 \uc774\ub77c\ub294 \uc758\ubbf8\uac00 \ud06c\ub2e4.", "sentence_answer": "T-2 \ud6c4\uacc4\uae30\ub85c\uc368 \uad50\uc721\ube44\ud589\ub300 21\uae30\ub294 \ub108\ubb34 \uc801\uc740 \uc22b\uc790\uba70(T-2\ub294 96\uae30 \uc870\ub2ec), \uc0dd\ub3c4 \uad50\uc721\ubcf4\ub2e4\ub294 \uae30\uc885 \uc804\ud658 \ud6c8\ub828\uc6a9 \uc774\ub77c\ub294 \uc758\ubbf8\uac00 \ud06c\ub2e4.", "paragraph_id": "6515252-18-1", "paragraph_question": "question: T-2 \ud6c4\uacc4\uae30\ub85c\uc11c \uad50\uc721\ube44\ud589\ub300 21\uae30\ub294 \uc0dd\ub3c4 \uad50\uc721\ubcf4\ub2e4 \uc5b4\ub5a4 \uc758\ubbf8\ub97c \ub744\ub294\uac00?, context: \ucd94\uac00\ub85c 2002\ub144 8\uc6d4 \uc870\ub2ec \uae30\uc218 \uac10\uc218\uac00 \uc774\ub8e8\uc5b4\uc838 \ube44\ud589\uad50\ub3c4\ub300 \ubc30\uce58 \uc608\uc815\uc778 8\uae30\uac00 \uc804\uae30 \uc18c\uac10, \ud604\uc7a5 \uc18c\ubaa8 \uc608\ube44\uae30\uac00 15\uae30\ub85c \uc18c\uac10\ub418\uc5b4 \ud569\uacc4 32\uae30\ub97c \uc904\uc778 98\uae30\uac00 \uc870\ub2ec\ub418\uc5c8\ub2e4. \uc18c\uac10 \ub300\uc0c1\uc740 \ud604\uc7ac F-15DJ\ub97c \uc0ac\uc6a9\ud558\uace0 \uc788\ub294 \ubd80\ub300\ub85c \uc774\ud6c4\ub3c4 \uc7a5\uae30\uac04 F-15DJ \uc0ac\uc6a9\uc774 \uac00\ub2a5\ud55c \ube44\ud589\uad50\ub3c4\ub300\uc774\ub2e4. \ucd94\uac00\ub85c \uc18c\ubaa8 \ubc0f \uc815\ube44 \ub85c\ud14c\uc774\uc158\uc744 \uc704\ud55c \ud604\uc7ac \uc18c\ubaa8 \uc608\uc815\uae30\ub97c 15\uae30\ub85c \uc555\ucd95\ud558\ub294 \ubc29\ubc95\uc73c\ub85c \uc870\ub2ec\uc218\ub97c \uc18c\uac10\ud588\ub2e4. \ube44\ud589\ub300 \ubc30\uce58 \uc815\uc218 81\uae30\uc5d0 \ube44\ud574 15\uae30\ub294 \ub9e4\uc6b0 \uc801\uc740 \uc22b\uc790\uc774\ub2e4(F-4, F-1 \ub4f1\uc740 \uae30\uccb4 \uc815\uc218 \uc57d 30 \ud37c\uc13c\ud2b8\ub97c \uc608\ube44\uae30\ub85c \uc870\ub2ec\ud588\ub2e4). T-2 \ud6c4\uacc4\uae30\ub85c\uc368 \uad50\uc721\ube44\ud589\ub300 21\uae30\ub294 \ub108\ubb34 \uc801\uc740 \uc22b\uc790\uba70(T-2\ub294 96\uae30 \uc870\ub2ec), \uc0dd\ub3c4 \uad50\uc721\ubcf4\ub2e4\ub294 \uae30\uc885 \uc804\ud658 \ud6c8\ub828\uc6a9\uc774\ub77c\ub294 \uc758\ubbf8\uac00 \ud06c\ub2e4."} {"question": "\ub2f9\uc2dc\uc758 \uc218\ub3c4\uc5d0 \uc124\uce58\ub418\uc5b4 \uc911\uad6d\uc758 \uc784\uc758\uc758 \uc704\uce58\uc5d0\uc11c \uc9c0\uc9c4\uc774 \uc77c\uc5b4\ub098\uba74 \uc9c4\uc559\uc9c0\uc758 \ubc29\ud5a5\uc744 \uc54c\ub824\uc900 \uc774 \uae30\uacc4\uc758 \uc774\ub984\uc740?", "paragraph": "\uc9c0\uc9c8\ud559: \ud070 \uc7ac\ub09c\uc5d0 \ub300\ube44\ud558\uae30 \uc704\ud574\uc11c \uc7a5\ud615\uc740 \uc9c0\ub3d9\uc758\ub97c \uac1c\ubc1c\ud588\ub2e4. \uc774 \uae30\uacc4\ub294 \ub2f9\uc2dc\uc758 \uc218\ub3c4(\u9996\u90fd)\uc5d0 \uc124\uce58\ub418\uc5b4 \uc788\uc5c8\uc73c\uba70, \uc911\uad6d\uc758 \uc784\uc758\uc758 \uc704\uce58\uc5d0\uc11c \uc9c0\uc9c4\uc774 \uc77c\uc5b4\ub098\uba74 \uc774 \uae30\uacc4\ub294 \uc9c4\uc559\uc9c0\uc758 \ubc29\ud5a5\uc744 \uc54c\ub824\uc8fc\uc5c8\ub2e4. \uc2e4\uc81c\ub85c \uc7a5\ud615\uc758 \uc9c0\ub3d9\uc758\uac00 \uc791\ub3d9\ud588\uc9c0\ub9cc \uc218\ub3c4 \uc9c0\uc5ed\uc5d0\uc11c\ub294 \uc9c4\ub3d9\uc774 \uc804\ud600 \uac10\uc9c0\ub418\uc9c0 \uc54a\uc544 \uc9c0\uc9c4\uc758 \ubc1c\uc0dd\uc0ac\uc2e4\uc5d0 \ub300\ud574 \ub9ce\uc740 \uad00\ub9ac\ub4e4\uc774 \uc758\uc544\ud574 \ud588\uc73c\ub098, \uc7a5\ud615\uc740 \uc9c0\ub3d9\uc758\uc758 \uc791\ub3d9\uacb0\uacfc\ub97c \ubd84\uc11d\ud558\uc5ec \ubd81\uc11c\uc9c0\ubc29\uc5d0\uc11c \uc9c0\uc9c4\uc774 \ubc1c\uc0dd\ud588\ub2e4\uace0 \uc815\ubd80\uae30\uad00\uc5d0 \ubcf4\uace0\ud588\uace0, \uc5bc\ub9c8 \ub4a4 \uc9c0\uc9c4 \ubc1c\uc0dd\uc744 \uc54c\ub9ac\ub294 \ud30c\ubc1c\uc774 \ub904\uc591\uc9c0\ubc29\uc73c\ub85c \ubd80\ud130 \ub3c4\ucc29\ud588\ub2e4. \uc7a5\ud615\uc740 \uadf8\uc758 \uc7a5\ube44\ub97c \u2018\uc9c0\uad6c\uc758 \uc6c0\uc9c1\uc784\uacfc \uacc4\uc808\uc758 \ud48d\ud5a5\uc744 \uce21\uc815\ud558\ub294 \uae30\uad6c\u2019\ub77c \uc124\uba85\ud588\ub294\ub370, \uc774\ub7ec\ud55c \uc124\uba85\uc740 \ub2f9\uc2dc\uc5d0 \uc9c0\uc9c4\uc774 \uadf9\ub3c4\ub85c \uc555\ucd95\ub41c \uacf5\uae30\uc5d0 \uc758\ud574 \ubc1c\uc0dd\ud55c\ub2e4\ub294 \uc778\uc2dd\uc5d0\uc11c \uae30\uc778\ud55c \uac83\uc774\uc5c8\ub2e4.", "answer": "\uc9c0\ub3d9\uc758", "sentence": "\uc9c0\uc9c8\ud559: \ud070 \uc7ac\ub09c\uc5d0 \ub300\ube44\ud558\uae30 \uc704\ud574\uc11c \uc7a5\ud615\uc740 \uc9c0\ub3d9\uc758 \ub97c \uac1c\ubc1c\ud588\ub2e4.", "paragraph_sentence": " \uc9c0\uc9c8\ud559: \ud070 \uc7ac\ub09c\uc5d0 \ub300\ube44\ud558\uae30 \uc704\ud574\uc11c \uc7a5\ud615\uc740 \uc9c0\ub3d9\uc758 \ub97c \uac1c\ubc1c\ud588\ub2e4. \uc774 \uae30\uacc4\ub294 \ub2f9\uc2dc\uc758 \uc218\ub3c4(\u9996\u90fd)\uc5d0 \uc124\uce58\ub418\uc5b4 \uc788\uc5c8\uc73c\uba70, \uc911\uad6d\uc758 \uc784\uc758\uc758 \uc704\uce58\uc5d0\uc11c \uc9c0\uc9c4\uc774 \uc77c\uc5b4\ub098\uba74 \uc774 \uae30\uacc4\ub294 \uc9c4\uc559\uc9c0\uc758 \ubc29\ud5a5\uc744 \uc54c\ub824\uc8fc\uc5c8\ub2e4. \uc2e4\uc81c\ub85c \uc7a5\ud615\uc758 \uc9c0\ub3d9\uc758\uac00 \uc791\ub3d9\ud588\uc9c0\ub9cc \uc218\ub3c4 \uc9c0\uc5ed\uc5d0\uc11c\ub294 \uc9c4\ub3d9\uc774 \uc804\ud600 \uac10\uc9c0\ub418\uc9c0 \uc54a\uc544 \uc9c0\uc9c4\uc758 \ubc1c\uc0dd\uc0ac\uc2e4\uc5d0 \ub300\ud574 \ub9ce\uc740 \uad00\ub9ac\ub4e4\uc774 \uc758\uc544\ud574 \ud588\uc73c\ub098, \uc7a5\ud615\uc740 \uc9c0\ub3d9\uc758\uc758 \uc791\ub3d9\uacb0\uacfc\ub97c \ubd84\uc11d\ud558\uc5ec \ubd81\uc11c\uc9c0\ubc29\uc5d0\uc11c \uc9c0\uc9c4\uc774 \ubc1c\uc0dd\ud588\ub2e4\uace0 \uc815\ubd80\uae30\uad00\uc5d0 \ubcf4\uace0\ud588\uace0, \uc5bc\ub9c8 \ub4a4 \uc9c0\uc9c4 \ubc1c\uc0dd\uc744 \uc54c\ub9ac\ub294 \ud30c\ubc1c\uc774 \ub904\uc591\uc9c0\ubc29\uc73c\ub85c \ubd80\ud130 \ub3c4\ucc29\ud588\ub2e4. \uc7a5\ud615\uc740 \uadf8\uc758 \uc7a5\ube44\ub97c \u2018\uc9c0\uad6c\uc758 \uc6c0\uc9c1\uc784\uacfc \uacc4\uc808\uc758 \ud48d\ud5a5\uc744 \uce21\uc815\ud558\ub294 \uae30\uad6c\u2019\ub77c \uc124\uba85\ud588\ub294\ub370, \uc774\ub7ec\ud55c \uc124\uba85\uc740 \ub2f9\uc2dc\uc5d0 \uc9c0\uc9c4\uc774 \uadf9\ub3c4\ub85c \uc555\ucd95\ub41c \uacf5\uae30\uc5d0 \uc758\ud574 \ubc1c\uc0dd\ud55c\ub2e4\ub294 \uc778\uc2dd\uc5d0\uc11c \uae30\uc778\ud55c \uac83\uc774\uc5c8\ub2e4.", "paragraph_answer": "\uc9c0\uc9c8\ud559: \ud070 \uc7ac\ub09c\uc5d0 \ub300\ube44\ud558\uae30 \uc704\ud574\uc11c \uc7a5\ud615\uc740 \uc9c0\ub3d9\uc758 \ub97c \uac1c\ubc1c\ud588\ub2e4. \uc774 \uae30\uacc4\ub294 \ub2f9\uc2dc\uc758 \uc218\ub3c4(\u9996\u90fd)\uc5d0 \uc124\uce58\ub418\uc5b4 \uc788\uc5c8\uc73c\uba70, \uc911\uad6d\uc758 \uc784\uc758\uc758 \uc704\uce58\uc5d0\uc11c \uc9c0\uc9c4\uc774 \uc77c\uc5b4\ub098\uba74 \uc774 \uae30\uacc4\ub294 \uc9c4\uc559\uc9c0\uc758 \ubc29\ud5a5\uc744 \uc54c\ub824\uc8fc\uc5c8\ub2e4. \uc2e4\uc81c\ub85c \uc7a5\ud615\uc758 \uc9c0\ub3d9\uc758\uac00 \uc791\ub3d9\ud588\uc9c0\ub9cc \uc218\ub3c4 \uc9c0\uc5ed\uc5d0\uc11c\ub294 \uc9c4\ub3d9\uc774 \uc804\ud600 \uac10\uc9c0\ub418\uc9c0 \uc54a\uc544 \uc9c0\uc9c4\uc758 \ubc1c\uc0dd\uc0ac\uc2e4\uc5d0 \ub300\ud574 \ub9ce\uc740 \uad00\ub9ac\ub4e4\uc774 \uc758\uc544\ud574 \ud588\uc73c\ub098, \uc7a5\ud615\uc740 \uc9c0\ub3d9\uc758\uc758 \uc791\ub3d9\uacb0\uacfc\ub97c \ubd84\uc11d\ud558\uc5ec \ubd81\uc11c\uc9c0\ubc29\uc5d0\uc11c \uc9c0\uc9c4\uc774 \ubc1c\uc0dd\ud588\ub2e4\uace0 \uc815\ubd80\uae30\uad00\uc5d0 \ubcf4\uace0\ud588\uace0, \uc5bc\ub9c8 \ub4a4 \uc9c0\uc9c4 \ubc1c\uc0dd\uc744 \uc54c\ub9ac\ub294 \ud30c\ubc1c\uc774 \ub904\uc591\uc9c0\ubc29\uc73c\ub85c \ubd80\ud130 \ub3c4\ucc29\ud588\ub2e4. \uc7a5\ud615\uc740 \uadf8\uc758 \uc7a5\ube44\ub97c \u2018\uc9c0\uad6c\uc758 \uc6c0\uc9c1\uc784\uacfc \uacc4\uc808\uc758 \ud48d\ud5a5\uc744 \uce21\uc815\ud558\ub294 \uae30\uad6c\u2019\ub77c \uc124\uba85\ud588\ub294\ub370, \uc774\ub7ec\ud55c \uc124\uba85\uc740 \ub2f9\uc2dc\uc5d0 \uc9c0\uc9c4\uc774 \uadf9\ub3c4\ub85c \uc555\ucd95\ub41c \uacf5\uae30\uc5d0 \uc758\ud574 \ubc1c\uc0dd\ud55c\ub2e4\ub294 \uc778\uc2dd\uc5d0\uc11c \uae30\uc778\ud55c \uac83\uc774\uc5c8\ub2e4.", "sentence_answer": "\uc9c0\uc9c8\ud559: \ud070 \uc7ac\ub09c\uc5d0 \ub300\ube44\ud558\uae30 \uc704\ud574\uc11c \uc7a5\ud615\uc740 \uc9c0\ub3d9\uc758 \ub97c \uac1c\ubc1c\ud588\ub2e4.", "paragraph_id": "6489929-9-0", "paragraph_question": "question: \ub2f9\uc2dc\uc758 \uc218\ub3c4\uc5d0 \uc124\uce58\ub418\uc5b4 \uc911\uad6d\uc758 \uc784\uc758\uc758 \uc704\uce58\uc5d0\uc11c \uc9c0\uc9c4\uc774 \uc77c\uc5b4\ub098\uba74 \uc9c4\uc559\uc9c0\uc758 \ubc29\ud5a5\uc744 \uc54c\ub824\uc900 \uc774 \uae30\uacc4\uc758 \uc774\ub984\uc740?, context: \uc9c0\uc9c8\ud559: \ud070 \uc7ac\ub09c\uc5d0 \ub300\ube44\ud558\uae30 \uc704\ud574\uc11c \uc7a5\ud615\uc740 \uc9c0\ub3d9\uc758\ub97c \uac1c\ubc1c\ud588\ub2e4. \uc774 \uae30\uacc4\ub294 \ub2f9\uc2dc\uc758 \uc218\ub3c4(\u9996\u90fd)\uc5d0 \uc124\uce58\ub418\uc5b4 \uc788\uc5c8\uc73c\uba70, \uc911\uad6d\uc758 \uc784\uc758\uc758 \uc704\uce58\uc5d0\uc11c \uc9c0\uc9c4\uc774 \uc77c\uc5b4\ub098\uba74 \uc774 \uae30\uacc4\ub294 \uc9c4\uc559\uc9c0\uc758 \ubc29\ud5a5\uc744 \uc54c\ub824\uc8fc\uc5c8\ub2e4. \uc2e4\uc81c\ub85c \uc7a5\ud615\uc758 \uc9c0\ub3d9\uc758\uac00 \uc791\ub3d9\ud588\uc9c0\ub9cc \uc218\ub3c4 \uc9c0\uc5ed\uc5d0\uc11c\ub294 \uc9c4\ub3d9\uc774 \uc804\ud600 \uac10\uc9c0\ub418\uc9c0 \uc54a\uc544 \uc9c0\uc9c4\uc758 \ubc1c\uc0dd\uc0ac\uc2e4\uc5d0 \ub300\ud574 \ub9ce\uc740 \uad00\ub9ac\ub4e4\uc774 \uc758\uc544\ud574 \ud588\uc73c\ub098, \uc7a5\ud615\uc740 \uc9c0\ub3d9\uc758\uc758 \uc791\ub3d9\uacb0\uacfc\ub97c \ubd84\uc11d\ud558\uc5ec \ubd81\uc11c\uc9c0\ubc29\uc5d0\uc11c \uc9c0\uc9c4\uc774 \ubc1c\uc0dd\ud588\ub2e4\uace0 \uc815\ubd80\uae30\uad00\uc5d0 \ubcf4\uace0\ud588\uace0, \uc5bc\ub9c8 \ub4a4 \uc9c0\uc9c4 \ubc1c\uc0dd\uc744 \uc54c\ub9ac\ub294 \ud30c\ubc1c\uc774 \ub904\uc591\uc9c0\ubc29\uc73c\ub85c \ubd80\ud130 \ub3c4\ucc29\ud588\ub2e4. \uc7a5\ud615\uc740 \uadf8\uc758 \uc7a5\ube44\ub97c \u2018\uc9c0\uad6c\uc758 \uc6c0\uc9c1\uc784\uacfc \uacc4\uc808\uc758 \ud48d\ud5a5\uc744 \uce21\uc815\ud558\ub294 \uae30\uad6c\u2019\ub77c \uc124\uba85\ud588\ub294\ub370, \uc774\ub7ec\ud55c \uc124\uba85\uc740 \ub2f9\uc2dc\uc5d0 \uc9c0\uc9c4\uc774 \uadf9\ub3c4\ub85c \uc555\ucd95\ub41c \uacf5\uae30\uc5d0 \uc758\ud574 \ubc1c\uc0dd\ud55c\ub2e4\ub294 \uc778\uc2dd\uc5d0\uc11c \uae30\uc778\ud55c \uac83\uc774\uc5c8\ub2e4."} {"question": "\uc7a5\uae30\ud558\uc758 \uccab \uc815\uaddc \uc74c\ubc18\uc758 \uc774\ub984\uc740 \ubb34\uc5c7\uc778\uac00?", "paragraph": "\uc7a5\uae30\ud558\ub294 2008\ub144 12\uc6d4 31\uc77c \uc790 \u300a\ud55c\uaca8\ub808\u300b \uae30\uc0ac\uc5d0\uc11c \"2\uc6d4 \ub9d0\uc5d0 \uc7a5\uae30\ud558\uc640 \uc5bc\uad74\ub4e4 \uc815\uaddc \uc74c\ubc18\uc774 \ub098\uc628\ub2e4.\"\uace0 \uc5b8\uae09\ud588\ub2e4. \uc18c\uc18d\uc0ac \ubd95\uac00\ubd95\uac00\ub808\ucf54\ub4dc \uad00\uacc4\uc790\ub294 2009\ub144 2\uc6d4 3\uc77c\uc5d0 \"\uc7a5\uae30\ud558\uc640 \uc5bc\uad74\ub4e4\uc758 \uc815\uaddc 1\uc9d1\uc774 \uc624\ub294 27\uc77c \ubc1c\ub9e4\ub420 \uc608\uc815\"\uc774\ub77c\uba70 \"\ubc1c\ub9e4\uc77c\uc5d0 \ub9de\ucdb0 \ubc1c\ub9e4 \uae30\ub150\uacf5\uc5f0\ub3c4 \ud568\uaed8 \uc5f0\ub2e4\"\uace0 \ubc1c\ub9e4 \uc77c\uc790\ub97c \ud655\uc815\uc9c0\uc5c8\ub2e4. \uc774 \uad00\uacc4\uc790\ub294 \"\ud604\uc7ac \ub9c8\ubb34\ub9ac \uc74c\ubc18 \uc791\uc5c5\uc744 \uc9c4\ud589 \uc911\"\uc774\ub77c\uba70 \"\uc74c\ubc18 \ud0c0\uc774\ud2c0\uacfc \uc218\ub85d\uace1 \uc804\ubd80\uac00 \uacb0\uc815\ub41c \uc0c1\ud0dc\ub294 \uc544\ub2c8\ub2e4\"\ub77c\uace0 \ub367\ubd99\uc600\ub2e4. \uc7a5\uae30\ud558\ub294 \uc608\uc2a424 \uc778\ud130\ubdf0\uc5d0\uc11c \"\uc0c1\uc0c1\ub9c8\ub2f9\uc5d0\uc11c 2\uc6d4 27\uc77c \uacf5\uc5f0 \uacb8 \uc1fc\ucf00\uc774\uc2a4\uac00 \uc7a1\ud600 \uc788\uc5b4\uc11c \uc790\uc5f0\uc2a4\ub7fd\uac8c 1\uc9d1 \ub9c8\uac10 \uc2dc\ud55c\uc774 \uacb0\uc815\ub418\uc5c8\ub2e4.\"\uace0 \ubc1d\ud614\ub2e4. \uccab \uc815\uaddc \uc74c\ubc18\uc778 \ub9cc\ud07c \u300a\ubcc4\uc77c \uc5c6\uc774 \uc0b0\ub2e4\u300b\ub294 \uac00\ub0b4\uc218\uacf5\uc5c5\uc744 \ud1b5\ud55c \uc18c\ub7c9 \uc0dd\uc0b0\ubc29\uc2dd\uc744 \ud0dd\ud558\uc9c0 \uc54a\uace0 \uacf5\uc7a5\uc5d0\uc11c \ucc0d\uc5b4\ub0c8\ub2e4.", "answer": "\ubcc4\uc77c \uc5c6\uc774 \uc0b0\ub2e4", "sentence": "\uccab \uc815\uaddc \uc74c\ubc18\uc778 \ub9cc\ud07c \u300a \ubcc4\uc77c \uc5c6\uc774 \uc0b0\ub2e4 \u300b\ub294 \uac00\ub0b4\uc218\uacf5\uc5c5\uc744 \ud1b5\ud55c \uc18c\ub7c9 \uc0dd\uc0b0\ubc29\uc2dd\uc744 \ud0dd\ud558\uc9c0 \uc54a\uace0 \uacf5\uc7a5\uc5d0\uc11c \ucc0d\uc5b4\ub0c8\ub2e4.", "paragraph_sentence": "\uc7a5\uae30\ud558\ub294 2008\ub144 12\uc6d4 31\uc77c \uc790 \u300a\ud55c\uaca8\ub808\u300b \uae30\uc0ac\uc5d0\uc11c \"2\uc6d4 \ub9d0\uc5d0 \uc7a5\uae30\ud558\uc640 \uc5bc\uad74\ub4e4 \uc815\uaddc \uc74c\ubc18\uc774 \ub098\uc628\ub2e4. \"\uace0 \uc5b8\uae09\ud588\ub2e4. \uc18c\uc18d\uc0ac \ubd95\uac00\ubd95\uac00\ub808\ucf54\ub4dc \uad00\uacc4\uc790\ub294 2009\ub144 2\uc6d4 3\uc77c\uc5d0 \"\uc7a5\uae30\ud558\uc640 \uc5bc\uad74\ub4e4\uc758 \uc815\uaddc 1\uc9d1\uc774 \uc624\ub294 27\uc77c \ubc1c\ub9e4\ub420 \uc608\uc815\"\uc774\ub77c\uba70 \"\ubc1c\ub9e4\uc77c\uc5d0 \ub9de\ucdb0 \ubc1c\ub9e4 \uae30\ub150\uacf5\uc5f0\ub3c4 \ud568\uaed8 \uc5f0\ub2e4\"\uace0 \ubc1c\ub9e4 \uc77c\uc790\ub97c \ud655\uc815\uc9c0\uc5c8\ub2e4. \uc774 \uad00\uacc4\uc790\ub294 \"\ud604\uc7ac \ub9c8\ubb34\ub9ac \uc74c\ubc18 \uc791\uc5c5\uc744 \uc9c4\ud589 \uc911\"\uc774\ub77c\uba70 \"\uc74c\ubc18 \ud0c0\uc774\ud2c0\uacfc \uc218\ub85d\uace1 \uc804\ubd80\uac00 \uacb0\uc815\ub41c \uc0c1\ud0dc\ub294 \uc544\ub2c8\ub2e4\"\ub77c\uace0 \ub367\ubd99\uc600\ub2e4. \uc7a5\uae30\ud558\ub294 \uc608\uc2a424 \uc778\ud130\ubdf0\uc5d0\uc11c \"\uc0c1\uc0c1\ub9c8\ub2f9\uc5d0\uc11c 2\uc6d4 27\uc77c \uacf5\uc5f0 \uacb8 \uc1fc\ucf00\uc774\uc2a4\uac00 \uc7a1\ud600 \uc788\uc5b4\uc11c \uc790\uc5f0\uc2a4\ub7fd\uac8c 1\uc9d1 \ub9c8\uac10 \uc2dc\ud55c\uc774 \uacb0\uc815\ub418\uc5c8\ub2e4. \"\uace0 \ubc1d\ud614\ub2e4. \uccab \uc815\uaddc \uc74c\ubc18\uc778 \ub9cc\ud07c \u300a \ubcc4\uc77c \uc5c6\uc774 \uc0b0\ub2e4 \u300b\ub294 \uac00\ub0b4\uc218\uacf5\uc5c5\uc744 \ud1b5\ud55c \uc18c\ub7c9 \uc0dd\uc0b0\ubc29\uc2dd\uc744 \ud0dd\ud558\uc9c0 \uc54a\uace0 \uacf5\uc7a5\uc5d0\uc11c \ucc0d\uc5b4\ub0c8\ub2e4. ", "paragraph_answer": "\uc7a5\uae30\ud558\ub294 2008\ub144 12\uc6d4 31\uc77c \uc790 \u300a\ud55c\uaca8\ub808\u300b \uae30\uc0ac\uc5d0\uc11c \"2\uc6d4 \ub9d0\uc5d0 \uc7a5\uae30\ud558\uc640 \uc5bc\uad74\ub4e4 \uc815\uaddc \uc74c\ubc18\uc774 \ub098\uc628\ub2e4.\"\uace0 \uc5b8\uae09\ud588\ub2e4. \uc18c\uc18d\uc0ac \ubd95\uac00\ubd95\uac00\ub808\ucf54\ub4dc \uad00\uacc4\uc790\ub294 2009\ub144 2\uc6d4 3\uc77c\uc5d0 \"\uc7a5\uae30\ud558\uc640 \uc5bc\uad74\ub4e4\uc758 \uc815\uaddc 1\uc9d1\uc774 \uc624\ub294 27\uc77c \ubc1c\ub9e4\ub420 \uc608\uc815\"\uc774\ub77c\uba70 \"\ubc1c\ub9e4\uc77c\uc5d0 \ub9de\ucdb0 \ubc1c\ub9e4 \uae30\ub150\uacf5\uc5f0\ub3c4 \ud568\uaed8 \uc5f0\ub2e4\"\uace0 \ubc1c\ub9e4 \uc77c\uc790\ub97c \ud655\uc815\uc9c0\uc5c8\ub2e4. \uc774 \uad00\uacc4\uc790\ub294 \"\ud604\uc7ac \ub9c8\ubb34\ub9ac \uc74c\ubc18 \uc791\uc5c5\uc744 \uc9c4\ud589 \uc911\"\uc774\ub77c\uba70 \"\uc74c\ubc18 \ud0c0\uc774\ud2c0\uacfc \uc218\ub85d\uace1 \uc804\ubd80\uac00 \uacb0\uc815\ub41c \uc0c1\ud0dc\ub294 \uc544\ub2c8\ub2e4\"\ub77c\uace0 \ub367\ubd99\uc600\ub2e4. \uc7a5\uae30\ud558\ub294 \uc608\uc2a424 \uc778\ud130\ubdf0\uc5d0\uc11c \"\uc0c1\uc0c1\ub9c8\ub2f9\uc5d0\uc11c 2\uc6d4 27\uc77c \uacf5\uc5f0 \uacb8 \uc1fc\ucf00\uc774\uc2a4\uac00 \uc7a1\ud600 \uc788\uc5b4\uc11c \uc790\uc5f0\uc2a4\ub7fd\uac8c 1\uc9d1 \ub9c8\uac10 \uc2dc\ud55c\uc774 \uacb0\uc815\ub418\uc5c8\ub2e4.\"\uace0 \ubc1d\ud614\ub2e4. \uccab \uc815\uaddc \uc74c\ubc18\uc778 \ub9cc\ud07c \u300a \ubcc4\uc77c \uc5c6\uc774 \uc0b0\ub2e4 \u300b\ub294 \uac00\ub0b4\uc218\uacf5\uc5c5\uc744 \ud1b5\ud55c \uc18c\ub7c9 \uc0dd\uc0b0\ubc29\uc2dd\uc744 \ud0dd\ud558\uc9c0 \uc54a\uace0 \uacf5\uc7a5\uc5d0\uc11c \ucc0d\uc5b4\ub0c8\ub2e4.", "sentence_answer": "\uccab \uc815\uaddc \uc74c\ubc18\uc778 \ub9cc\ud07c \u300a \ubcc4\uc77c \uc5c6\uc774 \uc0b0\ub2e4 \u300b\ub294 \uac00\ub0b4\uc218\uacf5\uc5c5\uc744 \ud1b5\ud55c \uc18c\ub7c9 \uc0dd\uc0b0\ubc29\uc2dd\uc744 \ud0dd\ud558\uc9c0 \uc54a\uace0 \uacf5\uc7a5\uc5d0\uc11c \ucc0d\uc5b4\ub0c8\ub2e4.", "paragraph_id": "6530311-1-0", "paragraph_question": "question: \uc7a5\uae30\ud558\uc758 \uccab \uc815\uaddc \uc74c\ubc18\uc758 \uc774\ub984\uc740 \ubb34\uc5c7\uc778\uac00?, context: \uc7a5\uae30\ud558\ub294 2008\ub144 12\uc6d4 31\uc77c \uc790 \u300a\ud55c\uaca8\ub808\u300b \uae30\uc0ac\uc5d0\uc11c \"2\uc6d4 \ub9d0\uc5d0 \uc7a5\uae30\ud558\uc640 \uc5bc\uad74\ub4e4 \uc815\uaddc \uc74c\ubc18\uc774 \ub098\uc628\ub2e4.\"\uace0 \uc5b8\uae09\ud588\ub2e4. \uc18c\uc18d\uc0ac \ubd95\uac00\ubd95\uac00\ub808\ucf54\ub4dc \uad00\uacc4\uc790\ub294 2009\ub144 2\uc6d4 3\uc77c\uc5d0 \"\uc7a5\uae30\ud558\uc640 \uc5bc\uad74\ub4e4\uc758 \uc815\uaddc 1\uc9d1\uc774 \uc624\ub294 27\uc77c \ubc1c\ub9e4\ub420 \uc608\uc815\"\uc774\ub77c\uba70 \"\ubc1c\ub9e4\uc77c\uc5d0 \ub9de\ucdb0 \ubc1c\ub9e4 \uae30\ub150\uacf5\uc5f0\ub3c4 \ud568\uaed8 \uc5f0\ub2e4\"\uace0 \ubc1c\ub9e4 \uc77c\uc790\ub97c \ud655\uc815\uc9c0\uc5c8\ub2e4. \uc774 \uad00\uacc4\uc790\ub294 \"\ud604\uc7ac \ub9c8\ubb34\ub9ac \uc74c\ubc18 \uc791\uc5c5\uc744 \uc9c4\ud589 \uc911\"\uc774\ub77c\uba70 \"\uc74c\ubc18 \ud0c0\uc774\ud2c0\uacfc \uc218\ub85d\uace1 \uc804\ubd80\uac00 \uacb0\uc815\ub41c \uc0c1\ud0dc\ub294 \uc544\ub2c8\ub2e4\"\ub77c\uace0 \ub367\ubd99\uc600\ub2e4. \uc7a5\uae30\ud558\ub294 \uc608\uc2a424 \uc778\ud130\ubdf0\uc5d0\uc11c \"\uc0c1\uc0c1\ub9c8\ub2f9\uc5d0\uc11c 2\uc6d4 27\uc77c \uacf5\uc5f0 \uacb8 \uc1fc\ucf00\uc774\uc2a4\uac00 \uc7a1\ud600 \uc788\uc5b4\uc11c \uc790\uc5f0\uc2a4\ub7fd\uac8c 1\uc9d1 \ub9c8\uac10 \uc2dc\ud55c\uc774 \uacb0\uc815\ub418\uc5c8\ub2e4.\"\uace0 \ubc1d\ud614\ub2e4. \uccab \uc815\uaddc \uc74c\ubc18\uc778 \ub9cc\ud07c \u300a\ubcc4\uc77c \uc5c6\uc774 \uc0b0\ub2e4\u300b\ub294 \uac00\ub0b4\uc218\uacf5\uc5c5\uc744 \ud1b5\ud55c \uc18c\ub7c9 \uc0dd\uc0b0\ubc29\uc2dd\uc744 \ud0dd\ud558\uc9c0 \uc54a\uace0 \uacf5\uc7a5\uc5d0\uc11c \ucc0d\uc5b4\ub0c8\ub2e4."} {"question": "PAOK\uc758 \ub300\uc8fc\uc8fc \uc774\ubc18 \uc0ac\ube44\ub514\uc2a4\uac00 \ud22c\uc790\ud55c \uae08\uc561\uc740 \uc5bc\ub9c8\uc778\uac00?", "paragraph": "2012\ub144 \uc5ec\ub984, \uba87\ub2ec\uac04\uc758 \ud611\uc0c1\ud55c \ub05d\uc5d0 \uc774\ubc18 \uc0ac\ube44\ub514\uc2a4\uac00 PAOK\uc758 \ub300\uc8fc\uc8fc\ub85c \ub4f1\uc7a5\ud588\ub2e4. \uadf8\ub294 \u20ac12M\uc744 \ud22c\uc790\ud558\uc600\uace0, 2012\ub144 5\uc6d4\uc5d0 2\uba85\uc744 \uc784\ub300\ud558\uc600\ub2e4. PAOK\uc758 \uc0c8 \uc8fc\uc8fc\ub294 \uc804 \uccb4\uc81c\uc5d0 \uc313\uc778 \ub300\uaddc\ubaa8\uc758 \ube5a\uc744 \uccad\uc0b0\ud558\uae30 \uc704\ud574 \ud074\ub7fd \uc7ac\uc815 \uc815\ucc45\uc744 \uc218\ub9bd\ud558\uc600\ub2e4. \ud074\ub7fd\uc740 \uc0c8 \uc815\ucc45\uc744 \ud1b5\ud574 \uac12\ube44\uc2fc \uc678\uad6d\uc778 \uc6a9\ubcd1\uc744 \ub4e4\uc5ec\uc624\ub294 \ub300\uc2e0\uc5d0 \uc544\uce74\ub370\ubbf8 \uc18c\uc18d\uc758 \uc120\uc218\ub4e4\uc744 \uc560\uc6a9\ud558\uae30 \uc2dc\uc791\ud558\uc600\ub2e4. \uc804 \uc120\uc218\ub4e4\uc774\uc790 \ud68c\uc7a5\uc744 \ub9e1\uc740 \ud14c\uc624\ub3c4\ub85c\uc2a4 \uc790\uace0\ub77c\ud0a4\uc2a4\uc640 \uc9c0\uc2dc\uc2a4 \ube0c\ub9ac\uc790\uc2a4\ub294 \ud074\ub7fd\uc758 \ucd95\uad6c \uc815\ucc45\uc744 \ud589\ub3d9\uc73c\ub85c \uc62e\uacbc\uace0, \ud55c\ud3b8 \uc0ac\ube44\ub514\uc2a4\ub294 \uadf8\uc758 \uc8fc\uc694 \uc0ac\uc6d0\ub4e4\uc744 \uc8fc\uc694\uc9c1\uc5d0 \ubc30\uce58\uc2dc\ucf30\ub2e4. PAOK \ubcf4\ub4dc\uc9c4\uc740 2012\ub144 \uc5ec\ub984\uc5d0 \uc804 \uac10\ub3c5\uc778 \ub77c\uc2e4\ub85c \ubd50\ub8b0\ub2c8\uc640\uc758 \uacc4\uc57d\uc744 \uc885\ub8cc\uc2dc\ud0a4\uace0 \uadf8\ub9ac\uc2a4\uc778 \uc694\ub974\uace0\uc2a4 \ub3c4\ub2c8\uc2a4\ub97c \uac10\ub3c5\uc73c\ub85c \ub0b4\uc815\ud558\uc600\ub2e4.", "answer": "\u20ac12M", "sentence": "\uadf8\ub294 \u20ac12M \uc744 \ud22c\uc790\ud558\uc600\uace0, 2012\ub144 5\uc6d4\uc5d0 2\uba85\uc744 \uc784\ub300\ud558\uc600\ub2e4.", "paragraph_sentence": "2012\ub144 \uc5ec\ub984, \uba87\ub2ec\uac04\uc758 \ud611\uc0c1\ud55c \ub05d\uc5d0 \uc774\ubc18 \uc0ac\ube44\ub514\uc2a4\uac00 PAOK\uc758 \ub300\uc8fc\uc8fc\ub85c \ub4f1\uc7a5\ud588\ub2e4. \uadf8\ub294 \u20ac12M \uc744 \ud22c\uc790\ud558\uc600\uace0, 2012\ub144 5\uc6d4\uc5d0 2\uba85\uc744 \uc784\ub300\ud558\uc600\ub2e4. PAOK\uc758 \uc0c8 \uc8fc\uc8fc\ub294 \uc804 \uccb4\uc81c\uc5d0 \uc313\uc778 \ub300\uaddc\ubaa8\uc758 \ube5a\uc744 \uccad\uc0b0\ud558\uae30 \uc704\ud574 \ud074\ub7fd \uc7ac\uc815 \uc815\ucc45\uc744 \uc218\ub9bd\ud558\uc600\ub2e4. \ud074\ub7fd\uc740 \uc0c8 \uc815\ucc45\uc744 \ud1b5\ud574 \uac12\ube44\uc2fc \uc678\uad6d\uc778 \uc6a9\ubcd1\uc744 \ub4e4\uc5ec\uc624\ub294 \ub300\uc2e0\uc5d0 \uc544\uce74\ub370\ubbf8 \uc18c\uc18d\uc758 \uc120\uc218\ub4e4\uc744 \uc560\uc6a9\ud558\uae30 \uc2dc\uc791\ud558\uc600\ub2e4. \uc804 \uc120\uc218\ub4e4\uc774\uc790 \ud68c\uc7a5\uc744 \ub9e1\uc740 \ud14c\uc624\ub3c4\ub85c\uc2a4 \uc790\uace0\ub77c\ud0a4\uc2a4\uc640 \uc9c0\uc2dc\uc2a4 \ube0c\ub9ac\uc790\uc2a4\ub294 \ud074\ub7fd\uc758 \ucd95\uad6c \uc815\ucc45\uc744 \ud589\ub3d9\uc73c\ub85c \uc62e\uacbc\uace0, \ud55c\ud3b8 \uc0ac\ube44\ub514\uc2a4\ub294 \uadf8\uc758 \uc8fc\uc694 \uc0ac\uc6d0\ub4e4\uc744 \uc8fc\uc694\uc9c1\uc5d0 \ubc30\uce58\uc2dc\ucf30\ub2e4. PAOK \ubcf4\ub4dc\uc9c4\uc740 2012\ub144 \uc5ec\ub984\uc5d0 \uc804 \uac10\ub3c5\uc778 \ub77c\uc2e4\ub85c \ubd50\ub8b0\ub2c8\uc640\uc758 \uacc4\uc57d\uc744 \uc885\ub8cc\uc2dc\ud0a4\uace0 \uadf8\ub9ac\uc2a4\uc778 \uc694\ub974\uace0\uc2a4 \ub3c4\ub2c8\uc2a4\ub97c \uac10\ub3c5\uc73c\ub85c \ub0b4\uc815\ud558\uc600\ub2e4.", "paragraph_answer": "2012\ub144 \uc5ec\ub984, \uba87\ub2ec\uac04\uc758 \ud611\uc0c1\ud55c \ub05d\uc5d0 \uc774\ubc18 \uc0ac\ube44\ub514\uc2a4\uac00 PAOK\uc758 \ub300\uc8fc\uc8fc\ub85c \ub4f1\uc7a5\ud588\ub2e4. \uadf8\ub294 \u20ac12M \uc744 \ud22c\uc790\ud558\uc600\uace0, 2012\ub144 5\uc6d4\uc5d0 2\uba85\uc744 \uc784\ub300\ud558\uc600\ub2e4. PAOK\uc758 \uc0c8 \uc8fc\uc8fc\ub294 \uc804 \uccb4\uc81c\uc5d0 \uc313\uc778 \ub300\uaddc\ubaa8\uc758 \ube5a\uc744 \uccad\uc0b0\ud558\uae30 \uc704\ud574 \ud074\ub7fd \uc7ac\uc815 \uc815\ucc45\uc744 \uc218\ub9bd\ud558\uc600\ub2e4. \ud074\ub7fd\uc740 \uc0c8 \uc815\ucc45\uc744 \ud1b5\ud574 \uac12\ube44\uc2fc \uc678\uad6d\uc778 \uc6a9\ubcd1\uc744 \ub4e4\uc5ec\uc624\ub294 \ub300\uc2e0\uc5d0 \uc544\uce74\ub370\ubbf8 \uc18c\uc18d\uc758 \uc120\uc218\ub4e4\uc744 \uc560\uc6a9\ud558\uae30 \uc2dc\uc791\ud558\uc600\ub2e4. \uc804 \uc120\uc218\ub4e4\uc774\uc790 \ud68c\uc7a5\uc744 \ub9e1\uc740 \ud14c\uc624\ub3c4\ub85c\uc2a4 \uc790\uace0\ub77c\ud0a4\uc2a4\uc640 \uc9c0\uc2dc\uc2a4 \ube0c\ub9ac\uc790\uc2a4\ub294 \ud074\ub7fd\uc758 \ucd95\uad6c \uc815\ucc45\uc744 \ud589\ub3d9\uc73c\ub85c \uc62e\uacbc\uace0, \ud55c\ud3b8 \uc0ac\ube44\ub514\uc2a4\ub294 \uadf8\uc758 \uc8fc\uc694 \uc0ac\uc6d0\ub4e4\uc744 \uc8fc\uc694\uc9c1\uc5d0 \ubc30\uce58\uc2dc\ucf30\ub2e4. PAOK \ubcf4\ub4dc\uc9c4\uc740 2012\ub144 \uc5ec\ub984\uc5d0 \uc804 \uac10\ub3c5\uc778 \ub77c\uc2e4\ub85c \ubd50\ub8b0\ub2c8\uc640\uc758 \uacc4\uc57d\uc744 \uc885\ub8cc\uc2dc\ud0a4\uace0 \uadf8\ub9ac\uc2a4\uc778 \uc694\ub974\uace0\uc2a4 \ub3c4\ub2c8\uc2a4\ub97c \uac10\ub3c5\uc73c\ub85c \ub0b4\uc815\ud558\uc600\ub2e4.", "sentence_answer": "\uadf8\ub294 \u20ac12M \uc744 \ud22c\uc790\ud558\uc600\uace0, 2012\ub144 5\uc6d4\uc5d0 2\uba85\uc744 \uc784\ub300\ud558\uc600\ub2e4.", "paragraph_id": "6472301-6-1", "paragraph_question": "question: PAOK\uc758 \ub300\uc8fc\uc8fc \uc774\ubc18 \uc0ac\ube44\ub514\uc2a4\uac00 \ud22c\uc790\ud55c \uae08\uc561\uc740 \uc5bc\ub9c8\uc778\uac00?, context: 2012\ub144 \uc5ec\ub984, \uba87\ub2ec\uac04\uc758 \ud611\uc0c1\ud55c \ub05d\uc5d0 \uc774\ubc18 \uc0ac\ube44\ub514\uc2a4\uac00 PAOK\uc758 \ub300\uc8fc\uc8fc\ub85c \ub4f1\uc7a5\ud588\ub2e4. \uadf8\ub294 \u20ac12M\uc744 \ud22c\uc790\ud558\uc600\uace0, 2012\ub144 5\uc6d4\uc5d0 2\uba85\uc744 \uc784\ub300\ud558\uc600\ub2e4. PAOK\uc758 \uc0c8 \uc8fc\uc8fc\ub294 \uc804 \uccb4\uc81c\uc5d0 \uc313\uc778 \ub300\uaddc\ubaa8\uc758 \ube5a\uc744 \uccad\uc0b0\ud558\uae30 \uc704\ud574 \ud074\ub7fd \uc7ac\uc815 \uc815\ucc45\uc744 \uc218\ub9bd\ud558\uc600\ub2e4. \ud074\ub7fd\uc740 \uc0c8 \uc815\ucc45\uc744 \ud1b5\ud574 \uac12\ube44\uc2fc \uc678\uad6d\uc778 \uc6a9\ubcd1\uc744 \ub4e4\uc5ec\uc624\ub294 \ub300\uc2e0\uc5d0 \uc544\uce74\ub370\ubbf8 \uc18c\uc18d\uc758 \uc120\uc218\ub4e4\uc744 \uc560\uc6a9\ud558\uae30 \uc2dc\uc791\ud558\uc600\ub2e4. \uc804 \uc120\uc218\ub4e4\uc774\uc790 \ud68c\uc7a5\uc744 \ub9e1\uc740 \ud14c\uc624\ub3c4\ub85c\uc2a4 \uc790\uace0\ub77c\ud0a4\uc2a4\uc640 \uc9c0\uc2dc\uc2a4 \ube0c\ub9ac\uc790\uc2a4\ub294 \ud074\ub7fd\uc758 \ucd95\uad6c \uc815\ucc45\uc744 \ud589\ub3d9\uc73c\ub85c \uc62e\uacbc\uace0, \ud55c\ud3b8 \uc0ac\ube44\ub514\uc2a4\ub294 \uadf8\uc758 \uc8fc\uc694 \uc0ac\uc6d0\ub4e4\uc744 \uc8fc\uc694\uc9c1\uc5d0 \ubc30\uce58\uc2dc\ucf30\ub2e4. PAOK \ubcf4\ub4dc\uc9c4\uc740 2012\ub144 \uc5ec\ub984\uc5d0 \uc804 \uac10\ub3c5\uc778 \ub77c\uc2e4\ub85c \ubd50\ub8b0\ub2c8\uc640\uc758 \uacc4\uc57d\uc744 \uc885\ub8cc\uc2dc\ud0a4\uace0 \uadf8\ub9ac\uc2a4\uc778 \uc694\ub974\uace0\uc2a4 \ub3c4\ub2c8\uc2a4\ub97c \uac10\ub3c5\uc73c\ub85c \ub0b4\uc815\ud558\uc600\ub2e4."} {"question": "\uc81c3\uacf5\ud654\uad6d \ucd08\uae30\uae4c\uc9c0 \ubbfc\uc871\uc801\uc774\ub77c\uace0 \ud3c9\uac00\ud558\uba70 \ubc15\uc815\ud76c\uc5d0 \uc9c0\uc9c0\ub97c \ud45c\uba85\ud588\ub358 \uc0ac\ub78c\uc740?", "paragraph": "\uc815\ubcc0 \ucd08\uae30\uc5d0\ub294 \uc77c\ubd80 \uc778\uc0ac\ub4e4\uc758 \uc9c0\uc9c0 \uc131\uba85\uc774 \uc788\uc5c8\ub294\ub370 \uc7a5\uc900\ud558\ub294 \uc0ac\uc0c1\uacc4 6\uc6d4\ud638\uc5d0\uc11c \u201c\uacfc\uac70\uc758 \ubc29\uc885, \ubb34\uc9c8\uc11c, \ud0c0\uc131, \ud3b8\uc758\uc8fc\uc758\uc758 \ub0a1\uc740 \uaecd\uc9c8\uc5d0\uc11c \ud0c8\ud53c\ud558\uc5ec, \uc77c\uccb4\uc758 \uad6c\uc545\uc744 \ubfcc\ub9ac \ubf51\uace0 \uc0c8\ub85c\uc6b4 \ubbfc\uc871\uc801 \ud65c\ub85c\ub97c \uac1c\ucc99\ud560 \uacc4\uae30\ub97c \ub9c8\ub828\ud55c \uac83\uc774\ub2e4\u201d\ub77c\uba70 \uc815\ubcc0\uc744 \uc9c0\uc9c0\ud558\uc600\uace0 \uc5b8\ub860\uc778 \uc1a1\uac74\ud638\ub3c4 \uc81c3\uacf5\ud654\uad6d \ucd08\uae30\uae4c\uc9c0 \ubbfc\uc871\uc801\uc774\ub77c\uace0 \ud3c9\uac00\ud558\uc5ec \ubc15\uc815\ud76c\uc5d0 \ub300\ud55c \uc9c0\uc9c0\ub97c \ud45c\uba85\ud558\uae30\ub3c4 \ud558\uc600\ub2e4. \ub610\ud55c \uc815\ubcc0 \ud55c \ub2ec \ub4a4, \uc77c\uc81c\uac15\uc810\uae30 \ub2f9\uc2dc \uc81c\uc554\ub9ac \ud559\uc0b4\uc0ac\uac74\uc744 \ud3ed\ub85c\ud55c \ud504\ub7ad\ud06c \uc2a4\ucf54\ud544\ub4dc \ubc15\uc0ac\ub294 1961\ub144 6\uc6d4 14\uc77c \u2018\ucf54\ub9ac\uc5b8 \ub9ac\ud37c\ube14\ub9ad\u2019\uc9c0\uc5d0 \u20185\u00b716 \ucfe0\ub370\ud0c0\uc5d0 \ub300\ud55c \ub098\uc758 \uacac\ud574\u2019\ub77c\ub294 \uae00\uc744 \ubc1c\ud45c\ud558\uc600\ub294\ub370 \uadf8\ub294 \ud22c\uace0\uc758 \uccab\uba38\ub9ac\uc5d0\uc11c \u20185\u00b716 \ucfe0\ub370\ud0c0\ub294 \ud544\uc694\ud558\uace0\ub3c4 \ubd88\uac00\ud53c\ud55c \uac83\u2019\uc744 \uc54c\uac8c \ub420 \uac83\uc774\ub77c\uace0 \uc9c0\uc801\ud558\uba74\uc11c \ubbfc\uc8fc\ub2f9 \uc815\uad8c\uc758 \ubd80\uc815\uacfc \ubb34\ub2a5\uc744 \ud3ed\ub85c\ud558\uba70 \u2018\ud55c\uad6d\uc5d0\ub294 \uc544\uc9c1 \uc9c4\uc815\ud55c \ubbfc\uc8fc\uc8fc\uc758\uac00 \uc2dc\ud5d8\ub41c \uc801\uc774 \uc5c6\ub2e4\u2019\uace0 \uc8fc\uc7a5\ud558\uc600\ub2e4.", "answer": "\uc1a1\uac74\ud638", "sentence": "\uc5b8\ub860\uc778 \uc1a1\uac74\ud638 \ub3c4 \uc81c3\uacf5\ud654\uad6d \ucd08\uae30\uae4c\uc9c0 \ubbfc\uc871\uc801\uc774\ub77c\uace0 \ud3c9\uac00\ud558\uc5ec \ubc15\uc815\ud76c\uc5d0 \ub300\ud55c \uc9c0\uc9c0\ub97c \ud45c\uba85\ud558\uae30\ub3c4 \ud558\uc600\ub2e4.", "paragraph_sentence": "\uc815\ubcc0 \ucd08\uae30\uc5d0\ub294 \uc77c\ubd80 \uc778\uc0ac\ub4e4\uc758 \uc9c0\uc9c0 \uc131\uba85\uc774 \uc788\uc5c8\ub294\ub370 \uc7a5\uc900\ud558\ub294 \uc0ac\uc0c1\uacc4 6\uc6d4\ud638\uc5d0\uc11c \u201c\uacfc\uac70\uc758 \ubc29\uc885, \ubb34\uc9c8\uc11c, \ud0c0\uc131, \ud3b8\uc758\uc8fc\uc758\uc758 \ub0a1\uc740 \uaecd\uc9c8\uc5d0\uc11c \ud0c8\ud53c\ud558\uc5ec, \uc77c\uccb4\uc758 \uad6c\uc545\uc744 \ubfcc\ub9ac \ubf51\uace0 \uc0c8\ub85c\uc6b4 \ubbfc\uc871\uc801 \ud65c\ub85c\ub97c \uac1c\ucc99\ud560 \uacc4\uae30\ub97c \ub9c8\ub828\ud55c \uac83\uc774\ub2e4\u201d\ub77c\uba70 \uc815\ubcc0\uc744 \uc9c0\uc9c0\ud558\uc600\uace0 \uc5b8\ub860\uc778 \uc1a1\uac74\ud638 \ub3c4 \uc81c3\uacf5\ud654\uad6d \ucd08\uae30\uae4c\uc9c0 \ubbfc\uc871\uc801\uc774\ub77c\uace0 \ud3c9\uac00\ud558\uc5ec \ubc15\uc815\ud76c\uc5d0 \ub300\ud55c \uc9c0\uc9c0\ub97c \ud45c\uba85\ud558\uae30\ub3c4 \ud558\uc600\ub2e4. \ub610\ud55c \uc815\ubcc0 \ud55c \ub2ec \ub4a4, \uc77c\uc81c\uac15\uc810\uae30 \ub2f9\uc2dc \uc81c\uc554\ub9ac \ud559\uc0b4\uc0ac\uac74\uc744 \ud3ed\ub85c\ud55c \ud504\ub7ad\ud06c \uc2a4\ucf54\ud544\ub4dc \ubc15\uc0ac\ub294 1961\ub144 6\uc6d4 14\uc77c \u2018\ucf54\ub9ac\uc5b8 \ub9ac\ud37c\ube14\ub9ad\u2019\uc9c0\uc5d0 \u20185\u00b716 \ucfe0\ub370\ud0c0\uc5d0 \ub300\ud55c \ub098\uc758 \uacac\ud574\u2019\ub77c\ub294 \uae00\uc744 \ubc1c\ud45c\ud558\uc600\ub294\ub370 \uadf8\ub294 \ud22c\uace0\uc758 \uccab\uba38\ub9ac\uc5d0\uc11c \u20185\u00b716 \ucfe0\ub370\ud0c0\ub294 \ud544\uc694\ud558\uace0\ub3c4 \ubd88\uac00\ud53c\ud55c \uac83\u2019\uc744 \uc54c\uac8c \ub420 \uac83\uc774\ub77c\uace0 \uc9c0\uc801\ud558\uba74\uc11c \ubbfc\uc8fc\ub2f9 \uc815\uad8c\uc758 \ubd80\uc815\uacfc \ubb34\ub2a5\uc744 \ud3ed\ub85c\ud558\uba70 \u2018\ud55c\uad6d\uc5d0\ub294 \uc544\uc9c1 \uc9c4\uc815\ud55c \ubbfc\uc8fc\uc8fc\uc758\uac00 \uc2dc\ud5d8\ub41c \uc801\uc774 \uc5c6\ub2e4\u2019\uace0 \uc8fc\uc7a5\ud558\uc600\ub2e4.", "paragraph_answer": "\uc815\ubcc0 \ucd08\uae30\uc5d0\ub294 \uc77c\ubd80 \uc778\uc0ac\ub4e4\uc758 \uc9c0\uc9c0 \uc131\uba85\uc774 \uc788\uc5c8\ub294\ub370 \uc7a5\uc900\ud558\ub294 \uc0ac\uc0c1\uacc4 6\uc6d4\ud638\uc5d0\uc11c \u201c\uacfc\uac70\uc758 \ubc29\uc885, \ubb34\uc9c8\uc11c, \ud0c0\uc131, \ud3b8\uc758\uc8fc\uc758\uc758 \ub0a1\uc740 \uaecd\uc9c8\uc5d0\uc11c \ud0c8\ud53c\ud558\uc5ec, \uc77c\uccb4\uc758 \uad6c\uc545\uc744 \ubfcc\ub9ac \ubf51\uace0 \uc0c8\ub85c\uc6b4 \ubbfc\uc871\uc801 \ud65c\ub85c\ub97c \uac1c\ucc99\ud560 \uacc4\uae30\ub97c \ub9c8\ub828\ud55c \uac83\uc774\ub2e4\u201d\ub77c\uba70 \uc815\ubcc0\uc744 \uc9c0\uc9c0\ud558\uc600\uace0 \uc5b8\ub860\uc778 \uc1a1\uac74\ud638 \ub3c4 \uc81c3\uacf5\ud654\uad6d \ucd08\uae30\uae4c\uc9c0 \ubbfc\uc871\uc801\uc774\ub77c\uace0 \ud3c9\uac00\ud558\uc5ec \ubc15\uc815\ud76c\uc5d0 \ub300\ud55c \uc9c0\uc9c0\ub97c \ud45c\uba85\ud558\uae30\ub3c4 \ud558\uc600\ub2e4. \ub610\ud55c \uc815\ubcc0 \ud55c \ub2ec \ub4a4, \uc77c\uc81c\uac15\uc810\uae30 \ub2f9\uc2dc \uc81c\uc554\ub9ac \ud559\uc0b4\uc0ac\uac74\uc744 \ud3ed\ub85c\ud55c \ud504\ub7ad\ud06c \uc2a4\ucf54\ud544\ub4dc \ubc15\uc0ac\ub294 1961\ub144 6\uc6d4 14\uc77c \u2018\ucf54\ub9ac\uc5b8 \ub9ac\ud37c\ube14\ub9ad\u2019\uc9c0\uc5d0 \u20185\u00b716 \ucfe0\ub370\ud0c0\uc5d0 \ub300\ud55c \ub098\uc758 \uacac\ud574\u2019\ub77c\ub294 \uae00\uc744 \ubc1c\ud45c\ud558\uc600\ub294\ub370 \uadf8\ub294 \ud22c\uace0\uc758 \uccab\uba38\ub9ac\uc5d0\uc11c \u20185\u00b716 \ucfe0\ub370\ud0c0\ub294 \ud544\uc694\ud558\uace0\ub3c4 \ubd88\uac00\ud53c\ud55c \uac83\u2019\uc744 \uc54c\uac8c \ub420 \uac83\uc774\ub77c\uace0 \uc9c0\uc801\ud558\uba74\uc11c \ubbfc\uc8fc\ub2f9 \uc815\uad8c\uc758 \ubd80\uc815\uacfc \ubb34\ub2a5\uc744 \ud3ed\ub85c\ud558\uba70 \u2018\ud55c\uad6d\uc5d0\ub294 \uc544\uc9c1 \uc9c4\uc815\ud55c \ubbfc\uc8fc\uc8fc\uc758\uac00 \uc2dc\ud5d8\ub41c \uc801\uc774 \uc5c6\ub2e4\u2019\uace0 \uc8fc\uc7a5\ud558\uc600\ub2e4.", "sentence_answer": "\uc5b8\ub860\uc778 \uc1a1\uac74\ud638 \ub3c4 \uc81c3\uacf5\ud654\uad6d \ucd08\uae30\uae4c\uc9c0 \ubbfc\uc871\uc801\uc774\ub77c\uace0 \ud3c9\uac00\ud558\uc5ec \ubc15\uc815\ud76c\uc5d0 \ub300\ud55c \uc9c0\uc9c0\ub97c \ud45c\uba85\ud558\uae30\ub3c4 \ud558\uc600\ub2e4.", "paragraph_id": "6530241-23-0", "paragraph_question": "question: \uc81c3\uacf5\ud654\uad6d \ucd08\uae30\uae4c\uc9c0 \ubbfc\uc871\uc801\uc774\ub77c\uace0 \ud3c9\uac00\ud558\uba70 \ubc15\uc815\ud76c\uc5d0 \uc9c0\uc9c0\ub97c \ud45c\uba85\ud588\ub358 \uc0ac\ub78c\uc740?, context: \uc815\ubcc0 \ucd08\uae30\uc5d0\ub294 \uc77c\ubd80 \uc778\uc0ac\ub4e4\uc758 \uc9c0\uc9c0 \uc131\uba85\uc774 \uc788\uc5c8\ub294\ub370 \uc7a5\uc900\ud558\ub294 \uc0ac\uc0c1\uacc4 6\uc6d4\ud638\uc5d0\uc11c \u201c\uacfc\uac70\uc758 \ubc29\uc885, \ubb34\uc9c8\uc11c, \ud0c0\uc131, \ud3b8\uc758\uc8fc\uc758\uc758 \ub0a1\uc740 \uaecd\uc9c8\uc5d0\uc11c \ud0c8\ud53c\ud558\uc5ec, \uc77c\uccb4\uc758 \uad6c\uc545\uc744 \ubfcc\ub9ac \ubf51\uace0 \uc0c8\ub85c\uc6b4 \ubbfc\uc871\uc801 \ud65c\ub85c\ub97c \uac1c\ucc99\ud560 \uacc4\uae30\ub97c \ub9c8\ub828\ud55c \uac83\uc774\ub2e4\u201d\ub77c\uba70 \uc815\ubcc0\uc744 \uc9c0\uc9c0\ud558\uc600\uace0 \uc5b8\ub860\uc778 \uc1a1\uac74\ud638\ub3c4 \uc81c3\uacf5\ud654\uad6d \ucd08\uae30\uae4c\uc9c0 \ubbfc\uc871\uc801\uc774\ub77c\uace0 \ud3c9\uac00\ud558\uc5ec \ubc15\uc815\ud76c\uc5d0 \ub300\ud55c \uc9c0\uc9c0\ub97c \ud45c\uba85\ud558\uae30\ub3c4 \ud558\uc600\ub2e4. \ub610\ud55c \uc815\ubcc0 \ud55c \ub2ec \ub4a4, \uc77c\uc81c\uac15\uc810\uae30 \ub2f9\uc2dc \uc81c\uc554\ub9ac \ud559\uc0b4\uc0ac\uac74\uc744 \ud3ed\ub85c\ud55c \ud504\ub7ad\ud06c \uc2a4\ucf54\ud544\ub4dc \ubc15\uc0ac\ub294 1961\ub144 6\uc6d4 14\uc77c \u2018\ucf54\ub9ac\uc5b8 \ub9ac\ud37c\ube14\ub9ad\u2019\uc9c0\uc5d0 \u20185\u00b716 \ucfe0\ub370\ud0c0\uc5d0 \ub300\ud55c \ub098\uc758 \uacac\ud574\u2019\ub77c\ub294 \uae00\uc744 \ubc1c\ud45c\ud558\uc600\ub294\ub370 \uadf8\ub294 \ud22c\uace0\uc758 \uccab\uba38\ub9ac\uc5d0\uc11c \u20185\u00b716 \ucfe0\ub370\ud0c0\ub294 \ud544\uc694\ud558\uace0\ub3c4 \ubd88\uac00\ud53c\ud55c \uac83\u2019\uc744 \uc54c\uac8c \ub420 \uac83\uc774\ub77c\uace0 \uc9c0\uc801\ud558\uba74\uc11c \ubbfc\uc8fc\ub2f9 \uc815\uad8c\uc758 \ubd80\uc815\uacfc \ubb34\ub2a5\uc744 \ud3ed\ub85c\ud558\uba70 \u2018\ud55c\uad6d\uc5d0\ub294 \uc544\uc9c1 \uc9c4\uc815\ud55c \ubbfc\uc8fc\uc8fc\uc758\uac00 \uc2dc\ud5d8\ub41c \uc801\uc774 \uc5c6\ub2e4\u2019\uace0 \uc8fc\uc7a5\ud558\uc600\ub2e4."} {"question": "2013\ub144 \uc544\uba54\ubc14\uceec\uccd0\uc5d0\uc11c \ud06c\ub7ec\uc26c\uc640 \ud568\uaed8 \ubc1c\ub9e4\ud55c \uc568\ubc94\uc758 \uc774\ub984\uc740", "paragraph": "\uc790\uc774\uc5b8\ud2f0(Zion.T, \ubcf8\uba85: \uae40\ud574\uc194, 1989\ub144 4\uc6d4 13\uc77c ~ )\ub294 \ub300\ud55c\ubbfc\uad6d\uc758 R&B \ud799\ud569 \uc74c\uc545\uac00\uc774\ub2e4. \ud06c\ub8e8\ub294 VV:D (\ube44\ube44\ub4dc) \ud06c\ub8e8\ub85c, \uadf8\ub808\uc774, \ub85c\uaf2c, \ud06c\ub7ec\uc26c, \uc5d8\ub85c\uac00 \uba64\ubc84\uc774\uba70 \ub124\uc624\uc18c\uc6b8 \ud48d\uc758 \uc74c\uc545\uc744 \uc8fc\ub85c \ud558\uace0 \uc788\ub2e4. 2011\ub144 \uc2f1\uae00 \uc74c\ubc18 Click Me\ub85c \ub370\ubdd4\ud574 \uc544\uba54\ubc14 \uceec\uccd0\uc5d0 \ub4e4\uc5b4\uac00\uba74\uc11c \uc0ac\uc774\uba3c \ub3c4\ubbf8\ub2c9\uc758 \u300aSimon Dominic Presents 'SNL LEAGUE BEGINS'\u300b\uc640 \ud504\ub77c\uc774\uba38\ub9ac\uc758 \u300aPrimary And The Messengers LP\u300b \ub4f1 \ub3d9\ub8cc\ub4e4\uc758 \uc74c\ubc18\uc5d0 \ud53c\ucc98\ub9c1\ud558\uba70 \uc774\ub984\uc744 \uc54c\ub838\ub2e4. 2013\ub144\uc5d0\ub294 \uc544\uba54\ubc14 \uceec\uccd0\uc758 \ud504\ub85c\uc81d\ud2b8\uc778 NOWorkend \ud504\ub85c\uc81d\ud2b8\uc758 \uc77c\ud658\uc73c\ub85c \ud06c\ub7ec\uc26c\uc640 \ud568\uaed8 \u3008\ubed4\ud55c \uba5c\ub85c\ub514\u3009\ub97c \ubc1c\ub9e4\ud588\ub2e4. \uc774\uc5b4 2013\ub144 12\uc6d4 19\uc77c\uc5d0\ub294 \uc2f1\uae00 \u3008\ubbf8\ub7ec\ubcfc\u3009\uc744 \ubc1c\ud45c\ud588\ub2e4. 2016\ub144 3\uc6d4, \uc790\uc774\uc5b8\ud2f0\ub294 YG \uc18c\uc18d \ud504\ub85c\ub4c0\uc11c \ud14c\ub514\uac00 \uc124\ub9bd\ud55c YG \uc5d4\ud130\ud14c\uc778\uba3c\ud2b8 \uc0b0\ud558 \ub808\uc774\ube14\ub85c \uc774\uc801\ud558\ub294 \uac83\uc774 \ubc1c\ud45c \ub418\uc5c8\ub2e4. \ub808\uc774\ube14 \uc774\ub984\uc740 \ub354\ube14\ub799\ub808\uc774\ube14\uc774\ub2e4.", "answer": "\ubed4\ud55c \uba5c\ub85c\ub514", "sentence": "2013\ub144\uc5d0\ub294 \uc544\uba54\ubc14 \uceec\uccd0\uc758 \ud504\ub85c\uc81d\ud2b8\uc778 NOWorkend \ud504\ub85c\uc81d\ud2b8\uc758 \uc77c\ud658\uc73c\ub85c \ud06c\ub7ec\uc26c\uc640 \ud568\uaed8 \u3008 \ubed4\ud55c \uba5c\ub85c\ub514 \u3009\ub97c \ubc1c\ub9e4\ud588\ub2e4.", "paragraph_sentence": "\uc790\uc774\uc5b8\ud2f0(Zion. T, \ubcf8\uba85: \uae40\ud574\uc194, 1989\ub144 4\uc6d4 13\uc77c ~ )\ub294 \ub300\ud55c\ubbfc\uad6d\uc758 R&B \ud799\ud569 \uc74c\uc545\uac00\uc774\ub2e4. \ud06c\ub8e8\ub294 VV:D (\ube44\ube44\ub4dc) \ud06c\ub8e8\ub85c, \uadf8\ub808\uc774, \ub85c\uaf2c, \ud06c\ub7ec\uc26c, \uc5d8\ub85c\uac00 \uba64\ubc84\uc774\uba70 \ub124\uc624\uc18c\uc6b8 \ud48d\uc758 \uc74c\uc545\uc744 \uc8fc\ub85c \ud558\uace0 \uc788\ub2e4. 2011\ub144 \uc2f1\uae00 \uc74c\ubc18 Click Me\ub85c \ub370\ubdd4\ud574 \uc544\uba54\ubc14 \uceec\uccd0\uc5d0 \ub4e4\uc5b4\uac00\uba74\uc11c \uc0ac\uc774\uba3c \ub3c4\ubbf8\ub2c9\uc758 \u300aSimon Dominic Presents 'SNL LEAGUE BEGINS'\u300b\uc640 \ud504\ub77c\uc774\uba38\ub9ac\uc758 \u300aPrimary And The Messengers LP\u300b \ub4f1 \ub3d9\ub8cc\ub4e4\uc758 \uc74c\ubc18\uc5d0 \ud53c\ucc98\ub9c1\ud558\uba70 \uc774\ub984\uc744 \uc54c\ub838\ub2e4. 2013\ub144\uc5d0\ub294 \uc544\uba54\ubc14 \uceec\uccd0\uc758 \ud504\ub85c\uc81d\ud2b8\uc778 NOWorkend \ud504\ub85c\uc81d\ud2b8\uc758 \uc77c\ud658\uc73c\ub85c \ud06c\ub7ec\uc26c\uc640 \ud568\uaed8 \u3008 \ubed4\ud55c \uba5c\ub85c\ub514 \u3009\ub97c \ubc1c\ub9e4\ud588\ub2e4. \uc774\uc5b4 2013\ub144 12\uc6d4 19\uc77c\uc5d0\ub294 \uc2f1\uae00 \u3008\ubbf8\ub7ec\ubcfc\u3009\uc744 \ubc1c\ud45c\ud588\ub2e4. 2016\ub144 3\uc6d4, \uc790\uc774\uc5b8\ud2f0\ub294 YG \uc18c\uc18d \ud504\ub85c\ub4c0\uc11c \ud14c\ub514\uac00 \uc124\ub9bd\ud55c YG \uc5d4\ud130\ud14c\uc778\uba3c\ud2b8 \uc0b0\ud558 \ub808\uc774\ube14\ub85c \uc774\uc801\ud558\ub294 \uac83\uc774 \ubc1c\ud45c \ub418\uc5c8\ub2e4. \ub808\uc774\ube14 \uc774\ub984\uc740 \ub354\ube14\ub799\ub808\uc774\ube14\uc774\ub2e4.", "paragraph_answer": "\uc790\uc774\uc5b8\ud2f0(Zion.T, \ubcf8\uba85: \uae40\ud574\uc194, 1989\ub144 4\uc6d4 13\uc77c ~ )\ub294 \ub300\ud55c\ubbfc\uad6d\uc758 R&B \ud799\ud569 \uc74c\uc545\uac00\uc774\ub2e4. \ud06c\ub8e8\ub294 VV:D (\ube44\ube44\ub4dc) \ud06c\ub8e8\ub85c, \uadf8\ub808\uc774, \ub85c\uaf2c, \ud06c\ub7ec\uc26c, \uc5d8\ub85c\uac00 \uba64\ubc84\uc774\uba70 \ub124\uc624\uc18c\uc6b8 \ud48d\uc758 \uc74c\uc545\uc744 \uc8fc\ub85c \ud558\uace0 \uc788\ub2e4. 2011\ub144 \uc2f1\uae00 \uc74c\ubc18 Click Me\ub85c \ub370\ubdd4\ud574 \uc544\uba54\ubc14 \uceec\uccd0\uc5d0 \ub4e4\uc5b4\uac00\uba74\uc11c \uc0ac\uc774\uba3c \ub3c4\ubbf8\ub2c9\uc758 \u300aSimon Dominic Presents 'SNL LEAGUE BEGINS'\u300b\uc640 \ud504\ub77c\uc774\uba38\ub9ac\uc758 \u300aPrimary And The Messengers LP\u300b \ub4f1 \ub3d9\ub8cc\ub4e4\uc758 \uc74c\ubc18\uc5d0 \ud53c\ucc98\ub9c1\ud558\uba70 \uc774\ub984\uc744 \uc54c\ub838\ub2e4. 2013\ub144\uc5d0\ub294 \uc544\uba54\ubc14 \uceec\uccd0\uc758 \ud504\ub85c\uc81d\ud2b8\uc778 NOWorkend \ud504\ub85c\uc81d\ud2b8\uc758 \uc77c\ud658\uc73c\ub85c \ud06c\ub7ec\uc26c\uc640 \ud568\uaed8 \u3008 \ubed4\ud55c \uba5c\ub85c\ub514 \u3009\ub97c \ubc1c\ub9e4\ud588\ub2e4. \uc774\uc5b4 2013\ub144 12\uc6d4 19\uc77c\uc5d0\ub294 \uc2f1\uae00 \u3008\ubbf8\ub7ec\ubcfc\u3009\uc744 \ubc1c\ud45c\ud588\ub2e4. 2016\ub144 3\uc6d4, \uc790\uc774\uc5b8\ud2f0\ub294 YG \uc18c\uc18d \ud504\ub85c\ub4c0\uc11c \ud14c\ub514\uac00 \uc124\ub9bd\ud55c YG \uc5d4\ud130\ud14c\uc778\uba3c\ud2b8 \uc0b0\ud558 \ub808\uc774\ube14\ub85c \uc774\uc801\ud558\ub294 \uac83\uc774 \ubc1c\ud45c \ub418\uc5c8\ub2e4. \ub808\uc774\ube14 \uc774\ub984\uc740 \ub354\ube14\ub799\ub808\uc774\ube14\uc774\ub2e4.", "sentence_answer": "2013\ub144\uc5d0\ub294 \uc544\uba54\ubc14 \uceec\uccd0\uc758 \ud504\ub85c\uc81d\ud2b8\uc778 NOWorkend \ud504\ub85c\uc81d\ud2b8\uc758 \uc77c\ud658\uc73c\ub85c \ud06c\ub7ec\uc26c\uc640 \ud568\uaed8 \u3008 \ubed4\ud55c \uba5c\ub85c\ub514 \u3009\ub97c \ubc1c\ub9e4\ud588\ub2e4.", "paragraph_id": "5856825-0-1", "paragraph_question": "question: 2013\ub144 \uc544\uba54\ubc14\uceec\uccd0\uc5d0\uc11c \ud06c\ub7ec\uc26c\uc640 \ud568\uaed8 \ubc1c\ub9e4\ud55c \uc568\ubc94\uc758 \uc774\ub984\uc740, context: \uc790\uc774\uc5b8\ud2f0(Zion.T, \ubcf8\uba85: \uae40\ud574\uc194, 1989\ub144 4\uc6d4 13\uc77c ~ )\ub294 \ub300\ud55c\ubbfc\uad6d\uc758 R&B \ud799\ud569 \uc74c\uc545\uac00\uc774\ub2e4. \ud06c\ub8e8\ub294 VV:D (\ube44\ube44\ub4dc) \ud06c\ub8e8\ub85c, \uadf8\ub808\uc774, \ub85c\uaf2c, \ud06c\ub7ec\uc26c, \uc5d8\ub85c\uac00 \uba64\ubc84\uc774\uba70 \ub124\uc624\uc18c\uc6b8 \ud48d\uc758 \uc74c\uc545\uc744 \uc8fc\ub85c \ud558\uace0 \uc788\ub2e4. 2011\ub144 \uc2f1\uae00 \uc74c\ubc18 Click Me\ub85c \ub370\ubdd4\ud574 \uc544\uba54\ubc14 \uceec\uccd0\uc5d0 \ub4e4\uc5b4\uac00\uba74\uc11c \uc0ac\uc774\uba3c \ub3c4\ubbf8\ub2c9\uc758 \u300aSimon Dominic Presents 'SNL LEAGUE BEGINS'\u300b\uc640 \ud504\ub77c\uc774\uba38\ub9ac\uc758 \u300aPrimary And The Messengers LP\u300b \ub4f1 \ub3d9\ub8cc\ub4e4\uc758 \uc74c\ubc18\uc5d0 \ud53c\ucc98\ub9c1\ud558\uba70 \uc774\ub984\uc744 \uc54c\ub838\ub2e4. 2013\ub144\uc5d0\ub294 \uc544\uba54\ubc14 \uceec\uccd0\uc758 \ud504\ub85c\uc81d\ud2b8\uc778 NOWorkend \ud504\ub85c\uc81d\ud2b8\uc758 \uc77c\ud658\uc73c\ub85c \ud06c\ub7ec\uc26c\uc640 \ud568\uaed8 \u3008\ubed4\ud55c \uba5c\ub85c\ub514\u3009\ub97c \ubc1c\ub9e4\ud588\ub2e4. \uc774\uc5b4 2013\ub144 12\uc6d4 19\uc77c\uc5d0\ub294 \uc2f1\uae00 \u3008\ubbf8\ub7ec\ubcfc\u3009\uc744 \ubc1c\ud45c\ud588\ub2e4. 2016\ub144 3\uc6d4, \uc790\uc774\uc5b8\ud2f0\ub294 YG \uc18c\uc18d \ud504\ub85c\ub4c0\uc11c \ud14c\ub514\uac00 \uc124\ub9bd\ud55c YG \uc5d4\ud130\ud14c\uc778\uba3c\ud2b8 \uc0b0\ud558 \ub808\uc774\ube14\ub85c \uc774\uc801\ud558\ub294 \uac83\uc774 \ubc1c\ud45c \ub418\uc5c8\ub2e4. \ub808\uc774\ube14 \uc774\ub984\uc740 \ub354\ube14\ub799\ub808\uc774\ube14\uc774\ub2e4."} {"question": "\uc8fc\uba39\uc774 \uc6b4\ub2e4\uc5d0\uc11c 4\uc778\uc758 \ubcf8\uc120 \ub3c4\uc804\uc790\ub97c \ubd80\uc0b0\uc73c\ub85c \ubd80\ub978 \uc0ac\ub78c\uc758 \uc774\ub984\uc740?", "paragraph": "\ubcf8\uc120 4\uac15 \ud1a0\ub108\uba3c\ud2b8\uc5d0 \ud569\ub958\ud55c 4\uc778 \ub3c4\uc804\uc790\ub4e4\uc740 \ubd80\uc0b0\uc73c\ub85c \uc678\ucd9c\uc744 \ub5a0\ub098 \ub9dd\ub9dd\ub300\ud574 \uc704 \uc694\ud2b8 \ud55c \ucc99\uc5d0 \uc624\ub974\uac8c \ub41c\ub2e4. 4\uc778 \ub3c4\uc804\uc790\ub4e4\uc744 \ubd80\ub978 \uc0ac\ub78c\uc740 \ub2e4\ub984\uc544\ub2cc \uc815\ubb38\ud64d \ub300\ud45c\uc600\ub2e4. \ub85c\ub4dc FC \ub77c\uc6b4\ub4dc \uac78 \ubbfc\uc81c\uc774. \ubc15\uc9c4\uc544\uc758 \ubcf4\uc88c \uc544\ub798 \ub098\ud0c0\ub09c \uc815\ubb38\ud64d \ub300\ud45c\ub294 \ubcf8\uc120 4\uac15 \ud1a0\ub108\uba3c\ud2b8\ub97c \uc55e\ub454 4\uc778 \ub3c4\uc804\uc790\ub4e4\uc5d0\uac8c \ubbf8\uc158\uc744 \uc218\ud589\ud558\uac8c \ud558\ub294\ub370, \uadf8\uac83\uc740 \ub2e4\ub984\uc544\ub2cc \ubcf8\uc120 4\uac15 \ub300\uc9c4\ud45c\ub97c \uacb0\uc815\ud558\uae30 \uc704\ud55c \ubbf8\uc158\uc774\uc5c8\ub2e4. \ud64d\uc131\ubbfc, \uad8c\ubbfc\uc11d, \uae40\uc2b9\uc5f0, \uc11c\ub3d9\uc218 \uc21c\uc73c\ub85c \ub300\uc9c4\ud45c\uac00 \ub4e4\uc5b4\uc788\ub294 \ud14c\uc774\ube14 \uc704\uc758 4\uac1c\uc758 \ubc15\uc2a4 \uc911 \ud55c\uac1c\uc529\uc744 \uac00\uc838\uac00\uace0 \ub9c8\ucc2c\uac00\uc9c0\ub85c \ud64d\uc131\ubbfc, \uad8c\ubbfc\uc11d, \uae40\uc2b9\uc5f0, \uc11c\ub3d9\uc218 \uc21c\uc73c\ub85c \ucc28\ub840\ub85c \ubc15\uc2a4\ub97c \uc5f4\ub78c\ud588\ub2e4. \uc5f4\ub78c \uacb0\uacfc \ud64d\uc131\ubbfc\uc740 2\uacbd\uae30 \ud64d\ucf54\ub108, \uad8c\ubbfc\uc11d\uc740 1\uacbd\uae30 \uccad\ucf54\ub108, \uae40\uc2b9\uc5f0\uc740 2\uacbd\uae30 \uccad\ucf54\ub108\ub97c \uc120\ud0dd\ud588\uace0, \uc11c\ub3d9\uc218\ub294 \uc790\uc5f0\uc2a4\ub7fd\uac8c 1\uacbd\uae30 \ud64d\ucf54\ub108\ub97c \uc120\ud0dd\ud558\uac8c \ub418\uc5c8\ub2e4.", "answer": "\uc815\ubb38\ud64d", "sentence": "4\uc778 \ub3c4\uc804\uc790\ub4e4\uc744 \ubd80\ub978 \uc0ac\ub78c\uc740 \ub2e4\ub984\uc544\ub2cc \uc815\ubb38\ud64d \ub300\ud45c\uc600\ub2e4.", "paragraph_sentence": "\ubcf8\uc120 4\uac15 \ud1a0\ub108\uba3c\ud2b8\uc5d0 \ud569\ub958\ud55c 4\uc778 \ub3c4\uc804\uc790\ub4e4\uc740 \ubd80\uc0b0\uc73c\ub85c \uc678\ucd9c\uc744 \ub5a0\ub098 \ub9dd\ub9dd\ub300\ud574 \uc704 \uc694\ud2b8 \ud55c \ucc99\uc5d0 \uc624\ub974\uac8c \ub41c\ub2e4. 4\uc778 \ub3c4\uc804\uc790\ub4e4\uc744 \ubd80\ub978 \uc0ac\ub78c\uc740 \ub2e4\ub984\uc544\ub2cc \uc815\ubb38\ud64d \ub300\ud45c\uc600\ub2e4. \ub85c\ub4dc FC \ub77c\uc6b4\ub4dc \uac78 \ubbfc\uc81c\uc774. \ubc15\uc9c4\uc544\uc758 \ubcf4\uc88c \uc544\ub798 \ub098\ud0c0\ub09c \uc815\ubb38\ud64d \ub300\ud45c\ub294 \ubcf8\uc120 4\uac15 \ud1a0\ub108\uba3c\ud2b8\ub97c \uc55e\ub454 4\uc778 \ub3c4\uc804\uc790\ub4e4\uc5d0\uac8c \ubbf8\uc158\uc744 \uc218\ud589\ud558\uac8c \ud558\ub294\ub370, \uadf8\uac83\uc740 \ub2e4\ub984\uc544\ub2cc \ubcf8\uc120 4\uac15 \ub300\uc9c4\ud45c\ub97c \uacb0\uc815\ud558\uae30 \uc704\ud55c \ubbf8\uc158\uc774\uc5c8\ub2e4. \ud64d\uc131\ubbfc, \uad8c\ubbfc\uc11d, \uae40\uc2b9\uc5f0, \uc11c\ub3d9\uc218 \uc21c\uc73c\ub85c \ub300\uc9c4\ud45c\uac00 \ub4e4\uc5b4\uc788\ub294 \ud14c\uc774\ube14 \uc704\uc758 4\uac1c\uc758 \ubc15\uc2a4 \uc911 \ud55c\uac1c\uc529\uc744 \uac00\uc838\uac00\uace0 \ub9c8\ucc2c\uac00\uc9c0\ub85c \ud64d\uc131\ubbfc, \uad8c\ubbfc\uc11d, \uae40\uc2b9\uc5f0, \uc11c\ub3d9\uc218 \uc21c\uc73c\ub85c \ucc28\ub840\ub85c \ubc15\uc2a4\ub97c \uc5f4\ub78c\ud588\ub2e4. \uc5f4\ub78c \uacb0\uacfc \ud64d\uc131\ubbfc\uc740 2\uacbd\uae30 \ud64d\ucf54\ub108, \uad8c\ubbfc\uc11d\uc740 1\uacbd\uae30 \uccad\ucf54\ub108, \uae40\uc2b9\uc5f0\uc740 2\uacbd\uae30 \uccad\ucf54\ub108\ub97c \uc120\ud0dd\ud588\uace0, \uc11c\ub3d9\uc218\ub294 \uc790\uc5f0\uc2a4\ub7fd\uac8c 1\uacbd\uae30 \ud64d\ucf54\ub108\ub97c \uc120\ud0dd\ud558\uac8c \ub418\uc5c8\ub2e4.", "paragraph_answer": "\ubcf8\uc120 4\uac15 \ud1a0\ub108\uba3c\ud2b8\uc5d0 \ud569\ub958\ud55c 4\uc778 \ub3c4\uc804\uc790\ub4e4\uc740 \ubd80\uc0b0\uc73c\ub85c \uc678\ucd9c\uc744 \ub5a0\ub098 \ub9dd\ub9dd\ub300\ud574 \uc704 \uc694\ud2b8 \ud55c \ucc99\uc5d0 \uc624\ub974\uac8c \ub41c\ub2e4. 4\uc778 \ub3c4\uc804\uc790\ub4e4\uc744 \ubd80\ub978 \uc0ac\ub78c\uc740 \ub2e4\ub984\uc544\ub2cc \uc815\ubb38\ud64d \ub300\ud45c\uc600\ub2e4. \ub85c\ub4dc FC \ub77c\uc6b4\ub4dc \uac78 \ubbfc\uc81c\uc774. \ubc15\uc9c4\uc544\uc758 \ubcf4\uc88c \uc544\ub798 \ub098\ud0c0\ub09c \uc815\ubb38\ud64d \ub300\ud45c\ub294 \ubcf8\uc120 4\uac15 \ud1a0\ub108\uba3c\ud2b8\ub97c \uc55e\ub454 4\uc778 \ub3c4\uc804\uc790\ub4e4\uc5d0\uac8c \ubbf8\uc158\uc744 \uc218\ud589\ud558\uac8c \ud558\ub294\ub370, \uadf8\uac83\uc740 \ub2e4\ub984\uc544\ub2cc \ubcf8\uc120 4\uac15 \ub300\uc9c4\ud45c\ub97c \uacb0\uc815\ud558\uae30 \uc704\ud55c \ubbf8\uc158\uc774\uc5c8\ub2e4. \ud64d\uc131\ubbfc, \uad8c\ubbfc\uc11d, \uae40\uc2b9\uc5f0, \uc11c\ub3d9\uc218 \uc21c\uc73c\ub85c \ub300\uc9c4\ud45c\uac00 \ub4e4\uc5b4\uc788\ub294 \ud14c\uc774\ube14 \uc704\uc758 4\uac1c\uc758 \ubc15\uc2a4 \uc911 \ud55c\uac1c\uc529\uc744 \uac00\uc838\uac00\uace0 \ub9c8\ucc2c\uac00\uc9c0\ub85c \ud64d\uc131\ubbfc, \uad8c\ubbfc\uc11d, \uae40\uc2b9\uc5f0, \uc11c\ub3d9\uc218 \uc21c\uc73c\ub85c \ucc28\ub840\ub85c \ubc15\uc2a4\ub97c \uc5f4\ub78c\ud588\ub2e4. \uc5f4\ub78c \uacb0\uacfc \ud64d\uc131\ubbfc\uc740 2\uacbd\uae30 \ud64d\ucf54\ub108, \uad8c\ubbfc\uc11d\uc740 1\uacbd\uae30 \uccad\ucf54\ub108, \uae40\uc2b9\uc5f0\uc740 2\uacbd\uae30 \uccad\ucf54\ub108\ub97c \uc120\ud0dd\ud588\uace0, \uc11c\ub3d9\uc218\ub294 \uc790\uc5f0\uc2a4\ub7fd\uac8c 1\uacbd\uae30 \ud64d\ucf54\ub108\ub97c \uc120\ud0dd\ud558\uac8c \ub418\uc5c8\ub2e4.", "sentence_answer": "4\uc778 \ub3c4\uc804\uc790\ub4e4\uc744 \ubd80\ub978 \uc0ac\ub78c\uc740 \ub2e4\ub984\uc544\ub2cc \uc815\ubb38\ud64d \ub300\ud45c\uc600\ub2e4.", "paragraph_id": "5890258-6-1", "paragraph_question": "question: \uc8fc\uba39\uc774 \uc6b4\ub2e4\uc5d0\uc11c 4\uc778\uc758 \ubcf8\uc120 \ub3c4\uc804\uc790\ub97c \ubd80\uc0b0\uc73c\ub85c \ubd80\ub978 \uc0ac\ub78c\uc758 \uc774\ub984\uc740?, context: \ubcf8\uc120 4\uac15 \ud1a0\ub108\uba3c\ud2b8\uc5d0 \ud569\ub958\ud55c 4\uc778 \ub3c4\uc804\uc790\ub4e4\uc740 \ubd80\uc0b0\uc73c\ub85c \uc678\ucd9c\uc744 \ub5a0\ub098 \ub9dd\ub9dd\ub300\ud574 \uc704 \uc694\ud2b8 \ud55c \ucc99\uc5d0 \uc624\ub974\uac8c \ub41c\ub2e4. 4\uc778 \ub3c4\uc804\uc790\ub4e4\uc744 \ubd80\ub978 \uc0ac\ub78c\uc740 \ub2e4\ub984\uc544\ub2cc \uc815\ubb38\ud64d \ub300\ud45c\uc600\ub2e4. \ub85c\ub4dc FC \ub77c\uc6b4\ub4dc \uac78 \ubbfc\uc81c\uc774. \ubc15\uc9c4\uc544\uc758 \ubcf4\uc88c \uc544\ub798 \ub098\ud0c0\ub09c \uc815\ubb38\ud64d \ub300\ud45c\ub294 \ubcf8\uc120 4\uac15 \ud1a0\ub108\uba3c\ud2b8\ub97c \uc55e\ub454 4\uc778 \ub3c4\uc804\uc790\ub4e4\uc5d0\uac8c \ubbf8\uc158\uc744 \uc218\ud589\ud558\uac8c \ud558\ub294\ub370, \uadf8\uac83\uc740 \ub2e4\ub984\uc544\ub2cc \ubcf8\uc120 4\uac15 \ub300\uc9c4\ud45c\ub97c \uacb0\uc815\ud558\uae30 \uc704\ud55c \ubbf8\uc158\uc774\uc5c8\ub2e4. \ud64d\uc131\ubbfc, \uad8c\ubbfc\uc11d, \uae40\uc2b9\uc5f0, \uc11c\ub3d9\uc218 \uc21c\uc73c\ub85c \ub300\uc9c4\ud45c\uac00 \ub4e4\uc5b4\uc788\ub294 \ud14c\uc774\ube14 \uc704\uc758 4\uac1c\uc758 \ubc15\uc2a4 \uc911 \ud55c\uac1c\uc529\uc744 \uac00\uc838\uac00\uace0 \ub9c8\ucc2c\uac00\uc9c0\ub85c \ud64d\uc131\ubbfc, \uad8c\ubbfc\uc11d, \uae40\uc2b9\uc5f0, \uc11c\ub3d9\uc218 \uc21c\uc73c\ub85c \ucc28\ub840\ub85c \ubc15\uc2a4\ub97c \uc5f4\ub78c\ud588\ub2e4. \uc5f4\ub78c \uacb0\uacfc \ud64d\uc131\ubbfc\uc740 2\uacbd\uae30 \ud64d\ucf54\ub108, \uad8c\ubbfc\uc11d\uc740 1\uacbd\uae30 \uccad\ucf54\ub108, \uae40\uc2b9\uc5f0\uc740 2\uacbd\uae30 \uccad\ucf54\ub108\ub97c \uc120\ud0dd\ud588\uace0, \uc11c\ub3d9\uc218\ub294 \uc790\uc5f0\uc2a4\ub7fd\uac8c 1\uacbd\uae30 \ud64d\ucf54\ub108\ub97c \uc120\ud0dd\ud558\uac8c \ub418\uc5c8\ub2e4."} {"question": "\uc2e4\ub85d\uc5d0\ub294 \uc815\uc885\uc774 \uc8fd\uc740 \ud6c4 \uacbd\ud61c\uacf5\uc8fc\uac00 \ubb34\uc5c7\uc774 \ub418\uc5c8\ub2e4\uace0 \uae30\ub85d\ub418\uc5b4 \uc788\ub294\uac00?", "paragraph": "\uc548\uc815\ubcf5\uc758 \uc21c\uc554\uc9d1\uc5d0\ub3c4 \uacbd\ud61c\uacf5\uc8fc\uac00 \uc7a5\ud765\uc758 \uad00\ub178\uac00 \ub418\uc5c8\ub2e4\ub294 \uae30\uc220\uc774 \uc788\ub2e4. \uadf8\ub7ec\ub098 \uc2e4\ub85d\uc5d0\ub294 \uacbd\ud61c\uacf5\uc8fc\uac00 \uad00\ub178\uac00 \ub418\uc5c8\ub2e4\ub294 \uae30\ub85d\uc774 \uc5c6\uace0 \ub300\uc2e0 \uc815\uc885\uc774 \uc8fd\uc740 \ud6c4 \uba38\ub9ac\ub97c \uae4e\uace0 \uc5ec\uc2b9\uc774 \ub418\uc5c8\ub294\ub370 \ubb34\ucc99 \uac00\ub09c\ud588\ub2e4\ub294 \uae30\ub85d\uc774 \uc788\ub2e4. \uc138\uc870\ub294 \uc774\ub97c \ubd88\uc30d\ud788 \uc5ec\uaca8 \uacbd\ud61c\uacf5\uc8fc\uc5d0\uac8c \uc9d1\uc744 \uc9c0\uc5b4\uc8fc\uace0 \ubab0\uc218\ud55c \uc7ac\uc0b0\uacfc \ub178\ube44\ub97c \ud558\uc0ac\ud588\ub2e4. \ub610\ud55c \uc9c0\ub09c 2012\ub144 7\uc6d4 24\uc77c\uc5d0 \ud55c\uad6d\ud559\uc911\uc559\uc5f0\uad6c\uc6d0 \uc7a5\uc11c\uac01\uc5d0 \uae30\uc99d\ub41c \ud574\uc8fc \uc815\uc528 \ub300\uc885\uac00 \uc18c\uc7a5 \uace0\ubb38\uc11c \uac00\uc6b4\ub370 \uacbd\ud61c\uacf5\uc8fc\uac00 \uc790\uc2e0\uc758 \uc720\uc77c\ud55c \uc544\ub4e4 \uc815\ubbf8\uc218\uc5d0\uac8c \uc7ac\uc0b0\uc744 \uc0c1\uc18d\ud55c \ubd84\uc7ac\uae30(\u5206\u8ca1\u8a18)\uac00 \uacf5\uac1c\ub418\uc5c8\ub294\ub370, \uacf5\uc8fc \uc790\uc2e0\uc774 \uc8fd\uae30 \uc0ac\ud758 \uc804\uc778 \uc131\uc8855\ub144(1474\ub144) \uc74c\ub825 12\uc6d4 27\uc77c\uc5d0 \uc81c\uc791\ub41c \uac83\uc73c\ub85c '\uacbd\ud61c\uacf5\uc8fc\uc9c0\uc778(\u656c\u60e0\u516c\u4e3b\u4e4b\u5370)'\uc774\ub77c\ub294 \ubd89\uc740 \ub3c4\uc7a5\uc774 \ucc0d\ud600 \uc788\uc5b4 \uacbd\ud61c\uacf5\uc8fc\uac00 \uc8fd\uc744 \ub54c\uae4c\uc9c0 \uacf5\uc8fc\uc758 \uc2e0\ubd84\uc744 \uc720\uc9c0\ud588\ub2e4\ub294 \uc99d\uac70\ub85c \ucc44\ud0dd\ub418\uae30\ub3c4 \ud588\ub2e4. \uc608\uc885\uc5d0 \uc774\ub974\ub7ec\uc11c\ub294 \uacbd\ud61c\uacf5\uc8fc \ub0b4\uc678\uc758 \uc544\ub4e4 \uc815\ubbf8\uc218\ub97c \uc885\uce5c\uc758 \uc608\ub85c \uc11c\uc6a9\ud558\uc600\ub2e4. 1473\ub144(\uc131\uc8854) 12\uc6d4 30\uc77c \uc878\ud558\uc600\uc73c\uba70 \uc131\uc885\uc740 \ubd80\uc758(\u8cfb\u5100)\ub85c \uc300\u00b7\ucf69 \uc544\uc6b8\ub7ec 70\uc11d(\u78a9), \uc815\ud3ec(\u6b63\u5e03) 50\ud544, \uc885\uc774 1\ubc31 \uad8c, \uc11d\ud68c(\u77f3\u7070) 60\uc11d, \ucd09\ub78d(\u71ed\u881f) 30\uadfc\uc744 \ud558\uc0ac\ud558\uc600\ub2e4. \uacf5\uc8fc\uc758 \ubb18\ub294 \uacbd\uae30\ub3c4 \uace0\uc591\uc2dc \ub355\uc591\uad6c \ub300\uc790\ub3d9 \ub300\uc790\uace8\uc5d0 \uc788\ub2e4.", "answer": "\uc5ec\uc2b9", "sentence": "\uadf8\ub7ec\ub098 \uc2e4\ub85d\uc5d0\ub294 \uacbd\ud61c\uacf5\uc8fc\uac00 \uad00\ub178\uac00 \ub418\uc5c8\ub2e4\ub294 \uae30\ub85d\uc774 \uc5c6\uace0 \ub300\uc2e0 \uc815\uc885\uc774 \uc8fd\uc740 \ud6c4 \uba38\ub9ac\ub97c \uae4e\uace0 \uc5ec\uc2b9 \uc774 \ub418\uc5c8\ub294\ub370 \ubb34\ucc99 \uac00\ub09c\ud588\ub2e4\ub294 \uae30\ub85d\uc774 \uc788\ub2e4.", "paragraph_sentence": "\uc548\uc815\ubcf5\uc758 \uc21c\uc554\uc9d1\uc5d0\ub3c4 \uacbd\ud61c\uacf5\uc8fc\uac00 \uc7a5\ud765\uc758 \uad00\ub178\uac00 \ub418\uc5c8\ub2e4\ub294 \uae30\uc220\uc774 \uc788\ub2e4. \uadf8\ub7ec\ub098 \uc2e4\ub85d\uc5d0\ub294 \uacbd\ud61c\uacf5\uc8fc\uac00 \uad00\ub178\uac00 \ub418\uc5c8\ub2e4\ub294 \uae30\ub85d\uc774 \uc5c6\uace0 \ub300\uc2e0 \uc815\uc885\uc774 \uc8fd\uc740 \ud6c4 \uba38\ub9ac\ub97c \uae4e\uace0 \uc5ec\uc2b9 \uc774 \ub418\uc5c8\ub294\ub370 \ubb34\ucc99 \uac00\ub09c\ud588\ub2e4\ub294 \uae30\ub85d\uc774 \uc788\ub2e4. \uc138\uc870\ub294 \uc774\ub97c \ubd88\uc30d\ud788 \uc5ec\uaca8 \uacbd\ud61c\uacf5\uc8fc\uc5d0\uac8c \uc9d1\uc744 \uc9c0\uc5b4\uc8fc\uace0 \ubab0\uc218\ud55c \uc7ac\uc0b0\uacfc \ub178\ube44\ub97c \ud558\uc0ac\ud588\ub2e4. \ub610\ud55c \uc9c0\ub09c 2012\ub144 7\uc6d4 24\uc77c\uc5d0 \ud55c\uad6d\ud559\uc911\uc559\uc5f0\uad6c\uc6d0 \uc7a5\uc11c\uac01\uc5d0 \uae30\uc99d\ub41c \ud574\uc8fc \uc815\uc528 \ub300\uc885\uac00 \uc18c\uc7a5 \uace0\ubb38\uc11c \uac00\uc6b4\ub370 \uacbd\ud61c\uacf5\uc8fc\uac00 \uc790\uc2e0\uc758 \uc720\uc77c\ud55c \uc544\ub4e4 \uc815\ubbf8\uc218\uc5d0\uac8c \uc7ac\uc0b0\uc744 \uc0c1\uc18d\ud55c \ubd84\uc7ac\uae30(\u5206\u8ca1\u8a18)\uac00 \uacf5\uac1c\ub418\uc5c8\ub294\ub370, \uacf5\uc8fc \uc790\uc2e0\uc774 \uc8fd\uae30 \uc0ac\ud758 \uc804\uc778 \uc131\uc8855\ub144(1474\ub144) \uc74c\ub825 12\uc6d4 27\uc77c\uc5d0 \uc81c\uc791\ub41c \uac83\uc73c\ub85c '\uacbd\ud61c\uacf5\uc8fc\uc9c0\uc778(\u656c\u60e0\u516c\u4e3b\u4e4b\u5370)'\uc774\ub77c\ub294 \ubd89\uc740 \ub3c4\uc7a5\uc774 \ucc0d\ud600 \uc788\uc5b4 \uacbd\ud61c\uacf5\uc8fc\uac00 \uc8fd\uc744 \ub54c\uae4c\uc9c0 \uacf5\uc8fc\uc758 \uc2e0\ubd84\uc744 \uc720\uc9c0\ud588\ub2e4\ub294 \uc99d\uac70\ub85c \ucc44\ud0dd\ub418\uae30\ub3c4 \ud588\ub2e4. \uc608\uc885\uc5d0 \uc774\ub974\ub7ec\uc11c\ub294 \uacbd\ud61c\uacf5\uc8fc \ub0b4\uc678\uc758 \uc544\ub4e4 \uc815\ubbf8\uc218\ub97c \uc885\uce5c\uc758 \uc608\ub85c \uc11c\uc6a9\ud558\uc600\ub2e4. 1473\ub144(\uc131\uc8854) 12\uc6d4 30\uc77c \uc878\ud558\uc600\uc73c\uba70 \uc131\uc885\uc740 \ubd80\uc758(\u8cfb\u5100)\ub85c \uc300\u00b7\ucf69 \uc544\uc6b8\ub7ec 70\uc11d(\u78a9), \uc815\ud3ec(\u6b63\u5e03) 50\ud544, \uc885\uc774 1\ubc31 \uad8c, \uc11d\ud68c(\u77f3\u7070) 60\uc11d, \ucd09\ub78d(\u71ed\u881f) 30\uadfc\uc744 \ud558\uc0ac\ud558\uc600\ub2e4. \uacf5\uc8fc\uc758 \ubb18\ub294 \uacbd\uae30\ub3c4 \uace0\uc591\uc2dc \ub355\uc591\uad6c \ub300\uc790\ub3d9 \ub300\uc790\uace8\uc5d0 \uc788\ub2e4.", "paragraph_answer": "\uc548\uc815\ubcf5\uc758 \uc21c\uc554\uc9d1\uc5d0\ub3c4 \uacbd\ud61c\uacf5\uc8fc\uac00 \uc7a5\ud765\uc758 \uad00\ub178\uac00 \ub418\uc5c8\ub2e4\ub294 \uae30\uc220\uc774 \uc788\ub2e4. \uadf8\ub7ec\ub098 \uc2e4\ub85d\uc5d0\ub294 \uacbd\ud61c\uacf5\uc8fc\uac00 \uad00\ub178\uac00 \ub418\uc5c8\ub2e4\ub294 \uae30\ub85d\uc774 \uc5c6\uace0 \ub300\uc2e0 \uc815\uc885\uc774 \uc8fd\uc740 \ud6c4 \uba38\ub9ac\ub97c \uae4e\uace0 \uc5ec\uc2b9 \uc774 \ub418\uc5c8\ub294\ub370 \ubb34\ucc99 \uac00\ub09c\ud588\ub2e4\ub294 \uae30\ub85d\uc774 \uc788\ub2e4. \uc138\uc870\ub294 \uc774\ub97c \ubd88\uc30d\ud788 \uc5ec\uaca8 \uacbd\ud61c\uacf5\uc8fc\uc5d0\uac8c \uc9d1\uc744 \uc9c0\uc5b4\uc8fc\uace0 \ubab0\uc218\ud55c \uc7ac\uc0b0\uacfc \ub178\ube44\ub97c \ud558\uc0ac\ud588\ub2e4. \ub610\ud55c \uc9c0\ub09c 2012\ub144 7\uc6d4 24\uc77c\uc5d0 \ud55c\uad6d\ud559\uc911\uc559\uc5f0\uad6c\uc6d0 \uc7a5\uc11c\uac01\uc5d0 \uae30\uc99d\ub41c \ud574\uc8fc \uc815\uc528 \ub300\uc885\uac00 \uc18c\uc7a5 \uace0\ubb38\uc11c \uac00\uc6b4\ub370 \uacbd\ud61c\uacf5\uc8fc\uac00 \uc790\uc2e0\uc758 \uc720\uc77c\ud55c \uc544\ub4e4 \uc815\ubbf8\uc218\uc5d0\uac8c \uc7ac\uc0b0\uc744 \uc0c1\uc18d\ud55c \ubd84\uc7ac\uae30(\u5206\u8ca1\u8a18)\uac00 \uacf5\uac1c\ub418\uc5c8\ub294\ub370, \uacf5\uc8fc \uc790\uc2e0\uc774 \uc8fd\uae30 \uc0ac\ud758 \uc804\uc778 \uc131\uc8855\ub144(1474\ub144) \uc74c\ub825 12\uc6d4 27\uc77c\uc5d0 \uc81c\uc791\ub41c \uac83\uc73c\ub85c '\uacbd\ud61c\uacf5\uc8fc\uc9c0\uc778(\u656c\u60e0\u516c\u4e3b\u4e4b\u5370)'\uc774\ub77c\ub294 \ubd89\uc740 \ub3c4\uc7a5\uc774 \ucc0d\ud600 \uc788\uc5b4 \uacbd\ud61c\uacf5\uc8fc\uac00 \uc8fd\uc744 \ub54c\uae4c\uc9c0 \uacf5\uc8fc\uc758 \uc2e0\ubd84\uc744 \uc720\uc9c0\ud588\ub2e4\ub294 \uc99d\uac70\ub85c \ucc44\ud0dd\ub418\uae30\ub3c4 \ud588\ub2e4. \uc608\uc885\uc5d0 \uc774\ub974\ub7ec\uc11c\ub294 \uacbd\ud61c\uacf5\uc8fc \ub0b4\uc678\uc758 \uc544\ub4e4 \uc815\ubbf8\uc218\ub97c \uc885\uce5c\uc758 \uc608\ub85c \uc11c\uc6a9\ud558\uc600\ub2e4. 1473\ub144(\uc131\uc8854) 12\uc6d4 30\uc77c \uc878\ud558\uc600\uc73c\uba70 \uc131\uc885\uc740 \ubd80\uc758(\u8cfb\u5100)\ub85c \uc300\u00b7\ucf69 \uc544\uc6b8\ub7ec 70\uc11d(\u78a9), \uc815\ud3ec(\u6b63\u5e03) 50\ud544, \uc885\uc774 1\ubc31 \uad8c, \uc11d\ud68c(\u77f3\u7070) 60\uc11d, \ucd09\ub78d(\u71ed\u881f) 30\uadfc\uc744 \ud558\uc0ac\ud558\uc600\ub2e4. \uacf5\uc8fc\uc758 \ubb18\ub294 \uacbd\uae30\ub3c4 \uace0\uc591\uc2dc \ub355\uc591\uad6c \ub300\uc790\ub3d9 \ub300\uc790\uace8\uc5d0 \uc788\ub2e4.", "sentence_answer": "\uadf8\ub7ec\ub098 \uc2e4\ub85d\uc5d0\ub294 \uacbd\ud61c\uacf5\uc8fc\uac00 \uad00\ub178\uac00 \ub418\uc5c8\ub2e4\ub294 \uae30\ub85d\uc774 \uc5c6\uace0 \ub300\uc2e0 \uc815\uc885\uc774 \uc8fd\uc740 \ud6c4 \uba38\ub9ac\ub97c \uae4e\uace0 \uc5ec\uc2b9 \uc774 \ub418\uc5c8\ub294\ub370 \ubb34\ucc99 \uac00\ub09c\ud588\ub2e4\ub294 \uae30\ub85d\uc774 \uc788\ub2e4.", "paragraph_id": "6343285-2-1", "paragraph_question": "question: \uc2e4\ub85d\uc5d0\ub294 \uc815\uc885\uc774 \uc8fd\uc740 \ud6c4 \uacbd\ud61c\uacf5\uc8fc\uac00 \ubb34\uc5c7\uc774 \ub418\uc5c8\ub2e4\uace0 \uae30\ub85d\ub418\uc5b4 \uc788\ub294\uac00?, context: \uc548\uc815\ubcf5\uc758 \uc21c\uc554\uc9d1\uc5d0\ub3c4 \uacbd\ud61c\uacf5\uc8fc\uac00 \uc7a5\ud765\uc758 \uad00\ub178\uac00 \ub418\uc5c8\ub2e4\ub294 \uae30\uc220\uc774 \uc788\ub2e4. \uadf8\ub7ec\ub098 \uc2e4\ub85d\uc5d0\ub294 \uacbd\ud61c\uacf5\uc8fc\uac00 \uad00\ub178\uac00 \ub418\uc5c8\ub2e4\ub294 \uae30\ub85d\uc774 \uc5c6\uace0 \ub300\uc2e0 \uc815\uc885\uc774 \uc8fd\uc740 \ud6c4 \uba38\ub9ac\ub97c \uae4e\uace0 \uc5ec\uc2b9\uc774 \ub418\uc5c8\ub294\ub370 \ubb34\ucc99 \uac00\ub09c\ud588\ub2e4\ub294 \uae30\ub85d\uc774 \uc788\ub2e4. \uc138\uc870\ub294 \uc774\ub97c \ubd88\uc30d\ud788 \uc5ec\uaca8 \uacbd\ud61c\uacf5\uc8fc\uc5d0\uac8c \uc9d1\uc744 \uc9c0\uc5b4\uc8fc\uace0 \ubab0\uc218\ud55c \uc7ac\uc0b0\uacfc \ub178\ube44\ub97c \ud558\uc0ac\ud588\ub2e4. \ub610\ud55c \uc9c0\ub09c 2012\ub144 7\uc6d4 24\uc77c\uc5d0 \ud55c\uad6d\ud559\uc911\uc559\uc5f0\uad6c\uc6d0 \uc7a5\uc11c\uac01\uc5d0 \uae30\uc99d\ub41c \ud574\uc8fc \uc815\uc528 \ub300\uc885\uac00 \uc18c\uc7a5 \uace0\ubb38\uc11c \uac00\uc6b4\ub370 \uacbd\ud61c\uacf5\uc8fc\uac00 \uc790\uc2e0\uc758 \uc720\uc77c\ud55c \uc544\ub4e4 \uc815\ubbf8\uc218\uc5d0\uac8c \uc7ac\uc0b0\uc744 \uc0c1\uc18d\ud55c \ubd84\uc7ac\uae30(\u5206\u8ca1\u8a18)\uac00 \uacf5\uac1c\ub418\uc5c8\ub294\ub370, \uacf5\uc8fc \uc790\uc2e0\uc774 \uc8fd\uae30 \uc0ac\ud758 \uc804\uc778 \uc131\uc8855\ub144(1474\ub144) \uc74c\ub825 12\uc6d4 27\uc77c\uc5d0 \uc81c\uc791\ub41c \uac83\uc73c\ub85c '\uacbd\ud61c\uacf5\uc8fc\uc9c0\uc778(\u656c\u60e0\u516c\u4e3b\u4e4b\u5370)'\uc774\ub77c\ub294 \ubd89\uc740 \ub3c4\uc7a5\uc774 \ucc0d\ud600 \uc788\uc5b4 \uacbd\ud61c\uacf5\uc8fc\uac00 \uc8fd\uc744 \ub54c\uae4c\uc9c0 \uacf5\uc8fc\uc758 \uc2e0\ubd84\uc744 \uc720\uc9c0\ud588\ub2e4\ub294 \uc99d\uac70\ub85c \ucc44\ud0dd\ub418\uae30\ub3c4 \ud588\ub2e4. \uc608\uc885\uc5d0 \uc774\ub974\ub7ec\uc11c\ub294 \uacbd\ud61c\uacf5\uc8fc \ub0b4\uc678\uc758 \uc544\ub4e4 \uc815\ubbf8\uc218\ub97c \uc885\uce5c\uc758 \uc608\ub85c \uc11c\uc6a9\ud558\uc600\ub2e4. 1473\ub144(\uc131\uc8854) 12\uc6d4 30\uc77c \uc878\ud558\uc600\uc73c\uba70 \uc131\uc885\uc740 \ubd80\uc758(\u8cfb\u5100)\ub85c \uc300\u00b7\ucf69 \uc544\uc6b8\ub7ec 70\uc11d(\u78a9), \uc815\ud3ec(\u6b63\u5e03) 50\ud544, \uc885\uc774 1\ubc31 \uad8c, \uc11d\ud68c(\u77f3\u7070) 60\uc11d, \ucd09\ub78d(\u71ed\u881f) 30\uadfc\uc744 \ud558\uc0ac\ud558\uc600\ub2e4. \uacf5\uc8fc\uc758 \ubb18\ub294 \uacbd\uae30\ub3c4 \uace0\uc591\uc2dc \ub355\uc591\uad6c \ub300\uc790\ub3d9 \ub300\uc790\uace8\uc5d0 \uc788\ub2e4."} {"question": "\ubca1\ud130\uc758 \uc5f0\uad6c\ub97c \uc774\ub8e8\ub294 \uc218\ud559\uc758 \uc911\uc694\ud55c \uc138\uac1c\uc758 \ubd84\uc57c\ub294?", "paragraph": "\uc218 \ub300\uc2e0 \ubb38\uc790\ub97c \uc368\uc11c \ubb38\uc81c\ud574\uacb0\uc744 \uc27d\uac8c \ud558\ub294 \uac83\uacfc, \ub9c8\ucc2c\uac00\uc9c0\ub85c \uc218\ud559\uc801 \ubc95\uce59\uc744 \uc77c\ubc18\uc801\uc774\uace0 \uac04\uba85\ud558\uac8c \ub098\ud0c0\ub0b4\ub294 \uac83\uc744 \ud3ec\ud568\ud55c\ub2e4. \uace0\uc804\ub300\uc218\ud559\uc740 \ub300\uc218\ubc29\uc815\uc2dd \ubc0f \uc5f0\ub9bd\ubc29\uc815\uc2dd\uc758 \ud574\ubc95\uc5d0\uc11c \uc2dc\uc791\ud558\uc5ec \uad70, \ud658, \uccb4 \ub4f1\uc758 \ucd94\uc0c1\ub300\uc218\ud559\uc744 \uac70\uccd0 \ud604\ub300\uc5d0 \uc640\uc11c\ub294 \ub300\uc218\uacc4\uc758 \uad6c\uc870\ub97c \ubcf4\ub294 \uac83\uc744 \uc911\uc2ec\uc73c\ub85c \ud558\ub294 \uc120\ud615\ub300\uc218\ud559\uc73c\ub85c \uc804\uac1c\ub418\uc5c8\ub2e4. \uc218\uc758 \uc9d1\ud569\uc774\ub098 \ud568\uc218\uc640 \uac19\uc740 \ub9ce\uc740 \uc218\ud559\uc801 \ub300\uc0c1\ub4e4\uc740 \ub0b4\uc7ac\uc801\uc778 \uad6c\uc870\ub97c \ubcf4\uc778\ub2e4. \uc774\ub7ec\ud55c \ub300\uc0c1\ub4e4\uc758 \uad6c\uc870\uc801 \ud2b9\uc131\ub4e4\uc774 \uad70\ub860, \ud658\ub860, \uccb4\ub860 \uadf8\ub9ac\uace0 \uadf8 \uc678\uc758 \uc218\ub9ce\uc740 \ub300\uc218\uc801 \uad6c\uc870\ub4e4\uc744 \uc5f0\uad6c\ud558\uba74\uc11c \ub2e4\ub8e8\uc5b4\uc9c0\uba70, \uadf8\uac83\ub4e4 \ud558\ub098\ud558\ub098\uac00 \ub0b4\uc7ac\uc801 \uad6c\uc870\ub97c \uc9c0\ub2cc \uc218\ud559\uc801 \ub300\uc0c1\uc774\ub2e4. \uc774 \ubd84\uc57c\uc5d0\uc11c \uc911\uc694\ud55c \uac1c\ub150\uc740 \ubca1\ud130, \ubca1\ud130 \uacf5\uac04\uc73c\ub85c\uc758 \uc77c\ubc18\ud654, \uadf8\ub9ac\uace0 \uc120\ud615\ub300\uc218\ud559\uc5d0\uc11c\uc758 \uc9c0\uc2dd\ub4e4\uc774\ub2e4. \ubca1\ud130\uc758 \uc5f0\uad6c\uc5d0\ub294 \uc0b0\uc220, \ub300\uc218, \uae30\ud558\ub77c\ub294 \uc218\ud559\uc758 \uc911\uc694\ud55c \uc138\uac1c\uc758 \ubd84\uc57c\uac00 \uc870\ud569\ub418\uc5b4 \uc788\ub2e4. \ubca1\ud130 \ubbf8\uc801\ubd84\ud559\uc740 \uc5ec\uae30\uc5d0 \ud574\uc11d\ud559\uc758 \uc601\uc5ed\uc774 \ucd94\uac00\ub41c\ub2e4. \ud150\uc11c \ubbf8\uc801\ubd84\ud559\uc740 \ub300\uce6d\uc131\uacfc \ud68c\uc804\ucd95\uc758 \uc601\ud5a5 \uc544\ub798\uc5d0\uc11c \ubca1\ud130\uc758 \uc6c0\uc9c1\uc784\uc744 \uc5f0\uad6c\ud55c\ub2e4. \ub208\uae08\uc5c6\ub294 \uc790\uc640 \ucef4\ud37c\uc2a4\uc640 \uad00\ub828\ub41c \ub9ce\uc740 \uace0\ub300\uc758 \ubbf8\ud574\uacb0 \ubb38\uc81c\ub4e4\uc774 \uac08\ub8e8\uc544 \uc774\ub860\uc744 \uc0ac\uc6a9\ud558\uc5ec \ube44\ub85c\uc18c \ud574\uacb0\ub418\uc5c8\ub2e4.", "answer": "\uc0b0\uc220, \ub300\uc218, \uae30\ud558", "sentence": "\ubca1\ud130\uc758 \uc5f0\uad6c\uc5d0\ub294 \uc0b0\uc220, \ub300\uc218, \uae30\ud558 \ub77c\ub294 \uc218\ud559\uc758 \uc911\uc694\ud55c \uc138\uac1c\uc758 \ubd84\uc57c\uac00 \uc870\ud569\ub418\uc5b4 \uc788\ub2e4.", "paragraph_sentence": "\uc218 \ub300\uc2e0 \ubb38\uc790\ub97c \uc368\uc11c \ubb38\uc81c\ud574\uacb0\uc744 \uc27d\uac8c \ud558\ub294 \uac83\uacfc, \ub9c8\ucc2c\uac00\uc9c0\ub85c \uc218\ud559\uc801 \ubc95\uce59\uc744 \uc77c\ubc18\uc801\uc774\uace0 \uac04\uba85\ud558\uac8c \ub098\ud0c0\ub0b4\ub294 \uac83\uc744 \ud3ec\ud568\ud55c\ub2e4. \uace0\uc804\ub300\uc218\ud559\uc740 \ub300\uc218\ubc29\uc815\uc2dd \ubc0f \uc5f0\ub9bd\ubc29\uc815\uc2dd\uc758 \ud574\ubc95\uc5d0\uc11c \uc2dc\uc791\ud558\uc5ec \uad70, \ud658, \uccb4 \ub4f1\uc758 \ucd94\uc0c1\ub300\uc218\ud559\uc744 \uac70\uccd0 \ud604\ub300\uc5d0 \uc640\uc11c\ub294 \ub300\uc218\uacc4\uc758 \uad6c\uc870\ub97c \ubcf4\ub294 \uac83\uc744 \uc911\uc2ec\uc73c\ub85c \ud558\ub294 \uc120\ud615\ub300\uc218\ud559\uc73c\ub85c \uc804\uac1c\ub418\uc5c8\ub2e4. \uc218\uc758 \uc9d1\ud569\uc774\ub098 \ud568\uc218\uc640 \uac19\uc740 \ub9ce\uc740 \uc218\ud559\uc801 \ub300\uc0c1\ub4e4\uc740 \ub0b4\uc7ac\uc801\uc778 \uad6c\uc870\ub97c \ubcf4\uc778\ub2e4. \uc774\ub7ec\ud55c \ub300\uc0c1\ub4e4\uc758 \uad6c\uc870\uc801 \ud2b9\uc131\ub4e4\uc774 \uad70\ub860, \ud658\ub860, \uccb4\ub860 \uadf8\ub9ac\uace0 \uadf8 \uc678\uc758 \uc218\ub9ce\uc740 \ub300\uc218\uc801 \uad6c\uc870\ub4e4\uc744 \uc5f0\uad6c\ud558\uba74\uc11c \ub2e4\ub8e8\uc5b4\uc9c0\uba70, \uadf8\uac83\ub4e4 \ud558\ub098\ud558\ub098\uac00 \ub0b4\uc7ac\uc801 \uad6c\uc870\ub97c \uc9c0\ub2cc \uc218\ud559\uc801 \ub300\uc0c1\uc774\ub2e4. \uc774 \ubd84\uc57c\uc5d0\uc11c \uc911\uc694\ud55c \uac1c\ub150\uc740 \ubca1\ud130, \ubca1\ud130 \uacf5\uac04\uc73c\ub85c\uc758 \uc77c\ubc18\ud654, \uadf8\ub9ac\uace0 \uc120\ud615\ub300\uc218\ud559\uc5d0\uc11c\uc758 \uc9c0\uc2dd\ub4e4\uc774\ub2e4. \ubca1\ud130\uc758 \uc5f0\uad6c\uc5d0\ub294 \uc0b0\uc220, \ub300\uc218, \uae30\ud558 \ub77c\ub294 \uc218\ud559\uc758 \uc911\uc694\ud55c \uc138\uac1c\uc758 \ubd84\uc57c\uac00 \uc870\ud569\ub418\uc5b4 \uc788\ub2e4. \ubca1\ud130 \ubbf8\uc801\ubd84\ud559\uc740 \uc5ec\uae30\uc5d0 \ud574\uc11d\ud559\uc758 \uc601\uc5ed\uc774 \ucd94\uac00\ub41c\ub2e4. \ud150\uc11c \ubbf8\uc801\ubd84\ud559\uc740 \ub300\uce6d\uc131\uacfc \ud68c\uc804\ucd95\uc758 \uc601\ud5a5 \uc544\ub798\uc5d0\uc11c \ubca1\ud130\uc758 \uc6c0\uc9c1\uc784\uc744 \uc5f0\uad6c\ud55c\ub2e4. \ub208\uae08\uc5c6\ub294 \uc790\uc640 \ucef4\ud37c\uc2a4\uc640 \uad00\ub828\ub41c \ub9ce\uc740 \uace0\ub300\uc758 \ubbf8\ud574\uacb0 \ubb38\uc81c\ub4e4\uc774 \uac08\ub8e8\uc544 \uc774\ub860\uc744 \uc0ac\uc6a9\ud558\uc5ec \ube44\ub85c\uc18c \ud574\uacb0\ub418\uc5c8\ub2e4.", "paragraph_answer": "\uc218 \ub300\uc2e0 \ubb38\uc790\ub97c \uc368\uc11c \ubb38\uc81c\ud574\uacb0\uc744 \uc27d\uac8c \ud558\ub294 \uac83\uacfc, \ub9c8\ucc2c\uac00\uc9c0\ub85c \uc218\ud559\uc801 \ubc95\uce59\uc744 \uc77c\ubc18\uc801\uc774\uace0 \uac04\uba85\ud558\uac8c \ub098\ud0c0\ub0b4\ub294 \uac83\uc744 \ud3ec\ud568\ud55c\ub2e4. \uace0\uc804\ub300\uc218\ud559\uc740 \ub300\uc218\ubc29\uc815\uc2dd \ubc0f \uc5f0\ub9bd\ubc29\uc815\uc2dd\uc758 \ud574\ubc95\uc5d0\uc11c \uc2dc\uc791\ud558\uc5ec \uad70, \ud658, \uccb4 \ub4f1\uc758 \ucd94\uc0c1\ub300\uc218\ud559\uc744 \uac70\uccd0 \ud604\ub300\uc5d0 \uc640\uc11c\ub294 \ub300\uc218\uacc4\uc758 \uad6c\uc870\ub97c \ubcf4\ub294 \uac83\uc744 \uc911\uc2ec\uc73c\ub85c \ud558\ub294 \uc120\ud615\ub300\uc218\ud559\uc73c\ub85c \uc804\uac1c\ub418\uc5c8\ub2e4. \uc218\uc758 \uc9d1\ud569\uc774\ub098 \ud568\uc218\uc640 \uac19\uc740 \ub9ce\uc740 \uc218\ud559\uc801 \ub300\uc0c1\ub4e4\uc740 \ub0b4\uc7ac\uc801\uc778 \uad6c\uc870\ub97c \ubcf4\uc778\ub2e4. \uc774\ub7ec\ud55c \ub300\uc0c1\ub4e4\uc758 \uad6c\uc870\uc801 \ud2b9\uc131\ub4e4\uc774 \uad70\ub860, \ud658\ub860, \uccb4\ub860 \uadf8\ub9ac\uace0 \uadf8 \uc678\uc758 \uc218\ub9ce\uc740 \ub300\uc218\uc801 \uad6c\uc870\ub4e4\uc744 \uc5f0\uad6c\ud558\uba74\uc11c \ub2e4\ub8e8\uc5b4\uc9c0\uba70, \uadf8\uac83\ub4e4 \ud558\ub098\ud558\ub098\uac00 \ub0b4\uc7ac\uc801 \uad6c\uc870\ub97c \uc9c0\ub2cc \uc218\ud559\uc801 \ub300\uc0c1\uc774\ub2e4. \uc774 \ubd84\uc57c\uc5d0\uc11c \uc911\uc694\ud55c \uac1c\ub150\uc740 \ubca1\ud130, \ubca1\ud130 \uacf5\uac04\uc73c\ub85c\uc758 \uc77c\ubc18\ud654, \uadf8\ub9ac\uace0 \uc120\ud615\ub300\uc218\ud559\uc5d0\uc11c\uc758 \uc9c0\uc2dd\ub4e4\uc774\ub2e4. \ubca1\ud130\uc758 \uc5f0\uad6c\uc5d0\ub294 \uc0b0\uc220, \ub300\uc218, \uae30\ud558 \ub77c\ub294 \uc218\ud559\uc758 \uc911\uc694\ud55c \uc138\uac1c\uc758 \ubd84\uc57c\uac00 \uc870\ud569\ub418\uc5b4 \uc788\ub2e4. \ubca1\ud130 \ubbf8\uc801\ubd84\ud559\uc740 \uc5ec\uae30\uc5d0 \ud574\uc11d\ud559\uc758 \uc601\uc5ed\uc774 \ucd94\uac00\ub41c\ub2e4. \ud150\uc11c \ubbf8\uc801\ubd84\ud559\uc740 \ub300\uce6d\uc131\uacfc \ud68c\uc804\ucd95\uc758 \uc601\ud5a5 \uc544\ub798\uc5d0\uc11c \ubca1\ud130\uc758 \uc6c0\uc9c1\uc784\uc744 \uc5f0\uad6c\ud55c\ub2e4. \ub208\uae08\uc5c6\ub294 \uc790\uc640 \ucef4\ud37c\uc2a4\uc640 \uad00\ub828\ub41c \ub9ce\uc740 \uace0\ub300\uc758 \ubbf8\ud574\uacb0 \ubb38\uc81c\ub4e4\uc774 \uac08\ub8e8\uc544 \uc774\ub860\uc744 \uc0ac\uc6a9\ud558\uc5ec \ube44\ub85c\uc18c \ud574\uacb0\ub418\uc5c8\ub2e4.", "sentence_answer": "\ubca1\ud130\uc758 \uc5f0\uad6c\uc5d0\ub294 \uc0b0\uc220, \ub300\uc218, \uae30\ud558 \ub77c\ub294 \uc218\ud559\uc758 \uc911\uc694\ud55c \uc138\uac1c\uc758 \ubd84\uc57c\uac00 \uc870\ud569\ub418\uc5b4 \uc788\ub2e4.", "paragraph_id": "6532557-0-1", "paragraph_question": "question: \ubca1\ud130\uc758 \uc5f0\uad6c\ub97c \uc774\ub8e8\ub294 \uc218\ud559\uc758 \uc911\uc694\ud55c \uc138\uac1c\uc758 \ubd84\uc57c\ub294?, context: \uc218 \ub300\uc2e0 \ubb38\uc790\ub97c \uc368\uc11c \ubb38\uc81c\ud574\uacb0\uc744 \uc27d\uac8c \ud558\ub294 \uac83\uacfc, \ub9c8\ucc2c\uac00\uc9c0\ub85c \uc218\ud559\uc801 \ubc95\uce59\uc744 \uc77c\ubc18\uc801\uc774\uace0 \uac04\uba85\ud558\uac8c \ub098\ud0c0\ub0b4\ub294 \uac83\uc744 \ud3ec\ud568\ud55c\ub2e4. \uace0\uc804\ub300\uc218\ud559\uc740 \ub300\uc218\ubc29\uc815\uc2dd \ubc0f \uc5f0\ub9bd\ubc29\uc815\uc2dd\uc758 \ud574\ubc95\uc5d0\uc11c \uc2dc\uc791\ud558\uc5ec \uad70, \ud658, \uccb4 \ub4f1\uc758 \ucd94\uc0c1\ub300\uc218\ud559\uc744 \uac70\uccd0 \ud604\ub300\uc5d0 \uc640\uc11c\ub294 \ub300\uc218\uacc4\uc758 \uad6c\uc870\ub97c \ubcf4\ub294 \uac83\uc744 \uc911\uc2ec\uc73c\ub85c \ud558\ub294 \uc120\ud615\ub300\uc218\ud559\uc73c\ub85c \uc804\uac1c\ub418\uc5c8\ub2e4. \uc218\uc758 \uc9d1\ud569\uc774\ub098 \ud568\uc218\uc640 \uac19\uc740 \ub9ce\uc740 \uc218\ud559\uc801 \ub300\uc0c1\ub4e4\uc740 \ub0b4\uc7ac\uc801\uc778 \uad6c\uc870\ub97c \ubcf4\uc778\ub2e4. \uc774\ub7ec\ud55c \ub300\uc0c1\ub4e4\uc758 \uad6c\uc870\uc801 \ud2b9\uc131\ub4e4\uc774 \uad70\ub860, \ud658\ub860, \uccb4\ub860 \uadf8\ub9ac\uace0 \uadf8 \uc678\uc758 \uc218\ub9ce\uc740 \ub300\uc218\uc801 \uad6c\uc870\ub4e4\uc744 \uc5f0\uad6c\ud558\uba74\uc11c \ub2e4\ub8e8\uc5b4\uc9c0\uba70, \uadf8\uac83\ub4e4 \ud558\ub098\ud558\ub098\uac00 \ub0b4\uc7ac\uc801 \uad6c\uc870\ub97c \uc9c0\ub2cc \uc218\ud559\uc801 \ub300\uc0c1\uc774\ub2e4. \uc774 \ubd84\uc57c\uc5d0\uc11c \uc911\uc694\ud55c \uac1c\ub150\uc740 \ubca1\ud130, \ubca1\ud130 \uacf5\uac04\uc73c\ub85c\uc758 \uc77c\ubc18\ud654, \uadf8\ub9ac\uace0 \uc120\ud615\ub300\uc218\ud559\uc5d0\uc11c\uc758 \uc9c0\uc2dd\ub4e4\uc774\ub2e4. \ubca1\ud130\uc758 \uc5f0\uad6c\uc5d0\ub294 \uc0b0\uc220, \ub300\uc218, \uae30\ud558\ub77c\ub294 \uc218\ud559\uc758 \uc911\uc694\ud55c \uc138\uac1c\uc758 \ubd84\uc57c\uac00 \uc870\ud569\ub418\uc5b4 \uc788\ub2e4. \ubca1\ud130 \ubbf8\uc801\ubd84\ud559\uc740 \uc5ec\uae30\uc5d0 \ud574\uc11d\ud559\uc758 \uc601\uc5ed\uc774 \ucd94\uac00\ub41c\ub2e4. \ud150\uc11c \ubbf8\uc801\ubd84\ud559\uc740 \ub300\uce6d\uc131\uacfc \ud68c\uc804\ucd95\uc758 \uc601\ud5a5 \uc544\ub798\uc5d0\uc11c \ubca1\ud130\uc758 \uc6c0\uc9c1\uc784\uc744 \uc5f0\uad6c\ud55c\ub2e4. \ub208\uae08\uc5c6\ub294 \uc790\uc640 \ucef4\ud37c\uc2a4\uc640 \uad00\ub828\ub41c \ub9ce\uc740 \uace0\ub300\uc758 \ubbf8\ud574\uacb0 \ubb38\uc81c\ub4e4\uc774 \uac08\ub8e8\uc544 \uc774\ub860\uc744 \uc0ac\uc6a9\ud558\uc5ec \ube44\ub85c\uc18c \ud574\uacb0\ub418\uc5c8\ub2e4."} {"question": "\uc870\uc120\uc758 \uc655\uc790\uc778 \ubd09\ub9bd\ub300\uad70\uacfc \uc778\ud3c9\ub300\uad70, \uc138\uc790\ube48 \uc7a5\uc528\uac00 \ud53c\uc2e0\ud574 \uc788\ub358 \uc7a5\uc18c\ub294 \uc5b4\ub290 \uacf3\uc778\uac00?", "paragraph": "\uac19\uc740 \ud574\uc5d0\ub294 \uc870\uc120\uc744 \uce68\ub7b5\ud558\uc5ec \ubcd1\uc790\ud638\ub780\uc744 \uc77c\uc73c\ucf30\uc73c\uba70 \ud3c9\uc548\ub3c4\uc640 \ud669\ud574\ub3c4, \ud568\uacbd\ub3c4 \uc9c0\uc5ed\uc744 \uc57d\ud0c8\ud558\uc600\ub2e4. \uc774\uc5b4\uc11c \uc870\uc120\uc758 \uc655\uc790\uc778 \ubd09\ub9bc\ub300\uad70\uacfc \uc778\ud3c9\ub300\uad70, \uadf8\ub9ac\uace0 \uc138\uc790\ube48 \uac15\uc528\uac00 \ud53c\uc2e0\ud574 \uc788\ub358 \uac15\ud654\ub3c4\ub97c \uacf5\ub7b5\ud558\uc600\uc73c\ub098 \ud64d\ud0c0\uc774\uc9c0\uac00 \uc5c4\uba85\uc744 \ub0b4\ub824 \uc870\uc120\uc758 \uc655\uc871\ub4e4\uc744 \uc0b4\uc721\ud558\uc9c0 \ub9d0\ub77c\ud558\ub2c8 \uadf8 \uba85\uc744 \ucad3\uc544 \ud3ec\uaca9\uc744 \ud558\ub294 \ub300\uc2e0 \uc131\uc744 \uacb9\uacb9\uc774 \ud3ec\uc704\ud558\uc600\ub2e4. \uacb0\uad6d \uac15\ud654\ub97c \uc9c0\ud0a4\ub358 \ubd09\ub9bc\ub300\uad70\uc740 \ub18d\uc131\uc744 \ud480\uace0 \ub3c4\ub974\uace4\uc744 \uc131 \uc548\uc73c\ub85c \uc601\uc811\ud558\uc600\uace0 \ub3c4\ub974\uace4\uc740 \ub300\uad70\ub4e4\uacfc \uc138\uc790\ube48 \uac15\uc528\ub97c \uc0ac\ub85c\uc7a1\uc544 \ub0a8\ud55c\uc0b0\uc131\uc5d0 \uc228\uc5b4\uc788\ub358 \uc870\uc120\uc758 \uad6d\uc655 \uc778\uc870\ub97c \uc555\ubc15\ud558\uc600\ub2e4. \uc774\ub4ec\ud574\uc778 1637\ub144(\uc22d\ub355 2\ub144, \uc870\uc120 \uc778\uc870 14\ub144) 1\uc6d4 30\uc77c, \uc778\uc870\ub294 \ub0a8\ud55c\uc0b0\uc131\uc5d0\uc11c \ub098\uc640 \uc0bc\uc804\ub3c4\uc5d0\uc11c \ud64d\ud0c0\uc774\uc9c0\uc5d0\uac8c \uc0bc\uada4\uad6c\uace0\ub450\ub97c \uc62c\ub838\ub2e4. \uc804\uc7c1\uc774 \ub05d\ub098\uc790 \uccad\uad70\uc740 \uc870\uc120\uc778 \ud3ec\ub85c\uc640 \uc778\uc0bc, \uae08 \ub4f1\uc758 \uacf5\ubb3c\uacfc \uacf5\ub140 \uc218\ubc31\uc5ec \uba85\uc744 \uc0ac\ub85c\uc7a1\uc544 \uc218\ub3c4\uc778 \ubb35\ub358\uc73c\ub85c \uadc0\ud658\ud558\uc600\ub2e4.", "answer": "\uac15\ud654\ub3c4", "sentence": "\uc774\uc5b4\uc11c \uc870\uc120\uc758 \uc655\uc790\uc778 \ubd09\ub9bc\ub300\uad70\uacfc \uc778\ud3c9\ub300\uad70, \uadf8\ub9ac\uace0 \uc138\uc790\ube48 \uac15\uc528\uac00 \ud53c\uc2e0\ud574 \uc788\ub358 \uac15\ud654\ub3c4 \ub97c \uacf5\ub7b5\ud558\uc600\uc73c\ub098 \ud64d\ud0c0\uc774\uc9c0\uac00 \uc5c4\uba85\uc744 \ub0b4\ub824 \uc870\uc120\uc758 \uc655\uc871\ub4e4\uc744 \uc0b4\uc721\ud558\uc9c0 \ub9d0\ub77c\ud558\ub2c8 \uadf8 \uba85\uc744 \ucad3\uc544 \ud3ec\uaca9\uc744 \ud558\ub294 \ub300\uc2e0 \uc131\uc744 \uacb9\uacb9\uc774 \ud3ec\uc704\ud558\uc600\ub2e4.", "paragraph_sentence": "\uac19\uc740 \ud574\uc5d0\ub294 \uc870\uc120\uc744 \uce68\ub7b5\ud558\uc5ec \ubcd1\uc790\ud638\ub780\uc744 \uc77c\uc73c\ucf30\uc73c\uba70 \ud3c9\uc548\ub3c4\uc640 \ud669\ud574\ub3c4, \ud568\uacbd\ub3c4 \uc9c0\uc5ed\uc744 \uc57d\ud0c8\ud558\uc600\ub2e4. \uc774\uc5b4\uc11c \uc870\uc120\uc758 \uc655\uc790\uc778 \ubd09\ub9bc\ub300\uad70\uacfc \uc778\ud3c9\ub300\uad70, \uadf8\ub9ac\uace0 \uc138\uc790\ube48 \uac15\uc528\uac00 \ud53c\uc2e0\ud574 \uc788\ub358 \uac15\ud654\ub3c4 \ub97c \uacf5\ub7b5\ud558\uc600\uc73c\ub098 \ud64d\ud0c0\uc774\uc9c0\uac00 \uc5c4\uba85\uc744 \ub0b4\ub824 \uc870\uc120\uc758 \uc655\uc871\ub4e4\uc744 \uc0b4\uc721\ud558\uc9c0 \ub9d0\ub77c\ud558\ub2c8 \uadf8 \uba85\uc744 \ucad3\uc544 \ud3ec\uaca9\uc744 \ud558\ub294 \ub300\uc2e0 \uc131\uc744 \uacb9\uacb9\uc774 \ud3ec\uc704\ud558\uc600\ub2e4. \uacb0\uad6d \uac15\ud654\ub97c \uc9c0\ud0a4\ub358 \ubd09\ub9bc\ub300\uad70\uc740 \ub18d\uc131\uc744 \ud480\uace0 \ub3c4\ub974\uace4\uc744 \uc131 \uc548\uc73c\ub85c \uc601\uc811\ud558\uc600\uace0 \ub3c4\ub974\uace4\uc740 \ub300\uad70\ub4e4\uacfc \uc138\uc790\ube48 \uac15\uc528\ub97c \uc0ac\ub85c\uc7a1\uc544 \ub0a8\ud55c\uc0b0\uc131\uc5d0 \uc228\uc5b4\uc788\ub358 \uc870\uc120\uc758 \uad6d\uc655 \uc778\uc870\ub97c \uc555\ubc15\ud558\uc600\ub2e4. \uc774\ub4ec\ud574\uc778 1637\ub144(\uc22d\ub355 2\ub144, \uc870\uc120 \uc778\uc870 14\ub144) 1\uc6d4 30\uc77c, \uc778\uc870\ub294 \ub0a8\ud55c\uc0b0\uc131\uc5d0\uc11c \ub098\uc640 \uc0bc\uc804\ub3c4\uc5d0\uc11c \ud64d\ud0c0\uc774\uc9c0\uc5d0\uac8c \uc0bc\uada4\uad6c\uace0\ub450\ub97c \uc62c\ub838\ub2e4. \uc804\uc7c1\uc774 \ub05d\ub098\uc790 \uccad\uad70\uc740 \uc870\uc120\uc778 \ud3ec\ub85c\uc640 \uc778\uc0bc, \uae08 \ub4f1\uc758 \uacf5\ubb3c\uacfc \uacf5\ub140 \uc218\ubc31\uc5ec \uba85\uc744 \uc0ac\ub85c\uc7a1\uc544 \uc218\ub3c4\uc778 \ubb35\ub358\uc73c\ub85c \uadc0\ud658\ud558\uc600\ub2e4.", "paragraph_answer": "\uac19\uc740 \ud574\uc5d0\ub294 \uc870\uc120\uc744 \uce68\ub7b5\ud558\uc5ec \ubcd1\uc790\ud638\ub780\uc744 \uc77c\uc73c\ucf30\uc73c\uba70 \ud3c9\uc548\ub3c4\uc640 \ud669\ud574\ub3c4, \ud568\uacbd\ub3c4 \uc9c0\uc5ed\uc744 \uc57d\ud0c8\ud558\uc600\ub2e4. \uc774\uc5b4\uc11c \uc870\uc120\uc758 \uc655\uc790\uc778 \ubd09\ub9bc\ub300\uad70\uacfc \uc778\ud3c9\ub300\uad70, \uadf8\ub9ac\uace0 \uc138\uc790\ube48 \uac15\uc528\uac00 \ud53c\uc2e0\ud574 \uc788\ub358 \uac15\ud654\ub3c4 \ub97c \uacf5\ub7b5\ud558\uc600\uc73c\ub098 \ud64d\ud0c0\uc774\uc9c0\uac00 \uc5c4\uba85\uc744 \ub0b4\ub824 \uc870\uc120\uc758 \uc655\uc871\ub4e4\uc744 \uc0b4\uc721\ud558\uc9c0 \ub9d0\ub77c\ud558\ub2c8 \uadf8 \uba85\uc744 \ucad3\uc544 \ud3ec\uaca9\uc744 \ud558\ub294 \ub300\uc2e0 \uc131\uc744 \uacb9\uacb9\uc774 \ud3ec\uc704\ud558\uc600\ub2e4. \uacb0\uad6d \uac15\ud654\ub97c \uc9c0\ud0a4\ub358 \ubd09\ub9bc\ub300\uad70\uc740 \ub18d\uc131\uc744 \ud480\uace0 \ub3c4\ub974\uace4\uc744 \uc131 \uc548\uc73c\ub85c \uc601\uc811\ud558\uc600\uace0 \ub3c4\ub974\uace4\uc740 \ub300\uad70\ub4e4\uacfc \uc138\uc790\ube48 \uac15\uc528\ub97c \uc0ac\ub85c\uc7a1\uc544 \ub0a8\ud55c\uc0b0\uc131\uc5d0 \uc228\uc5b4\uc788\ub358 \uc870\uc120\uc758 \uad6d\uc655 \uc778\uc870\ub97c \uc555\ubc15\ud558\uc600\ub2e4. \uc774\ub4ec\ud574\uc778 1637\ub144(\uc22d\ub355 2\ub144, \uc870\uc120 \uc778\uc870 14\ub144) 1\uc6d4 30\uc77c, \uc778\uc870\ub294 \ub0a8\ud55c\uc0b0\uc131\uc5d0\uc11c \ub098\uc640 \uc0bc\uc804\ub3c4\uc5d0\uc11c \ud64d\ud0c0\uc774\uc9c0\uc5d0\uac8c \uc0bc\uada4\uad6c\uace0\ub450\ub97c \uc62c\ub838\ub2e4. \uc804\uc7c1\uc774 \ub05d\ub098\uc790 \uccad\uad70\uc740 \uc870\uc120\uc778 \ud3ec\ub85c\uc640 \uc778\uc0bc, \uae08 \ub4f1\uc758 \uacf5\ubb3c\uacfc \uacf5\ub140 \uc218\ubc31\uc5ec \uba85\uc744 \uc0ac\ub85c\uc7a1\uc544 \uc218\ub3c4\uc778 \ubb35\ub358\uc73c\ub85c \uadc0\ud658\ud558\uc600\ub2e4.", "sentence_answer": "\uc774\uc5b4\uc11c \uc870\uc120\uc758 \uc655\uc790\uc778 \ubd09\ub9bc\ub300\uad70\uacfc \uc778\ud3c9\ub300\uad70, \uadf8\ub9ac\uace0 \uc138\uc790\ube48 \uac15\uc528\uac00 \ud53c\uc2e0\ud574 \uc788\ub358 \uac15\ud654\ub3c4 \ub97c \uacf5\ub7b5\ud558\uc600\uc73c\ub098 \ud64d\ud0c0\uc774\uc9c0\uac00 \uc5c4\uba85\uc744 \ub0b4\ub824 \uc870\uc120\uc758 \uc655\uc871\ub4e4\uc744 \uc0b4\uc721\ud558\uc9c0 \ub9d0\ub77c\ud558\ub2c8 \uadf8 \uba85\uc744 \ucad3\uc544 \ud3ec\uaca9\uc744 \ud558\ub294 \ub300\uc2e0 \uc131\uc744 \uacb9\uacb9\uc774 \ud3ec\uc704\ud558\uc600\ub2e4.", "paragraph_id": "6528619-1-0", "paragraph_question": "question: \uc870\uc120\uc758 \uc655\uc790\uc778 \ubd09\ub9bd\ub300\uad70\uacfc \uc778\ud3c9\ub300\uad70, \uc138\uc790\ube48 \uc7a5\uc528\uac00 \ud53c\uc2e0\ud574 \uc788\ub358 \uc7a5\uc18c\ub294 \uc5b4\ub290 \uacf3\uc778\uac00?, context: \uac19\uc740 \ud574\uc5d0\ub294 \uc870\uc120\uc744 \uce68\ub7b5\ud558\uc5ec \ubcd1\uc790\ud638\ub780\uc744 \uc77c\uc73c\ucf30\uc73c\uba70 \ud3c9\uc548\ub3c4\uc640 \ud669\ud574\ub3c4, \ud568\uacbd\ub3c4 \uc9c0\uc5ed\uc744 \uc57d\ud0c8\ud558\uc600\ub2e4. \uc774\uc5b4\uc11c \uc870\uc120\uc758 \uc655\uc790\uc778 \ubd09\ub9bc\ub300\uad70\uacfc \uc778\ud3c9\ub300\uad70, \uadf8\ub9ac\uace0 \uc138\uc790\ube48 \uac15\uc528\uac00 \ud53c\uc2e0\ud574 \uc788\ub358 \uac15\ud654\ub3c4\ub97c \uacf5\ub7b5\ud558\uc600\uc73c\ub098 \ud64d\ud0c0\uc774\uc9c0\uac00 \uc5c4\uba85\uc744 \ub0b4\ub824 \uc870\uc120\uc758 \uc655\uc871\ub4e4\uc744 \uc0b4\uc721\ud558\uc9c0 \ub9d0\ub77c\ud558\ub2c8 \uadf8 \uba85\uc744 \ucad3\uc544 \ud3ec\uaca9\uc744 \ud558\ub294 \ub300\uc2e0 \uc131\uc744 \uacb9\uacb9\uc774 \ud3ec\uc704\ud558\uc600\ub2e4. \uacb0\uad6d \uac15\ud654\ub97c \uc9c0\ud0a4\ub358 \ubd09\ub9bc\ub300\uad70\uc740 \ub18d\uc131\uc744 \ud480\uace0 \ub3c4\ub974\uace4\uc744 \uc131 \uc548\uc73c\ub85c \uc601\uc811\ud558\uc600\uace0 \ub3c4\ub974\uace4\uc740 \ub300\uad70\ub4e4\uacfc \uc138\uc790\ube48 \uac15\uc528\ub97c \uc0ac\ub85c\uc7a1\uc544 \ub0a8\ud55c\uc0b0\uc131\uc5d0 \uc228\uc5b4\uc788\ub358 \uc870\uc120\uc758 \uad6d\uc655 \uc778\uc870\ub97c \uc555\ubc15\ud558\uc600\ub2e4. \uc774\ub4ec\ud574\uc778 1637\ub144(\uc22d\ub355 2\ub144, \uc870\uc120 \uc778\uc870 14\ub144) 1\uc6d4 30\uc77c, \uc778\uc870\ub294 \ub0a8\ud55c\uc0b0\uc131\uc5d0\uc11c \ub098\uc640 \uc0bc\uc804\ub3c4\uc5d0\uc11c \ud64d\ud0c0\uc774\uc9c0\uc5d0\uac8c \uc0bc\uada4\uad6c\uace0\ub450\ub97c \uc62c\ub838\ub2e4. \uc804\uc7c1\uc774 \ub05d\ub098\uc790 \uccad\uad70\uc740 \uc870\uc120\uc778 \ud3ec\ub85c\uc640 \uc778\uc0bc, \uae08 \ub4f1\uc758 \uacf5\ubb3c\uacfc \uacf5\ub140 \uc218\ubc31\uc5ec \uba85\uc744 \uc0ac\ub85c\uc7a1\uc544 \uc218\ub3c4\uc778 \ubb35\ub358\uc73c\ub85c \uadc0\ud658\ud558\uc600\ub2e4."} {"question": "\uacfc\uac70 \ub192\uc740\uc9c0\uc704\uc758 \uc0ac\ub78c\uc774 \uc8fd\uc73c\uba74 \uadf8 \uc2e0\ud558\ub4e4\uacfc \ubb3c\uac74\ub4e4\uc744 \uac19\uc774 \ubb3b\ub294 \ud48d\uc2b5\uc744 \ubb34\uc5c7\uc774\ub77c\uace0 \ud558\ub294\uac00?", "paragraph": "1612\ub144 11\uc6d4 17\uc77c \ub2f9\uc2dc \ub9cc\uc8fc \uc77c\ub300\uc5d0\uc11c \uc138\ub825\uc744 \ub5a8\uce58\ub358 \uac74\uc8fc\uc5ec\uc9c4\uc758 \ucd94\uc7a5 \ub204\ub974\ud558\uce58\uc758 \uc5f4\ub124 \ubc88\uc9f8 \uc544\ub4e4\ub85c \ud0dc\uc5b4\ub0ac\ub2e4. \uadf8\uc758 \uc774\ub984 \ub3c4\ub974\uace4\uc740 \ub9cc\uc8fc\uc5b4\ub85c \uc624\uc18c\ub9ac\ub97c \ub73b\ud55c\ub2e4. \ub3c4\ub974\uace4\uc758 \uc5b4\uba38\ub2c8\ub294 \ub204\ub974\ud558\uce58\uc758 \ub124 \ubc88\uc9f8 \uc815\uc2e4, \uc989 \ub300\ubcf5\uc9c4(\u5927\u798f\u6649)\uc73c\ub85c \ubd09\ud574\uc838\uc788\ub358 \uc6b8\ub77c\ub098\ub77c\uc528\uc600\ub2e4. \uadf8\ub140\ub294 \uc774\ubbf8 \uc5b4\ub9b0 \ub098\uc774\uc5d0 \ub204\ub974\ud558\uce58\uc5d0\uac8c \uc2dc\uc9d1\uc628 \uc774\ud6c4 \uc904\uace7 \ub204\ub974\ud558\uce58\uc758 \ud070 \ucd1d\uc560\ub97c \ubc1b\uc558\uace0 \uadf8\ub140\uc758 \uc790\uc2dd\uc778 \ub3c4\ub974\uace4 \ud615\uc81c \uc5ed\uc2dc \ub369\ub2ec\uc544 \uc544\ubc84\uc9c0\uc758 \uc0ac\ub791\uc744 \ubc1b\uc558\ub2e4. \ub3c4\ub974\uace4\uc774 5\uc0b4 \ub54c\uc778 1616\ub144 1\uc6d4, \ub204\ub974\ud558\uce58\ub294 \uc2a4\uc2a4\ub85c \ud55c\uc774\ub77c \uce6d\ud558\uba70 \ub098\ub77c \uc774\ub984\uc744 \uae08(\u91d1)\uc774\ub77c \uc815\ud558\ub2c8 \uc774\uac83\uc774 \ubc14\ub85c \uccad\ub098\ub77c\uc758 \uc804\uc2e0\uc778 \ud6c4\uae08\uc774\ub2e4. 1626\ub144(\ucc9c\uba85 11\ub144) \ub204\ub974\ud558\uce58\ub294 \uc601\uc6d0\uc131 \uacf5\uaca9 \ub3c4\uc911 \uba85\ub098\ub77c\uc758 \uc7a5\uc218 \uc6d0\uc22d\ud658\uc758 \ud64d\uc758\ud3ec\ub97c \ub9de\uace0 \uc911\uc0c1\uc744 \uc785\uace0 \uc5bc\ub9c8 \ub4a4\uc5d0 \uc138\uc0c1\uc744 \ub5a0\ub0ac\ub2e4. \uc774\uc5d0 \ub204\ub974\ud558\uce58\uc758 \uc544\ub4e4\ub4e4\uacfc \uc870\uce74\ub4e4\uc740 \ucc28\uae30 \ud55c\uc73c\ub85c \ub204\ub974\ud558\uce58\uc758 8\ub0a8\uc774\uc790 \ub3c4\ub974\uace4\ubcf4\ub2e4 \uc2a4\ubb34 \uc0b4\uc774 \ub9ce\ub358 \uc774\ubcf5\ud615 \ud64d\ud0c0\uc774\uc9c0\ub97c \uc639\ub9bd\ud558\uc600\ub2e4. \ud64d\ud0c0\uc774\uc9c0\ub97c \ube44\ub86f\ud55c \ud328\ub975(\u8c9d\u52d2)\ub4e4\uc740 \ub300\ubcf5\uc9c4 \uc6b8\ub77c\ub098\ub77c\uc528\uc5d0\uac8c \uc21c\uc7a5\uc744 \ud558\uc5ec \ub204\ub974\ud558\uce58\uc758 \ub4a4\ub97c \ub530\ub97c \uac83\uc744 \uac15\uc694\ud558\uc600\uace0 \uacb0\uad6d \uc6b8\ub77c\ub098\ub77c\uc528\ub294 \uc21c\uc7a5\ub418\uc5c8\ub2e4. \ub3c4\ub974\uace4\uacfc \uadf8\uc758 \ud615 \uc544\uc9c0\uac70, \uc5b4\ub9b0 \ub3d9\uc0dd \ub3c4\ub3c4\ub294 \ud55c \uc21c\uac04\uc5d0 \uc544\ubc84\uc9c0\uc5d0 \uc774\uc5b4 \uc5b4\uba38\ub2c8\ub97c \uc783\uc5c8\uc73c\ub098 \ud64d\ud0c0\uc774\uc9c0\ub294 \ub3c4\ub974\uace4 \ud615\uc81c\ub97c \ubaa8\ub450 \ud328\ub975\uc5d0 \ubd09\ud558\uc5ec \uadf8\ub4e4\uc744 \ub2ec\ub7ac\uace0 \uc870\uc815\uc5d0 \ucd9c\uc0ac\ucf00 \ud558\uc600\ub2e4.", "answer": "\uc21c\uc7a5", "sentence": "\ud64d\ud0c0\uc774\uc9c0\ub97c \ube44\ub86f\ud55c \ud328\ub975(\u8c9d\u52d2)\ub4e4\uc740 \ub300\ubcf5\uc9c4 \uc6b8\ub77c\ub098\ub77c\uc528\uc5d0\uac8c \uc21c\uc7a5 \uc744 \ud558\uc5ec \ub204\ub974\ud558\uce58\uc758 \ub4a4\ub97c \ub530\ub97c \uac83\uc744 \uac15\uc694\ud558\uc600\uace0 \uacb0\uad6d \uc6b8\ub77c\ub098\ub77c\uc528\ub294 \uc21c\uc7a5\ub418\uc5c8\ub2e4.", "paragraph_sentence": "1612\ub144 11\uc6d4 17\uc77c \ub2f9\uc2dc \ub9cc\uc8fc \uc77c\ub300\uc5d0\uc11c \uc138\ub825\uc744 \ub5a8\uce58\ub358 \uac74\uc8fc\uc5ec\uc9c4\uc758 \ucd94\uc7a5 \ub204\ub974\ud558\uce58\uc758 \uc5f4\ub124 \ubc88\uc9f8 \uc544\ub4e4\ub85c \ud0dc\uc5b4\ub0ac\ub2e4. \uadf8\uc758 \uc774\ub984 \ub3c4\ub974\uace4\uc740 \ub9cc\uc8fc\uc5b4\ub85c \uc624\uc18c\ub9ac\ub97c \ub73b\ud55c\ub2e4. \ub3c4\ub974\uace4\uc758 \uc5b4\uba38\ub2c8\ub294 \ub204\ub974\ud558\uce58\uc758 \ub124 \ubc88\uc9f8 \uc815\uc2e4, \uc989 \ub300\ubcf5\uc9c4(\u5927\u798f\u6649)\uc73c\ub85c \ubd09\ud574\uc838\uc788\ub358 \uc6b8\ub77c\ub098\ub77c\uc528\uc600\ub2e4. \uadf8\ub140\ub294 \uc774\ubbf8 \uc5b4\ub9b0 \ub098\uc774\uc5d0 \ub204\ub974\ud558\uce58\uc5d0\uac8c \uc2dc\uc9d1\uc628 \uc774\ud6c4 \uc904\uace7 \ub204\ub974\ud558\uce58\uc758 \ud070 \ucd1d\uc560\ub97c \ubc1b\uc558\uace0 \uadf8\ub140\uc758 \uc790\uc2dd\uc778 \ub3c4\ub974\uace4 \ud615\uc81c \uc5ed\uc2dc \ub369\ub2ec\uc544 \uc544\ubc84\uc9c0\uc758 \uc0ac\ub791\uc744 \ubc1b\uc558\ub2e4. \ub3c4\ub974\uace4\uc774 5\uc0b4 \ub54c\uc778 1616\ub144 1\uc6d4, \ub204\ub974\ud558\uce58\ub294 \uc2a4\uc2a4\ub85c \ud55c\uc774\ub77c \uce6d\ud558\uba70 \ub098\ub77c \uc774\ub984\uc744 \uae08(\u91d1)\uc774\ub77c \uc815\ud558\ub2c8 \uc774\uac83\uc774 \ubc14\ub85c \uccad\ub098\ub77c\uc758 \uc804\uc2e0\uc778 \ud6c4\uae08\uc774\ub2e4. 1626\ub144(\ucc9c\uba85 11\ub144) \ub204\ub974\ud558\uce58\ub294 \uc601\uc6d0\uc131 \uacf5\uaca9 \ub3c4\uc911 \uba85\ub098\ub77c\uc758 \uc7a5\uc218 \uc6d0\uc22d\ud658\uc758 \ud64d\uc758\ud3ec\ub97c \ub9de\uace0 \uc911\uc0c1\uc744 \uc785\uace0 \uc5bc\ub9c8 \ub4a4\uc5d0 \uc138\uc0c1\uc744 \ub5a0\ub0ac\ub2e4. \uc774\uc5d0 \ub204\ub974\ud558\uce58\uc758 \uc544\ub4e4\ub4e4\uacfc \uc870\uce74\ub4e4\uc740 \ucc28\uae30 \ud55c\uc73c\ub85c \ub204\ub974\ud558\uce58\uc758 8\ub0a8\uc774\uc790 \ub3c4\ub974\uace4\ubcf4\ub2e4 \uc2a4\ubb34 \uc0b4\uc774 \ub9ce\ub358 \uc774\ubcf5\ud615 \ud64d\ud0c0\uc774\uc9c0\ub97c \uc639\ub9bd\ud558\uc600\ub2e4. \ud64d\ud0c0\uc774\uc9c0\ub97c \ube44\ub86f\ud55c \ud328\ub975(\u8c9d\u52d2)\ub4e4\uc740 \ub300\ubcf5\uc9c4 \uc6b8\ub77c\ub098\ub77c\uc528\uc5d0\uac8c \uc21c\uc7a5 \uc744 \ud558\uc5ec \ub204\ub974\ud558\uce58\uc758 \ub4a4\ub97c \ub530\ub97c \uac83\uc744 \uac15\uc694\ud558\uc600\uace0 \uacb0\uad6d \uc6b8\ub77c\ub098\ub77c\uc528\ub294 \uc21c\uc7a5\ub418\uc5c8\ub2e4. \ub3c4\ub974\uace4\uacfc \uadf8\uc758 \ud615 \uc544\uc9c0\uac70, \uc5b4\ub9b0 \ub3d9\uc0dd \ub3c4\ub3c4\ub294 \ud55c \uc21c\uac04\uc5d0 \uc544\ubc84\uc9c0\uc5d0 \uc774\uc5b4 \uc5b4\uba38\ub2c8\ub97c \uc783\uc5c8\uc73c\ub098 \ud64d\ud0c0\uc774\uc9c0\ub294 \ub3c4\ub974\uace4 \ud615\uc81c\ub97c \ubaa8\ub450 \ud328\ub975\uc5d0 \ubd09\ud558\uc5ec \uadf8\ub4e4\uc744 \ub2ec\ub7ac\uace0 \uc870\uc815\uc5d0 \ucd9c\uc0ac\ucf00 \ud558\uc600\ub2e4.", "paragraph_answer": "1612\ub144 11\uc6d4 17\uc77c \ub2f9\uc2dc \ub9cc\uc8fc \uc77c\ub300\uc5d0\uc11c \uc138\ub825\uc744 \ub5a8\uce58\ub358 \uac74\uc8fc\uc5ec\uc9c4\uc758 \ucd94\uc7a5 \ub204\ub974\ud558\uce58\uc758 \uc5f4\ub124 \ubc88\uc9f8 \uc544\ub4e4\ub85c \ud0dc\uc5b4\ub0ac\ub2e4. \uadf8\uc758 \uc774\ub984 \ub3c4\ub974\uace4\uc740 \ub9cc\uc8fc\uc5b4\ub85c \uc624\uc18c\ub9ac\ub97c \ub73b\ud55c\ub2e4. \ub3c4\ub974\uace4\uc758 \uc5b4\uba38\ub2c8\ub294 \ub204\ub974\ud558\uce58\uc758 \ub124 \ubc88\uc9f8 \uc815\uc2e4, \uc989 \ub300\ubcf5\uc9c4(\u5927\u798f\u6649)\uc73c\ub85c \ubd09\ud574\uc838\uc788\ub358 \uc6b8\ub77c\ub098\ub77c\uc528\uc600\ub2e4. \uadf8\ub140\ub294 \uc774\ubbf8 \uc5b4\ub9b0 \ub098\uc774\uc5d0 \ub204\ub974\ud558\uce58\uc5d0\uac8c \uc2dc\uc9d1\uc628 \uc774\ud6c4 \uc904\uace7 \ub204\ub974\ud558\uce58\uc758 \ud070 \ucd1d\uc560\ub97c \ubc1b\uc558\uace0 \uadf8\ub140\uc758 \uc790\uc2dd\uc778 \ub3c4\ub974\uace4 \ud615\uc81c \uc5ed\uc2dc \ub369\ub2ec\uc544 \uc544\ubc84\uc9c0\uc758 \uc0ac\ub791\uc744 \ubc1b\uc558\ub2e4. \ub3c4\ub974\uace4\uc774 5\uc0b4 \ub54c\uc778 1616\ub144 1\uc6d4, \ub204\ub974\ud558\uce58\ub294 \uc2a4\uc2a4\ub85c \ud55c\uc774\ub77c \uce6d\ud558\uba70 \ub098\ub77c \uc774\ub984\uc744 \uae08(\u91d1)\uc774\ub77c \uc815\ud558\ub2c8 \uc774\uac83\uc774 \ubc14\ub85c \uccad\ub098\ub77c\uc758 \uc804\uc2e0\uc778 \ud6c4\uae08\uc774\ub2e4. 1626\ub144(\ucc9c\uba85 11\ub144) \ub204\ub974\ud558\uce58\ub294 \uc601\uc6d0\uc131 \uacf5\uaca9 \ub3c4\uc911 \uba85\ub098\ub77c\uc758 \uc7a5\uc218 \uc6d0\uc22d\ud658\uc758 \ud64d\uc758\ud3ec\ub97c \ub9de\uace0 \uc911\uc0c1\uc744 \uc785\uace0 \uc5bc\ub9c8 \ub4a4\uc5d0 \uc138\uc0c1\uc744 \ub5a0\ub0ac\ub2e4. \uc774\uc5d0 \ub204\ub974\ud558\uce58\uc758 \uc544\ub4e4\ub4e4\uacfc \uc870\uce74\ub4e4\uc740 \ucc28\uae30 \ud55c\uc73c\ub85c \ub204\ub974\ud558\uce58\uc758 8\ub0a8\uc774\uc790 \ub3c4\ub974\uace4\ubcf4\ub2e4 \uc2a4\ubb34 \uc0b4\uc774 \ub9ce\ub358 \uc774\ubcf5\ud615 \ud64d\ud0c0\uc774\uc9c0\ub97c \uc639\ub9bd\ud558\uc600\ub2e4. \ud64d\ud0c0\uc774\uc9c0\ub97c \ube44\ub86f\ud55c \ud328\ub975(\u8c9d\u52d2)\ub4e4\uc740 \ub300\ubcf5\uc9c4 \uc6b8\ub77c\ub098\ub77c\uc528\uc5d0\uac8c \uc21c\uc7a5 \uc744 \ud558\uc5ec \ub204\ub974\ud558\uce58\uc758 \ub4a4\ub97c \ub530\ub97c \uac83\uc744 \uac15\uc694\ud558\uc600\uace0 \uacb0\uad6d \uc6b8\ub77c\ub098\ub77c\uc528\ub294 \uc21c\uc7a5\ub418\uc5c8\ub2e4. \ub3c4\ub974\uace4\uacfc \uadf8\uc758 \ud615 \uc544\uc9c0\uac70, \uc5b4\ub9b0 \ub3d9\uc0dd \ub3c4\ub3c4\ub294 \ud55c \uc21c\uac04\uc5d0 \uc544\ubc84\uc9c0\uc5d0 \uc774\uc5b4 \uc5b4\uba38\ub2c8\ub97c \uc783\uc5c8\uc73c\ub098 \ud64d\ud0c0\uc774\uc9c0\ub294 \ub3c4\ub974\uace4 \ud615\uc81c\ub97c \ubaa8\ub450 \ud328\ub975\uc5d0 \ubd09\ud558\uc5ec \uadf8\ub4e4\uc744 \ub2ec\ub7ac\uace0 \uc870\uc815\uc5d0 \ucd9c\uc0ac\ucf00 \ud558\uc600\ub2e4.", "sentence_answer": "\ud64d\ud0c0\uc774\uc9c0\ub97c \ube44\ub86f\ud55c \ud328\ub975(\u8c9d\u52d2)\ub4e4\uc740 \ub300\ubcf5\uc9c4 \uc6b8\ub77c\ub098\ub77c\uc528\uc5d0\uac8c \uc21c\uc7a5 \uc744 \ud558\uc5ec \ub204\ub974\ud558\uce58\uc758 \ub4a4\ub97c \ub530\ub97c \uac83\uc744 \uac15\uc694\ud558\uc600\uace0 \uacb0\uad6d \uc6b8\ub77c\ub098\ub77c\uc528\ub294 \uc21c\uc7a5\ub418\uc5c8\ub2e4.", "paragraph_id": "6512901-0-1", "paragraph_question": "question: \uacfc\uac70 \ub192\uc740\uc9c0\uc704\uc758 \uc0ac\ub78c\uc774 \uc8fd\uc73c\uba74 \uadf8 \uc2e0\ud558\ub4e4\uacfc \ubb3c\uac74\ub4e4\uc744 \uac19\uc774 \ubb3b\ub294 \ud48d\uc2b5\uc744 \ubb34\uc5c7\uc774\ub77c\uace0 \ud558\ub294\uac00?, context: 1612\ub144 11\uc6d4 17\uc77c \ub2f9\uc2dc \ub9cc\uc8fc \uc77c\ub300\uc5d0\uc11c \uc138\ub825\uc744 \ub5a8\uce58\ub358 \uac74\uc8fc\uc5ec\uc9c4\uc758 \ucd94\uc7a5 \ub204\ub974\ud558\uce58\uc758 \uc5f4\ub124 \ubc88\uc9f8 \uc544\ub4e4\ub85c \ud0dc\uc5b4\ub0ac\ub2e4. \uadf8\uc758 \uc774\ub984 \ub3c4\ub974\uace4\uc740 \ub9cc\uc8fc\uc5b4\ub85c \uc624\uc18c\ub9ac\ub97c \ub73b\ud55c\ub2e4. \ub3c4\ub974\uace4\uc758 \uc5b4\uba38\ub2c8\ub294 \ub204\ub974\ud558\uce58\uc758 \ub124 \ubc88\uc9f8 \uc815\uc2e4, \uc989 \ub300\ubcf5\uc9c4(\u5927\u798f\u6649)\uc73c\ub85c \ubd09\ud574\uc838\uc788\ub358 \uc6b8\ub77c\ub098\ub77c\uc528\uc600\ub2e4. \uadf8\ub140\ub294 \uc774\ubbf8 \uc5b4\ub9b0 \ub098\uc774\uc5d0 \ub204\ub974\ud558\uce58\uc5d0\uac8c \uc2dc\uc9d1\uc628 \uc774\ud6c4 \uc904\uace7 \ub204\ub974\ud558\uce58\uc758 \ud070 \ucd1d\uc560\ub97c \ubc1b\uc558\uace0 \uadf8\ub140\uc758 \uc790\uc2dd\uc778 \ub3c4\ub974\uace4 \ud615\uc81c \uc5ed\uc2dc \ub369\ub2ec\uc544 \uc544\ubc84\uc9c0\uc758 \uc0ac\ub791\uc744 \ubc1b\uc558\ub2e4. \ub3c4\ub974\uace4\uc774 5\uc0b4 \ub54c\uc778 1616\ub144 1\uc6d4, \ub204\ub974\ud558\uce58\ub294 \uc2a4\uc2a4\ub85c \ud55c\uc774\ub77c \uce6d\ud558\uba70 \ub098\ub77c \uc774\ub984\uc744 \uae08(\u91d1)\uc774\ub77c \uc815\ud558\ub2c8 \uc774\uac83\uc774 \ubc14\ub85c \uccad\ub098\ub77c\uc758 \uc804\uc2e0\uc778 \ud6c4\uae08\uc774\ub2e4. 1626\ub144(\ucc9c\uba85 11\ub144) \ub204\ub974\ud558\uce58\ub294 \uc601\uc6d0\uc131 \uacf5\uaca9 \ub3c4\uc911 \uba85\ub098\ub77c\uc758 \uc7a5\uc218 \uc6d0\uc22d\ud658\uc758 \ud64d\uc758\ud3ec\ub97c \ub9de\uace0 \uc911\uc0c1\uc744 \uc785\uace0 \uc5bc\ub9c8 \ub4a4\uc5d0 \uc138\uc0c1\uc744 \ub5a0\ub0ac\ub2e4. \uc774\uc5d0 \ub204\ub974\ud558\uce58\uc758 \uc544\ub4e4\ub4e4\uacfc \uc870\uce74\ub4e4\uc740 \ucc28\uae30 \ud55c\uc73c\ub85c \ub204\ub974\ud558\uce58\uc758 8\ub0a8\uc774\uc790 \ub3c4\ub974\uace4\ubcf4\ub2e4 \uc2a4\ubb34 \uc0b4\uc774 \ub9ce\ub358 \uc774\ubcf5\ud615 \ud64d\ud0c0\uc774\uc9c0\ub97c \uc639\ub9bd\ud558\uc600\ub2e4. \ud64d\ud0c0\uc774\uc9c0\ub97c \ube44\ub86f\ud55c \ud328\ub975(\u8c9d\u52d2)\ub4e4\uc740 \ub300\ubcf5\uc9c4 \uc6b8\ub77c\ub098\ub77c\uc528\uc5d0\uac8c \uc21c\uc7a5\uc744 \ud558\uc5ec \ub204\ub974\ud558\uce58\uc758 \ub4a4\ub97c \ub530\ub97c \uac83\uc744 \uac15\uc694\ud558\uc600\uace0 \uacb0\uad6d \uc6b8\ub77c\ub098\ub77c\uc528\ub294 \uc21c\uc7a5\ub418\uc5c8\ub2e4. \ub3c4\ub974\uace4\uacfc \uadf8\uc758 \ud615 \uc544\uc9c0\uac70, \uc5b4\ub9b0 \ub3d9\uc0dd \ub3c4\ub3c4\ub294 \ud55c \uc21c\uac04\uc5d0 \uc544\ubc84\uc9c0\uc5d0 \uc774\uc5b4 \uc5b4\uba38\ub2c8\ub97c \uc783\uc5c8\uc73c\ub098 \ud64d\ud0c0\uc774\uc9c0\ub294 \ub3c4\ub974\uace4 \ud615\uc81c\ub97c \ubaa8\ub450 \ud328\ub975\uc5d0 \ubd09\ud558\uc5ec \uadf8\ub4e4\uc744 \ub2ec\ub7ac\uace0 \uc870\uc815\uc5d0 \ucd9c\uc0ac\ucf00 \ud558\uc600\ub2e4."} {"question": "\ubb34\uc2a8 \uc0ac\uac74\uc740 \ub3c5\uc77c \uc790\uc720\uc8fc\uc758\uc758 \ud328\ubc30\ub97c \uc758\ubbf8\ud588\ub294\uac00?", "paragraph": "'\uc88c\uc808\ub41c \ud601\uba85'(gescheiterte Revolution)\uc740 \ub3c5\uc77c \uc790\uc720\uc8fc\uc758\uc758 \ud328\ubc30\ub97c \uc758\ubbf8\ud558\ub294 \uc0ac\uac74\uc774\uc5c8\ub2e4. \ubbfc\uc8fc\uc8fc\uc758\uc790\ub4e4\uacfc \uc790\uc720\uc8fc\uc758\uc790\ub4e4\uc774 \uc815\uce58\uc801 \uc2e0\ub9dd\uc744 \uc783\uace0 \uc18c\uc678\ub418\uba74\uc11c, \uc2e4\ud604\ub418\uc9c0 \ubabb\ud55c \uad6d\ubbfc\uad6d\uac00\ub97c \ud5a5\ud55c \uadf8\ub9ac\uc6c0\ub9cc\uc774 \ub0a8\uc558\ub2e4. \uc785\ud5cc\uad6d\uac00\uc640 \uad6d\ubbfc\uad6d\uac00 \ud615\uc131\uc744 \ub3d9\uc2dc\uc5d0 \ucd94\uad6c\ud558\uace0\uc790 \ud588\ub358 1848/49\ub144 \ud601\uba85\uc758 \uc2e4\ud328\ub294 \uc790\uc720\uc640 \ud1b5\uc77c\uc744 \ubd84\ub9ac\ud558\ub294 \uacb0\uacfc\ub85c \uc774\uc5b4\uc84c\uc73c\uba70, \uc774\ub85c \uc778\ud55c \uc790\uc720\uc640 \ud1b5\uc77c \uc0ac\uc774\uc758 \uc591\uc790 \uc120\ud0dd \uc0c1\ud669\uc740 20\uc138\uae30 \ub3c5\uc77c\uc0ac\uc5d0\uc11c \uc124\uba85\uc758 \uacac\ubcf8\uc744 \ucc3e\ub294 \uba87\uba87 \uc5ed\uc0ac\uac00\ub4e4\uc5d0 \uc758\ud574 \uc874\ub354\ubca0\ud06c(Deutscher Sonderweg; \ub3c5\uc77c\uc758 \ud2b9\uc218\ud55c \uae38)\uc758 \uc99d\uac70\ub85c \ud574\uc11d\ub418\uc5c8\ub2e4. \ud504\ub791\ud06c\ud478\ub974\ud2b8 \uad6d\ubbfc\uc758\ud68c\uc758 \uc88c\uc808\ub85c \ud1b5\uc77c\uc740 \uc790\uc720\uc8fc\uc758\uc801 \uc774\uc0c1 \ub300\uc2e0 \uad70\uc0ac\ub825\uacfc \uac19\uc740 \ud604\uc2e4\uc815\uce58(Realpolitik)\uc801 \uc218\ub2e8\uc744 \uc911\uc2ec\uc73c\ub85c \uc774\ub8e8\uc5b4\uc9c0\uac8c \ub418\uba70, \ub3c5\uc77c \uc81c\uad6d\uc740 \ubcf4\uc218\uc801\uc774\uace0 \uad8c\uc704\uc8fc\uc758\uc801 \uc0c9\ucc44\ub97c \ub760\uac8c \ub41c\ub2e4.", "answer": "\uc88c\uc808\ub41c \ud601\uba85", "sentence": "' \uc88c\uc808\ub41c \ud601\uba85 '(gescheiterte Revolution)\uc740 \ub3c5\uc77c \uc790\uc720\uc8fc\uc758\uc758 \ud328\ubc30\ub97c \uc758\ubbf8\ud558\ub294 \uc0ac\uac74\uc774\uc5c8\ub2e4.", "paragraph_sentence": " ' \uc88c\uc808\ub41c \ud601\uba85 '(gescheiterte Revolution)\uc740 \ub3c5\uc77c \uc790\uc720\uc8fc\uc758\uc758 \ud328\ubc30\ub97c \uc758\ubbf8\ud558\ub294 \uc0ac\uac74\uc774\uc5c8\ub2e4. \ubbfc\uc8fc\uc8fc\uc758\uc790\ub4e4\uacfc \uc790\uc720\uc8fc\uc758\uc790\ub4e4\uc774 \uc815\uce58\uc801 \uc2e0\ub9dd\uc744 \uc783\uace0 \uc18c\uc678\ub418\uba74\uc11c, \uc2e4\ud604\ub418\uc9c0 \ubabb\ud55c \uad6d\ubbfc\uad6d\uac00\ub97c \ud5a5\ud55c \uadf8\ub9ac\uc6c0\ub9cc\uc774 \ub0a8\uc558\ub2e4. \uc785\ud5cc\uad6d\uac00\uc640 \uad6d\ubbfc\uad6d\uac00 \ud615\uc131\uc744 \ub3d9\uc2dc\uc5d0 \ucd94\uad6c\ud558\uace0\uc790 \ud588\ub358 1848/49\ub144 \ud601\uba85\uc758 \uc2e4\ud328\ub294 \uc790\uc720\uc640 \ud1b5\uc77c\uc744 \ubd84\ub9ac\ud558\ub294 \uacb0\uacfc\ub85c \uc774\uc5b4\uc84c\uc73c\uba70, \uc774\ub85c \uc778\ud55c \uc790\uc720\uc640 \ud1b5\uc77c \uc0ac\uc774\uc758 \uc591\uc790 \uc120\ud0dd \uc0c1\ud669\uc740 20\uc138\uae30 \ub3c5\uc77c\uc0ac\uc5d0\uc11c \uc124\uba85\uc758 \uacac\ubcf8\uc744 \ucc3e\ub294 \uba87\uba87 \uc5ed\uc0ac\uac00\ub4e4\uc5d0 \uc758\ud574 \uc874\ub354\ubca0\ud06c(Deutscher Sonderweg; \ub3c5\uc77c\uc758 \ud2b9\uc218\ud55c \uae38)\uc758 \uc99d\uac70\ub85c \ud574\uc11d\ub418\uc5c8\ub2e4. \ud504\ub791\ud06c\ud478\ub974\ud2b8 \uad6d\ubbfc\uc758\ud68c\uc758 \uc88c\uc808\ub85c \ud1b5\uc77c\uc740 \uc790\uc720\uc8fc\uc758\uc801 \uc774\uc0c1 \ub300\uc2e0 \uad70\uc0ac\ub825\uacfc \uac19\uc740 \ud604\uc2e4\uc815\uce58(Realpolitik)\uc801 \uc218\ub2e8\uc744 \uc911\uc2ec\uc73c\ub85c \uc774\ub8e8\uc5b4\uc9c0\uac8c \ub418\uba70, \ub3c5\uc77c \uc81c\uad6d\uc740 \ubcf4\uc218\uc801\uc774\uace0 \uad8c\uc704\uc8fc\uc758\uc801 \uc0c9\ucc44\ub97c \ub760\uac8c \ub41c\ub2e4.", "paragraph_answer": "' \uc88c\uc808\ub41c \ud601\uba85 '(gescheiterte Revolution)\uc740 \ub3c5\uc77c \uc790\uc720\uc8fc\uc758\uc758 \ud328\ubc30\ub97c \uc758\ubbf8\ud558\ub294 \uc0ac\uac74\uc774\uc5c8\ub2e4. \ubbfc\uc8fc\uc8fc\uc758\uc790\ub4e4\uacfc \uc790\uc720\uc8fc\uc758\uc790\ub4e4\uc774 \uc815\uce58\uc801 \uc2e0\ub9dd\uc744 \uc783\uace0 \uc18c\uc678\ub418\uba74\uc11c, \uc2e4\ud604\ub418\uc9c0 \ubabb\ud55c \uad6d\ubbfc\uad6d\uac00\ub97c \ud5a5\ud55c \uadf8\ub9ac\uc6c0\ub9cc\uc774 \ub0a8\uc558\ub2e4. \uc785\ud5cc\uad6d\uac00\uc640 \uad6d\ubbfc\uad6d\uac00 \ud615\uc131\uc744 \ub3d9\uc2dc\uc5d0 \ucd94\uad6c\ud558\uace0\uc790 \ud588\ub358 1848/49\ub144 \ud601\uba85\uc758 \uc2e4\ud328\ub294 \uc790\uc720\uc640 \ud1b5\uc77c\uc744 \ubd84\ub9ac\ud558\ub294 \uacb0\uacfc\ub85c \uc774\uc5b4\uc84c\uc73c\uba70, \uc774\ub85c \uc778\ud55c \uc790\uc720\uc640 \ud1b5\uc77c \uc0ac\uc774\uc758 \uc591\uc790 \uc120\ud0dd \uc0c1\ud669\uc740 20\uc138\uae30 \ub3c5\uc77c\uc0ac\uc5d0\uc11c \uc124\uba85\uc758 \uacac\ubcf8\uc744 \ucc3e\ub294 \uba87\uba87 \uc5ed\uc0ac\uac00\ub4e4\uc5d0 \uc758\ud574 \uc874\ub354\ubca0\ud06c(Deutscher Sonderweg; \ub3c5\uc77c\uc758 \ud2b9\uc218\ud55c \uae38)\uc758 \uc99d\uac70\ub85c \ud574\uc11d\ub418\uc5c8\ub2e4. \ud504\ub791\ud06c\ud478\ub974\ud2b8 \uad6d\ubbfc\uc758\ud68c\uc758 \uc88c\uc808\ub85c \ud1b5\uc77c\uc740 \uc790\uc720\uc8fc\uc758\uc801 \uc774\uc0c1 \ub300\uc2e0 \uad70\uc0ac\ub825\uacfc \uac19\uc740 \ud604\uc2e4\uc815\uce58(Realpolitik)\uc801 \uc218\ub2e8\uc744 \uc911\uc2ec\uc73c\ub85c \uc774\ub8e8\uc5b4\uc9c0\uac8c \ub418\uba70, \ub3c5\uc77c \uc81c\uad6d\uc740 \ubcf4\uc218\uc801\uc774\uace0 \uad8c\uc704\uc8fc\uc758\uc801 \uc0c9\ucc44\ub97c \ub760\uac8c \ub41c\ub2e4.", "sentence_answer": "' \uc88c\uc808\ub41c \ud601\uba85 '(gescheiterte Revolution)\uc740 \ub3c5\uc77c \uc790\uc720\uc8fc\uc758\uc758 \ud328\ubc30\ub97c \uc758\ubbf8\ud558\ub294 \uc0ac\uac74\uc774\uc5c8\ub2e4.", "paragraph_id": "6568666-31-0", "paragraph_question": "question: \ubb34\uc2a8 \uc0ac\uac74\uc740 \ub3c5\uc77c \uc790\uc720\uc8fc\uc758\uc758 \ud328\ubc30\ub97c \uc758\ubbf8\ud588\ub294\uac00?, context: '\uc88c\uc808\ub41c \ud601\uba85'(gescheiterte Revolution)\uc740 \ub3c5\uc77c \uc790\uc720\uc8fc\uc758\uc758 \ud328\ubc30\ub97c \uc758\ubbf8\ud558\ub294 \uc0ac\uac74\uc774\uc5c8\ub2e4. \ubbfc\uc8fc\uc8fc\uc758\uc790\ub4e4\uacfc \uc790\uc720\uc8fc\uc758\uc790\ub4e4\uc774 \uc815\uce58\uc801 \uc2e0\ub9dd\uc744 \uc783\uace0 \uc18c\uc678\ub418\uba74\uc11c, \uc2e4\ud604\ub418\uc9c0 \ubabb\ud55c \uad6d\ubbfc\uad6d\uac00\ub97c \ud5a5\ud55c \uadf8\ub9ac\uc6c0\ub9cc\uc774 \ub0a8\uc558\ub2e4. \uc785\ud5cc\uad6d\uac00\uc640 \uad6d\ubbfc\uad6d\uac00 \ud615\uc131\uc744 \ub3d9\uc2dc\uc5d0 \ucd94\uad6c\ud558\uace0\uc790 \ud588\ub358 1848/49\ub144 \ud601\uba85\uc758 \uc2e4\ud328\ub294 \uc790\uc720\uc640 \ud1b5\uc77c\uc744 \ubd84\ub9ac\ud558\ub294 \uacb0\uacfc\ub85c \uc774\uc5b4\uc84c\uc73c\uba70, \uc774\ub85c \uc778\ud55c \uc790\uc720\uc640 \ud1b5\uc77c \uc0ac\uc774\uc758 \uc591\uc790 \uc120\ud0dd \uc0c1\ud669\uc740 20\uc138\uae30 \ub3c5\uc77c\uc0ac\uc5d0\uc11c \uc124\uba85\uc758 \uacac\ubcf8\uc744 \ucc3e\ub294 \uba87\uba87 \uc5ed\uc0ac\uac00\ub4e4\uc5d0 \uc758\ud574 \uc874\ub354\ubca0\ud06c(Deutscher Sonderweg; \ub3c5\uc77c\uc758 \ud2b9\uc218\ud55c \uae38)\uc758 \uc99d\uac70\ub85c \ud574\uc11d\ub418\uc5c8\ub2e4. \ud504\ub791\ud06c\ud478\ub974\ud2b8 \uad6d\ubbfc\uc758\ud68c\uc758 \uc88c\uc808\ub85c \ud1b5\uc77c\uc740 \uc790\uc720\uc8fc\uc758\uc801 \uc774\uc0c1 \ub300\uc2e0 \uad70\uc0ac\ub825\uacfc \uac19\uc740 \ud604\uc2e4\uc815\uce58(Realpolitik)\uc801 \uc218\ub2e8\uc744 \uc911\uc2ec\uc73c\ub85c \uc774\ub8e8\uc5b4\uc9c0\uac8c \ub418\uba70, \ub3c5\uc77c \uc81c\uad6d\uc740 \ubcf4\uc218\uc801\uc774\uace0 \uad8c\uc704\uc8fc\uc758\uc801 \uc0c9\ucc44\ub97c \ub760\uac8c \ub41c\ub2e4."} {"question": "\ud0a4\uc608\ud504 \uc804\ud22c \ub2f9\uc2dc \uc721\uad70\ucc38\ubaa8\ucd1d\uc7a5\uc740?", "paragraph": "8\uc6d4 3\uc77c, \ud788\ud2c0\ub7ec\ub294 \uc77c\uc2dc\uc801\uc73c\ub85c \ub0a8\ucabd\uc758 \uc6b0\ud06c\ub77c\uc774\ub098 \ud0a4\uc608\ud504 \uacf5\uaca9\uc744 \uc704\ud574 \ubaa8\uc2a4\ud06c\ubc14 \uc804\uc9c4\uc744 \uc911\uc9c0\ud588\ub2e4. \uadf8\ub7ec\ub098, 8\uc6d4 12\uc77c 34 \ucd1d\ud1b5 \uc9c0\uc2dc\ub839\uc5d0\uc11c \ud504\ub780\uce20 \ud560\ub354, \ud398\ub3c4\ub974 \ud3f0 \ubcf4\ud06c, \ud558\uc778\uce20 \uad6c\ub370\ub9ac\uc548 \ub4f1 \ubaa8\uc2a4\ud06c\ubc14\ub85c \ubc14\ub85c \uc9c4\uaca9\ud558\uc790\ub294 \uc8fc\uc7a5\uacfc \ubaa8\uc2a4\ud06c\ubc14 \uc9c4\uaca9\uc744 \uc2dc\uc791\ud558\uae30 \uc804 \uc911\ubd80 \uc9d1\ub2e8\uad70\uc744 \ub0a8\ubd80 \uc9d1\ub2e8\uad70\uc758 \ud0a4\uc608\ud504 \ubc29\uba74\uc73c\ub85c \uc9c4\uaca9\uc2dc\ucf1c\uc57c \ud55c\ub2e4\ub294 \ud788\ud2c0\ub7ec\uc758 \uc758\uacac \uc0ac\uc774\uc5d0\uc11c \ud0c0\ud611\uc744 \uc774\ub8e8\uc5c8\ub2e4. \ubd81\ubd80 \uc9d1\ub2e8\uad70\uacfc \uc911\ubd80 \uc9d1\ub2e8\uad70\uc744 \uac01\uac01 \uc9c0\uc6d0\ud558\uae30 \uc704\ud574 \uc7ac\ubc30\uce58\ub418\uc5c8\ub358 \uc911\ubd80 \uc9d1\ub2e8\uad70\uc758 \uc81c2\uae30\uac11\uc9d1\ub2e8\uacfc \uc81c3\uae30\uac11\uc9d1\ub2e8\uc744 \uc911\ubd80 \uc9d1\ub2e8\uad70\uc73c\ub85c \ub3cc\ub9b0 \uc774\ud6c4 \ubd81\ubd80 \uc9d1\ub2e8\uad70\uc758 \uc81c4\uae30\uac11\uc9d1\ub2e8\uacfc \ud568\uaed8 \ubaa9\ud45c\ub85c \ub3cc\ub9ac\uace0\uc790 \ud588\ub2e4. \uadf8\ub9ac\uace0 \uc774 \uc81c3\uae30\uac11\uc9d1\ub2e8\uc740 \uc911\ubd80 \uc9d1\ub2e8\uad70 \uc9c0\ud718 \ud558\uc5d0 \ubaa8\uc2a4\ud06c\ubc14\ub85c \uc9c4\uaca9\ud588\ub2e4. \ucc98\uc74c\uc5d0 \uc721\uad70\ucc38\ubaa8\ucd1d\uc7a5 \ud504\ub780\uce20 \ud560\ub354\uc640 \uc911\ubd80 \uc9d1\ub2e8\uad70 \uc0ac\ub839\uad00 \ud3f0 \ubcf4\ud06c\ub294 \ud0c0\ud611\uc5d0 \ub9cc\uc871\ud588\uc9c0\ub9cc, \uace7 \ud604\uc2e4\uc801\uc778 \uc791\uc804 \uacc4\ud68d \ud604\uc2e4\uc774 \ub108\ubb34 \uc5b4\ub824\uc6cc \ub099\uad00\ub860\uc774 \uc218\uadf8\ub7ec\ub4e4\uac8c \ub418\uc5c8\ub2e4.", "answer": "\ud504\ub780\uce20 \ud560\ub354", "sentence": "\uadf8\ub7ec\ub098, 8\uc6d4 12\uc77c 34 \ucd1d\ud1b5 \uc9c0\uc2dc\ub839\uc5d0\uc11c \ud504\ub780\uce20 \ud560\ub354 ,", "paragraph_sentence": "8\uc6d4 3\uc77c, \ud788\ud2c0\ub7ec\ub294 \uc77c\uc2dc\uc801\uc73c\ub85c \ub0a8\ucabd\uc758 \uc6b0\ud06c\ub77c\uc774\ub098 \ud0a4\uc608\ud504 \uacf5\uaca9\uc744 \uc704\ud574 \ubaa8\uc2a4\ud06c\ubc14 \uc804\uc9c4\uc744 \uc911\uc9c0\ud588\ub2e4. \uadf8\ub7ec\ub098, 8\uc6d4 12\uc77c 34 \ucd1d\ud1b5 \uc9c0\uc2dc\ub839\uc5d0\uc11c \ud504\ub780\uce20 \ud560\ub354 , \ud398\ub3c4\ub974 \ud3f0 \ubcf4\ud06c, \ud558\uc778\uce20 \uad6c\ub370\ub9ac\uc548 \ub4f1 \ubaa8\uc2a4\ud06c\ubc14\ub85c \ubc14\ub85c \uc9c4\uaca9\ud558\uc790\ub294 \uc8fc\uc7a5\uacfc \ubaa8\uc2a4\ud06c\ubc14 \uc9c4\uaca9\uc744 \uc2dc\uc791\ud558\uae30 \uc804 \uc911\ubd80 \uc9d1\ub2e8\uad70\uc744 \ub0a8\ubd80 \uc9d1\ub2e8\uad70\uc758 \ud0a4\uc608\ud504 \ubc29\uba74\uc73c\ub85c \uc9c4\uaca9\uc2dc\ucf1c\uc57c \ud55c\ub2e4\ub294 \ud788\ud2c0\ub7ec\uc758 \uc758\uacac \uc0ac\uc774\uc5d0\uc11c \ud0c0\ud611\uc744 \uc774\ub8e8\uc5c8\ub2e4. \ubd81\ubd80 \uc9d1\ub2e8\uad70\uacfc \uc911\ubd80 \uc9d1\ub2e8\uad70\uc744 \uac01\uac01 \uc9c0\uc6d0\ud558\uae30 \uc704\ud574 \uc7ac\ubc30\uce58\ub418\uc5c8\ub358 \uc911\ubd80 \uc9d1\ub2e8\uad70\uc758 \uc81c2\uae30\uac11\uc9d1\ub2e8\uacfc \uc81c3\uae30\uac11\uc9d1\ub2e8\uc744 \uc911\ubd80 \uc9d1\ub2e8\uad70\uc73c\ub85c \ub3cc\ub9b0 \uc774\ud6c4 \ubd81\ubd80 \uc9d1\ub2e8\uad70\uc758 \uc81c4\uae30\uac11\uc9d1\ub2e8\uacfc \ud568\uaed8 \ubaa9\ud45c\ub85c \ub3cc\ub9ac\uace0\uc790 \ud588\ub2e4. \uadf8\ub9ac\uace0 \uc774 \uc81c3\uae30\uac11\uc9d1\ub2e8\uc740 \uc911\ubd80 \uc9d1\ub2e8\uad70 \uc9c0\ud718 \ud558\uc5d0 \ubaa8\uc2a4\ud06c\ubc14\ub85c \uc9c4\uaca9\ud588\ub2e4. \ucc98\uc74c\uc5d0 \uc721\uad70\ucc38\ubaa8\ucd1d\uc7a5 \ud504\ub780\uce20 \ud560\ub354\uc640 \uc911\ubd80 \uc9d1\ub2e8\uad70 \uc0ac\ub839\uad00 \ud3f0 \ubcf4\ud06c\ub294 \ud0c0\ud611\uc5d0 \ub9cc\uc871\ud588\uc9c0\ub9cc, \uace7 \ud604\uc2e4\uc801\uc778 \uc791\uc804 \uacc4\ud68d \ud604\uc2e4\uc774 \ub108\ubb34 \uc5b4\ub824\uc6cc \ub099\uad00\ub860\uc774 \uc218\uadf8\ub7ec\ub4e4\uac8c \ub418\uc5c8\ub2e4.", "paragraph_answer": "8\uc6d4 3\uc77c, \ud788\ud2c0\ub7ec\ub294 \uc77c\uc2dc\uc801\uc73c\ub85c \ub0a8\ucabd\uc758 \uc6b0\ud06c\ub77c\uc774\ub098 \ud0a4\uc608\ud504 \uacf5\uaca9\uc744 \uc704\ud574 \ubaa8\uc2a4\ud06c\ubc14 \uc804\uc9c4\uc744 \uc911\uc9c0\ud588\ub2e4. \uadf8\ub7ec\ub098, 8\uc6d4 12\uc77c 34 \ucd1d\ud1b5 \uc9c0\uc2dc\ub839\uc5d0\uc11c \ud504\ub780\uce20 \ud560\ub354 , \ud398\ub3c4\ub974 \ud3f0 \ubcf4\ud06c, \ud558\uc778\uce20 \uad6c\ub370\ub9ac\uc548 \ub4f1 \ubaa8\uc2a4\ud06c\ubc14\ub85c \ubc14\ub85c \uc9c4\uaca9\ud558\uc790\ub294 \uc8fc\uc7a5\uacfc \ubaa8\uc2a4\ud06c\ubc14 \uc9c4\uaca9\uc744 \uc2dc\uc791\ud558\uae30 \uc804 \uc911\ubd80 \uc9d1\ub2e8\uad70\uc744 \ub0a8\ubd80 \uc9d1\ub2e8\uad70\uc758 \ud0a4\uc608\ud504 \ubc29\uba74\uc73c\ub85c \uc9c4\uaca9\uc2dc\ucf1c\uc57c \ud55c\ub2e4\ub294 \ud788\ud2c0\ub7ec\uc758 \uc758\uacac \uc0ac\uc774\uc5d0\uc11c \ud0c0\ud611\uc744 \uc774\ub8e8\uc5c8\ub2e4. \ubd81\ubd80 \uc9d1\ub2e8\uad70\uacfc \uc911\ubd80 \uc9d1\ub2e8\uad70\uc744 \uac01\uac01 \uc9c0\uc6d0\ud558\uae30 \uc704\ud574 \uc7ac\ubc30\uce58\ub418\uc5c8\ub358 \uc911\ubd80 \uc9d1\ub2e8\uad70\uc758 \uc81c2\uae30\uac11\uc9d1\ub2e8\uacfc \uc81c3\uae30\uac11\uc9d1\ub2e8\uc744 \uc911\ubd80 \uc9d1\ub2e8\uad70\uc73c\ub85c \ub3cc\ub9b0 \uc774\ud6c4 \ubd81\ubd80 \uc9d1\ub2e8\uad70\uc758 \uc81c4\uae30\uac11\uc9d1\ub2e8\uacfc \ud568\uaed8 \ubaa9\ud45c\ub85c \ub3cc\ub9ac\uace0\uc790 \ud588\ub2e4. \uadf8\ub9ac\uace0 \uc774 \uc81c3\uae30\uac11\uc9d1\ub2e8\uc740 \uc911\ubd80 \uc9d1\ub2e8\uad70 \uc9c0\ud718 \ud558\uc5d0 \ubaa8\uc2a4\ud06c\ubc14\ub85c \uc9c4\uaca9\ud588\ub2e4. \ucc98\uc74c\uc5d0 \uc721\uad70\ucc38\ubaa8\ucd1d\uc7a5 \ud504\ub780\uce20 \ud560\ub354\uc640 \uc911\ubd80 \uc9d1\ub2e8\uad70 \uc0ac\ub839\uad00 \ud3f0 \ubcf4\ud06c\ub294 \ud0c0\ud611\uc5d0 \ub9cc\uc871\ud588\uc9c0\ub9cc, \uace7 \ud604\uc2e4\uc801\uc778 \uc791\uc804 \uacc4\ud68d \ud604\uc2e4\uc774 \ub108\ubb34 \uc5b4\ub824\uc6cc \ub099\uad00\ub860\uc774 \uc218\uadf8\ub7ec\ub4e4\uac8c \ub418\uc5c8\ub2e4.", "sentence_answer": "\uadf8\ub7ec\ub098, 8\uc6d4 12\uc77c 34 \ucd1d\ud1b5 \uc9c0\uc2dc\ub839\uc5d0\uc11c \ud504\ub780\uce20 \ud560\ub354 ,", "paragraph_id": "6564888-1-2", "paragraph_question": "question: \ud0a4\uc608\ud504 \uc804\ud22c \ub2f9\uc2dc \uc721\uad70\ucc38\ubaa8\ucd1d\uc7a5\uc740?, context: 8\uc6d4 3\uc77c, \ud788\ud2c0\ub7ec\ub294 \uc77c\uc2dc\uc801\uc73c\ub85c \ub0a8\ucabd\uc758 \uc6b0\ud06c\ub77c\uc774\ub098 \ud0a4\uc608\ud504 \uacf5\uaca9\uc744 \uc704\ud574 \ubaa8\uc2a4\ud06c\ubc14 \uc804\uc9c4\uc744 \uc911\uc9c0\ud588\ub2e4. \uadf8\ub7ec\ub098, 8\uc6d4 12\uc77c 34 \ucd1d\ud1b5 \uc9c0\uc2dc\ub839\uc5d0\uc11c \ud504\ub780\uce20 \ud560\ub354, \ud398\ub3c4\ub974 \ud3f0 \ubcf4\ud06c, \ud558\uc778\uce20 \uad6c\ub370\ub9ac\uc548 \ub4f1 \ubaa8\uc2a4\ud06c\ubc14\ub85c \ubc14\ub85c \uc9c4\uaca9\ud558\uc790\ub294 \uc8fc\uc7a5\uacfc \ubaa8\uc2a4\ud06c\ubc14 \uc9c4\uaca9\uc744 \uc2dc\uc791\ud558\uae30 \uc804 \uc911\ubd80 \uc9d1\ub2e8\uad70\uc744 \ub0a8\ubd80 \uc9d1\ub2e8\uad70\uc758 \ud0a4\uc608\ud504 \ubc29\uba74\uc73c\ub85c \uc9c4\uaca9\uc2dc\ucf1c\uc57c \ud55c\ub2e4\ub294 \ud788\ud2c0\ub7ec\uc758 \uc758\uacac \uc0ac\uc774\uc5d0\uc11c \ud0c0\ud611\uc744 \uc774\ub8e8\uc5c8\ub2e4. \ubd81\ubd80 \uc9d1\ub2e8\uad70\uacfc \uc911\ubd80 \uc9d1\ub2e8\uad70\uc744 \uac01\uac01 \uc9c0\uc6d0\ud558\uae30 \uc704\ud574 \uc7ac\ubc30\uce58\ub418\uc5c8\ub358 \uc911\ubd80 \uc9d1\ub2e8\uad70\uc758 \uc81c2\uae30\uac11\uc9d1\ub2e8\uacfc \uc81c3\uae30\uac11\uc9d1\ub2e8\uc744 \uc911\ubd80 \uc9d1\ub2e8\uad70\uc73c\ub85c \ub3cc\ub9b0 \uc774\ud6c4 \ubd81\ubd80 \uc9d1\ub2e8\uad70\uc758 \uc81c4\uae30\uac11\uc9d1\ub2e8\uacfc \ud568\uaed8 \ubaa9\ud45c\ub85c \ub3cc\ub9ac\uace0\uc790 \ud588\ub2e4. \uadf8\ub9ac\uace0 \uc774 \uc81c3\uae30\uac11\uc9d1\ub2e8\uc740 \uc911\ubd80 \uc9d1\ub2e8\uad70 \uc9c0\ud718 \ud558\uc5d0 \ubaa8\uc2a4\ud06c\ubc14\ub85c \uc9c4\uaca9\ud588\ub2e4. \ucc98\uc74c\uc5d0 \uc721\uad70\ucc38\ubaa8\ucd1d\uc7a5 \ud504\ub780\uce20 \ud560\ub354\uc640 \uc911\ubd80 \uc9d1\ub2e8\uad70 \uc0ac\ub839\uad00 \ud3f0 \ubcf4\ud06c\ub294 \ud0c0\ud611\uc5d0 \ub9cc\uc871\ud588\uc9c0\ub9cc, \uace7 \ud604\uc2e4\uc801\uc778 \uc791\uc804 \uacc4\ud68d \ud604\uc2e4\uc774 \ub108\ubb34 \uc5b4\ub824\uc6cc \ub099\uad00\ub860\uc774 \uc218\uadf8\ub7ec\ub4e4\uac8c \ub418\uc5c8\ub2e4."} {"question": "\ud638\ubd81\uacfc \uc591\uad11\uc5d0\uc11c \ubcf4\ub85c\uc6b4\ub3d9\uc774 \uc77c\uc5b4\ub0ac\ub294\ub370 \uadf8\uc911 \uac00\uc7a5 \uaca9\ub82c\ud558\uac8c \uc77c\uc5b4\ub09c\uacf3\uc740?", "paragraph": "\uccad\uc870\uc758 \uc774\ub7ec\ud55c \uc870\ucc98\ub294 \uacb0\uc815 \ub2f9\uc0ac\uc790\ub97c \uc81c\uc678\ud55c \ubaa8\ub4e0 \uc0ac\ub78c\uc744 \uaca9\ubd84\uc2dc\ucf30\ub2e4. \uc6b0\uc120 \ub0b4\uac01\uc774 \uc758\ud68c\uc5d0 \uc544\ubb34\ub7f0 \uc0c1\uc758 \uc5c6\uc774 \uc77c\ubc29\uc801\uc73c\ub85c \uacb0\uc815\ud558\uace0 \ud1b5\ubcf4\ud55c \uc0ac\ud56d\uc774\uc5c8\uace0 \uc815\ubd80\uac00 \uc57d\uc18d\ud55c \ubbfc\uc8fc\uc801\uc778 \uc808\ucc28\ub97c \uc644\uc804\ud788 \ubb34\uc2dc\ud588\uace0 \ub9cc\uc8fc\uc871\uc774 \ub3c5\uc810\ud55c \ub0b4\uac01\uc5d0\uc11c \uc678\uad6d \uc790\ubcf8\uc744 \ub04c\uc5b4\ub4e4\uc774\ub824 \ud588\ub2e4\ub294 \uac83\uc740 \ub2f9\uc2dc \uc911\uad6d \ubbfc\uc871\uc8fc\uc758\uc758 \uc815\uc11c\uc5d0 \ud06c\uac8c \uac70\uc2a4\ub974\ub294 \uacb0\uc815\uc774\uc5c8\uc73c\uba70 \uc774\ubbf8 \ud22c\uc790\ud55c \ubbfc\uac04\uc778\ub4e4\uc758 \uc774\uad8c\uc744 \ubc15\ud0c8\ud558\ub294 \uc870\ucc98\uc600\ub2e4. \uadf8\ub9ac\ud558\uc5ec \ud53c\ud574\ub97c \uc785\uc740 \uc9c0\uc5ed \uac00\uc6b4\ub370 \ud638\ub0a8 \uc7a5\uc0ac(\u9577\u6c99)\uc5d0\uc11c \ucc98\uc74c\uc73c\ub85c \uad6d\uc720\ud654\ub839\uc744 \ucde8\uc18c\ud558\ub3c4\ub85d \uc694\uad6c\ud558\uc600\ub2e4. \ub610, \uc870\uc138\uc5d0 \ud56d\uac70\ud558\uc790\ub294 \uc0c1\uc778\uc758 \ucca0\uc2dc\uc640 \ub3d9\ub9f9\ud734\ud559\ub3c4 \uc77c\uc5b4\ub0ac\ub2e4. \uc774\ud6c4 6\uc6d4 17\uc77c \u2018\ubcf4\ub85c \ub3d9\uc9c0\ud68c\u2019\uac00 \uc870\uc9c1\ub418\uc5b4 \ud638\ubd81\uacfc \uc591\uad11\uc5d0\uc11c\ub3c4 \ubcf4\ub85c\uc6b4\ub3d9\uc774 \uc77c\uc5b4\ub0ac\ub294\ub370, \uadf8 \uac00\uc6b4\ub370 \uc81c\uc77c \uaca9\ub82c\ud558\uac8c \uc77c\uc5b4\ub09c \uacf3\uc774 \uc4f0\ucd28 \uc131\uc774\uc5c8\ub2e4.", "answer": "\uc4f0\ucd28 \uc131", "sentence": "\uadf8 \uac00\uc6b4\ub370 \uc81c\uc77c \uaca9\ub82c\ud558\uac8c \uc77c\uc5b4\ub09c \uacf3\uc774 \uc4f0\ucd28 \uc131 \uc774\uc5c8\ub2e4.", "paragraph_sentence": "\uccad\uc870\uc758 \uc774\ub7ec\ud55c \uc870\ucc98\ub294 \uacb0\uc815 \ub2f9\uc0ac\uc790\ub97c \uc81c\uc678\ud55c \ubaa8\ub4e0 \uc0ac\ub78c\uc744 \uaca9\ubd84\uc2dc\ucf30\ub2e4. \uc6b0\uc120 \ub0b4\uac01\uc774 \uc758\ud68c\uc5d0 \uc544\ubb34\ub7f0 \uc0c1\uc758 \uc5c6\uc774 \uc77c\ubc29\uc801\uc73c\ub85c \uacb0\uc815\ud558\uace0 \ud1b5\ubcf4\ud55c \uc0ac\ud56d\uc774\uc5c8\uace0 \uc815\ubd80\uac00 \uc57d\uc18d\ud55c \ubbfc\uc8fc\uc801\uc778 \uc808\ucc28\ub97c \uc644\uc804\ud788 \ubb34\uc2dc\ud588\uace0 \ub9cc\uc8fc\uc871\uc774 \ub3c5\uc810\ud55c \ub0b4\uac01\uc5d0\uc11c \uc678\uad6d \uc790\ubcf8\uc744 \ub04c\uc5b4\ub4e4\uc774\ub824 \ud588\ub2e4\ub294 \uac83\uc740 \ub2f9\uc2dc \uc911\uad6d \ubbfc\uc871\uc8fc\uc758\uc758 \uc815\uc11c\uc5d0 \ud06c\uac8c \uac70\uc2a4\ub974\ub294 \uacb0\uc815\uc774\uc5c8\uc73c\uba70 \uc774\ubbf8 \ud22c\uc790\ud55c \ubbfc\uac04\uc778\ub4e4\uc758 \uc774\uad8c\uc744 \ubc15\ud0c8\ud558\ub294 \uc870\ucc98\uc600\ub2e4. \uadf8\ub9ac\ud558\uc5ec \ud53c\ud574\ub97c \uc785\uc740 \uc9c0\uc5ed \uac00\uc6b4\ub370 \ud638\ub0a8 \uc7a5\uc0ac(\u9577\u6c99)\uc5d0\uc11c \ucc98\uc74c\uc73c\ub85c \uad6d\uc720\ud654\ub839\uc744 \ucde8\uc18c\ud558\ub3c4\ub85d \uc694\uad6c\ud558\uc600\ub2e4. \ub610, \uc870\uc138\uc5d0 \ud56d\uac70\ud558\uc790\ub294 \uc0c1\uc778\uc758 \ucca0\uc2dc\uc640 \ub3d9\ub9f9\ud734\ud559\ub3c4 \uc77c\uc5b4\ub0ac\ub2e4. \uc774\ud6c4 6\uc6d4 17\uc77c \u2018\ubcf4\ub85c \ub3d9\uc9c0\ud68c\u2019\uac00 \uc870\uc9c1\ub418\uc5b4 \ud638\ubd81\uacfc \uc591\uad11\uc5d0\uc11c\ub3c4 \ubcf4\ub85c\uc6b4\ub3d9\uc774 \uc77c\uc5b4\ub0ac\ub294\ub370, \uadf8 \uac00\uc6b4\ub370 \uc81c\uc77c \uaca9\ub82c\ud558\uac8c \uc77c\uc5b4\ub09c \uacf3\uc774 \uc4f0\ucd28 \uc131 \uc774\uc5c8\ub2e4. ", "paragraph_answer": "\uccad\uc870\uc758 \uc774\ub7ec\ud55c \uc870\ucc98\ub294 \uacb0\uc815 \ub2f9\uc0ac\uc790\ub97c \uc81c\uc678\ud55c \ubaa8\ub4e0 \uc0ac\ub78c\uc744 \uaca9\ubd84\uc2dc\ucf30\ub2e4. \uc6b0\uc120 \ub0b4\uac01\uc774 \uc758\ud68c\uc5d0 \uc544\ubb34\ub7f0 \uc0c1\uc758 \uc5c6\uc774 \uc77c\ubc29\uc801\uc73c\ub85c \uacb0\uc815\ud558\uace0 \ud1b5\ubcf4\ud55c \uc0ac\ud56d\uc774\uc5c8\uace0 \uc815\ubd80\uac00 \uc57d\uc18d\ud55c \ubbfc\uc8fc\uc801\uc778 \uc808\ucc28\ub97c \uc644\uc804\ud788 \ubb34\uc2dc\ud588\uace0 \ub9cc\uc8fc\uc871\uc774 \ub3c5\uc810\ud55c \ub0b4\uac01\uc5d0\uc11c \uc678\uad6d \uc790\ubcf8\uc744 \ub04c\uc5b4\ub4e4\uc774\ub824 \ud588\ub2e4\ub294 \uac83\uc740 \ub2f9\uc2dc \uc911\uad6d \ubbfc\uc871\uc8fc\uc758\uc758 \uc815\uc11c\uc5d0 \ud06c\uac8c \uac70\uc2a4\ub974\ub294 \uacb0\uc815\uc774\uc5c8\uc73c\uba70 \uc774\ubbf8 \ud22c\uc790\ud55c \ubbfc\uac04\uc778\ub4e4\uc758 \uc774\uad8c\uc744 \ubc15\ud0c8\ud558\ub294 \uc870\ucc98\uc600\ub2e4. \uadf8\ub9ac\ud558\uc5ec \ud53c\ud574\ub97c \uc785\uc740 \uc9c0\uc5ed \uac00\uc6b4\ub370 \ud638\ub0a8 \uc7a5\uc0ac(\u9577\u6c99)\uc5d0\uc11c \ucc98\uc74c\uc73c\ub85c \uad6d\uc720\ud654\ub839\uc744 \ucde8\uc18c\ud558\ub3c4\ub85d \uc694\uad6c\ud558\uc600\ub2e4. \ub610, \uc870\uc138\uc5d0 \ud56d\uac70\ud558\uc790\ub294 \uc0c1\uc778\uc758 \ucca0\uc2dc\uc640 \ub3d9\ub9f9\ud734\ud559\ub3c4 \uc77c\uc5b4\ub0ac\ub2e4. \uc774\ud6c4 6\uc6d4 17\uc77c \u2018\ubcf4\ub85c \ub3d9\uc9c0\ud68c\u2019\uac00 \uc870\uc9c1\ub418\uc5b4 \ud638\ubd81\uacfc \uc591\uad11\uc5d0\uc11c\ub3c4 \ubcf4\ub85c\uc6b4\ub3d9\uc774 \uc77c\uc5b4\ub0ac\ub294\ub370, \uadf8 \uac00\uc6b4\ub370 \uc81c\uc77c \uaca9\ub82c\ud558\uac8c \uc77c\uc5b4\ub09c \uacf3\uc774 \uc4f0\ucd28 \uc131 \uc774\uc5c8\ub2e4.", "sentence_answer": "\uadf8 \uac00\uc6b4\ub370 \uc81c\uc77c \uaca9\ub82c\ud558\uac8c \uc77c\uc5b4\ub09c \uacf3\uc774 \uc4f0\ucd28 \uc131 \uc774\uc5c8\ub2e4.", "paragraph_id": "6533360-17-1", "paragraph_question": "question: \ud638\ubd81\uacfc \uc591\uad11\uc5d0\uc11c \ubcf4\ub85c\uc6b4\ub3d9\uc774 \uc77c\uc5b4\ub0ac\ub294\ub370 \uadf8\uc911 \uac00\uc7a5 \uaca9\ub82c\ud558\uac8c \uc77c\uc5b4\ub09c\uacf3\uc740?, context: \uccad\uc870\uc758 \uc774\ub7ec\ud55c \uc870\ucc98\ub294 \uacb0\uc815 \ub2f9\uc0ac\uc790\ub97c \uc81c\uc678\ud55c \ubaa8\ub4e0 \uc0ac\ub78c\uc744 \uaca9\ubd84\uc2dc\ucf30\ub2e4. \uc6b0\uc120 \ub0b4\uac01\uc774 \uc758\ud68c\uc5d0 \uc544\ubb34\ub7f0 \uc0c1\uc758 \uc5c6\uc774 \uc77c\ubc29\uc801\uc73c\ub85c \uacb0\uc815\ud558\uace0 \ud1b5\ubcf4\ud55c \uc0ac\ud56d\uc774\uc5c8\uace0 \uc815\ubd80\uac00 \uc57d\uc18d\ud55c \ubbfc\uc8fc\uc801\uc778 \uc808\ucc28\ub97c \uc644\uc804\ud788 \ubb34\uc2dc\ud588\uace0 \ub9cc\uc8fc\uc871\uc774 \ub3c5\uc810\ud55c \ub0b4\uac01\uc5d0\uc11c \uc678\uad6d \uc790\ubcf8\uc744 \ub04c\uc5b4\ub4e4\uc774\ub824 \ud588\ub2e4\ub294 \uac83\uc740 \ub2f9\uc2dc \uc911\uad6d \ubbfc\uc871\uc8fc\uc758\uc758 \uc815\uc11c\uc5d0 \ud06c\uac8c \uac70\uc2a4\ub974\ub294 \uacb0\uc815\uc774\uc5c8\uc73c\uba70 \uc774\ubbf8 \ud22c\uc790\ud55c \ubbfc\uac04\uc778\ub4e4\uc758 \uc774\uad8c\uc744 \ubc15\ud0c8\ud558\ub294 \uc870\ucc98\uc600\ub2e4. \uadf8\ub9ac\ud558\uc5ec \ud53c\ud574\ub97c \uc785\uc740 \uc9c0\uc5ed \uac00\uc6b4\ub370 \ud638\ub0a8 \uc7a5\uc0ac(\u9577\u6c99)\uc5d0\uc11c \ucc98\uc74c\uc73c\ub85c \uad6d\uc720\ud654\ub839\uc744 \ucde8\uc18c\ud558\ub3c4\ub85d \uc694\uad6c\ud558\uc600\ub2e4. \ub610, \uc870\uc138\uc5d0 \ud56d\uac70\ud558\uc790\ub294 \uc0c1\uc778\uc758 \ucca0\uc2dc\uc640 \ub3d9\ub9f9\ud734\ud559\ub3c4 \uc77c\uc5b4\ub0ac\ub2e4. \uc774\ud6c4 6\uc6d4 17\uc77c \u2018\ubcf4\ub85c \ub3d9\uc9c0\ud68c\u2019\uac00 \uc870\uc9c1\ub418\uc5b4 \ud638\ubd81\uacfc \uc591\uad11\uc5d0\uc11c\ub3c4 \ubcf4\ub85c\uc6b4\ub3d9\uc774 \uc77c\uc5b4\ub0ac\ub294\ub370, \uadf8 \uac00\uc6b4\ub370 \uc81c\uc77c \uaca9\ub82c\ud558\uac8c \uc77c\uc5b4\ub09c \uacf3\uc774 \uc4f0\ucd28 \uc131\uc774\uc5c8\ub2e4."} {"question": "\ub098\uce74\ubaa8\ub9ac \uc544\ud0a4\ub098\uac00 \ucee8\ud2f0\ub274 \uac1c\uc778\uc18c\uc18d\uc0ac\ub97c \uc124\ub9bd\ud55c \ub144\ub3c4\ub294?", "paragraph": "\u3008Dear Friend\u3009 \ubc1c\ub9e4 \uc9c1\uc804, \ub098\uce74\ubaa8\ub9ac\ub294 10\ub144 \uac00\uae4c\uc774 \uc18c\uc18d\ub418\uc5b4 \uc788\ub294 \ucf04\uc628\uacfc\uc758 \uc7ac\uacc4\uc57d\uc744 \ud3ec\uae30\ud558\uace0 \uc774\ub4ec\ud574\uc778 1991\ub144\uc5d0 \uc0c8\ub85c \uac1c\uc778\uc0ac\ubb34\uc18c\uc778 Collection(\ucf5c\ub809\uc158)\uc744 \uc138\uc6e0\ub2e4. \uadf8\ub7ec\ub098, \uace7 \uc18c\uc18d\uc0ac\uc758 \uc784\uc6d0\ub4e4\uacfc\uc758 \ud2b8\ub7ec\ube14 \ub54c\ubb38\uc5d0 \ub2e4\uc2dc \ub098\uc624\uace0 1992\ub144 \uc790\uc2e0\uc758 \uac1c\uc778 \uc18c\uc18d\uc0ac\uc778 Continue(\ucee8\ud2f0\ub274)\ub97c \ub2e4\uc2dc \uc138\uc6e0\ub2e4. \ub610\ud55c \uc6cc\ub108 \ud30c\uc774\uc624\ub2c8\uc5b4\uc640\ub3c4 \uc74c\uc545 \uc81c\uc791\uacfc \ucf58\uc149\ud2b8\uc0c1 \uc790\uc8fc \ucda9\ub3cc\uc774 \uc788\uc5b4\uc624\ub358 \ub05d\uc5d0, \ub098\uce74\ubaa8\ub9ac\ub294 \u3008\ud6c4\ud0c0\ub9ac\uc2dc\uc988\uce74 -\u300c\ucc9c\ud558\uc804\uc124\uc0b4\uc778\uc0ac\uac74\u300d\uc73c\ub85c\ubd80\ud130\u3009(\u4e8c\u4eba\u9759 -\u300c\u5929\u6cb3\u4f1d\u8aac\u6bba\u4eba\u4e8b\u4ef6\u300d\u3088\u308a)\ub97c \ub9c8\uc9c0\ub9c9\uc73c\ub85c \uc6cc\ub108 \ud30c\uc774\uc624\ub2c8\uc5b4\uc5d0\uc11c \ub3c5\ub9bd\ud558\uc5ec MCA\uc640 \uc0c8\ub85c \uacc4\uc57d\uc744 \uccb4\uacb0\ud558\uc600\ub2e4. \uc774 \uacfc\uc815\uc5d0\uc11c \ub808\uc774\ube14 \uc774\uc801\ub8cc\uc778 3\uc5b5 5000\ub9cc\uc5d4\uc758 \ubd84\ubc30\ub97c \ub450\uace0 \ub098\uce74\ubaa8\ub9ac\uc640 \uc18c\uc18d\uc0ac Continue\uac04\uc758 \ubc95\uc815 \uacf5\ubc29\uc774 \uc774\uc5b4\uc84c\uace0 \ubd84\uac1c\ud55c \ub098\uce74\ubaa8\ub9ac\ub294 \ub2e4\uc2dc \uc18c\uc18d\uc0ac\ub97c NAPC\ub85c \uc774\uc801\ud558\uc600\ub2e4. \uc774\ub7ec\ud55c \uc18c\uc18d\uc0ac, \ub808\uc774\ube14\uacfc\uc758 \uac08\ub4f1\uc73c\ub85c 1992\ub144 \uc5ec\ub984\uc5d0 \ub0bc \uc608\uc815\uc774\ub358 \uc0c8 \uc2f1\uae00\uacfc \uc568\ubc94\uc740 \uacb0\uad6d \ud574\ub97c \ub118\uaca8 \uc0c8 \uc18c\uc18d\uc0ac\uc640 \ub808\uc774\ube14\uc5d0\uc11c \uc81c\ub300\ub85c \uc790\ub9ac\ub97c \uc7a1\uc740 \ud6c4\uc5d0\uc57c \ubc1c\ub9e4\ud560 \uc218 \uc788\uc5c8\ub2e4.", "answer": "1992\ub144", "sentence": "\uadf8\ub7ec\ub098, \uace7 \uc18c\uc18d\uc0ac\uc758 \uc784\uc6d0\ub4e4\uacfc\uc758 \ud2b8\ub7ec\ube14 \ub54c\ubb38\uc5d0 \ub2e4\uc2dc \ub098\uc624\uace0 1992\ub144 \uc790\uc2e0\uc758 \uac1c\uc778 \uc18c\uc18d\uc0ac\uc778 Continue(\ucee8\ud2f0\ub274)\ub97c \ub2e4\uc2dc \uc138\uc6e0\ub2e4.", "paragraph_sentence": "\u3008Dear Friend\u3009 \ubc1c\ub9e4 \uc9c1\uc804, \ub098\uce74\ubaa8\ub9ac\ub294 10\ub144 \uac00\uae4c\uc774 \uc18c\uc18d\ub418\uc5b4 \uc788\ub294 \ucf04\uc628\uacfc\uc758 \uc7ac\uacc4\uc57d\uc744 \ud3ec\uae30\ud558\uace0 \uc774\ub4ec\ud574\uc778 1991\ub144\uc5d0 \uc0c8\ub85c \uac1c\uc778\uc0ac\ubb34\uc18c\uc778 Collection(\ucf5c\ub809\uc158)\uc744 \uc138\uc6e0\ub2e4. \uadf8\ub7ec\ub098, \uace7 \uc18c\uc18d\uc0ac\uc758 \uc784\uc6d0\ub4e4\uacfc\uc758 \ud2b8\ub7ec\ube14 \ub54c\ubb38\uc5d0 \ub2e4\uc2dc \ub098\uc624\uace0 1992\ub144 \uc790\uc2e0\uc758 \uac1c\uc778 \uc18c\uc18d\uc0ac\uc778 Continue(\ucee8\ud2f0\ub274)\ub97c \ub2e4\uc2dc \uc138\uc6e0\ub2e4. \ub610\ud55c \uc6cc\ub108 \ud30c\uc774\uc624\ub2c8\uc5b4\uc640\ub3c4 \uc74c\uc545 \uc81c\uc791\uacfc \ucf58\uc149\ud2b8\uc0c1 \uc790\uc8fc \ucda9\ub3cc\uc774 \uc788\uc5b4\uc624\ub358 \ub05d\uc5d0, \ub098\uce74\ubaa8\ub9ac\ub294 \u3008\ud6c4\ud0c0\ub9ac\uc2dc\uc988\uce74 -\u300c\ucc9c\ud558\uc804\uc124\uc0b4\uc778\uc0ac\uac74\u300d\uc73c\ub85c\ubd80\ud130\u3009(\u4e8c\u4eba\u9759 -\u300c\u5929\u6cb3\u4f1d\u8aac\u6bba\u4eba\u4e8b\u4ef6\u300d\u3088\u308a)\ub97c \ub9c8\uc9c0\ub9c9\uc73c\ub85c \uc6cc\ub108 \ud30c\uc774\uc624\ub2c8\uc5b4\uc5d0\uc11c \ub3c5\ub9bd\ud558\uc5ec MCA\uc640 \uc0c8\ub85c \uacc4\uc57d\uc744 \uccb4\uacb0\ud558\uc600\ub2e4. \uc774 \uacfc\uc815\uc5d0\uc11c \ub808\uc774\ube14 \uc774\uc801\ub8cc\uc778 3\uc5b5 5000\ub9cc\uc5d4\uc758 \ubd84\ubc30\ub97c \ub450\uace0 \ub098\uce74\ubaa8\ub9ac\uc640 \uc18c\uc18d\uc0ac Continue\uac04\uc758 \ubc95\uc815 \uacf5\ubc29\uc774 \uc774\uc5b4\uc84c\uace0 \ubd84\uac1c\ud55c \ub098\uce74\ubaa8\ub9ac\ub294 \ub2e4\uc2dc \uc18c\uc18d\uc0ac\ub97c NAPC\ub85c \uc774\uc801\ud558\uc600\ub2e4. \uc774\ub7ec\ud55c \uc18c\uc18d\uc0ac, \ub808\uc774\ube14\uacfc\uc758 \uac08\ub4f1\uc73c\ub85c 1992\ub144 \uc5ec\ub984\uc5d0 \ub0bc \uc608\uc815\uc774\ub358 \uc0c8 \uc2f1\uae00\uacfc \uc568\ubc94\uc740 \uacb0\uad6d \ud574\ub97c \ub118\uaca8 \uc0c8 \uc18c\uc18d\uc0ac\uc640 \ub808\uc774\ube14\uc5d0\uc11c \uc81c\ub300\ub85c \uc790\ub9ac\ub97c \uc7a1\uc740 \ud6c4\uc5d0\uc57c \ubc1c\ub9e4\ud560 \uc218 \uc788\uc5c8\ub2e4.", "paragraph_answer": "\u3008Dear Friend\u3009 \ubc1c\ub9e4 \uc9c1\uc804, \ub098\uce74\ubaa8\ub9ac\ub294 10\ub144 \uac00\uae4c\uc774 \uc18c\uc18d\ub418\uc5b4 \uc788\ub294 \ucf04\uc628\uacfc\uc758 \uc7ac\uacc4\uc57d\uc744 \ud3ec\uae30\ud558\uace0 \uc774\ub4ec\ud574\uc778 1991\ub144\uc5d0 \uc0c8\ub85c \uac1c\uc778\uc0ac\ubb34\uc18c\uc778 Collection(\ucf5c\ub809\uc158)\uc744 \uc138\uc6e0\ub2e4. \uadf8\ub7ec\ub098, \uace7 \uc18c\uc18d\uc0ac\uc758 \uc784\uc6d0\ub4e4\uacfc\uc758 \ud2b8\ub7ec\ube14 \ub54c\ubb38\uc5d0 \ub2e4\uc2dc \ub098\uc624\uace0 1992\ub144 \uc790\uc2e0\uc758 \uac1c\uc778 \uc18c\uc18d\uc0ac\uc778 Continue(\ucee8\ud2f0\ub274)\ub97c \ub2e4\uc2dc \uc138\uc6e0\ub2e4. \ub610\ud55c \uc6cc\ub108 \ud30c\uc774\uc624\ub2c8\uc5b4\uc640\ub3c4 \uc74c\uc545 \uc81c\uc791\uacfc \ucf58\uc149\ud2b8\uc0c1 \uc790\uc8fc \ucda9\ub3cc\uc774 \uc788\uc5b4\uc624\ub358 \ub05d\uc5d0, \ub098\uce74\ubaa8\ub9ac\ub294 \u3008\ud6c4\ud0c0\ub9ac\uc2dc\uc988\uce74 -\u300c\ucc9c\ud558\uc804\uc124\uc0b4\uc778\uc0ac\uac74\u300d\uc73c\ub85c\ubd80\ud130\u3009(\u4e8c\u4eba\u9759 -\u300c\u5929\u6cb3\u4f1d\u8aac\u6bba\u4eba\u4e8b\u4ef6\u300d\u3088\u308a)\ub97c \ub9c8\uc9c0\ub9c9\uc73c\ub85c \uc6cc\ub108 \ud30c\uc774\uc624\ub2c8\uc5b4\uc5d0\uc11c \ub3c5\ub9bd\ud558\uc5ec MCA\uc640 \uc0c8\ub85c \uacc4\uc57d\uc744 \uccb4\uacb0\ud558\uc600\ub2e4. \uc774 \uacfc\uc815\uc5d0\uc11c \ub808\uc774\ube14 \uc774\uc801\ub8cc\uc778 3\uc5b5 5000\ub9cc\uc5d4\uc758 \ubd84\ubc30\ub97c \ub450\uace0 \ub098\uce74\ubaa8\ub9ac\uc640 \uc18c\uc18d\uc0ac Continue\uac04\uc758 \ubc95\uc815 \uacf5\ubc29\uc774 \uc774\uc5b4\uc84c\uace0 \ubd84\uac1c\ud55c \ub098\uce74\ubaa8\ub9ac\ub294 \ub2e4\uc2dc \uc18c\uc18d\uc0ac\ub97c NAPC\ub85c \uc774\uc801\ud558\uc600\ub2e4. \uc774\ub7ec\ud55c \uc18c\uc18d\uc0ac, \ub808\uc774\ube14\uacfc\uc758 \uac08\ub4f1\uc73c\ub85c 1992\ub144 \uc5ec\ub984\uc5d0 \ub0bc \uc608\uc815\uc774\ub358 \uc0c8 \uc2f1\uae00\uacfc \uc568\ubc94\uc740 \uacb0\uad6d \ud574\ub97c \ub118\uaca8 \uc0c8 \uc18c\uc18d\uc0ac\uc640 \ub808\uc774\ube14\uc5d0\uc11c \uc81c\ub300\ub85c \uc790\ub9ac\ub97c \uc7a1\uc740 \ud6c4\uc5d0\uc57c \ubc1c\ub9e4\ud560 \uc218 \uc788\uc5c8\ub2e4.", "sentence_answer": "\uadf8\ub7ec\ub098, \uace7 \uc18c\uc18d\uc0ac\uc758 \uc784\uc6d0\ub4e4\uacfc\uc758 \ud2b8\ub7ec\ube14 \ub54c\ubb38\uc5d0 \ub2e4\uc2dc \ub098\uc624\uace0 1992\ub144 \uc790\uc2e0\uc758 \uac1c\uc778 \uc18c\uc18d\uc0ac\uc778 Continue(\ucee8\ud2f0\ub274)\ub97c \ub2e4\uc2dc \uc138\uc6e0\ub2e4.", "paragraph_id": "6546040-13-1", "paragraph_question": "question: \ub098\uce74\ubaa8\ub9ac \uc544\ud0a4\ub098\uac00 \ucee8\ud2f0\ub274 \uac1c\uc778\uc18c\uc18d\uc0ac\ub97c \uc124\ub9bd\ud55c \ub144\ub3c4\ub294?, context: \u3008Dear Friend\u3009 \ubc1c\ub9e4 \uc9c1\uc804, \ub098\uce74\ubaa8\ub9ac\ub294 10\ub144 \uac00\uae4c\uc774 \uc18c\uc18d\ub418\uc5b4 \uc788\ub294 \ucf04\uc628\uacfc\uc758 \uc7ac\uacc4\uc57d\uc744 \ud3ec\uae30\ud558\uace0 \uc774\ub4ec\ud574\uc778 1991\ub144\uc5d0 \uc0c8\ub85c \uac1c\uc778\uc0ac\ubb34\uc18c\uc778 Collection(\ucf5c\ub809\uc158)\uc744 \uc138\uc6e0\ub2e4. \uadf8\ub7ec\ub098, \uace7 \uc18c\uc18d\uc0ac\uc758 \uc784\uc6d0\ub4e4\uacfc\uc758 \ud2b8\ub7ec\ube14 \ub54c\ubb38\uc5d0 \ub2e4\uc2dc \ub098\uc624\uace0 1992\ub144 \uc790\uc2e0\uc758 \uac1c\uc778 \uc18c\uc18d\uc0ac\uc778 Continue(\ucee8\ud2f0\ub274)\ub97c \ub2e4\uc2dc \uc138\uc6e0\ub2e4. \ub610\ud55c \uc6cc\ub108 \ud30c\uc774\uc624\ub2c8\uc5b4\uc640\ub3c4 \uc74c\uc545 \uc81c\uc791\uacfc \ucf58\uc149\ud2b8\uc0c1 \uc790\uc8fc \ucda9\ub3cc\uc774 \uc788\uc5b4\uc624\ub358 \ub05d\uc5d0, \ub098\uce74\ubaa8\ub9ac\ub294 \u3008\ud6c4\ud0c0\ub9ac\uc2dc\uc988\uce74 -\u300c\ucc9c\ud558\uc804\uc124\uc0b4\uc778\uc0ac\uac74\u300d\uc73c\ub85c\ubd80\ud130\u3009(\u4e8c\u4eba\u9759 -\u300c\u5929\u6cb3\u4f1d\u8aac\u6bba\u4eba\u4e8b\u4ef6\u300d\u3088\u308a)\ub97c \ub9c8\uc9c0\ub9c9\uc73c\ub85c \uc6cc\ub108 \ud30c\uc774\uc624\ub2c8\uc5b4\uc5d0\uc11c \ub3c5\ub9bd\ud558\uc5ec MCA\uc640 \uc0c8\ub85c \uacc4\uc57d\uc744 \uccb4\uacb0\ud558\uc600\ub2e4. \uc774 \uacfc\uc815\uc5d0\uc11c \ub808\uc774\ube14 \uc774\uc801\ub8cc\uc778 3\uc5b5 5000\ub9cc\uc5d4\uc758 \ubd84\ubc30\ub97c \ub450\uace0 \ub098\uce74\ubaa8\ub9ac\uc640 \uc18c\uc18d\uc0ac Continue\uac04\uc758 \ubc95\uc815 \uacf5\ubc29\uc774 \uc774\uc5b4\uc84c\uace0 \ubd84\uac1c\ud55c \ub098\uce74\ubaa8\ub9ac\ub294 \ub2e4\uc2dc \uc18c\uc18d\uc0ac\ub97c NAPC\ub85c \uc774\uc801\ud558\uc600\ub2e4. \uc774\ub7ec\ud55c \uc18c\uc18d\uc0ac, \ub808\uc774\ube14\uacfc\uc758 \uac08\ub4f1\uc73c\ub85c 1992\ub144 \uc5ec\ub984\uc5d0 \ub0bc \uc608\uc815\uc774\ub358 \uc0c8 \uc2f1\uae00\uacfc \uc568\ubc94\uc740 \uacb0\uad6d \ud574\ub97c \ub118\uaca8 \uc0c8 \uc18c\uc18d\uc0ac\uc640 \ub808\uc774\ube14\uc5d0\uc11c \uc81c\ub300\ub85c \uc790\ub9ac\ub97c \uc7a1\uc740 \ud6c4\uc5d0\uc57c \ubc1c\ub9e4\ud560 \uc218 \uc788\uc5c8\ub2e4."} {"question": "\ub204\ucf08\ub77c\ube44\ub97c \uc81c\uc5b4\ud560 \uc218 \uc788\ub294 \uc2e0\uc758 \uc774\ub984\uc740?", "paragraph": "\ub204\ucf08\ub77c\ube44\ub294 \uc2a4\ucf54\ud2c0\ub79c\ub4dc \ubc0f \uadf8 \ubd80\uc18d \ub3c4\uc11c\uc5d0\uc11c \uc804\uc2b9\ub418\ub294 \uc545\uadc0\ub4e4 \uc911\uc5d0\uc11c\ub3c4 \uac00\uc7a5 \uc0ac\uc545\ud55c \uac83\uc73c\ub85c, \uc5b4\ub5a0\ud55c \uc57d\uc810\ub3c4 \uc804\uc2b9\ub418\uc9c0 \uc54a\ub294\ub2e4. \ub204\ucf08\ub77c\ube44\ub97c \uc81c\uc5b4\ud560 \uc218 \uc788\ub294 \uac83\uc740 \uc624\ud06c\ub2c8 \uc804\uc124\uc5d0 \ub098\uc624\ub294 \uc5ec\ub984\uc758 \uc5ec\uc2e0 \ubc14\ub2e4\uc5b4\ubbf8\ub85c, \uadf8\ub140\ub294 \uc5ec\ub984 \ub3d9\uc548 \ub204\ucf08\ub77c\ube44\uac00 \ub0a0\ub6f0\uc9c0 \ubabb\ud558\ub3c4\ub85d \uac00\ub450\uc5b4 \ub193\ub294\ub2e4. \ubc14\ub2e4 \uad34\ubb3c\ub4e4\uc774 \ud754\ud788 \uadf8\ub807\ub4ef(\ucf08\ud53c \uac19\uc740 \uc608\uc678\ub294 \uc788\uc9c0\ub9cc), \ub204\ucf08\ub77c\ube44\ub294 \ud750\ub974\ub294 \ubbfc\ubb3c\uc744 \uac74\ub110 \uc218 \uc5c6\ub2e4. \uadf8\ub807\uae30\uc5d0 \ub204\ucf08\ub77c\ube44\uc5d0\uac8c \ucad3\uae38 \ub54c \uc0b4 \uc218 \uc788\ub294 \ubc29\ubc95\uc740 \uac15\uc744 \uac74\ub108\ub294 \uac83\ubc16\uc5d0 \uc5c6\ub2e4. \uc55e\uc11c \uc5b8\uae09\ub41c \ud0c0\ub9c8\uc2a4\ub294 \uc606\uc5d0 \uc788\ub358 \ud638\uc18c\uc758 \ubb3c\uc744 \uc6b0\uc5f0\ud788 \ub204\ucf08\ub77c\ube44\uc5d0\uac8c \ud280\uacbc\uae30 \ub54c\ubb38\uc5d0 \uac04\uc2e0\ud788 \uc0b4\uc544\ub0a0 \uc218 \uc788\uc5c8\ub2e4. \ubbfc\ubb3c\ub85c \uc778\ud574 \uad34\ubb3c\uc774 \uc2e0\uacbd\uc774 \ud750\ud2b8\ub7ec\uc9c4 \ud2c8\uc744 \ud0c0 \ud0c0\ub9c8\uc2a4\ub294 \uc606\uc758 \ubbfc\ubb3c \uc218\ub85c\ub97c \uac74\ub108\ub6f0\uc5c8\uace0 \uc548\uc804\ud558\uac8c \ubc18\ub300\ud3b8 \uac15\ub451\uc5d0 \ub3c4\ucc29\ud588\ub2e4.", "answer": "\ubc14\ub2e4\uc5b4\ubbf8", "sentence": "\ub204\ucf08\ub77c\ube44\ub97c \uc81c\uc5b4\ud560 \uc218 \uc788\ub294 \uac83\uc740 \uc624\ud06c\ub2c8 \uc804\uc124\uc5d0 \ub098\uc624\ub294 \uc5ec\ub984\uc758 \uc5ec\uc2e0 \ubc14\ub2e4\uc5b4\ubbf8 \ub85c, \uadf8\ub140\ub294 \uc5ec\ub984 \ub3d9\uc548 \ub204\ucf08\ub77c\ube44\uac00 \ub0a0\ub6f0\uc9c0 \ubabb\ud558\ub3c4\ub85d \uac00\ub450\uc5b4 \ub193\ub294\ub2e4.", "paragraph_sentence": "\ub204\ucf08\ub77c\ube44\ub294 \uc2a4\ucf54\ud2c0\ub79c\ub4dc \ubc0f \uadf8 \ubd80\uc18d \ub3c4\uc11c\uc5d0\uc11c \uc804\uc2b9\ub418\ub294 \uc545\uadc0\ub4e4 \uc911\uc5d0\uc11c\ub3c4 \uac00\uc7a5 \uc0ac\uc545\ud55c \uac83\uc73c\ub85c, \uc5b4\ub5a0\ud55c \uc57d\uc810\ub3c4 \uc804\uc2b9\ub418\uc9c0 \uc54a\ub294\ub2e4. \ub204\ucf08\ub77c\ube44\ub97c \uc81c\uc5b4\ud560 \uc218 \uc788\ub294 \uac83\uc740 \uc624\ud06c\ub2c8 \uc804\uc124\uc5d0 \ub098\uc624\ub294 \uc5ec\ub984\uc758 \uc5ec\uc2e0 \ubc14\ub2e4\uc5b4\ubbf8 \ub85c, \uadf8\ub140\ub294 \uc5ec\ub984 \ub3d9\uc548 \ub204\ucf08\ub77c\ube44\uac00 \ub0a0\ub6f0\uc9c0 \ubabb\ud558\ub3c4\ub85d \uac00\ub450\uc5b4 \ub193\ub294\ub2e4. \ubc14\ub2e4 \uad34\ubb3c\ub4e4\uc774 \ud754\ud788 \uadf8\ub807\ub4ef(\ucf08\ud53c \uac19\uc740 \uc608\uc678\ub294 \uc788\uc9c0\ub9cc), \ub204\ucf08\ub77c\ube44\ub294 \ud750\ub974\ub294 \ubbfc\ubb3c\uc744 \uac74\ub110 \uc218 \uc5c6\ub2e4. \uadf8\ub807\uae30\uc5d0 \ub204\ucf08\ub77c\ube44\uc5d0\uac8c \ucad3\uae38 \ub54c \uc0b4 \uc218 \uc788\ub294 \ubc29\ubc95\uc740 \uac15\uc744 \uac74\ub108\ub294 \uac83\ubc16\uc5d0 \uc5c6\ub2e4. \uc55e\uc11c \uc5b8\uae09\ub41c \ud0c0\ub9c8\uc2a4\ub294 \uc606\uc5d0 \uc788\ub358 \ud638\uc18c\uc758 \ubb3c\uc744 \uc6b0\uc5f0\ud788 \ub204\ucf08\ub77c\ube44\uc5d0\uac8c \ud280\uacbc\uae30 \ub54c\ubb38\uc5d0 \uac04\uc2e0\ud788 \uc0b4\uc544\ub0a0 \uc218 \uc788\uc5c8\ub2e4. \ubbfc\ubb3c\ub85c \uc778\ud574 \uad34\ubb3c\uc774 \uc2e0\uacbd\uc774 \ud750\ud2b8\ub7ec\uc9c4 \ud2c8\uc744 \ud0c0 \ud0c0\ub9c8\uc2a4\ub294 \uc606\uc758 \ubbfc\ubb3c \uc218\ub85c\ub97c \uac74\ub108\ub6f0\uc5c8\uace0 \uc548\uc804\ud558\uac8c \ubc18\ub300\ud3b8 \uac15\ub451\uc5d0 \ub3c4\ucc29\ud588\ub2e4.", "paragraph_answer": "\ub204\ucf08\ub77c\ube44\ub294 \uc2a4\ucf54\ud2c0\ub79c\ub4dc \ubc0f \uadf8 \ubd80\uc18d \ub3c4\uc11c\uc5d0\uc11c \uc804\uc2b9\ub418\ub294 \uc545\uadc0\ub4e4 \uc911\uc5d0\uc11c\ub3c4 \uac00\uc7a5 \uc0ac\uc545\ud55c \uac83\uc73c\ub85c, \uc5b4\ub5a0\ud55c \uc57d\uc810\ub3c4 \uc804\uc2b9\ub418\uc9c0 \uc54a\ub294\ub2e4. \ub204\ucf08\ub77c\ube44\ub97c \uc81c\uc5b4\ud560 \uc218 \uc788\ub294 \uac83\uc740 \uc624\ud06c\ub2c8 \uc804\uc124\uc5d0 \ub098\uc624\ub294 \uc5ec\ub984\uc758 \uc5ec\uc2e0 \ubc14\ub2e4\uc5b4\ubbf8 \ub85c, \uadf8\ub140\ub294 \uc5ec\ub984 \ub3d9\uc548 \ub204\ucf08\ub77c\ube44\uac00 \ub0a0\ub6f0\uc9c0 \ubabb\ud558\ub3c4\ub85d \uac00\ub450\uc5b4 \ub193\ub294\ub2e4. \ubc14\ub2e4 \uad34\ubb3c\ub4e4\uc774 \ud754\ud788 \uadf8\ub807\ub4ef(\ucf08\ud53c \uac19\uc740 \uc608\uc678\ub294 \uc788\uc9c0\ub9cc), \ub204\ucf08\ub77c\ube44\ub294 \ud750\ub974\ub294 \ubbfc\ubb3c\uc744 \uac74\ub110 \uc218 \uc5c6\ub2e4. \uadf8\ub807\uae30\uc5d0 \ub204\ucf08\ub77c\ube44\uc5d0\uac8c \ucad3\uae38 \ub54c \uc0b4 \uc218 \uc788\ub294 \ubc29\ubc95\uc740 \uac15\uc744 \uac74\ub108\ub294 \uac83\ubc16\uc5d0 \uc5c6\ub2e4. \uc55e\uc11c \uc5b8\uae09\ub41c \ud0c0\ub9c8\uc2a4\ub294 \uc606\uc5d0 \uc788\ub358 \ud638\uc18c\uc758 \ubb3c\uc744 \uc6b0\uc5f0\ud788 \ub204\ucf08\ub77c\ube44\uc5d0\uac8c \ud280\uacbc\uae30 \ub54c\ubb38\uc5d0 \uac04\uc2e0\ud788 \uc0b4\uc544\ub0a0 \uc218 \uc788\uc5c8\ub2e4. \ubbfc\ubb3c\ub85c \uc778\ud574 \uad34\ubb3c\uc774 \uc2e0\uacbd\uc774 \ud750\ud2b8\ub7ec\uc9c4 \ud2c8\uc744 \ud0c0 \ud0c0\ub9c8\uc2a4\ub294 \uc606\uc758 \ubbfc\ubb3c \uc218\ub85c\ub97c \uac74\ub108\ub6f0\uc5c8\uace0 \uc548\uc804\ud558\uac8c \ubc18\ub300\ud3b8 \uac15\ub451\uc5d0 \ub3c4\ucc29\ud588\ub2e4.", "sentence_answer": "\ub204\ucf08\ub77c\ube44\ub97c \uc81c\uc5b4\ud560 \uc218 \uc788\ub294 \uac83\uc740 \uc624\ud06c\ub2c8 \uc804\uc124\uc5d0 \ub098\uc624\ub294 \uc5ec\ub984\uc758 \uc5ec\uc2e0 \ubc14\ub2e4\uc5b4\ubbf8 \ub85c, \uadf8\ub140\ub294 \uc5ec\ub984 \ub3d9\uc548 \ub204\ucf08\ub77c\ube44\uac00 \ub0a0\ub6f0\uc9c0 \ubabb\ud558\ub3c4\ub85d \uac00\ub450\uc5b4 \ub193\ub294\ub2e4.", "paragraph_id": "6484475-3-1", "paragraph_question": "question: \ub204\ucf08\ub77c\ube44\ub97c \uc81c\uc5b4\ud560 \uc218 \uc788\ub294 \uc2e0\uc758 \uc774\ub984\uc740?, context: \ub204\ucf08\ub77c\ube44\ub294 \uc2a4\ucf54\ud2c0\ub79c\ub4dc \ubc0f \uadf8 \ubd80\uc18d \ub3c4\uc11c\uc5d0\uc11c \uc804\uc2b9\ub418\ub294 \uc545\uadc0\ub4e4 \uc911\uc5d0\uc11c\ub3c4 \uac00\uc7a5 \uc0ac\uc545\ud55c \uac83\uc73c\ub85c, \uc5b4\ub5a0\ud55c \uc57d\uc810\ub3c4 \uc804\uc2b9\ub418\uc9c0 \uc54a\ub294\ub2e4. \ub204\ucf08\ub77c\ube44\ub97c \uc81c\uc5b4\ud560 \uc218 \uc788\ub294 \uac83\uc740 \uc624\ud06c\ub2c8 \uc804\uc124\uc5d0 \ub098\uc624\ub294 \uc5ec\ub984\uc758 \uc5ec\uc2e0 \ubc14\ub2e4\uc5b4\ubbf8\ub85c, \uadf8\ub140\ub294 \uc5ec\ub984 \ub3d9\uc548 \ub204\ucf08\ub77c\ube44\uac00 \ub0a0\ub6f0\uc9c0 \ubabb\ud558\ub3c4\ub85d \uac00\ub450\uc5b4 \ub193\ub294\ub2e4. \ubc14\ub2e4 \uad34\ubb3c\ub4e4\uc774 \ud754\ud788 \uadf8\ub807\ub4ef(\ucf08\ud53c \uac19\uc740 \uc608\uc678\ub294 \uc788\uc9c0\ub9cc), \ub204\ucf08\ub77c\ube44\ub294 \ud750\ub974\ub294 \ubbfc\ubb3c\uc744 \uac74\ub110 \uc218 \uc5c6\ub2e4. \uadf8\ub807\uae30\uc5d0 \ub204\ucf08\ub77c\ube44\uc5d0\uac8c \ucad3\uae38 \ub54c \uc0b4 \uc218 \uc788\ub294 \ubc29\ubc95\uc740 \uac15\uc744 \uac74\ub108\ub294 \uac83\ubc16\uc5d0 \uc5c6\ub2e4. \uc55e\uc11c \uc5b8\uae09\ub41c \ud0c0\ub9c8\uc2a4\ub294 \uc606\uc5d0 \uc788\ub358 \ud638\uc18c\uc758 \ubb3c\uc744 \uc6b0\uc5f0\ud788 \ub204\ucf08\ub77c\ube44\uc5d0\uac8c \ud280\uacbc\uae30 \ub54c\ubb38\uc5d0 \uac04\uc2e0\ud788 \uc0b4\uc544\ub0a0 \uc218 \uc788\uc5c8\ub2e4. \ubbfc\ubb3c\ub85c \uc778\ud574 \uad34\ubb3c\uc774 \uc2e0\uacbd\uc774 \ud750\ud2b8\ub7ec\uc9c4 \ud2c8\uc744 \ud0c0 \ud0c0\ub9c8\uc2a4\ub294 \uc606\uc758 \ubbfc\ubb3c \uc218\ub85c\ub97c \uac74\ub108\ub6f0\uc5c8\uace0 \uc548\uc804\ud558\uac8c \ubc18\ub300\ud3b8 \uac15\ub451\uc5d0 \ub3c4\ucc29\ud588\ub2e4."} {"question": "2010\ub144 8\uc6d416~17\uc77c\uc5d0 EBS \uacf5\uac10 \ubb34\ub300 \uc704\uc5d0\uc11c \uac00\ub9ac\uc628\uacfc \ud568\uaed8 \ud638\ud761\uc744 \ub9de\ucd98 \ub808\uac8c \ubc34\ub4dc\ub294?", "paragraph": "2010\ub144 8\uc6d4 16\uc77c~17\uc77c\uc5d0\ub294 EBS \uacf5\uac10 \ubb34\ub300 \uc704\uc5d0\uc11c \ub808\uac8c \ubc34\ub4dc \uc18c\uc6b8 \uc2a4\ud14c\ub514 \ub77d\ucee4\uc988\uc640 \ud638\ud761\uc744 \ub9de\ucdb0 \ubc1c\ud45c\uace1\uc778 \u3008\uc57d\uc18d\uc758 \uc7a5\uc18c\u3009\ub4f1\uacfc 2\uc9d1 \uc218\ub85d \uc608\uc815\uace1\uc778 \u3008\uc18c\ub9ac\ub97c \ub354 \ud06c\uac8c\u3009, \u3008\uc0dd\uba85\uc218\u3009\ub4f1\uc744 \ubd88\ub800\ub2e4. \uc774 \uacf5\uc5f0\uc740 10\uc6d4 8\uc77c\uc5d0 \ubc29\uc1a1\ub418\uc5c8\ub2e4. 8\uc6d4 19\uc77c\uc5d0\ub294 \uc7ac\uc988 \ubba4\uc9c0\uc158 \ud33b \uc874\uc758 \ub0b4\ud55c \uacf5\uc5f0\uc5d0 \ucc38\uc5ec\ud588\uace0, 28\uc77c\uc5d0\ub294 29\uc77c\uc5d0 \uc2a4\ud398\uc15c \uacf5\uc5f0\uc744 \uac16\ub294 \ub354\ube14\ucf00\uc774\uc640 \ud568\uaed8 8\uc6d4\uc758 \ud799\ud569\ud50c\ub808\uc774\uc57c \uc1fc\uc5d0 \ud5e4\ub4dc\ub77c\uc774\ub108\ub85c \ub098\uc11c \ubbf8\ubc1c\ud45c \uc2e0\uace1\ub4e4\uc744 \ubd80\ub974\uba70 \uacf5\uc2dd\uc801\uc73c\ub85c \ud799\ud569 \ubb34\ub300\uc5d0 \ubcf5\uadc0\ud588\ub2e4. \uadf8\ub7ec\ub098 \uc568\ubc94\uc774 \ubc1c\ud45c\ub418\uc9c0 \uc54a\uace0 \uc788\uc790 \uae30\ub300\ud558\ub358 \ud799\ud569\ud32c\ub4e4\uc740 \uc544\uc26c\uc6cc\ud588\ub2e4. \uadf8\ub7ec\ub358 \ub3c4\uc911 10\uc6d4 15\uc77c\uc5d0 \uac00\ub9ac\uc628\uc740 \ud2b8\uc704\ud130\ub97c \ud1b5\ud574 \uac00\ub9ac\uc628 2\uc9d1\uc758 \ubc1c\ub9e4\uc77c\uc744 10\uc6d4 26\uc77c\ub85c \ubc1d\ud614\uace0, \uc18c\uc18d\uc0ac \ud0c0\uc77c \ubba4\uc9c1\uc740 10\uc6d4 20\uc77c\uc5d0 \uc815\uc2dd \ubcf4\ub3c4\uc790\ub8cc\ub97c \ubc30\ud3ec\ud558\uba70 \ubc1c\ub9e4 \ub0a0\uc9dc\ub97c \ud655\uc815\uc9c0\uc5c8\ub2e4.", "answer": "\uc18c\uc6b8 \uc2a4\ud14c\ub514 \ub77d\ucee4\uc988", "sentence": "2010\ub144 8\uc6d4 16\uc77c~17\uc77c\uc5d0\ub294 EBS \uacf5\uac10 \ubb34\ub300 \uc704\uc5d0\uc11c \ub808\uac8c \ubc34\ub4dc \uc18c\uc6b8 \uc2a4\ud14c\ub514 \ub77d\ucee4\uc988 \uc640 \ud638\ud761\uc744 \ub9de\ucdb0 \ubc1c\ud45c\uace1\uc778 \u3008\uc57d\uc18d\uc758 \uc7a5\uc18c\u3009\ub4f1\uacfc 2\uc9d1 \uc218\ub85d \uc608\uc815\uace1\uc778 \u3008\uc18c\ub9ac\ub97c \ub354 \ud06c\uac8c\u3009, \u3008\uc0dd\uba85\uc218\u3009\ub4f1\uc744 \ubd88\ub800\ub2e4.", "paragraph_sentence": " 2010\ub144 8\uc6d4 16\uc77c~17\uc77c\uc5d0\ub294 EBS \uacf5\uac10 \ubb34\ub300 \uc704\uc5d0\uc11c \ub808\uac8c \ubc34\ub4dc \uc18c\uc6b8 \uc2a4\ud14c\ub514 \ub77d\ucee4\uc988 \uc640 \ud638\ud761\uc744 \ub9de\ucdb0 \ubc1c\ud45c\uace1\uc778 \u3008\uc57d\uc18d\uc758 \uc7a5\uc18c\u3009\ub4f1\uacfc 2\uc9d1 \uc218\ub85d \uc608\uc815\uace1\uc778 \u3008\uc18c\ub9ac\ub97c \ub354 \ud06c\uac8c\u3009, \u3008\uc0dd\uba85\uc218\u3009\ub4f1\uc744 \ubd88\ub800\ub2e4. \uc774 \uacf5\uc5f0\uc740 10\uc6d4 8\uc77c\uc5d0 \ubc29\uc1a1\ub418\uc5c8\ub2e4. 8\uc6d4 19\uc77c\uc5d0\ub294 \uc7ac\uc988 \ubba4\uc9c0\uc158 \ud33b \uc874\uc758 \ub0b4\ud55c \uacf5\uc5f0\uc5d0 \ucc38\uc5ec\ud588\uace0, 28\uc77c\uc5d0\ub294 29\uc77c\uc5d0 \uc2a4\ud398\uc15c \uacf5\uc5f0\uc744 \uac16\ub294 \ub354\ube14\ucf00\uc774\uc640 \ud568\uaed8 8\uc6d4\uc758 \ud799\ud569\ud50c\ub808\uc774\uc57c \uc1fc\uc5d0 \ud5e4\ub4dc\ub77c\uc774\ub108\ub85c \ub098\uc11c \ubbf8\ubc1c\ud45c \uc2e0\uace1\ub4e4\uc744 \ubd80\ub974\uba70 \uacf5\uc2dd\uc801\uc73c\ub85c \ud799\ud569 \ubb34\ub300\uc5d0 \ubcf5\uadc0\ud588\ub2e4. \uadf8\ub7ec\ub098 \uc568\ubc94\uc774 \ubc1c\ud45c\ub418\uc9c0 \uc54a\uace0 \uc788\uc790 \uae30\ub300\ud558\ub358 \ud799\ud569\ud32c\ub4e4\uc740 \uc544\uc26c\uc6cc\ud588\ub2e4. \uadf8\ub7ec\ub358 \ub3c4\uc911 10\uc6d4 15\uc77c\uc5d0 \uac00\ub9ac\uc628\uc740 \ud2b8\uc704\ud130\ub97c \ud1b5\ud574 \uac00\ub9ac\uc628 2\uc9d1\uc758 \ubc1c\ub9e4\uc77c\uc744 10\uc6d4 26\uc77c\ub85c \ubc1d\ud614\uace0, \uc18c\uc18d\uc0ac \ud0c0\uc77c \ubba4\uc9c1\uc740 10\uc6d4 20\uc77c\uc5d0 \uc815\uc2dd \ubcf4\ub3c4\uc790\ub8cc\ub97c \ubc30\ud3ec\ud558\uba70 \ubc1c\ub9e4 \ub0a0\uc9dc\ub97c \ud655\uc815\uc9c0\uc5c8\ub2e4.", "paragraph_answer": "2010\ub144 8\uc6d4 16\uc77c~17\uc77c\uc5d0\ub294 EBS \uacf5\uac10 \ubb34\ub300 \uc704\uc5d0\uc11c \ub808\uac8c \ubc34\ub4dc \uc18c\uc6b8 \uc2a4\ud14c\ub514 \ub77d\ucee4\uc988 \uc640 \ud638\ud761\uc744 \ub9de\ucdb0 \ubc1c\ud45c\uace1\uc778 \u3008\uc57d\uc18d\uc758 \uc7a5\uc18c\u3009\ub4f1\uacfc 2\uc9d1 \uc218\ub85d \uc608\uc815\uace1\uc778 \u3008\uc18c\ub9ac\ub97c \ub354 \ud06c\uac8c\u3009, \u3008\uc0dd\uba85\uc218\u3009\ub4f1\uc744 \ubd88\ub800\ub2e4. \uc774 \uacf5\uc5f0\uc740 10\uc6d4 8\uc77c\uc5d0 \ubc29\uc1a1\ub418\uc5c8\ub2e4. 8\uc6d4 19\uc77c\uc5d0\ub294 \uc7ac\uc988 \ubba4\uc9c0\uc158 \ud33b \uc874\uc758 \ub0b4\ud55c \uacf5\uc5f0\uc5d0 \ucc38\uc5ec\ud588\uace0, 28\uc77c\uc5d0\ub294 29\uc77c\uc5d0 \uc2a4\ud398\uc15c \uacf5\uc5f0\uc744 \uac16\ub294 \ub354\ube14\ucf00\uc774\uc640 \ud568\uaed8 8\uc6d4\uc758 \ud799\ud569\ud50c\ub808\uc774\uc57c \uc1fc\uc5d0 \ud5e4\ub4dc\ub77c\uc774\ub108\ub85c \ub098\uc11c \ubbf8\ubc1c\ud45c \uc2e0\uace1\ub4e4\uc744 \ubd80\ub974\uba70 \uacf5\uc2dd\uc801\uc73c\ub85c \ud799\ud569 \ubb34\ub300\uc5d0 \ubcf5\uadc0\ud588\ub2e4. \uadf8\ub7ec\ub098 \uc568\ubc94\uc774 \ubc1c\ud45c\ub418\uc9c0 \uc54a\uace0 \uc788\uc790 \uae30\ub300\ud558\ub358 \ud799\ud569\ud32c\ub4e4\uc740 \uc544\uc26c\uc6cc\ud588\ub2e4. \uadf8\ub7ec\ub358 \ub3c4\uc911 10\uc6d4 15\uc77c\uc5d0 \uac00\ub9ac\uc628\uc740 \ud2b8\uc704\ud130\ub97c \ud1b5\ud574 \uac00\ub9ac\uc628 2\uc9d1\uc758 \ubc1c\ub9e4\uc77c\uc744 10\uc6d4 26\uc77c\ub85c \ubc1d\ud614\uace0, \uc18c\uc18d\uc0ac \ud0c0\uc77c \ubba4\uc9c1\uc740 10\uc6d4 20\uc77c\uc5d0 \uc815\uc2dd \ubcf4\ub3c4\uc790\ub8cc\ub97c \ubc30\ud3ec\ud558\uba70 \ubc1c\ub9e4 \ub0a0\uc9dc\ub97c \ud655\uc815\uc9c0\uc5c8\ub2e4.", "sentence_answer": "2010\ub144 8\uc6d4 16\uc77c~17\uc77c\uc5d0\ub294 EBS \uacf5\uac10 \ubb34\ub300 \uc704\uc5d0\uc11c \ub808\uac8c \ubc34\ub4dc \uc18c\uc6b8 \uc2a4\ud14c\ub514 \ub77d\ucee4\uc988 \uc640 \ud638\ud761\uc744 \ub9de\ucdb0 \ubc1c\ud45c\uace1\uc778 \u3008\uc57d\uc18d\uc758 \uc7a5\uc18c\u3009\ub4f1\uacfc 2\uc9d1 \uc218\ub85d \uc608\uc815\uace1\uc778 \u3008\uc18c\ub9ac\ub97c \ub354 \ud06c\uac8c\u3009, \u3008\uc0dd\uba85\uc218\u3009\ub4f1\uc744 \ubd88\ub800\ub2e4.", "paragraph_id": "6554403-2-0", "paragraph_question": "question: 2010\ub144 8\uc6d416~17\uc77c\uc5d0 EBS \uacf5\uac10 \ubb34\ub300 \uc704\uc5d0\uc11c \uac00\ub9ac\uc628\uacfc \ud568\uaed8 \ud638\ud761\uc744 \ub9de\ucd98 \ub808\uac8c \ubc34\ub4dc\ub294?, context: 2010\ub144 8\uc6d4 16\uc77c~17\uc77c\uc5d0\ub294 EBS \uacf5\uac10 \ubb34\ub300 \uc704\uc5d0\uc11c \ub808\uac8c \ubc34\ub4dc \uc18c\uc6b8 \uc2a4\ud14c\ub514 \ub77d\ucee4\uc988\uc640 \ud638\ud761\uc744 \ub9de\ucdb0 \ubc1c\ud45c\uace1\uc778 \u3008\uc57d\uc18d\uc758 \uc7a5\uc18c\u3009\ub4f1\uacfc 2\uc9d1 \uc218\ub85d \uc608\uc815\uace1\uc778 \u3008\uc18c\ub9ac\ub97c \ub354 \ud06c\uac8c\u3009, \u3008\uc0dd\uba85\uc218\u3009\ub4f1\uc744 \ubd88\ub800\ub2e4. \uc774 \uacf5\uc5f0\uc740 10\uc6d4 8\uc77c\uc5d0 \ubc29\uc1a1\ub418\uc5c8\ub2e4. 8\uc6d4 19\uc77c\uc5d0\ub294 \uc7ac\uc988 \ubba4\uc9c0\uc158 \ud33b \uc874\uc758 \ub0b4\ud55c \uacf5\uc5f0\uc5d0 \ucc38\uc5ec\ud588\uace0, 28\uc77c\uc5d0\ub294 29\uc77c\uc5d0 \uc2a4\ud398\uc15c \uacf5\uc5f0\uc744 \uac16\ub294 \ub354\ube14\ucf00\uc774\uc640 \ud568\uaed8 8\uc6d4\uc758 \ud799\ud569\ud50c\ub808\uc774\uc57c \uc1fc\uc5d0 \ud5e4\ub4dc\ub77c\uc774\ub108\ub85c \ub098\uc11c \ubbf8\ubc1c\ud45c \uc2e0\uace1\ub4e4\uc744 \ubd80\ub974\uba70 \uacf5\uc2dd\uc801\uc73c\ub85c \ud799\ud569 \ubb34\ub300\uc5d0 \ubcf5\uadc0\ud588\ub2e4. \uadf8\ub7ec\ub098 \uc568\ubc94\uc774 \ubc1c\ud45c\ub418\uc9c0 \uc54a\uace0 \uc788\uc790 \uae30\ub300\ud558\ub358 \ud799\ud569\ud32c\ub4e4\uc740 \uc544\uc26c\uc6cc\ud588\ub2e4. \uadf8\ub7ec\ub358 \ub3c4\uc911 10\uc6d4 15\uc77c\uc5d0 \uac00\ub9ac\uc628\uc740 \ud2b8\uc704\ud130\ub97c \ud1b5\ud574 \uac00\ub9ac\uc628 2\uc9d1\uc758 \ubc1c\ub9e4\uc77c\uc744 10\uc6d4 26\uc77c\ub85c \ubc1d\ud614\uace0, \uc18c\uc18d\uc0ac \ud0c0\uc77c \ubba4\uc9c1\uc740 10\uc6d4 20\uc77c\uc5d0 \uc815\uc2dd \ubcf4\ub3c4\uc790\ub8cc\ub97c \ubc30\ud3ec\ud558\uba70 \ubc1c\ub9e4 \ub0a0\uc9dc\ub97c \ud655\uc815\uc9c0\uc5c8\ub2e4."} {"question": "\uc790\uc720\uc8fc\uc758\uc801 \uac1c\uc778\uc8fc\uc758\uc640 \ubbfc\uc8fc\uc8fc\uc758\uc801 \uc9d1\ub2e8\uc8fc\uc758\uc758 \ud0c0\ud611\uc73c\ub85c \ubcfc \uc218 \uc788\ub294 \uac83\uc740?", "paragraph": "\uc790\uc720\ubbfc\uc8fc\uc8fc\uc758\uac00 \uacc4\ubabd\uc8fc\uc758 \uc2dc\ub300\uc758 \uc790\uc720\uc8fc\uc758\uc790\ub4e4\ub85c\ubd80\ud130 \ube44\ub86f\ub418\uc5c8\uc9c0\ub9cc, \uc790\uc720\uc8fc\uc758\uc640 \ubbfc\uc8fc\uc8fc\uc758\uac04\uc758 \uad00\uacc4\uc5d0 \ub300\ud574\uc11c\ub294 \uacc4\uc18d \ub17c\uc7c1\uc774 \uc788\uc5b4\uc654\ub2e4. \uc790\uc720\uc8fc\uc758\uc758 \uc774\ub150\uc740 (\ud2b9\ud788 \uace0\uc804\uc8fc\uc758\uc801 \uc790\uc720\uc8fc\uc758\uc5d0\uc11c) \uace0\ub3c4\ub85c \uac1c\uc778\uc8fc\uc758\uc801\uc774\uace0 \uac1c\uc778\uacfc \uc815\ubd80\uc640\uc758 \uad00\uacc4\uc5d0\uc11c \uc815\ubd80\uc758 \ud798\uc744 \uc81c\ud55c\ud558\ub294 \uac83\uc5d0 \ucd08\uc810\uc744 \ub454\ub2e4. \ubc18\ub300\ub85c \ubbfc\uc8fc\uc8fc\uc758\ub294 \ub2e4\uc218\uc5d0\uac8c \uad8c\ud55c\uc744 \ubd80\uc5ec\ud558\ub294 \uc6d0\ub9ac\ub85c\uc11c \uc9d1\ub2e8\uc8fc\uc758\uc801\uc778 \uc131\uaca9\uc744 \uac00\uc9c0\uace0 \uc788\ub2e4\ub294 \ud3c9\uac00\ub97c \ubc1b\uae30\ub3c4 \ud55c\ub2e4(\uac1c\uc778\uc8fc\uc758\uc801 \uad00\uc810\uc5d0\uc11c \ubbfc\uc8fc\uc8fc\uc758\ub97c \uc815\uc758\ud558\ub294 \uacac\ud574\ub3c4 \uc0c1\ub2f9\ud568\uc5d0 \uc720\uc758\ud560 \uac83). \ub530\ub77c\uc11c \uc790\uc720\ubbfc\uc8fc\uc8fc\uc758\ub294 \uc790\uc720\uc8fc\uc758\uc801 \uac1c\uc778\uc8fc\uc758\uc640 \ubbfc\uc8fc\uc8fc\uc758\uc801 \uc9d1\ub2e8\uc8fc\uc758\uc758 \ud0c0\ud611\uc73c\ub85c\ub3c4 \ubcfc \uc218 \uc788\ub2e4. \uc774\ub7ec\ud55c \uacac\ud574\uc758 \uc9c0\uc9c0\uc790\ub4e4\uc740 \uc790\uc720\ub86d\uc9c0 \uc54a\uc740 \ubbfc\uc8fc\uc8fc\uc758(illiberal democracy)\uc640 \uc790\uc720\ub97c \ubcf4\uc7a5\ud558\ub294 \ub3c5\uc7ac\uc815(liberal dictatorship)\uc758 \uc874\uc7ac\ub97c \uadfc\uac70\ub85c \uc785\ud5cc\uc801 \uc790\uc720\uc8fc\uc758\uc640 \ubbfc\uc8fc\uc801\uc778 \uc815\ubd80\uac00 \ubc18\ub4dc\uc2dc \ud544\uc5f0\uc801\uc778 \uad00\uacc4\ub294 \uc544\ub2c8\ub77c\uace0 \ub9d0\ud55c\ub2e4. \uc774\uc5d0 \ubc18\ud574, \uc591\uc790\ub294 \uc815\uce58\uc801\uc778 \ud3c9\ub4f1\uc758 \uac1c\ub150\uc5d0 \uae30\ucd08\ud558\ubbc0\ub85c \uc785\ud5cc\uc801 \uc790\uc720\uc8fc\uc758\uc640 \ubbfc\uc8fc\uc801\uc778 \uc815\ubd80 \uac04\uc5d0\ub294 \ubaa8\uc21c\uc774 \uc5c6\uc744 \ubfd0\ub9cc \uc544\ub2c8\ub77c \uc11c\ub85c\uc758 \uc9c4\uc815\ud55c \uc874\ub9bd\uc744 \uc704\ud574 \ud544\uc694\ud558\ub2e4\ub294 \uacac\ud574\ub3c4 \uc788\ub2e4.", "answer": "\uc790\uc720\ubbfc\uc8fc\uc8fc\uc758", "sentence": "\uc790\uc720\ubbfc\uc8fc\uc8fc\uc758 \uac00 \uacc4\ubabd\uc8fc\uc758 \uc2dc\ub300\uc758 \uc790\uc720\uc8fc\uc758\uc790\ub4e4\ub85c\ubd80\ud130 \ube44\ub86f\ub418\uc5c8\uc9c0\ub9cc, \uc790\uc720\uc8fc\uc758\uc640 \ubbfc\uc8fc\uc8fc\uc758\uac04\uc758 \uad00\uacc4\uc5d0 \ub300\ud574\uc11c\ub294 \uacc4\uc18d \ub17c\uc7c1\uc774 \uc788\uc5b4\uc654\ub2e4.", "paragraph_sentence": " \uc790\uc720\ubbfc\uc8fc\uc8fc\uc758 \uac00 \uacc4\ubabd\uc8fc\uc758 \uc2dc\ub300\uc758 \uc790\uc720\uc8fc\uc758\uc790\ub4e4\ub85c\ubd80\ud130 \ube44\ub86f\ub418\uc5c8\uc9c0\ub9cc, \uc790\uc720\uc8fc\uc758\uc640 \ubbfc\uc8fc\uc8fc\uc758\uac04\uc758 \uad00\uacc4\uc5d0 \ub300\ud574\uc11c\ub294 \uacc4\uc18d \ub17c\uc7c1\uc774 \uc788\uc5b4\uc654\ub2e4. \uc790\uc720\uc8fc\uc758\uc758 \uc774\ub150\uc740 (\ud2b9\ud788 \uace0\uc804\uc8fc\uc758\uc801 \uc790\uc720\uc8fc\uc758\uc5d0\uc11c) \uace0\ub3c4\ub85c \uac1c\uc778\uc8fc\uc758\uc801\uc774\uace0 \uac1c\uc778\uacfc \uc815\ubd80\uc640\uc758 \uad00\uacc4\uc5d0\uc11c \uc815\ubd80\uc758 \ud798\uc744 \uc81c\ud55c\ud558\ub294 \uac83\uc5d0 \ucd08\uc810\uc744 \ub454\ub2e4. \ubc18\ub300\ub85c \ubbfc\uc8fc\uc8fc\uc758\ub294 \ub2e4\uc218\uc5d0\uac8c \uad8c\ud55c\uc744 \ubd80\uc5ec\ud558\ub294 \uc6d0\ub9ac\ub85c\uc11c \uc9d1\ub2e8\uc8fc\uc758\uc801\uc778 \uc131\uaca9\uc744 \uac00\uc9c0\uace0 \uc788\ub2e4\ub294 \ud3c9\uac00\ub97c \ubc1b\uae30\ub3c4 \ud55c\ub2e4(\uac1c\uc778\uc8fc\uc758\uc801 \uad00\uc810\uc5d0\uc11c \ubbfc\uc8fc\uc8fc\uc758\ub97c \uc815\uc758\ud558\ub294 \uacac\ud574\ub3c4 \uc0c1\ub2f9\ud568\uc5d0 \uc720\uc758\ud560 \uac83). \ub530\ub77c\uc11c \uc790\uc720\ubbfc\uc8fc\uc8fc\uc758\ub294 \uc790\uc720\uc8fc\uc758\uc801 \uac1c\uc778\uc8fc\uc758\uc640 \ubbfc\uc8fc\uc8fc\uc758\uc801 \uc9d1\ub2e8\uc8fc\uc758\uc758 \ud0c0\ud611\uc73c\ub85c\ub3c4 \ubcfc \uc218 \uc788\ub2e4. \uc774\ub7ec\ud55c \uacac\ud574\uc758 \uc9c0\uc9c0\uc790\ub4e4\uc740 \uc790\uc720\ub86d\uc9c0 \uc54a\uc740 \ubbfc\uc8fc\uc8fc\uc758(illiberal democracy)\uc640 \uc790\uc720\ub97c \ubcf4\uc7a5\ud558\ub294 \ub3c5\uc7ac\uc815(liberal dictatorship)\uc758 \uc874\uc7ac\ub97c \uadfc\uac70\ub85c \uc785\ud5cc\uc801 \uc790\uc720\uc8fc\uc758\uc640 \ubbfc\uc8fc\uc801\uc778 \uc815\ubd80\uac00 \ubc18\ub4dc\uc2dc \ud544\uc5f0\uc801\uc778 \uad00\uacc4\ub294 \uc544\ub2c8\ub77c\uace0 \ub9d0\ud55c\ub2e4. \uc774\uc5d0 \ubc18\ud574, \uc591\uc790\ub294 \uc815\uce58\uc801\uc778 \ud3c9\ub4f1\uc758 \uac1c\ub150\uc5d0 \uae30\ucd08\ud558\ubbc0\ub85c \uc785\ud5cc\uc801 \uc790\uc720\uc8fc\uc758\uc640 \ubbfc\uc8fc\uc801\uc778 \uc815\ubd80 \uac04\uc5d0\ub294 \ubaa8\uc21c\uc774 \uc5c6\uc744 \ubfd0\ub9cc \uc544\ub2c8\ub77c \uc11c\ub85c\uc758 \uc9c4\uc815\ud55c \uc874\ub9bd\uc744 \uc704\ud574 \ud544\uc694\ud558\ub2e4\ub294 \uacac\ud574\ub3c4 \uc788\ub2e4.", "paragraph_answer": " \uc790\uc720\ubbfc\uc8fc\uc8fc\uc758 \uac00 \uacc4\ubabd\uc8fc\uc758 \uc2dc\ub300\uc758 \uc790\uc720\uc8fc\uc758\uc790\ub4e4\ub85c\ubd80\ud130 \ube44\ub86f\ub418\uc5c8\uc9c0\ub9cc, \uc790\uc720\uc8fc\uc758\uc640 \ubbfc\uc8fc\uc8fc\uc758\uac04\uc758 \uad00\uacc4\uc5d0 \ub300\ud574\uc11c\ub294 \uacc4\uc18d \ub17c\uc7c1\uc774 \uc788\uc5b4\uc654\ub2e4. \uc790\uc720\uc8fc\uc758\uc758 \uc774\ub150\uc740 (\ud2b9\ud788 \uace0\uc804\uc8fc\uc758\uc801 \uc790\uc720\uc8fc\uc758\uc5d0\uc11c) \uace0\ub3c4\ub85c \uac1c\uc778\uc8fc\uc758\uc801\uc774\uace0 \uac1c\uc778\uacfc \uc815\ubd80\uc640\uc758 \uad00\uacc4\uc5d0\uc11c \uc815\ubd80\uc758 \ud798\uc744 \uc81c\ud55c\ud558\ub294 \uac83\uc5d0 \ucd08\uc810\uc744 \ub454\ub2e4. \ubc18\ub300\ub85c \ubbfc\uc8fc\uc8fc\uc758\ub294 \ub2e4\uc218\uc5d0\uac8c \uad8c\ud55c\uc744 \ubd80\uc5ec\ud558\ub294 \uc6d0\ub9ac\ub85c\uc11c \uc9d1\ub2e8\uc8fc\uc758\uc801\uc778 \uc131\uaca9\uc744 \uac00\uc9c0\uace0 \uc788\ub2e4\ub294 \ud3c9\uac00\ub97c \ubc1b\uae30\ub3c4 \ud55c\ub2e4(\uac1c\uc778\uc8fc\uc758\uc801 \uad00\uc810\uc5d0\uc11c \ubbfc\uc8fc\uc8fc\uc758\ub97c \uc815\uc758\ud558\ub294 \uacac\ud574\ub3c4 \uc0c1\ub2f9\ud568\uc5d0 \uc720\uc758\ud560 \uac83). \ub530\ub77c\uc11c \uc790\uc720\ubbfc\uc8fc\uc8fc\uc758\ub294 \uc790\uc720\uc8fc\uc758\uc801 \uac1c\uc778\uc8fc\uc758\uc640 \ubbfc\uc8fc\uc8fc\uc758\uc801 \uc9d1\ub2e8\uc8fc\uc758\uc758 \ud0c0\ud611\uc73c\ub85c\ub3c4 \ubcfc \uc218 \uc788\ub2e4. \uc774\ub7ec\ud55c \uacac\ud574\uc758 \uc9c0\uc9c0\uc790\ub4e4\uc740 \uc790\uc720\ub86d\uc9c0 \uc54a\uc740 \ubbfc\uc8fc\uc8fc\uc758(illiberal democracy)\uc640 \uc790\uc720\ub97c \ubcf4\uc7a5\ud558\ub294 \ub3c5\uc7ac\uc815(liberal dictatorship)\uc758 \uc874\uc7ac\ub97c \uadfc\uac70\ub85c \uc785\ud5cc\uc801 \uc790\uc720\uc8fc\uc758\uc640 \ubbfc\uc8fc\uc801\uc778 \uc815\ubd80\uac00 \ubc18\ub4dc\uc2dc \ud544\uc5f0\uc801\uc778 \uad00\uacc4\ub294 \uc544\ub2c8\ub77c\uace0 \ub9d0\ud55c\ub2e4. \uc774\uc5d0 \ubc18\ud574, \uc591\uc790\ub294 \uc815\uce58\uc801\uc778 \ud3c9\ub4f1\uc758 \uac1c\ub150\uc5d0 \uae30\ucd08\ud558\ubbc0\ub85c \uc785\ud5cc\uc801 \uc790\uc720\uc8fc\uc758\uc640 \ubbfc\uc8fc\uc801\uc778 \uc815\ubd80 \uac04\uc5d0\ub294 \ubaa8\uc21c\uc774 \uc5c6\uc744 \ubfd0\ub9cc \uc544\ub2c8\ub77c \uc11c\ub85c\uc758 \uc9c4\uc815\ud55c \uc874\ub9bd\uc744 \uc704\ud574 \ud544\uc694\ud558\ub2e4\ub294 \uacac\ud574\ub3c4 \uc788\ub2e4.", "sentence_answer": " \uc790\uc720\ubbfc\uc8fc\uc8fc\uc758 \uac00 \uacc4\ubabd\uc8fc\uc758 \uc2dc\ub300\uc758 \uc790\uc720\uc8fc\uc758\uc790\ub4e4\ub85c\ubd80\ud130 \ube44\ub86f\ub418\uc5c8\uc9c0\ub9cc, \uc790\uc720\uc8fc\uc758\uc640 \ubbfc\uc8fc\uc8fc\uc758\uac04\uc758 \uad00\uacc4\uc5d0 \ub300\ud574\uc11c\ub294 \uacc4\uc18d \ub17c\uc7c1\uc774 \uc788\uc5b4\uc654\ub2e4.", "paragraph_id": "6548896-1-1", "paragraph_question": "question: \uc790\uc720\uc8fc\uc758\uc801 \uac1c\uc778\uc8fc\uc758\uc640 \ubbfc\uc8fc\uc8fc\uc758\uc801 \uc9d1\ub2e8\uc8fc\uc758\uc758 \ud0c0\ud611\uc73c\ub85c \ubcfc \uc218 \uc788\ub294 \uac83\uc740?, context: \uc790\uc720\ubbfc\uc8fc\uc8fc\uc758\uac00 \uacc4\ubabd\uc8fc\uc758 \uc2dc\ub300\uc758 \uc790\uc720\uc8fc\uc758\uc790\ub4e4\ub85c\ubd80\ud130 \ube44\ub86f\ub418\uc5c8\uc9c0\ub9cc, \uc790\uc720\uc8fc\uc758\uc640 \ubbfc\uc8fc\uc8fc\uc758\uac04\uc758 \uad00\uacc4\uc5d0 \ub300\ud574\uc11c\ub294 \uacc4\uc18d \ub17c\uc7c1\uc774 \uc788\uc5b4\uc654\ub2e4. \uc790\uc720\uc8fc\uc758\uc758 \uc774\ub150\uc740 (\ud2b9\ud788 \uace0\uc804\uc8fc\uc758\uc801 \uc790\uc720\uc8fc\uc758\uc5d0\uc11c) \uace0\ub3c4\ub85c \uac1c\uc778\uc8fc\uc758\uc801\uc774\uace0 \uac1c\uc778\uacfc \uc815\ubd80\uc640\uc758 \uad00\uacc4\uc5d0\uc11c \uc815\ubd80\uc758 \ud798\uc744 \uc81c\ud55c\ud558\ub294 \uac83\uc5d0 \ucd08\uc810\uc744 \ub454\ub2e4. \ubc18\ub300\ub85c \ubbfc\uc8fc\uc8fc\uc758\ub294 \ub2e4\uc218\uc5d0\uac8c \uad8c\ud55c\uc744 \ubd80\uc5ec\ud558\ub294 \uc6d0\ub9ac\ub85c\uc11c \uc9d1\ub2e8\uc8fc\uc758\uc801\uc778 \uc131\uaca9\uc744 \uac00\uc9c0\uace0 \uc788\ub2e4\ub294 \ud3c9\uac00\ub97c \ubc1b\uae30\ub3c4 \ud55c\ub2e4(\uac1c\uc778\uc8fc\uc758\uc801 \uad00\uc810\uc5d0\uc11c \ubbfc\uc8fc\uc8fc\uc758\ub97c \uc815\uc758\ud558\ub294 \uacac\ud574\ub3c4 \uc0c1\ub2f9\ud568\uc5d0 \uc720\uc758\ud560 \uac83). \ub530\ub77c\uc11c \uc790\uc720\ubbfc\uc8fc\uc8fc\uc758\ub294 \uc790\uc720\uc8fc\uc758\uc801 \uac1c\uc778\uc8fc\uc758\uc640 \ubbfc\uc8fc\uc8fc\uc758\uc801 \uc9d1\ub2e8\uc8fc\uc758\uc758 \ud0c0\ud611\uc73c\ub85c\ub3c4 \ubcfc \uc218 \uc788\ub2e4. \uc774\ub7ec\ud55c \uacac\ud574\uc758 \uc9c0\uc9c0\uc790\ub4e4\uc740 \uc790\uc720\ub86d\uc9c0 \uc54a\uc740 \ubbfc\uc8fc\uc8fc\uc758(illiberal democracy)\uc640 \uc790\uc720\ub97c \ubcf4\uc7a5\ud558\ub294 \ub3c5\uc7ac\uc815(liberal dictatorship)\uc758 \uc874\uc7ac\ub97c \uadfc\uac70\ub85c \uc785\ud5cc\uc801 \uc790\uc720\uc8fc\uc758\uc640 \ubbfc\uc8fc\uc801\uc778 \uc815\ubd80\uac00 \ubc18\ub4dc\uc2dc \ud544\uc5f0\uc801\uc778 \uad00\uacc4\ub294 \uc544\ub2c8\ub77c\uace0 \ub9d0\ud55c\ub2e4. \uc774\uc5d0 \ubc18\ud574, \uc591\uc790\ub294 \uc815\uce58\uc801\uc778 \ud3c9\ub4f1\uc758 \uac1c\ub150\uc5d0 \uae30\ucd08\ud558\ubbc0\ub85c \uc785\ud5cc\uc801 \uc790\uc720\uc8fc\uc758\uc640 \ubbfc\uc8fc\uc801\uc778 \uc815\ubd80 \uac04\uc5d0\ub294 \ubaa8\uc21c\uc774 \uc5c6\uc744 \ubfd0\ub9cc \uc544\ub2c8\ub77c \uc11c\ub85c\uc758 \uc9c4\uc815\ud55c \uc874\ub9bd\uc744 \uc704\ud574 \ud544\uc694\ud558\ub2e4\ub294 \uacac\ud574\ub3c4 \uc788\ub2e4."} {"question": "\ub178\uc544\uc758 \uc7a5\ub0a8 \uc148\uc740 \uc5b4\ub290 \ub300\ub959\uc758 \uc120\uc870\uc778\uac00?", "paragraph": "\uc911\uc138 \uae30\ub3c5\uad50\uc5d0\uc11c\ub294 \ub178\uc544\uc758 \uc138 \uc544\ub4e4\uc774 \uc138 \ub300\ub959\uc758 \uc0ac\ub78c\ub4e4\uc758 \uc120\uc870\ub85c \uc5ec\uaca8\uc84c\ub2e4. \uc7a5\ub0a8\uc778 \uc148\uc740 \uc544\uc2dc\uc544\uc778, \ucc28\ub0a8\uc778 \ud568\uc740 \uc544\ud504\ub9ac\uce74\uc778, \uc0bc\ub0a8\uc778 \uc57c\ubcb3\uc740 \uc720\ub7fd\uc778\uc758 \uc120\uc870\uc778 \uac83\uc73c\ub85c \ud574\uc11d\ub418\uc5c8\ub2e4. \uc774\uc640 \ub2e4\ub978 \uc18c\uc218\uc758 \uacac\ud574\ub85c, \ub178\uc544\uc758 \uc138 \uc544\ub4e4\uc744 \uac01\uac01 \uc131\uc9c1\uc790(\uc148), \ub18d\ubbfc(\ud568), \uc804\uc0ac(\uc57c\ubcb3)\ub85c \ud574\uc11d\ud558\ub294 \uacac\ud574\uac00 \uc788\uc5c8\ub2e4. \uc138 \uc544\ub4e4\uc744 \uac01\uac01\uc758 \uc778\uc885\uc73c\ub85c \ubd84\ub958\ud558\ub294 \ubc29\uc2dd\uc744 \ub450\uace0\uc11c \ud604\ub300\uc5d0 \uc640\uc11c\ub294 \uc778\uc885\ucc28\ubcc4\uc801 \ud574\uc11d\uc774\ub77c\uace0 \ube44\ud310\ubc1b\uae30\ub294 \ud558\uc9c0\ub9cc, \uc774 \ud574\uc11d\uc740 \uc778\ub958\uc758 \uae30\uc6d0\uacfc \ud37c\uc838\ub098\uac04 \ubc29\uc2dd\uc744 \uc5f0\uad6c\ud558\ub294 \ud1b5\uc2dc\uc801\uc778 \ud574\uc11d\ubc29\ubc95\uc77c\ubfd0 \uc778\uc885\ucc28\ubcc4\uc801 \ud574\uc11d\uc774\ub77c\ub294 \ube44\ud310\uc740 \uc633\ubc14\ub974\uc9c0 \uc54a\ub2e4. \uc138 \uc544\ub4e4\uc758 \uc778\uc885\uc801 \ubd84\ub958\ud574\uc11d\ubc29\uc2dd\uc740 \uc8fc\ub85c \uae30\ub3c5\uad50\uadfc\ubcf8\uc8fc\uc758, \ubcf5\uc74c\uc8fc\uc758, \uc131\uacbd\uc2e0\uc790 \uc9c4\uc601\uc5d0\uc11c \uc624\ub7ab\ub3d9\uc548 \ud574\uc11d\ub418\uace0 \uc804\ud574\uc838\uc628 \uc804\ud1b5\uc801\uc778 \ud574\uc11d\ubc29\ubc95\uc774\ub2e4.", "answer": "\uc544\uc2dc\uc544", "sentence": "\uc7a5\ub0a8\uc778 \uc148\uc740 \uc544\uc2dc\uc544 \uc778, \ucc28\ub0a8\uc778 \ud568\uc740 \uc544\ud504\ub9ac\uce74\uc778, \uc0bc\ub0a8\uc778 \uc57c\ubcb3\uc740 \uc720\ub7fd\uc778\uc758 \uc120\uc870\uc778 \uac83\uc73c\ub85c \ud574\uc11d\ub418\uc5c8\ub2e4.", "paragraph_sentence": "\uc911\uc138 \uae30\ub3c5\uad50\uc5d0\uc11c\ub294 \ub178\uc544\uc758 \uc138 \uc544\ub4e4\uc774 \uc138 \ub300\ub959\uc758 \uc0ac\ub78c\ub4e4\uc758 \uc120\uc870\ub85c \uc5ec\uaca8\uc84c\ub2e4. \uc7a5\ub0a8\uc778 \uc148\uc740 \uc544\uc2dc\uc544 \uc778, \ucc28\ub0a8\uc778 \ud568\uc740 \uc544\ud504\ub9ac\uce74\uc778, \uc0bc\ub0a8\uc778 \uc57c\ubcb3\uc740 \uc720\ub7fd\uc778\uc758 \uc120\uc870\uc778 \uac83\uc73c\ub85c \ud574\uc11d\ub418\uc5c8\ub2e4. \uc774\uc640 \ub2e4\ub978 \uc18c\uc218\uc758 \uacac\ud574\ub85c, \ub178\uc544\uc758 \uc138 \uc544\ub4e4\uc744 \uac01\uac01 \uc131\uc9c1\uc790(\uc148), \ub18d\ubbfc(\ud568), \uc804\uc0ac(\uc57c\ubcb3)\ub85c \ud574\uc11d\ud558\ub294 \uacac\ud574\uac00 \uc788\uc5c8\ub2e4. \uc138 \uc544\ub4e4\uc744 \uac01\uac01\uc758 \uc778\uc885\uc73c\ub85c \ubd84\ub958\ud558\ub294 \ubc29\uc2dd\uc744 \ub450\uace0\uc11c \ud604\ub300\uc5d0 \uc640\uc11c\ub294 \uc778\uc885\ucc28\ubcc4\uc801 \ud574\uc11d\uc774\ub77c\uace0 \ube44\ud310\ubc1b\uae30\ub294 \ud558\uc9c0\ub9cc, \uc774 \ud574\uc11d\uc740 \uc778\ub958\uc758 \uae30\uc6d0\uacfc \ud37c\uc838\ub098\uac04 \ubc29\uc2dd\uc744 \uc5f0\uad6c\ud558\ub294 \ud1b5\uc2dc\uc801\uc778 \ud574\uc11d\ubc29\ubc95\uc77c\ubfd0 \uc778\uc885\ucc28\ubcc4\uc801 \ud574\uc11d\uc774\ub77c\ub294 \ube44\ud310\uc740 \uc633\ubc14\ub974\uc9c0 \uc54a\ub2e4. \uc138 \uc544\ub4e4\uc758 \uc778\uc885\uc801 \ubd84\ub958\ud574\uc11d\ubc29\uc2dd\uc740 \uc8fc\ub85c \uae30\ub3c5\uad50\uadfc\ubcf8\uc8fc\uc758, \ubcf5\uc74c\uc8fc\uc758, \uc131\uacbd\uc2e0\uc790 \uc9c4\uc601\uc5d0\uc11c \uc624\ub7ab\ub3d9\uc548 \ud574\uc11d\ub418\uace0 \uc804\ud574\uc838\uc628 \uc804\ud1b5\uc801\uc778 \ud574\uc11d\ubc29\ubc95\uc774\ub2e4.", "paragraph_answer": "\uc911\uc138 \uae30\ub3c5\uad50\uc5d0\uc11c\ub294 \ub178\uc544\uc758 \uc138 \uc544\ub4e4\uc774 \uc138 \ub300\ub959\uc758 \uc0ac\ub78c\ub4e4\uc758 \uc120\uc870\ub85c \uc5ec\uaca8\uc84c\ub2e4. \uc7a5\ub0a8\uc778 \uc148\uc740 \uc544\uc2dc\uc544 \uc778, \ucc28\ub0a8\uc778 \ud568\uc740 \uc544\ud504\ub9ac\uce74\uc778, \uc0bc\ub0a8\uc778 \uc57c\ubcb3\uc740 \uc720\ub7fd\uc778\uc758 \uc120\uc870\uc778 \uac83\uc73c\ub85c \ud574\uc11d\ub418\uc5c8\ub2e4. \uc774\uc640 \ub2e4\ub978 \uc18c\uc218\uc758 \uacac\ud574\ub85c, \ub178\uc544\uc758 \uc138 \uc544\ub4e4\uc744 \uac01\uac01 \uc131\uc9c1\uc790(\uc148), \ub18d\ubbfc(\ud568), \uc804\uc0ac(\uc57c\ubcb3)\ub85c \ud574\uc11d\ud558\ub294 \uacac\ud574\uac00 \uc788\uc5c8\ub2e4. \uc138 \uc544\ub4e4\uc744 \uac01\uac01\uc758 \uc778\uc885\uc73c\ub85c \ubd84\ub958\ud558\ub294 \ubc29\uc2dd\uc744 \ub450\uace0\uc11c \ud604\ub300\uc5d0 \uc640\uc11c\ub294 \uc778\uc885\ucc28\ubcc4\uc801 \ud574\uc11d\uc774\ub77c\uace0 \ube44\ud310\ubc1b\uae30\ub294 \ud558\uc9c0\ub9cc, \uc774 \ud574\uc11d\uc740 \uc778\ub958\uc758 \uae30\uc6d0\uacfc \ud37c\uc838\ub098\uac04 \ubc29\uc2dd\uc744 \uc5f0\uad6c\ud558\ub294 \ud1b5\uc2dc\uc801\uc778 \ud574\uc11d\ubc29\ubc95\uc77c\ubfd0 \uc778\uc885\ucc28\ubcc4\uc801 \ud574\uc11d\uc774\ub77c\ub294 \ube44\ud310\uc740 \uc633\ubc14\ub974\uc9c0 \uc54a\ub2e4. \uc138 \uc544\ub4e4\uc758 \uc778\uc885\uc801 \ubd84\ub958\ud574\uc11d\ubc29\uc2dd\uc740 \uc8fc\ub85c \uae30\ub3c5\uad50\uadfc\ubcf8\uc8fc\uc758, \ubcf5\uc74c\uc8fc\uc758, \uc131\uacbd\uc2e0\uc790 \uc9c4\uc601\uc5d0\uc11c \uc624\ub7ab\ub3d9\uc548 \ud574\uc11d\ub418\uace0 \uc804\ud574\uc838\uc628 \uc804\ud1b5\uc801\uc778 \ud574\uc11d\ubc29\ubc95\uc774\ub2e4.", "sentence_answer": "\uc7a5\ub0a8\uc778 \uc148\uc740 \uc544\uc2dc\uc544 \uc778, \ucc28\ub0a8\uc778 \ud568\uc740 \uc544\ud504\ub9ac\uce74\uc778, \uc0bc\ub0a8\uc778 \uc57c\ubcb3\uc740 \uc720\ub7fd\uc778\uc758 \uc120\uc870\uc778 \uac83\uc73c\ub85c \ud574\uc11d\ub418\uc5c8\ub2e4.", "paragraph_id": "6484143-1-0", "paragraph_question": "question: \ub178\uc544\uc758 \uc7a5\ub0a8 \uc148\uc740 \uc5b4\ub290 \ub300\ub959\uc758 \uc120\uc870\uc778\uac00?, context: \uc911\uc138 \uae30\ub3c5\uad50\uc5d0\uc11c\ub294 \ub178\uc544\uc758 \uc138 \uc544\ub4e4\uc774 \uc138 \ub300\ub959\uc758 \uc0ac\ub78c\ub4e4\uc758 \uc120\uc870\ub85c \uc5ec\uaca8\uc84c\ub2e4. \uc7a5\ub0a8\uc778 \uc148\uc740 \uc544\uc2dc\uc544\uc778, \ucc28\ub0a8\uc778 \ud568\uc740 \uc544\ud504\ub9ac\uce74\uc778, \uc0bc\ub0a8\uc778 \uc57c\ubcb3\uc740 \uc720\ub7fd\uc778\uc758 \uc120\uc870\uc778 \uac83\uc73c\ub85c \ud574\uc11d\ub418\uc5c8\ub2e4. \uc774\uc640 \ub2e4\ub978 \uc18c\uc218\uc758 \uacac\ud574\ub85c, \ub178\uc544\uc758 \uc138 \uc544\ub4e4\uc744 \uac01\uac01 \uc131\uc9c1\uc790(\uc148), \ub18d\ubbfc(\ud568), \uc804\uc0ac(\uc57c\ubcb3)\ub85c \ud574\uc11d\ud558\ub294 \uacac\ud574\uac00 \uc788\uc5c8\ub2e4. \uc138 \uc544\ub4e4\uc744 \uac01\uac01\uc758 \uc778\uc885\uc73c\ub85c \ubd84\ub958\ud558\ub294 \ubc29\uc2dd\uc744 \ub450\uace0\uc11c \ud604\ub300\uc5d0 \uc640\uc11c\ub294 \uc778\uc885\ucc28\ubcc4\uc801 \ud574\uc11d\uc774\ub77c\uace0 \ube44\ud310\ubc1b\uae30\ub294 \ud558\uc9c0\ub9cc, \uc774 \ud574\uc11d\uc740 \uc778\ub958\uc758 \uae30\uc6d0\uacfc \ud37c\uc838\ub098\uac04 \ubc29\uc2dd\uc744 \uc5f0\uad6c\ud558\ub294 \ud1b5\uc2dc\uc801\uc778 \ud574\uc11d\ubc29\ubc95\uc77c\ubfd0 \uc778\uc885\ucc28\ubcc4\uc801 \ud574\uc11d\uc774\ub77c\ub294 \ube44\ud310\uc740 \uc633\ubc14\ub974\uc9c0 \uc54a\ub2e4. \uc138 \uc544\ub4e4\uc758 \uc778\uc885\uc801 \ubd84\ub958\ud574\uc11d\ubc29\uc2dd\uc740 \uc8fc\ub85c \uae30\ub3c5\uad50\uadfc\ubcf8\uc8fc\uc758, \ubcf5\uc74c\uc8fc\uc758, \uc131\uacbd\uc2e0\uc790 \uc9c4\uc601\uc5d0\uc11c \uc624\ub7ab\ub3d9\uc548 \ud574\uc11d\ub418\uace0 \uc804\ud574\uc838\uc628 \uc804\ud1b5\uc801\uc778 \ud574\uc11d\ubc29\ubc95\uc774\ub2e4."} {"question": "\uc77c\uad11 \uc808\uc57d \uc2dc\uac04\uc81c\uac00 \uc2dc\ud589\ub418\ub294 \uacc4\uc808\uc740?", "paragraph": "\uc77c\uad11 \uc808\uc57d \uc2dc\uac04\uc81c(\u65e5\u5149\u7bc0\u7d04\u6642\u9593\u5236, \ubbf8\uad6d \uc601\uc5b4: Daylight saving time, DST) \ub610\ub294 \uc11c\uba38 \ud0c0\uc784(\uc601\uad6d \uc601\uc5b4: summer time)\uc740 \ud558\uc808\uae30\uc5d0 \ud45c\uc900\uc2dc\ub97c \uc6d0\ub798 \uc2dc\uac04\ubcf4\ub2e4 \ud55c \uc2dc\uac04 \uc55e\ub2f9\uae34 \uc2dc\uac04\uc744 \uc4f0\ub294 \uac83\uc744 \ub9d0\ud55c\ub2e4. \uc989, 0\uc2dc\uc5d0 \uc77c\uad11 \uc808\uc57d \uc2dc\uac04\uc81c\ub97c \uc2e4\uc2dc\ud558\uba74 1\uc2dc\ub85c \uc2dc\uac04\uc744 \uc870\uc815\ud574\uc57c \ud558\ub294 \uac83\uc774\ub2e4. \uc2e4\uc81c \ub0ae \uc2dc\uac04\uacfc \uc0ac\ub78c\ub4e4\uc774 \ud65c\ub3d9\ud558\ub294 \ub0ae \uc2dc\uac04 \uc0ac\uc774\uc758 \uaca9\ucc28\ub97c \uc904\uc774\uae30 \uc704\ud574 \uc0ac\uc6a9\ud55c\ub2e4. \uc5ec\ub984\uc5d0\ub294 \uc77c\uc870 \uc2dc\uac04\uc774 \uae38\ubbc0\ub85c \ud65c\ub3d9\uc744 \ubcf4\ub2e4 \uc77c\ucc0d \uc2dc\uc791\ud558\uc5ec \uc800\ub141 \ub54c \uc9c1\uc7a5\uc774\ub098 \ud559\uad50\uc5d0\uc11c \uc774\ub807\uac8c '\uc808\uc57d\ub41c \ub0ae \uc2dc\uac04'\uc744 \ub354 \ubc1d\uc740 \uc0c1\ud0dc\uc5d0\uc11c \uc624\ud6c4\uc5d0 \ud65c\ub3d9\ud560 \uc218 \uc788\uac8c \ud558\ub294 \ud6a8\uacfc\uac00 \uc788\uc73c\uba70, \ub610\ud55c \uc9c1\uc7a5\uc774\ub098 \ud559\uad50\uc5d0\uc11c\uc758 \uc870\uba85\uacfc \uc5f0\ub8cc \ub4f1\uc758 \uc808\uac10 \ud6a8\uacfc\ub97c \uae30\ub300\ud560 \uc218 \uc788\uae30 \ub54c\ubb38\uc774\ub2e4. \uc628\ub300 \uc9c0\uc5ed\uc5d0\uc11c\ub294 \uacc4\uc808\uc5d0 \ub530\ub978 \uc77c\uc870\ub7c9\uc758 \ucc28\uc774\uac00 \ud06c\ubbc0\ub85c \uc77c\uad11 \uc808\uc57d \uc2dc\uac04\uc81c\ub294 \ubcf4\ud1b5 \uc628\ub300 \uc9c0\uc5ed\uc5d0\uc11c \uc2dc\ud589\ub41c\ub2e4.", "answer": "\uc5ec\ub984", "sentence": "\uc5ec\ub984 \uc5d0\ub294 \uc77c\uc870 \uc2dc\uac04\uc774 \uae38\ubbc0\ub85c \ud65c\ub3d9\uc744 \ubcf4\ub2e4 \uc77c\ucc0d \uc2dc\uc791\ud558\uc5ec \uc800\ub141 \ub54c \uc9c1\uc7a5\uc774\ub098 \ud559\uad50\uc5d0\uc11c \uc774\ub807\uac8c '\uc808\uc57d\ub41c \ub0ae \uc2dc\uac04'\uc744 \ub354 \ubc1d\uc740 \uc0c1\ud0dc\uc5d0\uc11c \uc624\ud6c4\uc5d0 \ud65c\ub3d9\ud560 \uc218 \uc788\uac8c \ud558\ub294 \ud6a8\uacfc\uac00 \uc788\uc73c\uba70, \ub610\ud55c \uc9c1\uc7a5\uc774\ub098 \ud559\uad50\uc5d0\uc11c\uc758 \uc870\uba85\uacfc \uc5f0\ub8cc \ub4f1\uc758 \uc808\uac10 \ud6a8\uacfc\ub97c \uae30\ub300\ud560 \uc218 \uc788\uae30 \ub54c\ubb38\uc774\ub2e4.", "paragraph_sentence": "\uc77c\uad11 \uc808\uc57d \uc2dc\uac04\uc81c(\u65e5\u5149\u7bc0\u7d04\u6642\u9593\u5236, \ubbf8\uad6d \uc601\uc5b4: Daylight saving time, DST) \ub610\ub294 \uc11c\uba38 \ud0c0\uc784(\uc601\uad6d \uc601\uc5b4: summer time)\uc740 \ud558\uc808\uae30\uc5d0 \ud45c\uc900\uc2dc\ub97c \uc6d0\ub798 \uc2dc\uac04\ubcf4\ub2e4 \ud55c \uc2dc\uac04 \uc55e\ub2f9\uae34 \uc2dc\uac04\uc744 \uc4f0\ub294 \uac83\uc744 \ub9d0\ud55c\ub2e4. \uc989, 0\uc2dc\uc5d0 \uc77c\uad11 \uc808\uc57d \uc2dc\uac04\uc81c\ub97c \uc2e4\uc2dc\ud558\uba74 1\uc2dc\ub85c \uc2dc\uac04\uc744 \uc870\uc815\ud574\uc57c \ud558\ub294 \uac83\uc774\ub2e4. \uc2e4\uc81c \ub0ae \uc2dc\uac04\uacfc \uc0ac\ub78c\ub4e4\uc774 \ud65c\ub3d9\ud558\ub294 \ub0ae \uc2dc\uac04 \uc0ac\uc774\uc758 \uaca9\ucc28\ub97c \uc904\uc774\uae30 \uc704\ud574 \uc0ac\uc6a9\ud55c\ub2e4. \uc5ec\ub984 \uc5d0\ub294 \uc77c\uc870 \uc2dc\uac04\uc774 \uae38\ubbc0\ub85c \ud65c\ub3d9\uc744 \ubcf4\ub2e4 \uc77c\ucc0d \uc2dc\uc791\ud558\uc5ec \uc800\ub141 \ub54c \uc9c1\uc7a5\uc774\ub098 \ud559\uad50\uc5d0\uc11c \uc774\ub807\uac8c '\uc808\uc57d\ub41c \ub0ae \uc2dc\uac04'\uc744 \ub354 \ubc1d\uc740 \uc0c1\ud0dc\uc5d0\uc11c \uc624\ud6c4\uc5d0 \ud65c\ub3d9\ud560 \uc218 \uc788\uac8c \ud558\ub294 \ud6a8\uacfc\uac00 \uc788\uc73c\uba70, \ub610\ud55c \uc9c1\uc7a5\uc774\ub098 \ud559\uad50\uc5d0\uc11c\uc758 \uc870\uba85\uacfc \uc5f0\ub8cc \ub4f1\uc758 \uc808\uac10 \ud6a8\uacfc\ub97c \uae30\ub300\ud560 \uc218 \uc788\uae30 \ub54c\ubb38\uc774\ub2e4. \uc628\ub300 \uc9c0\uc5ed\uc5d0\uc11c\ub294 \uacc4\uc808\uc5d0 \ub530\ub978 \uc77c\uc870\ub7c9\uc758 \ucc28\uc774\uac00 \ud06c\ubbc0\ub85c \uc77c\uad11 \uc808\uc57d \uc2dc\uac04\uc81c\ub294 \ubcf4\ud1b5 \uc628\ub300 \uc9c0\uc5ed\uc5d0\uc11c \uc2dc\ud589\ub41c\ub2e4.", "paragraph_answer": "\uc77c\uad11 \uc808\uc57d \uc2dc\uac04\uc81c(\u65e5\u5149\u7bc0\u7d04\u6642\u9593\u5236, \ubbf8\uad6d \uc601\uc5b4: Daylight saving time, DST) \ub610\ub294 \uc11c\uba38 \ud0c0\uc784(\uc601\uad6d \uc601\uc5b4: summer time)\uc740 \ud558\uc808\uae30\uc5d0 \ud45c\uc900\uc2dc\ub97c \uc6d0\ub798 \uc2dc\uac04\ubcf4\ub2e4 \ud55c \uc2dc\uac04 \uc55e\ub2f9\uae34 \uc2dc\uac04\uc744 \uc4f0\ub294 \uac83\uc744 \ub9d0\ud55c\ub2e4. \uc989, 0\uc2dc\uc5d0 \uc77c\uad11 \uc808\uc57d \uc2dc\uac04\uc81c\ub97c \uc2e4\uc2dc\ud558\uba74 1\uc2dc\ub85c \uc2dc\uac04\uc744 \uc870\uc815\ud574\uc57c \ud558\ub294 \uac83\uc774\ub2e4. \uc2e4\uc81c \ub0ae \uc2dc\uac04\uacfc \uc0ac\ub78c\ub4e4\uc774 \ud65c\ub3d9\ud558\ub294 \ub0ae \uc2dc\uac04 \uc0ac\uc774\uc758 \uaca9\ucc28\ub97c \uc904\uc774\uae30 \uc704\ud574 \uc0ac\uc6a9\ud55c\ub2e4. \uc5ec\ub984 \uc5d0\ub294 \uc77c\uc870 \uc2dc\uac04\uc774 \uae38\ubbc0\ub85c \ud65c\ub3d9\uc744 \ubcf4\ub2e4 \uc77c\ucc0d \uc2dc\uc791\ud558\uc5ec \uc800\ub141 \ub54c \uc9c1\uc7a5\uc774\ub098 \ud559\uad50\uc5d0\uc11c \uc774\ub807\uac8c '\uc808\uc57d\ub41c \ub0ae \uc2dc\uac04'\uc744 \ub354 \ubc1d\uc740 \uc0c1\ud0dc\uc5d0\uc11c \uc624\ud6c4\uc5d0 \ud65c\ub3d9\ud560 \uc218 \uc788\uac8c \ud558\ub294 \ud6a8\uacfc\uac00 \uc788\uc73c\uba70, \ub610\ud55c \uc9c1\uc7a5\uc774\ub098 \ud559\uad50\uc5d0\uc11c\uc758 \uc870\uba85\uacfc \uc5f0\ub8cc \ub4f1\uc758 \uc808\uac10 \ud6a8\uacfc\ub97c \uae30\ub300\ud560 \uc218 \uc788\uae30 \ub54c\ubb38\uc774\ub2e4. \uc628\ub300 \uc9c0\uc5ed\uc5d0\uc11c\ub294 \uacc4\uc808\uc5d0 \ub530\ub978 \uc77c\uc870\ub7c9\uc758 \ucc28\uc774\uac00 \ud06c\ubbc0\ub85c \uc77c\uad11 \uc808\uc57d \uc2dc\uac04\uc81c\ub294 \ubcf4\ud1b5 \uc628\ub300 \uc9c0\uc5ed\uc5d0\uc11c \uc2dc\ud589\ub41c\ub2e4.", "sentence_answer": " \uc5ec\ub984 \uc5d0\ub294 \uc77c\uc870 \uc2dc\uac04\uc774 \uae38\ubbc0\ub85c \ud65c\ub3d9\uc744 \ubcf4\ub2e4 \uc77c\ucc0d \uc2dc\uc791\ud558\uc5ec \uc800\ub141 \ub54c \uc9c1\uc7a5\uc774\ub098 \ud559\uad50\uc5d0\uc11c \uc774\ub807\uac8c '\uc808\uc57d\ub41c \ub0ae \uc2dc\uac04'\uc744 \ub354 \ubc1d\uc740 \uc0c1\ud0dc\uc5d0\uc11c \uc624\ud6c4\uc5d0 \ud65c\ub3d9\ud560 \uc218 \uc788\uac8c \ud558\ub294 \ud6a8\uacfc\uac00 \uc788\uc73c\uba70, \ub610\ud55c \uc9c1\uc7a5\uc774\ub098 \ud559\uad50\uc5d0\uc11c\uc758 \uc870\uba85\uacfc \uc5f0\ub8cc \ub4f1\uc758 \uc808\uac10 \ud6a8\uacfc\ub97c \uae30\ub300\ud560 \uc218 \uc788\uae30 \ub54c\ubb38\uc774\ub2e4.", "paragraph_id": "5853631-0-1", "paragraph_question": "question: \uc77c\uad11 \uc808\uc57d \uc2dc\uac04\uc81c\uac00 \uc2dc\ud589\ub418\ub294 \uacc4\uc808\uc740?, context: \uc77c\uad11 \uc808\uc57d \uc2dc\uac04\uc81c(\u65e5\u5149\u7bc0\u7d04\u6642\u9593\u5236, \ubbf8\uad6d \uc601\uc5b4: Daylight saving time, DST) \ub610\ub294 \uc11c\uba38 \ud0c0\uc784(\uc601\uad6d \uc601\uc5b4: summer time)\uc740 \ud558\uc808\uae30\uc5d0 \ud45c\uc900\uc2dc\ub97c \uc6d0\ub798 \uc2dc\uac04\ubcf4\ub2e4 \ud55c \uc2dc\uac04 \uc55e\ub2f9\uae34 \uc2dc\uac04\uc744 \uc4f0\ub294 \uac83\uc744 \ub9d0\ud55c\ub2e4. \uc989, 0\uc2dc\uc5d0 \uc77c\uad11 \uc808\uc57d \uc2dc\uac04\uc81c\ub97c \uc2e4\uc2dc\ud558\uba74 1\uc2dc\ub85c \uc2dc\uac04\uc744 \uc870\uc815\ud574\uc57c \ud558\ub294 \uac83\uc774\ub2e4. \uc2e4\uc81c \ub0ae \uc2dc\uac04\uacfc \uc0ac\ub78c\ub4e4\uc774 \ud65c\ub3d9\ud558\ub294 \ub0ae \uc2dc\uac04 \uc0ac\uc774\uc758 \uaca9\ucc28\ub97c \uc904\uc774\uae30 \uc704\ud574 \uc0ac\uc6a9\ud55c\ub2e4. \uc5ec\ub984\uc5d0\ub294 \uc77c\uc870 \uc2dc\uac04\uc774 \uae38\ubbc0\ub85c \ud65c\ub3d9\uc744 \ubcf4\ub2e4 \uc77c\ucc0d \uc2dc\uc791\ud558\uc5ec \uc800\ub141 \ub54c \uc9c1\uc7a5\uc774\ub098 \ud559\uad50\uc5d0\uc11c \uc774\ub807\uac8c '\uc808\uc57d\ub41c \ub0ae \uc2dc\uac04'\uc744 \ub354 \ubc1d\uc740 \uc0c1\ud0dc\uc5d0\uc11c \uc624\ud6c4\uc5d0 \ud65c\ub3d9\ud560 \uc218 \uc788\uac8c \ud558\ub294 \ud6a8\uacfc\uac00 \uc788\uc73c\uba70, \ub610\ud55c \uc9c1\uc7a5\uc774\ub098 \ud559\uad50\uc5d0\uc11c\uc758 \uc870\uba85\uacfc \uc5f0\ub8cc \ub4f1\uc758 \uc808\uac10 \ud6a8\uacfc\ub97c \uae30\ub300\ud560 \uc218 \uc788\uae30 \ub54c\ubb38\uc774\ub2e4. \uc628\ub300 \uc9c0\uc5ed\uc5d0\uc11c\ub294 \uacc4\uc808\uc5d0 \ub530\ub978 \uc77c\uc870\ub7c9\uc758 \ucc28\uc774\uac00 \ud06c\ubbc0\ub85c \uc77c\uad11 \uc808\uc57d \uc2dc\uac04\uc81c\ub294 \ubcf4\ud1b5 \uc628\ub300 \uc9c0\uc5ed\uc5d0\uc11c \uc2dc\ud589\ub41c\ub2e4."} {"question": "\uc694\uc81c\ud504\uc758 \uc544\ubc84\uc9c0\ub294 \uc5b8\uc81c \ud0dc\uc5b4\ub0ac\ub098?", "paragraph": "\uc694\uc81c\ud504 \uad34\ubca8\uc2a4\ub294 1897\ub144 \ub3c5\uc77c \ub178\ub974\ud2b8\ub77c\uc778\ubca0\uc2a4\ud2b8\ud314\ub80c \uc8fc \ub77c\uc774\ud2b8(Rheydt)\uc5d0\uc11c \ucd9c\uc0dd\ud558\uc600\ub2e4. \uad34\ubca8\uc2a4\uc758 \ud560\uc544\ubc84\uc9c0 \ucf58\ub77c\ud2b8 \uad34\ubca8\uc2a4\ub294 \ub18d\ubd80\uc600\uc73c\ub098, \uadf8 \ub2f9\uc2dc \ub18d\ucd0c\uc758 \uc0ac\ub78c\ub4e4\uc740 \ub3c4\uc2dc\uc758 \ub178\ub3d9\uc790\ub85c\uc11c\uc758 \uc0b6\uc774 \uc790\uc2e0\ub4e4\uc5d0\uac8c \ub354 \ub098\uc740 \uc0b6\uc774 \ub420 \uac83\uc774\ub77c \ubbff\uace0 \uacf5\uc5c5 \ub3c4\uc2dc\ub85c \uc774\ub3d9\ud558\ub294 \uc0ac\ub78c\ub4e4\uc774 \ub9ce\uc558\ub2e4. \ucf58\ub77c\ud2b8\ub3c4 \uc774\uc911 \ud55c \uba85\uc774\uc5c8\ub294\ub370, \uacf5\uc5c5\ub3c4\uc2dc \ub77c\uc774\ud2b8\ub85c \uc774\uc8fc\ud574\uc11c \uacb0\ud63c\ud558\uace0 \uc694\uc81c\ud504 \uad34\ubca8\uc2a4\uc758 \uc544\ubc84\uc9c0\uc778 \ud504\ub9ac\uce20 \uad34\ubca8\uc2a4\ub97c 1867\ub144\uc5d0 \ub0b3\uc740 \ud6c4 \ud3c9\uc0dd\uc744 \ub178\ub3d9\uc790(\ud504\ub864\ub808\ud0c0\ub9ac\uc544)\ub85c \uc0b4\uc558\ub2e4. \uc694\uc81c\ud504 \uad34\ubca8\uc2a4\uc758 \uc544\ubc84\uc9c0 \ud504\ub9ac\uce20\ub3c4 \uc2ec\uc9c0 \uacf5\uc7a5\uc5d0\uc11c \ub2e8\uc21c \ub178\ub3d9\uc790\uc77c \ubfd0\uc774\uc5c8\uc73c\ub098, \ubd80\uc9c0\ub7f0\ud55c \uc0ac\ub78c\uc774\uc5c8\uae30 \ub54c\ubb38\uc5d0 \uacf5\uc7a5 \uc9c0\ubc30\uc778\uc774 \ub418\uc5b4 \uc18c\uc2dc\ubbfc(\ud504\ud2f0\ubd80\ub974\uc8fc\uc544) \uacc4\uae09\uc774 \ub41c\ub2e4. \ud504\ub9ac\uce20 \uad34\ubca8\uc2a4\ub294 1892\ub144\uc5d0 \uce74\ud0c0\ub9ac\ub098 \uc624\ub374\ud558\uc6b0\uc820\uacfc \uacb0\ud63c\ud558\uc5ec \uc544\ub4e4 \uc138 \uba85\uacfc \ub538 \ub450 \uba85\uc744 \ub0b3\uc558\ub294\ub370 \uc694\uc81c\ud504 \uad34\ubca8\uc2a4\ub294 \uadf8\ub4e4\uc758 \uc14b\uc9f8 \uc544\ub4e4\ub85c 1897\ub144\uc5d0 \ud0dc\uc5b4\ub0ac\ub2e4.", "answer": "1867\ub144", "sentence": "\ucf58\ub77c\ud2b8\ub3c4 \uc774\uc911 \ud55c \uba85\uc774\uc5c8\ub294\ub370, \uacf5\uc5c5\ub3c4\uc2dc \ub77c\uc774\ud2b8\ub85c \uc774\uc8fc\ud574\uc11c \uacb0\ud63c\ud558\uace0 \uc694\uc81c\ud504 \uad34\ubca8\uc2a4\uc758 \uc544\ubc84\uc9c0\uc778 \ud504\ub9ac\uce20 \uad34\ubca8\uc2a4\ub97c 1867\ub144 \uc5d0 \ub0b3\uc740 \ud6c4 \ud3c9\uc0dd\uc744 \ub178\ub3d9\uc790(\ud504\ub864\ub808\ud0c0\ub9ac\uc544)\ub85c \uc0b4\uc558\ub2e4.", "paragraph_sentence": "\uc694\uc81c\ud504 \uad34\ubca8\uc2a4\ub294 1897\ub144 \ub3c5\uc77c \ub178\ub974\ud2b8\ub77c\uc778\ubca0\uc2a4\ud2b8\ud314\ub80c \uc8fc \ub77c\uc774\ud2b8(Rheydt)\uc5d0\uc11c \ucd9c\uc0dd\ud558\uc600\ub2e4. \uad34\ubca8\uc2a4\uc758 \ud560\uc544\ubc84\uc9c0 \ucf58\ub77c\ud2b8 \uad34\ubca8\uc2a4\ub294 \ub18d\ubd80\uc600\uc73c\ub098, \uadf8 \ub2f9\uc2dc \ub18d\ucd0c\uc758 \uc0ac\ub78c\ub4e4\uc740 \ub3c4\uc2dc\uc758 \ub178\ub3d9\uc790\ub85c\uc11c\uc758 \uc0b6\uc774 \uc790\uc2e0\ub4e4\uc5d0\uac8c \ub354 \ub098\uc740 \uc0b6\uc774 \ub420 \uac83\uc774\ub77c \ubbff\uace0 \uacf5\uc5c5 \ub3c4\uc2dc\ub85c \uc774\ub3d9\ud558\ub294 \uc0ac\ub78c\ub4e4\uc774 \ub9ce\uc558\ub2e4. \ucf58\ub77c\ud2b8\ub3c4 \uc774\uc911 \ud55c \uba85\uc774\uc5c8\ub294\ub370, \uacf5\uc5c5\ub3c4\uc2dc \ub77c\uc774\ud2b8\ub85c \uc774\uc8fc\ud574\uc11c \uacb0\ud63c\ud558\uace0 \uc694\uc81c\ud504 \uad34\ubca8\uc2a4\uc758 \uc544\ubc84\uc9c0\uc778 \ud504\ub9ac\uce20 \uad34\ubca8\uc2a4\ub97c 1867\ub144 \uc5d0 \ub0b3\uc740 \ud6c4 \ud3c9\uc0dd\uc744 \ub178\ub3d9\uc790(\ud504\ub864\ub808\ud0c0\ub9ac\uc544)\ub85c \uc0b4\uc558\ub2e4. \uc694\uc81c\ud504 \uad34\ubca8\uc2a4\uc758 \uc544\ubc84\uc9c0 \ud504\ub9ac\uce20\ub3c4 \uc2ec\uc9c0 \uacf5\uc7a5\uc5d0\uc11c \ub2e8\uc21c \ub178\ub3d9\uc790\uc77c \ubfd0\uc774\uc5c8\uc73c\ub098, \ubd80\uc9c0\ub7f0\ud55c \uc0ac\ub78c\uc774\uc5c8\uae30 \ub54c\ubb38\uc5d0 \uacf5\uc7a5 \uc9c0\ubc30\uc778\uc774 \ub418\uc5b4 \uc18c\uc2dc\ubbfc(\ud504\ud2f0\ubd80\ub974\uc8fc\uc544) \uacc4\uae09\uc774 \ub41c\ub2e4. \ud504\ub9ac\uce20 \uad34\ubca8\uc2a4\ub294 1892\ub144\uc5d0 \uce74\ud0c0\ub9ac\ub098 \uc624\ub374\ud558\uc6b0\uc820\uacfc \uacb0\ud63c\ud558\uc5ec \uc544\ub4e4 \uc138 \uba85\uacfc \ub538 \ub450 \uba85\uc744 \ub0b3\uc558\ub294\ub370 \uc694\uc81c\ud504 \uad34\ubca8\uc2a4\ub294 \uadf8\ub4e4\uc758 \uc14b\uc9f8 \uc544\ub4e4\ub85c 1897\ub144\uc5d0 \ud0dc\uc5b4\ub0ac\ub2e4.", "paragraph_answer": "\uc694\uc81c\ud504 \uad34\ubca8\uc2a4\ub294 1897\ub144 \ub3c5\uc77c \ub178\ub974\ud2b8\ub77c\uc778\ubca0\uc2a4\ud2b8\ud314\ub80c \uc8fc \ub77c\uc774\ud2b8(Rheydt)\uc5d0\uc11c \ucd9c\uc0dd\ud558\uc600\ub2e4. \uad34\ubca8\uc2a4\uc758 \ud560\uc544\ubc84\uc9c0 \ucf58\ub77c\ud2b8 \uad34\ubca8\uc2a4\ub294 \ub18d\ubd80\uc600\uc73c\ub098, \uadf8 \ub2f9\uc2dc \ub18d\ucd0c\uc758 \uc0ac\ub78c\ub4e4\uc740 \ub3c4\uc2dc\uc758 \ub178\ub3d9\uc790\ub85c\uc11c\uc758 \uc0b6\uc774 \uc790\uc2e0\ub4e4\uc5d0\uac8c \ub354 \ub098\uc740 \uc0b6\uc774 \ub420 \uac83\uc774\ub77c \ubbff\uace0 \uacf5\uc5c5 \ub3c4\uc2dc\ub85c \uc774\ub3d9\ud558\ub294 \uc0ac\ub78c\ub4e4\uc774 \ub9ce\uc558\ub2e4. \ucf58\ub77c\ud2b8\ub3c4 \uc774\uc911 \ud55c \uba85\uc774\uc5c8\ub294\ub370, \uacf5\uc5c5\ub3c4\uc2dc \ub77c\uc774\ud2b8\ub85c \uc774\uc8fc\ud574\uc11c \uacb0\ud63c\ud558\uace0 \uc694\uc81c\ud504 \uad34\ubca8\uc2a4\uc758 \uc544\ubc84\uc9c0\uc778 \ud504\ub9ac\uce20 \uad34\ubca8\uc2a4\ub97c 1867\ub144 \uc5d0 \ub0b3\uc740 \ud6c4 \ud3c9\uc0dd\uc744 \ub178\ub3d9\uc790(\ud504\ub864\ub808\ud0c0\ub9ac\uc544)\ub85c \uc0b4\uc558\ub2e4. \uc694\uc81c\ud504 \uad34\ubca8\uc2a4\uc758 \uc544\ubc84\uc9c0 \ud504\ub9ac\uce20\ub3c4 \uc2ec\uc9c0 \uacf5\uc7a5\uc5d0\uc11c \ub2e8\uc21c \ub178\ub3d9\uc790\uc77c \ubfd0\uc774\uc5c8\uc73c\ub098, \ubd80\uc9c0\ub7f0\ud55c \uc0ac\ub78c\uc774\uc5c8\uae30 \ub54c\ubb38\uc5d0 \uacf5\uc7a5 \uc9c0\ubc30\uc778\uc774 \ub418\uc5b4 \uc18c\uc2dc\ubbfc(\ud504\ud2f0\ubd80\ub974\uc8fc\uc544) \uacc4\uae09\uc774 \ub41c\ub2e4. \ud504\ub9ac\uce20 \uad34\ubca8\uc2a4\ub294 1892\ub144\uc5d0 \uce74\ud0c0\ub9ac\ub098 \uc624\ub374\ud558\uc6b0\uc820\uacfc \uacb0\ud63c\ud558\uc5ec \uc544\ub4e4 \uc138 \uba85\uacfc \ub538 \ub450 \uba85\uc744 \ub0b3\uc558\ub294\ub370 \uc694\uc81c\ud504 \uad34\ubca8\uc2a4\ub294 \uadf8\ub4e4\uc758 \uc14b\uc9f8 \uc544\ub4e4\ub85c 1897\ub144\uc5d0 \ud0dc\uc5b4\ub0ac\ub2e4.", "sentence_answer": "\ucf58\ub77c\ud2b8\ub3c4 \uc774\uc911 \ud55c \uba85\uc774\uc5c8\ub294\ub370, \uacf5\uc5c5\ub3c4\uc2dc \ub77c\uc774\ud2b8\ub85c \uc774\uc8fc\ud574\uc11c \uacb0\ud63c\ud558\uace0 \uc694\uc81c\ud504 \uad34\ubca8\uc2a4\uc758 \uc544\ubc84\uc9c0\uc778 \ud504\ub9ac\uce20 \uad34\ubca8\uc2a4\ub97c 1867\ub144 \uc5d0 \ub0b3\uc740 \ud6c4 \ud3c9\uc0dd\uc744 \ub178\ub3d9\uc790(\ud504\ub864\ub808\ud0c0\ub9ac\uc544)\ub85c \uc0b4\uc558\ub2e4.", "paragraph_id": "5820364-1-1", "paragraph_question": "question: \uc694\uc81c\ud504\uc758 \uc544\ubc84\uc9c0\ub294 \uc5b8\uc81c \ud0dc\uc5b4\ub0ac\ub098?, context: \uc694\uc81c\ud504 \uad34\ubca8\uc2a4\ub294 1897\ub144 \ub3c5\uc77c \ub178\ub974\ud2b8\ub77c\uc778\ubca0\uc2a4\ud2b8\ud314\ub80c \uc8fc \ub77c\uc774\ud2b8(Rheydt)\uc5d0\uc11c \ucd9c\uc0dd\ud558\uc600\ub2e4. \uad34\ubca8\uc2a4\uc758 \ud560\uc544\ubc84\uc9c0 \ucf58\ub77c\ud2b8 \uad34\ubca8\uc2a4\ub294 \ub18d\ubd80\uc600\uc73c\ub098, \uadf8 \ub2f9\uc2dc \ub18d\ucd0c\uc758 \uc0ac\ub78c\ub4e4\uc740 \ub3c4\uc2dc\uc758 \ub178\ub3d9\uc790\ub85c\uc11c\uc758 \uc0b6\uc774 \uc790\uc2e0\ub4e4\uc5d0\uac8c \ub354 \ub098\uc740 \uc0b6\uc774 \ub420 \uac83\uc774\ub77c \ubbff\uace0 \uacf5\uc5c5 \ub3c4\uc2dc\ub85c \uc774\ub3d9\ud558\ub294 \uc0ac\ub78c\ub4e4\uc774 \ub9ce\uc558\ub2e4. \ucf58\ub77c\ud2b8\ub3c4 \uc774\uc911 \ud55c \uba85\uc774\uc5c8\ub294\ub370, \uacf5\uc5c5\ub3c4\uc2dc \ub77c\uc774\ud2b8\ub85c \uc774\uc8fc\ud574\uc11c \uacb0\ud63c\ud558\uace0 \uc694\uc81c\ud504 \uad34\ubca8\uc2a4\uc758 \uc544\ubc84\uc9c0\uc778 \ud504\ub9ac\uce20 \uad34\ubca8\uc2a4\ub97c 1867\ub144\uc5d0 \ub0b3\uc740 \ud6c4 \ud3c9\uc0dd\uc744 \ub178\ub3d9\uc790(\ud504\ub864\ub808\ud0c0\ub9ac\uc544)\ub85c \uc0b4\uc558\ub2e4. \uc694\uc81c\ud504 \uad34\ubca8\uc2a4\uc758 \uc544\ubc84\uc9c0 \ud504\ub9ac\uce20\ub3c4 \uc2ec\uc9c0 \uacf5\uc7a5\uc5d0\uc11c \ub2e8\uc21c \ub178\ub3d9\uc790\uc77c \ubfd0\uc774\uc5c8\uc73c\ub098, \ubd80\uc9c0\ub7f0\ud55c \uc0ac\ub78c\uc774\uc5c8\uae30 \ub54c\ubb38\uc5d0 \uacf5\uc7a5 \uc9c0\ubc30\uc778\uc774 \ub418\uc5b4 \uc18c\uc2dc\ubbfc(\ud504\ud2f0\ubd80\ub974\uc8fc\uc544) \uacc4\uae09\uc774 \ub41c\ub2e4. \ud504\ub9ac\uce20 \uad34\ubca8\uc2a4\ub294 1892\ub144\uc5d0 \uce74\ud0c0\ub9ac\ub098 \uc624\ub374\ud558\uc6b0\uc820\uacfc \uacb0\ud63c\ud558\uc5ec \uc544\ub4e4 \uc138 \uba85\uacfc \ub538 \ub450 \uba85\uc744 \ub0b3\uc558\ub294\ub370 \uc694\uc81c\ud504 \uad34\ubca8\uc2a4\ub294 \uadf8\ub4e4\uc758 \uc14b\uc9f8 \uc544\ub4e4\ub85c 1897\ub144\uc5d0 \ud0dc\uc5b4\ub0ac\ub2e4."} {"question": "\ub300\uc911 \ub77c\ud2f4\uc5b4\ub85c \ubc88\uc5ed\ud558\uc5ec \ud0c4\uc0dd\ud558\uac8c \ub41c \uc131\uacbd \uc774\ub984\uc740?", "paragraph": "\ub2e4\ub9c8\uc18c 1\uc138\ub294 \ube44\ubc00\ub9ac\uc5d0 \uc608\ub85c\ub2c8\ubaa8\ub97c \uc790\uc2e0\uc758 \ube44\uc11c\ub85c \uc784\uba85\ud558\uc600\ub2e4. 409\ub144 \uc608\ub85c\ub2c8\ubaa8\ub294 \u201c\uc5ec\ub7ec \ud574 \ub3d9\uc548 \ub098\ub294 \ub85c\ub9c8\uc758 \uc8fc\uad50 \ub2e4\ub9c8\uc18c\ub97c \ub3c4\uc640 \ub3d9\u00b7\uc11c \uad50\ud68c\ud68c\uc758\uc5d0\uc11c \uadf8\uc5d0\uac8c \ubb38\uc758\ub41c \uc9c8\ubb38\ub4e4\uc5d0 \ub300\ud55c \uadf8\uc758 \ub2f5\ubcc0\uc744 \uc801\ub294 \uac83\uacfc \uac19\uc740 \uc11c\uc2e0 \uc791\uc131\uc744 \ud558\uc600\ub2e4.\u201d\uace0 \uae30\ub85d\ud558\uc600\ub2e4. \uae30\ub3c5\uad50\uac00 \ub85c\ub9c8 \uc81c\uad6d\uc758 \uad6d\uad50\ub85c \uc120\ud3ec\ub41c \uac83\uc740 \ub2e4\ub9c8\uc18c 1\uc138\uac00 \uad50\ud669\uc73c\ub85c \uc7ac\uc784\ud558\ub358(380\ub144) \uc911\uc774\uc5c8\ub2e4. \uadf8\ub9ac\uace0 \ub2e4\ub9c8\uc18c 1\uc138\uc758 \uad50\ud68c \uac1c\ud601\uc5d0 \ub530\ub77c \ub77c\ud2f4\uc5b4\uac00 \uae30\ub3c5\uad50\uc758 \uc911\uc2ec\uc801 \uc804\ub840 \uc5b8\uc5b4\uac00 \ub418\uc5c8\ub2e4. \ub610\ud55c, \ub2f9\uc2dc\uc5d0\ub294 \uc131\uacbd\uc774 \uc5ec\ub7ec \uc5ed\uc790\uc5d0 \uc758\ud574\uc11c \ubc88\uc5ed\ub418\uc5b4 \ud1b5\uc77c\uc131\uc774 \uc5c6\uc774 \ub09c\uc7a1\ud558\uace0 \uc815\ub9ac\uac00 \uc548 \ub418\uc5b4 \uc788\uc5c8\ub358 \uc0c1\ud669\uc774\uc5c8\uae30 \ub54c\ubb38\uc5d0, \ud1b5\uc77c\uc131 \uc788\ub294 \uc0c8\ub85c\uc6b4 \uacf5\uc6a9 \uc131\uacbd \ubcf8\ubb38\uc774 \uc694\uad6c\ub418\ub294 \uc0c1\ud669\uc774\uc5c8\ub2e4. \uadf8\ub9ac\ud558\uc5ec \ub2e4\ub9c8\uc18c 1\uc138\ub294 \uc608\ub85c\ub2c8\ubaa8\uc5d0\uac8c \uadf8\ub9ac\uc2a4\uc5b4 \uc2e0\uc57d\uc131\uacbd\uacfc 70\uc778\uc5ed \ub4f1\uc744 \ubc88\uc5ed \ub300\ubcf8\uc73c\ub85c \uc0bc\uc544 \uc61b \uc0c1\uace0 \ub77c\ud2f4\uc5b4\ub85c \ubc88\uc5ed\ub41c \uc131\uacbd\uc744 \ubcf4\ub2e4 \uc815\ud655\ud558\uace0 \ub2f9\uc2dc \ub300\uc911\uc774 \ub110\ub9ac \uc0ac\uc6a9\ud558\ub294 \ub300\uc911 \ub77c\ud2f4\uc5b4\ub85c \ubc88\uc5ed\ud558\ub3c4\ub85d \uc7a5\ub824\ud558\uc600\ub2e4. \uadf8 \uacb0\uacfc \ubd88\uac00\ud0c0 \uc131\uacbd\uc774 \ub098\uc624\uac8c \ub418\uc5c8\ub2e4. \uc774 \uc131\uacbd\uc740 382\ub144 \ub85c\ub9c8 \uad50\ud68c\ud68c\uc758\ub97c \ud1b5\ud574 \uacf5\uc2dd\uc801\uc778 \ub77c\ud2f4\uc5b4 \uc131\uacbd\uc73c\ub85c \uc778\uc900\uc744 \ubc1b\uc558\ub2e4. \ub2e4\ub9c8\uc18c 1\uc138\uc758 \uc0ac\ud6c4, \uc608\ub85c\ub2c8\ubaa8\ub294 \uc800\uc11c \u300a\uc704\uc778\uc804\u300b(De Viris Illustribus)\uc5d0\uc11c \ub2e4\ub9c8\uc18c 1\uc138\uc5d0 \ub300\ud574 \ub2e4\uc74c\uacfc \uac19\uc774 \uc9e7\uc740 \uae00\uc744 \uc37c\ub2e4. \u201c\uc2dc\ub97c \uc4f0\ub294 \ub370 \ud0c1\uc6d4\ud55c \uc7ac\ub2a5\uc744 \uc9c0\ub154\ub358 \uadf8\ub294 \uc544\ub984\ub2e4\uc6b4 \uc6b4\uc728\uc744 \uc9c0\ub2cc \ub9ce\uc740 \uc2dc\ub97c \uc37c\ub2e4. \uadf8\ub294 80\uc138\uc5d0 \uac00\uae4c\uc6b4 \ub098\uc774\uc5d0 \ud14c\uc624\ub3c4\uc2dc\uc6b0\uc2a4 1\uc138 \ud669\uc81c\uc758 \uce58\uc138 \ub54c \uc120\uc885\ud558\uc600\ub2e4.\u201d", "answer": "\ubd88\uac00\ud0c0 \uc131\uacbd", "sentence": "\uadf8 \uacb0\uacfc \ubd88\uac00\ud0c0 \uc131\uacbd \uc774 \ub098\uc624\uac8c \ub418\uc5c8\ub2e4.", "paragraph_sentence": "\ub2e4\ub9c8\uc18c 1\uc138\ub294 \ube44\ubc00\ub9ac\uc5d0 \uc608\ub85c\ub2c8\ubaa8\ub97c \uc790\uc2e0\uc758 \ube44\uc11c\ub85c \uc784\uba85\ud558\uc600\ub2e4. 409\ub144 \uc608\ub85c\ub2c8\ubaa8\ub294 \u201c\uc5ec\ub7ec \ud574 \ub3d9\uc548 \ub098\ub294 \ub85c\ub9c8\uc758 \uc8fc\uad50 \ub2e4\ub9c8\uc18c\ub97c \ub3c4\uc640 \ub3d9\u00b7\uc11c \uad50\ud68c\ud68c\uc758\uc5d0\uc11c \uadf8\uc5d0\uac8c \ubb38\uc758\ub41c \uc9c8\ubb38\ub4e4\uc5d0 \ub300\ud55c \uadf8\uc758 \ub2f5\ubcc0\uc744 \uc801\ub294 \uac83\uacfc \uac19\uc740 \uc11c\uc2e0 \uc791\uc131\uc744 \ud558\uc600\ub2e4. \u201d\uace0 \uae30\ub85d\ud558\uc600\ub2e4. \uae30\ub3c5\uad50\uac00 \ub85c\ub9c8 \uc81c\uad6d\uc758 \uad6d\uad50\ub85c \uc120\ud3ec\ub41c \uac83\uc740 \ub2e4\ub9c8\uc18c 1\uc138\uac00 \uad50\ud669\uc73c\ub85c \uc7ac\uc784\ud558\ub358(380\ub144) \uc911\uc774\uc5c8\ub2e4. \uadf8\ub9ac\uace0 \ub2e4\ub9c8\uc18c 1\uc138\uc758 \uad50\ud68c \uac1c\ud601\uc5d0 \ub530\ub77c \ub77c\ud2f4\uc5b4\uac00 \uae30\ub3c5\uad50\uc758 \uc911\uc2ec\uc801 \uc804\ub840 \uc5b8\uc5b4\uac00 \ub418\uc5c8\ub2e4. \ub610\ud55c, \ub2f9\uc2dc\uc5d0\ub294 \uc131\uacbd\uc774 \uc5ec\ub7ec \uc5ed\uc790\uc5d0 \uc758\ud574\uc11c \ubc88\uc5ed\ub418\uc5b4 \ud1b5\uc77c\uc131\uc774 \uc5c6\uc774 \ub09c\uc7a1\ud558\uace0 \uc815\ub9ac\uac00 \uc548 \ub418\uc5b4 \uc788\uc5c8\ub358 \uc0c1\ud669\uc774\uc5c8\uae30 \ub54c\ubb38\uc5d0, \ud1b5\uc77c\uc131 \uc788\ub294 \uc0c8\ub85c\uc6b4 \uacf5\uc6a9 \uc131\uacbd \ubcf8\ubb38\uc774 \uc694\uad6c\ub418\ub294 \uc0c1\ud669\uc774\uc5c8\ub2e4. \uadf8\ub9ac\ud558\uc5ec \ub2e4\ub9c8\uc18c 1\uc138\ub294 \uc608\ub85c\ub2c8\ubaa8\uc5d0\uac8c \uadf8\ub9ac\uc2a4\uc5b4 \uc2e0\uc57d\uc131\uacbd\uacfc 70\uc778\uc5ed \ub4f1\uc744 \ubc88\uc5ed \ub300\ubcf8\uc73c\ub85c \uc0bc\uc544 \uc61b \uc0c1\uace0 \ub77c\ud2f4\uc5b4\ub85c \ubc88\uc5ed\ub41c \uc131\uacbd\uc744 \ubcf4\ub2e4 \uc815\ud655\ud558\uace0 \ub2f9\uc2dc \ub300\uc911\uc774 \ub110\ub9ac \uc0ac\uc6a9\ud558\ub294 \ub300\uc911 \ub77c\ud2f4\uc5b4\ub85c \ubc88\uc5ed\ud558\ub3c4\ub85d \uc7a5\ub824\ud558\uc600\ub2e4. \uadf8 \uacb0\uacfc \ubd88\uac00\ud0c0 \uc131\uacbd \uc774 \ub098\uc624\uac8c \ub418\uc5c8\ub2e4. \uc774 \uc131\uacbd\uc740 382\ub144 \ub85c\ub9c8 \uad50\ud68c\ud68c\uc758\ub97c \ud1b5\ud574 \uacf5\uc2dd\uc801\uc778 \ub77c\ud2f4\uc5b4 \uc131\uacbd\uc73c\ub85c \uc778\uc900\uc744 \ubc1b\uc558\ub2e4. \ub2e4\ub9c8\uc18c 1\uc138\uc758 \uc0ac\ud6c4, \uc608\ub85c\ub2c8\ubaa8\ub294 \uc800\uc11c \u300a\uc704\uc778\uc804\u300b(De Viris Illustribus)\uc5d0\uc11c \ub2e4\ub9c8\uc18c 1\uc138\uc5d0 \ub300\ud574 \ub2e4\uc74c\uacfc \uac19\uc774 \uc9e7\uc740 \uae00\uc744 \uc37c\ub2e4. \u201c\uc2dc\ub97c \uc4f0\ub294 \ub370 \ud0c1\uc6d4\ud55c \uc7ac\ub2a5\uc744 \uc9c0\ub154\ub358 \uadf8\ub294 \uc544\ub984\ub2e4\uc6b4 \uc6b4\uc728\uc744 \uc9c0\ub2cc \ub9ce\uc740 \uc2dc\ub97c \uc37c\ub2e4. \uadf8\ub294 80\uc138\uc5d0 \uac00\uae4c\uc6b4 \ub098\uc774\uc5d0 \ud14c\uc624\ub3c4\uc2dc\uc6b0\uc2a4 1\uc138 \ud669\uc81c\uc758 \uce58\uc138 \ub54c \uc120\uc885\ud558\uc600\ub2e4. \u201d", "paragraph_answer": "\ub2e4\ub9c8\uc18c 1\uc138\ub294 \ube44\ubc00\ub9ac\uc5d0 \uc608\ub85c\ub2c8\ubaa8\ub97c \uc790\uc2e0\uc758 \ube44\uc11c\ub85c \uc784\uba85\ud558\uc600\ub2e4. 409\ub144 \uc608\ub85c\ub2c8\ubaa8\ub294 \u201c\uc5ec\ub7ec \ud574 \ub3d9\uc548 \ub098\ub294 \ub85c\ub9c8\uc758 \uc8fc\uad50 \ub2e4\ub9c8\uc18c\ub97c \ub3c4\uc640 \ub3d9\u00b7\uc11c \uad50\ud68c\ud68c\uc758\uc5d0\uc11c \uadf8\uc5d0\uac8c \ubb38\uc758\ub41c \uc9c8\ubb38\ub4e4\uc5d0 \ub300\ud55c \uadf8\uc758 \ub2f5\ubcc0\uc744 \uc801\ub294 \uac83\uacfc \uac19\uc740 \uc11c\uc2e0 \uc791\uc131\uc744 \ud558\uc600\ub2e4.\u201d\uace0 \uae30\ub85d\ud558\uc600\ub2e4. \uae30\ub3c5\uad50\uac00 \ub85c\ub9c8 \uc81c\uad6d\uc758 \uad6d\uad50\ub85c \uc120\ud3ec\ub41c \uac83\uc740 \ub2e4\ub9c8\uc18c 1\uc138\uac00 \uad50\ud669\uc73c\ub85c \uc7ac\uc784\ud558\ub358(380\ub144) \uc911\uc774\uc5c8\ub2e4. \uadf8\ub9ac\uace0 \ub2e4\ub9c8\uc18c 1\uc138\uc758 \uad50\ud68c \uac1c\ud601\uc5d0 \ub530\ub77c \ub77c\ud2f4\uc5b4\uac00 \uae30\ub3c5\uad50\uc758 \uc911\uc2ec\uc801 \uc804\ub840 \uc5b8\uc5b4\uac00 \ub418\uc5c8\ub2e4. \ub610\ud55c, \ub2f9\uc2dc\uc5d0\ub294 \uc131\uacbd\uc774 \uc5ec\ub7ec \uc5ed\uc790\uc5d0 \uc758\ud574\uc11c \ubc88\uc5ed\ub418\uc5b4 \ud1b5\uc77c\uc131\uc774 \uc5c6\uc774 \ub09c\uc7a1\ud558\uace0 \uc815\ub9ac\uac00 \uc548 \ub418\uc5b4 \uc788\uc5c8\ub358 \uc0c1\ud669\uc774\uc5c8\uae30 \ub54c\ubb38\uc5d0, \ud1b5\uc77c\uc131 \uc788\ub294 \uc0c8\ub85c\uc6b4 \uacf5\uc6a9 \uc131\uacbd \ubcf8\ubb38\uc774 \uc694\uad6c\ub418\ub294 \uc0c1\ud669\uc774\uc5c8\ub2e4. \uadf8\ub9ac\ud558\uc5ec \ub2e4\ub9c8\uc18c 1\uc138\ub294 \uc608\ub85c\ub2c8\ubaa8\uc5d0\uac8c \uadf8\ub9ac\uc2a4\uc5b4 \uc2e0\uc57d\uc131\uacbd\uacfc 70\uc778\uc5ed \ub4f1\uc744 \ubc88\uc5ed \ub300\ubcf8\uc73c\ub85c \uc0bc\uc544 \uc61b \uc0c1\uace0 \ub77c\ud2f4\uc5b4\ub85c \ubc88\uc5ed\ub41c \uc131\uacbd\uc744 \ubcf4\ub2e4 \uc815\ud655\ud558\uace0 \ub2f9\uc2dc \ub300\uc911\uc774 \ub110\ub9ac \uc0ac\uc6a9\ud558\ub294 \ub300\uc911 \ub77c\ud2f4\uc5b4\ub85c \ubc88\uc5ed\ud558\ub3c4\ub85d \uc7a5\ub824\ud558\uc600\ub2e4. \uadf8 \uacb0\uacfc \ubd88\uac00\ud0c0 \uc131\uacbd \uc774 \ub098\uc624\uac8c \ub418\uc5c8\ub2e4. \uc774 \uc131\uacbd\uc740 382\ub144 \ub85c\ub9c8 \uad50\ud68c\ud68c\uc758\ub97c \ud1b5\ud574 \uacf5\uc2dd\uc801\uc778 \ub77c\ud2f4\uc5b4 \uc131\uacbd\uc73c\ub85c \uc778\uc900\uc744 \ubc1b\uc558\ub2e4. \ub2e4\ub9c8\uc18c 1\uc138\uc758 \uc0ac\ud6c4, \uc608\ub85c\ub2c8\ubaa8\ub294 \uc800\uc11c \u300a\uc704\uc778\uc804\u300b(De Viris Illustribus)\uc5d0\uc11c \ub2e4\ub9c8\uc18c 1\uc138\uc5d0 \ub300\ud574 \ub2e4\uc74c\uacfc \uac19\uc774 \uc9e7\uc740 \uae00\uc744 \uc37c\ub2e4. \u201c\uc2dc\ub97c \uc4f0\ub294 \ub370 \ud0c1\uc6d4\ud55c \uc7ac\ub2a5\uc744 \uc9c0\ub154\ub358 \uadf8\ub294 \uc544\ub984\ub2e4\uc6b4 \uc6b4\uc728\uc744 \uc9c0\ub2cc \ub9ce\uc740 \uc2dc\ub97c \uc37c\ub2e4. \uadf8\ub294 80\uc138\uc5d0 \uac00\uae4c\uc6b4 \ub098\uc774\uc5d0 \ud14c\uc624\ub3c4\uc2dc\uc6b0\uc2a4 1\uc138 \ud669\uc81c\uc758 \uce58\uc138 \ub54c \uc120\uc885\ud558\uc600\ub2e4.\u201d", "sentence_answer": "\uadf8 \uacb0\uacfc \ubd88\uac00\ud0c0 \uc131\uacbd \uc774 \ub098\uc624\uac8c \ub418\uc5c8\ub2e4.", "paragraph_id": "6256664-4-1", "paragraph_question": "question: \ub300\uc911 \ub77c\ud2f4\uc5b4\ub85c \ubc88\uc5ed\ud558\uc5ec \ud0c4\uc0dd\ud558\uac8c \ub41c \uc131\uacbd \uc774\ub984\uc740?, context: \ub2e4\ub9c8\uc18c 1\uc138\ub294 \ube44\ubc00\ub9ac\uc5d0 \uc608\ub85c\ub2c8\ubaa8\ub97c \uc790\uc2e0\uc758 \ube44\uc11c\ub85c \uc784\uba85\ud558\uc600\ub2e4. 409\ub144 \uc608\ub85c\ub2c8\ubaa8\ub294 \u201c\uc5ec\ub7ec \ud574 \ub3d9\uc548 \ub098\ub294 \ub85c\ub9c8\uc758 \uc8fc\uad50 \ub2e4\ub9c8\uc18c\ub97c \ub3c4\uc640 \ub3d9\u00b7\uc11c \uad50\ud68c\ud68c\uc758\uc5d0\uc11c \uadf8\uc5d0\uac8c \ubb38\uc758\ub41c \uc9c8\ubb38\ub4e4\uc5d0 \ub300\ud55c \uadf8\uc758 \ub2f5\ubcc0\uc744 \uc801\ub294 \uac83\uacfc \uac19\uc740 \uc11c\uc2e0 \uc791\uc131\uc744 \ud558\uc600\ub2e4.\u201d\uace0 \uae30\ub85d\ud558\uc600\ub2e4. \uae30\ub3c5\uad50\uac00 \ub85c\ub9c8 \uc81c\uad6d\uc758 \uad6d\uad50\ub85c \uc120\ud3ec\ub41c \uac83\uc740 \ub2e4\ub9c8\uc18c 1\uc138\uac00 \uad50\ud669\uc73c\ub85c \uc7ac\uc784\ud558\ub358(380\ub144) \uc911\uc774\uc5c8\ub2e4. \uadf8\ub9ac\uace0 \ub2e4\ub9c8\uc18c 1\uc138\uc758 \uad50\ud68c \uac1c\ud601\uc5d0 \ub530\ub77c \ub77c\ud2f4\uc5b4\uac00 \uae30\ub3c5\uad50\uc758 \uc911\uc2ec\uc801 \uc804\ub840 \uc5b8\uc5b4\uac00 \ub418\uc5c8\ub2e4. \ub610\ud55c, \ub2f9\uc2dc\uc5d0\ub294 \uc131\uacbd\uc774 \uc5ec\ub7ec \uc5ed\uc790\uc5d0 \uc758\ud574\uc11c \ubc88\uc5ed\ub418\uc5b4 \ud1b5\uc77c\uc131\uc774 \uc5c6\uc774 \ub09c\uc7a1\ud558\uace0 \uc815\ub9ac\uac00 \uc548 \ub418\uc5b4 \uc788\uc5c8\ub358 \uc0c1\ud669\uc774\uc5c8\uae30 \ub54c\ubb38\uc5d0, \ud1b5\uc77c\uc131 \uc788\ub294 \uc0c8\ub85c\uc6b4 \uacf5\uc6a9 \uc131\uacbd \ubcf8\ubb38\uc774 \uc694\uad6c\ub418\ub294 \uc0c1\ud669\uc774\uc5c8\ub2e4. \uadf8\ub9ac\ud558\uc5ec \ub2e4\ub9c8\uc18c 1\uc138\ub294 \uc608\ub85c\ub2c8\ubaa8\uc5d0\uac8c \uadf8\ub9ac\uc2a4\uc5b4 \uc2e0\uc57d\uc131\uacbd\uacfc 70\uc778\uc5ed \ub4f1\uc744 \ubc88\uc5ed \ub300\ubcf8\uc73c\ub85c \uc0bc\uc544 \uc61b \uc0c1\uace0 \ub77c\ud2f4\uc5b4\ub85c \ubc88\uc5ed\ub41c \uc131\uacbd\uc744 \ubcf4\ub2e4 \uc815\ud655\ud558\uace0 \ub2f9\uc2dc \ub300\uc911\uc774 \ub110\ub9ac \uc0ac\uc6a9\ud558\ub294 \ub300\uc911 \ub77c\ud2f4\uc5b4\ub85c \ubc88\uc5ed\ud558\ub3c4\ub85d \uc7a5\ub824\ud558\uc600\ub2e4. \uadf8 \uacb0\uacfc \ubd88\uac00\ud0c0 \uc131\uacbd\uc774 \ub098\uc624\uac8c \ub418\uc5c8\ub2e4. \uc774 \uc131\uacbd\uc740 382\ub144 \ub85c\ub9c8 \uad50\ud68c\ud68c\uc758\ub97c \ud1b5\ud574 \uacf5\uc2dd\uc801\uc778 \ub77c\ud2f4\uc5b4 \uc131\uacbd\uc73c\ub85c \uc778\uc900\uc744 \ubc1b\uc558\ub2e4. \ub2e4\ub9c8\uc18c 1\uc138\uc758 \uc0ac\ud6c4, \uc608\ub85c\ub2c8\ubaa8\ub294 \uc800\uc11c \u300a\uc704\uc778\uc804\u300b(De Viris Illustribus)\uc5d0\uc11c \ub2e4\ub9c8\uc18c 1\uc138\uc5d0 \ub300\ud574 \ub2e4\uc74c\uacfc \uac19\uc774 \uc9e7\uc740 \uae00\uc744 \uc37c\ub2e4. \u201c\uc2dc\ub97c \uc4f0\ub294 \ub370 \ud0c1\uc6d4\ud55c \uc7ac\ub2a5\uc744 \uc9c0\ub154\ub358 \uadf8\ub294 \uc544\ub984\ub2e4\uc6b4 \uc6b4\uc728\uc744 \uc9c0\ub2cc \ub9ce\uc740 \uc2dc\ub97c \uc37c\ub2e4. \uadf8\ub294 80\uc138\uc5d0 \uac00\uae4c\uc6b4 \ub098\uc774\uc5d0 \ud14c\uc624\ub3c4\uc2dc\uc6b0\uc2a4 1\uc138 \ud669\uc81c\uc758 \uce58\uc138 \ub54c \uc120\uc885\ud558\uc600\ub2e4.\u201d"} {"question": "single ladies\ub294 2009\ub144 \ud2f4 \ucd08\uc774\uc2a4 \uc5b4\uc6cc\ub4dc\uc5d0\uc11c \uc5b4\ub5a4 \uc0c1\uc744 \uc218\uc0c1\ud588\ub294\uac00?", "paragraph": "\u3008Single Ladies\u3009\ub294 \uc81c52\ud68c \uadf8\ub798\ubbf8 \uc0c1\uc5d0\uc11c \uc62c\ud574\uc758 \ub178\ub798, \ucd5c\uc6b0\uc218 \uc54c\uc564\ube44 \ub178\ub798 \uadf8\ub9ac\uace0 \uc62c\ud574\uc758 \uc5ec\uc790 \uc54c\uc564\ube44 \ubcf4\uceec \ud37c\ud3ec\uba3c\uc2a4 \uc0c1\uae4c\uc9c0 \uc138 \uac1c\uc758 \ubd80\ubb38\uc744 \uc11d\uad8c\ud588\ub2e4. 2009 \ud0a4\ub4dc \ucd08\uc774\uc2a4 \uc5b4\uc6cc\uc988\uc5d0\uc11c \ud398\ubc84\ub9ac\ud2f0 \uc1a1\uc744 \uc218\uc0c1\ubc1b\uc558\uace0, 2009 \uc18c\uc6b8 \ud2b8\ub808\uc778 \ubba4\uc9c1 \uc5b4\uc6cc\ub4dc\uc5d0\uc11c \uc62c\ud574\uc758 \ub178\ub798\uc0c1, \uadf8\ub9ac\uace0 2009 \ud2f4 \ucd08\uc774\uc2a4 \uc5b4\uc6cc\ub4dc\uc5d0\uc11c\ub294 \ucd5c\uc6b0\uc218 \uc54c\uc564\ube44 \ub178\ub798\ub97c \uc218\uc0c1\ud588\ub2e4. 2009\ub144 \ubbf8\uad6d\uc74c\uc545\uc800\uc791\uad8c\ud611\ud68c (ASCAP)\uc758 27\ud68c ASCAP \ud31d \ubba4\uc9c1 \uc5b4\uc6cc\ub4dc\uc5d0\uc11c\ub294 \uc62c\ud574\uc758 \ub178\ub798 \uc911 \ud558\ub098\ub85c \uc120\uc815\ub418\uc5c8\ub2e4. 2009\ub144 NAACP \uc774\ubbf8\uc9c0 \uc5b4\uc6cc\ub4dc\uc5d0\uc11c \ucd5c\uc6b0\uc218 \ub178\ub798 \ubd80\ubb38\uc5d0, 2009 \ud37c\ubbf8\uc6b0\uc2a4 \uc624\uc608! \uc5b4\uc6cc\ub4dc\uc5d0\uc11c\ub294 \"\uc62c\ud574\uc758 \ub808\ucf54\ub4dc\" \ubd80\ubb38 \ud6c4\ubcf4\ub85c \uc9c0\uba85\ub418\uc5c8\ub2e4. \ub610\ud55c 2009 \uc18c\uc6b8 \ud2b8\ub808\uc778 \ubba4\uc9c1 \uc5b4\uc6cc\ub4dc\uc5d0\uc11c \uc62c\ud574\uc758 \ub808\ucf54\ub4dc, 2009 BET \uc5b4\uc6cc\ub4dc\uc758 \ubdf0\uc5b4\uc2a4 \ucd08\uc774\uc2a4\uc0c1, 2009 \uad6d\uc81c \ub304\uc2a4 \ubba4\uc9c1 \uc5b4\uc6cc\ub4dc\uc758 \ucd5c\uc6b0\uc218 \uc54c\uc564\ube44/\uc5b4\ubc18 \ub304\uc2a4 \ud2b8\ub799, 2010 \uc6d4\ub4dc \ubba4\uc9c1 \uc5b4\uc6cc\ub4dc\uc758 \ucd5c\uc6b0\uc218 \uc6d4\ub4dc \uc2f1\uae00 \ubd80\ubb38 \ud6c4\ubcf4\ub85c \uc9c0\uba85\ub418\uc5c8\ub2e4.", "answer": "\ucd5c\uc6b0\uc218 \uc54c\uc564\ube44 \ub178\ub798", "sentence": "\u3008Single Ladies\u3009\ub294 \uc81c52\ud68c \uadf8\ub798\ubbf8 \uc0c1\uc5d0\uc11c \uc62c\ud574\uc758 \ub178\ub798, \ucd5c\uc6b0\uc218 \uc54c\uc564\ube44 \ub178\ub798 \uadf8\ub9ac\uace0 \uc62c\ud574\uc758 \uc5ec\uc790 \uc54c\uc564\ube44 \ubcf4\uceec \ud37c\ud3ec\uba3c\uc2a4 \uc0c1\uae4c\uc9c0 \uc138 \uac1c\uc758 \ubd80\ubb38\uc744 \uc11d\uad8c\ud588\ub2e4.", "paragraph_sentence": " \u3008Single Ladies\u3009\ub294 \uc81c52\ud68c \uadf8\ub798\ubbf8 \uc0c1\uc5d0\uc11c \uc62c\ud574\uc758 \ub178\ub798, \ucd5c\uc6b0\uc218 \uc54c\uc564\ube44 \ub178\ub798 \uadf8\ub9ac\uace0 \uc62c\ud574\uc758 \uc5ec\uc790 \uc54c\uc564\ube44 \ubcf4\uceec \ud37c\ud3ec\uba3c\uc2a4 \uc0c1\uae4c\uc9c0 \uc138 \uac1c\uc758 \ubd80\ubb38\uc744 \uc11d\uad8c\ud588\ub2e4. 2009 \ud0a4\ub4dc \ucd08\uc774\uc2a4 \uc5b4\uc6cc\uc988\uc5d0\uc11c \ud398\ubc84\ub9ac\ud2f0 \uc1a1\uc744 \uc218\uc0c1\ubc1b\uc558\uace0, 2009 \uc18c\uc6b8 \ud2b8\ub808\uc778 \ubba4\uc9c1 \uc5b4\uc6cc\ub4dc\uc5d0\uc11c \uc62c\ud574\uc758 \ub178\ub798\uc0c1, \uadf8\ub9ac\uace0 2009 \ud2f4 \ucd08\uc774\uc2a4 \uc5b4\uc6cc\ub4dc\uc5d0\uc11c\ub294 \ucd5c\uc6b0\uc218 \uc54c\uc564\ube44 \ub178\ub798\ub97c \uc218\uc0c1\ud588\ub2e4. 2009\ub144 \ubbf8\uad6d\uc74c\uc545\uc800\uc791\uad8c\ud611\ud68c (ASCAP)\uc758 27\ud68c ASCAP \ud31d \ubba4\uc9c1 \uc5b4\uc6cc\ub4dc\uc5d0\uc11c\ub294 \uc62c\ud574\uc758 \ub178\ub798 \uc911 \ud558\ub098\ub85c \uc120\uc815\ub418\uc5c8\ub2e4. 2009\ub144 NAACP \uc774\ubbf8\uc9c0 \uc5b4\uc6cc\ub4dc\uc5d0\uc11c \ucd5c\uc6b0\uc218 \ub178\ub798 \ubd80\ubb38\uc5d0, 2009 \ud37c\ubbf8\uc6b0\uc2a4 \uc624\uc608! \uc5b4\uc6cc\ub4dc\uc5d0\uc11c\ub294 \"\uc62c\ud574\uc758 \ub808\ucf54\ub4dc\" \ubd80\ubb38 \ud6c4\ubcf4\ub85c \uc9c0\uba85\ub418\uc5c8\ub2e4. \ub610\ud55c 2009 \uc18c\uc6b8 \ud2b8\ub808\uc778 \ubba4\uc9c1 \uc5b4\uc6cc\ub4dc\uc5d0\uc11c \uc62c\ud574\uc758 \ub808\ucf54\ub4dc, 2009 BET \uc5b4\uc6cc\ub4dc\uc758 \ubdf0\uc5b4\uc2a4 \ucd08\uc774\uc2a4\uc0c1, 2009 \uad6d\uc81c \ub304\uc2a4 \ubba4\uc9c1 \uc5b4\uc6cc\ub4dc\uc758 \ucd5c\uc6b0\uc218 \uc54c\uc564\ube44/\uc5b4\ubc18 \ub304\uc2a4 \ud2b8\ub799, 2010 \uc6d4\ub4dc \ubba4\uc9c1 \uc5b4\uc6cc\ub4dc\uc758 \ucd5c\uc6b0\uc218 \uc6d4\ub4dc \uc2f1\uae00 \ubd80\ubb38 \ud6c4\ubcf4\ub85c \uc9c0\uba85\ub418\uc5c8\ub2e4.", "paragraph_answer": "\u3008Single Ladies\u3009\ub294 \uc81c52\ud68c \uadf8\ub798\ubbf8 \uc0c1\uc5d0\uc11c \uc62c\ud574\uc758 \ub178\ub798, \ucd5c\uc6b0\uc218 \uc54c\uc564\ube44 \ub178\ub798 \uadf8\ub9ac\uace0 \uc62c\ud574\uc758 \uc5ec\uc790 \uc54c\uc564\ube44 \ubcf4\uceec \ud37c\ud3ec\uba3c\uc2a4 \uc0c1\uae4c\uc9c0 \uc138 \uac1c\uc758 \ubd80\ubb38\uc744 \uc11d\uad8c\ud588\ub2e4. 2009 \ud0a4\ub4dc \ucd08\uc774\uc2a4 \uc5b4\uc6cc\uc988\uc5d0\uc11c \ud398\ubc84\ub9ac\ud2f0 \uc1a1\uc744 \uc218\uc0c1\ubc1b\uc558\uace0, 2009 \uc18c\uc6b8 \ud2b8\ub808\uc778 \ubba4\uc9c1 \uc5b4\uc6cc\ub4dc\uc5d0\uc11c \uc62c\ud574\uc758 \ub178\ub798\uc0c1, \uadf8\ub9ac\uace0 2009 \ud2f4 \ucd08\uc774\uc2a4 \uc5b4\uc6cc\ub4dc\uc5d0\uc11c\ub294 \ucd5c\uc6b0\uc218 \uc54c\uc564\ube44 \ub178\ub798\ub97c \uc218\uc0c1\ud588\ub2e4. 2009\ub144 \ubbf8\uad6d\uc74c\uc545\uc800\uc791\uad8c\ud611\ud68c (ASCAP)\uc758 27\ud68c ASCAP \ud31d \ubba4\uc9c1 \uc5b4\uc6cc\ub4dc\uc5d0\uc11c\ub294 \uc62c\ud574\uc758 \ub178\ub798 \uc911 \ud558\ub098\ub85c \uc120\uc815\ub418\uc5c8\ub2e4. 2009\ub144 NAACP \uc774\ubbf8\uc9c0 \uc5b4\uc6cc\ub4dc\uc5d0\uc11c \ucd5c\uc6b0\uc218 \ub178\ub798 \ubd80\ubb38\uc5d0, 2009 \ud37c\ubbf8\uc6b0\uc2a4 \uc624\uc608! \uc5b4\uc6cc\ub4dc\uc5d0\uc11c\ub294 \"\uc62c\ud574\uc758 \ub808\ucf54\ub4dc\" \ubd80\ubb38 \ud6c4\ubcf4\ub85c \uc9c0\uba85\ub418\uc5c8\ub2e4. \ub610\ud55c 2009 \uc18c\uc6b8 \ud2b8\ub808\uc778 \ubba4\uc9c1 \uc5b4\uc6cc\ub4dc\uc5d0\uc11c \uc62c\ud574\uc758 \ub808\ucf54\ub4dc, 2009 BET \uc5b4\uc6cc\ub4dc\uc758 \ubdf0\uc5b4\uc2a4 \ucd08\uc774\uc2a4\uc0c1, 2009 \uad6d\uc81c \ub304\uc2a4 \ubba4\uc9c1 \uc5b4\uc6cc\ub4dc\uc758 \ucd5c\uc6b0\uc218 \uc54c\uc564\ube44/\uc5b4\ubc18 \ub304\uc2a4 \ud2b8\ub799, 2010 \uc6d4\ub4dc \ubba4\uc9c1 \uc5b4\uc6cc\ub4dc\uc758 \ucd5c\uc6b0\uc218 \uc6d4\ub4dc \uc2f1\uae00 \ubd80\ubb38 \ud6c4\ubcf4\ub85c \uc9c0\uba85\ub418\uc5c8\ub2e4.", "sentence_answer": "\u3008Single Ladies\u3009\ub294 \uc81c52\ud68c \uadf8\ub798\ubbf8 \uc0c1\uc5d0\uc11c \uc62c\ud574\uc758 \ub178\ub798, \ucd5c\uc6b0\uc218 \uc54c\uc564\ube44 \ub178\ub798 \uadf8\ub9ac\uace0 \uc62c\ud574\uc758 \uc5ec\uc790 \uc54c\uc564\ube44 \ubcf4\uceec \ud37c\ud3ec\uba3c\uc2a4 \uc0c1\uae4c\uc9c0 \uc138 \uac1c\uc758 \ubd80\ubb38\uc744 \uc11d\uad8c\ud588\ub2e4.", "paragraph_id": "6488992-13-1", "paragraph_question": "question: single ladies\ub294 2009\ub144 \ud2f4 \ucd08\uc774\uc2a4 \uc5b4\uc6cc\ub4dc\uc5d0\uc11c \uc5b4\ub5a4 \uc0c1\uc744 \uc218\uc0c1\ud588\ub294\uac00?, context: \u3008Single Ladies\u3009\ub294 \uc81c52\ud68c \uadf8\ub798\ubbf8 \uc0c1\uc5d0\uc11c \uc62c\ud574\uc758 \ub178\ub798, \ucd5c\uc6b0\uc218 \uc54c\uc564\ube44 \ub178\ub798 \uadf8\ub9ac\uace0 \uc62c\ud574\uc758 \uc5ec\uc790 \uc54c\uc564\ube44 \ubcf4\uceec \ud37c\ud3ec\uba3c\uc2a4 \uc0c1\uae4c\uc9c0 \uc138 \uac1c\uc758 \ubd80\ubb38\uc744 \uc11d\uad8c\ud588\ub2e4. 2009 \ud0a4\ub4dc \ucd08\uc774\uc2a4 \uc5b4\uc6cc\uc988\uc5d0\uc11c \ud398\ubc84\ub9ac\ud2f0 \uc1a1\uc744 \uc218\uc0c1\ubc1b\uc558\uace0, 2009 \uc18c\uc6b8 \ud2b8\ub808\uc778 \ubba4\uc9c1 \uc5b4\uc6cc\ub4dc\uc5d0\uc11c \uc62c\ud574\uc758 \ub178\ub798\uc0c1, \uadf8\ub9ac\uace0 2009 \ud2f4 \ucd08\uc774\uc2a4 \uc5b4\uc6cc\ub4dc\uc5d0\uc11c\ub294 \ucd5c\uc6b0\uc218 \uc54c\uc564\ube44 \ub178\ub798\ub97c \uc218\uc0c1\ud588\ub2e4. 2009\ub144 \ubbf8\uad6d\uc74c\uc545\uc800\uc791\uad8c\ud611\ud68c (ASCAP)\uc758 27\ud68c ASCAP \ud31d \ubba4\uc9c1 \uc5b4\uc6cc\ub4dc\uc5d0\uc11c\ub294 \uc62c\ud574\uc758 \ub178\ub798 \uc911 \ud558\ub098\ub85c \uc120\uc815\ub418\uc5c8\ub2e4. 2009\ub144 NAACP \uc774\ubbf8\uc9c0 \uc5b4\uc6cc\ub4dc\uc5d0\uc11c \ucd5c\uc6b0\uc218 \ub178\ub798 \ubd80\ubb38\uc5d0, 2009 \ud37c\ubbf8\uc6b0\uc2a4 \uc624\uc608! \uc5b4\uc6cc\ub4dc\uc5d0\uc11c\ub294 \"\uc62c\ud574\uc758 \ub808\ucf54\ub4dc\" \ubd80\ubb38 \ud6c4\ubcf4\ub85c \uc9c0\uba85\ub418\uc5c8\ub2e4. \ub610\ud55c 2009 \uc18c\uc6b8 \ud2b8\ub808\uc778 \ubba4\uc9c1 \uc5b4\uc6cc\ub4dc\uc5d0\uc11c \uc62c\ud574\uc758 \ub808\ucf54\ub4dc, 2009 BET \uc5b4\uc6cc\ub4dc\uc758 \ubdf0\uc5b4\uc2a4 \ucd08\uc774\uc2a4\uc0c1, 2009 \uad6d\uc81c \ub304\uc2a4 \ubba4\uc9c1 \uc5b4\uc6cc\ub4dc\uc758 \ucd5c\uc6b0\uc218 \uc54c\uc564\ube44/\uc5b4\ubc18 \ub304\uc2a4 \ud2b8\ub799, 2010 \uc6d4\ub4dc \ubba4\uc9c1 \uc5b4\uc6cc\ub4dc\uc758 \ucd5c\uc6b0\uc218 \uc6d4\ub4dc \uc2f1\uae00 \ubd80\ubb38 \ud6c4\ubcf4\ub85c \uc9c0\uba85\ub418\uc5c8\ub2e4."} {"question": "\uc2a4\uc704\ud504\ud2b8\uc758 \ub450 \ubc88\uc9f8 \uc2f1\uae00\uc740 \ubb34\uc5c7\uc778\uac00\uc694?", "paragraph": "\uc2a4\uc704\ud504\ud2b8\uc758 \ub450 \ubc88\uc9f8 \uc815\uaddc \uc74c\ubc18 Fearless\ub294 2008\ub144 11\uc6d4 11\uc77c \ubbf8\uad6d\uc5d0\uc11c \ubc1c\ub9e4\ub418\uc5c8\ub2e4. \uc2a4\uc704\ud504\ud2b8\ub294 \ub450 \uc7a5\uc758 \uc2f1\uae00\uc744 \ud3ec\ud568\ud574 \uc568\ubc94\uc758 \ub178\ub798 \uc77c\uacf1 \uace1\uc744 \ud63c\uc790 \uc791\uace1\ud588\uace0, \ub098\uba38\uc9c0 \uc5ec\uc12f \uace1\uc740 \ub9ac\uc988 \ub85c\uc988, \uc874 \ub9ac\uce58, \ucf5c\ube44 \uce74\ub808\uc774, \ud790\ub7ec\ub9ac \ub9b0\uc9c0\uc640 \ud568\uaed8 \uacf5\ub3d9 \uc791\uace1\ud588\ub2e4. \ub610\ud55c \ub124\uc774\uc2a8 \ucc44\ud504\uba3c\uacfc \ud568\uaed8 \uacf5\ub3d9 \ud504\ub85c\ub4c0\uc11c\ub97c \ub9e1\uc558\ub2e4. Fearless\ub294 \uccab \uc8fc 592,304 \uc7a5\uc744 \ud314\uc544 \ube4c\ubcf4\ub4dc 200 1\uc704\ub85c \ub370\ubdd4\ud588\ub2e4. \ub9ac\ub4dc \uc2f1\uae00 \"Love Story\"\ub294 \uac70\uc758 \ubaa8\ub4e0 \uad6d\uac00\uc758 \ud31d \ucc28\ud2b8\uc5d0\uc11c \uc131\uacf5\ud588\ub2e4. \uc2f1\uae00 \uacf5\uac1c \uccab \ub0a0 \ub514\uc9c0\ud138 \ub2e4\uc6b4\ub85c\ub4dc \ud69f\uc218 129,000\uac74\uc744 \ub118\uacbc\ub2e4. \uc774\ud6c4 \ub450 \ubc88\uc9f8 \uc2f1\uae00 \"White Horse\"\ub97c 2008\ub144 12\uc6d4 9\uc77c \ubc1c\ub9e4\ud558\uc600\uace0 2009\ub144 2\uc6d4 7\uc77c CMT\ub97c \ud1b5\ud558\uc5ec \ubba4\uc9c1 \ube44\ub514\uc624\ub97c \uacf5\uac1c\ud558\uc600\ub2e4. 2008\ub144 \uc870 \uc870\ub098\uc2a4\uc640 \uc0ac\uadc0\uace0 \ud5e4\uc5b4\uc9c4 \ud6c4 \"Forever & Always\"\ub97c \uc791\uace1\ud588\ub2e4. \uc774 \uc678\uc5d0 19\uace1\uc758 \ud2b8\ub799\ub9ac\uc2a4\ud2b8\ub4e4 \uc911 13\uace1\uc774 \ube4c\ubcf4\ub4dc \ucc28\ud2b8\uc5d0 40\uc704\uad8c\uc548\uc5d0 \uc9c4\uc785\ud558\uc600\ub2e4. \ub610\ud55c \ub2e8 2\uac1c\uc6d4\ub9cc\uc5d0 Fearless\uc758 \uc74c\ubc18\ud310\ub9e4\ub7c9\uc740 200\ub9cc\uc7a5\uc774 \ub118\uc5c8\uc73c\uba70 2008\ub144 \uac00\uc7a5 \ub9ce\uc774 \ud310 \uc74c\ubc18 3\uc704\uc5d0 \uc624\ub974\uae30\ub3c4 \ud558\uc600\uc73c\uba70 \ub610 \ud558\ub098\uc758 \uc74c\ubc18\uc744 10\uad8c \uc548\uc5d0 \uc9c4\uc785\uc2dc\ucf1c \ucd1d \uc74c\ubc18\ud310\ub9e4\ub7c9\uc774 400\ub9cc\uc7a5\uc744 \ub118\uc73c\uba70 2008\ub144 \uac00\uc7a5 \ub9ce\uc774 \ud310 \uac00\uc218\uac00 \ub418\uc5c8\ub2e4.", "answer": "White Horse", "sentence": "\uc774\ud6c4 \ub450 \ubc88\uc9f8 \uc2f1\uae00 \" White Horse \"\ub97c 2008\ub144 12\uc6d4 9\uc77c \ubc1c\ub9e4\ud558\uc600\uace0 2009\ub144 2\uc6d4 7\uc77c CMT\ub97c \ud1b5\ud558\uc5ec \ubba4\uc9c1 \ube44\ub514\uc624\ub97c \uacf5\uac1c\ud558\uc600\ub2e4.", "paragraph_sentence": "\uc2a4\uc704\ud504\ud2b8\uc758 \ub450 \ubc88\uc9f8 \uc815\uaddc \uc74c\ubc18 Fearless\ub294 2008\ub144 11\uc6d4 11\uc77c \ubbf8\uad6d\uc5d0\uc11c \ubc1c\ub9e4\ub418\uc5c8\ub2e4. \uc2a4\uc704\ud504\ud2b8\ub294 \ub450 \uc7a5\uc758 \uc2f1\uae00\uc744 \ud3ec\ud568\ud574 \uc568\ubc94\uc758 \ub178\ub798 \uc77c\uacf1 \uace1\uc744 \ud63c\uc790 \uc791\uace1\ud588\uace0, \ub098\uba38\uc9c0 \uc5ec\uc12f \uace1\uc740 \ub9ac\uc988 \ub85c\uc988, \uc874 \ub9ac\uce58, \ucf5c\ube44 \uce74\ub808\uc774, \ud790\ub7ec\ub9ac \ub9b0\uc9c0\uc640 \ud568\uaed8 \uacf5\ub3d9 \uc791\uace1\ud588\ub2e4. \ub610\ud55c \ub124\uc774\uc2a8 \ucc44\ud504\uba3c\uacfc \ud568\uaed8 \uacf5\ub3d9 \ud504\ub85c\ub4c0\uc11c\ub97c \ub9e1\uc558\ub2e4. Fearless\ub294 \uccab \uc8fc 592,304 \uc7a5\uc744 \ud314\uc544 \ube4c\ubcf4\ub4dc 200 1\uc704\ub85c \ub370\ubdd4\ud588\ub2e4. \ub9ac\ub4dc \uc2f1\uae00 \"Love Story\"\ub294 \uac70\uc758 \ubaa8\ub4e0 \uad6d\uac00\uc758 \ud31d \ucc28\ud2b8\uc5d0\uc11c \uc131\uacf5\ud588\ub2e4. \uc2f1\uae00 \uacf5\uac1c \uccab \ub0a0 \ub514\uc9c0\ud138 \ub2e4\uc6b4\ub85c\ub4dc \ud69f\uc218 129,000\uac74\uc744 \ub118\uacbc\ub2e4. \uc774\ud6c4 \ub450 \ubc88\uc9f8 \uc2f1\uae00 \" White Horse \"\ub97c 2008\ub144 12\uc6d4 9\uc77c \ubc1c\ub9e4\ud558\uc600\uace0 2009\ub144 2\uc6d4 7\uc77c CMT\ub97c \ud1b5\ud558\uc5ec \ubba4\uc9c1 \ube44\ub514\uc624\ub97c \uacf5\uac1c\ud558\uc600\ub2e4. 2008\ub144 \uc870 \uc870\ub098\uc2a4\uc640 \uc0ac\uadc0\uace0 \ud5e4\uc5b4\uc9c4 \ud6c4 \"Forever & Always\"\ub97c \uc791\uace1\ud588\ub2e4. \uc774 \uc678\uc5d0 19\uace1\uc758 \ud2b8\ub799\ub9ac\uc2a4\ud2b8\ub4e4 \uc911 13\uace1\uc774 \ube4c\ubcf4\ub4dc \ucc28\ud2b8\uc5d0 40\uc704\uad8c\uc548\uc5d0 \uc9c4\uc785\ud558\uc600\ub2e4. \ub610\ud55c \ub2e8 2\uac1c\uc6d4\ub9cc\uc5d0 Fearless\uc758 \uc74c\ubc18\ud310\ub9e4\ub7c9\uc740 200\ub9cc\uc7a5\uc774 \ub118\uc5c8\uc73c\uba70 2008\ub144 \uac00\uc7a5 \ub9ce\uc774 \ud310 \uc74c\ubc18 3\uc704\uc5d0 \uc624\ub974\uae30\ub3c4 \ud558\uc600\uc73c\uba70 \ub610 \ud558\ub098\uc758 \uc74c\ubc18\uc744 10\uad8c \uc548\uc5d0 \uc9c4\uc785\uc2dc\ucf1c \ucd1d \uc74c\ubc18\ud310\ub9e4\ub7c9\uc774 400\ub9cc\uc7a5\uc744 \ub118\uc73c\uba70 2008\ub144 \uac00\uc7a5 \ub9ce\uc774 \ud310 \uac00\uc218\uac00 \ub418\uc5c8\ub2e4.", "paragraph_answer": "\uc2a4\uc704\ud504\ud2b8\uc758 \ub450 \ubc88\uc9f8 \uc815\uaddc \uc74c\ubc18 Fearless\ub294 2008\ub144 11\uc6d4 11\uc77c \ubbf8\uad6d\uc5d0\uc11c \ubc1c\ub9e4\ub418\uc5c8\ub2e4. \uc2a4\uc704\ud504\ud2b8\ub294 \ub450 \uc7a5\uc758 \uc2f1\uae00\uc744 \ud3ec\ud568\ud574 \uc568\ubc94\uc758 \ub178\ub798 \uc77c\uacf1 \uace1\uc744 \ud63c\uc790 \uc791\uace1\ud588\uace0, \ub098\uba38\uc9c0 \uc5ec\uc12f \uace1\uc740 \ub9ac\uc988 \ub85c\uc988, \uc874 \ub9ac\uce58, \ucf5c\ube44 \uce74\ub808\uc774, \ud790\ub7ec\ub9ac \ub9b0\uc9c0\uc640 \ud568\uaed8 \uacf5\ub3d9 \uc791\uace1\ud588\ub2e4. \ub610\ud55c \ub124\uc774\uc2a8 \ucc44\ud504\uba3c\uacfc \ud568\uaed8 \uacf5\ub3d9 \ud504\ub85c\ub4c0\uc11c\ub97c \ub9e1\uc558\ub2e4. Fearless\ub294 \uccab \uc8fc 592,304 \uc7a5\uc744 \ud314\uc544 \ube4c\ubcf4\ub4dc 200 1\uc704\ub85c \ub370\ubdd4\ud588\ub2e4. \ub9ac\ub4dc \uc2f1\uae00 \"Love Story\"\ub294 \uac70\uc758 \ubaa8\ub4e0 \uad6d\uac00\uc758 \ud31d \ucc28\ud2b8\uc5d0\uc11c \uc131\uacf5\ud588\ub2e4. \uc2f1\uae00 \uacf5\uac1c \uccab \ub0a0 \ub514\uc9c0\ud138 \ub2e4\uc6b4\ub85c\ub4dc \ud69f\uc218 129,000\uac74\uc744 \ub118\uacbc\ub2e4. \uc774\ud6c4 \ub450 \ubc88\uc9f8 \uc2f1\uae00 \" White Horse \"\ub97c 2008\ub144 12\uc6d4 9\uc77c \ubc1c\ub9e4\ud558\uc600\uace0 2009\ub144 2\uc6d4 7\uc77c CMT\ub97c \ud1b5\ud558\uc5ec \ubba4\uc9c1 \ube44\ub514\uc624\ub97c \uacf5\uac1c\ud558\uc600\ub2e4. 2008\ub144 \uc870 \uc870\ub098\uc2a4\uc640 \uc0ac\uadc0\uace0 \ud5e4\uc5b4\uc9c4 \ud6c4 \"Forever & Always\"\ub97c \uc791\uace1\ud588\ub2e4. \uc774 \uc678\uc5d0 19\uace1\uc758 \ud2b8\ub799\ub9ac\uc2a4\ud2b8\ub4e4 \uc911 13\uace1\uc774 \ube4c\ubcf4\ub4dc \ucc28\ud2b8\uc5d0 40\uc704\uad8c\uc548\uc5d0 \uc9c4\uc785\ud558\uc600\ub2e4. \ub610\ud55c \ub2e8 2\uac1c\uc6d4\ub9cc\uc5d0 Fearless\uc758 \uc74c\ubc18\ud310\ub9e4\ub7c9\uc740 200\ub9cc\uc7a5\uc774 \ub118\uc5c8\uc73c\uba70 2008\ub144 \uac00\uc7a5 \ub9ce\uc774 \ud310 \uc74c\ubc18 3\uc704\uc5d0 \uc624\ub974\uae30\ub3c4 \ud558\uc600\uc73c\uba70 \ub610 \ud558\ub098\uc758 \uc74c\ubc18\uc744 10\uad8c \uc548\uc5d0 \uc9c4\uc785\uc2dc\ucf1c \ucd1d \uc74c\ubc18\ud310\ub9e4\ub7c9\uc774 400\ub9cc\uc7a5\uc744 \ub118\uc73c\uba70 2008\ub144 \uac00\uc7a5 \ub9ce\uc774 \ud310 \uac00\uc218\uac00 \ub418\uc5c8\ub2e4.", "sentence_answer": "\uc774\ud6c4 \ub450 \ubc88\uc9f8 \uc2f1\uae00 \" White Horse \"\ub97c 2008\ub144 12\uc6d4 9\uc77c \ubc1c\ub9e4\ud558\uc600\uace0 2009\ub144 2\uc6d4 7\uc77c CMT\ub97c \ud1b5\ud558\uc5ec \ubba4\uc9c1 \ube44\ub514\uc624\ub97c \uacf5\uac1c\ud558\uc600\ub2e4.", "paragraph_id": "6216459-5-0", "paragraph_question": "question: \uc2a4\uc704\ud504\ud2b8\uc758 \ub450 \ubc88\uc9f8 \uc2f1\uae00\uc740 \ubb34\uc5c7\uc778\uac00\uc694?, context: \uc2a4\uc704\ud504\ud2b8\uc758 \ub450 \ubc88\uc9f8 \uc815\uaddc \uc74c\ubc18 Fearless\ub294 2008\ub144 11\uc6d4 11\uc77c \ubbf8\uad6d\uc5d0\uc11c \ubc1c\ub9e4\ub418\uc5c8\ub2e4. \uc2a4\uc704\ud504\ud2b8\ub294 \ub450 \uc7a5\uc758 \uc2f1\uae00\uc744 \ud3ec\ud568\ud574 \uc568\ubc94\uc758 \ub178\ub798 \uc77c\uacf1 \uace1\uc744 \ud63c\uc790 \uc791\uace1\ud588\uace0, \ub098\uba38\uc9c0 \uc5ec\uc12f \uace1\uc740 \ub9ac\uc988 \ub85c\uc988, \uc874 \ub9ac\uce58, \ucf5c\ube44 \uce74\ub808\uc774, \ud790\ub7ec\ub9ac \ub9b0\uc9c0\uc640 \ud568\uaed8 \uacf5\ub3d9 \uc791\uace1\ud588\ub2e4. \ub610\ud55c \ub124\uc774\uc2a8 \ucc44\ud504\uba3c\uacfc \ud568\uaed8 \uacf5\ub3d9 \ud504\ub85c\ub4c0\uc11c\ub97c \ub9e1\uc558\ub2e4. Fearless\ub294 \uccab \uc8fc 592,304 \uc7a5\uc744 \ud314\uc544 \ube4c\ubcf4\ub4dc 200 1\uc704\ub85c \ub370\ubdd4\ud588\ub2e4. \ub9ac\ub4dc \uc2f1\uae00 \"Love Story\"\ub294 \uac70\uc758 \ubaa8\ub4e0 \uad6d\uac00\uc758 \ud31d \ucc28\ud2b8\uc5d0\uc11c \uc131\uacf5\ud588\ub2e4. \uc2f1\uae00 \uacf5\uac1c \uccab \ub0a0 \ub514\uc9c0\ud138 \ub2e4\uc6b4\ub85c\ub4dc \ud69f\uc218 129,000\uac74\uc744 \ub118\uacbc\ub2e4. \uc774\ud6c4 \ub450 \ubc88\uc9f8 \uc2f1\uae00 \"White Horse\"\ub97c 2008\ub144 12\uc6d4 9\uc77c \ubc1c\ub9e4\ud558\uc600\uace0 2009\ub144 2\uc6d4 7\uc77c CMT\ub97c \ud1b5\ud558\uc5ec \ubba4\uc9c1 \ube44\ub514\uc624\ub97c \uacf5\uac1c\ud558\uc600\ub2e4. 2008\ub144 \uc870 \uc870\ub098\uc2a4\uc640 \uc0ac\uadc0\uace0 \ud5e4\uc5b4\uc9c4 \ud6c4 \"Forever & Always\"\ub97c \uc791\uace1\ud588\ub2e4. \uc774 \uc678\uc5d0 19\uace1\uc758 \ud2b8\ub799\ub9ac\uc2a4\ud2b8\ub4e4 \uc911 13\uace1\uc774 \ube4c\ubcf4\ub4dc \ucc28\ud2b8\uc5d0 40\uc704\uad8c\uc548\uc5d0 \uc9c4\uc785\ud558\uc600\ub2e4. \ub610\ud55c \ub2e8 2\uac1c\uc6d4\ub9cc\uc5d0 Fearless\uc758 \uc74c\ubc18\ud310\ub9e4\ub7c9\uc740 200\ub9cc\uc7a5\uc774 \ub118\uc5c8\uc73c\uba70 2008\ub144 \uac00\uc7a5 \ub9ce\uc774 \ud310 \uc74c\ubc18 3\uc704\uc5d0 \uc624\ub974\uae30\ub3c4 \ud558\uc600\uc73c\uba70 \ub610 \ud558\ub098\uc758 \uc74c\ubc18\uc744 10\uad8c \uc548\uc5d0 \uc9c4\uc785\uc2dc\ucf1c \ucd1d \uc74c\ubc18\ud310\ub9e4\ub7c9\uc774 400\ub9cc\uc7a5\uc744 \ub118\uc73c\uba70 2008\ub144 \uac00\uc7a5 \ub9ce\uc774 \ud310 \uac00\uc218\uac00 \ub418\uc5c8\ub2e4."} {"question": "\uc138\uc6d4\ud638 \uc0ac\uace0 \uc0dd\uc874\uc790\uc758 \uc218\ub294 \uba87\uc778\uac00?", "paragraph": "\uc138\uc6d4\ud638 \uce68\ubab0 \uc0ac\uace0(\uc601\uc5b4: Sinking of MV Sewol)\ub294 2014\ub144 4\uc6d4 16\uc77c \uc624\uc804 8\uc2dc 50\ubd84\uacbd \ub300\ud55c\ubbfc\uad6d \uc804\ub77c\ub0a8\ub3c4 \uc9c4\ub3c4\uad70 \uc870\ub3c4\uba74 \ubd80\uadfc \ud574\uc0c1\uc5d0\uc11c \uc5ec\uac1d\uc120 \uc138\uc6d4\ud638\uac00 \uc804\ubcf5\ub418\uc5b4 \uce68\ubab0\ud55c \uc0ac\uace0\uc774\ub2e4. \uc138\uc6d4\ud638\ub294 \uc548\uc0b0\uc2dc\uc758 \ub2e8\uc6d0\uace0\ub4f1\ud559\uad50 \ud559\uc0dd\uc774 \uc8fc\uc694 \uad6c\uc131\uc6d0\uc744 \uc774\ub8e8\ub294 \ud0d1\uc2b9\uc778\uc6d0 476\uba85\uc744 \uc218\uc6a9\ud55c \uccad\ud574\uc9c4\ud574\uc6b4 \uc18c\uc18d\uc758 \uc778\ucc9c\ubc1c \uc81c\uc8fc\ud589 \uc5f0\uc548 \uc5ec\uac1d\uc120\uc73c\ub85c 4\uc6d4 16\uc77c \uc624\uc804 8\uc2dc 58\ubd84\uc5d0 \ubcd1\ud48d\ub3c4 \ubd81\ucabd 20km \uc778\uadfc\uc5d0\uc11c \uc870\ub09c \uc2e0\ud638\ub97c \ubcf4\ub0c8\ub2e4. 2014\ub144 4\uc6d4 18\uc77c \uc138\uc6d4\ud638\ub294 \uc644\uc804\ud788 \uce68\ubab0\ud558\uc600\uc73c\uba70, \uc774 \uc0ac\uace0\ub85c \uc2dc\uc2e0 \ubbf8\uc218\uc2b5\uc790 9\uba85\uc744 \ud3ec\ud568\ud55c 304\uba85\uc774 \uc0ac\ub9dd\ud558\uc600\ub2e4. \uce68\ubab0 \uc0ac\uace0 \uc0dd\uc874\uc790 172\uba85 \uc911 \uc808\ubc18 \uc774\uc0c1\uc740 \ud574\uc591\uacbd\ucc30\ubcf4\ub2e4 \uc57d 40\ubd84 \ub2a6\uac8c \ub3c4\ucc29\ud55c \uc5b4\uc120 \ub4f1 \ubbfc\uac04 \uc120\ubc15\uc5d0 \uc758\ud574 \uad6c\uc870\ub418\uc5c8\ub2e4. 3\ub144 \ub3d9\uc548 \uc778\uc591\uc744 \ubbf8\ub904\uc624\ub2e4\uac00 2017\ub144 3\uc6d4 10\uc77c \uc81c18\ub300 \ub300\ud1b5\ub839 \ubc15\uadfc\ud61c\uac00 \ud30c\uba74\ub418\uace0 12\uc77c \ud6c4\uc778 2017\ub144 3\uc6d4 22\uc77c\ubd80\ud130 \uc778\uc591\uc744 \uc2dc\uc791\ud588\ub2e4. 2017\ub144 3\uc6d4 28\uc77c \uad6d\ud68c\uc5d0\uc11c \uc138\uc6d4\ud638 \uc120\uccb4\uc870\uc0ac\uc704\uc6d0\uc120\ucd9c\uc548\uc774 \uc758\uacb0\ub418\uc5c8\ub2e4.", "answer": "172\uba85", "sentence": "\uce68\ubab0 \uc0ac\uace0 \uc0dd\uc874\uc790 172\uba85 \uc911 \uc808\ubc18 \uc774\uc0c1\uc740 \ud574\uc591\uacbd\ucc30\ubcf4\ub2e4 \uc57d 40\ubd84 \ub2a6\uac8c \ub3c4\ucc29\ud55c \uc5b4\uc120 \ub4f1 \ubbfc\uac04 \uc120\ubc15\uc5d0 \uc758\ud574 \uad6c\uc870\ub418\uc5c8\ub2e4.", "paragraph_sentence": "\uc138\uc6d4\ud638 \uce68\ubab0 \uc0ac\uace0(\uc601\uc5b4: Sinking of MV Sewol)\ub294 2014\ub144 4\uc6d4 16\uc77c \uc624\uc804 8\uc2dc 50\ubd84\uacbd \ub300\ud55c\ubbfc\uad6d \uc804\ub77c\ub0a8\ub3c4 \uc9c4\ub3c4\uad70 \uc870\ub3c4\uba74 \ubd80\uadfc \ud574\uc0c1\uc5d0\uc11c \uc5ec\uac1d\uc120 \uc138\uc6d4\ud638\uac00 \uc804\ubcf5\ub418\uc5b4 \uce68\ubab0\ud55c \uc0ac\uace0\uc774\ub2e4. \uc138\uc6d4\ud638\ub294 \uc548\uc0b0\uc2dc\uc758 \ub2e8\uc6d0\uace0\ub4f1\ud559\uad50 \ud559\uc0dd\uc774 \uc8fc\uc694 \uad6c\uc131\uc6d0\uc744 \uc774\ub8e8\ub294 \ud0d1\uc2b9\uc778\uc6d0 476\uba85\uc744 \uc218\uc6a9\ud55c \uccad\ud574\uc9c4\ud574\uc6b4 \uc18c\uc18d\uc758 \uc778\ucc9c\ubc1c \uc81c\uc8fc\ud589 \uc5f0\uc548 \uc5ec\uac1d\uc120\uc73c\ub85c 4\uc6d4 16\uc77c \uc624\uc804 8\uc2dc 58\ubd84\uc5d0 \ubcd1\ud48d\ub3c4 \ubd81\ucabd 20km \uc778\uadfc\uc5d0\uc11c \uc870\ub09c \uc2e0\ud638\ub97c \ubcf4\ub0c8\ub2e4. 2014\ub144 4\uc6d4 18\uc77c \uc138\uc6d4\ud638\ub294 \uc644\uc804\ud788 \uce68\ubab0\ud558\uc600\uc73c\uba70, \uc774 \uc0ac\uace0\ub85c \uc2dc\uc2e0 \ubbf8\uc218\uc2b5\uc790 9\uba85\uc744 \ud3ec\ud568\ud55c 304\uba85\uc774 \uc0ac\ub9dd\ud558\uc600\ub2e4. \uce68\ubab0 \uc0ac\uace0 \uc0dd\uc874\uc790 172\uba85 \uc911 \uc808\ubc18 \uc774\uc0c1\uc740 \ud574\uc591\uacbd\ucc30\ubcf4\ub2e4 \uc57d 40\ubd84 \ub2a6\uac8c \ub3c4\ucc29\ud55c \uc5b4\uc120 \ub4f1 \ubbfc\uac04 \uc120\ubc15\uc5d0 \uc758\ud574 \uad6c\uc870\ub418\uc5c8\ub2e4. 3\ub144 \ub3d9\uc548 \uc778\uc591\uc744 \ubbf8\ub904\uc624\ub2e4\uac00 2017\ub144 3\uc6d4 10\uc77c \uc81c18\ub300 \ub300\ud1b5\ub839 \ubc15\uadfc\ud61c\uac00 \ud30c\uba74\ub418\uace0 12\uc77c \ud6c4\uc778 2017\ub144 3\uc6d4 22\uc77c\ubd80\ud130 \uc778\uc591\uc744 \uc2dc\uc791\ud588\ub2e4. 2017\ub144 3\uc6d4 28\uc77c \uad6d\ud68c\uc5d0\uc11c \uc138\uc6d4\ud638 \uc120\uccb4\uc870\uc0ac\uc704\uc6d0\uc120\ucd9c\uc548\uc774 \uc758\uacb0\ub418\uc5c8\ub2e4.", "paragraph_answer": "\uc138\uc6d4\ud638 \uce68\ubab0 \uc0ac\uace0(\uc601\uc5b4: Sinking of MV Sewol)\ub294 2014\ub144 4\uc6d4 16\uc77c \uc624\uc804 8\uc2dc 50\ubd84\uacbd \ub300\ud55c\ubbfc\uad6d \uc804\ub77c\ub0a8\ub3c4 \uc9c4\ub3c4\uad70 \uc870\ub3c4\uba74 \ubd80\uadfc \ud574\uc0c1\uc5d0\uc11c \uc5ec\uac1d\uc120 \uc138\uc6d4\ud638\uac00 \uc804\ubcf5\ub418\uc5b4 \uce68\ubab0\ud55c \uc0ac\uace0\uc774\ub2e4. \uc138\uc6d4\ud638\ub294 \uc548\uc0b0\uc2dc\uc758 \ub2e8\uc6d0\uace0\ub4f1\ud559\uad50 \ud559\uc0dd\uc774 \uc8fc\uc694 \uad6c\uc131\uc6d0\uc744 \uc774\ub8e8\ub294 \ud0d1\uc2b9\uc778\uc6d0 476\uba85\uc744 \uc218\uc6a9\ud55c \uccad\ud574\uc9c4\ud574\uc6b4 \uc18c\uc18d\uc758 \uc778\ucc9c\ubc1c \uc81c\uc8fc\ud589 \uc5f0\uc548 \uc5ec\uac1d\uc120\uc73c\ub85c 4\uc6d4 16\uc77c \uc624\uc804 8\uc2dc 58\ubd84\uc5d0 \ubcd1\ud48d\ub3c4 \ubd81\ucabd 20km \uc778\uadfc\uc5d0\uc11c \uc870\ub09c \uc2e0\ud638\ub97c \ubcf4\ub0c8\ub2e4. 2014\ub144 4\uc6d4 18\uc77c \uc138\uc6d4\ud638\ub294 \uc644\uc804\ud788 \uce68\ubab0\ud558\uc600\uc73c\uba70, \uc774 \uc0ac\uace0\ub85c \uc2dc\uc2e0 \ubbf8\uc218\uc2b5\uc790 9\uba85\uc744 \ud3ec\ud568\ud55c 304\uba85\uc774 \uc0ac\ub9dd\ud558\uc600\ub2e4. \uce68\ubab0 \uc0ac\uace0 \uc0dd\uc874\uc790 172\uba85 \uc911 \uc808\ubc18 \uc774\uc0c1\uc740 \ud574\uc591\uacbd\ucc30\ubcf4\ub2e4 \uc57d 40\ubd84 \ub2a6\uac8c \ub3c4\ucc29\ud55c \uc5b4\uc120 \ub4f1 \ubbfc\uac04 \uc120\ubc15\uc5d0 \uc758\ud574 \uad6c\uc870\ub418\uc5c8\ub2e4. 3\ub144 \ub3d9\uc548 \uc778\uc591\uc744 \ubbf8\ub904\uc624\ub2e4\uac00 2017\ub144 3\uc6d4 10\uc77c \uc81c18\ub300 \ub300\ud1b5\ub839 \ubc15\uadfc\ud61c\uac00 \ud30c\uba74\ub418\uace0 12\uc77c \ud6c4\uc778 2017\ub144 3\uc6d4 22\uc77c\ubd80\ud130 \uc778\uc591\uc744 \uc2dc\uc791\ud588\ub2e4. 2017\ub144 3\uc6d4 28\uc77c \uad6d\ud68c\uc5d0\uc11c \uc138\uc6d4\ud638 \uc120\uccb4\uc870\uc0ac\uc704\uc6d0\uc120\ucd9c\uc548\uc774 \uc758\uacb0\ub418\uc5c8\ub2e4.", "sentence_answer": "\uce68\ubab0 \uc0ac\uace0 \uc0dd\uc874\uc790 172\uba85 \uc911 \uc808\ubc18 \uc774\uc0c1\uc740 \ud574\uc591\uacbd\ucc30\ubcf4\ub2e4 \uc57d 40\ubd84 \ub2a6\uac8c \ub3c4\ucc29\ud55c \uc5b4\uc120 \ub4f1 \ubbfc\uac04 \uc120\ubc15\uc5d0 \uc758\ud574 \uad6c\uc870\ub418\uc5c8\ub2e4.", "paragraph_id": "6532130-0-2", "paragraph_question": "question: \uc138\uc6d4\ud638 \uc0ac\uace0 \uc0dd\uc874\uc790\uc758 \uc218\ub294 \uba87\uc778\uac00?, context: \uc138\uc6d4\ud638 \uce68\ubab0 \uc0ac\uace0(\uc601\uc5b4: Sinking of MV Sewol)\ub294 2014\ub144 4\uc6d4 16\uc77c \uc624\uc804 8\uc2dc 50\ubd84\uacbd \ub300\ud55c\ubbfc\uad6d \uc804\ub77c\ub0a8\ub3c4 \uc9c4\ub3c4\uad70 \uc870\ub3c4\uba74 \ubd80\uadfc \ud574\uc0c1\uc5d0\uc11c \uc5ec\uac1d\uc120 \uc138\uc6d4\ud638\uac00 \uc804\ubcf5\ub418\uc5b4 \uce68\ubab0\ud55c \uc0ac\uace0\uc774\ub2e4. \uc138\uc6d4\ud638\ub294 \uc548\uc0b0\uc2dc\uc758 \ub2e8\uc6d0\uace0\ub4f1\ud559\uad50 \ud559\uc0dd\uc774 \uc8fc\uc694 \uad6c\uc131\uc6d0\uc744 \uc774\ub8e8\ub294 \ud0d1\uc2b9\uc778\uc6d0 476\uba85\uc744 \uc218\uc6a9\ud55c \uccad\ud574\uc9c4\ud574\uc6b4 \uc18c\uc18d\uc758 \uc778\ucc9c\ubc1c \uc81c\uc8fc\ud589 \uc5f0\uc548 \uc5ec\uac1d\uc120\uc73c\ub85c 4\uc6d4 16\uc77c \uc624\uc804 8\uc2dc 58\ubd84\uc5d0 \ubcd1\ud48d\ub3c4 \ubd81\ucabd 20km \uc778\uadfc\uc5d0\uc11c \uc870\ub09c \uc2e0\ud638\ub97c \ubcf4\ub0c8\ub2e4. 2014\ub144 4\uc6d4 18\uc77c \uc138\uc6d4\ud638\ub294 \uc644\uc804\ud788 \uce68\ubab0\ud558\uc600\uc73c\uba70, \uc774 \uc0ac\uace0\ub85c \uc2dc\uc2e0 \ubbf8\uc218\uc2b5\uc790 9\uba85\uc744 \ud3ec\ud568\ud55c 304\uba85\uc774 \uc0ac\ub9dd\ud558\uc600\ub2e4. \uce68\ubab0 \uc0ac\uace0 \uc0dd\uc874\uc790 172\uba85 \uc911 \uc808\ubc18 \uc774\uc0c1\uc740 \ud574\uc591\uacbd\ucc30\ubcf4\ub2e4 \uc57d 40\ubd84 \ub2a6\uac8c \ub3c4\ucc29\ud55c \uc5b4\uc120 \ub4f1 \ubbfc\uac04 \uc120\ubc15\uc5d0 \uc758\ud574 \uad6c\uc870\ub418\uc5c8\ub2e4. 3\ub144 \ub3d9\uc548 \uc778\uc591\uc744 \ubbf8\ub904\uc624\ub2e4\uac00 2017\ub144 3\uc6d4 10\uc77c \uc81c18\ub300 \ub300\ud1b5\ub839 \ubc15\uadfc\ud61c\uac00 \ud30c\uba74\ub418\uace0 12\uc77c \ud6c4\uc778 2017\ub144 3\uc6d4 22\uc77c\ubd80\ud130 \uc778\uc591\uc744 \uc2dc\uc791\ud588\ub2e4. 2017\ub144 3\uc6d4 28\uc77c \uad6d\ud68c\uc5d0\uc11c \uc138\uc6d4\ud638 \uc120\uccb4\uc870\uc0ac\uc704\uc6d0\uc120\ucd9c\uc548\uc774 \uc758\uacb0\ub418\uc5c8\ub2e4."} {"question": "\ubaa8\ub450\uac00 \uc62c\ub9ac\ub294\ub370 \uc2e4\ud328\ud55c \uc774\uac83\uc740 \ubb34\uc5c7\uc778\uac00?", "paragraph": "23\uc2dc 50\ubd84 20\ucd08\uc5d0 \uc0ac\uace0\uae30\uac00 4R \ud65c\uc8fc\ub85c\uc5d0 \ub2ff\uc558\ub2e4. \ucc29\ub959\ubc14\ud034\uac00 \ub2ff\uc740\uc9c0 2\ucd08\ud6c4 \ubd80\uae30\uc7a5 \uc624\ub9ac\uac94\uc774 \"\uc6b0\ub9ac\ub294 \ubbf8\ub044\ub7ec\uc9c0\uace0 \uc788\uc5b4\uc694\"\ub77c\uace0 \uc0c1\uae30\ud588\ub2e4. \uc5b4\ub290 \ub204\uad6c\ub3c4 \uc2a4\ud3ec\uc77c\ub7ec\ub97c \uc62c\ub9ac\ub294\ub370 \uc2e4\ud328\ud588\uae30 \ub54c\ubb38\uc774\uc5c8\ub294\ub370 \ucc29\ub959\ud588\uc744\ub54c \uc2a4\ud3ec\uc77c\ub7ec\uac00 \uc790\ub3d9\uc73c\ub85c \uc804\uac1c\ub418\uc9c0 \uc54a\uc740\ub370\ub2e4 \ube44\ud589 \uc2b9\ubb34\uc6d0\ub4e4\ub3c4 \uc218\ub3d9\uc73c\ub85c \uc2a4\ud3ec\uc77c\ub7ec\ub97c \uc62c\ub9ac\ub294\ub370 \uc2e4\ud328\ud588\uae30 \ub54c\ubb38\uc774\uc5c8\ub2e4. \uc2a4\ud3ec\uc77c\ub7ec\uac00 \uacf5\uae30\uc800\ud56d\uc744 \ud615\uc131\ud568\uc73c\ub85c \uc778\ud574 \ud56d\uacf5\uae30\uc758 \uc911\ub7c9\uc740 \ucc29\ub959\ubc14\ud034\ub85c \uc9c0\uc9c0\ub418\ub294\ub370 \ub9cc\uc57d \uc2a4\ud3ec\uc77c\ub7ec\uac00 \uc804\uac1c\ub418\uc5c8\uc744 \uacbd\uc6b0\uc5d0\ub294 \uc0ac\uace0\uae30 \uc911\ub7c9\uc758 65%\ub294 \ucda9\ubd84\ud788 \ucc29\ub959\ubc14\ud034\ub9cc\uc73c\ub85c\ub3c4 \uc9c0\uc9c0\ub420 \uc218 \uc788\uc9c0\ub9cc \uc2a4\ud3ec\uc77c\ub7ec\uac00 \uc5c6\uc744 \uacbd\uc6b0 \uc774 \uc218\uce58\uc5d0\uc11c \uac70\uc758 \ube60\uc9c4 15%\ub85c \ub5a8\uc5b4\uc9c0\uac8c \ub41c\ub2e4. \ucc29\ub959\ubc14\ud034\uc758 \uac00\ubcbc\uc6b4 \uc911\ub7c9\uacfc \ud568\uaed8 \uc0ac\uace0\uae30\uc758 \ube0c\ub808\uc774\ud06c\ub294 \uac10\uc18d\ud558\ub294\ub370 \ud6a8\uacfc\uac00 \uc5c6\uc5c8\uc73c\uba70 \uacc4\uc18d \uace0\uc18d\uc73c\ub85c \ud65c\uc8fc\ub85c\ub97c \uc9c8\uc8fc\ud558\uc600\ub2e4. \ube0c\uc2dc\uba3c \uae30\uc7a5\uc774 \uc5d4\uc9c4 \uc5ed\ucd94\uc9c4\uc744 \uac1c\uc2dc\ud558\uba74\uc11c \ubc29\ud5a5 \uc81c\uc5b4\ub97c \uc0c1\uc2e4\ud558\uace0 \uc0ac\uace0\uae30\uc758 \ubc29\ud5a5\ud0a4\uc640 \uc218\uc9c1 \uc548\uc815\ud310\uc758 \ud6a8\uacfc\ub3c4 \uac70\uc758 \uc904\uc5b4\ub4e4\uc5c8\ub2e4.", "answer": "\uc2a4\ud3ec\uc77c\ub7ec", "sentence": "\uc5b4\ub290 \ub204\uad6c\ub3c4 \uc2a4\ud3ec\uc77c\ub7ec \ub97c \uc62c\ub9ac\ub294\ub370 \uc2e4\ud328\ud588\uae30 \ub54c\ubb38\uc774\uc5c8\ub294\ub370 \ucc29\ub959\ud588\uc744\ub54c \uc2a4\ud3ec\uc77c\ub7ec\uac00 \uc790\ub3d9\uc73c\ub85c \uc804\uac1c\ub418\uc9c0 \uc54a\uc740\ub370\ub2e4 \ube44\ud589 \uc2b9\ubb34\uc6d0\ub4e4\ub3c4 \uc218\ub3d9\uc73c\ub85c \uc2a4\ud3ec\uc77c\ub7ec\ub97c \uc62c\ub9ac\ub294\ub370 \uc2e4\ud328\ud588\uae30 \ub54c\ubb38\uc774\uc5c8\ub2e4.", "paragraph_sentence": "23\uc2dc 50\ubd84 20\ucd08\uc5d0 \uc0ac\uace0\uae30\uac00 4R \ud65c\uc8fc\ub85c\uc5d0 \ub2ff\uc558\ub2e4. \ucc29\ub959\ubc14\ud034\uac00 \ub2ff\uc740\uc9c0 2\ucd08\ud6c4 \ubd80\uae30\uc7a5 \uc624\ub9ac\uac94\uc774 \"\uc6b0\ub9ac\ub294 \ubbf8\ub044\ub7ec\uc9c0\uace0 \uc788\uc5b4\uc694\"\ub77c\uace0 \uc0c1\uae30\ud588\ub2e4. \uc5b4\ub290 \ub204\uad6c\ub3c4 \uc2a4\ud3ec\uc77c\ub7ec \ub97c \uc62c\ub9ac\ub294\ub370 \uc2e4\ud328\ud588\uae30 \ub54c\ubb38\uc774\uc5c8\ub294\ub370 \ucc29\ub959\ud588\uc744\ub54c \uc2a4\ud3ec\uc77c\ub7ec\uac00 \uc790\ub3d9\uc73c\ub85c \uc804\uac1c\ub418\uc9c0 \uc54a\uc740\ub370\ub2e4 \ube44\ud589 \uc2b9\ubb34\uc6d0\ub4e4\ub3c4 \uc218\ub3d9\uc73c\ub85c \uc2a4\ud3ec\uc77c\ub7ec\ub97c \uc62c\ub9ac\ub294\ub370 \uc2e4\ud328\ud588\uae30 \ub54c\ubb38\uc774\uc5c8\ub2e4. \uc2a4\ud3ec\uc77c\ub7ec\uac00 \uacf5\uae30\uc800\ud56d\uc744 \ud615\uc131\ud568\uc73c\ub85c \uc778\ud574 \ud56d\uacf5\uae30\uc758 \uc911\ub7c9\uc740 \ucc29\ub959\ubc14\ud034\ub85c \uc9c0\uc9c0\ub418\ub294\ub370 \ub9cc\uc57d \uc2a4\ud3ec\uc77c\ub7ec\uac00 \uc804\uac1c\ub418\uc5c8\uc744 \uacbd\uc6b0\uc5d0\ub294 \uc0ac\uace0\uae30 \uc911\ub7c9\uc758 65%\ub294 \ucda9\ubd84\ud788 \ucc29\ub959\ubc14\ud034\ub9cc\uc73c\ub85c\ub3c4 \uc9c0\uc9c0\ub420 \uc218 \uc788\uc9c0\ub9cc \uc2a4\ud3ec\uc77c\ub7ec\uac00 \uc5c6\uc744 \uacbd\uc6b0 \uc774 \uc218\uce58\uc5d0\uc11c \uac70\uc758 \ube60\uc9c4 15%\ub85c \ub5a8\uc5b4\uc9c0\uac8c \ub41c\ub2e4. \ucc29\ub959\ubc14\ud034\uc758 \uac00\ubcbc\uc6b4 \uc911\ub7c9\uacfc \ud568\uaed8 \uc0ac\uace0\uae30\uc758 \ube0c\ub808\uc774\ud06c\ub294 \uac10\uc18d\ud558\ub294\ub370 \ud6a8\uacfc\uac00 \uc5c6\uc5c8\uc73c\uba70 \uacc4\uc18d \uace0\uc18d\uc73c\ub85c \ud65c\uc8fc\ub85c\ub97c \uc9c8\uc8fc\ud558\uc600\ub2e4. \ube0c\uc2dc\uba3c \uae30\uc7a5\uc774 \uc5d4\uc9c4 \uc5ed\ucd94\uc9c4\uc744 \uac1c\uc2dc\ud558\uba74\uc11c \ubc29\ud5a5 \uc81c\uc5b4\ub97c \uc0c1\uc2e4\ud558\uace0 \uc0ac\uace0\uae30\uc758 \ubc29\ud5a5\ud0a4\uc640 \uc218\uc9c1 \uc548\uc815\ud310\uc758 \ud6a8\uacfc\ub3c4 \uac70\uc758 \uc904\uc5b4\ub4e4\uc5c8\ub2e4.", "paragraph_answer": "23\uc2dc 50\ubd84 20\ucd08\uc5d0 \uc0ac\uace0\uae30\uac00 4R \ud65c\uc8fc\ub85c\uc5d0 \ub2ff\uc558\ub2e4. \ucc29\ub959\ubc14\ud034\uac00 \ub2ff\uc740\uc9c0 2\ucd08\ud6c4 \ubd80\uae30\uc7a5 \uc624\ub9ac\uac94\uc774 \"\uc6b0\ub9ac\ub294 \ubbf8\ub044\ub7ec\uc9c0\uace0 \uc788\uc5b4\uc694\"\ub77c\uace0 \uc0c1\uae30\ud588\ub2e4. \uc5b4\ub290 \ub204\uad6c\ub3c4 \uc2a4\ud3ec\uc77c\ub7ec \ub97c \uc62c\ub9ac\ub294\ub370 \uc2e4\ud328\ud588\uae30 \ub54c\ubb38\uc774\uc5c8\ub294\ub370 \ucc29\ub959\ud588\uc744\ub54c \uc2a4\ud3ec\uc77c\ub7ec\uac00 \uc790\ub3d9\uc73c\ub85c \uc804\uac1c\ub418\uc9c0 \uc54a\uc740\ub370\ub2e4 \ube44\ud589 \uc2b9\ubb34\uc6d0\ub4e4\ub3c4 \uc218\ub3d9\uc73c\ub85c \uc2a4\ud3ec\uc77c\ub7ec\ub97c \uc62c\ub9ac\ub294\ub370 \uc2e4\ud328\ud588\uae30 \ub54c\ubb38\uc774\uc5c8\ub2e4. \uc2a4\ud3ec\uc77c\ub7ec\uac00 \uacf5\uae30\uc800\ud56d\uc744 \ud615\uc131\ud568\uc73c\ub85c \uc778\ud574 \ud56d\uacf5\uae30\uc758 \uc911\ub7c9\uc740 \ucc29\ub959\ubc14\ud034\ub85c \uc9c0\uc9c0\ub418\ub294\ub370 \ub9cc\uc57d \uc2a4\ud3ec\uc77c\ub7ec\uac00 \uc804\uac1c\ub418\uc5c8\uc744 \uacbd\uc6b0\uc5d0\ub294 \uc0ac\uace0\uae30 \uc911\ub7c9\uc758 65%\ub294 \ucda9\ubd84\ud788 \ucc29\ub959\ubc14\ud034\ub9cc\uc73c\ub85c\ub3c4 \uc9c0\uc9c0\ub420 \uc218 \uc788\uc9c0\ub9cc \uc2a4\ud3ec\uc77c\ub7ec\uac00 \uc5c6\uc744 \uacbd\uc6b0 \uc774 \uc218\uce58\uc5d0\uc11c \uac70\uc758 \ube60\uc9c4 15%\ub85c \ub5a8\uc5b4\uc9c0\uac8c \ub41c\ub2e4. \ucc29\ub959\ubc14\ud034\uc758 \uac00\ubcbc\uc6b4 \uc911\ub7c9\uacfc \ud568\uaed8 \uc0ac\uace0\uae30\uc758 \ube0c\ub808\uc774\ud06c\ub294 \uac10\uc18d\ud558\ub294\ub370 \ud6a8\uacfc\uac00 \uc5c6\uc5c8\uc73c\uba70 \uacc4\uc18d \uace0\uc18d\uc73c\ub85c \ud65c\uc8fc\ub85c\ub97c \uc9c8\uc8fc\ud558\uc600\ub2e4. \ube0c\uc2dc\uba3c \uae30\uc7a5\uc774 \uc5d4\uc9c4 \uc5ed\ucd94\uc9c4\uc744 \uac1c\uc2dc\ud558\uba74\uc11c \ubc29\ud5a5 \uc81c\uc5b4\ub97c \uc0c1\uc2e4\ud558\uace0 \uc0ac\uace0\uae30\uc758 \ubc29\ud5a5\ud0a4\uc640 \uc218\uc9c1 \uc548\uc815\ud310\uc758 \ud6a8\uacfc\ub3c4 \uac70\uc758 \uc904\uc5b4\ub4e4\uc5c8\ub2e4.", "sentence_answer": "\uc5b4\ub290 \ub204\uad6c\ub3c4 \uc2a4\ud3ec\uc77c\ub7ec \ub97c \uc62c\ub9ac\ub294\ub370 \uc2e4\ud328\ud588\uae30 \ub54c\ubb38\uc774\uc5c8\ub294\ub370 \ucc29\ub959\ud588\uc744\ub54c \uc2a4\ud3ec\uc77c\ub7ec\uac00 \uc790\ub3d9\uc73c\ub85c \uc804\uac1c\ub418\uc9c0 \uc54a\uc740\ub370\ub2e4 \ube44\ud589 \uc2b9\ubb34\uc6d0\ub4e4\ub3c4 \uc218\ub3d9\uc73c\ub85c \uc2a4\ud3ec\uc77c\ub7ec\ub97c \uc62c\ub9ac\ub294\ub370 \uc2e4\ud328\ud588\uae30 \ub54c\ubb38\uc774\uc5c8\ub2e4.", "paragraph_id": "6533622-3-0", "paragraph_question": "question: \ubaa8\ub450\uac00 \uc62c\ub9ac\ub294\ub370 \uc2e4\ud328\ud55c \uc774\uac83\uc740 \ubb34\uc5c7\uc778\uac00?, context: 23\uc2dc 50\ubd84 20\ucd08\uc5d0 \uc0ac\uace0\uae30\uac00 4R \ud65c\uc8fc\ub85c\uc5d0 \ub2ff\uc558\ub2e4. \ucc29\ub959\ubc14\ud034\uac00 \ub2ff\uc740\uc9c0 2\ucd08\ud6c4 \ubd80\uae30\uc7a5 \uc624\ub9ac\uac94\uc774 \"\uc6b0\ub9ac\ub294 \ubbf8\ub044\ub7ec\uc9c0\uace0 \uc788\uc5b4\uc694\"\ub77c\uace0 \uc0c1\uae30\ud588\ub2e4. \uc5b4\ub290 \ub204\uad6c\ub3c4 \uc2a4\ud3ec\uc77c\ub7ec\ub97c \uc62c\ub9ac\ub294\ub370 \uc2e4\ud328\ud588\uae30 \ub54c\ubb38\uc774\uc5c8\ub294\ub370 \ucc29\ub959\ud588\uc744\ub54c \uc2a4\ud3ec\uc77c\ub7ec\uac00 \uc790\ub3d9\uc73c\ub85c \uc804\uac1c\ub418\uc9c0 \uc54a\uc740\ub370\ub2e4 \ube44\ud589 \uc2b9\ubb34\uc6d0\ub4e4\ub3c4 \uc218\ub3d9\uc73c\ub85c \uc2a4\ud3ec\uc77c\ub7ec\ub97c \uc62c\ub9ac\ub294\ub370 \uc2e4\ud328\ud588\uae30 \ub54c\ubb38\uc774\uc5c8\ub2e4. \uc2a4\ud3ec\uc77c\ub7ec\uac00 \uacf5\uae30\uc800\ud56d\uc744 \ud615\uc131\ud568\uc73c\ub85c \uc778\ud574 \ud56d\uacf5\uae30\uc758 \uc911\ub7c9\uc740 \ucc29\ub959\ubc14\ud034\ub85c \uc9c0\uc9c0\ub418\ub294\ub370 \ub9cc\uc57d \uc2a4\ud3ec\uc77c\ub7ec\uac00 \uc804\uac1c\ub418\uc5c8\uc744 \uacbd\uc6b0\uc5d0\ub294 \uc0ac\uace0\uae30 \uc911\ub7c9\uc758 65%\ub294 \ucda9\ubd84\ud788 \ucc29\ub959\ubc14\ud034\ub9cc\uc73c\ub85c\ub3c4 \uc9c0\uc9c0\ub420 \uc218 \uc788\uc9c0\ub9cc \uc2a4\ud3ec\uc77c\ub7ec\uac00 \uc5c6\uc744 \uacbd\uc6b0 \uc774 \uc218\uce58\uc5d0\uc11c \uac70\uc758 \ube60\uc9c4 15%\ub85c \ub5a8\uc5b4\uc9c0\uac8c \ub41c\ub2e4. \ucc29\ub959\ubc14\ud034\uc758 \uac00\ubcbc\uc6b4 \uc911\ub7c9\uacfc \ud568\uaed8 \uc0ac\uace0\uae30\uc758 \ube0c\ub808\uc774\ud06c\ub294 \uac10\uc18d\ud558\ub294\ub370 \ud6a8\uacfc\uac00 \uc5c6\uc5c8\uc73c\uba70 \uacc4\uc18d \uace0\uc18d\uc73c\ub85c \ud65c\uc8fc\ub85c\ub97c \uc9c8\uc8fc\ud558\uc600\ub2e4. \ube0c\uc2dc\uba3c \uae30\uc7a5\uc774 \uc5d4\uc9c4 \uc5ed\ucd94\uc9c4\uc744 \uac1c\uc2dc\ud558\uba74\uc11c \ubc29\ud5a5 \uc81c\uc5b4\ub97c \uc0c1\uc2e4\ud558\uace0 \uc0ac\uace0\uae30\uc758 \ubc29\ud5a5\ud0a4\uc640 \uc218\uc9c1 \uc548\uc815\ud310\uc758 \ud6a8\uacfc\ub3c4 \uac70\uc758 \uc904\uc5b4\ub4e4\uc5c8\ub2e4."} {"question": "\uc548\ucc3d\ud638\ub294 \uba87 \ub144\uc5d0 \uc0ac\ub9dd\ud588\ub294\uac00?", "paragraph": "\ub300\ud55c\uc758\uad70 \ud2b9\ud30c\ub3c5\ub9bd\ub300\uc7a5 \uacb8 \uc544\ub839\uc9c0\uad6c\uc0ac\ub839\uad00\uc744 \uc9c0\ub0b8 \ud56d\uc77c \ub3c5\ub9bd\uc6b4\ub3d9\uac00 \uc548\uc911\uadfc(\u5b89\u91cd\u6839) \uc120\uc0dd\uacfc\ub294 \ub3d9\uc131\ub3d9\ubcf8 \uce5c\uc871\ucc99\uc774\uae30\ub3c4 \ud558\uba70 1938\ub144 \uacbd\uc0c1\ub0a8\ub3c4 \ubd80\uc0b0\uc5d0\uc11c \uac04\uacbd\ud654\ub97c \ube44\ub86f\ud55c \ud3d0\ub834\uacfc \ub9cc\uc131\uae30\uad00\uc9c0\uc5fc \ubc0f \uc704\ud558\uc218\uc99d\u00b7 \ubcf5\ub9c9\uc5fc \ubc0f \ud53c\ubd80\uc5fc\uacfc \uc18c\ud654\ubd88\ub7c9 \ub4f1\uc758 \ud569\ubcd1\uc99d\uc73c\ub85c \ud0c0\uacc4\ud55c \uadf8\ub294 \uc0dd\uc804\uc5d0 \ud765\uc0ac\ub2e8, \ub300\ud55c\uc778\uad6d\ubbfc\ud68c \ub4f1 \uc7ac\ubbf8\ud55c\uc778\ub2e8\uccb4\ub97c \uc870\uc9c1, \uc8fc\uad00\ud558\uc600\uace0, 1919\ub144 4\uc6d4\ubd80\ud130 \ub300\ud55c\ubbfc\uad6d \uc784\uc2dc\uc815\ubd80 \uc218\ub9bd\uc5d0 \ucc38\uc5ec, \uc8fc\ub3c4\ud558\uc600\ub2e4. 1921\ub144\ubd80\ud130\ub294 \uc784\uc2dc\uc815\ubd80 \ucc3d\uc870\ub860\uacfc \uac1c\uc870\ub860\uc774 \ub098\ub260 \ub54c \uac1c\uc870\ub860\uc744 \uc8fc\uc7a5\ud558\uc600\uc73c\uba70, \uad6d\ubbfc\ub300\ud45c\uc790\ud68c\uc758\uac00 \uac15\uc81c \ud574\uc0b0\ub41c\ub4a4 \ubbf8\uad6d\uc73c\ub85c \uac74\ub108\uac00 \ub3c5\ub9bd\uc6b4\ub3d9\uc744 \ud558\uc600\ub2e4. \ubbf8\uad6d\uc758 \ud55c\uc778 \uad50\ud3ec\uc0ac\ud68c\uc758 \uc9c0\ub3c4\uc790\uc600\ub358 \uadf8\ub294 \ubc15\uc6a9\ub9cc, \uc774\uc2b9\ub9cc, \uc11c\uc7ac\ud544 \ub4f1\uacfc \uacbd\uc7c1, \uac08\ub4f1\uad00\uacc4\uc5d0 \uc788\uae30\ub3c4 \ud588\ub2e4. 1932\ub144 \uc724\ubd09\uae38\uc758 \ud6d9\ucee4\uc6b0 \uacf5\uc6d0 \ud3ed\ud0c4\ud22c\ucc99 \uc0ac\uac74 \uc774\ud6c4 \uc77c\ubcf8 \uc81c\uad6d \uc601\uc0ac\uad00 \uacbd\ucc30\uc758 \ubd88\uc2ec\uac80\ubb38\uc5d0 \uccb4\ud3ec\ub418\uc5b4 \uc1a1\ud658, \uc11c\ub300\ubb38\ud615\ubb34\uc18c\uc5d0 \ud22c\uc625\ub418\uc5b4 \ud615\ubb38\uc744 \ubc1b\uc558\uace0, \uace0\ubb38 \ud6c4\uc720\uc99d\uc73c\ub85c \uacbd\uc131\uc81c\uad6d\ub300\ud559 \ubcd1\uc6d0\uc5d0 \uc785\uc6d0\ud588\uc73c\ub098 \uc18c\ud654\ubd88\ub7c9\uacfc \uac04\uacbd\ud654, \ud3d0\uc9c8\ud658\uc744 \ube44\ub86f\ud55c \uc5ec\ub7ec \uc9c8\ubcd1\uc758 \ud569\ubcd1\uc99d\uc73c\ub85c \ucd9c\uc625 \uc9c1\ud6c4 \uc0ac\ub9dd\ud55c\ub2e4.", "answer": "1938\ub144", "sentence": "\ub300\ud55c\uc758\uad70 \ud2b9\ud30c\ub3c5\ub9bd\ub300\uc7a5 \uacb8 \uc544\ub839\uc9c0\uad6c\uc0ac\ub839\uad00\uc744 \uc9c0\ub0b8 \ud56d\uc77c \ub3c5\ub9bd\uc6b4\ub3d9\uac00 \uc548\uc911\uadfc(\u5b89\u91cd\u6839) \uc120\uc0dd\uacfc\ub294 \ub3d9\uc131\ub3d9\ubcf8 \uce5c\uc871\ucc99\uc774\uae30\ub3c4 \ud558\uba70 1938\ub144 \uacbd\uc0c1\ub0a8\ub3c4 \ubd80\uc0b0\uc5d0\uc11c \uac04\uacbd\ud654\ub97c \ube44\ub86f\ud55c \ud3d0\ub834\uacfc \ub9cc\uc131\uae30\uad00\uc9c0\uc5fc \ubc0f \uc704\ud558\uc218\uc99d\u00b7 \ubcf5\ub9c9\uc5fc \ubc0f \ud53c\ubd80\uc5fc\uacfc \uc18c\ud654\ubd88\ub7c9 \ub4f1\uc758 \ud569\ubcd1\uc99d\uc73c\ub85c \ud0c0\uacc4\ud55c \uadf8\ub294 \uc0dd\uc804\uc5d0 \ud765\uc0ac\ub2e8, \ub300\ud55c\uc778\uad6d\ubbfc\ud68c \ub4f1 \uc7ac\ubbf8\ud55c\uc778\ub2e8\uccb4\ub97c \uc870\uc9c1, \uc8fc\uad00\ud558\uc600\uace0, 1919\ub144 4\uc6d4\ubd80\ud130 \ub300\ud55c\ubbfc\uad6d \uc784\uc2dc\uc815\ubd80 \uc218\ub9bd\uc5d0 \ucc38\uc5ec, \uc8fc\ub3c4\ud558\uc600\ub2e4.", "paragraph_sentence": " \ub300\ud55c\uc758\uad70 \ud2b9\ud30c\ub3c5\ub9bd\ub300\uc7a5 \uacb8 \uc544\ub839\uc9c0\uad6c\uc0ac\ub839\uad00\uc744 \uc9c0\ub0b8 \ud56d\uc77c \ub3c5\ub9bd\uc6b4\ub3d9\uac00 \uc548\uc911\uadfc(\u5b89\u91cd\u6839) \uc120\uc0dd\uacfc\ub294 \ub3d9\uc131\ub3d9\ubcf8 \uce5c\uc871\ucc99\uc774\uae30\ub3c4 \ud558\uba70 1938\ub144 \uacbd\uc0c1\ub0a8\ub3c4 \ubd80\uc0b0\uc5d0\uc11c \uac04\uacbd\ud654\ub97c \ube44\ub86f\ud55c \ud3d0\ub834\uacfc \ub9cc\uc131\uae30\uad00\uc9c0\uc5fc \ubc0f \uc704\ud558\uc218\uc99d\u00b7 \ubcf5\ub9c9\uc5fc \ubc0f \ud53c\ubd80\uc5fc\uacfc \uc18c\ud654\ubd88\ub7c9 \ub4f1\uc758 \ud569\ubcd1\uc99d\uc73c\ub85c \ud0c0\uacc4\ud55c \uadf8\ub294 \uc0dd\uc804\uc5d0 \ud765\uc0ac\ub2e8, \ub300\ud55c\uc778\uad6d\ubbfc\ud68c \ub4f1 \uc7ac\ubbf8\ud55c\uc778\ub2e8\uccb4\ub97c \uc870\uc9c1, \uc8fc\uad00\ud558\uc600\uace0, 1919\ub144 4\uc6d4\ubd80\ud130 \ub300\ud55c\ubbfc\uad6d \uc784\uc2dc\uc815\ubd80 \uc218\ub9bd\uc5d0 \ucc38\uc5ec, \uc8fc\ub3c4\ud558\uc600\ub2e4. 1921\ub144\ubd80\ud130\ub294 \uc784\uc2dc\uc815\ubd80 \ucc3d\uc870\ub860\uacfc \uac1c\uc870\ub860\uc774 \ub098\ub260 \ub54c \uac1c\uc870\ub860\uc744 \uc8fc\uc7a5\ud558\uc600\uc73c\uba70, \uad6d\ubbfc\ub300\ud45c\uc790\ud68c\uc758\uac00 \uac15\uc81c \ud574\uc0b0\ub41c\ub4a4 \ubbf8\uad6d\uc73c\ub85c \uac74\ub108\uac00 \ub3c5\ub9bd\uc6b4\ub3d9\uc744 \ud558\uc600\ub2e4. \ubbf8\uad6d\uc758 \ud55c\uc778 \uad50\ud3ec\uc0ac\ud68c\uc758 \uc9c0\ub3c4\uc790\uc600\ub358 \uadf8\ub294 \ubc15\uc6a9\ub9cc, \uc774\uc2b9\ub9cc, \uc11c\uc7ac\ud544 \ub4f1\uacfc \uacbd\uc7c1, \uac08\ub4f1\uad00\uacc4\uc5d0 \uc788\uae30\ub3c4 \ud588\ub2e4. 1932\ub144 \uc724\ubd09\uae38\uc758 \ud6d9\ucee4\uc6b0 \uacf5\uc6d0 \ud3ed\ud0c4\ud22c\ucc99 \uc0ac\uac74 \uc774\ud6c4 \uc77c\ubcf8 \uc81c\uad6d \uc601\uc0ac\uad00 \uacbd\ucc30\uc758 \ubd88\uc2ec\uac80\ubb38\uc5d0 \uccb4\ud3ec\ub418\uc5b4 \uc1a1\ud658, \uc11c\ub300\ubb38\ud615\ubb34\uc18c\uc5d0 \ud22c\uc625\ub418\uc5b4 \ud615\ubb38\uc744 \ubc1b\uc558\uace0, \uace0\ubb38 \ud6c4\uc720\uc99d\uc73c\ub85c \uacbd\uc131\uc81c\uad6d\ub300\ud559 \ubcd1\uc6d0\uc5d0 \uc785\uc6d0\ud588\uc73c\ub098 \uc18c\ud654\ubd88\ub7c9\uacfc \uac04\uacbd\ud654, \ud3d0\uc9c8\ud658\uc744 \ube44\ub86f\ud55c \uc5ec\ub7ec \uc9c8\ubcd1\uc758 \ud569\ubcd1\uc99d\uc73c\ub85c \ucd9c\uc625 \uc9c1\ud6c4 \uc0ac\ub9dd\ud55c\ub2e4.", "paragraph_answer": "\ub300\ud55c\uc758\uad70 \ud2b9\ud30c\ub3c5\ub9bd\ub300\uc7a5 \uacb8 \uc544\ub839\uc9c0\uad6c\uc0ac\ub839\uad00\uc744 \uc9c0\ub0b8 \ud56d\uc77c \ub3c5\ub9bd\uc6b4\ub3d9\uac00 \uc548\uc911\uadfc(\u5b89\u91cd\u6839) \uc120\uc0dd\uacfc\ub294 \ub3d9\uc131\ub3d9\ubcf8 \uce5c\uc871\ucc99\uc774\uae30\ub3c4 \ud558\uba70 1938\ub144 \uacbd\uc0c1\ub0a8\ub3c4 \ubd80\uc0b0\uc5d0\uc11c \uac04\uacbd\ud654\ub97c \ube44\ub86f\ud55c \ud3d0\ub834\uacfc \ub9cc\uc131\uae30\uad00\uc9c0\uc5fc \ubc0f \uc704\ud558\uc218\uc99d\u00b7 \ubcf5\ub9c9\uc5fc \ubc0f \ud53c\ubd80\uc5fc\uacfc \uc18c\ud654\ubd88\ub7c9 \ub4f1\uc758 \ud569\ubcd1\uc99d\uc73c\ub85c \ud0c0\uacc4\ud55c \uadf8\ub294 \uc0dd\uc804\uc5d0 \ud765\uc0ac\ub2e8, \ub300\ud55c\uc778\uad6d\ubbfc\ud68c \ub4f1 \uc7ac\ubbf8\ud55c\uc778\ub2e8\uccb4\ub97c \uc870\uc9c1, \uc8fc\uad00\ud558\uc600\uace0, 1919\ub144 4\uc6d4\ubd80\ud130 \ub300\ud55c\ubbfc\uad6d \uc784\uc2dc\uc815\ubd80 \uc218\ub9bd\uc5d0 \ucc38\uc5ec, \uc8fc\ub3c4\ud558\uc600\ub2e4. 1921\ub144\ubd80\ud130\ub294 \uc784\uc2dc\uc815\ubd80 \ucc3d\uc870\ub860\uacfc \uac1c\uc870\ub860\uc774 \ub098\ub260 \ub54c \uac1c\uc870\ub860\uc744 \uc8fc\uc7a5\ud558\uc600\uc73c\uba70, \uad6d\ubbfc\ub300\ud45c\uc790\ud68c\uc758\uac00 \uac15\uc81c \ud574\uc0b0\ub41c\ub4a4 \ubbf8\uad6d\uc73c\ub85c \uac74\ub108\uac00 \ub3c5\ub9bd\uc6b4\ub3d9\uc744 \ud558\uc600\ub2e4. \ubbf8\uad6d\uc758 \ud55c\uc778 \uad50\ud3ec\uc0ac\ud68c\uc758 \uc9c0\ub3c4\uc790\uc600\ub358 \uadf8\ub294 \ubc15\uc6a9\ub9cc, \uc774\uc2b9\ub9cc, \uc11c\uc7ac\ud544 \ub4f1\uacfc \uacbd\uc7c1, \uac08\ub4f1\uad00\uacc4\uc5d0 \uc788\uae30\ub3c4 \ud588\ub2e4. 1932\ub144 \uc724\ubd09\uae38\uc758 \ud6d9\ucee4\uc6b0 \uacf5\uc6d0 \ud3ed\ud0c4\ud22c\ucc99 \uc0ac\uac74 \uc774\ud6c4 \uc77c\ubcf8 \uc81c\uad6d \uc601\uc0ac\uad00 \uacbd\ucc30\uc758 \ubd88\uc2ec\uac80\ubb38\uc5d0 \uccb4\ud3ec\ub418\uc5b4 \uc1a1\ud658, \uc11c\ub300\ubb38\ud615\ubb34\uc18c\uc5d0 \ud22c\uc625\ub418\uc5b4 \ud615\ubb38\uc744 \ubc1b\uc558\uace0, \uace0\ubb38 \ud6c4\uc720\uc99d\uc73c\ub85c \uacbd\uc131\uc81c\uad6d\ub300\ud559 \ubcd1\uc6d0\uc5d0 \uc785\uc6d0\ud588\uc73c\ub098 \uc18c\ud654\ubd88\ub7c9\uacfc \uac04\uacbd\ud654, \ud3d0\uc9c8\ud658\uc744 \ube44\ub86f\ud55c \uc5ec\ub7ec \uc9c8\ubcd1\uc758 \ud569\ubcd1\uc99d\uc73c\ub85c \ucd9c\uc625 \uc9c1\ud6c4 \uc0ac\ub9dd\ud55c\ub2e4.", "sentence_answer": "\ub300\ud55c\uc758\uad70 \ud2b9\ud30c\ub3c5\ub9bd\ub300\uc7a5 \uacb8 \uc544\ub839\uc9c0\uad6c\uc0ac\ub839\uad00\uc744 \uc9c0\ub0b8 \ud56d\uc77c \ub3c5\ub9bd\uc6b4\ub3d9\uac00 \uc548\uc911\uadfc(\u5b89\u91cd\u6839) \uc120\uc0dd\uacfc\ub294 \ub3d9\uc131\ub3d9\ubcf8 \uce5c\uc871\ucc99\uc774\uae30\ub3c4 \ud558\uba70 1938\ub144 \uacbd\uc0c1\ub0a8\ub3c4 \ubd80\uc0b0\uc5d0\uc11c \uac04\uacbd\ud654\ub97c \ube44\ub86f\ud55c \ud3d0\ub834\uacfc \ub9cc\uc131\uae30\uad00\uc9c0\uc5fc \ubc0f \uc704\ud558\uc218\uc99d\u00b7 \ubcf5\ub9c9\uc5fc \ubc0f \ud53c\ubd80\uc5fc\uacfc \uc18c\ud654\ubd88\ub7c9 \ub4f1\uc758 \ud569\ubcd1\uc99d\uc73c\ub85c \ud0c0\uacc4\ud55c \uadf8\ub294 \uc0dd\uc804\uc5d0 \ud765\uc0ac\ub2e8, \ub300\ud55c\uc778\uad6d\ubbfc\ud68c \ub4f1 \uc7ac\ubbf8\ud55c\uc778\ub2e8\uccb4\ub97c \uc870\uc9c1, \uc8fc\uad00\ud558\uc600\uace0, 1919\ub144 4\uc6d4\ubd80\ud130 \ub300\ud55c\ubbfc\uad6d \uc784\uc2dc\uc815\ubd80 \uc218\ub9bd\uc5d0 \ucc38\uc5ec, \uc8fc\ub3c4\ud558\uc600\ub2e4.", "paragraph_id": "6521619-0-0", "paragraph_question": "question: \uc548\ucc3d\ud638\ub294 \uba87 \ub144\uc5d0 \uc0ac\ub9dd\ud588\ub294\uac00?, context: \ub300\ud55c\uc758\uad70 \ud2b9\ud30c\ub3c5\ub9bd\ub300\uc7a5 \uacb8 \uc544\ub839\uc9c0\uad6c\uc0ac\ub839\uad00\uc744 \uc9c0\ub0b8 \ud56d\uc77c \ub3c5\ub9bd\uc6b4\ub3d9\uac00 \uc548\uc911\uadfc(\u5b89\u91cd\u6839) \uc120\uc0dd\uacfc\ub294 \ub3d9\uc131\ub3d9\ubcf8 \uce5c\uc871\ucc99\uc774\uae30\ub3c4 \ud558\uba70 1938\ub144 \uacbd\uc0c1\ub0a8\ub3c4 \ubd80\uc0b0\uc5d0\uc11c \uac04\uacbd\ud654\ub97c \ube44\ub86f\ud55c \ud3d0\ub834\uacfc \ub9cc\uc131\uae30\uad00\uc9c0\uc5fc \ubc0f \uc704\ud558\uc218\uc99d\u00b7 \ubcf5\ub9c9\uc5fc \ubc0f \ud53c\ubd80\uc5fc\uacfc \uc18c\ud654\ubd88\ub7c9 \ub4f1\uc758 \ud569\ubcd1\uc99d\uc73c\ub85c \ud0c0\uacc4\ud55c \uadf8\ub294 \uc0dd\uc804\uc5d0 \ud765\uc0ac\ub2e8, \ub300\ud55c\uc778\uad6d\ubbfc\ud68c \ub4f1 \uc7ac\ubbf8\ud55c\uc778\ub2e8\uccb4\ub97c \uc870\uc9c1, \uc8fc\uad00\ud558\uc600\uace0, 1919\ub144 4\uc6d4\ubd80\ud130 \ub300\ud55c\ubbfc\uad6d \uc784\uc2dc\uc815\ubd80 \uc218\ub9bd\uc5d0 \ucc38\uc5ec, \uc8fc\ub3c4\ud558\uc600\ub2e4. 1921\ub144\ubd80\ud130\ub294 \uc784\uc2dc\uc815\ubd80 \ucc3d\uc870\ub860\uacfc \uac1c\uc870\ub860\uc774 \ub098\ub260 \ub54c \uac1c\uc870\ub860\uc744 \uc8fc\uc7a5\ud558\uc600\uc73c\uba70, \uad6d\ubbfc\ub300\ud45c\uc790\ud68c\uc758\uac00 \uac15\uc81c \ud574\uc0b0\ub41c\ub4a4 \ubbf8\uad6d\uc73c\ub85c \uac74\ub108\uac00 \ub3c5\ub9bd\uc6b4\ub3d9\uc744 \ud558\uc600\ub2e4. \ubbf8\uad6d\uc758 \ud55c\uc778 \uad50\ud3ec\uc0ac\ud68c\uc758 \uc9c0\ub3c4\uc790\uc600\ub358 \uadf8\ub294 \ubc15\uc6a9\ub9cc, \uc774\uc2b9\ub9cc, \uc11c\uc7ac\ud544 \ub4f1\uacfc \uacbd\uc7c1, \uac08\ub4f1\uad00\uacc4\uc5d0 \uc788\uae30\ub3c4 \ud588\ub2e4. 1932\ub144 \uc724\ubd09\uae38\uc758 \ud6d9\ucee4\uc6b0 \uacf5\uc6d0 \ud3ed\ud0c4\ud22c\ucc99 \uc0ac\uac74 \uc774\ud6c4 \uc77c\ubcf8 \uc81c\uad6d \uc601\uc0ac\uad00 \uacbd\ucc30\uc758 \ubd88\uc2ec\uac80\ubb38\uc5d0 \uccb4\ud3ec\ub418\uc5b4 \uc1a1\ud658, \uc11c\ub300\ubb38\ud615\ubb34\uc18c\uc5d0 \ud22c\uc625\ub418\uc5b4 \ud615\ubb38\uc744 \ubc1b\uc558\uace0, \uace0\ubb38 \ud6c4\uc720\uc99d\uc73c\ub85c \uacbd\uc131\uc81c\uad6d\ub300\ud559 \ubcd1\uc6d0\uc5d0 \uc785\uc6d0\ud588\uc73c\ub098 \uc18c\ud654\ubd88\ub7c9\uacfc \uac04\uacbd\ud654, \ud3d0\uc9c8\ud658\uc744 \ube44\ub86f\ud55c \uc5ec\ub7ec \uc9c8\ubcd1\uc758 \ud569\ubcd1\uc99d\uc73c\ub85c \ucd9c\uc625 \uc9c1\ud6c4 \uc0ac\ub9dd\ud55c\ub2e4."} {"question": "\uc131\ud615\uc774\ub098 \uac00\uacf5\uc5d0 \ud3b8\ub9ac\ud558\ub3c4\ub85d \uc720\ub9ac \uc548\uc5d0 \uc11e\ub294 \uac83\uc740?", "paragraph": "\uc720\ub9ac\uc758 \uad6c\uc870\uc5d0\ub294 \uad6c\uba4d\uc774 \uc788\uc5b4 \uace0\uc628\uc774 \ub418\uc5b4 \uc6d0\uc790\uac00 \ud06c\uac8c \uc9c4\ub3d9\ud558\uc5ec\ub3c4, \uc774 \uad6c\uba4d\uc774 \ucfe0\uc158\uacfc \uac19\uc740 \uc5ed\ud560\uc744 \ud558\uc5ec \uc798 \uae68\uc5b4\uc9c0\uc9c0 \uc54a\ub294\ub2e4. \uadf8\ub7ec\ub098 \uc18c\ub2e4\uc11d\ud68c\uc720\ub9ac\uc640 \uac19\uc774 \uc131\ud615\uc774\ub098 \uac00\uacf5\uc5d0 \ud3b8\ub9ac\ud558\ub3c4\ub85d \uc54c\uce7c\ub9ac\ub97c \uc11e\uc73c\uba74, \uc54c\uce7c\ub9ac\uc774\uc628\uc774 \uc6a9\ud574\ub418\uc5b4 \uc774 \uad6c\uba4d \uc18d\uc73c\ub85c \ub4e4\uc5b4\uac04\ub2e4. \uc774 \ub54c\ubb38\uc5d0 \uace0\uc628\uc774 \ub418\uc5b4 \uc6d0\uc790\uac00 \ud06c\uac8c \uc9c4\ub3d9\ud558\uae30 \uc2dc\uc791\ud574\ub3c4 \uad6c\uba4d\uc774 \ucfe0\uc158\uc73c\ub85c \uc791\uc6a9\ud558\uc9c0 \ubabb\ud558\ubbc0\ub85c, \uc6d0\uc790 \uc0ac\uc774\uc758 \uac70\ub9ac\uac00 \ub113\uc5b4\uc9c0\ub294\ub370, \uc774\uac83\uc774 \ubc14\ub85c \uc5f4\ud33d\ucc3d\uc774\ub2e4. \uacb0\uc815\uc758 \uacbd\uc6b0\uc5d0\uc11c\ub3c4 \uc6d0\uc790\uac00 \ubc00\uc811\ud788 \uacb0\ud569\ud558\uc5ec \uad6c\uba4d\uc774 \uc5c6\ub294 \uac83\uc740 \uc5f4\ud33d\ucc3d\uc774 \ud06c\uace0, \uad6c\uba4d\uc774 \uc788\ub294 \uac83\uc740 \uc5f4\ud33d\ucc3d\uc758 \uc815\ub3c4\uac00 \ub0ae\ub2e4. \uc6d0\uc790\uac00 \ubc00\uc811\ud788 \uacb0\ud569\ub418\uc5b4 \uc788\ub294 \uac83\uc73c\ub85c\ub294 \uc5fc\ud654\ub098\ud2b8\ub968(\uc2dd\uc5fc)\uc774 \uc788\uc73c\uba70, 100\u2103\uc758 \uc628\ub3c4\ucc28\uc5d0\uc11c 0.4%\uc758 \ud33d\ucc3d\u00b7\uc218\ucd95\uc774 \uc77c\uc5b4\ub09c\ub2e4. \uad6c\uba4d\uc774 \uc788\ub294 \uac83\uc73c\ub85c\ub294 \ub2e4\uc774\uc544\ubaac\ub4dc\u00b7\uc11d\uc601 \ub4f1\uc774 \uc788\ub294\ub370 \uc5f4\ud33d\ucc3d\ub960\uc740 \uac70\uc758 0\uc774\ub2e4.", "answer": "\uc54c\uce7c\ub9ac", "sentence": "\uadf8\ub7ec\ub098 \uc18c\ub2e4\uc11d\ud68c\uc720\ub9ac\uc640 \uac19\uc774 \uc131\ud615\uc774\ub098 \uac00\uacf5\uc5d0 \ud3b8\ub9ac\ud558\ub3c4\ub85d \uc54c\uce7c\ub9ac \ub97c \uc11e\uc73c\uba74, \uc54c\uce7c\ub9ac\uc774\uc628\uc774 \uc6a9\ud574\ub418\uc5b4 \uc774 \uad6c\uba4d \uc18d\uc73c\ub85c \ub4e4\uc5b4\uac04\ub2e4.", "paragraph_sentence": "\uc720\ub9ac\uc758 \uad6c\uc870\uc5d0\ub294 \uad6c\uba4d\uc774 \uc788\uc5b4 \uace0\uc628\uc774 \ub418\uc5b4 \uc6d0\uc790\uac00 \ud06c\uac8c \uc9c4\ub3d9\ud558\uc5ec\ub3c4, \uc774 \uad6c\uba4d\uc774 \ucfe0\uc158\uacfc \uac19\uc740 \uc5ed\ud560\uc744 \ud558\uc5ec \uc798 \uae68\uc5b4\uc9c0\uc9c0 \uc54a\ub294\ub2e4. \uadf8\ub7ec\ub098 \uc18c\ub2e4\uc11d\ud68c\uc720\ub9ac\uc640 \uac19\uc774 \uc131\ud615\uc774\ub098 \uac00\uacf5\uc5d0 \ud3b8\ub9ac\ud558\ub3c4\ub85d \uc54c\uce7c\ub9ac \ub97c \uc11e\uc73c\uba74, \uc54c\uce7c\ub9ac\uc774\uc628\uc774 \uc6a9\ud574\ub418\uc5b4 \uc774 \uad6c\uba4d \uc18d\uc73c\ub85c \ub4e4\uc5b4\uac04\ub2e4. \uc774 \ub54c\ubb38\uc5d0 \uace0\uc628\uc774 \ub418\uc5b4 \uc6d0\uc790\uac00 \ud06c\uac8c \uc9c4\ub3d9\ud558\uae30 \uc2dc\uc791\ud574\ub3c4 \uad6c\uba4d\uc774 \ucfe0\uc158\uc73c\ub85c \uc791\uc6a9\ud558\uc9c0 \ubabb\ud558\ubbc0\ub85c, \uc6d0\uc790 \uc0ac\uc774\uc758 \uac70\ub9ac\uac00 \ub113\uc5b4\uc9c0\ub294\ub370, \uc774\uac83\uc774 \ubc14\ub85c \uc5f4\ud33d\ucc3d\uc774\ub2e4. \uacb0\uc815\uc758 \uacbd\uc6b0\uc5d0\uc11c\ub3c4 \uc6d0\uc790\uac00 \ubc00\uc811\ud788 \uacb0\ud569\ud558\uc5ec \uad6c\uba4d\uc774 \uc5c6\ub294 \uac83\uc740 \uc5f4\ud33d\ucc3d\uc774 \ud06c\uace0, \uad6c\uba4d\uc774 \uc788\ub294 \uac83\uc740 \uc5f4\ud33d\ucc3d\uc758 \uc815\ub3c4\uac00 \ub0ae\ub2e4. \uc6d0\uc790\uac00 \ubc00\uc811\ud788 \uacb0\ud569\ub418\uc5b4 \uc788\ub294 \uac83\uc73c\ub85c\ub294 \uc5fc\ud654\ub098\ud2b8\ub968(\uc2dd\uc5fc)\uc774 \uc788\uc73c\uba70, 100\u2103\uc758 \uc628\ub3c4\ucc28\uc5d0\uc11c 0.4%\uc758 \ud33d\ucc3d\u00b7\uc218\ucd95\uc774 \uc77c\uc5b4\ub09c\ub2e4. \uad6c\uba4d\uc774 \uc788\ub294 \uac83\uc73c\ub85c\ub294 \ub2e4\uc774\uc544\ubaac\ub4dc\u00b7\uc11d\uc601 \ub4f1\uc774 \uc788\ub294\ub370 \uc5f4\ud33d\ucc3d\ub960\uc740 \uac70\uc758 0\uc774\ub2e4.", "paragraph_answer": "\uc720\ub9ac\uc758 \uad6c\uc870\uc5d0\ub294 \uad6c\uba4d\uc774 \uc788\uc5b4 \uace0\uc628\uc774 \ub418\uc5b4 \uc6d0\uc790\uac00 \ud06c\uac8c \uc9c4\ub3d9\ud558\uc5ec\ub3c4, \uc774 \uad6c\uba4d\uc774 \ucfe0\uc158\uacfc \uac19\uc740 \uc5ed\ud560\uc744 \ud558\uc5ec \uc798 \uae68\uc5b4\uc9c0\uc9c0 \uc54a\ub294\ub2e4. \uadf8\ub7ec\ub098 \uc18c\ub2e4\uc11d\ud68c\uc720\ub9ac\uc640 \uac19\uc774 \uc131\ud615\uc774\ub098 \uac00\uacf5\uc5d0 \ud3b8\ub9ac\ud558\ub3c4\ub85d \uc54c\uce7c\ub9ac \ub97c \uc11e\uc73c\uba74, \uc54c\uce7c\ub9ac\uc774\uc628\uc774 \uc6a9\ud574\ub418\uc5b4 \uc774 \uad6c\uba4d \uc18d\uc73c\ub85c \ub4e4\uc5b4\uac04\ub2e4. \uc774 \ub54c\ubb38\uc5d0 \uace0\uc628\uc774 \ub418\uc5b4 \uc6d0\uc790\uac00 \ud06c\uac8c \uc9c4\ub3d9\ud558\uae30 \uc2dc\uc791\ud574\ub3c4 \uad6c\uba4d\uc774 \ucfe0\uc158\uc73c\ub85c \uc791\uc6a9\ud558\uc9c0 \ubabb\ud558\ubbc0\ub85c, \uc6d0\uc790 \uc0ac\uc774\uc758 \uac70\ub9ac\uac00 \ub113\uc5b4\uc9c0\ub294\ub370, \uc774\uac83\uc774 \ubc14\ub85c \uc5f4\ud33d\ucc3d\uc774\ub2e4. \uacb0\uc815\uc758 \uacbd\uc6b0\uc5d0\uc11c\ub3c4 \uc6d0\uc790\uac00 \ubc00\uc811\ud788 \uacb0\ud569\ud558\uc5ec \uad6c\uba4d\uc774 \uc5c6\ub294 \uac83\uc740 \uc5f4\ud33d\ucc3d\uc774 \ud06c\uace0, \uad6c\uba4d\uc774 \uc788\ub294 \uac83\uc740 \uc5f4\ud33d\ucc3d\uc758 \uc815\ub3c4\uac00 \ub0ae\ub2e4. \uc6d0\uc790\uac00 \ubc00\uc811\ud788 \uacb0\ud569\ub418\uc5b4 \uc788\ub294 \uac83\uc73c\ub85c\ub294 \uc5fc\ud654\ub098\ud2b8\ub968(\uc2dd\uc5fc)\uc774 \uc788\uc73c\uba70, 100\u2103\uc758 \uc628\ub3c4\ucc28\uc5d0\uc11c 0.4%\uc758 \ud33d\ucc3d\u00b7\uc218\ucd95\uc774 \uc77c\uc5b4\ub09c\ub2e4. \uad6c\uba4d\uc774 \uc788\ub294 \uac83\uc73c\ub85c\ub294 \ub2e4\uc774\uc544\ubaac\ub4dc\u00b7\uc11d\uc601 \ub4f1\uc774 \uc788\ub294\ub370 \uc5f4\ud33d\ucc3d\ub960\uc740 \uac70\uc758 0\uc774\ub2e4.", "sentence_answer": "\uadf8\ub7ec\ub098 \uc18c\ub2e4\uc11d\ud68c\uc720\ub9ac\uc640 \uac19\uc774 \uc131\ud615\uc774\ub098 \uac00\uacf5\uc5d0 \ud3b8\ub9ac\ud558\ub3c4\ub85d \uc54c\uce7c\ub9ac \ub97c \uc11e\uc73c\uba74, \uc54c\uce7c\ub9ac\uc774\uc628\uc774 \uc6a9\ud574\ub418\uc5b4 \uc774 \uad6c\uba4d \uc18d\uc73c\ub85c \ub4e4\uc5b4\uac04\ub2e4.", "paragraph_id": "6147943-0-0", "paragraph_question": "question: \uc131\ud615\uc774\ub098 \uac00\uacf5\uc5d0 \ud3b8\ub9ac\ud558\ub3c4\ub85d \uc720\ub9ac \uc548\uc5d0 \uc11e\ub294 \uac83\uc740?, context: \uc720\ub9ac\uc758 \uad6c\uc870\uc5d0\ub294 \uad6c\uba4d\uc774 \uc788\uc5b4 \uace0\uc628\uc774 \ub418\uc5b4 \uc6d0\uc790\uac00 \ud06c\uac8c \uc9c4\ub3d9\ud558\uc5ec\ub3c4, \uc774 \uad6c\uba4d\uc774 \ucfe0\uc158\uacfc \uac19\uc740 \uc5ed\ud560\uc744 \ud558\uc5ec \uc798 \uae68\uc5b4\uc9c0\uc9c0 \uc54a\ub294\ub2e4. \uadf8\ub7ec\ub098 \uc18c\ub2e4\uc11d\ud68c\uc720\ub9ac\uc640 \uac19\uc774 \uc131\ud615\uc774\ub098 \uac00\uacf5\uc5d0 \ud3b8\ub9ac\ud558\ub3c4\ub85d \uc54c\uce7c\ub9ac\ub97c \uc11e\uc73c\uba74, \uc54c\uce7c\ub9ac\uc774\uc628\uc774 \uc6a9\ud574\ub418\uc5b4 \uc774 \uad6c\uba4d \uc18d\uc73c\ub85c \ub4e4\uc5b4\uac04\ub2e4. \uc774 \ub54c\ubb38\uc5d0 \uace0\uc628\uc774 \ub418\uc5b4 \uc6d0\uc790\uac00 \ud06c\uac8c \uc9c4\ub3d9\ud558\uae30 \uc2dc\uc791\ud574\ub3c4 \uad6c\uba4d\uc774 \ucfe0\uc158\uc73c\ub85c \uc791\uc6a9\ud558\uc9c0 \ubabb\ud558\ubbc0\ub85c, \uc6d0\uc790 \uc0ac\uc774\uc758 \uac70\ub9ac\uac00 \ub113\uc5b4\uc9c0\ub294\ub370, \uc774\uac83\uc774 \ubc14\ub85c \uc5f4\ud33d\ucc3d\uc774\ub2e4. \uacb0\uc815\uc758 \uacbd\uc6b0\uc5d0\uc11c\ub3c4 \uc6d0\uc790\uac00 \ubc00\uc811\ud788 \uacb0\ud569\ud558\uc5ec \uad6c\uba4d\uc774 \uc5c6\ub294 \uac83\uc740 \uc5f4\ud33d\ucc3d\uc774 \ud06c\uace0, \uad6c\uba4d\uc774 \uc788\ub294 \uac83\uc740 \uc5f4\ud33d\ucc3d\uc758 \uc815\ub3c4\uac00 \ub0ae\ub2e4. \uc6d0\uc790\uac00 \ubc00\uc811\ud788 \uacb0\ud569\ub418\uc5b4 \uc788\ub294 \uac83\uc73c\ub85c\ub294 \uc5fc\ud654\ub098\ud2b8\ub968(\uc2dd\uc5fc)\uc774 \uc788\uc73c\uba70, 100\u2103\uc758 \uc628\ub3c4\ucc28\uc5d0\uc11c 0.4%\uc758 \ud33d\ucc3d\u00b7\uc218\ucd95\uc774 \uc77c\uc5b4\ub09c\ub2e4. \uad6c\uba4d\uc774 \uc788\ub294 \uac83\uc73c\ub85c\ub294 \ub2e4\uc774\uc544\ubaac\ub4dc\u00b7\uc11d\uc601 \ub4f1\uc774 \uc788\ub294\ub370 \uc5f4\ud33d\ucc3d\ub960\uc740 \uac70\uc758 0\uc774\ub2e4."} {"question": "\uad6d\ud638\ub97c \ub300\ud55c\uc81c\uad6d\uc73c\ub85c \uc120\ud3ec\ud55c \ub144\ub3c4\ub294?", "paragraph": "\uc774\uc5b4 1897\ub144(\uad11\ubb34 \uc6d0\ub144), \uad6d\ud638\ub97c \ub300\ud55c\uc81c\uad6d\uc73c\ub85c \uc120\ud3ec\ud558\uace0 \ud669\uc81c\uc5d0 \uc624\ub978 \uace0\uc885\ud669\uc81c\ub294 \ud669\uac00\uc758 \uc815\ud1b5\uc131\uc744 \uc704\ud574 \ud669\uac00\uc758 \uc9c1\uacc4 \uc870\uc0c1\uc778 \uc7a5\uc885, \uc815\uc885, \uc21c\uc870, \uc775\uc885\uc744 \ubaa8\ub450 \ud669\uc81c\ub85c \ub192\uc774\uace0, \uc7a5\uc885, \uc815\uc885, \uc775\uc885\uc758 \ubb18\ud638 \ub610\ud55c \ub192\uc5ec \uac01\uac01 \uc7a5\uc870, \uc815\uc870, \ubb38\uc870\ub85c \uace0\ucce4\ub2e4. \ub610\ud55c 1908\ub144(\uc735\ud76c 2\ub144)\uc5d0 \uc21c\uc885\ud669\uc81c\ub294 \ud669\uc81c\uad6d\uc758 \uc885\ubb18 \uc81c\ub3c4\uc778 \uce60\ubb18\uc81c\uc5d0 \ub530\ub77c \uc9c4\uc885, \ud5cc\uc885, \ucca0\uc885\ub3c4 \ubaa8\ub450 \ud669\uc81c\ub85c \ucd94\uc22d\ud558\uc600\ub2e4. 1910\ub144(\uc735\ud76c 4\ub144), \ud55c\uc77c\ud569\ubcd1\uc870\uc57d\uc73c\ub85c \uc778\ud574 \ub300\ud55c\uc81c\uad6d\uc774 \uba78\ub9dd\ud55c \ud6c4 \uc774\ubbf8 \ubb3c\ub7ec\ub098\uc788\ub358 \uace0\uc885\uacfc \uc21c\uc885\uc740 \uac01\uac01 \ub355\uc218\uad81 \uc774\ud0dc\uc655, \ucc3d\ub355\uad81 \uc774\uc655 \ub4f1\uc73c\ub85c \uaca9\ud558\ub418\uc5c8\ub2e4. 1919\ub144\uc5d0 \uace0\uc885\uc774, 1926\ub144\uc5d0 \uc21c\uc885\uc774 \uc138\uc0c1\uc744 \ub5a0\ub0a0 \ub54c, \ubcf8\ub798 \ub9dd\uad6d\uc758 \uad70\uc8fc\uc5d0\uac8c \ubb18\ud638\ub97c \ubd99\uc774\ub294 \uc77c\uc740 \ub4dc\ubb3c\uc5c8\uc73c\ub098 \uc774\uc655\uc9c1\uacfc \uc870\uc120\ucd1d\ub3c5\ubd80\uc5d0\uc11c \uc870\uc120 \uc655\uc2e4\uc758 \uc608\uc640 \ud615\uc2dd\uc5d0 \ub530\ub77c \ubb18\ud638\ub97c \uc62c\ub9ac\uae30\ub3c4 \ud588\ub2e4.", "answer": "1897\ub144", "sentence": "\uc774\uc5b4 1897\ub144 (\uad11\ubb34 \uc6d0\ub144), \uad6d\ud638\ub97c \ub300\ud55c\uc81c\uad6d\uc73c\ub85c \uc120\ud3ec\ud558\uace0 \ud669\uc81c\uc5d0 \uc624\ub978 \uace0\uc885\ud669\uc81c\ub294 \ud669\uac00\uc758 \uc815\ud1b5\uc131\uc744 \uc704\ud574 \ud669\uac00\uc758 \uc9c1\uacc4 \uc870\uc0c1\uc778 \uc7a5\uc885, \uc815\uc885, \uc21c\uc870, \uc775\uc885\uc744 \ubaa8\ub450 \ud669\uc81c\ub85c \ub192\uc774\uace0, \uc7a5\uc885, \uc815\uc885, \uc775\uc885\uc758 \ubb18\ud638 \ub610\ud55c \ub192\uc5ec \uac01\uac01 \uc7a5\uc870, \uc815\uc870, \ubb38\uc870\ub85c \uace0\ucce4\ub2e4.", "paragraph_sentence": " \uc774\uc5b4 1897\ub144 (\uad11\ubb34 \uc6d0\ub144), \uad6d\ud638\ub97c \ub300\ud55c\uc81c\uad6d\uc73c\ub85c \uc120\ud3ec\ud558\uace0 \ud669\uc81c\uc5d0 \uc624\ub978 \uace0\uc885\ud669\uc81c\ub294 \ud669\uac00\uc758 \uc815\ud1b5\uc131\uc744 \uc704\ud574 \ud669\uac00\uc758 \uc9c1\uacc4 \uc870\uc0c1\uc778 \uc7a5\uc885, \uc815\uc885, \uc21c\uc870, \uc775\uc885\uc744 \ubaa8\ub450 \ud669\uc81c\ub85c \ub192\uc774\uace0, \uc7a5\uc885, \uc815\uc885, \uc775\uc885\uc758 \ubb18\ud638 \ub610\ud55c \ub192\uc5ec \uac01\uac01 \uc7a5\uc870, \uc815\uc870, \ubb38\uc870\ub85c \uace0\ucce4\ub2e4. \ub610\ud55c 1908\ub144(\uc735\ud76c 2\ub144)\uc5d0 \uc21c\uc885\ud669\uc81c\ub294 \ud669\uc81c\uad6d\uc758 \uc885\ubb18 \uc81c\ub3c4\uc778 \uce60\ubb18\uc81c\uc5d0 \ub530\ub77c \uc9c4\uc885, \ud5cc\uc885, \ucca0\uc885\ub3c4 \ubaa8\ub450 \ud669\uc81c\ub85c \ucd94\uc22d\ud558\uc600\ub2e4. 1910\ub144(\uc735\ud76c 4\ub144), \ud55c\uc77c\ud569\ubcd1\uc870\uc57d\uc73c\ub85c \uc778\ud574 \ub300\ud55c\uc81c\uad6d\uc774 \uba78\ub9dd\ud55c \ud6c4 \uc774\ubbf8 \ubb3c\ub7ec\ub098\uc788\ub358 \uace0\uc885\uacfc \uc21c\uc885\uc740 \uac01\uac01 \ub355\uc218\uad81 \uc774\ud0dc\uc655, \ucc3d\ub355\uad81 \uc774\uc655 \ub4f1\uc73c\ub85c \uaca9\ud558\ub418\uc5c8\ub2e4. 1919\ub144\uc5d0 \uace0\uc885\uc774, 1926\ub144\uc5d0 \uc21c\uc885\uc774 \uc138\uc0c1\uc744 \ub5a0\ub0a0 \ub54c, \ubcf8\ub798 \ub9dd\uad6d\uc758 \uad70\uc8fc\uc5d0\uac8c \ubb18\ud638\ub97c \ubd99\uc774\ub294 \uc77c\uc740 \ub4dc\ubb3c\uc5c8\uc73c\ub098 \uc774\uc655\uc9c1\uacfc \uc870\uc120\ucd1d\ub3c5\ubd80\uc5d0\uc11c \uc870\uc120 \uc655\uc2e4\uc758 \uc608\uc640 \ud615\uc2dd\uc5d0 \ub530\ub77c \ubb18\ud638\ub97c \uc62c\ub9ac\uae30\ub3c4 \ud588\ub2e4.", "paragraph_answer": "\uc774\uc5b4 1897\ub144 (\uad11\ubb34 \uc6d0\ub144), \uad6d\ud638\ub97c \ub300\ud55c\uc81c\uad6d\uc73c\ub85c \uc120\ud3ec\ud558\uace0 \ud669\uc81c\uc5d0 \uc624\ub978 \uace0\uc885\ud669\uc81c\ub294 \ud669\uac00\uc758 \uc815\ud1b5\uc131\uc744 \uc704\ud574 \ud669\uac00\uc758 \uc9c1\uacc4 \uc870\uc0c1\uc778 \uc7a5\uc885, \uc815\uc885, \uc21c\uc870, \uc775\uc885\uc744 \ubaa8\ub450 \ud669\uc81c\ub85c \ub192\uc774\uace0, \uc7a5\uc885, \uc815\uc885, \uc775\uc885\uc758 \ubb18\ud638 \ub610\ud55c \ub192\uc5ec \uac01\uac01 \uc7a5\uc870, \uc815\uc870, \ubb38\uc870\ub85c \uace0\ucce4\ub2e4. \ub610\ud55c 1908\ub144(\uc735\ud76c 2\ub144)\uc5d0 \uc21c\uc885\ud669\uc81c\ub294 \ud669\uc81c\uad6d\uc758 \uc885\ubb18 \uc81c\ub3c4\uc778 \uce60\ubb18\uc81c\uc5d0 \ub530\ub77c \uc9c4\uc885, \ud5cc\uc885, \ucca0\uc885\ub3c4 \ubaa8\ub450 \ud669\uc81c\ub85c \ucd94\uc22d\ud558\uc600\ub2e4. 1910\ub144(\uc735\ud76c 4\ub144), \ud55c\uc77c\ud569\ubcd1\uc870\uc57d\uc73c\ub85c \uc778\ud574 \ub300\ud55c\uc81c\uad6d\uc774 \uba78\ub9dd\ud55c \ud6c4 \uc774\ubbf8 \ubb3c\ub7ec\ub098\uc788\ub358 \uace0\uc885\uacfc \uc21c\uc885\uc740 \uac01\uac01 \ub355\uc218\uad81 \uc774\ud0dc\uc655, \ucc3d\ub355\uad81 \uc774\uc655 \ub4f1\uc73c\ub85c \uaca9\ud558\ub418\uc5c8\ub2e4. 1919\ub144\uc5d0 \uace0\uc885\uc774, 1926\ub144\uc5d0 \uc21c\uc885\uc774 \uc138\uc0c1\uc744 \ub5a0\ub0a0 \ub54c, \ubcf8\ub798 \ub9dd\uad6d\uc758 \uad70\uc8fc\uc5d0\uac8c \ubb18\ud638\ub97c \ubd99\uc774\ub294 \uc77c\uc740 \ub4dc\ubb3c\uc5c8\uc73c\ub098 \uc774\uc655\uc9c1\uacfc \uc870\uc120\ucd1d\ub3c5\ubd80\uc5d0\uc11c \uc870\uc120 \uc655\uc2e4\uc758 \uc608\uc640 \ud615\uc2dd\uc5d0 \ub530\ub77c \ubb18\ud638\ub97c \uc62c\ub9ac\uae30\ub3c4 \ud588\ub2e4.", "sentence_answer": "\uc774\uc5b4 1897\ub144 (\uad11\ubb34 \uc6d0\ub144), \uad6d\ud638\ub97c \ub300\ud55c\uc81c\uad6d\uc73c\ub85c \uc120\ud3ec\ud558\uace0 \ud669\uc81c\uc5d0 \uc624\ub978 \uace0\uc885\ud669\uc81c\ub294 \ud669\uac00\uc758 \uc815\ud1b5\uc131\uc744 \uc704\ud574 \ud669\uac00\uc758 \uc9c1\uacc4 \uc870\uc0c1\uc778 \uc7a5\uc885, \uc815\uc885, \uc21c\uc870, \uc775\uc885\uc744 \ubaa8\ub450 \ud669\uc81c\ub85c \ub192\uc774\uace0, \uc7a5\uc885, \uc815\uc885, \uc775\uc885\uc758 \ubb18\ud638 \ub610\ud55c \ub192\uc5ec \uac01\uac01 \uc7a5\uc870, \uc815\uc870, \ubb38\uc870\ub85c \uace0\ucce4\ub2e4.", "paragraph_id": "6657037-11-0", "paragraph_question": "question: \uad6d\ud638\ub97c \ub300\ud55c\uc81c\uad6d\uc73c\ub85c \uc120\ud3ec\ud55c \ub144\ub3c4\ub294?, context: \uc774\uc5b4 1897\ub144(\uad11\ubb34 \uc6d0\ub144), \uad6d\ud638\ub97c \ub300\ud55c\uc81c\uad6d\uc73c\ub85c \uc120\ud3ec\ud558\uace0 \ud669\uc81c\uc5d0 \uc624\ub978 \uace0\uc885\ud669\uc81c\ub294 \ud669\uac00\uc758 \uc815\ud1b5\uc131\uc744 \uc704\ud574 \ud669\uac00\uc758 \uc9c1\uacc4 \uc870\uc0c1\uc778 \uc7a5\uc885, \uc815\uc885, \uc21c\uc870, \uc775\uc885\uc744 \ubaa8\ub450 \ud669\uc81c\ub85c \ub192\uc774\uace0, \uc7a5\uc885, \uc815\uc885, \uc775\uc885\uc758 \ubb18\ud638 \ub610\ud55c \ub192\uc5ec \uac01\uac01 \uc7a5\uc870, \uc815\uc870, \ubb38\uc870\ub85c \uace0\ucce4\ub2e4. \ub610\ud55c 1908\ub144(\uc735\ud76c 2\ub144)\uc5d0 \uc21c\uc885\ud669\uc81c\ub294 \ud669\uc81c\uad6d\uc758 \uc885\ubb18 \uc81c\ub3c4\uc778 \uce60\ubb18\uc81c\uc5d0 \ub530\ub77c \uc9c4\uc885, \ud5cc\uc885, \ucca0\uc885\ub3c4 \ubaa8\ub450 \ud669\uc81c\ub85c \ucd94\uc22d\ud558\uc600\ub2e4. 1910\ub144(\uc735\ud76c 4\ub144), \ud55c\uc77c\ud569\ubcd1\uc870\uc57d\uc73c\ub85c \uc778\ud574 \ub300\ud55c\uc81c\uad6d\uc774 \uba78\ub9dd\ud55c \ud6c4 \uc774\ubbf8 \ubb3c\ub7ec\ub098\uc788\ub358 \uace0\uc885\uacfc \uc21c\uc885\uc740 \uac01\uac01 \ub355\uc218\uad81 \uc774\ud0dc\uc655, \ucc3d\ub355\uad81 \uc774\uc655 \ub4f1\uc73c\ub85c \uaca9\ud558\ub418\uc5c8\ub2e4. 1919\ub144\uc5d0 \uace0\uc885\uc774, 1926\ub144\uc5d0 \uc21c\uc885\uc774 \uc138\uc0c1\uc744 \ub5a0\ub0a0 \ub54c, \ubcf8\ub798 \ub9dd\uad6d\uc758 \uad70\uc8fc\uc5d0\uac8c \ubb18\ud638\ub97c \ubd99\uc774\ub294 \uc77c\uc740 \ub4dc\ubb3c\uc5c8\uc73c\ub098 \uc774\uc655\uc9c1\uacfc \uc870\uc120\ucd1d\ub3c5\ubd80\uc5d0\uc11c \uc870\uc120 \uc655\uc2e4\uc758 \uc608\uc640 \ud615\uc2dd\uc5d0 \ub530\ub77c \ubb18\ud638\ub97c \uc62c\ub9ac\uae30\ub3c4 \ud588\ub2e4."} {"question": "\uc6d0\uc2dc \ud589\uc131 \uc911 \uc77c\ubd80\uac00 \uc6d0\ub798\uc758 \uada4\ub3c4\uc5d0\uc11c \uc774\ud0c8\ud558\uac8c \ub41c \uac83\uc740 \ubaa9\uc131\uc758 \ubb34\uc5c7 \ub54c\ubb38\uc778\uac00?", "paragraph": "\ubaa9\uc131\uc774 \ud0c4\uc0dd\ud558\uace0 \ud0dc\uc591\uc5d0 \ub354\uc6b1 \uac00\uae4c\uc6b4 \uacf3\uc73c\ub85c \uada4\ub3c4\ub97c \uc62e\uae30\uba74\uc11c, \uada4\ub3c4 \uacf5\uba85 \ub54c\ubb38\uc5d0 \uc18c\ud589\uc131\ub300\uc758 \ucc9c\uccb4\ub4e4\uc740 \ud769\uc5b4\uc84c\uc744 \uac83\uc774\uba70, \ub3d9\uc5ed\ud559\uc801\uc73c\ub85c \uc18c\ud589\uc131\ub300\uc758 \ucc9c\uccb4\ub4e4\uc758 \uada4\ub3c4\ub294 \ubd88\uaddc\uce59\ud574\uc84c\uace0, \uc11c\ub85c\uc5d0 \ub300\ud55c \uacf5\uc804 \uc18d\ub3c4\ub294 \uc62c\ub77c\uac14\uc744 \uac83\uc774\ub2e4. \uc6d0\uc2dc \ud589\uc131\ub4e4\uc758 \uada4\ub3c4 \uacf5\uba85\uc73c\ub85c, \uc18c\ud589\uc131\ub300\uc5d0 \uc874\uc7ac\ud588\ub358 \ubbf8\ud589\uc131\ub4e4\uc740 \ud769\uc5b4\uc84c\uac70\ub098 \ud639\uc740 \uada4\ub3c4 \uacbd\uc0ac\uac01\uacfc \uada4\ub3c4 \uc774\uc2ec\ub960\uc774 \ubd88\uaddc\uce59\ud574\uc84c\ub2e4. \uc774 \uc6d0\uc2dc \ud589\uc131 \uc911 \uc77c\ubd80\ub294 \ubaa9\uc131\uc758 \uc911\ub825\uc5d0 \uc774\ub04c\ub824 \uc6d0\ub798\uc758 \uada4\ub3c4\uc5d0\uc11c \uc774\ud0c8\ud558\uac8c \ub418\uc5c8\uace0, \ub098\uba38\uc9c0\ub294 \ub0b4\ud589\uc131 \uc9c0\ub300\ub85c \uada4\ub3c4\ub97c \uc62e\uacbc\uc73c\uba70, \uc774\ub85c\uc368 \uc9c0\uad6c\ud615 \ud589\uc131\uc758 \ub9c8\uc9c0\ub9c9 \uac15\ucc29 \ub2e8\uacc4\uac00 \uc774\ub8e8\uc5b4\uc9c4\ub2e4. \uc774 \u2018\uccab \ubc88\uc9f8 \uace0\uac08\uc758 \uc2dc\uae30\u2019 \ub3d9\uc548, \uac00\uc2a4 \ud589\uc131 \ubc0f \uc6d0\uc2dc \ud589\uc131\uc740 \uc18c\ud589\uc131\ub300\uc5d0 \uc788\ub358 \ubb3c\uc9c8(\ub300\ubd80\ubd84 \uc791\uc740 \ubbf8\ud589\uc131\uc73c\ub85c \uad6c\uc131\ub418\uc5b4 \uc788\uc5c8\uc74c)\uc744 \ud604\uc7ac \uc9c0\uad6c \uc9c8\ub7c9\uc758 1\ud37c\uc13c\ud2b8 \uc218\uc900\ub9cc \ub0a8\uae30\uace0 \ub2e4 \ud769\uc5b4\uc838 \ubc84\ub838\ub2e4. \uadf8\ub7fc\uc5d0\ub3c4, \uc544\uc9c1 \uc774\ub54c \uc18c\ud589\uc131\ub300\uc5d0\ub294 \uc9c0\uae08\ubcf4\ub2e4 10~20\ubc30\ub294 \ub9ce\uc740 \ubb3c\uc9c8\uc774 \uc874\uc7ac\ud558\uace0 \uc788\uc5c8\ub2e4. \uc9c0\uae08 \ub0a8\uc544 \uc788\ub294 \uc18c\ud589\uc131\ub300 \ubb3c\uc9c8\uc758 \ucd1d \uc9c8\ub7c9\uc740 \uc9c0\uad6c\uc758 2\ucc9c \ubd84\uc758 1 \uc218\uc900\uc774\ub2e4. \u2018\ub450 \ubc88\uc9f8 \uace0\uac08\uc758 \uc2dc\uae30\u2019\ub294 \ubaa9\uc131\uacfc \ud1a0\uc131\uc774 \uc77c\uc2dc\uc801\uc778 2 \ub300 1 \uada4\ub3c4 \uacf5\uba85(\uc544\ub798 \ucc38\uace0)\uc744 \ubcf4\uc774\uba74\uc11c \ucc3e\uc544\uc654\ub2e4\uace0 \ucd94\uce21\ub418\uba70, \uc774\ub54c \uc18c\ud589\uc131\ub300\uc5d0 \uc788\ub358 \ubb3c\uc9c8\uc740 \ud604\uc7ac \ub0a8\uc544 \uc788\ub294 \uc218\uc900\uc744 \uc81c\uc678\ud558\uace0 \ub2e4\uc2dc \ud55c \ubc88 \ud769\uc5b4\uc838 \ub098\uac14\ub2e4.", "answer": "\uc911\ub825", "sentence": "\uc774 \uc6d0\uc2dc \ud589\uc131 \uc911 \uc77c\ubd80\ub294 \ubaa9\uc131\uc758 \uc911\ub825 \uc5d0", "paragraph_sentence": "\ubaa9\uc131\uc774 \ud0c4\uc0dd\ud558\uace0 \ud0dc\uc591\uc5d0 \ub354\uc6b1 \uac00\uae4c\uc6b4 \uacf3\uc73c\ub85c \uada4\ub3c4\ub97c \uc62e\uae30\uba74\uc11c, \uada4\ub3c4 \uacf5\uba85 \ub54c\ubb38\uc5d0 \uc18c\ud589\uc131\ub300\uc758 \ucc9c\uccb4\ub4e4\uc740 \ud769\uc5b4\uc84c\uc744 \uac83\uc774\uba70, \ub3d9\uc5ed\ud559\uc801\uc73c\ub85c \uc18c\ud589\uc131\ub300\uc758 \ucc9c\uccb4\ub4e4\uc758 \uada4\ub3c4\ub294 \ubd88\uaddc\uce59\ud574\uc84c\uace0, \uc11c\ub85c\uc5d0 \ub300\ud55c \uacf5\uc804 \uc18d\ub3c4\ub294 \uc62c\ub77c\uac14\uc744 \uac83\uc774\ub2e4. \uc6d0\uc2dc \ud589\uc131\ub4e4\uc758 \uada4\ub3c4 \uacf5\uba85\uc73c\ub85c, \uc18c\ud589\uc131\ub300\uc5d0 \uc874\uc7ac\ud588\ub358 \ubbf8\ud589\uc131\ub4e4\uc740 \ud769\uc5b4\uc84c\uac70\ub098 \ud639\uc740 \uada4\ub3c4 \uacbd\uc0ac\uac01\uacfc \uada4\ub3c4 \uc774\uc2ec\ub960\uc774 \ubd88\uaddc\uce59\ud574\uc84c\ub2e4. \uc774 \uc6d0\uc2dc \ud589\uc131 \uc911 \uc77c\ubd80\ub294 \ubaa9\uc131\uc758 \uc911\ub825 \uc5d0 \uc774\ub04c\ub824 \uc6d0\ub798\uc758 \uada4\ub3c4\uc5d0\uc11c \uc774\ud0c8\ud558\uac8c \ub418\uc5c8\uace0, \ub098\uba38\uc9c0\ub294 \ub0b4\ud589\uc131 \uc9c0\ub300\ub85c \uada4\ub3c4\ub97c \uc62e\uacbc\uc73c\uba70, \uc774\ub85c\uc368 \uc9c0\uad6c\ud615 \ud589\uc131\uc758 \ub9c8\uc9c0\ub9c9 \uac15\ucc29 \ub2e8\uacc4\uac00 \uc774\ub8e8\uc5b4\uc9c4\ub2e4. \uc774 \u2018\uccab \ubc88\uc9f8 \uace0\uac08\uc758 \uc2dc\uae30\u2019 \ub3d9\uc548, \uac00\uc2a4 \ud589\uc131 \ubc0f \uc6d0\uc2dc \ud589\uc131\uc740 \uc18c\ud589\uc131\ub300\uc5d0 \uc788\ub358 \ubb3c\uc9c8(\ub300\ubd80\ubd84 \uc791\uc740 \ubbf8\ud589\uc131\uc73c\ub85c \uad6c\uc131\ub418\uc5b4 \uc788\uc5c8\uc74c)\uc744 \ud604\uc7ac \uc9c0\uad6c \uc9c8\ub7c9\uc758 1\ud37c\uc13c\ud2b8 \uc218\uc900\ub9cc \ub0a8\uae30\uace0 \ub2e4 \ud769\uc5b4\uc838 \ubc84\ub838\ub2e4. \uadf8\ub7fc\uc5d0\ub3c4, \uc544\uc9c1 \uc774\ub54c \uc18c\ud589\uc131\ub300\uc5d0\ub294 \uc9c0\uae08\ubcf4\ub2e4 10~20\ubc30\ub294 \ub9ce\uc740 \ubb3c\uc9c8\uc774 \uc874\uc7ac\ud558\uace0 \uc788\uc5c8\ub2e4. \uc9c0\uae08 \ub0a8\uc544 \uc788\ub294 \uc18c\ud589\uc131\ub300 \ubb3c\uc9c8\uc758 \ucd1d \uc9c8\ub7c9\uc740 \uc9c0\uad6c\uc758 2\ucc9c \ubd84\uc758 1 \uc218\uc900\uc774\ub2e4. \u2018\ub450 \ubc88\uc9f8 \uace0\uac08\uc758 \uc2dc\uae30\u2019\ub294 \ubaa9\uc131\uacfc \ud1a0\uc131\uc774 \uc77c\uc2dc\uc801\uc778 2 \ub300 1 \uada4\ub3c4 \uacf5\uba85(\uc544\ub798 \ucc38\uace0)\uc744 \ubcf4\uc774\uba74\uc11c \ucc3e\uc544\uc654\ub2e4\uace0 \ucd94\uce21\ub418\uba70, \uc774\ub54c \uc18c\ud589\uc131\ub300\uc5d0 \uc788\ub358 \ubb3c\uc9c8\uc740 \ud604\uc7ac \ub0a8\uc544 \uc788\ub294 \uc218\uc900\uc744 \uc81c\uc678\ud558\uace0 \ub2e4\uc2dc \ud55c \ubc88 \ud769\uc5b4\uc838 \ub098\uac14\ub2e4.", "paragraph_answer": "\ubaa9\uc131\uc774 \ud0c4\uc0dd\ud558\uace0 \ud0dc\uc591\uc5d0 \ub354\uc6b1 \uac00\uae4c\uc6b4 \uacf3\uc73c\ub85c \uada4\ub3c4\ub97c \uc62e\uae30\uba74\uc11c, \uada4\ub3c4 \uacf5\uba85 \ub54c\ubb38\uc5d0 \uc18c\ud589\uc131\ub300\uc758 \ucc9c\uccb4\ub4e4\uc740 \ud769\uc5b4\uc84c\uc744 \uac83\uc774\uba70, \ub3d9\uc5ed\ud559\uc801\uc73c\ub85c \uc18c\ud589\uc131\ub300\uc758 \ucc9c\uccb4\ub4e4\uc758 \uada4\ub3c4\ub294 \ubd88\uaddc\uce59\ud574\uc84c\uace0, \uc11c\ub85c\uc5d0 \ub300\ud55c \uacf5\uc804 \uc18d\ub3c4\ub294 \uc62c\ub77c\uac14\uc744 \uac83\uc774\ub2e4. \uc6d0\uc2dc \ud589\uc131\ub4e4\uc758 \uada4\ub3c4 \uacf5\uba85\uc73c\ub85c, \uc18c\ud589\uc131\ub300\uc5d0 \uc874\uc7ac\ud588\ub358 \ubbf8\ud589\uc131\ub4e4\uc740 \ud769\uc5b4\uc84c\uac70\ub098 \ud639\uc740 \uada4\ub3c4 \uacbd\uc0ac\uac01\uacfc \uada4\ub3c4 \uc774\uc2ec\ub960\uc774 \ubd88\uaddc\uce59\ud574\uc84c\ub2e4. \uc774 \uc6d0\uc2dc \ud589\uc131 \uc911 \uc77c\ubd80\ub294 \ubaa9\uc131\uc758 \uc911\ub825 \uc5d0 \uc774\ub04c\ub824 \uc6d0\ub798\uc758 \uada4\ub3c4\uc5d0\uc11c \uc774\ud0c8\ud558\uac8c \ub418\uc5c8\uace0, \ub098\uba38\uc9c0\ub294 \ub0b4\ud589\uc131 \uc9c0\ub300\ub85c \uada4\ub3c4\ub97c \uc62e\uacbc\uc73c\uba70, \uc774\ub85c\uc368 \uc9c0\uad6c\ud615 \ud589\uc131\uc758 \ub9c8\uc9c0\ub9c9 \uac15\ucc29 \ub2e8\uacc4\uac00 \uc774\ub8e8\uc5b4\uc9c4\ub2e4. \uc774 \u2018\uccab \ubc88\uc9f8 \uace0\uac08\uc758 \uc2dc\uae30\u2019 \ub3d9\uc548, \uac00\uc2a4 \ud589\uc131 \ubc0f \uc6d0\uc2dc \ud589\uc131\uc740 \uc18c\ud589\uc131\ub300\uc5d0 \uc788\ub358 \ubb3c\uc9c8(\ub300\ubd80\ubd84 \uc791\uc740 \ubbf8\ud589\uc131\uc73c\ub85c \uad6c\uc131\ub418\uc5b4 \uc788\uc5c8\uc74c)\uc744 \ud604\uc7ac \uc9c0\uad6c \uc9c8\ub7c9\uc758 1\ud37c\uc13c\ud2b8 \uc218\uc900\ub9cc \ub0a8\uae30\uace0 \ub2e4 \ud769\uc5b4\uc838 \ubc84\ub838\ub2e4. \uadf8\ub7fc\uc5d0\ub3c4, \uc544\uc9c1 \uc774\ub54c \uc18c\ud589\uc131\ub300\uc5d0\ub294 \uc9c0\uae08\ubcf4\ub2e4 10~20\ubc30\ub294 \ub9ce\uc740 \ubb3c\uc9c8\uc774 \uc874\uc7ac\ud558\uace0 \uc788\uc5c8\ub2e4. \uc9c0\uae08 \ub0a8\uc544 \uc788\ub294 \uc18c\ud589\uc131\ub300 \ubb3c\uc9c8\uc758 \ucd1d \uc9c8\ub7c9\uc740 \uc9c0\uad6c\uc758 2\ucc9c \ubd84\uc758 1 \uc218\uc900\uc774\ub2e4. \u2018\ub450 \ubc88\uc9f8 \uace0\uac08\uc758 \uc2dc\uae30\u2019\ub294 \ubaa9\uc131\uacfc \ud1a0\uc131\uc774 \uc77c\uc2dc\uc801\uc778 2 \ub300 1 \uada4\ub3c4 \uacf5\uba85(\uc544\ub798 \ucc38\uace0)\uc744 \ubcf4\uc774\uba74\uc11c \ucc3e\uc544\uc654\ub2e4\uace0 \ucd94\uce21\ub418\uba70, \uc774\ub54c \uc18c\ud589\uc131\ub300\uc5d0 \uc788\ub358 \ubb3c\uc9c8\uc740 \ud604\uc7ac \ub0a8\uc544 \uc788\ub294 \uc218\uc900\uc744 \uc81c\uc678\ud558\uace0 \ub2e4\uc2dc \ud55c \ubc88 \ud769\uc5b4\uc838 \ub098\uac14\ub2e4.", "sentence_answer": "\uc774 \uc6d0\uc2dc \ud589\uc131 \uc911 \uc77c\ubd80\ub294 \ubaa9\uc131\uc758 \uc911\ub825 \uc5d0", "paragraph_id": "6598972-9-0", "paragraph_question": "question: \uc6d0\uc2dc \ud589\uc131 \uc911 \uc77c\ubd80\uac00 \uc6d0\ub798\uc758 \uada4\ub3c4\uc5d0\uc11c \uc774\ud0c8\ud558\uac8c \ub41c \uac83\uc740 \ubaa9\uc131\uc758 \ubb34\uc5c7 \ub54c\ubb38\uc778\uac00?, context: \ubaa9\uc131\uc774 \ud0c4\uc0dd\ud558\uace0 \ud0dc\uc591\uc5d0 \ub354\uc6b1 \uac00\uae4c\uc6b4 \uacf3\uc73c\ub85c \uada4\ub3c4\ub97c \uc62e\uae30\uba74\uc11c, \uada4\ub3c4 \uacf5\uba85 \ub54c\ubb38\uc5d0 \uc18c\ud589\uc131\ub300\uc758 \ucc9c\uccb4\ub4e4\uc740 \ud769\uc5b4\uc84c\uc744 \uac83\uc774\uba70, \ub3d9\uc5ed\ud559\uc801\uc73c\ub85c \uc18c\ud589\uc131\ub300\uc758 \ucc9c\uccb4\ub4e4\uc758 \uada4\ub3c4\ub294 \ubd88\uaddc\uce59\ud574\uc84c\uace0, \uc11c\ub85c\uc5d0 \ub300\ud55c \uacf5\uc804 \uc18d\ub3c4\ub294 \uc62c\ub77c\uac14\uc744 \uac83\uc774\ub2e4. \uc6d0\uc2dc \ud589\uc131\ub4e4\uc758 \uada4\ub3c4 \uacf5\uba85\uc73c\ub85c, \uc18c\ud589\uc131\ub300\uc5d0 \uc874\uc7ac\ud588\ub358 \ubbf8\ud589\uc131\ub4e4\uc740 \ud769\uc5b4\uc84c\uac70\ub098 \ud639\uc740 \uada4\ub3c4 \uacbd\uc0ac\uac01\uacfc \uada4\ub3c4 \uc774\uc2ec\ub960\uc774 \ubd88\uaddc\uce59\ud574\uc84c\ub2e4. \uc774 \uc6d0\uc2dc \ud589\uc131 \uc911 \uc77c\ubd80\ub294 \ubaa9\uc131\uc758 \uc911\ub825\uc5d0 \uc774\ub04c\ub824 \uc6d0\ub798\uc758 \uada4\ub3c4\uc5d0\uc11c \uc774\ud0c8\ud558\uac8c \ub418\uc5c8\uace0, \ub098\uba38\uc9c0\ub294 \ub0b4\ud589\uc131 \uc9c0\ub300\ub85c \uada4\ub3c4\ub97c \uc62e\uacbc\uc73c\uba70, \uc774\ub85c\uc368 \uc9c0\uad6c\ud615 \ud589\uc131\uc758 \ub9c8\uc9c0\ub9c9 \uac15\ucc29 \ub2e8\uacc4\uac00 \uc774\ub8e8\uc5b4\uc9c4\ub2e4. \uc774 \u2018\uccab \ubc88\uc9f8 \uace0\uac08\uc758 \uc2dc\uae30\u2019 \ub3d9\uc548, \uac00\uc2a4 \ud589\uc131 \ubc0f \uc6d0\uc2dc \ud589\uc131\uc740 \uc18c\ud589\uc131\ub300\uc5d0 \uc788\ub358 \ubb3c\uc9c8(\ub300\ubd80\ubd84 \uc791\uc740 \ubbf8\ud589\uc131\uc73c\ub85c \uad6c\uc131\ub418\uc5b4 \uc788\uc5c8\uc74c)\uc744 \ud604\uc7ac \uc9c0\uad6c \uc9c8\ub7c9\uc758 1\ud37c\uc13c\ud2b8 \uc218\uc900\ub9cc \ub0a8\uae30\uace0 \ub2e4 \ud769\uc5b4\uc838 \ubc84\ub838\ub2e4. \uadf8\ub7fc\uc5d0\ub3c4, \uc544\uc9c1 \uc774\ub54c \uc18c\ud589\uc131\ub300\uc5d0\ub294 \uc9c0\uae08\ubcf4\ub2e4 10~20\ubc30\ub294 \ub9ce\uc740 \ubb3c\uc9c8\uc774 \uc874\uc7ac\ud558\uace0 \uc788\uc5c8\ub2e4. \uc9c0\uae08 \ub0a8\uc544 \uc788\ub294 \uc18c\ud589\uc131\ub300 \ubb3c\uc9c8\uc758 \ucd1d \uc9c8\ub7c9\uc740 \uc9c0\uad6c\uc758 2\ucc9c \ubd84\uc758 1 \uc218\uc900\uc774\ub2e4. \u2018\ub450 \ubc88\uc9f8 \uace0\uac08\uc758 \uc2dc\uae30\u2019\ub294 \ubaa9\uc131\uacfc \ud1a0\uc131\uc774 \uc77c\uc2dc\uc801\uc778 2 \ub300 1 \uada4\ub3c4 \uacf5\uba85(\uc544\ub798 \ucc38\uace0)\uc744 \ubcf4\uc774\uba74\uc11c \ucc3e\uc544\uc654\ub2e4\uace0 \ucd94\uce21\ub418\uba70, \uc774\ub54c \uc18c\ud589\uc131\ub300\uc5d0 \uc788\ub358 \ubb3c\uc9c8\uc740 \ud604\uc7ac \ub0a8\uc544 \uc788\ub294 \uc218\uc900\uc744 \uc81c\uc678\ud558\uace0 \ub2e4\uc2dc \ud55c \ubc88 \ud769\uc5b4\uc838 \ub098\uac14\ub2e4."} {"question": "\ub098\uce74\ubaa8\ub9ac \uc544\ud0a4\ub098\uac00 \ub300\ud45c\ud55c \uc774\ubbf8\uc9c0\ub294 \ubb34\uc5c7\uc77c\uae4c?", "paragraph": "1980\ub144\ub300 \ucd08\ubc18\uc5d0 \uc5ed\uc2dc \uc77c\ubcf8 \uc5ec\uc790 \uc544\uc774\ub3cc\uacc4\ub97c \uc774\ub04c\ub358 \ub77c\uc774\ubc8c \ub9c8\uce20\ub2e4 \uc138\uc774\ucf54\uc640\ub294 \uace1\uc870\uc640 \uce90\ub9ad\ud130 \uba74\uc5d0\uc11c \ub354\uc6b1 \uadf9\uba85\ud55c \ucc28\uc774\ub97c \ubcf4\uc600\ub294\ub370, \ub9c8\uce20\ub2e4\uac00 \ubd80\ub9bf\ucf54(\u3076\u308a\u3063\u5b50) \ucf58\uc149\ud2b8\ub85c \ub0a8\uc131\ud32c\ub4e4\uc744 \uacf5\ub7b5\ud588\ub2e4\uba74, \ub098\uce74\ubaa8\ub9ac\ub294 \uadf8\uc640 \ub300\ube44\ub418\ub294 \uc548\ud2f0 \ubd80\ub9bf\ucf54(\u30a2\u30f3\u30c1\u00b7\u3076\u308a\u3063\u5b50) \uc774\ubbf8\uc9c0\ub97c \ub300\ud45c\ud558\uc600\ub2e4. \uc774\ub178\uc6b0\uc5d0 \uc694\uc2a4\uc774\uc640 \uc791\uc5c5\ud55c \u3008\uc7a5\uc2dd\uc740 \uc544\ub2c8\uc57c, \ub208\ubb3c\uc740\u3009\uc73c\ub85c \uc544\uc774\ub3cc \uc774\ubbf8\uc9c0\uc5d0\uc11c \ud0c8\ud53c\ud568\uacfc \ub3d9\uc2dc\uc5d0 \uc0c8\ub85c\uc6b4 \uacbd\uc9c0\ub97c \uac1c\ucc99\ud558\uc5ec \ud6c4\ub300 \uc544\uc774\ub3cc\uc5d0\uac8c \uc601\ud5a5\uc744 \uc92c\ub2e4. \uc774 \uc774\ud6c4, \ub098\uce74\ubaa8\ub9ac\uc758 \ud2b9\uc0c9 \uc788\ub294 \uac00\ucc3d\ub825\uacfc \ub178\ub798\uc758 \uc5b4\ub450\uc6b4 \ubd84\uc704\uae30\ub294 \ub2f9\uc2dc \ubd88\uc548\uacfc \uc808\ub9dd\uc5d0 \ube60\uc838\uc788\ub294 \uc80a\uc740\uc774\ub4e4, \ud2b9\ud788 \uc5ec\uc131\ub4e4\uc758 \uacf5\uac10\uc744 \uc0ac\uba70 \uc778\uae30\ub97c \ub204\ub838\ub2e4. \ub610\ud55c \ub2f9\uc2dc \uc720\ud589\ud558\ub358 \ubc30\uae30 \ud32c\uce20 \ub4f1\uc758 \uc758\uc0c1\uc744 \uc801\uadf9\uc801\uc73c\ub85c \uc218\uc6a9\ud558\uc5ec \uc790\uc2e0\uc758 \ubb34\ub300\uc5d0 \ubc18\uc601\ud558\ub294 \ub4f1 \ud328\uc158 \ub9ac\ub354\ub85c\uc11c\uc758 \uba74\ubaa8\ub3c4 \ubcf4\uc5ec\uc8fc\uc5c8\ub2e4. \uadf8\ub7ec\ub098 \ub098\uce74\ubaa8\ub9ac\uc758 \ud589\ubcf4\uc5d0\ub3c4 \ube44\ud310\uc740 \ub530\ub974\ub294\ub370 \uadf8\ub140\uc758 \uc790\uc0b4\ubbf8\uc218 \uc0ac\uac74\uc744 \ube44\ub86f\ud558\uc5ec \ud6c4\ubc30 \uc544\uc774\ub3cc\uc774\uc5c8\ub358 \uc624\uce74\ub2e4 \uc720\ud0a4\ucf54\uc758 \ud22c\uc2e0\uc790\uc0b4 \ub4f1 \uc544\uc774\ub3cc \uc74c\uc545 \uc0b0\uc5c5\uc5d0\uc11c \ube44\ub86f\ub41c \uc77c\ub828\uc758 \uc0ac\uac74\ub4e4\uc774 \uc544\uc774\ub3cc \uc138\uacc4\uc5d0 \ub300\ud574 \ub9c9\uc5f0\ud55c \ub3d9\uacbd\uc744 \ud488\ub358 10\ub300 \uccad\uc18c\ub144\ub4e4\uc5d0\uac8c \uacbd\uc885\uc744 \uc6b8\ub838\ub2e4\uace0 \uc9c0\uc801\ud558\uc600\ub2e4.", "answer": "\uc548\ud2f0 \ubd80\ub9bf\ucf54", "sentence": "1980\ub144\ub300 \ucd08\ubc18\uc5d0 \uc5ed\uc2dc \uc77c\ubcf8 \uc5ec\uc790 \uc544\uc774\ub3cc\uacc4\ub97c \uc774\ub04c\ub358 \ub77c\uc774\ubc8c \ub9c8\uce20\ub2e4 \uc138\uc774\ucf54\uc640\ub294 \uace1\uc870\uc640 \uce90\ub9ad\ud130 \uba74\uc5d0\uc11c \ub354\uc6b1 \uadf9\uba85\ud55c \ucc28\uc774\ub97c \ubcf4\uc600\ub294\ub370, \ub9c8\uce20\ub2e4\uac00 \ubd80\ub9bf\ucf54(\u3076\u308a\u3063\u5b50) \ucf58\uc149\ud2b8\ub85c \ub0a8\uc131\ud32c\ub4e4\uc744 \uacf5\ub7b5\ud588\ub2e4\uba74, \ub098\uce74\ubaa8\ub9ac\ub294 \uadf8\uc640 \ub300\ube44\ub418\ub294 \uc548\ud2f0 \ubd80\ub9bf\ucf54 (\u30a2\u30f3\u30c1\u00b7\u3076\u308a\u3063\u5b50) \uc774\ubbf8\uc9c0\ub97c \ub300\ud45c\ud558\uc600\ub2e4.", "paragraph_sentence": " 1980\ub144\ub300 \ucd08\ubc18\uc5d0 \uc5ed\uc2dc \uc77c\ubcf8 \uc5ec\uc790 \uc544\uc774\ub3cc\uacc4\ub97c \uc774\ub04c\ub358 \ub77c\uc774\ubc8c \ub9c8\uce20\ub2e4 \uc138\uc774\ucf54\uc640\ub294 \uace1\uc870\uc640 \uce90\ub9ad\ud130 \uba74\uc5d0\uc11c \ub354\uc6b1 \uadf9\uba85\ud55c \ucc28\uc774\ub97c \ubcf4\uc600\ub294\ub370, \ub9c8\uce20\ub2e4\uac00 \ubd80\ub9bf\ucf54(\u3076\u308a\u3063\u5b50) \ucf58\uc149\ud2b8\ub85c \ub0a8\uc131\ud32c\ub4e4\uc744 \uacf5\ub7b5\ud588\ub2e4\uba74, \ub098\uce74\ubaa8\ub9ac\ub294 \uadf8\uc640 \ub300\ube44\ub418\ub294 \uc548\ud2f0 \ubd80\ub9bf\ucf54 (\u30a2\u30f3\u30c1\u00b7\u3076\u308a\u3063\u5b50) \uc774\ubbf8\uc9c0\ub97c \ub300\ud45c\ud558\uc600\ub2e4. \uc774\ub178\uc6b0\uc5d0 \uc694\uc2a4\uc774\uc640 \uc791\uc5c5\ud55c \u3008\uc7a5\uc2dd\uc740 \uc544\ub2c8\uc57c, \ub208\ubb3c\uc740\u3009\uc73c\ub85c \uc544\uc774\ub3cc \uc774\ubbf8\uc9c0\uc5d0\uc11c \ud0c8\ud53c\ud568\uacfc \ub3d9\uc2dc\uc5d0 \uc0c8\ub85c\uc6b4 \uacbd\uc9c0\ub97c \uac1c\ucc99\ud558\uc5ec \ud6c4\ub300 \uc544\uc774\ub3cc\uc5d0\uac8c \uc601\ud5a5\uc744 \uc92c\ub2e4. \uc774 \uc774\ud6c4, \ub098\uce74\ubaa8\ub9ac\uc758 \ud2b9\uc0c9 \uc788\ub294 \uac00\ucc3d\ub825\uacfc \ub178\ub798\uc758 \uc5b4\ub450\uc6b4 \ubd84\uc704\uae30\ub294 \ub2f9\uc2dc \ubd88\uc548\uacfc \uc808\ub9dd\uc5d0 \ube60\uc838\uc788\ub294 \uc80a\uc740\uc774\ub4e4, \ud2b9\ud788 \uc5ec\uc131\ub4e4\uc758 \uacf5\uac10\uc744 \uc0ac\uba70 \uc778\uae30\ub97c \ub204\ub838\ub2e4. \ub610\ud55c \ub2f9\uc2dc \uc720\ud589\ud558\ub358 \ubc30\uae30 \ud32c\uce20 \ub4f1\uc758 \uc758\uc0c1\uc744 \uc801\uadf9\uc801\uc73c\ub85c \uc218\uc6a9\ud558\uc5ec \uc790\uc2e0\uc758 \ubb34\ub300\uc5d0 \ubc18\uc601\ud558\ub294 \ub4f1 \ud328\uc158 \ub9ac\ub354\ub85c\uc11c\uc758 \uba74\ubaa8\ub3c4 \ubcf4\uc5ec\uc8fc\uc5c8\ub2e4. \uadf8\ub7ec\ub098 \ub098\uce74\ubaa8\ub9ac\uc758 \ud589\ubcf4\uc5d0\ub3c4 \ube44\ud310\uc740 \ub530\ub974\ub294\ub370 \uadf8\ub140\uc758 \uc790\uc0b4\ubbf8\uc218 \uc0ac\uac74\uc744 \ube44\ub86f\ud558\uc5ec \ud6c4\ubc30 \uc544\uc774\ub3cc\uc774\uc5c8\ub358 \uc624\uce74\ub2e4 \uc720\ud0a4\ucf54\uc758 \ud22c\uc2e0\uc790\uc0b4 \ub4f1 \uc544\uc774\ub3cc \uc74c\uc545 \uc0b0\uc5c5\uc5d0\uc11c \ube44\ub86f\ub41c \uc77c\ub828\uc758 \uc0ac\uac74\ub4e4\uc774 \uc544\uc774\ub3cc \uc138\uacc4\uc5d0 \ub300\ud574 \ub9c9\uc5f0\ud55c \ub3d9\uacbd\uc744 \ud488\ub358 10\ub300 \uccad\uc18c\ub144\ub4e4\uc5d0\uac8c \uacbd\uc885\uc744 \uc6b8\ub838\ub2e4\uace0 \uc9c0\uc801\ud558\uc600\ub2e4.", "paragraph_answer": "1980\ub144\ub300 \ucd08\ubc18\uc5d0 \uc5ed\uc2dc \uc77c\ubcf8 \uc5ec\uc790 \uc544\uc774\ub3cc\uacc4\ub97c \uc774\ub04c\ub358 \ub77c\uc774\ubc8c \ub9c8\uce20\ub2e4 \uc138\uc774\ucf54\uc640\ub294 \uace1\uc870\uc640 \uce90\ub9ad\ud130 \uba74\uc5d0\uc11c \ub354\uc6b1 \uadf9\uba85\ud55c \ucc28\uc774\ub97c \ubcf4\uc600\ub294\ub370, \ub9c8\uce20\ub2e4\uac00 \ubd80\ub9bf\ucf54(\u3076\u308a\u3063\u5b50) \ucf58\uc149\ud2b8\ub85c \ub0a8\uc131\ud32c\ub4e4\uc744 \uacf5\ub7b5\ud588\ub2e4\uba74, \ub098\uce74\ubaa8\ub9ac\ub294 \uadf8\uc640 \ub300\ube44\ub418\ub294 \uc548\ud2f0 \ubd80\ub9bf\ucf54 (\u30a2\u30f3\u30c1\u00b7\u3076\u308a\u3063\u5b50) \uc774\ubbf8\uc9c0\ub97c \ub300\ud45c\ud558\uc600\ub2e4. \uc774\ub178\uc6b0\uc5d0 \uc694\uc2a4\uc774\uc640 \uc791\uc5c5\ud55c \u3008\uc7a5\uc2dd\uc740 \uc544\ub2c8\uc57c, \ub208\ubb3c\uc740\u3009\uc73c\ub85c \uc544\uc774\ub3cc \uc774\ubbf8\uc9c0\uc5d0\uc11c \ud0c8\ud53c\ud568\uacfc \ub3d9\uc2dc\uc5d0 \uc0c8\ub85c\uc6b4 \uacbd\uc9c0\ub97c \uac1c\ucc99\ud558\uc5ec \ud6c4\ub300 \uc544\uc774\ub3cc\uc5d0\uac8c \uc601\ud5a5\uc744 \uc92c\ub2e4. \uc774 \uc774\ud6c4, \ub098\uce74\ubaa8\ub9ac\uc758 \ud2b9\uc0c9 \uc788\ub294 \uac00\ucc3d\ub825\uacfc \ub178\ub798\uc758 \uc5b4\ub450\uc6b4 \ubd84\uc704\uae30\ub294 \ub2f9\uc2dc \ubd88\uc548\uacfc \uc808\ub9dd\uc5d0 \ube60\uc838\uc788\ub294 \uc80a\uc740\uc774\ub4e4, \ud2b9\ud788 \uc5ec\uc131\ub4e4\uc758 \uacf5\uac10\uc744 \uc0ac\uba70 \uc778\uae30\ub97c \ub204\ub838\ub2e4. \ub610\ud55c \ub2f9\uc2dc \uc720\ud589\ud558\ub358 \ubc30\uae30 \ud32c\uce20 \ub4f1\uc758 \uc758\uc0c1\uc744 \uc801\uadf9\uc801\uc73c\ub85c \uc218\uc6a9\ud558\uc5ec \uc790\uc2e0\uc758 \ubb34\ub300\uc5d0 \ubc18\uc601\ud558\ub294 \ub4f1 \ud328\uc158 \ub9ac\ub354\ub85c\uc11c\uc758 \uba74\ubaa8\ub3c4 \ubcf4\uc5ec\uc8fc\uc5c8\ub2e4. \uadf8\ub7ec\ub098 \ub098\uce74\ubaa8\ub9ac\uc758 \ud589\ubcf4\uc5d0\ub3c4 \ube44\ud310\uc740 \ub530\ub974\ub294\ub370 \uadf8\ub140\uc758 \uc790\uc0b4\ubbf8\uc218 \uc0ac\uac74\uc744 \ube44\ub86f\ud558\uc5ec \ud6c4\ubc30 \uc544\uc774\ub3cc\uc774\uc5c8\ub358 \uc624\uce74\ub2e4 \uc720\ud0a4\ucf54\uc758 \ud22c\uc2e0\uc790\uc0b4 \ub4f1 \uc544\uc774\ub3cc \uc74c\uc545 \uc0b0\uc5c5\uc5d0\uc11c \ube44\ub86f\ub41c \uc77c\ub828\uc758 \uc0ac\uac74\ub4e4\uc774 \uc544\uc774\ub3cc \uc138\uacc4\uc5d0 \ub300\ud574 \ub9c9\uc5f0\ud55c \ub3d9\uacbd\uc744 \ud488\ub358 10\ub300 \uccad\uc18c\ub144\ub4e4\uc5d0\uac8c \uacbd\uc885\uc744 \uc6b8\ub838\ub2e4\uace0 \uc9c0\uc801\ud558\uc600\ub2e4.", "sentence_answer": "1980\ub144\ub300 \ucd08\ubc18\uc5d0 \uc5ed\uc2dc \uc77c\ubcf8 \uc5ec\uc790 \uc544\uc774\ub3cc\uacc4\ub97c \uc774\ub04c\ub358 \ub77c\uc774\ubc8c \ub9c8\uce20\ub2e4 \uc138\uc774\ucf54\uc640\ub294 \uace1\uc870\uc640 \uce90\ub9ad\ud130 \uba74\uc5d0\uc11c \ub354\uc6b1 \uadf9\uba85\ud55c \ucc28\uc774\ub97c \ubcf4\uc600\ub294\ub370, \ub9c8\uce20\ub2e4\uac00 \ubd80\ub9bf\ucf54(\u3076\u308a\u3063\u5b50) \ucf58\uc149\ud2b8\ub85c \ub0a8\uc131\ud32c\ub4e4\uc744 \uacf5\ub7b5\ud588\ub2e4\uba74, \ub098\uce74\ubaa8\ub9ac\ub294 \uadf8\uc640 \ub300\ube44\ub418\ub294 \uc548\ud2f0 \ubd80\ub9bf\ucf54 (\u30a2\u30f3\u30c1\u00b7\u3076\u308a\u3063\u5b50) \uc774\ubbf8\uc9c0\ub97c \ub300\ud45c\ud558\uc600\ub2e4.", "paragraph_id": "6533918-32-0", "paragraph_question": "question: \ub098\uce74\ubaa8\ub9ac \uc544\ud0a4\ub098\uac00 \ub300\ud45c\ud55c \uc774\ubbf8\uc9c0\ub294 \ubb34\uc5c7\uc77c\uae4c?, context: 1980\ub144\ub300 \ucd08\ubc18\uc5d0 \uc5ed\uc2dc \uc77c\ubcf8 \uc5ec\uc790 \uc544\uc774\ub3cc\uacc4\ub97c \uc774\ub04c\ub358 \ub77c\uc774\ubc8c \ub9c8\uce20\ub2e4 \uc138\uc774\ucf54\uc640\ub294 \uace1\uc870\uc640 \uce90\ub9ad\ud130 \uba74\uc5d0\uc11c \ub354\uc6b1 \uadf9\uba85\ud55c \ucc28\uc774\ub97c \ubcf4\uc600\ub294\ub370, \ub9c8\uce20\ub2e4\uac00 \ubd80\ub9bf\ucf54(\u3076\u308a\u3063\u5b50) \ucf58\uc149\ud2b8\ub85c \ub0a8\uc131\ud32c\ub4e4\uc744 \uacf5\ub7b5\ud588\ub2e4\uba74, \ub098\uce74\ubaa8\ub9ac\ub294 \uadf8\uc640 \ub300\ube44\ub418\ub294 \uc548\ud2f0 \ubd80\ub9bf\ucf54(\u30a2\u30f3\u30c1\u00b7\u3076\u308a\u3063\u5b50) \uc774\ubbf8\uc9c0\ub97c \ub300\ud45c\ud558\uc600\ub2e4. \uc774\ub178\uc6b0\uc5d0 \uc694\uc2a4\uc774\uc640 \uc791\uc5c5\ud55c \u3008\uc7a5\uc2dd\uc740 \uc544\ub2c8\uc57c, \ub208\ubb3c\uc740\u3009\uc73c\ub85c \uc544\uc774\ub3cc \uc774\ubbf8\uc9c0\uc5d0\uc11c \ud0c8\ud53c\ud568\uacfc \ub3d9\uc2dc\uc5d0 \uc0c8\ub85c\uc6b4 \uacbd\uc9c0\ub97c \uac1c\ucc99\ud558\uc5ec \ud6c4\ub300 \uc544\uc774\ub3cc\uc5d0\uac8c \uc601\ud5a5\uc744 \uc92c\ub2e4. \uc774 \uc774\ud6c4, \ub098\uce74\ubaa8\ub9ac\uc758 \ud2b9\uc0c9 \uc788\ub294 \uac00\ucc3d\ub825\uacfc \ub178\ub798\uc758 \uc5b4\ub450\uc6b4 \ubd84\uc704\uae30\ub294 \ub2f9\uc2dc \ubd88\uc548\uacfc \uc808\ub9dd\uc5d0 \ube60\uc838\uc788\ub294 \uc80a\uc740\uc774\ub4e4, \ud2b9\ud788 \uc5ec\uc131\ub4e4\uc758 \uacf5\uac10\uc744 \uc0ac\uba70 \uc778\uae30\ub97c \ub204\ub838\ub2e4. \ub610\ud55c \ub2f9\uc2dc \uc720\ud589\ud558\ub358 \ubc30\uae30 \ud32c\uce20 \ub4f1\uc758 \uc758\uc0c1\uc744 \uc801\uadf9\uc801\uc73c\ub85c \uc218\uc6a9\ud558\uc5ec \uc790\uc2e0\uc758 \ubb34\ub300\uc5d0 \ubc18\uc601\ud558\ub294 \ub4f1 \ud328\uc158 \ub9ac\ub354\ub85c\uc11c\uc758 \uba74\ubaa8\ub3c4 \ubcf4\uc5ec\uc8fc\uc5c8\ub2e4. \uadf8\ub7ec\ub098 \ub098\uce74\ubaa8\ub9ac\uc758 \ud589\ubcf4\uc5d0\ub3c4 \ube44\ud310\uc740 \ub530\ub974\ub294\ub370 \uadf8\ub140\uc758 \uc790\uc0b4\ubbf8\uc218 \uc0ac\uac74\uc744 \ube44\ub86f\ud558\uc5ec \ud6c4\ubc30 \uc544\uc774\ub3cc\uc774\uc5c8\ub358 \uc624\uce74\ub2e4 \uc720\ud0a4\ucf54\uc758 \ud22c\uc2e0\uc790\uc0b4 \ub4f1 \uc544\uc774\ub3cc \uc74c\uc545 \uc0b0\uc5c5\uc5d0\uc11c \ube44\ub86f\ub41c \uc77c\ub828\uc758 \uc0ac\uac74\ub4e4\uc774 \uc544\uc774\ub3cc \uc138\uacc4\uc5d0 \ub300\ud574 \ub9c9\uc5f0\ud55c \ub3d9\uacbd\uc744 \ud488\ub358 10\ub300 \uccad\uc18c\ub144\ub4e4\uc5d0\uac8c \uacbd\uc885\uc744 \uc6b8\ub838\ub2e4\uace0 \uc9c0\uc801\ud558\uc600\ub2e4."} {"question": "\uc790\uc720\uc758 \uc5ec\uc2e0\uc0c1\uc744 \ud3ec\ud568\ud55c \ub9ce\uc740 \ub79c\ub4dc\ub9c8\ud06c\ub97c \ubcf4\uc720\ud558\uace0 \uc788\uc73c\uba70, \uc5f0\uac04 5\ucc9c\ub9cc \uba85\uc758 \uad00\uad11\uac1d\uc774 \ubc29\ubb38\ud558\ub294 \ub3c4\uc2dc\ub294?", "paragraph": "\ub274\uc695\uc5d0\ub294 5\ubc88\ub85c\ub97c \ud3ec\ud568\ud55c \uac70\ub9ac, \uc790\uc720\uc758 \uc5ec\uc2e0\uc0c1\uc744 \ud3ec\ud568\ud55c \ub79c\ub4dc\ub9c8\ud06c\uac00 \ub9ce\uc774 \uc788\uc73c\uba70, \uc5f0\uac04 5\ucc9c\ub9cc \uba85\uc758 \uad00\uad11\uac1d\uc774 \ubc29\ubb38\ud55c\ub2e4. \ud0c0\uc784\uc2a4 \uc2a4\ud018\uc5b4\ub294 '\uc138\uacc4\uc758 \uad50\ucc28\ub85c'(The Crossroads of the World)\ub77c\uace0 \ubd88\ub9ac\uace0 \uc788\ub2e4. \ud0c0\uc784\uc2a4 \uc2a4\ud018\uc5b4 \ubd80\uadfc\uc5d0\uc11c\ub294 \ube0c\ub85c\ub4dc\uc6e8\uc774 \uc5f0\uadf9\uc774 \uc0c1\uc5f0\ub418\uba70, \ub274\uc695\uc740 \uc5d4\ud130\ud14c\uc778\uba3c\ud2b8 \uc0b0\uc5c5\uc758 \uc911\uc2ec\uc9c0\ub85c \uc77c\uceec\uc5b4\uc9c4\ub2e4. \uc5e0\ud30c\uc774\uc5b4 \uc2a4\ud14c\uc774\ud2b8 \ube4c\ub529, \ub85d\ud3a0\ub7ec\uc13c\ud130, \ud06c\ub77c\uc774\uc2ac\ub7ec \ube4c\ub529\uc744 \ud3ec\ud568\ud55c \ucd08\uace0\uce35 \uac74\ubb3c, \uc13c\ud2b8\ub7f4 \ud30c\ud06c\ub97c \ud3ec\ud568\ud55c \uacf5\uc6d0, \ube0c\ub8e8\ud074\ub9b0 \ub2e4\ub9ac \ub4f1\uc744 \ud3ec\ud568\ud55c \ub2e4\ub9ac\ub3c4 \ub9ce\uc774 \uc788\ub2e4. \uacbd\uc81c \uc218\ub3c4\ub85c\ub3c4 \ubd88\ub9ac\ub294 \ub274\uc695\uc5d0\ub294 \uc6d4 \uac00\uac00 \uc788\uc73c\uba70, \ub274\uc695 \uc99d\uad8c\uac70\ub798\uc18c(NYSE)\uc640 NASDAQ\uc774 \uc774 \uac70\ub9ac\uc5d0 \uc788\ub2e4. \ub9e8\ud574\ud2bc\uc758 \ubd80\ub3d9\uc0b0 \uc2dc\uc7a5\uc740 \uc138\uacc4\uc5d0\uc11c \uac00\uc7a5 \ube44\uc2f8\ub2e4. \ub274\uc695 \uc9c0\ud558\ucca0\uc740 \uc138\uacc4 \ucd5c\ub300\uc758 \uc9c0\ud558\ucca0\ub9dd \uc911 \ud558\ub098\uc774\uba70, \uceec\ub7fc\ube44\uc544 \ub300\ud559\uad50, \ub274\uc695 \ub300\ud559\uad50\ub97c \ud3ec\ud568\ud55c \uc218\ub9ce\uc740 \ub300\ud559\uad50\ub3c4 \ub274\uc695\uc5d0 \uc788\ub2e4.", "answer": "\ub274\uc695", "sentence": "\ub274\uc695 \uc5d0\ub294 5\ubc88\ub85c\ub97c \ud3ec\ud568\ud55c \uac70\ub9ac, \uc790\uc720\uc758 \uc5ec\uc2e0\uc0c1\uc744 \ud3ec\ud568\ud55c \ub79c\ub4dc\ub9c8\ud06c\uac00 \ub9ce\uc774 \uc788\uc73c\uba70, \uc5f0\uac04 5\ucc9c\ub9cc \uba85\uc758 \uad00\uad11\uac1d\uc774 \ubc29\ubb38\ud55c\ub2e4.", "paragraph_sentence": " \ub274\uc695 \uc5d0\ub294 5\ubc88\ub85c\ub97c \ud3ec\ud568\ud55c \uac70\ub9ac, \uc790\uc720\uc758 \uc5ec\uc2e0\uc0c1\uc744 \ud3ec\ud568\ud55c \ub79c\ub4dc\ub9c8\ud06c\uac00 \ub9ce\uc774 \uc788\uc73c\uba70, \uc5f0\uac04 5\ucc9c\ub9cc \uba85\uc758 \uad00\uad11\uac1d\uc774 \ubc29\ubb38\ud55c\ub2e4. \ud0c0\uc784\uc2a4 \uc2a4\ud018\uc5b4\ub294 '\uc138\uacc4\uc758 \uad50\ucc28\ub85c'(The Crossroads of the World)\ub77c\uace0 \ubd88\ub9ac\uace0 \uc788\ub2e4. \ud0c0\uc784\uc2a4 \uc2a4\ud018\uc5b4 \ubd80\uadfc\uc5d0\uc11c\ub294 \ube0c\ub85c\ub4dc\uc6e8\uc774 \uc5f0\uadf9\uc774 \uc0c1\uc5f0\ub418\uba70, \ub274\uc695\uc740 \uc5d4\ud130\ud14c\uc778\uba3c\ud2b8 \uc0b0\uc5c5\uc758 \uc911\uc2ec\uc9c0\ub85c \uc77c\uceec\uc5b4\uc9c4\ub2e4. \uc5e0\ud30c\uc774\uc5b4 \uc2a4\ud14c\uc774\ud2b8 \ube4c\ub529, \ub85d\ud3a0\ub7ec\uc13c\ud130, \ud06c\ub77c\uc774\uc2ac\ub7ec \ube4c\ub529\uc744 \ud3ec\ud568\ud55c \ucd08\uace0\uce35 \uac74\ubb3c, \uc13c\ud2b8\ub7f4 \ud30c\ud06c\ub97c \ud3ec\ud568\ud55c \uacf5\uc6d0, \ube0c\ub8e8\ud074\ub9b0 \ub2e4\ub9ac \ub4f1\uc744 \ud3ec\ud568\ud55c \ub2e4\ub9ac\ub3c4 \ub9ce\uc774 \uc788\ub2e4. \uacbd\uc81c \uc218\ub3c4\ub85c\ub3c4 \ubd88\ub9ac\ub294 \ub274\uc695\uc5d0\ub294 \uc6d4 \uac00\uac00 \uc788\uc73c\uba70, \ub274\uc695 \uc99d\uad8c\uac70\ub798\uc18c(NYSE)\uc640 NASDAQ\uc774 \uc774 \uac70\ub9ac\uc5d0 \uc788\ub2e4. \ub9e8\ud574\ud2bc\uc758 \ubd80\ub3d9\uc0b0 \uc2dc\uc7a5\uc740 \uc138\uacc4\uc5d0\uc11c \uac00\uc7a5 \ube44\uc2f8\ub2e4. \ub274\uc695 \uc9c0\ud558\ucca0\uc740 \uc138\uacc4 \ucd5c\ub300\uc758 \uc9c0\ud558\ucca0\ub9dd \uc911 \ud558\ub098\uc774\uba70, \uceec\ub7fc\ube44\uc544 \ub300\ud559\uad50, \ub274\uc695 \ub300\ud559\uad50\ub97c \ud3ec\ud568\ud55c \uc218\ub9ce\uc740 \ub300\ud559\uad50\ub3c4 \ub274\uc695\uc5d0 \uc788\ub2e4.", "paragraph_answer": " \ub274\uc695 \uc5d0\ub294 5\ubc88\ub85c\ub97c \ud3ec\ud568\ud55c \uac70\ub9ac, \uc790\uc720\uc758 \uc5ec\uc2e0\uc0c1\uc744 \ud3ec\ud568\ud55c \ub79c\ub4dc\ub9c8\ud06c\uac00 \ub9ce\uc774 \uc788\uc73c\uba70, \uc5f0\uac04 5\ucc9c\ub9cc \uba85\uc758 \uad00\uad11\uac1d\uc774 \ubc29\ubb38\ud55c\ub2e4. \ud0c0\uc784\uc2a4 \uc2a4\ud018\uc5b4\ub294 '\uc138\uacc4\uc758 \uad50\ucc28\ub85c'(The Crossroads of the World)\ub77c\uace0 \ubd88\ub9ac\uace0 \uc788\ub2e4. \ud0c0\uc784\uc2a4 \uc2a4\ud018\uc5b4 \ubd80\uadfc\uc5d0\uc11c\ub294 \ube0c\ub85c\ub4dc\uc6e8\uc774 \uc5f0\uadf9\uc774 \uc0c1\uc5f0\ub418\uba70, \ub274\uc695\uc740 \uc5d4\ud130\ud14c\uc778\uba3c\ud2b8 \uc0b0\uc5c5\uc758 \uc911\uc2ec\uc9c0\ub85c \uc77c\uceec\uc5b4\uc9c4\ub2e4. \uc5e0\ud30c\uc774\uc5b4 \uc2a4\ud14c\uc774\ud2b8 \ube4c\ub529, \ub85d\ud3a0\ub7ec\uc13c\ud130, \ud06c\ub77c\uc774\uc2ac\ub7ec \ube4c\ub529\uc744 \ud3ec\ud568\ud55c \ucd08\uace0\uce35 \uac74\ubb3c, \uc13c\ud2b8\ub7f4 \ud30c\ud06c\ub97c \ud3ec\ud568\ud55c \uacf5\uc6d0, \ube0c\ub8e8\ud074\ub9b0 \ub2e4\ub9ac \ub4f1\uc744 \ud3ec\ud568\ud55c \ub2e4\ub9ac\ub3c4 \ub9ce\uc774 \uc788\ub2e4. \uacbd\uc81c \uc218\ub3c4\ub85c\ub3c4 \ubd88\ub9ac\ub294 \ub274\uc695\uc5d0\ub294 \uc6d4 \uac00\uac00 \uc788\uc73c\uba70, \ub274\uc695 \uc99d\uad8c\uac70\ub798\uc18c(NYSE)\uc640 NASDAQ\uc774 \uc774 \uac70\ub9ac\uc5d0 \uc788\ub2e4. \ub9e8\ud574\ud2bc\uc758 \ubd80\ub3d9\uc0b0 \uc2dc\uc7a5\uc740 \uc138\uacc4\uc5d0\uc11c \uac00\uc7a5 \ube44\uc2f8\ub2e4. \ub274\uc695 \uc9c0\ud558\ucca0\uc740 \uc138\uacc4 \ucd5c\ub300\uc758 \uc9c0\ud558\ucca0\ub9dd \uc911 \ud558\ub098\uc774\uba70, \uceec\ub7fc\ube44\uc544 \ub300\ud559\uad50, \ub274\uc695 \ub300\ud559\uad50\ub97c \ud3ec\ud568\ud55c \uc218\ub9ce\uc740 \ub300\ud559\uad50\ub3c4 \ub274\uc695\uc5d0 \uc788\ub2e4.", "sentence_answer": " \ub274\uc695 \uc5d0\ub294 5\ubc88\ub85c\ub97c \ud3ec\ud568\ud55c \uac70\ub9ac, \uc790\uc720\uc758 \uc5ec\uc2e0\uc0c1\uc744 \ud3ec\ud568\ud55c \ub79c\ub4dc\ub9c8\ud06c\uac00 \ub9ce\uc774 \uc788\uc73c\uba70, \uc5f0\uac04 5\ucc9c\ub9cc \uba85\uc758 \uad00\uad11\uac1d\uc774 \ubc29\ubb38\ud55c\ub2e4.", "paragraph_id": "6489368-0-0", "paragraph_question": "question: \uc790\uc720\uc758 \uc5ec\uc2e0\uc0c1\uc744 \ud3ec\ud568\ud55c \ub9ce\uc740 \ub79c\ub4dc\ub9c8\ud06c\ub97c \ubcf4\uc720\ud558\uace0 \uc788\uc73c\uba70, \uc5f0\uac04 5\ucc9c\ub9cc \uba85\uc758 \uad00\uad11\uac1d\uc774 \ubc29\ubb38\ud558\ub294 \ub3c4\uc2dc\ub294?, context: \ub274\uc695\uc5d0\ub294 5\ubc88\ub85c\ub97c \ud3ec\ud568\ud55c \uac70\ub9ac, \uc790\uc720\uc758 \uc5ec\uc2e0\uc0c1\uc744 \ud3ec\ud568\ud55c \ub79c\ub4dc\ub9c8\ud06c\uac00 \ub9ce\uc774 \uc788\uc73c\uba70, \uc5f0\uac04 5\ucc9c\ub9cc \uba85\uc758 \uad00\uad11\uac1d\uc774 \ubc29\ubb38\ud55c\ub2e4. \ud0c0\uc784\uc2a4 \uc2a4\ud018\uc5b4\ub294 '\uc138\uacc4\uc758 \uad50\ucc28\ub85c'(The Crossroads of the World)\ub77c\uace0 \ubd88\ub9ac\uace0 \uc788\ub2e4. \ud0c0\uc784\uc2a4 \uc2a4\ud018\uc5b4 \ubd80\uadfc\uc5d0\uc11c\ub294 \ube0c\ub85c\ub4dc\uc6e8\uc774 \uc5f0\uadf9\uc774 \uc0c1\uc5f0\ub418\uba70, \ub274\uc695\uc740 \uc5d4\ud130\ud14c\uc778\uba3c\ud2b8 \uc0b0\uc5c5\uc758 \uc911\uc2ec\uc9c0\ub85c \uc77c\uceec\uc5b4\uc9c4\ub2e4. \uc5e0\ud30c\uc774\uc5b4 \uc2a4\ud14c\uc774\ud2b8 \ube4c\ub529, \ub85d\ud3a0\ub7ec\uc13c\ud130, \ud06c\ub77c\uc774\uc2ac\ub7ec \ube4c\ub529\uc744 \ud3ec\ud568\ud55c \ucd08\uace0\uce35 \uac74\ubb3c, \uc13c\ud2b8\ub7f4 \ud30c\ud06c\ub97c \ud3ec\ud568\ud55c \uacf5\uc6d0, \ube0c\ub8e8\ud074\ub9b0 \ub2e4\ub9ac \ub4f1\uc744 \ud3ec\ud568\ud55c \ub2e4\ub9ac\ub3c4 \ub9ce\uc774 \uc788\ub2e4. \uacbd\uc81c \uc218\ub3c4\ub85c\ub3c4 \ubd88\ub9ac\ub294 \ub274\uc695\uc5d0\ub294 \uc6d4 \uac00\uac00 \uc788\uc73c\uba70, \ub274\uc695 \uc99d\uad8c\uac70\ub798\uc18c(NYSE)\uc640 NASDAQ\uc774 \uc774 \uac70\ub9ac\uc5d0 \uc788\ub2e4. \ub9e8\ud574\ud2bc\uc758 \ubd80\ub3d9\uc0b0 \uc2dc\uc7a5\uc740 \uc138\uacc4\uc5d0\uc11c \uac00\uc7a5 \ube44\uc2f8\ub2e4. \ub274\uc695 \uc9c0\ud558\ucca0\uc740 \uc138\uacc4 \ucd5c\ub300\uc758 \uc9c0\ud558\ucca0\ub9dd \uc911 \ud558\ub098\uc774\uba70, \uceec\ub7fc\ube44\uc544 \ub300\ud559\uad50, \ub274\uc695 \ub300\ud559\uad50\ub97c \ud3ec\ud568\ud55c \uc218\ub9ce\uc740 \ub300\ud559\uad50\ub3c4 \ub274\uc695\uc5d0 \uc788\ub2e4."} {"question": "\ubd80\uc0b0\uc218\uc0b0\uc804\ubb38\ud559\uad50\uac00 \ub300\ud559\uc73c\ub85c \uc2b9\uaca9\ub41c \ud6c4\uc758 \uc774\ub984\uc740 \ubb34\uc5c7\uc778\uac00?", "paragraph": "1946\ub144 5\uc6d4 15\uc77c\uc5d0 \ubb38\uad50\ubd80\ub294 \ubd80\uc0b0\uc218\uc0b0\uc804\ubb38\ud559\uad50\ub97c \ub300\ud559\uc73c\ub85c \uc2b9\uaca9\uc2dc\ucf1c \uc885\ud569\ub300\ud559\uc758 1\uac1c \ub2e8\uacfc\ub300\ud559(\uc218\uc0b0\ud559\ubd80)\uc73c\ub85c, \uadf8\uc640 \ubcc4\ub3c4\ub85c \uc778\ubb38\uacc4\ub300\ud559\uc744 \uc2e0\uc124\ud558\uc5ec \uc778\ubb38\uacc4 1\uac1c \ub2e8\uacfc\ub300\ud559(\uc778\ubb38\ud559\ubd80)\uacfc \uc790\uc5f0\uacc4 1\uac1c \ub2e8\uacfc\ub300\ud559(\uc218\uc0b0\ud559\ubd80) \uccb4\uc81c\uc758 \uad6d\ub9bd \ubd80\uc0b0\ub300\ud559\uad50(\uc885\ud569\ub300\ud559) \uc124\ub9bd\uc744 \uc778\uac00\ud558\uc600\ub294\ub370, \ud559\uad50\uc758 \uba85\uce6d\uc740 \u2018\ubd80\uc0b0\ub300\ud559\u2019\uc73c\ub85c, \ud559\ubd80(\ub2e8\uacfc\ub300\ud559)\ub294 \uc778\ubb38\ud559\ubd80\uc640 \uc218\uc0b0\ud559\ubd80\ub85c \uac1c\uad50\ud558\uc600\uc73c\uba70(\uc774\ub54c\uc758 \ud559\ubd80\ub294 \uc624\ub298\ub0a0\uc758 \ub2e8\uacfc\ub300\ud559(Faculty)\uc5d0 \ud574\ub2f9\ud558\ub294 \uac83\uc73c\ub85c, 1946\ub144 3\uc6d4 7\uc77c\uc5d0 \uacf5\ud3ec\ub41c \ub300\ud559\ub839 \uc81c2\uc870 \ud6c4\ub2e8\uc5d0\ub294 \uc778\ubb38\uacc4 \ubc0f \uc790\uc5f0\uacc4\uc758 \ud559\ubd80\uac00 \ubcd1\uc124\ub420 \ub54c\ub294 2\uac1c \uc774\uc0c1\uc758 \ud559\ubd80\ub85c\uc368 '\uc885\ud569\ub300\ud559\uad50'\ub97c \uad6c\uc131\ud560 \uc218 \uc788\ub3c4\ub85d \uaddc\uc815\ub418\uc5b4 \uc788\uc5c8\ub2e4.) \uadf8\ud6c4 1946\ub144 7\uc6d4 \uc608\uacfc \uc2dc\ud5d8\uc744 \uac70\uce58\uace0, 1946\ub144 8\uc6d4\uc5d0\ub294 \ub300\ud559 \ucd1d\uc7a5\uc73c\ub85c \uc544\uc11c \ub9b0 \ubca0\ucee4(Arthur Lynn Becker 1879~1979, \ud55c\uad6d\uba85 \ubc31\uc544\ub355\u767d\u96c5\u60b3) \ubc15\uc0ac\ub97c \uc784\uba85\ud558\uc5ec 1946\ub144 9\uc6d4 \uc785\ud559\uc2dd\uc744 \uac70\ud589\ud558\uace0 1\ub144 \uc815\ub3c4 \uc885\ud569\ub300\ud559\uc73c\ub85c \uc6b4\uc601\ub418\ub2e4\uac00 1947\ub144 7\uc6d4 \uc218\uc0b0\uacfc\ub300\ud559\uc774 '\uad6d\ub9bd \ubd80\uc0b0\uc218\uc0b0\ub300\ud559'\uc73c\ub85c \ubd84\ub9ac\ub428\uc73c\ub85c\uc368 \uc885\ud569\ub300\ud559\uc73c\ub85c\uc368 '\uad6d\ub9bd \ubd80\uc0b0\ub300\ud559\uad50'\uc740 \uc0ac\uc2e4\uc0c1 \uc640\ud574\ub418\uc5c8\ub2e4. \uadf8 \ud6c4 1953\ub144 4\uc6d4 1\uc77c\uc5d0 \uad6d\ub9bd \ubd80\uc0b0\ub300\ud559\uad50 \uc124\ub9bd\uc548\uc774 \uad6d\ubb34\ud68c\uc758\ub97c \ud1b5\uacfc\ud558\uace0, 4\uc6d4 3\uc77c \uad6d\ubb34\ucd1d\ub9ac\uc758 \uc7ac\uac00\ub97c \ubc1b\uc544 \uac19\uc740 \ud574 9\uc6d415\uc77c \uad6d\ub9bd\ud559\uad50\uc124\uce58\ub839\uc774 \uacf5\ud3ec\ub428\uc5d0 \ub530\ub77c \ub2e4\uc2dc \uc885\ud569\ub300\ud559\uc73c\ub85c \uc2b9\uaca9\ub418\uc5c8\ub2e4.", "answer": "\ubd80\uc0b0\ub300\ud559", "sentence": "1946\ub144 5\uc6d4 15\uc77c\uc5d0 \ubb38\uad50\ubd80\ub294 \ubd80\uc0b0\uc218\uc0b0\uc804\ubb38\ud559\uad50\ub97c \ub300\ud559\uc73c\ub85c \uc2b9\uaca9\uc2dc\ucf1c \uc885\ud569\ub300\ud559\uc758 1\uac1c \ub2e8\uacfc\ub300\ud559(\uc218\uc0b0\ud559\ubd80)\uc73c\ub85c, \uadf8\uc640 \ubcc4\ub3c4\ub85c \uc778\ubb38\uacc4\ub300\ud559\uc744 \uc2e0\uc124\ud558\uc5ec \uc778\ubb38\uacc4 1\uac1c \ub2e8\uacfc\ub300\ud559(\uc778\ubb38\ud559\ubd80)\uacfc \uc790\uc5f0\uacc4 1\uac1c \ub2e8\uacfc\ub300\ud559(\uc218\uc0b0\ud559\ubd80) \uccb4\uc81c\uc758 \uad6d\ub9bd \ubd80\uc0b0\ub300\ud559 \uad50(\uc885\ud569\ub300\ud559) \uc124\ub9bd\uc744 \uc778\uac00\ud558\uc600\ub294\ub370, \ud559\uad50\uc758 \uba85\uce6d\uc740 \u2018\ubd80\uc0b0\ub300\ud559\u2019\uc73c\ub85c, \ud559\ubd80(\ub2e8\uacfc\ub300\ud559)\ub294 \uc778\ubb38\ud559\ubd80\uc640 \uc218\uc0b0\ud559\ubd80\ub85c \uac1c\uad50\ud558\uc600\uc73c\uba70(\uc774\ub54c\uc758 \ud559\ubd80\ub294 \uc624\ub298\ub0a0\uc758 \ub2e8\uacfc\ub300\ud559(Faculty)\uc5d0 \ud574\ub2f9\ud558\ub294 \uac83\uc73c\ub85c, 1946\ub144 3\uc6d4 7\uc77c\uc5d0 \uacf5\ud3ec\ub41c \ub300\ud559\ub839 \uc81c2\uc870 \ud6c4\ub2e8\uc5d0\ub294 \uc778\ubb38\uacc4 \ubc0f \uc790\uc5f0\uacc4\uc758 \ud559\ubd80\uac00 \ubcd1\uc124\ub420 \ub54c\ub294 2\uac1c \uc774\uc0c1\uc758 \ud559\ubd80\ub85c\uc368 '\uc885\ud569\ub300\ud559\uad50'\ub97c \uad6c\uc131\ud560 \uc218 \uc788\ub3c4\ub85d \uaddc\uc815\ub418\uc5b4 \uc788\uc5c8\ub2e4.", "paragraph_sentence": " 1946\ub144 5\uc6d4 15\uc77c\uc5d0 \ubb38\uad50\ubd80\ub294 \ubd80\uc0b0\uc218\uc0b0\uc804\ubb38\ud559\uad50\ub97c \ub300\ud559\uc73c\ub85c \uc2b9\uaca9\uc2dc\ucf1c \uc885\ud569\ub300\ud559\uc758 1\uac1c \ub2e8\uacfc\ub300\ud559(\uc218\uc0b0\ud559\ubd80)\uc73c\ub85c, \uadf8\uc640 \ubcc4\ub3c4\ub85c \uc778\ubb38\uacc4\ub300\ud559\uc744 \uc2e0\uc124\ud558\uc5ec \uc778\ubb38\uacc4 1\uac1c \ub2e8\uacfc\ub300\ud559(\uc778\ubb38\ud559\ubd80)\uacfc \uc790\uc5f0\uacc4 1\uac1c \ub2e8\uacfc\ub300\ud559(\uc218\uc0b0\ud559\ubd80) \uccb4\uc81c\uc758 \uad6d\ub9bd \ubd80\uc0b0\ub300\ud559 \uad50(\uc885\ud569\ub300\ud559) \uc124\ub9bd\uc744 \uc778\uac00\ud558\uc600\ub294\ub370, \ud559\uad50\uc758 \uba85\uce6d\uc740 \u2018\ubd80\uc0b0\ub300\ud559\u2019\uc73c\ub85c, \ud559\ubd80(\ub2e8\uacfc\ub300\ud559)\ub294 \uc778\ubb38\ud559\ubd80\uc640 \uc218\uc0b0\ud559\ubd80\ub85c \uac1c\uad50\ud558\uc600\uc73c\uba70(\uc774\ub54c\uc758 \ud559\ubd80\ub294 \uc624\ub298\ub0a0\uc758 \ub2e8\uacfc\ub300\ud559(Faculty)\uc5d0 \ud574\ub2f9\ud558\ub294 \uac83\uc73c\ub85c, 1946\ub144 3\uc6d4 7\uc77c\uc5d0 \uacf5\ud3ec\ub41c \ub300\ud559\ub839 \uc81c2\uc870 \ud6c4\ub2e8\uc5d0\ub294 \uc778\ubb38\uacc4 \ubc0f \uc790\uc5f0\uacc4\uc758 \ud559\ubd80\uac00 \ubcd1\uc124\ub420 \ub54c\ub294 2\uac1c \uc774\uc0c1\uc758 \ud559\ubd80\ub85c\uc368 '\uc885\ud569\ub300\ud559\uad50'\ub97c \uad6c\uc131\ud560 \uc218 \uc788\ub3c4\ub85d \uaddc\uc815\ub418\uc5b4 \uc788\uc5c8\ub2e4. ) \uadf8\ud6c4 1946\ub144 7\uc6d4 \uc608\uacfc \uc2dc\ud5d8\uc744 \uac70\uce58\uace0, 1946\ub144 8\uc6d4\uc5d0\ub294 \ub300\ud559 \ucd1d\uc7a5\uc73c\ub85c \uc544\uc11c \ub9b0 \ubca0\ucee4(Arthur Lynn Becker 1879~1979, \ud55c\uad6d\uba85 \ubc31\uc544\ub355\u767d\u96c5\u60b3) \ubc15\uc0ac\ub97c \uc784\uba85\ud558\uc5ec 1946\ub144 9\uc6d4 \uc785\ud559\uc2dd\uc744 \uac70\ud589\ud558\uace0 1\ub144 \uc815\ub3c4 \uc885\ud569\ub300\ud559\uc73c\ub85c \uc6b4\uc601\ub418\ub2e4\uac00 1947\ub144 7\uc6d4 \uc218\uc0b0\uacfc\ub300\ud559\uc774 '\uad6d\ub9bd \ubd80\uc0b0\uc218\uc0b0\ub300\ud559'\uc73c\ub85c \ubd84\ub9ac\ub428\uc73c\ub85c\uc368 \uc885\ud569\ub300\ud559\uc73c\ub85c\uc368 '\uad6d\ub9bd \ubd80\uc0b0\ub300\ud559\uad50'\uc740 \uc0ac\uc2e4\uc0c1 \uc640\ud574\ub418\uc5c8\ub2e4. \uadf8 \ud6c4 1953\ub144 4\uc6d4 1\uc77c\uc5d0 \uad6d\ub9bd \ubd80\uc0b0\ub300\ud559\uad50 \uc124\ub9bd\uc548\uc774 \uad6d\ubb34\ud68c\uc758\ub97c \ud1b5\uacfc\ud558\uace0, 4\uc6d4 3\uc77c \uad6d\ubb34\ucd1d\ub9ac\uc758 \uc7ac\uac00\ub97c \ubc1b\uc544 \uac19\uc740 \ud574 9\uc6d415\uc77c \uad6d\ub9bd\ud559\uad50\uc124\uce58\ub839\uc774 \uacf5\ud3ec\ub428\uc5d0 \ub530\ub77c \ub2e4\uc2dc \uc885\ud569\ub300\ud559\uc73c\ub85c \uc2b9\uaca9\ub418\uc5c8\ub2e4.", "paragraph_answer": "1946\ub144 5\uc6d4 15\uc77c\uc5d0 \ubb38\uad50\ubd80\ub294 \ubd80\uc0b0\uc218\uc0b0\uc804\ubb38\ud559\uad50\ub97c \ub300\ud559\uc73c\ub85c \uc2b9\uaca9\uc2dc\ucf1c \uc885\ud569\ub300\ud559\uc758 1\uac1c \ub2e8\uacfc\ub300\ud559(\uc218\uc0b0\ud559\ubd80)\uc73c\ub85c, \uadf8\uc640 \ubcc4\ub3c4\ub85c \uc778\ubb38\uacc4\ub300\ud559\uc744 \uc2e0\uc124\ud558\uc5ec \uc778\ubb38\uacc4 1\uac1c \ub2e8\uacfc\ub300\ud559(\uc778\ubb38\ud559\ubd80)\uacfc \uc790\uc5f0\uacc4 1\uac1c \ub2e8\uacfc\ub300\ud559(\uc218\uc0b0\ud559\ubd80) \uccb4\uc81c\uc758 \uad6d\ub9bd \ubd80\uc0b0\ub300\ud559 \uad50(\uc885\ud569\ub300\ud559) \uc124\ub9bd\uc744 \uc778\uac00\ud558\uc600\ub294\ub370, \ud559\uad50\uc758 \uba85\uce6d\uc740 \u2018\ubd80\uc0b0\ub300\ud559\u2019\uc73c\ub85c, \ud559\ubd80(\ub2e8\uacfc\ub300\ud559)\ub294 \uc778\ubb38\ud559\ubd80\uc640 \uc218\uc0b0\ud559\ubd80\ub85c \uac1c\uad50\ud558\uc600\uc73c\uba70(\uc774\ub54c\uc758 \ud559\ubd80\ub294 \uc624\ub298\ub0a0\uc758 \ub2e8\uacfc\ub300\ud559(Faculty)\uc5d0 \ud574\ub2f9\ud558\ub294 \uac83\uc73c\ub85c, 1946\ub144 3\uc6d4 7\uc77c\uc5d0 \uacf5\ud3ec\ub41c \ub300\ud559\ub839 \uc81c2\uc870 \ud6c4\ub2e8\uc5d0\ub294 \uc778\ubb38\uacc4 \ubc0f \uc790\uc5f0\uacc4\uc758 \ud559\ubd80\uac00 \ubcd1\uc124\ub420 \ub54c\ub294 2\uac1c \uc774\uc0c1\uc758 \ud559\ubd80\ub85c\uc368 '\uc885\ud569\ub300\ud559\uad50'\ub97c \uad6c\uc131\ud560 \uc218 \uc788\ub3c4\ub85d \uaddc\uc815\ub418\uc5b4 \uc788\uc5c8\ub2e4.) \uadf8\ud6c4 1946\ub144 7\uc6d4 \uc608\uacfc \uc2dc\ud5d8\uc744 \uac70\uce58\uace0, 1946\ub144 8\uc6d4\uc5d0\ub294 \ub300\ud559 \ucd1d\uc7a5\uc73c\ub85c \uc544\uc11c \ub9b0 \ubca0\ucee4(Arthur Lynn Becker 1879~1979, \ud55c\uad6d\uba85 \ubc31\uc544\ub355\u767d\u96c5\u60b3) \ubc15\uc0ac\ub97c \uc784\uba85\ud558\uc5ec 1946\ub144 9\uc6d4 \uc785\ud559\uc2dd\uc744 \uac70\ud589\ud558\uace0 1\ub144 \uc815\ub3c4 \uc885\ud569\ub300\ud559\uc73c\ub85c \uc6b4\uc601\ub418\ub2e4\uac00 1947\ub144 7\uc6d4 \uc218\uc0b0\uacfc\ub300\ud559\uc774 '\uad6d\ub9bd \ubd80\uc0b0\uc218\uc0b0\ub300\ud559'\uc73c\ub85c \ubd84\ub9ac\ub428\uc73c\ub85c\uc368 \uc885\ud569\ub300\ud559\uc73c\ub85c\uc368 '\uad6d\ub9bd \ubd80\uc0b0\ub300\ud559\uad50'\uc740 \uc0ac\uc2e4\uc0c1 \uc640\ud574\ub418\uc5c8\ub2e4. \uadf8 \ud6c4 1953\ub144 4\uc6d4 1\uc77c\uc5d0 \uad6d\ub9bd \ubd80\uc0b0\ub300\ud559\uad50 \uc124\ub9bd\uc548\uc774 \uad6d\ubb34\ud68c\uc758\ub97c \ud1b5\uacfc\ud558\uace0, 4\uc6d4 3\uc77c \uad6d\ubb34\ucd1d\ub9ac\uc758 \uc7ac\uac00\ub97c \ubc1b\uc544 \uac19\uc740 \ud574 9\uc6d415\uc77c \uad6d\ub9bd\ud559\uad50\uc124\uce58\ub839\uc774 \uacf5\ud3ec\ub428\uc5d0 \ub530\ub77c \ub2e4\uc2dc \uc885\ud569\ub300\ud559\uc73c\ub85c \uc2b9\uaca9\ub418\uc5c8\ub2e4.", "sentence_answer": "1946\ub144 5\uc6d4 15\uc77c\uc5d0 \ubb38\uad50\ubd80\ub294 \ubd80\uc0b0\uc218\uc0b0\uc804\ubb38\ud559\uad50\ub97c \ub300\ud559\uc73c\ub85c \uc2b9\uaca9\uc2dc\ucf1c \uc885\ud569\ub300\ud559\uc758 1\uac1c \ub2e8\uacfc\ub300\ud559(\uc218\uc0b0\ud559\ubd80)\uc73c\ub85c, \uadf8\uc640 \ubcc4\ub3c4\ub85c \uc778\ubb38\uacc4\ub300\ud559\uc744 \uc2e0\uc124\ud558\uc5ec \uc778\ubb38\uacc4 1\uac1c \ub2e8\uacfc\ub300\ud559(\uc778\ubb38\ud559\ubd80)\uacfc \uc790\uc5f0\uacc4 1\uac1c \ub2e8\uacfc\ub300\ud559(\uc218\uc0b0\ud559\ubd80) \uccb4\uc81c\uc758 \uad6d\ub9bd \ubd80\uc0b0\ub300\ud559 \uad50(\uc885\ud569\ub300\ud559) \uc124\ub9bd\uc744 \uc778\uac00\ud558\uc600\ub294\ub370, \ud559\uad50\uc758 \uba85\uce6d\uc740 \u2018\ubd80\uc0b0\ub300\ud559\u2019\uc73c\ub85c, \ud559\ubd80(\ub2e8\uacfc\ub300\ud559)\ub294 \uc778\ubb38\ud559\ubd80\uc640 \uc218\uc0b0\ud559\ubd80\ub85c \uac1c\uad50\ud558\uc600\uc73c\uba70(\uc774\ub54c\uc758 \ud559\ubd80\ub294 \uc624\ub298\ub0a0\uc758 \ub2e8\uacfc\ub300\ud559(Faculty)\uc5d0 \ud574\ub2f9\ud558\ub294 \uac83\uc73c\ub85c, 1946\ub144 3\uc6d4 7\uc77c\uc5d0 \uacf5\ud3ec\ub41c \ub300\ud559\ub839 \uc81c2\uc870 \ud6c4\ub2e8\uc5d0\ub294 \uc778\ubb38\uacc4 \ubc0f \uc790\uc5f0\uacc4\uc758 \ud559\ubd80\uac00 \ubcd1\uc124\ub420 \ub54c\ub294 2\uac1c \uc774\uc0c1\uc758 \ud559\ubd80\ub85c\uc368 '\uc885\ud569\ub300\ud559\uad50'\ub97c \uad6c\uc131\ud560 \uc218 \uc788\ub3c4\ub85d \uaddc\uc815\ub418\uc5b4 \uc788\uc5c8\ub2e4.", "paragraph_id": "6516756-0-0", "paragraph_question": "question: \ubd80\uc0b0\uc218\uc0b0\uc804\ubb38\ud559\uad50\uac00 \ub300\ud559\uc73c\ub85c \uc2b9\uaca9\ub41c \ud6c4\uc758 \uc774\ub984\uc740 \ubb34\uc5c7\uc778\uac00?, context: 1946\ub144 5\uc6d4 15\uc77c\uc5d0 \ubb38\uad50\ubd80\ub294 \ubd80\uc0b0\uc218\uc0b0\uc804\ubb38\ud559\uad50\ub97c \ub300\ud559\uc73c\ub85c \uc2b9\uaca9\uc2dc\ucf1c \uc885\ud569\ub300\ud559\uc758 1\uac1c \ub2e8\uacfc\ub300\ud559(\uc218\uc0b0\ud559\ubd80)\uc73c\ub85c, \uadf8\uc640 \ubcc4\ub3c4\ub85c \uc778\ubb38\uacc4\ub300\ud559\uc744 \uc2e0\uc124\ud558\uc5ec \uc778\ubb38\uacc4 1\uac1c \ub2e8\uacfc\ub300\ud559(\uc778\ubb38\ud559\ubd80)\uacfc \uc790\uc5f0\uacc4 1\uac1c \ub2e8\uacfc\ub300\ud559(\uc218\uc0b0\ud559\ubd80) \uccb4\uc81c\uc758 \uad6d\ub9bd \ubd80\uc0b0\ub300\ud559\uad50(\uc885\ud569\ub300\ud559) \uc124\ub9bd\uc744 \uc778\uac00\ud558\uc600\ub294\ub370, \ud559\uad50\uc758 \uba85\uce6d\uc740 \u2018\ubd80\uc0b0\ub300\ud559\u2019\uc73c\ub85c, \ud559\ubd80(\ub2e8\uacfc\ub300\ud559)\ub294 \uc778\ubb38\ud559\ubd80\uc640 \uc218\uc0b0\ud559\ubd80\ub85c \uac1c\uad50\ud558\uc600\uc73c\uba70(\uc774\ub54c\uc758 \ud559\ubd80\ub294 \uc624\ub298\ub0a0\uc758 \ub2e8\uacfc\ub300\ud559(Faculty)\uc5d0 \ud574\ub2f9\ud558\ub294 \uac83\uc73c\ub85c, 1946\ub144 3\uc6d4 7\uc77c\uc5d0 \uacf5\ud3ec\ub41c \ub300\ud559\ub839 \uc81c2\uc870 \ud6c4\ub2e8\uc5d0\ub294 \uc778\ubb38\uacc4 \ubc0f \uc790\uc5f0\uacc4\uc758 \ud559\ubd80\uac00 \ubcd1\uc124\ub420 \ub54c\ub294 2\uac1c \uc774\uc0c1\uc758 \ud559\ubd80\ub85c\uc368 '\uc885\ud569\ub300\ud559\uad50'\ub97c \uad6c\uc131\ud560 \uc218 \uc788\ub3c4\ub85d \uaddc\uc815\ub418\uc5b4 \uc788\uc5c8\ub2e4.) \uadf8\ud6c4 1946\ub144 7\uc6d4 \uc608\uacfc \uc2dc\ud5d8\uc744 \uac70\uce58\uace0, 1946\ub144 8\uc6d4\uc5d0\ub294 \ub300\ud559 \ucd1d\uc7a5\uc73c\ub85c \uc544\uc11c \ub9b0 \ubca0\ucee4(Arthur Lynn Becker 1879~1979, \ud55c\uad6d\uba85 \ubc31\uc544\ub355\u767d\u96c5\u60b3) \ubc15\uc0ac\ub97c \uc784\uba85\ud558\uc5ec 1946\ub144 9\uc6d4 \uc785\ud559\uc2dd\uc744 \uac70\ud589\ud558\uace0 1\ub144 \uc815\ub3c4 \uc885\ud569\ub300\ud559\uc73c\ub85c \uc6b4\uc601\ub418\ub2e4\uac00 1947\ub144 7\uc6d4 \uc218\uc0b0\uacfc\ub300\ud559\uc774 '\uad6d\ub9bd \ubd80\uc0b0\uc218\uc0b0\ub300\ud559'\uc73c\ub85c \ubd84\ub9ac\ub428\uc73c\ub85c\uc368 \uc885\ud569\ub300\ud559\uc73c\ub85c\uc368 '\uad6d\ub9bd \ubd80\uc0b0\ub300\ud559\uad50'\uc740 \uc0ac\uc2e4\uc0c1 \uc640\ud574\ub418\uc5c8\ub2e4. \uadf8 \ud6c4 1953\ub144 4\uc6d4 1\uc77c\uc5d0 \uad6d\ub9bd \ubd80\uc0b0\ub300\ud559\uad50 \uc124\ub9bd\uc548\uc774 \uad6d\ubb34\ud68c\uc758\ub97c \ud1b5\uacfc\ud558\uace0, 4\uc6d4 3\uc77c \uad6d\ubb34\ucd1d\ub9ac\uc758 \uc7ac\uac00\ub97c \ubc1b\uc544 \uac19\uc740 \ud574 9\uc6d415\uc77c \uad6d\ub9bd\ud559\uad50\uc124\uce58\ub839\uc774 \uacf5\ud3ec\ub428\uc5d0 \ub530\ub77c \ub2e4\uc2dc \uc885\ud569\ub300\ud559\uc73c\ub85c \uc2b9\uaca9\ub418\uc5c8\ub2e4."} {"question": "\ub098\uce58 \ub3c5\uc77c\uc5d0 \ud761\uc218\ub418\uc5b4 \uc6d4\ub4dc\ucef5 \ubcf8\uc120\uc5d0 \uc9c4\ucd9c\ud558\uc9c0 \ubabb\ud55c \ub098\ub77c\ub294?", "paragraph": "1934\ub144\ubd80\ud130 1978\ub144 \ub300\ud68c\uae4c\uc9c0\ub294 1938\ub144 \ub300\ud68c\uc5d0\uc11c \uc624\uc2a4\ud2b8\ub9ac\uc544\uac00 \ub098\uce58 \ub3c5\uc77c\uc5d0 \uc608\uc120 \ud1b5\uacfc \ud6c4\uc5d0 \ud761\uc218\ub428\uc5d0 \ub530\ub77c 15\uac1c \ud300\uc758 \ucc38\uac00\ub85c \uce58\ub7ec\uc9c4 \uacbd\uc6b0\uc640, 1950\ub144 \ub300\ud68c\uc5d0\uc11c \uc778\ub3c4, \uc2a4\ucf54\ud2c0\ub79c\ub4dc, \ud130\ud0a4\uac00 \uae30\uad8c\ud574\uc11c 13\uac1c \ud300\uc758 \ucc38\uac00\ub85c \uce58\ub7ec\uc9c4 \uacbd\uc6b0\ub97c \uc81c\uc678\ud558\uba74 \ucd1d 16\uac1c\uc758 \ud300\uc774 \ubcf8\uc120\uc5d0\uc11c \uacbd\uc7c1\uc744 \uce58\ub800\ub2e4. \uac00\uc7a5 \ub9ce\uc740 \ucc38\uac00\ub97c \ud55c \uad6d\uac00\ub4e4\uc740 \ub300\ubd80\ubd84 \uc720\ub7fd\uacfc \ub0a8\ubbf8\uc5d0\uc11c \uc628 \uad6d\uac00\ub4e4\uc774\uc5c8\uc73c\uba70 \uc77c\ubd80 \uc18c\uc218\uc758 \ubd81\uc911\ubbf8\uc640 \uc544\ud504\ub9ac\uce74, \uc544\uc2dc\uc544, \uc624\uc138\uc544\ub2c8\uc544\uc758 \ud300\ub4e4\uc774 \ucc38\uac00\ud588\ub2e4. \uc774\ub4e4 \ud300\uc740 \ub300\ubd80\ubd84 \uc27d\uac8c \uc720\ub7fd \ud300\ub4e4\uacfc \ub0a8\ubbf8 \ud300\ub4e4\uc5d0 \ud328\ud588\ub2e4. 1982\ub144\uae4c\uc9c0 \uc720\ub7fd\uacfc \ub0a8\ubbf8\ub97c \uc81c\uc678\ud558\uace0 1\ub77c\uc6b4\ub4dc\ub97c \ubc97\uc5b4\ub09c \ud300\ub4e4\uc740 1930\ub144 \ub300\ud68c\uc5d0\uc11c 4\uac15\uc5d0 \ub4e4\uc5c8\ub358 \ubbf8\uad6d, 1938\ub144 \ub300\ud68c\uc5d0\uc11c 8\uac15\uc5d0 \ub4e4\uc5c8\ub358 \ucfe0\ubc14, 1966\ub144 \ub300\ud68c\uc5d0\uc11c 8\uac15\uc5d0 \ub4e4\uc5c8\ub358 \uc870\uc120\ubbfc\uc8fc\uc8fc\uc758\uc778\ubbfc\uacf5\ud654\uad6d, 1970\ub144 \ub300\ud68c\uc5d0\uc11c 8\uac15\uc5d0 \ub4e4\uc5c8\ub358 \uba55\uc2dc\ucf54\uac00 \uc804\ubd80\uc600\ub2e4.", "answer": "\uc624\uc2a4\ud2b8\ub9ac\uc544", "sentence": "1934\ub144\ubd80\ud130 1978\ub144 \ub300\ud68c\uae4c\uc9c0\ub294 1938\ub144 \ub300\ud68c\uc5d0\uc11c \uc624\uc2a4\ud2b8\ub9ac\uc544 \uac00 \ub098\uce58 \ub3c5\uc77c\uc5d0 \uc608\uc120 \ud1b5\uacfc \ud6c4\uc5d0 \ud761\uc218\ub428\uc5d0 \ub530\ub77c 15\uac1c \ud300\uc758 \ucc38\uac00\ub85c \uce58\ub7ec\uc9c4 \uacbd\uc6b0\uc640, 1950\ub144 \ub300\ud68c\uc5d0\uc11c \uc778\ub3c4, \uc2a4\ucf54\ud2c0\ub79c\ub4dc, \ud130\ud0a4\uac00 \uae30\uad8c\ud574\uc11c 13\uac1c \ud300\uc758 \ucc38\uac00\ub85c \uce58\ub7ec\uc9c4 \uacbd\uc6b0\ub97c \uc81c\uc678\ud558\uba74 \ucd1d 16\uac1c\uc758 \ud300\uc774 \ubcf8\uc120\uc5d0\uc11c \uacbd\uc7c1\uc744 \uce58\ub800\ub2e4.", "paragraph_sentence": " 1934\ub144\ubd80\ud130 1978\ub144 \ub300\ud68c\uae4c\uc9c0\ub294 1938\ub144 \ub300\ud68c\uc5d0\uc11c \uc624\uc2a4\ud2b8\ub9ac\uc544 \uac00 \ub098\uce58 \ub3c5\uc77c\uc5d0 \uc608\uc120 \ud1b5\uacfc \ud6c4\uc5d0 \ud761\uc218\ub428\uc5d0 \ub530\ub77c 15\uac1c \ud300\uc758 \ucc38\uac00\ub85c \uce58\ub7ec\uc9c4 \uacbd\uc6b0\uc640, 1950\ub144 \ub300\ud68c\uc5d0\uc11c \uc778\ub3c4, \uc2a4\ucf54\ud2c0\ub79c\ub4dc, \ud130\ud0a4\uac00 \uae30\uad8c\ud574\uc11c 13\uac1c \ud300\uc758 \ucc38\uac00\ub85c \uce58\ub7ec\uc9c4 \uacbd\uc6b0\ub97c \uc81c\uc678\ud558\uba74 \ucd1d 16\uac1c\uc758 \ud300\uc774 \ubcf8\uc120\uc5d0\uc11c \uacbd\uc7c1\uc744 \uce58\ub800\ub2e4. \uac00\uc7a5 \ub9ce\uc740 \ucc38\uac00\ub97c \ud55c \uad6d\uac00\ub4e4\uc740 \ub300\ubd80\ubd84 \uc720\ub7fd\uacfc \ub0a8\ubbf8\uc5d0\uc11c \uc628 \uad6d\uac00\ub4e4\uc774\uc5c8\uc73c\uba70 \uc77c\ubd80 \uc18c\uc218\uc758 \ubd81\uc911\ubbf8\uc640 \uc544\ud504\ub9ac\uce74, \uc544\uc2dc\uc544, \uc624\uc138\uc544\ub2c8\uc544\uc758 \ud300\ub4e4\uc774 \ucc38\uac00\ud588\ub2e4. \uc774\ub4e4 \ud300\uc740 \ub300\ubd80\ubd84 \uc27d\uac8c \uc720\ub7fd \ud300\ub4e4\uacfc \ub0a8\ubbf8 \ud300\ub4e4\uc5d0 \ud328\ud588\ub2e4. 1982\ub144\uae4c\uc9c0 \uc720\ub7fd\uacfc \ub0a8\ubbf8\ub97c \uc81c\uc678\ud558\uace0 1\ub77c\uc6b4\ub4dc\ub97c \ubc97\uc5b4\ub09c \ud300\ub4e4\uc740 1930\ub144 \ub300\ud68c\uc5d0\uc11c 4\uac15\uc5d0 \ub4e4\uc5c8\ub358 \ubbf8\uad6d, 1938\ub144 \ub300\ud68c\uc5d0\uc11c 8\uac15\uc5d0 \ub4e4\uc5c8\ub358 \ucfe0\ubc14, 1966\ub144 \ub300\ud68c\uc5d0\uc11c 8\uac15\uc5d0 \ub4e4\uc5c8\ub358 \uc870\uc120\ubbfc\uc8fc\uc8fc\uc758\uc778\ubbfc\uacf5\ud654\uad6d, 1970\ub144 \ub300\ud68c\uc5d0\uc11c 8\uac15\uc5d0 \ub4e4\uc5c8\ub358 \uba55\uc2dc\ucf54\uac00 \uc804\ubd80\uc600\ub2e4.", "paragraph_answer": "1934\ub144\ubd80\ud130 1978\ub144 \ub300\ud68c\uae4c\uc9c0\ub294 1938\ub144 \ub300\ud68c\uc5d0\uc11c \uc624\uc2a4\ud2b8\ub9ac\uc544 \uac00 \ub098\uce58 \ub3c5\uc77c\uc5d0 \uc608\uc120 \ud1b5\uacfc \ud6c4\uc5d0 \ud761\uc218\ub428\uc5d0 \ub530\ub77c 15\uac1c \ud300\uc758 \ucc38\uac00\ub85c \uce58\ub7ec\uc9c4 \uacbd\uc6b0\uc640, 1950\ub144 \ub300\ud68c\uc5d0\uc11c \uc778\ub3c4, \uc2a4\ucf54\ud2c0\ub79c\ub4dc, \ud130\ud0a4\uac00 \uae30\uad8c\ud574\uc11c 13\uac1c \ud300\uc758 \ucc38\uac00\ub85c \uce58\ub7ec\uc9c4 \uacbd\uc6b0\ub97c \uc81c\uc678\ud558\uba74 \ucd1d 16\uac1c\uc758 \ud300\uc774 \ubcf8\uc120\uc5d0\uc11c \uacbd\uc7c1\uc744 \uce58\ub800\ub2e4. \uac00\uc7a5 \ub9ce\uc740 \ucc38\uac00\ub97c \ud55c \uad6d\uac00\ub4e4\uc740 \ub300\ubd80\ubd84 \uc720\ub7fd\uacfc \ub0a8\ubbf8\uc5d0\uc11c \uc628 \uad6d\uac00\ub4e4\uc774\uc5c8\uc73c\uba70 \uc77c\ubd80 \uc18c\uc218\uc758 \ubd81\uc911\ubbf8\uc640 \uc544\ud504\ub9ac\uce74, \uc544\uc2dc\uc544, \uc624\uc138\uc544\ub2c8\uc544\uc758 \ud300\ub4e4\uc774 \ucc38\uac00\ud588\ub2e4. \uc774\ub4e4 \ud300\uc740 \ub300\ubd80\ubd84 \uc27d\uac8c \uc720\ub7fd \ud300\ub4e4\uacfc \ub0a8\ubbf8 \ud300\ub4e4\uc5d0 \ud328\ud588\ub2e4. 1982\ub144\uae4c\uc9c0 \uc720\ub7fd\uacfc \ub0a8\ubbf8\ub97c \uc81c\uc678\ud558\uace0 1\ub77c\uc6b4\ub4dc\ub97c \ubc97\uc5b4\ub09c \ud300\ub4e4\uc740 1930\ub144 \ub300\ud68c\uc5d0\uc11c 4\uac15\uc5d0 \ub4e4\uc5c8\ub358 \ubbf8\uad6d, 1938\ub144 \ub300\ud68c\uc5d0\uc11c 8\uac15\uc5d0 \ub4e4\uc5c8\ub358 \ucfe0\ubc14, 1966\ub144 \ub300\ud68c\uc5d0\uc11c 8\uac15\uc5d0 \ub4e4\uc5c8\ub358 \uc870\uc120\ubbfc\uc8fc\uc8fc\uc758\uc778\ubbfc\uacf5\ud654\uad6d, 1970\ub144 \ub300\ud68c\uc5d0\uc11c 8\uac15\uc5d0 \ub4e4\uc5c8\ub358 \uba55\uc2dc\ucf54\uac00 \uc804\ubd80\uc600\ub2e4.", "sentence_answer": "1934\ub144\ubd80\ud130 1978\ub144 \ub300\ud68c\uae4c\uc9c0\ub294 1938\ub144 \ub300\ud68c\uc5d0\uc11c \uc624\uc2a4\ud2b8\ub9ac\uc544 \uac00 \ub098\uce58 \ub3c5\uc77c\uc5d0 \uc608\uc120 \ud1b5\uacfc \ud6c4\uc5d0 \ud761\uc218\ub428\uc5d0 \ub530\ub77c 15\uac1c \ud300\uc758 \ucc38\uac00\ub85c \uce58\ub7ec\uc9c4 \uacbd\uc6b0\uc640, 1950\ub144 \ub300\ud68c\uc5d0\uc11c \uc778\ub3c4, \uc2a4\ucf54\ud2c0\ub79c\ub4dc, \ud130\ud0a4\uac00 \uae30\uad8c\ud574\uc11c 13\uac1c \ud300\uc758 \ucc38\uac00\ub85c \uce58\ub7ec\uc9c4 \uacbd\uc6b0\ub97c \uc81c\uc678\ud558\uba74 \ucd1d 16\uac1c\uc758 \ud300\uc774 \ubcf8\uc120\uc5d0\uc11c \uacbd\uc7c1\uc744 \uce58\ub800\ub2e4.", "paragraph_id": "6523258-3-2", "paragraph_question": "question: \ub098\uce58 \ub3c5\uc77c\uc5d0 \ud761\uc218\ub418\uc5b4 \uc6d4\ub4dc\ucef5 \ubcf8\uc120\uc5d0 \uc9c4\ucd9c\ud558\uc9c0 \ubabb\ud55c \ub098\ub77c\ub294?, context: 1934\ub144\ubd80\ud130 1978\ub144 \ub300\ud68c\uae4c\uc9c0\ub294 1938\ub144 \ub300\ud68c\uc5d0\uc11c \uc624\uc2a4\ud2b8\ub9ac\uc544\uac00 \ub098\uce58 \ub3c5\uc77c\uc5d0 \uc608\uc120 \ud1b5\uacfc \ud6c4\uc5d0 \ud761\uc218\ub428\uc5d0 \ub530\ub77c 15\uac1c \ud300\uc758 \ucc38\uac00\ub85c \uce58\ub7ec\uc9c4 \uacbd\uc6b0\uc640, 1950\ub144 \ub300\ud68c\uc5d0\uc11c \uc778\ub3c4, \uc2a4\ucf54\ud2c0\ub79c\ub4dc, \ud130\ud0a4\uac00 \uae30\uad8c\ud574\uc11c 13\uac1c \ud300\uc758 \ucc38\uac00\ub85c \uce58\ub7ec\uc9c4 \uacbd\uc6b0\ub97c \uc81c\uc678\ud558\uba74 \ucd1d 16\uac1c\uc758 \ud300\uc774 \ubcf8\uc120\uc5d0\uc11c \uacbd\uc7c1\uc744 \uce58\ub800\ub2e4. \uac00\uc7a5 \ub9ce\uc740 \ucc38\uac00\ub97c \ud55c \uad6d\uac00\ub4e4\uc740 \ub300\ubd80\ubd84 \uc720\ub7fd\uacfc \ub0a8\ubbf8\uc5d0\uc11c \uc628 \uad6d\uac00\ub4e4\uc774\uc5c8\uc73c\uba70 \uc77c\ubd80 \uc18c\uc218\uc758 \ubd81\uc911\ubbf8\uc640 \uc544\ud504\ub9ac\uce74, \uc544\uc2dc\uc544, \uc624\uc138\uc544\ub2c8\uc544\uc758 \ud300\ub4e4\uc774 \ucc38\uac00\ud588\ub2e4. \uc774\ub4e4 \ud300\uc740 \ub300\ubd80\ubd84 \uc27d\uac8c \uc720\ub7fd \ud300\ub4e4\uacfc \ub0a8\ubbf8 \ud300\ub4e4\uc5d0 \ud328\ud588\ub2e4. 1982\ub144\uae4c\uc9c0 \uc720\ub7fd\uacfc \ub0a8\ubbf8\ub97c \uc81c\uc678\ud558\uace0 1\ub77c\uc6b4\ub4dc\ub97c \ubc97\uc5b4\ub09c \ud300\ub4e4\uc740 1930\ub144 \ub300\ud68c\uc5d0\uc11c 4\uac15\uc5d0 \ub4e4\uc5c8\ub358 \ubbf8\uad6d, 1938\ub144 \ub300\ud68c\uc5d0\uc11c 8\uac15\uc5d0 \ub4e4\uc5c8\ub358 \ucfe0\ubc14, 1966\ub144 \ub300\ud68c\uc5d0\uc11c 8\uac15\uc5d0 \ub4e4\uc5c8\ub358 \uc870\uc120\ubbfc\uc8fc\uc8fc\uc758\uc778\ubbfc\uacf5\ud654\uad6d, 1970\ub144 \ub300\ud68c\uc5d0\uc11c 8\uac15\uc5d0 \ub4e4\uc5c8\ub358 \uba55\uc2dc\ucf54\uac00 \uc804\ubd80\uc600\ub2e4."} {"question": "\ucc48\ub4e4\ub7ec\ub294 \ubaa8\ub2c8\uce74\uc640 \ub3d9\uac70\ud558\uae30 \uc804 6\uc2dc\uc98c\uae4c\uc9c0 \ub204\uad6c\uc640 \uc0ac\ub294\uac00\uc694?", "paragraph": "\ucc48\ub4e4\ub7ec \ube59\uc740 \ub85c\uc2a4 \uac94\ub7ec\uc758 \ub300\ud559\uc2dc\uc808 \ub8f8\uba54\uc774\ud2b8\ub85c \uc5d0\ub85c \uc18c\uc124 \uc791\uac00\uc778 \ub178\ub77c \ud0c0\uc77c\ub7ec \ube59(Nora Tyler Bing)\uacfc \ub77c\uc2a4\ubca0\uac00\uc2a4 \ubc84\ub77c\uc774\uc5b4\ud2f0 \uc1fc \uc2a4\ud0c0\uc778 \ucc30\uc2a4 \ube59(Charels Bing)\uc0ac\uc774\uc758 \uc678\uc544\ub4e4\uc774\ub2e4. \ub300\ubd80\ubd84\uc758 \uc2dc\ub9ac\uc988\uc5d0\uc11c \ucc48\ub4e4\ub7ec\ub294 '\ud1b5\uacc4\uc801 \ubd84\uc11d'\uacfc '\ub370\uc774\ud130 \uc778\uc2dd' \uad00\ub9ac \uc804\ubb38\uac00\ub85c \ub098\uc628\ub2e4.\ub098\uc911\uc5d0\ub294 \uad11\uace0\ud68c\uc0ac\uc5d0\uc11c \uce74\ud53c\ub77c\uc774\ud130\ub85c \uc77c\ud558\uac8c \ub41c\ub2e4. \ub300\ud559 \uc7ac\ud559\uc2dc\uc808\uc758 \ucd94\uc218\uac10\uc0ac\uc808\ub54c \ub2f9\uc2dc \ub8f8\uba54\uc774\ud2b8 \ub85c\uc2a4 \uac94\ub7ec\uc758 \uc5ec\ub3d9\uc0dd\uc778 \ubaa8\ub2c8\uce74 \uac94\ub7ec\uc640 \uadf8\ub140\uc758 \uce5c\uad6c \ub808\uc774\uccbc \uadf8\ub9b0\uc744 \ub9cc\ub098\uace0, \ud6c4\uc5d0 \ubaa8\ub2c8\uce74\uc758 \ub274\uc695 \uc544\ud30c\ud2b8 \uc55e\uc9d1(19\ud638)\uc73c\ub85c \uc774\uc0ac\ud558\uac8c \ub418\uba74\uc11c \ubaa8\ub2c8\uce74\uc758 \ub8f8\uba54\uc774\ud2b8\uc600\ub358 \ud53c\ube44 \ubd80\ud398\uc774\uc640 \ub9cc\ub098\uac8c \ub41c\ub2e4. \uadf8 \ub54c \ub2f9\uc2dc \ucc48\ub4e4\ub7ec\uc758 \ub8f8\uba54\uc774\ud2b8\ub294 \ud0b5(Kip,\uc2dc\ub9ac\uc988\uc5d0 \ub4f1\uc7a5\ud558\uc9c0 \uc54a\ub294\ub2e4)\uc774\uc5c8\uc9c0\ub9cc \ud0b5\uc740 \uccab \uc2dc\ub9ac\uc988\uac00 \uc2dc\uc791\ub418\uae30 \uc804\uc5d0 \ub5a0\ub09c\ub2e4. \uc870\uc774 \ud2b8\ub9ac\ube44\uc544\ub2c8\uac00 \ucc48\ub4e4\ub7ec\uc758 \ub2e4\uc74c \ub8f8\uba54\uc774\ud2b8\ub85c \ub4e4\uc5b4\uc624\uac8c \ub418\uba74\uc11c, <\ud504\ub80c\uc988>\uc758 \uc5ec\uc12f \uce5c\uad6c\ub4e4 \uadf8\ub8f9\uc774 \ud0c4\uc0dd\ud558\uac8c \ub41c\ub2e4. \ucc48\ub4e4\ub7ec\ub294 \ubaa8\ub2c8\uce74\uc640 \ub3d9\uac70\ud558\uae30 \uc804 6\uc2dc\uc98c\uae4c\uc9c0 \uc870\uc774\uc640 \ud568\uaed8 \uc0b4\uba70,7\uc2dc\uc98c\uc5d0\uc11c \ubaa8\ub2c8\uce74\uc640 \uacb0\ud63c\ud558\uac8c \ub41c\ub2e4.", "answer": "\uc870\uc774", "sentence": "\uc870\uc774 \ud2b8\ub9ac\ube44\uc544\ub2c8\uac00 \ucc48\ub4e4\ub7ec\uc758 \ub2e4\uc74c \ub8f8\uba54\uc774\ud2b8\ub85c \ub4e4\uc5b4\uc624\uac8c \ub418\uba74\uc11c, <\ud504\ub80c\uc988>\uc758 \uc5ec\uc12f \uce5c\uad6c\ub4e4 \uadf8\ub8f9\uc774 \ud0c4\uc0dd\ud558\uac8c \ub41c\ub2e4.", "paragraph_sentence": "\ucc48\ub4e4\ub7ec \ube59\uc740 \ub85c\uc2a4 \uac94\ub7ec\uc758 \ub300\ud559\uc2dc\uc808 \ub8f8\uba54\uc774\ud2b8\ub85c \uc5d0\ub85c \uc18c\uc124 \uc791\uac00\uc778 \ub178\ub77c \ud0c0\uc77c\ub7ec \ube59(Nora Tyler Bing)\uacfc \ub77c\uc2a4\ubca0\uac00\uc2a4 \ubc84\ub77c\uc774\uc5b4\ud2f0 \uc1fc \uc2a4\ud0c0\uc778 \ucc30\uc2a4 \ube59(Charels Bing)\uc0ac\uc774\uc758 \uc678\uc544\ub4e4\uc774\ub2e4. \ub300\ubd80\ubd84\uc758 \uc2dc\ub9ac\uc988\uc5d0\uc11c \ucc48\ub4e4\ub7ec\ub294 '\ud1b5\uacc4\uc801 \ubd84\uc11d'\uacfc '\ub370\uc774\ud130 \uc778\uc2dd' \uad00\ub9ac \uc804\ubb38\uac00\ub85c \ub098\uc628\ub2e4.\ub098\uc911\uc5d0\ub294 \uad11\uace0\ud68c\uc0ac\uc5d0\uc11c \uce74\ud53c\ub77c\uc774\ud130\ub85c \uc77c\ud558\uac8c \ub41c\ub2e4. \ub300\ud559 \uc7ac\ud559\uc2dc\uc808\uc758 \ucd94\uc218\uac10\uc0ac\uc808\ub54c \ub2f9\uc2dc \ub8f8\uba54\uc774\ud2b8 \ub85c\uc2a4 \uac94\ub7ec\uc758 \uc5ec\ub3d9\uc0dd\uc778 \ubaa8\ub2c8\uce74 \uac94\ub7ec\uc640 \uadf8\ub140\uc758 \uce5c\uad6c \ub808\uc774\uccbc \uadf8\ub9b0\uc744 \ub9cc\ub098\uace0, \ud6c4\uc5d0 \ubaa8\ub2c8\uce74\uc758 \ub274\uc695 \uc544\ud30c\ud2b8 \uc55e\uc9d1(19\ud638)\uc73c\ub85c \uc774\uc0ac\ud558\uac8c \ub418\uba74\uc11c \ubaa8\ub2c8\uce74\uc758 \ub8f8\uba54\uc774\ud2b8\uc600\ub358 \ud53c\ube44 \ubd80\ud398\uc774\uc640 \ub9cc\ub098\uac8c \ub41c\ub2e4. \uadf8 \ub54c \ub2f9\uc2dc \ucc48\ub4e4\ub7ec\uc758 \ub8f8\uba54\uc774\ud2b8\ub294 \ud0b5(Kip,\uc2dc\ub9ac\uc988\uc5d0 \ub4f1\uc7a5\ud558\uc9c0 \uc54a\ub294\ub2e4)\uc774\uc5c8\uc9c0\ub9cc \ud0b5\uc740 \uccab \uc2dc\ub9ac\uc988\uac00 \uc2dc\uc791\ub418\uae30 \uc804\uc5d0 \ub5a0\ub09c\ub2e4. \uc870\uc774 \ud2b8\ub9ac\ube44\uc544\ub2c8\uac00 \ucc48\ub4e4\ub7ec\uc758 \ub2e4\uc74c \ub8f8\uba54\uc774\ud2b8\ub85c \ub4e4\uc5b4\uc624\uac8c \ub418\uba74\uc11c, <\ud504\ub80c\uc988>\uc758 \uc5ec\uc12f \uce5c\uad6c\ub4e4 \uadf8\ub8f9\uc774 \ud0c4\uc0dd\ud558\uac8c \ub41c\ub2e4. \ucc48\ub4e4\ub7ec\ub294 \ubaa8\ub2c8\uce74\uc640 \ub3d9\uac70\ud558\uae30 \uc804 6\uc2dc\uc98c\uae4c\uc9c0 \uc870\uc774\uc640 \ud568\uaed8 \uc0b4\uba70,7\uc2dc\uc98c\uc5d0\uc11c \ubaa8\ub2c8\uce74\uc640 \uacb0\ud63c\ud558\uac8c \ub41c\ub2e4.", "paragraph_answer": "\ucc48\ub4e4\ub7ec \ube59\uc740 \ub85c\uc2a4 \uac94\ub7ec\uc758 \ub300\ud559\uc2dc\uc808 \ub8f8\uba54\uc774\ud2b8\ub85c \uc5d0\ub85c \uc18c\uc124 \uc791\uac00\uc778 \ub178\ub77c \ud0c0\uc77c\ub7ec \ube59(Nora Tyler Bing)\uacfc \ub77c\uc2a4\ubca0\uac00\uc2a4 \ubc84\ub77c\uc774\uc5b4\ud2f0 \uc1fc \uc2a4\ud0c0\uc778 \ucc30\uc2a4 \ube59(Charels Bing)\uc0ac\uc774\uc758 \uc678\uc544\ub4e4\uc774\ub2e4. \ub300\ubd80\ubd84\uc758 \uc2dc\ub9ac\uc988\uc5d0\uc11c \ucc48\ub4e4\ub7ec\ub294 '\ud1b5\uacc4\uc801 \ubd84\uc11d'\uacfc '\ub370\uc774\ud130 \uc778\uc2dd' \uad00\ub9ac \uc804\ubb38\uac00\ub85c \ub098\uc628\ub2e4.\ub098\uc911\uc5d0\ub294 \uad11\uace0\ud68c\uc0ac\uc5d0\uc11c \uce74\ud53c\ub77c\uc774\ud130\ub85c \uc77c\ud558\uac8c \ub41c\ub2e4. \ub300\ud559 \uc7ac\ud559\uc2dc\uc808\uc758 \ucd94\uc218\uac10\uc0ac\uc808\ub54c \ub2f9\uc2dc \ub8f8\uba54\uc774\ud2b8 \ub85c\uc2a4 \uac94\ub7ec\uc758 \uc5ec\ub3d9\uc0dd\uc778 \ubaa8\ub2c8\uce74 \uac94\ub7ec\uc640 \uadf8\ub140\uc758 \uce5c\uad6c \ub808\uc774\uccbc \uadf8\ub9b0\uc744 \ub9cc\ub098\uace0, \ud6c4\uc5d0 \ubaa8\ub2c8\uce74\uc758 \ub274\uc695 \uc544\ud30c\ud2b8 \uc55e\uc9d1(19\ud638)\uc73c\ub85c \uc774\uc0ac\ud558\uac8c \ub418\uba74\uc11c \ubaa8\ub2c8\uce74\uc758 \ub8f8\uba54\uc774\ud2b8\uc600\ub358 \ud53c\ube44 \ubd80\ud398\uc774\uc640 \ub9cc\ub098\uac8c \ub41c\ub2e4. \uadf8 \ub54c \ub2f9\uc2dc \ucc48\ub4e4\ub7ec\uc758 \ub8f8\uba54\uc774\ud2b8\ub294 \ud0b5(Kip,\uc2dc\ub9ac\uc988\uc5d0 \ub4f1\uc7a5\ud558\uc9c0 \uc54a\ub294\ub2e4)\uc774\uc5c8\uc9c0\ub9cc \ud0b5\uc740 \uccab \uc2dc\ub9ac\uc988\uac00 \uc2dc\uc791\ub418\uae30 \uc804\uc5d0 \ub5a0\ub09c\ub2e4. \uc870\uc774 \ud2b8\ub9ac\ube44\uc544\ub2c8\uac00 \ucc48\ub4e4\ub7ec\uc758 \ub2e4\uc74c \ub8f8\uba54\uc774\ud2b8\ub85c \ub4e4\uc5b4\uc624\uac8c \ub418\uba74\uc11c, <\ud504\ub80c\uc988>\uc758 \uc5ec\uc12f \uce5c\uad6c\ub4e4 \uadf8\ub8f9\uc774 \ud0c4\uc0dd\ud558\uac8c \ub41c\ub2e4. \ucc48\ub4e4\ub7ec\ub294 \ubaa8\ub2c8\uce74\uc640 \ub3d9\uac70\ud558\uae30 \uc804 6\uc2dc\uc98c\uae4c\uc9c0 \uc870\uc774\uc640 \ud568\uaed8 \uc0b4\uba70,7\uc2dc\uc98c\uc5d0\uc11c \ubaa8\ub2c8\uce74\uc640 \uacb0\ud63c\ud558\uac8c \ub41c\ub2e4.", "sentence_answer": " \uc870\uc774 \ud2b8\ub9ac\ube44\uc544\ub2c8\uac00 \ucc48\ub4e4\ub7ec\uc758 \ub2e4\uc74c \ub8f8\uba54\uc774\ud2b8\ub85c \ub4e4\uc5b4\uc624\uac8c \ub418\uba74\uc11c, <\ud504\ub80c\uc988>\uc758 \uc5ec\uc12f \uce5c\uad6c\ub4e4 \uadf8\ub8f9\uc774 \ud0c4\uc0dd\ud558\uac8c \ub41c\ub2e4.", "paragraph_id": "5897296-0-2", "paragraph_question": "question: \ucc48\ub4e4\ub7ec\ub294 \ubaa8\ub2c8\uce74\uc640 \ub3d9\uac70\ud558\uae30 \uc804 6\uc2dc\uc98c\uae4c\uc9c0 \ub204\uad6c\uc640 \uc0ac\ub294\uac00\uc694?, context: \ucc48\ub4e4\ub7ec \ube59\uc740 \ub85c\uc2a4 \uac94\ub7ec\uc758 \ub300\ud559\uc2dc\uc808 \ub8f8\uba54\uc774\ud2b8\ub85c \uc5d0\ub85c \uc18c\uc124 \uc791\uac00\uc778 \ub178\ub77c \ud0c0\uc77c\ub7ec \ube59(Nora Tyler Bing)\uacfc \ub77c\uc2a4\ubca0\uac00\uc2a4 \ubc84\ub77c\uc774\uc5b4\ud2f0 \uc1fc \uc2a4\ud0c0\uc778 \ucc30\uc2a4 \ube59(Charels Bing)\uc0ac\uc774\uc758 \uc678\uc544\ub4e4\uc774\ub2e4. \ub300\ubd80\ubd84\uc758 \uc2dc\ub9ac\uc988\uc5d0\uc11c \ucc48\ub4e4\ub7ec\ub294 '\ud1b5\uacc4\uc801 \ubd84\uc11d'\uacfc '\ub370\uc774\ud130 \uc778\uc2dd' \uad00\ub9ac \uc804\ubb38\uac00\ub85c \ub098\uc628\ub2e4.\ub098\uc911\uc5d0\ub294 \uad11\uace0\ud68c\uc0ac\uc5d0\uc11c \uce74\ud53c\ub77c\uc774\ud130\ub85c \uc77c\ud558\uac8c \ub41c\ub2e4. \ub300\ud559 \uc7ac\ud559\uc2dc\uc808\uc758 \ucd94\uc218\uac10\uc0ac\uc808\ub54c \ub2f9\uc2dc \ub8f8\uba54\uc774\ud2b8 \ub85c\uc2a4 \uac94\ub7ec\uc758 \uc5ec\ub3d9\uc0dd\uc778 \ubaa8\ub2c8\uce74 \uac94\ub7ec\uc640 \uadf8\ub140\uc758 \uce5c\uad6c \ub808\uc774\uccbc \uadf8\ub9b0\uc744 \ub9cc\ub098\uace0, \ud6c4\uc5d0 \ubaa8\ub2c8\uce74\uc758 \ub274\uc695 \uc544\ud30c\ud2b8 \uc55e\uc9d1(19\ud638)\uc73c\ub85c \uc774\uc0ac\ud558\uac8c \ub418\uba74\uc11c \ubaa8\ub2c8\uce74\uc758 \ub8f8\uba54\uc774\ud2b8\uc600\ub358 \ud53c\ube44 \ubd80\ud398\uc774\uc640 \ub9cc\ub098\uac8c \ub41c\ub2e4. \uadf8 \ub54c \ub2f9\uc2dc \ucc48\ub4e4\ub7ec\uc758 \ub8f8\uba54\uc774\ud2b8\ub294 \ud0b5(Kip,\uc2dc\ub9ac\uc988\uc5d0 \ub4f1\uc7a5\ud558\uc9c0 \uc54a\ub294\ub2e4)\uc774\uc5c8\uc9c0\ub9cc \ud0b5\uc740 \uccab \uc2dc\ub9ac\uc988\uac00 \uc2dc\uc791\ub418\uae30 \uc804\uc5d0 \ub5a0\ub09c\ub2e4. \uc870\uc774 \ud2b8\ub9ac\ube44\uc544\ub2c8\uac00 \ucc48\ub4e4\ub7ec\uc758 \ub2e4\uc74c \ub8f8\uba54\uc774\ud2b8\ub85c \ub4e4\uc5b4\uc624\uac8c \ub418\uba74\uc11c, <\ud504\ub80c\uc988>\uc758 \uc5ec\uc12f \uce5c\uad6c\ub4e4 \uadf8\ub8f9\uc774 \ud0c4\uc0dd\ud558\uac8c \ub41c\ub2e4. \ucc48\ub4e4\ub7ec\ub294 \ubaa8\ub2c8\uce74\uc640 \ub3d9\uac70\ud558\uae30 \uc804 6\uc2dc\uc98c\uae4c\uc9c0 \uc870\uc774\uc640 \ud568\uaed8 \uc0b4\uba70,7\uc2dc\uc98c\uc5d0\uc11c \ubaa8\ub2c8\uce74\uc640 \uacb0\ud63c\ud558\uac8c \ub41c\ub2e4."} {"question": "\uccad\uc640\ub300\ub294 \uc815\uad8c\uc774 \ubc14\ub00c\uac70\ub098 \uc815\uce58\uc801 \uc0c1\ud669\uc774 \ubcc0\ud654\ub418\ub354\ub77c\ub3c4 \ud310\ubb38\uc810 \uc120\uc5b8\uc758 \ub0b4\uc6a9\uc774 \uc9c0\uc18d\ub420 \uc218 \uc788\ub3c4\ub85d \ubc1b\uace0\uc790 \ud55c \uac83\uc740?", "paragraph": "\ud310\ubb38\uc810 \uc120\uc5b8\uc740 \ubd84\ub2e8\uacfc \ub300\uacb0\uc758 \uc885\uc2dd, \ubbfc\uc871 \ud654\ud574\uc640 \ud3c9\ud654 \ubc88\uc601\uc758 \uc0c8 \uc2dc\ub300, \ub0a8\ubd81\uad00\uacc4\uc758 \uac1c\uc120\uacfc \ubc1c\uc804\uc774\ub77c\ub294 \ud655\uace0\ud55c \uc758\uc9c0\ub97c \ub2f4\uc558\uc73c\uba70 \ud569\uc758 \ub0b4\uc6a9\uc5d0 \ub300\ud574 \ubb38\uc7ac\uc778\uc740 \"\uacb0\ucf54 \ub4a4\ub3cc\uc544\uac00\uc9c0 \uc54a\uc744 \uac83\"\uc774\uace0 \uae40\uc815\uc740\uc740 \"\uae34\ubc00\ud788 \ud611\ub825\ud574 \ubc18\ub4dc\uc2dc \uc88b\uc740 \uacb0\uc2e4\uc774 \ub9fa\uc5b4\uc9c0\ub3c4\ub85d \ub178\ub825\ud574\ub098\uac08 \uac83\"\uc774\ub77c\uace0 \uac15\uc870\ud588\ub2e4. \uc138\ubd80\uc801\uc73c\ub85c \ub2e8\uacc4\uc801 \uad70\ucd95, \uc801\ub300\uc801 \ud589\uc704 \uc911\uc9c0, \ubd81\ubc29\ud55c\uacc4\uc120\uc744 \ud3c9\ud654\uc218\uc5ed\ud654 \ub4f1\uc744 \ud569\uc758\ud588\ub2e4. \ub2e4\ub9cc, \uc7a5\uae30 \uacfc\uc81c\ub85c\uc11c \ud1b5\uc77c\uacfc \uad00\ub828\ud558\uc5ec \uad6c\uccb4\uc801\uc778 \ub300\ud654\ub294 \uc774\ub8e8\uc5b4\uc9c0\uc9c0 \uc54a\uc558\ub2e4. \uac01\uad6d\uc758 \uc9c0\uc9c0\ub3c4 \uc787\ub530\ub790\ub2e4. \ub3c4\ub110\ub4dc \ud2b8\ub7fc\ud504 \ubbf8\uad6d \ub300\ud1b5\ub839\uc740 \"\uc88b\uc740 \uc77c\ub4e4\uc774 \uc77c\uc5b4\ub098\uace0 \uc788\ub2e4. \uadf8\ub7ec\ub098 \uc624\uc9c1 \uc2dc\uac04\uc774 \ub9d0\ud574\uc904 \uac83\"\uc774\ub77c\uba70 \ud658\uc601\uc774 \ub73b\uc744 \ubc1d\ud614\ub2e4. \uc911\ud654\uc778\ubbfc\uacf5\ud654\uad6d \uc678\uad50\ubd80\ub294 \ub0a8\ubd81\uacfc \uc9c0\uc5ed\uc0ac\ud68c \ubc0f \uad6d\uc81c\uc0ac\ud68c\uc758 \uacf5\ud1b5\ub41c \uae30\ub300\uc5d0 \ubd80\ud569\ud55c\ub2e4\uace0 \uc9c0\uc9c0 \uc758\uc0ac\ub97c \ubcf4\ub0c8\uc73c\uba70 \ub7ec\uc2dc\uc544\ub3c4 \"\ud68c\ub2f4 \uacb0\uacfc\ub97c \uc544\uc8fc \uae0d\uc815\uc801\uc73c\ub85c \ud3c9\uac00\ud55c\ub2e4\"\uace0 \uac15\uc870\ud588\ub2e4. \uc77c\ubcf8 \uc5ed\uc2dc \"\ud55c\uad6d \uc815\ubd80\uc758 \ub178\ub825\uc744 \uce6d\ucc2c\ud558\uace0 \uc2f6\ub2e4\"\uace0 \uae0d\uc815\uc801\uc778 \ud3c9\uac00\ub97c \ubcf4\uc600\uc9c0\ub9cc \"\uc55e\uc73c\ub85c \ubd81\ud55c\uc758 \ub3d9\ud5a5\uc744 \uc8fc\uc2dc\ud560 \uac83\"\uc774\ub77c\uba70 \uacbd\uacc4\ub97c \ubcf4\uc774\uae30\ub3c4 \ud588\ub2e4. \uc601\uad6d, \ud504\ub791\uc2a4, \uc774\ud0c8\ub9ac\uc544\ub294 \ubb3c\ub860 \uc911\ub9bd\uad6d\uac10\ub3c5\uc704\uc6d0\ud68c\uc758 \uc77c\uc6d0\uc778 \uc2a4\uc6e8\ub374\uacfc \uc2a4\uc704\uc2a4\ub3c4 \ub0a8\ubd81\uad00\uacc4 \ud68c\ubcf5\uc5d0 \ud658\uc601\uc744 \ub73b\uc744 \ud45c\ud588\ub2e4. \ub0b4\uc6a9\uc5d0 \ub300\ud574\uc11c\ub294 \uae30\ub300\uc640 \uc6b0\ub824\uac00 \uad50\ucc28\ud588\ub2e4. '\uc644\uc804\ud55c \ube44\ud575\ud654'\uc640 '\ud575 \uc5c6\ub294 \ud55c\ubc18\ub3c4'\ub77c\ub294 \uc9c0\ud5a5\uc810\uc744 \ubc1d\ud788\uace0 \ubd81\ud55c\uc774 \uc2e4\uc9c8\uc801\uc778 \ube44\ud575\ud654\uac00 \uc544\ub2cc \ud575\ub3d9\uacb0\uc5d0 \uba38\ubb34\ub974\ub294 \uac83 \uc544\ub2c8\ub0d0\ub294 \uc758\uad6c\uc2ec\uc744 \uc5b4\ub290 \uc815\ub3c4 \uac77\uc5b4\ub0b4\ub294 \uac83\uc740 \uc758\ubbf8\uac00 \uc788\uc9c0\ub9cc \ube44\ud575\ud654\uac00 \uc120\uc5b8\ubb38\uc758 \ub9c8\uc9c0\ub9c9\uc5d0 \uc704\uce58\ud558\uace0 \ub2e4\ub978 \ud569\uc758\uc0ac\ud56d\uc5d0 \ube44\ud574 \uc2dc\ud55c\uc744 \ubabb\ubc15\uc9c0\ub3c4 \ubabb\ud558\uace0 \ub290\uc2a8\ud55c \ud45c\ud604\uc73c\ub85c \ub2f4\uae34 \uc810\uc5d0 \ub300\ud574\uc11c\ub294 \uc6b0\ub824\ub3c4 \uc874\uc7ac\ud588\ub2e4. \ubd81\ud55c\ub3c4 \uadf8\uac04 \uc778\uc815\ud558\uc9c0 \uc54a\ub358 \ubd81\ubc29\ud55c\uacc4\uc120\uc774\ub780 \ud45c\ud604\ub3c4 \uadf8\ub300\ub85c \uc0ac\uc6a9\ud558\uba74\uc11c \uc870\uc120\uc911\uc559\ud1b5\uc2e0\uc0ac\uac00 \ud310\ubb38\uc810 \uc120\uc5b8 \uc804\ubb38\uc744 \uac8c\uc7ac\ud588\ub2e4. \uc815\uc0c1\ud68c\ub2f4\uacfc \uad00\ub828\ud55c \uae30\uc0ac\ub3c4 \uc790\uc138\ud558\uac8c \ubcf4\ub3c4\ud588\ub294\ub370 \ubd81\ud55c\uc774 \uacf5\uc2dd\ub9e4\uccb4 \ubcf4\ub3c4\ub97c \ud1b5\ud574 \ube44\ud575\ud654 \uc758\uc9c0\ub97c \uacf5\uc2dd\ud654\ud55c \uac83\uc740 \ub9e4\uc6b0 \uc774\ub840\uc801\uc778 \uac83\uc774\uc5b4\uc11c \uc8fc\ubaa9\uc744 \ubc1b\uc558\ub2e4. \ub610\ud55c \uccad\uc640\ub300\ub294 \uc815\uad8c\uc774 \ubc14\ub00c\uac70\ub098 \uc815\uce58\uc801 \uc0c1\ud669\uc774 \ubcc0\ud654\ub418\ub354\ub77c\ub3c4 \ud310\ubb38\uc810 \uc120\uc5b8\uc758 \ub0b4\uc6a9\uc774 \uc9c0\uc18d\ub420 \uc218 \uc788\ub3c4\ub85d \uad6d\ud68c\uc758 \ube44\uc900\uc744 \ubc1b\uace0\uc790 \ud588\ub2e4. \ud558\uc9c0\ub9cc \uc5ec\uc57c\ub294 \uc9c0\uc9c0 \uacb0\uc758\uc548 \ud45c\ud604\uacfc \ubb38\uad6c\uc5d0 \ub300\ud55c \uc774\uacac\uc744 \uc881\ud788\uc9c0 \ubabb\ud574 5\uc6d4 28\uc77c \uad6d\ud68c \ube44\uc900\uc5d0 \uc2e4\ud328\ud588\ub2e4.", "answer": "\uad6d\ud68c\uc758 \ube44\uc900", "sentence": "\ub610\ud55c \uccad\uc640\ub300\ub294 \uc815\uad8c\uc774 \ubc14\ub00c\uac70\ub098 \uc815\uce58\uc801 \uc0c1\ud669\uc774 \ubcc0\ud654\ub418\ub354\ub77c\ub3c4 \ud310\ubb38\uc810 \uc120\uc5b8\uc758 \ub0b4\uc6a9\uc774 \uc9c0\uc18d\ub420 \uc218 \uc788\ub3c4\ub85d \uad6d\ud68c\uc758 \ube44\uc900 \uc744 \ubc1b\uace0\uc790 \ud588\ub2e4.", "paragraph_sentence": "\ud310\ubb38\uc810 \uc120\uc5b8\uc740 \ubd84\ub2e8\uacfc \ub300\uacb0\uc758 \uc885\uc2dd, \ubbfc\uc871 \ud654\ud574\uc640 \ud3c9\ud654 \ubc88\uc601\uc758 \uc0c8 \uc2dc\ub300, \ub0a8\ubd81\uad00\uacc4\uc758 \uac1c\uc120\uacfc \ubc1c\uc804\uc774\ub77c\ub294 \ud655\uace0\ud55c \uc758\uc9c0\ub97c \ub2f4\uc558\uc73c\uba70 \ud569\uc758 \ub0b4\uc6a9\uc5d0 \ub300\ud574 \ubb38\uc7ac\uc778\uc740 \"\uacb0\ucf54 \ub4a4\ub3cc\uc544\uac00\uc9c0 \uc54a\uc744 \uac83\"\uc774\uace0 \uae40\uc815\uc740\uc740 \"\uae34\ubc00\ud788 \ud611\ub825\ud574 \ubc18\ub4dc\uc2dc \uc88b\uc740 \uacb0\uc2e4\uc774 \ub9fa\uc5b4\uc9c0\ub3c4\ub85d \ub178\ub825\ud574\ub098\uac08 \uac83\"\uc774\ub77c\uace0 \uac15\uc870\ud588\ub2e4. \uc138\ubd80\uc801\uc73c\ub85c \ub2e8\uacc4\uc801 \uad70\ucd95, \uc801\ub300\uc801 \ud589\uc704 \uc911\uc9c0, \ubd81\ubc29\ud55c\uacc4\uc120\uc744 \ud3c9\ud654\uc218\uc5ed\ud654 \ub4f1\uc744 \ud569\uc758\ud588\ub2e4. \ub2e4\ub9cc, \uc7a5\uae30 \uacfc\uc81c\ub85c\uc11c \ud1b5\uc77c\uacfc \uad00\ub828\ud558\uc5ec \uad6c\uccb4\uc801\uc778 \ub300\ud654\ub294 \uc774\ub8e8\uc5b4\uc9c0\uc9c0 \uc54a\uc558\ub2e4. \uac01\uad6d\uc758 \uc9c0\uc9c0\ub3c4 \uc787\ub530\ub790\ub2e4. \ub3c4\ub110\ub4dc \ud2b8\ub7fc\ud504 \ubbf8\uad6d \ub300\ud1b5\ub839\uc740 \"\uc88b\uc740 \uc77c\ub4e4\uc774 \uc77c\uc5b4\ub098\uace0 \uc788\ub2e4. \uadf8\ub7ec\ub098 \uc624\uc9c1 \uc2dc\uac04\uc774 \ub9d0\ud574\uc904 \uac83\"\uc774\ub77c\uba70 \ud658\uc601\uc774 \ub73b\uc744 \ubc1d\ud614\ub2e4. \uc911\ud654\uc778\ubbfc\uacf5\ud654\uad6d \uc678\uad50\ubd80\ub294 \ub0a8\ubd81\uacfc \uc9c0\uc5ed\uc0ac\ud68c \ubc0f \uad6d\uc81c\uc0ac\ud68c\uc758 \uacf5\ud1b5\ub41c \uae30\ub300\uc5d0 \ubd80\ud569\ud55c\ub2e4\uace0 \uc9c0\uc9c0 \uc758\uc0ac\ub97c \ubcf4\ub0c8\uc73c\uba70 \ub7ec\uc2dc\uc544\ub3c4 \"\ud68c\ub2f4 \uacb0\uacfc\ub97c \uc544\uc8fc \uae0d\uc815\uc801\uc73c\ub85c \ud3c9\uac00\ud55c\ub2e4\"\uace0 \uac15\uc870\ud588\ub2e4. \uc77c\ubcf8 \uc5ed\uc2dc \"\ud55c\uad6d \uc815\ubd80\uc758 \ub178\ub825\uc744 \uce6d\ucc2c\ud558\uace0 \uc2f6\ub2e4\"\uace0 \uae0d\uc815\uc801\uc778 \ud3c9\uac00\ub97c \ubcf4\uc600\uc9c0\ub9cc \"\uc55e\uc73c\ub85c \ubd81\ud55c\uc758 \ub3d9\ud5a5\uc744 \uc8fc\uc2dc\ud560 \uac83\"\uc774\ub77c\uba70 \uacbd\uacc4\ub97c \ubcf4\uc774\uae30\ub3c4 \ud588\ub2e4. \uc601\uad6d, \ud504\ub791\uc2a4, \uc774\ud0c8\ub9ac\uc544\ub294 \ubb3c\ub860 \uc911\ub9bd\uad6d\uac10\ub3c5\uc704\uc6d0\ud68c\uc758 \uc77c\uc6d0\uc778 \uc2a4\uc6e8\ub374\uacfc \uc2a4\uc704\uc2a4\ub3c4 \ub0a8\ubd81\uad00\uacc4 \ud68c\ubcf5\uc5d0 \ud658\uc601\uc744 \ub73b\uc744 \ud45c\ud588\ub2e4. \ub0b4\uc6a9\uc5d0 \ub300\ud574\uc11c\ub294 \uae30\ub300\uc640 \uc6b0\ub824\uac00 \uad50\ucc28\ud588\ub2e4. '\uc644\uc804\ud55c \ube44\ud575\ud654'\uc640 '\ud575 \uc5c6\ub294 \ud55c\ubc18\ub3c4'\ub77c\ub294 \uc9c0\ud5a5\uc810\uc744 \ubc1d\ud788\uace0 \ubd81\ud55c\uc774 \uc2e4\uc9c8\uc801\uc778 \ube44\ud575\ud654\uac00 \uc544\ub2cc \ud575\ub3d9\uacb0\uc5d0 \uba38\ubb34\ub974\ub294 \uac83 \uc544\ub2c8\ub0d0\ub294 \uc758\uad6c\uc2ec\uc744 \uc5b4\ub290 \uc815\ub3c4 \uac77\uc5b4\ub0b4\ub294 \uac83\uc740 \uc758\ubbf8\uac00 \uc788\uc9c0\ub9cc \ube44\ud575\ud654\uac00 \uc120\uc5b8\ubb38\uc758 \ub9c8\uc9c0\ub9c9\uc5d0 \uc704\uce58\ud558\uace0 \ub2e4\ub978 \ud569\uc758\uc0ac\ud56d\uc5d0 \ube44\ud574 \uc2dc\ud55c\uc744 \ubabb\ubc15\uc9c0\ub3c4 \ubabb\ud558\uace0 \ub290\uc2a8\ud55c \ud45c\ud604\uc73c\ub85c \ub2f4\uae34 \uc810\uc5d0 \ub300\ud574\uc11c\ub294 \uc6b0\ub824\ub3c4 \uc874\uc7ac\ud588\ub2e4. \ubd81\ud55c\ub3c4 \uadf8\uac04 \uc778\uc815\ud558\uc9c0 \uc54a\ub358 \ubd81\ubc29\ud55c\uacc4\uc120\uc774\ub780 \ud45c\ud604\ub3c4 \uadf8\ub300\ub85c \uc0ac\uc6a9\ud558\uba74\uc11c \uc870\uc120\uc911\uc559\ud1b5\uc2e0\uc0ac\uac00 \ud310\ubb38\uc810 \uc120\uc5b8 \uc804\ubb38\uc744 \uac8c\uc7ac\ud588\ub2e4. \uc815\uc0c1\ud68c\ub2f4\uacfc \uad00\ub828\ud55c \uae30\uc0ac\ub3c4 \uc790\uc138\ud558\uac8c \ubcf4\ub3c4\ud588\ub294\ub370 \ubd81\ud55c\uc774 \uacf5\uc2dd\ub9e4\uccb4 \ubcf4\ub3c4\ub97c \ud1b5\ud574 \ube44\ud575\ud654 \uc758\uc9c0\ub97c \uacf5\uc2dd\ud654\ud55c \uac83\uc740 \ub9e4\uc6b0 \uc774\ub840\uc801\uc778 \uac83\uc774\uc5b4\uc11c \uc8fc\ubaa9\uc744 \ubc1b\uc558\ub2e4. \ub610\ud55c \uccad\uc640\ub300\ub294 \uc815\uad8c\uc774 \ubc14\ub00c\uac70\ub098 \uc815\uce58\uc801 \uc0c1\ud669\uc774 \ubcc0\ud654\ub418\ub354\ub77c\ub3c4 \ud310\ubb38\uc810 \uc120\uc5b8\uc758 \ub0b4\uc6a9\uc774 \uc9c0\uc18d\ub420 \uc218 \uc788\ub3c4\ub85d \uad6d\ud68c\uc758 \ube44\uc900 \uc744 \ubc1b\uace0\uc790 \ud588\ub2e4. \ud558\uc9c0\ub9cc \uc5ec\uc57c\ub294 \uc9c0\uc9c0 \uacb0\uc758\uc548 \ud45c\ud604\uacfc \ubb38\uad6c\uc5d0 \ub300\ud55c \uc774\uacac\uc744 \uc881\ud788\uc9c0 \ubabb\ud574 5\uc6d4 28\uc77c \uad6d\ud68c \ube44\uc900\uc5d0 \uc2e4\ud328\ud588\ub2e4.", "paragraph_answer": "\ud310\ubb38\uc810 \uc120\uc5b8\uc740 \ubd84\ub2e8\uacfc \ub300\uacb0\uc758 \uc885\uc2dd, \ubbfc\uc871 \ud654\ud574\uc640 \ud3c9\ud654 \ubc88\uc601\uc758 \uc0c8 \uc2dc\ub300, \ub0a8\ubd81\uad00\uacc4\uc758 \uac1c\uc120\uacfc \ubc1c\uc804\uc774\ub77c\ub294 \ud655\uace0\ud55c \uc758\uc9c0\ub97c \ub2f4\uc558\uc73c\uba70 \ud569\uc758 \ub0b4\uc6a9\uc5d0 \ub300\ud574 \ubb38\uc7ac\uc778\uc740 \"\uacb0\ucf54 \ub4a4\ub3cc\uc544\uac00\uc9c0 \uc54a\uc744 \uac83\"\uc774\uace0 \uae40\uc815\uc740\uc740 \"\uae34\ubc00\ud788 \ud611\ub825\ud574 \ubc18\ub4dc\uc2dc \uc88b\uc740 \uacb0\uc2e4\uc774 \ub9fa\uc5b4\uc9c0\ub3c4\ub85d \ub178\ub825\ud574\ub098\uac08 \uac83\"\uc774\ub77c\uace0 \uac15\uc870\ud588\ub2e4. \uc138\ubd80\uc801\uc73c\ub85c \ub2e8\uacc4\uc801 \uad70\ucd95, \uc801\ub300\uc801 \ud589\uc704 \uc911\uc9c0, \ubd81\ubc29\ud55c\uacc4\uc120\uc744 \ud3c9\ud654\uc218\uc5ed\ud654 \ub4f1\uc744 \ud569\uc758\ud588\ub2e4. \ub2e4\ub9cc, \uc7a5\uae30 \uacfc\uc81c\ub85c\uc11c \ud1b5\uc77c\uacfc \uad00\ub828\ud558\uc5ec \uad6c\uccb4\uc801\uc778 \ub300\ud654\ub294 \uc774\ub8e8\uc5b4\uc9c0\uc9c0 \uc54a\uc558\ub2e4. \uac01\uad6d\uc758 \uc9c0\uc9c0\ub3c4 \uc787\ub530\ub790\ub2e4. \ub3c4\ub110\ub4dc \ud2b8\ub7fc\ud504 \ubbf8\uad6d \ub300\ud1b5\ub839\uc740 \"\uc88b\uc740 \uc77c\ub4e4\uc774 \uc77c\uc5b4\ub098\uace0 \uc788\ub2e4. \uadf8\ub7ec\ub098 \uc624\uc9c1 \uc2dc\uac04\uc774 \ub9d0\ud574\uc904 \uac83\"\uc774\ub77c\uba70 \ud658\uc601\uc774 \ub73b\uc744 \ubc1d\ud614\ub2e4. \uc911\ud654\uc778\ubbfc\uacf5\ud654\uad6d \uc678\uad50\ubd80\ub294 \ub0a8\ubd81\uacfc \uc9c0\uc5ed\uc0ac\ud68c \ubc0f \uad6d\uc81c\uc0ac\ud68c\uc758 \uacf5\ud1b5\ub41c \uae30\ub300\uc5d0 \ubd80\ud569\ud55c\ub2e4\uace0 \uc9c0\uc9c0 \uc758\uc0ac\ub97c \ubcf4\ub0c8\uc73c\uba70 \ub7ec\uc2dc\uc544\ub3c4 \"\ud68c\ub2f4 \uacb0\uacfc\ub97c \uc544\uc8fc \uae0d\uc815\uc801\uc73c\ub85c \ud3c9\uac00\ud55c\ub2e4\"\uace0 \uac15\uc870\ud588\ub2e4. \uc77c\ubcf8 \uc5ed\uc2dc \"\ud55c\uad6d \uc815\ubd80\uc758 \ub178\ub825\uc744 \uce6d\ucc2c\ud558\uace0 \uc2f6\ub2e4\"\uace0 \uae0d\uc815\uc801\uc778 \ud3c9\uac00\ub97c \ubcf4\uc600\uc9c0\ub9cc \"\uc55e\uc73c\ub85c \ubd81\ud55c\uc758 \ub3d9\ud5a5\uc744 \uc8fc\uc2dc\ud560 \uac83\"\uc774\ub77c\uba70 \uacbd\uacc4\ub97c \ubcf4\uc774\uae30\ub3c4 \ud588\ub2e4. \uc601\uad6d, \ud504\ub791\uc2a4, \uc774\ud0c8\ub9ac\uc544\ub294 \ubb3c\ub860 \uc911\ub9bd\uad6d\uac10\ub3c5\uc704\uc6d0\ud68c\uc758 \uc77c\uc6d0\uc778 \uc2a4\uc6e8\ub374\uacfc \uc2a4\uc704\uc2a4\ub3c4 \ub0a8\ubd81\uad00\uacc4 \ud68c\ubcf5\uc5d0 \ud658\uc601\uc744 \ub73b\uc744 \ud45c\ud588\ub2e4. \ub0b4\uc6a9\uc5d0 \ub300\ud574\uc11c\ub294 \uae30\ub300\uc640 \uc6b0\ub824\uac00 \uad50\ucc28\ud588\ub2e4. '\uc644\uc804\ud55c \ube44\ud575\ud654'\uc640 '\ud575 \uc5c6\ub294 \ud55c\ubc18\ub3c4'\ub77c\ub294 \uc9c0\ud5a5\uc810\uc744 \ubc1d\ud788\uace0 \ubd81\ud55c\uc774 \uc2e4\uc9c8\uc801\uc778 \ube44\ud575\ud654\uac00 \uc544\ub2cc \ud575\ub3d9\uacb0\uc5d0 \uba38\ubb34\ub974\ub294 \uac83 \uc544\ub2c8\ub0d0\ub294 \uc758\uad6c\uc2ec\uc744 \uc5b4\ub290 \uc815\ub3c4 \uac77\uc5b4\ub0b4\ub294 \uac83\uc740 \uc758\ubbf8\uac00 \uc788\uc9c0\ub9cc \ube44\ud575\ud654\uac00 \uc120\uc5b8\ubb38\uc758 \ub9c8\uc9c0\ub9c9\uc5d0 \uc704\uce58\ud558\uace0 \ub2e4\ub978 \ud569\uc758\uc0ac\ud56d\uc5d0 \ube44\ud574 \uc2dc\ud55c\uc744 \ubabb\ubc15\uc9c0\ub3c4 \ubabb\ud558\uace0 \ub290\uc2a8\ud55c \ud45c\ud604\uc73c\ub85c \ub2f4\uae34 \uc810\uc5d0 \ub300\ud574\uc11c\ub294 \uc6b0\ub824\ub3c4 \uc874\uc7ac\ud588\ub2e4. \ubd81\ud55c\ub3c4 \uadf8\uac04 \uc778\uc815\ud558\uc9c0 \uc54a\ub358 \ubd81\ubc29\ud55c\uacc4\uc120\uc774\ub780 \ud45c\ud604\ub3c4 \uadf8\ub300\ub85c \uc0ac\uc6a9\ud558\uba74\uc11c \uc870\uc120\uc911\uc559\ud1b5\uc2e0\uc0ac\uac00 \ud310\ubb38\uc810 \uc120\uc5b8 \uc804\ubb38\uc744 \uac8c\uc7ac\ud588\ub2e4. \uc815\uc0c1\ud68c\ub2f4\uacfc \uad00\ub828\ud55c \uae30\uc0ac\ub3c4 \uc790\uc138\ud558\uac8c \ubcf4\ub3c4\ud588\ub294\ub370 \ubd81\ud55c\uc774 \uacf5\uc2dd\ub9e4\uccb4 \ubcf4\ub3c4\ub97c \ud1b5\ud574 \ube44\ud575\ud654 \uc758\uc9c0\ub97c \uacf5\uc2dd\ud654\ud55c \uac83\uc740 \ub9e4\uc6b0 \uc774\ub840\uc801\uc778 \uac83\uc774\uc5b4\uc11c \uc8fc\ubaa9\uc744 \ubc1b\uc558\ub2e4. \ub610\ud55c \uccad\uc640\ub300\ub294 \uc815\uad8c\uc774 \ubc14\ub00c\uac70\ub098 \uc815\uce58\uc801 \uc0c1\ud669\uc774 \ubcc0\ud654\ub418\ub354\ub77c\ub3c4 \ud310\ubb38\uc810 \uc120\uc5b8\uc758 \ub0b4\uc6a9\uc774 \uc9c0\uc18d\ub420 \uc218 \uc788\ub3c4\ub85d \uad6d\ud68c\uc758 \ube44\uc900 \uc744 \ubc1b\uace0\uc790 \ud588\ub2e4. \ud558\uc9c0\ub9cc \uc5ec\uc57c\ub294 \uc9c0\uc9c0 \uacb0\uc758\uc548 \ud45c\ud604\uacfc \ubb38\uad6c\uc5d0 \ub300\ud55c \uc774\uacac\uc744 \uc881\ud788\uc9c0 \ubabb\ud574 5\uc6d4 28\uc77c \uad6d\ud68c \ube44\uc900\uc5d0 \uc2e4\ud328\ud588\ub2e4.", "sentence_answer": "\ub610\ud55c \uccad\uc640\ub300\ub294 \uc815\uad8c\uc774 \ubc14\ub00c\uac70\ub098 \uc815\uce58\uc801 \uc0c1\ud669\uc774 \ubcc0\ud654\ub418\ub354\ub77c\ub3c4 \ud310\ubb38\uc810 \uc120\uc5b8\uc758 \ub0b4\uc6a9\uc774 \uc9c0\uc18d\ub420 \uc218 \uc788\ub3c4\ub85d \uad6d\ud68c\uc758 \ube44\uc900 \uc744 \ubc1b\uace0\uc790 \ud588\ub2e4.", "paragraph_id": "6580792-50-1", "paragraph_question": "question: \uccad\uc640\ub300\ub294 \uc815\uad8c\uc774 \ubc14\ub00c\uac70\ub098 \uc815\uce58\uc801 \uc0c1\ud669\uc774 \ubcc0\ud654\ub418\ub354\ub77c\ub3c4 \ud310\ubb38\uc810 \uc120\uc5b8\uc758 \ub0b4\uc6a9\uc774 \uc9c0\uc18d\ub420 \uc218 \uc788\ub3c4\ub85d \ubc1b\uace0\uc790 \ud55c \uac83\uc740?, context: \ud310\ubb38\uc810 \uc120\uc5b8\uc740 \ubd84\ub2e8\uacfc \ub300\uacb0\uc758 \uc885\uc2dd, \ubbfc\uc871 \ud654\ud574\uc640 \ud3c9\ud654 \ubc88\uc601\uc758 \uc0c8 \uc2dc\ub300, \ub0a8\ubd81\uad00\uacc4\uc758 \uac1c\uc120\uacfc \ubc1c\uc804\uc774\ub77c\ub294 \ud655\uace0\ud55c \uc758\uc9c0\ub97c \ub2f4\uc558\uc73c\uba70 \ud569\uc758 \ub0b4\uc6a9\uc5d0 \ub300\ud574 \ubb38\uc7ac\uc778\uc740 \"\uacb0\ucf54 \ub4a4\ub3cc\uc544\uac00\uc9c0 \uc54a\uc744 \uac83\"\uc774\uace0 \uae40\uc815\uc740\uc740 \"\uae34\ubc00\ud788 \ud611\ub825\ud574 \ubc18\ub4dc\uc2dc \uc88b\uc740 \uacb0\uc2e4\uc774 \ub9fa\uc5b4\uc9c0\ub3c4\ub85d \ub178\ub825\ud574\ub098\uac08 \uac83\"\uc774\ub77c\uace0 \uac15\uc870\ud588\ub2e4. \uc138\ubd80\uc801\uc73c\ub85c \ub2e8\uacc4\uc801 \uad70\ucd95, \uc801\ub300\uc801 \ud589\uc704 \uc911\uc9c0, \ubd81\ubc29\ud55c\uacc4\uc120\uc744 \ud3c9\ud654\uc218\uc5ed\ud654 \ub4f1\uc744 \ud569\uc758\ud588\ub2e4. \ub2e4\ub9cc, \uc7a5\uae30 \uacfc\uc81c\ub85c\uc11c \ud1b5\uc77c\uacfc \uad00\ub828\ud558\uc5ec \uad6c\uccb4\uc801\uc778 \ub300\ud654\ub294 \uc774\ub8e8\uc5b4\uc9c0\uc9c0 \uc54a\uc558\ub2e4. \uac01\uad6d\uc758 \uc9c0\uc9c0\ub3c4 \uc787\ub530\ub790\ub2e4. \ub3c4\ub110\ub4dc \ud2b8\ub7fc\ud504 \ubbf8\uad6d \ub300\ud1b5\ub839\uc740 \"\uc88b\uc740 \uc77c\ub4e4\uc774 \uc77c\uc5b4\ub098\uace0 \uc788\ub2e4. \uadf8\ub7ec\ub098 \uc624\uc9c1 \uc2dc\uac04\uc774 \ub9d0\ud574\uc904 \uac83\"\uc774\ub77c\uba70 \ud658\uc601\uc774 \ub73b\uc744 \ubc1d\ud614\ub2e4. \uc911\ud654\uc778\ubbfc\uacf5\ud654\uad6d \uc678\uad50\ubd80\ub294 \ub0a8\ubd81\uacfc \uc9c0\uc5ed\uc0ac\ud68c \ubc0f \uad6d\uc81c\uc0ac\ud68c\uc758 \uacf5\ud1b5\ub41c \uae30\ub300\uc5d0 \ubd80\ud569\ud55c\ub2e4\uace0 \uc9c0\uc9c0 \uc758\uc0ac\ub97c \ubcf4\ub0c8\uc73c\uba70 \ub7ec\uc2dc\uc544\ub3c4 \"\ud68c\ub2f4 \uacb0\uacfc\ub97c \uc544\uc8fc \uae0d\uc815\uc801\uc73c\ub85c \ud3c9\uac00\ud55c\ub2e4\"\uace0 \uac15\uc870\ud588\ub2e4. \uc77c\ubcf8 \uc5ed\uc2dc \"\ud55c\uad6d \uc815\ubd80\uc758 \ub178\ub825\uc744 \uce6d\ucc2c\ud558\uace0 \uc2f6\ub2e4\"\uace0 \uae0d\uc815\uc801\uc778 \ud3c9\uac00\ub97c \ubcf4\uc600\uc9c0\ub9cc \"\uc55e\uc73c\ub85c \ubd81\ud55c\uc758 \ub3d9\ud5a5\uc744 \uc8fc\uc2dc\ud560 \uac83\"\uc774\ub77c\uba70 \uacbd\uacc4\ub97c \ubcf4\uc774\uae30\ub3c4 \ud588\ub2e4. \uc601\uad6d, \ud504\ub791\uc2a4, \uc774\ud0c8\ub9ac\uc544\ub294 \ubb3c\ub860 \uc911\ub9bd\uad6d\uac10\ub3c5\uc704\uc6d0\ud68c\uc758 \uc77c\uc6d0\uc778 \uc2a4\uc6e8\ub374\uacfc \uc2a4\uc704\uc2a4\ub3c4 \ub0a8\ubd81\uad00\uacc4 \ud68c\ubcf5\uc5d0 \ud658\uc601\uc744 \ub73b\uc744 \ud45c\ud588\ub2e4. \ub0b4\uc6a9\uc5d0 \ub300\ud574\uc11c\ub294 \uae30\ub300\uc640 \uc6b0\ub824\uac00 \uad50\ucc28\ud588\ub2e4. '\uc644\uc804\ud55c \ube44\ud575\ud654'\uc640 '\ud575 \uc5c6\ub294 \ud55c\ubc18\ub3c4'\ub77c\ub294 \uc9c0\ud5a5\uc810\uc744 \ubc1d\ud788\uace0 \ubd81\ud55c\uc774 \uc2e4\uc9c8\uc801\uc778 \ube44\ud575\ud654\uac00 \uc544\ub2cc \ud575\ub3d9\uacb0\uc5d0 \uba38\ubb34\ub974\ub294 \uac83 \uc544\ub2c8\ub0d0\ub294 \uc758\uad6c\uc2ec\uc744 \uc5b4\ub290 \uc815\ub3c4 \uac77\uc5b4\ub0b4\ub294 \uac83\uc740 \uc758\ubbf8\uac00 \uc788\uc9c0\ub9cc \ube44\ud575\ud654\uac00 \uc120\uc5b8\ubb38\uc758 \ub9c8\uc9c0\ub9c9\uc5d0 \uc704\uce58\ud558\uace0 \ub2e4\ub978 \ud569\uc758\uc0ac\ud56d\uc5d0 \ube44\ud574 \uc2dc\ud55c\uc744 \ubabb\ubc15\uc9c0\ub3c4 \ubabb\ud558\uace0 \ub290\uc2a8\ud55c \ud45c\ud604\uc73c\ub85c \ub2f4\uae34 \uc810\uc5d0 \ub300\ud574\uc11c\ub294 \uc6b0\ub824\ub3c4 \uc874\uc7ac\ud588\ub2e4. \ubd81\ud55c\ub3c4 \uadf8\uac04 \uc778\uc815\ud558\uc9c0 \uc54a\ub358 \ubd81\ubc29\ud55c\uacc4\uc120\uc774\ub780 \ud45c\ud604\ub3c4 \uadf8\ub300\ub85c \uc0ac\uc6a9\ud558\uba74\uc11c \uc870\uc120\uc911\uc559\ud1b5\uc2e0\uc0ac\uac00 \ud310\ubb38\uc810 \uc120\uc5b8 \uc804\ubb38\uc744 \uac8c\uc7ac\ud588\ub2e4. \uc815\uc0c1\ud68c\ub2f4\uacfc \uad00\ub828\ud55c \uae30\uc0ac\ub3c4 \uc790\uc138\ud558\uac8c \ubcf4\ub3c4\ud588\ub294\ub370 \ubd81\ud55c\uc774 \uacf5\uc2dd\ub9e4\uccb4 \ubcf4\ub3c4\ub97c \ud1b5\ud574 \ube44\ud575\ud654 \uc758\uc9c0\ub97c \uacf5\uc2dd\ud654\ud55c \uac83\uc740 \ub9e4\uc6b0 \uc774\ub840\uc801\uc778 \uac83\uc774\uc5b4\uc11c \uc8fc\ubaa9\uc744 \ubc1b\uc558\ub2e4. \ub610\ud55c \uccad\uc640\ub300\ub294 \uc815\uad8c\uc774 \ubc14\ub00c\uac70\ub098 \uc815\uce58\uc801 \uc0c1\ud669\uc774 \ubcc0\ud654\ub418\ub354\ub77c\ub3c4 \ud310\ubb38\uc810 \uc120\uc5b8\uc758 \ub0b4\uc6a9\uc774 \uc9c0\uc18d\ub420 \uc218 \uc788\ub3c4\ub85d \uad6d\ud68c\uc758 \ube44\uc900\uc744 \ubc1b\uace0\uc790 \ud588\ub2e4. \ud558\uc9c0\ub9cc \uc5ec\uc57c\ub294 \uc9c0\uc9c0 \uacb0\uc758\uc548 \ud45c\ud604\uacfc \ubb38\uad6c\uc5d0 \ub300\ud55c \uc774\uacac\uc744 \uc881\ud788\uc9c0 \ubabb\ud574 5\uc6d4 28\uc77c \uad6d\ud68c \ube44\uc900\uc5d0 \uc2e4\ud328\ud588\ub2e4."} {"question": "\uc720\ub7fd\ucc0c\ub974\ub808\uae30\uc758 \uc54c \ud488\uae30\uc758 \uae30\uac04\uc740 \ub300\ub7b5 \uba87\uc77c\uc778\uac00?", "paragraph": "\uc54c \ud488\uae30\ub294 13\uc77c \uc815\ub3c4 \uac78\ub9ac\uba70, \ub9c8\uc9c0\ub9c9\uc5d0 \ub0b3\uc740 \uc54c\uc774 \uac00\uc7a5 \uba3c\uc800 \uc54a\uc740 \uc54c\ubcf4\ub2e4 \uc57d 24\uc2dc\uac04 \ub2a6\uac8c \ubd80\ud654\ud55c\ub2e4. \uc554\uc218 \ubaa8\ub450 \uc54c\uc744 \ub3cc\ubcf4\uc9c0\ub9cc, \uc554\ucef7\uc774 \uc218\ucef7\ubcf4\ub2e4 \ub354 \uc624\ub798 \uc54c\uc744 \ud488\ub294\ub2e4. \ub610\ud55c \ubc24\uc774 \ub418\uba74 \uc218\ucef7\uc740 \ud683\ub300\ub85c \ub3cc\uc544\uac00 \ub2e4\ub978 \uc0c8\ub4e4\uacfc \uc5b4\uc6b8\ub9ac\uc9c0\ub9cc \uc554\ucef7\uc740 \uacc4\uc18d \ub465\uc9c0\uc5d0 \ub0a8\uc544 \uc54c\uc744 \ud488\ub294\ub2e4. \uac13 \uae68\uc5b4\ub09c \uc0c8\ub294 \ub208\ub3c4 \ub728\uc9c0 \ubabb\ud558\uace0 \ud138\ub3c4 \ud558\ub098 \uc5c6\ub2e4. \uc774\ud6c4 7\uc77c\uc5d0 \uac78\uccd0 \uc19c\ud138\uc774 \ub3cb\uc544\ub098\uba70, 9\uc77c\uc9f8\uac00 \ub418\uba74 \ub208\uc774 \ub728\uc5ec \uc55e\uc744 \ubcfc \uc218 \uc788\ub2e4. \ubd80\ud654 \ud6c4 \uc57d 6\uc77c\uc774 \uc9c0\ub098\uba74 \ubcd1\uc544\ub9ac\ub4e4\uc774 \uc790\uae30 \uccb4\uc628\uc744 \uc720\uc9c0\ud560 \uc218 \uc788\uac8c \ub41c\ub2e4. \uadf8\ub7ec\uba74 \ubd80\ubaa8 \uc0c8\ub4e4\uc740 \ubc30\uc124\ubb3c\uc744 \ub465\uc9c0 \ubc16\uc73c\ub85c \ubc84\ub9ac\uae30\ub97c \uadf8\ub9cc\ub454\ub2e4. \uadf8 \uc804\uae4c\uc9c0\ub294 \ubc30\uc124\ubb3c\uc774 \ub465\uc9c0\uc640 \ubcd1\uc544\ub9ac\uc758 \ud138\uc744 \uc801\uc154\uc11c \ub2e8\uc5f4\ub2a5\ub825\uc744 \uc800\ud574\ud558\uace0 \uac13 \ud0dc\uc5b4\ub09c \uc0c8\ub07c\ub97c \uc800\uccb4\uc628\uc99d\uc5d0 \uac78\ub9ac\uac8c \ud560 \uc218 \uc788\uae30\uc5d0 \ubd80\uc9c0\ub7f0\ud788 \ub465\uc9c0 \ubc16\uc73c\ub85c \ub0b4\ub2e4 \ubc84\ub9b0\ub2e4. \uc0c8\ub07c \uc0c8\ub294 3\uc8fc \uac00\ub7c9 \ub465\uc9c0\uc5d0 \uba38\ubb3c\uba74\uc11c \ubd80\ubaa8\uc5d0\uac8c \uacc4\uc18d \uba39\uc774\ub97c \ubc1b\uc544\uba39\ub294\ub2e4. \uae43\ud138\uacfc \uadfc\uc721\uc774 \ubc1c\ub2ec\ud558\uace0 \ub098\uc11c\ub3c4 1, 2\uc8fc \ub354 \ubd80\ubaa8\uc5d0\uac8c \uba39\uc774\ub97c \ubc1b\uc544\uba39\ub294\ub2e4. \ud55c \uc30d\uc758 \ubd80\ubd80\ub294 1\ub144\uc5d0 \uc138 \ubc88 \ubc88\uc2dd\uc744 \ud560 \uc218 \uc788\uc73c\uba70, \ub300\uac1c \uc4f0\ub358 \ub465\uc9c0\uc758 \ub0b4\ubd80\uc7ac\ub9cc \uc190\ubd10\uc11c \ub2e4\uc2dc \uc0ac\uc6a9\ud55c\ub2e4. \ud558\uc9c0\ub9cc \uc2e4\uc81c\ub85c \uc138 \ubc88\uc744 \ucc44\uc6b0\uae30\ubcf4\ub2e4\ub294 \ub450 \ubc88 \uc815\ub3c4\uac00 \ud754\ud558\uace0, \ubd81\uc704 48\ub3c4 \uc774\uc0c1\uc5d0\uc11c\ub294 1\ub144\uc5d0 \ud55c \ubc88 \ubc16\uc5d0 \uc0c8\ub07c\ub97c \uce58\uc9c0 \uc54a\ub294\ub2e4. 2\uac1c\uc6d4 \ub0b4\uc5d0 \uc0c8\ub07c\ub4e4\uc740 \ud138\uac08\uc774\ub97c \ud558\uace0 \uc19c\ud138 \ub300\uc2e0 \uccab \uae43\ud138\uc744 \uc5bb\uac8c \ub41c\ub2e4. \uc5b4\ub978 \uae43\ud138\uc774 \ub098\ub824\uba74 \uc774\ub4ec\ud574\uae4c\uc9c0 \ub354 \uae30\ub2e4\ub824\uc57c \ud55c\ub2e4. \ub2e4\ub978 \uc5f0\uc791\ub958\uc640 \ub9c8\ucc2c\uac00\uc9c0\ub85c \ub465\uc9c0\ub294 \uae68\ub057\ud558\uac8c \uccad\uc18c\ud558\uace0 \ubcd1\uc544\ub9ac\ub4e4\uc758 \ubc30\uc124\ub0ad\uc740 \ubd80\ubaa8\uac00 \uce58\uc6b4\ub2e4.", "answer": "13\uc77c", "sentence": "\uc54c \ud488\uae30\ub294 13\uc77c \uc815\ub3c4 \uac78\ub9ac\uba70, \ub9c8\uc9c0\ub9c9\uc5d0 \ub0b3\uc740 \uc54c\uc774 \uac00\uc7a5 \uba3c\uc800 \uc54a\uc740 \uc54c\ubcf4\ub2e4 \uc57d 24\uc2dc\uac04 \ub2a6\uac8c \ubd80\ud654\ud55c\ub2e4.", "paragraph_sentence": " \uc54c \ud488\uae30\ub294 13\uc77c \uc815\ub3c4 \uac78\ub9ac\uba70, \ub9c8\uc9c0\ub9c9\uc5d0 \ub0b3\uc740 \uc54c\uc774 \uac00\uc7a5 \uba3c\uc800 \uc54a\uc740 \uc54c\ubcf4\ub2e4 \uc57d 24\uc2dc\uac04 \ub2a6\uac8c \ubd80\ud654\ud55c\ub2e4. \uc554\uc218 \ubaa8\ub450 \uc54c\uc744 \ub3cc\ubcf4\uc9c0\ub9cc, \uc554\ucef7\uc774 \uc218\ucef7\ubcf4\ub2e4 \ub354 \uc624\ub798 \uc54c\uc744 \ud488\ub294\ub2e4. \ub610\ud55c \ubc24\uc774 \ub418\uba74 \uc218\ucef7\uc740 \ud683\ub300\ub85c \ub3cc\uc544\uac00 \ub2e4\ub978 \uc0c8\ub4e4\uacfc \uc5b4\uc6b8\ub9ac\uc9c0\ub9cc \uc554\ucef7\uc740 \uacc4\uc18d \ub465\uc9c0\uc5d0 \ub0a8\uc544 \uc54c\uc744 \ud488\ub294\ub2e4. \uac13 \uae68\uc5b4\ub09c \uc0c8\ub294 \ub208\ub3c4 \ub728\uc9c0 \ubabb\ud558\uace0 \ud138\ub3c4 \ud558\ub098 \uc5c6\ub2e4. \uc774\ud6c4 7\uc77c\uc5d0 \uac78\uccd0 \uc19c\ud138\uc774 \ub3cb\uc544\ub098\uba70, 9\uc77c\uc9f8\uac00 \ub418\uba74 \ub208\uc774 \ub728\uc5ec \uc55e\uc744 \ubcfc \uc218 \uc788\ub2e4. \ubd80\ud654 \ud6c4 \uc57d 6\uc77c\uc774 \uc9c0\ub098\uba74 \ubcd1\uc544\ub9ac\ub4e4\uc774 \uc790\uae30 \uccb4\uc628\uc744 \uc720\uc9c0\ud560 \uc218 \uc788\uac8c \ub41c\ub2e4. \uadf8\ub7ec\uba74 \ubd80\ubaa8 \uc0c8\ub4e4\uc740 \ubc30\uc124\ubb3c\uc744 \ub465\uc9c0 \ubc16\uc73c\ub85c \ubc84\ub9ac\uae30\ub97c \uadf8\ub9cc\ub454\ub2e4. \uadf8 \uc804\uae4c\uc9c0\ub294 \ubc30\uc124\ubb3c\uc774 \ub465\uc9c0\uc640 \ubcd1\uc544\ub9ac\uc758 \ud138\uc744 \uc801\uc154\uc11c \ub2e8\uc5f4\ub2a5\ub825\uc744 \uc800\ud574\ud558\uace0 \uac13 \ud0dc\uc5b4\ub09c \uc0c8\ub07c\ub97c \uc800\uccb4\uc628\uc99d\uc5d0 \uac78\ub9ac\uac8c \ud560 \uc218 \uc788\uae30\uc5d0 \ubd80\uc9c0\ub7f0\ud788 \ub465\uc9c0 \ubc16\uc73c\ub85c \ub0b4\ub2e4 \ubc84\ub9b0\ub2e4. \uc0c8\ub07c \uc0c8\ub294 3\uc8fc \uac00\ub7c9 \ub465\uc9c0\uc5d0 \uba38\ubb3c\uba74\uc11c \ubd80\ubaa8\uc5d0\uac8c \uacc4\uc18d \uba39\uc774\ub97c \ubc1b\uc544\uba39\ub294\ub2e4. \uae43\ud138\uacfc \uadfc\uc721\uc774 \ubc1c\ub2ec\ud558\uace0 \ub098\uc11c\ub3c4 1, 2\uc8fc \ub354 \ubd80\ubaa8\uc5d0\uac8c \uba39\uc774\ub97c \ubc1b\uc544\uba39\ub294\ub2e4. \ud55c \uc30d\uc758 \ubd80\ubd80\ub294 1\ub144\uc5d0 \uc138 \ubc88 \ubc88\uc2dd\uc744 \ud560 \uc218 \uc788\uc73c\uba70, \ub300\uac1c \uc4f0\ub358 \ub465\uc9c0\uc758 \ub0b4\ubd80\uc7ac\ub9cc \uc190\ubd10\uc11c \ub2e4\uc2dc \uc0ac\uc6a9\ud55c\ub2e4. \ud558\uc9c0\ub9cc \uc2e4\uc81c\ub85c \uc138 \ubc88\uc744 \ucc44\uc6b0\uae30\ubcf4\ub2e4\ub294 \ub450 \ubc88 \uc815\ub3c4\uac00 \ud754\ud558\uace0, \ubd81\uc704 48\ub3c4 \uc774\uc0c1\uc5d0\uc11c\ub294 1\ub144\uc5d0 \ud55c \ubc88 \ubc16\uc5d0 \uc0c8\ub07c\ub97c \uce58\uc9c0 \uc54a\ub294\ub2e4. 2\uac1c\uc6d4 \ub0b4\uc5d0 \uc0c8\ub07c\ub4e4\uc740 \ud138\uac08\uc774\ub97c \ud558\uace0 \uc19c\ud138 \ub300\uc2e0 \uccab \uae43\ud138\uc744 \uc5bb\uac8c \ub41c\ub2e4. \uc5b4\ub978 \uae43\ud138\uc774 \ub098\ub824\uba74 \uc774\ub4ec\ud574\uae4c\uc9c0 \ub354 \uae30\ub2e4\ub824\uc57c \ud55c\ub2e4. \ub2e4\ub978 \uc5f0\uc791\ub958\uc640 \ub9c8\ucc2c\uac00\uc9c0\ub85c \ub465\uc9c0\ub294 \uae68\ub057\ud558\uac8c \uccad\uc18c\ud558\uace0 \ubcd1\uc544\ub9ac\ub4e4\uc758 \ubc30\uc124\ub0ad\uc740 \ubd80\ubaa8\uac00 \uce58\uc6b4\ub2e4.", "paragraph_answer": "\uc54c \ud488\uae30\ub294 13\uc77c \uc815\ub3c4 \uac78\ub9ac\uba70, \ub9c8\uc9c0\ub9c9\uc5d0 \ub0b3\uc740 \uc54c\uc774 \uac00\uc7a5 \uba3c\uc800 \uc54a\uc740 \uc54c\ubcf4\ub2e4 \uc57d 24\uc2dc\uac04 \ub2a6\uac8c \ubd80\ud654\ud55c\ub2e4. \uc554\uc218 \ubaa8\ub450 \uc54c\uc744 \ub3cc\ubcf4\uc9c0\ub9cc, \uc554\ucef7\uc774 \uc218\ucef7\ubcf4\ub2e4 \ub354 \uc624\ub798 \uc54c\uc744 \ud488\ub294\ub2e4. \ub610\ud55c \ubc24\uc774 \ub418\uba74 \uc218\ucef7\uc740 \ud683\ub300\ub85c \ub3cc\uc544\uac00 \ub2e4\ub978 \uc0c8\ub4e4\uacfc \uc5b4\uc6b8\ub9ac\uc9c0\ub9cc \uc554\ucef7\uc740 \uacc4\uc18d \ub465\uc9c0\uc5d0 \ub0a8\uc544 \uc54c\uc744 \ud488\ub294\ub2e4. \uac13 \uae68\uc5b4\ub09c \uc0c8\ub294 \ub208\ub3c4 \ub728\uc9c0 \ubabb\ud558\uace0 \ud138\ub3c4 \ud558\ub098 \uc5c6\ub2e4. \uc774\ud6c4 7\uc77c\uc5d0 \uac78\uccd0 \uc19c\ud138\uc774 \ub3cb\uc544\ub098\uba70, 9\uc77c\uc9f8\uac00 \ub418\uba74 \ub208\uc774 \ub728\uc5ec \uc55e\uc744 \ubcfc \uc218 \uc788\ub2e4. \ubd80\ud654 \ud6c4 \uc57d 6\uc77c\uc774 \uc9c0\ub098\uba74 \ubcd1\uc544\ub9ac\ub4e4\uc774 \uc790\uae30 \uccb4\uc628\uc744 \uc720\uc9c0\ud560 \uc218 \uc788\uac8c \ub41c\ub2e4. \uadf8\ub7ec\uba74 \ubd80\ubaa8 \uc0c8\ub4e4\uc740 \ubc30\uc124\ubb3c\uc744 \ub465\uc9c0 \ubc16\uc73c\ub85c \ubc84\ub9ac\uae30\ub97c \uadf8\ub9cc\ub454\ub2e4. \uadf8 \uc804\uae4c\uc9c0\ub294 \ubc30\uc124\ubb3c\uc774 \ub465\uc9c0\uc640 \ubcd1\uc544\ub9ac\uc758 \ud138\uc744 \uc801\uc154\uc11c \ub2e8\uc5f4\ub2a5\ub825\uc744 \uc800\ud574\ud558\uace0 \uac13 \ud0dc\uc5b4\ub09c \uc0c8\ub07c\ub97c \uc800\uccb4\uc628\uc99d\uc5d0 \uac78\ub9ac\uac8c \ud560 \uc218 \uc788\uae30\uc5d0 \ubd80\uc9c0\ub7f0\ud788 \ub465\uc9c0 \ubc16\uc73c\ub85c \ub0b4\ub2e4 \ubc84\ub9b0\ub2e4. \uc0c8\ub07c \uc0c8\ub294 3\uc8fc \uac00\ub7c9 \ub465\uc9c0\uc5d0 \uba38\ubb3c\uba74\uc11c \ubd80\ubaa8\uc5d0\uac8c \uacc4\uc18d \uba39\uc774\ub97c \ubc1b\uc544\uba39\ub294\ub2e4. \uae43\ud138\uacfc \uadfc\uc721\uc774 \ubc1c\ub2ec\ud558\uace0 \ub098\uc11c\ub3c4 1, 2\uc8fc \ub354 \ubd80\ubaa8\uc5d0\uac8c \uba39\uc774\ub97c \ubc1b\uc544\uba39\ub294\ub2e4. \ud55c \uc30d\uc758 \ubd80\ubd80\ub294 1\ub144\uc5d0 \uc138 \ubc88 \ubc88\uc2dd\uc744 \ud560 \uc218 \uc788\uc73c\uba70, \ub300\uac1c \uc4f0\ub358 \ub465\uc9c0\uc758 \ub0b4\ubd80\uc7ac\ub9cc \uc190\ubd10\uc11c \ub2e4\uc2dc \uc0ac\uc6a9\ud55c\ub2e4. \ud558\uc9c0\ub9cc \uc2e4\uc81c\ub85c \uc138 \ubc88\uc744 \ucc44\uc6b0\uae30\ubcf4\ub2e4\ub294 \ub450 \ubc88 \uc815\ub3c4\uac00 \ud754\ud558\uace0, \ubd81\uc704 48\ub3c4 \uc774\uc0c1\uc5d0\uc11c\ub294 1\ub144\uc5d0 \ud55c \ubc88 \ubc16\uc5d0 \uc0c8\ub07c\ub97c \uce58\uc9c0 \uc54a\ub294\ub2e4. 2\uac1c\uc6d4 \ub0b4\uc5d0 \uc0c8\ub07c\ub4e4\uc740 \ud138\uac08\uc774\ub97c \ud558\uace0 \uc19c\ud138 \ub300\uc2e0 \uccab \uae43\ud138\uc744 \uc5bb\uac8c \ub41c\ub2e4. \uc5b4\ub978 \uae43\ud138\uc774 \ub098\ub824\uba74 \uc774\ub4ec\ud574\uae4c\uc9c0 \ub354 \uae30\ub2e4\ub824\uc57c \ud55c\ub2e4. \ub2e4\ub978 \uc5f0\uc791\ub958\uc640 \ub9c8\ucc2c\uac00\uc9c0\ub85c \ub465\uc9c0\ub294 \uae68\ub057\ud558\uac8c \uccad\uc18c\ud558\uace0 \ubcd1\uc544\ub9ac\ub4e4\uc758 \ubc30\uc124\ub0ad\uc740 \ubd80\ubaa8\uac00 \uce58\uc6b4\ub2e4.", "sentence_answer": "\uc54c \ud488\uae30\ub294 13\uc77c \uc815\ub3c4 \uac78\ub9ac\uba70, \ub9c8\uc9c0\ub9c9\uc5d0 \ub0b3\uc740 \uc54c\uc774 \uac00\uc7a5 \uba3c\uc800 \uc54a\uc740 \uc54c\ubcf4\ub2e4 \uc57d 24\uc2dc\uac04 \ub2a6\uac8c \ubd80\ud654\ud55c\ub2e4.", "paragraph_id": "6464640-14-2", "paragraph_question": "question: \uc720\ub7fd\ucc0c\ub974\ub808\uae30\uc758 \uc54c \ud488\uae30\uc758 \uae30\uac04\uc740 \ub300\ub7b5 \uba87\uc77c\uc778\uac00?, context: \uc54c \ud488\uae30\ub294 13\uc77c \uc815\ub3c4 \uac78\ub9ac\uba70, \ub9c8\uc9c0\ub9c9\uc5d0 \ub0b3\uc740 \uc54c\uc774 \uac00\uc7a5 \uba3c\uc800 \uc54a\uc740 \uc54c\ubcf4\ub2e4 \uc57d 24\uc2dc\uac04 \ub2a6\uac8c \ubd80\ud654\ud55c\ub2e4. \uc554\uc218 \ubaa8\ub450 \uc54c\uc744 \ub3cc\ubcf4\uc9c0\ub9cc, \uc554\ucef7\uc774 \uc218\ucef7\ubcf4\ub2e4 \ub354 \uc624\ub798 \uc54c\uc744 \ud488\ub294\ub2e4. \ub610\ud55c \ubc24\uc774 \ub418\uba74 \uc218\ucef7\uc740 \ud683\ub300\ub85c \ub3cc\uc544\uac00 \ub2e4\ub978 \uc0c8\ub4e4\uacfc \uc5b4\uc6b8\ub9ac\uc9c0\ub9cc \uc554\ucef7\uc740 \uacc4\uc18d \ub465\uc9c0\uc5d0 \ub0a8\uc544 \uc54c\uc744 \ud488\ub294\ub2e4. \uac13 \uae68\uc5b4\ub09c \uc0c8\ub294 \ub208\ub3c4 \ub728\uc9c0 \ubabb\ud558\uace0 \ud138\ub3c4 \ud558\ub098 \uc5c6\ub2e4. \uc774\ud6c4 7\uc77c\uc5d0 \uac78\uccd0 \uc19c\ud138\uc774 \ub3cb\uc544\ub098\uba70, 9\uc77c\uc9f8\uac00 \ub418\uba74 \ub208\uc774 \ub728\uc5ec \uc55e\uc744 \ubcfc \uc218 \uc788\ub2e4. \ubd80\ud654 \ud6c4 \uc57d 6\uc77c\uc774 \uc9c0\ub098\uba74 \ubcd1\uc544\ub9ac\ub4e4\uc774 \uc790\uae30 \uccb4\uc628\uc744 \uc720\uc9c0\ud560 \uc218 \uc788\uac8c \ub41c\ub2e4. \uadf8\ub7ec\uba74 \ubd80\ubaa8 \uc0c8\ub4e4\uc740 \ubc30\uc124\ubb3c\uc744 \ub465\uc9c0 \ubc16\uc73c\ub85c \ubc84\ub9ac\uae30\ub97c \uadf8\ub9cc\ub454\ub2e4. \uadf8 \uc804\uae4c\uc9c0\ub294 \ubc30\uc124\ubb3c\uc774 \ub465\uc9c0\uc640 \ubcd1\uc544\ub9ac\uc758 \ud138\uc744 \uc801\uc154\uc11c \ub2e8\uc5f4\ub2a5\ub825\uc744 \uc800\ud574\ud558\uace0 \uac13 \ud0dc\uc5b4\ub09c \uc0c8\ub07c\ub97c \uc800\uccb4\uc628\uc99d\uc5d0 \uac78\ub9ac\uac8c \ud560 \uc218 \uc788\uae30\uc5d0 \ubd80\uc9c0\ub7f0\ud788 \ub465\uc9c0 \ubc16\uc73c\ub85c \ub0b4\ub2e4 \ubc84\ub9b0\ub2e4. \uc0c8\ub07c \uc0c8\ub294 3\uc8fc \uac00\ub7c9 \ub465\uc9c0\uc5d0 \uba38\ubb3c\uba74\uc11c \ubd80\ubaa8\uc5d0\uac8c \uacc4\uc18d \uba39\uc774\ub97c \ubc1b\uc544\uba39\ub294\ub2e4. \uae43\ud138\uacfc \uadfc\uc721\uc774 \ubc1c\ub2ec\ud558\uace0 \ub098\uc11c\ub3c4 1, 2\uc8fc \ub354 \ubd80\ubaa8\uc5d0\uac8c \uba39\uc774\ub97c \ubc1b\uc544\uba39\ub294\ub2e4. \ud55c \uc30d\uc758 \ubd80\ubd80\ub294 1\ub144\uc5d0 \uc138 \ubc88 \ubc88\uc2dd\uc744 \ud560 \uc218 \uc788\uc73c\uba70, \ub300\uac1c \uc4f0\ub358 \ub465\uc9c0\uc758 \ub0b4\ubd80\uc7ac\ub9cc \uc190\ubd10\uc11c \ub2e4\uc2dc \uc0ac\uc6a9\ud55c\ub2e4. \ud558\uc9c0\ub9cc \uc2e4\uc81c\ub85c \uc138 \ubc88\uc744 \ucc44\uc6b0\uae30\ubcf4\ub2e4\ub294 \ub450 \ubc88 \uc815\ub3c4\uac00 \ud754\ud558\uace0, \ubd81\uc704 48\ub3c4 \uc774\uc0c1\uc5d0\uc11c\ub294 1\ub144\uc5d0 \ud55c \ubc88 \ubc16\uc5d0 \uc0c8\ub07c\ub97c \uce58\uc9c0 \uc54a\ub294\ub2e4. 2\uac1c\uc6d4 \ub0b4\uc5d0 \uc0c8\ub07c\ub4e4\uc740 \ud138\uac08\uc774\ub97c \ud558\uace0 \uc19c\ud138 \ub300\uc2e0 \uccab \uae43\ud138\uc744 \uc5bb\uac8c \ub41c\ub2e4. \uc5b4\ub978 \uae43\ud138\uc774 \ub098\ub824\uba74 \uc774\ub4ec\ud574\uae4c\uc9c0 \ub354 \uae30\ub2e4\ub824\uc57c \ud55c\ub2e4. \ub2e4\ub978 \uc5f0\uc791\ub958\uc640 \ub9c8\ucc2c\uac00\uc9c0\ub85c \ub465\uc9c0\ub294 \uae68\ub057\ud558\uac8c \uccad\uc18c\ud558\uace0 \ubcd1\uc544\ub9ac\ub4e4\uc758 \ubc30\uc124\ub0ad\uc740 \ubd80\ubaa8\uac00 \uce58\uc6b4\ub2e4."} {"question": "\uc774\uc131\uacc4\uac00 \uac00\uc9c0\uace0 \uc788\ub358 \uba85\ub9c8 \uc911 \uc81c\uc8fc\ub9c8\uc758 \uc774\ub984\uc740?", "paragraph": "\uc870\uc120 \uc655\uc870\uc758 \uac1c\ucc3d\uc790\uc774\uc790 \uace0\ub824 \ub9d0\uc758 \ubb34\uc7a5\uc73c\ub85c\uc11c \ud65c\uc57d\ud588\ub358 \uc774\uc131\uacc4\uc5d0\uac8c\ub294 \ud314\uc900\ub9c8\ub77c \ubd88\ub9ac\ub294 \uc5ec\ub35f \ub9c8\ub9ac\uc758 \uba85\ub9c8\uac00 \uc788\uc5c8\ub294\ub370, \uc774 \uc911 \uc704\ud654\ub3c4 \ud68c\uad70 \ub2f9\uc2dc \ud0c0\uace0 \uc788\uc5c8\ub358 \uc751\uc0c1\ubc31(\u51dd\u971c\u767d)\uc774 \uc81c\uc8fc\ub9c8\uc600\ub2e4\uace0 \ud55c\ub2e4. \ub9d0\uc758 \uc218\ub97c \ub298\ub9ac\uae30 \uc704\ud574 \ub9d0\uace0\uae30 \uba39\ub294 \uac83\ub3c4 \uae08\uc9c0\ub418\uc5b4, \ud0dc\uc870 7\ub144(1398\ub144)\uc5d0 4,414\ud544\uc774\ub358 \uc81c\uc8fc\ub9c8\ub294 \uc138\uc885(\u4e16\u5b97) 11\ub144(1429\ub144)\ubd80\ud130 16\ub144(1434\ub144)\uae4c\uc9c0 1\ub9cc\uc5ec \ud544\ub85c \ubd88\uc5b4\ub0ac\ub2e4. \u300a\uc138\uc885\uc2e4\ub85d\u300b\uc5d0\ub294 \uc138\uc885 11\ub144(1429\ub144)\uc5d0 \uc81c\uc8fc \ucd9c\uc2e0\uc758 \uad00\uc778 \uace0\ub4dd\uc885(\u9ad8\u5f97\u5b97)\uc758 \uac74\uc758\uc5d0 \ub530\ub77c \ud55c\ub77c\uc0b0(\u6f22\u62cf\u5c71) \uc911\uc0b0\uac04 \uc9c0\uc5ed\uc5d0 \uc870\uc131\ub418\uc5c8\ub294\ub370, \uc774\ub54c \ud574\uc548 \uc9c0\uc5ed\uc758 \ucd0c\ub77d\uacfc \ub18d\uacbd\uc9c0\uc640\uc758 \uacbd\uacc4\ub97c \uc704\ud574 \ub3cc\ub85c \ud558\uc7a3(\ub3cc\ub2f4)\uc774\ub77c\ub294 \ub3cc\ub2f4\uc744 \uc313\uc558\uace0, \uc131\uc885(\u6210\u5b97) 24\ub144(1493\ub144) \uc774\uc804\uc5d0 \uc644\uc131\ub41c \ud558\uc7a3\uc744 \uae30\uc900\uc73c\ub85c 10\uac1c\uc758 \uad6c\uc5ed\uc73c\ub85c \ub098\ub208 10\uc18c\uc7a5\uc774 \uc124\uce58\ub410\ub2e4. \uac01 \uc18c\uc7a5\uc758 \ub458\ub808\ub294 45\u223c60\ub9ac\uc600\ub2e4.", "answer": "\uc751\uc0c1\ubc31", "sentence": "\uc870\uc120 \uc655\uc870\uc758 \uac1c\ucc3d\uc790\uc774\uc790 \uace0\ub824 \ub9d0\uc758 \ubb34\uc7a5\uc73c\ub85c\uc11c \ud65c\uc57d\ud588\ub358 \uc774\uc131\uacc4\uc5d0\uac8c\ub294 \ud314\uc900\ub9c8\ub77c \ubd88\ub9ac\ub294 \uc5ec\ub35f \ub9c8\ub9ac\uc758 \uba85\ub9c8\uac00 \uc788\uc5c8\ub294\ub370, \uc774 \uc911 \uc704\ud654\ub3c4 \ud68c\uad70 \ub2f9\uc2dc \ud0c0\uace0 \uc788\uc5c8\ub358 \uc751\uc0c1\ubc31 (\u51dd\u971c\u767d)", "paragraph_sentence": " \uc870\uc120 \uc655\uc870\uc758 \uac1c\ucc3d\uc790\uc774\uc790 \uace0\ub824 \ub9d0\uc758 \ubb34\uc7a5\uc73c\ub85c\uc11c \ud65c\uc57d\ud588\ub358 \uc774\uc131\uacc4\uc5d0\uac8c\ub294 \ud314\uc900\ub9c8\ub77c \ubd88\ub9ac\ub294 \uc5ec\ub35f \ub9c8\ub9ac\uc758 \uba85\ub9c8\uac00 \uc788\uc5c8\ub294\ub370, \uc774 \uc911 \uc704\ud654\ub3c4 \ud68c\uad70 \ub2f9\uc2dc \ud0c0\uace0 \uc788\uc5c8\ub358 \uc751\uc0c1\ubc31 (\u51dd\u971c\u767d) \uc774 \uc81c\uc8fc\ub9c8\uc600\ub2e4\uace0 \ud55c\ub2e4. \ub9d0\uc758 \uc218\ub97c \ub298\ub9ac\uae30 \uc704\ud574 \ub9d0\uace0\uae30 \uba39\ub294 \uac83\ub3c4 \uae08\uc9c0\ub418\uc5b4, \ud0dc\uc870 7\ub144(1398\ub144)\uc5d0 4,414\ud544\uc774\ub358 \uc81c\uc8fc\ub9c8\ub294 \uc138\uc885(\u4e16\u5b97) 11\ub144(1429\ub144)\ubd80\ud130 16\ub144(1434\ub144)\uae4c\uc9c0 1\ub9cc\uc5ec \ud544\ub85c \ubd88\uc5b4\ub0ac\ub2e4. \u300a\uc138\uc885\uc2e4\ub85d\u300b\uc5d0\ub294 \uc138\uc885 11\ub144(1429\ub144)\uc5d0 \uc81c\uc8fc \ucd9c\uc2e0\uc758 \uad00\uc778 \uace0\ub4dd\uc885(\u9ad8\u5f97\u5b97)\uc758 \uac74\uc758\uc5d0 \ub530\ub77c \ud55c\ub77c\uc0b0(\u6f22\u62cf\u5c71) \uc911\uc0b0\uac04 \uc9c0\uc5ed\uc5d0 \uc870\uc131\ub418\uc5c8\ub294\ub370, \uc774\ub54c \ud574\uc548 \uc9c0\uc5ed\uc758 \ucd0c\ub77d\uacfc \ub18d\uacbd\uc9c0\uc640\uc758 \uacbd\uacc4\ub97c \uc704\ud574 \ub3cc\ub85c \ud558\uc7a3(\ub3cc\ub2f4)\uc774\ub77c\ub294 \ub3cc\ub2f4\uc744 \uc313\uc558\uace0, \uc131\uc885(\u6210\u5b97) 24\ub144(1493\ub144) \uc774\uc804\uc5d0 \uc644\uc131\ub41c \ud558\uc7a3\uc744 \uae30\uc900\uc73c\ub85c 10\uac1c\uc758 \uad6c\uc5ed\uc73c\ub85c \ub098\ub208 10\uc18c\uc7a5\uc774 \uc124\uce58\ub410\ub2e4. \uac01 \uc18c\uc7a5\uc758 \ub458\ub808\ub294 45\u223c60\ub9ac\uc600\ub2e4.", "paragraph_answer": "\uc870\uc120 \uc655\uc870\uc758 \uac1c\ucc3d\uc790\uc774\uc790 \uace0\ub824 \ub9d0\uc758 \ubb34\uc7a5\uc73c\ub85c\uc11c \ud65c\uc57d\ud588\ub358 \uc774\uc131\uacc4\uc5d0\uac8c\ub294 \ud314\uc900\ub9c8\ub77c \ubd88\ub9ac\ub294 \uc5ec\ub35f \ub9c8\ub9ac\uc758 \uba85\ub9c8\uac00 \uc788\uc5c8\ub294\ub370, \uc774 \uc911 \uc704\ud654\ub3c4 \ud68c\uad70 \ub2f9\uc2dc \ud0c0\uace0 \uc788\uc5c8\ub358 \uc751\uc0c1\ubc31 (\u51dd\u971c\u767d)\uc774 \uc81c\uc8fc\ub9c8\uc600\ub2e4\uace0 \ud55c\ub2e4. \ub9d0\uc758 \uc218\ub97c \ub298\ub9ac\uae30 \uc704\ud574 \ub9d0\uace0\uae30 \uba39\ub294 \uac83\ub3c4 \uae08\uc9c0\ub418\uc5b4, \ud0dc\uc870 7\ub144(1398\ub144)\uc5d0 4,414\ud544\uc774\ub358 \uc81c\uc8fc\ub9c8\ub294 \uc138\uc885(\u4e16\u5b97) 11\ub144(1429\ub144)\ubd80\ud130 16\ub144(1434\ub144)\uae4c\uc9c0 1\ub9cc\uc5ec \ud544\ub85c \ubd88\uc5b4\ub0ac\ub2e4. \u300a\uc138\uc885\uc2e4\ub85d\u300b\uc5d0\ub294 \uc138\uc885 11\ub144(1429\ub144)\uc5d0 \uc81c\uc8fc \ucd9c\uc2e0\uc758 \uad00\uc778 \uace0\ub4dd\uc885(\u9ad8\u5f97\u5b97)\uc758 \uac74\uc758\uc5d0 \ub530\ub77c \ud55c\ub77c\uc0b0(\u6f22\u62cf\u5c71) \uc911\uc0b0\uac04 \uc9c0\uc5ed\uc5d0 \uc870\uc131\ub418\uc5c8\ub294\ub370, \uc774\ub54c \ud574\uc548 \uc9c0\uc5ed\uc758 \ucd0c\ub77d\uacfc \ub18d\uacbd\uc9c0\uc640\uc758 \uacbd\uacc4\ub97c \uc704\ud574 \ub3cc\ub85c \ud558\uc7a3(\ub3cc\ub2f4)\uc774\ub77c\ub294 \ub3cc\ub2f4\uc744 \uc313\uc558\uace0, \uc131\uc885(\u6210\u5b97) 24\ub144(1493\ub144) \uc774\uc804\uc5d0 \uc644\uc131\ub41c \ud558\uc7a3\uc744 \uae30\uc900\uc73c\ub85c 10\uac1c\uc758 \uad6c\uc5ed\uc73c\ub85c \ub098\ub208 10\uc18c\uc7a5\uc774 \uc124\uce58\ub410\ub2e4. \uac01 \uc18c\uc7a5\uc758 \ub458\ub808\ub294 45\u223c60\ub9ac\uc600\ub2e4.", "sentence_answer": "\uc870\uc120 \uc655\uc870\uc758 \uac1c\ucc3d\uc790\uc774\uc790 \uace0\ub824 \ub9d0\uc758 \ubb34\uc7a5\uc73c\ub85c\uc11c \ud65c\uc57d\ud588\ub358 \uc774\uc131\uacc4\uc5d0\uac8c\ub294 \ud314\uc900\ub9c8\ub77c \ubd88\ub9ac\ub294 \uc5ec\ub35f \ub9c8\ub9ac\uc758 \uba85\ub9c8\uac00 \uc788\uc5c8\ub294\ub370, \uc774 \uc911 \uc704\ud654\ub3c4 \ud68c\uad70 \ub2f9\uc2dc \ud0c0\uace0 \uc788\uc5c8\ub358 \uc751\uc0c1\ubc31 (\u51dd\u971c\u767d)", "paragraph_id": "6594323-1-0", "paragraph_question": "question: \uc774\uc131\uacc4\uac00 \uac00\uc9c0\uace0 \uc788\ub358 \uba85\ub9c8 \uc911 \uc81c\uc8fc\ub9c8\uc758 \uc774\ub984\uc740?, context: \uc870\uc120 \uc655\uc870\uc758 \uac1c\ucc3d\uc790\uc774\uc790 \uace0\ub824 \ub9d0\uc758 \ubb34\uc7a5\uc73c\ub85c\uc11c \ud65c\uc57d\ud588\ub358 \uc774\uc131\uacc4\uc5d0\uac8c\ub294 \ud314\uc900\ub9c8\ub77c \ubd88\ub9ac\ub294 \uc5ec\ub35f \ub9c8\ub9ac\uc758 \uba85\ub9c8\uac00 \uc788\uc5c8\ub294\ub370, \uc774 \uc911 \uc704\ud654\ub3c4 \ud68c\uad70 \ub2f9\uc2dc \ud0c0\uace0 \uc788\uc5c8\ub358 \uc751\uc0c1\ubc31(\u51dd\u971c\u767d)\uc774 \uc81c\uc8fc\ub9c8\uc600\ub2e4\uace0 \ud55c\ub2e4. \ub9d0\uc758 \uc218\ub97c \ub298\ub9ac\uae30 \uc704\ud574 \ub9d0\uace0\uae30 \uba39\ub294 \uac83\ub3c4 \uae08\uc9c0\ub418\uc5b4, \ud0dc\uc870 7\ub144(1398\ub144)\uc5d0 4,414\ud544\uc774\ub358 \uc81c\uc8fc\ub9c8\ub294 \uc138\uc885(\u4e16\u5b97) 11\ub144(1429\ub144)\ubd80\ud130 16\ub144(1434\ub144)\uae4c\uc9c0 1\ub9cc\uc5ec \ud544\ub85c \ubd88\uc5b4\ub0ac\ub2e4. \u300a\uc138\uc885\uc2e4\ub85d\u300b\uc5d0\ub294 \uc138\uc885 11\ub144(1429\ub144)\uc5d0 \uc81c\uc8fc \ucd9c\uc2e0\uc758 \uad00\uc778 \uace0\ub4dd\uc885(\u9ad8\u5f97\u5b97)\uc758 \uac74\uc758\uc5d0 \ub530\ub77c \ud55c\ub77c\uc0b0(\u6f22\u62cf\u5c71) \uc911\uc0b0\uac04 \uc9c0\uc5ed\uc5d0 \uc870\uc131\ub418\uc5c8\ub294\ub370, \uc774\ub54c \ud574\uc548 \uc9c0\uc5ed\uc758 \ucd0c\ub77d\uacfc \ub18d\uacbd\uc9c0\uc640\uc758 \uacbd\uacc4\ub97c \uc704\ud574 \ub3cc\ub85c \ud558\uc7a3(\ub3cc\ub2f4)\uc774\ub77c\ub294 \ub3cc\ub2f4\uc744 \uc313\uc558\uace0, \uc131\uc885(\u6210\u5b97) 24\ub144(1493\ub144) \uc774\uc804\uc5d0 \uc644\uc131\ub41c \ud558\uc7a3\uc744 \uae30\uc900\uc73c\ub85c 10\uac1c\uc758 \uad6c\uc5ed\uc73c\ub85c \ub098\ub208 10\uc18c\uc7a5\uc774 \uc124\uce58\ub410\ub2e4. \uac01 \uc18c\uc7a5\uc758 \ub458\ub808\ub294 45\u223c60\ub9ac\uc600\ub2e4."} {"question": "NEADS\uc5d0\uc11c F-15 2\uae30\ub97c \uc774\ub959\uc2dc\ud0a4\ub77c\ub294 \uba85\ub839\uc744 \ubc1b\uc740 \uae30\uc9c0\ub294 \uc5b4\ub514\uc778\uac00?", "paragraph": "8\uc2dc 33\ubd84 59\ucd08, \uc544\ud0c0\ub294 \u201c\uc6c0\uc9c1\uc774\uc9c0 \ub9c8\ub77c. \uc6b0\ub9ac\ub294 \uacf5\ud56d\uc73c\ub85c \ub3cc\uc544\uac00\uace0 \uc788\ub2e4. \ud5db\uc9d3\uac70\ub9ac \ud558\uc9c0 \ub9d0\ub77c.\u201d\ub77c\uace0 \ubc29\uc1a1\ud588\ub2e4. 8\uc2dc 37\ubd84 8\ucd08, \uc720\ub098\uc774\ud2f0\ub4dc \ud56d\uacf5 175\ud3b8\uc758 \uae30\uc7a5\uc774 11\ud3b8\uc758 \uc704\uce58\ub97c \ud655\uc778\ud588\ub2e4. \ubcf4\uc2a4\ud134 ARTCC\ub294 \ud45c\uc900 \uaddc\uc815\uc744 \uc6b0\ud68c\ud558\uc5ec \uc9c1\uc811 \ub274\uc695 \uc8fc \ub86c\uc5d0 \uc788\ub294 \ubd81\ubbf8\ud56d\uacf5\uc6b0\uc8fc\ubc29\uc704\uc0ac\ub839\ubd80(NORAD) \ubd81\ub3d9\ubd80 \ud56d\uacf5 \ubc29\uc704 \uad6c\uc5ed(NEADS)\uc5d0 \uc5f0\ub77d\ud588\ub2e4. NEADS\ub294 \uc774 \uc5f0\ub77d\uc744 \ubc1b\uace0 \uba54\uc774\uc288\ud53c\uc5d0 \uc788\ub294 \uc624\ud2f0\uc2a4 \uc8fc \uacf5\uad70 \uae30\uc9c0\uc5d0\uc11c F-15 2\uae30\ub97c \uc774\ub959 \uba85\ub839\uc744 \ub0b4\ub838\ub2e4. \uc624\ud2f0\uc2a4 \uae30\uc9c0\uc5d0\uc11c\ub294 \uba85\ub839\uc744 \ubc1b\uace0 \uc218\ubd84 \ud6c4 \uc774\ub959\ud588\ub2e4. 8\uc2dc 43\ubd84, \uc544\ud0c0\ub294 \ub9c8\uc9c0\ub9c9\uc73c\ub85c \ub9e8\ud574\ud2bc \ucabd\uc73c\ub85c \ube44\ud589\uae30 \ubc29\ud5a5\uc744 \ub3cc\ub838\ub2e4. \uc624\ud2f0\uc2a4 \uae30\uc9c0\ub294 8\uc2dc 46\ubd84 \uba85\ub839\uc744 \ubc1b\uace0 \uc544\uba54\ub9ac\uce78 \ud56d\uacf5 11\ud3b8\uc774 \uc774\ubbf8 \uc138\uacc4 \ubb34\uc5ed \uc13c\ud130 \ubd81\ucabd \ud0c0\uc6cc\uc5d0 \ub4e4\uc774\ubc1b\uc740 \uc774\ud6c4\uc778 8\uc2dc 53\ubd84\uc5d0 F-15\uae30\uac00 \ucd9c\uaca9\ud588\ub2e4. 9.11 \ud14c\ub7ec\uc5d0\uc11c \ud56d\uacf5\uae30 4\ub300\uac00 \ub0a9\uce58\ub418\uc5c8\uc744 \ub54c, \uc544\uba54\ub9ac\uce78 \ud56d\uacf5 11\ud3b8\uc758 \uacbd\uc6b0\uc5d0\ub294 NORAD\uac00 \ud56d\uacf5\uae30\uac00 \ubaa9\ud45c\ubb3c\uc5d0 \ucda9\ub3cc\ud558\uae30 \uc9c1\uc804\uae4c\uc9c0 \ub300\uc751\ud560 \uc218 \uc788\uc744 \uc2dc\uac04\uc740 \ubd88\uacfc 9\ubd84\uc774\uc5c8\uc9c0\ub9cc, \uc774 \uc2dc\uac04\uc740 4\ub300 \uc911 \uc81c\uc77c \uae38\uc5c8\ub2e4.", "answer": "\uc624\ud2f0\uc2a4 \uae30\uc9c0", "sentence": "\uc624\ud2f0\uc2a4 \uae30\uc9c0 \uc5d0\uc11c\ub294 \uba85\ub839\uc744 \ubc1b\uace0 \uc218\ubd84 \ud6c4 \uc774\ub959\ud588\ub2e4.", "paragraph_sentence": "8\uc2dc 33\ubd84 59\ucd08, \uc544\ud0c0\ub294 \u201c\uc6c0\uc9c1\uc774\uc9c0 \ub9c8\ub77c. \uc6b0\ub9ac\ub294 \uacf5\ud56d\uc73c\ub85c \ub3cc\uc544\uac00\uace0 \uc788\ub2e4. \ud5db\uc9d3\uac70\ub9ac \ud558\uc9c0 \ub9d0\ub77c. \u201d\ub77c\uace0 \ubc29\uc1a1\ud588\ub2e4. 8\uc2dc 37\ubd84 8\ucd08, \uc720\ub098\uc774\ud2f0\ub4dc \ud56d\uacf5 175\ud3b8\uc758 \uae30\uc7a5\uc774 11\ud3b8\uc758 \uc704\uce58\ub97c \ud655\uc778\ud588\ub2e4. \ubcf4\uc2a4\ud134 ARTCC\ub294 \ud45c\uc900 \uaddc\uc815\uc744 \uc6b0\ud68c\ud558\uc5ec \uc9c1\uc811 \ub274\uc695 \uc8fc \ub86c\uc5d0 \uc788\ub294 \ubd81\ubbf8\ud56d\uacf5\uc6b0\uc8fc\ubc29\uc704\uc0ac\ub839\ubd80(NORAD) \ubd81\ub3d9\ubd80 \ud56d\uacf5 \ubc29\uc704 \uad6c\uc5ed(NEADS)\uc5d0 \uc5f0\ub77d\ud588\ub2e4. NEADS\ub294 \uc774 \uc5f0\ub77d\uc744 \ubc1b\uace0 \uba54\uc774\uc288\ud53c\uc5d0 \uc788\ub294 \uc624\ud2f0\uc2a4 \uc8fc \uacf5\uad70 \uae30\uc9c0\uc5d0\uc11c F-15 2\uae30\ub97c \uc774\ub959 \uba85\ub839\uc744 \ub0b4\ub838\ub2e4. \uc624\ud2f0\uc2a4 \uae30\uc9c0 \uc5d0\uc11c\ub294 \uba85\ub839\uc744 \ubc1b\uace0 \uc218\ubd84 \ud6c4 \uc774\ub959\ud588\ub2e4. 8\uc2dc 43\ubd84, \uc544\ud0c0\ub294 \ub9c8\uc9c0\ub9c9\uc73c\ub85c \ub9e8\ud574\ud2bc \ucabd\uc73c\ub85c \ube44\ud589\uae30 \ubc29\ud5a5\uc744 \ub3cc\ub838\ub2e4. \uc624\ud2f0\uc2a4 \uae30\uc9c0\ub294 8\uc2dc 46\ubd84 \uba85\ub839\uc744 \ubc1b\uace0 \uc544\uba54\ub9ac\uce78 \ud56d\uacf5 11\ud3b8\uc774 \uc774\ubbf8 \uc138\uacc4 \ubb34\uc5ed \uc13c\ud130 \ubd81\ucabd \ud0c0\uc6cc\uc5d0 \ub4e4\uc774\ubc1b\uc740 \uc774\ud6c4\uc778 8\uc2dc 53\ubd84\uc5d0 F-15\uae30\uac00 \ucd9c\uaca9\ud588\ub2e4. 9.11 \ud14c\ub7ec\uc5d0\uc11c \ud56d\uacf5\uae30 4\ub300\uac00 \ub0a9\uce58\ub418\uc5c8\uc744 \ub54c, \uc544\uba54\ub9ac\uce78 \ud56d\uacf5 11\ud3b8\uc758 \uacbd\uc6b0\uc5d0\ub294 NORAD\uac00 \ud56d\uacf5\uae30\uac00 \ubaa9\ud45c\ubb3c\uc5d0 \ucda9\ub3cc\ud558\uae30 \uc9c1\uc804\uae4c\uc9c0 \ub300\uc751\ud560 \uc218 \uc788\uc744 \uc2dc\uac04\uc740 \ubd88\uacfc 9\ubd84\uc774\uc5c8\uc9c0\ub9cc, \uc774 \uc2dc\uac04\uc740 4\ub300 \uc911 \uc81c\uc77c \uae38\uc5c8\ub2e4.", "paragraph_answer": "8\uc2dc 33\ubd84 59\ucd08, \uc544\ud0c0\ub294 \u201c\uc6c0\uc9c1\uc774\uc9c0 \ub9c8\ub77c. \uc6b0\ub9ac\ub294 \uacf5\ud56d\uc73c\ub85c \ub3cc\uc544\uac00\uace0 \uc788\ub2e4. \ud5db\uc9d3\uac70\ub9ac \ud558\uc9c0 \ub9d0\ub77c.\u201d\ub77c\uace0 \ubc29\uc1a1\ud588\ub2e4. 8\uc2dc 37\ubd84 8\ucd08, \uc720\ub098\uc774\ud2f0\ub4dc \ud56d\uacf5 175\ud3b8\uc758 \uae30\uc7a5\uc774 11\ud3b8\uc758 \uc704\uce58\ub97c \ud655\uc778\ud588\ub2e4. \ubcf4\uc2a4\ud134 ARTCC\ub294 \ud45c\uc900 \uaddc\uc815\uc744 \uc6b0\ud68c\ud558\uc5ec \uc9c1\uc811 \ub274\uc695 \uc8fc \ub86c\uc5d0 \uc788\ub294 \ubd81\ubbf8\ud56d\uacf5\uc6b0\uc8fc\ubc29\uc704\uc0ac\ub839\ubd80(NORAD) \ubd81\ub3d9\ubd80 \ud56d\uacf5 \ubc29\uc704 \uad6c\uc5ed(NEADS)\uc5d0 \uc5f0\ub77d\ud588\ub2e4. NEADS\ub294 \uc774 \uc5f0\ub77d\uc744 \ubc1b\uace0 \uba54\uc774\uc288\ud53c\uc5d0 \uc788\ub294 \uc624\ud2f0\uc2a4 \uc8fc \uacf5\uad70 \uae30\uc9c0\uc5d0\uc11c F-15 2\uae30\ub97c \uc774\ub959 \uba85\ub839\uc744 \ub0b4\ub838\ub2e4. \uc624\ud2f0\uc2a4 \uae30\uc9c0 \uc5d0\uc11c\ub294 \uba85\ub839\uc744 \ubc1b\uace0 \uc218\ubd84 \ud6c4 \uc774\ub959\ud588\ub2e4. 8\uc2dc 43\ubd84, \uc544\ud0c0\ub294 \ub9c8\uc9c0\ub9c9\uc73c\ub85c \ub9e8\ud574\ud2bc \ucabd\uc73c\ub85c \ube44\ud589\uae30 \ubc29\ud5a5\uc744 \ub3cc\ub838\ub2e4. \uc624\ud2f0\uc2a4 \uae30\uc9c0\ub294 8\uc2dc 46\ubd84 \uba85\ub839\uc744 \ubc1b\uace0 \uc544\uba54\ub9ac\uce78 \ud56d\uacf5 11\ud3b8\uc774 \uc774\ubbf8 \uc138\uacc4 \ubb34\uc5ed \uc13c\ud130 \ubd81\ucabd \ud0c0\uc6cc\uc5d0 \ub4e4\uc774\ubc1b\uc740 \uc774\ud6c4\uc778 8\uc2dc 53\ubd84\uc5d0 F-15\uae30\uac00 \ucd9c\uaca9\ud588\ub2e4. 9.11 \ud14c\ub7ec\uc5d0\uc11c \ud56d\uacf5\uae30 4\ub300\uac00 \ub0a9\uce58\ub418\uc5c8\uc744 \ub54c, \uc544\uba54\ub9ac\uce78 \ud56d\uacf5 11\ud3b8\uc758 \uacbd\uc6b0\uc5d0\ub294 NORAD\uac00 \ud56d\uacf5\uae30\uac00 \ubaa9\ud45c\ubb3c\uc5d0 \ucda9\ub3cc\ud558\uae30 \uc9c1\uc804\uae4c\uc9c0 \ub300\uc751\ud560 \uc218 \uc788\uc744 \uc2dc\uac04\uc740 \ubd88\uacfc 9\ubd84\uc774\uc5c8\uc9c0\ub9cc, \uc774 \uc2dc\uac04\uc740 4\ub300 \uc911 \uc81c\uc77c \uae38\uc5c8\ub2e4.", "sentence_answer": " \uc624\ud2f0\uc2a4 \uae30\uc9c0 \uc5d0\uc11c\ub294 \uba85\ub839\uc744 \ubc1b\uace0 \uc218\ubd84 \ud6c4 \uc774\ub959\ud588\ub2e4.", "paragraph_id": "6457571-5-0", "paragraph_question": "question: NEADS\uc5d0\uc11c F-15 2\uae30\ub97c \uc774\ub959\uc2dc\ud0a4\ub77c\ub294 \uba85\ub839\uc744 \ubc1b\uc740 \uae30\uc9c0\ub294 \uc5b4\ub514\uc778\uac00?, context: 8\uc2dc 33\ubd84 59\ucd08, \uc544\ud0c0\ub294 \u201c\uc6c0\uc9c1\uc774\uc9c0 \ub9c8\ub77c. \uc6b0\ub9ac\ub294 \uacf5\ud56d\uc73c\ub85c \ub3cc\uc544\uac00\uace0 \uc788\ub2e4. \ud5db\uc9d3\uac70\ub9ac \ud558\uc9c0 \ub9d0\ub77c.\u201d\ub77c\uace0 \ubc29\uc1a1\ud588\ub2e4. 8\uc2dc 37\ubd84 8\ucd08, \uc720\ub098\uc774\ud2f0\ub4dc \ud56d\uacf5 175\ud3b8\uc758 \uae30\uc7a5\uc774 11\ud3b8\uc758 \uc704\uce58\ub97c \ud655\uc778\ud588\ub2e4. \ubcf4\uc2a4\ud134 ARTCC\ub294 \ud45c\uc900 \uaddc\uc815\uc744 \uc6b0\ud68c\ud558\uc5ec \uc9c1\uc811 \ub274\uc695 \uc8fc \ub86c\uc5d0 \uc788\ub294 \ubd81\ubbf8\ud56d\uacf5\uc6b0\uc8fc\ubc29\uc704\uc0ac\ub839\ubd80(NORAD) \ubd81\ub3d9\ubd80 \ud56d\uacf5 \ubc29\uc704 \uad6c\uc5ed(NEADS)\uc5d0 \uc5f0\ub77d\ud588\ub2e4. NEADS\ub294 \uc774 \uc5f0\ub77d\uc744 \ubc1b\uace0 \uba54\uc774\uc288\ud53c\uc5d0 \uc788\ub294 \uc624\ud2f0\uc2a4 \uc8fc \uacf5\uad70 \uae30\uc9c0\uc5d0\uc11c F-15 2\uae30\ub97c \uc774\ub959 \uba85\ub839\uc744 \ub0b4\ub838\ub2e4. \uc624\ud2f0\uc2a4 \uae30\uc9c0\uc5d0\uc11c\ub294 \uba85\ub839\uc744 \ubc1b\uace0 \uc218\ubd84 \ud6c4 \uc774\ub959\ud588\ub2e4. 8\uc2dc 43\ubd84, \uc544\ud0c0\ub294 \ub9c8\uc9c0\ub9c9\uc73c\ub85c \ub9e8\ud574\ud2bc \ucabd\uc73c\ub85c \ube44\ud589\uae30 \ubc29\ud5a5\uc744 \ub3cc\ub838\ub2e4. \uc624\ud2f0\uc2a4 \uae30\uc9c0\ub294 8\uc2dc 46\ubd84 \uba85\ub839\uc744 \ubc1b\uace0 \uc544\uba54\ub9ac\uce78 \ud56d\uacf5 11\ud3b8\uc774 \uc774\ubbf8 \uc138\uacc4 \ubb34\uc5ed \uc13c\ud130 \ubd81\ucabd \ud0c0\uc6cc\uc5d0 \ub4e4\uc774\ubc1b\uc740 \uc774\ud6c4\uc778 8\uc2dc 53\ubd84\uc5d0 F-15\uae30\uac00 \ucd9c\uaca9\ud588\ub2e4. 9.11 \ud14c\ub7ec\uc5d0\uc11c \ud56d\uacf5\uae30 4\ub300\uac00 \ub0a9\uce58\ub418\uc5c8\uc744 \ub54c, \uc544\uba54\ub9ac\uce78 \ud56d\uacf5 11\ud3b8\uc758 \uacbd\uc6b0\uc5d0\ub294 NORAD\uac00 \ud56d\uacf5\uae30\uac00 \ubaa9\ud45c\ubb3c\uc5d0 \ucda9\ub3cc\ud558\uae30 \uc9c1\uc804\uae4c\uc9c0 \ub300\uc751\ud560 \uc218 \uc788\uc744 \uc2dc\uac04\uc740 \ubd88\uacfc 9\ubd84\uc774\uc5c8\uc9c0\ub9cc, \uc774 \uc2dc\uac04\uc740 4\ub300 \uc911 \uc81c\uc77c \uae38\uc5c8\ub2e4."} {"question": "\ud3ec\ucf13\ubaac \uc2dc\ub9ac\uc988\ub97c \uc77c\ubcf8\uc5d0 \ucc98\uc74c\uc73c\ub85c \ucd9c\uc2dc\ud55c \uacf3\uc740 \uc5b4\ub514\uc778\uac00\uc694?", "paragraph": "\uac8c\uc784 \ud504\ub9ac\ud06c\uac00 \uac1c\ubc1c\ud55c \ud3ec\ucf13\ubaac \uc2dc\ub9ac\uc988\ub294 1996\ub144 \ub2cc\ud150\ub3c4\uc5d0 \uc758\ud574 \uc77c\ubcf8\uc5d0\uc11c \uccab \ucd9c\uc2dc\ub418\uc5c8\ub2e4. \ud50c\ub808\uc774\uc5b4\ub294 \ud3ec\ucf13\ubaac \ud2b8\ub808\uc774\ub108 \uc5ed\ud560\uc744 \ub9e1\uc544 \ud3ec\ucf13\ubaac\uc744 \ud3ec\ud68d\ud558\uc5ec \ud6c8\ub828\uc2dc\ud0a4\uace0, \uac01 \ud3ec\ucf13\ubaac\ub4e4\uc758 \uace0\uc720 \uae30\uc220\uc744 \uc774\uc6a9\ud558\uc5ec \uc11c\ub85c \ub300\uc804\uc2dc\ud0a8\ub2e4. \ub2cc\ud150\ub3c4 \uce21\uc740 \u300a\ub2cc\ud150\ub3c4 \ud30c\uc6cc\u300b 1999\ub144 5\uc6d4\ud638\uc5d0\uc11c \ubbf8\uc2f1\ub178\uc758 \ucd9c\ud604\uc5d0 \ub300\ud574, \"\ubbf8\uc2f1\ub178\uc640 \uc811\ucd09\ud560 \uacbd\uc6b0, \ubbf8\uc2f1\ub178\ub97c \ud3ec\ud68d\ud558\uc9c0 \uc54a\ub354\ub77c\ub3c4 \uac8c\uc784 \ud30c\uc77c\uc774 \uc0ad\uc81c\ub420 \uc218 \uc788\uc73c\uba70, \uadf8\ub798\ud53d \uae68\uc9d0 \ud604\uc0c1\uc774 \ubc1c\uc0dd\ud560 \uc218 \uc788\ub2e4\u201d\ub294 \uacbd\uace0\uc640 \ud568\uaed8 \ucc98\uc74c\uc73c\ub85c \uacf5\uc2dd\uc801\uc73c\ub85c \uc5b8\uae09\ud558\uc600\ub2e4. \ubbf8\uc2f1\ub178\ub294 \ud2b9\uc815 \uc774\ubca4\ud2b8\ub4e4\uc774 \uc5f0\uc18d\uc801\uc73c\ub85c \uc2e4\ud589\ub418\uc5c8\uc744 \ub54c \ub098\ud0c0\ub098\uac8c \ub41c\ub2e4. \uc989, \ud50c\ub808\uc774\uc5b4\uac00 \ud3ec\ucf13\ubaac \ud3ec\ud68d\uc744 \uc704\ud55c \ud29c\ud1a0\ub9ac\uc5bc\uc744 \ubcf8 \ub2e4\uc74c, \uacf5\uc911\ub0a0\uae30\ub97c \uc774\uc6a9\ud558\uc5ec \ud64d\ub828\uc12c\uc73c\ub85c \uc774\ub3d9\ud55c \ud6c4, \ub4a4\uc774\uc5b4 \ud30c\ub3c4\ud0c0\uae30\ub85c \ub3d9\ucabd \ud574\uc548\uc5d0 \ub3c4\ub2ec\ud558\uba74 \ubbf8\uc2f1\ub178\ub97c \ub9cc\ub098\uac8c \ub41c\ub2e4.", "answer": "\ub2cc\ud150\ub3c4", "sentence": "\uac8c\uc784 \ud504\ub9ac\ud06c\uac00 \uac1c\ubc1c\ud55c \ud3ec\ucf13\ubaac \uc2dc\ub9ac\uc988\ub294 1996\ub144 \ub2cc\ud150\ub3c4 \uc5d0 \uc758\ud574 \uc77c\ubcf8\uc5d0\uc11c \uccab \ucd9c\uc2dc\ub418\uc5c8\ub2e4.", "paragraph_sentence": " \uac8c\uc784 \ud504\ub9ac\ud06c\uac00 \uac1c\ubc1c\ud55c \ud3ec\ucf13\ubaac \uc2dc\ub9ac\uc988\ub294 1996\ub144 \ub2cc\ud150\ub3c4 \uc5d0 \uc758\ud574 \uc77c\ubcf8\uc5d0\uc11c \uccab \ucd9c\uc2dc\ub418\uc5c8\ub2e4. \ud50c\ub808\uc774\uc5b4\ub294 \ud3ec\ucf13\ubaac \ud2b8\ub808\uc774\ub108 \uc5ed\ud560\uc744 \ub9e1\uc544 \ud3ec\ucf13\ubaac\uc744 \ud3ec\ud68d\ud558\uc5ec \ud6c8\ub828\uc2dc\ud0a4\uace0, \uac01 \ud3ec\ucf13\ubaac\ub4e4\uc758 \uace0\uc720 \uae30\uc220\uc744 \uc774\uc6a9\ud558\uc5ec \uc11c\ub85c \ub300\uc804\uc2dc\ud0a8\ub2e4. \ub2cc\ud150\ub3c4 \uce21\uc740 \u300a\ub2cc\ud150\ub3c4 \ud30c\uc6cc\u300b 1999\ub144 5\uc6d4\ud638\uc5d0\uc11c \ubbf8\uc2f1\ub178\uc758 \ucd9c\ud604\uc5d0 \ub300\ud574, \"\ubbf8\uc2f1\ub178\uc640 \uc811\ucd09\ud560 \uacbd\uc6b0, \ubbf8\uc2f1\ub178\ub97c \ud3ec\ud68d\ud558\uc9c0 \uc54a\ub354\ub77c\ub3c4 \uac8c\uc784 \ud30c\uc77c\uc774 \uc0ad\uc81c\ub420 \uc218 \uc788\uc73c\uba70, \uadf8\ub798\ud53d \uae68\uc9d0 \ud604\uc0c1\uc774 \ubc1c\uc0dd\ud560 \uc218 \uc788\ub2e4\u201d\ub294 \uacbd\uace0\uc640 \ud568\uaed8 \ucc98\uc74c\uc73c\ub85c \uacf5\uc2dd\uc801\uc73c\ub85c \uc5b8\uae09\ud558\uc600\ub2e4. \ubbf8\uc2f1\ub178\ub294 \ud2b9\uc815 \uc774\ubca4\ud2b8\ub4e4\uc774 \uc5f0\uc18d\uc801\uc73c\ub85c \uc2e4\ud589\ub418\uc5c8\uc744 \ub54c \ub098\ud0c0\ub098\uac8c \ub41c\ub2e4. \uc989, \ud50c\ub808\uc774\uc5b4\uac00 \ud3ec\ucf13\ubaac \ud3ec\ud68d\uc744 \uc704\ud55c \ud29c\ud1a0\ub9ac\uc5bc\uc744 \ubcf8 \ub2e4\uc74c, \uacf5\uc911\ub0a0\uae30\ub97c \uc774\uc6a9\ud558\uc5ec \ud64d\ub828\uc12c\uc73c\ub85c \uc774\ub3d9\ud55c \ud6c4, \ub4a4\uc774\uc5b4 \ud30c\ub3c4\ud0c0\uae30\ub85c \ub3d9\ucabd \ud574\uc548\uc5d0 \ub3c4\ub2ec\ud558\uba74 \ubbf8\uc2f1\ub178\ub97c \ub9cc\ub098\uac8c \ub41c\ub2e4.", "paragraph_answer": "\uac8c\uc784 \ud504\ub9ac\ud06c\uac00 \uac1c\ubc1c\ud55c \ud3ec\ucf13\ubaac \uc2dc\ub9ac\uc988\ub294 1996\ub144 \ub2cc\ud150\ub3c4 \uc5d0 \uc758\ud574 \uc77c\ubcf8\uc5d0\uc11c \uccab \ucd9c\uc2dc\ub418\uc5c8\ub2e4. \ud50c\ub808\uc774\uc5b4\ub294 \ud3ec\ucf13\ubaac \ud2b8\ub808\uc774\ub108 \uc5ed\ud560\uc744 \ub9e1\uc544 \ud3ec\ucf13\ubaac\uc744 \ud3ec\ud68d\ud558\uc5ec \ud6c8\ub828\uc2dc\ud0a4\uace0, \uac01 \ud3ec\ucf13\ubaac\ub4e4\uc758 \uace0\uc720 \uae30\uc220\uc744 \uc774\uc6a9\ud558\uc5ec \uc11c\ub85c \ub300\uc804\uc2dc\ud0a8\ub2e4. \ub2cc\ud150\ub3c4 \uce21\uc740 \u300a\ub2cc\ud150\ub3c4 \ud30c\uc6cc\u300b 1999\ub144 5\uc6d4\ud638\uc5d0\uc11c \ubbf8\uc2f1\ub178\uc758 \ucd9c\ud604\uc5d0 \ub300\ud574, \"\ubbf8\uc2f1\ub178\uc640 \uc811\ucd09\ud560 \uacbd\uc6b0, \ubbf8\uc2f1\ub178\ub97c \ud3ec\ud68d\ud558\uc9c0 \uc54a\ub354\ub77c\ub3c4 \uac8c\uc784 \ud30c\uc77c\uc774 \uc0ad\uc81c\ub420 \uc218 \uc788\uc73c\uba70, \uadf8\ub798\ud53d \uae68\uc9d0 \ud604\uc0c1\uc774 \ubc1c\uc0dd\ud560 \uc218 \uc788\ub2e4\u201d\ub294 \uacbd\uace0\uc640 \ud568\uaed8 \ucc98\uc74c\uc73c\ub85c \uacf5\uc2dd\uc801\uc73c\ub85c \uc5b8\uae09\ud558\uc600\ub2e4. \ubbf8\uc2f1\ub178\ub294 \ud2b9\uc815 \uc774\ubca4\ud2b8\ub4e4\uc774 \uc5f0\uc18d\uc801\uc73c\ub85c \uc2e4\ud589\ub418\uc5c8\uc744 \ub54c \ub098\ud0c0\ub098\uac8c \ub41c\ub2e4. \uc989, \ud50c\ub808\uc774\uc5b4\uac00 \ud3ec\ucf13\ubaac \ud3ec\ud68d\uc744 \uc704\ud55c \ud29c\ud1a0\ub9ac\uc5bc\uc744 \ubcf8 \ub2e4\uc74c, \uacf5\uc911\ub0a0\uae30\ub97c \uc774\uc6a9\ud558\uc5ec \ud64d\ub828\uc12c\uc73c\ub85c \uc774\ub3d9\ud55c \ud6c4, \ub4a4\uc774\uc5b4 \ud30c\ub3c4\ud0c0\uae30\ub85c \ub3d9\ucabd \ud574\uc548\uc5d0 \ub3c4\ub2ec\ud558\uba74 \ubbf8\uc2f1\ub178\ub97c \ub9cc\ub098\uac8c \ub41c\ub2e4.", "sentence_answer": "\uac8c\uc784 \ud504\ub9ac\ud06c\uac00 \uac1c\ubc1c\ud55c \ud3ec\ucf13\ubaac \uc2dc\ub9ac\uc988\ub294 1996\ub144 \ub2cc\ud150\ub3c4 \uc5d0 \uc758\ud574 \uc77c\ubcf8\uc5d0\uc11c \uccab \ucd9c\uc2dc\ub418\uc5c8\ub2e4.", "paragraph_id": "6505638-0-1", "paragraph_question": "question: \ud3ec\ucf13\ubaac \uc2dc\ub9ac\uc988\ub97c \uc77c\ubcf8\uc5d0 \ucc98\uc74c\uc73c\ub85c \ucd9c\uc2dc\ud55c \uacf3\uc740 \uc5b4\ub514\uc778\uac00\uc694?, context: \uac8c\uc784 \ud504\ub9ac\ud06c\uac00 \uac1c\ubc1c\ud55c \ud3ec\ucf13\ubaac \uc2dc\ub9ac\uc988\ub294 1996\ub144 \ub2cc\ud150\ub3c4\uc5d0 \uc758\ud574 \uc77c\ubcf8\uc5d0\uc11c \uccab \ucd9c\uc2dc\ub418\uc5c8\ub2e4. \ud50c\ub808\uc774\uc5b4\ub294 \ud3ec\ucf13\ubaac \ud2b8\ub808\uc774\ub108 \uc5ed\ud560\uc744 \ub9e1\uc544 \ud3ec\ucf13\ubaac\uc744 \ud3ec\ud68d\ud558\uc5ec \ud6c8\ub828\uc2dc\ud0a4\uace0, \uac01 \ud3ec\ucf13\ubaac\ub4e4\uc758 \uace0\uc720 \uae30\uc220\uc744 \uc774\uc6a9\ud558\uc5ec \uc11c\ub85c \ub300\uc804\uc2dc\ud0a8\ub2e4. \ub2cc\ud150\ub3c4 \uce21\uc740 \u300a\ub2cc\ud150\ub3c4 \ud30c\uc6cc\u300b 1999\ub144 5\uc6d4\ud638\uc5d0\uc11c \ubbf8\uc2f1\ub178\uc758 \ucd9c\ud604\uc5d0 \ub300\ud574, \"\ubbf8\uc2f1\ub178\uc640 \uc811\ucd09\ud560 \uacbd\uc6b0, \ubbf8\uc2f1\ub178\ub97c \ud3ec\ud68d\ud558\uc9c0 \uc54a\ub354\ub77c\ub3c4 \uac8c\uc784 \ud30c\uc77c\uc774 \uc0ad\uc81c\ub420 \uc218 \uc788\uc73c\uba70, \uadf8\ub798\ud53d \uae68\uc9d0 \ud604\uc0c1\uc774 \ubc1c\uc0dd\ud560 \uc218 \uc788\ub2e4\u201d\ub294 \uacbd\uace0\uc640 \ud568\uaed8 \ucc98\uc74c\uc73c\ub85c \uacf5\uc2dd\uc801\uc73c\ub85c \uc5b8\uae09\ud558\uc600\ub2e4. \ubbf8\uc2f1\ub178\ub294 \ud2b9\uc815 \uc774\ubca4\ud2b8\ub4e4\uc774 \uc5f0\uc18d\uc801\uc73c\ub85c \uc2e4\ud589\ub418\uc5c8\uc744 \ub54c \ub098\ud0c0\ub098\uac8c \ub41c\ub2e4. \uc989, \ud50c\ub808\uc774\uc5b4\uac00 \ud3ec\ucf13\ubaac \ud3ec\ud68d\uc744 \uc704\ud55c \ud29c\ud1a0\ub9ac\uc5bc\uc744 \ubcf8 \ub2e4\uc74c, \uacf5\uc911\ub0a0\uae30\ub97c \uc774\uc6a9\ud558\uc5ec \ud64d\ub828\uc12c\uc73c\ub85c \uc774\ub3d9\ud55c \ud6c4, \ub4a4\uc774\uc5b4 \ud30c\ub3c4\ud0c0\uae30\ub85c \ub3d9\ucabd \ud574\uc548\uc5d0 \ub3c4\ub2ec\ud558\uba74 \ubbf8\uc2f1\ub178\ub97c \ub9cc\ub098\uac8c \ub41c\ub2e4."} {"question": "\ub18d\ub178\ub4e4\uc774 \ub354 \uc774\uc0c1\uc740 \uc601\uc8fc\ub97c \uc704\ud574 \uc0b4\uc544\uac08 \ud544\uc694\uac00 \uc5c6\ub2e4\uace0 \uc8fc\uc7a5\ud588\ub358 \ucc45\uc740?", "paragraph": "\ubd88\ub9cc\uc774 \uc313\uc774\uc790 \uc800\ud56d\uc758 \uae38\uc774 \uc5f4\ub838\ub2e4. 1377\ub144, \uc789\uae00\ub79c\ub4dc \ub3d9\ub0a8\ubd80\uc640 \uc11c\ub0a8\ubd80\uc5d0\uc11c \"\uc5c4\uccad\ub09c \uc720\uc5b8\ube44\uc5b4\"(Great Rumour)\uac00 \ubc1c\uc0dd\ud588\ub2e4. \uc804\uc6d0\uc9c0\ub300\uc758 \uc77c\uafbc\ub4e4\uc740 \uc9d1\ub2e8\uc744 \uc870\uc9c1\ud558\uc5ec \uc601\uc8fc\ub4e4\uc744 \uc704\ud574 \uc77c\ud560 \uac83\uc744 \uac70\ubd80\ud588\uace0, \u300a\ub460\uc2a4\ub370\uc774 \ubd81\u300b\uc5d0 \ub530\ub974\uba74 \uc790\uc2e0\ub4e4\uc740 \uc601\uc8fc\ub4e4\uc758 \uc694\uad6c\ub97c \uba74\uc81c\ubc1b\uc744 \uc218 \uc788\ub2e4\uace0 \uc8fc\uc7a5\ud588\ub2e4. \ud558\uc9c0\ub9cc \uc7ac\ud310\uc18c\uc640 \uad6d\uc655\uc5d0 \ub300\ud55c \uadf8\ub4e4\uc758 \ud638\uc18c\ub294 \uc131\uacf5\uc801\uc774\uc9c0 \ubabb\ud588\ub2e4. \ub3c4\uc2dc \uc9c0\uc5ed, \ud2b9\ud788 \ub7f0\ub358\uc5d0\uc11c\ub3c4 \uad11\ubc94\uc704\ud55c \uae34\uc7a5 \uc0c1\ud0dc\uac00 \uc720\uc9c0\ub418\uc5c8\ub2e4. \uace4\ud2b8\uc758 \uc874\uc740 \ub9b0\uce58\ub97c \ub2f9\ud560 \ubed4 \ud588\uc73c\ub098 \uaca8\uc6b0\uaca8\uc6b0 \ub7f0\ub358\uc744 \ube60\uc838\ub098\uac14\ub2e4. \uc7c1\uc758\ub294 1380\ub144\uc5d0 \uc7ac\ucc28 \uc99d\uac00\ud558\uc5ec, \uc800\ud56d\uacfc \uc18c\ub780\uc774 \uc789\uae00\ub79c\ub4dc \ubd81\ubd80\uc640 \uc11c\ubd80\uc758 \uc74d\ub0b4\ub4e4(\uc288\ub8e8\uc988\ubc84\ub9ac, \ube0c\ub9ac\uc9c0\uc6cc\ud130 \ub4f1)\uc744 \ud729\uc4f8\uc5c8\ub2e4. \uc694\ud06c\uc5d0\uc11c\ub294 \uc2dc\uc7a5 \uae30\uc988\ubc88\uc758 \uc874(John de Gisborne)\uc774 \ud574\uc784\ub418\uc790 \ubd09\uae30\uac00 \uc77c\uc5b4\ub0ac\uace0, 1381\ub144 \ucd08\uc5d0\ub294 \uc138\uae08 \ud3ed\ub3d9\uc774 \ub4a4\ub530\ub790\ub2e4. 1381\ub144 5\uc6d4, \ub300\ud3ed\ud48d\uc6b0\uac00 \uc789\uae00\ub79c\ub4dc\ub97c \ub36e\ucce4\ub2e4. \ub9ce\uc740 \uc774\ub4e4\uc774 \ubbf8\ub798\uc5d0 \ub300\uaca9\ubcc0\uc774 \uc77c\uc5b4\ub0a0 \uac83\uc774\ub77c\uace0 \ub290\uaf08\uace0, \ub3d9\uc694\ud558\ub294 \ubd84\uc704\uae30\ub294 \ub354\uc6b1 \uac00\uc911\ub418\uc5c8\ub2e4.", "answer": "\ub460\uc2a4\ub370\uc774 \ubd81", "sentence": "\u300a \ub460\uc2a4\ub370\uc774 \ubd81 \u300b\uc5d0 \ub530\ub974\uba74 \uc790\uc2e0\ub4e4\uc740 \uc601\uc8fc\ub4e4\uc758 \uc694\uad6c\ub97c \uba74\uc81c\ubc1b\uc744 \uc218 \uc788\ub2e4\uace0 \uc8fc\uc7a5\ud588\ub2e4.", "paragraph_sentence": "\ubd88\ub9cc\uc774 \uc313\uc774\uc790 \uc800\ud56d\uc758 \uae38\uc774 \uc5f4\ub838\ub2e4. 1377\ub144, \uc789\uae00\ub79c\ub4dc \ub3d9\ub0a8\ubd80\uc640 \uc11c\ub0a8\ubd80\uc5d0\uc11c \"\uc5c4\uccad\ub09c \uc720\uc5b8\ube44\uc5b4\"(Great Rumour)\uac00 \ubc1c\uc0dd\ud588\ub2e4. \uc804\uc6d0\uc9c0\ub300\uc758 \uc77c\uafbc\ub4e4\uc740 \uc9d1\ub2e8\uc744 \uc870\uc9c1\ud558\uc5ec \uc601\uc8fc\ub4e4\uc744 \uc704\ud574 \uc77c\ud560 \uac83\uc744 \uac70\ubd80\ud588\uace0, \u300a \ub460\uc2a4\ub370\uc774 \ubd81 \u300b\uc5d0 \ub530\ub974\uba74 \uc790\uc2e0\ub4e4\uc740 \uc601\uc8fc\ub4e4\uc758 \uc694\uad6c\ub97c \uba74\uc81c\ubc1b\uc744 \uc218 \uc788\ub2e4\uace0 \uc8fc\uc7a5\ud588\ub2e4. \ud558\uc9c0\ub9cc \uc7ac\ud310\uc18c\uc640 \uad6d\uc655\uc5d0 \ub300\ud55c \uadf8\ub4e4\uc758 \ud638\uc18c\ub294 \uc131\uacf5\uc801\uc774\uc9c0 \ubabb\ud588\ub2e4. \ub3c4\uc2dc \uc9c0\uc5ed, \ud2b9\ud788 \ub7f0\ub358\uc5d0\uc11c\ub3c4 \uad11\ubc94\uc704\ud55c \uae34\uc7a5 \uc0c1\ud0dc\uac00 \uc720\uc9c0\ub418\uc5c8\ub2e4. \uace4\ud2b8\uc758 \uc874\uc740 \ub9b0\uce58\ub97c \ub2f9\ud560 \ubed4 \ud588\uc73c\ub098 \uaca8\uc6b0\uaca8\uc6b0 \ub7f0\ub358\uc744 \ube60\uc838\ub098\uac14\ub2e4. \uc7c1\uc758\ub294 1380\ub144\uc5d0 \uc7ac\ucc28 \uc99d\uac00\ud558\uc5ec, \uc800\ud56d\uacfc \uc18c\ub780\uc774 \uc789\uae00\ub79c\ub4dc \ubd81\ubd80\uc640 \uc11c\ubd80\uc758 \uc74d\ub0b4\ub4e4(\uc288\ub8e8\uc988\ubc84\ub9ac, \ube0c\ub9ac\uc9c0\uc6cc\ud130 \ub4f1)\uc744 \ud729\uc4f8\uc5c8\ub2e4. \uc694\ud06c\uc5d0\uc11c\ub294 \uc2dc\uc7a5 \uae30\uc988\ubc88\uc758 \uc874(John de Gisborne)\uc774 \ud574\uc784\ub418\uc790 \ubd09\uae30\uac00 \uc77c\uc5b4\ub0ac\uace0, 1381\ub144 \ucd08\uc5d0\ub294 \uc138\uae08 \ud3ed\ub3d9\uc774 \ub4a4\ub530\ub790\ub2e4. 1381\ub144 5\uc6d4, \ub300\ud3ed\ud48d\uc6b0\uac00 \uc789\uae00\ub79c\ub4dc\ub97c \ub36e\ucce4\ub2e4. \ub9ce\uc740 \uc774\ub4e4\uc774 \ubbf8\ub798\uc5d0 \ub300\uaca9\ubcc0\uc774 \uc77c\uc5b4\ub0a0 \uac83\uc774\ub77c\uace0 \ub290\uaf08\uace0, \ub3d9\uc694\ud558\ub294 \ubd84\uc704\uae30\ub294 \ub354\uc6b1 \uac00\uc911\ub418\uc5c8\ub2e4.", "paragraph_answer": "\ubd88\ub9cc\uc774 \uc313\uc774\uc790 \uc800\ud56d\uc758 \uae38\uc774 \uc5f4\ub838\ub2e4. 1377\ub144, \uc789\uae00\ub79c\ub4dc \ub3d9\ub0a8\ubd80\uc640 \uc11c\ub0a8\ubd80\uc5d0\uc11c \"\uc5c4\uccad\ub09c \uc720\uc5b8\ube44\uc5b4\"(Great Rumour)\uac00 \ubc1c\uc0dd\ud588\ub2e4. \uc804\uc6d0\uc9c0\ub300\uc758 \uc77c\uafbc\ub4e4\uc740 \uc9d1\ub2e8\uc744 \uc870\uc9c1\ud558\uc5ec \uc601\uc8fc\ub4e4\uc744 \uc704\ud574 \uc77c\ud560 \uac83\uc744 \uac70\ubd80\ud588\uace0, \u300a \ub460\uc2a4\ub370\uc774 \ubd81 \u300b\uc5d0 \ub530\ub974\uba74 \uc790\uc2e0\ub4e4\uc740 \uc601\uc8fc\ub4e4\uc758 \uc694\uad6c\ub97c \uba74\uc81c\ubc1b\uc744 \uc218 \uc788\ub2e4\uace0 \uc8fc\uc7a5\ud588\ub2e4. \ud558\uc9c0\ub9cc \uc7ac\ud310\uc18c\uc640 \uad6d\uc655\uc5d0 \ub300\ud55c \uadf8\ub4e4\uc758 \ud638\uc18c\ub294 \uc131\uacf5\uc801\uc774\uc9c0 \ubabb\ud588\ub2e4. \ub3c4\uc2dc \uc9c0\uc5ed, \ud2b9\ud788 \ub7f0\ub358\uc5d0\uc11c\ub3c4 \uad11\ubc94\uc704\ud55c \uae34\uc7a5 \uc0c1\ud0dc\uac00 \uc720\uc9c0\ub418\uc5c8\ub2e4. \uace4\ud2b8\uc758 \uc874\uc740 \ub9b0\uce58\ub97c \ub2f9\ud560 \ubed4 \ud588\uc73c\ub098 \uaca8\uc6b0\uaca8\uc6b0 \ub7f0\ub358\uc744 \ube60\uc838\ub098\uac14\ub2e4. \uc7c1\uc758\ub294 1380\ub144\uc5d0 \uc7ac\ucc28 \uc99d\uac00\ud558\uc5ec, \uc800\ud56d\uacfc \uc18c\ub780\uc774 \uc789\uae00\ub79c\ub4dc \ubd81\ubd80\uc640 \uc11c\ubd80\uc758 \uc74d\ub0b4\ub4e4(\uc288\ub8e8\uc988\ubc84\ub9ac, \ube0c\ub9ac\uc9c0\uc6cc\ud130 \ub4f1)\uc744 \ud729\uc4f8\uc5c8\ub2e4. \uc694\ud06c\uc5d0\uc11c\ub294 \uc2dc\uc7a5 \uae30\uc988\ubc88\uc758 \uc874(John de Gisborne)\uc774 \ud574\uc784\ub418\uc790 \ubd09\uae30\uac00 \uc77c\uc5b4\ub0ac\uace0, 1381\ub144 \ucd08\uc5d0\ub294 \uc138\uae08 \ud3ed\ub3d9\uc774 \ub4a4\ub530\ub790\ub2e4. 1381\ub144 5\uc6d4, \ub300\ud3ed\ud48d\uc6b0\uac00 \uc789\uae00\ub79c\ub4dc\ub97c \ub36e\ucce4\ub2e4. \ub9ce\uc740 \uc774\ub4e4\uc774 \ubbf8\ub798\uc5d0 \ub300\uaca9\ubcc0\uc774 \uc77c\uc5b4\ub0a0 \uac83\uc774\ub77c\uace0 \ub290\uaf08\uace0, \ub3d9\uc694\ud558\ub294 \ubd84\uc704\uae30\ub294 \ub354\uc6b1 \uac00\uc911\ub418\uc5c8\ub2e4.", "sentence_answer": "\u300a \ub460\uc2a4\ub370\uc774 \ubd81 \u300b\uc5d0 \ub530\ub974\uba74 \uc790\uc2e0\ub4e4\uc740 \uc601\uc8fc\ub4e4\uc758 \uc694\uad6c\ub97c \uba74\uc81c\ubc1b\uc744 \uc218 \uc788\ub2e4\uace0 \uc8fc\uc7a5\ud588\ub2e4.", "paragraph_id": "6541489-14-0", "paragraph_question": "question: \ub18d\ub178\ub4e4\uc774 \ub354 \uc774\uc0c1\uc740 \uc601\uc8fc\ub97c \uc704\ud574 \uc0b4\uc544\uac08 \ud544\uc694\uac00 \uc5c6\ub2e4\uace0 \uc8fc\uc7a5\ud588\ub358 \ucc45\uc740?, context: \ubd88\ub9cc\uc774 \uc313\uc774\uc790 \uc800\ud56d\uc758 \uae38\uc774 \uc5f4\ub838\ub2e4. 1377\ub144, \uc789\uae00\ub79c\ub4dc \ub3d9\ub0a8\ubd80\uc640 \uc11c\ub0a8\ubd80\uc5d0\uc11c \"\uc5c4\uccad\ub09c \uc720\uc5b8\ube44\uc5b4\"(Great Rumour)\uac00 \ubc1c\uc0dd\ud588\ub2e4. \uc804\uc6d0\uc9c0\ub300\uc758 \uc77c\uafbc\ub4e4\uc740 \uc9d1\ub2e8\uc744 \uc870\uc9c1\ud558\uc5ec \uc601\uc8fc\ub4e4\uc744 \uc704\ud574 \uc77c\ud560 \uac83\uc744 \uac70\ubd80\ud588\uace0, \u300a\ub460\uc2a4\ub370\uc774 \ubd81\u300b\uc5d0 \ub530\ub974\uba74 \uc790\uc2e0\ub4e4\uc740 \uc601\uc8fc\ub4e4\uc758 \uc694\uad6c\ub97c \uba74\uc81c\ubc1b\uc744 \uc218 \uc788\ub2e4\uace0 \uc8fc\uc7a5\ud588\ub2e4. \ud558\uc9c0\ub9cc \uc7ac\ud310\uc18c\uc640 \uad6d\uc655\uc5d0 \ub300\ud55c \uadf8\ub4e4\uc758 \ud638\uc18c\ub294 \uc131\uacf5\uc801\uc774\uc9c0 \ubabb\ud588\ub2e4. \ub3c4\uc2dc \uc9c0\uc5ed, \ud2b9\ud788 \ub7f0\ub358\uc5d0\uc11c\ub3c4 \uad11\ubc94\uc704\ud55c \uae34\uc7a5 \uc0c1\ud0dc\uac00 \uc720\uc9c0\ub418\uc5c8\ub2e4. \uace4\ud2b8\uc758 \uc874\uc740 \ub9b0\uce58\ub97c \ub2f9\ud560 \ubed4 \ud588\uc73c\ub098 \uaca8\uc6b0\uaca8\uc6b0 \ub7f0\ub358\uc744 \ube60\uc838\ub098\uac14\ub2e4. \uc7c1\uc758\ub294 1380\ub144\uc5d0 \uc7ac\ucc28 \uc99d\uac00\ud558\uc5ec, \uc800\ud56d\uacfc \uc18c\ub780\uc774 \uc789\uae00\ub79c\ub4dc \ubd81\ubd80\uc640 \uc11c\ubd80\uc758 \uc74d\ub0b4\ub4e4(\uc288\ub8e8\uc988\ubc84\ub9ac, \ube0c\ub9ac\uc9c0\uc6cc\ud130 \ub4f1)\uc744 \ud729\uc4f8\uc5c8\ub2e4. \uc694\ud06c\uc5d0\uc11c\ub294 \uc2dc\uc7a5 \uae30\uc988\ubc88\uc758 \uc874(John de Gisborne)\uc774 \ud574\uc784\ub418\uc790 \ubd09\uae30\uac00 \uc77c\uc5b4\ub0ac\uace0, 1381\ub144 \ucd08\uc5d0\ub294 \uc138\uae08 \ud3ed\ub3d9\uc774 \ub4a4\ub530\ub790\ub2e4. 1381\ub144 5\uc6d4, \ub300\ud3ed\ud48d\uc6b0\uac00 \uc789\uae00\ub79c\ub4dc\ub97c \ub36e\ucce4\ub2e4. \ub9ce\uc740 \uc774\ub4e4\uc774 \ubbf8\ub798\uc5d0 \ub300\uaca9\ubcc0\uc774 \uc77c\uc5b4\ub0a0 \uac83\uc774\ub77c\uace0 \ub290\uaf08\uace0, \ub3d9\uc694\ud558\ub294 \ubd84\uc704\uae30\ub294 \ub354\uc6b1 \uac00\uc911\ub418\uc5c8\ub2e4."} {"question": "\uc815\uc2e0\uc774 \ub098\uac04 \uc124\uc815\uc758 \ubb34\uc7a5 \uc911\ub9bd \ubbfc\uac04\uc778 NPC \uce90\ub9ad\ud130\ub294 \ub204\uad6c\uc778\uac00?", "paragraph": "\ubb34\ud574\ud55c \ubbf8\uce58\uad11\uc774: \uc815\uc2e0\uc774 \ub098\uac04 \uc124\uc815\uc758 \ubb34\uc7a5 \uc911\ub9bd \ubbfc\uac04\uc778 NPC 1\uba85\uc774 \uaf2d\ub300\uae30\uc5d0 \uac70\uc8fc\ud558\uace0 \uc788\uc73c\uba70 \uadf8 \uc0ac\uc774\uc758 \uae38\ub4e4\uc744 \ub300\ub7c9\uc758 \uc794\ud574\ub354\ubbf8\uac00 \ub9c9\uace0 \uc788\ub2e4. \uc794\ud574\ub97c \uc815\ub9ac\ud558\ub294\ub370 \uc2dc\uac04\uc774 \ube44\uad50\uc801 \ub9ce\uc774 \uc18c\uc694\ub41c\ub2e4. \uc0ac\uc720\uc7ac\uc0b0\uc5d0 \uc811\uadfc\ud560 \uacbd\uc6b0 \uacbd\uace0\ub294 \ud558\uc9c0\ub9cc \uc801\ub300\ud558\uc9c0\ub294 \uc54a\uae30 \ub54c\ubb38\uc5d0 \uc57d\ud0c8 \ub09c\uc774\ub3c4\ub294 \uad49\uc7a5\ud788 \ub0ae\uc740 \ud3b8\uc774\ub2e4. \ud558\uc9c0\ub9cc \ubbfc\uac04\uc778\uc758 \uc0ac\uc720\uc7ac\uc0b0\uc744 \uc57d\ud0c8\ud55c \uc8c4\ub85c \uc0ac\uae30\uac00 \uc57d\uac04 \ub5a8\uc5b4\uc9c4\ub2e4. 4\uce35 \uc624\ub978\ud3b8 \uce90\ube44\ub137\uc5d0 \uc0b0\ud0c4\ucd1d\ub3c4 \uc788\ub2e4. \uc8fc\ubbfc \uc5ed\uc2dc \uc0b0\ud0c4\ucd1d\uc73c\ub85c \ubb34\uc7a5\ud558\uace0 \uc788\uc5b4 \uc554\uc0b4 \ub4f1\uc744 \ud1b5\ud574 \uc8fc\ubbfc\uc758 \ucd1d\uae4c\uc9c0 \uc5bb\uc744 \uc218 \uc788\ub2e4. \ubb3c\ub860 \ubbfc\uac04\uc778 NPC\uc774\ubbc0\ub85c \uc0ac\uae30\uac00 \ud558\ub77d\ud55c\ub2e4. \uac74\ubb3c \uc548\uc758 \uba54\ubaa8\ub97c \ud655\uc778\ud558\uba74 \uac74\ubb3c \uc624\ub978\ucabd \ubb38 \ubc14\uae65\uc5d0 \ubd80\uc11c\uc9c4 \uc0b0\ud0c4\ucd1d, \uba40\uca61\ud55c \ubc29\ud0c4\ubaa8, \ubc29\ud0c4\ubcf5 \ub4f1 \uc228\uaca8\uc9c4 \ubb3c\ud488\ub4e4\uc774 \ub098\ud0c0\ub09c\ub2e4.", "answer": "\ubb34\ud574\ud55c \ubbf8\uce58\uad11\uc774", "sentence": "\ubb34\ud574\ud55c \ubbf8\uce58\uad11\uc774 :", "paragraph_sentence": " \ubb34\ud574\ud55c \ubbf8\uce58\uad11\uc774 : \uc815\uc2e0\uc774 \ub098\uac04 \uc124\uc815\uc758 \ubb34\uc7a5 \uc911\ub9bd \ubbfc\uac04\uc778 NPC 1\uba85\uc774 \uaf2d\ub300\uae30\uc5d0 \uac70\uc8fc\ud558\uace0 \uc788\uc73c\uba70 \uadf8 \uc0ac\uc774\uc758 \uae38\ub4e4\uc744 \ub300\ub7c9\uc758 \uc794\ud574\ub354\ubbf8\uac00 \ub9c9\uace0 \uc788\ub2e4. \uc794\ud574\ub97c \uc815\ub9ac\ud558\ub294\ub370 \uc2dc\uac04\uc774 \ube44\uad50\uc801 \ub9ce\uc774 \uc18c\uc694\ub41c\ub2e4. \uc0ac\uc720\uc7ac\uc0b0\uc5d0 \uc811\uadfc\ud560 \uacbd\uc6b0 \uacbd\uace0\ub294 \ud558\uc9c0\ub9cc \uc801\ub300\ud558\uc9c0\ub294 \uc54a\uae30 \ub54c\ubb38\uc5d0 \uc57d\ud0c8 \ub09c\uc774\ub3c4\ub294 \uad49\uc7a5\ud788 \ub0ae\uc740 \ud3b8\uc774\ub2e4. \ud558\uc9c0\ub9cc \ubbfc\uac04\uc778\uc758 \uc0ac\uc720\uc7ac\uc0b0\uc744 \uc57d\ud0c8\ud55c \uc8c4\ub85c \uc0ac\uae30\uac00 \uc57d\uac04 \ub5a8\uc5b4\uc9c4\ub2e4. 4\uce35 \uc624\ub978\ud3b8 \uce90\ube44\ub137\uc5d0 \uc0b0\ud0c4\ucd1d\ub3c4 \uc788\ub2e4. \uc8fc\ubbfc \uc5ed\uc2dc \uc0b0\ud0c4\ucd1d\uc73c\ub85c \ubb34\uc7a5\ud558\uace0 \uc788\uc5b4 \uc554\uc0b4 \ub4f1\uc744 \ud1b5\ud574 \uc8fc\ubbfc\uc758 \ucd1d\uae4c\uc9c0 \uc5bb\uc744 \uc218 \uc788\ub2e4. \ubb3c\ub860 \ubbfc\uac04\uc778 NPC\uc774\ubbc0\ub85c \uc0ac\uae30\uac00 \ud558\ub77d\ud55c\ub2e4. \uac74\ubb3c \uc548\uc758 \uba54\ubaa8\ub97c \ud655\uc778\ud558\uba74 \uac74\ubb3c \uc624\ub978\ucabd \ubb38 \ubc14\uae65\uc5d0 \ubd80\uc11c\uc9c4 \uc0b0\ud0c4\ucd1d, \uba40\uca61\ud55c \ubc29\ud0c4\ubaa8, \ubc29\ud0c4\ubcf5 \ub4f1 \uc228\uaca8\uc9c4 \ubb3c\ud488\ub4e4\uc774 \ub098\ud0c0\ub09c\ub2e4.", "paragraph_answer": " \ubb34\ud574\ud55c \ubbf8\uce58\uad11\uc774 : \uc815\uc2e0\uc774 \ub098\uac04 \uc124\uc815\uc758 \ubb34\uc7a5 \uc911\ub9bd \ubbfc\uac04\uc778 NPC 1\uba85\uc774 \uaf2d\ub300\uae30\uc5d0 \uac70\uc8fc\ud558\uace0 \uc788\uc73c\uba70 \uadf8 \uc0ac\uc774\uc758 \uae38\ub4e4\uc744 \ub300\ub7c9\uc758 \uc794\ud574\ub354\ubbf8\uac00 \ub9c9\uace0 \uc788\ub2e4. \uc794\ud574\ub97c \uc815\ub9ac\ud558\ub294\ub370 \uc2dc\uac04\uc774 \ube44\uad50\uc801 \ub9ce\uc774 \uc18c\uc694\ub41c\ub2e4. \uc0ac\uc720\uc7ac\uc0b0\uc5d0 \uc811\uadfc\ud560 \uacbd\uc6b0 \uacbd\uace0\ub294 \ud558\uc9c0\ub9cc \uc801\ub300\ud558\uc9c0\ub294 \uc54a\uae30 \ub54c\ubb38\uc5d0 \uc57d\ud0c8 \ub09c\uc774\ub3c4\ub294 \uad49\uc7a5\ud788 \ub0ae\uc740 \ud3b8\uc774\ub2e4. \ud558\uc9c0\ub9cc \ubbfc\uac04\uc778\uc758 \uc0ac\uc720\uc7ac\uc0b0\uc744 \uc57d\ud0c8\ud55c \uc8c4\ub85c \uc0ac\uae30\uac00 \uc57d\uac04 \ub5a8\uc5b4\uc9c4\ub2e4. 4\uce35 \uc624\ub978\ud3b8 \uce90\ube44\ub137\uc5d0 \uc0b0\ud0c4\ucd1d\ub3c4 \uc788\ub2e4. \uc8fc\ubbfc \uc5ed\uc2dc \uc0b0\ud0c4\ucd1d\uc73c\ub85c \ubb34\uc7a5\ud558\uace0 \uc788\uc5b4 \uc554\uc0b4 \ub4f1\uc744 \ud1b5\ud574 \uc8fc\ubbfc\uc758 \ucd1d\uae4c\uc9c0 \uc5bb\uc744 \uc218 \uc788\ub2e4. \ubb3c\ub860 \ubbfc\uac04\uc778 NPC\uc774\ubbc0\ub85c \uc0ac\uae30\uac00 \ud558\ub77d\ud55c\ub2e4. \uac74\ubb3c \uc548\uc758 \uba54\ubaa8\ub97c \ud655\uc778\ud558\uba74 \uac74\ubb3c \uc624\ub978\ucabd \ubb38 \ubc14\uae65\uc5d0 \ubd80\uc11c\uc9c4 \uc0b0\ud0c4\ucd1d, \uba40\uca61\ud55c \ubc29\ud0c4\ubaa8, \ubc29\ud0c4\ubcf5 \ub4f1 \uc228\uaca8\uc9c4 \ubb3c\ud488\ub4e4\uc774 \ub098\ud0c0\ub09c\ub2e4.", "sentence_answer": " \ubb34\ud574\ud55c \ubbf8\uce58\uad11\uc774 :", "paragraph_id": "6528791-24-0", "paragraph_question": "question: \uc815\uc2e0\uc774 \ub098\uac04 \uc124\uc815\uc758 \ubb34\uc7a5 \uc911\ub9bd \ubbfc\uac04\uc778 NPC \uce90\ub9ad\ud130\ub294 \ub204\uad6c\uc778\uac00?, context: \ubb34\ud574\ud55c \ubbf8\uce58\uad11\uc774: \uc815\uc2e0\uc774 \ub098\uac04 \uc124\uc815\uc758 \ubb34\uc7a5 \uc911\ub9bd \ubbfc\uac04\uc778 NPC 1\uba85\uc774 \uaf2d\ub300\uae30\uc5d0 \uac70\uc8fc\ud558\uace0 \uc788\uc73c\uba70 \uadf8 \uc0ac\uc774\uc758 \uae38\ub4e4\uc744 \ub300\ub7c9\uc758 \uc794\ud574\ub354\ubbf8\uac00 \ub9c9\uace0 \uc788\ub2e4. \uc794\ud574\ub97c \uc815\ub9ac\ud558\ub294\ub370 \uc2dc\uac04\uc774 \ube44\uad50\uc801 \ub9ce\uc774 \uc18c\uc694\ub41c\ub2e4. \uc0ac\uc720\uc7ac\uc0b0\uc5d0 \uc811\uadfc\ud560 \uacbd\uc6b0 \uacbd\uace0\ub294 \ud558\uc9c0\ub9cc \uc801\ub300\ud558\uc9c0\ub294 \uc54a\uae30 \ub54c\ubb38\uc5d0 \uc57d\ud0c8 \ub09c\uc774\ub3c4\ub294 \uad49\uc7a5\ud788 \ub0ae\uc740 \ud3b8\uc774\ub2e4. \ud558\uc9c0\ub9cc \ubbfc\uac04\uc778\uc758 \uc0ac\uc720\uc7ac\uc0b0\uc744 \uc57d\ud0c8\ud55c \uc8c4\ub85c \uc0ac\uae30\uac00 \uc57d\uac04 \ub5a8\uc5b4\uc9c4\ub2e4. 4\uce35 \uc624\ub978\ud3b8 \uce90\ube44\ub137\uc5d0 \uc0b0\ud0c4\ucd1d\ub3c4 \uc788\ub2e4. \uc8fc\ubbfc \uc5ed\uc2dc \uc0b0\ud0c4\ucd1d\uc73c\ub85c \ubb34\uc7a5\ud558\uace0 \uc788\uc5b4 \uc554\uc0b4 \ub4f1\uc744 \ud1b5\ud574 \uc8fc\ubbfc\uc758 \ucd1d\uae4c\uc9c0 \uc5bb\uc744 \uc218 \uc788\ub2e4. \ubb3c\ub860 \ubbfc\uac04\uc778 NPC\uc774\ubbc0\ub85c \uc0ac\uae30\uac00 \ud558\ub77d\ud55c\ub2e4. \uac74\ubb3c \uc548\uc758 \uba54\ubaa8\ub97c \ud655\uc778\ud558\uba74 \uac74\ubb3c \uc624\ub978\ucabd \ubb38 \ubc14\uae65\uc5d0 \ubd80\uc11c\uc9c4 \uc0b0\ud0c4\ucd1d, \uba40\uca61\ud55c \ubc29\ud0c4\ubaa8, \ubc29\ud0c4\ubcf5 \ub4f1 \uc228\uaca8\uc9c4 \ubb3c\ud488\ub4e4\uc774 \ub098\ud0c0\ub09c\ub2e4."} {"question": "\uc7a0\ub4e4\uc5b4\uc788\ub294 \uc544\ub77c\uace4\uc758 \ubcd1\uc0ac\ub4e4\uc740 \uae68\uc6b4 \uc545\uae30\ub294?", "paragraph": "\ub8e8\uccb4\ub9ac\uc624\ub294 \uadf8\uc758 \ubcd1\uc0ac\ub4e4\uc5d0\uac8c \ubb34\uc7a5 \ubc0f \uc804\ud22c \uc900\ube44\ub97c \uba85\ub839\ud588\ub2e4. \uc544\ub77c\uace4 \ubcd1\uc0ac\ub4e4\uc740 \ube68\ub9ac \uc785\ud56d\ud574 \uae30\uc2b5 \uacf5\uaca9\uc744 \uac00\ud558\uace0\uc790 \ud588\uc73c\uba70, \"\uc6b0\ub9ac\ub4e4\uc5d0\uac8c \uacf5\uaca9\ud558\uac8c \ud574\ub2ec\ub77c. \uadf8\ub4e4\uc740 \ubaa8\ub450 \uc6b0\ub9ac \uac83\uc774\ub2e4.\"\ub77c\uace0 \uc678\ucce4\ub2e4. \ub8e8\uccb4\ub9ac\uc624\ub294 \uc544\ub77c\uace4 \ubcd1\uc0ac\ub4e4\uc774 \uc7a0\ub4e4\uc5b4 \uc788\ub294 \uc801\uc744 \ud328\ubc30\uc2dc\ucf30\ub2e4\ub294 \ub9d0\uc744 \ub4e3\uace0 \uc2f6\uc9c0 \uc54a\ub2e4\uace0 \ub9d0\ud558\uba70 \ud2b8\ub7fc\ud3ab\uc744 \ubd88\uc5b4 \ud56d\uad6c\uc5d0 \uc790\uc2e0\ub4e4\uc774 \ub3c4\ucc29\ud588\uc74c\uc744 \uc54c\ub9b4 \uac83\uc744 \uc694\uad6c\ud588\ub2e4. \ub8e8\uccb4\ub9ac\uc624\ub294 \ud2b8\ub7fc\ud3ab\uc744 \ubd84 \ud6c4 \uadf8\uc758 \uac24\ub9ac\uc120\ub4e4\uc758 \uc5f4\uc744 \ub9de\ucd98 \ud6c4 \uadf8\ub79c\ub4dc\ud558\ubc84\uc5d0 \uc785\uc131\ud574 \uacf5\uaca9\uc744 \uac1c\uc2dc\ud588\ub2e4. \ud504\ub85c\ubc29\uc2a4\uc778\ub4e4\uc740 \uc77c\uc5b4\ub09c \ud6c4 \ub8e8\uccb4\ub9ac\uc624\uac00 \ub178\ub97c \ub4e4\uace0 \uc788\uc74c\uc744 \ubc1c\uacac\ud588\ub2e4. 100\uc5ec \uba85\uc758 \ud504\ub791\uc2a4 \uadc0\uc871\ub4e4\uc774 \uc131\uc5d0\uc11c \ub0b4\ub824\uc640 \uac24\ub9ac\uc120\ub4e4\uc758 \ud56d\ud574\uc0ac\ub4e4\uc744 \uc9c0\uc6d0\ud588\ub2e4. \ucf54\ub974\ub258\ub3c4 \uadf8\uc758 \ud2b8\ub7fc\ud3ab\uc744 \ubd88\uc5b4 \ub8e8\uccb4\ub9ac\uc624\uc758 \ud568\ub300\uc5d0 \uadf8\uc758 \uac24\ub9ac\uc120\uc744 \uc9c4\uaca9\uc2dc\ud0a4\ub77c\uace0 \uba85\ub839\ud588\ub2e4.", "answer": "\ud2b8\ub7fc\ud3ab", "sentence": "\ub8e8\uccb4\ub9ac\uc624\ub294 \uc544\ub77c\uace4 \ubcd1\uc0ac\ub4e4\uc774 \uc7a0\ub4e4\uc5b4 \uc788\ub294 \uc801\uc744 \ud328\ubc30\uc2dc\ucf30\ub2e4\ub294 \ub9d0\uc744 \ub4e3\uace0 \uc2f6\uc9c0 \uc54a\ub2e4\uace0 \ub9d0\ud558\uba70 \ud2b8\ub7fc\ud3ab \uc744 \ubd88\uc5b4 \ud56d\uad6c\uc5d0 \uc790\uc2e0\ub4e4\uc774 \ub3c4\ucc29\ud588\uc74c\uc744 \uc54c\ub9b4 \uac83\uc744 \uc694\uad6c\ud588\ub2e4.", "paragraph_sentence": "\ub8e8\uccb4\ub9ac\uc624\ub294 \uadf8\uc758 \ubcd1\uc0ac\ub4e4\uc5d0\uac8c \ubb34\uc7a5 \ubc0f \uc804\ud22c \uc900\ube44\ub97c \uba85\ub839\ud588\ub2e4. \uc544\ub77c\uace4 \ubcd1\uc0ac\ub4e4\uc740 \ube68\ub9ac \uc785\ud56d\ud574 \uae30\uc2b5 \uacf5\uaca9\uc744 \uac00\ud558\uace0\uc790 \ud588\uc73c\uba70, \"\uc6b0\ub9ac\ub4e4\uc5d0\uac8c \uacf5\uaca9\ud558\uac8c \ud574\ub2ec\ub77c. \uadf8\ub4e4\uc740 \ubaa8\ub450 \uc6b0\ub9ac \uac83\uc774\ub2e4. \"\ub77c\uace0 \uc678\ucce4\ub2e4. \ub8e8\uccb4\ub9ac\uc624\ub294 \uc544\ub77c\uace4 \ubcd1\uc0ac\ub4e4\uc774 \uc7a0\ub4e4\uc5b4 \uc788\ub294 \uc801\uc744 \ud328\ubc30\uc2dc\ucf30\ub2e4\ub294 \ub9d0\uc744 \ub4e3\uace0 \uc2f6\uc9c0 \uc54a\ub2e4\uace0 \ub9d0\ud558\uba70 \ud2b8\ub7fc\ud3ab \uc744 \ubd88\uc5b4 \ud56d\uad6c\uc5d0 \uc790\uc2e0\ub4e4\uc774 \ub3c4\ucc29\ud588\uc74c\uc744 \uc54c\ub9b4 \uac83\uc744 \uc694\uad6c\ud588\ub2e4. \ub8e8\uccb4\ub9ac\uc624\ub294 \ud2b8\ub7fc\ud3ab\uc744 \ubd84 \ud6c4 \uadf8\uc758 \uac24\ub9ac\uc120\ub4e4\uc758 \uc5f4\uc744 \ub9de\ucd98 \ud6c4 \uadf8\ub79c\ub4dc\ud558\ubc84\uc5d0 \uc785\uc131\ud574 \uacf5\uaca9\uc744 \uac1c\uc2dc\ud588\ub2e4. \ud504\ub85c\ubc29\uc2a4\uc778\ub4e4\uc740 \uc77c\uc5b4\ub09c \ud6c4 \ub8e8\uccb4\ub9ac\uc624\uac00 \ub178\ub97c \ub4e4\uace0 \uc788\uc74c\uc744 \ubc1c\uacac\ud588\ub2e4. 100\uc5ec \uba85\uc758 \ud504\ub791\uc2a4 \uadc0\uc871\ub4e4\uc774 \uc131\uc5d0\uc11c \ub0b4\ub824\uc640 \uac24\ub9ac\uc120\ub4e4\uc758 \ud56d\ud574\uc0ac\ub4e4\uc744 \uc9c0\uc6d0\ud588\ub2e4. \ucf54\ub974\ub258\ub3c4 \uadf8\uc758 \ud2b8\ub7fc\ud3ab\uc744 \ubd88\uc5b4 \ub8e8\uccb4\ub9ac\uc624\uc758 \ud568\ub300\uc5d0 \uadf8\uc758 \uac24\ub9ac\uc120\uc744 \uc9c4\uaca9\uc2dc\ud0a4\ub77c\uace0 \uba85\ub839\ud588\ub2e4.", "paragraph_answer": "\ub8e8\uccb4\ub9ac\uc624\ub294 \uadf8\uc758 \ubcd1\uc0ac\ub4e4\uc5d0\uac8c \ubb34\uc7a5 \ubc0f \uc804\ud22c \uc900\ube44\ub97c \uba85\ub839\ud588\ub2e4. \uc544\ub77c\uace4 \ubcd1\uc0ac\ub4e4\uc740 \ube68\ub9ac \uc785\ud56d\ud574 \uae30\uc2b5 \uacf5\uaca9\uc744 \uac00\ud558\uace0\uc790 \ud588\uc73c\uba70, \"\uc6b0\ub9ac\ub4e4\uc5d0\uac8c \uacf5\uaca9\ud558\uac8c \ud574\ub2ec\ub77c. \uadf8\ub4e4\uc740 \ubaa8\ub450 \uc6b0\ub9ac \uac83\uc774\ub2e4.\"\ub77c\uace0 \uc678\ucce4\ub2e4. \ub8e8\uccb4\ub9ac\uc624\ub294 \uc544\ub77c\uace4 \ubcd1\uc0ac\ub4e4\uc774 \uc7a0\ub4e4\uc5b4 \uc788\ub294 \uc801\uc744 \ud328\ubc30\uc2dc\ucf30\ub2e4\ub294 \ub9d0\uc744 \ub4e3\uace0 \uc2f6\uc9c0 \uc54a\ub2e4\uace0 \ub9d0\ud558\uba70 \ud2b8\ub7fc\ud3ab \uc744 \ubd88\uc5b4 \ud56d\uad6c\uc5d0 \uc790\uc2e0\ub4e4\uc774 \ub3c4\ucc29\ud588\uc74c\uc744 \uc54c\ub9b4 \uac83\uc744 \uc694\uad6c\ud588\ub2e4. \ub8e8\uccb4\ub9ac\uc624\ub294 \ud2b8\ub7fc\ud3ab\uc744 \ubd84 \ud6c4 \uadf8\uc758 \uac24\ub9ac\uc120\ub4e4\uc758 \uc5f4\uc744 \ub9de\ucd98 \ud6c4 \uadf8\ub79c\ub4dc\ud558\ubc84\uc5d0 \uc785\uc131\ud574 \uacf5\uaca9\uc744 \uac1c\uc2dc\ud588\ub2e4. \ud504\ub85c\ubc29\uc2a4\uc778\ub4e4\uc740 \uc77c\uc5b4\ub09c \ud6c4 \ub8e8\uccb4\ub9ac\uc624\uac00 \ub178\ub97c \ub4e4\uace0 \uc788\uc74c\uc744 \ubc1c\uacac\ud588\ub2e4. 100\uc5ec \uba85\uc758 \ud504\ub791\uc2a4 \uadc0\uc871\ub4e4\uc774 \uc131\uc5d0\uc11c \ub0b4\ub824\uc640 \uac24\ub9ac\uc120\ub4e4\uc758 \ud56d\ud574\uc0ac\ub4e4\uc744 \uc9c0\uc6d0\ud588\ub2e4. \ucf54\ub974\ub258\ub3c4 \uadf8\uc758 \ud2b8\ub7fc\ud3ab\uc744 \ubd88\uc5b4 \ub8e8\uccb4\ub9ac\uc624\uc758 \ud568\ub300\uc5d0 \uadf8\uc758 \uac24\ub9ac\uc120\uc744 \uc9c4\uaca9\uc2dc\ud0a4\ub77c\uace0 \uba85\ub839\ud588\ub2e4.", "sentence_answer": "\ub8e8\uccb4\ub9ac\uc624\ub294 \uc544\ub77c\uace4 \ubcd1\uc0ac\ub4e4\uc774 \uc7a0\ub4e4\uc5b4 \uc788\ub294 \uc801\uc744 \ud328\ubc30\uc2dc\ucf30\ub2e4\ub294 \ub9d0\uc744 \ub4e3\uace0 \uc2f6\uc9c0 \uc54a\ub2e4\uace0 \ub9d0\ud558\uba70 \ud2b8\ub7fc\ud3ab \uc744 \ubd88\uc5b4 \ud56d\uad6c\uc5d0 \uc790\uc2e0\ub4e4\uc774 \ub3c4\ucc29\ud588\uc74c\uc744 \uc54c\ub9b4 \uac83\uc744 \uc694\uad6c\ud588\ub2e4.", "paragraph_id": "6465088-1-0", "paragraph_question": "question: \uc7a0\ub4e4\uc5b4\uc788\ub294 \uc544\ub77c\uace4\uc758 \ubcd1\uc0ac\ub4e4\uc740 \uae68\uc6b4 \uc545\uae30\ub294?, context: \ub8e8\uccb4\ub9ac\uc624\ub294 \uadf8\uc758 \ubcd1\uc0ac\ub4e4\uc5d0\uac8c \ubb34\uc7a5 \ubc0f \uc804\ud22c \uc900\ube44\ub97c \uba85\ub839\ud588\ub2e4. \uc544\ub77c\uace4 \ubcd1\uc0ac\ub4e4\uc740 \ube68\ub9ac \uc785\ud56d\ud574 \uae30\uc2b5 \uacf5\uaca9\uc744 \uac00\ud558\uace0\uc790 \ud588\uc73c\uba70, \"\uc6b0\ub9ac\ub4e4\uc5d0\uac8c \uacf5\uaca9\ud558\uac8c \ud574\ub2ec\ub77c. \uadf8\ub4e4\uc740 \ubaa8\ub450 \uc6b0\ub9ac \uac83\uc774\ub2e4.\"\ub77c\uace0 \uc678\ucce4\ub2e4. \ub8e8\uccb4\ub9ac\uc624\ub294 \uc544\ub77c\uace4 \ubcd1\uc0ac\ub4e4\uc774 \uc7a0\ub4e4\uc5b4 \uc788\ub294 \uc801\uc744 \ud328\ubc30\uc2dc\ucf30\ub2e4\ub294 \ub9d0\uc744 \ub4e3\uace0 \uc2f6\uc9c0 \uc54a\ub2e4\uace0 \ub9d0\ud558\uba70 \ud2b8\ub7fc\ud3ab\uc744 \ubd88\uc5b4 \ud56d\uad6c\uc5d0 \uc790\uc2e0\ub4e4\uc774 \ub3c4\ucc29\ud588\uc74c\uc744 \uc54c\ub9b4 \uac83\uc744 \uc694\uad6c\ud588\ub2e4. \ub8e8\uccb4\ub9ac\uc624\ub294 \ud2b8\ub7fc\ud3ab\uc744 \ubd84 \ud6c4 \uadf8\uc758 \uac24\ub9ac\uc120\ub4e4\uc758 \uc5f4\uc744 \ub9de\ucd98 \ud6c4 \uadf8\ub79c\ub4dc\ud558\ubc84\uc5d0 \uc785\uc131\ud574 \uacf5\uaca9\uc744 \uac1c\uc2dc\ud588\ub2e4. \ud504\ub85c\ubc29\uc2a4\uc778\ub4e4\uc740 \uc77c\uc5b4\ub09c \ud6c4 \ub8e8\uccb4\ub9ac\uc624\uac00 \ub178\ub97c \ub4e4\uace0 \uc788\uc74c\uc744 \ubc1c\uacac\ud588\ub2e4. 100\uc5ec \uba85\uc758 \ud504\ub791\uc2a4 \uadc0\uc871\ub4e4\uc774 \uc131\uc5d0\uc11c \ub0b4\ub824\uc640 \uac24\ub9ac\uc120\ub4e4\uc758 \ud56d\ud574\uc0ac\ub4e4\uc744 \uc9c0\uc6d0\ud588\ub2e4. \ucf54\ub974\ub258\ub3c4 \uadf8\uc758 \ud2b8\ub7fc\ud3ab\uc744 \ubd88\uc5b4 \ub8e8\uccb4\ub9ac\uc624\uc758 \ud568\ub300\uc5d0 \uadf8\uc758 \uac24\ub9ac\uc120\uc744 \uc9c4\uaca9\uc2dc\ud0a4\ub77c\uace0 \uba85\ub839\ud588\ub2e4."} {"question": "\ub9ac\uc9c0\uc6e8\uc774\ub294 \uc0c8\ub85c\uc6b4 \ubc29\uc5b4\uc120\uc744 \uc124\ub9bd\ud558\uae30 \uc704\ud574 \uba87\ub3c4\uc120 \uc774\ub0a8\uae4c\uc9c0 \ucca0\uc218\ud560 \uac83\uc744 \uc9c0\uc2dc\ud588\ub294\uac00?", "paragraph": "\uc911\uad6d\uad70\uacfc \ubd81\ud55c\uad70\uc758 \ucd08\uae30 \uacf5\uaca9\uc774 \uc131\uacf5\ud558\uc5ec \ub300\ud55c\ubbfc\uad6d \uc81c1\uad70\ub2e8\uc740 \uc6d0\ub798 \ubc29\uc5b4\uc9c0\uc810\uc774\uc5c8\ub358 \ud604\ub9ac\uc5d0\uc11c\uc758 \uc0c1\ud669 \ud68c\ubcf5 \uc2dc\ub3c4\ub97c \ud3ec\uae30\ud560 \uc218\ubc16\uc5d0 \uc5c6\uc5c8\ub2e4. \uc774\ud6c4 \ubd81\ud55c\uad70 \ubcd1\ub825 \ub2e4\uc218\uac00 \uc6d0\uc8fc\uc758 \ubbf8\uad6d \uc81c2\ubcf4\ubcd1\uc0ac\ub2e8\uacfc \ub3d9\ud574\uc548\uc758 \ub300\ud55c\ubbfc\uad6d \uc81c1\uad70\ub2e8 \uc0ac\uc774\uc5d0 \uc0dd\uae34 \ud2c8 \uc0ac\uc774\ub85c \ubc00\uace0 \ub4e4\uc5b4\uc654\ub2e4. \ub9ac\uc9c0\uc6e8\uc774\ub294 \uc911\ubd80 \uc804\uc120\uc5d0 \ub300\ud55c \uc870\uc911 \uc5f0\ud569\uad70\uc758 \uacf5\uaca9\uc774 \uc11c\uc6b8\uc758 \uc720\uc5d4\uad70 \ubc29\uc5b4\uc790\ub4e4\uc744 \ud3ec\uc704\ud558\uae30 \uc704\ud55c \uc2dc\ub3c4\ub77c\uace0 \ud574\uc11d\ud558\uace0 \uadf8\ub294 1\uc6d4 3\uc77c \uc11c\uc6b8 \uc8fc\ub454 \uc720\uc5d4\uad70\uc5d0\uac8c \uc989\uac01\uc801\uc778 \ucca0\uc218\ub97c \uba85\ub839\ud588\ub2e4. 1\uc6d4 5\uc77c, \ub9ac\uc9c0\uc6e8\uc774\ub294 D \ub77c\uc778\uc774\ub77c \uba85\uba85\ub41c \uc0c8\ub85c\uc6b4 \ubc29\uc5b4\uc120\uc744 \uc124\ub9bd\ud558\uae30 \uc704\ud574 \ubaa8\ub4e0 \uc720\uc5d4\uad70 \ubcd1\ub825\uc5d0\uac8c 37\ub3c4\uc120 \uc774\ub0a8\uc744 \ucca0\uc218\ud560 \uac83\uc744 \uc9c0\uc2dc\ud588\uace0, \uc911\ubd80 \ubc0f \ub3d9\ubd80 \uc804\uc120\uc758 \uc720\uc5d4\uad70\uc740 \uc6d0\uc8fc\uc640 \ud574\uc548\ub9c8\uc744\uc778 \uc8fc\ubb38\uc9c4 \uc0ac\uc774\uc5d0 \ubc29\uc5b4\uc9c4\uc9c0\ub4e4\uc744 \uc124\uce58\ud588\ub2e4. \uac19\uc740 \uc2dc\uae30\uc5d0 \uc911\uad6d\uc778\ubbfc\uc9c0\uc6d0\uad70 \uc81c42\uad70\ub2e8\uacfc \uc81c66\uad70\ub2e8\uc744 \uad6c\ud638\ud558\uae30 \uc704\ud574 \uc911\uad6d\uad70\uc740 \uc870\uc120\uc778\ubbfc\uad70 \uc81c2\uad70\ub2e8\uacfc \uc81c5\uad70\ub2e8\uacfc \ud568\uaed8 \uacf5\uc138 \uc791\uc804\uc744 \uc911\ub2e8\ud588\ub2e4.", "answer": "37\ub3c4\uc120", "sentence": "1\uc6d4 5\uc77c, \ub9ac\uc9c0\uc6e8\uc774\ub294 D \ub77c\uc778\uc774\ub77c \uba85\uba85\ub41c \uc0c8\ub85c\uc6b4 \ubc29\uc5b4\uc120\uc744 \uc124\ub9bd\ud558\uae30 \uc704\ud574 \ubaa8\ub4e0 \uc720\uc5d4\uad70 \ubcd1\ub825\uc5d0\uac8c 37\ub3c4\uc120 \uc774\ub0a8\uc744 \ucca0\uc218\ud560 \uac83\uc744 \uc9c0\uc2dc\ud588\uace0, \uc911\ubd80 \ubc0f \ub3d9\ubd80 \uc804\uc120\uc758 \uc720\uc5d4\uad70\uc740 \uc6d0\uc8fc\uc640 \ud574\uc548\ub9c8\uc744\uc778 \uc8fc\ubb38\uc9c4 \uc0ac\uc774\uc5d0 \ubc29\uc5b4\uc9c4\uc9c0\ub4e4\uc744 \uc124\uce58\ud588\ub2e4.", "paragraph_sentence": "\uc911\uad6d\uad70\uacfc \ubd81\ud55c\uad70\uc758 \ucd08\uae30 \uacf5\uaca9\uc774 \uc131\uacf5\ud558\uc5ec \ub300\ud55c\ubbfc\uad6d \uc81c1\uad70\ub2e8\uc740 \uc6d0\ub798 \ubc29\uc5b4\uc9c0\uc810\uc774\uc5c8\ub358 \ud604\ub9ac\uc5d0\uc11c\uc758 \uc0c1\ud669 \ud68c\ubcf5 \uc2dc\ub3c4\ub97c \ud3ec\uae30\ud560 \uc218\ubc16\uc5d0 \uc5c6\uc5c8\ub2e4. \uc774\ud6c4 \ubd81\ud55c\uad70 \ubcd1\ub825 \ub2e4\uc218\uac00 \uc6d0\uc8fc\uc758 \ubbf8\uad6d \uc81c2\ubcf4\ubcd1\uc0ac\ub2e8\uacfc \ub3d9\ud574\uc548\uc758 \ub300\ud55c\ubbfc\uad6d \uc81c1\uad70\ub2e8 \uc0ac\uc774\uc5d0 \uc0dd\uae34 \ud2c8 \uc0ac\uc774\ub85c \ubc00\uace0 \ub4e4\uc5b4\uc654\ub2e4. \ub9ac\uc9c0\uc6e8\uc774\ub294 \uc911\ubd80 \uc804\uc120\uc5d0 \ub300\ud55c \uc870\uc911 \uc5f0\ud569\uad70\uc758 \uacf5\uaca9\uc774 \uc11c\uc6b8\uc758 \uc720\uc5d4\uad70 \ubc29\uc5b4\uc790\ub4e4\uc744 \ud3ec\uc704\ud558\uae30 \uc704\ud55c \uc2dc\ub3c4\ub77c\uace0 \ud574\uc11d\ud558\uace0 \uadf8\ub294 1\uc6d4 3\uc77c \uc11c\uc6b8 \uc8fc\ub454 \uc720\uc5d4\uad70\uc5d0\uac8c \uc989\uac01\uc801\uc778 \ucca0\uc218\ub97c \uba85\ub839\ud588\ub2e4. 1\uc6d4 5\uc77c, \ub9ac\uc9c0\uc6e8\uc774\ub294 D \ub77c\uc778\uc774\ub77c \uba85\uba85\ub41c \uc0c8\ub85c\uc6b4 \ubc29\uc5b4\uc120\uc744 \uc124\ub9bd\ud558\uae30 \uc704\ud574 \ubaa8\ub4e0 \uc720\uc5d4\uad70 \ubcd1\ub825\uc5d0\uac8c 37\ub3c4\uc120 \uc774\ub0a8\uc744 \ucca0\uc218\ud560 \uac83\uc744 \uc9c0\uc2dc\ud588\uace0, \uc911\ubd80 \ubc0f \ub3d9\ubd80 \uc804\uc120\uc758 \uc720\uc5d4\uad70\uc740 \uc6d0\uc8fc\uc640 \ud574\uc548\ub9c8\uc744\uc778 \uc8fc\ubb38\uc9c4 \uc0ac\uc774\uc5d0 \ubc29\uc5b4\uc9c4\uc9c0\ub4e4\uc744 \uc124\uce58\ud588\ub2e4. \uac19\uc740 \uc2dc\uae30\uc5d0 \uc911\uad6d\uc778\ubbfc\uc9c0\uc6d0\uad70 \uc81c42\uad70\ub2e8\uacfc \uc81c66\uad70\ub2e8\uc744 \uad6c\ud638\ud558\uae30 \uc704\ud574 \uc911\uad6d\uad70\uc740 \uc870\uc120\uc778\ubbfc\uad70 \uc81c2\uad70\ub2e8\uacfc \uc81c5\uad70\ub2e8\uacfc \ud568\uaed8 \uacf5\uc138 \uc791\uc804\uc744 \uc911\ub2e8\ud588\ub2e4.", "paragraph_answer": "\uc911\uad6d\uad70\uacfc \ubd81\ud55c\uad70\uc758 \ucd08\uae30 \uacf5\uaca9\uc774 \uc131\uacf5\ud558\uc5ec \ub300\ud55c\ubbfc\uad6d \uc81c1\uad70\ub2e8\uc740 \uc6d0\ub798 \ubc29\uc5b4\uc9c0\uc810\uc774\uc5c8\ub358 \ud604\ub9ac\uc5d0\uc11c\uc758 \uc0c1\ud669 \ud68c\ubcf5 \uc2dc\ub3c4\ub97c \ud3ec\uae30\ud560 \uc218\ubc16\uc5d0 \uc5c6\uc5c8\ub2e4. \uc774\ud6c4 \ubd81\ud55c\uad70 \ubcd1\ub825 \ub2e4\uc218\uac00 \uc6d0\uc8fc\uc758 \ubbf8\uad6d \uc81c2\ubcf4\ubcd1\uc0ac\ub2e8\uacfc \ub3d9\ud574\uc548\uc758 \ub300\ud55c\ubbfc\uad6d \uc81c1\uad70\ub2e8 \uc0ac\uc774\uc5d0 \uc0dd\uae34 \ud2c8 \uc0ac\uc774\ub85c \ubc00\uace0 \ub4e4\uc5b4\uc654\ub2e4. \ub9ac\uc9c0\uc6e8\uc774\ub294 \uc911\ubd80 \uc804\uc120\uc5d0 \ub300\ud55c \uc870\uc911 \uc5f0\ud569\uad70\uc758 \uacf5\uaca9\uc774 \uc11c\uc6b8\uc758 \uc720\uc5d4\uad70 \ubc29\uc5b4\uc790\ub4e4\uc744 \ud3ec\uc704\ud558\uae30 \uc704\ud55c \uc2dc\ub3c4\ub77c\uace0 \ud574\uc11d\ud558\uace0 \uadf8\ub294 1\uc6d4 3\uc77c \uc11c\uc6b8 \uc8fc\ub454 \uc720\uc5d4\uad70\uc5d0\uac8c \uc989\uac01\uc801\uc778 \ucca0\uc218\ub97c \uba85\ub839\ud588\ub2e4. 1\uc6d4 5\uc77c, \ub9ac\uc9c0\uc6e8\uc774\ub294 D \ub77c\uc778\uc774\ub77c \uba85\uba85\ub41c \uc0c8\ub85c\uc6b4 \ubc29\uc5b4\uc120\uc744 \uc124\ub9bd\ud558\uae30 \uc704\ud574 \ubaa8\ub4e0 \uc720\uc5d4\uad70 \ubcd1\ub825\uc5d0\uac8c 37\ub3c4\uc120 \uc774\ub0a8\uc744 \ucca0\uc218\ud560 \uac83\uc744 \uc9c0\uc2dc\ud588\uace0, \uc911\ubd80 \ubc0f \ub3d9\ubd80 \uc804\uc120\uc758 \uc720\uc5d4\uad70\uc740 \uc6d0\uc8fc\uc640 \ud574\uc548\ub9c8\uc744\uc778 \uc8fc\ubb38\uc9c4 \uc0ac\uc774\uc5d0 \ubc29\uc5b4\uc9c4\uc9c0\ub4e4\uc744 \uc124\uce58\ud588\ub2e4. \uac19\uc740 \uc2dc\uae30\uc5d0 \uc911\uad6d\uc778\ubbfc\uc9c0\uc6d0\uad70 \uc81c42\uad70\ub2e8\uacfc \uc81c66\uad70\ub2e8\uc744 \uad6c\ud638\ud558\uae30 \uc704\ud574 \uc911\uad6d\uad70\uc740 \uc870\uc120\uc778\ubbfc\uad70 \uc81c2\uad70\ub2e8\uacfc \uc81c5\uad70\ub2e8\uacfc \ud568\uaed8 \uacf5\uc138 \uc791\uc804\uc744 \uc911\ub2e8\ud588\ub2e4.", "sentence_answer": "1\uc6d4 5\uc77c, \ub9ac\uc9c0\uc6e8\uc774\ub294 D \ub77c\uc778\uc774\ub77c \uba85\uba85\ub41c \uc0c8\ub85c\uc6b4 \ubc29\uc5b4\uc120\uc744 \uc124\ub9bd\ud558\uae30 \uc704\ud574 \ubaa8\ub4e0 \uc720\uc5d4\uad70 \ubcd1\ub825\uc5d0\uac8c 37\ub3c4\uc120 \uc774\ub0a8\uc744 \ucca0\uc218\ud560 \uac83\uc744 \uc9c0\uc2dc\ud588\uace0, \uc911\ubd80 \ubc0f \ub3d9\ubd80 \uc804\uc120\uc758 \uc720\uc5d4\uad70\uc740 \uc6d0\uc8fc\uc640 \ud574\uc548\ub9c8\uc744\uc778 \uc8fc\ubb38\uc9c4 \uc0ac\uc774\uc5d0 \ubc29\uc5b4\uc9c4\uc9c0\ub4e4\uc744 \uc124\uce58\ud588\ub2e4.", "paragraph_id": "6550565-7-1", "paragraph_question": "question: \ub9ac\uc9c0\uc6e8\uc774\ub294 \uc0c8\ub85c\uc6b4 \ubc29\uc5b4\uc120\uc744 \uc124\ub9bd\ud558\uae30 \uc704\ud574 \uba87\ub3c4\uc120 \uc774\ub0a8\uae4c\uc9c0 \ucca0\uc218\ud560 \uac83\uc744 \uc9c0\uc2dc\ud588\ub294\uac00?, context: \uc911\uad6d\uad70\uacfc \ubd81\ud55c\uad70\uc758 \ucd08\uae30 \uacf5\uaca9\uc774 \uc131\uacf5\ud558\uc5ec \ub300\ud55c\ubbfc\uad6d \uc81c1\uad70\ub2e8\uc740 \uc6d0\ub798 \ubc29\uc5b4\uc9c0\uc810\uc774\uc5c8\ub358 \ud604\ub9ac\uc5d0\uc11c\uc758 \uc0c1\ud669 \ud68c\ubcf5 \uc2dc\ub3c4\ub97c \ud3ec\uae30\ud560 \uc218\ubc16\uc5d0 \uc5c6\uc5c8\ub2e4. \uc774\ud6c4 \ubd81\ud55c\uad70 \ubcd1\ub825 \ub2e4\uc218\uac00 \uc6d0\uc8fc\uc758 \ubbf8\uad6d \uc81c2\ubcf4\ubcd1\uc0ac\ub2e8\uacfc \ub3d9\ud574\uc548\uc758 \ub300\ud55c\ubbfc\uad6d \uc81c1\uad70\ub2e8 \uc0ac\uc774\uc5d0 \uc0dd\uae34 \ud2c8 \uc0ac\uc774\ub85c \ubc00\uace0 \ub4e4\uc5b4\uc654\ub2e4. \ub9ac\uc9c0\uc6e8\uc774\ub294 \uc911\ubd80 \uc804\uc120\uc5d0 \ub300\ud55c \uc870\uc911 \uc5f0\ud569\uad70\uc758 \uacf5\uaca9\uc774 \uc11c\uc6b8\uc758 \uc720\uc5d4\uad70 \ubc29\uc5b4\uc790\ub4e4\uc744 \ud3ec\uc704\ud558\uae30 \uc704\ud55c \uc2dc\ub3c4\ub77c\uace0 \ud574\uc11d\ud558\uace0 \uadf8\ub294 1\uc6d4 3\uc77c \uc11c\uc6b8 \uc8fc\ub454 \uc720\uc5d4\uad70\uc5d0\uac8c \uc989\uac01\uc801\uc778 \ucca0\uc218\ub97c \uba85\ub839\ud588\ub2e4. 1\uc6d4 5\uc77c, \ub9ac\uc9c0\uc6e8\uc774\ub294 D \ub77c\uc778\uc774\ub77c \uba85\uba85\ub41c \uc0c8\ub85c\uc6b4 \ubc29\uc5b4\uc120\uc744 \uc124\ub9bd\ud558\uae30 \uc704\ud574 \ubaa8\ub4e0 \uc720\uc5d4\uad70 \ubcd1\ub825\uc5d0\uac8c 37\ub3c4\uc120 \uc774\ub0a8\uc744 \ucca0\uc218\ud560 \uac83\uc744 \uc9c0\uc2dc\ud588\uace0, \uc911\ubd80 \ubc0f \ub3d9\ubd80 \uc804\uc120\uc758 \uc720\uc5d4\uad70\uc740 \uc6d0\uc8fc\uc640 \ud574\uc548\ub9c8\uc744\uc778 \uc8fc\ubb38\uc9c4 \uc0ac\uc774\uc5d0 \ubc29\uc5b4\uc9c4\uc9c0\ub4e4\uc744 \uc124\uce58\ud588\ub2e4. \uac19\uc740 \uc2dc\uae30\uc5d0 \uc911\uad6d\uc778\ubbfc\uc9c0\uc6d0\uad70 \uc81c42\uad70\ub2e8\uacfc \uc81c66\uad70\ub2e8\uc744 \uad6c\ud638\ud558\uae30 \uc704\ud574 \uc911\uad6d\uad70\uc740 \uc870\uc120\uc778\ubbfc\uad70 \uc81c2\uad70\ub2e8\uacfc \uc81c5\uad70\ub2e8\uacfc \ud568\uaed8 \uacf5\uc138 \uc791\uc804\uc744 \uc911\ub2e8\ud588\ub2e4."} {"question": "1917\ub144 \ud2f0\ubca0\ud2b8 \ucef4\uc744 \uce68\ub7b5\ud55c \uc911\uad6d\uc778\uc740 \ub204\uad6c\uc778\uac00?", "paragraph": "13\uc138\ub294 \uad70\uc758 \uadfc\ub300\ud654\ub97c \ub3c4\ubaa8\ud558\uae30 \uc704\ud558\uc5ec \uce20\uc544\ub871\u00b7\uc0e4\ud398\ub97c \uad70\uc758 \ucd5c\uace0 \uc0ac\ub839\uad00\uc5d0 \uc784\uba85\ud588\ub2e4. \uce20\uc544\ub871\u00b7\uc0e4\ud398\ub294 \ubabd\uace8 \ub9dd\uba85 \uc2dc\uc5d0 \ub3d9\ud589\ud55c \uac00\uc2e0\uc758 \ub354 \uc885\uc790\uc778\uc9c0\ub9cc \ubabd\uace8\uc5b4\uc758 \uc2b5\ub4dd\uc774 \ube68\ub790\ub2e4 \ub54c\ubb38\uc5d0 \uadf8\uc758\uc5d0\ub3c4 \ub9c9\ubd80\uc5d0 \uc9c1\uc18d\ud588\ub358 \ubb34\uc0ac\ub85c \uccad\uad70 \uce68\uacf5 \ub54c\ub294 \uadf8\uc758 \ubaa9\uc228\uc744 \uad6c\ud588\uae30 \ub54c\ubb38\uc5d0 13\uc138\uc758 \ub9c8\uc74c\uc5d0 \ub4e0\ub2e4\uace0 \ub418\uc5b4 \uc788\uc5c8\ub2e4. 1916\ub144 \uc704\uc548\uc2a4\uce74\uc774\uac00 \uc911\uad6d \ud669\uc81c\uac00 \ub418\ub294\ub370 \uc2e4\ud328\ud558\uc790 \uc911\ud654 \ubbfc\uad6d\uc740 \uad70\uc6c5 \ud560\uac70\uc758 \uc2dc\ub300\uac00 \ub418\uc5c8\ub2e4. 13\uc138\ub294 \uc774 \ud63c\ub780\uc5d0 \uc911\ub9bd\uc744 \uc720\uc9c0\ud588\uc73c\ub098 1917\ub144\uc5d0 \uc911\uad6d\uc758 \ud391\ub354\ud654\uc774 \uc77c\uc2b9\uc774 \ud2f0\ubca0\ud2b8\uc758 \ucef4\uc5d0 \uccd0\ub4e4\uc5b4\uac00\uba74 \ud604\uc9c0 \ud2f0\ubca0\ud2b8\uad70\uc740 \ubc18\uaca9\uc5d0 \uc131\uacf5\ud558\uba74\uc11c \ud55c\ub54c \uc584\ub8fd\uc7a5\ubf40 \uac15\uae4c\uc9c0 \uad70\uc744 \uc9c4\ud589\ud588\ub2e4. 1918\ub144 8\uc6d4 \uc601\uad6d\uc758 \uc8fc\uc120\uc73c\ub85c \uba54\ucf69 \uac15\uacfc \uc584\ub8fd\uc7a5\ubf40 \uac15 \uc0ac\uc774\uc5d0 \uc788\ub294 \ub098\uac00\uc5d0\uac00 \uc77c\uc2dc\uc801 \uad6d\uacbd\uc73c\ub85c \uc815\ud574\uc84c\uc73c\uba70 \uc774\ub4ec\ud574\uc778 1919\ub144\uc5d0\ub294 \uadf8\uac83\uc774 \ud655\uc815\ud588\ub2e4.\uc911\uad6d\uc758 \uc774\ub978\ubc14 \ubca0\uc774\uc9d5 \uc815\ubd80\ub294 \ud2f0\ubca0\ud2b8\uc758 \ub3c5\ub9bd\ub9cc \ud5c8\uc6a9\ud558\uc9c0 \uc54a\uc558\ub2e4.", "answer": "\ud391\ub354\ud654\uc774 \uc77c\uc2b9", "sentence": "13\uc138\ub294 \uc774 \ud63c\ub780\uc5d0 \uc911\ub9bd\uc744 \uc720\uc9c0\ud588\uc73c\ub098 1917\ub144\uc5d0 \uc911\uad6d\uc758 \ud391\ub354\ud654\uc774 \uc77c\uc2b9 \uc774 \ud2f0\ubca0\ud2b8\uc758 \ucef4\uc5d0 \uccd0\ub4e4\uc5b4\uac00\uba74 \ud604\uc9c0 \ud2f0\ubca0\ud2b8\uad70\uc740 \ubc18\uaca9\uc5d0 \uc131\uacf5\ud558\uba74\uc11c \ud55c\ub54c \uc584\ub8fd\uc7a5\ubf40 \uac15\uae4c\uc9c0 \uad70\uc744 \uc9c4\ud589\ud588\ub2e4.", "paragraph_sentence": "13\uc138\ub294 \uad70\uc758 \uadfc\ub300\ud654\ub97c \ub3c4\ubaa8\ud558\uae30 \uc704\ud558\uc5ec \uce20\uc544\ub871\u00b7\uc0e4\ud398\ub97c \uad70\uc758 \ucd5c\uace0 \uc0ac\ub839\uad00\uc5d0 \uc784\uba85\ud588\ub2e4. \uce20\uc544\ub871\u00b7\uc0e4\ud398\ub294 \ubabd\uace8 \ub9dd\uba85 \uc2dc\uc5d0 \ub3d9\ud589\ud55c \uac00\uc2e0\uc758 \ub354 \uc885\uc790\uc778\uc9c0\ub9cc \ubabd\uace8\uc5b4\uc758 \uc2b5\ub4dd\uc774 \ube68\ub790\ub2e4 \ub54c\ubb38\uc5d0 \uadf8\uc758\uc5d0\ub3c4 \ub9c9\ubd80\uc5d0 \uc9c1\uc18d\ud588\ub358 \ubb34\uc0ac\ub85c \uccad\uad70 \uce68\uacf5 \ub54c\ub294 \uadf8\uc758 \ubaa9\uc228\uc744 \uad6c\ud588\uae30 \ub54c\ubb38\uc5d0 13\uc138\uc758 \ub9c8\uc74c\uc5d0 \ub4e0\ub2e4\uace0 \ub418\uc5b4 \uc788\uc5c8\ub2e4. 1916\ub144 \uc704\uc548\uc2a4\uce74\uc774\uac00 \uc911\uad6d \ud669\uc81c\uac00 \ub418\ub294\ub370 \uc2e4\ud328\ud558\uc790 \uc911\ud654 \ubbfc\uad6d\uc740 \uad70\uc6c5 \ud560\uac70\uc758 \uc2dc\ub300\uac00 \ub418\uc5c8\ub2e4. 13\uc138\ub294 \uc774 \ud63c\ub780\uc5d0 \uc911\ub9bd\uc744 \uc720\uc9c0\ud588\uc73c\ub098 1917\ub144\uc5d0 \uc911\uad6d\uc758 \ud391\ub354\ud654\uc774 \uc77c\uc2b9 \uc774 \ud2f0\ubca0\ud2b8\uc758 \ucef4\uc5d0 \uccd0\ub4e4\uc5b4\uac00\uba74 \ud604\uc9c0 \ud2f0\ubca0\ud2b8\uad70\uc740 \ubc18\uaca9\uc5d0 \uc131\uacf5\ud558\uba74\uc11c \ud55c\ub54c \uc584\ub8fd\uc7a5\ubf40 \uac15\uae4c\uc9c0 \uad70\uc744 \uc9c4\ud589\ud588\ub2e4. 1918\ub144 8\uc6d4 \uc601\uad6d\uc758 \uc8fc\uc120\uc73c\ub85c \uba54\ucf69 \uac15\uacfc \uc584\ub8fd\uc7a5\ubf40 \uac15 \uc0ac\uc774\uc5d0 \uc788\ub294 \ub098\uac00\uc5d0\uac00 \uc77c\uc2dc\uc801 \uad6d\uacbd\uc73c\ub85c \uc815\ud574\uc84c\uc73c\uba70 \uc774\ub4ec\ud574\uc778 1919\ub144\uc5d0\ub294 \uadf8\uac83\uc774 \ud655\uc815\ud588\ub2e4.\uc911\uad6d\uc758 \uc774\ub978\ubc14 \ubca0\uc774\uc9d5 \uc815\ubd80\ub294 \ud2f0\ubca0\ud2b8\uc758 \ub3c5\ub9bd\ub9cc \ud5c8\uc6a9\ud558\uc9c0 \uc54a\uc558\ub2e4.", "paragraph_answer": "13\uc138\ub294 \uad70\uc758 \uadfc\ub300\ud654\ub97c \ub3c4\ubaa8\ud558\uae30 \uc704\ud558\uc5ec \uce20\uc544\ub871\u00b7\uc0e4\ud398\ub97c \uad70\uc758 \ucd5c\uace0 \uc0ac\ub839\uad00\uc5d0 \uc784\uba85\ud588\ub2e4. \uce20\uc544\ub871\u00b7\uc0e4\ud398\ub294 \ubabd\uace8 \ub9dd\uba85 \uc2dc\uc5d0 \ub3d9\ud589\ud55c \uac00\uc2e0\uc758 \ub354 \uc885\uc790\uc778\uc9c0\ub9cc \ubabd\uace8\uc5b4\uc758 \uc2b5\ub4dd\uc774 \ube68\ub790\ub2e4 \ub54c\ubb38\uc5d0 \uadf8\uc758\uc5d0\ub3c4 \ub9c9\ubd80\uc5d0 \uc9c1\uc18d\ud588\ub358 \ubb34\uc0ac\ub85c \uccad\uad70 \uce68\uacf5 \ub54c\ub294 \uadf8\uc758 \ubaa9\uc228\uc744 \uad6c\ud588\uae30 \ub54c\ubb38\uc5d0 13\uc138\uc758 \ub9c8\uc74c\uc5d0 \ub4e0\ub2e4\uace0 \ub418\uc5b4 \uc788\uc5c8\ub2e4. 1916\ub144 \uc704\uc548\uc2a4\uce74\uc774\uac00 \uc911\uad6d \ud669\uc81c\uac00 \ub418\ub294\ub370 \uc2e4\ud328\ud558\uc790 \uc911\ud654 \ubbfc\uad6d\uc740 \uad70\uc6c5 \ud560\uac70\uc758 \uc2dc\ub300\uac00 \ub418\uc5c8\ub2e4. 13\uc138\ub294 \uc774 \ud63c\ub780\uc5d0 \uc911\ub9bd\uc744 \uc720\uc9c0\ud588\uc73c\ub098 1917\ub144\uc5d0 \uc911\uad6d\uc758 \ud391\ub354\ud654\uc774 \uc77c\uc2b9 \uc774 \ud2f0\ubca0\ud2b8\uc758 \ucef4\uc5d0 \uccd0\ub4e4\uc5b4\uac00\uba74 \ud604\uc9c0 \ud2f0\ubca0\ud2b8\uad70\uc740 \ubc18\uaca9\uc5d0 \uc131\uacf5\ud558\uba74\uc11c \ud55c\ub54c \uc584\ub8fd\uc7a5\ubf40 \uac15\uae4c\uc9c0 \uad70\uc744 \uc9c4\ud589\ud588\ub2e4. 1918\ub144 8\uc6d4 \uc601\uad6d\uc758 \uc8fc\uc120\uc73c\ub85c \uba54\ucf69 \uac15\uacfc \uc584\ub8fd\uc7a5\ubf40 \uac15 \uc0ac\uc774\uc5d0 \uc788\ub294 \ub098\uac00\uc5d0\uac00 \uc77c\uc2dc\uc801 \uad6d\uacbd\uc73c\ub85c \uc815\ud574\uc84c\uc73c\uba70 \uc774\ub4ec\ud574\uc778 1919\ub144\uc5d0\ub294 \uadf8\uac83\uc774 \ud655\uc815\ud588\ub2e4.\uc911\uad6d\uc758 \uc774\ub978\ubc14 \ubca0\uc774\uc9d5 \uc815\ubd80\ub294 \ud2f0\ubca0\ud2b8\uc758 \ub3c5\ub9bd\ub9cc \ud5c8\uc6a9\ud558\uc9c0 \uc54a\uc558\ub2e4.", "sentence_answer": "13\uc138\ub294 \uc774 \ud63c\ub780\uc5d0 \uc911\ub9bd\uc744 \uc720\uc9c0\ud588\uc73c\ub098 1917\ub144\uc5d0 \uc911\uad6d\uc758 \ud391\ub354\ud654\uc774 \uc77c\uc2b9 \uc774 \ud2f0\ubca0\ud2b8\uc758 \ucef4\uc5d0 \uccd0\ub4e4\uc5b4\uac00\uba74 \ud604\uc9c0 \ud2f0\ubca0\ud2b8\uad70\uc740 \ubc18\uaca9\uc5d0 \uc131\uacf5\ud558\uba74\uc11c \ud55c\ub54c \uc584\ub8fd\uc7a5\ubf40 \uac15\uae4c\uc9c0 \uad70\uc744 \uc9c4\ud589\ud588\ub2e4.", "paragraph_id": "6550707-15-1", "paragraph_question": "question: 1917\ub144 \ud2f0\ubca0\ud2b8 \ucef4\uc744 \uce68\ub7b5\ud55c \uc911\uad6d\uc778\uc740 \ub204\uad6c\uc778\uac00?, context: 13\uc138\ub294 \uad70\uc758 \uadfc\ub300\ud654\ub97c \ub3c4\ubaa8\ud558\uae30 \uc704\ud558\uc5ec \uce20\uc544\ub871\u00b7\uc0e4\ud398\ub97c \uad70\uc758 \ucd5c\uace0 \uc0ac\ub839\uad00\uc5d0 \uc784\uba85\ud588\ub2e4. \uce20\uc544\ub871\u00b7\uc0e4\ud398\ub294 \ubabd\uace8 \ub9dd\uba85 \uc2dc\uc5d0 \ub3d9\ud589\ud55c \uac00\uc2e0\uc758 \ub354 \uc885\uc790\uc778\uc9c0\ub9cc \ubabd\uace8\uc5b4\uc758 \uc2b5\ub4dd\uc774 \ube68\ub790\ub2e4 \ub54c\ubb38\uc5d0 \uadf8\uc758\uc5d0\ub3c4 \ub9c9\ubd80\uc5d0 \uc9c1\uc18d\ud588\ub358 \ubb34\uc0ac\ub85c \uccad\uad70 \uce68\uacf5 \ub54c\ub294 \uadf8\uc758 \ubaa9\uc228\uc744 \uad6c\ud588\uae30 \ub54c\ubb38\uc5d0 13\uc138\uc758 \ub9c8\uc74c\uc5d0 \ub4e0\ub2e4\uace0 \ub418\uc5b4 \uc788\uc5c8\ub2e4. 1916\ub144 \uc704\uc548\uc2a4\uce74\uc774\uac00 \uc911\uad6d \ud669\uc81c\uac00 \ub418\ub294\ub370 \uc2e4\ud328\ud558\uc790 \uc911\ud654 \ubbfc\uad6d\uc740 \uad70\uc6c5 \ud560\uac70\uc758 \uc2dc\ub300\uac00 \ub418\uc5c8\ub2e4. 13\uc138\ub294 \uc774 \ud63c\ub780\uc5d0 \uc911\ub9bd\uc744 \uc720\uc9c0\ud588\uc73c\ub098 1917\ub144\uc5d0 \uc911\uad6d\uc758 \ud391\ub354\ud654\uc774 \uc77c\uc2b9\uc774 \ud2f0\ubca0\ud2b8\uc758 \ucef4\uc5d0 \uccd0\ub4e4\uc5b4\uac00\uba74 \ud604\uc9c0 \ud2f0\ubca0\ud2b8\uad70\uc740 \ubc18\uaca9\uc5d0 \uc131\uacf5\ud558\uba74\uc11c \ud55c\ub54c \uc584\ub8fd\uc7a5\ubf40 \uac15\uae4c\uc9c0 \uad70\uc744 \uc9c4\ud589\ud588\ub2e4. 1918\ub144 8\uc6d4 \uc601\uad6d\uc758 \uc8fc\uc120\uc73c\ub85c \uba54\ucf69 \uac15\uacfc \uc584\ub8fd\uc7a5\ubf40 \uac15 \uc0ac\uc774\uc5d0 \uc788\ub294 \ub098\uac00\uc5d0\uac00 \uc77c\uc2dc\uc801 \uad6d\uacbd\uc73c\ub85c \uc815\ud574\uc84c\uc73c\uba70 \uc774\ub4ec\ud574\uc778 1919\ub144\uc5d0\ub294 \uadf8\uac83\uc774 \ud655\uc815\ud588\ub2e4.\uc911\uad6d\uc758 \uc774\ub978\ubc14 \ubca0\uc774\uc9d5 \uc815\ubd80\ub294 \ud2f0\ubca0\ud2b8\uc758 \ub3c5\ub9bd\ub9cc \ud5c8\uc6a9\ud558\uc9c0 \uc54a\uc558\ub2e4."} {"question": "\uac15\ud76c\uc81c\uc758 \ubb18\ud638\ub294?", "paragraph": "\ub2a5\ud638\ub294 \uacbd\ub989(\u666f\u9675)\uc73c\ub85c \ubd80\ud669 \uc21c\uce58\uc81c\uc758 \ud669\ub989\uc778 \ud6a8\ub989(\u5b5d\u9675) \uc606\uc5d0 \uc788\uc73c\uba70, \uccad\ub3d9\ub989(\u6df8\u6771\u9675)\uc758 \ud558\ub098\uc774\ub2e4. \ubb18\ud638\ub294 \uc131\uc778\uc758 \ub73b\uc73c\ub85c \uad6d\uac00\ub97c \ub2e4\uc2a4\ub824 \uc9c4\uc815\uc73c\ub85c \ud1b5\uc77c\uc2dc\ud0a8 \ud070 \uc5c5\uc801\uc774 \uc788\ub294 \ud669\uc81c\ub77c \ud558\uc5ec \uc131\uc870(\u8056\u7956), \uc2dc\ud638\ub294 \uc0dd\uc804\uc5d0 \uac15\ud76c\uc81c\uac00 \uc778\uacfc \ub355\uc744 \uc911\uc2dc\ud55c \uac83\uc744 \ub530 \uc778\ud669\uc81c(\u4ec1\u7687\u5e1d)\ub85c \uba85\uba85\ud558\uc600\ub2e4. \uc815\uc2dd \uc2dc\ud638\ub294 \ud569\ucc9c\ud64d\uc6b4\ubb38\ubb34\uc608\ucca0\uacf5\uac80\uad00\uc720\ud6a8\uacbd\uc131\uc2e0\uc911\ud654\uacf5\ub355\ub300\uc131\uc778\ud669\uc81c(\u5408\u5929\u5f18\u904b\u6587\u6b66\u777f\u54f2\u606d\u5109\u5bec\u88d5\u5b5d\u656c\u8aa0\u4fe1\u4e2d\u548c\u529f\u5fb7\u5927\u6210\u4ec1\u7687\u5e1d)\ub85c \uc774 \uae34 \uc2dc\ud638\uc5d0\uc11c\ub3c4 \uadf8\uc758 \uc5c5\uc801\uc744 \ud655\uc778\ud560 \uc218 \uc788\ub2e4. \u2018\ud569\ucc9c\u2019(\u5408\u5929)\uc740 \ubd84\uc5f4\ub41c \ucc9c\ud558\ub97c \ub2e4\uc2dc \ud1b5\uc77c\uc2dc\ucf30\ub2e4\ub294 \ub73b\uc774\uace0 \u2018\ud64d\uc6b4\u2019(\u5f18\u904b)\uc740 \uad6d\uc6b4\uc744 \ud06c\uac8c \ub113\ud614\ub2e4\ub294 \ub73b\uc774\ub2e4. \u2018\ubb38\ubb34\u2019(\u6587\u6b66) \uc5ed\uc2dc \ub098\ub77c\uc758 \uae30\ud2c0\uc744 \uc7a1\uace0 \ubb38\uacfc \ubb34\ub97c \uace0\ub8e8 \uc774\uc6a9\ud558\uc5ec \uc804\uc131\uae30\ub97c \uc774\ub8e9\ud55c \ud669\uc81c\uc5d0\uac8c \uc62c\ub9ac\ub294 \uc2dc\ud638\ub85c \ub098\ub77c\ub97c \uc138\uc6b4 \uac1c\uad6d \ud669\uc81c\uc5d0\uac8c \uc62c\ub824\uc9c0\ub294 \u2018\uace0\u2019(\u9ad8) \uc790\uc640 \ub354\ubd88\uc5b4 \ud669\uc81c\uac00 \ubc1b\uc744 \uc218 \uc788\ub294 \ucd5c\uace0\uc758 \uc2dc\ud638\uc774\uba70 \uadf8\ub9ac\uace0 \u2018\uc911\ud654\u2019(\u4e2d\u548c)\uc640 \u2018\ub300\uc131\u2019(\u5927\u6210)\uc740 \uccad\ub098\ub77c\ub97c \uc911\ud765\uc2dc\ucf1c\uc11c \ub098\ub77c\ub97c \ub354\uc6b1 \ubc88\ucc3d\uc2dc\ud0a4\uace0 \uc5ec\ub7ec \ubbfc\uc871\uc744 \ud558\ub098\ub85c \ubaa8\uc544 \ud06c\uac8c \uc774\ub8e8\uc5c8\ub2e4\ub294 \ub73b\uc774 \ub2f4\uaca8 \uc788\ub2e4.", "answer": "\uc131\uc870", "sentence": "\ubb18\ud638\ub294 \uc131\uc778\uc758 \ub73b\uc73c\ub85c \uad6d\uac00\ub97c \ub2e4\uc2a4\ub824 \uc9c4\uc815\uc73c\ub85c \ud1b5\uc77c\uc2dc\ud0a8 \ud070 \uc5c5\uc801\uc774 \uc788\ub294 \ud669\uc81c\ub77c \ud558\uc5ec \uc131\uc870 (\u8056\u7956),", "paragraph_sentence": "\ub2a5\ud638\ub294 \uacbd\ub989(\u666f\u9675)\uc73c\ub85c \ubd80\ud669 \uc21c\uce58\uc81c\uc758 \ud669\ub989\uc778 \ud6a8\ub989(\u5b5d\u9675) \uc606\uc5d0 \uc788\uc73c\uba70, \uccad\ub3d9\ub989(\u6df8\u6771\u9675)\uc758 \ud558\ub098\uc774\ub2e4. \ubb18\ud638\ub294 \uc131\uc778\uc758 \ub73b\uc73c\ub85c \uad6d\uac00\ub97c \ub2e4\uc2a4\ub824 \uc9c4\uc815\uc73c\ub85c \ud1b5\uc77c\uc2dc\ud0a8 \ud070 \uc5c5\uc801\uc774 \uc788\ub294 \ud669\uc81c\ub77c \ud558\uc5ec \uc131\uc870 (\u8056\u7956), \uc2dc\ud638\ub294 \uc0dd\uc804\uc5d0 \uac15\ud76c\uc81c\uac00 \uc778\uacfc \ub355\uc744 \uc911\uc2dc\ud55c \uac83\uc744 \ub530 \uc778\ud669\uc81c(\u4ec1\u7687\u5e1d)\ub85c \uba85\uba85\ud558\uc600\ub2e4. \uc815\uc2dd \uc2dc\ud638\ub294 \ud569\ucc9c\ud64d\uc6b4\ubb38\ubb34\uc608\ucca0\uacf5\uac80\uad00\uc720\ud6a8\uacbd\uc131\uc2e0\uc911\ud654\uacf5\ub355\ub300\uc131\uc778\ud669\uc81c(\u5408\u5929\u5f18\u904b\u6587\u6b66\u777f\u54f2\u606d\u5109\u5bec\u88d5\u5b5d\u656c\u8aa0\u4fe1\u4e2d\u548c\u529f\u5fb7\u5927\u6210\u4ec1\u7687\u5e1d)\ub85c \uc774 \uae34 \uc2dc\ud638\uc5d0\uc11c\ub3c4 \uadf8\uc758 \uc5c5\uc801\uc744 \ud655\uc778\ud560 \uc218 \uc788\ub2e4. \u2018\ud569\ucc9c\u2019(\u5408\u5929)\uc740 \ubd84\uc5f4\ub41c \ucc9c\ud558\ub97c \ub2e4\uc2dc \ud1b5\uc77c\uc2dc\ucf30\ub2e4\ub294 \ub73b\uc774\uace0 \u2018\ud64d\uc6b4\u2019(\u5f18\u904b)\uc740 \uad6d\uc6b4\uc744 \ud06c\uac8c \ub113\ud614\ub2e4\ub294 \ub73b\uc774\ub2e4. \u2018\ubb38\ubb34\u2019(\u6587\u6b66) \uc5ed\uc2dc \ub098\ub77c\uc758 \uae30\ud2c0\uc744 \uc7a1\uace0 \ubb38\uacfc \ubb34\ub97c \uace0\ub8e8 \uc774\uc6a9\ud558\uc5ec \uc804\uc131\uae30\ub97c \uc774\ub8e9\ud55c \ud669\uc81c\uc5d0\uac8c \uc62c\ub9ac\ub294 \uc2dc\ud638\ub85c \ub098\ub77c\ub97c \uc138\uc6b4 \uac1c\uad6d \ud669\uc81c\uc5d0\uac8c \uc62c\ub824\uc9c0\ub294 \u2018\uace0\u2019(\u9ad8) \uc790\uc640 \ub354\ubd88\uc5b4 \ud669\uc81c\uac00 \ubc1b\uc744 \uc218 \uc788\ub294 \ucd5c\uace0\uc758 \uc2dc\ud638\uc774\uba70 \uadf8\ub9ac\uace0 \u2018\uc911\ud654\u2019(\u4e2d\u548c)\uc640 \u2018\ub300\uc131\u2019(\u5927\u6210)\uc740 \uccad\ub098\ub77c\ub97c \uc911\ud765\uc2dc\ucf1c\uc11c \ub098\ub77c\ub97c \ub354\uc6b1 \ubc88\ucc3d\uc2dc\ud0a4\uace0 \uc5ec\ub7ec \ubbfc\uc871\uc744 \ud558\ub098\ub85c \ubaa8\uc544 \ud06c\uac8c \uc774\ub8e8\uc5c8\ub2e4\ub294 \ub73b\uc774 \ub2f4\uaca8 \uc788\ub2e4.", "paragraph_answer": "\ub2a5\ud638\ub294 \uacbd\ub989(\u666f\u9675)\uc73c\ub85c \ubd80\ud669 \uc21c\uce58\uc81c\uc758 \ud669\ub989\uc778 \ud6a8\ub989(\u5b5d\u9675) \uc606\uc5d0 \uc788\uc73c\uba70, \uccad\ub3d9\ub989(\u6df8\u6771\u9675)\uc758 \ud558\ub098\uc774\ub2e4. \ubb18\ud638\ub294 \uc131\uc778\uc758 \ub73b\uc73c\ub85c \uad6d\uac00\ub97c \ub2e4\uc2a4\ub824 \uc9c4\uc815\uc73c\ub85c \ud1b5\uc77c\uc2dc\ud0a8 \ud070 \uc5c5\uc801\uc774 \uc788\ub294 \ud669\uc81c\ub77c \ud558\uc5ec \uc131\uc870 (\u8056\u7956), \uc2dc\ud638\ub294 \uc0dd\uc804\uc5d0 \uac15\ud76c\uc81c\uac00 \uc778\uacfc \ub355\uc744 \uc911\uc2dc\ud55c \uac83\uc744 \ub530 \uc778\ud669\uc81c(\u4ec1\u7687\u5e1d)\ub85c \uba85\uba85\ud558\uc600\ub2e4. \uc815\uc2dd \uc2dc\ud638\ub294 \ud569\ucc9c\ud64d\uc6b4\ubb38\ubb34\uc608\ucca0\uacf5\uac80\uad00\uc720\ud6a8\uacbd\uc131\uc2e0\uc911\ud654\uacf5\ub355\ub300\uc131\uc778\ud669\uc81c(\u5408\u5929\u5f18\u904b\u6587\u6b66\u777f\u54f2\u606d\u5109\u5bec\u88d5\u5b5d\u656c\u8aa0\u4fe1\u4e2d\u548c\u529f\u5fb7\u5927\u6210\u4ec1\u7687\u5e1d)\ub85c \uc774 \uae34 \uc2dc\ud638\uc5d0\uc11c\ub3c4 \uadf8\uc758 \uc5c5\uc801\uc744 \ud655\uc778\ud560 \uc218 \uc788\ub2e4. \u2018\ud569\ucc9c\u2019(\u5408\u5929)\uc740 \ubd84\uc5f4\ub41c \ucc9c\ud558\ub97c \ub2e4\uc2dc \ud1b5\uc77c\uc2dc\ucf30\ub2e4\ub294 \ub73b\uc774\uace0 \u2018\ud64d\uc6b4\u2019(\u5f18\u904b)\uc740 \uad6d\uc6b4\uc744 \ud06c\uac8c \ub113\ud614\ub2e4\ub294 \ub73b\uc774\ub2e4. \u2018\ubb38\ubb34\u2019(\u6587\u6b66) \uc5ed\uc2dc \ub098\ub77c\uc758 \uae30\ud2c0\uc744 \uc7a1\uace0 \ubb38\uacfc \ubb34\ub97c \uace0\ub8e8 \uc774\uc6a9\ud558\uc5ec \uc804\uc131\uae30\ub97c \uc774\ub8e9\ud55c \ud669\uc81c\uc5d0\uac8c \uc62c\ub9ac\ub294 \uc2dc\ud638\ub85c \ub098\ub77c\ub97c \uc138\uc6b4 \uac1c\uad6d \ud669\uc81c\uc5d0\uac8c \uc62c\ub824\uc9c0\ub294 \u2018\uace0\u2019(\u9ad8) \uc790\uc640 \ub354\ubd88\uc5b4 \ud669\uc81c\uac00 \ubc1b\uc744 \uc218 \uc788\ub294 \ucd5c\uace0\uc758 \uc2dc\ud638\uc774\uba70 \uadf8\ub9ac\uace0 \u2018\uc911\ud654\u2019(\u4e2d\u548c)\uc640 \u2018\ub300\uc131\u2019(\u5927\u6210)\uc740 \uccad\ub098\ub77c\ub97c \uc911\ud765\uc2dc\ucf1c\uc11c \ub098\ub77c\ub97c \ub354\uc6b1 \ubc88\ucc3d\uc2dc\ud0a4\uace0 \uc5ec\ub7ec \ubbfc\uc871\uc744 \ud558\ub098\ub85c \ubaa8\uc544 \ud06c\uac8c \uc774\ub8e8\uc5c8\ub2e4\ub294 \ub73b\uc774 \ub2f4\uaca8 \uc788\ub2e4.", "sentence_answer": "\ubb18\ud638\ub294 \uc131\uc778\uc758 \ub73b\uc73c\ub85c \uad6d\uac00\ub97c \ub2e4\uc2a4\ub824 \uc9c4\uc815\uc73c\ub85c \ud1b5\uc77c\uc2dc\ud0a8 \ud070 \uc5c5\uc801\uc774 \uc788\ub294 \ud669\uc81c\ub77c \ud558\uc5ec \uc131\uc870 (\u8056\u7956),", "paragraph_id": "6387920-36-1", "paragraph_question": "question: \uac15\ud76c\uc81c\uc758 \ubb18\ud638\ub294?, context: \ub2a5\ud638\ub294 \uacbd\ub989(\u666f\u9675)\uc73c\ub85c \ubd80\ud669 \uc21c\uce58\uc81c\uc758 \ud669\ub989\uc778 \ud6a8\ub989(\u5b5d\u9675) \uc606\uc5d0 \uc788\uc73c\uba70, \uccad\ub3d9\ub989(\u6df8\u6771\u9675)\uc758 \ud558\ub098\uc774\ub2e4. \ubb18\ud638\ub294 \uc131\uc778\uc758 \ub73b\uc73c\ub85c \uad6d\uac00\ub97c \ub2e4\uc2a4\ub824 \uc9c4\uc815\uc73c\ub85c \ud1b5\uc77c\uc2dc\ud0a8 \ud070 \uc5c5\uc801\uc774 \uc788\ub294 \ud669\uc81c\ub77c \ud558\uc5ec \uc131\uc870(\u8056\u7956), \uc2dc\ud638\ub294 \uc0dd\uc804\uc5d0 \uac15\ud76c\uc81c\uac00 \uc778\uacfc \ub355\uc744 \uc911\uc2dc\ud55c \uac83\uc744 \ub530 \uc778\ud669\uc81c(\u4ec1\u7687\u5e1d)\ub85c \uba85\uba85\ud558\uc600\ub2e4. \uc815\uc2dd \uc2dc\ud638\ub294 \ud569\ucc9c\ud64d\uc6b4\ubb38\ubb34\uc608\ucca0\uacf5\uac80\uad00\uc720\ud6a8\uacbd\uc131\uc2e0\uc911\ud654\uacf5\ub355\ub300\uc131\uc778\ud669\uc81c(\u5408\u5929\u5f18\u904b\u6587\u6b66\u777f\u54f2\u606d\u5109\u5bec\u88d5\u5b5d\u656c\u8aa0\u4fe1\u4e2d\u548c\u529f\u5fb7\u5927\u6210\u4ec1\u7687\u5e1d)\ub85c \uc774 \uae34 \uc2dc\ud638\uc5d0\uc11c\ub3c4 \uadf8\uc758 \uc5c5\uc801\uc744 \ud655\uc778\ud560 \uc218 \uc788\ub2e4. \u2018\ud569\ucc9c\u2019(\u5408\u5929)\uc740 \ubd84\uc5f4\ub41c \ucc9c\ud558\ub97c \ub2e4\uc2dc \ud1b5\uc77c\uc2dc\ucf30\ub2e4\ub294 \ub73b\uc774\uace0 \u2018\ud64d\uc6b4\u2019(\u5f18\u904b)\uc740 \uad6d\uc6b4\uc744 \ud06c\uac8c \ub113\ud614\ub2e4\ub294 \ub73b\uc774\ub2e4. \u2018\ubb38\ubb34\u2019(\u6587\u6b66) \uc5ed\uc2dc \ub098\ub77c\uc758 \uae30\ud2c0\uc744 \uc7a1\uace0 \ubb38\uacfc \ubb34\ub97c \uace0\ub8e8 \uc774\uc6a9\ud558\uc5ec \uc804\uc131\uae30\ub97c \uc774\ub8e9\ud55c \ud669\uc81c\uc5d0\uac8c \uc62c\ub9ac\ub294 \uc2dc\ud638\ub85c \ub098\ub77c\ub97c \uc138\uc6b4 \uac1c\uad6d \ud669\uc81c\uc5d0\uac8c \uc62c\ub824\uc9c0\ub294 \u2018\uace0\u2019(\u9ad8) \uc790\uc640 \ub354\ubd88\uc5b4 \ud669\uc81c\uac00 \ubc1b\uc744 \uc218 \uc788\ub294 \ucd5c\uace0\uc758 \uc2dc\ud638\uc774\uba70 \uadf8\ub9ac\uace0 \u2018\uc911\ud654\u2019(\u4e2d\u548c)\uc640 \u2018\ub300\uc131\u2019(\u5927\u6210)\uc740 \uccad\ub098\ub77c\ub97c \uc911\ud765\uc2dc\ucf1c\uc11c \ub098\ub77c\ub97c \ub354\uc6b1 \ubc88\ucc3d\uc2dc\ud0a4\uace0 \uc5ec\ub7ec \ubbfc\uc871\uc744 \ud558\ub098\ub85c \ubaa8\uc544 \ud06c\uac8c \uc774\ub8e8\uc5c8\ub2e4\ub294 \ub73b\uc774 \ub2f4\uaca8 \uc788\ub2e4."} {"question": "\ucd09\uac01\uacfc \uc560\ucc29\uc774\ub860\uc758 \uad00\ub828\uc131\uc5d0 \ub300\ud558\uc5ec \ub9e4\ub9ac \uc124\ud130 \uc560\uc778\uc988\uc6cc\uc2a4\uc640 \uacf5\ub3d9\uc5f0\uad6c\ub97c \uc9c4\ud589\ud55c \uc0ac\ub78c\uc740 \ub204\uad6c\uc778\uac00?", "paragraph": "\uac10\uac01 \uc0c1\uc2e4 (sense deprivation)\uc774\ub780 \uc5b4\ub5a4 \uc0ac\ub78c\uc774 \ucd09\uac01\uc744 \uac70\uc758 \ub290\ub07c\uc9c0 \ubabb\ud588\uc744 \ub54c\uc758 \ud604\uc0c1\uc774\uba70, \ud2b9\ud788 \uc601\uc544\uae30 \ub54c \ud070 \ubb38\uc81c\uac00 \ub41c\ub2e4. \uc601\uc544\uc758 \uac10\uac01 \uc0c1\uc2e4\uc740 \uc774\ud6c4 \ud589\ub3d9\uc801, \uac74\uac15\uc801, \uc0dd\ub9ac\uc801 \ubc1c\ub2ec\uc5d0 \uc788\uc5b4 \uc5ec\ub7ec \uac00\uc9c0 \ubb38\uc81c\ub97c \ub0b3\uac8c \ub41c\ub2e4. \uc544\uc27d\uac8c\ub3c4 \uc774 \ubd84\uc57c\ub294 \uc5f0\uad6c\uac00 \uae4a\uc774 \ub418\uc9c0 \uc54a\uc544 \uadf8 \uc601\ud5a5\uc5d0 \ub300\ud574 \uc54c\ub824\uc9c4 \uac83\uc774 \ub9ce\uc9c0\ub294 \uc54a\ub2e4. \uc5f0\uad6c\uac00 \uc9c4\ud589\ub418\uae30 \uc804, \ucd09\uac01\uc740 \uc0ac\ub78c\uc758 \ubc1c\ub2ec\uc5d0 \uc788\uc5b4 \ubbf8\ubbf8\ud55c \uc601\ud5a5\uc744 \ub07c\uce58\ub294 \uac83\uc73c\ub85c \uc5ec\uaca8\uc84c\ub2e4. \ud558\uc9c0\ub9cc 1945-47\ub144, \uc2e0\uc2dc\ub0b4\ud2f0 \ub300\ud559\uc758 \ub85c\ubc84\ud2b8 \ud574\ud2b8\ud544\ub4dc (Robert Hatfield) \ubc15\uc0ac\uc758 \ub17c\ubb38\uc5d0 \ub530\ub77c \uc774 \uad00\uc810\uc774 \ubb34\ub108\uc9c0\uae30 \uc2dc\uc791\ud558\uc600\ub2e4. \ud55c\ud3b8\uc73c\ub85c \ub2f9\uc2dc \uc544\uc774\ub4e4\uc744 \ub3cc\ubcf4\ub294 \uc758\ub8cc\uc778\uc774\uc5c8\ub358 \ub80c \uc2a4\ud53c\uce20 (Rene Spitz) \ubc15\uc0ac\ub294, \ud2b9\uc815 \uc601\uc544 \ubc0f \uc720\uc544\ub4e4\uc758 \uac11\uc791\uc2a4\ub7f0 \uc0ac\ub9dd\uc5d0 \ub300\ud55c \uc6d0\uc778 \uaddc\uba85\uc5d0 \ucc29\uc218\ud558\uc600\ub2e4. 1958-1962\ub144\uc774 \ub418\uc5b4 \ud574\ub9ac \ud560\ub85c\uc6b0 (Harry Harlow)\uc758 \uc5f0\uad6c\uac00 \uc774\ub904\uc9c0\uace0 \ub098\uc11c\uc57c \uc6d0\uc778\uc774 \uaddc\uba85\ub418\uc5c8\ub2e4. \uc544\uc774\ub4e4\uc740 \ucd09\uac01\uc744 \uac70\uc758 \ub290\ub07c\uc9c0 \ubabb\ud588\ub358 \uac83\uc774\ub2e4. \uc6d0\uc22d\uc774 \uc0c8\ub07c\ub4e4\uc744 \uac00\uc9c0\uace0 \ud55c \ud560\ub85c\uc6b0\uc758 \uc5f0\uad6c\uc5d0 \ub530\ub77c \uadf8\ub294 \ucd09\uac01\uc758 \uc911\uc694\uc131\uc744 \uae68\ub2eb\uac8c \ub418\uc5c8\ub2e4. \uc774\ud6c4 \uc874 \ubcfc\ube44 (John Bowlby)\uc640 \ub9e4\ub9ac \uc124\ud130 \uc560\uc778\uc988\uc6cc\uc2a4 (Mary Salter Ainsworth)\uc758 \uc5f0\uad6c\ub97c \ud1b5\ud574 \uc560\ucc29 \uc774\ub860\uacfc \uad00\ub828\ud558\uc5ec \ucd09\uac01\uc774 \uc5b4\ub5a4 \uc601\ud5a5\uc744 \ubbf8\uce58\ub294\uc9c0\uac00 \ud655\uc778\ub418\uc5c8\ub2e4. \ub85c\ubc84\ud2b8 \ud574\ud2b8\ud544\ub4dc\ub294 \uc774\ub7f0 \uc2e4\ud5d8\ub4e4\uc758 \uacb0\uacfc\ub97c \ub17c\ud558\uba74\uc11c \u201c\uc560\uc815\uc774 \ub2f4\uae34 \uc190\uae38\uacfc \ubb34\uc2dc, \ubc8c\uc744 \uc8fc\ub294 \uc190\uae38 \uac04\uc758 \ucc28\uc774\ub294 \uc560\ucc29 \uc774\ub860\uc758 \uc911\uc2ec \ud654\ub450\uc774\uba70, \uc774\ub7f0 \uc5f0\uad6c\ub4e4\uc740 \ud560\ub85c\uc6b0\uc758 \uc5f0\uad6c\ub97c \uc778\uac04\uc5d0\uac8c \uc801\uc6a9\ud55c \uac83\uc774\ub77c \ubcfc \uc218 \uc788\ub2e4.\u201d\ub77c\uace0 \uc8fc\uc7a5\ud558\uc600\ub2e4.", "answer": "\uc874 \ubcfc\ube44", "sentence": "\uc774\ud6c4 \uc874 \ubcfc\ube44 (John Bowlby)\uc640 \ub9e4\ub9ac \uc124\ud130 \uc560\uc778\uc988\uc6cc\uc2a4 (Mary Salter Ainsworth)\uc758 \uc5f0\uad6c\ub97c \ud1b5\ud574 \uc560\ucc29 \uc774\ub860\uacfc \uad00\ub828\ud558\uc5ec \ucd09\uac01\uc774 \uc5b4\ub5a4 \uc601\ud5a5\uc744 \ubbf8\uce58\ub294\uc9c0\uac00 \ud655\uc778\ub418\uc5c8\ub2e4.", "paragraph_sentence": "\uac10\uac01 \uc0c1\uc2e4 (sense deprivation)\uc774\ub780 \uc5b4\ub5a4 \uc0ac\ub78c\uc774 \ucd09\uac01\uc744 \uac70\uc758 \ub290\ub07c\uc9c0 \ubabb\ud588\uc744 \ub54c\uc758 \ud604\uc0c1\uc774\uba70, \ud2b9\ud788 \uc601\uc544\uae30 \ub54c \ud070 \ubb38\uc81c\uac00 \ub41c\ub2e4. \uc601\uc544\uc758 \uac10\uac01 \uc0c1\uc2e4\uc740 \uc774\ud6c4 \ud589\ub3d9\uc801, \uac74\uac15\uc801, \uc0dd\ub9ac\uc801 \ubc1c\ub2ec\uc5d0 \uc788\uc5b4 \uc5ec\ub7ec \uac00\uc9c0 \ubb38\uc81c\ub97c \ub0b3\uac8c \ub41c\ub2e4. \uc544\uc27d\uac8c\ub3c4 \uc774 \ubd84\uc57c\ub294 \uc5f0\uad6c\uac00 \uae4a\uc774 \ub418\uc9c0 \uc54a\uc544 \uadf8 \uc601\ud5a5\uc5d0 \ub300\ud574 \uc54c\ub824\uc9c4 \uac83\uc774 \ub9ce\uc9c0\ub294 \uc54a\ub2e4. \uc5f0\uad6c\uac00 \uc9c4\ud589\ub418\uae30 \uc804, \ucd09\uac01\uc740 \uc0ac\ub78c\uc758 \ubc1c\ub2ec\uc5d0 \uc788\uc5b4 \ubbf8\ubbf8\ud55c \uc601\ud5a5\uc744 \ub07c\uce58\ub294 \uac83\uc73c\ub85c \uc5ec\uaca8\uc84c\ub2e4. \ud558\uc9c0\ub9cc 1945-47\ub144, \uc2e0\uc2dc\ub0b4\ud2f0 \ub300\ud559\uc758 \ub85c\ubc84\ud2b8 \ud574\ud2b8\ud544\ub4dc (Robert Hatfield) \ubc15\uc0ac\uc758 \ub17c\ubb38\uc5d0 \ub530\ub77c \uc774 \uad00\uc810\uc774 \ubb34\ub108\uc9c0\uae30 \uc2dc\uc791\ud558\uc600\ub2e4. \ud55c\ud3b8\uc73c\ub85c \ub2f9\uc2dc \uc544\uc774\ub4e4\uc744 \ub3cc\ubcf4\ub294 \uc758\ub8cc\uc778\uc774\uc5c8\ub358 \ub80c \uc2a4\ud53c\uce20 (Rene Spitz) \ubc15\uc0ac\ub294, \ud2b9\uc815 \uc601\uc544 \ubc0f \uc720\uc544\ub4e4\uc758 \uac11\uc791\uc2a4\ub7f0 \uc0ac\ub9dd\uc5d0 \ub300\ud55c \uc6d0\uc778 \uaddc\uba85\uc5d0 \ucc29\uc218\ud558\uc600\ub2e4. 1958-1962\ub144\uc774 \ub418\uc5b4 \ud574\ub9ac \ud560\ub85c\uc6b0 (Harry Harlow)\uc758 \uc5f0\uad6c\uac00 \uc774\ub904\uc9c0\uace0 \ub098\uc11c\uc57c \uc6d0\uc778\uc774 \uaddc\uba85\ub418\uc5c8\ub2e4. \uc544\uc774\ub4e4\uc740 \ucd09\uac01\uc744 \uac70\uc758 \ub290\ub07c\uc9c0 \ubabb\ud588\ub358 \uac83\uc774\ub2e4. \uc6d0\uc22d\uc774 \uc0c8\ub07c\ub4e4\uc744 \uac00\uc9c0\uace0 \ud55c \ud560\ub85c\uc6b0\uc758 \uc5f0\uad6c\uc5d0 \ub530\ub77c \uadf8\ub294 \ucd09\uac01\uc758 \uc911\uc694\uc131\uc744 \uae68\ub2eb\uac8c \ub418\uc5c8\ub2e4. \uc774\ud6c4 \uc874 \ubcfc\ube44 (John Bowlby)\uc640 \ub9e4\ub9ac \uc124\ud130 \uc560\uc778\uc988\uc6cc\uc2a4 (Mary Salter Ainsworth)\uc758 \uc5f0\uad6c\ub97c \ud1b5\ud574 \uc560\ucc29 \uc774\ub860\uacfc \uad00\ub828\ud558\uc5ec \ucd09\uac01\uc774 \uc5b4\ub5a4 \uc601\ud5a5\uc744 \ubbf8\uce58\ub294\uc9c0\uac00 \ud655\uc778\ub418\uc5c8\ub2e4. \ub85c\ubc84\ud2b8 \ud574\ud2b8\ud544\ub4dc\ub294 \uc774\ub7f0 \uc2e4\ud5d8\ub4e4\uc758 \uacb0\uacfc\ub97c \ub17c\ud558\uba74\uc11c \u201c\uc560\uc815\uc774 \ub2f4\uae34 \uc190\uae38\uacfc \ubb34\uc2dc, \ubc8c\uc744 \uc8fc\ub294 \uc190\uae38 \uac04\uc758 \ucc28\uc774\ub294 \uc560\ucc29 \uc774\ub860\uc758 \uc911\uc2ec \ud654\ub450\uc774\uba70, \uc774\ub7f0 \uc5f0\uad6c\ub4e4\uc740 \ud560\ub85c\uc6b0\uc758 \uc5f0\uad6c\ub97c \uc778\uac04\uc5d0\uac8c \uc801\uc6a9\ud55c \uac83\uc774\ub77c \ubcfc \uc218 \uc788\ub2e4. \u201d\ub77c\uace0 \uc8fc\uc7a5\ud558\uc600\ub2e4.", "paragraph_answer": "\uac10\uac01 \uc0c1\uc2e4 (sense deprivation)\uc774\ub780 \uc5b4\ub5a4 \uc0ac\ub78c\uc774 \ucd09\uac01\uc744 \uac70\uc758 \ub290\ub07c\uc9c0 \ubabb\ud588\uc744 \ub54c\uc758 \ud604\uc0c1\uc774\uba70, \ud2b9\ud788 \uc601\uc544\uae30 \ub54c \ud070 \ubb38\uc81c\uac00 \ub41c\ub2e4. \uc601\uc544\uc758 \uac10\uac01 \uc0c1\uc2e4\uc740 \uc774\ud6c4 \ud589\ub3d9\uc801, \uac74\uac15\uc801, \uc0dd\ub9ac\uc801 \ubc1c\ub2ec\uc5d0 \uc788\uc5b4 \uc5ec\ub7ec \uac00\uc9c0 \ubb38\uc81c\ub97c \ub0b3\uac8c \ub41c\ub2e4. \uc544\uc27d\uac8c\ub3c4 \uc774 \ubd84\uc57c\ub294 \uc5f0\uad6c\uac00 \uae4a\uc774 \ub418\uc9c0 \uc54a\uc544 \uadf8 \uc601\ud5a5\uc5d0 \ub300\ud574 \uc54c\ub824\uc9c4 \uac83\uc774 \ub9ce\uc9c0\ub294 \uc54a\ub2e4. \uc5f0\uad6c\uac00 \uc9c4\ud589\ub418\uae30 \uc804, \ucd09\uac01\uc740 \uc0ac\ub78c\uc758 \ubc1c\ub2ec\uc5d0 \uc788\uc5b4 \ubbf8\ubbf8\ud55c \uc601\ud5a5\uc744 \ub07c\uce58\ub294 \uac83\uc73c\ub85c \uc5ec\uaca8\uc84c\ub2e4. \ud558\uc9c0\ub9cc 1945-47\ub144, \uc2e0\uc2dc\ub0b4\ud2f0 \ub300\ud559\uc758 \ub85c\ubc84\ud2b8 \ud574\ud2b8\ud544\ub4dc (Robert Hatfield) \ubc15\uc0ac\uc758 \ub17c\ubb38\uc5d0 \ub530\ub77c \uc774 \uad00\uc810\uc774 \ubb34\ub108\uc9c0\uae30 \uc2dc\uc791\ud558\uc600\ub2e4. \ud55c\ud3b8\uc73c\ub85c \ub2f9\uc2dc \uc544\uc774\ub4e4\uc744 \ub3cc\ubcf4\ub294 \uc758\ub8cc\uc778\uc774\uc5c8\ub358 \ub80c \uc2a4\ud53c\uce20 (Rene Spitz) \ubc15\uc0ac\ub294, \ud2b9\uc815 \uc601\uc544 \ubc0f \uc720\uc544\ub4e4\uc758 \uac11\uc791\uc2a4\ub7f0 \uc0ac\ub9dd\uc5d0 \ub300\ud55c \uc6d0\uc778 \uaddc\uba85\uc5d0 \ucc29\uc218\ud558\uc600\ub2e4. 1958-1962\ub144\uc774 \ub418\uc5b4 \ud574\ub9ac \ud560\ub85c\uc6b0 (Harry Harlow)\uc758 \uc5f0\uad6c\uac00 \uc774\ub904\uc9c0\uace0 \ub098\uc11c\uc57c \uc6d0\uc778\uc774 \uaddc\uba85\ub418\uc5c8\ub2e4. \uc544\uc774\ub4e4\uc740 \ucd09\uac01\uc744 \uac70\uc758 \ub290\ub07c\uc9c0 \ubabb\ud588\ub358 \uac83\uc774\ub2e4. \uc6d0\uc22d\uc774 \uc0c8\ub07c\ub4e4\uc744 \uac00\uc9c0\uace0 \ud55c \ud560\ub85c\uc6b0\uc758 \uc5f0\uad6c\uc5d0 \ub530\ub77c \uadf8\ub294 \ucd09\uac01\uc758 \uc911\uc694\uc131\uc744 \uae68\ub2eb\uac8c \ub418\uc5c8\ub2e4. \uc774\ud6c4 \uc874 \ubcfc\ube44 (John Bowlby)\uc640 \ub9e4\ub9ac \uc124\ud130 \uc560\uc778\uc988\uc6cc\uc2a4 (Mary Salter Ainsworth)\uc758 \uc5f0\uad6c\ub97c \ud1b5\ud574 \uc560\ucc29 \uc774\ub860\uacfc \uad00\ub828\ud558\uc5ec \ucd09\uac01\uc774 \uc5b4\ub5a4 \uc601\ud5a5\uc744 \ubbf8\uce58\ub294\uc9c0\uac00 \ud655\uc778\ub418\uc5c8\ub2e4. \ub85c\ubc84\ud2b8 \ud574\ud2b8\ud544\ub4dc\ub294 \uc774\ub7f0 \uc2e4\ud5d8\ub4e4\uc758 \uacb0\uacfc\ub97c \ub17c\ud558\uba74\uc11c \u201c\uc560\uc815\uc774 \ub2f4\uae34 \uc190\uae38\uacfc \ubb34\uc2dc, \ubc8c\uc744 \uc8fc\ub294 \uc190\uae38 \uac04\uc758 \ucc28\uc774\ub294 \uc560\ucc29 \uc774\ub860\uc758 \uc911\uc2ec \ud654\ub450\uc774\uba70, \uc774\ub7f0 \uc5f0\uad6c\ub4e4\uc740 \ud560\ub85c\uc6b0\uc758 \uc5f0\uad6c\ub97c \uc778\uac04\uc5d0\uac8c \uc801\uc6a9\ud55c \uac83\uc774\ub77c \ubcfc \uc218 \uc788\ub2e4.\u201d\ub77c\uace0 \uc8fc\uc7a5\ud558\uc600\ub2e4.", "sentence_answer": "\uc774\ud6c4 \uc874 \ubcfc\ube44 (John Bowlby)\uc640 \ub9e4\ub9ac \uc124\ud130 \uc560\uc778\uc988\uc6cc\uc2a4 (Mary Salter Ainsworth)\uc758 \uc5f0\uad6c\ub97c \ud1b5\ud574 \uc560\ucc29 \uc774\ub860\uacfc \uad00\ub828\ud558\uc5ec \ucd09\uac01\uc774 \uc5b4\ub5a4 \uc601\ud5a5\uc744 \ubbf8\uce58\ub294\uc9c0\uac00 \ud655\uc778\ub418\uc5c8\ub2e4.", "paragraph_id": "6504660-3-2", "paragraph_question": "question: \ucd09\uac01\uacfc \uc560\ucc29\uc774\ub860\uc758 \uad00\ub828\uc131\uc5d0 \ub300\ud558\uc5ec \ub9e4\ub9ac \uc124\ud130 \uc560\uc778\uc988\uc6cc\uc2a4\uc640 \uacf5\ub3d9\uc5f0\uad6c\ub97c \uc9c4\ud589\ud55c \uc0ac\ub78c\uc740 \ub204\uad6c\uc778\uac00?, context: \uac10\uac01 \uc0c1\uc2e4 (sense deprivation)\uc774\ub780 \uc5b4\ub5a4 \uc0ac\ub78c\uc774 \ucd09\uac01\uc744 \uac70\uc758 \ub290\ub07c\uc9c0 \ubabb\ud588\uc744 \ub54c\uc758 \ud604\uc0c1\uc774\uba70, \ud2b9\ud788 \uc601\uc544\uae30 \ub54c \ud070 \ubb38\uc81c\uac00 \ub41c\ub2e4. \uc601\uc544\uc758 \uac10\uac01 \uc0c1\uc2e4\uc740 \uc774\ud6c4 \ud589\ub3d9\uc801, \uac74\uac15\uc801, \uc0dd\ub9ac\uc801 \ubc1c\ub2ec\uc5d0 \uc788\uc5b4 \uc5ec\ub7ec \uac00\uc9c0 \ubb38\uc81c\ub97c \ub0b3\uac8c \ub41c\ub2e4. \uc544\uc27d\uac8c\ub3c4 \uc774 \ubd84\uc57c\ub294 \uc5f0\uad6c\uac00 \uae4a\uc774 \ub418\uc9c0 \uc54a\uc544 \uadf8 \uc601\ud5a5\uc5d0 \ub300\ud574 \uc54c\ub824\uc9c4 \uac83\uc774 \ub9ce\uc9c0\ub294 \uc54a\ub2e4. \uc5f0\uad6c\uac00 \uc9c4\ud589\ub418\uae30 \uc804, \ucd09\uac01\uc740 \uc0ac\ub78c\uc758 \ubc1c\ub2ec\uc5d0 \uc788\uc5b4 \ubbf8\ubbf8\ud55c \uc601\ud5a5\uc744 \ub07c\uce58\ub294 \uac83\uc73c\ub85c \uc5ec\uaca8\uc84c\ub2e4. \ud558\uc9c0\ub9cc 1945-47\ub144, \uc2e0\uc2dc\ub0b4\ud2f0 \ub300\ud559\uc758 \ub85c\ubc84\ud2b8 \ud574\ud2b8\ud544\ub4dc (Robert Hatfield) \ubc15\uc0ac\uc758 \ub17c\ubb38\uc5d0 \ub530\ub77c \uc774 \uad00\uc810\uc774 \ubb34\ub108\uc9c0\uae30 \uc2dc\uc791\ud558\uc600\ub2e4. \ud55c\ud3b8\uc73c\ub85c \ub2f9\uc2dc \uc544\uc774\ub4e4\uc744 \ub3cc\ubcf4\ub294 \uc758\ub8cc\uc778\uc774\uc5c8\ub358 \ub80c \uc2a4\ud53c\uce20 (Rene Spitz) \ubc15\uc0ac\ub294, \ud2b9\uc815 \uc601\uc544 \ubc0f \uc720\uc544\ub4e4\uc758 \uac11\uc791\uc2a4\ub7f0 \uc0ac\ub9dd\uc5d0 \ub300\ud55c \uc6d0\uc778 \uaddc\uba85\uc5d0 \ucc29\uc218\ud558\uc600\ub2e4. 1958-1962\ub144\uc774 \ub418\uc5b4 \ud574\ub9ac \ud560\ub85c\uc6b0 (Harry Harlow)\uc758 \uc5f0\uad6c\uac00 \uc774\ub904\uc9c0\uace0 \ub098\uc11c\uc57c \uc6d0\uc778\uc774 \uaddc\uba85\ub418\uc5c8\ub2e4. \uc544\uc774\ub4e4\uc740 \ucd09\uac01\uc744 \uac70\uc758 \ub290\ub07c\uc9c0 \ubabb\ud588\ub358 \uac83\uc774\ub2e4. \uc6d0\uc22d\uc774 \uc0c8\ub07c\ub4e4\uc744 \uac00\uc9c0\uace0 \ud55c \ud560\ub85c\uc6b0\uc758 \uc5f0\uad6c\uc5d0 \ub530\ub77c \uadf8\ub294 \ucd09\uac01\uc758 \uc911\uc694\uc131\uc744 \uae68\ub2eb\uac8c \ub418\uc5c8\ub2e4. \uc774\ud6c4 \uc874 \ubcfc\ube44 (John Bowlby)\uc640 \ub9e4\ub9ac \uc124\ud130 \uc560\uc778\uc988\uc6cc\uc2a4 (Mary Salter Ainsworth)\uc758 \uc5f0\uad6c\ub97c \ud1b5\ud574 \uc560\ucc29 \uc774\ub860\uacfc \uad00\ub828\ud558\uc5ec \ucd09\uac01\uc774 \uc5b4\ub5a4 \uc601\ud5a5\uc744 \ubbf8\uce58\ub294\uc9c0\uac00 \ud655\uc778\ub418\uc5c8\ub2e4. \ub85c\ubc84\ud2b8 \ud574\ud2b8\ud544\ub4dc\ub294 \uc774\ub7f0 \uc2e4\ud5d8\ub4e4\uc758 \uacb0\uacfc\ub97c \ub17c\ud558\uba74\uc11c \u201c\uc560\uc815\uc774 \ub2f4\uae34 \uc190\uae38\uacfc \ubb34\uc2dc, \ubc8c\uc744 \uc8fc\ub294 \uc190\uae38 \uac04\uc758 \ucc28\uc774\ub294 \uc560\ucc29 \uc774\ub860\uc758 \uc911\uc2ec \ud654\ub450\uc774\uba70, \uc774\ub7f0 \uc5f0\uad6c\ub4e4\uc740 \ud560\ub85c\uc6b0\uc758 \uc5f0\uad6c\ub97c \uc778\uac04\uc5d0\uac8c \uc801\uc6a9\ud55c \uac83\uc774\ub77c \ubcfc \uc218 \uc788\ub2e4.\u201d\ub77c\uace0 \uc8fc\uc7a5\ud558\uc600\ub2e4."} {"question": "2006\ub144\uc5d0 \ud3ec\uc720\ub958\uc548\uc758 \uc784\ud30c \uc870\uc9c1\uacfc \ube44\uc2b7\ud55c \uc784\ud30c\uc870\uc9c1\uc774 \uacbd\uace8\uc5b4\ub958\ub4e4 \uc911 \uc5b4\ub514\uc5d0\uc11c \ubc1c\uacac\ub418\uc5c8\ub294\uac00?", "paragraph": "\uc870\uae30\uc5b4\ub958\ucc98\ub7fc, \uacbd\uace8\uc5b4\ub958\uc758 \uc8fc\ub41c \uba74\uc5ed \uc870\uc9c1\uc740 \uc2e0\uc7a5, \ud2b9\ud788 \ub9ce\uc740 \ub2e4\ub978 \uba74\uc5ed \uc138\ud3ec\ub4e4\uc744 \ud3ec\ud568\ud558\ub294 \uc2e0\uc7a5\uc804\uc5fd(\uc601\uc5b4: anterior kidney)\uc744 \ud3ec\ud568\ud55c\ub2e4. \ub354\ud558\uc5ec \uacbd\uace8\uc5b4\ub958\ub294 \ud749\uc120(\uc601\uc5b4: thymus), \ube44\uc7a5(\uc601\uc5b4: spleen) \uadf8\ub9ac\uace0 \ud53c\ubd80\uc640 \uc544\uac00\ubbf8, \ub0b4\uc7a5, \uadf8\ub9ac\uace0 \uc0dd\uc2dd\uc120\uacfc \uac19\uc740 \uc810\uc561\uc9c8\uc758 \uc870\uc9c1 \uc548\uc5d0 \uc0b0\uc7ac\ub41c \uba74\uc5ed\ubd80\ub97c \uac16\ub294\ub2e4. \ubcf4\ub2e4 \ud3ec\uc720\ub958\uc758 \uba74\uc5ed\uccb4\uacc4\uc640 \ube44\uc2b7\ud558\uac8c \uacbd\uace8\uc5b4\ub958\uc758 \uc801\ud608\uad6c(\uc601\uc5b4: erythrocyte), \ud638\uc911\uc131 \ubc31\ud608\uad6c(\uc601\uc5b4: neutrophil) \uadf8\ub9ac\uace0 \uacfc\ub9bd\uc131 \ubc31\ud608\uad6c(\uc601\uc5b4: granulocyte)\ub4e4\uc740, \ud749\uc120\uc548\uc5d0\uc11c \ubc1c\uacac\ub418\ub294 \uc784\ud30c\uad6c\uac00 \uc8fc\ub41c \uc138\ud3ec\ud615\ud0dc\uc778, \ube44\uc7a5 \uc548\uc758 \uc5b4\ub514\uc5d0\ub4e0 \uc874\uc7ac\ud55c\ub2e4\uace0 \ubbff\uc5b4\uc9c4\ub2e4. 2006\ub144, \ud3ec\uc720\ub958\uc548\uc758 \uc784\ud30c \uc870\uc9c1\uacfc \ube44\uc2b7\ud55c \uc784\ud30c\uc870\uc9c1\uc774 \uacbd\uace8\uc5b4\ub958\uc758 \uc885\ub4e4 \uc911 \ud558\ub098\uc778 \uc81c\ube0c\ub77c \ub2e4\ub2c8\uc624\uc5d0\uc11c \ubc1c\uacac\ub418\uc5c8\ub2e4. \uc544\uc9c1 \ud655\uc815\ub418\uc9c0\ub294 \uc54a\uc558\uc74c\uc5d0\ub3c4 \ubd88\uad6c\ud558\uace0, \uc774 \uc870\uc9c1\uc740 \ud56d\uc6d0\uc744 \uae30\ub2e4\ub9ac\ub294 \ub3d9\uc548 \uc544\uc9c1 \uc54c\ub824\uc9c0\uc9c0 \uc54a\uc740, \uc989 \uc544\uc9c1 \uc790\uadf9 \ubc1b\uc9c0 \uc54a\uc740 T \uc138\ud3ec\ub4e4\uc774 \ucd95\uc801\ub418\ub294 \uc7a5\uc18c\uc77c \uac83\uc73c\ub85c \uc5ec\uaca8\uc9c4\ub2e4.", "answer": "\uc81c\ube0c\ub77c \ub2e4\ub2c8\uc624", "sentence": "2006\ub144, \ud3ec\uc720\ub958\uc548\uc758 \uc784\ud30c \uc870\uc9c1\uacfc \ube44\uc2b7\ud55c \uc784\ud30c\uc870\uc9c1\uc774 \uacbd\uace8\uc5b4\ub958\uc758 \uc885\ub4e4 \uc911 \ud558\ub098\uc778 \uc81c\ube0c\ub77c \ub2e4\ub2c8\uc624 \uc5d0\uc11c \ubc1c\uacac\ub418\uc5c8\ub2e4.", "paragraph_sentence": "\uc870\uae30\uc5b4\ub958\ucc98\ub7fc, \uacbd\uace8\uc5b4\ub958\uc758 \uc8fc\ub41c \uba74\uc5ed \uc870\uc9c1\uc740 \uc2e0\uc7a5, \ud2b9\ud788 \ub9ce\uc740 \ub2e4\ub978 \uba74\uc5ed \uc138\ud3ec\ub4e4\uc744 \ud3ec\ud568\ud558\ub294 \uc2e0\uc7a5\uc804\uc5fd(\uc601\uc5b4: anterior kidney)\uc744 \ud3ec\ud568\ud55c\ub2e4. \ub354\ud558\uc5ec \uacbd\uace8\uc5b4\ub958\ub294 \ud749\uc120(\uc601\uc5b4: thymus), \ube44\uc7a5(\uc601\uc5b4: spleen) \uadf8\ub9ac\uace0 \ud53c\ubd80\uc640 \uc544\uac00\ubbf8, \ub0b4\uc7a5, \uadf8\ub9ac\uace0 \uc0dd\uc2dd\uc120\uacfc \uac19\uc740 \uc810\uc561\uc9c8\uc758 \uc870\uc9c1 \uc548\uc5d0 \uc0b0\uc7ac\ub41c \uba74\uc5ed\ubd80\ub97c \uac16\ub294\ub2e4. \ubcf4\ub2e4 \ud3ec\uc720\ub958\uc758 \uba74\uc5ed\uccb4\uacc4\uc640 \ube44\uc2b7\ud558\uac8c \uacbd\uace8\uc5b4\ub958\uc758 \uc801\ud608\uad6c(\uc601\uc5b4: erythrocyte), \ud638\uc911\uc131 \ubc31\ud608\uad6c(\uc601\uc5b4: neutrophil) \uadf8\ub9ac\uace0 \uacfc\ub9bd\uc131 \ubc31\ud608\uad6c(\uc601\uc5b4: granulocyte)\ub4e4\uc740, \ud749\uc120\uc548\uc5d0\uc11c \ubc1c\uacac\ub418\ub294 \uc784\ud30c\uad6c\uac00 \uc8fc\ub41c \uc138\ud3ec\ud615\ud0dc\uc778, \ube44\uc7a5 \uc548\uc758 \uc5b4\ub514\uc5d0\ub4e0 \uc874\uc7ac\ud55c\ub2e4\uace0 \ubbff\uc5b4\uc9c4\ub2e4. 2006\ub144, \ud3ec\uc720\ub958\uc548\uc758 \uc784\ud30c \uc870\uc9c1\uacfc \ube44\uc2b7\ud55c \uc784\ud30c\uc870\uc9c1\uc774 \uacbd\uace8\uc5b4\ub958\uc758 \uc885\ub4e4 \uc911 \ud558\ub098\uc778 \uc81c\ube0c\ub77c \ub2e4\ub2c8\uc624 \uc5d0\uc11c \ubc1c\uacac\ub418\uc5c8\ub2e4. \uc544\uc9c1 \ud655\uc815\ub418\uc9c0\ub294 \uc54a\uc558\uc74c\uc5d0\ub3c4 \ubd88\uad6c\ud558\uace0, \uc774 \uc870\uc9c1\uc740 \ud56d\uc6d0\uc744 \uae30\ub2e4\ub9ac\ub294 \ub3d9\uc548 \uc544\uc9c1 \uc54c\ub824\uc9c0\uc9c0 \uc54a\uc740, \uc989 \uc544\uc9c1 \uc790\uadf9 \ubc1b\uc9c0 \uc54a\uc740 T \uc138\ud3ec\ub4e4\uc774 \ucd95\uc801\ub418\ub294 \uc7a5\uc18c\uc77c \uac83\uc73c\ub85c \uc5ec\uaca8\uc9c4\ub2e4.", "paragraph_answer": "\uc870\uae30\uc5b4\ub958\ucc98\ub7fc, \uacbd\uace8\uc5b4\ub958\uc758 \uc8fc\ub41c \uba74\uc5ed \uc870\uc9c1\uc740 \uc2e0\uc7a5, \ud2b9\ud788 \ub9ce\uc740 \ub2e4\ub978 \uba74\uc5ed \uc138\ud3ec\ub4e4\uc744 \ud3ec\ud568\ud558\ub294 \uc2e0\uc7a5\uc804\uc5fd(\uc601\uc5b4: anterior kidney)\uc744 \ud3ec\ud568\ud55c\ub2e4. \ub354\ud558\uc5ec \uacbd\uace8\uc5b4\ub958\ub294 \ud749\uc120(\uc601\uc5b4: thymus), \ube44\uc7a5(\uc601\uc5b4: spleen) \uadf8\ub9ac\uace0 \ud53c\ubd80\uc640 \uc544\uac00\ubbf8, \ub0b4\uc7a5, \uadf8\ub9ac\uace0 \uc0dd\uc2dd\uc120\uacfc \uac19\uc740 \uc810\uc561\uc9c8\uc758 \uc870\uc9c1 \uc548\uc5d0 \uc0b0\uc7ac\ub41c \uba74\uc5ed\ubd80\ub97c \uac16\ub294\ub2e4. \ubcf4\ub2e4 \ud3ec\uc720\ub958\uc758 \uba74\uc5ed\uccb4\uacc4\uc640 \ube44\uc2b7\ud558\uac8c \uacbd\uace8\uc5b4\ub958\uc758 \uc801\ud608\uad6c(\uc601\uc5b4: erythrocyte), \ud638\uc911\uc131 \ubc31\ud608\uad6c(\uc601\uc5b4: neutrophil) \uadf8\ub9ac\uace0 \uacfc\ub9bd\uc131 \ubc31\ud608\uad6c(\uc601\uc5b4: granulocyte)\ub4e4\uc740, \ud749\uc120\uc548\uc5d0\uc11c \ubc1c\uacac\ub418\ub294 \uc784\ud30c\uad6c\uac00 \uc8fc\ub41c \uc138\ud3ec\ud615\ud0dc\uc778, \ube44\uc7a5 \uc548\uc758 \uc5b4\ub514\uc5d0\ub4e0 \uc874\uc7ac\ud55c\ub2e4\uace0 \ubbff\uc5b4\uc9c4\ub2e4. 2006\ub144, \ud3ec\uc720\ub958\uc548\uc758 \uc784\ud30c \uc870\uc9c1\uacfc \ube44\uc2b7\ud55c \uc784\ud30c\uc870\uc9c1\uc774 \uacbd\uace8\uc5b4\ub958\uc758 \uc885\ub4e4 \uc911 \ud558\ub098\uc778 \uc81c\ube0c\ub77c \ub2e4\ub2c8\uc624 \uc5d0\uc11c \ubc1c\uacac\ub418\uc5c8\ub2e4. \uc544\uc9c1 \ud655\uc815\ub418\uc9c0\ub294 \uc54a\uc558\uc74c\uc5d0\ub3c4 \ubd88\uad6c\ud558\uace0, \uc774 \uc870\uc9c1\uc740 \ud56d\uc6d0\uc744 \uae30\ub2e4\ub9ac\ub294 \ub3d9\uc548 \uc544\uc9c1 \uc54c\ub824\uc9c0\uc9c0 \uc54a\uc740, \uc989 \uc544\uc9c1 \uc790\uadf9 \ubc1b\uc9c0 \uc54a\uc740 T \uc138\ud3ec\ub4e4\uc774 \ucd95\uc801\ub418\ub294 \uc7a5\uc18c\uc77c \uac83\uc73c\ub85c \uc5ec\uaca8\uc9c4\ub2e4.", "sentence_answer": "2006\ub144, \ud3ec\uc720\ub958\uc548\uc758 \uc784\ud30c \uc870\uc9c1\uacfc \ube44\uc2b7\ud55c \uc784\ud30c\uc870\uc9c1\uc774 \uacbd\uace8\uc5b4\ub958\uc758 \uc885\ub4e4 \uc911 \ud558\ub098\uc778 \uc81c\ube0c\ub77c \ub2e4\ub2c8\uc624 \uc5d0\uc11c \ubc1c\uacac\ub418\uc5c8\ub2e4.", "paragraph_id": "6505170-11-1", "paragraph_question": "question: 2006\ub144\uc5d0 \ud3ec\uc720\ub958\uc548\uc758 \uc784\ud30c \uc870\uc9c1\uacfc \ube44\uc2b7\ud55c \uc784\ud30c\uc870\uc9c1\uc774 \uacbd\uace8\uc5b4\ub958\ub4e4 \uc911 \uc5b4\ub514\uc5d0\uc11c \ubc1c\uacac\ub418\uc5c8\ub294\uac00?, context: \uc870\uae30\uc5b4\ub958\ucc98\ub7fc, \uacbd\uace8\uc5b4\ub958\uc758 \uc8fc\ub41c \uba74\uc5ed \uc870\uc9c1\uc740 \uc2e0\uc7a5, \ud2b9\ud788 \ub9ce\uc740 \ub2e4\ub978 \uba74\uc5ed \uc138\ud3ec\ub4e4\uc744 \ud3ec\ud568\ud558\ub294 \uc2e0\uc7a5\uc804\uc5fd(\uc601\uc5b4: anterior kidney)\uc744 \ud3ec\ud568\ud55c\ub2e4. \ub354\ud558\uc5ec \uacbd\uace8\uc5b4\ub958\ub294 \ud749\uc120(\uc601\uc5b4: thymus), \ube44\uc7a5(\uc601\uc5b4: spleen) \uadf8\ub9ac\uace0 \ud53c\ubd80\uc640 \uc544\uac00\ubbf8, \ub0b4\uc7a5, \uadf8\ub9ac\uace0 \uc0dd\uc2dd\uc120\uacfc \uac19\uc740 \uc810\uc561\uc9c8\uc758 \uc870\uc9c1 \uc548\uc5d0 \uc0b0\uc7ac\ub41c \uba74\uc5ed\ubd80\ub97c \uac16\ub294\ub2e4. \ubcf4\ub2e4 \ud3ec\uc720\ub958\uc758 \uba74\uc5ed\uccb4\uacc4\uc640 \ube44\uc2b7\ud558\uac8c \uacbd\uace8\uc5b4\ub958\uc758 \uc801\ud608\uad6c(\uc601\uc5b4: erythrocyte), \ud638\uc911\uc131 \ubc31\ud608\uad6c(\uc601\uc5b4: neutrophil) \uadf8\ub9ac\uace0 \uacfc\ub9bd\uc131 \ubc31\ud608\uad6c(\uc601\uc5b4: granulocyte)\ub4e4\uc740, \ud749\uc120\uc548\uc5d0\uc11c \ubc1c\uacac\ub418\ub294 \uc784\ud30c\uad6c\uac00 \uc8fc\ub41c \uc138\ud3ec\ud615\ud0dc\uc778, \ube44\uc7a5 \uc548\uc758 \uc5b4\ub514\uc5d0\ub4e0 \uc874\uc7ac\ud55c\ub2e4\uace0 \ubbff\uc5b4\uc9c4\ub2e4. 2006\ub144, \ud3ec\uc720\ub958\uc548\uc758 \uc784\ud30c \uc870\uc9c1\uacfc \ube44\uc2b7\ud55c \uc784\ud30c\uc870\uc9c1\uc774 \uacbd\uace8\uc5b4\ub958\uc758 \uc885\ub4e4 \uc911 \ud558\ub098\uc778 \uc81c\ube0c\ub77c \ub2e4\ub2c8\uc624\uc5d0\uc11c \ubc1c\uacac\ub418\uc5c8\ub2e4. \uc544\uc9c1 \ud655\uc815\ub418\uc9c0\ub294 \uc54a\uc558\uc74c\uc5d0\ub3c4 \ubd88\uad6c\ud558\uace0, \uc774 \uc870\uc9c1\uc740 \ud56d\uc6d0\uc744 \uae30\ub2e4\ub9ac\ub294 \ub3d9\uc548 \uc544\uc9c1 \uc54c\ub824\uc9c0\uc9c0 \uc54a\uc740, \uc989 \uc544\uc9c1 \uc790\uadf9 \ubc1b\uc9c0 \uc54a\uc740 T \uc138\ud3ec\ub4e4\uc774 \ucd95\uc801\ub418\ub294 \uc7a5\uc18c\uc77c \uac83\uc73c\ub85c \uc5ec\uaca8\uc9c4\ub2e4."} {"question": "2NE1\uc740 \ub370\ubdd4\ub97c \uc704\ud558\uc5ec \uba87 \ub144\ub3d9\uc548 \uc900\ube44\ud588\ub294\uac00?", "paragraph": "2NE1\uc740 YG \uc5d4\ud130\ud14c\uc778\uba3c\ud2b8\uc5d0\uc11c 10\ub144\ub9cc\uc5d0 \ub098\uc624\ub294 \uac78 \uadf8\ub8f9\uc73c\ub85c, 4\ub144\ub3d9\uc548 \uc900\ube44\ud574\uc654\ub2e4. \uc5f0\uc2b5\uc0dd \uc2dc\uc808 \ud2b9\ubcc4\ud55c \uc77c\uc815\uc774 \uc5c6\uc73c\uba74 \ud558\ub8e8 12\uc2dc\uac04\uc529 \ud2b8\ub808\uc774\ub2dd\uc744 \ubc1b\uc558\ub294\ub370, \ub2e4\ub978 \uc544\uc774\ub3cc\uacfc\ub294 \ub2ec\ub9ac \ud569\uc219\uc744 \ud558\uc9c0 \uc54a\uc558\ub2e4. \uc774\ud6c4 \ub370\ubdd4 \uc804\ubd80\ud130 \uc778\ud130\ub137\uc0c1\uc5d0\uc11c \ud654\uc81c\uac00 \ub418\uba70 '\uc5ec\uc790 \ube45\ubc45'\uc774\ub77c\uace0 \ubd88\ub838\ub2e4. \ubcf8\ub798 \uadf8\ub8f9\uba85\uc740 21\uc774\uc600\uc73c\ub098, \uc774\ubbf8 2005\ub144 21(To Anyone)\uc774\ub77c\ub294 \ub0a8\uc790 \uc194\ub85c \uac00\uc218\uac00 \uc788\ub358 \uac83\uc73c\ub85c \uc54c\ub824\uc838 \ud604\uc7ac\uc758 2NE1\uc73c\ub85c \uadf8\ub8f9\uba85\uc744 \ubc14\uafe8\ub2e4. \uadf8\ub8f9\uba85\uc73c\ub85c \uc4f0\ub098\ubbf8, \ucfe8\uce59\uc2a4, \uc591\uc2f8\uc758 \uc544\uc774\ub4e4, YG\uac78\uc2a4 \ub4f1 \ub2e4\uc591\ud55c \uc774\ub984\uc774 \uac70\ub860 \ub418\uc5c8\uc73c\ub098, \uadf8\ub8f9\uba85 \uacf5\uac1c \uba87 \uc2dc\uac04 \uc804 \uc0c8\ubcbd\uc5d0 2NE1\uc774\ub77c\ub294 \uc774\ub984\uc73c\ub85c \uae09\ud558\uac8c \uc815\ud574\uc84c\ub2e4. 2NE1\uc774\ub77c\ub294 \uadf8\ub8f9\uba85\uc740 \"21\uc138\uae30\uc758 \uc0c8\ub85c\uc6b4 \uc9c4\ud654 (21C New Evolution)\", \"21\uc0b4\uc758 \ub098\uc774\ucc98\ub7fc \ud56d\uc0c1 \ub3c4\uc804\uc801\uc774\uace0 \uc2e0\uc120\ud55c \uc74c\uc545\uc744 \uc120\ubcf4\uc778\ub2e4\"\ub294 \uc758\ubbf8\ub97c \uac00\uc9c0\uace0 \uc788\ub2e4.", "answer": "4\ub144", "sentence": "2NE1\uc740 YG \uc5d4\ud130\ud14c\uc778\uba3c\ud2b8\uc5d0\uc11c 10\ub144\ub9cc\uc5d0 \ub098\uc624\ub294 \uac78 \uadf8\ub8f9\uc73c\ub85c, 4\ub144 \ub3d9\uc548 \uc900\ube44\ud574\uc654\ub2e4.", "paragraph_sentence": " 2NE1\uc740 YG \uc5d4\ud130\ud14c\uc778\uba3c\ud2b8\uc5d0\uc11c 10\ub144\ub9cc\uc5d0 \ub098\uc624\ub294 \uac78 \uadf8\ub8f9\uc73c\ub85c, 4\ub144 \ub3d9\uc548 \uc900\ube44\ud574\uc654\ub2e4. \uc5f0\uc2b5\uc0dd \uc2dc\uc808 \ud2b9\ubcc4\ud55c \uc77c\uc815\uc774 \uc5c6\uc73c\uba74 \ud558\ub8e8 12\uc2dc\uac04\uc529 \ud2b8\ub808\uc774\ub2dd\uc744 \ubc1b\uc558\ub294\ub370, \ub2e4\ub978 \uc544\uc774\ub3cc\uacfc\ub294 \ub2ec\ub9ac \ud569\uc219\uc744 \ud558\uc9c0 \uc54a\uc558\ub2e4. \uc774\ud6c4 \ub370\ubdd4 \uc804\ubd80\ud130 \uc778\ud130\ub137\uc0c1\uc5d0\uc11c \ud654\uc81c\uac00 \ub418\uba70 '\uc5ec\uc790 \ube45\ubc45'\uc774\ub77c\uace0 \ubd88\ub838\ub2e4. \ubcf8\ub798 \uadf8\ub8f9\uba85\uc740 21\uc774\uc600\uc73c\ub098, \uc774\ubbf8 2005\ub144 21(To Anyone)\uc774\ub77c\ub294 \ub0a8\uc790 \uc194\ub85c \uac00\uc218\uac00 \uc788\ub358 \uac83\uc73c\ub85c \uc54c\ub824\uc838 \ud604\uc7ac\uc758 2NE1\uc73c\ub85c \uadf8\ub8f9\uba85\uc744 \ubc14\uafe8\ub2e4. \uadf8\ub8f9\uba85\uc73c\ub85c \uc4f0\ub098\ubbf8, \ucfe8\uce59\uc2a4, \uc591\uc2f8\uc758 \uc544\uc774\ub4e4, YG\uac78\uc2a4 \ub4f1 \ub2e4\uc591\ud55c \uc774\ub984\uc774 \uac70\ub860 \ub418\uc5c8\uc73c\ub098, \uadf8\ub8f9\uba85 \uacf5\uac1c \uba87 \uc2dc\uac04 \uc804 \uc0c8\ubcbd\uc5d0 2NE1\uc774\ub77c\ub294 \uc774\ub984\uc73c\ub85c \uae09\ud558\uac8c \uc815\ud574\uc84c\ub2e4. 2NE1\uc774\ub77c\ub294 \uadf8\ub8f9\uba85\uc740 \"21\uc138\uae30\uc758 \uc0c8\ub85c\uc6b4 \uc9c4\ud654 (21C New Evolution)\", \"21\uc0b4\uc758 \ub098\uc774\ucc98\ub7fc \ud56d\uc0c1 \ub3c4\uc804\uc801\uc774\uace0 \uc2e0\uc120\ud55c \uc74c\uc545\uc744 \uc120\ubcf4\uc778\ub2e4\"\ub294 \uc758\ubbf8\ub97c \uac00\uc9c0\uace0 \uc788\ub2e4.", "paragraph_answer": "2NE1\uc740 YG \uc5d4\ud130\ud14c\uc778\uba3c\ud2b8\uc5d0\uc11c 10\ub144\ub9cc\uc5d0 \ub098\uc624\ub294 \uac78 \uadf8\ub8f9\uc73c\ub85c, 4\ub144 \ub3d9\uc548 \uc900\ube44\ud574\uc654\ub2e4. \uc5f0\uc2b5\uc0dd \uc2dc\uc808 \ud2b9\ubcc4\ud55c \uc77c\uc815\uc774 \uc5c6\uc73c\uba74 \ud558\ub8e8 12\uc2dc\uac04\uc529 \ud2b8\ub808\uc774\ub2dd\uc744 \ubc1b\uc558\ub294\ub370, \ub2e4\ub978 \uc544\uc774\ub3cc\uacfc\ub294 \ub2ec\ub9ac \ud569\uc219\uc744 \ud558\uc9c0 \uc54a\uc558\ub2e4. \uc774\ud6c4 \ub370\ubdd4 \uc804\ubd80\ud130 \uc778\ud130\ub137\uc0c1\uc5d0\uc11c \ud654\uc81c\uac00 \ub418\uba70 '\uc5ec\uc790 \ube45\ubc45'\uc774\ub77c\uace0 \ubd88\ub838\ub2e4. \ubcf8\ub798 \uadf8\ub8f9\uba85\uc740 21\uc774\uc600\uc73c\ub098, \uc774\ubbf8 2005\ub144 21(To Anyone)\uc774\ub77c\ub294 \ub0a8\uc790 \uc194\ub85c \uac00\uc218\uac00 \uc788\ub358 \uac83\uc73c\ub85c \uc54c\ub824\uc838 \ud604\uc7ac\uc758 2NE1\uc73c\ub85c \uadf8\ub8f9\uba85\uc744 \ubc14\uafe8\ub2e4. \uadf8\ub8f9\uba85\uc73c\ub85c \uc4f0\ub098\ubbf8, \ucfe8\uce59\uc2a4, \uc591\uc2f8\uc758 \uc544\uc774\ub4e4, YG\uac78\uc2a4 \ub4f1 \ub2e4\uc591\ud55c \uc774\ub984\uc774 \uac70\ub860 \ub418\uc5c8\uc73c\ub098, \uadf8\ub8f9\uba85 \uacf5\uac1c \uba87 \uc2dc\uac04 \uc804 \uc0c8\ubcbd\uc5d0 2NE1\uc774\ub77c\ub294 \uc774\ub984\uc73c\ub85c \uae09\ud558\uac8c \uc815\ud574\uc84c\ub2e4. 2NE1\uc774\ub77c\ub294 \uadf8\ub8f9\uba85\uc740 \"21\uc138\uae30\uc758 \uc0c8\ub85c\uc6b4 \uc9c4\ud654 (21C New Evolution)\", \"21\uc0b4\uc758 \ub098\uc774\ucc98\ub7fc \ud56d\uc0c1 \ub3c4\uc804\uc801\uc774\uace0 \uc2e0\uc120\ud55c \uc74c\uc545\uc744 \uc120\ubcf4\uc778\ub2e4\"\ub294 \uc758\ubbf8\ub97c \uac00\uc9c0\uace0 \uc788\ub2e4.", "sentence_answer": "2NE1\uc740 YG \uc5d4\ud130\ud14c\uc778\uba3c\ud2b8\uc5d0\uc11c 10\ub144\ub9cc\uc5d0 \ub098\uc624\ub294 \uac78 \uadf8\ub8f9\uc73c\ub85c, 4\ub144 \ub3d9\uc548 \uc900\ube44\ud574\uc654\ub2e4.", "paragraph_id": "6471356-2-1", "paragraph_question": "question: 2NE1\uc740 \ub370\ubdd4\ub97c \uc704\ud558\uc5ec \uba87 \ub144\ub3d9\uc548 \uc900\ube44\ud588\ub294\uac00?, context: 2NE1\uc740 YG \uc5d4\ud130\ud14c\uc778\uba3c\ud2b8\uc5d0\uc11c 10\ub144\ub9cc\uc5d0 \ub098\uc624\ub294 \uac78 \uadf8\ub8f9\uc73c\ub85c, 4\ub144\ub3d9\uc548 \uc900\ube44\ud574\uc654\ub2e4. \uc5f0\uc2b5\uc0dd \uc2dc\uc808 \ud2b9\ubcc4\ud55c \uc77c\uc815\uc774 \uc5c6\uc73c\uba74 \ud558\ub8e8 12\uc2dc\uac04\uc529 \ud2b8\ub808\uc774\ub2dd\uc744 \ubc1b\uc558\ub294\ub370, \ub2e4\ub978 \uc544\uc774\ub3cc\uacfc\ub294 \ub2ec\ub9ac \ud569\uc219\uc744 \ud558\uc9c0 \uc54a\uc558\ub2e4. \uc774\ud6c4 \ub370\ubdd4 \uc804\ubd80\ud130 \uc778\ud130\ub137\uc0c1\uc5d0\uc11c \ud654\uc81c\uac00 \ub418\uba70 '\uc5ec\uc790 \ube45\ubc45'\uc774\ub77c\uace0 \ubd88\ub838\ub2e4. \ubcf8\ub798 \uadf8\ub8f9\uba85\uc740 21\uc774\uc600\uc73c\ub098, \uc774\ubbf8 2005\ub144 21(To Anyone)\uc774\ub77c\ub294 \ub0a8\uc790 \uc194\ub85c \uac00\uc218\uac00 \uc788\ub358 \uac83\uc73c\ub85c \uc54c\ub824\uc838 \ud604\uc7ac\uc758 2NE1\uc73c\ub85c \uadf8\ub8f9\uba85\uc744 \ubc14\uafe8\ub2e4. \uadf8\ub8f9\uba85\uc73c\ub85c \uc4f0\ub098\ubbf8, \ucfe8\uce59\uc2a4, \uc591\uc2f8\uc758 \uc544\uc774\ub4e4, YG\uac78\uc2a4 \ub4f1 \ub2e4\uc591\ud55c \uc774\ub984\uc774 \uac70\ub860 \ub418\uc5c8\uc73c\ub098, \uadf8\ub8f9\uba85 \uacf5\uac1c \uba87 \uc2dc\uac04 \uc804 \uc0c8\ubcbd\uc5d0 2NE1\uc774\ub77c\ub294 \uc774\ub984\uc73c\ub85c \uae09\ud558\uac8c \uc815\ud574\uc84c\ub2e4. 2NE1\uc774\ub77c\ub294 \uadf8\ub8f9\uba85\uc740 \"21\uc138\uae30\uc758 \uc0c8\ub85c\uc6b4 \uc9c4\ud654 (21C New Evolution)\", \"21\uc0b4\uc758 \ub098\uc774\ucc98\ub7fc \ud56d\uc0c1 \ub3c4\uc804\uc801\uc774\uace0 \uc2e0\uc120\ud55c \uc74c\uc545\uc744 \uc120\ubcf4\uc778\ub2e4\"\ub294 \uc758\ubbf8\ub97c \uac00\uc9c0\uace0 \uc788\ub2e4."} {"question": "\ud3f4\ub9c1\uc740 \ubca4\uc820\uc758 \uad6c\uc870\ub97c \ubb34\uc5c7\uc5d0 \uae30\ucd08\ud558\uc5ec \uc124\uba85\ud558\uc600\ub294\uac00", "paragraph": "\ud3f4\ub9c1\uc758 \ud654\ud559\uacb0\ud569\uc5d0 \uad00\ud55c \uc5c5\uc801 \uc911 \uc138 \ubc88\uc9f8 \uac83\uc740 \ubc29\ud5a5\uc871 \ud0c4\ud654\uc218\uc18c\uc758 \uad6c\uc870\ub97c \uaddc\uba85\ud558\ub294 \uac83\uc774\uc600\ub2e4. \uc774 \uc774\uc804\uae4c\uc9c0 \ubca4\uc820\uc758 \uad6c\uc870\uc5d0 \ub300\ud55c \uac00\uc7a5 \ub098\uc740 \uc124\uba85\uc740 \ub3c5\uc77c\uc758 \ud654\ud559\uc790\uc778 \ucf00\ucfe8\ub808\uc758 \uc124\uba85\uc774\uc600\ub2e4. \ucf00\ucfe8\ub808\ub294 \ubca4\uc820\uc774 \ub450 \uac00\uc9c0\uc758 \uad6c\uc870\uc0ac\uc774\ub97c \ube60\ub974\uac8c \uc654\ub2e4\uac14\ub2e4 \ud558\uba74\uc11c \uad6c\uc870\uac00 \uc720\uc9c0\ub418\uace0 \uc788\ub2e4\uace0 \ub9d0\ud558\uc600\ub294\ub370 \uc774 \ub54c \uccab \ubc88\uc9f8 \uad6c\uc870\uc5d0\uc11c \ub2e8\uc77c\uacb0\ud569\uc774\uc600\ub358 \ud0c4\uc18c-\ud0c4\uc18c\uac04\uc758 \uacb0\ud569\uc740 \ub450 \ubc88\uc9f8 \uad6c\uc870\uc5d0\uc11c \uc774\uc911\uacb0\ud569\uc73c\ub85c \uc774\uc911\uacb0\ud569\uc774\uc600\ub358 \ud0c4\uc18c-\ud0c4\uc18c\uac04\uc758 \uacb0\ud569\uc740 \ub450 \ubc88\uc9f8 \uad6c\uc870\uc5d0\uc11c \ub2e8\uc77c\uacb0\ud569\uc73c\ub85c\uc11c \uc874\uc7ac\ud55c\ub2e4\uace0 \uc124\uba85\ud558\uc600\ub2e4. \ubc18\uba74 \ud3f4\ub9c1\uc740 \uc591\uc790\uc5ed\ud559\uc5d0 \uae30\ucd08\ud558\uc5ec \uc0c8\ub85c\uc6b4 \uc124\uba85\uc744 \uc81c\uc2dc\ud558\uc600\ub294\ub370 \uc774\ub294 \ubca4\uc820\uc758 \ud0c4\uc18c-\ud0c4\uc18c\uac04\uc758 \uacb0\ud569\uc740 \uc774\uc911\uacb0\ud569\uacfc \ub2e8\uc77c\uacb0\ud569\uc758 \uc911\uac04\ucbe4 \ub418\ub294 \uacb0\ud569\uc744 \ud558\uace0 \uc788\ub2e4\ub294 \uac83\uc774\ub2e4. \uc989 \ubca4\uc820\uc758 \uad6c\uc870\ub294 \ucf00\ucfe8\ub808\uac00 \uc81c\uc2dc\ud55c \ub450 \uac00\uc9c0 \uad6c\uc870\uc758 \uc0ac\uc774\uc758 \uad6c\uc870\ub85c\uc11c \uc874\uc7ac\ud558\ub294 \uac83\uc774\uc9c0 \ub450 \uac00\uc9c0 \uad6c\uc870\uac00 \uc874\uc7ac\ud558\uc5ec \uadf8 \ub450 \uac00\uc9c0 \uad6c\uc870\uc0ac\uc774\ub97c \uc654\ub2e4\uac14\ub2e4 \ud558\ub294 \uac83\uc774 \uc544\ub2c8\ub77c\ub294 \uac83\uc774\uc600\ub2e4. \ud6c4\uc5d0 \uc774\ub7ec\ud55c \ud604\uc0c1\uc5d0 \ub300\ud558\uc5ec '\uacf5\uba85(resonance)' \ub77c\ub294 \uc774\ub984\uc774 \uc8fc\uc5b4\uc9c0\uac8c \ub418\uc5c8\ub2e4. \uc774\ub7ec\ud55c \ud3f4\ub9c1\uc758 \ubca4\uc820\uad6c\uc870\uc5d0 \ub300\ud55c \uc124\uba85\uc740 \uc55e\uc11c \uadf8\uac00 \uc81c\uc2dc\ud55c \uc624\ube44\ud0c8\uc758 \ud63c\uc131\ud654\uc640 \uac19\uc740 \ub9e5\ub77d\uc5d0 \uc788\ub2e4\uace0 \ubcfc \uc218 \uc788\ub294\ub370 \uc774\ub294 \ub450 \uac00\uc9c0 \uc124\uba85 \ubaa8\ub450 \uc5ec\ub7ec \uac1c\uc758 \uc804\uc790\uad6c\uc870\uac00 \ud569\uccd0\uc800\uc11c \uadf8\ub7ec\ud55c \uad6c\uc870\ub4e4\uc758 \uc0ac\uc774 \uc5b4\ub518\uac00\uc5d0 \uc704\uce58\ud558\ub294 \uad6c\uc870\ub97c \ub9cc\ub4e4\uc5b4 \ub0b4\uc5c8\uae30 \ub54c\ubb38\uc774\ub2e4.", "answer": "\uc591\uc790\uc5ed\ud559", "sentence": "\ubc18\uba74 \ud3f4\ub9c1\uc740 \uc591\uc790\uc5ed\ud559 \uc5d0", "paragraph_sentence": "\ud3f4\ub9c1\uc758 \ud654\ud559\uacb0\ud569\uc5d0 \uad00\ud55c \uc5c5\uc801 \uc911 \uc138 \ubc88\uc9f8 \uac83\uc740 \ubc29\ud5a5\uc871 \ud0c4\ud654\uc218\uc18c\uc758 \uad6c\uc870\ub97c \uaddc\uba85\ud558\ub294 \uac83\uc774\uc600\ub2e4. \uc774 \uc774\uc804\uae4c\uc9c0 \ubca4\uc820\uc758 \uad6c\uc870\uc5d0 \ub300\ud55c \uac00\uc7a5 \ub098\uc740 \uc124\uba85\uc740 \ub3c5\uc77c\uc758 \ud654\ud559\uc790\uc778 \ucf00\ucfe8\ub808\uc758 \uc124\uba85\uc774\uc600\ub2e4. \ucf00\ucfe8\ub808\ub294 \ubca4\uc820\uc774 \ub450 \uac00\uc9c0\uc758 \uad6c\uc870\uc0ac\uc774\ub97c \ube60\ub974\uac8c \uc654\ub2e4\uac14\ub2e4 \ud558\uba74\uc11c \uad6c\uc870\uac00 \uc720\uc9c0\ub418\uace0 \uc788\ub2e4\uace0 \ub9d0\ud558\uc600\ub294\ub370 \uc774 \ub54c \uccab \ubc88\uc9f8 \uad6c\uc870\uc5d0\uc11c \ub2e8\uc77c\uacb0\ud569\uc774\uc600\ub358 \ud0c4\uc18c-\ud0c4\uc18c\uac04\uc758 \uacb0\ud569\uc740 \ub450 \ubc88\uc9f8 \uad6c\uc870\uc5d0\uc11c \uc774\uc911\uacb0\ud569\uc73c\ub85c \uc774\uc911\uacb0\ud569\uc774\uc600\ub358 \ud0c4\uc18c-\ud0c4\uc18c\uac04\uc758 \uacb0\ud569\uc740 \ub450 \ubc88\uc9f8 \uad6c\uc870\uc5d0\uc11c \ub2e8\uc77c\uacb0\ud569\uc73c\ub85c\uc11c \uc874\uc7ac\ud55c\ub2e4\uace0 \uc124\uba85\ud558\uc600\ub2e4. \ubc18\uba74 \ud3f4\ub9c1\uc740 \uc591\uc790\uc5ed\ud559 \uc5d0 \uae30\ucd08\ud558\uc5ec \uc0c8\ub85c\uc6b4 \uc124\uba85\uc744 \uc81c\uc2dc\ud558\uc600\ub294\ub370 \uc774\ub294 \ubca4\uc820\uc758 \ud0c4\uc18c-\ud0c4\uc18c\uac04\uc758 \uacb0\ud569\uc740 \uc774\uc911\uacb0\ud569\uacfc \ub2e8\uc77c\uacb0\ud569\uc758 \uc911\uac04\ucbe4 \ub418\ub294 \uacb0\ud569\uc744 \ud558\uace0 \uc788\ub2e4\ub294 \uac83\uc774\ub2e4. \uc989 \ubca4\uc820\uc758 \uad6c\uc870\ub294 \ucf00\ucfe8\ub808\uac00 \uc81c\uc2dc\ud55c \ub450 \uac00\uc9c0 \uad6c\uc870\uc758 \uc0ac\uc774\uc758 \uad6c\uc870\ub85c\uc11c \uc874\uc7ac\ud558\ub294 \uac83\uc774\uc9c0 \ub450 \uac00\uc9c0 \uad6c\uc870\uac00 \uc874\uc7ac\ud558\uc5ec \uadf8 \ub450 \uac00\uc9c0 \uad6c\uc870\uc0ac\uc774\ub97c \uc654\ub2e4\uac14\ub2e4 \ud558\ub294 \uac83\uc774 \uc544\ub2c8\ub77c\ub294 \uac83\uc774\uc600\ub2e4. \ud6c4\uc5d0 \uc774\ub7ec\ud55c \ud604\uc0c1\uc5d0 \ub300\ud558\uc5ec '\uacf5\uba85(resonance)' \ub77c\ub294 \uc774\ub984\uc774 \uc8fc\uc5b4\uc9c0\uac8c \ub418\uc5c8\ub2e4. \uc774\ub7ec\ud55c \ud3f4\ub9c1\uc758 \ubca4\uc820\uad6c\uc870\uc5d0 \ub300\ud55c \uc124\uba85\uc740 \uc55e\uc11c \uadf8\uac00 \uc81c\uc2dc\ud55c \uc624\ube44\ud0c8\uc758 \ud63c\uc131\ud654\uc640 \uac19\uc740 \ub9e5\ub77d\uc5d0 \uc788\ub2e4\uace0 \ubcfc \uc218 \uc788\ub294\ub370 \uc774\ub294 \ub450 \uac00\uc9c0 \uc124\uba85 \ubaa8\ub450 \uc5ec\ub7ec \uac1c\uc758 \uc804\uc790\uad6c\uc870\uac00 \ud569\uccd0\uc800\uc11c \uadf8\ub7ec\ud55c \uad6c\uc870\ub4e4\uc758 \uc0ac\uc774 \uc5b4\ub518\uac00\uc5d0 \uc704\uce58\ud558\ub294 \uad6c\uc870\ub97c \ub9cc\ub4e4\uc5b4 \ub0b4\uc5c8\uae30 \ub54c\ubb38\uc774\ub2e4.", "paragraph_answer": "\ud3f4\ub9c1\uc758 \ud654\ud559\uacb0\ud569\uc5d0 \uad00\ud55c \uc5c5\uc801 \uc911 \uc138 \ubc88\uc9f8 \uac83\uc740 \ubc29\ud5a5\uc871 \ud0c4\ud654\uc218\uc18c\uc758 \uad6c\uc870\ub97c \uaddc\uba85\ud558\ub294 \uac83\uc774\uc600\ub2e4. \uc774 \uc774\uc804\uae4c\uc9c0 \ubca4\uc820\uc758 \uad6c\uc870\uc5d0 \ub300\ud55c \uac00\uc7a5 \ub098\uc740 \uc124\uba85\uc740 \ub3c5\uc77c\uc758 \ud654\ud559\uc790\uc778 \ucf00\ucfe8\ub808\uc758 \uc124\uba85\uc774\uc600\ub2e4. \ucf00\ucfe8\ub808\ub294 \ubca4\uc820\uc774 \ub450 \uac00\uc9c0\uc758 \uad6c\uc870\uc0ac\uc774\ub97c \ube60\ub974\uac8c \uc654\ub2e4\uac14\ub2e4 \ud558\uba74\uc11c \uad6c\uc870\uac00 \uc720\uc9c0\ub418\uace0 \uc788\ub2e4\uace0 \ub9d0\ud558\uc600\ub294\ub370 \uc774 \ub54c \uccab \ubc88\uc9f8 \uad6c\uc870\uc5d0\uc11c \ub2e8\uc77c\uacb0\ud569\uc774\uc600\ub358 \ud0c4\uc18c-\ud0c4\uc18c\uac04\uc758 \uacb0\ud569\uc740 \ub450 \ubc88\uc9f8 \uad6c\uc870\uc5d0\uc11c \uc774\uc911\uacb0\ud569\uc73c\ub85c \uc774\uc911\uacb0\ud569\uc774\uc600\ub358 \ud0c4\uc18c-\ud0c4\uc18c\uac04\uc758 \uacb0\ud569\uc740 \ub450 \ubc88\uc9f8 \uad6c\uc870\uc5d0\uc11c \ub2e8\uc77c\uacb0\ud569\uc73c\ub85c\uc11c \uc874\uc7ac\ud55c\ub2e4\uace0 \uc124\uba85\ud558\uc600\ub2e4. \ubc18\uba74 \ud3f4\ub9c1\uc740 \uc591\uc790\uc5ed\ud559 \uc5d0 \uae30\ucd08\ud558\uc5ec \uc0c8\ub85c\uc6b4 \uc124\uba85\uc744 \uc81c\uc2dc\ud558\uc600\ub294\ub370 \uc774\ub294 \ubca4\uc820\uc758 \ud0c4\uc18c-\ud0c4\uc18c\uac04\uc758 \uacb0\ud569\uc740 \uc774\uc911\uacb0\ud569\uacfc \ub2e8\uc77c\uacb0\ud569\uc758 \uc911\uac04\ucbe4 \ub418\ub294 \uacb0\ud569\uc744 \ud558\uace0 \uc788\ub2e4\ub294 \uac83\uc774\ub2e4. \uc989 \ubca4\uc820\uc758 \uad6c\uc870\ub294 \ucf00\ucfe8\ub808\uac00 \uc81c\uc2dc\ud55c \ub450 \uac00\uc9c0 \uad6c\uc870\uc758 \uc0ac\uc774\uc758 \uad6c\uc870\ub85c\uc11c \uc874\uc7ac\ud558\ub294 \uac83\uc774\uc9c0 \ub450 \uac00\uc9c0 \uad6c\uc870\uac00 \uc874\uc7ac\ud558\uc5ec \uadf8 \ub450 \uac00\uc9c0 \uad6c\uc870\uc0ac\uc774\ub97c \uc654\ub2e4\uac14\ub2e4 \ud558\ub294 \uac83\uc774 \uc544\ub2c8\ub77c\ub294 \uac83\uc774\uc600\ub2e4. \ud6c4\uc5d0 \uc774\ub7ec\ud55c \ud604\uc0c1\uc5d0 \ub300\ud558\uc5ec '\uacf5\uba85(resonance)' \ub77c\ub294 \uc774\ub984\uc774 \uc8fc\uc5b4\uc9c0\uac8c \ub418\uc5c8\ub2e4. \uc774\ub7ec\ud55c \ud3f4\ub9c1\uc758 \ubca4\uc820\uad6c\uc870\uc5d0 \ub300\ud55c \uc124\uba85\uc740 \uc55e\uc11c \uadf8\uac00 \uc81c\uc2dc\ud55c \uc624\ube44\ud0c8\uc758 \ud63c\uc131\ud654\uc640 \uac19\uc740 \ub9e5\ub77d\uc5d0 \uc788\ub2e4\uace0 \ubcfc \uc218 \uc788\ub294\ub370 \uc774\ub294 \ub450 \uac00\uc9c0 \uc124\uba85 \ubaa8\ub450 \uc5ec\ub7ec \uac1c\uc758 \uc804\uc790\uad6c\uc870\uac00 \ud569\uccd0\uc800\uc11c \uadf8\ub7ec\ud55c \uad6c\uc870\ub4e4\uc758 \uc0ac\uc774 \uc5b4\ub518\uac00\uc5d0 \uc704\uce58\ud558\ub294 \uad6c\uc870\ub97c \ub9cc\ub4e4\uc5b4 \ub0b4\uc5c8\uae30 \ub54c\ubb38\uc774\ub2e4.", "sentence_answer": "\ubc18\uba74 \ud3f4\ub9c1\uc740 \uc591\uc790\uc5ed\ud559 \uc5d0", "paragraph_id": "6404796-8-1", "paragraph_question": "question: \ud3f4\ub9c1\uc740 \ubca4\uc820\uc758 \uad6c\uc870\ub97c \ubb34\uc5c7\uc5d0 \uae30\ucd08\ud558\uc5ec \uc124\uba85\ud558\uc600\ub294\uac00, context: \ud3f4\ub9c1\uc758 \ud654\ud559\uacb0\ud569\uc5d0 \uad00\ud55c \uc5c5\uc801 \uc911 \uc138 \ubc88\uc9f8 \uac83\uc740 \ubc29\ud5a5\uc871 \ud0c4\ud654\uc218\uc18c\uc758 \uad6c\uc870\ub97c \uaddc\uba85\ud558\ub294 \uac83\uc774\uc600\ub2e4. \uc774 \uc774\uc804\uae4c\uc9c0 \ubca4\uc820\uc758 \uad6c\uc870\uc5d0 \ub300\ud55c \uac00\uc7a5 \ub098\uc740 \uc124\uba85\uc740 \ub3c5\uc77c\uc758 \ud654\ud559\uc790\uc778 \ucf00\ucfe8\ub808\uc758 \uc124\uba85\uc774\uc600\ub2e4. \ucf00\ucfe8\ub808\ub294 \ubca4\uc820\uc774 \ub450 \uac00\uc9c0\uc758 \uad6c\uc870\uc0ac\uc774\ub97c \ube60\ub974\uac8c \uc654\ub2e4\uac14\ub2e4 \ud558\uba74\uc11c \uad6c\uc870\uac00 \uc720\uc9c0\ub418\uace0 \uc788\ub2e4\uace0 \ub9d0\ud558\uc600\ub294\ub370 \uc774 \ub54c \uccab \ubc88\uc9f8 \uad6c\uc870\uc5d0\uc11c \ub2e8\uc77c\uacb0\ud569\uc774\uc600\ub358 \ud0c4\uc18c-\ud0c4\uc18c\uac04\uc758 \uacb0\ud569\uc740 \ub450 \ubc88\uc9f8 \uad6c\uc870\uc5d0\uc11c \uc774\uc911\uacb0\ud569\uc73c\ub85c \uc774\uc911\uacb0\ud569\uc774\uc600\ub358 \ud0c4\uc18c-\ud0c4\uc18c\uac04\uc758 \uacb0\ud569\uc740 \ub450 \ubc88\uc9f8 \uad6c\uc870\uc5d0\uc11c \ub2e8\uc77c\uacb0\ud569\uc73c\ub85c\uc11c \uc874\uc7ac\ud55c\ub2e4\uace0 \uc124\uba85\ud558\uc600\ub2e4. \ubc18\uba74 \ud3f4\ub9c1\uc740 \uc591\uc790\uc5ed\ud559\uc5d0 \uae30\ucd08\ud558\uc5ec \uc0c8\ub85c\uc6b4 \uc124\uba85\uc744 \uc81c\uc2dc\ud558\uc600\ub294\ub370 \uc774\ub294 \ubca4\uc820\uc758 \ud0c4\uc18c-\ud0c4\uc18c\uac04\uc758 \uacb0\ud569\uc740 \uc774\uc911\uacb0\ud569\uacfc \ub2e8\uc77c\uacb0\ud569\uc758 \uc911\uac04\ucbe4 \ub418\ub294 \uacb0\ud569\uc744 \ud558\uace0 \uc788\ub2e4\ub294 \uac83\uc774\ub2e4. \uc989 \ubca4\uc820\uc758 \uad6c\uc870\ub294 \ucf00\ucfe8\ub808\uac00 \uc81c\uc2dc\ud55c \ub450 \uac00\uc9c0 \uad6c\uc870\uc758 \uc0ac\uc774\uc758 \uad6c\uc870\ub85c\uc11c \uc874\uc7ac\ud558\ub294 \uac83\uc774\uc9c0 \ub450 \uac00\uc9c0 \uad6c\uc870\uac00 \uc874\uc7ac\ud558\uc5ec \uadf8 \ub450 \uac00\uc9c0 \uad6c\uc870\uc0ac\uc774\ub97c \uc654\ub2e4\uac14\ub2e4 \ud558\ub294 \uac83\uc774 \uc544\ub2c8\ub77c\ub294 \uac83\uc774\uc600\ub2e4. \ud6c4\uc5d0 \uc774\ub7ec\ud55c \ud604\uc0c1\uc5d0 \ub300\ud558\uc5ec '\uacf5\uba85(resonance)' \ub77c\ub294 \uc774\ub984\uc774 \uc8fc\uc5b4\uc9c0\uac8c \ub418\uc5c8\ub2e4. \uc774\ub7ec\ud55c \ud3f4\ub9c1\uc758 \ubca4\uc820\uad6c\uc870\uc5d0 \ub300\ud55c \uc124\uba85\uc740 \uc55e\uc11c \uadf8\uac00 \uc81c\uc2dc\ud55c \uc624\ube44\ud0c8\uc758 \ud63c\uc131\ud654\uc640 \uac19\uc740 \ub9e5\ub77d\uc5d0 \uc788\ub2e4\uace0 \ubcfc \uc218 \uc788\ub294\ub370 \uc774\ub294 \ub450 \uac00\uc9c0 \uc124\uba85 \ubaa8\ub450 \uc5ec\ub7ec \uac1c\uc758 \uc804\uc790\uad6c\uc870\uac00 \ud569\uccd0\uc800\uc11c \uadf8\ub7ec\ud55c \uad6c\uc870\ub4e4\uc758 \uc0ac\uc774 \uc5b4\ub518\uac00\uc5d0 \uc704\uce58\ud558\ub294 \uad6c\uc870\ub97c \ub9cc\ub4e4\uc5b4 \ub0b4\uc5c8\uae30 \ub54c\ubb38\uc774\ub2e4."} {"question": "\ubc15\uadfc\ud61c\uc758 \ub192\uc740 \uc9c0\uc9c0\uc728\uc774 '\uac10\uc131\uc758 \uc815\uce58' \ub54c\ubb38\uc774\ub77c\uace0 \ube44\ud310\ud55c \uc778\ubb3c\uc740?", "paragraph": "2010\ub144 12\uc6d4, \uc0c8\ud574 \uc608\uc0b0\uc548\uc774 \uad6d\ud68c\uc758\uc7a5\uc758 \uc9c1\uad8c\uc0c1\uc815\uc73c\ub85c \ud1b5\uacfc\ub418\uc5b4 \uc5ec\uc57c\uac04\uc758 \uac08\ub4f1\uc774 \uadf9\ud654\ub41c \uc0c1\ud669\uc5d0\uc11c \ubc15\uadfc\ud61c\uac00 \ubcf5\uc9c0 \uc815\ucc45\uc744 \uac15\uc870\ud558\uba70 \ub098\ud0c0\ub098\uc790 \ubbfc\uc8fc\ub2f9 \ubc15\uc9c0\uc6d0 \uc6d0\ub0b4\ub300\ud45c\ub294 \"\ubc15 \uc804 \ub300\ud45c\ub294 \ub0a0\uce58\uae30\ub85c \uadf8 \ub9ce\uc740 \ubcf5\uc9c0 \uc608\uc0b0\uc774 \uc644\uc804\ud788 \uc0ad\uac10\ub420 \ub54c \uc544\ubb34\ub9d0\ub3c4 \ud558\uc9c0 \uc54a\uc558\ub2e4\"\uace0 \uc9c0\uc801\ud558\uba70 \"\uc720\ub9ac\ud560\ub54c\ub9cc \uace0\uac1c\ub97c \ub4e0\ub2e4\"\uba70 \ubc15\uadfc\ud61c\uc758 \ud589\ubcf4\uc5d0 \ube44\ud310\uc744 \uc81c\uae30\ud588\ub2e4. \uc870\uc120\uc77c\ubcf4\ub3c4 \uc0ac\uc124\uc5d0\uc11c \"\ubc15 \uc804 \ub300\ud45c\ub294 \uc65c \ub0b4\ud0a4\ub294 \uc8fc\uc81c\uc5d0 \ub300\ud574 \ud558\uace0 \uc2f6\uc740 \ub9d0\ub9cc \ud558\uace0, \uad6d\ubbfc\uc774 \ubc15 \uc804 \ub300\ud45c\uc5d0 \ub300\ud574 \uad81\uae08\ud55c \uc77c\uc744 \ubb3c\uc744 \uae30\ud68c\ub294 \ub9cc\ub4e4\uc9c0 \uc54a\ub290\ub0d0\ub294 \uac83\uc774\ub2e4. \uadf8\ub798\uc11c \uac70\ubd81\ud55c \uc8fc\uc81c\ub97c \ud53c\ud558\ub824 \ud55c\ub2e4\ub294 \ub290\ub08c\uc744 \uc900\ub2e4.\"\uace0 \uc9c0\uc801\ud588\ub2e4. \ubc15\uadfc\ud61c\uc758 \uc774\ub7ec\ud55c \ubbf8\uc9c0\uadfc\ud55c \ud589\ubcf4\uc5d0\ub3c4 \ub192\uc740 \uc9c0\uc9c0\uc728\uc744 \uc720\uc9c0\ud558\uace0 \uc788\ub294\ub370 \ub300\ud55c \ube44\ud310\ub3c4 \uc788\ub2e4. \uc11c\uc6b8\ub300\ud559\uad50 \ubc95\ud559\uc804\ubb38\ub300\ud559\uc6d0 \uc870\uad6d \uad50\uc218\ub294 \"\ubc15\uadfc\ud61c \uc774 \uc0ac\ub78c\uc774 \ubb34\uc2a8 \uc598\uae30\ub97c \ud558\ub294\uc9c0 \uc0ac\ub78c\ub4e4\uc774 \uc798 \ubaa8\ub974\ub294\ub370\ub3c4 \uc9c0\uc9c0\ud558\ub294 \uc0ac\ub78c\uc740 \uc0bc\uc2ed\uba87 %\ub098 \ub41c\ub2e4\"\ub77c\uba70 \"\ub0b4\uc6a9\ub3c4 \ubaa8\ub974\uace0 \uc88b\uc544\ud558\ub294 \uac83\uc740 \uac10\uc131\uc758 \uc815\uce58\uc77c \uac83\"\uc774\ub77c\uace0 \ube44\ud310\ud558\uc600\ub2e4. \uadf8\ub7ec\ub098 \ubc15\uadfc\ud61c\uc758 \uc9c0\uc9c0\uc790\ub4e4\uc740 \ubc15\uadfc\ud61c\uc758 \ub9d0\uc5d0 \uc2e0\ub8b0\ub97c \ubcf4\ub0b8\ub2e4. \"\ubc15\uadfc\ud61c\uc758 \uc2e0\uc911\ud568\uc740 \ubbf8\ub798\uc758 \ubd88\ud655\uc2e4\uc131\uc73c\ub85c \ubd88\uc548\uac10\uc5d0 \uc816\uc5b4\uc788\ub294 \uad6d\ubbfc\uc5d0\uac8c \uc704\ub85c\uc640 \uc2e0\ub8b0\ub97c \uc900\ub2e4\"\ub294 \ud3c9\uac00\uac00 \uc788\ub2e4.", "answer": "\uc870\uad6d", "sentence": "\uc11c\uc6b8\ub300\ud559\uad50 \ubc95\ud559\uc804\ubb38\ub300\ud559\uc6d0 \uc870\uad6d \uad50\uc218\ub294 \"\ubc15\uadfc\ud61c \uc774 \uc0ac\ub78c\uc774 \ubb34\uc2a8 \uc598\uae30\ub97c \ud558\ub294\uc9c0 \uc0ac\ub78c\ub4e4\uc774 \uc798 \ubaa8\ub974\ub294\ub370\ub3c4 \uc9c0\uc9c0\ud558\ub294 \uc0ac\ub78c\uc740 \uc0bc\uc2ed\uba87 %\ub098 \ub41c\ub2e4\"\ub77c\uba70 \"\ub0b4\uc6a9\ub3c4 \ubaa8\ub974\uace0 \uc88b\uc544\ud558\ub294 \uac83\uc740 \uac10\uc131\uc758 \uc815\uce58\uc77c \uac83\"\uc774\ub77c\uace0 \ube44\ud310\ud558\uc600\ub2e4.", "paragraph_sentence": "2010\ub144 12\uc6d4, \uc0c8\ud574 \uc608\uc0b0\uc548\uc774 \uad6d\ud68c\uc758\uc7a5\uc758 \uc9c1\uad8c\uc0c1\uc815\uc73c\ub85c \ud1b5\uacfc\ub418\uc5b4 \uc5ec\uc57c\uac04\uc758 \uac08\ub4f1\uc774 \uadf9\ud654\ub41c \uc0c1\ud669\uc5d0\uc11c \ubc15\uadfc\ud61c\uac00 \ubcf5\uc9c0 \uc815\ucc45\uc744 \uac15\uc870\ud558\uba70 \ub098\ud0c0\ub098\uc790 \ubbfc\uc8fc\ub2f9 \ubc15\uc9c0\uc6d0 \uc6d0\ub0b4\ub300\ud45c\ub294 \"\ubc15 \uc804 \ub300\ud45c\ub294 \ub0a0\uce58\uae30\ub85c \uadf8 \ub9ce\uc740 \ubcf5\uc9c0 \uc608\uc0b0\uc774 \uc644\uc804\ud788 \uc0ad\uac10\ub420 \ub54c \uc544\ubb34\ub9d0\ub3c4 \ud558\uc9c0 \uc54a\uc558\ub2e4\"\uace0 \uc9c0\uc801\ud558\uba70 \"\uc720\ub9ac\ud560\ub54c\ub9cc \uace0\uac1c\ub97c \ub4e0\ub2e4\"\uba70 \ubc15\uadfc\ud61c\uc758 \ud589\ubcf4\uc5d0 \ube44\ud310\uc744 \uc81c\uae30\ud588\ub2e4. \uc870\uc120\uc77c\ubcf4\ub3c4 \uc0ac\uc124\uc5d0\uc11c \"\ubc15 \uc804 \ub300\ud45c\ub294 \uc65c \ub0b4\ud0a4\ub294 \uc8fc\uc81c\uc5d0 \ub300\ud574 \ud558\uace0 \uc2f6\uc740 \ub9d0\ub9cc \ud558\uace0, \uad6d\ubbfc\uc774 \ubc15 \uc804 \ub300\ud45c\uc5d0 \ub300\ud574 \uad81\uae08\ud55c \uc77c\uc744 \ubb3c\uc744 \uae30\ud68c\ub294 \ub9cc\ub4e4\uc9c0 \uc54a\ub290\ub0d0\ub294 \uac83\uc774\ub2e4. \uadf8\ub798\uc11c \uac70\ubd81\ud55c \uc8fc\uc81c\ub97c \ud53c\ud558\ub824 \ud55c\ub2e4\ub294 \ub290\ub08c\uc744 \uc900\ub2e4. \"\uace0 \uc9c0\uc801\ud588\ub2e4. \ubc15\uadfc\ud61c\uc758 \uc774\ub7ec\ud55c \ubbf8\uc9c0\uadfc\ud55c \ud589\ubcf4\uc5d0\ub3c4 \ub192\uc740 \uc9c0\uc9c0\uc728\uc744 \uc720\uc9c0\ud558\uace0 \uc788\ub294\ub370 \ub300\ud55c \ube44\ud310\ub3c4 \uc788\ub2e4. \uc11c\uc6b8\ub300\ud559\uad50 \ubc95\ud559\uc804\ubb38\ub300\ud559\uc6d0 \uc870\uad6d \uad50\uc218\ub294 \"\ubc15\uadfc\ud61c \uc774 \uc0ac\ub78c\uc774 \ubb34\uc2a8 \uc598\uae30\ub97c \ud558\ub294\uc9c0 \uc0ac\ub78c\ub4e4\uc774 \uc798 \ubaa8\ub974\ub294\ub370\ub3c4 \uc9c0\uc9c0\ud558\ub294 \uc0ac\ub78c\uc740 \uc0bc\uc2ed\uba87 %\ub098 \ub41c\ub2e4\"\ub77c\uba70 \"\ub0b4\uc6a9\ub3c4 \ubaa8\ub974\uace0 \uc88b\uc544\ud558\ub294 \uac83\uc740 \uac10\uc131\uc758 \uc815\uce58\uc77c \uac83\"\uc774\ub77c\uace0 \ube44\ud310\ud558\uc600\ub2e4. \uadf8\ub7ec\ub098 \ubc15\uadfc\ud61c\uc758 \uc9c0\uc9c0\uc790\ub4e4\uc740 \ubc15\uadfc\ud61c\uc758 \ub9d0\uc5d0 \uc2e0\ub8b0\ub97c \ubcf4\ub0b8\ub2e4. \"\ubc15\uadfc\ud61c\uc758 \uc2e0\uc911\ud568\uc740 \ubbf8\ub798\uc758 \ubd88\ud655\uc2e4\uc131\uc73c\ub85c \ubd88\uc548\uac10\uc5d0 \uc816\uc5b4\uc788\ub294 \uad6d\ubbfc\uc5d0\uac8c \uc704\ub85c\uc640 \uc2e0\ub8b0\ub97c \uc900\ub2e4\"\ub294 \ud3c9\uac00\uac00 \uc788\ub2e4.", "paragraph_answer": "2010\ub144 12\uc6d4, \uc0c8\ud574 \uc608\uc0b0\uc548\uc774 \uad6d\ud68c\uc758\uc7a5\uc758 \uc9c1\uad8c\uc0c1\uc815\uc73c\ub85c \ud1b5\uacfc\ub418\uc5b4 \uc5ec\uc57c\uac04\uc758 \uac08\ub4f1\uc774 \uadf9\ud654\ub41c \uc0c1\ud669\uc5d0\uc11c \ubc15\uadfc\ud61c\uac00 \ubcf5\uc9c0 \uc815\ucc45\uc744 \uac15\uc870\ud558\uba70 \ub098\ud0c0\ub098\uc790 \ubbfc\uc8fc\ub2f9 \ubc15\uc9c0\uc6d0 \uc6d0\ub0b4\ub300\ud45c\ub294 \"\ubc15 \uc804 \ub300\ud45c\ub294 \ub0a0\uce58\uae30\ub85c \uadf8 \ub9ce\uc740 \ubcf5\uc9c0 \uc608\uc0b0\uc774 \uc644\uc804\ud788 \uc0ad\uac10\ub420 \ub54c \uc544\ubb34\ub9d0\ub3c4 \ud558\uc9c0 \uc54a\uc558\ub2e4\"\uace0 \uc9c0\uc801\ud558\uba70 \"\uc720\ub9ac\ud560\ub54c\ub9cc \uace0\uac1c\ub97c \ub4e0\ub2e4\"\uba70 \ubc15\uadfc\ud61c\uc758 \ud589\ubcf4\uc5d0 \ube44\ud310\uc744 \uc81c\uae30\ud588\ub2e4. \uc870\uc120\uc77c\ubcf4\ub3c4 \uc0ac\uc124\uc5d0\uc11c \"\ubc15 \uc804 \ub300\ud45c\ub294 \uc65c \ub0b4\ud0a4\ub294 \uc8fc\uc81c\uc5d0 \ub300\ud574 \ud558\uace0 \uc2f6\uc740 \ub9d0\ub9cc \ud558\uace0, \uad6d\ubbfc\uc774 \ubc15 \uc804 \ub300\ud45c\uc5d0 \ub300\ud574 \uad81\uae08\ud55c \uc77c\uc744 \ubb3c\uc744 \uae30\ud68c\ub294 \ub9cc\ub4e4\uc9c0 \uc54a\ub290\ub0d0\ub294 \uac83\uc774\ub2e4. \uadf8\ub798\uc11c \uac70\ubd81\ud55c \uc8fc\uc81c\ub97c \ud53c\ud558\ub824 \ud55c\ub2e4\ub294 \ub290\ub08c\uc744 \uc900\ub2e4.\"\uace0 \uc9c0\uc801\ud588\ub2e4. \ubc15\uadfc\ud61c\uc758 \uc774\ub7ec\ud55c \ubbf8\uc9c0\uadfc\ud55c \ud589\ubcf4\uc5d0\ub3c4 \ub192\uc740 \uc9c0\uc9c0\uc728\uc744 \uc720\uc9c0\ud558\uace0 \uc788\ub294\ub370 \ub300\ud55c \ube44\ud310\ub3c4 \uc788\ub2e4. \uc11c\uc6b8\ub300\ud559\uad50 \ubc95\ud559\uc804\ubb38\ub300\ud559\uc6d0 \uc870\uad6d \uad50\uc218\ub294 \"\ubc15\uadfc\ud61c \uc774 \uc0ac\ub78c\uc774 \ubb34\uc2a8 \uc598\uae30\ub97c \ud558\ub294\uc9c0 \uc0ac\ub78c\ub4e4\uc774 \uc798 \ubaa8\ub974\ub294\ub370\ub3c4 \uc9c0\uc9c0\ud558\ub294 \uc0ac\ub78c\uc740 \uc0bc\uc2ed\uba87 %\ub098 \ub41c\ub2e4\"\ub77c\uba70 \"\ub0b4\uc6a9\ub3c4 \ubaa8\ub974\uace0 \uc88b\uc544\ud558\ub294 \uac83\uc740 \uac10\uc131\uc758 \uc815\uce58\uc77c \uac83\"\uc774\ub77c\uace0 \ube44\ud310\ud558\uc600\ub2e4. \uadf8\ub7ec\ub098 \ubc15\uadfc\ud61c\uc758 \uc9c0\uc9c0\uc790\ub4e4\uc740 \ubc15\uadfc\ud61c\uc758 \ub9d0\uc5d0 \uc2e0\ub8b0\ub97c \ubcf4\ub0b8\ub2e4. \"\ubc15\uadfc\ud61c\uc758 \uc2e0\uc911\ud568\uc740 \ubbf8\ub798\uc758 \ubd88\ud655\uc2e4\uc131\uc73c\ub85c \ubd88\uc548\uac10\uc5d0 \uc816\uc5b4\uc788\ub294 \uad6d\ubbfc\uc5d0\uac8c \uc704\ub85c\uc640 \uc2e0\ub8b0\ub97c \uc900\ub2e4\"\ub294 \ud3c9\uac00\uac00 \uc788\ub2e4.", "sentence_answer": "\uc11c\uc6b8\ub300\ud559\uad50 \ubc95\ud559\uc804\ubb38\ub300\ud559\uc6d0 \uc870\uad6d \uad50\uc218\ub294 \"\ubc15\uadfc\ud61c \uc774 \uc0ac\ub78c\uc774 \ubb34\uc2a8 \uc598\uae30\ub97c \ud558\ub294\uc9c0 \uc0ac\ub78c\ub4e4\uc774 \uc798 \ubaa8\ub974\ub294\ub370\ub3c4 \uc9c0\uc9c0\ud558\ub294 \uc0ac\ub78c\uc740 \uc0bc\uc2ed\uba87 %\ub098 \ub41c\ub2e4\"\ub77c\uba70 \"\ub0b4\uc6a9\ub3c4 \ubaa8\ub974\uace0 \uc88b\uc544\ud558\ub294 \uac83\uc740 \uac10\uc131\uc758 \uc815\uce58\uc77c \uac83\"\uc774\ub77c\uace0 \ube44\ud310\ud558\uc600\ub2e4.", "paragraph_id": "6515822-14-1", "paragraph_question": "question: \ubc15\uadfc\ud61c\uc758 \ub192\uc740 \uc9c0\uc9c0\uc728\uc774 '\uac10\uc131\uc758 \uc815\uce58' \ub54c\ubb38\uc774\ub77c\uace0 \ube44\ud310\ud55c \uc778\ubb3c\uc740?, context: 2010\ub144 12\uc6d4, \uc0c8\ud574 \uc608\uc0b0\uc548\uc774 \uad6d\ud68c\uc758\uc7a5\uc758 \uc9c1\uad8c\uc0c1\uc815\uc73c\ub85c \ud1b5\uacfc\ub418\uc5b4 \uc5ec\uc57c\uac04\uc758 \uac08\ub4f1\uc774 \uadf9\ud654\ub41c \uc0c1\ud669\uc5d0\uc11c \ubc15\uadfc\ud61c\uac00 \ubcf5\uc9c0 \uc815\ucc45\uc744 \uac15\uc870\ud558\uba70 \ub098\ud0c0\ub098\uc790 \ubbfc\uc8fc\ub2f9 \ubc15\uc9c0\uc6d0 \uc6d0\ub0b4\ub300\ud45c\ub294 \"\ubc15 \uc804 \ub300\ud45c\ub294 \ub0a0\uce58\uae30\ub85c \uadf8 \ub9ce\uc740 \ubcf5\uc9c0 \uc608\uc0b0\uc774 \uc644\uc804\ud788 \uc0ad\uac10\ub420 \ub54c \uc544\ubb34\ub9d0\ub3c4 \ud558\uc9c0 \uc54a\uc558\ub2e4\"\uace0 \uc9c0\uc801\ud558\uba70 \"\uc720\ub9ac\ud560\ub54c\ub9cc \uace0\uac1c\ub97c \ub4e0\ub2e4\"\uba70 \ubc15\uadfc\ud61c\uc758 \ud589\ubcf4\uc5d0 \ube44\ud310\uc744 \uc81c\uae30\ud588\ub2e4. \uc870\uc120\uc77c\ubcf4\ub3c4 \uc0ac\uc124\uc5d0\uc11c \"\ubc15 \uc804 \ub300\ud45c\ub294 \uc65c \ub0b4\ud0a4\ub294 \uc8fc\uc81c\uc5d0 \ub300\ud574 \ud558\uace0 \uc2f6\uc740 \ub9d0\ub9cc \ud558\uace0, \uad6d\ubbfc\uc774 \ubc15 \uc804 \ub300\ud45c\uc5d0 \ub300\ud574 \uad81\uae08\ud55c \uc77c\uc744 \ubb3c\uc744 \uae30\ud68c\ub294 \ub9cc\ub4e4\uc9c0 \uc54a\ub290\ub0d0\ub294 \uac83\uc774\ub2e4. \uadf8\ub798\uc11c \uac70\ubd81\ud55c \uc8fc\uc81c\ub97c \ud53c\ud558\ub824 \ud55c\ub2e4\ub294 \ub290\ub08c\uc744 \uc900\ub2e4.\"\uace0 \uc9c0\uc801\ud588\ub2e4. \ubc15\uadfc\ud61c\uc758 \uc774\ub7ec\ud55c \ubbf8\uc9c0\uadfc\ud55c \ud589\ubcf4\uc5d0\ub3c4 \ub192\uc740 \uc9c0\uc9c0\uc728\uc744 \uc720\uc9c0\ud558\uace0 \uc788\ub294\ub370 \ub300\ud55c \ube44\ud310\ub3c4 \uc788\ub2e4. \uc11c\uc6b8\ub300\ud559\uad50 \ubc95\ud559\uc804\ubb38\ub300\ud559\uc6d0 \uc870\uad6d \uad50\uc218\ub294 \"\ubc15\uadfc\ud61c \uc774 \uc0ac\ub78c\uc774 \ubb34\uc2a8 \uc598\uae30\ub97c \ud558\ub294\uc9c0 \uc0ac\ub78c\ub4e4\uc774 \uc798 \ubaa8\ub974\ub294\ub370\ub3c4 \uc9c0\uc9c0\ud558\ub294 \uc0ac\ub78c\uc740 \uc0bc\uc2ed\uba87 %\ub098 \ub41c\ub2e4\"\ub77c\uba70 \"\ub0b4\uc6a9\ub3c4 \ubaa8\ub974\uace0 \uc88b\uc544\ud558\ub294 \uac83\uc740 \uac10\uc131\uc758 \uc815\uce58\uc77c \uac83\"\uc774\ub77c\uace0 \ube44\ud310\ud558\uc600\ub2e4. \uadf8\ub7ec\ub098 \ubc15\uadfc\ud61c\uc758 \uc9c0\uc9c0\uc790\ub4e4\uc740 \ubc15\uadfc\ud61c\uc758 \ub9d0\uc5d0 \uc2e0\ub8b0\ub97c \ubcf4\ub0b8\ub2e4. \"\ubc15\uadfc\ud61c\uc758 \uc2e0\uc911\ud568\uc740 \ubbf8\ub798\uc758 \ubd88\ud655\uc2e4\uc131\uc73c\ub85c \ubd88\uc548\uac10\uc5d0 \uc816\uc5b4\uc788\ub294 \uad6d\ubbfc\uc5d0\uac8c \uc704\ub85c\uc640 \uc2e0\ub8b0\ub97c \uc900\ub2e4\"\ub294 \ud3c9\uac00\uac00 \uc788\ub2e4."} {"question": "\ub4dc\ub8e8\uc988\ud30c\ub294 \ub204\uac00 \uc2e0\uc758 \ud604\uc2e0\uc774\ub77c \uc8fc\uc7a5\ud558\ub294\uac00?", "paragraph": "\ub4dc\ub8e8\uc988\ud30c(Druze)\ub294 11\uc138\uae30 \uc774\uc2a4\ub9c8\uc77c\ud30c\uc5d0\uc11c \ub098\uc628 \ub2e4\ub77c\uc9c0(Darazi)\ub77c\ub294 \uc778\ubb3c\uc774 \uc228\uc740 \uc774\ub9d8 '\ub9c8\ud750\ub514'\ub97c \uc790\ucc98\ud558\uba74\uc11c \ucc3d\uc2dc\ud588\ub2e4. \ube44\ubc00\uc8fc\uc758\uac00 \uac15\ud558\uba70, \uc2dc\ub9ac\uc544\uc640 \ub808\ubc14\ub17c\uc5d0 \uc8fc\ub85c \ubd84\ud3ec\ud558\uace0 \uc788\ub2e4. \uc2e0\uc758 \uc608\uc5b8\uc790 \ubb34\ud568\ub9c8\ub4dc\ub97c \ub530\ub974\ub294 \uc774\uc2ac\ub78c\uacfc \ub2ec\ub9ac \ud558\ud0b4\uc774\ub77c\ub294 \uce7c\ub9ac\ud30c\uac00 \uc2e0\uc758 \ud604\uc2e0\uc774\ub77c \uc8fc\uc7a5\ud558\uba70 \uc77c\uc2e0\ub860\uc744 \ubbff\ub294\ub2e4. \uc77c\ubd80\ub2e4\ucc98\uc81c\ub294 \uae08\uc9c0\ub3fc \uc788\uace0 \uc5ec\uc131\ub4e4\uc758 \uc9c0\uc704\ub294 \ub0a8\uc131\uacfc \ub3d9\ub4f1\ud558\uba70 \uc601\ud63c\uc740 \uc724\ud68c\ud55c\ub2e4\uace0 \ubbff\ub294\ub2e4. \uc774\uc2ac\ub78c\uc758 \ud558\uc988(\uc21c\ub840) \uae30\uac04 \ucd95\uc81c\ub97c \uc5f4\uae34 \ud558\uc9c0\ub9cc \uc21c\ub840 \uc790\uccb4\ub294 \ud558\uc9c0 \uc54a\uc73c\uba70 \ub77c\ub9c8\ub2e8 \uae08\uc2dd\ub3c4 \ud558\uc9c0 \uc54a\ub294\ub2e4. \uacf5\ub3d9\uccb4\ub97c \ub9cc\ub4e4\uc5b4 \uc0dd\ud65c\ud558\ub294 \uacbd\uc6b0\uac00 \ub9ce\uc73c\uba70 \uae08\uc695\uc801\uc778 \uc0dd\ud65c\uc744 \ucc2c\uc591\ud55c\ub2e4. \uad50\ub9ac\uc640 \uad00\ud589\uc73c\ub85c \ubcf4\uc544, \uc774\uc2ac\ub78c\uad50\uc758 \uc885\ud30c\ub77c\uae30\ubcf4\ub2e4\ub294 \ubcc4\ub3c4\uc758 \uc885\uad50\ub85c \ubcf4\ub294 \uc2dc\uac01\uc774 \uc9c0\ubc30\uc801\uc774\ub2e4.", "answer": "\ud558\ud0b4", "sentence": "\uc2e0\uc758 \uc608\uc5b8\uc790 \ubb34\ud568\ub9c8\ub4dc\ub97c \ub530\ub974\ub294 \uc774\uc2ac\ub78c\uacfc \ub2ec\ub9ac \ud558\ud0b4 \uc774\ub77c\ub294 \uce7c\ub9ac\ud30c\uac00 \uc2e0\uc758 \ud604\uc2e0\uc774\ub77c \uc8fc\uc7a5\ud558\uba70 \uc77c\uc2e0\ub860\uc744 \ubbff\ub294\ub2e4.", "paragraph_sentence": "\ub4dc\ub8e8\uc988\ud30c(Druze)\ub294 11\uc138\uae30 \uc774\uc2a4\ub9c8\uc77c\ud30c\uc5d0\uc11c \ub098\uc628 \ub2e4\ub77c\uc9c0(Darazi)\ub77c\ub294 \uc778\ubb3c\uc774 \uc228\uc740 \uc774\ub9d8 '\ub9c8\ud750\ub514'\ub97c \uc790\ucc98\ud558\uba74\uc11c \ucc3d\uc2dc\ud588\ub2e4. \ube44\ubc00\uc8fc\uc758\uac00 \uac15\ud558\uba70, \uc2dc\ub9ac\uc544\uc640 \ub808\ubc14\ub17c\uc5d0 \uc8fc\ub85c \ubd84\ud3ec\ud558\uace0 \uc788\ub2e4. \uc2e0\uc758 \uc608\uc5b8\uc790 \ubb34\ud568\ub9c8\ub4dc\ub97c \ub530\ub974\ub294 \uc774\uc2ac\ub78c\uacfc \ub2ec\ub9ac \ud558\ud0b4 \uc774\ub77c\ub294 \uce7c\ub9ac\ud30c\uac00 \uc2e0\uc758 \ud604\uc2e0\uc774\ub77c \uc8fc\uc7a5\ud558\uba70 \uc77c\uc2e0\ub860\uc744 \ubbff\ub294\ub2e4. \uc77c\ubd80\ub2e4\ucc98\uc81c\ub294 \uae08\uc9c0\ub3fc \uc788\uace0 \uc5ec\uc131\ub4e4\uc758 \uc9c0\uc704\ub294 \ub0a8\uc131\uacfc \ub3d9\ub4f1\ud558\uba70 \uc601\ud63c\uc740 \uc724\ud68c\ud55c\ub2e4\uace0 \ubbff\ub294\ub2e4. \uc774\uc2ac\ub78c\uc758 \ud558\uc988(\uc21c\ub840) \uae30\uac04 \ucd95\uc81c\ub97c \uc5f4\uae34 \ud558\uc9c0\ub9cc \uc21c\ub840 \uc790\uccb4\ub294 \ud558\uc9c0 \uc54a\uc73c\uba70 \ub77c\ub9c8\ub2e8 \uae08\uc2dd\ub3c4 \ud558\uc9c0 \uc54a\ub294\ub2e4. \uacf5\ub3d9\uccb4\ub97c \ub9cc\ub4e4\uc5b4 \uc0dd\ud65c\ud558\ub294 \uacbd\uc6b0\uac00 \ub9ce\uc73c\uba70 \uae08\uc695\uc801\uc778 \uc0dd\ud65c\uc744 \ucc2c\uc591\ud55c\ub2e4. \uad50\ub9ac\uc640 \uad00\ud589\uc73c\ub85c \ubcf4\uc544, \uc774\uc2ac\ub78c\uad50\uc758 \uc885\ud30c\ub77c\uae30\ubcf4\ub2e4\ub294 \ubcc4\ub3c4\uc758 \uc885\uad50\ub85c \ubcf4\ub294 \uc2dc\uac01\uc774 \uc9c0\ubc30\uc801\uc774\ub2e4.", "paragraph_answer": "\ub4dc\ub8e8\uc988\ud30c(Druze)\ub294 11\uc138\uae30 \uc774\uc2a4\ub9c8\uc77c\ud30c\uc5d0\uc11c \ub098\uc628 \ub2e4\ub77c\uc9c0(Darazi)\ub77c\ub294 \uc778\ubb3c\uc774 \uc228\uc740 \uc774\ub9d8 '\ub9c8\ud750\ub514'\ub97c \uc790\ucc98\ud558\uba74\uc11c \ucc3d\uc2dc\ud588\ub2e4. \ube44\ubc00\uc8fc\uc758\uac00 \uac15\ud558\uba70, \uc2dc\ub9ac\uc544\uc640 \ub808\ubc14\ub17c\uc5d0 \uc8fc\ub85c \ubd84\ud3ec\ud558\uace0 \uc788\ub2e4. \uc2e0\uc758 \uc608\uc5b8\uc790 \ubb34\ud568\ub9c8\ub4dc\ub97c \ub530\ub974\ub294 \uc774\uc2ac\ub78c\uacfc \ub2ec\ub9ac \ud558\ud0b4 \uc774\ub77c\ub294 \uce7c\ub9ac\ud30c\uac00 \uc2e0\uc758 \ud604\uc2e0\uc774\ub77c \uc8fc\uc7a5\ud558\uba70 \uc77c\uc2e0\ub860\uc744 \ubbff\ub294\ub2e4. \uc77c\ubd80\ub2e4\ucc98\uc81c\ub294 \uae08\uc9c0\ub3fc \uc788\uace0 \uc5ec\uc131\ub4e4\uc758 \uc9c0\uc704\ub294 \ub0a8\uc131\uacfc \ub3d9\ub4f1\ud558\uba70 \uc601\ud63c\uc740 \uc724\ud68c\ud55c\ub2e4\uace0 \ubbff\ub294\ub2e4. \uc774\uc2ac\ub78c\uc758 \ud558\uc988(\uc21c\ub840) \uae30\uac04 \ucd95\uc81c\ub97c \uc5f4\uae34 \ud558\uc9c0\ub9cc \uc21c\ub840 \uc790\uccb4\ub294 \ud558\uc9c0 \uc54a\uc73c\uba70 \ub77c\ub9c8\ub2e8 \uae08\uc2dd\ub3c4 \ud558\uc9c0 \uc54a\ub294\ub2e4. \uacf5\ub3d9\uccb4\ub97c \ub9cc\ub4e4\uc5b4 \uc0dd\ud65c\ud558\ub294 \uacbd\uc6b0\uac00 \ub9ce\uc73c\uba70 \uae08\uc695\uc801\uc778 \uc0dd\ud65c\uc744 \ucc2c\uc591\ud55c\ub2e4. \uad50\ub9ac\uc640 \uad00\ud589\uc73c\ub85c \ubcf4\uc544, \uc774\uc2ac\ub78c\uad50\uc758 \uc885\ud30c\ub77c\uae30\ubcf4\ub2e4\ub294 \ubcc4\ub3c4\uc758 \uc885\uad50\ub85c \ubcf4\ub294 \uc2dc\uac01\uc774 \uc9c0\ubc30\uc801\uc774\ub2e4.", "sentence_answer": "\uc2e0\uc758 \uc608\uc5b8\uc790 \ubb34\ud568\ub9c8\ub4dc\ub97c \ub530\ub974\ub294 \uc774\uc2ac\ub78c\uacfc \ub2ec\ub9ac \ud558\ud0b4 \uc774\ub77c\ub294 \uce7c\ub9ac\ud30c\uac00 \uc2e0\uc758 \ud604\uc2e0\uc774\ub77c \uc8fc\uc7a5\ud558\uba70 \uc77c\uc2e0\ub860\uc744 \ubbff\ub294\ub2e4.", "paragraph_id": "6545694-8-2", "paragraph_question": "question: \ub4dc\ub8e8\uc988\ud30c\ub294 \ub204\uac00 \uc2e0\uc758 \ud604\uc2e0\uc774\ub77c \uc8fc\uc7a5\ud558\ub294\uac00?, context: \ub4dc\ub8e8\uc988\ud30c(Druze)\ub294 11\uc138\uae30 \uc774\uc2a4\ub9c8\uc77c\ud30c\uc5d0\uc11c \ub098\uc628 \ub2e4\ub77c\uc9c0(Darazi)\ub77c\ub294 \uc778\ubb3c\uc774 \uc228\uc740 \uc774\ub9d8 '\ub9c8\ud750\ub514'\ub97c \uc790\ucc98\ud558\uba74\uc11c \ucc3d\uc2dc\ud588\ub2e4. \ube44\ubc00\uc8fc\uc758\uac00 \uac15\ud558\uba70, \uc2dc\ub9ac\uc544\uc640 \ub808\ubc14\ub17c\uc5d0 \uc8fc\ub85c \ubd84\ud3ec\ud558\uace0 \uc788\ub2e4. \uc2e0\uc758 \uc608\uc5b8\uc790 \ubb34\ud568\ub9c8\ub4dc\ub97c \ub530\ub974\ub294 \uc774\uc2ac\ub78c\uacfc \ub2ec\ub9ac \ud558\ud0b4\uc774\ub77c\ub294 \uce7c\ub9ac\ud30c\uac00 \uc2e0\uc758 \ud604\uc2e0\uc774\ub77c \uc8fc\uc7a5\ud558\uba70 \uc77c\uc2e0\ub860\uc744 \ubbff\ub294\ub2e4. \uc77c\ubd80\ub2e4\ucc98\uc81c\ub294 \uae08\uc9c0\ub3fc \uc788\uace0 \uc5ec\uc131\ub4e4\uc758 \uc9c0\uc704\ub294 \ub0a8\uc131\uacfc \ub3d9\ub4f1\ud558\uba70 \uc601\ud63c\uc740 \uc724\ud68c\ud55c\ub2e4\uace0 \ubbff\ub294\ub2e4. \uc774\uc2ac\ub78c\uc758 \ud558\uc988(\uc21c\ub840) \uae30\uac04 \ucd95\uc81c\ub97c \uc5f4\uae34 \ud558\uc9c0\ub9cc \uc21c\ub840 \uc790\uccb4\ub294 \ud558\uc9c0 \uc54a\uc73c\uba70 \ub77c\ub9c8\ub2e8 \uae08\uc2dd\ub3c4 \ud558\uc9c0 \uc54a\ub294\ub2e4. \uacf5\ub3d9\uccb4\ub97c \ub9cc\ub4e4\uc5b4 \uc0dd\ud65c\ud558\ub294 \uacbd\uc6b0\uac00 \ub9ce\uc73c\uba70 \uae08\uc695\uc801\uc778 \uc0dd\ud65c\uc744 \ucc2c\uc591\ud55c\ub2e4. \uad50\ub9ac\uc640 \uad00\ud589\uc73c\ub85c \ubcf4\uc544, \uc774\uc2ac\ub78c\uad50\uc758 \uc885\ud30c\ub77c\uae30\ubcf4\ub2e4\ub294 \ubcc4\ub3c4\uc758 \uc885\uad50\ub85c \ubcf4\ub294 \uc2dc\uac01\uc774 \uc9c0\ubc30\uc801\uc774\ub2e4."} {"question": "\uc21c\ucc9c\ub300\ud559\uad50\ub294 1982\ub144 4\ub144\uc81c \ub300\ud559\uc73c\ub85c \uc7ac\uc815\ube44\ub418\uba74\uc11c \uba87 \uac1c\uc758 \ud559\uacfc\ub97c \uac1c\uc124\ud558\uc600\ub294\uac00?", "paragraph": "1982\ub144 4\ub144\uc81c \ub300\ud559\uc73c\ub85c\uc758 \uac1c\ud3b8\uacfc \ud568\uaed8 \ub18d\ud559\uacfc \ub4f1 10\uac1c \ud559\uacfc\ub97c \uac1c\uc124, 1987\ub144\uc5d0\ub294 \uc11d\uc0ac\uacfc\uc815\uc758 \ub300\ud559\uc6d0\uc774 \uc124\uce58, 1991\ub144\uc5d0 5\uac1c \ub2e8\uacfc\ub300\ud559\uc744 \uac16\ucd98 \uc885\ud569\ub300\ud559\uad50\ub85c \uc2b9\uaca9\ub410\ub2e4. \ubd80\uc18d\uc2dc\uc124\ub85c 70\uc8fc\ub144 \uae30\ub150\uad00, \ub3c4\uc11c\uad00, \ubc15\ubb3c\uad00, \uc815\ubcf4\uc804\uc0b0\uc6d0, \ud559\uc0dd\uc0dd\ud65c\uad00, \ud3c9\uc0dd\uad50\uc721\uc6d0, \uc601\ub18d\uad50\uc721\uc6d0, \uc5b8\ub860\uc0ac, \uc5b4\ud559\uc6d0, \uacf5\ub3d9\uc2e4\ud5d8\uc2e4\uc2b5\uad00, \uace0\uc2dc\uc6d0, \uacfc\ud559\uc601\uc7ac\uad50\uc721\uc6d0, \uc0dd\ud65c\uccb4\uc721\uc9c0\ub3c4\uc790\uc5f0\uc218\uc6d0, \uc0b0\ud559\ud611\ub825\ub2e8, \uc5ec\ub300\uc0dd\ucee4\ub9ac\uc5b4\uac1c\ubc1c\uc13c\ud130\uac00 \uc124\uce58\ub418\uc5b4\uc788\uc73c\uba70, \uc5f0\uad6c\uc2dc\uc124\uc740 \ucd1d 21\uae30\uad00\uc73c\ub85c \uc9c0\uc5ed\uac1c\ubc1c\uc5f0\uad6c\uc18c \ub18d\uc5c5\uacfc\ud559\uc5f0\uad6c\uc18c \uae30\ucd08\uacfc\ud559\uc5f0\uad6c\uc18c \uacf5\uc5c5\uae30\uc220\uc5f0\uad6c\uc18c \ub0a8\ub3c4\ubb38\ud654\uc5f0\uad6c\uc18c \uc0ac\ud68c\uacfc\ud559\uc5f0\uad6c\uc18c \uc911\uc18c\uae30\uc5c5\uacbd\uc601\uc5f0\uad6c\uc18c \uc720\uc6a9\ucc9c\uc5f0\uc790\uc6d0\uc5f0\uad6c\uc18c \uc720\uc804\uacf5\ud559\uc5f0\uad6c\uc18c, \ud658\uacbd \ubc0f \ub3c4\uc2dc\ubb38\uc81c \uc5f0\uad6c\uc18c, \uc0dd\ud65c\uccb4\uc721\uc5f0\uad6c\uc18c, \uc2dd\ud488\uc0b0\uc5c5\uc5f0\uad6c\uc18c, \uace0\ubd84\uc790\ud654\ud559\uc5f0\uad6c\uc18c, \ub098\ub178\uae30\uc220\uc13c\ud130, \uc0b0\uc5c5\ud3d0\uae30\ubb3c\uc18c\uc7ac\ud654 \uc5f0\uad6c\uc13c\ud130, \ucd08\uace0\uc18d\uc815\ubcf4\ud1b5\uc2e0\uc5f0\uad6c\uc18c, \ub098\ub178\uc18c\uc7ac\ubb3c\uc131\ud3c9\uac00\uc5f0\uad6c\ub2e8, \uacfc\ud559\uad50\uc721\uc5f0\uad6c\uc18c, \uc804\ub0a8\uc2e4\ubc84\ubcf5\uc9c0\uc5f0\uad6c\uc13c\ud130, \uacf5\uacf5\uc11c\ube44\uc2a4\ud601\uc2e0\uc5f0\uad6c\uc13c\ud130\uac00 \uc804\ub0a8 \ub3d9\ubd80\uad8c\uc758 \ub3c4\uc2dc \ubc0f \uc9c0\uc5ed\uc0ac\ud68c\uc758 \uac1c\ubc1c\uc744 \uc704\ud55c \uc815\uce58\u318d\uacbd\uc81c\u318d\uc0ac\ud68c\u318d\ubb38\ud654\u318d\uad50\uc721\u318d\ubc95\ub960\u318d\ud658\uacbd \ubc0f \uacf5\uac04\uccb4\uc81c \ub4f1\uc758 \uc81c\ubd84\uc57c\ub97c \uc885\ud569\uc801\uc73c\ub85c \ubd84\uc11d\ud558\uc5ec \uadf8 \ubc1c\uc804 \ubaa8\ud615\uc744 \uc5f0\uad6c \ud3c9\uac00\ud568\uc744 \ubaa9\uc801\uc73c\ub85c \uc124\uce58\ub418\uc5b4\uc788\ub2e4. \uad6d\uc81c\uad50\ub958\ub85c\ub294 \uc138\uacc4 15\uac1c\uad6d 45\uac1c \ub300\ud559 \ubc0f 2\uac1c\uc758 \uad50\uc721\uae30\uad00\uacfc \ub124\ud2b8\uc6cc\ud06c\ub97c \ud615\uc131\ud558\uace0 \uc788\ub2e4.", "answer": "10\uac1c", "sentence": "1982\ub144 4\ub144\uc81c \ub300\ud559\uc73c\ub85c\uc758 \uac1c\ud3b8\uacfc \ud568\uaed8 \ub18d\ud559\uacfc \ub4f1 10\uac1c \ud559\uacfc\ub97c \uac1c\uc124, 1987\ub144\uc5d0\ub294 \uc11d\uc0ac\uacfc\uc815\uc758 \ub300\ud559\uc6d0\uc774 \uc124\uce58, 1991\ub144\uc5d0 5\uac1c \ub2e8\uacfc\ub300\ud559\uc744 \uac16\ucd98 \uc885\ud569\ub300\ud559\uad50\ub85c \uc2b9\uaca9\ub410\ub2e4.", "paragraph_sentence": " 1982\ub144 4\ub144\uc81c \ub300\ud559\uc73c\ub85c\uc758 \uac1c\ud3b8\uacfc \ud568\uaed8 \ub18d\ud559\uacfc \ub4f1 10\uac1c \ud559\uacfc\ub97c \uac1c\uc124, 1987\ub144\uc5d0\ub294 \uc11d\uc0ac\uacfc\uc815\uc758 \ub300\ud559\uc6d0\uc774 \uc124\uce58, 1991\ub144\uc5d0 5\uac1c \ub2e8\uacfc\ub300\ud559\uc744 \uac16\ucd98 \uc885\ud569\ub300\ud559\uad50\ub85c \uc2b9\uaca9\ub410\ub2e4. \ubd80\uc18d\uc2dc\uc124\ub85c 70\uc8fc\ub144 \uae30\ub150\uad00, \ub3c4\uc11c\uad00, \ubc15\ubb3c\uad00, \uc815\ubcf4\uc804\uc0b0\uc6d0, \ud559\uc0dd\uc0dd\ud65c\uad00, \ud3c9\uc0dd\uad50\uc721\uc6d0, \uc601\ub18d\uad50\uc721\uc6d0, \uc5b8\ub860\uc0ac, \uc5b4\ud559\uc6d0, \uacf5\ub3d9\uc2e4\ud5d8\uc2e4\uc2b5\uad00, \uace0\uc2dc\uc6d0, \uacfc\ud559\uc601\uc7ac\uad50\uc721\uc6d0, \uc0dd\ud65c\uccb4\uc721\uc9c0\ub3c4\uc790\uc5f0\uc218\uc6d0, \uc0b0\ud559\ud611\ub825\ub2e8, \uc5ec\ub300\uc0dd\ucee4\ub9ac\uc5b4\uac1c\ubc1c\uc13c\ud130\uac00 \uc124\uce58\ub418\uc5b4\uc788\uc73c\uba70, \uc5f0\uad6c\uc2dc\uc124\uc740 \ucd1d 21\uae30\uad00\uc73c\ub85c \uc9c0\uc5ed\uac1c\ubc1c\uc5f0\uad6c\uc18c \ub18d\uc5c5\uacfc\ud559\uc5f0\uad6c\uc18c \uae30\ucd08\uacfc\ud559\uc5f0\uad6c\uc18c \uacf5\uc5c5\uae30\uc220\uc5f0\uad6c\uc18c \ub0a8\ub3c4\ubb38\ud654\uc5f0\uad6c\uc18c \uc0ac\ud68c\uacfc\ud559\uc5f0\uad6c\uc18c \uc911\uc18c\uae30\uc5c5\uacbd\uc601\uc5f0\uad6c\uc18c \uc720\uc6a9\ucc9c\uc5f0\uc790\uc6d0\uc5f0\uad6c\uc18c \uc720\uc804\uacf5\ud559\uc5f0\uad6c\uc18c, \ud658\uacbd \ubc0f \ub3c4\uc2dc\ubb38\uc81c \uc5f0\uad6c\uc18c, \uc0dd\ud65c\uccb4\uc721\uc5f0\uad6c\uc18c, \uc2dd\ud488\uc0b0\uc5c5\uc5f0\uad6c\uc18c, \uace0\ubd84\uc790\ud654\ud559\uc5f0\uad6c\uc18c, \ub098\ub178\uae30\uc220\uc13c\ud130, \uc0b0\uc5c5\ud3d0\uae30\ubb3c\uc18c\uc7ac\ud654 \uc5f0\uad6c\uc13c\ud130, \ucd08\uace0\uc18d\uc815\ubcf4\ud1b5\uc2e0\uc5f0\uad6c\uc18c, \ub098\ub178\uc18c\uc7ac\ubb3c\uc131\ud3c9\uac00\uc5f0\uad6c\ub2e8, \uacfc\ud559\uad50\uc721\uc5f0\uad6c\uc18c, \uc804\ub0a8\uc2e4\ubc84\ubcf5\uc9c0\uc5f0\uad6c\uc13c\ud130, \uacf5\uacf5\uc11c\ube44\uc2a4\ud601\uc2e0\uc5f0\uad6c\uc13c\ud130\uac00 \uc804\ub0a8 \ub3d9\ubd80\uad8c\uc758 \ub3c4\uc2dc \ubc0f \uc9c0\uc5ed\uc0ac\ud68c\uc758 \uac1c\ubc1c\uc744 \uc704\ud55c \uc815\uce58\u318d\uacbd\uc81c\u318d\uc0ac\ud68c\u318d\ubb38\ud654\u318d\uad50\uc721\u318d\ubc95\ub960\u318d\ud658\uacbd \ubc0f \uacf5\uac04\uccb4\uc81c \ub4f1\uc758 \uc81c\ubd84\uc57c\ub97c \uc885\ud569\uc801\uc73c\ub85c \ubd84\uc11d\ud558\uc5ec \uadf8 \ubc1c\uc804 \ubaa8\ud615\uc744 \uc5f0\uad6c \ud3c9\uac00\ud568\uc744 \ubaa9\uc801\uc73c\ub85c \uc124\uce58\ub418\uc5b4\uc788\ub2e4. \uad6d\uc81c\uad50\ub958\ub85c\ub294 \uc138\uacc4 15\uac1c\uad6d 45\uac1c \ub300\ud559 \ubc0f 2\uac1c\uc758 \uad50\uc721\uae30\uad00\uacfc \ub124\ud2b8\uc6cc\ud06c\ub97c \ud615\uc131\ud558\uace0 \uc788\ub2e4.", "paragraph_answer": "1982\ub144 4\ub144\uc81c \ub300\ud559\uc73c\ub85c\uc758 \uac1c\ud3b8\uacfc \ud568\uaed8 \ub18d\ud559\uacfc \ub4f1 10\uac1c \ud559\uacfc\ub97c \uac1c\uc124, 1987\ub144\uc5d0\ub294 \uc11d\uc0ac\uacfc\uc815\uc758 \ub300\ud559\uc6d0\uc774 \uc124\uce58, 1991\ub144\uc5d0 5\uac1c \ub2e8\uacfc\ub300\ud559\uc744 \uac16\ucd98 \uc885\ud569\ub300\ud559\uad50\ub85c \uc2b9\uaca9\ub410\ub2e4. \ubd80\uc18d\uc2dc\uc124\ub85c 70\uc8fc\ub144 \uae30\ub150\uad00, \ub3c4\uc11c\uad00, \ubc15\ubb3c\uad00, \uc815\ubcf4\uc804\uc0b0\uc6d0, \ud559\uc0dd\uc0dd\ud65c\uad00, \ud3c9\uc0dd\uad50\uc721\uc6d0, \uc601\ub18d\uad50\uc721\uc6d0, \uc5b8\ub860\uc0ac, \uc5b4\ud559\uc6d0, \uacf5\ub3d9\uc2e4\ud5d8\uc2e4\uc2b5\uad00, \uace0\uc2dc\uc6d0, \uacfc\ud559\uc601\uc7ac\uad50\uc721\uc6d0, \uc0dd\ud65c\uccb4\uc721\uc9c0\ub3c4\uc790\uc5f0\uc218\uc6d0, \uc0b0\ud559\ud611\ub825\ub2e8, \uc5ec\ub300\uc0dd\ucee4\ub9ac\uc5b4\uac1c\ubc1c\uc13c\ud130\uac00 \uc124\uce58\ub418\uc5b4\uc788\uc73c\uba70, \uc5f0\uad6c\uc2dc\uc124\uc740 \ucd1d 21\uae30\uad00\uc73c\ub85c \uc9c0\uc5ed\uac1c\ubc1c\uc5f0\uad6c\uc18c \ub18d\uc5c5\uacfc\ud559\uc5f0\uad6c\uc18c \uae30\ucd08\uacfc\ud559\uc5f0\uad6c\uc18c \uacf5\uc5c5\uae30\uc220\uc5f0\uad6c\uc18c \ub0a8\ub3c4\ubb38\ud654\uc5f0\uad6c\uc18c \uc0ac\ud68c\uacfc\ud559\uc5f0\uad6c\uc18c \uc911\uc18c\uae30\uc5c5\uacbd\uc601\uc5f0\uad6c\uc18c \uc720\uc6a9\ucc9c\uc5f0\uc790\uc6d0\uc5f0\uad6c\uc18c \uc720\uc804\uacf5\ud559\uc5f0\uad6c\uc18c, \ud658\uacbd \ubc0f \ub3c4\uc2dc\ubb38\uc81c \uc5f0\uad6c\uc18c, \uc0dd\ud65c\uccb4\uc721\uc5f0\uad6c\uc18c, \uc2dd\ud488\uc0b0\uc5c5\uc5f0\uad6c\uc18c, \uace0\ubd84\uc790\ud654\ud559\uc5f0\uad6c\uc18c, \ub098\ub178\uae30\uc220\uc13c\ud130, \uc0b0\uc5c5\ud3d0\uae30\ubb3c\uc18c\uc7ac\ud654 \uc5f0\uad6c\uc13c\ud130, \ucd08\uace0\uc18d\uc815\ubcf4\ud1b5\uc2e0\uc5f0\uad6c\uc18c, \ub098\ub178\uc18c\uc7ac\ubb3c\uc131\ud3c9\uac00\uc5f0\uad6c\ub2e8, \uacfc\ud559\uad50\uc721\uc5f0\uad6c\uc18c, \uc804\ub0a8\uc2e4\ubc84\ubcf5\uc9c0\uc5f0\uad6c\uc13c\ud130, \uacf5\uacf5\uc11c\ube44\uc2a4\ud601\uc2e0\uc5f0\uad6c\uc13c\ud130\uac00 \uc804\ub0a8 \ub3d9\ubd80\uad8c\uc758 \ub3c4\uc2dc \ubc0f \uc9c0\uc5ed\uc0ac\ud68c\uc758 \uac1c\ubc1c\uc744 \uc704\ud55c \uc815\uce58\u318d\uacbd\uc81c\u318d\uc0ac\ud68c\u318d\ubb38\ud654\u318d\uad50\uc721\u318d\ubc95\ub960\u318d\ud658\uacbd \ubc0f \uacf5\uac04\uccb4\uc81c \ub4f1\uc758 \uc81c\ubd84\uc57c\ub97c \uc885\ud569\uc801\uc73c\ub85c \ubd84\uc11d\ud558\uc5ec \uadf8 \ubc1c\uc804 \ubaa8\ud615\uc744 \uc5f0\uad6c \ud3c9\uac00\ud568\uc744 \ubaa9\uc801\uc73c\ub85c \uc124\uce58\ub418\uc5b4\uc788\ub2e4. \uad6d\uc81c\uad50\ub958\ub85c\ub294 \uc138\uacc4 15\uac1c\uad6d 45\uac1c \ub300\ud559 \ubc0f 2\uac1c\uc758 \uad50\uc721\uae30\uad00\uacfc \ub124\ud2b8\uc6cc\ud06c\ub97c \ud615\uc131\ud558\uace0 \uc788\ub2e4.", "sentence_answer": "1982\ub144 4\ub144\uc81c \ub300\ud559\uc73c\ub85c\uc758 \uac1c\ud3b8\uacfc \ud568\uaed8 \ub18d\ud559\uacfc \ub4f1 10\uac1c \ud559\uacfc\ub97c \uac1c\uc124, 1987\ub144\uc5d0\ub294 \uc11d\uc0ac\uacfc\uc815\uc758 \ub300\ud559\uc6d0\uc774 \uc124\uce58, 1991\ub144\uc5d0 5\uac1c \ub2e8\uacfc\ub300\ud559\uc744 \uac16\ucd98 \uc885\ud569\ub300\ud559\uad50\ub85c \uc2b9\uaca9\ub410\ub2e4.", "paragraph_id": "6537159-1-0", "paragraph_question": "question: \uc21c\ucc9c\ub300\ud559\uad50\ub294 1982\ub144 4\ub144\uc81c \ub300\ud559\uc73c\ub85c \uc7ac\uc815\ube44\ub418\uba74\uc11c \uba87 \uac1c\uc758 \ud559\uacfc\ub97c \uac1c\uc124\ud558\uc600\ub294\uac00?, context: 1982\ub144 4\ub144\uc81c \ub300\ud559\uc73c\ub85c\uc758 \uac1c\ud3b8\uacfc \ud568\uaed8 \ub18d\ud559\uacfc \ub4f1 10\uac1c \ud559\uacfc\ub97c \uac1c\uc124, 1987\ub144\uc5d0\ub294 \uc11d\uc0ac\uacfc\uc815\uc758 \ub300\ud559\uc6d0\uc774 \uc124\uce58, 1991\ub144\uc5d0 5\uac1c \ub2e8\uacfc\ub300\ud559\uc744 \uac16\ucd98 \uc885\ud569\ub300\ud559\uad50\ub85c \uc2b9\uaca9\ub410\ub2e4. \ubd80\uc18d\uc2dc\uc124\ub85c 70\uc8fc\ub144 \uae30\ub150\uad00, \ub3c4\uc11c\uad00, \ubc15\ubb3c\uad00, \uc815\ubcf4\uc804\uc0b0\uc6d0, \ud559\uc0dd\uc0dd\ud65c\uad00, \ud3c9\uc0dd\uad50\uc721\uc6d0, \uc601\ub18d\uad50\uc721\uc6d0, \uc5b8\ub860\uc0ac, \uc5b4\ud559\uc6d0, \uacf5\ub3d9\uc2e4\ud5d8\uc2e4\uc2b5\uad00, \uace0\uc2dc\uc6d0, \uacfc\ud559\uc601\uc7ac\uad50\uc721\uc6d0, \uc0dd\ud65c\uccb4\uc721\uc9c0\ub3c4\uc790\uc5f0\uc218\uc6d0, \uc0b0\ud559\ud611\ub825\ub2e8, \uc5ec\ub300\uc0dd\ucee4\ub9ac\uc5b4\uac1c\ubc1c\uc13c\ud130\uac00 \uc124\uce58\ub418\uc5b4\uc788\uc73c\uba70, \uc5f0\uad6c\uc2dc\uc124\uc740 \ucd1d 21\uae30\uad00\uc73c\ub85c \uc9c0\uc5ed\uac1c\ubc1c\uc5f0\uad6c\uc18c \ub18d\uc5c5\uacfc\ud559\uc5f0\uad6c\uc18c \uae30\ucd08\uacfc\ud559\uc5f0\uad6c\uc18c \uacf5\uc5c5\uae30\uc220\uc5f0\uad6c\uc18c \ub0a8\ub3c4\ubb38\ud654\uc5f0\uad6c\uc18c \uc0ac\ud68c\uacfc\ud559\uc5f0\uad6c\uc18c \uc911\uc18c\uae30\uc5c5\uacbd\uc601\uc5f0\uad6c\uc18c \uc720\uc6a9\ucc9c\uc5f0\uc790\uc6d0\uc5f0\uad6c\uc18c \uc720\uc804\uacf5\ud559\uc5f0\uad6c\uc18c, \ud658\uacbd \ubc0f \ub3c4\uc2dc\ubb38\uc81c \uc5f0\uad6c\uc18c, \uc0dd\ud65c\uccb4\uc721\uc5f0\uad6c\uc18c, \uc2dd\ud488\uc0b0\uc5c5\uc5f0\uad6c\uc18c, \uace0\ubd84\uc790\ud654\ud559\uc5f0\uad6c\uc18c, \ub098\ub178\uae30\uc220\uc13c\ud130, \uc0b0\uc5c5\ud3d0\uae30\ubb3c\uc18c\uc7ac\ud654 \uc5f0\uad6c\uc13c\ud130, \ucd08\uace0\uc18d\uc815\ubcf4\ud1b5\uc2e0\uc5f0\uad6c\uc18c, \ub098\ub178\uc18c\uc7ac\ubb3c\uc131\ud3c9\uac00\uc5f0\uad6c\ub2e8, \uacfc\ud559\uad50\uc721\uc5f0\uad6c\uc18c, \uc804\ub0a8\uc2e4\ubc84\ubcf5\uc9c0\uc5f0\uad6c\uc13c\ud130, \uacf5\uacf5\uc11c\ube44\uc2a4\ud601\uc2e0\uc5f0\uad6c\uc13c\ud130\uac00 \uc804\ub0a8 \ub3d9\ubd80\uad8c\uc758 \ub3c4\uc2dc \ubc0f \uc9c0\uc5ed\uc0ac\ud68c\uc758 \uac1c\ubc1c\uc744 \uc704\ud55c \uc815\uce58\u318d\uacbd\uc81c\u318d\uc0ac\ud68c\u318d\ubb38\ud654\u318d\uad50\uc721\u318d\ubc95\ub960\u318d\ud658\uacbd \ubc0f \uacf5\uac04\uccb4\uc81c \ub4f1\uc758 \uc81c\ubd84\uc57c\ub97c \uc885\ud569\uc801\uc73c\ub85c \ubd84\uc11d\ud558\uc5ec \uadf8 \ubc1c\uc804 \ubaa8\ud615\uc744 \uc5f0\uad6c \ud3c9\uac00\ud568\uc744 \ubaa9\uc801\uc73c\ub85c \uc124\uce58\ub418\uc5b4\uc788\ub2e4. \uad6d\uc81c\uad50\ub958\ub85c\ub294 \uc138\uacc4 15\uac1c\uad6d 45\uac1c \ub300\ud559 \ubc0f 2\uac1c\uc758 \uad50\uc721\uae30\uad00\uacfc \ub124\ud2b8\uc6cc\ud06c\ub97c \ud615\uc131\ud558\uace0 \uc788\ub2e4."} {"question": "2014\ub144 \uc138\uacc4 \uc1fc\ud2b8\ud2b8\ub799 \uc120\uc218\uad8c \ub300\ud68c\uac00 \uac1c\ucd5c\ub41c \uad6d\uac00\uc640 \ub3c4\uc2dc\ub294 \uc5b4\ub514\uc778\uac00?", "paragraph": "2014\ub144 \uce90\ub098\ub2e4 \ubaac\ud2b8\ub9ac\uc62c \uc138\uacc4 \uc1fc\ud2b8\ud2b8\ub799 \uc120\uc218\uad8c \ub300\ud68c\uc5d0\uc11c 3\uc6d4 15\uc77c \uc790\uc2e0\uc758 \uc138\uacc4 \uc120\uc218\uad8c \ub300\ud68c 1500m \uccab \ubc88\uc9f8 \uae08\uba54\ub2ec\uc744 \ud68d\ub4dd\ud558\uc600\ub2e4. 3\uc6d4 16\uc77c \uc5ec\uc790 500m\uc5d0\uc11c\ub294 \uc900\uacb0\uc2b9\uc804\uc5d0\uc11c \uc911\uad6d\uc758 \ud310\ucee4\uc2e0 \uc120\uc218\uc5d0\uac8c \ubc18\uce59\uc744 \ub2f9\ud588\uc9c0\ub9cc, \uc2ec\ud310\uc774 \ud310\ucee4\uc2e0\uc5d0\uac8c \ud328\ub110\ud2f0\ub97c \uc8fc\uc9c0 \uc54a\uc544, \uacb0\uc2b9\uc804\uc5d0 \uc9c4\ucd9c\ud558\uc9c0 \ubabb\ud558\uc600\ub2e4. 3\uc6d4 17\uc77c \uc790\uc2e0\uc758 \uc138\uacc4 \uc120\uc218\uad8c \ub300\ud68c 1000m \uccab \ubc88\uc9f8 \uae08\uba54\ub2ec\uc744 \ud68d\ub4dd\ud558\uc600\uace0, \uc288\ud37c\ud30c\uc774\ub110 3000m\uc5d0\uc11c\ub3c4 \uae08\uba54\ub2ec\uc744 \ud68d\ub4dd\ud558\uc5ec, \uc885\ud569 \uc6b0\uc2b9\uc744 \ucc28\uc9c0\ud558\uc600\ub2e4. \uc885\ud569 \uc6b0\uc2b9\uae4c\uc9c0 \ub530\uc838, \ub300\ud68c 4\uad00\uc655\uc774 \ub418\uc5c8\ub2e4. \ud2b9\ud788, \uc288\ud37c\ud30c\uc774\ub110 3000m\ub294 2013\ub144 \uc138\uacc4 \uc1fc\ud2b8\ud2b8\ub799 \uc120\uc218\uad8c \ub300\ud68c \uae08\uba54\ub2ec\uc5d0 \uc774\uc5b4 2\uc5f0\ud328\ub97c \ub2ec\uc131\ud558\uc600\ub2e4. \uac19\uc740 \ub0a0 \uc5f4\ub9b0 \uc5ec\uc790 3000m \uacc4\uc8fc \uacb0\uc2b9\uc804\uc5d0\uc11c\ub294 \uc2ec\uc11d\ud76c \uc120\uc218\uac00 \ub9c8\uc9c0\ub9c9 \ud55c \ubc14\ud034\ub97c \ub0a8\uaca8\ub193\uace0 \ud310\ucee4\uc2e0 \uc120\uc218\uc5d0\uac8c \ubc18\uce59\uc744 \ud588\ub2e4\ub294 \uc2ec\ud310\uc758 \ud310\uc815\uc73c\ub85c, \uc5ec\uc790 \uacc4\uc8fc \ud300\uc740 \uc544\uc27d\uac8c \uc2e4\uaca9\ub2f9\ud558\uc600\ub2e4.", "answer": "\uce90\ub098\ub2e4 \ubaac\ud2b8\ub9ac\uc62c", "sentence": "2014\ub144 \uce90\ub098\ub2e4 \ubaac\ud2b8\ub9ac\uc62c \uc138\uacc4 \uc1fc\ud2b8\ud2b8\ub799 \uc120\uc218\uad8c \ub300\ud68c\uc5d0\uc11c 3\uc6d4 15\uc77c \uc790\uc2e0\uc758 \uc138\uacc4 \uc120\uc218\uad8c \ub300\ud68c 1500m \uccab \ubc88\uc9f8 \uae08\uba54\ub2ec\uc744 \ud68d\ub4dd\ud558\uc600\ub2e4.", "paragraph_sentence": " 2014\ub144 \uce90\ub098\ub2e4 \ubaac\ud2b8\ub9ac\uc62c \uc138\uacc4 \uc1fc\ud2b8\ud2b8\ub799 \uc120\uc218\uad8c \ub300\ud68c\uc5d0\uc11c 3\uc6d4 15\uc77c \uc790\uc2e0\uc758 \uc138\uacc4 \uc120\uc218\uad8c \ub300\ud68c 1500m \uccab \ubc88\uc9f8 \uae08\uba54\ub2ec\uc744 \ud68d\ub4dd\ud558\uc600\ub2e4. 3\uc6d4 16\uc77c \uc5ec\uc790 500m\uc5d0\uc11c\ub294 \uc900\uacb0\uc2b9\uc804\uc5d0\uc11c \uc911\uad6d\uc758 \ud310\ucee4\uc2e0 \uc120\uc218\uc5d0\uac8c \ubc18\uce59\uc744 \ub2f9\ud588\uc9c0\ub9cc, \uc2ec\ud310\uc774 \ud310\ucee4\uc2e0\uc5d0\uac8c \ud328\ub110\ud2f0\ub97c \uc8fc\uc9c0 \uc54a\uc544, \uacb0\uc2b9\uc804\uc5d0 \uc9c4\ucd9c\ud558\uc9c0 \ubabb\ud558\uc600\ub2e4. 3\uc6d4 17\uc77c \uc790\uc2e0\uc758 \uc138\uacc4 \uc120\uc218\uad8c \ub300\ud68c 1000m \uccab \ubc88\uc9f8 \uae08\uba54\ub2ec\uc744 \ud68d\ub4dd\ud558\uc600\uace0, \uc288\ud37c\ud30c\uc774\ub110 3000m\uc5d0\uc11c\ub3c4 \uae08\uba54\ub2ec\uc744 \ud68d\ub4dd\ud558\uc5ec, \uc885\ud569 \uc6b0\uc2b9\uc744 \ucc28\uc9c0\ud558\uc600\ub2e4. \uc885\ud569 \uc6b0\uc2b9\uae4c\uc9c0 \ub530\uc838, \ub300\ud68c 4\uad00\uc655\uc774 \ub418\uc5c8\ub2e4. \ud2b9\ud788, \uc288\ud37c\ud30c\uc774\ub110 3000m\ub294 2013\ub144 \uc138\uacc4 \uc1fc\ud2b8\ud2b8\ub799 \uc120\uc218\uad8c \ub300\ud68c \uae08\uba54\ub2ec\uc5d0 \uc774\uc5b4 2\uc5f0\ud328\ub97c \ub2ec\uc131\ud558\uc600\ub2e4. \uac19\uc740 \ub0a0 \uc5f4\ub9b0 \uc5ec\uc790 3000m \uacc4\uc8fc \uacb0\uc2b9\uc804\uc5d0\uc11c\ub294 \uc2ec\uc11d\ud76c \uc120\uc218\uac00 \ub9c8\uc9c0\ub9c9 \ud55c \ubc14\ud034\ub97c \ub0a8\uaca8\ub193\uace0 \ud310\ucee4\uc2e0 \uc120\uc218\uc5d0\uac8c \ubc18\uce59\uc744 \ud588\ub2e4\ub294 \uc2ec\ud310\uc758 \ud310\uc815\uc73c\ub85c, \uc5ec\uc790 \uacc4\uc8fc \ud300\uc740 \uc544\uc27d\uac8c \uc2e4\uaca9\ub2f9\ud558\uc600\ub2e4.", "paragraph_answer": "2014\ub144 \uce90\ub098\ub2e4 \ubaac\ud2b8\ub9ac\uc62c \uc138\uacc4 \uc1fc\ud2b8\ud2b8\ub799 \uc120\uc218\uad8c \ub300\ud68c\uc5d0\uc11c 3\uc6d4 15\uc77c \uc790\uc2e0\uc758 \uc138\uacc4 \uc120\uc218\uad8c \ub300\ud68c 1500m \uccab \ubc88\uc9f8 \uae08\uba54\ub2ec\uc744 \ud68d\ub4dd\ud558\uc600\ub2e4. 3\uc6d4 16\uc77c \uc5ec\uc790 500m\uc5d0\uc11c\ub294 \uc900\uacb0\uc2b9\uc804\uc5d0\uc11c \uc911\uad6d\uc758 \ud310\ucee4\uc2e0 \uc120\uc218\uc5d0\uac8c \ubc18\uce59\uc744 \ub2f9\ud588\uc9c0\ub9cc, \uc2ec\ud310\uc774 \ud310\ucee4\uc2e0\uc5d0\uac8c \ud328\ub110\ud2f0\ub97c \uc8fc\uc9c0 \uc54a\uc544, \uacb0\uc2b9\uc804\uc5d0 \uc9c4\ucd9c\ud558\uc9c0 \ubabb\ud558\uc600\ub2e4. 3\uc6d4 17\uc77c \uc790\uc2e0\uc758 \uc138\uacc4 \uc120\uc218\uad8c \ub300\ud68c 1000m \uccab \ubc88\uc9f8 \uae08\uba54\ub2ec\uc744 \ud68d\ub4dd\ud558\uc600\uace0, \uc288\ud37c\ud30c\uc774\ub110 3000m\uc5d0\uc11c\ub3c4 \uae08\uba54\ub2ec\uc744 \ud68d\ub4dd\ud558\uc5ec, \uc885\ud569 \uc6b0\uc2b9\uc744 \ucc28\uc9c0\ud558\uc600\ub2e4. \uc885\ud569 \uc6b0\uc2b9\uae4c\uc9c0 \ub530\uc838, \ub300\ud68c 4\uad00\uc655\uc774 \ub418\uc5c8\ub2e4. \ud2b9\ud788, \uc288\ud37c\ud30c\uc774\ub110 3000m\ub294 2013\ub144 \uc138\uacc4 \uc1fc\ud2b8\ud2b8\ub799 \uc120\uc218\uad8c \ub300\ud68c \uae08\uba54\ub2ec\uc5d0 \uc774\uc5b4 2\uc5f0\ud328\ub97c \ub2ec\uc131\ud558\uc600\ub2e4. \uac19\uc740 \ub0a0 \uc5f4\ub9b0 \uc5ec\uc790 3000m \uacc4\uc8fc \uacb0\uc2b9\uc804\uc5d0\uc11c\ub294 \uc2ec\uc11d\ud76c \uc120\uc218\uac00 \ub9c8\uc9c0\ub9c9 \ud55c \ubc14\ud034\ub97c \ub0a8\uaca8\ub193\uace0 \ud310\ucee4\uc2e0 \uc120\uc218\uc5d0\uac8c \ubc18\uce59\uc744 \ud588\ub2e4\ub294 \uc2ec\ud310\uc758 \ud310\uc815\uc73c\ub85c, \uc5ec\uc790 \uacc4\uc8fc \ud300\uc740 \uc544\uc27d\uac8c \uc2e4\uaca9\ub2f9\ud558\uc600\ub2e4.", "sentence_answer": "2014\ub144 \uce90\ub098\ub2e4 \ubaac\ud2b8\ub9ac\uc62c \uc138\uacc4 \uc1fc\ud2b8\ud2b8\ub799 \uc120\uc218\uad8c \ub300\ud68c\uc5d0\uc11c 3\uc6d4 15\uc77c \uc790\uc2e0\uc758 \uc138\uacc4 \uc120\uc218\uad8c \ub300\ud68c 1500m \uccab \ubc88\uc9f8 \uae08\uba54\ub2ec\uc744 \ud68d\ub4dd\ud558\uc600\ub2e4.", "paragraph_id": "6519945-2-1", "paragraph_question": "question: 2014\ub144 \uc138\uacc4 \uc1fc\ud2b8\ud2b8\ub799 \uc120\uc218\uad8c \ub300\ud68c\uac00 \uac1c\ucd5c\ub41c \uad6d\uac00\uc640 \ub3c4\uc2dc\ub294 \uc5b4\ub514\uc778\uac00?, context: 2014\ub144 \uce90\ub098\ub2e4 \ubaac\ud2b8\ub9ac\uc62c \uc138\uacc4 \uc1fc\ud2b8\ud2b8\ub799 \uc120\uc218\uad8c \ub300\ud68c\uc5d0\uc11c 3\uc6d4 15\uc77c \uc790\uc2e0\uc758 \uc138\uacc4 \uc120\uc218\uad8c \ub300\ud68c 1500m \uccab \ubc88\uc9f8 \uae08\uba54\ub2ec\uc744 \ud68d\ub4dd\ud558\uc600\ub2e4. 3\uc6d4 16\uc77c \uc5ec\uc790 500m\uc5d0\uc11c\ub294 \uc900\uacb0\uc2b9\uc804\uc5d0\uc11c \uc911\uad6d\uc758 \ud310\ucee4\uc2e0 \uc120\uc218\uc5d0\uac8c \ubc18\uce59\uc744 \ub2f9\ud588\uc9c0\ub9cc, \uc2ec\ud310\uc774 \ud310\ucee4\uc2e0\uc5d0\uac8c \ud328\ub110\ud2f0\ub97c \uc8fc\uc9c0 \uc54a\uc544, \uacb0\uc2b9\uc804\uc5d0 \uc9c4\ucd9c\ud558\uc9c0 \ubabb\ud558\uc600\ub2e4. 3\uc6d4 17\uc77c \uc790\uc2e0\uc758 \uc138\uacc4 \uc120\uc218\uad8c \ub300\ud68c 1000m \uccab \ubc88\uc9f8 \uae08\uba54\ub2ec\uc744 \ud68d\ub4dd\ud558\uc600\uace0, \uc288\ud37c\ud30c\uc774\ub110 3000m\uc5d0\uc11c\ub3c4 \uae08\uba54\ub2ec\uc744 \ud68d\ub4dd\ud558\uc5ec, \uc885\ud569 \uc6b0\uc2b9\uc744 \ucc28\uc9c0\ud558\uc600\ub2e4. \uc885\ud569 \uc6b0\uc2b9\uae4c\uc9c0 \ub530\uc838, \ub300\ud68c 4\uad00\uc655\uc774 \ub418\uc5c8\ub2e4. \ud2b9\ud788, \uc288\ud37c\ud30c\uc774\ub110 3000m\ub294 2013\ub144 \uc138\uacc4 \uc1fc\ud2b8\ud2b8\ub799 \uc120\uc218\uad8c \ub300\ud68c \uae08\uba54\ub2ec\uc5d0 \uc774\uc5b4 2\uc5f0\ud328\ub97c \ub2ec\uc131\ud558\uc600\ub2e4. \uac19\uc740 \ub0a0 \uc5f4\ub9b0 \uc5ec\uc790 3000m \uacc4\uc8fc \uacb0\uc2b9\uc804\uc5d0\uc11c\ub294 \uc2ec\uc11d\ud76c \uc120\uc218\uac00 \ub9c8\uc9c0\ub9c9 \ud55c \ubc14\ud034\ub97c \ub0a8\uaca8\ub193\uace0 \ud310\ucee4\uc2e0 \uc120\uc218\uc5d0\uac8c \ubc18\uce59\uc744 \ud588\ub2e4\ub294 \uc2ec\ud310\uc758 \ud310\uc815\uc73c\ub85c, \uc5ec\uc790 \uacc4\uc8fc \ud300\uc740 \uc544\uc27d\uac8c \uc2e4\uaca9\ub2f9\ud558\uc600\ub2e4."} {"question": "\ubc25 \ub51c\ub7f0\uc774 \ub4dc\ub85c\uc789 \ubc0f \uadf8\ub9bc \uc11c\uc801\uc744 \ubc1c\uac04\ud558\uae30 \uc2dc\uc791\ud55c \ud574\ub294?", "paragraph": "\ub51c\ub7f0\uc758 \uac00\uc0ac\ub294 \ub113\uc740 \ubc94\uc704\ub97c \uc544\uc6b0\ub974\uba70 \uc815\uce58, \uc0ac\ud68c, \ucca0\ud559, \ubb38\ud559\uc758 \uc601\ud5a5\uc744 \ubc1b\ub294\ub2e4. \uc774\ub4e4\uc740 \ud604\uc874\ud55c \ub300\uc911\uc74c\uc545\uc758 \uad00\uc2b5\uc5d0 \uc800\ud56d\ud558\uba70 \uae09\uc131\uc7a5\ud558\ub294 \ubc18\ubb38\ud654\uc5d0 \ud638\uc18c\ud588\ub2e4. \ucd08\uae30 \ub9ac\ud2c0 \ub9ac\ucc98\ub4dc\uc758 \uacf5\uc5f0\uacfc \uc6b0\ub514 \uac70\uc2a4\ub9ac, \ub85c\ubc84\ud2b8 \uc874\uc2a8, \ud589\ud06c \uc70c\ub9ac\uc5c4\uc2a4\uc758 \uc791\uace1\uc5d0 \uc601\ud5a5\uc744 \ubc1b\uc740 \ube4c\ub7f0\uc740 \uc74c\uc545 \uc7a5\ub974\ub97c \uc99d\ud3ed\ud558\uace0 \uc790\uae30 \uac83\uc73c\ub85c \ud65c\uc6a9\ud560 \uc904 \uc54c\uc558\ub2e4. \uadf8\uc758 \uc74c\ubc18 \ud65c\ub3d9\uc740 50\ub144 \ub118\uac8c \uc9c0\uc18d\ub418\uc5c8\uc73c\uba70 \ud3ec\ud06c, \ube14\ub8e8\uc2a4, \ucee8\ud2b8\ub9ac\uc5d0\uc11c \uac00\uc2a4\ud3a0, \ub85c\ud070\ub864\uacfc \ub85c\ucee4\ube4c\ub9ac\uc5d0\uc11c \uc601\uad6d, \uc2a4\ucf54\ud2c0\ub79c\ub4dc, \uc544\uc77c\ub79c\ub4dc \ud3ec\ud06c \uc74c\uc545\uae4c\uc9c0\ub97c \ud3ec\uc12d\ud558\uba74\uc11c \uace0\uc804 \ubbf8\uad6d \ub178\ub798\ub97c \ud0d0\uad6c\ud588\uace0, \uc2ec\uc9c0\uc5b4 \uc7ac\uc988\uc640 \uadf8\ub808\uc774\ud2b8 \uc544\uba54\ub9ac\uce78 \uc1a1\ubd81\uae4c\uc9c0 \uc218\uc6a9\ud588\ub2e4. \uae30\ud0c0, \uac74\ubc18, \ud558\ubaa8\ub2c8\uce74\ub97c \uc0ac\uc6a9\ud55c \uacf5\uc5f0\uc744 \ud588\ub2e4. \ubc18\uc8fc\ub294 \uc2dc\uc2dc\uac01\uac01 \ubc14\ub00c\ub294 \ub77c\uc778\uc5c5\uc758 \uc74c\uc545\uac00\uac00 \uc81c\uacf5\ud558\uba70 1980\ub144\ub300 \ub9d0\ubd80\ud130 \uac74\uc2e4\ud788 \uc218\ud589\ud55c \uc21c\ud68c\uacf5\uc5f0\uc740 \ub124\ubc84 \uc5d4\ub529 \ud22c\uc5b4\ub85c \ubd88\ub9ac\uace0 \uc788\ub2e4. \ub179\uc74c \uc544\ud2f0\uc2a4\ud2b8 \ubc0f \uacf5\uc5f0\uc790\ub85c\uc11c \uadf8\uac00 \uc774\ub8e9\ud55c \uc5c5\uc801\uc740 \uadf8\uc758 \uacbd\ub825 \uc911\uc2ec\uc5d0 \uc704\uce58\ud574 \uc788\uc9c0\ub9cc, \uadf8\uc758 \uac00\uc7a5 \uc704\ub300\ud55c \uacf5\ud5cc\uc73c\ub85c \ud3c9\uac00\ub418\ub294 \uac83\uc740 \uc791\uace1\u00b7\uc791\uc0ac\ub2e4. 1994\ub144\ubd80\ud130 \uc2dc\uc791\ud574 \uc77c\uacf1 \uad8c\uc758 \ub4dc\ub85c\uc789 \ubc0f \uadf8\ub9bc \uc11c\uc801\uc744 \ubc1c\uac04\ud588\uace0 \uadf8\uc758 \uc791\uc5c5\ubb3c\uc740 \uc8fc\uc694 \uc608\uc220 \ud654\ub791\uc5d0 \uc804\uc2dc\ub418\uace0 \uc788\ub2e4.", "answer": "1994\ub144", "sentence": "1994\ub144 \ubd80\ud130 \uc2dc\uc791\ud574 \uc77c\uacf1 \uad8c\uc758 \ub4dc\ub85c\uc789 \ubc0f \uadf8\ub9bc \uc11c\uc801\uc744 \ubc1c\uac04\ud588\uace0 \uadf8\uc758 \uc791\uc5c5\ubb3c\uc740 \uc8fc\uc694 \uc608\uc220 \ud654\ub791\uc5d0 \uc804\uc2dc\ub418\uace0 \uc788\ub2e4.", "paragraph_sentence": "\ub51c\ub7f0\uc758 \uac00\uc0ac\ub294 \ub113\uc740 \ubc94\uc704\ub97c \uc544\uc6b0\ub974\uba70 \uc815\uce58, \uc0ac\ud68c, \ucca0\ud559, \ubb38\ud559\uc758 \uc601\ud5a5\uc744 \ubc1b\ub294\ub2e4. \uc774\ub4e4\uc740 \ud604\uc874\ud55c \ub300\uc911\uc74c\uc545\uc758 \uad00\uc2b5\uc5d0 \uc800\ud56d\ud558\uba70 \uae09\uc131\uc7a5\ud558\ub294 \ubc18\ubb38\ud654\uc5d0 \ud638\uc18c\ud588\ub2e4. \ucd08\uae30 \ub9ac\ud2c0 \ub9ac\ucc98\ub4dc\uc758 \uacf5\uc5f0\uacfc \uc6b0\ub514 \uac70\uc2a4\ub9ac, \ub85c\ubc84\ud2b8 \uc874\uc2a8, \ud589\ud06c \uc70c\ub9ac\uc5c4\uc2a4\uc758 \uc791\uace1\uc5d0 \uc601\ud5a5\uc744 \ubc1b\uc740 \ube4c\ub7f0\uc740 \uc74c\uc545 \uc7a5\ub974\ub97c \uc99d\ud3ed\ud558\uace0 \uc790\uae30 \uac83\uc73c\ub85c \ud65c\uc6a9\ud560 \uc904 \uc54c\uc558\ub2e4. \uadf8\uc758 \uc74c\ubc18 \ud65c\ub3d9\uc740 50\ub144 \ub118\uac8c \uc9c0\uc18d\ub418\uc5c8\uc73c\uba70 \ud3ec\ud06c, \ube14\ub8e8\uc2a4, \ucee8\ud2b8\ub9ac\uc5d0\uc11c \uac00\uc2a4\ud3a0, \ub85c\ud070\ub864\uacfc \ub85c\ucee4\ube4c\ub9ac\uc5d0\uc11c \uc601\uad6d, \uc2a4\ucf54\ud2c0\ub79c\ub4dc, \uc544\uc77c\ub79c\ub4dc \ud3ec\ud06c \uc74c\uc545\uae4c\uc9c0\ub97c \ud3ec\uc12d\ud558\uba74\uc11c \uace0\uc804 \ubbf8\uad6d \ub178\ub798\ub97c \ud0d0\uad6c\ud588\uace0, \uc2ec\uc9c0\uc5b4 \uc7ac\uc988\uc640 \uadf8\ub808\uc774\ud2b8 \uc544\uba54\ub9ac\uce78 \uc1a1\ubd81\uae4c\uc9c0 \uc218\uc6a9\ud588\ub2e4. \uae30\ud0c0, \uac74\ubc18, \ud558\ubaa8\ub2c8\uce74\ub97c \uc0ac\uc6a9\ud55c \uacf5\uc5f0\uc744 \ud588\ub2e4. \ubc18\uc8fc\ub294 \uc2dc\uc2dc\uac01\uac01 \ubc14\ub00c\ub294 \ub77c\uc778\uc5c5\uc758 \uc74c\uc545\uac00\uac00 \uc81c\uacf5\ud558\uba70 1980\ub144\ub300 \ub9d0\ubd80\ud130 \uac74\uc2e4\ud788 \uc218\ud589\ud55c \uc21c\ud68c\uacf5\uc5f0\uc740 \ub124\ubc84 \uc5d4\ub529 \ud22c\uc5b4\ub85c \ubd88\ub9ac\uace0 \uc788\ub2e4. \ub179\uc74c \uc544\ud2f0\uc2a4\ud2b8 \ubc0f \uacf5\uc5f0\uc790\ub85c\uc11c \uadf8\uac00 \uc774\ub8e9\ud55c \uc5c5\uc801\uc740 \uadf8\uc758 \uacbd\ub825 \uc911\uc2ec\uc5d0 \uc704\uce58\ud574 \uc788\uc9c0\ub9cc, \uadf8\uc758 \uac00\uc7a5 \uc704\ub300\ud55c \uacf5\ud5cc\uc73c\ub85c \ud3c9\uac00\ub418\ub294 \uac83\uc740 \uc791\uace1\u00b7\uc791\uc0ac\ub2e4. 1994\ub144 \ubd80\ud130 \uc2dc\uc791\ud574 \uc77c\uacf1 \uad8c\uc758 \ub4dc\ub85c\uc789 \ubc0f \uadf8\ub9bc \uc11c\uc801\uc744 \ubc1c\uac04\ud588\uace0 \uadf8\uc758 \uc791\uc5c5\ubb3c\uc740 \uc8fc\uc694 \uc608\uc220 \ud654\ub791\uc5d0 \uc804\uc2dc\ub418\uace0 \uc788\ub2e4. ", "paragraph_answer": "\ub51c\ub7f0\uc758 \uac00\uc0ac\ub294 \ub113\uc740 \ubc94\uc704\ub97c \uc544\uc6b0\ub974\uba70 \uc815\uce58, \uc0ac\ud68c, \ucca0\ud559, \ubb38\ud559\uc758 \uc601\ud5a5\uc744 \ubc1b\ub294\ub2e4. \uc774\ub4e4\uc740 \ud604\uc874\ud55c \ub300\uc911\uc74c\uc545\uc758 \uad00\uc2b5\uc5d0 \uc800\ud56d\ud558\uba70 \uae09\uc131\uc7a5\ud558\ub294 \ubc18\ubb38\ud654\uc5d0 \ud638\uc18c\ud588\ub2e4. \ucd08\uae30 \ub9ac\ud2c0 \ub9ac\ucc98\ub4dc\uc758 \uacf5\uc5f0\uacfc \uc6b0\ub514 \uac70\uc2a4\ub9ac, \ub85c\ubc84\ud2b8 \uc874\uc2a8, \ud589\ud06c \uc70c\ub9ac\uc5c4\uc2a4\uc758 \uc791\uace1\uc5d0 \uc601\ud5a5\uc744 \ubc1b\uc740 \ube4c\ub7f0\uc740 \uc74c\uc545 \uc7a5\ub974\ub97c \uc99d\ud3ed\ud558\uace0 \uc790\uae30 \uac83\uc73c\ub85c \ud65c\uc6a9\ud560 \uc904 \uc54c\uc558\ub2e4. \uadf8\uc758 \uc74c\ubc18 \ud65c\ub3d9\uc740 50\ub144 \ub118\uac8c \uc9c0\uc18d\ub418\uc5c8\uc73c\uba70 \ud3ec\ud06c, \ube14\ub8e8\uc2a4, \ucee8\ud2b8\ub9ac\uc5d0\uc11c \uac00\uc2a4\ud3a0, \ub85c\ud070\ub864\uacfc \ub85c\ucee4\ube4c\ub9ac\uc5d0\uc11c \uc601\uad6d, \uc2a4\ucf54\ud2c0\ub79c\ub4dc, \uc544\uc77c\ub79c\ub4dc \ud3ec\ud06c \uc74c\uc545\uae4c\uc9c0\ub97c \ud3ec\uc12d\ud558\uba74\uc11c \uace0\uc804 \ubbf8\uad6d \ub178\ub798\ub97c \ud0d0\uad6c\ud588\uace0, \uc2ec\uc9c0\uc5b4 \uc7ac\uc988\uc640 \uadf8\ub808\uc774\ud2b8 \uc544\uba54\ub9ac\uce78 \uc1a1\ubd81\uae4c\uc9c0 \uc218\uc6a9\ud588\ub2e4. \uae30\ud0c0, \uac74\ubc18, \ud558\ubaa8\ub2c8\uce74\ub97c \uc0ac\uc6a9\ud55c \uacf5\uc5f0\uc744 \ud588\ub2e4. \ubc18\uc8fc\ub294 \uc2dc\uc2dc\uac01\uac01 \ubc14\ub00c\ub294 \ub77c\uc778\uc5c5\uc758 \uc74c\uc545\uac00\uac00 \uc81c\uacf5\ud558\uba70 1980\ub144\ub300 \ub9d0\ubd80\ud130 \uac74\uc2e4\ud788 \uc218\ud589\ud55c \uc21c\ud68c\uacf5\uc5f0\uc740 \ub124\ubc84 \uc5d4\ub529 \ud22c\uc5b4\ub85c \ubd88\ub9ac\uace0 \uc788\ub2e4. \ub179\uc74c \uc544\ud2f0\uc2a4\ud2b8 \ubc0f \uacf5\uc5f0\uc790\ub85c\uc11c \uadf8\uac00 \uc774\ub8e9\ud55c \uc5c5\uc801\uc740 \uadf8\uc758 \uacbd\ub825 \uc911\uc2ec\uc5d0 \uc704\uce58\ud574 \uc788\uc9c0\ub9cc, \uadf8\uc758 \uac00\uc7a5 \uc704\ub300\ud55c \uacf5\ud5cc\uc73c\ub85c \ud3c9\uac00\ub418\ub294 \uac83\uc740 \uc791\uace1\u00b7\uc791\uc0ac\ub2e4. 1994\ub144 \ubd80\ud130 \uc2dc\uc791\ud574 \uc77c\uacf1 \uad8c\uc758 \ub4dc\ub85c\uc789 \ubc0f \uadf8\ub9bc \uc11c\uc801\uc744 \ubc1c\uac04\ud588\uace0 \uadf8\uc758 \uc791\uc5c5\ubb3c\uc740 \uc8fc\uc694 \uc608\uc220 \ud654\ub791\uc5d0 \uc804\uc2dc\ub418\uace0 \uc788\ub2e4.", "sentence_answer": " 1994\ub144 \ubd80\ud130 \uc2dc\uc791\ud574 \uc77c\uacf1 \uad8c\uc758 \ub4dc\ub85c\uc789 \ubc0f \uadf8\ub9bc \uc11c\uc801\uc744 \ubc1c\uac04\ud588\uace0 \uadf8\uc758 \uc791\uc5c5\ubb3c\uc740 \uc8fc\uc694 \uc608\uc220 \ud654\ub791\uc5d0 \uc804\uc2dc\ub418\uace0 \uc788\ub2e4.", "paragraph_id": "6507863-1-1", "paragraph_question": "question: \ubc25 \ub51c\ub7f0\uc774 \ub4dc\ub85c\uc789 \ubc0f \uadf8\ub9bc \uc11c\uc801\uc744 \ubc1c\uac04\ud558\uae30 \uc2dc\uc791\ud55c \ud574\ub294?, context: \ub51c\ub7f0\uc758 \uac00\uc0ac\ub294 \ub113\uc740 \ubc94\uc704\ub97c \uc544\uc6b0\ub974\uba70 \uc815\uce58, \uc0ac\ud68c, \ucca0\ud559, \ubb38\ud559\uc758 \uc601\ud5a5\uc744 \ubc1b\ub294\ub2e4. \uc774\ub4e4\uc740 \ud604\uc874\ud55c \ub300\uc911\uc74c\uc545\uc758 \uad00\uc2b5\uc5d0 \uc800\ud56d\ud558\uba70 \uae09\uc131\uc7a5\ud558\ub294 \ubc18\ubb38\ud654\uc5d0 \ud638\uc18c\ud588\ub2e4. \ucd08\uae30 \ub9ac\ud2c0 \ub9ac\ucc98\ub4dc\uc758 \uacf5\uc5f0\uacfc \uc6b0\ub514 \uac70\uc2a4\ub9ac, \ub85c\ubc84\ud2b8 \uc874\uc2a8, \ud589\ud06c \uc70c\ub9ac\uc5c4\uc2a4\uc758 \uc791\uace1\uc5d0 \uc601\ud5a5\uc744 \ubc1b\uc740 \ube4c\ub7f0\uc740 \uc74c\uc545 \uc7a5\ub974\ub97c \uc99d\ud3ed\ud558\uace0 \uc790\uae30 \uac83\uc73c\ub85c \ud65c\uc6a9\ud560 \uc904 \uc54c\uc558\ub2e4. \uadf8\uc758 \uc74c\ubc18 \ud65c\ub3d9\uc740 50\ub144 \ub118\uac8c \uc9c0\uc18d\ub418\uc5c8\uc73c\uba70 \ud3ec\ud06c, \ube14\ub8e8\uc2a4, \ucee8\ud2b8\ub9ac\uc5d0\uc11c \uac00\uc2a4\ud3a0, \ub85c\ud070\ub864\uacfc \ub85c\ucee4\ube4c\ub9ac\uc5d0\uc11c \uc601\uad6d, \uc2a4\ucf54\ud2c0\ub79c\ub4dc, \uc544\uc77c\ub79c\ub4dc \ud3ec\ud06c \uc74c\uc545\uae4c\uc9c0\ub97c \ud3ec\uc12d\ud558\uba74\uc11c \uace0\uc804 \ubbf8\uad6d \ub178\ub798\ub97c \ud0d0\uad6c\ud588\uace0, \uc2ec\uc9c0\uc5b4 \uc7ac\uc988\uc640 \uadf8\ub808\uc774\ud2b8 \uc544\uba54\ub9ac\uce78 \uc1a1\ubd81\uae4c\uc9c0 \uc218\uc6a9\ud588\ub2e4. \uae30\ud0c0, \uac74\ubc18, \ud558\ubaa8\ub2c8\uce74\ub97c \uc0ac\uc6a9\ud55c \uacf5\uc5f0\uc744 \ud588\ub2e4. \ubc18\uc8fc\ub294 \uc2dc\uc2dc\uac01\uac01 \ubc14\ub00c\ub294 \ub77c\uc778\uc5c5\uc758 \uc74c\uc545\uac00\uac00 \uc81c\uacf5\ud558\uba70 1980\ub144\ub300 \ub9d0\ubd80\ud130 \uac74\uc2e4\ud788 \uc218\ud589\ud55c \uc21c\ud68c\uacf5\uc5f0\uc740 \ub124\ubc84 \uc5d4\ub529 \ud22c\uc5b4\ub85c \ubd88\ub9ac\uace0 \uc788\ub2e4. \ub179\uc74c \uc544\ud2f0\uc2a4\ud2b8 \ubc0f \uacf5\uc5f0\uc790\ub85c\uc11c \uadf8\uac00 \uc774\ub8e9\ud55c \uc5c5\uc801\uc740 \uadf8\uc758 \uacbd\ub825 \uc911\uc2ec\uc5d0 \uc704\uce58\ud574 \uc788\uc9c0\ub9cc, \uadf8\uc758 \uac00\uc7a5 \uc704\ub300\ud55c \uacf5\ud5cc\uc73c\ub85c \ud3c9\uac00\ub418\ub294 \uac83\uc740 \uc791\uace1\u00b7\uc791\uc0ac\ub2e4. 1994\ub144\ubd80\ud130 \uc2dc\uc791\ud574 \uc77c\uacf1 \uad8c\uc758 \ub4dc\ub85c\uc789 \ubc0f \uadf8\ub9bc \uc11c\uc801\uc744 \ubc1c\uac04\ud588\uace0 \uadf8\uc758 \uc791\uc5c5\ubb3c\uc740 \uc8fc\uc694 \uc608\uc220 \ud654\ub791\uc5d0 \uc804\uc2dc\ub418\uace0 \uc788\ub2e4."} {"question": "\ubc29\uc1a1\ud615\uc790\ub3d9\uc885\uc18d\uac10\uc2dc \uc2e0\ud638\uac00 \ud3ec\ud568\ud558\uace0 \uc788\ub294 \uc815\ubcf4 \uc911 \ub450\uac00\uc9c0\ub97c \ub9d0\ud558\uc2dc\uc624.", "paragraph": "\uc801\ud569\ud55c \ud2b8\ub79c\uc2a4\ud3f0\ub354\ub97c \ud0d1\uc7ac\ud55c \ud56d\uacf5\uae30\uc5d0\uc11c\ub294 \uc2dd\ubcc4\uc790, \ud604\uc7ac \uc704\uce58, \uace0\ub3c4, \ub300\uae30\uc18d\ub3c4\uc640 \uac19\uc740 \uc815\ubcf4\ub97c \ud3ec\ud568\ud55c \ubc29\uc1a1\ud615\uc790\ub3d9\uc885\uc18d\uac10\uc2dc(Automatic Dependent Surveillance-Broadcast : ADS-B)\uc2e0\ud638\uac00 \uc1a1\uc2e0\ub41c\ub2e4. \uc774 \uc2e0\ud638\ub294 TCAS\uc758 \uc751\ub2f5\uacfc \uac19\uc774 1090MHz \uc8fc\ud30c\uc218\ub85c \uc1a1\uc2e0\ub418\uc5b4 ADS-B \uba54\uc2dc\uc9c0\ub97c \ucc98\ub9ac\ud560 \uc218 \uc788\ub2e4. ADS-B \uba54\uc2dc\uc9c0\ub97c \ucc98\ub9ac\ud560 \uc218 \uc788\ub294 TCAS \uc7a5\uce58\ub294 \ud1b5\uc0c1 TCAS \uba54\uc2dc\uc9c0\uc640 \ud568\uaed8 ADS-B \uba54\uc2dc\uc9c0\ub97c \uc0ac\uc6a9\ud558\uace0, \uc608\uce21 \ub2a5\ub825\uacfc \uc0c1\ud669 \ud45c\uc2dc\uc758 \uac15\ud654\uac00 \uac00\ub2a5\ud574\uc9c4\ub2e4. \uc774 \ubc29\ubc95\uc744 \u300c\ud558\uc774\ube0c\ub9ac\ub4dc \uac10\uc2dc\u300d\ub77c\uace0 \ubd88\ub9ac\uace0 \uc788\ub2e4. \ub2a5\ub3d9\ud615 TCAS\uac00 \uac10\uc2dc\ud560 \uc218 \uc788\ub294 \ubc94\uc704\ub294 40 \ud574\ub9ac\uc778 \uc810\uc744 \ube44\uad50\ud574 \ubcf4\uba74 ADS-B\ub294 \uc57d 100 \ud574\ub9ac \uc774\uc0c1\uc758 \uc6d0\uac70\ub9ac\ub85c\ubd80\ud130 \uc218\ub3d9\uc801\uc73c\ub85c \uc218\uc2e0\ud560 \uc218 \uc788\ub2e4\uace0 \ud558\ub294 \uac83\ubfd0\ub9cc\uc774 \uc544\ub2c8\ub77c, ADS-B \uba54\uc2dc\uc9c0\uc5d0\ub294 \ucd94\uac00 \uc815\ubcf4(\ub300\uae30\uc18d\ub3c4\ub4f1)\uac00 \ud3ec\ud568\ub418\uc5b4 \uc788\ub294 \uac83\uc73c\ub85c \uc608\uce21 \ub2a5\ub825\uc774 \ud5a5\uc0c1\ub41c\ub2e4. ADS-B \uba54\uc2dc\uc9c0 \uc548\uc5d0 \uc788\ub294 \uc2dd\ubcc4 \uc815\ubcf4\ub294 \uc870\uc815\uc11d\uc5d0 \uc788\ub294 \uc804\uc2dc\uae30\uc5d0\uc11c \ub2e4\ub978 \ud56d\uacf5\uae30\uc5d0 \uc2dd\ubcc4\uc790\ub97c \ubd99\uc774\uae30 \uc704\ud574\uc11c \uc0ac\uc6a9\ud558\uac70\ub098 \uc0c1\ud669\uc778\uc2dd\uc744 \uac1c\uc120\ud560 \uc218 \uc788\ub2e4. \ud558\uc774\ube0c\ub9ac\ub4dc \uac10\uc2dc\ub97c \uc0ac\uc6a9\ud588\uc744 \uacbd\uc6b0\uc5d0\uc11c\ub3c4 TCAS \uc758 \uae30\ubcf8\uc801\uc778 \ucda9\ub3cc\ubc29\uc9c0 \uae30\ub2a5\uc5d0\ub294 \ubcc0\ud654\ub294 \uc5c6\ub2e4.", "answer": "\uc2dd\ubcc4\uc790, \ud604\uc7ac \uc704\uce58", "sentence": "\uc801\ud569\ud55c \ud2b8\ub79c\uc2a4\ud3f0\ub354\ub97c \ud0d1\uc7ac\ud55c \ud56d\uacf5\uae30\uc5d0\uc11c\ub294 \uc2dd\ubcc4\uc790, \ud604\uc7ac \uc704\uce58 ,", "paragraph_sentence": " \uc801\ud569\ud55c \ud2b8\ub79c\uc2a4\ud3f0\ub354\ub97c \ud0d1\uc7ac\ud55c \ud56d\uacf5\uae30\uc5d0\uc11c\ub294 \uc2dd\ubcc4\uc790, \ud604\uc7ac \uc704\uce58 , \uace0\ub3c4, \ub300\uae30\uc18d\ub3c4\uc640 \uac19\uc740 \uc815\ubcf4\ub97c \ud3ec\ud568\ud55c \ubc29\uc1a1\ud615\uc790\ub3d9\uc885\uc18d\uac10\uc2dc(Automatic Dependent Surveillance-Broadcast : ADS-B)\uc2e0\ud638\uac00 \uc1a1\uc2e0\ub41c\ub2e4. \uc774 \uc2e0\ud638\ub294 TCAS\uc758 \uc751\ub2f5\uacfc \uac19\uc774 1090MHz \uc8fc\ud30c\uc218\ub85c \uc1a1\uc2e0\ub418\uc5b4 ADS-B \uba54\uc2dc\uc9c0\ub97c \ucc98\ub9ac\ud560 \uc218 \uc788\ub2e4. ADS-B \uba54\uc2dc\uc9c0\ub97c \ucc98\ub9ac\ud560 \uc218 \uc788\ub294 TCAS \uc7a5\uce58\ub294 \ud1b5\uc0c1 TCAS \uba54\uc2dc\uc9c0\uc640 \ud568\uaed8 ADS-B \uba54\uc2dc\uc9c0\ub97c \uc0ac\uc6a9\ud558\uace0, \uc608\uce21 \ub2a5\ub825\uacfc \uc0c1\ud669 \ud45c\uc2dc\uc758 \uac15\ud654\uac00 \uac00\ub2a5\ud574\uc9c4\ub2e4. \uc774 \ubc29\ubc95\uc744 \u300c\ud558\uc774\ube0c\ub9ac\ub4dc \uac10\uc2dc\u300d\ub77c\uace0 \ubd88\ub9ac\uace0 \uc788\ub2e4. \ub2a5\ub3d9\ud615 TCAS\uac00 \uac10\uc2dc\ud560 \uc218 \uc788\ub294 \ubc94\uc704\ub294 40 \ud574\ub9ac\uc778 \uc810\uc744 \ube44\uad50\ud574 \ubcf4\uba74 ADS-B\ub294 \uc57d 100 \ud574\ub9ac \uc774\uc0c1\uc758 \uc6d0\uac70\ub9ac\ub85c\ubd80\ud130 \uc218\ub3d9\uc801\uc73c\ub85c \uc218\uc2e0\ud560 \uc218 \uc788\ub2e4\uace0 \ud558\ub294 \uac83\ubfd0\ub9cc\uc774 \uc544\ub2c8\ub77c, ADS-B \uba54\uc2dc\uc9c0\uc5d0\ub294 \ucd94\uac00 \uc815\ubcf4(\ub300\uae30\uc18d\ub3c4\ub4f1)\uac00 \ud3ec\ud568\ub418\uc5b4 \uc788\ub294 \uac83\uc73c\ub85c \uc608\uce21 \ub2a5\ub825\uc774 \ud5a5\uc0c1\ub41c\ub2e4. ADS-B \uba54\uc2dc\uc9c0 \uc548\uc5d0 \uc788\ub294 \uc2dd\ubcc4 \uc815\ubcf4\ub294 \uc870\uc815\uc11d\uc5d0 \uc788\ub294 \uc804\uc2dc\uae30\uc5d0\uc11c \ub2e4\ub978 \ud56d\uacf5\uae30\uc5d0 \uc2dd\ubcc4\uc790\ub97c \ubd99\uc774\uae30 \uc704\ud574\uc11c \uc0ac\uc6a9\ud558\uac70\ub098 \uc0c1\ud669\uc778\uc2dd\uc744 \uac1c\uc120\ud560 \uc218 \uc788\ub2e4. \ud558\uc774\ube0c\ub9ac\ub4dc \uac10\uc2dc\ub97c \uc0ac\uc6a9\ud588\uc744 \uacbd\uc6b0\uc5d0\uc11c\ub3c4 TCAS \uc758 \uae30\ubcf8\uc801\uc778 \ucda9\ub3cc\ubc29\uc9c0 \uae30\ub2a5\uc5d0\ub294 \ubcc0\ud654\ub294 \uc5c6\ub2e4.", "paragraph_answer": "\uc801\ud569\ud55c \ud2b8\ub79c\uc2a4\ud3f0\ub354\ub97c \ud0d1\uc7ac\ud55c \ud56d\uacf5\uae30\uc5d0\uc11c\ub294 \uc2dd\ubcc4\uc790, \ud604\uc7ac \uc704\uce58 , \uace0\ub3c4, \ub300\uae30\uc18d\ub3c4\uc640 \uac19\uc740 \uc815\ubcf4\ub97c \ud3ec\ud568\ud55c \ubc29\uc1a1\ud615\uc790\ub3d9\uc885\uc18d\uac10\uc2dc(Automatic Dependent Surveillance-Broadcast : ADS-B)\uc2e0\ud638\uac00 \uc1a1\uc2e0\ub41c\ub2e4. \uc774 \uc2e0\ud638\ub294 TCAS\uc758 \uc751\ub2f5\uacfc \uac19\uc774 1090MHz \uc8fc\ud30c\uc218\ub85c \uc1a1\uc2e0\ub418\uc5b4 ADS-B \uba54\uc2dc\uc9c0\ub97c \ucc98\ub9ac\ud560 \uc218 \uc788\ub2e4. ADS-B \uba54\uc2dc\uc9c0\ub97c \ucc98\ub9ac\ud560 \uc218 \uc788\ub294 TCAS \uc7a5\uce58\ub294 \ud1b5\uc0c1 TCAS \uba54\uc2dc\uc9c0\uc640 \ud568\uaed8 ADS-B \uba54\uc2dc\uc9c0\ub97c \uc0ac\uc6a9\ud558\uace0, \uc608\uce21 \ub2a5\ub825\uacfc \uc0c1\ud669 \ud45c\uc2dc\uc758 \uac15\ud654\uac00 \uac00\ub2a5\ud574\uc9c4\ub2e4. \uc774 \ubc29\ubc95\uc744 \u300c\ud558\uc774\ube0c\ub9ac\ub4dc \uac10\uc2dc\u300d\ub77c\uace0 \ubd88\ub9ac\uace0 \uc788\ub2e4. \ub2a5\ub3d9\ud615 TCAS\uac00 \uac10\uc2dc\ud560 \uc218 \uc788\ub294 \ubc94\uc704\ub294 40 \ud574\ub9ac\uc778 \uc810\uc744 \ube44\uad50\ud574 \ubcf4\uba74 ADS-B\ub294 \uc57d 100 \ud574\ub9ac \uc774\uc0c1\uc758 \uc6d0\uac70\ub9ac\ub85c\ubd80\ud130 \uc218\ub3d9\uc801\uc73c\ub85c \uc218\uc2e0\ud560 \uc218 \uc788\ub2e4\uace0 \ud558\ub294 \uac83\ubfd0\ub9cc\uc774 \uc544\ub2c8\ub77c, ADS-B \uba54\uc2dc\uc9c0\uc5d0\ub294 \ucd94\uac00 \uc815\ubcf4(\ub300\uae30\uc18d\ub3c4\ub4f1)\uac00 \ud3ec\ud568\ub418\uc5b4 \uc788\ub294 \uac83\uc73c\ub85c \uc608\uce21 \ub2a5\ub825\uc774 \ud5a5\uc0c1\ub41c\ub2e4. ADS-B \uba54\uc2dc\uc9c0 \uc548\uc5d0 \uc788\ub294 \uc2dd\ubcc4 \uc815\ubcf4\ub294 \uc870\uc815\uc11d\uc5d0 \uc788\ub294 \uc804\uc2dc\uae30\uc5d0\uc11c \ub2e4\ub978 \ud56d\uacf5\uae30\uc5d0 \uc2dd\ubcc4\uc790\ub97c \ubd99\uc774\uae30 \uc704\ud574\uc11c \uc0ac\uc6a9\ud558\uac70\ub098 \uc0c1\ud669\uc778\uc2dd\uc744 \uac1c\uc120\ud560 \uc218 \uc788\ub2e4. \ud558\uc774\ube0c\ub9ac\ub4dc \uac10\uc2dc\ub97c \uc0ac\uc6a9\ud588\uc744 \uacbd\uc6b0\uc5d0\uc11c\ub3c4 TCAS \uc758 \uae30\ubcf8\uc801\uc778 \ucda9\ub3cc\ubc29\uc9c0 \uae30\ub2a5\uc5d0\ub294 \ubcc0\ud654\ub294 \uc5c6\ub2e4.", "sentence_answer": "\uc801\ud569\ud55c \ud2b8\ub79c\uc2a4\ud3f0\ub354\ub97c \ud0d1\uc7ac\ud55c \ud56d\uacf5\uae30\uc5d0\uc11c\ub294 \uc2dd\ubcc4\uc790, \ud604\uc7ac \uc704\uce58 ,", "paragraph_id": "5951603-1-1", "paragraph_question": "question: \ubc29\uc1a1\ud615\uc790\ub3d9\uc885\uc18d\uac10\uc2dc \uc2e0\ud638\uac00 \ud3ec\ud568\ud558\uace0 \uc788\ub294 \uc815\ubcf4 \uc911 \ub450\uac00\uc9c0\ub97c \ub9d0\ud558\uc2dc\uc624., context: \uc801\ud569\ud55c \ud2b8\ub79c\uc2a4\ud3f0\ub354\ub97c \ud0d1\uc7ac\ud55c \ud56d\uacf5\uae30\uc5d0\uc11c\ub294 \uc2dd\ubcc4\uc790, \ud604\uc7ac \uc704\uce58, \uace0\ub3c4, \ub300\uae30\uc18d\ub3c4\uc640 \uac19\uc740 \uc815\ubcf4\ub97c \ud3ec\ud568\ud55c \ubc29\uc1a1\ud615\uc790\ub3d9\uc885\uc18d\uac10\uc2dc(Automatic Dependent Surveillance-Broadcast : ADS-B)\uc2e0\ud638\uac00 \uc1a1\uc2e0\ub41c\ub2e4. \uc774 \uc2e0\ud638\ub294 TCAS\uc758 \uc751\ub2f5\uacfc \uac19\uc774 1090MHz \uc8fc\ud30c\uc218\ub85c \uc1a1\uc2e0\ub418\uc5b4 ADS-B \uba54\uc2dc\uc9c0\ub97c \ucc98\ub9ac\ud560 \uc218 \uc788\ub2e4. ADS-B \uba54\uc2dc\uc9c0\ub97c \ucc98\ub9ac\ud560 \uc218 \uc788\ub294 TCAS \uc7a5\uce58\ub294 \ud1b5\uc0c1 TCAS \uba54\uc2dc\uc9c0\uc640 \ud568\uaed8 ADS-B \uba54\uc2dc\uc9c0\ub97c \uc0ac\uc6a9\ud558\uace0, \uc608\uce21 \ub2a5\ub825\uacfc \uc0c1\ud669 \ud45c\uc2dc\uc758 \uac15\ud654\uac00 \uac00\ub2a5\ud574\uc9c4\ub2e4. \uc774 \ubc29\ubc95\uc744 \u300c\ud558\uc774\ube0c\ub9ac\ub4dc \uac10\uc2dc\u300d\ub77c\uace0 \ubd88\ub9ac\uace0 \uc788\ub2e4. \ub2a5\ub3d9\ud615 TCAS\uac00 \uac10\uc2dc\ud560 \uc218 \uc788\ub294 \ubc94\uc704\ub294 40 \ud574\ub9ac\uc778 \uc810\uc744 \ube44\uad50\ud574 \ubcf4\uba74 ADS-B\ub294 \uc57d 100 \ud574\ub9ac \uc774\uc0c1\uc758 \uc6d0\uac70\ub9ac\ub85c\ubd80\ud130 \uc218\ub3d9\uc801\uc73c\ub85c \uc218\uc2e0\ud560 \uc218 \uc788\ub2e4\uace0 \ud558\ub294 \uac83\ubfd0\ub9cc\uc774 \uc544\ub2c8\ub77c, ADS-B \uba54\uc2dc\uc9c0\uc5d0\ub294 \ucd94\uac00 \uc815\ubcf4(\ub300\uae30\uc18d\ub3c4\ub4f1)\uac00 \ud3ec\ud568\ub418\uc5b4 \uc788\ub294 \uac83\uc73c\ub85c \uc608\uce21 \ub2a5\ub825\uc774 \ud5a5\uc0c1\ub41c\ub2e4. ADS-B \uba54\uc2dc\uc9c0 \uc548\uc5d0 \uc788\ub294 \uc2dd\ubcc4 \uc815\ubcf4\ub294 \uc870\uc815\uc11d\uc5d0 \uc788\ub294 \uc804\uc2dc\uae30\uc5d0\uc11c \ub2e4\ub978 \ud56d\uacf5\uae30\uc5d0 \uc2dd\ubcc4\uc790\ub97c \ubd99\uc774\uae30 \uc704\ud574\uc11c \uc0ac\uc6a9\ud558\uac70\ub098 \uc0c1\ud669\uc778\uc2dd\uc744 \uac1c\uc120\ud560 \uc218 \uc788\ub2e4. \ud558\uc774\ube0c\ub9ac\ub4dc \uac10\uc2dc\ub97c \uc0ac\uc6a9\ud588\uc744 \uacbd\uc6b0\uc5d0\uc11c\ub3c4 TCAS \uc758 \uae30\ubcf8\uc801\uc778 \ucda9\ub3cc\ubc29\uc9c0 \uae30\ub2a5\uc5d0\ub294 \ubcc0\ud654\ub294 \uc5c6\ub2e4."} {"question": "\ubbf8\uad6d\uc758 \uc218\ub3c4\uc774\uc790 \ub300\ud1b5\ub839\uc774 \uc0ac\ub294 \ubc31\uc545\uad00\uc774 \uc704\uce58\ud55c \uc9c0\uc5ed\uc740?", "paragraph": "\ub300\ud1b5\ub839 \ud589\uc815\uc2e4\uacfc \uc77c\ubc18 \ud589\uc815 \ubd80\uc11c, \ub300\ud1b5\ub839 \uc9c1\uc18d\ub3c5\ub9bd\uae30\uad00\uc73c\ub85c \uad6c\uc131\ub418\uc5b4 \uc788\ub2e4. \ub300\ud1b5\ub839\uc774 \ud589\uc815\ubd80\uc758 \uc218\ubc18\uc774\uc790 \ub098\ub77c\ub97c \ub300\ud45c\ud558\ub294 \uad6d\uac00\uc6d0\uc218\uc774\ub2e4. \ub300\ud1b5\ub839\uc740 \uc218\ub3c4 \uc6cc\uc2f1\ud134\uc5d0 \uc788\ub294 \ubc31\uc545\uad00\uc5d0 \uc0b4\uba70 \uc774 \uacf3\uc5d0\uc11c \ub300\ud1b5\ub839 \uc5c5\ubb34\ub97c \ubcf8\ub2e4. \ub300\ud1b5\ub839\uc740 \ud589\uc815\ubd80\uc758 \uc218\ubc18\uc73c\ub85c\uc11c \uc5f0\ubc29\ubc95\uc744 \uc9d1\ud589\ud560 \uad8c\ud55c\uc774 \uc788\uc73c\uba70 \uace0\uc704 \uc5f0\ubc29 \uacf5\ubb34\uc6d0\uc5d0 \ub300\ud55c \uc784\uba85\uad8c\uacfc \ud574\uc784\uad8c\uc744 \uac00\uc9c4\ub2e4. \ub610\ud55c \uad70\ud1b5\uc218\uad8c\uacfc \uc678\uad50\uad8c\uc744 \uac00\uc9c0\uba70 \uc758\ud68c\uc5d0 \ubc95\uc548\uc744 \uc0c1\uc815\ud560 \uc218 \uc5c6\uace0 \uad6d\uc81c\uae30\uad6c\uc640 \uc7ac\uc678 \uacf5\uad00\uc5d0 \ubbf8\uad6d \ub300\ud45c\ub97c \uc784\uba85\ud55c\ub2e4. \ub300\ud1b5\ub839\uc740 \uad6d\ubbfc\uc774 \uc120\ucd9c\ud558\uba70 \uc784\uae30\ub294 4\ub144\uc774\uace0 \ub450 \ubc88 \uc774\uc0c1 \uc5f0\uc784\ud560 \uc218 \uc5c6\ub2e4. \ub300\ud1b5\ub839 \ubc11\uc5d0\ub294 \ub300\ud1b5\ub839\uc758 \ud589\uc815\uc5c5\ubb34\ub97c \ubcf4\ubd10\ud574\uc904 12\uac1c\uc758 \ud589\uc815\uc2e4\uc774 \uc788\ub2e4. \uc77c\ubc18 \ud589\uc815 \ubd80\uc11c\ub294 \uc911\uc559\uc815\ubd80\uc758 \ud589\uc815\uc744 \ub9e1\ub294 15\uac1c \ubd80\uc11c\ub85c \uad6c\uc131\ub418\uba70, \uac01 \ubd80\uc758 \uc7a5\uad00\uc744 \ub300\ud1b5\ub839\uc774 \uc0c1\uc6d0\uc758 \uc2b9\uc778\uc744 \uc5bb\uc5b4 \uc784\uba85\ud55c\ub2e4. 15\ubd80 \uc7a5\uad00\uc740 \ub0b4\uac01\uc758 \uad6c\uc131\uc6d0\uc774 \ub41c\ub2e4. \uc5f0\ubc29\uc815\ubd80\ub294 15\uac1c\uc758 \ubd80\uc640 \ub2e4\ub978 \ub3c5\ub9bd \uc815\ubd80\uae30\uad00\uc73c\ub85c \uad6c\uc131\ub418\uc5b4 \uc788\ub2e4. \uac01 \uc131\uc758 \uc7a5\uad00\uc740 \ub2e8\uc9c0 \ub300\ud1b5\ub839\uc744 \ubcf4\uc88c\ud558\ub294 \uc785\uc7a5\uc5d0 \uc788\uc744 \ubfd0, \uc758\ud68c\uc5d0 \uc758\uc11d\ub3c4 \uac00\uc9c0\uc9c0 \uc54a\uc73c\uba70, \uc9c0\uc704\ub85c\uc11c\ub294 \ub2e4\ub978 \ub098\ub77c\uc758 \uac01\ub8cc\ubcf4\ub2e4\ub3c4 \ub0ae\ub2e4\uace0 \ud558\uaca0\ub2e4. \ub300\ud1b5\ub839\uc744 \uc9c1\uc811 \ubcf4\uc88c\ud558\ub294 \uae30\uad00\uc73c\ub85c\uc11c\ub294 \ub300\ud1b5\ub839\ubd80\u00b7\uad6d\uac00\uc548\uc804\ubcf4\uc7a5\ud68c\uc758\u00b7\uacbd\uc81c\uc790\ubb38\uc704\uc6d0\ud68c \uc678\uc5d0 \uc5f0\ubc29\uc608\uc0b0\uad6d\u00b7\uc911\uc559\uc815\ubcf4\uad6d\u00b7\uae34\uae09\uacc4\ud68d\uad6d \ub4f1\uc774 \uc788\ub2e4.", "answer": "\uc6cc\uc2f1\ud134", "sentence": "\ub300\ud1b5\ub839\uc740 \uc218\ub3c4 \uc6cc\uc2f1\ud134 \uc5d0 \uc788\ub294 \ubc31\uc545\uad00\uc5d0 \uc0b4\uba70 \uc774 \uacf3\uc5d0\uc11c \ub300\ud1b5\ub839 \uc5c5\ubb34\ub97c \ubcf8\ub2e4.", "paragraph_sentence": "\ub300\ud1b5\ub839 \ud589\uc815\uc2e4\uacfc \uc77c\ubc18 \ud589\uc815 \ubd80\uc11c, \ub300\ud1b5\ub839 \uc9c1\uc18d\ub3c5\ub9bd\uae30\uad00\uc73c\ub85c \uad6c\uc131\ub418\uc5b4 \uc788\ub2e4. \ub300\ud1b5\ub839\uc774 \ud589\uc815\ubd80\uc758 \uc218\ubc18\uc774\uc790 \ub098\ub77c\ub97c \ub300\ud45c\ud558\ub294 \uad6d\uac00\uc6d0\uc218\uc774\ub2e4. \ub300\ud1b5\ub839\uc740 \uc218\ub3c4 \uc6cc\uc2f1\ud134 \uc5d0 \uc788\ub294 \ubc31\uc545\uad00\uc5d0 \uc0b4\uba70 \uc774 \uacf3\uc5d0\uc11c \ub300\ud1b5\ub839 \uc5c5\ubb34\ub97c \ubcf8\ub2e4. \ub300\ud1b5\ub839\uc740 \ud589\uc815\ubd80\uc758 \uc218\ubc18\uc73c\ub85c\uc11c \uc5f0\ubc29\ubc95\uc744 \uc9d1\ud589\ud560 \uad8c\ud55c\uc774 \uc788\uc73c\uba70 \uace0\uc704 \uc5f0\ubc29 \uacf5\ubb34\uc6d0\uc5d0 \ub300\ud55c \uc784\uba85\uad8c\uacfc \ud574\uc784\uad8c\uc744 \uac00\uc9c4\ub2e4. \ub610\ud55c \uad70\ud1b5\uc218\uad8c\uacfc \uc678\uad50\uad8c\uc744 \uac00\uc9c0\uba70 \uc758\ud68c\uc5d0 \ubc95\uc548\uc744 \uc0c1\uc815\ud560 \uc218 \uc5c6\uace0 \uad6d\uc81c\uae30\uad6c\uc640 \uc7ac\uc678 \uacf5\uad00\uc5d0 \ubbf8\uad6d \ub300\ud45c\ub97c \uc784\uba85\ud55c\ub2e4. \ub300\ud1b5\ub839\uc740 \uad6d\ubbfc\uc774 \uc120\ucd9c\ud558\uba70 \uc784\uae30\ub294 4\ub144\uc774\uace0 \ub450 \ubc88 \uc774\uc0c1 \uc5f0\uc784\ud560 \uc218 \uc5c6\ub2e4. \ub300\ud1b5\ub839 \ubc11\uc5d0\ub294 \ub300\ud1b5\ub839\uc758 \ud589\uc815\uc5c5\ubb34\ub97c \ubcf4\ubd10\ud574\uc904 12\uac1c\uc758 \ud589\uc815\uc2e4\uc774 \uc788\ub2e4. \uc77c\ubc18 \ud589\uc815 \ubd80\uc11c\ub294 \uc911\uc559\uc815\ubd80\uc758 \ud589\uc815\uc744 \ub9e1\ub294 15\uac1c \ubd80\uc11c\ub85c \uad6c\uc131\ub418\uba70, \uac01 \ubd80\uc758 \uc7a5\uad00\uc744 \ub300\ud1b5\ub839\uc774 \uc0c1\uc6d0\uc758 \uc2b9\uc778\uc744 \uc5bb\uc5b4 \uc784\uba85\ud55c\ub2e4. 15\ubd80 \uc7a5\uad00\uc740 \ub0b4\uac01\uc758 \uad6c\uc131\uc6d0\uc774 \ub41c\ub2e4. \uc5f0\ubc29\uc815\ubd80\ub294 15\uac1c\uc758 \ubd80\uc640 \ub2e4\ub978 \ub3c5\ub9bd \uc815\ubd80\uae30\uad00\uc73c\ub85c \uad6c\uc131\ub418\uc5b4 \uc788\ub2e4. \uac01 \uc131\uc758 \uc7a5\uad00\uc740 \ub2e8\uc9c0 \ub300\ud1b5\ub839\uc744 \ubcf4\uc88c\ud558\ub294 \uc785\uc7a5\uc5d0 \uc788\uc744 \ubfd0, \uc758\ud68c\uc5d0 \uc758\uc11d\ub3c4 \uac00\uc9c0\uc9c0 \uc54a\uc73c\uba70, \uc9c0\uc704\ub85c\uc11c\ub294 \ub2e4\ub978 \ub098\ub77c\uc758 \uac01\ub8cc\ubcf4\ub2e4\ub3c4 \ub0ae\ub2e4\uace0 \ud558\uaca0\ub2e4. \ub300\ud1b5\ub839\uc744 \uc9c1\uc811 \ubcf4\uc88c\ud558\ub294 \uae30\uad00\uc73c\ub85c\uc11c\ub294 \ub300\ud1b5\ub839\ubd80\u00b7\uad6d\uac00\uc548\uc804\ubcf4\uc7a5\ud68c\uc758\u00b7\uacbd\uc81c\uc790\ubb38\uc704\uc6d0\ud68c \uc678\uc5d0 \uc5f0\ubc29\uc608\uc0b0\uad6d\u00b7\uc911\uc559\uc815\ubcf4\uad6d\u00b7\uae34\uae09\uacc4\ud68d\uad6d \ub4f1\uc774 \uc788\ub2e4.", "paragraph_answer": "\ub300\ud1b5\ub839 \ud589\uc815\uc2e4\uacfc \uc77c\ubc18 \ud589\uc815 \ubd80\uc11c, \ub300\ud1b5\ub839 \uc9c1\uc18d\ub3c5\ub9bd\uae30\uad00\uc73c\ub85c \uad6c\uc131\ub418\uc5b4 \uc788\ub2e4. \ub300\ud1b5\ub839\uc774 \ud589\uc815\ubd80\uc758 \uc218\ubc18\uc774\uc790 \ub098\ub77c\ub97c \ub300\ud45c\ud558\ub294 \uad6d\uac00\uc6d0\uc218\uc774\ub2e4. \ub300\ud1b5\ub839\uc740 \uc218\ub3c4 \uc6cc\uc2f1\ud134 \uc5d0 \uc788\ub294 \ubc31\uc545\uad00\uc5d0 \uc0b4\uba70 \uc774 \uacf3\uc5d0\uc11c \ub300\ud1b5\ub839 \uc5c5\ubb34\ub97c \ubcf8\ub2e4. \ub300\ud1b5\ub839\uc740 \ud589\uc815\ubd80\uc758 \uc218\ubc18\uc73c\ub85c\uc11c \uc5f0\ubc29\ubc95\uc744 \uc9d1\ud589\ud560 \uad8c\ud55c\uc774 \uc788\uc73c\uba70 \uace0\uc704 \uc5f0\ubc29 \uacf5\ubb34\uc6d0\uc5d0 \ub300\ud55c \uc784\uba85\uad8c\uacfc \ud574\uc784\uad8c\uc744 \uac00\uc9c4\ub2e4. \ub610\ud55c \uad70\ud1b5\uc218\uad8c\uacfc \uc678\uad50\uad8c\uc744 \uac00\uc9c0\uba70 \uc758\ud68c\uc5d0 \ubc95\uc548\uc744 \uc0c1\uc815\ud560 \uc218 \uc5c6\uace0 \uad6d\uc81c\uae30\uad6c\uc640 \uc7ac\uc678 \uacf5\uad00\uc5d0 \ubbf8\uad6d \ub300\ud45c\ub97c \uc784\uba85\ud55c\ub2e4. \ub300\ud1b5\ub839\uc740 \uad6d\ubbfc\uc774 \uc120\ucd9c\ud558\uba70 \uc784\uae30\ub294 4\ub144\uc774\uace0 \ub450 \ubc88 \uc774\uc0c1 \uc5f0\uc784\ud560 \uc218 \uc5c6\ub2e4. \ub300\ud1b5\ub839 \ubc11\uc5d0\ub294 \ub300\ud1b5\ub839\uc758 \ud589\uc815\uc5c5\ubb34\ub97c \ubcf4\ubd10\ud574\uc904 12\uac1c\uc758 \ud589\uc815\uc2e4\uc774 \uc788\ub2e4. \uc77c\ubc18 \ud589\uc815 \ubd80\uc11c\ub294 \uc911\uc559\uc815\ubd80\uc758 \ud589\uc815\uc744 \ub9e1\ub294 15\uac1c \ubd80\uc11c\ub85c \uad6c\uc131\ub418\uba70, \uac01 \ubd80\uc758 \uc7a5\uad00\uc744 \ub300\ud1b5\ub839\uc774 \uc0c1\uc6d0\uc758 \uc2b9\uc778\uc744 \uc5bb\uc5b4 \uc784\uba85\ud55c\ub2e4. 15\ubd80 \uc7a5\uad00\uc740 \ub0b4\uac01\uc758 \uad6c\uc131\uc6d0\uc774 \ub41c\ub2e4. \uc5f0\ubc29\uc815\ubd80\ub294 15\uac1c\uc758 \ubd80\uc640 \ub2e4\ub978 \ub3c5\ub9bd \uc815\ubd80\uae30\uad00\uc73c\ub85c \uad6c\uc131\ub418\uc5b4 \uc788\ub2e4. \uac01 \uc131\uc758 \uc7a5\uad00\uc740 \ub2e8\uc9c0 \ub300\ud1b5\ub839\uc744 \ubcf4\uc88c\ud558\ub294 \uc785\uc7a5\uc5d0 \uc788\uc744 \ubfd0, \uc758\ud68c\uc5d0 \uc758\uc11d\ub3c4 \uac00\uc9c0\uc9c0 \uc54a\uc73c\uba70, \uc9c0\uc704\ub85c\uc11c\ub294 \ub2e4\ub978 \ub098\ub77c\uc758 \uac01\ub8cc\ubcf4\ub2e4\ub3c4 \ub0ae\ub2e4\uace0 \ud558\uaca0\ub2e4. \ub300\ud1b5\ub839\uc744 \uc9c1\uc811 \ubcf4\uc88c\ud558\ub294 \uae30\uad00\uc73c\ub85c\uc11c\ub294 \ub300\ud1b5\ub839\ubd80\u00b7\uad6d\uac00\uc548\uc804\ubcf4\uc7a5\ud68c\uc758\u00b7\uacbd\uc81c\uc790\ubb38\uc704\uc6d0\ud68c \uc678\uc5d0 \uc5f0\ubc29\uc608\uc0b0\uad6d\u00b7\uc911\uc559\uc815\ubcf4\uad6d\u00b7\uae34\uae09\uacc4\ud68d\uad6d \ub4f1\uc774 \uc788\ub2e4.", "sentence_answer": "\ub300\ud1b5\ub839\uc740 \uc218\ub3c4 \uc6cc\uc2f1\ud134 \uc5d0 \uc788\ub294 \ubc31\uc545\uad00\uc5d0 \uc0b4\uba70 \uc774 \uacf3\uc5d0\uc11c \ub300\ud1b5\ub839 \uc5c5\ubb34\ub97c \ubcf8\ub2e4.", "paragraph_id": "6529668-8-1", "paragraph_question": "question: \ubbf8\uad6d\uc758 \uc218\ub3c4\uc774\uc790 \ub300\ud1b5\ub839\uc774 \uc0ac\ub294 \ubc31\uc545\uad00\uc774 \uc704\uce58\ud55c \uc9c0\uc5ed\uc740?, context: \ub300\ud1b5\ub839 \ud589\uc815\uc2e4\uacfc \uc77c\ubc18 \ud589\uc815 \ubd80\uc11c, \ub300\ud1b5\ub839 \uc9c1\uc18d\ub3c5\ub9bd\uae30\uad00\uc73c\ub85c \uad6c\uc131\ub418\uc5b4 \uc788\ub2e4. \ub300\ud1b5\ub839\uc774 \ud589\uc815\ubd80\uc758 \uc218\ubc18\uc774\uc790 \ub098\ub77c\ub97c \ub300\ud45c\ud558\ub294 \uad6d\uac00\uc6d0\uc218\uc774\ub2e4. \ub300\ud1b5\ub839\uc740 \uc218\ub3c4 \uc6cc\uc2f1\ud134\uc5d0 \uc788\ub294 \ubc31\uc545\uad00\uc5d0 \uc0b4\uba70 \uc774 \uacf3\uc5d0\uc11c \ub300\ud1b5\ub839 \uc5c5\ubb34\ub97c \ubcf8\ub2e4. \ub300\ud1b5\ub839\uc740 \ud589\uc815\ubd80\uc758 \uc218\ubc18\uc73c\ub85c\uc11c \uc5f0\ubc29\ubc95\uc744 \uc9d1\ud589\ud560 \uad8c\ud55c\uc774 \uc788\uc73c\uba70 \uace0\uc704 \uc5f0\ubc29 \uacf5\ubb34\uc6d0\uc5d0 \ub300\ud55c \uc784\uba85\uad8c\uacfc \ud574\uc784\uad8c\uc744 \uac00\uc9c4\ub2e4. \ub610\ud55c \uad70\ud1b5\uc218\uad8c\uacfc \uc678\uad50\uad8c\uc744 \uac00\uc9c0\uba70 \uc758\ud68c\uc5d0 \ubc95\uc548\uc744 \uc0c1\uc815\ud560 \uc218 \uc5c6\uace0 \uad6d\uc81c\uae30\uad6c\uc640 \uc7ac\uc678 \uacf5\uad00\uc5d0 \ubbf8\uad6d \ub300\ud45c\ub97c \uc784\uba85\ud55c\ub2e4. \ub300\ud1b5\ub839\uc740 \uad6d\ubbfc\uc774 \uc120\ucd9c\ud558\uba70 \uc784\uae30\ub294 4\ub144\uc774\uace0 \ub450 \ubc88 \uc774\uc0c1 \uc5f0\uc784\ud560 \uc218 \uc5c6\ub2e4. \ub300\ud1b5\ub839 \ubc11\uc5d0\ub294 \ub300\ud1b5\ub839\uc758 \ud589\uc815\uc5c5\ubb34\ub97c \ubcf4\ubd10\ud574\uc904 12\uac1c\uc758 \ud589\uc815\uc2e4\uc774 \uc788\ub2e4. \uc77c\ubc18 \ud589\uc815 \ubd80\uc11c\ub294 \uc911\uc559\uc815\ubd80\uc758 \ud589\uc815\uc744 \ub9e1\ub294 15\uac1c \ubd80\uc11c\ub85c \uad6c\uc131\ub418\uba70, \uac01 \ubd80\uc758 \uc7a5\uad00\uc744 \ub300\ud1b5\ub839\uc774 \uc0c1\uc6d0\uc758 \uc2b9\uc778\uc744 \uc5bb\uc5b4 \uc784\uba85\ud55c\ub2e4. 15\ubd80 \uc7a5\uad00\uc740 \ub0b4\uac01\uc758 \uad6c\uc131\uc6d0\uc774 \ub41c\ub2e4. \uc5f0\ubc29\uc815\ubd80\ub294 15\uac1c\uc758 \ubd80\uc640 \ub2e4\ub978 \ub3c5\ub9bd \uc815\ubd80\uae30\uad00\uc73c\ub85c \uad6c\uc131\ub418\uc5b4 \uc788\ub2e4. \uac01 \uc131\uc758 \uc7a5\uad00\uc740 \ub2e8\uc9c0 \ub300\ud1b5\ub839\uc744 \ubcf4\uc88c\ud558\ub294 \uc785\uc7a5\uc5d0 \uc788\uc744 \ubfd0, \uc758\ud68c\uc5d0 \uc758\uc11d\ub3c4 \uac00\uc9c0\uc9c0 \uc54a\uc73c\uba70, \uc9c0\uc704\ub85c\uc11c\ub294 \ub2e4\ub978 \ub098\ub77c\uc758 \uac01\ub8cc\ubcf4\ub2e4\ub3c4 \ub0ae\ub2e4\uace0 \ud558\uaca0\ub2e4. \ub300\ud1b5\ub839\uc744 \uc9c1\uc811 \ubcf4\uc88c\ud558\ub294 \uae30\uad00\uc73c\ub85c\uc11c\ub294 \ub300\ud1b5\ub839\ubd80\u00b7\uad6d\uac00\uc548\uc804\ubcf4\uc7a5\ud68c\uc758\u00b7\uacbd\uc81c\uc790\ubb38\uc704\uc6d0\ud68c \uc678\uc5d0 \uc5f0\ubc29\uc608\uc0b0\uad6d\u00b7\uc911\uc559\uc815\ubcf4\uad6d\u00b7\uae34\uae09\uacc4\ud68d\uad6d \ub4f1\uc774 \uc788\ub2e4."} {"question": "\ud53c\ud56d\uc18c\uc778\uc774 \ud56d\uc18c\uc5d0 \ubd80\uc218\ud574\uc11c \uc81c1\uc2ec \ud310\uacb0\uc744 \uc790\uae30\uc5d0\uac8c \uc720\ub9ac\ud558\ub3c4\ub85d \ubcc0\uacbd\ud558\ub294 \uac83\uc740?", "paragraph": "\ubd80\ub300\ud56d\uc18c(\u9644\u5e36\u6297\u8a34)\ub294 \ud56d\uc18c\uc778\uc758 \uc0c1\ub300\ubc29\uc778 \ud53c\ud56d\uc18c\uc778\uc774 \ud56d\uc18c\uc5d0 \ubd80\uc218\ud574\uc11c \uc81c1\uc2ec \ud310\uacb0\uc744 \uc790\uae30\uc5d0\uac8c \uc720\ub9ac\ud558\ub3c4\ub85d \ubcc0\uacbd\ud568\uc744 \uad6c\ud558\ub294 \uc2e0\uccad\uc73c\ub85c\uc11c \ud56d\uc18c\ub294 \uc544\ub2c8\ub2e4. \ub530\ub77c\uc11c \ud56d\uc18c\uad8c\uc758 \ud3ec\uae30\ub098 \ud56d\uc18c\uae30\uac04\uc758 \uacbd\uacfc\ub85c \ub9d0\ubbf8\uc554\uc544 \ud56d\uc18c\uad8c\uc774 \uc18c\uba78\ub418\ub354\ub77c\ub3c4 \uc0c1\ub300\ubc29\uc774 \ud56d\uc18c\ud558\uba74 \ubd80\ub300\ud56d\uc18c\ub97c \ud560 \uc218\uac00 \uc788\ub2e4. \ub610\ud55c \ud56d\uc18c\uac00 \uc544\ub2c8\uae30 \ub54c\ubb38\uc5d0 \uc804\ubd80\uc2b9\uc18c\ud55c \uc790\ub77c \ud558\ub354\ub77c\ub3c4 \ubd80\ub300\ud56d\uc18c\uc5d0 \uc758\ud558\uc5ec \uc18c\uc758 \ubcc0\uacbd\uc744 \ud558\uace0 \ubc18\uc18c\ub97c \uc81c\uae30\ud560 \uc218\ub3c4 \uc788\uc73c\uba70 \uc18c\uc1a1\ube44\uc6a9\uc5d0 \ud55c\ud55c \ubd80\ub300\ud56d\uc18c\ub3c4 \ud5c8\uc6a9\ub418\uace0 \uc788\ub2e4. \uc774\uc5d0 \ub530\ub77c\uc11c \ud56d\uc18c\uc5d0 \uc758\ud55c \uc2ec\ud310\uc758 \ubc94\uc704\uac00 \ubcc0\uacbd\ub418\ub294 \uacbd\uc6b0\uac00 \uc788\uc73c\uba70 \ud56d\uc18c\uc778\uc5d0 \ub530\ub77c\uc11c\ub294 \uc81c1\uc2ec\ubcf4\ub2e4 \ubd88\uc774\uc775\ud55c \ud56d\uc18c\ud310\uacb0\uc744 \ubc1b\ub294 \uacbd\uc6b0\ub3c4 \uc0dd\uae34\ub2e4. \ubd80\ub300\ud56d\uc18c\ub294 \ud56d\uc18c\uc758 \uc81c\uae30\ub85c\ubd80\ud130 \ud56d\uc18c\uc2ec\uc758 \ubcc0\ub860\uc774 \uc885\uacb0\ub420 \ub54c\uae4c\uc9c0\ub294 \uc5b8\uc81c\ub4e0\uc9c0 \ud589\ud560 \uc218\uac00 \uc788\uc73c\uba70 \uc774\uac83\uc740 \ud56d\uc18c\uc81c\uae30\uc640 \ub3d9\uc77c\ud55c \ubc29\uc2dd\uc5d0 \uc758\ud55c\ub2e4. \ub610\ud55c \ud56d\uc18c\uc5d0 \ubd80\uc218\ub41c \uac83\uc774\ubbc0\ub85c \ud56d\uc18c\uc758 \ucde8\ud558\ub098 \uac01\ud558 \ub4f1\uc774 \uc788\ub294 \ub54c\uc5d0\ub294 \ubd80\ub300\ud56d\uc18c\ub3c4 \uadf8 \ud6a8\ub825\uc744 \uc783\uac8c \ub418\ub098 \ub2e4\ub9cc \ud56d\uc18c\uae30\uac04 \uc548\uc5d0 \ud589\ud55c \ubd80\ub300\ud56d\uc18c\ub294 \ub3c5\ub9bd\ub41c \ud56d\uc18c\ub85c\uc11c \ub0a8\uac8c \ub41c\ub2e4.", "answer": "\ubd80\ub300\ud56d\uc18c", "sentence": "\ubd80\ub300\ud56d\uc18c (\u9644\u5e36\u6297\u8a34)", "paragraph_sentence": " \ubd80\ub300\ud56d\uc18c (\u9644\u5e36\u6297\u8a34) \ub294 \ud56d\uc18c\uc778\uc758 \uc0c1\ub300\ubc29\uc778 \ud53c\ud56d\uc18c\uc778\uc774 \ud56d\uc18c\uc5d0 \ubd80\uc218\ud574\uc11c \uc81c1\uc2ec \ud310\uacb0\uc744 \uc790\uae30\uc5d0\uac8c \uc720\ub9ac\ud558\ub3c4\ub85d \ubcc0\uacbd\ud568\uc744 \uad6c\ud558\ub294 \uc2e0\uccad\uc73c\ub85c\uc11c \ud56d\uc18c\ub294 \uc544\ub2c8\ub2e4. \ub530\ub77c\uc11c \ud56d\uc18c\uad8c\uc758 \ud3ec\uae30\ub098 \ud56d\uc18c\uae30\uac04\uc758 \uacbd\uacfc\ub85c \ub9d0\ubbf8\uc554\uc544 \ud56d\uc18c\uad8c\uc774 \uc18c\uba78\ub418\ub354\ub77c\ub3c4 \uc0c1\ub300\ubc29\uc774 \ud56d\uc18c\ud558\uba74 \ubd80\ub300\ud56d\uc18c\ub97c \ud560 \uc218\uac00 \uc788\ub2e4. \ub610\ud55c \ud56d\uc18c\uac00 \uc544\ub2c8\uae30 \ub54c\ubb38\uc5d0 \uc804\ubd80\uc2b9\uc18c\ud55c \uc790\ub77c \ud558\ub354\ub77c\ub3c4 \ubd80\ub300\ud56d\uc18c\uc5d0 \uc758\ud558\uc5ec \uc18c\uc758 \ubcc0\uacbd\uc744 \ud558\uace0 \ubc18\uc18c\ub97c \uc81c\uae30\ud560 \uc218\ub3c4 \uc788\uc73c\uba70 \uc18c\uc1a1\ube44\uc6a9\uc5d0 \ud55c\ud55c \ubd80\ub300\ud56d\uc18c\ub3c4 \ud5c8\uc6a9\ub418\uace0 \uc788\ub2e4. \uc774\uc5d0 \ub530\ub77c\uc11c \ud56d\uc18c\uc5d0 \uc758\ud55c \uc2ec\ud310\uc758 \ubc94\uc704\uac00 \ubcc0\uacbd\ub418\ub294 \uacbd\uc6b0\uac00 \uc788\uc73c\uba70 \ud56d\uc18c\uc778\uc5d0 \ub530\ub77c\uc11c\ub294 \uc81c1\uc2ec\ubcf4\ub2e4 \ubd88\uc774\uc775\ud55c \ud56d\uc18c\ud310\uacb0\uc744 \ubc1b\ub294 \uacbd\uc6b0\ub3c4 \uc0dd\uae34\ub2e4. \ubd80\ub300\ud56d\uc18c\ub294 \ud56d\uc18c\uc758 \uc81c\uae30\ub85c\ubd80\ud130 \ud56d\uc18c\uc2ec\uc758 \ubcc0\ub860\uc774 \uc885\uacb0\ub420 \ub54c\uae4c\uc9c0\ub294 \uc5b8\uc81c\ub4e0\uc9c0 \ud589\ud560 \uc218\uac00 \uc788\uc73c\uba70 \uc774\uac83\uc740 \ud56d\uc18c\uc81c\uae30\uc640 \ub3d9\uc77c\ud55c \ubc29\uc2dd\uc5d0 \uc758\ud55c\ub2e4. \ub610\ud55c \ud56d\uc18c\uc5d0 \ubd80\uc218\ub41c \uac83\uc774\ubbc0\ub85c \ud56d\uc18c\uc758 \ucde8\ud558\ub098 \uac01\ud558 \ub4f1\uc774 \uc788\ub294 \ub54c\uc5d0\ub294 \ubd80\ub300\ud56d\uc18c\ub3c4 \uadf8 \ud6a8\ub825\uc744 \uc783\uac8c \ub418\ub098 \ub2e4\ub9cc \ud56d\uc18c\uae30\uac04 \uc548\uc5d0 \ud589\ud55c \ubd80\ub300\ud56d\uc18c\ub294 \ub3c5\ub9bd\ub41c \ud56d\uc18c\ub85c\uc11c \ub0a8\uac8c \ub41c\ub2e4.", "paragraph_answer": " \ubd80\ub300\ud56d\uc18c (\u9644\u5e36\u6297\u8a34)\ub294 \ud56d\uc18c\uc778\uc758 \uc0c1\ub300\ubc29\uc778 \ud53c\ud56d\uc18c\uc778\uc774 \ud56d\uc18c\uc5d0 \ubd80\uc218\ud574\uc11c \uc81c1\uc2ec \ud310\uacb0\uc744 \uc790\uae30\uc5d0\uac8c \uc720\ub9ac\ud558\ub3c4\ub85d \ubcc0\uacbd\ud568\uc744 \uad6c\ud558\ub294 \uc2e0\uccad\uc73c\ub85c\uc11c \ud56d\uc18c\ub294 \uc544\ub2c8\ub2e4. \ub530\ub77c\uc11c \ud56d\uc18c\uad8c\uc758 \ud3ec\uae30\ub098 \ud56d\uc18c\uae30\uac04\uc758 \uacbd\uacfc\ub85c \ub9d0\ubbf8\uc554\uc544 \ud56d\uc18c\uad8c\uc774 \uc18c\uba78\ub418\ub354\ub77c\ub3c4 \uc0c1\ub300\ubc29\uc774 \ud56d\uc18c\ud558\uba74 \ubd80\ub300\ud56d\uc18c\ub97c \ud560 \uc218\uac00 \uc788\ub2e4. \ub610\ud55c \ud56d\uc18c\uac00 \uc544\ub2c8\uae30 \ub54c\ubb38\uc5d0 \uc804\ubd80\uc2b9\uc18c\ud55c \uc790\ub77c \ud558\ub354\ub77c\ub3c4 \ubd80\ub300\ud56d\uc18c\uc5d0 \uc758\ud558\uc5ec \uc18c\uc758 \ubcc0\uacbd\uc744 \ud558\uace0 \ubc18\uc18c\ub97c \uc81c\uae30\ud560 \uc218\ub3c4 \uc788\uc73c\uba70 \uc18c\uc1a1\ube44\uc6a9\uc5d0 \ud55c\ud55c \ubd80\ub300\ud56d\uc18c\ub3c4 \ud5c8\uc6a9\ub418\uace0 \uc788\ub2e4. \uc774\uc5d0 \ub530\ub77c\uc11c \ud56d\uc18c\uc5d0 \uc758\ud55c \uc2ec\ud310\uc758 \ubc94\uc704\uac00 \ubcc0\uacbd\ub418\ub294 \uacbd\uc6b0\uac00 \uc788\uc73c\uba70 \ud56d\uc18c\uc778\uc5d0 \ub530\ub77c\uc11c\ub294 \uc81c1\uc2ec\ubcf4\ub2e4 \ubd88\uc774\uc775\ud55c \ud56d\uc18c\ud310\uacb0\uc744 \ubc1b\ub294 \uacbd\uc6b0\ub3c4 \uc0dd\uae34\ub2e4. \ubd80\ub300\ud56d\uc18c\ub294 \ud56d\uc18c\uc758 \uc81c\uae30\ub85c\ubd80\ud130 \ud56d\uc18c\uc2ec\uc758 \ubcc0\ub860\uc774 \uc885\uacb0\ub420 \ub54c\uae4c\uc9c0\ub294 \uc5b8\uc81c\ub4e0\uc9c0 \ud589\ud560 \uc218\uac00 \uc788\uc73c\uba70 \uc774\uac83\uc740 \ud56d\uc18c\uc81c\uae30\uc640 \ub3d9\uc77c\ud55c \ubc29\uc2dd\uc5d0 \uc758\ud55c\ub2e4. \ub610\ud55c \ud56d\uc18c\uc5d0 \ubd80\uc218\ub41c \uac83\uc774\ubbc0\ub85c \ud56d\uc18c\uc758 \ucde8\ud558\ub098 \uac01\ud558 \ub4f1\uc774 \uc788\ub294 \ub54c\uc5d0\ub294 \ubd80\ub300\ud56d\uc18c\ub3c4 \uadf8 \ud6a8\ub825\uc744 \uc783\uac8c \ub418\ub098 \ub2e4\ub9cc \ud56d\uc18c\uae30\uac04 \uc548\uc5d0 \ud589\ud55c \ubd80\ub300\ud56d\uc18c\ub294 \ub3c5\ub9bd\ub41c \ud56d\uc18c\ub85c\uc11c \ub0a8\uac8c \ub41c\ub2e4.", "sentence_answer": " \ubd80\ub300\ud56d\uc18c (\u9644\u5e36\u6297\u8a34)", "paragraph_id": "6570352-2-0", "paragraph_question": "question: \ud53c\ud56d\uc18c\uc778\uc774 \ud56d\uc18c\uc5d0 \ubd80\uc218\ud574\uc11c \uc81c1\uc2ec \ud310\uacb0\uc744 \uc790\uae30\uc5d0\uac8c \uc720\ub9ac\ud558\ub3c4\ub85d \ubcc0\uacbd\ud558\ub294 \uac83\uc740?, context: \ubd80\ub300\ud56d\uc18c(\u9644\u5e36\u6297\u8a34)\ub294 \ud56d\uc18c\uc778\uc758 \uc0c1\ub300\ubc29\uc778 \ud53c\ud56d\uc18c\uc778\uc774 \ud56d\uc18c\uc5d0 \ubd80\uc218\ud574\uc11c \uc81c1\uc2ec \ud310\uacb0\uc744 \uc790\uae30\uc5d0\uac8c \uc720\ub9ac\ud558\ub3c4\ub85d \ubcc0\uacbd\ud568\uc744 \uad6c\ud558\ub294 \uc2e0\uccad\uc73c\ub85c\uc11c \ud56d\uc18c\ub294 \uc544\ub2c8\ub2e4. \ub530\ub77c\uc11c \ud56d\uc18c\uad8c\uc758 \ud3ec\uae30\ub098 \ud56d\uc18c\uae30\uac04\uc758 \uacbd\uacfc\ub85c \ub9d0\ubbf8\uc554\uc544 \ud56d\uc18c\uad8c\uc774 \uc18c\uba78\ub418\ub354\ub77c\ub3c4 \uc0c1\ub300\ubc29\uc774 \ud56d\uc18c\ud558\uba74 \ubd80\ub300\ud56d\uc18c\ub97c \ud560 \uc218\uac00 \uc788\ub2e4. \ub610\ud55c \ud56d\uc18c\uac00 \uc544\ub2c8\uae30 \ub54c\ubb38\uc5d0 \uc804\ubd80\uc2b9\uc18c\ud55c \uc790\ub77c \ud558\ub354\ub77c\ub3c4 \ubd80\ub300\ud56d\uc18c\uc5d0 \uc758\ud558\uc5ec \uc18c\uc758 \ubcc0\uacbd\uc744 \ud558\uace0 \ubc18\uc18c\ub97c \uc81c\uae30\ud560 \uc218\ub3c4 \uc788\uc73c\uba70 \uc18c\uc1a1\ube44\uc6a9\uc5d0 \ud55c\ud55c \ubd80\ub300\ud56d\uc18c\ub3c4 \ud5c8\uc6a9\ub418\uace0 \uc788\ub2e4. \uc774\uc5d0 \ub530\ub77c\uc11c \ud56d\uc18c\uc5d0 \uc758\ud55c \uc2ec\ud310\uc758 \ubc94\uc704\uac00 \ubcc0\uacbd\ub418\ub294 \uacbd\uc6b0\uac00 \uc788\uc73c\uba70 \ud56d\uc18c\uc778\uc5d0 \ub530\ub77c\uc11c\ub294 \uc81c1\uc2ec\ubcf4\ub2e4 \ubd88\uc774\uc775\ud55c \ud56d\uc18c\ud310\uacb0\uc744 \ubc1b\ub294 \uacbd\uc6b0\ub3c4 \uc0dd\uae34\ub2e4. \ubd80\ub300\ud56d\uc18c\ub294 \ud56d\uc18c\uc758 \uc81c\uae30\ub85c\ubd80\ud130 \ud56d\uc18c\uc2ec\uc758 \ubcc0\ub860\uc774 \uc885\uacb0\ub420 \ub54c\uae4c\uc9c0\ub294 \uc5b8\uc81c\ub4e0\uc9c0 \ud589\ud560 \uc218\uac00 \uc788\uc73c\uba70 \uc774\uac83\uc740 \ud56d\uc18c\uc81c\uae30\uc640 \ub3d9\uc77c\ud55c \ubc29\uc2dd\uc5d0 \uc758\ud55c\ub2e4. \ub610\ud55c \ud56d\uc18c\uc5d0 \ubd80\uc218\ub41c \uac83\uc774\ubbc0\ub85c \ud56d\uc18c\uc758 \ucde8\ud558\ub098 \uac01\ud558 \ub4f1\uc774 \uc788\ub294 \ub54c\uc5d0\ub294 \ubd80\ub300\ud56d\uc18c\ub3c4 \uadf8 \ud6a8\ub825\uc744 \uc783\uac8c \ub418\ub098 \ub2e4\ub9cc \ud56d\uc18c\uae30\uac04 \uc548\uc5d0 \ud589\ud55c \ubd80\ub300\ud56d\uc18c\ub294 \ub3c5\ub9bd\ub41c \ud56d\uc18c\ub85c\uc11c \ub0a8\uac8c \ub41c\ub2e4."} {"question": "\uc77c\ubd80 \uc0ac\ub0e5\uafbc\ub4e4\uc774 \ub291\ub300\ub97c \uc0ac\ub0e5\ud560 \ub54c \uc774\uc6a9\ud558\ub294 \uac83\uc740 \ubb34\uc5c7\uc778\uac00?", "paragraph": "\ub291\ub300\ub294 \ub6f0\uc5b4\ub09c \uac10\uac01, \ube60\ub978 \uc18d\ub3c4\uc640 \ube60\ub974\uac8c \uc0ac\ub0e5\uac1c\ub97c \uc8fd\uc77c \uc218 \uc788\ub294 \ub2a5\ub825, \uac15\ud55c \uccb4\ub825 \ub4f1\uc73c\ub85c \uc778\ud574 \ub291\ub300 \uc0ac\ub0e5\uc740 \ub9e4\uc6b0 \uc5b4\ub824\uc6b4 \uac83\uc73c\ub85c \uc54c\ub824\uc838 \uc788\ub2e4. \uc5ed\uc0ac\uc801\uc73c\ub85c \ub291\ub300\ub97c \uc0ac\ub0e5\ud558\ub294 \ub9ce\uc740 \ubc29\uc548\uc774 \uace0\uc548\ub418\uc5c8\uc73c\uba70 \uc774\ub294 \ubd04\uc5d0 \ub465\uc9c0\uc5d0\uc11c \ud0dc\uc5b4\ub09c \uc0c8\ub07c\ub97c \uc8fd\uc774\ub294 \uac83\uc744 \ud3ec\ud568\ud558\uc5ec \uac1c\ub97c \uc774\uc6a9\ud55c \uc0ac\ub0e5(\ubcf4\ud1b5 \uc2dc\uac01 \ud558\uc6b4\ub4dc, \ube14\ub7ec\ub4dc \ud558\uc6b4\ub4dc, \ud3ed\uc2a4 \ud14c\ub9ac\uc5b4\ub97c \uc774\uc6a9), \uc2a4\ud2b8\ub9ac\ud06c\ub2cc \uc911\ub3c5, \uacfc \ubc1c\ud310, \ub36b, \ub7ec\uc2dc\uc544\uc5d0\uc11c \uc778\uae30 \uc788\ub294 \uc0ac\ub0e5 \ubc29\ubc95\uc778 \uc778\uac04 \ub0c4\uc0c8\ub97c \ubfcc\ub9ac\uace0 \ud50c\ub77c\ub4dc\ub9ac\ub85c \uc560\uc6cc\uc2f8\uc6b4 \ub2e4\uc74c \ubb34\ub9ac \uc804\uccb4\ub97c \uc0ac\ub0e5\ud558\ub294 \uac83\uc774 \uc788\ub2e4. \uc774 \ubc29\ubc95\uc740 \ub291\ub300\uc758 \uc778\uac04 \ub0c4\uc0c8\uc5d0 \ub300\ud55c \uacf5\ud3ec\uc5d0 \uc758\uc874\ud558\uba70 \ub291\ub300\uac00 \ub0c4\uc0c8\uc5d0 \uc775\uc219\ud574\uc9c0\uba74 \ud6a8\uacfc\ub97c \uc783\uc744 \uc218 \uc788\ub2e4. \uc77c\ubd80 \uc0ac\ub0e5\uafbc\ub4e4\uc740 \uc804\ud654\ub97c \uc774\uc6a9\ud558\uc5ec \ub291\ub300\ub97c \uc0ac\ub0e5\ud558\uae30\ub3c4 \ud55c\ub2e4. \uce74\uc790\ud750\uc2a4\ud0c4\uacfc \ubabd\uace8\uc5d0\uc11c\ub294 \uc804\ud1b5\uc801\uc73c\ub85c \ub291\ub300\ub97c \ub3c5\uc218\ub9ac\uc640 \ub9e4\ub85c \uc0ac\ub0e5\ud558\uba70 \ud604\uc7ac\ub294 \uacbd\ud5d8\uc774 \ud48d\ubd80\ud55c \ub9e4\uc7a1\uc774\uac00 \uc904\uc5b4\ub4e4\uc5b4 \uac10\uc18c\ud558\uace0 \uc788\ub2e4. \ud56d\uacf5\uae30\ub85c \ub291\ub300\ub97c \uc3d8\ub294 \uac83\uc740 \uc9c0\uc0c1\uc5d0\uc11c\ubcf4\ub2e4 \uac00\uc2dc\ub3c4\uac00 \ub192\uae30 \ub54c\ubb38\uc5d0 \ub9e4\uc6b0 \ud6a8\uc728\uc801\uc774\uba70 \uc774 \ubc29\ubc95\uc740 \ub17c\uc7c1\uc774 \uc788\uc73c\ub098 \ub291\ub300\uc5d0\uac8c \ud0c8\ucd9c\ud558\uac70\ub098 \ubcf4\ud638\ud558\ub294 \uae30\ud68c\ub97c \uc8fc\uae30\ub3c4 \ud55c\ub2e4.", "answer": "\uc804\ud654", "sentence": "\uc77c\ubd80 \uc0ac\ub0e5\uafbc\ub4e4\uc740 \uc804\ud654 \ub97c \uc774\uc6a9\ud558\uc5ec \ub291\ub300\ub97c \uc0ac\ub0e5\ud558\uae30\ub3c4 \ud55c\ub2e4.", "paragraph_sentence": "\ub291\ub300\ub294 \ub6f0\uc5b4\ub09c \uac10\uac01, \ube60\ub978 \uc18d\ub3c4\uc640 \ube60\ub974\uac8c \uc0ac\ub0e5\uac1c\ub97c \uc8fd\uc77c \uc218 \uc788\ub294 \ub2a5\ub825, \uac15\ud55c \uccb4\ub825 \ub4f1\uc73c\ub85c \uc778\ud574 \ub291\ub300 \uc0ac\ub0e5\uc740 \ub9e4\uc6b0 \uc5b4\ub824\uc6b4 \uac83\uc73c\ub85c \uc54c\ub824\uc838 \uc788\ub2e4. \uc5ed\uc0ac\uc801\uc73c\ub85c \ub291\ub300\ub97c \uc0ac\ub0e5\ud558\ub294 \ub9ce\uc740 \ubc29\uc548\uc774 \uace0\uc548\ub418\uc5c8\uc73c\uba70 \uc774\ub294 \ubd04\uc5d0 \ub465\uc9c0\uc5d0\uc11c \ud0dc\uc5b4\ub09c \uc0c8\ub07c\ub97c \uc8fd\uc774\ub294 \uac83\uc744 \ud3ec\ud568\ud558\uc5ec \uac1c\ub97c \uc774\uc6a9\ud55c \uc0ac\ub0e5(\ubcf4\ud1b5 \uc2dc\uac01 \ud558\uc6b4\ub4dc, \ube14\ub7ec\ub4dc \ud558\uc6b4\ub4dc, \ud3ed\uc2a4 \ud14c\ub9ac\uc5b4\ub97c \uc774\uc6a9), \uc2a4\ud2b8\ub9ac\ud06c\ub2cc \uc911\ub3c5, \uacfc \ubc1c\ud310, \ub36b, \ub7ec\uc2dc\uc544\uc5d0\uc11c \uc778\uae30 \uc788\ub294 \uc0ac\ub0e5 \ubc29\ubc95\uc778 \uc778\uac04 \ub0c4\uc0c8\ub97c \ubfcc\ub9ac\uace0 \ud50c\ub77c\ub4dc\ub9ac\ub85c \uc560\uc6cc\uc2f8\uc6b4 \ub2e4\uc74c \ubb34\ub9ac \uc804\uccb4\ub97c \uc0ac\ub0e5\ud558\ub294 \uac83\uc774 \uc788\ub2e4. \uc774 \ubc29\ubc95\uc740 \ub291\ub300\uc758 \uc778\uac04 \ub0c4\uc0c8\uc5d0 \ub300\ud55c \uacf5\ud3ec\uc5d0 \uc758\uc874\ud558\uba70 \ub291\ub300\uac00 \ub0c4\uc0c8\uc5d0 \uc775\uc219\ud574\uc9c0\uba74 \ud6a8\uacfc\ub97c \uc783\uc744 \uc218 \uc788\ub2e4. \uc77c\ubd80 \uc0ac\ub0e5\uafbc\ub4e4\uc740 \uc804\ud654 \ub97c \uc774\uc6a9\ud558\uc5ec \ub291\ub300\ub97c \uc0ac\ub0e5\ud558\uae30\ub3c4 \ud55c\ub2e4. \uce74\uc790\ud750\uc2a4\ud0c4\uacfc \ubabd\uace8\uc5d0\uc11c\ub294 \uc804\ud1b5\uc801\uc73c\ub85c \ub291\ub300\ub97c \ub3c5\uc218\ub9ac\uc640 \ub9e4\ub85c \uc0ac\ub0e5\ud558\uba70 \ud604\uc7ac\ub294 \uacbd\ud5d8\uc774 \ud48d\ubd80\ud55c \ub9e4\uc7a1\uc774\uac00 \uc904\uc5b4\ub4e4\uc5b4 \uac10\uc18c\ud558\uace0 \uc788\ub2e4. \ud56d\uacf5\uae30\ub85c \ub291\ub300\ub97c \uc3d8\ub294 \uac83\uc740 \uc9c0\uc0c1\uc5d0\uc11c\ubcf4\ub2e4 \uac00\uc2dc\ub3c4\uac00 \ub192\uae30 \ub54c\ubb38\uc5d0 \ub9e4\uc6b0 \ud6a8\uc728\uc801\uc774\uba70 \uc774 \ubc29\ubc95\uc740 \ub17c\uc7c1\uc774 \uc788\uc73c\ub098 \ub291\ub300\uc5d0\uac8c \ud0c8\ucd9c\ud558\uac70\ub098 \ubcf4\ud638\ud558\ub294 \uae30\ud68c\ub97c \uc8fc\uae30\ub3c4 \ud55c\ub2e4.", "paragraph_answer": "\ub291\ub300\ub294 \ub6f0\uc5b4\ub09c \uac10\uac01, \ube60\ub978 \uc18d\ub3c4\uc640 \ube60\ub974\uac8c \uc0ac\ub0e5\uac1c\ub97c \uc8fd\uc77c \uc218 \uc788\ub294 \ub2a5\ub825, \uac15\ud55c \uccb4\ub825 \ub4f1\uc73c\ub85c \uc778\ud574 \ub291\ub300 \uc0ac\ub0e5\uc740 \ub9e4\uc6b0 \uc5b4\ub824\uc6b4 \uac83\uc73c\ub85c \uc54c\ub824\uc838 \uc788\ub2e4. \uc5ed\uc0ac\uc801\uc73c\ub85c \ub291\ub300\ub97c \uc0ac\ub0e5\ud558\ub294 \ub9ce\uc740 \ubc29\uc548\uc774 \uace0\uc548\ub418\uc5c8\uc73c\uba70 \uc774\ub294 \ubd04\uc5d0 \ub465\uc9c0\uc5d0\uc11c \ud0dc\uc5b4\ub09c \uc0c8\ub07c\ub97c \uc8fd\uc774\ub294 \uac83\uc744 \ud3ec\ud568\ud558\uc5ec \uac1c\ub97c \uc774\uc6a9\ud55c \uc0ac\ub0e5(\ubcf4\ud1b5 \uc2dc\uac01 \ud558\uc6b4\ub4dc, \ube14\ub7ec\ub4dc \ud558\uc6b4\ub4dc, \ud3ed\uc2a4 \ud14c\ub9ac\uc5b4\ub97c \uc774\uc6a9), \uc2a4\ud2b8\ub9ac\ud06c\ub2cc \uc911\ub3c5, \uacfc \ubc1c\ud310, \ub36b, \ub7ec\uc2dc\uc544\uc5d0\uc11c \uc778\uae30 \uc788\ub294 \uc0ac\ub0e5 \ubc29\ubc95\uc778 \uc778\uac04 \ub0c4\uc0c8\ub97c \ubfcc\ub9ac\uace0 \ud50c\ub77c\ub4dc\ub9ac\ub85c \uc560\uc6cc\uc2f8\uc6b4 \ub2e4\uc74c \ubb34\ub9ac \uc804\uccb4\ub97c \uc0ac\ub0e5\ud558\ub294 \uac83\uc774 \uc788\ub2e4. \uc774 \ubc29\ubc95\uc740 \ub291\ub300\uc758 \uc778\uac04 \ub0c4\uc0c8\uc5d0 \ub300\ud55c \uacf5\ud3ec\uc5d0 \uc758\uc874\ud558\uba70 \ub291\ub300\uac00 \ub0c4\uc0c8\uc5d0 \uc775\uc219\ud574\uc9c0\uba74 \ud6a8\uacfc\ub97c \uc783\uc744 \uc218 \uc788\ub2e4. \uc77c\ubd80 \uc0ac\ub0e5\uafbc\ub4e4\uc740 \uc804\ud654 \ub97c \uc774\uc6a9\ud558\uc5ec \ub291\ub300\ub97c \uc0ac\ub0e5\ud558\uae30\ub3c4 \ud55c\ub2e4. \uce74\uc790\ud750\uc2a4\ud0c4\uacfc \ubabd\uace8\uc5d0\uc11c\ub294 \uc804\ud1b5\uc801\uc73c\ub85c \ub291\ub300\ub97c \ub3c5\uc218\ub9ac\uc640 \ub9e4\ub85c \uc0ac\ub0e5\ud558\uba70 \ud604\uc7ac\ub294 \uacbd\ud5d8\uc774 \ud48d\ubd80\ud55c \ub9e4\uc7a1\uc774\uac00 \uc904\uc5b4\ub4e4\uc5b4 \uac10\uc18c\ud558\uace0 \uc788\ub2e4. \ud56d\uacf5\uae30\ub85c \ub291\ub300\ub97c \uc3d8\ub294 \uac83\uc740 \uc9c0\uc0c1\uc5d0\uc11c\ubcf4\ub2e4 \uac00\uc2dc\ub3c4\uac00 \ub192\uae30 \ub54c\ubb38\uc5d0 \ub9e4\uc6b0 \ud6a8\uc728\uc801\uc774\uba70 \uc774 \ubc29\ubc95\uc740 \ub17c\uc7c1\uc774 \uc788\uc73c\ub098 \ub291\ub300\uc5d0\uac8c \ud0c8\ucd9c\ud558\uac70\ub098 \ubcf4\ud638\ud558\ub294 \uae30\ud68c\ub97c \uc8fc\uae30\ub3c4 \ud55c\ub2e4.", "sentence_answer": "\uc77c\ubd80 \uc0ac\ub0e5\uafbc\ub4e4\uc740 \uc804\ud654 \ub97c \uc774\uc6a9\ud558\uc5ec \ub291\ub300\ub97c \uc0ac\ub0e5\ud558\uae30\ub3c4 \ud55c\ub2e4.", "paragraph_id": "6572454-53-1", "paragraph_question": "question: \uc77c\ubd80 \uc0ac\ub0e5\uafbc\ub4e4\uc774 \ub291\ub300\ub97c \uc0ac\ub0e5\ud560 \ub54c \uc774\uc6a9\ud558\ub294 \uac83\uc740 \ubb34\uc5c7\uc778\uac00?, context: \ub291\ub300\ub294 \ub6f0\uc5b4\ub09c \uac10\uac01, \ube60\ub978 \uc18d\ub3c4\uc640 \ube60\ub974\uac8c \uc0ac\ub0e5\uac1c\ub97c \uc8fd\uc77c \uc218 \uc788\ub294 \ub2a5\ub825, \uac15\ud55c \uccb4\ub825 \ub4f1\uc73c\ub85c \uc778\ud574 \ub291\ub300 \uc0ac\ub0e5\uc740 \ub9e4\uc6b0 \uc5b4\ub824\uc6b4 \uac83\uc73c\ub85c \uc54c\ub824\uc838 \uc788\ub2e4. \uc5ed\uc0ac\uc801\uc73c\ub85c \ub291\ub300\ub97c \uc0ac\ub0e5\ud558\ub294 \ub9ce\uc740 \ubc29\uc548\uc774 \uace0\uc548\ub418\uc5c8\uc73c\uba70 \uc774\ub294 \ubd04\uc5d0 \ub465\uc9c0\uc5d0\uc11c \ud0dc\uc5b4\ub09c \uc0c8\ub07c\ub97c \uc8fd\uc774\ub294 \uac83\uc744 \ud3ec\ud568\ud558\uc5ec \uac1c\ub97c \uc774\uc6a9\ud55c \uc0ac\ub0e5(\ubcf4\ud1b5 \uc2dc\uac01 \ud558\uc6b4\ub4dc, \ube14\ub7ec\ub4dc \ud558\uc6b4\ub4dc, \ud3ed\uc2a4 \ud14c\ub9ac\uc5b4\ub97c \uc774\uc6a9), \uc2a4\ud2b8\ub9ac\ud06c\ub2cc \uc911\ub3c5, \uacfc \ubc1c\ud310, \ub36b, \ub7ec\uc2dc\uc544\uc5d0\uc11c \uc778\uae30 \uc788\ub294 \uc0ac\ub0e5 \ubc29\ubc95\uc778 \uc778\uac04 \ub0c4\uc0c8\ub97c \ubfcc\ub9ac\uace0 \ud50c\ub77c\ub4dc\ub9ac\ub85c \uc560\uc6cc\uc2f8\uc6b4 \ub2e4\uc74c \ubb34\ub9ac \uc804\uccb4\ub97c \uc0ac\ub0e5\ud558\ub294 \uac83\uc774 \uc788\ub2e4. \uc774 \ubc29\ubc95\uc740 \ub291\ub300\uc758 \uc778\uac04 \ub0c4\uc0c8\uc5d0 \ub300\ud55c \uacf5\ud3ec\uc5d0 \uc758\uc874\ud558\uba70 \ub291\ub300\uac00 \ub0c4\uc0c8\uc5d0 \uc775\uc219\ud574\uc9c0\uba74 \ud6a8\uacfc\ub97c \uc783\uc744 \uc218 \uc788\ub2e4. \uc77c\ubd80 \uc0ac\ub0e5\uafbc\ub4e4\uc740 \uc804\ud654\ub97c \uc774\uc6a9\ud558\uc5ec \ub291\ub300\ub97c \uc0ac\ub0e5\ud558\uae30\ub3c4 \ud55c\ub2e4. \uce74\uc790\ud750\uc2a4\ud0c4\uacfc \ubabd\uace8\uc5d0\uc11c\ub294 \uc804\ud1b5\uc801\uc73c\ub85c \ub291\ub300\ub97c \ub3c5\uc218\ub9ac\uc640 \ub9e4\ub85c \uc0ac\ub0e5\ud558\uba70 \ud604\uc7ac\ub294 \uacbd\ud5d8\uc774 \ud48d\ubd80\ud55c \ub9e4\uc7a1\uc774\uac00 \uc904\uc5b4\ub4e4\uc5b4 \uac10\uc18c\ud558\uace0 \uc788\ub2e4. \ud56d\uacf5\uae30\ub85c \ub291\ub300\ub97c \uc3d8\ub294 \uac83\uc740 \uc9c0\uc0c1\uc5d0\uc11c\ubcf4\ub2e4 \uac00\uc2dc\ub3c4\uac00 \ub192\uae30 \ub54c\ubb38\uc5d0 \ub9e4\uc6b0 \ud6a8\uc728\uc801\uc774\uba70 \uc774 \ubc29\ubc95\uc740 \ub17c\uc7c1\uc774 \uc788\uc73c\ub098 \ub291\ub300\uc5d0\uac8c \ud0c8\ucd9c\ud558\uac70\ub098 \ubcf4\ud638\ud558\ub294 \uae30\ud68c\ub97c \uc8fc\uae30\ub3c4 \ud55c\ub2e4."} {"question": "\ud074\ub9ad\ub960\uacfc \uc720\uc0ac\ud55c \ud45c\ud604\uc73c\ub85c \uc77c\ubc18\uc801\uc73c\ub85c \ub110\ub9ac \uc4f0\uc774\ub294 \uc6a9\uc5b4\ub294 \ubb34\uc5c7\uc778\uac00?", "paragraph": "\ub9c8\ucf00\ud305 \uc804\ubb38\uac00\ub4e4\uc740 \uc6f9 \ubc29\ubb38\uc790\ub4e4\uc758 \ubc18\uc751\uc744 \uc54c\uace0 \uc2f6\uc5b4\ud558\uc9c0\ub9cc, \ud2b9\uc815 \uc6f9\uc0ac\uc774\ud2b8\uc5d0 \ub300\ud55c \uc0ac\uc6a9\uc790\ub4e4\uc758 \uac10\uc815\uc801 \ubc18\uc751 \ubc0f \uc6f9\uc0ac\uc774\ud2b8\ub97c \ud1b5\ud55c \uae30\uc5c5\uc758 \ube0c\ub79c\ub4dc \uc774\ubbf8\uc9c0 \uc0c1\uc2b9 \ud6a8\uacfc\ub97c \uc644\uc804\ud788 \uc815\ub7c9\ud654 \uc2dc\ud0a4\ub294 \uac83\uc740 \ud604\uc7ac\uc758 \uae30\uc220\ub85c\uc11c\ub294 \uac70\uc758 \ubd88\uac00\ub2a5\ud558\ub2e4\uace0 \ubcfc \uc218 \uc788\ub2e4. \uc774\uc5d0 \ubc18\ud574 \ubcf4\ub2e4 \uc27d\uac8c \uc5bb\uc5b4\ub0bc \uc218 \uc788\ub294 \uc815\ubcf4\uac00 \ubc14\ub85c \ud074\ub9ad\ub960\uc774\ub2e4. \ud074\ub9ad\ub960\uc740 \uad11\uace0\uc5d0 \uc758\ud574 \uc758\ub3c4\ub41c \ud398\uc774\uc9c0\ub85c \uc5f0\uacb0\ub41c \ubc29\ubb38\uac1d \uc911 \uc5b4\ub5a0\ud55c \ud488\ubaa9\uc744 \uad6c\uc785\ud558\uac70\ub098 \uc81c\ud488 \ub610\ub294 \uc11c\ube44\uc2a4\uc5d0 \ub300\ud55c \uc815\ubcf4\ub97c \uc5bb\ub294 \ub4f1, \uad11\uace0\ub85c\ubd80\ud130 \uc720\ub3c4\ub41c \ud589\uc704\ub97c \uc2dc\ud589\ud55c \ubc29\ubb38\uc790\uc758 \ube44\uc728\uc744 \uce21\uc815\ud55c\ub2e4. \ub2e4\uc591\ud55c \ud615\ud0dc\uc758 \uc0c1\ud638 \uc791\uc6a9\uc774 \uac00\ub2a5\ud558\uc9c0\ub9cc \uc5ec\uae30\uc11c \uad11\uace0(\ud639\uc740 \ub9c1\ud06c)\uc5d0 \ub300\ud574 \"\ub9c8\uc6b0\uc2a4 \ud074\ub9ad\"\uc774\ub77c\ub294 \ud45c\ud604\uc744 \uc0ac\uc6a9\ud55c \uac83\uc740 \uc774 \ud45c\ud604\uc774 \uc77c\ubc18\uc801\uc73c\ub85c \ub110\ub9ac \uc4f0\uc774\ub294 \uc6a9\uc5b4\uc774\uae30 \ub54c\ubb38\uc774\ub2e4.", "answer": "\ub9c8\uc6b0\uc2a4 \ud074\ub9ad", "sentence": "\ub2e4\uc591\ud55c \ud615\ud0dc\uc758 \uc0c1\ud638 \uc791\uc6a9\uc774 \uac00\ub2a5\ud558\uc9c0\ub9cc \uc5ec\uae30\uc11c \uad11\uace0(\ud639\uc740 \ub9c1\ud06c)\uc5d0 \ub300\ud574 \" \ub9c8\uc6b0\uc2a4 \ud074\ub9ad \"\uc774\ub77c\ub294 \ud45c\ud604\uc744 \uc0ac\uc6a9\ud55c \uac83\uc740 \uc774 \ud45c\ud604\uc774 \uc77c\ubc18\uc801\uc73c\ub85c \ub110\ub9ac \uc4f0\uc774\ub294 \uc6a9\uc5b4\uc774\uae30 \ub54c\ubb38\uc774\ub2e4.", "paragraph_sentence": "\ub9c8\ucf00\ud305 \uc804\ubb38\uac00\ub4e4\uc740 \uc6f9 \ubc29\ubb38\uc790\ub4e4\uc758 \ubc18\uc751\uc744 \uc54c\uace0 \uc2f6\uc5b4\ud558\uc9c0\ub9cc, \ud2b9\uc815 \uc6f9\uc0ac\uc774\ud2b8\uc5d0 \ub300\ud55c \uc0ac\uc6a9\uc790\ub4e4\uc758 \uac10\uc815\uc801 \ubc18\uc751 \ubc0f \uc6f9\uc0ac\uc774\ud2b8\ub97c \ud1b5\ud55c \uae30\uc5c5\uc758 \ube0c\ub79c\ub4dc \uc774\ubbf8\uc9c0 \uc0c1\uc2b9 \ud6a8\uacfc\ub97c \uc644\uc804\ud788 \uc815\ub7c9\ud654 \uc2dc\ud0a4\ub294 \uac83\uc740 \ud604\uc7ac\uc758 \uae30\uc220\ub85c\uc11c\ub294 \uac70\uc758 \ubd88\uac00\ub2a5\ud558\ub2e4\uace0 \ubcfc \uc218 \uc788\ub2e4. \uc774\uc5d0 \ubc18\ud574 \ubcf4\ub2e4 \uc27d\uac8c \uc5bb\uc5b4\ub0bc \uc218 \uc788\ub294 \uc815\ubcf4\uac00 \ubc14\ub85c \ud074\ub9ad\ub960\uc774\ub2e4. \ud074\ub9ad\ub960\uc740 \uad11\uace0\uc5d0 \uc758\ud574 \uc758\ub3c4\ub41c \ud398\uc774\uc9c0\ub85c \uc5f0\uacb0\ub41c \ubc29\ubb38\uac1d \uc911 \uc5b4\ub5a0\ud55c \ud488\ubaa9\uc744 \uad6c\uc785\ud558\uac70\ub098 \uc81c\ud488 \ub610\ub294 \uc11c\ube44\uc2a4\uc5d0 \ub300\ud55c \uc815\ubcf4\ub97c \uc5bb\ub294 \ub4f1, \uad11\uace0\ub85c\ubd80\ud130 \uc720\ub3c4\ub41c \ud589\uc704\ub97c \uc2dc\ud589\ud55c \ubc29\ubb38\uc790\uc758 \ube44\uc728\uc744 \uce21\uc815\ud55c\ub2e4. \ub2e4\uc591\ud55c \ud615\ud0dc\uc758 \uc0c1\ud638 \uc791\uc6a9\uc774 \uac00\ub2a5\ud558\uc9c0\ub9cc \uc5ec\uae30\uc11c \uad11\uace0(\ud639\uc740 \ub9c1\ud06c)\uc5d0 \ub300\ud574 \" \ub9c8\uc6b0\uc2a4 \ud074\ub9ad \"\uc774\ub77c\ub294 \ud45c\ud604\uc744 \uc0ac\uc6a9\ud55c \uac83\uc740 \uc774 \ud45c\ud604\uc774 \uc77c\ubc18\uc801\uc73c\ub85c \ub110\ub9ac \uc4f0\uc774\ub294 \uc6a9\uc5b4\uc774\uae30 \ub54c\ubb38\uc774\ub2e4. ", "paragraph_answer": "\ub9c8\ucf00\ud305 \uc804\ubb38\uac00\ub4e4\uc740 \uc6f9 \ubc29\ubb38\uc790\ub4e4\uc758 \ubc18\uc751\uc744 \uc54c\uace0 \uc2f6\uc5b4\ud558\uc9c0\ub9cc, \ud2b9\uc815 \uc6f9\uc0ac\uc774\ud2b8\uc5d0 \ub300\ud55c \uc0ac\uc6a9\uc790\ub4e4\uc758 \uac10\uc815\uc801 \ubc18\uc751 \ubc0f \uc6f9\uc0ac\uc774\ud2b8\ub97c \ud1b5\ud55c \uae30\uc5c5\uc758 \ube0c\ub79c\ub4dc \uc774\ubbf8\uc9c0 \uc0c1\uc2b9 \ud6a8\uacfc\ub97c \uc644\uc804\ud788 \uc815\ub7c9\ud654 \uc2dc\ud0a4\ub294 \uac83\uc740 \ud604\uc7ac\uc758 \uae30\uc220\ub85c\uc11c\ub294 \uac70\uc758 \ubd88\uac00\ub2a5\ud558\ub2e4\uace0 \ubcfc \uc218 \uc788\ub2e4. \uc774\uc5d0 \ubc18\ud574 \ubcf4\ub2e4 \uc27d\uac8c \uc5bb\uc5b4\ub0bc \uc218 \uc788\ub294 \uc815\ubcf4\uac00 \ubc14\ub85c \ud074\ub9ad\ub960\uc774\ub2e4. \ud074\ub9ad\ub960\uc740 \uad11\uace0\uc5d0 \uc758\ud574 \uc758\ub3c4\ub41c \ud398\uc774\uc9c0\ub85c \uc5f0\uacb0\ub41c \ubc29\ubb38\uac1d \uc911 \uc5b4\ub5a0\ud55c \ud488\ubaa9\uc744 \uad6c\uc785\ud558\uac70\ub098 \uc81c\ud488 \ub610\ub294 \uc11c\ube44\uc2a4\uc5d0 \ub300\ud55c \uc815\ubcf4\ub97c \uc5bb\ub294 \ub4f1, \uad11\uace0\ub85c\ubd80\ud130 \uc720\ub3c4\ub41c \ud589\uc704\ub97c \uc2dc\ud589\ud55c \ubc29\ubb38\uc790\uc758 \ube44\uc728\uc744 \uce21\uc815\ud55c\ub2e4. \ub2e4\uc591\ud55c \ud615\ud0dc\uc758 \uc0c1\ud638 \uc791\uc6a9\uc774 \uac00\ub2a5\ud558\uc9c0\ub9cc \uc5ec\uae30\uc11c \uad11\uace0(\ud639\uc740 \ub9c1\ud06c)\uc5d0 \ub300\ud574 \" \ub9c8\uc6b0\uc2a4 \ud074\ub9ad \"\uc774\ub77c\ub294 \ud45c\ud604\uc744 \uc0ac\uc6a9\ud55c \uac83\uc740 \uc774 \ud45c\ud604\uc774 \uc77c\ubc18\uc801\uc73c\ub85c \ub110\ub9ac \uc4f0\uc774\ub294 \uc6a9\uc5b4\uc774\uae30 \ub54c\ubb38\uc774\ub2e4.", "sentence_answer": "\ub2e4\uc591\ud55c \ud615\ud0dc\uc758 \uc0c1\ud638 \uc791\uc6a9\uc774 \uac00\ub2a5\ud558\uc9c0\ub9cc \uc5ec\uae30\uc11c \uad11\uace0(\ud639\uc740 \ub9c1\ud06c)\uc5d0 \ub300\ud574 \" \ub9c8\uc6b0\uc2a4 \ud074\ub9ad \"\uc774\ub77c\ub294 \ud45c\ud604\uc744 \uc0ac\uc6a9\ud55c \uac83\uc740 \uc774 \ud45c\ud604\uc774 \uc77c\ubc18\uc801\uc73c\ub85c \ub110\ub9ac \uc4f0\uc774\ub294 \uc6a9\uc5b4\uc774\uae30 \ub54c\ubb38\uc774\ub2e4.", "paragraph_id": "5910562-0-1", "paragraph_question": "question: \ud074\ub9ad\ub960\uacfc \uc720\uc0ac\ud55c \ud45c\ud604\uc73c\ub85c \uc77c\ubc18\uc801\uc73c\ub85c \ub110\ub9ac \uc4f0\uc774\ub294 \uc6a9\uc5b4\ub294 \ubb34\uc5c7\uc778\uac00?, context: \ub9c8\ucf00\ud305 \uc804\ubb38\uac00\ub4e4\uc740 \uc6f9 \ubc29\ubb38\uc790\ub4e4\uc758 \ubc18\uc751\uc744 \uc54c\uace0 \uc2f6\uc5b4\ud558\uc9c0\ub9cc, \ud2b9\uc815 \uc6f9\uc0ac\uc774\ud2b8\uc5d0 \ub300\ud55c \uc0ac\uc6a9\uc790\ub4e4\uc758 \uac10\uc815\uc801 \ubc18\uc751 \ubc0f \uc6f9\uc0ac\uc774\ud2b8\ub97c \ud1b5\ud55c \uae30\uc5c5\uc758 \ube0c\ub79c\ub4dc \uc774\ubbf8\uc9c0 \uc0c1\uc2b9 \ud6a8\uacfc\ub97c \uc644\uc804\ud788 \uc815\ub7c9\ud654 \uc2dc\ud0a4\ub294 \uac83\uc740 \ud604\uc7ac\uc758 \uae30\uc220\ub85c\uc11c\ub294 \uac70\uc758 \ubd88\uac00\ub2a5\ud558\ub2e4\uace0 \ubcfc \uc218 \uc788\ub2e4. \uc774\uc5d0 \ubc18\ud574 \ubcf4\ub2e4 \uc27d\uac8c \uc5bb\uc5b4\ub0bc \uc218 \uc788\ub294 \uc815\ubcf4\uac00 \ubc14\ub85c \ud074\ub9ad\ub960\uc774\ub2e4. \ud074\ub9ad\ub960\uc740 \uad11\uace0\uc5d0 \uc758\ud574 \uc758\ub3c4\ub41c \ud398\uc774\uc9c0\ub85c \uc5f0\uacb0\ub41c \ubc29\ubb38\uac1d \uc911 \uc5b4\ub5a0\ud55c \ud488\ubaa9\uc744 \uad6c\uc785\ud558\uac70\ub098 \uc81c\ud488 \ub610\ub294 \uc11c\ube44\uc2a4\uc5d0 \ub300\ud55c \uc815\ubcf4\ub97c \uc5bb\ub294 \ub4f1, \uad11\uace0\ub85c\ubd80\ud130 \uc720\ub3c4\ub41c \ud589\uc704\ub97c \uc2dc\ud589\ud55c \ubc29\ubb38\uc790\uc758 \ube44\uc728\uc744 \uce21\uc815\ud55c\ub2e4. \ub2e4\uc591\ud55c \ud615\ud0dc\uc758 \uc0c1\ud638 \uc791\uc6a9\uc774 \uac00\ub2a5\ud558\uc9c0\ub9cc \uc5ec\uae30\uc11c \uad11\uace0(\ud639\uc740 \ub9c1\ud06c)\uc5d0 \ub300\ud574 \"\ub9c8\uc6b0\uc2a4 \ud074\ub9ad\"\uc774\ub77c\ub294 \ud45c\ud604\uc744 \uc0ac\uc6a9\ud55c \uac83\uc740 \uc774 \ud45c\ud604\uc774 \uc77c\ubc18\uc801\uc73c\ub85c \ub110\ub9ac \uc4f0\uc774\ub294 \uc6a9\uc5b4\uc774\uae30 \ub54c\ubb38\uc774\ub2e4."} {"question": "\ubd88\ubc95 \ub3c4\uccad\uc5d0 \ub300\ud55c \uad6d\uc815\uc6d0 \ubcf4\uace0\uc11c\uc5d0\uc11c \ucd9c\uc785\uad6d \uaddc\uc81c\uac00 \ud798\ub4e0 \uc778\uc0ac\uc5d0 \ub300\ud574\uc11c\ub3c4 \ucd9c\uae08\uae08\uc9c0\ub97c \ub0b4\ub838\ub358 \ub0a0\uc9dc\ub294 \uba87 \uc77c\uc774\ub77c \uae30\uc7ac\ub418\uc5b4 \uc788\ub098?", "paragraph": "\uc5ec\uc57c \uc758\uc6d0\ub4e4\uc740 \uc815\ubcf4\uae30\uad00\uc758 \ubd88\ubc95 \ub3c4\uac10\uccad \uc545\uc2b5\uc744 \ud55c\ubaa9\uc18c\ub9ac\ub85c \ube44\ud310\ud558\uba74\uc11c \uc815\ud655\ud55c \uc9c4\uc0c1\uaddc\uba85\uacfc \uad00\ub828 \ucc45\uc784\uc790 \ucc98\ubc8c, \uc7ac\ubc1c \ubc29\uc9c0 \ub300\ucc45 \ub4f1\uc744 \uc694\uad6c\ud588\ub2e4. \uad6d\uc815\uc6d0 \ubcf4\uace0\ub0b4\uc6a9\uc5d0\uc11c \uae40 \uc6d0\uc7a5\uc740 \"\uac80\ucc30 \ud611\uc870\ub97c \ud1b5\ud574 \uc7ac\ubbf8\uad50\ud3ec \ubc15\uc778\ud68c\uc528 \ub4f1 \ud575\uc2ec \uad00\ub828\uc790 20\uba85\uc5d0 \ub300\ud574 \uc9c0\ub09c\ub2ec 22\uc77c\ubd80\ud130 \ucd9c\uc785\uad6d \uaddc\uc81c \ub4f1 \uc870\uce58\ub97c \ucde8\ud588\uace0, \ucd9c\uc785\uad6d \uaddc\uc81c\uac00 \uc5b4\ub824\uc6b4 \uc8fc\uc694 \uc778\uc0ac\uc5d0 \ub300\ud574\uc11c\ub3c4 \uc9c0\ub09c\ub2ec 29\uc77c \ucd9c\uad6d\uae08\uc9c0\ud1a0\ub85d \ud588\ub2e4\"\uba70 \uadf8\uac04\uc758 \uc870\uc0ac \ub0b4\uc6a9\uc744 \ubcf4\uace0\ud588\ub2e4. \uadf8\ub7ec\uba74\uc11c \uc9c0\ub09c\ub2ec 29\uc77c \uac80\ucc30\uc5d0 \uad6c\uc18d\ub41c \ubc15\uc528\uc758 \uacbd\uc6b0 \uad6d\uc815\uc6d0\uc758 \ucd9c\uad6d\uc815\uc9c0 \uc870\uce58\uac00 \uc5c6\uc5b4 \uc774\ubbf8 \ucd9c\uad6d\ud588\ub2e4\uba74 \uad00\ub828 \uc218\uc0ac\uac00 \ubbf8\uad81\uc5d0 \ube60\uc84c\uc744 \uac83\uc774\ub77c\uace0 \uc790\ud3c9\ud588\ub2e4. \uae40 \uc6d0\uc7a5\uc740 \uc774\uc5b4 \"\ud575\uc2ec \uad00\ub828\uc790\uc778 \uacf5\uc6b4\uc601\uc528\uc758 \uc790\ud574\ub85c \uc870\uc0ac \uc77c\uc815\uc5d0 \ucc28\uc9c8\uc744 \ube5a\uace0 \uc788\uc73c\ub098 \uac80\ucc30\uacfc \uae34\ubc00\ud55c \ud611\uc870\ud558\uc5d0 \uc785\uc6d0 \uc911\uc778 \ubcd1\uc2e4\uc5d0\uc11c \ubbf8\ub9bc\ud300\uc758 \ubd88\ubc95\uac10\uccad \uc9c0\uc2dc \ubc0f \ub77c\uc778, \ub179\uc74c \ud14c\uc774\ud504 \uc720\ucd9c \uacbd\uc704 \ub4f1\uc5d0 \ub300\ud574 \uad11\ubc94\uc704\ud558\uac8c \uc870\uc0ac \uc911\"\uc774\ub77c\uace0 \uac15\uc870\ud588\ub2e4.", "answer": "29\uc77c", "sentence": "\uad6d\uc815\uc6d0 \ubcf4\uace0\ub0b4\uc6a9\uc5d0\uc11c \uae40 \uc6d0\uc7a5\uc740 \"\uac80\ucc30 \ud611\uc870\ub97c \ud1b5\ud574 \uc7ac\ubbf8\uad50\ud3ec \ubc15\uc778\ud68c\uc528 \ub4f1 \ud575\uc2ec \uad00\ub828\uc790 20\uba85\uc5d0 \ub300\ud574 \uc9c0\ub09c\ub2ec 22\uc77c\ubd80\ud130 \ucd9c\uc785\uad6d \uaddc\uc81c \ub4f1 \uc870\uce58\ub97c \ucde8\ud588\uace0, \ucd9c\uc785\uad6d \uaddc\uc81c\uac00 \uc5b4\ub824\uc6b4 \uc8fc\uc694 \uc778\uc0ac\uc5d0 \ub300\ud574\uc11c\ub3c4 \uc9c0\ub09c\ub2ec 29\uc77c \ucd9c\uad6d\uae08\uc9c0\ud1a0\ub85d \ud588\ub2e4\"\uba70 \uadf8\uac04\uc758 \uc870\uc0ac \ub0b4\uc6a9\uc744 \ubcf4\uace0\ud588\ub2e4.", "paragraph_sentence": "\uc5ec\uc57c \uc758\uc6d0\ub4e4\uc740 \uc815\ubcf4\uae30\uad00\uc758 \ubd88\ubc95 \ub3c4\uac10\uccad \uc545\uc2b5\uc744 \ud55c\ubaa9\uc18c\ub9ac\ub85c \ube44\ud310\ud558\uba74\uc11c \uc815\ud655\ud55c \uc9c4\uc0c1\uaddc\uba85\uacfc \uad00\ub828 \ucc45\uc784\uc790 \ucc98\ubc8c, \uc7ac\ubc1c \ubc29\uc9c0 \ub300\ucc45 \ub4f1\uc744 \uc694\uad6c\ud588\ub2e4. \uad6d\uc815\uc6d0 \ubcf4\uace0\ub0b4\uc6a9\uc5d0\uc11c \uae40 \uc6d0\uc7a5\uc740 \"\uac80\ucc30 \ud611\uc870\ub97c \ud1b5\ud574 \uc7ac\ubbf8\uad50\ud3ec \ubc15\uc778\ud68c\uc528 \ub4f1 \ud575\uc2ec \uad00\ub828\uc790 20\uba85\uc5d0 \ub300\ud574 \uc9c0\ub09c\ub2ec 22\uc77c\ubd80\ud130 \ucd9c\uc785\uad6d \uaddc\uc81c \ub4f1 \uc870\uce58\ub97c \ucde8\ud588\uace0, \ucd9c\uc785\uad6d \uaddc\uc81c\uac00 \uc5b4\ub824\uc6b4 \uc8fc\uc694 \uc778\uc0ac\uc5d0 \ub300\ud574\uc11c\ub3c4 \uc9c0\ub09c\ub2ec 29\uc77c \ucd9c\uad6d\uae08\uc9c0\ud1a0\ub85d \ud588\ub2e4\"\uba70 \uadf8\uac04\uc758 \uc870\uc0ac \ub0b4\uc6a9\uc744 \ubcf4\uace0\ud588\ub2e4. \uadf8\ub7ec\uba74\uc11c \uc9c0\ub09c\ub2ec 29\uc77c \uac80\ucc30\uc5d0 \uad6c\uc18d\ub41c \ubc15\uc528\uc758 \uacbd\uc6b0 \uad6d\uc815\uc6d0\uc758 \ucd9c\uad6d\uc815\uc9c0 \uc870\uce58\uac00 \uc5c6\uc5b4 \uc774\ubbf8 \ucd9c\uad6d\ud588\ub2e4\uba74 \uad00\ub828 \uc218\uc0ac\uac00 \ubbf8\uad81\uc5d0 \ube60\uc84c\uc744 \uac83\uc774\ub77c\uace0 \uc790\ud3c9\ud588\ub2e4. \uae40 \uc6d0\uc7a5\uc740 \uc774\uc5b4 \"\ud575\uc2ec \uad00\ub828\uc790\uc778 \uacf5\uc6b4\uc601\uc528\uc758 \uc790\ud574\ub85c \uc870\uc0ac \uc77c\uc815\uc5d0 \ucc28\uc9c8\uc744 \ube5a\uace0 \uc788\uc73c\ub098 \uac80\ucc30\uacfc \uae34\ubc00\ud55c \ud611\uc870\ud558\uc5d0 \uc785\uc6d0 \uc911\uc778 \ubcd1\uc2e4\uc5d0\uc11c \ubbf8\ub9bc\ud300\uc758 \ubd88\ubc95\uac10\uccad \uc9c0\uc2dc \ubc0f \ub77c\uc778, \ub179\uc74c \ud14c\uc774\ud504 \uc720\ucd9c \uacbd\uc704 \ub4f1\uc5d0 \ub300\ud574 \uad11\ubc94\uc704\ud558\uac8c \uc870\uc0ac \uc911\"\uc774\ub77c\uace0 \uac15\uc870\ud588\ub2e4.", "paragraph_answer": "\uc5ec\uc57c \uc758\uc6d0\ub4e4\uc740 \uc815\ubcf4\uae30\uad00\uc758 \ubd88\ubc95 \ub3c4\uac10\uccad \uc545\uc2b5\uc744 \ud55c\ubaa9\uc18c\ub9ac\ub85c \ube44\ud310\ud558\uba74\uc11c \uc815\ud655\ud55c \uc9c4\uc0c1\uaddc\uba85\uacfc \uad00\ub828 \ucc45\uc784\uc790 \ucc98\ubc8c, \uc7ac\ubc1c \ubc29\uc9c0 \ub300\ucc45 \ub4f1\uc744 \uc694\uad6c\ud588\ub2e4. \uad6d\uc815\uc6d0 \ubcf4\uace0\ub0b4\uc6a9\uc5d0\uc11c \uae40 \uc6d0\uc7a5\uc740 \"\uac80\ucc30 \ud611\uc870\ub97c \ud1b5\ud574 \uc7ac\ubbf8\uad50\ud3ec \ubc15\uc778\ud68c\uc528 \ub4f1 \ud575\uc2ec \uad00\ub828\uc790 20\uba85\uc5d0 \ub300\ud574 \uc9c0\ub09c\ub2ec 22\uc77c\ubd80\ud130 \ucd9c\uc785\uad6d \uaddc\uc81c \ub4f1 \uc870\uce58\ub97c \ucde8\ud588\uace0, \ucd9c\uc785\uad6d \uaddc\uc81c\uac00 \uc5b4\ub824\uc6b4 \uc8fc\uc694 \uc778\uc0ac\uc5d0 \ub300\ud574\uc11c\ub3c4 \uc9c0\ub09c\ub2ec 29\uc77c \ucd9c\uad6d\uae08\uc9c0\ud1a0\ub85d \ud588\ub2e4\"\uba70 \uadf8\uac04\uc758 \uc870\uc0ac \ub0b4\uc6a9\uc744 \ubcf4\uace0\ud588\ub2e4. \uadf8\ub7ec\uba74\uc11c \uc9c0\ub09c\ub2ec 29\uc77c \uac80\ucc30\uc5d0 \uad6c\uc18d\ub41c \ubc15\uc528\uc758 \uacbd\uc6b0 \uad6d\uc815\uc6d0\uc758 \ucd9c\uad6d\uc815\uc9c0 \uc870\uce58\uac00 \uc5c6\uc5b4 \uc774\ubbf8 \ucd9c\uad6d\ud588\ub2e4\uba74 \uad00\ub828 \uc218\uc0ac\uac00 \ubbf8\uad81\uc5d0 \ube60\uc84c\uc744 \uac83\uc774\ub77c\uace0 \uc790\ud3c9\ud588\ub2e4. \uae40 \uc6d0\uc7a5\uc740 \uc774\uc5b4 \"\ud575\uc2ec \uad00\ub828\uc790\uc778 \uacf5\uc6b4\uc601\uc528\uc758 \uc790\ud574\ub85c \uc870\uc0ac \uc77c\uc815\uc5d0 \ucc28\uc9c8\uc744 \ube5a\uace0 \uc788\uc73c\ub098 \uac80\ucc30\uacfc \uae34\ubc00\ud55c \ud611\uc870\ud558\uc5d0 \uc785\uc6d0 \uc911\uc778 \ubcd1\uc2e4\uc5d0\uc11c \ubbf8\ub9bc\ud300\uc758 \ubd88\ubc95\uac10\uccad \uc9c0\uc2dc \ubc0f \ub77c\uc778, \ub179\uc74c \ud14c\uc774\ud504 \uc720\ucd9c \uacbd\uc704 \ub4f1\uc5d0 \ub300\ud574 \uad11\ubc94\uc704\ud558\uac8c \uc870\uc0ac \uc911\"\uc774\ub77c\uace0 \uac15\uc870\ud588\ub2e4.", "sentence_answer": "\uad6d\uc815\uc6d0 \ubcf4\uace0\ub0b4\uc6a9\uc5d0\uc11c \uae40 \uc6d0\uc7a5\uc740 \"\uac80\ucc30 \ud611\uc870\ub97c \ud1b5\ud574 \uc7ac\ubbf8\uad50\ud3ec \ubc15\uc778\ud68c\uc528 \ub4f1 \ud575\uc2ec \uad00\ub828\uc790 20\uba85\uc5d0 \ub300\ud574 \uc9c0\ub09c\ub2ec 22\uc77c\ubd80\ud130 \ucd9c\uc785\uad6d \uaddc\uc81c \ub4f1 \uc870\uce58\ub97c \ucde8\ud588\uace0, \ucd9c\uc785\uad6d \uaddc\uc81c\uac00 \uc5b4\ub824\uc6b4 \uc8fc\uc694 \uc778\uc0ac\uc5d0 \ub300\ud574\uc11c\ub3c4 \uc9c0\ub09c\ub2ec 29\uc77c \ucd9c\uad6d\uae08\uc9c0\ud1a0\ub85d \ud588\ub2e4\"\uba70 \uadf8\uac04\uc758 \uc870\uc0ac \ub0b4\uc6a9\uc744 \ubcf4\uace0\ud588\ub2e4.", "paragraph_id": "6577692-1-1", "paragraph_question": "question: \ubd88\ubc95 \ub3c4\uccad\uc5d0 \ub300\ud55c \uad6d\uc815\uc6d0 \ubcf4\uace0\uc11c\uc5d0\uc11c \ucd9c\uc785\uad6d \uaddc\uc81c\uac00 \ud798\ub4e0 \uc778\uc0ac\uc5d0 \ub300\ud574\uc11c\ub3c4 \ucd9c\uae08\uae08\uc9c0\ub97c \ub0b4\ub838\ub358 \ub0a0\uc9dc\ub294 \uba87 \uc77c\uc774\ub77c \uae30\uc7ac\ub418\uc5b4 \uc788\ub098?, context: \uc5ec\uc57c \uc758\uc6d0\ub4e4\uc740 \uc815\ubcf4\uae30\uad00\uc758 \ubd88\ubc95 \ub3c4\uac10\uccad \uc545\uc2b5\uc744 \ud55c\ubaa9\uc18c\ub9ac\ub85c \ube44\ud310\ud558\uba74\uc11c \uc815\ud655\ud55c \uc9c4\uc0c1\uaddc\uba85\uacfc \uad00\ub828 \ucc45\uc784\uc790 \ucc98\ubc8c, \uc7ac\ubc1c \ubc29\uc9c0 \ub300\ucc45 \ub4f1\uc744 \uc694\uad6c\ud588\ub2e4. \uad6d\uc815\uc6d0 \ubcf4\uace0\ub0b4\uc6a9\uc5d0\uc11c \uae40 \uc6d0\uc7a5\uc740 \"\uac80\ucc30 \ud611\uc870\ub97c \ud1b5\ud574 \uc7ac\ubbf8\uad50\ud3ec \ubc15\uc778\ud68c\uc528 \ub4f1 \ud575\uc2ec \uad00\ub828\uc790 20\uba85\uc5d0 \ub300\ud574 \uc9c0\ub09c\ub2ec 22\uc77c\ubd80\ud130 \ucd9c\uc785\uad6d \uaddc\uc81c \ub4f1 \uc870\uce58\ub97c \ucde8\ud588\uace0, \ucd9c\uc785\uad6d \uaddc\uc81c\uac00 \uc5b4\ub824\uc6b4 \uc8fc\uc694 \uc778\uc0ac\uc5d0 \ub300\ud574\uc11c\ub3c4 \uc9c0\ub09c\ub2ec 29\uc77c \ucd9c\uad6d\uae08\uc9c0\ud1a0\ub85d \ud588\ub2e4\"\uba70 \uadf8\uac04\uc758 \uc870\uc0ac \ub0b4\uc6a9\uc744 \ubcf4\uace0\ud588\ub2e4. \uadf8\ub7ec\uba74\uc11c \uc9c0\ub09c\ub2ec 29\uc77c \uac80\ucc30\uc5d0 \uad6c\uc18d\ub41c \ubc15\uc528\uc758 \uacbd\uc6b0 \uad6d\uc815\uc6d0\uc758 \ucd9c\uad6d\uc815\uc9c0 \uc870\uce58\uac00 \uc5c6\uc5b4 \uc774\ubbf8 \ucd9c\uad6d\ud588\ub2e4\uba74 \uad00\ub828 \uc218\uc0ac\uac00 \ubbf8\uad81\uc5d0 \ube60\uc84c\uc744 \uac83\uc774\ub77c\uace0 \uc790\ud3c9\ud588\ub2e4. \uae40 \uc6d0\uc7a5\uc740 \uc774\uc5b4 \"\ud575\uc2ec \uad00\ub828\uc790\uc778 \uacf5\uc6b4\uc601\uc528\uc758 \uc790\ud574\ub85c \uc870\uc0ac \uc77c\uc815\uc5d0 \ucc28\uc9c8\uc744 \ube5a\uace0 \uc788\uc73c\ub098 \uac80\ucc30\uacfc \uae34\ubc00\ud55c \ud611\uc870\ud558\uc5d0 \uc785\uc6d0 \uc911\uc778 \ubcd1\uc2e4\uc5d0\uc11c \ubbf8\ub9bc\ud300\uc758 \ubd88\ubc95\uac10\uccad \uc9c0\uc2dc \ubc0f \ub77c\uc778, \ub179\uc74c \ud14c\uc774\ud504 \uc720\ucd9c \uacbd\uc704 \ub4f1\uc5d0 \ub300\ud574 \uad11\ubc94\uc704\ud558\uac8c \uc870\uc0ac \uc911\"\uc774\ub77c\uace0 \uac15\uc870\ud588\ub2e4."} {"question": "\uc1a1\ud61c\uad50\uac00 \uc544\uc2dc\uc544 \ubc30\uc6b0 \ucd5c\ucd08\ub85c \uacc4\uc57d\uc744 \ub9fa\uc740 \uc5d0\uc774\uc804\uc2dc\uac00 \uc18c\uc7ac\ud55c \uacf3\uc740?", "paragraph": "\uc1a1\ud61c\uad50\ub294 2011\ub144 \uc774\uc815\ud5a5 \uac10\ub3c5\uc758 \uc601\ud654 \u300a\uc624\ub298\u300b\ub85c \ub2e4\ud050\uba58\ud130\ub9ac PD \ub2e4\ud61c \uc5ed\uc744 \ub9e1\uc544 4\ub144 \ub9cc\uc5d0 \uad6d\ub0b4 \uc2a4\ud06c\ub9b0\uc5d0 \ucef4\ubc31\ud588\ub2e4. \uc774 \uc791\ud488\uc740 \uc57d\ud63c\uc790\ub97c \uc8fd\uc778 17\uc138 \uc18c\ub144\uc744 \uc6a9\uc11c\ud55c \ub2e4\ud050\uba58\ud130\ub9ac PD \ub2e4\ud61c\uc640 \uac00\uc871\uc5d0\uac8c \ubc84\ub9bc\ubc1b\uc740 \ucc9c\uc7ac \uc18c\ub140 \uc9c0\ubbfc(\ub0a8\uc9c0\ud604 \ubd84)\uc758 \uc0c1\ucc98\ubc1b\uc740 \ub0b4\uba74\uc744 \uadf8\ub9b0 \uc791\ud488\uc73c\ub85c 1\ub144 \ud6c4 \uc790\uc2e0\uc758 \uc6a9\uc11c\uac00 \ub73b\ud558\uc9c0 \uc54a\uc740 \uacb0\uacfc\ub97c \ubd88\ub7ec\uc624\uba74\uc11c \ub2e4\ud61c\uac00 \uacaa\ub294 \ud63c\ub780\uacfc \uc2ac\ud514, \uadf8\ub9ac\uace0 \uadf8 \ub05d\uc5d0\uc11c \ucc3e\uc544\ub0b8 \ucc2c\ub780\ud55c \uac10\ub3d9\uc744 \uadf8\ub824\ub0c8\ub2e4\ub294 \ud3c9\uc744 \ubc1b\uc558\ub2e4. \uc1a1\ud61c\uad50\ub294 \ub9c8\uc9c0\ub9c9 \uae30\ub150\ucd2c\uc601\uc744 \ub9c8\uce5c \ud6c4 \u201c\u2018\uc624\ub298\u2019\uc740 \ub0b4 \uc5f0\uae30 \uc778\uc0dd\uc758 \ud130\ub2dd\ud3ec\uc778\ud2b8\uac00 \ub418\ub294 \uc791\ud488\uc774\ub2e4.\u201d\ub77c\uace0 \ub9d0\ud588\ub2e4. \ub610\ud55c, \uadf8\ub140\uc758 \uc12c\uc138\ud55c \uac10\uc815\uc5f0\uae30\uc640 \uc808\uc81c\ub41c \uc2ec\ub9ac\uc5f0\uae30\ub294 \ud3c9\ub2e8\uc758 \ud638\ud3c9\uc744 \ubc1b\uc544 2011 \uc62c\ud574\uc758 \uc5ec\uc131\uc601\ud654\uc778 \uc2dc\uc0c1\uc2dd\uc5d0\uc11c \uc62c\ud574\uc758 \uc5ec\uc131\uc601\ud654\uc778 \uc5f0\uae30\uc0c1\uc744 \uc218\uc0c1\ud588\ub2e4. \uc218\uc0c1 \uc18c\uac10\uc5d0\uc11c \uc1a1\ud61c\uad50\ub294 \u201c\ub370\ubdd4\ud55c\uc9c0\ub294 \uc624\ub798\ub410\ub294\ub370 \uc601\ud654 \uc5f0\uae30\uc0c1\uc740 \ucc98\uc74c \ubc1b\ub294\ub2e4. \uc815\ub9d0 \uac10\uc0ac\ub4dc\ub9b0\ub2e4. \uc88b\uc740 \uc601\ud654\uc640 \uce90\ub9ad\ud130\ub97c \ub9cc\ub0a0 \uc218 \uc788\uac8c \ud574\uc900 \uc774\uc815\ud5a5 \uac10\ub3c5\ub2d8\uc774 \uacc4\uc168\ub354\ub77c\uba74 \ub354 \uc88b\uc558\uc744 \ud150\ub370 \uc544\uc27d\ub2e4\u201d\uc5d0 \uc774\uc5b4 \u201c\uc601\ud654 \uc5f0\uae30\uc0c1\uc744 \ubc1b\uc740 \uc9c0\uae08 \uc774 \uc21c\uac04\ub3c4 \ub108\ubb34 \ub5a8\ub9ac\ub294\ub370, \uccad\ub8e1 \uc601\ud654\uc81c\ub77c\uba74 \uc5b4\ub5a8\uc9c0 \uc0c1\uc0c1\uc870\ucc28 \ud558\uc9c0 \ubabb\ud558\uaca0\ub2e4.\u201d\uba70 \uac10\uaca9\uc758 \ub208\ubb3c\uc744 \ubcf4\uc600\uace0, \ub9c8\uc9c0\ub9c9\uc73c\ub85c \u201c\uc601\ud654 \u2018\uc624\ub298\u2019\uc774 \ub354 \uc88b\uc740 \ud658\uacbd\uc5d0\uc11c \uad00\uac1d\ub4e4\uacfc \ub9cc\ub0a0 \uc218 \uc788\uc5c8\ub2e4\uba74 \uc88b\uc558\uc744\ud150\ub370 \uadf8\ub7ec\uc9c0 \ubabb\ud574 \uc544\uc27d\ub2e4. \uc8fc\uc704 \ubaa8\ub4e0 \uc9c0\uc778\ubd84\ub4e4\uaed8 \uac10\uc0ac\ub4dc\ub9ac\uace0 \ud56d\uc0c1 \uc5f4\uc2ec\ud788 \ud574\uc11c \ub354 \uc88b\uc740 \uc5f0\uae30\ub97c \ubcf4\uc5ec\ub4dc\ub9ac\uaca0\ub2e4.\u201d\uace0 \ub9d0\ud588\ub2e4. \uac19\uc740 \ud574, \uadf8\ub140\ub294 \uc544\uc2dc\uc544 \ubc30\uc6b0 \ucd5c\ucd08\ub85c \ud504\ub791\uc2a4 \ucd5c\ub300 \uc5d0\uc774\uc804\uc2dc\uc640 \uacc4\uc57d\uc744 \uccb4\uacb0\ud558\uba70 \uc720\ub7fd \uc9c4\ucd9c\uc5d0 \uae30\ub300\ub97c \ubaa8\uc558\ub2e4. \uc1a1\ud61c\uad50\ub294 \ub2e8\uc21c\ud55c 20\ub300 \uccad\ucd98\uc2a4\ud0c0\uc5d0\uc11c \ub9cc\uc871\ud558\uc9c0 \uc54a\uace0 \uc5ec\ubc30\uc6b0\ub85c\uc11c \uc790\uc2e0\uc774 \ud45c\ud604\ud560 \uc218 \uc788\ub294 \uce90\ub9ad\ud130\uc5d0 \ub300\ud55c \uac70\ub4ed \uace0\ubbfc\ud558\uba70 \ubc30\uc6b0\ub85c\uc11c \uc131\uc7a5\ud558\ub294 \ubaa8\uc2b5\uc744 \ubcf4\uc5ec\uc92c\ub2e4.", "answer": "\ud504\ub791\uc2a4", "sentence": "\uac19\uc740 \ud574, \uadf8\ub140\ub294 \uc544\uc2dc\uc544 \ubc30\uc6b0 \ucd5c\ucd08\ub85c \ud504\ub791\uc2a4 \ucd5c\ub300 \uc5d0\uc774\uc804\uc2dc\uc640 \uacc4\uc57d\uc744 \uccb4\uacb0\ud558\uba70 \uc720\ub7fd \uc9c4\ucd9c\uc5d0 \uae30\ub300\ub97c \ubaa8\uc558\ub2e4.", "paragraph_sentence": "\uc1a1\ud61c\uad50\ub294 2011\ub144 \uc774\uc815\ud5a5 \uac10\ub3c5\uc758 \uc601\ud654 \u300a\uc624\ub298\u300b\ub85c \ub2e4\ud050\uba58\ud130\ub9ac PD \ub2e4\ud61c \uc5ed\uc744 \ub9e1\uc544 4\ub144 \ub9cc\uc5d0 \uad6d\ub0b4 \uc2a4\ud06c\ub9b0\uc5d0 \ucef4\ubc31\ud588\ub2e4. \uc774 \uc791\ud488\uc740 \uc57d\ud63c\uc790\ub97c \uc8fd\uc778 17\uc138 \uc18c\ub144\uc744 \uc6a9\uc11c\ud55c \ub2e4\ud050\uba58\ud130\ub9ac PD \ub2e4\ud61c\uc640 \uac00\uc871\uc5d0\uac8c \ubc84\ub9bc\ubc1b\uc740 \ucc9c\uc7ac \uc18c\ub140 \uc9c0\ubbfc(\ub0a8\uc9c0\ud604 \ubd84)\uc758 \uc0c1\ucc98\ubc1b\uc740 \ub0b4\uba74\uc744 \uadf8\ub9b0 \uc791\ud488\uc73c\ub85c 1\ub144 \ud6c4 \uc790\uc2e0\uc758 \uc6a9\uc11c\uac00 \ub73b\ud558\uc9c0 \uc54a\uc740 \uacb0\uacfc\ub97c \ubd88\ub7ec\uc624\uba74\uc11c \ub2e4\ud61c\uac00 \uacaa\ub294 \ud63c\ub780\uacfc \uc2ac\ud514, \uadf8\ub9ac\uace0 \uadf8 \ub05d\uc5d0\uc11c \ucc3e\uc544\ub0b8 \ucc2c\ub780\ud55c \uac10\ub3d9\uc744 \uadf8\ub824\ub0c8\ub2e4\ub294 \ud3c9\uc744 \ubc1b\uc558\ub2e4. \uc1a1\ud61c\uad50\ub294 \ub9c8\uc9c0\ub9c9 \uae30\ub150\ucd2c\uc601\uc744 \ub9c8\uce5c \ud6c4 \u201c\u2018\uc624\ub298\u2019\uc740 \ub0b4 \uc5f0\uae30 \uc778\uc0dd\uc758 \ud130\ub2dd\ud3ec\uc778\ud2b8\uac00 \ub418\ub294 \uc791\ud488\uc774\ub2e4. \u201d\ub77c\uace0 \ub9d0\ud588\ub2e4. \ub610\ud55c, \uadf8\ub140\uc758 \uc12c\uc138\ud55c \uac10\uc815\uc5f0\uae30\uc640 \uc808\uc81c\ub41c \uc2ec\ub9ac\uc5f0\uae30\ub294 \ud3c9\ub2e8\uc758 \ud638\ud3c9\uc744 \ubc1b\uc544 2011 \uc62c\ud574\uc758 \uc5ec\uc131\uc601\ud654\uc778 \uc2dc\uc0c1\uc2dd\uc5d0\uc11c \uc62c\ud574\uc758 \uc5ec\uc131\uc601\ud654\uc778 \uc5f0\uae30\uc0c1\uc744 \uc218\uc0c1\ud588\ub2e4. \uc218\uc0c1 \uc18c\uac10\uc5d0\uc11c \uc1a1\ud61c\uad50\ub294 \u201c\ub370\ubdd4\ud55c\uc9c0\ub294 \uc624\ub798\ub410\ub294\ub370 \uc601\ud654 \uc5f0\uae30\uc0c1\uc740 \ucc98\uc74c \ubc1b\ub294\ub2e4. \uc815\ub9d0 \uac10\uc0ac\ub4dc\ub9b0\ub2e4. \uc88b\uc740 \uc601\ud654\uc640 \uce90\ub9ad\ud130\ub97c \ub9cc\ub0a0 \uc218 \uc788\uac8c \ud574\uc900 \uc774\uc815\ud5a5 \uac10\ub3c5\ub2d8\uc774 \uacc4\uc168\ub354\ub77c\uba74 \ub354 \uc88b\uc558\uc744 \ud150\ub370 \uc544\uc27d\ub2e4\u201d\uc5d0 \uc774\uc5b4 \u201c\uc601\ud654 \uc5f0\uae30\uc0c1\uc744 \ubc1b\uc740 \uc9c0\uae08 \uc774 \uc21c\uac04\ub3c4 \ub108\ubb34 \ub5a8\ub9ac\ub294\ub370, \uccad\ub8e1 \uc601\ud654\uc81c\ub77c\uba74 \uc5b4\ub5a8\uc9c0 \uc0c1\uc0c1\uc870\ucc28 \ud558\uc9c0 \ubabb\ud558\uaca0\ub2e4. \u201d\uba70 \uac10\uaca9\uc758 \ub208\ubb3c\uc744 \ubcf4\uc600\uace0, \ub9c8\uc9c0\ub9c9\uc73c\ub85c \u201c\uc601\ud654 \u2018\uc624\ub298\u2019\uc774 \ub354 \uc88b\uc740 \ud658\uacbd\uc5d0\uc11c \uad00\uac1d\ub4e4\uacfc \ub9cc\ub0a0 \uc218 \uc788\uc5c8\ub2e4\uba74 \uc88b\uc558\uc744\ud150\ub370 \uadf8\ub7ec\uc9c0 \ubabb\ud574 \uc544\uc27d\ub2e4. \uc8fc\uc704 \ubaa8\ub4e0 \uc9c0\uc778\ubd84\ub4e4\uaed8 \uac10\uc0ac\ub4dc\ub9ac\uace0 \ud56d\uc0c1 \uc5f4\uc2ec\ud788 \ud574\uc11c \ub354 \uc88b\uc740 \uc5f0\uae30\ub97c \ubcf4\uc5ec\ub4dc\ub9ac\uaca0\ub2e4. \u201d\uace0 \ub9d0\ud588\ub2e4. \uac19\uc740 \ud574, \uadf8\ub140\ub294 \uc544\uc2dc\uc544 \ubc30\uc6b0 \ucd5c\ucd08\ub85c \ud504\ub791\uc2a4 \ucd5c\ub300 \uc5d0\uc774\uc804\uc2dc\uc640 \uacc4\uc57d\uc744 \uccb4\uacb0\ud558\uba70 \uc720\ub7fd \uc9c4\ucd9c\uc5d0 \uae30\ub300\ub97c \ubaa8\uc558\ub2e4. \uc1a1\ud61c\uad50\ub294 \ub2e8\uc21c\ud55c 20\ub300 \uccad\ucd98\uc2a4\ud0c0\uc5d0\uc11c \ub9cc\uc871\ud558\uc9c0 \uc54a\uace0 \uc5ec\ubc30\uc6b0\ub85c\uc11c \uc790\uc2e0\uc774 \ud45c\ud604\ud560 \uc218 \uc788\ub294 \uce90\ub9ad\ud130\uc5d0 \ub300\ud55c \uac70\ub4ed \uace0\ubbfc\ud558\uba70 \ubc30\uc6b0\ub85c\uc11c \uc131\uc7a5\ud558\ub294 \ubaa8\uc2b5\uc744 \ubcf4\uc5ec\uc92c\ub2e4.", "paragraph_answer": "\uc1a1\ud61c\uad50\ub294 2011\ub144 \uc774\uc815\ud5a5 \uac10\ub3c5\uc758 \uc601\ud654 \u300a\uc624\ub298\u300b\ub85c \ub2e4\ud050\uba58\ud130\ub9ac PD \ub2e4\ud61c \uc5ed\uc744 \ub9e1\uc544 4\ub144 \ub9cc\uc5d0 \uad6d\ub0b4 \uc2a4\ud06c\ub9b0\uc5d0 \ucef4\ubc31\ud588\ub2e4. \uc774 \uc791\ud488\uc740 \uc57d\ud63c\uc790\ub97c \uc8fd\uc778 17\uc138 \uc18c\ub144\uc744 \uc6a9\uc11c\ud55c \ub2e4\ud050\uba58\ud130\ub9ac PD \ub2e4\ud61c\uc640 \uac00\uc871\uc5d0\uac8c \ubc84\ub9bc\ubc1b\uc740 \ucc9c\uc7ac \uc18c\ub140 \uc9c0\ubbfc(\ub0a8\uc9c0\ud604 \ubd84)\uc758 \uc0c1\ucc98\ubc1b\uc740 \ub0b4\uba74\uc744 \uadf8\ub9b0 \uc791\ud488\uc73c\ub85c 1\ub144 \ud6c4 \uc790\uc2e0\uc758 \uc6a9\uc11c\uac00 \ub73b\ud558\uc9c0 \uc54a\uc740 \uacb0\uacfc\ub97c \ubd88\ub7ec\uc624\uba74\uc11c \ub2e4\ud61c\uac00 \uacaa\ub294 \ud63c\ub780\uacfc \uc2ac\ud514, \uadf8\ub9ac\uace0 \uadf8 \ub05d\uc5d0\uc11c \ucc3e\uc544\ub0b8 \ucc2c\ub780\ud55c \uac10\ub3d9\uc744 \uadf8\ub824\ub0c8\ub2e4\ub294 \ud3c9\uc744 \ubc1b\uc558\ub2e4. \uc1a1\ud61c\uad50\ub294 \ub9c8\uc9c0\ub9c9 \uae30\ub150\ucd2c\uc601\uc744 \ub9c8\uce5c \ud6c4 \u201c\u2018\uc624\ub298\u2019\uc740 \ub0b4 \uc5f0\uae30 \uc778\uc0dd\uc758 \ud130\ub2dd\ud3ec\uc778\ud2b8\uac00 \ub418\ub294 \uc791\ud488\uc774\ub2e4.\u201d\ub77c\uace0 \ub9d0\ud588\ub2e4. \ub610\ud55c, \uadf8\ub140\uc758 \uc12c\uc138\ud55c \uac10\uc815\uc5f0\uae30\uc640 \uc808\uc81c\ub41c \uc2ec\ub9ac\uc5f0\uae30\ub294 \ud3c9\ub2e8\uc758 \ud638\ud3c9\uc744 \ubc1b\uc544 2011 \uc62c\ud574\uc758 \uc5ec\uc131\uc601\ud654\uc778 \uc2dc\uc0c1\uc2dd\uc5d0\uc11c \uc62c\ud574\uc758 \uc5ec\uc131\uc601\ud654\uc778 \uc5f0\uae30\uc0c1\uc744 \uc218\uc0c1\ud588\ub2e4. \uc218\uc0c1 \uc18c\uac10\uc5d0\uc11c \uc1a1\ud61c\uad50\ub294 \u201c\ub370\ubdd4\ud55c\uc9c0\ub294 \uc624\ub798\ub410\ub294\ub370 \uc601\ud654 \uc5f0\uae30\uc0c1\uc740 \ucc98\uc74c \ubc1b\ub294\ub2e4. \uc815\ub9d0 \uac10\uc0ac\ub4dc\ub9b0\ub2e4. \uc88b\uc740 \uc601\ud654\uc640 \uce90\ub9ad\ud130\ub97c \ub9cc\ub0a0 \uc218 \uc788\uac8c \ud574\uc900 \uc774\uc815\ud5a5 \uac10\ub3c5\ub2d8\uc774 \uacc4\uc168\ub354\ub77c\uba74 \ub354 \uc88b\uc558\uc744 \ud150\ub370 \uc544\uc27d\ub2e4\u201d\uc5d0 \uc774\uc5b4 \u201c\uc601\ud654 \uc5f0\uae30\uc0c1\uc744 \ubc1b\uc740 \uc9c0\uae08 \uc774 \uc21c\uac04\ub3c4 \ub108\ubb34 \ub5a8\ub9ac\ub294\ub370, \uccad\ub8e1 \uc601\ud654\uc81c\ub77c\uba74 \uc5b4\ub5a8\uc9c0 \uc0c1\uc0c1\uc870\ucc28 \ud558\uc9c0 \ubabb\ud558\uaca0\ub2e4.\u201d\uba70 \uac10\uaca9\uc758 \ub208\ubb3c\uc744 \ubcf4\uc600\uace0, \ub9c8\uc9c0\ub9c9\uc73c\ub85c \u201c\uc601\ud654 \u2018\uc624\ub298\u2019\uc774 \ub354 \uc88b\uc740 \ud658\uacbd\uc5d0\uc11c \uad00\uac1d\ub4e4\uacfc \ub9cc\ub0a0 \uc218 \uc788\uc5c8\ub2e4\uba74 \uc88b\uc558\uc744\ud150\ub370 \uadf8\ub7ec\uc9c0 \ubabb\ud574 \uc544\uc27d\ub2e4. \uc8fc\uc704 \ubaa8\ub4e0 \uc9c0\uc778\ubd84\ub4e4\uaed8 \uac10\uc0ac\ub4dc\ub9ac\uace0 \ud56d\uc0c1 \uc5f4\uc2ec\ud788 \ud574\uc11c \ub354 \uc88b\uc740 \uc5f0\uae30\ub97c \ubcf4\uc5ec\ub4dc\ub9ac\uaca0\ub2e4.\u201d\uace0 \ub9d0\ud588\ub2e4. \uac19\uc740 \ud574, \uadf8\ub140\ub294 \uc544\uc2dc\uc544 \ubc30\uc6b0 \ucd5c\ucd08\ub85c \ud504\ub791\uc2a4 \ucd5c\ub300 \uc5d0\uc774\uc804\uc2dc\uc640 \uacc4\uc57d\uc744 \uccb4\uacb0\ud558\uba70 \uc720\ub7fd \uc9c4\ucd9c\uc5d0 \uae30\ub300\ub97c \ubaa8\uc558\ub2e4. \uc1a1\ud61c\uad50\ub294 \ub2e8\uc21c\ud55c 20\ub300 \uccad\ucd98\uc2a4\ud0c0\uc5d0\uc11c \ub9cc\uc871\ud558\uc9c0 \uc54a\uace0 \uc5ec\ubc30\uc6b0\ub85c\uc11c \uc790\uc2e0\uc774 \ud45c\ud604\ud560 \uc218 \uc788\ub294 \uce90\ub9ad\ud130\uc5d0 \ub300\ud55c \uac70\ub4ed \uace0\ubbfc\ud558\uba70 \ubc30\uc6b0\ub85c\uc11c \uc131\uc7a5\ud558\ub294 \ubaa8\uc2b5\uc744 \ubcf4\uc5ec\uc92c\ub2e4.", "sentence_answer": "\uac19\uc740 \ud574, \uadf8\ub140\ub294 \uc544\uc2dc\uc544 \ubc30\uc6b0 \ucd5c\ucd08\ub85c \ud504\ub791\uc2a4 \ucd5c\ub300 \uc5d0\uc774\uc804\uc2dc\uc640 \uacc4\uc57d\uc744 \uccb4\uacb0\ud558\uba70 \uc720\ub7fd \uc9c4\ucd9c\uc5d0 \uae30\ub300\ub97c \ubaa8\uc558\ub2e4.", "paragraph_id": "6518513-4-1", "paragraph_question": "question: \uc1a1\ud61c\uad50\uac00 \uc544\uc2dc\uc544 \ubc30\uc6b0 \ucd5c\ucd08\ub85c \uacc4\uc57d\uc744 \ub9fa\uc740 \uc5d0\uc774\uc804\uc2dc\uac00 \uc18c\uc7ac\ud55c \uacf3\uc740?, context: \uc1a1\ud61c\uad50\ub294 2011\ub144 \uc774\uc815\ud5a5 \uac10\ub3c5\uc758 \uc601\ud654 \u300a\uc624\ub298\u300b\ub85c \ub2e4\ud050\uba58\ud130\ub9ac PD \ub2e4\ud61c \uc5ed\uc744 \ub9e1\uc544 4\ub144 \ub9cc\uc5d0 \uad6d\ub0b4 \uc2a4\ud06c\ub9b0\uc5d0 \ucef4\ubc31\ud588\ub2e4. \uc774 \uc791\ud488\uc740 \uc57d\ud63c\uc790\ub97c \uc8fd\uc778 17\uc138 \uc18c\ub144\uc744 \uc6a9\uc11c\ud55c \ub2e4\ud050\uba58\ud130\ub9ac PD \ub2e4\ud61c\uc640 \uac00\uc871\uc5d0\uac8c \ubc84\ub9bc\ubc1b\uc740 \ucc9c\uc7ac \uc18c\ub140 \uc9c0\ubbfc(\ub0a8\uc9c0\ud604 \ubd84)\uc758 \uc0c1\ucc98\ubc1b\uc740 \ub0b4\uba74\uc744 \uadf8\ub9b0 \uc791\ud488\uc73c\ub85c 1\ub144 \ud6c4 \uc790\uc2e0\uc758 \uc6a9\uc11c\uac00 \ub73b\ud558\uc9c0 \uc54a\uc740 \uacb0\uacfc\ub97c \ubd88\ub7ec\uc624\uba74\uc11c \ub2e4\ud61c\uac00 \uacaa\ub294 \ud63c\ub780\uacfc \uc2ac\ud514, \uadf8\ub9ac\uace0 \uadf8 \ub05d\uc5d0\uc11c \ucc3e\uc544\ub0b8 \ucc2c\ub780\ud55c \uac10\ub3d9\uc744 \uadf8\ub824\ub0c8\ub2e4\ub294 \ud3c9\uc744 \ubc1b\uc558\ub2e4. \uc1a1\ud61c\uad50\ub294 \ub9c8\uc9c0\ub9c9 \uae30\ub150\ucd2c\uc601\uc744 \ub9c8\uce5c \ud6c4 \u201c\u2018\uc624\ub298\u2019\uc740 \ub0b4 \uc5f0\uae30 \uc778\uc0dd\uc758 \ud130\ub2dd\ud3ec\uc778\ud2b8\uac00 \ub418\ub294 \uc791\ud488\uc774\ub2e4.\u201d\ub77c\uace0 \ub9d0\ud588\ub2e4. \ub610\ud55c, \uadf8\ub140\uc758 \uc12c\uc138\ud55c \uac10\uc815\uc5f0\uae30\uc640 \uc808\uc81c\ub41c \uc2ec\ub9ac\uc5f0\uae30\ub294 \ud3c9\ub2e8\uc758 \ud638\ud3c9\uc744 \ubc1b\uc544 2011 \uc62c\ud574\uc758 \uc5ec\uc131\uc601\ud654\uc778 \uc2dc\uc0c1\uc2dd\uc5d0\uc11c \uc62c\ud574\uc758 \uc5ec\uc131\uc601\ud654\uc778 \uc5f0\uae30\uc0c1\uc744 \uc218\uc0c1\ud588\ub2e4. \uc218\uc0c1 \uc18c\uac10\uc5d0\uc11c \uc1a1\ud61c\uad50\ub294 \u201c\ub370\ubdd4\ud55c\uc9c0\ub294 \uc624\ub798\ub410\ub294\ub370 \uc601\ud654 \uc5f0\uae30\uc0c1\uc740 \ucc98\uc74c \ubc1b\ub294\ub2e4. \uc815\ub9d0 \uac10\uc0ac\ub4dc\ub9b0\ub2e4. \uc88b\uc740 \uc601\ud654\uc640 \uce90\ub9ad\ud130\ub97c \ub9cc\ub0a0 \uc218 \uc788\uac8c \ud574\uc900 \uc774\uc815\ud5a5 \uac10\ub3c5\ub2d8\uc774 \uacc4\uc168\ub354\ub77c\uba74 \ub354 \uc88b\uc558\uc744 \ud150\ub370 \uc544\uc27d\ub2e4\u201d\uc5d0 \uc774\uc5b4 \u201c\uc601\ud654 \uc5f0\uae30\uc0c1\uc744 \ubc1b\uc740 \uc9c0\uae08 \uc774 \uc21c\uac04\ub3c4 \ub108\ubb34 \ub5a8\ub9ac\ub294\ub370, \uccad\ub8e1 \uc601\ud654\uc81c\ub77c\uba74 \uc5b4\ub5a8\uc9c0 \uc0c1\uc0c1\uc870\ucc28 \ud558\uc9c0 \ubabb\ud558\uaca0\ub2e4.\u201d\uba70 \uac10\uaca9\uc758 \ub208\ubb3c\uc744 \ubcf4\uc600\uace0, \ub9c8\uc9c0\ub9c9\uc73c\ub85c \u201c\uc601\ud654 \u2018\uc624\ub298\u2019\uc774 \ub354 \uc88b\uc740 \ud658\uacbd\uc5d0\uc11c \uad00\uac1d\ub4e4\uacfc \ub9cc\ub0a0 \uc218 \uc788\uc5c8\ub2e4\uba74 \uc88b\uc558\uc744\ud150\ub370 \uadf8\ub7ec\uc9c0 \ubabb\ud574 \uc544\uc27d\ub2e4. \uc8fc\uc704 \ubaa8\ub4e0 \uc9c0\uc778\ubd84\ub4e4\uaed8 \uac10\uc0ac\ub4dc\ub9ac\uace0 \ud56d\uc0c1 \uc5f4\uc2ec\ud788 \ud574\uc11c \ub354 \uc88b\uc740 \uc5f0\uae30\ub97c \ubcf4\uc5ec\ub4dc\ub9ac\uaca0\ub2e4.\u201d\uace0 \ub9d0\ud588\ub2e4. \uac19\uc740 \ud574, \uadf8\ub140\ub294 \uc544\uc2dc\uc544 \ubc30\uc6b0 \ucd5c\ucd08\ub85c \ud504\ub791\uc2a4 \ucd5c\ub300 \uc5d0\uc774\uc804\uc2dc\uc640 \uacc4\uc57d\uc744 \uccb4\uacb0\ud558\uba70 \uc720\ub7fd \uc9c4\ucd9c\uc5d0 \uae30\ub300\ub97c \ubaa8\uc558\ub2e4. \uc1a1\ud61c\uad50\ub294 \ub2e8\uc21c\ud55c 20\ub300 \uccad\ucd98\uc2a4\ud0c0\uc5d0\uc11c \ub9cc\uc871\ud558\uc9c0 \uc54a\uace0 \uc5ec\ubc30\uc6b0\ub85c\uc11c \uc790\uc2e0\uc774 \ud45c\ud604\ud560 \uc218 \uc788\ub294 \uce90\ub9ad\ud130\uc5d0 \ub300\ud55c \uac70\ub4ed \uace0\ubbfc\ud558\uba70 \ubc30\uc6b0\ub85c\uc11c \uc131\uc7a5\ud558\ub294 \ubaa8\uc2b5\uc744 \ubcf4\uc5ec\uc92c\ub2e4."} {"question": "1979\ub144\uc5d0\uc11c 80\ub144\uae4c\uc9c0 \uc2dc\uc98c\uc5d0 \uc140\ud0c0 \ube44\uace0\uac00 \uac15\ub4f1\ub418\uace0\ub9cc \ub9ac\uadf8\ub294?", "paragraph": "1954\ub144\ubd80\ud130 1969\ub144\uae4c\uc9c0 \uc140\ud0c0 \ube44\uace0\ub294 2\ubd80 \ub9ac\uadf8\uc5d0\uc11c \ud504\ub9ac\uba54\ub77c\ub85c \uc62c\ub77c\uc624\uae30 \uc704\ud574 \uc5c4\uccad\ub09c \ud22c\uc7c1\uc744 \ud588\ub294\ub370 \ubb3c\ub860 59-60, 60-61, 65-66 \uc2dc\uc98c\uc5d0 \uadf8\ub4e4\uc774 \ud504\ub9ac\uba54\ub77c \ub9ac\uac00\uc5d0\uc11c \ub6f8 \uc218 \uc788\ub294 \ub2a5\ub825\uc774 \uc788\uc74c\uc744 \uc99d\uba85\ud588\uc9c0\ub9cc \uc544\uc27d\uac8c \uc2e4\ud328\ud558\uace0 \ub9cc\ub2e4. \uadf8\ub7ec\ub098 \uadf8\ub4e4\uc740 Uefa \ucef5 \ucd9c\uc804 \uc790\uaca9\uc744 \uc5bb\uc74c\uc73c\ub85c\uc11c \uadf8\ub4e4\uc758 \uc5ed\uc0ac\uc0c1 \ucc98\uc74c\uc73c\ub85c \uc720\ub7fd \ub300\ud56d\uc804\uc5d0 \ub098\uac08 \uae30\ud68c\ub97c \uc7a1\ub294\ub2e4.\uadf8\ub9ac\uace0 1979-80 \uc2dc\uc98c\uc5d0 \uc140\ud0c0 \ube44\uace0\ub294 \uae30\uc5b4\uc774 3\ubd80 \ub9ac\uadf8\uae4c\uc9c0 \uac15\ub4f1\ub418\uace0 \ub9cc\ub2e4. \uadf8\ub4e4\uc758 \uc5ed\uc0ac\uc0c1 \uac00\uc7a5 \uce58\uc695\uc801\uc778 \uae30\ub85d\uc774 \uc544\ub2d0 \uc218 \uc5c6\uc9c0\ub9cc \uc774\ub85c\uc368 \uadf8\ub4e4\uc740 \ubc14\ub2e5\uc744 \uacbd\ud5d8\ud588\uace0 \uadf8\ub4e4\uc740 92\ub144 \ub4dc\ub514\uc5b4 \ub2e4\uc2dc \ubcf5\uadc0\ud558\uba74\uc11c \uadf8\ub4e4\uc758 \uc2e4\ub825\uc774 \ud504\ub9ac\uba54\ub77c \ub9ac\uac00\uc758 \uc815\uc0c1\uc740 \uc544\ub2c8\uc9c0\ub9cc \ud504\ub9ac\uba54\ub77c \ub9ac\uac00\uc5d0\uc11c \ucad3\uaca8\ub0a0 \uc815\ub3c4\ub3c4 \uc5ed\uc2dc \uc544\ub2c8\ub77c\ub294 \uac83\uc744 \uc99d\uba85\ud558\uc600\ub2e4. \uadf8\ub9ac\uace0 1994\ub144 \ub4dc\ub514\uc5b4 \uadf8\ub4e4\uc740 \uc2a4\ud398\uc778 \ucef5\uc758 \uacb0\uc2b9\uae4c\uc9c0 \uc624\ub974\uc9c0\ub9cc \uc6b0\uc138\ud55c \uacbd\uae30 \ub0b4\uc6a9\uc5d0\ub3c4 \ubd88\uad6c\ud558\uace0 \uc9d1\uc911\ub825 \ubd80\uc871\uc73c\ub85c \ub808\uc54c \uc0ac\ub77c\uace0\uc0ac\uc5d0\uac8c \ud328\ub110\ud2f0 \ud0a5\uc73c\ub85c \uc9c0\uace0 \ub9cc\ub2e4.", "answer": "3\ubd80 \ub9ac\uadf8", "sentence": "\uadf8\ub7ec\ub098 \uadf8\ub4e4\uc740 Uefa \ucef5 \ucd9c\uc804 \uc790\uaca9\uc744 \uc5bb\uc74c\uc73c\ub85c\uc11c \uadf8\ub4e4\uc758 \uc5ed\uc0ac\uc0c1 \ucc98\uc74c\uc73c\ub85c \uc720\ub7fd \ub300\ud56d\uc804\uc5d0 \ub098\uac08 \uae30\ud68c\ub97c \uc7a1\ub294\ub2e4.\uadf8\ub9ac\uace0 1979-80 \uc2dc\uc98c\uc5d0 \uc140\ud0c0 \ube44\uace0\ub294 \uae30\uc5b4\uc774 3\ubd80 \ub9ac\uadf8 \uae4c\uc9c0 \uac15\ub4f1\ub418\uace0 \ub9cc\ub2e4.", "paragraph_sentence": "1954\ub144\ubd80\ud130 1969\ub144\uae4c\uc9c0 \uc140\ud0c0 \ube44\uace0\ub294 2\ubd80 \ub9ac\uadf8\uc5d0\uc11c \ud504\ub9ac\uba54\ub77c\ub85c \uc62c\ub77c\uc624\uae30 \uc704\ud574 \uc5c4\uccad\ub09c \ud22c\uc7c1\uc744 \ud588\ub294\ub370 \ubb3c\ub860 59-60, 60-61, 65-66 \uc2dc\uc98c\uc5d0 \uadf8\ub4e4\uc774 \ud504\ub9ac\uba54\ub77c \ub9ac\uac00\uc5d0\uc11c \ub6f8 \uc218 \uc788\ub294 \ub2a5\ub825\uc774 \uc788\uc74c\uc744 \uc99d\uba85\ud588\uc9c0\ub9cc \uc544\uc27d\uac8c \uc2e4\ud328\ud558\uace0 \ub9cc\ub2e4. \uadf8\ub7ec\ub098 \uadf8\ub4e4\uc740 Uefa \ucef5 \ucd9c\uc804 \uc790\uaca9\uc744 \uc5bb\uc74c\uc73c\ub85c\uc11c \uadf8\ub4e4\uc758 \uc5ed\uc0ac\uc0c1 \ucc98\uc74c\uc73c\ub85c \uc720\ub7fd \ub300\ud56d\uc804\uc5d0 \ub098\uac08 \uae30\ud68c\ub97c \uc7a1\ub294\ub2e4.\uadf8\ub9ac\uace0 1979-80 \uc2dc\uc98c\uc5d0 \uc140\ud0c0 \ube44\uace0\ub294 \uae30\uc5b4\uc774 3\ubd80 \ub9ac\uadf8 \uae4c\uc9c0 \uac15\ub4f1\ub418\uace0 \ub9cc\ub2e4. \uadf8\ub4e4\uc758 \uc5ed\uc0ac\uc0c1 \uac00\uc7a5 \uce58\uc695\uc801\uc778 \uae30\ub85d\uc774 \uc544\ub2d0 \uc218 \uc5c6\uc9c0\ub9cc \uc774\ub85c\uc368 \uadf8\ub4e4\uc740 \ubc14\ub2e5\uc744 \uacbd\ud5d8\ud588\uace0 \uadf8\ub4e4\uc740 92\ub144 \ub4dc\ub514\uc5b4 \ub2e4\uc2dc \ubcf5\uadc0\ud558\uba74\uc11c \uadf8\ub4e4\uc758 \uc2e4\ub825\uc774 \ud504\ub9ac\uba54\ub77c \ub9ac\uac00\uc758 \uc815\uc0c1\uc740 \uc544\ub2c8\uc9c0\ub9cc \ud504\ub9ac\uba54\ub77c \ub9ac\uac00\uc5d0\uc11c \ucad3\uaca8\ub0a0 \uc815\ub3c4\ub3c4 \uc5ed\uc2dc \uc544\ub2c8\ub77c\ub294 \uac83\uc744 \uc99d\uba85\ud558\uc600\ub2e4. \uadf8\ub9ac\uace0 1994\ub144 \ub4dc\ub514\uc5b4 \uadf8\ub4e4\uc740 \uc2a4\ud398\uc778 \ucef5\uc758 \uacb0\uc2b9\uae4c\uc9c0 \uc624\ub974\uc9c0\ub9cc \uc6b0\uc138\ud55c \uacbd\uae30 \ub0b4\uc6a9\uc5d0\ub3c4 \ubd88\uad6c\ud558\uace0 \uc9d1\uc911\ub825 \ubd80\uc871\uc73c\ub85c \ub808\uc54c \uc0ac\ub77c\uace0\uc0ac\uc5d0\uac8c \ud328\ub110\ud2f0 \ud0a5\uc73c\ub85c \uc9c0\uace0 \ub9cc\ub2e4.", "paragraph_answer": "1954\ub144\ubd80\ud130 1969\ub144\uae4c\uc9c0 \uc140\ud0c0 \ube44\uace0\ub294 2\ubd80 \ub9ac\uadf8\uc5d0\uc11c \ud504\ub9ac\uba54\ub77c\ub85c \uc62c\ub77c\uc624\uae30 \uc704\ud574 \uc5c4\uccad\ub09c \ud22c\uc7c1\uc744 \ud588\ub294\ub370 \ubb3c\ub860 59-60, 60-61, 65-66 \uc2dc\uc98c\uc5d0 \uadf8\ub4e4\uc774 \ud504\ub9ac\uba54\ub77c \ub9ac\uac00\uc5d0\uc11c \ub6f8 \uc218 \uc788\ub294 \ub2a5\ub825\uc774 \uc788\uc74c\uc744 \uc99d\uba85\ud588\uc9c0\ub9cc \uc544\uc27d\uac8c \uc2e4\ud328\ud558\uace0 \ub9cc\ub2e4. \uadf8\ub7ec\ub098 \uadf8\ub4e4\uc740 Uefa \ucef5 \ucd9c\uc804 \uc790\uaca9\uc744 \uc5bb\uc74c\uc73c\ub85c\uc11c \uadf8\ub4e4\uc758 \uc5ed\uc0ac\uc0c1 \ucc98\uc74c\uc73c\ub85c \uc720\ub7fd \ub300\ud56d\uc804\uc5d0 \ub098\uac08 \uae30\ud68c\ub97c \uc7a1\ub294\ub2e4.\uadf8\ub9ac\uace0 1979-80 \uc2dc\uc98c\uc5d0 \uc140\ud0c0 \ube44\uace0\ub294 \uae30\uc5b4\uc774 3\ubd80 \ub9ac\uadf8 \uae4c\uc9c0 \uac15\ub4f1\ub418\uace0 \ub9cc\ub2e4. \uadf8\ub4e4\uc758 \uc5ed\uc0ac\uc0c1 \uac00\uc7a5 \uce58\uc695\uc801\uc778 \uae30\ub85d\uc774 \uc544\ub2d0 \uc218 \uc5c6\uc9c0\ub9cc \uc774\ub85c\uc368 \uadf8\ub4e4\uc740 \ubc14\ub2e5\uc744 \uacbd\ud5d8\ud588\uace0 \uadf8\ub4e4\uc740 92\ub144 \ub4dc\ub514\uc5b4 \ub2e4\uc2dc \ubcf5\uadc0\ud558\uba74\uc11c \uadf8\ub4e4\uc758 \uc2e4\ub825\uc774 \ud504\ub9ac\uba54\ub77c \ub9ac\uac00\uc758 \uc815\uc0c1\uc740 \uc544\ub2c8\uc9c0\ub9cc \ud504\ub9ac\uba54\ub77c \ub9ac\uac00\uc5d0\uc11c \ucad3\uaca8\ub0a0 \uc815\ub3c4\ub3c4 \uc5ed\uc2dc \uc544\ub2c8\ub77c\ub294 \uac83\uc744 \uc99d\uba85\ud558\uc600\ub2e4. \uadf8\ub9ac\uace0 1994\ub144 \ub4dc\ub514\uc5b4 \uadf8\ub4e4\uc740 \uc2a4\ud398\uc778 \ucef5\uc758 \uacb0\uc2b9\uae4c\uc9c0 \uc624\ub974\uc9c0\ub9cc \uc6b0\uc138\ud55c \uacbd\uae30 \ub0b4\uc6a9\uc5d0\ub3c4 \ubd88\uad6c\ud558\uace0 \uc9d1\uc911\ub825 \ubd80\uc871\uc73c\ub85c \ub808\uc54c \uc0ac\ub77c\uace0\uc0ac\uc5d0\uac8c \ud328\ub110\ud2f0 \ud0a5\uc73c\ub85c \uc9c0\uace0 \ub9cc\ub2e4.", "sentence_answer": "\uadf8\ub7ec\ub098 \uadf8\ub4e4\uc740 Uefa \ucef5 \ucd9c\uc804 \uc790\uaca9\uc744 \uc5bb\uc74c\uc73c\ub85c\uc11c \uadf8\ub4e4\uc758 \uc5ed\uc0ac\uc0c1 \ucc98\uc74c\uc73c\ub85c \uc720\ub7fd \ub300\ud56d\uc804\uc5d0 \ub098\uac08 \uae30\ud68c\ub97c \uc7a1\ub294\ub2e4.\uadf8\ub9ac\uace0 1979-80 \uc2dc\uc98c\uc5d0 \uc140\ud0c0 \ube44\uace0\ub294 \uae30\uc5b4\uc774 3\ubd80 \ub9ac\uadf8 \uae4c\uc9c0 \uac15\ub4f1\ub418\uace0 \ub9cc\ub2e4.", "paragraph_id": "6518223-1-0", "paragraph_question": "question: 1979\ub144\uc5d0\uc11c 80\ub144\uae4c\uc9c0 \uc2dc\uc98c\uc5d0 \uc140\ud0c0 \ube44\uace0\uac00 \uac15\ub4f1\ub418\uace0\ub9cc \ub9ac\uadf8\ub294?, context: 1954\ub144\ubd80\ud130 1969\ub144\uae4c\uc9c0 \uc140\ud0c0 \ube44\uace0\ub294 2\ubd80 \ub9ac\uadf8\uc5d0\uc11c \ud504\ub9ac\uba54\ub77c\ub85c \uc62c\ub77c\uc624\uae30 \uc704\ud574 \uc5c4\uccad\ub09c \ud22c\uc7c1\uc744 \ud588\ub294\ub370 \ubb3c\ub860 59-60, 60-61, 65-66 \uc2dc\uc98c\uc5d0 \uadf8\ub4e4\uc774 \ud504\ub9ac\uba54\ub77c \ub9ac\uac00\uc5d0\uc11c \ub6f8 \uc218 \uc788\ub294 \ub2a5\ub825\uc774 \uc788\uc74c\uc744 \uc99d\uba85\ud588\uc9c0\ub9cc \uc544\uc27d\uac8c \uc2e4\ud328\ud558\uace0 \ub9cc\ub2e4. \uadf8\ub7ec\ub098 \uadf8\ub4e4\uc740 Uefa \ucef5 \ucd9c\uc804 \uc790\uaca9\uc744 \uc5bb\uc74c\uc73c\ub85c\uc11c \uadf8\ub4e4\uc758 \uc5ed\uc0ac\uc0c1 \ucc98\uc74c\uc73c\ub85c \uc720\ub7fd \ub300\ud56d\uc804\uc5d0 \ub098\uac08 \uae30\ud68c\ub97c \uc7a1\ub294\ub2e4.\uadf8\ub9ac\uace0 1979-80 \uc2dc\uc98c\uc5d0 \uc140\ud0c0 \ube44\uace0\ub294 \uae30\uc5b4\uc774 3\ubd80 \ub9ac\uadf8\uae4c\uc9c0 \uac15\ub4f1\ub418\uace0 \ub9cc\ub2e4. \uadf8\ub4e4\uc758 \uc5ed\uc0ac\uc0c1 \uac00\uc7a5 \uce58\uc695\uc801\uc778 \uae30\ub85d\uc774 \uc544\ub2d0 \uc218 \uc5c6\uc9c0\ub9cc \uc774\ub85c\uc368 \uadf8\ub4e4\uc740 \ubc14\ub2e5\uc744 \uacbd\ud5d8\ud588\uace0 \uadf8\ub4e4\uc740 92\ub144 \ub4dc\ub514\uc5b4 \ub2e4\uc2dc \ubcf5\uadc0\ud558\uba74\uc11c \uadf8\ub4e4\uc758 \uc2e4\ub825\uc774 \ud504\ub9ac\uba54\ub77c \ub9ac\uac00\uc758 \uc815\uc0c1\uc740 \uc544\ub2c8\uc9c0\ub9cc \ud504\ub9ac\uba54\ub77c \ub9ac\uac00\uc5d0\uc11c \ucad3\uaca8\ub0a0 \uc815\ub3c4\ub3c4 \uc5ed\uc2dc \uc544\ub2c8\ub77c\ub294 \uac83\uc744 \uc99d\uba85\ud558\uc600\ub2e4. \uadf8\ub9ac\uace0 1994\ub144 \ub4dc\ub514\uc5b4 \uadf8\ub4e4\uc740 \uc2a4\ud398\uc778 \ucef5\uc758 \uacb0\uc2b9\uae4c\uc9c0 \uc624\ub974\uc9c0\ub9cc \uc6b0\uc138\ud55c \uacbd\uae30 \ub0b4\uc6a9\uc5d0\ub3c4 \ubd88\uad6c\ud558\uace0 \uc9d1\uc911\ub825 \ubd80\uc871\uc73c\ub85c \ub808\uc54c \uc0ac\ub77c\uace0\uc0ac\uc5d0\uac8c \ud328\ub110\ud2f0 \ud0a5\uc73c\ub85c \uc9c0\uace0 \ub9cc\ub2e4."} {"question": "\uae40\ub300\uc911\uc744 \uc911\ub3c4\uc801\uc778 \uc778\ub3c4\uc8fc\uc758\uc790\ub85c \ud3c9\uac00\ud55c \ud3c9\ud654\ud559\uc790\ub294?", "paragraph": "\uc870\uc9c0\ud504 \ubc14\uc774\ub4e0 \ubbf8\uad6d \ubd80\ud1b5\ub839\uc740 \uac00\uc7a5 \uc874\uacbd\ud558\ub294 \uc9c0\ub3c4\uc790\ub97c \uae40\ub300\uc911\uc744 \uaf3d\uc558\ub2e4. \ubc14\uc774\ub4e0 \ubd80\ud1b5\ub839\uc740 \ud587\ubcd5\uc815\ucc45\uc758 \uc5f4\ub82c\ud55c \uc9c0\uc9c0\uc790\uc774\uae30\ub3c4 \ud558\ub2e4. \ud638\ucf58 \ub9dd\ub204\uc2a4 \ub178\ub974\uc6e8\uc774 \ud669\ud0dc\uc790\ub294 \"\uae40 \uc804 \ub300\ud1b5\ub839\uc774\uc57c\ub9d0\ub85c \ubbfc\uc8fc\ud654\uc640 \ud55c\ubc18\ub3c4 \ud3c9\ud654 \ud504\ub85c\uc138\uc2a4 \uadf8 \uc790\uccb4\ub97c \uc0b4\uc544\uc624\uc2e0 \ubd84\"\uc774\ub77c\uace0 \ud3c9\uac00\ud588\ub2e4. \uc5d0\ub9ac\ud06c \uc190\ud558\uc784 \ub178\ub974\uc6e8\uc774 \ud658\uacbd\uac1c\ubc1c\ubd80 \uc7a5\uad00\uc740 \"\uc800\uc758 \uac1c\uc778\uc801 \uc601\uc6c5\uc774\ub2e4. \ub2e4\ub978 \ub098\ub77c\uc5d0\uc11c \ud558\uc9c0 \ubabb\ud588\ub358 \ud3c9\ud654\uc5d0 \ub300\ud55c \ub9ce\uc740 \uc77c\uc744 \ud558\uc168\ub2e4\"\ub77c\uace0 \ud3c9\uac00\ud588\ub2e4. \ud3c9\ud654\ud559\uc790 \uc694\ud55c \uac08\ud241 \uad50\uc218\ub3c4 \"\uadf8\ub294 \uc88c\ud30c\ub3c4 \uc6b0\ud30c\ub3c4 \uc544\ub2cc '\uc911\ub3c4'\uc774\uba70 \ub300\ub2e8\ud788 \uc778\ub3c4\uc8fc\uc758\uc801\uc778 \uc778\ubb3c\uc774\ub2e4\"\ub77c\uace0 \ud3c9\uac00\ud588\ub2e4. \ub9ac\ud558\ub974\ud2b8 \ud3f0 \ubc14\uc774\uce20\uccb4\ucee4 \uc804 \ub3c5\uc77c \ub300\ud1b5\ub839\uc740 \"\uce5c\uad6c\ub97c \ub118\uc5b4 \uc778\ub958\uc560\ub97c \uac16\ucd98 \uc778\uaca9\uc790\"\ub77c\uace0 \uae40\ub300\uc911\uc744 \ud3c9\ud558\uace4 \ud588\ub2e4. \ub178\ub974\ubca0\ub974\ud2b8 \ub78c\uba38\ud2b8 \ub3c5\uc77c \uad6d\ud68c\uc758\uc7a5\uc740 \uae40\ub300\uc911\uc744 \ub9cc\ub09c \uc790\ub9ac\uc5d0\uc11c \"\ub3c5\uc77c\uc5d0\uc11c \ub2f9\uc2e0\ub9cc\ud07c \uc601\ud5a5\ub825 \uc788\ub294 \ud55c\uad6d\uc778\uc740 \uc5c6\ub2e4\"\ub77c\uace0 \uac70\ub4ed \ubc1d\ud78c\ubc14 \uc788\ub2e4. \ub3c4\ub110\ub4dc \uadf8\ub808\uadf8 \uc804 \uc8fc\ud55c \ubbf8\uad6d\ub300\uc0ac\ub294 \"\ud55c\uad6d \uad6d\ubbfc\uacfc \uc720\uac00\uc871\ub4e4\uc774 \uae40 \uc804 \ub300\ud1b5\ub839\uc758 \uc0b6\uc5d0 \ub300\ud574 \ud070 \uc790\ubd80\uc2ec\uc744 \ub290\uaef4\uc57c \ud55c\ub2e4\uace0 \uc0dd\uac01\ud55c\ub2e4\"\uba74\uc11c \uadf8\ub294 \uc6a9\uac10\ud588\uace0 \ubbfc\uc8fc\uc8fc\uc758\ub97c \uc704\ud574 \uacc4\uc18d \ud22c\uc7c1\ud588\uc73c\uba70 \ud55c\uad6d \ubbfc\uc8fc\uc8fc\uc758\ub97c \uc704\ud574 \ub9ce\uc740 \uae30\uc5ec\ub97c \ud558\ub294 \ud55c\ud3b8 \ud587\ubcd5\uc815\ucc45\uc744 \ud1b5\ud574 \ub300\ubd81\uad00\uacc4\uc5d0 \ud070 \uc9c4\uc804\uc744 \uc774\ub918\ub2e4\uace0 \ud3c9\uac00\ud588\ub2e4.", "answer": "\uc694\ud55c \uac08\ud241", "sentence": "\ud3c9\ud654\ud559\uc790 \uc694\ud55c \uac08\ud241 \uad50\uc218\ub3c4 \"\uadf8\ub294 \uc88c\ud30c\ub3c4 \uc6b0\ud30c\ub3c4 \uc544\ub2cc '\uc911\ub3c4'\uc774\uba70 \ub300\ub2e8\ud788 \uc778\ub3c4\uc8fc\uc758\uc801\uc778 \uc778\ubb3c\uc774\ub2e4\"\ub77c\uace0 \ud3c9\uac00\ud588\ub2e4.", "paragraph_sentence": "\uc870\uc9c0\ud504 \ubc14\uc774\ub4e0 \ubbf8\uad6d \ubd80\ud1b5\ub839\uc740 \uac00\uc7a5 \uc874\uacbd\ud558\ub294 \uc9c0\ub3c4\uc790\ub97c \uae40\ub300\uc911\uc744 \uaf3d\uc558\ub2e4. \ubc14\uc774\ub4e0 \ubd80\ud1b5\ub839\uc740 \ud587\ubcd5\uc815\ucc45\uc758 \uc5f4\ub82c\ud55c \uc9c0\uc9c0\uc790\uc774\uae30\ub3c4 \ud558\ub2e4. \ud638\ucf58 \ub9dd\ub204\uc2a4 \ub178\ub974\uc6e8\uc774 \ud669\ud0dc\uc790\ub294 \"\uae40 \uc804 \ub300\ud1b5\ub839\uc774\uc57c\ub9d0\ub85c \ubbfc\uc8fc\ud654\uc640 \ud55c\ubc18\ub3c4 \ud3c9\ud654 \ud504\ub85c\uc138\uc2a4 \uadf8 \uc790\uccb4\ub97c \uc0b4\uc544\uc624\uc2e0 \ubd84\"\uc774\ub77c\uace0 \ud3c9\uac00\ud588\ub2e4. \uc5d0\ub9ac\ud06c \uc190\ud558\uc784 \ub178\ub974\uc6e8\uc774 \ud658\uacbd\uac1c\ubc1c\ubd80 \uc7a5\uad00\uc740 \"\uc800\uc758 \uac1c\uc778\uc801 \uc601\uc6c5\uc774\ub2e4. \ub2e4\ub978 \ub098\ub77c\uc5d0\uc11c \ud558\uc9c0 \ubabb\ud588\ub358 \ud3c9\ud654\uc5d0 \ub300\ud55c \ub9ce\uc740 \uc77c\uc744 \ud558\uc168\ub2e4\"\ub77c\uace0 \ud3c9\uac00\ud588\ub2e4. \ud3c9\ud654\ud559\uc790 \uc694\ud55c \uac08\ud241 \uad50\uc218\ub3c4 \"\uadf8\ub294 \uc88c\ud30c\ub3c4 \uc6b0\ud30c\ub3c4 \uc544\ub2cc '\uc911\ub3c4'\uc774\uba70 \ub300\ub2e8\ud788 \uc778\ub3c4\uc8fc\uc758\uc801\uc778 \uc778\ubb3c\uc774\ub2e4\"\ub77c\uace0 \ud3c9\uac00\ud588\ub2e4. \ub9ac\ud558\ub974\ud2b8 \ud3f0 \ubc14\uc774\uce20\uccb4\ucee4 \uc804 \ub3c5\uc77c \ub300\ud1b5\ub839\uc740 \"\uce5c\uad6c\ub97c \ub118\uc5b4 \uc778\ub958\uc560\ub97c \uac16\ucd98 \uc778\uaca9\uc790\"\ub77c\uace0 \uae40\ub300\uc911\uc744 \ud3c9\ud558\uace4 \ud588\ub2e4. \ub178\ub974\ubca0\ub974\ud2b8 \ub78c\uba38\ud2b8 \ub3c5\uc77c \uad6d\ud68c\uc758\uc7a5\uc740 \uae40\ub300\uc911\uc744 \ub9cc\ub09c \uc790\ub9ac\uc5d0\uc11c \"\ub3c5\uc77c\uc5d0\uc11c \ub2f9\uc2e0\ub9cc\ud07c \uc601\ud5a5\ub825 \uc788\ub294 \ud55c\uad6d\uc778\uc740 \uc5c6\ub2e4\"\ub77c\uace0 \uac70\ub4ed \ubc1d\ud78c\ubc14 \uc788\ub2e4. \ub3c4\ub110\ub4dc \uadf8\ub808\uadf8 \uc804 \uc8fc\ud55c \ubbf8\uad6d\ub300\uc0ac\ub294 \"\ud55c\uad6d \uad6d\ubbfc\uacfc \uc720\uac00\uc871\ub4e4\uc774 \uae40 \uc804 \ub300\ud1b5\ub839\uc758 \uc0b6\uc5d0 \ub300\ud574 \ud070 \uc790\ubd80\uc2ec\uc744 \ub290\uaef4\uc57c \ud55c\ub2e4\uace0 \uc0dd\uac01\ud55c\ub2e4\"\uba74\uc11c \uadf8\ub294 \uc6a9\uac10\ud588\uace0 \ubbfc\uc8fc\uc8fc\uc758\ub97c \uc704\ud574 \uacc4\uc18d \ud22c\uc7c1\ud588\uc73c\uba70 \ud55c\uad6d \ubbfc\uc8fc\uc8fc\uc758\ub97c \uc704\ud574 \ub9ce\uc740 \uae30\uc5ec\ub97c \ud558\ub294 \ud55c\ud3b8 \ud587\ubcd5\uc815\ucc45\uc744 \ud1b5\ud574 \ub300\ubd81\uad00\uacc4\uc5d0 \ud070 \uc9c4\uc804\uc744 \uc774\ub918\ub2e4\uace0 \ud3c9\uac00\ud588\ub2e4.", "paragraph_answer": "\uc870\uc9c0\ud504 \ubc14\uc774\ub4e0 \ubbf8\uad6d \ubd80\ud1b5\ub839\uc740 \uac00\uc7a5 \uc874\uacbd\ud558\ub294 \uc9c0\ub3c4\uc790\ub97c \uae40\ub300\uc911\uc744 \uaf3d\uc558\ub2e4. \ubc14\uc774\ub4e0 \ubd80\ud1b5\ub839\uc740 \ud587\ubcd5\uc815\ucc45\uc758 \uc5f4\ub82c\ud55c \uc9c0\uc9c0\uc790\uc774\uae30\ub3c4 \ud558\ub2e4. \ud638\ucf58 \ub9dd\ub204\uc2a4 \ub178\ub974\uc6e8\uc774 \ud669\ud0dc\uc790\ub294 \"\uae40 \uc804 \ub300\ud1b5\ub839\uc774\uc57c\ub9d0\ub85c \ubbfc\uc8fc\ud654\uc640 \ud55c\ubc18\ub3c4 \ud3c9\ud654 \ud504\ub85c\uc138\uc2a4 \uadf8 \uc790\uccb4\ub97c \uc0b4\uc544\uc624\uc2e0 \ubd84\"\uc774\ub77c\uace0 \ud3c9\uac00\ud588\ub2e4. \uc5d0\ub9ac\ud06c \uc190\ud558\uc784 \ub178\ub974\uc6e8\uc774 \ud658\uacbd\uac1c\ubc1c\ubd80 \uc7a5\uad00\uc740 \"\uc800\uc758 \uac1c\uc778\uc801 \uc601\uc6c5\uc774\ub2e4. \ub2e4\ub978 \ub098\ub77c\uc5d0\uc11c \ud558\uc9c0 \ubabb\ud588\ub358 \ud3c9\ud654\uc5d0 \ub300\ud55c \ub9ce\uc740 \uc77c\uc744 \ud558\uc168\ub2e4\"\ub77c\uace0 \ud3c9\uac00\ud588\ub2e4. \ud3c9\ud654\ud559\uc790 \uc694\ud55c \uac08\ud241 \uad50\uc218\ub3c4 \"\uadf8\ub294 \uc88c\ud30c\ub3c4 \uc6b0\ud30c\ub3c4 \uc544\ub2cc '\uc911\ub3c4'\uc774\uba70 \ub300\ub2e8\ud788 \uc778\ub3c4\uc8fc\uc758\uc801\uc778 \uc778\ubb3c\uc774\ub2e4\"\ub77c\uace0 \ud3c9\uac00\ud588\ub2e4. \ub9ac\ud558\ub974\ud2b8 \ud3f0 \ubc14\uc774\uce20\uccb4\ucee4 \uc804 \ub3c5\uc77c \ub300\ud1b5\ub839\uc740 \"\uce5c\uad6c\ub97c \ub118\uc5b4 \uc778\ub958\uc560\ub97c \uac16\ucd98 \uc778\uaca9\uc790\"\ub77c\uace0 \uae40\ub300\uc911\uc744 \ud3c9\ud558\uace4 \ud588\ub2e4. \ub178\ub974\ubca0\ub974\ud2b8 \ub78c\uba38\ud2b8 \ub3c5\uc77c \uad6d\ud68c\uc758\uc7a5\uc740 \uae40\ub300\uc911\uc744 \ub9cc\ub09c \uc790\ub9ac\uc5d0\uc11c \"\ub3c5\uc77c\uc5d0\uc11c \ub2f9\uc2e0\ub9cc\ud07c \uc601\ud5a5\ub825 \uc788\ub294 \ud55c\uad6d\uc778\uc740 \uc5c6\ub2e4\"\ub77c\uace0 \uac70\ub4ed \ubc1d\ud78c\ubc14 \uc788\ub2e4. \ub3c4\ub110\ub4dc \uadf8\ub808\uadf8 \uc804 \uc8fc\ud55c \ubbf8\uad6d\ub300\uc0ac\ub294 \"\ud55c\uad6d \uad6d\ubbfc\uacfc \uc720\uac00\uc871\ub4e4\uc774 \uae40 \uc804 \ub300\ud1b5\ub839\uc758 \uc0b6\uc5d0 \ub300\ud574 \ud070 \uc790\ubd80\uc2ec\uc744 \ub290\uaef4\uc57c \ud55c\ub2e4\uace0 \uc0dd\uac01\ud55c\ub2e4\"\uba74\uc11c \uadf8\ub294 \uc6a9\uac10\ud588\uace0 \ubbfc\uc8fc\uc8fc\uc758\ub97c \uc704\ud574 \uacc4\uc18d \ud22c\uc7c1\ud588\uc73c\uba70 \ud55c\uad6d \ubbfc\uc8fc\uc8fc\uc758\ub97c \uc704\ud574 \ub9ce\uc740 \uae30\uc5ec\ub97c \ud558\ub294 \ud55c\ud3b8 \ud587\ubcd5\uc815\ucc45\uc744 \ud1b5\ud574 \ub300\ubd81\uad00\uacc4\uc5d0 \ud070 \uc9c4\uc804\uc744 \uc774\ub918\ub2e4\uace0 \ud3c9\uac00\ud588\ub2e4.", "sentence_answer": "\ud3c9\ud654\ud559\uc790 \uc694\ud55c \uac08\ud241 \uad50\uc218\ub3c4 \"\uadf8\ub294 \uc88c\ud30c\ub3c4 \uc6b0\ud30c\ub3c4 \uc544\ub2cc '\uc911\ub3c4'\uc774\uba70 \ub300\ub2e8\ud788 \uc778\ub3c4\uc8fc\uc758\uc801\uc778 \uc778\ubb3c\uc774\ub2e4\"\ub77c\uace0 \ud3c9\uac00\ud588\ub2e4.", "paragraph_id": "6488203-65-1", "paragraph_question": "question: \uae40\ub300\uc911\uc744 \uc911\ub3c4\uc801\uc778 \uc778\ub3c4\uc8fc\uc758\uc790\ub85c \ud3c9\uac00\ud55c \ud3c9\ud654\ud559\uc790\ub294?, context: \uc870\uc9c0\ud504 \ubc14\uc774\ub4e0 \ubbf8\uad6d \ubd80\ud1b5\ub839\uc740 \uac00\uc7a5 \uc874\uacbd\ud558\ub294 \uc9c0\ub3c4\uc790\ub97c \uae40\ub300\uc911\uc744 \uaf3d\uc558\ub2e4. \ubc14\uc774\ub4e0 \ubd80\ud1b5\ub839\uc740 \ud587\ubcd5\uc815\ucc45\uc758 \uc5f4\ub82c\ud55c \uc9c0\uc9c0\uc790\uc774\uae30\ub3c4 \ud558\ub2e4. \ud638\ucf58 \ub9dd\ub204\uc2a4 \ub178\ub974\uc6e8\uc774 \ud669\ud0dc\uc790\ub294 \"\uae40 \uc804 \ub300\ud1b5\ub839\uc774\uc57c\ub9d0\ub85c \ubbfc\uc8fc\ud654\uc640 \ud55c\ubc18\ub3c4 \ud3c9\ud654 \ud504\ub85c\uc138\uc2a4 \uadf8 \uc790\uccb4\ub97c \uc0b4\uc544\uc624\uc2e0 \ubd84\"\uc774\ub77c\uace0 \ud3c9\uac00\ud588\ub2e4. \uc5d0\ub9ac\ud06c \uc190\ud558\uc784 \ub178\ub974\uc6e8\uc774 \ud658\uacbd\uac1c\ubc1c\ubd80 \uc7a5\uad00\uc740 \"\uc800\uc758 \uac1c\uc778\uc801 \uc601\uc6c5\uc774\ub2e4. \ub2e4\ub978 \ub098\ub77c\uc5d0\uc11c \ud558\uc9c0 \ubabb\ud588\ub358 \ud3c9\ud654\uc5d0 \ub300\ud55c \ub9ce\uc740 \uc77c\uc744 \ud558\uc168\ub2e4\"\ub77c\uace0 \ud3c9\uac00\ud588\ub2e4. \ud3c9\ud654\ud559\uc790 \uc694\ud55c \uac08\ud241 \uad50\uc218\ub3c4 \"\uadf8\ub294 \uc88c\ud30c\ub3c4 \uc6b0\ud30c\ub3c4 \uc544\ub2cc '\uc911\ub3c4'\uc774\uba70 \ub300\ub2e8\ud788 \uc778\ub3c4\uc8fc\uc758\uc801\uc778 \uc778\ubb3c\uc774\ub2e4\"\ub77c\uace0 \ud3c9\uac00\ud588\ub2e4. \ub9ac\ud558\ub974\ud2b8 \ud3f0 \ubc14\uc774\uce20\uccb4\ucee4 \uc804 \ub3c5\uc77c \ub300\ud1b5\ub839\uc740 \"\uce5c\uad6c\ub97c \ub118\uc5b4 \uc778\ub958\uc560\ub97c \uac16\ucd98 \uc778\uaca9\uc790\"\ub77c\uace0 \uae40\ub300\uc911\uc744 \ud3c9\ud558\uace4 \ud588\ub2e4. \ub178\ub974\ubca0\ub974\ud2b8 \ub78c\uba38\ud2b8 \ub3c5\uc77c \uad6d\ud68c\uc758\uc7a5\uc740 \uae40\ub300\uc911\uc744 \ub9cc\ub09c \uc790\ub9ac\uc5d0\uc11c \"\ub3c5\uc77c\uc5d0\uc11c \ub2f9\uc2e0\ub9cc\ud07c \uc601\ud5a5\ub825 \uc788\ub294 \ud55c\uad6d\uc778\uc740 \uc5c6\ub2e4\"\ub77c\uace0 \uac70\ub4ed \ubc1d\ud78c\ubc14 \uc788\ub2e4. \ub3c4\ub110\ub4dc \uadf8\ub808\uadf8 \uc804 \uc8fc\ud55c \ubbf8\uad6d\ub300\uc0ac\ub294 \"\ud55c\uad6d \uad6d\ubbfc\uacfc \uc720\uac00\uc871\ub4e4\uc774 \uae40 \uc804 \ub300\ud1b5\ub839\uc758 \uc0b6\uc5d0 \ub300\ud574 \ud070 \uc790\ubd80\uc2ec\uc744 \ub290\uaef4\uc57c \ud55c\ub2e4\uace0 \uc0dd\uac01\ud55c\ub2e4\"\uba74\uc11c \uadf8\ub294 \uc6a9\uac10\ud588\uace0 \ubbfc\uc8fc\uc8fc\uc758\ub97c \uc704\ud574 \uacc4\uc18d \ud22c\uc7c1\ud588\uc73c\uba70 \ud55c\uad6d \ubbfc\uc8fc\uc8fc\uc758\ub97c \uc704\ud574 \ub9ce\uc740 \uae30\uc5ec\ub97c \ud558\ub294 \ud55c\ud3b8 \ud587\ubcd5\uc815\ucc45\uc744 \ud1b5\ud574 \ub300\ubd81\uad00\uacc4\uc5d0 \ud070 \uc9c4\uc804\uc744 \uc774\ub918\ub2e4\uace0 \ud3c9\uac00\ud588\ub2e4."} {"question": "\ubb38\uc7ac\uc778\ub300\ud1b5\ub839\uc774 \ud604\uc7a5\uc744 \ubc29\ubb38\ud55c \ub0a0\uc9dc\ub294?", "paragraph": "2017\ub144 12\uc6d4 22\uc77c \ubb38\uc7ac\uc778 \ub300\ud1b5\ub839\uc774 \ud604\uc7a5\uc744 \ubc29\ubb38\ud558\uc790 \uac10\uc815\uc801\uc73c\ub85c \uaca9\uc559\ub3fc \uc788\ub358 \uc77c\ubd80 \uc720\uac00\uc871\uc740 \ub300\uc120 \uad6c\ud638\uc600\ub358 '\uc0ac\ub78c\uc774 \uba3c\uc800\ub2e4'\uc744 \uc0c1\uae30\uc2dc\ud0a4\uba74\uc11c \"\uc774\ubc88\uc5d0 \uc0ac\ub78c\uc774\uace0 \ubb50\uace0 \uc5c6\uc5c8\ub2e4\"\uba70 \ubd84\ub178\ud588\uace0 \"\uc815\ubd80\uac00 \uc774\ub7f0 \uc2dd\uc73c\ub85c \ub300\ucc98\ud558\ub294 \uac8c \ud55c\ub450 \ubc88\uc774\ub0d0\", \"\ucd08\uae30\ub300\uc751\ub9cc \uc798\ud588\uc5b4\ub3c4 \uc0ac\ub78c\uc774 \uc774\ub807\uac8c \ub9ce\uc774 \uc8fd\uc9c0\ub294 \uc54a\uc558\uc744 \uac83\"\uc774\ub77c\uba70 \"\uc138\uc6d4\ud638 \uc774\ud6c4\uc5d0\ub294 \uc880 \ub098\uc544\uc9c0\ub294\uac00 \ud588\ub294\ub370 \uc548\uc804 \uc2dc\uc2a4\ud15c\uc774 \ub098\uc544\uc9c4 \uac83\uc774 \uc5c6\ub2e4\"\ub294 \uac83\ub3c4 \uc9c0\uc801\ud588\ub2e4. \ub610\ud55c \uc138\uc6d4\ud638 \uc0ac\uace0\uc5d0 \ube57\ub300 \ud654\uc7ac \ubc1c\uc0dd 2\uc2dc\uac04\uc774 \ub118\uc5b4\uc11c\uc57c \uc218\uc2b5\ud558\ub77c \uc9c0\uc2dc\ud588\ub358 \uac83\uc744 \ube44\ud310\ud588\uace0 5\uc2dc 20\ubd84\uae4c\uc9c0 \uc0dd\uc874\ud55c \ud76c\uc0dd\uc790\ub4e4\uacfc \ud1b5\ud654\ud55c \uc720\uc871\ub4e4\uc774 \uc788\uc5c8\ub2e4\uba74\uc11c, 4\uc2dc 7\ubd84\uc5d0 \uc804\ud654\ud588\uc744 \ub54c \uc9c0\ub839\uc774 \ub0b4\ub824\uc9c0\uace0 \uc9c0\uce68\ub300\ub85c \ud588\ub2e4\uba74 \uc778\uba85 \ud53c\ud574\uac00 \uc801\uc5c8\uc744 \uac83\uc774\ub77c \uc8fc\uc7a5\ud588\ub2e4.", "answer": "2017\ub144 12\uc6d4 22\uc77c", "sentence": "2017\ub144 12\uc6d4 22\uc77c \ubb38\uc7ac\uc778 \ub300\ud1b5\ub839\uc774 \ud604\uc7a5\uc744 \ubc29\ubb38\ud558\uc790 \uac10\uc815\uc801\uc73c\ub85c \uaca9\uc559\ub3fc \uc788\ub358 \uc77c\ubd80 \uc720\uac00\uc871\uc740 \ub300\uc120 \uad6c\ud638\uc600\ub358 '\uc0ac\ub78c\uc774 \uba3c\uc800\ub2e4'\uc744 \uc0c1\uae30\uc2dc\ud0a4\uba74\uc11c \"\uc774\ubc88\uc5d0 \uc0ac\ub78c\uc774\uace0 \ubb50\uace0 \uc5c6\uc5c8\ub2e4\"\uba70 \ubd84\ub178\ud588\uace0 \"\uc815\ubd80\uac00 \uc774\ub7f0 \uc2dd\uc73c\ub85c \ub300\ucc98\ud558\ub294 \uac8c \ud55c\ub450 \ubc88\uc774\ub0d0\", \"\ucd08\uae30\ub300\uc751\ub9cc \uc798\ud588\uc5b4\ub3c4 \uc0ac\ub78c\uc774 \uc774\ub807\uac8c \ub9ce\uc774 \uc8fd\uc9c0\ub294 \uc54a\uc558\uc744 \uac83\"\uc774\ub77c\uba70 \"\uc138\uc6d4\ud638 \uc774\ud6c4\uc5d0\ub294 \uc880 \ub098\uc544\uc9c0\ub294\uac00 \ud588\ub294\ub370 \uc548\uc804 \uc2dc\uc2a4\ud15c\uc774 \ub098\uc544\uc9c4 \uac83\uc774 \uc5c6\ub2e4\"\ub294 \uac83\ub3c4 \uc9c0\uc801\ud588\ub2e4.", "paragraph_sentence": " 2017\ub144 12\uc6d4 22\uc77c \ubb38\uc7ac\uc778 \ub300\ud1b5\ub839\uc774 \ud604\uc7a5\uc744 \ubc29\ubb38\ud558\uc790 \uac10\uc815\uc801\uc73c\ub85c \uaca9\uc559\ub3fc \uc788\ub358 \uc77c\ubd80 \uc720\uac00\uc871\uc740 \ub300\uc120 \uad6c\ud638\uc600\ub358 '\uc0ac\ub78c\uc774 \uba3c\uc800\ub2e4'\uc744 \uc0c1\uae30\uc2dc\ud0a4\uba74\uc11c \"\uc774\ubc88\uc5d0 \uc0ac\ub78c\uc774\uace0 \ubb50\uace0 \uc5c6\uc5c8\ub2e4\"\uba70 \ubd84\ub178\ud588\uace0 \"\uc815\ubd80\uac00 \uc774\ub7f0 \uc2dd\uc73c\ub85c \ub300\ucc98\ud558\ub294 \uac8c \ud55c\ub450 \ubc88\uc774\ub0d0\", \"\ucd08\uae30\ub300\uc751\ub9cc \uc798\ud588\uc5b4\ub3c4 \uc0ac\ub78c\uc774 \uc774\ub807\uac8c \ub9ce\uc774 \uc8fd\uc9c0\ub294 \uc54a\uc558\uc744 \uac83\"\uc774\ub77c\uba70 \"\uc138\uc6d4\ud638 \uc774\ud6c4\uc5d0\ub294 \uc880 \ub098\uc544\uc9c0\ub294\uac00 \ud588\ub294\ub370 \uc548\uc804 \uc2dc\uc2a4\ud15c\uc774 \ub098\uc544\uc9c4 \uac83\uc774 \uc5c6\ub2e4\"\ub294 \uac83\ub3c4 \uc9c0\uc801\ud588\ub2e4. \ub610\ud55c \uc138\uc6d4\ud638 \uc0ac\uace0\uc5d0 \ube57\ub300 \ud654\uc7ac \ubc1c\uc0dd 2\uc2dc\uac04\uc774 \ub118\uc5b4\uc11c\uc57c \uc218\uc2b5\ud558\ub77c \uc9c0\uc2dc\ud588\ub358 \uac83\uc744 \ube44\ud310\ud588\uace0 5\uc2dc 20\ubd84\uae4c\uc9c0 \uc0dd\uc874\ud55c \ud76c\uc0dd\uc790\ub4e4\uacfc \ud1b5\ud654\ud55c \uc720\uc871\ub4e4\uc774 \uc788\uc5c8\ub2e4\uba74\uc11c, 4\uc2dc 7\ubd84\uc5d0 \uc804\ud654\ud588\uc744 \ub54c \uc9c0\ub839\uc774 \ub0b4\ub824\uc9c0\uace0 \uc9c0\uce68\ub300\ub85c \ud588\ub2e4\uba74 \uc778\uba85 \ud53c\ud574\uac00 \uc801\uc5c8\uc744 \uac83\uc774\ub77c \uc8fc\uc7a5\ud588\ub2e4.", "paragraph_answer": " 2017\ub144 12\uc6d4 22\uc77c \ubb38\uc7ac\uc778 \ub300\ud1b5\ub839\uc774 \ud604\uc7a5\uc744 \ubc29\ubb38\ud558\uc790 \uac10\uc815\uc801\uc73c\ub85c \uaca9\uc559\ub3fc \uc788\ub358 \uc77c\ubd80 \uc720\uac00\uc871\uc740 \ub300\uc120 \uad6c\ud638\uc600\ub358 '\uc0ac\ub78c\uc774 \uba3c\uc800\ub2e4'\uc744 \uc0c1\uae30\uc2dc\ud0a4\uba74\uc11c \"\uc774\ubc88\uc5d0 \uc0ac\ub78c\uc774\uace0 \ubb50\uace0 \uc5c6\uc5c8\ub2e4\"\uba70 \ubd84\ub178\ud588\uace0 \"\uc815\ubd80\uac00 \uc774\ub7f0 \uc2dd\uc73c\ub85c \ub300\ucc98\ud558\ub294 \uac8c \ud55c\ub450 \ubc88\uc774\ub0d0\", \"\ucd08\uae30\ub300\uc751\ub9cc \uc798\ud588\uc5b4\ub3c4 \uc0ac\ub78c\uc774 \uc774\ub807\uac8c \ub9ce\uc774 \uc8fd\uc9c0\ub294 \uc54a\uc558\uc744 \uac83\"\uc774\ub77c\uba70 \"\uc138\uc6d4\ud638 \uc774\ud6c4\uc5d0\ub294 \uc880 \ub098\uc544\uc9c0\ub294\uac00 \ud588\ub294\ub370 \uc548\uc804 \uc2dc\uc2a4\ud15c\uc774 \ub098\uc544\uc9c4 \uac83\uc774 \uc5c6\ub2e4\"\ub294 \uac83\ub3c4 \uc9c0\uc801\ud588\ub2e4. \ub610\ud55c \uc138\uc6d4\ud638 \uc0ac\uace0\uc5d0 \ube57\ub300 \ud654\uc7ac \ubc1c\uc0dd 2\uc2dc\uac04\uc774 \ub118\uc5b4\uc11c\uc57c \uc218\uc2b5\ud558\ub77c \uc9c0\uc2dc\ud588\ub358 \uac83\uc744 \ube44\ud310\ud588\uace0 5\uc2dc 20\ubd84\uae4c\uc9c0 \uc0dd\uc874\ud55c \ud76c\uc0dd\uc790\ub4e4\uacfc \ud1b5\ud654\ud55c \uc720\uc871\ub4e4\uc774 \uc788\uc5c8\ub2e4\uba74\uc11c, 4\uc2dc 7\ubd84\uc5d0 \uc804\ud654\ud588\uc744 \ub54c \uc9c0\ub839\uc774 \ub0b4\ub824\uc9c0\uace0 \uc9c0\uce68\ub300\ub85c \ud588\ub2e4\uba74 \uc778\uba85 \ud53c\ud574\uac00 \uc801\uc5c8\uc744 \uac83\uc774\ub77c \uc8fc\uc7a5\ud588\ub2e4.", "sentence_answer": " 2017\ub144 12\uc6d4 22\uc77c \ubb38\uc7ac\uc778 \ub300\ud1b5\ub839\uc774 \ud604\uc7a5\uc744 \ubc29\ubb38\ud558\uc790 \uac10\uc815\uc801\uc73c\ub85c \uaca9\uc559\ub3fc \uc788\ub358 \uc77c\ubd80 \uc720\uac00\uc871\uc740 \ub300\uc120 \uad6c\ud638\uc600\ub358 '\uc0ac\ub78c\uc774 \uba3c\uc800\ub2e4'\uc744 \uc0c1\uae30\uc2dc\ud0a4\uba74\uc11c \"\uc774\ubc88\uc5d0 \uc0ac\ub78c\uc774\uace0 \ubb50\uace0 \uc5c6\uc5c8\ub2e4\"\uba70 \ubd84\ub178\ud588\uace0 \"\uc815\ubd80\uac00 \uc774\ub7f0 \uc2dd\uc73c\ub85c \ub300\ucc98\ud558\ub294 \uac8c \ud55c\ub450 \ubc88\uc774\ub0d0\", \"\ucd08\uae30\ub300\uc751\ub9cc \uc798\ud588\uc5b4\ub3c4 \uc0ac\ub78c\uc774 \uc774\ub807\uac8c \ub9ce\uc774 \uc8fd\uc9c0\ub294 \uc54a\uc558\uc744 \uac83\"\uc774\ub77c\uba70 \"\uc138\uc6d4\ud638 \uc774\ud6c4\uc5d0\ub294 \uc880 \ub098\uc544\uc9c0\ub294\uac00 \ud588\ub294\ub370 \uc548\uc804 \uc2dc\uc2a4\ud15c\uc774 \ub098\uc544\uc9c4 \uac83\uc774 \uc5c6\ub2e4\"\ub294 \uac83\ub3c4 \uc9c0\uc801\ud588\ub2e4.", "paragraph_id": "6187310-1-0", "paragraph_question": "question: \ubb38\uc7ac\uc778\ub300\ud1b5\ub839\uc774 \ud604\uc7a5\uc744 \ubc29\ubb38\ud55c \ub0a0\uc9dc\ub294?, context: 2017\ub144 12\uc6d4 22\uc77c \ubb38\uc7ac\uc778 \ub300\ud1b5\ub839\uc774 \ud604\uc7a5\uc744 \ubc29\ubb38\ud558\uc790 \uac10\uc815\uc801\uc73c\ub85c \uaca9\uc559\ub3fc \uc788\ub358 \uc77c\ubd80 \uc720\uac00\uc871\uc740 \ub300\uc120 \uad6c\ud638\uc600\ub358 '\uc0ac\ub78c\uc774 \uba3c\uc800\ub2e4'\uc744 \uc0c1\uae30\uc2dc\ud0a4\uba74\uc11c \"\uc774\ubc88\uc5d0 \uc0ac\ub78c\uc774\uace0 \ubb50\uace0 \uc5c6\uc5c8\ub2e4\"\uba70 \ubd84\ub178\ud588\uace0 \"\uc815\ubd80\uac00 \uc774\ub7f0 \uc2dd\uc73c\ub85c \ub300\ucc98\ud558\ub294 \uac8c \ud55c\ub450 \ubc88\uc774\ub0d0\", \"\ucd08\uae30\ub300\uc751\ub9cc \uc798\ud588\uc5b4\ub3c4 \uc0ac\ub78c\uc774 \uc774\ub807\uac8c \ub9ce\uc774 \uc8fd\uc9c0\ub294 \uc54a\uc558\uc744 \uac83\"\uc774\ub77c\uba70 \"\uc138\uc6d4\ud638 \uc774\ud6c4\uc5d0\ub294 \uc880 \ub098\uc544\uc9c0\ub294\uac00 \ud588\ub294\ub370 \uc548\uc804 \uc2dc\uc2a4\ud15c\uc774 \ub098\uc544\uc9c4 \uac83\uc774 \uc5c6\ub2e4\"\ub294 \uac83\ub3c4 \uc9c0\uc801\ud588\ub2e4. \ub610\ud55c \uc138\uc6d4\ud638 \uc0ac\uace0\uc5d0 \ube57\ub300 \ud654\uc7ac \ubc1c\uc0dd 2\uc2dc\uac04\uc774 \ub118\uc5b4\uc11c\uc57c \uc218\uc2b5\ud558\ub77c \uc9c0\uc2dc\ud588\ub358 \uac83\uc744 \ube44\ud310\ud588\uace0 5\uc2dc 20\ubd84\uae4c\uc9c0 \uc0dd\uc874\ud55c \ud76c\uc0dd\uc790\ub4e4\uacfc \ud1b5\ud654\ud55c \uc720\uc871\ub4e4\uc774 \uc788\uc5c8\ub2e4\uba74\uc11c, 4\uc2dc 7\ubd84\uc5d0 \uc804\ud654\ud588\uc744 \ub54c \uc9c0\ub839\uc774 \ub0b4\ub824\uc9c0\uace0 \uc9c0\uce68\ub300\ub85c \ud588\ub2e4\uba74 \uc778\uba85 \ud53c\ud574\uac00 \uc801\uc5c8\uc744 \uac83\uc774\ub77c \uc8fc\uc7a5\ud588\ub2e4."} {"question": "1980\ub144 \ub3d9\uacc4 \uc62c\ub9bc\ud53d\uc758 \uac1c\ucd5c\uad6d\uc740?", "paragraph": "1980\ub144 \ub3d9\uacc4 \uc62c\ub9bc\ud53d\uc740 1932\ub144 \ub3d9\uacc4\uc62c\ub9bc\ud53d\uc744 \uac1c\ucd5c\ud55c \uc804\ub825\uc774 \uc788\uc5c8\ub358 \ubbf8\uad6d\uc758 \ub808\uc774\ud06c\ud50c\ub798\uc2dc\ub4dc\uc5d0\uc11c \ub2e4\uc2dc \uc5f4\ub838\ub2e4. \uc774 \ub300\ud68c\uc5d0\uc11c\ub294 \ucc98\uc74c\uc73c\ub85c \ub3d9\uacc4\uc62c\ub9bc\ud53d \ubcf4\uc774\ucf67 \uc0ac\ub840\uac00 \ub098\uc654\ub294\ub370, IOC\uac00 \uc911\ud654\ubbfc\uad6d \uc120\uc218\ub2e8 \uce21\uc5d0 \uc120\uc218\ub2e8\uba85\uacfc \uad6d\uac00\ub97c \ubc14\uafc0 \uac83\uc744 \uc9c0\uc2dc\ud558\ub294 \uad8c\uace0\ub97c \ub0b4\ub9ac\uba74\uc11c \uc911\ud654\ubbfc\uad6d \uc120\uc218\ub4e4\uc774 \ub300\ud68c \ucc38\uac00\ub97c \uc790\uc9c4\ucde8\uc18c\ud55c \uac83\uc774\uc5c8\ub2e4. \uc774 \ub2f9\uc2dc IOC\ub294 \uc911\ud654\uc778\ubbfc\uacf5\ud654\uad6d\uc744 \uc62c\ub9bc\ud53d \ubb34\ub300\uc5d0 \ucd08\ub300\ud558\uace0\uc790 \ud588\uace0, \uc911\ud654\uc778\ubbfc\uacf5\ud654\uad6d \uce21\uc740 \uc911\ud654\ubbfc\uad6d\uc774 \uc4f0\ub358 \uad6d\uac00\uba85 ('\uc911\uad6d')\uacfc \uad6d\uac00\ub97c \uadf8\ub300\ub85c \uc4f0\uae38 \uc6d0\ud558\ub358 \uc0c1\ud669\uc774\uc5c8\ub2e4. \uacb0\uad6d \uc911\ud654\ubbfc\uad6d \uc120\uc218\ub2e8\uc758 \ubcf4\uc774\ucf67\uc73c\ub85c \uc911\ud654\uc778\ubbfc\uacf5\ud654\uad6d \uc120\uc218\ub2e8\uc73c\ub85c \ub300\uc2e0 \uad6c\uc131\ub41c '\uc911\uad6d' \uc120\uc218\ub2e8\uc740 1952\ub144 \ud558\uacc4 \uc62c\ub9bc\ud53d\ubd80\ud130 \ubd88\ucc38\ud55c \uc774\ud6c4 \ucc98\uc74c\uc73c\ub85c \uc62c\ub9bc\ud53d\uc5d0 \ucc38\uac00\ud558\uac8c \ub418\uc5c8\ub2e4. \ud55c\ud3b8\uc73c\ub85c \ubbf8\uad6d\uc758 \uc5d0\ub9ad \ud558\uc774\ub4e0\uc774 \uc2a4\ud53c\ub4dc\uc2a4\ucf00\uc774\ud305 \uc804 5\uc885\ubaa9\uc5d0\uc11c \uc62c\ub9bc\ud53d\uacfc \uc138\uacc4 \uc2e0\uae30\ub85d\uc744 \ubaa8\ub450 \ub2ec\uc131\ud558\ub294 \uc9c4\ud48d\uacbd\uc744 \uc120\ubcf4\uc600\ub2e4. \ub610 \ub3d9\uc2dc\uc5d0 \ub2e4\uc12f \uac1c\uc778\uacbd\uae30\uc5d0\uc11c \ubaa8\ub450 \uae08\uba54\ub2ec\uc744 \ub540\uc73c\ub85c\uc11c, \ub3d9\ud558\uacc4\ub97c \ud1b5\ud2c0\uc5b4 \ud55c \ub300\ud68c\uc5d0\uc11c \uac1c\uc778\uc885\ubaa9\uc744 \uac00\uc7a5 \ub9ce\uc774 \uc6b0\uc2b9\ud55c \uae30\ub85d\uc744 \uacbd\uc2e0\ud558\uac8c \ub418\uc5c8\ub2e4. \ub610 \ub9ac\ud788\ud150\uc288\ud0c0\uc778\uc758 \ud558\ub2c8 \ubca4\uccbc\uc774 \uc54c\ud30c\uc778\uc2a4\ud0a4 \ud68c\uc804\uacfc \ub300\ud68c\uc804\uc5d0 \ucd9c\uc804\ud574 \ubaa8\ub450 \uc6b0\uc2b9\ud558\uba74\uc11c, \ub9ac\ud788\ud150\uc288\ud0c0\uc778\uc740 \uc138\uacc4\uc5d0\uc11c \uac00\uc7a5 \uc791\uc740 \uad6d\uac00 \uc911 \ud558\ub098\uc784\uc5d0\ub3c4 \ubd88\uad6c\ud558\uace0 \uc62c\ub9bc\ud53d \uae08\uba54\ub2ec\ub9ac\uc2a4\ud2b8\ub97c \ubc30\ucd9c\ud55c \uad6d\uac00\ub77c\ub294 \uc9c4\uae30\ub85d\uc744 \uc138\uc6b0\uac8c \ub418\uc5c8\ub2e4. \uc774 \ub300\ud68c\ub97c \uc5b8\uae09\ud560 \ub54c \ub193\uce60 \uc218 \uc5c6\ub294 \ub610 \ud55c\uac00\uc9c0 \ud558\uc774\ub77c\uc774\ud2b8, \uc774\ub978\ubc14 '\uc740\ubc18\uc704\uc758 \uae30\uc801'\uc740 \uc77c\ubc18 \ub300\ud559\uc120\uc218\ub85c \uad6c\uc131\ub41c \ubbf8\uad6d \uc544\uc774\uc2a4\ud558\ud0a4 \uad6d\uac00\ub300\ud45c\ud300\uc774 \ub178\ub828\ud55c \uc804\ubb38\uc120\uc218\ub85c \uad6c\uc131\ub41c \uc18c\ub828 \uc544\uc774\uc2a4\ud558\ud0a4 \uad6d\uac00\ub300\ud45c\ud300\uacfc \uacb0\uc2b9\uc804\uc5d0\uc11c \ub9de\ubd99\uace0, \ub05d\ub0b4 \uc2b9\ub9ac\ud574 \uae08\uba54\ub2ec\uc744 \ucc28\uc9c0\ud588\ub358 \uc0ac\uac74\uc774\ub2e4.", "answer": "\ubbf8\uad6d", "sentence": "1980\ub144 \ub3d9\uacc4 \uc62c\ub9bc\ud53d\uc740 1932\ub144 \ub3d9\uacc4\uc62c\ub9bc\ud53d\uc744 \uac1c\ucd5c\ud55c \uc804\ub825\uc774 \uc788\uc5c8\ub358 \ubbf8\uad6d \uc758 \ub808\uc774\ud06c\ud50c\ub798\uc2dc\ub4dc\uc5d0\uc11c \ub2e4\uc2dc \uc5f4\ub838\ub2e4.", "paragraph_sentence": " 1980\ub144 \ub3d9\uacc4 \uc62c\ub9bc\ud53d\uc740 1932\ub144 \ub3d9\uacc4\uc62c\ub9bc\ud53d\uc744 \uac1c\ucd5c\ud55c \uc804\ub825\uc774 \uc788\uc5c8\ub358 \ubbf8\uad6d \uc758 \ub808\uc774\ud06c\ud50c\ub798\uc2dc\ub4dc\uc5d0\uc11c \ub2e4\uc2dc \uc5f4\ub838\ub2e4. \uc774 \ub300\ud68c\uc5d0\uc11c\ub294 \ucc98\uc74c\uc73c\ub85c \ub3d9\uacc4\uc62c\ub9bc\ud53d \ubcf4\uc774\ucf67 \uc0ac\ub840\uac00 \ub098\uc654\ub294\ub370, IOC\uac00 \uc911\ud654\ubbfc\uad6d \uc120\uc218\ub2e8 \uce21\uc5d0 \uc120\uc218\ub2e8\uba85\uacfc \uad6d\uac00\ub97c \ubc14\uafc0 \uac83\uc744 \uc9c0\uc2dc\ud558\ub294 \uad8c\uace0\ub97c \ub0b4\ub9ac\uba74\uc11c \uc911\ud654\ubbfc\uad6d \uc120\uc218\ub4e4\uc774 \ub300\ud68c \ucc38\uac00\ub97c \uc790\uc9c4\ucde8\uc18c\ud55c \uac83\uc774\uc5c8\ub2e4. \uc774 \ub2f9\uc2dc IOC\ub294 \uc911\ud654\uc778\ubbfc\uacf5\ud654\uad6d\uc744 \uc62c\ub9bc\ud53d \ubb34\ub300\uc5d0 \ucd08\ub300\ud558\uace0\uc790 \ud588\uace0, \uc911\ud654\uc778\ubbfc\uacf5\ud654\uad6d \uce21\uc740 \uc911\ud654\ubbfc\uad6d\uc774 \uc4f0\ub358 \uad6d\uac00\uba85 ('\uc911\uad6d')\uacfc \uad6d\uac00\ub97c \uadf8\ub300\ub85c \uc4f0\uae38 \uc6d0\ud558\ub358 \uc0c1\ud669\uc774\uc5c8\ub2e4. \uacb0\uad6d \uc911\ud654\ubbfc\uad6d \uc120\uc218\ub2e8\uc758 \ubcf4\uc774\ucf67\uc73c\ub85c \uc911\ud654\uc778\ubbfc\uacf5\ud654\uad6d \uc120\uc218\ub2e8\uc73c\ub85c \ub300\uc2e0 \uad6c\uc131\ub41c '\uc911\uad6d' \uc120\uc218\ub2e8\uc740 1952\ub144 \ud558\uacc4 \uc62c\ub9bc\ud53d\ubd80\ud130 \ubd88\ucc38\ud55c \uc774\ud6c4 \ucc98\uc74c\uc73c\ub85c \uc62c\ub9bc\ud53d\uc5d0 \ucc38\uac00\ud558\uac8c \ub418\uc5c8\ub2e4. \ud55c\ud3b8\uc73c\ub85c \ubbf8\uad6d\uc758 \uc5d0\ub9ad \ud558\uc774\ub4e0\uc774 \uc2a4\ud53c\ub4dc\uc2a4\ucf00\uc774\ud305 \uc804 5\uc885\ubaa9\uc5d0\uc11c \uc62c\ub9bc\ud53d\uacfc \uc138\uacc4 \uc2e0\uae30\ub85d\uc744 \ubaa8\ub450 \ub2ec\uc131\ud558\ub294 \uc9c4\ud48d\uacbd\uc744 \uc120\ubcf4\uc600\ub2e4. \ub610 \ub3d9\uc2dc\uc5d0 \ub2e4\uc12f \uac1c\uc778\uacbd\uae30\uc5d0\uc11c \ubaa8\ub450 \uae08\uba54\ub2ec\uc744 \ub540\uc73c\ub85c\uc11c, \ub3d9\ud558\uacc4\ub97c \ud1b5\ud2c0\uc5b4 \ud55c \ub300\ud68c\uc5d0\uc11c \uac1c\uc778\uc885\ubaa9\uc744 \uac00\uc7a5 \ub9ce\uc774 \uc6b0\uc2b9\ud55c \uae30\ub85d\uc744 \uacbd\uc2e0\ud558\uac8c \ub418\uc5c8\ub2e4. \ub610 \ub9ac\ud788\ud150\uc288\ud0c0\uc778\uc758 \ud558\ub2c8 \ubca4\uccbc\uc774 \uc54c\ud30c\uc778\uc2a4\ud0a4 \ud68c\uc804\uacfc \ub300\ud68c\uc804\uc5d0 \ucd9c\uc804\ud574 \ubaa8\ub450 \uc6b0\uc2b9\ud558\uba74\uc11c, \ub9ac\ud788\ud150\uc288\ud0c0\uc778\uc740 \uc138\uacc4\uc5d0\uc11c \uac00\uc7a5 \uc791\uc740 \uad6d\uac00 \uc911 \ud558\ub098\uc784\uc5d0\ub3c4 \ubd88\uad6c\ud558\uace0 \uc62c\ub9bc\ud53d \uae08\uba54\ub2ec\ub9ac\uc2a4\ud2b8\ub97c \ubc30\ucd9c\ud55c \uad6d\uac00\ub77c\ub294 \uc9c4\uae30\ub85d\uc744 \uc138\uc6b0\uac8c \ub418\uc5c8\ub2e4. \uc774 \ub300\ud68c\ub97c \uc5b8\uae09\ud560 \ub54c \ub193\uce60 \uc218 \uc5c6\ub294 \ub610 \ud55c\uac00\uc9c0 \ud558\uc774\ub77c\uc774\ud2b8, \uc774\ub978\ubc14 '\uc740\ubc18\uc704\uc758 \uae30\uc801'\uc740 \uc77c\ubc18 \ub300\ud559\uc120\uc218\ub85c \uad6c\uc131\ub41c \ubbf8\uad6d \uc544\uc774\uc2a4\ud558\ud0a4 \uad6d\uac00\ub300\ud45c\ud300\uc774 \ub178\ub828\ud55c \uc804\ubb38\uc120\uc218\ub85c \uad6c\uc131\ub41c \uc18c\ub828 \uc544\uc774\uc2a4\ud558\ud0a4 \uad6d\uac00\ub300\ud45c\ud300\uacfc \uacb0\uc2b9\uc804\uc5d0\uc11c \ub9de\ubd99\uace0, \ub05d\ub0b4 \uc2b9\ub9ac\ud574 \uae08\uba54\ub2ec\uc744 \ucc28\uc9c0\ud588\ub358 \uc0ac\uac74\uc774\ub2e4.", "paragraph_answer": "1980\ub144 \ub3d9\uacc4 \uc62c\ub9bc\ud53d\uc740 1932\ub144 \ub3d9\uacc4\uc62c\ub9bc\ud53d\uc744 \uac1c\ucd5c\ud55c \uc804\ub825\uc774 \uc788\uc5c8\ub358 \ubbf8\uad6d \uc758 \ub808\uc774\ud06c\ud50c\ub798\uc2dc\ub4dc\uc5d0\uc11c \ub2e4\uc2dc \uc5f4\ub838\ub2e4. \uc774 \ub300\ud68c\uc5d0\uc11c\ub294 \ucc98\uc74c\uc73c\ub85c \ub3d9\uacc4\uc62c\ub9bc\ud53d \ubcf4\uc774\ucf67 \uc0ac\ub840\uac00 \ub098\uc654\ub294\ub370, IOC\uac00 \uc911\ud654\ubbfc\uad6d \uc120\uc218\ub2e8 \uce21\uc5d0 \uc120\uc218\ub2e8\uba85\uacfc \uad6d\uac00\ub97c \ubc14\uafc0 \uac83\uc744 \uc9c0\uc2dc\ud558\ub294 \uad8c\uace0\ub97c \ub0b4\ub9ac\uba74\uc11c \uc911\ud654\ubbfc\uad6d \uc120\uc218\ub4e4\uc774 \ub300\ud68c \ucc38\uac00\ub97c \uc790\uc9c4\ucde8\uc18c\ud55c \uac83\uc774\uc5c8\ub2e4. \uc774 \ub2f9\uc2dc IOC\ub294 \uc911\ud654\uc778\ubbfc\uacf5\ud654\uad6d\uc744 \uc62c\ub9bc\ud53d \ubb34\ub300\uc5d0 \ucd08\ub300\ud558\uace0\uc790 \ud588\uace0, \uc911\ud654\uc778\ubbfc\uacf5\ud654\uad6d \uce21\uc740 \uc911\ud654\ubbfc\uad6d\uc774 \uc4f0\ub358 \uad6d\uac00\uba85 ('\uc911\uad6d')\uacfc \uad6d\uac00\ub97c \uadf8\ub300\ub85c \uc4f0\uae38 \uc6d0\ud558\ub358 \uc0c1\ud669\uc774\uc5c8\ub2e4. \uacb0\uad6d \uc911\ud654\ubbfc\uad6d \uc120\uc218\ub2e8\uc758 \ubcf4\uc774\ucf67\uc73c\ub85c \uc911\ud654\uc778\ubbfc\uacf5\ud654\uad6d \uc120\uc218\ub2e8\uc73c\ub85c \ub300\uc2e0 \uad6c\uc131\ub41c '\uc911\uad6d' \uc120\uc218\ub2e8\uc740 1952\ub144 \ud558\uacc4 \uc62c\ub9bc\ud53d\ubd80\ud130 \ubd88\ucc38\ud55c \uc774\ud6c4 \ucc98\uc74c\uc73c\ub85c \uc62c\ub9bc\ud53d\uc5d0 \ucc38\uac00\ud558\uac8c \ub418\uc5c8\ub2e4. \ud55c\ud3b8\uc73c\ub85c \ubbf8\uad6d\uc758 \uc5d0\ub9ad \ud558\uc774\ub4e0\uc774 \uc2a4\ud53c\ub4dc\uc2a4\ucf00\uc774\ud305 \uc804 5\uc885\ubaa9\uc5d0\uc11c \uc62c\ub9bc\ud53d\uacfc \uc138\uacc4 \uc2e0\uae30\ub85d\uc744 \ubaa8\ub450 \ub2ec\uc131\ud558\ub294 \uc9c4\ud48d\uacbd\uc744 \uc120\ubcf4\uc600\ub2e4. \ub610 \ub3d9\uc2dc\uc5d0 \ub2e4\uc12f \uac1c\uc778\uacbd\uae30\uc5d0\uc11c \ubaa8\ub450 \uae08\uba54\ub2ec\uc744 \ub540\uc73c\ub85c\uc11c, \ub3d9\ud558\uacc4\ub97c \ud1b5\ud2c0\uc5b4 \ud55c \ub300\ud68c\uc5d0\uc11c \uac1c\uc778\uc885\ubaa9\uc744 \uac00\uc7a5 \ub9ce\uc774 \uc6b0\uc2b9\ud55c \uae30\ub85d\uc744 \uacbd\uc2e0\ud558\uac8c \ub418\uc5c8\ub2e4. \ub610 \ub9ac\ud788\ud150\uc288\ud0c0\uc778\uc758 \ud558\ub2c8 \ubca4\uccbc\uc774 \uc54c\ud30c\uc778\uc2a4\ud0a4 \ud68c\uc804\uacfc \ub300\ud68c\uc804\uc5d0 \ucd9c\uc804\ud574 \ubaa8\ub450 \uc6b0\uc2b9\ud558\uba74\uc11c, \ub9ac\ud788\ud150\uc288\ud0c0\uc778\uc740 \uc138\uacc4\uc5d0\uc11c \uac00\uc7a5 \uc791\uc740 \uad6d\uac00 \uc911 \ud558\ub098\uc784\uc5d0\ub3c4 \ubd88\uad6c\ud558\uace0 \uc62c\ub9bc\ud53d \uae08\uba54\ub2ec\ub9ac\uc2a4\ud2b8\ub97c \ubc30\ucd9c\ud55c \uad6d\uac00\ub77c\ub294 \uc9c4\uae30\ub85d\uc744 \uc138\uc6b0\uac8c \ub418\uc5c8\ub2e4. \uc774 \ub300\ud68c\ub97c \uc5b8\uae09\ud560 \ub54c \ub193\uce60 \uc218 \uc5c6\ub294 \ub610 \ud55c\uac00\uc9c0 \ud558\uc774\ub77c\uc774\ud2b8, \uc774\ub978\ubc14 '\uc740\ubc18\uc704\uc758 \uae30\uc801'\uc740 \uc77c\ubc18 \ub300\ud559\uc120\uc218\ub85c \uad6c\uc131\ub41c \ubbf8\uad6d \uc544\uc774\uc2a4\ud558\ud0a4 \uad6d\uac00\ub300\ud45c\ud300\uc774 \ub178\ub828\ud55c \uc804\ubb38\uc120\uc218\ub85c \uad6c\uc131\ub41c \uc18c\ub828 \uc544\uc774\uc2a4\ud558\ud0a4 \uad6d\uac00\ub300\ud45c\ud300\uacfc \uacb0\uc2b9\uc804\uc5d0\uc11c \ub9de\ubd99\uace0, \ub05d\ub0b4 \uc2b9\ub9ac\ud574 \uae08\uba54\ub2ec\uc744 \ucc28\uc9c0\ud588\ub358 \uc0ac\uac74\uc774\ub2e4.", "sentence_answer": "1980\ub144 \ub3d9\uacc4 \uc62c\ub9bc\ud53d\uc740 1932\ub144 \ub3d9\uacc4\uc62c\ub9bc\ud53d\uc744 \uac1c\ucd5c\ud55c \uc804\ub825\uc774 \uc788\uc5c8\ub358 \ubbf8\uad6d \uc758 \ub808\uc774\ud06c\ud50c\ub798\uc2dc\ub4dc\uc5d0\uc11c \ub2e4\uc2dc \uc5f4\ub838\ub2e4.", "paragraph_id": "6656930-8-0", "paragraph_question": "question: 1980\ub144 \ub3d9\uacc4 \uc62c\ub9bc\ud53d\uc758 \uac1c\ucd5c\uad6d\uc740?, context: 1980\ub144 \ub3d9\uacc4 \uc62c\ub9bc\ud53d\uc740 1932\ub144 \ub3d9\uacc4\uc62c\ub9bc\ud53d\uc744 \uac1c\ucd5c\ud55c \uc804\ub825\uc774 \uc788\uc5c8\ub358 \ubbf8\uad6d\uc758 \ub808\uc774\ud06c\ud50c\ub798\uc2dc\ub4dc\uc5d0\uc11c \ub2e4\uc2dc \uc5f4\ub838\ub2e4. \uc774 \ub300\ud68c\uc5d0\uc11c\ub294 \ucc98\uc74c\uc73c\ub85c \ub3d9\uacc4\uc62c\ub9bc\ud53d \ubcf4\uc774\ucf67 \uc0ac\ub840\uac00 \ub098\uc654\ub294\ub370, IOC\uac00 \uc911\ud654\ubbfc\uad6d \uc120\uc218\ub2e8 \uce21\uc5d0 \uc120\uc218\ub2e8\uba85\uacfc \uad6d\uac00\ub97c \ubc14\uafc0 \uac83\uc744 \uc9c0\uc2dc\ud558\ub294 \uad8c\uace0\ub97c \ub0b4\ub9ac\uba74\uc11c \uc911\ud654\ubbfc\uad6d \uc120\uc218\ub4e4\uc774 \ub300\ud68c \ucc38\uac00\ub97c \uc790\uc9c4\ucde8\uc18c\ud55c \uac83\uc774\uc5c8\ub2e4. \uc774 \ub2f9\uc2dc IOC\ub294 \uc911\ud654\uc778\ubbfc\uacf5\ud654\uad6d\uc744 \uc62c\ub9bc\ud53d \ubb34\ub300\uc5d0 \ucd08\ub300\ud558\uace0\uc790 \ud588\uace0, \uc911\ud654\uc778\ubbfc\uacf5\ud654\uad6d \uce21\uc740 \uc911\ud654\ubbfc\uad6d\uc774 \uc4f0\ub358 \uad6d\uac00\uba85 ('\uc911\uad6d')\uacfc \uad6d\uac00\ub97c \uadf8\ub300\ub85c \uc4f0\uae38 \uc6d0\ud558\ub358 \uc0c1\ud669\uc774\uc5c8\ub2e4. \uacb0\uad6d \uc911\ud654\ubbfc\uad6d \uc120\uc218\ub2e8\uc758 \ubcf4\uc774\ucf67\uc73c\ub85c \uc911\ud654\uc778\ubbfc\uacf5\ud654\uad6d \uc120\uc218\ub2e8\uc73c\ub85c \ub300\uc2e0 \uad6c\uc131\ub41c '\uc911\uad6d' \uc120\uc218\ub2e8\uc740 1952\ub144 \ud558\uacc4 \uc62c\ub9bc\ud53d\ubd80\ud130 \ubd88\ucc38\ud55c \uc774\ud6c4 \ucc98\uc74c\uc73c\ub85c \uc62c\ub9bc\ud53d\uc5d0 \ucc38\uac00\ud558\uac8c \ub418\uc5c8\ub2e4. \ud55c\ud3b8\uc73c\ub85c \ubbf8\uad6d\uc758 \uc5d0\ub9ad \ud558\uc774\ub4e0\uc774 \uc2a4\ud53c\ub4dc\uc2a4\ucf00\uc774\ud305 \uc804 5\uc885\ubaa9\uc5d0\uc11c \uc62c\ub9bc\ud53d\uacfc \uc138\uacc4 \uc2e0\uae30\ub85d\uc744 \ubaa8\ub450 \ub2ec\uc131\ud558\ub294 \uc9c4\ud48d\uacbd\uc744 \uc120\ubcf4\uc600\ub2e4. \ub610 \ub3d9\uc2dc\uc5d0 \ub2e4\uc12f \uac1c\uc778\uacbd\uae30\uc5d0\uc11c \ubaa8\ub450 \uae08\uba54\ub2ec\uc744 \ub540\uc73c\ub85c\uc11c, \ub3d9\ud558\uacc4\ub97c \ud1b5\ud2c0\uc5b4 \ud55c \ub300\ud68c\uc5d0\uc11c \uac1c\uc778\uc885\ubaa9\uc744 \uac00\uc7a5 \ub9ce\uc774 \uc6b0\uc2b9\ud55c \uae30\ub85d\uc744 \uacbd\uc2e0\ud558\uac8c \ub418\uc5c8\ub2e4. \ub610 \ub9ac\ud788\ud150\uc288\ud0c0\uc778\uc758 \ud558\ub2c8 \ubca4\uccbc\uc774 \uc54c\ud30c\uc778\uc2a4\ud0a4 \ud68c\uc804\uacfc \ub300\ud68c\uc804\uc5d0 \ucd9c\uc804\ud574 \ubaa8\ub450 \uc6b0\uc2b9\ud558\uba74\uc11c, \ub9ac\ud788\ud150\uc288\ud0c0\uc778\uc740 \uc138\uacc4\uc5d0\uc11c \uac00\uc7a5 \uc791\uc740 \uad6d\uac00 \uc911 \ud558\ub098\uc784\uc5d0\ub3c4 \ubd88\uad6c\ud558\uace0 \uc62c\ub9bc\ud53d \uae08\uba54\ub2ec\ub9ac\uc2a4\ud2b8\ub97c \ubc30\ucd9c\ud55c \uad6d\uac00\ub77c\ub294 \uc9c4\uae30\ub85d\uc744 \uc138\uc6b0\uac8c \ub418\uc5c8\ub2e4. \uc774 \ub300\ud68c\ub97c \uc5b8\uae09\ud560 \ub54c \ub193\uce60 \uc218 \uc5c6\ub294 \ub610 \ud55c\uac00\uc9c0 \ud558\uc774\ub77c\uc774\ud2b8, \uc774\ub978\ubc14 '\uc740\ubc18\uc704\uc758 \uae30\uc801'\uc740 \uc77c\ubc18 \ub300\ud559\uc120\uc218\ub85c \uad6c\uc131\ub41c \ubbf8\uad6d \uc544\uc774\uc2a4\ud558\ud0a4 \uad6d\uac00\ub300\ud45c\ud300\uc774 \ub178\ub828\ud55c \uc804\ubb38\uc120\uc218\ub85c \uad6c\uc131\ub41c \uc18c\ub828 \uc544\uc774\uc2a4\ud558\ud0a4 \uad6d\uac00\ub300\ud45c\ud300\uacfc \uacb0\uc2b9\uc804\uc5d0\uc11c \ub9de\ubd99\uace0, \ub05d\ub0b4 \uc2b9\ub9ac\ud574 \uae08\uba54\ub2ec\uc744 \ucc28\uc9c0\ud588\ub358 \uc0ac\uac74\uc774\ub2e4."} {"question": "\uc900\uacb0\uc2b9\uc5d0\uc11c \uce90\uc11c\ub9b0 \ub85c\uc774\ud130\uc640 \ubab8\uc2f8\uc6c0\uc744 \ubc8c\uc778 \uc120\uc218\ub294 \ub204\uad6c\uc778\uac00?", "paragraph": "3000m \uacc4\uc8fc\uc5d0\uc11c \uc568\ub9ac\uc2a8 \ub450\ub371, \uc568\ub9ac\uc2a8 \ubca0\uc774\ubc84, \ud0b4\ubc8c\ub9ac \ub370\ub9ad\uacfc \ud568\uaed8 \ucd9c\uc804\ud588\ub2e4. \ubbf8\uad6d \ud300\uc740 \uc900\uacb0\uc2b9\uc5d0\uc11c 4\ubd84 15.376\ucd08\ub97c \uae30\ub85d\ud558\uba70 \ub300\ud55c\ubbfc\uad6d\uc5d0 \ub4a4\uc774\uc5b4 2\uc704\ub97c \ucc28\uc9c0, \uacb0\uc2b9\uc5d0 \uc9c4\ucd9c\ud588\ub2e4. 1500m \uacb0\uc2b9\uc5d0\uc11c\ub294 \uc911\uad6d\uc758 \uc800\uc6b0\uc591, \ub300\ud55c\ubbfc\uad6d\uc758 \uc774\uc740\ubcc4\uacfc \ubc15\uc2b9\ud76c\uc5d0 \ubc00\ub824 4\uc704\ub97c \ucc28\uc9c0\ud588\ub2e4. \uce90\uc11c\ub9b0 \ub85c\uc774\ud130\ub294 \uc900\uacb0\uc2b9\uc5d0\uc11c \uc655\uba4d\uacfc \ubab8\uc2f8\uc6c0\uc744 \ubc8c\uc774\ub2e4 \uc5c9\ucf1c \ub118\uc5b4\uc84c\uace0, \ub300\ud55c\ubbfc\uad6d\uc758 \uc870\ud574\ub9ac\ub3c4 \uac19\uc774 \ub118\uc5b4\uc84c\ub2e4. \uacb0\uad6d \uc655\uba4d\uc740 \uc2e4\uaca9 \ucc98\ub9ac\ub97c \ubc1b\uc558\uace0 \uce90\uc11c\ub9b0 \ub85c\uc774\ud130\uc640 \ubc15\uc2b9\ud76c\ub294 \uc5b4\ub4dc\ubc34\ud2f0\uc9c0\ub85c \uacb0\uc2b9\uc5d0 \uc9c4\ucd9c\ud588\ub2e4. \uacb0\uc2b9\uc804\uc5d0\uc11c\ub3c4 \ubc15\uc2b9\ud76c\uc640 \ubd80\ub52a\ud788\uba70 \uac19\uc740 \ud589\ub3d9\uc744 \ubc18\ubcf5\ud588\uace0, \ub098\uc911\uc5d0 \ub450 \ubc88\uc758 \uc2e4\uc218\uc5d0 \ub300\ud574 \ub2e4\uc74c\uacfc \uac19\uc740 \uc0ac\uacfc\uc758 \ub9d0\uc744 \ud588\ub2e4. \u201c\ub0b4 \uc0dd\uac01\uc5d4 \uc5ec\uc720 \uacf5\uac04\uc774 \uc788\uc5c8\ub358 \uac83 \uac19\uc558\uc9c0\ub9cc \uadf8\ub807\uc9c0 \uc54a\uc558\ub2e4. \uc798 \ubaa8\ub974\uaca0\ub2e4. \uadf8\ub7ec\ub098 \ub0b4 \uc8fc\ubcc0\uc5d0 \uc788\ub294 \uc120\uc218\ub4e4\uc5d0\uac8c \ud3d0\ub97c \ub07c\uce5c \uc810\uc5d0 \ub300\ud574\uc11c\ub294 \ubbf8\uc548\ud568\uc744 \ub290\ub080\ub2e4.\u201d 1000m \uc608\uc120\uc5d0\uc11c \uc62c\ub9bc\ud53d \uae30\ub85d\uc744 \uc138\uc6b0\uba70 \uc870 1\uc704\ub97c \ucc28\uc9c0\ud588\ub2e4. \uadf8\ub85c\ubd80\ud130 \uc57d \ud55c \uc2dc\uac04 \ubc18 \ub4a4, 3000m \uacc4\uc8fc\uc5d0 \uc568\ub9ac\uc2a8 \ubca0\uc774\ubc84, \uc568\ub9ac\uc2a8 \ub450\ub371, \ub77c\ub098 \uac8c\ub9c1\uacfc \ud568\uaed8 \ucd9c\uc804\ud574 \ub300\ud55c\ubbfc\uad6d\uc758 \uc2e4\uaca9 \ub355\ud0dd\uc73c\ub85c \ub3d9\uba54\ub2ec\uc744 \ubaa9\uc5d0 \uac78\uc5c8\ub2e4. 1000m \uacb0\uc2b9\uc5d0\uc11c 1\ubd84 29.324\ucd08\ub97c \ub2ec\ub824, 1\ubd84 29.213\ucd08\ub97c \uae30\ub85d\ud55c \uc655\uba4d\uc5d0\uac8c \uadfc\uc18c\ud55c \ucc28\uc774\ub85c \uae08\uba54\ub2ec\uc744 \ub0b4\uc8fc\uc5c8\ub2e4.", "answer": "\uc655\uba4d", "sentence": "\uce90\uc11c\ub9b0 \ub85c\uc774\ud130\ub294 \uc900\uacb0\uc2b9\uc5d0\uc11c \uc655\uba4d \uacfc \ubab8\uc2f8\uc6c0\uc744 \ubc8c\uc774\ub2e4 \uc5c9\ucf1c \ub118\uc5b4\uc84c\uace0, \ub300\ud55c\ubbfc\uad6d\uc758 \uc870\ud574\ub9ac\ub3c4 \uac19\uc774 \ub118\uc5b4\uc84c\ub2e4.", "paragraph_sentence": "3000m \uacc4\uc8fc\uc5d0\uc11c \uc568\ub9ac\uc2a8 \ub450\ub371, \uc568\ub9ac\uc2a8 \ubca0\uc774\ubc84, \ud0b4\ubc8c\ub9ac \ub370\ub9ad\uacfc \ud568\uaed8 \ucd9c\uc804\ud588\ub2e4. \ubbf8\uad6d \ud300\uc740 \uc900\uacb0\uc2b9\uc5d0\uc11c 4\ubd84 15.376\ucd08\ub97c \uae30\ub85d\ud558\uba70 \ub300\ud55c\ubbfc\uad6d\uc5d0 \ub4a4\uc774\uc5b4 2\uc704\ub97c \ucc28\uc9c0, \uacb0\uc2b9\uc5d0 \uc9c4\ucd9c\ud588\ub2e4. 1500m \uacb0\uc2b9\uc5d0\uc11c\ub294 \uc911\uad6d\uc758 \uc800\uc6b0\uc591, \ub300\ud55c\ubbfc\uad6d\uc758 \uc774\uc740\ubcc4\uacfc \ubc15\uc2b9\ud76c\uc5d0 \ubc00\ub824 4\uc704\ub97c \ucc28\uc9c0\ud588\ub2e4. \uce90\uc11c\ub9b0 \ub85c\uc774\ud130\ub294 \uc900\uacb0\uc2b9\uc5d0\uc11c \uc655\uba4d \uacfc \ubab8\uc2f8\uc6c0\uc744 \ubc8c\uc774\ub2e4 \uc5c9\ucf1c \ub118\uc5b4\uc84c\uace0, \ub300\ud55c\ubbfc\uad6d\uc758 \uc870\ud574\ub9ac\ub3c4 \uac19\uc774 \ub118\uc5b4\uc84c\ub2e4. \uacb0\uad6d \uc655\uba4d\uc740 \uc2e4\uaca9 \ucc98\ub9ac\ub97c \ubc1b\uc558\uace0 \uce90\uc11c\ub9b0 \ub85c\uc774\ud130\uc640 \ubc15\uc2b9\ud76c\ub294 \uc5b4\ub4dc\ubc34\ud2f0\uc9c0\ub85c \uacb0\uc2b9\uc5d0 \uc9c4\ucd9c\ud588\ub2e4. \uacb0\uc2b9\uc804\uc5d0\uc11c\ub3c4 \ubc15\uc2b9\ud76c\uc640 \ubd80\ub52a\ud788\uba70 \uac19\uc740 \ud589\ub3d9\uc744 \ubc18\ubcf5\ud588\uace0, \ub098\uc911\uc5d0 \ub450 \ubc88\uc758 \uc2e4\uc218\uc5d0 \ub300\ud574 \ub2e4\uc74c\uacfc \uac19\uc740 \uc0ac\uacfc\uc758 \ub9d0\uc744 \ud588\ub2e4. \u201c\ub0b4 \uc0dd\uac01\uc5d4 \uc5ec\uc720 \uacf5\uac04\uc774 \uc788\uc5c8\ub358 \uac83 \uac19\uc558\uc9c0\ub9cc \uadf8\ub807\uc9c0 \uc54a\uc558\ub2e4. \uc798 \ubaa8\ub974\uaca0\ub2e4. \uadf8\ub7ec\ub098 \ub0b4 \uc8fc\ubcc0\uc5d0 \uc788\ub294 \uc120\uc218\ub4e4\uc5d0\uac8c \ud3d0\ub97c \ub07c\uce5c \uc810\uc5d0 \ub300\ud574\uc11c\ub294 \ubbf8\uc548\ud568\uc744 \ub290\ub080\ub2e4. \u201d 1000m \uc608\uc120\uc5d0\uc11c \uc62c\ub9bc\ud53d \uae30\ub85d\uc744 \uc138\uc6b0\uba70 \uc870 1\uc704\ub97c \ucc28\uc9c0\ud588\ub2e4. \uadf8\ub85c\ubd80\ud130 \uc57d \ud55c \uc2dc\uac04 \ubc18 \ub4a4, 3000m \uacc4\uc8fc\uc5d0 \uc568\ub9ac\uc2a8 \ubca0\uc774\ubc84, \uc568\ub9ac\uc2a8 \ub450\ub371, \ub77c\ub098 \uac8c\ub9c1\uacfc \ud568\uaed8 \ucd9c\uc804\ud574 \ub300\ud55c\ubbfc\uad6d\uc758 \uc2e4\uaca9 \ub355\ud0dd\uc73c\ub85c \ub3d9\uba54\ub2ec\uc744 \ubaa9\uc5d0 \uac78\uc5c8\ub2e4. 1000m \uacb0\uc2b9\uc5d0\uc11c 1\ubd84 29.324\ucd08\ub97c \ub2ec\ub824, 1\ubd84 29.213\ucd08\ub97c \uae30\ub85d\ud55c \uc655\uba4d\uc5d0\uac8c \uadfc\uc18c\ud55c \ucc28\uc774\ub85c \uae08\uba54\ub2ec\uc744 \ub0b4\uc8fc\uc5c8\ub2e4.", "paragraph_answer": "3000m \uacc4\uc8fc\uc5d0\uc11c \uc568\ub9ac\uc2a8 \ub450\ub371, \uc568\ub9ac\uc2a8 \ubca0\uc774\ubc84, \ud0b4\ubc8c\ub9ac \ub370\ub9ad\uacfc \ud568\uaed8 \ucd9c\uc804\ud588\ub2e4. \ubbf8\uad6d \ud300\uc740 \uc900\uacb0\uc2b9\uc5d0\uc11c 4\ubd84 15.376\ucd08\ub97c \uae30\ub85d\ud558\uba70 \ub300\ud55c\ubbfc\uad6d\uc5d0 \ub4a4\uc774\uc5b4 2\uc704\ub97c \ucc28\uc9c0, \uacb0\uc2b9\uc5d0 \uc9c4\ucd9c\ud588\ub2e4. 1500m \uacb0\uc2b9\uc5d0\uc11c\ub294 \uc911\uad6d\uc758 \uc800\uc6b0\uc591, \ub300\ud55c\ubbfc\uad6d\uc758 \uc774\uc740\ubcc4\uacfc \ubc15\uc2b9\ud76c\uc5d0 \ubc00\ub824 4\uc704\ub97c \ucc28\uc9c0\ud588\ub2e4. \uce90\uc11c\ub9b0 \ub85c\uc774\ud130\ub294 \uc900\uacb0\uc2b9\uc5d0\uc11c \uc655\uba4d \uacfc \ubab8\uc2f8\uc6c0\uc744 \ubc8c\uc774\ub2e4 \uc5c9\ucf1c \ub118\uc5b4\uc84c\uace0, \ub300\ud55c\ubbfc\uad6d\uc758 \uc870\ud574\ub9ac\ub3c4 \uac19\uc774 \ub118\uc5b4\uc84c\ub2e4. \uacb0\uad6d \uc655\uba4d\uc740 \uc2e4\uaca9 \ucc98\ub9ac\ub97c \ubc1b\uc558\uace0 \uce90\uc11c\ub9b0 \ub85c\uc774\ud130\uc640 \ubc15\uc2b9\ud76c\ub294 \uc5b4\ub4dc\ubc34\ud2f0\uc9c0\ub85c \uacb0\uc2b9\uc5d0 \uc9c4\ucd9c\ud588\ub2e4. \uacb0\uc2b9\uc804\uc5d0\uc11c\ub3c4 \ubc15\uc2b9\ud76c\uc640 \ubd80\ub52a\ud788\uba70 \uac19\uc740 \ud589\ub3d9\uc744 \ubc18\ubcf5\ud588\uace0, \ub098\uc911\uc5d0 \ub450 \ubc88\uc758 \uc2e4\uc218\uc5d0 \ub300\ud574 \ub2e4\uc74c\uacfc \uac19\uc740 \uc0ac\uacfc\uc758 \ub9d0\uc744 \ud588\ub2e4. \u201c\ub0b4 \uc0dd\uac01\uc5d4 \uc5ec\uc720 \uacf5\uac04\uc774 \uc788\uc5c8\ub358 \uac83 \uac19\uc558\uc9c0\ub9cc \uadf8\ub807\uc9c0 \uc54a\uc558\ub2e4. \uc798 \ubaa8\ub974\uaca0\ub2e4. \uadf8\ub7ec\ub098 \ub0b4 \uc8fc\ubcc0\uc5d0 \uc788\ub294 \uc120\uc218\ub4e4\uc5d0\uac8c \ud3d0\ub97c \ub07c\uce5c \uc810\uc5d0 \ub300\ud574\uc11c\ub294 \ubbf8\uc548\ud568\uc744 \ub290\ub080\ub2e4.\u201d 1000m \uc608\uc120\uc5d0\uc11c \uc62c\ub9bc\ud53d \uae30\ub85d\uc744 \uc138\uc6b0\uba70 \uc870 1\uc704\ub97c \ucc28\uc9c0\ud588\ub2e4. \uadf8\ub85c\ubd80\ud130 \uc57d \ud55c \uc2dc\uac04 \ubc18 \ub4a4, 3000m \uacc4\uc8fc\uc5d0 \uc568\ub9ac\uc2a8 \ubca0\uc774\ubc84, \uc568\ub9ac\uc2a8 \ub450\ub371, \ub77c\ub098 \uac8c\ub9c1\uacfc \ud568\uaed8 \ucd9c\uc804\ud574 \ub300\ud55c\ubbfc\uad6d\uc758 \uc2e4\uaca9 \ub355\ud0dd\uc73c\ub85c \ub3d9\uba54\ub2ec\uc744 \ubaa9\uc5d0 \uac78\uc5c8\ub2e4. 1000m \uacb0\uc2b9\uc5d0\uc11c 1\ubd84 29.324\ucd08\ub97c \ub2ec\ub824, 1\ubd84 29.213\ucd08\ub97c \uae30\ub85d\ud55c \uc655\uba4d\uc5d0\uac8c \uadfc\uc18c\ud55c \ucc28\uc774\ub85c \uae08\uba54\ub2ec\uc744 \ub0b4\uc8fc\uc5c8\ub2e4.", "sentence_answer": "\uce90\uc11c\ub9b0 \ub85c\uc774\ud130\ub294 \uc900\uacb0\uc2b9\uc5d0\uc11c \uc655\uba4d \uacfc \ubab8\uc2f8\uc6c0\uc744 \ubc8c\uc774\ub2e4 \uc5c9\ucf1c \ub118\uc5b4\uc84c\uace0, \ub300\ud55c\ubbfc\uad6d\uc758 \uc870\ud574\ub9ac\ub3c4 \uac19\uc774 \ub118\uc5b4\uc84c\ub2e4.", "paragraph_id": "6561573-0-1", "paragraph_question": "question: \uc900\uacb0\uc2b9\uc5d0\uc11c \uce90\uc11c\ub9b0 \ub85c\uc774\ud130\uc640 \ubab8\uc2f8\uc6c0\uc744 \ubc8c\uc778 \uc120\uc218\ub294 \ub204\uad6c\uc778\uac00?, context: 3000m \uacc4\uc8fc\uc5d0\uc11c \uc568\ub9ac\uc2a8 \ub450\ub371, \uc568\ub9ac\uc2a8 \ubca0\uc774\ubc84, \ud0b4\ubc8c\ub9ac \ub370\ub9ad\uacfc \ud568\uaed8 \ucd9c\uc804\ud588\ub2e4. \ubbf8\uad6d \ud300\uc740 \uc900\uacb0\uc2b9\uc5d0\uc11c 4\ubd84 15.376\ucd08\ub97c \uae30\ub85d\ud558\uba70 \ub300\ud55c\ubbfc\uad6d\uc5d0 \ub4a4\uc774\uc5b4 2\uc704\ub97c \ucc28\uc9c0, \uacb0\uc2b9\uc5d0 \uc9c4\ucd9c\ud588\ub2e4. 1500m \uacb0\uc2b9\uc5d0\uc11c\ub294 \uc911\uad6d\uc758 \uc800\uc6b0\uc591, \ub300\ud55c\ubbfc\uad6d\uc758 \uc774\uc740\ubcc4\uacfc \ubc15\uc2b9\ud76c\uc5d0 \ubc00\ub824 4\uc704\ub97c \ucc28\uc9c0\ud588\ub2e4. \uce90\uc11c\ub9b0 \ub85c\uc774\ud130\ub294 \uc900\uacb0\uc2b9\uc5d0\uc11c \uc655\uba4d\uacfc \ubab8\uc2f8\uc6c0\uc744 \ubc8c\uc774\ub2e4 \uc5c9\ucf1c \ub118\uc5b4\uc84c\uace0, \ub300\ud55c\ubbfc\uad6d\uc758 \uc870\ud574\ub9ac\ub3c4 \uac19\uc774 \ub118\uc5b4\uc84c\ub2e4. \uacb0\uad6d \uc655\uba4d\uc740 \uc2e4\uaca9 \ucc98\ub9ac\ub97c \ubc1b\uc558\uace0 \uce90\uc11c\ub9b0 \ub85c\uc774\ud130\uc640 \ubc15\uc2b9\ud76c\ub294 \uc5b4\ub4dc\ubc34\ud2f0\uc9c0\ub85c \uacb0\uc2b9\uc5d0 \uc9c4\ucd9c\ud588\ub2e4. \uacb0\uc2b9\uc804\uc5d0\uc11c\ub3c4 \ubc15\uc2b9\ud76c\uc640 \ubd80\ub52a\ud788\uba70 \uac19\uc740 \ud589\ub3d9\uc744 \ubc18\ubcf5\ud588\uace0, \ub098\uc911\uc5d0 \ub450 \ubc88\uc758 \uc2e4\uc218\uc5d0 \ub300\ud574 \ub2e4\uc74c\uacfc \uac19\uc740 \uc0ac\uacfc\uc758 \ub9d0\uc744 \ud588\ub2e4. \u201c\ub0b4 \uc0dd\uac01\uc5d4 \uc5ec\uc720 \uacf5\uac04\uc774 \uc788\uc5c8\ub358 \uac83 \uac19\uc558\uc9c0\ub9cc \uadf8\ub807\uc9c0 \uc54a\uc558\ub2e4. \uc798 \ubaa8\ub974\uaca0\ub2e4. \uadf8\ub7ec\ub098 \ub0b4 \uc8fc\ubcc0\uc5d0 \uc788\ub294 \uc120\uc218\ub4e4\uc5d0\uac8c \ud3d0\ub97c \ub07c\uce5c \uc810\uc5d0 \ub300\ud574\uc11c\ub294 \ubbf8\uc548\ud568\uc744 \ub290\ub080\ub2e4.\u201d 1000m \uc608\uc120\uc5d0\uc11c \uc62c\ub9bc\ud53d \uae30\ub85d\uc744 \uc138\uc6b0\uba70 \uc870 1\uc704\ub97c \ucc28\uc9c0\ud588\ub2e4. \uadf8\ub85c\ubd80\ud130 \uc57d \ud55c \uc2dc\uac04 \ubc18 \ub4a4, 3000m \uacc4\uc8fc\uc5d0 \uc568\ub9ac\uc2a8 \ubca0\uc774\ubc84, \uc568\ub9ac\uc2a8 \ub450\ub371, \ub77c\ub098 \uac8c\ub9c1\uacfc \ud568\uaed8 \ucd9c\uc804\ud574 \ub300\ud55c\ubbfc\uad6d\uc758 \uc2e4\uaca9 \ub355\ud0dd\uc73c\ub85c \ub3d9\uba54\ub2ec\uc744 \ubaa9\uc5d0 \uac78\uc5c8\ub2e4. 1000m \uacb0\uc2b9\uc5d0\uc11c 1\ubd84 29.324\ucd08\ub97c \ub2ec\ub824, 1\ubd84 29.213\ucd08\ub97c \uae30\ub85d\ud55c \uc655\uba4d\uc5d0\uac8c \uadfc\uc18c\ud55c \ucc28\uc774\ub85c \uae08\uba54\ub2ec\uc744 \ub0b4\uc8fc\uc5c8\ub2e4."} {"question": "\uac00\uc0c1\ud604\uc2e4\uc5d0 \ub300\ud55c \uc801\uadf9\uc801\uc778 \ucca0\ud559\uc801 \ub17c\uc758\uc640 \uace0\ucc30\uc758 \ud544\uc694\uc131\uc774 \ub300\ub450\ub41c \uac83\uc740 \uc6d0\ucd08\uc801\uc73c\ub85c \ubb34\uc5c7 \ub54c\ubb38\uc774\uac00?", "paragraph": "\uac00\uc0c1\ud604\uc2e4\uc5d0 \ub300\ud55c \ucca0\ud559\uc801 \ub17c\uc758\uac00 \ubcf8\uaca9\ud654\ub418\uae30 \uc774\uc804\ubd80\ud130 \uac00\uc0c1\ud604\uc2e4\uc740 SF\uc601\ud654\ub098 \ubb38\ud559\uc791\ud488, \uae30\uc220 \ubd84\uc57c\uc5d0\uc11c \ub2e4\uc591\ud55c \uc774\ub984\uc73c\ub85c \uba85\uba85\ub418\uba70 \uc0ac\ub78c\ub4e4\uc5d0\uac8c \uc778\uc2dd\ub418\uc5c8\ub2e4. \ub2d0 \uc2a4\ud2f0\ube10\uc2a8\uc758 \uc0ac\uc774\ubc84 \ud391\ud06c \uc18c\uc124 \uc2a4\ub178\uc6b0 \ud06c\ub798\uc26c(1991)\uc5d0\uc11c \uba85\uba85\ub41c '\uba54\ud0c0\ubc84\uc2a4(Metaverse)'\uc640 \uc70c\ub9ac\uc5c4 \uae41\uc2a8\uc758 \uc18c\uc124 \ub274\ub85c\ub9e8\uc11c(1984)\uc5d0\uc11c \u2018\uc0ac\uc774\ubc84\uc2a4\ud398\uc774\uc2a4(cyberspace)'\uac00 \uadf8 \uc608\uc774\ub2e4. \uadf8\ub7ec\ub098 \ucef4\ud4e8\ud130 \uae30\uc220\uc758 \uc9c0\uc18d\uc801 \ubc1c\ub2ec\ub85c \uc778\ud574 \uac00\uc0c1\ud604\uc2e4 \uc2dc\uc2a4\ud15c\uc774 \uc2e4\uc81c\ub85c \uc778\uac04\uc758 \uc0b6 \uc18d\uc5d0\uc11c \uc601\ud5a5\ub825\uc744 \ubc1c\ud718\ud558\uace0, \ud604\ub300\uc778\uc758 \uc0b6\uc758 \ubb34\ub300\uac00 \uac00\uc0c1\uc758 \uacf5\uac04\uc73c\ub85c\uae4c\uc9c0 \ud655\uc7a5\ub428\uc5d0 \ub530\ub77c \uac00\uc0c1\ud604\uc2e4\uc5d0 \ub300\ud55c \uc801\uadf9\uc801\uc778 \ucca0\ud559\uc801 \ub17c\uc758\uc640 \uace0\ucc30\uc758 \ud544\uc694\uc131\uc774 \ub300\ub450\ub418\uc5c8\ub2e4. \uac00\uc0c1\ud604\uc2e4\uc5d0 \ub300\ud55c \ucca0\ud559\uc801 \ub17c\uc758\ub294 '\uac00\uc0c1\ud604\uc2e4'\uc5d0\uc11c \uac00\uc0c1, \uc989 '\ubc84\ucd94\uc5bc(virtual)'\uc758 \uac1c\ub150 \uc790\uccb4\uc5d0 \ub300\ud55c \uac83\ub3c4 \uc788\uace0, \uc778\uac04 \uac10\uac01\uc758 \ud55c\uacc4 \ubc0f \uac00\uc0c1\ud604\uc2e4 \uae30\uc220\uc758 \ud2b9\uc131\uacfc \uad00\ub828\ud558\uc5ec \uc874\uc7ac\ub860\uc801\uc778 \uace0\ucc30, \uc2e4\uc7ac\uc131\uacfc \uacf5\uac04\uc131 \uac1c\ub150\uc5d0 \ub300\ud55c \ub17c\uc758\ub4e4\ub3c4 \ud65c\ubc1c\ud788 \uc81c\uae30\ub418\uace0 \uc788\ub2e4. \uac00\uc0c1\ud604\uc2e4\uc5d0 \ub300\ud55c \ucca0\ud559\uc801\uc73c\ub85c \uace0\ucc30\ud55c \uc11c\uc801\uc73c\ub85c \ub9c8\uc774\ud074 \ud558\uc784(Michael Heim)\uc758 \uac00\uc0c1\ud604\uc2e4\uc758 \ucca0\ud559\uc801 \uc758\ubbf8(1993)\uac00 \uc788\ub2e4. \ub610, \uac00\uc0c1\ud604\uc2e4\uc5d0 \ub300\ud55c \uc9c1\uc811\uc801 \ub17c\uc758\ub97c \ub2e4\ub8e8\uace0 \uc788\uc9c0\ub294 \uc54a\uc9c0\ub9cc '\uac00\uc0c1\uc131'\uacfc \uad00\ub828\ud558\uc5ec \uc7a5 \ubcf4\ub4dc\ub9ac\uc57c\ub974(Jean baudrillard)\uc758 \uc2dc\ubbac\ub77c\uc2dc\uc639\uc774 \uc720\uba85\ud558\ub2e4. \uac00\uc0c1\ud604\uc2e4\uc5d0 \ub300\ud55c \ucca0\ud559\uc801 \uc0ac\uc720\ub97c \uc790\uadf9\ud558\ub294 \uc601\ud654\ub85c\ub294 \ub9e4\ud2b8\ub9ad\uc2a4, \ud1a0\ud0c8\ub9ac\ucf5c, \uacf5\uac01\uae30\ub3d9\ub300 \ub4f1\uc774 \uc788\ub2e4.", "answer": "\ucef4\ud4e8\ud130 \uae30\uc220\uc758 \uc9c0\uc18d\uc801 \ubc1c\ub2ec", "sentence": "\uadf8\ub7ec\ub098 \ucef4\ud4e8\ud130 \uae30\uc220\uc758 \uc9c0\uc18d\uc801 \ubc1c\ub2ec \ub85c \uc778\ud574 \uac00\uc0c1\ud604\uc2e4 \uc2dc\uc2a4\ud15c\uc774 \uc2e4\uc81c\ub85c \uc778\uac04\uc758 \uc0b6 \uc18d\uc5d0\uc11c \uc601\ud5a5\ub825\uc744 \ubc1c\ud718\ud558\uace0, \ud604\ub300\uc778\uc758 \uc0b6\uc758 \ubb34\ub300\uac00 \uac00\uc0c1\uc758 \uacf5\uac04\uc73c\ub85c\uae4c\uc9c0 \ud655\uc7a5\ub428\uc5d0 \ub530\ub77c \uac00\uc0c1\ud604\uc2e4\uc5d0 \ub300\ud55c \uc801\uadf9\uc801\uc778 \ucca0\ud559\uc801 \ub17c\uc758\uc640 \uace0\ucc30\uc758 \ud544\uc694\uc131\uc774 \ub300\ub450\ub418\uc5c8\ub2e4.", "paragraph_sentence": "\uac00\uc0c1\ud604\uc2e4\uc5d0 \ub300\ud55c \ucca0\ud559\uc801 \ub17c\uc758\uac00 \ubcf8\uaca9\ud654\ub418\uae30 \uc774\uc804\ubd80\ud130 \uac00\uc0c1\ud604\uc2e4\uc740 SF\uc601\ud654\ub098 \ubb38\ud559\uc791\ud488, \uae30\uc220 \ubd84\uc57c\uc5d0\uc11c \ub2e4\uc591\ud55c \uc774\ub984\uc73c\ub85c \uba85\uba85\ub418\uba70 \uc0ac\ub78c\ub4e4\uc5d0\uac8c \uc778\uc2dd\ub418\uc5c8\ub2e4. \ub2d0 \uc2a4\ud2f0\ube10\uc2a8\uc758 \uc0ac\uc774\ubc84 \ud391\ud06c \uc18c\uc124 \uc2a4\ub178\uc6b0 \ud06c\ub798\uc26c(1991)\uc5d0\uc11c \uba85\uba85\ub41c '\uba54\ud0c0\ubc84\uc2a4(Metaverse)'\uc640 \uc70c\ub9ac\uc5c4 \uae41\uc2a8\uc758 \uc18c\uc124 \ub274\ub85c\ub9e8\uc11c(1984)\uc5d0\uc11c \u2018\uc0ac\uc774\ubc84\uc2a4\ud398\uc774\uc2a4(cyberspace)'\uac00 \uadf8 \uc608\uc774\ub2e4. \uadf8\ub7ec\ub098 \ucef4\ud4e8\ud130 \uae30\uc220\uc758 \uc9c0\uc18d\uc801 \ubc1c\ub2ec \ub85c \uc778\ud574 \uac00\uc0c1\ud604\uc2e4 \uc2dc\uc2a4\ud15c\uc774 \uc2e4\uc81c\ub85c \uc778\uac04\uc758 \uc0b6 \uc18d\uc5d0\uc11c \uc601\ud5a5\ub825\uc744 \ubc1c\ud718\ud558\uace0, \ud604\ub300\uc778\uc758 \uc0b6\uc758 \ubb34\ub300\uac00 \uac00\uc0c1\uc758 \uacf5\uac04\uc73c\ub85c\uae4c\uc9c0 \ud655\uc7a5\ub428\uc5d0 \ub530\ub77c \uac00\uc0c1\ud604\uc2e4\uc5d0 \ub300\ud55c \uc801\uadf9\uc801\uc778 \ucca0\ud559\uc801 \ub17c\uc758\uc640 \uace0\ucc30\uc758 \ud544\uc694\uc131\uc774 \ub300\ub450\ub418\uc5c8\ub2e4. \uac00\uc0c1\ud604\uc2e4\uc5d0 \ub300\ud55c \ucca0\ud559\uc801 \ub17c\uc758\ub294 '\uac00\uc0c1\ud604\uc2e4'\uc5d0\uc11c \uac00\uc0c1, \uc989 '\ubc84\ucd94\uc5bc(virtual)'\uc758 \uac1c\ub150 \uc790\uccb4\uc5d0 \ub300\ud55c \uac83\ub3c4 \uc788\uace0, \uc778\uac04 \uac10\uac01\uc758 \ud55c\uacc4 \ubc0f \uac00\uc0c1\ud604\uc2e4 \uae30\uc220\uc758 \ud2b9\uc131\uacfc \uad00\ub828\ud558\uc5ec \uc874\uc7ac\ub860\uc801\uc778 \uace0\ucc30, \uc2e4\uc7ac\uc131\uacfc \uacf5\uac04\uc131 \uac1c\ub150\uc5d0 \ub300\ud55c \ub17c\uc758\ub4e4\ub3c4 \ud65c\ubc1c\ud788 \uc81c\uae30\ub418\uace0 \uc788\ub2e4. \uac00\uc0c1\ud604\uc2e4\uc5d0 \ub300\ud55c \ucca0\ud559\uc801\uc73c\ub85c \uace0\ucc30\ud55c \uc11c\uc801\uc73c\ub85c \ub9c8\uc774\ud074 \ud558\uc784(Michael Heim)\uc758 \uac00\uc0c1\ud604\uc2e4\uc758 \ucca0\ud559\uc801 \uc758\ubbf8(1993)\uac00 \uc788\ub2e4. \ub610, \uac00\uc0c1\ud604\uc2e4\uc5d0 \ub300\ud55c \uc9c1\uc811\uc801 \ub17c\uc758\ub97c \ub2e4\ub8e8\uace0 \uc788\uc9c0\ub294 \uc54a\uc9c0\ub9cc '\uac00\uc0c1\uc131'\uacfc \uad00\ub828\ud558\uc5ec \uc7a5 \ubcf4\ub4dc\ub9ac\uc57c\ub974(Jean baudrillard)\uc758 \uc2dc\ubbac\ub77c\uc2dc\uc639\uc774 \uc720\uba85\ud558\ub2e4. \uac00\uc0c1\ud604\uc2e4\uc5d0 \ub300\ud55c \ucca0\ud559\uc801 \uc0ac\uc720\ub97c \uc790\uadf9\ud558\ub294 \uc601\ud654\ub85c\ub294 \ub9e4\ud2b8\ub9ad\uc2a4, \ud1a0\ud0c8\ub9ac\ucf5c, \uacf5\uac01\uae30\ub3d9\ub300 \ub4f1\uc774 \uc788\ub2e4.", "paragraph_answer": "\uac00\uc0c1\ud604\uc2e4\uc5d0 \ub300\ud55c \ucca0\ud559\uc801 \ub17c\uc758\uac00 \ubcf8\uaca9\ud654\ub418\uae30 \uc774\uc804\ubd80\ud130 \uac00\uc0c1\ud604\uc2e4\uc740 SF\uc601\ud654\ub098 \ubb38\ud559\uc791\ud488, \uae30\uc220 \ubd84\uc57c\uc5d0\uc11c \ub2e4\uc591\ud55c \uc774\ub984\uc73c\ub85c \uba85\uba85\ub418\uba70 \uc0ac\ub78c\ub4e4\uc5d0\uac8c \uc778\uc2dd\ub418\uc5c8\ub2e4. \ub2d0 \uc2a4\ud2f0\ube10\uc2a8\uc758 \uc0ac\uc774\ubc84 \ud391\ud06c \uc18c\uc124 \uc2a4\ub178\uc6b0 \ud06c\ub798\uc26c(1991)\uc5d0\uc11c \uba85\uba85\ub41c '\uba54\ud0c0\ubc84\uc2a4(Metaverse)'\uc640 \uc70c\ub9ac\uc5c4 \uae41\uc2a8\uc758 \uc18c\uc124 \ub274\ub85c\ub9e8\uc11c(1984)\uc5d0\uc11c \u2018\uc0ac\uc774\ubc84\uc2a4\ud398\uc774\uc2a4(cyberspace)'\uac00 \uadf8 \uc608\uc774\ub2e4. \uadf8\ub7ec\ub098 \ucef4\ud4e8\ud130 \uae30\uc220\uc758 \uc9c0\uc18d\uc801 \ubc1c\ub2ec \ub85c \uc778\ud574 \uac00\uc0c1\ud604\uc2e4 \uc2dc\uc2a4\ud15c\uc774 \uc2e4\uc81c\ub85c \uc778\uac04\uc758 \uc0b6 \uc18d\uc5d0\uc11c \uc601\ud5a5\ub825\uc744 \ubc1c\ud718\ud558\uace0, \ud604\ub300\uc778\uc758 \uc0b6\uc758 \ubb34\ub300\uac00 \uac00\uc0c1\uc758 \uacf5\uac04\uc73c\ub85c\uae4c\uc9c0 \ud655\uc7a5\ub428\uc5d0 \ub530\ub77c \uac00\uc0c1\ud604\uc2e4\uc5d0 \ub300\ud55c \uc801\uadf9\uc801\uc778 \ucca0\ud559\uc801 \ub17c\uc758\uc640 \uace0\ucc30\uc758 \ud544\uc694\uc131\uc774 \ub300\ub450\ub418\uc5c8\ub2e4. \uac00\uc0c1\ud604\uc2e4\uc5d0 \ub300\ud55c \ucca0\ud559\uc801 \ub17c\uc758\ub294 '\uac00\uc0c1\ud604\uc2e4'\uc5d0\uc11c \uac00\uc0c1, \uc989 '\ubc84\ucd94\uc5bc(virtual)'\uc758 \uac1c\ub150 \uc790\uccb4\uc5d0 \ub300\ud55c \uac83\ub3c4 \uc788\uace0, \uc778\uac04 \uac10\uac01\uc758 \ud55c\uacc4 \ubc0f \uac00\uc0c1\ud604\uc2e4 \uae30\uc220\uc758 \ud2b9\uc131\uacfc \uad00\ub828\ud558\uc5ec \uc874\uc7ac\ub860\uc801\uc778 \uace0\ucc30, \uc2e4\uc7ac\uc131\uacfc \uacf5\uac04\uc131 \uac1c\ub150\uc5d0 \ub300\ud55c \ub17c\uc758\ub4e4\ub3c4 \ud65c\ubc1c\ud788 \uc81c\uae30\ub418\uace0 \uc788\ub2e4. \uac00\uc0c1\ud604\uc2e4\uc5d0 \ub300\ud55c \ucca0\ud559\uc801\uc73c\ub85c \uace0\ucc30\ud55c \uc11c\uc801\uc73c\ub85c \ub9c8\uc774\ud074 \ud558\uc784(Michael Heim)\uc758 \uac00\uc0c1\ud604\uc2e4\uc758 \ucca0\ud559\uc801 \uc758\ubbf8(1993)\uac00 \uc788\ub2e4. \ub610, \uac00\uc0c1\ud604\uc2e4\uc5d0 \ub300\ud55c \uc9c1\uc811\uc801 \ub17c\uc758\ub97c \ub2e4\ub8e8\uace0 \uc788\uc9c0\ub294 \uc54a\uc9c0\ub9cc '\uac00\uc0c1\uc131'\uacfc \uad00\ub828\ud558\uc5ec \uc7a5 \ubcf4\ub4dc\ub9ac\uc57c\ub974(Jean baudrillard)\uc758 \uc2dc\ubbac\ub77c\uc2dc\uc639\uc774 \uc720\uba85\ud558\ub2e4. \uac00\uc0c1\ud604\uc2e4\uc5d0 \ub300\ud55c \ucca0\ud559\uc801 \uc0ac\uc720\ub97c \uc790\uadf9\ud558\ub294 \uc601\ud654\ub85c\ub294 \ub9e4\ud2b8\ub9ad\uc2a4, \ud1a0\ud0c8\ub9ac\ucf5c, \uacf5\uac01\uae30\ub3d9\ub300 \ub4f1\uc774 \uc788\ub2e4.", "sentence_answer": "\uadf8\ub7ec\ub098 \ucef4\ud4e8\ud130 \uae30\uc220\uc758 \uc9c0\uc18d\uc801 \ubc1c\ub2ec \ub85c \uc778\ud574 \uac00\uc0c1\ud604\uc2e4 \uc2dc\uc2a4\ud15c\uc774 \uc2e4\uc81c\ub85c \uc778\uac04\uc758 \uc0b6 \uc18d\uc5d0\uc11c \uc601\ud5a5\ub825\uc744 \ubc1c\ud718\ud558\uace0, \ud604\ub300\uc778\uc758 \uc0b6\uc758 \ubb34\ub300\uac00 \uac00\uc0c1\uc758 \uacf5\uac04\uc73c\ub85c\uae4c\uc9c0 \ud655\uc7a5\ub428\uc5d0 \ub530\ub77c \uac00\uc0c1\ud604\uc2e4\uc5d0 \ub300\ud55c \uc801\uadf9\uc801\uc778 \ucca0\ud559\uc801 \ub17c\uc758\uc640 \uace0\ucc30\uc758 \ud544\uc694\uc131\uc774 \ub300\ub450\ub418\uc5c8\ub2e4.", "paragraph_id": "6471999-4-0", "paragraph_question": "question: \uac00\uc0c1\ud604\uc2e4\uc5d0 \ub300\ud55c \uc801\uadf9\uc801\uc778 \ucca0\ud559\uc801 \ub17c\uc758\uc640 \uace0\ucc30\uc758 \ud544\uc694\uc131\uc774 \ub300\ub450\ub41c \uac83\uc740 \uc6d0\ucd08\uc801\uc73c\ub85c \ubb34\uc5c7 \ub54c\ubb38\uc774\uac00?, context: \uac00\uc0c1\ud604\uc2e4\uc5d0 \ub300\ud55c \ucca0\ud559\uc801 \ub17c\uc758\uac00 \ubcf8\uaca9\ud654\ub418\uae30 \uc774\uc804\ubd80\ud130 \uac00\uc0c1\ud604\uc2e4\uc740 SF\uc601\ud654\ub098 \ubb38\ud559\uc791\ud488, \uae30\uc220 \ubd84\uc57c\uc5d0\uc11c \ub2e4\uc591\ud55c \uc774\ub984\uc73c\ub85c \uba85\uba85\ub418\uba70 \uc0ac\ub78c\ub4e4\uc5d0\uac8c \uc778\uc2dd\ub418\uc5c8\ub2e4. \ub2d0 \uc2a4\ud2f0\ube10\uc2a8\uc758 \uc0ac\uc774\ubc84 \ud391\ud06c \uc18c\uc124 \uc2a4\ub178\uc6b0 \ud06c\ub798\uc26c(1991)\uc5d0\uc11c \uba85\uba85\ub41c '\uba54\ud0c0\ubc84\uc2a4(Metaverse)'\uc640 \uc70c\ub9ac\uc5c4 \uae41\uc2a8\uc758 \uc18c\uc124 \ub274\ub85c\ub9e8\uc11c(1984)\uc5d0\uc11c \u2018\uc0ac\uc774\ubc84\uc2a4\ud398\uc774\uc2a4(cyberspace)'\uac00 \uadf8 \uc608\uc774\ub2e4. \uadf8\ub7ec\ub098 \ucef4\ud4e8\ud130 \uae30\uc220\uc758 \uc9c0\uc18d\uc801 \ubc1c\ub2ec\ub85c \uc778\ud574 \uac00\uc0c1\ud604\uc2e4 \uc2dc\uc2a4\ud15c\uc774 \uc2e4\uc81c\ub85c \uc778\uac04\uc758 \uc0b6 \uc18d\uc5d0\uc11c \uc601\ud5a5\ub825\uc744 \ubc1c\ud718\ud558\uace0, \ud604\ub300\uc778\uc758 \uc0b6\uc758 \ubb34\ub300\uac00 \uac00\uc0c1\uc758 \uacf5\uac04\uc73c\ub85c\uae4c\uc9c0 \ud655\uc7a5\ub428\uc5d0 \ub530\ub77c \uac00\uc0c1\ud604\uc2e4\uc5d0 \ub300\ud55c \uc801\uadf9\uc801\uc778 \ucca0\ud559\uc801 \ub17c\uc758\uc640 \uace0\ucc30\uc758 \ud544\uc694\uc131\uc774 \ub300\ub450\ub418\uc5c8\ub2e4. \uac00\uc0c1\ud604\uc2e4\uc5d0 \ub300\ud55c \ucca0\ud559\uc801 \ub17c\uc758\ub294 '\uac00\uc0c1\ud604\uc2e4'\uc5d0\uc11c \uac00\uc0c1, \uc989 '\ubc84\ucd94\uc5bc(virtual)'\uc758 \uac1c\ub150 \uc790\uccb4\uc5d0 \ub300\ud55c \uac83\ub3c4 \uc788\uace0, \uc778\uac04 \uac10\uac01\uc758 \ud55c\uacc4 \ubc0f \uac00\uc0c1\ud604\uc2e4 \uae30\uc220\uc758 \ud2b9\uc131\uacfc \uad00\ub828\ud558\uc5ec \uc874\uc7ac\ub860\uc801\uc778 \uace0\ucc30, \uc2e4\uc7ac\uc131\uacfc \uacf5\uac04\uc131 \uac1c\ub150\uc5d0 \ub300\ud55c \ub17c\uc758\ub4e4\ub3c4 \ud65c\ubc1c\ud788 \uc81c\uae30\ub418\uace0 \uc788\ub2e4. \uac00\uc0c1\ud604\uc2e4\uc5d0 \ub300\ud55c \ucca0\ud559\uc801\uc73c\ub85c \uace0\ucc30\ud55c \uc11c\uc801\uc73c\ub85c \ub9c8\uc774\ud074 \ud558\uc784(Michael Heim)\uc758 \uac00\uc0c1\ud604\uc2e4\uc758 \ucca0\ud559\uc801 \uc758\ubbf8(1993)\uac00 \uc788\ub2e4. \ub610, \uac00\uc0c1\ud604\uc2e4\uc5d0 \ub300\ud55c \uc9c1\uc811\uc801 \ub17c\uc758\ub97c \ub2e4\ub8e8\uace0 \uc788\uc9c0\ub294 \uc54a\uc9c0\ub9cc '\uac00\uc0c1\uc131'\uacfc \uad00\ub828\ud558\uc5ec \uc7a5 \ubcf4\ub4dc\ub9ac\uc57c\ub974(Jean baudrillard)\uc758 \uc2dc\ubbac\ub77c\uc2dc\uc639\uc774 \uc720\uba85\ud558\ub2e4. \uac00\uc0c1\ud604\uc2e4\uc5d0 \ub300\ud55c \ucca0\ud559\uc801 \uc0ac\uc720\ub97c \uc790\uadf9\ud558\ub294 \uc601\ud654\ub85c\ub294 \ub9e4\ud2b8\ub9ad\uc2a4, \ud1a0\ud0c8\ub9ac\ucf5c, \uacf5\uac01\uae30\ub3d9\ub300 \ub4f1\uc774 \uc788\ub2e4."} {"question": "\ube0c\uc640\ub514\uc2a4\uc640\ud504 2\uc138\uc758 \ub3d9\uc0dd\uc73c\ub85c 1146\ub144 \ud3f4\ub780\ub4dc\uc758 \uad8c\ub825\uc744 \uc7a5\uc545\ud55c \uc778\ubb3c\uc740?", "paragraph": "1146\ub144 \ud3ec\uc988\ub09c \uc804\ud22c\uc5d0\uc11c \ud328\ud55c \ube0c\uc640\ub514\uc2a4\uc640\ud504 2\uc138\ub294 \ub3c5\uc77c\ub85c \ub9dd\uba85\ud558\uc600\uace0 \uadf8\uc758 \ub3d9\uc0dd \uace0\uc218\uba38\ub9ac \ubcfc\ub808\uc2a4\uc640\ud504 4\uc138\uac00 \uad8c\ub825\uc744 \uc7a5\uc545\ud558\uace0 \ub300\uacf5\uc758 \uc9c0\uc704\ub97c \ub204\ub9ac\uac8c \ub418\uc5c8\ub2e4. 1152\ub144 \ub3c5\uc77c \ud669\uc81c\uac00 \ub41c \ud504\ub9ac\ub4dc\ub9ac\ud788 1\uc138\ub294 \ube0c\uc640\ub514\uc2a4\uc640\ud504 2\uc138\uc758 \uc694\uccad\uc5d0 \ub530\ub77c 1157\ub144 \ud3f4\ub780\ub4dc \uc6d0\uc815\uc5d0 \ub098\uc130\ub2e4. \ubcfc\ub808\uc2a4\uc640\ud504 4\uc138\uac00 \uc0ac\ub9dd\ud55c \ud6c4 \ubbf8\uc5d0\uc288\ucf54 3\uc138\ub294 \ud589\uc815\uac00\ub85c\uc11c \ub2a5\ub825\uc744 \ubc1c\ud718\ud558\uc5ec \uad8c\ub825\uc744 \uc790\uae30\uc5d0\uac8c\ub85c \uc9d1\uc911\uc2dc\ud0a4\ub824\uace0 \ub178\ub825\ud588\uc9c0\ub9cc \uadf8\uc758 \uc911\uc559\uc9d1\uad8c\ud654 \uc815\ucc45\uc740 \ub300\uadc0\uc871\ub4e4\uc758 \uc800\ud56d\uc5d0 \ubd80\ub52a\ud788\uace0 \ub9d0\uc558\ub2e4. 1180\ub144 \uc6fd\uce58\ucc28 \ud68c\uc758\uc5d0\uc11c \uce74\uc9c0\ubbf8\uc5d0\uc2dc\ub294 \uc790\uc2e0\uc758 \uad8c\ub825 \uc815\ud1b5\uc131\uc744 \ud655\uc778\ubc1b\uc744 \uc218 \uc788\uc5c8\ub2e4. \uadf8\ub294 \uac00\ud1a8\ub9ad\uad50\ud68c\uac00 \uc0ac\uc2e4\uc0c1 \uad6d\uac00\ud589\uc815\uc5d0 \uc601\ud5a5\uc744 \ubbf8\uce60 \uc218 \uc788\ub294 \uae38\uc744 \ud130\ub193\uc558\uc73c\uba70 \uc5f0\uc7a5\uc790 \uc6b0\uc120\uc81c\ub3c4\ub97c \ud3d0\uc9c0\ud588\ub2e4. \uadf8\uacb0\uacfc \ud3f4\ub780\ub4dc\uac00 \ubd84\ud560 \uacf5\uad6d\uc73c\ub85c \ub354\uc6b1 \uc138\ubd84\ud654\ub418\ub294 \uac83\uc744 \ubc29\uc9c0\ud560 \uc218 \uc788\uc5c8\uace0 \uad6d\ub0b4\uc758 \uc815\ucc45\uc744 \uc218\ud589\ud558\ub294 \ub370\uc5d0 \ub300\uacf5\uc758 \ud589\ub3d9 \ubc18\uacbd\uc774 \uadf8\ub9cc\ud07c \ud655\ub300\ub418\uc5c8\ub2e4.", "answer": "\ubcfc\ub808\uc2a4\uc640\ud504 4\uc138", "sentence": "\uadf8\uc758 \ub3d9\uc0dd \uace0\uc218\uba38\ub9ac \ubcfc\ub808\uc2a4\uc640\ud504 4\uc138 \uac00 \uad8c\ub825\uc744 \uc7a5\uc545\ud558\uace0 \ub300\uacf5\uc758 \uc9c0\uc704\ub97c \ub204\ub9ac\uac8c \ub418\uc5c8\ub2e4.", "paragraph_sentence": "1146\ub144 \ud3ec\uc988\ub09c \uc804\ud22c\uc5d0\uc11c \ud328\ud55c \ube0c\uc640\ub514\uc2a4\uc640\ud504 2\uc138\ub294 \ub3c5\uc77c\ub85c \ub9dd\uba85\ud558\uc600\uace0 \uadf8\uc758 \ub3d9\uc0dd \uace0\uc218\uba38\ub9ac \ubcfc\ub808\uc2a4\uc640\ud504 4\uc138 \uac00 \uad8c\ub825\uc744 \uc7a5\uc545\ud558\uace0 \ub300\uacf5\uc758 \uc9c0\uc704\ub97c \ub204\ub9ac\uac8c \ub418\uc5c8\ub2e4. 1152\ub144 \ub3c5\uc77c \ud669\uc81c\uac00 \ub41c \ud504\ub9ac\ub4dc\ub9ac\ud788 1\uc138\ub294 \ube0c\uc640\ub514\uc2a4\uc640\ud504 2\uc138\uc758 \uc694\uccad\uc5d0 \ub530\ub77c 1157\ub144 \ud3f4\ub780\ub4dc \uc6d0\uc815\uc5d0 \ub098\uc130\ub2e4. \ubcfc\ub808\uc2a4\uc640\ud504 4\uc138\uac00 \uc0ac\ub9dd\ud55c \ud6c4 \ubbf8\uc5d0\uc288\ucf54 3\uc138\ub294 \ud589\uc815\uac00\ub85c\uc11c \ub2a5\ub825\uc744 \ubc1c\ud718\ud558\uc5ec \uad8c\ub825\uc744 \uc790\uae30\uc5d0\uac8c\ub85c \uc9d1\uc911\uc2dc\ud0a4\ub824\uace0 \ub178\ub825\ud588\uc9c0\ub9cc \uadf8\uc758 \uc911\uc559\uc9d1\uad8c\ud654 \uc815\ucc45\uc740 \ub300\uadc0\uc871\ub4e4\uc758 \uc800\ud56d\uc5d0 \ubd80\ub52a\ud788\uace0 \ub9d0\uc558\ub2e4. 1180\ub144 \uc6fd\uce58\ucc28 \ud68c\uc758\uc5d0\uc11c \uce74\uc9c0\ubbf8\uc5d0\uc2dc\ub294 \uc790\uc2e0\uc758 \uad8c\ub825 \uc815\ud1b5\uc131\uc744 \ud655\uc778\ubc1b\uc744 \uc218 \uc788\uc5c8\ub2e4. \uadf8\ub294 \uac00\ud1a8\ub9ad\uad50\ud68c\uac00 \uc0ac\uc2e4\uc0c1 \uad6d\uac00\ud589\uc815\uc5d0 \uc601\ud5a5\uc744 \ubbf8\uce60 \uc218 \uc788\ub294 \uae38\uc744 \ud130\ub193\uc558\uc73c\uba70 \uc5f0\uc7a5\uc790 \uc6b0\uc120\uc81c\ub3c4\ub97c \ud3d0\uc9c0\ud588\ub2e4. \uadf8\uacb0\uacfc \ud3f4\ub780\ub4dc\uac00 \ubd84\ud560 \uacf5\uad6d\uc73c\ub85c \ub354\uc6b1 \uc138\ubd84\ud654\ub418\ub294 \uac83\uc744 \ubc29\uc9c0\ud560 \uc218 \uc788\uc5c8\uace0 \uad6d\ub0b4\uc758 \uc815\ucc45\uc744 \uc218\ud589\ud558\ub294 \ub370\uc5d0 \ub300\uacf5\uc758 \ud589\ub3d9 \ubc18\uacbd\uc774 \uadf8\ub9cc\ud07c \ud655\ub300\ub418\uc5c8\ub2e4.", "paragraph_answer": "1146\ub144 \ud3ec\uc988\ub09c \uc804\ud22c\uc5d0\uc11c \ud328\ud55c \ube0c\uc640\ub514\uc2a4\uc640\ud504 2\uc138\ub294 \ub3c5\uc77c\ub85c \ub9dd\uba85\ud558\uc600\uace0 \uadf8\uc758 \ub3d9\uc0dd \uace0\uc218\uba38\ub9ac \ubcfc\ub808\uc2a4\uc640\ud504 4\uc138 \uac00 \uad8c\ub825\uc744 \uc7a5\uc545\ud558\uace0 \ub300\uacf5\uc758 \uc9c0\uc704\ub97c \ub204\ub9ac\uac8c \ub418\uc5c8\ub2e4. 1152\ub144 \ub3c5\uc77c \ud669\uc81c\uac00 \ub41c \ud504\ub9ac\ub4dc\ub9ac\ud788 1\uc138\ub294 \ube0c\uc640\ub514\uc2a4\uc640\ud504 2\uc138\uc758 \uc694\uccad\uc5d0 \ub530\ub77c 1157\ub144 \ud3f4\ub780\ub4dc \uc6d0\uc815\uc5d0 \ub098\uc130\ub2e4. \ubcfc\ub808\uc2a4\uc640\ud504 4\uc138\uac00 \uc0ac\ub9dd\ud55c \ud6c4 \ubbf8\uc5d0\uc288\ucf54 3\uc138\ub294 \ud589\uc815\uac00\ub85c\uc11c \ub2a5\ub825\uc744 \ubc1c\ud718\ud558\uc5ec \uad8c\ub825\uc744 \uc790\uae30\uc5d0\uac8c\ub85c \uc9d1\uc911\uc2dc\ud0a4\ub824\uace0 \ub178\ub825\ud588\uc9c0\ub9cc \uadf8\uc758 \uc911\uc559\uc9d1\uad8c\ud654 \uc815\ucc45\uc740 \ub300\uadc0\uc871\ub4e4\uc758 \uc800\ud56d\uc5d0 \ubd80\ub52a\ud788\uace0 \ub9d0\uc558\ub2e4. 1180\ub144 \uc6fd\uce58\ucc28 \ud68c\uc758\uc5d0\uc11c \uce74\uc9c0\ubbf8\uc5d0\uc2dc\ub294 \uc790\uc2e0\uc758 \uad8c\ub825 \uc815\ud1b5\uc131\uc744 \ud655\uc778\ubc1b\uc744 \uc218 \uc788\uc5c8\ub2e4. \uadf8\ub294 \uac00\ud1a8\ub9ad\uad50\ud68c\uac00 \uc0ac\uc2e4\uc0c1 \uad6d\uac00\ud589\uc815\uc5d0 \uc601\ud5a5\uc744 \ubbf8\uce60 \uc218 \uc788\ub294 \uae38\uc744 \ud130\ub193\uc558\uc73c\uba70 \uc5f0\uc7a5\uc790 \uc6b0\uc120\uc81c\ub3c4\ub97c \ud3d0\uc9c0\ud588\ub2e4. \uadf8\uacb0\uacfc \ud3f4\ub780\ub4dc\uac00 \ubd84\ud560 \uacf5\uad6d\uc73c\ub85c \ub354\uc6b1 \uc138\ubd84\ud654\ub418\ub294 \uac83\uc744 \ubc29\uc9c0\ud560 \uc218 \uc788\uc5c8\uace0 \uad6d\ub0b4\uc758 \uc815\ucc45\uc744 \uc218\ud589\ud558\ub294 \ub370\uc5d0 \ub300\uacf5\uc758 \ud589\ub3d9 \ubc18\uacbd\uc774 \uadf8\ub9cc\ud07c \ud655\ub300\ub418\uc5c8\ub2e4.", "sentence_answer": "\uadf8\uc758 \ub3d9\uc0dd \uace0\uc218\uba38\ub9ac \ubcfc\ub808\uc2a4\uc640\ud504 4\uc138 \uac00 \uad8c\ub825\uc744 \uc7a5\uc545\ud558\uace0 \ub300\uacf5\uc758 \uc9c0\uc704\ub97c \ub204\ub9ac\uac8c \ub418\uc5c8\ub2e4.", "paragraph_id": "6600375-3-1", "paragraph_question": "question: \ube0c\uc640\ub514\uc2a4\uc640\ud504 2\uc138\uc758 \ub3d9\uc0dd\uc73c\ub85c 1146\ub144 \ud3f4\ub780\ub4dc\uc758 \uad8c\ub825\uc744 \uc7a5\uc545\ud55c \uc778\ubb3c\uc740?, context: 1146\ub144 \ud3ec\uc988\ub09c \uc804\ud22c\uc5d0\uc11c \ud328\ud55c \ube0c\uc640\ub514\uc2a4\uc640\ud504 2\uc138\ub294 \ub3c5\uc77c\ub85c \ub9dd\uba85\ud558\uc600\uace0 \uadf8\uc758 \ub3d9\uc0dd \uace0\uc218\uba38\ub9ac \ubcfc\ub808\uc2a4\uc640\ud504 4\uc138\uac00 \uad8c\ub825\uc744 \uc7a5\uc545\ud558\uace0 \ub300\uacf5\uc758 \uc9c0\uc704\ub97c \ub204\ub9ac\uac8c \ub418\uc5c8\ub2e4. 1152\ub144 \ub3c5\uc77c \ud669\uc81c\uac00 \ub41c \ud504\ub9ac\ub4dc\ub9ac\ud788 1\uc138\ub294 \ube0c\uc640\ub514\uc2a4\uc640\ud504 2\uc138\uc758 \uc694\uccad\uc5d0 \ub530\ub77c 1157\ub144 \ud3f4\ub780\ub4dc \uc6d0\uc815\uc5d0 \ub098\uc130\ub2e4. \ubcfc\ub808\uc2a4\uc640\ud504 4\uc138\uac00 \uc0ac\ub9dd\ud55c \ud6c4 \ubbf8\uc5d0\uc288\ucf54 3\uc138\ub294 \ud589\uc815\uac00\ub85c\uc11c \ub2a5\ub825\uc744 \ubc1c\ud718\ud558\uc5ec \uad8c\ub825\uc744 \uc790\uae30\uc5d0\uac8c\ub85c \uc9d1\uc911\uc2dc\ud0a4\ub824\uace0 \ub178\ub825\ud588\uc9c0\ub9cc \uadf8\uc758 \uc911\uc559\uc9d1\uad8c\ud654 \uc815\ucc45\uc740 \ub300\uadc0\uc871\ub4e4\uc758 \uc800\ud56d\uc5d0 \ubd80\ub52a\ud788\uace0 \ub9d0\uc558\ub2e4. 1180\ub144 \uc6fd\uce58\ucc28 \ud68c\uc758\uc5d0\uc11c \uce74\uc9c0\ubbf8\uc5d0\uc2dc\ub294 \uc790\uc2e0\uc758 \uad8c\ub825 \uc815\ud1b5\uc131\uc744 \ud655\uc778\ubc1b\uc744 \uc218 \uc788\uc5c8\ub2e4. \uadf8\ub294 \uac00\ud1a8\ub9ad\uad50\ud68c\uac00 \uc0ac\uc2e4\uc0c1 \uad6d\uac00\ud589\uc815\uc5d0 \uc601\ud5a5\uc744 \ubbf8\uce60 \uc218 \uc788\ub294 \uae38\uc744 \ud130\ub193\uc558\uc73c\uba70 \uc5f0\uc7a5\uc790 \uc6b0\uc120\uc81c\ub3c4\ub97c \ud3d0\uc9c0\ud588\ub2e4. \uadf8\uacb0\uacfc \ud3f4\ub780\ub4dc\uac00 \ubd84\ud560 \uacf5\uad6d\uc73c\ub85c \ub354\uc6b1 \uc138\ubd84\ud654\ub418\ub294 \uac83\uc744 \ubc29\uc9c0\ud560 \uc218 \uc788\uc5c8\uace0 \uad6d\ub0b4\uc758 \uc815\ucc45\uc744 \uc218\ud589\ud558\ub294 \ub370\uc5d0 \ub300\uacf5\uc758 \ud589\ub3d9 \ubc18\uacbd\uc774 \uadf8\ub9cc\ud07c \ud655\ub300\ub418\uc5c8\ub2e4."} {"question": "\uc559\ub4dc\ub808 \uc544\uc720\uac00 \uc544\ud504\ub9ac\uce74 U-20 \ucc54\ud53c\uc5b8\uc27d\uacfc FIFA U-20 \uc6d4\ub4dc\ucef5 \uc6b0\uc2b9 \uc8fc\uc5ed\uc774 \ub41c \ub144\ub3c4\ub294?", "paragraph": "\uc720\uc18c\ub144 \ub300\ud45c\ud300\uc5d0\uc11c, \uc544\uc720\ub294 \uac00\ub098\uc758 U-20 \ub300\ud45c\ud300 \uc77c\uc6d0\uc73c\ub85c \uc8fc\uc7a5\uc774 \ub418\uc5b4, 2009\ub144 \uc544\ud504\ub9ac\uce74 U-20 \ucc54\ud53c\uc5b8\uc27d\uacfc 2009\ub144 FIFA U-20 \uc6d4\ub4dc\ucef5 \uc6b0\uc2b9 \uc8fc\uc5ed\uc774 \ub418\uc5c8\ub2e4. \uc544\ud504\ub9ac\uce74 U-20 \ucc54\ud53c\uc5b8\uc27d\uc5d0\uc11c, \uc544\uc720\ub294 \uce74\uba54\ub8ec\uacfc \ub0a8\uc544\ud504\ub9ac\uce74 \uacf5\ud654\uad6d\uc744 \uc0c1\ub300\ub85c \ud55c\uace8\uc529 \uae30\ub85d\ud558\uc600\ub2e4. \ucc54\ud53c\uc5b8\uc27d\uc5d0\uc11c\uc758 \ub0a8\uc544\uacf5 \uc900\uacb0\uc2b9\uc804 4-3 \uc2b9\ub9ac\uc5d0 \uc774\uc5b4 \ub2e4\uc2dc \ub9cc\ub09c \uce74\uba54\ub8ec\uc744 2-0\uc73c\ub85c \uaebe\uc73c\uba70 \uc6b0\uc2b9\ud55c \uac00\ub098\ub294 2009\ub144 FIFA U-20 \uc6d4\ub4dc\ucef5 \ubcf8\uc120\uc5d0 \uc9c4\ucd9c\ud558\uc600\ub2e4. \uc774 \ub300\ud68c\uc5d0\uc11c \uc544\uc720\ub294 \uc789\uae00\ub79c\ub4dc\uc804\uc5d0\uc11c \ub450\uace8\uc744 \uae30\ub85d, 4-0 \uc2b9\ub9ac\ub97c \uc774\ub04c\uc5b4\ub0c8\uace0, 16\uac15\uc804\uc5d0\uc11c \ub0a8\uc544\uacf5\uc804\uc5d0\uc11c 1-1 \ub3d9\uc810\uace8\uc744 \uae30\ub85d\ud558\uc600\ub2e4. \uac19\uc740 \uacbd\uae30\uc5d0\uc11c \uac00\ub098\ub294 \ub3c4\ubbf8\ub2c9 \uc544\ub514\uc774\uc544\uc758 \uc5f0\uc7a5\uc804 \uacb0\uc2b9\uace8\ub85c 8\uac15\uc5d0 \uc9c4\ucd9c\ud558\uc600\ub2e4. \uc544\uc720\uc758 \uac00\ub098\ub294 \uc774\ud6c4 \ub300\ud55c\ubbfc\uad6d, \ud5dd\uac00\ub9ac, \ube0c\ub77c\uc9c8\uc744 \ucc28\ub840\ub85c \ubb34\ub108\ub728\ub9ac\uace0 \uc6b0\uc2b9\uc744 \ud558\uc600\ub2e4. \uac00\ub098\ub294 \ube0c\ub77c\uc9c8\uacfc\uc758 \uacb0\uc2b9\uc804\uc5d0\uc11c \uc2b9\ubd80\ucc28\uae30\ub05d\uc5d0 4-3 \uc2b9\ub9ac\ub97c \uac70\ub450\uc5c8\ub2e4. \uc544\uc720\ub294 \uccab \ud0a4\ucee4\ub85c \ub098\uc11c\uc11c \uc131\uacf5\ud558\uc600\ub2e4.", "answer": "2009\ub144", "sentence": "\uc720\uc18c\ub144 \ub300\ud45c\ud300\uc5d0\uc11c, \uc544\uc720\ub294 \uac00\ub098\uc758 U-20 \ub300\ud45c\ud300 \uc77c\uc6d0\uc73c\ub85c \uc8fc\uc7a5\uc774 \ub418\uc5b4, 2009\ub144 \uc544\ud504\ub9ac\uce74 U-20 \ucc54\ud53c\uc5b8\uc27d\uacfc 2009\ub144 FIFA U-20 \uc6d4\ub4dc\ucef5 \uc6b0\uc2b9 \uc8fc\uc5ed\uc774 \ub418\uc5c8\ub2e4.", "paragraph_sentence": " \uc720\uc18c\ub144 \ub300\ud45c\ud300\uc5d0\uc11c, \uc544\uc720\ub294 \uac00\ub098\uc758 U-20 \ub300\ud45c\ud300 \uc77c\uc6d0\uc73c\ub85c \uc8fc\uc7a5\uc774 \ub418\uc5b4, 2009\ub144 \uc544\ud504\ub9ac\uce74 U-20 \ucc54\ud53c\uc5b8\uc27d\uacfc 2009\ub144 FIFA U-20 \uc6d4\ub4dc\ucef5 \uc6b0\uc2b9 \uc8fc\uc5ed\uc774 \ub418\uc5c8\ub2e4. \uc544\ud504\ub9ac\uce74 U-20 \ucc54\ud53c\uc5b8\uc27d\uc5d0\uc11c, \uc544\uc720\ub294 \uce74\uba54\ub8ec\uacfc \ub0a8\uc544\ud504\ub9ac\uce74 \uacf5\ud654\uad6d\uc744 \uc0c1\ub300\ub85c \ud55c\uace8\uc529 \uae30\ub85d\ud558\uc600\ub2e4. \ucc54\ud53c\uc5b8\uc27d\uc5d0\uc11c\uc758 \ub0a8\uc544\uacf5 \uc900\uacb0\uc2b9\uc804 4-3 \uc2b9\ub9ac\uc5d0 \uc774\uc5b4 \ub2e4\uc2dc \ub9cc\ub09c \uce74\uba54\ub8ec\uc744 2-0\uc73c\ub85c \uaebe\uc73c\uba70 \uc6b0\uc2b9\ud55c \uac00\ub098\ub294 2009\ub144 FIFA U-20 \uc6d4\ub4dc\ucef5 \ubcf8\uc120\uc5d0 \uc9c4\ucd9c\ud558\uc600\ub2e4. \uc774 \ub300\ud68c\uc5d0\uc11c \uc544\uc720\ub294 \uc789\uae00\ub79c\ub4dc\uc804\uc5d0\uc11c \ub450\uace8\uc744 \uae30\ub85d, 4-0 \uc2b9\ub9ac\ub97c \uc774\ub04c\uc5b4\ub0c8\uace0, 16\uac15\uc804\uc5d0\uc11c \ub0a8\uc544\uacf5\uc804\uc5d0\uc11c 1-1 \ub3d9\uc810\uace8\uc744 \uae30\ub85d\ud558\uc600\ub2e4. \uac19\uc740 \uacbd\uae30\uc5d0\uc11c \uac00\ub098\ub294 \ub3c4\ubbf8\ub2c9 \uc544\ub514\uc774\uc544\uc758 \uc5f0\uc7a5\uc804 \uacb0\uc2b9\uace8\ub85c 8\uac15\uc5d0 \uc9c4\ucd9c\ud558\uc600\ub2e4. \uc544\uc720\uc758 \uac00\ub098\ub294 \uc774\ud6c4 \ub300\ud55c\ubbfc\uad6d, \ud5dd\uac00\ub9ac, \ube0c\ub77c\uc9c8\uc744 \ucc28\ub840\ub85c \ubb34\ub108\ub728\ub9ac\uace0 \uc6b0\uc2b9\uc744 \ud558\uc600\ub2e4. \uac00\ub098\ub294 \ube0c\ub77c\uc9c8\uacfc\uc758 \uacb0\uc2b9\uc804\uc5d0\uc11c \uc2b9\ubd80\ucc28\uae30\ub05d\uc5d0 4-3 \uc2b9\ub9ac\ub97c \uac70\ub450\uc5c8\ub2e4. \uc544\uc720\ub294 \uccab \ud0a4\ucee4\ub85c \ub098\uc11c\uc11c \uc131\uacf5\ud558\uc600\ub2e4.", "paragraph_answer": "\uc720\uc18c\ub144 \ub300\ud45c\ud300\uc5d0\uc11c, \uc544\uc720\ub294 \uac00\ub098\uc758 U-20 \ub300\ud45c\ud300 \uc77c\uc6d0\uc73c\ub85c \uc8fc\uc7a5\uc774 \ub418\uc5b4, 2009\ub144 \uc544\ud504\ub9ac\uce74 U-20 \ucc54\ud53c\uc5b8\uc27d\uacfc 2009\ub144 FIFA U-20 \uc6d4\ub4dc\ucef5 \uc6b0\uc2b9 \uc8fc\uc5ed\uc774 \ub418\uc5c8\ub2e4. \uc544\ud504\ub9ac\uce74 U-20 \ucc54\ud53c\uc5b8\uc27d\uc5d0\uc11c, \uc544\uc720\ub294 \uce74\uba54\ub8ec\uacfc \ub0a8\uc544\ud504\ub9ac\uce74 \uacf5\ud654\uad6d\uc744 \uc0c1\ub300\ub85c \ud55c\uace8\uc529 \uae30\ub85d\ud558\uc600\ub2e4. \ucc54\ud53c\uc5b8\uc27d\uc5d0\uc11c\uc758 \ub0a8\uc544\uacf5 \uc900\uacb0\uc2b9\uc804 4-3 \uc2b9\ub9ac\uc5d0 \uc774\uc5b4 \ub2e4\uc2dc \ub9cc\ub09c \uce74\uba54\ub8ec\uc744 2-0\uc73c\ub85c \uaebe\uc73c\uba70 \uc6b0\uc2b9\ud55c \uac00\ub098\ub294 2009\ub144 FIFA U-20 \uc6d4\ub4dc\ucef5 \ubcf8\uc120\uc5d0 \uc9c4\ucd9c\ud558\uc600\ub2e4. \uc774 \ub300\ud68c\uc5d0\uc11c \uc544\uc720\ub294 \uc789\uae00\ub79c\ub4dc\uc804\uc5d0\uc11c \ub450\uace8\uc744 \uae30\ub85d, 4-0 \uc2b9\ub9ac\ub97c \uc774\ub04c\uc5b4\ub0c8\uace0, 16\uac15\uc804\uc5d0\uc11c \ub0a8\uc544\uacf5\uc804\uc5d0\uc11c 1-1 \ub3d9\uc810\uace8\uc744 \uae30\ub85d\ud558\uc600\ub2e4. \uac19\uc740 \uacbd\uae30\uc5d0\uc11c \uac00\ub098\ub294 \ub3c4\ubbf8\ub2c9 \uc544\ub514\uc774\uc544\uc758 \uc5f0\uc7a5\uc804 \uacb0\uc2b9\uace8\ub85c 8\uac15\uc5d0 \uc9c4\ucd9c\ud558\uc600\ub2e4. \uc544\uc720\uc758 \uac00\ub098\ub294 \uc774\ud6c4 \ub300\ud55c\ubbfc\uad6d, \ud5dd\uac00\ub9ac, \ube0c\ub77c\uc9c8\uc744 \ucc28\ub840\ub85c \ubb34\ub108\ub728\ub9ac\uace0 \uc6b0\uc2b9\uc744 \ud558\uc600\ub2e4. \uac00\ub098\ub294 \ube0c\ub77c\uc9c8\uacfc\uc758 \uacb0\uc2b9\uc804\uc5d0\uc11c \uc2b9\ubd80\ucc28\uae30\ub05d\uc5d0 4-3 \uc2b9\ub9ac\ub97c \uac70\ub450\uc5c8\ub2e4. \uc544\uc720\ub294 \uccab \ud0a4\ucee4\ub85c \ub098\uc11c\uc11c \uc131\uacf5\ud558\uc600\ub2e4.", "sentence_answer": "\uc720\uc18c\ub144 \ub300\ud45c\ud300\uc5d0\uc11c, \uc544\uc720\ub294 \uac00\ub098\uc758 U-20 \ub300\ud45c\ud300 \uc77c\uc6d0\uc73c\ub85c \uc8fc\uc7a5\uc774 \ub418\uc5b4, 2009\ub144 \uc544\ud504\ub9ac\uce74 U-20 \ucc54\ud53c\uc5b8\uc27d\uacfc 2009\ub144 FIFA U-20 \uc6d4\ub4dc\ucef5 \uc6b0\uc2b9 \uc8fc\uc5ed\uc774 \ub418\uc5c8\ub2e4.", "paragraph_id": "6539920-6-0", "paragraph_question": "question: \uc559\ub4dc\ub808 \uc544\uc720\uac00 \uc544\ud504\ub9ac\uce74 U-20 \ucc54\ud53c\uc5b8\uc27d\uacfc FIFA U-20 \uc6d4\ub4dc\ucef5 \uc6b0\uc2b9 \uc8fc\uc5ed\uc774 \ub41c \ub144\ub3c4\ub294?, context: \uc720\uc18c\ub144 \ub300\ud45c\ud300\uc5d0\uc11c, \uc544\uc720\ub294 \uac00\ub098\uc758 U-20 \ub300\ud45c\ud300 \uc77c\uc6d0\uc73c\ub85c \uc8fc\uc7a5\uc774 \ub418\uc5b4, 2009\ub144 \uc544\ud504\ub9ac\uce74 U-20 \ucc54\ud53c\uc5b8\uc27d\uacfc 2009\ub144 FIFA U-20 \uc6d4\ub4dc\ucef5 \uc6b0\uc2b9 \uc8fc\uc5ed\uc774 \ub418\uc5c8\ub2e4. \uc544\ud504\ub9ac\uce74 U-20 \ucc54\ud53c\uc5b8\uc27d\uc5d0\uc11c, \uc544\uc720\ub294 \uce74\uba54\ub8ec\uacfc \ub0a8\uc544\ud504\ub9ac\uce74 \uacf5\ud654\uad6d\uc744 \uc0c1\ub300\ub85c \ud55c\uace8\uc529 \uae30\ub85d\ud558\uc600\ub2e4. \ucc54\ud53c\uc5b8\uc27d\uc5d0\uc11c\uc758 \ub0a8\uc544\uacf5 \uc900\uacb0\uc2b9\uc804 4-3 \uc2b9\ub9ac\uc5d0 \uc774\uc5b4 \ub2e4\uc2dc \ub9cc\ub09c \uce74\uba54\ub8ec\uc744 2-0\uc73c\ub85c \uaebe\uc73c\uba70 \uc6b0\uc2b9\ud55c \uac00\ub098\ub294 2009\ub144 FIFA U-20 \uc6d4\ub4dc\ucef5 \ubcf8\uc120\uc5d0 \uc9c4\ucd9c\ud558\uc600\ub2e4. \uc774 \ub300\ud68c\uc5d0\uc11c \uc544\uc720\ub294 \uc789\uae00\ub79c\ub4dc\uc804\uc5d0\uc11c \ub450\uace8\uc744 \uae30\ub85d, 4-0 \uc2b9\ub9ac\ub97c \uc774\ub04c\uc5b4\ub0c8\uace0, 16\uac15\uc804\uc5d0\uc11c \ub0a8\uc544\uacf5\uc804\uc5d0\uc11c 1-1 \ub3d9\uc810\uace8\uc744 \uae30\ub85d\ud558\uc600\ub2e4. \uac19\uc740 \uacbd\uae30\uc5d0\uc11c \uac00\ub098\ub294 \ub3c4\ubbf8\ub2c9 \uc544\ub514\uc774\uc544\uc758 \uc5f0\uc7a5\uc804 \uacb0\uc2b9\uace8\ub85c 8\uac15\uc5d0 \uc9c4\ucd9c\ud558\uc600\ub2e4. \uc544\uc720\uc758 \uac00\ub098\ub294 \uc774\ud6c4 \ub300\ud55c\ubbfc\uad6d, \ud5dd\uac00\ub9ac, \ube0c\ub77c\uc9c8\uc744 \ucc28\ub840\ub85c \ubb34\ub108\ub728\ub9ac\uace0 \uc6b0\uc2b9\uc744 \ud558\uc600\ub2e4. \uac00\ub098\ub294 \ube0c\ub77c\uc9c8\uacfc\uc758 \uacb0\uc2b9\uc804\uc5d0\uc11c \uc2b9\ubd80\ucc28\uae30\ub05d\uc5d0 4-3 \uc2b9\ub9ac\ub97c \uac70\ub450\uc5c8\ub2e4. \uc544\uc720\ub294 \uccab \ud0a4\ucee4\ub85c \ub098\uc11c\uc11c \uc131\uacf5\ud558\uc600\ub2e4."} {"question": "\ub9e4\uce74\ud2b8\ub2c8 \uc74c\ubc18\uc5d0 \ub300\ud574 \ud639\ud3c9\uc744 \uac00\ud588\ub358 \uba5c\ub85c\ub514 \uba54\uc774\ucee4\uc9c0\uc758 \ud3c9\ub860\uac00\ub294 \ub204\uad6c\uc778\uac00?", "paragraph": "\u300aMcCartney\u300b\ub294 \ucd9c\uc2dc\ub418\uc790 \ubd88\ucda9\ubd84\ud55c \uc81c\uc791\uacfc \ubbf8\uc644\uc131 \uace1\uc73c\ub85c \uc778\ud574 \uc804 \uc138\uacc4\uc801\uc73c\ub85c \ud639\ud3c9\uc744 \ub4e4\uc5c8\ub2e4. \ub367\ubd99\uc5ec\uc11c \ub2c8\ucf5c\ub77c\uc2a4 \uc0e4\ud504\ub108\uc758 1977\ub144 \uc800\uc11c \u300a\ube44\ud2c0\uc988 \ud3ec\uc5d0\ubc84\u300b(The Beatles Forever)\uc5d0 \uc758\ud558\uba74, \ub9e4\uce74\ud2b8\ub2c8\ub294 \uc790\uc2e0\uc744 \ud589\ubcf5\ud55c \uac00\uc815\uc758 \ub0a8\uc790\ub85c \ub300\ud45c\ud558\uba74\uc11c \ube44\ud2c0\uc988\uc758 \ud574\uccb4\ub97c \uc790\uc2e0\uc758 \uc194\ub85c \uc74c\ubc18 \ud64d\ubcf4\uc5d0 \uc0ac\uc6a9\ud558\ub824\uace0 \ud588\uace0, \uc774\ub294 \"\ub9ce\uc740 \ubaa9\uaca9\uc790\ub4e4\uc5d0 \uc758\ud574 \uc74c\ubc18 \uc804\ubd80\uac00 \uac10\uac01\uc774 \uc5c6\uace0, \uc9c0\ub3c5\ud558\uac8c \uace0\uc548\ub428\uc774 \ub4e4\ud1b5\ub098\uba74\uc11c\"\ubd80\ud130 \"\ubd84\uba85\ud55c \uc5ed\ud6a8\uacfc\"\ub97c \ub0b3\uc558\ub2e4. \ub9e4\ub529\uac70\uc640 \uc774\uc2a4\ud130\ub294 \uc74c\ubc18\uc774 \"\uc5c4\uccad\ub09c \ud639\ud3c9\"\uc744 \ubc1b\uc558\uc73c\uba70, \ud3c9\ub860\uac00 \uc0ac\uc774\uc5d0\uc11c\uc758 \uc774\ub7ec\ud55c \"\uacf5\ud1b5\ub41c \uc9c4\uc220\"\uc740 \"'\ube44\ud2c0\uc988\ub97c \uace0\uc791 \uc774\uac70 \ub54c\ubb38\uc5d0 \ud574\uccb4\uc2dc\ucf30\ub2e4\uace0!?'\uc640 \uc720\uc0ac\ud558\ub2e4\"\uace0 \uc37c\ub2e4. \u300a\uba5c\ub85c\ub514 \uba54\uc774\ucee4\u300b\uc9c0\uc758 \ub9ac\ucc98\ub4dc \uc70c\ub9ac\uc5c4\uc2a4\ub294 \"\uc74c\ubc18\uc758 \ubd84\uba85\ud55c \uc0ac\uc6b4\ub4dc\ub294 \uc870\uc9c0 \ub9c8\ud2f4\uc758 \uacf5\"\uc774\ub77c\uace0 \ubd84\uc11d\ud588\ub2e4. \uc70c\ub9ac\uc5c4\uc2a4\ub294 \u3008Maybe I'm Amazed\u3009\uc5d0\ub294 \"\uc21c\uc804\ud788 \ub530\ubd84\ud568\"\uc774 \uc788\uc744 \ubfd0\uc774\ub77c\uace0 \uc9c0\uc801\ud588\uc73c\uba70, \u3008Man We Was Lonely\u3009\uc5d0 \ub300\ud574\uc11c\ub294 \"\uadf8\uc758 \ubcf4\ub4dc\ube4c \uba74\uc5d0\uc11c\uc758 \ucd5c\uc545\uc758 \uc608\uc2dc\"\ub77c\uace0 \ud45c\ud604\ud588\ub2e4.", "answer": "\ub9ac\ucc98\ub4dc \uc70c\ub9ac\uc5c4\uc2a4", "sentence": "\u300a\uba5c\ub85c\ub514 \uba54\uc774\ucee4\u300b\uc9c0\uc758 \ub9ac\ucc98\ub4dc \uc70c\ub9ac\uc5c4\uc2a4 \ub294 \"\uc74c\ubc18\uc758 \ubd84\uba85\ud55c \uc0ac\uc6b4\ub4dc\ub294 \uc870\uc9c0 \ub9c8\ud2f4\uc758 \uacf5\"\uc774\ub77c\uace0 \ubd84\uc11d\ud588\ub2e4.", "paragraph_sentence": "\u300aMcCartney\u300b\ub294 \ucd9c\uc2dc\ub418\uc790 \ubd88\ucda9\ubd84\ud55c \uc81c\uc791\uacfc \ubbf8\uc644\uc131 \uace1\uc73c\ub85c \uc778\ud574 \uc804 \uc138\uacc4\uc801\uc73c\ub85c \ud639\ud3c9\uc744 \ub4e4\uc5c8\ub2e4. \ub367\ubd99\uc5ec\uc11c \ub2c8\ucf5c\ub77c\uc2a4 \uc0e4\ud504\ub108\uc758 1977\ub144 \uc800\uc11c \u300a\ube44\ud2c0\uc988 \ud3ec\uc5d0\ubc84\u300b(The Beatles Forever)\uc5d0 \uc758\ud558\uba74, \ub9e4\uce74\ud2b8\ub2c8\ub294 \uc790\uc2e0\uc744 \ud589\ubcf5\ud55c \uac00\uc815\uc758 \ub0a8\uc790\ub85c \ub300\ud45c\ud558\uba74\uc11c \ube44\ud2c0\uc988\uc758 \ud574\uccb4\ub97c \uc790\uc2e0\uc758 \uc194\ub85c \uc74c\ubc18 \ud64d\ubcf4\uc5d0 \uc0ac\uc6a9\ud558\ub824\uace0 \ud588\uace0, \uc774\ub294 \"\ub9ce\uc740 \ubaa9\uaca9\uc790\ub4e4\uc5d0 \uc758\ud574 \uc74c\ubc18 \uc804\ubd80\uac00 \uac10\uac01\uc774 \uc5c6\uace0, \uc9c0\ub3c5\ud558\uac8c \uace0\uc548\ub428\uc774 \ub4e4\ud1b5\ub098\uba74\uc11c\"\ubd80\ud130 \"\ubd84\uba85\ud55c \uc5ed\ud6a8\uacfc\"\ub97c \ub0b3\uc558\ub2e4. \ub9e4\ub529\uac70\uc640 \uc774\uc2a4\ud130\ub294 \uc74c\ubc18\uc774 \"\uc5c4\uccad\ub09c \ud639\ud3c9\"\uc744 \ubc1b\uc558\uc73c\uba70, \ud3c9\ub860\uac00 \uc0ac\uc774\uc5d0\uc11c\uc758 \uc774\ub7ec\ud55c \"\uacf5\ud1b5\ub41c \uc9c4\uc220\"\uc740 \"'\ube44\ud2c0\uc988\ub97c \uace0\uc791 \uc774\uac70 \ub54c\ubb38\uc5d0 \ud574\uccb4\uc2dc\ucf30\ub2e4\uace0!?'\uc640 \uc720\uc0ac\ud558\ub2e4\"\uace0 \uc37c\ub2e4. \u300a\uba5c\ub85c\ub514 \uba54\uc774\ucee4\u300b\uc9c0\uc758 \ub9ac\ucc98\ub4dc \uc70c\ub9ac\uc5c4\uc2a4 \ub294 \"\uc74c\ubc18\uc758 \ubd84\uba85\ud55c \uc0ac\uc6b4\ub4dc\ub294 \uc870\uc9c0 \ub9c8\ud2f4\uc758 \uacf5\"\uc774\ub77c\uace0 \ubd84\uc11d\ud588\ub2e4. \uc70c\ub9ac\uc5c4\uc2a4\ub294 \u3008Maybe I'm Amazed\u3009\uc5d0\ub294 \"\uc21c\uc804\ud788 \ub530\ubd84\ud568\"\uc774 \uc788\uc744 \ubfd0\uc774\ub77c\uace0 \uc9c0\uc801\ud588\uc73c\uba70, \u3008Man We Was Lonely\u3009\uc5d0 \ub300\ud574\uc11c\ub294 \"\uadf8\uc758 \ubcf4\ub4dc\ube4c \uba74\uc5d0\uc11c\uc758 \ucd5c\uc545\uc758 \uc608\uc2dc\"\ub77c\uace0 \ud45c\ud604\ud588\ub2e4.", "paragraph_answer": "\u300aMcCartney\u300b\ub294 \ucd9c\uc2dc\ub418\uc790 \ubd88\ucda9\ubd84\ud55c \uc81c\uc791\uacfc \ubbf8\uc644\uc131 \uace1\uc73c\ub85c \uc778\ud574 \uc804 \uc138\uacc4\uc801\uc73c\ub85c \ud639\ud3c9\uc744 \ub4e4\uc5c8\ub2e4. \ub367\ubd99\uc5ec\uc11c \ub2c8\ucf5c\ub77c\uc2a4 \uc0e4\ud504\ub108\uc758 1977\ub144 \uc800\uc11c \u300a\ube44\ud2c0\uc988 \ud3ec\uc5d0\ubc84\u300b(The Beatles Forever)\uc5d0 \uc758\ud558\uba74, \ub9e4\uce74\ud2b8\ub2c8\ub294 \uc790\uc2e0\uc744 \ud589\ubcf5\ud55c \uac00\uc815\uc758 \ub0a8\uc790\ub85c \ub300\ud45c\ud558\uba74\uc11c \ube44\ud2c0\uc988\uc758 \ud574\uccb4\ub97c \uc790\uc2e0\uc758 \uc194\ub85c \uc74c\ubc18 \ud64d\ubcf4\uc5d0 \uc0ac\uc6a9\ud558\ub824\uace0 \ud588\uace0, \uc774\ub294 \"\ub9ce\uc740 \ubaa9\uaca9\uc790\ub4e4\uc5d0 \uc758\ud574 \uc74c\ubc18 \uc804\ubd80\uac00 \uac10\uac01\uc774 \uc5c6\uace0, \uc9c0\ub3c5\ud558\uac8c \uace0\uc548\ub428\uc774 \ub4e4\ud1b5\ub098\uba74\uc11c\"\ubd80\ud130 \"\ubd84\uba85\ud55c \uc5ed\ud6a8\uacfc\"\ub97c \ub0b3\uc558\ub2e4. \ub9e4\ub529\uac70\uc640 \uc774\uc2a4\ud130\ub294 \uc74c\ubc18\uc774 \"\uc5c4\uccad\ub09c \ud639\ud3c9\"\uc744 \ubc1b\uc558\uc73c\uba70, \ud3c9\ub860\uac00 \uc0ac\uc774\uc5d0\uc11c\uc758 \uc774\ub7ec\ud55c \"\uacf5\ud1b5\ub41c \uc9c4\uc220\"\uc740 \"'\ube44\ud2c0\uc988\ub97c \uace0\uc791 \uc774\uac70 \ub54c\ubb38\uc5d0 \ud574\uccb4\uc2dc\ucf30\ub2e4\uace0!?'\uc640 \uc720\uc0ac\ud558\ub2e4\"\uace0 \uc37c\ub2e4. \u300a\uba5c\ub85c\ub514 \uba54\uc774\ucee4\u300b\uc9c0\uc758 \ub9ac\ucc98\ub4dc \uc70c\ub9ac\uc5c4\uc2a4 \ub294 \"\uc74c\ubc18\uc758 \ubd84\uba85\ud55c \uc0ac\uc6b4\ub4dc\ub294 \uc870\uc9c0 \ub9c8\ud2f4\uc758 \uacf5\"\uc774\ub77c\uace0 \ubd84\uc11d\ud588\ub2e4. \uc70c\ub9ac\uc5c4\uc2a4\ub294 \u3008Maybe I'm Amazed\u3009\uc5d0\ub294 \"\uc21c\uc804\ud788 \ub530\ubd84\ud568\"\uc774 \uc788\uc744 \ubfd0\uc774\ub77c\uace0 \uc9c0\uc801\ud588\uc73c\uba70, \u3008Man We Was Lonely\u3009\uc5d0 \ub300\ud574\uc11c\ub294 \"\uadf8\uc758 \ubcf4\ub4dc\ube4c \uba74\uc5d0\uc11c\uc758 \ucd5c\uc545\uc758 \uc608\uc2dc\"\ub77c\uace0 \ud45c\ud604\ud588\ub2e4.", "sentence_answer": "\u300a\uba5c\ub85c\ub514 \uba54\uc774\ucee4\u300b\uc9c0\uc758 \ub9ac\ucc98\ub4dc \uc70c\ub9ac\uc5c4\uc2a4 \ub294 \"\uc74c\ubc18\uc758 \ubd84\uba85\ud55c \uc0ac\uc6b4\ub4dc\ub294 \uc870\uc9c0 \ub9c8\ud2f4\uc758 \uacf5\"\uc774\ub77c\uace0 \ubd84\uc11d\ud588\ub2e4.", "paragraph_id": "6539634-2-1", "paragraph_question": "question: \ub9e4\uce74\ud2b8\ub2c8 \uc74c\ubc18\uc5d0 \ub300\ud574 \ud639\ud3c9\uc744 \uac00\ud588\ub358 \uba5c\ub85c\ub514 \uba54\uc774\ucee4\uc9c0\uc758 \ud3c9\ub860\uac00\ub294 \ub204\uad6c\uc778\uac00?, context: \u300aMcCartney\u300b\ub294 \ucd9c\uc2dc\ub418\uc790 \ubd88\ucda9\ubd84\ud55c \uc81c\uc791\uacfc \ubbf8\uc644\uc131 \uace1\uc73c\ub85c \uc778\ud574 \uc804 \uc138\uacc4\uc801\uc73c\ub85c \ud639\ud3c9\uc744 \ub4e4\uc5c8\ub2e4. \ub367\ubd99\uc5ec\uc11c \ub2c8\ucf5c\ub77c\uc2a4 \uc0e4\ud504\ub108\uc758 1977\ub144 \uc800\uc11c \u300a\ube44\ud2c0\uc988 \ud3ec\uc5d0\ubc84\u300b(The Beatles Forever)\uc5d0 \uc758\ud558\uba74, \ub9e4\uce74\ud2b8\ub2c8\ub294 \uc790\uc2e0\uc744 \ud589\ubcf5\ud55c \uac00\uc815\uc758 \ub0a8\uc790\ub85c \ub300\ud45c\ud558\uba74\uc11c \ube44\ud2c0\uc988\uc758 \ud574\uccb4\ub97c \uc790\uc2e0\uc758 \uc194\ub85c \uc74c\ubc18 \ud64d\ubcf4\uc5d0 \uc0ac\uc6a9\ud558\ub824\uace0 \ud588\uace0, \uc774\ub294 \"\ub9ce\uc740 \ubaa9\uaca9\uc790\ub4e4\uc5d0 \uc758\ud574 \uc74c\ubc18 \uc804\ubd80\uac00 \uac10\uac01\uc774 \uc5c6\uace0, \uc9c0\ub3c5\ud558\uac8c \uace0\uc548\ub428\uc774 \ub4e4\ud1b5\ub098\uba74\uc11c\"\ubd80\ud130 \"\ubd84\uba85\ud55c \uc5ed\ud6a8\uacfc\"\ub97c \ub0b3\uc558\ub2e4. \ub9e4\ub529\uac70\uc640 \uc774\uc2a4\ud130\ub294 \uc74c\ubc18\uc774 \"\uc5c4\uccad\ub09c \ud639\ud3c9\"\uc744 \ubc1b\uc558\uc73c\uba70, \ud3c9\ub860\uac00 \uc0ac\uc774\uc5d0\uc11c\uc758 \uc774\ub7ec\ud55c \"\uacf5\ud1b5\ub41c \uc9c4\uc220\"\uc740 \"'\ube44\ud2c0\uc988\ub97c \uace0\uc791 \uc774\uac70 \ub54c\ubb38\uc5d0 \ud574\uccb4\uc2dc\ucf30\ub2e4\uace0!?'\uc640 \uc720\uc0ac\ud558\ub2e4\"\uace0 \uc37c\ub2e4. \u300a\uba5c\ub85c\ub514 \uba54\uc774\ucee4\u300b\uc9c0\uc758 \ub9ac\ucc98\ub4dc \uc70c\ub9ac\uc5c4\uc2a4\ub294 \"\uc74c\ubc18\uc758 \ubd84\uba85\ud55c \uc0ac\uc6b4\ub4dc\ub294 \uc870\uc9c0 \ub9c8\ud2f4\uc758 \uacf5\"\uc774\ub77c\uace0 \ubd84\uc11d\ud588\ub2e4. \uc70c\ub9ac\uc5c4\uc2a4\ub294 \u3008Maybe I'm Amazed\u3009\uc5d0\ub294 \"\uc21c\uc804\ud788 \ub530\ubd84\ud568\"\uc774 \uc788\uc744 \ubfd0\uc774\ub77c\uace0 \uc9c0\uc801\ud588\uc73c\uba70, \u3008Man We Was Lonely\u3009\uc5d0 \ub300\ud574\uc11c\ub294 \"\uadf8\uc758 \ubcf4\ub4dc\ube4c \uba74\uc5d0\uc11c\uc758 \ucd5c\uc545\uc758 \uc608\uc2dc\"\ub77c\uace0 \ud45c\ud604\ud588\ub2e4."} {"question": "\ub860 \uc190\ub354\uc2a4 \uac10\ub3c5\uc740 \uc5b4\ub5a4 \ud300\uc73c\ub85c\ubd80\ud130 \ub9e8\uccb4\uc2a4\ud130 \uc2dc\ud2f0\ub85c \uc624\uac8c \ub418\uc5c8\ub294\uac00?", "paragraph": "1972\ub144\ubd80\ud130 1974\ub144\uae4c\uc9c0, \ub9e8\uccb4\uc2a4\ud130 \uc2dc\ud2f0\ub294 \uacbd\uae30\uc7a5 \uc548\ud30e\uc73c\ub85c \uc5ec\ub7ec\uac00\uc9c0 \ubcc0\ud654\ub97c \uacaa\uac8c \ub41c\ub2e4. \ube44\uc988\ub2c8\uc2a4\ub9e8\uc778 \ud53c\ud130 \uc2a4\uc6e8\uc77c\uc2a4\uac00 \ud68c\uc7a5\uc774 \ub418\uba70, \ub9d0\ucf64 \uc568\ub9ac\uc2a8\uc740 \uac15\ub4f1\uad8c\uc73c\ub85c \uce58\ub2eb\ub358 1972 - 73\uc2dc\uc98c \ub3c4\uc911, \uac10\ub3c5\uc9c1\uc744 \uc0ac\uc784\ud558\uac8c \ub41c\ub2e4. \uc624\ub79c\uae30\uac04 \ucf54\uce58\ub85c \uc5ed\uc784\ud574 \uc788\ub358 \uc870\ub2c8 \ud558\ud2b8\uac00 \uac10\ub3c5\ub300\ud589\uc744 \ub9e1\uc544 \ud300\uc744 \uac15\ub4f1\uad8c\uc5d0\uc11c \uad6c\ud574\ub0b4\uace0 \uad1c\ucc2e\uc740 \uc131\uc801\uc744 \uac70\ub450\uac8c \ub418\uc5b4 \uacb0\uad6d \uc2dc\uc98c\uc774 \ub05d\ub098\uace0 \uc815\uc2dd\uac10\ub3c5\uc9c1\uc5d0 \uc624\ub974\uac8c \ub41c\ub2e4. \ud558\uc9c0\ub9cc \uac74\uac15\uc0c1\uc758 \ubb38\uc81c\uac00 \uadf8\ub97c \uc555\ubc15\ud588\uace0 \uadf8\uc758 \uac10\ub3c5 \uae30\uac04\uc740 \uc218 \uac1c\uc6d4\ubc16\uc5d0 \uac00\uc9c0 \ubabb\ud588\ub2e4. \ub178\ub9ac\uce58 \uc2dc\ud2f0\uc758 \uac10\ub3c5\uc744 \ub9e1\uace0 \uc788\uc5c8\ub358 \ub860 \uc190\ub354\uc2a4\uac00 \ud558\ud2b8\uc758 \uc790\ub9ac\ub97c \ub300\uc2e0\ud558\uac8c \ub418\ub294\ub370 \uadf8\uc758 \uc9c0\ud718 \uc544\ub798 \ub9e8\uccb4\uc2a4\ud130 \uc2dc\ud2f0\ub294 1974\ub144, \ub9ac\uadf8 \ucef5 \uacb0\uc2b9\uc5d0 \uc9c4\ucd9c\ud558\uc9c0\ub9cc \uc544\uae5d\uac8c \uc900\uc6b0\uc2b9\uc5d0 \uadf8\uce58\uace0 \ub9cc\ub2e4. \ub9ac\uadf8\uc5d0\uc120 \ubd80\uc9c4\ud558\uac8c \ub418\uc5c8\uace0 \uc190\ub354\uc2a4\ub294 1973 - 74\uc2dc\uc98c \ud55c\ub2ec\uc744 \ub0a8\uaca8\ub193\uace0 \ud574\uace0 \ub2f9\ud558\uac8c \ub418\uc5c8\uace0 \ub9e8\uccb4\uc2a4\ud130 \uc2dc\ud2f0\ub3c4 \uac15\ub4f1\ub2f9\ud560 \uc704\uae30\uc5d0 \ucc98\ud558\uac8c \ub41c\ub2e4. \uc774\uc5d0 \u524d \uc8fc\uc7a5\uc774\uc5c8\ub358 \ud1a0\ub2c8 \ubd81\uc774 \uc120\uc218 \uacb8 \uac10\ub3c5\uc73c\ub85c \ud300\uc744 \uc774\ub04c\uc5b4 \uc548\uc815\uc744 \ub418\ucc3e\uac8c \ub41c\ub2e4. \uc2dc\uc98c \ub9c8\uc9c0\ub9c9\ub0a0\uc758 \uc0c1\ub300\ub294 \uc5f0\uace0\uc9c0 \ub77c\uc774\ubc8c\uc778 \ub9e8\uccb4\uc2a4\ud130 \uc720\ub098\uc774\ud2f0\ub4dc \ub85c\uc368 \uac15\ub4f1\uc744 \uba74\ud558\uae30 \uc704\ud574\uc11c \uc591\ud300 \ubaa8\ub450 \uc2b9\ub9ac\uac00 \uc808\uc2e4\ud588\ub358 \uc0c1\ud669\uc774\uc5c8\ub2e4. \u524d \uc720\ub098\uc774\ud2f0\ub4dc \uc120\uc218 \uc600\ub358 \ub370\ub2c8\uc2a4 \ub85c\uac00 \ud790\ud0a5\uc73c\ub85c \ub4dd\uc810\uc744 \uc131\uacf5\uc2dc\ucf1c 1-0 \uc2b9\ub9ac\ub97c \uac70\ub450\uace0 \ub9e8\uccb4\uc2a4\ud130 \uc720\ub098\uc774\ud2f0\ub4dc\uc758 \uac15\ub4f1\uc744 \ud655\uc815\uc9d3\uac8c \ub9cc\ub4e4\uc5c8\ub2e4. \ub9e8\uccb4\uc2a4\ud130 \uc2dc\ud2f0\ub294 \ud1a0\ub2c8 \ubd81\uc758 \uc7ac\uc784\uae30\uac04 \ub3d9\uc548 \uc791\uc740 \uc131\uacf5\uc744 \uac70\ub450\uac8c \ub418\ub294\ub370 1976\ub144, \ub274\uce90\uc2ac \uc720\ub098\uc774\ud2f0\ub4dc\ub97c 2-1\ub85c \uaebe\uace0 \ub450 \ubc88\uc9f8 \ub9ac\uadf8 \ucef5 \uc6b0\uc2b9\uc744 \uac70\uba38\uc950\uc5c8\uace0 1977\ub144\uc5d0\ub294 \ub9ac\uadf8\uc5d0\uc11c 2\uc704\ub97c \uae30\ub85d\ud558\uc600\ub2e4.", "answer": "\ub178\ub9ac\uce58 \uc2dc\ud2f0", "sentence": "\ub178\ub9ac\uce58 \uc2dc\ud2f0 \uc758 \uac10\ub3c5\uc744 \ub9e1\uace0 \uc788\uc5c8\ub358 \ub860 \uc190\ub354\uc2a4\uac00 \ud558\ud2b8\uc758 \uc790\ub9ac\ub97c \ub300\uc2e0\ud558\uac8c \ub418\ub294\ub370 \uadf8\uc758 \uc9c0\ud718 \uc544\ub798 \ub9e8\uccb4\uc2a4\ud130 \uc2dc\ud2f0\ub294 1974\ub144, \ub9ac\uadf8 \ucef5 \uacb0\uc2b9\uc5d0 \uc9c4\ucd9c\ud558\uc9c0\ub9cc \uc544\uae5d\uac8c \uc900\uc6b0\uc2b9\uc5d0 \uadf8\uce58\uace0 \ub9cc\ub2e4.", "paragraph_sentence": "1972\ub144\ubd80\ud130 1974\ub144\uae4c\uc9c0, \ub9e8\uccb4\uc2a4\ud130 \uc2dc\ud2f0\ub294 \uacbd\uae30\uc7a5 \uc548\ud30e\uc73c\ub85c \uc5ec\ub7ec\uac00\uc9c0 \ubcc0\ud654\ub97c \uacaa\uac8c \ub41c\ub2e4. \ube44\uc988\ub2c8\uc2a4\ub9e8\uc778 \ud53c\ud130 \uc2a4\uc6e8\uc77c\uc2a4\uac00 \ud68c\uc7a5\uc774 \ub418\uba70, \ub9d0\ucf64 \uc568\ub9ac\uc2a8\uc740 \uac15\ub4f1\uad8c\uc73c\ub85c \uce58\ub2eb\ub358 1972 - 73\uc2dc\uc98c \ub3c4\uc911, \uac10\ub3c5\uc9c1\uc744 \uc0ac\uc784\ud558\uac8c \ub41c\ub2e4. \uc624\ub79c\uae30\uac04 \ucf54\uce58\ub85c \uc5ed\uc784\ud574 \uc788\ub358 \uc870\ub2c8 \ud558\ud2b8\uac00 \uac10\ub3c5\ub300\ud589\uc744 \ub9e1\uc544 \ud300\uc744 \uac15\ub4f1\uad8c\uc5d0\uc11c \uad6c\ud574\ub0b4\uace0 \uad1c\ucc2e\uc740 \uc131\uc801\uc744 \uac70\ub450\uac8c \ub418\uc5b4 \uacb0\uad6d \uc2dc\uc98c\uc774 \ub05d\ub098\uace0 \uc815\uc2dd\uac10\ub3c5\uc9c1\uc5d0 \uc624\ub974\uac8c \ub41c\ub2e4. \ud558\uc9c0\ub9cc \uac74\uac15\uc0c1\uc758 \ubb38\uc81c\uac00 \uadf8\ub97c \uc555\ubc15\ud588\uace0 \uadf8\uc758 \uac10\ub3c5 \uae30\uac04\uc740 \uc218 \uac1c\uc6d4\ubc16\uc5d0 \uac00\uc9c0 \ubabb\ud588\ub2e4. \ub178\ub9ac\uce58 \uc2dc\ud2f0 \uc758 \uac10\ub3c5\uc744 \ub9e1\uace0 \uc788\uc5c8\ub358 \ub860 \uc190\ub354\uc2a4\uac00 \ud558\ud2b8\uc758 \uc790\ub9ac\ub97c \ub300\uc2e0\ud558\uac8c \ub418\ub294\ub370 \uadf8\uc758 \uc9c0\ud718 \uc544\ub798 \ub9e8\uccb4\uc2a4\ud130 \uc2dc\ud2f0\ub294 1974\ub144, \ub9ac\uadf8 \ucef5 \uacb0\uc2b9\uc5d0 \uc9c4\ucd9c\ud558\uc9c0\ub9cc \uc544\uae5d\uac8c \uc900\uc6b0\uc2b9\uc5d0 \uadf8\uce58\uace0 \ub9cc\ub2e4. \ub9ac\uadf8\uc5d0\uc120 \ubd80\uc9c4\ud558\uac8c \ub418\uc5c8\uace0 \uc190\ub354\uc2a4\ub294 1973 - 74\uc2dc\uc98c \ud55c\ub2ec\uc744 \ub0a8\uaca8\ub193\uace0 \ud574\uace0 \ub2f9\ud558\uac8c \ub418\uc5c8\uace0 \ub9e8\uccb4\uc2a4\ud130 \uc2dc\ud2f0\ub3c4 \uac15\ub4f1\ub2f9\ud560 \uc704\uae30\uc5d0 \ucc98\ud558\uac8c \ub41c\ub2e4. \uc774\uc5d0 \u524d \uc8fc\uc7a5\uc774\uc5c8\ub358 \ud1a0\ub2c8 \ubd81\uc774 \uc120\uc218 \uacb8 \uac10\ub3c5\uc73c\ub85c \ud300\uc744 \uc774\ub04c\uc5b4 \uc548\uc815\uc744 \ub418\ucc3e\uac8c \ub41c\ub2e4. \uc2dc\uc98c \ub9c8\uc9c0\ub9c9\ub0a0\uc758 \uc0c1\ub300\ub294 \uc5f0\uace0\uc9c0 \ub77c\uc774\ubc8c\uc778 \ub9e8\uccb4\uc2a4\ud130 \uc720\ub098\uc774\ud2f0\ub4dc \ub85c\uc368 \uac15\ub4f1\uc744 \uba74\ud558\uae30 \uc704\ud574\uc11c \uc591\ud300 \ubaa8\ub450 \uc2b9\ub9ac\uac00 \uc808\uc2e4\ud588\ub358 \uc0c1\ud669\uc774\uc5c8\ub2e4. \u524d \uc720\ub098\uc774\ud2f0\ub4dc \uc120\uc218 \uc600\ub358 \ub370\ub2c8\uc2a4 \ub85c\uac00 \ud790\ud0a5\uc73c\ub85c \ub4dd\uc810\uc744 \uc131\uacf5\uc2dc\ucf1c 1-0 \uc2b9\ub9ac\ub97c \uac70\ub450\uace0 \ub9e8\uccb4\uc2a4\ud130 \uc720\ub098\uc774\ud2f0\ub4dc\uc758 \uac15\ub4f1\uc744 \ud655\uc815\uc9d3\uac8c \ub9cc\ub4e4\uc5c8\ub2e4. \ub9e8\uccb4\uc2a4\ud130 \uc2dc\ud2f0\ub294 \ud1a0\ub2c8 \ubd81\uc758 \uc7ac\uc784\uae30\uac04 \ub3d9\uc548 \uc791\uc740 \uc131\uacf5\uc744 \uac70\ub450\uac8c \ub418\ub294\ub370 1976\ub144, \ub274\uce90\uc2ac \uc720\ub098\uc774\ud2f0\ub4dc\ub97c 2-1\ub85c \uaebe\uace0 \ub450 \ubc88\uc9f8 \ub9ac\uadf8 \ucef5 \uc6b0\uc2b9\uc744 \uac70\uba38\uc950\uc5c8\uace0 1977\ub144\uc5d0\ub294 \ub9ac\uadf8\uc5d0\uc11c 2\uc704\ub97c \uae30\ub85d\ud558\uc600\ub2e4.", "paragraph_answer": "1972\ub144\ubd80\ud130 1974\ub144\uae4c\uc9c0, \ub9e8\uccb4\uc2a4\ud130 \uc2dc\ud2f0\ub294 \uacbd\uae30\uc7a5 \uc548\ud30e\uc73c\ub85c \uc5ec\ub7ec\uac00\uc9c0 \ubcc0\ud654\ub97c \uacaa\uac8c \ub41c\ub2e4. \ube44\uc988\ub2c8\uc2a4\ub9e8\uc778 \ud53c\ud130 \uc2a4\uc6e8\uc77c\uc2a4\uac00 \ud68c\uc7a5\uc774 \ub418\uba70, \ub9d0\ucf64 \uc568\ub9ac\uc2a8\uc740 \uac15\ub4f1\uad8c\uc73c\ub85c \uce58\ub2eb\ub358 1972 - 73\uc2dc\uc98c \ub3c4\uc911, \uac10\ub3c5\uc9c1\uc744 \uc0ac\uc784\ud558\uac8c \ub41c\ub2e4. \uc624\ub79c\uae30\uac04 \ucf54\uce58\ub85c \uc5ed\uc784\ud574 \uc788\ub358 \uc870\ub2c8 \ud558\ud2b8\uac00 \uac10\ub3c5\ub300\ud589\uc744 \ub9e1\uc544 \ud300\uc744 \uac15\ub4f1\uad8c\uc5d0\uc11c \uad6c\ud574\ub0b4\uace0 \uad1c\ucc2e\uc740 \uc131\uc801\uc744 \uac70\ub450\uac8c \ub418\uc5b4 \uacb0\uad6d \uc2dc\uc98c\uc774 \ub05d\ub098\uace0 \uc815\uc2dd\uac10\ub3c5\uc9c1\uc5d0 \uc624\ub974\uac8c \ub41c\ub2e4. \ud558\uc9c0\ub9cc \uac74\uac15\uc0c1\uc758 \ubb38\uc81c\uac00 \uadf8\ub97c \uc555\ubc15\ud588\uace0 \uadf8\uc758 \uac10\ub3c5 \uae30\uac04\uc740 \uc218 \uac1c\uc6d4\ubc16\uc5d0 \uac00\uc9c0 \ubabb\ud588\ub2e4. \ub178\ub9ac\uce58 \uc2dc\ud2f0 \uc758 \uac10\ub3c5\uc744 \ub9e1\uace0 \uc788\uc5c8\ub358 \ub860 \uc190\ub354\uc2a4\uac00 \ud558\ud2b8\uc758 \uc790\ub9ac\ub97c \ub300\uc2e0\ud558\uac8c \ub418\ub294\ub370 \uadf8\uc758 \uc9c0\ud718 \uc544\ub798 \ub9e8\uccb4\uc2a4\ud130 \uc2dc\ud2f0\ub294 1974\ub144, \ub9ac\uadf8 \ucef5 \uacb0\uc2b9\uc5d0 \uc9c4\ucd9c\ud558\uc9c0\ub9cc \uc544\uae5d\uac8c \uc900\uc6b0\uc2b9\uc5d0 \uadf8\uce58\uace0 \ub9cc\ub2e4. \ub9ac\uadf8\uc5d0\uc120 \ubd80\uc9c4\ud558\uac8c \ub418\uc5c8\uace0 \uc190\ub354\uc2a4\ub294 1973 - 74\uc2dc\uc98c \ud55c\ub2ec\uc744 \ub0a8\uaca8\ub193\uace0 \ud574\uace0 \ub2f9\ud558\uac8c \ub418\uc5c8\uace0 \ub9e8\uccb4\uc2a4\ud130 \uc2dc\ud2f0\ub3c4 \uac15\ub4f1\ub2f9\ud560 \uc704\uae30\uc5d0 \ucc98\ud558\uac8c \ub41c\ub2e4. \uc774\uc5d0 \u524d \uc8fc\uc7a5\uc774\uc5c8\ub358 \ud1a0\ub2c8 \ubd81\uc774 \uc120\uc218 \uacb8 \uac10\ub3c5\uc73c\ub85c \ud300\uc744 \uc774\ub04c\uc5b4 \uc548\uc815\uc744 \ub418\ucc3e\uac8c \ub41c\ub2e4. \uc2dc\uc98c \ub9c8\uc9c0\ub9c9\ub0a0\uc758 \uc0c1\ub300\ub294 \uc5f0\uace0\uc9c0 \ub77c\uc774\ubc8c\uc778 \ub9e8\uccb4\uc2a4\ud130 \uc720\ub098\uc774\ud2f0\ub4dc \ub85c\uc368 \uac15\ub4f1\uc744 \uba74\ud558\uae30 \uc704\ud574\uc11c \uc591\ud300 \ubaa8\ub450 \uc2b9\ub9ac\uac00 \uc808\uc2e4\ud588\ub358 \uc0c1\ud669\uc774\uc5c8\ub2e4. \u524d \uc720\ub098\uc774\ud2f0\ub4dc \uc120\uc218 \uc600\ub358 \ub370\ub2c8\uc2a4 \ub85c\uac00 \ud790\ud0a5\uc73c\ub85c \ub4dd\uc810\uc744 \uc131\uacf5\uc2dc\ucf1c 1-0 \uc2b9\ub9ac\ub97c \uac70\ub450\uace0 \ub9e8\uccb4\uc2a4\ud130 \uc720\ub098\uc774\ud2f0\ub4dc\uc758 \uac15\ub4f1\uc744 \ud655\uc815\uc9d3\uac8c \ub9cc\ub4e4\uc5c8\ub2e4. \ub9e8\uccb4\uc2a4\ud130 \uc2dc\ud2f0\ub294 \ud1a0\ub2c8 \ubd81\uc758 \uc7ac\uc784\uae30\uac04 \ub3d9\uc548 \uc791\uc740 \uc131\uacf5\uc744 \uac70\ub450\uac8c \ub418\ub294\ub370 1976\ub144, \ub274\uce90\uc2ac \uc720\ub098\uc774\ud2f0\ub4dc\ub97c 2-1\ub85c \uaebe\uace0 \ub450 \ubc88\uc9f8 \ub9ac\uadf8 \ucef5 \uc6b0\uc2b9\uc744 \uac70\uba38\uc950\uc5c8\uace0 1977\ub144\uc5d0\ub294 \ub9ac\uadf8\uc5d0\uc11c 2\uc704\ub97c \uae30\ub85d\ud558\uc600\ub2e4.", "sentence_answer": " \ub178\ub9ac\uce58 \uc2dc\ud2f0 \uc758 \uac10\ub3c5\uc744 \ub9e1\uace0 \uc788\uc5c8\ub358 \ub860 \uc190\ub354\uc2a4\uac00 \ud558\ud2b8\uc758 \uc790\ub9ac\ub97c \ub300\uc2e0\ud558\uac8c \ub418\ub294\ub370 \uadf8\uc758 \uc9c0\ud718 \uc544\ub798 \ub9e8\uccb4\uc2a4\ud130 \uc2dc\ud2f0\ub294 1974\ub144, \ub9ac\uadf8 \ucef5 \uacb0\uc2b9\uc5d0 \uc9c4\ucd9c\ud558\uc9c0\ub9cc \uc544\uae5d\uac8c \uc900\uc6b0\uc2b9\uc5d0 \uadf8\uce58\uace0 \ub9cc\ub2e4.", "paragraph_id": "6502151-8-0", "paragraph_question": "question: \ub860 \uc190\ub354\uc2a4 \uac10\ub3c5\uc740 \uc5b4\ub5a4 \ud300\uc73c\ub85c\ubd80\ud130 \ub9e8\uccb4\uc2a4\ud130 \uc2dc\ud2f0\ub85c \uc624\uac8c \ub418\uc5c8\ub294\uac00?, context: 1972\ub144\ubd80\ud130 1974\ub144\uae4c\uc9c0, \ub9e8\uccb4\uc2a4\ud130 \uc2dc\ud2f0\ub294 \uacbd\uae30\uc7a5 \uc548\ud30e\uc73c\ub85c \uc5ec\ub7ec\uac00\uc9c0 \ubcc0\ud654\ub97c \uacaa\uac8c \ub41c\ub2e4. \ube44\uc988\ub2c8\uc2a4\ub9e8\uc778 \ud53c\ud130 \uc2a4\uc6e8\uc77c\uc2a4\uac00 \ud68c\uc7a5\uc774 \ub418\uba70, \ub9d0\ucf64 \uc568\ub9ac\uc2a8\uc740 \uac15\ub4f1\uad8c\uc73c\ub85c \uce58\ub2eb\ub358 1972 - 73\uc2dc\uc98c \ub3c4\uc911, \uac10\ub3c5\uc9c1\uc744 \uc0ac\uc784\ud558\uac8c \ub41c\ub2e4. \uc624\ub79c\uae30\uac04 \ucf54\uce58\ub85c \uc5ed\uc784\ud574 \uc788\ub358 \uc870\ub2c8 \ud558\ud2b8\uac00 \uac10\ub3c5\ub300\ud589\uc744 \ub9e1\uc544 \ud300\uc744 \uac15\ub4f1\uad8c\uc5d0\uc11c \uad6c\ud574\ub0b4\uace0 \uad1c\ucc2e\uc740 \uc131\uc801\uc744 \uac70\ub450\uac8c \ub418\uc5b4 \uacb0\uad6d \uc2dc\uc98c\uc774 \ub05d\ub098\uace0 \uc815\uc2dd\uac10\ub3c5\uc9c1\uc5d0 \uc624\ub974\uac8c \ub41c\ub2e4. \ud558\uc9c0\ub9cc \uac74\uac15\uc0c1\uc758 \ubb38\uc81c\uac00 \uadf8\ub97c \uc555\ubc15\ud588\uace0 \uadf8\uc758 \uac10\ub3c5 \uae30\uac04\uc740 \uc218 \uac1c\uc6d4\ubc16\uc5d0 \uac00\uc9c0 \ubabb\ud588\ub2e4. \ub178\ub9ac\uce58 \uc2dc\ud2f0\uc758 \uac10\ub3c5\uc744 \ub9e1\uace0 \uc788\uc5c8\ub358 \ub860 \uc190\ub354\uc2a4\uac00 \ud558\ud2b8\uc758 \uc790\ub9ac\ub97c \ub300\uc2e0\ud558\uac8c \ub418\ub294\ub370 \uadf8\uc758 \uc9c0\ud718 \uc544\ub798 \ub9e8\uccb4\uc2a4\ud130 \uc2dc\ud2f0\ub294 1974\ub144, \ub9ac\uadf8 \ucef5 \uacb0\uc2b9\uc5d0 \uc9c4\ucd9c\ud558\uc9c0\ub9cc \uc544\uae5d\uac8c \uc900\uc6b0\uc2b9\uc5d0 \uadf8\uce58\uace0 \ub9cc\ub2e4. \ub9ac\uadf8\uc5d0\uc120 \ubd80\uc9c4\ud558\uac8c \ub418\uc5c8\uace0 \uc190\ub354\uc2a4\ub294 1973 - 74\uc2dc\uc98c \ud55c\ub2ec\uc744 \ub0a8\uaca8\ub193\uace0 \ud574\uace0 \ub2f9\ud558\uac8c \ub418\uc5c8\uace0 \ub9e8\uccb4\uc2a4\ud130 \uc2dc\ud2f0\ub3c4 \uac15\ub4f1\ub2f9\ud560 \uc704\uae30\uc5d0 \ucc98\ud558\uac8c \ub41c\ub2e4. \uc774\uc5d0 \u524d \uc8fc\uc7a5\uc774\uc5c8\ub358 \ud1a0\ub2c8 \ubd81\uc774 \uc120\uc218 \uacb8 \uac10\ub3c5\uc73c\ub85c \ud300\uc744 \uc774\ub04c\uc5b4 \uc548\uc815\uc744 \ub418\ucc3e\uac8c \ub41c\ub2e4. \uc2dc\uc98c \ub9c8\uc9c0\ub9c9\ub0a0\uc758 \uc0c1\ub300\ub294 \uc5f0\uace0\uc9c0 \ub77c\uc774\ubc8c\uc778 \ub9e8\uccb4\uc2a4\ud130 \uc720\ub098\uc774\ud2f0\ub4dc \ub85c\uc368 \uac15\ub4f1\uc744 \uba74\ud558\uae30 \uc704\ud574\uc11c \uc591\ud300 \ubaa8\ub450 \uc2b9\ub9ac\uac00 \uc808\uc2e4\ud588\ub358 \uc0c1\ud669\uc774\uc5c8\ub2e4. \u524d \uc720\ub098\uc774\ud2f0\ub4dc \uc120\uc218 \uc600\ub358 \ub370\ub2c8\uc2a4 \ub85c\uac00 \ud790\ud0a5\uc73c\ub85c \ub4dd\uc810\uc744 \uc131\uacf5\uc2dc\ucf1c 1-0 \uc2b9\ub9ac\ub97c \uac70\ub450\uace0 \ub9e8\uccb4\uc2a4\ud130 \uc720\ub098\uc774\ud2f0\ub4dc\uc758 \uac15\ub4f1\uc744 \ud655\uc815\uc9d3\uac8c \ub9cc\ub4e4\uc5c8\ub2e4. \ub9e8\uccb4\uc2a4\ud130 \uc2dc\ud2f0\ub294 \ud1a0\ub2c8 \ubd81\uc758 \uc7ac\uc784\uae30\uac04 \ub3d9\uc548 \uc791\uc740 \uc131\uacf5\uc744 \uac70\ub450\uac8c \ub418\ub294\ub370 1976\ub144, \ub274\uce90\uc2ac \uc720\ub098\uc774\ud2f0\ub4dc\ub97c 2-1\ub85c \uaebe\uace0 \ub450 \ubc88\uc9f8 \ub9ac\uadf8 \ucef5 \uc6b0\uc2b9\uc744 \uac70\uba38\uc950\uc5c8\uace0 1977\ub144\uc5d0\ub294 \ub9ac\uadf8\uc5d0\uc11c 2\uc704\ub97c \uae30\ub85d\ud558\uc600\ub2e4."} {"question": "\ucf00\uc774\ube14 \uce21 \ube44\ub300\uc704\uac00 \uc9c0\uc0c1\ud30c \uce21\uc5d0 \ucd09\uad6c\ud55c \uc0ac\ud56d\uc740?", "paragraph": "1\uc6d4 17\uc77c\uc5d0 \ucf00\uc774\ube14 \ubc29\uc1a1 \ube44\uc0c1\ub300\ucc45\uc704\uc6d0\ud68c\ub294 \ubc29\uc1a1\ud1b5\uc2e0\uc704\uc6d0\ud68c\uc758 \uc81c\uc81c \uc870\uce58\ub97c \ubc18\ubc15\ud558\ub294 \uc131\uba85\uc11c\ub97c \ubc1c\ud45c\ud558\uc600\ub2e4. \ucf00\uc774\ube14 \uc5c5\uacc4\ub294 \uc131\uba85\uc11c\ub97c \ud1b5\ud574, '\uc9c0\uc0c1\ud30c \uc7ac\uc1a1\uc2e0 \ubb38\uc81c\ub294 \ubc29\ud1b5\uc704\uac00 \uc81c\ub3c4 \uac1c\uc120\uc744 \ud1b5\ud574 \uadfc\ubcf8\uc801\uc778 \ub300\ucc45\uc744 \ub9c8\ub828\ud574\uc57c \ud588\uc74c\uc5d0\ub3c4 \ubd88\uad6c\ud558\uace0 \uc2dc\uc815\uba85\ub839\uc744 \ud1b5\ud574 \uc0ac\ud6c4\uc57d\ubc29\ubb38 \uc2dd\uc73c\ub85c \uc77c\uc744 \ud574\uacb0\ud558\ub824 \ud55c\ub2e4.' \uba70 \ubc29\uc1a1\ud1b5\uc2e0\uc704\uc6d0\ud68c\ub97c \ube44\ub09c\ud558\uc600\ub2e4. \ub610\ud55c \uc790\uc2e0\ub4e4\uc740 \ubc95\uc6d0\uc758 \ud310\uacb0\uc744 \uc874\uc911\ud558\uc5ec \uc9c0\uc0c1\ud30c \uc7ac\uc1a1\uc2e0\uc744 \uc911\ub2e8\ud558\uc600\ub294\ub370, \uc65c \ucf00\uc774\ube14 \ud154\ub808\ube44\uc804\uc5d0\ub9cc \uc81c\uc81c\ub97c \uac00\ud558\ub290\ub0d0\ub294 \ubd88\ub9cc\uc744 \ud45c\uc2dc\ud558\uc600\ub2e4. \uadf8\ub7ec\uba74\uc11c \uc774\ud589\uac15\uc81c\uae08\uc774 100\uc5b5 \uc6d0\uc744 \ub118\uc5b4\uac10\uc5d0 \ub530\ub77c \ubc95\uc801 \uc555\ubc15\uc744 \ud574 \uc628 \uc9c0\uc0c1\ud30c \ud154\ub808\ube44\uc804 \ubc29\uc1a1 3\uc0ac\uc640 \uc774\ub4e4\uc758 \ub208\uce58 \ubcf4\uae30\uc5d0\ub9cc \uae09\uae09\ud55c \ubc29\ud1b5\uc704\uc5d0 \uc774 \uc0ac\ud0dc\uc5d0 \ub300\ud55c \ucc45\uc784\uc744 \ubb3c\uc74c\uacfc \ub3d9\uc2dc\uc5d0, \uc9c0\uc0c1\ud30c \ud154\ub808\ube44\uc804 \ubc29\uc1a1 3\uc0ac\uc5d0\ub294 \uc804\ud5a5\uc801\uc778 \ud611\uc0c1 \ud0dc\ub3c4\uc640, \ubc29\uc1a1\ud1b5\uc2e0\uc704\uc6d0\ud68c\uc5d0\ub294 \uc7ac\uc1a1\uc2e0 \uc81c\ub3c4 \uac1c\uc120\uc744 \ucd09\uad6c\ud558\uc600\ub2e4.", "answer": "\uc804\ud5a5\uc801\uc778 \ud611\uc0c1 \ud0dc\ub3c4", "sentence": "\uadf8\ub7ec\uba74\uc11c \uc774\ud589\uac15\uc81c\uae08\uc774 100\uc5b5 \uc6d0\uc744 \ub118\uc5b4\uac10\uc5d0 \ub530\ub77c \ubc95\uc801 \uc555\ubc15\uc744 \ud574 \uc628 \uc9c0\uc0c1\ud30c \ud154\ub808\ube44\uc804 \ubc29\uc1a1 3\uc0ac\uc640 \uc774\ub4e4\uc758 \ub208\uce58 \ubcf4\uae30\uc5d0\ub9cc \uae09\uae09\ud55c \ubc29\ud1b5\uc704\uc5d0 \uc774 \uc0ac\ud0dc\uc5d0 \ub300\ud55c \ucc45\uc784\uc744 \ubb3c\uc74c\uacfc \ub3d9\uc2dc\uc5d0, \uc9c0\uc0c1\ud30c \ud154\ub808\ube44\uc804 \ubc29\uc1a1 3\uc0ac\uc5d0\ub294 \uc804\ud5a5\uc801\uc778 \ud611\uc0c1 \ud0dc\ub3c4 \uc640, \ubc29\uc1a1\ud1b5\uc2e0\uc704\uc6d0\ud68c\uc5d0\ub294 \uc7ac\uc1a1\uc2e0 \uc81c\ub3c4 \uac1c\uc120\uc744 \ucd09\uad6c\ud558\uc600\ub2e4.", "paragraph_sentence": "1\uc6d4 17\uc77c\uc5d0 \ucf00\uc774\ube14 \ubc29\uc1a1 \ube44\uc0c1\ub300\ucc45\uc704\uc6d0\ud68c\ub294 \ubc29\uc1a1\ud1b5\uc2e0\uc704\uc6d0\ud68c\uc758 \uc81c\uc81c \uc870\uce58\ub97c \ubc18\ubc15\ud558\ub294 \uc131\uba85\uc11c\ub97c \ubc1c\ud45c\ud558\uc600\ub2e4. \ucf00\uc774\ube14 \uc5c5\uacc4\ub294 \uc131\uba85\uc11c\ub97c \ud1b5\ud574, '\uc9c0\uc0c1\ud30c \uc7ac\uc1a1\uc2e0 \ubb38\uc81c\ub294 \ubc29\ud1b5\uc704\uac00 \uc81c\ub3c4 \uac1c\uc120\uc744 \ud1b5\ud574 \uadfc\ubcf8\uc801\uc778 \ub300\ucc45\uc744 \ub9c8\ub828\ud574\uc57c \ud588\uc74c\uc5d0\ub3c4 \ubd88\uad6c\ud558\uace0 \uc2dc\uc815\uba85\ub839\uc744 \ud1b5\ud574 \uc0ac\ud6c4\uc57d\ubc29\ubb38 \uc2dd\uc73c\ub85c \uc77c\uc744 \ud574\uacb0\ud558\ub824 \ud55c\ub2e4. ' \uba70 \ubc29\uc1a1\ud1b5\uc2e0\uc704\uc6d0\ud68c\ub97c \ube44\ub09c\ud558\uc600\ub2e4. \ub610\ud55c \uc790\uc2e0\ub4e4\uc740 \ubc95\uc6d0\uc758 \ud310\uacb0\uc744 \uc874\uc911\ud558\uc5ec \uc9c0\uc0c1\ud30c \uc7ac\uc1a1\uc2e0\uc744 \uc911\ub2e8\ud558\uc600\ub294\ub370, \uc65c \ucf00\uc774\ube14 \ud154\ub808\ube44\uc804\uc5d0\ub9cc \uc81c\uc81c\ub97c \uac00\ud558\ub290\ub0d0\ub294 \ubd88\ub9cc\uc744 \ud45c\uc2dc\ud558\uc600\ub2e4. \uadf8\ub7ec\uba74\uc11c \uc774\ud589\uac15\uc81c\uae08\uc774 100\uc5b5 \uc6d0\uc744 \ub118\uc5b4\uac10\uc5d0 \ub530\ub77c \ubc95\uc801 \uc555\ubc15\uc744 \ud574 \uc628 \uc9c0\uc0c1\ud30c \ud154\ub808\ube44\uc804 \ubc29\uc1a1 3\uc0ac\uc640 \uc774\ub4e4\uc758 \ub208\uce58 \ubcf4\uae30\uc5d0\ub9cc \uae09\uae09\ud55c \ubc29\ud1b5\uc704\uc5d0 \uc774 \uc0ac\ud0dc\uc5d0 \ub300\ud55c \ucc45\uc784\uc744 \ubb3c\uc74c\uacfc \ub3d9\uc2dc\uc5d0, \uc9c0\uc0c1\ud30c \ud154\ub808\ube44\uc804 \ubc29\uc1a1 3\uc0ac\uc5d0\ub294 \uc804\ud5a5\uc801\uc778 \ud611\uc0c1 \ud0dc\ub3c4 \uc640, \ubc29\uc1a1\ud1b5\uc2e0\uc704\uc6d0\ud68c\uc5d0\ub294 \uc7ac\uc1a1\uc2e0 \uc81c\ub3c4 \uac1c\uc120\uc744 \ucd09\uad6c\ud558\uc600\ub2e4. ", "paragraph_answer": "1\uc6d4 17\uc77c\uc5d0 \ucf00\uc774\ube14 \ubc29\uc1a1 \ube44\uc0c1\ub300\ucc45\uc704\uc6d0\ud68c\ub294 \ubc29\uc1a1\ud1b5\uc2e0\uc704\uc6d0\ud68c\uc758 \uc81c\uc81c \uc870\uce58\ub97c \ubc18\ubc15\ud558\ub294 \uc131\uba85\uc11c\ub97c \ubc1c\ud45c\ud558\uc600\ub2e4. \ucf00\uc774\ube14 \uc5c5\uacc4\ub294 \uc131\uba85\uc11c\ub97c \ud1b5\ud574, '\uc9c0\uc0c1\ud30c \uc7ac\uc1a1\uc2e0 \ubb38\uc81c\ub294 \ubc29\ud1b5\uc704\uac00 \uc81c\ub3c4 \uac1c\uc120\uc744 \ud1b5\ud574 \uadfc\ubcf8\uc801\uc778 \ub300\ucc45\uc744 \ub9c8\ub828\ud574\uc57c \ud588\uc74c\uc5d0\ub3c4 \ubd88\uad6c\ud558\uace0 \uc2dc\uc815\uba85\ub839\uc744 \ud1b5\ud574 \uc0ac\ud6c4\uc57d\ubc29\ubb38 \uc2dd\uc73c\ub85c \uc77c\uc744 \ud574\uacb0\ud558\ub824 \ud55c\ub2e4.' \uba70 \ubc29\uc1a1\ud1b5\uc2e0\uc704\uc6d0\ud68c\ub97c \ube44\ub09c\ud558\uc600\ub2e4. \ub610\ud55c \uc790\uc2e0\ub4e4\uc740 \ubc95\uc6d0\uc758 \ud310\uacb0\uc744 \uc874\uc911\ud558\uc5ec \uc9c0\uc0c1\ud30c \uc7ac\uc1a1\uc2e0\uc744 \uc911\ub2e8\ud558\uc600\ub294\ub370, \uc65c \ucf00\uc774\ube14 \ud154\ub808\ube44\uc804\uc5d0\ub9cc \uc81c\uc81c\ub97c \uac00\ud558\ub290\ub0d0\ub294 \ubd88\ub9cc\uc744 \ud45c\uc2dc\ud558\uc600\ub2e4. \uadf8\ub7ec\uba74\uc11c \uc774\ud589\uac15\uc81c\uae08\uc774 100\uc5b5 \uc6d0\uc744 \ub118\uc5b4\uac10\uc5d0 \ub530\ub77c \ubc95\uc801 \uc555\ubc15\uc744 \ud574 \uc628 \uc9c0\uc0c1\ud30c \ud154\ub808\ube44\uc804 \ubc29\uc1a1 3\uc0ac\uc640 \uc774\ub4e4\uc758 \ub208\uce58 \ubcf4\uae30\uc5d0\ub9cc \uae09\uae09\ud55c \ubc29\ud1b5\uc704\uc5d0 \uc774 \uc0ac\ud0dc\uc5d0 \ub300\ud55c \ucc45\uc784\uc744 \ubb3c\uc74c\uacfc \ub3d9\uc2dc\uc5d0, \uc9c0\uc0c1\ud30c \ud154\ub808\ube44\uc804 \ubc29\uc1a1 3\uc0ac\uc5d0\ub294 \uc804\ud5a5\uc801\uc778 \ud611\uc0c1 \ud0dc\ub3c4 \uc640, \ubc29\uc1a1\ud1b5\uc2e0\uc704\uc6d0\ud68c\uc5d0\ub294 \uc7ac\uc1a1\uc2e0 \uc81c\ub3c4 \uac1c\uc120\uc744 \ucd09\uad6c\ud558\uc600\ub2e4.", "sentence_answer": "\uadf8\ub7ec\uba74\uc11c \uc774\ud589\uac15\uc81c\uae08\uc774 100\uc5b5 \uc6d0\uc744 \ub118\uc5b4\uac10\uc5d0 \ub530\ub77c \ubc95\uc801 \uc555\ubc15\uc744 \ud574 \uc628 \uc9c0\uc0c1\ud30c \ud154\ub808\ube44\uc804 \ubc29\uc1a1 3\uc0ac\uc640 \uc774\ub4e4\uc758 \ub208\uce58 \ubcf4\uae30\uc5d0\ub9cc \uae09\uae09\ud55c \ubc29\ud1b5\uc704\uc5d0 \uc774 \uc0ac\ud0dc\uc5d0 \ub300\ud55c \ucc45\uc784\uc744 \ubb3c\uc74c\uacfc \ub3d9\uc2dc\uc5d0, \uc9c0\uc0c1\ud30c \ud154\ub808\ube44\uc804 \ubc29\uc1a1 3\uc0ac\uc5d0\ub294 \uc804\ud5a5\uc801\uc778 \ud611\uc0c1 \ud0dc\ub3c4 \uc640, \ubc29\uc1a1\ud1b5\uc2e0\uc704\uc6d0\ud68c\uc5d0\ub294 \uc7ac\uc1a1\uc2e0 \uc81c\ub3c4 \uac1c\uc120\uc744 \ucd09\uad6c\ud558\uc600\ub2e4.", "paragraph_id": "6486479-3-0", "paragraph_question": "question: \ucf00\uc774\ube14 \uce21 \ube44\ub300\uc704\uac00 \uc9c0\uc0c1\ud30c \uce21\uc5d0 \ucd09\uad6c\ud55c \uc0ac\ud56d\uc740?, context: 1\uc6d4 17\uc77c\uc5d0 \ucf00\uc774\ube14 \ubc29\uc1a1 \ube44\uc0c1\ub300\ucc45\uc704\uc6d0\ud68c\ub294 \ubc29\uc1a1\ud1b5\uc2e0\uc704\uc6d0\ud68c\uc758 \uc81c\uc81c \uc870\uce58\ub97c \ubc18\ubc15\ud558\ub294 \uc131\uba85\uc11c\ub97c \ubc1c\ud45c\ud558\uc600\ub2e4. \ucf00\uc774\ube14 \uc5c5\uacc4\ub294 \uc131\uba85\uc11c\ub97c \ud1b5\ud574, '\uc9c0\uc0c1\ud30c \uc7ac\uc1a1\uc2e0 \ubb38\uc81c\ub294 \ubc29\ud1b5\uc704\uac00 \uc81c\ub3c4 \uac1c\uc120\uc744 \ud1b5\ud574 \uadfc\ubcf8\uc801\uc778 \ub300\ucc45\uc744 \ub9c8\ub828\ud574\uc57c \ud588\uc74c\uc5d0\ub3c4 \ubd88\uad6c\ud558\uace0 \uc2dc\uc815\uba85\ub839\uc744 \ud1b5\ud574 \uc0ac\ud6c4\uc57d\ubc29\ubb38 \uc2dd\uc73c\ub85c \uc77c\uc744 \ud574\uacb0\ud558\ub824 \ud55c\ub2e4.' \uba70 \ubc29\uc1a1\ud1b5\uc2e0\uc704\uc6d0\ud68c\ub97c \ube44\ub09c\ud558\uc600\ub2e4. \ub610\ud55c \uc790\uc2e0\ub4e4\uc740 \ubc95\uc6d0\uc758 \ud310\uacb0\uc744 \uc874\uc911\ud558\uc5ec \uc9c0\uc0c1\ud30c \uc7ac\uc1a1\uc2e0\uc744 \uc911\ub2e8\ud558\uc600\ub294\ub370, \uc65c \ucf00\uc774\ube14 \ud154\ub808\ube44\uc804\uc5d0\ub9cc \uc81c\uc81c\ub97c \uac00\ud558\ub290\ub0d0\ub294 \ubd88\ub9cc\uc744 \ud45c\uc2dc\ud558\uc600\ub2e4. \uadf8\ub7ec\uba74\uc11c \uc774\ud589\uac15\uc81c\uae08\uc774 100\uc5b5 \uc6d0\uc744 \ub118\uc5b4\uac10\uc5d0 \ub530\ub77c \ubc95\uc801 \uc555\ubc15\uc744 \ud574 \uc628 \uc9c0\uc0c1\ud30c \ud154\ub808\ube44\uc804 \ubc29\uc1a1 3\uc0ac\uc640 \uc774\ub4e4\uc758 \ub208\uce58 \ubcf4\uae30\uc5d0\ub9cc \uae09\uae09\ud55c \ubc29\ud1b5\uc704\uc5d0 \uc774 \uc0ac\ud0dc\uc5d0 \ub300\ud55c \ucc45\uc784\uc744 \ubb3c\uc74c\uacfc \ub3d9\uc2dc\uc5d0, \uc9c0\uc0c1\ud30c \ud154\ub808\ube44\uc804 \ubc29\uc1a1 3\uc0ac\uc5d0\ub294 \uc804\ud5a5\uc801\uc778 \ud611\uc0c1 \ud0dc\ub3c4\uc640, \ubc29\uc1a1\ud1b5\uc2e0\uc704\uc6d0\ud68c\uc5d0\ub294 \uc7ac\uc1a1\uc2e0 \uc81c\ub3c4 \uac1c\uc120\uc744 \ucd09\uad6c\ud558\uc600\ub2e4."} {"question": "\ud3f4\uc740 2013\ub144 ESPN\uacfc \uc2a4\ud3ec\uce20 \uc77c\ub7ec\uc2a4\ud2b8\ub808\uc774\ud2f0\ub4dc\uac00 \uc120\uc815\ud55c \ub9ac\uadf8 \ucd5c\uc6b0\uc218 \uc120\uc218 \uba87\uc704\uc5d0 \uc62c\ub790\ub294\uac00?", "paragraph": "6\ud53c\ud2b8\uc5d0 175\ud30c\uc6b4\ub4dc\uc5d0 \ub2ec\ud558\ub294 \uc2e0\uc7a5\uacfc \uccb4\uc911\uc73c\ub85c, \ud3f4\uc740 \ud3ec\uc778\ud2b8\uac00\ub4dc\ub97c \uc804\ubb38\uc801\uc73c\ub85c \ub2f4\ub2f9\ud558\uace0 \uc788\ub2e4. 2014 NBA \ub9ac\ubdf0\uc5d0\uc11c ESPN\uc758 \ucf00\ud540 \ud3a0\ud2bc\uc740 \ud3f4\uc744 \ub9ac\uadf8 \ucd5c\uace0\uc758 \ud3ec\uc778\ud2b8\uac00\ub4dc\ub77c\uace0 \uadf9\ucc2c\ud588\ub2e4. \uadf8\uc758 \ucee4\ub9ac\uc5b4 \uae30\ub85d\uc740 \uacbd\uae30 \ud3c9\uade0 18.6\ub4dd\uc810 4.4\ub9ac\ubc14\uc6b4\ub4dc 9.9\uc5b4\uc2dc\uc2a4\ud2b8 2.4\uc2a4\ud2f8\uc774\ub2e4. \uadf8\ub294 \uc62c-NBA\ud300\uc5d0 5\ud68c (2009-09, 2011-13), \uc62c-\ub514\ud39c\uc2dc\ube0c\ud300\uc5d0 5\ud68c (2008-09, 2011-13) \uc120\uc815\ub418\uc5c8\uc73c\uba70, NBA \ucd5c\ub2e4 \uc2a4\ud2f8 6\ud68c (2008-09, 2011-14), \ucd5c\ub2e4 \uc5b4\uc2dc\uc2a4\ud2b8\ub97c 3\ud68c (2008-09, 2014) \uae30\ub85d\ud588\ub2e4. 2013\ub144, \ud3f4\uc740 ESPN\uacfc \u2018\u2019\uc2a4\ud3ec\uce20 \uc77c\ub7ec\uc2a4\ud2b8\ub808\uc774\ud2f0\ub4dc\u2019\u2019\uac00 \uc120\uc815\ud55c \ub9ac\uadf8 \ucd5c\uc6b0\uc218 \uc120\uc218\uc5d0\uc11c 3\uc704\uc5d0 \uc120\uc815\ub418\uc5c8\ub2e4.", "answer": "3\uc704", "sentence": "2013\ub144, \ud3f4\uc740 ESPN\uacfc \u2018\u2019\uc2a4\ud3ec\uce20 \uc77c\ub7ec\uc2a4\ud2b8\ub808\uc774\ud2f0\ub4dc\u2019\u2019\uac00 \uc120\uc815\ud55c \ub9ac\uadf8 \ucd5c\uc6b0\uc218 \uc120\uc218\uc5d0\uc11c 3\uc704 \uc5d0 \uc120\uc815\ub418\uc5c8\ub2e4.", "paragraph_sentence": "6\ud53c\ud2b8\uc5d0 175\ud30c\uc6b4\ub4dc\uc5d0 \ub2ec\ud558\ub294 \uc2e0\uc7a5\uacfc \uccb4\uc911\uc73c\ub85c, \ud3f4\uc740 \ud3ec\uc778\ud2b8\uac00\ub4dc\ub97c \uc804\ubb38\uc801\uc73c\ub85c \ub2f4\ub2f9\ud558\uace0 \uc788\ub2e4. 2014 NBA \ub9ac\ubdf0\uc5d0\uc11c ESPN\uc758 \ucf00\ud540 \ud3a0\ud2bc\uc740 \ud3f4\uc744 \ub9ac\uadf8 \ucd5c\uace0\uc758 \ud3ec\uc778\ud2b8\uac00\ub4dc\ub77c\uace0 \uadf9\ucc2c\ud588\ub2e4. \uadf8\uc758 \ucee4\ub9ac\uc5b4 \uae30\ub85d\uc740 \uacbd\uae30 \ud3c9\uade0 18.6\ub4dd\uc810 4.4\ub9ac\ubc14\uc6b4\ub4dc 9.9\uc5b4\uc2dc\uc2a4\ud2b8 2.4\uc2a4\ud2f8\uc774\ub2e4. \uadf8\ub294 \uc62c-NBA\ud300\uc5d0 5\ud68c (2009-09, 2011-13), \uc62c-\ub514\ud39c\uc2dc\ube0c\ud300\uc5d0 5\ud68c (2008-09, 2011-13) \uc120\uc815\ub418\uc5c8\uc73c\uba70, NBA \ucd5c\ub2e4 \uc2a4\ud2f8 6\ud68c (2008-09, 2011-14), \ucd5c\ub2e4 \uc5b4\uc2dc\uc2a4\ud2b8\ub97c 3\ud68c (2008-09, 2014) \uae30\ub85d\ud588\ub2e4. 2013\ub144, \ud3f4\uc740 ESPN\uacfc \u2018\u2019\uc2a4\ud3ec\uce20 \uc77c\ub7ec\uc2a4\ud2b8\ub808\uc774\ud2f0\ub4dc\u2019\u2019\uac00 \uc120\uc815\ud55c \ub9ac\uadf8 \ucd5c\uc6b0\uc218 \uc120\uc218\uc5d0\uc11c 3\uc704 \uc5d0 \uc120\uc815\ub418\uc5c8\ub2e4. ", "paragraph_answer": "6\ud53c\ud2b8\uc5d0 175\ud30c\uc6b4\ub4dc\uc5d0 \ub2ec\ud558\ub294 \uc2e0\uc7a5\uacfc \uccb4\uc911\uc73c\ub85c, \ud3f4\uc740 \ud3ec\uc778\ud2b8\uac00\ub4dc\ub97c \uc804\ubb38\uc801\uc73c\ub85c \ub2f4\ub2f9\ud558\uace0 \uc788\ub2e4. 2014 NBA \ub9ac\ubdf0\uc5d0\uc11c ESPN\uc758 \ucf00\ud540 \ud3a0\ud2bc\uc740 \ud3f4\uc744 \ub9ac\uadf8 \ucd5c\uace0\uc758 \ud3ec\uc778\ud2b8\uac00\ub4dc\ub77c\uace0 \uadf9\ucc2c\ud588\ub2e4. \uadf8\uc758 \ucee4\ub9ac\uc5b4 \uae30\ub85d\uc740 \uacbd\uae30 \ud3c9\uade0 18.6\ub4dd\uc810 4.4\ub9ac\ubc14\uc6b4\ub4dc 9.9\uc5b4\uc2dc\uc2a4\ud2b8 2.4\uc2a4\ud2f8\uc774\ub2e4. \uadf8\ub294 \uc62c-NBA\ud300\uc5d0 5\ud68c (2009-09, 2011-13), \uc62c-\ub514\ud39c\uc2dc\ube0c\ud300\uc5d0 5\ud68c (2008-09, 2011-13) \uc120\uc815\ub418\uc5c8\uc73c\uba70, NBA \ucd5c\ub2e4 \uc2a4\ud2f8 6\ud68c (2008-09, 2011-14), \ucd5c\ub2e4 \uc5b4\uc2dc\uc2a4\ud2b8\ub97c 3\ud68c (2008-09, 2014) \uae30\ub85d\ud588\ub2e4. 2013\ub144, \ud3f4\uc740 ESPN\uacfc \u2018\u2019\uc2a4\ud3ec\uce20 \uc77c\ub7ec\uc2a4\ud2b8\ub808\uc774\ud2f0\ub4dc\u2019\u2019\uac00 \uc120\uc815\ud55c \ub9ac\uadf8 \ucd5c\uc6b0\uc218 \uc120\uc218\uc5d0\uc11c 3\uc704 \uc5d0 \uc120\uc815\ub418\uc5c8\ub2e4.", "sentence_answer": "2013\ub144, \ud3f4\uc740 ESPN\uacfc \u2018\u2019\uc2a4\ud3ec\uce20 \uc77c\ub7ec\uc2a4\ud2b8\ub808\uc774\ud2f0\ub4dc\u2019\u2019\uac00 \uc120\uc815\ud55c \ub9ac\uadf8 \ucd5c\uc6b0\uc218 \uc120\uc218\uc5d0\uc11c 3\uc704 \uc5d0 \uc120\uc815\ub418\uc5c8\ub2e4.", "paragraph_id": "6212000-8-2", "paragraph_question": "question: \ud3f4\uc740 2013\ub144 ESPN\uacfc \uc2a4\ud3ec\uce20 \uc77c\ub7ec\uc2a4\ud2b8\ub808\uc774\ud2f0\ub4dc\uac00 \uc120\uc815\ud55c \ub9ac\uadf8 \ucd5c\uc6b0\uc218 \uc120\uc218 \uba87\uc704\uc5d0 \uc62c\ub790\ub294\uac00?, context: 6\ud53c\ud2b8\uc5d0 175\ud30c\uc6b4\ub4dc\uc5d0 \ub2ec\ud558\ub294 \uc2e0\uc7a5\uacfc \uccb4\uc911\uc73c\ub85c, \ud3f4\uc740 \ud3ec\uc778\ud2b8\uac00\ub4dc\ub97c \uc804\ubb38\uc801\uc73c\ub85c \ub2f4\ub2f9\ud558\uace0 \uc788\ub2e4. 2014 NBA \ub9ac\ubdf0\uc5d0\uc11c ESPN\uc758 \ucf00\ud540 \ud3a0\ud2bc\uc740 \ud3f4\uc744 \ub9ac\uadf8 \ucd5c\uace0\uc758 \ud3ec\uc778\ud2b8\uac00\ub4dc\ub77c\uace0 \uadf9\ucc2c\ud588\ub2e4. \uadf8\uc758 \ucee4\ub9ac\uc5b4 \uae30\ub85d\uc740 \uacbd\uae30 \ud3c9\uade0 18.6\ub4dd\uc810 4.4\ub9ac\ubc14\uc6b4\ub4dc 9.9\uc5b4\uc2dc\uc2a4\ud2b8 2.4\uc2a4\ud2f8\uc774\ub2e4. \uadf8\ub294 \uc62c-NBA\ud300\uc5d0 5\ud68c (2009-09, 2011-13), \uc62c-\ub514\ud39c\uc2dc\ube0c\ud300\uc5d0 5\ud68c (2008-09, 2011-13) \uc120\uc815\ub418\uc5c8\uc73c\uba70, NBA \ucd5c\ub2e4 \uc2a4\ud2f8 6\ud68c (2008-09, 2011-14), \ucd5c\ub2e4 \uc5b4\uc2dc\uc2a4\ud2b8\ub97c 3\ud68c (2008-09, 2014) \uae30\ub85d\ud588\ub2e4. 2013\ub144, \ud3f4\uc740 ESPN\uacfc \u2018\u2019\uc2a4\ud3ec\uce20 \uc77c\ub7ec\uc2a4\ud2b8\ub808\uc774\ud2f0\ub4dc\u2019\u2019\uac00 \uc120\uc815\ud55c \ub9ac\uadf8 \ucd5c\uc6b0\uc218 \uc120\uc218\uc5d0\uc11c 3\uc704\uc5d0 \uc120\uc815\ub418\uc5c8\ub2e4."} {"question": "\ubc30\uad00 \uc124\ube44\uc0ac \uae40\uc528\ub294 1\uce35\uc5d0 \uc788\ub294 \ud559\uc0dd \uba87\uc5ec\uba85\uc744 \uc704\uce35 \ub09c\uac04\uc73c\ub85c \uc62c\ub824\uc8fc\uc5c8\ub098?", "paragraph": "\ubc30\uad00 \uc124\ube44\uc0ac \uae40\u25cb\u25cb(59)\uc740 \uc81c\uc8fc\ub3c4\uc5d0 \uc788\ub294 \uac74\ucd95\ud68c\uc0ac\uc5d0 \uccab \ucd9c\uadfc\uc744 \ud558\ub358 \uc911 \uc0ac\uace0\ub97c \uacaa\uc5c8\ub2e4. \uadf8\ub294 \ubc30\uac00 \uae30\uc6b8\uc5b4\uc9c0\ub358 30\uc5ec\ubd84 \ub3d9\uc548 \uc8fc\ubcc0 \uc0ac\ub78c\ub4e4\uacfc \ud568\uaed8 \uc18c\ubc29\ud638\uc2a4\uc640 \ucee4\ud2bc\uc744 \ubb36\uc5b4 \uc120\ubc15 1\uce35\uc5d0 \uc788\ub358 \ud559\uc0dd 20\uc5ec \uba85\uc744 6~7m\uac00\ub7c9\uc758 \uc704\uce35 \ub09c\uac04\uc73c\ub85c \uc62c\ub824\uc8fc\uc5c8\uace0 \uadf8 \uc790\uc2e0\uc744 \ud3ec\ud568\ud558\uc5ec \uc774\ub4e4\uc740 \ubaa8\ub450 \uc0dd\ud658\ud558\uc600\ub2e4. \uadf8\ub294, \uadf8 \uc0ac\uc774\uc5d0 1\uce35\uc5d0\ub294 \ubb3c\uc774 \uac00\ub4dd \ucc28\ub294 \ubc14\ub78c\uc5d0 \ub354 \ub9ce\uc740 \ud559\uc0dd\ub4e4\uc744 \uad6c\ud558\uc9c0 \ubabb\ud574 \uc548\ud0c0\uae5d\ub2e4\uace0 \ubc1d\ud614\ub2e4. \ud654\ubb3c \uc6b4\uc804\uae30\uc0ac \uae40\u25cb\u25cb(50)\uc740 \uc0ac\uace0 \ub2f9\uc2dc \uc5ec\uac1d\uc120 3\uce35 \ud654\ubb3c\uc6b4\uc1a1 \uae30\uc0ac \uc804\uc6a9 \uac1d\uc2e4\uc5d0\uc11c \ub3d9\ub8cc \uae30\uc0ac\ub4e4\uacfc \ud568\uaed8 \uc788\uc5c8\ub2e4. \uadf8\ub294 \ubc30\uac00 \uae30\uc6b8\uc790 \uc77c\ubc18 \uac1d\uc2e4\uc774 \ubab0\ub9b0 4\uce35\uc73c\ub85c \uc774\ub3d9\ud574 \ud638\uc2a4\uc640 \ubc27\uc904\uc744 \ub358\uc838 10\uc5ec \uba85\uc758 \uc0ac\ub78c\ub4e4\uc744 \uad6c\ud558\uace0 \uc0dd\ud658\ud558\uc600\ub2e4. \ud2b9\ud788, \uadf8\uac00 \uad6c\ud55c \uc774\ub4e4 \uac00\uc6b4\ub370\ub294 \uac00\uc871\ub4e4\uacfc \ud568\uaed8 \uc81c\uc8fc\ub3c4\ub85c \uc774\uc0ac\ub97c \uac00\ub2e4\uac00 \uc0ac\uace0\ub97c \ub2f9\ud55c \uc5b4\ub9b0\uc774 \uad8c\uc9c0\uc5f0(6, \uc5ec)\ub3c4 \ud3ec\ud568\ub418\uc5c8\ub358 \uac83\uc73c\ub85c \uc54c\ub824\uc838 \uc788\ub2e4.", "answer": "20\uc5ec \uba85", "sentence": "\uadf8\ub294 \ubc30\uac00 \uae30\uc6b8\uc5b4\uc9c0\ub358 30\uc5ec\ubd84 \ub3d9\uc548 \uc8fc\ubcc0 \uc0ac\ub78c\ub4e4\uacfc \ud568\uaed8 \uc18c\ubc29\ud638\uc2a4\uc640 \ucee4\ud2bc\uc744 \ubb36\uc5b4 \uc120\ubc15 1\uce35\uc5d0 \uc788\ub358 \ud559\uc0dd 20\uc5ec \uba85 \uc744 6~7m\uac00\ub7c9\uc758 \uc704\uce35 \ub09c\uac04\uc73c\ub85c \uc62c\ub824\uc8fc\uc5c8\uace0 \uadf8 \uc790\uc2e0\uc744 \ud3ec\ud568\ud558\uc5ec \uc774\ub4e4\uc740 \ubaa8\ub450 \uc0dd\ud658\ud558\uc600\ub2e4.", "paragraph_sentence": "\ubc30\uad00 \uc124\ube44\uc0ac \uae40\u25cb\u25cb(59)\uc740 \uc81c\uc8fc\ub3c4\uc5d0 \uc788\ub294 \uac74\ucd95\ud68c\uc0ac\uc5d0 \uccab \ucd9c\uadfc\uc744 \ud558\ub358 \uc911 \uc0ac\uace0\ub97c \uacaa\uc5c8\ub2e4. \uadf8\ub294 \ubc30\uac00 \uae30\uc6b8\uc5b4\uc9c0\ub358 30\uc5ec\ubd84 \ub3d9\uc548 \uc8fc\ubcc0 \uc0ac\ub78c\ub4e4\uacfc \ud568\uaed8 \uc18c\ubc29\ud638\uc2a4\uc640 \ucee4\ud2bc\uc744 \ubb36\uc5b4 \uc120\ubc15 1\uce35\uc5d0 \uc788\ub358 \ud559\uc0dd 20\uc5ec \uba85 \uc744 6~7m\uac00\ub7c9\uc758 \uc704\uce35 \ub09c\uac04\uc73c\ub85c \uc62c\ub824\uc8fc\uc5c8\uace0 \uadf8 \uc790\uc2e0\uc744 \ud3ec\ud568\ud558\uc5ec \uc774\ub4e4\uc740 \ubaa8\ub450 \uc0dd\ud658\ud558\uc600\ub2e4. \uadf8\ub294, \uadf8 \uc0ac\uc774\uc5d0 1\uce35\uc5d0\ub294 \ubb3c\uc774 \uac00\ub4dd \ucc28\ub294 \ubc14\ub78c\uc5d0 \ub354 \ub9ce\uc740 \ud559\uc0dd\ub4e4\uc744 \uad6c\ud558\uc9c0 \ubabb\ud574 \uc548\ud0c0\uae5d\ub2e4\uace0 \ubc1d\ud614\ub2e4. \ud654\ubb3c \uc6b4\uc804\uae30\uc0ac \uae40\u25cb\u25cb(50)\uc740 \uc0ac\uace0 \ub2f9\uc2dc \uc5ec\uac1d\uc120 3\uce35 \ud654\ubb3c\uc6b4\uc1a1 \uae30\uc0ac \uc804\uc6a9 \uac1d\uc2e4\uc5d0\uc11c \ub3d9\ub8cc \uae30\uc0ac\ub4e4\uacfc \ud568\uaed8 \uc788\uc5c8\ub2e4. \uadf8\ub294 \ubc30\uac00 \uae30\uc6b8\uc790 \uc77c\ubc18 \uac1d\uc2e4\uc774 \ubab0\ub9b0 4\uce35\uc73c\ub85c \uc774\ub3d9\ud574 \ud638\uc2a4\uc640 \ubc27\uc904\uc744 \ub358\uc838 10\uc5ec \uba85\uc758 \uc0ac\ub78c\ub4e4\uc744 \uad6c\ud558\uace0 \uc0dd\ud658\ud558\uc600\ub2e4. \ud2b9\ud788, \uadf8\uac00 \uad6c\ud55c \uc774\ub4e4 \uac00\uc6b4\ub370\ub294 \uac00\uc871\ub4e4\uacfc \ud568\uaed8 \uc81c\uc8fc\ub3c4\ub85c \uc774\uc0ac\ub97c \uac00\ub2e4\uac00 \uc0ac\uace0\ub97c \ub2f9\ud55c \uc5b4\ub9b0\uc774 \uad8c\uc9c0\uc5f0(6, \uc5ec)\ub3c4 \ud3ec\ud568\ub418\uc5c8\ub358 \uac83\uc73c\ub85c \uc54c\ub824\uc838 \uc788\ub2e4.", "paragraph_answer": "\ubc30\uad00 \uc124\ube44\uc0ac \uae40\u25cb\u25cb(59)\uc740 \uc81c\uc8fc\ub3c4\uc5d0 \uc788\ub294 \uac74\ucd95\ud68c\uc0ac\uc5d0 \uccab \ucd9c\uadfc\uc744 \ud558\ub358 \uc911 \uc0ac\uace0\ub97c \uacaa\uc5c8\ub2e4. \uadf8\ub294 \ubc30\uac00 \uae30\uc6b8\uc5b4\uc9c0\ub358 30\uc5ec\ubd84 \ub3d9\uc548 \uc8fc\ubcc0 \uc0ac\ub78c\ub4e4\uacfc \ud568\uaed8 \uc18c\ubc29\ud638\uc2a4\uc640 \ucee4\ud2bc\uc744 \ubb36\uc5b4 \uc120\ubc15 1\uce35\uc5d0 \uc788\ub358 \ud559\uc0dd 20\uc5ec \uba85 \uc744 6~7m\uac00\ub7c9\uc758 \uc704\uce35 \ub09c\uac04\uc73c\ub85c \uc62c\ub824\uc8fc\uc5c8\uace0 \uadf8 \uc790\uc2e0\uc744 \ud3ec\ud568\ud558\uc5ec \uc774\ub4e4\uc740 \ubaa8\ub450 \uc0dd\ud658\ud558\uc600\ub2e4. \uadf8\ub294, \uadf8 \uc0ac\uc774\uc5d0 1\uce35\uc5d0\ub294 \ubb3c\uc774 \uac00\ub4dd \ucc28\ub294 \ubc14\ub78c\uc5d0 \ub354 \ub9ce\uc740 \ud559\uc0dd\ub4e4\uc744 \uad6c\ud558\uc9c0 \ubabb\ud574 \uc548\ud0c0\uae5d\ub2e4\uace0 \ubc1d\ud614\ub2e4. \ud654\ubb3c \uc6b4\uc804\uae30\uc0ac \uae40\u25cb\u25cb(50)\uc740 \uc0ac\uace0 \ub2f9\uc2dc \uc5ec\uac1d\uc120 3\uce35 \ud654\ubb3c\uc6b4\uc1a1 \uae30\uc0ac \uc804\uc6a9 \uac1d\uc2e4\uc5d0\uc11c \ub3d9\ub8cc \uae30\uc0ac\ub4e4\uacfc \ud568\uaed8 \uc788\uc5c8\ub2e4. \uadf8\ub294 \ubc30\uac00 \uae30\uc6b8\uc790 \uc77c\ubc18 \uac1d\uc2e4\uc774 \ubab0\ub9b0 4\uce35\uc73c\ub85c \uc774\ub3d9\ud574 \ud638\uc2a4\uc640 \ubc27\uc904\uc744 \ub358\uc838 10\uc5ec \uba85\uc758 \uc0ac\ub78c\ub4e4\uc744 \uad6c\ud558\uace0 \uc0dd\ud658\ud558\uc600\ub2e4. \ud2b9\ud788, \uadf8\uac00 \uad6c\ud55c \uc774\ub4e4 \uac00\uc6b4\ub370\ub294 \uac00\uc871\ub4e4\uacfc \ud568\uaed8 \uc81c\uc8fc\ub3c4\ub85c \uc774\uc0ac\ub97c \uac00\ub2e4\uac00 \uc0ac\uace0\ub97c \ub2f9\ud55c \uc5b4\ub9b0\uc774 \uad8c\uc9c0\uc5f0(6, \uc5ec)\ub3c4 \ud3ec\ud568\ub418\uc5c8\ub358 \uac83\uc73c\ub85c \uc54c\ub824\uc838 \uc788\ub2e4.", "sentence_answer": "\uadf8\ub294 \ubc30\uac00 \uae30\uc6b8\uc5b4\uc9c0\ub358 30\uc5ec\ubd84 \ub3d9\uc548 \uc8fc\ubcc0 \uc0ac\ub78c\ub4e4\uacfc \ud568\uaed8 \uc18c\ubc29\ud638\uc2a4\uc640 \ucee4\ud2bc\uc744 \ubb36\uc5b4 \uc120\ubc15 1\uce35\uc5d0 \uc788\ub358 \ud559\uc0dd 20\uc5ec \uba85 \uc744 6~7m\uac00\ub7c9\uc758 \uc704\uce35 \ub09c\uac04\uc73c\ub85c \uc62c\ub824\uc8fc\uc5c8\uace0 \uadf8 \uc790\uc2e0\uc744 \ud3ec\ud568\ud558\uc5ec \uc774\ub4e4\uc740 \ubaa8\ub450 \uc0dd\ud658\ud558\uc600\ub2e4.", "paragraph_id": "6657196-24-1", "paragraph_question": "question: \ubc30\uad00 \uc124\ube44\uc0ac \uae40\uc528\ub294 1\uce35\uc5d0 \uc788\ub294 \ud559\uc0dd \uba87\uc5ec\uba85\uc744 \uc704\uce35 \ub09c\uac04\uc73c\ub85c \uc62c\ub824\uc8fc\uc5c8\ub098?, context: \ubc30\uad00 \uc124\ube44\uc0ac \uae40\u25cb\u25cb(59)\uc740 \uc81c\uc8fc\ub3c4\uc5d0 \uc788\ub294 \uac74\ucd95\ud68c\uc0ac\uc5d0 \uccab \ucd9c\uadfc\uc744 \ud558\ub358 \uc911 \uc0ac\uace0\ub97c \uacaa\uc5c8\ub2e4. \uadf8\ub294 \ubc30\uac00 \uae30\uc6b8\uc5b4\uc9c0\ub358 30\uc5ec\ubd84 \ub3d9\uc548 \uc8fc\ubcc0 \uc0ac\ub78c\ub4e4\uacfc \ud568\uaed8 \uc18c\ubc29\ud638\uc2a4\uc640 \ucee4\ud2bc\uc744 \ubb36\uc5b4 \uc120\ubc15 1\uce35\uc5d0 \uc788\ub358 \ud559\uc0dd 20\uc5ec \uba85\uc744 6~7m\uac00\ub7c9\uc758 \uc704\uce35 \ub09c\uac04\uc73c\ub85c \uc62c\ub824\uc8fc\uc5c8\uace0 \uadf8 \uc790\uc2e0\uc744 \ud3ec\ud568\ud558\uc5ec \uc774\ub4e4\uc740 \ubaa8\ub450 \uc0dd\ud658\ud558\uc600\ub2e4. \uadf8\ub294, \uadf8 \uc0ac\uc774\uc5d0 1\uce35\uc5d0\ub294 \ubb3c\uc774 \uac00\ub4dd \ucc28\ub294 \ubc14\ub78c\uc5d0 \ub354 \ub9ce\uc740 \ud559\uc0dd\ub4e4\uc744 \uad6c\ud558\uc9c0 \ubabb\ud574 \uc548\ud0c0\uae5d\ub2e4\uace0 \ubc1d\ud614\ub2e4. \ud654\ubb3c \uc6b4\uc804\uae30\uc0ac \uae40\u25cb\u25cb(50)\uc740 \uc0ac\uace0 \ub2f9\uc2dc \uc5ec\uac1d\uc120 3\uce35 \ud654\ubb3c\uc6b4\uc1a1 \uae30\uc0ac \uc804\uc6a9 \uac1d\uc2e4\uc5d0\uc11c \ub3d9\ub8cc \uae30\uc0ac\ub4e4\uacfc \ud568\uaed8 \uc788\uc5c8\ub2e4. \uadf8\ub294 \ubc30\uac00 \uae30\uc6b8\uc790 \uc77c\ubc18 \uac1d\uc2e4\uc774 \ubab0\ub9b0 4\uce35\uc73c\ub85c \uc774\ub3d9\ud574 \ud638\uc2a4\uc640 \ubc27\uc904\uc744 \ub358\uc838 10\uc5ec \uba85\uc758 \uc0ac\ub78c\ub4e4\uc744 \uad6c\ud558\uace0 \uc0dd\ud658\ud558\uc600\ub2e4. \ud2b9\ud788, \uadf8\uac00 \uad6c\ud55c \uc774\ub4e4 \uac00\uc6b4\ub370\ub294 \uac00\uc871\ub4e4\uacfc \ud568\uaed8 \uc81c\uc8fc\ub3c4\ub85c \uc774\uc0ac\ub97c \uac00\ub2e4\uac00 \uc0ac\uace0\ub97c \ub2f9\ud55c \uc5b4\ub9b0\uc774 \uad8c\uc9c0\uc5f0(6, \uc5ec)\ub3c4 \ud3ec\ud568\ub418\uc5c8\ub358 \uac83\uc73c\ub85c \uc54c\ub824\uc838 \uc788\ub2e4."} {"question": "\uc640\ud2b8 \ud0c0\uc77c\ub7ec\uc758 \ub09c\uc744 \uc77c\uc73c\ud0a8 \uc138\ub825\uc5d0\uc11c \uc0ac\ud68c \uadf9\ube48\uce35\uc774 \ucc28\uc9c0\ud558\ub294 \ube44\uc728\uc740 \uc5b4\ub290\uc815\ub3c4\uc600\ub2e4\uace0 \ud558\ub294\uac00?", "paragraph": "\uc804\uc6d0\uc9c0\ub300 \ub09c\ubbfc\ub4e4\uc758 \ubc30\uacbd\uc740 \uc81c\uac01\uae30 \ub2e4\uc591\ud558\ub098, \uc77c\ubc18\uc801\uc73c\ub85c \uadf8\ub4e4\uc740, \uc0ac\ud559\uc790 \ud06c\ub9ac\uc2a4\ud1a0\ud37c \ub2e4\uc774\uc5b4(Christopher Dyer)\uc758 \uc124\uba85\ub300\ub85c, \u201c\uc2e0\uc0ac\uce35 \uc774\ud558 \uacc4\uae09\uc758 \ubbfc\uc911\uc774\uc5c8\uc73c\ub098, \ub300\uac1c \uc5b4\ub290 \uc815\ub3c4\uc758 \ud1a0\uc9c0\uc640 \uc7ac\ud654\ub97c \uc18c\uc720\ud558\uace0 \uc788\ub294\u201d \uc790\ub4e4\uc774\uc5c8\uc9c0, \uc0ac\ud68c \uadf9\ube48 \uacc4\uce35\uc774 \uc544\ub2c8\uc5c8\ub2e4. \uc624\ud788\ub824 \uadf9\ube48\uce35\ub4e4\uc740 \ubc18\ub780 \uad6c\uc131\uc6d0\ub4e4 \uc911 \uc18c\uc218\uc5d0 \ubd88\uacfc\ud588\ub2e4. \ub9ce\uc740 \ub09c\ubbfc\ub4e4\uc740 \uc9c0\uc5ed \ucd0c\ub77d \ud589\uc815\ub2f9\uad6d\uc5d0\uc11c \uc5b4\ub290 \uc815\ub3c4 \uc9c0\uc704\ub97c \uac00\uc9c0\uace0 \uc788\uc5c8\uc73c\uba70, \ub610 \uadf8\ub7ac\uae30 \ub54c\ubb38\uc5d0 \ubc18\ub780 \uacfc\uc815\uc5d0\uc11c \ub9ac\ub354\uc2ed\uc744 \ubc1c\ud718\ud560 \uc218 \uc788\uc5c8\ub358 \uac83\uc73c\ub85c \ubcf4\uc778\ub2e4. \ub09c\ubbfc\ub4e4 \uc911 \uc77c\ubd80\ub294 \uc7a5\uc778\ub4e4\uc774\uc5c8\uace0, \ub85c\ub4dc\ub2c8 \ud790\ud2bc\uc758 \ub098\uc5f4\uc5d0 \ub530\ub974\uba74 \u201c\ubaa9\uc218, \ud1b1\uc9c8\uafbc, \uc11d\uacf5, \uc2e0\uae30\ub8cc, \uc7ac\ub2e8\uc0ac, \ubc29\uc9c1\uacf5, \ucd95\uc735\uacf5, \uc7a5\uac11\uacf5, \uc591\ub9d0\uacf5, \ud53c\ud601\uc0c1, \uc81c\ube75\uc0ac, \ud478\uc8fc\ud55c, \uc5ec\uad00\uc8fc, \uc694\ub9ac\uc0ac, \uc11d\ud68c\uacf5\u201d \ub4f1 \ub2e4\uc591\ud588\ub2e4. \ub09c\ubbfc\ub4e4\uc740 \ub300\ubd80\ubd84 \ub0a8\uc131\uc774\uc5c8\uc73c\ub098, \uac19\uc740 \uacc4\uae09\uc758 \uc5ec\uc131\ub4e4\ub3c4 \uc77c\ubd80 \uc788\uc5c8\ub2e4. \ub09c\ubbfc\ub4e4\uc740 \ub300\uac1c \ubb38\ub9f9\uc774\uc5c8\ub294\ub370, \ub2f9\uc2dc \uc789\uae00\ub79c\ub4dc \uc804 \uc778\uad6c\uc758 5 ~ 15%\ub9cc\uc774 \uae00\uc744 \uc77d\uc744 \uc218 \uc788\uc5c8\ub2e4. \ub610\ud55c \uadf8\ub4e4\uc740 \uad11\ubc94\uc704\ud55c \uc9c0\uc5ed \uacf5\ub3d9\uccb4\ub85c\ubd80\ud130 \ubaa8\uc5ec\ub4e4\uc5c8\ub2e4. \ub3d9\ub0a8\ubd80\ub9cc \ud574\ub3c4 \ub09c\ubbfc\ub4e4\uc774 \ubc1c\uc0dd\ud55c \ucd0c\ub77d\uc740 \ucd5c\uc18c 330\uac1c\uc5d0 \uc774\ub974\ub800\ub2e4.", "answer": "\uc18c\uc218", "sentence": "\ubc18\ub780 \uad6c\uc131\uc6d0\ub4e4 \uc911 \uc18c\uc218 \uc5d0 \ubd88\uacfc\ud588\ub2e4.", "paragraph_sentence": "\uc804\uc6d0\uc9c0\ub300 \ub09c\ubbfc\ub4e4\uc758 \ubc30\uacbd\uc740 \uc81c\uac01\uae30 \ub2e4\uc591\ud558\ub098, \uc77c\ubc18\uc801\uc73c\ub85c \uadf8\ub4e4\uc740, \uc0ac\ud559\uc790 \ud06c\ub9ac\uc2a4\ud1a0\ud37c \ub2e4\uc774\uc5b4(Christopher Dyer)\uc758 \uc124\uba85\ub300\ub85c, \u201c\uc2e0\uc0ac\uce35 \uc774\ud558 \uacc4\uae09\uc758 \ubbfc\uc911\uc774\uc5c8\uc73c\ub098, \ub300\uac1c \uc5b4\ub290 \uc815\ub3c4\uc758 \ud1a0\uc9c0\uc640 \uc7ac\ud654\ub97c \uc18c\uc720\ud558\uace0 \uc788\ub294\u201d \uc790\ub4e4\uc774\uc5c8\uc9c0, \uc0ac\ud68c \uadf9\ube48 \uacc4\uce35\uc774 \uc544\ub2c8\uc5c8\ub2e4. \uc624\ud788\ub824 \uadf9\ube48\uce35\ub4e4\uc740 \ubc18\ub780 \uad6c\uc131\uc6d0\ub4e4 \uc911 \uc18c\uc218 \uc5d0 \ubd88\uacfc\ud588\ub2e4. \ub9ce\uc740 \ub09c\ubbfc\ub4e4\uc740 \uc9c0\uc5ed \ucd0c\ub77d \ud589\uc815\ub2f9\uad6d\uc5d0\uc11c \uc5b4\ub290 \uc815\ub3c4 \uc9c0\uc704\ub97c \uac00\uc9c0\uace0 \uc788\uc5c8\uc73c\uba70, \ub610 \uadf8\ub7ac\uae30 \ub54c\ubb38\uc5d0 \ubc18\ub780 \uacfc\uc815\uc5d0\uc11c \ub9ac\ub354\uc2ed\uc744 \ubc1c\ud718\ud560 \uc218 \uc788\uc5c8\ub358 \uac83\uc73c\ub85c \ubcf4\uc778\ub2e4. \ub09c\ubbfc\ub4e4 \uc911 \uc77c\ubd80\ub294 \uc7a5\uc778\ub4e4\uc774\uc5c8\uace0, \ub85c\ub4dc\ub2c8 \ud790\ud2bc\uc758 \ub098\uc5f4\uc5d0 \ub530\ub974\uba74 \u201c\ubaa9\uc218, \ud1b1\uc9c8\uafbc, \uc11d\uacf5, \uc2e0\uae30\ub8cc, \uc7ac\ub2e8\uc0ac, \ubc29\uc9c1\uacf5, \ucd95\uc735\uacf5, \uc7a5\uac11\uacf5, \uc591\ub9d0\uacf5, \ud53c\ud601\uc0c1, \uc81c\ube75\uc0ac, \ud478\uc8fc\ud55c, \uc5ec\uad00\uc8fc, \uc694\ub9ac\uc0ac, \uc11d\ud68c\uacf5\u201d \ub4f1 \ub2e4\uc591\ud588\ub2e4. \ub09c\ubbfc\ub4e4\uc740 \ub300\ubd80\ubd84 \ub0a8\uc131\uc774\uc5c8\uc73c\ub098, \uac19\uc740 \uacc4\uae09\uc758 \uc5ec\uc131\ub4e4\ub3c4 \uc77c\ubd80 \uc788\uc5c8\ub2e4. \ub09c\ubbfc\ub4e4\uc740 \ub300\uac1c \ubb38\ub9f9\uc774\uc5c8\ub294\ub370, \ub2f9\uc2dc \uc789\uae00\ub79c\ub4dc \uc804 \uc778\uad6c\uc758 5 ~ 15%\ub9cc\uc774 \uae00\uc744 \uc77d\uc744 \uc218 \uc788\uc5c8\ub2e4. \ub610\ud55c \uadf8\ub4e4\uc740 \uad11\ubc94\uc704\ud55c \uc9c0\uc5ed \uacf5\ub3d9\uccb4\ub85c\ubd80\ud130 \ubaa8\uc5ec\ub4e4\uc5c8\ub2e4. \ub3d9\ub0a8\ubd80\ub9cc \ud574\ub3c4 \ub09c\ubbfc\ub4e4\uc774 \ubc1c\uc0dd\ud55c \ucd0c\ub77d\uc740 \ucd5c\uc18c 330\uac1c\uc5d0 \uc774\ub974\ub800\ub2e4.", "paragraph_answer": "\uc804\uc6d0\uc9c0\ub300 \ub09c\ubbfc\ub4e4\uc758 \ubc30\uacbd\uc740 \uc81c\uac01\uae30 \ub2e4\uc591\ud558\ub098, \uc77c\ubc18\uc801\uc73c\ub85c \uadf8\ub4e4\uc740, \uc0ac\ud559\uc790 \ud06c\ub9ac\uc2a4\ud1a0\ud37c \ub2e4\uc774\uc5b4(Christopher Dyer)\uc758 \uc124\uba85\ub300\ub85c, \u201c\uc2e0\uc0ac\uce35 \uc774\ud558 \uacc4\uae09\uc758 \ubbfc\uc911\uc774\uc5c8\uc73c\ub098, \ub300\uac1c \uc5b4\ub290 \uc815\ub3c4\uc758 \ud1a0\uc9c0\uc640 \uc7ac\ud654\ub97c \uc18c\uc720\ud558\uace0 \uc788\ub294\u201d \uc790\ub4e4\uc774\uc5c8\uc9c0, \uc0ac\ud68c \uadf9\ube48 \uacc4\uce35\uc774 \uc544\ub2c8\uc5c8\ub2e4. \uc624\ud788\ub824 \uadf9\ube48\uce35\ub4e4\uc740 \ubc18\ub780 \uad6c\uc131\uc6d0\ub4e4 \uc911 \uc18c\uc218 \uc5d0 \ubd88\uacfc\ud588\ub2e4. \ub9ce\uc740 \ub09c\ubbfc\ub4e4\uc740 \uc9c0\uc5ed \ucd0c\ub77d \ud589\uc815\ub2f9\uad6d\uc5d0\uc11c \uc5b4\ub290 \uc815\ub3c4 \uc9c0\uc704\ub97c \uac00\uc9c0\uace0 \uc788\uc5c8\uc73c\uba70, \ub610 \uadf8\ub7ac\uae30 \ub54c\ubb38\uc5d0 \ubc18\ub780 \uacfc\uc815\uc5d0\uc11c \ub9ac\ub354\uc2ed\uc744 \ubc1c\ud718\ud560 \uc218 \uc788\uc5c8\ub358 \uac83\uc73c\ub85c \ubcf4\uc778\ub2e4. \ub09c\ubbfc\ub4e4 \uc911 \uc77c\ubd80\ub294 \uc7a5\uc778\ub4e4\uc774\uc5c8\uace0, \ub85c\ub4dc\ub2c8 \ud790\ud2bc\uc758 \ub098\uc5f4\uc5d0 \ub530\ub974\uba74 \u201c\ubaa9\uc218, \ud1b1\uc9c8\uafbc, \uc11d\uacf5, \uc2e0\uae30\ub8cc, \uc7ac\ub2e8\uc0ac, \ubc29\uc9c1\uacf5, \ucd95\uc735\uacf5, \uc7a5\uac11\uacf5, \uc591\ub9d0\uacf5, \ud53c\ud601\uc0c1, \uc81c\ube75\uc0ac, \ud478\uc8fc\ud55c, \uc5ec\uad00\uc8fc, \uc694\ub9ac\uc0ac, \uc11d\ud68c\uacf5\u201d \ub4f1 \ub2e4\uc591\ud588\ub2e4. \ub09c\ubbfc\ub4e4\uc740 \ub300\ubd80\ubd84 \ub0a8\uc131\uc774\uc5c8\uc73c\ub098, \uac19\uc740 \uacc4\uae09\uc758 \uc5ec\uc131\ub4e4\ub3c4 \uc77c\ubd80 \uc788\uc5c8\ub2e4. \ub09c\ubbfc\ub4e4\uc740 \ub300\uac1c \ubb38\ub9f9\uc774\uc5c8\ub294\ub370, \ub2f9\uc2dc \uc789\uae00\ub79c\ub4dc \uc804 \uc778\uad6c\uc758 5 ~ 15%\ub9cc\uc774 \uae00\uc744 \uc77d\uc744 \uc218 \uc788\uc5c8\ub2e4. \ub610\ud55c \uadf8\ub4e4\uc740 \uad11\ubc94\uc704\ud55c \uc9c0\uc5ed \uacf5\ub3d9\uccb4\ub85c\ubd80\ud130 \ubaa8\uc5ec\ub4e4\uc5c8\ub2e4. \ub3d9\ub0a8\ubd80\ub9cc \ud574\ub3c4 \ub09c\ubbfc\ub4e4\uc774 \ubc1c\uc0dd\ud55c \ucd0c\ub77d\uc740 \ucd5c\uc18c 330\uac1c\uc5d0 \uc774\ub974\ub800\ub2e4.", "sentence_answer": "\ubc18\ub780 \uad6c\uc131\uc6d0\ub4e4 \uc911 \uc18c\uc218 \uc5d0 \ubd88\uacfc\ud588\ub2e4.", "paragraph_id": "6541489-34-0", "paragraph_question": "question: \uc640\ud2b8 \ud0c0\uc77c\ub7ec\uc758 \ub09c\uc744 \uc77c\uc73c\ud0a8 \uc138\ub825\uc5d0\uc11c \uc0ac\ud68c \uadf9\ube48\uce35\uc774 \ucc28\uc9c0\ud558\ub294 \ube44\uc728\uc740 \uc5b4\ub290\uc815\ub3c4\uc600\ub2e4\uace0 \ud558\ub294\uac00?, context: \uc804\uc6d0\uc9c0\ub300 \ub09c\ubbfc\ub4e4\uc758 \ubc30\uacbd\uc740 \uc81c\uac01\uae30 \ub2e4\uc591\ud558\ub098, \uc77c\ubc18\uc801\uc73c\ub85c \uadf8\ub4e4\uc740, \uc0ac\ud559\uc790 \ud06c\ub9ac\uc2a4\ud1a0\ud37c \ub2e4\uc774\uc5b4(Christopher Dyer)\uc758 \uc124\uba85\ub300\ub85c, \u201c\uc2e0\uc0ac\uce35 \uc774\ud558 \uacc4\uae09\uc758 \ubbfc\uc911\uc774\uc5c8\uc73c\ub098, \ub300\uac1c \uc5b4\ub290 \uc815\ub3c4\uc758 \ud1a0\uc9c0\uc640 \uc7ac\ud654\ub97c \uc18c\uc720\ud558\uace0 \uc788\ub294\u201d \uc790\ub4e4\uc774\uc5c8\uc9c0, \uc0ac\ud68c \uadf9\ube48 \uacc4\uce35\uc774 \uc544\ub2c8\uc5c8\ub2e4. \uc624\ud788\ub824 \uadf9\ube48\uce35\ub4e4\uc740 \ubc18\ub780 \uad6c\uc131\uc6d0\ub4e4 \uc911 \uc18c\uc218\uc5d0 \ubd88\uacfc\ud588\ub2e4. \ub9ce\uc740 \ub09c\ubbfc\ub4e4\uc740 \uc9c0\uc5ed \ucd0c\ub77d \ud589\uc815\ub2f9\uad6d\uc5d0\uc11c \uc5b4\ub290 \uc815\ub3c4 \uc9c0\uc704\ub97c \uac00\uc9c0\uace0 \uc788\uc5c8\uc73c\uba70, \ub610 \uadf8\ub7ac\uae30 \ub54c\ubb38\uc5d0 \ubc18\ub780 \uacfc\uc815\uc5d0\uc11c \ub9ac\ub354\uc2ed\uc744 \ubc1c\ud718\ud560 \uc218 \uc788\uc5c8\ub358 \uac83\uc73c\ub85c \ubcf4\uc778\ub2e4. \ub09c\ubbfc\ub4e4 \uc911 \uc77c\ubd80\ub294 \uc7a5\uc778\ub4e4\uc774\uc5c8\uace0, \ub85c\ub4dc\ub2c8 \ud790\ud2bc\uc758 \ub098\uc5f4\uc5d0 \ub530\ub974\uba74 \u201c\ubaa9\uc218, \ud1b1\uc9c8\uafbc, \uc11d\uacf5, \uc2e0\uae30\ub8cc, \uc7ac\ub2e8\uc0ac, \ubc29\uc9c1\uacf5, \ucd95\uc735\uacf5, \uc7a5\uac11\uacf5, \uc591\ub9d0\uacf5, \ud53c\ud601\uc0c1, \uc81c\ube75\uc0ac, \ud478\uc8fc\ud55c, \uc5ec\uad00\uc8fc, \uc694\ub9ac\uc0ac, \uc11d\ud68c\uacf5\u201d \ub4f1 \ub2e4\uc591\ud588\ub2e4. \ub09c\ubbfc\ub4e4\uc740 \ub300\ubd80\ubd84 \ub0a8\uc131\uc774\uc5c8\uc73c\ub098, \uac19\uc740 \uacc4\uae09\uc758 \uc5ec\uc131\ub4e4\ub3c4 \uc77c\ubd80 \uc788\uc5c8\ub2e4. \ub09c\ubbfc\ub4e4\uc740 \ub300\uac1c \ubb38\ub9f9\uc774\uc5c8\ub294\ub370, \ub2f9\uc2dc \uc789\uae00\ub79c\ub4dc \uc804 \uc778\uad6c\uc758 5 ~ 15%\ub9cc\uc774 \uae00\uc744 \uc77d\uc744 \uc218 \uc788\uc5c8\ub2e4. \ub610\ud55c \uadf8\ub4e4\uc740 \uad11\ubc94\uc704\ud55c \uc9c0\uc5ed \uacf5\ub3d9\uccb4\ub85c\ubd80\ud130 \ubaa8\uc5ec\ub4e4\uc5c8\ub2e4. \ub3d9\ub0a8\ubd80\ub9cc \ud574\ub3c4 \ub09c\ubbfc\ub4e4\uc774 \ubc1c\uc0dd\ud55c \ucd0c\ub77d\uc740 \ucd5c\uc18c 330\uac1c\uc5d0 \uc774\ub974\ub800\ub2e4."} {"question": "\uc2a4\ud0c0\ub514\uc6c0\uc758 \ucd9c\uc785\ubb38\uc774 \uc5f4\ub9b0 \uc2dc\uac04\uc740?", "paragraph": "\ucd9c\uc785\ubb38\uc740 \uc54c\ub824\uc9c4 \ub300\ub85c \uacbd\uae30 \uc2dc\uc791 \uc2dc\uac04 3\uc2dc\uac04 30\ubd84 \uc804\uc778 \uc624\uc80411\uc2dc 30\ubd84\uc5d0 \uc5f4\ub838\uace0 \uc624\ud6c41\uc2dc\uae4c\uc9c0 \ubab0\ub824\ub4e0 \uad00\uc911\ub4e4\uc740 \uc9c8\uc11c\uc788\uac8c \uacbd\uae30\uc7a5\uc5d0 \uc785\uc7a5\ud588\ub2e4. \uadf8\ub7ec\ub098 1\uc2dc\uae4c\uc9c0 \ubab0\ub824\ub4e0 \uad00\uc911\ub4e4\uc774 \uac10\ub2f9\ud558\uc9c0 \ubabb\ud560 \uc815\ub3c4\ub85c \ubd88\uc5b4\ub098\uc790 \uacbd\uae30\uc7a5 \uad00\ub9ac\uce21\uc758 \uc870\uc0ac \ud6c4\uc5d0 \uc624\ud6c4 1\uc2dc 45\ubd84\uc5d0 \ucd9c\uc785\uad6c\ub97c \ub2eb\uace0 \uad00\uc911\uc758 \uc785\uc7a5\uc744 \ub9c9\uc558\ub2e4. \uad00\uc911\uc774\uc5c8\ub358 \uc70c\ub9ac\uc5c4 \ub85c\uc2a4\ub294 \uc62c\ub9bc\ud53d\uae38(Olympic Way)\uc774 \"\uc0ac\ub78c\ub4e4\ub85c \ub4e4\ub053\uc5c8\"\uace0 \"\uacbd\uae30\uc7a5\uc5d0 \uac00\uae4c\uc6cc\uc9c8\uc218\ub85d \uadf8 \uc815\ub3c4\uac00 \uc2ec\ud574\uc9c0\ub2e4 \uacb0\uad6d \uacbd\uae30\uc7a5\uc5d0 \ub3c4\ucc29\ud588\uc744 \ub54c\ub294 \ud68c\uc804\uc2dd \ubb38\uc774 \ub2eb\ud600\uc788\uc5c8\ub2e4\"\ub77c\uace0 \ub098\uc911\uc5d0 \ud68c\uace0\ud558\uae30\ub3c4 \ud588\ub2e4. \uacbd\uae30\uc7a5 \uc0c1\ud0dc\uc5d0 \ub300\ud55c \uc815\ubcf4\uac00 \uc5ec\ub7ec \uae30\ucc28\uc5ed\uc5d0 \uc804\ub2ec\ub418\uc5c8\uc9c0\ub9cc, \uba87 \ucc9c \uba85\uc758 \uc0ac\ub78c\ub4e4\uc774 \uacc4\uc18d \uacbd\uae30\uc7a5\uc5d0 \ub3c4\ucc29\ud574 \ucd9c\uc785\uad6c \uc8fc\ubcc0\uc5d0 \ud070 \ud63c\ub780\uc774 \uc77c\uc5b4\ub0ac\ub2e4. \uacbd\uae30\uc7a5\uc5d0\uc11c\uc758 \uacbd\uae30 \uc900\ube44\ub294 \ubd80\uc2e4\ud588\uc5c8\ub294\ub370 \u300a\ub370\uc77c\ub9ac \uba54\uc77c\u300b\uc758 \uae30\uc790\ub294 \uacbd\uae30\uc7a5\uc758 \uc0c1\ud0dc\uc5d0 \ub300\ud574 \uad00\ub9ac\uc778\ub4e4\uc740 \"\uc4f8\ubaa8\uac00 \uc5c6\uc5c8\"\uace0 \uacbd\uae30\uc7a5 \uce21 \uc0ac\ub78c\ub4e4\uc740 \uacbd\uae30\uc7a5 \ubc16\uacfc \uc548\uc758 \uc0c1\ud669\uc5d0 \ub300\ud574 \"\uc544\ubb34\uac83\ub3c4 \ubaa8\ub974\ub294 \uac83 \ucc98\ub7fc \ubcf4\uc600\ub2e4\"\uace0 \uc790\uc2e0\uc758 \ub9ac\ud3ec\ud2b8\uc5d0 \uc801\uae30\ub3c4 \ud588\ub2e4. \uacbd\uae30\uc7a5\uc744 \ucc3e\uc740 \ud32c\ub4e4\uc740 \uc544\ubb34\ub7f0 \ub9c8\ub828\ub41c \uc7a5\uc18c\ub97c \uc18c\uac1c\ubc1b\uc9c0 \ubabb\ud588\uace0, \uad00\uc911\uc11d\uc758 \uc544\ub798\uce35\uc744 \uc704\uce35\ubcf4\ub2e4 \uba87 \ubc30\ub098 \ube60\ub974\uac8c \ucc44\uc6e0\ub2e4. \uacbd\uae30\uc7a5 \ubc16\uc758 \uc778\ud30c\uac00 \uc810\uc810 \ub298\uc5b4\ub098\uc11c \uc9c0\uc5ed \uacbd\ucc30\uae4c\uc9c0 \ub3d9\uc6d0\ub418\uc5c8\uc9c0\ub9cc \uc774\ubbf8 \uc778\ud30c\uac00 \ub108\ubb34 \ucee4\uc838\ubc84\ub9ac\ub294 \ubc14\ub78c\uc5d0 \uacbd\ucc30\uc740 \ucde8\ud560 \uc218 \uc788\ub294 \ud589\ub3d9\uc774 \uc544\ubb34\uac83\ub3c4 \uc5c6\ub2e4\uace0 \ud310\ub2e8\ud588\ub2e4. \uc624\ud6c42\uc2dc 15\ubd84, \uacbd\uae30\uc7a5 \ubc16\uc758 \uc0ac\ub78c\ub4e4\uc740 \uacbd\uae30\uc7a5\uc758 \ubcbd\uc744 \ud5a5\ud574 \ub3cc\uc9c4\ud574\uc11c \uc5b5\uc9c0\ub85c \uacbd\uae30\uc7a5\uc5d0 \uc9c4\uc785\ud558\ub824\uace0 \ud588\ub2e4. \uc544\ub798\uce35\uc5d0 \uc788\ub358 \uad00\uc911\ub4e4\uc740 \uc774 \uad70\uc911\uc744 \ud53c\ud558\uae30 \uc704\ud574 \uc774 \ud39c\uc2a4\ub97c \ub118\uc5b4\uc11c \uadf8\ub77c\uc6b4\ub4dc\uae4c\uc9c0 \ub4e4\uc5b4\uac00\uac8c \ub418\uc5c8\ub2e4. \ub2f9\uc2dc \uad00\uc911 \ud14c\ub9ac \ud788\ud0a4\ub294 \"\uc904\uc5ec\uc11c \ub9d0\ud558\uc790\uba74, \ubaa8\ub4e0 \uac83\ub4e4\uc774 \uc9dc\uc99d\ub098\ub294 \ub09c\uc7a5\ud310\uc774\uc5c8\ub2e4.\"\ub77c\uba70 \ud6d7\ub0a0 \ub2f9\uc2dc \uc0c1\ud669\uc758 \uadf9\ub2e8\uc131\uc5d0 \ub300\ud574\uc11c \uc124\uba85\ud558\uae30\ub3c4 \ud588\ub2e4. \uacf5\uc2dd\uc801\uc778 \uad00\uc911 \uc218\ub294 126,047\uba85\uc73c\ub85c \uc9d1\uacc4\ub418\uc5c8\uc9c0\ub9cc \uc2e4\uc81c\ub85c \uc785\uc7a5\ud55c \uad00\uc911 \uc218\ub294 15\ub9cc\uba85\uc5d0\uc11c 30\ub9cc\uba85\uae4c\uc9c0 \uc774\ub97c \uac83\uc73c\ub85c \ucd94\uce21\ub41c\ub2e4. \uc789\uae00\ub79c\ub4dc \ucd95\uad6c\ud611\ud68c\ub294 \uc785\uc7a5\uad8c\uc744 \uc608\ub9e4\ud588\uc73c\ub098 \uc9c0\uc815\ub41c \uc88c\uc11d\uc5d0 \uc549\uc744 \uc218 \uc5c6\uc5c8\ub358 \uad00\uc911\ub4e4\uc5d0\uac8c \ud45c \uac12\uc758 10%\ub97c \ud658\ubd88\ud588\ub2e4. \uacbd\uae30\uc7a5 \uc8fc\ubcc0 \ub3c4\ub85c\ub4e4\uc774 \ubaa8\ub450 \ucc28\ub2e8\ub418\ub294 \ubc14\ub78c\uc5d0 \ubcfc\ud134\uc758 \uc120\uc218\ub4e4\uc740 \uacbd\uae30\uc7a5\uc5d0\uc11c 1\ub9c8\uc77c \ub5a8\uc5b4\uc9c4 \uacf3\uc5d0\uc11c \ubd80\ud130 \ucf54\uce58\uc640 \ub530\ub85c \ub5a8\uc5b4\uc838 \uc778\ud30c \uc18d\uc744 \ub6ab\uace0 \uacbd\uae30\uc7a5\uc5d0 \uc9c4\uc785\ud588\ub2e4. \u300a\ud0c0\uc784\u300b\uc740 \ud55c \ub54c \uacbd\uae30\uac00 \uc5f4\ub9ac\ub294 \uac83 \uc790\uccb4\uac00 \ubd88\uac00\ub2a5\ud574\ubcf4\uc600\uc73c\ub098 \uc601\uad6d\uc758 \uc655 \uc870\uc9c0 5\uc138\uac00 \ub3c4\ucc29\ud558\uc790 \uad00\uc911\ub4e4 \uc0ac\uc774\uc758 \ubd84\uc704\uae30\uac00 \ubc14\ub00c\uc5c8\ub2e4\uace0 \uae30\ub85d\ud588\ub2e4. \uad00\uc911\ub4e4\uc740 \uc5f4\ub82c\ud788 \uc601\uad6d\uc758 \uad6d\uac00\uc778 \u3008\ud558\ub290\ub2d8, \uad6d\uc655 \ud3d0\ud558\ub97c \uc9c0\ucf1c\uc8fc\uc18c\uc11c\u3009\ub97c \ub530\ub77c \ubd80\ub978 \ub4a4\uc5d0 \uc120\uc218\ub4e4\uc774 \uacbd\uae30\ud558\ub294 \uacf5\uac04\uc744 \uc815\ub9ac\ud558\ub294 \uac83\uc744 \ub3d5\uae30 \uc2dc\uc791\ud588\ub2e4.", "answer": "\uc624\uc80411\uc2dc 30\ubd84", "sentence": "\ucd9c\uc785\ubb38\uc740 \uc54c\ub824\uc9c4 \ub300\ub85c \uacbd\uae30 \uc2dc\uc791 \uc2dc\uac04 3\uc2dc\uac04 30\ubd84 \uc804\uc778 \uc624\uc80411\uc2dc 30\ubd84 \uc5d0 \uc5f4\ub838\uace0 \uc624\ud6c41\uc2dc\uae4c\uc9c0 \ubab0\ub824\ub4e0 \uad00\uc911\ub4e4\uc740 \uc9c8\uc11c\uc788\uac8c \uacbd\uae30\uc7a5\uc5d0 \uc785\uc7a5\ud588\ub2e4.", "paragraph_sentence": " \ucd9c\uc785\ubb38\uc740 \uc54c\ub824\uc9c4 \ub300\ub85c \uacbd\uae30 \uc2dc\uc791 \uc2dc\uac04 3\uc2dc\uac04 30\ubd84 \uc804\uc778 \uc624\uc80411\uc2dc 30\ubd84 \uc5d0 \uc5f4\ub838\uace0 \uc624\ud6c41\uc2dc\uae4c\uc9c0 \ubab0\ub824\ub4e0 \uad00\uc911\ub4e4\uc740 \uc9c8\uc11c\uc788\uac8c \uacbd\uae30\uc7a5\uc5d0 \uc785\uc7a5\ud588\ub2e4. \uadf8\ub7ec\ub098 1\uc2dc\uae4c\uc9c0 \ubab0\ub824\ub4e0 \uad00\uc911\ub4e4\uc774 \uac10\ub2f9\ud558\uc9c0 \ubabb\ud560 \uc815\ub3c4\ub85c \ubd88\uc5b4\ub098\uc790 \uacbd\uae30\uc7a5 \uad00\ub9ac\uce21\uc758 \uc870\uc0ac \ud6c4\uc5d0 \uc624\ud6c4 1\uc2dc 45\ubd84\uc5d0 \ucd9c\uc785\uad6c\ub97c \ub2eb\uace0 \uad00\uc911\uc758 \uc785\uc7a5\uc744 \ub9c9\uc558\ub2e4. \uad00\uc911\uc774\uc5c8\ub358 \uc70c\ub9ac\uc5c4 \ub85c\uc2a4\ub294 \uc62c\ub9bc\ud53d\uae38(Olympic Way)\uc774 \"\uc0ac\ub78c\ub4e4\ub85c \ub4e4\ub053\uc5c8\"\uace0 \"\uacbd\uae30\uc7a5\uc5d0 \uac00\uae4c\uc6cc\uc9c8\uc218\ub85d \uadf8 \uc815\ub3c4\uac00 \uc2ec\ud574\uc9c0\ub2e4 \uacb0\uad6d \uacbd\uae30\uc7a5\uc5d0 \ub3c4\ucc29\ud588\uc744 \ub54c\ub294 \ud68c\uc804\uc2dd \ubb38\uc774 \ub2eb\ud600\uc788\uc5c8\ub2e4\"\ub77c\uace0 \ub098\uc911\uc5d0 \ud68c\uace0\ud558\uae30\ub3c4 \ud588\ub2e4. \uacbd\uae30\uc7a5 \uc0c1\ud0dc\uc5d0 \ub300\ud55c \uc815\ubcf4\uac00 \uc5ec\ub7ec \uae30\ucc28\uc5ed\uc5d0 \uc804\ub2ec\ub418\uc5c8\uc9c0\ub9cc, \uba87 \ucc9c \uba85\uc758 \uc0ac\ub78c\ub4e4\uc774 \uacc4\uc18d \uacbd\uae30\uc7a5\uc5d0 \ub3c4\ucc29\ud574 \ucd9c\uc785\uad6c \uc8fc\ubcc0\uc5d0 \ud070 \ud63c\ub780\uc774 \uc77c\uc5b4\ub0ac\ub2e4. \uacbd\uae30\uc7a5\uc5d0\uc11c\uc758 \uacbd\uae30 \uc900\ube44\ub294 \ubd80\uc2e4\ud588\uc5c8\ub294\ub370 \u300a\ub370\uc77c\ub9ac \uba54\uc77c\u300b\uc758 \uae30\uc790\ub294 \uacbd\uae30\uc7a5\uc758 \uc0c1\ud0dc\uc5d0 \ub300\ud574 \uad00\ub9ac\uc778\ub4e4\uc740 \"\uc4f8\ubaa8\uac00 \uc5c6\uc5c8\"\uace0 \uacbd\uae30\uc7a5 \uce21 \uc0ac\ub78c\ub4e4\uc740 \uacbd\uae30\uc7a5 \ubc16\uacfc \uc548\uc758 \uc0c1\ud669\uc5d0 \ub300\ud574 \"\uc544\ubb34\uac83\ub3c4 \ubaa8\ub974\ub294 \uac83 \ucc98\ub7fc \ubcf4\uc600\ub2e4\"\uace0 \uc790\uc2e0\uc758 \ub9ac\ud3ec\ud2b8\uc5d0 \uc801\uae30\ub3c4 \ud588\ub2e4. \uacbd\uae30\uc7a5\uc744 \ucc3e\uc740 \ud32c\ub4e4\uc740 \uc544\ubb34\ub7f0 \ub9c8\ub828\ub41c \uc7a5\uc18c\ub97c \uc18c\uac1c\ubc1b\uc9c0 \ubabb\ud588\uace0, \uad00\uc911\uc11d\uc758 \uc544\ub798\uce35\uc744 \uc704\uce35\ubcf4\ub2e4 \uba87 \ubc30\ub098 \ube60\ub974\uac8c \ucc44\uc6e0\ub2e4. \uacbd\uae30\uc7a5 \ubc16\uc758 \uc778\ud30c\uac00 \uc810\uc810 \ub298\uc5b4\ub098\uc11c \uc9c0\uc5ed \uacbd\ucc30\uae4c\uc9c0 \ub3d9\uc6d0\ub418\uc5c8\uc9c0\ub9cc \uc774\ubbf8 \uc778\ud30c\uac00 \ub108\ubb34 \ucee4\uc838\ubc84\ub9ac\ub294 \ubc14\ub78c\uc5d0 \uacbd\ucc30\uc740 \ucde8\ud560 \uc218 \uc788\ub294 \ud589\ub3d9\uc774 \uc544\ubb34\uac83\ub3c4 \uc5c6\ub2e4\uace0 \ud310\ub2e8\ud588\ub2e4. \uc624\ud6c42\uc2dc 15\ubd84, \uacbd\uae30\uc7a5 \ubc16\uc758 \uc0ac\ub78c\ub4e4\uc740 \uacbd\uae30\uc7a5\uc758 \ubcbd\uc744 \ud5a5\ud574 \ub3cc\uc9c4\ud574\uc11c \uc5b5\uc9c0\ub85c \uacbd\uae30\uc7a5\uc5d0 \uc9c4\uc785\ud558\ub824\uace0 \ud588\ub2e4. \uc544\ub798\uce35\uc5d0 \uc788\ub358 \uad00\uc911\ub4e4\uc740 \uc774 \uad70\uc911\uc744 \ud53c\ud558\uae30 \uc704\ud574 \uc774 \ud39c\uc2a4\ub97c \ub118\uc5b4\uc11c \uadf8\ub77c\uc6b4\ub4dc\uae4c\uc9c0 \ub4e4\uc5b4\uac00\uac8c \ub418\uc5c8\ub2e4. \ub2f9\uc2dc \uad00\uc911 \ud14c\ub9ac \ud788\ud0a4\ub294 \"\uc904\uc5ec\uc11c \ub9d0\ud558\uc790\uba74, \ubaa8\ub4e0 \uac83\ub4e4\uc774 \uc9dc\uc99d\ub098\ub294 \ub09c\uc7a5\ud310\uc774\uc5c8\ub2e4. \"\ub77c\uba70 \ud6d7\ub0a0 \ub2f9\uc2dc \uc0c1\ud669\uc758 \uadf9\ub2e8\uc131\uc5d0 \ub300\ud574\uc11c \uc124\uba85\ud558\uae30\ub3c4 \ud588\ub2e4. \uacf5\uc2dd\uc801\uc778 \uad00\uc911 \uc218\ub294 126,047\uba85\uc73c\ub85c \uc9d1\uacc4\ub418\uc5c8\uc9c0\ub9cc \uc2e4\uc81c\ub85c \uc785\uc7a5\ud55c \uad00\uc911 \uc218\ub294 15\ub9cc\uba85\uc5d0\uc11c 30\ub9cc\uba85\uae4c\uc9c0 \uc774\ub97c \uac83\uc73c\ub85c \ucd94\uce21\ub41c\ub2e4. \uc789\uae00\ub79c\ub4dc \ucd95\uad6c\ud611\ud68c\ub294 \uc785\uc7a5\uad8c\uc744 \uc608\ub9e4\ud588\uc73c\ub098 \uc9c0\uc815\ub41c \uc88c\uc11d\uc5d0 \uc549\uc744 \uc218 \uc5c6\uc5c8\ub358 \uad00\uc911\ub4e4\uc5d0\uac8c \ud45c \uac12\uc758 10%\ub97c \ud658\ubd88\ud588\ub2e4. \uacbd\uae30\uc7a5 \uc8fc\ubcc0 \ub3c4\ub85c\ub4e4\uc774 \ubaa8\ub450 \ucc28\ub2e8\ub418\ub294 \ubc14\ub78c\uc5d0 \ubcfc\ud134\uc758 \uc120\uc218\ub4e4\uc740 \uacbd\uae30\uc7a5\uc5d0\uc11c 1\ub9c8\uc77c \ub5a8\uc5b4\uc9c4 \uacf3\uc5d0\uc11c \ubd80\ud130 \ucf54\uce58\uc640 \ub530\ub85c \ub5a8\uc5b4\uc838 \uc778\ud30c \uc18d\uc744 \ub6ab\uace0 \uacbd\uae30\uc7a5\uc5d0 \uc9c4\uc785\ud588\ub2e4. \u300a\ud0c0\uc784\u300b\uc740 \ud55c \ub54c \uacbd\uae30\uac00 \uc5f4\ub9ac\ub294 \uac83 \uc790\uccb4\uac00 \ubd88\uac00\ub2a5\ud574\ubcf4\uc600\uc73c\ub098 \uc601\uad6d\uc758 \uc655 \uc870\uc9c0 5\uc138\uac00 \ub3c4\ucc29\ud558\uc790 \uad00\uc911\ub4e4 \uc0ac\uc774\uc758 \ubd84\uc704\uae30\uac00 \ubc14\ub00c\uc5c8\ub2e4\uace0 \uae30\ub85d\ud588\ub2e4. \uad00\uc911\ub4e4\uc740 \uc5f4\ub82c\ud788 \uc601\uad6d\uc758 \uad6d\uac00\uc778 \u3008\ud558\ub290\ub2d8, \uad6d\uc655 \ud3d0\ud558\ub97c \uc9c0\ucf1c\uc8fc\uc18c\uc11c\u3009\ub97c \ub530\ub77c \ubd80\ub978 \ub4a4\uc5d0 \uc120\uc218\ub4e4\uc774 \uacbd\uae30\ud558\ub294 \uacf5\uac04\uc744 \uc815\ub9ac\ud558\ub294 \uac83\uc744 \ub3d5\uae30 \uc2dc\uc791\ud588\ub2e4.", "paragraph_answer": "\ucd9c\uc785\ubb38\uc740 \uc54c\ub824\uc9c4 \ub300\ub85c \uacbd\uae30 \uc2dc\uc791 \uc2dc\uac04 3\uc2dc\uac04 30\ubd84 \uc804\uc778 \uc624\uc80411\uc2dc 30\ubd84 \uc5d0 \uc5f4\ub838\uace0 \uc624\ud6c41\uc2dc\uae4c\uc9c0 \ubab0\ub824\ub4e0 \uad00\uc911\ub4e4\uc740 \uc9c8\uc11c\uc788\uac8c \uacbd\uae30\uc7a5\uc5d0 \uc785\uc7a5\ud588\ub2e4. \uadf8\ub7ec\ub098 1\uc2dc\uae4c\uc9c0 \ubab0\ub824\ub4e0 \uad00\uc911\ub4e4\uc774 \uac10\ub2f9\ud558\uc9c0 \ubabb\ud560 \uc815\ub3c4\ub85c \ubd88\uc5b4\ub098\uc790 \uacbd\uae30\uc7a5 \uad00\ub9ac\uce21\uc758 \uc870\uc0ac \ud6c4\uc5d0 \uc624\ud6c4 1\uc2dc 45\ubd84\uc5d0 \ucd9c\uc785\uad6c\ub97c \ub2eb\uace0 \uad00\uc911\uc758 \uc785\uc7a5\uc744 \ub9c9\uc558\ub2e4. \uad00\uc911\uc774\uc5c8\ub358 \uc70c\ub9ac\uc5c4 \ub85c\uc2a4\ub294 \uc62c\ub9bc\ud53d\uae38(Olympic Way)\uc774 \"\uc0ac\ub78c\ub4e4\ub85c \ub4e4\ub053\uc5c8\"\uace0 \"\uacbd\uae30\uc7a5\uc5d0 \uac00\uae4c\uc6cc\uc9c8\uc218\ub85d \uadf8 \uc815\ub3c4\uac00 \uc2ec\ud574\uc9c0\ub2e4 \uacb0\uad6d \uacbd\uae30\uc7a5\uc5d0 \ub3c4\ucc29\ud588\uc744 \ub54c\ub294 \ud68c\uc804\uc2dd \ubb38\uc774 \ub2eb\ud600\uc788\uc5c8\ub2e4\"\ub77c\uace0 \ub098\uc911\uc5d0 \ud68c\uace0\ud558\uae30\ub3c4 \ud588\ub2e4. \uacbd\uae30\uc7a5 \uc0c1\ud0dc\uc5d0 \ub300\ud55c \uc815\ubcf4\uac00 \uc5ec\ub7ec \uae30\ucc28\uc5ed\uc5d0 \uc804\ub2ec\ub418\uc5c8\uc9c0\ub9cc, \uba87 \ucc9c \uba85\uc758 \uc0ac\ub78c\ub4e4\uc774 \uacc4\uc18d \uacbd\uae30\uc7a5\uc5d0 \ub3c4\ucc29\ud574 \ucd9c\uc785\uad6c \uc8fc\ubcc0\uc5d0 \ud070 \ud63c\ub780\uc774 \uc77c\uc5b4\ub0ac\ub2e4. \uacbd\uae30\uc7a5\uc5d0\uc11c\uc758 \uacbd\uae30 \uc900\ube44\ub294 \ubd80\uc2e4\ud588\uc5c8\ub294\ub370 \u300a\ub370\uc77c\ub9ac \uba54\uc77c\u300b\uc758 \uae30\uc790\ub294 \uacbd\uae30\uc7a5\uc758 \uc0c1\ud0dc\uc5d0 \ub300\ud574 \uad00\ub9ac\uc778\ub4e4\uc740 \"\uc4f8\ubaa8\uac00 \uc5c6\uc5c8\"\uace0 \uacbd\uae30\uc7a5 \uce21 \uc0ac\ub78c\ub4e4\uc740 \uacbd\uae30\uc7a5 \ubc16\uacfc \uc548\uc758 \uc0c1\ud669\uc5d0 \ub300\ud574 \"\uc544\ubb34\uac83\ub3c4 \ubaa8\ub974\ub294 \uac83 \ucc98\ub7fc \ubcf4\uc600\ub2e4\"\uace0 \uc790\uc2e0\uc758 \ub9ac\ud3ec\ud2b8\uc5d0 \uc801\uae30\ub3c4 \ud588\ub2e4. \uacbd\uae30\uc7a5\uc744 \ucc3e\uc740 \ud32c\ub4e4\uc740 \uc544\ubb34\ub7f0 \ub9c8\ub828\ub41c \uc7a5\uc18c\ub97c \uc18c\uac1c\ubc1b\uc9c0 \ubabb\ud588\uace0, \uad00\uc911\uc11d\uc758 \uc544\ub798\uce35\uc744 \uc704\uce35\ubcf4\ub2e4 \uba87 \ubc30\ub098 \ube60\ub974\uac8c \ucc44\uc6e0\ub2e4. \uacbd\uae30\uc7a5 \ubc16\uc758 \uc778\ud30c\uac00 \uc810\uc810 \ub298\uc5b4\ub098\uc11c \uc9c0\uc5ed \uacbd\ucc30\uae4c\uc9c0 \ub3d9\uc6d0\ub418\uc5c8\uc9c0\ub9cc \uc774\ubbf8 \uc778\ud30c\uac00 \ub108\ubb34 \ucee4\uc838\ubc84\ub9ac\ub294 \ubc14\ub78c\uc5d0 \uacbd\ucc30\uc740 \ucde8\ud560 \uc218 \uc788\ub294 \ud589\ub3d9\uc774 \uc544\ubb34\uac83\ub3c4 \uc5c6\ub2e4\uace0 \ud310\ub2e8\ud588\ub2e4. \uc624\ud6c42\uc2dc 15\ubd84, \uacbd\uae30\uc7a5 \ubc16\uc758 \uc0ac\ub78c\ub4e4\uc740 \uacbd\uae30\uc7a5\uc758 \ubcbd\uc744 \ud5a5\ud574 \ub3cc\uc9c4\ud574\uc11c \uc5b5\uc9c0\ub85c \uacbd\uae30\uc7a5\uc5d0 \uc9c4\uc785\ud558\ub824\uace0 \ud588\ub2e4. \uc544\ub798\uce35\uc5d0 \uc788\ub358 \uad00\uc911\ub4e4\uc740 \uc774 \uad70\uc911\uc744 \ud53c\ud558\uae30 \uc704\ud574 \uc774 \ud39c\uc2a4\ub97c \ub118\uc5b4\uc11c \uadf8\ub77c\uc6b4\ub4dc\uae4c\uc9c0 \ub4e4\uc5b4\uac00\uac8c \ub418\uc5c8\ub2e4. \ub2f9\uc2dc \uad00\uc911 \ud14c\ub9ac \ud788\ud0a4\ub294 \"\uc904\uc5ec\uc11c \ub9d0\ud558\uc790\uba74, \ubaa8\ub4e0 \uac83\ub4e4\uc774 \uc9dc\uc99d\ub098\ub294 \ub09c\uc7a5\ud310\uc774\uc5c8\ub2e4.\"\ub77c\uba70 \ud6d7\ub0a0 \ub2f9\uc2dc \uc0c1\ud669\uc758 \uadf9\ub2e8\uc131\uc5d0 \ub300\ud574\uc11c \uc124\uba85\ud558\uae30\ub3c4 \ud588\ub2e4. \uacf5\uc2dd\uc801\uc778 \uad00\uc911 \uc218\ub294 126,047\uba85\uc73c\ub85c \uc9d1\uacc4\ub418\uc5c8\uc9c0\ub9cc \uc2e4\uc81c\ub85c \uc785\uc7a5\ud55c \uad00\uc911 \uc218\ub294 15\ub9cc\uba85\uc5d0\uc11c 30\ub9cc\uba85\uae4c\uc9c0 \uc774\ub97c \uac83\uc73c\ub85c \ucd94\uce21\ub41c\ub2e4. \uc789\uae00\ub79c\ub4dc \ucd95\uad6c\ud611\ud68c\ub294 \uc785\uc7a5\uad8c\uc744 \uc608\ub9e4\ud588\uc73c\ub098 \uc9c0\uc815\ub41c \uc88c\uc11d\uc5d0 \uc549\uc744 \uc218 \uc5c6\uc5c8\ub358 \uad00\uc911\ub4e4\uc5d0\uac8c \ud45c \uac12\uc758 10%\ub97c \ud658\ubd88\ud588\ub2e4. \uacbd\uae30\uc7a5 \uc8fc\ubcc0 \ub3c4\ub85c\ub4e4\uc774 \ubaa8\ub450 \ucc28\ub2e8\ub418\ub294 \ubc14\ub78c\uc5d0 \ubcfc\ud134\uc758 \uc120\uc218\ub4e4\uc740 \uacbd\uae30\uc7a5\uc5d0\uc11c 1\ub9c8\uc77c \ub5a8\uc5b4\uc9c4 \uacf3\uc5d0\uc11c \ubd80\ud130 \ucf54\uce58\uc640 \ub530\ub85c \ub5a8\uc5b4\uc838 \uc778\ud30c \uc18d\uc744 \ub6ab\uace0 \uacbd\uae30\uc7a5\uc5d0 \uc9c4\uc785\ud588\ub2e4. \u300a\ud0c0\uc784\u300b\uc740 \ud55c \ub54c \uacbd\uae30\uac00 \uc5f4\ub9ac\ub294 \uac83 \uc790\uccb4\uac00 \ubd88\uac00\ub2a5\ud574\ubcf4\uc600\uc73c\ub098 \uc601\uad6d\uc758 \uc655 \uc870\uc9c0 5\uc138\uac00 \ub3c4\ucc29\ud558\uc790 \uad00\uc911\ub4e4 \uc0ac\uc774\uc758 \ubd84\uc704\uae30\uac00 \ubc14\ub00c\uc5c8\ub2e4\uace0 \uae30\ub85d\ud588\ub2e4. \uad00\uc911\ub4e4\uc740 \uc5f4\ub82c\ud788 \uc601\uad6d\uc758 \uad6d\uac00\uc778 \u3008\ud558\ub290\ub2d8, \uad6d\uc655 \ud3d0\ud558\ub97c \uc9c0\ucf1c\uc8fc\uc18c\uc11c\u3009\ub97c \ub530\ub77c \ubd80\ub978 \ub4a4\uc5d0 \uc120\uc218\ub4e4\uc774 \uacbd\uae30\ud558\ub294 \uacf5\uac04\uc744 \uc815\ub9ac\ud558\ub294 \uac83\uc744 \ub3d5\uae30 \uc2dc\uc791\ud588\ub2e4.", "sentence_answer": "\ucd9c\uc785\ubb38\uc740 \uc54c\ub824\uc9c4 \ub300\ub85c \uacbd\uae30 \uc2dc\uc791 \uc2dc\uac04 3\uc2dc\uac04 30\ubd84 \uc804\uc778 \uc624\uc80411\uc2dc 30\ubd84 \uc5d0 \uc5f4\ub838\uace0 \uc624\ud6c41\uc2dc\uae4c\uc9c0 \ubab0\ub824\ub4e0 \uad00\uc911\ub4e4\uc740 \uc9c8\uc11c\uc788\uac8c \uacbd\uae30\uc7a5\uc5d0 \uc785\uc7a5\ud588\ub2e4.", "paragraph_id": "6489739-4-0", "paragraph_question": "question: \uc2a4\ud0c0\ub514\uc6c0\uc758 \ucd9c\uc785\ubb38\uc774 \uc5f4\ub9b0 \uc2dc\uac04\uc740?, context: \ucd9c\uc785\ubb38\uc740 \uc54c\ub824\uc9c4 \ub300\ub85c \uacbd\uae30 \uc2dc\uc791 \uc2dc\uac04 3\uc2dc\uac04 30\ubd84 \uc804\uc778 \uc624\uc80411\uc2dc 30\ubd84\uc5d0 \uc5f4\ub838\uace0 \uc624\ud6c41\uc2dc\uae4c\uc9c0 \ubab0\ub824\ub4e0 \uad00\uc911\ub4e4\uc740 \uc9c8\uc11c\uc788\uac8c \uacbd\uae30\uc7a5\uc5d0 \uc785\uc7a5\ud588\ub2e4. \uadf8\ub7ec\ub098 1\uc2dc\uae4c\uc9c0 \ubab0\ub824\ub4e0 \uad00\uc911\ub4e4\uc774 \uac10\ub2f9\ud558\uc9c0 \ubabb\ud560 \uc815\ub3c4\ub85c \ubd88\uc5b4\ub098\uc790 \uacbd\uae30\uc7a5 \uad00\ub9ac\uce21\uc758 \uc870\uc0ac \ud6c4\uc5d0 \uc624\ud6c4 1\uc2dc 45\ubd84\uc5d0 \ucd9c\uc785\uad6c\ub97c \ub2eb\uace0 \uad00\uc911\uc758 \uc785\uc7a5\uc744 \ub9c9\uc558\ub2e4. \uad00\uc911\uc774\uc5c8\ub358 \uc70c\ub9ac\uc5c4 \ub85c\uc2a4\ub294 \uc62c\ub9bc\ud53d\uae38(Olympic Way)\uc774 \"\uc0ac\ub78c\ub4e4\ub85c \ub4e4\ub053\uc5c8\"\uace0 \"\uacbd\uae30\uc7a5\uc5d0 \uac00\uae4c\uc6cc\uc9c8\uc218\ub85d \uadf8 \uc815\ub3c4\uac00 \uc2ec\ud574\uc9c0\ub2e4 \uacb0\uad6d \uacbd\uae30\uc7a5\uc5d0 \ub3c4\ucc29\ud588\uc744 \ub54c\ub294 \ud68c\uc804\uc2dd \ubb38\uc774 \ub2eb\ud600\uc788\uc5c8\ub2e4\"\ub77c\uace0 \ub098\uc911\uc5d0 \ud68c\uace0\ud558\uae30\ub3c4 \ud588\ub2e4. \uacbd\uae30\uc7a5 \uc0c1\ud0dc\uc5d0 \ub300\ud55c \uc815\ubcf4\uac00 \uc5ec\ub7ec \uae30\ucc28\uc5ed\uc5d0 \uc804\ub2ec\ub418\uc5c8\uc9c0\ub9cc, \uba87 \ucc9c \uba85\uc758 \uc0ac\ub78c\ub4e4\uc774 \uacc4\uc18d \uacbd\uae30\uc7a5\uc5d0 \ub3c4\ucc29\ud574 \ucd9c\uc785\uad6c \uc8fc\ubcc0\uc5d0 \ud070 \ud63c\ub780\uc774 \uc77c\uc5b4\ub0ac\ub2e4. \uacbd\uae30\uc7a5\uc5d0\uc11c\uc758 \uacbd\uae30 \uc900\ube44\ub294 \ubd80\uc2e4\ud588\uc5c8\ub294\ub370 \u300a\ub370\uc77c\ub9ac \uba54\uc77c\u300b\uc758 \uae30\uc790\ub294 \uacbd\uae30\uc7a5\uc758 \uc0c1\ud0dc\uc5d0 \ub300\ud574 \uad00\ub9ac\uc778\ub4e4\uc740 \"\uc4f8\ubaa8\uac00 \uc5c6\uc5c8\"\uace0 \uacbd\uae30\uc7a5 \uce21 \uc0ac\ub78c\ub4e4\uc740 \uacbd\uae30\uc7a5 \ubc16\uacfc \uc548\uc758 \uc0c1\ud669\uc5d0 \ub300\ud574 \"\uc544\ubb34\uac83\ub3c4 \ubaa8\ub974\ub294 \uac83 \ucc98\ub7fc \ubcf4\uc600\ub2e4\"\uace0 \uc790\uc2e0\uc758 \ub9ac\ud3ec\ud2b8\uc5d0 \uc801\uae30\ub3c4 \ud588\ub2e4. \uacbd\uae30\uc7a5\uc744 \ucc3e\uc740 \ud32c\ub4e4\uc740 \uc544\ubb34\ub7f0 \ub9c8\ub828\ub41c \uc7a5\uc18c\ub97c \uc18c\uac1c\ubc1b\uc9c0 \ubabb\ud588\uace0, \uad00\uc911\uc11d\uc758 \uc544\ub798\uce35\uc744 \uc704\uce35\ubcf4\ub2e4 \uba87 \ubc30\ub098 \ube60\ub974\uac8c \ucc44\uc6e0\ub2e4. \uacbd\uae30\uc7a5 \ubc16\uc758 \uc778\ud30c\uac00 \uc810\uc810 \ub298\uc5b4\ub098\uc11c \uc9c0\uc5ed \uacbd\ucc30\uae4c\uc9c0 \ub3d9\uc6d0\ub418\uc5c8\uc9c0\ub9cc \uc774\ubbf8 \uc778\ud30c\uac00 \ub108\ubb34 \ucee4\uc838\ubc84\ub9ac\ub294 \ubc14\ub78c\uc5d0 \uacbd\ucc30\uc740 \ucde8\ud560 \uc218 \uc788\ub294 \ud589\ub3d9\uc774 \uc544\ubb34\uac83\ub3c4 \uc5c6\ub2e4\uace0 \ud310\ub2e8\ud588\ub2e4. \uc624\ud6c42\uc2dc 15\ubd84, \uacbd\uae30\uc7a5 \ubc16\uc758 \uc0ac\ub78c\ub4e4\uc740 \uacbd\uae30\uc7a5\uc758 \ubcbd\uc744 \ud5a5\ud574 \ub3cc\uc9c4\ud574\uc11c \uc5b5\uc9c0\ub85c \uacbd\uae30\uc7a5\uc5d0 \uc9c4\uc785\ud558\ub824\uace0 \ud588\ub2e4. \uc544\ub798\uce35\uc5d0 \uc788\ub358 \uad00\uc911\ub4e4\uc740 \uc774 \uad70\uc911\uc744 \ud53c\ud558\uae30 \uc704\ud574 \uc774 \ud39c\uc2a4\ub97c \ub118\uc5b4\uc11c \uadf8\ub77c\uc6b4\ub4dc\uae4c\uc9c0 \ub4e4\uc5b4\uac00\uac8c \ub418\uc5c8\ub2e4. \ub2f9\uc2dc \uad00\uc911 \ud14c\ub9ac \ud788\ud0a4\ub294 \"\uc904\uc5ec\uc11c \ub9d0\ud558\uc790\uba74, \ubaa8\ub4e0 \uac83\ub4e4\uc774 \uc9dc\uc99d\ub098\ub294 \ub09c\uc7a5\ud310\uc774\uc5c8\ub2e4.\"\ub77c\uba70 \ud6d7\ub0a0 \ub2f9\uc2dc \uc0c1\ud669\uc758 \uadf9\ub2e8\uc131\uc5d0 \ub300\ud574\uc11c \uc124\uba85\ud558\uae30\ub3c4 \ud588\ub2e4. \uacf5\uc2dd\uc801\uc778 \uad00\uc911 \uc218\ub294 126,047\uba85\uc73c\ub85c \uc9d1\uacc4\ub418\uc5c8\uc9c0\ub9cc \uc2e4\uc81c\ub85c \uc785\uc7a5\ud55c \uad00\uc911 \uc218\ub294 15\ub9cc\uba85\uc5d0\uc11c 30\ub9cc\uba85\uae4c\uc9c0 \uc774\ub97c \uac83\uc73c\ub85c \ucd94\uce21\ub41c\ub2e4. \uc789\uae00\ub79c\ub4dc \ucd95\uad6c\ud611\ud68c\ub294 \uc785\uc7a5\uad8c\uc744 \uc608\ub9e4\ud588\uc73c\ub098 \uc9c0\uc815\ub41c \uc88c\uc11d\uc5d0 \uc549\uc744 \uc218 \uc5c6\uc5c8\ub358 \uad00\uc911\ub4e4\uc5d0\uac8c \ud45c \uac12\uc758 10%\ub97c \ud658\ubd88\ud588\ub2e4. \uacbd\uae30\uc7a5 \uc8fc\ubcc0 \ub3c4\ub85c\ub4e4\uc774 \ubaa8\ub450 \ucc28\ub2e8\ub418\ub294 \ubc14\ub78c\uc5d0 \ubcfc\ud134\uc758 \uc120\uc218\ub4e4\uc740 \uacbd\uae30\uc7a5\uc5d0\uc11c 1\ub9c8\uc77c \ub5a8\uc5b4\uc9c4 \uacf3\uc5d0\uc11c \ubd80\ud130 \ucf54\uce58\uc640 \ub530\ub85c \ub5a8\uc5b4\uc838 \uc778\ud30c \uc18d\uc744 \ub6ab\uace0 \uacbd\uae30\uc7a5\uc5d0 \uc9c4\uc785\ud588\ub2e4. \u300a\ud0c0\uc784\u300b\uc740 \ud55c \ub54c \uacbd\uae30\uac00 \uc5f4\ub9ac\ub294 \uac83 \uc790\uccb4\uac00 \ubd88\uac00\ub2a5\ud574\ubcf4\uc600\uc73c\ub098 \uc601\uad6d\uc758 \uc655 \uc870\uc9c0 5\uc138\uac00 \ub3c4\ucc29\ud558\uc790 \uad00\uc911\ub4e4 \uc0ac\uc774\uc758 \ubd84\uc704\uae30\uac00 \ubc14\ub00c\uc5c8\ub2e4\uace0 \uae30\ub85d\ud588\ub2e4. \uad00\uc911\ub4e4\uc740 \uc5f4\ub82c\ud788 \uc601\uad6d\uc758 \uad6d\uac00\uc778 \u3008\ud558\ub290\ub2d8, \uad6d\uc655 \ud3d0\ud558\ub97c \uc9c0\ucf1c\uc8fc\uc18c\uc11c\u3009\ub97c \ub530\ub77c \ubd80\ub978 \ub4a4\uc5d0 \uc120\uc218\ub4e4\uc774 \uacbd\uae30\ud558\ub294 \uacf5\uac04\uc744 \uc815\ub9ac\ud558\ub294 \uac83\uc744 \ub3d5\uae30 \uc2dc\uc791\ud588\ub2e4."} {"question": "\ud6c4\ubbf8\uc544\ud0a4\uac00 \uc815\ub144\ud1f4\uc784\uae4c\uc9c0 \uc77c\ud55c \uacf3\uc740?", "paragraph": "\ubd80\ub9ac\ub354 \ud558\uc138\uac00\uc640 \ud6c4\ubbf8\uc544\ud0a4\ub294 \ucd9c\uc18c \ud6c4\uc5d0 \ubc14\ub85c \ub3c4\ucfc4\ub85c \uc62c\ub77c\uac00 \uc815\ub144\uc73c\ub85c \ud1f4\uc784\ud560 \ub54c\uae4c\uc9c0 \ud55c \uc778\uc1c4\ud68c\uc0ac\uc5d0\uc11c \uc77c\ud588\ub2e4. \uadf8\ub9ac\uace0 \uc77c\ubcf8\uc5b4 \ud0c0\uc790\uae30(\uc77c\ubcf8\uc5b4: \u548c\u6587\u30bf\u30a4\u30d7\u30e9\u30a4\u30bf\u30fc \uc640\ubaac \ud0c0\uc774\ud478 \ub77c\uc774\ud0c0)\ub97c \uc774\uc6a9\ud55c \uc77c\ubcf8\uc5b4 \uc545\ubcf4 \uc778\uc1c4\uc758 \uc790\ub3d9\ud654\uc5d0 \uc131\uacf5\ud588\ub2e4. \uc790\uc2e0\uc758 \uae30\uc220 \uc219\ub828\uacfc \ud568\uaed8, \uc624\uce74\ucfe0\ub77c \ub374\uc2e0\uc758 \ubca0\ud1a0\ubca4\uc5d0 \ub300\ud55c \uc774\ud574\uc5d0 \uc601\ud5a5\uc744 \ubc1b\uc740 \uac83\uc774\uc5c8\ub2e4. \uc624\uce74\ucfe0\ub77c\ub294 \ubca0\ud1a0\ubca4\uc744 \uc608\uc220\ub85c\uc11c \uc720\uc77c\ud558\uac8c \uc720\ub7fd\uc774 \uc544\uc2dc\uc544\ub97c \uc774\uae30\uac8c \ud55c \uc778\ubb3c\ub85c\uae4c\uc9c0 \ubcf4\uace0 \uc788\uc5c8\ub2e4. \uc774\uac83\uc744 \uc57c\uc2a4\ub2e4 \uc694\uc8fc\ub85c\uc758 \uc800\uc11c \u300a\uba54\uc774\uc9c0\uc758 \uc815\uc2e0\u300b\uc744 \uc77d\uc73c\uba74\uc11c \uc54c\uac8c \ub41c \ud558\uc138\uac00\uc640\ub294 \uc11c\uc591 \uc74c\uc545\uc758 \uc545\ubcf4\ub97c \uc218\uc9d1\ud574\uac14\uc9c0\ub9cc, \uc790\uc2e0\uc774 \uc6d0\ud558\ub294 \uac83\uc744 \uc5bb\uae30\ub294 \ud798\ub4e4\uc5c8\ub2e4. \uacb0\uad6d \ud558\uc138\uac00\uc640\ub294 \uadf8 \ub54c\ubb38\uc5d0 \uc2a4\uc2a4\ub85c \uc545\ubcf4\ub97c \uc870\ud310\ud560 \uc218 \uc5c6\uc744\uae4c \ud558\uace0 \uc0dd\uac01\ud558\uc5ec, \uc545\ubcf4 \uc778\uc1c4\uc758 \uc790\ub3d9\ud654\uc5d0 \ub3c4\uc804\ud558\uc600\ub2e4.", "answer": "\uc778\uc1c4\ud68c\uc0ac", "sentence": "\uc815\ub144\uc73c\ub85c \ud1f4\uc784\ud560 \ub54c\uae4c\uc9c0 \ud55c \uc778\uc1c4\ud68c\uc0ac \uc5d0\uc11c \uc77c\ud588\ub2e4.", "paragraph_sentence": "\ubd80\ub9ac\ub354 \ud558\uc138\uac00\uc640 \ud6c4\ubbf8\uc544\ud0a4\ub294 \ucd9c\uc18c \ud6c4\uc5d0 \ubc14\ub85c \ub3c4\ucfc4\ub85c \uc62c\ub77c\uac00 \uc815\ub144\uc73c\ub85c \ud1f4\uc784\ud560 \ub54c\uae4c\uc9c0 \ud55c \uc778\uc1c4\ud68c\uc0ac \uc5d0\uc11c \uc77c\ud588\ub2e4. \uadf8\ub9ac\uace0 \uc77c\ubcf8\uc5b4 \ud0c0\uc790\uae30(\uc77c\ubcf8\uc5b4: \u548c\u6587\u30bf\u30a4\u30d7\u30e9\u30a4\u30bf\u30fc \uc640\ubaac \ud0c0\uc774\ud478 \ub77c\uc774\ud0c0)\ub97c \uc774\uc6a9\ud55c \uc77c\ubcf8\uc5b4 \uc545\ubcf4 \uc778\uc1c4\uc758 \uc790\ub3d9\ud654\uc5d0 \uc131\uacf5\ud588\ub2e4. \uc790\uc2e0\uc758 \uae30\uc220 \uc219\ub828\uacfc \ud568\uaed8, \uc624\uce74\ucfe0\ub77c \ub374\uc2e0\uc758 \ubca0\ud1a0\ubca4\uc5d0 \ub300\ud55c \uc774\ud574\uc5d0 \uc601\ud5a5\uc744 \ubc1b\uc740 \uac83\uc774\uc5c8\ub2e4. \uc624\uce74\ucfe0\ub77c\ub294 \ubca0\ud1a0\ubca4\uc744 \uc608\uc220\ub85c\uc11c \uc720\uc77c\ud558\uac8c \uc720\ub7fd\uc774 \uc544\uc2dc\uc544\ub97c \uc774\uae30\uac8c \ud55c \uc778\ubb3c\ub85c\uae4c\uc9c0 \ubcf4\uace0 \uc788\uc5c8\ub2e4. \uc774\uac83\uc744 \uc57c\uc2a4\ub2e4 \uc694\uc8fc\ub85c\uc758 \uc800\uc11c \u300a\uba54\uc774\uc9c0\uc758 \uc815\uc2e0\u300b\uc744 \uc77d\uc73c\uba74\uc11c \uc54c\uac8c \ub41c \ud558\uc138\uac00\uc640\ub294 \uc11c\uc591 \uc74c\uc545\uc758 \uc545\ubcf4\ub97c \uc218\uc9d1\ud574\uac14\uc9c0\ub9cc, \uc790\uc2e0\uc774 \uc6d0\ud558\ub294 \uac83\uc744 \uc5bb\uae30\ub294 \ud798\ub4e4\uc5c8\ub2e4. \uacb0\uad6d \ud558\uc138\uac00\uc640\ub294 \uadf8 \ub54c\ubb38\uc5d0 \uc2a4\uc2a4\ub85c \uc545\ubcf4\ub97c \uc870\ud310\ud560 \uc218 \uc5c6\uc744\uae4c \ud558\uace0 \uc0dd\uac01\ud558\uc5ec, \uc545\ubcf4 \uc778\uc1c4\uc758 \uc790\ub3d9\ud654\uc5d0 \ub3c4\uc804\ud558\uc600\ub2e4.", "paragraph_answer": "\ubd80\ub9ac\ub354 \ud558\uc138\uac00\uc640 \ud6c4\ubbf8\uc544\ud0a4\ub294 \ucd9c\uc18c \ud6c4\uc5d0 \ubc14\ub85c \ub3c4\ucfc4\ub85c \uc62c\ub77c\uac00 \uc815\ub144\uc73c\ub85c \ud1f4\uc784\ud560 \ub54c\uae4c\uc9c0 \ud55c \uc778\uc1c4\ud68c\uc0ac \uc5d0\uc11c \uc77c\ud588\ub2e4. \uadf8\ub9ac\uace0 \uc77c\ubcf8\uc5b4 \ud0c0\uc790\uae30(\uc77c\ubcf8\uc5b4: \u548c\u6587\u30bf\u30a4\u30d7\u30e9\u30a4\u30bf\u30fc \uc640\ubaac \ud0c0\uc774\ud478 \ub77c\uc774\ud0c0)\ub97c \uc774\uc6a9\ud55c \uc77c\ubcf8\uc5b4 \uc545\ubcf4 \uc778\uc1c4\uc758 \uc790\ub3d9\ud654\uc5d0 \uc131\uacf5\ud588\ub2e4. \uc790\uc2e0\uc758 \uae30\uc220 \uc219\ub828\uacfc \ud568\uaed8, \uc624\uce74\ucfe0\ub77c \ub374\uc2e0\uc758 \ubca0\ud1a0\ubca4\uc5d0 \ub300\ud55c \uc774\ud574\uc5d0 \uc601\ud5a5\uc744 \ubc1b\uc740 \uac83\uc774\uc5c8\ub2e4. \uc624\uce74\ucfe0\ub77c\ub294 \ubca0\ud1a0\ubca4\uc744 \uc608\uc220\ub85c\uc11c \uc720\uc77c\ud558\uac8c \uc720\ub7fd\uc774 \uc544\uc2dc\uc544\ub97c \uc774\uae30\uac8c \ud55c \uc778\ubb3c\ub85c\uae4c\uc9c0 \ubcf4\uace0 \uc788\uc5c8\ub2e4. \uc774\uac83\uc744 \uc57c\uc2a4\ub2e4 \uc694\uc8fc\ub85c\uc758 \uc800\uc11c \u300a\uba54\uc774\uc9c0\uc758 \uc815\uc2e0\u300b\uc744 \uc77d\uc73c\uba74\uc11c \uc54c\uac8c \ub41c \ud558\uc138\uac00\uc640\ub294 \uc11c\uc591 \uc74c\uc545\uc758 \uc545\ubcf4\ub97c \uc218\uc9d1\ud574\uac14\uc9c0\ub9cc, \uc790\uc2e0\uc774 \uc6d0\ud558\ub294 \uac83\uc744 \uc5bb\uae30\ub294 \ud798\ub4e4\uc5c8\ub2e4. \uacb0\uad6d \ud558\uc138\uac00\uc640\ub294 \uadf8 \ub54c\ubb38\uc5d0 \uc2a4\uc2a4\ub85c \uc545\ubcf4\ub97c \uc870\ud310\ud560 \uc218 \uc5c6\uc744\uae4c \ud558\uace0 \uc0dd\uac01\ud558\uc5ec, \uc545\ubcf4 \uc778\uc1c4\uc758 \uc790\ub3d9\ud654\uc5d0 \ub3c4\uc804\ud558\uc600\ub2e4.", "sentence_answer": "\uc815\ub144\uc73c\ub85c \ud1f4\uc784\ud560 \ub54c\uae4c\uc9c0 \ud55c \uc778\uc1c4\ud68c\uc0ac \uc5d0\uc11c \uc77c\ud588\ub2e4.", "paragraph_id": "6514016-24-0", "paragraph_question": "question: \ud6c4\ubbf8\uc544\ud0a4\uac00 \uc815\ub144\ud1f4\uc784\uae4c\uc9c0 \uc77c\ud55c \uacf3\uc740?, context: \ubd80\ub9ac\ub354 \ud558\uc138\uac00\uc640 \ud6c4\ubbf8\uc544\ud0a4\ub294 \ucd9c\uc18c \ud6c4\uc5d0 \ubc14\ub85c \ub3c4\ucfc4\ub85c \uc62c\ub77c\uac00 \uc815\ub144\uc73c\ub85c \ud1f4\uc784\ud560 \ub54c\uae4c\uc9c0 \ud55c \uc778\uc1c4\ud68c\uc0ac\uc5d0\uc11c \uc77c\ud588\ub2e4. \uadf8\ub9ac\uace0 \uc77c\ubcf8\uc5b4 \ud0c0\uc790\uae30(\uc77c\ubcf8\uc5b4: \u548c\u6587\u30bf\u30a4\u30d7\u30e9\u30a4\u30bf\u30fc \uc640\ubaac \ud0c0\uc774\ud478 \ub77c\uc774\ud0c0)\ub97c \uc774\uc6a9\ud55c \uc77c\ubcf8\uc5b4 \uc545\ubcf4 \uc778\uc1c4\uc758 \uc790\ub3d9\ud654\uc5d0 \uc131\uacf5\ud588\ub2e4. \uc790\uc2e0\uc758 \uae30\uc220 \uc219\ub828\uacfc \ud568\uaed8, \uc624\uce74\ucfe0\ub77c \ub374\uc2e0\uc758 \ubca0\ud1a0\ubca4\uc5d0 \ub300\ud55c \uc774\ud574\uc5d0 \uc601\ud5a5\uc744 \ubc1b\uc740 \uac83\uc774\uc5c8\ub2e4. \uc624\uce74\ucfe0\ub77c\ub294 \ubca0\ud1a0\ubca4\uc744 \uc608\uc220\ub85c\uc11c \uc720\uc77c\ud558\uac8c \uc720\ub7fd\uc774 \uc544\uc2dc\uc544\ub97c \uc774\uae30\uac8c \ud55c \uc778\ubb3c\ub85c\uae4c\uc9c0 \ubcf4\uace0 \uc788\uc5c8\ub2e4. \uc774\uac83\uc744 \uc57c\uc2a4\ub2e4 \uc694\uc8fc\ub85c\uc758 \uc800\uc11c \u300a\uba54\uc774\uc9c0\uc758 \uc815\uc2e0\u300b\uc744 \uc77d\uc73c\uba74\uc11c \uc54c\uac8c \ub41c \ud558\uc138\uac00\uc640\ub294 \uc11c\uc591 \uc74c\uc545\uc758 \uc545\ubcf4\ub97c \uc218\uc9d1\ud574\uac14\uc9c0\ub9cc, \uc790\uc2e0\uc774 \uc6d0\ud558\ub294 \uac83\uc744 \uc5bb\uae30\ub294 \ud798\ub4e4\uc5c8\ub2e4. \uacb0\uad6d \ud558\uc138\uac00\uc640\ub294 \uadf8 \ub54c\ubb38\uc5d0 \uc2a4\uc2a4\ub85c \uc545\ubcf4\ub97c \uc870\ud310\ud560 \uc218 \uc5c6\uc744\uae4c \ud558\uace0 \uc0dd\uac01\ud558\uc5ec, \uc545\ubcf4 \uc778\uc1c4\uc758 \uc790\ub3d9\ud654\uc5d0 \ub3c4\uc804\ud558\uc600\ub2e4."} {"question": "\ud3ed\uc2a4\ubc14\uac90 \ubc30\uae30\uac00\uc2a4 \uc870\uc791 \uc0ac\uac74\uc744 \uc870\uc0ac\ud558\ub294 \ubbf8\uad6d \uc870\uc0ac\uad00\ub4e4\uc758 \ud575\uc2ec \uc218\uc0ac\ub300\uc0c1\uc758 \uc774\ub984\uc740?", "paragraph": "\ubbf8\uad6d\uc758 \uc870\uc0ac\uad00\ub4e4\uc740 \uc2a4\uce94\ub4e4 \ubc94\uc8c4 \ud610\uc758 \uc785\uc99d\uc758 \uc77c\ud658\uc73c\ub85c 150\ub9cc \uac1c\uc758 \uc11c\ub958\ub97c \uac80\ud1a0\ud588\ub2e4. \ud575\uc2ec \uc218\uc0ac\ub300\uc0c1\uc740 \ubbf8\uad6d\uc5d0\uc11c \ud3ed\uc2a4\ubc14\uac90 \ub514\uc824 \ucf64\ud53c\ub358\uc2a4 \ud300\uc744 \uc774\ub044\ub294 \ub3c5\uc77c \uc2dc\ubbfc\uad8c\uc790 \uc81c\uc784\uc2a4 \ub85c\ubc84\ud2b8 \uc591\uc774\uc5c8\ub2e4. \uadf8\ub294 \ub3c5\uc77c\uc5d0\uc11c \uc18c\uc704 '\ud074\ub9b0 \ub514\uc824' \uc5d4\uc9c4\uc744 \uac1c\ubc1c\ud55c \uc5f0\uad6c\uc9c4\uc758 \uc77c\uc6d0\uc774\uc5c8\ub2e4. \ubc95\uc815 \ubb38\uc11c\uc5d0 \ub530\ub974\uba74 \uadf8\uc640 \uadf8 \ub3d9\ub8cc\ub4e4\uc740 \ubbf8\uad6d\uc758 \uc5c4\uaca9\ud55c \ubc30\ucd9c\uac00\uc2a4 \uae30\uc900\uc744 \ucda9\uc871\ud558\ub294 \ub514\uc824 \uc5d4\uc9c4\uc744 \uc124\uacc4\ud560 \uc218 \uc5c6\ub2e4\ub294 \uacb0\ub860\uc744 \ub0b4\ub9ac\uace0 \uadf8 \ub300\uc2e0 \ubc30\ucd9c\uac00\uc2a4 \uac80\uc0ac\ub97c \uc870\uc791\ud558\ub294 \uc18c\ud504\ud2b8\uc6e8\uc5b4\ub97c \uc124\uacc4\ud574\uc11c \uc124\uce58\ud588\ub2e4. 2016\ub144 6\uc6d4\uc5d0 \ud3ed\uc2a4\ubc14\uac90\uc740 \uaddc\uc81c\ub2f9\uad6d\uacfc \uc18c\ube44\uc790\ub4e4\uc5d0\uac8c \ucd5c\uace0 15\uc5b5 \ub2ec\ub7ec\ub97c \ub0b4\ub294 \ubbfc\uc0ac\ud569\uc758\uc5d0 \ub3d9\uc758\ud588\ub2e4. 2016\ub144 8\uc6d4\uc5d0 \ubc1c\ud45c\ub41c \uc774 \ud569\uc758\uc5d0\ub294 652\uac1c\uc5d0 \uc774\ub974\ub294 \ubbf8\uad6d \uc790\ub3d9\ucc28 \ud310\ub9e4\uc0c1\ub3c4 \uad00\ub828\ub418\uc5b4 \uc788\ub2e4. 2016\ub144 6\uc6d4\uc5d0 \uc591\uc740 \uc790\uc2e0\uc774 \uc2a4\uce94\ub4e4\uc5d0\uc11c \ub9e1\uc558\ub358 \uc5ed\ud560\uc5d0 \ub300\ud574 \uc720\uc8c4\ub97c \uc778\uc815\ud558\uace0 \ub2e4\ub978 \ud3ed\uc2a4\ubc14\uac90 \uace0\uc6a9\uc778\ub4e4\uc758 \uc218\uc0ac\uc5d0 \ud611\uc870\ud558\uae30\ub85c \ud55c\ub2e4. \uc591\uc740 \uc5d4\uc9c4 \ud14c\uc2a4\ud2b8 \uacb0\uacfc\uac00 \uac70\uc9d3\uc778 \uac83\uc744 \uc54c\uace0 \uc788\uc73c\uba74\uc11c\ub3c4 \ud658\uacbd\uaddc\uc81c \ub2f9\uad6d\uc758 \uc9c8\ubb38\uc5d0 \uadf8 \ub0b4\uc6a9\uc744 \uac00\uc9c0\uace0 \ub300\ub2f5\ud588\uc5c8\ub358 \uac83\uc73c\ub85c \ubc1d\ud600\uc84c\ub2e4. \ub354 \ub098\uc544\uac00 \uc870\uc0ac\uad00\ub4e4\uc740 \uc591\uc774 \ubbf8\uad6d\uc5d0\uc11c \ubc94\uc8c4\ub97c \uc800\uc9c0\ub974\uace0 \uc788\ub294 \ub3d9\uc548 \uadf8 \uc18c\ud504\ud2b8\uc6e8\uc5b4\uc5d0 \ub300\ud574\uc11c \ub3c5\uc77c\uc758 \ub3d9\ub8cc\ub4e4\uc5d0\uac8c\ub3c4 \uc774\uba54\uc77c\uc744 \ubcf4\ub0c8\uc5c8\ub2e4\uace0 \ud55c\ub2e4. \uc591\uc740 \ucd5c\uace0 5\ub144\uae4c\uc9c0 \ubbf8\uad6d\uc5d0\uc11c \uc9d5\uc5ed\uc744 \uc0b4\uace0\ub09c \ud6c4\uc5d0 \uad6d\uc678\ucd94\ubc29\uc744 \ub2f9\ud558\uac8c \ub418\uba70, \ucd5c\uace0 25\ub9cc \ub2ec\ub7ec\uc758 \ubc8c\uae08\uc744 \ubb3c\uac8c \ub41c\ub2e4.", "answer": "\uc81c\uc784\uc2a4 \ub85c\ubc84\ud2b8 \uc591", "sentence": "\ud575\uc2ec \uc218\uc0ac\ub300\uc0c1\uc740 \ubbf8\uad6d\uc5d0\uc11c \ud3ed\uc2a4\ubc14\uac90 \ub514\uc824 \ucf64\ud53c\ub358\uc2a4 \ud300\uc744 \uc774\ub044\ub294 \ub3c5\uc77c \uc2dc\ubbfc\uad8c\uc790 \uc81c\uc784\uc2a4 \ub85c\ubc84\ud2b8 \uc591 \uc774\uc5c8\ub2e4.", "paragraph_sentence": "\ubbf8\uad6d\uc758 \uc870\uc0ac\uad00\ub4e4\uc740 \uc2a4\uce94\ub4e4 \ubc94\uc8c4 \ud610\uc758 \uc785\uc99d\uc758 \uc77c\ud658\uc73c\ub85c 150\ub9cc \uac1c\uc758 \uc11c\ub958\ub97c \uac80\ud1a0\ud588\ub2e4. \ud575\uc2ec \uc218\uc0ac\ub300\uc0c1\uc740 \ubbf8\uad6d\uc5d0\uc11c \ud3ed\uc2a4\ubc14\uac90 \ub514\uc824 \ucf64\ud53c\ub358\uc2a4 \ud300\uc744 \uc774\ub044\ub294 \ub3c5\uc77c \uc2dc\ubbfc\uad8c\uc790 \uc81c\uc784\uc2a4 \ub85c\ubc84\ud2b8 \uc591 \uc774\uc5c8\ub2e4. \uadf8\ub294 \ub3c5\uc77c\uc5d0\uc11c \uc18c\uc704 '\ud074\ub9b0 \ub514\uc824' \uc5d4\uc9c4\uc744 \uac1c\ubc1c\ud55c \uc5f0\uad6c\uc9c4\uc758 \uc77c\uc6d0\uc774\uc5c8\ub2e4. \ubc95\uc815 \ubb38\uc11c\uc5d0 \ub530\ub974\uba74 \uadf8\uc640 \uadf8 \ub3d9\ub8cc\ub4e4\uc740 \ubbf8\uad6d\uc758 \uc5c4\uaca9\ud55c \ubc30\ucd9c\uac00\uc2a4 \uae30\uc900\uc744 \ucda9\uc871\ud558\ub294 \ub514\uc824 \uc5d4\uc9c4\uc744 \uc124\uacc4\ud560 \uc218 \uc5c6\ub2e4\ub294 \uacb0\ub860\uc744 \ub0b4\ub9ac\uace0 \uadf8 \ub300\uc2e0 \ubc30\ucd9c\uac00\uc2a4 \uac80\uc0ac\ub97c \uc870\uc791\ud558\ub294 \uc18c\ud504\ud2b8\uc6e8\uc5b4\ub97c \uc124\uacc4\ud574\uc11c \uc124\uce58\ud588\ub2e4. 2016\ub144 6\uc6d4\uc5d0 \ud3ed\uc2a4\ubc14\uac90\uc740 \uaddc\uc81c\ub2f9\uad6d\uacfc \uc18c\ube44\uc790\ub4e4\uc5d0\uac8c \ucd5c\uace0 15\uc5b5 \ub2ec\ub7ec\ub97c \ub0b4\ub294 \ubbfc\uc0ac\ud569\uc758\uc5d0 \ub3d9\uc758\ud588\ub2e4. 2016\ub144 8\uc6d4\uc5d0 \ubc1c\ud45c\ub41c \uc774 \ud569\uc758\uc5d0\ub294 652\uac1c\uc5d0 \uc774\ub974\ub294 \ubbf8\uad6d \uc790\ub3d9\ucc28 \ud310\ub9e4\uc0c1\ub3c4 \uad00\ub828\ub418\uc5b4 \uc788\ub2e4. 2016\ub144 6\uc6d4\uc5d0 \uc591\uc740 \uc790\uc2e0\uc774 \uc2a4\uce94\ub4e4\uc5d0\uc11c \ub9e1\uc558\ub358 \uc5ed\ud560\uc5d0 \ub300\ud574 \uc720\uc8c4\ub97c \uc778\uc815\ud558\uace0 \ub2e4\ub978 \ud3ed\uc2a4\ubc14\uac90 \uace0\uc6a9\uc778\ub4e4\uc758 \uc218\uc0ac\uc5d0 \ud611\uc870\ud558\uae30\ub85c \ud55c\ub2e4. \uc591\uc740 \uc5d4\uc9c4 \ud14c\uc2a4\ud2b8 \uacb0\uacfc\uac00 \uac70\uc9d3\uc778 \uac83\uc744 \uc54c\uace0 \uc788\uc73c\uba74\uc11c\ub3c4 \ud658\uacbd\uaddc\uc81c \ub2f9\uad6d\uc758 \uc9c8\ubb38\uc5d0 \uadf8 \ub0b4\uc6a9\uc744 \uac00\uc9c0\uace0 \ub300\ub2f5\ud588\uc5c8\ub358 \uac83\uc73c\ub85c \ubc1d\ud600\uc84c\ub2e4. \ub354 \ub098\uc544\uac00 \uc870\uc0ac\uad00\ub4e4\uc740 \uc591\uc774 \ubbf8\uad6d\uc5d0\uc11c \ubc94\uc8c4\ub97c \uc800\uc9c0\ub974\uace0 \uc788\ub294 \ub3d9\uc548 \uadf8 \uc18c\ud504\ud2b8\uc6e8\uc5b4\uc5d0 \ub300\ud574\uc11c \ub3c5\uc77c\uc758 \ub3d9\ub8cc\ub4e4\uc5d0\uac8c\ub3c4 \uc774\uba54\uc77c\uc744 \ubcf4\ub0c8\uc5c8\ub2e4\uace0 \ud55c\ub2e4. \uc591\uc740 \ucd5c\uace0 5\ub144\uae4c\uc9c0 \ubbf8\uad6d\uc5d0\uc11c \uc9d5\uc5ed\uc744 \uc0b4\uace0\ub09c \ud6c4\uc5d0 \uad6d\uc678\ucd94\ubc29\uc744 \ub2f9\ud558\uac8c \ub418\uba70, \ucd5c\uace0 25\ub9cc \ub2ec\ub7ec\uc758 \ubc8c\uae08\uc744 \ubb3c\uac8c \ub41c\ub2e4.", "paragraph_answer": "\ubbf8\uad6d\uc758 \uc870\uc0ac\uad00\ub4e4\uc740 \uc2a4\uce94\ub4e4 \ubc94\uc8c4 \ud610\uc758 \uc785\uc99d\uc758 \uc77c\ud658\uc73c\ub85c 150\ub9cc \uac1c\uc758 \uc11c\ub958\ub97c \uac80\ud1a0\ud588\ub2e4. \ud575\uc2ec \uc218\uc0ac\ub300\uc0c1\uc740 \ubbf8\uad6d\uc5d0\uc11c \ud3ed\uc2a4\ubc14\uac90 \ub514\uc824 \ucf64\ud53c\ub358\uc2a4 \ud300\uc744 \uc774\ub044\ub294 \ub3c5\uc77c \uc2dc\ubbfc\uad8c\uc790 \uc81c\uc784\uc2a4 \ub85c\ubc84\ud2b8 \uc591 \uc774\uc5c8\ub2e4. \uadf8\ub294 \ub3c5\uc77c\uc5d0\uc11c \uc18c\uc704 '\ud074\ub9b0 \ub514\uc824' \uc5d4\uc9c4\uc744 \uac1c\ubc1c\ud55c \uc5f0\uad6c\uc9c4\uc758 \uc77c\uc6d0\uc774\uc5c8\ub2e4. \ubc95\uc815 \ubb38\uc11c\uc5d0 \ub530\ub974\uba74 \uadf8\uc640 \uadf8 \ub3d9\ub8cc\ub4e4\uc740 \ubbf8\uad6d\uc758 \uc5c4\uaca9\ud55c \ubc30\ucd9c\uac00\uc2a4 \uae30\uc900\uc744 \ucda9\uc871\ud558\ub294 \ub514\uc824 \uc5d4\uc9c4\uc744 \uc124\uacc4\ud560 \uc218 \uc5c6\ub2e4\ub294 \uacb0\ub860\uc744 \ub0b4\ub9ac\uace0 \uadf8 \ub300\uc2e0 \ubc30\ucd9c\uac00\uc2a4 \uac80\uc0ac\ub97c \uc870\uc791\ud558\ub294 \uc18c\ud504\ud2b8\uc6e8\uc5b4\ub97c \uc124\uacc4\ud574\uc11c \uc124\uce58\ud588\ub2e4. 2016\ub144 6\uc6d4\uc5d0 \ud3ed\uc2a4\ubc14\uac90\uc740 \uaddc\uc81c\ub2f9\uad6d\uacfc \uc18c\ube44\uc790\ub4e4\uc5d0\uac8c \ucd5c\uace0 15\uc5b5 \ub2ec\ub7ec\ub97c \ub0b4\ub294 \ubbfc\uc0ac\ud569\uc758\uc5d0 \ub3d9\uc758\ud588\ub2e4. 2016\ub144 8\uc6d4\uc5d0 \ubc1c\ud45c\ub41c \uc774 \ud569\uc758\uc5d0\ub294 652\uac1c\uc5d0 \uc774\ub974\ub294 \ubbf8\uad6d \uc790\ub3d9\ucc28 \ud310\ub9e4\uc0c1\ub3c4 \uad00\ub828\ub418\uc5b4 \uc788\ub2e4. 2016\ub144 6\uc6d4\uc5d0 \uc591\uc740 \uc790\uc2e0\uc774 \uc2a4\uce94\ub4e4\uc5d0\uc11c \ub9e1\uc558\ub358 \uc5ed\ud560\uc5d0 \ub300\ud574 \uc720\uc8c4\ub97c \uc778\uc815\ud558\uace0 \ub2e4\ub978 \ud3ed\uc2a4\ubc14\uac90 \uace0\uc6a9\uc778\ub4e4\uc758 \uc218\uc0ac\uc5d0 \ud611\uc870\ud558\uae30\ub85c \ud55c\ub2e4. \uc591\uc740 \uc5d4\uc9c4 \ud14c\uc2a4\ud2b8 \uacb0\uacfc\uac00 \uac70\uc9d3\uc778 \uac83\uc744 \uc54c\uace0 \uc788\uc73c\uba74\uc11c\ub3c4 \ud658\uacbd\uaddc\uc81c \ub2f9\uad6d\uc758 \uc9c8\ubb38\uc5d0 \uadf8 \ub0b4\uc6a9\uc744 \uac00\uc9c0\uace0 \ub300\ub2f5\ud588\uc5c8\ub358 \uac83\uc73c\ub85c \ubc1d\ud600\uc84c\ub2e4. \ub354 \ub098\uc544\uac00 \uc870\uc0ac\uad00\ub4e4\uc740 \uc591\uc774 \ubbf8\uad6d\uc5d0\uc11c \ubc94\uc8c4\ub97c \uc800\uc9c0\ub974\uace0 \uc788\ub294 \ub3d9\uc548 \uadf8 \uc18c\ud504\ud2b8\uc6e8\uc5b4\uc5d0 \ub300\ud574\uc11c \ub3c5\uc77c\uc758 \ub3d9\ub8cc\ub4e4\uc5d0\uac8c\ub3c4 \uc774\uba54\uc77c\uc744 \ubcf4\ub0c8\uc5c8\ub2e4\uace0 \ud55c\ub2e4. \uc591\uc740 \ucd5c\uace0 5\ub144\uae4c\uc9c0 \ubbf8\uad6d\uc5d0\uc11c \uc9d5\uc5ed\uc744 \uc0b4\uace0\ub09c \ud6c4\uc5d0 \uad6d\uc678\ucd94\ubc29\uc744 \ub2f9\ud558\uac8c \ub418\uba70, \ucd5c\uace0 25\ub9cc \ub2ec\ub7ec\uc758 \ubc8c\uae08\uc744 \ubb3c\uac8c \ub41c\ub2e4.", "sentence_answer": "\ud575\uc2ec \uc218\uc0ac\ub300\uc0c1\uc740 \ubbf8\uad6d\uc5d0\uc11c \ud3ed\uc2a4\ubc14\uac90 \ub514\uc824 \ucf64\ud53c\ub358\uc2a4 \ud300\uc744 \uc774\ub044\ub294 \ub3c5\uc77c \uc2dc\ubbfc\uad8c\uc790 \uc81c\uc784\uc2a4 \ub85c\ubc84\ud2b8 \uc591 \uc774\uc5c8\ub2e4.", "paragraph_id": "5922246-2-1", "paragraph_question": "question: \ud3ed\uc2a4\ubc14\uac90 \ubc30\uae30\uac00\uc2a4 \uc870\uc791 \uc0ac\uac74\uc744 \uc870\uc0ac\ud558\ub294 \ubbf8\uad6d \uc870\uc0ac\uad00\ub4e4\uc758 \ud575\uc2ec \uc218\uc0ac\ub300\uc0c1\uc758 \uc774\ub984\uc740?, context: \ubbf8\uad6d\uc758 \uc870\uc0ac\uad00\ub4e4\uc740 \uc2a4\uce94\ub4e4 \ubc94\uc8c4 \ud610\uc758 \uc785\uc99d\uc758 \uc77c\ud658\uc73c\ub85c 150\ub9cc \uac1c\uc758 \uc11c\ub958\ub97c \uac80\ud1a0\ud588\ub2e4. \ud575\uc2ec \uc218\uc0ac\ub300\uc0c1\uc740 \ubbf8\uad6d\uc5d0\uc11c \ud3ed\uc2a4\ubc14\uac90 \ub514\uc824 \ucf64\ud53c\ub358\uc2a4 \ud300\uc744 \uc774\ub044\ub294 \ub3c5\uc77c \uc2dc\ubbfc\uad8c\uc790 \uc81c\uc784\uc2a4 \ub85c\ubc84\ud2b8 \uc591\uc774\uc5c8\ub2e4. \uadf8\ub294 \ub3c5\uc77c\uc5d0\uc11c \uc18c\uc704 '\ud074\ub9b0 \ub514\uc824' \uc5d4\uc9c4\uc744 \uac1c\ubc1c\ud55c \uc5f0\uad6c\uc9c4\uc758 \uc77c\uc6d0\uc774\uc5c8\ub2e4. \ubc95\uc815 \ubb38\uc11c\uc5d0 \ub530\ub974\uba74 \uadf8\uc640 \uadf8 \ub3d9\ub8cc\ub4e4\uc740 \ubbf8\uad6d\uc758 \uc5c4\uaca9\ud55c \ubc30\ucd9c\uac00\uc2a4 \uae30\uc900\uc744 \ucda9\uc871\ud558\ub294 \ub514\uc824 \uc5d4\uc9c4\uc744 \uc124\uacc4\ud560 \uc218 \uc5c6\ub2e4\ub294 \uacb0\ub860\uc744 \ub0b4\ub9ac\uace0 \uadf8 \ub300\uc2e0 \ubc30\ucd9c\uac00\uc2a4 \uac80\uc0ac\ub97c \uc870\uc791\ud558\ub294 \uc18c\ud504\ud2b8\uc6e8\uc5b4\ub97c \uc124\uacc4\ud574\uc11c \uc124\uce58\ud588\ub2e4. 2016\ub144 6\uc6d4\uc5d0 \ud3ed\uc2a4\ubc14\uac90\uc740 \uaddc\uc81c\ub2f9\uad6d\uacfc \uc18c\ube44\uc790\ub4e4\uc5d0\uac8c \ucd5c\uace0 15\uc5b5 \ub2ec\ub7ec\ub97c \ub0b4\ub294 \ubbfc\uc0ac\ud569\uc758\uc5d0 \ub3d9\uc758\ud588\ub2e4. 2016\ub144 8\uc6d4\uc5d0 \ubc1c\ud45c\ub41c \uc774 \ud569\uc758\uc5d0\ub294 652\uac1c\uc5d0 \uc774\ub974\ub294 \ubbf8\uad6d \uc790\ub3d9\ucc28 \ud310\ub9e4\uc0c1\ub3c4 \uad00\ub828\ub418\uc5b4 \uc788\ub2e4. 2016\ub144 6\uc6d4\uc5d0 \uc591\uc740 \uc790\uc2e0\uc774 \uc2a4\uce94\ub4e4\uc5d0\uc11c \ub9e1\uc558\ub358 \uc5ed\ud560\uc5d0 \ub300\ud574 \uc720\uc8c4\ub97c \uc778\uc815\ud558\uace0 \ub2e4\ub978 \ud3ed\uc2a4\ubc14\uac90 \uace0\uc6a9\uc778\ub4e4\uc758 \uc218\uc0ac\uc5d0 \ud611\uc870\ud558\uae30\ub85c \ud55c\ub2e4. \uc591\uc740 \uc5d4\uc9c4 \ud14c\uc2a4\ud2b8 \uacb0\uacfc\uac00 \uac70\uc9d3\uc778 \uac83\uc744 \uc54c\uace0 \uc788\uc73c\uba74\uc11c\ub3c4 \ud658\uacbd\uaddc\uc81c \ub2f9\uad6d\uc758 \uc9c8\ubb38\uc5d0 \uadf8 \ub0b4\uc6a9\uc744 \uac00\uc9c0\uace0 \ub300\ub2f5\ud588\uc5c8\ub358 \uac83\uc73c\ub85c \ubc1d\ud600\uc84c\ub2e4. \ub354 \ub098\uc544\uac00 \uc870\uc0ac\uad00\ub4e4\uc740 \uc591\uc774 \ubbf8\uad6d\uc5d0\uc11c \ubc94\uc8c4\ub97c \uc800\uc9c0\ub974\uace0 \uc788\ub294 \ub3d9\uc548 \uadf8 \uc18c\ud504\ud2b8\uc6e8\uc5b4\uc5d0 \ub300\ud574\uc11c \ub3c5\uc77c\uc758 \ub3d9\ub8cc\ub4e4\uc5d0\uac8c\ub3c4 \uc774\uba54\uc77c\uc744 \ubcf4\ub0c8\uc5c8\ub2e4\uace0 \ud55c\ub2e4. \uc591\uc740 \ucd5c\uace0 5\ub144\uae4c\uc9c0 \ubbf8\uad6d\uc5d0\uc11c \uc9d5\uc5ed\uc744 \uc0b4\uace0\ub09c \ud6c4\uc5d0 \uad6d\uc678\ucd94\ubc29\uc744 \ub2f9\ud558\uac8c \ub418\uba70, \ucd5c\uace0 25\ub9cc \ub2ec\ub7ec\uc758 \ubc8c\uae08\uc744 \ubb3c\uac8c \ub41c\ub2e4."} {"question": "\ud0b9 \ub300\uc704\uc640 \ud328\ub2cc \uc6b0\ub808\uc544 \uc7a5\uad70\uc758 \uc120\ubc1c\ubd80\ub300\uac00 \ub9cc\ub09c \uacf3\uc740 \uc5b4\ub514\uc778\uac00?", "paragraph": "3\uc6d4 12\uc77c, \ud328\ub2cc\uc740 \ub808\ud478\uc9c0\uc624\ub85c \ub9c8\ucc28\ub97c \uac00\uc838\uac00\uae30 \uc704\ud574 \uc5d0\uc774\ubabd \ubc84\ud2c0\ub7ec \ud0b9 \ub300\uc704\uc640 \uc57d 28\uba85\uc758 \ubcd1\uc0ac\ub97c \ud30c\uacac\ud558\uc5ec \uac70\uae30\uc5d0 \ub0a8\uc544 \uc788\ub358 \uac00\uc871\uc744 \ucca0\uc218\ud558\ub3c4\ub85d \ub3c4\uc654\ub2e4. \ud0b9\uacfc \uadf8\uc758 \ubcd1\uc0ac\ub294 \ub808\ud478\uc9c0\uc624\uc5d0\uc11c \uc6b0\ub808\uc544 \uc7a5\uad70\uc758 \uae30\ub9c8\ub300 \uc911 \uc120\ubc1c\ubd80\ub300\ub97c \ub9cc\ub098 \uc61b\uc804\ub3c4\uc18c\uac00 \uc788\ub294 \uacf3\uc73c\ub85c \ucca0\uc218\ud588\ub2e4. \uadf8 \uc9c0\uc5ed\uc758 \uc18c\ub144\uc774 \ub9c8\uc744\uc5d0\uc11c \uac04\uc2e0\ud788 \ub3c4\ub9dd\uccd0 \ud328\ub2cc\uc5d0\uac8c \uc791\uc740 \ucda9\ub3cc\uc744 \uacbd\uace0\ud588\ub2e4. \ud328\ub2cc\uc740 \uc70c\ub9ac\uc5c4 \uc6cc\ub4dc \uc911\ub839\uacfc 120 \uba85\uc758 \ubcd1\uc0ac\ub97c \uad6c\ucd9c\ud558\uae30 \uc704\ud574 \ud0b9\uc73c\ub85c \ud5a5\ud588\ub2e4. \uc6cc\ub4dc\ub294 \uba55\uc2dc\ucf54\uad70 \uc18c\ubd80\ub300\ub97c \uc798 \uaca9\ud1f4\ud558\uace0, \uad70\uc778\uc744 \ud734\uc2dd\ud558\uae30 \uc704\ud558\uc5ec 1\ubc15\ud558\uae30\ub85c \ud588\ub2e4. 1836\ub144 3\uc6d4 14\uc77c, \uc6cc\ub4dc\uc640 \ud0b9\uc740 \uc6b0\ub808\uc544 \uc7a5\uad70\uacfc 200\uba85 \uc774\uc0c1\uc758 \uba55\uc2dc\ucf54 \uad70\uc758 \uc2b5\uaca9\uc744 \ubc1b\uc558\ub2e4. \uac19\uc740 \ub0a0 \ud734\uc2a4\ud134 \uc7a5\uad70\uc740 \ud328\ub2cc\uc5d0\uac8c \ube45\ud1a0\ub9ac\uc544\ub85c \ucca0\uc218\ud558\ub77c\uace0 \uba85\ub839\ud588\ub2e4.", "answer": "\ub808\ud478\uc9c0\uc624", "sentence": "3\uc6d4 12\uc77c, \ud328\ub2cc\uc740 \ub808\ud478\uc9c0\uc624 \ub85c \ub9c8\ucc28\ub97c \uac00\uc838\uac00\uae30 \uc704\ud574 \uc5d0\uc774\ubabd \ubc84\ud2c0\ub7ec \ud0b9 \ub300\uc704\uc640 \uc57d 28\uba85\uc758 \ubcd1\uc0ac\ub97c \ud30c\uacac\ud558\uc5ec \uac70\uae30\uc5d0 \ub0a8\uc544 \uc788\ub358 \uac00\uc871\uc744 \ucca0\uc218\ud558\ub3c4\ub85d \ub3c4\uc654\ub2e4.", "paragraph_sentence": " 3\uc6d4 12\uc77c, \ud328\ub2cc\uc740 \ub808\ud478\uc9c0\uc624 \ub85c \ub9c8\ucc28\ub97c \uac00\uc838\uac00\uae30 \uc704\ud574 \uc5d0\uc774\ubabd \ubc84\ud2c0\ub7ec \ud0b9 \ub300\uc704\uc640 \uc57d 28\uba85\uc758 \ubcd1\uc0ac\ub97c \ud30c\uacac\ud558\uc5ec \uac70\uae30\uc5d0 \ub0a8\uc544 \uc788\ub358 \uac00\uc871\uc744 \ucca0\uc218\ud558\ub3c4\ub85d \ub3c4\uc654\ub2e4. \ud0b9\uacfc \uadf8\uc758 \ubcd1\uc0ac\ub294 \ub808\ud478\uc9c0\uc624\uc5d0\uc11c \uc6b0\ub808\uc544 \uc7a5\uad70\uc758 \uae30\ub9c8\ub300 \uc911 \uc120\ubc1c\ubd80\ub300\ub97c \ub9cc\ub098 \uc61b\uc804\ub3c4\uc18c\uac00 \uc788\ub294 \uacf3\uc73c\ub85c \ucca0\uc218\ud588\ub2e4. \uadf8 \uc9c0\uc5ed\uc758 \uc18c\ub144\uc774 \ub9c8\uc744\uc5d0\uc11c \uac04\uc2e0\ud788 \ub3c4\ub9dd\uccd0 \ud328\ub2cc\uc5d0\uac8c \uc791\uc740 \ucda9\ub3cc\uc744 \uacbd\uace0\ud588\ub2e4. \ud328\ub2cc\uc740 \uc70c\ub9ac\uc5c4 \uc6cc\ub4dc \uc911\ub839\uacfc 120 \uba85\uc758 \ubcd1\uc0ac\ub97c \uad6c\ucd9c\ud558\uae30 \uc704\ud574 \ud0b9\uc73c\ub85c \ud5a5\ud588\ub2e4. \uc6cc\ub4dc\ub294 \uba55\uc2dc\ucf54\uad70 \uc18c\ubd80\ub300\ub97c \uc798 \uaca9\ud1f4\ud558\uace0, \uad70\uc778\uc744 \ud734\uc2dd\ud558\uae30 \uc704\ud558\uc5ec 1\ubc15\ud558\uae30\ub85c \ud588\ub2e4. 1836\ub144 3\uc6d4 14\uc77c, \uc6cc\ub4dc\uc640 \ud0b9\uc740 \uc6b0\ub808\uc544 \uc7a5\uad70\uacfc 200\uba85 \uc774\uc0c1\uc758 \uba55\uc2dc\ucf54 \uad70\uc758 \uc2b5\uaca9\uc744 \ubc1b\uc558\ub2e4. \uac19\uc740 \ub0a0 \ud734\uc2a4\ud134 \uc7a5\uad70\uc740 \ud328\ub2cc\uc5d0\uac8c \ube45\ud1a0\ub9ac\uc544\ub85c \ucca0\uc218\ud558\ub77c\uace0 \uba85\ub839\ud588\ub2e4.", "paragraph_answer": "3\uc6d4 12\uc77c, \ud328\ub2cc\uc740 \ub808\ud478\uc9c0\uc624 \ub85c \ub9c8\ucc28\ub97c \uac00\uc838\uac00\uae30 \uc704\ud574 \uc5d0\uc774\ubabd \ubc84\ud2c0\ub7ec \ud0b9 \ub300\uc704\uc640 \uc57d 28\uba85\uc758 \ubcd1\uc0ac\ub97c \ud30c\uacac\ud558\uc5ec \uac70\uae30\uc5d0 \ub0a8\uc544 \uc788\ub358 \uac00\uc871\uc744 \ucca0\uc218\ud558\ub3c4\ub85d \ub3c4\uc654\ub2e4. \ud0b9\uacfc \uadf8\uc758 \ubcd1\uc0ac\ub294 \ub808\ud478\uc9c0\uc624\uc5d0\uc11c \uc6b0\ub808\uc544 \uc7a5\uad70\uc758 \uae30\ub9c8\ub300 \uc911 \uc120\ubc1c\ubd80\ub300\ub97c \ub9cc\ub098 \uc61b\uc804\ub3c4\uc18c\uac00 \uc788\ub294 \uacf3\uc73c\ub85c \ucca0\uc218\ud588\ub2e4. \uadf8 \uc9c0\uc5ed\uc758 \uc18c\ub144\uc774 \ub9c8\uc744\uc5d0\uc11c \uac04\uc2e0\ud788 \ub3c4\ub9dd\uccd0 \ud328\ub2cc\uc5d0\uac8c \uc791\uc740 \ucda9\ub3cc\uc744 \uacbd\uace0\ud588\ub2e4. \ud328\ub2cc\uc740 \uc70c\ub9ac\uc5c4 \uc6cc\ub4dc \uc911\ub839\uacfc 120 \uba85\uc758 \ubcd1\uc0ac\ub97c \uad6c\ucd9c\ud558\uae30 \uc704\ud574 \ud0b9\uc73c\ub85c \ud5a5\ud588\ub2e4. \uc6cc\ub4dc\ub294 \uba55\uc2dc\ucf54\uad70 \uc18c\ubd80\ub300\ub97c \uc798 \uaca9\ud1f4\ud558\uace0, \uad70\uc778\uc744 \ud734\uc2dd\ud558\uae30 \uc704\ud558\uc5ec 1\ubc15\ud558\uae30\ub85c \ud588\ub2e4. 1836\ub144 3\uc6d4 14\uc77c, \uc6cc\ub4dc\uc640 \ud0b9\uc740 \uc6b0\ub808\uc544 \uc7a5\uad70\uacfc 200\uba85 \uc774\uc0c1\uc758 \uba55\uc2dc\ucf54 \uad70\uc758 \uc2b5\uaca9\uc744 \ubc1b\uc558\ub2e4. \uac19\uc740 \ub0a0 \ud734\uc2a4\ud134 \uc7a5\uad70\uc740 \ud328\ub2cc\uc5d0\uac8c \ube45\ud1a0\ub9ac\uc544\ub85c \ucca0\uc218\ud558\ub77c\uace0 \uba85\ub839\ud588\ub2e4.", "sentence_answer": "3\uc6d4 12\uc77c, \ud328\ub2cc\uc740 \ub808\ud478\uc9c0\uc624 \ub85c \ub9c8\ucc28\ub97c \uac00\uc838\uac00\uae30 \uc704\ud574 \uc5d0\uc774\ubabd \ubc84\ud2c0\ub7ec \ud0b9 \ub300\uc704\uc640 \uc57d 28\uba85\uc758 \ubcd1\uc0ac\ub97c \ud30c\uacac\ud558\uc5ec \uac70\uae30\uc5d0 \ub0a8\uc544 \uc788\ub358 \uac00\uc871\uc744 \ucca0\uc218\ud558\ub3c4\ub85d \ub3c4\uc654\ub2e4.", "paragraph_id": "6550882-1-0", "paragraph_question": "question: \ud0b9 \ub300\uc704\uc640 \ud328\ub2cc \uc6b0\ub808\uc544 \uc7a5\uad70\uc758 \uc120\ubc1c\ubd80\ub300\uac00 \ub9cc\ub09c \uacf3\uc740 \uc5b4\ub514\uc778\uac00?, context: 3\uc6d4 12\uc77c, \ud328\ub2cc\uc740 \ub808\ud478\uc9c0\uc624\ub85c \ub9c8\ucc28\ub97c \uac00\uc838\uac00\uae30 \uc704\ud574 \uc5d0\uc774\ubabd \ubc84\ud2c0\ub7ec \ud0b9 \ub300\uc704\uc640 \uc57d 28\uba85\uc758 \ubcd1\uc0ac\ub97c \ud30c\uacac\ud558\uc5ec \uac70\uae30\uc5d0 \ub0a8\uc544 \uc788\ub358 \uac00\uc871\uc744 \ucca0\uc218\ud558\ub3c4\ub85d \ub3c4\uc654\ub2e4. \ud0b9\uacfc \uadf8\uc758 \ubcd1\uc0ac\ub294 \ub808\ud478\uc9c0\uc624\uc5d0\uc11c \uc6b0\ub808\uc544 \uc7a5\uad70\uc758 \uae30\ub9c8\ub300 \uc911 \uc120\ubc1c\ubd80\ub300\ub97c \ub9cc\ub098 \uc61b\uc804\ub3c4\uc18c\uac00 \uc788\ub294 \uacf3\uc73c\ub85c \ucca0\uc218\ud588\ub2e4. \uadf8 \uc9c0\uc5ed\uc758 \uc18c\ub144\uc774 \ub9c8\uc744\uc5d0\uc11c \uac04\uc2e0\ud788 \ub3c4\ub9dd\uccd0 \ud328\ub2cc\uc5d0\uac8c \uc791\uc740 \ucda9\ub3cc\uc744 \uacbd\uace0\ud588\ub2e4. \ud328\ub2cc\uc740 \uc70c\ub9ac\uc5c4 \uc6cc\ub4dc \uc911\ub839\uacfc 120 \uba85\uc758 \ubcd1\uc0ac\ub97c \uad6c\ucd9c\ud558\uae30 \uc704\ud574 \ud0b9\uc73c\ub85c \ud5a5\ud588\ub2e4. \uc6cc\ub4dc\ub294 \uba55\uc2dc\ucf54\uad70 \uc18c\ubd80\ub300\ub97c \uc798 \uaca9\ud1f4\ud558\uace0, \uad70\uc778\uc744 \ud734\uc2dd\ud558\uae30 \uc704\ud558\uc5ec 1\ubc15\ud558\uae30\ub85c \ud588\ub2e4. 1836\ub144 3\uc6d4 14\uc77c, \uc6cc\ub4dc\uc640 \ud0b9\uc740 \uc6b0\ub808\uc544 \uc7a5\uad70\uacfc 200\uba85 \uc774\uc0c1\uc758 \uba55\uc2dc\ucf54 \uad70\uc758 \uc2b5\uaca9\uc744 \ubc1b\uc558\ub2e4. \uac19\uc740 \ub0a0 \ud734\uc2a4\ud134 \uc7a5\uad70\uc740 \ud328\ub2cc\uc5d0\uac8c \ube45\ud1a0\ub9ac\uc544\ub85c \ucca0\uc218\ud558\ub77c\uace0 \uba85\ub839\ud588\ub2e4."} {"question": "\ub9c8\uc624\uca4c\ub465\uc774 \uad6d\uac00\uc8fc\uc11d\uc9c1\uc5d0\uc11c \ubb3c\ub7ec\ub09c \ud574\ub294?", "paragraph": "\ub9c8\uc624\uca4c\ub465\uc774 \uc911\uad6d \ud604\ub300\uc0ac\uc5d0 \ud070 \uc601\ud5a5\uc744 \ubbf8\uce5c \ub370\uc5d0\ub294 \uc774\uacac\uc774 \uc5c6\uc73c\ub098, \uc624\ub298\ub0a0\uae4c\uc9c0\ub3c4 \uadf8\uc5d0 \ub300\ud55c \ud3c9\uac00\ub294 \uc544\uc9c1\ub3c4 \ub17c\uc7c1\uc774 \ubd84\ubd84\ud558\ub2e4. \uc77c\uac01\uc5d0\uc11c\ub294 \uc7a5\uc81c\uc2a4\ub97c \uaca9\ud30c\ud558\uace0 \uc911\uad6d \ub300\ub959\uc5d0 \uacf5\uc0b0\uc8fc\uc758 \uad6d\uac00\ub97c \uac74\uc124\ud55c \ud601\uba85\uac00\uc774\uc790 \uc804\ub7b5\uac00\ub85c \ud3c9\uac00\ud558\uace0 \uc788\uc9c0\ub9cc, \ub300\uc57d\uc9c4 \uc6b4\ub3d9 \uc2e4\ud328\uc640 \ubb38\ud654\ub300\ud601\uba85\uc73c\ub85c \uc57d 2\ucc9c 5\ubc31\ub9cc \uba85\uc758 \uc778\uba85 \ud53c\ud574\ub97c \ucd08\ub798\ud558\uc600\ub2e4. \ub300\uc57d\uc9c4 \uc6b4\ub3d9\uacfc \ubb38\ud654 \ub300\ud601\uba85\uc5d0 \uc774\ub974\uae30\uae4c\uc9c0 \uacc4\uc18d\ub41c \uae09\uc9c4\uc801\uc778 \uc815\ucc45\uc774 \uc911\uad6d\uc758 \ubb38\ud654, \uc0ac\ud68c, \uacbd\uc81c, \uc678\uad50\uad00\uacc4\uc5d0 \uc785\ud78c \ubb3c\uc801, \uc778\uc801, \ubb38\ud654\uc801 \ud53c\ud574\ub294 \uc218\uce58\ub85c \uacc4\uc0b0\uc774 \ud798\ub4e4 \uc815\ub3c4\ub2e4. 1931\ub144 \uc774\ud6c4 \uc911\uad6d \uacf5\uc0b0\ub2f9\uc758 \uc9c0\ub3c4\uc801 \uc5ed\ud560\uc744 \ub9e1\uc544 \uc654\uc73c\uba70, 1949\ub144\ubd80\ud130 1959\ub144\uae4c\uc9c0 \uc911\ud654\uc778\ubbfc\uacf5\ud654\uad6d\uc758 \uad6d\uac00\uc8fc\uc11d\uc744 \uc9c0\ub0c8\ub2e4. \uad6d\uac00\uc8fc\uc11d \uc9c1\uc5d0\uc11c \ubb3c\ub7ec\ub09c \ud6c4\uc5d0\ub3c4 \uc911\uad6d \uacf5\uc0b0\ub2f9 \uc8fc\uc11d\uc5d0 \uc804\uc784\ud558\uba70 \uc0ac\ub9dd\ud560 \ub54c\uae4c\uc9c0 \uc2e4\uad8c\uc744 \ud589\uc0ac\ud588\ub2e4.", "answer": "1959\ub144", "sentence": "1949\ub144\ubd80\ud130 1959\ub144 \uae4c\uc9c0 \uc911\ud654\uc778\ubbfc\uacf5\ud654\uad6d\uc758 \uad6d\uac00\uc8fc\uc11d\uc744 \uc9c0\ub0c8\ub2e4.", "paragraph_sentence": "\ub9c8\uc624\uca4c\ub465\uc774 \uc911\uad6d \ud604\ub300\uc0ac\uc5d0 \ud070 \uc601\ud5a5\uc744 \ubbf8\uce5c \ub370\uc5d0\ub294 \uc774\uacac\uc774 \uc5c6\uc73c\ub098, \uc624\ub298\ub0a0\uae4c\uc9c0\ub3c4 \uadf8\uc5d0 \ub300\ud55c \ud3c9\uac00\ub294 \uc544\uc9c1\ub3c4 \ub17c\uc7c1\uc774 \ubd84\ubd84\ud558\ub2e4. \uc77c\uac01\uc5d0\uc11c\ub294 \uc7a5\uc81c\uc2a4\ub97c \uaca9\ud30c\ud558\uace0 \uc911\uad6d \ub300\ub959\uc5d0 \uacf5\uc0b0\uc8fc\uc758 \uad6d\uac00\ub97c \uac74\uc124\ud55c \ud601\uba85\uac00\uc774\uc790 \uc804\ub7b5\uac00\ub85c \ud3c9\uac00\ud558\uace0 \uc788\uc9c0\ub9cc, \ub300\uc57d\uc9c4 \uc6b4\ub3d9 \uc2e4\ud328\uc640 \ubb38\ud654\ub300\ud601\uba85\uc73c\ub85c \uc57d 2\ucc9c 5\ubc31\ub9cc \uba85\uc758 \uc778\uba85 \ud53c\ud574\ub97c \ucd08\ub798\ud558\uc600\ub2e4. \ub300\uc57d\uc9c4 \uc6b4\ub3d9\uacfc \ubb38\ud654 \ub300\ud601\uba85\uc5d0 \uc774\ub974\uae30\uae4c\uc9c0 \uacc4\uc18d\ub41c \uae09\uc9c4\uc801\uc778 \uc815\ucc45\uc774 \uc911\uad6d\uc758 \ubb38\ud654, \uc0ac\ud68c, \uacbd\uc81c, \uc678\uad50\uad00\uacc4\uc5d0 \uc785\ud78c \ubb3c\uc801, \uc778\uc801, \ubb38\ud654\uc801 \ud53c\ud574\ub294 \uc218\uce58\ub85c \uacc4\uc0b0\uc774 \ud798\ub4e4 \uc815\ub3c4\ub2e4. 1931\ub144 \uc774\ud6c4 \uc911\uad6d \uacf5\uc0b0\ub2f9\uc758 \uc9c0\ub3c4\uc801 \uc5ed\ud560\uc744 \ub9e1\uc544 \uc654\uc73c\uba70, 1949\ub144\ubd80\ud130 1959\ub144 \uae4c\uc9c0 \uc911\ud654\uc778\ubbfc\uacf5\ud654\uad6d\uc758 \uad6d\uac00\uc8fc\uc11d\uc744 \uc9c0\ub0c8\ub2e4. \uad6d\uac00\uc8fc\uc11d \uc9c1\uc5d0\uc11c \ubb3c\ub7ec\ub09c \ud6c4\uc5d0\ub3c4 \uc911\uad6d \uacf5\uc0b0\ub2f9 \uc8fc\uc11d\uc5d0 \uc804\uc784\ud558\uba70 \uc0ac\ub9dd\ud560 \ub54c\uae4c\uc9c0 \uc2e4\uad8c\uc744 \ud589\uc0ac\ud588\ub2e4.", "paragraph_answer": "\ub9c8\uc624\uca4c\ub465\uc774 \uc911\uad6d \ud604\ub300\uc0ac\uc5d0 \ud070 \uc601\ud5a5\uc744 \ubbf8\uce5c \ub370\uc5d0\ub294 \uc774\uacac\uc774 \uc5c6\uc73c\ub098, \uc624\ub298\ub0a0\uae4c\uc9c0\ub3c4 \uadf8\uc5d0 \ub300\ud55c \ud3c9\uac00\ub294 \uc544\uc9c1\ub3c4 \ub17c\uc7c1\uc774 \ubd84\ubd84\ud558\ub2e4. \uc77c\uac01\uc5d0\uc11c\ub294 \uc7a5\uc81c\uc2a4\ub97c \uaca9\ud30c\ud558\uace0 \uc911\uad6d \ub300\ub959\uc5d0 \uacf5\uc0b0\uc8fc\uc758 \uad6d\uac00\ub97c \uac74\uc124\ud55c \ud601\uba85\uac00\uc774\uc790 \uc804\ub7b5\uac00\ub85c \ud3c9\uac00\ud558\uace0 \uc788\uc9c0\ub9cc, \ub300\uc57d\uc9c4 \uc6b4\ub3d9 \uc2e4\ud328\uc640 \ubb38\ud654\ub300\ud601\uba85\uc73c\ub85c \uc57d 2\ucc9c 5\ubc31\ub9cc \uba85\uc758 \uc778\uba85 \ud53c\ud574\ub97c \ucd08\ub798\ud558\uc600\ub2e4. \ub300\uc57d\uc9c4 \uc6b4\ub3d9\uacfc \ubb38\ud654 \ub300\ud601\uba85\uc5d0 \uc774\ub974\uae30\uae4c\uc9c0 \uacc4\uc18d\ub41c \uae09\uc9c4\uc801\uc778 \uc815\ucc45\uc774 \uc911\uad6d\uc758 \ubb38\ud654, \uc0ac\ud68c, \uacbd\uc81c, \uc678\uad50\uad00\uacc4\uc5d0 \uc785\ud78c \ubb3c\uc801, \uc778\uc801, \ubb38\ud654\uc801 \ud53c\ud574\ub294 \uc218\uce58\ub85c \uacc4\uc0b0\uc774 \ud798\ub4e4 \uc815\ub3c4\ub2e4. 1931\ub144 \uc774\ud6c4 \uc911\uad6d \uacf5\uc0b0\ub2f9\uc758 \uc9c0\ub3c4\uc801 \uc5ed\ud560\uc744 \ub9e1\uc544 \uc654\uc73c\uba70, 1949\ub144\ubd80\ud130 1959\ub144 \uae4c\uc9c0 \uc911\ud654\uc778\ubbfc\uacf5\ud654\uad6d\uc758 \uad6d\uac00\uc8fc\uc11d\uc744 \uc9c0\ub0c8\ub2e4. \uad6d\uac00\uc8fc\uc11d \uc9c1\uc5d0\uc11c \ubb3c\ub7ec\ub09c \ud6c4\uc5d0\ub3c4 \uc911\uad6d \uacf5\uc0b0\ub2f9 \uc8fc\uc11d\uc5d0 \uc804\uc784\ud558\uba70 \uc0ac\ub9dd\ud560 \ub54c\uae4c\uc9c0 \uc2e4\uad8c\uc744 \ud589\uc0ac\ud588\ub2e4.", "sentence_answer": "1949\ub144\ubd80\ud130 1959\ub144 \uae4c\uc9c0 \uc911\ud654\uc778\ubbfc\uacf5\ud654\uad6d\uc758 \uad6d\uac00\uc8fc\uc11d\uc744 \uc9c0\ub0c8\ub2e4.", "paragraph_id": "6316419-1-1", "paragraph_question": "question: \ub9c8\uc624\uca4c\ub465\uc774 \uad6d\uac00\uc8fc\uc11d\uc9c1\uc5d0\uc11c \ubb3c\ub7ec\ub09c \ud574\ub294?, context: \ub9c8\uc624\uca4c\ub465\uc774 \uc911\uad6d \ud604\ub300\uc0ac\uc5d0 \ud070 \uc601\ud5a5\uc744 \ubbf8\uce5c \ub370\uc5d0\ub294 \uc774\uacac\uc774 \uc5c6\uc73c\ub098, \uc624\ub298\ub0a0\uae4c\uc9c0\ub3c4 \uadf8\uc5d0 \ub300\ud55c \ud3c9\uac00\ub294 \uc544\uc9c1\ub3c4 \ub17c\uc7c1\uc774 \ubd84\ubd84\ud558\ub2e4. \uc77c\uac01\uc5d0\uc11c\ub294 \uc7a5\uc81c\uc2a4\ub97c \uaca9\ud30c\ud558\uace0 \uc911\uad6d \ub300\ub959\uc5d0 \uacf5\uc0b0\uc8fc\uc758 \uad6d\uac00\ub97c \uac74\uc124\ud55c \ud601\uba85\uac00\uc774\uc790 \uc804\ub7b5\uac00\ub85c \ud3c9\uac00\ud558\uace0 \uc788\uc9c0\ub9cc, \ub300\uc57d\uc9c4 \uc6b4\ub3d9 \uc2e4\ud328\uc640 \ubb38\ud654\ub300\ud601\uba85\uc73c\ub85c \uc57d 2\ucc9c 5\ubc31\ub9cc \uba85\uc758 \uc778\uba85 \ud53c\ud574\ub97c \ucd08\ub798\ud558\uc600\ub2e4. \ub300\uc57d\uc9c4 \uc6b4\ub3d9\uacfc \ubb38\ud654 \ub300\ud601\uba85\uc5d0 \uc774\ub974\uae30\uae4c\uc9c0 \uacc4\uc18d\ub41c \uae09\uc9c4\uc801\uc778 \uc815\ucc45\uc774 \uc911\uad6d\uc758 \ubb38\ud654, \uc0ac\ud68c, \uacbd\uc81c, \uc678\uad50\uad00\uacc4\uc5d0 \uc785\ud78c \ubb3c\uc801, \uc778\uc801, \ubb38\ud654\uc801 \ud53c\ud574\ub294 \uc218\uce58\ub85c \uacc4\uc0b0\uc774 \ud798\ub4e4 \uc815\ub3c4\ub2e4. 1931\ub144 \uc774\ud6c4 \uc911\uad6d \uacf5\uc0b0\ub2f9\uc758 \uc9c0\ub3c4\uc801 \uc5ed\ud560\uc744 \ub9e1\uc544 \uc654\uc73c\uba70, 1949\ub144\ubd80\ud130 1959\ub144\uae4c\uc9c0 \uc911\ud654\uc778\ubbfc\uacf5\ud654\uad6d\uc758 \uad6d\uac00\uc8fc\uc11d\uc744 \uc9c0\ub0c8\ub2e4. \uad6d\uac00\uc8fc\uc11d \uc9c1\uc5d0\uc11c \ubb3c\ub7ec\ub09c \ud6c4\uc5d0\ub3c4 \uc911\uad6d \uacf5\uc0b0\ub2f9 \uc8fc\uc11d\uc5d0 \uc804\uc784\ud558\uba70 \uc0ac\ub9dd\ud560 \ub54c\uae4c\uc9c0 \uc2e4\uad8c\uc744 \ud589\uc0ac\ud588\ub2e4."} {"question": "\ubba4\uc628\uc774 \ubc1c\uacac\ub41c \uc5f0\ub3c4\ub294?", "paragraph": "\ubba4\uc628\uc740 1936\ub144\uc5d0 Carl D. Anderson\uc774 \ubc1c\uacac\ud588\ub2e4. \ubc1c\uacac \ub2f9\uc2dc \uadf8\ub4e4\uc740 \ubba4\uc628\uc758 \uc9c8\ub7c9 \ub54c\ubb38\uc5d0 \ubba4\uc628\uc744 \uba54\uc874\uc73c\ub85c \uc798\ubabb \ubd84\ub958\ud588\uc5c8\ub2e4. \uadf8\ub7ec\ub098 \ubba4\uc628\uc774 \uc804\uc790\ucc98\ub7fc \uac15\ud55c \uc0c1\ud638\uc791\uc6a9\uc5d0 \uc601\ud5a5\uc744 \ubc1b\uc9c0 \uc54a\ub294\ub2e4\ub294 \uc2e4\ud5d8 \uacb0\uacfc\ub97c \ud1a0\ub300\ub85c \ud558\uc5ec, \uc0c8\ub85c \ubc1c\uacac\ud55c \ubba4\uc628\uc774 \uba54\uc874\ubcf4\ub2e4\ub294 \uc804\uc790\uc5d0 \ub354 \uac00\uae5d\ub2e4\uace0 \uacb0\ub860\uc744 \ub0b4\ub838\ub2e4. 1947\ub144\uc5d0\uc11c\uc57c \uc804\uc790\uc640 \uac19\uc774 \ud589\ub3d9\ud558\ub294 \uc785\uc790\ub4e4\uc744 \u2018\ub819\ud1a4\u2019\uc774\ub77c \uc774\ub984 \ubd99\uc5ec\uc8fc\uba70, \uc804\uc790\uc640 \ud568\uaed8, \ubba4\uc628\uacfc (\uc804\uc790) \uc911\uc131\ubbf8\uc790\ub97c \ub819\ud1a4\uc5d0 \ud3ec\ud568\uc2dc\ucf30\ub2e4. 1962\ub144 Leon M. Lederman, Melvin Schwartz \uadf8\ub9ac\uace0 Jack Steinberger\ub294 \ubba4\uc628 \uc911\uc131\ubbf8\uc790\uc758 \uc0c1\ud638\uc791\uc6a9\uc744 \ucc98\uc74c \ubc1c\uacac\ud558\uba74\uc11c, \uc804\uc790 \uc911\uc131\ubbf8\uc790 \uc774\uc678\uc5d0 \ub610 \ub2e4\ub978 \uc885\ub958\uc758 \uc911\uc131\ubbf8\uc790\uac00 \uc874\uc7ac\ud568\uc744 \uc785\uc99d\ud588\ub2e4. \uc774 \uacf5\ub85c\ub97c \uae30\ub9ac\uae30 \uc704\ud558\uc5ec, 1988\ub144 \uc138 \uc0ac\ub78c\uc5d0\uac8c \ub178\ubca8 \ubb3c\ub9ac\ud559\uc0c1\uc774 \uc218\uc5ec\ub418\uc5c8\ub2e4.", "answer": "1936\ub144", "sentence": "\ubba4\uc628\uc740 1936\ub144 \uc5d0 Carl D. Anderson\uc774 \ubc1c\uacac\ud588\ub2e4.", "paragraph_sentence": " \ubba4\uc628\uc740 1936\ub144 \uc5d0 Carl D. Anderson\uc774 \ubc1c\uacac\ud588\ub2e4. \ubc1c\uacac \ub2f9\uc2dc \uadf8\ub4e4\uc740 \ubba4\uc628\uc758 \uc9c8\ub7c9 \ub54c\ubb38\uc5d0 \ubba4\uc628\uc744 \uba54\uc874\uc73c\ub85c \uc798\ubabb \ubd84\ub958\ud588\uc5c8\ub2e4. \uadf8\ub7ec\ub098 \ubba4\uc628\uc774 \uc804\uc790\ucc98\ub7fc \uac15\ud55c \uc0c1\ud638\uc791\uc6a9\uc5d0 \uc601\ud5a5\uc744 \ubc1b\uc9c0 \uc54a\ub294\ub2e4\ub294 \uc2e4\ud5d8 \uacb0\uacfc\ub97c \ud1a0\ub300\ub85c \ud558\uc5ec, \uc0c8\ub85c \ubc1c\uacac\ud55c \ubba4\uc628\uc774 \uba54\uc874\ubcf4\ub2e4\ub294 \uc804\uc790\uc5d0 \ub354 \uac00\uae5d\ub2e4\uace0 \uacb0\ub860\uc744 \ub0b4\ub838\ub2e4. 1947\ub144\uc5d0\uc11c\uc57c \uc804\uc790\uc640 \uac19\uc774 \ud589\ub3d9\ud558\ub294 \uc785\uc790\ub4e4\uc744 \u2018\ub819\ud1a4\u2019\uc774\ub77c \uc774\ub984 \ubd99\uc5ec\uc8fc\uba70, \uc804\uc790\uc640 \ud568\uaed8, \ubba4\uc628\uacfc (\uc804\uc790) \uc911\uc131\ubbf8\uc790\ub97c \ub819\ud1a4\uc5d0 \ud3ec\ud568\uc2dc\ucf30\ub2e4. 1962\ub144 Leon M. Lederman, Melvin Schwartz \uadf8\ub9ac\uace0 Jack Steinberger\ub294 \ubba4\uc628 \uc911\uc131\ubbf8\uc790\uc758 \uc0c1\ud638\uc791\uc6a9\uc744 \ucc98\uc74c \ubc1c\uacac\ud558\uba74\uc11c, \uc804\uc790 \uc911\uc131\ubbf8\uc790 \uc774\uc678\uc5d0 \ub610 \ub2e4\ub978 \uc885\ub958\uc758 \uc911\uc131\ubbf8\uc790\uac00 \uc874\uc7ac\ud568\uc744 \uc785\uc99d\ud588\ub2e4. \uc774 \uacf5\ub85c\ub97c \uae30\ub9ac\uae30 \uc704\ud558\uc5ec, 1988\ub144 \uc138 \uc0ac\ub78c\uc5d0\uac8c \ub178\ubca8 \ubb3c\ub9ac\ud559\uc0c1\uc774 \uc218\uc5ec\ub418\uc5c8\ub2e4.", "paragraph_answer": "\ubba4\uc628\uc740 1936\ub144 \uc5d0 Carl D. Anderson\uc774 \ubc1c\uacac\ud588\ub2e4. \ubc1c\uacac \ub2f9\uc2dc \uadf8\ub4e4\uc740 \ubba4\uc628\uc758 \uc9c8\ub7c9 \ub54c\ubb38\uc5d0 \ubba4\uc628\uc744 \uba54\uc874\uc73c\ub85c \uc798\ubabb \ubd84\ub958\ud588\uc5c8\ub2e4. \uadf8\ub7ec\ub098 \ubba4\uc628\uc774 \uc804\uc790\ucc98\ub7fc \uac15\ud55c \uc0c1\ud638\uc791\uc6a9\uc5d0 \uc601\ud5a5\uc744 \ubc1b\uc9c0 \uc54a\ub294\ub2e4\ub294 \uc2e4\ud5d8 \uacb0\uacfc\ub97c \ud1a0\ub300\ub85c \ud558\uc5ec, \uc0c8\ub85c \ubc1c\uacac\ud55c \ubba4\uc628\uc774 \uba54\uc874\ubcf4\ub2e4\ub294 \uc804\uc790\uc5d0 \ub354 \uac00\uae5d\ub2e4\uace0 \uacb0\ub860\uc744 \ub0b4\ub838\ub2e4. 1947\ub144\uc5d0\uc11c\uc57c \uc804\uc790\uc640 \uac19\uc774 \ud589\ub3d9\ud558\ub294 \uc785\uc790\ub4e4\uc744 \u2018\ub819\ud1a4\u2019\uc774\ub77c \uc774\ub984 \ubd99\uc5ec\uc8fc\uba70, \uc804\uc790\uc640 \ud568\uaed8, \ubba4\uc628\uacfc (\uc804\uc790) \uc911\uc131\ubbf8\uc790\ub97c \ub819\ud1a4\uc5d0 \ud3ec\ud568\uc2dc\ucf30\ub2e4. 1962\ub144 Leon M. Lederman, Melvin Schwartz \uadf8\ub9ac\uace0 Jack Steinberger\ub294 \ubba4\uc628 \uc911\uc131\ubbf8\uc790\uc758 \uc0c1\ud638\uc791\uc6a9\uc744 \ucc98\uc74c \ubc1c\uacac\ud558\uba74\uc11c, \uc804\uc790 \uc911\uc131\ubbf8\uc790 \uc774\uc678\uc5d0 \ub610 \ub2e4\ub978 \uc885\ub958\uc758 \uc911\uc131\ubbf8\uc790\uac00 \uc874\uc7ac\ud568\uc744 \uc785\uc99d\ud588\ub2e4. \uc774 \uacf5\ub85c\ub97c \uae30\ub9ac\uae30 \uc704\ud558\uc5ec, 1988\ub144 \uc138 \uc0ac\ub78c\uc5d0\uac8c \ub178\ubca8 \ubb3c\ub9ac\ud559\uc0c1\uc774 \uc218\uc5ec\ub418\uc5c8\ub2e4.", "sentence_answer": "\ubba4\uc628\uc740 1936\ub144 \uc5d0 Carl D. Anderson\uc774 \ubc1c\uacac\ud588\ub2e4.", "paragraph_id": "6657775-3-0", "paragraph_question": "question: \ubba4\uc628\uc774 \ubc1c\uacac\ub41c \uc5f0\ub3c4\ub294?, context: \ubba4\uc628\uc740 1936\ub144\uc5d0 Carl D. Anderson\uc774 \ubc1c\uacac\ud588\ub2e4. \ubc1c\uacac \ub2f9\uc2dc \uadf8\ub4e4\uc740 \ubba4\uc628\uc758 \uc9c8\ub7c9 \ub54c\ubb38\uc5d0 \ubba4\uc628\uc744 \uba54\uc874\uc73c\ub85c \uc798\ubabb \ubd84\ub958\ud588\uc5c8\ub2e4. \uadf8\ub7ec\ub098 \ubba4\uc628\uc774 \uc804\uc790\ucc98\ub7fc \uac15\ud55c \uc0c1\ud638\uc791\uc6a9\uc5d0 \uc601\ud5a5\uc744 \ubc1b\uc9c0 \uc54a\ub294\ub2e4\ub294 \uc2e4\ud5d8 \uacb0\uacfc\ub97c \ud1a0\ub300\ub85c \ud558\uc5ec, \uc0c8\ub85c \ubc1c\uacac\ud55c \ubba4\uc628\uc774 \uba54\uc874\ubcf4\ub2e4\ub294 \uc804\uc790\uc5d0 \ub354 \uac00\uae5d\ub2e4\uace0 \uacb0\ub860\uc744 \ub0b4\ub838\ub2e4. 1947\ub144\uc5d0\uc11c\uc57c \uc804\uc790\uc640 \uac19\uc774 \ud589\ub3d9\ud558\ub294 \uc785\uc790\ub4e4\uc744 \u2018\ub819\ud1a4\u2019\uc774\ub77c \uc774\ub984 \ubd99\uc5ec\uc8fc\uba70, \uc804\uc790\uc640 \ud568\uaed8, \ubba4\uc628\uacfc (\uc804\uc790) \uc911\uc131\ubbf8\uc790\ub97c \ub819\ud1a4\uc5d0 \ud3ec\ud568\uc2dc\ucf30\ub2e4. 1962\ub144 Leon M. Lederman, Melvin Schwartz \uadf8\ub9ac\uace0 Jack Steinberger\ub294 \ubba4\uc628 \uc911\uc131\ubbf8\uc790\uc758 \uc0c1\ud638\uc791\uc6a9\uc744 \ucc98\uc74c \ubc1c\uacac\ud558\uba74\uc11c, \uc804\uc790 \uc911\uc131\ubbf8\uc790 \uc774\uc678\uc5d0 \ub610 \ub2e4\ub978 \uc885\ub958\uc758 \uc911\uc131\ubbf8\uc790\uac00 \uc874\uc7ac\ud568\uc744 \uc785\uc99d\ud588\ub2e4. \uc774 \uacf5\ub85c\ub97c \uae30\ub9ac\uae30 \uc704\ud558\uc5ec, 1988\ub144 \uc138 \uc0ac\ub78c\uc5d0\uac8c \ub178\ubca8 \ubb3c\ub9ac\ud559\uc0c1\uc774 \uc218\uc5ec\ub418\uc5c8\ub2e4."} {"question": "\ub85c\uc988\ub294 \uc5b8\uc81c \ucc98\uc74c \ubc29\uc601\ub418\uc5c8\ub098?", "paragraph": "\u3008Rose\u3009\ub294 2005\ub144 3\uc6d4 26\uc77c BBC One\uc744 \ud1b5\ud574 \ucc98\uc74c\uc73c\ub85c \uc601\uad6d\uc5d0\uc11c \ubc29\uc601\ub418\uc5c8\uace0, 1996\ub144\uc758 \ub2e5\ud130 \ud6c4 \ud154\ub808\ube44\uc804 \uc601\ud654 \uc774\ub798\ub85c \ucc98\uc74c\uc73c\ub85c \ubc29\uc601\ub41c \ub2e5\ud130 \ud6c4 \uc5d0\ud53c\uc18c\ub4dc\uac00 \ub418\uc5c8\ub2e4. \ubc29\uc1a1\uc0ac \uc2dc\uccad\uc790 \uc870\uc0ac \uc704\uc6d0\ud68c (BARB)\uc758 \ube44\uacf5\uc2dd \ubc24\uc0ac\uc774 \uc2dc\uccad\uc218\uce58\ub294 \uc5d0\ud53c\uc18c\ub4dc\uac00 \uc800\ub141 \ub3d9\uc548\uc5d0 \ud3c9\uade0 990\ub9cc \uba85\uc758 \uc2dc\uccad\uc790, \uc720\ud6a8 \ud154\ub808\ube44\uc804 \uc2dc\uccad\uc790\uc758 43.2%\ub97c \ub04c\uc5b4\ubaa8\uc740 \uac83\uc73c\ub85c \ub098\ud0c0\ub0ac\ub2e4. \ucd5c\uace0 \uc2dc\uac04\ub300\uc5d0\ub294 1050\ub9cc \uba85\uc758 \uc2dc\uccad\uc790, 44.3%\uc758 \uc2dc\uccad \uc810\uc720\uc728\uc774\uc5c8\ub2e4. \uc5d0\ud53c\uc18c\ub4dc\uc758 \ucd5c\uc885 \uc218\uce58\ub294 \uccab\ubc29\uc601 \uc8fc\uac04\ub0b4\uc5d0 \uc2dc\uccad\ub41c \ube44\ub514\uc624 \ub179\ud654\ub97c \ud3ec\ud568\ud574\uc11c 1081\ub9cc \uba85\uc73c\ub85c, \ud574\ub2f9 \uc8fc\uc5d0\ub294 BBC One\uc5d0\uc11c \uc138 \ubc88\uc9f8, \ubaa8\ub4e0 \ucc44\ub110\uc744 \ud1b5\ud2c0\uc5b4\uc11c \uc77c\uacf1 \ubc88\uc9f8\ub85c \ucef8\ub2e4. \uc774 \uc5d0\ud53c\uc18c\ub4dc\uc758 3\uc6d4 26\uc77c BBC \uccab \ubc29\uc601 \uc2dc \uc77c\ubd80 \uc9c0\uc5ed\uc5d0\uc11c\ub294 \ucc98\uc74c \uba87 \ubd84\uc774 \uadf8\ub808\uc774\uc5c4 \ub178\ud134\uc774 \uc9c4\ud589\ud558\ub294 \u300a\uc2a4\ud2b8\ub9ad\ud2c0\ub9ac \ub304\uc2a4 \ud53c\ubc84\u300b\uc758 \uba87 \ucd08 \uac04 \uc18c\ub9ac\uc640 \uc6b0\uc5f0\ud788 \ud63c\uc120\ub418\uc5b4 \ub9dd\uccd0\uc9c0\uae30\ub3c4 \ud588\ub2e4.", "answer": "2005\ub144 3\uc6d4 26\uc77c", "sentence": "\u3008Rose\u3009\ub294 2005\ub144 3\uc6d4 26\uc77c BBC One\uc744 \ud1b5\ud574 \ucc98\uc74c\uc73c\ub85c \uc601\uad6d\uc5d0\uc11c \ubc29\uc601\ub418\uc5c8\uace0, 1996\ub144\uc758 \ub2e5\ud130 \ud6c4 \ud154\ub808\ube44\uc804 \uc601\ud654 \uc774\ub798\ub85c \ucc98\uc74c\uc73c\ub85c \ubc29\uc601\ub41c \ub2e5\ud130 \ud6c4 \uc5d0\ud53c\uc18c\ub4dc\uac00 \ub418\uc5c8\ub2e4.", "paragraph_sentence": " \u3008Rose\u3009\ub294 2005\ub144 3\uc6d4 26\uc77c BBC One\uc744 \ud1b5\ud574 \ucc98\uc74c\uc73c\ub85c \uc601\uad6d\uc5d0\uc11c \ubc29\uc601\ub418\uc5c8\uace0, 1996\ub144\uc758 \ub2e5\ud130 \ud6c4 \ud154\ub808\ube44\uc804 \uc601\ud654 \uc774\ub798\ub85c \ucc98\uc74c\uc73c\ub85c \ubc29\uc601\ub41c \ub2e5\ud130 \ud6c4 \uc5d0\ud53c\uc18c\ub4dc\uac00 \ub418\uc5c8\ub2e4. \ubc29\uc1a1\uc0ac \uc2dc\uccad\uc790 \uc870\uc0ac \uc704\uc6d0\ud68c (BARB)\uc758 \ube44\uacf5\uc2dd \ubc24\uc0ac\uc774 \uc2dc\uccad\uc218\uce58\ub294 \uc5d0\ud53c\uc18c\ub4dc\uac00 \uc800\ub141 \ub3d9\uc548\uc5d0 \ud3c9\uade0 990\ub9cc \uba85\uc758 \uc2dc\uccad\uc790, \uc720\ud6a8 \ud154\ub808\ube44\uc804 \uc2dc\uccad\uc790\uc758 43.2%\ub97c \ub04c\uc5b4\ubaa8\uc740 \uac83\uc73c\ub85c \ub098\ud0c0\ub0ac\ub2e4. \ucd5c\uace0 \uc2dc\uac04\ub300\uc5d0\ub294 1050\ub9cc \uba85\uc758 \uc2dc\uccad\uc790, 44.3%\uc758 \uc2dc\uccad \uc810\uc720\uc728\uc774\uc5c8\ub2e4. \uc5d0\ud53c\uc18c\ub4dc\uc758 \ucd5c\uc885 \uc218\uce58\ub294 \uccab\ubc29\uc601 \uc8fc\uac04\ub0b4\uc5d0 \uc2dc\uccad\ub41c \ube44\ub514\uc624 \ub179\ud654\ub97c \ud3ec\ud568\ud574\uc11c 1081\ub9cc \uba85\uc73c\ub85c, \ud574\ub2f9 \uc8fc\uc5d0\ub294 BBC One\uc5d0\uc11c \uc138 \ubc88\uc9f8, \ubaa8\ub4e0 \ucc44\ub110\uc744 \ud1b5\ud2c0\uc5b4\uc11c \uc77c\uacf1 \ubc88\uc9f8\ub85c \ucef8\ub2e4. \uc774 \uc5d0\ud53c\uc18c\ub4dc\uc758 3\uc6d4 26\uc77c BBC \uccab \ubc29\uc601 \uc2dc \uc77c\ubd80 \uc9c0\uc5ed\uc5d0\uc11c\ub294 \ucc98\uc74c \uba87 \ubd84\uc774 \uadf8\ub808\uc774\uc5c4 \ub178\ud134\uc774 \uc9c4\ud589\ud558\ub294 \u300a\uc2a4\ud2b8\ub9ad\ud2c0\ub9ac \ub304\uc2a4 \ud53c\ubc84\u300b\uc758 \uba87 \ucd08 \uac04 \uc18c\ub9ac\uc640 \uc6b0\uc5f0\ud788 \ud63c\uc120\ub418\uc5b4 \ub9dd\uccd0\uc9c0\uae30\ub3c4 \ud588\ub2e4.", "paragraph_answer": "\u3008Rose\u3009\ub294 2005\ub144 3\uc6d4 26\uc77c BBC One\uc744 \ud1b5\ud574 \ucc98\uc74c\uc73c\ub85c \uc601\uad6d\uc5d0\uc11c \ubc29\uc601\ub418\uc5c8\uace0, 1996\ub144\uc758 \ub2e5\ud130 \ud6c4 \ud154\ub808\ube44\uc804 \uc601\ud654 \uc774\ub798\ub85c \ucc98\uc74c\uc73c\ub85c \ubc29\uc601\ub41c \ub2e5\ud130 \ud6c4 \uc5d0\ud53c\uc18c\ub4dc\uac00 \ub418\uc5c8\ub2e4. \ubc29\uc1a1\uc0ac \uc2dc\uccad\uc790 \uc870\uc0ac \uc704\uc6d0\ud68c (BARB)\uc758 \ube44\uacf5\uc2dd \ubc24\uc0ac\uc774 \uc2dc\uccad\uc218\uce58\ub294 \uc5d0\ud53c\uc18c\ub4dc\uac00 \uc800\ub141 \ub3d9\uc548\uc5d0 \ud3c9\uade0 990\ub9cc \uba85\uc758 \uc2dc\uccad\uc790, \uc720\ud6a8 \ud154\ub808\ube44\uc804 \uc2dc\uccad\uc790\uc758 43.2%\ub97c \ub04c\uc5b4\ubaa8\uc740 \uac83\uc73c\ub85c \ub098\ud0c0\ub0ac\ub2e4. \ucd5c\uace0 \uc2dc\uac04\ub300\uc5d0\ub294 1050\ub9cc \uba85\uc758 \uc2dc\uccad\uc790, 44.3%\uc758 \uc2dc\uccad \uc810\uc720\uc728\uc774\uc5c8\ub2e4. \uc5d0\ud53c\uc18c\ub4dc\uc758 \ucd5c\uc885 \uc218\uce58\ub294 \uccab\ubc29\uc601 \uc8fc\uac04\ub0b4\uc5d0 \uc2dc\uccad\ub41c \ube44\ub514\uc624 \ub179\ud654\ub97c \ud3ec\ud568\ud574\uc11c 1081\ub9cc \uba85\uc73c\ub85c, \ud574\ub2f9 \uc8fc\uc5d0\ub294 BBC One\uc5d0\uc11c \uc138 \ubc88\uc9f8, \ubaa8\ub4e0 \ucc44\ub110\uc744 \ud1b5\ud2c0\uc5b4\uc11c \uc77c\uacf1 \ubc88\uc9f8\ub85c \ucef8\ub2e4. \uc774 \uc5d0\ud53c\uc18c\ub4dc\uc758 3\uc6d4 26\uc77c BBC \uccab \ubc29\uc601 \uc2dc \uc77c\ubd80 \uc9c0\uc5ed\uc5d0\uc11c\ub294 \ucc98\uc74c \uba87 \ubd84\uc774 \uadf8\ub808\uc774\uc5c4 \ub178\ud134\uc774 \uc9c4\ud589\ud558\ub294 \u300a\uc2a4\ud2b8\ub9ad\ud2c0\ub9ac \ub304\uc2a4 \ud53c\ubc84\u300b\uc758 \uba87 \ucd08 \uac04 \uc18c\ub9ac\uc640 \uc6b0\uc5f0\ud788 \ud63c\uc120\ub418\uc5b4 \ub9dd\uccd0\uc9c0\uae30\ub3c4 \ud588\ub2e4.", "sentence_answer": "\u3008Rose\u3009\ub294 2005\ub144 3\uc6d4 26\uc77c BBC One\uc744 \ud1b5\ud574 \ucc98\uc74c\uc73c\ub85c \uc601\uad6d\uc5d0\uc11c \ubc29\uc601\ub418\uc5c8\uace0, 1996\ub144\uc758 \ub2e5\ud130 \ud6c4 \ud154\ub808\ube44\uc804 \uc601\ud654 \uc774\ub798\ub85c \ucc98\uc74c\uc73c\ub85c \ubc29\uc601\ub41c \ub2e5\ud130 \ud6c4 \uc5d0\ud53c\uc18c\ub4dc\uac00 \ub418\uc5c8\ub2e4.", "paragraph_id": "6483060-10-0", "paragraph_question": "question: \ub85c\uc988\ub294 \uc5b8\uc81c \ucc98\uc74c \ubc29\uc601\ub418\uc5c8\ub098?, context: \u3008Rose\u3009\ub294 2005\ub144 3\uc6d4 26\uc77c BBC One\uc744 \ud1b5\ud574 \ucc98\uc74c\uc73c\ub85c \uc601\uad6d\uc5d0\uc11c \ubc29\uc601\ub418\uc5c8\uace0, 1996\ub144\uc758 \ub2e5\ud130 \ud6c4 \ud154\ub808\ube44\uc804 \uc601\ud654 \uc774\ub798\ub85c \ucc98\uc74c\uc73c\ub85c \ubc29\uc601\ub41c \ub2e5\ud130 \ud6c4 \uc5d0\ud53c\uc18c\ub4dc\uac00 \ub418\uc5c8\ub2e4. \ubc29\uc1a1\uc0ac \uc2dc\uccad\uc790 \uc870\uc0ac \uc704\uc6d0\ud68c (BARB)\uc758 \ube44\uacf5\uc2dd \ubc24\uc0ac\uc774 \uc2dc\uccad\uc218\uce58\ub294 \uc5d0\ud53c\uc18c\ub4dc\uac00 \uc800\ub141 \ub3d9\uc548\uc5d0 \ud3c9\uade0 990\ub9cc \uba85\uc758 \uc2dc\uccad\uc790, \uc720\ud6a8 \ud154\ub808\ube44\uc804 \uc2dc\uccad\uc790\uc758 43.2%\ub97c \ub04c\uc5b4\ubaa8\uc740 \uac83\uc73c\ub85c \ub098\ud0c0\ub0ac\ub2e4. \ucd5c\uace0 \uc2dc\uac04\ub300\uc5d0\ub294 1050\ub9cc \uba85\uc758 \uc2dc\uccad\uc790, 44.3%\uc758 \uc2dc\uccad \uc810\uc720\uc728\uc774\uc5c8\ub2e4. \uc5d0\ud53c\uc18c\ub4dc\uc758 \ucd5c\uc885 \uc218\uce58\ub294 \uccab\ubc29\uc601 \uc8fc\uac04\ub0b4\uc5d0 \uc2dc\uccad\ub41c \ube44\ub514\uc624 \ub179\ud654\ub97c \ud3ec\ud568\ud574\uc11c 1081\ub9cc \uba85\uc73c\ub85c, \ud574\ub2f9 \uc8fc\uc5d0\ub294 BBC One\uc5d0\uc11c \uc138 \ubc88\uc9f8, \ubaa8\ub4e0 \ucc44\ub110\uc744 \ud1b5\ud2c0\uc5b4\uc11c \uc77c\uacf1 \ubc88\uc9f8\ub85c \ucef8\ub2e4. \uc774 \uc5d0\ud53c\uc18c\ub4dc\uc758 3\uc6d4 26\uc77c BBC \uccab \ubc29\uc601 \uc2dc \uc77c\ubd80 \uc9c0\uc5ed\uc5d0\uc11c\ub294 \ucc98\uc74c \uba87 \ubd84\uc774 \uadf8\ub808\uc774\uc5c4 \ub178\ud134\uc774 \uc9c4\ud589\ud558\ub294 \u300a\uc2a4\ud2b8\ub9ad\ud2c0\ub9ac \ub304\uc2a4 \ud53c\ubc84\u300b\uc758 \uba87 \ucd08 \uac04 \uc18c\ub9ac\uc640 \uc6b0\uc5f0\ud788 \ud63c\uc120\ub418\uc5b4 \ub9dd\uccd0\uc9c0\uae30\ub3c4 \ud588\ub2e4."} {"question": "1797\ub144 \uc804\uc7c1\uc774 \ubc1c\ubc1c\ud55c \ud6c4 \uba87 \ub144\uc774 \uc9c0\ub098 \ud504\ub791\uc2a4 \uacf5\ud654\uad6d\uc774 \uc81c 1\ucc28 \ub300\ud504\ub791\uc2a4 \ub3d9\ub9f9\uc744 \uaca9\ud30c\ud588\uc744\uae4c\uc694?", "paragraph": "\uc720\ub7fd\uc740 1792\ub144 \ud504\ub791\uc2a4 \ud601\uba85\uc804\uc7c1\uc774 \ubc1c\ubc1c\ud55c \uc774\ub798\ub85c \ud070 \ud63c\ub780\uc5d0 \ube60\uc838 \uc788\uc5c8\ub2e4. 1797\ub144 \uc804\uc7c1\uc774 \ubc1c\ubc1c\ud55c \uc9c0 5\ub144\uc774 \uc9c0\ub098, \ud504\ub791\uc2a4 \uacf5\ud654\uad6d\uc740 \uc81c1\ucc28 \ub300\ud504\ub791\uc2a4 \ub3d9\ub9f9\uc744 \uaca9\ud30c\ud588\ub2e4. \uc81c2\ucc28 \ub300\ud504\ub791\uc2a4 \ub3d9\ub9f9\uc740 1798\ub144\uc5d0 \uacb0\uc131\ub418\uc5c8\uc73c\ub098, 1801\ub144 \uc774 \uc5ed\uc2dc\ub3c4 \ud328\ubc30\ud558\uace0, \uc624\uc9c1 \uc601\uad6d\ub9cc\uc774 \uc0c8\ub85c\uc6b4 \ud504\ub791\uc2a4 \uc6d0\ub85c\uc6d0\uc758 \uc720\uc77c\ud55c \uc801\uc73c\ub85c \ub0a8\uc544\uc788\uc744 \ubfd0\uc774\uc5c8\ub2e4. \uc601\uad6d\uacfc \ud504\ub791\uc2a4\ub294 \uc544\ubbf8\uc575 \uc870\uc57d(Treaty of Amiens)\uc73c\ub85c \uc801\ub300 \uad00\uacc4\ub97c \ub05d\ub0b4\ub294 \ub370 \ub3d9\uc758\ud558\uc600\ub2e4. \uc9c0\ub09c 10\ub144\uac04 \ucc98\uc74c\uc73c\ub85c \ubaa8\ub4e0 \uc720\ub7fd\uc774 \ud3c9\ud654\uc0c1\ud0dc\ub97c \ub204\ub9ac\uac8c \ub418\uc5c8\ub2e4. \uadf8\ub7ec\ub098 \uc591\uce21\uc5d0\ub294 \uc870\uc57d\uc744 \uc774\ud589\ud558\ub294 \ub370 \uc810\uc810 \uc5b4\ub824\uc6c0\uc774 \ub204\uc801\ub418\ub294 \ub4f1 \ub9ce\uc740 \ubb38\uc81c\ub4e4\uc774 \uc9c0\uc18d\ub418\uace0 \uc788\uc5c8\ub2e4. \uc601\uad6d\uc815\ubd80\ub294 1793\ub144\ubd80\ud130 \uc774\ub8e8\uc5b4\uc9c4 \uc2dd\ubbfc\uc9c0 \uc815\ubcf5\uc744 \ub418\ub3cc\ub824\uc57c \ud55c\ub2e4\ub294 \ub370 \ubd84\uac1c\ud588\uace0, \ub098\ud3f4\ub808\uc639\uc740 \uc601\uad6d\uad70\uc774 \ubab0\ud0c0(Malta) \uc12c\uc5d0\uc11c \ucca0\uc218\ud558\uc9c0 \uc54a\ub294 \ub370 \ud654\uac00 \ub0ac\ub2e4. \uae34\uc7a5 \uc0c1\ud0dc\ub294 \ub098\ud3f4\ub808\uc639\uc774 \uc544\uc774\ud2f0 \ud601\uba85\uc744 \uc9c4\uc555\ud558\uae30 \uc704\ud574 \uc6d0\uc815\uad70\uc744 \ud30c\uacac\ud588\uc744 \ub54c \ub354\uc6b1 \uc545\ud654\ub418\uc5c8\ub2e4. 1803\ub144 5\uc6d4, \uc601\uad6d\uc740 \ud504\ub791\uc2a4\uc5d0 \ub300\ud55c \uc804\uc7c1\uc744 \uc120\ud3ec\ud588\ub2e4.", "answer": "5\ub144", "sentence": "1797\ub144 \uc804\uc7c1\uc774 \ubc1c\ubc1c\ud55c \uc9c0 5\ub144 \uc774 \uc9c0\ub098, \ud504\ub791\uc2a4 \uacf5\ud654\uad6d\uc740 \uc81c1\ucc28 \ub300\ud504\ub791\uc2a4 \ub3d9\ub9f9\uc744 \uaca9\ud30c\ud588\ub2e4.", "paragraph_sentence": "\uc720\ub7fd\uc740 1792\ub144 \ud504\ub791\uc2a4 \ud601\uba85\uc804\uc7c1\uc774 \ubc1c\ubc1c\ud55c \uc774\ub798\ub85c \ud070 \ud63c\ub780\uc5d0 \ube60\uc838 \uc788\uc5c8\ub2e4. 1797\ub144 \uc804\uc7c1\uc774 \ubc1c\ubc1c\ud55c \uc9c0 5\ub144 \uc774 \uc9c0\ub098, \ud504\ub791\uc2a4 \uacf5\ud654\uad6d\uc740 \uc81c1\ucc28 \ub300\ud504\ub791\uc2a4 \ub3d9\ub9f9\uc744 \uaca9\ud30c\ud588\ub2e4. \uc81c2\ucc28 \ub300\ud504\ub791\uc2a4 \ub3d9\ub9f9\uc740 1798\ub144\uc5d0 \uacb0\uc131\ub418\uc5c8\uc73c\ub098, 1801\ub144 \uc774 \uc5ed\uc2dc\ub3c4 \ud328\ubc30\ud558\uace0, \uc624\uc9c1 \uc601\uad6d\ub9cc\uc774 \uc0c8\ub85c\uc6b4 \ud504\ub791\uc2a4 \uc6d0\ub85c\uc6d0\uc758 \uc720\uc77c\ud55c \uc801\uc73c\ub85c \ub0a8\uc544\uc788\uc744 \ubfd0\uc774\uc5c8\ub2e4. \uc601\uad6d\uacfc \ud504\ub791\uc2a4\ub294 \uc544\ubbf8\uc575 \uc870\uc57d(Treaty of Amiens)\uc73c\ub85c \uc801\ub300 \uad00\uacc4\ub97c \ub05d\ub0b4\ub294 \ub370 \ub3d9\uc758\ud558\uc600\ub2e4. \uc9c0\ub09c 10\ub144\uac04 \ucc98\uc74c\uc73c\ub85c \ubaa8\ub4e0 \uc720\ub7fd\uc774 \ud3c9\ud654\uc0c1\ud0dc\ub97c \ub204\ub9ac\uac8c \ub418\uc5c8\ub2e4. \uadf8\ub7ec\ub098 \uc591\uce21\uc5d0\ub294 \uc870\uc57d\uc744 \uc774\ud589\ud558\ub294 \ub370 \uc810\uc810 \uc5b4\ub824\uc6c0\uc774 \ub204\uc801\ub418\ub294 \ub4f1 \ub9ce\uc740 \ubb38\uc81c\ub4e4\uc774 \uc9c0\uc18d\ub418\uace0 \uc788\uc5c8\ub2e4. \uc601\uad6d\uc815\ubd80\ub294 1793\ub144\ubd80\ud130 \uc774\ub8e8\uc5b4\uc9c4 \uc2dd\ubbfc\uc9c0 \uc815\ubcf5\uc744 \ub418\ub3cc\ub824\uc57c \ud55c\ub2e4\ub294 \ub370 \ubd84\uac1c\ud588\uace0, \ub098\ud3f4\ub808\uc639\uc740 \uc601\uad6d\uad70\uc774 \ubab0\ud0c0(Malta) \uc12c\uc5d0\uc11c \ucca0\uc218\ud558\uc9c0 \uc54a\ub294 \ub370 \ud654\uac00 \ub0ac\ub2e4. \uae34\uc7a5 \uc0c1\ud0dc\ub294 \ub098\ud3f4\ub808\uc639\uc774 \uc544\uc774\ud2f0 \ud601\uba85\uc744 \uc9c4\uc555\ud558\uae30 \uc704\ud574 \uc6d0\uc815\uad70\uc744 \ud30c\uacac\ud588\uc744 \ub54c \ub354\uc6b1 \uc545\ud654\ub418\uc5c8\ub2e4. 1803\ub144 5\uc6d4, \uc601\uad6d\uc740 \ud504\ub791\uc2a4\uc5d0 \ub300\ud55c \uc804\uc7c1\uc744 \uc120\ud3ec\ud588\ub2e4.", "paragraph_answer": "\uc720\ub7fd\uc740 1792\ub144 \ud504\ub791\uc2a4 \ud601\uba85\uc804\uc7c1\uc774 \ubc1c\ubc1c\ud55c \uc774\ub798\ub85c \ud070 \ud63c\ub780\uc5d0 \ube60\uc838 \uc788\uc5c8\ub2e4. 1797\ub144 \uc804\uc7c1\uc774 \ubc1c\ubc1c\ud55c \uc9c0 5\ub144 \uc774 \uc9c0\ub098, \ud504\ub791\uc2a4 \uacf5\ud654\uad6d\uc740 \uc81c1\ucc28 \ub300\ud504\ub791\uc2a4 \ub3d9\ub9f9\uc744 \uaca9\ud30c\ud588\ub2e4. \uc81c2\ucc28 \ub300\ud504\ub791\uc2a4 \ub3d9\ub9f9\uc740 1798\ub144\uc5d0 \uacb0\uc131\ub418\uc5c8\uc73c\ub098, 1801\ub144 \uc774 \uc5ed\uc2dc\ub3c4 \ud328\ubc30\ud558\uace0, \uc624\uc9c1 \uc601\uad6d\ub9cc\uc774 \uc0c8\ub85c\uc6b4 \ud504\ub791\uc2a4 \uc6d0\ub85c\uc6d0\uc758 \uc720\uc77c\ud55c \uc801\uc73c\ub85c \ub0a8\uc544\uc788\uc744 \ubfd0\uc774\uc5c8\ub2e4. \uc601\uad6d\uacfc \ud504\ub791\uc2a4\ub294 \uc544\ubbf8\uc575 \uc870\uc57d(Treaty of Amiens)\uc73c\ub85c \uc801\ub300 \uad00\uacc4\ub97c \ub05d\ub0b4\ub294 \ub370 \ub3d9\uc758\ud558\uc600\ub2e4. \uc9c0\ub09c 10\ub144\uac04 \ucc98\uc74c\uc73c\ub85c \ubaa8\ub4e0 \uc720\ub7fd\uc774 \ud3c9\ud654\uc0c1\ud0dc\ub97c \ub204\ub9ac\uac8c \ub418\uc5c8\ub2e4. \uadf8\ub7ec\ub098 \uc591\uce21\uc5d0\ub294 \uc870\uc57d\uc744 \uc774\ud589\ud558\ub294 \ub370 \uc810\uc810 \uc5b4\ub824\uc6c0\uc774 \ub204\uc801\ub418\ub294 \ub4f1 \ub9ce\uc740 \ubb38\uc81c\ub4e4\uc774 \uc9c0\uc18d\ub418\uace0 \uc788\uc5c8\ub2e4. \uc601\uad6d\uc815\ubd80\ub294 1793\ub144\ubd80\ud130 \uc774\ub8e8\uc5b4\uc9c4 \uc2dd\ubbfc\uc9c0 \uc815\ubcf5\uc744 \ub418\ub3cc\ub824\uc57c \ud55c\ub2e4\ub294 \ub370 \ubd84\uac1c\ud588\uace0, \ub098\ud3f4\ub808\uc639\uc740 \uc601\uad6d\uad70\uc774 \ubab0\ud0c0(Malta) \uc12c\uc5d0\uc11c \ucca0\uc218\ud558\uc9c0 \uc54a\ub294 \ub370 \ud654\uac00 \ub0ac\ub2e4. \uae34\uc7a5 \uc0c1\ud0dc\ub294 \ub098\ud3f4\ub808\uc639\uc774 \uc544\uc774\ud2f0 \ud601\uba85\uc744 \uc9c4\uc555\ud558\uae30 \uc704\ud574 \uc6d0\uc815\uad70\uc744 \ud30c\uacac\ud588\uc744 \ub54c \ub354\uc6b1 \uc545\ud654\ub418\uc5c8\ub2e4. 1803\ub144 5\uc6d4, \uc601\uad6d\uc740 \ud504\ub791\uc2a4\uc5d0 \ub300\ud55c \uc804\uc7c1\uc744 \uc120\ud3ec\ud588\ub2e4.", "sentence_answer": "1797\ub144 \uc804\uc7c1\uc774 \ubc1c\ubc1c\ud55c \uc9c0 5\ub144 \uc774 \uc9c0\ub098, \ud504\ub791\uc2a4 \uacf5\ud654\uad6d\uc740 \uc81c1\ucc28 \ub300\ud504\ub791\uc2a4 \ub3d9\ub9f9\uc744 \uaca9\ud30c\ud588\ub2e4.", "paragraph_id": "6533833-2-0", "paragraph_question": "question: 1797\ub144 \uc804\uc7c1\uc774 \ubc1c\ubc1c\ud55c \ud6c4 \uba87 \ub144\uc774 \uc9c0\ub098 \ud504\ub791\uc2a4 \uacf5\ud654\uad6d\uc774 \uc81c 1\ucc28 \ub300\ud504\ub791\uc2a4 \ub3d9\ub9f9\uc744 \uaca9\ud30c\ud588\uc744\uae4c\uc694?, context: \uc720\ub7fd\uc740 1792\ub144 \ud504\ub791\uc2a4 \ud601\uba85\uc804\uc7c1\uc774 \ubc1c\ubc1c\ud55c \uc774\ub798\ub85c \ud070 \ud63c\ub780\uc5d0 \ube60\uc838 \uc788\uc5c8\ub2e4. 1797\ub144 \uc804\uc7c1\uc774 \ubc1c\ubc1c\ud55c \uc9c0 5\ub144\uc774 \uc9c0\ub098, \ud504\ub791\uc2a4 \uacf5\ud654\uad6d\uc740 \uc81c1\ucc28 \ub300\ud504\ub791\uc2a4 \ub3d9\ub9f9\uc744 \uaca9\ud30c\ud588\ub2e4. \uc81c2\ucc28 \ub300\ud504\ub791\uc2a4 \ub3d9\ub9f9\uc740 1798\ub144\uc5d0 \uacb0\uc131\ub418\uc5c8\uc73c\ub098, 1801\ub144 \uc774 \uc5ed\uc2dc\ub3c4 \ud328\ubc30\ud558\uace0, \uc624\uc9c1 \uc601\uad6d\ub9cc\uc774 \uc0c8\ub85c\uc6b4 \ud504\ub791\uc2a4 \uc6d0\ub85c\uc6d0\uc758 \uc720\uc77c\ud55c \uc801\uc73c\ub85c \ub0a8\uc544\uc788\uc744 \ubfd0\uc774\uc5c8\ub2e4. \uc601\uad6d\uacfc \ud504\ub791\uc2a4\ub294 \uc544\ubbf8\uc575 \uc870\uc57d(Treaty of Amiens)\uc73c\ub85c \uc801\ub300 \uad00\uacc4\ub97c \ub05d\ub0b4\ub294 \ub370 \ub3d9\uc758\ud558\uc600\ub2e4. \uc9c0\ub09c 10\ub144\uac04 \ucc98\uc74c\uc73c\ub85c \ubaa8\ub4e0 \uc720\ub7fd\uc774 \ud3c9\ud654\uc0c1\ud0dc\ub97c \ub204\ub9ac\uac8c \ub418\uc5c8\ub2e4. \uadf8\ub7ec\ub098 \uc591\uce21\uc5d0\ub294 \uc870\uc57d\uc744 \uc774\ud589\ud558\ub294 \ub370 \uc810\uc810 \uc5b4\ub824\uc6c0\uc774 \ub204\uc801\ub418\ub294 \ub4f1 \ub9ce\uc740 \ubb38\uc81c\ub4e4\uc774 \uc9c0\uc18d\ub418\uace0 \uc788\uc5c8\ub2e4. \uc601\uad6d\uc815\ubd80\ub294 1793\ub144\ubd80\ud130 \uc774\ub8e8\uc5b4\uc9c4 \uc2dd\ubbfc\uc9c0 \uc815\ubcf5\uc744 \ub418\ub3cc\ub824\uc57c \ud55c\ub2e4\ub294 \ub370 \ubd84\uac1c\ud588\uace0, \ub098\ud3f4\ub808\uc639\uc740 \uc601\uad6d\uad70\uc774 \ubab0\ud0c0(Malta) \uc12c\uc5d0\uc11c \ucca0\uc218\ud558\uc9c0 \uc54a\ub294 \ub370 \ud654\uac00 \ub0ac\ub2e4. \uae34\uc7a5 \uc0c1\ud0dc\ub294 \ub098\ud3f4\ub808\uc639\uc774 \uc544\uc774\ud2f0 \ud601\uba85\uc744 \uc9c4\uc555\ud558\uae30 \uc704\ud574 \uc6d0\uc815\uad70\uc744 \ud30c\uacac\ud588\uc744 \ub54c \ub354\uc6b1 \uc545\ud654\ub418\uc5c8\ub2e4. 1803\ub144 5\uc6d4, \uc601\uad6d\uc740 \ud504\ub791\uc2a4\uc5d0 \ub300\ud55c \uc804\uc7c1\uc744 \uc120\ud3ec\ud588\ub2e4."} {"question": "\ub85c\ub9c8 \uacf5\ud654\uc815 \uc2dc\uae30 \uc9d1\uc815\uad00\uc81c\ub3c4\uc5d0\uc11c \uc9d1\uc815\uad00\uc740 \uba87 \uba85\uc774\uc5c8\ub098?", "paragraph": "\uc5d0\ud2b8\ub8e8\ub9ac\uc544\uac00 \uce84\ud30c\ub2c8\uc544\uc5d0\uc11c \ud328\ubc30\ud558\uace0, \ub77c\ud2f0\uc6c0\uc778\ub4e4\uc774 \ubc18\ub780\uc744 \uc77c\uc73c\ud0a4\uba70, \uac8c\ub2e4\uac00 \uc0b0\uc9c0 \uc885\uc871\ub4e4\uc774 \ub77c\ud2f0\uc6c0(latium)\uc744 \uce68\uacf5\ud558\ub294 \ub4f1 \uba87 \ucc28\ub840\uc5d0 \uac78\uce5c \uad70\uc0ac\uc801 \ud328\ubc30\ub85c \ub85c\ub9c8\uc5d0 \uc0ac\ud68c\u00b7\uc815\uce58\uc801 \uc704\uae30\uac00 \uc77c\uc5b4\ub098\uba74\uc11c \uc655\uc815\uc774 \ubab0\ub77d\ud558\uac8c \ub41c \uac83\uc73c\ub85c \ubcf4\uc778\ub2e4. \uadf8\ub7ec\ub098 \uc5ec\ub7ec \ud559\uc790\ub4e4\uc740 \uc5f0\ub300 \ubaa9\ub85d(fasti)\uc5d0 \uae30\ub85d\ub41c \uae30\uc6d0\uc804 509~507\ub144\uc744 \uc655\uc815 \ubab0\ub77d\uc758 \uc2dc\uc810\uc73c\ub85c \ubcf4\uae30\ub3c4 \ud55c\ub2e4. \ucd08\uae30\uc5d0\ub294 \uad70\uc0ac \uc704\uc6d0\ud68c (military tribune)\uc5d0 \uc758\ud55c \uacf5\ub3d9\ud1b5\uce58\uc758 \ud615\ud0dc\uc600\ub2e4\uac00 \ub3c5\uc7ac\uad00\uc744 \uc790\uc8fc \uc120\ucd9c\ud558\ub294 \ubb38\uc81c\uac00 \uc0dd\uae30\uc790 2\uba85\uc758 \uc9d1\uc815\uad00 \uc81c\ub3c4\ub85c \ubc14\uafb8\uc5c8\ub2e4. \ub85c\ub9c8\uc758 \uc815\uccb4\ub294 \uacac\uc81c\uc640 \uade0\ud615, \uad8c\ub825 \ubd84\uc810\uc73c\ub85c \uad6c\uc131\ub41c\ub2e4. \uac00\uc7a5 \uc911\uc694\ud55c \uc815\ubb34\uad00\uc740 \ub450 \uc9d1\uc815\uad00\uc73c\ub85c \uad70\uc0ac \ud1b5\uc194\uad8c\uc778 \uba85\ub839\uad8c(imperium)\uc744 \ud1b5\ud574 \uc9d1\ud589 \uad8c\ub825\uc744 \ud568\uaed8 \ud589\uc0ac\ud588\ub2e4. \uc9d1\uc815\uad00\uc9c1\uc740 \uc624\ub79c \uae30\uac04\uc758 \uc2e4\ud5d8\uacfc \ubc1c\uc804\uc744 \uac70\uce5c\ub4a4\uc5d0\uc57c \ube44\ub85c\uc18c \uc815\ucc29\ub41c \uac83\uc73c\ub85c \ubcf4\uc774\uba70, \uae30\uc6d0\uc804 5\uc138\uae30 \uc911\uc5fd\ucbe4\uc5d0\ub294 \uacf5\ud654\uc815\uc758 \uace0\uc704 \uc815\ubb34\uad00\uc9c1\uc744 \uc9d1\uc815\uad00\uc774\ub77c \ud558\uc600\ub2e4. \uc9d1\uc815\uad00\uc740 \uc6d0\ub798 \uadc0\uc871\ub4e4\uc758 \uc790\ubb38 \ud68c\uc758\uc600\ub358 \uc6d0\ub85c\uc6d0\uacfc \ud611\uc870\ud588\ub294\ub370, \uc2dc\uac04\uc774 \uc9c0\ub098\uba74\uc11c \uc6d0\ub85c\uc6d0\uc740 \uc591\uc801\uc73c\ub85c \ud655\ub300\ub418\uc5c8\ub2e4. \uacf5\ud654\uc815\uc758 \ub2e4\ub978 \uc815\ubb34\uad00\uc73c\ub85c\ub294 \ubc95\ubb34\uad00, \uc870\uc601\uad00, \uc7ac\ubb34\uad00\uc774 \uc788\uc5c8\ub2e4. \uc815\ubb34\uad00\uc9c1\uc740 \uc6d0\ub798 \uadc0\uc871 \ucd9c\uc2e0\uc73c\ub85c \uc81c\ud55c\ub418\uc5c8\uc73c\ub098 \ub098\uc911\uc5d0\ub294 \ud3c9\ubbfc\uc5d0\uac8c\ub3c4 \uac1c\ubc29\ub418\uc5c8\ub2e4. \uacf5\ud654\uc815\uc758 \ubbfc\ud68c\ub85c\ub294 \uc655\uc815 \uc2dc\ub300\uc5d0 \uae30\uc6d0\uc744 \ub450\uba70, \uc804\uc7c1\uacfc \ud654\uc758 \uc5ec\ubd80\ub97c \uacb0\uc815\ud558\uace0 \uc694\uc9c1 \uad00\ub9ac\ub97c \uc120\ucd9c\ud558\ub294 \ucf04\ud22c\ub9ac\uc544\ud68c(comitia centuriata), \ud558\uc704\uc9c1 \uc8fc\uc694 \uad00\ub9ac\ub97c \uc120\ucd9c\ud558\ub294 \ud2b8\ub9ac\ubd80\uc2a4\ud68c(comitia tributa)\uac00 \uc788\uc5c8\ub2e4.", "answer": "2\uba85", "sentence": "\ucd08\uae30\uc5d0\ub294 \uad70\uc0ac \uc704\uc6d0\ud68c (military tribune)\uc5d0 \uc758\ud55c \uacf5\ub3d9\ud1b5\uce58\uc758 \ud615\ud0dc\uc600\ub2e4\uac00 \ub3c5\uc7ac\uad00\uc744 \uc790\uc8fc \uc120\ucd9c\ud558\ub294 \ubb38\uc81c\uac00 \uc0dd\uae30\uc790 2\uba85 \uc758 \uc9d1\uc815\uad00 \uc81c\ub3c4\ub85c \ubc14\uafb8\uc5c8\ub2e4.", "paragraph_sentence": "\uc5d0\ud2b8\ub8e8\ub9ac\uc544\uac00 \uce84\ud30c\ub2c8\uc544\uc5d0\uc11c \ud328\ubc30\ud558\uace0, \ub77c\ud2f0\uc6c0\uc778\ub4e4\uc774 \ubc18\ub780\uc744 \uc77c\uc73c\ud0a4\uba70, \uac8c\ub2e4\uac00 \uc0b0\uc9c0 \uc885\uc871\ub4e4\uc774 \ub77c\ud2f0\uc6c0(latium)\uc744 \uce68\uacf5\ud558\ub294 \ub4f1 \uba87 \ucc28\ub840\uc5d0 \uac78\uce5c \uad70\uc0ac\uc801 \ud328\ubc30\ub85c \ub85c\ub9c8\uc5d0 \uc0ac\ud68c\u00b7\uc815\uce58\uc801 \uc704\uae30\uac00 \uc77c\uc5b4\ub098\uba74\uc11c \uc655\uc815\uc774 \ubab0\ub77d\ud558\uac8c \ub41c \uac83\uc73c\ub85c \ubcf4\uc778\ub2e4. \uadf8\ub7ec\ub098 \uc5ec\ub7ec \ud559\uc790\ub4e4\uc740 \uc5f0\ub300 \ubaa9\ub85d(fasti)\uc5d0 \uae30\ub85d\ub41c \uae30\uc6d0\uc804 509~507\ub144\uc744 \uc655\uc815 \ubab0\ub77d\uc758 \uc2dc\uc810\uc73c\ub85c \ubcf4\uae30\ub3c4 \ud55c\ub2e4. \ucd08\uae30\uc5d0\ub294 \uad70\uc0ac \uc704\uc6d0\ud68c (military tribune)\uc5d0 \uc758\ud55c \uacf5\ub3d9\ud1b5\uce58\uc758 \ud615\ud0dc\uc600\ub2e4\uac00 \ub3c5\uc7ac\uad00\uc744 \uc790\uc8fc \uc120\ucd9c\ud558\ub294 \ubb38\uc81c\uac00 \uc0dd\uae30\uc790 2\uba85 \uc758 \uc9d1\uc815\uad00 \uc81c\ub3c4\ub85c \ubc14\uafb8\uc5c8\ub2e4. \ub85c\ub9c8\uc758 \uc815\uccb4\ub294 \uacac\uc81c\uc640 \uade0\ud615, \uad8c\ub825 \ubd84\uc810\uc73c\ub85c \uad6c\uc131\ub41c\ub2e4. \uac00\uc7a5 \uc911\uc694\ud55c \uc815\ubb34\uad00\uc740 \ub450 \uc9d1\uc815\uad00\uc73c\ub85c \uad70\uc0ac \ud1b5\uc194\uad8c\uc778 \uba85\ub839\uad8c(imperium)\uc744 \ud1b5\ud574 \uc9d1\ud589 \uad8c\ub825\uc744 \ud568\uaed8 \ud589\uc0ac\ud588\ub2e4. \uc9d1\uc815\uad00\uc9c1\uc740 \uc624\ub79c \uae30\uac04\uc758 \uc2e4\ud5d8\uacfc \ubc1c\uc804\uc744 \uac70\uce5c\ub4a4\uc5d0\uc57c \ube44\ub85c\uc18c \uc815\ucc29\ub41c \uac83\uc73c\ub85c \ubcf4\uc774\uba70, \uae30\uc6d0\uc804 5\uc138\uae30 \uc911\uc5fd\ucbe4\uc5d0\ub294 \uacf5\ud654\uc815\uc758 \uace0\uc704 \uc815\ubb34\uad00\uc9c1\uc744 \uc9d1\uc815\uad00\uc774\ub77c \ud558\uc600\ub2e4. \uc9d1\uc815\uad00\uc740 \uc6d0\ub798 \uadc0\uc871\ub4e4\uc758 \uc790\ubb38 \ud68c\uc758\uc600\ub358 \uc6d0\ub85c\uc6d0\uacfc \ud611\uc870\ud588\ub294\ub370, \uc2dc\uac04\uc774 \uc9c0\ub098\uba74\uc11c \uc6d0\ub85c\uc6d0\uc740 \uc591\uc801\uc73c\ub85c \ud655\ub300\ub418\uc5c8\ub2e4. \uacf5\ud654\uc815\uc758 \ub2e4\ub978 \uc815\ubb34\uad00\uc73c\ub85c\ub294 \ubc95\ubb34\uad00, \uc870\uc601\uad00, \uc7ac\ubb34\uad00\uc774 \uc788\uc5c8\ub2e4. \uc815\ubb34\uad00\uc9c1\uc740 \uc6d0\ub798 \uadc0\uc871 \ucd9c\uc2e0\uc73c\ub85c \uc81c\ud55c\ub418\uc5c8\uc73c\ub098 \ub098\uc911\uc5d0\ub294 \ud3c9\ubbfc\uc5d0\uac8c\ub3c4 \uac1c\ubc29\ub418\uc5c8\ub2e4. \uacf5\ud654\uc815\uc758 \ubbfc\ud68c\ub85c\ub294 \uc655\uc815 \uc2dc\ub300\uc5d0 \uae30\uc6d0\uc744 \ub450\uba70, \uc804\uc7c1\uacfc \ud654\uc758 \uc5ec\ubd80\ub97c \uacb0\uc815\ud558\uace0 \uc694\uc9c1 \uad00\ub9ac\ub97c \uc120\ucd9c\ud558\ub294 \ucf04\ud22c\ub9ac\uc544\ud68c(comitia centuriata), \ud558\uc704\uc9c1 \uc8fc\uc694 \uad00\ub9ac\ub97c \uc120\ucd9c\ud558\ub294 \ud2b8\ub9ac\ubd80\uc2a4\ud68c(comitia tributa)\uac00 \uc788\uc5c8\ub2e4.", "paragraph_answer": "\uc5d0\ud2b8\ub8e8\ub9ac\uc544\uac00 \uce84\ud30c\ub2c8\uc544\uc5d0\uc11c \ud328\ubc30\ud558\uace0, \ub77c\ud2f0\uc6c0\uc778\ub4e4\uc774 \ubc18\ub780\uc744 \uc77c\uc73c\ud0a4\uba70, \uac8c\ub2e4\uac00 \uc0b0\uc9c0 \uc885\uc871\ub4e4\uc774 \ub77c\ud2f0\uc6c0(latium)\uc744 \uce68\uacf5\ud558\ub294 \ub4f1 \uba87 \ucc28\ub840\uc5d0 \uac78\uce5c \uad70\uc0ac\uc801 \ud328\ubc30\ub85c \ub85c\ub9c8\uc5d0 \uc0ac\ud68c\u00b7\uc815\uce58\uc801 \uc704\uae30\uac00 \uc77c\uc5b4\ub098\uba74\uc11c \uc655\uc815\uc774 \ubab0\ub77d\ud558\uac8c \ub41c \uac83\uc73c\ub85c \ubcf4\uc778\ub2e4. \uadf8\ub7ec\ub098 \uc5ec\ub7ec \ud559\uc790\ub4e4\uc740 \uc5f0\ub300 \ubaa9\ub85d(fasti)\uc5d0 \uae30\ub85d\ub41c \uae30\uc6d0\uc804 509~507\ub144\uc744 \uc655\uc815 \ubab0\ub77d\uc758 \uc2dc\uc810\uc73c\ub85c \ubcf4\uae30\ub3c4 \ud55c\ub2e4. \ucd08\uae30\uc5d0\ub294 \uad70\uc0ac \uc704\uc6d0\ud68c (military tribune)\uc5d0 \uc758\ud55c \uacf5\ub3d9\ud1b5\uce58\uc758 \ud615\ud0dc\uc600\ub2e4\uac00 \ub3c5\uc7ac\uad00\uc744 \uc790\uc8fc \uc120\ucd9c\ud558\ub294 \ubb38\uc81c\uac00 \uc0dd\uae30\uc790 2\uba85 \uc758 \uc9d1\uc815\uad00 \uc81c\ub3c4\ub85c \ubc14\uafb8\uc5c8\ub2e4. \ub85c\ub9c8\uc758 \uc815\uccb4\ub294 \uacac\uc81c\uc640 \uade0\ud615, \uad8c\ub825 \ubd84\uc810\uc73c\ub85c \uad6c\uc131\ub41c\ub2e4. \uac00\uc7a5 \uc911\uc694\ud55c \uc815\ubb34\uad00\uc740 \ub450 \uc9d1\uc815\uad00\uc73c\ub85c \uad70\uc0ac \ud1b5\uc194\uad8c\uc778 \uba85\ub839\uad8c(imperium)\uc744 \ud1b5\ud574 \uc9d1\ud589 \uad8c\ub825\uc744 \ud568\uaed8 \ud589\uc0ac\ud588\ub2e4. \uc9d1\uc815\uad00\uc9c1\uc740 \uc624\ub79c \uae30\uac04\uc758 \uc2e4\ud5d8\uacfc \ubc1c\uc804\uc744 \uac70\uce5c\ub4a4\uc5d0\uc57c \ube44\ub85c\uc18c \uc815\ucc29\ub41c \uac83\uc73c\ub85c \ubcf4\uc774\uba70, \uae30\uc6d0\uc804 5\uc138\uae30 \uc911\uc5fd\ucbe4\uc5d0\ub294 \uacf5\ud654\uc815\uc758 \uace0\uc704 \uc815\ubb34\uad00\uc9c1\uc744 \uc9d1\uc815\uad00\uc774\ub77c \ud558\uc600\ub2e4. \uc9d1\uc815\uad00\uc740 \uc6d0\ub798 \uadc0\uc871\ub4e4\uc758 \uc790\ubb38 \ud68c\uc758\uc600\ub358 \uc6d0\ub85c\uc6d0\uacfc \ud611\uc870\ud588\ub294\ub370, \uc2dc\uac04\uc774 \uc9c0\ub098\uba74\uc11c \uc6d0\ub85c\uc6d0\uc740 \uc591\uc801\uc73c\ub85c \ud655\ub300\ub418\uc5c8\ub2e4. \uacf5\ud654\uc815\uc758 \ub2e4\ub978 \uc815\ubb34\uad00\uc73c\ub85c\ub294 \ubc95\ubb34\uad00, \uc870\uc601\uad00, \uc7ac\ubb34\uad00\uc774 \uc788\uc5c8\ub2e4. \uc815\ubb34\uad00\uc9c1\uc740 \uc6d0\ub798 \uadc0\uc871 \ucd9c\uc2e0\uc73c\ub85c \uc81c\ud55c\ub418\uc5c8\uc73c\ub098 \ub098\uc911\uc5d0\ub294 \ud3c9\ubbfc\uc5d0\uac8c\ub3c4 \uac1c\ubc29\ub418\uc5c8\ub2e4. \uacf5\ud654\uc815\uc758 \ubbfc\ud68c\ub85c\ub294 \uc655\uc815 \uc2dc\ub300\uc5d0 \uae30\uc6d0\uc744 \ub450\uba70, \uc804\uc7c1\uacfc \ud654\uc758 \uc5ec\ubd80\ub97c \uacb0\uc815\ud558\uace0 \uc694\uc9c1 \uad00\ub9ac\ub97c \uc120\ucd9c\ud558\ub294 \ucf04\ud22c\ub9ac\uc544\ud68c(comitia centuriata), \ud558\uc704\uc9c1 \uc8fc\uc694 \uad00\ub9ac\ub97c \uc120\ucd9c\ud558\ub294 \ud2b8\ub9ac\ubd80\uc2a4\ud68c(comitia tributa)\uac00 \uc788\uc5c8\ub2e4.", "sentence_answer": "\ucd08\uae30\uc5d0\ub294 \uad70\uc0ac \uc704\uc6d0\ud68c (military tribune)\uc5d0 \uc758\ud55c \uacf5\ub3d9\ud1b5\uce58\uc758 \ud615\ud0dc\uc600\ub2e4\uac00 \ub3c5\uc7ac\uad00\uc744 \uc790\uc8fc \uc120\ucd9c\ud558\ub294 \ubb38\uc81c\uac00 \uc0dd\uae30\uc790 2\uba85 \uc758 \uc9d1\uc815\uad00 \uc81c\ub3c4\ub85c \ubc14\uafb8\uc5c8\ub2e4.", "paragraph_id": "6466082-1-0", "paragraph_question": "question: \ub85c\ub9c8 \uacf5\ud654\uc815 \uc2dc\uae30 \uc9d1\uc815\uad00\uc81c\ub3c4\uc5d0\uc11c \uc9d1\uc815\uad00\uc740 \uba87 \uba85\uc774\uc5c8\ub098?, context: \uc5d0\ud2b8\ub8e8\ub9ac\uc544\uac00 \uce84\ud30c\ub2c8\uc544\uc5d0\uc11c \ud328\ubc30\ud558\uace0, \ub77c\ud2f0\uc6c0\uc778\ub4e4\uc774 \ubc18\ub780\uc744 \uc77c\uc73c\ud0a4\uba70, \uac8c\ub2e4\uac00 \uc0b0\uc9c0 \uc885\uc871\ub4e4\uc774 \ub77c\ud2f0\uc6c0(latium)\uc744 \uce68\uacf5\ud558\ub294 \ub4f1 \uba87 \ucc28\ub840\uc5d0 \uac78\uce5c \uad70\uc0ac\uc801 \ud328\ubc30\ub85c \ub85c\ub9c8\uc5d0 \uc0ac\ud68c\u00b7\uc815\uce58\uc801 \uc704\uae30\uac00 \uc77c\uc5b4\ub098\uba74\uc11c \uc655\uc815\uc774 \ubab0\ub77d\ud558\uac8c \ub41c \uac83\uc73c\ub85c \ubcf4\uc778\ub2e4. \uadf8\ub7ec\ub098 \uc5ec\ub7ec \ud559\uc790\ub4e4\uc740 \uc5f0\ub300 \ubaa9\ub85d(fasti)\uc5d0 \uae30\ub85d\ub41c \uae30\uc6d0\uc804 509~507\ub144\uc744 \uc655\uc815 \ubab0\ub77d\uc758 \uc2dc\uc810\uc73c\ub85c \ubcf4\uae30\ub3c4 \ud55c\ub2e4. \ucd08\uae30\uc5d0\ub294 \uad70\uc0ac \uc704\uc6d0\ud68c (military tribune)\uc5d0 \uc758\ud55c \uacf5\ub3d9\ud1b5\uce58\uc758 \ud615\ud0dc\uc600\ub2e4\uac00 \ub3c5\uc7ac\uad00\uc744 \uc790\uc8fc \uc120\ucd9c\ud558\ub294 \ubb38\uc81c\uac00 \uc0dd\uae30\uc790 2\uba85\uc758 \uc9d1\uc815\uad00 \uc81c\ub3c4\ub85c \ubc14\uafb8\uc5c8\ub2e4. \ub85c\ub9c8\uc758 \uc815\uccb4\ub294 \uacac\uc81c\uc640 \uade0\ud615, \uad8c\ub825 \ubd84\uc810\uc73c\ub85c \uad6c\uc131\ub41c\ub2e4. \uac00\uc7a5 \uc911\uc694\ud55c \uc815\ubb34\uad00\uc740 \ub450 \uc9d1\uc815\uad00\uc73c\ub85c \uad70\uc0ac \ud1b5\uc194\uad8c\uc778 \uba85\ub839\uad8c(imperium)\uc744 \ud1b5\ud574 \uc9d1\ud589 \uad8c\ub825\uc744 \ud568\uaed8 \ud589\uc0ac\ud588\ub2e4. \uc9d1\uc815\uad00\uc9c1\uc740 \uc624\ub79c \uae30\uac04\uc758 \uc2e4\ud5d8\uacfc \ubc1c\uc804\uc744 \uac70\uce5c\ub4a4\uc5d0\uc57c \ube44\ub85c\uc18c \uc815\ucc29\ub41c \uac83\uc73c\ub85c \ubcf4\uc774\uba70, \uae30\uc6d0\uc804 5\uc138\uae30 \uc911\uc5fd\ucbe4\uc5d0\ub294 \uacf5\ud654\uc815\uc758 \uace0\uc704 \uc815\ubb34\uad00\uc9c1\uc744 \uc9d1\uc815\uad00\uc774\ub77c \ud558\uc600\ub2e4. \uc9d1\uc815\uad00\uc740 \uc6d0\ub798 \uadc0\uc871\ub4e4\uc758 \uc790\ubb38 \ud68c\uc758\uc600\ub358 \uc6d0\ub85c\uc6d0\uacfc \ud611\uc870\ud588\ub294\ub370, \uc2dc\uac04\uc774 \uc9c0\ub098\uba74\uc11c \uc6d0\ub85c\uc6d0\uc740 \uc591\uc801\uc73c\ub85c \ud655\ub300\ub418\uc5c8\ub2e4. \uacf5\ud654\uc815\uc758 \ub2e4\ub978 \uc815\ubb34\uad00\uc73c\ub85c\ub294 \ubc95\ubb34\uad00, \uc870\uc601\uad00, \uc7ac\ubb34\uad00\uc774 \uc788\uc5c8\ub2e4. \uc815\ubb34\uad00\uc9c1\uc740 \uc6d0\ub798 \uadc0\uc871 \ucd9c\uc2e0\uc73c\ub85c \uc81c\ud55c\ub418\uc5c8\uc73c\ub098 \ub098\uc911\uc5d0\ub294 \ud3c9\ubbfc\uc5d0\uac8c\ub3c4 \uac1c\ubc29\ub418\uc5c8\ub2e4. \uacf5\ud654\uc815\uc758 \ubbfc\ud68c\ub85c\ub294 \uc655\uc815 \uc2dc\ub300\uc5d0 \uae30\uc6d0\uc744 \ub450\uba70, \uc804\uc7c1\uacfc \ud654\uc758 \uc5ec\ubd80\ub97c \uacb0\uc815\ud558\uace0 \uc694\uc9c1 \uad00\ub9ac\ub97c \uc120\ucd9c\ud558\ub294 \ucf04\ud22c\ub9ac\uc544\ud68c(comitia centuriata), \ud558\uc704\uc9c1 \uc8fc\uc694 \uad00\ub9ac\ub97c \uc120\ucd9c\ud558\ub294 \ud2b8\ub9ac\ubd80\uc2a4\ud68c(comitia tributa)\uac00 \uc788\uc5c8\ub2e4."} {"question": "\ud2b8\ub9ac\ud50c \ub514\ub7ed\uc2a4\uc5d0\uc11c \uc5d1\uc2a4\ud2b8\ub77c \uc2a4\ud14c\uc774\uc9c0\uc758 \uc774\ub984\uc740 \ubb34\uc5c7\uc778\uac00?", "paragraph": "\uba54\uc778\ubaa8\ub4dc\ub97c \ud074\ub9ac\uc5b4\ud558\uba74 \uac1c\ubc29\ub418\ub294 \ubaa8\ub4dc. \uc9c4\ud589\ubc29\uc2dd\uc740 \uba54\uc778\ubaa8\ub4dc\uc640 \uc644\ubcbd\ud788 \ub3d9\uc77c\ud558\ub098 \ubcf4\uc2a4\ub4e4\uc758 \uc678\ud615\ubcc0\ud654 + \ud328\ud134 \ucd94\uac00, \uc77c\ubd80 \uc5d0\ub108\uc9c0 \uc2a4\ud53c\uc5b4\uc758 \ud68d\ub4dd \ub09c\uc774\ub3c4 \uc99d\uac00, \ucd5c\ub300 \uccb4\ub825\uc774 \uba54\uc778\uc5d0 \ube44\ud574 \uc808\ubc18\uac00\ub7c9\uc778 \ud558\ub4dc\ubaa8\ub4dc\uc774\ub2e4. \ub2f9\uc5f0\ud788 \ucc4c\ub9b0\uc9c0 \ubaa8\ub4dc\ub3c4 \ub178\uba40\uacfc \uc5d1\uc2a4\ud2b8\ub77c\uac00 \ub530\ub85c \uc874\uc7ac\ud55c\ub2e4. \uc774 \ubb38\uc11c\ub97c \uc5ec\uae30\uae4c\uc9c0\ub9cc \ubd10\ub3c4 \uc54c\uaca0\uc9c0\ub9cc \ucc4c\ub9b0\uc9c0\uc640 \ud568\uaed8 \ud37c\ud399\ud2b8 \ud074\ub9ac\uc5b4\uc758 \ubb38\ud131\uc744 \uc5c4\uccad\ub098\uac8c \uc62c\ub824\ubc84\ub9b0 \uc8fc\ubc94\uc774\ub2e4. \ud6c4\uc18d\uc791\uc778 \ubcc4\uc758 \ucee4\ube44 \ud2b8\ub9ac\ud50c \ub514\ub7ed\uc2a4, \ubcc4\uc758 \ucee4\ube44 \ub85c\ubcf4\ubcf4 \ud50c\ub798\ub2db, \ubcc4\uc758 \ucee4\ube44 \uc2a4\ud0c0 \uc5bc\ub77c\uc774\uc988\uc5d0\uc11c\ub294 \uc774 \ubaa8\ub4dc \ub300\uc2e0 \uc5d1\uc2a4\ud2b8\ub77c \uc2a4\ud14c\uc774\uc9c0\uac00 \uc788\ub2e4. \ub300\uc2e0 \ud2b8\ub9ac\ud50c \ub514\ub7ed\uc2a4\ub294 \uace0 \ub514\ub514\ub514\uac00 \uc788\uace0 \ub85c\ubcf4\ubcf4\ud50c\ub798\ub2db\uc5d0\uc11c\ub294 \uace0 \uba54\ud0c0\ub098\uc774\ud2b8 \ub9ac\ud134\uc988\uac00 \uc788\uace0 \uc2a4\ud0c0 \uc5bc\ub77c\uc774\uc988\ub294 \ubcc4\uc758 \u25cb\u25cb \ud504\ub80c\uc988\ub85c \uace0 \uac00 \uc788\ub2e4. \ud2b8\ub9ac\ud50c \ub514\ub7ed\uc2a4, \ub85c\ubcf4\ubcf4\ud50c\ub798\ub2db\uc5d0 \ube44\ud558\uba74 wii\ub294 \ub9ce\uc774 \uc544\uc27d\ub2e4. \uc544\uc26c\uc6b4 \uc774\uc720\ub294 \uc5d1\uc2a4\ud2b8\ub77c \ubaa8\ub4dc\uc5d0\uc11c \ubcc4\ub85c \ub2ec\ub77c\uc9c4\uac8c \ub9ce\uc9c0 \uc54a\uae30\ub54c\ubb38\uc774\ub2e4.", "answer": "\uace0 \ub514\ub514\ub514", "sentence": "\ub300\uc2e0 \ud2b8\ub9ac\ud50c \ub514\ub7ed\uc2a4\ub294 \uace0 \ub514\ub514\ub514 \uac00 \uc788\uace0", "paragraph_sentence": "\uba54\uc778\ubaa8\ub4dc\ub97c \ud074\ub9ac\uc5b4\ud558\uba74 \uac1c\ubc29\ub418\ub294 \ubaa8\ub4dc. \uc9c4\ud589\ubc29\uc2dd\uc740 \uba54\uc778\ubaa8\ub4dc\uc640 \uc644\ubcbd\ud788 \ub3d9\uc77c\ud558\ub098 \ubcf4\uc2a4\ub4e4\uc758 \uc678\ud615\ubcc0\ud654 + \ud328\ud134 \ucd94\uac00, \uc77c\ubd80 \uc5d0\ub108\uc9c0 \uc2a4\ud53c\uc5b4\uc758 \ud68d\ub4dd \ub09c\uc774\ub3c4 \uc99d\uac00, \ucd5c\ub300 \uccb4\ub825\uc774 \uba54\uc778\uc5d0 \ube44\ud574 \uc808\ubc18\uac00\ub7c9\uc778 \ud558\ub4dc\ubaa8\ub4dc\uc774\ub2e4. \ub2f9\uc5f0\ud788 \ucc4c\ub9b0\uc9c0 \ubaa8\ub4dc\ub3c4 \ub178\uba40\uacfc \uc5d1\uc2a4\ud2b8\ub77c\uac00 \ub530\ub85c \uc874\uc7ac\ud55c\ub2e4. \uc774 \ubb38\uc11c\ub97c \uc5ec\uae30\uae4c\uc9c0\ub9cc \ubd10\ub3c4 \uc54c\uaca0\uc9c0\ub9cc \ucc4c\ub9b0\uc9c0\uc640 \ud568\uaed8 \ud37c\ud399\ud2b8 \ud074\ub9ac\uc5b4\uc758 \ubb38\ud131\uc744 \uc5c4\uccad\ub098\uac8c \uc62c\ub824\ubc84\ub9b0 \uc8fc\ubc94\uc774\ub2e4. \ud6c4\uc18d\uc791\uc778 \ubcc4\uc758 \ucee4\ube44 \ud2b8\ub9ac\ud50c \ub514\ub7ed\uc2a4, \ubcc4\uc758 \ucee4\ube44 \ub85c\ubcf4\ubcf4 \ud50c\ub798\ub2db, \ubcc4\uc758 \ucee4\ube44 \uc2a4\ud0c0 \uc5bc\ub77c\uc774\uc988\uc5d0\uc11c\ub294 \uc774 \ubaa8\ub4dc \ub300\uc2e0 \uc5d1\uc2a4\ud2b8\ub77c \uc2a4\ud14c\uc774\uc9c0\uac00 \uc788\ub2e4. \ub300\uc2e0 \ud2b8\ub9ac\ud50c \ub514\ub7ed\uc2a4\ub294 \uace0 \ub514\ub514\ub514 \uac00 \uc788\uace0 \ub85c\ubcf4\ubcf4\ud50c\ub798\ub2db\uc5d0\uc11c\ub294 \uace0 \uba54\ud0c0\ub098\uc774\ud2b8 \ub9ac\ud134\uc988\uac00 \uc788\uace0 \uc2a4\ud0c0 \uc5bc\ub77c\uc774\uc988\ub294 \ubcc4\uc758 \u25cb\u25cb \ud504\ub80c\uc988\ub85c \uace0 \uac00 \uc788\ub2e4. \ud2b8\ub9ac\ud50c \ub514\ub7ed\uc2a4, \ub85c\ubcf4\ubcf4\ud50c\ub798\ub2db\uc5d0 \ube44\ud558\uba74 wii\ub294 \ub9ce\uc774 \uc544\uc27d\ub2e4. \uc544\uc26c\uc6b4 \uc774\uc720\ub294 \uc5d1\uc2a4\ud2b8\ub77c \ubaa8\ub4dc\uc5d0\uc11c \ubcc4\ub85c \ub2ec\ub77c\uc9c4\uac8c \ub9ce\uc9c0 \uc54a\uae30\ub54c\ubb38\uc774\ub2e4.", "paragraph_answer": "\uba54\uc778\ubaa8\ub4dc\ub97c \ud074\ub9ac\uc5b4\ud558\uba74 \uac1c\ubc29\ub418\ub294 \ubaa8\ub4dc. \uc9c4\ud589\ubc29\uc2dd\uc740 \uba54\uc778\ubaa8\ub4dc\uc640 \uc644\ubcbd\ud788 \ub3d9\uc77c\ud558\ub098 \ubcf4\uc2a4\ub4e4\uc758 \uc678\ud615\ubcc0\ud654 + \ud328\ud134 \ucd94\uac00, \uc77c\ubd80 \uc5d0\ub108\uc9c0 \uc2a4\ud53c\uc5b4\uc758 \ud68d\ub4dd \ub09c\uc774\ub3c4 \uc99d\uac00, \ucd5c\ub300 \uccb4\ub825\uc774 \uba54\uc778\uc5d0 \ube44\ud574 \uc808\ubc18\uac00\ub7c9\uc778 \ud558\ub4dc\ubaa8\ub4dc\uc774\ub2e4. \ub2f9\uc5f0\ud788 \ucc4c\ub9b0\uc9c0 \ubaa8\ub4dc\ub3c4 \ub178\uba40\uacfc \uc5d1\uc2a4\ud2b8\ub77c\uac00 \ub530\ub85c \uc874\uc7ac\ud55c\ub2e4. \uc774 \ubb38\uc11c\ub97c \uc5ec\uae30\uae4c\uc9c0\ub9cc \ubd10\ub3c4 \uc54c\uaca0\uc9c0\ub9cc \ucc4c\ub9b0\uc9c0\uc640 \ud568\uaed8 \ud37c\ud399\ud2b8 \ud074\ub9ac\uc5b4\uc758 \ubb38\ud131\uc744 \uc5c4\uccad\ub098\uac8c \uc62c\ub824\ubc84\ub9b0 \uc8fc\ubc94\uc774\ub2e4. \ud6c4\uc18d\uc791\uc778 \ubcc4\uc758 \ucee4\ube44 \ud2b8\ub9ac\ud50c \ub514\ub7ed\uc2a4, \ubcc4\uc758 \ucee4\ube44 \ub85c\ubcf4\ubcf4 \ud50c\ub798\ub2db, \ubcc4\uc758 \ucee4\ube44 \uc2a4\ud0c0 \uc5bc\ub77c\uc774\uc988\uc5d0\uc11c\ub294 \uc774 \ubaa8\ub4dc \ub300\uc2e0 \uc5d1\uc2a4\ud2b8\ub77c \uc2a4\ud14c\uc774\uc9c0\uac00 \uc788\ub2e4. \ub300\uc2e0 \ud2b8\ub9ac\ud50c \ub514\ub7ed\uc2a4\ub294 \uace0 \ub514\ub514\ub514 \uac00 \uc788\uace0 \ub85c\ubcf4\ubcf4\ud50c\ub798\ub2db\uc5d0\uc11c\ub294 \uace0 \uba54\ud0c0\ub098\uc774\ud2b8 \ub9ac\ud134\uc988\uac00 \uc788\uace0 \uc2a4\ud0c0 \uc5bc\ub77c\uc774\uc988\ub294 \ubcc4\uc758 \u25cb\u25cb \ud504\ub80c\uc988\ub85c \uace0 \uac00 \uc788\ub2e4. \ud2b8\ub9ac\ud50c \ub514\ub7ed\uc2a4, \ub85c\ubcf4\ubcf4\ud50c\ub798\ub2db\uc5d0 \ube44\ud558\uba74 wii\ub294 \ub9ce\uc774 \uc544\uc27d\ub2e4. \uc544\uc26c\uc6b4 \uc774\uc720\ub294 \uc5d1\uc2a4\ud2b8\ub77c \ubaa8\ub4dc\uc5d0\uc11c \ubcc4\ub85c \ub2ec\ub77c\uc9c4\uac8c \ub9ce\uc9c0 \uc54a\uae30\ub54c\ubb38\uc774\ub2e4.", "sentence_answer": "\ub300\uc2e0 \ud2b8\ub9ac\ud50c \ub514\ub7ed\uc2a4\ub294 \uace0 \ub514\ub514\ub514 \uac00 \uc788\uace0", "paragraph_id": "6603557-3-2", "paragraph_question": "question: \ud2b8\ub9ac\ud50c \ub514\ub7ed\uc2a4\uc5d0\uc11c \uc5d1\uc2a4\ud2b8\ub77c \uc2a4\ud14c\uc774\uc9c0\uc758 \uc774\ub984\uc740 \ubb34\uc5c7\uc778\uac00?, context: \uba54\uc778\ubaa8\ub4dc\ub97c \ud074\ub9ac\uc5b4\ud558\uba74 \uac1c\ubc29\ub418\ub294 \ubaa8\ub4dc. \uc9c4\ud589\ubc29\uc2dd\uc740 \uba54\uc778\ubaa8\ub4dc\uc640 \uc644\ubcbd\ud788 \ub3d9\uc77c\ud558\ub098 \ubcf4\uc2a4\ub4e4\uc758 \uc678\ud615\ubcc0\ud654 + \ud328\ud134 \ucd94\uac00, \uc77c\ubd80 \uc5d0\ub108\uc9c0 \uc2a4\ud53c\uc5b4\uc758 \ud68d\ub4dd \ub09c\uc774\ub3c4 \uc99d\uac00, \ucd5c\ub300 \uccb4\ub825\uc774 \uba54\uc778\uc5d0 \ube44\ud574 \uc808\ubc18\uac00\ub7c9\uc778 \ud558\ub4dc\ubaa8\ub4dc\uc774\ub2e4. \ub2f9\uc5f0\ud788 \ucc4c\ub9b0\uc9c0 \ubaa8\ub4dc\ub3c4 \ub178\uba40\uacfc \uc5d1\uc2a4\ud2b8\ub77c\uac00 \ub530\ub85c \uc874\uc7ac\ud55c\ub2e4. \uc774 \ubb38\uc11c\ub97c \uc5ec\uae30\uae4c\uc9c0\ub9cc \ubd10\ub3c4 \uc54c\uaca0\uc9c0\ub9cc \ucc4c\ub9b0\uc9c0\uc640 \ud568\uaed8 \ud37c\ud399\ud2b8 \ud074\ub9ac\uc5b4\uc758 \ubb38\ud131\uc744 \uc5c4\uccad\ub098\uac8c \uc62c\ub824\ubc84\ub9b0 \uc8fc\ubc94\uc774\ub2e4. \ud6c4\uc18d\uc791\uc778 \ubcc4\uc758 \ucee4\ube44 \ud2b8\ub9ac\ud50c \ub514\ub7ed\uc2a4, \ubcc4\uc758 \ucee4\ube44 \ub85c\ubcf4\ubcf4 \ud50c\ub798\ub2db, \ubcc4\uc758 \ucee4\ube44 \uc2a4\ud0c0 \uc5bc\ub77c\uc774\uc988\uc5d0\uc11c\ub294 \uc774 \ubaa8\ub4dc \ub300\uc2e0 \uc5d1\uc2a4\ud2b8\ub77c \uc2a4\ud14c\uc774\uc9c0\uac00 \uc788\ub2e4. \ub300\uc2e0 \ud2b8\ub9ac\ud50c \ub514\ub7ed\uc2a4\ub294 \uace0 \ub514\ub514\ub514\uac00 \uc788\uace0 \ub85c\ubcf4\ubcf4\ud50c\ub798\ub2db\uc5d0\uc11c\ub294 \uace0 \uba54\ud0c0\ub098\uc774\ud2b8 \ub9ac\ud134\uc988\uac00 \uc788\uace0 \uc2a4\ud0c0 \uc5bc\ub77c\uc774\uc988\ub294 \ubcc4\uc758 \u25cb\u25cb \ud504\ub80c\uc988\ub85c \uace0 \uac00 \uc788\ub2e4. \ud2b8\ub9ac\ud50c \ub514\ub7ed\uc2a4, \ub85c\ubcf4\ubcf4\ud50c\ub798\ub2db\uc5d0 \ube44\ud558\uba74 wii\ub294 \ub9ce\uc774 \uc544\uc27d\ub2e4. \uc544\uc26c\uc6b4 \uc774\uc720\ub294 \uc5d1\uc2a4\ud2b8\ub77c \ubaa8\ub4dc\uc5d0\uc11c \ubcc4\ub85c \ub2ec\ub77c\uc9c4\uac8c \ub9ce\uc9c0 \uc54a\uae30\ub54c\ubb38\uc774\ub2e4."} {"question": "\ucc48\ub4e4\ub7ec\uc640 \ubaa8\ub2c8\uce74\ub294 \ub9c8\uc9c0\ub9c9 \uc5d0\ud53c\uc18c\ub4dc\uc5d0\uc11c \uc30d\ub465\uc774 \ub204\uad6c \ub204\uad6c\ub97c \uc785\uc591\ud558\ub294\uac00\uc694?", "paragraph": "4\uc2dc\uc98c\uc758 \ub9c8\uc9c0\ub9c9 \uc5d0\ud53c\uc18c\ub4dc\uc778 '\ub85c\uc2a4\uc758 \uacb0\ud63c\uc2dd'\ud3b8\uc5d0\uc11c \ucc48\ub4e4\ub7ec\ub294 \uac00\uc7a5 \uac00\uae4c\uc6b4 \uce5c\uad6c\uc911 \ud55c\uba85\uc778 \ubaa8\ub2c8\uce74 \uac94\ub7ec\uc640 \uc0ac\uadc0\uae30 \uc2dc\uc791\ud55c\ub2e4. \uccab \uc2dc\uc98c\uc5d0\uc11c \uc774\ubbf8 \uadf8\ub294 \ubaa8\ub2c8\uce74\uc5d0 \ub300\ud55c \uad00\uc2ec\uc744 \ud45c\ucd9c\ud55c\ub2e4.3\uc2dc\uc98c\uc5d0\uc11c\ub3c4 \ucc48\ub4e4\ub7ec\ub294 \uadf8\ub140\uc5d0 \ub300\ud55c \uc9c4\uc2ec\uc744 \ubcf4\uc774\uc9c0\ub9cc,\ubaa8\ub2c8\uce74\ub294 \uadf8\uc800 \uadf8\uac00 \ub18d\ub2f4\ud558\ub294\uac70\ub77c \uc0dd\uac01\ud574\uc11c \uc2dc\ud070\ub465\ud558\uac8c \ubc18\uc751\ud55c\ub2e4. \ub7f0\ub358\uc5d0\uc11c \ub85c\uc2a4\uc640 \uc5d0\ubc00\ub9ac\uc758 \uacb0\ud63c\uc2dd \uc804\uc57c\uc5d0 \ucc48\ub4e4\ub7ec\uc640 \ubaa8\ub2c8\uce74\ub294 \uc220\uc5d0 \ucde8\ud574 \uac19\uc774 \uc7a0\uc790\ub9ac\ub97c \ud558\uac8c \ub41c\ub2e4.(\uba87 \ub144 \ub4a4, \ubaa8\ub2c8\uce74\ub294 \uc0ac\uc2e4 \ucc48\ub4e4\ub7ec\uc758 \ud638\ud154 \ubc29\uc5d0 \uc870\uc774\ub97c \ub9cc\ub098\ub7ec \uac14\uc5c8\ub2e4\uace0 \ud138\uc5b4\ub193\uac8c \ub41c\ub2e4.) \ucc48\ub4e4\ub7ec\uc640 \ubaa8\ub2c8\uce74\ub294 \ub2e4\ub978 \uce5c\uad6c\ub4e4\uc5d0\uac8c \uc0ac\uadc0\uace0 \uc788\ub2e4\ub294 \uc0ac\uc2e4\uc744 \uc228\uae30\ub824\uace0 \ud558\uc9c0\ub9cc,\uacb0\uad6d \ubaa8\ub450\uac00 \uc54c\uac8c \ub41c\ub2e4. \ucc48\ub4e4\ub7ec\ub294 \ubaa8\ub2c8\uce74\uc640 6\uc2dc\uc98c\uc5d0\uc11c \ub3d9\uac70\ub97c \uc2dc\uc791\ud558\uace0 7\uc2dc\uc98c \ub9c8\uc9c0\ub9c9\uc5d0 \uc870\uc774\uc758 \uc8fc\ub840\ub85c \uacb0\ud63c\ud55c\ub2e4.\uc2dc\ub9ac\uc988 \ub9c8\uc9c0\ub9c9 \uc5d0\ud53c\uc18c\ub4dc\uc5d0\uc11c \uadf8\ub4e4\uc740 \uc30d\ub465\uc774 \uc5d0\ub9ac\uce74\uc640 \uc7ad\uc744 \uc785\uc591\ud55c\ub2e4.", "answer": "\uc5d0\ub9ac\uce74\uc640 \uc7ad", "sentence": "\uadf8\ub4e4\uc740 \uc30d\ub465\uc774 \uc5d0\ub9ac\uce74\uc640 \uc7ad \uc744 \uc785\uc591\ud55c\ub2e4.", "paragraph_sentence": "4\uc2dc\uc98c\uc758 \ub9c8\uc9c0\ub9c9 \uc5d0\ud53c\uc18c\ub4dc\uc778 '\ub85c\uc2a4\uc758 \uacb0\ud63c\uc2dd'\ud3b8\uc5d0\uc11c \ucc48\ub4e4\ub7ec\ub294 \uac00\uc7a5 \uac00\uae4c\uc6b4 \uce5c\uad6c\uc911 \ud55c\uba85\uc778 \ubaa8\ub2c8\uce74 \uac94\ub7ec\uc640 \uc0ac\uadc0\uae30 \uc2dc\uc791\ud55c\ub2e4. \uccab \uc2dc\uc98c\uc5d0\uc11c \uc774\ubbf8 \uadf8\ub294 \ubaa8\ub2c8\uce74\uc5d0 \ub300\ud55c \uad00\uc2ec\uc744 \ud45c\ucd9c\ud55c\ub2e4.3\uc2dc\uc98c\uc5d0\uc11c\ub3c4 \ucc48\ub4e4\ub7ec\ub294 \uadf8\ub140\uc5d0 \ub300\ud55c \uc9c4\uc2ec\uc744 \ubcf4\uc774\uc9c0\ub9cc,\ubaa8\ub2c8\uce74\ub294 \uadf8\uc800 \uadf8\uac00 \ub18d\ub2f4\ud558\ub294\uac70\ub77c \uc0dd\uac01\ud574\uc11c \uc2dc\ud070\ub465\ud558\uac8c \ubc18\uc751\ud55c\ub2e4. \ub7f0\ub358\uc5d0\uc11c \ub85c\uc2a4\uc640 \uc5d0\ubc00\ub9ac\uc758 \uacb0\ud63c\uc2dd \uc804\uc57c\uc5d0 \ucc48\ub4e4\ub7ec\uc640 \ubaa8\ub2c8\uce74\ub294 \uc220\uc5d0 \ucde8\ud574 \uac19\uc774 \uc7a0\uc790\ub9ac\ub97c \ud558\uac8c \ub41c\ub2e4.(\uba87 \ub144 \ub4a4, \ubaa8\ub2c8\uce74\ub294 \uc0ac\uc2e4 \ucc48\ub4e4\ub7ec\uc758 \ud638\ud154 \ubc29\uc5d0 \uc870\uc774\ub97c \ub9cc\ub098\ub7ec \uac14\uc5c8\ub2e4\uace0 \ud138\uc5b4\ub193\uac8c \ub41c\ub2e4. ) \ucc48\ub4e4\ub7ec\uc640 \ubaa8\ub2c8\uce74\ub294 \ub2e4\ub978 \uce5c\uad6c\ub4e4\uc5d0\uac8c \uc0ac\uadc0\uace0 \uc788\ub2e4\ub294 \uc0ac\uc2e4\uc744 \uc228\uae30\ub824\uace0 \ud558\uc9c0\ub9cc,\uacb0\uad6d \ubaa8\ub450\uac00 \uc54c\uac8c \ub41c\ub2e4. \ucc48\ub4e4\ub7ec\ub294 \ubaa8\ub2c8\uce74\uc640 6\uc2dc\uc98c\uc5d0\uc11c \ub3d9\uac70\ub97c \uc2dc\uc791\ud558\uace0 7\uc2dc\uc98c \ub9c8\uc9c0\ub9c9\uc5d0 \uc870\uc774\uc758 \uc8fc\ub840\ub85c \uacb0\ud63c\ud55c\ub2e4.\uc2dc\ub9ac\uc988 \ub9c8\uc9c0\ub9c9 \uc5d0\ud53c\uc18c\ub4dc\uc5d0\uc11c \uadf8\ub4e4\uc740 \uc30d\ub465\uc774 \uc5d0\ub9ac\uce74\uc640 \uc7ad \uc744 \uc785\uc591\ud55c\ub2e4. ", "paragraph_answer": "4\uc2dc\uc98c\uc758 \ub9c8\uc9c0\ub9c9 \uc5d0\ud53c\uc18c\ub4dc\uc778 '\ub85c\uc2a4\uc758 \uacb0\ud63c\uc2dd'\ud3b8\uc5d0\uc11c \ucc48\ub4e4\ub7ec\ub294 \uac00\uc7a5 \uac00\uae4c\uc6b4 \uce5c\uad6c\uc911 \ud55c\uba85\uc778 \ubaa8\ub2c8\uce74 \uac94\ub7ec\uc640 \uc0ac\uadc0\uae30 \uc2dc\uc791\ud55c\ub2e4. \uccab \uc2dc\uc98c\uc5d0\uc11c \uc774\ubbf8 \uadf8\ub294 \ubaa8\ub2c8\uce74\uc5d0 \ub300\ud55c \uad00\uc2ec\uc744 \ud45c\ucd9c\ud55c\ub2e4.3\uc2dc\uc98c\uc5d0\uc11c\ub3c4 \ucc48\ub4e4\ub7ec\ub294 \uadf8\ub140\uc5d0 \ub300\ud55c \uc9c4\uc2ec\uc744 \ubcf4\uc774\uc9c0\ub9cc,\ubaa8\ub2c8\uce74\ub294 \uadf8\uc800 \uadf8\uac00 \ub18d\ub2f4\ud558\ub294\uac70\ub77c \uc0dd\uac01\ud574\uc11c \uc2dc\ud070\ub465\ud558\uac8c \ubc18\uc751\ud55c\ub2e4. \ub7f0\ub358\uc5d0\uc11c \ub85c\uc2a4\uc640 \uc5d0\ubc00\ub9ac\uc758 \uacb0\ud63c\uc2dd \uc804\uc57c\uc5d0 \ucc48\ub4e4\ub7ec\uc640 \ubaa8\ub2c8\uce74\ub294 \uc220\uc5d0 \ucde8\ud574 \uac19\uc774 \uc7a0\uc790\ub9ac\ub97c \ud558\uac8c \ub41c\ub2e4.(\uba87 \ub144 \ub4a4, \ubaa8\ub2c8\uce74\ub294 \uc0ac\uc2e4 \ucc48\ub4e4\ub7ec\uc758 \ud638\ud154 \ubc29\uc5d0 \uc870\uc774\ub97c \ub9cc\ub098\ub7ec \uac14\uc5c8\ub2e4\uace0 \ud138\uc5b4\ub193\uac8c \ub41c\ub2e4.) \ucc48\ub4e4\ub7ec\uc640 \ubaa8\ub2c8\uce74\ub294 \ub2e4\ub978 \uce5c\uad6c\ub4e4\uc5d0\uac8c \uc0ac\uadc0\uace0 \uc788\ub2e4\ub294 \uc0ac\uc2e4\uc744 \uc228\uae30\ub824\uace0 \ud558\uc9c0\ub9cc,\uacb0\uad6d \ubaa8\ub450\uac00 \uc54c\uac8c \ub41c\ub2e4. \ucc48\ub4e4\ub7ec\ub294 \ubaa8\ub2c8\uce74\uc640 6\uc2dc\uc98c\uc5d0\uc11c \ub3d9\uac70\ub97c \uc2dc\uc791\ud558\uace0 7\uc2dc\uc98c \ub9c8\uc9c0\ub9c9\uc5d0 \uc870\uc774\uc758 \uc8fc\ub840\ub85c \uacb0\ud63c\ud55c\ub2e4.\uc2dc\ub9ac\uc988 \ub9c8\uc9c0\ub9c9 \uc5d0\ud53c\uc18c\ub4dc\uc5d0\uc11c \uadf8\ub4e4\uc740 \uc30d\ub465\uc774 \uc5d0\ub9ac\uce74\uc640 \uc7ad \uc744 \uc785\uc591\ud55c\ub2e4.", "sentence_answer": "\uadf8\ub4e4\uc740 \uc30d\ub465\uc774 \uc5d0\ub9ac\uce74\uc640 \uc7ad \uc744 \uc785\uc591\ud55c\ub2e4.", "paragraph_id": "5897296-1-2", "paragraph_question": "question: \ucc48\ub4e4\ub7ec\uc640 \ubaa8\ub2c8\uce74\ub294 \ub9c8\uc9c0\ub9c9 \uc5d0\ud53c\uc18c\ub4dc\uc5d0\uc11c \uc30d\ub465\uc774 \ub204\uad6c \ub204\uad6c\ub97c \uc785\uc591\ud558\ub294\uac00\uc694?, context: 4\uc2dc\uc98c\uc758 \ub9c8\uc9c0\ub9c9 \uc5d0\ud53c\uc18c\ub4dc\uc778 '\ub85c\uc2a4\uc758 \uacb0\ud63c\uc2dd'\ud3b8\uc5d0\uc11c \ucc48\ub4e4\ub7ec\ub294 \uac00\uc7a5 \uac00\uae4c\uc6b4 \uce5c\uad6c\uc911 \ud55c\uba85\uc778 \ubaa8\ub2c8\uce74 \uac94\ub7ec\uc640 \uc0ac\uadc0\uae30 \uc2dc\uc791\ud55c\ub2e4. \uccab \uc2dc\uc98c\uc5d0\uc11c \uc774\ubbf8 \uadf8\ub294 \ubaa8\ub2c8\uce74\uc5d0 \ub300\ud55c \uad00\uc2ec\uc744 \ud45c\ucd9c\ud55c\ub2e4.3\uc2dc\uc98c\uc5d0\uc11c\ub3c4 \ucc48\ub4e4\ub7ec\ub294 \uadf8\ub140\uc5d0 \ub300\ud55c \uc9c4\uc2ec\uc744 \ubcf4\uc774\uc9c0\ub9cc,\ubaa8\ub2c8\uce74\ub294 \uadf8\uc800 \uadf8\uac00 \ub18d\ub2f4\ud558\ub294\uac70\ub77c \uc0dd\uac01\ud574\uc11c \uc2dc\ud070\ub465\ud558\uac8c \ubc18\uc751\ud55c\ub2e4. \ub7f0\ub358\uc5d0\uc11c \ub85c\uc2a4\uc640 \uc5d0\ubc00\ub9ac\uc758 \uacb0\ud63c\uc2dd \uc804\uc57c\uc5d0 \ucc48\ub4e4\ub7ec\uc640 \ubaa8\ub2c8\uce74\ub294 \uc220\uc5d0 \ucde8\ud574 \uac19\uc774 \uc7a0\uc790\ub9ac\ub97c \ud558\uac8c \ub41c\ub2e4.(\uba87 \ub144 \ub4a4, \ubaa8\ub2c8\uce74\ub294 \uc0ac\uc2e4 \ucc48\ub4e4\ub7ec\uc758 \ud638\ud154 \ubc29\uc5d0 \uc870\uc774\ub97c \ub9cc\ub098\ub7ec \uac14\uc5c8\ub2e4\uace0 \ud138\uc5b4\ub193\uac8c \ub41c\ub2e4.) \ucc48\ub4e4\ub7ec\uc640 \ubaa8\ub2c8\uce74\ub294 \ub2e4\ub978 \uce5c\uad6c\ub4e4\uc5d0\uac8c \uc0ac\uadc0\uace0 \uc788\ub2e4\ub294 \uc0ac\uc2e4\uc744 \uc228\uae30\ub824\uace0 \ud558\uc9c0\ub9cc,\uacb0\uad6d \ubaa8\ub450\uac00 \uc54c\uac8c \ub41c\ub2e4. \ucc48\ub4e4\ub7ec\ub294 \ubaa8\ub2c8\uce74\uc640 6\uc2dc\uc98c\uc5d0\uc11c \ub3d9\uac70\ub97c \uc2dc\uc791\ud558\uace0 7\uc2dc\uc98c \ub9c8\uc9c0\ub9c9\uc5d0 \uc870\uc774\uc758 \uc8fc\ub840\ub85c \uacb0\ud63c\ud55c\ub2e4.\uc2dc\ub9ac\uc988 \ub9c8\uc9c0\ub9c9 \uc5d0\ud53c\uc18c\ub4dc\uc5d0\uc11c \uadf8\ub4e4\uc740 \uc30d\ub465\uc774 \uc5d0\ub9ac\uce74\uc640 \uc7ad\uc744 \uc785\uc591\ud55c\ub2e4."} {"question": "\uc778\uc870\uac00 \uc0bc\uc804\ub3c4\uc5d0\uc11c \ud56d\ubcf5\uc758 \uc608\ub97c \uac00\uc9c0\uae30 \uc804 \ub0a8\ud55c\uc0b0\uc131\uc5d0\uc11c \uba87\uc77c\uc744 \ubc84\ud2f0\uc5c8\ub294\uac00?", "paragraph": "\uc774\uc5d0 \uc870\uc120\uc740 \uba3c\uc800 \ubd09\ub9bc\ub300\uad70, \uc778\ud3c9\ub300\uad70 \ub4f1\uc758 \uc655\uc790\ub97c \uba3c\uc800 \uac15\ud654\ub3c4\ub85c \ud53c\uc2e0\ud558\uac8c \ud558\uace0, \uc870\uc815 \ub610\ud55c \uac15\ud654\ub3c4\ub85c \ud53c\ub09c\ud558\ub824 \ud588\uc73c\ub098, \uccad\uad70\uc758 \uc120\ubc1c\ub300\uac00 \uc774\ubbf8 \uac15\ud654\ub3c4\ub85c \uac00\ub294 \uae38\ubaa9\uc744 \uac00\ub85c\ub9c9\uc73c\uba74\uc11c \uac15\ud654\ub3c4\ub97c \ud3ec\uae30\ud558\uace0 \ub0a8\ud55c\uc0b0\uc131\uc73c\ub85c \uac08 \uc218\ubc16\uc5d0 \uc5c6\uc5c8\ub2e4. \ub2f9\uc2dc \uc131 \uc548\uc5d0\ub294 1\ub9cc 3\ucc9c\uc5ec\uba85\uc758 \uad70\uc0ac\uac00 \ubc29\uc5b4\ub97c \ud558\uace0 \uc788\uc5c8\uace0, \uc591\uace1 1\ub9cc 4300\uc5ec \uc11d\uacfc \uc18c\uae08 90\uc5ec \uc11d\uc774 \uc788\uc5b4 \uaca8\uc6b0 50\uc77c \ubd84\uc758 \uc2dd\ub7c9\uc774 \ube44\ucd95\ub418\uc5b4 \uc788\uc5c8\ub2e4. \uccad\uad70\uc740 \ud070 \uc800\ud56d\uc744 \ubc1b\uc9c0 \uc54a\uc740 \ucc44 \ub0a8\ud55c\uc0b0\uc131\uc5d0 \ub2f9\ub3c4\uc5d0 \uc0b0\uc131 \ubc11\uc758 \ud0c4\ucc9c \ubd80\uadfc\uc5d0 \ud3ec\uc9c4\ud558\uc600\ub2e4. \uc774\uc5d0 \uc804\uad6d\uc758 \uad6c\uc6d0\ubcd1\ub4e4\uc774 \ucd9c\ubcd1\ud558\uc600\uc9c0\ub9cc, \ubaa8\ub450 \ub0a8\ud55c\uc0b0\uc131\uc5d0 \ub2f9\ub3c4\ud558\uae30 \uc804\uc5d0 \uada4\uba78\ub418\uc5c8\ub2e4. \uad6c\uc6d0\ubcd1\uc774 \ubcc4\ub85c \ub3c4\uc6c0\uc744 \uc8fc\uc9c0 \ubabb\ud558\uc790 \uc131\uc548\uc758 \uc870\uc815\uc5d0\uc11c\ub294 \uac15\ud654\ub860\uc774 \uc81c\uae30 \ub418\uc5c8\ub2e4. \uc8fc\ud654\ud30c\uc640 \uc8fc\uc804\ud30c \uc0ac\uc774\uc5d0 \uc5ec\ub7ec\ucc28\ub840 \uaca9\ub82c\ud55c \ub17c\uc7c1\uc774 \ubc8c\uc5b4\uc84c\uc9c0\ub9cc \uc774\ub4ec\ud574 1\uc6d4 22\uc77c \uac15\ud654\ub3c4\uac00 \uccad\uad70\uc5d0 \ud568\ub77d\ub2f9\ud558\uace0 \uac15\ud654\ub3c4\uc5d0 \ud53c\uc2e0\ud574 \uc788\ub358 \uc655\uc790\uc640 \uad70\uc2e0\ub4e4\uc758 \ucc98\uc790 200\uc5ec \uba85\uc774 \uccad\uad70\uc758 \ud3ec\ub85c\ub85c \uc7a1\ud788\uba74\uc11c \ub300\uc138\ub294 \uac15\ud654 \ucabd\uc73c\ub85c \uae30\uc6b8\uac8c \ub418\uc5c8\ub2e4. \uadf8 \uc774\ud6c4 \uccad\uad70\uc740 \ud654\ud3ec\ub97c \uc3d8\uc544\ub300\uc5b4 \uc131\ubcbd\uc758 \uc77c\ubd80\ub97c \ubb34\ub108\ub728\ub9ac\ub294 \ub4f1 \uc555\ubc15\uc744 \uac00\ud558\uc600\ub2e4. 1\uc6d4 26\uc77c \uc870\uc120\uc758 \uc0ac\uc2e0\uc774 \ud611\uc0c1\uc744 \uc704\ud574 \uccad \uc9c4\uc601\uc5d0 \ub3c4\ucc29\ud588\uc744\ub54c \uccad\uad70\uc740 \uac15\ud654\uc5d0\uc11c \ud3ec\ub85c\uac00 \ub41c \uc655\uc790\ub97c \ub300\uba74\uc2dc\ucf30\ub2e4. \uc774\uc5d0 \uc870\uc120\uc740 \ub354 \uc774\uc0c1 \ubc84\ud2f8 \ud798\uc744 \uc0c1\uc2e4\ud558\uace0, 1\uc6d4 30\uc77c \uc778\uc870\uac00 45\uc77c \ub9cc\uc5d0 \uc2dd\ub7c9 \ubd80\uc871\uc73c\ub85c \uc131\ubb38\uc744 \uc5f4\uace0 \ub098\uac00 \uc0bc\uc804\ub3c4\uc5d0\uc11c \ud56d\ubcf5\uc758 \uc608\ub97c \uac16\uac8c \ub418\uc5c8\ub2e4. \uc774 \uacf3\uc5d0\uc11c \uc778\uc870\ub294 \uc18c\ubcf5\uc744 \uc785\uace0 \uccad\ub098\ub77c \ud669\uc81c \ud669\ud0dc\uadf9\uc740 \ub192\uc740 \ub2e8\uc0c1\uc5d0 \uc549\uc544\uc11c \uc0bc\uada4\uad6c\uace0\ub450\uc758 \uc608\ub97c \ubc1b\uc558\ub2e4. \uc0bc\uada4\uad6c\uace0\ub450\ub294 \uc138 \ubc88 \uc808\ud558\uace0 \uc544\ud649 \ubc88 \uba38\ub9ac\ub97c \uc870\uc544\ub9ac\ub294 \uc5ec\uc9c4\uc871\uc758 \uc758\uc2dd\uc774\uc5c8\ub2e4.", "answer": "45\uc77c", "sentence": "\uc774\uc5d0 \uc870\uc120\uc740 \ub354 \uc774\uc0c1 \ubc84\ud2f8 \ud798\uc744 \uc0c1\uc2e4\ud558\uace0, 1\uc6d4 30\uc77c \uc778\uc870\uac00 45\uc77c \ub9cc\uc5d0 \uc2dd\ub7c9 \ubd80\uc871\uc73c\ub85c \uc131\ubb38\uc744 \uc5f4\uace0 \ub098\uac00 \uc0bc\uc804\ub3c4\uc5d0\uc11c \ud56d\ubcf5\uc758 \uc608\ub97c \uac16\uac8c \ub418\uc5c8\ub2e4.", "paragraph_sentence": "\uc774\uc5d0 \uc870\uc120\uc740 \uba3c\uc800 \ubd09\ub9bc\ub300\uad70, \uc778\ud3c9\ub300\uad70 \ub4f1\uc758 \uc655\uc790\ub97c \uba3c\uc800 \uac15\ud654\ub3c4\ub85c \ud53c\uc2e0\ud558\uac8c \ud558\uace0, \uc870\uc815 \ub610\ud55c \uac15\ud654\ub3c4\ub85c \ud53c\ub09c\ud558\ub824 \ud588\uc73c\ub098, \uccad\uad70\uc758 \uc120\ubc1c\ub300\uac00 \uc774\ubbf8 \uac15\ud654\ub3c4\ub85c \uac00\ub294 \uae38\ubaa9\uc744 \uac00\ub85c\ub9c9\uc73c\uba74\uc11c \uac15\ud654\ub3c4\ub97c \ud3ec\uae30\ud558\uace0 \ub0a8\ud55c\uc0b0\uc131\uc73c\ub85c \uac08 \uc218\ubc16\uc5d0 \uc5c6\uc5c8\ub2e4. \ub2f9\uc2dc \uc131 \uc548\uc5d0\ub294 1\ub9cc 3\ucc9c\uc5ec\uba85\uc758 \uad70\uc0ac\uac00 \ubc29\uc5b4\ub97c \ud558\uace0 \uc788\uc5c8\uace0, \uc591\uace1 1\ub9cc 4300\uc5ec \uc11d\uacfc \uc18c\uae08 90\uc5ec \uc11d\uc774 \uc788\uc5b4 \uaca8\uc6b0 50\uc77c \ubd84\uc758 \uc2dd\ub7c9\uc774 \ube44\ucd95\ub418\uc5b4 \uc788\uc5c8\ub2e4. \uccad\uad70\uc740 \ud070 \uc800\ud56d\uc744 \ubc1b\uc9c0 \uc54a\uc740 \ucc44 \ub0a8\ud55c\uc0b0\uc131\uc5d0 \ub2f9\ub3c4\uc5d0 \uc0b0\uc131 \ubc11\uc758 \ud0c4\ucc9c \ubd80\uadfc\uc5d0 \ud3ec\uc9c4\ud558\uc600\ub2e4. \uc774\uc5d0 \uc804\uad6d\uc758 \uad6c\uc6d0\ubcd1\ub4e4\uc774 \ucd9c\ubcd1\ud558\uc600\uc9c0\ub9cc, \ubaa8\ub450 \ub0a8\ud55c\uc0b0\uc131\uc5d0 \ub2f9\ub3c4\ud558\uae30 \uc804\uc5d0 \uada4\uba78\ub418\uc5c8\ub2e4. \uad6c\uc6d0\ubcd1\uc774 \ubcc4\ub85c \ub3c4\uc6c0\uc744 \uc8fc\uc9c0 \ubabb\ud558\uc790 \uc131\uc548\uc758 \uc870\uc815\uc5d0\uc11c\ub294 \uac15\ud654\ub860\uc774 \uc81c\uae30 \ub418\uc5c8\ub2e4. \uc8fc\ud654\ud30c\uc640 \uc8fc\uc804\ud30c \uc0ac\uc774\uc5d0 \uc5ec\ub7ec\ucc28\ub840 \uaca9\ub82c\ud55c \ub17c\uc7c1\uc774 \ubc8c\uc5b4\uc84c\uc9c0\ub9cc \uc774\ub4ec\ud574 1\uc6d4 22\uc77c \uac15\ud654\ub3c4\uac00 \uccad\uad70\uc5d0 \ud568\ub77d\ub2f9\ud558\uace0 \uac15\ud654\ub3c4\uc5d0 \ud53c\uc2e0\ud574 \uc788\ub358 \uc655\uc790\uc640 \uad70\uc2e0\ub4e4\uc758 \ucc98\uc790 200\uc5ec \uba85\uc774 \uccad\uad70\uc758 \ud3ec\ub85c\ub85c \uc7a1\ud788\uba74\uc11c \ub300\uc138\ub294 \uac15\ud654 \ucabd\uc73c\ub85c \uae30\uc6b8\uac8c \ub418\uc5c8\ub2e4. \uadf8 \uc774\ud6c4 \uccad\uad70\uc740 \ud654\ud3ec\ub97c \uc3d8\uc544\ub300\uc5b4 \uc131\ubcbd\uc758 \uc77c\ubd80\ub97c \ubb34\ub108\ub728\ub9ac\ub294 \ub4f1 \uc555\ubc15\uc744 \uac00\ud558\uc600\ub2e4. 1\uc6d4 26\uc77c \uc870\uc120\uc758 \uc0ac\uc2e0\uc774 \ud611\uc0c1\uc744 \uc704\ud574 \uccad \uc9c4\uc601\uc5d0 \ub3c4\ucc29\ud588\uc744\ub54c \uccad\uad70\uc740 \uac15\ud654\uc5d0\uc11c \ud3ec\ub85c\uac00 \ub41c \uc655\uc790\ub97c \ub300\uba74\uc2dc\ucf30\ub2e4. \uc774\uc5d0 \uc870\uc120\uc740 \ub354 \uc774\uc0c1 \ubc84\ud2f8 \ud798\uc744 \uc0c1\uc2e4\ud558\uace0, 1\uc6d4 30\uc77c \uc778\uc870\uac00 45\uc77c \ub9cc\uc5d0 \uc2dd\ub7c9 \ubd80\uc871\uc73c\ub85c \uc131\ubb38\uc744 \uc5f4\uace0 \ub098\uac00 \uc0bc\uc804\ub3c4\uc5d0\uc11c \ud56d\ubcf5\uc758 \uc608\ub97c \uac16\uac8c \ub418\uc5c8\ub2e4. \uc774 \uacf3\uc5d0\uc11c \uc778\uc870\ub294 \uc18c\ubcf5\uc744 \uc785\uace0 \uccad\ub098\ub77c \ud669\uc81c \ud669\ud0dc\uadf9\uc740 \ub192\uc740 \ub2e8\uc0c1\uc5d0 \uc549\uc544\uc11c \uc0bc\uada4\uad6c\uace0\ub450\uc758 \uc608\ub97c \ubc1b\uc558\ub2e4. \uc0bc\uada4\uad6c\uace0\ub450\ub294 \uc138 \ubc88 \uc808\ud558\uace0 \uc544\ud649 \ubc88 \uba38\ub9ac\ub97c \uc870\uc544\ub9ac\ub294 \uc5ec\uc9c4\uc871\uc758 \uc758\uc2dd\uc774\uc5c8\ub2e4.", "paragraph_answer": "\uc774\uc5d0 \uc870\uc120\uc740 \uba3c\uc800 \ubd09\ub9bc\ub300\uad70, \uc778\ud3c9\ub300\uad70 \ub4f1\uc758 \uc655\uc790\ub97c \uba3c\uc800 \uac15\ud654\ub3c4\ub85c \ud53c\uc2e0\ud558\uac8c \ud558\uace0, \uc870\uc815 \ub610\ud55c \uac15\ud654\ub3c4\ub85c \ud53c\ub09c\ud558\ub824 \ud588\uc73c\ub098, \uccad\uad70\uc758 \uc120\ubc1c\ub300\uac00 \uc774\ubbf8 \uac15\ud654\ub3c4\ub85c \uac00\ub294 \uae38\ubaa9\uc744 \uac00\ub85c\ub9c9\uc73c\uba74\uc11c \uac15\ud654\ub3c4\ub97c \ud3ec\uae30\ud558\uace0 \ub0a8\ud55c\uc0b0\uc131\uc73c\ub85c \uac08 \uc218\ubc16\uc5d0 \uc5c6\uc5c8\ub2e4. \ub2f9\uc2dc \uc131 \uc548\uc5d0\ub294 1\ub9cc 3\ucc9c\uc5ec\uba85\uc758 \uad70\uc0ac\uac00 \ubc29\uc5b4\ub97c \ud558\uace0 \uc788\uc5c8\uace0, \uc591\uace1 1\ub9cc 4300\uc5ec \uc11d\uacfc \uc18c\uae08 90\uc5ec \uc11d\uc774 \uc788\uc5b4 \uaca8\uc6b0 50\uc77c \ubd84\uc758 \uc2dd\ub7c9\uc774 \ube44\ucd95\ub418\uc5b4 \uc788\uc5c8\ub2e4. \uccad\uad70\uc740 \ud070 \uc800\ud56d\uc744 \ubc1b\uc9c0 \uc54a\uc740 \ucc44 \ub0a8\ud55c\uc0b0\uc131\uc5d0 \ub2f9\ub3c4\uc5d0 \uc0b0\uc131 \ubc11\uc758 \ud0c4\ucc9c \ubd80\uadfc\uc5d0 \ud3ec\uc9c4\ud558\uc600\ub2e4. \uc774\uc5d0 \uc804\uad6d\uc758 \uad6c\uc6d0\ubcd1\ub4e4\uc774 \ucd9c\ubcd1\ud558\uc600\uc9c0\ub9cc, \ubaa8\ub450 \ub0a8\ud55c\uc0b0\uc131\uc5d0 \ub2f9\ub3c4\ud558\uae30 \uc804\uc5d0 \uada4\uba78\ub418\uc5c8\ub2e4. \uad6c\uc6d0\ubcd1\uc774 \ubcc4\ub85c \ub3c4\uc6c0\uc744 \uc8fc\uc9c0 \ubabb\ud558\uc790 \uc131\uc548\uc758 \uc870\uc815\uc5d0\uc11c\ub294 \uac15\ud654\ub860\uc774 \uc81c\uae30 \ub418\uc5c8\ub2e4. \uc8fc\ud654\ud30c\uc640 \uc8fc\uc804\ud30c \uc0ac\uc774\uc5d0 \uc5ec\ub7ec\ucc28\ub840 \uaca9\ub82c\ud55c \ub17c\uc7c1\uc774 \ubc8c\uc5b4\uc84c\uc9c0\ub9cc \uc774\ub4ec\ud574 1\uc6d4 22\uc77c \uac15\ud654\ub3c4\uac00 \uccad\uad70\uc5d0 \ud568\ub77d\ub2f9\ud558\uace0 \uac15\ud654\ub3c4\uc5d0 \ud53c\uc2e0\ud574 \uc788\ub358 \uc655\uc790\uc640 \uad70\uc2e0\ub4e4\uc758 \ucc98\uc790 200\uc5ec \uba85\uc774 \uccad\uad70\uc758 \ud3ec\ub85c\ub85c \uc7a1\ud788\uba74\uc11c \ub300\uc138\ub294 \uac15\ud654 \ucabd\uc73c\ub85c \uae30\uc6b8\uac8c \ub418\uc5c8\ub2e4. \uadf8 \uc774\ud6c4 \uccad\uad70\uc740 \ud654\ud3ec\ub97c \uc3d8\uc544\ub300\uc5b4 \uc131\ubcbd\uc758 \uc77c\ubd80\ub97c \ubb34\ub108\ub728\ub9ac\ub294 \ub4f1 \uc555\ubc15\uc744 \uac00\ud558\uc600\ub2e4. 1\uc6d4 26\uc77c \uc870\uc120\uc758 \uc0ac\uc2e0\uc774 \ud611\uc0c1\uc744 \uc704\ud574 \uccad \uc9c4\uc601\uc5d0 \ub3c4\ucc29\ud588\uc744\ub54c \uccad\uad70\uc740 \uac15\ud654\uc5d0\uc11c \ud3ec\ub85c\uac00 \ub41c \uc655\uc790\ub97c \ub300\uba74\uc2dc\ucf30\ub2e4. \uc774\uc5d0 \uc870\uc120\uc740 \ub354 \uc774\uc0c1 \ubc84\ud2f8 \ud798\uc744 \uc0c1\uc2e4\ud558\uace0, 1\uc6d4 30\uc77c \uc778\uc870\uac00 45\uc77c \ub9cc\uc5d0 \uc2dd\ub7c9 \ubd80\uc871\uc73c\ub85c \uc131\ubb38\uc744 \uc5f4\uace0 \ub098\uac00 \uc0bc\uc804\ub3c4\uc5d0\uc11c \ud56d\ubcf5\uc758 \uc608\ub97c \uac16\uac8c \ub418\uc5c8\ub2e4. \uc774 \uacf3\uc5d0\uc11c \uc778\uc870\ub294 \uc18c\ubcf5\uc744 \uc785\uace0 \uccad\ub098\ub77c \ud669\uc81c \ud669\ud0dc\uadf9\uc740 \ub192\uc740 \ub2e8\uc0c1\uc5d0 \uc549\uc544\uc11c \uc0bc\uada4\uad6c\uace0\ub450\uc758 \uc608\ub97c \ubc1b\uc558\ub2e4. \uc0bc\uada4\uad6c\uace0\ub450\ub294 \uc138 \ubc88 \uc808\ud558\uace0 \uc544\ud649 \ubc88 \uba38\ub9ac\ub97c \uc870\uc544\ub9ac\ub294 \uc5ec\uc9c4\uc871\uc758 \uc758\uc2dd\uc774\uc5c8\ub2e4.", "sentence_answer": "\uc774\uc5d0 \uc870\uc120\uc740 \ub354 \uc774\uc0c1 \ubc84\ud2f8 \ud798\uc744 \uc0c1\uc2e4\ud558\uace0, 1\uc6d4 30\uc77c \uc778\uc870\uac00 45\uc77c \ub9cc\uc5d0 \uc2dd\ub7c9 \ubd80\uc871\uc73c\ub85c \uc131\ubb38\uc744 \uc5f4\uace0 \ub098\uac00 \uc0bc\uc804\ub3c4\uc5d0\uc11c \ud56d\ubcf5\uc758 \uc608\ub97c \uac16\uac8c \ub418\uc5c8\ub2e4.", "paragraph_id": "6483536-11-1", "paragraph_question": "question: \uc778\uc870\uac00 \uc0bc\uc804\ub3c4\uc5d0\uc11c \ud56d\ubcf5\uc758 \uc608\ub97c \uac00\uc9c0\uae30 \uc804 \ub0a8\ud55c\uc0b0\uc131\uc5d0\uc11c \uba87\uc77c\uc744 \ubc84\ud2f0\uc5c8\ub294\uac00?, context: \uc774\uc5d0 \uc870\uc120\uc740 \uba3c\uc800 \ubd09\ub9bc\ub300\uad70, \uc778\ud3c9\ub300\uad70 \ub4f1\uc758 \uc655\uc790\ub97c \uba3c\uc800 \uac15\ud654\ub3c4\ub85c \ud53c\uc2e0\ud558\uac8c \ud558\uace0, \uc870\uc815 \ub610\ud55c \uac15\ud654\ub3c4\ub85c \ud53c\ub09c\ud558\ub824 \ud588\uc73c\ub098, \uccad\uad70\uc758 \uc120\ubc1c\ub300\uac00 \uc774\ubbf8 \uac15\ud654\ub3c4\ub85c \uac00\ub294 \uae38\ubaa9\uc744 \uac00\ub85c\ub9c9\uc73c\uba74\uc11c \uac15\ud654\ub3c4\ub97c \ud3ec\uae30\ud558\uace0 \ub0a8\ud55c\uc0b0\uc131\uc73c\ub85c \uac08 \uc218\ubc16\uc5d0 \uc5c6\uc5c8\ub2e4. \ub2f9\uc2dc \uc131 \uc548\uc5d0\ub294 1\ub9cc 3\ucc9c\uc5ec\uba85\uc758 \uad70\uc0ac\uac00 \ubc29\uc5b4\ub97c \ud558\uace0 \uc788\uc5c8\uace0, \uc591\uace1 1\ub9cc 4300\uc5ec \uc11d\uacfc \uc18c\uae08 90\uc5ec \uc11d\uc774 \uc788\uc5b4 \uaca8\uc6b0 50\uc77c \ubd84\uc758 \uc2dd\ub7c9\uc774 \ube44\ucd95\ub418\uc5b4 \uc788\uc5c8\ub2e4. \uccad\uad70\uc740 \ud070 \uc800\ud56d\uc744 \ubc1b\uc9c0 \uc54a\uc740 \ucc44 \ub0a8\ud55c\uc0b0\uc131\uc5d0 \ub2f9\ub3c4\uc5d0 \uc0b0\uc131 \ubc11\uc758 \ud0c4\ucc9c \ubd80\uadfc\uc5d0 \ud3ec\uc9c4\ud558\uc600\ub2e4. \uc774\uc5d0 \uc804\uad6d\uc758 \uad6c\uc6d0\ubcd1\ub4e4\uc774 \ucd9c\ubcd1\ud558\uc600\uc9c0\ub9cc, \ubaa8\ub450 \ub0a8\ud55c\uc0b0\uc131\uc5d0 \ub2f9\ub3c4\ud558\uae30 \uc804\uc5d0 \uada4\uba78\ub418\uc5c8\ub2e4. \uad6c\uc6d0\ubcd1\uc774 \ubcc4\ub85c \ub3c4\uc6c0\uc744 \uc8fc\uc9c0 \ubabb\ud558\uc790 \uc131\uc548\uc758 \uc870\uc815\uc5d0\uc11c\ub294 \uac15\ud654\ub860\uc774 \uc81c\uae30 \ub418\uc5c8\ub2e4. \uc8fc\ud654\ud30c\uc640 \uc8fc\uc804\ud30c \uc0ac\uc774\uc5d0 \uc5ec\ub7ec\ucc28\ub840 \uaca9\ub82c\ud55c \ub17c\uc7c1\uc774 \ubc8c\uc5b4\uc84c\uc9c0\ub9cc \uc774\ub4ec\ud574 1\uc6d4 22\uc77c \uac15\ud654\ub3c4\uac00 \uccad\uad70\uc5d0 \ud568\ub77d\ub2f9\ud558\uace0 \uac15\ud654\ub3c4\uc5d0 \ud53c\uc2e0\ud574 \uc788\ub358 \uc655\uc790\uc640 \uad70\uc2e0\ub4e4\uc758 \ucc98\uc790 200\uc5ec \uba85\uc774 \uccad\uad70\uc758 \ud3ec\ub85c\ub85c \uc7a1\ud788\uba74\uc11c \ub300\uc138\ub294 \uac15\ud654 \ucabd\uc73c\ub85c \uae30\uc6b8\uac8c \ub418\uc5c8\ub2e4. \uadf8 \uc774\ud6c4 \uccad\uad70\uc740 \ud654\ud3ec\ub97c \uc3d8\uc544\ub300\uc5b4 \uc131\ubcbd\uc758 \uc77c\ubd80\ub97c \ubb34\ub108\ub728\ub9ac\ub294 \ub4f1 \uc555\ubc15\uc744 \uac00\ud558\uc600\ub2e4. 1\uc6d4 26\uc77c \uc870\uc120\uc758 \uc0ac\uc2e0\uc774 \ud611\uc0c1\uc744 \uc704\ud574 \uccad \uc9c4\uc601\uc5d0 \ub3c4\ucc29\ud588\uc744\ub54c \uccad\uad70\uc740 \uac15\ud654\uc5d0\uc11c \ud3ec\ub85c\uac00 \ub41c \uc655\uc790\ub97c \ub300\uba74\uc2dc\ucf30\ub2e4. \uc774\uc5d0 \uc870\uc120\uc740 \ub354 \uc774\uc0c1 \ubc84\ud2f8 \ud798\uc744 \uc0c1\uc2e4\ud558\uace0, 1\uc6d4 30\uc77c \uc778\uc870\uac00 45\uc77c \ub9cc\uc5d0 \uc2dd\ub7c9 \ubd80\uc871\uc73c\ub85c \uc131\ubb38\uc744 \uc5f4\uace0 \ub098\uac00 \uc0bc\uc804\ub3c4\uc5d0\uc11c \ud56d\ubcf5\uc758 \uc608\ub97c \uac16\uac8c \ub418\uc5c8\ub2e4. \uc774 \uacf3\uc5d0\uc11c \uc778\uc870\ub294 \uc18c\ubcf5\uc744 \uc785\uace0 \uccad\ub098\ub77c \ud669\uc81c \ud669\ud0dc\uadf9\uc740 \ub192\uc740 \ub2e8\uc0c1\uc5d0 \uc549\uc544\uc11c \uc0bc\uada4\uad6c\uace0\ub450\uc758 \uc608\ub97c \ubc1b\uc558\ub2e4. \uc0bc\uada4\uad6c\uace0\ub450\ub294 \uc138 \ubc88 \uc808\ud558\uace0 \uc544\ud649 \ubc88 \uba38\ub9ac\ub97c \uc870\uc544\ub9ac\ub294 \uc5ec\uc9c4\uc871\uc758 \uc758\uc2dd\uc774\uc5c8\ub2e4."} {"question": "\ucf00\uc774\ud2f0 \ud398\ub9ac\uac00 2011\ub144 2\uc6d4 20\uc77c \ubd80\ud130 2012\ub144 1\uc6d4 22\uc77c\uae4c\uc9c0 \ud3bc\uce5c \ucf58\uc11c\ud2b8 \ud22c\uc5b4\uc758 \uc774\ub984\uc740?", "paragraph": "2011\ub144 2\uc6d4 13\uc77c \ud398\ub9ac\ub294 \uc81c53\ud68c \uadf8\ub798\ubbf8 \uc0c1\uc5d0\uc11c \uc62c\ud574\uc758 \uc568\ubc94\uc744 \ud3ec\ud568\ud55c \ub124 \ubd80\ubb38\uc758 \ud6c4\ubcf4\ub85c \uc62c\ub790\uc9c0\ub9cc \uc218\uc0c1\uc740 \ubabb\ud588\ub2e4. \uac19\uc740 \ub2ec \uce74\ub2c8\uc608 \uc6e8\uc2a4\ud2b8\uac00 \ud53c\ucc98\ub9c1\uc73c\ub85c \ucc38\uc5ec\ud55c \ub124 \ubc88\uc9f8 \uc2f1\uae00 \"E.T.\"\ub97c \ubc1c\ub9e4\ud574 5\uc8fc \ub3d9\uc548 1\uc704\ub97c \ud588\ub2e4. \ub610\ud55c 2011\ub144 5\uc6d4 12\uc77c\uc790 \ube4c\ubcf4\ub4dc \ud56b 100 \ucc28\ud2b8\uc5d0\uc11c \ud398\ub9ac\ub294 \ud56b 100 \ucc28\ud2b8 \uc5ed\uc0ac\uc0c1 \ucc98\uc74c\uc73c\ub85c 52\uc8fc \uc5f0\uc18d 10\uc704\uad8c \uc548\uc5d0 \uba38\ubb34\ub294 \uae30\ub85d\uc744 \uc138\uc6e0\ub2e4. 2011\ub144 6\uc6d4\uc5d0\ub294 \ub2e4\uc12f \ubc88\uc9f8 \uc2f1\uae00 \"Last Friday Night (T.G.I.F.)\"\ub97c \ubc1c\ub9e4\ud588\uc73c\uba70, \ubbf8\uad6d\uc758 \ub798\ud37c \ubbf8\uc2dc \uc5d8\ub9ac\uc5c7\uc774 \ucc38\uc5ec\ud55c \ub9ac\ubbf9\uc2a4 \ubc84\uc804\uc740 8\uc6d4\uc5d0 \uacf5\uac1c\ud588\ub2e4. \uc774 \uc2f1\uae00\uc740 \ube4c\ubcf4\ub4dc \ud56b 100 2011\ub144 8\uc6d4 17\uc77c\uc790\uc5d0 1\uc704\uc5d0 \uc624\ub974\uba70 \uc5ec\uc790 \uac00\uc218\ub85c\ub294 \ucd5c\ucd08\uc774\uc790 \uc804\uccb4\uc5d0\uc11c\ub294 \ub450 \ubc88\uc9f8\ub85c, \ub9c8\uc774\ud074 \uc7ad\uc2a8\uc758 Bad \uc774\ud6c4 20\ub144 \ub9cc\uc5d0 \ud55c \uc568\ubc94\uc5d0\uc11c 5\uc7a5\uc758 \uc2f1\uae00\uc774 \uc5f0\uc18d\uc73c\ub85c 1\uc704\ub97c \ucc28\uc9c0\ud558\ub294 \uae30\ub85d\uc744 \uc138\uc6e0\ub2e4. \ub610\ud55c \uc55e\uc758 \uae30\ub85d\uacfc \ub9c8\ucc2c\uac00\uc9c0\ub85c \ube4c\ubcf4\ub4dc \ud31d \uc1a1 \uc5d0\uc5b4\ud50c\ub808\uc774 \ucc28\ud2b8\uc5d0\uc11c \ucd5c\ucd08\ub85c \ub2e4\uc12f \ubc88 \uc5f0\uc18d 1\uc704\uc5d0 \uc62c\ub790\ub2e4. 2011\ub144 10\uc6d4\uc5d0\ub294 \uc5ec\uc12f \ubc88\uc9f8 \uc2f1\uae00 \"The One That Got Away\"\ub97c \ubc1c\ub9e4\ud588\ub2e4. \uc774 \ub178\ub798\ub294 \ud56b 100 \ucc28\ud2b8 3\uc704\uae4c\uc9c0 \uc62c\ub790\uace0, Teenage Dream\uc740 \ud55c \uc568\ubc94\uc5d0\uc11c 6\uc7a5\uc758 \uc2f1\uae00\uc774 \ubaa8\ub450 5\uc704\uad8c\uc5d0 \uc9c4\uc785\ud55c \uc5ed\uc0ac\uc0c1 \uc138 \ubc88\uc9f8 \uc568\ubc94\uc774 \ub418\uc5c8\ub2e4. \uc5b4\ub35c\ud2b8 \ud31d \uc1a1 \ucc28\ud2b8\uc5d0\uc11c\ub3c4 1\uc704\ub97c \ud558\uba70 \uc5ec\uc12f \ubc88 \uc5f0\uc18d 1\uc704(\"Hot N Cold\", \"California Gurls\", \"Teenage Dream\", \"Firework\", \"Last Friday Night (T.G.I.F.), \"The One That Got Away\")\ub97c \ud558\uba70 \ub2e4\uc12f \ubc88 1\uc704\ub97c \ud588\ub358 \ud551\ud06c\uc640 \ub2c8\ucf08\ubc31\uc744 \uc81c\uce58\uace0 \uc2e0\uae30\ub85d\uc744 \uc138\uc6e0\ub2e4. \ud398\ub9ac\ub294 \uc774 \ub178\ub798\ub97c \ube4c\ubcf4\ub4dc \ud56b 100 1\uc704\ub97c \uc704\ud574 B.o.B\uac00 \ud53c\ucc98\ub9c1\uc73c\ub85c \ucc38\uc5ec\ud55c \ub9ac\ubbf9\uc2a4 \ubc84\uc804\uc744 \uacf5\uac1c\ud558\uace0, 2012\ub144 1\uc6d4\uc5d0\ub294 \uc5b4\ucfe0\uc2a4\ud2f1 \ubc84\uc804\uc744 \uacf5\uac1c\ud558\ub294 \ub4f1 \ub9ce\uc740 \ub178\ub825\uc744 \ud588\uc9c0\ub9cc \uc2e4\ud328\ud588\ub2e4. \ud398\ub9ac\ub294 2011\ub144 2\uc6d4 20\uc77c\ubd80\ud130 2012\ub144 1\uc6d4 22\uc77c\uae4c\uc9c0 \uc720\ub7fd, \uc624\uc2a4\ud2b8\ub808\uc77c\ub9ac\uc544, \uc544\uc2dc\uc544, \uc544\uba54\ub9ac\uce74 \ub300\ub959\uc5d0\uc11c \ucd1d 124\ud68c \uacf5\uc5f0\uc744 \ud3bc\uce58\ub294 \ucf58\uc11c\ud2b8 \ud22c\uc5b4 California Dreams Tour\ub97c \uc9c4\ud589\ud588\uace0, \uc138\uacc4\uc801\uc73c\ub85c \ucd1d 6,000\ub9cc \ub2ec\ub7ec\uc758 \uc218\uc775\uc744 \uac70\ub480\ub2e4. \ud398\ub9ac\ub294 2010\ub144 11\uc6d4 \"\ud37c\ub974\"\ub77c\ub294 \uc790\uc2e0\uc758 \uccab \ud5a5\uc218\ub97c \ub860\uce6d\ud588\ub2e4. \uc774 \ud5a5\uc218\ub294 \uace0\uc591\uc774 \ubaa8\uc591\uc758 \ubcd1\uc5d0 \ub2f4\uaca8 \uc788\uc73c\uba70, \ub178\ub4dc\uc2a4\ud2b8\ub86c\uc5d0 \ucd9c\uc2dc\ub418\uc5c8\ub2e4. 2011\ub144 11\uc6d4\uc5d0\ub294 \ud2b8\uc704\ud130\ub97c \ud1b5\ud574 \uc790\uc2e0\uc758 \ub450 \ubc88\uc9f8 \ud5a5\uc218 \"\ubbf8\uc544\uc6b0\"\ub97c \ucd9c\uc2dc\ud55c\ub2e4\uace0 \ubc1d\ud614\ub2e4.", "answer": "California Dreams Tour", "sentence": "\ud398\ub9ac\ub294 2011\ub144 2\uc6d4 20\uc77c\ubd80\ud130 2012\ub144 1\uc6d4 22\uc77c\uae4c\uc9c0 \uc720\ub7fd, \uc624\uc2a4\ud2b8\ub808\uc77c\ub9ac\uc544, \uc544\uc2dc\uc544, \uc544\uba54\ub9ac\uce74 \ub300\ub959\uc5d0\uc11c \ucd1d 124\ud68c \uacf5\uc5f0\uc744 \ud3bc\uce58\ub294 \ucf58\uc11c\ud2b8 \ud22c\uc5b4 California Dreams Tour \ub97c \uc9c4\ud589\ud588\uace0, \uc138\uacc4\uc801\uc73c\ub85c \ucd1d 6,000\ub9cc \ub2ec\ub7ec\uc758 \uc218\uc775\uc744 \uac70\ub480\ub2e4.", "paragraph_sentence": "2011\ub144 2\uc6d4 13\uc77c \ud398\ub9ac\ub294 \uc81c53\ud68c \uadf8\ub798\ubbf8 \uc0c1\uc5d0\uc11c \uc62c\ud574\uc758 \uc568\ubc94\uc744 \ud3ec\ud568\ud55c \ub124 \ubd80\ubb38\uc758 \ud6c4\ubcf4\ub85c \uc62c\ub790\uc9c0\ub9cc \uc218\uc0c1\uc740 \ubabb\ud588\ub2e4. \uac19\uc740 \ub2ec \uce74\ub2c8\uc608 \uc6e8\uc2a4\ud2b8\uac00 \ud53c\ucc98\ub9c1\uc73c\ub85c \ucc38\uc5ec\ud55c \ub124 \ubc88\uc9f8 \uc2f1\uae00 \"E.T.\"\ub97c \ubc1c\ub9e4\ud574 5\uc8fc \ub3d9\uc548 1\uc704\ub97c \ud588\ub2e4. \ub610\ud55c 2011\ub144 5\uc6d4 12\uc77c\uc790 \ube4c\ubcf4\ub4dc \ud56b 100 \ucc28\ud2b8\uc5d0\uc11c \ud398\ub9ac\ub294 \ud56b 100 \ucc28\ud2b8 \uc5ed\uc0ac\uc0c1 \ucc98\uc74c\uc73c\ub85c 52\uc8fc \uc5f0\uc18d 10\uc704\uad8c \uc548\uc5d0 \uba38\ubb34\ub294 \uae30\ub85d\uc744 \uc138\uc6e0\ub2e4. 2011\ub144 6\uc6d4\uc5d0\ub294 \ub2e4\uc12f \ubc88\uc9f8 \uc2f1\uae00 \"Last Friday Night (T.G.I.F.)\"\ub97c \ubc1c\ub9e4\ud588\uc73c\uba70, \ubbf8\uad6d\uc758 \ub798\ud37c \ubbf8\uc2dc \uc5d8\ub9ac\uc5c7\uc774 \ucc38\uc5ec\ud55c \ub9ac\ubbf9\uc2a4 \ubc84\uc804\uc740 8\uc6d4\uc5d0 \uacf5\uac1c\ud588\ub2e4. \uc774 \uc2f1\uae00\uc740 \ube4c\ubcf4\ub4dc \ud56b 100 2011\ub144 8\uc6d4 17\uc77c\uc790\uc5d0 1\uc704\uc5d0 \uc624\ub974\uba70 \uc5ec\uc790 \uac00\uc218\ub85c\ub294 \ucd5c\ucd08\uc774\uc790 \uc804\uccb4\uc5d0\uc11c\ub294 \ub450 \ubc88\uc9f8\ub85c, \ub9c8\uc774\ud074 \uc7ad\uc2a8\uc758 Bad \uc774\ud6c4 20\ub144 \ub9cc\uc5d0 \ud55c \uc568\ubc94\uc5d0\uc11c 5\uc7a5\uc758 \uc2f1\uae00\uc774 \uc5f0\uc18d\uc73c\ub85c 1\uc704\ub97c \ucc28\uc9c0\ud558\ub294 \uae30\ub85d\uc744 \uc138\uc6e0\ub2e4. \ub610\ud55c \uc55e\uc758 \uae30\ub85d\uacfc \ub9c8\ucc2c\uac00\uc9c0\ub85c \ube4c\ubcf4\ub4dc \ud31d \uc1a1 \uc5d0\uc5b4\ud50c\ub808\uc774 \ucc28\ud2b8\uc5d0\uc11c \ucd5c\ucd08\ub85c \ub2e4\uc12f \ubc88 \uc5f0\uc18d 1\uc704\uc5d0 \uc62c\ub790\ub2e4. 2011\ub144 10\uc6d4\uc5d0\ub294 \uc5ec\uc12f \ubc88\uc9f8 \uc2f1\uae00 \"The One That Got Away\"\ub97c \ubc1c\ub9e4\ud588\ub2e4. \uc774 \ub178\ub798\ub294 \ud56b 100 \ucc28\ud2b8 3\uc704\uae4c\uc9c0 \uc62c\ub790\uace0, Teenage Dream\uc740 \ud55c \uc568\ubc94\uc5d0\uc11c 6\uc7a5\uc758 \uc2f1\uae00\uc774 \ubaa8\ub450 5\uc704\uad8c\uc5d0 \uc9c4\uc785\ud55c \uc5ed\uc0ac\uc0c1 \uc138 \ubc88\uc9f8 \uc568\ubc94\uc774 \ub418\uc5c8\ub2e4. \uc5b4\ub35c\ud2b8 \ud31d \uc1a1 \ucc28\ud2b8\uc5d0\uc11c\ub3c4 1\uc704\ub97c \ud558\uba70 \uc5ec\uc12f \ubc88 \uc5f0\uc18d 1\uc704(\"Hot N Cold\", \"California Gurls\", \"Teenage Dream\", \"Firework\", \"Last Friday Night (T.G.I.F.), \"The One That Got Away\")\ub97c \ud558\uba70 \ub2e4\uc12f \ubc88 1\uc704\ub97c \ud588\ub358 \ud551\ud06c\uc640 \ub2c8\ucf08\ubc31\uc744 \uc81c\uce58\uace0 \uc2e0\uae30\ub85d\uc744 \uc138\uc6e0\ub2e4. \ud398\ub9ac\ub294 \uc774 \ub178\ub798\ub97c \ube4c\ubcf4\ub4dc \ud56b 100 1\uc704\ub97c \uc704\ud574 B.o. B\uac00 \ud53c\ucc98\ub9c1\uc73c\ub85c \ucc38\uc5ec\ud55c \ub9ac\ubbf9\uc2a4 \ubc84\uc804\uc744 \uacf5\uac1c\ud558\uace0, 2012\ub144 1\uc6d4\uc5d0\ub294 \uc5b4\ucfe0\uc2a4\ud2f1 \ubc84\uc804\uc744 \uacf5\uac1c\ud558\ub294 \ub4f1 \ub9ce\uc740 \ub178\ub825\uc744 \ud588\uc9c0\ub9cc \uc2e4\ud328\ud588\ub2e4. \ud398\ub9ac\ub294 2011\ub144 2\uc6d4 20\uc77c\ubd80\ud130 2012\ub144 1\uc6d4 22\uc77c\uae4c\uc9c0 \uc720\ub7fd, \uc624\uc2a4\ud2b8\ub808\uc77c\ub9ac\uc544, \uc544\uc2dc\uc544, \uc544\uba54\ub9ac\uce74 \ub300\ub959\uc5d0\uc11c \ucd1d 124\ud68c \uacf5\uc5f0\uc744 \ud3bc\uce58\ub294 \ucf58\uc11c\ud2b8 \ud22c\uc5b4 California Dreams Tour \ub97c \uc9c4\ud589\ud588\uace0, \uc138\uacc4\uc801\uc73c\ub85c \ucd1d 6,000\ub9cc \ub2ec\ub7ec\uc758 \uc218\uc775\uc744 \uac70\ub480\ub2e4. \ud398\ub9ac\ub294 2010\ub144 11\uc6d4 \"\ud37c\ub974\"\ub77c\ub294 \uc790\uc2e0\uc758 \uccab \ud5a5\uc218\ub97c \ub860\uce6d\ud588\ub2e4. \uc774 \ud5a5\uc218\ub294 \uace0\uc591\uc774 \ubaa8\uc591\uc758 \ubcd1\uc5d0 \ub2f4\uaca8 \uc788\uc73c\uba70, \ub178\ub4dc\uc2a4\ud2b8\ub86c\uc5d0 \ucd9c\uc2dc\ub418\uc5c8\ub2e4. 2011\ub144 11\uc6d4\uc5d0\ub294 \ud2b8\uc704\ud130\ub97c \ud1b5\ud574 \uc790\uc2e0\uc758 \ub450 \ubc88\uc9f8 \ud5a5\uc218 \"\ubbf8\uc544\uc6b0\"\ub97c \ucd9c\uc2dc\ud55c\ub2e4\uace0 \ubc1d\ud614\ub2e4.", "paragraph_answer": "2011\ub144 2\uc6d4 13\uc77c \ud398\ub9ac\ub294 \uc81c53\ud68c \uadf8\ub798\ubbf8 \uc0c1\uc5d0\uc11c \uc62c\ud574\uc758 \uc568\ubc94\uc744 \ud3ec\ud568\ud55c \ub124 \ubd80\ubb38\uc758 \ud6c4\ubcf4\ub85c \uc62c\ub790\uc9c0\ub9cc \uc218\uc0c1\uc740 \ubabb\ud588\ub2e4. \uac19\uc740 \ub2ec \uce74\ub2c8\uc608 \uc6e8\uc2a4\ud2b8\uac00 \ud53c\ucc98\ub9c1\uc73c\ub85c \ucc38\uc5ec\ud55c \ub124 \ubc88\uc9f8 \uc2f1\uae00 \"E.T.\"\ub97c \ubc1c\ub9e4\ud574 5\uc8fc \ub3d9\uc548 1\uc704\ub97c \ud588\ub2e4. \ub610\ud55c 2011\ub144 5\uc6d4 12\uc77c\uc790 \ube4c\ubcf4\ub4dc \ud56b 100 \ucc28\ud2b8\uc5d0\uc11c \ud398\ub9ac\ub294 \ud56b 100 \ucc28\ud2b8 \uc5ed\uc0ac\uc0c1 \ucc98\uc74c\uc73c\ub85c 52\uc8fc \uc5f0\uc18d 10\uc704\uad8c \uc548\uc5d0 \uba38\ubb34\ub294 \uae30\ub85d\uc744 \uc138\uc6e0\ub2e4. 2011\ub144 6\uc6d4\uc5d0\ub294 \ub2e4\uc12f \ubc88\uc9f8 \uc2f1\uae00 \"Last Friday Night (T.G.I.F.)\"\ub97c \ubc1c\ub9e4\ud588\uc73c\uba70, \ubbf8\uad6d\uc758 \ub798\ud37c \ubbf8\uc2dc \uc5d8\ub9ac\uc5c7\uc774 \ucc38\uc5ec\ud55c \ub9ac\ubbf9\uc2a4 \ubc84\uc804\uc740 8\uc6d4\uc5d0 \uacf5\uac1c\ud588\ub2e4. \uc774 \uc2f1\uae00\uc740 \ube4c\ubcf4\ub4dc \ud56b 100 2011\ub144 8\uc6d4 17\uc77c\uc790\uc5d0 1\uc704\uc5d0 \uc624\ub974\uba70 \uc5ec\uc790 \uac00\uc218\ub85c\ub294 \ucd5c\ucd08\uc774\uc790 \uc804\uccb4\uc5d0\uc11c\ub294 \ub450 \ubc88\uc9f8\ub85c, \ub9c8\uc774\ud074 \uc7ad\uc2a8\uc758 Bad \uc774\ud6c4 20\ub144 \ub9cc\uc5d0 \ud55c \uc568\ubc94\uc5d0\uc11c 5\uc7a5\uc758 \uc2f1\uae00\uc774 \uc5f0\uc18d\uc73c\ub85c 1\uc704\ub97c \ucc28\uc9c0\ud558\ub294 \uae30\ub85d\uc744 \uc138\uc6e0\ub2e4. \ub610\ud55c \uc55e\uc758 \uae30\ub85d\uacfc \ub9c8\ucc2c\uac00\uc9c0\ub85c \ube4c\ubcf4\ub4dc \ud31d \uc1a1 \uc5d0\uc5b4\ud50c\ub808\uc774 \ucc28\ud2b8\uc5d0\uc11c \ucd5c\ucd08\ub85c \ub2e4\uc12f \ubc88 \uc5f0\uc18d 1\uc704\uc5d0 \uc62c\ub790\ub2e4. 2011\ub144 10\uc6d4\uc5d0\ub294 \uc5ec\uc12f \ubc88\uc9f8 \uc2f1\uae00 \"The One That Got Away\"\ub97c \ubc1c\ub9e4\ud588\ub2e4. \uc774 \ub178\ub798\ub294 \ud56b 100 \ucc28\ud2b8 3\uc704\uae4c\uc9c0 \uc62c\ub790\uace0, Teenage Dream\uc740 \ud55c \uc568\ubc94\uc5d0\uc11c 6\uc7a5\uc758 \uc2f1\uae00\uc774 \ubaa8\ub450 5\uc704\uad8c\uc5d0 \uc9c4\uc785\ud55c \uc5ed\uc0ac\uc0c1 \uc138 \ubc88\uc9f8 \uc568\ubc94\uc774 \ub418\uc5c8\ub2e4. \uc5b4\ub35c\ud2b8 \ud31d \uc1a1 \ucc28\ud2b8\uc5d0\uc11c\ub3c4 1\uc704\ub97c \ud558\uba70 \uc5ec\uc12f \ubc88 \uc5f0\uc18d 1\uc704(\"Hot N Cold\", \"California Gurls\", \"Teenage Dream\", \"Firework\", \"Last Friday Night (T.G.I.F.), \"The One That Got Away\")\ub97c \ud558\uba70 \ub2e4\uc12f \ubc88 1\uc704\ub97c \ud588\ub358 \ud551\ud06c\uc640 \ub2c8\ucf08\ubc31\uc744 \uc81c\uce58\uace0 \uc2e0\uae30\ub85d\uc744 \uc138\uc6e0\ub2e4. \ud398\ub9ac\ub294 \uc774 \ub178\ub798\ub97c \ube4c\ubcf4\ub4dc \ud56b 100 1\uc704\ub97c \uc704\ud574 B.o.B\uac00 \ud53c\ucc98\ub9c1\uc73c\ub85c \ucc38\uc5ec\ud55c \ub9ac\ubbf9\uc2a4 \ubc84\uc804\uc744 \uacf5\uac1c\ud558\uace0, 2012\ub144 1\uc6d4\uc5d0\ub294 \uc5b4\ucfe0\uc2a4\ud2f1 \ubc84\uc804\uc744 \uacf5\uac1c\ud558\ub294 \ub4f1 \ub9ce\uc740 \ub178\ub825\uc744 \ud588\uc9c0\ub9cc \uc2e4\ud328\ud588\ub2e4. \ud398\ub9ac\ub294 2011\ub144 2\uc6d4 20\uc77c\ubd80\ud130 2012\ub144 1\uc6d4 22\uc77c\uae4c\uc9c0 \uc720\ub7fd, \uc624\uc2a4\ud2b8\ub808\uc77c\ub9ac\uc544, \uc544\uc2dc\uc544, \uc544\uba54\ub9ac\uce74 \ub300\ub959\uc5d0\uc11c \ucd1d 124\ud68c \uacf5\uc5f0\uc744 \ud3bc\uce58\ub294 \ucf58\uc11c\ud2b8 \ud22c\uc5b4 California Dreams Tour \ub97c \uc9c4\ud589\ud588\uace0, \uc138\uacc4\uc801\uc73c\ub85c \ucd1d 6,000\ub9cc \ub2ec\ub7ec\uc758 \uc218\uc775\uc744 \uac70\ub480\ub2e4. \ud398\ub9ac\ub294 2010\ub144 11\uc6d4 \"\ud37c\ub974\"\ub77c\ub294 \uc790\uc2e0\uc758 \uccab \ud5a5\uc218\ub97c \ub860\uce6d\ud588\ub2e4. \uc774 \ud5a5\uc218\ub294 \uace0\uc591\uc774 \ubaa8\uc591\uc758 \ubcd1\uc5d0 \ub2f4\uaca8 \uc788\uc73c\uba70, \ub178\ub4dc\uc2a4\ud2b8\ub86c\uc5d0 \ucd9c\uc2dc\ub418\uc5c8\ub2e4. 2011\ub144 11\uc6d4\uc5d0\ub294 \ud2b8\uc704\ud130\ub97c \ud1b5\ud574 \uc790\uc2e0\uc758 \ub450 \ubc88\uc9f8 \ud5a5\uc218 \"\ubbf8\uc544\uc6b0\"\ub97c \ucd9c\uc2dc\ud55c\ub2e4\uace0 \ubc1d\ud614\ub2e4.", "sentence_answer": "\ud398\ub9ac\ub294 2011\ub144 2\uc6d4 20\uc77c\ubd80\ud130 2012\ub144 1\uc6d4 22\uc77c\uae4c\uc9c0 \uc720\ub7fd, \uc624\uc2a4\ud2b8\ub808\uc77c\ub9ac\uc544, \uc544\uc2dc\uc544, \uc544\uba54\ub9ac\uce74 \ub300\ub959\uc5d0\uc11c \ucd1d 124\ud68c \uacf5\uc5f0\uc744 \ud3bc\uce58\ub294 \ucf58\uc11c\ud2b8 \ud22c\uc5b4 California Dreams Tour \ub97c \uc9c4\ud589\ud588\uace0, \uc138\uacc4\uc801\uc73c\ub85c \ucd1d 6,000\ub9cc \ub2ec\ub7ec\uc758 \uc218\uc775\uc744 \uac70\ub480\ub2e4.", "paragraph_id": "6209726-12-1", "paragraph_question": "question: \ucf00\uc774\ud2f0 \ud398\ub9ac\uac00 2011\ub144 2\uc6d4 20\uc77c \ubd80\ud130 2012\ub144 1\uc6d4 22\uc77c\uae4c\uc9c0 \ud3bc\uce5c \ucf58\uc11c\ud2b8 \ud22c\uc5b4\uc758 \uc774\ub984\uc740?, context: 2011\ub144 2\uc6d4 13\uc77c \ud398\ub9ac\ub294 \uc81c53\ud68c \uadf8\ub798\ubbf8 \uc0c1\uc5d0\uc11c \uc62c\ud574\uc758 \uc568\ubc94\uc744 \ud3ec\ud568\ud55c \ub124 \ubd80\ubb38\uc758 \ud6c4\ubcf4\ub85c \uc62c\ub790\uc9c0\ub9cc \uc218\uc0c1\uc740 \ubabb\ud588\ub2e4. \uac19\uc740 \ub2ec \uce74\ub2c8\uc608 \uc6e8\uc2a4\ud2b8\uac00 \ud53c\ucc98\ub9c1\uc73c\ub85c \ucc38\uc5ec\ud55c \ub124 \ubc88\uc9f8 \uc2f1\uae00 \"E.T.\"\ub97c \ubc1c\ub9e4\ud574 5\uc8fc \ub3d9\uc548 1\uc704\ub97c \ud588\ub2e4. \ub610\ud55c 2011\ub144 5\uc6d4 12\uc77c\uc790 \ube4c\ubcf4\ub4dc \ud56b 100 \ucc28\ud2b8\uc5d0\uc11c \ud398\ub9ac\ub294 \ud56b 100 \ucc28\ud2b8 \uc5ed\uc0ac\uc0c1 \ucc98\uc74c\uc73c\ub85c 52\uc8fc \uc5f0\uc18d 10\uc704\uad8c \uc548\uc5d0 \uba38\ubb34\ub294 \uae30\ub85d\uc744 \uc138\uc6e0\ub2e4. 2011\ub144 6\uc6d4\uc5d0\ub294 \ub2e4\uc12f \ubc88\uc9f8 \uc2f1\uae00 \"Last Friday Night (T.G.I.F.)\"\ub97c \ubc1c\ub9e4\ud588\uc73c\uba70, \ubbf8\uad6d\uc758 \ub798\ud37c \ubbf8\uc2dc \uc5d8\ub9ac\uc5c7\uc774 \ucc38\uc5ec\ud55c \ub9ac\ubbf9\uc2a4 \ubc84\uc804\uc740 8\uc6d4\uc5d0 \uacf5\uac1c\ud588\ub2e4. \uc774 \uc2f1\uae00\uc740 \ube4c\ubcf4\ub4dc \ud56b 100 2011\ub144 8\uc6d4 17\uc77c\uc790\uc5d0 1\uc704\uc5d0 \uc624\ub974\uba70 \uc5ec\uc790 \uac00\uc218\ub85c\ub294 \ucd5c\ucd08\uc774\uc790 \uc804\uccb4\uc5d0\uc11c\ub294 \ub450 \ubc88\uc9f8\ub85c, \ub9c8\uc774\ud074 \uc7ad\uc2a8\uc758 Bad \uc774\ud6c4 20\ub144 \ub9cc\uc5d0 \ud55c \uc568\ubc94\uc5d0\uc11c 5\uc7a5\uc758 \uc2f1\uae00\uc774 \uc5f0\uc18d\uc73c\ub85c 1\uc704\ub97c \ucc28\uc9c0\ud558\ub294 \uae30\ub85d\uc744 \uc138\uc6e0\ub2e4. \ub610\ud55c \uc55e\uc758 \uae30\ub85d\uacfc \ub9c8\ucc2c\uac00\uc9c0\ub85c \ube4c\ubcf4\ub4dc \ud31d \uc1a1 \uc5d0\uc5b4\ud50c\ub808\uc774 \ucc28\ud2b8\uc5d0\uc11c \ucd5c\ucd08\ub85c \ub2e4\uc12f \ubc88 \uc5f0\uc18d 1\uc704\uc5d0 \uc62c\ub790\ub2e4. 2011\ub144 10\uc6d4\uc5d0\ub294 \uc5ec\uc12f \ubc88\uc9f8 \uc2f1\uae00 \"The One That Got Away\"\ub97c \ubc1c\ub9e4\ud588\ub2e4. \uc774 \ub178\ub798\ub294 \ud56b 100 \ucc28\ud2b8 3\uc704\uae4c\uc9c0 \uc62c\ub790\uace0, Teenage Dream\uc740 \ud55c \uc568\ubc94\uc5d0\uc11c 6\uc7a5\uc758 \uc2f1\uae00\uc774 \ubaa8\ub450 5\uc704\uad8c\uc5d0 \uc9c4\uc785\ud55c \uc5ed\uc0ac\uc0c1 \uc138 \ubc88\uc9f8 \uc568\ubc94\uc774 \ub418\uc5c8\ub2e4. \uc5b4\ub35c\ud2b8 \ud31d \uc1a1 \ucc28\ud2b8\uc5d0\uc11c\ub3c4 1\uc704\ub97c \ud558\uba70 \uc5ec\uc12f \ubc88 \uc5f0\uc18d 1\uc704(\"Hot N Cold\", \"California Gurls\", \"Teenage Dream\", \"Firework\", \"Last Friday Night (T.G.I.F.), \"The One That Got Away\")\ub97c \ud558\uba70 \ub2e4\uc12f \ubc88 1\uc704\ub97c \ud588\ub358 \ud551\ud06c\uc640 \ub2c8\ucf08\ubc31\uc744 \uc81c\uce58\uace0 \uc2e0\uae30\ub85d\uc744 \uc138\uc6e0\ub2e4. \ud398\ub9ac\ub294 \uc774 \ub178\ub798\ub97c \ube4c\ubcf4\ub4dc \ud56b 100 1\uc704\ub97c \uc704\ud574 B.o.B\uac00 \ud53c\ucc98\ub9c1\uc73c\ub85c \ucc38\uc5ec\ud55c \ub9ac\ubbf9\uc2a4 \ubc84\uc804\uc744 \uacf5\uac1c\ud558\uace0, 2012\ub144 1\uc6d4\uc5d0\ub294 \uc5b4\ucfe0\uc2a4\ud2f1 \ubc84\uc804\uc744 \uacf5\uac1c\ud558\ub294 \ub4f1 \ub9ce\uc740 \ub178\ub825\uc744 \ud588\uc9c0\ub9cc \uc2e4\ud328\ud588\ub2e4. \ud398\ub9ac\ub294 2011\ub144 2\uc6d4 20\uc77c\ubd80\ud130 2012\ub144 1\uc6d4 22\uc77c\uae4c\uc9c0 \uc720\ub7fd, \uc624\uc2a4\ud2b8\ub808\uc77c\ub9ac\uc544, \uc544\uc2dc\uc544, \uc544\uba54\ub9ac\uce74 \ub300\ub959\uc5d0\uc11c \ucd1d 124\ud68c \uacf5\uc5f0\uc744 \ud3bc\uce58\ub294 \ucf58\uc11c\ud2b8 \ud22c\uc5b4 California Dreams Tour\ub97c \uc9c4\ud589\ud588\uace0, \uc138\uacc4\uc801\uc73c\ub85c \ucd1d 6,000\ub9cc \ub2ec\ub7ec\uc758 \uc218\uc775\uc744 \uac70\ub480\ub2e4. \ud398\ub9ac\ub294 2010\ub144 11\uc6d4 \"\ud37c\ub974\"\ub77c\ub294 \uc790\uc2e0\uc758 \uccab \ud5a5\uc218\ub97c \ub860\uce6d\ud588\ub2e4. \uc774 \ud5a5\uc218\ub294 \uace0\uc591\uc774 \ubaa8\uc591\uc758 \ubcd1\uc5d0 \ub2f4\uaca8 \uc788\uc73c\uba70, \ub178\ub4dc\uc2a4\ud2b8\ub86c\uc5d0 \ucd9c\uc2dc\ub418\uc5c8\ub2e4. 2011\ub144 11\uc6d4\uc5d0\ub294 \ud2b8\uc704\ud130\ub97c \ud1b5\ud574 \uc790\uc2e0\uc758 \ub450 \ubc88\uc9f8 \ud5a5\uc218 \"\ubbf8\uc544\uc6b0\"\ub97c \ucd9c\uc2dc\ud55c\ub2e4\uace0 \ubc1d\ud614\ub2e4."} {"question": "\ud3ec\ub85c \ud559\uc0b4\uc774 \uc774\ub8e8\uc5b4\uc9c4 \uc815\ud655\ud55c \uc704\uce58\ub97c \ucc3e\uae30 \uc704\ud574 1985\ub144 \uc774\ud6c4 \uc5ec\ub7ec \ubc29\uba74\uc73c\ub85c \uc218\uace0\ud55c \uc0ac\ub78c\uc740?", "paragraph": "303\uace0\uc9c0 \uc774\uc57c\uae30\ub294 \ubbf8\uad6d \uc5b8\ub860\uc744 \ud1b5\ud574 \ub110\ub9ac \uc54c\ub824\uc84c\uace0, \uc0dd\uc874\uc790\ub4e4\uc758 \uc99d\uc5b8\uc740 \u300a\ud0c0\uc784\u300b, \u300a\ub77c\uc774\ud504\u300b\ub97c \ube44\ub86f\ud55c \uc8fc\uc694 \uc5b8\ub860\uc758 \ubcf4\ub3c4\ub97c \ud0d4\ub2e4. \ud55c\uad6d\uc804\uc7c1 \uc774\ud6c4 \uc8fc\ud55c\ubbf8\uad70\uc740 \uc65c\uad00\uc5d0 \uc601\uad6c\uc801 \uc8fc\ub454\uc9c0\uc778 \ucea0\ud504 \uce90\ub7f4\uc744 \uc124\uce58\ud588\ub294\ub370, \uc774 \ubd80\ub300\ub294 \uc790\uace0\uc0b0(= 303\uace0\uc9c0)\uc5d0 \ub9e4\uc6b0 \uac00\uae4c\uc6b4 \uc704\uce58\uc5d0 \uc788\ub2e4. \uadf8\ub9ac\uace0 303\uace0\uc9c0 \uc0ac\uac74\uc740 \ud55c\ub3d9\uc548 \uc78a\ud600\uc838 \uc788\uc5c8\ub294\ub370, 1985\ub144 \ub370\uc774\ube44\ub4dc \ucea5\uac70\uc2a4(David Kangas) \uc911\uc704\uac00 \u300a\ub0a8\uc73c\ub85c\ub294 \ub099\ub3d9\uae4c\uc9c0, \ubd81\uc73c\ub85c\ub294 \uc555\ub85d\uae4c\uc9c0\u300b\ub97c \uc77d\uace0 \ub610 \ubbf8\uc721\uad70\uacfc \ub300\ud55c\ubbfc\uad6d \uce21\uc758 \uc5ec\ub7ec \uc790\ub8cc\ub4e4\uc744 \uc0b4\ud3b4\ubd24\uc73c\ub098 \ud559\uc0b4\uc774 \uc774\ub8e8\uc5b4\uc9c4 \uc7a5\uc18c\uac00 \uc5b4\ub514\uc778\uc9c0 \ubaa8\ub978\ub2e4\ub294 \uac83\uc744 \uc54c\uc544\ub0c8\ub2e4. \uadf8\ub798\uc11c \ucea5\uac70\uc2a4\ub294 \ud3ec\ub85c \ud559\uc0b4\uc774 \uc774\ub8e8\uc5b4\uc9c4 \uc815\ud655\ud55c \uc704\uce58\ub97c \ucc3e\uc544\ub0b4\uae30 \uc704\ud574 \uad6d\uac00\uae30\ub85d\uc6d0\uc758 \uc804\ud22c \uae30\ub85d\uc744 \ucc3e\uc544\ubcf4\uc558\uace0, \uc0dd\uc874\uc790\ub4e4\uc758 \uac70\ucde8\ub97c \ucd94\uc801\ud588\ub2e4.", "answer": "\ub370\uc774\ube44\ub4dc \ucea5\uac70\uc2a4", "sentence": "1985\ub144 \ub370\uc774\ube44\ub4dc \ucea5\uac70\uc2a4 (David Kangas) \uc911\uc704\uac00 \u300a\ub0a8\uc73c\ub85c\ub294 \ub099\ub3d9\uae4c\uc9c0, \ubd81\uc73c\ub85c\ub294 \uc555\ub85d\uae4c\uc9c0\u300b\ub97c \uc77d\uace0 \ub610 \ubbf8\uc721\uad70\uacfc \ub300\ud55c\ubbfc\uad6d \uce21\uc758 \uc5ec\ub7ec \uc790\ub8cc\ub4e4\uc744 \uc0b4\ud3b4\ubd24\uc73c\ub098 \ud559\uc0b4\uc774 \uc774\ub8e8\uc5b4\uc9c4 \uc7a5\uc18c\uac00 \uc5b4\ub514\uc778\uc9c0 \ubaa8\ub978\ub2e4\ub294 \uac83\uc744 \uc54c\uc544\ub0c8\ub2e4.", "paragraph_sentence": "303\uace0\uc9c0 \uc774\uc57c\uae30\ub294 \ubbf8\uad6d \uc5b8\ub860\uc744 \ud1b5\ud574 \ub110\ub9ac \uc54c\ub824\uc84c\uace0, \uc0dd\uc874\uc790\ub4e4\uc758 \uc99d\uc5b8\uc740 \u300a\ud0c0\uc784\u300b, \u300a\ub77c\uc774\ud504\u300b\ub97c \ube44\ub86f\ud55c \uc8fc\uc694 \uc5b8\ub860\uc758 \ubcf4\ub3c4\ub97c \ud0d4\ub2e4. \ud55c\uad6d\uc804\uc7c1 \uc774\ud6c4 \uc8fc\ud55c\ubbf8\uad70\uc740 \uc65c\uad00\uc5d0 \uc601\uad6c\uc801 \uc8fc\ub454\uc9c0\uc778 \ucea0\ud504 \uce90\ub7f4\uc744 \uc124\uce58\ud588\ub294\ub370, \uc774 \ubd80\ub300\ub294 \uc790\uace0\uc0b0(= 303\uace0\uc9c0)\uc5d0 \ub9e4\uc6b0 \uac00\uae4c\uc6b4 \uc704\uce58\uc5d0 \uc788\ub2e4. \uadf8\ub9ac\uace0 303\uace0\uc9c0 \uc0ac\uac74\uc740 \ud55c\ub3d9\uc548 \uc78a\ud600\uc838 \uc788\uc5c8\ub294\ub370, 1985\ub144 \ub370\uc774\ube44\ub4dc \ucea5\uac70\uc2a4 (David Kangas) \uc911\uc704\uac00 \u300a\ub0a8\uc73c\ub85c\ub294 \ub099\ub3d9\uae4c\uc9c0, \ubd81\uc73c\ub85c\ub294 \uc555\ub85d\uae4c\uc9c0\u300b\ub97c \uc77d\uace0 \ub610 \ubbf8\uc721\uad70\uacfc \ub300\ud55c\ubbfc\uad6d \uce21\uc758 \uc5ec\ub7ec \uc790\ub8cc\ub4e4\uc744 \uc0b4\ud3b4\ubd24\uc73c\ub098 \ud559\uc0b4\uc774 \uc774\ub8e8\uc5b4\uc9c4 \uc7a5\uc18c\uac00 \uc5b4\ub514\uc778\uc9c0 \ubaa8\ub978\ub2e4\ub294 \uac83\uc744 \uc54c\uc544\ub0c8\ub2e4. \uadf8\ub798\uc11c \ucea5\uac70\uc2a4\ub294 \ud3ec\ub85c \ud559\uc0b4\uc774 \uc774\ub8e8\uc5b4\uc9c4 \uc815\ud655\ud55c \uc704\uce58\ub97c \ucc3e\uc544\ub0b4\uae30 \uc704\ud574 \uad6d\uac00\uae30\ub85d\uc6d0\uc758 \uc804\ud22c \uae30\ub85d\uc744 \ucc3e\uc544\ubcf4\uc558\uace0, \uc0dd\uc874\uc790\ub4e4\uc758 \uac70\ucde8\ub97c \ucd94\uc801\ud588\ub2e4.", "paragraph_answer": "303\uace0\uc9c0 \uc774\uc57c\uae30\ub294 \ubbf8\uad6d \uc5b8\ub860\uc744 \ud1b5\ud574 \ub110\ub9ac \uc54c\ub824\uc84c\uace0, \uc0dd\uc874\uc790\ub4e4\uc758 \uc99d\uc5b8\uc740 \u300a\ud0c0\uc784\u300b, \u300a\ub77c\uc774\ud504\u300b\ub97c \ube44\ub86f\ud55c \uc8fc\uc694 \uc5b8\ub860\uc758 \ubcf4\ub3c4\ub97c \ud0d4\ub2e4. \ud55c\uad6d\uc804\uc7c1 \uc774\ud6c4 \uc8fc\ud55c\ubbf8\uad70\uc740 \uc65c\uad00\uc5d0 \uc601\uad6c\uc801 \uc8fc\ub454\uc9c0\uc778 \ucea0\ud504 \uce90\ub7f4\uc744 \uc124\uce58\ud588\ub294\ub370, \uc774 \ubd80\ub300\ub294 \uc790\uace0\uc0b0(= 303\uace0\uc9c0)\uc5d0 \ub9e4\uc6b0 \uac00\uae4c\uc6b4 \uc704\uce58\uc5d0 \uc788\ub2e4. \uadf8\ub9ac\uace0 303\uace0\uc9c0 \uc0ac\uac74\uc740 \ud55c\ub3d9\uc548 \uc78a\ud600\uc838 \uc788\uc5c8\ub294\ub370, 1985\ub144 \ub370\uc774\ube44\ub4dc \ucea5\uac70\uc2a4 (David Kangas) \uc911\uc704\uac00 \u300a\ub0a8\uc73c\ub85c\ub294 \ub099\ub3d9\uae4c\uc9c0, \ubd81\uc73c\ub85c\ub294 \uc555\ub85d\uae4c\uc9c0\u300b\ub97c \uc77d\uace0 \ub610 \ubbf8\uc721\uad70\uacfc \ub300\ud55c\ubbfc\uad6d \uce21\uc758 \uc5ec\ub7ec \uc790\ub8cc\ub4e4\uc744 \uc0b4\ud3b4\ubd24\uc73c\ub098 \ud559\uc0b4\uc774 \uc774\ub8e8\uc5b4\uc9c4 \uc7a5\uc18c\uac00 \uc5b4\ub514\uc778\uc9c0 \ubaa8\ub978\ub2e4\ub294 \uac83\uc744 \uc54c\uc544\ub0c8\ub2e4. \uadf8\ub798\uc11c \ucea5\uac70\uc2a4\ub294 \ud3ec\ub85c \ud559\uc0b4\uc774 \uc774\ub8e8\uc5b4\uc9c4 \uc815\ud655\ud55c \uc704\uce58\ub97c \ucc3e\uc544\ub0b4\uae30 \uc704\ud574 \uad6d\uac00\uae30\ub85d\uc6d0\uc758 \uc804\ud22c \uae30\ub85d\uc744 \ucc3e\uc544\ubcf4\uc558\uace0, \uc0dd\uc874\uc790\ub4e4\uc758 \uac70\ucde8\ub97c \ucd94\uc801\ud588\ub2e4.", "sentence_answer": "1985\ub144 \ub370\uc774\ube44\ub4dc \ucea5\uac70\uc2a4 (David Kangas) \uc911\uc704\uac00 \u300a\ub0a8\uc73c\ub85c\ub294 \ub099\ub3d9\uae4c\uc9c0, \ubd81\uc73c\ub85c\ub294 \uc555\ub85d\uae4c\uc9c0\u300b\ub97c \uc77d\uace0 \ub610 \ubbf8\uc721\uad70\uacfc \ub300\ud55c\ubbfc\uad6d \uce21\uc758 \uc5ec\ub7ec \uc790\ub8cc\ub4e4\uc744 \uc0b4\ud3b4\ubd24\uc73c\ub098 \ud559\uc0b4\uc774 \uc774\ub8e8\uc5b4\uc9c4 \uc7a5\uc18c\uac00 \uc5b4\ub514\uc778\uc9c0 \ubaa8\ub978\ub2e4\ub294 \uac83\uc744 \uc54c\uc544\ub0c8\ub2e4.", "paragraph_id": "6470861-10-2", "paragraph_question": "question: \ud3ec\ub85c \ud559\uc0b4\uc774 \uc774\ub8e8\uc5b4\uc9c4 \uc815\ud655\ud55c \uc704\uce58\ub97c \ucc3e\uae30 \uc704\ud574 1985\ub144 \uc774\ud6c4 \uc5ec\ub7ec \ubc29\uba74\uc73c\ub85c \uc218\uace0\ud55c \uc0ac\ub78c\uc740?, context: 303\uace0\uc9c0 \uc774\uc57c\uae30\ub294 \ubbf8\uad6d \uc5b8\ub860\uc744 \ud1b5\ud574 \ub110\ub9ac \uc54c\ub824\uc84c\uace0, \uc0dd\uc874\uc790\ub4e4\uc758 \uc99d\uc5b8\uc740 \u300a\ud0c0\uc784\u300b, \u300a\ub77c\uc774\ud504\u300b\ub97c \ube44\ub86f\ud55c \uc8fc\uc694 \uc5b8\ub860\uc758 \ubcf4\ub3c4\ub97c \ud0d4\ub2e4. \ud55c\uad6d\uc804\uc7c1 \uc774\ud6c4 \uc8fc\ud55c\ubbf8\uad70\uc740 \uc65c\uad00\uc5d0 \uc601\uad6c\uc801 \uc8fc\ub454\uc9c0\uc778 \ucea0\ud504 \uce90\ub7f4\uc744 \uc124\uce58\ud588\ub294\ub370, \uc774 \ubd80\ub300\ub294 \uc790\uace0\uc0b0(= 303\uace0\uc9c0)\uc5d0 \ub9e4\uc6b0 \uac00\uae4c\uc6b4 \uc704\uce58\uc5d0 \uc788\ub2e4. \uadf8\ub9ac\uace0 303\uace0\uc9c0 \uc0ac\uac74\uc740 \ud55c\ub3d9\uc548 \uc78a\ud600\uc838 \uc788\uc5c8\ub294\ub370, 1985\ub144 \ub370\uc774\ube44\ub4dc \ucea5\uac70\uc2a4(David Kangas) \uc911\uc704\uac00 \u300a\ub0a8\uc73c\ub85c\ub294 \ub099\ub3d9\uae4c\uc9c0, \ubd81\uc73c\ub85c\ub294 \uc555\ub85d\uae4c\uc9c0\u300b\ub97c \uc77d\uace0 \ub610 \ubbf8\uc721\uad70\uacfc \ub300\ud55c\ubbfc\uad6d \uce21\uc758 \uc5ec\ub7ec \uc790\ub8cc\ub4e4\uc744 \uc0b4\ud3b4\ubd24\uc73c\ub098 \ud559\uc0b4\uc774 \uc774\ub8e8\uc5b4\uc9c4 \uc7a5\uc18c\uac00 \uc5b4\ub514\uc778\uc9c0 \ubaa8\ub978\ub2e4\ub294 \uac83\uc744 \uc54c\uc544\ub0c8\ub2e4. \uadf8\ub798\uc11c \ucea5\uac70\uc2a4\ub294 \ud3ec\ub85c \ud559\uc0b4\uc774 \uc774\ub8e8\uc5b4\uc9c4 \uc815\ud655\ud55c \uc704\uce58\ub97c \ucc3e\uc544\ub0b4\uae30 \uc704\ud574 \uad6d\uac00\uae30\ub85d\uc6d0\uc758 \uc804\ud22c \uae30\ub85d\uc744 \ucc3e\uc544\ubcf4\uc558\uace0, \uc0dd\uc874\uc790\ub4e4\uc758 \uac70\ucde8\ub97c \ucd94\uc801\ud588\ub2e4."} {"question": "\uc624\uc2a4\ud2b8\ub9ac\uc544\uccad\uac1c\uad6c\ub9ac\uc18d\uc740 \ub2e4\ub978 \uccad\uac1c\uad6c\ub9ac\ub4e4\uacfc\ub294 \ub2ec\ub9ac \ub208\uaebc\ud480\uc5d0 \ubb34\uc5c7\uc774 \uc5c6\ub2e4 \ud558\uc600\ub294\uac00?", "paragraph": "\uc624\uc2a4\ud2b8\ub808\uc77c\ub9ac\uc544\uccad\uac1c\uad6c\ub9ac\ub294 \uc2e0\ub300\ub959 \uccad\uac1c\uad6c\ub9ac\ub4e4\uc778 \uccad\uac1c\uad6c\ub9ac\uacfc(Hylidae)\uc758 \uc77c\uc6d0\uc73c\ub85c\uc11c \uc624\uc2a4\ud2b8\ub808\uc77c\ub9ac\uc544\uc640 \ub274\uae30\ub2c8 \uace0\uc720\uc885\uc778 \uc624\uc2a4\ud2b8\ub808\uc77c\ub9ac\uc544\uccad\uac1c\uad6c\ub9ac\uc544\uacfc(Pelodryadinae)\uc5d0 \uc18d\ud55c\ub2e4. \uc774 \uc544\uacfc\uc5d0\ub294 \ubb3c\uc9c0\ub2d8\uac1c\uad6c\ub9ac\uc18d(Cyclorana), \uc624\uc2a4\ud2b8\ub808\uc77c\ub9ac\uc544\uccad\uac1c\uad6c\ub9ac\uc18d(Litoria), \uc655\ub208\uc774\uccad\uac1c\uad6c\ub9ac\uc18d(Nyctimystes) 3\uc18d\uc5d0 160\uc5ec \uc885\uc774 \ud3ec\ud568\ub418\uc5b4 \uc788\ub2e4. \uc624\uc2a4\ud2b8\ub808\uc77c\ub9ac\uc544\uccad\uac1c\uad6c\ub9ac\uc18d\uc758 \uac1c\uad6c\ub9ac\ub4e4\uc740 \ud64d\ucc44\uac00 \uac00\ub85c\ub85c \ucc22\uc5b4\uc838 \uc788\uace0 \ub208\uaebc\ud480\uc5d0 \uc0c9\uc18c\uac00 \uc5c6\ub294 \uc810\uc73c\ub85c \ub2e4\ub978 \uccad\uac1c\uad6c\ub9ac\ub4e4\uacfc \uad6c\ubd84\ub41c\ub2e4. \uc624\uc2a4\ud2b8\ub808\uc77c\ub9ac\uc544\uccad\uac1c\uad6c\ub9ac\uc758 \ub2e4\ub978 \uc774\ub984\uc778 \"\ud654\uc774\ud2b8\uccad\uac1c\uad6c\ub9ac\"\ub294 1790\ub144 \ub188\uc758 \ud559\uba85\uc744 \ucc98\uc74c \uba85\uba85\ud55c \uc874 \ud654\uc774\ud2b8\uc758 \uc774\ub984\uc774 \ubd99\uc740 \uac83\uc774\ub2e4. \uc624\uc2a4\ud2b8\ub808\uc77c\ub9ac\uc544\uccad\uac1c\uad6c\ub9ac\ub294 \uc624\uc2a4\ud2b8\ub808\uc77c\ub9ac\uc544\uc5d0 \uc11c\uc2dd\ud558\ub294 \uac1c\uad6c\ub9ac\ub4e4 \uc911 \ucc98\uc74c\uc73c\ub85c \uae30\uc7ac\ub41c \uc885\uc73c\ub85c\uc11c, \uadf8 \ucd5c\ucd08 \ud45c\ubcf8\uc740 \uc81c1\ub300 \ubc45\ud06c\uc2a4 \uc900\ub0a8\uc791 \uc870\uc9c0\ud504 \ubc45\ud06c\uc2a4\uc758 \uc218\uc9d1\ud488 \uceec\ub809\uc158\uc5d0 \ud3ec\ud568\ub418\uc5b4 \uc788\uc5c8\ub2e4. \uadf8\ub7ec\ub098 \ubd88\ud589\ud788\ub3c4 \uc774 \ud45c\ubcf8\uc740 \uc81c2\ucc28 \uc138\uacc4 \ub300\uc804 \ub54c \ud45c\ubcf8\uc744 \uc18c\uc7a5\ud558\ub358 \ubc15\ubb3c\uad00\uc774 \ud3ed\uaca9\ub2f9\ud568\uc73c\ub85c\uc368 \uc18c\uc2e4\ub418\uc5c8\ub2e4.", "answer": "\uc0c9\uc18c", "sentence": "\uc624\uc2a4\ud2b8\ub808\uc77c\ub9ac\uc544\uccad\uac1c\uad6c\ub9ac\uc18d\uc758 \uac1c\uad6c\ub9ac\ub4e4\uc740 \ud64d\ucc44\uac00 \uac00\ub85c\ub85c \ucc22\uc5b4\uc838 \uc788\uace0 \ub208\uaebc\ud480\uc5d0 \uc0c9\uc18c \uac00 \uc5c6\ub294 \uc810\uc73c\ub85c \ub2e4\ub978 \uccad\uac1c\uad6c\ub9ac\ub4e4\uacfc \uad6c\ubd84\ub41c\ub2e4.", "paragraph_sentence": "\uc624\uc2a4\ud2b8\ub808\uc77c\ub9ac\uc544\uccad\uac1c\uad6c\ub9ac\ub294 \uc2e0\ub300\ub959 \uccad\uac1c\uad6c\ub9ac\ub4e4\uc778 \uccad\uac1c\uad6c\ub9ac\uacfc(Hylidae)\uc758 \uc77c\uc6d0\uc73c\ub85c\uc11c \uc624\uc2a4\ud2b8\ub808\uc77c\ub9ac\uc544\uc640 \ub274\uae30\ub2c8 \uace0\uc720\uc885\uc778 \uc624\uc2a4\ud2b8\ub808\uc77c\ub9ac\uc544\uccad\uac1c\uad6c\ub9ac\uc544\uacfc(Pelodryadinae)\uc5d0 \uc18d\ud55c\ub2e4. \uc774 \uc544\uacfc\uc5d0\ub294 \ubb3c\uc9c0\ub2d8\uac1c\uad6c\ub9ac\uc18d(Cyclorana), \uc624\uc2a4\ud2b8\ub808\uc77c\ub9ac\uc544\uccad\uac1c\uad6c\ub9ac\uc18d(Litoria), \uc655\ub208\uc774\uccad\uac1c\uad6c\ub9ac\uc18d(Nyctimystes) 3\uc18d\uc5d0 160\uc5ec \uc885\uc774 \ud3ec\ud568\ub418\uc5b4 \uc788\ub2e4. \uc624\uc2a4\ud2b8\ub808\uc77c\ub9ac\uc544\uccad\uac1c\uad6c\ub9ac\uc18d\uc758 \uac1c\uad6c\ub9ac\ub4e4\uc740 \ud64d\ucc44\uac00 \uac00\ub85c\ub85c \ucc22\uc5b4\uc838 \uc788\uace0 \ub208\uaebc\ud480\uc5d0 \uc0c9\uc18c \uac00 \uc5c6\ub294 \uc810\uc73c\ub85c \ub2e4\ub978 \uccad\uac1c\uad6c\ub9ac\ub4e4\uacfc \uad6c\ubd84\ub41c\ub2e4. \uc624\uc2a4\ud2b8\ub808\uc77c\ub9ac\uc544\uccad\uac1c\uad6c\ub9ac\uc758 \ub2e4\ub978 \uc774\ub984\uc778 \"\ud654\uc774\ud2b8\uccad\uac1c\uad6c\ub9ac\"\ub294 1790\ub144 \ub188\uc758 \ud559\uba85\uc744 \ucc98\uc74c \uba85\uba85\ud55c \uc874 \ud654\uc774\ud2b8\uc758 \uc774\ub984\uc774 \ubd99\uc740 \uac83\uc774\ub2e4. \uc624\uc2a4\ud2b8\ub808\uc77c\ub9ac\uc544\uccad\uac1c\uad6c\ub9ac\ub294 \uc624\uc2a4\ud2b8\ub808\uc77c\ub9ac\uc544\uc5d0 \uc11c\uc2dd\ud558\ub294 \uac1c\uad6c\ub9ac\ub4e4 \uc911 \ucc98\uc74c\uc73c\ub85c \uae30\uc7ac\ub41c \uc885\uc73c\ub85c\uc11c, \uadf8 \ucd5c\ucd08 \ud45c\ubcf8\uc740 \uc81c1\ub300 \ubc45\ud06c\uc2a4 \uc900\ub0a8\uc791 \uc870\uc9c0\ud504 \ubc45\ud06c\uc2a4\uc758 \uc218\uc9d1\ud488 \uceec\ub809\uc158\uc5d0 \ud3ec\ud568\ub418\uc5b4 \uc788\uc5c8\ub2e4. \uadf8\ub7ec\ub098 \ubd88\ud589\ud788\ub3c4 \uc774 \ud45c\ubcf8\uc740 \uc81c2\ucc28 \uc138\uacc4 \ub300\uc804 \ub54c \ud45c\ubcf8\uc744 \uc18c\uc7a5\ud558\ub358 \ubc15\ubb3c\uad00\uc774 \ud3ed\uaca9\ub2f9\ud568\uc73c\ub85c\uc368 \uc18c\uc2e4\ub418\uc5c8\ub2e4.", "paragraph_answer": "\uc624\uc2a4\ud2b8\ub808\uc77c\ub9ac\uc544\uccad\uac1c\uad6c\ub9ac\ub294 \uc2e0\ub300\ub959 \uccad\uac1c\uad6c\ub9ac\ub4e4\uc778 \uccad\uac1c\uad6c\ub9ac\uacfc(Hylidae)\uc758 \uc77c\uc6d0\uc73c\ub85c\uc11c \uc624\uc2a4\ud2b8\ub808\uc77c\ub9ac\uc544\uc640 \ub274\uae30\ub2c8 \uace0\uc720\uc885\uc778 \uc624\uc2a4\ud2b8\ub808\uc77c\ub9ac\uc544\uccad\uac1c\uad6c\ub9ac\uc544\uacfc(Pelodryadinae)\uc5d0 \uc18d\ud55c\ub2e4. \uc774 \uc544\uacfc\uc5d0\ub294 \ubb3c\uc9c0\ub2d8\uac1c\uad6c\ub9ac\uc18d(Cyclorana), \uc624\uc2a4\ud2b8\ub808\uc77c\ub9ac\uc544\uccad\uac1c\uad6c\ub9ac\uc18d(Litoria), \uc655\ub208\uc774\uccad\uac1c\uad6c\ub9ac\uc18d(Nyctimystes) 3\uc18d\uc5d0 160\uc5ec \uc885\uc774 \ud3ec\ud568\ub418\uc5b4 \uc788\ub2e4. \uc624\uc2a4\ud2b8\ub808\uc77c\ub9ac\uc544\uccad\uac1c\uad6c\ub9ac\uc18d\uc758 \uac1c\uad6c\ub9ac\ub4e4\uc740 \ud64d\ucc44\uac00 \uac00\ub85c\ub85c \ucc22\uc5b4\uc838 \uc788\uace0 \ub208\uaebc\ud480\uc5d0 \uc0c9\uc18c \uac00 \uc5c6\ub294 \uc810\uc73c\ub85c \ub2e4\ub978 \uccad\uac1c\uad6c\ub9ac\ub4e4\uacfc \uad6c\ubd84\ub41c\ub2e4. \uc624\uc2a4\ud2b8\ub808\uc77c\ub9ac\uc544\uccad\uac1c\uad6c\ub9ac\uc758 \ub2e4\ub978 \uc774\ub984\uc778 \"\ud654\uc774\ud2b8\uccad\uac1c\uad6c\ub9ac\"\ub294 1790\ub144 \ub188\uc758 \ud559\uba85\uc744 \ucc98\uc74c \uba85\uba85\ud55c \uc874 \ud654\uc774\ud2b8\uc758 \uc774\ub984\uc774 \ubd99\uc740 \uac83\uc774\ub2e4. \uc624\uc2a4\ud2b8\ub808\uc77c\ub9ac\uc544\uccad\uac1c\uad6c\ub9ac\ub294 \uc624\uc2a4\ud2b8\ub808\uc77c\ub9ac\uc544\uc5d0 \uc11c\uc2dd\ud558\ub294 \uac1c\uad6c\ub9ac\ub4e4 \uc911 \ucc98\uc74c\uc73c\ub85c \uae30\uc7ac\ub41c \uc885\uc73c\ub85c\uc11c, \uadf8 \ucd5c\ucd08 \ud45c\ubcf8\uc740 \uc81c1\ub300 \ubc45\ud06c\uc2a4 \uc900\ub0a8\uc791 \uc870\uc9c0\ud504 \ubc45\ud06c\uc2a4\uc758 \uc218\uc9d1\ud488 \uceec\ub809\uc158\uc5d0 \ud3ec\ud568\ub418\uc5b4 \uc788\uc5c8\ub2e4. \uadf8\ub7ec\ub098 \ubd88\ud589\ud788\ub3c4 \uc774 \ud45c\ubcf8\uc740 \uc81c2\ucc28 \uc138\uacc4 \ub300\uc804 \ub54c \ud45c\ubcf8\uc744 \uc18c\uc7a5\ud558\ub358 \ubc15\ubb3c\uad00\uc774 \ud3ed\uaca9\ub2f9\ud568\uc73c\ub85c\uc368 \uc18c\uc2e4\ub418\uc5c8\ub2e4.", "sentence_answer": "\uc624\uc2a4\ud2b8\ub808\uc77c\ub9ac\uc544\uccad\uac1c\uad6c\ub9ac\uc18d\uc758 \uac1c\uad6c\ub9ac\ub4e4\uc740 \ud64d\ucc44\uac00 \uac00\ub85c\ub85c \ucc22\uc5b4\uc838 \uc788\uace0 \ub208\uaebc\ud480\uc5d0 \uc0c9\uc18c \uac00 \uc5c6\ub294 \uc810\uc73c\ub85c \ub2e4\ub978 \uccad\uac1c\uad6c\ub9ac\ub4e4\uacfc \uad6c\ubd84\ub41c\ub2e4.", "paragraph_id": "6541274-0-1", "paragraph_question": "question: \uc624\uc2a4\ud2b8\ub9ac\uc544\uccad\uac1c\uad6c\ub9ac\uc18d\uc740 \ub2e4\ub978 \uccad\uac1c\uad6c\ub9ac\ub4e4\uacfc\ub294 \ub2ec\ub9ac \ub208\uaebc\ud480\uc5d0 \ubb34\uc5c7\uc774 \uc5c6\ub2e4 \ud558\uc600\ub294\uac00?, context: \uc624\uc2a4\ud2b8\ub808\uc77c\ub9ac\uc544\uccad\uac1c\uad6c\ub9ac\ub294 \uc2e0\ub300\ub959 \uccad\uac1c\uad6c\ub9ac\ub4e4\uc778 \uccad\uac1c\uad6c\ub9ac\uacfc(Hylidae)\uc758 \uc77c\uc6d0\uc73c\ub85c\uc11c \uc624\uc2a4\ud2b8\ub808\uc77c\ub9ac\uc544\uc640 \ub274\uae30\ub2c8 \uace0\uc720\uc885\uc778 \uc624\uc2a4\ud2b8\ub808\uc77c\ub9ac\uc544\uccad\uac1c\uad6c\ub9ac\uc544\uacfc(Pelodryadinae)\uc5d0 \uc18d\ud55c\ub2e4. \uc774 \uc544\uacfc\uc5d0\ub294 \ubb3c\uc9c0\ub2d8\uac1c\uad6c\ub9ac\uc18d(Cyclorana), \uc624\uc2a4\ud2b8\ub808\uc77c\ub9ac\uc544\uccad\uac1c\uad6c\ub9ac\uc18d(Litoria), \uc655\ub208\uc774\uccad\uac1c\uad6c\ub9ac\uc18d(Nyctimystes) 3\uc18d\uc5d0 160\uc5ec \uc885\uc774 \ud3ec\ud568\ub418\uc5b4 \uc788\ub2e4. \uc624\uc2a4\ud2b8\ub808\uc77c\ub9ac\uc544\uccad\uac1c\uad6c\ub9ac\uc18d\uc758 \uac1c\uad6c\ub9ac\ub4e4\uc740 \ud64d\ucc44\uac00 \uac00\ub85c\ub85c \ucc22\uc5b4\uc838 \uc788\uace0 \ub208\uaebc\ud480\uc5d0 \uc0c9\uc18c\uac00 \uc5c6\ub294 \uc810\uc73c\ub85c \ub2e4\ub978 \uccad\uac1c\uad6c\ub9ac\ub4e4\uacfc \uad6c\ubd84\ub41c\ub2e4. \uc624\uc2a4\ud2b8\ub808\uc77c\ub9ac\uc544\uccad\uac1c\uad6c\ub9ac\uc758 \ub2e4\ub978 \uc774\ub984\uc778 \"\ud654\uc774\ud2b8\uccad\uac1c\uad6c\ub9ac\"\ub294 1790\ub144 \ub188\uc758 \ud559\uba85\uc744 \ucc98\uc74c \uba85\uba85\ud55c \uc874 \ud654\uc774\ud2b8\uc758 \uc774\ub984\uc774 \ubd99\uc740 \uac83\uc774\ub2e4. \uc624\uc2a4\ud2b8\ub808\uc77c\ub9ac\uc544\uccad\uac1c\uad6c\ub9ac\ub294 \uc624\uc2a4\ud2b8\ub808\uc77c\ub9ac\uc544\uc5d0 \uc11c\uc2dd\ud558\ub294 \uac1c\uad6c\ub9ac\ub4e4 \uc911 \ucc98\uc74c\uc73c\ub85c \uae30\uc7ac\ub41c \uc885\uc73c\ub85c\uc11c, \uadf8 \ucd5c\ucd08 \ud45c\ubcf8\uc740 \uc81c1\ub300 \ubc45\ud06c\uc2a4 \uc900\ub0a8\uc791 \uc870\uc9c0\ud504 \ubc45\ud06c\uc2a4\uc758 \uc218\uc9d1\ud488 \uceec\ub809\uc158\uc5d0 \ud3ec\ud568\ub418\uc5b4 \uc788\uc5c8\ub2e4. \uadf8\ub7ec\ub098 \ubd88\ud589\ud788\ub3c4 \uc774 \ud45c\ubcf8\uc740 \uc81c2\ucc28 \uc138\uacc4 \ub300\uc804 \ub54c \ud45c\ubcf8\uc744 \uc18c\uc7a5\ud558\ub358 \ubc15\ubb3c\uad00\uc774 \ud3ed\uaca9\ub2f9\ud568\uc73c\ub85c\uc368 \uc18c\uc2e4\ub418\uc5c8\ub2e4."} {"question": "\ud558\ub098\ud68c \uc138\ub825\ub4e4\uc774 \ub300\ud1b5\ub839\uc758 \uc0ac\uc804 \uc2b9\uc778 \uc5c6\uc774 \uccb4\ud3ec\ud55c \uc0ac\ub78c\uc740?", "paragraph": "\uc81c5\uacf5\ud654\uad6d \ucd9c\ubc94 \uc804 \ud5c8\ud654\ud3c9\uc740 \ud5c8\uc0bc\uc218 \ubcf4\uc548\uc0ac\ub839\ubd80 \uc778\uc0ac\ucc98\uc7a5, \uc774\ud559\ubd09 \ubcf4\uc548\uc0ac\ub839\ubd80 \ub300\uacf5\ucc98\uc7a5 \ub4f1\uacfc \ud568\uaed8 '\ubcf4\uc548\uc0ac 3\uc778\ubc29'\uc73c\ub85c \ubd88\ub838\uc73c\uba70, \uc804\ub450\ud658\uc758 \uc9d1\uad8c \uc2dc\ub098\ub9ac\uc624\ub97c \uadf8\ub9b0 \uac83\uc73c\ub85c \uc54c\ub824\uc84c\ub2e4. \uadf8\ub7ec\ub098 \uc804\ub450\ud658 \ubcf8\uc778\uc740 \uc9d1\uad8c \uacc4\ud68d\uc774 \uc5c6\uc5c8\ub2e4\uace0 \uc8fc\uc7a5\ud588\ub2e4. \ub2e4\ub9cc \uad11\uc8fc\uc0ac\ud0dc \uc9c1\ud6c4 \ud63c\ub780\uc774 \uc624\uac8c\ub418\uace0 \ub2e4\ub4e4 \uc804\ub450\ud658\uc758 \ub4f1\uc744 \ub5a0\ubc00\uc5b4\uc11c 5\uacf5\ud654\uad6d\uc774 \ucd9c\ubc94\ud588\ub2e4\uace0 \ud588\ub2e4. \uadf8\ub7ec\ub098 \uc774\ub294 \ud558\uadf9\uc0c1\uc744 \uc77c\uc73c\ud0a8 \ud0c0\ub77d\ud55c \uad70\uc778\uc758 \ubcc0\uba85\uc77c\ubfd0, \ud5c8\ud654\ud3c9\uc744 \ud3ec\ud568\ud55c \ud558\ub098\ud68c 12.12 \uc138\ub825\ub4e4\uc740 \uc0c1\uad00\uc778 \uc815\uc2b9\ud654 \uc721\ucc38\ucd1d\uc7a5\uc744 \ub300\ud1b5\ub839\uc758 \uc0ac\uc804 \uc2b9\uc778 \uc5c6\uc774 \uccb4\ud3ec\ud558\ub294 \uac83\uc744 \uacc4\uae30\ub85c \uc815\uad6d\uc758 \uc8fc\ub3c4\uad8c\uc744 \uc7a5\uc545\ud558\uace0, \ubbfc\uc8fc\ud654\ub97c \uc694\uad6c\ud558\ub294 \uad11\uc8fc\uc2dc\ubbfc\ub4e4\uc744 \ubb34\ub825\uc73c\ub85c \uc9c4\uc555\ud558\ub294 \ub9cc\ud589\uc744 \uc800\uc9c8\ub800\ub2e4. \uc815\uad8c\uc744 \ucc2c\ud0c8\ud55c \ub4a4\uc5d0\ub294 \uae30\uc5c5\uc778\ub4e4\ub85c \ubd80\ud130 \uc815\uce58\uc790\uae08\uc744 \uac08\ucde8\ud558\uac70\ub098 \ud608\uc138\ub97c \uac1c\uc778\uc801\uc73c\ub85c \uc720\uc6a9\ud558\ub294 \ub4f1 \ube44\ub9ac\ub3c4 \uc2ec\uac01\ud558\uc5ec, \uc804\ub450\ud658 \ub4f1\uc740 \ubc95\uc6d0\uc73c\ub85c \ubd80\ud130 2205\uc5b5\uc6d0\uc73c\ub85c \ucd94\uc9d5\uae08\uc744 \uc120\uace0 \ubc1b\uae30\ub3c4 \ud558\uc600\ub2e4. 5\uacf5 \uc138\ub825\ub4e4\uc758 \uc77c\ub828\uc758 \uc545\ud589\ub4e4\uc740 \uc5ed\uc0ac\uc801 \uace0\uc99d\uc744 \uac70\uccd0 MBC \ub4dc\ub77c\ub9c8 \"\uc81c5\uacf5\ud654\uad6d\"\uc73c\ub85c \ubc29\uc601\ub418\uae30\ub3c4 \ud558\uc600\ub2e4.", "answer": "\uc815\uc2b9\ud654 \uc721\ucc38\ucd1d\uc7a5", "sentence": "\uc0c1\uad00\uc778 \uc815\uc2b9\ud654 \uc721\ucc38\ucd1d\uc7a5 \uc744 \ub300\ud1b5\ub839\uc758 \uc0ac\uc804 \uc2b9\uc778 \uc5c6\uc774 \uccb4\ud3ec\ud558\ub294 \uac83\uc744 \uacc4\uae30\ub85c \uc815\uad6d\uc758 \uc8fc\ub3c4\uad8c\uc744 \uc7a5\uc545\ud558\uace0, \ubbfc\uc8fc\ud654\ub97c \uc694\uad6c\ud558\ub294 \uad11\uc8fc\uc2dc\ubbfc\ub4e4\uc744 \ubb34\ub825\uc73c\ub85c \uc9c4\uc555\ud558\ub294 \ub9cc\ud589\uc744 \uc800\uc9c8\ub800\ub2e4.", "paragraph_sentence": "\uc81c5\uacf5\ud654\uad6d \ucd9c\ubc94 \uc804 \ud5c8\ud654\ud3c9\uc740 \ud5c8\uc0bc\uc218 \ubcf4\uc548\uc0ac\ub839\ubd80 \uc778\uc0ac\ucc98\uc7a5, \uc774\ud559\ubd09 \ubcf4\uc548\uc0ac\ub839\ubd80 \ub300\uacf5\ucc98\uc7a5 \ub4f1\uacfc \ud568\uaed8 '\ubcf4\uc548\uc0ac 3\uc778\ubc29'\uc73c\ub85c \ubd88\ub838\uc73c\uba70, \uc804\ub450\ud658\uc758 \uc9d1\uad8c \uc2dc\ub098\ub9ac\uc624\ub97c \uadf8\ub9b0 \uac83\uc73c\ub85c \uc54c\ub824\uc84c\ub2e4. \uadf8\ub7ec\ub098 \uc804\ub450\ud658 \ubcf8\uc778\uc740 \uc9d1\uad8c \uacc4\ud68d\uc774 \uc5c6\uc5c8\ub2e4\uace0 \uc8fc\uc7a5\ud588\ub2e4. \ub2e4\ub9cc \uad11\uc8fc\uc0ac\ud0dc \uc9c1\ud6c4 \ud63c\ub780\uc774 \uc624\uac8c\ub418\uace0 \ub2e4\ub4e4 \uc804\ub450\ud658\uc758 \ub4f1\uc744 \ub5a0\ubc00\uc5b4\uc11c 5\uacf5\ud654\uad6d\uc774 \ucd9c\ubc94\ud588\ub2e4\uace0 \ud588\ub2e4. \uadf8\ub7ec\ub098 \uc774\ub294 \ud558\uadf9\uc0c1\uc744 \uc77c\uc73c\ud0a8 \ud0c0\ub77d\ud55c \uad70\uc778\uc758 \ubcc0\uba85\uc77c\ubfd0, \ud5c8\ud654\ud3c9\uc744 \ud3ec\ud568\ud55c \ud558\ub098\ud68c 12.12 \uc138\ub825\ub4e4\uc740 \uc0c1\uad00\uc778 \uc815\uc2b9\ud654 \uc721\ucc38\ucd1d\uc7a5 \uc744 \ub300\ud1b5\ub839\uc758 \uc0ac\uc804 \uc2b9\uc778 \uc5c6\uc774 \uccb4\ud3ec\ud558\ub294 \uac83\uc744 \uacc4\uae30\ub85c \uc815\uad6d\uc758 \uc8fc\ub3c4\uad8c\uc744 \uc7a5\uc545\ud558\uace0, \ubbfc\uc8fc\ud654\ub97c \uc694\uad6c\ud558\ub294 \uad11\uc8fc\uc2dc\ubbfc\ub4e4\uc744 \ubb34\ub825\uc73c\ub85c \uc9c4\uc555\ud558\ub294 \ub9cc\ud589\uc744 \uc800\uc9c8\ub800\ub2e4. \uc815\uad8c\uc744 \ucc2c\ud0c8\ud55c \ub4a4\uc5d0\ub294 \uae30\uc5c5\uc778\ub4e4\ub85c \ubd80\ud130 \uc815\uce58\uc790\uae08\uc744 \uac08\ucde8\ud558\uac70\ub098 \ud608\uc138\ub97c \uac1c\uc778\uc801\uc73c\ub85c \uc720\uc6a9\ud558\ub294 \ub4f1 \ube44\ub9ac\ub3c4 \uc2ec\uac01\ud558\uc5ec, \uc804\ub450\ud658 \ub4f1\uc740 \ubc95\uc6d0\uc73c\ub85c \ubd80\ud130 2205\uc5b5\uc6d0\uc73c\ub85c \ucd94\uc9d5\uae08\uc744 \uc120\uace0 \ubc1b\uae30\ub3c4 \ud558\uc600\ub2e4. 5\uacf5 \uc138\ub825\ub4e4\uc758 \uc77c\ub828\uc758 \uc545\ud589\ub4e4\uc740 \uc5ed\uc0ac\uc801 \uace0\uc99d\uc744 \uac70\uccd0 MBC \ub4dc\ub77c\ub9c8 \"\uc81c5\uacf5\ud654\uad6d\"\uc73c\ub85c \ubc29\uc601\ub418\uae30\ub3c4 \ud558\uc600\ub2e4.", "paragraph_answer": "\uc81c5\uacf5\ud654\uad6d \ucd9c\ubc94 \uc804 \ud5c8\ud654\ud3c9\uc740 \ud5c8\uc0bc\uc218 \ubcf4\uc548\uc0ac\ub839\ubd80 \uc778\uc0ac\ucc98\uc7a5, \uc774\ud559\ubd09 \ubcf4\uc548\uc0ac\ub839\ubd80 \ub300\uacf5\ucc98\uc7a5 \ub4f1\uacfc \ud568\uaed8 '\ubcf4\uc548\uc0ac 3\uc778\ubc29'\uc73c\ub85c \ubd88\ub838\uc73c\uba70, \uc804\ub450\ud658\uc758 \uc9d1\uad8c \uc2dc\ub098\ub9ac\uc624\ub97c \uadf8\ub9b0 \uac83\uc73c\ub85c \uc54c\ub824\uc84c\ub2e4. \uadf8\ub7ec\ub098 \uc804\ub450\ud658 \ubcf8\uc778\uc740 \uc9d1\uad8c \uacc4\ud68d\uc774 \uc5c6\uc5c8\ub2e4\uace0 \uc8fc\uc7a5\ud588\ub2e4. \ub2e4\ub9cc \uad11\uc8fc\uc0ac\ud0dc \uc9c1\ud6c4 \ud63c\ub780\uc774 \uc624\uac8c\ub418\uace0 \ub2e4\ub4e4 \uc804\ub450\ud658\uc758 \ub4f1\uc744 \ub5a0\ubc00\uc5b4\uc11c 5\uacf5\ud654\uad6d\uc774 \ucd9c\ubc94\ud588\ub2e4\uace0 \ud588\ub2e4. \uadf8\ub7ec\ub098 \uc774\ub294 \ud558\uadf9\uc0c1\uc744 \uc77c\uc73c\ud0a8 \ud0c0\ub77d\ud55c \uad70\uc778\uc758 \ubcc0\uba85\uc77c\ubfd0, \ud5c8\ud654\ud3c9\uc744 \ud3ec\ud568\ud55c \ud558\ub098\ud68c 12.12 \uc138\ub825\ub4e4\uc740 \uc0c1\uad00\uc778 \uc815\uc2b9\ud654 \uc721\ucc38\ucd1d\uc7a5 \uc744 \ub300\ud1b5\ub839\uc758 \uc0ac\uc804 \uc2b9\uc778 \uc5c6\uc774 \uccb4\ud3ec\ud558\ub294 \uac83\uc744 \uacc4\uae30\ub85c \uc815\uad6d\uc758 \uc8fc\ub3c4\uad8c\uc744 \uc7a5\uc545\ud558\uace0, \ubbfc\uc8fc\ud654\ub97c \uc694\uad6c\ud558\ub294 \uad11\uc8fc\uc2dc\ubbfc\ub4e4\uc744 \ubb34\ub825\uc73c\ub85c \uc9c4\uc555\ud558\ub294 \ub9cc\ud589\uc744 \uc800\uc9c8\ub800\ub2e4. \uc815\uad8c\uc744 \ucc2c\ud0c8\ud55c \ub4a4\uc5d0\ub294 \uae30\uc5c5\uc778\ub4e4\ub85c \ubd80\ud130 \uc815\uce58\uc790\uae08\uc744 \uac08\ucde8\ud558\uac70\ub098 \ud608\uc138\ub97c \uac1c\uc778\uc801\uc73c\ub85c \uc720\uc6a9\ud558\ub294 \ub4f1 \ube44\ub9ac\ub3c4 \uc2ec\uac01\ud558\uc5ec, \uc804\ub450\ud658 \ub4f1\uc740 \ubc95\uc6d0\uc73c\ub85c \ubd80\ud130 2205\uc5b5\uc6d0\uc73c\ub85c \ucd94\uc9d5\uae08\uc744 \uc120\uace0 \ubc1b\uae30\ub3c4 \ud558\uc600\ub2e4. 5\uacf5 \uc138\ub825\ub4e4\uc758 \uc77c\ub828\uc758 \uc545\ud589\ub4e4\uc740 \uc5ed\uc0ac\uc801 \uace0\uc99d\uc744 \uac70\uccd0 MBC \ub4dc\ub77c\ub9c8 \"\uc81c5\uacf5\ud654\uad6d\"\uc73c\ub85c \ubc29\uc601\ub418\uae30\ub3c4 \ud558\uc600\ub2e4.", "sentence_answer": "\uc0c1\uad00\uc778 \uc815\uc2b9\ud654 \uc721\ucc38\ucd1d\uc7a5 \uc744 \ub300\ud1b5\ub839\uc758 \uc0ac\uc804 \uc2b9\uc778 \uc5c6\uc774 \uccb4\ud3ec\ud558\ub294 \uac83\uc744 \uacc4\uae30\ub85c \uc815\uad6d\uc758 \uc8fc\ub3c4\uad8c\uc744 \uc7a5\uc545\ud558\uace0, \ubbfc\uc8fc\ud654\ub97c \uc694\uad6c\ud558\ub294 \uad11\uc8fc\uc2dc\ubbfc\ub4e4\uc744 \ubb34\ub825\uc73c\ub85c \uc9c4\uc555\ud558\ub294 \ub9cc\ud589\uc744 \uc800\uc9c8\ub800\ub2e4.", "paragraph_id": "5943442-0-1", "paragraph_question": "question: \ud558\ub098\ud68c \uc138\ub825\ub4e4\uc774 \ub300\ud1b5\ub839\uc758 \uc0ac\uc804 \uc2b9\uc778 \uc5c6\uc774 \uccb4\ud3ec\ud55c \uc0ac\ub78c\uc740?, context: \uc81c5\uacf5\ud654\uad6d \ucd9c\ubc94 \uc804 \ud5c8\ud654\ud3c9\uc740 \ud5c8\uc0bc\uc218 \ubcf4\uc548\uc0ac\ub839\ubd80 \uc778\uc0ac\ucc98\uc7a5, \uc774\ud559\ubd09 \ubcf4\uc548\uc0ac\ub839\ubd80 \ub300\uacf5\ucc98\uc7a5 \ub4f1\uacfc \ud568\uaed8 '\ubcf4\uc548\uc0ac 3\uc778\ubc29'\uc73c\ub85c \ubd88\ub838\uc73c\uba70, \uc804\ub450\ud658\uc758 \uc9d1\uad8c \uc2dc\ub098\ub9ac\uc624\ub97c \uadf8\ub9b0 \uac83\uc73c\ub85c \uc54c\ub824\uc84c\ub2e4. \uadf8\ub7ec\ub098 \uc804\ub450\ud658 \ubcf8\uc778\uc740 \uc9d1\uad8c \uacc4\ud68d\uc774 \uc5c6\uc5c8\ub2e4\uace0 \uc8fc\uc7a5\ud588\ub2e4. \ub2e4\ub9cc \uad11\uc8fc\uc0ac\ud0dc \uc9c1\ud6c4 \ud63c\ub780\uc774 \uc624\uac8c\ub418\uace0 \ub2e4\ub4e4 \uc804\ub450\ud658\uc758 \ub4f1\uc744 \ub5a0\ubc00\uc5b4\uc11c 5\uacf5\ud654\uad6d\uc774 \ucd9c\ubc94\ud588\ub2e4\uace0 \ud588\ub2e4. \uadf8\ub7ec\ub098 \uc774\ub294 \ud558\uadf9\uc0c1\uc744 \uc77c\uc73c\ud0a8 \ud0c0\ub77d\ud55c \uad70\uc778\uc758 \ubcc0\uba85\uc77c\ubfd0, \ud5c8\ud654\ud3c9\uc744 \ud3ec\ud568\ud55c \ud558\ub098\ud68c 12.12 \uc138\ub825\ub4e4\uc740 \uc0c1\uad00\uc778 \uc815\uc2b9\ud654 \uc721\ucc38\ucd1d\uc7a5\uc744 \ub300\ud1b5\ub839\uc758 \uc0ac\uc804 \uc2b9\uc778 \uc5c6\uc774 \uccb4\ud3ec\ud558\ub294 \uac83\uc744 \uacc4\uae30\ub85c \uc815\uad6d\uc758 \uc8fc\ub3c4\uad8c\uc744 \uc7a5\uc545\ud558\uace0, \ubbfc\uc8fc\ud654\ub97c \uc694\uad6c\ud558\ub294 \uad11\uc8fc\uc2dc\ubbfc\ub4e4\uc744 \ubb34\ub825\uc73c\ub85c \uc9c4\uc555\ud558\ub294 \ub9cc\ud589\uc744 \uc800\uc9c8\ub800\ub2e4. \uc815\uad8c\uc744 \ucc2c\ud0c8\ud55c \ub4a4\uc5d0\ub294 \uae30\uc5c5\uc778\ub4e4\ub85c \ubd80\ud130 \uc815\uce58\uc790\uae08\uc744 \uac08\ucde8\ud558\uac70\ub098 \ud608\uc138\ub97c \uac1c\uc778\uc801\uc73c\ub85c \uc720\uc6a9\ud558\ub294 \ub4f1 \ube44\ub9ac\ub3c4 \uc2ec\uac01\ud558\uc5ec, \uc804\ub450\ud658 \ub4f1\uc740 \ubc95\uc6d0\uc73c\ub85c \ubd80\ud130 2205\uc5b5\uc6d0\uc73c\ub85c \ucd94\uc9d5\uae08\uc744 \uc120\uace0 \ubc1b\uae30\ub3c4 \ud558\uc600\ub2e4. 5\uacf5 \uc138\ub825\ub4e4\uc758 \uc77c\ub828\uc758 \uc545\ud589\ub4e4\uc740 \uc5ed\uc0ac\uc801 \uace0\uc99d\uc744 \uac70\uccd0 MBC \ub4dc\ub77c\ub9c8 \"\uc81c5\uacf5\ud654\uad6d\"\uc73c\ub85c \ubc29\uc601\ub418\uae30\ub3c4 \ud558\uc600\ub2e4."} {"question": "\uc804\ub450\ud658\uc774 \uc721\uc0ac\ub97c \uc7a5\uc545 \ud560 \uc218 \uc788\uac8c \ub3c4\uc6c0\uc744 \uc900 \uc0ac\ub78c\uc740?", "paragraph": "\uac19\uc740 \ud574 5\uc6d4 16\uc77c\uc5d0 5\u00b716 \uad70\uc0ac \ucfe0\ub370\ud0c0\uac00 \ubc1c\uc0dd\ud558\uc790 \uadf8\ub294 \uc721\uad70\uc0ac\uad00\ud559\uad50\ub85c \uac00\uc11c \uc721\uc0ac \uc0dd\ub3c4\ub4e4\uc758 5\u00b716 \uad70\uc0ac \ucfe0\ub370\ud0c0 \uc9c0\uc9c0 \uc2dc\uc704\ub97c \uc8fc\ub3c4\ud574 \uad6d\uac00\uc7ac\uac74\ucd5c\uace0\ud68c\uc758 \ubd80\uc758\uc7a5 \ubc15\uc815\ud76c\uc758 \ucd1d\uc560\uc744 \ubc1b\uc558\ub2e4. 5\u00b716 \uad70\uc0ac \ucfe0\ub370\ud0c0 \uacfc\uc815\uc5d0\uc11c \ubc15\uc815\ud76c\uc758 \ubd80\ud558 \uc7a5\uad50\ub4e4\uc740 \uc721\uad70\uc0ac\uad00\ud559\uad50\ub97c \uc7a5\uc545\ud558\uae30 \uc704\ud574 3\uba85\uc758 \uc7a5\uad50\ub4e4\uc758 \uc778\uc194 \ud558\uc5d0 \ubb34\uc7a5\ubcd1\ub825\ub4e4\uc744 \uc721\uad70\uc0ac\uad00\ud559\uad50\uc5d0 \ud30c\uacac\ud588\ub294\ub370 \uadf8 \uc9c0\ud718\uad00\uc740 \uc624\uce58\uc131 \ub300\ub839(\uc721\uc0ac 8\uae30), \ubc15\ucc3d\uc554 \ub300\ub839(\uc721\uc0ac 8\uae30) \uadf8\ub9ac\uace0 \ud6d7\ub0a0 \ubc15\uc815\ud76c\uc758 \ucd1d\uc560\ub97c \ubc1b\uac8c \ub420 \ucc28\uc9c0\ucca0 \ub300\uc704\uc600\ub2e4. \uadf8\ub7f0\ub370 \uc774\ub4e4\uc774 \ub9c9\uc0c1 \uc721\uc0ac\uc0dd\ub3c4\ub4e4\uc5d0\uac8c \ucfe0\ub370\ud0c0 \uc138\ub825\uc744 \uc9c0\uc9c0\ud558\uae30 \uc704\ud55c \uc2dc\uac00\ud589\uc9c4\uc744 \ud558\ub77c\uace0 \uc694\uad6c\ud588\uc73c\ub098 \uc0dd\ub3c4\ub4e4\uc740 \uac70\uc808\ud588\ub2e4. \uc774 \uacfc\uc815\uc5d0\uc11c \uc804\ub450\ud658\uc740 \ub2f9\uc2dc \uc721\uad70\uc0ac\uad00\ud559\uad50 \uad50\uc7a5\uc778 \uac15\uc601\ud6c8\uc758 \uc9c0\uc2dc\ub85c \uc0dd\ub3c4\ub4e4\uc774 \ucfe0\ub370\ud0c0\ub97c \uc9c0\uc9c0\ud558\ub294 \ud37c\ub808\uc774\ub4dc\ub97c \ud558\uc9c0 \uc54a\uace0 \uc788\ub2e4\ub294 \uc0ac\uc2e4\uc744 \ud30c\uc545\ud55c \ub4a4 \uc0c1\ubd80\uc5d0 \ubc00\uace0\ud588\ub2e4. \uc774\ud6c4 \uae40\uc885\ud544 \ub4f1\uc758 \uc9c0\uc6d0\ud558\uc5d0 \uc804\ub450\ud658\uc740 \uc721\uc0ac\ub97c \uc7a5\uc545\ud560 \uc218 \uc788\uc5c8\uace0 \uc721\uc0ac \uad50\uc7a5 \uac15\uc601\ud6c8\uc740 \uad6c\uae08\ub418\uc5c8\ub2e4. \uc774\ud6c4 \uc804\ub450\ud658\uc740 \ucd9c\uc138\uac00\ub3c4\uc758 \uae30\ubc18\uc744 \ub9c8\ub828\ud55c\ub2e4.", "answer": "\uae40\uc885\ud544", "sentence": "\uc774\ud6c4 \uae40\uc885\ud544 \ub4f1\uc758 \uc9c0\uc6d0\ud558\uc5d0 \uc804\ub450\ud658\uc740 \uc721\uc0ac\ub97c \uc7a5\uc545\ud560 \uc218 \uc788\uc5c8\uace0 \uc721\uc0ac \uad50\uc7a5 \uac15\uc601\ud6c8\uc740 \uad6c\uae08\ub418\uc5c8\ub2e4.", "paragraph_sentence": "\uac19\uc740 \ud574 5\uc6d4 16\uc77c\uc5d0 5\u00b716 \uad70\uc0ac \ucfe0\ub370\ud0c0\uac00 \ubc1c\uc0dd\ud558\uc790 \uadf8\ub294 \uc721\uad70\uc0ac\uad00\ud559\uad50\ub85c \uac00\uc11c \uc721\uc0ac \uc0dd\ub3c4\ub4e4\uc758 5\u00b716 \uad70\uc0ac \ucfe0\ub370\ud0c0 \uc9c0\uc9c0 \uc2dc\uc704\ub97c \uc8fc\ub3c4\ud574 \uad6d\uac00\uc7ac\uac74\ucd5c\uace0\ud68c\uc758 \ubd80\uc758\uc7a5 \ubc15\uc815\ud76c\uc758 \ucd1d\uc560\uc744 \ubc1b\uc558\ub2e4. 5\u00b716 \uad70\uc0ac \ucfe0\ub370\ud0c0 \uacfc\uc815\uc5d0\uc11c \ubc15\uc815\ud76c\uc758 \ubd80\ud558 \uc7a5\uad50\ub4e4\uc740 \uc721\uad70\uc0ac\uad00\ud559\uad50\ub97c \uc7a5\uc545\ud558\uae30 \uc704\ud574 3\uba85\uc758 \uc7a5\uad50\ub4e4\uc758 \uc778\uc194 \ud558\uc5d0 \ubb34\uc7a5\ubcd1\ub825\ub4e4\uc744 \uc721\uad70\uc0ac\uad00\ud559\uad50\uc5d0 \ud30c\uacac\ud588\ub294\ub370 \uadf8 \uc9c0\ud718\uad00\uc740 \uc624\uce58\uc131 \ub300\ub839(\uc721\uc0ac 8\uae30), \ubc15\ucc3d\uc554 \ub300\ub839(\uc721\uc0ac 8\uae30) \uadf8\ub9ac\uace0 \ud6d7\ub0a0 \ubc15\uc815\ud76c\uc758 \ucd1d\uc560\ub97c \ubc1b\uac8c \ub420 \ucc28\uc9c0\ucca0 \ub300\uc704\uc600\ub2e4. \uadf8\ub7f0\ub370 \uc774\ub4e4\uc774 \ub9c9\uc0c1 \uc721\uc0ac\uc0dd\ub3c4\ub4e4\uc5d0\uac8c \ucfe0\ub370\ud0c0 \uc138\ub825\uc744 \uc9c0\uc9c0\ud558\uae30 \uc704\ud55c \uc2dc\uac00\ud589\uc9c4\uc744 \ud558\ub77c\uace0 \uc694\uad6c\ud588\uc73c\ub098 \uc0dd\ub3c4\ub4e4\uc740 \uac70\uc808\ud588\ub2e4. \uc774 \uacfc\uc815\uc5d0\uc11c \uc804\ub450\ud658\uc740 \ub2f9\uc2dc \uc721\uad70\uc0ac\uad00\ud559\uad50 \uad50\uc7a5\uc778 \uac15\uc601\ud6c8\uc758 \uc9c0\uc2dc\ub85c \uc0dd\ub3c4\ub4e4\uc774 \ucfe0\ub370\ud0c0\ub97c \uc9c0\uc9c0\ud558\ub294 \ud37c\ub808\uc774\ub4dc\ub97c \ud558\uc9c0 \uc54a\uace0 \uc788\ub2e4\ub294 \uc0ac\uc2e4\uc744 \ud30c\uc545\ud55c \ub4a4 \uc0c1\ubd80\uc5d0 \ubc00\uace0\ud588\ub2e4. \uc774\ud6c4 \uae40\uc885\ud544 \ub4f1\uc758 \uc9c0\uc6d0\ud558\uc5d0 \uc804\ub450\ud658\uc740 \uc721\uc0ac\ub97c \uc7a5\uc545\ud560 \uc218 \uc788\uc5c8\uace0 \uc721\uc0ac \uad50\uc7a5 \uac15\uc601\ud6c8\uc740 \uad6c\uae08\ub418\uc5c8\ub2e4. \uc774\ud6c4 \uc804\ub450\ud658\uc740 \ucd9c\uc138\uac00\ub3c4\uc758 \uae30\ubc18\uc744 \ub9c8\ub828\ud55c\ub2e4.", "paragraph_answer": "\uac19\uc740 \ud574 5\uc6d4 16\uc77c\uc5d0 5\u00b716 \uad70\uc0ac \ucfe0\ub370\ud0c0\uac00 \ubc1c\uc0dd\ud558\uc790 \uadf8\ub294 \uc721\uad70\uc0ac\uad00\ud559\uad50\ub85c \uac00\uc11c \uc721\uc0ac \uc0dd\ub3c4\ub4e4\uc758 5\u00b716 \uad70\uc0ac \ucfe0\ub370\ud0c0 \uc9c0\uc9c0 \uc2dc\uc704\ub97c \uc8fc\ub3c4\ud574 \uad6d\uac00\uc7ac\uac74\ucd5c\uace0\ud68c\uc758 \ubd80\uc758\uc7a5 \ubc15\uc815\ud76c\uc758 \ucd1d\uc560\uc744 \ubc1b\uc558\ub2e4. 5\u00b716 \uad70\uc0ac \ucfe0\ub370\ud0c0 \uacfc\uc815\uc5d0\uc11c \ubc15\uc815\ud76c\uc758 \ubd80\ud558 \uc7a5\uad50\ub4e4\uc740 \uc721\uad70\uc0ac\uad00\ud559\uad50\ub97c \uc7a5\uc545\ud558\uae30 \uc704\ud574 3\uba85\uc758 \uc7a5\uad50\ub4e4\uc758 \uc778\uc194 \ud558\uc5d0 \ubb34\uc7a5\ubcd1\ub825\ub4e4\uc744 \uc721\uad70\uc0ac\uad00\ud559\uad50\uc5d0 \ud30c\uacac\ud588\ub294\ub370 \uadf8 \uc9c0\ud718\uad00\uc740 \uc624\uce58\uc131 \ub300\ub839(\uc721\uc0ac 8\uae30), \ubc15\ucc3d\uc554 \ub300\ub839(\uc721\uc0ac 8\uae30) \uadf8\ub9ac\uace0 \ud6d7\ub0a0 \ubc15\uc815\ud76c\uc758 \ucd1d\uc560\ub97c \ubc1b\uac8c \ub420 \ucc28\uc9c0\ucca0 \ub300\uc704\uc600\ub2e4. \uadf8\ub7f0\ub370 \uc774\ub4e4\uc774 \ub9c9\uc0c1 \uc721\uc0ac\uc0dd\ub3c4\ub4e4\uc5d0\uac8c \ucfe0\ub370\ud0c0 \uc138\ub825\uc744 \uc9c0\uc9c0\ud558\uae30 \uc704\ud55c \uc2dc\uac00\ud589\uc9c4\uc744 \ud558\ub77c\uace0 \uc694\uad6c\ud588\uc73c\ub098 \uc0dd\ub3c4\ub4e4\uc740 \uac70\uc808\ud588\ub2e4. \uc774 \uacfc\uc815\uc5d0\uc11c \uc804\ub450\ud658\uc740 \ub2f9\uc2dc \uc721\uad70\uc0ac\uad00\ud559\uad50 \uad50\uc7a5\uc778 \uac15\uc601\ud6c8\uc758 \uc9c0\uc2dc\ub85c \uc0dd\ub3c4\ub4e4\uc774 \ucfe0\ub370\ud0c0\ub97c \uc9c0\uc9c0\ud558\ub294 \ud37c\ub808\uc774\ub4dc\ub97c \ud558\uc9c0 \uc54a\uace0 \uc788\ub2e4\ub294 \uc0ac\uc2e4\uc744 \ud30c\uc545\ud55c \ub4a4 \uc0c1\ubd80\uc5d0 \ubc00\uace0\ud588\ub2e4. \uc774\ud6c4 \uae40\uc885\ud544 \ub4f1\uc758 \uc9c0\uc6d0\ud558\uc5d0 \uc804\ub450\ud658\uc740 \uc721\uc0ac\ub97c \uc7a5\uc545\ud560 \uc218 \uc788\uc5c8\uace0 \uc721\uc0ac \uad50\uc7a5 \uac15\uc601\ud6c8\uc740 \uad6c\uae08\ub418\uc5c8\ub2e4. \uc774\ud6c4 \uc804\ub450\ud658\uc740 \ucd9c\uc138\uac00\ub3c4\uc758 \uae30\ubc18\uc744 \ub9c8\ub828\ud55c\ub2e4.", "sentence_answer": "\uc774\ud6c4 \uae40\uc885\ud544 \ub4f1\uc758 \uc9c0\uc6d0\ud558\uc5d0 \uc804\ub450\ud658\uc740 \uc721\uc0ac\ub97c \uc7a5\uc545\ud560 \uc218 \uc788\uc5c8\uace0 \uc721\uc0ac \uad50\uc7a5 \uac15\uc601\ud6c8\uc740 \uad6c\uae08\ub418\uc5c8\ub2e4.", "paragraph_id": "6390080-4-1", "paragraph_question": "question: \uc804\ub450\ud658\uc774 \uc721\uc0ac\ub97c \uc7a5\uc545 \ud560 \uc218 \uc788\uac8c \ub3c4\uc6c0\uc744 \uc900 \uc0ac\ub78c\uc740?, context: \uac19\uc740 \ud574 5\uc6d4 16\uc77c\uc5d0 5\u00b716 \uad70\uc0ac \ucfe0\ub370\ud0c0\uac00 \ubc1c\uc0dd\ud558\uc790 \uadf8\ub294 \uc721\uad70\uc0ac\uad00\ud559\uad50\ub85c \uac00\uc11c \uc721\uc0ac \uc0dd\ub3c4\ub4e4\uc758 5\u00b716 \uad70\uc0ac \ucfe0\ub370\ud0c0 \uc9c0\uc9c0 \uc2dc\uc704\ub97c \uc8fc\ub3c4\ud574 \uad6d\uac00\uc7ac\uac74\ucd5c\uace0\ud68c\uc758 \ubd80\uc758\uc7a5 \ubc15\uc815\ud76c\uc758 \ucd1d\uc560\uc744 \ubc1b\uc558\ub2e4. 5\u00b716 \uad70\uc0ac \ucfe0\ub370\ud0c0 \uacfc\uc815\uc5d0\uc11c \ubc15\uc815\ud76c\uc758 \ubd80\ud558 \uc7a5\uad50\ub4e4\uc740 \uc721\uad70\uc0ac\uad00\ud559\uad50\ub97c \uc7a5\uc545\ud558\uae30 \uc704\ud574 3\uba85\uc758 \uc7a5\uad50\ub4e4\uc758 \uc778\uc194 \ud558\uc5d0 \ubb34\uc7a5\ubcd1\ub825\ub4e4\uc744 \uc721\uad70\uc0ac\uad00\ud559\uad50\uc5d0 \ud30c\uacac\ud588\ub294\ub370 \uadf8 \uc9c0\ud718\uad00\uc740 \uc624\uce58\uc131 \ub300\ub839(\uc721\uc0ac 8\uae30), \ubc15\ucc3d\uc554 \ub300\ub839(\uc721\uc0ac 8\uae30) \uadf8\ub9ac\uace0 \ud6d7\ub0a0 \ubc15\uc815\ud76c\uc758 \ucd1d\uc560\ub97c \ubc1b\uac8c \ub420 \ucc28\uc9c0\ucca0 \ub300\uc704\uc600\ub2e4. \uadf8\ub7f0\ub370 \uc774\ub4e4\uc774 \ub9c9\uc0c1 \uc721\uc0ac\uc0dd\ub3c4\ub4e4\uc5d0\uac8c \ucfe0\ub370\ud0c0 \uc138\ub825\uc744 \uc9c0\uc9c0\ud558\uae30 \uc704\ud55c \uc2dc\uac00\ud589\uc9c4\uc744 \ud558\ub77c\uace0 \uc694\uad6c\ud588\uc73c\ub098 \uc0dd\ub3c4\ub4e4\uc740 \uac70\uc808\ud588\ub2e4. \uc774 \uacfc\uc815\uc5d0\uc11c \uc804\ub450\ud658\uc740 \ub2f9\uc2dc \uc721\uad70\uc0ac\uad00\ud559\uad50 \uad50\uc7a5\uc778 \uac15\uc601\ud6c8\uc758 \uc9c0\uc2dc\ub85c \uc0dd\ub3c4\ub4e4\uc774 \ucfe0\ub370\ud0c0\ub97c \uc9c0\uc9c0\ud558\ub294 \ud37c\ub808\uc774\ub4dc\ub97c \ud558\uc9c0 \uc54a\uace0 \uc788\ub2e4\ub294 \uc0ac\uc2e4\uc744 \ud30c\uc545\ud55c \ub4a4 \uc0c1\ubd80\uc5d0 \ubc00\uace0\ud588\ub2e4. \uc774\ud6c4 \uae40\uc885\ud544 \ub4f1\uc758 \uc9c0\uc6d0\ud558\uc5d0 \uc804\ub450\ud658\uc740 \uc721\uc0ac\ub97c \uc7a5\uc545\ud560 \uc218 \uc788\uc5c8\uace0 \uc721\uc0ac \uad50\uc7a5 \uac15\uc601\ud6c8\uc740 \uad6c\uae08\ub418\uc5c8\ub2e4. \uc774\ud6c4 \uc804\ub450\ud658\uc740 \ucd9c\uc138\uac00\ub3c4\uc758 \uae30\ubc18\uc744 \ub9c8\ub828\ud55c\ub2e4."} {"question": "\uc7ad \ucff0\uc2a4\ub97c \uc560\uc2ac\ub808\ud2f1\uc2a4\uc5d0 \ucd94\ucc9c\ud55c \uc778\ubb3c\uc740 \ub204\uad6c\uc778\uac00?", "paragraph": "\ucff0\uc2a4 \uc790\uc2e0\uc740 \ub300\ud559 \uc878\uc5c5 \ud6c4 \ub9e4\uc0ac\ucd94\uc138\uce20 \uacf5\uacfc\ub300\ud559\uad50\uc5d0\uc11c \ud559\uc5c5\uc744 \uc774\uc5b4\uac08 \uacc4\ud68d\uc774\uc5c8\uc73c\ub098, \ud3c9\uc18c \uadf8\uc758 \uc2e4\ub825\uc744 \uc775\ud788 \uc54c\uace0 \uc788\uc5c8\ub358 \ud544\ub77c\ub378\ud53c\uc544 \uc560\uc2ac\ub808\ud2f1\uc2a4 \uac10\ub3c5 \ucf54\ub2c8 \ub9e5\uc758 \ub3d9\uc0dd \ud1b0 \ub9e5\uc774 \ud300\uc5d0 \uadf8\ub97c \ucd94\ucc9c\ud588\ub2e4. \uacb0\uad6d 1905\ub144 12\uc6d4\uc5d0 \uc560\uc2ac\ub808\ud2f1\uc2a4\uc640 \uc2dc\uc98c 2,400 \ub2ec\ub7ec\uc758 \uae08\uc561\uc73c\ub85c \uacc4\uc57d\ud588\ub2e4. \ub300\ud559\uc744 \uc878\uc5c5\ud55c \ud6c4 1906\ub144 7\uc6d4 2\uc77c, \uc7ad \ucff0\uc2a4\ub294 \uc560\uc2ac\ub808\ud2f1\uc2a4 \uc120\uc218\ub2e8\uc5d0 \ucc98\uc74c\uc73c\ub85c \ud569\ub958\ud574 \uc5f0\uc2b5 \ud22c\uad6c\ub97c \uc120\ubcf4\uc600\ub2e4. \uc0ac\ud758 \ub4a4\uc778 7\uc6d4 5\uc77c, \uc7ad \ucff0\uc2a4\ub294 \ud544\ub77c\ub378\ud53c\uc544 \uc560\uc2ac\ub808\ud2f1\uc2a4\uc640 \uc6cc\uc2f1\ud134 \uc138\ub108\ud130\uc2a4\uc758 \uacbd\uae30\uc5d0 \uba54\uc774\uc800 \ub9ac\uadf8 \ub370\ubdd4 \uc120\ubc1c \ub4f1\ud310\ud574 7\ud53c\uc548\ud0c0 5\ubcfc\ub137 6\ud0c8\uc0bc\uc9c4 \uc644\ubd09\uc2b9\uc744 \uae30\ub85d\ud588\uace0, \ud300\uc740 3-0\uc73c\ub85c \uc2b9\ub9ac\ud588\ub2e4. \uc774\ud6c4 7\uc6d4\uacfc 8\uc6d4 \uadf8\uc800\uadf8\ub7f0 \ud65c\uc57d\uc73c\ub85c 5\uc2b9 7\ud328\ub9cc\uc744 \uae30\ub85d\ud588\uc73c\ub098, 9\uc6d4 1\uc77c \ubcf4\uc2a4\ud134 \uc544\uba54\ub9ac\uce78\uc2a4\uc640 \ub354\ube14\ud5e4\ub354\ub85c \uc608\uc815\ub418\uc5c8\ub358 \uacbd\uae30 \uc911 \uccab \uc2dc\ud569\uc5d0\uc11c\ub294 \uc870 \ud574\ub9ac\uc2a4\uc758 \uc0c1\ub300\ub85c \uc120\ubc1c \ub4f1\ud310\ud574 24\uc774\ub2dd 15\ud53c\uc548\ud0c0 18\ud0c8\uc0bc\uc9c4 \uc644\ud22c\uc2b9\uc744 \uac70\ub450\uc5c8\ub2e4. \uc774\ubc88 \uc2dc\uc98c 23\uacbd\uae30(\uc120\ubc1c 18\uacbd\uae30)\uc5d0\uc11c 173\uc774\ub2dd\uc744 \ub358\uc9c0\uba70 10\uc2b9 10\ud328, 2.50\uc758 \ud3c9\uade0\uc790\ucc45\uc810\uc744 \uae30\ub85d\ud588\ub2e4.", "answer": "\ud1b0 \ub9e5", "sentence": "\ud3c9\uc18c \uadf8\uc758 \uc2e4\ub825\uc744 \uc775\ud788 \uc54c\uace0 \uc788\uc5c8\ub358 \ud544\ub77c\ub378\ud53c\uc544 \uc560\uc2ac\ub808\ud2f1\uc2a4 \uac10\ub3c5 \ucf54\ub2c8 \ub9e5\uc758 \ub3d9\uc0dd \ud1b0 \ub9e5 \uc774 \ud300\uc5d0 \uadf8\ub97c \ucd94\ucc9c\ud588\ub2e4.", "paragraph_sentence": "\ucff0\uc2a4 \uc790\uc2e0\uc740 \ub300\ud559 \uc878\uc5c5 \ud6c4 \ub9e4\uc0ac\ucd94\uc138\uce20 \uacf5\uacfc\ub300\ud559\uad50\uc5d0\uc11c \ud559\uc5c5\uc744 \uc774\uc5b4\uac08 \uacc4\ud68d\uc774\uc5c8\uc73c\ub098, \ud3c9\uc18c \uadf8\uc758 \uc2e4\ub825\uc744 \uc775\ud788 \uc54c\uace0 \uc788\uc5c8\ub358 \ud544\ub77c\ub378\ud53c\uc544 \uc560\uc2ac\ub808\ud2f1\uc2a4 \uac10\ub3c5 \ucf54\ub2c8 \ub9e5\uc758 \ub3d9\uc0dd \ud1b0 \ub9e5 \uc774 \ud300\uc5d0 \uadf8\ub97c \ucd94\ucc9c\ud588\ub2e4. \uacb0\uad6d 1905\ub144 12\uc6d4\uc5d0 \uc560\uc2ac\ub808\ud2f1\uc2a4\uc640 \uc2dc\uc98c 2,400 \ub2ec\ub7ec\uc758 \uae08\uc561\uc73c\ub85c \uacc4\uc57d\ud588\ub2e4. \ub300\ud559\uc744 \uc878\uc5c5\ud55c \ud6c4 1906\ub144 7\uc6d4 2\uc77c, \uc7ad \ucff0\uc2a4\ub294 \uc560\uc2ac\ub808\ud2f1\uc2a4 \uc120\uc218\ub2e8\uc5d0 \ucc98\uc74c\uc73c\ub85c \ud569\ub958\ud574 \uc5f0\uc2b5 \ud22c\uad6c\ub97c \uc120\ubcf4\uc600\ub2e4. \uc0ac\ud758 \ub4a4\uc778 7\uc6d4 5\uc77c, \uc7ad \ucff0\uc2a4\ub294 \ud544\ub77c\ub378\ud53c\uc544 \uc560\uc2ac\ub808\ud2f1\uc2a4\uc640 \uc6cc\uc2f1\ud134 \uc138\ub108\ud130\uc2a4\uc758 \uacbd\uae30\uc5d0 \uba54\uc774\uc800 \ub9ac\uadf8 \ub370\ubdd4 \uc120\ubc1c \ub4f1\ud310\ud574 7\ud53c\uc548\ud0c0 5\ubcfc\ub137 6\ud0c8\uc0bc\uc9c4 \uc644\ubd09\uc2b9\uc744 \uae30\ub85d\ud588\uace0, \ud300\uc740 3-0\uc73c\ub85c \uc2b9\ub9ac\ud588\ub2e4. \uc774\ud6c4 7\uc6d4\uacfc 8\uc6d4 \uadf8\uc800\uadf8\ub7f0 \ud65c\uc57d\uc73c\ub85c 5\uc2b9 7\ud328\ub9cc\uc744 \uae30\ub85d\ud588\uc73c\ub098, 9\uc6d4 1\uc77c \ubcf4\uc2a4\ud134 \uc544\uba54\ub9ac\uce78\uc2a4\uc640 \ub354\ube14\ud5e4\ub354\ub85c \uc608\uc815\ub418\uc5c8\ub358 \uacbd\uae30 \uc911 \uccab \uc2dc\ud569\uc5d0\uc11c\ub294 \uc870 \ud574\ub9ac\uc2a4\uc758 \uc0c1\ub300\ub85c \uc120\ubc1c \ub4f1\ud310\ud574 24\uc774\ub2dd 15\ud53c\uc548\ud0c0 18\ud0c8\uc0bc\uc9c4 \uc644\ud22c\uc2b9\uc744 \uac70\ub450\uc5c8\ub2e4. \uc774\ubc88 \uc2dc\uc98c 23\uacbd\uae30(\uc120\ubc1c 18\uacbd\uae30)\uc5d0\uc11c 173\uc774\ub2dd\uc744 \ub358\uc9c0\uba70 10\uc2b9 10\ud328, 2.50\uc758 \ud3c9\uade0\uc790\ucc45\uc810\uc744 \uae30\ub85d\ud588\ub2e4.", "paragraph_answer": "\ucff0\uc2a4 \uc790\uc2e0\uc740 \ub300\ud559 \uc878\uc5c5 \ud6c4 \ub9e4\uc0ac\ucd94\uc138\uce20 \uacf5\uacfc\ub300\ud559\uad50\uc5d0\uc11c \ud559\uc5c5\uc744 \uc774\uc5b4\uac08 \uacc4\ud68d\uc774\uc5c8\uc73c\ub098, \ud3c9\uc18c \uadf8\uc758 \uc2e4\ub825\uc744 \uc775\ud788 \uc54c\uace0 \uc788\uc5c8\ub358 \ud544\ub77c\ub378\ud53c\uc544 \uc560\uc2ac\ub808\ud2f1\uc2a4 \uac10\ub3c5 \ucf54\ub2c8 \ub9e5\uc758 \ub3d9\uc0dd \ud1b0 \ub9e5 \uc774 \ud300\uc5d0 \uadf8\ub97c \ucd94\ucc9c\ud588\ub2e4. \uacb0\uad6d 1905\ub144 12\uc6d4\uc5d0 \uc560\uc2ac\ub808\ud2f1\uc2a4\uc640 \uc2dc\uc98c 2,400 \ub2ec\ub7ec\uc758 \uae08\uc561\uc73c\ub85c \uacc4\uc57d\ud588\ub2e4. \ub300\ud559\uc744 \uc878\uc5c5\ud55c \ud6c4 1906\ub144 7\uc6d4 2\uc77c, \uc7ad \ucff0\uc2a4\ub294 \uc560\uc2ac\ub808\ud2f1\uc2a4 \uc120\uc218\ub2e8\uc5d0 \ucc98\uc74c\uc73c\ub85c \ud569\ub958\ud574 \uc5f0\uc2b5 \ud22c\uad6c\ub97c \uc120\ubcf4\uc600\ub2e4. \uc0ac\ud758 \ub4a4\uc778 7\uc6d4 5\uc77c, \uc7ad \ucff0\uc2a4\ub294 \ud544\ub77c\ub378\ud53c\uc544 \uc560\uc2ac\ub808\ud2f1\uc2a4\uc640 \uc6cc\uc2f1\ud134 \uc138\ub108\ud130\uc2a4\uc758 \uacbd\uae30\uc5d0 \uba54\uc774\uc800 \ub9ac\uadf8 \ub370\ubdd4 \uc120\ubc1c \ub4f1\ud310\ud574 7\ud53c\uc548\ud0c0 5\ubcfc\ub137 6\ud0c8\uc0bc\uc9c4 \uc644\ubd09\uc2b9\uc744 \uae30\ub85d\ud588\uace0, \ud300\uc740 3-0\uc73c\ub85c \uc2b9\ub9ac\ud588\ub2e4. \uc774\ud6c4 7\uc6d4\uacfc 8\uc6d4 \uadf8\uc800\uadf8\ub7f0 \ud65c\uc57d\uc73c\ub85c 5\uc2b9 7\ud328\ub9cc\uc744 \uae30\ub85d\ud588\uc73c\ub098, 9\uc6d4 1\uc77c \ubcf4\uc2a4\ud134 \uc544\uba54\ub9ac\uce78\uc2a4\uc640 \ub354\ube14\ud5e4\ub354\ub85c \uc608\uc815\ub418\uc5c8\ub358 \uacbd\uae30 \uc911 \uccab \uc2dc\ud569\uc5d0\uc11c\ub294 \uc870 \ud574\ub9ac\uc2a4\uc758 \uc0c1\ub300\ub85c \uc120\ubc1c \ub4f1\ud310\ud574 24\uc774\ub2dd 15\ud53c\uc548\ud0c0 18\ud0c8\uc0bc\uc9c4 \uc644\ud22c\uc2b9\uc744 \uac70\ub450\uc5c8\ub2e4. \uc774\ubc88 \uc2dc\uc98c 23\uacbd\uae30(\uc120\ubc1c 18\uacbd\uae30)\uc5d0\uc11c 173\uc774\ub2dd\uc744 \ub358\uc9c0\uba70 10\uc2b9 10\ud328, 2.50\uc758 \ud3c9\uade0\uc790\ucc45\uc810\uc744 \uae30\ub85d\ud588\ub2e4.", "sentence_answer": "\ud3c9\uc18c \uadf8\uc758 \uc2e4\ub825\uc744 \uc775\ud788 \uc54c\uace0 \uc788\uc5c8\ub358 \ud544\ub77c\ub378\ud53c\uc544 \uc560\uc2ac\ub808\ud2f1\uc2a4 \uac10\ub3c5 \ucf54\ub2c8 \ub9e5\uc758 \ub3d9\uc0dd \ud1b0 \ub9e5 \uc774 \ud300\uc5d0 \uadf8\ub97c \ucd94\ucc9c\ud588\ub2e4.", "paragraph_id": "5860426-1-1", "paragraph_question": "question: \uc7ad \ucff0\uc2a4\ub97c \uc560\uc2ac\ub808\ud2f1\uc2a4\uc5d0 \ucd94\ucc9c\ud55c \uc778\ubb3c\uc740 \ub204\uad6c\uc778\uac00?, context: \ucff0\uc2a4 \uc790\uc2e0\uc740 \ub300\ud559 \uc878\uc5c5 \ud6c4 \ub9e4\uc0ac\ucd94\uc138\uce20 \uacf5\uacfc\ub300\ud559\uad50\uc5d0\uc11c \ud559\uc5c5\uc744 \uc774\uc5b4\uac08 \uacc4\ud68d\uc774\uc5c8\uc73c\ub098, \ud3c9\uc18c \uadf8\uc758 \uc2e4\ub825\uc744 \uc775\ud788 \uc54c\uace0 \uc788\uc5c8\ub358 \ud544\ub77c\ub378\ud53c\uc544 \uc560\uc2ac\ub808\ud2f1\uc2a4 \uac10\ub3c5 \ucf54\ub2c8 \ub9e5\uc758 \ub3d9\uc0dd \ud1b0 \ub9e5\uc774 \ud300\uc5d0 \uadf8\ub97c \ucd94\ucc9c\ud588\ub2e4. \uacb0\uad6d 1905\ub144 12\uc6d4\uc5d0 \uc560\uc2ac\ub808\ud2f1\uc2a4\uc640 \uc2dc\uc98c 2,400 \ub2ec\ub7ec\uc758 \uae08\uc561\uc73c\ub85c \uacc4\uc57d\ud588\ub2e4. \ub300\ud559\uc744 \uc878\uc5c5\ud55c \ud6c4 1906\ub144 7\uc6d4 2\uc77c, \uc7ad \ucff0\uc2a4\ub294 \uc560\uc2ac\ub808\ud2f1\uc2a4 \uc120\uc218\ub2e8\uc5d0 \ucc98\uc74c\uc73c\ub85c \ud569\ub958\ud574 \uc5f0\uc2b5 \ud22c\uad6c\ub97c \uc120\ubcf4\uc600\ub2e4. \uc0ac\ud758 \ub4a4\uc778 7\uc6d4 5\uc77c, \uc7ad \ucff0\uc2a4\ub294 \ud544\ub77c\ub378\ud53c\uc544 \uc560\uc2ac\ub808\ud2f1\uc2a4\uc640 \uc6cc\uc2f1\ud134 \uc138\ub108\ud130\uc2a4\uc758 \uacbd\uae30\uc5d0 \uba54\uc774\uc800 \ub9ac\uadf8 \ub370\ubdd4 \uc120\ubc1c \ub4f1\ud310\ud574 7\ud53c\uc548\ud0c0 5\ubcfc\ub137 6\ud0c8\uc0bc\uc9c4 \uc644\ubd09\uc2b9\uc744 \uae30\ub85d\ud588\uace0, \ud300\uc740 3-0\uc73c\ub85c \uc2b9\ub9ac\ud588\ub2e4. \uc774\ud6c4 7\uc6d4\uacfc 8\uc6d4 \uadf8\uc800\uadf8\ub7f0 \ud65c\uc57d\uc73c\ub85c 5\uc2b9 7\ud328\ub9cc\uc744 \uae30\ub85d\ud588\uc73c\ub098, 9\uc6d4 1\uc77c \ubcf4\uc2a4\ud134 \uc544\uba54\ub9ac\uce78\uc2a4\uc640 \ub354\ube14\ud5e4\ub354\ub85c \uc608\uc815\ub418\uc5c8\ub358 \uacbd\uae30 \uc911 \uccab \uc2dc\ud569\uc5d0\uc11c\ub294 \uc870 \ud574\ub9ac\uc2a4\uc758 \uc0c1\ub300\ub85c \uc120\ubc1c \ub4f1\ud310\ud574 24\uc774\ub2dd 15\ud53c\uc548\ud0c0 18\ud0c8\uc0bc\uc9c4 \uc644\ud22c\uc2b9\uc744 \uac70\ub450\uc5c8\ub2e4. \uc774\ubc88 \uc2dc\uc98c 23\uacbd\uae30(\uc120\ubc1c 18\uacbd\uae30)\uc5d0\uc11c 173\uc774\ub2dd\uc744 \ub358\uc9c0\uba70 10\uc2b9 10\ud328, 2.50\uc758 \ud3c9\uade0\uc790\ucc45\uc810\uc744 \uae30\ub85d\ud588\ub2e4."} {"question": "1923\ub144 \uc789\uae00\ub79c\ub4dc FA\ucef5 \uacb0\uc2b9\uc804\uc758 \uc785\uc7a5\ub8cc \ucd1d\uc561\uc740?", "paragraph": "\ud55c \ub2e8\uccb4\uac00 \uacbd\uae30 \ud55c \ub2ec \ud6c4\uc5d0 \uacbd\uae30\uc7a5\uc744 \uc870\uc0ac\ud588\uace0, \uacbd\uae30\uc7a5 \uad00\ub9ac\uce21\uc5d0 \uc5ec\ub7ec \uad8c\uace0\uc0ac\ud56d\uc744 \uc81c\uc2dc\ud588\ub2e4. \uadf8\ub4e4\uc758 \uc81c\uc548\ub4e4 \uc911\uc5d0\ub294 \ud68c\uc804\uc2dd \ubb38\uc744 \ucd5c\uc2e0\uc2dd \ubb38\uc73c\ub85c \uad50\uccb4\ud558\ub294 \uac83, \ucd9c\uc785\ubb38\uacfc \ub09c\uac04\uc758 \ucd94\uac00 \uc124\uce58, \uadf8\ub9ac\uace0 \ud14c\ub77c\uc2a4\ub97c \ud55c \ud14c\ub77c\uc2a4\ub2f9 \ud558\ub098\uc758 \ubb38\uc744 \uac16\uac8c\ub054 \ub098\ub204\ub294 \uac83 \ub4f1\uc774 \ud3ec\ud568\ub418\uc5b4\uc788\ub2e4. \uadf8 \uc678\uc5d0\ub3c4 \uad00\ub9ac\uce21\uc740 \ub2e4\uc74c \ud574\ubd80\ud130 \uc5f4\ub9b4 \ubaa8\ub4e0 \uacb0\uc2b9\uc804\uc5d0\ub294 \ud45c\uc758 \uc608\ub9e4\ub97c \uc758\ubb34\ud654\ud568\uc73c\ub85c\uc368 \ub2f9\uc77c\uc5d0 \uacbd\uae30\uc7a5\uc758 \uc218\uc6a9 \ub2a5\ub825\uc744 \ub118\uae30\ub294 \uc0ac\ub78c\ub4e4\uc774 \ud639\uc2dc \ucd9c\uc785\uad6c\uc5d0\uc11c \ud45c\ub97c \uad6c\ud560 \uc218 \uc788\uc744\uae4c\ub77c\ub294 \uae30\ub300\ub97c \uac16\uace0 \uacbd\uae30\uc7a5\uc744 \ucc3e\uc544\uc624\uc9c0 \ubabb\ud558\uac8c \ubc29\uc9c0\ud558\uc600\ub2e4. \ucd1d \uc785\uc7a5\ub8cc\ub294 27,776 \ud30c\uc6b4\ub4dc\uc600\ub2e4. \uacbd\uae30\uc7a5 \uad00\ub9ac \ube44\uc6a9\uc744 \ube80 \ub4a4\uc5d0, \uc789\uae00\ub79c\ub4dc \ucd95\uad6c\ud611\ud68c\uc640 \uacb0\uc2b9\uc804\uc758 \ub450 \ud300\uc774 \uac01\uac01 \u00a36,365\uc529 \ub098\ub220\uac00\uc84c\ub294\ub370 \ud45c\ub97c \uc608\ub9e4\ud55c \uad00\uc911\ub4e4\uc774 \uc790\uc2e0\ub4e4\uc758 \uc9c0\uc815\uc11d\uc5d0 \uc549\uc9c0 \ubabb\ud568\uc73c\ub85c \uc778\ud574 \ubc1c\uc0dd\ud55c \ud658\ubd88 \ube44\uc6a9\uc740 \ucd95\uad6c\ud611\ud68c\uc758 \ubaab\uc5d0\uc11c \ube60\uc84c\ub2e4.", "answer": "27,776 \ud30c\uc6b4\ub4dc", "sentence": "\ucd1d \uc785\uc7a5\ub8cc\ub294 27,776 \ud30c\uc6b4\ub4dc \uc600\ub2e4.", "paragraph_sentence": "\ud55c \ub2e8\uccb4\uac00 \uacbd\uae30 \ud55c \ub2ec \ud6c4\uc5d0 \uacbd\uae30\uc7a5\uc744 \uc870\uc0ac\ud588\uace0, \uacbd\uae30\uc7a5 \uad00\ub9ac\uce21\uc5d0 \uc5ec\ub7ec \uad8c\uace0\uc0ac\ud56d\uc744 \uc81c\uc2dc\ud588\ub2e4. \uadf8\ub4e4\uc758 \uc81c\uc548\ub4e4 \uc911\uc5d0\ub294 \ud68c\uc804\uc2dd \ubb38\uc744 \ucd5c\uc2e0\uc2dd \ubb38\uc73c\ub85c \uad50\uccb4\ud558\ub294 \uac83, \ucd9c\uc785\ubb38\uacfc \ub09c\uac04\uc758 \ucd94\uac00 \uc124\uce58, \uadf8\ub9ac\uace0 \ud14c\ub77c\uc2a4\ub97c \ud55c \ud14c\ub77c\uc2a4\ub2f9 \ud558\ub098\uc758 \ubb38\uc744 \uac16\uac8c\ub054 \ub098\ub204\ub294 \uac83 \ub4f1\uc774 \ud3ec\ud568\ub418\uc5b4\uc788\ub2e4. \uadf8 \uc678\uc5d0\ub3c4 \uad00\ub9ac\uce21\uc740 \ub2e4\uc74c \ud574\ubd80\ud130 \uc5f4\ub9b4 \ubaa8\ub4e0 \uacb0\uc2b9\uc804\uc5d0\ub294 \ud45c\uc758 \uc608\ub9e4\ub97c \uc758\ubb34\ud654\ud568\uc73c\ub85c\uc368 \ub2f9\uc77c\uc5d0 \uacbd\uae30\uc7a5\uc758 \uc218\uc6a9 \ub2a5\ub825\uc744 \ub118\uae30\ub294 \uc0ac\ub78c\ub4e4\uc774 \ud639\uc2dc \ucd9c\uc785\uad6c\uc5d0\uc11c \ud45c\ub97c \uad6c\ud560 \uc218 \uc788\uc744\uae4c\ub77c\ub294 \uae30\ub300\ub97c \uac16\uace0 \uacbd\uae30\uc7a5\uc744 \ucc3e\uc544\uc624\uc9c0 \ubabb\ud558\uac8c \ubc29\uc9c0\ud558\uc600\ub2e4. \ucd1d \uc785\uc7a5\ub8cc\ub294 27,776 \ud30c\uc6b4\ub4dc \uc600\ub2e4. \uacbd\uae30\uc7a5 \uad00\ub9ac \ube44\uc6a9\uc744 \ube80 \ub4a4\uc5d0, \uc789\uae00\ub79c\ub4dc \ucd95\uad6c\ud611\ud68c\uc640 \uacb0\uc2b9\uc804\uc758 \ub450 \ud300\uc774 \uac01\uac01 \u00a36,365\uc529 \ub098\ub220\uac00\uc84c\ub294\ub370 \ud45c\ub97c \uc608\ub9e4\ud55c \uad00\uc911\ub4e4\uc774 \uc790\uc2e0\ub4e4\uc758 \uc9c0\uc815\uc11d\uc5d0 \uc549\uc9c0 \ubabb\ud568\uc73c\ub85c \uc778\ud574 \ubc1c\uc0dd\ud55c \ud658\ubd88 \ube44\uc6a9\uc740 \ucd95\uad6c\ud611\ud68c\uc758 \ubaab\uc5d0\uc11c \ube60\uc84c\ub2e4.", "paragraph_answer": "\ud55c \ub2e8\uccb4\uac00 \uacbd\uae30 \ud55c \ub2ec \ud6c4\uc5d0 \uacbd\uae30\uc7a5\uc744 \uc870\uc0ac\ud588\uace0, \uacbd\uae30\uc7a5 \uad00\ub9ac\uce21\uc5d0 \uc5ec\ub7ec \uad8c\uace0\uc0ac\ud56d\uc744 \uc81c\uc2dc\ud588\ub2e4. \uadf8\ub4e4\uc758 \uc81c\uc548\ub4e4 \uc911\uc5d0\ub294 \ud68c\uc804\uc2dd \ubb38\uc744 \ucd5c\uc2e0\uc2dd \ubb38\uc73c\ub85c \uad50\uccb4\ud558\ub294 \uac83, \ucd9c\uc785\ubb38\uacfc \ub09c\uac04\uc758 \ucd94\uac00 \uc124\uce58, \uadf8\ub9ac\uace0 \ud14c\ub77c\uc2a4\ub97c \ud55c \ud14c\ub77c\uc2a4\ub2f9 \ud558\ub098\uc758 \ubb38\uc744 \uac16\uac8c\ub054 \ub098\ub204\ub294 \uac83 \ub4f1\uc774 \ud3ec\ud568\ub418\uc5b4\uc788\ub2e4. \uadf8 \uc678\uc5d0\ub3c4 \uad00\ub9ac\uce21\uc740 \ub2e4\uc74c \ud574\ubd80\ud130 \uc5f4\ub9b4 \ubaa8\ub4e0 \uacb0\uc2b9\uc804\uc5d0\ub294 \ud45c\uc758 \uc608\ub9e4\ub97c \uc758\ubb34\ud654\ud568\uc73c\ub85c\uc368 \ub2f9\uc77c\uc5d0 \uacbd\uae30\uc7a5\uc758 \uc218\uc6a9 \ub2a5\ub825\uc744 \ub118\uae30\ub294 \uc0ac\ub78c\ub4e4\uc774 \ud639\uc2dc \ucd9c\uc785\uad6c\uc5d0\uc11c \ud45c\ub97c \uad6c\ud560 \uc218 \uc788\uc744\uae4c\ub77c\ub294 \uae30\ub300\ub97c \uac16\uace0 \uacbd\uae30\uc7a5\uc744 \ucc3e\uc544\uc624\uc9c0 \ubabb\ud558\uac8c \ubc29\uc9c0\ud558\uc600\ub2e4. \ucd1d \uc785\uc7a5\ub8cc\ub294 27,776 \ud30c\uc6b4\ub4dc \uc600\ub2e4. \uacbd\uae30\uc7a5 \uad00\ub9ac \ube44\uc6a9\uc744 \ube80 \ub4a4\uc5d0, \uc789\uae00\ub79c\ub4dc \ucd95\uad6c\ud611\ud68c\uc640 \uacb0\uc2b9\uc804\uc758 \ub450 \ud300\uc774 \uac01\uac01 \u00a36,365\uc529 \ub098\ub220\uac00\uc84c\ub294\ub370 \ud45c\ub97c \uc608\ub9e4\ud55c \uad00\uc911\ub4e4\uc774 \uc790\uc2e0\ub4e4\uc758 \uc9c0\uc815\uc11d\uc5d0 \uc549\uc9c0 \ubabb\ud568\uc73c\ub85c \uc778\ud574 \ubc1c\uc0dd\ud55c \ud658\ubd88 \ube44\uc6a9\uc740 \ucd95\uad6c\ud611\ud68c\uc758 \ubaab\uc5d0\uc11c \ube60\uc84c\ub2e4.", "sentence_answer": "\ucd1d \uc785\uc7a5\ub8cc\ub294 27,776 \ud30c\uc6b4\ub4dc \uc600\ub2e4.", "paragraph_id": "5782571-9-0", "paragraph_question": "question: 1923\ub144 \uc789\uae00\ub79c\ub4dc FA\ucef5 \uacb0\uc2b9\uc804\uc758 \uc785\uc7a5\ub8cc \ucd1d\uc561\uc740?, context: \ud55c \ub2e8\uccb4\uac00 \uacbd\uae30 \ud55c \ub2ec \ud6c4\uc5d0 \uacbd\uae30\uc7a5\uc744 \uc870\uc0ac\ud588\uace0, \uacbd\uae30\uc7a5 \uad00\ub9ac\uce21\uc5d0 \uc5ec\ub7ec \uad8c\uace0\uc0ac\ud56d\uc744 \uc81c\uc2dc\ud588\ub2e4. \uadf8\ub4e4\uc758 \uc81c\uc548\ub4e4 \uc911\uc5d0\ub294 \ud68c\uc804\uc2dd \ubb38\uc744 \ucd5c\uc2e0\uc2dd \ubb38\uc73c\ub85c \uad50\uccb4\ud558\ub294 \uac83, \ucd9c\uc785\ubb38\uacfc \ub09c\uac04\uc758 \ucd94\uac00 \uc124\uce58, \uadf8\ub9ac\uace0 \ud14c\ub77c\uc2a4\ub97c \ud55c \ud14c\ub77c\uc2a4\ub2f9 \ud558\ub098\uc758 \ubb38\uc744 \uac16\uac8c\ub054 \ub098\ub204\ub294 \uac83 \ub4f1\uc774 \ud3ec\ud568\ub418\uc5b4\uc788\ub2e4. \uadf8 \uc678\uc5d0\ub3c4 \uad00\ub9ac\uce21\uc740 \ub2e4\uc74c \ud574\ubd80\ud130 \uc5f4\ub9b4 \ubaa8\ub4e0 \uacb0\uc2b9\uc804\uc5d0\ub294 \ud45c\uc758 \uc608\ub9e4\ub97c \uc758\ubb34\ud654\ud568\uc73c\ub85c\uc368 \ub2f9\uc77c\uc5d0 \uacbd\uae30\uc7a5\uc758 \uc218\uc6a9 \ub2a5\ub825\uc744 \ub118\uae30\ub294 \uc0ac\ub78c\ub4e4\uc774 \ud639\uc2dc \ucd9c\uc785\uad6c\uc5d0\uc11c \ud45c\ub97c \uad6c\ud560 \uc218 \uc788\uc744\uae4c\ub77c\ub294 \uae30\ub300\ub97c \uac16\uace0 \uacbd\uae30\uc7a5\uc744 \ucc3e\uc544\uc624\uc9c0 \ubabb\ud558\uac8c \ubc29\uc9c0\ud558\uc600\ub2e4. \ucd1d \uc785\uc7a5\ub8cc\ub294 27,776 \ud30c\uc6b4\ub4dc\uc600\ub2e4. \uacbd\uae30\uc7a5 \uad00\ub9ac \ube44\uc6a9\uc744 \ube80 \ub4a4\uc5d0, \uc789\uae00\ub79c\ub4dc \ucd95\uad6c\ud611\ud68c\uc640 \uacb0\uc2b9\uc804\uc758 \ub450 \ud300\uc774 \uac01\uac01 \u00a36,365\uc529 \ub098\ub220\uac00\uc84c\ub294\ub370 \ud45c\ub97c \uc608\ub9e4\ud55c \uad00\uc911\ub4e4\uc774 \uc790\uc2e0\ub4e4\uc758 \uc9c0\uc815\uc11d\uc5d0 \uc549\uc9c0 \ubabb\ud568\uc73c\ub85c \uc778\ud574 \ubc1c\uc0dd\ud55c \ud658\ubd88 \ube44\uc6a9\uc740 \ucd95\uad6c\ud611\ud68c\uc758 \ubaab\uc5d0\uc11c \ube60\uc84c\ub2e4."} {"question": "\uccad\ub144\ud559\ud30c\ub294 \uc815\uce58\uc778\uacfc \ub204\uac00 \ub9d0\ub824\ub4e4\uba70 \ub17c\ub780\uc744 \ubd88\ub7ec\uc654\ub098?", "paragraph": "\uace7 \uccad\ub144\ud559\ud30c\ub294 \uc815\uce58\uc778\uacfc \uc5b8\ub860\ub3c4 \ub9d0\ub824\ub4e4\uba70 \ub17c\ub780\uc744 \ubd88\ub7ec\uc654\ub2e4. 1886\ub144 1\uc6d4 7\uc77c\uc5d0 \uc624\ube0c\ub294 \ud574\uad70 \uc7a5\uad00\uc5d0 \uc784\uba85\ub418\uc5c8\uc9c0\ub9cc, \uc774\uac83\uc740 \uc624\ube0c\uc758 \uc0ac\uc0c1\uc744 \uc778\uc815\ud55c \uac83\uc774\ub77c\uae30 \ubcf4\ub2e4\ub294 \uc815\uce58\uc801 \ud310\ub2e8\uc5d0 \uc758\ud55c \uac83\uc774\uc5c8\ub2e4. \ud574\uad70\uc7a5\uad00\uc774 \ub41c \uc624\ube0c\ub294 \uc7ac\ube60\ub974\uac8c \uadf8\uc758 \uc0ac\uc0c1\uc758 \uc2e4\ud604\ud558\ub824 \ud588\ub2e4. 1861\ub144 2\uc6d4\uc5d0\ub294 \uc258\ubd80\ub974\uc5d0\uc11c \uc9c0\ube0c\ub864\ud130 \ud574\ud611\uc744 \ub3cc\uc544\uc11c \ud234\ub871\uae4c\uc9c0 \uc5b4\ub8b0\uc815\uc744 \ud56d\ud574\uc2dc\ucf30\ub294\ub370, \uae38\uc774 33m \uc815\ub3c4\uc758 \uc5b4\ub8b0\uc815\uc73c\ub85c \uacf5\ud574\ub97c \ud56d\ud574\ud558\ub294 \uac83\uc740 \uc5b4\ub824\uc6e0\ub2e4. \uac19\uc740 \ud574 5\uc6d4\uacfc 6\uc6d4\uc5d0\ub294 8\ucc99\uc758 \uc804\ud568\uc73c\ub85c \uc774\ub8e8\uc5b4\uc9c4 \uacf5\uaca9\uce21\uc744 20\ucc99 \ub0a8\uc9d3\uc758 \uc5b4\ub8b0\uc815(\uc21c\uc591\ud568 3\ucc99\uacfc \ud574\ubc29\uc804\ud568 1\ucc99\uc774 \uc9c0\uc6d0)\uc73c\ub85c \uc800\uc9c0\ud558\ub294 \uc5f0\uc2b5\uc744 \ud588\ub2e4. \uacf5\uaca9\uce21\uc740 \ud234\ub871\uc744 \ud568\ud3ec \uc0ac\uaca9\ud558\ub294 \ub370\ub294 \uc131\uacf5\ud588\uc9c0\ub9cc, \ub300\ubd80\ubd84\uc774 \uaca9\uce68 \ud310\uc815\uc744 \ubc1b\uc558\ub2e4. \uc774\ub85c \uc778\ud574 \ub300\ud615\ud568\uc5d0 \uc758\ud55c \uadfc\uc811 \ubd09\uc1c4\uac00 \ubd88\uac00\ub2a5\ud558\ub2e4\ub294 \uac83\uc774 \uc785\uc99d\ub418\uc5c8\ub2e4. \uac19\uc740 \ud574 12\uc6d4, \uc624\ube0c\ub294 \ub2e4\uc74c\uacfc \uac19\uc774 \uc804\ub7b5\uc744 \ubcc0\uacbd\ud588\ub2e4.", "answer": "\uc5b8\ub860", "sentence": "\uace7 \uccad\ub144\ud559\ud30c\ub294 \uc815\uce58\uc778\uacfc \uc5b8\ub860 \ub3c4 \ub9d0\ub824\ub4e4\uba70 \ub17c\ub780\uc744 \ubd88\ub7ec\uc654\ub2e4.", "paragraph_sentence": " \uace7 \uccad\ub144\ud559\ud30c\ub294 \uc815\uce58\uc778\uacfc \uc5b8\ub860 \ub3c4 \ub9d0\ub824\ub4e4\uba70 \ub17c\ub780\uc744 \ubd88\ub7ec\uc654\ub2e4. 1886\ub144 1\uc6d4 7\uc77c\uc5d0 \uc624\ube0c\ub294 \ud574\uad70 \uc7a5\uad00\uc5d0 \uc784\uba85\ub418\uc5c8\uc9c0\ub9cc, \uc774\uac83\uc740 \uc624\ube0c\uc758 \uc0ac\uc0c1\uc744 \uc778\uc815\ud55c \uac83\uc774\ub77c\uae30 \ubcf4\ub2e4\ub294 \uc815\uce58\uc801 \ud310\ub2e8\uc5d0 \uc758\ud55c \uac83\uc774\uc5c8\ub2e4. \ud574\uad70\uc7a5\uad00\uc774 \ub41c \uc624\ube0c\ub294 \uc7ac\ube60\ub974\uac8c \uadf8\uc758 \uc0ac\uc0c1\uc758 \uc2e4\ud604\ud558\ub824 \ud588\ub2e4. 1861\ub144 2\uc6d4\uc5d0\ub294 \uc258\ubd80\ub974\uc5d0\uc11c \uc9c0\ube0c\ub864\ud130 \ud574\ud611\uc744 \ub3cc\uc544\uc11c \ud234\ub871\uae4c\uc9c0 \uc5b4\ub8b0\uc815\uc744 \ud56d\ud574\uc2dc\ucf30\ub294\ub370, \uae38\uc774 33m \uc815\ub3c4\uc758 \uc5b4\ub8b0\uc815\uc73c\ub85c \uacf5\ud574\ub97c \ud56d\ud574\ud558\ub294 \uac83\uc740 \uc5b4\ub824\uc6e0\ub2e4. \uac19\uc740 \ud574 5\uc6d4\uacfc 6\uc6d4\uc5d0\ub294 8\ucc99\uc758 \uc804\ud568\uc73c\ub85c \uc774\ub8e8\uc5b4\uc9c4 \uacf5\uaca9\uce21\uc744 20\ucc99 \ub0a8\uc9d3\uc758 \uc5b4\ub8b0\uc815(\uc21c\uc591\ud568 3\ucc99\uacfc \ud574\ubc29\uc804\ud568 1\ucc99\uc774 \uc9c0\uc6d0)\uc73c\ub85c \uc800\uc9c0\ud558\ub294 \uc5f0\uc2b5\uc744 \ud588\ub2e4. \uacf5\uaca9\uce21\uc740 \ud234\ub871\uc744 \ud568\ud3ec \uc0ac\uaca9\ud558\ub294 \ub370\ub294 \uc131\uacf5\ud588\uc9c0\ub9cc, \ub300\ubd80\ubd84\uc774 \uaca9\uce68 \ud310\uc815\uc744 \ubc1b\uc558\ub2e4. \uc774\ub85c \uc778\ud574 \ub300\ud615\ud568\uc5d0 \uc758\ud55c \uadfc\uc811 \ubd09\uc1c4\uac00 \ubd88\uac00\ub2a5\ud558\ub2e4\ub294 \uac83\uc774 \uc785\uc99d\ub418\uc5c8\ub2e4. \uac19\uc740 \ud574 12\uc6d4, \uc624\ube0c\ub294 \ub2e4\uc74c\uacfc \uac19\uc774 \uc804\ub7b5\uc744 \ubcc0\uacbd\ud588\ub2e4.", "paragraph_answer": "\uace7 \uccad\ub144\ud559\ud30c\ub294 \uc815\uce58\uc778\uacfc \uc5b8\ub860 \ub3c4 \ub9d0\ub824\ub4e4\uba70 \ub17c\ub780\uc744 \ubd88\ub7ec\uc654\ub2e4. 1886\ub144 1\uc6d4 7\uc77c\uc5d0 \uc624\ube0c\ub294 \ud574\uad70 \uc7a5\uad00\uc5d0 \uc784\uba85\ub418\uc5c8\uc9c0\ub9cc, \uc774\uac83\uc740 \uc624\ube0c\uc758 \uc0ac\uc0c1\uc744 \uc778\uc815\ud55c \uac83\uc774\ub77c\uae30 \ubcf4\ub2e4\ub294 \uc815\uce58\uc801 \ud310\ub2e8\uc5d0 \uc758\ud55c \uac83\uc774\uc5c8\ub2e4. \ud574\uad70\uc7a5\uad00\uc774 \ub41c \uc624\ube0c\ub294 \uc7ac\ube60\ub974\uac8c \uadf8\uc758 \uc0ac\uc0c1\uc758 \uc2e4\ud604\ud558\ub824 \ud588\ub2e4. 1861\ub144 2\uc6d4\uc5d0\ub294 \uc258\ubd80\ub974\uc5d0\uc11c \uc9c0\ube0c\ub864\ud130 \ud574\ud611\uc744 \ub3cc\uc544\uc11c \ud234\ub871\uae4c\uc9c0 \uc5b4\ub8b0\uc815\uc744 \ud56d\ud574\uc2dc\ucf30\ub294\ub370, \uae38\uc774 33m \uc815\ub3c4\uc758 \uc5b4\ub8b0\uc815\uc73c\ub85c \uacf5\ud574\ub97c \ud56d\ud574\ud558\ub294 \uac83\uc740 \uc5b4\ub824\uc6e0\ub2e4. \uac19\uc740 \ud574 5\uc6d4\uacfc 6\uc6d4\uc5d0\ub294 8\ucc99\uc758 \uc804\ud568\uc73c\ub85c \uc774\ub8e8\uc5b4\uc9c4 \uacf5\uaca9\uce21\uc744 20\ucc99 \ub0a8\uc9d3\uc758 \uc5b4\ub8b0\uc815(\uc21c\uc591\ud568 3\ucc99\uacfc \ud574\ubc29\uc804\ud568 1\ucc99\uc774 \uc9c0\uc6d0)\uc73c\ub85c \uc800\uc9c0\ud558\ub294 \uc5f0\uc2b5\uc744 \ud588\ub2e4. \uacf5\uaca9\uce21\uc740 \ud234\ub871\uc744 \ud568\ud3ec \uc0ac\uaca9\ud558\ub294 \ub370\ub294 \uc131\uacf5\ud588\uc9c0\ub9cc, \ub300\ubd80\ubd84\uc774 \uaca9\uce68 \ud310\uc815\uc744 \ubc1b\uc558\ub2e4. \uc774\ub85c \uc778\ud574 \ub300\ud615\ud568\uc5d0 \uc758\ud55c \uadfc\uc811 \ubd09\uc1c4\uac00 \ubd88\uac00\ub2a5\ud558\ub2e4\ub294 \uac83\uc774 \uc785\uc99d\ub418\uc5c8\ub2e4. \uac19\uc740 \ud574 12\uc6d4, \uc624\ube0c\ub294 \ub2e4\uc74c\uacfc \uac19\uc774 \uc804\ub7b5\uc744 \ubcc0\uacbd\ud588\ub2e4.", "sentence_answer": "\uace7 \uccad\ub144\ud559\ud30c\ub294 \uc815\uce58\uc778\uacfc \uc5b8\ub860 \ub3c4 \ub9d0\ub824\ub4e4\uba70 \ub17c\ub780\uc744 \ubd88\ub7ec\uc654\ub2e4.", "paragraph_id": "6596841-1-0", "paragraph_question": "question: \uccad\ub144\ud559\ud30c\ub294 \uc815\uce58\uc778\uacfc \ub204\uac00 \ub9d0\ub824\ub4e4\uba70 \ub17c\ub780\uc744 \ubd88\ub7ec\uc654\ub098?, context: \uace7 \uccad\ub144\ud559\ud30c\ub294 \uc815\uce58\uc778\uacfc \uc5b8\ub860\ub3c4 \ub9d0\ub824\ub4e4\uba70 \ub17c\ub780\uc744 \ubd88\ub7ec\uc654\ub2e4. 1886\ub144 1\uc6d4 7\uc77c\uc5d0 \uc624\ube0c\ub294 \ud574\uad70 \uc7a5\uad00\uc5d0 \uc784\uba85\ub418\uc5c8\uc9c0\ub9cc, \uc774\uac83\uc740 \uc624\ube0c\uc758 \uc0ac\uc0c1\uc744 \uc778\uc815\ud55c \uac83\uc774\ub77c\uae30 \ubcf4\ub2e4\ub294 \uc815\uce58\uc801 \ud310\ub2e8\uc5d0 \uc758\ud55c \uac83\uc774\uc5c8\ub2e4. \ud574\uad70\uc7a5\uad00\uc774 \ub41c \uc624\ube0c\ub294 \uc7ac\ube60\ub974\uac8c \uadf8\uc758 \uc0ac\uc0c1\uc758 \uc2e4\ud604\ud558\ub824 \ud588\ub2e4. 1861\ub144 2\uc6d4\uc5d0\ub294 \uc258\ubd80\ub974\uc5d0\uc11c \uc9c0\ube0c\ub864\ud130 \ud574\ud611\uc744 \ub3cc\uc544\uc11c \ud234\ub871\uae4c\uc9c0 \uc5b4\ub8b0\uc815\uc744 \ud56d\ud574\uc2dc\ucf30\ub294\ub370, \uae38\uc774 33m \uc815\ub3c4\uc758 \uc5b4\ub8b0\uc815\uc73c\ub85c \uacf5\ud574\ub97c \ud56d\ud574\ud558\ub294 \uac83\uc740 \uc5b4\ub824\uc6e0\ub2e4. \uac19\uc740 \ud574 5\uc6d4\uacfc 6\uc6d4\uc5d0\ub294 8\ucc99\uc758 \uc804\ud568\uc73c\ub85c \uc774\ub8e8\uc5b4\uc9c4 \uacf5\uaca9\uce21\uc744 20\ucc99 \ub0a8\uc9d3\uc758 \uc5b4\ub8b0\uc815(\uc21c\uc591\ud568 3\ucc99\uacfc \ud574\ubc29\uc804\ud568 1\ucc99\uc774 \uc9c0\uc6d0)\uc73c\ub85c \uc800\uc9c0\ud558\ub294 \uc5f0\uc2b5\uc744 \ud588\ub2e4. \uacf5\uaca9\uce21\uc740 \ud234\ub871\uc744 \ud568\ud3ec \uc0ac\uaca9\ud558\ub294 \ub370\ub294 \uc131\uacf5\ud588\uc9c0\ub9cc, \ub300\ubd80\ubd84\uc774 \uaca9\uce68 \ud310\uc815\uc744 \ubc1b\uc558\ub2e4. \uc774\ub85c \uc778\ud574 \ub300\ud615\ud568\uc5d0 \uc758\ud55c \uadfc\uc811 \ubd09\uc1c4\uac00 \ubd88\uac00\ub2a5\ud558\ub2e4\ub294 \uac83\uc774 \uc785\uc99d\ub418\uc5c8\ub2e4. \uac19\uc740 \ud574 12\uc6d4, \uc624\ube0c\ub294 \ub2e4\uc74c\uacfc \uac19\uc774 \uc804\ub7b5\uc744 \ubcc0\uacbd\ud588\ub2e4."} {"question": "\ud55c\uc640 \uc120 \uc9c1\ud1b5 \ucf8c\uc18d\uc774 \uac01\uc5ed\uc5d0 \uc815\ucc28\ud558\ub294 \ub178\uc120\uc740?", "paragraph": "\uc9c1\ud1b5 \ucf8c\uc18d\uc740 2008\ub144 3\uc6d4 15\uc77c\uc758 \uc2dc\uac04\ud45c \uac1c\uc815\uc73c\ub85c \ud3c9\uc77c \uc544\uce68 \ucd9c\uadfc \uc2dc\uac04\ub300\uc5d0 \uc2e0\uc124\ub41c \uc624\uc0ac\uce74 \uc21c\ud658\uc120 \uc9c1\ud1b5 \uc5f4\ucc28\uc774\ub2e4. \uc815\ucc28\uc5ed\uc740 \ud55c\uc640 \uc120 \ub0b4\uc5d0\uc11c\ub294 \ucf8c\uc18d\uacfc \ub3d9\uc77c\ud558\uba70 \uc624\uc0ac\uce74 \uc21c\ud658\uc120 \uc548\uc5d0\uc11c\ub294 \uac01\uc5ed\uc5d0 \uc815\ucc28\ud55c\ub2e4. \ucc28\ub7c9\uc740 223\uacc4\ub97c \uc0ac\uc6a9\ud558\uace0 \uc788\ub2e4. \uc0c1\ud589\uc740 10\ud3b8\uc774 \uc624\uc0ac\uce74 \uc21c\ud658\uc120 \uc678\uc120\uc744 \uacbd\uc720\ud574 \uc624\uc0ac\uce74 \uc5ed\u00b7\uad50\ubc14\uc2dc \ubc29\uba74\uc73c\ub85c \uc6b4\uc804\ub41c\ub2e4. \uc624\uc0ac\uce74 \uc21c\ud658\uc120\uc744 \uc77c\uc8fc\ud558\ub294 \uc5f4\ucc28 \ub4f1\uacfc \uac19\uc774 \ub3c4\uc911\uc5ed\uc73c\ub85c\uc11c \uc77c\ub2e8 \ub374\ub178\uc9c0 \uc5ed\uc5d0 \uc815\ucc28\ud558\ub294 \uac83\uc5d0\uc11c \uc624\uc0ac\uce74 \ubc29\uba74\uc73c\ub85c \uc6b4\ud589\ud558\uc9c0 \uc54a\ub294\ub2e4\ub294 \uc624\ud574\ub97c \ud53c\ud558\uae30 \uc704\ud574 \uad50\ubc14\uc2dc \ud589\uc744 \uc81c\uc678\ud558\uace0 \uc624\uc0ac\uce74 \uc5ed\uae4c\uc9c0\ub294 \uc624\uc0ac\uce74 \uc21c\ud658\uc120\uc73c\ub85c \uc548\ub0b4\ud558\uace0 \uc788\ub2e4. \ud558\ud589\uc740 \uc624\uc0ac\uce74 \uc21c\ud658\uc120 \ub374\ub178\uc9c0 \uc5ed\uc5d0\uc11c \uad50\ubc14\uc2dc \uc5ed\uc744 \uacbd\uc720\ud574 \uc624\ud1a0\ub9ac \uc5ed\uc774\ub098 \uc640\uce74\uc57c\ub9c8 \uc5ed \ud589\uc774 \uac01\uac01 1\ud3b8\uc529 \uc6b4\ud589\ud558\uace0 \uc788\ub2e4.", "answer": "\uc624\uc0ac\uce74 \uc21c\ud658\uc120", "sentence": "\uc9c1\ud1b5 \ucf8c\uc18d\uc740 2008\ub144 3\uc6d4 15\uc77c\uc758 \uc2dc\uac04\ud45c \uac1c\uc815\uc73c\ub85c \ud3c9\uc77c \uc544\uce68 \ucd9c\uadfc \uc2dc\uac04\ub300\uc5d0 \uc2e0\uc124\ub41c \uc624\uc0ac\uce74 \uc21c\ud658\uc120 \uc9c1\ud1b5 \uc5f4\ucc28\uc774\ub2e4.", "paragraph_sentence": " \uc9c1\ud1b5 \ucf8c\uc18d\uc740 2008\ub144 3\uc6d4 15\uc77c\uc758 \uc2dc\uac04\ud45c \uac1c\uc815\uc73c\ub85c \ud3c9\uc77c \uc544\uce68 \ucd9c\uadfc \uc2dc\uac04\ub300\uc5d0 \uc2e0\uc124\ub41c \uc624\uc0ac\uce74 \uc21c\ud658\uc120 \uc9c1\ud1b5 \uc5f4\ucc28\uc774\ub2e4. \uc815\ucc28\uc5ed\uc740 \ud55c\uc640 \uc120 \ub0b4\uc5d0\uc11c\ub294 \ucf8c\uc18d\uacfc \ub3d9\uc77c\ud558\uba70 \uc624\uc0ac\uce74 \uc21c\ud658\uc120 \uc548\uc5d0\uc11c\ub294 \uac01\uc5ed\uc5d0 \uc815\ucc28\ud55c\ub2e4. \ucc28\ub7c9\uc740 223\uacc4\ub97c \uc0ac\uc6a9\ud558\uace0 \uc788\ub2e4. \uc0c1\ud589\uc740 10\ud3b8\uc774 \uc624\uc0ac\uce74 \uc21c\ud658\uc120 \uc678\uc120\uc744 \uacbd\uc720\ud574 \uc624\uc0ac\uce74 \uc5ed\u00b7\uad50\ubc14\uc2dc \ubc29\uba74\uc73c\ub85c \uc6b4\uc804\ub41c\ub2e4. \uc624\uc0ac\uce74 \uc21c\ud658\uc120\uc744 \uc77c\uc8fc\ud558\ub294 \uc5f4\ucc28 \ub4f1\uacfc \uac19\uc774 \ub3c4\uc911\uc5ed\uc73c\ub85c\uc11c \uc77c\ub2e8 \ub374\ub178\uc9c0 \uc5ed\uc5d0 \uc815\ucc28\ud558\ub294 \uac83\uc5d0\uc11c \uc624\uc0ac\uce74 \ubc29\uba74\uc73c\ub85c \uc6b4\ud589\ud558\uc9c0 \uc54a\ub294\ub2e4\ub294 \uc624\ud574\ub97c \ud53c\ud558\uae30 \uc704\ud574 \uad50\ubc14\uc2dc \ud589\uc744 \uc81c\uc678\ud558\uace0 \uc624\uc0ac\uce74 \uc5ed\uae4c\uc9c0\ub294 \uc624\uc0ac\uce74 \uc21c\ud658\uc120\uc73c\ub85c \uc548\ub0b4\ud558\uace0 \uc788\ub2e4. \ud558\ud589\uc740 \uc624\uc0ac\uce74 \uc21c\ud658\uc120 \ub374\ub178\uc9c0 \uc5ed\uc5d0\uc11c \uad50\ubc14\uc2dc \uc5ed\uc744 \uacbd\uc720\ud574 \uc624\ud1a0\ub9ac \uc5ed\uc774\ub098 \uc640\uce74\uc57c\ub9c8 \uc5ed \ud589\uc774 \uac01\uac01 1\ud3b8\uc529 \uc6b4\ud589\ud558\uace0 \uc788\ub2e4.", "paragraph_answer": "\uc9c1\ud1b5 \ucf8c\uc18d\uc740 2008\ub144 3\uc6d4 15\uc77c\uc758 \uc2dc\uac04\ud45c \uac1c\uc815\uc73c\ub85c \ud3c9\uc77c \uc544\uce68 \ucd9c\uadfc \uc2dc\uac04\ub300\uc5d0 \uc2e0\uc124\ub41c \uc624\uc0ac\uce74 \uc21c\ud658\uc120 \uc9c1\ud1b5 \uc5f4\ucc28\uc774\ub2e4. \uc815\ucc28\uc5ed\uc740 \ud55c\uc640 \uc120 \ub0b4\uc5d0\uc11c\ub294 \ucf8c\uc18d\uacfc \ub3d9\uc77c\ud558\uba70 \uc624\uc0ac\uce74 \uc21c\ud658\uc120 \uc548\uc5d0\uc11c\ub294 \uac01\uc5ed\uc5d0 \uc815\ucc28\ud55c\ub2e4. \ucc28\ub7c9\uc740 223\uacc4\ub97c \uc0ac\uc6a9\ud558\uace0 \uc788\ub2e4. \uc0c1\ud589\uc740 10\ud3b8\uc774 \uc624\uc0ac\uce74 \uc21c\ud658\uc120 \uc678\uc120\uc744 \uacbd\uc720\ud574 \uc624\uc0ac\uce74 \uc5ed\u00b7\uad50\ubc14\uc2dc \ubc29\uba74\uc73c\ub85c \uc6b4\uc804\ub41c\ub2e4. \uc624\uc0ac\uce74 \uc21c\ud658\uc120\uc744 \uc77c\uc8fc\ud558\ub294 \uc5f4\ucc28 \ub4f1\uacfc \uac19\uc774 \ub3c4\uc911\uc5ed\uc73c\ub85c\uc11c \uc77c\ub2e8 \ub374\ub178\uc9c0 \uc5ed\uc5d0 \uc815\ucc28\ud558\ub294 \uac83\uc5d0\uc11c \uc624\uc0ac\uce74 \ubc29\uba74\uc73c\ub85c \uc6b4\ud589\ud558\uc9c0 \uc54a\ub294\ub2e4\ub294 \uc624\ud574\ub97c \ud53c\ud558\uae30 \uc704\ud574 \uad50\ubc14\uc2dc \ud589\uc744 \uc81c\uc678\ud558\uace0 \uc624\uc0ac\uce74 \uc5ed\uae4c\uc9c0\ub294 \uc624\uc0ac\uce74 \uc21c\ud658\uc120\uc73c\ub85c \uc548\ub0b4\ud558\uace0 \uc788\ub2e4. \ud558\ud589\uc740 \uc624\uc0ac\uce74 \uc21c\ud658\uc120 \ub374\ub178\uc9c0 \uc5ed\uc5d0\uc11c \uad50\ubc14\uc2dc \uc5ed\uc744 \uacbd\uc720\ud574 \uc624\ud1a0\ub9ac \uc5ed\uc774\ub098 \uc640\uce74\uc57c\ub9c8 \uc5ed \ud589\uc774 \uac01\uac01 1\ud3b8\uc529 \uc6b4\ud589\ud558\uace0 \uc788\ub2e4.", "sentence_answer": "\uc9c1\ud1b5 \ucf8c\uc18d\uc740 2008\ub144 3\uc6d4 15\uc77c\uc758 \uc2dc\uac04\ud45c \uac1c\uc815\uc73c\ub85c \ud3c9\uc77c \uc544\uce68 \ucd9c\uadfc \uc2dc\uac04\ub300\uc5d0 \uc2e0\uc124\ub41c \uc624\uc0ac\uce74 \uc21c\ud658\uc120 \uc9c1\ud1b5 \uc5f4\ucc28\uc774\ub2e4.", "paragraph_id": "6570133-7-1", "paragraph_question": "question: \ud55c\uc640 \uc120 \uc9c1\ud1b5 \ucf8c\uc18d\uc774 \uac01\uc5ed\uc5d0 \uc815\ucc28\ud558\ub294 \ub178\uc120\uc740?, context: \uc9c1\ud1b5 \ucf8c\uc18d\uc740 2008\ub144 3\uc6d4 15\uc77c\uc758 \uc2dc\uac04\ud45c \uac1c\uc815\uc73c\ub85c \ud3c9\uc77c \uc544\uce68 \ucd9c\uadfc \uc2dc\uac04\ub300\uc5d0 \uc2e0\uc124\ub41c \uc624\uc0ac\uce74 \uc21c\ud658\uc120 \uc9c1\ud1b5 \uc5f4\ucc28\uc774\ub2e4. \uc815\ucc28\uc5ed\uc740 \ud55c\uc640 \uc120 \ub0b4\uc5d0\uc11c\ub294 \ucf8c\uc18d\uacfc \ub3d9\uc77c\ud558\uba70 \uc624\uc0ac\uce74 \uc21c\ud658\uc120 \uc548\uc5d0\uc11c\ub294 \uac01\uc5ed\uc5d0 \uc815\ucc28\ud55c\ub2e4. \ucc28\ub7c9\uc740 223\uacc4\ub97c \uc0ac\uc6a9\ud558\uace0 \uc788\ub2e4. \uc0c1\ud589\uc740 10\ud3b8\uc774 \uc624\uc0ac\uce74 \uc21c\ud658\uc120 \uc678\uc120\uc744 \uacbd\uc720\ud574 \uc624\uc0ac\uce74 \uc5ed\u00b7\uad50\ubc14\uc2dc \ubc29\uba74\uc73c\ub85c \uc6b4\uc804\ub41c\ub2e4. \uc624\uc0ac\uce74 \uc21c\ud658\uc120\uc744 \uc77c\uc8fc\ud558\ub294 \uc5f4\ucc28 \ub4f1\uacfc \uac19\uc774 \ub3c4\uc911\uc5ed\uc73c\ub85c\uc11c \uc77c\ub2e8 \ub374\ub178\uc9c0 \uc5ed\uc5d0 \uc815\ucc28\ud558\ub294 \uac83\uc5d0\uc11c \uc624\uc0ac\uce74 \ubc29\uba74\uc73c\ub85c \uc6b4\ud589\ud558\uc9c0 \uc54a\ub294\ub2e4\ub294 \uc624\ud574\ub97c \ud53c\ud558\uae30 \uc704\ud574 \uad50\ubc14\uc2dc \ud589\uc744 \uc81c\uc678\ud558\uace0 \uc624\uc0ac\uce74 \uc5ed\uae4c\uc9c0\ub294 \uc624\uc0ac\uce74 \uc21c\ud658\uc120\uc73c\ub85c \uc548\ub0b4\ud558\uace0 \uc788\ub2e4. \ud558\ud589\uc740 \uc624\uc0ac\uce74 \uc21c\ud658\uc120 \ub374\ub178\uc9c0 \uc5ed\uc5d0\uc11c \uad50\ubc14\uc2dc \uc5ed\uc744 \uacbd\uc720\ud574 \uc624\ud1a0\ub9ac \uc5ed\uc774\ub098 \uc640\uce74\uc57c\ub9c8 \uc5ed \ud589\uc774 \uac01\uac01 1\ud3b8\uc529 \uc6b4\ud589\ud558\uace0 \uc788\ub2e4."} {"question": "\uc544\uae38\ub808\ub77c\uac00 \ube0c\ub9ac\ud2b8\ub2c8 \uc2a4\ud53c\uc5b4\uc2a4\uc640\uc758 \ube44\uad50 \uc790\uccb4\uac00 \uc2a4\ud2b8\ub808\uc2a4\ub77c\uace0 \ubc1d\ud78c \ub9e4\uccb4\ub294?", "paragraph": "\uc2a4\ud53c\uc5b4\uc2a4\ub294 \ucf58\ud2b8\ub784\ud1a0\uc758 \ubcf4\uceec \ubc94\uc704\ub97c \uac00\uc9c0\uace0 \uc788\ub2e4. \uc2a4\ud53c\uc5b4\uc2a4\uc758 \uac00\ucc3d\ub825\uc740 \uc5b4\ub838\uc744\uc801 \ud31d \ub77c\uc774\ubc8c \ud06c\ub9ac\uc2a4\ud2f0\ub098 \uc544\uae38\ub808\ub77c\uc640 \uc885\uc885 \ube44\uad50\ub3fc \ube44\ud310\uc744 \ubc1b\uc544\uc654\ub294\ub370, \ud3c9\ub860\uac00 \uc568\ub7f0 \ub808\uc774\ube14\uc740 \uc2a4\ud53c\uc5b4\uc2a4\uc758 \ub300\ubd80\ubd84\uc758 \ub178\ub798\uac00 \ub514\uc9c0\ud138 \ud6a8\uacfc\uc640 \uae30\uacc4\uc74c\uc73c\ub85c \ub9cc\ub4e4\uc5b4\uc9c4\ub2e4\uba70 \ube44\ud310\ud588\ub2e4. \"\ube0c\ub9ac\ud2b8\ub2c8\ub294 \uac15\ud55c \ubcf4\uceec\ub9ac\uc2a4\ud2b8\ub85c \ub3cc\uc544\uac08 \uc218 \uc5c6\ub2e4\"\uba70 \"\uc2a4\ud53c\uc5b4\uc2a4\uac00 \uc637\uc744 \ubc97\uc5c8\ub2e4\uace0 \uc74c\ud5a5 \ud6a8\uacfc \uc5c6\uc774 \uc774 \ub178\ub798\ub97c \uc18c\ud654\ud560 \uc218 \uc788\uc744\uae4c? \ub354 \uc911\uc694\ud55c\uac74, \ub300\uc911\ub4e4\uc774 \uc2a4\ud53c\uc5b4\uc2a4\uc758 \ub77c\uc774\ube0c\ub97c \ub4e3\uace0 \uc2f6\uc5b4\ud558\ub294 \uac74\uac00? \ud558\uc9c0\ub9cc, \uc0ac\ub78c\ub4e4\uc740 \uc5ec\uc804\ud788 \ub208\uc73c\ub85c \ubcf4\ub294 \uc774\ubbf8\uc9c0\ub97c \uc88b\uc544\ud55c\ub2e4.\"\ub77c\uace0 \uc9d1\ud544\ud588\ub2e4. \uac00\ucc3d\ub825 \uc774\uc678\uc5d0 \uc774\ubbf8\uc9c0\ub3c4 \ud06c\ub9ac\uc2a4\ud2f0\ub098 \uc544\uae38\ub808\ub77c\uc640 \ube44\uad50\ub418\uc5b4 \uc654\ub2e4. \u300a\uc5d4\ud130\ud14c\uc778\uba3c\ud2b8 \uc704\ud074\ub9ac\u300b\uc758 \ub370\uc774\ube44\ub4dc \ube0c\ub77c\uc6b4\uc740 \"\ud06c\ub9ac\uc2a4\ud2f0\ub098 \uc544\uae38\ub808\ub77c\ub294 \uc790\uae30\uc758 \uc74c\uc545\uc744 \uc704\ud574 \ud53c\ubd80\ub97c \ud0dc\uc6b0\uace0 \ubc30\uaf3d\uc744 \ub9cc\ub4e4\uc5c8\ub294\ub370, \ub108\ubb34 \ubd88\ucf8c\ud574\ud558\uc9c0 \ub9c8\ub77c. \uadf8\ub140\ub294 \ub098\uc05c \ucc99\ud558\ub294 \ucc29\ud55c \uc5ec\uc790\ub2e4. \uadf8\ub7ec\ub098, \uc2a4\ud53c\uc5b4\uc2a4\ub294 \uc88b\uc740 \uc5ec\uc790\uc600\uc9c0\ub9cc, \ub098\uc05c \uc5ec\uc790\ub85c \ub2e4\uac00\uc624\uace0 \uc788\ub2e4. \uc2a4\ud53c\uc5b4\uc2a4\uc758 \uc778\uacf5\uc801\uc778 \ubcf4\uc774\uc2a4\ub294 \ub2f4\ubc31\ud560 \uc815\ub3c4\ub85c \ud765\ubbf8\ub86d\uc9c0\ub9cc, \uc2e4\uc81c\ub85c\ub294 \uc544\uae38\ub808\ub77c\uc5d0\uac8c \uc804\ud600 \ubabb \ubbf8\uce5c\ub2e4.\" Slant \uba54\uac70\uc9c4\uc5d0\uc11c\ub294 \ub2e4\uc74c\uacfc \uac19\uc774 \ub9d0\ud588\ub2e4. \"\ube0c\ub9ac\ud2b8\ub2c8\uc640 \uc544\uae38\ub808\ub77c \uadf8\ub4e4 \ubaa9\uc18c\ub9ac\uc758 \uc74c\uc0c9\uacfc \uc625\ud0c0\ube0c\ub294 \uc11c\ub85c \ub2ec\ub77c \uacb0\uc815\uc744 \ub0b4\ub9b4 \uc218 \uc5c6\ub2e4. \uc544\uae38\ub808\ub77c\ub294 \u300a\ud53c\uba54\uc77c \ud37c\uc2a4\ud2b8\u300b\uc640\uc758 \uc778\ud130\ubdf0\uc5d0\uc11c \"\ub370\ubdd4 \uc774\ud6c4 \uc904\uace7 \ube0c\ub9ac\ud2b8\ub2c8 \uc2a4\ud53c\uc5b4\uc2a4\uc640 \ube44\uad50\ub418\uc5c8\ub294\ub370, \ube44\uad50 \uc790\uccb4\ub97c \uac70\ubd80\ud558\ub294 \ub098\ub85c\uc11c \uadf8\uac83\uc774 \uc5bc\ub9c8\ub098 \ud070 \uc2a4\ud2b8\ub808\uc2a4\uc778\uc9c0 \uc544\ubb34\ub3c4 \ubaa8\ub97c \uac83\"\uc774\ub77c\uba70 \ube44\uad50\ub97c \uaebc\ub824\ud588\ub2e4.", "answer": "\ud53c\uba54\uc77c \ud37c\uc2a4\ud2b8", "sentence": "\uc544\uae38\ub808\ub77c\ub294 \u300a \ud53c\uba54\uc77c \ud37c\uc2a4\ud2b8 \u300b\uc640\uc758 \uc778\ud130\ubdf0\uc5d0\uc11c \"\ub370\ubdd4 \uc774\ud6c4 \uc904\uace7 \ube0c\ub9ac\ud2b8\ub2c8 \uc2a4\ud53c\uc5b4\uc2a4\uc640 \ube44\uad50\ub418\uc5c8\ub294\ub370, \ube44\uad50 \uc790\uccb4\ub97c \uac70\ubd80\ud558\ub294 \ub098\ub85c\uc11c \uadf8\uac83\uc774 \uc5bc\ub9c8\ub098 \ud070 \uc2a4\ud2b8\ub808\uc2a4\uc778\uc9c0 \uc544\ubb34\ub3c4 \ubaa8\ub97c \uac83\"\uc774\ub77c\uba70 \ube44\uad50\ub97c \uaebc\ub824\ud588\ub2e4.", "paragraph_sentence": "\uc2a4\ud53c\uc5b4\uc2a4\ub294 \ucf58\ud2b8\ub784\ud1a0\uc758 \ubcf4\uceec \ubc94\uc704\ub97c \uac00\uc9c0\uace0 \uc788\ub2e4. \uc2a4\ud53c\uc5b4\uc2a4\uc758 \uac00\ucc3d\ub825\uc740 \uc5b4\ub838\uc744\uc801 \ud31d \ub77c\uc774\ubc8c \ud06c\ub9ac\uc2a4\ud2f0\ub098 \uc544\uae38\ub808\ub77c\uc640 \uc885\uc885 \ube44\uad50\ub3fc \ube44\ud310\uc744 \ubc1b\uc544\uc654\ub294\ub370, \ud3c9\ub860\uac00 \uc568\ub7f0 \ub808\uc774\ube14\uc740 \uc2a4\ud53c\uc5b4\uc2a4\uc758 \ub300\ubd80\ubd84\uc758 \ub178\ub798\uac00 \ub514\uc9c0\ud138 \ud6a8\uacfc\uc640 \uae30\uacc4\uc74c\uc73c\ub85c \ub9cc\ub4e4\uc5b4\uc9c4\ub2e4\uba70 \ube44\ud310\ud588\ub2e4. \"\ube0c\ub9ac\ud2b8\ub2c8\ub294 \uac15\ud55c \ubcf4\uceec\ub9ac\uc2a4\ud2b8\ub85c \ub3cc\uc544\uac08 \uc218 \uc5c6\ub2e4\"\uba70 \"\uc2a4\ud53c\uc5b4\uc2a4\uac00 \uc637\uc744 \ubc97\uc5c8\ub2e4\uace0 \uc74c\ud5a5 \ud6a8\uacfc \uc5c6\uc774 \uc774 \ub178\ub798\ub97c \uc18c\ud654\ud560 \uc218 \uc788\uc744\uae4c? \ub354 \uc911\uc694\ud55c\uac74, \ub300\uc911\ub4e4\uc774 \uc2a4\ud53c\uc5b4\uc2a4\uc758 \ub77c\uc774\ube0c\ub97c \ub4e3\uace0 \uc2f6\uc5b4\ud558\ub294 \uac74\uac00? \ud558\uc9c0\ub9cc, \uc0ac\ub78c\ub4e4\uc740 \uc5ec\uc804\ud788 \ub208\uc73c\ub85c \ubcf4\ub294 \uc774\ubbf8\uc9c0\ub97c \uc88b\uc544\ud55c\ub2e4. \"\ub77c\uace0 \uc9d1\ud544\ud588\ub2e4. \uac00\ucc3d\ub825 \uc774\uc678\uc5d0 \uc774\ubbf8\uc9c0\ub3c4 \ud06c\ub9ac\uc2a4\ud2f0\ub098 \uc544\uae38\ub808\ub77c\uc640 \ube44\uad50\ub418\uc5b4 \uc654\ub2e4. \u300a\uc5d4\ud130\ud14c\uc778\uba3c\ud2b8 \uc704\ud074\ub9ac\u300b\uc758 \ub370\uc774\ube44\ub4dc \ube0c\ub77c\uc6b4\uc740 \"\ud06c\ub9ac\uc2a4\ud2f0\ub098 \uc544\uae38\ub808\ub77c\ub294 \uc790\uae30\uc758 \uc74c\uc545\uc744 \uc704\ud574 \ud53c\ubd80\ub97c \ud0dc\uc6b0\uace0 \ubc30\uaf3d\uc744 \ub9cc\ub4e4\uc5c8\ub294\ub370, \ub108\ubb34 \ubd88\ucf8c\ud574\ud558\uc9c0 \ub9c8\ub77c. \uadf8\ub140\ub294 \ub098\uc05c \ucc99\ud558\ub294 \ucc29\ud55c \uc5ec\uc790\ub2e4. \uadf8\ub7ec\ub098, \uc2a4\ud53c\uc5b4\uc2a4\ub294 \uc88b\uc740 \uc5ec\uc790\uc600\uc9c0\ub9cc, \ub098\uc05c \uc5ec\uc790\ub85c \ub2e4\uac00\uc624\uace0 \uc788\ub2e4. \uc2a4\ud53c\uc5b4\uc2a4\uc758 \uc778\uacf5\uc801\uc778 \ubcf4\uc774\uc2a4\ub294 \ub2f4\ubc31\ud560 \uc815\ub3c4\ub85c \ud765\ubbf8\ub86d\uc9c0\ub9cc, \uc2e4\uc81c\ub85c\ub294 \uc544\uae38\ub808\ub77c\uc5d0\uac8c \uc804\ud600 \ubabb \ubbf8\uce5c\ub2e4.\" Slant \uba54\uac70\uc9c4\uc5d0\uc11c\ub294 \ub2e4\uc74c\uacfc \uac19\uc774 \ub9d0\ud588\ub2e4. \"\ube0c\ub9ac\ud2b8\ub2c8\uc640 \uc544\uae38\ub808\ub77c \uadf8\ub4e4 \ubaa9\uc18c\ub9ac\uc758 \uc74c\uc0c9\uacfc \uc625\ud0c0\ube0c\ub294 \uc11c\ub85c \ub2ec\ub77c \uacb0\uc815\uc744 \ub0b4\ub9b4 \uc218 \uc5c6\ub2e4. \uc544\uae38\ub808\ub77c\ub294 \u300a \ud53c\uba54\uc77c \ud37c\uc2a4\ud2b8 \u300b\uc640\uc758 \uc778\ud130\ubdf0\uc5d0\uc11c \"\ub370\ubdd4 \uc774\ud6c4 \uc904\uace7 \ube0c\ub9ac\ud2b8\ub2c8 \uc2a4\ud53c\uc5b4\uc2a4\uc640 \ube44\uad50\ub418\uc5c8\ub294\ub370, \ube44\uad50 \uc790\uccb4\ub97c \uac70\ubd80\ud558\ub294 \ub098\ub85c\uc11c \uadf8\uac83\uc774 \uc5bc\ub9c8\ub098 \ud070 \uc2a4\ud2b8\ub808\uc2a4\uc778\uc9c0 \uc544\ubb34\ub3c4 \ubaa8\ub97c \uac83\"\uc774\ub77c\uba70 \ube44\uad50\ub97c \uaebc\ub824\ud588\ub2e4. ", "paragraph_answer": "\uc2a4\ud53c\uc5b4\uc2a4\ub294 \ucf58\ud2b8\ub784\ud1a0\uc758 \ubcf4\uceec \ubc94\uc704\ub97c \uac00\uc9c0\uace0 \uc788\ub2e4. \uc2a4\ud53c\uc5b4\uc2a4\uc758 \uac00\ucc3d\ub825\uc740 \uc5b4\ub838\uc744\uc801 \ud31d \ub77c\uc774\ubc8c \ud06c\ub9ac\uc2a4\ud2f0\ub098 \uc544\uae38\ub808\ub77c\uc640 \uc885\uc885 \ube44\uad50\ub3fc \ube44\ud310\uc744 \ubc1b\uc544\uc654\ub294\ub370, \ud3c9\ub860\uac00 \uc568\ub7f0 \ub808\uc774\ube14\uc740 \uc2a4\ud53c\uc5b4\uc2a4\uc758 \ub300\ubd80\ubd84\uc758 \ub178\ub798\uac00 \ub514\uc9c0\ud138 \ud6a8\uacfc\uc640 \uae30\uacc4\uc74c\uc73c\ub85c \ub9cc\ub4e4\uc5b4\uc9c4\ub2e4\uba70 \ube44\ud310\ud588\ub2e4. \"\ube0c\ub9ac\ud2b8\ub2c8\ub294 \uac15\ud55c \ubcf4\uceec\ub9ac\uc2a4\ud2b8\ub85c \ub3cc\uc544\uac08 \uc218 \uc5c6\ub2e4\"\uba70 \"\uc2a4\ud53c\uc5b4\uc2a4\uac00 \uc637\uc744 \ubc97\uc5c8\ub2e4\uace0 \uc74c\ud5a5 \ud6a8\uacfc \uc5c6\uc774 \uc774 \ub178\ub798\ub97c \uc18c\ud654\ud560 \uc218 \uc788\uc744\uae4c? \ub354 \uc911\uc694\ud55c\uac74, \ub300\uc911\ub4e4\uc774 \uc2a4\ud53c\uc5b4\uc2a4\uc758 \ub77c\uc774\ube0c\ub97c \ub4e3\uace0 \uc2f6\uc5b4\ud558\ub294 \uac74\uac00? \ud558\uc9c0\ub9cc, \uc0ac\ub78c\ub4e4\uc740 \uc5ec\uc804\ud788 \ub208\uc73c\ub85c \ubcf4\ub294 \uc774\ubbf8\uc9c0\ub97c \uc88b\uc544\ud55c\ub2e4.\"\ub77c\uace0 \uc9d1\ud544\ud588\ub2e4. \uac00\ucc3d\ub825 \uc774\uc678\uc5d0 \uc774\ubbf8\uc9c0\ub3c4 \ud06c\ub9ac\uc2a4\ud2f0\ub098 \uc544\uae38\ub808\ub77c\uc640 \ube44\uad50\ub418\uc5b4 \uc654\ub2e4. \u300a\uc5d4\ud130\ud14c\uc778\uba3c\ud2b8 \uc704\ud074\ub9ac\u300b\uc758 \ub370\uc774\ube44\ub4dc \ube0c\ub77c\uc6b4\uc740 \"\ud06c\ub9ac\uc2a4\ud2f0\ub098 \uc544\uae38\ub808\ub77c\ub294 \uc790\uae30\uc758 \uc74c\uc545\uc744 \uc704\ud574 \ud53c\ubd80\ub97c \ud0dc\uc6b0\uace0 \ubc30\uaf3d\uc744 \ub9cc\ub4e4\uc5c8\ub294\ub370, \ub108\ubb34 \ubd88\ucf8c\ud574\ud558\uc9c0 \ub9c8\ub77c. \uadf8\ub140\ub294 \ub098\uc05c \ucc99\ud558\ub294 \ucc29\ud55c \uc5ec\uc790\ub2e4. \uadf8\ub7ec\ub098, \uc2a4\ud53c\uc5b4\uc2a4\ub294 \uc88b\uc740 \uc5ec\uc790\uc600\uc9c0\ub9cc, \ub098\uc05c \uc5ec\uc790\ub85c \ub2e4\uac00\uc624\uace0 \uc788\ub2e4. \uc2a4\ud53c\uc5b4\uc2a4\uc758 \uc778\uacf5\uc801\uc778 \ubcf4\uc774\uc2a4\ub294 \ub2f4\ubc31\ud560 \uc815\ub3c4\ub85c \ud765\ubbf8\ub86d\uc9c0\ub9cc, \uc2e4\uc81c\ub85c\ub294 \uc544\uae38\ub808\ub77c\uc5d0\uac8c \uc804\ud600 \ubabb \ubbf8\uce5c\ub2e4.\" Slant \uba54\uac70\uc9c4\uc5d0\uc11c\ub294 \ub2e4\uc74c\uacfc \uac19\uc774 \ub9d0\ud588\ub2e4. \"\ube0c\ub9ac\ud2b8\ub2c8\uc640 \uc544\uae38\ub808\ub77c \uadf8\ub4e4 \ubaa9\uc18c\ub9ac\uc758 \uc74c\uc0c9\uacfc \uc625\ud0c0\ube0c\ub294 \uc11c\ub85c \ub2ec\ub77c \uacb0\uc815\uc744 \ub0b4\ub9b4 \uc218 \uc5c6\ub2e4. \uc544\uae38\ub808\ub77c\ub294 \u300a \ud53c\uba54\uc77c \ud37c\uc2a4\ud2b8 \u300b\uc640\uc758 \uc778\ud130\ubdf0\uc5d0\uc11c \"\ub370\ubdd4 \uc774\ud6c4 \uc904\uace7 \ube0c\ub9ac\ud2b8\ub2c8 \uc2a4\ud53c\uc5b4\uc2a4\uc640 \ube44\uad50\ub418\uc5c8\ub294\ub370, \ube44\uad50 \uc790\uccb4\ub97c \uac70\ubd80\ud558\ub294 \ub098\ub85c\uc11c \uadf8\uac83\uc774 \uc5bc\ub9c8\ub098 \ud070 \uc2a4\ud2b8\ub808\uc2a4\uc778\uc9c0 \uc544\ubb34\ub3c4 \ubaa8\ub97c \uac83\"\uc774\ub77c\uba70 \ube44\uad50\ub97c \uaebc\ub824\ud588\ub2e4.", "sentence_answer": "\uc544\uae38\ub808\ub77c\ub294 \u300a \ud53c\uba54\uc77c \ud37c\uc2a4\ud2b8 \u300b\uc640\uc758 \uc778\ud130\ubdf0\uc5d0\uc11c \"\ub370\ubdd4 \uc774\ud6c4 \uc904\uace7 \ube0c\ub9ac\ud2b8\ub2c8 \uc2a4\ud53c\uc5b4\uc2a4\uc640 \ube44\uad50\ub418\uc5c8\ub294\ub370, \ube44\uad50 \uc790\uccb4\ub97c \uac70\ubd80\ud558\ub294 \ub098\ub85c\uc11c \uadf8\uac83\uc774 \uc5bc\ub9c8\ub098 \ud070 \uc2a4\ud2b8\ub808\uc2a4\uc778\uc9c0 \uc544\ubb34\ub3c4 \ubaa8\ub97c \uac83\"\uc774\ub77c\uba70 \ube44\uad50\ub97c \uaebc\ub824\ud588\ub2e4.", "paragraph_id": "6509466-26-2", "paragraph_question": "question: \uc544\uae38\ub808\ub77c\uac00 \ube0c\ub9ac\ud2b8\ub2c8 \uc2a4\ud53c\uc5b4\uc2a4\uc640\uc758 \ube44\uad50 \uc790\uccb4\uac00 \uc2a4\ud2b8\ub808\uc2a4\ub77c\uace0 \ubc1d\ud78c \ub9e4\uccb4\ub294?, context: \uc2a4\ud53c\uc5b4\uc2a4\ub294 \ucf58\ud2b8\ub784\ud1a0\uc758 \ubcf4\uceec \ubc94\uc704\ub97c \uac00\uc9c0\uace0 \uc788\ub2e4. \uc2a4\ud53c\uc5b4\uc2a4\uc758 \uac00\ucc3d\ub825\uc740 \uc5b4\ub838\uc744\uc801 \ud31d \ub77c\uc774\ubc8c \ud06c\ub9ac\uc2a4\ud2f0\ub098 \uc544\uae38\ub808\ub77c\uc640 \uc885\uc885 \ube44\uad50\ub3fc \ube44\ud310\uc744 \ubc1b\uc544\uc654\ub294\ub370, \ud3c9\ub860\uac00 \uc568\ub7f0 \ub808\uc774\ube14\uc740 \uc2a4\ud53c\uc5b4\uc2a4\uc758 \ub300\ubd80\ubd84\uc758 \ub178\ub798\uac00 \ub514\uc9c0\ud138 \ud6a8\uacfc\uc640 \uae30\uacc4\uc74c\uc73c\ub85c \ub9cc\ub4e4\uc5b4\uc9c4\ub2e4\uba70 \ube44\ud310\ud588\ub2e4. \"\ube0c\ub9ac\ud2b8\ub2c8\ub294 \uac15\ud55c \ubcf4\uceec\ub9ac\uc2a4\ud2b8\ub85c \ub3cc\uc544\uac08 \uc218 \uc5c6\ub2e4\"\uba70 \"\uc2a4\ud53c\uc5b4\uc2a4\uac00 \uc637\uc744 \ubc97\uc5c8\ub2e4\uace0 \uc74c\ud5a5 \ud6a8\uacfc \uc5c6\uc774 \uc774 \ub178\ub798\ub97c \uc18c\ud654\ud560 \uc218 \uc788\uc744\uae4c? \ub354 \uc911\uc694\ud55c\uac74, \ub300\uc911\ub4e4\uc774 \uc2a4\ud53c\uc5b4\uc2a4\uc758 \ub77c\uc774\ube0c\ub97c \ub4e3\uace0 \uc2f6\uc5b4\ud558\ub294 \uac74\uac00? \ud558\uc9c0\ub9cc, \uc0ac\ub78c\ub4e4\uc740 \uc5ec\uc804\ud788 \ub208\uc73c\ub85c \ubcf4\ub294 \uc774\ubbf8\uc9c0\ub97c \uc88b\uc544\ud55c\ub2e4.\"\ub77c\uace0 \uc9d1\ud544\ud588\ub2e4. \uac00\ucc3d\ub825 \uc774\uc678\uc5d0 \uc774\ubbf8\uc9c0\ub3c4 \ud06c\ub9ac\uc2a4\ud2f0\ub098 \uc544\uae38\ub808\ub77c\uc640 \ube44\uad50\ub418\uc5b4 \uc654\ub2e4. \u300a\uc5d4\ud130\ud14c\uc778\uba3c\ud2b8 \uc704\ud074\ub9ac\u300b\uc758 \ub370\uc774\ube44\ub4dc \ube0c\ub77c\uc6b4\uc740 \"\ud06c\ub9ac\uc2a4\ud2f0\ub098 \uc544\uae38\ub808\ub77c\ub294 \uc790\uae30\uc758 \uc74c\uc545\uc744 \uc704\ud574 \ud53c\ubd80\ub97c \ud0dc\uc6b0\uace0 \ubc30\uaf3d\uc744 \ub9cc\ub4e4\uc5c8\ub294\ub370, \ub108\ubb34 \ubd88\ucf8c\ud574\ud558\uc9c0 \ub9c8\ub77c. \uadf8\ub140\ub294 \ub098\uc05c \ucc99\ud558\ub294 \ucc29\ud55c \uc5ec\uc790\ub2e4. \uadf8\ub7ec\ub098, \uc2a4\ud53c\uc5b4\uc2a4\ub294 \uc88b\uc740 \uc5ec\uc790\uc600\uc9c0\ub9cc, \ub098\uc05c \uc5ec\uc790\ub85c \ub2e4\uac00\uc624\uace0 \uc788\ub2e4. \uc2a4\ud53c\uc5b4\uc2a4\uc758 \uc778\uacf5\uc801\uc778 \ubcf4\uc774\uc2a4\ub294 \ub2f4\ubc31\ud560 \uc815\ub3c4\ub85c \ud765\ubbf8\ub86d\uc9c0\ub9cc, \uc2e4\uc81c\ub85c\ub294 \uc544\uae38\ub808\ub77c\uc5d0\uac8c \uc804\ud600 \ubabb \ubbf8\uce5c\ub2e4.\" Slant \uba54\uac70\uc9c4\uc5d0\uc11c\ub294 \ub2e4\uc74c\uacfc \uac19\uc774 \ub9d0\ud588\ub2e4. \"\ube0c\ub9ac\ud2b8\ub2c8\uc640 \uc544\uae38\ub808\ub77c \uadf8\ub4e4 \ubaa9\uc18c\ub9ac\uc758 \uc74c\uc0c9\uacfc \uc625\ud0c0\ube0c\ub294 \uc11c\ub85c \ub2ec\ub77c \uacb0\uc815\uc744 \ub0b4\ub9b4 \uc218 \uc5c6\ub2e4. \uc544\uae38\ub808\ub77c\ub294 \u300a\ud53c\uba54\uc77c \ud37c\uc2a4\ud2b8\u300b\uc640\uc758 \uc778\ud130\ubdf0\uc5d0\uc11c \"\ub370\ubdd4 \uc774\ud6c4 \uc904\uace7 \ube0c\ub9ac\ud2b8\ub2c8 \uc2a4\ud53c\uc5b4\uc2a4\uc640 \ube44\uad50\ub418\uc5c8\ub294\ub370, \ube44\uad50 \uc790\uccb4\ub97c \uac70\ubd80\ud558\ub294 \ub098\ub85c\uc11c \uadf8\uac83\uc774 \uc5bc\ub9c8\ub098 \ud070 \uc2a4\ud2b8\ub808\uc2a4\uc778\uc9c0 \uc544\ubb34\ub3c4 \ubaa8\ub97c \uac83\"\uc774\ub77c\uba70 \ube44\uad50\ub97c \uaebc\ub824\ud588\ub2e4."} {"question": "\uc18c\ub140\uc2dc\ub300 \ud65c\ub3d9 \ub2f9\uc2dc \uc8fc\ud55c\uc77c\ubcf8\uacf5\uc0ac\uc758 \uc774\ub984\uc740?", "paragraph": "\uc18c\ub140\uc2dc\ub300\ub294 \ud55c\ub958\ub97c \uc774\ub044\ub294 \uc8fc\uc5ed\uc73c\ub85c\ub3c4 \uc778\uc815 \ubc1b\uace0 \uc788\ub2e4. \uc8fc\ud55c\uc77c\ubcf8\uacf5\uc0ac \uc2a4\uc988\ud0a4 \ud788\ub85c\uc2dc\ub294 \uc774\ub4e4\uc744 \uce74\ub77c\uc640 \ud568\uaed8 \uc77c\ubcf8\uc758 \uc2e0\ud55c\ub958\ub97c \uc774\ub044\ub294 \uadf8\ub8f9\uc774\ub77c\uace0 \ub9d0\ud588\uc73c\uba70, \u300a\uacbd\ud5a5\uc2e0\ubb38\u300b\uc5d0\uc11c\ub294 \"\uc18c\ub140\uc2dc\ub300\ub294 \ub300\uc911\ubb38\ud654\uc758 \uc544\uc774\ucf58, \uc544\uc2dc\uc544\ub97c \ub118\uc5b4 \uc720\ub7fd\uae4c\uc9c0 \uc9c4\ucd9c\ud55c \ud55c\ub958\uc758 \uc120\ub450\uc8fc\uc790\uac00 \ub410\ub2e4\"\ub77c\uba70 \"\ud55c\ub958 \uc5f4\ud48d \uc774\ub044\ub294 \uc544\uc2dc\uc544 \ucd5c\uace0 \uac78\uadf8\ub8f9\"\uc774\ub77c\uace0 \ub9d0\ud588\ub2e4. \ube0c\ub77c\uc9c8\uc758 \uc5f0\uc608 \ud504\ub85c\uadf8\ub7a8\uc778 \u2018Leitura Dinamica\u2019\ub294 \ube0c\ub77c\uc9c8\uc758 k-pop \uc5f4\ud48d\uc744 \uc18c\uac1c\ud558\uba70 \uc18c\ub140\uc2dc\ub300\ub97c \uc608\ub85c \ub4e4\uba70, \u201c\uc2e0\uace1 \u2018\ud6d7\u2019\uc740 \ube0c\ub77c\uc9c8 \ubfd0 \uc544\ub2c8\ub77c \uc138\uacc4 \uc720\uba85 DJ\ub4e4\uc5d0\uac8c\ub3c4 \ud070 \uc778\uae30\ub2e4. \uadf8\uc57c\ub9d0\ub85c \uc18c\ub140\uc2dc\ub300\uac00 \uc3dc \ud654\uc0b4\uc774 \uc544\uc2dc\uc544\ub97c \ub118\uc5b4 \uc138\uacc4\ub85c \ub0a0\uc544\uac00\uace0 \uc788\ub294 \uc148\u201d\uc774\ub77c\uace0 \ud638\ud3c9\ud588\ub2e4. \ub610 \uc544\ub9ac\ub791TV\uac00 \uc5f0 \u2018K POP - \ub530\ub77c\ud558\uae30\u2019\uc5d0\uc11c \uac00\uc7a5 \ub9ce\uc774 \ud328\ub7ec\ub514\ub41c \uac00\uc218\ub294 \uc18c\ub140\uc2dc\ub300\uc600\uc73c\uba70, \uc778\ub3c4\ub124\uc2dc\uc544 \ud32c\ub4e4\uc740 \uc18c\ub140\uc2dc\ub300\uc758 \ub77c\uc774\ube0c \uacf5\uc5f0\uc744 \uc790\uad6d\uc5d0\uc11c \ud574 \ub2ec\ub77c\uace0 \uc11c\uba85 \uc6b4\ub3d9\uc744 \ubc8c\uc774\uae30\ub3c4 \ud588\ub2e4. \uc18c\uc2dc\uc9c0\ub8e9, \ub9c8\ub9b0\ub8e9 \ub4f1 \ud328\uc158\uc740 \ub300\ud55c\ubbfc\uad6d\uc5d0\uc11c \uc778\uae30\ub97c \ub04c\uc5c8\uc73c\uba70, \ud2b9\ud788 \ud6c4\ub8e8\uc57c \ub9c8\uc0ac\uc720\ud0a4\ub294 \uadf8\ub4e4\uc774 \uc77c\ubcf8 \uc80a\uc740 \uc5ec\uc131\ub4e4\uc758 \ud328\uc158 \uc544\uc774\ucf58\uc774\ub77c\uace0 \ub9d0\ud588\ub2e4. \ub610\ud55c \uc77c\ubcf8 \uc601\uc5b4 \uad50\uacfc\uc11c\uc5d0 \u201c\uc804 \uc138\uacc4\uc5d0\uc11c \uc778\uae30\ub97c \uc5bb\uace0 \uc788\ub294 K-pop \uc2a4\ud0c0\ub4e4\u201d\uc774\ub77c\ub294 \uc81c\ubaa9\uc73c\ub85c K-pop\uc758 \ub300\ud45c\ub85c \uc18c\ub140\uc2dc\ub300\ub97c \uc18c\uac1c\ud558\uba70 \uc0ac\uc9c4\uc774 \uc0bd\uc785\ub418\uc5c8\ub2e4. KIIS\uc758 \uc74c\uc545 \uac10\ub3c5\uc778 \uc904\ub9ac \ud544\ub78f (Julie Pilat)\uc740 2012\ub144 \uc8fc\ubaa9\ud560 \uadf8\ub8f9\uc73c\ub85c \ubf51\uae30\ub3c4 \ud588\ub2e4.", "answer": "\uc2a4\uc988\ud0a4 \ud788\ub85c\uc2dc", "sentence": "\uc8fc\ud55c\uc77c\ubcf8\uacf5\uc0ac \uc2a4\uc988\ud0a4 \ud788\ub85c\uc2dc \ub294 \uc774\ub4e4\uc744 \uce74\ub77c\uc640 \ud568\uaed8 \uc77c\ubcf8\uc758 \uc2e0\ud55c\ub958\ub97c \uc774\ub044\ub294 \uadf8\ub8f9\uc774\ub77c\uace0 \ub9d0\ud588\uc73c\uba70, \u300a\uacbd\ud5a5\uc2e0\ubb38\u300b\uc5d0\uc11c\ub294 \"\uc18c\ub140\uc2dc\ub300\ub294 \ub300\uc911\ubb38\ud654\uc758 \uc544\uc774\ucf58, \uc544\uc2dc\uc544\ub97c \ub118\uc5b4 \uc720\ub7fd\uae4c\uc9c0 \uc9c4\ucd9c\ud55c \ud55c\ub958\uc758 \uc120\ub450\uc8fc\uc790\uac00 \ub410\ub2e4\"\ub77c\uba70 \"\ud55c\ub958 \uc5f4\ud48d \uc774\ub044\ub294 \uc544\uc2dc\uc544 \ucd5c\uace0 \uac78\uadf8\ub8f9\"\uc774\ub77c\uace0 \ub9d0\ud588\ub2e4.", "paragraph_sentence": "\uc18c\ub140\uc2dc\ub300\ub294 \ud55c\ub958\ub97c \uc774\ub044\ub294 \uc8fc\uc5ed\uc73c\ub85c\ub3c4 \uc778\uc815 \ubc1b\uace0 \uc788\ub2e4. \uc8fc\ud55c\uc77c\ubcf8\uacf5\uc0ac \uc2a4\uc988\ud0a4 \ud788\ub85c\uc2dc \ub294 \uc774\ub4e4\uc744 \uce74\ub77c\uc640 \ud568\uaed8 \uc77c\ubcf8\uc758 \uc2e0\ud55c\ub958\ub97c \uc774\ub044\ub294 \uadf8\ub8f9\uc774\ub77c\uace0 \ub9d0\ud588\uc73c\uba70, \u300a\uacbd\ud5a5\uc2e0\ubb38\u300b\uc5d0\uc11c\ub294 \"\uc18c\ub140\uc2dc\ub300\ub294 \ub300\uc911\ubb38\ud654\uc758 \uc544\uc774\ucf58, \uc544\uc2dc\uc544\ub97c \ub118\uc5b4 \uc720\ub7fd\uae4c\uc9c0 \uc9c4\ucd9c\ud55c \ud55c\ub958\uc758 \uc120\ub450\uc8fc\uc790\uac00 \ub410\ub2e4\"\ub77c\uba70 \"\ud55c\ub958 \uc5f4\ud48d \uc774\ub044\ub294 \uc544\uc2dc\uc544 \ucd5c\uace0 \uac78\uadf8\ub8f9\"\uc774\ub77c\uace0 \ub9d0\ud588\ub2e4. \ube0c\ub77c\uc9c8\uc758 \uc5f0\uc608 \ud504\ub85c\uadf8\ub7a8\uc778 \u2018Leitura Dinamica\u2019\ub294 \ube0c\ub77c\uc9c8\uc758 k-pop \uc5f4\ud48d\uc744 \uc18c\uac1c\ud558\uba70 \uc18c\ub140\uc2dc\ub300\ub97c \uc608\ub85c \ub4e4\uba70, \u201c\uc2e0\uace1 \u2018\ud6d7\u2019\uc740 \ube0c\ub77c\uc9c8 \ubfd0 \uc544\ub2c8\ub77c \uc138\uacc4 \uc720\uba85 DJ\ub4e4\uc5d0\uac8c\ub3c4 \ud070 \uc778\uae30\ub2e4. \uadf8\uc57c\ub9d0\ub85c \uc18c\ub140\uc2dc\ub300\uac00 \uc3dc \ud654\uc0b4\uc774 \uc544\uc2dc\uc544\ub97c \ub118\uc5b4 \uc138\uacc4\ub85c \ub0a0\uc544\uac00\uace0 \uc788\ub294 \uc148\u201d\uc774\ub77c\uace0 \ud638\ud3c9\ud588\ub2e4. \ub610 \uc544\ub9ac\ub791TV\uac00 \uc5f0 \u2018K POP - \ub530\ub77c\ud558\uae30\u2019\uc5d0\uc11c \uac00\uc7a5 \ub9ce\uc774 \ud328\ub7ec\ub514\ub41c \uac00\uc218\ub294 \uc18c\ub140\uc2dc\ub300\uc600\uc73c\uba70, \uc778\ub3c4\ub124\uc2dc\uc544 \ud32c\ub4e4\uc740 \uc18c\ub140\uc2dc\ub300\uc758 \ub77c\uc774\ube0c \uacf5\uc5f0\uc744 \uc790\uad6d\uc5d0\uc11c \ud574 \ub2ec\ub77c\uace0 \uc11c\uba85 \uc6b4\ub3d9\uc744 \ubc8c\uc774\uae30\ub3c4 \ud588\ub2e4. \uc18c\uc2dc\uc9c0\ub8e9, \ub9c8\ub9b0\ub8e9 \ub4f1 \ud328\uc158\uc740 \ub300\ud55c\ubbfc\uad6d\uc5d0\uc11c \uc778\uae30\ub97c \ub04c\uc5c8\uc73c\uba70, \ud2b9\ud788 \ud6c4\ub8e8\uc57c \ub9c8\uc0ac\uc720\ud0a4\ub294 \uadf8\ub4e4\uc774 \uc77c\ubcf8 \uc80a\uc740 \uc5ec\uc131\ub4e4\uc758 \ud328\uc158 \uc544\uc774\ucf58\uc774\ub77c\uace0 \ub9d0\ud588\ub2e4. \ub610\ud55c \uc77c\ubcf8 \uc601\uc5b4 \uad50\uacfc\uc11c\uc5d0 \u201c\uc804 \uc138\uacc4\uc5d0\uc11c \uc778\uae30\ub97c \uc5bb\uace0 \uc788\ub294 K-pop \uc2a4\ud0c0\ub4e4\u201d\uc774\ub77c\ub294 \uc81c\ubaa9\uc73c\ub85c K-pop\uc758 \ub300\ud45c\ub85c \uc18c\ub140\uc2dc\ub300\ub97c \uc18c\uac1c\ud558\uba70 \uc0ac\uc9c4\uc774 \uc0bd\uc785\ub418\uc5c8\ub2e4. KIIS\uc758 \uc74c\uc545 \uac10\ub3c5\uc778 \uc904\ub9ac \ud544\ub78f (Julie Pilat)\uc740 2012\ub144 \uc8fc\ubaa9\ud560 \uadf8\ub8f9\uc73c\ub85c \ubf51\uae30\ub3c4 \ud588\ub2e4.", "paragraph_answer": "\uc18c\ub140\uc2dc\ub300\ub294 \ud55c\ub958\ub97c \uc774\ub044\ub294 \uc8fc\uc5ed\uc73c\ub85c\ub3c4 \uc778\uc815 \ubc1b\uace0 \uc788\ub2e4. \uc8fc\ud55c\uc77c\ubcf8\uacf5\uc0ac \uc2a4\uc988\ud0a4 \ud788\ub85c\uc2dc \ub294 \uc774\ub4e4\uc744 \uce74\ub77c\uc640 \ud568\uaed8 \uc77c\ubcf8\uc758 \uc2e0\ud55c\ub958\ub97c \uc774\ub044\ub294 \uadf8\ub8f9\uc774\ub77c\uace0 \ub9d0\ud588\uc73c\uba70, \u300a\uacbd\ud5a5\uc2e0\ubb38\u300b\uc5d0\uc11c\ub294 \"\uc18c\ub140\uc2dc\ub300\ub294 \ub300\uc911\ubb38\ud654\uc758 \uc544\uc774\ucf58, \uc544\uc2dc\uc544\ub97c \ub118\uc5b4 \uc720\ub7fd\uae4c\uc9c0 \uc9c4\ucd9c\ud55c \ud55c\ub958\uc758 \uc120\ub450\uc8fc\uc790\uac00 \ub410\ub2e4\"\ub77c\uba70 \"\ud55c\ub958 \uc5f4\ud48d \uc774\ub044\ub294 \uc544\uc2dc\uc544 \ucd5c\uace0 \uac78\uadf8\ub8f9\"\uc774\ub77c\uace0 \ub9d0\ud588\ub2e4. \ube0c\ub77c\uc9c8\uc758 \uc5f0\uc608 \ud504\ub85c\uadf8\ub7a8\uc778 \u2018Leitura Dinamica\u2019\ub294 \ube0c\ub77c\uc9c8\uc758 k-pop \uc5f4\ud48d\uc744 \uc18c\uac1c\ud558\uba70 \uc18c\ub140\uc2dc\ub300\ub97c \uc608\ub85c \ub4e4\uba70, \u201c\uc2e0\uace1 \u2018\ud6d7\u2019\uc740 \ube0c\ub77c\uc9c8 \ubfd0 \uc544\ub2c8\ub77c \uc138\uacc4 \uc720\uba85 DJ\ub4e4\uc5d0\uac8c\ub3c4 \ud070 \uc778\uae30\ub2e4. \uadf8\uc57c\ub9d0\ub85c \uc18c\ub140\uc2dc\ub300\uac00 \uc3dc \ud654\uc0b4\uc774 \uc544\uc2dc\uc544\ub97c \ub118\uc5b4 \uc138\uacc4\ub85c \ub0a0\uc544\uac00\uace0 \uc788\ub294 \uc148\u201d\uc774\ub77c\uace0 \ud638\ud3c9\ud588\ub2e4. \ub610 \uc544\ub9ac\ub791TV\uac00 \uc5f0 \u2018K POP - \ub530\ub77c\ud558\uae30\u2019\uc5d0\uc11c \uac00\uc7a5 \ub9ce\uc774 \ud328\ub7ec\ub514\ub41c \uac00\uc218\ub294 \uc18c\ub140\uc2dc\ub300\uc600\uc73c\uba70, \uc778\ub3c4\ub124\uc2dc\uc544 \ud32c\ub4e4\uc740 \uc18c\ub140\uc2dc\ub300\uc758 \ub77c\uc774\ube0c \uacf5\uc5f0\uc744 \uc790\uad6d\uc5d0\uc11c \ud574 \ub2ec\ub77c\uace0 \uc11c\uba85 \uc6b4\ub3d9\uc744 \ubc8c\uc774\uae30\ub3c4 \ud588\ub2e4. \uc18c\uc2dc\uc9c0\ub8e9, \ub9c8\ub9b0\ub8e9 \ub4f1 \ud328\uc158\uc740 \ub300\ud55c\ubbfc\uad6d\uc5d0\uc11c \uc778\uae30\ub97c \ub04c\uc5c8\uc73c\uba70, \ud2b9\ud788 \ud6c4\ub8e8\uc57c \ub9c8\uc0ac\uc720\ud0a4\ub294 \uadf8\ub4e4\uc774 \uc77c\ubcf8 \uc80a\uc740 \uc5ec\uc131\ub4e4\uc758 \ud328\uc158 \uc544\uc774\ucf58\uc774\ub77c\uace0 \ub9d0\ud588\ub2e4. \ub610\ud55c \uc77c\ubcf8 \uc601\uc5b4 \uad50\uacfc\uc11c\uc5d0 \u201c\uc804 \uc138\uacc4\uc5d0\uc11c \uc778\uae30\ub97c \uc5bb\uace0 \uc788\ub294 K-pop \uc2a4\ud0c0\ub4e4\u201d\uc774\ub77c\ub294 \uc81c\ubaa9\uc73c\ub85c K-pop\uc758 \ub300\ud45c\ub85c \uc18c\ub140\uc2dc\ub300\ub97c \uc18c\uac1c\ud558\uba70 \uc0ac\uc9c4\uc774 \uc0bd\uc785\ub418\uc5c8\ub2e4. KIIS\uc758 \uc74c\uc545 \uac10\ub3c5\uc778 \uc904\ub9ac \ud544\ub78f (Julie Pilat)\uc740 2012\ub144 \uc8fc\ubaa9\ud560 \uadf8\ub8f9\uc73c\ub85c \ubf51\uae30\ub3c4 \ud588\ub2e4.", "sentence_answer": "\uc8fc\ud55c\uc77c\ubcf8\uacf5\uc0ac \uc2a4\uc988\ud0a4 \ud788\ub85c\uc2dc \ub294 \uc774\ub4e4\uc744 \uce74\ub77c\uc640 \ud568\uaed8 \uc77c\ubcf8\uc758 \uc2e0\ud55c\ub958\ub97c \uc774\ub044\ub294 \uadf8\ub8f9\uc774\ub77c\uace0 \ub9d0\ud588\uc73c\uba70, \u300a\uacbd\ud5a5\uc2e0\ubb38\u300b\uc5d0\uc11c\ub294 \"\uc18c\ub140\uc2dc\ub300\ub294 \ub300\uc911\ubb38\ud654\uc758 \uc544\uc774\ucf58, \uc544\uc2dc\uc544\ub97c \ub118\uc5b4 \uc720\ub7fd\uae4c\uc9c0 \uc9c4\ucd9c\ud55c \ud55c\ub958\uc758 \uc120\ub450\uc8fc\uc790\uac00 \ub410\ub2e4\"\ub77c\uba70 \"\ud55c\ub958 \uc5f4\ud48d \uc774\ub044\ub294 \uc544\uc2dc\uc544 \ucd5c\uace0 \uac78\uadf8\ub8f9\"\uc774\ub77c\uace0 \ub9d0\ud588\ub2e4.", "paragraph_id": "6445662-18-0", "paragraph_question": "question: \uc18c\ub140\uc2dc\ub300 \ud65c\ub3d9 \ub2f9\uc2dc \uc8fc\ud55c\uc77c\ubcf8\uacf5\uc0ac\uc758 \uc774\ub984\uc740?, context: \uc18c\ub140\uc2dc\ub300\ub294 \ud55c\ub958\ub97c \uc774\ub044\ub294 \uc8fc\uc5ed\uc73c\ub85c\ub3c4 \uc778\uc815 \ubc1b\uace0 \uc788\ub2e4. \uc8fc\ud55c\uc77c\ubcf8\uacf5\uc0ac \uc2a4\uc988\ud0a4 \ud788\ub85c\uc2dc\ub294 \uc774\ub4e4\uc744 \uce74\ub77c\uc640 \ud568\uaed8 \uc77c\ubcf8\uc758 \uc2e0\ud55c\ub958\ub97c \uc774\ub044\ub294 \uadf8\ub8f9\uc774\ub77c\uace0 \ub9d0\ud588\uc73c\uba70, \u300a\uacbd\ud5a5\uc2e0\ubb38\u300b\uc5d0\uc11c\ub294 \"\uc18c\ub140\uc2dc\ub300\ub294 \ub300\uc911\ubb38\ud654\uc758 \uc544\uc774\ucf58, \uc544\uc2dc\uc544\ub97c \ub118\uc5b4 \uc720\ub7fd\uae4c\uc9c0 \uc9c4\ucd9c\ud55c \ud55c\ub958\uc758 \uc120\ub450\uc8fc\uc790\uac00 \ub410\ub2e4\"\ub77c\uba70 \"\ud55c\ub958 \uc5f4\ud48d \uc774\ub044\ub294 \uc544\uc2dc\uc544 \ucd5c\uace0 \uac78\uadf8\ub8f9\"\uc774\ub77c\uace0 \ub9d0\ud588\ub2e4. \ube0c\ub77c\uc9c8\uc758 \uc5f0\uc608 \ud504\ub85c\uadf8\ub7a8\uc778 \u2018Leitura Dinamica\u2019\ub294 \ube0c\ub77c\uc9c8\uc758 k-pop \uc5f4\ud48d\uc744 \uc18c\uac1c\ud558\uba70 \uc18c\ub140\uc2dc\ub300\ub97c \uc608\ub85c \ub4e4\uba70, \u201c\uc2e0\uace1 \u2018\ud6d7\u2019\uc740 \ube0c\ub77c\uc9c8 \ubfd0 \uc544\ub2c8\ub77c \uc138\uacc4 \uc720\uba85 DJ\ub4e4\uc5d0\uac8c\ub3c4 \ud070 \uc778\uae30\ub2e4. \uadf8\uc57c\ub9d0\ub85c \uc18c\ub140\uc2dc\ub300\uac00 \uc3dc \ud654\uc0b4\uc774 \uc544\uc2dc\uc544\ub97c \ub118\uc5b4 \uc138\uacc4\ub85c \ub0a0\uc544\uac00\uace0 \uc788\ub294 \uc148\u201d\uc774\ub77c\uace0 \ud638\ud3c9\ud588\ub2e4. \ub610 \uc544\ub9ac\ub791TV\uac00 \uc5f0 \u2018K POP - \ub530\ub77c\ud558\uae30\u2019\uc5d0\uc11c \uac00\uc7a5 \ub9ce\uc774 \ud328\ub7ec\ub514\ub41c \uac00\uc218\ub294 \uc18c\ub140\uc2dc\ub300\uc600\uc73c\uba70, \uc778\ub3c4\ub124\uc2dc\uc544 \ud32c\ub4e4\uc740 \uc18c\ub140\uc2dc\ub300\uc758 \ub77c\uc774\ube0c \uacf5\uc5f0\uc744 \uc790\uad6d\uc5d0\uc11c \ud574 \ub2ec\ub77c\uace0 \uc11c\uba85 \uc6b4\ub3d9\uc744 \ubc8c\uc774\uae30\ub3c4 \ud588\ub2e4. \uc18c\uc2dc\uc9c0\ub8e9, \ub9c8\ub9b0\ub8e9 \ub4f1 \ud328\uc158\uc740 \ub300\ud55c\ubbfc\uad6d\uc5d0\uc11c \uc778\uae30\ub97c \ub04c\uc5c8\uc73c\uba70, \ud2b9\ud788 \ud6c4\ub8e8\uc57c \ub9c8\uc0ac\uc720\ud0a4\ub294 \uadf8\ub4e4\uc774 \uc77c\ubcf8 \uc80a\uc740 \uc5ec\uc131\ub4e4\uc758 \ud328\uc158 \uc544\uc774\ucf58\uc774\ub77c\uace0 \ub9d0\ud588\ub2e4. \ub610\ud55c \uc77c\ubcf8 \uc601\uc5b4 \uad50\uacfc\uc11c\uc5d0 \u201c\uc804 \uc138\uacc4\uc5d0\uc11c \uc778\uae30\ub97c \uc5bb\uace0 \uc788\ub294 K-pop \uc2a4\ud0c0\ub4e4\u201d\uc774\ub77c\ub294 \uc81c\ubaa9\uc73c\ub85c K-pop\uc758 \ub300\ud45c\ub85c \uc18c\ub140\uc2dc\ub300\ub97c \uc18c\uac1c\ud558\uba70 \uc0ac\uc9c4\uc774 \uc0bd\uc785\ub418\uc5c8\ub2e4. KIIS\uc758 \uc74c\uc545 \uac10\ub3c5\uc778 \uc904\ub9ac \ud544\ub78f (Julie Pilat)\uc740 2012\ub144 \uc8fc\ubaa9\ud560 \uadf8\ub8f9\uc73c\ub85c \ubf51\uae30\ub3c4 \ud588\ub2e4."} {"question": "\uc870\ub2c8 \ud558\ud2b8\ub294 \uc65c \uac10\ub3c5\uc744 \uba87 \uac1c\uc6d4 \ud558\uc9c0 \ubabb\ud588\ub294\uac00?", "paragraph": "1972\ub144\ubd80\ud130 1974\ub144\uae4c\uc9c0, \ub9e8\uccb4\uc2a4\ud130 \uc2dc\ud2f0\ub294 \uacbd\uae30\uc7a5 \uc548\ud30e\uc73c\ub85c \uc5ec\ub7ec\uac00\uc9c0 \ubcc0\ud654\ub97c \uacaa\uac8c \ub41c\ub2e4. \ube44\uc988\ub2c8\uc2a4\ub9e8\uc778 \ud53c\ud130 \uc2a4\uc6e8\uc77c\uc2a4\uac00 \ud68c\uc7a5\uc774 \ub418\uba70, \ub9d0\ucf64 \uc568\ub9ac\uc2a8\uc740 \uac15\ub4f1\uad8c\uc73c\ub85c \uce58\ub2eb\ub358 1972 - 73\uc2dc\uc98c \ub3c4\uc911, \uac10\ub3c5\uc9c1\uc744 \uc0ac\uc784\ud558\uac8c \ub41c\ub2e4. \uc624\ub79c\uae30\uac04 \ucf54\uce58\ub85c \uc5ed\uc784\ud574 \uc788\ub358 \uc870\ub2c8 \ud558\ud2b8\uac00 \uac10\ub3c5\ub300\ud589\uc744 \ub9e1\uc544 \ud300\uc744 \uac15\ub4f1\uad8c\uc5d0\uc11c \uad6c\ud574\ub0b4\uace0 \uad1c\ucc2e\uc740 \uc131\uc801\uc744 \uac70\ub450\uac8c \ub418\uc5b4 \uacb0\uad6d \uc2dc\uc98c\uc774 \ub05d\ub098\uace0 \uc815\uc2dd\uac10\ub3c5\uc9c1\uc5d0 \uc624\ub974\uac8c \ub41c\ub2e4. \ud558\uc9c0\ub9cc \uac74\uac15\uc0c1\uc758 \ubb38\uc81c\uac00 \uadf8\ub97c \uc555\ubc15\ud588\uace0 \uadf8\uc758 \uac10\ub3c5 \uae30\uac04\uc740 \uc218 \uac1c\uc6d4\ubc16\uc5d0 \uac00\uc9c0 \ubabb\ud588\ub2e4. \ub178\ub9ac\uce58 \uc2dc\ud2f0\uc758 \uac10\ub3c5\uc744 \ub9e1\uace0 \uc788\uc5c8\ub358 \ub860 \uc190\ub354\uc2a4\uac00 \ud558\ud2b8\uc758 \uc790\ub9ac\ub97c \ub300\uc2e0\ud558\uac8c \ub418\ub294\ub370 \uadf8\uc758 \uc9c0\ud718 \uc544\ub798 \ub9e8\uccb4\uc2a4\ud130 \uc2dc\ud2f0\ub294 1974\ub144, \ub9ac\uadf8 \ucef5 \uacb0\uc2b9\uc5d0 \uc9c4\ucd9c\ud558\uc9c0\ub9cc \uc544\uae5d\uac8c \uc900\uc6b0\uc2b9\uc5d0 \uadf8\uce58\uace0 \ub9cc\ub2e4. \ub9ac\uadf8\uc5d0\uc120 \ubd80\uc9c4\ud558\uac8c \ub418\uc5c8\uace0 \uc190\ub354\uc2a4\ub294 1973 - 74\uc2dc\uc98c \ud55c\ub2ec\uc744 \ub0a8\uaca8\ub193\uace0 \ud574\uace0 \ub2f9\ud558\uac8c \ub418\uc5c8\uace0 \ub9e8\uccb4\uc2a4\ud130 \uc2dc\ud2f0\ub3c4 \uac15\ub4f1\ub2f9\ud560 \uc704\uae30\uc5d0 \ucc98\ud558\uac8c \ub41c\ub2e4. \uc774\uc5d0 \u524d \uc8fc\uc7a5\uc774\uc5c8\ub358 \ud1a0\ub2c8 \ubd81\uc774 \uc120\uc218 \uacb8 \uac10\ub3c5\uc73c\ub85c \ud300\uc744 \uc774\ub04c\uc5b4 \uc548\uc815\uc744 \ub418\ucc3e\uac8c \ub41c\ub2e4. \uc2dc\uc98c \ub9c8\uc9c0\ub9c9\ub0a0\uc758 \uc0c1\ub300\ub294 \uc5f0\uace0\uc9c0 \ub77c\uc774\ubc8c\uc778 \ub9e8\uccb4\uc2a4\ud130 \uc720\ub098\uc774\ud2f0\ub4dc \ub85c\uc368 \uac15\ub4f1\uc744 \uba74\ud558\uae30 \uc704\ud574\uc11c \uc591\ud300 \ubaa8\ub450 \uc2b9\ub9ac\uac00 \uc808\uc2e4\ud588\ub358 \uc0c1\ud669\uc774\uc5c8\ub2e4. \u524d \uc720\ub098\uc774\ud2f0\ub4dc \uc120\uc218 \uc600\ub358 \ub370\ub2c8\uc2a4 \ub85c\uac00 \ud790\ud0a5\uc73c\ub85c \ub4dd\uc810\uc744 \uc131\uacf5\uc2dc\ucf1c 1-0 \uc2b9\ub9ac\ub97c \uac70\ub450\uace0 \ub9e8\uccb4\uc2a4\ud130 \uc720\ub098\uc774\ud2f0\ub4dc\uc758 \uac15\ub4f1\uc744 \ud655\uc815\uc9d3\uac8c \ub9cc\ub4e4\uc5c8\ub2e4. \ub9e8\uccb4\uc2a4\ud130 \uc2dc\ud2f0\ub294 \ud1a0\ub2c8 \ubd81\uc758 \uc7ac\uc784\uae30\uac04 \ub3d9\uc548 \uc791\uc740 \uc131\uacf5\uc744 \uac70\ub450\uac8c \ub418\ub294\ub370 1976\ub144, \ub274\uce90\uc2ac \uc720\ub098\uc774\ud2f0\ub4dc\ub97c 2-1\ub85c \uaebe\uace0 \ub450 \ubc88\uc9f8 \ub9ac\uadf8 \ucef5 \uc6b0\uc2b9\uc744 \uac70\uba38\uc950\uc5c8\uace0 1977\ub144\uc5d0\ub294 \ub9ac\uadf8\uc5d0\uc11c 2\uc704\ub97c \uae30\ub85d\ud558\uc600\ub2e4.", "answer": "\uac74\uac15\uc0c1\uc758 \ubb38\uc81c", "sentence": "\ud558\uc9c0\ub9cc \uac74\uac15\uc0c1\uc758 \ubb38\uc81c \uac00 \uadf8\ub97c \uc555\ubc15\ud588\uace0 \uadf8\uc758 \uac10\ub3c5 \uae30\uac04\uc740 \uc218 \uac1c\uc6d4\ubc16\uc5d0 \uac00\uc9c0 \ubabb\ud588\ub2e4.", "paragraph_sentence": "1972\ub144\ubd80\ud130 1974\ub144\uae4c\uc9c0, \ub9e8\uccb4\uc2a4\ud130 \uc2dc\ud2f0\ub294 \uacbd\uae30\uc7a5 \uc548\ud30e\uc73c\ub85c \uc5ec\ub7ec\uac00\uc9c0 \ubcc0\ud654\ub97c \uacaa\uac8c \ub41c\ub2e4. \ube44\uc988\ub2c8\uc2a4\ub9e8\uc778 \ud53c\ud130 \uc2a4\uc6e8\uc77c\uc2a4\uac00 \ud68c\uc7a5\uc774 \ub418\uba70, \ub9d0\ucf64 \uc568\ub9ac\uc2a8\uc740 \uac15\ub4f1\uad8c\uc73c\ub85c \uce58\ub2eb\ub358 1972 - 73\uc2dc\uc98c \ub3c4\uc911, \uac10\ub3c5\uc9c1\uc744 \uc0ac\uc784\ud558\uac8c \ub41c\ub2e4. \uc624\ub79c\uae30\uac04 \ucf54\uce58\ub85c \uc5ed\uc784\ud574 \uc788\ub358 \uc870\ub2c8 \ud558\ud2b8\uac00 \uac10\ub3c5\ub300\ud589\uc744 \ub9e1\uc544 \ud300\uc744 \uac15\ub4f1\uad8c\uc5d0\uc11c \uad6c\ud574\ub0b4\uace0 \uad1c\ucc2e\uc740 \uc131\uc801\uc744 \uac70\ub450\uac8c \ub418\uc5b4 \uacb0\uad6d \uc2dc\uc98c\uc774 \ub05d\ub098\uace0 \uc815\uc2dd\uac10\ub3c5\uc9c1\uc5d0 \uc624\ub974\uac8c \ub41c\ub2e4. \ud558\uc9c0\ub9cc \uac74\uac15\uc0c1\uc758 \ubb38\uc81c \uac00 \uadf8\ub97c \uc555\ubc15\ud588\uace0 \uadf8\uc758 \uac10\ub3c5 \uae30\uac04\uc740 \uc218 \uac1c\uc6d4\ubc16\uc5d0 \uac00\uc9c0 \ubabb\ud588\ub2e4. \ub178\ub9ac\uce58 \uc2dc\ud2f0\uc758 \uac10\ub3c5\uc744 \ub9e1\uace0 \uc788\uc5c8\ub358 \ub860 \uc190\ub354\uc2a4\uac00 \ud558\ud2b8\uc758 \uc790\ub9ac\ub97c \ub300\uc2e0\ud558\uac8c \ub418\ub294\ub370 \uadf8\uc758 \uc9c0\ud718 \uc544\ub798 \ub9e8\uccb4\uc2a4\ud130 \uc2dc\ud2f0\ub294 1974\ub144, \ub9ac\uadf8 \ucef5 \uacb0\uc2b9\uc5d0 \uc9c4\ucd9c\ud558\uc9c0\ub9cc \uc544\uae5d\uac8c \uc900\uc6b0\uc2b9\uc5d0 \uadf8\uce58\uace0 \ub9cc\ub2e4. \ub9ac\uadf8\uc5d0\uc120 \ubd80\uc9c4\ud558\uac8c \ub418\uc5c8\uace0 \uc190\ub354\uc2a4\ub294 1973 - 74\uc2dc\uc98c \ud55c\ub2ec\uc744 \ub0a8\uaca8\ub193\uace0 \ud574\uace0 \ub2f9\ud558\uac8c \ub418\uc5c8\uace0 \ub9e8\uccb4\uc2a4\ud130 \uc2dc\ud2f0\ub3c4 \uac15\ub4f1\ub2f9\ud560 \uc704\uae30\uc5d0 \ucc98\ud558\uac8c \ub41c\ub2e4. \uc774\uc5d0 \u524d \uc8fc\uc7a5\uc774\uc5c8\ub358 \ud1a0\ub2c8 \ubd81\uc774 \uc120\uc218 \uacb8 \uac10\ub3c5\uc73c\ub85c \ud300\uc744 \uc774\ub04c\uc5b4 \uc548\uc815\uc744 \ub418\ucc3e\uac8c \ub41c\ub2e4. \uc2dc\uc98c \ub9c8\uc9c0\ub9c9\ub0a0\uc758 \uc0c1\ub300\ub294 \uc5f0\uace0\uc9c0 \ub77c\uc774\ubc8c\uc778 \ub9e8\uccb4\uc2a4\ud130 \uc720\ub098\uc774\ud2f0\ub4dc \ub85c\uc368 \uac15\ub4f1\uc744 \uba74\ud558\uae30 \uc704\ud574\uc11c \uc591\ud300 \ubaa8\ub450 \uc2b9\ub9ac\uac00 \uc808\uc2e4\ud588\ub358 \uc0c1\ud669\uc774\uc5c8\ub2e4. \u524d \uc720\ub098\uc774\ud2f0\ub4dc \uc120\uc218 \uc600\ub358 \ub370\ub2c8\uc2a4 \ub85c\uac00 \ud790\ud0a5\uc73c\ub85c \ub4dd\uc810\uc744 \uc131\uacf5\uc2dc\ucf1c 1-0 \uc2b9\ub9ac\ub97c \uac70\ub450\uace0 \ub9e8\uccb4\uc2a4\ud130 \uc720\ub098\uc774\ud2f0\ub4dc\uc758 \uac15\ub4f1\uc744 \ud655\uc815\uc9d3\uac8c \ub9cc\ub4e4\uc5c8\ub2e4. \ub9e8\uccb4\uc2a4\ud130 \uc2dc\ud2f0\ub294 \ud1a0\ub2c8 \ubd81\uc758 \uc7ac\uc784\uae30\uac04 \ub3d9\uc548 \uc791\uc740 \uc131\uacf5\uc744 \uac70\ub450\uac8c \ub418\ub294\ub370 1976\ub144, \ub274\uce90\uc2ac \uc720\ub098\uc774\ud2f0\ub4dc\ub97c 2-1\ub85c \uaebe\uace0 \ub450 \ubc88\uc9f8 \ub9ac\uadf8 \ucef5 \uc6b0\uc2b9\uc744 \uac70\uba38\uc950\uc5c8\uace0 1977\ub144\uc5d0\ub294 \ub9ac\uadf8\uc5d0\uc11c 2\uc704\ub97c \uae30\ub85d\ud558\uc600\ub2e4.", "paragraph_answer": "1972\ub144\ubd80\ud130 1974\ub144\uae4c\uc9c0, \ub9e8\uccb4\uc2a4\ud130 \uc2dc\ud2f0\ub294 \uacbd\uae30\uc7a5 \uc548\ud30e\uc73c\ub85c \uc5ec\ub7ec\uac00\uc9c0 \ubcc0\ud654\ub97c \uacaa\uac8c \ub41c\ub2e4. \ube44\uc988\ub2c8\uc2a4\ub9e8\uc778 \ud53c\ud130 \uc2a4\uc6e8\uc77c\uc2a4\uac00 \ud68c\uc7a5\uc774 \ub418\uba70, \ub9d0\ucf64 \uc568\ub9ac\uc2a8\uc740 \uac15\ub4f1\uad8c\uc73c\ub85c \uce58\ub2eb\ub358 1972 - 73\uc2dc\uc98c \ub3c4\uc911, \uac10\ub3c5\uc9c1\uc744 \uc0ac\uc784\ud558\uac8c \ub41c\ub2e4. \uc624\ub79c\uae30\uac04 \ucf54\uce58\ub85c \uc5ed\uc784\ud574 \uc788\ub358 \uc870\ub2c8 \ud558\ud2b8\uac00 \uac10\ub3c5\ub300\ud589\uc744 \ub9e1\uc544 \ud300\uc744 \uac15\ub4f1\uad8c\uc5d0\uc11c \uad6c\ud574\ub0b4\uace0 \uad1c\ucc2e\uc740 \uc131\uc801\uc744 \uac70\ub450\uac8c \ub418\uc5b4 \uacb0\uad6d \uc2dc\uc98c\uc774 \ub05d\ub098\uace0 \uc815\uc2dd\uac10\ub3c5\uc9c1\uc5d0 \uc624\ub974\uac8c \ub41c\ub2e4. \ud558\uc9c0\ub9cc \uac74\uac15\uc0c1\uc758 \ubb38\uc81c \uac00 \uadf8\ub97c \uc555\ubc15\ud588\uace0 \uadf8\uc758 \uac10\ub3c5 \uae30\uac04\uc740 \uc218 \uac1c\uc6d4\ubc16\uc5d0 \uac00\uc9c0 \ubabb\ud588\ub2e4. \ub178\ub9ac\uce58 \uc2dc\ud2f0\uc758 \uac10\ub3c5\uc744 \ub9e1\uace0 \uc788\uc5c8\ub358 \ub860 \uc190\ub354\uc2a4\uac00 \ud558\ud2b8\uc758 \uc790\ub9ac\ub97c \ub300\uc2e0\ud558\uac8c \ub418\ub294\ub370 \uadf8\uc758 \uc9c0\ud718 \uc544\ub798 \ub9e8\uccb4\uc2a4\ud130 \uc2dc\ud2f0\ub294 1974\ub144, \ub9ac\uadf8 \ucef5 \uacb0\uc2b9\uc5d0 \uc9c4\ucd9c\ud558\uc9c0\ub9cc \uc544\uae5d\uac8c \uc900\uc6b0\uc2b9\uc5d0 \uadf8\uce58\uace0 \ub9cc\ub2e4. \ub9ac\uadf8\uc5d0\uc120 \ubd80\uc9c4\ud558\uac8c \ub418\uc5c8\uace0 \uc190\ub354\uc2a4\ub294 1973 - 74\uc2dc\uc98c \ud55c\ub2ec\uc744 \ub0a8\uaca8\ub193\uace0 \ud574\uace0 \ub2f9\ud558\uac8c \ub418\uc5c8\uace0 \ub9e8\uccb4\uc2a4\ud130 \uc2dc\ud2f0\ub3c4 \uac15\ub4f1\ub2f9\ud560 \uc704\uae30\uc5d0 \ucc98\ud558\uac8c \ub41c\ub2e4. \uc774\uc5d0 \u524d \uc8fc\uc7a5\uc774\uc5c8\ub358 \ud1a0\ub2c8 \ubd81\uc774 \uc120\uc218 \uacb8 \uac10\ub3c5\uc73c\ub85c \ud300\uc744 \uc774\ub04c\uc5b4 \uc548\uc815\uc744 \ub418\ucc3e\uac8c \ub41c\ub2e4. \uc2dc\uc98c \ub9c8\uc9c0\ub9c9\ub0a0\uc758 \uc0c1\ub300\ub294 \uc5f0\uace0\uc9c0 \ub77c\uc774\ubc8c\uc778 \ub9e8\uccb4\uc2a4\ud130 \uc720\ub098\uc774\ud2f0\ub4dc \ub85c\uc368 \uac15\ub4f1\uc744 \uba74\ud558\uae30 \uc704\ud574\uc11c \uc591\ud300 \ubaa8\ub450 \uc2b9\ub9ac\uac00 \uc808\uc2e4\ud588\ub358 \uc0c1\ud669\uc774\uc5c8\ub2e4. \u524d \uc720\ub098\uc774\ud2f0\ub4dc \uc120\uc218 \uc600\ub358 \ub370\ub2c8\uc2a4 \ub85c\uac00 \ud790\ud0a5\uc73c\ub85c \ub4dd\uc810\uc744 \uc131\uacf5\uc2dc\ucf1c 1-0 \uc2b9\ub9ac\ub97c \uac70\ub450\uace0 \ub9e8\uccb4\uc2a4\ud130 \uc720\ub098\uc774\ud2f0\ub4dc\uc758 \uac15\ub4f1\uc744 \ud655\uc815\uc9d3\uac8c \ub9cc\ub4e4\uc5c8\ub2e4. \ub9e8\uccb4\uc2a4\ud130 \uc2dc\ud2f0\ub294 \ud1a0\ub2c8 \ubd81\uc758 \uc7ac\uc784\uae30\uac04 \ub3d9\uc548 \uc791\uc740 \uc131\uacf5\uc744 \uac70\ub450\uac8c \ub418\ub294\ub370 1976\ub144, \ub274\uce90\uc2ac \uc720\ub098\uc774\ud2f0\ub4dc\ub97c 2-1\ub85c \uaebe\uace0 \ub450 \ubc88\uc9f8 \ub9ac\uadf8 \ucef5 \uc6b0\uc2b9\uc744 \uac70\uba38\uc950\uc5c8\uace0 1977\ub144\uc5d0\ub294 \ub9ac\uadf8\uc5d0\uc11c 2\uc704\ub97c \uae30\ub85d\ud558\uc600\ub2e4.", "sentence_answer": "\ud558\uc9c0\ub9cc \uac74\uac15\uc0c1\uc758 \ubb38\uc81c \uac00 \uadf8\ub97c \uc555\ubc15\ud588\uace0 \uadf8\uc758 \uac10\ub3c5 \uae30\uac04\uc740 \uc218 \uac1c\uc6d4\ubc16\uc5d0 \uac00\uc9c0 \ubabb\ud588\ub2e4.", "paragraph_id": "6465982-8-1", "paragraph_question": "question: \uc870\ub2c8 \ud558\ud2b8\ub294 \uc65c \uac10\ub3c5\uc744 \uba87 \uac1c\uc6d4 \ud558\uc9c0 \ubabb\ud588\ub294\uac00?, context: 1972\ub144\ubd80\ud130 1974\ub144\uae4c\uc9c0, \ub9e8\uccb4\uc2a4\ud130 \uc2dc\ud2f0\ub294 \uacbd\uae30\uc7a5 \uc548\ud30e\uc73c\ub85c \uc5ec\ub7ec\uac00\uc9c0 \ubcc0\ud654\ub97c \uacaa\uac8c \ub41c\ub2e4. \ube44\uc988\ub2c8\uc2a4\ub9e8\uc778 \ud53c\ud130 \uc2a4\uc6e8\uc77c\uc2a4\uac00 \ud68c\uc7a5\uc774 \ub418\uba70, \ub9d0\ucf64 \uc568\ub9ac\uc2a8\uc740 \uac15\ub4f1\uad8c\uc73c\ub85c \uce58\ub2eb\ub358 1972 - 73\uc2dc\uc98c \ub3c4\uc911, \uac10\ub3c5\uc9c1\uc744 \uc0ac\uc784\ud558\uac8c \ub41c\ub2e4. \uc624\ub79c\uae30\uac04 \ucf54\uce58\ub85c \uc5ed\uc784\ud574 \uc788\ub358 \uc870\ub2c8 \ud558\ud2b8\uac00 \uac10\ub3c5\ub300\ud589\uc744 \ub9e1\uc544 \ud300\uc744 \uac15\ub4f1\uad8c\uc5d0\uc11c \uad6c\ud574\ub0b4\uace0 \uad1c\ucc2e\uc740 \uc131\uc801\uc744 \uac70\ub450\uac8c \ub418\uc5b4 \uacb0\uad6d \uc2dc\uc98c\uc774 \ub05d\ub098\uace0 \uc815\uc2dd\uac10\ub3c5\uc9c1\uc5d0 \uc624\ub974\uac8c \ub41c\ub2e4. \ud558\uc9c0\ub9cc \uac74\uac15\uc0c1\uc758 \ubb38\uc81c\uac00 \uadf8\ub97c \uc555\ubc15\ud588\uace0 \uadf8\uc758 \uac10\ub3c5 \uae30\uac04\uc740 \uc218 \uac1c\uc6d4\ubc16\uc5d0 \uac00\uc9c0 \ubabb\ud588\ub2e4. \ub178\ub9ac\uce58 \uc2dc\ud2f0\uc758 \uac10\ub3c5\uc744 \ub9e1\uace0 \uc788\uc5c8\ub358 \ub860 \uc190\ub354\uc2a4\uac00 \ud558\ud2b8\uc758 \uc790\ub9ac\ub97c \ub300\uc2e0\ud558\uac8c \ub418\ub294\ub370 \uadf8\uc758 \uc9c0\ud718 \uc544\ub798 \ub9e8\uccb4\uc2a4\ud130 \uc2dc\ud2f0\ub294 1974\ub144, \ub9ac\uadf8 \ucef5 \uacb0\uc2b9\uc5d0 \uc9c4\ucd9c\ud558\uc9c0\ub9cc \uc544\uae5d\uac8c \uc900\uc6b0\uc2b9\uc5d0 \uadf8\uce58\uace0 \ub9cc\ub2e4. \ub9ac\uadf8\uc5d0\uc120 \ubd80\uc9c4\ud558\uac8c \ub418\uc5c8\uace0 \uc190\ub354\uc2a4\ub294 1973 - 74\uc2dc\uc98c \ud55c\ub2ec\uc744 \ub0a8\uaca8\ub193\uace0 \ud574\uace0 \ub2f9\ud558\uac8c \ub418\uc5c8\uace0 \ub9e8\uccb4\uc2a4\ud130 \uc2dc\ud2f0\ub3c4 \uac15\ub4f1\ub2f9\ud560 \uc704\uae30\uc5d0 \ucc98\ud558\uac8c \ub41c\ub2e4. \uc774\uc5d0 \u524d \uc8fc\uc7a5\uc774\uc5c8\ub358 \ud1a0\ub2c8 \ubd81\uc774 \uc120\uc218 \uacb8 \uac10\ub3c5\uc73c\ub85c \ud300\uc744 \uc774\ub04c\uc5b4 \uc548\uc815\uc744 \ub418\ucc3e\uac8c \ub41c\ub2e4. \uc2dc\uc98c \ub9c8\uc9c0\ub9c9\ub0a0\uc758 \uc0c1\ub300\ub294 \uc5f0\uace0\uc9c0 \ub77c\uc774\ubc8c\uc778 \ub9e8\uccb4\uc2a4\ud130 \uc720\ub098\uc774\ud2f0\ub4dc \ub85c\uc368 \uac15\ub4f1\uc744 \uba74\ud558\uae30 \uc704\ud574\uc11c \uc591\ud300 \ubaa8\ub450 \uc2b9\ub9ac\uac00 \uc808\uc2e4\ud588\ub358 \uc0c1\ud669\uc774\uc5c8\ub2e4. \u524d \uc720\ub098\uc774\ud2f0\ub4dc \uc120\uc218 \uc600\ub358 \ub370\ub2c8\uc2a4 \ub85c\uac00 \ud790\ud0a5\uc73c\ub85c \ub4dd\uc810\uc744 \uc131\uacf5\uc2dc\ucf1c 1-0 \uc2b9\ub9ac\ub97c \uac70\ub450\uace0 \ub9e8\uccb4\uc2a4\ud130 \uc720\ub098\uc774\ud2f0\ub4dc\uc758 \uac15\ub4f1\uc744 \ud655\uc815\uc9d3\uac8c \ub9cc\ub4e4\uc5c8\ub2e4. \ub9e8\uccb4\uc2a4\ud130 \uc2dc\ud2f0\ub294 \ud1a0\ub2c8 \ubd81\uc758 \uc7ac\uc784\uae30\uac04 \ub3d9\uc548 \uc791\uc740 \uc131\uacf5\uc744 \uac70\ub450\uac8c \ub418\ub294\ub370 1976\ub144, \ub274\uce90\uc2ac \uc720\ub098\uc774\ud2f0\ub4dc\ub97c 2-1\ub85c \uaebe\uace0 \ub450 \ubc88\uc9f8 \ub9ac\uadf8 \ucef5 \uc6b0\uc2b9\uc744 \uac70\uba38\uc950\uc5c8\uace0 1977\ub144\uc5d0\ub294 \ub9ac\uadf8\uc5d0\uc11c 2\uc704\ub97c \uae30\ub85d\ud558\uc600\ub2e4."} {"question": "\ud0c0\ub9c8\ub974\uc758 \uacc4\uc2b9\uc744 \ubc18\ub300\ud588\ub358 \ubc18\ub300\ud30c\ub4e4\uc758 \uc8fc\uc694 \uc774\uc720\ub294?", "paragraph": "\ud0c0\ub9c8\ub974\ub294 6\ub144\ub3d9\uc548 \uadf8\uc758 \ubd80\uce5c\uc774 \uc8fd\ub358 1184\ub144\uae4c\uc9c0 \uc12d\uc815\ud588\uace0, \uc774\uc5b4\uc11c \ub2e8\uc77c \uad70\uc8fc\uac00 \ub418\uc5c8\uc73c\uba70, \uc870\uc9c0\uc544 \ub3d9\ubd80\uc758 \ucfe0\ud0c0\uc774\uc2dc \uadfc\uad50\uc5d0 \uc788\ub294 \uac94\ub77c\ud2f0 \uc218\ub3c4\uc6d0\uc5d0\uc11c \uc989\uc704\uc2dd\uc744 \uce58\ub800\ub2e4. \uadf8\ub140\ub294 \ube44\uad50\uc801 \uac15\ub825\ud55c \uc655\uad6d\uc744 \uc2b9\uacc4\ud588\uc9c0\ub9cc, \uace0\uc704\uae09 \uadc0\uc871\ub4e4\uc5d0 \uc758\ud574 \uc870\uc7a5\ub41c \uc911\uc559\uc9d1\uad8c\uc744 \ud750\ud2b8\ub824\ud2b8\ub824 \ub193\uc73c\ub824\ub294 \ud48d\uc870\ub294 \uac00\ub77c \uc549\uc744 \uc904\uc744 \ubab0\ub790\ub2e4. \ud0c0\ub9c8\ub974\uc758 \uacc4\uc2b9\uc744 \ubc18\ub300\ud558\ub358 \uc2ec\uc0c1\uce58 \uc54a\uc740 \uc790\ub4e4\uc774 \uc788\uc5c8\ub2e4. \uadf8\ub4e4\uc740 \uadf8\ub140\uc758 \ubd80\uce5c\uc774 \uadf8\ub4e4\uc744 \uc5b5\uc555\ud588\ub358 \uc815\ucc45\uc5d0 \ub300\ud574 \ubc18\uae30\ub97c \ub4e4\uc5b4 \uc62c\ub838\uc9c0\ub9cc, \uc0c8\ub85c\uc6b4 \uad70\uc8fc\uc758 \ucde8\uc57d\uc810, \uc989 \uc5ec\uc131\uc774\ub77c\ub294 \uc0ac\uc2e4\uc774 \uadf8\ub4e4\uc744 \ub354 \ubd80\ucd94\uacbc\ub358 \uac83\uc774\ub2e4. \uadf8\ub140 \uc804\uae4c\uc9c0\ub9cc \ud574\ub3c4 \uc870\uc9c0\uc544\uc5d0\uc11c\ub294 \uc5ec\uc131 \uad70\uc8fc\uac00 \ub2e8 \ud55c \uba85\ub3c4 \uc5c6\uc5c8\uae30 \ub54c\ubb38\uc5d0, \uadc0\uc871\ub4e4\uc758 \uc77c\ubd80\ub294 \ud0c0\ub9c8\ub974 \uc5ec\uc655\uc758 \uc801\ubc95\uc131\uc5d0 \ub300\ud574 \uc758\ubb38\uc744 \ud488\uc5c8\uc9c0\ub9cc, \ubc18\uba74\uc5d0 \ub2e4\ub978 \uc0ac\ub78c\ub4e4\uc740 \uadf8\ub140\uc758 \uc5b4\ub9b0 \uc810\uc744 \uc774\uc6a9\ud574 \uadf8\ub140\uc758 \uacb0\uc810\uc774 \uc624\ud788\ub824 \uadf8\ub4e4\uc758 \ub300\uc790\uce58\uad8c\uc744 \ud655\uace0\ud788 \ud574\uc904 \uc218 \uc788\uc744 \uac83\uc73c\ub85c \uc608\uc0c1\ud588\ub2e4. \ud0c0\ub9c8\ub974\uc758 \uc601\ud5a5\ub825 \uc788\ub294 \uace0\ubaa8\uc778 \ub8e8\uc218\ub2e8 \uc5ec\uc655\uacfc \uc870\uc9c0\uc544\uc758 \uac00\ud1a8\ub9ac\ucf54\uc2a4 \ubbf8\ucc28\uc5d8 4\uc138 \ubbf8\ub9ac\uc544\ub2c8\uc2dc\uc81c\uc758 \uc5f4\uc815\uc801\uc778 \ub3c4\uc6c0\uc740 \uc870\uc9c0\uc544 \uc655\uad6d\uc744 \uc2b9\uacc4\ud55c \ud0c0\ub9c8\ub974 \uc5ec\uc655\uc744 \ud569\ubc95\ud654\ud558\ub294\ub370 \uacb0\uc815\uc801\uc778 \uc5ed\ud560\uc744 \ud588\ub2e4. \uadf8\ub140\ub294 \uac00\ud1a8\ub9ac\ucf54\uc2a4 \ubbf8\ucc28\uc5d8\uc758 \ub3c4\uc6c0\uc5d0 \ub300\ud55c \ubcf4\uc0c1\uc73c\ub85c \uadf8\ub97c \ub300\ubc95\uad00\uc5d0 \uc784\uba85\ud588\uace0, \uadf8\ub9ac\ud558\uc5ec \ubbf8\ucc28\uc5d8\uc740 \uc131\uc9c1\uacfc \uc138\uc18d\uc758 \uacf5\ubb34 \uccb4\uacc4 \uc591\ucabd \ubaa8\ub450\uc5d0\uc11c \uc218\uc7a5\uc758 \uc9c0\uc704\uc5d0 \uc624\ub974\uac8c \ub418\uc5c8\ub2e4. \ud0c0\ub9c8\ub974\ub294 \uadf8\ub140\uc758 \ubd80\uce5c\uc778 \uac8c\uc624\ub974\uac8c 3\uc138\uac00 \uadf8\uc5d0\uac8c \ub3c4\uc804\ud558\ub358 \uadc0\uc871\ub4e4\uc744 \uc5c4\uc911 \ucc98\ubc8c\ud558\ub294\ub370 \ub3c4\uc6c0\uc744 \uc92c\ub358 \uadf8\uc758 \ubd80\uce5c\uc774 \uc784\uba85\ud588\ub358 \uc0ac\ub78c\ub4e4 \uc911\uc5d0\uc11c \ub0ae\uc740 \uc2e0\ubd84 \ud0dc\uc0dd\uc758 \uc870\uc9c0\uc544\uc778\ud654 \ub41c \ud0b5\ucc28\ud06c\uc778\uc778 \ucfe0\ubc14\uc0ac\uc2a4 \ubcf4\uc548\uad00\uc758 \ud574\uc784\uc744 \ucd09\uad6c\ud558\uae30\ub3c4 \ud588\ub2e4. \uac8c\uc624\ub974\uac8c 3\uc138\uac00 \ube44\ucc9c\ud55c \uc2e0\ubd84\uc73c\ub85c\uc758 \uc219\uba85\uc744 \ubc97\uc5b4\ub0a0 \uc218 \uc788\uac8c \ud574\uc900 \uadf8\uc758 \ubb34\uba85\uc758 \uc2dc\uc911\ub4e4 \uc911\uc5d0 \ud55c\uba85\uc740 \uadf8 \ub54c\uc5d0 \uc0c8\ub85c\uc6b4 \uc758\ud68c\uc778 \uce74\ub77c\ube44\ub97c \uc870\uc9c1\ud574 \uc758\ud68c\ub9cc \ub530\ub85c \ub5a8\uc5b4\uc838 \uc2ec\uc0ac\uc219\uace0 \ud558\uc5ec \uc815\ucc45\uc744 \uacb0\uc815\ud588\ub358 \uadf8\ub807\uac8c \uc655\uad8c\uc744 \uc81c\ud55c\ud558\ub824\ub294 \ud22c\uc7c1 \uc911\uc5d0 \uc788\uc5c8\ub358 \uadc0\uc871 \ub2e8\uccb4\ub4e4\uacfc \ubd80\uc720\ud55c \uc2dc\ubbfc\ub4e4\uc744 \uc774\ub04c\ub358 \uc7ac\ubb34 \ub2f4\ub2f9\uc790 \ucfe0\ud2c0\ub8e8 \uc544\uc2ac\ub780\uc774\uc5c8\ub2e4. \ud0c0\ub9c8\ub974\uac00 \ucfe0\ud2c0\ub8e8 \uc544\uc2ac\ub780\uc744 \uccb4\ud3ec\ud558\uace0 \uadf8\uc758 \ucd94\uc885\uc790\ub4e4\uc744 \uad6c\uc2ac\ub824\uc11c \ud56d\ubcf5\ud558\uac8c \ud568\uc73c\ub85c \"\ubd09\uac74 \uc785\ud5cc \uc815\uce58\"\uc758 \uc2dc\ub3c4\ub294 \uc218\ud3ec\ub85c \ub3cc\uc544\uac14\ub2e4. \ud558\uc9c0\ub9cc, \uc0c1\ub958\uce35 \uadc0\uc871\ub4e4\uc758 \uad8c\ub825\uc744 \uc57d\ud654\uc2dc\ud0a4\ub824\ub294 \ud0c0\ub9c8\ub974\uc758 \uccab \ud589\ubcf4\ub294 \uc644\uc804\ud788 \uc131\uacf5\uc744 \uac70\ub450\uc9c0 \ubabb\ud588\ub2e4. \uadf8\ub140\ub294 \uad50\ud68c \ud68c\uc758\ub97c \ud1b5\ud558\uc5ec \uac00\ud1a8\ub9ac\ucf54\uc2a4 \ubbf8\ucc28\uc5d8\uc744 \ud574\uc784\uc2dc\ud0a4\ub824 \ud588\uc73c\ub098 \uc2e4\ud328\ud588\uace0, \uadc0\uc871 \uc758\ud68c \ub2e4\ub974\ubc14\uc9c0\uac00 \uad6d\uc655 \ubc95\ub839\uc758 \uc2b9\uc778\uad8c\uc744 \uc8fc\uc7a5\ud588\ub2e4. \uadf8\ub140\uc758 \uccab \ubc88\uc9f8 \ub0a8\ud3b8 \ub9c8\ucc98\ub3c4 \uadc0\uc871\ub4e4\uc5d0 \uc758\ud558\uc5ec \uac15\uc694\ub41c \ub8e8\uc2a4\uc871 \uc655\uc790 \uc720\ub9ac \ubcf4\uace8\ub958\ud504\uc2a4\ud0a4\uc600\ub2e4.", "answer": "\uc870\uc9c0\uc544\uc5d0\uc11c\ub294 \uc5ec\uc131 \uad70\uc8fc\uac00 \ub2e8 \ud55c \uba85\ub3c4 \uc5c6\uc5c8\uae30 \ub54c\ubb38", "sentence": "\uadf8\ub140 \uc804\uae4c\uc9c0\ub9cc \ud574\ub3c4 \uc870\uc9c0\uc544\uc5d0\uc11c\ub294 \uc5ec\uc131 \uad70\uc8fc\uac00 \ub2e8 \ud55c \uba85\ub3c4 \uc5c6\uc5c8\uae30 \ub54c\ubb38 \uc5d0, \uadc0\uc871\ub4e4\uc758 \uc77c\ubd80\ub294 \ud0c0\ub9c8\ub974 \uc5ec\uc655\uc758 \uc801\ubc95\uc131\uc5d0 \ub300\ud574 \uc758\ubb38\uc744 \ud488\uc5c8\uc9c0\ub9cc, \ubc18\uba74\uc5d0 \ub2e4\ub978 \uc0ac\ub78c\ub4e4\uc740 \uadf8\ub140\uc758 \uc5b4\ub9b0 \uc810\uc744 \uc774\uc6a9\ud574 \uadf8\ub140\uc758 \uacb0\uc810\uc774 \uc624\ud788\ub824 \uadf8\ub4e4\uc758 \ub300\uc790\uce58\uad8c\uc744 \ud655\uace0\ud788 \ud574\uc904 \uc218 \uc788\uc744 \uac83\uc73c\ub85c \uc608\uc0c1\ud588\ub2e4.", "paragraph_sentence": "\ud0c0\ub9c8\ub974\ub294 6\ub144\ub3d9\uc548 \uadf8\uc758 \ubd80\uce5c\uc774 \uc8fd\ub358 1184\ub144\uae4c\uc9c0 \uc12d\uc815\ud588\uace0, \uc774\uc5b4\uc11c \ub2e8\uc77c \uad70\uc8fc\uac00 \ub418\uc5c8\uc73c\uba70, \uc870\uc9c0\uc544 \ub3d9\ubd80\uc758 \ucfe0\ud0c0\uc774\uc2dc \uadfc\uad50\uc5d0 \uc788\ub294 \uac94\ub77c\ud2f0 \uc218\ub3c4\uc6d0\uc5d0\uc11c \uc989\uc704\uc2dd\uc744 \uce58\ub800\ub2e4. \uadf8\ub140\ub294 \ube44\uad50\uc801 \uac15\ub825\ud55c \uc655\uad6d\uc744 \uc2b9\uacc4\ud588\uc9c0\ub9cc, \uace0\uc704\uae09 \uadc0\uc871\ub4e4\uc5d0 \uc758\ud574 \uc870\uc7a5\ub41c \uc911\uc559\uc9d1\uad8c\uc744 \ud750\ud2b8\ub824\ud2b8\ub824 \ub193\uc73c\ub824\ub294 \ud48d\uc870\ub294 \uac00\ub77c \uc549\uc744 \uc904\uc744 \ubab0\ub790\ub2e4. \ud0c0\ub9c8\ub974\uc758 \uacc4\uc2b9\uc744 \ubc18\ub300\ud558\ub358 \uc2ec\uc0c1\uce58 \uc54a\uc740 \uc790\ub4e4\uc774 \uc788\uc5c8\ub2e4. \uadf8\ub4e4\uc740 \uadf8\ub140\uc758 \ubd80\uce5c\uc774 \uadf8\ub4e4\uc744 \uc5b5\uc555\ud588\ub358 \uc815\ucc45\uc5d0 \ub300\ud574 \ubc18\uae30\ub97c \ub4e4\uc5b4 \uc62c\ub838\uc9c0\ub9cc, \uc0c8\ub85c\uc6b4 \uad70\uc8fc\uc758 \ucde8\uc57d\uc810, \uc989 \uc5ec\uc131\uc774\ub77c\ub294 \uc0ac\uc2e4\uc774 \uadf8\ub4e4\uc744 \ub354 \ubd80\ucd94\uacbc\ub358 \uac83\uc774\ub2e4. \uadf8\ub140 \uc804\uae4c\uc9c0\ub9cc \ud574\ub3c4 \uc870\uc9c0\uc544\uc5d0\uc11c\ub294 \uc5ec\uc131 \uad70\uc8fc\uac00 \ub2e8 \ud55c \uba85\ub3c4 \uc5c6\uc5c8\uae30 \ub54c\ubb38 \uc5d0, \uadc0\uc871\ub4e4\uc758 \uc77c\ubd80\ub294 \ud0c0\ub9c8\ub974 \uc5ec\uc655\uc758 \uc801\ubc95\uc131\uc5d0 \ub300\ud574 \uc758\ubb38\uc744 \ud488\uc5c8\uc9c0\ub9cc, \ubc18\uba74\uc5d0 \ub2e4\ub978 \uc0ac\ub78c\ub4e4\uc740 \uadf8\ub140\uc758 \uc5b4\ub9b0 \uc810\uc744 \uc774\uc6a9\ud574 \uadf8\ub140\uc758 \uacb0\uc810\uc774 \uc624\ud788\ub824 \uadf8\ub4e4\uc758 \ub300\uc790\uce58\uad8c\uc744 \ud655\uace0\ud788 \ud574\uc904 \uc218 \uc788\uc744 \uac83\uc73c\ub85c \uc608\uc0c1\ud588\ub2e4. \ud0c0\ub9c8\ub974\uc758 \uc601\ud5a5\ub825 \uc788\ub294 \uace0\ubaa8\uc778 \ub8e8\uc218\ub2e8 \uc5ec\uc655\uacfc \uc870\uc9c0\uc544\uc758 \uac00\ud1a8\ub9ac\ucf54\uc2a4 \ubbf8\ucc28\uc5d8 4\uc138 \ubbf8\ub9ac\uc544\ub2c8\uc2dc\uc81c\uc758 \uc5f4\uc815\uc801\uc778 \ub3c4\uc6c0\uc740 \uc870\uc9c0\uc544 \uc655\uad6d\uc744 \uc2b9\uacc4\ud55c \ud0c0\ub9c8\ub974 \uc5ec\uc655\uc744 \ud569\ubc95\ud654\ud558\ub294\ub370 \uacb0\uc815\uc801\uc778 \uc5ed\ud560\uc744 \ud588\ub2e4. \uadf8\ub140\ub294 \uac00\ud1a8\ub9ac\ucf54\uc2a4 \ubbf8\ucc28\uc5d8\uc758 \ub3c4\uc6c0\uc5d0 \ub300\ud55c \ubcf4\uc0c1\uc73c\ub85c \uadf8\ub97c \ub300\ubc95\uad00\uc5d0 \uc784\uba85\ud588\uace0, \uadf8\ub9ac\ud558\uc5ec \ubbf8\ucc28\uc5d8\uc740 \uc131\uc9c1\uacfc \uc138\uc18d\uc758 \uacf5\ubb34 \uccb4\uacc4 \uc591\ucabd \ubaa8\ub450\uc5d0\uc11c \uc218\uc7a5\uc758 \uc9c0\uc704\uc5d0 \uc624\ub974\uac8c \ub418\uc5c8\ub2e4. \ud0c0\ub9c8\ub974\ub294 \uadf8\ub140\uc758 \ubd80\uce5c\uc778 \uac8c\uc624\ub974\uac8c 3\uc138\uac00 \uadf8\uc5d0\uac8c \ub3c4\uc804\ud558\ub358 \uadc0\uc871\ub4e4\uc744 \uc5c4\uc911 \ucc98\ubc8c\ud558\ub294\ub370 \ub3c4\uc6c0\uc744 \uc92c\ub358 \uadf8\uc758 \ubd80\uce5c\uc774 \uc784\uba85\ud588\ub358 \uc0ac\ub78c\ub4e4 \uc911\uc5d0\uc11c \ub0ae\uc740 \uc2e0\ubd84 \ud0dc\uc0dd\uc758 \uc870\uc9c0\uc544\uc778\ud654 \ub41c \ud0b5\ucc28\ud06c\uc778\uc778 \ucfe0\ubc14\uc0ac\uc2a4 \ubcf4\uc548\uad00\uc758 \ud574\uc784\uc744 \ucd09\uad6c\ud558\uae30\ub3c4 \ud588\ub2e4. \uac8c\uc624\ub974\uac8c 3\uc138\uac00 \ube44\ucc9c\ud55c \uc2e0\ubd84\uc73c\ub85c\uc758 \uc219\uba85\uc744 \ubc97\uc5b4\ub0a0 \uc218 \uc788\uac8c \ud574\uc900 \uadf8\uc758 \ubb34\uba85\uc758 \uc2dc\uc911\ub4e4 \uc911\uc5d0 \ud55c\uba85\uc740 \uadf8 \ub54c\uc5d0 \uc0c8\ub85c\uc6b4 \uc758\ud68c\uc778 \uce74\ub77c\ube44\ub97c \uc870\uc9c1\ud574 \uc758\ud68c\ub9cc \ub530\ub85c \ub5a8\uc5b4\uc838 \uc2ec\uc0ac\uc219\uace0 \ud558\uc5ec \uc815\ucc45\uc744 \uacb0\uc815\ud588\ub358 \uadf8\ub807\uac8c \uc655\uad8c\uc744 \uc81c\ud55c\ud558\ub824\ub294 \ud22c\uc7c1 \uc911\uc5d0 \uc788\uc5c8\ub358 \uadc0\uc871 \ub2e8\uccb4\ub4e4\uacfc \ubd80\uc720\ud55c \uc2dc\ubbfc\ub4e4\uc744 \uc774\ub04c\ub358 \uc7ac\ubb34 \ub2f4\ub2f9\uc790 \ucfe0\ud2c0\ub8e8 \uc544\uc2ac\ub780\uc774\uc5c8\ub2e4. \ud0c0\ub9c8\ub974\uac00 \ucfe0\ud2c0\ub8e8 \uc544\uc2ac\ub780\uc744 \uccb4\ud3ec\ud558\uace0 \uadf8\uc758 \ucd94\uc885\uc790\ub4e4\uc744 \uad6c\uc2ac\ub824\uc11c \ud56d\ubcf5\ud558\uac8c \ud568\uc73c\ub85c \"\ubd09\uac74 \uc785\ud5cc \uc815\uce58\"\uc758 \uc2dc\ub3c4\ub294 \uc218\ud3ec\ub85c \ub3cc\uc544\uac14\ub2e4. \ud558\uc9c0\ub9cc, \uc0c1\ub958\uce35 \uadc0\uc871\ub4e4\uc758 \uad8c\ub825\uc744 \uc57d\ud654\uc2dc\ud0a4\ub824\ub294 \ud0c0\ub9c8\ub974\uc758 \uccab \ud589\ubcf4\ub294 \uc644\uc804\ud788 \uc131\uacf5\uc744 \uac70\ub450\uc9c0 \ubabb\ud588\ub2e4. \uadf8\ub140\ub294 \uad50\ud68c \ud68c\uc758\ub97c \ud1b5\ud558\uc5ec \uac00\ud1a8\ub9ac\ucf54\uc2a4 \ubbf8\ucc28\uc5d8\uc744 \ud574\uc784\uc2dc\ud0a4\ub824 \ud588\uc73c\ub098 \uc2e4\ud328\ud588\uace0, \uadc0\uc871 \uc758\ud68c \ub2e4\ub974\ubc14\uc9c0\uac00 \uad6d\uc655 \ubc95\ub839\uc758 \uc2b9\uc778\uad8c\uc744 \uc8fc\uc7a5\ud588\ub2e4. \uadf8\ub140\uc758 \uccab \ubc88\uc9f8 \ub0a8\ud3b8 \ub9c8\ucc98\ub3c4 \uadc0\uc871\ub4e4\uc5d0 \uc758\ud558\uc5ec \uac15\uc694\ub41c \ub8e8\uc2a4\uc871 \uc655\uc790 \uc720\ub9ac \ubcf4\uace8\ub958\ud504\uc2a4\ud0a4\uc600\ub2e4.", "paragraph_answer": "\ud0c0\ub9c8\ub974\ub294 6\ub144\ub3d9\uc548 \uadf8\uc758 \ubd80\uce5c\uc774 \uc8fd\ub358 1184\ub144\uae4c\uc9c0 \uc12d\uc815\ud588\uace0, \uc774\uc5b4\uc11c \ub2e8\uc77c \uad70\uc8fc\uac00 \ub418\uc5c8\uc73c\uba70, \uc870\uc9c0\uc544 \ub3d9\ubd80\uc758 \ucfe0\ud0c0\uc774\uc2dc \uadfc\uad50\uc5d0 \uc788\ub294 \uac94\ub77c\ud2f0 \uc218\ub3c4\uc6d0\uc5d0\uc11c \uc989\uc704\uc2dd\uc744 \uce58\ub800\ub2e4. \uadf8\ub140\ub294 \ube44\uad50\uc801 \uac15\ub825\ud55c \uc655\uad6d\uc744 \uc2b9\uacc4\ud588\uc9c0\ub9cc, \uace0\uc704\uae09 \uadc0\uc871\ub4e4\uc5d0 \uc758\ud574 \uc870\uc7a5\ub41c \uc911\uc559\uc9d1\uad8c\uc744 \ud750\ud2b8\ub824\ud2b8\ub824 \ub193\uc73c\ub824\ub294 \ud48d\uc870\ub294 \uac00\ub77c \uc549\uc744 \uc904\uc744 \ubab0\ub790\ub2e4. \ud0c0\ub9c8\ub974\uc758 \uacc4\uc2b9\uc744 \ubc18\ub300\ud558\ub358 \uc2ec\uc0c1\uce58 \uc54a\uc740 \uc790\ub4e4\uc774 \uc788\uc5c8\ub2e4. \uadf8\ub4e4\uc740 \uadf8\ub140\uc758 \ubd80\uce5c\uc774 \uadf8\ub4e4\uc744 \uc5b5\uc555\ud588\ub358 \uc815\ucc45\uc5d0 \ub300\ud574 \ubc18\uae30\ub97c \ub4e4\uc5b4 \uc62c\ub838\uc9c0\ub9cc, \uc0c8\ub85c\uc6b4 \uad70\uc8fc\uc758 \ucde8\uc57d\uc810, \uc989 \uc5ec\uc131\uc774\ub77c\ub294 \uc0ac\uc2e4\uc774 \uadf8\ub4e4\uc744 \ub354 \ubd80\ucd94\uacbc\ub358 \uac83\uc774\ub2e4. \uadf8\ub140 \uc804\uae4c\uc9c0\ub9cc \ud574\ub3c4 \uc870\uc9c0\uc544\uc5d0\uc11c\ub294 \uc5ec\uc131 \uad70\uc8fc\uac00 \ub2e8 \ud55c \uba85\ub3c4 \uc5c6\uc5c8\uae30 \ub54c\ubb38 \uc5d0, \uadc0\uc871\ub4e4\uc758 \uc77c\ubd80\ub294 \ud0c0\ub9c8\ub974 \uc5ec\uc655\uc758 \uc801\ubc95\uc131\uc5d0 \ub300\ud574 \uc758\ubb38\uc744 \ud488\uc5c8\uc9c0\ub9cc, \ubc18\uba74\uc5d0 \ub2e4\ub978 \uc0ac\ub78c\ub4e4\uc740 \uadf8\ub140\uc758 \uc5b4\ub9b0 \uc810\uc744 \uc774\uc6a9\ud574 \uadf8\ub140\uc758 \uacb0\uc810\uc774 \uc624\ud788\ub824 \uadf8\ub4e4\uc758 \ub300\uc790\uce58\uad8c\uc744 \ud655\uace0\ud788 \ud574\uc904 \uc218 \uc788\uc744 \uac83\uc73c\ub85c \uc608\uc0c1\ud588\ub2e4. \ud0c0\ub9c8\ub974\uc758 \uc601\ud5a5\ub825 \uc788\ub294 \uace0\ubaa8\uc778 \ub8e8\uc218\ub2e8 \uc5ec\uc655\uacfc \uc870\uc9c0\uc544\uc758 \uac00\ud1a8\ub9ac\ucf54\uc2a4 \ubbf8\ucc28\uc5d8 4\uc138 \ubbf8\ub9ac\uc544\ub2c8\uc2dc\uc81c\uc758 \uc5f4\uc815\uc801\uc778 \ub3c4\uc6c0\uc740 \uc870\uc9c0\uc544 \uc655\uad6d\uc744 \uc2b9\uacc4\ud55c \ud0c0\ub9c8\ub974 \uc5ec\uc655\uc744 \ud569\ubc95\ud654\ud558\ub294\ub370 \uacb0\uc815\uc801\uc778 \uc5ed\ud560\uc744 \ud588\ub2e4. \uadf8\ub140\ub294 \uac00\ud1a8\ub9ac\ucf54\uc2a4 \ubbf8\ucc28\uc5d8\uc758 \ub3c4\uc6c0\uc5d0 \ub300\ud55c \ubcf4\uc0c1\uc73c\ub85c \uadf8\ub97c \ub300\ubc95\uad00\uc5d0 \uc784\uba85\ud588\uace0, \uadf8\ub9ac\ud558\uc5ec \ubbf8\ucc28\uc5d8\uc740 \uc131\uc9c1\uacfc \uc138\uc18d\uc758 \uacf5\ubb34 \uccb4\uacc4 \uc591\ucabd \ubaa8\ub450\uc5d0\uc11c \uc218\uc7a5\uc758 \uc9c0\uc704\uc5d0 \uc624\ub974\uac8c \ub418\uc5c8\ub2e4. \ud0c0\ub9c8\ub974\ub294 \uadf8\ub140\uc758 \ubd80\uce5c\uc778 \uac8c\uc624\ub974\uac8c 3\uc138\uac00 \uadf8\uc5d0\uac8c \ub3c4\uc804\ud558\ub358 \uadc0\uc871\ub4e4\uc744 \uc5c4\uc911 \ucc98\ubc8c\ud558\ub294\ub370 \ub3c4\uc6c0\uc744 \uc92c\ub358 \uadf8\uc758 \ubd80\uce5c\uc774 \uc784\uba85\ud588\ub358 \uc0ac\ub78c\ub4e4 \uc911\uc5d0\uc11c \ub0ae\uc740 \uc2e0\ubd84 \ud0dc\uc0dd\uc758 \uc870\uc9c0\uc544\uc778\ud654 \ub41c \ud0b5\ucc28\ud06c\uc778\uc778 \ucfe0\ubc14\uc0ac\uc2a4 \ubcf4\uc548\uad00\uc758 \ud574\uc784\uc744 \ucd09\uad6c\ud558\uae30\ub3c4 \ud588\ub2e4. \uac8c\uc624\ub974\uac8c 3\uc138\uac00 \ube44\ucc9c\ud55c \uc2e0\ubd84\uc73c\ub85c\uc758 \uc219\uba85\uc744 \ubc97\uc5b4\ub0a0 \uc218 \uc788\uac8c \ud574\uc900 \uadf8\uc758 \ubb34\uba85\uc758 \uc2dc\uc911\ub4e4 \uc911\uc5d0 \ud55c\uba85\uc740 \uadf8 \ub54c\uc5d0 \uc0c8\ub85c\uc6b4 \uc758\ud68c\uc778 \uce74\ub77c\ube44\ub97c \uc870\uc9c1\ud574 \uc758\ud68c\ub9cc \ub530\ub85c \ub5a8\uc5b4\uc838 \uc2ec\uc0ac\uc219\uace0 \ud558\uc5ec \uc815\ucc45\uc744 \uacb0\uc815\ud588\ub358 \uadf8\ub807\uac8c \uc655\uad8c\uc744 \uc81c\ud55c\ud558\ub824\ub294 \ud22c\uc7c1 \uc911\uc5d0 \uc788\uc5c8\ub358 \uadc0\uc871 \ub2e8\uccb4\ub4e4\uacfc \ubd80\uc720\ud55c \uc2dc\ubbfc\ub4e4\uc744 \uc774\ub04c\ub358 \uc7ac\ubb34 \ub2f4\ub2f9\uc790 \ucfe0\ud2c0\ub8e8 \uc544\uc2ac\ub780\uc774\uc5c8\ub2e4. \ud0c0\ub9c8\ub974\uac00 \ucfe0\ud2c0\ub8e8 \uc544\uc2ac\ub780\uc744 \uccb4\ud3ec\ud558\uace0 \uadf8\uc758 \ucd94\uc885\uc790\ub4e4\uc744 \uad6c\uc2ac\ub824\uc11c \ud56d\ubcf5\ud558\uac8c \ud568\uc73c\ub85c \"\ubd09\uac74 \uc785\ud5cc \uc815\uce58\"\uc758 \uc2dc\ub3c4\ub294 \uc218\ud3ec\ub85c \ub3cc\uc544\uac14\ub2e4. \ud558\uc9c0\ub9cc, \uc0c1\ub958\uce35 \uadc0\uc871\ub4e4\uc758 \uad8c\ub825\uc744 \uc57d\ud654\uc2dc\ud0a4\ub824\ub294 \ud0c0\ub9c8\ub974\uc758 \uccab \ud589\ubcf4\ub294 \uc644\uc804\ud788 \uc131\uacf5\uc744 \uac70\ub450\uc9c0 \ubabb\ud588\ub2e4. \uadf8\ub140\ub294 \uad50\ud68c \ud68c\uc758\ub97c \ud1b5\ud558\uc5ec \uac00\ud1a8\ub9ac\ucf54\uc2a4 \ubbf8\ucc28\uc5d8\uc744 \ud574\uc784\uc2dc\ud0a4\ub824 \ud588\uc73c\ub098 \uc2e4\ud328\ud588\uace0, \uadc0\uc871 \uc758\ud68c \ub2e4\ub974\ubc14\uc9c0\uac00 \uad6d\uc655 \ubc95\ub839\uc758 \uc2b9\uc778\uad8c\uc744 \uc8fc\uc7a5\ud588\ub2e4. \uadf8\ub140\uc758 \uccab \ubc88\uc9f8 \ub0a8\ud3b8 \ub9c8\ucc98\ub3c4 \uadc0\uc871\ub4e4\uc5d0 \uc758\ud558\uc5ec \uac15\uc694\ub41c \ub8e8\uc2a4\uc871 \uc655\uc790 \uc720\ub9ac \ubcf4\uace8\ub958\ud504\uc2a4\ud0a4\uc600\ub2e4.", "sentence_answer": "\uadf8\ub140 \uc804\uae4c\uc9c0\ub9cc \ud574\ub3c4 \uc870\uc9c0\uc544\uc5d0\uc11c\ub294 \uc5ec\uc131 \uad70\uc8fc\uac00 \ub2e8 \ud55c \uba85\ub3c4 \uc5c6\uc5c8\uae30 \ub54c\ubb38 \uc5d0, \uadc0\uc871\ub4e4\uc758 \uc77c\ubd80\ub294 \ud0c0\ub9c8\ub974 \uc5ec\uc655\uc758 \uc801\ubc95\uc131\uc5d0 \ub300\ud574 \uc758\ubb38\uc744 \ud488\uc5c8\uc9c0\ub9cc, \ubc18\uba74\uc5d0 \ub2e4\ub978 \uc0ac\ub78c\ub4e4\uc740 \uadf8\ub140\uc758 \uc5b4\ub9b0 \uc810\uc744 \uc774\uc6a9\ud574 \uadf8\ub140\uc758 \uacb0\uc810\uc774 \uc624\ud788\ub824 \uadf8\ub4e4\uc758 \ub300\uc790\uce58\uad8c\uc744 \ud655\uace0\ud788 \ud574\uc904 \uc218 \uc788\uc744 \uac83\uc73c\ub85c \uc608\uc0c1\ud588\ub2e4.", "paragraph_id": "6192313-1-1", "paragraph_question": "question: \ud0c0\ub9c8\ub974\uc758 \uacc4\uc2b9\uc744 \ubc18\ub300\ud588\ub358 \ubc18\ub300\ud30c\ub4e4\uc758 \uc8fc\uc694 \uc774\uc720\ub294?, context: \ud0c0\ub9c8\ub974\ub294 6\ub144\ub3d9\uc548 \uadf8\uc758 \ubd80\uce5c\uc774 \uc8fd\ub358 1184\ub144\uae4c\uc9c0 \uc12d\uc815\ud588\uace0, \uc774\uc5b4\uc11c \ub2e8\uc77c \uad70\uc8fc\uac00 \ub418\uc5c8\uc73c\uba70, \uc870\uc9c0\uc544 \ub3d9\ubd80\uc758 \ucfe0\ud0c0\uc774\uc2dc \uadfc\uad50\uc5d0 \uc788\ub294 \uac94\ub77c\ud2f0 \uc218\ub3c4\uc6d0\uc5d0\uc11c \uc989\uc704\uc2dd\uc744 \uce58\ub800\ub2e4. \uadf8\ub140\ub294 \ube44\uad50\uc801 \uac15\ub825\ud55c \uc655\uad6d\uc744 \uc2b9\uacc4\ud588\uc9c0\ub9cc, \uace0\uc704\uae09 \uadc0\uc871\ub4e4\uc5d0 \uc758\ud574 \uc870\uc7a5\ub41c \uc911\uc559\uc9d1\uad8c\uc744 \ud750\ud2b8\ub824\ud2b8\ub824 \ub193\uc73c\ub824\ub294 \ud48d\uc870\ub294 \uac00\ub77c \uc549\uc744 \uc904\uc744 \ubab0\ub790\ub2e4. \ud0c0\ub9c8\ub974\uc758 \uacc4\uc2b9\uc744 \ubc18\ub300\ud558\ub358 \uc2ec\uc0c1\uce58 \uc54a\uc740 \uc790\ub4e4\uc774 \uc788\uc5c8\ub2e4. \uadf8\ub4e4\uc740 \uadf8\ub140\uc758 \ubd80\uce5c\uc774 \uadf8\ub4e4\uc744 \uc5b5\uc555\ud588\ub358 \uc815\ucc45\uc5d0 \ub300\ud574 \ubc18\uae30\ub97c \ub4e4\uc5b4 \uc62c\ub838\uc9c0\ub9cc, \uc0c8\ub85c\uc6b4 \uad70\uc8fc\uc758 \ucde8\uc57d\uc810, \uc989 \uc5ec\uc131\uc774\ub77c\ub294 \uc0ac\uc2e4\uc774 \uadf8\ub4e4\uc744 \ub354 \ubd80\ucd94\uacbc\ub358 \uac83\uc774\ub2e4. \uadf8\ub140 \uc804\uae4c\uc9c0\ub9cc \ud574\ub3c4 \uc870\uc9c0\uc544\uc5d0\uc11c\ub294 \uc5ec\uc131 \uad70\uc8fc\uac00 \ub2e8 \ud55c \uba85\ub3c4 \uc5c6\uc5c8\uae30 \ub54c\ubb38\uc5d0, \uadc0\uc871\ub4e4\uc758 \uc77c\ubd80\ub294 \ud0c0\ub9c8\ub974 \uc5ec\uc655\uc758 \uc801\ubc95\uc131\uc5d0 \ub300\ud574 \uc758\ubb38\uc744 \ud488\uc5c8\uc9c0\ub9cc, \ubc18\uba74\uc5d0 \ub2e4\ub978 \uc0ac\ub78c\ub4e4\uc740 \uadf8\ub140\uc758 \uc5b4\ub9b0 \uc810\uc744 \uc774\uc6a9\ud574 \uadf8\ub140\uc758 \uacb0\uc810\uc774 \uc624\ud788\ub824 \uadf8\ub4e4\uc758 \ub300\uc790\uce58\uad8c\uc744 \ud655\uace0\ud788 \ud574\uc904 \uc218 \uc788\uc744 \uac83\uc73c\ub85c \uc608\uc0c1\ud588\ub2e4. \ud0c0\ub9c8\ub974\uc758 \uc601\ud5a5\ub825 \uc788\ub294 \uace0\ubaa8\uc778 \ub8e8\uc218\ub2e8 \uc5ec\uc655\uacfc \uc870\uc9c0\uc544\uc758 \uac00\ud1a8\ub9ac\ucf54\uc2a4 \ubbf8\ucc28\uc5d8 4\uc138 \ubbf8\ub9ac\uc544\ub2c8\uc2dc\uc81c\uc758 \uc5f4\uc815\uc801\uc778 \ub3c4\uc6c0\uc740 \uc870\uc9c0\uc544 \uc655\uad6d\uc744 \uc2b9\uacc4\ud55c \ud0c0\ub9c8\ub974 \uc5ec\uc655\uc744 \ud569\ubc95\ud654\ud558\ub294\ub370 \uacb0\uc815\uc801\uc778 \uc5ed\ud560\uc744 \ud588\ub2e4. \uadf8\ub140\ub294 \uac00\ud1a8\ub9ac\ucf54\uc2a4 \ubbf8\ucc28\uc5d8\uc758 \ub3c4\uc6c0\uc5d0 \ub300\ud55c \ubcf4\uc0c1\uc73c\ub85c \uadf8\ub97c \ub300\ubc95\uad00\uc5d0 \uc784\uba85\ud588\uace0, \uadf8\ub9ac\ud558\uc5ec \ubbf8\ucc28\uc5d8\uc740 \uc131\uc9c1\uacfc \uc138\uc18d\uc758 \uacf5\ubb34 \uccb4\uacc4 \uc591\ucabd \ubaa8\ub450\uc5d0\uc11c \uc218\uc7a5\uc758 \uc9c0\uc704\uc5d0 \uc624\ub974\uac8c \ub418\uc5c8\ub2e4. \ud0c0\ub9c8\ub974\ub294 \uadf8\ub140\uc758 \ubd80\uce5c\uc778 \uac8c\uc624\ub974\uac8c 3\uc138\uac00 \uadf8\uc5d0\uac8c \ub3c4\uc804\ud558\ub358 \uadc0\uc871\ub4e4\uc744 \uc5c4\uc911 \ucc98\ubc8c\ud558\ub294\ub370 \ub3c4\uc6c0\uc744 \uc92c\ub358 \uadf8\uc758 \ubd80\uce5c\uc774 \uc784\uba85\ud588\ub358 \uc0ac\ub78c\ub4e4 \uc911\uc5d0\uc11c \ub0ae\uc740 \uc2e0\ubd84 \ud0dc\uc0dd\uc758 \uc870\uc9c0\uc544\uc778\ud654 \ub41c \ud0b5\ucc28\ud06c\uc778\uc778 \ucfe0\ubc14\uc0ac\uc2a4 \ubcf4\uc548\uad00\uc758 \ud574\uc784\uc744 \ucd09\uad6c\ud558\uae30\ub3c4 \ud588\ub2e4. \uac8c\uc624\ub974\uac8c 3\uc138\uac00 \ube44\ucc9c\ud55c \uc2e0\ubd84\uc73c\ub85c\uc758 \uc219\uba85\uc744 \ubc97\uc5b4\ub0a0 \uc218 \uc788\uac8c \ud574\uc900 \uadf8\uc758 \ubb34\uba85\uc758 \uc2dc\uc911\ub4e4 \uc911\uc5d0 \ud55c\uba85\uc740 \uadf8 \ub54c\uc5d0 \uc0c8\ub85c\uc6b4 \uc758\ud68c\uc778 \uce74\ub77c\ube44\ub97c \uc870\uc9c1\ud574 \uc758\ud68c\ub9cc \ub530\ub85c \ub5a8\uc5b4\uc838 \uc2ec\uc0ac\uc219\uace0 \ud558\uc5ec \uc815\ucc45\uc744 \uacb0\uc815\ud588\ub358 \uadf8\ub807\uac8c \uc655\uad8c\uc744 \uc81c\ud55c\ud558\ub824\ub294 \ud22c\uc7c1 \uc911\uc5d0 \uc788\uc5c8\ub358 \uadc0\uc871 \ub2e8\uccb4\ub4e4\uacfc \ubd80\uc720\ud55c \uc2dc\ubbfc\ub4e4\uc744 \uc774\ub04c\ub358 \uc7ac\ubb34 \ub2f4\ub2f9\uc790 \ucfe0\ud2c0\ub8e8 \uc544\uc2ac\ub780\uc774\uc5c8\ub2e4. \ud0c0\ub9c8\ub974\uac00 \ucfe0\ud2c0\ub8e8 \uc544\uc2ac\ub780\uc744 \uccb4\ud3ec\ud558\uace0 \uadf8\uc758 \ucd94\uc885\uc790\ub4e4\uc744 \uad6c\uc2ac\ub824\uc11c \ud56d\ubcf5\ud558\uac8c \ud568\uc73c\ub85c \"\ubd09\uac74 \uc785\ud5cc \uc815\uce58\"\uc758 \uc2dc\ub3c4\ub294 \uc218\ud3ec\ub85c \ub3cc\uc544\uac14\ub2e4. \ud558\uc9c0\ub9cc, \uc0c1\ub958\uce35 \uadc0\uc871\ub4e4\uc758 \uad8c\ub825\uc744 \uc57d\ud654\uc2dc\ud0a4\ub824\ub294 \ud0c0\ub9c8\ub974\uc758 \uccab \ud589\ubcf4\ub294 \uc644\uc804\ud788 \uc131\uacf5\uc744 \uac70\ub450\uc9c0 \ubabb\ud588\ub2e4. \uadf8\ub140\ub294 \uad50\ud68c \ud68c\uc758\ub97c \ud1b5\ud558\uc5ec \uac00\ud1a8\ub9ac\ucf54\uc2a4 \ubbf8\ucc28\uc5d8\uc744 \ud574\uc784\uc2dc\ud0a4\ub824 \ud588\uc73c\ub098 \uc2e4\ud328\ud588\uace0, \uadc0\uc871 \uc758\ud68c \ub2e4\ub974\ubc14\uc9c0\uac00 \uad6d\uc655 \ubc95\ub839\uc758 \uc2b9\uc778\uad8c\uc744 \uc8fc\uc7a5\ud588\ub2e4. \uadf8\ub140\uc758 \uccab \ubc88\uc9f8 \ub0a8\ud3b8 \ub9c8\ucc98\ub3c4 \uadc0\uc871\ub4e4\uc5d0 \uc758\ud558\uc5ec \uac15\uc694\ub41c \ub8e8\uc2a4\uc871 \uc655\uc790 \uc720\ub9ac \ubcf4\uace8\ub958\ud504\uc2a4\ud0a4\uc600\ub2e4."} {"question": "\ub85c\ub10c\uc774 \ud55c\ub54c \ub2e4\ub154\ub358 \ud559\uad50\ub294?", "paragraph": "\ub85c\ub10c\uc740 1994\ub144 4\uc6d4 12\uc77c, \ubbf8\uad6d \ub274\uc695 \uc8fc, \ube0c\ub871\ud06c\uc2a4\uc5d0\uc11c \ud0dc\uc5b4\ub0ac\ub2e4. \ub85c\ub10c\uc740 \uc544\uc77c\ub79c\ub4dc\uc778\uc758 \ubd80\ubaa8 \ud3f4 \ub85c\ub10c\uacfc \ubaa8\ub2c8\uce74 \ub85c\ub10c (\uc61b \uc131\uc528\ub294 \ube0c\ub808\ub10c)\uc758 \uc678\ub3d9\ub538\uc774\ub2e4. \uc774\ub4e4\uc740 \ubaa8\ub450 \uc544\uc77c\ub79c\ub4dc \ub354\ube14\ub9b0 \ucd9c\uc2e0\uc73c\ub85c 1980\ub144\ub300 \ud6c4\ubc18 \ubbf8\uad6d\uc73c\ub85c \uc774\uc8fc\ud588\ub2e4. \uadf8\ub140\uc758 \uc544\ubc84\uc9c0\ub294 \ub274\uc695\uc5d0\uc11c \ubc30\uc6b0\ub85c \ud65c\ub3d9\ud558\uae30 \uc804\uc5d0 \uac74\uc124\uc5c5\uacfc \ubc14\uc5d0\uc11c \uc77c\ud588\uc73c\uba70, \uadf8\ub140\uc758 \uc5b4\uba38\ub2c8\ub294 \uac8c\uc774\uc5b4\ud2f0\uadf9\uc7a5\uc5d0\uc11c \ubc30\uc6b0\ub85c \uc77c\ud588\uc73c\ub098 \uc774\ud6c4\uc5d0\ub294 \ud65c\ub3d9\ud558\uc9c0 \uc54a\uace0 \uc788\ub2e4. \ub85c\ub10c\uc758 \uac00\uc871\uc740 \uadf8\ub140\uac00 3\uc0b4 \ub54c \uc544\uc77c\ub79c\ub4dc \ub354\ube14\ub9b0\uc73c\ub85c \uc774\uc8fc\ud588\ub2e4. \uadf8\ub140\ub294 \uce7c\ub85c \uc8fc\uc5d0\uc11c \uc0b4\uba70 \uc544\ub974\ub2e4\ud2f4 \uad6d\ub9bd\ud559\uad50\uc5d0 \ub2e4\ub154\uc9c0\ub9cc, \uadf8\ub7ec\ub098 \uc774\ud6c4\uc5d0 \uac00\uc815\uad50\uc0ac\uc5d0 \uc758\ud574 \ud648\uc2a4\ucfe8\ub9c1\uc744 \ubc1b\uc558\ub2e4. 10\ub300 \ucd08\ubc18\uc5d0 \ub85c\ub10c\uc740 \ud574\ubcc0 \ub9c8\uc744 \ud638\uc2a4\uc5d0\uc11c \uc815\ucc29\ud574 \uc0b4\ub2e4 \ubd80\ubaa8\uc640 \ud568\uaed8 \ub354\ube14\ub9b0\uc73c\ub85c \ub2e4\uc2dc \ub3cc\uc544\uc654\ub2e4.", "answer": "\uc544\ub974\ub2e4\ud2f4 \uad6d\ub9bd\ud559\uad50", "sentence": "\uadf8\ub140\ub294 \uce7c\ub85c \uc8fc\uc5d0\uc11c \uc0b4\uba70 \uc544\ub974\ub2e4\ud2f4 \uad6d\ub9bd\ud559\uad50 \uc5d0", "paragraph_sentence": "\ub85c\ub10c\uc740 1994\ub144 4\uc6d4 12\uc77c, \ubbf8\uad6d \ub274\uc695 \uc8fc, \ube0c\ub871\ud06c\uc2a4\uc5d0\uc11c \ud0dc\uc5b4\ub0ac\ub2e4. \ub85c\ub10c\uc740 \uc544\uc77c\ub79c\ub4dc\uc778\uc758 \ubd80\ubaa8 \ud3f4 \ub85c\ub10c\uacfc \ubaa8\ub2c8\uce74 \ub85c\ub10c (\uc61b \uc131\uc528\ub294 \ube0c\ub808\ub10c)\uc758 \uc678\ub3d9\ub538\uc774\ub2e4. \uc774\ub4e4\uc740 \ubaa8\ub450 \uc544\uc77c\ub79c\ub4dc \ub354\ube14\ub9b0 \ucd9c\uc2e0\uc73c\ub85c 1980\ub144\ub300 \ud6c4\ubc18 \ubbf8\uad6d\uc73c\ub85c \uc774\uc8fc\ud588\ub2e4. \uadf8\ub140\uc758 \uc544\ubc84\uc9c0\ub294 \ub274\uc695\uc5d0\uc11c \ubc30\uc6b0\ub85c \ud65c\ub3d9\ud558\uae30 \uc804\uc5d0 \uac74\uc124\uc5c5\uacfc \ubc14\uc5d0\uc11c \uc77c\ud588\uc73c\uba70, \uadf8\ub140\uc758 \uc5b4\uba38\ub2c8\ub294 \uac8c\uc774\uc5b4\ud2f0\uadf9\uc7a5\uc5d0\uc11c \ubc30\uc6b0\ub85c \uc77c\ud588\uc73c\ub098 \uc774\ud6c4\uc5d0\ub294 \ud65c\ub3d9\ud558\uc9c0 \uc54a\uace0 \uc788\ub2e4. \ub85c\ub10c\uc758 \uac00\uc871\uc740 \uadf8\ub140\uac00 3\uc0b4 \ub54c \uc544\uc77c\ub79c\ub4dc \ub354\ube14\ub9b0\uc73c\ub85c \uc774\uc8fc\ud588\ub2e4. \uadf8\ub140\ub294 \uce7c\ub85c \uc8fc\uc5d0\uc11c \uc0b4\uba70 \uc544\ub974\ub2e4\ud2f4 \uad6d\ub9bd\ud559\uad50 \uc5d0 \ub2e4\ub154\uc9c0\ub9cc, \uadf8\ub7ec\ub098 \uc774\ud6c4\uc5d0 \uac00\uc815\uad50\uc0ac\uc5d0 \uc758\ud574 \ud648\uc2a4\ucfe8\ub9c1\uc744 \ubc1b\uc558\ub2e4. 10\ub300 \ucd08\ubc18\uc5d0 \ub85c\ub10c\uc740 \ud574\ubcc0 \ub9c8\uc744 \ud638\uc2a4\uc5d0\uc11c \uc815\ucc29\ud574 \uc0b4\ub2e4 \ubd80\ubaa8\uc640 \ud568\uaed8 \ub354\ube14\ub9b0\uc73c\ub85c \ub2e4\uc2dc \ub3cc\uc544\uc654\ub2e4.", "paragraph_answer": "\ub85c\ub10c\uc740 1994\ub144 4\uc6d4 12\uc77c, \ubbf8\uad6d \ub274\uc695 \uc8fc, \ube0c\ub871\ud06c\uc2a4\uc5d0\uc11c \ud0dc\uc5b4\ub0ac\ub2e4. \ub85c\ub10c\uc740 \uc544\uc77c\ub79c\ub4dc\uc778\uc758 \ubd80\ubaa8 \ud3f4 \ub85c\ub10c\uacfc \ubaa8\ub2c8\uce74 \ub85c\ub10c (\uc61b \uc131\uc528\ub294 \ube0c\ub808\ub10c)\uc758 \uc678\ub3d9\ub538\uc774\ub2e4. \uc774\ub4e4\uc740 \ubaa8\ub450 \uc544\uc77c\ub79c\ub4dc \ub354\ube14\ub9b0 \ucd9c\uc2e0\uc73c\ub85c 1980\ub144\ub300 \ud6c4\ubc18 \ubbf8\uad6d\uc73c\ub85c \uc774\uc8fc\ud588\ub2e4. \uadf8\ub140\uc758 \uc544\ubc84\uc9c0\ub294 \ub274\uc695\uc5d0\uc11c \ubc30\uc6b0\ub85c \ud65c\ub3d9\ud558\uae30 \uc804\uc5d0 \uac74\uc124\uc5c5\uacfc \ubc14\uc5d0\uc11c \uc77c\ud588\uc73c\uba70, \uadf8\ub140\uc758 \uc5b4\uba38\ub2c8\ub294 \uac8c\uc774\uc5b4\ud2f0\uadf9\uc7a5\uc5d0\uc11c \ubc30\uc6b0\ub85c \uc77c\ud588\uc73c\ub098 \uc774\ud6c4\uc5d0\ub294 \ud65c\ub3d9\ud558\uc9c0 \uc54a\uace0 \uc788\ub2e4. \ub85c\ub10c\uc758 \uac00\uc871\uc740 \uadf8\ub140\uac00 3\uc0b4 \ub54c \uc544\uc77c\ub79c\ub4dc \ub354\ube14\ub9b0\uc73c\ub85c \uc774\uc8fc\ud588\ub2e4. \uadf8\ub140\ub294 \uce7c\ub85c \uc8fc\uc5d0\uc11c \uc0b4\uba70 \uc544\ub974\ub2e4\ud2f4 \uad6d\ub9bd\ud559\uad50 \uc5d0 \ub2e4\ub154\uc9c0\ub9cc, \uadf8\ub7ec\ub098 \uc774\ud6c4\uc5d0 \uac00\uc815\uad50\uc0ac\uc5d0 \uc758\ud574 \ud648\uc2a4\ucfe8\ub9c1\uc744 \ubc1b\uc558\ub2e4. 10\ub300 \ucd08\ubc18\uc5d0 \ub85c\ub10c\uc740 \ud574\ubcc0 \ub9c8\uc744 \ud638\uc2a4\uc5d0\uc11c \uc815\ucc29\ud574 \uc0b4\ub2e4 \ubd80\ubaa8\uc640 \ud568\uaed8 \ub354\ube14\ub9b0\uc73c\ub85c \ub2e4\uc2dc \ub3cc\uc544\uc654\ub2e4.", "sentence_answer": "\uadf8\ub140\ub294 \uce7c\ub85c \uc8fc\uc5d0\uc11c \uc0b4\uba70 \uc544\ub974\ub2e4\ud2f4 \uad6d\ub9bd\ud559\uad50 \uc5d0", "paragraph_id": "6457389-1-1", "paragraph_question": "question: \ub85c\ub10c\uc774 \ud55c\ub54c \ub2e4\ub154\ub358 \ud559\uad50\ub294?, context: \ub85c\ub10c\uc740 1994\ub144 4\uc6d4 12\uc77c, \ubbf8\uad6d \ub274\uc695 \uc8fc, \ube0c\ub871\ud06c\uc2a4\uc5d0\uc11c \ud0dc\uc5b4\ub0ac\ub2e4. \ub85c\ub10c\uc740 \uc544\uc77c\ub79c\ub4dc\uc778\uc758 \ubd80\ubaa8 \ud3f4 \ub85c\ub10c\uacfc \ubaa8\ub2c8\uce74 \ub85c\ub10c (\uc61b \uc131\uc528\ub294 \ube0c\ub808\ub10c)\uc758 \uc678\ub3d9\ub538\uc774\ub2e4. \uc774\ub4e4\uc740 \ubaa8\ub450 \uc544\uc77c\ub79c\ub4dc \ub354\ube14\ub9b0 \ucd9c\uc2e0\uc73c\ub85c 1980\ub144\ub300 \ud6c4\ubc18 \ubbf8\uad6d\uc73c\ub85c \uc774\uc8fc\ud588\ub2e4. \uadf8\ub140\uc758 \uc544\ubc84\uc9c0\ub294 \ub274\uc695\uc5d0\uc11c \ubc30\uc6b0\ub85c \ud65c\ub3d9\ud558\uae30 \uc804\uc5d0 \uac74\uc124\uc5c5\uacfc \ubc14\uc5d0\uc11c \uc77c\ud588\uc73c\uba70, \uadf8\ub140\uc758 \uc5b4\uba38\ub2c8\ub294 \uac8c\uc774\uc5b4\ud2f0\uadf9\uc7a5\uc5d0\uc11c \ubc30\uc6b0\ub85c \uc77c\ud588\uc73c\ub098 \uc774\ud6c4\uc5d0\ub294 \ud65c\ub3d9\ud558\uc9c0 \uc54a\uace0 \uc788\ub2e4. \ub85c\ub10c\uc758 \uac00\uc871\uc740 \uadf8\ub140\uac00 3\uc0b4 \ub54c \uc544\uc77c\ub79c\ub4dc \ub354\ube14\ub9b0\uc73c\ub85c \uc774\uc8fc\ud588\ub2e4. \uadf8\ub140\ub294 \uce7c\ub85c \uc8fc\uc5d0\uc11c \uc0b4\uba70 \uc544\ub974\ub2e4\ud2f4 \uad6d\ub9bd\ud559\uad50\uc5d0 \ub2e4\ub154\uc9c0\ub9cc, \uadf8\ub7ec\ub098 \uc774\ud6c4\uc5d0 \uac00\uc815\uad50\uc0ac\uc5d0 \uc758\ud574 \ud648\uc2a4\ucfe8\ub9c1\uc744 \ubc1b\uc558\ub2e4. 10\ub300 \ucd08\ubc18\uc5d0 \ub85c\ub10c\uc740 \ud574\ubcc0 \ub9c8\uc744 \ud638\uc2a4\uc5d0\uc11c \uc815\ucc29\ud574 \uc0b4\ub2e4 \ubd80\ubaa8\uc640 \ud568\uaed8 \ub354\ube14\ub9b0\uc73c\ub85c \ub2e4\uc2dc \ub3cc\uc544\uc654\ub2e4."} {"question": "\ucd5c\uc9c4\uc2e4\uc774 \uc2a4\uc2a4\ub85c \ubaa9\uc228\uc744 \ub04a\uc740 \ub54c\ub294?", "paragraph": "2008\ub144 9\uc6d4 30\uc77c, \ucd5c\uc9c4\uc2e4\uc740 \uce5c\uad6c\uc778 \ud038\uc758 \uae40\uc7ac\uc6b0 \uae30\uc790\uc5d0\uac8c \"\uc0ac\ucc44\uc124 \uc720\ud3ec\uc790\uac00 \uc7a1\ud614\uc73c\ub2c8 \uc774\uc81c\ub294 \uc0ac\ub78c\ub4e4\uc774 \ub0b4 \uc9c4\uc2e4\uc744 \ubbff\uc5b4\uc904 \uc904 \uc54c\uc558\uc5b4. \uadf8\ub7f0\ub370 \ud1a0\ub860 \uac8c\uc2dc\ud310\uc5d0\uc11c \ub0b4 \uc774\uc57c\uae30\uc5d0 \uc11c\ub85c \ucc2c\ubc18\uc591\ub860\uc744 \ud3bc\uce58\ub294\ub370, \uc544\uc9c1\ub3c4 \ub0b4\uac00 \uc548\uc7ac\ud658\uc744 \uc8fd\uc74c\uc73c\ub85c \ubab0\uace0 \uac04 \uc545\ub355 \uc0ac\ucc44\uc5c5\uc790\ub85c \uadf8\ub300\ub85c \ubb35\uc778\ud558\ub294 \uc0ac\ub78c\ub4e4\uc774 \ub9ce\ub354\ub77c. \ub098 \ub108\ubb34 \ub5a8\ub824. \uadf8 \uac8c\uc2dc\ud310\uc5d0 \uae00\uc744 \ub0a8\uae30\uace0 \uc2f6\uc5b4. \ub098 \uc774\uc81c \ub354 \uc774\uc0c1 \ucc38\uc744 \uc218\uac00 \uc5c6\uc5b4. \uadf8\ub0e5 \uadf8 \uc0ac\ub78c\ub4e4\ud558\uace0 \ud53c \ud130\uc9c0\uac8c \uc2f8\uc6c0\uc744 \ud558\uace0 \uc2f6\uc5b4. \uc774\ub300\ub85c \uac00\ub2e4\uac04 \uc815\ub9d0 \uc8fd\uc744 \uac83 \uac19\uc544\"\ub77c\uace0 \ub9d0\ud588\ub2e4. \ucd5c\uc9c4\uc2e4\uc740 \ud1a0\ub860\uc744 \ud1b5\ud574 '\uc5ec\ub860\uc7ac\ud310'\uc744 \ub17c\ud30c\ud558\ub824\uace0 \ud588\ub358 \uac83\uc774\ub2e4. \uc774\ud6c4 \ucd5c\uc9c4\uc2e4\uc740 \uae40\uc7ac\uc6b0\uc5d0\uac8c \"\uc218\ub9ce\uc740 \uc545\ud50c\ub7ec\ub4e4\uacfc \uc2f8\uc6b0\uae30\uc5d0\ub294 \ub098 \uc790\uc2e0\uc774 \uadf8 \uc21c\uac04 \ub108\ubb34\ub3c4 \uc57d\ud55c \uc874\uc7ac\ub77c\ub294 \uac83\uc744 \ubc14\ub85c \ub290\uaf08\uace0 \uc624\ud574\uc758 \ubcbd\uc740 \ub108\ubb34\ub3c4 \ub192\uc544 \uc5b4\ub5bb\uac8c \uc190\uc744 \uc4f8 \uc218\uac00 \uc5c6\uc5b4 \ub2e8 \ud55c \uc904\uc758 \uae00\ub3c4 \uc62c\ub9b4 \uc218 \uc5c6\uc5c8\ub2e4\"\ub77c\uace0 \ub9d0\ud588\ub2e4. 2008\ub144 10\uc6d4 2\uc77c, \ucd5c\uc9c4\uc2e4\uc740 \ub9c8\ub140\uc0ac\ub0e5\uc2dd '\uc5ec\ub860\uc7ac\ud310'\uc758 \uace0\ud1b5\uc744 \ud1a0\ub85c\ud558\ub2e4\uac00 \uacb0\uad6d \uc2a4\uc2a4\ub85c \ubaa9\uc228\uc744 \ub04a\uc5c8\ub2e4.", "answer": "2008\ub144 10\uc6d4 2\uc77c", "sentence": "2008\ub144 10\uc6d4 2\uc77c ,", "paragraph_sentence": "2008\ub144 9\uc6d4 30\uc77c, \ucd5c\uc9c4\uc2e4\uc740 \uce5c\uad6c\uc778 \ud038\uc758 \uae40\uc7ac\uc6b0 \uae30\uc790\uc5d0\uac8c \"\uc0ac\ucc44\uc124 \uc720\ud3ec\uc790\uac00 \uc7a1\ud614\uc73c\ub2c8 \uc774\uc81c\ub294 \uc0ac\ub78c\ub4e4\uc774 \ub0b4 \uc9c4\uc2e4\uc744 \ubbff\uc5b4\uc904 \uc904 \uc54c\uc558\uc5b4. \uadf8\ub7f0\ub370 \ud1a0\ub860 \uac8c\uc2dc\ud310\uc5d0\uc11c \ub0b4 \uc774\uc57c\uae30\uc5d0 \uc11c\ub85c \ucc2c\ubc18\uc591\ub860\uc744 \ud3bc\uce58\ub294\ub370, \uc544\uc9c1\ub3c4 \ub0b4\uac00 \uc548\uc7ac\ud658\uc744 \uc8fd\uc74c\uc73c\ub85c \ubab0\uace0 \uac04 \uc545\ub355 \uc0ac\ucc44\uc5c5\uc790\ub85c \uadf8\ub300\ub85c \ubb35\uc778\ud558\ub294 \uc0ac\ub78c\ub4e4\uc774 \ub9ce\ub354\ub77c. \ub098 \ub108\ubb34 \ub5a8\ub824. \uadf8 \uac8c\uc2dc\ud310\uc5d0 \uae00\uc744 \ub0a8\uae30\uace0 \uc2f6\uc5b4. \ub098 \uc774\uc81c \ub354 \uc774\uc0c1 \ucc38\uc744 \uc218\uac00 \uc5c6\uc5b4. \uadf8\ub0e5 \uadf8 \uc0ac\ub78c\ub4e4\ud558\uace0 \ud53c \ud130\uc9c0\uac8c \uc2f8\uc6c0\uc744 \ud558\uace0 \uc2f6\uc5b4. \uc774\ub300\ub85c \uac00\ub2e4\uac04 \uc815\ub9d0 \uc8fd\uc744 \uac83 \uac19\uc544\"\ub77c\uace0 \ub9d0\ud588\ub2e4. \ucd5c\uc9c4\uc2e4\uc740 \ud1a0\ub860\uc744 \ud1b5\ud574 '\uc5ec\ub860\uc7ac\ud310'\uc744 \ub17c\ud30c\ud558\ub824\uace0 \ud588\ub358 \uac83\uc774\ub2e4. \uc774\ud6c4 \ucd5c\uc9c4\uc2e4\uc740 \uae40\uc7ac\uc6b0\uc5d0\uac8c \"\uc218\ub9ce\uc740 \uc545\ud50c\ub7ec\ub4e4\uacfc \uc2f8\uc6b0\uae30\uc5d0\ub294 \ub098 \uc790\uc2e0\uc774 \uadf8 \uc21c\uac04 \ub108\ubb34\ub3c4 \uc57d\ud55c \uc874\uc7ac\ub77c\ub294 \uac83\uc744 \ubc14\ub85c \ub290\uaf08\uace0 \uc624\ud574\uc758 \ubcbd\uc740 \ub108\ubb34\ub3c4 \ub192\uc544 \uc5b4\ub5bb\uac8c \uc190\uc744 \uc4f8 \uc218\uac00 \uc5c6\uc5b4 \ub2e8 \ud55c \uc904\uc758 \uae00\ub3c4 \uc62c\ub9b4 \uc218 \uc5c6\uc5c8\ub2e4\"\ub77c\uace0 \ub9d0\ud588\ub2e4. 2008\ub144 10\uc6d4 2\uc77c , \ucd5c\uc9c4\uc2e4\uc740 \ub9c8\ub140\uc0ac\ub0e5\uc2dd '\uc5ec\ub860\uc7ac\ud310'\uc758 \uace0\ud1b5\uc744 \ud1a0\ub85c\ud558\ub2e4\uac00 \uacb0\uad6d \uc2a4\uc2a4\ub85c \ubaa9\uc228\uc744 \ub04a\uc5c8\ub2e4.", "paragraph_answer": "2008\ub144 9\uc6d4 30\uc77c, \ucd5c\uc9c4\uc2e4\uc740 \uce5c\uad6c\uc778 \ud038\uc758 \uae40\uc7ac\uc6b0 \uae30\uc790\uc5d0\uac8c \"\uc0ac\ucc44\uc124 \uc720\ud3ec\uc790\uac00 \uc7a1\ud614\uc73c\ub2c8 \uc774\uc81c\ub294 \uc0ac\ub78c\ub4e4\uc774 \ub0b4 \uc9c4\uc2e4\uc744 \ubbff\uc5b4\uc904 \uc904 \uc54c\uc558\uc5b4. \uadf8\ub7f0\ub370 \ud1a0\ub860 \uac8c\uc2dc\ud310\uc5d0\uc11c \ub0b4 \uc774\uc57c\uae30\uc5d0 \uc11c\ub85c \ucc2c\ubc18\uc591\ub860\uc744 \ud3bc\uce58\ub294\ub370, \uc544\uc9c1\ub3c4 \ub0b4\uac00 \uc548\uc7ac\ud658\uc744 \uc8fd\uc74c\uc73c\ub85c \ubab0\uace0 \uac04 \uc545\ub355 \uc0ac\ucc44\uc5c5\uc790\ub85c \uadf8\ub300\ub85c \ubb35\uc778\ud558\ub294 \uc0ac\ub78c\ub4e4\uc774 \ub9ce\ub354\ub77c. \ub098 \ub108\ubb34 \ub5a8\ub824. \uadf8 \uac8c\uc2dc\ud310\uc5d0 \uae00\uc744 \ub0a8\uae30\uace0 \uc2f6\uc5b4. \ub098 \uc774\uc81c \ub354 \uc774\uc0c1 \ucc38\uc744 \uc218\uac00 \uc5c6\uc5b4. \uadf8\ub0e5 \uadf8 \uc0ac\ub78c\ub4e4\ud558\uace0 \ud53c \ud130\uc9c0\uac8c \uc2f8\uc6c0\uc744 \ud558\uace0 \uc2f6\uc5b4. \uc774\ub300\ub85c \uac00\ub2e4\uac04 \uc815\ub9d0 \uc8fd\uc744 \uac83 \uac19\uc544\"\ub77c\uace0 \ub9d0\ud588\ub2e4. \ucd5c\uc9c4\uc2e4\uc740 \ud1a0\ub860\uc744 \ud1b5\ud574 '\uc5ec\ub860\uc7ac\ud310'\uc744 \ub17c\ud30c\ud558\ub824\uace0 \ud588\ub358 \uac83\uc774\ub2e4. \uc774\ud6c4 \ucd5c\uc9c4\uc2e4\uc740 \uae40\uc7ac\uc6b0\uc5d0\uac8c \"\uc218\ub9ce\uc740 \uc545\ud50c\ub7ec\ub4e4\uacfc \uc2f8\uc6b0\uae30\uc5d0\ub294 \ub098 \uc790\uc2e0\uc774 \uadf8 \uc21c\uac04 \ub108\ubb34\ub3c4 \uc57d\ud55c \uc874\uc7ac\ub77c\ub294 \uac83\uc744 \ubc14\ub85c \ub290\uaf08\uace0 \uc624\ud574\uc758 \ubcbd\uc740 \ub108\ubb34\ub3c4 \ub192\uc544 \uc5b4\ub5bb\uac8c \uc190\uc744 \uc4f8 \uc218\uac00 \uc5c6\uc5b4 \ub2e8 \ud55c \uc904\uc758 \uae00\ub3c4 \uc62c\ub9b4 \uc218 \uc5c6\uc5c8\ub2e4\"\ub77c\uace0 \ub9d0\ud588\ub2e4. 2008\ub144 10\uc6d4 2\uc77c , \ucd5c\uc9c4\uc2e4\uc740 \ub9c8\ub140\uc0ac\ub0e5\uc2dd '\uc5ec\ub860\uc7ac\ud310'\uc758 \uace0\ud1b5\uc744 \ud1a0\ub85c\ud558\ub2e4\uac00 \uacb0\uad6d \uc2a4\uc2a4\ub85c \ubaa9\uc228\uc744 \ub04a\uc5c8\ub2e4.", "sentence_answer": " 2008\ub144 10\uc6d4 2\uc77c ,", "paragraph_id": "6560235-28-1", "paragraph_question": "question: \ucd5c\uc9c4\uc2e4\uc774 \uc2a4\uc2a4\ub85c \ubaa9\uc228\uc744 \ub04a\uc740 \ub54c\ub294?, context: 2008\ub144 9\uc6d4 30\uc77c, \ucd5c\uc9c4\uc2e4\uc740 \uce5c\uad6c\uc778 \ud038\uc758 \uae40\uc7ac\uc6b0 \uae30\uc790\uc5d0\uac8c \"\uc0ac\ucc44\uc124 \uc720\ud3ec\uc790\uac00 \uc7a1\ud614\uc73c\ub2c8 \uc774\uc81c\ub294 \uc0ac\ub78c\ub4e4\uc774 \ub0b4 \uc9c4\uc2e4\uc744 \ubbff\uc5b4\uc904 \uc904 \uc54c\uc558\uc5b4. \uadf8\ub7f0\ub370 \ud1a0\ub860 \uac8c\uc2dc\ud310\uc5d0\uc11c \ub0b4 \uc774\uc57c\uae30\uc5d0 \uc11c\ub85c \ucc2c\ubc18\uc591\ub860\uc744 \ud3bc\uce58\ub294\ub370, \uc544\uc9c1\ub3c4 \ub0b4\uac00 \uc548\uc7ac\ud658\uc744 \uc8fd\uc74c\uc73c\ub85c \ubab0\uace0 \uac04 \uc545\ub355 \uc0ac\ucc44\uc5c5\uc790\ub85c \uadf8\ub300\ub85c \ubb35\uc778\ud558\ub294 \uc0ac\ub78c\ub4e4\uc774 \ub9ce\ub354\ub77c. \ub098 \ub108\ubb34 \ub5a8\ub824. \uadf8 \uac8c\uc2dc\ud310\uc5d0 \uae00\uc744 \ub0a8\uae30\uace0 \uc2f6\uc5b4. \ub098 \uc774\uc81c \ub354 \uc774\uc0c1 \ucc38\uc744 \uc218\uac00 \uc5c6\uc5b4. \uadf8\ub0e5 \uadf8 \uc0ac\ub78c\ub4e4\ud558\uace0 \ud53c \ud130\uc9c0\uac8c \uc2f8\uc6c0\uc744 \ud558\uace0 \uc2f6\uc5b4. \uc774\ub300\ub85c \uac00\ub2e4\uac04 \uc815\ub9d0 \uc8fd\uc744 \uac83 \uac19\uc544\"\ub77c\uace0 \ub9d0\ud588\ub2e4. \ucd5c\uc9c4\uc2e4\uc740 \ud1a0\ub860\uc744 \ud1b5\ud574 '\uc5ec\ub860\uc7ac\ud310'\uc744 \ub17c\ud30c\ud558\ub824\uace0 \ud588\ub358 \uac83\uc774\ub2e4. \uc774\ud6c4 \ucd5c\uc9c4\uc2e4\uc740 \uae40\uc7ac\uc6b0\uc5d0\uac8c \"\uc218\ub9ce\uc740 \uc545\ud50c\ub7ec\ub4e4\uacfc \uc2f8\uc6b0\uae30\uc5d0\ub294 \ub098 \uc790\uc2e0\uc774 \uadf8 \uc21c\uac04 \ub108\ubb34\ub3c4 \uc57d\ud55c \uc874\uc7ac\ub77c\ub294 \uac83\uc744 \ubc14\ub85c \ub290\uaf08\uace0 \uc624\ud574\uc758 \ubcbd\uc740 \ub108\ubb34\ub3c4 \ub192\uc544 \uc5b4\ub5bb\uac8c \uc190\uc744 \uc4f8 \uc218\uac00 \uc5c6\uc5b4 \ub2e8 \ud55c \uc904\uc758 \uae00\ub3c4 \uc62c\ub9b4 \uc218 \uc5c6\uc5c8\ub2e4\"\ub77c\uace0 \ub9d0\ud588\ub2e4. 2008\ub144 10\uc6d4 2\uc77c, \ucd5c\uc9c4\uc2e4\uc740 \ub9c8\ub140\uc0ac\ub0e5\uc2dd '\uc5ec\ub860\uc7ac\ud310'\uc758 \uace0\ud1b5\uc744 \ud1a0\ub85c\ud558\ub2e4\uac00 \uacb0\uad6d \uc2a4\uc2a4\ub85c \ubaa9\uc228\uc744 \ub04a\uc5c8\ub2e4."} {"question": "\uc790\uc0b4\uacfc \uad00\ub828\ud558\uc5ec \uc694\uc778\uc5d0 \ub530\ub77c \uc704\ud5d8\ub3c4\ub97c \ubd84\ub958\ud558\uc5ec \uace0\uc704\ud5d8\uad70 \ubd84\ub958 \ud504\ub808\uc784\uc6cc\ud06c\ub97c \ub9cc\ub4e0 \uc0ac\ub78c\uc740?", "paragraph": "2008\ub144 9\uc6d4, \ucf54\ubbf8\ub514\uc5b8 \uc815\uc120\ud76c\uc758 \ub0a8\ud3b8\uc778 \ubc30\uc6b0 \uc548\uc7ac\ud658\uc774 \uc0ac\ub9dd\ud558\ub294 \uc0ac\uac74\uc774 \ubc1c\uc0dd\ud588\uace0 \uc774 \uc0ac\uac74\uc740 \uc5b8\ub860\uc758 \uc8fc\ubaa9\uc744 \ubc1b\uc558\ub2e4. \uc774 \uc0ac\uac74\uc744 \uc811\ud55c \ucd5c\uc9c4\uc2e4\uc740 \ud06c\uac8c \uc2ac\ud37c\ud558\uc600\uace0 \uc815\uc120\ud76c\uc758 \uacc1\uc744 \uc9c0\ucf30\ub2e4. \ub300\uc911\uc740 \uc5b8\ub860\uc744 \ud1b5\ud574 \ucd5c\uc9c4\uc2e4\uc758 \uc774\ub7f0 \ubaa8\uc2b5\uc744 \uc9c0\ucf1c\ubcf4\uc558\ub2e4. \ucd5c\uc9c4\uc2e4\uc758 \uc5b4\uba38\ub2c8 \uc815\uc625\uc219\uc740, \uc5c6\ub294 \uc77c\ub3c4 \ub9cc\ub4e4\uc5b4\ub0b4\ub294 \uac83\uc774 \uc138\uc0c1 \uc0ac\ub78c\ub4e4\uc774\ub2c8 \uc815\uc120\ud76c\ub97c \uc801\uadf9\uc801\uc73c\ub85c \ub3c4\uc6b0\ub824\ub294 \ucd5c\uc9c4\uc2e4\uc5d0\uac8c \uadf8 \ud589\ub3d9\uc744 \uc790\uc81c\ud560 \uac83\uc744 \uc8fc\ubb38\ud558\uc600\ub2e4. \ud558\uc9c0\ub9cc \ucd5c\uc9c4\uc2e4\uc740 \uadf8 \ub9d0\uc5d0 \uac1c\uc758\uce58 \uc54a\uc558\ub2e4. \uc5b4\ub290\ub367 \uc815\uc625\uc219\uc758 \uc6b0\ub824\ub294 \ud604\uc2e4\uc774 \ub418\uc5c8\ub2e4. \uc548\uc7ac\ud658\uc758 \uc8fd\uc74c\uc5d0 \ucd5c\uc9c4\uc2e4\uc774 \uc5f0\ub8e8\ub418\uc5c8\ub2e4\ub294 \ud5c8\uc704\uc0ac\uc2e4\uc774 \uc778\ud130\ub137\uc0c1\uc5d0 \uc720\ud3ec\ub418\uc5c8\ub358 \uac83\uc774\ub2e4. \uc774\uc640 \uad00\ub828\ud558\uc5ec \uc5b8\ub860 \ub610\ud55c \uc77c\uc774 \ucee4\uc9c0\ub294 \ub370 \uc77c\uc870\ud558\uc600\ub294\ub370, \uc778\ud130\ub137\uc0c1\uc5d0 \uc720\ud3ec\ub41c \uadfc\uac70 \uc5c6\ub294 \uc774\uc57c\uae30\ub4e4\uc744 \uadf8\ub300\ub85c \ud2b9\uc885\uc73c\ub85c \uc2e4\uc5c8\ub358 \uac83\uc774\ub2e4. \ub354\uc6b1\uc774 \ucd5c\uc9c4\uc2e4\uc740 20\uc5ec \ub144 \ub3d9\uc548 \uc5f0\uc608\uacc4 \uc0dd\ud65c\uc744 \ud558\uba74\uc11c \ud06c\uace0 \uc791\uac8c \ud5c8\uc704\uc0ac\uc2e4 \uc720\ud3ec\ub97c \uacaa\uc5c8\ub294\ub370, \uacfc\uac70\uc758 \uac70\uc9d3 \uc18c\ubb38\ub4e4\uae4c\uc9c0 \uc778\ud130\ub137\uc0c1\uc5d0\uc11c \ubaa8\ub450 \uc218\uba74 \uc704\ub85c \ub5a0\uc62c\ub77c \uadfc\uac70 \uc5c6\ub294 \uc18c\ubb38\uc774 \ub208\ub369\uc774\ucc98\ub7fc \ubd88\uc5b4\ub098 \ucd5c\uc9c4\uc2e4\uc744 \uac15\ud0c0\ud588\ub2e4. \ub610, \uc0ac\ub78c\ub4e4\uc740 \uc815\uc625\uc219\uc744 \ub450\uace0 '\uc18c\uc124'\uc744 \uc4f0\ub294 \uc77c\ub3c4 \uc11c\uc2b4\uc9c0 \uc54a\uc558\ub2e4. \uc774 '\uc18c\uc124'\uc740 \uadf8\ub0e5 \uc18c\uc124\ub85c \uadf8\uce58\ub294 \uac83\uc774 \uc544\ub2c8\ub77c, \uc815\uc625\uc219\uc740 \ud604\uc2e4 \uc18d\uc5d0\uc11c \uadf8 \ud53c\ud574\ub97c \uacaa\uc5b4\uc57c \ud588\ub2e4. \uc815\uc625\uc219\uc740 \uc790\uc2e0\uc744 \ub450\uace0 \uc218\uad70\uac70\ub9ac\ub294 \uc0ac\ub78c\ub4e4\uc744 \ubaa9\uaca9\ud588\ub358 \uac83\uc774\ub2e4. \uadf8 \ub2f9\uc2dc \uc0c1\ud669\uc5d0 \ub300\ud574 \uc815\uc625\uc219\uc740 \uc790\uc2e0\uc758 \uc800\uc11c \u300a\uc5c4\ub9c8\uac00, \ubbf8\uc548\ud574 \uadf8\ub9ac\uace0 \uc0ac\ub791\ud574\u300b(2011\ub144)\uc5d0\uc11c \"\uc5bc\uad74\uc744 \ubabb \ub4e4 \uc815\ub3c4\ub85c \ubaa8\uba78\uac10\uc744 \ub290\uaf08\ub2e4\"\ub77c\uace0 \ubc1d\ud614\ub2e4. \ucd5c\uc9c4\uc2e4\uc740 \uc790\uc2e0\uc740 \ubb3c\ub860 \uac00\uc871\ub4e4\uae4c\uc9c0 \uac10\uc625 \uc544\ub2cc \uac10\uc625 \uc0dd\ud65c\uc744 \ud558\ub294 \uc77c\uc744 \uacaa\uc5c8\ub358 \uac83\uc774\ub2e4. \uadf8 \ub2f9\uc2dc \ucd5c\uc9c4\uc2e4\uc758 \ub3d9\uc0dd\uc740 \uc5f0\uc608\uc778\uc774\uc5c8\uace0, \ucd5c\uc9c4\uc2e4\uc758 \uc544\ub4e4\uc740 \ucd08\ub4f1\ud559\uc0dd\uc774\uc5c8\uc73c\uba70, \ucd5c\uc9c4\uc2e4\uc758 \ub538\uc740 \uc720\uce58\uc6d0\uc0dd\uc774\uc5c8\uae30 \ub54c\ubb38\uc5d0 \ucd5c\uc9c4\uc2e4\uc758 \uc2ec\uc801 \ubd80\ub2f4\uc740 \uc0c1\ub2f9\ud588\ub2e4. \ucd5c\uc9c4\uc2e4\uc740 \uc815\uc625\uc219\uc5d0\uac8c \"\ub098 \ud5db\uc0b4\uc558\ub098\ubd10. \ub098\ub294 \uc644\uc804\ud788 \uc655\ub530\uc57c. \ub204\uad6c \ud558\ub098 \uc815\ub9d0 \uc9c4\uc2ec\uc73c\ub85c \ub098\ub97c \uc0dd\uac01\ud574\uc8fc\ub294 \uc0ac\ub78c\uc774 \uc5c6\ub124. \uadf8 \ub3d9\uc548 \uc798\ubabb \uc0b0 \uac70\uc57c. \ucd5c\uc9c4\uc2e4\uc740 \ub3c8 \ube4c\ub824\uc900 \uc801 \uc5c6\ub2e4\uace0 \ud55c \ub9c8\ub514\ub9cc \ud574\uc92c\ub2e4\uba74, \uadf8 \ud55c \ub9c8\ub514\ub9cc \ub204\uac00 \ud574\uc92c\ub2e4\uba74, \ub204\uad6c \ud55c \uba85\uc774\ub77c\ub3c4 \ub098\ub97c \uc544\ub294 \uc0ac\ub78c \uadf8 \ub204\uad6c\ub77c\ub3c4\u2026\"\ub77c\ub294 \ub9d0\uc744 \ud568\uc73c\ub85c\uc368 \uc5f0\uc608\uacc4 \ub0b4\ubd80\uc758 \ube44\uc815\ud568\uc5d0\ub3c4 \uc12d\uc12d\ud568\uc744 \ub4dc\ub7ec\ub0b4\uae30\ub3c4 \ud558\uc600\ub2e4. \uacb0\uad6d, \uadfc\uac70 \uc5c6\ub294 \uc18c\ubb38\uc740 \ucd5c\uc9c4\uc2e4\uc744 \uc8fd\uc74c\uc5d0 \uc774\ub974\uac8c \ud558\uc600\ub2e4. \uacbd\ucc30\uccad \ud504\ub85c\ud30c\uc77c\ub7ec \ucd9c\uc2e0 \ub300\ud55c\ubbfc\uad6d \ub0b4 \ucd5c\ucd08 \uc2ec\ub9ac \ubd80\uac80 \uc804\ubb38\uac00 \uc11c\uc885\ud55c\uc740 \uc790\uc0b4\uacfc \uad00\ub828\ub41c \uc9c1\uc811\uc801 \uc694\uc778\uacfc \ucd94\uac00\uc801 \uc704\ud5d8 \uc694\uc778\uc5d0 \ub530\ub77c \uc704\ud5d8\ub3c4\ub97c \ubd84\ub958\ud558\uc5ec '\uace0\uc704\ud5d8\uad70 \ubd84\ub958 \ud504\ub808\uc784\uc6cc\ud06c'\ub97c \ub9cc\ub4e4\uc5b4\ub0c8\ub294\ub370, \ucd5c\uc9c4\uc2e4\uc740 \ud544\uc218 \uc704\ud5d8 \uc694\uc778('\uc8fd\uace0 \uc2f6\ub2e4', '\uc544\uc774\ub4e4\uc744 \ubd80\ud0c1\ud55c\ub2e4') + \ucd94\uac00\uc801 \uc704\ud5d8 \uc694\uc778 4\uac1c \uc774\uc0c1(\uc545\uc131 \ub313\uae00, \ubd88\uba74\uc99d, \uc774\ud63c, \ubd80\ubd80\ud3ed\ub825, \uc6b0\uc6b8\uc99d, \uc790\uc0b4\uc790 \uacbd\ud5d8 \ub4f1)\uc5d0 \uc758\uac70\ud574 \uace0\uc704\ud5d8\uad70\uc5d0 \uc18d\ud588\ub2e4\uace0 \uc124\uba85\ud588\ub2e4.", "answer": "\uc11c\uc885\ud55c", "sentence": "\uacbd\ucc30\uccad \ud504\ub85c\ud30c\uc77c\ub7ec \ucd9c\uc2e0 \ub300\ud55c\ubbfc\uad6d \ub0b4 \ucd5c\ucd08 \uc2ec\ub9ac \ubd80\uac80 \uc804\ubb38\uac00 \uc11c\uc885\ud55c \uc740 \uc790\uc0b4\uacfc \uad00\ub828\ub41c \uc9c1\uc811\uc801 \uc694\uc778\uacfc \ucd94\uac00\uc801 \uc704\ud5d8 \uc694\uc778\uc5d0 \ub530\ub77c \uc704\ud5d8\ub3c4\ub97c \ubd84\ub958\ud558\uc5ec '\uace0\uc704\ud5d8\uad70 \ubd84\ub958 \ud504\ub808\uc784\uc6cc\ud06c'\ub97c \ub9cc\ub4e4\uc5b4\ub0c8\ub294\ub370, \ucd5c\uc9c4\uc2e4\uc740 \ud544\uc218 \uc704\ud5d8 \uc694\uc778('\uc8fd\uace0 \uc2f6\ub2e4', '\uc544\uc774\ub4e4\uc744 \ubd80\ud0c1\ud55c\ub2e4') + \ucd94\uac00\uc801 \uc704\ud5d8 \uc694\uc778 4\uac1c \uc774\uc0c1(\uc545\uc131 \ub313\uae00, \ubd88\uba74\uc99d, \uc774\ud63c, \ubd80\ubd80\ud3ed\ub825, \uc6b0\uc6b8\uc99d, \uc790\uc0b4\uc790 \uacbd\ud5d8 \ub4f1)\uc5d0 \uc758\uac70\ud574 \uace0\uc704\ud5d8\uad70\uc5d0 \uc18d\ud588\ub2e4\uace0 \uc124\uba85\ud588\ub2e4.", "paragraph_sentence": "2008\ub144 9\uc6d4, \ucf54\ubbf8\ub514\uc5b8 \uc815\uc120\ud76c\uc758 \ub0a8\ud3b8\uc778 \ubc30\uc6b0 \uc548\uc7ac\ud658\uc774 \uc0ac\ub9dd\ud558\ub294 \uc0ac\uac74\uc774 \ubc1c\uc0dd\ud588\uace0 \uc774 \uc0ac\uac74\uc740 \uc5b8\ub860\uc758 \uc8fc\ubaa9\uc744 \ubc1b\uc558\ub2e4. \uc774 \uc0ac\uac74\uc744 \uc811\ud55c \ucd5c\uc9c4\uc2e4\uc740 \ud06c\uac8c \uc2ac\ud37c\ud558\uc600\uace0 \uc815\uc120\ud76c\uc758 \uacc1\uc744 \uc9c0\ucf30\ub2e4. \ub300\uc911\uc740 \uc5b8\ub860\uc744 \ud1b5\ud574 \ucd5c\uc9c4\uc2e4\uc758 \uc774\ub7f0 \ubaa8\uc2b5\uc744 \uc9c0\ucf1c\ubcf4\uc558\ub2e4. \ucd5c\uc9c4\uc2e4\uc758 \uc5b4\uba38\ub2c8 \uc815\uc625\uc219\uc740, \uc5c6\ub294 \uc77c\ub3c4 \ub9cc\ub4e4\uc5b4\ub0b4\ub294 \uac83\uc774 \uc138\uc0c1 \uc0ac\ub78c\ub4e4\uc774\ub2c8 \uc815\uc120\ud76c\ub97c \uc801\uadf9\uc801\uc73c\ub85c \ub3c4\uc6b0\ub824\ub294 \ucd5c\uc9c4\uc2e4\uc5d0\uac8c \uadf8 \ud589\ub3d9\uc744 \uc790\uc81c\ud560 \uac83\uc744 \uc8fc\ubb38\ud558\uc600\ub2e4. \ud558\uc9c0\ub9cc \ucd5c\uc9c4\uc2e4\uc740 \uadf8 \ub9d0\uc5d0 \uac1c\uc758\uce58 \uc54a\uc558\ub2e4. \uc5b4\ub290\ub367 \uc815\uc625\uc219\uc758 \uc6b0\ub824\ub294 \ud604\uc2e4\uc774 \ub418\uc5c8\ub2e4. \uc548\uc7ac\ud658\uc758 \uc8fd\uc74c\uc5d0 \ucd5c\uc9c4\uc2e4\uc774 \uc5f0\ub8e8\ub418\uc5c8\ub2e4\ub294 \ud5c8\uc704\uc0ac\uc2e4\uc774 \uc778\ud130\ub137\uc0c1\uc5d0 \uc720\ud3ec\ub418\uc5c8\ub358 \uac83\uc774\ub2e4. \uc774\uc640 \uad00\ub828\ud558\uc5ec \uc5b8\ub860 \ub610\ud55c \uc77c\uc774 \ucee4\uc9c0\ub294 \ub370 \uc77c\uc870\ud558\uc600\ub294\ub370, \uc778\ud130\ub137\uc0c1\uc5d0 \uc720\ud3ec\ub41c \uadfc\uac70 \uc5c6\ub294 \uc774\uc57c\uae30\ub4e4\uc744 \uadf8\ub300\ub85c \ud2b9\uc885\uc73c\ub85c \uc2e4\uc5c8\ub358 \uac83\uc774\ub2e4. \ub354\uc6b1\uc774 \ucd5c\uc9c4\uc2e4\uc740 20\uc5ec \ub144 \ub3d9\uc548 \uc5f0\uc608\uacc4 \uc0dd\ud65c\uc744 \ud558\uba74\uc11c \ud06c\uace0 \uc791\uac8c \ud5c8\uc704\uc0ac\uc2e4 \uc720\ud3ec\ub97c \uacaa\uc5c8\ub294\ub370, \uacfc\uac70\uc758 \uac70\uc9d3 \uc18c\ubb38\ub4e4\uae4c\uc9c0 \uc778\ud130\ub137\uc0c1\uc5d0\uc11c \ubaa8\ub450 \uc218\uba74 \uc704\ub85c \ub5a0\uc62c\ub77c \uadfc\uac70 \uc5c6\ub294 \uc18c\ubb38\uc774 \ub208\ub369\uc774\ucc98\ub7fc \ubd88\uc5b4\ub098 \ucd5c\uc9c4\uc2e4\uc744 \uac15\ud0c0\ud588\ub2e4. \ub610, \uc0ac\ub78c\ub4e4\uc740 \uc815\uc625\uc219\uc744 \ub450\uace0 '\uc18c\uc124'\uc744 \uc4f0\ub294 \uc77c\ub3c4 \uc11c\uc2b4\uc9c0 \uc54a\uc558\ub2e4. \uc774 '\uc18c\uc124'\uc740 \uadf8\ub0e5 \uc18c\uc124\ub85c \uadf8\uce58\ub294 \uac83\uc774 \uc544\ub2c8\ub77c, \uc815\uc625\uc219\uc740 \ud604\uc2e4 \uc18d\uc5d0\uc11c \uadf8 \ud53c\ud574\ub97c \uacaa\uc5b4\uc57c \ud588\ub2e4. \uc815\uc625\uc219\uc740 \uc790\uc2e0\uc744 \ub450\uace0 \uc218\uad70\uac70\ub9ac\ub294 \uc0ac\ub78c\ub4e4\uc744 \ubaa9\uaca9\ud588\ub358 \uac83\uc774\ub2e4. \uadf8 \ub2f9\uc2dc \uc0c1\ud669\uc5d0 \ub300\ud574 \uc815\uc625\uc219\uc740 \uc790\uc2e0\uc758 \uc800\uc11c \u300a\uc5c4\ub9c8\uac00, \ubbf8\uc548\ud574 \uadf8\ub9ac\uace0 \uc0ac\ub791\ud574\u300b(2011\ub144)\uc5d0\uc11c \"\uc5bc\uad74\uc744 \ubabb \ub4e4 \uc815\ub3c4\ub85c \ubaa8\uba78\uac10\uc744 \ub290\uaf08\ub2e4\"\ub77c\uace0 \ubc1d\ud614\ub2e4. \ucd5c\uc9c4\uc2e4\uc740 \uc790\uc2e0\uc740 \ubb3c\ub860 \uac00\uc871\ub4e4\uae4c\uc9c0 \uac10\uc625 \uc544\ub2cc \uac10\uc625 \uc0dd\ud65c\uc744 \ud558\ub294 \uc77c\uc744 \uacaa\uc5c8\ub358 \uac83\uc774\ub2e4. \uadf8 \ub2f9\uc2dc \ucd5c\uc9c4\uc2e4\uc758 \ub3d9\uc0dd\uc740 \uc5f0\uc608\uc778\uc774\uc5c8\uace0, \ucd5c\uc9c4\uc2e4\uc758 \uc544\ub4e4\uc740 \ucd08\ub4f1\ud559\uc0dd\uc774\uc5c8\uc73c\uba70, \ucd5c\uc9c4\uc2e4\uc758 \ub538\uc740 \uc720\uce58\uc6d0\uc0dd\uc774\uc5c8\uae30 \ub54c\ubb38\uc5d0 \ucd5c\uc9c4\uc2e4\uc758 \uc2ec\uc801 \ubd80\ub2f4\uc740 \uc0c1\ub2f9\ud588\ub2e4. \ucd5c\uc9c4\uc2e4\uc740 \uc815\uc625\uc219\uc5d0\uac8c \"\ub098 \ud5db\uc0b4\uc558\ub098\ubd10. \ub098\ub294 \uc644\uc804\ud788 \uc655\ub530\uc57c. \ub204\uad6c \ud558\ub098 \uc815\ub9d0 \uc9c4\uc2ec\uc73c\ub85c \ub098\ub97c \uc0dd\uac01\ud574\uc8fc\ub294 \uc0ac\ub78c\uc774 \uc5c6\ub124. \uadf8 \ub3d9\uc548 \uc798\ubabb \uc0b0 \uac70\uc57c. \ucd5c\uc9c4\uc2e4\uc740 \ub3c8 \ube4c\ub824\uc900 \uc801 \uc5c6\ub2e4\uace0 \ud55c \ub9c8\ub514\ub9cc \ud574\uc92c\ub2e4\uba74, \uadf8 \ud55c \ub9c8\ub514\ub9cc \ub204\uac00 \ud574\uc92c\ub2e4\uba74, \ub204\uad6c \ud55c \uba85\uc774\ub77c\ub3c4 \ub098\ub97c \uc544\ub294 \uc0ac\ub78c \uadf8 \ub204\uad6c\ub77c\ub3c4\u2026\"\ub77c\ub294 \ub9d0\uc744 \ud568\uc73c\ub85c\uc368 \uc5f0\uc608\uacc4 \ub0b4\ubd80\uc758 \ube44\uc815\ud568\uc5d0\ub3c4 \uc12d\uc12d\ud568\uc744 \ub4dc\ub7ec\ub0b4\uae30\ub3c4 \ud558\uc600\ub2e4. \uacb0\uad6d, \uadfc\uac70 \uc5c6\ub294 \uc18c\ubb38\uc740 \ucd5c\uc9c4\uc2e4\uc744 \uc8fd\uc74c\uc5d0 \uc774\ub974\uac8c \ud558\uc600\ub2e4. \uacbd\ucc30\uccad \ud504\ub85c\ud30c\uc77c\ub7ec \ucd9c\uc2e0 \ub300\ud55c\ubbfc\uad6d \ub0b4 \ucd5c\ucd08 \uc2ec\ub9ac \ubd80\uac80 \uc804\ubb38\uac00 \uc11c\uc885\ud55c \uc740 \uc790\uc0b4\uacfc \uad00\ub828\ub41c \uc9c1\uc811\uc801 \uc694\uc778\uacfc \ucd94\uac00\uc801 \uc704\ud5d8 \uc694\uc778\uc5d0 \ub530\ub77c \uc704\ud5d8\ub3c4\ub97c \ubd84\ub958\ud558\uc5ec '\uace0\uc704\ud5d8\uad70 \ubd84\ub958 \ud504\ub808\uc784\uc6cc\ud06c'\ub97c \ub9cc\ub4e4\uc5b4\ub0c8\ub294\ub370, \ucd5c\uc9c4\uc2e4\uc740 \ud544\uc218 \uc704\ud5d8 \uc694\uc778('\uc8fd\uace0 \uc2f6\ub2e4', '\uc544\uc774\ub4e4\uc744 \ubd80\ud0c1\ud55c\ub2e4') + \ucd94\uac00\uc801 \uc704\ud5d8 \uc694\uc778 4\uac1c \uc774\uc0c1(\uc545\uc131 \ub313\uae00, \ubd88\uba74\uc99d, \uc774\ud63c, \ubd80\ubd80\ud3ed\ub825, \uc6b0\uc6b8\uc99d, \uc790\uc0b4\uc790 \uacbd\ud5d8 \ub4f1)\uc5d0 \uc758\uac70\ud574 \uace0\uc704\ud5d8\uad70\uc5d0 \uc18d\ud588\ub2e4\uace0 \uc124\uba85\ud588\ub2e4. ", "paragraph_answer": "2008\ub144 9\uc6d4, \ucf54\ubbf8\ub514\uc5b8 \uc815\uc120\ud76c\uc758 \ub0a8\ud3b8\uc778 \ubc30\uc6b0 \uc548\uc7ac\ud658\uc774 \uc0ac\ub9dd\ud558\ub294 \uc0ac\uac74\uc774 \ubc1c\uc0dd\ud588\uace0 \uc774 \uc0ac\uac74\uc740 \uc5b8\ub860\uc758 \uc8fc\ubaa9\uc744 \ubc1b\uc558\ub2e4. \uc774 \uc0ac\uac74\uc744 \uc811\ud55c \ucd5c\uc9c4\uc2e4\uc740 \ud06c\uac8c \uc2ac\ud37c\ud558\uc600\uace0 \uc815\uc120\ud76c\uc758 \uacc1\uc744 \uc9c0\ucf30\ub2e4. \ub300\uc911\uc740 \uc5b8\ub860\uc744 \ud1b5\ud574 \ucd5c\uc9c4\uc2e4\uc758 \uc774\ub7f0 \ubaa8\uc2b5\uc744 \uc9c0\ucf1c\ubcf4\uc558\ub2e4. \ucd5c\uc9c4\uc2e4\uc758 \uc5b4\uba38\ub2c8 \uc815\uc625\uc219\uc740, \uc5c6\ub294 \uc77c\ub3c4 \ub9cc\ub4e4\uc5b4\ub0b4\ub294 \uac83\uc774 \uc138\uc0c1 \uc0ac\ub78c\ub4e4\uc774\ub2c8 \uc815\uc120\ud76c\ub97c \uc801\uadf9\uc801\uc73c\ub85c \ub3c4\uc6b0\ub824\ub294 \ucd5c\uc9c4\uc2e4\uc5d0\uac8c \uadf8 \ud589\ub3d9\uc744 \uc790\uc81c\ud560 \uac83\uc744 \uc8fc\ubb38\ud558\uc600\ub2e4. \ud558\uc9c0\ub9cc \ucd5c\uc9c4\uc2e4\uc740 \uadf8 \ub9d0\uc5d0 \uac1c\uc758\uce58 \uc54a\uc558\ub2e4. \uc5b4\ub290\ub367 \uc815\uc625\uc219\uc758 \uc6b0\ub824\ub294 \ud604\uc2e4\uc774 \ub418\uc5c8\ub2e4. \uc548\uc7ac\ud658\uc758 \uc8fd\uc74c\uc5d0 \ucd5c\uc9c4\uc2e4\uc774 \uc5f0\ub8e8\ub418\uc5c8\ub2e4\ub294 \ud5c8\uc704\uc0ac\uc2e4\uc774 \uc778\ud130\ub137\uc0c1\uc5d0 \uc720\ud3ec\ub418\uc5c8\ub358 \uac83\uc774\ub2e4. \uc774\uc640 \uad00\ub828\ud558\uc5ec \uc5b8\ub860 \ub610\ud55c \uc77c\uc774 \ucee4\uc9c0\ub294 \ub370 \uc77c\uc870\ud558\uc600\ub294\ub370, \uc778\ud130\ub137\uc0c1\uc5d0 \uc720\ud3ec\ub41c \uadfc\uac70 \uc5c6\ub294 \uc774\uc57c\uae30\ub4e4\uc744 \uadf8\ub300\ub85c \ud2b9\uc885\uc73c\ub85c \uc2e4\uc5c8\ub358 \uac83\uc774\ub2e4. \ub354\uc6b1\uc774 \ucd5c\uc9c4\uc2e4\uc740 20\uc5ec \ub144 \ub3d9\uc548 \uc5f0\uc608\uacc4 \uc0dd\ud65c\uc744 \ud558\uba74\uc11c \ud06c\uace0 \uc791\uac8c \ud5c8\uc704\uc0ac\uc2e4 \uc720\ud3ec\ub97c \uacaa\uc5c8\ub294\ub370, \uacfc\uac70\uc758 \uac70\uc9d3 \uc18c\ubb38\ub4e4\uae4c\uc9c0 \uc778\ud130\ub137\uc0c1\uc5d0\uc11c \ubaa8\ub450 \uc218\uba74 \uc704\ub85c \ub5a0\uc62c\ub77c \uadfc\uac70 \uc5c6\ub294 \uc18c\ubb38\uc774 \ub208\ub369\uc774\ucc98\ub7fc \ubd88\uc5b4\ub098 \ucd5c\uc9c4\uc2e4\uc744 \uac15\ud0c0\ud588\ub2e4. \ub610, \uc0ac\ub78c\ub4e4\uc740 \uc815\uc625\uc219\uc744 \ub450\uace0 '\uc18c\uc124'\uc744 \uc4f0\ub294 \uc77c\ub3c4 \uc11c\uc2b4\uc9c0 \uc54a\uc558\ub2e4. \uc774 '\uc18c\uc124'\uc740 \uadf8\ub0e5 \uc18c\uc124\ub85c \uadf8\uce58\ub294 \uac83\uc774 \uc544\ub2c8\ub77c, \uc815\uc625\uc219\uc740 \ud604\uc2e4 \uc18d\uc5d0\uc11c \uadf8 \ud53c\ud574\ub97c \uacaa\uc5b4\uc57c \ud588\ub2e4. \uc815\uc625\uc219\uc740 \uc790\uc2e0\uc744 \ub450\uace0 \uc218\uad70\uac70\ub9ac\ub294 \uc0ac\ub78c\ub4e4\uc744 \ubaa9\uaca9\ud588\ub358 \uac83\uc774\ub2e4. \uadf8 \ub2f9\uc2dc \uc0c1\ud669\uc5d0 \ub300\ud574 \uc815\uc625\uc219\uc740 \uc790\uc2e0\uc758 \uc800\uc11c \u300a\uc5c4\ub9c8\uac00, \ubbf8\uc548\ud574 \uadf8\ub9ac\uace0 \uc0ac\ub791\ud574\u300b(2011\ub144)\uc5d0\uc11c \"\uc5bc\uad74\uc744 \ubabb \ub4e4 \uc815\ub3c4\ub85c \ubaa8\uba78\uac10\uc744 \ub290\uaf08\ub2e4\"\ub77c\uace0 \ubc1d\ud614\ub2e4. \ucd5c\uc9c4\uc2e4\uc740 \uc790\uc2e0\uc740 \ubb3c\ub860 \uac00\uc871\ub4e4\uae4c\uc9c0 \uac10\uc625 \uc544\ub2cc \uac10\uc625 \uc0dd\ud65c\uc744 \ud558\ub294 \uc77c\uc744 \uacaa\uc5c8\ub358 \uac83\uc774\ub2e4. \uadf8 \ub2f9\uc2dc \ucd5c\uc9c4\uc2e4\uc758 \ub3d9\uc0dd\uc740 \uc5f0\uc608\uc778\uc774\uc5c8\uace0, \ucd5c\uc9c4\uc2e4\uc758 \uc544\ub4e4\uc740 \ucd08\ub4f1\ud559\uc0dd\uc774\uc5c8\uc73c\uba70, \ucd5c\uc9c4\uc2e4\uc758 \ub538\uc740 \uc720\uce58\uc6d0\uc0dd\uc774\uc5c8\uae30 \ub54c\ubb38\uc5d0 \ucd5c\uc9c4\uc2e4\uc758 \uc2ec\uc801 \ubd80\ub2f4\uc740 \uc0c1\ub2f9\ud588\ub2e4. \ucd5c\uc9c4\uc2e4\uc740 \uc815\uc625\uc219\uc5d0\uac8c \"\ub098 \ud5db\uc0b4\uc558\ub098\ubd10. \ub098\ub294 \uc644\uc804\ud788 \uc655\ub530\uc57c. \ub204\uad6c \ud558\ub098 \uc815\ub9d0 \uc9c4\uc2ec\uc73c\ub85c \ub098\ub97c \uc0dd\uac01\ud574\uc8fc\ub294 \uc0ac\ub78c\uc774 \uc5c6\ub124. \uadf8 \ub3d9\uc548 \uc798\ubabb \uc0b0 \uac70\uc57c. \ucd5c\uc9c4\uc2e4\uc740 \ub3c8 \ube4c\ub824\uc900 \uc801 \uc5c6\ub2e4\uace0 \ud55c \ub9c8\ub514\ub9cc \ud574\uc92c\ub2e4\uba74, \uadf8 \ud55c \ub9c8\ub514\ub9cc \ub204\uac00 \ud574\uc92c\ub2e4\uba74, \ub204\uad6c \ud55c \uba85\uc774\ub77c\ub3c4 \ub098\ub97c \uc544\ub294 \uc0ac\ub78c \uadf8 \ub204\uad6c\ub77c\ub3c4\u2026\"\ub77c\ub294 \ub9d0\uc744 \ud568\uc73c\ub85c\uc368 \uc5f0\uc608\uacc4 \ub0b4\ubd80\uc758 \ube44\uc815\ud568\uc5d0\ub3c4 \uc12d\uc12d\ud568\uc744 \ub4dc\ub7ec\ub0b4\uae30\ub3c4 \ud558\uc600\ub2e4. \uacb0\uad6d, \uadfc\uac70 \uc5c6\ub294 \uc18c\ubb38\uc740 \ucd5c\uc9c4\uc2e4\uc744 \uc8fd\uc74c\uc5d0 \uc774\ub974\uac8c \ud558\uc600\ub2e4. \uacbd\ucc30\uccad \ud504\ub85c\ud30c\uc77c\ub7ec \ucd9c\uc2e0 \ub300\ud55c\ubbfc\uad6d \ub0b4 \ucd5c\ucd08 \uc2ec\ub9ac \ubd80\uac80 \uc804\ubb38\uac00 \uc11c\uc885\ud55c \uc740 \uc790\uc0b4\uacfc \uad00\ub828\ub41c \uc9c1\uc811\uc801 \uc694\uc778\uacfc \ucd94\uac00\uc801 \uc704\ud5d8 \uc694\uc778\uc5d0 \ub530\ub77c \uc704\ud5d8\ub3c4\ub97c \ubd84\ub958\ud558\uc5ec '\uace0\uc704\ud5d8\uad70 \ubd84\ub958 \ud504\ub808\uc784\uc6cc\ud06c'\ub97c \ub9cc\ub4e4\uc5b4\ub0c8\ub294\ub370, \ucd5c\uc9c4\uc2e4\uc740 \ud544\uc218 \uc704\ud5d8 \uc694\uc778('\uc8fd\uace0 \uc2f6\ub2e4', '\uc544\uc774\ub4e4\uc744 \ubd80\ud0c1\ud55c\ub2e4') + \ucd94\uac00\uc801 \uc704\ud5d8 \uc694\uc778 4\uac1c \uc774\uc0c1(\uc545\uc131 \ub313\uae00, \ubd88\uba74\uc99d, \uc774\ud63c, \ubd80\ubd80\ud3ed\ub825, \uc6b0\uc6b8\uc99d, \uc790\uc0b4\uc790 \uacbd\ud5d8 \ub4f1)\uc5d0 \uc758\uac70\ud574 \uace0\uc704\ud5d8\uad70\uc5d0 \uc18d\ud588\ub2e4\uace0 \uc124\uba85\ud588\ub2e4.", "sentence_answer": "\uacbd\ucc30\uccad \ud504\ub85c\ud30c\uc77c\ub7ec \ucd9c\uc2e0 \ub300\ud55c\ubbfc\uad6d \ub0b4 \ucd5c\ucd08 \uc2ec\ub9ac \ubd80\uac80 \uc804\ubb38\uac00 \uc11c\uc885\ud55c \uc740 \uc790\uc0b4\uacfc \uad00\ub828\ub41c \uc9c1\uc811\uc801 \uc694\uc778\uacfc \ucd94\uac00\uc801 \uc704\ud5d8 \uc694\uc778\uc5d0 \ub530\ub77c \uc704\ud5d8\ub3c4\ub97c \ubd84\ub958\ud558\uc5ec '\uace0\uc704\ud5d8\uad70 \ubd84\ub958 \ud504\ub808\uc784\uc6cc\ud06c'\ub97c \ub9cc\ub4e4\uc5b4\ub0c8\ub294\ub370, \ucd5c\uc9c4\uc2e4\uc740 \ud544\uc218 \uc704\ud5d8 \uc694\uc778('\uc8fd\uace0 \uc2f6\ub2e4', '\uc544\uc774\ub4e4\uc744 \ubd80\ud0c1\ud55c\ub2e4') + \ucd94\uac00\uc801 \uc704\ud5d8 \uc694\uc778 4\uac1c \uc774\uc0c1(\uc545\uc131 \ub313\uae00, \ubd88\uba74\uc99d, \uc774\ud63c, \ubd80\ubd80\ud3ed\ub825, \uc6b0\uc6b8\uc99d, \uc790\uc0b4\uc790 \uacbd\ud5d8 \ub4f1)\uc5d0 \uc758\uac70\ud574 \uace0\uc704\ud5d8\uad70\uc5d0 \uc18d\ud588\ub2e4\uace0 \uc124\uba85\ud588\ub2e4.", "paragraph_id": "6560156-12-1", "paragraph_question": "question: \uc790\uc0b4\uacfc \uad00\ub828\ud558\uc5ec \uc694\uc778\uc5d0 \ub530\ub77c \uc704\ud5d8\ub3c4\ub97c \ubd84\ub958\ud558\uc5ec \uace0\uc704\ud5d8\uad70 \ubd84\ub958 \ud504\ub808\uc784\uc6cc\ud06c\ub97c \ub9cc\ub4e0 \uc0ac\ub78c\uc740?, context: 2008\ub144 9\uc6d4, \ucf54\ubbf8\ub514\uc5b8 \uc815\uc120\ud76c\uc758 \ub0a8\ud3b8\uc778 \ubc30\uc6b0 \uc548\uc7ac\ud658\uc774 \uc0ac\ub9dd\ud558\ub294 \uc0ac\uac74\uc774 \ubc1c\uc0dd\ud588\uace0 \uc774 \uc0ac\uac74\uc740 \uc5b8\ub860\uc758 \uc8fc\ubaa9\uc744 \ubc1b\uc558\ub2e4. \uc774 \uc0ac\uac74\uc744 \uc811\ud55c \ucd5c\uc9c4\uc2e4\uc740 \ud06c\uac8c \uc2ac\ud37c\ud558\uc600\uace0 \uc815\uc120\ud76c\uc758 \uacc1\uc744 \uc9c0\ucf30\ub2e4. \ub300\uc911\uc740 \uc5b8\ub860\uc744 \ud1b5\ud574 \ucd5c\uc9c4\uc2e4\uc758 \uc774\ub7f0 \ubaa8\uc2b5\uc744 \uc9c0\ucf1c\ubcf4\uc558\ub2e4. \ucd5c\uc9c4\uc2e4\uc758 \uc5b4\uba38\ub2c8 \uc815\uc625\uc219\uc740, \uc5c6\ub294 \uc77c\ub3c4 \ub9cc\ub4e4\uc5b4\ub0b4\ub294 \uac83\uc774 \uc138\uc0c1 \uc0ac\ub78c\ub4e4\uc774\ub2c8 \uc815\uc120\ud76c\ub97c \uc801\uadf9\uc801\uc73c\ub85c \ub3c4\uc6b0\ub824\ub294 \ucd5c\uc9c4\uc2e4\uc5d0\uac8c \uadf8 \ud589\ub3d9\uc744 \uc790\uc81c\ud560 \uac83\uc744 \uc8fc\ubb38\ud558\uc600\ub2e4. \ud558\uc9c0\ub9cc \ucd5c\uc9c4\uc2e4\uc740 \uadf8 \ub9d0\uc5d0 \uac1c\uc758\uce58 \uc54a\uc558\ub2e4. \uc5b4\ub290\ub367 \uc815\uc625\uc219\uc758 \uc6b0\ub824\ub294 \ud604\uc2e4\uc774 \ub418\uc5c8\ub2e4. \uc548\uc7ac\ud658\uc758 \uc8fd\uc74c\uc5d0 \ucd5c\uc9c4\uc2e4\uc774 \uc5f0\ub8e8\ub418\uc5c8\ub2e4\ub294 \ud5c8\uc704\uc0ac\uc2e4\uc774 \uc778\ud130\ub137\uc0c1\uc5d0 \uc720\ud3ec\ub418\uc5c8\ub358 \uac83\uc774\ub2e4. \uc774\uc640 \uad00\ub828\ud558\uc5ec \uc5b8\ub860 \ub610\ud55c \uc77c\uc774 \ucee4\uc9c0\ub294 \ub370 \uc77c\uc870\ud558\uc600\ub294\ub370, \uc778\ud130\ub137\uc0c1\uc5d0 \uc720\ud3ec\ub41c \uadfc\uac70 \uc5c6\ub294 \uc774\uc57c\uae30\ub4e4\uc744 \uadf8\ub300\ub85c \ud2b9\uc885\uc73c\ub85c \uc2e4\uc5c8\ub358 \uac83\uc774\ub2e4. \ub354\uc6b1\uc774 \ucd5c\uc9c4\uc2e4\uc740 20\uc5ec \ub144 \ub3d9\uc548 \uc5f0\uc608\uacc4 \uc0dd\ud65c\uc744 \ud558\uba74\uc11c \ud06c\uace0 \uc791\uac8c \ud5c8\uc704\uc0ac\uc2e4 \uc720\ud3ec\ub97c \uacaa\uc5c8\ub294\ub370, \uacfc\uac70\uc758 \uac70\uc9d3 \uc18c\ubb38\ub4e4\uae4c\uc9c0 \uc778\ud130\ub137\uc0c1\uc5d0\uc11c \ubaa8\ub450 \uc218\uba74 \uc704\ub85c \ub5a0\uc62c\ub77c \uadfc\uac70 \uc5c6\ub294 \uc18c\ubb38\uc774 \ub208\ub369\uc774\ucc98\ub7fc \ubd88\uc5b4\ub098 \ucd5c\uc9c4\uc2e4\uc744 \uac15\ud0c0\ud588\ub2e4. \ub610, \uc0ac\ub78c\ub4e4\uc740 \uc815\uc625\uc219\uc744 \ub450\uace0 '\uc18c\uc124'\uc744 \uc4f0\ub294 \uc77c\ub3c4 \uc11c\uc2b4\uc9c0 \uc54a\uc558\ub2e4. \uc774 '\uc18c\uc124'\uc740 \uadf8\ub0e5 \uc18c\uc124\ub85c \uadf8\uce58\ub294 \uac83\uc774 \uc544\ub2c8\ub77c, \uc815\uc625\uc219\uc740 \ud604\uc2e4 \uc18d\uc5d0\uc11c \uadf8 \ud53c\ud574\ub97c \uacaa\uc5b4\uc57c \ud588\ub2e4. \uc815\uc625\uc219\uc740 \uc790\uc2e0\uc744 \ub450\uace0 \uc218\uad70\uac70\ub9ac\ub294 \uc0ac\ub78c\ub4e4\uc744 \ubaa9\uaca9\ud588\ub358 \uac83\uc774\ub2e4. \uadf8 \ub2f9\uc2dc \uc0c1\ud669\uc5d0 \ub300\ud574 \uc815\uc625\uc219\uc740 \uc790\uc2e0\uc758 \uc800\uc11c \u300a\uc5c4\ub9c8\uac00, \ubbf8\uc548\ud574 \uadf8\ub9ac\uace0 \uc0ac\ub791\ud574\u300b(2011\ub144)\uc5d0\uc11c \"\uc5bc\uad74\uc744 \ubabb \ub4e4 \uc815\ub3c4\ub85c \ubaa8\uba78\uac10\uc744 \ub290\uaf08\ub2e4\"\ub77c\uace0 \ubc1d\ud614\ub2e4. \ucd5c\uc9c4\uc2e4\uc740 \uc790\uc2e0\uc740 \ubb3c\ub860 \uac00\uc871\ub4e4\uae4c\uc9c0 \uac10\uc625 \uc544\ub2cc \uac10\uc625 \uc0dd\ud65c\uc744 \ud558\ub294 \uc77c\uc744 \uacaa\uc5c8\ub358 \uac83\uc774\ub2e4. \uadf8 \ub2f9\uc2dc \ucd5c\uc9c4\uc2e4\uc758 \ub3d9\uc0dd\uc740 \uc5f0\uc608\uc778\uc774\uc5c8\uace0, \ucd5c\uc9c4\uc2e4\uc758 \uc544\ub4e4\uc740 \ucd08\ub4f1\ud559\uc0dd\uc774\uc5c8\uc73c\uba70, \ucd5c\uc9c4\uc2e4\uc758 \ub538\uc740 \uc720\uce58\uc6d0\uc0dd\uc774\uc5c8\uae30 \ub54c\ubb38\uc5d0 \ucd5c\uc9c4\uc2e4\uc758 \uc2ec\uc801 \ubd80\ub2f4\uc740 \uc0c1\ub2f9\ud588\ub2e4. \ucd5c\uc9c4\uc2e4\uc740 \uc815\uc625\uc219\uc5d0\uac8c \"\ub098 \ud5db\uc0b4\uc558\ub098\ubd10. \ub098\ub294 \uc644\uc804\ud788 \uc655\ub530\uc57c. \ub204\uad6c \ud558\ub098 \uc815\ub9d0 \uc9c4\uc2ec\uc73c\ub85c \ub098\ub97c \uc0dd\uac01\ud574\uc8fc\ub294 \uc0ac\ub78c\uc774 \uc5c6\ub124. \uadf8 \ub3d9\uc548 \uc798\ubabb \uc0b0 \uac70\uc57c. \ucd5c\uc9c4\uc2e4\uc740 \ub3c8 \ube4c\ub824\uc900 \uc801 \uc5c6\ub2e4\uace0 \ud55c \ub9c8\ub514\ub9cc \ud574\uc92c\ub2e4\uba74, \uadf8 \ud55c \ub9c8\ub514\ub9cc \ub204\uac00 \ud574\uc92c\ub2e4\uba74, \ub204\uad6c \ud55c \uba85\uc774\ub77c\ub3c4 \ub098\ub97c \uc544\ub294 \uc0ac\ub78c \uadf8 \ub204\uad6c\ub77c\ub3c4\u2026\"\ub77c\ub294 \ub9d0\uc744 \ud568\uc73c\ub85c\uc368 \uc5f0\uc608\uacc4 \ub0b4\ubd80\uc758 \ube44\uc815\ud568\uc5d0\ub3c4 \uc12d\uc12d\ud568\uc744 \ub4dc\ub7ec\ub0b4\uae30\ub3c4 \ud558\uc600\ub2e4. \uacb0\uad6d, \uadfc\uac70 \uc5c6\ub294 \uc18c\ubb38\uc740 \ucd5c\uc9c4\uc2e4\uc744 \uc8fd\uc74c\uc5d0 \uc774\ub974\uac8c \ud558\uc600\ub2e4. \uacbd\ucc30\uccad \ud504\ub85c\ud30c\uc77c\ub7ec \ucd9c\uc2e0 \ub300\ud55c\ubbfc\uad6d \ub0b4 \ucd5c\ucd08 \uc2ec\ub9ac \ubd80\uac80 \uc804\ubb38\uac00 \uc11c\uc885\ud55c\uc740 \uc790\uc0b4\uacfc \uad00\ub828\ub41c \uc9c1\uc811\uc801 \uc694\uc778\uacfc \ucd94\uac00\uc801 \uc704\ud5d8 \uc694\uc778\uc5d0 \ub530\ub77c \uc704\ud5d8\ub3c4\ub97c \ubd84\ub958\ud558\uc5ec '\uace0\uc704\ud5d8\uad70 \ubd84\ub958 \ud504\ub808\uc784\uc6cc\ud06c'\ub97c \ub9cc\ub4e4\uc5b4\ub0c8\ub294\ub370, \ucd5c\uc9c4\uc2e4\uc740 \ud544\uc218 \uc704\ud5d8 \uc694\uc778('\uc8fd\uace0 \uc2f6\ub2e4', '\uc544\uc774\ub4e4\uc744 \ubd80\ud0c1\ud55c\ub2e4') + \ucd94\uac00\uc801 \uc704\ud5d8 \uc694\uc778 4\uac1c \uc774\uc0c1(\uc545\uc131 \ub313\uae00, \ubd88\uba74\uc99d, \uc774\ud63c, \ubd80\ubd80\ud3ed\ub825, \uc6b0\uc6b8\uc99d, \uc790\uc0b4\uc790 \uacbd\ud5d8 \ub4f1)\uc5d0 \uc758\uac70\ud574 \uace0\uc704\ud5d8\uad70\uc5d0 \uc18d\ud588\ub2e4\uace0 \uc124\uba85\ud588\ub2e4."} {"question": "\uc774 \ub300\ud68c\uc5d0\uc11c \ucd5c\ubbfc\uc815\uc758 \uc21c\uc704\ub294?", "paragraph": "2015-2016 \uc2dc\uc98c ISU \uc1fc\ud2b8\ud2b8\ub799 \uc6d4\ub4dc\ucef5\uc5d0\uc11c \uae40\uc544\ub791\uc740 \uc8fc \uc885\ubaa9\uc778 1000\ubbf8\ud130\uc640 1500\ubbf8\ud130\ub97c \ub178\ub3c4\ud76c\uc640 \ubc88\uac08\uc544 \uac00\uba70 \ucd9c\uc804\ud558\uc5ec \uac1c\uc778\uc804 \ub3d9\uba54\ub2ec 4\uac1c\ub97c \ud68d\ub4dd\ud558\ub294 \ub370 \uadf8\ucce4\uc73c\ub098 3000m \uacc4\uc8fc\uc5d0\uc11c\ub294 5\ubc88 \ucd9c\uc804\ud558\uc5ec 4\uc5f0\uc18d \uae08\uba54\ub2ec\uc744 \ucc28\uc9c0\ud558\uc600\ub2e4. \ud558\uc9c0\ub9cc \uae40\uc544\ub791\uc740 2016\ub144 2\uc6d4 \ub124\ub35c\ub780\ub4dc \ub3c4\ub974\ub4dc\ub808\ud750\ud2b8\uc758 ISU \uc1fc\ud2b8\ud2b8\ub799 \uc6d4\ub4dc\ucef5 6\ucc28 \ub300\ud68c\ub97c \uc55e\ub450\uace0 \ud6c8\ub828\ud558\ub358 \uc911 \ubc1c\ubaa9 \ubd80\uc0c1\uc73c\ub85c \uadf8 \ub300\ud68c 1000\ubbf8\ud130 (2), 1500\ubbf8\ud130 \ucd9c\uc804 \uc5d4\ud2b8\ub9ac\ub97c \uc81c\ucd9c\ud558\uace0\ub3c4 \uacbd\uae30\ub97c \ud3ec\uae30\ud560 \uc218\ubc16\uc5d0 \uc5c6\uc5c8\uc73c\uba70, 2016\ub144 3\uc6d4 \uc11c\uc6b8\uc5d0\uc11c \uc5f4\ub9b0 2016 \uc138\uacc4 \uc120\uc218\uad8c \ub300\ud68c\uc5d0\ub3c4 \uae40\uc544\ub791\uc740 \uadf8 \ubd80\uc0c1\uc73c\ub85c \uc778\ud558\uc5ec \uac1c\uc778 \uc885\ubaa9\uc5d0 \ucc38\uac00\ud560 \uc218 \uc5c6\uc5c8\uc73c\ub098 \uc720\uc77c\ud558\uac8c 3000\ubbf8\ud130 \uacc4\uc8fc \uc900\uacb0\uc2b9\uc5d0\ub9cc \ucc38\uc5ec\ud558\uc5ec \ub300\ud45c\ud300\uc758 3000\ubbf8\ud130 \uacc4\uc8fc \uc6b0\uc2b9\uc5d0 \uae30\uc5ec\ud558\uc600\ub2e4. \ud55c\ud3b8 \uc774 \ub300\ud68c\uc5d0\uc11c \ucd5c\ubbfc\uc815\uc740 \uac1c\uc778 \uc885\ud569 \uc21c\uc704 1\uc704, \uc2ec\uc11d\ud76c\ub294 13\uc704, \ub178\ub3c4\ud76c\ub294 21\uc704\ub97c \uae30\ub85d\ud558\uc600\ub2e4.", "answer": "1\uc704", "sentence": "\ud55c\ud3b8 \uc774 \ub300\ud68c\uc5d0\uc11c \ucd5c\ubbfc\uc815\uc740 \uac1c\uc778 \uc885\ud569 \uc21c\uc704 1\uc704 ,", "paragraph_sentence": "2015-2016 \uc2dc\uc98c ISU \uc1fc\ud2b8\ud2b8\ub799 \uc6d4\ub4dc\ucef5\uc5d0\uc11c \uae40\uc544\ub791\uc740 \uc8fc \uc885\ubaa9\uc778 1000\ubbf8\ud130\uc640 1500\ubbf8\ud130\ub97c \ub178\ub3c4\ud76c\uc640 \ubc88\uac08\uc544 \uac00\uba70 \ucd9c\uc804\ud558\uc5ec \uac1c\uc778\uc804 \ub3d9\uba54\ub2ec 4\uac1c\ub97c \ud68d\ub4dd\ud558\ub294 \ub370 \uadf8\ucce4\uc73c\ub098 3000m \uacc4\uc8fc\uc5d0\uc11c\ub294 5\ubc88 \ucd9c\uc804\ud558\uc5ec 4\uc5f0\uc18d \uae08\uba54\ub2ec\uc744 \ucc28\uc9c0\ud558\uc600\ub2e4. \ud558\uc9c0\ub9cc \uae40\uc544\ub791\uc740 2016\ub144 2\uc6d4 \ub124\ub35c\ub780\ub4dc \ub3c4\ub974\ub4dc\ub808\ud750\ud2b8\uc758 ISU \uc1fc\ud2b8\ud2b8\ub799 \uc6d4\ub4dc\ucef5 6\ucc28 \ub300\ud68c\ub97c \uc55e\ub450\uace0 \ud6c8\ub828\ud558\ub358 \uc911 \ubc1c\ubaa9 \ubd80\uc0c1\uc73c\ub85c \uadf8 \ub300\ud68c 1000\ubbf8\ud130 (2), 1500\ubbf8\ud130 \ucd9c\uc804 \uc5d4\ud2b8\ub9ac\ub97c \uc81c\ucd9c\ud558\uace0\ub3c4 \uacbd\uae30\ub97c \ud3ec\uae30\ud560 \uc218\ubc16\uc5d0 \uc5c6\uc5c8\uc73c\uba70, 2016\ub144 3\uc6d4 \uc11c\uc6b8\uc5d0\uc11c \uc5f4\ub9b0 2016 \uc138\uacc4 \uc120\uc218\uad8c \ub300\ud68c\uc5d0\ub3c4 \uae40\uc544\ub791\uc740 \uadf8 \ubd80\uc0c1\uc73c\ub85c \uc778\ud558\uc5ec \uac1c\uc778 \uc885\ubaa9\uc5d0 \ucc38\uac00\ud560 \uc218 \uc5c6\uc5c8\uc73c\ub098 \uc720\uc77c\ud558\uac8c 3000\ubbf8\ud130 \uacc4\uc8fc \uc900\uacb0\uc2b9\uc5d0\ub9cc \ucc38\uc5ec\ud558\uc5ec \ub300\ud45c\ud300\uc758 3000\ubbf8\ud130 \uacc4\uc8fc \uc6b0\uc2b9\uc5d0 \uae30\uc5ec\ud558\uc600\ub2e4. \ud55c\ud3b8 \uc774 \ub300\ud68c\uc5d0\uc11c \ucd5c\ubbfc\uc815\uc740 \uac1c\uc778 \uc885\ud569 \uc21c\uc704 1\uc704 , \uc2ec\uc11d\ud76c\ub294 13\uc704, \ub178\ub3c4\ud76c\ub294 21\uc704\ub97c \uae30\ub85d\ud558\uc600\ub2e4.", "paragraph_answer": "2015-2016 \uc2dc\uc98c ISU \uc1fc\ud2b8\ud2b8\ub799 \uc6d4\ub4dc\ucef5\uc5d0\uc11c \uae40\uc544\ub791\uc740 \uc8fc \uc885\ubaa9\uc778 1000\ubbf8\ud130\uc640 1500\ubbf8\ud130\ub97c \ub178\ub3c4\ud76c\uc640 \ubc88\uac08\uc544 \uac00\uba70 \ucd9c\uc804\ud558\uc5ec \uac1c\uc778\uc804 \ub3d9\uba54\ub2ec 4\uac1c\ub97c \ud68d\ub4dd\ud558\ub294 \ub370 \uadf8\ucce4\uc73c\ub098 3000m \uacc4\uc8fc\uc5d0\uc11c\ub294 5\ubc88 \ucd9c\uc804\ud558\uc5ec 4\uc5f0\uc18d \uae08\uba54\ub2ec\uc744 \ucc28\uc9c0\ud558\uc600\ub2e4. \ud558\uc9c0\ub9cc \uae40\uc544\ub791\uc740 2016\ub144 2\uc6d4 \ub124\ub35c\ub780\ub4dc \ub3c4\ub974\ub4dc\ub808\ud750\ud2b8\uc758 ISU \uc1fc\ud2b8\ud2b8\ub799 \uc6d4\ub4dc\ucef5 6\ucc28 \ub300\ud68c\ub97c \uc55e\ub450\uace0 \ud6c8\ub828\ud558\ub358 \uc911 \ubc1c\ubaa9 \ubd80\uc0c1\uc73c\ub85c \uadf8 \ub300\ud68c 1000\ubbf8\ud130 (2), 1500\ubbf8\ud130 \ucd9c\uc804 \uc5d4\ud2b8\ub9ac\ub97c \uc81c\ucd9c\ud558\uace0\ub3c4 \uacbd\uae30\ub97c \ud3ec\uae30\ud560 \uc218\ubc16\uc5d0 \uc5c6\uc5c8\uc73c\uba70, 2016\ub144 3\uc6d4 \uc11c\uc6b8\uc5d0\uc11c \uc5f4\ub9b0 2016 \uc138\uacc4 \uc120\uc218\uad8c \ub300\ud68c\uc5d0\ub3c4 \uae40\uc544\ub791\uc740 \uadf8 \ubd80\uc0c1\uc73c\ub85c \uc778\ud558\uc5ec \uac1c\uc778 \uc885\ubaa9\uc5d0 \ucc38\uac00\ud560 \uc218 \uc5c6\uc5c8\uc73c\ub098 \uc720\uc77c\ud558\uac8c 3000\ubbf8\ud130 \uacc4\uc8fc \uc900\uacb0\uc2b9\uc5d0\ub9cc \ucc38\uc5ec\ud558\uc5ec \ub300\ud45c\ud300\uc758 3000\ubbf8\ud130 \uacc4\uc8fc \uc6b0\uc2b9\uc5d0 \uae30\uc5ec\ud558\uc600\ub2e4. \ud55c\ud3b8 \uc774 \ub300\ud68c\uc5d0\uc11c \ucd5c\ubbfc\uc815\uc740 \uac1c\uc778 \uc885\ud569 \uc21c\uc704 1\uc704 , \uc2ec\uc11d\ud76c\ub294 13\uc704, \ub178\ub3c4\ud76c\ub294 21\uc704\ub97c \uae30\ub85d\ud558\uc600\ub2e4.", "sentence_answer": "\ud55c\ud3b8 \uc774 \ub300\ud68c\uc5d0\uc11c \ucd5c\ubbfc\uc815\uc740 \uac1c\uc778 \uc885\ud569 \uc21c\uc704 1\uc704 ,", "paragraph_id": "6481874-3-1", "paragraph_question": "question: \uc774 \ub300\ud68c\uc5d0\uc11c \ucd5c\ubbfc\uc815\uc758 \uc21c\uc704\ub294?, context: 2015-2016 \uc2dc\uc98c ISU \uc1fc\ud2b8\ud2b8\ub799 \uc6d4\ub4dc\ucef5\uc5d0\uc11c \uae40\uc544\ub791\uc740 \uc8fc \uc885\ubaa9\uc778 1000\ubbf8\ud130\uc640 1500\ubbf8\ud130\ub97c \ub178\ub3c4\ud76c\uc640 \ubc88\uac08\uc544 \uac00\uba70 \ucd9c\uc804\ud558\uc5ec \uac1c\uc778\uc804 \ub3d9\uba54\ub2ec 4\uac1c\ub97c \ud68d\ub4dd\ud558\ub294 \ub370 \uadf8\ucce4\uc73c\ub098 3000m \uacc4\uc8fc\uc5d0\uc11c\ub294 5\ubc88 \ucd9c\uc804\ud558\uc5ec 4\uc5f0\uc18d \uae08\uba54\ub2ec\uc744 \ucc28\uc9c0\ud558\uc600\ub2e4. \ud558\uc9c0\ub9cc \uae40\uc544\ub791\uc740 2016\ub144 2\uc6d4 \ub124\ub35c\ub780\ub4dc \ub3c4\ub974\ub4dc\ub808\ud750\ud2b8\uc758 ISU \uc1fc\ud2b8\ud2b8\ub799 \uc6d4\ub4dc\ucef5 6\ucc28 \ub300\ud68c\ub97c \uc55e\ub450\uace0 \ud6c8\ub828\ud558\ub358 \uc911 \ubc1c\ubaa9 \ubd80\uc0c1\uc73c\ub85c \uadf8 \ub300\ud68c 1000\ubbf8\ud130 (2), 1500\ubbf8\ud130 \ucd9c\uc804 \uc5d4\ud2b8\ub9ac\ub97c \uc81c\ucd9c\ud558\uace0\ub3c4 \uacbd\uae30\ub97c \ud3ec\uae30\ud560 \uc218\ubc16\uc5d0 \uc5c6\uc5c8\uc73c\uba70, 2016\ub144 3\uc6d4 \uc11c\uc6b8\uc5d0\uc11c \uc5f4\ub9b0 2016 \uc138\uacc4 \uc120\uc218\uad8c \ub300\ud68c\uc5d0\ub3c4 \uae40\uc544\ub791\uc740 \uadf8 \ubd80\uc0c1\uc73c\ub85c \uc778\ud558\uc5ec \uac1c\uc778 \uc885\ubaa9\uc5d0 \ucc38\uac00\ud560 \uc218 \uc5c6\uc5c8\uc73c\ub098 \uc720\uc77c\ud558\uac8c 3000\ubbf8\ud130 \uacc4\uc8fc \uc900\uacb0\uc2b9\uc5d0\ub9cc \ucc38\uc5ec\ud558\uc5ec \ub300\ud45c\ud300\uc758 3000\ubbf8\ud130 \uacc4\uc8fc \uc6b0\uc2b9\uc5d0 \uae30\uc5ec\ud558\uc600\ub2e4. \ud55c\ud3b8 \uc774 \ub300\ud68c\uc5d0\uc11c \ucd5c\ubbfc\uc815\uc740 \uac1c\uc778 \uc885\ud569 \uc21c\uc704 1\uc704, \uc2ec\uc11d\ud76c\ub294 13\uc704, \ub178\ub3c4\ud76c\ub294 21\uc704\ub97c \uae30\ub85d\ud558\uc600\ub2e4."} {"question": "\uc560\ub85c\uc6b0 \ud638\uac00 \uc8fc\uac15\uc744 \ud56d\ud574\ud560 \ub54c \ub2ec\uc558\ub358 \uae43\ubc1c\uc740?", "paragraph": "1856\ub144 10\uc6d4 8\uc77c \uc911\uad6d \uad6d\uc801 \ubc94\uc120 \uc560\ub85c\uc6b0 \ud638\uac00 \uc601\uad6d \uc0c1\uc120\uae30(the Red Ensign)\ub97c \ub2ec\uace0 \uc8fc\uac15\uc744 \ud56d\ud589 \uc911\uc774\uc5c8\ub2e4. \uc120\uc6d0 \uc911 \ud574\uc801\ub4e4\uc774 \uc704\uc7a5\uc7a0\uc785\ud574 \uc788\ub2e4\uba70 \uccad\uc758 \uad00\ub9ac\ub4e4\uc774 \uc120\uc7a5\uacfc \uc120\uc6d0 \uc804\uc6d0\uc744 \uccb4\ud3ec\ud558\ub824 \ud588\ub2e4. \uc120\uc7a5\uc740 \ubc30\uc5d0 \uc5c6\uc5c8\uace0 12\uba85\uc758 \uc120\uc6d0\uc744 \uccb4\ud3ec\ud55c \uadf8\ub4e4\uc740 \uc601\uad6d \uc0c1\uc120\uae30\ub97c \uac15\uc5d0 \ub358\uc838 \ubc84\ub838\ub2e4.(\uc560\ub85c\ud638 \uc0ac\uac74) \ud30c\ud06c\uc2a4\ub294 \uc774\ub97c \uac1c\uc804\uc758 \ud6cc\ub96d\ud55c \uacc4\uae30\ub85c \uc5ec\uacbc\ub2e4. \uc774\uc5d0 \uc601\uad6d\uc5d0 \ub300\ud55c \uc2ec\uac01\ud55c \ub3c4\uc804\uc774\ub77c\uba70 \uc5c4\uc911\ud55c \ud56d\uc758\ub97c \uad11\uc800\uc6b0 \ucd1d\ub3c5 \uc608\ubc0d\ucc9c\uc5d0\uac8c \uc804\ub2ec\ud588\ub2e4. \uc608\ucd1d\ub3c5\uc740 \uc0c1\uc120\uae30\uc77c \ubfd0 \uc601\uad6d \uad6d\uae30\ub3c4 \uc544\ub2c8\uace0 \ub354\uad6c\ub098 \uc911\uad6d \uc601\ud1a0\uc5d0\uc11c \ub2e8\uc9c0 \ud574\uc801\uc120\uc744 \ub098\ud3ec\ud55c \ub2f9\uc5f0\ud55c \uad8c\ub9ac\ub97c \ud589\uc0ac\ud55c \uac83\uc77c \ubfd0\uc774\ub77c\uba70 \uc77c\ucd95\ud588\ub2e4. \ud30c\ud06c\uc2a4\ub294 \uac1c\uc804\uc758 \ucda9\ubd84\ud55c \uba85\ubd84\uc73c\ub85c \uac04\uc8fc\ud558\uace0 \ucd08\ub300 \ud64d\ucf69 \ucd1d\ub3c5\uc774 \ub41c \uc874 \ubcf4\uc6b0\ub9c1 \uacbd\uc5d0\uac8c \uc989\uc2dc \ubcf4\uace0\ud588\ub2e4.", "answer": "\uc601\uad6d \uc0c1\uc120\uae30", "sentence": "1856\ub144 10\uc6d4 8\uc77c \uc911\uad6d \uad6d\uc801 \ubc94\uc120 \uc560\ub85c\uc6b0 \ud638\uac00 \uc601\uad6d \uc0c1\uc120\uae30 (the Red Ensign)", "paragraph_sentence": " 1856\ub144 10\uc6d4 8\uc77c \uc911\uad6d \uad6d\uc801 \ubc94\uc120 \uc560\ub85c\uc6b0 \ud638\uac00 \uc601\uad6d \uc0c1\uc120\uae30 (the Red Ensign) \ub97c \ub2ec\uace0 \uc8fc\uac15\uc744 \ud56d\ud589 \uc911\uc774\uc5c8\ub2e4. \uc120\uc6d0 \uc911 \ud574\uc801\ub4e4\uc774 \uc704\uc7a5\uc7a0\uc785\ud574 \uc788\ub2e4\uba70 \uccad\uc758 \uad00\ub9ac\ub4e4\uc774 \uc120\uc7a5\uacfc \uc120\uc6d0 \uc804\uc6d0\uc744 \uccb4\ud3ec\ud558\ub824 \ud588\ub2e4. \uc120\uc7a5\uc740 \ubc30\uc5d0 \uc5c6\uc5c8\uace0 12\uba85\uc758 \uc120\uc6d0\uc744 \uccb4\ud3ec\ud55c \uadf8\ub4e4\uc740 \uc601\uad6d \uc0c1\uc120\uae30\ub97c \uac15\uc5d0 \ub358\uc838 \ubc84\ub838\ub2e4.(\uc560\ub85c\ud638 \uc0ac\uac74) \ud30c\ud06c\uc2a4\ub294 \uc774\ub97c \uac1c\uc804\uc758 \ud6cc\ub96d\ud55c \uacc4\uae30\ub85c \uc5ec\uacbc\ub2e4. \uc774\uc5d0 \uc601\uad6d\uc5d0 \ub300\ud55c \uc2ec\uac01\ud55c \ub3c4\uc804\uc774\ub77c\uba70 \uc5c4\uc911\ud55c \ud56d\uc758\ub97c \uad11\uc800\uc6b0 \ucd1d\ub3c5 \uc608\ubc0d\ucc9c\uc5d0\uac8c \uc804\ub2ec\ud588\ub2e4. \uc608\ucd1d\ub3c5\uc740 \uc0c1\uc120\uae30\uc77c \ubfd0 \uc601\uad6d \uad6d\uae30\ub3c4 \uc544\ub2c8\uace0 \ub354\uad6c\ub098 \uc911\uad6d \uc601\ud1a0\uc5d0\uc11c \ub2e8\uc9c0 \ud574\uc801\uc120\uc744 \ub098\ud3ec\ud55c \ub2f9\uc5f0\ud55c \uad8c\ub9ac\ub97c \ud589\uc0ac\ud55c \uac83\uc77c \ubfd0\uc774\ub77c\uba70 \uc77c\ucd95\ud588\ub2e4. \ud30c\ud06c\uc2a4\ub294 \uac1c\uc804\uc758 \ucda9\ubd84\ud55c \uba85\ubd84\uc73c\ub85c \uac04\uc8fc\ud558\uace0 \ucd08\ub300 \ud64d\ucf69 \ucd1d\ub3c5\uc774 \ub41c \uc874 \ubcf4\uc6b0\ub9c1 \uacbd\uc5d0\uac8c \uc989\uc2dc \ubcf4\uace0\ud588\ub2e4.", "paragraph_answer": "1856\ub144 10\uc6d4 8\uc77c \uc911\uad6d \uad6d\uc801 \ubc94\uc120 \uc560\ub85c\uc6b0 \ud638\uac00 \uc601\uad6d \uc0c1\uc120\uae30 (the Red Ensign)\ub97c \ub2ec\uace0 \uc8fc\uac15\uc744 \ud56d\ud589 \uc911\uc774\uc5c8\ub2e4. \uc120\uc6d0 \uc911 \ud574\uc801\ub4e4\uc774 \uc704\uc7a5\uc7a0\uc785\ud574 \uc788\ub2e4\uba70 \uccad\uc758 \uad00\ub9ac\ub4e4\uc774 \uc120\uc7a5\uacfc \uc120\uc6d0 \uc804\uc6d0\uc744 \uccb4\ud3ec\ud558\ub824 \ud588\ub2e4. \uc120\uc7a5\uc740 \ubc30\uc5d0 \uc5c6\uc5c8\uace0 12\uba85\uc758 \uc120\uc6d0\uc744 \uccb4\ud3ec\ud55c \uadf8\ub4e4\uc740 \uc601\uad6d \uc0c1\uc120\uae30\ub97c \uac15\uc5d0 \ub358\uc838 \ubc84\ub838\ub2e4.(\uc560\ub85c\ud638 \uc0ac\uac74) \ud30c\ud06c\uc2a4\ub294 \uc774\ub97c \uac1c\uc804\uc758 \ud6cc\ub96d\ud55c \uacc4\uae30\ub85c \uc5ec\uacbc\ub2e4. \uc774\uc5d0 \uc601\uad6d\uc5d0 \ub300\ud55c \uc2ec\uac01\ud55c \ub3c4\uc804\uc774\ub77c\uba70 \uc5c4\uc911\ud55c \ud56d\uc758\ub97c \uad11\uc800\uc6b0 \ucd1d\ub3c5 \uc608\ubc0d\ucc9c\uc5d0\uac8c \uc804\ub2ec\ud588\ub2e4. \uc608\ucd1d\ub3c5\uc740 \uc0c1\uc120\uae30\uc77c \ubfd0 \uc601\uad6d \uad6d\uae30\ub3c4 \uc544\ub2c8\uace0 \ub354\uad6c\ub098 \uc911\uad6d \uc601\ud1a0\uc5d0\uc11c \ub2e8\uc9c0 \ud574\uc801\uc120\uc744 \ub098\ud3ec\ud55c \ub2f9\uc5f0\ud55c \uad8c\ub9ac\ub97c \ud589\uc0ac\ud55c \uac83\uc77c \ubfd0\uc774\ub77c\uba70 \uc77c\ucd95\ud588\ub2e4. \ud30c\ud06c\uc2a4\ub294 \uac1c\uc804\uc758 \ucda9\ubd84\ud55c \uba85\ubd84\uc73c\ub85c \uac04\uc8fc\ud558\uace0 \ucd08\ub300 \ud64d\ucf69 \ucd1d\ub3c5\uc774 \ub41c \uc874 \ubcf4\uc6b0\ub9c1 \uacbd\uc5d0\uac8c \uc989\uc2dc \ubcf4\uace0\ud588\ub2e4.", "sentence_answer": "1856\ub144 10\uc6d4 8\uc77c \uc911\uad6d \uad6d\uc801 \ubc94\uc120 \uc560\ub85c\uc6b0 \ud638\uac00 \uc601\uad6d \uc0c1\uc120\uae30 (the Red Ensign)", "paragraph_id": "6600738-2-1", "paragraph_question": "question: \uc560\ub85c\uc6b0 \ud638\uac00 \uc8fc\uac15\uc744 \ud56d\ud574\ud560 \ub54c \ub2ec\uc558\ub358 \uae43\ubc1c\uc740?, context: 1856\ub144 10\uc6d4 8\uc77c \uc911\uad6d \uad6d\uc801 \ubc94\uc120 \uc560\ub85c\uc6b0 \ud638\uac00 \uc601\uad6d \uc0c1\uc120\uae30(the Red Ensign)\ub97c \ub2ec\uace0 \uc8fc\uac15\uc744 \ud56d\ud589 \uc911\uc774\uc5c8\ub2e4. \uc120\uc6d0 \uc911 \ud574\uc801\ub4e4\uc774 \uc704\uc7a5\uc7a0\uc785\ud574 \uc788\ub2e4\uba70 \uccad\uc758 \uad00\ub9ac\ub4e4\uc774 \uc120\uc7a5\uacfc \uc120\uc6d0 \uc804\uc6d0\uc744 \uccb4\ud3ec\ud558\ub824 \ud588\ub2e4. \uc120\uc7a5\uc740 \ubc30\uc5d0 \uc5c6\uc5c8\uace0 12\uba85\uc758 \uc120\uc6d0\uc744 \uccb4\ud3ec\ud55c \uadf8\ub4e4\uc740 \uc601\uad6d \uc0c1\uc120\uae30\ub97c \uac15\uc5d0 \ub358\uc838 \ubc84\ub838\ub2e4.(\uc560\ub85c\ud638 \uc0ac\uac74) \ud30c\ud06c\uc2a4\ub294 \uc774\ub97c \uac1c\uc804\uc758 \ud6cc\ub96d\ud55c \uacc4\uae30\ub85c \uc5ec\uacbc\ub2e4. \uc774\uc5d0 \uc601\uad6d\uc5d0 \ub300\ud55c \uc2ec\uac01\ud55c \ub3c4\uc804\uc774\ub77c\uba70 \uc5c4\uc911\ud55c \ud56d\uc758\ub97c \uad11\uc800\uc6b0 \ucd1d\ub3c5 \uc608\ubc0d\ucc9c\uc5d0\uac8c \uc804\ub2ec\ud588\ub2e4. \uc608\ucd1d\ub3c5\uc740 \uc0c1\uc120\uae30\uc77c \ubfd0 \uc601\uad6d \uad6d\uae30\ub3c4 \uc544\ub2c8\uace0 \ub354\uad6c\ub098 \uc911\uad6d \uc601\ud1a0\uc5d0\uc11c \ub2e8\uc9c0 \ud574\uc801\uc120\uc744 \ub098\ud3ec\ud55c \ub2f9\uc5f0\ud55c \uad8c\ub9ac\ub97c \ud589\uc0ac\ud55c \uac83\uc77c \ubfd0\uc774\ub77c\uba70 \uc77c\ucd95\ud588\ub2e4. \ud30c\ud06c\uc2a4\ub294 \uac1c\uc804\uc758 \ucda9\ubd84\ud55c \uba85\ubd84\uc73c\ub85c \uac04\uc8fc\ud558\uace0 \ucd08\ub300 \ud64d\ucf69 \ucd1d\ub3c5\uc774 \ub41c \uc874 \ubcf4\uc6b0\ub9c1 \uacbd\uc5d0\uac8c \uc989\uc2dc \ubcf4\uace0\ud588\ub2e4."} {"question": "\ud56d\uc131 \ud669\ub3c4\ub300\uc5d0 \uc788\ub294 \ub098\ud06c\uc0e4\ud2b8\ub77c\ub294 \uadf8 \uac1c\uc218\ub97c \uace0\ub824\ud560 \ub54c \ud558\ub098 \ub2f9 \uba87 \ub3c4\uc778\uac00?", "paragraph": "\uc804\ud1b5\uc801\uc778 \ud78c\ub450 \uc810\uc131\uc220\uc740 \uae30\ubcf8\uc801\uc73c\ub85c \ub098\ud06c\uc0e4\ud2b8\ub77c\ub77c\uace0 \ubd88\ub9ac\ub294 \ud56d\uc131\uad70\uc5d0 \uc788\ub294 \ud589\uc131\uc758 \uace0\uc591\uc5d0 \ub300\ud55c \uac1c\ub150\uc744 \uac16\uace0 \uc788\ub2e4. \ud56d\uc131 \ud669\ub3c4\ub300\uc5d0\ub294 27\uac1c\uc758 \ub098\ud06c\uc0e4\ud2b8\ub77c\uac00 \uc788\ub2e4. 360\u00b0/27\ub85c \ud558\uc5ec 1\u00b0\uac00 60\u2032\uc784\uc744 \uac10\uc548\ud558\uba74 \ud55c \uac1c\uc758 \ub098\ud06c\uc0e4\ud2b8\ub77c \ub2f9 \uc815\ud655\ud788 13\u00b0 20\u2032\uc774\ub2e4. \uc608\ub97c \ub4e4\uc5b4, \ubaa9\uc131\uc740 \uac8c\uc790\ub9ac\uc5d0\uc11c \uace0\uc591\ud558\uc9c0\ub9cc, \uac8c\uc790\ub9ac\uc758 30\u00b0 \ub0b4\uc5d0\ub294 (\uc30d\ub465\uc774\uc790\ub9ac 20\u00b000\u2032\ubd80\ud130 \uac8c\uc790\ub9ac 3\u00b020\uae4c\uc9c0\uc758) \ud478\ub098\ubc14\uc218\uc640 (\uac8c\uc790\ub9ac 3\u00b020\ubd80\ud130 16\u00b040\u2032\uae4c\uc9c0\uc758) \ud478\uc2dc\uc57c \uadf8\ub9ac\uace0 (\uac8c\uc790\ub9ac 16\u00b040\u2032\ubd80\ud130 29\u00b059\u2032\uae4c\uc9c0\uc758) \uc544\uc290\ub808\uc0e4 \uc774\ub807\uac8c \uc138 \uac1c\uc758 \ub098\ud06c\uc0e4\ud2b8\ub77c\uac00 \uc788\ub2e4. \ubaa9\uc131\uc740 \uac8c\uc790\ub9ac 5\u00b0 \uc5d0\uc11c \uace0\uc591\ud558\ubbc0\ub85c, \uadf8 \uc704\uce58\ub294 \ud478\uc2dc\uc57c \ub098\ud06c\uc0e4\ud2b8\ub77c\uc5d0\uc11c\uc758 \uc9c4\uc9dc \uace0\uc591\uc744 \uc758\ubbf8\ud55c\ub2e4. \ud478\uc2dc\uc57c \ub098\ud06c\uc0e4\ud2b8\ub77c \ub370\ube44\ud0c0\ub294 \uc2e0\ub4e4\uc758 \uc2a4\uc2b9\uc778 \ube0c\ub77c\uc2a4\ud30c\ud2f0\uc774\ub2e4. \ubaa9\uc131\uc740 \uc77c\ubc18\uc801\uc73c\ub85c \uac8c\uc790\ub9ac\uc5d0\uc11c \uace0\uc591\ud558\uc9c0\ub9cc, \ud478\uc2dc\uc57c \ubc14\ub85c \uc606\uc758 \ub098\ud06c\uc0e4\ud2b8\ub77c\uc5d0 \uc788\uc744 \ub54c\ub294 \uadf8\uac83\uc758 \uc644\uc804\ud55c \uace0\uc591\uc774 \ubd80\uc5ec\ub418\uc9c0 \uc54a\ub294\ub2e4. \uac8c\ub2e4\uac00, \uac01\uac01\uc758 \ub098\ud06c\uc0e4\ud2b8\ub77c\ub294 \ud30c\ub2e4\ub77c\uace0 \ubd88\ub9ac\ub294 \ub124 \uad6c\uac04\uc73c\ub85c \ub098\ub258\ub294\ub370, 13\u00b020\u2032/4\ub85c \ud558\uba74 \ud55c \ud30c\ub2e4 \ub2f9 \uc815\ud655\ud788 3\u00b020\u2032 \uc774\ub2e4. \ubca0\ub2e4\uc758 \uc804\ud1b5\uc5d0 \ub530\ub974\uba74, \ub124 \uac1c\uc758 \ud30c\ub2e4\ub294 \uac01\uac01 \ub2e4\ub974\ub9c8\uc640 \uc544\ub974\ud0c0, \uce74\ub9c8 \uadf8\ub9ac\uace0 \ubaa8\ud06c\uc0ac \uc774\ub807\uac8c \ub124 \uac1c\uc758 \uc0b6\uc758 \ubaa9\ud45c\ub97c \ub098\ud0c0\ub0b8\ub2e4. \ub9cc\uc77c, \ubaa9\uc131\uc774 \ud478\uc2dc\uc57c\uc5d0\uc11c \uac15\ud558\uac8c \uace0\uc591\ud558\uace0 \uc788\ub2e4\uba74, (\uac8c\uc790\ub9ac 3\u00b020\u2032~6\u00b040\u2032\uc758) \ub2e4\ub974\ub9c8 \ud30c\ub2e4, (\uac8c\uc790\ub9ac 6\u00b040\u2032~10\u00b000\u2032\uc758) \uc544\ub974\ud0c0 \ud30c\ub2e4, (\uac8c\uc790\ub9ac 10\u00b000\u2032~13\u00b020\u2032\uc758) \uce74\ub9c8 \ud30c\ub2e4 \uadf8\ub9ac\uace0 (\uac8c\uc790\ub9ac 13\u00b020\u2032-16\u00b040\u2032\uc758) \ubaa8\uadf8\uc0ac \ud30c\ub2e4 \uac00\uc6b4\ub370 \ud558\ub098\uc5d0 \uc18d\ud55c \uac83\uc774\uae30\ub3c4 \ud558\ub2e4. \uc5ec\uae30\uc11c\uc758 \ub3c4\uc218\ub294 \ucc9c\ubb38\uc801\uc778 \uac83\uc73c\ub85c \ucd94\ub860\ub418\ub294\ub370, \ubaa9\uc131\uc740 \uac8c\uc790\ub9ac\uc758 5\u00b0\uc5d0\uc11c \uace0\uc591\ud558\ubbc0\ub85c, \uadf8\ub7ec\ud568\uc740 \ud478\uc2dc\uc57c \ub098\ud06c\uc0e4\ud2b8\ub77c\uc758 \ub2e4\ub974\ub9c8 \ud30c\ub2e4\uc758 \ub9c8\uc74c\uc5d0\uc11c\uc758 \"\uae4a\uc740\" \uace0\uc591\uc744 \uc758\ubbf8\ud55c\ub2e4. \ud78c\ub450 \uc810\uc131\uc220 \ube44\uc804\uc5d0\uc11c, \ubaa9\uc131\uc774 (\ub9c8\uc74c\uc744 \ub73b\ud558\ub294) \uac8c\uc790\ub9ac\uc5d0 \uc788\uace0 (\uc2e0\ub4e4\uc758 \uc2a4\uc2b9\uc778) \ube0c\ub77c\uc2a4\ud30c\ud2f0\uc758 \uc601\ud5a5\uc744 \ubc1b\uc558\uc73c\uba70, (\ub2e4\ub974\ub9c8 \ud30c\ub2e4\uc758) \uc815\ub2f9\ud55c \uc704\uce58\uc5d0 \uc788\ub2e4\uba74, \ubaa9\uc131\uc740 \uadf8\uac83\uc774 \ud45c\ud604\ud558\ub294 \ubaa8\ub4e0 \uac83\uc5d0 \uad00\ud558\uc5ec 100% \uace0\uc591\ub41c \uae30\ub2a5\uc744 \ud558\uace0 \uc788\ub294 \uac83\uc774\ub2e4", "answer": "13\u00b0 20\u2032", "sentence": "\ub2f9 \uc815\ud655\ud788 13\u00b0 20\u2032 \uc774\ub2e4.", "paragraph_sentence": "\uc804\ud1b5\uc801\uc778 \ud78c\ub450 \uc810\uc131\uc220\uc740 \uae30\ubcf8\uc801\uc73c\ub85c \ub098\ud06c\uc0e4\ud2b8\ub77c\ub77c\uace0 \ubd88\ub9ac\ub294 \ud56d\uc131\uad70\uc5d0 \uc788\ub294 \ud589\uc131\uc758 \uace0\uc591\uc5d0 \ub300\ud55c \uac1c\ub150\uc744 \uac16\uace0 \uc788\ub2e4. \ud56d\uc131 \ud669\ub3c4\ub300\uc5d0\ub294 27\uac1c\uc758 \ub098\ud06c\uc0e4\ud2b8\ub77c\uac00 \uc788\ub2e4. 360\u00b0/27\ub85c \ud558\uc5ec 1\u00b0\uac00 60\u2032\uc784\uc744 \uac10\uc548\ud558\uba74 \ud55c \uac1c\uc758 \ub098\ud06c\uc0e4\ud2b8\ub77c \ub2f9 \uc815\ud655\ud788 13\u00b0 20\u2032 \uc774\ub2e4. \uc608\ub97c \ub4e4\uc5b4, \ubaa9\uc131\uc740 \uac8c\uc790\ub9ac\uc5d0\uc11c \uace0\uc591\ud558\uc9c0\ub9cc, \uac8c\uc790\ub9ac\uc758 30\u00b0 \ub0b4\uc5d0\ub294 (\uc30d\ub465\uc774\uc790\ub9ac 20\u00b000\u2032\ubd80\ud130 \uac8c\uc790\ub9ac 3\u00b020\uae4c\uc9c0\uc758) \ud478\ub098\ubc14\uc218\uc640 (\uac8c\uc790\ub9ac 3\u00b020\ubd80\ud130 16\u00b040\u2032\uae4c\uc9c0\uc758) \ud478\uc2dc\uc57c \uadf8\ub9ac\uace0 (\uac8c\uc790\ub9ac 16\u00b040\u2032\ubd80\ud130 29\u00b059\u2032\uae4c\uc9c0\uc758) \uc544\uc290\ub808\uc0e4 \uc774\ub807\uac8c \uc138 \uac1c\uc758 \ub098\ud06c\uc0e4\ud2b8\ub77c\uac00 \uc788\ub2e4. \ubaa9\uc131\uc740 \uac8c\uc790\ub9ac 5\u00b0 \uc5d0\uc11c \uace0\uc591\ud558\ubbc0\ub85c, \uadf8 \uc704\uce58\ub294 \ud478\uc2dc\uc57c \ub098\ud06c\uc0e4\ud2b8\ub77c\uc5d0\uc11c\uc758 \uc9c4\uc9dc \uace0\uc591\uc744 \uc758\ubbf8\ud55c\ub2e4. \ud478\uc2dc\uc57c \ub098\ud06c\uc0e4\ud2b8\ub77c \ub370\ube44\ud0c0\ub294 \uc2e0\ub4e4\uc758 \uc2a4\uc2b9\uc778 \ube0c\ub77c\uc2a4\ud30c\ud2f0\uc774\ub2e4. \ubaa9\uc131\uc740 \uc77c\ubc18\uc801\uc73c\ub85c \uac8c\uc790\ub9ac\uc5d0\uc11c \uace0\uc591\ud558\uc9c0\ub9cc, \ud478\uc2dc\uc57c \ubc14\ub85c \uc606\uc758 \ub098\ud06c\uc0e4\ud2b8\ub77c\uc5d0 \uc788\uc744 \ub54c\ub294 \uadf8\uac83\uc758 \uc644\uc804\ud55c \uace0\uc591\uc774 \ubd80\uc5ec\ub418\uc9c0 \uc54a\ub294\ub2e4. \uac8c\ub2e4\uac00, \uac01\uac01\uc758 \ub098\ud06c\uc0e4\ud2b8\ub77c\ub294 \ud30c\ub2e4\ub77c\uace0 \ubd88\ub9ac\ub294 \ub124 \uad6c\uac04\uc73c\ub85c \ub098\ub258\ub294\ub370, 13\u00b020\u2032/4\ub85c \ud558\uba74 \ud55c \ud30c\ub2e4 \ub2f9 \uc815\ud655\ud788 3\u00b020\u2032 \uc774\ub2e4. \ubca0\ub2e4\uc758 \uc804\ud1b5\uc5d0 \ub530\ub974\uba74, \ub124 \uac1c\uc758 \ud30c\ub2e4\ub294 \uac01\uac01 \ub2e4\ub974\ub9c8\uc640 \uc544\ub974\ud0c0, \uce74\ub9c8 \uadf8\ub9ac\uace0 \ubaa8\ud06c\uc0ac \uc774\ub807\uac8c \ub124 \uac1c\uc758 \uc0b6\uc758 \ubaa9\ud45c\ub97c \ub098\ud0c0\ub0b8\ub2e4. \ub9cc\uc77c, \ubaa9\uc131\uc774 \ud478\uc2dc\uc57c\uc5d0\uc11c \uac15\ud558\uac8c \uace0\uc591\ud558\uace0 \uc788\ub2e4\uba74, (\uac8c\uc790\ub9ac 3\u00b020\u2032~6\u00b040\u2032\uc758) \ub2e4\ub974\ub9c8 \ud30c\ub2e4, (\uac8c\uc790\ub9ac 6\u00b040\u2032~10\u00b000\u2032\uc758) \uc544\ub974\ud0c0 \ud30c\ub2e4, (\uac8c\uc790\ub9ac 10\u00b000\u2032~13\u00b020\u2032\uc758) \uce74\ub9c8 \ud30c\ub2e4 \uadf8\ub9ac\uace0 (\uac8c\uc790\ub9ac 13\u00b020\u2032-16\u00b040\u2032\uc758) \ubaa8\uadf8\uc0ac \ud30c\ub2e4 \uac00\uc6b4\ub370 \ud558\ub098\uc5d0 \uc18d\ud55c \uac83\uc774\uae30\ub3c4 \ud558\ub2e4. \uc5ec\uae30\uc11c\uc758 \ub3c4\uc218\ub294 \ucc9c\ubb38\uc801\uc778 \uac83\uc73c\ub85c \ucd94\ub860\ub418\ub294\ub370, \ubaa9\uc131\uc740 \uac8c\uc790\ub9ac\uc758 5\u00b0\uc5d0\uc11c \uace0\uc591\ud558\ubbc0\ub85c, \uadf8\ub7ec\ud568\uc740 \ud478\uc2dc\uc57c \ub098\ud06c\uc0e4\ud2b8\ub77c\uc758 \ub2e4\ub974\ub9c8 \ud30c\ub2e4\uc758 \ub9c8\uc74c\uc5d0\uc11c\uc758 \"\uae4a\uc740\" \uace0\uc591\uc744 \uc758\ubbf8\ud55c\ub2e4. \ud78c\ub450 \uc810\uc131\uc220 \ube44\uc804\uc5d0\uc11c, \ubaa9\uc131\uc774 (\ub9c8\uc74c\uc744 \ub73b\ud558\ub294) \uac8c\uc790\ub9ac\uc5d0 \uc788\uace0 (\uc2e0\ub4e4\uc758 \uc2a4\uc2b9\uc778) \ube0c\ub77c\uc2a4\ud30c\ud2f0\uc758 \uc601\ud5a5\uc744 \ubc1b\uc558\uc73c\uba70, (\ub2e4\ub974\ub9c8 \ud30c\ub2e4\uc758) \uc815\ub2f9\ud55c \uc704\uce58\uc5d0 \uc788\ub2e4\uba74, \ubaa9\uc131\uc740 \uadf8\uac83\uc774 \ud45c\ud604\ud558\ub294 \ubaa8\ub4e0 \uac83\uc5d0 \uad00\ud558\uc5ec 100% \uace0\uc591\ub41c \uae30\ub2a5\uc744 \ud558\uace0 \uc788\ub294 \uac83\uc774\ub2e4", "paragraph_answer": "\uc804\ud1b5\uc801\uc778 \ud78c\ub450 \uc810\uc131\uc220\uc740 \uae30\ubcf8\uc801\uc73c\ub85c \ub098\ud06c\uc0e4\ud2b8\ub77c\ub77c\uace0 \ubd88\ub9ac\ub294 \ud56d\uc131\uad70\uc5d0 \uc788\ub294 \ud589\uc131\uc758 \uace0\uc591\uc5d0 \ub300\ud55c \uac1c\ub150\uc744 \uac16\uace0 \uc788\ub2e4. \ud56d\uc131 \ud669\ub3c4\ub300\uc5d0\ub294 27\uac1c\uc758 \ub098\ud06c\uc0e4\ud2b8\ub77c\uac00 \uc788\ub2e4. 360\u00b0/27\ub85c \ud558\uc5ec 1\u00b0\uac00 60\u2032\uc784\uc744 \uac10\uc548\ud558\uba74 \ud55c \uac1c\uc758 \ub098\ud06c\uc0e4\ud2b8\ub77c \ub2f9 \uc815\ud655\ud788 13\u00b0 20\u2032 \uc774\ub2e4. \uc608\ub97c \ub4e4\uc5b4, \ubaa9\uc131\uc740 \uac8c\uc790\ub9ac\uc5d0\uc11c \uace0\uc591\ud558\uc9c0\ub9cc, \uac8c\uc790\ub9ac\uc758 30\u00b0 \ub0b4\uc5d0\ub294 (\uc30d\ub465\uc774\uc790\ub9ac 20\u00b000\u2032\ubd80\ud130 \uac8c\uc790\ub9ac 3\u00b020\uae4c\uc9c0\uc758) \ud478\ub098\ubc14\uc218\uc640 (\uac8c\uc790\ub9ac 3\u00b020\ubd80\ud130 16\u00b040\u2032\uae4c\uc9c0\uc758) \ud478\uc2dc\uc57c \uadf8\ub9ac\uace0 (\uac8c\uc790\ub9ac 16\u00b040\u2032\ubd80\ud130 29\u00b059\u2032\uae4c\uc9c0\uc758) \uc544\uc290\ub808\uc0e4 \uc774\ub807\uac8c \uc138 \uac1c\uc758 \ub098\ud06c\uc0e4\ud2b8\ub77c\uac00 \uc788\ub2e4. \ubaa9\uc131\uc740 \uac8c\uc790\ub9ac 5\u00b0 \uc5d0\uc11c \uace0\uc591\ud558\ubbc0\ub85c, \uadf8 \uc704\uce58\ub294 \ud478\uc2dc\uc57c \ub098\ud06c\uc0e4\ud2b8\ub77c\uc5d0\uc11c\uc758 \uc9c4\uc9dc \uace0\uc591\uc744 \uc758\ubbf8\ud55c\ub2e4. \ud478\uc2dc\uc57c \ub098\ud06c\uc0e4\ud2b8\ub77c \ub370\ube44\ud0c0\ub294 \uc2e0\ub4e4\uc758 \uc2a4\uc2b9\uc778 \ube0c\ub77c\uc2a4\ud30c\ud2f0\uc774\ub2e4. \ubaa9\uc131\uc740 \uc77c\ubc18\uc801\uc73c\ub85c \uac8c\uc790\ub9ac\uc5d0\uc11c \uace0\uc591\ud558\uc9c0\ub9cc, \ud478\uc2dc\uc57c \ubc14\ub85c \uc606\uc758 \ub098\ud06c\uc0e4\ud2b8\ub77c\uc5d0 \uc788\uc744 \ub54c\ub294 \uadf8\uac83\uc758 \uc644\uc804\ud55c \uace0\uc591\uc774 \ubd80\uc5ec\ub418\uc9c0 \uc54a\ub294\ub2e4. \uac8c\ub2e4\uac00, \uac01\uac01\uc758 \ub098\ud06c\uc0e4\ud2b8\ub77c\ub294 \ud30c\ub2e4\ub77c\uace0 \ubd88\ub9ac\ub294 \ub124 \uad6c\uac04\uc73c\ub85c \ub098\ub258\ub294\ub370, 13\u00b020\u2032/4\ub85c \ud558\uba74 \ud55c \ud30c\ub2e4 \ub2f9 \uc815\ud655\ud788 3\u00b020\u2032 \uc774\ub2e4. \ubca0\ub2e4\uc758 \uc804\ud1b5\uc5d0 \ub530\ub974\uba74, \ub124 \uac1c\uc758 \ud30c\ub2e4\ub294 \uac01\uac01 \ub2e4\ub974\ub9c8\uc640 \uc544\ub974\ud0c0, \uce74\ub9c8 \uadf8\ub9ac\uace0 \ubaa8\ud06c\uc0ac \uc774\ub807\uac8c \ub124 \uac1c\uc758 \uc0b6\uc758 \ubaa9\ud45c\ub97c \ub098\ud0c0\ub0b8\ub2e4. \ub9cc\uc77c, \ubaa9\uc131\uc774 \ud478\uc2dc\uc57c\uc5d0\uc11c \uac15\ud558\uac8c \uace0\uc591\ud558\uace0 \uc788\ub2e4\uba74, (\uac8c\uc790\ub9ac 3\u00b020\u2032~6\u00b040\u2032\uc758) \ub2e4\ub974\ub9c8 \ud30c\ub2e4, (\uac8c\uc790\ub9ac 6\u00b040\u2032~10\u00b000\u2032\uc758) \uc544\ub974\ud0c0 \ud30c\ub2e4, (\uac8c\uc790\ub9ac 10\u00b000\u2032~13\u00b020\u2032\uc758) \uce74\ub9c8 \ud30c\ub2e4 \uadf8\ub9ac\uace0 (\uac8c\uc790\ub9ac 13\u00b020\u2032-16\u00b040\u2032\uc758) \ubaa8\uadf8\uc0ac \ud30c\ub2e4 \uac00\uc6b4\ub370 \ud558\ub098\uc5d0 \uc18d\ud55c \uac83\uc774\uae30\ub3c4 \ud558\ub2e4. \uc5ec\uae30\uc11c\uc758 \ub3c4\uc218\ub294 \ucc9c\ubb38\uc801\uc778 \uac83\uc73c\ub85c \ucd94\ub860\ub418\ub294\ub370, \ubaa9\uc131\uc740 \uac8c\uc790\ub9ac\uc758 5\u00b0\uc5d0\uc11c \uace0\uc591\ud558\ubbc0\ub85c, \uadf8\ub7ec\ud568\uc740 \ud478\uc2dc\uc57c \ub098\ud06c\uc0e4\ud2b8\ub77c\uc758 \ub2e4\ub974\ub9c8 \ud30c\ub2e4\uc758 \ub9c8\uc74c\uc5d0\uc11c\uc758 \"\uae4a\uc740\" \uace0\uc591\uc744 \uc758\ubbf8\ud55c\ub2e4. \ud78c\ub450 \uc810\uc131\uc220 \ube44\uc804\uc5d0\uc11c, \ubaa9\uc131\uc774 (\ub9c8\uc74c\uc744 \ub73b\ud558\ub294) \uac8c\uc790\ub9ac\uc5d0 \uc788\uace0 (\uc2e0\ub4e4\uc758 \uc2a4\uc2b9\uc778) \ube0c\ub77c\uc2a4\ud30c\ud2f0\uc758 \uc601\ud5a5\uc744 \ubc1b\uc558\uc73c\uba70, (\ub2e4\ub974\ub9c8 \ud30c\ub2e4\uc758) \uc815\ub2f9\ud55c \uc704\uce58\uc5d0 \uc788\ub2e4\uba74, \ubaa9\uc131\uc740 \uadf8\uac83\uc774 \ud45c\ud604\ud558\ub294 \ubaa8\ub4e0 \uac83\uc5d0 \uad00\ud558\uc5ec 100% \uace0\uc591\ub41c \uae30\ub2a5\uc744 \ud558\uace0 \uc788\ub294 \uac83\uc774\ub2e4", "sentence_answer": "\ub2f9 \uc815\ud655\ud788 13\u00b0 20\u2032 \uc774\ub2e4.", "paragraph_id": "6355134-1-0", "paragraph_question": "question: \ud56d\uc131 \ud669\ub3c4\ub300\uc5d0 \uc788\ub294 \ub098\ud06c\uc0e4\ud2b8\ub77c\ub294 \uadf8 \uac1c\uc218\ub97c \uace0\ub824\ud560 \ub54c \ud558\ub098 \ub2f9 \uba87 \ub3c4\uc778\uac00?, context: \uc804\ud1b5\uc801\uc778 \ud78c\ub450 \uc810\uc131\uc220\uc740 \uae30\ubcf8\uc801\uc73c\ub85c \ub098\ud06c\uc0e4\ud2b8\ub77c\ub77c\uace0 \ubd88\ub9ac\ub294 \ud56d\uc131\uad70\uc5d0 \uc788\ub294 \ud589\uc131\uc758 \uace0\uc591\uc5d0 \ub300\ud55c \uac1c\ub150\uc744 \uac16\uace0 \uc788\ub2e4. \ud56d\uc131 \ud669\ub3c4\ub300\uc5d0\ub294 27\uac1c\uc758 \ub098\ud06c\uc0e4\ud2b8\ub77c\uac00 \uc788\ub2e4. 360\u00b0/27\ub85c \ud558\uc5ec 1\u00b0\uac00 60\u2032\uc784\uc744 \uac10\uc548\ud558\uba74 \ud55c \uac1c\uc758 \ub098\ud06c\uc0e4\ud2b8\ub77c \ub2f9 \uc815\ud655\ud788 13\u00b0 20\u2032\uc774\ub2e4. \uc608\ub97c \ub4e4\uc5b4, \ubaa9\uc131\uc740 \uac8c\uc790\ub9ac\uc5d0\uc11c \uace0\uc591\ud558\uc9c0\ub9cc, \uac8c\uc790\ub9ac\uc758 30\u00b0 \ub0b4\uc5d0\ub294 (\uc30d\ub465\uc774\uc790\ub9ac 20\u00b000\u2032\ubd80\ud130 \uac8c\uc790\ub9ac 3\u00b020\uae4c\uc9c0\uc758) \ud478\ub098\ubc14\uc218\uc640 (\uac8c\uc790\ub9ac 3\u00b020\ubd80\ud130 16\u00b040\u2032\uae4c\uc9c0\uc758) \ud478\uc2dc\uc57c \uadf8\ub9ac\uace0 (\uac8c\uc790\ub9ac 16\u00b040\u2032\ubd80\ud130 29\u00b059\u2032\uae4c\uc9c0\uc758) \uc544\uc290\ub808\uc0e4 \uc774\ub807\uac8c \uc138 \uac1c\uc758 \ub098\ud06c\uc0e4\ud2b8\ub77c\uac00 \uc788\ub2e4. \ubaa9\uc131\uc740 \uac8c\uc790\ub9ac 5\u00b0 \uc5d0\uc11c \uace0\uc591\ud558\ubbc0\ub85c, \uadf8 \uc704\uce58\ub294 \ud478\uc2dc\uc57c \ub098\ud06c\uc0e4\ud2b8\ub77c\uc5d0\uc11c\uc758 \uc9c4\uc9dc \uace0\uc591\uc744 \uc758\ubbf8\ud55c\ub2e4. \ud478\uc2dc\uc57c \ub098\ud06c\uc0e4\ud2b8\ub77c \ub370\ube44\ud0c0\ub294 \uc2e0\ub4e4\uc758 \uc2a4\uc2b9\uc778 \ube0c\ub77c\uc2a4\ud30c\ud2f0\uc774\ub2e4. \ubaa9\uc131\uc740 \uc77c\ubc18\uc801\uc73c\ub85c \uac8c\uc790\ub9ac\uc5d0\uc11c \uace0\uc591\ud558\uc9c0\ub9cc, \ud478\uc2dc\uc57c \ubc14\ub85c \uc606\uc758 \ub098\ud06c\uc0e4\ud2b8\ub77c\uc5d0 \uc788\uc744 \ub54c\ub294 \uadf8\uac83\uc758 \uc644\uc804\ud55c \uace0\uc591\uc774 \ubd80\uc5ec\ub418\uc9c0 \uc54a\ub294\ub2e4. \uac8c\ub2e4\uac00, \uac01\uac01\uc758 \ub098\ud06c\uc0e4\ud2b8\ub77c\ub294 \ud30c\ub2e4\ub77c\uace0 \ubd88\ub9ac\ub294 \ub124 \uad6c\uac04\uc73c\ub85c \ub098\ub258\ub294\ub370, 13\u00b020\u2032/4\ub85c \ud558\uba74 \ud55c \ud30c\ub2e4 \ub2f9 \uc815\ud655\ud788 3\u00b020\u2032 \uc774\ub2e4. \ubca0\ub2e4\uc758 \uc804\ud1b5\uc5d0 \ub530\ub974\uba74, \ub124 \uac1c\uc758 \ud30c\ub2e4\ub294 \uac01\uac01 \ub2e4\ub974\ub9c8\uc640 \uc544\ub974\ud0c0, \uce74\ub9c8 \uadf8\ub9ac\uace0 \ubaa8\ud06c\uc0ac \uc774\ub807\uac8c \ub124 \uac1c\uc758 \uc0b6\uc758 \ubaa9\ud45c\ub97c \ub098\ud0c0\ub0b8\ub2e4. \ub9cc\uc77c, \ubaa9\uc131\uc774 \ud478\uc2dc\uc57c\uc5d0\uc11c \uac15\ud558\uac8c \uace0\uc591\ud558\uace0 \uc788\ub2e4\uba74, (\uac8c\uc790\ub9ac 3\u00b020\u2032~6\u00b040\u2032\uc758) \ub2e4\ub974\ub9c8 \ud30c\ub2e4, (\uac8c\uc790\ub9ac 6\u00b040\u2032~10\u00b000\u2032\uc758) \uc544\ub974\ud0c0 \ud30c\ub2e4, (\uac8c\uc790\ub9ac 10\u00b000\u2032~13\u00b020\u2032\uc758) \uce74\ub9c8 \ud30c\ub2e4 \uadf8\ub9ac\uace0 (\uac8c\uc790\ub9ac 13\u00b020\u2032-16\u00b040\u2032\uc758) \ubaa8\uadf8\uc0ac \ud30c\ub2e4 \uac00\uc6b4\ub370 \ud558\ub098\uc5d0 \uc18d\ud55c \uac83\uc774\uae30\ub3c4 \ud558\ub2e4. \uc5ec\uae30\uc11c\uc758 \ub3c4\uc218\ub294 \ucc9c\ubb38\uc801\uc778 \uac83\uc73c\ub85c \ucd94\ub860\ub418\ub294\ub370, \ubaa9\uc131\uc740 \uac8c\uc790\ub9ac\uc758 5\u00b0\uc5d0\uc11c \uace0\uc591\ud558\ubbc0\ub85c, \uadf8\ub7ec\ud568\uc740 \ud478\uc2dc\uc57c \ub098\ud06c\uc0e4\ud2b8\ub77c\uc758 \ub2e4\ub974\ub9c8 \ud30c\ub2e4\uc758 \ub9c8\uc74c\uc5d0\uc11c\uc758 \"\uae4a\uc740\" \uace0\uc591\uc744 \uc758\ubbf8\ud55c\ub2e4. \ud78c\ub450 \uc810\uc131\uc220 \ube44\uc804\uc5d0\uc11c, \ubaa9\uc131\uc774 (\ub9c8\uc74c\uc744 \ub73b\ud558\ub294) \uac8c\uc790\ub9ac\uc5d0 \uc788\uace0 (\uc2e0\ub4e4\uc758 \uc2a4\uc2b9\uc778) \ube0c\ub77c\uc2a4\ud30c\ud2f0\uc758 \uc601\ud5a5\uc744 \ubc1b\uc558\uc73c\uba70, (\ub2e4\ub974\ub9c8 \ud30c\ub2e4\uc758) \uc815\ub2f9\ud55c \uc704\uce58\uc5d0 \uc788\ub2e4\uba74, \ubaa9\uc131\uc740 \uadf8\uac83\uc774 \ud45c\ud604\ud558\ub294 \ubaa8\ub4e0 \uac83\uc5d0 \uad00\ud558\uc5ec 100% \uace0\uc591\ub41c \uae30\ub2a5\uc744 \ud558\uace0 \uc788\ub294 \uac83\uc774\ub2e4"} {"question": "\ud070\uace0\ub798\ub294 \uba87\uc30d\uc758 \uc228\uad6c\uba4d\uc744 \uac00\uc9c0\uace0 \uc788\ub098?", "paragraph": "\ud070\uace0\ub798\uc758 \ub4f1 \ubd80\ubd84\uc740 \ubc24\ud68c\uc0c9\uc774\uba70, \ubc30 \ucabd\uc740 \ud558\uc597\ub2e4. \ud280\uc5b4\ub098\uc628 \ub450 \uc30d\uc758 \uc228\uad6c\uba4d\uc774 \uc788\uc73c\uba70, \ub0a9\uc791\ud558\uace0 \ub113\uc740 \uc8fc\ub465\uc774\ub97c \uac00\uc9c0\uace0 \uc788\ub2e4. \ub450 \uac1c\uc758 \ubc1d\uc740 \uc0c9 \ubb38\uc591\uc774 \uc228\uad6c\uba4d \ub4a4\uc5d0\uc11c \uc2dc\uc791\ud574 \ubab8\uc758 \uce21\uba74\uc73c\ub85c \ub530\ub77c\uac00 \uaf2c\ub9ac\ub85c \uc774\uc5b4\uc9c4\ub2e4. \uc624\ub978\ucabd \ud131\uc5d0 \ud558\uc580\uc0c9 \ubb34\ub2ac\uac00 \uc788\uc73c\uba70, \uc67c\ucabd\uc740 \ud68c\uc0c9 \ub610\ub294 \uac80\uc740\uc0c9\uc774\ub2e4. \uc774\ub7ec\ud55c \ube44\ub300\uce6d\uc131\uc740 \uc1e0\uc815\uc5b4\ub9ac\uace0\ub798\uc5d0\uac8c\ub3c4 \uc885\uc885 \ubc1c\uacac\ub418\uc9c0\ub9cc, \ud070\uace0\ub798\ub4e4\uc5d0\uac8c\ub294 \ube44\ub300\uce6d\uc131\uc774 \uc9c4\ubd80\ud558\uba70, \ub2e4\ub978 \uace0\ub798\uc5d0\uac8c\ub294 \uc774\ub7ec\ud55c \ud2b9\uc131\uc744 \ucc3e\uc544\ubcfc \uc218 \uc5c6\uc73c\ubbc0\ub85c \ub2e4\ub978 \uace0\ub798\ub4e4\uacfc \uad6c\ubd84\ud558\ub294 \ud55c \uac00\uc9c0 \ucc99\ub3c4\uac00 \ub418\uace0 \uc788\ub2e4. \ube44\ub300\uce6d\uc131\uc740 \uadf8\ub4e4\uc774 \ub3cc \ub54c \uc624\ub978\ucabd\uc73c\ub85c \ub3cc\uc544 \uadf8\ub807\uac8c \ub410\ub2e4\ub294 \uac00\uc124\uc774 \uc788\uc9c0\ub9cc, \uc2e4\uc81c\ub85c\ub294 \uc67c\ucabd\uc73c\ub85c\ub3c4 \ub3cc\uae30\ub3c4 \ud55c\ub2e4. \uc544\uc9c1\uae4c\uc9c0 \uc774\ub7ec\ud55c \ube44\ub300\uce6d\uc131\uc744 \uc790\uc138\ud788 \uc124\uba85\ud560 \ub9cc\ud55c \uac00\uc124\uc740 \uc5c6\ub2e4.", "answer": "\ub450 \uc30d", "sentence": "\ud280\uc5b4\ub098\uc628 \ub450 \uc30d \uc758 \uc228\uad6c\uba4d\uc774 \uc788\uc73c\uba70, \ub0a9\uc791\ud558\uace0 \ub113\uc740 \uc8fc\ub465\uc774\ub97c \uac00\uc9c0\uace0 \uc788\ub2e4.", "paragraph_sentence": "\ud070\uace0\ub798\uc758 \ub4f1 \ubd80\ubd84\uc740 \ubc24\ud68c\uc0c9\uc774\uba70, \ubc30 \ucabd\uc740 \ud558\uc597\ub2e4. \ud280\uc5b4\ub098\uc628 \ub450 \uc30d \uc758 \uc228\uad6c\uba4d\uc774 \uc788\uc73c\uba70, \ub0a9\uc791\ud558\uace0 \ub113\uc740 \uc8fc\ub465\uc774\ub97c \uac00\uc9c0\uace0 \uc788\ub2e4. \ub450 \uac1c\uc758 \ubc1d\uc740 \uc0c9 \ubb38\uc591\uc774 \uc228\uad6c\uba4d \ub4a4\uc5d0\uc11c \uc2dc\uc791\ud574 \ubab8\uc758 \uce21\uba74\uc73c\ub85c \ub530\ub77c\uac00 \uaf2c\ub9ac\ub85c \uc774\uc5b4\uc9c4\ub2e4. \uc624\ub978\ucabd \ud131\uc5d0 \ud558\uc580\uc0c9 \ubb34\ub2ac\uac00 \uc788\uc73c\uba70, \uc67c\ucabd\uc740 \ud68c\uc0c9 \ub610\ub294 \uac80\uc740\uc0c9\uc774\ub2e4. \uc774\ub7ec\ud55c \ube44\ub300\uce6d\uc131\uc740 \uc1e0\uc815\uc5b4\ub9ac\uace0\ub798\uc5d0\uac8c\ub3c4 \uc885\uc885 \ubc1c\uacac\ub418\uc9c0\ub9cc, \ud070\uace0\ub798\ub4e4\uc5d0\uac8c\ub294 \ube44\ub300\uce6d\uc131\uc774 \uc9c4\ubd80\ud558\uba70, \ub2e4\ub978 \uace0\ub798\uc5d0\uac8c\ub294 \uc774\ub7ec\ud55c \ud2b9\uc131\uc744 \ucc3e\uc544\ubcfc \uc218 \uc5c6\uc73c\ubbc0\ub85c \ub2e4\ub978 \uace0\ub798\ub4e4\uacfc \uad6c\ubd84\ud558\ub294 \ud55c \uac00\uc9c0 \ucc99\ub3c4\uac00 \ub418\uace0 \uc788\ub2e4. \ube44\ub300\uce6d\uc131\uc740 \uadf8\ub4e4\uc774 \ub3cc \ub54c \uc624\ub978\ucabd\uc73c\ub85c \ub3cc\uc544 \uadf8\ub807\uac8c \ub410\ub2e4\ub294 \uac00\uc124\uc774 \uc788\uc9c0\ub9cc, \uc2e4\uc81c\ub85c\ub294 \uc67c\ucabd\uc73c\ub85c\ub3c4 \ub3cc\uae30\ub3c4 \ud55c\ub2e4. \uc544\uc9c1\uae4c\uc9c0 \uc774\ub7ec\ud55c \ube44\ub300\uce6d\uc131\uc744 \uc790\uc138\ud788 \uc124\uba85\ud560 \ub9cc\ud55c \uac00\uc124\uc740 \uc5c6\ub2e4.", "paragraph_answer": "\ud070\uace0\ub798\uc758 \ub4f1 \ubd80\ubd84\uc740 \ubc24\ud68c\uc0c9\uc774\uba70, \ubc30 \ucabd\uc740 \ud558\uc597\ub2e4. \ud280\uc5b4\ub098\uc628 \ub450 \uc30d \uc758 \uc228\uad6c\uba4d\uc774 \uc788\uc73c\uba70, \ub0a9\uc791\ud558\uace0 \ub113\uc740 \uc8fc\ub465\uc774\ub97c \uac00\uc9c0\uace0 \uc788\ub2e4. \ub450 \uac1c\uc758 \ubc1d\uc740 \uc0c9 \ubb38\uc591\uc774 \uc228\uad6c\uba4d \ub4a4\uc5d0\uc11c \uc2dc\uc791\ud574 \ubab8\uc758 \uce21\uba74\uc73c\ub85c \ub530\ub77c\uac00 \uaf2c\ub9ac\ub85c \uc774\uc5b4\uc9c4\ub2e4. \uc624\ub978\ucabd \ud131\uc5d0 \ud558\uc580\uc0c9 \ubb34\ub2ac\uac00 \uc788\uc73c\uba70, \uc67c\ucabd\uc740 \ud68c\uc0c9 \ub610\ub294 \uac80\uc740\uc0c9\uc774\ub2e4. \uc774\ub7ec\ud55c \ube44\ub300\uce6d\uc131\uc740 \uc1e0\uc815\uc5b4\ub9ac\uace0\ub798\uc5d0\uac8c\ub3c4 \uc885\uc885 \ubc1c\uacac\ub418\uc9c0\ub9cc, \ud070\uace0\ub798\ub4e4\uc5d0\uac8c\ub294 \ube44\ub300\uce6d\uc131\uc774 \uc9c4\ubd80\ud558\uba70, \ub2e4\ub978 \uace0\ub798\uc5d0\uac8c\ub294 \uc774\ub7ec\ud55c \ud2b9\uc131\uc744 \ucc3e\uc544\ubcfc \uc218 \uc5c6\uc73c\ubbc0\ub85c \ub2e4\ub978 \uace0\ub798\ub4e4\uacfc \uad6c\ubd84\ud558\ub294 \ud55c \uac00\uc9c0 \ucc99\ub3c4\uac00 \ub418\uace0 \uc788\ub2e4. \ube44\ub300\uce6d\uc131\uc740 \uadf8\ub4e4\uc774 \ub3cc \ub54c \uc624\ub978\ucabd\uc73c\ub85c \ub3cc\uc544 \uadf8\ub807\uac8c \ub410\ub2e4\ub294 \uac00\uc124\uc774 \uc788\uc9c0\ub9cc, \uc2e4\uc81c\ub85c\ub294 \uc67c\ucabd\uc73c\ub85c\ub3c4 \ub3cc\uae30\ub3c4 \ud55c\ub2e4. \uc544\uc9c1\uae4c\uc9c0 \uc774\ub7ec\ud55c \ube44\ub300\uce6d\uc131\uc744 \uc790\uc138\ud788 \uc124\uba85\ud560 \ub9cc\ud55c \uac00\uc124\uc740 \uc5c6\ub2e4.", "sentence_answer": "\ud280\uc5b4\ub098\uc628 \ub450 \uc30d \uc758 \uc228\uad6c\uba4d\uc774 \uc788\uc73c\uba70, \ub0a9\uc791\ud558\uace0 \ub113\uc740 \uc8fc\ub465\uc774\ub97c \uac00\uc9c0\uace0 \uc788\ub2e4.", "paragraph_id": "6564198-3-2", "paragraph_question": "question: \ud070\uace0\ub798\ub294 \uba87\uc30d\uc758 \uc228\uad6c\uba4d\uc744 \uac00\uc9c0\uace0 \uc788\ub098?, context: \ud070\uace0\ub798\uc758 \ub4f1 \ubd80\ubd84\uc740 \ubc24\ud68c\uc0c9\uc774\uba70, \ubc30 \ucabd\uc740 \ud558\uc597\ub2e4. \ud280\uc5b4\ub098\uc628 \ub450 \uc30d\uc758 \uc228\uad6c\uba4d\uc774 \uc788\uc73c\uba70, \ub0a9\uc791\ud558\uace0 \ub113\uc740 \uc8fc\ub465\uc774\ub97c \uac00\uc9c0\uace0 \uc788\ub2e4. \ub450 \uac1c\uc758 \ubc1d\uc740 \uc0c9 \ubb38\uc591\uc774 \uc228\uad6c\uba4d \ub4a4\uc5d0\uc11c \uc2dc\uc791\ud574 \ubab8\uc758 \uce21\uba74\uc73c\ub85c \ub530\ub77c\uac00 \uaf2c\ub9ac\ub85c \uc774\uc5b4\uc9c4\ub2e4. \uc624\ub978\ucabd \ud131\uc5d0 \ud558\uc580\uc0c9 \ubb34\ub2ac\uac00 \uc788\uc73c\uba70, \uc67c\ucabd\uc740 \ud68c\uc0c9 \ub610\ub294 \uac80\uc740\uc0c9\uc774\ub2e4. \uc774\ub7ec\ud55c \ube44\ub300\uce6d\uc131\uc740 \uc1e0\uc815\uc5b4\ub9ac\uace0\ub798\uc5d0\uac8c\ub3c4 \uc885\uc885 \ubc1c\uacac\ub418\uc9c0\ub9cc, \ud070\uace0\ub798\ub4e4\uc5d0\uac8c\ub294 \ube44\ub300\uce6d\uc131\uc774 \uc9c4\ubd80\ud558\uba70, \ub2e4\ub978 \uace0\ub798\uc5d0\uac8c\ub294 \uc774\ub7ec\ud55c \ud2b9\uc131\uc744 \ucc3e\uc544\ubcfc \uc218 \uc5c6\uc73c\ubbc0\ub85c \ub2e4\ub978 \uace0\ub798\ub4e4\uacfc \uad6c\ubd84\ud558\ub294 \ud55c \uac00\uc9c0 \ucc99\ub3c4\uac00 \ub418\uace0 \uc788\ub2e4. \ube44\ub300\uce6d\uc131\uc740 \uadf8\ub4e4\uc774 \ub3cc \ub54c \uc624\ub978\ucabd\uc73c\ub85c \ub3cc\uc544 \uadf8\ub807\uac8c \ub410\ub2e4\ub294 \uac00\uc124\uc774 \uc788\uc9c0\ub9cc, \uc2e4\uc81c\ub85c\ub294 \uc67c\ucabd\uc73c\ub85c\ub3c4 \ub3cc\uae30\ub3c4 \ud55c\ub2e4. \uc544\uc9c1\uae4c\uc9c0 \uc774\ub7ec\ud55c \ube44\ub300\uce6d\uc131\uc744 \uc790\uc138\ud788 \uc124\uba85\ud560 \ub9cc\ud55c \uac00\uc124\uc740 \uc5c6\ub2e4."} {"question": "\uba85\ub098\ub77c\uc758 \uacfc\uac70\uc81c\ub3c4\uc778 \ud5a5\uc2dc, \ud68c\uc2dc, \uc804\uc2dc\ub294 \uc5b4\ub290 \ub098\ub77c\uc758 \uc81c\ub3c4\ub97c \uacc4\uc2b9\ud55c \uac83\uc778\uac00?", "paragraph": "\uba85\ub300\uc758 \uacfc\uac70\uc81c\ub3c4\ub294 \uc1a1\u00b7\uc6d0(\u5b8b\u5143)\uc758 \uc81c\ub3c4\ub97c \uacc4\uc2b9\ud558\uc5ec \ud5a5\uc2dc(\u9115\u8a66)\u00b7\ud68c\uc2dc(\u6703\u8a66)\u00b7\uc804\uc2dc(\u6bbf\u8a66)\uc758 3\uc885\uc774 \uc788\ub2e4. \ud5a5\uc2dc\ub294 \uac01\uc131(\u5404\u7701)\uc5d0\uc11c, \ud68c\uc2dc\ub294 \uc608\ubd80(\u79ae\u90e8)\uc5d0\uc11c \uc804\uc2dc\ub294 \uad81\uc804(\u5bae\u6bbf)\uc5d0\uc11c \uc2dc\ud589\ud558\uc600\ub2e4. \uac01 \ubd80\u00b7\uc8fc\u00b7\ud604\uc758 \ud559\uc0dd\uc740 \uc81c\uac70\ud559\uad00(\u63d0\u64e7\u5b78\u5b98)\uc774 \uc8fc\uad00\ud558\ub294 \uc138\uace0(\u6b72\u8003)\uc5d0 1\uae09\uc73c\ub85c \ud569\uaca9\ud568\uc73c\ub85c\uc368 \ud5a5\uc2dc\uc5d0 \uc751\uc2dc\ud560 \uc790\uaca9\uc744 \uc5bb\ub294\ub2e4. \ud5a5\uc2dc\uc5d0 \ud569\uaca9\ud558\uba74 \uac70\uc778(\u64e7\u4eba)\uc774 \ub418\uace0 \ud68c\uc2dc(\u6703\u8a66)\uc5d0 \uc751\uc2dc\ud560 \uc790\uaca9\uc744 \uc5bb\ub294\ub2e4. \ub610 \ud68c\uc2dc\uc5d0 \ud569\uaca9\ud558\uba74 \ucc9c\uc790(\u5929\u5b50)\uac00 \uc784\uc11d\ud558\uc5ec \uc2dc\ud589\ud558\ub294 \uc804\uc2dc\uc5d0 \uc751\uc2dc\ud55c\ub2e4. \uace0\uc2dc\ub0b4\uc6a9\uc740 \uacbd\uc758(\u7d93\u7fa9)\u00b7\uc18c\uace0\uc728\ub839(\u8a34\u8aa5\u5f8b\u4ee4)\u00b7\uacbd\uc0ac\uc2dc\ubb34\ucc45(\u7d93\u53f2\u6642\u52d9\u7b56) \ub4f1 3\uc885\uc73c\ub85c, \uacbd\uc758\ub294 4\uc11c(\u56db\u66f8) \ubc0f \uc5ed\u00b7\uc11c\u00b7\uc2dc\u00b7\uc608\u00b7\ucd98\ucd94(\u6613\u00b7\u66f8\u00b7\u8a69\u00b7\u79ae\u00b7\u6625\u79cb)\uc758 5\uacbd\uc5d0\uc11c \ucd9c\uc81c\ud55c\ub2e4. \ud5a5\uc2dc\uc640 \ud68c\uc2dc\ub294 \uac01\uac01 3\ucc28\uc758 \uc2dc\ud5d8\uc744 \uac70\uce5c\ub2e4. 1\ucc28\ub294 4\uc11c\uc758(\u56db\u66f8\u7fa9) 3\ubb38\uc81c\u00b7\uacbd\uc758 4\ubb38\uc81c\ub97c \uc791\uc131\ud558\ub294 \uac83\uc774\ub2e4. 2\ucc28\ub294 \ub17c\ubb38 \ud55c \ud3b8\uc744 300\uc790 \uc774\uc0c1 \uc791\uc131\ud558\ub294 \uac83\uacfc \uc728\ub839\uc5d0 \uad00\ud55c \uac83 \ud55c \ud3b8, \uc81c\ub3c4\uc5d0 \uad00\ud55c \uac83 5\uc870\ubaa9\uc744 \uc4f0\ub294 \uac83\uc774\ub2e4. 3\ucc28\ub294 \uacbd\uc0ac\uc2dc\ubb34\ucc45(\u7d93\u53f2\u6642\u52d9\u7b56) 5\ubb38\uc81c\ub97c \uc791\uc131\ud558\ub294 \uac83\uc774\ub2e4. \uadf8\uc911 1\ucc28 \uc2dc\ud5d8\uc774 \uac00\uc7a5 \uc911\uc2dc\ub418\uc5c8\uace0, \ub2f5\uc548 \uc791\uc131\uc740 \uc61b \uc0ac\ub78c\uc758 \ubb38\uccb4\ub97c \ubaa8\ubc29\ud558\uc5ec \uaca9\ub960(\u683c\u5f8b)\uc774 \uc5c4\uaca9\ud558\uac8c \uc815\ud574\uc9c4 8\uace0\ubb38(\u516b\u80a1\u6587)\uc73c\ub85c \ud558\uc5ec\uc57c \ud588\ub2e4. 8\uace0\ubb38\uc740 \uaca9\uc2dd\uc744 \uc815\ud558\uc5ec, \uc124\uba85\uc5d0 \ud3b8\ub9ac\ud558\uace0 \uc6b4\uc728\uc5d0 \ub9de\ucd94\uc5b4 \ub4e3\uae30 \uc88b\ub3c4\ub85d \ud558\uae30 \uc704\ud574 \ub9cc\ub4e4\uc5b4\uc9c4 \uc77c\uc885\uc758 \uc218\uc0ac\ubb38(\u4fee\u8a5e\u6587)\uc774\uc9c0\ub9cc, \uc9c0\ub098\uce58\uac8c \uaddc\uaca9\ud654\ud568\uc73c\ub85c\uc368 \ub9ce\uc740 \ud3d0\ub2e8\uc744 \ub0b3\uc558\ub2e4. 8\uace0\ubb38\uc774 \uba85\ub098\ub77c\uc758 \uacfc\uac70\uc2dc\ud5d8\uc5d0 \uc801\uc6a9\ub428\uc73c\ub85c\uc368 \uad50\uc721\uacfc \ud559\uc220\ubd84\uc57c\uc5d0 \ub07c\uce5c \ud574\ub3c5\uc740 \ub9e4\uc6b0 \ucef8\ub2e4. \uadf8\ub798\uc11c \uacbd\uc11c(\u7d93\u66f8)\uc758 \ub73b\uc744 \ud640\uc2dc(\u5ffd\u8996)\ud558\uace0 \ubb38\uc7a5\uc758 \uc218\uc2dd\uc5d0\ub9cc \uc804\ub150\ud558\ub294 \ub098\uc05c \ud559\ud48d\uc744 \uc870\uc131\ud558\uc600\ub2e4.", "answer": "\uc1a1\u00b7\uc6d0", "sentence": "\uba85\ub300\uc758 \uacfc\uac70\uc81c\ub3c4\ub294 \uc1a1\u00b7\uc6d0 (\u5b8b\u5143)\uc758 \uc81c\ub3c4\ub97c \uacc4\uc2b9\ud558\uc5ec \ud5a5\uc2dc(\u9115\u8a66)\u00b7\ud68c\uc2dc(\u6703\u8a66)\u00b7\uc804\uc2dc(\u6bbf\u8a66)\uc758 3\uc885\uc774 \uc788\ub2e4.", "paragraph_sentence": " \uba85\ub300\uc758 \uacfc\uac70\uc81c\ub3c4\ub294 \uc1a1\u00b7\uc6d0 (\u5b8b\u5143)\uc758 \uc81c\ub3c4\ub97c \uacc4\uc2b9\ud558\uc5ec \ud5a5\uc2dc(\u9115\u8a66)\u00b7\ud68c\uc2dc(\u6703\u8a66)\u00b7\uc804\uc2dc(\u6bbf\u8a66)\uc758 3\uc885\uc774 \uc788\ub2e4. \ud5a5\uc2dc\ub294 \uac01\uc131(\u5404\u7701)\uc5d0\uc11c, \ud68c\uc2dc\ub294 \uc608\ubd80(\u79ae\u90e8)\uc5d0\uc11c \uc804\uc2dc\ub294 \uad81\uc804(\u5bae\u6bbf)\uc5d0\uc11c \uc2dc\ud589\ud558\uc600\ub2e4. \uac01 \ubd80\u00b7\uc8fc\u00b7\ud604\uc758 \ud559\uc0dd\uc740 \uc81c\uac70\ud559\uad00(\u63d0\u64e7\u5b78\u5b98)\uc774 \uc8fc\uad00\ud558\ub294 \uc138\uace0(\u6b72\u8003)\uc5d0 1\uae09\uc73c\ub85c \ud569\uaca9\ud568\uc73c\ub85c\uc368 \ud5a5\uc2dc\uc5d0 \uc751\uc2dc\ud560 \uc790\uaca9\uc744 \uc5bb\ub294\ub2e4. \ud5a5\uc2dc\uc5d0 \ud569\uaca9\ud558\uba74 \uac70\uc778(\u64e7\u4eba)\uc774 \ub418\uace0 \ud68c\uc2dc(\u6703\u8a66)\uc5d0 \uc751\uc2dc\ud560 \uc790\uaca9\uc744 \uc5bb\ub294\ub2e4. \ub610 \ud68c\uc2dc\uc5d0 \ud569\uaca9\ud558\uba74 \ucc9c\uc790(\u5929\u5b50)\uac00 \uc784\uc11d\ud558\uc5ec \uc2dc\ud589\ud558\ub294 \uc804\uc2dc\uc5d0 \uc751\uc2dc\ud55c\ub2e4. \uace0\uc2dc\ub0b4\uc6a9\uc740 \uacbd\uc758(\u7d93\u7fa9)\u00b7\uc18c\uace0\uc728\ub839(\u8a34\u8aa5\u5f8b\u4ee4)\u00b7\uacbd\uc0ac\uc2dc\ubb34\ucc45(\u7d93\u53f2\u6642\u52d9\u7b56) \ub4f1 3\uc885\uc73c\ub85c, \uacbd\uc758\ub294 4\uc11c(\u56db\u66f8) \ubc0f \uc5ed\u00b7\uc11c\u00b7\uc2dc\u00b7\uc608\u00b7\ucd98\ucd94(\u6613\u00b7\u66f8\u00b7\u8a69\u00b7\u79ae\u00b7\u6625\u79cb)\uc758 5\uacbd\uc5d0\uc11c \ucd9c\uc81c\ud55c\ub2e4. \ud5a5\uc2dc\uc640 \ud68c\uc2dc\ub294 \uac01\uac01 3\ucc28\uc758 \uc2dc\ud5d8\uc744 \uac70\uce5c\ub2e4. 1\ucc28\ub294 4\uc11c\uc758(\u56db\u66f8\u7fa9) 3\ubb38\uc81c\u00b7\uacbd\uc758 4\ubb38\uc81c\ub97c \uc791\uc131\ud558\ub294 \uac83\uc774\ub2e4. 2\ucc28\ub294 \ub17c\ubb38 \ud55c \ud3b8\uc744 300\uc790 \uc774\uc0c1 \uc791\uc131\ud558\ub294 \uac83\uacfc \uc728\ub839\uc5d0 \uad00\ud55c \uac83 \ud55c \ud3b8, \uc81c\ub3c4\uc5d0 \uad00\ud55c \uac83 5\uc870\ubaa9\uc744 \uc4f0\ub294 \uac83\uc774\ub2e4. 3\ucc28\ub294 \uacbd\uc0ac\uc2dc\ubb34\ucc45(\u7d93\u53f2\u6642\u52d9\u7b56) 5\ubb38\uc81c\ub97c \uc791\uc131\ud558\ub294 \uac83\uc774\ub2e4. \uadf8\uc911 1\ucc28 \uc2dc\ud5d8\uc774 \uac00\uc7a5 \uc911\uc2dc\ub418\uc5c8\uace0, \ub2f5\uc548 \uc791\uc131\uc740 \uc61b \uc0ac\ub78c\uc758 \ubb38\uccb4\ub97c \ubaa8\ubc29\ud558\uc5ec \uaca9\ub960(\u683c\u5f8b)\uc774 \uc5c4\uaca9\ud558\uac8c \uc815\ud574\uc9c4 8\uace0\ubb38(\u516b\u80a1\u6587)\uc73c\ub85c \ud558\uc5ec\uc57c \ud588\ub2e4. 8\uace0\ubb38\uc740 \uaca9\uc2dd\uc744 \uc815\ud558\uc5ec, \uc124\uba85\uc5d0 \ud3b8\ub9ac\ud558\uace0 \uc6b4\uc728\uc5d0 \ub9de\ucd94\uc5b4 \ub4e3\uae30 \uc88b\ub3c4\ub85d \ud558\uae30 \uc704\ud574 \ub9cc\ub4e4\uc5b4\uc9c4 \uc77c\uc885\uc758 \uc218\uc0ac\ubb38(\u4fee\u8a5e\u6587)\uc774\uc9c0\ub9cc, \uc9c0\ub098\uce58\uac8c \uaddc\uaca9\ud654\ud568\uc73c\ub85c\uc368 \ub9ce\uc740 \ud3d0\ub2e8\uc744 \ub0b3\uc558\ub2e4. 8\uace0\ubb38\uc774 \uba85\ub098\ub77c\uc758 \uacfc\uac70\uc2dc\ud5d8\uc5d0 \uc801\uc6a9\ub428\uc73c\ub85c\uc368 \uad50\uc721\uacfc \ud559\uc220\ubd84\uc57c\uc5d0 \ub07c\uce5c \ud574\ub3c5\uc740 \ub9e4\uc6b0 \ucef8\ub2e4. \uadf8\ub798\uc11c \uacbd\uc11c(\u7d93\u66f8)\uc758 \ub73b\uc744 \ud640\uc2dc(\u5ffd\u8996)\ud558\uace0 \ubb38\uc7a5\uc758 \uc218\uc2dd\uc5d0\ub9cc \uc804\ub150\ud558\ub294 \ub098\uc05c \ud559\ud48d\uc744 \uc870\uc131\ud558\uc600\ub2e4.", "paragraph_answer": "\uba85\ub300\uc758 \uacfc\uac70\uc81c\ub3c4\ub294 \uc1a1\u00b7\uc6d0 (\u5b8b\u5143)\uc758 \uc81c\ub3c4\ub97c \uacc4\uc2b9\ud558\uc5ec \ud5a5\uc2dc(\u9115\u8a66)\u00b7\ud68c\uc2dc(\u6703\u8a66)\u00b7\uc804\uc2dc(\u6bbf\u8a66)\uc758 3\uc885\uc774 \uc788\ub2e4. \ud5a5\uc2dc\ub294 \uac01\uc131(\u5404\u7701)\uc5d0\uc11c, \ud68c\uc2dc\ub294 \uc608\ubd80(\u79ae\u90e8)\uc5d0\uc11c \uc804\uc2dc\ub294 \uad81\uc804(\u5bae\u6bbf)\uc5d0\uc11c \uc2dc\ud589\ud558\uc600\ub2e4. \uac01 \ubd80\u00b7\uc8fc\u00b7\ud604\uc758 \ud559\uc0dd\uc740 \uc81c\uac70\ud559\uad00(\u63d0\u64e7\u5b78\u5b98)\uc774 \uc8fc\uad00\ud558\ub294 \uc138\uace0(\u6b72\u8003)\uc5d0 1\uae09\uc73c\ub85c \ud569\uaca9\ud568\uc73c\ub85c\uc368 \ud5a5\uc2dc\uc5d0 \uc751\uc2dc\ud560 \uc790\uaca9\uc744 \uc5bb\ub294\ub2e4. \ud5a5\uc2dc\uc5d0 \ud569\uaca9\ud558\uba74 \uac70\uc778(\u64e7\u4eba)\uc774 \ub418\uace0 \ud68c\uc2dc(\u6703\u8a66)\uc5d0 \uc751\uc2dc\ud560 \uc790\uaca9\uc744 \uc5bb\ub294\ub2e4. \ub610 \ud68c\uc2dc\uc5d0 \ud569\uaca9\ud558\uba74 \ucc9c\uc790(\u5929\u5b50)\uac00 \uc784\uc11d\ud558\uc5ec \uc2dc\ud589\ud558\ub294 \uc804\uc2dc\uc5d0 \uc751\uc2dc\ud55c\ub2e4. \uace0\uc2dc\ub0b4\uc6a9\uc740 \uacbd\uc758(\u7d93\u7fa9)\u00b7\uc18c\uace0\uc728\ub839(\u8a34\u8aa5\u5f8b\u4ee4)\u00b7\uacbd\uc0ac\uc2dc\ubb34\ucc45(\u7d93\u53f2\u6642\u52d9\u7b56) \ub4f1 3\uc885\uc73c\ub85c, \uacbd\uc758\ub294 4\uc11c(\u56db\u66f8) \ubc0f \uc5ed\u00b7\uc11c\u00b7\uc2dc\u00b7\uc608\u00b7\ucd98\ucd94(\u6613\u00b7\u66f8\u00b7\u8a69\u00b7\u79ae\u00b7\u6625\u79cb)\uc758 5\uacbd\uc5d0\uc11c \ucd9c\uc81c\ud55c\ub2e4. \ud5a5\uc2dc\uc640 \ud68c\uc2dc\ub294 \uac01\uac01 3\ucc28\uc758 \uc2dc\ud5d8\uc744 \uac70\uce5c\ub2e4. 1\ucc28\ub294 4\uc11c\uc758(\u56db\u66f8\u7fa9) 3\ubb38\uc81c\u00b7\uacbd\uc758 4\ubb38\uc81c\ub97c \uc791\uc131\ud558\ub294 \uac83\uc774\ub2e4. 2\ucc28\ub294 \ub17c\ubb38 \ud55c \ud3b8\uc744 300\uc790 \uc774\uc0c1 \uc791\uc131\ud558\ub294 \uac83\uacfc \uc728\ub839\uc5d0 \uad00\ud55c \uac83 \ud55c \ud3b8, \uc81c\ub3c4\uc5d0 \uad00\ud55c \uac83 5\uc870\ubaa9\uc744 \uc4f0\ub294 \uac83\uc774\ub2e4. 3\ucc28\ub294 \uacbd\uc0ac\uc2dc\ubb34\ucc45(\u7d93\u53f2\u6642\u52d9\u7b56) 5\ubb38\uc81c\ub97c \uc791\uc131\ud558\ub294 \uac83\uc774\ub2e4. \uadf8\uc911 1\ucc28 \uc2dc\ud5d8\uc774 \uac00\uc7a5 \uc911\uc2dc\ub418\uc5c8\uace0, \ub2f5\uc548 \uc791\uc131\uc740 \uc61b \uc0ac\ub78c\uc758 \ubb38\uccb4\ub97c \ubaa8\ubc29\ud558\uc5ec \uaca9\ub960(\u683c\u5f8b)\uc774 \uc5c4\uaca9\ud558\uac8c \uc815\ud574\uc9c4 8\uace0\ubb38(\u516b\u80a1\u6587)\uc73c\ub85c \ud558\uc5ec\uc57c \ud588\ub2e4. 8\uace0\ubb38\uc740 \uaca9\uc2dd\uc744 \uc815\ud558\uc5ec, \uc124\uba85\uc5d0 \ud3b8\ub9ac\ud558\uace0 \uc6b4\uc728\uc5d0 \ub9de\ucd94\uc5b4 \ub4e3\uae30 \uc88b\ub3c4\ub85d \ud558\uae30 \uc704\ud574 \ub9cc\ub4e4\uc5b4\uc9c4 \uc77c\uc885\uc758 \uc218\uc0ac\ubb38(\u4fee\u8a5e\u6587)\uc774\uc9c0\ub9cc, \uc9c0\ub098\uce58\uac8c \uaddc\uaca9\ud654\ud568\uc73c\ub85c\uc368 \ub9ce\uc740 \ud3d0\ub2e8\uc744 \ub0b3\uc558\ub2e4. 8\uace0\ubb38\uc774 \uba85\ub098\ub77c\uc758 \uacfc\uac70\uc2dc\ud5d8\uc5d0 \uc801\uc6a9\ub428\uc73c\ub85c\uc368 \uad50\uc721\uacfc \ud559\uc220\ubd84\uc57c\uc5d0 \ub07c\uce5c \ud574\ub3c5\uc740 \ub9e4\uc6b0 \ucef8\ub2e4. \uadf8\ub798\uc11c \uacbd\uc11c(\u7d93\u66f8)\uc758 \ub73b\uc744 \ud640\uc2dc(\u5ffd\u8996)\ud558\uace0 \ubb38\uc7a5\uc758 \uc218\uc2dd\uc5d0\ub9cc \uc804\ub150\ud558\ub294 \ub098\uc05c \ud559\ud48d\uc744 \uc870\uc131\ud558\uc600\ub2e4.", "sentence_answer": "\uba85\ub300\uc758 \uacfc\uac70\uc81c\ub3c4\ub294 \uc1a1\u00b7\uc6d0 (\u5b8b\u5143)\uc758 \uc81c\ub3c4\ub97c \uacc4\uc2b9\ud558\uc5ec \ud5a5\uc2dc(\u9115\u8a66)\u00b7\ud68c\uc2dc(\u6703\u8a66)\u00b7\uc804\uc2dc(\u6bbf\u8a66)\uc758 3\uc885\uc774 \uc788\ub2e4.", "paragraph_id": "6503076-2-0", "paragraph_question": "question: \uba85\ub098\ub77c\uc758 \uacfc\uac70\uc81c\ub3c4\uc778 \ud5a5\uc2dc, \ud68c\uc2dc, \uc804\uc2dc\ub294 \uc5b4\ub290 \ub098\ub77c\uc758 \uc81c\ub3c4\ub97c \uacc4\uc2b9\ud55c \uac83\uc778\uac00?, context: \uba85\ub300\uc758 \uacfc\uac70\uc81c\ub3c4\ub294 \uc1a1\u00b7\uc6d0(\u5b8b\u5143)\uc758 \uc81c\ub3c4\ub97c \uacc4\uc2b9\ud558\uc5ec \ud5a5\uc2dc(\u9115\u8a66)\u00b7\ud68c\uc2dc(\u6703\u8a66)\u00b7\uc804\uc2dc(\u6bbf\u8a66)\uc758 3\uc885\uc774 \uc788\ub2e4. \ud5a5\uc2dc\ub294 \uac01\uc131(\u5404\u7701)\uc5d0\uc11c, \ud68c\uc2dc\ub294 \uc608\ubd80(\u79ae\u90e8)\uc5d0\uc11c \uc804\uc2dc\ub294 \uad81\uc804(\u5bae\u6bbf)\uc5d0\uc11c \uc2dc\ud589\ud558\uc600\ub2e4. \uac01 \ubd80\u00b7\uc8fc\u00b7\ud604\uc758 \ud559\uc0dd\uc740 \uc81c\uac70\ud559\uad00(\u63d0\u64e7\u5b78\u5b98)\uc774 \uc8fc\uad00\ud558\ub294 \uc138\uace0(\u6b72\u8003)\uc5d0 1\uae09\uc73c\ub85c \ud569\uaca9\ud568\uc73c\ub85c\uc368 \ud5a5\uc2dc\uc5d0 \uc751\uc2dc\ud560 \uc790\uaca9\uc744 \uc5bb\ub294\ub2e4. \ud5a5\uc2dc\uc5d0 \ud569\uaca9\ud558\uba74 \uac70\uc778(\u64e7\u4eba)\uc774 \ub418\uace0 \ud68c\uc2dc(\u6703\u8a66)\uc5d0 \uc751\uc2dc\ud560 \uc790\uaca9\uc744 \uc5bb\ub294\ub2e4. \ub610 \ud68c\uc2dc\uc5d0 \ud569\uaca9\ud558\uba74 \ucc9c\uc790(\u5929\u5b50)\uac00 \uc784\uc11d\ud558\uc5ec \uc2dc\ud589\ud558\ub294 \uc804\uc2dc\uc5d0 \uc751\uc2dc\ud55c\ub2e4. \uace0\uc2dc\ub0b4\uc6a9\uc740 \uacbd\uc758(\u7d93\u7fa9)\u00b7\uc18c\uace0\uc728\ub839(\u8a34\u8aa5\u5f8b\u4ee4)\u00b7\uacbd\uc0ac\uc2dc\ubb34\ucc45(\u7d93\u53f2\u6642\u52d9\u7b56) \ub4f1 3\uc885\uc73c\ub85c, \uacbd\uc758\ub294 4\uc11c(\u56db\u66f8) \ubc0f \uc5ed\u00b7\uc11c\u00b7\uc2dc\u00b7\uc608\u00b7\ucd98\ucd94(\u6613\u00b7\u66f8\u00b7\u8a69\u00b7\u79ae\u00b7\u6625\u79cb)\uc758 5\uacbd\uc5d0\uc11c \ucd9c\uc81c\ud55c\ub2e4. \ud5a5\uc2dc\uc640 \ud68c\uc2dc\ub294 \uac01\uac01 3\ucc28\uc758 \uc2dc\ud5d8\uc744 \uac70\uce5c\ub2e4. 1\ucc28\ub294 4\uc11c\uc758(\u56db\u66f8\u7fa9) 3\ubb38\uc81c\u00b7\uacbd\uc758 4\ubb38\uc81c\ub97c \uc791\uc131\ud558\ub294 \uac83\uc774\ub2e4. 2\ucc28\ub294 \ub17c\ubb38 \ud55c \ud3b8\uc744 300\uc790 \uc774\uc0c1 \uc791\uc131\ud558\ub294 \uac83\uacfc \uc728\ub839\uc5d0 \uad00\ud55c \uac83 \ud55c \ud3b8, \uc81c\ub3c4\uc5d0 \uad00\ud55c \uac83 5\uc870\ubaa9\uc744 \uc4f0\ub294 \uac83\uc774\ub2e4. 3\ucc28\ub294 \uacbd\uc0ac\uc2dc\ubb34\ucc45(\u7d93\u53f2\u6642\u52d9\u7b56) 5\ubb38\uc81c\ub97c \uc791\uc131\ud558\ub294 \uac83\uc774\ub2e4. \uadf8\uc911 1\ucc28 \uc2dc\ud5d8\uc774 \uac00\uc7a5 \uc911\uc2dc\ub418\uc5c8\uace0, \ub2f5\uc548 \uc791\uc131\uc740 \uc61b \uc0ac\ub78c\uc758 \ubb38\uccb4\ub97c \ubaa8\ubc29\ud558\uc5ec \uaca9\ub960(\u683c\u5f8b)\uc774 \uc5c4\uaca9\ud558\uac8c \uc815\ud574\uc9c4 8\uace0\ubb38(\u516b\u80a1\u6587)\uc73c\ub85c \ud558\uc5ec\uc57c \ud588\ub2e4. 8\uace0\ubb38\uc740 \uaca9\uc2dd\uc744 \uc815\ud558\uc5ec, \uc124\uba85\uc5d0 \ud3b8\ub9ac\ud558\uace0 \uc6b4\uc728\uc5d0 \ub9de\ucd94\uc5b4 \ub4e3\uae30 \uc88b\ub3c4\ub85d \ud558\uae30 \uc704\ud574 \ub9cc\ub4e4\uc5b4\uc9c4 \uc77c\uc885\uc758 \uc218\uc0ac\ubb38(\u4fee\u8a5e\u6587)\uc774\uc9c0\ub9cc, \uc9c0\ub098\uce58\uac8c \uaddc\uaca9\ud654\ud568\uc73c\ub85c\uc368 \ub9ce\uc740 \ud3d0\ub2e8\uc744 \ub0b3\uc558\ub2e4. 8\uace0\ubb38\uc774 \uba85\ub098\ub77c\uc758 \uacfc\uac70\uc2dc\ud5d8\uc5d0 \uc801\uc6a9\ub428\uc73c\ub85c\uc368 \uad50\uc721\uacfc \ud559\uc220\ubd84\uc57c\uc5d0 \ub07c\uce5c \ud574\ub3c5\uc740 \ub9e4\uc6b0 \ucef8\ub2e4. \uadf8\ub798\uc11c \uacbd\uc11c(\u7d93\u66f8)\uc758 \ub73b\uc744 \ud640\uc2dc(\u5ffd\u8996)\ud558\uace0 \ubb38\uc7a5\uc758 \uc218\uc2dd\uc5d0\ub9cc \uc804\ub150\ud558\ub294 \ub098\uc05c \ud559\ud48d\uc744 \uc870\uc131\ud558\uc600\ub2e4."} {"question": "\uc728\uace1 \uc774\uc774\ub294 \ubaa8\uce5c\uc0c1\uc744 \ub2f9\ud558\uace0 \uc2ac\ud514\uc5d0 \ube60\uc838 \uc5b4\ub514\uc5d0 \ub4e4\uc5b4\uac00 \uce69\uac70\ub97c \ud588\ub098?", "paragraph": "\uadf8\ubc16\uc5d0 '\uc790\uc2dd\uc5d0\uac8c\ub294 \uc5b4\uc9c4 \uc5b4\uba38\ub2c8\uc774\uace0 \ub0a8\ud3b8\uc5d0\uac8c\ub294 \ucc29\ud55c \uc544\ub0b4'\ub77c\ub294 \ub9d0\ub73b \uadf8\ub300\ub85c\uc758 \ud604\ubaa8\uc591\ucc98\ub77c\uba74 \uc2e0\uc0ac\uc784\ub2f9\uc774 \ud604\ubaa8\uc591\ucc98\ub77c\ub294 \ub9d0\uc740 \ub9de\uace0\ub3c4 \ud2c0\ub9ac\ub2e4.'\ub294 \uc758\uacac\ub3c4 \uc788\ub2e4. \uc791\uac00 \uae40\ubcc4\uc544\ub294 '\uc2a4\uc2a4\ub85c \uc9c0\uc5b4 \ubd80\ub978 \uc0ac\uc784\ub2f9\uc774\ub77c\ub294 \ud638\uac00 \uc131\uad70\uc758 \ub300\ud45c \uaca9\uc778 \uc911\uad6d \uc8fc\ub098\ub77c \ubb38\uc655\uc758 \ud6cc\ub96d\ud55c \uc5b4\uba38\ub2c8 \ud0dc\uc784\uc744 \ubc30\uc6b0\uace0 \ubcf8\ubc1b\ub294\ub2e4\ub294 \ub73b\uc778 \ub9cc\ud07c, \ub300\ud559\uc790\uc774\uc790 \uc815\uce58\uac00\uc778 \uc728\uace1\uc744 \ud3ec\ud568\ud55c 4\ub0a8 3\ub140\uc758 \uc790\uc2dd\ub4e4\uc5d0\uac8c \uc0ac\uc784\ub2f9\uc740 \ub354\ud560 \ub098\uc704 \uc5c6\uc774 \uc88b\uc740 \uc5b4\uba38\ub2c8\uc600\ub2e4. \uc2e4\ub85c \uc0ac\uc784\ub2f9\uc774 \uc6b0\ub9ac \uc5ed\uc0ac \uc18d\uc5d0\uc11c `\ud76c\uadc0'\ud558\ub2e4\uc2dc\ud53c \ud55c \uc5ec\uc131 \uc778\ubb3c\ub85c \uc6b0\ub69d \uc790\ub9ac \uc7a1\uc740 \ub370\ub294 \uc728\uace1 \uc774\uc774\uac00 \uc4f4 `\uc5b4\uba38\ub2c8\uc758 \uc77c\ub300\uae30'(\u5148\u884c\u72c0)\uac00 \uacb0\uc815\uc801\uc778 \uc5ed\ud560\uc744 \ud588\ub2e4. \uc728\uace1\uc740 \uc544\ubc84\uc9c0 \uc774\uc6d0\uc218\uc758 \ud589\uc7a5\uc740 \uc4f4 \uc801\uc774 \uc5c6\uc9c0\ub9cc, \uc5b4\uba38\ub2c8 \uc0ac\uc784\ub2f9\uc5d0 \ub300\ud574\uc11c\ub294 \uc808\uc808\ud55c \uadf8\ub9ac\uc6c0\uc774 \ub2f4\uae34 \ud589\uc7a5\ubfd0\ub9cc\uc774 \uc544\ub2c8\ub77c \ubaa8\uce5c\uc0c1\uc744 \ub2f9\ud55c \uc9c1\ud6c4 \uc2ac\ud514\uacfc \ud5c8\ubb34\uac10\uc5d0 \ube60\uc838 \uae08\uac15\uc0b0\uc5d0 \ub4e4\uc5b4\uac00 \uce69\uac70\ud588\ub2e4\ub294 \uae30\ub85d\uc744 \ub0a8\uae30\uae30\ub3c4 \ud588\ub2e4.'\uace0 \uc804\uc81c\ud558\uc600\ub2e4.", "answer": "\uae08\uac15\uc0b0", "sentence": "\uc728\uace1\uc740 \uc544\ubc84\uc9c0 \uc774\uc6d0\uc218\uc758 \ud589\uc7a5\uc740 \uc4f4 \uc801\uc774 \uc5c6\uc9c0\ub9cc, \uc5b4\uba38\ub2c8 \uc0ac\uc784\ub2f9\uc5d0 \ub300\ud574\uc11c\ub294 \uc808\uc808\ud55c \uadf8\ub9ac\uc6c0\uc774 \ub2f4\uae34 \ud589\uc7a5\ubfd0\ub9cc\uc774 \uc544\ub2c8\ub77c \ubaa8\uce5c\uc0c1\uc744 \ub2f9\ud55c \uc9c1\ud6c4 \uc2ac\ud514\uacfc \ud5c8\ubb34\uac10\uc5d0 \ube60\uc838 \uae08\uac15\uc0b0 \uc5d0 \ub4e4\uc5b4\uac00 \uce69\uac70\ud588\ub2e4\ub294 \uae30\ub85d\uc744 \ub0a8\uae30\uae30\ub3c4 \ud588\ub2e4.", "paragraph_sentence": "\uadf8\ubc16\uc5d0 '\uc790\uc2dd\uc5d0\uac8c\ub294 \uc5b4\uc9c4 \uc5b4\uba38\ub2c8\uc774\uace0 \ub0a8\ud3b8\uc5d0\uac8c\ub294 \ucc29\ud55c \uc544\ub0b4'\ub77c\ub294 \ub9d0\ub73b \uadf8\ub300\ub85c\uc758 \ud604\ubaa8\uc591\ucc98\ub77c\uba74 \uc2e0\uc0ac\uc784\ub2f9\uc774 \ud604\ubaa8\uc591\ucc98\ub77c\ub294 \ub9d0\uc740 \ub9de\uace0\ub3c4 \ud2c0\ub9ac\ub2e4. '\ub294 \uc758\uacac\ub3c4 \uc788\ub2e4. \uc791\uac00 \uae40\ubcc4\uc544\ub294 '\uc2a4\uc2a4\ub85c \uc9c0\uc5b4 \ubd80\ub978 \uc0ac\uc784\ub2f9\uc774\ub77c\ub294 \ud638\uac00 \uc131\uad70\uc758 \ub300\ud45c \uaca9\uc778 \uc911\uad6d \uc8fc\ub098\ub77c \ubb38\uc655\uc758 \ud6cc\ub96d\ud55c \uc5b4\uba38\ub2c8 \ud0dc\uc784\uc744 \ubc30\uc6b0\uace0 \ubcf8\ubc1b\ub294\ub2e4\ub294 \ub73b\uc778 \ub9cc\ud07c, \ub300\ud559\uc790\uc774\uc790 \uc815\uce58\uac00\uc778 \uc728\uace1\uc744 \ud3ec\ud568\ud55c 4\ub0a8 3\ub140\uc758 \uc790\uc2dd\ub4e4\uc5d0\uac8c \uc0ac\uc784\ub2f9\uc740 \ub354\ud560 \ub098\uc704 \uc5c6\uc774 \uc88b\uc740 \uc5b4\uba38\ub2c8\uc600\ub2e4. \uc2e4\ub85c \uc0ac\uc784\ub2f9\uc774 \uc6b0\ub9ac \uc5ed\uc0ac \uc18d\uc5d0\uc11c `\ud76c\uadc0'\ud558\ub2e4\uc2dc\ud53c \ud55c \uc5ec\uc131 \uc778\ubb3c\ub85c \uc6b0\ub69d \uc790\ub9ac \uc7a1\uc740 \ub370\ub294 \uc728\uace1 \uc774\uc774\uac00 \uc4f4 `\uc5b4\uba38\ub2c8\uc758 \uc77c\ub300\uae30'(\u5148\u884c\u72c0)\uac00 \uacb0\uc815\uc801\uc778 \uc5ed\ud560\uc744 \ud588\ub2e4. \uc728\uace1\uc740 \uc544\ubc84\uc9c0 \uc774\uc6d0\uc218\uc758 \ud589\uc7a5\uc740 \uc4f4 \uc801\uc774 \uc5c6\uc9c0\ub9cc, \uc5b4\uba38\ub2c8 \uc0ac\uc784\ub2f9\uc5d0 \ub300\ud574\uc11c\ub294 \uc808\uc808\ud55c \uadf8\ub9ac\uc6c0\uc774 \ub2f4\uae34 \ud589\uc7a5\ubfd0\ub9cc\uc774 \uc544\ub2c8\ub77c \ubaa8\uce5c\uc0c1\uc744 \ub2f9\ud55c \uc9c1\ud6c4 \uc2ac\ud514\uacfc \ud5c8\ubb34\uac10\uc5d0 \ube60\uc838 \uae08\uac15\uc0b0 \uc5d0 \ub4e4\uc5b4\uac00 \uce69\uac70\ud588\ub2e4\ub294 \uae30\ub85d\uc744 \ub0a8\uae30\uae30\ub3c4 \ud588\ub2e4. '\uace0 \uc804\uc81c\ud558\uc600\ub2e4.", "paragraph_answer": "\uadf8\ubc16\uc5d0 '\uc790\uc2dd\uc5d0\uac8c\ub294 \uc5b4\uc9c4 \uc5b4\uba38\ub2c8\uc774\uace0 \ub0a8\ud3b8\uc5d0\uac8c\ub294 \ucc29\ud55c \uc544\ub0b4'\ub77c\ub294 \ub9d0\ub73b \uadf8\ub300\ub85c\uc758 \ud604\ubaa8\uc591\ucc98\ub77c\uba74 \uc2e0\uc0ac\uc784\ub2f9\uc774 \ud604\ubaa8\uc591\ucc98\ub77c\ub294 \ub9d0\uc740 \ub9de\uace0\ub3c4 \ud2c0\ub9ac\ub2e4.'\ub294 \uc758\uacac\ub3c4 \uc788\ub2e4. \uc791\uac00 \uae40\ubcc4\uc544\ub294 '\uc2a4\uc2a4\ub85c \uc9c0\uc5b4 \ubd80\ub978 \uc0ac\uc784\ub2f9\uc774\ub77c\ub294 \ud638\uac00 \uc131\uad70\uc758 \ub300\ud45c \uaca9\uc778 \uc911\uad6d \uc8fc\ub098\ub77c \ubb38\uc655\uc758 \ud6cc\ub96d\ud55c \uc5b4\uba38\ub2c8 \ud0dc\uc784\uc744 \ubc30\uc6b0\uace0 \ubcf8\ubc1b\ub294\ub2e4\ub294 \ub73b\uc778 \ub9cc\ud07c, \ub300\ud559\uc790\uc774\uc790 \uc815\uce58\uac00\uc778 \uc728\uace1\uc744 \ud3ec\ud568\ud55c 4\ub0a8 3\ub140\uc758 \uc790\uc2dd\ub4e4\uc5d0\uac8c \uc0ac\uc784\ub2f9\uc740 \ub354\ud560 \ub098\uc704 \uc5c6\uc774 \uc88b\uc740 \uc5b4\uba38\ub2c8\uc600\ub2e4. \uc2e4\ub85c \uc0ac\uc784\ub2f9\uc774 \uc6b0\ub9ac \uc5ed\uc0ac \uc18d\uc5d0\uc11c `\ud76c\uadc0'\ud558\ub2e4\uc2dc\ud53c \ud55c \uc5ec\uc131 \uc778\ubb3c\ub85c \uc6b0\ub69d \uc790\ub9ac \uc7a1\uc740 \ub370\ub294 \uc728\uace1 \uc774\uc774\uac00 \uc4f4 `\uc5b4\uba38\ub2c8\uc758 \uc77c\ub300\uae30'(\u5148\u884c\u72c0)\uac00 \uacb0\uc815\uc801\uc778 \uc5ed\ud560\uc744 \ud588\ub2e4. \uc728\uace1\uc740 \uc544\ubc84\uc9c0 \uc774\uc6d0\uc218\uc758 \ud589\uc7a5\uc740 \uc4f4 \uc801\uc774 \uc5c6\uc9c0\ub9cc, \uc5b4\uba38\ub2c8 \uc0ac\uc784\ub2f9\uc5d0 \ub300\ud574\uc11c\ub294 \uc808\uc808\ud55c \uadf8\ub9ac\uc6c0\uc774 \ub2f4\uae34 \ud589\uc7a5\ubfd0\ub9cc\uc774 \uc544\ub2c8\ub77c \ubaa8\uce5c\uc0c1\uc744 \ub2f9\ud55c \uc9c1\ud6c4 \uc2ac\ud514\uacfc \ud5c8\ubb34\uac10\uc5d0 \ube60\uc838 \uae08\uac15\uc0b0 \uc5d0 \ub4e4\uc5b4\uac00 \uce69\uac70\ud588\ub2e4\ub294 \uae30\ub85d\uc744 \ub0a8\uae30\uae30\ub3c4 \ud588\ub2e4.'\uace0 \uc804\uc81c\ud558\uc600\ub2e4.", "sentence_answer": "\uc728\uace1\uc740 \uc544\ubc84\uc9c0 \uc774\uc6d0\uc218\uc758 \ud589\uc7a5\uc740 \uc4f4 \uc801\uc774 \uc5c6\uc9c0\ub9cc, \uc5b4\uba38\ub2c8 \uc0ac\uc784\ub2f9\uc5d0 \ub300\ud574\uc11c\ub294 \uc808\uc808\ud55c \uadf8\ub9ac\uc6c0\uc774 \ub2f4\uae34 \ud589\uc7a5\ubfd0\ub9cc\uc774 \uc544\ub2c8\ub77c \ubaa8\uce5c\uc0c1\uc744 \ub2f9\ud55c \uc9c1\ud6c4 \uc2ac\ud514\uacfc \ud5c8\ubb34\uac10\uc5d0 \ube60\uc838 \uae08\uac15\uc0b0 \uc5d0 \ub4e4\uc5b4\uac00 \uce69\uac70\ud588\ub2e4\ub294 \uae30\ub85d\uc744 \ub0a8\uae30\uae30\ub3c4 \ud588\ub2e4.", "paragraph_id": "6657247-3-2", "paragraph_question": "question: \uc728\uace1 \uc774\uc774\ub294 \ubaa8\uce5c\uc0c1\uc744 \ub2f9\ud558\uace0 \uc2ac\ud514\uc5d0 \ube60\uc838 \uc5b4\ub514\uc5d0 \ub4e4\uc5b4\uac00 \uce69\uac70\ub97c \ud588\ub098?, context: \uadf8\ubc16\uc5d0 '\uc790\uc2dd\uc5d0\uac8c\ub294 \uc5b4\uc9c4 \uc5b4\uba38\ub2c8\uc774\uace0 \ub0a8\ud3b8\uc5d0\uac8c\ub294 \ucc29\ud55c \uc544\ub0b4'\ub77c\ub294 \ub9d0\ub73b \uadf8\ub300\ub85c\uc758 \ud604\ubaa8\uc591\ucc98\ub77c\uba74 \uc2e0\uc0ac\uc784\ub2f9\uc774 \ud604\ubaa8\uc591\ucc98\ub77c\ub294 \ub9d0\uc740 \ub9de\uace0\ub3c4 \ud2c0\ub9ac\ub2e4.'\ub294 \uc758\uacac\ub3c4 \uc788\ub2e4. \uc791\uac00 \uae40\ubcc4\uc544\ub294 '\uc2a4\uc2a4\ub85c \uc9c0\uc5b4 \ubd80\ub978 \uc0ac\uc784\ub2f9\uc774\ub77c\ub294 \ud638\uac00 \uc131\uad70\uc758 \ub300\ud45c \uaca9\uc778 \uc911\uad6d \uc8fc\ub098\ub77c \ubb38\uc655\uc758 \ud6cc\ub96d\ud55c \uc5b4\uba38\ub2c8 \ud0dc\uc784\uc744 \ubc30\uc6b0\uace0 \ubcf8\ubc1b\ub294\ub2e4\ub294 \ub73b\uc778 \ub9cc\ud07c, \ub300\ud559\uc790\uc774\uc790 \uc815\uce58\uac00\uc778 \uc728\uace1\uc744 \ud3ec\ud568\ud55c 4\ub0a8 3\ub140\uc758 \uc790\uc2dd\ub4e4\uc5d0\uac8c \uc0ac\uc784\ub2f9\uc740 \ub354\ud560 \ub098\uc704 \uc5c6\uc774 \uc88b\uc740 \uc5b4\uba38\ub2c8\uc600\ub2e4. \uc2e4\ub85c \uc0ac\uc784\ub2f9\uc774 \uc6b0\ub9ac \uc5ed\uc0ac \uc18d\uc5d0\uc11c `\ud76c\uadc0'\ud558\ub2e4\uc2dc\ud53c \ud55c \uc5ec\uc131 \uc778\ubb3c\ub85c \uc6b0\ub69d \uc790\ub9ac \uc7a1\uc740 \ub370\ub294 \uc728\uace1 \uc774\uc774\uac00 \uc4f4 `\uc5b4\uba38\ub2c8\uc758 \uc77c\ub300\uae30'(\u5148\u884c\u72c0)\uac00 \uacb0\uc815\uc801\uc778 \uc5ed\ud560\uc744 \ud588\ub2e4. \uc728\uace1\uc740 \uc544\ubc84\uc9c0 \uc774\uc6d0\uc218\uc758 \ud589\uc7a5\uc740 \uc4f4 \uc801\uc774 \uc5c6\uc9c0\ub9cc, \uc5b4\uba38\ub2c8 \uc0ac\uc784\ub2f9\uc5d0 \ub300\ud574\uc11c\ub294 \uc808\uc808\ud55c \uadf8\ub9ac\uc6c0\uc774 \ub2f4\uae34 \ud589\uc7a5\ubfd0\ub9cc\uc774 \uc544\ub2c8\ub77c \ubaa8\uce5c\uc0c1\uc744 \ub2f9\ud55c \uc9c1\ud6c4 \uc2ac\ud514\uacfc \ud5c8\ubb34\uac10\uc5d0 \ube60\uc838 \uae08\uac15\uc0b0\uc5d0 \ub4e4\uc5b4\uac00 \uce69\uac70\ud588\ub2e4\ub294 \uae30\ub85d\uc744 \ub0a8\uae30\uae30\ub3c4 \ud588\ub2e4.'\uace0 \uc804\uc81c\ud558\uc600\ub2e4."} {"question": "1970\ub144\ub300 \ucd08 AKA\uac00 \ubcf4\ud638\ud558\uae30 \uc704\ud574 2\ub9cc \ub2ec\ub7ec\ub97c \uae30\ubd80\ud588\ub358 \ub300\uc0c1\uc740 \ub204\uad6c\uc778\uac00?", "paragraph": "\uc5ec\uc131\ud68c\ub294 1968\ub144\uc5d0\uc11c 1972\ub144\uae4c\uc9c0\uc758 \uc720\uc0b0 \uc2dc\ub9ac\uc988\ub97c \ubc1c\uac04\ud568\uc73c\ub85c\uc368 \ub6f0\uc5b4\ub09c \uc5c5\uc801\uc744 \ub0a8\uae34 \uc0ac\ub78c\ub4e4\uc744 \uc9d1\uc911\uc870\uba85\ud558\uba74\uc11c \uacf5\ub3d9\uccb4\ub97c \uad50\uc721\ud558\uc600\ub2e4. \uc774\ub4e4 \uc18c\ucc45\uc790\ub4e4\uc740 \uc804\uae30 \ucd5c\uace0\uc758 \uc544\ud504\ub9ac\uce74\uacc4 \ubbf8\uad6d\uc778 \uc5ec\uc131\uc758 \uc77c\ub300\uae30 \uc2dc\ub9ac\uc988\uc774\ub2e4. \uadf8\ub9ac\uace0, \uc804\uccb4 \ucf5c\ub809\uc158\uc740 '\uc0ac\ubc95\uacc4 \uc5ec\uc131', '\uc815\uacc4 \uc5ec\uc131', '\uc758\ub8cc\uacc4 \uc5ec\uc131', '\ube44\uc988\ub2c8\uc2a4\uacc4 \uc5ec\uc131', '\uce58\uc758\ud559\uacc4 \uc5ec\uc131'\ub4e4\uc744 \ud3ec\ud568\ud558\uace0 \uc788\ub2e4. 1970\ub144\ub300 \ucd08 AKA\ub294 \uc870\uc9c0\uc544 \uc8fc \uc560\ud2c0\ub780\ud0c0\uc5d0 \uc788\ub294 \ub9c8\ud2f4 \ub8e8\uc11c \ud0b9 \uc8fc\ub2c8\uc5b4\uc758 \uc0dd\uc77c \uc7a5\uc18c\uc5d0\uc11c \ub9c8\ud2f4 \ub8e8\uc11c \ud0b9\uc744 \ubcf4\ud638\ud558\ub294 \uc77c\uc5d0 2\ub9cc \ub2ec\ub7ec\ub97c \uae30\ubd80\ud558\uc600\ub2e4. \uc5ec\uc131\ud68c 70\uc8fc\ub144\uc778 1978\ub144, \ud558\uc6cc\ub4dc \ub300\ud559\uad50\uc5d0 AKA\uc758 \uc124\ub9bd\uc790\ub4e4\uc744 \uae30\ub9ac\ub294 '\uae30\ub150 \ucc3d\ubb38'\ub97c \uc124\uce58\ud588\ub2e4. '\uae30\ub150 \ucc3d\ubb38'\uc740 \ub85c\uc774\uc2a4 \ub9c8\uc77c\ub85c \uc874\uc2a4\uac00 \ub514\uc790\uc778\ud55c \uac83\uc774\uba70, \ud604\uc874\ud558\ub358 \uc124\ub9bd\uc790\uc778 \ub77c\ube44\ub2c8\uc544 \ub178\uba3c\uacfc \ub178\ub9c8 \ubcf4\uc774\ub4dc\uac00 '\uae30\ub150 \ucc3d\ubb38'\uc758 \uacf5\uac1c\uc2dd\uc5d0 \ucc38\uc11d\ud558\uc600\ub2e4.", "answer": "\ub9c8\ud2f4 \ub8e8\uc11c \ud0b9", "sentence": "1970\ub144\ub300 \ucd08 AKA\ub294 \uc870\uc9c0\uc544 \uc8fc \uc560\ud2c0\ub780\ud0c0\uc5d0 \uc788\ub294 \ub9c8\ud2f4 \ub8e8\uc11c \ud0b9 \uc8fc\ub2c8\uc5b4\uc758 \uc0dd\uc77c \uc7a5\uc18c\uc5d0\uc11c \ub9c8\ud2f4 \ub8e8\uc11c \ud0b9\uc744 \ubcf4\ud638\ud558\ub294 \uc77c\uc5d0 2\ub9cc \ub2ec\ub7ec\ub97c \uae30\ubd80\ud558\uc600\ub2e4.", "paragraph_sentence": "\uc5ec\uc131\ud68c\ub294 1968\ub144\uc5d0\uc11c 1972\ub144\uae4c\uc9c0\uc758 \uc720\uc0b0 \uc2dc\ub9ac\uc988\ub97c \ubc1c\uac04\ud568\uc73c\ub85c\uc368 \ub6f0\uc5b4\ub09c \uc5c5\uc801\uc744 \ub0a8\uae34 \uc0ac\ub78c\ub4e4\uc744 \uc9d1\uc911\uc870\uba85\ud558\uba74\uc11c \uacf5\ub3d9\uccb4\ub97c \uad50\uc721\ud558\uc600\ub2e4. \uc774\ub4e4 \uc18c\ucc45\uc790\ub4e4\uc740 \uc804\uae30 \ucd5c\uace0\uc758 \uc544\ud504\ub9ac\uce74\uacc4 \ubbf8\uad6d\uc778 \uc5ec\uc131\uc758 \uc77c\ub300\uae30 \uc2dc\ub9ac\uc988\uc774\ub2e4. \uadf8\ub9ac\uace0, \uc804\uccb4 \ucf5c\ub809\uc158\uc740 '\uc0ac\ubc95\uacc4 \uc5ec\uc131', '\uc815\uacc4 \uc5ec\uc131', '\uc758\ub8cc\uacc4 \uc5ec\uc131', '\ube44\uc988\ub2c8\uc2a4\uacc4 \uc5ec\uc131', '\uce58\uc758\ud559\uacc4 \uc5ec\uc131'\ub4e4\uc744 \ud3ec\ud568\ud558\uace0 \uc788\ub2e4. 1970\ub144\ub300 \ucd08 AKA\ub294 \uc870\uc9c0\uc544 \uc8fc \uc560\ud2c0\ub780\ud0c0\uc5d0 \uc788\ub294 \ub9c8\ud2f4 \ub8e8\uc11c \ud0b9 \uc8fc\ub2c8\uc5b4\uc758 \uc0dd\uc77c \uc7a5\uc18c\uc5d0\uc11c \ub9c8\ud2f4 \ub8e8\uc11c \ud0b9\uc744 \ubcf4\ud638\ud558\ub294 \uc77c\uc5d0 2\ub9cc \ub2ec\ub7ec\ub97c \uae30\ubd80\ud558\uc600\ub2e4. \uc5ec\uc131\ud68c 70\uc8fc\ub144\uc778 1978\ub144, \ud558\uc6cc\ub4dc \ub300\ud559\uad50\uc5d0 AKA\uc758 \uc124\ub9bd\uc790\ub4e4\uc744 \uae30\ub9ac\ub294 '\uae30\ub150 \ucc3d\ubb38'\ub97c \uc124\uce58\ud588\ub2e4. '\uae30\ub150 \ucc3d\ubb38'\uc740 \ub85c\uc774\uc2a4 \ub9c8\uc77c\ub85c \uc874\uc2a4\uac00 \ub514\uc790\uc778\ud55c \uac83\uc774\uba70, \ud604\uc874\ud558\ub358 \uc124\ub9bd\uc790\uc778 \ub77c\ube44\ub2c8\uc544 \ub178\uba3c\uacfc \ub178\ub9c8 \ubcf4\uc774\ub4dc\uac00 '\uae30\ub150 \ucc3d\ubb38'\uc758 \uacf5\uac1c\uc2dd\uc5d0 \ucc38\uc11d\ud558\uc600\ub2e4.", "paragraph_answer": "\uc5ec\uc131\ud68c\ub294 1968\ub144\uc5d0\uc11c 1972\ub144\uae4c\uc9c0\uc758 \uc720\uc0b0 \uc2dc\ub9ac\uc988\ub97c \ubc1c\uac04\ud568\uc73c\ub85c\uc368 \ub6f0\uc5b4\ub09c \uc5c5\uc801\uc744 \ub0a8\uae34 \uc0ac\ub78c\ub4e4\uc744 \uc9d1\uc911\uc870\uba85\ud558\uba74\uc11c \uacf5\ub3d9\uccb4\ub97c \uad50\uc721\ud558\uc600\ub2e4. \uc774\ub4e4 \uc18c\ucc45\uc790\ub4e4\uc740 \uc804\uae30 \ucd5c\uace0\uc758 \uc544\ud504\ub9ac\uce74\uacc4 \ubbf8\uad6d\uc778 \uc5ec\uc131\uc758 \uc77c\ub300\uae30 \uc2dc\ub9ac\uc988\uc774\ub2e4. \uadf8\ub9ac\uace0, \uc804\uccb4 \ucf5c\ub809\uc158\uc740 '\uc0ac\ubc95\uacc4 \uc5ec\uc131', '\uc815\uacc4 \uc5ec\uc131', '\uc758\ub8cc\uacc4 \uc5ec\uc131', '\ube44\uc988\ub2c8\uc2a4\uacc4 \uc5ec\uc131', '\uce58\uc758\ud559\uacc4 \uc5ec\uc131'\ub4e4\uc744 \ud3ec\ud568\ud558\uace0 \uc788\ub2e4. 1970\ub144\ub300 \ucd08 AKA\ub294 \uc870\uc9c0\uc544 \uc8fc \uc560\ud2c0\ub780\ud0c0\uc5d0 \uc788\ub294 \ub9c8\ud2f4 \ub8e8\uc11c \ud0b9 \uc8fc\ub2c8\uc5b4\uc758 \uc0dd\uc77c \uc7a5\uc18c\uc5d0\uc11c \ub9c8\ud2f4 \ub8e8\uc11c \ud0b9\uc744 \ubcf4\ud638\ud558\ub294 \uc77c\uc5d0 2\ub9cc \ub2ec\ub7ec\ub97c \uae30\ubd80\ud558\uc600\ub2e4. \uc5ec\uc131\ud68c 70\uc8fc\ub144\uc778 1978\ub144, \ud558\uc6cc\ub4dc \ub300\ud559\uad50\uc5d0 AKA\uc758 \uc124\ub9bd\uc790\ub4e4\uc744 \uae30\ub9ac\ub294 '\uae30\ub150 \ucc3d\ubb38'\ub97c \uc124\uce58\ud588\ub2e4. '\uae30\ub150 \ucc3d\ubb38'\uc740 \ub85c\uc774\uc2a4 \ub9c8\uc77c\ub85c \uc874\uc2a4\uac00 \ub514\uc790\uc778\ud55c \uac83\uc774\uba70, \ud604\uc874\ud558\ub358 \uc124\ub9bd\uc790\uc778 \ub77c\ube44\ub2c8\uc544 \ub178\uba3c\uacfc \ub178\ub9c8 \ubcf4\uc774\ub4dc\uac00 '\uae30\ub150 \ucc3d\ubb38'\uc758 \uacf5\uac1c\uc2dd\uc5d0 \ucc38\uc11d\ud558\uc600\ub2e4.", "sentence_answer": "1970\ub144\ub300 \ucd08 AKA\ub294 \uc870\uc9c0\uc544 \uc8fc \uc560\ud2c0\ub780\ud0c0\uc5d0 \uc788\ub294 \ub9c8\ud2f4 \ub8e8\uc11c \ud0b9 \uc8fc\ub2c8\uc5b4\uc758 \uc0dd\uc77c \uc7a5\uc18c\uc5d0\uc11c \ub9c8\ud2f4 \ub8e8\uc11c \ud0b9\uc744 \ubcf4\ud638\ud558\ub294 \uc77c\uc5d0 2\ub9cc \ub2ec\ub7ec\ub97c \uae30\ubd80\ud558\uc600\ub2e4.", "paragraph_id": "6604746-7-1", "paragraph_question": "question: 1970\ub144\ub300 \ucd08 AKA\uac00 \ubcf4\ud638\ud558\uae30 \uc704\ud574 2\ub9cc \ub2ec\ub7ec\ub97c \uae30\ubd80\ud588\ub358 \ub300\uc0c1\uc740 \ub204\uad6c\uc778\uac00?, context: \uc5ec\uc131\ud68c\ub294 1968\ub144\uc5d0\uc11c 1972\ub144\uae4c\uc9c0\uc758 \uc720\uc0b0 \uc2dc\ub9ac\uc988\ub97c \ubc1c\uac04\ud568\uc73c\ub85c\uc368 \ub6f0\uc5b4\ub09c \uc5c5\uc801\uc744 \ub0a8\uae34 \uc0ac\ub78c\ub4e4\uc744 \uc9d1\uc911\uc870\uba85\ud558\uba74\uc11c \uacf5\ub3d9\uccb4\ub97c \uad50\uc721\ud558\uc600\ub2e4. \uc774\ub4e4 \uc18c\ucc45\uc790\ub4e4\uc740 \uc804\uae30 \ucd5c\uace0\uc758 \uc544\ud504\ub9ac\uce74\uacc4 \ubbf8\uad6d\uc778 \uc5ec\uc131\uc758 \uc77c\ub300\uae30 \uc2dc\ub9ac\uc988\uc774\ub2e4. \uadf8\ub9ac\uace0, \uc804\uccb4 \ucf5c\ub809\uc158\uc740 '\uc0ac\ubc95\uacc4 \uc5ec\uc131', '\uc815\uacc4 \uc5ec\uc131', '\uc758\ub8cc\uacc4 \uc5ec\uc131', '\ube44\uc988\ub2c8\uc2a4\uacc4 \uc5ec\uc131', '\uce58\uc758\ud559\uacc4 \uc5ec\uc131'\ub4e4\uc744 \ud3ec\ud568\ud558\uace0 \uc788\ub2e4. 1970\ub144\ub300 \ucd08 AKA\ub294 \uc870\uc9c0\uc544 \uc8fc \uc560\ud2c0\ub780\ud0c0\uc5d0 \uc788\ub294 \ub9c8\ud2f4 \ub8e8\uc11c \ud0b9 \uc8fc\ub2c8\uc5b4\uc758 \uc0dd\uc77c \uc7a5\uc18c\uc5d0\uc11c \ub9c8\ud2f4 \ub8e8\uc11c \ud0b9\uc744 \ubcf4\ud638\ud558\ub294 \uc77c\uc5d0 2\ub9cc \ub2ec\ub7ec\ub97c \uae30\ubd80\ud558\uc600\ub2e4. \uc5ec\uc131\ud68c 70\uc8fc\ub144\uc778 1978\ub144, \ud558\uc6cc\ub4dc \ub300\ud559\uad50\uc5d0 AKA\uc758 \uc124\ub9bd\uc790\ub4e4\uc744 \uae30\ub9ac\ub294 '\uae30\ub150 \ucc3d\ubb38'\ub97c \uc124\uce58\ud588\ub2e4. '\uae30\ub150 \ucc3d\ubb38'\uc740 \ub85c\uc774\uc2a4 \ub9c8\uc77c\ub85c \uc874\uc2a4\uac00 \ub514\uc790\uc778\ud55c \uac83\uc774\uba70, \ud604\uc874\ud558\ub358 \uc124\ub9bd\uc790\uc778 \ub77c\ube44\ub2c8\uc544 \ub178\uba3c\uacfc \ub178\ub9c8 \ubcf4\uc774\ub4dc\uac00 '\uae30\ub150 \ucc3d\ubb38'\uc758 \uacf5\uac1c\uc2dd\uc5d0 \ucc38\uc11d\ud558\uc600\ub2e4."} {"question": "\uc774\uc21c\uc2e0\uc774 \uc804\uc0ac\ud55c \uacf3\uc740?", "paragraph": "1598\ub144 \uc74c\ub825 11\uc6d4 18\uc77c\uc5d0 \uc870\uc120 \uc218\uad70 70\uc5ec \ucc99, \uba85\ub098\ub77c \uc218\uad70 400\ucc99\uc774 \ub178\ub7c9\uc73c\ub85c \uc9c4\uad70\ud588\ub2e4. \uad70\uc0ac\ub294 1\ub9cc 5\ucc9c\uba85\uc774\uc5c8\ub2e4. \uc774\uc21c\uc2e0\uc740 \uba85\ub098\ub77c \ubd80\ucd1d\ubcd1 \uc9c4\ub9b0(\u9673\u7498)\uacfc \ud568\uaed8 1598\ub144 \uc74c\ub825 11\uc6d4 19\uc77c \uc0c8\ubcbd\ubd80\ud130 \ub178\ub7c9\ud574\ud611\uc5d0 \ubaa8\uc5ec \uc788\ub294 \uc77c\ubcf8\uad70\uc744 \uacf5\uaca9\ud558\uc600\uace0, \uc77c\ubcf8\uc73c\ub85c \uac74\ub108\uac08 \uc900\ube44\ub97c \ud558\uace0 \uc788\ub358 \uc65c\uad70 \uc120\ub2e8 500\uc5ec \ucc99 \uac00\uc6b4\ub370 200\uc5ec \ucc99\uc744 \uaca9\ud30c, 150\uc5ec \ucc99\uc744 \ud30c\uc190\uc2dc\ucf30\ub2e4. \uc804\ud22c\ub294 \uc815\uc624\uae4c\uc9c0 \uc774\uc5b4\uc84c\uace0, \uad00\uc74c\ud3ec\ub85c \ub2ec\uc544\ub098\ub294 \uc65c\uad70\uc744 \ucd94\uc801\ud558\ub358 \uc774\uc21c\uc2e0\uc740 \ub0a0\uc544\uc628 \ud0c4\ud658\uc5d0 \ub9de\uc544 \uc804\uc0ac\ud558\uc600\ub2e4. \uc8fd\uae30 \uc804\uc5d0 \uadf8\ub294 \"\uc9c0\uae08\uc740 \uc2f8\uc6c0\uc774 \uae09\ud558\ub2e4. \ub098\uc758 \uc8fd\uc74c\uc744 \uc54c\ub9ac\uc9c0 \ub9d0\ub77c.\"\ub294 \ub9d0\uc744 \ub0a8\uacbc\ub2e4\uace0 \ud55c\ub2e4. \ud5a5\ub144 54\uc138. \ub099\uc548\uad70\uc218 \ubc29\ub355\ub8e1(\u65b9\u5fb7\u9f8d), \uac00\ub9ac\ud3ec\ucca8\uc0ac \uc774\uc601\ub0a8(\u674e\u82f1\u7537)\uacfc \uba85\uc758 \uc7a5\uc218 \ub4f1\uc790\ub8e1(\u9127\u5b50\u9f8d)\ub3c4 \ud568\uaed8 \uc804\uc0ac\ud588\ub2e4.", "answer": "\ub178\ub7c9\ud574\ud611", "sentence": "\uc774\uc21c\uc2e0\uc740 \uba85\ub098\ub77c \ubd80\ucd1d\ubcd1 \uc9c4\ub9b0(\u9673\u7498)\uacfc \ud568\uaed8 1598\ub144 \uc74c\ub825 11\uc6d4 19\uc77c \uc0c8\ubcbd\ubd80\ud130 \ub178\ub7c9\ud574\ud611 \uc5d0 \ubaa8\uc5ec \uc788\ub294 \uc77c\ubcf8\uad70\uc744 \uacf5\uaca9\ud558\uc600\uace0, \uc77c\ubcf8\uc73c\ub85c \uac74\ub108\uac08 \uc900\ube44\ub97c \ud558\uace0 \uc788\ub358 \uc65c\uad70 \uc120\ub2e8 500\uc5ec \ucc99 \uac00\uc6b4\ub370 200\uc5ec \ucc99\uc744 \uaca9\ud30c, 150\uc5ec \ucc99\uc744 \ud30c\uc190\uc2dc\ucf30\ub2e4.", "paragraph_sentence": "1598\ub144 \uc74c\ub825 11\uc6d4 18\uc77c\uc5d0 \uc870\uc120 \uc218\uad70 70\uc5ec \ucc99, \uba85\ub098\ub77c \uc218\uad70 400\ucc99\uc774 \ub178\ub7c9\uc73c\ub85c \uc9c4\uad70\ud588\ub2e4. \uad70\uc0ac\ub294 1\ub9cc 5\ucc9c\uba85\uc774\uc5c8\ub2e4. \uc774\uc21c\uc2e0\uc740 \uba85\ub098\ub77c \ubd80\ucd1d\ubcd1 \uc9c4\ub9b0(\u9673\u7498)\uacfc \ud568\uaed8 1598\ub144 \uc74c\ub825 11\uc6d4 19\uc77c \uc0c8\ubcbd\ubd80\ud130 \ub178\ub7c9\ud574\ud611 \uc5d0 \ubaa8\uc5ec \uc788\ub294 \uc77c\ubcf8\uad70\uc744 \uacf5\uaca9\ud558\uc600\uace0, \uc77c\ubcf8\uc73c\ub85c \uac74\ub108\uac08 \uc900\ube44\ub97c \ud558\uace0 \uc788\ub358 \uc65c\uad70 \uc120\ub2e8 500\uc5ec \ucc99 \uac00\uc6b4\ub370 200\uc5ec \ucc99\uc744 \uaca9\ud30c, 150\uc5ec \ucc99\uc744 \ud30c\uc190\uc2dc\ucf30\ub2e4. \uc804\ud22c\ub294 \uc815\uc624\uae4c\uc9c0 \uc774\uc5b4\uc84c\uace0, \uad00\uc74c\ud3ec\ub85c \ub2ec\uc544\ub098\ub294 \uc65c\uad70\uc744 \ucd94\uc801\ud558\ub358 \uc774\uc21c\uc2e0\uc740 \ub0a0\uc544\uc628 \ud0c4\ud658\uc5d0 \ub9de\uc544 \uc804\uc0ac\ud558\uc600\ub2e4. \uc8fd\uae30 \uc804\uc5d0 \uadf8\ub294 \"\uc9c0\uae08\uc740 \uc2f8\uc6c0\uc774 \uae09\ud558\ub2e4. \ub098\uc758 \uc8fd\uc74c\uc744 \uc54c\ub9ac\uc9c0 \ub9d0\ub77c. \"\ub294 \ub9d0\uc744 \ub0a8\uacbc\ub2e4\uace0 \ud55c\ub2e4. \ud5a5\ub144 54\uc138. \ub099\uc548\uad70\uc218 \ubc29\ub355\ub8e1(\u65b9\u5fb7\u9f8d), \uac00\ub9ac\ud3ec\ucca8\uc0ac \uc774\uc601\ub0a8(\u674e\u82f1\u7537)\uacfc \uba85\uc758 \uc7a5\uc218 \ub4f1\uc790\ub8e1(\u9127\u5b50\u9f8d)\ub3c4 \ud568\uaed8 \uc804\uc0ac\ud588\ub2e4.", "paragraph_answer": "1598\ub144 \uc74c\ub825 11\uc6d4 18\uc77c\uc5d0 \uc870\uc120 \uc218\uad70 70\uc5ec \ucc99, \uba85\ub098\ub77c \uc218\uad70 400\ucc99\uc774 \ub178\ub7c9\uc73c\ub85c \uc9c4\uad70\ud588\ub2e4. \uad70\uc0ac\ub294 1\ub9cc 5\ucc9c\uba85\uc774\uc5c8\ub2e4. \uc774\uc21c\uc2e0\uc740 \uba85\ub098\ub77c \ubd80\ucd1d\ubcd1 \uc9c4\ub9b0(\u9673\u7498)\uacfc \ud568\uaed8 1598\ub144 \uc74c\ub825 11\uc6d4 19\uc77c \uc0c8\ubcbd\ubd80\ud130 \ub178\ub7c9\ud574\ud611 \uc5d0 \ubaa8\uc5ec \uc788\ub294 \uc77c\ubcf8\uad70\uc744 \uacf5\uaca9\ud558\uc600\uace0, \uc77c\ubcf8\uc73c\ub85c \uac74\ub108\uac08 \uc900\ube44\ub97c \ud558\uace0 \uc788\ub358 \uc65c\uad70 \uc120\ub2e8 500\uc5ec \ucc99 \uac00\uc6b4\ub370 200\uc5ec \ucc99\uc744 \uaca9\ud30c, 150\uc5ec \ucc99\uc744 \ud30c\uc190\uc2dc\ucf30\ub2e4. \uc804\ud22c\ub294 \uc815\uc624\uae4c\uc9c0 \uc774\uc5b4\uc84c\uace0, \uad00\uc74c\ud3ec\ub85c \ub2ec\uc544\ub098\ub294 \uc65c\uad70\uc744 \ucd94\uc801\ud558\ub358 \uc774\uc21c\uc2e0\uc740 \ub0a0\uc544\uc628 \ud0c4\ud658\uc5d0 \ub9de\uc544 \uc804\uc0ac\ud558\uc600\ub2e4. \uc8fd\uae30 \uc804\uc5d0 \uadf8\ub294 \"\uc9c0\uae08\uc740 \uc2f8\uc6c0\uc774 \uae09\ud558\ub2e4. \ub098\uc758 \uc8fd\uc74c\uc744 \uc54c\ub9ac\uc9c0 \ub9d0\ub77c.\"\ub294 \ub9d0\uc744 \ub0a8\uacbc\ub2e4\uace0 \ud55c\ub2e4. \ud5a5\ub144 54\uc138. \ub099\uc548\uad70\uc218 \ubc29\ub355\ub8e1(\u65b9\u5fb7\u9f8d), \uac00\ub9ac\ud3ec\ucca8\uc0ac \uc774\uc601\ub0a8(\u674e\u82f1\u7537)\uacfc \uba85\uc758 \uc7a5\uc218 \ub4f1\uc790\ub8e1(\u9127\u5b50\u9f8d)\ub3c4 \ud568\uaed8 \uc804\uc0ac\ud588\ub2e4.", "sentence_answer": "\uc774\uc21c\uc2e0\uc740 \uba85\ub098\ub77c \ubd80\ucd1d\ubcd1 \uc9c4\ub9b0(\u9673\u7498)\uacfc \ud568\uaed8 1598\ub144 \uc74c\ub825 11\uc6d4 19\uc77c \uc0c8\ubcbd\ubd80\ud130 \ub178\ub7c9\ud574\ud611 \uc5d0 \ubaa8\uc5ec \uc788\ub294 \uc77c\ubcf8\uad70\uc744 \uacf5\uaca9\ud558\uc600\uace0, \uc77c\ubcf8\uc73c\ub85c \uac74\ub108\uac08 \uc900\ube44\ub97c \ud558\uace0 \uc788\ub358 \uc65c\uad70 \uc120\ub2e8 500\uc5ec \ucc99 \uac00\uc6b4\ub370 200\uc5ec \ucc99\uc744 \uaca9\ud30c, 150\uc5ec \ucc99\uc744 \ud30c\uc190\uc2dc\ucf30\ub2e4.", "paragraph_id": "6161989-11-0", "paragraph_question": "question: \uc774\uc21c\uc2e0\uc774 \uc804\uc0ac\ud55c \uacf3\uc740?, context: 1598\ub144 \uc74c\ub825 11\uc6d4 18\uc77c\uc5d0 \uc870\uc120 \uc218\uad70 70\uc5ec \ucc99, \uba85\ub098\ub77c \uc218\uad70 400\ucc99\uc774 \ub178\ub7c9\uc73c\ub85c \uc9c4\uad70\ud588\ub2e4. \uad70\uc0ac\ub294 1\ub9cc 5\ucc9c\uba85\uc774\uc5c8\ub2e4. \uc774\uc21c\uc2e0\uc740 \uba85\ub098\ub77c \ubd80\ucd1d\ubcd1 \uc9c4\ub9b0(\u9673\u7498)\uacfc \ud568\uaed8 1598\ub144 \uc74c\ub825 11\uc6d4 19\uc77c \uc0c8\ubcbd\ubd80\ud130 \ub178\ub7c9\ud574\ud611\uc5d0 \ubaa8\uc5ec \uc788\ub294 \uc77c\ubcf8\uad70\uc744 \uacf5\uaca9\ud558\uc600\uace0, \uc77c\ubcf8\uc73c\ub85c \uac74\ub108\uac08 \uc900\ube44\ub97c \ud558\uace0 \uc788\ub358 \uc65c\uad70 \uc120\ub2e8 500\uc5ec \ucc99 \uac00\uc6b4\ub370 200\uc5ec \ucc99\uc744 \uaca9\ud30c, 150\uc5ec \ucc99\uc744 \ud30c\uc190\uc2dc\ucf30\ub2e4. \uc804\ud22c\ub294 \uc815\uc624\uae4c\uc9c0 \uc774\uc5b4\uc84c\uace0, \uad00\uc74c\ud3ec\ub85c \ub2ec\uc544\ub098\ub294 \uc65c\uad70\uc744 \ucd94\uc801\ud558\ub358 \uc774\uc21c\uc2e0\uc740 \ub0a0\uc544\uc628 \ud0c4\ud658\uc5d0 \ub9de\uc544 \uc804\uc0ac\ud558\uc600\ub2e4. \uc8fd\uae30 \uc804\uc5d0 \uadf8\ub294 \"\uc9c0\uae08\uc740 \uc2f8\uc6c0\uc774 \uae09\ud558\ub2e4. \ub098\uc758 \uc8fd\uc74c\uc744 \uc54c\ub9ac\uc9c0 \ub9d0\ub77c.\"\ub294 \ub9d0\uc744 \ub0a8\uacbc\ub2e4\uace0 \ud55c\ub2e4. \ud5a5\ub144 54\uc138. \ub099\uc548\uad70\uc218 \ubc29\ub355\ub8e1(\u65b9\u5fb7\u9f8d), \uac00\ub9ac\ud3ec\ucca8\uc0ac \uc774\uc601\ub0a8(\u674e\u82f1\u7537)\uacfc \uba85\uc758 \uc7a5\uc218 \ub4f1\uc790\ub8e1(\u9127\u5b50\u9f8d)\ub3c4 \ud568\uaed8 \uc804\uc0ac\ud588\ub2e4."} {"question": "\uc720\ub7fd\ucc0c\ub974\ub808\uae30\uac00 \ud143\uc0c8\ub85c \uc0ac\ub294 \uacf3\uc740?", "paragraph": "\uc720\ub7fd \uc11c\ubd80\uc640 \uc704\ub3c4 \ubd81\uc704 40\ub3c4 \uc774\ud558\uc758 \ub0a8\ubd80 \uc9c0\uc5ed\uc5d0 \uc0ac\ub294 \uc720\ub7fd\ucc0c\ub974\ub808\uae30\ub4e4\uc740 \uc8fc\ub85c \ud143\uc0c8\uc774\ub098, \ub2e4\ub978 \uc9c0\uc5ed\uc758 \uc720\ub7fd\ucc0c\ub974\ub808\uae30\ub4e4\uc740 \uaca8\uc6b8\uc774 \ud639\ub3c5\ud574\uc838 \ub545\uc774 \uc5bc\uace0 \uba39\uc744 \uac8c \uc801\uc5b4\uc9c0\uba74 \ub2e4\ub978 \uc9c0\uc5ed\uc73c\ub85c \uc774\ub3d9\ud558\ub294 \ucca0\uc0c8\ub85c \uc0dd\ud65c\ud55c\ub2e4. \uaca8\uc6b8\uc774 \ub418\uba74 \ubd81\uc720\ub7fd, \ub7ec\uc2dc\uc544, \uc6b0\ud06c\ub77c\uc774\ub098 \uc77c\ub300\uc5d0\uc11c \ub300\uaddc\ubaa8\uc758 \uc720\ub7fd\ucc0c\ub974\ub808\uae30\uac00 \uc11c\ub0a8\ucabd \ub610\ub294 \ub3d9\ub0a8\ucabd\uc73c\ub85c \uc774\ub3d9\ud55c\ub2e4. \ub3d9\uc720\ub7fd\uc758 \uac1c\uccb4\ub4e4\uc774 \ub3c4\ucc29\ud560 \uc988\uc74c\uc778 \uac00\uc744\uc774 \ub418\uba74 \uc601\uad6d\uc758 \uc720\ub7fd\ucc0c\ub974\ub808\uae30 \uac1c\uccb4\ub4e4\uc740 \uc774\ubca0\ub9ac\uc544\uc640 \ubd81\uc544\ud504\ub9ac\uce74\ub85c \ub5a0\ub09c\ub2e4. \uc774\ub807\uac8c \uc774\ub3d9\ud558\uba74\uc11c \ub2e4\ub978 \uc9d1\ub2e8\ub4e4\ub07c\ub9ac \uc870\uc6b0\ud558\uae30\ub3c4 \ud558\uace0 \uc11c\ub85c \ub2e4\ub978 \uc0c8 \ub5bc\uc758 \ud750\ub984\uc744 \ub9c8\uc8fc\uccd0 \uc9c0\ub098\uac00\uae30\ub3c4 \ud55c\ub2e4. \uc789\uae00\ub79c\ub4dc \uba38\uc2dc\uc0ac\uc774\ub4dc\uc5d0\uc11c \uc544\uc9c1 \ub465\uc9c0\ub97c \ubabb \ubc97\uc5b4\ub09c \ubcd1\uc544\ub9ac 15,000 \ub9c8\ub9ac\uc758 \ub2e4\ub9ac\uc5d0 \ubc18\uc9c0\ub97c \ub07c\uc6e0\ub294\ub370, \uc2dc\uac04\uc774 \uc9c0\ub098\uba74\uc11c \uc774 \uac1c\uccb4\ub4e4\uc740 \uba40\ub9ac \ub178\ub974\uc6e8\uc774, \uc2a4\uc6e8\ub374, \ud540\ub780\ub4dc, \ub7ec\uc2dc\uc544, \uc6b0\ud06c\ub77c\uc774\ub098, \ud3f4\ub780\ub4dc, \ub3c5\uc77c, \ub2c8\ub354\ub780\ud2b8 \ub4f1\uc9c0\uc5d0\uc11c \ubc1c\uacac\ub418\uc5c8\ub2e4. \uc77c\ubcf8\uacfc \ud64d\ucf69\uc5d0\uc11c \uc18c\uc218\uc758 \uc720\ub7fd\ucc0c\ub974\ub808\uae30\ub97c \ubcf4\uc558\ub2e4\ub294 \ubaa9\uaca9\ub2f4\uc774 \uc0b0\ubc1c\uc801\uc73c\ub85c \ub4e4\ub824\uc624\uc9c0\ub9cc \uc774 \uc0c8\ub4e4\uc774 \uc5b4\ub514\uc11c \uc654\ub294\uc9c0\ub294 \ubd88\ud655\uc2e4\ud558\ub2e4. \ubd81\uc544\uba54\ub9ac\uce74\uc5d0\uc11c\ub294 \ubd81\ubd80 \uac1c\uccb4\uad70\uc740 \ucca0\uc0c8\ucc98\ub7fc \ud589\ub3d9\ud558\uc5ec \uaca8\uc6b8\uc774 \ub418\uba74 \uce90\ub098\ub2e4\uc5d0\uc120 \uc720\ub7fd\ucc0c\ub974\ub808\uae30\ub97c \ucc3e\uc744 \uc218\uac00 \uc5c6\uac8c \ub41c\ub2e4. \ub3d9\ubd80\uc758 \uc0c8\ub4e4\uc740 \ub0a8\ucabd\uc73c\ub85c \uc774\ub3d9\ud558\uace0, \ub354\uc6b1 \ub0a8\ucabd\uc73c\ub85c \uac00 \ubbf8\uad6d \ub0a8\uc11c\ubd80\uc5d0\uc11c \uaca8\uc6b8\uc744 \ub09c\ub2e4.", "answer": "\uc720\ub7fd \uc11c\ubd80\uc640 \uc704\ub3c4 \ubd81\uc704 40\ub3c4 \uc774\ud558\uc758 \ub0a8\ubd80 \uc9c0\uc5ed", "sentence": "\uc720\ub7fd \uc11c\ubd80\uc640 \uc704\ub3c4 \ubd81\uc704 40\ub3c4 \uc774\ud558\uc758 \ub0a8\ubd80 \uc9c0\uc5ed \uc5d0 \uc0ac\ub294 \uc720\ub7fd\ucc0c\ub974\ub808\uae30\ub4e4\uc740 \uc8fc\ub85c \ud143\uc0c8\uc774\ub098, \ub2e4\ub978 \uc9c0\uc5ed\uc758 \uc720\ub7fd\ucc0c\ub974\ub808\uae30\ub4e4\uc740 \uaca8\uc6b8\uc774 \ud639\ub3c5\ud574\uc838 \ub545\uc774 \uc5bc\uace0 \uba39\uc744 \uac8c \uc801\uc5b4\uc9c0\uba74 \ub2e4\ub978 \uc9c0\uc5ed\uc73c\ub85c \uc774\ub3d9\ud558\ub294 \ucca0\uc0c8\ub85c \uc0dd\ud65c\ud55c\ub2e4.", "paragraph_sentence": " \uc720\ub7fd \uc11c\ubd80\uc640 \uc704\ub3c4 \ubd81\uc704 40\ub3c4 \uc774\ud558\uc758 \ub0a8\ubd80 \uc9c0\uc5ed \uc5d0 \uc0ac\ub294 \uc720\ub7fd\ucc0c\ub974\ub808\uae30\ub4e4\uc740 \uc8fc\ub85c \ud143\uc0c8\uc774\ub098, \ub2e4\ub978 \uc9c0\uc5ed\uc758 \uc720\ub7fd\ucc0c\ub974\ub808\uae30\ub4e4\uc740 \uaca8\uc6b8\uc774 \ud639\ub3c5\ud574\uc838 \ub545\uc774 \uc5bc\uace0 \uba39\uc744 \uac8c \uc801\uc5b4\uc9c0\uba74 \ub2e4\ub978 \uc9c0\uc5ed\uc73c\ub85c \uc774\ub3d9\ud558\ub294 \ucca0\uc0c8\ub85c \uc0dd\ud65c\ud55c\ub2e4. \uaca8\uc6b8\uc774 \ub418\uba74 \ubd81\uc720\ub7fd, \ub7ec\uc2dc\uc544, \uc6b0\ud06c\ub77c\uc774\ub098 \uc77c\ub300\uc5d0\uc11c \ub300\uaddc\ubaa8\uc758 \uc720\ub7fd\ucc0c\ub974\ub808\uae30\uac00 \uc11c\ub0a8\ucabd \ub610\ub294 \ub3d9\ub0a8\ucabd\uc73c\ub85c \uc774\ub3d9\ud55c\ub2e4. \ub3d9\uc720\ub7fd\uc758 \uac1c\uccb4\ub4e4\uc774 \ub3c4\ucc29\ud560 \uc988\uc74c\uc778 \uac00\uc744\uc774 \ub418\uba74 \uc601\uad6d\uc758 \uc720\ub7fd\ucc0c\ub974\ub808\uae30 \uac1c\uccb4\ub4e4\uc740 \uc774\ubca0\ub9ac\uc544\uc640 \ubd81\uc544\ud504\ub9ac\uce74\ub85c \ub5a0\ub09c\ub2e4. \uc774\ub807\uac8c \uc774\ub3d9\ud558\uba74\uc11c \ub2e4\ub978 \uc9d1\ub2e8\ub4e4\ub07c\ub9ac \uc870\uc6b0\ud558\uae30\ub3c4 \ud558\uace0 \uc11c\ub85c \ub2e4\ub978 \uc0c8 \ub5bc\uc758 \ud750\ub984\uc744 \ub9c8\uc8fc\uccd0 \uc9c0\ub098\uac00\uae30\ub3c4 \ud55c\ub2e4. \uc789\uae00\ub79c\ub4dc \uba38\uc2dc\uc0ac\uc774\ub4dc\uc5d0\uc11c \uc544\uc9c1 \ub465\uc9c0\ub97c \ubabb \ubc97\uc5b4\ub09c \ubcd1\uc544\ub9ac 15,000 \ub9c8\ub9ac\uc758 \ub2e4\ub9ac\uc5d0 \ubc18\uc9c0\ub97c \ub07c\uc6e0\ub294\ub370, \uc2dc\uac04\uc774 \uc9c0\ub098\uba74\uc11c \uc774 \uac1c\uccb4\ub4e4\uc740 \uba40\ub9ac \ub178\ub974\uc6e8\uc774, \uc2a4\uc6e8\ub374, \ud540\ub780\ub4dc, \ub7ec\uc2dc\uc544, \uc6b0\ud06c\ub77c\uc774\ub098, \ud3f4\ub780\ub4dc, \ub3c5\uc77c, \ub2c8\ub354\ub780\ud2b8 \ub4f1\uc9c0\uc5d0\uc11c \ubc1c\uacac\ub418\uc5c8\ub2e4. \uc77c\ubcf8\uacfc \ud64d\ucf69\uc5d0\uc11c \uc18c\uc218\uc758 \uc720\ub7fd\ucc0c\ub974\ub808\uae30\ub97c \ubcf4\uc558\ub2e4\ub294 \ubaa9\uaca9\ub2f4\uc774 \uc0b0\ubc1c\uc801\uc73c\ub85c \ub4e4\ub824\uc624\uc9c0\ub9cc \uc774 \uc0c8\ub4e4\uc774 \uc5b4\ub514\uc11c \uc654\ub294\uc9c0\ub294 \ubd88\ud655\uc2e4\ud558\ub2e4. \ubd81\uc544\uba54\ub9ac\uce74\uc5d0\uc11c\ub294 \ubd81\ubd80 \uac1c\uccb4\uad70\uc740 \ucca0\uc0c8\ucc98\ub7fc \ud589\ub3d9\ud558\uc5ec \uaca8\uc6b8\uc774 \ub418\uba74 \uce90\ub098\ub2e4\uc5d0\uc120 \uc720\ub7fd\ucc0c\ub974\ub808\uae30\ub97c \ucc3e\uc744 \uc218\uac00 \uc5c6\uac8c \ub41c\ub2e4. \ub3d9\ubd80\uc758 \uc0c8\ub4e4\uc740 \ub0a8\ucabd\uc73c\ub85c \uc774\ub3d9\ud558\uace0, \ub354\uc6b1 \ub0a8\ucabd\uc73c\ub85c \uac00 \ubbf8\uad6d \ub0a8\uc11c\ubd80\uc5d0\uc11c \uaca8\uc6b8\uc744 \ub09c\ub2e4.", "paragraph_answer": " \uc720\ub7fd \uc11c\ubd80\uc640 \uc704\ub3c4 \ubd81\uc704 40\ub3c4 \uc774\ud558\uc758 \ub0a8\ubd80 \uc9c0\uc5ed \uc5d0 \uc0ac\ub294 \uc720\ub7fd\ucc0c\ub974\ub808\uae30\ub4e4\uc740 \uc8fc\ub85c \ud143\uc0c8\uc774\ub098, \ub2e4\ub978 \uc9c0\uc5ed\uc758 \uc720\ub7fd\ucc0c\ub974\ub808\uae30\ub4e4\uc740 \uaca8\uc6b8\uc774 \ud639\ub3c5\ud574\uc838 \ub545\uc774 \uc5bc\uace0 \uba39\uc744 \uac8c \uc801\uc5b4\uc9c0\uba74 \ub2e4\ub978 \uc9c0\uc5ed\uc73c\ub85c \uc774\ub3d9\ud558\ub294 \ucca0\uc0c8\ub85c \uc0dd\ud65c\ud55c\ub2e4. \uaca8\uc6b8\uc774 \ub418\uba74 \ubd81\uc720\ub7fd, \ub7ec\uc2dc\uc544, \uc6b0\ud06c\ub77c\uc774\ub098 \uc77c\ub300\uc5d0\uc11c \ub300\uaddc\ubaa8\uc758 \uc720\ub7fd\ucc0c\ub974\ub808\uae30\uac00 \uc11c\ub0a8\ucabd \ub610\ub294 \ub3d9\ub0a8\ucabd\uc73c\ub85c \uc774\ub3d9\ud55c\ub2e4. \ub3d9\uc720\ub7fd\uc758 \uac1c\uccb4\ub4e4\uc774 \ub3c4\ucc29\ud560 \uc988\uc74c\uc778 \uac00\uc744\uc774 \ub418\uba74 \uc601\uad6d\uc758 \uc720\ub7fd\ucc0c\ub974\ub808\uae30 \uac1c\uccb4\ub4e4\uc740 \uc774\ubca0\ub9ac\uc544\uc640 \ubd81\uc544\ud504\ub9ac\uce74\ub85c \ub5a0\ub09c\ub2e4. \uc774\ub807\uac8c \uc774\ub3d9\ud558\uba74\uc11c \ub2e4\ub978 \uc9d1\ub2e8\ub4e4\ub07c\ub9ac \uc870\uc6b0\ud558\uae30\ub3c4 \ud558\uace0 \uc11c\ub85c \ub2e4\ub978 \uc0c8 \ub5bc\uc758 \ud750\ub984\uc744 \ub9c8\uc8fc\uccd0 \uc9c0\ub098\uac00\uae30\ub3c4 \ud55c\ub2e4. \uc789\uae00\ub79c\ub4dc \uba38\uc2dc\uc0ac\uc774\ub4dc\uc5d0\uc11c \uc544\uc9c1 \ub465\uc9c0\ub97c \ubabb \ubc97\uc5b4\ub09c \ubcd1\uc544\ub9ac 15,000 \ub9c8\ub9ac\uc758 \ub2e4\ub9ac\uc5d0 \ubc18\uc9c0\ub97c \ub07c\uc6e0\ub294\ub370, \uc2dc\uac04\uc774 \uc9c0\ub098\uba74\uc11c \uc774 \uac1c\uccb4\ub4e4\uc740 \uba40\ub9ac \ub178\ub974\uc6e8\uc774, \uc2a4\uc6e8\ub374, \ud540\ub780\ub4dc, \ub7ec\uc2dc\uc544, \uc6b0\ud06c\ub77c\uc774\ub098, \ud3f4\ub780\ub4dc, \ub3c5\uc77c, \ub2c8\ub354\ub780\ud2b8 \ub4f1\uc9c0\uc5d0\uc11c \ubc1c\uacac\ub418\uc5c8\ub2e4. \uc77c\ubcf8\uacfc \ud64d\ucf69\uc5d0\uc11c \uc18c\uc218\uc758 \uc720\ub7fd\ucc0c\ub974\ub808\uae30\ub97c \ubcf4\uc558\ub2e4\ub294 \ubaa9\uaca9\ub2f4\uc774 \uc0b0\ubc1c\uc801\uc73c\ub85c \ub4e4\ub824\uc624\uc9c0\ub9cc \uc774 \uc0c8\ub4e4\uc774 \uc5b4\ub514\uc11c \uc654\ub294\uc9c0\ub294 \ubd88\ud655\uc2e4\ud558\ub2e4. \ubd81\uc544\uba54\ub9ac\uce74\uc5d0\uc11c\ub294 \ubd81\ubd80 \uac1c\uccb4\uad70\uc740 \ucca0\uc0c8\ucc98\ub7fc \ud589\ub3d9\ud558\uc5ec \uaca8\uc6b8\uc774 \ub418\uba74 \uce90\ub098\ub2e4\uc5d0\uc120 \uc720\ub7fd\ucc0c\ub974\ub808\uae30\ub97c \ucc3e\uc744 \uc218\uac00 \uc5c6\uac8c \ub41c\ub2e4. \ub3d9\ubd80\uc758 \uc0c8\ub4e4\uc740 \ub0a8\ucabd\uc73c\ub85c \uc774\ub3d9\ud558\uace0, \ub354\uc6b1 \ub0a8\ucabd\uc73c\ub85c \uac00 \ubbf8\uad6d \ub0a8\uc11c\ubd80\uc5d0\uc11c \uaca8\uc6b8\uc744 \ub09c\ub2e4.", "sentence_answer": " \uc720\ub7fd \uc11c\ubd80\uc640 \uc704\ub3c4 \ubd81\uc704 40\ub3c4 \uc774\ud558\uc758 \ub0a8\ubd80 \uc9c0\uc5ed \uc5d0 \uc0ac\ub294 \uc720\ub7fd\ucc0c\ub974\ub808\uae30\ub4e4\uc740 \uc8fc\ub85c \ud143\uc0c8\uc774\ub098, \ub2e4\ub978 \uc9c0\uc5ed\uc758 \uc720\ub7fd\ucc0c\ub974\ub808\uae30\ub4e4\uc740 \uaca8\uc6b8\uc774 \ud639\ub3c5\ud574\uc838 \ub545\uc774 \uc5bc\uace0 \uba39\uc744 \uac8c \uc801\uc5b4\uc9c0\uba74 \ub2e4\ub978 \uc9c0\uc5ed\uc73c\ub85c \uc774\ub3d9\ud558\ub294 \ucca0\uc0c8\ub85c \uc0dd\ud65c\ud55c\ub2e4.", "paragraph_id": "6542858-19-0", "paragraph_question": "question: \uc720\ub7fd\ucc0c\ub974\ub808\uae30\uac00 \ud143\uc0c8\ub85c \uc0ac\ub294 \uacf3\uc740?, context: \uc720\ub7fd \uc11c\ubd80\uc640 \uc704\ub3c4 \ubd81\uc704 40\ub3c4 \uc774\ud558\uc758 \ub0a8\ubd80 \uc9c0\uc5ed\uc5d0 \uc0ac\ub294 \uc720\ub7fd\ucc0c\ub974\ub808\uae30\ub4e4\uc740 \uc8fc\ub85c \ud143\uc0c8\uc774\ub098, \ub2e4\ub978 \uc9c0\uc5ed\uc758 \uc720\ub7fd\ucc0c\ub974\ub808\uae30\ub4e4\uc740 \uaca8\uc6b8\uc774 \ud639\ub3c5\ud574\uc838 \ub545\uc774 \uc5bc\uace0 \uba39\uc744 \uac8c \uc801\uc5b4\uc9c0\uba74 \ub2e4\ub978 \uc9c0\uc5ed\uc73c\ub85c \uc774\ub3d9\ud558\ub294 \ucca0\uc0c8\ub85c \uc0dd\ud65c\ud55c\ub2e4. \uaca8\uc6b8\uc774 \ub418\uba74 \ubd81\uc720\ub7fd, \ub7ec\uc2dc\uc544, \uc6b0\ud06c\ub77c\uc774\ub098 \uc77c\ub300\uc5d0\uc11c \ub300\uaddc\ubaa8\uc758 \uc720\ub7fd\ucc0c\ub974\ub808\uae30\uac00 \uc11c\ub0a8\ucabd \ub610\ub294 \ub3d9\ub0a8\ucabd\uc73c\ub85c \uc774\ub3d9\ud55c\ub2e4. \ub3d9\uc720\ub7fd\uc758 \uac1c\uccb4\ub4e4\uc774 \ub3c4\ucc29\ud560 \uc988\uc74c\uc778 \uac00\uc744\uc774 \ub418\uba74 \uc601\uad6d\uc758 \uc720\ub7fd\ucc0c\ub974\ub808\uae30 \uac1c\uccb4\ub4e4\uc740 \uc774\ubca0\ub9ac\uc544\uc640 \ubd81\uc544\ud504\ub9ac\uce74\ub85c \ub5a0\ub09c\ub2e4. \uc774\ub807\uac8c \uc774\ub3d9\ud558\uba74\uc11c \ub2e4\ub978 \uc9d1\ub2e8\ub4e4\ub07c\ub9ac \uc870\uc6b0\ud558\uae30\ub3c4 \ud558\uace0 \uc11c\ub85c \ub2e4\ub978 \uc0c8 \ub5bc\uc758 \ud750\ub984\uc744 \ub9c8\uc8fc\uccd0 \uc9c0\ub098\uac00\uae30\ub3c4 \ud55c\ub2e4. \uc789\uae00\ub79c\ub4dc \uba38\uc2dc\uc0ac\uc774\ub4dc\uc5d0\uc11c \uc544\uc9c1 \ub465\uc9c0\ub97c \ubabb \ubc97\uc5b4\ub09c \ubcd1\uc544\ub9ac 15,000 \ub9c8\ub9ac\uc758 \ub2e4\ub9ac\uc5d0 \ubc18\uc9c0\ub97c \ub07c\uc6e0\ub294\ub370, \uc2dc\uac04\uc774 \uc9c0\ub098\uba74\uc11c \uc774 \uac1c\uccb4\ub4e4\uc740 \uba40\ub9ac \ub178\ub974\uc6e8\uc774, \uc2a4\uc6e8\ub374, \ud540\ub780\ub4dc, \ub7ec\uc2dc\uc544, \uc6b0\ud06c\ub77c\uc774\ub098, \ud3f4\ub780\ub4dc, \ub3c5\uc77c, \ub2c8\ub354\ub780\ud2b8 \ub4f1\uc9c0\uc5d0\uc11c \ubc1c\uacac\ub418\uc5c8\ub2e4. \uc77c\ubcf8\uacfc \ud64d\ucf69\uc5d0\uc11c \uc18c\uc218\uc758 \uc720\ub7fd\ucc0c\ub974\ub808\uae30\ub97c \ubcf4\uc558\ub2e4\ub294 \ubaa9\uaca9\ub2f4\uc774 \uc0b0\ubc1c\uc801\uc73c\ub85c \ub4e4\ub824\uc624\uc9c0\ub9cc \uc774 \uc0c8\ub4e4\uc774 \uc5b4\ub514\uc11c \uc654\ub294\uc9c0\ub294 \ubd88\ud655\uc2e4\ud558\ub2e4. \ubd81\uc544\uba54\ub9ac\uce74\uc5d0\uc11c\ub294 \ubd81\ubd80 \uac1c\uccb4\uad70\uc740 \ucca0\uc0c8\ucc98\ub7fc \ud589\ub3d9\ud558\uc5ec \uaca8\uc6b8\uc774 \ub418\uba74 \uce90\ub098\ub2e4\uc5d0\uc120 \uc720\ub7fd\ucc0c\ub974\ub808\uae30\ub97c \ucc3e\uc744 \uc218\uac00 \uc5c6\uac8c \ub41c\ub2e4. \ub3d9\ubd80\uc758 \uc0c8\ub4e4\uc740 \ub0a8\ucabd\uc73c\ub85c \uc774\ub3d9\ud558\uace0, \ub354\uc6b1 \ub0a8\ucabd\uc73c\ub85c \uac00 \ubbf8\uad6d \ub0a8\uc11c\ubd80\uc5d0\uc11c \uaca8\uc6b8\uc744 \ub09c\ub2e4."} {"question": "\uae40\uc5f0\uacbd\uc774 \ubca0\uc774\uc9d5 \uc62c\ub9bc\ud53d \ucd5c\uc885 \uc608\uc120\uc744 \uc55e\ub450\uace0 \uc218\uc220\ub300\uc5d0 \uc624\ub978 \uc774\uc720\ub294 \ubb34\uc5c7\uc778\uac00?", "paragraph": "\ud558\uc9c0\ub9cc 2006~2007 \uc2dc\uc98c \uc774\ud6c4\uc5d0\ub3c4 \uc67c\ucabd \ubb34\ub98e \uc5f0\uace8 \ud30c\uc5f4\ub85c \uc778\ud55c \ubd80\uc0c1\uc73c\ub85c \uc218\uc220\uc744 \ubc1b\uc544\uc57c \ud588\ub2e4. \uc7ac\ud65c\uc744 \ub9c8\uce58\uc790\ub9c8\uc790 2007\ub144 \ubc30\uad6c \uc6d4\ub4dc\ucef5 \uad6d\uac00\ub300\ud45c\ub85c \uc120\ubc1c\ub418\uc5b4 \uac70\uc758 \ubaa8\ub4e0 \uacbd\uae30\ub97c \ub6f0\uc5c8\uc73c\uba70, \uc6d4\ub4dc\ucef5 \uc77c\uc815\uc744 \ub9c8\uce58\uace0 \ub09c \ub4a4\uc5d0\ub294 \uc18c\uc18d \ud300\uc73c\ub85c \ub3cc\uc544\uc640 4\ub2ec \uac04\uc758 2007~2008 \uc2dc\uc98c\uc744 \uc18c\ud654\ud574\uc57c \ud588\ub2e4. 2007~2008 \uc2dc\uc98c\uc5d0\uc11c \uc18c\uc18d \ud300 \ud765\uad6d\uc0dd\uba85\uc740 \ucc54\ud53c\uc5b8 \uacb0\uc815\uc804\uc5d0\uc11c \uc778\ucc9c GS\uce7c\ud14d\uc2a4\uc5d0\uac8c \ub35c\ubbf8\ub97c \uc7a1\ud600 \ud1b5\ud569 \uc6b0\uc2b9\uc5d0\ub294 \uc2e4\ud328\ud588\uc9c0\ub9cc, \uae40\uc5f0\uacbd\uc740 \uc5ed\ub300 \ucd5c\uace0\uc758 \uacf5\uaca9 \uc131\uacf5\ub960\uc778 47.59%\uc744 \uae30\ub85d\ud558\uba74\uc11c \uacf5\uaca9\uc0c1\uc744 3\ub144 \uc5f0\uc18d \uc218\uc0c1\ud588\uace0, \ud765\uad6d\uc0dd\uba85\uc744 \uc815\uaddc\ub9ac\uadf8 1\uc704\ub85c \uc62c\ub9ac\ub294 \ub370\uc5d0 \uc131\uacf5\ud558\uba70 \uc815\uaddc\ub9ac\uadf8 MVP\ub3c4 3\ub144 \uc5f0\uc18d \uc218\uc0c1\ud588\ub2e4. \uadf8\ub7ec\ub098 \ub450 \ubc88\uc9f8 \uc218\uc220 \ud6c4 \ubb34\ub9ac\ud55c \uc77c\uc815 \ud0d3\uc5d0 \ubca0\uc774\uc9d5 \uc62c\ub9bc\ud53d \ucd5c\uc885 \uc608\uc120\uc744 \uc55e\ub450\uace0 \ubb34\ub98e \uc5f0\uace8\uc774 \ub2e4\uc2dc \ud30c\uc5f4\ub418\uba70 3\ub144 \uc5f0\uc18d\uc73c\ub85c \uc218\uc220\ub300\uc5d0 \uc624\ub974\uace0 \ub9d0\uc558\ub2e4.", "answer": "\ubb34\ub98e \uc5f0\uace8\uc774 \ub2e4\uc2dc \ud30c\uc5f4", "sentence": "\uadf8\ub7ec\ub098 \ub450 \ubc88\uc9f8 \uc218\uc220 \ud6c4 \ubb34\ub9ac\ud55c \uc77c\uc815 \ud0d3\uc5d0 \ubca0\uc774\uc9d5 \uc62c\ub9bc\ud53d \ucd5c\uc885 \uc608\uc120\uc744 \uc55e\ub450\uace0 \ubb34\ub98e \uc5f0\uace8\uc774 \ub2e4\uc2dc \ud30c\uc5f4 \ub418\uba70 3\ub144 \uc5f0\uc18d\uc73c\ub85c \uc218\uc220\ub300\uc5d0 \uc624\ub974\uace0 \ub9d0\uc558\ub2e4.", "paragraph_sentence": "\ud558\uc9c0\ub9cc 2006~2007 \uc2dc\uc98c \uc774\ud6c4\uc5d0\ub3c4 \uc67c\ucabd \ubb34\ub98e \uc5f0\uace8 \ud30c\uc5f4\ub85c \uc778\ud55c \ubd80\uc0c1\uc73c\ub85c \uc218\uc220\uc744 \ubc1b\uc544\uc57c \ud588\ub2e4. \uc7ac\ud65c\uc744 \ub9c8\uce58\uc790\ub9c8\uc790 2007\ub144 \ubc30\uad6c \uc6d4\ub4dc\ucef5 \uad6d\uac00\ub300\ud45c\ub85c \uc120\ubc1c\ub418\uc5b4 \uac70\uc758 \ubaa8\ub4e0 \uacbd\uae30\ub97c \ub6f0\uc5c8\uc73c\uba70, \uc6d4\ub4dc\ucef5 \uc77c\uc815\uc744 \ub9c8\uce58\uace0 \ub09c \ub4a4\uc5d0\ub294 \uc18c\uc18d \ud300\uc73c\ub85c \ub3cc\uc544\uc640 4\ub2ec \uac04\uc758 2007~2008 \uc2dc\uc98c\uc744 \uc18c\ud654\ud574\uc57c \ud588\ub2e4. 2007~2008 \uc2dc\uc98c\uc5d0\uc11c \uc18c\uc18d \ud300 \ud765\uad6d\uc0dd\uba85\uc740 \ucc54\ud53c\uc5b8 \uacb0\uc815\uc804\uc5d0\uc11c \uc778\ucc9c GS\uce7c\ud14d\uc2a4\uc5d0\uac8c \ub35c\ubbf8\ub97c \uc7a1\ud600 \ud1b5\ud569 \uc6b0\uc2b9\uc5d0\ub294 \uc2e4\ud328\ud588\uc9c0\ub9cc, \uae40\uc5f0\uacbd\uc740 \uc5ed\ub300 \ucd5c\uace0\uc758 \uacf5\uaca9 \uc131\uacf5\ub960\uc778 47.59%\uc744 \uae30\ub85d\ud558\uba74\uc11c \uacf5\uaca9\uc0c1\uc744 3\ub144 \uc5f0\uc18d \uc218\uc0c1\ud588\uace0, \ud765\uad6d\uc0dd\uba85\uc744 \uc815\uaddc\ub9ac\uadf8 1\uc704\ub85c \uc62c\ub9ac\ub294 \ub370\uc5d0 \uc131\uacf5\ud558\uba70 \uc815\uaddc\ub9ac\uadf8 MVP\ub3c4 3\ub144 \uc5f0\uc18d \uc218\uc0c1\ud588\ub2e4. \uadf8\ub7ec\ub098 \ub450 \ubc88\uc9f8 \uc218\uc220 \ud6c4 \ubb34\ub9ac\ud55c \uc77c\uc815 \ud0d3\uc5d0 \ubca0\uc774\uc9d5 \uc62c\ub9bc\ud53d \ucd5c\uc885 \uc608\uc120\uc744 \uc55e\ub450\uace0 \ubb34\ub98e \uc5f0\uace8\uc774 \ub2e4\uc2dc \ud30c\uc5f4 \ub418\uba70 3\ub144 \uc5f0\uc18d\uc73c\ub85c \uc218\uc220\ub300\uc5d0 \uc624\ub974\uace0 \ub9d0\uc558\ub2e4. ", "paragraph_answer": "\ud558\uc9c0\ub9cc 2006~2007 \uc2dc\uc98c \uc774\ud6c4\uc5d0\ub3c4 \uc67c\ucabd \ubb34\ub98e \uc5f0\uace8 \ud30c\uc5f4\ub85c \uc778\ud55c \ubd80\uc0c1\uc73c\ub85c \uc218\uc220\uc744 \ubc1b\uc544\uc57c \ud588\ub2e4. \uc7ac\ud65c\uc744 \ub9c8\uce58\uc790\ub9c8\uc790 2007\ub144 \ubc30\uad6c \uc6d4\ub4dc\ucef5 \uad6d\uac00\ub300\ud45c\ub85c \uc120\ubc1c\ub418\uc5b4 \uac70\uc758 \ubaa8\ub4e0 \uacbd\uae30\ub97c \ub6f0\uc5c8\uc73c\uba70, \uc6d4\ub4dc\ucef5 \uc77c\uc815\uc744 \ub9c8\uce58\uace0 \ub09c \ub4a4\uc5d0\ub294 \uc18c\uc18d \ud300\uc73c\ub85c \ub3cc\uc544\uc640 4\ub2ec \uac04\uc758 2007~2008 \uc2dc\uc98c\uc744 \uc18c\ud654\ud574\uc57c \ud588\ub2e4. 2007~2008 \uc2dc\uc98c\uc5d0\uc11c \uc18c\uc18d \ud300 \ud765\uad6d\uc0dd\uba85\uc740 \ucc54\ud53c\uc5b8 \uacb0\uc815\uc804\uc5d0\uc11c \uc778\ucc9c GS\uce7c\ud14d\uc2a4\uc5d0\uac8c \ub35c\ubbf8\ub97c \uc7a1\ud600 \ud1b5\ud569 \uc6b0\uc2b9\uc5d0\ub294 \uc2e4\ud328\ud588\uc9c0\ub9cc, \uae40\uc5f0\uacbd\uc740 \uc5ed\ub300 \ucd5c\uace0\uc758 \uacf5\uaca9 \uc131\uacf5\ub960\uc778 47.59%\uc744 \uae30\ub85d\ud558\uba74\uc11c \uacf5\uaca9\uc0c1\uc744 3\ub144 \uc5f0\uc18d \uc218\uc0c1\ud588\uace0, \ud765\uad6d\uc0dd\uba85\uc744 \uc815\uaddc\ub9ac\uadf8 1\uc704\ub85c \uc62c\ub9ac\ub294 \ub370\uc5d0 \uc131\uacf5\ud558\uba70 \uc815\uaddc\ub9ac\uadf8 MVP\ub3c4 3\ub144 \uc5f0\uc18d \uc218\uc0c1\ud588\ub2e4. \uadf8\ub7ec\ub098 \ub450 \ubc88\uc9f8 \uc218\uc220 \ud6c4 \ubb34\ub9ac\ud55c \uc77c\uc815 \ud0d3\uc5d0 \ubca0\uc774\uc9d5 \uc62c\ub9bc\ud53d \ucd5c\uc885 \uc608\uc120\uc744 \uc55e\ub450\uace0 \ubb34\ub98e \uc5f0\uace8\uc774 \ub2e4\uc2dc \ud30c\uc5f4 \ub418\uba70 3\ub144 \uc5f0\uc18d\uc73c\ub85c \uc218\uc220\ub300\uc5d0 \uc624\ub974\uace0 \ub9d0\uc558\ub2e4.", "sentence_answer": "\uadf8\ub7ec\ub098 \ub450 \ubc88\uc9f8 \uc218\uc220 \ud6c4 \ubb34\ub9ac\ud55c \uc77c\uc815 \ud0d3\uc5d0 \ubca0\uc774\uc9d5 \uc62c\ub9bc\ud53d \ucd5c\uc885 \uc608\uc120\uc744 \uc55e\ub450\uace0 \ubb34\ub98e \uc5f0\uace8\uc774 \ub2e4\uc2dc \ud30c\uc5f4 \ub418\uba70 3\ub144 \uc5f0\uc18d\uc73c\ub85c \uc218\uc220\ub300\uc5d0 \uc624\ub974\uace0 \ub9d0\uc558\ub2e4.", "paragraph_id": "5998742-0-2", "paragraph_question": "question: \uae40\uc5f0\uacbd\uc774 \ubca0\uc774\uc9d5 \uc62c\ub9bc\ud53d \ucd5c\uc885 \uc608\uc120\uc744 \uc55e\ub450\uace0 \uc218\uc220\ub300\uc5d0 \uc624\ub978 \uc774\uc720\ub294 \ubb34\uc5c7\uc778\uac00?, context: \ud558\uc9c0\ub9cc 2006~2007 \uc2dc\uc98c \uc774\ud6c4\uc5d0\ub3c4 \uc67c\ucabd \ubb34\ub98e \uc5f0\uace8 \ud30c\uc5f4\ub85c \uc778\ud55c \ubd80\uc0c1\uc73c\ub85c \uc218\uc220\uc744 \ubc1b\uc544\uc57c \ud588\ub2e4. \uc7ac\ud65c\uc744 \ub9c8\uce58\uc790\ub9c8\uc790 2007\ub144 \ubc30\uad6c \uc6d4\ub4dc\ucef5 \uad6d\uac00\ub300\ud45c\ub85c \uc120\ubc1c\ub418\uc5b4 \uac70\uc758 \ubaa8\ub4e0 \uacbd\uae30\ub97c \ub6f0\uc5c8\uc73c\uba70, \uc6d4\ub4dc\ucef5 \uc77c\uc815\uc744 \ub9c8\uce58\uace0 \ub09c \ub4a4\uc5d0\ub294 \uc18c\uc18d \ud300\uc73c\ub85c \ub3cc\uc544\uc640 4\ub2ec \uac04\uc758 2007~2008 \uc2dc\uc98c\uc744 \uc18c\ud654\ud574\uc57c \ud588\ub2e4. 2007~2008 \uc2dc\uc98c\uc5d0\uc11c \uc18c\uc18d \ud300 \ud765\uad6d\uc0dd\uba85\uc740 \ucc54\ud53c\uc5b8 \uacb0\uc815\uc804\uc5d0\uc11c \uc778\ucc9c GS\uce7c\ud14d\uc2a4\uc5d0\uac8c \ub35c\ubbf8\ub97c \uc7a1\ud600 \ud1b5\ud569 \uc6b0\uc2b9\uc5d0\ub294 \uc2e4\ud328\ud588\uc9c0\ub9cc, \uae40\uc5f0\uacbd\uc740 \uc5ed\ub300 \ucd5c\uace0\uc758 \uacf5\uaca9 \uc131\uacf5\ub960\uc778 47.59%\uc744 \uae30\ub85d\ud558\uba74\uc11c \uacf5\uaca9\uc0c1\uc744 3\ub144 \uc5f0\uc18d \uc218\uc0c1\ud588\uace0, \ud765\uad6d\uc0dd\uba85\uc744 \uc815\uaddc\ub9ac\uadf8 1\uc704\ub85c \uc62c\ub9ac\ub294 \ub370\uc5d0 \uc131\uacf5\ud558\uba70 \uc815\uaddc\ub9ac\uadf8 MVP\ub3c4 3\ub144 \uc5f0\uc18d \uc218\uc0c1\ud588\ub2e4. \uadf8\ub7ec\ub098 \ub450 \ubc88\uc9f8 \uc218\uc220 \ud6c4 \ubb34\ub9ac\ud55c \uc77c\uc815 \ud0d3\uc5d0 \ubca0\uc774\uc9d5 \uc62c\ub9bc\ud53d \ucd5c\uc885 \uc608\uc120\uc744 \uc55e\ub450\uace0 \ubb34\ub98e \uc5f0\uace8\uc774 \ub2e4\uc2dc \ud30c\uc5f4\ub418\uba70 3\ub144 \uc5f0\uc18d\uc73c\ub85c \uc218\uc220\ub300\uc5d0 \uc624\ub974\uace0 \ub9d0\uc558\ub2e4."} {"question": "\uae40\uad11\uc11d\uc740 \uc5b4\ub514\uc5d0 \ube44\uc2a4\ub4ec\ud788 \uc549\uc740 \ucc44\ub85c \ubc1c\uacac\ub418\uc5c8\ub294\uac00?", "paragraph": "\ucd1d 3\uac00\uc9c0 \uc8fc\uc694 \uc758\ubb38\uc810\uc774 \uc788\uc5c8\ub294\ub370 1.\uae40\uad11\uc11d\uc774 \ubaa9\uc744 \uc2a4\uc2a4\ub85c \ub9f8\ub0d0\ub294\uac83, 2. \uc720\uc11c\uac00 \ubc1c\uacac\ub418\uc9c0 \uc54a\uc740 \uc810, 3. \uc815\ub9d0\ub85c \uc6b0\uc6b8\uc99d\uc774 \uc788\uc5c8\ub0d0\ub294\uac83\uc774\ub2e4. 1\ubc88 \uc758\ubb38\uc758 \uadfc\uac70\ub294 \ubaa9\uc744 \ub9e4\ub2ec \uc804\uae43\uc904\uc744 \ubb36\uae30 \uc704\ud574 \uc4f0\uc600\uc5b4\uc57c \ud588\uc744 \uc758\uc790\uac00 \ubc1c\uacac\ub418\uc9c0 \uc54a\uc558\uace0 \ubaa9\uc744 \ub9e4\ub2ec \uc704\uce58\uc870\ucc28 \uc544\ub2c8\ub77c\ub294 \uac83\uc774\ub2e4. \uacb0\ub860\uc801\uc73c\ub85c \uc790\uc0b4\uc744 \ud560\ub9cc\ud55c \uc7a5\uc18c\uac00 \uc544\ub2c8\uc5c8\ub2e4\ub294 \uc8fc\uc7a5\uc774\ub2e4. \ub610 \uc9c0\uc778\uc740 \"\uacc4\ub2e8\uc5d0 \ube44\uc2a4\ub4ec\ud788 \uc549\uc544\uc788\ub294 \uac83\ucc98\ub7fc \ud574\uc11c \ubc1c\uacac\ub410\ub2e4. \uc904\uc774 \ub298\uc5b4\ub098\uc57c \uc0ac\ub78c\uc774 \uc11c\uc788\uc744\uac70\ub2e4. \uadfc\ub370 \uc804\uae43\uc904\uc774 \ub298\uc5b4\ub098\ub0d0\"\ub77c\uace0 \uc758\ubb38\uc744 \uc81c\uae30\ud588\ub2e4. \ud558\uc9c0\ub9cc \ub2f9\uc2dc \uace0 \uae40\uad11\uc11d \uc790\ud0dd\uc740 \ud604\uc7ac \ub0b4\ubd80 \uacf5\uc0ac\uac00 \uc774\ub8e8\uc5b4\uc838 \ud655\uc778\ud560 \uc218 \uc5c6\ub294 \uc0c1\ud0dc\uc600\ub2e4. 2\ubc88 \uc758\ubb38\uc740 \uace0 \uae40\uad11\uc11d\uc528\ub294 \uba54\ubaa8\uad11\uc73c\ub85c \uc720\uba85\ud558\uc600\ub294\ub370, \uc544\ubb34\ub7f0 \uc720\uc11c\ub3c4 \uc5c6\uc5c8\uae30 \ub54c\ubb38\uc774\ub2e4. \uace0 \uae40\uad11\uc11d \uccab\uc9f8 \uc790\ud615\uc740 \"(\uae40\uad11\uc11d\uc740) \uba54\ubaa8\uac00 \uc544\uc8fc \ud544\uc218\uc801\uc774\uace0 \uae30\ubcf8\uc801\uc778 \uc2b5\uad00\uc774 \uc788\ub2e4. \ud2c0\ub9bc\uc5c6\uc774 \uc720\uc11c\uac00 \uc788\uc744 \uac83\uc774\ub2e4\"\ub77c\uace0 \ud138\uc5b4\ub1a8\ub2e4. 3\ubc88\uc9f8 \uc774\uc720\uac00 \ub41c \uace0 \uae40\uad11\uc11d\uc528\uc758 \ubd80\uc778 \uc11c\ud574\uc21c\uc528\uac00 \uc8fc\uc7a5\ud55c \uc6b0\uc6b8\uc99d \uc5ed\uc2dc \uc9c0\uc778 \ub9d0\uc5d0 \ub530\ub974\uba74 \uc774\ud574\ub418\uc9c0 \uc54a\ub294 \ubd80\ubd84\uc774\ub2e4. \uc74c\uc545 \ub3d9\ud638\ud68c \uc9c0\uc778\uc740 \"\ubd84\uba85 \ub9d0\uc500 \ub4dc\ub9b4 \uc218 \uc788\ub2e4. \uae40\uad11\uc11d \uc528\uac00 \uc6b0\uc6b8\uc99d\uc774 \uc788\uc5c8\uc73c\uba74 \ub0b4\uac00 \ubab0\ub790\uc744 \ub9ac\uac00 \uc5c6\ub2e4\"\uba70 \"\uc801\uc5b4\ub3c4 1\ub144\ub3d9\uc548 \uac00\uc7a5 \uac00\uae4c\uc774\uc11c \uc9c0\ucf1c\ubd24\ub2e4. \ub77c\uc774\ube0c 1,000\ud68c \uacf5\uc5f0 \ud560\ub54c \ub9e4\uc77c \ubd24\uc5c8\uace0 \uadf8\ub3d9\uc548 \ubcd1\uc6d0 \uac00\ub294\uac78 \ubcf8\uc801\uc774 \uc5c6\ub2e4\"\ub77c\uace0 \uc124\uba85\ud588\ub2e4.", "answer": "\uacc4\ub2e8", "sentence": "\ub610 \uc9c0\uc778\uc740 \" \uacc4\ub2e8 \uc5d0 \ube44\uc2a4\ub4ec\ud788 \uc549\uc544\uc788\ub294 \uac83\ucc98\ub7fc \ud574\uc11c \ubc1c\uacac\ub410\ub2e4.", "paragraph_sentence": "\ucd1d 3\uac00\uc9c0 \uc8fc\uc694 \uc758\ubb38\uc810\uc774 \uc788\uc5c8\ub294\ub370 1.\uae40\uad11\uc11d\uc774 \ubaa9\uc744 \uc2a4\uc2a4\ub85c \ub9f8\ub0d0\ub294\uac83, 2. \uc720\uc11c\uac00 \ubc1c\uacac\ub418\uc9c0 \uc54a\uc740 \uc810, 3. \uc815\ub9d0\ub85c \uc6b0\uc6b8\uc99d\uc774 \uc788\uc5c8\ub0d0\ub294\uac83\uc774\ub2e4. 1\ubc88 \uc758\ubb38\uc758 \uadfc\uac70\ub294 \ubaa9\uc744 \ub9e4\ub2ec \uc804\uae43\uc904\uc744 \ubb36\uae30 \uc704\ud574 \uc4f0\uc600\uc5b4\uc57c \ud588\uc744 \uc758\uc790\uac00 \ubc1c\uacac\ub418\uc9c0 \uc54a\uc558\uace0 \ubaa9\uc744 \ub9e4\ub2ec \uc704\uce58\uc870\ucc28 \uc544\ub2c8\ub77c\ub294 \uac83\uc774\ub2e4. \uacb0\ub860\uc801\uc73c\ub85c \uc790\uc0b4\uc744 \ud560\ub9cc\ud55c \uc7a5\uc18c\uac00 \uc544\ub2c8\uc5c8\ub2e4\ub294 \uc8fc\uc7a5\uc774\ub2e4. \ub610 \uc9c0\uc778\uc740 \" \uacc4\ub2e8 \uc5d0 \ube44\uc2a4\ub4ec\ud788 \uc549\uc544\uc788\ub294 \uac83\ucc98\ub7fc \ud574\uc11c \ubc1c\uacac\ub410\ub2e4. \uc904\uc774 \ub298\uc5b4\ub098\uc57c \uc0ac\ub78c\uc774 \uc11c\uc788\uc744\uac70\ub2e4. \uadfc\ub370 \uc804\uae43\uc904\uc774 \ub298\uc5b4\ub098\ub0d0\"\ub77c\uace0 \uc758\ubb38\uc744 \uc81c\uae30\ud588\ub2e4. \ud558\uc9c0\ub9cc \ub2f9\uc2dc \uace0 \uae40\uad11\uc11d \uc790\ud0dd\uc740 \ud604\uc7ac \ub0b4\ubd80 \uacf5\uc0ac\uac00 \uc774\ub8e8\uc5b4\uc838 \ud655\uc778\ud560 \uc218 \uc5c6\ub294 \uc0c1\ud0dc\uc600\ub2e4. 2\ubc88 \uc758\ubb38\uc740 \uace0 \uae40\uad11\uc11d\uc528\ub294 \uba54\ubaa8\uad11\uc73c\ub85c \uc720\uba85\ud558\uc600\ub294\ub370, \uc544\ubb34\ub7f0 \uc720\uc11c\ub3c4 \uc5c6\uc5c8\uae30 \ub54c\ubb38\uc774\ub2e4. \uace0 \uae40\uad11\uc11d \uccab\uc9f8 \uc790\ud615\uc740 \"(\uae40\uad11\uc11d\uc740) \uba54\ubaa8\uac00 \uc544\uc8fc \ud544\uc218\uc801\uc774\uace0 \uae30\ubcf8\uc801\uc778 \uc2b5\uad00\uc774 \uc788\ub2e4. \ud2c0\ub9bc\uc5c6\uc774 \uc720\uc11c\uac00 \uc788\uc744 \uac83\uc774\ub2e4\"\ub77c\uace0 \ud138\uc5b4\ub1a8\ub2e4. 3\ubc88\uc9f8 \uc774\uc720\uac00 \ub41c \uace0 \uae40\uad11\uc11d\uc528\uc758 \ubd80\uc778 \uc11c\ud574\uc21c\uc528\uac00 \uc8fc\uc7a5\ud55c \uc6b0\uc6b8\uc99d \uc5ed\uc2dc \uc9c0\uc778 \ub9d0\uc5d0 \ub530\ub974\uba74 \uc774\ud574\ub418\uc9c0 \uc54a\ub294 \ubd80\ubd84\uc774\ub2e4. \uc74c\uc545 \ub3d9\ud638\ud68c \uc9c0\uc778\uc740 \"\ubd84\uba85 \ub9d0\uc500 \ub4dc\ub9b4 \uc218 \uc788\ub2e4. \uae40\uad11\uc11d \uc528\uac00 \uc6b0\uc6b8\uc99d\uc774 \uc788\uc5c8\uc73c\uba74 \ub0b4\uac00 \ubab0\ub790\uc744 \ub9ac\uac00 \uc5c6\ub2e4\"\uba70 \"\uc801\uc5b4\ub3c4 1\ub144\ub3d9\uc548 \uac00\uc7a5 \uac00\uae4c\uc774\uc11c \uc9c0\ucf1c\ubd24\ub2e4. \ub77c\uc774\ube0c 1,000\ud68c \uacf5\uc5f0 \ud560\ub54c \ub9e4\uc77c \ubd24\uc5c8\uace0 \uadf8\ub3d9\uc548 \ubcd1\uc6d0 \uac00\ub294\uac78 \ubcf8\uc801\uc774 \uc5c6\ub2e4\"\ub77c\uace0 \uc124\uba85\ud588\ub2e4.", "paragraph_answer": "\ucd1d 3\uac00\uc9c0 \uc8fc\uc694 \uc758\ubb38\uc810\uc774 \uc788\uc5c8\ub294\ub370 1.\uae40\uad11\uc11d\uc774 \ubaa9\uc744 \uc2a4\uc2a4\ub85c \ub9f8\ub0d0\ub294\uac83, 2. \uc720\uc11c\uac00 \ubc1c\uacac\ub418\uc9c0 \uc54a\uc740 \uc810, 3. \uc815\ub9d0\ub85c \uc6b0\uc6b8\uc99d\uc774 \uc788\uc5c8\ub0d0\ub294\uac83\uc774\ub2e4. 1\ubc88 \uc758\ubb38\uc758 \uadfc\uac70\ub294 \ubaa9\uc744 \ub9e4\ub2ec \uc804\uae43\uc904\uc744 \ubb36\uae30 \uc704\ud574 \uc4f0\uc600\uc5b4\uc57c \ud588\uc744 \uc758\uc790\uac00 \ubc1c\uacac\ub418\uc9c0 \uc54a\uc558\uace0 \ubaa9\uc744 \ub9e4\ub2ec \uc704\uce58\uc870\ucc28 \uc544\ub2c8\ub77c\ub294 \uac83\uc774\ub2e4. \uacb0\ub860\uc801\uc73c\ub85c \uc790\uc0b4\uc744 \ud560\ub9cc\ud55c \uc7a5\uc18c\uac00 \uc544\ub2c8\uc5c8\ub2e4\ub294 \uc8fc\uc7a5\uc774\ub2e4. \ub610 \uc9c0\uc778\uc740 \" \uacc4\ub2e8 \uc5d0 \ube44\uc2a4\ub4ec\ud788 \uc549\uc544\uc788\ub294 \uac83\ucc98\ub7fc \ud574\uc11c \ubc1c\uacac\ub410\ub2e4. \uc904\uc774 \ub298\uc5b4\ub098\uc57c \uc0ac\ub78c\uc774 \uc11c\uc788\uc744\uac70\ub2e4. \uadfc\ub370 \uc804\uae43\uc904\uc774 \ub298\uc5b4\ub098\ub0d0\"\ub77c\uace0 \uc758\ubb38\uc744 \uc81c\uae30\ud588\ub2e4. \ud558\uc9c0\ub9cc \ub2f9\uc2dc \uace0 \uae40\uad11\uc11d \uc790\ud0dd\uc740 \ud604\uc7ac \ub0b4\ubd80 \uacf5\uc0ac\uac00 \uc774\ub8e8\uc5b4\uc838 \ud655\uc778\ud560 \uc218 \uc5c6\ub294 \uc0c1\ud0dc\uc600\ub2e4. 2\ubc88 \uc758\ubb38\uc740 \uace0 \uae40\uad11\uc11d\uc528\ub294 \uba54\ubaa8\uad11\uc73c\ub85c \uc720\uba85\ud558\uc600\ub294\ub370, \uc544\ubb34\ub7f0 \uc720\uc11c\ub3c4 \uc5c6\uc5c8\uae30 \ub54c\ubb38\uc774\ub2e4. \uace0 \uae40\uad11\uc11d \uccab\uc9f8 \uc790\ud615\uc740 \"(\uae40\uad11\uc11d\uc740) \uba54\ubaa8\uac00 \uc544\uc8fc \ud544\uc218\uc801\uc774\uace0 \uae30\ubcf8\uc801\uc778 \uc2b5\uad00\uc774 \uc788\ub2e4. \ud2c0\ub9bc\uc5c6\uc774 \uc720\uc11c\uac00 \uc788\uc744 \uac83\uc774\ub2e4\"\ub77c\uace0 \ud138\uc5b4\ub1a8\ub2e4. 3\ubc88\uc9f8 \uc774\uc720\uac00 \ub41c \uace0 \uae40\uad11\uc11d\uc528\uc758 \ubd80\uc778 \uc11c\ud574\uc21c\uc528\uac00 \uc8fc\uc7a5\ud55c \uc6b0\uc6b8\uc99d \uc5ed\uc2dc \uc9c0\uc778 \ub9d0\uc5d0 \ub530\ub974\uba74 \uc774\ud574\ub418\uc9c0 \uc54a\ub294 \ubd80\ubd84\uc774\ub2e4. \uc74c\uc545 \ub3d9\ud638\ud68c \uc9c0\uc778\uc740 \"\ubd84\uba85 \ub9d0\uc500 \ub4dc\ub9b4 \uc218 \uc788\ub2e4. \uae40\uad11\uc11d \uc528\uac00 \uc6b0\uc6b8\uc99d\uc774 \uc788\uc5c8\uc73c\uba74 \ub0b4\uac00 \ubab0\ub790\uc744 \ub9ac\uac00 \uc5c6\ub2e4\"\uba70 \"\uc801\uc5b4\ub3c4 1\ub144\ub3d9\uc548 \uac00\uc7a5 \uac00\uae4c\uc774\uc11c \uc9c0\ucf1c\ubd24\ub2e4. \ub77c\uc774\ube0c 1,000\ud68c \uacf5\uc5f0 \ud560\ub54c \ub9e4\uc77c \ubd24\uc5c8\uace0 \uadf8\ub3d9\uc548 \ubcd1\uc6d0 \uac00\ub294\uac78 \ubcf8\uc801\uc774 \uc5c6\ub2e4\"\ub77c\uace0 \uc124\uba85\ud588\ub2e4.", "sentence_answer": "\ub610 \uc9c0\uc778\uc740 \" \uacc4\ub2e8 \uc5d0 \ube44\uc2a4\ub4ec\ud788 \uc549\uc544\uc788\ub294 \uac83\ucc98\ub7fc \ud574\uc11c \ubc1c\uacac\ub410\ub2e4.", "paragraph_id": "6576721-2-2", "paragraph_question": "question: \uae40\uad11\uc11d\uc740 \uc5b4\ub514\uc5d0 \ube44\uc2a4\ub4ec\ud788 \uc549\uc740 \ucc44\ub85c \ubc1c\uacac\ub418\uc5c8\ub294\uac00?, context: \ucd1d 3\uac00\uc9c0 \uc8fc\uc694 \uc758\ubb38\uc810\uc774 \uc788\uc5c8\ub294\ub370 1.\uae40\uad11\uc11d\uc774 \ubaa9\uc744 \uc2a4\uc2a4\ub85c \ub9f8\ub0d0\ub294\uac83, 2. \uc720\uc11c\uac00 \ubc1c\uacac\ub418\uc9c0 \uc54a\uc740 \uc810, 3. \uc815\ub9d0\ub85c \uc6b0\uc6b8\uc99d\uc774 \uc788\uc5c8\ub0d0\ub294\uac83\uc774\ub2e4. 1\ubc88 \uc758\ubb38\uc758 \uadfc\uac70\ub294 \ubaa9\uc744 \ub9e4\ub2ec \uc804\uae43\uc904\uc744 \ubb36\uae30 \uc704\ud574 \uc4f0\uc600\uc5b4\uc57c \ud588\uc744 \uc758\uc790\uac00 \ubc1c\uacac\ub418\uc9c0 \uc54a\uc558\uace0 \ubaa9\uc744 \ub9e4\ub2ec \uc704\uce58\uc870\ucc28 \uc544\ub2c8\ub77c\ub294 \uac83\uc774\ub2e4. \uacb0\ub860\uc801\uc73c\ub85c \uc790\uc0b4\uc744 \ud560\ub9cc\ud55c \uc7a5\uc18c\uac00 \uc544\ub2c8\uc5c8\ub2e4\ub294 \uc8fc\uc7a5\uc774\ub2e4. \ub610 \uc9c0\uc778\uc740 \"\uacc4\ub2e8\uc5d0 \ube44\uc2a4\ub4ec\ud788 \uc549\uc544\uc788\ub294 \uac83\ucc98\ub7fc \ud574\uc11c \ubc1c\uacac\ub410\ub2e4. \uc904\uc774 \ub298\uc5b4\ub098\uc57c \uc0ac\ub78c\uc774 \uc11c\uc788\uc744\uac70\ub2e4. \uadfc\ub370 \uc804\uae43\uc904\uc774 \ub298\uc5b4\ub098\ub0d0\"\ub77c\uace0 \uc758\ubb38\uc744 \uc81c\uae30\ud588\ub2e4. \ud558\uc9c0\ub9cc \ub2f9\uc2dc \uace0 \uae40\uad11\uc11d \uc790\ud0dd\uc740 \ud604\uc7ac \ub0b4\ubd80 \uacf5\uc0ac\uac00 \uc774\ub8e8\uc5b4\uc838 \ud655\uc778\ud560 \uc218 \uc5c6\ub294 \uc0c1\ud0dc\uc600\ub2e4. 2\ubc88 \uc758\ubb38\uc740 \uace0 \uae40\uad11\uc11d\uc528\ub294 \uba54\ubaa8\uad11\uc73c\ub85c \uc720\uba85\ud558\uc600\ub294\ub370, \uc544\ubb34\ub7f0 \uc720\uc11c\ub3c4 \uc5c6\uc5c8\uae30 \ub54c\ubb38\uc774\ub2e4. \uace0 \uae40\uad11\uc11d \uccab\uc9f8 \uc790\ud615\uc740 \"(\uae40\uad11\uc11d\uc740) \uba54\ubaa8\uac00 \uc544\uc8fc \ud544\uc218\uc801\uc774\uace0 \uae30\ubcf8\uc801\uc778 \uc2b5\uad00\uc774 \uc788\ub2e4. \ud2c0\ub9bc\uc5c6\uc774 \uc720\uc11c\uac00 \uc788\uc744 \uac83\uc774\ub2e4\"\ub77c\uace0 \ud138\uc5b4\ub1a8\ub2e4. 3\ubc88\uc9f8 \uc774\uc720\uac00 \ub41c \uace0 \uae40\uad11\uc11d\uc528\uc758 \ubd80\uc778 \uc11c\ud574\uc21c\uc528\uac00 \uc8fc\uc7a5\ud55c \uc6b0\uc6b8\uc99d \uc5ed\uc2dc \uc9c0\uc778 \ub9d0\uc5d0 \ub530\ub974\uba74 \uc774\ud574\ub418\uc9c0 \uc54a\ub294 \ubd80\ubd84\uc774\ub2e4. \uc74c\uc545 \ub3d9\ud638\ud68c \uc9c0\uc778\uc740 \"\ubd84\uba85 \ub9d0\uc500 \ub4dc\ub9b4 \uc218 \uc788\ub2e4. \uae40\uad11\uc11d \uc528\uac00 \uc6b0\uc6b8\uc99d\uc774 \uc788\uc5c8\uc73c\uba74 \ub0b4\uac00 \ubab0\ub790\uc744 \ub9ac\uac00 \uc5c6\ub2e4\"\uba70 \"\uc801\uc5b4\ub3c4 1\ub144\ub3d9\uc548 \uac00\uc7a5 \uac00\uae4c\uc774\uc11c \uc9c0\ucf1c\ubd24\ub2e4. \ub77c\uc774\ube0c 1,000\ud68c \uacf5\uc5f0 \ud560\ub54c \ub9e4\uc77c \ubd24\uc5c8\uace0 \uadf8\ub3d9\uc548 \ubcd1\uc6d0 \uac00\ub294\uac78 \ubcf8\uc801\uc774 \uc5c6\ub2e4\"\ub77c\uace0 \uc124\uba85\ud588\ub2e4."} {"question": "\uac1c\uc2e0\uad50 \uc81c\ud6c4\ub3d9\ub9f9\uc744 \uc774\ub04c\uace0 \uc788\ub358 \uc0ac\ub78c\uc740?", "paragraph": "\ubcf4\ud5e4\ubbf8\uc544 \ubd84\uc7c1\uc740 \uc5ec\uc804\ud788 \uc9c0\uc5ed\uc801\uc778 \ubd84\ub780\uc73c\ub85c \ub0a8\uc544\uc788\uc5c8\ub2e4. \ud558\uc9c0\ub9cc \ub9c8\ud2f0\uc544\uc2a4 \ud669\uc81c\uc758 \uc8fd\uc74c \uc774\ud6c4 \uac1c\uc2e0\uad50 \ubc18\ub780 \uc9c0\ub3c4\uc790\ub4e4\uc740 \ub300\ub2f4\ud574\uc84c\ub2e4. \ub9c8\ud2f0\uc544\uc2a4 \ud669\uc81c\uc758 \uc8fd\uc74c\uc73c\ub85c \uc778\ud574 \uacf5\uc2dd\uc801\uc778 \ud669\uc81c\uac00 \ub41c \ud398\ub974\ub514\ub09c\ud2b8 2\uc138\uc640 \ubcf4\ud5e4\ubbf8\uc544 \ubc18\ub780\uad70\uc774 \ub450\ub824\uc6cc\ud558\ub294 \uac83\uc740 \ubd84\uc7c1\uc774 \uc11c\ubd80 \ub3c5\uc77c\ub85c \uc774\ub3d9\ud558\ub294 \uac83\uc774\uc5c8\ub2e4. \ud398\ub974\ub514\ub09c\ud2b8\ub294 \uc774 \ubb38\uc81c\ub97c \ud574\uacb0\ud558\uae30 \uc704\ud574 \uac19\uc740 \ud569\uc2a4\ubd80\ub974\ud06c \uac00\ubb38\uc778 \uadf8\uc758 \uc870\uce74 \ud544\ub9ac\ud504 4\uc138\uc5d0\uac8c \ub3c4\uc6c0\uc744 \uc694\uccad\ud588\ub2e4. \ud669\uc81c\uc5d0 \ub9de\uc120 \ub3d9\ub9f9\uc5d0 \ub300\ud574 \uacb0\uc0ac\uc801\uc774\uc5c8\ub358 \ubcf4\ud5e4\ubbf8\uc544 \uc778\ub4e4\uc740 \uce7c\ubc45\ud30c\uc600\ub358 \ud314\uce20 \uc120\uc81c\ud6c4\uc600\ub358 \uce7c\ubc45\uad50\ub3c4 \ud504\ub9ac\ub4dc\ub9ac\ud788 5\uc138\ub97c \ubcf4\ud5e4\ubbf8\uc544 \uc655\uc758 \ud6c4\ubcf4\ub85c \ub0b4\uc138\uc6b0\uace0 \uc788\uc5c8\uace0 \ud504\ub9ac\ub4dc\ub9ac\ud788 5\uc138\ub294 \uac1c\uc2e0\uad50 \uc81c\ud6c4\ub3d9\ub9f9\uc744 \uc774\ub04c\uace0 \uc788\uc5c8\ub2e4. \ubcf4\ud5e4\ubbf8\uc544\ub294 \uc81c\ud6c4\ub3d9\ub9f9\uc5d0 \uc790\uad6d\uc744 \uac00\uc785\uc2dc\ucf1c\uc900\ub2e4\uba74 \ud504\ub9ac\ub4dc\ub9ac\ud788 5\uc138\uc5d0\uac8c \ubcf4\ud5e4\ubbf8\uc544 \uc655\uc704\ub97c \uc904 \uac83\uc774\ub77c\uace0 \uc57d\uc18d\ud588\ub2e4. \ud558\uc9c0\ub9cc \uc0ac\ubcf4\uc774 \uacf5\uc791 \uce74\ub97c\ub85c \uc5d0\ub9c8\ub204\uc5d8\ub808 1\uc138, \uc791\uc13c \uc120\uc81c\ud6c4\uad6d\uc758 \uc694\ud55c \uac8c\uc624\ub974\ud06c 1\uc138, \ud5dd\uac00\ub9ac \uc655 \ubca0\ud2c0\ub80c \uac00\ubcf4\ub974\uc5d0\uac8c\ub3c4 \uac19\uc740 \uc57d\uc18d\uc774 \uc8fc\uc5b4\uc84c\ub2e4. \ud504\ub77c\ud558\ub97c \ub5a0\ub098\ub294 \ubaa8\ub4e0 \uc11c\uc2e0\ub4e4\uc744 \uac00\ub85c\ucc4c \uc218 \uc788\uc5c8\ub358 \uac83\uc73c\ub85c \ubcf4\uc774\ub294 \uc624\uc2a4\ud2b8\ub9ac\uc544 \uc778\uc740 \uc774\ub7ec\ud55c \uc774\uc911\uc801 \ud0dc\ub3c4\ub97c \uacf5\uc5f0\ud788 \uc54c\ub838\ub2e4. \uc774\ub294 \ubcf4\ud5e4\ubbf8\uc544\uc778\ub4e4\uc5d0 \ub300\ud55c \uc9c0\uc9c0\uac00 \ucd94\ub77d\ud558\ub294 \uc6d0\uc778\uc774 \ub418\uc5c8\uace0, \uc791\uc13c\uc740 \uc544\uc608 \uc774\ub4e4\uc5d0\uac8c \uc2e0\uc784\uc744 \uc8fc\uc9c0 \uc54a\uc558\ub2e4.", "answer": "\ud504\ub9ac\ub4dc\ub9ac\ud788 5\uc138", "sentence": "\ud669\uc81c\uc5d0 \ub9de\uc120 \ub3d9\ub9f9\uc5d0 \ub300\ud574 \uacb0\uc0ac\uc801\uc774\uc5c8\ub358 \ubcf4\ud5e4\ubbf8\uc544 \uc778\ub4e4\uc740 \uce7c\ubc45\ud30c\uc600\ub358 \ud314\uce20 \uc120\uc81c\ud6c4\uc600\ub358 \uce7c\ubc45\uad50\ub3c4 \ud504\ub9ac\ub4dc\ub9ac\ud788 5\uc138 \ub97c \ubcf4\ud5e4\ubbf8\uc544 \uc655\uc758 \ud6c4\ubcf4\ub85c \ub0b4\uc138\uc6b0\uace0 \uc788\uc5c8\uace0 \ud504\ub9ac\ub4dc\ub9ac\ud788 5\uc138\ub294 \uac1c\uc2e0\uad50 \uc81c\ud6c4\ub3d9\ub9f9\uc744 \uc774\ub04c\uace0 \uc788\uc5c8\ub2e4.", "paragraph_sentence": "\ubcf4\ud5e4\ubbf8\uc544 \ubd84\uc7c1\uc740 \uc5ec\uc804\ud788 \uc9c0\uc5ed\uc801\uc778 \ubd84\ub780\uc73c\ub85c \ub0a8\uc544\uc788\uc5c8\ub2e4. \ud558\uc9c0\ub9cc \ub9c8\ud2f0\uc544\uc2a4 \ud669\uc81c\uc758 \uc8fd\uc74c \uc774\ud6c4 \uac1c\uc2e0\uad50 \ubc18\ub780 \uc9c0\ub3c4\uc790\ub4e4\uc740 \ub300\ub2f4\ud574\uc84c\ub2e4. \ub9c8\ud2f0\uc544\uc2a4 \ud669\uc81c\uc758 \uc8fd\uc74c\uc73c\ub85c \uc778\ud574 \uacf5\uc2dd\uc801\uc778 \ud669\uc81c\uac00 \ub41c \ud398\ub974\ub514\ub09c\ud2b8 2\uc138\uc640 \ubcf4\ud5e4\ubbf8\uc544 \ubc18\ub780\uad70\uc774 \ub450\ub824\uc6cc\ud558\ub294 \uac83\uc740 \ubd84\uc7c1\uc774 \uc11c\ubd80 \ub3c5\uc77c\ub85c \uc774\ub3d9\ud558\ub294 \uac83\uc774\uc5c8\ub2e4. \ud398\ub974\ub514\ub09c\ud2b8\ub294 \uc774 \ubb38\uc81c\ub97c \ud574\uacb0\ud558\uae30 \uc704\ud574 \uac19\uc740 \ud569\uc2a4\ubd80\ub974\ud06c \uac00\ubb38\uc778 \uadf8\uc758 \uc870\uce74 \ud544\ub9ac\ud504 4\uc138\uc5d0\uac8c \ub3c4\uc6c0\uc744 \uc694\uccad\ud588\ub2e4. \ud669\uc81c\uc5d0 \ub9de\uc120 \ub3d9\ub9f9\uc5d0 \ub300\ud574 \uacb0\uc0ac\uc801\uc774\uc5c8\ub358 \ubcf4\ud5e4\ubbf8\uc544 \uc778\ub4e4\uc740 \uce7c\ubc45\ud30c\uc600\ub358 \ud314\uce20 \uc120\uc81c\ud6c4\uc600\ub358 \uce7c\ubc45\uad50\ub3c4 \ud504\ub9ac\ub4dc\ub9ac\ud788 5\uc138 \ub97c \ubcf4\ud5e4\ubbf8\uc544 \uc655\uc758 \ud6c4\ubcf4\ub85c \ub0b4\uc138\uc6b0\uace0 \uc788\uc5c8\uace0 \ud504\ub9ac\ub4dc\ub9ac\ud788 5\uc138\ub294 \uac1c\uc2e0\uad50 \uc81c\ud6c4\ub3d9\ub9f9\uc744 \uc774\ub04c\uace0 \uc788\uc5c8\ub2e4. \ubcf4\ud5e4\ubbf8\uc544\ub294 \uc81c\ud6c4\ub3d9\ub9f9\uc5d0 \uc790\uad6d\uc744 \uac00\uc785\uc2dc\ucf1c\uc900\ub2e4\uba74 \ud504\ub9ac\ub4dc\ub9ac\ud788 5\uc138\uc5d0\uac8c \ubcf4\ud5e4\ubbf8\uc544 \uc655\uc704\ub97c \uc904 \uac83\uc774\ub77c\uace0 \uc57d\uc18d\ud588\ub2e4. \ud558\uc9c0\ub9cc \uc0ac\ubcf4\uc774 \uacf5\uc791 \uce74\ub97c\ub85c \uc5d0\ub9c8\ub204\uc5d8\ub808 1\uc138, \uc791\uc13c \uc120\uc81c\ud6c4\uad6d\uc758 \uc694\ud55c \uac8c\uc624\ub974\ud06c 1\uc138, \ud5dd\uac00\ub9ac \uc655 \ubca0\ud2c0\ub80c \uac00\ubcf4\ub974\uc5d0\uac8c\ub3c4 \uac19\uc740 \uc57d\uc18d\uc774 \uc8fc\uc5b4\uc84c\ub2e4. \ud504\ub77c\ud558\ub97c \ub5a0\ub098\ub294 \ubaa8\ub4e0 \uc11c\uc2e0\ub4e4\uc744 \uac00\ub85c\ucc4c \uc218 \uc788\uc5c8\ub358 \uac83\uc73c\ub85c \ubcf4\uc774\ub294 \uc624\uc2a4\ud2b8\ub9ac\uc544 \uc778\uc740 \uc774\ub7ec\ud55c \uc774\uc911\uc801 \ud0dc\ub3c4\ub97c \uacf5\uc5f0\ud788 \uc54c\ub838\ub2e4. \uc774\ub294 \ubcf4\ud5e4\ubbf8\uc544\uc778\ub4e4\uc5d0 \ub300\ud55c \uc9c0\uc9c0\uac00 \ucd94\ub77d\ud558\ub294 \uc6d0\uc778\uc774 \ub418\uc5c8\uace0, \uc791\uc13c\uc740 \uc544\uc608 \uc774\ub4e4\uc5d0\uac8c \uc2e0\uc784\uc744 \uc8fc\uc9c0 \uc54a\uc558\ub2e4.", "paragraph_answer": "\ubcf4\ud5e4\ubbf8\uc544 \ubd84\uc7c1\uc740 \uc5ec\uc804\ud788 \uc9c0\uc5ed\uc801\uc778 \ubd84\ub780\uc73c\ub85c \ub0a8\uc544\uc788\uc5c8\ub2e4. \ud558\uc9c0\ub9cc \ub9c8\ud2f0\uc544\uc2a4 \ud669\uc81c\uc758 \uc8fd\uc74c \uc774\ud6c4 \uac1c\uc2e0\uad50 \ubc18\ub780 \uc9c0\ub3c4\uc790\ub4e4\uc740 \ub300\ub2f4\ud574\uc84c\ub2e4. \ub9c8\ud2f0\uc544\uc2a4 \ud669\uc81c\uc758 \uc8fd\uc74c\uc73c\ub85c \uc778\ud574 \uacf5\uc2dd\uc801\uc778 \ud669\uc81c\uac00 \ub41c \ud398\ub974\ub514\ub09c\ud2b8 2\uc138\uc640 \ubcf4\ud5e4\ubbf8\uc544 \ubc18\ub780\uad70\uc774 \ub450\ub824\uc6cc\ud558\ub294 \uac83\uc740 \ubd84\uc7c1\uc774 \uc11c\ubd80 \ub3c5\uc77c\ub85c \uc774\ub3d9\ud558\ub294 \uac83\uc774\uc5c8\ub2e4. \ud398\ub974\ub514\ub09c\ud2b8\ub294 \uc774 \ubb38\uc81c\ub97c \ud574\uacb0\ud558\uae30 \uc704\ud574 \uac19\uc740 \ud569\uc2a4\ubd80\ub974\ud06c \uac00\ubb38\uc778 \uadf8\uc758 \uc870\uce74 \ud544\ub9ac\ud504 4\uc138\uc5d0\uac8c \ub3c4\uc6c0\uc744 \uc694\uccad\ud588\ub2e4. \ud669\uc81c\uc5d0 \ub9de\uc120 \ub3d9\ub9f9\uc5d0 \ub300\ud574 \uacb0\uc0ac\uc801\uc774\uc5c8\ub358 \ubcf4\ud5e4\ubbf8\uc544 \uc778\ub4e4\uc740 \uce7c\ubc45\ud30c\uc600\ub358 \ud314\uce20 \uc120\uc81c\ud6c4\uc600\ub358 \uce7c\ubc45\uad50\ub3c4 \ud504\ub9ac\ub4dc\ub9ac\ud788 5\uc138 \ub97c \ubcf4\ud5e4\ubbf8\uc544 \uc655\uc758 \ud6c4\ubcf4\ub85c \ub0b4\uc138\uc6b0\uace0 \uc788\uc5c8\uace0 \ud504\ub9ac\ub4dc\ub9ac\ud788 5\uc138\ub294 \uac1c\uc2e0\uad50 \uc81c\ud6c4\ub3d9\ub9f9\uc744 \uc774\ub04c\uace0 \uc788\uc5c8\ub2e4. \ubcf4\ud5e4\ubbf8\uc544\ub294 \uc81c\ud6c4\ub3d9\ub9f9\uc5d0 \uc790\uad6d\uc744 \uac00\uc785\uc2dc\ucf1c\uc900\ub2e4\uba74 \ud504\ub9ac\ub4dc\ub9ac\ud788 5\uc138\uc5d0\uac8c \ubcf4\ud5e4\ubbf8\uc544 \uc655\uc704\ub97c \uc904 \uac83\uc774\ub77c\uace0 \uc57d\uc18d\ud588\ub2e4. \ud558\uc9c0\ub9cc \uc0ac\ubcf4\uc774 \uacf5\uc791 \uce74\ub97c\ub85c \uc5d0\ub9c8\ub204\uc5d8\ub808 1\uc138, \uc791\uc13c \uc120\uc81c\ud6c4\uad6d\uc758 \uc694\ud55c \uac8c\uc624\ub974\ud06c 1\uc138, \ud5dd\uac00\ub9ac \uc655 \ubca0\ud2c0\ub80c \uac00\ubcf4\ub974\uc5d0\uac8c\ub3c4 \uac19\uc740 \uc57d\uc18d\uc774 \uc8fc\uc5b4\uc84c\ub2e4. \ud504\ub77c\ud558\ub97c \ub5a0\ub098\ub294 \ubaa8\ub4e0 \uc11c\uc2e0\ub4e4\uc744 \uac00\ub85c\ucc4c \uc218 \uc788\uc5c8\ub358 \uac83\uc73c\ub85c \ubcf4\uc774\ub294 \uc624\uc2a4\ud2b8\ub9ac\uc544 \uc778\uc740 \uc774\ub7ec\ud55c \uc774\uc911\uc801 \ud0dc\ub3c4\ub97c \uacf5\uc5f0\ud788 \uc54c\ub838\ub2e4. \uc774\ub294 \ubcf4\ud5e4\ubbf8\uc544\uc778\ub4e4\uc5d0 \ub300\ud55c \uc9c0\uc9c0\uac00 \ucd94\ub77d\ud558\ub294 \uc6d0\uc778\uc774 \ub418\uc5c8\uace0, \uc791\uc13c\uc740 \uc544\uc608 \uc774\ub4e4\uc5d0\uac8c \uc2e0\uc784\uc744 \uc8fc\uc9c0 \uc54a\uc558\ub2e4.", "sentence_answer": "\ud669\uc81c\uc5d0 \ub9de\uc120 \ub3d9\ub9f9\uc5d0 \ub300\ud574 \uacb0\uc0ac\uc801\uc774\uc5c8\ub358 \ubcf4\ud5e4\ubbf8\uc544 \uc778\ub4e4\uc740 \uce7c\ubc45\ud30c\uc600\ub358 \ud314\uce20 \uc120\uc81c\ud6c4\uc600\ub358 \uce7c\ubc45\uad50\ub3c4 \ud504\ub9ac\ub4dc\ub9ac\ud788 5\uc138 \ub97c \ubcf4\ud5e4\ubbf8\uc544 \uc655\uc758 \ud6c4\ubcf4\ub85c \ub0b4\uc138\uc6b0\uace0 \uc788\uc5c8\uace0 \ud504\ub9ac\ub4dc\ub9ac\ud788 5\uc138\ub294 \uac1c\uc2e0\uad50 \uc81c\ud6c4\ub3d9\ub9f9\uc744 \uc774\ub04c\uace0 \uc788\uc5c8\ub2e4.", "paragraph_id": "6499337-11-2", "paragraph_question": "question: \uac1c\uc2e0\uad50 \uc81c\ud6c4\ub3d9\ub9f9\uc744 \uc774\ub04c\uace0 \uc788\ub358 \uc0ac\ub78c\uc740?, context: \ubcf4\ud5e4\ubbf8\uc544 \ubd84\uc7c1\uc740 \uc5ec\uc804\ud788 \uc9c0\uc5ed\uc801\uc778 \ubd84\ub780\uc73c\ub85c \ub0a8\uc544\uc788\uc5c8\ub2e4. \ud558\uc9c0\ub9cc \ub9c8\ud2f0\uc544\uc2a4 \ud669\uc81c\uc758 \uc8fd\uc74c \uc774\ud6c4 \uac1c\uc2e0\uad50 \ubc18\ub780 \uc9c0\ub3c4\uc790\ub4e4\uc740 \ub300\ub2f4\ud574\uc84c\ub2e4. \ub9c8\ud2f0\uc544\uc2a4 \ud669\uc81c\uc758 \uc8fd\uc74c\uc73c\ub85c \uc778\ud574 \uacf5\uc2dd\uc801\uc778 \ud669\uc81c\uac00 \ub41c \ud398\ub974\ub514\ub09c\ud2b8 2\uc138\uc640 \ubcf4\ud5e4\ubbf8\uc544 \ubc18\ub780\uad70\uc774 \ub450\ub824\uc6cc\ud558\ub294 \uac83\uc740 \ubd84\uc7c1\uc774 \uc11c\ubd80 \ub3c5\uc77c\ub85c \uc774\ub3d9\ud558\ub294 \uac83\uc774\uc5c8\ub2e4. \ud398\ub974\ub514\ub09c\ud2b8\ub294 \uc774 \ubb38\uc81c\ub97c \ud574\uacb0\ud558\uae30 \uc704\ud574 \uac19\uc740 \ud569\uc2a4\ubd80\ub974\ud06c \uac00\ubb38\uc778 \uadf8\uc758 \uc870\uce74 \ud544\ub9ac\ud504 4\uc138\uc5d0\uac8c \ub3c4\uc6c0\uc744 \uc694\uccad\ud588\ub2e4. \ud669\uc81c\uc5d0 \ub9de\uc120 \ub3d9\ub9f9\uc5d0 \ub300\ud574 \uacb0\uc0ac\uc801\uc774\uc5c8\ub358 \ubcf4\ud5e4\ubbf8\uc544 \uc778\ub4e4\uc740 \uce7c\ubc45\ud30c\uc600\ub358 \ud314\uce20 \uc120\uc81c\ud6c4\uc600\ub358 \uce7c\ubc45\uad50\ub3c4 \ud504\ub9ac\ub4dc\ub9ac\ud788 5\uc138\ub97c \ubcf4\ud5e4\ubbf8\uc544 \uc655\uc758 \ud6c4\ubcf4\ub85c \ub0b4\uc138\uc6b0\uace0 \uc788\uc5c8\uace0 \ud504\ub9ac\ub4dc\ub9ac\ud788 5\uc138\ub294 \uac1c\uc2e0\uad50 \uc81c\ud6c4\ub3d9\ub9f9\uc744 \uc774\ub04c\uace0 \uc788\uc5c8\ub2e4. \ubcf4\ud5e4\ubbf8\uc544\ub294 \uc81c\ud6c4\ub3d9\ub9f9\uc5d0 \uc790\uad6d\uc744 \uac00\uc785\uc2dc\ucf1c\uc900\ub2e4\uba74 \ud504\ub9ac\ub4dc\ub9ac\ud788 5\uc138\uc5d0\uac8c \ubcf4\ud5e4\ubbf8\uc544 \uc655\uc704\ub97c \uc904 \uac83\uc774\ub77c\uace0 \uc57d\uc18d\ud588\ub2e4. \ud558\uc9c0\ub9cc \uc0ac\ubcf4\uc774 \uacf5\uc791 \uce74\ub97c\ub85c \uc5d0\ub9c8\ub204\uc5d8\ub808 1\uc138, \uc791\uc13c \uc120\uc81c\ud6c4\uad6d\uc758 \uc694\ud55c \uac8c\uc624\ub974\ud06c 1\uc138, \ud5dd\uac00\ub9ac \uc655 \ubca0\ud2c0\ub80c \uac00\ubcf4\ub974\uc5d0\uac8c\ub3c4 \uac19\uc740 \uc57d\uc18d\uc774 \uc8fc\uc5b4\uc84c\ub2e4. \ud504\ub77c\ud558\ub97c \ub5a0\ub098\ub294 \ubaa8\ub4e0 \uc11c\uc2e0\ub4e4\uc744 \uac00\ub85c\ucc4c \uc218 \uc788\uc5c8\ub358 \uac83\uc73c\ub85c \ubcf4\uc774\ub294 \uc624\uc2a4\ud2b8\ub9ac\uc544 \uc778\uc740 \uc774\ub7ec\ud55c \uc774\uc911\uc801 \ud0dc\ub3c4\ub97c \uacf5\uc5f0\ud788 \uc54c\ub838\ub2e4. \uc774\ub294 \ubcf4\ud5e4\ubbf8\uc544\uc778\ub4e4\uc5d0 \ub300\ud55c \uc9c0\uc9c0\uac00 \ucd94\ub77d\ud558\ub294 \uc6d0\uc778\uc774 \ub418\uc5c8\uace0, \uc791\uc13c\uc740 \uc544\uc608 \uc774\ub4e4\uc5d0\uac8c \uc2e0\uc784\uc744 \uc8fc\uc9c0 \uc54a\uc558\ub2e4."} {"question": "\ud55c \uace0\uc870\uc758 \uc190\uc790\ub85c \uc81c\ub098\ub77c\uc758 \uc655\uc774\uc5c8\ub358 \uc778\ubb3c\uc740?", "paragraph": "\uace0\uc870 \uc0ac\ud6c4, \ud61c\uc81c\uac00 \uc989\uc704\ud558\uace0 \uace0\ud669\ud6c4\uac00 \uc2e4\uad8c\uc744 \uc7a1\uc544 \ub098\ub77c\ub97c \ub2e4\uc2a4\ub838\ub2e4. \uad00\uc601\uc740 \uc5f4\ud6c4\ub85c\uc11c \uace0\ud669\ud6c4\ub97c \ubcf4\uc88c\ud588\ub2e4. \ud0dc\ud6c4\uac00 \uc8fd\uc790, \uc5ec\ub85d \ub4f1\uc758 \uc5ec\uc528 \uc77c\uc871\uc774 \uc7a5\uc548\uc758 \uad70\uc0ac\ub97c \uc7a5\uc545\ud558\uace0 \uc788\uc5c8\uace0, \uace0\uc870\uc758 \uc190\uc790, \uc720\ube44\uc758 \uc544\ub4e4\ub85c \uc81c\ub098\ub77c\uc758 \uc655\uc744 \uc9c0\ub0b4\ub294 \uc720\uc591\uc740 \ubd88\ub9cc\uc744 \ud488\uace0 \ubc18\ub780\uc744 \uc77c\uc73c\ucf1c \uc5ec\uc528 \uc138\ub825\uc744 \uc4f8\uc5b4\ubc84\ub9ac\uace0\uc790 \ud588\ub2e4. \uc5ec\uc528 \uc77c\uc871\uc740 \uad00\uc601\uc5d0\uac8c \uad70\uc0ac\ub97c \ub9e1\uaca8 \uc81c\ub098\ub77c\uc758 \ubc18\ub780\uc744 \uc9c4\uc555\ud558\ub3c4\ub85d \ud588\uc73c\ub098, \uad00\uc601\uc740 \ud0dc\uc704 \uc8fc\ubc1c, \uc2b9\uc0c1 \uc9c4\ud3c9 \ub4f1\uacfc \ubaa8\uc758\ud574 \uc5ec\uc528\ub97c \uce58\ub3c4\ub85d \uc791\uc815\ud588\ub2e4. \uadf8\ub7ec\uae30\uc5d0 \uad00\uc601\uc740 \ud615\uc591\uc5d0 \uc8fc\ub454\ud558\uba74\uc11c \uc18c\ubb38\uc744 \ud37c\ub728\ub824 \uc81c\ub098\ub77c \uad70\uc0ac\uac00 \uc9c4\uaca9\uc744 \uba48\ucd94\uac8c \ud558\uace0, \uc8fc\ubc1c \ub4f1\uc774 \uc774 \ud2c8\uc5d0 \uc5ec\uc528\ub4e4\uc744 \ub9d0\uac1b\uac8c \uc4f8\uc5c8\ub2e4. \uc720\uc591\uacfc \uad00\uc601 \ubaa8\ub450 \uad70\uc0ac\ub97c \ud574\uc0b0\ud574 \ub3cc\uc544\uac14\uace0, \uad00\uc601\uc740 \uc8fc\ubc1c, \uc9c4\ud3c9\uacfc \ud568\uaed8 \uace0\uc870\uc758 \uc544\ub4e4 \ub300\uc655\uc744 \ud669\uc81c\ub85c \uc138\uc6b0\ub2c8, \uc774\uac00 \uace7 \ubb38\uc81c\ub2e4.", "answer": "\uc720\uc591", "sentence": "\ud0dc\ud6c4\uac00 \uc8fd\uc790, \uc5ec\ub85d \ub4f1\uc758 \uc5ec\uc528 \uc77c\uc871\uc774 \uc7a5\uc548\uc758 \uad70\uc0ac\ub97c \uc7a5\uc545\ud558\uace0 \uc788\uc5c8\uace0, \uace0\uc870\uc758 \uc190\uc790, \uc720\ube44\uc758 \uc544\ub4e4\ub85c \uc81c\ub098\ub77c\uc758 \uc655\uc744 \uc9c0\ub0b4\ub294 \uc720\uc591 \uc740 \ubd88\ub9cc\uc744 \ud488\uace0 \ubc18\ub780\uc744 \uc77c\uc73c\ucf1c \uc5ec\uc528 \uc138\ub825\uc744 \uc4f8\uc5b4\ubc84\ub9ac\uace0\uc790 \ud588\ub2e4.", "paragraph_sentence": "\uace0\uc870 \uc0ac\ud6c4, \ud61c\uc81c\uac00 \uc989\uc704\ud558\uace0 \uace0\ud669\ud6c4\uac00 \uc2e4\uad8c\uc744 \uc7a1\uc544 \ub098\ub77c\ub97c \ub2e4\uc2a4\ub838\ub2e4. \uad00\uc601\uc740 \uc5f4\ud6c4\ub85c\uc11c \uace0\ud669\ud6c4\ub97c \ubcf4\uc88c\ud588\ub2e4. \ud0dc\ud6c4\uac00 \uc8fd\uc790, \uc5ec\ub85d \ub4f1\uc758 \uc5ec\uc528 \uc77c\uc871\uc774 \uc7a5\uc548\uc758 \uad70\uc0ac\ub97c \uc7a5\uc545\ud558\uace0 \uc788\uc5c8\uace0, \uace0\uc870\uc758 \uc190\uc790, \uc720\ube44\uc758 \uc544\ub4e4\ub85c \uc81c\ub098\ub77c\uc758 \uc655\uc744 \uc9c0\ub0b4\ub294 \uc720\uc591 \uc740 \ubd88\ub9cc\uc744 \ud488\uace0 \ubc18\ub780\uc744 \uc77c\uc73c\ucf1c \uc5ec\uc528 \uc138\ub825\uc744 \uc4f8\uc5b4\ubc84\ub9ac\uace0\uc790 \ud588\ub2e4. \uc5ec\uc528 \uc77c\uc871\uc740 \uad00\uc601\uc5d0\uac8c \uad70\uc0ac\ub97c \ub9e1\uaca8 \uc81c\ub098\ub77c\uc758 \ubc18\ub780\uc744 \uc9c4\uc555\ud558\ub3c4\ub85d \ud588\uc73c\ub098, \uad00\uc601\uc740 \ud0dc\uc704 \uc8fc\ubc1c, \uc2b9\uc0c1 \uc9c4\ud3c9 \ub4f1\uacfc \ubaa8\uc758\ud574 \uc5ec\uc528\ub97c \uce58\ub3c4\ub85d \uc791\uc815\ud588\ub2e4. \uadf8\ub7ec\uae30\uc5d0 \uad00\uc601\uc740 \ud615\uc591\uc5d0 \uc8fc\ub454\ud558\uba74\uc11c \uc18c\ubb38\uc744 \ud37c\ub728\ub824 \uc81c\ub098\ub77c \uad70\uc0ac\uac00 \uc9c4\uaca9\uc744 \uba48\ucd94\uac8c \ud558\uace0, \uc8fc\ubc1c \ub4f1\uc774 \uc774 \ud2c8\uc5d0 \uc5ec\uc528\ub4e4\uc744 \ub9d0\uac1b\uac8c \uc4f8\uc5c8\ub2e4. \uc720\uc591\uacfc \uad00\uc601 \ubaa8\ub450 \uad70\uc0ac\ub97c \ud574\uc0b0\ud574 \ub3cc\uc544\uac14\uace0, \uad00\uc601\uc740 \uc8fc\ubc1c, \uc9c4\ud3c9\uacfc \ud568\uaed8 \uace0\uc870\uc758 \uc544\ub4e4 \ub300\uc655\uc744 \ud669\uc81c\ub85c \uc138\uc6b0\ub2c8, \uc774\uac00 \uace7 \ubb38\uc81c\ub2e4.", "paragraph_answer": "\uace0\uc870 \uc0ac\ud6c4, \ud61c\uc81c\uac00 \uc989\uc704\ud558\uace0 \uace0\ud669\ud6c4\uac00 \uc2e4\uad8c\uc744 \uc7a1\uc544 \ub098\ub77c\ub97c \ub2e4\uc2a4\ub838\ub2e4. \uad00\uc601\uc740 \uc5f4\ud6c4\ub85c\uc11c \uace0\ud669\ud6c4\ub97c \ubcf4\uc88c\ud588\ub2e4. \ud0dc\ud6c4\uac00 \uc8fd\uc790, \uc5ec\ub85d \ub4f1\uc758 \uc5ec\uc528 \uc77c\uc871\uc774 \uc7a5\uc548\uc758 \uad70\uc0ac\ub97c \uc7a5\uc545\ud558\uace0 \uc788\uc5c8\uace0, \uace0\uc870\uc758 \uc190\uc790, \uc720\ube44\uc758 \uc544\ub4e4\ub85c \uc81c\ub098\ub77c\uc758 \uc655\uc744 \uc9c0\ub0b4\ub294 \uc720\uc591 \uc740 \ubd88\ub9cc\uc744 \ud488\uace0 \ubc18\ub780\uc744 \uc77c\uc73c\ucf1c \uc5ec\uc528 \uc138\ub825\uc744 \uc4f8\uc5b4\ubc84\ub9ac\uace0\uc790 \ud588\ub2e4. \uc5ec\uc528 \uc77c\uc871\uc740 \uad00\uc601\uc5d0\uac8c \uad70\uc0ac\ub97c \ub9e1\uaca8 \uc81c\ub098\ub77c\uc758 \ubc18\ub780\uc744 \uc9c4\uc555\ud558\ub3c4\ub85d \ud588\uc73c\ub098, \uad00\uc601\uc740 \ud0dc\uc704 \uc8fc\ubc1c, \uc2b9\uc0c1 \uc9c4\ud3c9 \ub4f1\uacfc \ubaa8\uc758\ud574 \uc5ec\uc528\ub97c \uce58\ub3c4\ub85d \uc791\uc815\ud588\ub2e4. \uadf8\ub7ec\uae30\uc5d0 \uad00\uc601\uc740 \ud615\uc591\uc5d0 \uc8fc\ub454\ud558\uba74\uc11c \uc18c\ubb38\uc744 \ud37c\ub728\ub824 \uc81c\ub098\ub77c \uad70\uc0ac\uac00 \uc9c4\uaca9\uc744 \uba48\ucd94\uac8c \ud558\uace0, \uc8fc\ubc1c \ub4f1\uc774 \uc774 \ud2c8\uc5d0 \uc5ec\uc528\ub4e4\uc744 \ub9d0\uac1b\uac8c \uc4f8\uc5c8\ub2e4. \uc720\uc591\uacfc \uad00\uc601 \ubaa8\ub450 \uad70\uc0ac\ub97c \ud574\uc0b0\ud574 \ub3cc\uc544\uac14\uace0, \uad00\uc601\uc740 \uc8fc\ubc1c, \uc9c4\ud3c9\uacfc \ud568\uaed8 \uace0\uc870\uc758 \uc544\ub4e4 \ub300\uc655\uc744 \ud669\uc81c\ub85c \uc138\uc6b0\ub2c8, \uc774\uac00 \uace7 \ubb38\uc81c\ub2e4.", "sentence_answer": "\ud0dc\ud6c4\uac00 \uc8fd\uc790, \uc5ec\ub85d \ub4f1\uc758 \uc5ec\uc528 \uc77c\uc871\uc774 \uc7a5\uc548\uc758 \uad70\uc0ac\ub97c \uc7a5\uc545\ud558\uace0 \uc788\uc5c8\uace0, \uace0\uc870\uc758 \uc190\uc790, \uc720\ube44\uc758 \uc544\ub4e4\ub85c \uc81c\ub098\ub77c\uc758 \uc655\uc744 \uc9c0\ub0b4\ub294 \uc720\uc591 \uc740 \ubd88\ub9cc\uc744 \ud488\uace0 \ubc18\ub780\uc744 \uc77c\uc73c\ucf1c \uc5ec\uc528 \uc138\ub825\uc744 \uc4f8\uc5b4\ubc84\ub9ac\uace0\uc790 \ud588\ub2e4.", "paragraph_id": "6471994-2-0", "paragraph_question": "question: \ud55c \uace0\uc870\uc758 \uc190\uc790\ub85c \uc81c\ub098\ub77c\uc758 \uc655\uc774\uc5c8\ub358 \uc778\ubb3c\uc740?, context: \uace0\uc870 \uc0ac\ud6c4, \ud61c\uc81c\uac00 \uc989\uc704\ud558\uace0 \uace0\ud669\ud6c4\uac00 \uc2e4\uad8c\uc744 \uc7a1\uc544 \ub098\ub77c\ub97c \ub2e4\uc2a4\ub838\ub2e4. \uad00\uc601\uc740 \uc5f4\ud6c4\ub85c\uc11c \uace0\ud669\ud6c4\ub97c \ubcf4\uc88c\ud588\ub2e4. \ud0dc\ud6c4\uac00 \uc8fd\uc790, \uc5ec\ub85d \ub4f1\uc758 \uc5ec\uc528 \uc77c\uc871\uc774 \uc7a5\uc548\uc758 \uad70\uc0ac\ub97c \uc7a5\uc545\ud558\uace0 \uc788\uc5c8\uace0, \uace0\uc870\uc758 \uc190\uc790, \uc720\ube44\uc758 \uc544\ub4e4\ub85c \uc81c\ub098\ub77c\uc758 \uc655\uc744 \uc9c0\ub0b4\ub294 \uc720\uc591\uc740 \ubd88\ub9cc\uc744 \ud488\uace0 \ubc18\ub780\uc744 \uc77c\uc73c\ucf1c \uc5ec\uc528 \uc138\ub825\uc744 \uc4f8\uc5b4\ubc84\ub9ac\uace0\uc790 \ud588\ub2e4. \uc5ec\uc528 \uc77c\uc871\uc740 \uad00\uc601\uc5d0\uac8c \uad70\uc0ac\ub97c \ub9e1\uaca8 \uc81c\ub098\ub77c\uc758 \ubc18\ub780\uc744 \uc9c4\uc555\ud558\ub3c4\ub85d \ud588\uc73c\ub098, \uad00\uc601\uc740 \ud0dc\uc704 \uc8fc\ubc1c, \uc2b9\uc0c1 \uc9c4\ud3c9 \ub4f1\uacfc \ubaa8\uc758\ud574 \uc5ec\uc528\ub97c \uce58\ub3c4\ub85d \uc791\uc815\ud588\ub2e4. \uadf8\ub7ec\uae30\uc5d0 \uad00\uc601\uc740 \ud615\uc591\uc5d0 \uc8fc\ub454\ud558\uba74\uc11c \uc18c\ubb38\uc744 \ud37c\ub728\ub824 \uc81c\ub098\ub77c \uad70\uc0ac\uac00 \uc9c4\uaca9\uc744 \uba48\ucd94\uac8c \ud558\uace0, \uc8fc\ubc1c \ub4f1\uc774 \uc774 \ud2c8\uc5d0 \uc5ec\uc528\ub4e4\uc744 \ub9d0\uac1b\uac8c \uc4f8\uc5c8\ub2e4. \uc720\uc591\uacfc \uad00\uc601 \ubaa8\ub450 \uad70\uc0ac\ub97c \ud574\uc0b0\ud574 \ub3cc\uc544\uac14\uace0, \uad00\uc601\uc740 \uc8fc\ubc1c, \uc9c4\ud3c9\uacfc \ud568\uaed8 \uace0\uc870\uc758 \uc544\ub4e4 \ub300\uc655\uc744 \ud669\uc81c\ub85c \uc138\uc6b0\ub2c8, \uc774\uac00 \uace7 \ubb38\uc81c\ub2e4."} {"question": "\uc11c\uc815\uc6d0\uc5d0\uac8c \uc601\uc785 \uc81c\uc758\ub97c \ud55c \ubca4\ud53c\uce74\uc5d0\uc11c \ubc30\ubc88\ud55c \ub4f1\ubc88\ud638\ub294?", "paragraph": "1997\ub144\uc5d0\ub294 \ubca4\ud53c\uce74\uac00 \ubc30\ubc88 9\ubc88\uc744 \uc8fc\uba70 \uc601\uc785 \uc81c\uc758\ub97c \ud588\uace0 \ubca4\ud53c\uce74 \uc120\uc218\ub2e8\uc5d0 \ud569\ub958\ud574\uc11c \ud568\uaed8 \ud6c8\ub828\uc744 \ud560 \uc815\ub3c4\ub85c \uc774\uc801\uc774 \ud655\uc815\ub410\uc9c0\ub9cc \ubca4\ud53c\uce74\uac00 \ub2f9\uc2dc \ub9ac\uadf8\uc5d0\uc11c \uc131\uc801\uc774 \uc2e0\ud1b5\uce58 \uc54a\uc558\ub358 \uc774\uc720\ub85c \uc11c\uc815\uc6d0\uc774 \uc6d4\ub4dc\ucef5 \uc544\uc2dc\uc544 \uc608\uc120\uc5d0\uc11c \ub6f0\uc9c0 \uc54a\ub294 \uc870\uac74\uc744 \ubd80\uc5ec\ud588\ub294\ub370 \ub2f9\uc2dc \uac10\ub3c5\uc774\uc5c8\ub358 \ucc28\ubc94\uadfc\ub3c4 \uc6d4\ub4dc\ucef5 \uc9c4\ucd9c\uc744 \uc704\ud574 \ud300\uc758 \ud575\uc2ec \uba64\ubc84\uc911 1\uba85\uc774\uc5c8\ub358 \uc11c\uc815\uc6d0\uc744 \ubd99\uc7a1\uc73c\ub824 \ud588\ub2e4.\uc5b4\uca0b\ub4e0 \ub300\ud55c\ucd95\uad6c\ud611\ud68c\uc758 \uc774\uc801\ub3d9\uc758\uc11c\ub294 \ub098\uc624\uc9c0\uc54a\uac8c \ub418\uace0 \uc774\ub85c\uc778\ud574 \ubb34\uc0b0\ub418\uc5c8\ub2e4. \uc774 \ub2f9\uc2dc \uc5b8\ub860\uc0ac \uc778\ud130\ubdf0\uc5d0\uc11c\ub294 \ucc28\ubc94\uadfc\uc774 \uc11c\uc815\uc6d0\uc774 \ubca4\ud53c\uce74\uc5d0 \uc785\ub2e8\ud14c\uc2a4\ud2b8\ub97c \uad6c\uac78\ud558\ub7ec \uac14\ub2e4\uace0 \ubcf4\ub3c4\ud588\ub2e4. \ud3ec\ub974\ud22c\uac08\uc758 \ucd95\uad6c \uc601\uc6c5 \uc5d0\uc6b0\uc81c\ube44\uc624\uac00 2009\ub144\uc5d0 \ud64d\uba85\ubcf4\ub97c \ub9cc\ub098\uc11c \uc11c\uc815\uc6d0\uc758 \uc548\ubd80\ub97c \ubb3c\uc5c8\ub2e4\ub294 \uc0ac\ub840\ub97c \ubcfc \uc218 \uc788\ub4ef\uc774 \uadf8\uc758 \ubca4\ud53c\uce74\ud589\uc740 \ud655\uc815\uc801\uc778 \uac83\uc774\uc5c8\ub2e4. \uadf8\ub7ec\ub098 \ub2f9\uc2dc \ucc28\ubc94\uadfc\uc740 \uc2e0\ubb38\uc0ac\uc640 \ubc29\uc1a1\uc0ac \uc778\ud130\ubdf0\uc5d0\uc11c \"\ud55c\uad6d \ucd95\uad6c\uc758 \uc218\uc900\uc740 \ub192\uc544\uc84c\uc2b5\ub2c8\ub2e4. \uad73\uc774 \ud3ec\ub974\ud22c\uac08 \uad6c\ub2e8\uc5d0 \uc785\ub2e8\uc744 \uad6c\uac78\ud560 \ud544\uc694\uac00 \uc788\uc2b5\ub2c8\uae4c?\"\ub77c\uace0 \ubc1c\uc5b8\ud558\uc600\ub2e4\uace0 \uae30\uc790\ub4e4\uc740 \ubcf4\ub3c4 \ud588\ub2e4. \uc11c\uc815\uc6d0\uc740 \uac1c\uc778\uc801\uc73c\ub85c \ubca4\ud53c\uce74\uc758 9\ubc88 \uc720\ub2c8\ud3fc\ub3c4 \uc18c\uc7a5\uc911\uc774\ub77c\uace0 \ud55c\ub2e4. \uba85\ubb38\uc778 \ubca4\ud53c\uce74\ub77c \uc544\uc26c\uc6c0\uc774 \uc788\uc9c0\ub9cc \uc11c\uc815\uc6d0\uc740 \uadf8 \ud6c4\uc5d0 \ub300\ud55c\ubbfc\uad6d 1\ud638\ub85c \ud504\ub791\uc2a4\uc5d0 \uc9c4\ucd9c\ud55c\ub2e4. \uadf8\ub7ec\ub098 \uc774 \uc0ac\uac74\uc73c\ub85c \ucc28\ubc94\uadfc\uacfc \uc11c\uc815\uc6d0\uc758 \uad00\uacc4\ub294 \uae08\uc774 \uac14\ub2e4\uace0 \ubcfc \uc218 \uc5c6\ub2e4.\uace0\ub824\ub300 \uc120\ud6c4\ubc30 \uc0ac\uc774\uc774\uace0 \ucc28\ubc94\uadfc \uce7c\ub7fc\uc778 '\ucc28\ubc94\uadfc\uc758 \ub530\ub73b\ud55c \ucd95\uad6c'\uc5d0\uc11c \ucc28\ubc94\uadfc\uc774 \uc11c\uc815\uc6d0\uc744 \uc88b\uac8c \uc5b8\uae09\ud558\ub294 \ubaa8\uc2b5\uc774\ub098 \uc11c\uc815\uc6d0\uc774 \uc870\uad11\ub798 \uac10\ub3c5 \uc2dc\uc808 \ucf54\uce58\ub85c \uc77c\ud560\ub54c \ucc28\ub450\ub9ac\uc640 \uc7a5\ub09c\uce58\ub294 \ubaa8\uc2b5\uc774 \uc2e0\ubb38\uc0ac \uc0ac\uc9c4\uc5d0 \uc2e4\ub9b0 \uac83\uc744 \ubcf4\uba74 \ucd94\uc815\ud560 \uc218 \uc788\ub2e4.", "answer": "9\ubc88", "sentence": "1997\ub144\uc5d0\ub294 \ubca4\ud53c\uce74\uac00 \ubc30\ubc88 9\ubc88 \uc744 \uc8fc\uba70 \uc601\uc785 \uc81c\uc758\ub97c \ud588\uace0 \ubca4\ud53c\uce74 \uc120\uc218\ub2e8\uc5d0 \ud569\ub958\ud574\uc11c \ud568\uaed8 \ud6c8\ub828\uc744 \ud560 \uc815\ub3c4\ub85c \uc774\uc801\uc774 \ud655\uc815\ub410\uc9c0\ub9cc \ubca4\ud53c\uce74\uac00 \ub2f9\uc2dc \ub9ac\uadf8\uc5d0\uc11c \uc131\uc801\uc774 \uc2e0\ud1b5\uce58 \uc54a\uc558\ub358 \uc774\uc720\ub85c \uc11c\uc815\uc6d0\uc774 \uc6d4\ub4dc\ucef5 \uc544\uc2dc\uc544 \uc608\uc120\uc5d0\uc11c \ub6f0\uc9c0 \uc54a\ub294 \uc870\uac74\uc744 \ubd80\uc5ec\ud588\ub294\ub370 \ub2f9\uc2dc \uac10\ub3c5\uc774\uc5c8\ub358 \ucc28\ubc94\uadfc\ub3c4 \uc6d4\ub4dc\ucef5 \uc9c4\ucd9c\uc744 \uc704\ud574 \ud300\uc758 \ud575\uc2ec \uba64\ubc84\uc911 1\uba85\uc774\uc5c8\ub358 \uc11c\uc815\uc6d0\uc744 \ubd99\uc7a1\uc73c\ub824 \ud588\ub2e4.\uc5b4\uca0b\ub4e0 \ub300\ud55c\ucd95\uad6c\ud611\ud68c\uc758 \uc774\uc801\ub3d9\uc758\uc11c\ub294 \ub098\uc624\uc9c0\uc54a\uac8c \ub418\uace0 \uc774\ub85c\uc778\ud574 \ubb34\uc0b0\ub418\uc5c8\ub2e4.", "paragraph_sentence": " 1997\ub144\uc5d0\ub294 \ubca4\ud53c\uce74\uac00 \ubc30\ubc88 9\ubc88 \uc744 \uc8fc\uba70 \uc601\uc785 \uc81c\uc758\ub97c \ud588\uace0 \ubca4\ud53c\uce74 \uc120\uc218\ub2e8\uc5d0 \ud569\ub958\ud574\uc11c \ud568\uaed8 \ud6c8\ub828\uc744 \ud560 \uc815\ub3c4\ub85c \uc774\uc801\uc774 \ud655\uc815\ub410\uc9c0\ub9cc \ubca4\ud53c\uce74\uac00 \ub2f9\uc2dc \ub9ac\uadf8\uc5d0\uc11c \uc131\uc801\uc774 \uc2e0\ud1b5\uce58 \uc54a\uc558\ub358 \uc774\uc720\ub85c \uc11c\uc815\uc6d0\uc774 \uc6d4\ub4dc\ucef5 \uc544\uc2dc\uc544 \uc608\uc120\uc5d0\uc11c \ub6f0\uc9c0 \uc54a\ub294 \uc870\uac74\uc744 \ubd80\uc5ec\ud588\ub294\ub370 \ub2f9\uc2dc \uac10\ub3c5\uc774\uc5c8\ub358 \ucc28\ubc94\uadfc\ub3c4 \uc6d4\ub4dc\ucef5 \uc9c4\ucd9c\uc744 \uc704\ud574 \ud300\uc758 \ud575\uc2ec \uba64\ubc84\uc911 1\uba85\uc774\uc5c8\ub358 \uc11c\uc815\uc6d0\uc744 \ubd99\uc7a1\uc73c\ub824 \ud588\ub2e4.\uc5b4\uca0b\ub4e0 \ub300\ud55c\ucd95\uad6c\ud611\ud68c\uc758 \uc774\uc801\ub3d9\uc758\uc11c\ub294 \ub098\uc624\uc9c0\uc54a\uac8c \ub418\uace0 \uc774\ub85c\uc778\ud574 \ubb34\uc0b0\ub418\uc5c8\ub2e4. \uc774 \ub2f9\uc2dc \uc5b8\ub860\uc0ac \uc778\ud130\ubdf0\uc5d0\uc11c\ub294 \ucc28\ubc94\uadfc\uc774 \uc11c\uc815\uc6d0\uc774 \ubca4\ud53c\uce74\uc5d0 \uc785\ub2e8\ud14c\uc2a4\ud2b8\ub97c \uad6c\uac78\ud558\ub7ec \uac14\ub2e4\uace0 \ubcf4\ub3c4\ud588\ub2e4. \ud3ec\ub974\ud22c\uac08\uc758 \ucd95\uad6c \uc601\uc6c5 \uc5d0\uc6b0\uc81c\ube44\uc624\uac00 2009\ub144\uc5d0 \ud64d\uba85\ubcf4\ub97c \ub9cc\ub098\uc11c \uc11c\uc815\uc6d0\uc758 \uc548\ubd80\ub97c \ubb3c\uc5c8\ub2e4\ub294 \uc0ac\ub840\ub97c \ubcfc \uc218 \uc788\ub4ef\uc774 \uadf8\uc758 \ubca4\ud53c\uce74\ud589\uc740 \ud655\uc815\uc801\uc778 \uac83\uc774\uc5c8\ub2e4. \uadf8\ub7ec\ub098 \ub2f9\uc2dc \ucc28\ubc94\uadfc\uc740 \uc2e0\ubb38\uc0ac\uc640 \ubc29\uc1a1\uc0ac \uc778\ud130\ubdf0\uc5d0\uc11c \"\ud55c\uad6d \ucd95\uad6c\uc758 \uc218\uc900\uc740 \ub192\uc544\uc84c\uc2b5\ub2c8\ub2e4. \uad73\uc774 \ud3ec\ub974\ud22c\uac08 \uad6c\ub2e8\uc5d0 \uc785\ub2e8\uc744 \uad6c\uac78\ud560 \ud544\uc694\uac00 \uc788\uc2b5\ub2c8\uae4c?\"\ub77c\uace0 \ubc1c\uc5b8\ud558\uc600\ub2e4\uace0 \uae30\uc790\ub4e4\uc740 \ubcf4\ub3c4 \ud588\ub2e4. \uc11c\uc815\uc6d0\uc740 \uac1c\uc778\uc801\uc73c\ub85c \ubca4\ud53c\uce74\uc758 9\ubc88 \uc720\ub2c8\ud3fc\ub3c4 \uc18c\uc7a5\uc911\uc774\ub77c\uace0 \ud55c\ub2e4. \uba85\ubb38\uc778 \ubca4\ud53c\uce74\ub77c \uc544\uc26c\uc6c0\uc774 \uc788\uc9c0\ub9cc \uc11c\uc815\uc6d0\uc740 \uadf8 \ud6c4\uc5d0 \ub300\ud55c\ubbfc\uad6d 1\ud638\ub85c \ud504\ub791\uc2a4\uc5d0 \uc9c4\ucd9c\ud55c\ub2e4. \uadf8\ub7ec\ub098 \uc774 \uc0ac\uac74\uc73c\ub85c \ucc28\ubc94\uadfc\uacfc \uc11c\uc815\uc6d0\uc758 \uad00\uacc4\ub294 \uae08\uc774 \uac14\ub2e4\uace0 \ubcfc \uc218 \uc5c6\ub2e4.\uace0\ub824\ub300 \uc120\ud6c4\ubc30 \uc0ac\uc774\uc774\uace0 \ucc28\ubc94\uadfc \uce7c\ub7fc\uc778 '\ucc28\ubc94\uadfc\uc758 \ub530\ub73b\ud55c \ucd95\uad6c'\uc5d0\uc11c \ucc28\ubc94\uadfc\uc774 \uc11c\uc815\uc6d0\uc744 \uc88b\uac8c \uc5b8\uae09\ud558\ub294 \ubaa8\uc2b5\uc774\ub098 \uc11c\uc815\uc6d0\uc774 \uc870\uad11\ub798 \uac10\ub3c5 \uc2dc\uc808 \ucf54\uce58\ub85c \uc77c\ud560\ub54c \ucc28\ub450\ub9ac\uc640 \uc7a5\ub09c\uce58\ub294 \ubaa8\uc2b5\uc774 \uc2e0\ubb38\uc0ac \uc0ac\uc9c4\uc5d0 \uc2e4\ub9b0 \uac83\uc744 \ubcf4\uba74 \ucd94\uc815\ud560 \uc218 \uc788\ub2e4.", "paragraph_answer": "1997\ub144\uc5d0\ub294 \ubca4\ud53c\uce74\uac00 \ubc30\ubc88 9\ubc88 \uc744 \uc8fc\uba70 \uc601\uc785 \uc81c\uc758\ub97c \ud588\uace0 \ubca4\ud53c\uce74 \uc120\uc218\ub2e8\uc5d0 \ud569\ub958\ud574\uc11c \ud568\uaed8 \ud6c8\ub828\uc744 \ud560 \uc815\ub3c4\ub85c \uc774\uc801\uc774 \ud655\uc815\ub410\uc9c0\ub9cc \ubca4\ud53c\uce74\uac00 \ub2f9\uc2dc \ub9ac\uadf8\uc5d0\uc11c \uc131\uc801\uc774 \uc2e0\ud1b5\uce58 \uc54a\uc558\ub358 \uc774\uc720\ub85c \uc11c\uc815\uc6d0\uc774 \uc6d4\ub4dc\ucef5 \uc544\uc2dc\uc544 \uc608\uc120\uc5d0\uc11c \ub6f0\uc9c0 \uc54a\ub294 \uc870\uac74\uc744 \ubd80\uc5ec\ud588\ub294\ub370 \ub2f9\uc2dc \uac10\ub3c5\uc774\uc5c8\ub358 \ucc28\ubc94\uadfc\ub3c4 \uc6d4\ub4dc\ucef5 \uc9c4\ucd9c\uc744 \uc704\ud574 \ud300\uc758 \ud575\uc2ec \uba64\ubc84\uc911 1\uba85\uc774\uc5c8\ub358 \uc11c\uc815\uc6d0\uc744 \ubd99\uc7a1\uc73c\ub824 \ud588\ub2e4.\uc5b4\uca0b\ub4e0 \ub300\ud55c\ucd95\uad6c\ud611\ud68c\uc758 \uc774\uc801\ub3d9\uc758\uc11c\ub294 \ub098\uc624\uc9c0\uc54a\uac8c \ub418\uace0 \uc774\ub85c\uc778\ud574 \ubb34\uc0b0\ub418\uc5c8\ub2e4. \uc774 \ub2f9\uc2dc \uc5b8\ub860\uc0ac \uc778\ud130\ubdf0\uc5d0\uc11c\ub294 \ucc28\ubc94\uadfc\uc774 \uc11c\uc815\uc6d0\uc774 \ubca4\ud53c\uce74\uc5d0 \uc785\ub2e8\ud14c\uc2a4\ud2b8\ub97c \uad6c\uac78\ud558\ub7ec \uac14\ub2e4\uace0 \ubcf4\ub3c4\ud588\ub2e4. \ud3ec\ub974\ud22c\uac08\uc758 \ucd95\uad6c \uc601\uc6c5 \uc5d0\uc6b0\uc81c\ube44\uc624\uac00 2009\ub144\uc5d0 \ud64d\uba85\ubcf4\ub97c \ub9cc\ub098\uc11c \uc11c\uc815\uc6d0\uc758 \uc548\ubd80\ub97c \ubb3c\uc5c8\ub2e4\ub294 \uc0ac\ub840\ub97c \ubcfc \uc218 \uc788\ub4ef\uc774 \uadf8\uc758 \ubca4\ud53c\uce74\ud589\uc740 \ud655\uc815\uc801\uc778 \uac83\uc774\uc5c8\ub2e4. \uadf8\ub7ec\ub098 \ub2f9\uc2dc \ucc28\ubc94\uadfc\uc740 \uc2e0\ubb38\uc0ac\uc640 \ubc29\uc1a1\uc0ac \uc778\ud130\ubdf0\uc5d0\uc11c \"\ud55c\uad6d \ucd95\uad6c\uc758 \uc218\uc900\uc740 \ub192\uc544\uc84c\uc2b5\ub2c8\ub2e4. \uad73\uc774 \ud3ec\ub974\ud22c\uac08 \uad6c\ub2e8\uc5d0 \uc785\ub2e8\uc744 \uad6c\uac78\ud560 \ud544\uc694\uac00 \uc788\uc2b5\ub2c8\uae4c?\"\ub77c\uace0 \ubc1c\uc5b8\ud558\uc600\ub2e4\uace0 \uae30\uc790\ub4e4\uc740 \ubcf4\ub3c4 \ud588\ub2e4. \uc11c\uc815\uc6d0\uc740 \uac1c\uc778\uc801\uc73c\ub85c \ubca4\ud53c\uce74\uc758 9\ubc88 \uc720\ub2c8\ud3fc\ub3c4 \uc18c\uc7a5\uc911\uc774\ub77c\uace0 \ud55c\ub2e4. \uba85\ubb38\uc778 \ubca4\ud53c\uce74\ub77c \uc544\uc26c\uc6c0\uc774 \uc788\uc9c0\ub9cc \uc11c\uc815\uc6d0\uc740 \uadf8 \ud6c4\uc5d0 \ub300\ud55c\ubbfc\uad6d 1\ud638\ub85c \ud504\ub791\uc2a4\uc5d0 \uc9c4\ucd9c\ud55c\ub2e4. \uadf8\ub7ec\ub098 \uc774 \uc0ac\uac74\uc73c\ub85c \ucc28\ubc94\uadfc\uacfc \uc11c\uc815\uc6d0\uc758 \uad00\uacc4\ub294 \uae08\uc774 \uac14\ub2e4\uace0 \ubcfc \uc218 \uc5c6\ub2e4.\uace0\ub824\ub300 \uc120\ud6c4\ubc30 \uc0ac\uc774\uc774\uace0 \ucc28\ubc94\uadfc \uce7c\ub7fc\uc778 '\ucc28\ubc94\uadfc\uc758 \ub530\ub73b\ud55c \ucd95\uad6c'\uc5d0\uc11c \ucc28\ubc94\uadfc\uc774 \uc11c\uc815\uc6d0\uc744 \uc88b\uac8c \uc5b8\uae09\ud558\ub294 \ubaa8\uc2b5\uc774\ub098 \uc11c\uc815\uc6d0\uc774 \uc870\uad11\ub798 \uac10\ub3c5 \uc2dc\uc808 \ucf54\uce58\ub85c \uc77c\ud560\ub54c \ucc28\ub450\ub9ac\uc640 \uc7a5\ub09c\uce58\ub294 \ubaa8\uc2b5\uc774 \uc2e0\ubb38\uc0ac \uc0ac\uc9c4\uc5d0 \uc2e4\ub9b0 \uac83\uc744 \ubcf4\uba74 \ucd94\uc815\ud560 \uc218 \uc788\ub2e4.", "sentence_answer": "1997\ub144\uc5d0\ub294 \ubca4\ud53c\uce74\uac00 \ubc30\ubc88 9\ubc88 \uc744 \uc8fc\uba70 \uc601\uc785 \uc81c\uc758\ub97c \ud588\uace0 \ubca4\ud53c\uce74 \uc120\uc218\ub2e8\uc5d0 \ud569\ub958\ud574\uc11c \ud568\uaed8 \ud6c8\ub828\uc744 \ud560 \uc815\ub3c4\ub85c \uc774\uc801\uc774 \ud655\uc815\ub410\uc9c0\ub9cc \ubca4\ud53c\uce74\uac00 \ub2f9\uc2dc \ub9ac\uadf8\uc5d0\uc11c \uc131\uc801\uc774 \uc2e0\ud1b5\uce58 \uc54a\uc558\ub358 \uc774\uc720\ub85c \uc11c\uc815\uc6d0\uc774 \uc6d4\ub4dc\ucef5 \uc544\uc2dc\uc544 \uc608\uc120\uc5d0\uc11c \ub6f0\uc9c0 \uc54a\ub294 \uc870\uac74\uc744 \ubd80\uc5ec\ud588\ub294\ub370 \ub2f9\uc2dc \uac10\ub3c5\uc774\uc5c8\ub358 \ucc28\ubc94\uadfc\ub3c4 \uc6d4\ub4dc\ucef5 \uc9c4\ucd9c\uc744 \uc704\ud574 \ud300\uc758 \ud575\uc2ec \uba64\ubc84\uc911 1\uba85\uc774\uc5c8\ub358 \uc11c\uc815\uc6d0\uc744 \ubd99\uc7a1\uc73c\ub824 \ud588\ub2e4.\uc5b4\uca0b\ub4e0 \ub300\ud55c\ucd95\uad6c\ud611\ud68c\uc758 \uc774\uc801\ub3d9\uc758\uc11c\ub294 \ub098\uc624\uc9c0\uc54a\uac8c \ub418\uace0 \uc774\ub85c\uc778\ud574 \ubb34\uc0b0\ub418\uc5c8\ub2e4.", "paragraph_id": "6517632-1-0", "paragraph_question": "question: \uc11c\uc815\uc6d0\uc5d0\uac8c \uc601\uc785 \uc81c\uc758\ub97c \ud55c \ubca4\ud53c\uce74\uc5d0\uc11c \ubc30\ubc88\ud55c \ub4f1\ubc88\ud638\ub294?, context: 1997\ub144\uc5d0\ub294 \ubca4\ud53c\uce74\uac00 \ubc30\ubc88 9\ubc88\uc744 \uc8fc\uba70 \uc601\uc785 \uc81c\uc758\ub97c \ud588\uace0 \ubca4\ud53c\uce74 \uc120\uc218\ub2e8\uc5d0 \ud569\ub958\ud574\uc11c \ud568\uaed8 \ud6c8\ub828\uc744 \ud560 \uc815\ub3c4\ub85c \uc774\uc801\uc774 \ud655\uc815\ub410\uc9c0\ub9cc \ubca4\ud53c\uce74\uac00 \ub2f9\uc2dc \ub9ac\uadf8\uc5d0\uc11c \uc131\uc801\uc774 \uc2e0\ud1b5\uce58 \uc54a\uc558\ub358 \uc774\uc720\ub85c \uc11c\uc815\uc6d0\uc774 \uc6d4\ub4dc\ucef5 \uc544\uc2dc\uc544 \uc608\uc120\uc5d0\uc11c \ub6f0\uc9c0 \uc54a\ub294 \uc870\uac74\uc744 \ubd80\uc5ec\ud588\ub294\ub370 \ub2f9\uc2dc \uac10\ub3c5\uc774\uc5c8\ub358 \ucc28\ubc94\uadfc\ub3c4 \uc6d4\ub4dc\ucef5 \uc9c4\ucd9c\uc744 \uc704\ud574 \ud300\uc758 \ud575\uc2ec \uba64\ubc84\uc911 1\uba85\uc774\uc5c8\ub358 \uc11c\uc815\uc6d0\uc744 \ubd99\uc7a1\uc73c\ub824 \ud588\ub2e4.\uc5b4\uca0b\ub4e0 \ub300\ud55c\ucd95\uad6c\ud611\ud68c\uc758 \uc774\uc801\ub3d9\uc758\uc11c\ub294 \ub098\uc624\uc9c0\uc54a\uac8c \ub418\uace0 \uc774\ub85c\uc778\ud574 \ubb34\uc0b0\ub418\uc5c8\ub2e4. \uc774 \ub2f9\uc2dc \uc5b8\ub860\uc0ac \uc778\ud130\ubdf0\uc5d0\uc11c\ub294 \ucc28\ubc94\uadfc\uc774 \uc11c\uc815\uc6d0\uc774 \ubca4\ud53c\uce74\uc5d0 \uc785\ub2e8\ud14c\uc2a4\ud2b8\ub97c \uad6c\uac78\ud558\ub7ec \uac14\ub2e4\uace0 \ubcf4\ub3c4\ud588\ub2e4. \ud3ec\ub974\ud22c\uac08\uc758 \ucd95\uad6c \uc601\uc6c5 \uc5d0\uc6b0\uc81c\ube44\uc624\uac00 2009\ub144\uc5d0 \ud64d\uba85\ubcf4\ub97c \ub9cc\ub098\uc11c \uc11c\uc815\uc6d0\uc758 \uc548\ubd80\ub97c \ubb3c\uc5c8\ub2e4\ub294 \uc0ac\ub840\ub97c \ubcfc \uc218 \uc788\ub4ef\uc774 \uadf8\uc758 \ubca4\ud53c\uce74\ud589\uc740 \ud655\uc815\uc801\uc778 \uac83\uc774\uc5c8\ub2e4. \uadf8\ub7ec\ub098 \ub2f9\uc2dc \ucc28\ubc94\uadfc\uc740 \uc2e0\ubb38\uc0ac\uc640 \ubc29\uc1a1\uc0ac \uc778\ud130\ubdf0\uc5d0\uc11c \"\ud55c\uad6d \ucd95\uad6c\uc758 \uc218\uc900\uc740 \ub192\uc544\uc84c\uc2b5\ub2c8\ub2e4. \uad73\uc774 \ud3ec\ub974\ud22c\uac08 \uad6c\ub2e8\uc5d0 \uc785\ub2e8\uc744 \uad6c\uac78\ud560 \ud544\uc694\uac00 \uc788\uc2b5\ub2c8\uae4c?\"\ub77c\uace0 \ubc1c\uc5b8\ud558\uc600\ub2e4\uace0 \uae30\uc790\ub4e4\uc740 \ubcf4\ub3c4 \ud588\ub2e4. \uc11c\uc815\uc6d0\uc740 \uac1c\uc778\uc801\uc73c\ub85c \ubca4\ud53c\uce74\uc758 9\ubc88 \uc720\ub2c8\ud3fc\ub3c4 \uc18c\uc7a5\uc911\uc774\ub77c\uace0 \ud55c\ub2e4. \uba85\ubb38\uc778 \ubca4\ud53c\uce74\ub77c \uc544\uc26c\uc6c0\uc774 \uc788\uc9c0\ub9cc \uc11c\uc815\uc6d0\uc740 \uadf8 \ud6c4\uc5d0 \ub300\ud55c\ubbfc\uad6d 1\ud638\ub85c \ud504\ub791\uc2a4\uc5d0 \uc9c4\ucd9c\ud55c\ub2e4. \uadf8\ub7ec\ub098 \uc774 \uc0ac\uac74\uc73c\ub85c \ucc28\ubc94\uadfc\uacfc \uc11c\uc815\uc6d0\uc758 \uad00\uacc4\ub294 \uae08\uc774 \uac14\ub2e4\uace0 \ubcfc \uc218 \uc5c6\ub2e4.\uace0\ub824\ub300 \uc120\ud6c4\ubc30 \uc0ac\uc774\uc774\uace0 \ucc28\ubc94\uadfc \uce7c\ub7fc\uc778 '\ucc28\ubc94\uadfc\uc758 \ub530\ub73b\ud55c \ucd95\uad6c'\uc5d0\uc11c \ucc28\ubc94\uadfc\uc774 \uc11c\uc815\uc6d0\uc744 \uc88b\uac8c \uc5b8\uae09\ud558\ub294 \ubaa8\uc2b5\uc774\ub098 \uc11c\uc815\uc6d0\uc774 \uc870\uad11\ub798 \uac10\ub3c5 \uc2dc\uc808 \ucf54\uce58\ub85c \uc77c\ud560\ub54c \ucc28\ub450\ub9ac\uc640 \uc7a5\ub09c\uce58\ub294 \ubaa8\uc2b5\uc774 \uc2e0\ubb38\uc0ac \uc0ac\uc9c4\uc5d0 \uc2e4\ub9b0 \uac83\uc744 \ubcf4\uba74 \ucd94\uc815\ud560 \uc218 \uc788\ub2e4."} {"question": "\ud55c\uad6d\uce5c\uc6b0\ud68c\ub294 \uc5b4\ub5a4\ud55c \uc601\ud5a5\ub825\uc744 \uac00\uc9c0\uace0 \uc788\uc5c8\ub098?", "paragraph": "\ud55c\uad6d\uce5c\uc6b0\ud68c\ub294 \ubbf8\uad6d\ubfd0\ub9cc \uc544\ub2c8\ub77c \uc601\uad6d\uacfc \ud504\ub791\uc2a4\uc5d0\ub3c4 \uc870\uc9c1\ub418\uc5b4 \uad11\ubc94\uc704\ud55c \uad6d\uc81c\uc801\uc778 \uc870\uc9c1\uc744 \uac16\ucda4\uc5d0 \ub530\ub77c 3\u00b71\uc6b4\ub3d9\uc73c\ub85c \ub098\ud0c0\ub09c \ud55c\uad6d\uc758 \uc2e4\uc0c1\uc744 \uc804\ud30c\ud558\uace0 \uce5c\ud55c\uc5ec\ub860\uc744 \ud615\uc131\ud558\ub294\ub370 \uc0c1\ub2f9\ud55c \uc601\ud5a5\ub825\uc744 \ubbf8\ucce4\ub2e4. \uc77c\ubcf8\uce21\uc740 \ubbf8\uad6d\uc5d0 \uc788\ub294 \uc5ec\ub7ec \ud55c\uc778\ub2e8\uccb4 \uac00\uc6b4\ub370 \u201c\uc870\uc120\uc6b0\uc560\ub2e8\uc758 \ud589\ub3d9\uc5d0 \uad00\ud574\uc11c\ub294 \uae08\ud6c4 \ub2e4\uc18c \uc8fc\uc758\ub97c \uc694\ud560 \ud544\uc694\uac00 \uc788\uc744 \uac83\uc774\ub2e4\u201d\ud558\uc5ec \ud55c\uad6d\uce5c\uc6b0\ud68c\uc758 \uacb0\uc131\uacfc \ud65c\ub3d9\uc5d0 \ud2b9\ud788 \uc8fc\ubaa9\ud558\uc600\ub2e4. \uc6cc\uc2f1\ud134\uacfc \uac01 \uc8fc \uc8fc\uc7ac \uc77c\ubcf8\uc601\uc0ac\uad00\uc740 \uc774\ub4e4 \ub2e8\uccb4\uac00 \ubc18\uc77c\ubcf8 \uc120\ub3d9\uc744 \ud558\ub294 \ub2e8\uccb4\uc778\uc9c0 \uc5ec\ubd80\ub97c \uc0ac\ub78c\ub4e4\uc744 \uace0\uc6a9, \ud30c\uacac\ud558\uc5ec \uac10\uc2dc\ud558\uc600\ub2e4. \uadf8\ub7ec\ub098 \ucc3d\ub9bd \ucd08\uae30, \ube44\uc815\uce58\uc801 \ubaa9\uc801\uc73c\ub85c \uc124\ub9bd\ud558\uc600\uc73c\uba70 \uad6c\ubbf8 \uae30\ub3c5\uad50\uacc4 \uc778\uc0ac\ub4e4\uc5d0\uac8c \uc870\uc120\uc5d0 \ub300\ud55c \uc6b0\ud638\uc801 \uc5ec\ub860\uacfc \uce5c\ubaa9\uc744 \ub3c4\ubaa8\ud55c\ub2e4\ub294 \ub0b4\uc6a9\ub9cc\uc774 \uc788\uc5c8\uc73c\ubbc0\ub85c \ubcc4\ub2e4\ub978 \uad6c\uc2e4\uc744 \ucc3e\uc9c0 \ubabb\ud558\uc600\ub2e4.", "answer": "\ud55c\uad6d\uc758 \uc2e4\uc0c1\uc744 \uc804\ud30c\ud558\uace0 \uce5c\ud55c\uc5ec\ub860\uc744 \ud615\uc131", "sentence": "\ud55c\uad6d\uce5c\uc6b0\ud68c\ub294 \ubbf8\uad6d\ubfd0\ub9cc \uc544\ub2c8\ub77c \uc601\uad6d\uacfc \ud504\ub791\uc2a4\uc5d0\ub3c4 \uc870\uc9c1\ub418\uc5b4 \uad11\ubc94\uc704\ud55c \uad6d\uc81c\uc801\uc778 \uc870\uc9c1\uc744 \uac16\ucda4\uc5d0 \ub530\ub77c 3\u00b71\uc6b4\ub3d9\uc73c\ub85c \ub098\ud0c0\ub09c \ud55c\uad6d\uc758 \uc2e4\uc0c1\uc744 \uc804\ud30c\ud558\uace0 \uce5c\ud55c\uc5ec\ub860\uc744 \ud615\uc131 \ud558\ub294\ub370 \uc0c1\ub2f9\ud55c \uc601\ud5a5\ub825\uc744 \ubbf8\ucce4\ub2e4.", "paragraph_sentence": " \ud55c\uad6d\uce5c\uc6b0\ud68c\ub294 \ubbf8\uad6d\ubfd0\ub9cc \uc544\ub2c8\ub77c \uc601\uad6d\uacfc \ud504\ub791\uc2a4\uc5d0\ub3c4 \uc870\uc9c1\ub418\uc5b4 \uad11\ubc94\uc704\ud55c \uad6d\uc81c\uc801\uc778 \uc870\uc9c1\uc744 \uac16\ucda4\uc5d0 \ub530\ub77c 3\u00b71\uc6b4\ub3d9\uc73c\ub85c \ub098\ud0c0\ub09c \ud55c\uad6d\uc758 \uc2e4\uc0c1\uc744 \uc804\ud30c\ud558\uace0 \uce5c\ud55c\uc5ec\ub860\uc744 \ud615\uc131 \ud558\ub294\ub370 \uc0c1\ub2f9\ud55c \uc601\ud5a5\ub825\uc744 \ubbf8\ucce4\ub2e4. \uc77c\ubcf8\uce21\uc740 \ubbf8\uad6d\uc5d0 \uc788\ub294 \uc5ec\ub7ec \ud55c\uc778\ub2e8\uccb4 \uac00\uc6b4\ub370 \u201c\uc870\uc120\uc6b0\uc560\ub2e8\uc758 \ud589\ub3d9\uc5d0 \uad00\ud574\uc11c\ub294 \uae08\ud6c4 \ub2e4\uc18c \uc8fc\uc758\ub97c \uc694\ud560 \ud544\uc694\uac00 \uc788\uc744 \uac83\uc774\ub2e4\u201d\ud558\uc5ec \ud55c\uad6d\uce5c\uc6b0\ud68c\uc758 \uacb0\uc131\uacfc \ud65c\ub3d9\uc5d0 \ud2b9\ud788 \uc8fc\ubaa9\ud558\uc600\ub2e4. \uc6cc\uc2f1\ud134\uacfc \uac01 \uc8fc \uc8fc\uc7ac \uc77c\ubcf8\uc601\uc0ac\uad00\uc740 \uc774\ub4e4 \ub2e8\uccb4\uac00 \ubc18\uc77c\ubcf8 \uc120\ub3d9\uc744 \ud558\ub294 \ub2e8\uccb4\uc778\uc9c0 \uc5ec\ubd80\ub97c \uc0ac\ub78c\ub4e4\uc744 \uace0\uc6a9, \ud30c\uacac\ud558\uc5ec \uac10\uc2dc\ud558\uc600\ub2e4. \uadf8\ub7ec\ub098 \ucc3d\ub9bd \ucd08\uae30, \ube44\uc815\uce58\uc801 \ubaa9\uc801\uc73c\ub85c \uc124\ub9bd\ud558\uc600\uc73c\uba70 \uad6c\ubbf8 \uae30\ub3c5\uad50\uacc4 \uc778\uc0ac\ub4e4\uc5d0\uac8c \uc870\uc120\uc5d0 \ub300\ud55c \uc6b0\ud638\uc801 \uc5ec\ub860\uacfc \uce5c\ubaa9\uc744 \ub3c4\ubaa8\ud55c\ub2e4\ub294 \ub0b4\uc6a9\ub9cc\uc774 \uc788\uc5c8\uc73c\ubbc0\ub85c \ubcc4\ub2e4\ub978 \uad6c\uc2e4\uc744 \ucc3e\uc9c0 \ubabb\ud558\uc600\ub2e4.", "paragraph_answer": "\ud55c\uad6d\uce5c\uc6b0\ud68c\ub294 \ubbf8\uad6d\ubfd0\ub9cc \uc544\ub2c8\ub77c \uc601\uad6d\uacfc \ud504\ub791\uc2a4\uc5d0\ub3c4 \uc870\uc9c1\ub418\uc5b4 \uad11\ubc94\uc704\ud55c \uad6d\uc81c\uc801\uc778 \uc870\uc9c1\uc744 \uac16\ucda4\uc5d0 \ub530\ub77c 3\u00b71\uc6b4\ub3d9\uc73c\ub85c \ub098\ud0c0\ub09c \ud55c\uad6d\uc758 \uc2e4\uc0c1\uc744 \uc804\ud30c\ud558\uace0 \uce5c\ud55c\uc5ec\ub860\uc744 \ud615\uc131 \ud558\ub294\ub370 \uc0c1\ub2f9\ud55c \uc601\ud5a5\ub825\uc744 \ubbf8\ucce4\ub2e4. \uc77c\ubcf8\uce21\uc740 \ubbf8\uad6d\uc5d0 \uc788\ub294 \uc5ec\ub7ec \ud55c\uc778\ub2e8\uccb4 \uac00\uc6b4\ub370 \u201c\uc870\uc120\uc6b0\uc560\ub2e8\uc758 \ud589\ub3d9\uc5d0 \uad00\ud574\uc11c\ub294 \uae08\ud6c4 \ub2e4\uc18c \uc8fc\uc758\ub97c \uc694\ud560 \ud544\uc694\uac00 \uc788\uc744 \uac83\uc774\ub2e4\u201d\ud558\uc5ec \ud55c\uad6d\uce5c\uc6b0\ud68c\uc758 \uacb0\uc131\uacfc \ud65c\ub3d9\uc5d0 \ud2b9\ud788 \uc8fc\ubaa9\ud558\uc600\ub2e4. \uc6cc\uc2f1\ud134\uacfc \uac01 \uc8fc \uc8fc\uc7ac \uc77c\ubcf8\uc601\uc0ac\uad00\uc740 \uc774\ub4e4 \ub2e8\uccb4\uac00 \ubc18\uc77c\ubcf8 \uc120\ub3d9\uc744 \ud558\ub294 \ub2e8\uccb4\uc778\uc9c0 \uc5ec\ubd80\ub97c \uc0ac\ub78c\ub4e4\uc744 \uace0\uc6a9, \ud30c\uacac\ud558\uc5ec \uac10\uc2dc\ud558\uc600\ub2e4. \uadf8\ub7ec\ub098 \ucc3d\ub9bd \ucd08\uae30, \ube44\uc815\uce58\uc801 \ubaa9\uc801\uc73c\ub85c \uc124\ub9bd\ud558\uc600\uc73c\uba70 \uad6c\ubbf8 \uae30\ub3c5\uad50\uacc4 \uc778\uc0ac\ub4e4\uc5d0\uac8c \uc870\uc120\uc5d0 \ub300\ud55c \uc6b0\ud638\uc801 \uc5ec\ub860\uacfc \uce5c\ubaa9\uc744 \ub3c4\ubaa8\ud55c\ub2e4\ub294 \ub0b4\uc6a9\ub9cc\uc774 \uc788\uc5c8\uc73c\ubbc0\ub85c \ubcc4\ub2e4\ub978 \uad6c\uc2e4\uc744 \ucc3e\uc9c0 \ubabb\ud558\uc600\ub2e4.", "sentence_answer": "\ud55c\uad6d\uce5c\uc6b0\ud68c\ub294 \ubbf8\uad6d\ubfd0\ub9cc \uc544\ub2c8\ub77c \uc601\uad6d\uacfc \ud504\ub791\uc2a4\uc5d0\ub3c4 \uc870\uc9c1\ub418\uc5b4 \uad11\ubc94\uc704\ud55c \uad6d\uc81c\uc801\uc778 \uc870\uc9c1\uc744 \uac16\ucda4\uc5d0 \ub530\ub77c 3\u00b71\uc6b4\ub3d9\uc73c\ub85c \ub098\ud0c0\ub09c \ud55c\uad6d\uc758 \uc2e4\uc0c1\uc744 \uc804\ud30c\ud558\uace0 \uce5c\ud55c\uc5ec\ub860\uc744 \ud615\uc131 \ud558\ub294\ub370 \uc0c1\ub2f9\ud55c \uc601\ud5a5\ub825\uc744 \ubbf8\ucce4\ub2e4.", "paragraph_id": "5936562-3-1", "paragraph_question": "question: \ud55c\uad6d\uce5c\uc6b0\ud68c\ub294 \uc5b4\ub5a4\ud55c \uc601\ud5a5\ub825\uc744 \uac00\uc9c0\uace0 \uc788\uc5c8\ub098?, context: \ud55c\uad6d\uce5c\uc6b0\ud68c\ub294 \ubbf8\uad6d\ubfd0\ub9cc \uc544\ub2c8\ub77c \uc601\uad6d\uacfc \ud504\ub791\uc2a4\uc5d0\ub3c4 \uc870\uc9c1\ub418\uc5b4 \uad11\ubc94\uc704\ud55c \uad6d\uc81c\uc801\uc778 \uc870\uc9c1\uc744 \uac16\ucda4\uc5d0 \ub530\ub77c 3\u00b71\uc6b4\ub3d9\uc73c\ub85c \ub098\ud0c0\ub09c \ud55c\uad6d\uc758 \uc2e4\uc0c1\uc744 \uc804\ud30c\ud558\uace0 \uce5c\ud55c\uc5ec\ub860\uc744 \ud615\uc131\ud558\ub294\ub370 \uc0c1\ub2f9\ud55c \uc601\ud5a5\ub825\uc744 \ubbf8\ucce4\ub2e4. \uc77c\ubcf8\uce21\uc740 \ubbf8\uad6d\uc5d0 \uc788\ub294 \uc5ec\ub7ec \ud55c\uc778\ub2e8\uccb4 \uac00\uc6b4\ub370 \u201c\uc870\uc120\uc6b0\uc560\ub2e8\uc758 \ud589\ub3d9\uc5d0 \uad00\ud574\uc11c\ub294 \uae08\ud6c4 \ub2e4\uc18c \uc8fc\uc758\ub97c \uc694\ud560 \ud544\uc694\uac00 \uc788\uc744 \uac83\uc774\ub2e4\u201d\ud558\uc5ec \ud55c\uad6d\uce5c\uc6b0\ud68c\uc758 \uacb0\uc131\uacfc \ud65c\ub3d9\uc5d0 \ud2b9\ud788 \uc8fc\ubaa9\ud558\uc600\ub2e4. \uc6cc\uc2f1\ud134\uacfc \uac01 \uc8fc \uc8fc\uc7ac \uc77c\ubcf8\uc601\uc0ac\uad00\uc740 \uc774\ub4e4 \ub2e8\uccb4\uac00 \ubc18\uc77c\ubcf8 \uc120\ub3d9\uc744 \ud558\ub294 \ub2e8\uccb4\uc778\uc9c0 \uc5ec\ubd80\ub97c \uc0ac\ub78c\ub4e4\uc744 \uace0\uc6a9, \ud30c\uacac\ud558\uc5ec \uac10\uc2dc\ud558\uc600\ub2e4. \uadf8\ub7ec\ub098 \ucc3d\ub9bd \ucd08\uae30, \ube44\uc815\uce58\uc801 \ubaa9\uc801\uc73c\ub85c \uc124\ub9bd\ud558\uc600\uc73c\uba70 \uad6c\ubbf8 \uae30\ub3c5\uad50\uacc4 \uc778\uc0ac\ub4e4\uc5d0\uac8c \uc870\uc120\uc5d0 \ub300\ud55c \uc6b0\ud638\uc801 \uc5ec\ub860\uacfc \uce5c\ubaa9\uc744 \ub3c4\ubaa8\ud55c\ub2e4\ub294 \ub0b4\uc6a9\ub9cc\uc774 \uc788\uc5c8\uc73c\ubbc0\ub85c \ubcc4\ub2e4\ub978 \uad6c\uc2e4\uc744 \ucc3e\uc9c0 \ubabb\ud558\uc600\ub2e4."} {"question": "\ud504\ub9ac\ub4dc\ub9ac\ud788 3\uc138\ub294 \ub3c5\uc77c \uc81c\uad6d\uc758 \uba87 \ub300 \uc655\uc778\uac00?", "paragraph": "\ud504\ub9ac\ub4dc\ub9ac\ud788 3\uc138(\ub3c5\uc77c\uc5b4: Friedrich III \ud504\ub9ac\ub4dc\ub9ac\ud788 \ub370\uc5b4 \ub4dc\ub9ac\ud14c, 1831\ub144 10\uc6d4 18\uc77c ~ 1888\ub144 6\uc6d4 15\uc77c)\ub294 \ub3c5\uc77c \uc81c\uad6d\uc758 \uc81c2\ub300 \ud669\uc81c \uacb8 \ud504\ub85c\uc774\uc13c \uc655\uad6d\uc758 \uc655(Deutscher Kaiser und K\u00f6nig von Preu\u00dfen, \uc7ac\uc704: 1888\ub144 3\uc6d4 9\uc77c ~ 6\uc6d4 15\uc77c)\uc774\ub2e4. \ubcf8\uba85\uc740 \ud504\ub9ac\ub4dc\ub9ac\ud788 \ube4c\ud5ec\ub984 \ub2c8\ucf5c\ub77c\uc6b0\uc2a4 \uce74\ub97c \ud638\uc5d4\ucd10\ub808\ub978 \ud3f0 \ud504\ub85c\uc774\uc13c(Friedrich Wilhelm Nikolaus Karl Hohenzollern von Preu\u00dfen)\uc774\uba70 \ud504\ub9ac\uce20(Fritz)\ub77c\ub294 \uc560\uce6d\uc73c\ub85c\ub3c4 \ubd88\ub9ac\uc5c8\ub2e4. \ub3c5\uc77c \uc81c\uad6d\uc758 \ucd08\ub300 \ud669\uc81c \ube4c\ud5ec\ub984 1\uc138\uc640 \uc544\uc6b0\uad6c\uc2a4\ud0c0 \ud669\ud6c4\uc758 \uc678\ub3d9\uc544\ub4e4\ub85c \ud0dc\uc5b4\ub098 \uac00\ud48d\uc5d0 \ub530\ub77c \uad70\uc778\uc73c\ub85c \uc790\ub790\ub2e4. 1861\ub144 \uc544\ubc84\uc9c0\uac00 \ud504\ub85c\uc774\uc13c \uc655\uc5d0 \uc989\uc704\ud55c \ud6c4 \uc655\uc138\uc790\uc5d0 \ucc45\ubd09\ub418\uc5c8\ub2e4. \uc81c2\ucc28 \uc290\ub808\uc2a4\ube44\ud788 \uc804\uc7c1\uacfc \ud504\ub85c\uc774\uc13c-\uc624\uc2a4\ud2b8\ub9ac\uc544 \uc804\uc7c1, \uadf8\ub9ac\uace0 \ud504\ub85c\uc774\uc13c-\ud504\ub791\uc2a4 \uc804\uc7c1\uc5d0\uc11c \uad70\uacf5\uc744 \uc138\uc6b0\uace0 \uadf8 \uc9c0\ub3c4\ub825\uacfc \ub2a5\ub825\uc744 \uc778\uc815\ubc1b\uc558\uc73c\ub098, \uc804\uc7c1\uc5d0 \ub300\ud55c \uc801\uac1c\uc2ec\uc744 \uac16\uace0 \uc788\uc5c8\uace0, \uadf8\uc758 \uc778\ub3c4\uc8fc\uc758\uc801 \ucc98\uc0ac\uc5d0 \ub9ce\uc740 \uc6b0\ubc29\uacfc \uc801\uc774 \uac10\ud0c4\ud558\uc600\ub2e4. 1871\ub144 \ub3c5\uc77c \uc81c\uad6d\uc774 \uc131\ub9bd\ub41c \ud6c4, \ub2e4\uc2dc \ud669\ud0dc\uc790\ub85c \ucc45\ubd09\ub418\uc5c8\uace0 1888\ub144, \ubd80\ud669 \ube4c\ud5ec\ub984 1\uc138\uac00 90\uc138\ub97c \uc77c\uae30\ub85c \uc0ac\ub9dd\ud558\uc790 \uadf8 \ub4a4\ub97c \uc774\uc5b4 27\ub144\uac04 \ud6c4\uacc4\uc790 \uc790\ub9ac\uc5d0 \uc788\ub358 \ud504\ub9ac\ub4dc\ub9ac\ud788\uac00 \ub3c5\uc77c \uc81c\uad6d \ud669\uc81c \uacb8 \ud504\ub85c\uc774\uc13c \uc655\uc5d0 \uc989\uc704\ud558\uc600\ub2e4. \uadf8\ub7ec\ub098 \ud669\ud0dc\uc790 \uc2dc\uc808\ubd80\ud130 \ud6c4\ub450\uc554\uc73c\ub85c \ud22c\ubcd1\uc911\uc774\uc5c8\ub358 \uadf8\ub294 \uacb0\uad6d \uc989\uc704 99\uc77c \ub9cc\uc778 6\uc6d4 15\uc77c\uc5d0 56\uc138\ub97c \uc77c\uae30\ub85c \uae09\uc0ac\ud558\uc600\ub2e4.", "answer": "\uc81c2\ub300", "sentence": "\ud504\ub9ac\ub4dc\ub9ac\ud788 3\uc138(\ub3c5\uc77c\uc5b4: Friedrich III \ud504\ub9ac\ub4dc\ub9ac\ud788 \ub370\uc5b4 \ub4dc\ub9ac\ud14c, 1831\ub144 10\uc6d4 18\uc77c ~ 1888\ub144 6\uc6d4 15\uc77c)\ub294 \ub3c5\uc77c \uc81c\uad6d\uc758 \uc81c2\ub300 \ud669\uc81c \uacb8 \ud504\ub85c\uc774\uc13c \uc655\uad6d\uc758 \uc655(Deutscher Kaiser und K\u00f6nig von Preu\u00dfen, \uc7ac\uc704: 1888\ub144 3\uc6d4 9\uc77c ~ 6\uc6d4 15\uc77c)\uc774\ub2e4.", "paragraph_sentence": " \ud504\ub9ac\ub4dc\ub9ac\ud788 3\uc138(\ub3c5\uc77c\uc5b4: Friedrich III \ud504\ub9ac\ub4dc\ub9ac\ud788 \ub370\uc5b4 \ub4dc\ub9ac\ud14c, 1831\ub144 10\uc6d4 18\uc77c ~ 1888\ub144 6\uc6d4 15\uc77c)\ub294 \ub3c5\uc77c \uc81c\uad6d\uc758 \uc81c2\ub300 \ud669\uc81c \uacb8 \ud504\ub85c\uc774\uc13c \uc655\uad6d\uc758 \uc655(Deutscher Kaiser und K\u00f6nig von Preu\u00dfen, \uc7ac\uc704: 1888\ub144 3\uc6d4 9\uc77c ~ 6\uc6d4 15\uc77c)\uc774\ub2e4. \ubcf8\uba85\uc740 \ud504\ub9ac\ub4dc\ub9ac\ud788 \ube4c\ud5ec\ub984 \ub2c8\ucf5c\ub77c\uc6b0\uc2a4 \uce74\ub97c \ud638\uc5d4\ucd10\ub808\ub978 \ud3f0 \ud504\ub85c\uc774\uc13c(Friedrich Wilhelm Nikolaus Karl Hohenzollern von Preu\u00dfen)\uc774\uba70 \ud504\ub9ac\uce20(Fritz)\ub77c\ub294 \uc560\uce6d\uc73c\ub85c\ub3c4 \ubd88\ub9ac\uc5c8\ub2e4. \ub3c5\uc77c \uc81c\uad6d\uc758 \ucd08\ub300 \ud669\uc81c \ube4c\ud5ec\ub984 1\uc138\uc640 \uc544\uc6b0\uad6c\uc2a4\ud0c0 \ud669\ud6c4\uc758 \uc678\ub3d9\uc544\ub4e4\ub85c \ud0dc\uc5b4\ub098 \uac00\ud48d\uc5d0 \ub530\ub77c \uad70\uc778\uc73c\ub85c \uc790\ub790\ub2e4. 1861\ub144 \uc544\ubc84\uc9c0\uac00 \ud504\ub85c\uc774\uc13c \uc655\uc5d0 \uc989\uc704\ud55c \ud6c4 \uc655\uc138\uc790\uc5d0 \ucc45\ubd09\ub418\uc5c8\ub2e4. \uc81c2\ucc28 \uc290\ub808\uc2a4\ube44\ud788 \uc804\uc7c1\uacfc \ud504\ub85c\uc774\uc13c-\uc624\uc2a4\ud2b8\ub9ac\uc544 \uc804\uc7c1, \uadf8\ub9ac\uace0 \ud504\ub85c\uc774\uc13c-\ud504\ub791\uc2a4 \uc804\uc7c1\uc5d0\uc11c \uad70\uacf5\uc744 \uc138\uc6b0\uace0 \uadf8 \uc9c0\ub3c4\ub825\uacfc \ub2a5\ub825\uc744 \uc778\uc815\ubc1b\uc558\uc73c\ub098, \uc804\uc7c1\uc5d0 \ub300\ud55c \uc801\uac1c\uc2ec\uc744 \uac16\uace0 \uc788\uc5c8\uace0, \uadf8\uc758 \uc778\ub3c4\uc8fc\uc758\uc801 \ucc98\uc0ac\uc5d0 \ub9ce\uc740 \uc6b0\ubc29\uacfc \uc801\uc774 \uac10\ud0c4\ud558\uc600\ub2e4. 1871\ub144 \ub3c5\uc77c \uc81c\uad6d\uc774 \uc131\ub9bd\ub41c \ud6c4, \ub2e4\uc2dc \ud669\ud0dc\uc790\ub85c \ucc45\ubd09\ub418\uc5c8\uace0 1888\ub144, \ubd80\ud669 \ube4c\ud5ec\ub984 1\uc138\uac00 90\uc138\ub97c \uc77c\uae30\ub85c \uc0ac\ub9dd\ud558\uc790 \uadf8 \ub4a4\ub97c \uc774\uc5b4 27\ub144\uac04 \ud6c4\uacc4\uc790 \uc790\ub9ac\uc5d0 \uc788\ub358 \ud504\ub9ac\ub4dc\ub9ac\ud788\uac00 \ub3c5\uc77c \uc81c\uad6d \ud669\uc81c \uacb8 \ud504\ub85c\uc774\uc13c \uc655\uc5d0 \uc989\uc704\ud558\uc600\ub2e4. \uadf8\ub7ec\ub098 \ud669\ud0dc\uc790 \uc2dc\uc808\ubd80\ud130 \ud6c4\ub450\uc554\uc73c\ub85c \ud22c\ubcd1\uc911\uc774\uc5c8\ub358 \uadf8\ub294 \uacb0\uad6d \uc989\uc704 99\uc77c \ub9cc\uc778 6\uc6d4 15\uc77c\uc5d0 56\uc138\ub97c \uc77c\uae30\ub85c \uae09\uc0ac\ud558\uc600\ub2e4.", "paragraph_answer": "\ud504\ub9ac\ub4dc\ub9ac\ud788 3\uc138(\ub3c5\uc77c\uc5b4: Friedrich III \ud504\ub9ac\ub4dc\ub9ac\ud788 \ub370\uc5b4 \ub4dc\ub9ac\ud14c, 1831\ub144 10\uc6d4 18\uc77c ~ 1888\ub144 6\uc6d4 15\uc77c)\ub294 \ub3c5\uc77c \uc81c\uad6d\uc758 \uc81c2\ub300 \ud669\uc81c \uacb8 \ud504\ub85c\uc774\uc13c \uc655\uad6d\uc758 \uc655(Deutscher Kaiser und K\u00f6nig von Preu\u00dfen, \uc7ac\uc704: 1888\ub144 3\uc6d4 9\uc77c ~ 6\uc6d4 15\uc77c)\uc774\ub2e4. \ubcf8\uba85\uc740 \ud504\ub9ac\ub4dc\ub9ac\ud788 \ube4c\ud5ec\ub984 \ub2c8\ucf5c\ub77c\uc6b0\uc2a4 \uce74\ub97c \ud638\uc5d4\ucd10\ub808\ub978 \ud3f0 \ud504\ub85c\uc774\uc13c(Friedrich Wilhelm Nikolaus Karl Hohenzollern von Preu\u00dfen)\uc774\uba70 \ud504\ub9ac\uce20(Fritz)\ub77c\ub294 \uc560\uce6d\uc73c\ub85c\ub3c4 \ubd88\ub9ac\uc5c8\ub2e4. \ub3c5\uc77c \uc81c\uad6d\uc758 \ucd08\ub300 \ud669\uc81c \ube4c\ud5ec\ub984 1\uc138\uc640 \uc544\uc6b0\uad6c\uc2a4\ud0c0 \ud669\ud6c4\uc758 \uc678\ub3d9\uc544\ub4e4\ub85c \ud0dc\uc5b4\ub098 \uac00\ud48d\uc5d0 \ub530\ub77c \uad70\uc778\uc73c\ub85c \uc790\ub790\ub2e4. 1861\ub144 \uc544\ubc84\uc9c0\uac00 \ud504\ub85c\uc774\uc13c \uc655\uc5d0 \uc989\uc704\ud55c \ud6c4 \uc655\uc138\uc790\uc5d0 \ucc45\ubd09\ub418\uc5c8\ub2e4. \uc81c2\ucc28 \uc290\ub808\uc2a4\ube44\ud788 \uc804\uc7c1\uacfc \ud504\ub85c\uc774\uc13c-\uc624\uc2a4\ud2b8\ub9ac\uc544 \uc804\uc7c1, \uadf8\ub9ac\uace0 \ud504\ub85c\uc774\uc13c-\ud504\ub791\uc2a4 \uc804\uc7c1\uc5d0\uc11c \uad70\uacf5\uc744 \uc138\uc6b0\uace0 \uadf8 \uc9c0\ub3c4\ub825\uacfc \ub2a5\ub825\uc744 \uc778\uc815\ubc1b\uc558\uc73c\ub098, \uc804\uc7c1\uc5d0 \ub300\ud55c \uc801\uac1c\uc2ec\uc744 \uac16\uace0 \uc788\uc5c8\uace0, \uadf8\uc758 \uc778\ub3c4\uc8fc\uc758\uc801 \ucc98\uc0ac\uc5d0 \ub9ce\uc740 \uc6b0\ubc29\uacfc \uc801\uc774 \uac10\ud0c4\ud558\uc600\ub2e4. 1871\ub144 \ub3c5\uc77c \uc81c\uad6d\uc774 \uc131\ub9bd\ub41c \ud6c4, \ub2e4\uc2dc \ud669\ud0dc\uc790\ub85c \ucc45\ubd09\ub418\uc5c8\uace0 1888\ub144, \ubd80\ud669 \ube4c\ud5ec\ub984 1\uc138\uac00 90\uc138\ub97c \uc77c\uae30\ub85c \uc0ac\ub9dd\ud558\uc790 \uadf8 \ub4a4\ub97c \uc774\uc5b4 27\ub144\uac04 \ud6c4\uacc4\uc790 \uc790\ub9ac\uc5d0 \uc788\ub358 \ud504\ub9ac\ub4dc\ub9ac\ud788\uac00 \ub3c5\uc77c \uc81c\uad6d \ud669\uc81c \uacb8 \ud504\ub85c\uc774\uc13c \uc655\uc5d0 \uc989\uc704\ud558\uc600\ub2e4. \uadf8\ub7ec\ub098 \ud669\ud0dc\uc790 \uc2dc\uc808\ubd80\ud130 \ud6c4\ub450\uc554\uc73c\ub85c \ud22c\ubcd1\uc911\uc774\uc5c8\ub358 \uadf8\ub294 \uacb0\uad6d \uc989\uc704 99\uc77c \ub9cc\uc778 6\uc6d4 15\uc77c\uc5d0 56\uc138\ub97c \uc77c\uae30\ub85c \uae09\uc0ac\ud558\uc600\ub2e4.", "sentence_answer": "\ud504\ub9ac\ub4dc\ub9ac\ud788 3\uc138(\ub3c5\uc77c\uc5b4: Friedrich III \ud504\ub9ac\ub4dc\ub9ac\ud788 \ub370\uc5b4 \ub4dc\ub9ac\ud14c, 1831\ub144 10\uc6d4 18\uc77c ~ 1888\ub144 6\uc6d4 15\uc77c)\ub294 \ub3c5\uc77c \uc81c\uad6d\uc758 \uc81c2\ub300 \ud669\uc81c \uacb8 \ud504\ub85c\uc774\uc13c \uc655\uad6d\uc758 \uc655(Deutscher Kaiser und K\u00f6nig von Preu\u00dfen, \uc7ac\uc704: 1888\ub144 3\uc6d4 9\uc77c ~ 6\uc6d4 15\uc77c)\uc774\ub2e4.", "paragraph_id": "6223777-0-0", "paragraph_question": "question: \ud504\ub9ac\ub4dc\ub9ac\ud788 3\uc138\ub294 \ub3c5\uc77c \uc81c\uad6d\uc758 \uba87 \ub300 \uc655\uc778\uac00?, context: \ud504\ub9ac\ub4dc\ub9ac\ud788 3\uc138(\ub3c5\uc77c\uc5b4: Friedrich III \ud504\ub9ac\ub4dc\ub9ac\ud788 \ub370\uc5b4 \ub4dc\ub9ac\ud14c, 1831\ub144 10\uc6d4 18\uc77c ~ 1888\ub144 6\uc6d4 15\uc77c)\ub294 \ub3c5\uc77c \uc81c\uad6d\uc758 \uc81c2\ub300 \ud669\uc81c \uacb8 \ud504\ub85c\uc774\uc13c \uc655\uad6d\uc758 \uc655(Deutscher Kaiser und K\u00f6nig von Preu\u00dfen, \uc7ac\uc704: 1888\ub144 3\uc6d4 9\uc77c ~ 6\uc6d4 15\uc77c)\uc774\ub2e4. \ubcf8\uba85\uc740 \ud504\ub9ac\ub4dc\ub9ac\ud788 \ube4c\ud5ec\ub984 \ub2c8\ucf5c\ub77c\uc6b0\uc2a4 \uce74\ub97c \ud638\uc5d4\ucd10\ub808\ub978 \ud3f0 \ud504\ub85c\uc774\uc13c(Friedrich Wilhelm Nikolaus Karl Hohenzollern von Preu\u00dfen)\uc774\uba70 \ud504\ub9ac\uce20(Fritz)\ub77c\ub294 \uc560\uce6d\uc73c\ub85c\ub3c4 \ubd88\ub9ac\uc5c8\ub2e4. \ub3c5\uc77c \uc81c\uad6d\uc758 \ucd08\ub300 \ud669\uc81c \ube4c\ud5ec\ub984 1\uc138\uc640 \uc544\uc6b0\uad6c\uc2a4\ud0c0 \ud669\ud6c4\uc758 \uc678\ub3d9\uc544\ub4e4\ub85c \ud0dc\uc5b4\ub098 \uac00\ud48d\uc5d0 \ub530\ub77c \uad70\uc778\uc73c\ub85c \uc790\ub790\ub2e4. 1861\ub144 \uc544\ubc84\uc9c0\uac00 \ud504\ub85c\uc774\uc13c \uc655\uc5d0 \uc989\uc704\ud55c \ud6c4 \uc655\uc138\uc790\uc5d0 \ucc45\ubd09\ub418\uc5c8\ub2e4. \uc81c2\ucc28 \uc290\ub808\uc2a4\ube44\ud788 \uc804\uc7c1\uacfc \ud504\ub85c\uc774\uc13c-\uc624\uc2a4\ud2b8\ub9ac\uc544 \uc804\uc7c1, \uadf8\ub9ac\uace0 \ud504\ub85c\uc774\uc13c-\ud504\ub791\uc2a4 \uc804\uc7c1\uc5d0\uc11c \uad70\uacf5\uc744 \uc138\uc6b0\uace0 \uadf8 \uc9c0\ub3c4\ub825\uacfc \ub2a5\ub825\uc744 \uc778\uc815\ubc1b\uc558\uc73c\ub098, \uc804\uc7c1\uc5d0 \ub300\ud55c \uc801\uac1c\uc2ec\uc744 \uac16\uace0 \uc788\uc5c8\uace0, \uadf8\uc758 \uc778\ub3c4\uc8fc\uc758\uc801 \ucc98\uc0ac\uc5d0 \ub9ce\uc740 \uc6b0\ubc29\uacfc \uc801\uc774 \uac10\ud0c4\ud558\uc600\ub2e4. 1871\ub144 \ub3c5\uc77c \uc81c\uad6d\uc774 \uc131\ub9bd\ub41c \ud6c4, \ub2e4\uc2dc \ud669\ud0dc\uc790\ub85c \ucc45\ubd09\ub418\uc5c8\uace0 1888\ub144, \ubd80\ud669 \ube4c\ud5ec\ub984 1\uc138\uac00 90\uc138\ub97c \uc77c\uae30\ub85c \uc0ac\ub9dd\ud558\uc790 \uadf8 \ub4a4\ub97c \uc774\uc5b4 27\ub144\uac04 \ud6c4\uacc4\uc790 \uc790\ub9ac\uc5d0 \uc788\ub358 \ud504\ub9ac\ub4dc\ub9ac\ud788\uac00 \ub3c5\uc77c \uc81c\uad6d \ud669\uc81c \uacb8 \ud504\ub85c\uc774\uc13c \uc655\uc5d0 \uc989\uc704\ud558\uc600\ub2e4. \uadf8\ub7ec\ub098 \ud669\ud0dc\uc790 \uc2dc\uc808\ubd80\ud130 \ud6c4\ub450\uc554\uc73c\ub85c \ud22c\ubcd1\uc911\uc774\uc5c8\ub358 \uadf8\ub294 \uacb0\uad6d \uc989\uc704 99\uc77c \ub9cc\uc778 6\uc6d4 15\uc77c\uc5d0 56\uc138\ub97c \uc77c\uae30\ub85c \uae09\uc0ac\ud558\uc600\ub2e4."} {"question": "\uccad\ubb38\ud68c\uc5d0\uc11c \uae40\ud5dd\uc6b1\uc740 \uae40\uc131\uace4\uc774 \uac77\uc740 \uc815\uce58\uc790\uae08 \uc911 \uba87 \ub9cc \ub2ec\ub7ec\ub97c \uac1c\uc778\uc801 \uc6a9\ub3c4\ub85c \ubcf4\uad00\ud558\uc600\ub2e4 \ud558\uc600\ub098?", "paragraph": "\ubcf4\uace0\uc11c\ub294 \ubc15\uc815\ud76c \ub300\ud1b5\ub839\uacfc \uadf8\uc758 \uce21\uadfc\ub4e4\uc774 \uae30\uc5c5\ub4e4\uacfc \uac01\uc885 \uc774\uad8c\uc0ac\uc5c5\uc744 \ud1b5\ud574 \ubd88\ubc95 \uc815\uce58\uc790\uae08\uc744 \uc5b4\ub5bb\uac8c \ubaa8\uc73c\uace0 \uad00\ub9ac\ud588\ub294\uc9c0\ub97c \ubc1d\ud788\uace0 \uc788\uc73c\uba70, \uc774 \uacfc\uc815\uc5d0\uc11c \u2018\ubc15\uc815\ud76c \uc2a4\uc704\uc2a4 \uc740\ud589 \uacc4\uc88c\uc124\u2019\uc5d0 \ub300\ud55c \uc5b8\uae09\ub3c4 \ub098\uc628\ub2e4. \ubcf4\uace0\uc11c\ub294 \u201c\ubcf8 \uc18c\uc704 \uccad\ubb38\ud68c\uc5d0\uc11c \uae40\ud615\uc6b1\uc740 \uae40\uc131\uace4\uc774 \uac77\uc740 \uc815\uce58\uc790\uae08 \uac00\uc6b4\ub370 75\ub9cc\ub2ec\ub7ec\ub97c \uac1c\uc778\uc801 \uc6a9\ub3c4\ub85c \ubcf4\uad00\ud588\uace0, \ubc15 \ub300\ud1b5\ub839\uacfc \uadf8 \ubd80\uc778(\uc721\uc601\uc218), \uc815\uc77c\uad8c, \uc774\ud6c4\ub77d, \ubc15\uc885\uaddc \ub4f1\uc758 \uac1c\uc778\uc801 \uc774\uc775\uc744 \uc704\ud574 \ube44\uc2b7\ud558\uac8c \uc81c\uacf5\ub41c \uc790\uae08\ub3c4 \uae40\uc131\uace4\uc774 \ubcf4\uad00\ud588\ub2e4\uace0 \uc99d\uc5b8\ud588\ub2e4\u201d\uace0 \ubc1d\ud788\uace0 \uc788\ub2e4. \ub610\ud55c \u201c\ubc15 \ub300\ud1b5\ub839\uc5d0\uac8c \uc804\ub2ec\ub41c \uc790\uae08\ub4e4\uacfc \uad00\ub828\ud574, \uc99d\uc5b8\u00b7\uae30\ub85d\ub4e4\uc740 \uc774\ud6c4\ub77d\uc5d0 \uc758\ud574 \uc218\uc9d1\ub41c \uc790\uae08\ub4e4\uc774 \uc2a4\uc704\uc2a4 \uc740\ud589\uacc4\uc88c\uc5d0 \uc608\uce58\ub418\uc5c8\ub2e4\uace0 \ud588\ub2e4\u201d\uba70 \u201c\uc2a4\uc704\uc2a4 \uacc4\uc88c\uc758 \uc874\uc7ac\ub294 \uc740\ud589 \uae30\ub85d\ub4e4\ub85c \uad6c\uccb4\ud654\ub418\uc5c8\uace0, \uc774\ub3d9\ud6c8(\uc774\ud6c4\ub77d \uc544\ub4e4)\uc5d0 \uc758\ud574, \uadf8\ub9ac\uace0 \ub2e4\uc218\uc758 \ucd5c\uce21\uadfc\uc5d0 \uc758\ud574 \ud655\uc778\ub418\uc5c8\ub2e4\u201d\uace0 \ubc1d\ud788\uace0 \uc788\ub2e4. \uc774 \uc2a4\uc704\uc2a4 \uacc4\uc88c\uc758 \ub3c8\uc740 \ub300\ud1b5\ub839\uc774 \uc0ac\uc6a9\ud558\uae30 \uc704\ud55c \u2018\uc815\ubd80\uc790\uae08\u2019\uc774\uc5c8\uace0, \ubc15 \uc804 \ub300\ud1b5\ub839\uc740 \uc774 \uc790\uae08\ub4e4\uc744 \uc57c\ub2f9 \uad6d\ud68c\uc758\uc6d0, \uad70\ubd80 \ub4f1\uc744 \u2018\ub9e4\uc218\u2019\ud558\uae30 \uc704\ud574 \ub098\ub220\uc92c\ub2e4\uace0 \ud55c\ub2e4.", "answer": "75\ub9cc\ub2ec\ub7ec", "sentence": "\ubcf4\uace0\uc11c\ub294 \u201c\ubcf8 \uc18c\uc704 \uccad\ubb38\ud68c\uc5d0\uc11c \uae40\ud615\uc6b1\uc740 \uae40\uc131\uace4\uc774 \uac77\uc740 \uc815\uce58\uc790\uae08 \uac00\uc6b4\ub370 75\ub9cc\ub2ec\ub7ec \ub97c \uac1c\uc778\uc801 \uc6a9\ub3c4\ub85c \ubcf4\uad00\ud588\uace0, \ubc15 \ub300\ud1b5\ub839\uacfc \uadf8 \ubd80\uc778(\uc721\uc601\uc218), \uc815\uc77c\uad8c, \uc774\ud6c4\ub77d, \ubc15\uc885\uaddc \ub4f1\uc758 \uac1c\uc778\uc801 \uc774\uc775\uc744 \uc704\ud574 \ube44\uc2b7\ud558\uac8c \uc81c\uacf5\ub41c \uc790\uae08\ub3c4 \uae40\uc131\uace4\uc774 \ubcf4\uad00\ud588\ub2e4\uace0 \uc99d\uc5b8\ud588\ub2e4\u201d\uace0 \ubc1d\ud788\uace0 \uc788\ub2e4.", "paragraph_sentence": "\ubcf4\uace0\uc11c\ub294 \ubc15\uc815\ud76c \ub300\ud1b5\ub839\uacfc \uadf8\uc758 \uce21\uadfc\ub4e4\uc774 \uae30\uc5c5\ub4e4\uacfc \uac01\uc885 \uc774\uad8c\uc0ac\uc5c5\uc744 \ud1b5\ud574 \ubd88\ubc95 \uc815\uce58\uc790\uae08\uc744 \uc5b4\ub5bb\uac8c \ubaa8\uc73c\uace0 \uad00\ub9ac\ud588\ub294\uc9c0\ub97c \ubc1d\ud788\uace0 \uc788\uc73c\uba70, \uc774 \uacfc\uc815\uc5d0\uc11c \u2018\ubc15\uc815\ud76c \uc2a4\uc704\uc2a4 \uc740\ud589 \uacc4\uc88c\uc124\u2019\uc5d0 \ub300\ud55c \uc5b8\uae09\ub3c4 \ub098\uc628\ub2e4. \ubcf4\uace0\uc11c\ub294 \u201c\ubcf8 \uc18c\uc704 \uccad\ubb38\ud68c\uc5d0\uc11c \uae40\ud615\uc6b1\uc740 \uae40\uc131\uace4\uc774 \uac77\uc740 \uc815\uce58\uc790\uae08 \uac00\uc6b4\ub370 75\ub9cc\ub2ec\ub7ec \ub97c \uac1c\uc778\uc801 \uc6a9\ub3c4\ub85c \ubcf4\uad00\ud588\uace0, \ubc15 \ub300\ud1b5\ub839\uacfc \uadf8 \ubd80\uc778(\uc721\uc601\uc218), \uc815\uc77c\uad8c, \uc774\ud6c4\ub77d, \ubc15\uc885\uaddc \ub4f1\uc758 \uac1c\uc778\uc801 \uc774\uc775\uc744 \uc704\ud574 \ube44\uc2b7\ud558\uac8c \uc81c\uacf5\ub41c \uc790\uae08\ub3c4 \uae40\uc131\uace4\uc774 \ubcf4\uad00\ud588\ub2e4\uace0 \uc99d\uc5b8\ud588\ub2e4\u201d\uace0 \ubc1d\ud788\uace0 \uc788\ub2e4. \ub610\ud55c \u201c\ubc15 \ub300\ud1b5\ub839\uc5d0\uac8c \uc804\ub2ec\ub41c \uc790\uae08\ub4e4\uacfc \uad00\ub828\ud574, \uc99d\uc5b8\u00b7\uae30\ub85d\ub4e4\uc740 \uc774\ud6c4\ub77d\uc5d0 \uc758\ud574 \uc218\uc9d1\ub41c \uc790\uae08\ub4e4\uc774 \uc2a4\uc704\uc2a4 \uc740\ud589\uacc4\uc88c\uc5d0 \uc608\uce58\ub418\uc5c8\ub2e4\uace0 \ud588\ub2e4\u201d\uba70 \u201c\uc2a4\uc704\uc2a4 \uacc4\uc88c\uc758 \uc874\uc7ac\ub294 \uc740\ud589 \uae30\ub85d\ub4e4\ub85c \uad6c\uccb4\ud654\ub418\uc5c8\uace0, \uc774\ub3d9\ud6c8(\uc774\ud6c4\ub77d \uc544\ub4e4)\uc5d0 \uc758\ud574, \uadf8\ub9ac\uace0 \ub2e4\uc218\uc758 \ucd5c\uce21\uadfc\uc5d0 \uc758\ud574 \ud655\uc778\ub418\uc5c8\ub2e4\u201d\uace0 \ubc1d\ud788\uace0 \uc788\ub2e4. \uc774 \uc2a4\uc704\uc2a4 \uacc4\uc88c\uc758 \ub3c8\uc740 \ub300\ud1b5\ub839\uc774 \uc0ac\uc6a9\ud558\uae30 \uc704\ud55c \u2018\uc815\ubd80\uc790\uae08\u2019\uc774\uc5c8\uace0, \ubc15 \uc804 \ub300\ud1b5\ub839\uc740 \uc774 \uc790\uae08\ub4e4\uc744 \uc57c\ub2f9 \uad6d\ud68c\uc758\uc6d0, \uad70\ubd80 \ub4f1\uc744 \u2018\ub9e4\uc218\u2019\ud558\uae30 \uc704\ud574 \ub098\ub220\uc92c\ub2e4\uace0 \ud55c\ub2e4.", "paragraph_answer": "\ubcf4\uace0\uc11c\ub294 \ubc15\uc815\ud76c \ub300\ud1b5\ub839\uacfc \uadf8\uc758 \uce21\uadfc\ub4e4\uc774 \uae30\uc5c5\ub4e4\uacfc \uac01\uc885 \uc774\uad8c\uc0ac\uc5c5\uc744 \ud1b5\ud574 \ubd88\ubc95 \uc815\uce58\uc790\uae08\uc744 \uc5b4\ub5bb\uac8c \ubaa8\uc73c\uace0 \uad00\ub9ac\ud588\ub294\uc9c0\ub97c \ubc1d\ud788\uace0 \uc788\uc73c\uba70, \uc774 \uacfc\uc815\uc5d0\uc11c \u2018\ubc15\uc815\ud76c \uc2a4\uc704\uc2a4 \uc740\ud589 \uacc4\uc88c\uc124\u2019\uc5d0 \ub300\ud55c \uc5b8\uae09\ub3c4 \ub098\uc628\ub2e4. \ubcf4\uace0\uc11c\ub294 \u201c\ubcf8 \uc18c\uc704 \uccad\ubb38\ud68c\uc5d0\uc11c \uae40\ud615\uc6b1\uc740 \uae40\uc131\uace4\uc774 \uac77\uc740 \uc815\uce58\uc790\uae08 \uac00\uc6b4\ub370 75\ub9cc\ub2ec\ub7ec \ub97c \uac1c\uc778\uc801 \uc6a9\ub3c4\ub85c \ubcf4\uad00\ud588\uace0, \ubc15 \ub300\ud1b5\ub839\uacfc \uadf8 \ubd80\uc778(\uc721\uc601\uc218), \uc815\uc77c\uad8c, \uc774\ud6c4\ub77d, \ubc15\uc885\uaddc \ub4f1\uc758 \uac1c\uc778\uc801 \uc774\uc775\uc744 \uc704\ud574 \ube44\uc2b7\ud558\uac8c \uc81c\uacf5\ub41c \uc790\uae08\ub3c4 \uae40\uc131\uace4\uc774 \ubcf4\uad00\ud588\ub2e4\uace0 \uc99d\uc5b8\ud588\ub2e4\u201d\uace0 \ubc1d\ud788\uace0 \uc788\ub2e4. \ub610\ud55c \u201c\ubc15 \ub300\ud1b5\ub839\uc5d0\uac8c \uc804\ub2ec\ub41c \uc790\uae08\ub4e4\uacfc \uad00\ub828\ud574, \uc99d\uc5b8\u00b7\uae30\ub85d\ub4e4\uc740 \uc774\ud6c4\ub77d\uc5d0 \uc758\ud574 \uc218\uc9d1\ub41c \uc790\uae08\ub4e4\uc774 \uc2a4\uc704\uc2a4 \uc740\ud589\uacc4\uc88c\uc5d0 \uc608\uce58\ub418\uc5c8\ub2e4\uace0 \ud588\ub2e4\u201d\uba70 \u201c\uc2a4\uc704\uc2a4 \uacc4\uc88c\uc758 \uc874\uc7ac\ub294 \uc740\ud589 \uae30\ub85d\ub4e4\ub85c \uad6c\uccb4\ud654\ub418\uc5c8\uace0, \uc774\ub3d9\ud6c8(\uc774\ud6c4\ub77d \uc544\ub4e4)\uc5d0 \uc758\ud574, \uadf8\ub9ac\uace0 \ub2e4\uc218\uc758 \ucd5c\uce21\uadfc\uc5d0 \uc758\ud574 \ud655\uc778\ub418\uc5c8\ub2e4\u201d\uace0 \ubc1d\ud788\uace0 \uc788\ub2e4. \uc774 \uc2a4\uc704\uc2a4 \uacc4\uc88c\uc758 \ub3c8\uc740 \ub300\ud1b5\ub839\uc774 \uc0ac\uc6a9\ud558\uae30 \uc704\ud55c \u2018\uc815\ubd80\uc790\uae08\u2019\uc774\uc5c8\uace0, \ubc15 \uc804 \ub300\ud1b5\ub839\uc740 \uc774 \uc790\uae08\ub4e4\uc744 \uc57c\ub2f9 \uad6d\ud68c\uc758\uc6d0, \uad70\ubd80 \ub4f1\uc744 \u2018\ub9e4\uc218\u2019\ud558\uae30 \uc704\ud574 \ub098\ub220\uc92c\ub2e4\uace0 \ud55c\ub2e4.", "sentence_answer": "\ubcf4\uace0\uc11c\ub294 \u201c\ubcf8 \uc18c\uc704 \uccad\ubb38\ud68c\uc5d0\uc11c \uae40\ud615\uc6b1\uc740 \uae40\uc131\uace4\uc774 \uac77\uc740 \uc815\uce58\uc790\uae08 \uac00\uc6b4\ub370 75\ub9cc\ub2ec\ub7ec \ub97c \uac1c\uc778\uc801 \uc6a9\ub3c4\ub85c \ubcf4\uad00\ud588\uace0, \ubc15 \ub300\ud1b5\ub839\uacfc \uadf8 \ubd80\uc778(\uc721\uc601\uc218), \uc815\uc77c\uad8c, \uc774\ud6c4\ub77d, \ubc15\uc885\uaddc \ub4f1\uc758 \uac1c\uc778\uc801 \uc774\uc775\uc744 \uc704\ud574 \ube44\uc2b7\ud558\uac8c \uc81c\uacf5\ub41c \uc790\uae08\ub3c4 \uae40\uc131\uace4\uc774 \ubcf4\uad00\ud588\ub2e4\uace0 \uc99d\uc5b8\ud588\ub2e4\u201d\uace0 \ubc1d\ud788\uace0 \uc788\ub2e4.", "paragraph_id": "6516370-65-1", "paragraph_question": "question: \uccad\ubb38\ud68c\uc5d0\uc11c \uae40\ud5dd\uc6b1\uc740 \uae40\uc131\uace4\uc774 \uac77\uc740 \uc815\uce58\uc790\uae08 \uc911 \uba87 \ub9cc \ub2ec\ub7ec\ub97c \uac1c\uc778\uc801 \uc6a9\ub3c4\ub85c \ubcf4\uad00\ud558\uc600\ub2e4 \ud558\uc600\ub098?, context: \ubcf4\uace0\uc11c\ub294 \ubc15\uc815\ud76c \ub300\ud1b5\ub839\uacfc \uadf8\uc758 \uce21\uadfc\ub4e4\uc774 \uae30\uc5c5\ub4e4\uacfc \uac01\uc885 \uc774\uad8c\uc0ac\uc5c5\uc744 \ud1b5\ud574 \ubd88\ubc95 \uc815\uce58\uc790\uae08\uc744 \uc5b4\ub5bb\uac8c \ubaa8\uc73c\uace0 \uad00\ub9ac\ud588\ub294\uc9c0\ub97c \ubc1d\ud788\uace0 \uc788\uc73c\uba70, \uc774 \uacfc\uc815\uc5d0\uc11c \u2018\ubc15\uc815\ud76c \uc2a4\uc704\uc2a4 \uc740\ud589 \uacc4\uc88c\uc124\u2019\uc5d0 \ub300\ud55c \uc5b8\uae09\ub3c4 \ub098\uc628\ub2e4. \ubcf4\uace0\uc11c\ub294 \u201c\ubcf8 \uc18c\uc704 \uccad\ubb38\ud68c\uc5d0\uc11c \uae40\ud615\uc6b1\uc740 \uae40\uc131\uace4\uc774 \uac77\uc740 \uc815\uce58\uc790\uae08 \uac00\uc6b4\ub370 75\ub9cc\ub2ec\ub7ec\ub97c \uac1c\uc778\uc801 \uc6a9\ub3c4\ub85c \ubcf4\uad00\ud588\uace0, \ubc15 \ub300\ud1b5\ub839\uacfc \uadf8 \ubd80\uc778(\uc721\uc601\uc218), \uc815\uc77c\uad8c, \uc774\ud6c4\ub77d, \ubc15\uc885\uaddc \ub4f1\uc758 \uac1c\uc778\uc801 \uc774\uc775\uc744 \uc704\ud574 \ube44\uc2b7\ud558\uac8c \uc81c\uacf5\ub41c \uc790\uae08\ub3c4 \uae40\uc131\uace4\uc774 \ubcf4\uad00\ud588\ub2e4\uace0 \uc99d\uc5b8\ud588\ub2e4\u201d\uace0 \ubc1d\ud788\uace0 \uc788\ub2e4. \ub610\ud55c \u201c\ubc15 \ub300\ud1b5\ub839\uc5d0\uac8c \uc804\ub2ec\ub41c \uc790\uae08\ub4e4\uacfc \uad00\ub828\ud574, \uc99d\uc5b8\u00b7\uae30\ub85d\ub4e4\uc740 \uc774\ud6c4\ub77d\uc5d0 \uc758\ud574 \uc218\uc9d1\ub41c \uc790\uae08\ub4e4\uc774 \uc2a4\uc704\uc2a4 \uc740\ud589\uacc4\uc88c\uc5d0 \uc608\uce58\ub418\uc5c8\ub2e4\uace0 \ud588\ub2e4\u201d\uba70 \u201c\uc2a4\uc704\uc2a4 \uacc4\uc88c\uc758 \uc874\uc7ac\ub294 \uc740\ud589 \uae30\ub85d\ub4e4\ub85c \uad6c\uccb4\ud654\ub418\uc5c8\uace0, \uc774\ub3d9\ud6c8(\uc774\ud6c4\ub77d \uc544\ub4e4)\uc5d0 \uc758\ud574, \uadf8\ub9ac\uace0 \ub2e4\uc218\uc758 \ucd5c\uce21\uadfc\uc5d0 \uc758\ud574 \ud655\uc778\ub418\uc5c8\ub2e4\u201d\uace0 \ubc1d\ud788\uace0 \uc788\ub2e4. \uc774 \uc2a4\uc704\uc2a4 \uacc4\uc88c\uc758 \ub3c8\uc740 \ub300\ud1b5\ub839\uc774 \uc0ac\uc6a9\ud558\uae30 \uc704\ud55c \u2018\uc815\ubd80\uc790\uae08\u2019\uc774\uc5c8\uace0, \ubc15 \uc804 \ub300\ud1b5\ub839\uc740 \uc774 \uc790\uae08\ub4e4\uc744 \uc57c\ub2f9 \uad6d\ud68c\uc758\uc6d0, \uad70\ubd80 \ub4f1\uc744 \u2018\ub9e4\uc218\u2019\ud558\uae30 \uc704\ud574 \ub098\ub220\uc92c\ub2e4\uace0 \ud55c\ub2e4."} {"question": "\ud574\ub2ec\uacfc \uc778\uac04 \uc5b4\uc5c5\ud65c\ub3d9\uc758 \ucda9\ub3cc\uc744 \uc870\uc815\ud558\uae30 \uc704\ud574 \uacf5\ud45c\ud55c \uac83\uc740?", "paragraph": "FWS\uac00 \ud574\ub2ec \uc774\uc8fc \uacc4\ud68d\uc744 \uc2dc\ud589\ud588\uc744 \ub2f9\uc2dc, \ub2f9\uad6d\uc5d0\uc11c\ub294 \uce98\ub9ac\ud3ec\ub2c8\uc544 \uac1c\uccb4\uad70\uc758 \"\uc601\uc5ed \uad00\ub9ac\"(zonal management) \uc5ed\uc2dc \uc2dc\ud589\ud558\ub824 \uc2dc\ub3c4\ud55c \ubc14 \uc788\ub2e4. \ud574\ub2ec\uacfc \uc778\uac04 \uc5b4\uc5c5 \ud65c\ub3d9\uc758 \ucda9\ub3cc\uc744 \uc870\uc815\ud558\uae30 \uc704\ud574, \ud3ec\uc778\ud2b8\ucee8\uc149\uc158\uc5d0\uc11c \uba55\uc2dc\ucf54\uc640\uc758 \uad6d\uacbd\uc5d0 \uc774\ub974\ub294 \"\ud574\ub2ec \uc5c6\ub294 \uc601\uc5ed\"(otter-free zone)\uc744 \uacf5\ud45c\ud55c \uac83\uc774\ub2e4. \uc774 \uc601\uc5ed \uc548\uc5d0\uc11c\ub294 \uc0b0\ub2c8\ucf5c\ub77c\uc2a4 \uc12c\ub9cc \ud574\ub2ec \uc11c\uc2dd\uc9c0\ub85c \uc9c0\uc815\ub418\uc5c8\uace0, \uadf8\uc678 \ub2e4\ub978 \uacf3\uc5d0\uc11c \ubc1c\uacac\ub41c \ud574\ub2ec\uc740 \ud3ec\ud68d\ud558\uc5ec \uc774\ub3d9\uc2dc\ud0a8\ub2e4\ub294 \uacc4\ud68d\uc774\uc5c8\ub2e4. \uadf8\ub7ec\ub098 \uc774\ub3d9\ub41c \ud574\ub2ec \uc911 \ub9ce\uc740 \uc218\uac00 \ud3d0\uc0ac\ud588\uace0, \uaddc\uc81c\ub97c \ubb34\uc2dc\ud558\uace0 \uc6d4\uacbd\ud558\ub294 \uc218\ubc31 \ub9c8\ub9ac\uc758 \ud574\ub2ec\uc744 \ubaa8\ub450 \ud3ec\ud68d\ud560 \uc2e4\uc9c8\uc801 \uc218\ub2e8\uc774 \uc5c6\uc5c8\uae30 \ub54c\ubb38\uc5d0 \uc774 \uacc4\ud68d\uc740 \ud3d0\uae30\ub418\uc5c8\ub2e4. \uc0b0\ud0c0\ubc14\ubc14\ub77c \uc9c0\uc5ed\uc758 \ud658\uacbd\ubcf4\ud638\uc13c\ud130\uc640 Otter Project\uac00 \uc81c\uae30\ud55c \uc18c\uc1a1\uc5d0 \ub300\uc751\ud558\uc5ec, 2012\ub144 12\uc6d4 19\uc77c USFWS\ub294 \"\ud574\ub2ec \uc5c6\ub294 \uc601\uc5ed\" \uc2e4\ud5d8\uc740 \uc2e4\ud328\ub85c \ub05d\ub0ac\uc74c\uc744 \uc778\uc815\ud558\uace0, \ud574\ub2ec\uc744 \ub2e4\uc2dc \uba78\uc885\uc704\uae30\uc885\uc73c\ub85c \ucde8\uae09, \ud3ec\uc778\ud2b8\ucee8\uc149\uc158 \ub0a8\uc548\uc5d0 \uad70\uc9d1\uc744 \uc7ac\uc870\uc131\ud558\uc5ec \ubcf4\ud638\ud558\uaca0\ub2e4\uace0 \uc120\uc5b8\ud588\ub2e4.", "answer": "\ud574\ub2ec \uc5c6\ub294 \uc601\uc5ed", "sentence": "\ud574\ub2ec\uacfc \uc778\uac04 \uc5b4\uc5c5 \ud65c\ub3d9\uc758 \ucda9\ub3cc\uc744 \uc870\uc815\ud558\uae30 \uc704\ud574, \ud3ec\uc778\ud2b8\ucee8\uc149\uc158\uc5d0\uc11c \uba55\uc2dc\ucf54\uc640\uc758 \uad6d\uacbd\uc5d0 \uc774\ub974\ub294 \" \ud574\ub2ec \uc5c6\ub294 \uc601\uc5ed \"(otter-free zone)\uc744 \uacf5\ud45c\ud55c \uac83\uc774\ub2e4.", "paragraph_sentence": "FWS\uac00 \ud574\ub2ec \uc774\uc8fc \uacc4\ud68d\uc744 \uc2dc\ud589\ud588\uc744 \ub2f9\uc2dc, \ub2f9\uad6d\uc5d0\uc11c\ub294 \uce98\ub9ac\ud3ec\ub2c8\uc544 \uac1c\uccb4\uad70\uc758 \"\uc601\uc5ed \uad00\ub9ac\"(zonal management) \uc5ed\uc2dc \uc2dc\ud589\ud558\ub824 \uc2dc\ub3c4\ud55c \ubc14 \uc788\ub2e4. \ud574\ub2ec\uacfc \uc778\uac04 \uc5b4\uc5c5 \ud65c\ub3d9\uc758 \ucda9\ub3cc\uc744 \uc870\uc815\ud558\uae30 \uc704\ud574, \ud3ec\uc778\ud2b8\ucee8\uc149\uc158\uc5d0\uc11c \uba55\uc2dc\ucf54\uc640\uc758 \uad6d\uacbd\uc5d0 \uc774\ub974\ub294 \" \ud574\ub2ec \uc5c6\ub294 \uc601\uc5ed \"(otter-free zone)\uc744 \uacf5\ud45c\ud55c \uac83\uc774\ub2e4. \uc774 \uc601\uc5ed \uc548\uc5d0\uc11c\ub294 \uc0b0\ub2c8\ucf5c\ub77c\uc2a4 \uc12c\ub9cc \ud574\ub2ec \uc11c\uc2dd\uc9c0\ub85c \uc9c0\uc815\ub418\uc5c8\uace0, \uadf8\uc678 \ub2e4\ub978 \uacf3\uc5d0\uc11c \ubc1c\uacac\ub41c \ud574\ub2ec\uc740 \ud3ec\ud68d\ud558\uc5ec \uc774\ub3d9\uc2dc\ud0a8\ub2e4\ub294 \uacc4\ud68d\uc774\uc5c8\ub2e4. \uadf8\ub7ec\ub098 \uc774\ub3d9\ub41c \ud574\ub2ec \uc911 \ub9ce\uc740 \uc218\uac00 \ud3d0\uc0ac\ud588\uace0, \uaddc\uc81c\ub97c \ubb34\uc2dc\ud558\uace0 \uc6d4\uacbd\ud558\ub294 \uc218\ubc31 \ub9c8\ub9ac\uc758 \ud574\ub2ec\uc744 \ubaa8\ub450 \ud3ec\ud68d\ud560 \uc2e4\uc9c8\uc801 \uc218\ub2e8\uc774 \uc5c6\uc5c8\uae30 \ub54c\ubb38\uc5d0 \uc774 \uacc4\ud68d\uc740 \ud3d0\uae30\ub418\uc5c8\ub2e4. \uc0b0\ud0c0\ubc14\ubc14\ub77c \uc9c0\uc5ed\uc758 \ud658\uacbd\ubcf4\ud638\uc13c\ud130\uc640 Otter Project\uac00 \uc81c\uae30\ud55c \uc18c\uc1a1\uc5d0 \ub300\uc751\ud558\uc5ec, 2012\ub144 12\uc6d4 19\uc77c USFWS\ub294 \"\ud574\ub2ec \uc5c6\ub294 \uc601\uc5ed\" \uc2e4\ud5d8\uc740 \uc2e4\ud328\ub85c \ub05d\ub0ac\uc74c\uc744 \uc778\uc815\ud558\uace0, \ud574\ub2ec\uc744 \ub2e4\uc2dc \uba78\uc885\uc704\uae30\uc885\uc73c\ub85c \ucde8\uae09, \ud3ec\uc778\ud2b8\ucee8\uc149\uc158 \ub0a8\uc548\uc5d0 \uad70\uc9d1\uc744 \uc7ac\uc870\uc131\ud558\uc5ec \ubcf4\ud638\ud558\uaca0\ub2e4\uace0 \uc120\uc5b8\ud588\ub2e4.", "paragraph_answer": "FWS\uac00 \ud574\ub2ec \uc774\uc8fc \uacc4\ud68d\uc744 \uc2dc\ud589\ud588\uc744 \ub2f9\uc2dc, \ub2f9\uad6d\uc5d0\uc11c\ub294 \uce98\ub9ac\ud3ec\ub2c8\uc544 \uac1c\uccb4\uad70\uc758 \"\uc601\uc5ed \uad00\ub9ac\"(zonal management) \uc5ed\uc2dc \uc2dc\ud589\ud558\ub824 \uc2dc\ub3c4\ud55c \ubc14 \uc788\ub2e4. \ud574\ub2ec\uacfc \uc778\uac04 \uc5b4\uc5c5 \ud65c\ub3d9\uc758 \ucda9\ub3cc\uc744 \uc870\uc815\ud558\uae30 \uc704\ud574, \ud3ec\uc778\ud2b8\ucee8\uc149\uc158\uc5d0\uc11c \uba55\uc2dc\ucf54\uc640\uc758 \uad6d\uacbd\uc5d0 \uc774\ub974\ub294 \" \ud574\ub2ec \uc5c6\ub294 \uc601\uc5ed \"(otter-free zone)\uc744 \uacf5\ud45c\ud55c \uac83\uc774\ub2e4. \uc774 \uc601\uc5ed \uc548\uc5d0\uc11c\ub294 \uc0b0\ub2c8\ucf5c\ub77c\uc2a4 \uc12c\ub9cc \ud574\ub2ec \uc11c\uc2dd\uc9c0\ub85c \uc9c0\uc815\ub418\uc5c8\uace0, \uadf8\uc678 \ub2e4\ub978 \uacf3\uc5d0\uc11c \ubc1c\uacac\ub41c \ud574\ub2ec\uc740 \ud3ec\ud68d\ud558\uc5ec \uc774\ub3d9\uc2dc\ud0a8\ub2e4\ub294 \uacc4\ud68d\uc774\uc5c8\ub2e4. \uadf8\ub7ec\ub098 \uc774\ub3d9\ub41c \ud574\ub2ec \uc911 \ub9ce\uc740 \uc218\uac00 \ud3d0\uc0ac\ud588\uace0, \uaddc\uc81c\ub97c \ubb34\uc2dc\ud558\uace0 \uc6d4\uacbd\ud558\ub294 \uc218\ubc31 \ub9c8\ub9ac\uc758 \ud574\ub2ec\uc744 \ubaa8\ub450 \ud3ec\ud68d\ud560 \uc2e4\uc9c8\uc801 \uc218\ub2e8\uc774 \uc5c6\uc5c8\uae30 \ub54c\ubb38\uc5d0 \uc774 \uacc4\ud68d\uc740 \ud3d0\uae30\ub418\uc5c8\ub2e4. \uc0b0\ud0c0\ubc14\ubc14\ub77c \uc9c0\uc5ed\uc758 \ud658\uacbd\ubcf4\ud638\uc13c\ud130\uc640 Otter Project\uac00 \uc81c\uae30\ud55c \uc18c\uc1a1\uc5d0 \ub300\uc751\ud558\uc5ec, 2012\ub144 12\uc6d4 19\uc77c USFWS\ub294 \"\ud574\ub2ec \uc5c6\ub294 \uc601\uc5ed\" \uc2e4\ud5d8\uc740 \uc2e4\ud328\ub85c \ub05d\ub0ac\uc74c\uc744 \uc778\uc815\ud558\uace0, \ud574\ub2ec\uc744 \ub2e4\uc2dc \uba78\uc885\uc704\uae30\uc885\uc73c\ub85c \ucde8\uae09, \ud3ec\uc778\ud2b8\ucee8\uc149\uc158 \ub0a8\uc548\uc5d0 \uad70\uc9d1\uc744 \uc7ac\uc870\uc131\ud558\uc5ec \ubcf4\ud638\ud558\uaca0\ub2e4\uace0 \uc120\uc5b8\ud588\ub2e4.", "sentence_answer": "\ud574\ub2ec\uacfc \uc778\uac04 \uc5b4\uc5c5 \ud65c\ub3d9\uc758 \ucda9\ub3cc\uc744 \uc870\uc815\ud558\uae30 \uc704\ud574, \ud3ec\uc778\ud2b8\ucee8\uc149\uc158\uc5d0\uc11c \uba55\uc2dc\ucf54\uc640\uc758 \uad6d\uacbd\uc5d0 \uc774\ub974\ub294 \" \ud574\ub2ec \uc5c6\ub294 \uc601\uc5ed \"(otter-free zone)\uc744 \uacf5\ud45c\ud55c \uac83\uc774\ub2e4.", "paragraph_id": "6570452-11-0", "paragraph_question": "question: \ud574\ub2ec\uacfc \uc778\uac04 \uc5b4\uc5c5\ud65c\ub3d9\uc758 \ucda9\ub3cc\uc744 \uc870\uc815\ud558\uae30 \uc704\ud574 \uacf5\ud45c\ud55c \uac83\uc740?, context: FWS\uac00 \ud574\ub2ec \uc774\uc8fc \uacc4\ud68d\uc744 \uc2dc\ud589\ud588\uc744 \ub2f9\uc2dc, \ub2f9\uad6d\uc5d0\uc11c\ub294 \uce98\ub9ac\ud3ec\ub2c8\uc544 \uac1c\uccb4\uad70\uc758 \"\uc601\uc5ed \uad00\ub9ac\"(zonal management) \uc5ed\uc2dc \uc2dc\ud589\ud558\ub824 \uc2dc\ub3c4\ud55c \ubc14 \uc788\ub2e4. \ud574\ub2ec\uacfc \uc778\uac04 \uc5b4\uc5c5 \ud65c\ub3d9\uc758 \ucda9\ub3cc\uc744 \uc870\uc815\ud558\uae30 \uc704\ud574, \ud3ec\uc778\ud2b8\ucee8\uc149\uc158\uc5d0\uc11c \uba55\uc2dc\ucf54\uc640\uc758 \uad6d\uacbd\uc5d0 \uc774\ub974\ub294 \"\ud574\ub2ec \uc5c6\ub294 \uc601\uc5ed\"(otter-free zone)\uc744 \uacf5\ud45c\ud55c \uac83\uc774\ub2e4. \uc774 \uc601\uc5ed \uc548\uc5d0\uc11c\ub294 \uc0b0\ub2c8\ucf5c\ub77c\uc2a4 \uc12c\ub9cc \ud574\ub2ec \uc11c\uc2dd\uc9c0\ub85c \uc9c0\uc815\ub418\uc5c8\uace0, \uadf8\uc678 \ub2e4\ub978 \uacf3\uc5d0\uc11c \ubc1c\uacac\ub41c \ud574\ub2ec\uc740 \ud3ec\ud68d\ud558\uc5ec \uc774\ub3d9\uc2dc\ud0a8\ub2e4\ub294 \uacc4\ud68d\uc774\uc5c8\ub2e4. \uadf8\ub7ec\ub098 \uc774\ub3d9\ub41c \ud574\ub2ec \uc911 \ub9ce\uc740 \uc218\uac00 \ud3d0\uc0ac\ud588\uace0, \uaddc\uc81c\ub97c \ubb34\uc2dc\ud558\uace0 \uc6d4\uacbd\ud558\ub294 \uc218\ubc31 \ub9c8\ub9ac\uc758 \ud574\ub2ec\uc744 \ubaa8\ub450 \ud3ec\ud68d\ud560 \uc2e4\uc9c8\uc801 \uc218\ub2e8\uc774 \uc5c6\uc5c8\uae30 \ub54c\ubb38\uc5d0 \uc774 \uacc4\ud68d\uc740 \ud3d0\uae30\ub418\uc5c8\ub2e4. \uc0b0\ud0c0\ubc14\ubc14\ub77c \uc9c0\uc5ed\uc758 \ud658\uacbd\ubcf4\ud638\uc13c\ud130\uc640 Otter Project\uac00 \uc81c\uae30\ud55c \uc18c\uc1a1\uc5d0 \ub300\uc751\ud558\uc5ec, 2012\ub144 12\uc6d4 19\uc77c USFWS\ub294 \"\ud574\ub2ec \uc5c6\ub294 \uc601\uc5ed\" \uc2e4\ud5d8\uc740 \uc2e4\ud328\ub85c \ub05d\ub0ac\uc74c\uc744 \uc778\uc815\ud558\uace0, \ud574\ub2ec\uc744 \ub2e4\uc2dc \uba78\uc885\uc704\uae30\uc885\uc73c\ub85c \ucde8\uae09, \ud3ec\uc778\ud2b8\ucee8\uc149\uc158 \ub0a8\uc548\uc5d0 \uad70\uc9d1\uc744 \uc7ac\uc870\uc131\ud558\uc5ec \ubcf4\ud638\ud558\uaca0\ub2e4\uace0 \uc120\uc5b8\ud588\ub2e4."} {"question": "\uac01 \ud559\uad50\uc758 \uc2f8\uc6c0\uafbc\ub4e4\uc744 \ubaa8\uc740 \uc9c0\uc5ed \ubaa8\uc784\uc758 \uba85\uce6d\uc740?", "paragraph": "\ud55c \uc5b8\ub860\uc758 \ucde8\uc7ac \uacb0\uacfc \uac01 \ud559\uad50 \uc2f8\uc6c0\uafbc\ub4e4\uc758 \uc9c0\uc5ed \ubaa8\uc784\uc73c\ub85c\uc11c \ubc00\uc591\uc5f0\ud569\uc740 \uc874\uc7ac\ud558\uace0 \uc788\uc5c8\ub2e4. \ub2e4\uc74c\uce74\ud398\ub098 \uc138\uc774\ud074\ub7fd \ub4f1\uc5d0 \u2018\ubc00\uc591\uc5f0\ud569\u2018\uc774\ub77c\ub294 \uc774\ub984\uc744 \ub0b4\uac78\uace0 \uc788\ub294 \ud648\ud398\uc774\uc9c0\ub3c4 \ud655\uc778\ud560 \uc218 \uc788\uc5c8\ub2e4. \ubc00\uc591\uc9c0\uc5ed \u3145\uace0\ub4f1\ud559\uad50 \ud55c \uc7ac\ud559\uc0dd\uc740 \u201c\ubc00\uc591\uc5f0\ud569\uc740 \ud559\uad50\ub97c \uc8fc\ub984\uc7a1\ub294 \uc544\uc774\ub4e4\uc758 \uce5c\ubaa9\ub2e8\uccb4\ub2e4. \ubc00\uc591\uc5f0\ud569\uc774\ub77c\uace0 \ud558\uba74 \uc544\ubb34\ub098 \ubabb \uac74\ub4dc\ub9b0\ub2e4. \uccad\ub3c4\uc5f0\ud569\uc774\ub098 \ucc3d\uc6d0\uc5f0\ud569\ud558\uace0 \ube44\uc2b7\ud558\ub2e4. \uc694\uc998 \uc6ec\ub9cc\ud55c \uc9c0\uc5ed\uc5d0\ub294 \ub2e4 \uc5f0\ud569\uc774 \uc788\ub2e4\u201d\ub77c\uace0 \ub9d0\ud588\ub2e4. \uc778\ud130\ubdf0\uac00 \uc788\uae30 \uc804\ub0a0 \ub9cc\ub09c \ud574\ub2f9 \ud559\uad50 \u2018\uc790\uce6d \uc5f0\ud569\u2019 \uc18c\uc18d \ud559\uc0dd\uc740 \u201c\uc624\ub298\ub3c4 \ud55c \ub140\uc11d\uacfc \uc2f8\uc6c0\uc744 \ud588\ub2e4\u201d\ub77c\uba70 \uc5bc\uad74\uc5d0 \uc0dd\uae34 \uc0c1\ucc98\ub97c \ubcf4\uc5ec\uc8fc\uc5c8\ub2e4. \uc774 \ud559\uc0dd\uc740 \u201c\ubc00\uc591\uc5d0\ub294 \uc5f0\ud569 \uc18c\uc18d\uc774 40~100\uba85\ucbe4 \ub418\ub294\ub370 \uc815\ud655\ud788 \ub204\uac00 \ud68c\uc6d0\uc774\ub2e4 \uc544\ub2c8\ub2e4\ub77c\ub294 \ub9d0\uc740 \ubabb\ud55c\ub2e4\u201d\ub77c\uace0 \ub2f5\ud588\ub2e4.", "answer": "\ubc00\uc591\uc5f0\ud569", "sentence": "\ud55c \uc5b8\ub860\uc758 \ucde8\uc7ac \uacb0\uacfc \uac01 \ud559\uad50 \uc2f8\uc6c0\uafbc\ub4e4\uc758 \uc9c0\uc5ed \ubaa8\uc784\uc73c\ub85c\uc11c \ubc00\uc591\uc5f0\ud569 \uc740 \uc874\uc7ac\ud558\uace0 \uc788\uc5c8\ub2e4.", "paragraph_sentence": " \ud55c \uc5b8\ub860\uc758 \ucde8\uc7ac \uacb0\uacfc \uac01 \ud559\uad50 \uc2f8\uc6c0\uafbc\ub4e4\uc758 \uc9c0\uc5ed \ubaa8\uc784\uc73c\ub85c\uc11c \ubc00\uc591\uc5f0\ud569 \uc740 \uc874\uc7ac\ud558\uace0 \uc788\uc5c8\ub2e4. \ub2e4\uc74c\uce74\ud398\ub098 \uc138\uc774\ud074\ub7fd \ub4f1\uc5d0 \u2018\ubc00\uc591\uc5f0\ud569\u2018\uc774\ub77c\ub294 \uc774\ub984\uc744 \ub0b4\uac78\uace0 \uc788\ub294 \ud648\ud398\uc774\uc9c0\ub3c4 \ud655\uc778\ud560 \uc218 \uc788\uc5c8\ub2e4. \ubc00\uc591\uc9c0\uc5ed \u3145\uace0\ub4f1\ud559\uad50 \ud55c \uc7ac\ud559\uc0dd\uc740 \u201c\ubc00\uc591\uc5f0\ud569\uc740 \ud559\uad50\ub97c \uc8fc\ub984\uc7a1\ub294 \uc544\uc774\ub4e4\uc758 \uce5c\ubaa9\ub2e8\uccb4\ub2e4. \ubc00\uc591\uc5f0\ud569\uc774\ub77c\uace0 \ud558\uba74 \uc544\ubb34\ub098 \ubabb \uac74\ub4dc\ub9b0\ub2e4. \uccad\ub3c4\uc5f0\ud569\uc774\ub098 \ucc3d\uc6d0\uc5f0\ud569\ud558\uace0 \ube44\uc2b7\ud558\ub2e4. \uc694\uc998 \uc6ec\ub9cc\ud55c \uc9c0\uc5ed\uc5d0\ub294 \ub2e4 \uc5f0\ud569\uc774 \uc788\ub2e4\u201d\ub77c\uace0 \ub9d0\ud588\ub2e4. \uc778\ud130\ubdf0\uac00 \uc788\uae30 \uc804\ub0a0 \ub9cc\ub09c \ud574\ub2f9 \ud559\uad50 \u2018\uc790\uce6d \uc5f0\ud569\u2019 \uc18c\uc18d \ud559\uc0dd\uc740 \u201c\uc624\ub298\ub3c4 \ud55c \ub140\uc11d\uacfc \uc2f8\uc6c0\uc744 \ud588\ub2e4\u201d\ub77c\uba70 \uc5bc\uad74\uc5d0 \uc0dd\uae34 \uc0c1\ucc98\ub97c \ubcf4\uc5ec\uc8fc\uc5c8\ub2e4. \uc774 \ud559\uc0dd\uc740 \u201c\ubc00\uc591\uc5d0\ub294 \uc5f0\ud569 \uc18c\uc18d\uc774 40~100\uba85\ucbe4 \ub418\ub294\ub370 \uc815\ud655\ud788 \ub204\uac00 \ud68c\uc6d0\uc774\ub2e4 \uc544\ub2c8\ub2e4\ub77c\ub294 \ub9d0\uc740 \ubabb\ud55c\ub2e4\u201d\ub77c\uace0 \ub2f5\ud588\ub2e4.", "paragraph_answer": "\ud55c \uc5b8\ub860\uc758 \ucde8\uc7ac \uacb0\uacfc \uac01 \ud559\uad50 \uc2f8\uc6c0\uafbc\ub4e4\uc758 \uc9c0\uc5ed \ubaa8\uc784\uc73c\ub85c\uc11c \ubc00\uc591\uc5f0\ud569 \uc740 \uc874\uc7ac\ud558\uace0 \uc788\uc5c8\ub2e4. \ub2e4\uc74c\uce74\ud398\ub098 \uc138\uc774\ud074\ub7fd \ub4f1\uc5d0 \u2018\ubc00\uc591\uc5f0\ud569\u2018\uc774\ub77c\ub294 \uc774\ub984\uc744 \ub0b4\uac78\uace0 \uc788\ub294 \ud648\ud398\uc774\uc9c0\ub3c4 \ud655\uc778\ud560 \uc218 \uc788\uc5c8\ub2e4. \ubc00\uc591\uc9c0\uc5ed \u3145\uace0\ub4f1\ud559\uad50 \ud55c \uc7ac\ud559\uc0dd\uc740 \u201c\ubc00\uc591\uc5f0\ud569\uc740 \ud559\uad50\ub97c \uc8fc\ub984\uc7a1\ub294 \uc544\uc774\ub4e4\uc758 \uce5c\ubaa9\ub2e8\uccb4\ub2e4. \ubc00\uc591\uc5f0\ud569\uc774\ub77c\uace0 \ud558\uba74 \uc544\ubb34\ub098 \ubabb \uac74\ub4dc\ub9b0\ub2e4. \uccad\ub3c4\uc5f0\ud569\uc774\ub098 \ucc3d\uc6d0\uc5f0\ud569\ud558\uace0 \ube44\uc2b7\ud558\ub2e4. \uc694\uc998 \uc6ec\ub9cc\ud55c \uc9c0\uc5ed\uc5d0\ub294 \ub2e4 \uc5f0\ud569\uc774 \uc788\ub2e4\u201d\ub77c\uace0 \ub9d0\ud588\ub2e4. \uc778\ud130\ubdf0\uac00 \uc788\uae30 \uc804\ub0a0 \ub9cc\ub09c \ud574\ub2f9 \ud559\uad50 \u2018\uc790\uce6d \uc5f0\ud569\u2019 \uc18c\uc18d \ud559\uc0dd\uc740 \u201c\uc624\ub298\ub3c4 \ud55c \ub140\uc11d\uacfc \uc2f8\uc6c0\uc744 \ud588\ub2e4\u201d\ub77c\uba70 \uc5bc\uad74\uc5d0 \uc0dd\uae34 \uc0c1\ucc98\ub97c \ubcf4\uc5ec\uc8fc\uc5c8\ub2e4. \uc774 \ud559\uc0dd\uc740 \u201c\ubc00\uc591\uc5d0\ub294 \uc5f0\ud569 \uc18c\uc18d\uc774 40~100\uba85\ucbe4 \ub418\ub294\ub370 \uc815\ud655\ud788 \ub204\uac00 \ud68c\uc6d0\uc774\ub2e4 \uc544\ub2c8\ub2e4\ub77c\ub294 \ub9d0\uc740 \ubabb\ud55c\ub2e4\u201d\ub77c\uace0 \ub2f5\ud588\ub2e4.", "sentence_answer": "\ud55c \uc5b8\ub860\uc758 \ucde8\uc7ac \uacb0\uacfc \uac01 \ud559\uad50 \uc2f8\uc6c0\uafbc\ub4e4\uc758 \uc9c0\uc5ed \ubaa8\uc784\uc73c\ub85c\uc11c \ubc00\uc591\uc5f0\ud569 \uc740 \uc874\uc7ac\ud558\uace0 \uc788\uc5c8\ub2e4.", "paragraph_id": "6603072-6-0", "paragraph_question": "question: \uac01 \ud559\uad50\uc758 \uc2f8\uc6c0\uafbc\ub4e4\uc744 \ubaa8\uc740 \uc9c0\uc5ed \ubaa8\uc784\uc758 \uba85\uce6d\uc740?, context: \ud55c \uc5b8\ub860\uc758 \ucde8\uc7ac \uacb0\uacfc \uac01 \ud559\uad50 \uc2f8\uc6c0\uafbc\ub4e4\uc758 \uc9c0\uc5ed \ubaa8\uc784\uc73c\ub85c\uc11c \ubc00\uc591\uc5f0\ud569\uc740 \uc874\uc7ac\ud558\uace0 \uc788\uc5c8\ub2e4. \ub2e4\uc74c\uce74\ud398\ub098 \uc138\uc774\ud074\ub7fd \ub4f1\uc5d0 \u2018\ubc00\uc591\uc5f0\ud569\u2018\uc774\ub77c\ub294 \uc774\ub984\uc744 \ub0b4\uac78\uace0 \uc788\ub294 \ud648\ud398\uc774\uc9c0\ub3c4 \ud655\uc778\ud560 \uc218 \uc788\uc5c8\ub2e4. \ubc00\uc591\uc9c0\uc5ed \u3145\uace0\ub4f1\ud559\uad50 \ud55c \uc7ac\ud559\uc0dd\uc740 \u201c\ubc00\uc591\uc5f0\ud569\uc740 \ud559\uad50\ub97c \uc8fc\ub984\uc7a1\ub294 \uc544\uc774\ub4e4\uc758 \uce5c\ubaa9\ub2e8\uccb4\ub2e4. \ubc00\uc591\uc5f0\ud569\uc774\ub77c\uace0 \ud558\uba74 \uc544\ubb34\ub098 \ubabb \uac74\ub4dc\ub9b0\ub2e4. \uccad\ub3c4\uc5f0\ud569\uc774\ub098 \ucc3d\uc6d0\uc5f0\ud569\ud558\uace0 \ube44\uc2b7\ud558\ub2e4. \uc694\uc998 \uc6ec\ub9cc\ud55c \uc9c0\uc5ed\uc5d0\ub294 \ub2e4 \uc5f0\ud569\uc774 \uc788\ub2e4\u201d\ub77c\uace0 \ub9d0\ud588\ub2e4. \uc778\ud130\ubdf0\uac00 \uc788\uae30 \uc804\ub0a0 \ub9cc\ub09c \ud574\ub2f9 \ud559\uad50 \u2018\uc790\uce6d \uc5f0\ud569\u2019 \uc18c\uc18d \ud559\uc0dd\uc740 \u201c\uc624\ub298\ub3c4 \ud55c \ub140\uc11d\uacfc \uc2f8\uc6c0\uc744 \ud588\ub2e4\u201d\ub77c\uba70 \uc5bc\uad74\uc5d0 \uc0dd\uae34 \uc0c1\ucc98\ub97c \ubcf4\uc5ec\uc8fc\uc5c8\ub2e4. \uc774 \ud559\uc0dd\uc740 \u201c\ubc00\uc591\uc5d0\ub294 \uc5f0\ud569 \uc18c\uc18d\uc774 40~100\uba85\ucbe4 \ub418\ub294\ub370 \uc815\ud655\ud788 \ub204\uac00 \ud68c\uc6d0\uc774\ub2e4 \uc544\ub2c8\ub2e4\ub77c\ub294 \ub9d0\uc740 \ubabb\ud55c\ub2e4\u201d\ub77c\uace0 \ub2f5\ud588\ub2e4."} {"question": "2014\ub144 4\uc6d4 16\uc77c \ubc1c\uc0dd\ud55c \uc138\uc6d4\ud638 \uce68\ubab0 \uc0ac\uace0\uc5d0 \ub300\ud55c \ub300\uc751 \uc2e4\ud328\ub85c \ud5cc\ubc95 10\uc870\uc778 \ubb34\uc5c7\uc744 \uc704\ubc18\ud558\uc600\ub098?", "paragraph": "2016\ub144 12\uc6d4 3\uc77c \uc624\ud6c4 4\uc2dc 10\ubd84, \ub354\ubd88\uc5b4\ubbfc\uc8fc\ub2f9, \uad6d\ubbfc\uc758\ub2f9, \uc815\uc758\ub2f9\uacfc \ubb34\uc18c\uc18d \uc758\uc6d0 171\uba85\uc774 \ud5cc\ubc95\uacfc \ubc95\ub960 \uc704\ubc18\uc744 \uc774\uc720\ub85c \"\ub300\ud1b5\ub839(\ubc15\uadfc\ud61c) \ud0c4\ud575\uc18c\ucd94\uc548\"\uc744 \uad6d\ud68c\uc5d0\uc11c \ubc1c\uc758\ud588\ub2e4. \ub354\ubd88\uc5b4\ubbfc\uc8fc\ub2f9 \uc6b0\uc0c1\ud638, \uad6d\ubbfc\uc758\ub2f9 \ubc15\uc9c0\uc6d0, \uc815\uc758\ub2f9 \ub178\ud68c\ucc2c \uc758\uc6d0\uc774 \ub300\ud45c \ubc1c\uc758\ud588\uc73c\uba70, \uad6c\uccb4\uc801\uc778 \uc0ac\uc720\ub85c\ub294 \ud5cc\ubc95\uacfc \ubc95\ub960 \uc704\ubc18\uc774 \uc81c\uc2dc\ub418\uc5c8\ub2e4. \ud5cc\ubc95 \uc704\ubc18 \ud589\uc704\ub85c\ub294 \ucd5c\uc21c\uc2e4\uc744 \ube44\ub86f\ud55c \uce21\uadfc\ub4e4\uc774 \uc815\ucc45\uc5d0 \uac1c\uc785\ud558\uace0 \uad6d\ubb34\ud68c\uc758\uc5d0 \uc601\ud5a5\ub825\uc744 \ud589\uc0ac\ud1a0\ub85d \ud588\ub2e4\ub294 \uc810\uc5d0\uc11c \ub300\uc758\ubbfc\uc8fc\uc8fc\uc758 \uc758\ubb34\ub97c \uc704\ubc30\ud588\uc73c\uba70, \uc774\ub4e4\uc774 \uc778\uc0ac\uc5d0 \uac1c\uc785\ud558\uc5ec \uc9c1\uc5c5\uacf5\ubb34\uc6d0\uc81c \uc704\ubc18, \uc0ac\uae30\uc5c5\uc5d0 \uae08\ud488 \ucd9c\uc5f0\uc744 \uac15\uc694\ud558\uace0 \ub1cc\ubb3c\uc744 \uc218\uc218\ud588\ub2e4\ub294 \uc810\uc5d0\uc11c \uad6d\ubbfc \uc7ac\uc0b0\uad8c \ubcf4\uc7a5\u00b7\uc2dc\uc7a5\uacbd\uc81c\uc9c8\uc11c \ubc0f \ud5cc\ubc95\uc218\ud638 \uc758\ubb34\ub97c \uc704\ubc18\ud588\ub2e4\uace0 \uc9c0\uc801\ud588\ub2e4. \ub610\ud55c 2014\ub144 4\uc6d4 16\uc77c\uc5d0 \ubc1c\uc0dd\ud55c \uc138\uc6d4\ud638 \uce68\ubab0 \uc0ac\uace0\uc5d0 \ub300\ud55c \ub300\uc751 \uc2e4\ud328\ub85c \ud5cc\ubc95 10\uc870\uc778 '\uc0dd\uba85\uad8c \ubcf4\uc7a5'\uc744 \uc704\ubc18\ud588\uc73c\uba70, \uad6d\ubbfc\uc758 \uc0dd\uba85\uacfc \uc548\uc804\uc744 \ubcf4\ud638\ud558\uae30 \uc704\ud55c \uc801\uadf9\uc801 \uc870\uce58\ub97c \ucde8\ud558\uc9c0 \uc54a\uc544 \uc9c1\ubb34 \uc720\uae30\uc5d0 \uac00\uae5d\ub2e4\uace0 \uc801\uc2dc\ud588\ub2e4. \ubc95\ub960 \uc704\ubc18 \ud589\uc704\ub85c\ub294 \uc7ac\ub2e8\ubc95\uc778 \ubbf8\ub974\uc640 \uc7ac\ub2e8\ubc95\uc778 \ucf00\uc774\uc2a4\ud3ec\uce20\uc5d0 \uc0bc\uc131\uacfc SK, \ub86f\ub370 \ub4f1\uc758 \uae30\uc5c5\uc774 \ucd9c\uc5f0\ud55c 360\uc5b5 \uc6d0 \ub1cc\ubb3c\ub85c \ud310\ub2e8\ud588\uace0, \ub86f\ub370\uac00 70\uc5b5\uc6d0\uc744 \ucd94\uac00 \ucd9c\uc5f0\ud55c \uac83 \ub4f1\uc5d0 \ub300\ud574 \ub1cc\ubb3c\uc8c4\uc640 \uc9c1\uad8c\ub0a8\uc6a9, \uac15\uc694\uc8c4\ub97c \uc801\uc6a9\ud588\ub2e4.", "answer": "\uc0dd\uba85\uad8c \ubcf4\uc7a5", "sentence": "\ub610\ud55c 2014\ub144 4\uc6d4 16\uc77c\uc5d0 \ubc1c\uc0dd\ud55c \uc138\uc6d4\ud638 \uce68\ubab0 \uc0ac\uace0\uc5d0 \ub300\ud55c \ub300\uc751 \uc2e4\ud328\ub85c \ud5cc\ubc95 10\uc870\uc778 ' \uc0dd\uba85\uad8c \ubcf4\uc7a5 '\uc744 \uc704\ubc18\ud588\uc73c\uba70, \uad6d\ubbfc\uc758 \uc0dd\uba85\uacfc \uc548\uc804\uc744 \ubcf4\ud638\ud558\uae30 \uc704\ud55c \uc801\uadf9\uc801 \uc870\uce58\ub97c \ucde8\ud558\uc9c0 \uc54a\uc544 \uc9c1\ubb34 \uc720\uae30\uc5d0 \uac00\uae5d\ub2e4\uace0 \uc801\uc2dc\ud588\ub2e4.", "paragraph_sentence": "2016\ub144 12\uc6d4 3\uc77c \uc624\ud6c4 4\uc2dc 10\ubd84, \ub354\ubd88\uc5b4\ubbfc\uc8fc\ub2f9, \uad6d\ubbfc\uc758\ub2f9, \uc815\uc758\ub2f9\uacfc \ubb34\uc18c\uc18d \uc758\uc6d0 171\uba85\uc774 \ud5cc\ubc95\uacfc \ubc95\ub960 \uc704\ubc18\uc744 \uc774\uc720\ub85c \"\ub300\ud1b5\ub839(\ubc15\uadfc\ud61c) \ud0c4\ud575\uc18c\ucd94\uc548\"\uc744 \uad6d\ud68c\uc5d0\uc11c \ubc1c\uc758\ud588\ub2e4. \ub354\ubd88\uc5b4\ubbfc\uc8fc\ub2f9 \uc6b0\uc0c1\ud638, \uad6d\ubbfc\uc758\ub2f9 \ubc15\uc9c0\uc6d0, \uc815\uc758\ub2f9 \ub178\ud68c\ucc2c \uc758\uc6d0\uc774 \ub300\ud45c \ubc1c\uc758\ud588\uc73c\uba70, \uad6c\uccb4\uc801\uc778 \uc0ac\uc720\ub85c\ub294 \ud5cc\ubc95\uacfc \ubc95\ub960 \uc704\ubc18\uc774 \uc81c\uc2dc\ub418\uc5c8\ub2e4. \ud5cc\ubc95 \uc704\ubc18 \ud589\uc704\ub85c\ub294 \ucd5c\uc21c\uc2e4\uc744 \ube44\ub86f\ud55c \uce21\uadfc\ub4e4\uc774 \uc815\ucc45\uc5d0 \uac1c\uc785\ud558\uace0 \uad6d\ubb34\ud68c\uc758\uc5d0 \uc601\ud5a5\ub825\uc744 \ud589\uc0ac\ud1a0\ub85d \ud588\ub2e4\ub294 \uc810\uc5d0\uc11c \ub300\uc758\ubbfc\uc8fc\uc8fc\uc758 \uc758\ubb34\ub97c \uc704\ubc30\ud588\uc73c\uba70, \uc774\ub4e4\uc774 \uc778\uc0ac\uc5d0 \uac1c\uc785\ud558\uc5ec \uc9c1\uc5c5\uacf5\ubb34\uc6d0\uc81c \uc704\ubc18, \uc0ac\uae30\uc5c5\uc5d0 \uae08\ud488 \ucd9c\uc5f0\uc744 \uac15\uc694\ud558\uace0 \ub1cc\ubb3c\uc744 \uc218\uc218\ud588\ub2e4\ub294 \uc810\uc5d0\uc11c \uad6d\ubbfc \uc7ac\uc0b0\uad8c \ubcf4\uc7a5\u00b7\uc2dc\uc7a5\uacbd\uc81c\uc9c8\uc11c \ubc0f \ud5cc\ubc95\uc218\ud638 \uc758\ubb34\ub97c \uc704\ubc18\ud588\ub2e4\uace0 \uc9c0\uc801\ud588\ub2e4. \ub610\ud55c 2014\ub144 4\uc6d4 16\uc77c\uc5d0 \ubc1c\uc0dd\ud55c \uc138\uc6d4\ud638 \uce68\ubab0 \uc0ac\uace0\uc5d0 \ub300\ud55c \ub300\uc751 \uc2e4\ud328\ub85c \ud5cc\ubc95 10\uc870\uc778 ' \uc0dd\uba85\uad8c \ubcf4\uc7a5 '\uc744 \uc704\ubc18\ud588\uc73c\uba70, \uad6d\ubbfc\uc758 \uc0dd\uba85\uacfc \uc548\uc804\uc744 \ubcf4\ud638\ud558\uae30 \uc704\ud55c \uc801\uadf9\uc801 \uc870\uce58\ub97c \ucde8\ud558\uc9c0 \uc54a\uc544 \uc9c1\ubb34 \uc720\uae30\uc5d0 \uac00\uae5d\ub2e4\uace0 \uc801\uc2dc\ud588\ub2e4. \ubc95\ub960 \uc704\ubc18 \ud589\uc704\ub85c\ub294 \uc7ac\ub2e8\ubc95\uc778 \ubbf8\ub974\uc640 \uc7ac\ub2e8\ubc95\uc778 \ucf00\uc774\uc2a4\ud3ec\uce20\uc5d0 \uc0bc\uc131\uacfc SK, \ub86f\ub370 \ub4f1\uc758 \uae30\uc5c5\uc774 \ucd9c\uc5f0\ud55c 360\uc5b5 \uc6d0 \ub1cc\ubb3c\ub85c \ud310\ub2e8\ud588\uace0, \ub86f\ub370\uac00 70\uc5b5\uc6d0\uc744 \ucd94\uac00 \ucd9c\uc5f0\ud55c \uac83 \ub4f1\uc5d0 \ub300\ud574 \ub1cc\ubb3c\uc8c4\uc640 \uc9c1\uad8c\ub0a8\uc6a9, \uac15\uc694\uc8c4\ub97c \uc801\uc6a9\ud588\ub2e4.", "paragraph_answer": "2016\ub144 12\uc6d4 3\uc77c \uc624\ud6c4 4\uc2dc 10\ubd84, \ub354\ubd88\uc5b4\ubbfc\uc8fc\ub2f9, \uad6d\ubbfc\uc758\ub2f9, \uc815\uc758\ub2f9\uacfc \ubb34\uc18c\uc18d \uc758\uc6d0 171\uba85\uc774 \ud5cc\ubc95\uacfc \ubc95\ub960 \uc704\ubc18\uc744 \uc774\uc720\ub85c \"\ub300\ud1b5\ub839(\ubc15\uadfc\ud61c) \ud0c4\ud575\uc18c\ucd94\uc548\"\uc744 \uad6d\ud68c\uc5d0\uc11c \ubc1c\uc758\ud588\ub2e4. \ub354\ubd88\uc5b4\ubbfc\uc8fc\ub2f9 \uc6b0\uc0c1\ud638, \uad6d\ubbfc\uc758\ub2f9 \ubc15\uc9c0\uc6d0, \uc815\uc758\ub2f9 \ub178\ud68c\ucc2c \uc758\uc6d0\uc774 \ub300\ud45c \ubc1c\uc758\ud588\uc73c\uba70, \uad6c\uccb4\uc801\uc778 \uc0ac\uc720\ub85c\ub294 \ud5cc\ubc95\uacfc \ubc95\ub960 \uc704\ubc18\uc774 \uc81c\uc2dc\ub418\uc5c8\ub2e4. \ud5cc\ubc95 \uc704\ubc18 \ud589\uc704\ub85c\ub294 \ucd5c\uc21c\uc2e4\uc744 \ube44\ub86f\ud55c \uce21\uadfc\ub4e4\uc774 \uc815\ucc45\uc5d0 \uac1c\uc785\ud558\uace0 \uad6d\ubb34\ud68c\uc758\uc5d0 \uc601\ud5a5\ub825\uc744 \ud589\uc0ac\ud1a0\ub85d \ud588\ub2e4\ub294 \uc810\uc5d0\uc11c \ub300\uc758\ubbfc\uc8fc\uc8fc\uc758 \uc758\ubb34\ub97c \uc704\ubc30\ud588\uc73c\uba70, \uc774\ub4e4\uc774 \uc778\uc0ac\uc5d0 \uac1c\uc785\ud558\uc5ec \uc9c1\uc5c5\uacf5\ubb34\uc6d0\uc81c \uc704\ubc18, \uc0ac\uae30\uc5c5\uc5d0 \uae08\ud488 \ucd9c\uc5f0\uc744 \uac15\uc694\ud558\uace0 \ub1cc\ubb3c\uc744 \uc218\uc218\ud588\ub2e4\ub294 \uc810\uc5d0\uc11c \uad6d\ubbfc \uc7ac\uc0b0\uad8c \ubcf4\uc7a5\u00b7\uc2dc\uc7a5\uacbd\uc81c\uc9c8\uc11c \ubc0f \ud5cc\ubc95\uc218\ud638 \uc758\ubb34\ub97c \uc704\ubc18\ud588\ub2e4\uace0 \uc9c0\uc801\ud588\ub2e4. \ub610\ud55c 2014\ub144 4\uc6d4 16\uc77c\uc5d0 \ubc1c\uc0dd\ud55c \uc138\uc6d4\ud638 \uce68\ubab0 \uc0ac\uace0\uc5d0 \ub300\ud55c \ub300\uc751 \uc2e4\ud328\ub85c \ud5cc\ubc95 10\uc870\uc778 ' \uc0dd\uba85\uad8c \ubcf4\uc7a5 '\uc744 \uc704\ubc18\ud588\uc73c\uba70, \uad6d\ubbfc\uc758 \uc0dd\uba85\uacfc \uc548\uc804\uc744 \ubcf4\ud638\ud558\uae30 \uc704\ud55c \uc801\uadf9\uc801 \uc870\uce58\ub97c \ucde8\ud558\uc9c0 \uc54a\uc544 \uc9c1\ubb34 \uc720\uae30\uc5d0 \uac00\uae5d\ub2e4\uace0 \uc801\uc2dc\ud588\ub2e4. \ubc95\ub960 \uc704\ubc18 \ud589\uc704\ub85c\ub294 \uc7ac\ub2e8\ubc95\uc778 \ubbf8\ub974\uc640 \uc7ac\ub2e8\ubc95\uc778 \ucf00\uc774\uc2a4\ud3ec\uce20\uc5d0 \uc0bc\uc131\uacfc SK, \ub86f\ub370 \ub4f1\uc758 \uae30\uc5c5\uc774 \ucd9c\uc5f0\ud55c 360\uc5b5 \uc6d0 \ub1cc\ubb3c\ub85c \ud310\ub2e8\ud588\uace0, \ub86f\ub370\uac00 70\uc5b5\uc6d0\uc744 \ucd94\uac00 \ucd9c\uc5f0\ud55c \uac83 \ub4f1\uc5d0 \ub300\ud574 \ub1cc\ubb3c\uc8c4\uc640 \uc9c1\uad8c\ub0a8\uc6a9, \uac15\uc694\uc8c4\ub97c \uc801\uc6a9\ud588\ub2e4.", "sentence_answer": "\ub610\ud55c 2014\ub144 4\uc6d4 16\uc77c\uc5d0 \ubc1c\uc0dd\ud55c \uc138\uc6d4\ud638 \uce68\ubab0 \uc0ac\uace0\uc5d0 \ub300\ud55c \ub300\uc751 \uc2e4\ud328\ub85c \ud5cc\ubc95 10\uc870\uc778 ' \uc0dd\uba85\uad8c \ubcf4\uc7a5 '\uc744 \uc704\ubc18\ud588\uc73c\uba70, \uad6d\ubbfc\uc758 \uc0dd\uba85\uacfc \uc548\uc804\uc744 \ubcf4\ud638\ud558\uae30 \uc704\ud55c \uc801\uadf9\uc801 \uc870\uce58\ub97c \ucde8\ud558\uc9c0 \uc54a\uc544 \uc9c1\ubb34 \uc720\uae30\uc5d0 \uac00\uae5d\ub2e4\uace0 \uc801\uc2dc\ud588\ub2e4.", "paragraph_id": "6515809-1-1", "paragraph_question": "question: 2014\ub144 4\uc6d4 16\uc77c \ubc1c\uc0dd\ud55c \uc138\uc6d4\ud638 \uce68\ubab0 \uc0ac\uace0\uc5d0 \ub300\ud55c \ub300\uc751 \uc2e4\ud328\ub85c \ud5cc\ubc95 10\uc870\uc778 \ubb34\uc5c7\uc744 \uc704\ubc18\ud558\uc600\ub098?, context: 2016\ub144 12\uc6d4 3\uc77c \uc624\ud6c4 4\uc2dc 10\ubd84, \ub354\ubd88\uc5b4\ubbfc\uc8fc\ub2f9, \uad6d\ubbfc\uc758\ub2f9, \uc815\uc758\ub2f9\uacfc \ubb34\uc18c\uc18d \uc758\uc6d0 171\uba85\uc774 \ud5cc\ubc95\uacfc \ubc95\ub960 \uc704\ubc18\uc744 \uc774\uc720\ub85c \"\ub300\ud1b5\ub839(\ubc15\uadfc\ud61c) \ud0c4\ud575\uc18c\ucd94\uc548\"\uc744 \uad6d\ud68c\uc5d0\uc11c \ubc1c\uc758\ud588\ub2e4. \ub354\ubd88\uc5b4\ubbfc\uc8fc\ub2f9 \uc6b0\uc0c1\ud638, \uad6d\ubbfc\uc758\ub2f9 \ubc15\uc9c0\uc6d0, \uc815\uc758\ub2f9 \ub178\ud68c\ucc2c \uc758\uc6d0\uc774 \ub300\ud45c \ubc1c\uc758\ud588\uc73c\uba70, \uad6c\uccb4\uc801\uc778 \uc0ac\uc720\ub85c\ub294 \ud5cc\ubc95\uacfc \ubc95\ub960 \uc704\ubc18\uc774 \uc81c\uc2dc\ub418\uc5c8\ub2e4. \ud5cc\ubc95 \uc704\ubc18 \ud589\uc704\ub85c\ub294 \ucd5c\uc21c\uc2e4\uc744 \ube44\ub86f\ud55c \uce21\uadfc\ub4e4\uc774 \uc815\ucc45\uc5d0 \uac1c\uc785\ud558\uace0 \uad6d\ubb34\ud68c\uc758\uc5d0 \uc601\ud5a5\ub825\uc744 \ud589\uc0ac\ud1a0\ub85d \ud588\ub2e4\ub294 \uc810\uc5d0\uc11c \ub300\uc758\ubbfc\uc8fc\uc8fc\uc758 \uc758\ubb34\ub97c \uc704\ubc30\ud588\uc73c\uba70, \uc774\ub4e4\uc774 \uc778\uc0ac\uc5d0 \uac1c\uc785\ud558\uc5ec \uc9c1\uc5c5\uacf5\ubb34\uc6d0\uc81c \uc704\ubc18, \uc0ac\uae30\uc5c5\uc5d0 \uae08\ud488 \ucd9c\uc5f0\uc744 \uac15\uc694\ud558\uace0 \ub1cc\ubb3c\uc744 \uc218\uc218\ud588\ub2e4\ub294 \uc810\uc5d0\uc11c \uad6d\ubbfc \uc7ac\uc0b0\uad8c \ubcf4\uc7a5\u00b7\uc2dc\uc7a5\uacbd\uc81c\uc9c8\uc11c \ubc0f \ud5cc\ubc95\uc218\ud638 \uc758\ubb34\ub97c \uc704\ubc18\ud588\ub2e4\uace0 \uc9c0\uc801\ud588\ub2e4. \ub610\ud55c 2014\ub144 4\uc6d4 16\uc77c\uc5d0 \ubc1c\uc0dd\ud55c \uc138\uc6d4\ud638 \uce68\ubab0 \uc0ac\uace0\uc5d0 \ub300\ud55c \ub300\uc751 \uc2e4\ud328\ub85c \ud5cc\ubc95 10\uc870\uc778 '\uc0dd\uba85\uad8c \ubcf4\uc7a5'\uc744 \uc704\ubc18\ud588\uc73c\uba70, \uad6d\ubbfc\uc758 \uc0dd\uba85\uacfc \uc548\uc804\uc744 \ubcf4\ud638\ud558\uae30 \uc704\ud55c \uc801\uadf9\uc801 \uc870\uce58\ub97c \ucde8\ud558\uc9c0 \uc54a\uc544 \uc9c1\ubb34 \uc720\uae30\uc5d0 \uac00\uae5d\ub2e4\uace0 \uc801\uc2dc\ud588\ub2e4. \ubc95\ub960 \uc704\ubc18 \ud589\uc704\ub85c\ub294 \uc7ac\ub2e8\ubc95\uc778 \ubbf8\ub974\uc640 \uc7ac\ub2e8\ubc95\uc778 \ucf00\uc774\uc2a4\ud3ec\uce20\uc5d0 \uc0bc\uc131\uacfc SK, \ub86f\ub370 \ub4f1\uc758 \uae30\uc5c5\uc774 \ucd9c\uc5f0\ud55c 360\uc5b5 \uc6d0 \ub1cc\ubb3c\ub85c \ud310\ub2e8\ud588\uace0, \ub86f\ub370\uac00 70\uc5b5\uc6d0\uc744 \ucd94\uac00 \ucd9c\uc5f0\ud55c \uac83 \ub4f1\uc5d0 \ub300\ud574 \ub1cc\ubb3c\uc8c4\uc640 \uc9c1\uad8c\ub0a8\uc6a9, \uac15\uc694\uc8c4\ub97c \uc801\uc6a9\ud588\ub2e4."} {"question": "\ubbf8\uad6d \uad6d\ub9bd \uc608\uc220\uae30\uae08\uc774 1967\ub144 \ucc3d\ub9bd\ud55c \ub3c5\ub9bd \ube44\uc601\ub9ac\uae30\uad00\uc740 \ubb34\uc5c7\uc778\uac00?", "paragraph": "\ubca4\ud5c8\ub294 \ubbf8\uad6d \uc601\ud654 \uc5f0\uad6c\uc18c(AFI, \ubbf8\uad6d \uad6d\ub9bd \uc608\uc220\uae30\uae08\uc774 1967\ub144 \ucc3d\ub9bd\ud55c \ub3c5\ub9bd \ube44\uc601\ub9ac\uae30\uad00)\uac00 \uc120\uc815\ud55c \uc5ec\ub7ec '\ucd5c\uace0' \ubaa9\ub85d\uc5d0 \uc774\ub984\uc744 \uc62c\ub838\ub2e4. AFI \ubbf8\uad6d\uc601\ud654 100\ub144 \uc2dc\ub9ac\uc988\ub294 1500\uba85\uc774 \ub118\ub294 \uc608\uc220\uac00\u00b7\ud559\uc790\u00b7\ube44\ud3c9\uac00\u00b7\uc5ed\uc0ac\uac00\ub4e4\ub85c \uad6c\uc131\ub41c \ubc30\uc2ec\uc6d0\ub4e4\uc774 \ub2f9\ub300 \uc778\uae30\u00b7\uc5ed\uc0ac\uc801 \uc758\uc758\u00b7\ubb38\ud654\uc801 \uc601\ud5a5\ub825 \ub4f1\uc744 \uace0\ub824\ud558\uc5ec \uc120\uc815\ud55c \uc601\ud654 \ubaa9\ub85d\ub4e4\uc774\ub2e4. \uc774 \uc911 \ubca4\ud5c8\ub294 \u300a\ubbf8\uad6d\uc601\ud654 \ubca0\uc2a4\ud2b8 100\u300b\uc5d0\uc11c 72\uc704, \u300a\uc2a4\ub9b4\uc788\ub294 \uc601\ud654 \ubca0\uc2a4\ud2b8 100\u300b\uc5d0\uc11c 49\uc704, \u300a\uc601\ud654\uc74c\uc545 \ubca0\uc2a4\ud2b8 100\u300b\uc5d0\uc11c 21\uc704, \u300a\uac10\ub3d9\uc801\uc778 \uc601\ud654 \ubca0\uc2a4\ud2b8 100\u300b\uc5d0\uc11c 56\uc704, \u300a\uc11c\uc0ac\uc601\ud654 \ubca0\uc2a4\ud2b8 10\u300b\uc5d0\uc11c 2\uc704\uc5d0 \uc62c\ub790\ub2e4. \uc720\ub2e4 \ubca4\ud5c8\uc640 \uba54\uc0b4\ub77c\ub294 \u300a\ud788\uc5b4\ub85c\uc640 \uc545\uc5ed \ubca0\uc2a4\ud2b8 100\u300b\uc5d0 \uc120\uc815\ub418\uc5c8\ub2e4. 2004\ub144 \ubbf8\uad6d \uad6d\ub9bd \uc601\ud654\ubcf4\uc874\uc704\uc6d0\ud68c\ub294 \ubca4\ud5c8\ub97c \ubbf8\uad6d \uad6d\ub9bd\uc601\ud654 \ub4f1\ub85d\ubd80 \ub4f1\uc7ac\ub300\uc0c1\uc73c\ub85c \uc120\uc815\ud588\ub294\ub370 \uadf8 \uc774\uc720\ub97c '\ubb38\ud654\uc801\u00b7\uc5ed\uc0ac\uc801\u00b7\uc2ec\ubbf8\uc801\uc73c\ub85c \ud070 \uc758\ubbf8\uac00 \uc788\ub294 \uc601\ud654\uc774\uae30 \ub54c\ubb38.'\uc774\ub77c\uace0 \uae30\uc220\ud588\ub2e4.", "answer": "\ubbf8\uad6d \uc601\ud654 \uc5f0\uad6c\uc18c", "sentence": "\ubca4\ud5c8\ub294 \ubbf8\uad6d \uc601\ud654 \uc5f0\uad6c\uc18c (AFI, \ubbf8\uad6d \uad6d\ub9bd \uc608\uc220\uae30\uae08\uc774 1967\ub144 \ucc3d\ub9bd\ud55c \ub3c5\ub9bd \ube44\uc601\ub9ac\uae30\uad00)", "paragraph_sentence": " \ubca4\ud5c8\ub294 \ubbf8\uad6d \uc601\ud654 \uc5f0\uad6c\uc18c (AFI, \ubbf8\uad6d \uad6d\ub9bd \uc608\uc220\uae30\uae08\uc774 1967\ub144 \ucc3d\ub9bd\ud55c \ub3c5\ub9bd \ube44\uc601\ub9ac\uae30\uad00) \uac00 \uc120\uc815\ud55c \uc5ec\ub7ec '\ucd5c\uace0' \ubaa9\ub85d\uc5d0 \uc774\ub984\uc744 \uc62c\ub838\ub2e4. AFI \ubbf8\uad6d\uc601\ud654 100\ub144 \uc2dc\ub9ac\uc988\ub294 1500\uba85\uc774 \ub118\ub294 \uc608\uc220\uac00\u00b7\ud559\uc790\u00b7\ube44\ud3c9\uac00\u00b7\uc5ed\uc0ac\uac00\ub4e4\ub85c \uad6c\uc131\ub41c \ubc30\uc2ec\uc6d0\ub4e4\uc774 \ub2f9\ub300 \uc778\uae30\u00b7\uc5ed\uc0ac\uc801 \uc758\uc758\u00b7\ubb38\ud654\uc801 \uc601\ud5a5\ub825 \ub4f1\uc744 \uace0\ub824\ud558\uc5ec \uc120\uc815\ud55c \uc601\ud654 \ubaa9\ub85d\ub4e4\uc774\ub2e4. \uc774 \uc911 \ubca4\ud5c8\ub294 \u300a\ubbf8\uad6d\uc601\ud654 \ubca0\uc2a4\ud2b8 100\u300b\uc5d0\uc11c 72\uc704, \u300a\uc2a4\ub9b4\uc788\ub294 \uc601\ud654 \ubca0\uc2a4\ud2b8 100\u300b\uc5d0\uc11c 49\uc704, \u300a\uc601\ud654\uc74c\uc545 \ubca0\uc2a4\ud2b8 100\u300b\uc5d0\uc11c 21\uc704, \u300a\uac10\ub3d9\uc801\uc778 \uc601\ud654 \ubca0\uc2a4\ud2b8 100\u300b\uc5d0\uc11c 56\uc704, \u300a\uc11c\uc0ac\uc601\ud654 \ubca0\uc2a4\ud2b8 10\u300b\uc5d0\uc11c 2\uc704\uc5d0 \uc62c\ub790\ub2e4. \uc720\ub2e4 \ubca4\ud5c8\uc640 \uba54\uc0b4\ub77c\ub294 \u300a\ud788\uc5b4\ub85c\uc640 \uc545\uc5ed \ubca0\uc2a4\ud2b8 100\u300b\uc5d0 \uc120\uc815\ub418\uc5c8\ub2e4. 2004\ub144 \ubbf8\uad6d \uad6d\ub9bd \uc601\ud654\ubcf4\uc874\uc704\uc6d0\ud68c\ub294 \ubca4\ud5c8\ub97c \ubbf8\uad6d \uad6d\ub9bd\uc601\ud654 \ub4f1\ub85d\ubd80 \ub4f1\uc7ac\ub300\uc0c1\uc73c\ub85c \uc120\uc815\ud588\ub294\ub370 \uadf8 \uc774\uc720\ub97c '\ubb38\ud654\uc801\u00b7\uc5ed\uc0ac\uc801\u00b7\uc2ec\ubbf8\uc801\uc73c\ub85c \ud070 \uc758\ubbf8\uac00 \uc788\ub294 \uc601\ud654\uc774\uae30 \ub54c\ubb38. '\uc774\ub77c\uace0 \uae30\uc220\ud588\ub2e4.", "paragraph_answer": "\ubca4\ud5c8\ub294 \ubbf8\uad6d \uc601\ud654 \uc5f0\uad6c\uc18c (AFI, \ubbf8\uad6d \uad6d\ub9bd \uc608\uc220\uae30\uae08\uc774 1967\ub144 \ucc3d\ub9bd\ud55c \ub3c5\ub9bd \ube44\uc601\ub9ac\uae30\uad00)\uac00 \uc120\uc815\ud55c \uc5ec\ub7ec '\ucd5c\uace0' \ubaa9\ub85d\uc5d0 \uc774\ub984\uc744 \uc62c\ub838\ub2e4. AFI \ubbf8\uad6d\uc601\ud654 100\ub144 \uc2dc\ub9ac\uc988\ub294 1500\uba85\uc774 \ub118\ub294 \uc608\uc220\uac00\u00b7\ud559\uc790\u00b7\ube44\ud3c9\uac00\u00b7\uc5ed\uc0ac\uac00\ub4e4\ub85c \uad6c\uc131\ub41c \ubc30\uc2ec\uc6d0\ub4e4\uc774 \ub2f9\ub300 \uc778\uae30\u00b7\uc5ed\uc0ac\uc801 \uc758\uc758\u00b7\ubb38\ud654\uc801 \uc601\ud5a5\ub825 \ub4f1\uc744 \uace0\ub824\ud558\uc5ec \uc120\uc815\ud55c \uc601\ud654 \ubaa9\ub85d\ub4e4\uc774\ub2e4. \uc774 \uc911 \ubca4\ud5c8\ub294 \u300a\ubbf8\uad6d\uc601\ud654 \ubca0\uc2a4\ud2b8 100\u300b\uc5d0\uc11c 72\uc704, \u300a\uc2a4\ub9b4\uc788\ub294 \uc601\ud654 \ubca0\uc2a4\ud2b8 100\u300b\uc5d0\uc11c 49\uc704, \u300a\uc601\ud654\uc74c\uc545 \ubca0\uc2a4\ud2b8 100\u300b\uc5d0\uc11c 21\uc704, \u300a\uac10\ub3d9\uc801\uc778 \uc601\ud654 \ubca0\uc2a4\ud2b8 100\u300b\uc5d0\uc11c 56\uc704, \u300a\uc11c\uc0ac\uc601\ud654 \ubca0\uc2a4\ud2b8 10\u300b\uc5d0\uc11c 2\uc704\uc5d0 \uc62c\ub790\ub2e4. \uc720\ub2e4 \ubca4\ud5c8\uc640 \uba54\uc0b4\ub77c\ub294 \u300a\ud788\uc5b4\ub85c\uc640 \uc545\uc5ed \ubca0\uc2a4\ud2b8 100\u300b\uc5d0 \uc120\uc815\ub418\uc5c8\ub2e4. 2004\ub144 \ubbf8\uad6d \uad6d\ub9bd \uc601\ud654\ubcf4\uc874\uc704\uc6d0\ud68c\ub294 \ubca4\ud5c8\ub97c \ubbf8\uad6d \uad6d\ub9bd\uc601\ud654 \ub4f1\ub85d\ubd80 \ub4f1\uc7ac\ub300\uc0c1\uc73c\ub85c \uc120\uc815\ud588\ub294\ub370 \uadf8 \uc774\uc720\ub97c '\ubb38\ud654\uc801\u00b7\uc5ed\uc0ac\uc801\u00b7\uc2ec\ubbf8\uc801\uc73c\ub85c \ud070 \uc758\ubbf8\uac00 \uc788\ub294 \uc601\ud654\uc774\uae30 \ub54c\ubb38.'\uc774\ub77c\uace0 \uae30\uc220\ud588\ub2e4.", "sentence_answer": "\ubca4\ud5c8\ub294 \ubbf8\uad6d \uc601\ud654 \uc5f0\uad6c\uc18c (AFI, \ubbf8\uad6d \uad6d\ub9bd \uc608\uc220\uae30\uae08\uc774 1967\ub144 \ucc3d\ub9bd\ud55c \ub3c5\ub9bd \ube44\uc601\ub9ac\uae30\uad00)", "paragraph_id": "6516549-38-0", "paragraph_question": "question: \ubbf8\uad6d \uad6d\ub9bd \uc608\uc220\uae30\uae08\uc774 1967\ub144 \ucc3d\ub9bd\ud55c \ub3c5\ub9bd \ube44\uc601\ub9ac\uae30\uad00\uc740 \ubb34\uc5c7\uc778\uac00?, context: \ubca4\ud5c8\ub294 \ubbf8\uad6d \uc601\ud654 \uc5f0\uad6c\uc18c(AFI, \ubbf8\uad6d \uad6d\ub9bd \uc608\uc220\uae30\uae08\uc774 1967\ub144 \ucc3d\ub9bd\ud55c \ub3c5\ub9bd \ube44\uc601\ub9ac\uae30\uad00)\uac00 \uc120\uc815\ud55c \uc5ec\ub7ec '\ucd5c\uace0' \ubaa9\ub85d\uc5d0 \uc774\ub984\uc744 \uc62c\ub838\ub2e4. AFI \ubbf8\uad6d\uc601\ud654 100\ub144 \uc2dc\ub9ac\uc988\ub294 1500\uba85\uc774 \ub118\ub294 \uc608\uc220\uac00\u00b7\ud559\uc790\u00b7\ube44\ud3c9\uac00\u00b7\uc5ed\uc0ac\uac00\ub4e4\ub85c \uad6c\uc131\ub41c \ubc30\uc2ec\uc6d0\ub4e4\uc774 \ub2f9\ub300 \uc778\uae30\u00b7\uc5ed\uc0ac\uc801 \uc758\uc758\u00b7\ubb38\ud654\uc801 \uc601\ud5a5\ub825 \ub4f1\uc744 \uace0\ub824\ud558\uc5ec \uc120\uc815\ud55c \uc601\ud654 \ubaa9\ub85d\ub4e4\uc774\ub2e4. \uc774 \uc911 \ubca4\ud5c8\ub294 \u300a\ubbf8\uad6d\uc601\ud654 \ubca0\uc2a4\ud2b8 100\u300b\uc5d0\uc11c 72\uc704, \u300a\uc2a4\ub9b4\uc788\ub294 \uc601\ud654 \ubca0\uc2a4\ud2b8 100\u300b\uc5d0\uc11c 49\uc704, \u300a\uc601\ud654\uc74c\uc545 \ubca0\uc2a4\ud2b8 100\u300b\uc5d0\uc11c 21\uc704, \u300a\uac10\ub3d9\uc801\uc778 \uc601\ud654 \ubca0\uc2a4\ud2b8 100\u300b\uc5d0\uc11c 56\uc704, \u300a\uc11c\uc0ac\uc601\ud654 \ubca0\uc2a4\ud2b8 10\u300b\uc5d0\uc11c 2\uc704\uc5d0 \uc62c\ub790\ub2e4. \uc720\ub2e4 \ubca4\ud5c8\uc640 \uba54\uc0b4\ub77c\ub294 \u300a\ud788\uc5b4\ub85c\uc640 \uc545\uc5ed \ubca0\uc2a4\ud2b8 100\u300b\uc5d0 \uc120\uc815\ub418\uc5c8\ub2e4. 2004\ub144 \ubbf8\uad6d \uad6d\ub9bd \uc601\ud654\ubcf4\uc874\uc704\uc6d0\ud68c\ub294 \ubca4\ud5c8\ub97c \ubbf8\uad6d \uad6d\ub9bd\uc601\ud654 \ub4f1\ub85d\ubd80 \ub4f1\uc7ac\ub300\uc0c1\uc73c\ub85c \uc120\uc815\ud588\ub294\ub370 \uadf8 \uc774\uc720\ub97c '\ubb38\ud654\uc801\u00b7\uc5ed\uc0ac\uc801\u00b7\uc2ec\ubbf8\uc801\uc73c\ub85c \ud070 \uc758\ubbf8\uac00 \uc788\ub294 \uc601\ud654\uc774\uae30 \ub54c\ubb38.'\uc774\ub77c\uace0 \uae30\uc220\ud588\ub2e4."} {"question": "\uc544\uc2a4\uac00\ub974\ub4dc\uc758 \uc9c0\ub984\uc740?", "paragraph": "\uce7c\ub9ac\uc2a4\ud1a0 \ud45c\uba74\uc5d0\uc11c\uc758 \uac00\uc7a5 \ud070 \ucda9\ub3cc \uc9c0\ud615\uc740 \u201c\ub2e4\uc911 \ub300\uc57c\u201d(\ub300\uc57c \ubaa8\uc591\uc73c\ub85c \uc0dd\uae34 \uc5ec\ub7ec \uacb9\uc758 \ucda9\ub3cc\uad6c)\uc774\uace0, \uadf8 \uc911 2\uac1c\ub294 \ub9e4\uc6b0 \ud06c\ub2e4. \ubc1c\ud560\ub77c\ub294 \uac00\uc7a5 \ud06c\uba70, \ubc1d\uc740 \uc911\uc2ec \uc9c0\uc5ed\uc740 \uc9c0\ub984 600 km, \uace0\ub9ac\ub294 \uc911\uc2ec\uc73c\ub85c\ubd80\ud130 \ucd5c\uc7a5 1800 km \uac00\ub7c9 \ub5a8\uc5b4\uc838 \uc788\uc73c\uba70, \uc9c0\ub984\uc740 \uc57d 3800 km \uac00\ub7c9\uc774\ub2e4. \ub450 \ubc88\uc9f8\ub85c \ud070 \uac83\uc740 \uc544\uc2a4\uac00\ub974\ub4dc\uc774\uba70, \uc9c0\ub984\uc740 \uc57d 1600 km \uac00\ub7c9\uc73c\ub85c \uce21\uc815\ub41c\ub2e4. \ub2e4\ud658 \ucda9\ub3cc\uad6c\ub4e4\uc740 \ubd80\ub4dc\ub7fd\uac70\ub098 \uc561\uccb4(\ubc14\ub2e4)\ub85c \uad6c\uc131\ub41c \uce35 \uc704\uc5d0 \ub5a0 \uc788\ub294 \uc554\uad8c\uc5d0 \uc0dd\uae34 \ub3d9\uc2ec\uc6d0 \ubaa8\uc591\uc758 \uade0\uc5f4\uc5d0\uc11c \uc720\ub798\ud588\uc744 \uc218 \uc788\ub2e4. \uce74\ud14c\ub098(\uc608\uc2dc\ub85c \uace0\ubb3c \uce74\ud14c\ub098)\ub4e4\uc740 \ucda9\ub3cc\uad6c\ub4e4\uc774 \ud45c\uba74\uc744 \uac00\ub85c\uc9c8\ub7ec \ub098 \uc788\ub294 \uae34 \uc0ac\uc2ac\uc744 \uc758\ubbf8\ud55c\ub2e4. \uc774\ub4e4\uc740 \uc544\ub9c8\ub3c4 \ubaa9\uc131\uc5d0 \ub108\ubb34 \uadfc\uc811\ud558\uc5ec \uae30\uc870\ub825\uc5d0 \uc758\ud574 \uc0b0\uc0b0\ud788 \uc870\uac01\ub09c \ucc9c\uccb4\uac00 \uce7c\ub9ac\uc2a4\ud1a0\uc5d0 \ucda9\ub3cc\ud574\uc11c \uc0dd\uacbc\uac70\ub098, \ucc9c\uccb4\uac00 \ub9e4\uc6b0 \ube44\uc2a4\ub4ec\ud558\uac8c \ucda9\ub3cc\ud574\uc11c \uc0dd\uc131\ub418\uc5c8\uc744 \uac83\uc774\ub2e4. \uae30\uc870\ub825\uc5d0 \uc758\ud574 \ucc9c\uccb4\uac00 \uc0b0\uc0b0\uc870\uac01\ub098\ub294 \ud604\uc0c1\uc758 \uc608\uc2dc\ub85c\ub294 \uc288\uba54\uc774\ucee4-\ub808\ube44 \uc81c9\ud61c\uc131\uc774 \uc788\ub2e4.", "answer": "\uc57d 1600 km", "sentence": "\ub450 \ubc88\uc9f8\ub85c \ud070 \uac83\uc740 \uc544\uc2a4\uac00\ub974\ub4dc\uc774\uba70, \uc9c0\ub984\uc740 \uc57d 1600 km \uac00\ub7c9\uc73c\ub85c \uce21\uc815\ub41c\ub2e4.", "paragraph_sentence": "\uce7c\ub9ac\uc2a4\ud1a0 \ud45c\uba74\uc5d0\uc11c\uc758 \uac00\uc7a5 \ud070 \ucda9\ub3cc \uc9c0\ud615\uc740 \u201c\ub2e4\uc911 \ub300\uc57c\u201d(\ub300\uc57c \ubaa8\uc591\uc73c\ub85c \uc0dd\uae34 \uc5ec\ub7ec \uacb9\uc758 \ucda9\ub3cc\uad6c)\uc774\uace0, \uadf8 \uc911 2\uac1c\ub294 \ub9e4\uc6b0 \ud06c\ub2e4. \ubc1c\ud560\ub77c\ub294 \uac00\uc7a5 \ud06c\uba70, \ubc1d\uc740 \uc911\uc2ec \uc9c0\uc5ed\uc740 \uc9c0\ub984 600 km, \uace0\ub9ac\ub294 \uc911\uc2ec\uc73c\ub85c\ubd80\ud130 \ucd5c\uc7a5 1800 km \uac00\ub7c9 \ub5a8\uc5b4\uc838 \uc788\uc73c\uba70, \uc9c0\ub984\uc740 \uc57d 3800 km \uac00\ub7c9\uc774\ub2e4. \ub450 \ubc88\uc9f8\ub85c \ud070 \uac83\uc740 \uc544\uc2a4\uac00\ub974\ub4dc\uc774\uba70, \uc9c0\ub984\uc740 \uc57d 1600 km \uac00\ub7c9\uc73c\ub85c \uce21\uc815\ub41c\ub2e4. \ub2e4\ud658 \ucda9\ub3cc\uad6c\ub4e4\uc740 \ubd80\ub4dc\ub7fd\uac70\ub098 \uc561\uccb4(\ubc14\ub2e4)\ub85c \uad6c\uc131\ub41c \uce35 \uc704\uc5d0 \ub5a0 \uc788\ub294 \uc554\uad8c\uc5d0 \uc0dd\uae34 \ub3d9\uc2ec\uc6d0 \ubaa8\uc591\uc758 \uade0\uc5f4\uc5d0\uc11c \uc720\ub798\ud588\uc744 \uc218 \uc788\ub2e4. \uce74\ud14c\ub098(\uc608\uc2dc\ub85c \uace0\ubb3c \uce74\ud14c\ub098)\ub4e4\uc740 \ucda9\ub3cc\uad6c\ub4e4\uc774 \ud45c\uba74\uc744 \uac00\ub85c\uc9c8\ub7ec \ub098 \uc788\ub294 \uae34 \uc0ac\uc2ac\uc744 \uc758\ubbf8\ud55c\ub2e4. \uc774\ub4e4\uc740 \uc544\ub9c8\ub3c4 \ubaa9\uc131\uc5d0 \ub108\ubb34 \uadfc\uc811\ud558\uc5ec \uae30\uc870\ub825\uc5d0 \uc758\ud574 \uc0b0\uc0b0\ud788 \uc870\uac01\ub09c \ucc9c\uccb4\uac00 \uce7c\ub9ac\uc2a4\ud1a0\uc5d0 \ucda9\ub3cc\ud574\uc11c \uc0dd\uacbc\uac70\ub098, \ucc9c\uccb4\uac00 \ub9e4\uc6b0 \ube44\uc2a4\ub4ec\ud558\uac8c \ucda9\ub3cc\ud574\uc11c \uc0dd\uc131\ub418\uc5c8\uc744 \uac83\uc774\ub2e4. \uae30\uc870\ub825\uc5d0 \uc758\ud574 \ucc9c\uccb4\uac00 \uc0b0\uc0b0\uc870\uac01\ub098\ub294 \ud604\uc0c1\uc758 \uc608\uc2dc\ub85c\ub294 \uc288\uba54\uc774\ucee4-\ub808\ube44 \uc81c9\ud61c\uc131\uc774 \uc788\ub2e4.", "paragraph_answer": "\uce7c\ub9ac\uc2a4\ud1a0 \ud45c\uba74\uc5d0\uc11c\uc758 \uac00\uc7a5 \ud070 \ucda9\ub3cc \uc9c0\ud615\uc740 \u201c\ub2e4\uc911 \ub300\uc57c\u201d(\ub300\uc57c \ubaa8\uc591\uc73c\ub85c \uc0dd\uae34 \uc5ec\ub7ec \uacb9\uc758 \ucda9\ub3cc\uad6c)\uc774\uace0, \uadf8 \uc911 2\uac1c\ub294 \ub9e4\uc6b0 \ud06c\ub2e4. \ubc1c\ud560\ub77c\ub294 \uac00\uc7a5 \ud06c\uba70, \ubc1d\uc740 \uc911\uc2ec \uc9c0\uc5ed\uc740 \uc9c0\ub984 600 km, \uace0\ub9ac\ub294 \uc911\uc2ec\uc73c\ub85c\ubd80\ud130 \ucd5c\uc7a5 1800 km \uac00\ub7c9 \ub5a8\uc5b4\uc838 \uc788\uc73c\uba70, \uc9c0\ub984\uc740 \uc57d 3800 km \uac00\ub7c9\uc774\ub2e4. \ub450 \ubc88\uc9f8\ub85c \ud070 \uac83\uc740 \uc544\uc2a4\uac00\ub974\ub4dc\uc774\uba70, \uc9c0\ub984\uc740 \uc57d 1600 km \uac00\ub7c9\uc73c\ub85c \uce21\uc815\ub41c\ub2e4. \ub2e4\ud658 \ucda9\ub3cc\uad6c\ub4e4\uc740 \ubd80\ub4dc\ub7fd\uac70\ub098 \uc561\uccb4(\ubc14\ub2e4)\ub85c \uad6c\uc131\ub41c \uce35 \uc704\uc5d0 \ub5a0 \uc788\ub294 \uc554\uad8c\uc5d0 \uc0dd\uae34 \ub3d9\uc2ec\uc6d0 \ubaa8\uc591\uc758 \uade0\uc5f4\uc5d0\uc11c \uc720\ub798\ud588\uc744 \uc218 \uc788\ub2e4. \uce74\ud14c\ub098(\uc608\uc2dc\ub85c \uace0\ubb3c \uce74\ud14c\ub098)\ub4e4\uc740 \ucda9\ub3cc\uad6c\ub4e4\uc774 \ud45c\uba74\uc744 \uac00\ub85c\uc9c8\ub7ec \ub098 \uc788\ub294 \uae34 \uc0ac\uc2ac\uc744 \uc758\ubbf8\ud55c\ub2e4. \uc774\ub4e4\uc740 \uc544\ub9c8\ub3c4 \ubaa9\uc131\uc5d0 \ub108\ubb34 \uadfc\uc811\ud558\uc5ec \uae30\uc870\ub825\uc5d0 \uc758\ud574 \uc0b0\uc0b0\ud788 \uc870\uac01\ub09c \ucc9c\uccb4\uac00 \uce7c\ub9ac\uc2a4\ud1a0\uc5d0 \ucda9\ub3cc\ud574\uc11c \uc0dd\uacbc\uac70\ub098, \ucc9c\uccb4\uac00 \ub9e4\uc6b0 \ube44\uc2a4\ub4ec\ud558\uac8c \ucda9\ub3cc\ud574\uc11c \uc0dd\uc131\ub418\uc5c8\uc744 \uac83\uc774\ub2e4. \uae30\uc870\ub825\uc5d0 \uc758\ud574 \ucc9c\uccb4\uac00 \uc0b0\uc0b0\uc870\uac01\ub098\ub294 \ud604\uc0c1\uc758 \uc608\uc2dc\ub85c\ub294 \uc288\uba54\uc774\ucee4-\ub808\ube44 \uc81c9\ud61c\uc131\uc774 \uc788\ub2e4.", "sentence_answer": "\ub450 \ubc88\uc9f8\ub85c \ud070 \uac83\uc740 \uc544\uc2a4\uac00\ub974\ub4dc\uc774\uba70, \uc9c0\ub984\uc740 \uc57d 1600 km \uac00\ub7c9\uc73c\ub85c \uce21\uc815\ub41c\ub2e4.", "paragraph_id": "6208599-6-2", "paragraph_question": "question: \uc544\uc2a4\uac00\ub974\ub4dc\uc758 \uc9c0\ub984\uc740?, context: \uce7c\ub9ac\uc2a4\ud1a0 \ud45c\uba74\uc5d0\uc11c\uc758 \uac00\uc7a5 \ud070 \ucda9\ub3cc \uc9c0\ud615\uc740 \u201c\ub2e4\uc911 \ub300\uc57c\u201d(\ub300\uc57c \ubaa8\uc591\uc73c\ub85c \uc0dd\uae34 \uc5ec\ub7ec \uacb9\uc758 \ucda9\ub3cc\uad6c)\uc774\uace0, \uadf8 \uc911 2\uac1c\ub294 \ub9e4\uc6b0 \ud06c\ub2e4. \ubc1c\ud560\ub77c\ub294 \uac00\uc7a5 \ud06c\uba70, \ubc1d\uc740 \uc911\uc2ec \uc9c0\uc5ed\uc740 \uc9c0\ub984 600 km, \uace0\ub9ac\ub294 \uc911\uc2ec\uc73c\ub85c\ubd80\ud130 \ucd5c\uc7a5 1800 km \uac00\ub7c9 \ub5a8\uc5b4\uc838 \uc788\uc73c\uba70, \uc9c0\ub984\uc740 \uc57d 3800 km \uac00\ub7c9\uc774\ub2e4. \ub450 \ubc88\uc9f8\ub85c \ud070 \uac83\uc740 \uc544\uc2a4\uac00\ub974\ub4dc\uc774\uba70, \uc9c0\ub984\uc740 \uc57d 1600 km \uac00\ub7c9\uc73c\ub85c \uce21\uc815\ub41c\ub2e4. \ub2e4\ud658 \ucda9\ub3cc\uad6c\ub4e4\uc740 \ubd80\ub4dc\ub7fd\uac70\ub098 \uc561\uccb4(\ubc14\ub2e4)\ub85c \uad6c\uc131\ub41c \uce35 \uc704\uc5d0 \ub5a0 \uc788\ub294 \uc554\uad8c\uc5d0 \uc0dd\uae34 \ub3d9\uc2ec\uc6d0 \ubaa8\uc591\uc758 \uade0\uc5f4\uc5d0\uc11c \uc720\ub798\ud588\uc744 \uc218 \uc788\ub2e4. \uce74\ud14c\ub098(\uc608\uc2dc\ub85c \uace0\ubb3c \uce74\ud14c\ub098)\ub4e4\uc740 \ucda9\ub3cc\uad6c\ub4e4\uc774 \ud45c\uba74\uc744 \uac00\ub85c\uc9c8\ub7ec \ub098 \uc788\ub294 \uae34 \uc0ac\uc2ac\uc744 \uc758\ubbf8\ud55c\ub2e4. \uc774\ub4e4\uc740 \uc544\ub9c8\ub3c4 \ubaa9\uc131\uc5d0 \ub108\ubb34 \uadfc\uc811\ud558\uc5ec \uae30\uc870\ub825\uc5d0 \uc758\ud574 \uc0b0\uc0b0\ud788 \uc870\uac01\ub09c \ucc9c\uccb4\uac00 \uce7c\ub9ac\uc2a4\ud1a0\uc5d0 \ucda9\ub3cc\ud574\uc11c \uc0dd\uacbc\uac70\ub098, \ucc9c\uccb4\uac00 \ub9e4\uc6b0 \ube44\uc2a4\ub4ec\ud558\uac8c \ucda9\ub3cc\ud574\uc11c \uc0dd\uc131\ub418\uc5c8\uc744 \uac83\uc774\ub2e4. \uae30\uc870\ub825\uc5d0 \uc758\ud574 \ucc9c\uccb4\uac00 \uc0b0\uc0b0\uc870\uac01\ub098\ub294 \ud604\uc0c1\uc758 \uc608\uc2dc\ub85c\ub294 \uc288\uba54\uc774\ucee4-\ub808\ube44 \uc81c9\ud61c\uc131\uc774 \uc788\ub2e4."} {"question": "2009\ub144 \uc124\uce58\uac00 \uc2dc\uc791\ub418\uc5c8\uc73c\uba70, \uc88c\uc11d\uc774 180\ub3c4\ub85c \uc816\ud600\uc9c0\uace0 \uc88c\uc11d \uac04 \uc55e\ub4a4 \uac04\uaca9\uc774 \ud55c\uce35 \ub113\uc5b4\uc9c4 \ud504\ub808\uc2a4\ud2f0\uc9c0 \uc11d\uc740?", "paragraph": "\uc77c\ubc18\uc801\uc73c\ub85c \ube44\uc988\ub2c8\uc2a4\uc11d\uc774\ub77c\uace0 \ubd88\ub9ac\ub294 2\ub4f1\uc11d\uc740 \ub300\ud55c\ud56d\uacf5\uc5d0\uc11c\ub294 \ud504\ub808\uc2a4\ud2f0\uc9c0 \ud074\ub798\uc2a4\ub77c\uace0 \uce6d\ud574\uc9c0\uba70, 1980\ub144\ub300 \ubbf8\uc8fc\ub178\uc120 \ube44\uc988\ub2c8\uc2a4 \uc0ac\uc6a9\uc790\ub97c \uc704\ud558\uc5ec \uc0dd\uaca8\ub0ac\ub2e4. \ub610\ud55c 2006\ub144, \uc2e0 \ube44\uc988\ub2c8\uc2a4 \ud074\ub798\uc2a4\uc758 \ub3c4\uc785\uc73c\ub85c \uc774\ubbf8\uc9c0\ub97c \uc1c4\uc2e0\ud558\ub824\uace0 \ud558\uace0 \uc788\ub2e4. \ub124 \uc885\ub958\uc758 \ud504\ub808\uc2a4\ud2f0\uc9c0 \uc11d\uc774 \uc788\uc73c\uba70, \"\ud504\ub808\uc2a4\ud2f0\uc9c0 \uc2a4\uc704\ud2b8 \uc2dc\ud2b8\", \"\ud504\ub808\uc2a4\ud2f0\uc9c0 \uc2ac\ub9ac\ud37c \uc2dc\ud2b8\", \"\ud504\ub808\uc2a4\ud2f0\uc9c0 \ud50c\ub7ec\uc2a4 \uc2dc\ud2b8\", \"\ud504\ub808\uc2a4\ud2f0\uc9c0\uc11d\"\uc73c\ub85c \ubd84\ub958\ub41c\ub2e4. \"\ud504\ub808\uc2a4\ud2f0\uc9c0\uc11d\" \uc740 139\ub3c4 \uc816\ud600\uc9c0\uace0 \uc55e\ub4a4\uc88c\uc11d\uac04\uaca9\uc774 1.3\ubbf8\ud130\ub85c \ub113\uace0 \ud3b8\uc548\ud55c \uc11c\ube44\uc2a4\ub97c \uc81c\uacf5\ud55c\ub2e4. \ud504\ub808\uc2a4\ud2f0\uc9c0 \ud50c\ub7ec\uc2a4 \uc2dc\ud2b8\ub294 2006\ub144 \uc0c8\ub85c \ub3c4\uc785\ub41c \uc88c\uc11d\uc73c\ub85c \uac1c\uc778 \ud504\ub77c\uc774\ubc84\uc2dc\ub97c \ubcf4\uc7a5\ud558\ub294 \uce78\ub9c9\uc774\uac00 \uc124\uce58\ub418\uc5c8\uace0 \uc88c\uc11d\uc774 170\ub3c4\uae4c\uc9c0 \uc816\ud600\uc9c0\uba70, \ube44\ud589\uae30 \uc88c\uc11d \uace0\uae09\ud654\uc758 \uc77c\ud658\uc73c\ub85c \uc124\uce58\ub418\uc5c8\ub2e4. \ud604\uc7ac \ubbf8\uc8fc \ubc0f \uc720\ub7fd \ub2e4\uc218 \ub178\uc120\uc5d0 \uc11c\ube44\uc2a4 \uc911\uc774\ub2e4. \"\ud504\ub808\uc2a4\ud2f0\uc9c0 \uc2ac\ub9ac\ud37c \uc2dc\ud2b8\"\ub294 2009\ub144\ubd80\ud130 \uc124\uce58\uac00 \uc2dc\uc791\ub418\uc5c8\ub2e4. \uc88c\uc11d\uc774 180\ub3c4\ub85c \uc816\ud600\uc9c0\uace0 \uc55e\ub4a4 \uac04\uaca9\uc774 \uae30\uc874 \ud504\ub808\uc2a4\ud2f0\uc9c0\uc11d\ubcf4\ub2e4 66cm \ub354 \ub113\ub2e4. \"\ud504\ub808\uc2a4\ud2f0\uc9c0 \uc2a4\uc704\ud2b8 \uc2dc\ud2b8\"\ub294 2014\ub144\ubd80\ud130 \uc124\uce58\uac00 \uc2dc\uc791\ub418\uc5c8\uc73c\uba70 \uae30\uc874 \ud504\ub808\uc2a4\ud2f0\uc9c0\uc11d\ubcf4\ub2e4 LCD \ubaa8\ub2c8\ud130\uac00 \ub354 \ud06c\ub2e4.", "answer": "\ud504\ub808\uc2a4\ud2f0\uc9c0 \uc2ac\ub9ac\ud37c \uc2dc\ud2b8", "sentence": "\ub124 \uc885\ub958\uc758 \ud504\ub808\uc2a4\ud2f0\uc9c0 \uc11d\uc774 \uc788\uc73c\uba70, \"\ud504\ub808\uc2a4\ud2f0\uc9c0 \uc2a4\uc704\ud2b8 \uc2dc\ud2b8\", \" \ud504\ub808\uc2a4\ud2f0\uc9c0 \uc2ac\ub9ac\ud37c \uc2dc\ud2b8 \", \"\ud504\ub808\uc2a4\ud2f0\uc9c0 \ud50c\ub7ec\uc2a4 \uc2dc\ud2b8\", \"\ud504\ub808\uc2a4\ud2f0\uc9c0\uc11d\"\uc73c\ub85c \ubd84\ub958\ub41c\ub2e4.", "paragraph_sentence": "\uc77c\ubc18\uc801\uc73c\ub85c \ube44\uc988\ub2c8\uc2a4\uc11d\uc774\ub77c\uace0 \ubd88\ub9ac\ub294 2\ub4f1\uc11d\uc740 \ub300\ud55c\ud56d\uacf5\uc5d0\uc11c\ub294 \ud504\ub808\uc2a4\ud2f0\uc9c0 \ud074\ub798\uc2a4\ub77c\uace0 \uce6d\ud574\uc9c0\uba70, 1980\ub144\ub300 \ubbf8\uc8fc\ub178\uc120 \ube44\uc988\ub2c8\uc2a4 \uc0ac\uc6a9\uc790\ub97c \uc704\ud558\uc5ec \uc0dd\uaca8\ub0ac\ub2e4. \ub610\ud55c 2006\ub144, \uc2e0 \ube44\uc988\ub2c8\uc2a4 \ud074\ub798\uc2a4\uc758 \ub3c4\uc785\uc73c\ub85c \uc774\ubbf8\uc9c0\ub97c \uc1c4\uc2e0\ud558\ub824\uace0 \ud558\uace0 \uc788\ub2e4. \ub124 \uc885\ub958\uc758 \ud504\ub808\uc2a4\ud2f0\uc9c0 \uc11d\uc774 \uc788\uc73c\uba70, \"\ud504\ub808\uc2a4\ud2f0\uc9c0 \uc2a4\uc704\ud2b8 \uc2dc\ud2b8\", \" \ud504\ub808\uc2a4\ud2f0\uc9c0 \uc2ac\ub9ac\ud37c \uc2dc\ud2b8 \", \"\ud504\ub808\uc2a4\ud2f0\uc9c0 \ud50c\ub7ec\uc2a4 \uc2dc\ud2b8\", \"\ud504\ub808\uc2a4\ud2f0\uc9c0\uc11d\"\uc73c\ub85c \ubd84\ub958\ub41c\ub2e4. \"\ud504\ub808\uc2a4\ud2f0\uc9c0\uc11d\" \uc740 139\ub3c4 \uc816\ud600\uc9c0\uace0 \uc55e\ub4a4\uc88c\uc11d\uac04\uaca9\uc774 1.3\ubbf8\ud130\ub85c \ub113\uace0 \ud3b8\uc548\ud55c \uc11c\ube44\uc2a4\ub97c \uc81c\uacf5\ud55c\ub2e4. \ud504\ub808\uc2a4\ud2f0\uc9c0 \ud50c\ub7ec\uc2a4 \uc2dc\ud2b8\ub294 2006\ub144 \uc0c8\ub85c \ub3c4\uc785\ub41c \uc88c\uc11d\uc73c\ub85c \uac1c\uc778 \ud504\ub77c\uc774\ubc84\uc2dc\ub97c \ubcf4\uc7a5\ud558\ub294 \uce78\ub9c9\uc774\uac00 \uc124\uce58\ub418\uc5c8\uace0 \uc88c\uc11d\uc774 170\ub3c4\uae4c\uc9c0 \uc816\ud600\uc9c0\uba70, \ube44\ud589\uae30 \uc88c\uc11d \uace0\uae09\ud654\uc758 \uc77c\ud658\uc73c\ub85c \uc124\uce58\ub418\uc5c8\ub2e4. \ud604\uc7ac \ubbf8\uc8fc \ubc0f \uc720\ub7fd \ub2e4\uc218 \ub178\uc120\uc5d0 \uc11c\ube44\uc2a4 \uc911\uc774\ub2e4. \"\ud504\ub808\uc2a4\ud2f0\uc9c0 \uc2ac\ub9ac\ud37c \uc2dc\ud2b8\"\ub294 2009\ub144\ubd80\ud130 \uc124\uce58\uac00 \uc2dc\uc791\ub418\uc5c8\ub2e4. \uc88c\uc11d\uc774 180\ub3c4\ub85c \uc816\ud600\uc9c0\uace0 \uc55e\ub4a4 \uac04\uaca9\uc774 \uae30\uc874 \ud504\ub808\uc2a4\ud2f0\uc9c0\uc11d\ubcf4\ub2e4 66cm \ub354 \ub113\ub2e4. \"\ud504\ub808\uc2a4\ud2f0\uc9c0 \uc2a4\uc704\ud2b8 \uc2dc\ud2b8\"\ub294 2014\ub144\ubd80\ud130 \uc124\uce58\uac00 \uc2dc\uc791\ub418\uc5c8\uc73c\uba70 \uae30\uc874 \ud504\ub808\uc2a4\ud2f0\uc9c0\uc11d\ubcf4\ub2e4 LCD \ubaa8\ub2c8\ud130\uac00 \ub354 \ud06c\ub2e4.", "paragraph_answer": "\uc77c\ubc18\uc801\uc73c\ub85c \ube44\uc988\ub2c8\uc2a4\uc11d\uc774\ub77c\uace0 \ubd88\ub9ac\ub294 2\ub4f1\uc11d\uc740 \ub300\ud55c\ud56d\uacf5\uc5d0\uc11c\ub294 \ud504\ub808\uc2a4\ud2f0\uc9c0 \ud074\ub798\uc2a4\ub77c\uace0 \uce6d\ud574\uc9c0\uba70, 1980\ub144\ub300 \ubbf8\uc8fc\ub178\uc120 \ube44\uc988\ub2c8\uc2a4 \uc0ac\uc6a9\uc790\ub97c \uc704\ud558\uc5ec \uc0dd\uaca8\ub0ac\ub2e4. \ub610\ud55c 2006\ub144, \uc2e0 \ube44\uc988\ub2c8\uc2a4 \ud074\ub798\uc2a4\uc758 \ub3c4\uc785\uc73c\ub85c \uc774\ubbf8\uc9c0\ub97c \uc1c4\uc2e0\ud558\ub824\uace0 \ud558\uace0 \uc788\ub2e4. \ub124 \uc885\ub958\uc758 \ud504\ub808\uc2a4\ud2f0\uc9c0 \uc11d\uc774 \uc788\uc73c\uba70, \"\ud504\ub808\uc2a4\ud2f0\uc9c0 \uc2a4\uc704\ud2b8 \uc2dc\ud2b8\", \" \ud504\ub808\uc2a4\ud2f0\uc9c0 \uc2ac\ub9ac\ud37c \uc2dc\ud2b8 \", \"\ud504\ub808\uc2a4\ud2f0\uc9c0 \ud50c\ub7ec\uc2a4 \uc2dc\ud2b8\", \"\ud504\ub808\uc2a4\ud2f0\uc9c0\uc11d\"\uc73c\ub85c \ubd84\ub958\ub41c\ub2e4. \"\ud504\ub808\uc2a4\ud2f0\uc9c0\uc11d\" \uc740 139\ub3c4 \uc816\ud600\uc9c0\uace0 \uc55e\ub4a4\uc88c\uc11d\uac04\uaca9\uc774 1.3\ubbf8\ud130\ub85c \ub113\uace0 \ud3b8\uc548\ud55c \uc11c\ube44\uc2a4\ub97c \uc81c\uacf5\ud55c\ub2e4. \ud504\ub808\uc2a4\ud2f0\uc9c0 \ud50c\ub7ec\uc2a4 \uc2dc\ud2b8\ub294 2006\ub144 \uc0c8\ub85c \ub3c4\uc785\ub41c \uc88c\uc11d\uc73c\ub85c \uac1c\uc778 \ud504\ub77c\uc774\ubc84\uc2dc\ub97c \ubcf4\uc7a5\ud558\ub294 \uce78\ub9c9\uc774\uac00 \uc124\uce58\ub418\uc5c8\uace0 \uc88c\uc11d\uc774 170\ub3c4\uae4c\uc9c0 \uc816\ud600\uc9c0\uba70, \ube44\ud589\uae30 \uc88c\uc11d \uace0\uae09\ud654\uc758 \uc77c\ud658\uc73c\ub85c \uc124\uce58\ub418\uc5c8\ub2e4. \ud604\uc7ac \ubbf8\uc8fc \ubc0f \uc720\ub7fd \ub2e4\uc218 \ub178\uc120\uc5d0 \uc11c\ube44\uc2a4 \uc911\uc774\ub2e4. \"\ud504\ub808\uc2a4\ud2f0\uc9c0 \uc2ac\ub9ac\ud37c \uc2dc\ud2b8\"\ub294 2009\ub144\ubd80\ud130 \uc124\uce58\uac00 \uc2dc\uc791\ub418\uc5c8\ub2e4. \uc88c\uc11d\uc774 180\ub3c4\ub85c \uc816\ud600\uc9c0\uace0 \uc55e\ub4a4 \uac04\uaca9\uc774 \uae30\uc874 \ud504\ub808\uc2a4\ud2f0\uc9c0\uc11d\ubcf4\ub2e4 66cm \ub354 \ub113\ub2e4. \"\ud504\ub808\uc2a4\ud2f0\uc9c0 \uc2a4\uc704\ud2b8 \uc2dc\ud2b8\"\ub294 2014\ub144\ubd80\ud130 \uc124\uce58\uac00 \uc2dc\uc791\ub418\uc5c8\uc73c\uba70 \uae30\uc874 \ud504\ub808\uc2a4\ud2f0\uc9c0\uc11d\ubcf4\ub2e4 LCD \ubaa8\ub2c8\ud130\uac00 \ub354 \ud06c\ub2e4.", "sentence_answer": "\ub124 \uc885\ub958\uc758 \ud504\ub808\uc2a4\ud2f0\uc9c0 \uc11d\uc774 \uc788\uc73c\uba70, \"\ud504\ub808\uc2a4\ud2f0\uc9c0 \uc2a4\uc704\ud2b8 \uc2dc\ud2b8\", \" \ud504\ub808\uc2a4\ud2f0\uc9c0 \uc2ac\ub9ac\ud37c \uc2dc\ud2b8 \", \"\ud504\ub808\uc2a4\ud2f0\uc9c0 \ud50c\ub7ec\uc2a4 \uc2dc\ud2b8\", \"\ud504\ub808\uc2a4\ud2f0\uc9c0\uc11d\"\uc73c\ub85c \ubd84\ub958\ub41c\ub2e4.", "paragraph_id": "6491032-7-1", "paragraph_question": "question: 2009\ub144 \uc124\uce58\uac00 \uc2dc\uc791\ub418\uc5c8\uc73c\uba70, \uc88c\uc11d\uc774 180\ub3c4\ub85c \uc816\ud600\uc9c0\uace0 \uc88c\uc11d \uac04 \uc55e\ub4a4 \uac04\uaca9\uc774 \ud55c\uce35 \ub113\uc5b4\uc9c4 \ud504\ub808\uc2a4\ud2f0\uc9c0 \uc11d\uc740?, context: \uc77c\ubc18\uc801\uc73c\ub85c \ube44\uc988\ub2c8\uc2a4\uc11d\uc774\ub77c\uace0 \ubd88\ub9ac\ub294 2\ub4f1\uc11d\uc740 \ub300\ud55c\ud56d\uacf5\uc5d0\uc11c\ub294 \ud504\ub808\uc2a4\ud2f0\uc9c0 \ud074\ub798\uc2a4\ub77c\uace0 \uce6d\ud574\uc9c0\uba70, 1980\ub144\ub300 \ubbf8\uc8fc\ub178\uc120 \ube44\uc988\ub2c8\uc2a4 \uc0ac\uc6a9\uc790\ub97c \uc704\ud558\uc5ec \uc0dd\uaca8\ub0ac\ub2e4. \ub610\ud55c 2006\ub144, \uc2e0 \ube44\uc988\ub2c8\uc2a4 \ud074\ub798\uc2a4\uc758 \ub3c4\uc785\uc73c\ub85c \uc774\ubbf8\uc9c0\ub97c \uc1c4\uc2e0\ud558\ub824\uace0 \ud558\uace0 \uc788\ub2e4. \ub124 \uc885\ub958\uc758 \ud504\ub808\uc2a4\ud2f0\uc9c0 \uc11d\uc774 \uc788\uc73c\uba70, \"\ud504\ub808\uc2a4\ud2f0\uc9c0 \uc2a4\uc704\ud2b8 \uc2dc\ud2b8\", \"\ud504\ub808\uc2a4\ud2f0\uc9c0 \uc2ac\ub9ac\ud37c \uc2dc\ud2b8\", \"\ud504\ub808\uc2a4\ud2f0\uc9c0 \ud50c\ub7ec\uc2a4 \uc2dc\ud2b8\", \"\ud504\ub808\uc2a4\ud2f0\uc9c0\uc11d\"\uc73c\ub85c \ubd84\ub958\ub41c\ub2e4. \"\ud504\ub808\uc2a4\ud2f0\uc9c0\uc11d\" \uc740 139\ub3c4 \uc816\ud600\uc9c0\uace0 \uc55e\ub4a4\uc88c\uc11d\uac04\uaca9\uc774 1.3\ubbf8\ud130\ub85c \ub113\uace0 \ud3b8\uc548\ud55c \uc11c\ube44\uc2a4\ub97c \uc81c\uacf5\ud55c\ub2e4. \ud504\ub808\uc2a4\ud2f0\uc9c0 \ud50c\ub7ec\uc2a4 \uc2dc\ud2b8\ub294 2006\ub144 \uc0c8\ub85c \ub3c4\uc785\ub41c \uc88c\uc11d\uc73c\ub85c \uac1c\uc778 \ud504\ub77c\uc774\ubc84\uc2dc\ub97c \ubcf4\uc7a5\ud558\ub294 \uce78\ub9c9\uc774\uac00 \uc124\uce58\ub418\uc5c8\uace0 \uc88c\uc11d\uc774 170\ub3c4\uae4c\uc9c0 \uc816\ud600\uc9c0\uba70, \ube44\ud589\uae30 \uc88c\uc11d \uace0\uae09\ud654\uc758 \uc77c\ud658\uc73c\ub85c \uc124\uce58\ub418\uc5c8\ub2e4. \ud604\uc7ac \ubbf8\uc8fc \ubc0f \uc720\ub7fd \ub2e4\uc218 \ub178\uc120\uc5d0 \uc11c\ube44\uc2a4 \uc911\uc774\ub2e4. \"\ud504\ub808\uc2a4\ud2f0\uc9c0 \uc2ac\ub9ac\ud37c \uc2dc\ud2b8\"\ub294 2009\ub144\ubd80\ud130 \uc124\uce58\uac00 \uc2dc\uc791\ub418\uc5c8\ub2e4. \uc88c\uc11d\uc774 180\ub3c4\ub85c \uc816\ud600\uc9c0\uace0 \uc55e\ub4a4 \uac04\uaca9\uc774 \uae30\uc874 \ud504\ub808\uc2a4\ud2f0\uc9c0\uc11d\ubcf4\ub2e4 66cm \ub354 \ub113\ub2e4. \"\ud504\ub808\uc2a4\ud2f0\uc9c0 \uc2a4\uc704\ud2b8 \uc2dc\ud2b8\"\ub294 2014\ub144\ubd80\ud130 \uc124\uce58\uac00 \uc2dc\uc791\ub418\uc5c8\uc73c\uba70 \uae30\uc874 \ud504\ub808\uc2a4\ud2f0\uc9c0\uc11d\ubcf4\ub2e4 LCD \ubaa8\ub2c8\ud130\uac00 \ub354 \ud06c\ub2e4."} {"question": "\ub4dc\ub77c\ub9c8 \uce58\uc988\uc778\ub354\ud2b8\ub7a9\uc740 \uba87 \ub144\ub3c4\uc5d0 \ubc29\uc601\ub410\ub294\uac00?", "paragraph": "\u300a\uce58\uc988\uc778\ub354\ud2b8\ub7a9\u300b\uc740 2016\ub144 1\uc6d4 4\uc77c\ubd80\ud130 2016\ub144 3\uc6d4 1\uc77c\uae4c\uc9c0 tvN\uc5d0\uc11c \ubc29\uc601\ub41c \uc6d4\ud654\ub4dc\ub77c\ub9c8\uc774\ub2e4. \uc21c\ub07c\uc758 \ub3d9\uba85 \uc6f9\ud230\uc774 \uc6d0\uc791\uc774\ub2e4. \ud558\uc9c0\ub9cc \uc6d0\uc791\uacfc\uc758 \uad34\ub9ac\uac00 \uc5c4\uccad\ub098\uac8c \ud070\uc9c0\ub77c \ubc29\uc601\ucd08\uae30\ubd80\ud130 \ub9d0\uc774 \ub9ce\uc558\uace0 \uadf9\uc744 \uc774\ub04c\uc5b4\ub098\uac00\uc57c\ud560 \uc8fc\uc5ed\ub4e4 \ub610\ud55c \uc720\uc815 \uc5ed\uc758 \ubc15\ud574\uc9c4\uc744 \uc81c\uc678\ud558\uace0\ub294 \ubaa8\ub450 \uc5f0\uae30\ub825 \ub17c\ub780\uc774 \uc77c\uba70 \ud639\ud3c9\uc774 \uc3df\uc544\uc9c0\uae30 \uc2dc\uc791\ud588\ub2e4. \uac8c\ub2e4\uac00 \uadf9\ud6c4\ubc18\uc73c\ub85c \uac08\uc218\ub85d \uba54\uc778\ub0a8\uc8fc \uc720\uc815\uc758 \ube44\uc911\uc774 \uc904\uc5b4\ub4e4\uace0 \uc11c\ube0c\ub0a8\uc8fc \ubc31\uc778\ud638\uc758 \ubd84\ub7c9\uc774 \uae09\uaca9\ud558\uac8c \ub9ce\uc544\uc9c0\uae30 \uc2dc\uc791\ud588\ub2e4. \ubb38\uc81c\ub294 \uc774\ub7ec\ud55c \uac01\uc0c9\uc774 \ub2e8\uc21c\ud788 \uc11c\ube0c\ub0a8\uc8fc\ub97c \ubc00\uc5b4\uc8fc\uae30 \uc704\ud55c \uc57d\uac04\uc758 \uac01\uc0c9 \uc815\ub3c4\uac00 \uc544\ub2c8\ub77c \uc544\uc608 \uc5ec\uc8fc\uc640 \ub0a8\uc8fc\uc758 \uad00\uacc4\ub97c \ub2e8\uc808\uc2dc\ucf1c\ubc84\ub9ac\ub294 \uacfc\ub3c4\ud55c \uc11c\ube0c\ub0a8\uc8fc \ubc00\uc5b4\uc8fc\uae30 \uc218\uc900\uc774\ub77c\ub294 \uc810\uc774\uc5c8\ub2e4. \uc774\ub7ec\ud55c \uc545\ud3c9\ub4e4\ub85c \uc778\ud574 \ub4dc\ub77c\ub9c8\ub294 \uc6d0\uc791 \ud0c0\uc774\ud2c0\uc774 \ubb34\uc0c9\ud558\uac8c \ucc38\ud328\ud588\ub2e4.", "answer": "2016\ub144", "sentence": "\u300a\uce58\uc988\uc778\ub354\ud2b8\ub7a9\u300b\uc740 2016\ub144 1\uc6d4 4\uc77c\ubd80\ud130 2016\ub144 3\uc6d4 1\uc77c\uae4c\uc9c0 tvN\uc5d0\uc11c \ubc29\uc601\ub41c \uc6d4\ud654\ub4dc\ub77c\ub9c8\uc774\ub2e4.", "paragraph_sentence": " \u300a\uce58\uc988\uc778\ub354\ud2b8\ub7a9\u300b\uc740 2016\ub144 1\uc6d4 4\uc77c\ubd80\ud130 2016\ub144 3\uc6d4 1\uc77c\uae4c\uc9c0 tvN\uc5d0\uc11c \ubc29\uc601\ub41c \uc6d4\ud654\ub4dc\ub77c\ub9c8\uc774\ub2e4. \uc21c\ub07c\uc758 \ub3d9\uba85 \uc6f9\ud230\uc774 \uc6d0\uc791\uc774\ub2e4. \ud558\uc9c0\ub9cc \uc6d0\uc791\uacfc\uc758 \uad34\ub9ac\uac00 \uc5c4\uccad\ub098\uac8c \ud070\uc9c0\ub77c \ubc29\uc601\ucd08\uae30\ubd80\ud130 \ub9d0\uc774 \ub9ce\uc558\uace0 \uadf9\uc744 \uc774\ub04c\uc5b4\ub098\uac00\uc57c\ud560 \uc8fc\uc5ed\ub4e4 \ub610\ud55c \uc720\uc815 \uc5ed\uc758 \ubc15\ud574\uc9c4\uc744 \uc81c\uc678\ud558\uace0\ub294 \ubaa8\ub450 \uc5f0\uae30\ub825 \ub17c\ub780\uc774 \uc77c\uba70 \ud639\ud3c9\uc774 \uc3df\uc544\uc9c0\uae30 \uc2dc\uc791\ud588\ub2e4. \uac8c\ub2e4\uac00 \uadf9\ud6c4\ubc18\uc73c\ub85c \uac08\uc218\ub85d \uba54\uc778\ub0a8\uc8fc \uc720\uc815\uc758 \ube44\uc911\uc774 \uc904\uc5b4\ub4e4\uace0 \uc11c\ube0c\ub0a8\uc8fc \ubc31\uc778\ud638\uc758 \ubd84\ub7c9\uc774 \uae09\uaca9\ud558\uac8c \ub9ce\uc544\uc9c0\uae30 \uc2dc\uc791\ud588\ub2e4. \ubb38\uc81c\ub294 \uc774\ub7ec\ud55c \uac01\uc0c9\uc774 \ub2e8\uc21c\ud788 \uc11c\ube0c\ub0a8\uc8fc\ub97c \ubc00\uc5b4\uc8fc\uae30 \uc704\ud55c \uc57d\uac04\uc758 \uac01\uc0c9 \uc815\ub3c4\uac00 \uc544\ub2c8\ub77c \uc544\uc608 \uc5ec\uc8fc\uc640 \ub0a8\uc8fc\uc758 \uad00\uacc4\ub97c \ub2e8\uc808\uc2dc\ucf1c\ubc84\ub9ac\ub294 \uacfc\ub3c4\ud55c \uc11c\ube0c\ub0a8\uc8fc \ubc00\uc5b4\uc8fc\uae30 \uc218\uc900\uc774\ub77c\ub294 \uc810\uc774\uc5c8\ub2e4. \uc774\ub7ec\ud55c \uc545\ud3c9\ub4e4\ub85c \uc778\ud574 \ub4dc\ub77c\ub9c8\ub294 \uc6d0\uc791 \ud0c0\uc774\ud2c0\uc774 \ubb34\uc0c9\ud558\uac8c \ucc38\ud328\ud588\ub2e4.", "paragraph_answer": "\u300a\uce58\uc988\uc778\ub354\ud2b8\ub7a9\u300b\uc740 2016\ub144 1\uc6d4 4\uc77c\ubd80\ud130 2016\ub144 3\uc6d4 1\uc77c\uae4c\uc9c0 tvN\uc5d0\uc11c \ubc29\uc601\ub41c \uc6d4\ud654\ub4dc\ub77c\ub9c8\uc774\ub2e4. \uc21c\ub07c\uc758 \ub3d9\uba85 \uc6f9\ud230\uc774 \uc6d0\uc791\uc774\ub2e4. \ud558\uc9c0\ub9cc \uc6d0\uc791\uacfc\uc758 \uad34\ub9ac\uac00 \uc5c4\uccad\ub098\uac8c \ud070\uc9c0\ub77c \ubc29\uc601\ucd08\uae30\ubd80\ud130 \ub9d0\uc774 \ub9ce\uc558\uace0 \uadf9\uc744 \uc774\ub04c\uc5b4\ub098\uac00\uc57c\ud560 \uc8fc\uc5ed\ub4e4 \ub610\ud55c \uc720\uc815 \uc5ed\uc758 \ubc15\ud574\uc9c4\uc744 \uc81c\uc678\ud558\uace0\ub294 \ubaa8\ub450 \uc5f0\uae30\ub825 \ub17c\ub780\uc774 \uc77c\uba70 \ud639\ud3c9\uc774 \uc3df\uc544\uc9c0\uae30 \uc2dc\uc791\ud588\ub2e4. \uac8c\ub2e4\uac00 \uadf9\ud6c4\ubc18\uc73c\ub85c \uac08\uc218\ub85d \uba54\uc778\ub0a8\uc8fc \uc720\uc815\uc758 \ube44\uc911\uc774 \uc904\uc5b4\ub4e4\uace0 \uc11c\ube0c\ub0a8\uc8fc \ubc31\uc778\ud638\uc758 \ubd84\ub7c9\uc774 \uae09\uaca9\ud558\uac8c \ub9ce\uc544\uc9c0\uae30 \uc2dc\uc791\ud588\ub2e4. \ubb38\uc81c\ub294 \uc774\ub7ec\ud55c \uac01\uc0c9\uc774 \ub2e8\uc21c\ud788 \uc11c\ube0c\ub0a8\uc8fc\ub97c \ubc00\uc5b4\uc8fc\uae30 \uc704\ud55c \uc57d\uac04\uc758 \uac01\uc0c9 \uc815\ub3c4\uac00 \uc544\ub2c8\ub77c \uc544\uc608 \uc5ec\uc8fc\uc640 \ub0a8\uc8fc\uc758 \uad00\uacc4\ub97c \ub2e8\uc808\uc2dc\ucf1c\ubc84\ub9ac\ub294 \uacfc\ub3c4\ud55c \uc11c\ube0c\ub0a8\uc8fc \ubc00\uc5b4\uc8fc\uae30 \uc218\uc900\uc774\ub77c\ub294 \uc810\uc774\uc5c8\ub2e4. \uc774\ub7ec\ud55c \uc545\ud3c9\ub4e4\ub85c \uc778\ud574 \ub4dc\ub77c\ub9c8\ub294 \uc6d0\uc791 \ud0c0\uc774\ud2c0\uc774 \ubb34\uc0c9\ud558\uac8c \ucc38\ud328\ud588\ub2e4.", "sentence_answer": "\u300a\uce58\uc988\uc778\ub354\ud2b8\ub7a9\u300b\uc740 2016\ub144 1\uc6d4 4\uc77c\ubd80\ud130 2016\ub144 3\uc6d4 1\uc77c\uae4c\uc9c0 tvN\uc5d0\uc11c \ubc29\uc601\ub41c \uc6d4\ud654\ub4dc\ub77c\ub9c8\uc774\ub2e4.", "paragraph_id": "6560836-0-0", "paragraph_question": "question: \ub4dc\ub77c\ub9c8 \uce58\uc988\uc778\ub354\ud2b8\ub7a9\uc740 \uba87 \ub144\ub3c4\uc5d0 \ubc29\uc601\ub410\ub294\uac00?, context: \u300a\uce58\uc988\uc778\ub354\ud2b8\ub7a9\u300b\uc740 2016\ub144 1\uc6d4 4\uc77c\ubd80\ud130 2016\ub144 3\uc6d4 1\uc77c\uae4c\uc9c0 tvN\uc5d0\uc11c \ubc29\uc601\ub41c \uc6d4\ud654\ub4dc\ub77c\ub9c8\uc774\ub2e4. \uc21c\ub07c\uc758 \ub3d9\uba85 \uc6f9\ud230\uc774 \uc6d0\uc791\uc774\ub2e4. \ud558\uc9c0\ub9cc \uc6d0\uc791\uacfc\uc758 \uad34\ub9ac\uac00 \uc5c4\uccad\ub098\uac8c \ud070\uc9c0\ub77c \ubc29\uc601\ucd08\uae30\ubd80\ud130 \ub9d0\uc774 \ub9ce\uc558\uace0 \uadf9\uc744 \uc774\ub04c\uc5b4\ub098\uac00\uc57c\ud560 \uc8fc\uc5ed\ub4e4 \ub610\ud55c \uc720\uc815 \uc5ed\uc758 \ubc15\ud574\uc9c4\uc744 \uc81c\uc678\ud558\uace0\ub294 \ubaa8\ub450 \uc5f0\uae30\ub825 \ub17c\ub780\uc774 \uc77c\uba70 \ud639\ud3c9\uc774 \uc3df\uc544\uc9c0\uae30 \uc2dc\uc791\ud588\ub2e4. \uac8c\ub2e4\uac00 \uadf9\ud6c4\ubc18\uc73c\ub85c \uac08\uc218\ub85d \uba54\uc778\ub0a8\uc8fc \uc720\uc815\uc758 \ube44\uc911\uc774 \uc904\uc5b4\ub4e4\uace0 \uc11c\ube0c\ub0a8\uc8fc \ubc31\uc778\ud638\uc758 \ubd84\ub7c9\uc774 \uae09\uaca9\ud558\uac8c \ub9ce\uc544\uc9c0\uae30 \uc2dc\uc791\ud588\ub2e4. \ubb38\uc81c\ub294 \uc774\ub7ec\ud55c \uac01\uc0c9\uc774 \ub2e8\uc21c\ud788 \uc11c\ube0c\ub0a8\uc8fc\ub97c \ubc00\uc5b4\uc8fc\uae30 \uc704\ud55c \uc57d\uac04\uc758 \uac01\uc0c9 \uc815\ub3c4\uac00 \uc544\ub2c8\ub77c \uc544\uc608 \uc5ec\uc8fc\uc640 \ub0a8\uc8fc\uc758 \uad00\uacc4\ub97c \ub2e8\uc808\uc2dc\ucf1c\ubc84\ub9ac\ub294 \uacfc\ub3c4\ud55c \uc11c\ube0c\ub0a8\uc8fc \ubc00\uc5b4\uc8fc\uae30 \uc218\uc900\uc774\ub77c\ub294 \uc810\uc774\uc5c8\ub2e4. \uc774\ub7ec\ud55c \uc545\ud3c9\ub4e4\ub85c \uc778\ud574 \ub4dc\ub77c\ub9c8\ub294 \uc6d0\uc791 \ud0c0\uc774\ud2c0\uc774 \ubb34\uc0c9\ud558\uac8c \ucc38\ud328\ud588\ub2e4."} {"question": "\uc6b0\uc8fc\uc655\ubcf5\uc120\uc73c\ub85c \ud504\ub85d\uc2dc\ub9c8\uae4c\uc9c0 \uac00\ub294 \uc2dc\uac04\uc740?", "paragraph": "\ud56d\uc131 \uc790\uccb4\ub294 \uc9c0\uad6c\uc5d0 \ube44\ud558\uba74 \uc555\ub3c4\uc801\uc73c\ub85c \ubb34\uac81\uace0 \ubc1d\uc740 \uc874\uc7ac\uc774\uc9c0\ub9cc \ud56d\uc131\uacfc \ud56d\uc131 \uc0ac\uc774\ub294 \uc5c4\uccad\ub098\uac8c \ub5a8\uc5b4\uc838 \uc788\ub2e4. \ud0dc\uc591\uc744 \uc81c\uc678\ud558\uace0 \uc9c0\uad6c\uc5d0\uc11c \uac00\uc7a5 \uac00\uae4c\uc6b4 \ud56d\uc131\uc740 \ucf04\ud0c0\uc6b0\ub8e8\uc2a4\uc790\ub9ac \ud504\ub85d\uc2dc\ub9c8\ub85c \ube5b\uc758 \uc18d\ub3c4\ub85c 4.2\ub144\uc774 \uac78\ub9ac\ub294 \uac70\ub9ac\uc5d0 \uc788\ub2e4. \uc774 \uac70\ub9ac\ub294 39\uc870 9\ucc9c\uc5b5 \ud0ac\ub85c\ubbf8\ud130\uc5d0 \uc774\ub974\uba70, \ud0dc\uc591\uc5d0\uc11c \uba85\uc655\uc131\uae4c\uc9c0 \uac70\ub9ac\uc758 6000\ubc30\uc5d0 \uc774\ub974\ub294 \uac04\uaca9\uc774\ub2e4. \ud504\ub85d\uc2dc\ub9c8\uc5d0\uc11c \ub5a0\ub09c \ube5b\uc774 \uc9c0\uad6c\uc5d0 \uc774\ub974\uae30\uae4c\uc9c0\ub294 4.2\ub144\uc774 \uac78\ub9b0\ub2e4. \uc6b0\uc8fc \uc655\ubcf5\uc120\uc758 \uc9c0\uad6c \uada4\ub3c4\uc120\uc0c1 \uc18d\ub3c4(\uc2dc\uc18d 3\ub9cc \ud0ac\ub85c\ubbf8\ud130)\ub85c \ud504\ub85d\uc2dc\ub9c8\uae4c\uc9c0 \uac00\ub824\uba74 15\ub9cc \ub144\uc774 \uac78\ub9b0\ub2e4. \uc774 \uc815\ub3c4\ub294 \uc740\ud558\uba74\uc5d0 \uc874\uc7ac\ud558\ub294 \ubcc4 \uc0ac\uc774\uc5d0\uc11c\ub294 \ud3c9\uade0\ubcf4\ub2e4 \uc57d\uac04 \uac00\uae4c\uc6b4 \uac70\ub9ac\uc5d0 \ud574\ub2f9\ud55c\ub2e4. \uc740\ud558 \uc911\uc2ec \ub610\ub294 \uad6c\uc0c1 \uc131\ub2e8\uc5d0 \uc788\ub294 \ubcc4\ub4e4 \uc0ac\uc774\uc758 \uac70\ub9ac\ub294 \ud6e8\uc52c \uac00\uae5d\uace0, \uc740\ud558 \ud5e4\uc77c\ub85c\uc5d0 \uc788\ub294 \ubcc4\ub4e4 \uc0ac\uc774\uc758 \uac70\ub9ac\ub294 \ud6e8\uc52c \uba40\ub2e4.", "answer": "15\ub9cc \ub144", "sentence": "\uc6b0\uc8fc \uc655\ubcf5\uc120\uc758 \uc9c0\uad6c \uada4\ub3c4\uc120\uc0c1 \uc18d\ub3c4(\uc2dc\uc18d 3\ub9cc \ud0ac\ub85c\ubbf8\ud130)\ub85c \ud504\ub85d\uc2dc\ub9c8\uae4c\uc9c0 \uac00\ub824\uba74 15\ub9cc \ub144 \uc774 \uac78\ub9b0\ub2e4.", "paragraph_sentence": "\ud56d\uc131 \uc790\uccb4\ub294 \uc9c0\uad6c\uc5d0 \ube44\ud558\uba74 \uc555\ub3c4\uc801\uc73c\ub85c \ubb34\uac81\uace0 \ubc1d\uc740 \uc874\uc7ac\uc774\uc9c0\ub9cc \ud56d\uc131\uacfc \ud56d\uc131 \uc0ac\uc774\ub294 \uc5c4\uccad\ub098\uac8c \ub5a8\uc5b4\uc838 \uc788\ub2e4. \ud0dc\uc591\uc744 \uc81c\uc678\ud558\uace0 \uc9c0\uad6c\uc5d0\uc11c \uac00\uc7a5 \uac00\uae4c\uc6b4 \ud56d\uc131\uc740 \ucf04\ud0c0\uc6b0\ub8e8\uc2a4\uc790\ub9ac \ud504\ub85d\uc2dc\ub9c8\ub85c \ube5b\uc758 \uc18d\ub3c4\ub85c 4.2\ub144\uc774 \uac78\ub9ac\ub294 \uac70\ub9ac\uc5d0 \uc788\ub2e4. \uc774 \uac70\ub9ac\ub294 39\uc870 9\ucc9c\uc5b5 \ud0ac\ub85c\ubbf8\ud130\uc5d0 \uc774\ub974\uba70, \ud0dc\uc591\uc5d0\uc11c \uba85\uc655\uc131\uae4c\uc9c0 \uac70\ub9ac\uc758 6000\ubc30\uc5d0 \uc774\ub974\ub294 \uac04\uaca9\uc774\ub2e4. \ud504\ub85d\uc2dc\ub9c8\uc5d0\uc11c \ub5a0\ub09c \ube5b\uc774 \uc9c0\uad6c\uc5d0 \uc774\ub974\uae30\uae4c\uc9c0\ub294 4.2\ub144\uc774 \uac78\ub9b0\ub2e4. \uc6b0\uc8fc \uc655\ubcf5\uc120\uc758 \uc9c0\uad6c \uada4\ub3c4\uc120\uc0c1 \uc18d\ub3c4(\uc2dc\uc18d 3\ub9cc \ud0ac\ub85c\ubbf8\ud130)\ub85c \ud504\ub85d\uc2dc\ub9c8\uae4c\uc9c0 \uac00\ub824\uba74 15\ub9cc \ub144 \uc774 \uac78\ub9b0\ub2e4. \uc774 \uc815\ub3c4\ub294 \uc740\ud558\uba74\uc5d0 \uc874\uc7ac\ud558\ub294 \ubcc4 \uc0ac\uc774\uc5d0\uc11c\ub294 \ud3c9\uade0\ubcf4\ub2e4 \uc57d\uac04 \uac00\uae4c\uc6b4 \uac70\ub9ac\uc5d0 \ud574\ub2f9\ud55c\ub2e4. \uc740\ud558 \uc911\uc2ec \ub610\ub294 \uad6c\uc0c1 \uc131\ub2e8\uc5d0 \uc788\ub294 \ubcc4\ub4e4 \uc0ac\uc774\uc758 \uac70\ub9ac\ub294 \ud6e8\uc52c \uac00\uae5d\uace0, \uc740\ud558 \ud5e4\uc77c\ub85c\uc5d0 \uc788\ub294 \ubcc4\ub4e4 \uc0ac\uc774\uc758 \uac70\ub9ac\ub294 \ud6e8\uc52c \uba40\ub2e4.", "paragraph_answer": "\ud56d\uc131 \uc790\uccb4\ub294 \uc9c0\uad6c\uc5d0 \ube44\ud558\uba74 \uc555\ub3c4\uc801\uc73c\ub85c \ubb34\uac81\uace0 \ubc1d\uc740 \uc874\uc7ac\uc774\uc9c0\ub9cc \ud56d\uc131\uacfc \ud56d\uc131 \uc0ac\uc774\ub294 \uc5c4\uccad\ub098\uac8c \ub5a8\uc5b4\uc838 \uc788\ub2e4. \ud0dc\uc591\uc744 \uc81c\uc678\ud558\uace0 \uc9c0\uad6c\uc5d0\uc11c \uac00\uc7a5 \uac00\uae4c\uc6b4 \ud56d\uc131\uc740 \ucf04\ud0c0\uc6b0\ub8e8\uc2a4\uc790\ub9ac \ud504\ub85d\uc2dc\ub9c8\ub85c \ube5b\uc758 \uc18d\ub3c4\ub85c 4.2\ub144\uc774 \uac78\ub9ac\ub294 \uac70\ub9ac\uc5d0 \uc788\ub2e4. \uc774 \uac70\ub9ac\ub294 39\uc870 9\ucc9c\uc5b5 \ud0ac\ub85c\ubbf8\ud130\uc5d0 \uc774\ub974\uba70, \ud0dc\uc591\uc5d0\uc11c \uba85\uc655\uc131\uae4c\uc9c0 \uac70\ub9ac\uc758 6000\ubc30\uc5d0 \uc774\ub974\ub294 \uac04\uaca9\uc774\ub2e4. \ud504\ub85d\uc2dc\ub9c8\uc5d0\uc11c \ub5a0\ub09c \ube5b\uc774 \uc9c0\uad6c\uc5d0 \uc774\ub974\uae30\uae4c\uc9c0\ub294 4.2\ub144\uc774 \uac78\ub9b0\ub2e4. \uc6b0\uc8fc \uc655\ubcf5\uc120\uc758 \uc9c0\uad6c \uada4\ub3c4\uc120\uc0c1 \uc18d\ub3c4(\uc2dc\uc18d 3\ub9cc \ud0ac\ub85c\ubbf8\ud130)\ub85c \ud504\ub85d\uc2dc\ub9c8\uae4c\uc9c0 \uac00\ub824\uba74 15\ub9cc \ub144 \uc774 \uac78\ub9b0\ub2e4. \uc774 \uc815\ub3c4\ub294 \uc740\ud558\uba74\uc5d0 \uc874\uc7ac\ud558\ub294 \ubcc4 \uc0ac\uc774\uc5d0\uc11c\ub294 \ud3c9\uade0\ubcf4\ub2e4 \uc57d\uac04 \uac00\uae4c\uc6b4 \uac70\ub9ac\uc5d0 \ud574\ub2f9\ud55c\ub2e4. \uc740\ud558 \uc911\uc2ec \ub610\ub294 \uad6c\uc0c1 \uc131\ub2e8\uc5d0 \uc788\ub294 \ubcc4\ub4e4 \uc0ac\uc774\uc758 \uac70\ub9ac\ub294 \ud6e8\uc52c \uac00\uae5d\uace0, \uc740\ud558 \ud5e4\uc77c\ub85c\uc5d0 \uc788\ub294 \ubcc4\ub4e4 \uc0ac\uc774\uc758 \uac70\ub9ac\ub294 \ud6e8\uc52c \uba40\ub2e4.", "sentence_answer": "\uc6b0\uc8fc \uc655\ubcf5\uc120\uc758 \uc9c0\uad6c \uada4\ub3c4\uc120\uc0c1 \uc18d\ub3c4(\uc2dc\uc18d 3\ub9cc \ud0ac\ub85c\ubbf8\ud130)\ub85c \ud504\ub85d\uc2dc\ub9c8\uae4c\uc9c0 \uac00\ub824\uba74 15\ub9cc \ub144 \uc774 \uac78\ub9b0\ub2e4.", "paragraph_id": "6570256-6-1", "paragraph_question": "question: \uc6b0\uc8fc\uc655\ubcf5\uc120\uc73c\ub85c \ud504\ub85d\uc2dc\ub9c8\uae4c\uc9c0 \uac00\ub294 \uc2dc\uac04\uc740?, context: \ud56d\uc131 \uc790\uccb4\ub294 \uc9c0\uad6c\uc5d0 \ube44\ud558\uba74 \uc555\ub3c4\uc801\uc73c\ub85c \ubb34\uac81\uace0 \ubc1d\uc740 \uc874\uc7ac\uc774\uc9c0\ub9cc \ud56d\uc131\uacfc \ud56d\uc131 \uc0ac\uc774\ub294 \uc5c4\uccad\ub098\uac8c \ub5a8\uc5b4\uc838 \uc788\ub2e4. \ud0dc\uc591\uc744 \uc81c\uc678\ud558\uace0 \uc9c0\uad6c\uc5d0\uc11c \uac00\uc7a5 \uac00\uae4c\uc6b4 \ud56d\uc131\uc740 \ucf04\ud0c0\uc6b0\ub8e8\uc2a4\uc790\ub9ac \ud504\ub85d\uc2dc\ub9c8\ub85c \ube5b\uc758 \uc18d\ub3c4\ub85c 4.2\ub144\uc774 \uac78\ub9ac\ub294 \uac70\ub9ac\uc5d0 \uc788\ub2e4. \uc774 \uac70\ub9ac\ub294 39\uc870 9\ucc9c\uc5b5 \ud0ac\ub85c\ubbf8\ud130\uc5d0 \uc774\ub974\uba70, \ud0dc\uc591\uc5d0\uc11c \uba85\uc655\uc131\uae4c\uc9c0 \uac70\ub9ac\uc758 6000\ubc30\uc5d0 \uc774\ub974\ub294 \uac04\uaca9\uc774\ub2e4. \ud504\ub85d\uc2dc\ub9c8\uc5d0\uc11c \ub5a0\ub09c \ube5b\uc774 \uc9c0\uad6c\uc5d0 \uc774\ub974\uae30\uae4c\uc9c0\ub294 4.2\ub144\uc774 \uac78\ub9b0\ub2e4. \uc6b0\uc8fc \uc655\ubcf5\uc120\uc758 \uc9c0\uad6c \uada4\ub3c4\uc120\uc0c1 \uc18d\ub3c4(\uc2dc\uc18d 3\ub9cc \ud0ac\ub85c\ubbf8\ud130)\ub85c \ud504\ub85d\uc2dc\ub9c8\uae4c\uc9c0 \uac00\ub824\uba74 15\ub9cc \ub144\uc774 \uac78\ub9b0\ub2e4. \uc774 \uc815\ub3c4\ub294 \uc740\ud558\uba74\uc5d0 \uc874\uc7ac\ud558\ub294 \ubcc4 \uc0ac\uc774\uc5d0\uc11c\ub294 \ud3c9\uade0\ubcf4\ub2e4 \uc57d\uac04 \uac00\uae4c\uc6b4 \uac70\ub9ac\uc5d0 \ud574\ub2f9\ud55c\ub2e4. \uc740\ud558 \uc911\uc2ec \ub610\ub294 \uad6c\uc0c1 \uc131\ub2e8\uc5d0 \uc788\ub294 \ubcc4\ub4e4 \uc0ac\uc774\uc758 \uac70\ub9ac\ub294 \ud6e8\uc52c \uac00\uae5d\uace0, \uc740\ud558 \ud5e4\uc77c\ub85c\uc5d0 \uc788\ub294 \ubcc4\ub4e4 \uc0ac\uc774\uc758 \uac70\ub9ac\ub294 \ud6e8\uc52c \uba40\ub2e4."} {"question": "\ub808\ub2cc\uc740 \ud504\ub864\ub808\ud0c0\ub9ac\uc544 \ud601\uba85\uc744 \uc5b4\ub5a4 \ubc29\ubc95\uc73c\ub85c \ub2ec\uc131\ud574\uc57c \ud55c\ub2e4\uace0 \uc8fc\uc7a5\ud588\ub294\uac00?", "paragraph": "\ub7ec\uc2dc\uc544 \uc0ac\ud68c\ubbfc\uc8fc\ub178\ub3d9\ub2f9\uc758 \uc8fc\ub958\ub294 \uc8fc\ub85c \ud50c\ub808\ud558\ub178\ud504\uac00 \ucd08\uae30\uc5d0 \uc124\uc815\ud588\ub358 \ub300\ub85c \ubd80\ub974\uc8fc\uc544 \ud601\uba85\uacfc \ud504\ub864\ub808\ud0c0\ub9ac\uc544 \ud601\uba85\uc758 2\ub2e8\uacc4 \uacfc\uc815\uc744 \uac70\uccd0 \uc0ac\ud68c\uc8fc\uc758 \ud601\uba85\uc774 \uc644\uc131\ub41c\ub2e4\uace0 \uc804\ub9dd\ud558\uc600\ub2e4. \uc5ec\uae30\uc5d0 \uc11c\uad6c \uc720\ud559\ud30c \ucd9c\uc2e0\uc73c\ub85c \uc5d0\ub450\uc544\ub974\ud2b8 \ubca0\ub978\uc288\ud0c0\uc778\uc758 \uc218\uc815\uc8fc\uc758 \ubc0f \uc758\ud68c\uc9c4\ucd9c\ub860\uc744 \uc2e0\ubd09\ud558\ub294 \uc774\ub4e4\ub3c4 \uc788\uc5c8\ub2e4. \uadf8\ub7ec\ub098 \ub808\ub2cc\uc740 \uc774\ub4e4\uc758 \uacac\ud574\ub97c \ube44\ud310\ud558\uc600\ub2e4. \ubd80\ub974\uc8fc\uc544 \ud601\uba85\uc740 \ubd80\ub974\uc8fc\uc544\uac00, \ud504\ub864\ub808\ud0c0\ub9ac\uc544 \ud601\uba85\uc740 \ud504\ub864\ub808\ud0c0\ub9ac\uc544 \uacc4\uae09\uc774 \uc8fc\ub3c4\ud574\uc57c \ud55c\ub2e4\ub294 \uac83\uc774 \uba58\uc170\ube44\ud0a4\uc758 \uc785\uc7a5\uc774\uc5c8\ub294\ub370, \ub808\ub2cc\uc740 \"\ubd80\ub974\uc8fc\uc544 \ud601\uba85 \uc774\ud6c4 \ud504\ub864\ub808\ud0c0\ub9ac\uc544 \ud601\uba85\uc73c\ub85c \ub118\uc5b4\uac00\uc57c \ub418\uba70, \uc758\ud68c \uc9c4\ucd9c\uc744 \ud1b5\ud574 \ubaa9\uc801\uc744 \ub2ec\uc131\ud574\uc57c \ud55c\ub2e4\"\uace0 \ub9d0\ud558\ub294 \uc628\uac74\ub860\uc870\ucc28 \ube44\ud310\ud558\uc600\ub2e4. \ub808\ub2cc\uc740 \ubd80\ub974\uc8fc\uc544 \ud601\uba85\ub2e8\uacc4\ub97c \uc0dd\ub7b5\ud558\uace0 \uace7\uc7a5 \ud504\ub864\ub808\ud0c0\ub9ac\uc544 \ud601\uba85\uc744 \ucd94\uc9c4\ud574\uc57c \ud558\uba70, \ud601\uba85\uc758 \ubc29\ubc95\uc740 \ubb34\ub825\uc73c\ub85c \ub2ec\uc131\ud574\uc57c \ub418\uace0 \uadf8 \uc8fc\ub3c4\uc138\ub825\uc740 \ud504\ub864\ub808\ud0c0\ub9ac\uc544\uac00 \ub418\uc5b4\uc57c \ud558\uace0 \ubbff\uc744 \uc218 \uc788\ub294 \ub3d9\ub9f9\uc790\ub294 \uc624\uc9c1 \ub18d\ubbfc\uacfc \ub178\ub3d9\uc790\ub4e4 \ubfd0\uc774\ub77c\uace0 \uc8fc\uc7a5\ud588\ub2e4. \uc628\uac74\ub860\uc790\ub4e4\uc740 \uadf8\uc758 \uacfc\uaca9\ud568\uc744 \uc9c0\uc801\ud558\uba70. \ubd80\ub974\uc8fc\uc544 \uce35\uae4c\uc9c0 \uc801\uc73c\ub85c \ub3cc\ub9ac\uba74 \ud601\uba85\uc758 \uc2e4\ud328\ub97c \uac00\uc838\uc628\ub2e4\uace0 \ube44\ud310\ud558\uc600\ub2e4.", "answer": "\ubb34\ub825", "sentence": "\ud601\uba85\uc758 \ubc29\ubc95\uc740 \ubb34\ub825 \uc73c\ub85c \ub2ec\uc131\ud574\uc57c \ub418\uace0 \uadf8 \uc8fc\ub3c4\uc138\ub825\uc740 \ud504\ub864\ub808\ud0c0\ub9ac\uc544\uac00 \ub418\uc5b4\uc57c \ud558\uace0 \ubbff\uc744 \uc218 \uc788\ub294 \ub3d9\ub9f9\uc790\ub294 \uc624\uc9c1 \ub18d\ubbfc\uacfc \ub178\ub3d9\uc790\ub4e4 \ubfd0\uc774\ub77c\uace0 \uc8fc\uc7a5\ud588\ub2e4.", "paragraph_sentence": "\ub7ec\uc2dc\uc544 \uc0ac\ud68c\ubbfc\uc8fc\ub178\ub3d9\ub2f9\uc758 \uc8fc\ub958\ub294 \uc8fc\ub85c \ud50c\ub808\ud558\ub178\ud504\uac00 \ucd08\uae30\uc5d0 \uc124\uc815\ud588\ub358 \ub300\ub85c \ubd80\ub974\uc8fc\uc544 \ud601\uba85\uacfc \ud504\ub864\ub808\ud0c0\ub9ac\uc544 \ud601\uba85\uc758 2\ub2e8\uacc4 \uacfc\uc815\uc744 \uac70\uccd0 \uc0ac\ud68c\uc8fc\uc758 \ud601\uba85\uc774 \uc644\uc131\ub41c\ub2e4\uace0 \uc804\ub9dd\ud558\uc600\ub2e4. \uc5ec\uae30\uc5d0 \uc11c\uad6c \uc720\ud559\ud30c \ucd9c\uc2e0\uc73c\ub85c \uc5d0\ub450\uc544\ub974\ud2b8 \ubca0\ub978\uc288\ud0c0\uc778\uc758 \uc218\uc815\uc8fc\uc758 \ubc0f \uc758\ud68c\uc9c4\ucd9c\ub860\uc744 \uc2e0\ubd09\ud558\ub294 \uc774\ub4e4\ub3c4 \uc788\uc5c8\ub2e4. \uadf8\ub7ec\ub098 \ub808\ub2cc\uc740 \uc774\ub4e4\uc758 \uacac\ud574\ub97c \ube44\ud310\ud558\uc600\ub2e4. \ubd80\ub974\uc8fc\uc544 \ud601\uba85\uc740 \ubd80\ub974\uc8fc\uc544\uac00, \ud504\ub864\ub808\ud0c0\ub9ac\uc544 \ud601\uba85\uc740 \ud504\ub864\ub808\ud0c0\ub9ac\uc544 \uacc4\uae09\uc774 \uc8fc\ub3c4\ud574\uc57c \ud55c\ub2e4\ub294 \uac83\uc774 \uba58\uc170\ube44\ud0a4\uc758 \uc785\uc7a5\uc774\uc5c8\ub294\ub370, \ub808\ub2cc\uc740 \"\ubd80\ub974\uc8fc\uc544 \ud601\uba85 \uc774\ud6c4 \ud504\ub864\ub808\ud0c0\ub9ac\uc544 \ud601\uba85\uc73c\ub85c \ub118\uc5b4\uac00\uc57c \ub418\uba70, \uc758\ud68c \uc9c4\ucd9c\uc744 \ud1b5\ud574 \ubaa9\uc801\uc744 \ub2ec\uc131\ud574\uc57c \ud55c\ub2e4\"\uace0 \ub9d0\ud558\ub294 \uc628\uac74\ub860\uc870\ucc28 \ube44\ud310\ud558\uc600\ub2e4. \ub808\ub2cc\uc740 \ubd80\ub974\uc8fc\uc544 \ud601\uba85\ub2e8\uacc4\ub97c \uc0dd\ub7b5\ud558\uace0 \uace7\uc7a5 \ud504\ub864\ub808\ud0c0\ub9ac\uc544 \ud601\uba85\uc744 \ucd94\uc9c4\ud574\uc57c \ud558\uba70, \ud601\uba85\uc758 \ubc29\ubc95\uc740 \ubb34\ub825 \uc73c\ub85c \ub2ec\uc131\ud574\uc57c \ub418\uace0 \uadf8 \uc8fc\ub3c4\uc138\ub825\uc740 \ud504\ub864\ub808\ud0c0\ub9ac\uc544\uac00 \ub418\uc5b4\uc57c \ud558\uace0 \ubbff\uc744 \uc218 \uc788\ub294 \ub3d9\ub9f9\uc790\ub294 \uc624\uc9c1 \ub18d\ubbfc\uacfc \ub178\ub3d9\uc790\ub4e4 \ubfd0\uc774\ub77c\uace0 \uc8fc\uc7a5\ud588\ub2e4. \uc628\uac74\ub860\uc790\ub4e4\uc740 \uadf8\uc758 \uacfc\uaca9\ud568\uc744 \uc9c0\uc801\ud558\uba70. \ubd80\ub974\uc8fc\uc544 \uce35\uae4c\uc9c0 \uc801\uc73c\ub85c \ub3cc\ub9ac\uba74 \ud601\uba85\uc758 \uc2e4\ud328\ub97c \uac00\uc838\uc628\ub2e4\uace0 \ube44\ud310\ud558\uc600\ub2e4.", "paragraph_answer": "\ub7ec\uc2dc\uc544 \uc0ac\ud68c\ubbfc\uc8fc\ub178\ub3d9\ub2f9\uc758 \uc8fc\ub958\ub294 \uc8fc\ub85c \ud50c\ub808\ud558\ub178\ud504\uac00 \ucd08\uae30\uc5d0 \uc124\uc815\ud588\ub358 \ub300\ub85c \ubd80\ub974\uc8fc\uc544 \ud601\uba85\uacfc \ud504\ub864\ub808\ud0c0\ub9ac\uc544 \ud601\uba85\uc758 2\ub2e8\uacc4 \uacfc\uc815\uc744 \uac70\uccd0 \uc0ac\ud68c\uc8fc\uc758 \ud601\uba85\uc774 \uc644\uc131\ub41c\ub2e4\uace0 \uc804\ub9dd\ud558\uc600\ub2e4. \uc5ec\uae30\uc5d0 \uc11c\uad6c \uc720\ud559\ud30c \ucd9c\uc2e0\uc73c\ub85c \uc5d0\ub450\uc544\ub974\ud2b8 \ubca0\ub978\uc288\ud0c0\uc778\uc758 \uc218\uc815\uc8fc\uc758 \ubc0f \uc758\ud68c\uc9c4\ucd9c\ub860\uc744 \uc2e0\ubd09\ud558\ub294 \uc774\ub4e4\ub3c4 \uc788\uc5c8\ub2e4. \uadf8\ub7ec\ub098 \ub808\ub2cc\uc740 \uc774\ub4e4\uc758 \uacac\ud574\ub97c \ube44\ud310\ud558\uc600\ub2e4. \ubd80\ub974\uc8fc\uc544 \ud601\uba85\uc740 \ubd80\ub974\uc8fc\uc544\uac00, \ud504\ub864\ub808\ud0c0\ub9ac\uc544 \ud601\uba85\uc740 \ud504\ub864\ub808\ud0c0\ub9ac\uc544 \uacc4\uae09\uc774 \uc8fc\ub3c4\ud574\uc57c \ud55c\ub2e4\ub294 \uac83\uc774 \uba58\uc170\ube44\ud0a4\uc758 \uc785\uc7a5\uc774\uc5c8\ub294\ub370, \ub808\ub2cc\uc740 \"\ubd80\ub974\uc8fc\uc544 \ud601\uba85 \uc774\ud6c4 \ud504\ub864\ub808\ud0c0\ub9ac\uc544 \ud601\uba85\uc73c\ub85c \ub118\uc5b4\uac00\uc57c \ub418\uba70, \uc758\ud68c \uc9c4\ucd9c\uc744 \ud1b5\ud574 \ubaa9\uc801\uc744 \ub2ec\uc131\ud574\uc57c \ud55c\ub2e4\"\uace0 \ub9d0\ud558\ub294 \uc628\uac74\ub860\uc870\ucc28 \ube44\ud310\ud558\uc600\ub2e4. \ub808\ub2cc\uc740 \ubd80\ub974\uc8fc\uc544 \ud601\uba85\ub2e8\uacc4\ub97c \uc0dd\ub7b5\ud558\uace0 \uace7\uc7a5 \ud504\ub864\ub808\ud0c0\ub9ac\uc544 \ud601\uba85\uc744 \ucd94\uc9c4\ud574\uc57c \ud558\uba70, \ud601\uba85\uc758 \ubc29\ubc95\uc740 \ubb34\ub825 \uc73c\ub85c \ub2ec\uc131\ud574\uc57c \ub418\uace0 \uadf8 \uc8fc\ub3c4\uc138\ub825\uc740 \ud504\ub864\ub808\ud0c0\ub9ac\uc544\uac00 \ub418\uc5b4\uc57c \ud558\uace0 \ubbff\uc744 \uc218 \uc788\ub294 \ub3d9\ub9f9\uc790\ub294 \uc624\uc9c1 \ub18d\ubbfc\uacfc \ub178\ub3d9\uc790\ub4e4 \ubfd0\uc774\ub77c\uace0 \uc8fc\uc7a5\ud588\ub2e4. \uc628\uac74\ub860\uc790\ub4e4\uc740 \uadf8\uc758 \uacfc\uaca9\ud568\uc744 \uc9c0\uc801\ud558\uba70. \ubd80\ub974\uc8fc\uc544 \uce35\uae4c\uc9c0 \uc801\uc73c\ub85c \ub3cc\ub9ac\uba74 \ud601\uba85\uc758 \uc2e4\ud328\ub97c \uac00\uc838\uc628\ub2e4\uace0 \ube44\ud310\ud558\uc600\ub2e4.", "sentence_answer": "\ud601\uba85\uc758 \ubc29\ubc95\uc740 \ubb34\ub825 \uc73c\ub85c \ub2ec\uc131\ud574\uc57c \ub418\uace0 \uadf8 \uc8fc\ub3c4\uc138\ub825\uc740 \ud504\ub864\ub808\ud0c0\ub9ac\uc544\uac00 \ub418\uc5b4\uc57c \ud558\uace0 \ubbff\uc744 \uc218 \uc788\ub294 \ub3d9\ub9f9\uc790\ub294 \uc624\uc9c1 \ub18d\ubbfc\uacfc \ub178\ub3d9\uc790\ub4e4 \ubfd0\uc774\ub77c\uace0 \uc8fc\uc7a5\ud588\ub2e4.", "paragraph_id": "6122234-2-1", "paragraph_question": "question: \ub808\ub2cc\uc740 \ud504\ub864\ub808\ud0c0\ub9ac\uc544 \ud601\uba85\uc744 \uc5b4\ub5a4 \ubc29\ubc95\uc73c\ub85c \ub2ec\uc131\ud574\uc57c \ud55c\ub2e4\uace0 \uc8fc\uc7a5\ud588\ub294\uac00?, context: \ub7ec\uc2dc\uc544 \uc0ac\ud68c\ubbfc\uc8fc\ub178\ub3d9\ub2f9\uc758 \uc8fc\ub958\ub294 \uc8fc\ub85c \ud50c\ub808\ud558\ub178\ud504\uac00 \ucd08\uae30\uc5d0 \uc124\uc815\ud588\ub358 \ub300\ub85c \ubd80\ub974\uc8fc\uc544 \ud601\uba85\uacfc \ud504\ub864\ub808\ud0c0\ub9ac\uc544 \ud601\uba85\uc758 2\ub2e8\uacc4 \uacfc\uc815\uc744 \uac70\uccd0 \uc0ac\ud68c\uc8fc\uc758 \ud601\uba85\uc774 \uc644\uc131\ub41c\ub2e4\uace0 \uc804\ub9dd\ud558\uc600\ub2e4. \uc5ec\uae30\uc5d0 \uc11c\uad6c \uc720\ud559\ud30c \ucd9c\uc2e0\uc73c\ub85c \uc5d0\ub450\uc544\ub974\ud2b8 \ubca0\ub978\uc288\ud0c0\uc778\uc758 \uc218\uc815\uc8fc\uc758 \ubc0f \uc758\ud68c\uc9c4\ucd9c\ub860\uc744 \uc2e0\ubd09\ud558\ub294 \uc774\ub4e4\ub3c4 \uc788\uc5c8\ub2e4. \uadf8\ub7ec\ub098 \ub808\ub2cc\uc740 \uc774\ub4e4\uc758 \uacac\ud574\ub97c \ube44\ud310\ud558\uc600\ub2e4. \ubd80\ub974\uc8fc\uc544 \ud601\uba85\uc740 \ubd80\ub974\uc8fc\uc544\uac00, \ud504\ub864\ub808\ud0c0\ub9ac\uc544 \ud601\uba85\uc740 \ud504\ub864\ub808\ud0c0\ub9ac\uc544 \uacc4\uae09\uc774 \uc8fc\ub3c4\ud574\uc57c \ud55c\ub2e4\ub294 \uac83\uc774 \uba58\uc170\ube44\ud0a4\uc758 \uc785\uc7a5\uc774\uc5c8\ub294\ub370, \ub808\ub2cc\uc740 \"\ubd80\ub974\uc8fc\uc544 \ud601\uba85 \uc774\ud6c4 \ud504\ub864\ub808\ud0c0\ub9ac\uc544 \ud601\uba85\uc73c\ub85c \ub118\uc5b4\uac00\uc57c \ub418\uba70, \uc758\ud68c \uc9c4\ucd9c\uc744 \ud1b5\ud574 \ubaa9\uc801\uc744 \ub2ec\uc131\ud574\uc57c \ud55c\ub2e4\"\uace0 \ub9d0\ud558\ub294 \uc628\uac74\ub860\uc870\ucc28 \ube44\ud310\ud558\uc600\ub2e4. \ub808\ub2cc\uc740 \ubd80\ub974\uc8fc\uc544 \ud601\uba85\ub2e8\uacc4\ub97c \uc0dd\ub7b5\ud558\uace0 \uace7\uc7a5 \ud504\ub864\ub808\ud0c0\ub9ac\uc544 \ud601\uba85\uc744 \ucd94\uc9c4\ud574\uc57c \ud558\uba70, \ud601\uba85\uc758 \ubc29\ubc95\uc740 \ubb34\ub825\uc73c\ub85c \ub2ec\uc131\ud574\uc57c \ub418\uace0 \uadf8 \uc8fc\ub3c4\uc138\ub825\uc740 \ud504\ub864\ub808\ud0c0\ub9ac\uc544\uac00 \ub418\uc5b4\uc57c \ud558\uace0 \ubbff\uc744 \uc218 \uc788\ub294 \ub3d9\ub9f9\uc790\ub294 \uc624\uc9c1 \ub18d\ubbfc\uacfc \ub178\ub3d9\uc790\ub4e4 \ubfd0\uc774\ub77c\uace0 \uc8fc\uc7a5\ud588\ub2e4. \uc628\uac74\ub860\uc790\ub4e4\uc740 \uadf8\uc758 \uacfc\uaca9\ud568\uc744 \uc9c0\uc801\ud558\uba70. \ubd80\ub974\uc8fc\uc544 \uce35\uae4c\uc9c0 \uc801\uc73c\ub85c \ub3cc\ub9ac\uba74 \ud601\uba85\uc758 \uc2e4\ud328\ub97c \uac00\uc838\uc628\ub2e4\uace0 \ube44\ud310\ud558\uc600\ub2e4."} {"question": "\ud734\uc77c \ub0ae \uc2dc\uac04\ub300\uc758 \uc9c0\ucfe0\ud638 \uc804\uae30 \ucca0\ub3c4\uc120 \ubc30\ucc28\uac04\uaca9\uc740 \uba87\ubd84\uc785\ub2c8\uae4c?", "paragraph": "\ud3c9\uc77c\u30fb\ud734\uc77c \ubaa8\ub450 \ub0ae\uc2dc\uac04\ub300\ub294 \uad6c\ub85c\uc0ac\ud0a4 \uc5ed \uc55e - \uc9c0\ucfe0\ud638 \ub178\uac00\ud0c0 \uc0ac\uc774\uc758 \uc804\uc120 \uc5f4\ucc28\uc640 \uad6c\ub85c\uc0ac\ud0a4 \uc5ed \uc55e- \uc9c0\ucfe0\ud638 \ub098\uce74\ub9c8 \uc0ac\uc774\uc758 \uad6c\uac04 \uc5f4\ucc28\uac00 12\ubd84 \uac04\uaca9\uc73c\ub85c \uad50\ub300\ub85c, \ud3c9\uc77c \uc544\uce68\uc758 \ub7ec\uc26c \uc2dc\uac04\ub300\ub294 \uc804\uc120 \uc5f4\ucc28\uc640 \uad6c\ub85c\uc0ac\ud0a4 \uc5ed \uc55e-\uc9c0\ucfe0\ud638 \ub098\uce74\ub9c8\u30fb\uad6c\uc2a4\ubc14\uc2dc \uc0ac\uc774\uc758 \uad6c\uac04 \uc5f4\ucc28\uac00 5 - 10\ubd84 \uac04\uaca9\uc73c\ub85c \uac70\uc758 \uad50\ub300\ub85c, \ud3c9\uc77c \uc800\ub141\uc758 \ub7ec\uc26c \uc2dc\uac04\ub300\ub294 \uc804\uc120 \uc5f4\ucc28\uc640 \uad6c\ub85c\uc0ac\ud0a4 \uc5d0\ud0a4\ub9c8\uc5d0-\uad6c\uc2a4\ubc14\uc2dc\uac04\uc758 \uad6c\uac04 \uc5f4\ucc28\uac00 7 - 10\ubd84 \uac04\uaca9\uc73c\ub85c \uad50\ub300\ub85c \uc6b4\ud589\ub418\uace0 \uc788\ub2e4. \ub610, \uc774\ub978 \uc544\uce68\u30fb\uc2ec\uc57c\uc5d0\ub294 \ucc28\uace0\ub85c\uc758 \ucd9c\uc785\uace0\ub97c \uacb8\ud55c \uad6c\uc2a4\ubc14\uc2dc-\uc9c0\ucfe0\ud638 \ub178\uac00\ud0c0\uac04\uc758 \uad6c\uac04 \uc5f4\ucc28\uac00 \uc788\ub2e4. \uc6d0\ub9e8 \uc6b4\uc804\uc740 \ud558\uc9c0 \uc54a\uace0, \ubaa8\ub4e0 \uc5f4\ucc28\uc5d0 \ucc28\uc7a5\uc774 \uc2b9\ubb34\ud558\uace0 \uc788\ub2e4. \ub367\ubd99\uc5ec \uc624\uc2b9\ubc29\uc9c0\ub97c \uc704\ud574 3000\ud615\ub9cc \ubc29\ud5a5\ub9c9\uc744 \uce7c\ub77c\ub85c \uac31\uc2e0\ud558\uc600\ub2e4.", "answer": "12\ubd84", "sentence": "\ud3c9\uc77c\u30fb\ud734\uc77c \ubaa8\ub450 \ub0ae\uc2dc\uac04\ub300\ub294 \uad6c\ub85c\uc0ac\ud0a4 \uc5ed \uc55e - \uc9c0\ucfe0\ud638 \ub178\uac00\ud0c0 \uc0ac\uc774\uc758 \uc804\uc120 \uc5f4\ucc28\uc640 \uad6c\ub85c\uc0ac\ud0a4 \uc5ed \uc55e- \uc9c0\ucfe0\ud638 \ub098\uce74\ub9c8 \uc0ac\uc774\uc758 \uad6c\uac04 \uc5f4\ucc28\uac00 12\ubd84 \uac04\uaca9\uc73c\ub85c \uad50\ub300\ub85c, \ud3c9\uc77c \uc544\uce68\uc758 \ub7ec\uc26c \uc2dc\uac04\ub300\ub294 \uc804\uc120 \uc5f4\ucc28\uc640 \uad6c\ub85c\uc0ac\ud0a4 \uc5ed \uc55e-\uc9c0\ucfe0\ud638 \ub098\uce74\ub9c8\u30fb\uad6c\uc2a4\ubc14\uc2dc \uc0ac\uc774\uc758 \uad6c\uac04 \uc5f4\ucc28\uac00 5 - 10\ubd84 \uac04\uaca9\uc73c\ub85c \uac70\uc758 \uad50\ub300\ub85c, \ud3c9\uc77c \uc800\ub141\uc758 \ub7ec\uc26c \uc2dc\uac04\ub300\ub294 \uc804\uc120 \uc5f4\ucc28\uc640 \uad6c\ub85c\uc0ac\ud0a4 \uc5d0\ud0a4\ub9c8\uc5d0-\uad6c\uc2a4\ubc14\uc2dc\uac04\uc758 \uad6c\uac04 \uc5f4\ucc28\uac00 7 - 10\ubd84 \uac04\uaca9\uc73c\ub85c \uad50\ub300\ub85c \uc6b4\ud589\ub418\uace0 \uc788\ub2e4.", "paragraph_sentence": " \ud3c9\uc77c\u30fb\ud734\uc77c \ubaa8\ub450 \ub0ae\uc2dc\uac04\ub300\ub294 \uad6c\ub85c\uc0ac\ud0a4 \uc5ed \uc55e - \uc9c0\ucfe0\ud638 \ub178\uac00\ud0c0 \uc0ac\uc774\uc758 \uc804\uc120 \uc5f4\ucc28\uc640 \uad6c\ub85c\uc0ac\ud0a4 \uc5ed \uc55e- \uc9c0\ucfe0\ud638 \ub098\uce74\ub9c8 \uc0ac\uc774\uc758 \uad6c\uac04 \uc5f4\ucc28\uac00 12\ubd84 \uac04\uaca9\uc73c\ub85c \uad50\ub300\ub85c, \ud3c9\uc77c \uc544\uce68\uc758 \ub7ec\uc26c \uc2dc\uac04\ub300\ub294 \uc804\uc120 \uc5f4\ucc28\uc640 \uad6c\ub85c\uc0ac\ud0a4 \uc5ed \uc55e-\uc9c0\ucfe0\ud638 \ub098\uce74\ub9c8\u30fb\uad6c\uc2a4\ubc14\uc2dc \uc0ac\uc774\uc758 \uad6c\uac04 \uc5f4\ucc28\uac00 5 - 10\ubd84 \uac04\uaca9\uc73c\ub85c \uac70\uc758 \uad50\ub300\ub85c, \ud3c9\uc77c \uc800\ub141\uc758 \ub7ec\uc26c \uc2dc\uac04\ub300\ub294 \uc804\uc120 \uc5f4\ucc28\uc640 \uad6c\ub85c\uc0ac\ud0a4 \uc5d0\ud0a4\ub9c8\uc5d0-\uad6c\uc2a4\ubc14\uc2dc\uac04\uc758 \uad6c\uac04 \uc5f4\ucc28\uac00 7 - 10\ubd84 \uac04\uaca9\uc73c\ub85c \uad50\ub300\ub85c \uc6b4\ud589\ub418\uace0 \uc788\ub2e4. \ub610, \uc774\ub978 \uc544\uce68\u30fb\uc2ec\uc57c\uc5d0\ub294 \ucc28\uace0\ub85c\uc758 \ucd9c\uc785\uace0\ub97c \uacb8\ud55c \uad6c\uc2a4\ubc14\uc2dc-\uc9c0\ucfe0\ud638 \ub178\uac00\ud0c0\uac04\uc758 \uad6c\uac04 \uc5f4\ucc28\uac00 \uc788\ub2e4. \uc6d0\ub9e8 \uc6b4\uc804\uc740 \ud558\uc9c0 \uc54a\uace0, \ubaa8\ub4e0 \uc5f4\ucc28\uc5d0 \ucc28\uc7a5\uc774 \uc2b9\ubb34\ud558\uace0 \uc788\ub2e4. \ub367\ubd99\uc5ec \uc624\uc2b9\ubc29\uc9c0\ub97c \uc704\ud574 3000\ud615\ub9cc \ubc29\ud5a5\ub9c9\uc744 \uce7c\ub77c\ub85c \uac31\uc2e0\ud558\uc600\ub2e4.", "paragraph_answer": "\ud3c9\uc77c\u30fb\ud734\uc77c \ubaa8\ub450 \ub0ae\uc2dc\uac04\ub300\ub294 \uad6c\ub85c\uc0ac\ud0a4 \uc5ed \uc55e - \uc9c0\ucfe0\ud638 \ub178\uac00\ud0c0 \uc0ac\uc774\uc758 \uc804\uc120 \uc5f4\ucc28\uc640 \uad6c\ub85c\uc0ac\ud0a4 \uc5ed \uc55e- \uc9c0\ucfe0\ud638 \ub098\uce74\ub9c8 \uc0ac\uc774\uc758 \uad6c\uac04 \uc5f4\ucc28\uac00 12\ubd84 \uac04\uaca9\uc73c\ub85c \uad50\ub300\ub85c, \ud3c9\uc77c \uc544\uce68\uc758 \ub7ec\uc26c \uc2dc\uac04\ub300\ub294 \uc804\uc120 \uc5f4\ucc28\uc640 \uad6c\ub85c\uc0ac\ud0a4 \uc5ed \uc55e-\uc9c0\ucfe0\ud638 \ub098\uce74\ub9c8\u30fb\uad6c\uc2a4\ubc14\uc2dc \uc0ac\uc774\uc758 \uad6c\uac04 \uc5f4\ucc28\uac00 5 - 10\ubd84 \uac04\uaca9\uc73c\ub85c \uac70\uc758 \uad50\ub300\ub85c, \ud3c9\uc77c \uc800\ub141\uc758 \ub7ec\uc26c \uc2dc\uac04\ub300\ub294 \uc804\uc120 \uc5f4\ucc28\uc640 \uad6c\ub85c\uc0ac\ud0a4 \uc5d0\ud0a4\ub9c8\uc5d0-\uad6c\uc2a4\ubc14\uc2dc\uac04\uc758 \uad6c\uac04 \uc5f4\ucc28\uac00 7 - 10\ubd84 \uac04\uaca9\uc73c\ub85c \uad50\ub300\ub85c \uc6b4\ud589\ub418\uace0 \uc788\ub2e4. \ub610, \uc774\ub978 \uc544\uce68\u30fb\uc2ec\uc57c\uc5d0\ub294 \ucc28\uace0\ub85c\uc758 \ucd9c\uc785\uace0\ub97c \uacb8\ud55c \uad6c\uc2a4\ubc14\uc2dc-\uc9c0\ucfe0\ud638 \ub178\uac00\ud0c0\uac04\uc758 \uad6c\uac04 \uc5f4\ucc28\uac00 \uc788\ub2e4. \uc6d0\ub9e8 \uc6b4\uc804\uc740 \ud558\uc9c0 \uc54a\uace0, \ubaa8\ub4e0 \uc5f4\ucc28\uc5d0 \ucc28\uc7a5\uc774 \uc2b9\ubb34\ud558\uace0 \uc788\ub2e4. \ub367\ubd99\uc5ec \uc624\uc2b9\ubc29\uc9c0\ub97c \uc704\ud574 3000\ud615\ub9cc \ubc29\ud5a5\ub9c9\uc744 \uce7c\ub77c\ub85c \uac31\uc2e0\ud558\uc600\ub2e4.", "sentence_answer": "\ud3c9\uc77c\u30fb\ud734\uc77c \ubaa8\ub450 \ub0ae\uc2dc\uac04\ub300\ub294 \uad6c\ub85c\uc0ac\ud0a4 \uc5ed \uc55e - \uc9c0\ucfe0\ud638 \ub178\uac00\ud0c0 \uc0ac\uc774\uc758 \uc804\uc120 \uc5f4\ucc28\uc640 \uad6c\ub85c\uc0ac\ud0a4 \uc5ed \uc55e- \uc9c0\ucfe0\ud638 \ub098\uce74\ub9c8 \uc0ac\uc774\uc758 \uad6c\uac04 \uc5f4\ucc28\uac00 12\ubd84 \uac04\uaca9\uc73c\ub85c \uad50\ub300\ub85c, \ud3c9\uc77c \uc544\uce68\uc758 \ub7ec\uc26c \uc2dc\uac04\ub300\ub294 \uc804\uc120 \uc5f4\ucc28\uc640 \uad6c\ub85c\uc0ac\ud0a4 \uc5ed \uc55e-\uc9c0\ucfe0\ud638 \ub098\uce74\ub9c8\u30fb\uad6c\uc2a4\ubc14\uc2dc \uc0ac\uc774\uc758 \uad6c\uac04 \uc5f4\ucc28\uac00 5 - 10\ubd84 \uac04\uaca9\uc73c\ub85c \uac70\uc758 \uad50\ub300\ub85c, \ud3c9\uc77c \uc800\ub141\uc758 \ub7ec\uc26c \uc2dc\uac04\ub300\ub294 \uc804\uc120 \uc5f4\ucc28\uc640 \uad6c\ub85c\uc0ac\ud0a4 \uc5d0\ud0a4\ub9c8\uc5d0-\uad6c\uc2a4\ubc14\uc2dc\uac04\uc758 \uad6c\uac04 \uc5f4\ucc28\uac00 7 - 10\ubd84 \uac04\uaca9\uc73c\ub85c \uad50\ub300\ub85c \uc6b4\ud589\ub418\uace0 \uc788\ub2e4.", "paragraph_id": "6198200-1-0", "paragraph_question": "question: \ud734\uc77c \ub0ae \uc2dc\uac04\ub300\uc758 \uc9c0\ucfe0\ud638 \uc804\uae30 \ucca0\ub3c4\uc120 \ubc30\ucc28\uac04\uaca9\uc740 \uba87\ubd84\uc785\ub2c8\uae4c?, context: \ud3c9\uc77c\u30fb\ud734\uc77c \ubaa8\ub450 \ub0ae\uc2dc\uac04\ub300\ub294 \uad6c\ub85c\uc0ac\ud0a4 \uc5ed \uc55e - \uc9c0\ucfe0\ud638 \ub178\uac00\ud0c0 \uc0ac\uc774\uc758 \uc804\uc120 \uc5f4\ucc28\uc640 \uad6c\ub85c\uc0ac\ud0a4 \uc5ed \uc55e- \uc9c0\ucfe0\ud638 \ub098\uce74\ub9c8 \uc0ac\uc774\uc758 \uad6c\uac04 \uc5f4\ucc28\uac00 12\ubd84 \uac04\uaca9\uc73c\ub85c \uad50\ub300\ub85c, \ud3c9\uc77c \uc544\uce68\uc758 \ub7ec\uc26c \uc2dc\uac04\ub300\ub294 \uc804\uc120 \uc5f4\ucc28\uc640 \uad6c\ub85c\uc0ac\ud0a4 \uc5ed \uc55e-\uc9c0\ucfe0\ud638 \ub098\uce74\ub9c8\u30fb\uad6c\uc2a4\ubc14\uc2dc \uc0ac\uc774\uc758 \uad6c\uac04 \uc5f4\ucc28\uac00 5 - 10\ubd84 \uac04\uaca9\uc73c\ub85c \uac70\uc758 \uad50\ub300\ub85c, \ud3c9\uc77c \uc800\ub141\uc758 \ub7ec\uc26c \uc2dc\uac04\ub300\ub294 \uc804\uc120 \uc5f4\ucc28\uc640 \uad6c\ub85c\uc0ac\ud0a4 \uc5d0\ud0a4\ub9c8\uc5d0-\uad6c\uc2a4\ubc14\uc2dc\uac04\uc758 \uad6c\uac04 \uc5f4\ucc28\uac00 7 - 10\ubd84 \uac04\uaca9\uc73c\ub85c \uad50\ub300\ub85c \uc6b4\ud589\ub418\uace0 \uc788\ub2e4. \ub610, \uc774\ub978 \uc544\uce68\u30fb\uc2ec\uc57c\uc5d0\ub294 \ucc28\uace0\ub85c\uc758 \ucd9c\uc785\uace0\ub97c \uacb8\ud55c \uad6c\uc2a4\ubc14\uc2dc-\uc9c0\ucfe0\ud638 \ub178\uac00\ud0c0\uac04\uc758 \uad6c\uac04 \uc5f4\ucc28\uac00 \uc788\ub2e4. \uc6d0\ub9e8 \uc6b4\uc804\uc740 \ud558\uc9c0 \uc54a\uace0, \ubaa8\ub4e0 \uc5f4\ucc28\uc5d0 \ucc28\uc7a5\uc774 \uc2b9\ubb34\ud558\uace0 \uc788\ub2e4. \ub367\ubd99\uc5ec \uc624\uc2b9\ubc29\uc9c0\ub97c \uc704\ud574 3000\ud615\ub9cc \ubc29\ud5a5\ub9c9\uc744 \uce7c\ub77c\ub85c \uac31\uc2e0\ud558\uc600\ub2e4."} {"question": "\uc99d\uba85\uc11c\uc5d0 \uc804\uacf5\uc744 \ud45c\uae30\ud560 \ub54c \uc77c\ubc18\uacfc\uc815\uc744 \uc774\uc218\ud55c \uc878\uc5c5\uc0dd\uc758 \uc804\uacf5 \uba85\uce6d\uc5d0 \ud45c\uae30\ud558\ub294 \uac83\uc740?", "paragraph": "\uacf5\ud559\uad50\uc721\uc778\uc99d\uc81c\ub3c4\ub85c \uc778\uc99d\ubc1b\uc740 \ud559\ubd80\uc640 \ud559\uacfc \ub610\ub294 \uc804\uacf5\uc740 \ud55c\uad6d\uacf5\ud559\uad50\uc721\uc778\uc99d\uc6d0\uc758 \uc778\uc99d \uae30\uc900\uc5d0 \ub9de\ucdb0 \uad50\uc721 \ud504\ub85c\uadf8\ub7a8\uc744 \uad6c\uc131\ud55c\ub2e4. \uacf5\ud559\uad50\uc721\uc778\uc99d\uc81c\ub3c4\ub97c \uc2dc\ud589\ud558\ub294 \uacf3\uc758 \ud559\uc0dd\uc740 \uba3c\uc800 \uc778\uc99d\ud504\ub85c\uadf8\ub7a8\uc5d0 \ub530\ub77c \uc804\uacf5\uc744 \uc774\uc218\ud558\ub294 \uc804\ubb38\uacfc\uc815\uacfc \uc778\uc99d\ud504\ub85c\uadf8\ub7a8\uc744 \ud3ec\uae30\ud558\ub294 \uc77c\ubc18\uacfc\uc815\uc744 \ub450\uace0 \uc120\ud0dd\ud560 \uc218 \uc788\ub2e4. \uc804\ubb38\uacfc\uc815\uc744 \uc774\uc218\ud558\ub294 \ud559\uc0dd\uc740 \ud55c\uad6d\uacf5\ud559\uad50\uc721\uc778\uc99d\uc6d0\uacfc \ud574\ub2f9 \uc778\uc99d\ud504\ub85c\uadf8\ub7a8\uc5d0\uc11c \uc81c\uc2dc\ud558\ub294 \uc778\uc99d\uae30\uc900\uc744 \ub3d9\uc2dc\uc5d0 \ucda9\uc871\ud558\uc5ec\uc57c \ud558\uba70, \uc815\ud574\uc9c4 \uc778\uc99d\ud504\ub85c\uadf8\ub7a8\uc758 \uad50\uacfc\ubaa9 \ucee4\ub9ac\ud058\ub7fc\uc5d0 \ub530\ub77c \uacfc\ubaa9\uc744 \uc774\uc218\ud574\uc57c \uacf5\ud559\uc778\uc99d\uc744 \ubc1b\uc744 \uc218 \uc788\ub2e4. \uc804\ubb38\uacfc\uc815\uc744 \uc774\uc218\ud55c \uc878\uc5c5\uc0dd\uc740 \ud559\uc801\ubd80\uc640 \uac01\uc885 \uc99d\uba85\uc11c \ub4f1\uc5d0 \uc778\uc99d \uc0ac\uc2e4\uc774 \uae30\uc7ac\ub418\uba70, \uc99d\uba85\uc11c\uc5d0\uc11c \uc804\uacf5\uc744 \ud45c\uae30\ud560 \ub54c, \uc778\uc99d\ud504\ub85c\uadf8\ub7a8 \uc774\uc218\uc790\ub294 \uc804\uacf5 \uba85\uce6d\uc5d0 \u2018\uc804\ubb38\u2019\uc744 \ucd94\uac00\ud558\uc5ec \ud45c\uae30\ud55c\ub2e4. \uadf8\ub7ec\ub098 \uc77c\ubc18\uacfc\uc815\uc744 \uc774\uc218\ud55c \uc878\uc5c5\uc0dd\uc740 \uc804\uacf5 \uba85\uce6d\uc5d0 \u2018\uc77c\ubc18\u2019\uc744 \ucd94\uac00 \uae30\uc7ac\ud574 \uc804\ubb38\uacfc\uc815 \uc774\uc218\uc790\uc640 \ucc28\uc774\ub97c \ub450\uac8c \ub41c\ub2e4.", "answer": "\uc77c\ubc18", "sentence": "\uacf5\ud559\uad50\uc721\uc778\uc99d\uc81c\ub3c4\ub97c \uc2dc\ud589\ud558\ub294 \uacf3\uc758 \ud559\uc0dd\uc740 \uba3c\uc800 \uc778\uc99d\ud504\ub85c\uadf8\ub7a8\uc5d0 \ub530\ub77c \uc804\uacf5\uc744 \uc774\uc218\ud558\ub294 \uc804\ubb38\uacfc\uc815\uacfc \uc778\uc99d\ud504\ub85c\uadf8\ub7a8\uc744 \ud3ec\uae30\ud558\ub294 \uc77c\ubc18 \uacfc\uc815\uc744 \ub450\uace0 \uc120\ud0dd\ud560 \uc218 \uc788\ub2e4.", "paragraph_sentence": "\uacf5\ud559\uad50\uc721\uc778\uc99d\uc81c\ub3c4\ub85c \uc778\uc99d\ubc1b\uc740 \ud559\ubd80\uc640 \ud559\uacfc \ub610\ub294 \uc804\uacf5\uc740 \ud55c\uad6d\uacf5\ud559\uad50\uc721\uc778\uc99d\uc6d0\uc758 \uc778\uc99d \uae30\uc900\uc5d0 \ub9de\ucdb0 \uad50\uc721 \ud504\ub85c\uadf8\ub7a8\uc744 \uad6c\uc131\ud55c\ub2e4. \uacf5\ud559\uad50\uc721\uc778\uc99d\uc81c\ub3c4\ub97c \uc2dc\ud589\ud558\ub294 \uacf3\uc758 \ud559\uc0dd\uc740 \uba3c\uc800 \uc778\uc99d\ud504\ub85c\uadf8\ub7a8\uc5d0 \ub530\ub77c \uc804\uacf5\uc744 \uc774\uc218\ud558\ub294 \uc804\ubb38\uacfc\uc815\uacfc \uc778\uc99d\ud504\ub85c\uadf8\ub7a8\uc744 \ud3ec\uae30\ud558\ub294 \uc77c\ubc18 \uacfc\uc815\uc744 \ub450\uace0 \uc120\ud0dd\ud560 \uc218 \uc788\ub2e4. \uc804\ubb38\uacfc\uc815\uc744 \uc774\uc218\ud558\ub294 \ud559\uc0dd\uc740 \ud55c\uad6d\uacf5\ud559\uad50\uc721\uc778\uc99d\uc6d0\uacfc \ud574\ub2f9 \uc778\uc99d\ud504\ub85c\uadf8\ub7a8\uc5d0\uc11c \uc81c\uc2dc\ud558\ub294 \uc778\uc99d\uae30\uc900\uc744 \ub3d9\uc2dc\uc5d0 \ucda9\uc871\ud558\uc5ec\uc57c \ud558\uba70, \uc815\ud574\uc9c4 \uc778\uc99d\ud504\ub85c\uadf8\ub7a8\uc758 \uad50\uacfc\ubaa9 \ucee4\ub9ac\ud058\ub7fc\uc5d0 \ub530\ub77c \uacfc\ubaa9\uc744 \uc774\uc218\ud574\uc57c \uacf5\ud559\uc778\uc99d\uc744 \ubc1b\uc744 \uc218 \uc788\ub2e4. \uc804\ubb38\uacfc\uc815\uc744 \uc774\uc218\ud55c \uc878\uc5c5\uc0dd\uc740 \ud559\uc801\ubd80\uc640 \uac01\uc885 \uc99d\uba85\uc11c \ub4f1\uc5d0 \uc778\uc99d \uc0ac\uc2e4\uc774 \uae30\uc7ac\ub418\uba70, \uc99d\uba85\uc11c\uc5d0\uc11c \uc804\uacf5\uc744 \ud45c\uae30\ud560 \ub54c, \uc778\uc99d\ud504\ub85c\uadf8\ub7a8 \uc774\uc218\uc790\ub294 \uc804\uacf5 \uba85\uce6d\uc5d0 \u2018\uc804\ubb38\u2019\uc744 \ucd94\uac00\ud558\uc5ec \ud45c\uae30\ud55c\ub2e4. \uadf8\ub7ec\ub098 \uc77c\ubc18\uacfc\uc815\uc744 \uc774\uc218\ud55c \uc878\uc5c5\uc0dd\uc740 \uc804\uacf5 \uba85\uce6d\uc5d0 \u2018\uc77c\ubc18\u2019\uc744 \ucd94\uac00 \uae30\uc7ac\ud574 \uc804\ubb38\uacfc\uc815 \uc774\uc218\uc790\uc640 \ucc28\uc774\ub97c \ub450\uac8c \ub41c\ub2e4.", "paragraph_answer": "\uacf5\ud559\uad50\uc721\uc778\uc99d\uc81c\ub3c4\ub85c \uc778\uc99d\ubc1b\uc740 \ud559\ubd80\uc640 \ud559\uacfc \ub610\ub294 \uc804\uacf5\uc740 \ud55c\uad6d\uacf5\ud559\uad50\uc721\uc778\uc99d\uc6d0\uc758 \uc778\uc99d \uae30\uc900\uc5d0 \ub9de\ucdb0 \uad50\uc721 \ud504\ub85c\uadf8\ub7a8\uc744 \uad6c\uc131\ud55c\ub2e4. \uacf5\ud559\uad50\uc721\uc778\uc99d\uc81c\ub3c4\ub97c \uc2dc\ud589\ud558\ub294 \uacf3\uc758 \ud559\uc0dd\uc740 \uba3c\uc800 \uc778\uc99d\ud504\ub85c\uadf8\ub7a8\uc5d0 \ub530\ub77c \uc804\uacf5\uc744 \uc774\uc218\ud558\ub294 \uc804\ubb38\uacfc\uc815\uacfc \uc778\uc99d\ud504\ub85c\uadf8\ub7a8\uc744 \ud3ec\uae30\ud558\ub294 \uc77c\ubc18 \uacfc\uc815\uc744 \ub450\uace0 \uc120\ud0dd\ud560 \uc218 \uc788\ub2e4. \uc804\ubb38\uacfc\uc815\uc744 \uc774\uc218\ud558\ub294 \ud559\uc0dd\uc740 \ud55c\uad6d\uacf5\ud559\uad50\uc721\uc778\uc99d\uc6d0\uacfc \ud574\ub2f9 \uc778\uc99d\ud504\ub85c\uadf8\ub7a8\uc5d0\uc11c \uc81c\uc2dc\ud558\ub294 \uc778\uc99d\uae30\uc900\uc744 \ub3d9\uc2dc\uc5d0 \ucda9\uc871\ud558\uc5ec\uc57c \ud558\uba70, \uc815\ud574\uc9c4 \uc778\uc99d\ud504\ub85c\uadf8\ub7a8\uc758 \uad50\uacfc\ubaa9 \ucee4\ub9ac\ud058\ub7fc\uc5d0 \ub530\ub77c \uacfc\ubaa9\uc744 \uc774\uc218\ud574\uc57c \uacf5\ud559\uc778\uc99d\uc744 \ubc1b\uc744 \uc218 \uc788\ub2e4. \uc804\ubb38\uacfc\uc815\uc744 \uc774\uc218\ud55c \uc878\uc5c5\uc0dd\uc740 \ud559\uc801\ubd80\uc640 \uac01\uc885 \uc99d\uba85\uc11c \ub4f1\uc5d0 \uc778\uc99d \uc0ac\uc2e4\uc774 \uae30\uc7ac\ub418\uba70, \uc99d\uba85\uc11c\uc5d0\uc11c \uc804\uacf5\uc744 \ud45c\uae30\ud560 \ub54c, \uc778\uc99d\ud504\ub85c\uadf8\ub7a8 \uc774\uc218\uc790\ub294 \uc804\uacf5 \uba85\uce6d\uc5d0 \u2018\uc804\ubb38\u2019\uc744 \ucd94\uac00\ud558\uc5ec \ud45c\uae30\ud55c\ub2e4. \uadf8\ub7ec\ub098 \uc77c\ubc18\uacfc\uc815\uc744 \uc774\uc218\ud55c \uc878\uc5c5\uc0dd\uc740 \uc804\uacf5 \uba85\uce6d\uc5d0 \u2018\uc77c\ubc18\u2019\uc744 \ucd94\uac00 \uae30\uc7ac\ud574 \uc804\ubb38\uacfc\uc815 \uc774\uc218\uc790\uc640 \ucc28\uc774\ub97c \ub450\uac8c \ub41c\ub2e4.", "sentence_answer": "\uacf5\ud559\uad50\uc721\uc778\uc99d\uc81c\ub3c4\ub97c \uc2dc\ud589\ud558\ub294 \uacf3\uc758 \ud559\uc0dd\uc740 \uba3c\uc800 \uc778\uc99d\ud504\ub85c\uadf8\ub7a8\uc5d0 \ub530\ub77c \uc804\uacf5\uc744 \uc774\uc218\ud558\ub294 \uc804\ubb38\uacfc\uc815\uacfc \uc778\uc99d\ud504\ub85c\uadf8\ub7a8\uc744 \ud3ec\uae30\ud558\ub294 \uc77c\ubc18 \uacfc\uc815\uc744 \ub450\uace0 \uc120\ud0dd\ud560 \uc218 \uc788\ub2e4.", "paragraph_id": "6516756-9-1", "paragraph_question": "question: \uc99d\uba85\uc11c\uc5d0 \uc804\uacf5\uc744 \ud45c\uae30\ud560 \ub54c \uc77c\ubc18\uacfc\uc815\uc744 \uc774\uc218\ud55c \uc878\uc5c5\uc0dd\uc758 \uc804\uacf5 \uba85\uce6d\uc5d0 \ud45c\uae30\ud558\ub294 \uac83\uc740?, context: \uacf5\ud559\uad50\uc721\uc778\uc99d\uc81c\ub3c4\ub85c \uc778\uc99d\ubc1b\uc740 \ud559\ubd80\uc640 \ud559\uacfc \ub610\ub294 \uc804\uacf5\uc740 \ud55c\uad6d\uacf5\ud559\uad50\uc721\uc778\uc99d\uc6d0\uc758 \uc778\uc99d \uae30\uc900\uc5d0 \ub9de\ucdb0 \uad50\uc721 \ud504\ub85c\uadf8\ub7a8\uc744 \uad6c\uc131\ud55c\ub2e4. \uacf5\ud559\uad50\uc721\uc778\uc99d\uc81c\ub3c4\ub97c \uc2dc\ud589\ud558\ub294 \uacf3\uc758 \ud559\uc0dd\uc740 \uba3c\uc800 \uc778\uc99d\ud504\ub85c\uadf8\ub7a8\uc5d0 \ub530\ub77c \uc804\uacf5\uc744 \uc774\uc218\ud558\ub294 \uc804\ubb38\uacfc\uc815\uacfc \uc778\uc99d\ud504\ub85c\uadf8\ub7a8\uc744 \ud3ec\uae30\ud558\ub294 \uc77c\ubc18\uacfc\uc815\uc744 \ub450\uace0 \uc120\ud0dd\ud560 \uc218 \uc788\ub2e4. \uc804\ubb38\uacfc\uc815\uc744 \uc774\uc218\ud558\ub294 \ud559\uc0dd\uc740 \ud55c\uad6d\uacf5\ud559\uad50\uc721\uc778\uc99d\uc6d0\uacfc \ud574\ub2f9 \uc778\uc99d\ud504\ub85c\uadf8\ub7a8\uc5d0\uc11c \uc81c\uc2dc\ud558\ub294 \uc778\uc99d\uae30\uc900\uc744 \ub3d9\uc2dc\uc5d0 \ucda9\uc871\ud558\uc5ec\uc57c \ud558\uba70, \uc815\ud574\uc9c4 \uc778\uc99d\ud504\ub85c\uadf8\ub7a8\uc758 \uad50\uacfc\ubaa9 \ucee4\ub9ac\ud058\ub7fc\uc5d0 \ub530\ub77c \uacfc\ubaa9\uc744 \uc774\uc218\ud574\uc57c \uacf5\ud559\uc778\uc99d\uc744 \ubc1b\uc744 \uc218 \uc788\ub2e4. \uc804\ubb38\uacfc\uc815\uc744 \uc774\uc218\ud55c \uc878\uc5c5\uc0dd\uc740 \ud559\uc801\ubd80\uc640 \uac01\uc885 \uc99d\uba85\uc11c \ub4f1\uc5d0 \uc778\uc99d \uc0ac\uc2e4\uc774 \uae30\uc7ac\ub418\uba70, \uc99d\uba85\uc11c\uc5d0\uc11c \uc804\uacf5\uc744 \ud45c\uae30\ud560 \ub54c, \uc778\uc99d\ud504\ub85c\uadf8\ub7a8 \uc774\uc218\uc790\ub294 \uc804\uacf5 \uba85\uce6d\uc5d0 \u2018\uc804\ubb38\u2019\uc744 \ucd94\uac00\ud558\uc5ec \ud45c\uae30\ud55c\ub2e4. \uadf8\ub7ec\ub098 \uc77c\ubc18\uacfc\uc815\uc744 \uc774\uc218\ud55c \uc878\uc5c5\uc0dd\uc740 \uc804\uacf5 \uba85\uce6d\uc5d0 \u2018\uc77c\ubc18\u2019\uc744 \ucd94\uac00 \uae30\uc7ac\ud574 \uc804\ubb38\uacfc\uc815 \uc774\uc218\uc790\uc640 \ucc28\uc774\ub97c \ub450\uac8c \ub41c\ub2e4."} {"question": "\ud53c\uc5b4\uc120\uc774 \uc601\ud5a5\uc744 \uc900 \uc774\uc2ac\ub78c \uc120\uad50\uc758 \uac1c\ucc99\uc790\ub294?", "paragraph": "\uadf8\ub294 \uadfc\ub300 \ubcf5\uc74c\uc8fc\uc758 \uc120\uad50\uc6b4\ub3d9 \uc774\ub860\uac00\uc600\uc73c\uba70 13,000\uac1c \uc774\uc0c1\uc758 \uc124\uad50\ubb38\uacfc \uc5f0\uc124\uc744 \uc9d1\ud544\ud558\uc600\uace0 60\uad8c \uc774\uc0c1\uc758 \ucc45\uc744 \uc37c\ub2e4. \ud2b9\ud788 \ud55c\uad6d\uc758 \uc5b8\ub354\uc6b0\ub4dc\uc640 \ubd81\uc7a5\ub85c\uad50 \ud574\uc678\uc120\uad50\ubd80 \ucd1d\ubb34\uc774\uc790 \ud504\ub9b0\uc2a4\ud134 \uc2e0\ud559\uad50\uc758 \uad50\ud68c\uc0ac\uac00 \ub85c\ubc84\ud2b8 \uc5d8\ub9ac\uc5c7 \uc2a4\ud53c\uc5b4(R.E.Speer) \uad50\uc218, \uc774\uc2ac\ub78c \uc120\uad50\uc758 \uac1c\ucc99\uc790 \uc0c8\ubba4\uc5bc \uc988\uc6e8\uba38(Samuel Zwemer)[1], \uae30\ub3c5\uad50\uccad\ub144\ud68c(YMCA) \uc6b4\ub3d9\uacfc \uc5d0\ud050\uba54\ub2c8\uceec\uc6b4\ub3d9\uc758 \uc9c0\ub3c4\uc790\ub85c \ud6c4\uc5d0 \ub178\ubca8\ud3c9\ud654\uc0c1\uc744 \uc218\uc0c1\ud55c \uc874 \ubaa8\ud2b8(John R. Mott), \ud55c\uad6d\uc758 \uc120\uad50\uc0ac\uc600\ub358 \uc81c\uc784\uc2a4 \uac8c\uc77c(J. S. Gale)\ubc15\uc0ac [2] \ub4f1 \uc218\ub9ce\uc740 \uc120\uad50\uc0ac\uc640 \ud559\uc790\ub4e4\uc5d0\uac8c \uc601\ud5a5\uc744 \uc8fc\uc5c8\ub2e4. \ud2b9\ud788 1887\ub144 \uc0c8\ubb38\uc548\uad50\ud68c\uac00 \uc2dc\uc791\ub41c \ud574\uc5d0 \uc5b8\ub354\uc6b0\ub4dc\ub294 \ud53c\uc5b4\uc120\uc5d0\uac8c \uc131\ub839\uc758 \ub2a5\ub825\uc73c\ub85c \uc608\ubc30\uc5d0 \ub9ce\uc740 \uc0ac\ub78c\ub4e4\uc774 \ucc38\uc5ec\ud558\uace0 \uc788\ub2e4\uace0 \ud3b8\uc9c0\ub97c \ubcf4\ub0b4\uba70 \uae4a\uc740 \uad50\uc81c\ub97c \uc2dc\uc791\ud558\uc600\uace0, \uadf8 \ud6c4 \ud53c\uc5b4\uc2a8\uc774 \ud3b8\uc9d1\uc7a5\uc73c\ub85c \uc788\ub358 \uc138\uacc4\uc120\uad50\ud3c9\ub860\uc758 \uc870\uc120\uac70\uc8fc \ud3b8\uc9d1\ud1b5\uc2e0\uc6d0\uc73c\ub85c \ud65c\ub3d9\ud558\uc600\ub2e4. \ud6c4\uc5d0 \uc5b8\ub354\uc6b0\ub4dc\ub294 \ud53c\uc5b4\uc120\uc131\uacbd\uae30\ub150\ud559\uc6d0\uc758 \ucd08\ub300 \uc774\uc0ac\uc7a5\uc73c\ub85c\uc11c \ud559\uad50\uc124\ub9bd\uc744 \uc8fc\ub3c4\ud558\uc600\uace0 \ucd08\ub300 \uc6d0\uc7a5\uc744 \uc9c0\ub0b4\uae30\ub3c4 \ud558\uc600\ub2e4.", "answer": "\uc0c8\ubba4\uc5bc \uc988\uc6e8\uba38", "sentence": "\ud2b9\ud788 \ud55c\uad6d\uc758 \uc5b8\ub354\uc6b0\ub4dc\uc640 \ubd81\uc7a5\ub85c\uad50 \ud574\uc678\uc120\uad50\ubd80 \ucd1d\ubb34\uc774\uc790 \ud504\ub9b0\uc2a4\ud134 \uc2e0\ud559\uad50\uc758 \uad50\ud68c\uc0ac\uac00 \ub85c\ubc84\ud2b8 \uc5d8\ub9ac\uc5c7 \uc2a4\ud53c\uc5b4(R.E.Speer) \uad50\uc218, \uc774\uc2ac\ub78c \uc120\uad50\uc758 \uac1c\ucc99\uc790 \uc0c8\ubba4\uc5bc \uc988\uc6e8\uba38 (Samuel Zwemer)[1], \uae30\ub3c5\uad50\uccad\ub144\ud68c(YMCA) \uc6b4\ub3d9\uacfc \uc5d0\ud050\uba54\ub2c8\uceec\uc6b4\ub3d9\uc758 \uc9c0\ub3c4\uc790\ub85c \ud6c4\uc5d0 \ub178\ubca8\ud3c9\ud654\uc0c1\uc744 \uc218\uc0c1\ud55c \uc874 \ubaa8\ud2b8(John R. Mott), \ud55c\uad6d\uc758 \uc120\uad50\uc0ac\uc600\ub358 \uc81c\uc784\uc2a4 \uac8c\uc77c(J. S. Gale)\ubc15\uc0ac [2] \ub4f1 \uc218\ub9ce\uc740 \uc120\uad50\uc0ac\uc640 \ud559\uc790\ub4e4\uc5d0\uac8c \uc601\ud5a5\uc744 \uc8fc\uc5c8\ub2e4.", "paragraph_sentence": "\uadf8\ub294 \uadfc\ub300 \ubcf5\uc74c\uc8fc\uc758 \uc120\uad50\uc6b4\ub3d9 \uc774\ub860\uac00\uc600\uc73c\uba70 13,000\uac1c \uc774\uc0c1\uc758 \uc124\uad50\ubb38\uacfc \uc5f0\uc124\uc744 \uc9d1\ud544\ud558\uc600\uace0 60\uad8c \uc774\uc0c1\uc758 \ucc45\uc744 \uc37c\ub2e4. \ud2b9\ud788 \ud55c\uad6d\uc758 \uc5b8\ub354\uc6b0\ub4dc\uc640 \ubd81\uc7a5\ub85c\uad50 \ud574\uc678\uc120\uad50\ubd80 \ucd1d\ubb34\uc774\uc790 \ud504\ub9b0\uc2a4\ud134 \uc2e0\ud559\uad50\uc758 \uad50\ud68c\uc0ac\uac00 \ub85c\ubc84\ud2b8 \uc5d8\ub9ac\uc5c7 \uc2a4\ud53c\uc5b4(R.E.Speer) \uad50\uc218, \uc774\uc2ac\ub78c \uc120\uad50\uc758 \uac1c\ucc99\uc790 \uc0c8\ubba4\uc5bc \uc988\uc6e8\uba38 (Samuel Zwemer)[1], \uae30\ub3c5\uad50\uccad\ub144\ud68c(YMCA) \uc6b4\ub3d9\uacfc \uc5d0\ud050\uba54\ub2c8\uceec\uc6b4\ub3d9\uc758 \uc9c0\ub3c4\uc790\ub85c \ud6c4\uc5d0 \ub178\ubca8\ud3c9\ud654\uc0c1\uc744 \uc218\uc0c1\ud55c \uc874 \ubaa8\ud2b8(John R. Mott), \ud55c\uad6d\uc758 \uc120\uad50\uc0ac\uc600\ub358 \uc81c\uc784\uc2a4 \uac8c\uc77c(J. S. Gale)\ubc15\uc0ac [2] \ub4f1 \uc218\ub9ce\uc740 \uc120\uad50\uc0ac\uc640 \ud559\uc790\ub4e4\uc5d0\uac8c \uc601\ud5a5\uc744 \uc8fc\uc5c8\ub2e4. \ud2b9\ud788 1887\ub144 \uc0c8\ubb38\uc548\uad50\ud68c\uac00 \uc2dc\uc791\ub41c \ud574\uc5d0 \uc5b8\ub354\uc6b0\ub4dc\ub294 \ud53c\uc5b4\uc120\uc5d0\uac8c \uc131\ub839\uc758 \ub2a5\ub825\uc73c\ub85c \uc608\ubc30\uc5d0 \ub9ce\uc740 \uc0ac\ub78c\ub4e4\uc774 \ucc38\uc5ec\ud558\uace0 \uc788\ub2e4\uace0 \ud3b8\uc9c0\ub97c \ubcf4\ub0b4\uba70 \uae4a\uc740 \uad50\uc81c\ub97c \uc2dc\uc791\ud558\uc600\uace0, \uadf8 \ud6c4 \ud53c\uc5b4\uc2a8\uc774 \ud3b8\uc9d1\uc7a5\uc73c\ub85c \uc788\ub358 \uc138\uacc4\uc120\uad50\ud3c9\ub860\uc758 \uc870\uc120\uac70\uc8fc \ud3b8\uc9d1\ud1b5\uc2e0\uc6d0\uc73c\ub85c \ud65c\ub3d9\ud558\uc600\ub2e4. \ud6c4\uc5d0 \uc5b8\ub354\uc6b0\ub4dc\ub294 \ud53c\uc5b4\uc120\uc131\uacbd\uae30\ub150\ud559\uc6d0\uc758 \ucd08\ub300 \uc774\uc0ac\uc7a5\uc73c\ub85c\uc11c \ud559\uad50\uc124\ub9bd\uc744 \uc8fc\ub3c4\ud558\uc600\uace0 \ucd08\ub300 \uc6d0\uc7a5\uc744 \uc9c0\ub0b4\uae30\ub3c4 \ud558\uc600\ub2e4.", "paragraph_answer": "\uadf8\ub294 \uadfc\ub300 \ubcf5\uc74c\uc8fc\uc758 \uc120\uad50\uc6b4\ub3d9 \uc774\ub860\uac00\uc600\uc73c\uba70 13,000\uac1c \uc774\uc0c1\uc758 \uc124\uad50\ubb38\uacfc \uc5f0\uc124\uc744 \uc9d1\ud544\ud558\uc600\uace0 60\uad8c \uc774\uc0c1\uc758 \ucc45\uc744 \uc37c\ub2e4. \ud2b9\ud788 \ud55c\uad6d\uc758 \uc5b8\ub354\uc6b0\ub4dc\uc640 \ubd81\uc7a5\ub85c\uad50 \ud574\uc678\uc120\uad50\ubd80 \ucd1d\ubb34\uc774\uc790 \ud504\ub9b0\uc2a4\ud134 \uc2e0\ud559\uad50\uc758 \uad50\ud68c\uc0ac\uac00 \ub85c\ubc84\ud2b8 \uc5d8\ub9ac\uc5c7 \uc2a4\ud53c\uc5b4(R.E.Speer) \uad50\uc218, \uc774\uc2ac\ub78c \uc120\uad50\uc758 \uac1c\ucc99\uc790 \uc0c8\ubba4\uc5bc \uc988\uc6e8\uba38 (Samuel Zwemer)[1], \uae30\ub3c5\uad50\uccad\ub144\ud68c(YMCA) \uc6b4\ub3d9\uacfc \uc5d0\ud050\uba54\ub2c8\uceec\uc6b4\ub3d9\uc758 \uc9c0\ub3c4\uc790\ub85c \ud6c4\uc5d0 \ub178\ubca8\ud3c9\ud654\uc0c1\uc744 \uc218\uc0c1\ud55c \uc874 \ubaa8\ud2b8(John R. Mott), \ud55c\uad6d\uc758 \uc120\uad50\uc0ac\uc600\ub358 \uc81c\uc784\uc2a4 \uac8c\uc77c(J. S. Gale)\ubc15\uc0ac [2] \ub4f1 \uc218\ub9ce\uc740 \uc120\uad50\uc0ac\uc640 \ud559\uc790\ub4e4\uc5d0\uac8c \uc601\ud5a5\uc744 \uc8fc\uc5c8\ub2e4. \ud2b9\ud788 1887\ub144 \uc0c8\ubb38\uc548\uad50\ud68c\uac00 \uc2dc\uc791\ub41c \ud574\uc5d0 \uc5b8\ub354\uc6b0\ub4dc\ub294 \ud53c\uc5b4\uc120\uc5d0\uac8c \uc131\ub839\uc758 \ub2a5\ub825\uc73c\ub85c \uc608\ubc30\uc5d0 \ub9ce\uc740 \uc0ac\ub78c\ub4e4\uc774 \ucc38\uc5ec\ud558\uace0 \uc788\ub2e4\uace0 \ud3b8\uc9c0\ub97c \ubcf4\ub0b4\uba70 \uae4a\uc740 \uad50\uc81c\ub97c \uc2dc\uc791\ud558\uc600\uace0, \uadf8 \ud6c4 \ud53c\uc5b4\uc2a8\uc774 \ud3b8\uc9d1\uc7a5\uc73c\ub85c \uc788\ub358 \uc138\uacc4\uc120\uad50\ud3c9\ub860\uc758 \uc870\uc120\uac70\uc8fc \ud3b8\uc9d1\ud1b5\uc2e0\uc6d0\uc73c\ub85c \ud65c\ub3d9\ud558\uc600\ub2e4. \ud6c4\uc5d0 \uc5b8\ub354\uc6b0\ub4dc\ub294 \ud53c\uc5b4\uc120\uc131\uacbd\uae30\ub150\ud559\uc6d0\uc758 \ucd08\ub300 \uc774\uc0ac\uc7a5\uc73c\ub85c\uc11c \ud559\uad50\uc124\ub9bd\uc744 \uc8fc\ub3c4\ud558\uc600\uace0 \ucd08\ub300 \uc6d0\uc7a5\uc744 \uc9c0\ub0b4\uae30\ub3c4 \ud558\uc600\ub2e4.", "sentence_answer": "\ud2b9\ud788 \ud55c\uad6d\uc758 \uc5b8\ub354\uc6b0\ub4dc\uc640 \ubd81\uc7a5\ub85c\uad50 \ud574\uc678\uc120\uad50\ubd80 \ucd1d\ubb34\uc774\uc790 \ud504\ub9b0\uc2a4\ud134 \uc2e0\ud559\uad50\uc758 \uad50\ud68c\uc0ac\uac00 \ub85c\ubc84\ud2b8 \uc5d8\ub9ac\uc5c7 \uc2a4\ud53c\uc5b4(R.E.Speer) \uad50\uc218, \uc774\uc2ac\ub78c \uc120\uad50\uc758 \uac1c\ucc99\uc790 \uc0c8\ubba4\uc5bc \uc988\uc6e8\uba38 (Samuel Zwemer)[1], \uae30\ub3c5\uad50\uccad\ub144\ud68c(YMCA) \uc6b4\ub3d9\uacfc \uc5d0\ud050\uba54\ub2c8\uceec\uc6b4\ub3d9\uc758 \uc9c0\ub3c4\uc790\ub85c \ud6c4\uc5d0 \ub178\ubca8\ud3c9\ud654\uc0c1\uc744 \uc218\uc0c1\ud55c \uc874 \ubaa8\ud2b8(John R. Mott), \ud55c\uad6d\uc758 \uc120\uad50\uc0ac\uc600\ub358 \uc81c\uc784\uc2a4 \uac8c\uc77c(J. S. Gale)\ubc15\uc0ac [2] \ub4f1 \uc218\ub9ce\uc740 \uc120\uad50\uc0ac\uc640 \ud559\uc790\ub4e4\uc5d0\uac8c \uc601\ud5a5\uc744 \uc8fc\uc5c8\ub2e4.", "paragraph_id": "6533524-0-0", "paragraph_question": "question: \ud53c\uc5b4\uc120\uc774 \uc601\ud5a5\uc744 \uc900 \uc774\uc2ac\ub78c \uc120\uad50\uc758 \uac1c\ucc99\uc790\ub294?, context: \uadf8\ub294 \uadfc\ub300 \ubcf5\uc74c\uc8fc\uc758 \uc120\uad50\uc6b4\ub3d9 \uc774\ub860\uac00\uc600\uc73c\uba70 13,000\uac1c \uc774\uc0c1\uc758 \uc124\uad50\ubb38\uacfc \uc5f0\uc124\uc744 \uc9d1\ud544\ud558\uc600\uace0 60\uad8c \uc774\uc0c1\uc758 \ucc45\uc744 \uc37c\ub2e4. \ud2b9\ud788 \ud55c\uad6d\uc758 \uc5b8\ub354\uc6b0\ub4dc\uc640 \ubd81\uc7a5\ub85c\uad50 \ud574\uc678\uc120\uad50\ubd80 \ucd1d\ubb34\uc774\uc790 \ud504\ub9b0\uc2a4\ud134 \uc2e0\ud559\uad50\uc758 \uad50\ud68c\uc0ac\uac00 \ub85c\ubc84\ud2b8 \uc5d8\ub9ac\uc5c7 \uc2a4\ud53c\uc5b4(R.E.Speer) \uad50\uc218, \uc774\uc2ac\ub78c \uc120\uad50\uc758 \uac1c\ucc99\uc790 \uc0c8\ubba4\uc5bc \uc988\uc6e8\uba38(Samuel Zwemer)[1], \uae30\ub3c5\uad50\uccad\ub144\ud68c(YMCA) \uc6b4\ub3d9\uacfc \uc5d0\ud050\uba54\ub2c8\uceec\uc6b4\ub3d9\uc758 \uc9c0\ub3c4\uc790\ub85c \ud6c4\uc5d0 \ub178\ubca8\ud3c9\ud654\uc0c1\uc744 \uc218\uc0c1\ud55c \uc874 \ubaa8\ud2b8(John R. Mott), \ud55c\uad6d\uc758 \uc120\uad50\uc0ac\uc600\ub358 \uc81c\uc784\uc2a4 \uac8c\uc77c(J. S. Gale)\ubc15\uc0ac [2] \ub4f1 \uc218\ub9ce\uc740 \uc120\uad50\uc0ac\uc640 \ud559\uc790\ub4e4\uc5d0\uac8c \uc601\ud5a5\uc744 \uc8fc\uc5c8\ub2e4. \ud2b9\ud788 1887\ub144 \uc0c8\ubb38\uc548\uad50\ud68c\uac00 \uc2dc\uc791\ub41c \ud574\uc5d0 \uc5b8\ub354\uc6b0\ub4dc\ub294 \ud53c\uc5b4\uc120\uc5d0\uac8c \uc131\ub839\uc758 \ub2a5\ub825\uc73c\ub85c \uc608\ubc30\uc5d0 \ub9ce\uc740 \uc0ac\ub78c\ub4e4\uc774 \ucc38\uc5ec\ud558\uace0 \uc788\ub2e4\uace0 \ud3b8\uc9c0\ub97c \ubcf4\ub0b4\uba70 \uae4a\uc740 \uad50\uc81c\ub97c \uc2dc\uc791\ud558\uc600\uace0, \uadf8 \ud6c4 \ud53c\uc5b4\uc2a8\uc774 \ud3b8\uc9d1\uc7a5\uc73c\ub85c \uc788\ub358 \uc138\uacc4\uc120\uad50\ud3c9\ub860\uc758 \uc870\uc120\uac70\uc8fc \ud3b8\uc9d1\ud1b5\uc2e0\uc6d0\uc73c\ub85c \ud65c\ub3d9\ud558\uc600\ub2e4. \ud6c4\uc5d0 \uc5b8\ub354\uc6b0\ub4dc\ub294 \ud53c\uc5b4\uc120\uc131\uacbd\uae30\ub150\ud559\uc6d0\uc758 \ucd08\ub300 \uc774\uc0ac\uc7a5\uc73c\ub85c\uc11c \ud559\uad50\uc124\ub9bd\uc744 \uc8fc\ub3c4\ud558\uc600\uace0 \ucd08\ub300 \uc6d0\uc7a5\uc744 \uc9c0\ub0b4\uae30\ub3c4 \ud558\uc600\ub2e4."} {"question": "1970\ub144\ub300 \ub2f9\uc2dc \uc0ac\ud68c\uc8fc\uc758\ud611\ud68c\uc758 \uc9c0\ub3c4\uc790\ub294?", "paragraph": "1970\ub144\ub300\uc5d0 \ub4e4\uc5b4\uc11c\uba74\uc11c \uc0ac\ud68c\ub2f9\uc740 \uc77c\ubcf8\uacf5\uc0b0\ub2f9\uae4c\uc9c0 \ud3ec\ud568\ud55c \ubaa8\ub4e0 \uc57c\ub2f9\uc758 \uacf5\ub3d9\ud22c\uc7c1 \ub178\uc120\uc744 \ucde8\ud574, \uc9c0\uc790\uccb4\uc5d0\uc11c \uacf5\uc0b0\ub2f9\uacfc \ud568\uaed8 \uc120\uac70\ub97c \uc9c4\ud589\uc2dc\ucf30\ub2e4. \uadf8 \uacb0\uacfc \ub3c4\ucfc4\ub3c4, \uc624\uc0ac\uce74\ubd80 \ub4f1 \uac01\uc9c0\uc5d0\uc11c \ud601\uc2e0 \uc9c0\uc790\uccb4\uc758 \uc218\uc7a5\uc774 \ud0c4\uc0dd\ud588\ub2e4. \uc774\ub7f0 \ud601\uc2e0 \uc9c0\uc790\uccb4\ub4e4\uc740 \uc0ac\ud68c \ubcf5\uc9c0\uc758 \uac15\ud654 \ub4f1 \uc77c\uc815\ud55c \uacb0\uacfc\ub97c \ub0a8\uacbc\uc9c0\ub9cc, \uc7ac\uc815\uc758 \uc545\ud654\ub97c \ubd88\ub800\ub2e4\ub294 \ube44\ud310\ub3c4 \ubc1b\uace0\uc788\ub2e4. \uc774\ub7f0 \uac00\uc6b4\ub370 \uc77c\ubcf8\uc0ac\ud68c\uc8fc\uc758\uccad\ub144\ub3d9\ub9f9\uc5d0 \uc138\ub825\uc744 \uac00\uc9c0\uace0 \uc788\ub358 \uadf9\uc88c\ud30c\ub97c \ubc30\uc81c\ud558\uba74\uc11c \uc0ac\ud68c\uc8fc\uc758\ud611\ud68c\uc758 \uc601\ud5a5\ub825\uc774 \uc870\uc9c1\uc5d0\uc11c\ub3c4 \uac15\ud574\uc9c0\uae30 \uc2dc\uc791\ud588\ub2e4. \uc0ac\ud0a4\uc0ac\uce74 \uc774\uce20\ub85c\ub97c \uc9c0\ub3c4\uc790\ub85c \ud55c \ub2f9\uc2dc\uc758 \uc0ac\ud68c\uc8fc\uc758\ud611\ud68c\ub294 \ub9c8\ub974\ud06c\uc2a4\ub098 \ub808\ub2cc \ub4f1\uc758 \uc800\uc791\uc744 \uc808\ub300\uc2dc\ud558\uba74\uc11c, \uc18c\ub828\uc744 \u2018\uc0ac\ud68c\uc8fc\uc758\uc758 \uc870\uad6d\u2019\uc73c\ub85c \uc0dd\uac01\ud558\uba70 \uccb4\ucf54 \uce68\uacf5\uc744 \uacf5\uacf5\uc5f0\ud558\uac8c \uc9c0\uc9c0\ud558\ub294 \ub4f1, \uc0ac\ud68c\ub2f9\uc774 \uc81c\uc2dc\ud55c \ub2f9\uc2dc\uc758 \uc911\ub9bd\uc815\ucc45\uc774 \uc6a9\ub0a9\ud560 \uc218 \uc5c6\ub294 \uc9c0\uacbd\uc5d0\uae4c\uc9c0 \uc774\ub974\ub800\ub2e4. \uce5c\uc18c \uacbd\ud5a5\uc758 \uc0ac\ud68c\uc8fc\uc758\ud611\ud68c \uc138\ub825\uc774 \ud655\ub300\ub418\uba74\uc11c, \uc0ac\uc0ac\ud0a4 \uace0\uc870 \ub4f1\uc740 \uc608\uc804\uc5d0 \uc801\ub300\uc2dc\ud558\ub358 \uc5d0\ub2e4 \uc0ac\ubd80\ub85c\uc640 \uc5f0\ud569\ud558\uba74\uc11c, \uc774\ud6c4 \uc0ac\ud68c\uc8fc\uc758\ud611\ud68c \uc138\ub825\uacfc \ubc18\ud611\ud68c \uc138\ub825\uc758 \ub300\ub9bd\uc740 \uaca9\ud654\ub418\uc5b4 \uac14\ub2e4.", "answer": "\uc0ac\ud0a4\uc0ac\uce74 \uc774\uce20\ub85c", "sentence": "\uc0ac\ud0a4\uc0ac\uce74 \uc774\uce20\ub85c \ub97c \uc9c0\ub3c4\uc790\ub85c \ud55c \ub2f9\uc2dc\uc758 \uc0ac\ud68c\uc8fc\uc758\ud611\ud68c\ub294 \ub9c8\ub974\ud06c\uc2a4\ub098 \ub808\ub2cc \ub4f1\uc758 \uc800\uc791\uc744 \uc808\ub300\uc2dc\ud558\uba74\uc11c, \uc18c\ub828\uc744 \u2018\uc0ac\ud68c\uc8fc\uc758\uc758 \uc870\uad6d\u2019\uc73c\ub85c \uc0dd\uac01\ud558\uba70 \uccb4\ucf54 \uce68\uacf5\uc744 \uacf5\uacf5\uc5f0\ud558\uac8c \uc9c0\uc9c0\ud558\ub294 \ub4f1, \uc0ac\ud68c\ub2f9\uc774 \uc81c\uc2dc\ud55c \ub2f9\uc2dc\uc758 \uc911\ub9bd\uc815\ucc45\uc774 \uc6a9\ub0a9\ud560 \uc218 \uc5c6\ub294 \uc9c0\uacbd\uc5d0\uae4c\uc9c0 \uc774\ub974\ub800\ub2e4.", "paragraph_sentence": "1970\ub144\ub300\uc5d0 \ub4e4\uc5b4\uc11c\uba74\uc11c \uc0ac\ud68c\ub2f9\uc740 \uc77c\ubcf8\uacf5\uc0b0\ub2f9\uae4c\uc9c0 \ud3ec\ud568\ud55c \ubaa8\ub4e0 \uc57c\ub2f9\uc758 \uacf5\ub3d9\ud22c\uc7c1 \ub178\uc120\uc744 \ucde8\ud574, \uc9c0\uc790\uccb4\uc5d0\uc11c \uacf5\uc0b0\ub2f9\uacfc \ud568\uaed8 \uc120\uac70\ub97c \uc9c4\ud589\uc2dc\ucf30\ub2e4. \uadf8 \uacb0\uacfc \ub3c4\ucfc4\ub3c4, \uc624\uc0ac\uce74\ubd80 \ub4f1 \uac01\uc9c0\uc5d0\uc11c \ud601\uc2e0 \uc9c0\uc790\uccb4\uc758 \uc218\uc7a5\uc774 \ud0c4\uc0dd\ud588\ub2e4. \uc774\ub7f0 \ud601\uc2e0 \uc9c0\uc790\uccb4\ub4e4\uc740 \uc0ac\ud68c \ubcf5\uc9c0\uc758 \uac15\ud654 \ub4f1 \uc77c\uc815\ud55c \uacb0\uacfc\ub97c \ub0a8\uacbc\uc9c0\ub9cc, \uc7ac\uc815\uc758 \uc545\ud654\ub97c \ubd88\ub800\ub2e4\ub294 \ube44\ud310\ub3c4 \ubc1b\uace0\uc788\ub2e4. \uc774\ub7f0 \uac00\uc6b4\ub370 \uc77c\ubcf8\uc0ac\ud68c\uc8fc\uc758\uccad\ub144\ub3d9\ub9f9\uc5d0 \uc138\ub825\uc744 \uac00\uc9c0\uace0 \uc788\ub358 \uadf9\uc88c\ud30c\ub97c \ubc30\uc81c\ud558\uba74\uc11c \uc0ac\ud68c\uc8fc\uc758\ud611\ud68c\uc758 \uc601\ud5a5\ub825\uc774 \uc870\uc9c1\uc5d0\uc11c\ub3c4 \uac15\ud574\uc9c0\uae30 \uc2dc\uc791\ud588\ub2e4. \uc0ac\ud0a4\uc0ac\uce74 \uc774\uce20\ub85c \ub97c \uc9c0\ub3c4\uc790\ub85c \ud55c \ub2f9\uc2dc\uc758 \uc0ac\ud68c\uc8fc\uc758\ud611\ud68c\ub294 \ub9c8\ub974\ud06c\uc2a4\ub098 \ub808\ub2cc \ub4f1\uc758 \uc800\uc791\uc744 \uc808\ub300\uc2dc\ud558\uba74\uc11c, \uc18c\ub828\uc744 \u2018\uc0ac\ud68c\uc8fc\uc758\uc758 \uc870\uad6d\u2019\uc73c\ub85c \uc0dd\uac01\ud558\uba70 \uccb4\ucf54 \uce68\uacf5\uc744 \uacf5\uacf5\uc5f0\ud558\uac8c \uc9c0\uc9c0\ud558\ub294 \ub4f1, \uc0ac\ud68c\ub2f9\uc774 \uc81c\uc2dc\ud55c \ub2f9\uc2dc\uc758 \uc911\ub9bd\uc815\ucc45\uc774 \uc6a9\ub0a9\ud560 \uc218 \uc5c6\ub294 \uc9c0\uacbd\uc5d0\uae4c\uc9c0 \uc774\ub974\ub800\ub2e4. \uce5c\uc18c \uacbd\ud5a5\uc758 \uc0ac\ud68c\uc8fc\uc758\ud611\ud68c \uc138\ub825\uc774 \ud655\ub300\ub418\uba74\uc11c, \uc0ac\uc0ac\ud0a4 \uace0\uc870 \ub4f1\uc740 \uc608\uc804\uc5d0 \uc801\ub300\uc2dc\ud558\ub358 \uc5d0\ub2e4 \uc0ac\ubd80\ub85c\uc640 \uc5f0\ud569\ud558\uba74\uc11c, \uc774\ud6c4 \uc0ac\ud68c\uc8fc\uc758\ud611\ud68c \uc138\ub825\uacfc \ubc18\ud611\ud68c \uc138\ub825\uc758 \ub300\ub9bd\uc740 \uaca9\ud654\ub418\uc5b4 \uac14\ub2e4.", "paragraph_answer": "1970\ub144\ub300\uc5d0 \ub4e4\uc5b4\uc11c\uba74\uc11c \uc0ac\ud68c\ub2f9\uc740 \uc77c\ubcf8\uacf5\uc0b0\ub2f9\uae4c\uc9c0 \ud3ec\ud568\ud55c \ubaa8\ub4e0 \uc57c\ub2f9\uc758 \uacf5\ub3d9\ud22c\uc7c1 \ub178\uc120\uc744 \ucde8\ud574, \uc9c0\uc790\uccb4\uc5d0\uc11c \uacf5\uc0b0\ub2f9\uacfc \ud568\uaed8 \uc120\uac70\ub97c \uc9c4\ud589\uc2dc\ucf30\ub2e4. \uadf8 \uacb0\uacfc \ub3c4\ucfc4\ub3c4, \uc624\uc0ac\uce74\ubd80 \ub4f1 \uac01\uc9c0\uc5d0\uc11c \ud601\uc2e0 \uc9c0\uc790\uccb4\uc758 \uc218\uc7a5\uc774 \ud0c4\uc0dd\ud588\ub2e4. \uc774\ub7f0 \ud601\uc2e0 \uc9c0\uc790\uccb4\ub4e4\uc740 \uc0ac\ud68c \ubcf5\uc9c0\uc758 \uac15\ud654 \ub4f1 \uc77c\uc815\ud55c \uacb0\uacfc\ub97c \ub0a8\uacbc\uc9c0\ub9cc, \uc7ac\uc815\uc758 \uc545\ud654\ub97c \ubd88\ub800\ub2e4\ub294 \ube44\ud310\ub3c4 \ubc1b\uace0\uc788\ub2e4. \uc774\ub7f0 \uac00\uc6b4\ub370 \uc77c\ubcf8\uc0ac\ud68c\uc8fc\uc758\uccad\ub144\ub3d9\ub9f9\uc5d0 \uc138\ub825\uc744 \uac00\uc9c0\uace0 \uc788\ub358 \uadf9\uc88c\ud30c\ub97c \ubc30\uc81c\ud558\uba74\uc11c \uc0ac\ud68c\uc8fc\uc758\ud611\ud68c\uc758 \uc601\ud5a5\ub825\uc774 \uc870\uc9c1\uc5d0\uc11c\ub3c4 \uac15\ud574\uc9c0\uae30 \uc2dc\uc791\ud588\ub2e4. \uc0ac\ud0a4\uc0ac\uce74 \uc774\uce20\ub85c \ub97c \uc9c0\ub3c4\uc790\ub85c \ud55c \ub2f9\uc2dc\uc758 \uc0ac\ud68c\uc8fc\uc758\ud611\ud68c\ub294 \ub9c8\ub974\ud06c\uc2a4\ub098 \ub808\ub2cc \ub4f1\uc758 \uc800\uc791\uc744 \uc808\ub300\uc2dc\ud558\uba74\uc11c, \uc18c\ub828\uc744 \u2018\uc0ac\ud68c\uc8fc\uc758\uc758 \uc870\uad6d\u2019\uc73c\ub85c \uc0dd\uac01\ud558\uba70 \uccb4\ucf54 \uce68\uacf5\uc744 \uacf5\uacf5\uc5f0\ud558\uac8c \uc9c0\uc9c0\ud558\ub294 \ub4f1, \uc0ac\ud68c\ub2f9\uc774 \uc81c\uc2dc\ud55c \ub2f9\uc2dc\uc758 \uc911\ub9bd\uc815\ucc45\uc774 \uc6a9\ub0a9\ud560 \uc218 \uc5c6\ub294 \uc9c0\uacbd\uc5d0\uae4c\uc9c0 \uc774\ub974\ub800\ub2e4. \uce5c\uc18c \uacbd\ud5a5\uc758 \uc0ac\ud68c\uc8fc\uc758\ud611\ud68c \uc138\ub825\uc774 \ud655\ub300\ub418\uba74\uc11c, \uc0ac\uc0ac\ud0a4 \uace0\uc870 \ub4f1\uc740 \uc608\uc804\uc5d0 \uc801\ub300\uc2dc\ud558\ub358 \uc5d0\ub2e4 \uc0ac\ubd80\ub85c\uc640 \uc5f0\ud569\ud558\uba74\uc11c, \uc774\ud6c4 \uc0ac\ud68c\uc8fc\uc758\ud611\ud68c \uc138\ub825\uacfc \ubc18\ud611\ud68c \uc138\ub825\uc758 \ub300\ub9bd\uc740 \uaca9\ud654\ub418\uc5b4 \uac14\ub2e4.", "sentence_answer": " \uc0ac\ud0a4\uc0ac\uce74 \uc774\uce20\ub85c \ub97c \uc9c0\ub3c4\uc790\ub85c \ud55c \ub2f9\uc2dc\uc758 \uc0ac\ud68c\uc8fc\uc758\ud611\ud68c\ub294 \ub9c8\ub974\ud06c\uc2a4\ub098 \ub808\ub2cc \ub4f1\uc758 \uc800\uc791\uc744 \uc808\ub300\uc2dc\ud558\uba74\uc11c, \uc18c\ub828\uc744 \u2018\uc0ac\ud68c\uc8fc\uc758\uc758 \uc870\uad6d\u2019\uc73c\ub85c \uc0dd\uac01\ud558\uba70 \uccb4\ucf54 \uce68\uacf5\uc744 \uacf5\uacf5\uc5f0\ud558\uac8c \uc9c0\uc9c0\ud558\ub294 \ub4f1, \uc0ac\ud68c\ub2f9\uc774 \uc81c\uc2dc\ud55c \ub2f9\uc2dc\uc758 \uc911\ub9bd\uc815\ucc45\uc774 \uc6a9\ub0a9\ud560 \uc218 \uc5c6\ub294 \uc9c0\uacbd\uc5d0\uae4c\uc9c0 \uc774\ub974\ub800\ub2e4.", "paragraph_id": "6548574-3-1", "paragraph_question": "question: 1970\ub144\ub300 \ub2f9\uc2dc \uc0ac\ud68c\uc8fc\uc758\ud611\ud68c\uc758 \uc9c0\ub3c4\uc790\ub294?, context: 1970\ub144\ub300\uc5d0 \ub4e4\uc5b4\uc11c\uba74\uc11c \uc0ac\ud68c\ub2f9\uc740 \uc77c\ubcf8\uacf5\uc0b0\ub2f9\uae4c\uc9c0 \ud3ec\ud568\ud55c \ubaa8\ub4e0 \uc57c\ub2f9\uc758 \uacf5\ub3d9\ud22c\uc7c1 \ub178\uc120\uc744 \ucde8\ud574, \uc9c0\uc790\uccb4\uc5d0\uc11c \uacf5\uc0b0\ub2f9\uacfc \ud568\uaed8 \uc120\uac70\ub97c \uc9c4\ud589\uc2dc\ucf30\ub2e4. \uadf8 \uacb0\uacfc \ub3c4\ucfc4\ub3c4, \uc624\uc0ac\uce74\ubd80 \ub4f1 \uac01\uc9c0\uc5d0\uc11c \ud601\uc2e0 \uc9c0\uc790\uccb4\uc758 \uc218\uc7a5\uc774 \ud0c4\uc0dd\ud588\ub2e4. \uc774\ub7f0 \ud601\uc2e0 \uc9c0\uc790\uccb4\ub4e4\uc740 \uc0ac\ud68c \ubcf5\uc9c0\uc758 \uac15\ud654 \ub4f1 \uc77c\uc815\ud55c \uacb0\uacfc\ub97c \ub0a8\uacbc\uc9c0\ub9cc, \uc7ac\uc815\uc758 \uc545\ud654\ub97c \ubd88\ub800\ub2e4\ub294 \ube44\ud310\ub3c4 \ubc1b\uace0\uc788\ub2e4. \uc774\ub7f0 \uac00\uc6b4\ub370 \uc77c\ubcf8\uc0ac\ud68c\uc8fc\uc758\uccad\ub144\ub3d9\ub9f9\uc5d0 \uc138\ub825\uc744 \uac00\uc9c0\uace0 \uc788\ub358 \uadf9\uc88c\ud30c\ub97c \ubc30\uc81c\ud558\uba74\uc11c \uc0ac\ud68c\uc8fc\uc758\ud611\ud68c\uc758 \uc601\ud5a5\ub825\uc774 \uc870\uc9c1\uc5d0\uc11c\ub3c4 \uac15\ud574\uc9c0\uae30 \uc2dc\uc791\ud588\ub2e4. \uc0ac\ud0a4\uc0ac\uce74 \uc774\uce20\ub85c\ub97c \uc9c0\ub3c4\uc790\ub85c \ud55c \ub2f9\uc2dc\uc758 \uc0ac\ud68c\uc8fc\uc758\ud611\ud68c\ub294 \ub9c8\ub974\ud06c\uc2a4\ub098 \ub808\ub2cc \ub4f1\uc758 \uc800\uc791\uc744 \uc808\ub300\uc2dc\ud558\uba74\uc11c, \uc18c\ub828\uc744 \u2018\uc0ac\ud68c\uc8fc\uc758\uc758 \uc870\uad6d\u2019\uc73c\ub85c \uc0dd\uac01\ud558\uba70 \uccb4\ucf54 \uce68\uacf5\uc744 \uacf5\uacf5\uc5f0\ud558\uac8c \uc9c0\uc9c0\ud558\ub294 \ub4f1, \uc0ac\ud68c\ub2f9\uc774 \uc81c\uc2dc\ud55c \ub2f9\uc2dc\uc758 \uc911\ub9bd\uc815\ucc45\uc774 \uc6a9\ub0a9\ud560 \uc218 \uc5c6\ub294 \uc9c0\uacbd\uc5d0\uae4c\uc9c0 \uc774\ub974\ub800\ub2e4. \uce5c\uc18c \uacbd\ud5a5\uc758 \uc0ac\ud68c\uc8fc\uc758\ud611\ud68c \uc138\ub825\uc774 \ud655\ub300\ub418\uba74\uc11c, \uc0ac\uc0ac\ud0a4 \uace0\uc870 \ub4f1\uc740 \uc608\uc804\uc5d0 \uc801\ub300\uc2dc\ud558\ub358 \uc5d0\ub2e4 \uc0ac\ubd80\ub85c\uc640 \uc5f0\ud569\ud558\uba74\uc11c, \uc774\ud6c4 \uc0ac\ud68c\uc8fc\uc758\ud611\ud68c \uc138\ub825\uacfc \ubc18\ud611\ud68c \uc138\ub825\uc758 \ub300\ub9bd\uc740 \uaca9\ud654\ub418\uc5b4 \uac14\ub2e4."} {"question": "\uc591\uc801\uc5f0\uad6c \ubc29\ubc95\uc744 \ud1b5\ud574 \uacc4\ub7c9\uc801\uc778 \ubc29\ubc95\uc73c\ub85c \uc0ac\ud68c \uacc4\uce35 \uc0ac\ud68c \uc774\ub3d9\uc131 \ub4f1\uc744 \ud0d0\uad6c\ud558\ub294 \ud559\ubb38\uc740?", "paragraph": "\uc0ac\ud68c\ud559\uc740 \ubc29\ubc95\ub860\uc774\ub098 \ud0d0\uad6c \uc8fc\uc81c\uba74\uc5d0\uc11c \ub9e4\uc6b0 \uad11\ubc94\uc704\ud55c \ud559\ubb38\uc774\ub2e4. \uc804\ud1b5\uc801\uc73c\ub85c \uc0ac\ud68c\ud559\uc740 \uc591\uc801 \uc5f0\uad6c \ubc29\ubc95\uc744 \ud1b5\ud558\uc5ec \uacc4\ub7c9\uc801\uc778 \ubc29\ubc95\uc73c\ub85c \uacc4\uae09\uacfc \uac19\uc740 \uc0ac\ud68c \uacc4\uce35, \uc0ac\ud68c \uc774\ub3d9\uc131, \uc885\uad50, \uc138\uc18d\ud654, \ubc95\ub960, \ubb38\ud654, \uc77c\ud0c8, \uadfc\ub300\uc131 \ub4f1\uc744 \uc5f0\uad6c\ud588\ub2e4. \uadfc\ub798\uc5d0 \ub4e4\uc5b4\uc11c \uc0ac\ud68c\ud559\uc758 \uc5f0\uad6c \ubd84\uc57c\ub294 \uc758\ud559, \uad70\uc0ac, \ucc98\ubc8c\uacfc \uac19\uc740 \uac01\uc885 \uc0ac\ud68c \uc81c\ub3c4\uc640 \uc778\ud130\ub137\uacfc \uac19\uc740 \uc0c8\ub85c\uc6b4 \uc0ac\ud68c \uad00\uacc4, \uacfc\ud559 \uc9c0\uc2dd\uacfc \uac19\uc740 \uc9c0\uc2dd \uccb4\uacc4\uc5d0 \uae4c\uc9c0 \ud655\uc7a5\ub418\uace0 \uc788\ub2e4. \uc5f0\uad6c \ubd84\uc57c\uac00 \uad11\ubc94\uc704 \ud55c \ub9cc\ud07c \uc5f0\uad6c\uc758 \uae30\ubc95 \uc5ed\uc2dc \ub2e4\uc591\ud558\uc5ec \uc5b8\uc5b4\uc801 \ubc29\ubc95, \ubb38\ud654\uc801 \ubc29\ubc95, \uc774\ud574\uc0ac\ud68c\ud559, \ucca0\ud559\uc801 \ubc29\ubc95 \ub4f1\uc774 \uc0ac\ud68c\ud559 \uc5f0\uad6c\uc5d0 \uc0ac\uc6a9\ub418\uace0 \uc788\ub2e4. \ucd5c\uadfc\uc5d0\ub294 \uc5d0\uc774\uc804\ud2b8 \uae30\ubc18 \ubaa8\ud615\uc774\ub098 \uc0ac\ud68c \ub124\ud2b8\uc6cc\ud06c\uc758 \ubd84\uc11d\uc744 \uc704\ud574 \uc218\ud559\uc801 \ubc29\ubc95\uacfc \ucef4\ud4e8\ud130 \uc5f0\uc0b0 \uae30\ubc95 \ub4f1\uc774 \uc0ac\uc6a9\ub418\uace0 \uc788\ub2e4.", "answer": "\uc0ac\ud68c\ud559", "sentence": "\uc0ac\ud68c\ud559 \uc740 \ubc29\ubc95\ub860\uc774\ub098 \ud0d0\uad6c \uc8fc\uc81c\uba74\uc5d0\uc11c \ub9e4\uc6b0 \uad11\ubc94\uc704\ud55c \ud559\ubb38\uc774\ub2e4.", "paragraph_sentence": " \uc0ac\ud68c\ud559 \uc740 \ubc29\ubc95\ub860\uc774\ub098 \ud0d0\uad6c \uc8fc\uc81c\uba74\uc5d0\uc11c \ub9e4\uc6b0 \uad11\ubc94\uc704\ud55c \ud559\ubb38\uc774\ub2e4. \uc804\ud1b5\uc801\uc73c\ub85c \uc0ac\ud68c\ud559\uc740 \uc591\uc801 \uc5f0\uad6c \ubc29\ubc95\uc744 \ud1b5\ud558\uc5ec \uacc4\ub7c9\uc801\uc778 \ubc29\ubc95\uc73c\ub85c \uacc4\uae09\uacfc \uac19\uc740 \uc0ac\ud68c \uacc4\uce35, \uc0ac\ud68c \uc774\ub3d9\uc131, \uc885\uad50, \uc138\uc18d\ud654, \ubc95\ub960, \ubb38\ud654, \uc77c\ud0c8, \uadfc\ub300\uc131 \ub4f1\uc744 \uc5f0\uad6c\ud588\ub2e4. \uadfc\ub798\uc5d0 \ub4e4\uc5b4\uc11c \uc0ac\ud68c\ud559\uc758 \uc5f0\uad6c \ubd84\uc57c\ub294 \uc758\ud559, \uad70\uc0ac, \ucc98\ubc8c\uacfc \uac19\uc740 \uac01\uc885 \uc0ac\ud68c \uc81c\ub3c4\uc640 \uc778\ud130\ub137\uacfc \uac19\uc740 \uc0c8\ub85c\uc6b4 \uc0ac\ud68c \uad00\uacc4, \uacfc\ud559 \uc9c0\uc2dd\uacfc \uac19\uc740 \uc9c0\uc2dd \uccb4\uacc4\uc5d0 \uae4c\uc9c0 \ud655\uc7a5\ub418\uace0 \uc788\ub2e4. \uc5f0\uad6c \ubd84\uc57c\uac00 \uad11\ubc94\uc704 \ud55c \ub9cc\ud07c \uc5f0\uad6c\uc758 \uae30\ubc95 \uc5ed\uc2dc \ub2e4\uc591\ud558\uc5ec \uc5b8\uc5b4\uc801 \ubc29\ubc95, \ubb38\ud654\uc801 \ubc29\ubc95, \uc774\ud574\uc0ac\ud68c\ud559, \ucca0\ud559\uc801 \ubc29\ubc95 \ub4f1\uc774 \uc0ac\ud68c\ud559 \uc5f0\uad6c\uc5d0 \uc0ac\uc6a9\ub418\uace0 \uc788\ub2e4. \ucd5c\uadfc\uc5d0\ub294 \uc5d0\uc774\uc804\ud2b8 \uae30\ubc18 \ubaa8\ud615\uc774\ub098 \uc0ac\ud68c \ub124\ud2b8\uc6cc\ud06c\uc758 \ubd84\uc11d\uc744 \uc704\ud574 \uc218\ud559\uc801 \ubc29\ubc95\uacfc \ucef4\ud4e8\ud130 \uc5f0\uc0b0 \uae30\ubc95 \ub4f1\uc774 \uc0ac\uc6a9\ub418\uace0 \uc788\ub2e4.", "paragraph_answer": " \uc0ac\ud68c\ud559 \uc740 \ubc29\ubc95\ub860\uc774\ub098 \ud0d0\uad6c \uc8fc\uc81c\uba74\uc5d0\uc11c \ub9e4\uc6b0 \uad11\ubc94\uc704\ud55c \ud559\ubb38\uc774\ub2e4. \uc804\ud1b5\uc801\uc73c\ub85c \uc0ac\ud68c\ud559\uc740 \uc591\uc801 \uc5f0\uad6c \ubc29\ubc95\uc744 \ud1b5\ud558\uc5ec \uacc4\ub7c9\uc801\uc778 \ubc29\ubc95\uc73c\ub85c \uacc4\uae09\uacfc \uac19\uc740 \uc0ac\ud68c \uacc4\uce35, \uc0ac\ud68c \uc774\ub3d9\uc131, \uc885\uad50, \uc138\uc18d\ud654, \ubc95\ub960, \ubb38\ud654, \uc77c\ud0c8, \uadfc\ub300\uc131 \ub4f1\uc744 \uc5f0\uad6c\ud588\ub2e4. \uadfc\ub798\uc5d0 \ub4e4\uc5b4\uc11c \uc0ac\ud68c\ud559\uc758 \uc5f0\uad6c \ubd84\uc57c\ub294 \uc758\ud559, \uad70\uc0ac, \ucc98\ubc8c\uacfc \uac19\uc740 \uac01\uc885 \uc0ac\ud68c \uc81c\ub3c4\uc640 \uc778\ud130\ub137\uacfc \uac19\uc740 \uc0c8\ub85c\uc6b4 \uc0ac\ud68c \uad00\uacc4, \uacfc\ud559 \uc9c0\uc2dd\uacfc \uac19\uc740 \uc9c0\uc2dd \uccb4\uacc4\uc5d0 \uae4c\uc9c0 \ud655\uc7a5\ub418\uace0 \uc788\ub2e4. \uc5f0\uad6c \ubd84\uc57c\uac00 \uad11\ubc94\uc704 \ud55c \ub9cc\ud07c \uc5f0\uad6c\uc758 \uae30\ubc95 \uc5ed\uc2dc \ub2e4\uc591\ud558\uc5ec \uc5b8\uc5b4\uc801 \ubc29\ubc95, \ubb38\ud654\uc801 \ubc29\ubc95, \uc774\ud574\uc0ac\ud68c\ud559, \ucca0\ud559\uc801 \ubc29\ubc95 \ub4f1\uc774 \uc0ac\ud68c\ud559 \uc5f0\uad6c\uc5d0 \uc0ac\uc6a9\ub418\uace0 \uc788\ub2e4. \ucd5c\uadfc\uc5d0\ub294 \uc5d0\uc774\uc804\ud2b8 \uae30\ubc18 \ubaa8\ud615\uc774\ub098 \uc0ac\ud68c \ub124\ud2b8\uc6cc\ud06c\uc758 \ubd84\uc11d\uc744 \uc704\ud574 \uc218\ud559\uc801 \ubc29\ubc95\uacfc \ucef4\ud4e8\ud130 \uc5f0\uc0b0 \uae30\ubc95 \ub4f1\uc774 \uc0ac\uc6a9\ub418\uace0 \uc788\ub2e4.", "sentence_answer": " \uc0ac\ud68c\ud559 \uc740 \ubc29\ubc95\ub860\uc774\ub098 \ud0d0\uad6c \uc8fc\uc81c\uba74\uc5d0\uc11c \ub9e4\uc6b0 \uad11\ubc94\uc704\ud55c \ud559\ubb38\uc774\ub2e4.", "paragraph_id": "6435766-1-0", "paragraph_question": "question: \uc591\uc801\uc5f0\uad6c \ubc29\ubc95\uc744 \ud1b5\ud574 \uacc4\ub7c9\uc801\uc778 \ubc29\ubc95\uc73c\ub85c \uc0ac\ud68c \uacc4\uce35 \uc0ac\ud68c \uc774\ub3d9\uc131 \ub4f1\uc744 \ud0d0\uad6c\ud558\ub294 \ud559\ubb38\uc740?, context: \uc0ac\ud68c\ud559\uc740 \ubc29\ubc95\ub860\uc774\ub098 \ud0d0\uad6c \uc8fc\uc81c\uba74\uc5d0\uc11c \ub9e4\uc6b0 \uad11\ubc94\uc704\ud55c \ud559\ubb38\uc774\ub2e4. \uc804\ud1b5\uc801\uc73c\ub85c \uc0ac\ud68c\ud559\uc740 \uc591\uc801 \uc5f0\uad6c \ubc29\ubc95\uc744 \ud1b5\ud558\uc5ec \uacc4\ub7c9\uc801\uc778 \ubc29\ubc95\uc73c\ub85c \uacc4\uae09\uacfc \uac19\uc740 \uc0ac\ud68c \uacc4\uce35, \uc0ac\ud68c \uc774\ub3d9\uc131, \uc885\uad50, \uc138\uc18d\ud654, \ubc95\ub960, \ubb38\ud654, \uc77c\ud0c8, \uadfc\ub300\uc131 \ub4f1\uc744 \uc5f0\uad6c\ud588\ub2e4. \uadfc\ub798\uc5d0 \ub4e4\uc5b4\uc11c \uc0ac\ud68c\ud559\uc758 \uc5f0\uad6c \ubd84\uc57c\ub294 \uc758\ud559, \uad70\uc0ac, \ucc98\ubc8c\uacfc \uac19\uc740 \uac01\uc885 \uc0ac\ud68c \uc81c\ub3c4\uc640 \uc778\ud130\ub137\uacfc \uac19\uc740 \uc0c8\ub85c\uc6b4 \uc0ac\ud68c \uad00\uacc4, \uacfc\ud559 \uc9c0\uc2dd\uacfc \uac19\uc740 \uc9c0\uc2dd \uccb4\uacc4\uc5d0 \uae4c\uc9c0 \ud655\uc7a5\ub418\uace0 \uc788\ub2e4. \uc5f0\uad6c \ubd84\uc57c\uac00 \uad11\ubc94\uc704 \ud55c \ub9cc\ud07c \uc5f0\uad6c\uc758 \uae30\ubc95 \uc5ed\uc2dc \ub2e4\uc591\ud558\uc5ec \uc5b8\uc5b4\uc801 \ubc29\ubc95, \ubb38\ud654\uc801 \ubc29\ubc95, \uc774\ud574\uc0ac\ud68c\ud559, \ucca0\ud559\uc801 \ubc29\ubc95 \ub4f1\uc774 \uc0ac\ud68c\ud559 \uc5f0\uad6c\uc5d0 \uc0ac\uc6a9\ub418\uace0 \uc788\ub2e4. \ucd5c\uadfc\uc5d0\ub294 \uc5d0\uc774\uc804\ud2b8 \uae30\ubc18 \ubaa8\ud615\uc774\ub098 \uc0ac\ud68c \ub124\ud2b8\uc6cc\ud06c\uc758 \ubd84\uc11d\uc744 \uc704\ud574 \uc218\ud559\uc801 \ubc29\ubc95\uacfc \ucef4\ud4e8\ud130 \uc5f0\uc0b0 \uae30\ubc95 \ub4f1\uc774 \uc0ac\uc6a9\ub418\uace0 \uc788\ub2e4."} {"question": "2005\ub144 7\uc6d4 3\uc77c \uc804\ubd81 \ud604\ub300 \ubaa8\ud130\uc2a4\uc758 \uac10\ub3c5\uc73c\ub85c \ubd80\uc784\ud55c \uac83\uc740 \ub204\uad6c\uc778\uac00?", "paragraph": "2005\ub144 7\uc6d4 3\uc77c, \uc870\uc724\ud658\uc758 \ud6c4\uc784\uc73c\ub85c \uc804\ubd81 \ud604\ub300 \ubaa8\ud130\uc2a4\uc758 \uac10\ub3c5\uc73c\ub85c \ubd80\uc784\ud558\uc600\ub2e4. \ub2f9\uc2dc \uc911\ud558\uc704\uad8c\uc744 \ub9f4\ub3cc\uace0 \uc788\uc5c8\ub358 \uc804\ubd81\uc758 \uc9c0\ud718\ubd09\uc744 \uc7a1\uc740 \ucd5c\uac15\ud76c \uac10\ub3c5\uc740 \ubd80\uc784 \uccab \ud574 \uc815\uaddc\ub9ac\uadf8\uc5d0\uc11c\ub294 \uc88b\uc740 \uc131\uc801\uc744 \ub0b4\uc9c0 \ubabb\ud558\uc600\uc73c\ub098 FA\ucef5\uc5d0\uc11c \uc6b0\uc2b9\uc744 \ucc28\uc9c0\ud574 2006\ub144 ACL\uc5d0 \ucd9c\uc804\ud558\uac8c \ub418\uc5c8\ub2e4. AFC \ucc54\ud53c\uc5b8\uc2a4\ub9ac\uadf8\uc5d0\uc11c \uc5fc\uae30\ud6c8, \uae40\ud615\ubc94, \uc81c \uce74\ub97c\ub8e8\uc2a4 \ub4f1\uc744 \uc55e\uc138\uc6cc \uc870\ubcc4 \uc608\uc120\uacfc \ubcf8\uc120 \ud1a0\ub108\uba3c\ud2b8\uc5d0\uc11c \uc5f0\uc774\uc740 \uc5ed\uc804\uc2b9\uc744 \uc77c\uad6c\uc5b4 \ub0b4\uba70 \uadf9\uc801\uc778 \uc6b0\uc2b9\uc744 \ucc28\uc9c0\ud558\uc600\ub2e4. 2008 K\ub9ac\uadf8\ub97c \uc55e\ub450\uace0 \ub300\ub300\uc801\uc778 \ub9ac\ube4c\ub529\uc744 \uc2e4\uc2dc\ud558\uc600\ub294\ub370, \uc2dc\uc98c \ucd08\ubc18 \ubd80\uc9c4\ud55c \ubaa8\uc2b5\uc744 \ubcf4\uc5ec \uc9c8\ud0c0\ub97c \ubc1b\uae30\ub3c4 \ud558\uc600\ub2e4. \ud558\uc9c0\ub9cc \ub9ac\uadf8 \ud6c4\ubc18\uae30\uc5d0 \ub4e4\uc5b4\uc11c\uba70 \uc774\ub0b4 \uacbd\uae30\ub825\uc744 \ub04c\uc5b4\uc62c\ub824 \uc815\uaddc \ub9ac\uadf8 \ucd5c\uc885 4\uc704\ub85c \uc2dc\uc98c\uc744 \ub9c8\uac10\ud558\uc600\ub2e4. \ud2b9\uc720\uc758 \uce5c\ud654\ub825\uc73c\ub85c \uc778\ud574 \uc804\ubd81 \ud604\ub300 \ubaa8\ud130\uc2a4\uc758 \ud074\ub7fd \ud558\uc6b0\uc2a4 \ubc0f \ud6c8\ub828\uc7a5\uc774 \uc18c\uc7ac\ud558\uace0 \uc788\ub294 \uc804\ub77c\ubd81\ub3c4 \uc644\uc8fc\uad70 \ubd09\ub3d9\uc74d\uc758 \uc9c0\uba85\uc744 \ubcf8\ub530\uc11c \ubd09\ub3d9 \uc774\uc7a5\uc774\ub77c\ub294 \ubcc4\uba85\uc744 \uc5bb\uc5c8\ub2e4.", "answer": "\ucd5c\uac15\ud76c", "sentence": "\ub2f9\uc2dc \uc911\ud558\uc704\uad8c\uc744 \ub9f4\ub3cc\uace0 \uc788\uc5c8\ub358 \uc804\ubd81\uc758 \uc9c0\ud718\ubd09\uc744 \uc7a1\uc740 \ucd5c\uac15\ud76c \uac10\ub3c5\uc740 \ubd80\uc784 \uccab \ud574 \uc815\uaddc\ub9ac\uadf8\uc5d0\uc11c\ub294 \uc88b\uc740 \uc131\uc801\uc744 \ub0b4\uc9c0 \ubabb\ud558\uc600\uc73c\ub098 FA\ucef5\uc5d0\uc11c \uc6b0\uc2b9\uc744 \ucc28\uc9c0\ud574 2006\ub144 ACL\uc5d0 \ucd9c\uc804\ud558\uac8c \ub418\uc5c8\ub2e4.", "paragraph_sentence": "2005\ub144 7\uc6d4 3\uc77c, \uc870\uc724\ud658\uc758 \ud6c4\uc784\uc73c\ub85c \uc804\ubd81 \ud604\ub300 \ubaa8\ud130\uc2a4\uc758 \uac10\ub3c5\uc73c\ub85c \ubd80\uc784\ud558\uc600\ub2e4. \ub2f9\uc2dc \uc911\ud558\uc704\uad8c\uc744 \ub9f4\ub3cc\uace0 \uc788\uc5c8\ub358 \uc804\ubd81\uc758 \uc9c0\ud718\ubd09\uc744 \uc7a1\uc740 \ucd5c\uac15\ud76c \uac10\ub3c5\uc740 \ubd80\uc784 \uccab \ud574 \uc815\uaddc\ub9ac\uadf8\uc5d0\uc11c\ub294 \uc88b\uc740 \uc131\uc801\uc744 \ub0b4\uc9c0 \ubabb\ud558\uc600\uc73c\ub098 FA\ucef5\uc5d0\uc11c \uc6b0\uc2b9\uc744 \ucc28\uc9c0\ud574 2006\ub144 ACL\uc5d0 \ucd9c\uc804\ud558\uac8c \ub418\uc5c8\ub2e4. AFC \ucc54\ud53c\uc5b8\uc2a4\ub9ac\uadf8\uc5d0\uc11c \uc5fc\uae30\ud6c8, \uae40\ud615\ubc94, \uc81c \uce74\ub97c\ub8e8\uc2a4 \ub4f1\uc744 \uc55e\uc138\uc6cc \uc870\ubcc4 \uc608\uc120\uacfc \ubcf8\uc120 \ud1a0\ub108\uba3c\ud2b8\uc5d0\uc11c \uc5f0\uc774\uc740 \uc5ed\uc804\uc2b9\uc744 \uc77c\uad6c\uc5b4 \ub0b4\uba70 \uadf9\uc801\uc778 \uc6b0\uc2b9\uc744 \ucc28\uc9c0\ud558\uc600\ub2e4. 2008 K\ub9ac\uadf8\ub97c \uc55e\ub450\uace0 \ub300\ub300\uc801\uc778 \ub9ac\ube4c\ub529\uc744 \uc2e4\uc2dc\ud558\uc600\ub294\ub370, \uc2dc\uc98c \ucd08\ubc18 \ubd80\uc9c4\ud55c \ubaa8\uc2b5\uc744 \ubcf4\uc5ec \uc9c8\ud0c0\ub97c \ubc1b\uae30\ub3c4 \ud558\uc600\ub2e4. \ud558\uc9c0\ub9cc \ub9ac\uadf8 \ud6c4\ubc18\uae30\uc5d0 \ub4e4\uc5b4\uc11c\uba70 \uc774\ub0b4 \uacbd\uae30\ub825\uc744 \ub04c\uc5b4\uc62c\ub824 \uc815\uaddc \ub9ac\uadf8 \ucd5c\uc885 4\uc704\ub85c \uc2dc\uc98c\uc744 \ub9c8\uac10\ud558\uc600\ub2e4. \ud2b9\uc720\uc758 \uce5c\ud654\ub825\uc73c\ub85c \uc778\ud574 \uc804\ubd81 \ud604\ub300 \ubaa8\ud130\uc2a4\uc758 \ud074\ub7fd \ud558\uc6b0\uc2a4 \ubc0f \ud6c8\ub828\uc7a5\uc774 \uc18c\uc7ac\ud558\uace0 \uc788\ub294 \uc804\ub77c\ubd81\ub3c4 \uc644\uc8fc\uad70 \ubd09\ub3d9\uc74d\uc758 \uc9c0\uba85\uc744 \ubcf8\ub530\uc11c \ubd09\ub3d9 \uc774\uc7a5\uc774\ub77c\ub294 \ubcc4\uba85\uc744 \uc5bb\uc5c8\ub2e4.", "paragraph_answer": "2005\ub144 7\uc6d4 3\uc77c, \uc870\uc724\ud658\uc758 \ud6c4\uc784\uc73c\ub85c \uc804\ubd81 \ud604\ub300 \ubaa8\ud130\uc2a4\uc758 \uac10\ub3c5\uc73c\ub85c \ubd80\uc784\ud558\uc600\ub2e4. \ub2f9\uc2dc \uc911\ud558\uc704\uad8c\uc744 \ub9f4\ub3cc\uace0 \uc788\uc5c8\ub358 \uc804\ubd81\uc758 \uc9c0\ud718\ubd09\uc744 \uc7a1\uc740 \ucd5c\uac15\ud76c \uac10\ub3c5\uc740 \ubd80\uc784 \uccab \ud574 \uc815\uaddc\ub9ac\uadf8\uc5d0\uc11c\ub294 \uc88b\uc740 \uc131\uc801\uc744 \ub0b4\uc9c0 \ubabb\ud558\uc600\uc73c\ub098 FA\ucef5\uc5d0\uc11c \uc6b0\uc2b9\uc744 \ucc28\uc9c0\ud574 2006\ub144 ACL\uc5d0 \ucd9c\uc804\ud558\uac8c \ub418\uc5c8\ub2e4. AFC \ucc54\ud53c\uc5b8\uc2a4\ub9ac\uadf8\uc5d0\uc11c \uc5fc\uae30\ud6c8, \uae40\ud615\ubc94, \uc81c \uce74\ub97c\ub8e8\uc2a4 \ub4f1\uc744 \uc55e\uc138\uc6cc \uc870\ubcc4 \uc608\uc120\uacfc \ubcf8\uc120 \ud1a0\ub108\uba3c\ud2b8\uc5d0\uc11c \uc5f0\uc774\uc740 \uc5ed\uc804\uc2b9\uc744 \uc77c\uad6c\uc5b4 \ub0b4\uba70 \uadf9\uc801\uc778 \uc6b0\uc2b9\uc744 \ucc28\uc9c0\ud558\uc600\ub2e4. 2008 K\ub9ac\uadf8\ub97c \uc55e\ub450\uace0 \ub300\ub300\uc801\uc778 \ub9ac\ube4c\ub529\uc744 \uc2e4\uc2dc\ud558\uc600\ub294\ub370, \uc2dc\uc98c \ucd08\ubc18 \ubd80\uc9c4\ud55c \ubaa8\uc2b5\uc744 \ubcf4\uc5ec \uc9c8\ud0c0\ub97c \ubc1b\uae30\ub3c4 \ud558\uc600\ub2e4. \ud558\uc9c0\ub9cc \ub9ac\uadf8 \ud6c4\ubc18\uae30\uc5d0 \ub4e4\uc5b4\uc11c\uba70 \uc774\ub0b4 \uacbd\uae30\ub825\uc744 \ub04c\uc5b4\uc62c\ub824 \uc815\uaddc \ub9ac\uadf8 \ucd5c\uc885 4\uc704\ub85c \uc2dc\uc98c\uc744 \ub9c8\uac10\ud558\uc600\ub2e4. \ud2b9\uc720\uc758 \uce5c\ud654\ub825\uc73c\ub85c \uc778\ud574 \uc804\ubd81 \ud604\ub300 \ubaa8\ud130\uc2a4\uc758 \ud074\ub7fd \ud558\uc6b0\uc2a4 \ubc0f \ud6c8\ub828\uc7a5\uc774 \uc18c\uc7ac\ud558\uace0 \uc788\ub294 \uc804\ub77c\ubd81\ub3c4 \uc644\uc8fc\uad70 \ubd09\ub3d9\uc74d\uc758 \uc9c0\uba85\uc744 \ubcf8\ub530\uc11c \ubd09\ub3d9 \uc774\uc7a5\uc774\ub77c\ub294 \ubcc4\uba85\uc744 \uc5bb\uc5c8\ub2e4.", "sentence_answer": "\ub2f9\uc2dc \uc911\ud558\uc704\uad8c\uc744 \ub9f4\ub3cc\uace0 \uc788\uc5c8\ub358 \uc804\ubd81\uc758 \uc9c0\ud718\ubd09\uc744 \uc7a1\uc740 \ucd5c\uac15\ud76c \uac10\ub3c5\uc740 \ubd80\uc784 \uccab \ud574 \uc815\uaddc\ub9ac\uadf8\uc5d0\uc11c\ub294 \uc88b\uc740 \uc131\uc801\uc744 \ub0b4\uc9c0 \ubabb\ud558\uc600\uc73c\ub098 FA\ucef5\uc5d0\uc11c \uc6b0\uc2b9\uc744 \ucc28\uc9c0\ud574 2006\ub144 ACL\uc5d0 \ucd9c\uc804\ud558\uac8c \ub418\uc5c8\ub2e4.", "paragraph_id": "6596920-0-0", "paragraph_question": "question: 2005\ub144 7\uc6d4 3\uc77c \uc804\ubd81 \ud604\ub300 \ubaa8\ud130\uc2a4\uc758 \uac10\ub3c5\uc73c\ub85c \ubd80\uc784\ud55c \uac83\uc740 \ub204\uad6c\uc778\uac00?, context: 2005\ub144 7\uc6d4 3\uc77c, \uc870\uc724\ud658\uc758 \ud6c4\uc784\uc73c\ub85c \uc804\ubd81 \ud604\ub300 \ubaa8\ud130\uc2a4\uc758 \uac10\ub3c5\uc73c\ub85c \ubd80\uc784\ud558\uc600\ub2e4. \ub2f9\uc2dc \uc911\ud558\uc704\uad8c\uc744 \ub9f4\ub3cc\uace0 \uc788\uc5c8\ub358 \uc804\ubd81\uc758 \uc9c0\ud718\ubd09\uc744 \uc7a1\uc740 \ucd5c\uac15\ud76c \uac10\ub3c5\uc740 \ubd80\uc784 \uccab \ud574 \uc815\uaddc\ub9ac\uadf8\uc5d0\uc11c\ub294 \uc88b\uc740 \uc131\uc801\uc744 \ub0b4\uc9c0 \ubabb\ud558\uc600\uc73c\ub098 FA\ucef5\uc5d0\uc11c \uc6b0\uc2b9\uc744 \ucc28\uc9c0\ud574 2006\ub144 ACL\uc5d0 \ucd9c\uc804\ud558\uac8c \ub418\uc5c8\ub2e4. AFC \ucc54\ud53c\uc5b8\uc2a4\ub9ac\uadf8\uc5d0\uc11c \uc5fc\uae30\ud6c8, \uae40\ud615\ubc94, \uc81c \uce74\ub97c\ub8e8\uc2a4 \ub4f1\uc744 \uc55e\uc138\uc6cc \uc870\ubcc4 \uc608\uc120\uacfc \ubcf8\uc120 \ud1a0\ub108\uba3c\ud2b8\uc5d0\uc11c \uc5f0\uc774\uc740 \uc5ed\uc804\uc2b9\uc744 \uc77c\uad6c\uc5b4 \ub0b4\uba70 \uadf9\uc801\uc778 \uc6b0\uc2b9\uc744 \ucc28\uc9c0\ud558\uc600\ub2e4. 2008 K\ub9ac\uadf8\ub97c \uc55e\ub450\uace0 \ub300\ub300\uc801\uc778 \ub9ac\ube4c\ub529\uc744 \uc2e4\uc2dc\ud558\uc600\ub294\ub370, \uc2dc\uc98c \ucd08\ubc18 \ubd80\uc9c4\ud55c \ubaa8\uc2b5\uc744 \ubcf4\uc5ec \uc9c8\ud0c0\ub97c \ubc1b\uae30\ub3c4 \ud558\uc600\ub2e4. \ud558\uc9c0\ub9cc \ub9ac\uadf8 \ud6c4\ubc18\uae30\uc5d0 \ub4e4\uc5b4\uc11c\uba70 \uc774\ub0b4 \uacbd\uae30\ub825\uc744 \ub04c\uc5b4\uc62c\ub824 \uc815\uaddc \ub9ac\uadf8 \ucd5c\uc885 4\uc704\ub85c \uc2dc\uc98c\uc744 \ub9c8\uac10\ud558\uc600\ub2e4. \ud2b9\uc720\uc758 \uce5c\ud654\ub825\uc73c\ub85c \uc778\ud574 \uc804\ubd81 \ud604\ub300 \ubaa8\ud130\uc2a4\uc758 \ud074\ub7fd \ud558\uc6b0\uc2a4 \ubc0f \ud6c8\ub828\uc7a5\uc774 \uc18c\uc7ac\ud558\uace0 \uc788\ub294 \uc804\ub77c\ubd81\ub3c4 \uc644\uc8fc\uad70 \ubd09\ub3d9\uc74d\uc758 \uc9c0\uba85\uc744 \ubcf8\ub530\uc11c \ubd09\ub3d9 \uc774\uc7a5\uc774\ub77c\ub294 \ubcc4\uba85\uc744 \uc5bb\uc5c8\ub2e4."} {"question": "\uc218\ud559 \uadc0\uc2e0\uc774 \ub3c5\uc77c\uc5b4\ub85c \uccab \ucd9c\uac04\ub41c \ub144\ub3c4\ub294?", "paragraph": "\uc774 \ucc45\uc758 \uc791\uac00 \uc5d4\uccb8\uc2a4\ubca0\ub974\uac70\ub294 \ube44\ub85d \uc218\ud559\uc790\ub294 \uc544\ub2c8\uc5c8\uc9c0\ub9cc \ud3c9\uc18c\uc5d0 \uc218\ud559\uc5d0 \uad00\ud574 \ub9ce\uc740 \uad00\uc2ec\uc744 \uac00\uc9c0\uace0 \uc788\uc5c8\ub2e4. 1998\ub144\uc5d0\ub294 \uc138\uacc4 \uc218\ud559\uc790 \ub300\ud68c\uc5d0\uc11c \uc218\ud559\uc774 \ub300\uc911 \ubb38\ud654\uc5d0\uc11c \uba40\uc5b4\uc838 \uace0\ub9bd\ub418\uace0 \uc788\ub294 \ud604\uc2e4\uc744 \ube44\ud310\ud558\ub294 \uc5f0\uc124\uc744 \ud558\uae30\ub3c4 \ud588\ub2e4. \uc774 \ucc45 \uc218\ud559 \uadc0\uc2e0\uc740 \uc5d4\uccb8\uc2a4\ubca0\ub974\uac70\uac00 \uc5f4\ud55c \uc0b4 \ub538 \ub370\ub808\uc0ac\uc758 \ubd80\ud0c1\uc73c\ub85c \uc4f0\uac8c \ub418\uc5c8\ub2e4. \ud3c9\uc18c \ud559\uad50\uc5d0\uc11c \uc218\ud559\uc744 \uac00\ub974\uce58\ub294 \ubc29\ubc95\uc5d0 \ub300\ud574 \ubd88\ub9cc\uc774 \uc788\ub358 \uadf8\ub294 \uc218\ud559\uc744 \uc0c8\ub86d\uace0 \uc7ac\ubc0c\uac8c \uac00\ub974\uce58\uace0 \uc2f6\uc5b4 \ud588\ub2e4. \uc0bd\ud654\uac00 \ub85c\ud2b8\ub77c\uc6b0\ud2b8 \uc218\uc794\ub124 \ubca0\ub974\ub108\uac00 \uc774 \ucc45\uc758 \ud45c\uc9c0\uc640 \ub0b4\uc6a9 \uacf3\uacf3\uc5d0 \uc788\ub294 \uadf8\ub9bc\uc744 \uadf8\ub838\ub2e4. \uc218\ud559 \uadc0\uc2e0\uc740 1997\ub144\uc5d0 \ub3c5\uc77c\uc5b4\ub85c \uccab \ucd9c\uac04\ub418\uc5c8\ub2e4. \uc791\uac00\ub294 \uc218\ud559 \uac1c\ub150\uc744 \uc77c\ubc18\uc801\uc73c\ub85c \uc124\uba85\ud558\uae30\ubcf4\ub2e4 \uc0c8\ub85c\uc6b4 \ub2e8\uc5b4\ub97c \ub9cc\ub4e4\uc5b4\ub0b4\uace0 \uc774\ub97c \ucc3d\uc758\uc801\uc778 \ubc29\ubc95\uc73c\ub85c \uc774\ud574\uc2dc\ud0a4\ub824\uace0 \ud588\ub2e4. \uc608\ub97c \ub4e4\uc5b4 \uac70\ub4ed\uc81c\uacf1\uc740 '\uae61\ucda9 \ub6f0\uae30'\ub85c, \ubb34\ub9ac\uc218\ub294 '\uc774\uce58\uc5d0 \uc5b4\uae0b\ub098\ub294 \uc218'\ub77c\ub294 \uc774\ub984\uc73c\ub85c \ub300\uc2e0\ud588\ub2e4.", "answer": "1997\ub144", "sentence": "\uc218\ud559 \uadc0\uc2e0\uc740 1997\ub144 \uc5d0 \ub3c5\uc77c\uc5b4\ub85c \uccab \ucd9c\uac04\ub418\uc5c8\ub2e4.", "paragraph_sentence": "\uc774 \ucc45\uc758 \uc791\uac00 \uc5d4\uccb8\uc2a4\ubca0\ub974\uac70\ub294 \ube44\ub85d \uc218\ud559\uc790\ub294 \uc544\ub2c8\uc5c8\uc9c0\ub9cc \ud3c9\uc18c\uc5d0 \uc218\ud559\uc5d0 \uad00\ud574 \ub9ce\uc740 \uad00\uc2ec\uc744 \uac00\uc9c0\uace0 \uc788\uc5c8\ub2e4. 1998\ub144\uc5d0\ub294 \uc138\uacc4 \uc218\ud559\uc790 \ub300\ud68c\uc5d0\uc11c \uc218\ud559\uc774 \ub300\uc911 \ubb38\ud654\uc5d0\uc11c \uba40\uc5b4\uc838 \uace0\ub9bd\ub418\uace0 \uc788\ub294 \ud604\uc2e4\uc744 \ube44\ud310\ud558\ub294 \uc5f0\uc124\uc744 \ud558\uae30\ub3c4 \ud588\ub2e4. \uc774 \ucc45 \uc218\ud559 \uadc0\uc2e0\uc740 \uc5d4\uccb8\uc2a4\ubca0\ub974\uac70\uac00 \uc5f4\ud55c \uc0b4 \ub538 \ub370\ub808\uc0ac\uc758 \ubd80\ud0c1\uc73c\ub85c \uc4f0\uac8c \ub418\uc5c8\ub2e4. \ud3c9\uc18c \ud559\uad50\uc5d0\uc11c \uc218\ud559\uc744 \uac00\ub974\uce58\ub294 \ubc29\ubc95\uc5d0 \ub300\ud574 \ubd88\ub9cc\uc774 \uc788\ub358 \uadf8\ub294 \uc218\ud559\uc744 \uc0c8\ub86d\uace0 \uc7ac\ubc0c\uac8c \uac00\ub974\uce58\uace0 \uc2f6\uc5b4 \ud588\ub2e4. \uc0bd\ud654\uac00 \ub85c\ud2b8\ub77c\uc6b0\ud2b8 \uc218\uc794\ub124 \ubca0\ub974\ub108\uac00 \uc774 \ucc45\uc758 \ud45c\uc9c0\uc640 \ub0b4\uc6a9 \uacf3\uacf3\uc5d0 \uc788\ub294 \uadf8\ub9bc\uc744 \uadf8\ub838\ub2e4. \uc218\ud559 \uadc0\uc2e0\uc740 1997\ub144 \uc5d0 \ub3c5\uc77c\uc5b4\ub85c \uccab \ucd9c\uac04\ub418\uc5c8\ub2e4. \uc791\uac00\ub294 \uc218\ud559 \uac1c\ub150\uc744 \uc77c\ubc18\uc801\uc73c\ub85c \uc124\uba85\ud558\uae30\ubcf4\ub2e4 \uc0c8\ub85c\uc6b4 \ub2e8\uc5b4\ub97c \ub9cc\ub4e4\uc5b4\ub0b4\uace0 \uc774\ub97c \ucc3d\uc758\uc801\uc778 \ubc29\ubc95\uc73c\ub85c \uc774\ud574\uc2dc\ud0a4\ub824\uace0 \ud588\ub2e4. \uc608\ub97c \ub4e4\uc5b4 \uac70\ub4ed\uc81c\uacf1\uc740 '\uae61\ucda9 \ub6f0\uae30'\ub85c, \ubb34\ub9ac\uc218\ub294 '\uc774\uce58\uc5d0 \uc5b4\uae0b\ub098\ub294 \uc218'\ub77c\ub294 \uc774\ub984\uc73c\ub85c \ub300\uc2e0\ud588\ub2e4.", "paragraph_answer": "\uc774 \ucc45\uc758 \uc791\uac00 \uc5d4\uccb8\uc2a4\ubca0\ub974\uac70\ub294 \ube44\ub85d \uc218\ud559\uc790\ub294 \uc544\ub2c8\uc5c8\uc9c0\ub9cc \ud3c9\uc18c\uc5d0 \uc218\ud559\uc5d0 \uad00\ud574 \ub9ce\uc740 \uad00\uc2ec\uc744 \uac00\uc9c0\uace0 \uc788\uc5c8\ub2e4. 1998\ub144\uc5d0\ub294 \uc138\uacc4 \uc218\ud559\uc790 \ub300\ud68c\uc5d0\uc11c \uc218\ud559\uc774 \ub300\uc911 \ubb38\ud654\uc5d0\uc11c \uba40\uc5b4\uc838 \uace0\ub9bd\ub418\uace0 \uc788\ub294 \ud604\uc2e4\uc744 \ube44\ud310\ud558\ub294 \uc5f0\uc124\uc744 \ud558\uae30\ub3c4 \ud588\ub2e4. \uc774 \ucc45 \uc218\ud559 \uadc0\uc2e0\uc740 \uc5d4\uccb8\uc2a4\ubca0\ub974\uac70\uac00 \uc5f4\ud55c \uc0b4 \ub538 \ub370\ub808\uc0ac\uc758 \ubd80\ud0c1\uc73c\ub85c \uc4f0\uac8c \ub418\uc5c8\ub2e4. \ud3c9\uc18c \ud559\uad50\uc5d0\uc11c \uc218\ud559\uc744 \uac00\ub974\uce58\ub294 \ubc29\ubc95\uc5d0 \ub300\ud574 \ubd88\ub9cc\uc774 \uc788\ub358 \uadf8\ub294 \uc218\ud559\uc744 \uc0c8\ub86d\uace0 \uc7ac\ubc0c\uac8c \uac00\ub974\uce58\uace0 \uc2f6\uc5b4 \ud588\ub2e4. \uc0bd\ud654\uac00 \ub85c\ud2b8\ub77c\uc6b0\ud2b8 \uc218\uc794\ub124 \ubca0\ub974\ub108\uac00 \uc774 \ucc45\uc758 \ud45c\uc9c0\uc640 \ub0b4\uc6a9 \uacf3\uacf3\uc5d0 \uc788\ub294 \uadf8\ub9bc\uc744 \uadf8\ub838\ub2e4. \uc218\ud559 \uadc0\uc2e0\uc740 1997\ub144 \uc5d0 \ub3c5\uc77c\uc5b4\ub85c \uccab \ucd9c\uac04\ub418\uc5c8\ub2e4. \uc791\uac00\ub294 \uc218\ud559 \uac1c\ub150\uc744 \uc77c\ubc18\uc801\uc73c\ub85c \uc124\uba85\ud558\uae30\ubcf4\ub2e4 \uc0c8\ub85c\uc6b4 \ub2e8\uc5b4\ub97c \ub9cc\ub4e4\uc5b4\ub0b4\uace0 \uc774\ub97c \ucc3d\uc758\uc801\uc778 \ubc29\ubc95\uc73c\ub85c \uc774\ud574\uc2dc\ud0a4\ub824\uace0 \ud588\ub2e4. \uc608\ub97c \ub4e4\uc5b4 \uac70\ub4ed\uc81c\uacf1\uc740 '\uae61\ucda9 \ub6f0\uae30'\ub85c, \ubb34\ub9ac\uc218\ub294 '\uc774\uce58\uc5d0 \uc5b4\uae0b\ub098\ub294 \uc218'\ub77c\ub294 \uc774\ub984\uc73c\ub85c \ub300\uc2e0\ud588\ub2e4.", "sentence_answer": "\uc218\ud559 \uadc0\uc2e0\uc740 1997\ub144 \uc5d0 \ub3c5\uc77c\uc5b4\ub85c \uccab \ucd9c\uac04\ub418\uc5c8\ub2e4.", "paragraph_id": "6518864-2-2", "paragraph_question": "question: \uc218\ud559 \uadc0\uc2e0\uc774 \ub3c5\uc77c\uc5b4\ub85c \uccab \ucd9c\uac04\ub41c \ub144\ub3c4\ub294?, context: \uc774 \ucc45\uc758 \uc791\uac00 \uc5d4\uccb8\uc2a4\ubca0\ub974\uac70\ub294 \ube44\ub85d \uc218\ud559\uc790\ub294 \uc544\ub2c8\uc5c8\uc9c0\ub9cc \ud3c9\uc18c\uc5d0 \uc218\ud559\uc5d0 \uad00\ud574 \ub9ce\uc740 \uad00\uc2ec\uc744 \uac00\uc9c0\uace0 \uc788\uc5c8\ub2e4. 1998\ub144\uc5d0\ub294 \uc138\uacc4 \uc218\ud559\uc790 \ub300\ud68c\uc5d0\uc11c \uc218\ud559\uc774 \ub300\uc911 \ubb38\ud654\uc5d0\uc11c \uba40\uc5b4\uc838 \uace0\ub9bd\ub418\uace0 \uc788\ub294 \ud604\uc2e4\uc744 \ube44\ud310\ud558\ub294 \uc5f0\uc124\uc744 \ud558\uae30\ub3c4 \ud588\ub2e4. \uc774 \ucc45 \uc218\ud559 \uadc0\uc2e0\uc740 \uc5d4\uccb8\uc2a4\ubca0\ub974\uac70\uac00 \uc5f4\ud55c \uc0b4 \ub538 \ub370\ub808\uc0ac\uc758 \ubd80\ud0c1\uc73c\ub85c \uc4f0\uac8c \ub418\uc5c8\ub2e4. \ud3c9\uc18c \ud559\uad50\uc5d0\uc11c \uc218\ud559\uc744 \uac00\ub974\uce58\ub294 \ubc29\ubc95\uc5d0 \ub300\ud574 \ubd88\ub9cc\uc774 \uc788\ub358 \uadf8\ub294 \uc218\ud559\uc744 \uc0c8\ub86d\uace0 \uc7ac\ubc0c\uac8c \uac00\ub974\uce58\uace0 \uc2f6\uc5b4 \ud588\ub2e4. \uc0bd\ud654\uac00 \ub85c\ud2b8\ub77c\uc6b0\ud2b8 \uc218\uc794\ub124 \ubca0\ub974\ub108\uac00 \uc774 \ucc45\uc758 \ud45c\uc9c0\uc640 \ub0b4\uc6a9 \uacf3\uacf3\uc5d0 \uc788\ub294 \uadf8\ub9bc\uc744 \uadf8\ub838\ub2e4. \uc218\ud559 \uadc0\uc2e0\uc740 1997\ub144\uc5d0 \ub3c5\uc77c\uc5b4\ub85c \uccab \ucd9c\uac04\ub418\uc5c8\ub2e4. \uc791\uac00\ub294 \uc218\ud559 \uac1c\ub150\uc744 \uc77c\ubc18\uc801\uc73c\ub85c \uc124\uba85\ud558\uae30\ubcf4\ub2e4 \uc0c8\ub85c\uc6b4 \ub2e8\uc5b4\ub97c \ub9cc\ub4e4\uc5b4\ub0b4\uace0 \uc774\ub97c \ucc3d\uc758\uc801\uc778 \ubc29\ubc95\uc73c\ub85c \uc774\ud574\uc2dc\ud0a4\ub824\uace0 \ud588\ub2e4. \uc608\ub97c \ub4e4\uc5b4 \uac70\ub4ed\uc81c\uacf1\uc740 '\uae61\ucda9 \ub6f0\uae30'\ub85c, \ubb34\ub9ac\uc218\ub294 '\uc774\uce58\uc5d0 \uc5b4\uae0b\ub098\ub294 \uc218'\ub77c\ub294 \uc774\ub984\uc73c\ub85c \ub300\uc2e0\ud588\ub2e4."} {"question": "\ub204\uac00 \ub3c4\uc131\uc5d0 \ubcfc\ubaa8\ub85c \uc7a1\ud600\uc788\ub358 \uacf5\uc9c1\uc758 \uc544\ub4e4\uc744 \uccad\uc8fc\ub85c \ub370\ub9ac\uace0 \uac14\ub098?", "paragraph": "\ub610\ud55c \uc5fc\uc0c1\uc9c4\uc740 \uacbd\uc885\uc774 \uc624\ub798\uc804\ubd80\ud130 \uc5ed\ubaa8\ub97c \uacc4\ud68d\ud558\uace0 \uc788\uc5c8\uc73c\uba70, \uadf8 \uc99d\uac70\ub85c \uacbd\uc885\uc774 \ucd5c\uadfc \uc790\uc2e0\uc758 \uc870\uce74\ub97c \uccad\uc8fc\ub85c \ub370\ub824\uac00\ub824 \ud588\ub2e4\ub294 \uc0ac\uc2e4\uc774 \uc788\uc5c8\uc74c\uc744 \ud53c\ub825\ud55c\ub2e4. \ub2f9\uc2dc \uc9c0\ubc29 \uc131\uc8fc\ub4e4\uc740 \uc790\uc2e0\uc758 \uc544\ub4e4\uc744 \ub3c4\uc131\uc5d0 \ubcfc\ubaa8\ub85c \ub0a8\uaca8\ub450\uc5b4\uc57c \ud588\ub294\ub370, \uc774\uac83\uc740 \uc6d0\ub798 \uad81\uc608\uac00 \ubc18\ub780\uc744 \ubc29\uc9c0\ud558\uae30 \uc704\ud574 \ub9c8\ub828\ud55c \ub300\ube44\ucc45\uc774\uc5c8\ub2e4. \uccad\uc8fc \uc131\uc8fc \uacf5\uc9c1\uc758 \uc544\ub4e4 \uc5ed\uc2dc \uc774\ub7f0 \uc774\uc720\ub85c \ub3c4\uc131\uc5d0 \uba38\ubb3c\ub800\ub294\ub370, \uacf5\uc9c1\uc758 \uc544\ub0b4\ub294 \uc774 \ub54c\ubb38\uc5d0 \ud56d\uc0c1 \uadfc\uc2ec\uc5d0 \uc0ac\ub85c\uc7a1\ud600 \uc788\uc5c8\ub2e4. \uadf8\ub9ac\uace0 \ub3d9\uc0dd \uacbd\uc885\uc5d0\uac8c \uc740\ubc00\ud788 \uc790\uc2e0\uc758 \uc544\ub4e4\uc744 \ub370\ub824\uc62c \uac83\uc744 \uc9c0\uc2dc\ud588\ub2e4. \ubcfc\ubaa8\ub97c \ub370\ub824\uac00\ub294 \uac83\uc740 \ubc18\uc5ed\uc73c\ub85c \ud574\uc11d\ud558\uc600\ub2e4. \ub530\ub77c\uc11c \uacbd\uc885\uc774 \uc870\uce74\ub97c \ub370\ub824\uac08 \uacc4\ud68d\uc744 \uc138\uc6b4 \uc801\uc774 \uc788\ub2e4\ub294 \uac83\uc740 \uc774\ubbf8 \uc624\ub798\uc804\ubd80\ud130 \uc5ed\ubaa8\ub97c \uacc4\ud68d\ud588\ub2e4\ub294 \uac83\uc744 \ub73b\ud55c\ub2e4\ub294 \uac83\uc774\ub2e4. \uc655\uac74\uc740 \uc5fc\uc0c1\uc9c4\uc758 \uc8fc\uc7a5\uc744 \ub4e3\uace0 \uacb0\uad6d \uacbd\uc885\uc744 \ube44\ub86f\ud55c \uc5ed\ubaa8 \ud610\uc758\uc790\ub4e4\uc744 \ubaa8\ub450 \uc8fd\uc774\uac8c \ub41c\ub2e4.", "answer": "\uacbd\uc885", "sentence": "\uc5fc\uc0c1\uc9c4\uc740 \uacbd\uc885 \uc774 \uc624\ub798\uc804\ubd80\ud130 \uc5ed\ubaa8\ub97c \uacc4\ud68d\ud558\uace0 \uc788\uc5c8\uc73c\uba70, \uadf8 \uc99d\uac70\ub85c \uacbd\uc885\uc774 \ucd5c\uadfc \uc790\uc2e0\uc758 \uc870\uce74\ub97c \uccad\uc8fc\ub85c \ub370\ub824\uac00\ub824 \ud588\ub2e4\ub294 \uc0ac\uc2e4\uc774 \uc788\uc5c8\uc74c\uc744 \ud53c\ub825\ud55c\ub2e4.", "paragraph_sentence": "\ub610\ud55c \uc5fc\uc0c1\uc9c4\uc740 \uacbd\uc885 \uc774 \uc624\ub798\uc804\ubd80\ud130 \uc5ed\ubaa8\ub97c \uacc4\ud68d\ud558\uace0 \uc788\uc5c8\uc73c\uba70, \uadf8 \uc99d\uac70\ub85c \uacbd\uc885\uc774 \ucd5c\uadfc \uc790\uc2e0\uc758 \uc870\uce74\ub97c \uccad\uc8fc\ub85c \ub370\ub824\uac00\ub824 \ud588\ub2e4\ub294 \uc0ac\uc2e4\uc774 \uc788\uc5c8\uc74c\uc744 \ud53c\ub825\ud55c\ub2e4. \ub2f9\uc2dc \uc9c0\ubc29 \uc131\uc8fc\ub4e4\uc740 \uc790\uc2e0\uc758 \uc544\ub4e4\uc744 \ub3c4\uc131\uc5d0 \ubcfc\ubaa8\ub85c \ub0a8\uaca8\ub450\uc5b4\uc57c \ud588\ub294\ub370, \uc774\uac83\uc740 \uc6d0\ub798 \uad81\uc608\uac00 \ubc18\ub780\uc744 \ubc29\uc9c0\ud558\uae30 \uc704\ud574 \ub9c8\ub828\ud55c \ub300\ube44\ucc45\uc774\uc5c8\ub2e4. \uccad\uc8fc \uc131\uc8fc \uacf5\uc9c1\uc758 \uc544\ub4e4 \uc5ed\uc2dc \uc774\ub7f0 \uc774\uc720\ub85c \ub3c4\uc131\uc5d0 \uba38\ubb3c\ub800\ub294\ub370, \uacf5\uc9c1\uc758 \uc544\ub0b4\ub294 \uc774 \ub54c\ubb38\uc5d0 \ud56d\uc0c1 \uadfc\uc2ec\uc5d0 \uc0ac\ub85c\uc7a1\ud600 \uc788\uc5c8\ub2e4. \uadf8\ub9ac\uace0 \ub3d9\uc0dd \uacbd\uc885\uc5d0\uac8c \uc740\ubc00\ud788 \uc790\uc2e0\uc758 \uc544\ub4e4\uc744 \ub370\ub824\uc62c \uac83\uc744 \uc9c0\uc2dc\ud588\ub2e4. \ubcfc\ubaa8\ub97c \ub370\ub824\uac00\ub294 \uac83\uc740 \ubc18\uc5ed\uc73c\ub85c \ud574\uc11d\ud558\uc600\ub2e4. \ub530\ub77c\uc11c \uacbd\uc885\uc774 \uc870\uce74\ub97c \ub370\ub824\uac08 \uacc4\ud68d\uc744 \uc138\uc6b4 \uc801\uc774 \uc788\ub2e4\ub294 \uac83\uc740 \uc774\ubbf8 \uc624\ub798\uc804\ubd80\ud130 \uc5ed\ubaa8\ub97c \uacc4\ud68d\ud588\ub2e4\ub294 \uac83\uc744 \ub73b\ud55c\ub2e4\ub294 \uac83\uc774\ub2e4. \uc655\uac74\uc740 \uc5fc\uc0c1\uc9c4\uc758 \uc8fc\uc7a5\uc744 \ub4e3\uace0 \uacb0\uad6d \uacbd\uc885\uc744 \ube44\ub86f\ud55c \uc5ed\ubaa8 \ud610\uc758\uc790\ub4e4\uc744 \ubaa8\ub450 \uc8fd\uc774\uac8c \ub41c\ub2e4.", "paragraph_answer": "\ub610\ud55c \uc5fc\uc0c1\uc9c4\uc740 \uacbd\uc885 \uc774 \uc624\ub798\uc804\ubd80\ud130 \uc5ed\ubaa8\ub97c \uacc4\ud68d\ud558\uace0 \uc788\uc5c8\uc73c\uba70, \uadf8 \uc99d\uac70\ub85c \uacbd\uc885\uc774 \ucd5c\uadfc \uc790\uc2e0\uc758 \uc870\uce74\ub97c \uccad\uc8fc\ub85c \ub370\ub824\uac00\ub824 \ud588\ub2e4\ub294 \uc0ac\uc2e4\uc774 \uc788\uc5c8\uc74c\uc744 \ud53c\ub825\ud55c\ub2e4. \ub2f9\uc2dc \uc9c0\ubc29 \uc131\uc8fc\ub4e4\uc740 \uc790\uc2e0\uc758 \uc544\ub4e4\uc744 \ub3c4\uc131\uc5d0 \ubcfc\ubaa8\ub85c \ub0a8\uaca8\ub450\uc5b4\uc57c \ud588\ub294\ub370, \uc774\uac83\uc740 \uc6d0\ub798 \uad81\uc608\uac00 \ubc18\ub780\uc744 \ubc29\uc9c0\ud558\uae30 \uc704\ud574 \ub9c8\ub828\ud55c \ub300\ube44\ucc45\uc774\uc5c8\ub2e4. \uccad\uc8fc \uc131\uc8fc \uacf5\uc9c1\uc758 \uc544\ub4e4 \uc5ed\uc2dc \uc774\ub7f0 \uc774\uc720\ub85c \ub3c4\uc131\uc5d0 \uba38\ubb3c\ub800\ub294\ub370, \uacf5\uc9c1\uc758 \uc544\ub0b4\ub294 \uc774 \ub54c\ubb38\uc5d0 \ud56d\uc0c1 \uadfc\uc2ec\uc5d0 \uc0ac\ub85c\uc7a1\ud600 \uc788\uc5c8\ub2e4. \uadf8\ub9ac\uace0 \ub3d9\uc0dd \uacbd\uc885\uc5d0\uac8c \uc740\ubc00\ud788 \uc790\uc2e0\uc758 \uc544\ub4e4\uc744 \ub370\ub824\uc62c \uac83\uc744 \uc9c0\uc2dc\ud588\ub2e4. \ubcfc\ubaa8\ub97c \ub370\ub824\uac00\ub294 \uac83\uc740 \ubc18\uc5ed\uc73c\ub85c \ud574\uc11d\ud558\uc600\ub2e4. \ub530\ub77c\uc11c \uacbd\uc885\uc774 \uc870\uce74\ub97c \ub370\ub824\uac08 \uacc4\ud68d\uc744 \uc138\uc6b4 \uc801\uc774 \uc788\ub2e4\ub294 \uac83\uc740 \uc774\ubbf8 \uc624\ub798\uc804\ubd80\ud130 \uc5ed\ubaa8\ub97c \uacc4\ud68d\ud588\ub2e4\ub294 \uac83\uc744 \ub73b\ud55c\ub2e4\ub294 \uac83\uc774\ub2e4. \uc655\uac74\uc740 \uc5fc\uc0c1\uc9c4\uc758 \uc8fc\uc7a5\uc744 \ub4e3\uace0 \uacb0\uad6d \uacbd\uc885\uc744 \ube44\ub86f\ud55c \uc5ed\ubaa8 \ud610\uc758\uc790\ub4e4\uc744 \ubaa8\ub450 \uc8fd\uc774\uac8c \ub41c\ub2e4.", "sentence_answer": "\uc5fc\uc0c1\uc9c4\uc740 \uacbd\uc885 \uc774 \uc624\ub798\uc804\ubd80\ud130 \uc5ed\ubaa8\ub97c \uacc4\ud68d\ud558\uace0 \uc788\uc5c8\uc73c\uba70, \uadf8 \uc99d\uac70\ub85c \uacbd\uc885\uc774 \ucd5c\uadfc \uc790\uc2e0\uc758 \uc870\uce74\ub97c \uccad\uc8fc\ub85c \ub370\ub824\uac00\ub824 \ud588\ub2e4\ub294 \uc0ac\uc2e4\uc774 \uc788\uc5c8\uc74c\uc744 \ud53c\ub825\ud55c\ub2e4.", "paragraph_id": "6528126-8-2", "paragraph_question": "question: \ub204\uac00 \ub3c4\uc131\uc5d0 \ubcfc\ubaa8\ub85c \uc7a1\ud600\uc788\ub358 \uacf5\uc9c1\uc758 \uc544\ub4e4\uc744 \uccad\uc8fc\ub85c \ub370\ub9ac\uace0 \uac14\ub098?, context: \ub610\ud55c \uc5fc\uc0c1\uc9c4\uc740 \uacbd\uc885\uc774 \uc624\ub798\uc804\ubd80\ud130 \uc5ed\ubaa8\ub97c \uacc4\ud68d\ud558\uace0 \uc788\uc5c8\uc73c\uba70, \uadf8 \uc99d\uac70\ub85c \uacbd\uc885\uc774 \ucd5c\uadfc \uc790\uc2e0\uc758 \uc870\uce74\ub97c \uccad\uc8fc\ub85c \ub370\ub824\uac00\ub824 \ud588\ub2e4\ub294 \uc0ac\uc2e4\uc774 \uc788\uc5c8\uc74c\uc744 \ud53c\ub825\ud55c\ub2e4. \ub2f9\uc2dc \uc9c0\ubc29 \uc131\uc8fc\ub4e4\uc740 \uc790\uc2e0\uc758 \uc544\ub4e4\uc744 \ub3c4\uc131\uc5d0 \ubcfc\ubaa8\ub85c \ub0a8\uaca8\ub450\uc5b4\uc57c \ud588\ub294\ub370, \uc774\uac83\uc740 \uc6d0\ub798 \uad81\uc608\uac00 \ubc18\ub780\uc744 \ubc29\uc9c0\ud558\uae30 \uc704\ud574 \ub9c8\ub828\ud55c \ub300\ube44\ucc45\uc774\uc5c8\ub2e4. \uccad\uc8fc \uc131\uc8fc \uacf5\uc9c1\uc758 \uc544\ub4e4 \uc5ed\uc2dc \uc774\ub7f0 \uc774\uc720\ub85c \ub3c4\uc131\uc5d0 \uba38\ubb3c\ub800\ub294\ub370, \uacf5\uc9c1\uc758 \uc544\ub0b4\ub294 \uc774 \ub54c\ubb38\uc5d0 \ud56d\uc0c1 \uadfc\uc2ec\uc5d0 \uc0ac\ub85c\uc7a1\ud600 \uc788\uc5c8\ub2e4. \uadf8\ub9ac\uace0 \ub3d9\uc0dd \uacbd\uc885\uc5d0\uac8c \uc740\ubc00\ud788 \uc790\uc2e0\uc758 \uc544\ub4e4\uc744 \ub370\ub824\uc62c \uac83\uc744 \uc9c0\uc2dc\ud588\ub2e4. \ubcfc\ubaa8\ub97c \ub370\ub824\uac00\ub294 \uac83\uc740 \ubc18\uc5ed\uc73c\ub85c \ud574\uc11d\ud558\uc600\ub2e4. \ub530\ub77c\uc11c \uacbd\uc885\uc774 \uc870\uce74\ub97c \ub370\ub824\uac08 \uacc4\ud68d\uc744 \uc138\uc6b4 \uc801\uc774 \uc788\ub2e4\ub294 \uac83\uc740 \uc774\ubbf8 \uc624\ub798\uc804\ubd80\ud130 \uc5ed\ubaa8\ub97c \uacc4\ud68d\ud588\ub2e4\ub294 \uac83\uc744 \ub73b\ud55c\ub2e4\ub294 \uac83\uc774\ub2e4. \uc655\uac74\uc740 \uc5fc\uc0c1\uc9c4\uc758 \uc8fc\uc7a5\uc744 \ub4e3\uace0 \uacb0\uad6d \uacbd\uc885\uc744 \ube44\ub86f\ud55c \uc5ed\ubaa8 \ud610\uc758\uc790\ub4e4\uc744 \ubaa8\ub450 \uc8fd\uc774\uac8c \ub41c\ub2e4."} {"question": "\uce90\ub098\ub2e4 \ubba4\uc9c0\uc158 \uc624\uc6ec \ud314\ub81b\uc758 \ud65c\ub3d9 \uc774\ub984\uc740 \ubb34\uc5c7\uc778\uac00?", "paragraph": "\uc774 \uc568\ubc94\uc5d0 \uc2e4\ub9b0 Cliquot\uc740 \ubca0\uc774\ub8e8\ud2b8\uc758 \ubaa8\ub4e0 \uace1 \uac00\uc6b4\ub370\uc5d0\uc11c \ucf58\ub3c8\uc774 \ubd80\ub974\uc9c0 \uc54a\uc740 \uc720\uc77c\ud55c \uace1\uc774\ub2e4. \uac01\uc885 \ube0c\ub77c\uc2a4 \uc545\uae30\ub4e4\uacfc \ud0a4\ubcf4\ub4dc, \uc544\ucf54\ub514\uc5b8, \uc6b0\ucfe8\ub810\ub808, \ub9cc\ub3cc\ub9b0 \ub4f1 \ub2e4\uc591\ud55c \uc545\uae30\ub4e4\uc744 \uc5f0\uc8fc\ud558\ub294 \ucf58\ub3c8\uc774\uc9c0\ub9cc \ud604\uc545\uae30\ub9cc\ud07c\uc740 \uc190\uc73c\ub85c \ub72f\ub294 \uac83 \uc678\uc5d0 \uc81c\ub300\ub85c \ub2e4\ub8f0 \uc904\uc744 \ubaa8\ub974\uae30 \ub54c\ubb38\uc5d0 \uc678\ubd80\uc758 \ub3c4\uc6c0\uc744 \ubc1b\uc558\ub2e4. Arcade Fire\uc758 \uba64\ubc84\uc774\uae30\ub3c4 \ud588\uace0 \ud30c\uc774\ub110 \ud310\ud0c0\uc9c0\ub77c\ub294 \uc774\ub984\uc73c\ub85c \ud65c\ub3d9\ud558\ub294 \uce90\ub098\ub2e4 \ubba4\uc9c0\uc158 \uc624\uc6ec \ud314\ub81b\uc774 \uc568\ubc94\uc758 \uc804\uccb4\uc801\uc778 \uc2a4\ud2b8\ub9c1 \uc791\uc5c5\uc744 \ub3c4\ub9e1\uc544\uc11c \ud588\uc73c\uba70, \ud2b9\ud788 \uc774 \uace1\uc5d0\uc11c\ub294 \ucf58\ub3c8\uacfc \uc791\uace1\ub3c4 \uacf5\ub3d9\uc73c\ub85c \uc791\uc5c5\ud588\ub2e4. \ub178\ub798\uc758 \uc81c\ubaa9\uc740 \ucf58\ub3c8\uc774 \ubd99\uc600\uc9c0\ub9cc \uc791\uc0ac\uc640 \ubcf4\uceec\uc740 \ud314\ub81b\uc774 \uc9c1\uc811 \ub9e1\uc544\uc11c \ud588\uae30 \ub54c\ubb38\uc5d0 \uadf8\ub294 Cliquot\uc774 \ubca0\uc774\ub8e8\ud2b8\uc758 \uace1\uc774\uae30\ub3c4 \ud558\uc9c0\ub9cc \ubcf8\uc778\uc758 \uace1\uc774\uae30\ub3c4 \ud558\ub2e4\ub294 \uc790\ubd80\uc2ec\uc744 \ub4dc\ub7ec\ub0b4\uae30\ub3c4 \ud588\ub2e4. \ud314\ub81b\uc740 \uc774 \ub178\ub798\ub97c \uc704\ud574 \ubca0\uc774\ub8e8\ud2b8\uc758 \ucf58\uc11c\ud2b8\uc5d0\ub3c4 \uac8c\uc2a4\ud2b8\ub85c \ucc38\uc5ec\ud55c \uc801\uc774 \uc788\uc73c\uba70 \uc568\ubc94\uc791\uc5c5\uc744 \ud560 \uc218 \uc788\ub294 \uc2a4\ud29c\ub514\uc624\ub97c \ub9c8\ub828\ud574\uc8fc\ub294 \ub4f1 \ubca0\uc774\ub8e8\ud2b8\uc5d0\uac8c \ubb3c\uc2ec\uc591\uba74\uc73c\ub85c \ud070 \ub3c4\uc6c0\uc744 \uc8fc\uc5c8\ub2e4. Cliquot\uc758 \uc568\ubc94 \ubc84\uc804\uc5d0\ub294 \uc624\uc6ec \ud314\ub81b\uc758 \uccad\uc544\ud55c \ubaa9\uc18c\ub9ac\uac00 \ub2f4\uaca8 \uc788\uc73c\ub098 \ubba4\uc9c1\ube44\ub514\uc624\uc5d0\uc11c\ub294 Grizzly Bear\uc758 \uc5d0\ub4dc \ub4dc\ub85c\uc2a4\ud14c\uac00 \ubcf4\uceec\uc744 \ub9e1\uc558\ub2e4. \u2018\uc5b4\ub5a4 \uba5c\ub85c\ub514\uac00 \ub098\uc758 \uc0ac\ub791\uc744 \uadf8\uc758 \uce68\uc2e4\ub85c\ubd80\ud130 \uc774\ub04c\uc5b4\uc904\uae4c\u2019\ub77c\ub294 \uac00\uc0ac\uc758 \ud55c \uad6c\uc808 \ub54c\ubb38\uc5d0 \uc791\uc0ac\ub97c \ud55c \uc624\uc6ec \ud314\ub81b\uc774 \ucee4\ubc0d\uc544\uc6c3\ud55c \uac8c\uc774 \ubba4\uc9c0\uc158\uc774\ub77c\uc11c \u2018\uadf8\ub140\u2019\uac00 \uc544\ub2cc \u2018\uadf8\uc758 \uce68\uc2e4\u2019\uc774 \ub418\uc5c8\ub2e4\ub294 \ucd94\uce21\ub3c4 \uc624\uac00\uae30\ub3c4 \ud588\uc9c0\ub9cc \uadf8\ub294 \ud751\uc0ac\ubcd1\uc73c\ub85c \ub0a8\ud3b8\uc744 \uc783\uc740 \uc5ec\uc778\uc758 \uc774\uc57c\uae30\ub77c\uace0 \ubc1d\ud614\ub2e4.", "answer": "\ud30c\uc774\ub110 \ud310\ud0c0\uc9c0", "sentence": "Arcade Fire\uc758 \uba64\ubc84\uc774\uae30\ub3c4 \ud588\uace0 \ud30c\uc774\ub110 \ud310\ud0c0\uc9c0 \ub77c\ub294 \uc774\ub984\uc73c\ub85c \ud65c\ub3d9\ud558\ub294 \uce90\ub098\ub2e4 \ubba4\uc9c0\uc158 \uc624\uc6ec \ud314\ub81b\uc774 \uc568\ubc94\uc758 \uc804\uccb4\uc801\uc778 \uc2a4\ud2b8\ub9c1 \uc791\uc5c5\uc744 \ub3c4\ub9e1\uc544\uc11c \ud588\uc73c\uba70, \ud2b9\ud788 \uc774 \uace1\uc5d0\uc11c\ub294 \ucf58\ub3c8\uacfc \uc791\uace1\ub3c4 \uacf5\ub3d9\uc73c\ub85c \uc791\uc5c5\ud588\ub2e4.", "paragraph_sentence": "\uc774 \uc568\ubc94\uc5d0 \uc2e4\ub9b0 Cliquot\uc740 \ubca0\uc774\ub8e8\ud2b8\uc758 \ubaa8\ub4e0 \uace1 \uac00\uc6b4\ub370\uc5d0\uc11c \ucf58\ub3c8\uc774 \ubd80\ub974\uc9c0 \uc54a\uc740 \uc720\uc77c\ud55c \uace1\uc774\ub2e4. \uac01\uc885 \ube0c\ub77c\uc2a4 \uc545\uae30\ub4e4\uacfc \ud0a4\ubcf4\ub4dc, \uc544\ucf54\ub514\uc5b8, \uc6b0\ucfe8\ub810\ub808, \ub9cc\ub3cc\ub9b0 \ub4f1 \ub2e4\uc591\ud55c \uc545\uae30\ub4e4\uc744 \uc5f0\uc8fc\ud558\ub294 \ucf58\ub3c8\uc774\uc9c0\ub9cc \ud604\uc545\uae30\ub9cc\ud07c\uc740 \uc190\uc73c\ub85c \ub72f\ub294 \uac83 \uc678\uc5d0 \uc81c\ub300\ub85c \ub2e4\ub8f0 \uc904\uc744 \ubaa8\ub974\uae30 \ub54c\ubb38\uc5d0 \uc678\ubd80\uc758 \ub3c4\uc6c0\uc744 \ubc1b\uc558\ub2e4. Arcade Fire\uc758 \uba64\ubc84\uc774\uae30\ub3c4 \ud588\uace0 \ud30c\uc774\ub110 \ud310\ud0c0\uc9c0 \ub77c\ub294 \uc774\ub984\uc73c\ub85c \ud65c\ub3d9\ud558\ub294 \uce90\ub098\ub2e4 \ubba4\uc9c0\uc158 \uc624\uc6ec \ud314\ub81b\uc774 \uc568\ubc94\uc758 \uc804\uccb4\uc801\uc778 \uc2a4\ud2b8\ub9c1 \uc791\uc5c5\uc744 \ub3c4\ub9e1\uc544\uc11c \ud588\uc73c\uba70, \ud2b9\ud788 \uc774 \uace1\uc5d0\uc11c\ub294 \ucf58\ub3c8\uacfc \uc791\uace1\ub3c4 \uacf5\ub3d9\uc73c\ub85c \uc791\uc5c5\ud588\ub2e4. \ub178\ub798\uc758 \uc81c\ubaa9\uc740 \ucf58\ub3c8\uc774 \ubd99\uc600\uc9c0\ub9cc \uc791\uc0ac\uc640 \ubcf4\uceec\uc740 \ud314\ub81b\uc774 \uc9c1\uc811 \ub9e1\uc544\uc11c \ud588\uae30 \ub54c\ubb38\uc5d0 \uadf8\ub294 Cliquot\uc774 \ubca0\uc774\ub8e8\ud2b8\uc758 \uace1\uc774\uae30\ub3c4 \ud558\uc9c0\ub9cc \ubcf8\uc778\uc758 \uace1\uc774\uae30\ub3c4 \ud558\ub2e4\ub294 \uc790\ubd80\uc2ec\uc744 \ub4dc\ub7ec\ub0b4\uae30\ub3c4 \ud588\ub2e4. \ud314\ub81b\uc740 \uc774 \ub178\ub798\ub97c \uc704\ud574 \ubca0\uc774\ub8e8\ud2b8\uc758 \ucf58\uc11c\ud2b8\uc5d0\ub3c4 \uac8c\uc2a4\ud2b8\ub85c \ucc38\uc5ec\ud55c \uc801\uc774 \uc788\uc73c\uba70 \uc568\ubc94\uc791\uc5c5\uc744 \ud560 \uc218 \uc788\ub294 \uc2a4\ud29c\ub514\uc624\ub97c \ub9c8\ub828\ud574\uc8fc\ub294 \ub4f1 \ubca0\uc774\ub8e8\ud2b8\uc5d0\uac8c \ubb3c\uc2ec\uc591\uba74\uc73c\ub85c \ud070 \ub3c4\uc6c0\uc744 \uc8fc\uc5c8\ub2e4. Cliquot\uc758 \uc568\ubc94 \ubc84\uc804\uc5d0\ub294 \uc624\uc6ec \ud314\ub81b\uc758 \uccad\uc544\ud55c \ubaa9\uc18c\ub9ac\uac00 \ub2f4\uaca8 \uc788\uc73c\ub098 \ubba4\uc9c1\ube44\ub514\uc624\uc5d0\uc11c\ub294 Grizzly Bear\uc758 \uc5d0\ub4dc \ub4dc\ub85c\uc2a4\ud14c\uac00 \ubcf4\uceec\uc744 \ub9e1\uc558\ub2e4. \u2018\uc5b4\ub5a4 \uba5c\ub85c\ub514\uac00 \ub098\uc758 \uc0ac\ub791\uc744 \uadf8\uc758 \uce68\uc2e4\ub85c\ubd80\ud130 \uc774\ub04c\uc5b4\uc904\uae4c\u2019\ub77c\ub294 \uac00\uc0ac\uc758 \ud55c \uad6c\uc808 \ub54c\ubb38\uc5d0 \uc791\uc0ac\ub97c \ud55c \uc624\uc6ec \ud314\ub81b\uc774 \ucee4\ubc0d\uc544\uc6c3\ud55c \uac8c\uc774 \ubba4\uc9c0\uc158\uc774\ub77c\uc11c \u2018\uadf8\ub140\u2019\uac00 \uc544\ub2cc \u2018\uadf8\uc758 \uce68\uc2e4\u2019\uc774 \ub418\uc5c8\ub2e4\ub294 \ucd94\uce21\ub3c4 \uc624\uac00\uae30\ub3c4 \ud588\uc9c0\ub9cc \uadf8\ub294 \ud751\uc0ac\ubcd1\uc73c\ub85c \ub0a8\ud3b8\uc744 \uc783\uc740 \uc5ec\uc778\uc758 \uc774\uc57c\uae30\ub77c\uace0 \ubc1d\ud614\ub2e4.", "paragraph_answer": "\uc774 \uc568\ubc94\uc5d0 \uc2e4\ub9b0 Cliquot\uc740 \ubca0\uc774\ub8e8\ud2b8\uc758 \ubaa8\ub4e0 \uace1 \uac00\uc6b4\ub370\uc5d0\uc11c \ucf58\ub3c8\uc774 \ubd80\ub974\uc9c0 \uc54a\uc740 \uc720\uc77c\ud55c \uace1\uc774\ub2e4. \uac01\uc885 \ube0c\ub77c\uc2a4 \uc545\uae30\ub4e4\uacfc \ud0a4\ubcf4\ub4dc, \uc544\ucf54\ub514\uc5b8, \uc6b0\ucfe8\ub810\ub808, \ub9cc\ub3cc\ub9b0 \ub4f1 \ub2e4\uc591\ud55c \uc545\uae30\ub4e4\uc744 \uc5f0\uc8fc\ud558\ub294 \ucf58\ub3c8\uc774\uc9c0\ub9cc \ud604\uc545\uae30\ub9cc\ud07c\uc740 \uc190\uc73c\ub85c \ub72f\ub294 \uac83 \uc678\uc5d0 \uc81c\ub300\ub85c \ub2e4\ub8f0 \uc904\uc744 \ubaa8\ub974\uae30 \ub54c\ubb38\uc5d0 \uc678\ubd80\uc758 \ub3c4\uc6c0\uc744 \ubc1b\uc558\ub2e4. Arcade Fire\uc758 \uba64\ubc84\uc774\uae30\ub3c4 \ud588\uace0 \ud30c\uc774\ub110 \ud310\ud0c0\uc9c0 \ub77c\ub294 \uc774\ub984\uc73c\ub85c \ud65c\ub3d9\ud558\ub294 \uce90\ub098\ub2e4 \ubba4\uc9c0\uc158 \uc624\uc6ec \ud314\ub81b\uc774 \uc568\ubc94\uc758 \uc804\uccb4\uc801\uc778 \uc2a4\ud2b8\ub9c1 \uc791\uc5c5\uc744 \ub3c4\ub9e1\uc544\uc11c \ud588\uc73c\uba70, \ud2b9\ud788 \uc774 \uace1\uc5d0\uc11c\ub294 \ucf58\ub3c8\uacfc \uc791\uace1\ub3c4 \uacf5\ub3d9\uc73c\ub85c \uc791\uc5c5\ud588\ub2e4. \ub178\ub798\uc758 \uc81c\ubaa9\uc740 \ucf58\ub3c8\uc774 \ubd99\uc600\uc9c0\ub9cc \uc791\uc0ac\uc640 \ubcf4\uceec\uc740 \ud314\ub81b\uc774 \uc9c1\uc811 \ub9e1\uc544\uc11c \ud588\uae30 \ub54c\ubb38\uc5d0 \uadf8\ub294 Cliquot\uc774 \ubca0\uc774\ub8e8\ud2b8\uc758 \uace1\uc774\uae30\ub3c4 \ud558\uc9c0\ub9cc \ubcf8\uc778\uc758 \uace1\uc774\uae30\ub3c4 \ud558\ub2e4\ub294 \uc790\ubd80\uc2ec\uc744 \ub4dc\ub7ec\ub0b4\uae30\ub3c4 \ud588\ub2e4. \ud314\ub81b\uc740 \uc774 \ub178\ub798\ub97c \uc704\ud574 \ubca0\uc774\ub8e8\ud2b8\uc758 \ucf58\uc11c\ud2b8\uc5d0\ub3c4 \uac8c\uc2a4\ud2b8\ub85c \ucc38\uc5ec\ud55c \uc801\uc774 \uc788\uc73c\uba70 \uc568\ubc94\uc791\uc5c5\uc744 \ud560 \uc218 \uc788\ub294 \uc2a4\ud29c\ub514\uc624\ub97c \ub9c8\ub828\ud574\uc8fc\ub294 \ub4f1 \ubca0\uc774\ub8e8\ud2b8\uc5d0\uac8c \ubb3c\uc2ec\uc591\uba74\uc73c\ub85c \ud070 \ub3c4\uc6c0\uc744 \uc8fc\uc5c8\ub2e4. Cliquot\uc758 \uc568\ubc94 \ubc84\uc804\uc5d0\ub294 \uc624\uc6ec \ud314\ub81b\uc758 \uccad\uc544\ud55c \ubaa9\uc18c\ub9ac\uac00 \ub2f4\uaca8 \uc788\uc73c\ub098 \ubba4\uc9c1\ube44\ub514\uc624\uc5d0\uc11c\ub294 Grizzly Bear\uc758 \uc5d0\ub4dc \ub4dc\ub85c\uc2a4\ud14c\uac00 \ubcf4\uceec\uc744 \ub9e1\uc558\ub2e4. \u2018\uc5b4\ub5a4 \uba5c\ub85c\ub514\uac00 \ub098\uc758 \uc0ac\ub791\uc744 \uadf8\uc758 \uce68\uc2e4\ub85c\ubd80\ud130 \uc774\ub04c\uc5b4\uc904\uae4c\u2019\ub77c\ub294 \uac00\uc0ac\uc758 \ud55c \uad6c\uc808 \ub54c\ubb38\uc5d0 \uc791\uc0ac\ub97c \ud55c \uc624\uc6ec \ud314\ub81b\uc774 \ucee4\ubc0d\uc544\uc6c3\ud55c \uac8c\uc774 \ubba4\uc9c0\uc158\uc774\ub77c\uc11c \u2018\uadf8\ub140\u2019\uac00 \uc544\ub2cc \u2018\uadf8\uc758 \uce68\uc2e4\u2019\uc774 \ub418\uc5c8\ub2e4\ub294 \ucd94\uce21\ub3c4 \uc624\uac00\uae30\ub3c4 \ud588\uc9c0\ub9cc \uadf8\ub294 \ud751\uc0ac\ubcd1\uc73c\ub85c \ub0a8\ud3b8\uc744 \uc783\uc740 \uc5ec\uc778\uc758 \uc774\uc57c\uae30\ub77c\uace0 \ubc1d\ud614\ub2e4.", "sentence_answer": "Arcade Fire\uc758 \uba64\ubc84\uc774\uae30\ub3c4 \ud588\uace0 \ud30c\uc774\ub110 \ud310\ud0c0\uc9c0 \ub77c\ub294 \uc774\ub984\uc73c\ub85c \ud65c\ub3d9\ud558\ub294 \uce90\ub098\ub2e4 \ubba4\uc9c0\uc158 \uc624\uc6ec \ud314\ub81b\uc774 \uc568\ubc94\uc758 \uc804\uccb4\uc801\uc778 \uc2a4\ud2b8\ub9c1 \uc791\uc5c5\uc744 \ub3c4\ub9e1\uc544\uc11c \ud588\uc73c\uba70, \ud2b9\ud788 \uc774 \uace1\uc5d0\uc11c\ub294 \ucf58\ub3c8\uacfc \uc791\uace1\ub3c4 \uacf5\ub3d9\uc73c\ub85c \uc791\uc5c5\ud588\ub2e4.", "paragraph_id": "6418825-4-0", "paragraph_question": "question: \uce90\ub098\ub2e4 \ubba4\uc9c0\uc158 \uc624\uc6ec \ud314\ub81b\uc758 \ud65c\ub3d9 \uc774\ub984\uc740 \ubb34\uc5c7\uc778\uac00?, context: \uc774 \uc568\ubc94\uc5d0 \uc2e4\ub9b0 Cliquot\uc740 \ubca0\uc774\ub8e8\ud2b8\uc758 \ubaa8\ub4e0 \uace1 \uac00\uc6b4\ub370\uc5d0\uc11c \ucf58\ub3c8\uc774 \ubd80\ub974\uc9c0 \uc54a\uc740 \uc720\uc77c\ud55c \uace1\uc774\ub2e4. \uac01\uc885 \ube0c\ub77c\uc2a4 \uc545\uae30\ub4e4\uacfc \ud0a4\ubcf4\ub4dc, \uc544\ucf54\ub514\uc5b8, \uc6b0\ucfe8\ub810\ub808, \ub9cc\ub3cc\ub9b0 \ub4f1 \ub2e4\uc591\ud55c \uc545\uae30\ub4e4\uc744 \uc5f0\uc8fc\ud558\ub294 \ucf58\ub3c8\uc774\uc9c0\ub9cc \ud604\uc545\uae30\ub9cc\ud07c\uc740 \uc190\uc73c\ub85c \ub72f\ub294 \uac83 \uc678\uc5d0 \uc81c\ub300\ub85c \ub2e4\ub8f0 \uc904\uc744 \ubaa8\ub974\uae30 \ub54c\ubb38\uc5d0 \uc678\ubd80\uc758 \ub3c4\uc6c0\uc744 \ubc1b\uc558\ub2e4. Arcade Fire\uc758 \uba64\ubc84\uc774\uae30\ub3c4 \ud588\uace0 \ud30c\uc774\ub110 \ud310\ud0c0\uc9c0\ub77c\ub294 \uc774\ub984\uc73c\ub85c \ud65c\ub3d9\ud558\ub294 \uce90\ub098\ub2e4 \ubba4\uc9c0\uc158 \uc624\uc6ec \ud314\ub81b\uc774 \uc568\ubc94\uc758 \uc804\uccb4\uc801\uc778 \uc2a4\ud2b8\ub9c1 \uc791\uc5c5\uc744 \ub3c4\ub9e1\uc544\uc11c \ud588\uc73c\uba70, \ud2b9\ud788 \uc774 \uace1\uc5d0\uc11c\ub294 \ucf58\ub3c8\uacfc \uc791\uace1\ub3c4 \uacf5\ub3d9\uc73c\ub85c \uc791\uc5c5\ud588\ub2e4. \ub178\ub798\uc758 \uc81c\ubaa9\uc740 \ucf58\ub3c8\uc774 \ubd99\uc600\uc9c0\ub9cc \uc791\uc0ac\uc640 \ubcf4\uceec\uc740 \ud314\ub81b\uc774 \uc9c1\uc811 \ub9e1\uc544\uc11c \ud588\uae30 \ub54c\ubb38\uc5d0 \uadf8\ub294 Cliquot\uc774 \ubca0\uc774\ub8e8\ud2b8\uc758 \uace1\uc774\uae30\ub3c4 \ud558\uc9c0\ub9cc \ubcf8\uc778\uc758 \uace1\uc774\uae30\ub3c4 \ud558\ub2e4\ub294 \uc790\ubd80\uc2ec\uc744 \ub4dc\ub7ec\ub0b4\uae30\ub3c4 \ud588\ub2e4. \ud314\ub81b\uc740 \uc774 \ub178\ub798\ub97c \uc704\ud574 \ubca0\uc774\ub8e8\ud2b8\uc758 \ucf58\uc11c\ud2b8\uc5d0\ub3c4 \uac8c\uc2a4\ud2b8\ub85c \ucc38\uc5ec\ud55c \uc801\uc774 \uc788\uc73c\uba70 \uc568\ubc94\uc791\uc5c5\uc744 \ud560 \uc218 \uc788\ub294 \uc2a4\ud29c\ub514\uc624\ub97c \ub9c8\ub828\ud574\uc8fc\ub294 \ub4f1 \ubca0\uc774\ub8e8\ud2b8\uc5d0\uac8c \ubb3c\uc2ec\uc591\uba74\uc73c\ub85c \ud070 \ub3c4\uc6c0\uc744 \uc8fc\uc5c8\ub2e4. Cliquot\uc758 \uc568\ubc94 \ubc84\uc804\uc5d0\ub294 \uc624\uc6ec \ud314\ub81b\uc758 \uccad\uc544\ud55c \ubaa9\uc18c\ub9ac\uac00 \ub2f4\uaca8 \uc788\uc73c\ub098 \ubba4\uc9c1\ube44\ub514\uc624\uc5d0\uc11c\ub294 Grizzly Bear\uc758 \uc5d0\ub4dc \ub4dc\ub85c\uc2a4\ud14c\uac00 \ubcf4\uceec\uc744 \ub9e1\uc558\ub2e4. \u2018\uc5b4\ub5a4 \uba5c\ub85c\ub514\uac00 \ub098\uc758 \uc0ac\ub791\uc744 \uadf8\uc758 \uce68\uc2e4\ub85c\ubd80\ud130 \uc774\ub04c\uc5b4\uc904\uae4c\u2019\ub77c\ub294 \uac00\uc0ac\uc758 \ud55c \uad6c\uc808 \ub54c\ubb38\uc5d0 \uc791\uc0ac\ub97c \ud55c \uc624\uc6ec \ud314\ub81b\uc774 \ucee4\ubc0d\uc544\uc6c3\ud55c \uac8c\uc774 \ubba4\uc9c0\uc158\uc774\ub77c\uc11c \u2018\uadf8\ub140\u2019\uac00 \uc544\ub2cc \u2018\uadf8\uc758 \uce68\uc2e4\u2019\uc774 \ub418\uc5c8\ub2e4\ub294 \ucd94\uce21\ub3c4 \uc624\uac00\uae30\ub3c4 \ud588\uc9c0\ub9cc \uadf8\ub294 \ud751\uc0ac\ubcd1\uc73c\ub85c \ub0a8\ud3b8\uc744 \uc783\uc740 \uc5ec\uc778\uc758 \uc774\uc57c\uae30\ub77c\uace0 \ubc1d\ud614\ub2e4."} {"question": "\ud3ec\uc774\uc5b4\ubc14\ud750\uc758 \uc8fc\uc7a5\uc5d0 \uc758\ud558\uba74 \uc778\uac04\uc744 \uc678\uba74\ud654\ud55c \uac83\uc5d0 \ud574\ub2f9\ud558\ub294 \uac83\uc740 \ubb34\uc5c7\uc778\uac00?", "paragraph": "2\ubd80\uc5d0\uc11c\ub294 \uc774\uc804\uc758 \uc124\uba85\uc744 \ucca0\ud68c\ud558\ub294 \uac83\ucc98\ub7fc \ubcf4\uc774\ub294 \uac83\uc774 \uc694\uc810\uc774\ub2e4. \ud3ec\uc774\uc5b4\ubc14\ud750\ub294 \uc2e0\uc774 \uc720\uc77c\ud558\uac8c \ud558\ub294 \uac83\uc740 \"\ub3c4\ub355\uc131\uacfc \uc778\uac04\uc758 \uc601\uc6d0\ud55c \uad6c\uc6d0\uc774\ub2e4. \uadf8\ub7ec\ubbc0\ub85c \uc778\uac04\uc740 \uc0ac\uc2e4 \uc790\uae30 \uc790\uc2e0\uc774 \ubaa9\uc801\uc77c \ub530\ub984\uc774\ub2e4.\"\ub77c\uace0 \uc8fc\uc7a5\ud558\ub294\ub370, \uc778\uac04\uc758 \ud589\uc704\ub4e4\uc740 \uc2e0\uc5d0\uac8c \ub193\uc5ec\uc788\uae30 \ub54c\ubb38\uc774\ub2e4. \ud3ec\uc774\uc5b4\ubc14\ud750\ub294 \uc778\uac04\uc740 \uc790\uc2e0\uc758 \uac1c\uc778\uc131\uc744 \ud3ec\uae30\ud558\uace0 \uc774\ub97c \uc774\ubc88\uc5d0\ub294 \uc774\uae30\uc801\uc778 \uc874\uc7ac\uc778 \uc2e0\uc5d0\uac8c \ub9e1\uae34\ub2e4\uace0 \uc8fc\uc7a5\ud558\ub294\ub370, \uc5ec\uae30\uc11c \uc774\uc804\uc758 \uc790\uae30 \uc8fc\uc7a5\uacfc \ubaa8\uc21c\uc774 \uc0dd\uae34\ub2e4. \uc774\ub7ec\ud55c \uc2e0\uc758 \uc774\uae30\uc131\uc740 \uc778\uac04\uc744 \uc790\uadf9\ud558\uc5ec \uc778\uac04\uc774 \uc0ac\uc545\ud558\uace0 \ud0c0\ub77d\ud558\uac8c \ub418\ub3c4\ub85d \ud22c\uc0ac\ud558\ub294\ub370, \uc774\ub9ac\ud558\uc5ec \uc778\uac04\uc774 \"\uc120\ud560 \uc218\ub294 \uc5c6\uace0\", \"\uc120\ud55c \uc874\uc7ac\"\ub294 \uc624\uc9c1 \uc2e0\ubc16\uc5d0 \uc5c6\ub2e4. \uc774\ub7f0 \ubc29\ubc95\uc73c\ub85c \ud3ec\uc774\uc5b4\ubc14\ud750\ub294 \uc2e0\uc744 \uc22d\ubc30\ud568\uc73c\ub85c\uc368 \uc778\uac04\uc5d0\uac8c \uc77c\uc5b4\ub098\ub294 \ub2e8\uc808\uc744 \ubcf4\uc5ec\uc90c\uc73c\ub85c\uc368 \uc790\uc2e0\uc758 \ucd08\uae30 \uc8fc\uc7a5\ub4e4\uc744 \ub4a4\uc5ce\ub294\ub2e4. \ud3ec\uc774\uc5b4\ubc14\ud750\ub294 \"\uc2e0\uc73c\ub85c \uc758\uc778\ud654\ub41c\" \uc120\ud568\uc774 \uc2e0\uc744 \ub300\uc0c1\uc73c\ub85c \ub9cc\ub4e4\uc5c8\ub2e4\uace0 \ub2e8\uc5b8\ud558\ub294\ub370, \ub9cc\uc77c \uc2e0\uc774 \ub300\uc0c1\uc774 \uc544\ub2c8\ub77c\uba74 \uc2e0\uc744 \uc758\uc778\ud654\ud558\uae30 \uc704\ud574 \uc544\ubb34\uac83\ub3c4 \ud544\uc694\ud558\uc9c0 \uc54a\uae30 \ub54c\ubb38\uc774\ub2e4. \ub300\uc0c1\ub4e4\ub85c\uc11c\uc758 \uad00\uc810\uc740 \uc774\uc804\ubd80\ud130 \ub17c\uc758\ub418\uc5b4 \uc654\ub2e4. \uadf8\ub7f0 \uc810\uc5d0\uc11c \uc778\uac04\uc740 \ub300\uc0c1\ub4e4\uc744 \uc0dd\uac01\ud558\uba70 \uadf8 \ub300\uc0c1\ub4e4 \uc790\uccb4\uac00 \uc778\uac04\uc744 \uc678\uba74\ud654\ud558\ub294 \uac1c\ub150\uc744 \ub358\uc838\uc900\ub2e4. \uadf8\ub7ec\ubbc0\ub85c \ub9cc\uc57d \uc2e0\uc774 \uc120\ud558\ub2e4\uba74 \uc778\uac04\uc774\uc5b4\uc57c \ud558\ub294\ub370 \uc2e0\uc740 \ub300\uc0c1\uc774\ubbc0\ub85c \uc2e0\uc740 \ub2e8\uc9c0 \uc778\uac04\uc744 \uc678\uba74\ud654\ud55c \uac83\uc774\uae30 \ub54c\ubb38\uc774\ub2e4. \uadf8\ub7ec\ub098 \uc885\uad50\ub294 \uc778\uac04\uc774 \ubcf8\ub798\ubd80\ud130 \ubd80\ub3c4\ub355\ud558\ub2e4\uace0 \ud55c\ub2e4. \ud3ec\uc774\uc5b4\ubc14\ud750\ub294 \ub9cc\uc57d \"\ub9cc\uc57d \ub098\uc758 \uc815\uc2e0\uc774 \ubbf8\uc801\uc73c\ub85c \ud0c0\ub77d\ud55c \uc808\ub300\uc801\uc778 \uc874\uc7ac\ub77c\uba74 \ub0b4\uac00 \ud6cc\ub96d\ud55c \uadf8\ub9bc\uc758 \uc544\ub984\ub2e4\uc6c0\uc744 \uc778\uc2dd\ud560 \uc218 \uc788\uc744 \uac83\uc778\uac00?\"\ub77c\ub294 \uac00\ub2a5\uc131\uc744 \ubb3c\uc74c\uc73c\ub85c\uc368 \uc790\uc2e0\uc758 \ubaa8\uc21c\uc744 \uc904\uc774\ub824\uace0 \ud588\ub2e4. \ud3ec\uc774\uc5b4\ubc14\ud750\uc758 \ucd94\ub860\uc5d0 \ub530\ub974\uba74 \uc774\uac83\uc740 \ubd88\uac00\ub2a5\ud560 \uac83\uc774\ub098 \uac00\ub2a5\ud558\uace0, \ud6c4\uc5d0 \uc778\uac04\uc740 \uc544\ub984\ub2e4\uc6c0\uc744 \ubc1c\uacac\ud560 \ub2a5\ub825\uc774 \uc788\ub2e4\ub294 \uac83\uc744 \uae30\uc220\ud55c\ub2e4.", "answer": "\uc2e0", "sentence": "\ud3ec\uc774\uc5b4\ubc14\ud750\ub294 \uc2e0 \uc774 \uc720\uc77c\ud558\uac8c \ud558\ub294 \uac83\uc740 \"\ub3c4\ub355\uc131\uacfc \uc778\uac04\uc758 \uc601\uc6d0\ud55c \uad6c\uc6d0\uc774\ub2e4.", "paragraph_sentence": "2\ubd80\uc5d0\uc11c\ub294 \uc774\uc804\uc758 \uc124\uba85\uc744 \ucca0\ud68c\ud558\ub294 \uac83\ucc98\ub7fc \ubcf4\uc774\ub294 \uac83\uc774 \uc694\uc810\uc774\ub2e4. \ud3ec\uc774\uc5b4\ubc14\ud750\ub294 \uc2e0 \uc774 \uc720\uc77c\ud558\uac8c \ud558\ub294 \uac83\uc740 \"\ub3c4\ub355\uc131\uacfc \uc778\uac04\uc758 \uc601\uc6d0\ud55c \uad6c\uc6d0\uc774\ub2e4. \uadf8\ub7ec\ubbc0\ub85c \uc778\uac04\uc740 \uc0ac\uc2e4 \uc790\uae30 \uc790\uc2e0\uc774 \ubaa9\uc801\uc77c \ub530\ub984\uc774\ub2e4. \"\ub77c\uace0 \uc8fc\uc7a5\ud558\ub294\ub370, \uc778\uac04\uc758 \ud589\uc704\ub4e4\uc740 \uc2e0\uc5d0\uac8c \ub193\uc5ec\uc788\uae30 \ub54c\ubb38\uc774\ub2e4. \ud3ec\uc774\uc5b4\ubc14\ud750\ub294 \uc778\uac04\uc740 \uc790\uc2e0\uc758 \uac1c\uc778\uc131\uc744 \ud3ec\uae30\ud558\uace0 \uc774\ub97c \uc774\ubc88\uc5d0\ub294 \uc774\uae30\uc801\uc778 \uc874\uc7ac\uc778 \uc2e0\uc5d0\uac8c \ub9e1\uae34\ub2e4\uace0 \uc8fc\uc7a5\ud558\ub294\ub370, \uc5ec\uae30\uc11c \uc774\uc804\uc758 \uc790\uae30 \uc8fc\uc7a5\uacfc \ubaa8\uc21c\uc774 \uc0dd\uae34\ub2e4. \uc774\ub7ec\ud55c \uc2e0\uc758 \uc774\uae30\uc131\uc740 \uc778\uac04\uc744 \uc790\uadf9\ud558\uc5ec \uc778\uac04\uc774 \uc0ac\uc545\ud558\uace0 \ud0c0\ub77d\ud558\uac8c \ub418\ub3c4\ub85d \ud22c\uc0ac\ud558\ub294\ub370, \uc774\ub9ac\ud558\uc5ec \uc778\uac04\uc774 \"\uc120\ud560 \uc218\ub294 \uc5c6\uace0\", \"\uc120\ud55c \uc874\uc7ac\"\ub294 \uc624\uc9c1 \uc2e0\ubc16\uc5d0 \uc5c6\ub2e4. \uc774\ub7f0 \ubc29\ubc95\uc73c\ub85c \ud3ec\uc774\uc5b4\ubc14\ud750\ub294 \uc2e0\uc744 \uc22d\ubc30\ud568\uc73c\ub85c\uc368 \uc778\uac04\uc5d0\uac8c \uc77c\uc5b4\ub098\ub294 \ub2e8\uc808\uc744 \ubcf4\uc5ec\uc90c\uc73c\ub85c\uc368 \uc790\uc2e0\uc758 \ucd08\uae30 \uc8fc\uc7a5\ub4e4\uc744 \ub4a4\uc5ce\ub294\ub2e4. \ud3ec\uc774\uc5b4\ubc14\ud750\ub294 \"\uc2e0\uc73c\ub85c \uc758\uc778\ud654\ub41c\" \uc120\ud568\uc774 \uc2e0\uc744 \ub300\uc0c1\uc73c\ub85c \ub9cc\ub4e4\uc5c8\ub2e4\uace0 \ub2e8\uc5b8\ud558\ub294\ub370, \ub9cc\uc77c \uc2e0\uc774 \ub300\uc0c1\uc774 \uc544\ub2c8\ub77c\uba74 \uc2e0\uc744 \uc758\uc778\ud654\ud558\uae30 \uc704\ud574 \uc544\ubb34\uac83\ub3c4 \ud544\uc694\ud558\uc9c0 \uc54a\uae30 \ub54c\ubb38\uc774\ub2e4. \ub300\uc0c1\ub4e4\ub85c\uc11c\uc758 \uad00\uc810\uc740 \uc774\uc804\ubd80\ud130 \ub17c\uc758\ub418\uc5b4 \uc654\ub2e4. \uadf8\ub7f0 \uc810\uc5d0\uc11c \uc778\uac04\uc740 \ub300\uc0c1\ub4e4\uc744 \uc0dd\uac01\ud558\uba70 \uadf8 \ub300\uc0c1\ub4e4 \uc790\uccb4\uac00 \uc778\uac04\uc744 \uc678\uba74\ud654\ud558\ub294 \uac1c\ub150\uc744 \ub358\uc838\uc900\ub2e4. \uadf8\ub7ec\ubbc0\ub85c \ub9cc\uc57d \uc2e0\uc774 \uc120\ud558\ub2e4\uba74 \uc778\uac04\uc774\uc5b4\uc57c \ud558\ub294\ub370 \uc2e0\uc740 \ub300\uc0c1\uc774\ubbc0\ub85c \uc2e0\uc740 \ub2e8\uc9c0 \uc778\uac04\uc744 \uc678\uba74\ud654\ud55c \uac83\uc774\uae30 \ub54c\ubb38\uc774\ub2e4. \uadf8\ub7ec\ub098 \uc885\uad50\ub294 \uc778\uac04\uc774 \ubcf8\ub798\ubd80\ud130 \ubd80\ub3c4\ub355\ud558\ub2e4\uace0 \ud55c\ub2e4. \ud3ec\uc774\uc5b4\ubc14\ud750\ub294 \ub9cc\uc57d \"\ub9cc\uc57d \ub098\uc758 \uc815\uc2e0\uc774 \ubbf8\uc801\uc73c\ub85c \ud0c0\ub77d\ud55c \uc808\ub300\uc801\uc778 \uc874\uc7ac\ub77c\uba74 \ub0b4\uac00 \ud6cc\ub96d\ud55c \uadf8\ub9bc\uc758 \uc544\ub984\ub2e4\uc6c0\uc744 \uc778\uc2dd\ud560 \uc218 \uc788\uc744 \uac83\uc778\uac00?\"\ub77c\ub294 \uac00\ub2a5\uc131\uc744 \ubb3c\uc74c\uc73c\ub85c\uc368 \uc790\uc2e0\uc758 \ubaa8\uc21c\uc744 \uc904\uc774\ub824\uace0 \ud588\ub2e4. \ud3ec\uc774\uc5b4\ubc14\ud750\uc758 \ucd94\ub860\uc5d0 \ub530\ub974\uba74 \uc774\uac83\uc740 \ubd88\uac00\ub2a5\ud560 \uac83\uc774\ub098 \uac00\ub2a5\ud558\uace0, \ud6c4\uc5d0 \uc778\uac04\uc740 \uc544\ub984\ub2e4\uc6c0\uc744 \ubc1c\uacac\ud560 \ub2a5\ub825\uc774 \uc788\ub2e4\ub294 \uac83\uc744 \uae30\uc220\ud55c\ub2e4.", "paragraph_answer": "2\ubd80\uc5d0\uc11c\ub294 \uc774\uc804\uc758 \uc124\uba85\uc744 \ucca0\ud68c\ud558\ub294 \uac83\ucc98\ub7fc \ubcf4\uc774\ub294 \uac83\uc774 \uc694\uc810\uc774\ub2e4. \ud3ec\uc774\uc5b4\ubc14\ud750\ub294 \uc2e0 \uc774 \uc720\uc77c\ud558\uac8c \ud558\ub294 \uac83\uc740 \"\ub3c4\ub355\uc131\uacfc \uc778\uac04\uc758 \uc601\uc6d0\ud55c \uad6c\uc6d0\uc774\ub2e4. \uadf8\ub7ec\ubbc0\ub85c \uc778\uac04\uc740 \uc0ac\uc2e4 \uc790\uae30 \uc790\uc2e0\uc774 \ubaa9\uc801\uc77c \ub530\ub984\uc774\ub2e4.\"\ub77c\uace0 \uc8fc\uc7a5\ud558\ub294\ub370, \uc778\uac04\uc758 \ud589\uc704\ub4e4\uc740 \uc2e0\uc5d0\uac8c \ub193\uc5ec\uc788\uae30 \ub54c\ubb38\uc774\ub2e4. \ud3ec\uc774\uc5b4\ubc14\ud750\ub294 \uc778\uac04\uc740 \uc790\uc2e0\uc758 \uac1c\uc778\uc131\uc744 \ud3ec\uae30\ud558\uace0 \uc774\ub97c \uc774\ubc88\uc5d0\ub294 \uc774\uae30\uc801\uc778 \uc874\uc7ac\uc778 \uc2e0\uc5d0\uac8c \ub9e1\uae34\ub2e4\uace0 \uc8fc\uc7a5\ud558\ub294\ub370, \uc5ec\uae30\uc11c \uc774\uc804\uc758 \uc790\uae30 \uc8fc\uc7a5\uacfc \ubaa8\uc21c\uc774 \uc0dd\uae34\ub2e4. \uc774\ub7ec\ud55c \uc2e0\uc758 \uc774\uae30\uc131\uc740 \uc778\uac04\uc744 \uc790\uadf9\ud558\uc5ec \uc778\uac04\uc774 \uc0ac\uc545\ud558\uace0 \ud0c0\ub77d\ud558\uac8c \ub418\ub3c4\ub85d \ud22c\uc0ac\ud558\ub294\ub370, \uc774\ub9ac\ud558\uc5ec \uc778\uac04\uc774 \"\uc120\ud560 \uc218\ub294 \uc5c6\uace0\", \"\uc120\ud55c \uc874\uc7ac\"\ub294 \uc624\uc9c1 \uc2e0\ubc16\uc5d0 \uc5c6\ub2e4. \uc774\ub7f0 \ubc29\ubc95\uc73c\ub85c \ud3ec\uc774\uc5b4\ubc14\ud750\ub294 \uc2e0\uc744 \uc22d\ubc30\ud568\uc73c\ub85c\uc368 \uc778\uac04\uc5d0\uac8c \uc77c\uc5b4\ub098\ub294 \ub2e8\uc808\uc744 \ubcf4\uc5ec\uc90c\uc73c\ub85c\uc368 \uc790\uc2e0\uc758 \ucd08\uae30 \uc8fc\uc7a5\ub4e4\uc744 \ub4a4\uc5ce\ub294\ub2e4. \ud3ec\uc774\uc5b4\ubc14\ud750\ub294 \"\uc2e0\uc73c\ub85c \uc758\uc778\ud654\ub41c\" \uc120\ud568\uc774 \uc2e0\uc744 \ub300\uc0c1\uc73c\ub85c \ub9cc\ub4e4\uc5c8\ub2e4\uace0 \ub2e8\uc5b8\ud558\ub294\ub370, \ub9cc\uc77c \uc2e0\uc774 \ub300\uc0c1\uc774 \uc544\ub2c8\ub77c\uba74 \uc2e0\uc744 \uc758\uc778\ud654\ud558\uae30 \uc704\ud574 \uc544\ubb34\uac83\ub3c4 \ud544\uc694\ud558\uc9c0 \uc54a\uae30 \ub54c\ubb38\uc774\ub2e4. \ub300\uc0c1\ub4e4\ub85c\uc11c\uc758 \uad00\uc810\uc740 \uc774\uc804\ubd80\ud130 \ub17c\uc758\ub418\uc5b4 \uc654\ub2e4. \uadf8\ub7f0 \uc810\uc5d0\uc11c \uc778\uac04\uc740 \ub300\uc0c1\ub4e4\uc744 \uc0dd\uac01\ud558\uba70 \uadf8 \ub300\uc0c1\ub4e4 \uc790\uccb4\uac00 \uc778\uac04\uc744 \uc678\uba74\ud654\ud558\ub294 \uac1c\ub150\uc744 \ub358\uc838\uc900\ub2e4. \uadf8\ub7ec\ubbc0\ub85c \ub9cc\uc57d \uc2e0\uc774 \uc120\ud558\ub2e4\uba74 \uc778\uac04\uc774\uc5b4\uc57c \ud558\ub294\ub370 \uc2e0\uc740 \ub300\uc0c1\uc774\ubbc0\ub85c \uc2e0\uc740 \ub2e8\uc9c0 \uc778\uac04\uc744 \uc678\uba74\ud654\ud55c \uac83\uc774\uae30 \ub54c\ubb38\uc774\ub2e4. \uadf8\ub7ec\ub098 \uc885\uad50\ub294 \uc778\uac04\uc774 \ubcf8\ub798\ubd80\ud130 \ubd80\ub3c4\ub355\ud558\ub2e4\uace0 \ud55c\ub2e4. \ud3ec\uc774\uc5b4\ubc14\ud750\ub294 \ub9cc\uc57d \"\ub9cc\uc57d \ub098\uc758 \uc815\uc2e0\uc774 \ubbf8\uc801\uc73c\ub85c \ud0c0\ub77d\ud55c \uc808\ub300\uc801\uc778 \uc874\uc7ac\ub77c\uba74 \ub0b4\uac00 \ud6cc\ub96d\ud55c \uadf8\ub9bc\uc758 \uc544\ub984\ub2e4\uc6c0\uc744 \uc778\uc2dd\ud560 \uc218 \uc788\uc744 \uac83\uc778\uac00?\"\ub77c\ub294 \uac00\ub2a5\uc131\uc744 \ubb3c\uc74c\uc73c\ub85c\uc368 \uc790\uc2e0\uc758 \ubaa8\uc21c\uc744 \uc904\uc774\ub824\uace0 \ud588\ub2e4. \ud3ec\uc774\uc5b4\ubc14\ud750\uc758 \ucd94\ub860\uc5d0 \ub530\ub974\uba74 \uc774\uac83\uc740 \ubd88\uac00\ub2a5\ud560 \uac83\uc774\ub098 \uac00\ub2a5\ud558\uace0, \ud6c4\uc5d0 \uc778\uac04\uc740 \uc544\ub984\ub2e4\uc6c0\uc744 \ubc1c\uacac\ud560 \ub2a5\ub825\uc774 \uc788\ub2e4\ub294 \uac83\uc744 \uae30\uc220\ud55c\ub2e4.", "sentence_answer": "\ud3ec\uc774\uc5b4\ubc14\ud750\ub294 \uc2e0 \uc774 \uc720\uc77c\ud558\uac8c \ud558\ub294 \uac83\uc740 \"\ub3c4\ub355\uc131\uacfc \uc778\uac04\uc758 \uc601\uc6d0\ud55c \uad6c\uc6d0\uc774\ub2e4.", "paragraph_id": "6602446-3-1", "paragraph_question": "question: \ud3ec\uc774\uc5b4\ubc14\ud750\uc758 \uc8fc\uc7a5\uc5d0 \uc758\ud558\uba74 \uc778\uac04\uc744 \uc678\uba74\ud654\ud55c \uac83\uc5d0 \ud574\ub2f9\ud558\ub294 \uac83\uc740 \ubb34\uc5c7\uc778\uac00?, context: 2\ubd80\uc5d0\uc11c\ub294 \uc774\uc804\uc758 \uc124\uba85\uc744 \ucca0\ud68c\ud558\ub294 \uac83\ucc98\ub7fc \ubcf4\uc774\ub294 \uac83\uc774 \uc694\uc810\uc774\ub2e4. \ud3ec\uc774\uc5b4\ubc14\ud750\ub294 \uc2e0\uc774 \uc720\uc77c\ud558\uac8c \ud558\ub294 \uac83\uc740 \"\ub3c4\ub355\uc131\uacfc \uc778\uac04\uc758 \uc601\uc6d0\ud55c \uad6c\uc6d0\uc774\ub2e4. \uadf8\ub7ec\ubbc0\ub85c \uc778\uac04\uc740 \uc0ac\uc2e4 \uc790\uae30 \uc790\uc2e0\uc774 \ubaa9\uc801\uc77c \ub530\ub984\uc774\ub2e4.\"\ub77c\uace0 \uc8fc\uc7a5\ud558\ub294\ub370, \uc778\uac04\uc758 \ud589\uc704\ub4e4\uc740 \uc2e0\uc5d0\uac8c \ub193\uc5ec\uc788\uae30 \ub54c\ubb38\uc774\ub2e4. \ud3ec\uc774\uc5b4\ubc14\ud750\ub294 \uc778\uac04\uc740 \uc790\uc2e0\uc758 \uac1c\uc778\uc131\uc744 \ud3ec\uae30\ud558\uace0 \uc774\ub97c \uc774\ubc88\uc5d0\ub294 \uc774\uae30\uc801\uc778 \uc874\uc7ac\uc778 \uc2e0\uc5d0\uac8c \ub9e1\uae34\ub2e4\uace0 \uc8fc\uc7a5\ud558\ub294\ub370, \uc5ec\uae30\uc11c \uc774\uc804\uc758 \uc790\uae30 \uc8fc\uc7a5\uacfc \ubaa8\uc21c\uc774 \uc0dd\uae34\ub2e4. \uc774\ub7ec\ud55c \uc2e0\uc758 \uc774\uae30\uc131\uc740 \uc778\uac04\uc744 \uc790\uadf9\ud558\uc5ec \uc778\uac04\uc774 \uc0ac\uc545\ud558\uace0 \ud0c0\ub77d\ud558\uac8c \ub418\ub3c4\ub85d \ud22c\uc0ac\ud558\ub294\ub370, \uc774\ub9ac\ud558\uc5ec \uc778\uac04\uc774 \"\uc120\ud560 \uc218\ub294 \uc5c6\uace0\", \"\uc120\ud55c \uc874\uc7ac\"\ub294 \uc624\uc9c1 \uc2e0\ubc16\uc5d0 \uc5c6\ub2e4. \uc774\ub7f0 \ubc29\ubc95\uc73c\ub85c \ud3ec\uc774\uc5b4\ubc14\ud750\ub294 \uc2e0\uc744 \uc22d\ubc30\ud568\uc73c\ub85c\uc368 \uc778\uac04\uc5d0\uac8c \uc77c\uc5b4\ub098\ub294 \ub2e8\uc808\uc744 \ubcf4\uc5ec\uc90c\uc73c\ub85c\uc368 \uc790\uc2e0\uc758 \ucd08\uae30 \uc8fc\uc7a5\ub4e4\uc744 \ub4a4\uc5ce\ub294\ub2e4. \ud3ec\uc774\uc5b4\ubc14\ud750\ub294 \"\uc2e0\uc73c\ub85c \uc758\uc778\ud654\ub41c\" \uc120\ud568\uc774 \uc2e0\uc744 \ub300\uc0c1\uc73c\ub85c \ub9cc\ub4e4\uc5c8\ub2e4\uace0 \ub2e8\uc5b8\ud558\ub294\ub370, \ub9cc\uc77c \uc2e0\uc774 \ub300\uc0c1\uc774 \uc544\ub2c8\ub77c\uba74 \uc2e0\uc744 \uc758\uc778\ud654\ud558\uae30 \uc704\ud574 \uc544\ubb34\uac83\ub3c4 \ud544\uc694\ud558\uc9c0 \uc54a\uae30 \ub54c\ubb38\uc774\ub2e4. \ub300\uc0c1\ub4e4\ub85c\uc11c\uc758 \uad00\uc810\uc740 \uc774\uc804\ubd80\ud130 \ub17c\uc758\ub418\uc5b4 \uc654\ub2e4. \uadf8\ub7f0 \uc810\uc5d0\uc11c \uc778\uac04\uc740 \ub300\uc0c1\ub4e4\uc744 \uc0dd\uac01\ud558\uba70 \uadf8 \ub300\uc0c1\ub4e4 \uc790\uccb4\uac00 \uc778\uac04\uc744 \uc678\uba74\ud654\ud558\ub294 \uac1c\ub150\uc744 \ub358\uc838\uc900\ub2e4. \uadf8\ub7ec\ubbc0\ub85c \ub9cc\uc57d \uc2e0\uc774 \uc120\ud558\ub2e4\uba74 \uc778\uac04\uc774\uc5b4\uc57c \ud558\ub294\ub370 \uc2e0\uc740 \ub300\uc0c1\uc774\ubbc0\ub85c \uc2e0\uc740 \ub2e8\uc9c0 \uc778\uac04\uc744 \uc678\uba74\ud654\ud55c \uac83\uc774\uae30 \ub54c\ubb38\uc774\ub2e4. \uadf8\ub7ec\ub098 \uc885\uad50\ub294 \uc778\uac04\uc774 \ubcf8\ub798\ubd80\ud130 \ubd80\ub3c4\ub355\ud558\ub2e4\uace0 \ud55c\ub2e4. \ud3ec\uc774\uc5b4\ubc14\ud750\ub294 \ub9cc\uc57d \"\ub9cc\uc57d \ub098\uc758 \uc815\uc2e0\uc774 \ubbf8\uc801\uc73c\ub85c \ud0c0\ub77d\ud55c \uc808\ub300\uc801\uc778 \uc874\uc7ac\ub77c\uba74 \ub0b4\uac00 \ud6cc\ub96d\ud55c \uadf8\ub9bc\uc758 \uc544\ub984\ub2e4\uc6c0\uc744 \uc778\uc2dd\ud560 \uc218 \uc788\uc744 \uac83\uc778\uac00?\"\ub77c\ub294 \uac00\ub2a5\uc131\uc744 \ubb3c\uc74c\uc73c\ub85c\uc368 \uc790\uc2e0\uc758 \ubaa8\uc21c\uc744 \uc904\uc774\ub824\uace0 \ud588\ub2e4. \ud3ec\uc774\uc5b4\ubc14\ud750\uc758 \ucd94\ub860\uc5d0 \ub530\ub974\uba74 \uc774\uac83\uc740 \ubd88\uac00\ub2a5\ud560 \uac83\uc774\ub098 \uac00\ub2a5\ud558\uace0, \ud6c4\uc5d0 \uc778\uac04\uc740 \uc544\ub984\ub2e4\uc6c0\uc744 \ubc1c\uacac\ud560 \ub2a5\ub825\uc774 \uc788\ub2e4\ub294 \uac83\uc744 \uae30\uc220\ud55c\ub2e4."} {"question": "\ubc38\ub7ec\ub9ac \uc194\ub77c\ub098\uc2a4\uac00 \uc5ec\uc131\uc8fc\uc758 \uc6b4\ub3d9\uacfc \uc18c\uc6d0\ud588\ub358 \ubaa8\uc21c\uc744 \uc9c0\uc801\ud55c \uc0ac\ub78c\uc740?", "paragraph": "\uadf8\ub7ec\ub098 \ube0c\ub80c \ud30c\uc2a4\ub294 \uc194\ub77c\ub098\uc2a4\uac00 \uc5ec\uc131\uc8fc\uc758 \uc6b4\ub3d9\uacfc \uc18c\uc6d0\ud588\ub358 \ubaa8\uc21c\uc744 \uc9c0\uc801\ud55c\ub2e4. \uc194\ub77c\ub098\uc2a4\ub294 \u201c\uc6b4\ub3d9\u201d\uc5d0 \ub3d9\ucc38\ud558\uace0\uc790 \ud588\ub358 \uc801\uc774 \ud55c \ubc88\ub3c4 \uc5c6\uc73c\uba70, \uc624\ud788\ub824 \uc790\uc2e0\uc5d0\uac8c \ub3d9\uc758\ud558\uc9c0 \uc54a\ub294 NOW \uc0ac\ub78c\ub4e4\uc744 \uc870\ub871\ud558\uba74\uc11c \uc6b4\ub3d9\uc744 \ubd84\uc5f4\uc2dc\ud0a4\ub824\uace0 \ud588\ub2e4. \uadf8\ub140\uc758 \uc0dd\ud65c\uc5d0\ub3c4 \ub9ce\uc740 \ubaa8\uc21c\uc774 \uc874\uc7ac\ud558\uba70(\uadf8\ub140\ub294 \ub808\uc988\ube44\uc5b8\uc774\uba74\uc11c \ub0a8\uc131\uc5d0\uac8c \ub9e4\ucd98\uc744 \ud588\uace0, \ub098\uc911\uc5d0\ub294 \ud63c\ub3d9 \ub05d\uc5d0 \uc2a4\uc2a4\ub85c \ubb34\uc131\uc560\uc790\ub77c\uace0 \uc8fc\uc7a5\ud588\ub2e4), \ud034\uc5b4 \ubb38\ud654\ub97c \uac70\ubd80\ud588\uace0, \ub0a8\ub4e4\uc5d0\uac8c \uc758\uc874\ud558\ub294 \uc0b6\uc744 \uc0b4\uba74\uc11c \ub0a8\ub4e4\uc5d0\uac8c \uad00\uc2ec\uc774 \uc5c6\uc5c8\ub2e4. \ub610\ud55c \ud30c\uc2a4\ub294 \uc194\ub77c\ub098\uc2a4\uc758 \uc0b6\uc5d0 \uad00\ud55c \uc5ec\ub7ec \uc774\uc57c\uae30\ub4e4\uc758 \ubaa8\uc21c\ub3c4 \uc9c0\uc801\ud55c\ub2e4. \ud754\ud788 \uc194\ub77c\ub098\uc2a4\ub294 \ud53c\ud574\uc790, \ubc18\ub780\uc790, \uc808\ub9dd\ud55c \uace0\ub3c5\uc790\ub85c \ubb18\uc0ac\ub41c\ub2e4. \uadf8\ub7ec\ub098 \uc194\ub77c\ub098\uc2a4\uc758 \uc0ac\ucd0c\uc740 \uadf8\ub140\uac00 20\ub300 \ub9d0\uc5d0\uc11c 30\ub300\uae4c\uc9c0\ub294 \uc6e8\uc774\ud2b8\ub9ac\uc2a4\ub85c \uc77c\ud588\uace0 \ub9e4\ucd98\uc740 \ud558\uc9c0 \uc54a\uc558\ub2e4 \ud558\uba70, \uce5c\uad6c \uc81c\ud504\ub9ac \ub77c\uae30\uc5b4\ub294 \uadf8\ub140\uac00 \u201c\uba4b\uc9c4 \uc720\ub144\uae30\u201d\ub97c \ubcf4\ub0c8\ub2e4\uace0 \ub9d0\ud55c\ub2e4. \ub610 \uc194\ub77c\ub098\uc2a4\ub294 \uc0dd\uc560 \ub0b4\ub0b4 \uc544\ubc84\uc9c0\uc640 \uc5f0\ub77d\uc744 \uc720\uc9c0\ud588\uc73c\uba70, \uc774\ub294 \uc194\ub77c\ub098\uc2a4\uac00 \uc544\ubc84\uc9c0\ub97c \uc131\ud3ed\ud589\ubc94\uc774\ub77c\uace0 \uc9c0\ubaa9\ud558\uace0 \ube44\ub09c\ud588\uace0 \uc120\uc5b8\uc11c\uc5d0\uc11c \uc99d\uc624\ub97c \ud45c\ucd9c\ud588\ub358 \uac83\uc5d0 \ub300\ud574 \ubcf5\uc7a1\ud55c \uc758\ubb38\uc744 \ubd88\ub7ec\uc77c\uc73c\ud0a8\ub2e4. \ud30c\uc2a4\ub294 \uc194\ub77c\ub098\uc2a4\uac00 \uc774\ub7ec\ud55c \ubaa8\uc21c\ub4e4 \uc790\uccb4\ub97c \uc790\uc2e0\uc758 \uc815\uccb4\uc131 \uc77c\ubd80\ub85c \ud3ec\uc6a9\ud588\ub2e4\uace0 \uc0dd\uac01\ud55c\ub2e4.", "answer": "\ube0c\ub80c \ud30c\uc2a4", "sentence": "\uadf8\ub7ec\ub098 \ube0c\ub80c \ud30c\uc2a4 \ub294 \uc194\ub77c\ub098\uc2a4\uac00 \uc5ec\uc131\uc8fc\uc758 \uc6b4\ub3d9\uacfc \uc18c\uc6d0\ud588\ub358 \ubaa8\uc21c\uc744 \uc9c0\uc801\ud55c\ub2e4.", "paragraph_sentence": " \uadf8\ub7ec\ub098 \ube0c\ub80c \ud30c\uc2a4 \ub294 \uc194\ub77c\ub098\uc2a4\uac00 \uc5ec\uc131\uc8fc\uc758 \uc6b4\ub3d9\uacfc \uc18c\uc6d0\ud588\ub358 \ubaa8\uc21c\uc744 \uc9c0\uc801\ud55c\ub2e4. \uc194\ub77c\ub098\uc2a4\ub294 \u201c\uc6b4\ub3d9\u201d\uc5d0 \ub3d9\ucc38\ud558\uace0\uc790 \ud588\ub358 \uc801\uc774 \ud55c \ubc88\ub3c4 \uc5c6\uc73c\uba70, \uc624\ud788\ub824 \uc790\uc2e0\uc5d0\uac8c \ub3d9\uc758\ud558\uc9c0 \uc54a\ub294 NOW \uc0ac\ub78c\ub4e4\uc744 \uc870\ub871\ud558\uba74\uc11c \uc6b4\ub3d9\uc744 \ubd84\uc5f4\uc2dc\ud0a4\ub824\uace0 \ud588\ub2e4. \uadf8\ub140\uc758 \uc0dd\ud65c\uc5d0\ub3c4 \ub9ce\uc740 \ubaa8\uc21c\uc774 \uc874\uc7ac\ud558\uba70(\uadf8\ub140\ub294 \ub808\uc988\ube44\uc5b8\uc774\uba74\uc11c \ub0a8\uc131\uc5d0\uac8c \ub9e4\ucd98\uc744 \ud588\uace0, \ub098\uc911\uc5d0\ub294 \ud63c\ub3d9 \ub05d\uc5d0 \uc2a4\uc2a4\ub85c \ubb34\uc131\uc560\uc790\ub77c\uace0 \uc8fc\uc7a5\ud588\ub2e4), \ud034\uc5b4 \ubb38\ud654\ub97c \uac70\ubd80\ud588\uace0, \ub0a8\ub4e4\uc5d0\uac8c \uc758\uc874\ud558\ub294 \uc0b6\uc744 \uc0b4\uba74\uc11c \ub0a8\ub4e4\uc5d0\uac8c \uad00\uc2ec\uc774 \uc5c6\uc5c8\ub2e4. \ub610\ud55c \ud30c\uc2a4\ub294 \uc194\ub77c\ub098\uc2a4\uc758 \uc0b6\uc5d0 \uad00\ud55c \uc5ec\ub7ec \uc774\uc57c\uae30\ub4e4\uc758 \ubaa8\uc21c\ub3c4 \uc9c0\uc801\ud55c\ub2e4. \ud754\ud788 \uc194\ub77c\ub098\uc2a4\ub294 \ud53c\ud574\uc790, \ubc18\ub780\uc790, \uc808\ub9dd\ud55c \uace0\ub3c5\uc790\ub85c \ubb18\uc0ac\ub41c\ub2e4. \uadf8\ub7ec\ub098 \uc194\ub77c\ub098\uc2a4\uc758 \uc0ac\ucd0c\uc740 \uadf8\ub140\uac00 20\ub300 \ub9d0\uc5d0\uc11c 30\ub300\uae4c\uc9c0\ub294 \uc6e8\uc774\ud2b8\ub9ac\uc2a4\ub85c \uc77c\ud588\uace0 \ub9e4\ucd98\uc740 \ud558\uc9c0 \uc54a\uc558\ub2e4 \ud558\uba70, \uce5c\uad6c \uc81c\ud504\ub9ac \ub77c\uae30\uc5b4\ub294 \uadf8\ub140\uac00 \u201c\uba4b\uc9c4 \uc720\ub144\uae30\u201d\ub97c \ubcf4\ub0c8\ub2e4\uace0 \ub9d0\ud55c\ub2e4. \ub610 \uc194\ub77c\ub098\uc2a4\ub294 \uc0dd\uc560 \ub0b4\ub0b4 \uc544\ubc84\uc9c0\uc640 \uc5f0\ub77d\uc744 \uc720\uc9c0\ud588\uc73c\uba70, \uc774\ub294 \uc194\ub77c\ub098\uc2a4\uac00 \uc544\ubc84\uc9c0\ub97c \uc131\ud3ed\ud589\ubc94\uc774\ub77c\uace0 \uc9c0\ubaa9\ud558\uace0 \ube44\ub09c\ud588\uace0 \uc120\uc5b8\uc11c\uc5d0\uc11c \uc99d\uc624\ub97c \ud45c\ucd9c\ud588\ub358 \uac83\uc5d0 \ub300\ud574 \ubcf5\uc7a1\ud55c \uc758\ubb38\uc744 \ubd88\ub7ec\uc77c\uc73c\ud0a8\ub2e4. \ud30c\uc2a4\ub294 \uc194\ub77c\ub098\uc2a4\uac00 \uc774\ub7ec\ud55c \ubaa8\uc21c\ub4e4 \uc790\uccb4\ub97c \uc790\uc2e0\uc758 \uc815\uccb4\uc131 \uc77c\ubd80\ub85c \ud3ec\uc6a9\ud588\ub2e4\uace0 \uc0dd\uac01\ud55c\ub2e4.", "paragraph_answer": "\uadf8\ub7ec\ub098 \ube0c\ub80c \ud30c\uc2a4 \ub294 \uc194\ub77c\ub098\uc2a4\uac00 \uc5ec\uc131\uc8fc\uc758 \uc6b4\ub3d9\uacfc \uc18c\uc6d0\ud588\ub358 \ubaa8\uc21c\uc744 \uc9c0\uc801\ud55c\ub2e4. \uc194\ub77c\ub098\uc2a4\ub294 \u201c\uc6b4\ub3d9\u201d\uc5d0 \ub3d9\ucc38\ud558\uace0\uc790 \ud588\ub358 \uc801\uc774 \ud55c \ubc88\ub3c4 \uc5c6\uc73c\uba70, \uc624\ud788\ub824 \uc790\uc2e0\uc5d0\uac8c \ub3d9\uc758\ud558\uc9c0 \uc54a\ub294 NOW \uc0ac\ub78c\ub4e4\uc744 \uc870\ub871\ud558\uba74\uc11c \uc6b4\ub3d9\uc744 \ubd84\uc5f4\uc2dc\ud0a4\ub824\uace0 \ud588\ub2e4. \uadf8\ub140\uc758 \uc0dd\ud65c\uc5d0\ub3c4 \ub9ce\uc740 \ubaa8\uc21c\uc774 \uc874\uc7ac\ud558\uba70(\uadf8\ub140\ub294 \ub808\uc988\ube44\uc5b8\uc774\uba74\uc11c \ub0a8\uc131\uc5d0\uac8c \ub9e4\ucd98\uc744 \ud588\uace0, \ub098\uc911\uc5d0\ub294 \ud63c\ub3d9 \ub05d\uc5d0 \uc2a4\uc2a4\ub85c \ubb34\uc131\uc560\uc790\ub77c\uace0 \uc8fc\uc7a5\ud588\ub2e4), \ud034\uc5b4 \ubb38\ud654\ub97c \uac70\ubd80\ud588\uace0, \ub0a8\ub4e4\uc5d0\uac8c \uc758\uc874\ud558\ub294 \uc0b6\uc744 \uc0b4\uba74\uc11c \ub0a8\ub4e4\uc5d0\uac8c \uad00\uc2ec\uc774 \uc5c6\uc5c8\ub2e4. \ub610\ud55c \ud30c\uc2a4\ub294 \uc194\ub77c\ub098\uc2a4\uc758 \uc0b6\uc5d0 \uad00\ud55c \uc5ec\ub7ec \uc774\uc57c\uae30\ub4e4\uc758 \ubaa8\uc21c\ub3c4 \uc9c0\uc801\ud55c\ub2e4. \ud754\ud788 \uc194\ub77c\ub098\uc2a4\ub294 \ud53c\ud574\uc790, \ubc18\ub780\uc790, \uc808\ub9dd\ud55c \uace0\ub3c5\uc790\ub85c \ubb18\uc0ac\ub41c\ub2e4. \uadf8\ub7ec\ub098 \uc194\ub77c\ub098\uc2a4\uc758 \uc0ac\ucd0c\uc740 \uadf8\ub140\uac00 20\ub300 \ub9d0\uc5d0\uc11c 30\ub300\uae4c\uc9c0\ub294 \uc6e8\uc774\ud2b8\ub9ac\uc2a4\ub85c \uc77c\ud588\uace0 \ub9e4\ucd98\uc740 \ud558\uc9c0 \uc54a\uc558\ub2e4 \ud558\uba70, \uce5c\uad6c \uc81c\ud504\ub9ac \ub77c\uae30\uc5b4\ub294 \uadf8\ub140\uac00 \u201c\uba4b\uc9c4 \uc720\ub144\uae30\u201d\ub97c \ubcf4\ub0c8\ub2e4\uace0 \ub9d0\ud55c\ub2e4. \ub610 \uc194\ub77c\ub098\uc2a4\ub294 \uc0dd\uc560 \ub0b4\ub0b4 \uc544\ubc84\uc9c0\uc640 \uc5f0\ub77d\uc744 \uc720\uc9c0\ud588\uc73c\uba70, \uc774\ub294 \uc194\ub77c\ub098\uc2a4\uac00 \uc544\ubc84\uc9c0\ub97c \uc131\ud3ed\ud589\ubc94\uc774\ub77c\uace0 \uc9c0\ubaa9\ud558\uace0 \ube44\ub09c\ud588\uace0 \uc120\uc5b8\uc11c\uc5d0\uc11c \uc99d\uc624\ub97c \ud45c\ucd9c\ud588\ub358 \uac83\uc5d0 \ub300\ud574 \ubcf5\uc7a1\ud55c \uc758\ubb38\uc744 \ubd88\ub7ec\uc77c\uc73c\ud0a8\ub2e4. \ud30c\uc2a4\ub294 \uc194\ub77c\ub098\uc2a4\uac00 \uc774\ub7ec\ud55c \ubaa8\uc21c\ub4e4 \uc790\uccb4\ub97c \uc790\uc2e0\uc758 \uc815\uccb4\uc131 \uc77c\ubd80\ub85c \ud3ec\uc6a9\ud588\ub2e4\uace0 \uc0dd\uac01\ud55c\ub2e4.", "sentence_answer": "\uadf8\ub7ec\ub098 \ube0c\ub80c \ud30c\uc2a4 \ub294 \uc194\ub77c\ub098\uc2a4\uac00 \uc5ec\uc131\uc8fc\uc758 \uc6b4\ub3d9\uacfc \uc18c\uc6d0\ud588\ub358 \ubaa8\uc21c\uc744 \uc9c0\uc801\ud55c\ub2e4.", "paragraph_id": "6535070-14-0", "paragraph_question": "question: \ubc38\ub7ec\ub9ac \uc194\ub77c\ub098\uc2a4\uac00 \uc5ec\uc131\uc8fc\uc758 \uc6b4\ub3d9\uacfc \uc18c\uc6d0\ud588\ub358 \ubaa8\uc21c\uc744 \uc9c0\uc801\ud55c \uc0ac\ub78c\uc740?, context: \uadf8\ub7ec\ub098 \ube0c\ub80c \ud30c\uc2a4\ub294 \uc194\ub77c\ub098\uc2a4\uac00 \uc5ec\uc131\uc8fc\uc758 \uc6b4\ub3d9\uacfc \uc18c\uc6d0\ud588\ub358 \ubaa8\uc21c\uc744 \uc9c0\uc801\ud55c\ub2e4. \uc194\ub77c\ub098\uc2a4\ub294 \u201c\uc6b4\ub3d9\u201d\uc5d0 \ub3d9\ucc38\ud558\uace0\uc790 \ud588\ub358 \uc801\uc774 \ud55c \ubc88\ub3c4 \uc5c6\uc73c\uba70, \uc624\ud788\ub824 \uc790\uc2e0\uc5d0\uac8c \ub3d9\uc758\ud558\uc9c0 \uc54a\ub294 NOW \uc0ac\ub78c\ub4e4\uc744 \uc870\ub871\ud558\uba74\uc11c \uc6b4\ub3d9\uc744 \ubd84\uc5f4\uc2dc\ud0a4\ub824\uace0 \ud588\ub2e4. \uadf8\ub140\uc758 \uc0dd\ud65c\uc5d0\ub3c4 \ub9ce\uc740 \ubaa8\uc21c\uc774 \uc874\uc7ac\ud558\uba70(\uadf8\ub140\ub294 \ub808\uc988\ube44\uc5b8\uc774\uba74\uc11c \ub0a8\uc131\uc5d0\uac8c \ub9e4\ucd98\uc744 \ud588\uace0, \ub098\uc911\uc5d0\ub294 \ud63c\ub3d9 \ub05d\uc5d0 \uc2a4\uc2a4\ub85c \ubb34\uc131\uc560\uc790\ub77c\uace0 \uc8fc\uc7a5\ud588\ub2e4), \ud034\uc5b4 \ubb38\ud654\ub97c \uac70\ubd80\ud588\uace0, \ub0a8\ub4e4\uc5d0\uac8c \uc758\uc874\ud558\ub294 \uc0b6\uc744 \uc0b4\uba74\uc11c \ub0a8\ub4e4\uc5d0\uac8c \uad00\uc2ec\uc774 \uc5c6\uc5c8\ub2e4. \ub610\ud55c \ud30c\uc2a4\ub294 \uc194\ub77c\ub098\uc2a4\uc758 \uc0b6\uc5d0 \uad00\ud55c \uc5ec\ub7ec \uc774\uc57c\uae30\ub4e4\uc758 \ubaa8\uc21c\ub3c4 \uc9c0\uc801\ud55c\ub2e4. \ud754\ud788 \uc194\ub77c\ub098\uc2a4\ub294 \ud53c\ud574\uc790, \ubc18\ub780\uc790, \uc808\ub9dd\ud55c \uace0\ub3c5\uc790\ub85c \ubb18\uc0ac\ub41c\ub2e4. \uadf8\ub7ec\ub098 \uc194\ub77c\ub098\uc2a4\uc758 \uc0ac\ucd0c\uc740 \uadf8\ub140\uac00 20\ub300 \ub9d0\uc5d0\uc11c 30\ub300\uae4c\uc9c0\ub294 \uc6e8\uc774\ud2b8\ub9ac\uc2a4\ub85c \uc77c\ud588\uace0 \ub9e4\ucd98\uc740 \ud558\uc9c0 \uc54a\uc558\ub2e4 \ud558\uba70, \uce5c\uad6c \uc81c\ud504\ub9ac \ub77c\uae30\uc5b4\ub294 \uadf8\ub140\uac00 \u201c\uba4b\uc9c4 \uc720\ub144\uae30\u201d\ub97c \ubcf4\ub0c8\ub2e4\uace0 \ub9d0\ud55c\ub2e4. \ub610 \uc194\ub77c\ub098\uc2a4\ub294 \uc0dd\uc560 \ub0b4\ub0b4 \uc544\ubc84\uc9c0\uc640 \uc5f0\ub77d\uc744 \uc720\uc9c0\ud588\uc73c\uba70, \uc774\ub294 \uc194\ub77c\ub098\uc2a4\uac00 \uc544\ubc84\uc9c0\ub97c \uc131\ud3ed\ud589\ubc94\uc774\ub77c\uace0 \uc9c0\ubaa9\ud558\uace0 \ube44\ub09c\ud588\uace0 \uc120\uc5b8\uc11c\uc5d0\uc11c \uc99d\uc624\ub97c \ud45c\ucd9c\ud588\ub358 \uac83\uc5d0 \ub300\ud574 \ubcf5\uc7a1\ud55c \uc758\ubb38\uc744 \ubd88\ub7ec\uc77c\uc73c\ud0a8\ub2e4. \ud30c\uc2a4\ub294 \uc194\ub77c\ub098\uc2a4\uac00 \uc774\ub7ec\ud55c \ubaa8\uc21c\ub4e4 \uc790\uccb4\ub97c \uc790\uc2e0\uc758 \uc815\uccb4\uc131 \uc77c\ubd80\ub85c \ud3ec\uc6a9\ud588\ub2e4\uace0 \uc0dd\uac01\ud55c\ub2e4."} {"question": "\ub124\ub4dc\uac00 \uac00\uc785\ud55c \ub178\ub3d9\uc790\ub4e4\uc758 \uc0ac\uad50 \ud074\ub7fd\uc740?", "paragraph": "\ub124\ub4dc\ub294 \ud558\uce35\ubbfc \ub0a8\uc790 \ub178\ub3d9\uc790\ub4e4\uc758 \uc0ac\uad50 \ud074\ub7fd\uc778 \ubcfc\ub9c1 \ud300\uc5d0 \ub4e4\uc5b4\uac04\ub2e4. \uadf8\uacf3\uc5d0\ub294 \uc9d0, \uc568\ub7f0, \ubc25\uc774 \uc788\uc5c8\ub2e4. \uc9d0\uc740 \ud138\ud138\ud558\uace0 \uc0b4\uac00\uc6e0\uace0, \uc568\ub7f0\uc740 \ucfe8\ud55c \ud3b8\uc774\uc5c8\ub2e4. \ubc25\uc740 \uc880 \ubb34\ub69d\ub69d\ud588\ub2e4. \ub124\ub4dc\ub294 \uc774\uacf3\uc5d0\uc11c \ub0a8\uc790\ub4e4\ub9cc\uc758 \uc6b0\uc815 \ub9fa\ub294 \ubc29\uc2dc\uacfc \uc5f0\ub300\uc758\uc2dd\uc744 \ubc30\uc6e0\ub2e4. \uadf8\ub4e4\uc740 \ub9e4\uc0ac\uc5d0 \uc194\uc9c1\ud588\uace0, \uc0ac\ub78c\uc744 \ud3b8\uacac \uc5c6\uc774 \uc624\ub85c\uc9c0 \ub2a5\ub825\ub9cc\uc73c\ub85c \ud3c9\uac00\ud588\ub2e4. \uc774\ub530\uae08 \ucc28\ubcc4\uc801\uc778 \ub18d\ub2f4\uc744 \ud558\uc9c0\ub9cc \uc5b4\ub514\uae4c\uc9c0\ub098 \ud765\ubbf8 \ubcf8\uc704\uc600\ub2e4. \uc544\ub0b4\uac00 \uc788\uc5c8\uc9c0\ub9cc \uc2a4\ud2b8\ub9bd \ud074\ub7fd\uc5d0 \ub2e4\ub2c8\uba70 \uc131\uc695\uc744 \ubc1c\uc0b0\ud588\uace0 \uadf8\uac83\uc744 \ubd80\ub044\ub7ec\uc6cc \ud558\uc9c0 \uc54a\uc558\ub2e4. \uc624\ud788\ub824 \uc544\ub0b4\uc640 \uc790\uc2dd\ub4e4\uc744 \uc544\uc8fc \uc544\uaf08\ub2e4. \uc139\uc2a4 \ub18d\ub2f4\uacfc \ub2f4\ubc30, \uc220\uc740 \uc77c\uc0c1\uc801\uc778 \uc720\ud765\uac70\ub9ac\uc600\ub2e4. \ub0a8\uc790\ub4e4\ub07c\ub9ac\ub294 \uadf8\ub4e4 \uc0ac\uc774\uc5d0\uc11c\ub9cc \ud1b5\ud558\ub294 \ubcf4\ud3b8\uc801\uc778 \uc720\ub300\uac10\uacfc \uc874\uc911\uc758\uc2dd\uc774 \uc788\uc5c8\ub2e4. \ud55c\ud3b8\uc73c\ub85c \uc544\ubb34\uc5d0\uac8c\ub098 \uc27d\uac8c \ub9c8\uc74c\uc744 \uc5f4\uc5b4\uc8fc\uc9c0\ub294 \uc54a\uc558\ub2e4.", "answer": "\ubcfc\ub9c1 \ud300", "sentence": "\ub124\ub4dc\ub294 \ud558\uce35\ubbfc \ub0a8\uc790 \ub178\ub3d9\uc790\ub4e4\uc758 \uc0ac\uad50 \ud074\ub7fd\uc778 \ubcfc\ub9c1 \ud300 \uc5d0 \ub4e4\uc5b4\uac04\ub2e4.", "paragraph_sentence": " \ub124\ub4dc\ub294 \ud558\uce35\ubbfc \ub0a8\uc790 \ub178\ub3d9\uc790\ub4e4\uc758 \uc0ac\uad50 \ud074\ub7fd\uc778 \ubcfc\ub9c1 \ud300 \uc5d0 \ub4e4\uc5b4\uac04\ub2e4. \uadf8\uacf3\uc5d0\ub294 \uc9d0, \uc568\ub7f0, \ubc25\uc774 \uc788\uc5c8\ub2e4. \uc9d0\uc740 \ud138\ud138\ud558\uace0 \uc0b4\uac00\uc6e0\uace0, \uc568\ub7f0\uc740 \ucfe8\ud55c \ud3b8\uc774\uc5c8\ub2e4. \ubc25\uc740 \uc880 \ubb34\ub69d\ub69d\ud588\ub2e4. \ub124\ub4dc\ub294 \uc774\uacf3\uc5d0\uc11c \ub0a8\uc790\ub4e4\ub9cc\uc758 \uc6b0\uc815 \ub9fa\ub294 \ubc29\uc2dc\uacfc \uc5f0\ub300\uc758\uc2dd\uc744 \ubc30\uc6e0\ub2e4. \uadf8\ub4e4\uc740 \ub9e4\uc0ac\uc5d0 \uc194\uc9c1\ud588\uace0, \uc0ac\ub78c\uc744 \ud3b8\uacac \uc5c6\uc774 \uc624\ub85c\uc9c0 \ub2a5\ub825\ub9cc\uc73c\ub85c \ud3c9\uac00\ud588\ub2e4. \uc774\ub530\uae08 \ucc28\ubcc4\uc801\uc778 \ub18d\ub2f4\uc744 \ud558\uc9c0\ub9cc \uc5b4\ub514\uae4c\uc9c0\ub098 \ud765\ubbf8 \ubcf8\uc704\uc600\ub2e4. \uc544\ub0b4\uac00 \uc788\uc5c8\uc9c0\ub9cc \uc2a4\ud2b8\ub9bd \ud074\ub7fd\uc5d0 \ub2e4\ub2c8\uba70 \uc131\uc695\uc744 \ubc1c\uc0b0\ud588\uace0 \uadf8\uac83\uc744 \ubd80\ub044\ub7ec\uc6cc \ud558\uc9c0 \uc54a\uc558\ub2e4. \uc624\ud788\ub824 \uc544\ub0b4\uc640 \uc790\uc2dd\ub4e4\uc744 \uc544\uc8fc \uc544\uaf08\ub2e4. \uc139\uc2a4 \ub18d\ub2f4\uacfc \ub2f4\ubc30, \uc220\uc740 \uc77c\uc0c1\uc801\uc778 \uc720\ud765\uac70\ub9ac\uc600\ub2e4. \ub0a8\uc790\ub4e4\ub07c\ub9ac\ub294 \uadf8\ub4e4 \uc0ac\uc774\uc5d0\uc11c\ub9cc \ud1b5\ud558\ub294 \ubcf4\ud3b8\uc801\uc778 \uc720\ub300\uac10\uacfc \uc874\uc911\uc758\uc2dd\uc774 \uc788\uc5c8\ub2e4. \ud55c\ud3b8\uc73c\ub85c \uc544\ubb34\uc5d0\uac8c\ub098 \uc27d\uac8c \ub9c8\uc74c\uc744 \uc5f4\uc5b4\uc8fc\uc9c0\ub294 \uc54a\uc558\ub2e4.", "paragraph_answer": "\ub124\ub4dc\ub294 \ud558\uce35\ubbfc \ub0a8\uc790 \ub178\ub3d9\uc790\ub4e4\uc758 \uc0ac\uad50 \ud074\ub7fd\uc778 \ubcfc\ub9c1 \ud300 \uc5d0 \ub4e4\uc5b4\uac04\ub2e4. \uadf8\uacf3\uc5d0\ub294 \uc9d0, \uc568\ub7f0, \ubc25\uc774 \uc788\uc5c8\ub2e4. \uc9d0\uc740 \ud138\ud138\ud558\uace0 \uc0b4\uac00\uc6e0\uace0, \uc568\ub7f0\uc740 \ucfe8\ud55c \ud3b8\uc774\uc5c8\ub2e4. \ubc25\uc740 \uc880 \ubb34\ub69d\ub69d\ud588\ub2e4. \ub124\ub4dc\ub294 \uc774\uacf3\uc5d0\uc11c \ub0a8\uc790\ub4e4\ub9cc\uc758 \uc6b0\uc815 \ub9fa\ub294 \ubc29\uc2dc\uacfc \uc5f0\ub300\uc758\uc2dd\uc744 \ubc30\uc6e0\ub2e4. \uadf8\ub4e4\uc740 \ub9e4\uc0ac\uc5d0 \uc194\uc9c1\ud588\uace0, \uc0ac\ub78c\uc744 \ud3b8\uacac \uc5c6\uc774 \uc624\ub85c\uc9c0 \ub2a5\ub825\ub9cc\uc73c\ub85c \ud3c9\uac00\ud588\ub2e4. \uc774\ub530\uae08 \ucc28\ubcc4\uc801\uc778 \ub18d\ub2f4\uc744 \ud558\uc9c0\ub9cc \uc5b4\ub514\uae4c\uc9c0\ub098 \ud765\ubbf8 \ubcf8\uc704\uc600\ub2e4. \uc544\ub0b4\uac00 \uc788\uc5c8\uc9c0\ub9cc \uc2a4\ud2b8\ub9bd \ud074\ub7fd\uc5d0 \ub2e4\ub2c8\uba70 \uc131\uc695\uc744 \ubc1c\uc0b0\ud588\uace0 \uadf8\uac83\uc744 \ubd80\ub044\ub7ec\uc6cc \ud558\uc9c0 \uc54a\uc558\ub2e4. \uc624\ud788\ub824 \uc544\ub0b4\uc640 \uc790\uc2dd\ub4e4\uc744 \uc544\uc8fc \uc544\uaf08\ub2e4. \uc139\uc2a4 \ub18d\ub2f4\uacfc \ub2f4\ubc30, \uc220\uc740 \uc77c\uc0c1\uc801\uc778 \uc720\ud765\uac70\ub9ac\uc600\ub2e4. \ub0a8\uc790\ub4e4\ub07c\ub9ac\ub294 \uadf8\ub4e4 \uc0ac\uc774\uc5d0\uc11c\ub9cc \ud1b5\ud558\ub294 \ubcf4\ud3b8\uc801\uc778 \uc720\ub300\uac10\uacfc \uc874\uc911\uc758\uc2dd\uc774 \uc788\uc5c8\ub2e4. \ud55c\ud3b8\uc73c\ub85c \uc544\ubb34\uc5d0\uac8c\ub098 \uc27d\uac8c \ub9c8\uc74c\uc744 \uc5f4\uc5b4\uc8fc\uc9c0\ub294 \uc54a\uc558\ub2e4.", "sentence_answer": "\ub124\ub4dc\ub294 \ud558\uce35\ubbfc \ub0a8\uc790 \ub178\ub3d9\uc790\ub4e4\uc758 \uc0ac\uad50 \ud074\ub7fd\uc778 \ubcfc\ub9c1 \ud300 \uc5d0 \ub4e4\uc5b4\uac04\ub2e4.", "paragraph_id": "6470925-0-0", "paragraph_question": "question: \ub124\ub4dc\uac00 \uac00\uc785\ud55c \ub178\ub3d9\uc790\ub4e4\uc758 \uc0ac\uad50 \ud074\ub7fd\uc740?, context: \ub124\ub4dc\ub294 \ud558\uce35\ubbfc \ub0a8\uc790 \ub178\ub3d9\uc790\ub4e4\uc758 \uc0ac\uad50 \ud074\ub7fd\uc778 \ubcfc\ub9c1 \ud300\uc5d0 \ub4e4\uc5b4\uac04\ub2e4. \uadf8\uacf3\uc5d0\ub294 \uc9d0, \uc568\ub7f0, \ubc25\uc774 \uc788\uc5c8\ub2e4. \uc9d0\uc740 \ud138\ud138\ud558\uace0 \uc0b4\uac00\uc6e0\uace0, \uc568\ub7f0\uc740 \ucfe8\ud55c \ud3b8\uc774\uc5c8\ub2e4. \ubc25\uc740 \uc880 \ubb34\ub69d\ub69d\ud588\ub2e4. \ub124\ub4dc\ub294 \uc774\uacf3\uc5d0\uc11c \ub0a8\uc790\ub4e4\ub9cc\uc758 \uc6b0\uc815 \ub9fa\ub294 \ubc29\uc2dc\uacfc \uc5f0\ub300\uc758\uc2dd\uc744 \ubc30\uc6e0\ub2e4. \uadf8\ub4e4\uc740 \ub9e4\uc0ac\uc5d0 \uc194\uc9c1\ud588\uace0, \uc0ac\ub78c\uc744 \ud3b8\uacac \uc5c6\uc774 \uc624\ub85c\uc9c0 \ub2a5\ub825\ub9cc\uc73c\ub85c \ud3c9\uac00\ud588\ub2e4. \uc774\ub530\uae08 \ucc28\ubcc4\uc801\uc778 \ub18d\ub2f4\uc744 \ud558\uc9c0\ub9cc \uc5b4\ub514\uae4c\uc9c0\ub098 \ud765\ubbf8 \ubcf8\uc704\uc600\ub2e4. \uc544\ub0b4\uac00 \uc788\uc5c8\uc9c0\ub9cc \uc2a4\ud2b8\ub9bd \ud074\ub7fd\uc5d0 \ub2e4\ub2c8\uba70 \uc131\uc695\uc744 \ubc1c\uc0b0\ud588\uace0 \uadf8\uac83\uc744 \ubd80\ub044\ub7ec\uc6cc \ud558\uc9c0 \uc54a\uc558\ub2e4. \uc624\ud788\ub824 \uc544\ub0b4\uc640 \uc790\uc2dd\ub4e4\uc744 \uc544\uc8fc \uc544\uaf08\ub2e4. \uc139\uc2a4 \ub18d\ub2f4\uacfc \ub2f4\ubc30, \uc220\uc740 \uc77c\uc0c1\uc801\uc778 \uc720\ud765\uac70\ub9ac\uc600\ub2e4. \ub0a8\uc790\ub4e4\ub07c\ub9ac\ub294 \uadf8\ub4e4 \uc0ac\uc774\uc5d0\uc11c\ub9cc \ud1b5\ud558\ub294 \ubcf4\ud3b8\uc801\uc778 \uc720\ub300\uac10\uacfc \uc874\uc911\uc758\uc2dd\uc774 \uc788\uc5c8\ub2e4. \ud55c\ud3b8\uc73c\ub85c \uc544\ubb34\uc5d0\uac8c\ub098 \uc27d\uac8c \ub9c8\uc74c\uc744 \uc5f4\uc5b4\uc8fc\uc9c0\ub294 \uc54a\uc558\ub2e4."} {"question": "\ud29c\ub294 \ubaa8\ubc29\ud558\ub294 \uc790\ub4e4\uc5d0\uac8c \ubb34\uc5c7\uc744 \ube48\ub2e4\uace0 \ud558\uc600\ub294\uac00?", "paragraph": "\uc774 \ubc16\uc5d0\ub3c4 \ud654\uc18c \ub2e8\uc704\ub85c \uad11\uace0\ub97c \ud310\ub9e4\ud558\ub294 \ub2e4\ub978 \uc6f9\uc0ac\uc774\ud2b8\ub4e4\uc740 \uc218\uc5c6\uc774 \ub9ce\ub2e4. \ud29c\ub294 \uc774 \uc0ac\uc774\ud2b8\ub4e4\uc744 \ubcf4\uace0 \"(\uc774\ub4e4\uc740) \uac70\uc758 \ubc14\ub85c \ubc14\ub85c \ub739\ub2c8\ub2e4. \uc9c0\uae08\uc740 \ud654\uc18c\ub97c \ud310\ub9e4\ud558\ub294 \uc6f9 \uc0ac\uc774\ud2b8\ub4e4\uc774 \uc218\ubc31 \uacf3 \uc788\uc2b5\ub2c8\ub2e4. \ubaa8\ubc29\uc744 \ud558\ub294 \uacf3\ub4e4\uc740 \ubaa8\ub450 \ub2e4\ub978 \uacf3\uacfc \uacbd\uc7c1\uc744 \ud558\uace0 \uc788\uc2b5\ub2c8\ub2e4.\"\ub77c\uace0 \ub9d0\uc744 \ub358\uc84c\ub2e4. \ub610, \"\uc774\ub7ec\ud55c \uac83\ub4e4\uc740 \uad11\uace0\uac00 \uac70\uc758 \uc5c6\ub294 \ud3b8\uc774\uc5b4\uc11c \uc81c \uc0dd\uac01\uc5d0 \uadf8\ub807\uac8c \uc798 \ub420 \uac83 \uac19\uc9c0\ub294 \uc54a\uc558\uc2b5\ub2c8\ub2e4. \uc774\ub7ec\ud55c \uc544\uc774\ub514\uc5b4\ub294 \ub531 \ud55c \ubc88 \ub9de\uc544\ub5a8\uc5b4\uc84c\uace0 \uae30\ubc1c\ud568\uc73c\ub85c \uc2b9\ubd80\ud558\uace0 \uc788\uc2b5\ub2c8\ub2e4. \uc774\ub97c \ubaa8\ubc29\ud558\ub294 \ubaa8\ub4e0 \uc0ac\uc774\ud2b8\ub4e4\uc740 \uc21c\uc804\ud788 \ud76c\uadf9\uc801\uc778 \uac00\uce58\ub9cc\uc744 \uc9c0\ub2c8\uace0 \uc788\uc9c0\ub9cc \uc81c \uc0ac\uc774\ud2b8\uc758 \uacbd\uc6b0 \ud76c\uadf9\uc801\uc778 \uc694\uc18c\ub294 \uc5ec\ubd84\uc758 \uac00\uce58\uc785\ub2c8\ub2e4. \uadf8\ub798\uc11c \uc800\ub294 \ubaa8\ubc29\ud558\ub294 \uc790\ub4e4\uc5d0\uac8c \ud589\uc6b4\uc744 \ube48\ub2e4\uace0 \ud569\ub2c8\ub2e4.\"\ub77c\uace0 \ub9d0\ud588\ub2e4.", "answer": "\ud589\uc6b4", "sentence": "\uadf8\ub798\uc11c \uc800\ub294 \ubaa8\ubc29\ud558\ub294 \uc790\ub4e4\uc5d0\uac8c \ud589\uc6b4 \uc744 \ube48\ub2e4\uace0 \ud569\ub2c8\ub2e4.", "paragraph_sentence": "\uc774 \ubc16\uc5d0\ub3c4 \ud654\uc18c \ub2e8\uc704\ub85c \uad11\uace0\ub97c \ud310\ub9e4\ud558\ub294 \ub2e4\ub978 \uc6f9\uc0ac\uc774\ud2b8\ub4e4\uc740 \uc218\uc5c6\uc774 \ub9ce\ub2e4. \ud29c\ub294 \uc774 \uc0ac\uc774\ud2b8\ub4e4\uc744 \ubcf4\uace0 \"(\uc774\ub4e4\uc740) \uac70\uc758 \ubc14\ub85c \ubc14\ub85c \ub739\ub2c8\ub2e4. \uc9c0\uae08\uc740 \ud654\uc18c\ub97c \ud310\ub9e4\ud558\ub294 \uc6f9 \uc0ac\uc774\ud2b8\ub4e4\uc774 \uc218\ubc31 \uacf3 \uc788\uc2b5\ub2c8\ub2e4. \ubaa8\ubc29\uc744 \ud558\ub294 \uacf3\ub4e4\uc740 \ubaa8\ub450 \ub2e4\ub978 \uacf3\uacfc \uacbd\uc7c1\uc744 \ud558\uace0 \uc788\uc2b5\ub2c8\ub2e4. \"\ub77c\uace0 \ub9d0\uc744 \ub358\uc84c\ub2e4. \ub610, \"\uc774\ub7ec\ud55c \uac83\ub4e4\uc740 \uad11\uace0\uac00 \uac70\uc758 \uc5c6\ub294 \ud3b8\uc774\uc5b4\uc11c \uc81c \uc0dd\uac01\uc5d0 \uadf8\ub807\uac8c \uc798 \ub420 \uac83 \uac19\uc9c0\ub294 \uc54a\uc558\uc2b5\ub2c8\ub2e4. \uc774\ub7ec\ud55c \uc544\uc774\ub514\uc5b4\ub294 \ub531 \ud55c \ubc88 \ub9de\uc544\ub5a8\uc5b4\uc84c\uace0 \uae30\ubc1c\ud568\uc73c\ub85c \uc2b9\ubd80\ud558\uace0 \uc788\uc2b5\ub2c8\ub2e4. \uc774\ub97c \ubaa8\ubc29\ud558\ub294 \ubaa8\ub4e0 \uc0ac\uc774\ud2b8\ub4e4\uc740 \uc21c\uc804\ud788 \ud76c\uadf9\uc801\uc778 \uac00\uce58\ub9cc\uc744 \uc9c0\ub2c8\uace0 \uc788\uc9c0\ub9cc \uc81c \uc0ac\uc774\ud2b8\uc758 \uacbd\uc6b0 \ud76c\uadf9\uc801\uc778 \uc694\uc18c\ub294 \uc5ec\ubd84\uc758 \uac00\uce58\uc785\ub2c8\ub2e4. \uadf8\ub798\uc11c \uc800\ub294 \ubaa8\ubc29\ud558\ub294 \uc790\ub4e4\uc5d0\uac8c \ud589\uc6b4 \uc744 \ube48\ub2e4\uace0 \ud569\ub2c8\ub2e4. \"\ub77c\uace0 \ub9d0\ud588\ub2e4.", "paragraph_answer": "\uc774 \ubc16\uc5d0\ub3c4 \ud654\uc18c \ub2e8\uc704\ub85c \uad11\uace0\ub97c \ud310\ub9e4\ud558\ub294 \ub2e4\ub978 \uc6f9\uc0ac\uc774\ud2b8\ub4e4\uc740 \uc218\uc5c6\uc774 \ub9ce\ub2e4. \ud29c\ub294 \uc774 \uc0ac\uc774\ud2b8\ub4e4\uc744 \ubcf4\uace0 \"(\uc774\ub4e4\uc740) \uac70\uc758 \ubc14\ub85c \ubc14\ub85c \ub739\ub2c8\ub2e4. \uc9c0\uae08\uc740 \ud654\uc18c\ub97c \ud310\ub9e4\ud558\ub294 \uc6f9 \uc0ac\uc774\ud2b8\ub4e4\uc774 \uc218\ubc31 \uacf3 \uc788\uc2b5\ub2c8\ub2e4. \ubaa8\ubc29\uc744 \ud558\ub294 \uacf3\ub4e4\uc740 \ubaa8\ub450 \ub2e4\ub978 \uacf3\uacfc \uacbd\uc7c1\uc744 \ud558\uace0 \uc788\uc2b5\ub2c8\ub2e4.\"\ub77c\uace0 \ub9d0\uc744 \ub358\uc84c\ub2e4. \ub610, \"\uc774\ub7ec\ud55c \uac83\ub4e4\uc740 \uad11\uace0\uac00 \uac70\uc758 \uc5c6\ub294 \ud3b8\uc774\uc5b4\uc11c \uc81c \uc0dd\uac01\uc5d0 \uadf8\ub807\uac8c \uc798 \ub420 \uac83 \uac19\uc9c0\ub294 \uc54a\uc558\uc2b5\ub2c8\ub2e4. \uc774\ub7ec\ud55c \uc544\uc774\ub514\uc5b4\ub294 \ub531 \ud55c \ubc88 \ub9de\uc544\ub5a8\uc5b4\uc84c\uace0 \uae30\ubc1c\ud568\uc73c\ub85c \uc2b9\ubd80\ud558\uace0 \uc788\uc2b5\ub2c8\ub2e4. \uc774\ub97c \ubaa8\ubc29\ud558\ub294 \ubaa8\ub4e0 \uc0ac\uc774\ud2b8\ub4e4\uc740 \uc21c\uc804\ud788 \ud76c\uadf9\uc801\uc778 \uac00\uce58\ub9cc\uc744 \uc9c0\ub2c8\uace0 \uc788\uc9c0\ub9cc \uc81c \uc0ac\uc774\ud2b8\uc758 \uacbd\uc6b0 \ud76c\uadf9\uc801\uc778 \uc694\uc18c\ub294 \uc5ec\ubd84\uc758 \uac00\uce58\uc785\ub2c8\ub2e4. \uadf8\ub798\uc11c \uc800\ub294 \ubaa8\ubc29\ud558\ub294 \uc790\ub4e4\uc5d0\uac8c \ud589\uc6b4 \uc744 \ube48\ub2e4\uace0 \ud569\ub2c8\ub2e4.\"\ub77c\uace0 \ub9d0\ud588\ub2e4.", "sentence_answer": "\uadf8\ub798\uc11c \uc800\ub294 \ubaa8\ubc29\ud558\ub294 \uc790\ub4e4\uc5d0\uac8c \ud589\uc6b4 \uc744 \ube48\ub2e4\uace0 \ud569\ub2c8\ub2e4.", "paragraph_id": "6603055-7-1", "paragraph_question": "question: \ud29c\ub294 \ubaa8\ubc29\ud558\ub294 \uc790\ub4e4\uc5d0\uac8c \ubb34\uc5c7\uc744 \ube48\ub2e4\uace0 \ud558\uc600\ub294\uac00?, context: \uc774 \ubc16\uc5d0\ub3c4 \ud654\uc18c \ub2e8\uc704\ub85c \uad11\uace0\ub97c \ud310\ub9e4\ud558\ub294 \ub2e4\ub978 \uc6f9\uc0ac\uc774\ud2b8\ub4e4\uc740 \uc218\uc5c6\uc774 \ub9ce\ub2e4. \ud29c\ub294 \uc774 \uc0ac\uc774\ud2b8\ub4e4\uc744 \ubcf4\uace0 \"(\uc774\ub4e4\uc740) \uac70\uc758 \ubc14\ub85c \ubc14\ub85c \ub739\ub2c8\ub2e4. \uc9c0\uae08\uc740 \ud654\uc18c\ub97c \ud310\ub9e4\ud558\ub294 \uc6f9 \uc0ac\uc774\ud2b8\ub4e4\uc774 \uc218\ubc31 \uacf3 \uc788\uc2b5\ub2c8\ub2e4. \ubaa8\ubc29\uc744 \ud558\ub294 \uacf3\ub4e4\uc740 \ubaa8\ub450 \ub2e4\ub978 \uacf3\uacfc \uacbd\uc7c1\uc744 \ud558\uace0 \uc788\uc2b5\ub2c8\ub2e4.\"\ub77c\uace0 \ub9d0\uc744 \ub358\uc84c\ub2e4. \ub610, \"\uc774\ub7ec\ud55c \uac83\ub4e4\uc740 \uad11\uace0\uac00 \uac70\uc758 \uc5c6\ub294 \ud3b8\uc774\uc5b4\uc11c \uc81c \uc0dd\uac01\uc5d0 \uadf8\ub807\uac8c \uc798 \ub420 \uac83 \uac19\uc9c0\ub294 \uc54a\uc558\uc2b5\ub2c8\ub2e4. \uc774\ub7ec\ud55c \uc544\uc774\ub514\uc5b4\ub294 \ub531 \ud55c \ubc88 \ub9de\uc544\ub5a8\uc5b4\uc84c\uace0 \uae30\ubc1c\ud568\uc73c\ub85c \uc2b9\ubd80\ud558\uace0 \uc788\uc2b5\ub2c8\ub2e4. \uc774\ub97c \ubaa8\ubc29\ud558\ub294 \ubaa8\ub4e0 \uc0ac\uc774\ud2b8\ub4e4\uc740 \uc21c\uc804\ud788 \ud76c\uadf9\uc801\uc778 \uac00\uce58\ub9cc\uc744 \uc9c0\ub2c8\uace0 \uc788\uc9c0\ub9cc \uc81c \uc0ac\uc774\ud2b8\uc758 \uacbd\uc6b0 \ud76c\uadf9\uc801\uc778 \uc694\uc18c\ub294 \uc5ec\ubd84\uc758 \uac00\uce58\uc785\ub2c8\ub2e4. \uadf8\ub798\uc11c \uc800\ub294 \ubaa8\ubc29\ud558\ub294 \uc790\ub4e4\uc5d0\uac8c \ud589\uc6b4\uc744 \ube48\ub2e4\uace0 \ud569\ub2c8\ub2e4.\"\ub77c\uace0 \ub9d0\ud588\ub2e4."} {"question": "\uc624\ubc14\ub9c8\uc758 \ubd80\ubaa8\uac00 \uacb0\ud63c\ud558\uace0, \uc624\ubc14\ub9c8\uac00 \ud0dc\uc5b4\ub09c \ud574\ub294?", "paragraph": "\ubbf8\uad6d \ud558\uc640\uc774 \ud638\ub180\ub8f0\ub8e8\uc758 \uce74\ud53c\uc62c\ub77c\ub2c8 \ubcd1\uc6d0\uc5d0\uc11c \uce94\uc790\uc2a4 \uc8fc \uc704\uce58\ud1a0 \ucd9c\uc2e0\uc758 \uc601\uad6d\uacc4 \ubbf8\uad6d\uc778 \uc2a4\ud0e0\ub9ac \uc564 \ub358\ud584 \uacfc \ucf00\ub0d0 \uc2dd\ubbfc\uc9c0 \ub2c8\uc548\uc790 \uc8fc \ub2c8\uc591\uc624\ub9c8 \ucf54\uac94\ub85c \ucd9c\uc2e0\uc758 \ub8e8\uc624\uc871 \ubc84\ub77d \uc624\ubc14\ub9c8 \uc2dc\ub2c8\uc5b4 \uc0ac\uc774\uc5d0\uc11c \ud0dc\uc5b4\ub0ac\ub2e4. \uc624\ubc14\ub9c8\uc758 \ubd80\ubaa8\ub294 1960\ub144, \uc544\ubc84\uc9c0 \uc624\ubc14\ub9c8\uac00 \ud574\uc678 \uc7a5\ud559\uc0dd\uc73c\ub85c \uc788\ub358 \ub9c8\ub178\uc544\uc758 \ud558\uc640\uc774 \ub300\ud559\uad50\uc758 \ub7ec\uc2dc\uc544\uc5b4 \uc218\uc5c5\uc5d0\uc11c \ub9cc\ub0ac\ub2e4. \ub450 \uc0ac\ub78c\uc740 1961\ub144 2\uc6d4 2\uc77c\uc5d0 \uacb0\ud63c\ud558\uc600\uc73c\uba70, \uac19\uc740 \ud574 \ubc84\ub77d \uc624\ubc14\ub9c8\uac00 \ud0dc\uc5b4\ub0ac\ub2e4. \uadf8\uac00 2\uc0b4\uc774\ub358 \ud574 \uc624\ubc14\ub9c8 \ubd80\ubd80\ub294 \ubcc4\uac70\ud558\uc600\uc73c\uba70, 1964\ub144\uc5d0\ub294 \uc774\ud63c\uc5d0 \uc774\ub974\ub800\ub2e4. \uc624\ubc14\ub9c8\uc758 \uc544\ubc84\uc9c0\ub294 \uc7ac\ud63c\ud558\uc5ec \ucf00\ub0d0\ub85c \ub3cc\uc544\uac00 \ub450 \uc544\ub4e4 \ub370\uc774\ube44\ub4dc(David)\uc640 \ub9c8\ud06c \uc740\ub370\uc0b0\uc870(Mark Ndesandjo)\ub97c \ub0b3\uc558\ub2e4. \uadf8\ub294 1982\ub144 \uad50\ud1b5\uc0ac\uace0\ub85c \uc8fd\uae30 \uc804\uc5d0 \uc544\ub4e4 \uc624\ubc14\ub9c8\ub97c \ub2e8 \ud55c \ubc88\ubc16\uc5d0 \ubcf4\uc9c0 \ubabb\ud588\ub2e4.", "answer": "1961\ub144", "sentence": "\ub450 \uc0ac\ub78c\uc740 1961\ub144 2\uc6d4 2\uc77c\uc5d0 \uacb0\ud63c\ud558\uc600\uc73c\uba70, \uac19\uc740 \ud574 \ubc84\ub77d \uc624\ubc14\ub9c8\uac00 \ud0dc\uc5b4\ub0ac\ub2e4.", "paragraph_sentence": "\ubbf8\uad6d \ud558\uc640\uc774 \ud638\ub180\ub8f0\ub8e8\uc758 \uce74\ud53c\uc62c\ub77c\ub2c8 \ubcd1\uc6d0\uc5d0\uc11c \uce94\uc790\uc2a4 \uc8fc \uc704\uce58\ud1a0 \ucd9c\uc2e0\uc758 \uc601\uad6d\uacc4 \ubbf8\uad6d\uc778 \uc2a4\ud0e0\ub9ac \uc564 \ub358\ud584 \uacfc \ucf00\ub0d0 \uc2dd\ubbfc\uc9c0 \ub2c8\uc548\uc790 \uc8fc \ub2c8\uc591\uc624\ub9c8 \ucf54\uac94\ub85c \ucd9c\uc2e0\uc758 \ub8e8\uc624\uc871 \ubc84\ub77d \uc624\ubc14\ub9c8 \uc2dc\ub2c8\uc5b4 \uc0ac\uc774\uc5d0\uc11c \ud0dc\uc5b4\ub0ac\ub2e4. \uc624\ubc14\ub9c8\uc758 \ubd80\ubaa8\ub294 1960\ub144, \uc544\ubc84\uc9c0 \uc624\ubc14\ub9c8\uac00 \ud574\uc678 \uc7a5\ud559\uc0dd\uc73c\ub85c \uc788\ub358 \ub9c8\ub178\uc544\uc758 \ud558\uc640\uc774 \ub300\ud559\uad50\uc758 \ub7ec\uc2dc\uc544\uc5b4 \uc218\uc5c5\uc5d0\uc11c \ub9cc\ub0ac\ub2e4. \ub450 \uc0ac\ub78c\uc740 1961\ub144 2\uc6d4 2\uc77c\uc5d0 \uacb0\ud63c\ud558\uc600\uc73c\uba70, \uac19\uc740 \ud574 \ubc84\ub77d \uc624\ubc14\ub9c8\uac00 \ud0dc\uc5b4\ub0ac\ub2e4. \uadf8\uac00 2\uc0b4\uc774\ub358 \ud574 \uc624\ubc14\ub9c8 \ubd80\ubd80\ub294 \ubcc4\uac70\ud558\uc600\uc73c\uba70, 1964\ub144\uc5d0\ub294 \uc774\ud63c\uc5d0 \uc774\ub974\ub800\ub2e4. \uc624\ubc14\ub9c8\uc758 \uc544\ubc84\uc9c0\ub294 \uc7ac\ud63c\ud558\uc5ec \ucf00\ub0d0\ub85c \ub3cc\uc544\uac00 \ub450 \uc544\ub4e4 \ub370\uc774\ube44\ub4dc(David)\uc640 \ub9c8\ud06c \uc740\ub370\uc0b0\uc870(Mark Ndesandjo)\ub97c \ub0b3\uc558\ub2e4. \uadf8\ub294 1982\ub144 \uad50\ud1b5\uc0ac\uace0\ub85c \uc8fd\uae30 \uc804\uc5d0 \uc544\ub4e4 \uc624\ubc14\ub9c8\ub97c \ub2e8 \ud55c \ubc88\ubc16\uc5d0 \ubcf4\uc9c0 \ubabb\ud588\ub2e4.", "paragraph_answer": "\ubbf8\uad6d \ud558\uc640\uc774 \ud638\ub180\ub8f0\ub8e8\uc758 \uce74\ud53c\uc62c\ub77c\ub2c8 \ubcd1\uc6d0\uc5d0\uc11c \uce94\uc790\uc2a4 \uc8fc \uc704\uce58\ud1a0 \ucd9c\uc2e0\uc758 \uc601\uad6d\uacc4 \ubbf8\uad6d\uc778 \uc2a4\ud0e0\ub9ac \uc564 \ub358\ud584 \uacfc \ucf00\ub0d0 \uc2dd\ubbfc\uc9c0 \ub2c8\uc548\uc790 \uc8fc \ub2c8\uc591\uc624\ub9c8 \ucf54\uac94\ub85c \ucd9c\uc2e0\uc758 \ub8e8\uc624\uc871 \ubc84\ub77d \uc624\ubc14\ub9c8 \uc2dc\ub2c8\uc5b4 \uc0ac\uc774\uc5d0\uc11c \ud0dc\uc5b4\ub0ac\ub2e4. \uc624\ubc14\ub9c8\uc758 \ubd80\ubaa8\ub294 1960\ub144, \uc544\ubc84\uc9c0 \uc624\ubc14\ub9c8\uac00 \ud574\uc678 \uc7a5\ud559\uc0dd\uc73c\ub85c \uc788\ub358 \ub9c8\ub178\uc544\uc758 \ud558\uc640\uc774 \ub300\ud559\uad50\uc758 \ub7ec\uc2dc\uc544\uc5b4 \uc218\uc5c5\uc5d0\uc11c \ub9cc\ub0ac\ub2e4. \ub450 \uc0ac\ub78c\uc740 1961\ub144 2\uc6d4 2\uc77c\uc5d0 \uacb0\ud63c\ud558\uc600\uc73c\uba70, \uac19\uc740 \ud574 \ubc84\ub77d \uc624\ubc14\ub9c8\uac00 \ud0dc\uc5b4\ub0ac\ub2e4. \uadf8\uac00 2\uc0b4\uc774\ub358 \ud574 \uc624\ubc14\ub9c8 \ubd80\ubd80\ub294 \ubcc4\uac70\ud558\uc600\uc73c\uba70, 1964\ub144\uc5d0\ub294 \uc774\ud63c\uc5d0 \uc774\ub974\ub800\ub2e4. \uc624\ubc14\ub9c8\uc758 \uc544\ubc84\uc9c0\ub294 \uc7ac\ud63c\ud558\uc5ec \ucf00\ub0d0\ub85c \ub3cc\uc544\uac00 \ub450 \uc544\ub4e4 \ub370\uc774\ube44\ub4dc(David)\uc640 \ub9c8\ud06c \uc740\ub370\uc0b0\uc870(Mark Ndesandjo)\ub97c \ub0b3\uc558\ub2e4. \uadf8\ub294 1982\ub144 \uad50\ud1b5\uc0ac\uace0\ub85c \uc8fd\uae30 \uc804\uc5d0 \uc544\ub4e4 \uc624\ubc14\ub9c8\ub97c \ub2e8 \ud55c \ubc88\ubc16\uc5d0 \ubcf4\uc9c0 \ubabb\ud588\ub2e4.", "sentence_answer": "\ub450 \uc0ac\ub78c\uc740 1961\ub144 2\uc6d4 2\uc77c\uc5d0 \uacb0\ud63c\ud558\uc600\uc73c\uba70, \uac19\uc740 \ud574 \ubc84\ub77d \uc624\ubc14\ub9c8\uac00 \ud0dc\uc5b4\ub0ac\ub2e4.", "paragraph_id": "6108839-0-1", "paragraph_question": "question: \uc624\ubc14\ub9c8\uc758 \ubd80\ubaa8\uac00 \uacb0\ud63c\ud558\uace0, \uc624\ubc14\ub9c8\uac00 \ud0dc\uc5b4\ub09c \ud574\ub294?, context: \ubbf8\uad6d \ud558\uc640\uc774 \ud638\ub180\ub8f0\ub8e8\uc758 \uce74\ud53c\uc62c\ub77c\ub2c8 \ubcd1\uc6d0\uc5d0\uc11c \uce94\uc790\uc2a4 \uc8fc \uc704\uce58\ud1a0 \ucd9c\uc2e0\uc758 \uc601\uad6d\uacc4 \ubbf8\uad6d\uc778 \uc2a4\ud0e0\ub9ac \uc564 \ub358\ud584 \uacfc \ucf00\ub0d0 \uc2dd\ubbfc\uc9c0 \ub2c8\uc548\uc790 \uc8fc \ub2c8\uc591\uc624\ub9c8 \ucf54\uac94\ub85c \ucd9c\uc2e0\uc758 \ub8e8\uc624\uc871 \ubc84\ub77d \uc624\ubc14\ub9c8 \uc2dc\ub2c8\uc5b4 \uc0ac\uc774\uc5d0\uc11c \ud0dc\uc5b4\ub0ac\ub2e4. \uc624\ubc14\ub9c8\uc758 \ubd80\ubaa8\ub294 1960\ub144, \uc544\ubc84\uc9c0 \uc624\ubc14\ub9c8\uac00 \ud574\uc678 \uc7a5\ud559\uc0dd\uc73c\ub85c \uc788\ub358 \ub9c8\ub178\uc544\uc758 \ud558\uc640\uc774 \ub300\ud559\uad50\uc758 \ub7ec\uc2dc\uc544\uc5b4 \uc218\uc5c5\uc5d0\uc11c \ub9cc\ub0ac\ub2e4. \ub450 \uc0ac\ub78c\uc740 1961\ub144 2\uc6d4 2\uc77c\uc5d0 \uacb0\ud63c\ud558\uc600\uc73c\uba70, \uac19\uc740 \ud574 \ubc84\ub77d \uc624\ubc14\ub9c8\uac00 \ud0dc\uc5b4\ub0ac\ub2e4. \uadf8\uac00 2\uc0b4\uc774\ub358 \ud574 \uc624\ubc14\ub9c8 \ubd80\ubd80\ub294 \ubcc4\uac70\ud558\uc600\uc73c\uba70, 1964\ub144\uc5d0\ub294 \uc774\ud63c\uc5d0 \uc774\ub974\ub800\ub2e4. \uc624\ubc14\ub9c8\uc758 \uc544\ubc84\uc9c0\ub294 \uc7ac\ud63c\ud558\uc5ec \ucf00\ub0d0\ub85c \ub3cc\uc544\uac00 \ub450 \uc544\ub4e4 \ub370\uc774\ube44\ub4dc(David)\uc640 \ub9c8\ud06c \uc740\ub370\uc0b0\uc870(Mark Ndesandjo)\ub97c \ub0b3\uc558\ub2e4. \uadf8\ub294 1982\ub144 \uad50\ud1b5\uc0ac\uace0\ub85c \uc8fd\uae30 \uc804\uc5d0 \uc544\ub4e4 \uc624\ubc14\ub9c8\ub97c \ub2e8 \ud55c \ubc88\ubc16\uc5d0 \ubcf4\uc9c0 \ubabb\ud588\ub2e4."} {"question": "1848\ub144 3\uc6d4 10\uc77c\uc5d0 \ub3c5\uc77c \uc5f0\ubc29\uc758\ud68c\ub294 \uba87 \uba85\uc758 \uc704\uc6d0\uc744 \uc18c\uc9d1\ud588\ub294\uac00?", "paragraph": "\uc774\ub7ec\ud55c \ubc1c\uc804\uc740 3\uc6d4 \ucd08 \ubc14\ub374, \ubc14\uc774\uc5d0\ub978, \uc791\uc13c, \ubdd4\ub974\ud15c\ubca0\ub974\ud06c, \uc624\uc2a4\ud2b8\ub9ac\uc544, \ud504\ub85c\uc774\uc13c \ub4f1\ub4f1\uc758 \uc5ec\ub7ec \ub3c5\uc77c \uc9c0\uc5ed \uad6d\uac00\ub4e4\uc5d0\uc11c \ud56d\uc758 \uc2dc\uc704\uc640 \ubd09\uae30\uac00 \uc77c\uc5b4\ub098\uba74\uc11c \ubd81\ub3cb\uc544\uc84c\ub2e4. \uc774 \uac19\uc740 \uc555\ub825\uc740 \uac01\uad6d \uc81c\ud6c4\ub4e4\uc774 \uae30\uc874\uc758 \ubcf4\uc218\uc801 \uc815\uad8c\ub4e4\uc744 \ud574\uc784\ud558\uace0 \uc790\uc720\uc8fc\uc758\uc801 \uc704\uc6d0\ud68c\ub4e4\uc744 \ud1b5\ud574 \uc18c\uc704 3\uc6d4 \uc815\uad8c(M\u00e4rzregierung)\uc774\ub77c \ubd88\ub9ac\ub294 \uac83\uc73c\ub85c \ub300\uccb4\ud558\uac8c \ub9cc\ub4e4\uc5c8\ub2e4. 1848\ub144 3\uc6d4 10\uc77c \ub3c5\uc77c \uc5f0\ubc29\uc758 \uc5f0\ubc29\uc758\ud68c\ub294 17\uc778 \uc704\uc6d0\ud68c(Siebzehnerausschuss)\ub97c \uc18c\uc9d1\ud574 \ud5cc\ubc95 \ucd08\uc548\uc758 \ub9c8\ubb34\ub9ac\ub97c \ud558\ub3c4\ub85d \ud558\uc600\uace0, 3\uc6d4 20\uc77c\uc5d0\ub294 \ub2f9\uc2dc\uae4c\uc9c0 \ub3c5\uc77c \uc5f0\ubc29\uc758 \uc601\uc5ed \uc678\ubd80\uc5d0 \ub193\uc5ec\uc788\ub358 \ud504\ub85c\uc774\uc13c \uc9c0\ubc29\uc744 \ud3ec\ud568\ud558\uc5ec \ub3c5\uc77c \uc5f0\ubc29\uc758 \uad6d\uac00\ub4e4\uc5d0\uac8c \uc81c\ud5cc \uc758\ud68c\uc758 \ub0a0\uc9dc\ub97c \uacb0\uc815\ud558\uae30 \uc704\ud55c \ud22c\ud45c\ub97c \ud558\ub3c4\ub85d \uc7ac\ucd09\ud588\ub2e4. \ud504\ub85c\uc774\uc13c\uc5d0\uc11c\ub294 \uc720\ud608 \uc2dc\uac00\uc804\uc774 \ubc8c\uc5b4\uc9c4 \ud6c4 \uad6d\uc655\uc774 \uad74\ubcf5\ud558\uc5ec \uc790\uc720\uc8fc\uc758\uc801 \ub0b4\uac01\uc744 \uad6c\uc131\ud558\uace0 \ud504\ub85c\uc774\uc13c \uc655\uad6d\uc758 \ud5cc\ubc95\uc744 \ub9cc\ub4e4\ub3c4\ub85d \ud504\ub85c\uc774\uc13c \uad6d\ubbfc\uc758\ud68c\ub97c \uc18c\uc9d1\ud558\uc600\ub2e4.", "answer": "17\uc778", "sentence": "\uc5f0\ubc29\uc758\ud68c\ub294 17\uc778 \uc704\uc6d0\ud68c(Siebzehnerausschuss)\ub97c \uc18c\uc9d1\ud574 \ud5cc\ubc95 \ucd08\uc548\uc758 \ub9c8\ubb34\ub9ac\ub97c \ud558\ub3c4\ub85d \ud558\uc600\uace0, 3\uc6d4 20\uc77c\uc5d0\ub294 \ub2f9\uc2dc\uae4c\uc9c0 \ub3c5\uc77c \uc5f0\ubc29\uc758 \uc601\uc5ed \uc678\ubd80\uc5d0 \ub193\uc5ec\uc788\ub358 \ud504\ub85c\uc774\uc13c \uc9c0\ubc29\uc744 \ud3ec\ud568\ud558\uc5ec \ub3c5\uc77c \uc5f0\ubc29\uc758 \uad6d\uac00\ub4e4\uc5d0\uac8c \uc81c\ud5cc \uc758\ud68c\uc758 \ub0a0\uc9dc\ub97c \uacb0\uc815\ud558\uae30 \uc704\ud55c \ud22c\ud45c\ub97c \ud558\ub3c4\ub85d \uc7ac\ucd09\ud588\ub2e4.", "paragraph_sentence": "\uc774\ub7ec\ud55c \ubc1c\uc804\uc740 3\uc6d4 \ucd08 \ubc14\ub374, \ubc14\uc774\uc5d0\ub978, \uc791\uc13c, \ubdd4\ub974\ud15c\ubca0\ub974\ud06c, \uc624\uc2a4\ud2b8\ub9ac\uc544, \ud504\ub85c\uc774\uc13c \ub4f1\ub4f1\uc758 \uc5ec\ub7ec \ub3c5\uc77c \uc9c0\uc5ed \uad6d\uac00\ub4e4\uc5d0\uc11c \ud56d\uc758 \uc2dc\uc704\uc640 \ubd09\uae30\uac00 \uc77c\uc5b4\ub098\uba74\uc11c \ubd81\ub3cb\uc544\uc84c\ub2e4. \uc774 \uac19\uc740 \uc555\ub825\uc740 \uac01\uad6d \uc81c\ud6c4\ub4e4\uc774 \uae30\uc874\uc758 \ubcf4\uc218\uc801 \uc815\uad8c\ub4e4\uc744 \ud574\uc784\ud558\uace0 \uc790\uc720\uc8fc\uc758\uc801 \uc704\uc6d0\ud68c\ub4e4\uc744 \ud1b5\ud574 \uc18c\uc704 3\uc6d4 \uc815\uad8c(M\u00e4rzregierung)\uc774\ub77c \ubd88\ub9ac\ub294 \uac83\uc73c\ub85c \ub300\uccb4\ud558\uac8c \ub9cc\ub4e4\uc5c8\ub2e4. 1848\ub144 3\uc6d4 10\uc77c \ub3c5\uc77c \uc5f0\ubc29\uc758 \uc5f0\ubc29\uc758\ud68c\ub294 17\uc778 \uc704\uc6d0\ud68c(Siebzehnerausschuss)\ub97c \uc18c\uc9d1\ud574 \ud5cc\ubc95 \ucd08\uc548\uc758 \ub9c8\ubb34\ub9ac\ub97c \ud558\ub3c4\ub85d \ud558\uc600\uace0, 3\uc6d4 20\uc77c\uc5d0\ub294 \ub2f9\uc2dc\uae4c\uc9c0 \ub3c5\uc77c \uc5f0\ubc29\uc758 \uc601\uc5ed \uc678\ubd80\uc5d0 \ub193\uc5ec\uc788\ub358 \ud504\ub85c\uc774\uc13c \uc9c0\ubc29\uc744 \ud3ec\ud568\ud558\uc5ec \ub3c5\uc77c \uc5f0\ubc29\uc758 \uad6d\uac00\ub4e4\uc5d0\uac8c \uc81c\ud5cc \uc758\ud68c\uc758 \ub0a0\uc9dc\ub97c \uacb0\uc815\ud558\uae30 \uc704\ud55c \ud22c\ud45c\ub97c \ud558\ub3c4\ub85d \uc7ac\ucd09\ud588\ub2e4. \ud504\ub85c\uc774\uc13c\uc5d0\uc11c\ub294 \uc720\ud608 \uc2dc\uac00\uc804\uc774 \ubc8c\uc5b4\uc9c4 \ud6c4 \uad6d\uc655\uc774 \uad74\ubcf5\ud558\uc5ec \uc790\uc720\uc8fc\uc758\uc801 \ub0b4\uac01\uc744 \uad6c\uc131\ud558\uace0 \ud504\ub85c\uc774\uc13c \uc655\uad6d\uc758 \ud5cc\ubc95\uc744 \ub9cc\ub4e4\ub3c4\ub85d \ud504\ub85c\uc774\uc13c \uad6d\ubbfc\uc758\ud68c\ub97c \uc18c\uc9d1\ud558\uc600\ub2e4.", "paragraph_answer": "\uc774\ub7ec\ud55c \ubc1c\uc804\uc740 3\uc6d4 \ucd08 \ubc14\ub374, \ubc14\uc774\uc5d0\ub978, \uc791\uc13c, \ubdd4\ub974\ud15c\ubca0\ub974\ud06c, \uc624\uc2a4\ud2b8\ub9ac\uc544, \ud504\ub85c\uc774\uc13c \ub4f1\ub4f1\uc758 \uc5ec\ub7ec \ub3c5\uc77c \uc9c0\uc5ed \uad6d\uac00\ub4e4\uc5d0\uc11c \ud56d\uc758 \uc2dc\uc704\uc640 \ubd09\uae30\uac00 \uc77c\uc5b4\ub098\uba74\uc11c \ubd81\ub3cb\uc544\uc84c\ub2e4. \uc774 \uac19\uc740 \uc555\ub825\uc740 \uac01\uad6d \uc81c\ud6c4\ub4e4\uc774 \uae30\uc874\uc758 \ubcf4\uc218\uc801 \uc815\uad8c\ub4e4\uc744 \ud574\uc784\ud558\uace0 \uc790\uc720\uc8fc\uc758\uc801 \uc704\uc6d0\ud68c\ub4e4\uc744 \ud1b5\ud574 \uc18c\uc704 3\uc6d4 \uc815\uad8c(M\u00e4rzregierung)\uc774\ub77c \ubd88\ub9ac\ub294 \uac83\uc73c\ub85c \ub300\uccb4\ud558\uac8c \ub9cc\ub4e4\uc5c8\ub2e4. 1848\ub144 3\uc6d4 10\uc77c \ub3c5\uc77c \uc5f0\ubc29\uc758 \uc5f0\ubc29\uc758\ud68c\ub294 17\uc778 \uc704\uc6d0\ud68c(Siebzehnerausschuss)\ub97c \uc18c\uc9d1\ud574 \ud5cc\ubc95 \ucd08\uc548\uc758 \ub9c8\ubb34\ub9ac\ub97c \ud558\ub3c4\ub85d \ud558\uc600\uace0, 3\uc6d4 20\uc77c\uc5d0\ub294 \ub2f9\uc2dc\uae4c\uc9c0 \ub3c5\uc77c \uc5f0\ubc29\uc758 \uc601\uc5ed \uc678\ubd80\uc5d0 \ub193\uc5ec\uc788\ub358 \ud504\ub85c\uc774\uc13c \uc9c0\ubc29\uc744 \ud3ec\ud568\ud558\uc5ec \ub3c5\uc77c \uc5f0\ubc29\uc758 \uad6d\uac00\ub4e4\uc5d0\uac8c \uc81c\ud5cc \uc758\ud68c\uc758 \ub0a0\uc9dc\ub97c \uacb0\uc815\ud558\uae30 \uc704\ud55c \ud22c\ud45c\ub97c \ud558\ub3c4\ub85d \uc7ac\ucd09\ud588\ub2e4. \ud504\ub85c\uc774\uc13c\uc5d0\uc11c\ub294 \uc720\ud608 \uc2dc\uac00\uc804\uc774 \ubc8c\uc5b4\uc9c4 \ud6c4 \uad6d\uc655\uc774 \uad74\ubcf5\ud558\uc5ec \uc790\uc720\uc8fc\uc758\uc801 \ub0b4\uac01\uc744 \uad6c\uc131\ud558\uace0 \ud504\ub85c\uc774\uc13c \uc655\uad6d\uc758 \ud5cc\ubc95\uc744 \ub9cc\ub4e4\ub3c4\ub85d \ud504\ub85c\uc774\uc13c \uad6d\ubbfc\uc758\ud68c\ub97c \uc18c\uc9d1\ud558\uc600\ub2e4.", "sentence_answer": "\uc5f0\ubc29\uc758\ud68c\ub294 17\uc778 \uc704\uc6d0\ud68c(Siebzehnerausschuss)\ub97c \uc18c\uc9d1\ud574 \ud5cc\ubc95 \ucd08\uc548\uc758 \ub9c8\ubb34\ub9ac\ub97c \ud558\ub3c4\ub85d \ud558\uc600\uace0, 3\uc6d4 20\uc77c\uc5d0\ub294 \ub2f9\uc2dc\uae4c\uc9c0 \ub3c5\uc77c \uc5f0\ubc29\uc758 \uc601\uc5ed \uc678\ubd80\uc5d0 \ub193\uc5ec\uc788\ub358 \ud504\ub85c\uc774\uc13c \uc9c0\ubc29\uc744 \ud3ec\ud568\ud558\uc5ec \ub3c5\uc77c \uc5f0\ubc29\uc758 \uad6d\uac00\ub4e4\uc5d0\uac8c \uc81c\ud5cc \uc758\ud68c\uc758 \ub0a0\uc9dc\ub97c \uacb0\uc815\ud558\uae30 \uc704\ud55c \ud22c\ud45c\ub97c \ud558\ub3c4\ub85d \uc7ac\ucd09\ud588\ub2e4.", "paragraph_id": "6568666-7-0", "paragraph_question": "question: 1848\ub144 3\uc6d4 10\uc77c\uc5d0 \ub3c5\uc77c \uc5f0\ubc29\uc758\ud68c\ub294 \uba87 \uba85\uc758 \uc704\uc6d0\uc744 \uc18c\uc9d1\ud588\ub294\uac00?, context: \uc774\ub7ec\ud55c \ubc1c\uc804\uc740 3\uc6d4 \ucd08 \ubc14\ub374, \ubc14\uc774\uc5d0\ub978, \uc791\uc13c, \ubdd4\ub974\ud15c\ubca0\ub974\ud06c, \uc624\uc2a4\ud2b8\ub9ac\uc544, \ud504\ub85c\uc774\uc13c \ub4f1\ub4f1\uc758 \uc5ec\ub7ec \ub3c5\uc77c \uc9c0\uc5ed \uad6d\uac00\ub4e4\uc5d0\uc11c \ud56d\uc758 \uc2dc\uc704\uc640 \ubd09\uae30\uac00 \uc77c\uc5b4\ub098\uba74\uc11c \ubd81\ub3cb\uc544\uc84c\ub2e4. \uc774 \uac19\uc740 \uc555\ub825\uc740 \uac01\uad6d \uc81c\ud6c4\ub4e4\uc774 \uae30\uc874\uc758 \ubcf4\uc218\uc801 \uc815\uad8c\ub4e4\uc744 \ud574\uc784\ud558\uace0 \uc790\uc720\uc8fc\uc758\uc801 \uc704\uc6d0\ud68c\ub4e4\uc744 \ud1b5\ud574 \uc18c\uc704 3\uc6d4 \uc815\uad8c(M\u00e4rzregierung)\uc774\ub77c \ubd88\ub9ac\ub294 \uac83\uc73c\ub85c \ub300\uccb4\ud558\uac8c \ub9cc\ub4e4\uc5c8\ub2e4. 1848\ub144 3\uc6d4 10\uc77c \ub3c5\uc77c \uc5f0\ubc29\uc758 \uc5f0\ubc29\uc758\ud68c\ub294 17\uc778 \uc704\uc6d0\ud68c(Siebzehnerausschuss)\ub97c \uc18c\uc9d1\ud574 \ud5cc\ubc95 \ucd08\uc548\uc758 \ub9c8\ubb34\ub9ac\ub97c \ud558\ub3c4\ub85d \ud558\uc600\uace0, 3\uc6d4 20\uc77c\uc5d0\ub294 \ub2f9\uc2dc\uae4c\uc9c0 \ub3c5\uc77c \uc5f0\ubc29\uc758 \uc601\uc5ed \uc678\ubd80\uc5d0 \ub193\uc5ec\uc788\ub358 \ud504\ub85c\uc774\uc13c \uc9c0\ubc29\uc744 \ud3ec\ud568\ud558\uc5ec \ub3c5\uc77c \uc5f0\ubc29\uc758 \uad6d\uac00\ub4e4\uc5d0\uac8c \uc81c\ud5cc \uc758\ud68c\uc758 \ub0a0\uc9dc\ub97c \uacb0\uc815\ud558\uae30 \uc704\ud55c \ud22c\ud45c\ub97c \ud558\ub3c4\ub85d \uc7ac\ucd09\ud588\ub2e4. \ud504\ub85c\uc774\uc13c\uc5d0\uc11c\ub294 \uc720\ud608 \uc2dc\uac00\uc804\uc774 \ubc8c\uc5b4\uc9c4 \ud6c4 \uad6d\uc655\uc774 \uad74\ubcf5\ud558\uc5ec \uc790\uc720\uc8fc\uc758\uc801 \ub0b4\uac01\uc744 \uad6c\uc131\ud558\uace0 \ud504\ub85c\uc774\uc13c \uc655\uad6d\uc758 \ud5cc\ubc95\uc744 \ub9cc\ub4e4\ub3c4\ub85d \ud504\ub85c\uc774\uc13c \uad6d\ubbfc\uc758\ud68c\ub97c \uc18c\uc9d1\ud558\uc600\ub2e4."} {"question": "\uc5b4\ub514\uc5d0 \uac00\ub450\uc5c8\ub294\uac00?", "paragraph": "\uac15\ud76c\uc81c\ub294 \uc724\uc789\uc774 \uc0c8\ub85c\uc6b4 \uc0ac\ub78c\uc774 \ub418\uae38 \ubc14\ub790\uc73c\ub098 \uacb0\uad6d \uc724\uc789\uc740 \ub3cc\uc774\ud0ac \uc218 \uc5c6\ub294 \uc5ed\ubaa8\uc758 \uc8fc\ub3d9\uc790\uac00 \ub418\uc5c8\ub2e4. \uac15\ud76c\uc81c\ub294 \uc989\uc2dc \uadc0\uacbd\ud558\uc5ec \ud0c1\ud569\uc81c\uc640 \uacbd\uc561, \uc81c\uc138\ubb34 \ub4f1\uc744 \uad50\uc0ac\ud558\uace0 \ub3c4\ud1b5 \uc545\uc120(\u9102\u7e55)\uc740 \uc720\ud3d0\ub41c \ud6c4 \uc548\uce5c\uc655 \uc545\ub77d(\u5b89\u89aa\u738b \u5cb3\u6a02)\uc758 \ubb18\uc18c\uc758 \ub2a5\uc9c0\uae30\ub85c \ubcf4\ub0c8\ub2e4. \ud669\uc81c\uc640 \ud669\ud0dc\uc790\uac00 \uc624\ub7ab\ub3d9\uc548 \uac19\uc774 \uc874\uc7ac\ud558\uba74\uc11c \ud669\ud0dc\uc790\uac00 \uc870\uc815\uc5d0\uc11c \uc81c2\uc758 \ud669\uc81c\ub85c\uc11c \ud669\uad8c\uc744 \uc800\uc9c0\ud560 \uac00\uc7a5 \uac15\ub825\ud55c \uc7a5\uc560\ubb3c\uc774\ub77c\uace0 \uc0dd\uac01\ud55c \uac15\ud76c\uc81c\ub294 \ub2e4\uc2dc \uc724\uc789\uc744 \ud3d0\ud560 \uacb0\uc2ec\uc744 \uad73\ud788\uace0 1712\ub144(\uac15\ud76c 51\ub144) \uc74c\ub825 9\uc6d4 30\uc77c, \uac15\ud76c\uc81c\ub294 \uc724\uc789\uc744 \uc7a1\uc544\ub4e4\uc774\uace0 \ubcf5\uc704\ud55c \uc774\ub798 \uad11\uc99d\uc774 \uc544\uc9c1 \uce58\uc720\ub418\uc9c0 \uc54a\uc558\uace0 \uc778\uc2ec\uc744 \ud06c\uac8c \uc783\uc73c\ub2c8 \uc774\ub7ec\ud55c \uc790\uc5d0\uac8c \uc5f4\uc131\uc870\uc758 \uc720\uc5c5\uc744 \uc787\uac8c \ud560 \uc218 \uc5c6\uc73c\ub2c8 \ud568\uc548\uad81(\u54b8\u5b89\u5bae)\uc5d0 \uac00\ub450\uc5b4 \uc601\uc6d0\ud788 \uc11c\uc778\uc73c\ub85c \uc0bc\uaca0\ub2e4\ub294 \uc131\uc9c0\ub97c \ub0b4\ub838\ub2e4. \uc774\ud6c4\ub85c\ub294 \uc0dd\uc804\uc5d0 \ud669\ud0dc\uc790\ub97c \ubf51\uc9c0 \uc54a\uc744 \uac83\uc784\uc744 \uacf5\uc5b8\ud558\uace0 \uc77c\uccb4\uc758 \ud669\ud0dc\uc790 \ucc45\ubd09\uc5d0 \uad00\ud55c \uc870\uc815\uc758 \uacf5\ub860\ud654\ub97c \ucc28\ub2e8\ud558\uc600\ub2e4. \ub354\ubd88\uc5b4 \uac15\ud76c\uc81c\ub294 \ub9cc\uc57d \uc774\ub7ec\ud55c \uc758\uacac\uc774 \uc870\uc815\uc5d0\uc11c \ub098\uc62c \uc2dc, \uc9c0\uc704 \uace0\ud558\ub97c \ub9c9\ub860\ud558\uace0 \ucc38\ud615\uc5d0 \ucc98\ud558\uaca0\ub2e4\ub294 \uc131\uc9c0\ub97c \ud55c\ubc88 \ub354 \ub0b4\ub838\ub2e4.", "answer": "\ud568\uc548\uad81(\u54b8\u5b89\u5bae)", "sentence": "\ud669\uc81c\uc640 \ud669\ud0dc\uc790\uac00 \uc624\ub7ab\ub3d9\uc548 \uac19\uc774 \uc874\uc7ac\ud558\uba74\uc11c \ud669\ud0dc\uc790\uac00 \uc870\uc815\uc5d0\uc11c \uc81c2\uc758 \ud669\uc81c\ub85c\uc11c \ud669\uad8c\uc744 \uc800\uc9c0\ud560 \uac00\uc7a5 \uac15\ub825\ud55c \uc7a5\uc560\ubb3c\uc774\ub77c\uace0 \uc0dd\uac01\ud55c \uac15\ud76c\uc81c\ub294 \ub2e4\uc2dc \uc724\uc789\uc744 \ud3d0\ud560 \uacb0\uc2ec\uc744 \uad73\ud788\uace0 1712\ub144(\uac15\ud76c 51\ub144) \uc74c\ub825 9\uc6d4 30\uc77c, \uac15\ud76c\uc81c\ub294 \uc724\uc789\uc744 \uc7a1\uc544\ub4e4\uc774\uace0 \ubcf5\uc704\ud55c \uc774\ub798 \uad11\uc99d\uc774 \uc544\uc9c1 \uce58\uc720\ub418\uc9c0 \uc54a\uc558\uace0 \uc778\uc2ec\uc744 \ud06c\uac8c \uc783\uc73c\ub2c8 \uc774\ub7ec\ud55c \uc790\uc5d0\uac8c \uc5f4\uc131\uc870\uc758 \uc720\uc5c5\uc744 \uc787\uac8c \ud560 \uc218 \uc5c6\uc73c\ub2c8 \ud568\uc548\uad81(\u54b8\u5b89\u5bae) \uc5d0 \uac00\ub450\uc5b4 \uc601\uc6d0\ud788 \uc11c\uc778\uc73c\ub85c \uc0bc\uaca0\ub2e4\ub294 \uc131\uc9c0\ub97c \ub0b4\ub838\ub2e4.", "paragraph_sentence": "\uac15\ud76c\uc81c\ub294 \uc724\uc789\uc774 \uc0c8\ub85c\uc6b4 \uc0ac\ub78c\uc774 \ub418\uae38 \ubc14\ub790\uc73c\ub098 \uacb0\uad6d \uc724\uc789\uc740 \ub3cc\uc774\ud0ac \uc218 \uc5c6\ub294 \uc5ed\ubaa8\uc758 \uc8fc\ub3d9\uc790\uac00 \ub418\uc5c8\ub2e4. \uac15\ud76c\uc81c\ub294 \uc989\uc2dc \uadc0\uacbd\ud558\uc5ec \ud0c1\ud569\uc81c\uc640 \uacbd\uc561, \uc81c\uc138\ubb34 \ub4f1\uc744 \uad50\uc0ac\ud558\uace0 \ub3c4\ud1b5 \uc545\uc120(\u9102\u7e55)\uc740 \uc720\ud3d0\ub41c \ud6c4 \uc548\uce5c\uc655 \uc545\ub77d(\u5b89\u89aa\u738b \u5cb3\u6a02)\uc758 \ubb18\uc18c\uc758 \ub2a5\uc9c0\uae30\ub85c \ubcf4\ub0c8\ub2e4. \ud669\uc81c\uc640 \ud669\ud0dc\uc790\uac00 \uc624\ub7ab\ub3d9\uc548 \uac19\uc774 \uc874\uc7ac\ud558\uba74\uc11c \ud669\ud0dc\uc790\uac00 \uc870\uc815\uc5d0\uc11c \uc81c2\uc758 \ud669\uc81c\ub85c\uc11c \ud669\uad8c\uc744 \uc800\uc9c0\ud560 \uac00\uc7a5 \uac15\ub825\ud55c \uc7a5\uc560\ubb3c\uc774\ub77c\uace0 \uc0dd\uac01\ud55c \uac15\ud76c\uc81c\ub294 \ub2e4\uc2dc \uc724\uc789\uc744 \ud3d0\ud560 \uacb0\uc2ec\uc744 \uad73\ud788\uace0 1712\ub144(\uac15\ud76c 51\ub144) \uc74c\ub825 9\uc6d4 30\uc77c, \uac15\ud76c\uc81c\ub294 \uc724\uc789\uc744 \uc7a1\uc544\ub4e4\uc774\uace0 \ubcf5\uc704\ud55c \uc774\ub798 \uad11\uc99d\uc774 \uc544\uc9c1 \uce58\uc720\ub418\uc9c0 \uc54a\uc558\uace0 \uc778\uc2ec\uc744 \ud06c\uac8c \uc783\uc73c\ub2c8 \uc774\ub7ec\ud55c \uc790\uc5d0\uac8c \uc5f4\uc131\uc870\uc758 \uc720\uc5c5\uc744 \uc787\uac8c \ud560 \uc218 \uc5c6\uc73c\ub2c8 \ud568\uc548\uad81(\u54b8\u5b89\u5bae) \uc5d0 \uac00\ub450\uc5b4 \uc601\uc6d0\ud788 \uc11c\uc778\uc73c\ub85c \uc0bc\uaca0\ub2e4\ub294 \uc131\uc9c0\ub97c \ub0b4\ub838\ub2e4. \uc774\ud6c4\ub85c\ub294 \uc0dd\uc804\uc5d0 \ud669\ud0dc\uc790\ub97c \ubf51\uc9c0 \uc54a\uc744 \uac83\uc784\uc744 \uacf5\uc5b8\ud558\uace0 \uc77c\uccb4\uc758 \ud669\ud0dc\uc790 \ucc45\ubd09\uc5d0 \uad00\ud55c \uc870\uc815\uc758 \uacf5\ub860\ud654\ub97c \ucc28\ub2e8\ud558\uc600\ub2e4. \ub354\ubd88\uc5b4 \uac15\ud76c\uc81c\ub294 \ub9cc\uc57d \uc774\ub7ec\ud55c \uc758\uacac\uc774 \uc870\uc815\uc5d0\uc11c \ub098\uc62c \uc2dc, \uc9c0\uc704 \uace0\ud558\ub97c \ub9c9\ub860\ud558\uace0 \ucc38\ud615\uc5d0 \ucc98\ud558\uaca0\ub2e4\ub294 \uc131\uc9c0\ub97c \ud55c\ubc88 \ub354 \ub0b4\ub838\ub2e4.", "paragraph_answer": "\uac15\ud76c\uc81c\ub294 \uc724\uc789\uc774 \uc0c8\ub85c\uc6b4 \uc0ac\ub78c\uc774 \ub418\uae38 \ubc14\ub790\uc73c\ub098 \uacb0\uad6d \uc724\uc789\uc740 \ub3cc\uc774\ud0ac \uc218 \uc5c6\ub294 \uc5ed\ubaa8\uc758 \uc8fc\ub3d9\uc790\uac00 \ub418\uc5c8\ub2e4. \uac15\ud76c\uc81c\ub294 \uc989\uc2dc \uadc0\uacbd\ud558\uc5ec \ud0c1\ud569\uc81c\uc640 \uacbd\uc561, \uc81c\uc138\ubb34 \ub4f1\uc744 \uad50\uc0ac\ud558\uace0 \ub3c4\ud1b5 \uc545\uc120(\u9102\u7e55)\uc740 \uc720\ud3d0\ub41c \ud6c4 \uc548\uce5c\uc655 \uc545\ub77d(\u5b89\u89aa\u738b \u5cb3\u6a02)\uc758 \ubb18\uc18c\uc758 \ub2a5\uc9c0\uae30\ub85c \ubcf4\ub0c8\ub2e4. \ud669\uc81c\uc640 \ud669\ud0dc\uc790\uac00 \uc624\ub7ab\ub3d9\uc548 \uac19\uc774 \uc874\uc7ac\ud558\uba74\uc11c \ud669\ud0dc\uc790\uac00 \uc870\uc815\uc5d0\uc11c \uc81c2\uc758 \ud669\uc81c\ub85c\uc11c \ud669\uad8c\uc744 \uc800\uc9c0\ud560 \uac00\uc7a5 \uac15\ub825\ud55c \uc7a5\uc560\ubb3c\uc774\ub77c\uace0 \uc0dd\uac01\ud55c \uac15\ud76c\uc81c\ub294 \ub2e4\uc2dc \uc724\uc789\uc744 \ud3d0\ud560 \uacb0\uc2ec\uc744 \uad73\ud788\uace0 1712\ub144(\uac15\ud76c 51\ub144) \uc74c\ub825 9\uc6d4 30\uc77c, \uac15\ud76c\uc81c\ub294 \uc724\uc789\uc744 \uc7a1\uc544\ub4e4\uc774\uace0 \ubcf5\uc704\ud55c \uc774\ub798 \uad11\uc99d\uc774 \uc544\uc9c1 \uce58\uc720\ub418\uc9c0 \uc54a\uc558\uace0 \uc778\uc2ec\uc744 \ud06c\uac8c \uc783\uc73c\ub2c8 \uc774\ub7ec\ud55c \uc790\uc5d0\uac8c \uc5f4\uc131\uc870\uc758 \uc720\uc5c5\uc744 \uc787\uac8c \ud560 \uc218 \uc5c6\uc73c\ub2c8 \ud568\uc548\uad81(\u54b8\u5b89\u5bae) \uc5d0 \uac00\ub450\uc5b4 \uc601\uc6d0\ud788 \uc11c\uc778\uc73c\ub85c \uc0bc\uaca0\ub2e4\ub294 \uc131\uc9c0\ub97c \ub0b4\ub838\ub2e4. \uc774\ud6c4\ub85c\ub294 \uc0dd\uc804\uc5d0 \ud669\ud0dc\uc790\ub97c \ubf51\uc9c0 \uc54a\uc744 \uac83\uc784\uc744 \uacf5\uc5b8\ud558\uace0 \uc77c\uccb4\uc758 \ud669\ud0dc\uc790 \ucc45\ubd09\uc5d0 \uad00\ud55c \uc870\uc815\uc758 \uacf5\ub860\ud654\ub97c \ucc28\ub2e8\ud558\uc600\ub2e4. \ub354\ubd88\uc5b4 \uac15\ud76c\uc81c\ub294 \ub9cc\uc57d \uc774\ub7ec\ud55c \uc758\uacac\uc774 \uc870\uc815\uc5d0\uc11c \ub098\uc62c \uc2dc, \uc9c0\uc704 \uace0\ud558\ub97c \ub9c9\ub860\ud558\uace0 \ucc38\ud615\uc5d0 \ucc98\ud558\uaca0\ub2e4\ub294 \uc131\uc9c0\ub97c \ud55c\ubc88 \ub354 \ub0b4\ub838\ub2e4.", "sentence_answer": "\ud669\uc81c\uc640 \ud669\ud0dc\uc790\uac00 \uc624\ub7ab\ub3d9\uc548 \uac19\uc774 \uc874\uc7ac\ud558\uba74\uc11c \ud669\ud0dc\uc790\uac00 \uc870\uc815\uc5d0\uc11c \uc81c2\uc758 \ud669\uc81c\ub85c\uc11c \ud669\uad8c\uc744 \uc800\uc9c0\ud560 \uac00\uc7a5 \uac15\ub825\ud55c \uc7a5\uc560\ubb3c\uc774\ub77c\uace0 \uc0dd\uac01\ud55c \uac15\ud76c\uc81c\ub294 \ub2e4\uc2dc \uc724\uc789\uc744 \ud3d0\ud560 \uacb0\uc2ec\uc744 \uad73\ud788\uace0 1712\ub144(\uac15\ud76c 51\ub144) \uc74c\ub825 9\uc6d4 30\uc77c, \uac15\ud76c\uc81c\ub294 \uc724\uc789\uc744 \uc7a1\uc544\ub4e4\uc774\uace0 \ubcf5\uc704\ud55c \uc774\ub798 \uad11\uc99d\uc774 \uc544\uc9c1 \uce58\uc720\ub418\uc9c0 \uc54a\uc558\uace0 \uc778\uc2ec\uc744 \ud06c\uac8c \uc783\uc73c\ub2c8 \uc774\ub7ec\ud55c \uc790\uc5d0\uac8c \uc5f4\uc131\uc870\uc758 \uc720\uc5c5\uc744 \uc787\uac8c \ud560 \uc218 \uc5c6\uc73c\ub2c8 \ud568\uc548\uad81(\u54b8\u5b89\u5bae) \uc5d0 \uac00\ub450\uc5b4 \uc601\uc6d0\ud788 \uc11c\uc778\uc73c\ub85c \uc0bc\uaca0\ub2e4\ub294 \uc131\uc9c0\ub97c \ub0b4\ub838\ub2e4.", "paragraph_id": "5835795-6-1", "paragraph_question": "question: \uc5b4\ub514\uc5d0 \uac00\ub450\uc5c8\ub294\uac00?, context: \uac15\ud76c\uc81c\ub294 \uc724\uc789\uc774 \uc0c8\ub85c\uc6b4 \uc0ac\ub78c\uc774 \ub418\uae38 \ubc14\ub790\uc73c\ub098 \uacb0\uad6d \uc724\uc789\uc740 \ub3cc\uc774\ud0ac \uc218 \uc5c6\ub294 \uc5ed\ubaa8\uc758 \uc8fc\ub3d9\uc790\uac00 \ub418\uc5c8\ub2e4. \uac15\ud76c\uc81c\ub294 \uc989\uc2dc \uadc0\uacbd\ud558\uc5ec \ud0c1\ud569\uc81c\uc640 \uacbd\uc561, \uc81c\uc138\ubb34 \ub4f1\uc744 \uad50\uc0ac\ud558\uace0 \ub3c4\ud1b5 \uc545\uc120(\u9102\u7e55)\uc740 \uc720\ud3d0\ub41c \ud6c4 \uc548\uce5c\uc655 \uc545\ub77d(\u5b89\u89aa\u738b \u5cb3\u6a02)\uc758 \ubb18\uc18c\uc758 \ub2a5\uc9c0\uae30\ub85c \ubcf4\ub0c8\ub2e4. \ud669\uc81c\uc640 \ud669\ud0dc\uc790\uac00 \uc624\ub7ab\ub3d9\uc548 \uac19\uc774 \uc874\uc7ac\ud558\uba74\uc11c \ud669\ud0dc\uc790\uac00 \uc870\uc815\uc5d0\uc11c \uc81c2\uc758 \ud669\uc81c\ub85c\uc11c \ud669\uad8c\uc744 \uc800\uc9c0\ud560 \uac00\uc7a5 \uac15\ub825\ud55c \uc7a5\uc560\ubb3c\uc774\ub77c\uace0 \uc0dd\uac01\ud55c \uac15\ud76c\uc81c\ub294 \ub2e4\uc2dc \uc724\uc789\uc744 \ud3d0\ud560 \uacb0\uc2ec\uc744 \uad73\ud788\uace0 1712\ub144(\uac15\ud76c 51\ub144) \uc74c\ub825 9\uc6d4 30\uc77c, \uac15\ud76c\uc81c\ub294 \uc724\uc789\uc744 \uc7a1\uc544\ub4e4\uc774\uace0 \ubcf5\uc704\ud55c \uc774\ub798 \uad11\uc99d\uc774 \uc544\uc9c1 \uce58\uc720\ub418\uc9c0 \uc54a\uc558\uace0 \uc778\uc2ec\uc744 \ud06c\uac8c \uc783\uc73c\ub2c8 \uc774\ub7ec\ud55c \uc790\uc5d0\uac8c \uc5f4\uc131\uc870\uc758 \uc720\uc5c5\uc744 \uc787\uac8c \ud560 \uc218 \uc5c6\uc73c\ub2c8 \ud568\uc548\uad81(\u54b8\u5b89\u5bae)\uc5d0 \uac00\ub450\uc5b4 \uc601\uc6d0\ud788 \uc11c\uc778\uc73c\ub85c \uc0bc\uaca0\ub2e4\ub294 \uc131\uc9c0\ub97c \ub0b4\ub838\ub2e4. \uc774\ud6c4\ub85c\ub294 \uc0dd\uc804\uc5d0 \ud669\ud0dc\uc790\ub97c \ubf51\uc9c0 \uc54a\uc744 \uac83\uc784\uc744 \uacf5\uc5b8\ud558\uace0 \uc77c\uccb4\uc758 \ud669\ud0dc\uc790 \ucc45\ubd09\uc5d0 \uad00\ud55c \uc870\uc815\uc758 \uacf5\ub860\ud654\ub97c \ucc28\ub2e8\ud558\uc600\ub2e4. \ub354\ubd88\uc5b4 \uac15\ud76c\uc81c\ub294 \ub9cc\uc57d \uc774\ub7ec\ud55c \uc758\uacac\uc774 \uc870\uc815\uc5d0\uc11c \ub098\uc62c \uc2dc, \uc9c0\uc704 \uace0\ud558\ub97c \ub9c9\ub860\ud558\uace0 \ucc38\ud615\uc5d0 \ucc98\ud558\uaca0\ub2e4\ub294 \uc131\uc9c0\ub97c \ud55c\ubc88 \ub354 \ub0b4\ub838\ub2e4."} {"question": "\ub9ac\uba54\uc774\ud06c\uace1\uc778 'Rhythm of the Rain'\uc758 \uc7ac\ud604\ubd80\uc5d0 rain\uc744 \uc0bd\uc785\ud55c \uac00\uc218\ub294?", "paragraph": "\u3008Rain\u3009\uc740 \ud398\ud234\ub77c \ud074\ub77c\ud06c, \uc544\uc774\ubca1\uc2a4, \ubd09\uc6cc\ud130, \ud5d8\ube14 \ud30c\uc774, \ub354 \uc7bc, \ub354 \ud074\ub798\uc2dc, \uc1fc\ub128 \ub098\uc774\ud504, \ud38c\ud0a8\uc2a4, \uac24\ub7ed\uc2dc 500, \ud558\ub4dc \ubbf8\ud2b8, \ud3f4\ub9ac\ub85d, \uadf8\ub809 \uc62c\uba3c, \uc548\ud2f0\ud0ec, \uc655 \uccad \ub4f1\uc758 \uac00\uc218\uc640 \ubc34\ub4dc\uac00 \ub9ac\uba54\uc774\ud06c\ud588\ub2e4. \uadf8\ub808\uc774\ud2b8\ud480 \ub370\ub4dc\ub294 80\ub144\ub300\uc5d0 \uc559\ucf54\ub974\ub85c \uc774 \uace1\uc744 \uc885\uc885 \uacf5\uc5f0\ud558\uae30\ub3c4 \ud588\ub2e4. U2\ub3c4 \ud22c\uc5b4\ub97c \uc9c4\ud589\ud560 \ub54c\ub9c8\ub2e4 \uc774 \uace1 \uc804\uccb4\ub098 \uc77c\ubd80\ub97c \uacf5\uc5f0\ud558\uc600\uc73c\uba70, \uc57c\uc678 \ucf58\uc11c\ud2b8\uc5d0\uc11c \ube44\uac00 \ub0b4\ub9ac\uae30 \uc2dc\uc791\ud558\uba74 \ubd80\ub97c \ub54c\uac00 \ub9ce\uc558\ub2e4. \ud384 \uc7bc\uc740 1992\ub144 \ud551\ud06c\ud31d \ud398\uc2a4\ud2f0\ubc8c \uc1fc\uc5d0\uc11c \u3008Jeremy\u3009\ub97c \uacf5\uc5f0\ud560 \ub54c \u3008Rain\u3009\uc744 \uc989\ud765\uc801\uc73c\ub85c \ub123\uc5b4\uc11c \ubd88\ub800\uace0, 2012\ub144 \uc544\uc77c\uc624\ube0c\uc640\uc774\ud2b8 \ud398\uc2a4\ud2f0\ubc8c\uc5d0\uc11c\ub294 \uace1 \uc804\uccb4\ub97c \ubd88\ub800\ub2e4. \ucfe8\ub77c \uc170\uc774\ucee4\ub294 1966\ub144 \ub808\ub529 \ud398\uc2a4\ud2f0\ubc8c\uc5d0\uc11c \ub9ac\uba54\uc774\ud06c\ud588\uace0, \ud398\uc5b4\ud3ec\ud2b8 \ucee8\ubca4\uc158\uc740 1997\ub144 \ud06c\ub85c\ud2b8\ub808\ub514 \ud398\uc2a4\ud2f0\ubc8c\uc5d0\uc11c \ub304 \uc54c \ube0c\ub798\uc988\uc640 \ud568\uaed8 \ub9ac\uba54\uc774\ud06c\ud55c \uc801\uc774 \uc788\ub2e4. \ud1a0\ub4dc \ub8ec\ub4dc\uadf8\ub80c\ub3c4 \uc774 \uace1\uc744 \ub9ac\uba54\uc774\ud06c\ud55c \ubc14 \uc788\uc73c\uba70 \ub304 \ud3ec\uae00\ubc84\uadf8\ub294 \ud6c4\uc77c \uc790\uc2e0\uc758 \ub9ac\uba54\uc774\ud06c\uace1\u3008Rhythm of the Rain\u3009\uc758 \uc7ac\ud604\ubd80\uc5d0 \uc774 \uace1\uc744 \uc0bd\uc785\ud558\uae30\ub3c4 \ud588\ub2e4.", "answer": "\ub304 \ud3ec\uae00\ubc84\uadf8", "sentence": "\ud1a0\ub4dc \ub8ec\ub4dc\uadf8\ub80c\ub3c4 \uc774 \uace1\uc744 \ub9ac\uba54\uc774\ud06c\ud55c \ubc14 \uc788\uc73c\uba70 \ub304 \ud3ec\uae00\ubc84\uadf8 \ub294 \ud6c4\uc77c \uc790\uc2e0\uc758 \ub9ac\uba54\uc774\ud06c\uace1\u3008Rhythm of the Rain\u3009\uc758 \uc7ac\ud604\ubd80\uc5d0 \uc774 \uace1\uc744 \uc0bd\uc785\ud558\uae30\ub3c4 \ud588\ub2e4.", "paragraph_sentence": "\u3008Rain\u3009\uc740 \ud398\ud234\ub77c \ud074\ub77c\ud06c, \uc544\uc774\ubca1\uc2a4, \ubd09\uc6cc\ud130, \ud5d8\ube14 \ud30c\uc774, \ub354 \uc7bc, \ub354 \ud074\ub798\uc2dc, \uc1fc\ub128 \ub098\uc774\ud504, \ud38c\ud0a8\uc2a4, \uac24\ub7ed\uc2dc 500, \ud558\ub4dc \ubbf8\ud2b8, \ud3f4\ub9ac\ub85d, \uadf8\ub809 \uc62c\uba3c, \uc548\ud2f0\ud0ec, \uc655 \uccad \ub4f1\uc758 \uac00\uc218\uc640 \ubc34\ub4dc\uac00 \ub9ac\uba54\uc774\ud06c\ud588\ub2e4. \uadf8\ub808\uc774\ud2b8\ud480 \ub370\ub4dc\ub294 80\ub144\ub300\uc5d0 \uc559\ucf54\ub974\ub85c \uc774 \uace1\uc744 \uc885\uc885 \uacf5\uc5f0\ud558\uae30\ub3c4 \ud588\ub2e4. U2\ub3c4 \ud22c\uc5b4\ub97c \uc9c4\ud589\ud560 \ub54c\ub9c8\ub2e4 \uc774 \uace1 \uc804\uccb4\ub098 \uc77c\ubd80\ub97c \uacf5\uc5f0\ud558\uc600\uc73c\uba70, \uc57c\uc678 \ucf58\uc11c\ud2b8\uc5d0\uc11c \ube44\uac00 \ub0b4\ub9ac\uae30 \uc2dc\uc791\ud558\uba74 \ubd80\ub97c \ub54c\uac00 \ub9ce\uc558\ub2e4. \ud384 \uc7bc\uc740 1992\ub144 \ud551\ud06c\ud31d \ud398\uc2a4\ud2f0\ubc8c \uc1fc\uc5d0\uc11c \u3008Jeremy\u3009\ub97c \uacf5\uc5f0\ud560 \ub54c \u3008Rain\u3009\uc744 \uc989\ud765\uc801\uc73c\ub85c \ub123\uc5b4\uc11c \ubd88\ub800\uace0, 2012\ub144 \uc544\uc77c\uc624\ube0c\uc640\uc774\ud2b8 \ud398\uc2a4\ud2f0\ubc8c\uc5d0\uc11c\ub294 \uace1 \uc804\uccb4\ub97c \ubd88\ub800\ub2e4. \ucfe8\ub77c \uc170\uc774\ucee4\ub294 1966\ub144 \ub808\ub529 \ud398\uc2a4\ud2f0\ubc8c\uc5d0\uc11c \ub9ac\uba54\uc774\ud06c\ud588\uace0, \ud398\uc5b4\ud3ec\ud2b8 \ucee8\ubca4\uc158\uc740 1997\ub144 \ud06c\ub85c\ud2b8\ub808\ub514 \ud398\uc2a4\ud2f0\ubc8c\uc5d0\uc11c \ub304 \uc54c \ube0c\ub798\uc988\uc640 \ud568\uaed8 \ub9ac\uba54\uc774\ud06c\ud55c \uc801\uc774 \uc788\ub2e4. \ud1a0\ub4dc \ub8ec\ub4dc\uadf8\ub80c\ub3c4 \uc774 \uace1\uc744 \ub9ac\uba54\uc774\ud06c\ud55c \ubc14 \uc788\uc73c\uba70 \ub304 \ud3ec\uae00\ubc84\uadf8 \ub294 \ud6c4\uc77c \uc790\uc2e0\uc758 \ub9ac\uba54\uc774\ud06c\uace1\u3008Rhythm of the Rain\u3009\uc758 \uc7ac\ud604\ubd80\uc5d0 \uc774 \uace1\uc744 \uc0bd\uc785\ud558\uae30\ub3c4 \ud588\ub2e4. ", "paragraph_answer": "\u3008Rain\u3009\uc740 \ud398\ud234\ub77c \ud074\ub77c\ud06c, \uc544\uc774\ubca1\uc2a4, \ubd09\uc6cc\ud130, \ud5d8\ube14 \ud30c\uc774, \ub354 \uc7bc, \ub354 \ud074\ub798\uc2dc, \uc1fc\ub128 \ub098\uc774\ud504, \ud38c\ud0a8\uc2a4, \uac24\ub7ed\uc2dc 500, \ud558\ub4dc \ubbf8\ud2b8, \ud3f4\ub9ac\ub85d, \uadf8\ub809 \uc62c\uba3c, \uc548\ud2f0\ud0ec, \uc655 \uccad \ub4f1\uc758 \uac00\uc218\uc640 \ubc34\ub4dc\uac00 \ub9ac\uba54\uc774\ud06c\ud588\ub2e4. \uadf8\ub808\uc774\ud2b8\ud480 \ub370\ub4dc\ub294 80\ub144\ub300\uc5d0 \uc559\ucf54\ub974\ub85c \uc774 \uace1\uc744 \uc885\uc885 \uacf5\uc5f0\ud558\uae30\ub3c4 \ud588\ub2e4. U2\ub3c4 \ud22c\uc5b4\ub97c \uc9c4\ud589\ud560 \ub54c\ub9c8\ub2e4 \uc774 \uace1 \uc804\uccb4\ub098 \uc77c\ubd80\ub97c \uacf5\uc5f0\ud558\uc600\uc73c\uba70, \uc57c\uc678 \ucf58\uc11c\ud2b8\uc5d0\uc11c \ube44\uac00 \ub0b4\ub9ac\uae30 \uc2dc\uc791\ud558\uba74 \ubd80\ub97c \ub54c\uac00 \ub9ce\uc558\ub2e4. \ud384 \uc7bc\uc740 1992\ub144 \ud551\ud06c\ud31d \ud398\uc2a4\ud2f0\ubc8c \uc1fc\uc5d0\uc11c \u3008Jeremy\u3009\ub97c \uacf5\uc5f0\ud560 \ub54c \u3008Rain\u3009\uc744 \uc989\ud765\uc801\uc73c\ub85c \ub123\uc5b4\uc11c \ubd88\ub800\uace0, 2012\ub144 \uc544\uc77c\uc624\ube0c\uc640\uc774\ud2b8 \ud398\uc2a4\ud2f0\ubc8c\uc5d0\uc11c\ub294 \uace1 \uc804\uccb4\ub97c \ubd88\ub800\ub2e4. \ucfe8\ub77c \uc170\uc774\ucee4\ub294 1966\ub144 \ub808\ub529 \ud398\uc2a4\ud2f0\ubc8c\uc5d0\uc11c \ub9ac\uba54\uc774\ud06c\ud588\uace0, \ud398\uc5b4\ud3ec\ud2b8 \ucee8\ubca4\uc158\uc740 1997\ub144 \ud06c\ub85c\ud2b8\ub808\ub514 \ud398\uc2a4\ud2f0\ubc8c\uc5d0\uc11c \ub304 \uc54c \ube0c\ub798\uc988\uc640 \ud568\uaed8 \ub9ac\uba54\uc774\ud06c\ud55c \uc801\uc774 \uc788\ub2e4. \ud1a0\ub4dc \ub8ec\ub4dc\uadf8\ub80c\ub3c4 \uc774 \uace1\uc744 \ub9ac\uba54\uc774\ud06c\ud55c \ubc14 \uc788\uc73c\uba70 \ub304 \ud3ec\uae00\ubc84\uadf8 \ub294 \ud6c4\uc77c \uc790\uc2e0\uc758 \ub9ac\uba54\uc774\ud06c\uace1\u3008Rhythm of the Rain\u3009\uc758 \uc7ac\ud604\ubd80\uc5d0 \uc774 \uace1\uc744 \uc0bd\uc785\ud558\uae30\ub3c4 \ud588\ub2e4.", "sentence_answer": "\ud1a0\ub4dc \ub8ec\ub4dc\uadf8\ub80c\ub3c4 \uc774 \uace1\uc744 \ub9ac\uba54\uc774\ud06c\ud55c \ubc14 \uc788\uc73c\uba70 \ub304 \ud3ec\uae00\ubc84\uadf8 \ub294 \ud6c4\uc77c \uc790\uc2e0\uc758 \ub9ac\uba54\uc774\ud06c\uace1\u3008Rhythm of the Rain\u3009\uc758 \uc7ac\ud604\ubd80\uc5d0 \uc774 \uace1\uc744 \uc0bd\uc785\ud558\uae30\ub3c4 \ud588\ub2e4.", "paragraph_id": "6472313-4-1", "paragraph_question": "question: \ub9ac\uba54\uc774\ud06c\uace1\uc778 'Rhythm of the Rain'\uc758 \uc7ac\ud604\ubd80\uc5d0 rain\uc744 \uc0bd\uc785\ud55c \uac00\uc218\ub294?, context: \u3008Rain\u3009\uc740 \ud398\ud234\ub77c \ud074\ub77c\ud06c, \uc544\uc774\ubca1\uc2a4, \ubd09\uc6cc\ud130, \ud5d8\ube14 \ud30c\uc774, \ub354 \uc7bc, \ub354 \ud074\ub798\uc2dc, \uc1fc\ub128 \ub098\uc774\ud504, \ud38c\ud0a8\uc2a4, \uac24\ub7ed\uc2dc 500, \ud558\ub4dc \ubbf8\ud2b8, \ud3f4\ub9ac\ub85d, \uadf8\ub809 \uc62c\uba3c, \uc548\ud2f0\ud0ec, \uc655 \uccad \ub4f1\uc758 \uac00\uc218\uc640 \ubc34\ub4dc\uac00 \ub9ac\uba54\uc774\ud06c\ud588\ub2e4. \uadf8\ub808\uc774\ud2b8\ud480 \ub370\ub4dc\ub294 80\ub144\ub300\uc5d0 \uc559\ucf54\ub974\ub85c \uc774 \uace1\uc744 \uc885\uc885 \uacf5\uc5f0\ud558\uae30\ub3c4 \ud588\ub2e4. U2\ub3c4 \ud22c\uc5b4\ub97c \uc9c4\ud589\ud560 \ub54c\ub9c8\ub2e4 \uc774 \uace1 \uc804\uccb4\ub098 \uc77c\ubd80\ub97c \uacf5\uc5f0\ud558\uc600\uc73c\uba70, \uc57c\uc678 \ucf58\uc11c\ud2b8\uc5d0\uc11c \ube44\uac00 \ub0b4\ub9ac\uae30 \uc2dc\uc791\ud558\uba74 \ubd80\ub97c \ub54c\uac00 \ub9ce\uc558\ub2e4. \ud384 \uc7bc\uc740 1992\ub144 \ud551\ud06c\ud31d \ud398\uc2a4\ud2f0\ubc8c \uc1fc\uc5d0\uc11c \u3008Jeremy\u3009\ub97c \uacf5\uc5f0\ud560 \ub54c \u3008Rain\u3009\uc744 \uc989\ud765\uc801\uc73c\ub85c \ub123\uc5b4\uc11c \ubd88\ub800\uace0, 2012\ub144 \uc544\uc77c\uc624\ube0c\uc640\uc774\ud2b8 \ud398\uc2a4\ud2f0\ubc8c\uc5d0\uc11c\ub294 \uace1 \uc804\uccb4\ub97c \ubd88\ub800\ub2e4. \ucfe8\ub77c \uc170\uc774\ucee4\ub294 1966\ub144 \ub808\ub529 \ud398\uc2a4\ud2f0\ubc8c\uc5d0\uc11c \ub9ac\uba54\uc774\ud06c\ud588\uace0, \ud398\uc5b4\ud3ec\ud2b8 \ucee8\ubca4\uc158\uc740 1997\ub144 \ud06c\ub85c\ud2b8\ub808\ub514 \ud398\uc2a4\ud2f0\ubc8c\uc5d0\uc11c \ub304 \uc54c \ube0c\ub798\uc988\uc640 \ud568\uaed8 \ub9ac\uba54\uc774\ud06c\ud55c \uc801\uc774 \uc788\ub2e4. \ud1a0\ub4dc \ub8ec\ub4dc\uadf8\ub80c\ub3c4 \uc774 \uace1\uc744 \ub9ac\uba54\uc774\ud06c\ud55c \ubc14 \uc788\uc73c\uba70 \ub304 \ud3ec\uae00\ubc84\uadf8\ub294 \ud6c4\uc77c \uc790\uc2e0\uc758 \ub9ac\uba54\uc774\ud06c\uace1\u3008Rhythm of the Rain\u3009\uc758 \uc7ac\ud604\ubd80\uc5d0 \uc774 \uace1\uc744 \uc0bd\uc785\ud558\uae30\ub3c4 \ud588\ub2e4."} {"question": "\uac01\uad6d \uacf5\uc0ac\uad00\ub4e4\uc774 \uc815\ub3d9\uc5d0 \ub4e4\uc5b4\uc624\uae30 \uc2dc\uc791\ud588\ub358 \uc2dc\uae30\ub294?", "paragraph": "19\uc138\uae30 \ub9d0\ubd80\ud130 \uac01\uad6d \uacf5\uc0ac\uad00\ub4e4\uc774 \uc815\ub3d9\uc5d0 \ub4e4\uc5b4\uc624\uae30 \uc2dc\uc791\ud588\ub2e4. \uc774\uc21c\uc6b0\uc758 \ucc45\uc5d0\ub294 \uacfc\uac70 \uc815\ub3d9\uc5d0 \uc788\uc5c8\uac70\ub098,\ud604\uc7ac\uae4c\uc9c0 \uc874\uc7ac\ud558\uace0 \uc788\ub294 \uacf5\uc0ac\uad00\ub4e4\uc5d0 \ub300\ud574 \uc790\uc138\ud788 \uc124\uba85\ub418\uc5b4 \uc788\ub2e4. \ubbf8\uad6d\uacf5\uc0ac\uad00(1883\ub144 5\uc6d4 \uac1c\uc124, \uc815\ub3d910\ubc88\uc9c0), \uc601\uad6d\uacf5\uc0ac\uad00(1884\ub144 4\uc6d4 \uac1c\uc124, \uc815\ub3d94\ubc88\uc9c0), \ub3c5\uc77c\uc601\uc0ac\uad00(1891\ub144 \ud6c4\ubc18, \uc11c\uc18c\ubb38\ub3d9 38\ubc88\uc9c0\uc774\uc804), \ub7ec\uc2dc\uc544\uacf5\uc0ac\uad00(1885\ub144 10\uc6d4 \uac1c\uc124, \uc815\ub3d915-1\ubc88\uc9c0), \ud504\ub791\uc2a4\uacf5\uc0ac\uad00(1889\ub144 10\uc6d4 \uc815\ub3d9 28\ubc88\uc9c0\uc774\uc804), \ubca8\uae30\uc5d0\uc601\uc0ac\uad00(1901\ub144 10\uc6d4 \uac1c\uc124, \uc815\ub3d9 16-1\ubc88\uc9c0), \uc774\ud0c8\ub9ac\uc544\uacf5\uc0ac\uad00(1902\ub144 11\uc6d4 \uc774\uc804), \uc678\uad50\uad00\uad6c\ub77d\ubd80(1894\ub144 \uc815\ucd08\uc2dd(\u5b9a\u790e\u5f0f), \uc815\ub3d9 17\ubc88\uc9c0) \ub4f1\uc774 \uc788\uc5c8\ub2e4. \ud604\uc7ac \uc11c\uc6b8\uc2dc\ub9bd\ubbf8\uc220\uad00\uc774 \uc790\ub9ac\ud55c \uc11c\uc18c\ubb38\ub3d9 38\ubc88\uc9c0 \uc77c\ub300\ub294 \uc721\uc601\uacf5\uc6d0\uc744 \uac70\uccd0 \ub3c5\uc77c\uc601\uc0ac\uad00\uc774 \uc788\ub358 \uc790\ub9ac\uc774\ub2e4. '\uad6c \ub7ec\uc2dc\uc544\uacf5\uc0ac\uad00'\uc740 1977\ub144 \uc0ac\uc801\uc73c\ub85c \uc9c0\uc815\ub418\uc5b4 \uc624\ub298\uc5d0 \uc774\ub974\uace0 \uc788\ub2e4. \uacfc\uac70 \ud504\ub791\uc2a4\uacf5\uc0ac\uad00\uc758 \uc790\ub9ac\uc5d4 \uc624\ub298\ub0a0 \ucc3d\ub355\uc5ec\uc911(\u660c\u5fb7\u5973\u4e2d)\uac00 \uc704\uce58\ud574\uc788\ub2e4. \uacfc\uac70 \ubca8\uae30\uc5d0 \uc601\uc0ac\uad00 \uc790\ub9ac(\uc815\ub3d916-1\ubc88\uc9c0)\ub294 \ud574\ubc29\uc774\ud6c4 \ud558\ub0a8\ud638\ud154\uc744 \uac70\uccd0 \uc624\ub298\ub0a0 \uce90\ub098\ub2e4 \ub300\uc0ac\uad00\uc774 \uc790\ub9ac\ud558\uace0 \uc788\ub2e4. \uc678\uad50\uad00\uad6c\ub77d\ubd80\ub294 1892\ub144 \uacb0\uc131\ub418\uc5b4 \ud504\ub791\uc2a4\uacf5\uc0ac\uad00 \uc778\uc811\uac00\uc625\uc5d0\uc11c \ud68c\ud569\uc744 \uac16\ub2e4\uac00, 1894\ub144 \uc815\ub3d9 17\ubc88\uc9c0\uc5d0 \uac74\ubb3c\uc744 \uc2e0\ucd95\ud558\uc600\ub2e4. \uc774\ud6c4 1923\ub144\uc5d0 \uc774 \uc790\ub9ac\ub97c \uc11c\uc6b8\uc678\uad6d\uc778\ud559\uad50\uac00 \ub9e4\uc785\ud558\uc5ec 1957\ub144\uae4c\uc9c0 \uc0ac\uc6a9\ud558\uc600\ub2e4. \ud604\uc7ac\ub294 \ud504\ub780\uce58\uc2a4\ucf54\uad50\uc721\ud68c\uad00(Francisco Education Center) \uac74\ubb3c\uc774 \uc790\ub9ac\ud558\uace0 \uc788\ub2e4. \ud604\uc7ac \ubbf8\uad6d\ub300\uc0ac\uad00\uc800(\uc815\ub3d9 10\ubc88\uc9c0), \uc601\uad6d\ub300\uc0ac\uad00(\uc815\ub3d9 4\ubc88\uc9c0), \uc131\uacf5\ud68c\ub300\uc131\ub2f9(\uc815\ub3d9 3\ubc88\uc9c0), \uc774\ud654\uc5ec\uace0 \uad6c\ub0b4(\uc815\ub3d9 29\ubc88\uc9c0, 30\ubc88\uc9c0, 31, 32\ubc88\uc9c0)\uac00 \uac70\uc758 \ubcc0\ub3d9\uc774 \uc5c6\uc774 \ub0a8\uc544\uc788\ub2e4.", "answer": "19\uc138\uae30 \ub9d0", "sentence": "19\uc138\uae30 \ub9d0 \ubd80\ud130 \uac01\uad6d \uacf5\uc0ac\uad00\ub4e4\uc774 \uc815\ub3d9\uc5d0 \ub4e4\uc5b4\uc624\uae30 \uc2dc\uc791\ud588\ub2e4.", "paragraph_sentence": " 19\uc138\uae30 \ub9d0 \ubd80\ud130 \uac01\uad6d \uacf5\uc0ac\uad00\ub4e4\uc774 \uc815\ub3d9\uc5d0 \ub4e4\uc5b4\uc624\uae30 \uc2dc\uc791\ud588\ub2e4. \uc774\uc21c\uc6b0\uc758 \ucc45\uc5d0\ub294 \uacfc\uac70 \uc815\ub3d9\uc5d0 \uc788\uc5c8\uac70\ub098,\ud604\uc7ac\uae4c\uc9c0 \uc874\uc7ac\ud558\uace0 \uc788\ub294 \uacf5\uc0ac\uad00\ub4e4\uc5d0 \ub300\ud574 \uc790\uc138\ud788 \uc124\uba85\ub418\uc5b4 \uc788\ub2e4. \ubbf8\uad6d\uacf5\uc0ac\uad00(1883\ub144 5\uc6d4 \uac1c\uc124, \uc815\ub3d910\ubc88\uc9c0), \uc601\uad6d\uacf5\uc0ac\uad00(1884\ub144 4\uc6d4 \uac1c\uc124, \uc815\ub3d94\ubc88\uc9c0), \ub3c5\uc77c\uc601\uc0ac\uad00(1891\ub144 \ud6c4\ubc18, \uc11c\uc18c\ubb38\ub3d9 38\ubc88\uc9c0\uc774\uc804), \ub7ec\uc2dc\uc544\uacf5\uc0ac\uad00(1885\ub144 10\uc6d4 \uac1c\uc124, \uc815\ub3d915-1\ubc88\uc9c0), \ud504\ub791\uc2a4\uacf5\uc0ac\uad00(1889\ub144 10\uc6d4 \uc815\ub3d9 28\ubc88\uc9c0\uc774\uc804), \ubca8\uae30\uc5d0\uc601\uc0ac\uad00(1901\ub144 10\uc6d4 \uac1c\uc124, \uc815\ub3d9 16-1\ubc88\uc9c0), \uc774\ud0c8\ub9ac\uc544\uacf5\uc0ac\uad00(1902\ub144 11\uc6d4 \uc774\uc804), \uc678\uad50\uad00\uad6c\ub77d\ubd80(1894\ub144 \uc815\ucd08\uc2dd(\u5b9a\u790e\u5f0f), \uc815\ub3d9 17\ubc88\uc9c0) \ub4f1\uc774 \uc788\uc5c8\ub2e4. \ud604\uc7ac \uc11c\uc6b8\uc2dc\ub9bd\ubbf8\uc220\uad00\uc774 \uc790\ub9ac\ud55c \uc11c\uc18c\ubb38\ub3d9 38\ubc88\uc9c0 \uc77c\ub300\ub294 \uc721\uc601\uacf5\uc6d0\uc744 \uac70\uccd0 \ub3c5\uc77c\uc601\uc0ac\uad00\uc774 \uc788\ub358 \uc790\ub9ac\uc774\ub2e4. '\uad6c \ub7ec\uc2dc\uc544\uacf5\uc0ac\uad00'\uc740 1977\ub144 \uc0ac\uc801\uc73c\ub85c \uc9c0\uc815\ub418\uc5b4 \uc624\ub298\uc5d0 \uc774\ub974\uace0 \uc788\ub2e4. \uacfc\uac70 \ud504\ub791\uc2a4\uacf5\uc0ac\uad00\uc758 \uc790\ub9ac\uc5d4 \uc624\ub298\ub0a0 \ucc3d\ub355\uc5ec\uc911(\u660c\u5fb7\u5973\u4e2d)\uac00 \uc704\uce58\ud574\uc788\ub2e4. \uacfc\uac70 \ubca8\uae30\uc5d0 \uc601\uc0ac\uad00 \uc790\ub9ac(\uc815\ub3d916-1\ubc88\uc9c0)\ub294 \ud574\ubc29\uc774\ud6c4 \ud558\ub0a8\ud638\ud154\uc744 \uac70\uccd0 \uc624\ub298\ub0a0 \uce90\ub098\ub2e4 \ub300\uc0ac\uad00\uc774 \uc790\ub9ac\ud558\uace0 \uc788\ub2e4. \uc678\uad50\uad00\uad6c\ub77d\ubd80\ub294 1892\ub144 \uacb0\uc131\ub418\uc5b4 \ud504\ub791\uc2a4\uacf5\uc0ac\uad00 \uc778\uc811\uac00\uc625\uc5d0\uc11c \ud68c\ud569\uc744 \uac16\ub2e4\uac00, 1894\ub144 \uc815\ub3d9 17\ubc88\uc9c0\uc5d0 \uac74\ubb3c\uc744 \uc2e0\ucd95\ud558\uc600\ub2e4. \uc774\ud6c4 1923\ub144\uc5d0 \uc774 \uc790\ub9ac\ub97c \uc11c\uc6b8\uc678\uad6d\uc778\ud559\uad50\uac00 \ub9e4\uc785\ud558\uc5ec 1957\ub144\uae4c\uc9c0 \uc0ac\uc6a9\ud558\uc600\ub2e4. \ud604\uc7ac\ub294 \ud504\ub780\uce58\uc2a4\ucf54\uad50\uc721\ud68c\uad00(Francisco Education Center) \uac74\ubb3c\uc774 \uc790\ub9ac\ud558\uace0 \uc788\ub2e4. \ud604\uc7ac \ubbf8\uad6d\ub300\uc0ac\uad00\uc800(\uc815\ub3d9 10\ubc88\uc9c0), \uc601\uad6d\ub300\uc0ac\uad00(\uc815\ub3d9 4\ubc88\uc9c0), \uc131\uacf5\ud68c\ub300\uc131\ub2f9(\uc815\ub3d9 3\ubc88\uc9c0), \uc774\ud654\uc5ec\uace0 \uad6c\ub0b4(\uc815\ub3d9 29\ubc88\uc9c0, 30\ubc88\uc9c0, 31, 32\ubc88\uc9c0)\uac00 \uac70\uc758 \ubcc0\ub3d9\uc774 \uc5c6\uc774 \ub0a8\uc544\uc788\ub2e4.", "paragraph_answer": " 19\uc138\uae30 \ub9d0 \ubd80\ud130 \uac01\uad6d \uacf5\uc0ac\uad00\ub4e4\uc774 \uc815\ub3d9\uc5d0 \ub4e4\uc5b4\uc624\uae30 \uc2dc\uc791\ud588\ub2e4. \uc774\uc21c\uc6b0\uc758 \ucc45\uc5d0\ub294 \uacfc\uac70 \uc815\ub3d9\uc5d0 \uc788\uc5c8\uac70\ub098,\ud604\uc7ac\uae4c\uc9c0 \uc874\uc7ac\ud558\uace0 \uc788\ub294 \uacf5\uc0ac\uad00\ub4e4\uc5d0 \ub300\ud574 \uc790\uc138\ud788 \uc124\uba85\ub418\uc5b4 \uc788\ub2e4. \ubbf8\uad6d\uacf5\uc0ac\uad00(1883\ub144 5\uc6d4 \uac1c\uc124, \uc815\ub3d910\ubc88\uc9c0), \uc601\uad6d\uacf5\uc0ac\uad00(1884\ub144 4\uc6d4 \uac1c\uc124, \uc815\ub3d94\ubc88\uc9c0), \ub3c5\uc77c\uc601\uc0ac\uad00(1891\ub144 \ud6c4\ubc18, \uc11c\uc18c\ubb38\ub3d9 38\ubc88\uc9c0\uc774\uc804), \ub7ec\uc2dc\uc544\uacf5\uc0ac\uad00(1885\ub144 10\uc6d4 \uac1c\uc124, \uc815\ub3d915-1\ubc88\uc9c0), \ud504\ub791\uc2a4\uacf5\uc0ac\uad00(1889\ub144 10\uc6d4 \uc815\ub3d9 28\ubc88\uc9c0\uc774\uc804), \ubca8\uae30\uc5d0\uc601\uc0ac\uad00(1901\ub144 10\uc6d4 \uac1c\uc124, \uc815\ub3d9 16-1\ubc88\uc9c0), \uc774\ud0c8\ub9ac\uc544\uacf5\uc0ac\uad00(1902\ub144 11\uc6d4 \uc774\uc804), \uc678\uad50\uad00\uad6c\ub77d\ubd80(1894\ub144 \uc815\ucd08\uc2dd(\u5b9a\u790e\u5f0f), \uc815\ub3d9 17\ubc88\uc9c0) \ub4f1\uc774 \uc788\uc5c8\ub2e4. \ud604\uc7ac \uc11c\uc6b8\uc2dc\ub9bd\ubbf8\uc220\uad00\uc774 \uc790\ub9ac\ud55c \uc11c\uc18c\ubb38\ub3d9 38\ubc88\uc9c0 \uc77c\ub300\ub294 \uc721\uc601\uacf5\uc6d0\uc744 \uac70\uccd0 \ub3c5\uc77c\uc601\uc0ac\uad00\uc774 \uc788\ub358 \uc790\ub9ac\uc774\ub2e4. '\uad6c \ub7ec\uc2dc\uc544\uacf5\uc0ac\uad00'\uc740 1977\ub144 \uc0ac\uc801\uc73c\ub85c \uc9c0\uc815\ub418\uc5b4 \uc624\ub298\uc5d0 \uc774\ub974\uace0 \uc788\ub2e4. \uacfc\uac70 \ud504\ub791\uc2a4\uacf5\uc0ac\uad00\uc758 \uc790\ub9ac\uc5d4 \uc624\ub298\ub0a0 \ucc3d\ub355\uc5ec\uc911(\u660c\u5fb7\u5973\u4e2d)\uac00 \uc704\uce58\ud574\uc788\ub2e4. \uacfc\uac70 \ubca8\uae30\uc5d0 \uc601\uc0ac\uad00 \uc790\ub9ac(\uc815\ub3d916-1\ubc88\uc9c0)\ub294 \ud574\ubc29\uc774\ud6c4 \ud558\ub0a8\ud638\ud154\uc744 \uac70\uccd0 \uc624\ub298\ub0a0 \uce90\ub098\ub2e4 \ub300\uc0ac\uad00\uc774 \uc790\ub9ac\ud558\uace0 \uc788\ub2e4. \uc678\uad50\uad00\uad6c\ub77d\ubd80\ub294 1892\ub144 \uacb0\uc131\ub418\uc5b4 \ud504\ub791\uc2a4\uacf5\uc0ac\uad00 \uc778\uc811\uac00\uc625\uc5d0\uc11c \ud68c\ud569\uc744 \uac16\ub2e4\uac00, 1894\ub144 \uc815\ub3d9 17\ubc88\uc9c0\uc5d0 \uac74\ubb3c\uc744 \uc2e0\ucd95\ud558\uc600\ub2e4. \uc774\ud6c4 1923\ub144\uc5d0 \uc774 \uc790\ub9ac\ub97c \uc11c\uc6b8\uc678\uad6d\uc778\ud559\uad50\uac00 \ub9e4\uc785\ud558\uc5ec 1957\ub144\uae4c\uc9c0 \uc0ac\uc6a9\ud558\uc600\ub2e4. \ud604\uc7ac\ub294 \ud504\ub780\uce58\uc2a4\ucf54\uad50\uc721\ud68c\uad00(Francisco Education Center) \uac74\ubb3c\uc774 \uc790\ub9ac\ud558\uace0 \uc788\ub2e4. \ud604\uc7ac \ubbf8\uad6d\ub300\uc0ac\uad00\uc800(\uc815\ub3d9 10\ubc88\uc9c0), \uc601\uad6d\ub300\uc0ac\uad00(\uc815\ub3d9 4\ubc88\uc9c0), \uc131\uacf5\ud68c\ub300\uc131\ub2f9(\uc815\ub3d9 3\ubc88\uc9c0), \uc774\ud654\uc5ec\uace0 \uad6c\ub0b4(\uc815\ub3d9 29\ubc88\uc9c0, 30\ubc88\uc9c0, 31, 32\ubc88\uc9c0)\uac00 \uac70\uc758 \ubcc0\ub3d9\uc774 \uc5c6\uc774 \ub0a8\uc544\uc788\ub2e4.", "sentence_answer": " 19\uc138\uae30 \ub9d0 \ubd80\ud130 \uac01\uad6d \uacf5\uc0ac\uad00\ub4e4\uc774 \uc815\ub3d9\uc5d0 \ub4e4\uc5b4\uc624\uae30 \uc2dc\uc791\ud588\ub2e4.", "paragraph_id": "6550027-2-1", "paragraph_question": "question: \uac01\uad6d \uacf5\uc0ac\uad00\ub4e4\uc774 \uc815\ub3d9\uc5d0 \ub4e4\uc5b4\uc624\uae30 \uc2dc\uc791\ud588\ub358 \uc2dc\uae30\ub294?, context: 19\uc138\uae30 \ub9d0\ubd80\ud130 \uac01\uad6d \uacf5\uc0ac\uad00\ub4e4\uc774 \uc815\ub3d9\uc5d0 \ub4e4\uc5b4\uc624\uae30 \uc2dc\uc791\ud588\ub2e4. \uc774\uc21c\uc6b0\uc758 \ucc45\uc5d0\ub294 \uacfc\uac70 \uc815\ub3d9\uc5d0 \uc788\uc5c8\uac70\ub098,\ud604\uc7ac\uae4c\uc9c0 \uc874\uc7ac\ud558\uace0 \uc788\ub294 \uacf5\uc0ac\uad00\ub4e4\uc5d0 \ub300\ud574 \uc790\uc138\ud788 \uc124\uba85\ub418\uc5b4 \uc788\ub2e4. \ubbf8\uad6d\uacf5\uc0ac\uad00(1883\ub144 5\uc6d4 \uac1c\uc124, \uc815\ub3d910\ubc88\uc9c0), \uc601\uad6d\uacf5\uc0ac\uad00(1884\ub144 4\uc6d4 \uac1c\uc124, \uc815\ub3d94\ubc88\uc9c0), \ub3c5\uc77c\uc601\uc0ac\uad00(1891\ub144 \ud6c4\ubc18, \uc11c\uc18c\ubb38\ub3d9 38\ubc88\uc9c0\uc774\uc804), \ub7ec\uc2dc\uc544\uacf5\uc0ac\uad00(1885\ub144 10\uc6d4 \uac1c\uc124, \uc815\ub3d915-1\ubc88\uc9c0), \ud504\ub791\uc2a4\uacf5\uc0ac\uad00(1889\ub144 10\uc6d4 \uc815\ub3d9 28\ubc88\uc9c0\uc774\uc804), \ubca8\uae30\uc5d0\uc601\uc0ac\uad00(1901\ub144 10\uc6d4 \uac1c\uc124, \uc815\ub3d9 16-1\ubc88\uc9c0), \uc774\ud0c8\ub9ac\uc544\uacf5\uc0ac\uad00(1902\ub144 11\uc6d4 \uc774\uc804), \uc678\uad50\uad00\uad6c\ub77d\ubd80(1894\ub144 \uc815\ucd08\uc2dd(\u5b9a\u790e\u5f0f), \uc815\ub3d9 17\ubc88\uc9c0) \ub4f1\uc774 \uc788\uc5c8\ub2e4. \ud604\uc7ac \uc11c\uc6b8\uc2dc\ub9bd\ubbf8\uc220\uad00\uc774 \uc790\ub9ac\ud55c \uc11c\uc18c\ubb38\ub3d9 38\ubc88\uc9c0 \uc77c\ub300\ub294 \uc721\uc601\uacf5\uc6d0\uc744 \uac70\uccd0 \ub3c5\uc77c\uc601\uc0ac\uad00\uc774 \uc788\ub358 \uc790\ub9ac\uc774\ub2e4. '\uad6c \ub7ec\uc2dc\uc544\uacf5\uc0ac\uad00'\uc740 1977\ub144 \uc0ac\uc801\uc73c\ub85c \uc9c0\uc815\ub418\uc5b4 \uc624\ub298\uc5d0 \uc774\ub974\uace0 \uc788\ub2e4. \uacfc\uac70 \ud504\ub791\uc2a4\uacf5\uc0ac\uad00\uc758 \uc790\ub9ac\uc5d4 \uc624\ub298\ub0a0 \ucc3d\ub355\uc5ec\uc911(\u660c\u5fb7\u5973\u4e2d)\uac00 \uc704\uce58\ud574\uc788\ub2e4. \uacfc\uac70 \ubca8\uae30\uc5d0 \uc601\uc0ac\uad00 \uc790\ub9ac(\uc815\ub3d916-1\ubc88\uc9c0)\ub294 \ud574\ubc29\uc774\ud6c4 \ud558\ub0a8\ud638\ud154\uc744 \uac70\uccd0 \uc624\ub298\ub0a0 \uce90\ub098\ub2e4 \ub300\uc0ac\uad00\uc774 \uc790\ub9ac\ud558\uace0 \uc788\ub2e4. \uc678\uad50\uad00\uad6c\ub77d\ubd80\ub294 1892\ub144 \uacb0\uc131\ub418\uc5b4 \ud504\ub791\uc2a4\uacf5\uc0ac\uad00 \uc778\uc811\uac00\uc625\uc5d0\uc11c \ud68c\ud569\uc744 \uac16\ub2e4\uac00, 1894\ub144 \uc815\ub3d9 17\ubc88\uc9c0\uc5d0 \uac74\ubb3c\uc744 \uc2e0\ucd95\ud558\uc600\ub2e4. \uc774\ud6c4 1923\ub144\uc5d0 \uc774 \uc790\ub9ac\ub97c \uc11c\uc6b8\uc678\uad6d\uc778\ud559\uad50\uac00 \ub9e4\uc785\ud558\uc5ec 1957\ub144\uae4c\uc9c0 \uc0ac\uc6a9\ud558\uc600\ub2e4. \ud604\uc7ac\ub294 \ud504\ub780\uce58\uc2a4\ucf54\uad50\uc721\ud68c\uad00(Francisco Education Center) \uac74\ubb3c\uc774 \uc790\ub9ac\ud558\uace0 \uc788\ub2e4. \ud604\uc7ac \ubbf8\uad6d\ub300\uc0ac\uad00\uc800(\uc815\ub3d9 10\ubc88\uc9c0), \uc601\uad6d\ub300\uc0ac\uad00(\uc815\ub3d9 4\ubc88\uc9c0), \uc131\uacf5\ud68c\ub300\uc131\ub2f9(\uc815\ub3d9 3\ubc88\uc9c0), \uc774\ud654\uc5ec\uace0 \uad6c\ub0b4(\uc815\ub3d9 29\ubc88\uc9c0, 30\ubc88\uc9c0, 31, 32\ubc88\uc9c0)\uac00 \uac70\uc758 \ubcc0\ub3d9\uc774 \uc5c6\uc774 \ub0a8\uc544\uc788\ub2e4."} {"question": "\uc9c0\uae08 \uc0b4\uc544\uc788\ub294 \uc6b8\ub9ac\uc778\ub4e4\ubcf4\ub2e4 \ub354 \ub9ce\uc740 \uc6b8\ub77c\uc778\ub4e4\uc744 \uc8fd\uc600\ub2e4\uace0 \ub9d0\ud55c \ub0a8\uc790\ub4e4\uc740 \ub204\uad6c\uc758 \uc544\ub4e4\ub4e4\uc778\uac00?", "paragraph": "\uc774\uba58 \ub9c8\ud558\uc758 \uc5b4\ub290 \ub0a0, \ucfe0 \ud6cc\ub9b0\uc740 \uce74\ud750\ubc14\ub4dc\uac00 \ud559\uc0dd\ub4e4\uc744 \uac00\ub974\uce58\ub294 \uac83\uc744 \uc6b0\uc5f0\ud788 \ub4e3\uac8c \ub418\uc5c8\ub2e4. \ud55c \ud559\uc0dd\uc774 \uce74\ud750\ubc14\ub4dc\uc5d0\uac8c \uc624\ub298\uc774 \uc5b4\ub5a4 \uc885\ub958\uc758 \uc0c1\uc11c\ub85c\uc6b4 \ub0a0\uc774\ub0d0\uace0 \ubb3c\uc5c8\ub2e4. \uce74\ud750\ubc14\ub4dc\ub294 \uc624\ub298 \ubb34\uae30\ub97c \uac16\uac8c \ub418\ub294 \uc804\uc0ac\ub294 \ubd88\uba78\uc758 \uba85\uc131\uc744 \uac00\uc9c0\uac8c \ub420 \uac83\uc774\ub77c\uace0 \ub300\ub2f5\ud588\ub2e4. \uc774\ub54c \uace0\uc791 7\uc0b4\uc774\uc5c8\ub358 \ucfe0 \ud6cc\ub9b0\uc740 \ucf58\ucf54\ubc14\ub974 \uc655\uc5d0\uac8c \uac00\uc11c \ubb34\uae30\ub97c \ub2ec\ub77c\uace0 \ud588\ub2e4. \ubb34\uae30\ub4e4\uc774 \ucfe0 \ud6cc\ub9b0\uc758 \ud798\uc744 \uacac\ub38c\ub0b4\uc9c0 \ubabb\ud558\uc790 \ucf58\ucf54\ubc14\ub974\ub294 \uc790\uae30 \ubb34\uae30\ub97c \uadf8\uc5d0\uac8c \uc8fc\uc5c8\ub2e4. \uadf8\ub7ec\ub098 \uc774 \uaf34\uc744 \ubcf8 \uce74\ud750\ubc14\ub4dc\ub294 \uc560\ud1b5\ud574\ud588\ub2e4. \ucfe0 \ud6cc\ub9b0\uc740 \uce74\ud750\ubc14\ub4dc\uc758 \uc608\uc5b8\uc744 \uc55e\ubd80\ubd84\ub9cc \ub4e3\uace0 \uac00\ubc84\ub838\ub358 \uac83\uc774\ub2e4. \uc608\uc5b8\uc758 \uc804\uccb4 \ub0b4\uc6a9\uc740 \uc624\ub298 \ubb34\uae30\ub97c \uac16\uac8c \ub418\ub294 \uc804\uc0ac\ub294 \ubd88\uba78\uc758 \uba85\uc131\uc744 \uac00\uc9c0\uac8c \ub420 \uac83\uc774\ub098 \uadf8\uc758 \uc218\uba85\uc740 \uc9e7\uc744 \uac83\uc774\ub77c\ub294 \uac83\uc774\uc5c8\ub2e4. \uc5bc\ub9c8 \ub4a4 \uce74\ud750\ubc14\ub4dc\uc758 \ub2e4\ub978 \uc608\uc5b8\uc744 \ub610 \ub4e4\uc740 \ucfe0 \ud6cc\ub9b0\uc740 \ucf58\ucf54\ubc14\ub974\uc5d0\uac8c \uc804\ucc28\ub97c \ub2ec\ub77c\uace0 \ud588\uace0, \uc774\ubc88\uc5d0\ub3c4 \ucf58\ucf54\ubc14\ub974 \uc655\uc758 \uc804\ucc28\ub9cc\uc774 \uadf8\uc758 \ud798\uc744 \uacac\ub38c\ub0bc \uc218 \uc788\uc5c8\ub2e4. \ubb34\uae30\uc640 \ud0c8\uac83\uc744 \uac00\uc9c0\uac8c \ub41c \ucfe0 \ud6cc\ub9b0\uc740 \u2018\uc9c0\uae08 \uc0b4\uc544\uc788\ub294 \uc6b8\ub77c\uc778\ub4e4\ubcf4\ub2e4 \ub354 \ub9ce\uc740 \uc6b8\ub77c\uc778\ub4e4\uc744 \uc8fd\uc600\ub2e4\u2019\uace0 \uc790\ub791\ud558\uace0 \ub2e4\ub2c8\ub358 \ub124\ud06c\ud0c4 \uc2a4\ucf00\ub124\uc758 \uc138 \uc544\ub4e4\ub4e4\uc744 \uae09\uc2b5\ud574\uc11c \ubaa8\uc870\ub9ac \uc8fd\uc600\ub2e4. \uadf8\ub294 \uc804\ub9ac\ud488\uc744 \ucc59\uaca8\uc11c \uc774\uba58 \ub9c8\ud558\ub85c \ub3cc\uc544\uc654\uace0, \uc6b8\ub77c\uc778\ub4e4\uc740 \ub9ac\uc2a4\ud2b8\ub77c\ub4dc \uc0c1\ud0dc\uc5d0 \ube60\uc9c4 \ucfe0 \ud6cc\ub9b0\uc774 \uc790\uc2e0\ub4e4\uae4c\uc9c0 \ubaa8\ub450 \uc8fd\uc774\uc9c0 \uc54a\uc744\uc9c0 \ub450\ub824\uc6cc\ud588\ub2e4. \ucf58\ucf54\ubc14\ub974 \uc655\uc758 \uc544\ub0b4(\uc989 \ucfe0 \ud6cc\ub9b0\uc5d0\uac8c\ub294 \uc678\uc219\ubaa8) \ubb34\uac04 \uc655\ube44\uac00 \uc774\uba58 \ub9c8\ud558\uc758 \uc5ec\uc790\ub4e4\uc744 \uc774\ub04c\uace0 \ub098\uc640 \uc6c3\uc637\uc744 \uae4c\uace0 \uc816\uac00\uc2b4\uc744 \ub4dc\ub7ec\ub0b4 \ubcf4\uc600\ub2e4. \uc774\uac83\uc744 \ubcf8 \ucfe0 \ud6cc\ub9b0\uc740 \ubd80\ub044\ub7ec\uc6c0\uc5d0 \ub208\uc744 \ub3cc\ub838\uace0, \uc774\ub54c \ub0a8\uc790\ub4e4\uc774 \ub2ec\ub824\ub4e4\uc5b4 \ubab8\uc2f8\uc6c0 \ub05d\uc5d0 \uadf8\ub97c \ub0c9\uc218\uac00 \ub4e4\uc5b4\uc788\ub294 \ubb3c\ud1b5\uc5d0 \ucc98\ubc15\uc558\ub2e4. \ucfe0 \ud6cc\ub9b0\uc774 \ubc1c\uc0b0\ud558\ub294 \uad11\ub780\uc758 \uc5f4\uae30 \ub54c\ubb38\uc5d0 \ubb3c\ud1b5\uc774 \ud130\uc838 \ubc84\ub838\ub2e4. \uc6b8\ub77c \ub0a8\uc790\ub4e4\uc740 \ub450 \ubc88\uc9f8 \ubb3c\ud1b5\uc5d0 \ub2e4\uc2dc \uadf8\ub97c \ucc98\ubc15\uc558\uace0, \uc774\ubc88\uc5d0\ub294 \ubb3c\uc774 \ud384\ud384 \ub053\uc5b4\uc62c\ub790\ub2e4. \uc138 \ubc88\uc9f8 \ubb3c\ud1b5\uc5d0 \ub2e4\uc2dc \ucc98\ubc15\uc790 \uaca8\uc6b0 \uc801\ub2f9\ud55c \uc628\ub3c4\ub85c \uc5f4\uc744 \uc2dd\ud790 \uc218 \uc788\uc5c8\ub2e4.", "answer": "\ub124\ud06c\ud0c4 \uc2a4\ucf00\ub124", "sentence": "\ubb34\uae30\uc640 \ud0c8\uac83\uc744 \uac00\uc9c0\uac8c \ub41c \ucfe0 \ud6cc\ub9b0\uc740 \u2018\uc9c0\uae08 \uc0b4\uc544\uc788\ub294 \uc6b8\ub77c\uc778\ub4e4\ubcf4\ub2e4 \ub354 \ub9ce\uc740 \uc6b8\ub77c\uc778\ub4e4\uc744 \uc8fd\uc600\ub2e4\u2019\uace0 \uc790\ub791\ud558\uace0 \ub2e4\ub2c8\ub358 \ub124\ud06c\ud0c4 \uc2a4\ucf00\ub124 \uc758 \uc138 \uc544\ub4e4\ub4e4\uc744 \uae09\uc2b5\ud574\uc11c \ubaa8\uc870\ub9ac \uc8fd\uc600\ub2e4.", "paragraph_sentence": "\uc774\uba58 \ub9c8\ud558\uc758 \uc5b4\ub290 \ub0a0, \ucfe0 \ud6cc\ub9b0\uc740 \uce74\ud750\ubc14\ub4dc\uac00 \ud559\uc0dd\ub4e4\uc744 \uac00\ub974\uce58\ub294 \uac83\uc744 \uc6b0\uc5f0\ud788 \ub4e3\uac8c \ub418\uc5c8\ub2e4. \ud55c \ud559\uc0dd\uc774 \uce74\ud750\ubc14\ub4dc\uc5d0\uac8c \uc624\ub298\uc774 \uc5b4\ub5a4 \uc885\ub958\uc758 \uc0c1\uc11c\ub85c\uc6b4 \ub0a0\uc774\ub0d0\uace0 \ubb3c\uc5c8\ub2e4. \uce74\ud750\ubc14\ub4dc\ub294 \uc624\ub298 \ubb34\uae30\ub97c \uac16\uac8c \ub418\ub294 \uc804\uc0ac\ub294 \ubd88\uba78\uc758 \uba85\uc131\uc744 \uac00\uc9c0\uac8c \ub420 \uac83\uc774\ub77c\uace0 \ub300\ub2f5\ud588\ub2e4. \uc774\ub54c \uace0\uc791 7\uc0b4\uc774\uc5c8\ub358 \ucfe0 \ud6cc\ub9b0\uc740 \ucf58\ucf54\ubc14\ub974 \uc655\uc5d0\uac8c \uac00\uc11c \ubb34\uae30\ub97c \ub2ec\ub77c\uace0 \ud588\ub2e4. \ubb34\uae30\ub4e4\uc774 \ucfe0 \ud6cc\ub9b0\uc758 \ud798\uc744 \uacac\ub38c\ub0b4\uc9c0 \ubabb\ud558\uc790 \ucf58\ucf54\ubc14\ub974\ub294 \uc790\uae30 \ubb34\uae30\ub97c \uadf8\uc5d0\uac8c \uc8fc\uc5c8\ub2e4. \uadf8\ub7ec\ub098 \uc774 \uaf34\uc744 \ubcf8 \uce74\ud750\ubc14\ub4dc\ub294 \uc560\ud1b5\ud574\ud588\ub2e4. \ucfe0 \ud6cc\ub9b0\uc740 \uce74\ud750\ubc14\ub4dc\uc758 \uc608\uc5b8\uc744 \uc55e\ubd80\ubd84\ub9cc \ub4e3\uace0 \uac00\ubc84\ub838\ub358 \uac83\uc774\ub2e4. \uc608\uc5b8\uc758 \uc804\uccb4 \ub0b4\uc6a9\uc740 \uc624\ub298 \ubb34\uae30\ub97c \uac16\uac8c \ub418\ub294 \uc804\uc0ac\ub294 \ubd88\uba78\uc758 \uba85\uc131\uc744 \uac00\uc9c0\uac8c \ub420 \uac83\uc774\ub098 \uadf8\uc758 \uc218\uba85\uc740 \uc9e7\uc744 \uac83\uc774\ub77c\ub294 \uac83\uc774\uc5c8\ub2e4. \uc5bc\ub9c8 \ub4a4 \uce74\ud750\ubc14\ub4dc\uc758 \ub2e4\ub978 \uc608\uc5b8\uc744 \ub610 \ub4e4\uc740 \ucfe0 \ud6cc\ub9b0\uc740 \ucf58\ucf54\ubc14\ub974\uc5d0\uac8c \uc804\ucc28\ub97c \ub2ec\ub77c\uace0 \ud588\uace0, \uc774\ubc88\uc5d0\ub3c4 \ucf58\ucf54\ubc14\ub974 \uc655\uc758 \uc804\ucc28\ub9cc\uc774 \uadf8\uc758 \ud798\uc744 \uacac\ub38c\ub0bc \uc218 \uc788\uc5c8\ub2e4. \ubb34\uae30\uc640 \ud0c8\uac83\uc744 \uac00\uc9c0\uac8c \ub41c \ucfe0 \ud6cc\ub9b0\uc740 \u2018\uc9c0\uae08 \uc0b4\uc544\uc788\ub294 \uc6b8\ub77c\uc778\ub4e4\ubcf4\ub2e4 \ub354 \ub9ce\uc740 \uc6b8\ub77c\uc778\ub4e4\uc744 \uc8fd\uc600\ub2e4\u2019\uace0 \uc790\ub791\ud558\uace0 \ub2e4\ub2c8\ub358 \ub124\ud06c\ud0c4 \uc2a4\ucf00\ub124 \uc758 \uc138 \uc544\ub4e4\ub4e4\uc744 \uae09\uc2b5\ud574\uc11c \ubaa8\uc870\ub9ac \uc8fd\uc600\ub2e4. \uadf8\ub294 \uc804\ub9ac\ud488\uc744 \ucc59\uaca8\uc11c \uc774\uba58 \ub9c8\ud558\ub85c \ub3cc\uc544\uc654\uace0, \uc6b8\ub77c\uc778\ub4e4\uc740 \ub9ac\uc2a4\ud2b8\ub77c\ub4dc \uc0c1\ud0dc\uc5d0 \ube60\uc9c4 \ucfe0 \ud6cc\ub9b0\uc774 \uc790\uc2e0\ub4e4\uae4c\uc9c0 \ubaa8\ub450 \uc8fd\uc774\uc9c0 \uc54a\uc744\uc9c0 \ub450\ub824\uc6cc\ud588\ub2e4. \ucf58\ucf54\ubc14\ub974 \uc655\uc758 \uc544\ub0b4(\uc989 \ucfe0 \ud6cc\ub9b0\uc5d0\uac8c\ub294 \uc678\uc219\ubaa8) \ubb34\uac04 \uc655\ube44\uac00 \uc774\uba58 \ub9c8\ud558\uc758 \uc5ec\uc790\ub4e4\uc744 \uc774\ub04c\uace0 \ub098\uc640 \uc6c3\uc637\uc744 \uae4c\uace0 \uc816\uac00\uc2b4\uc744 \ub4dc\ub7ec\ub0b4 \ubcf4\uc600\ub2e4. \uc774\uac83\uc744 \ubcf8 \ucfe0 \ud6cc\ub9b0\uc740 \ubd80\ub044\ub7ec\uc6c0\uc5d0 \ub208\uc744 \ub3cc\ub838\uace0, \uc774\ub54c \ub0a8\uc790\ub4e4\uc774 \ub2ec\ub824\ub4e4\uc5b4 \ubab8\uc2f8\uc6c0 \ub05d\uc5d0 \uadf8\ub97c \ub0c9\uc218\uac00 \ub4e4\uc5b4\uc788\ub294 \ubb3c\ud1b5\uc5d0 \ucc98\ubc15\uc558\ub2e4. \ucfe0 \ud6cc\ub9b0\uc774 \ubc1c\uc0b0\ud558\ub294 \uad11\ub780\uc758 \uc5f4\uae30 \ub54c\ubb38\uc5d0 \ubb3c\ud1b5\uc774 \ud130\uc838 \ubc84\ub838\ub2e4. \uc6b8\ub77c \ub0a8\uc790\ub4e4\uc740 \ub450 \ubc88\uc9f8 \ubb3c\ud1b5\uc5d0 \ub2e4\uc2dc \uadf8\ub97c \ucc98\ubc15\uc558\uace0, \uc774\ubc88\uc5d0\ub294 \ubb3c\uc774 \ud384\ud384 \ub053\uc5b4\uc62c\ub790\ub2e4. \uc138 \ubc88\uc9f8 \ubb3c\ud1b5\uc5d0 \ub2e4\uc2dc \ucc98\ubc15\uc790 \uaca8\uc6b0 \uc801\ub2f9\ud55c \uc628\ub3c4\ub85c \uc5f4\uc744 \uc2dd\ud790 \uc218 \uc788\uc5c8\ub2e4.", "paragraph_answer": "\uc774\uba58 \ub9c8\ud558\uc758 \uc5b4\ub290 \ub0a0, \ucfe0 \ud6cc\ub9b0\uc740 \uce74\ud750\ubc14\ub4dc\uac00 \ud559\uc0dd\ub4e4\uc744 \uac00\ub974\uce58\ub294 \uac83\uc744 \uc6b0\uc5f0\ud788 \ub4e3\uac8c \ub418\uc5c8\ub2e4. \ud55c \ud559\uc0dd\uc774 \uce74\ud750\ubc14\ub4dc\uc5d0\uac8c \uc624\ub298\uc774 \uc5b4\ub5a4 \uc885\ub958\uc758 \uc0c1\uc11c\ub85c\uc6b4 \ub0a0\uc774\ub0d0\uace0 \ubb3c\uc5c8\ub2e4. \uce74\ud750\ubc14\ub4dc\ub294 \uc624\ub298 \ubb34\uae30\ub97c \uac16\uac8c \ub418\ub294 \uc804\uc0ac\ub294 \ubd88\uba78\uc758 \uba85\uc131\uc744 \uac00\uc9c0\uac8c \ub420 \uac83\uc774\ub77c\uace0 \ub300\ub2f5\ud588\ub2e4. \uc774\ub54c \uace0\uc791 7\uc0b4\uc774\uc5c8\ub358 \ucfe0 \ud6cc\ub9b0\uc740 \ucf58\ucf54\ubc14\ub974 \uc655\uc5d0\uac8c \uac00\uc11c \ubb34\uae30\ub97c \ub2ec\ub77c\uace0 \ud588\ub2e4. \ubb34\uae30\ub4e4\uc774 \ucfe0 \ud6cc\ub9b0\uc758 \ud798\uc744 \uacac\ub38c\ub0b4\uc9c0 \ubabb\ud558\uc790 \ucf58\ucf54\ubc14\ub974\ub294 \uc790\uae30 \ubb34\uae30\ub97c \uadf8\uc5d0\uac8c \uc8fc\uc5c8\ub2e4. \uadf8\ub7ec\ub098 \uc774 \uaf34\uc744 \ubcf8 \uce74\ud750\ubc14\ub4dc\ub294 \uc560\ud1b5\ud574\ud588\ub2e4. \ucfe0 \ud6cc\ub9b0\uc740 \uce74\ud750\ubc14\ub4dc\uc758 \uc608\uc5b8\uc744 \uc55e\ubd80\ubd84\ub9cc \ub4e3\uace0 \uac00\ubc84\ub838\ub358 \uac83\uc774\ub2e4. \uc608\uc5b8\uc758 \uc804\uccb4 \ub0b4\uc6a9\uc740 \uc624\ub298 \ubb34\uae30\ub97c \uac16\uac8c \ub418\ub294 \uc804\uc0ac\ub294 \ubd88\uba78\uc758 \uba85\uc131\uc744 \uac00\uc9c0\uac8c \ub420 \uac83\uc774\ub098 \uadf8\uc758 \uc218\uba85\uc740 \uc9e7\uc744 \uac83\uc774\ub77c\ub294 \uac83\uc774\uc5c8\ub2e4. \uc5bc\ub9c8 \ub4a4 \uce74\ud750\ubc14\ub4dc\uc758 \ub2e4\ub978 \uc608\uc5b8\uc744 \ub610 \ub4e4\uc740 \ucfe0 \ud6cc\ub9b0\uc740 \ucf58\ucf54\ubc14\ub974\uc5d0\uac8c \uc804\ucc28\ub97c \ub2ec\ub77c\uace0 \ud588\uace0, \uc774\ubc88\uc5d0\ub3c4 \ucf58\ucf54\ubc14\ub974 \uc655\uc758 \uc804\ucc28\ub9cc\uc774 \uadf8\uc758 \ud798\uc744 \uacac\ub38c\ub0bc \uc218 \uc788\uc5c8\ub2e4. \ubb34\uae30\uc640 \ud0c8\uac83\uc744 \uac00\uc9c0\uac8c \ub41c \ucfe0 \ud6cc\ub9b0\uc740 \u2018\uc9c0\uae08 \uc0b4\uc544\uc788\ub294 \uc6b8\ub77c\uc778\ub4e4\ubcf4\ub2e4 \ub354 \ub9ce\uc740 \uc6b8\ub77c\uc778\ub4e4\uc744 \uc8fd\uc600\ub2e4\u2019\uace0 \uc790\ub791\ud558\uace0 \ub2e4\ub2c8\ub358 \ub124\ud06c\ud0c4 \uc2a4\ucf00\ub124 \uc758 \uc138 \uc544\ub4e4\ub4e4\uc744 \uae09\uc2b5\ud574\uc11c \ubaa8\uc870\ub9ac \uc8fd\uc600\ub2e4. \uadf8\ub294 \uc804\ub9ac\ud488\uc744 \ucc59\uaca8\uc11c \uc774\uba58 \ub9c8\ud558\ub85c \ub3cc\uc544\uc654\uace0, \uc6b8\ub77c\uc778\ub4e4\uc740 \ub9ac\uc2a4\ud2b8\ub77c\ub4dc \uc0c1\ud0dc\uc5d0 \ube60\uc9c4 \ucfe0 \ud6cc\ub9b0\uc774 \uc790\uc2e0\ub4e4\uae4c\uc9c0 \ubaa8\ub450 \uc8fd\uc774\uc9c0 \uc54a\uc744\uc9c0 \ub450\ub824\uc6cc\ud588\ub2e4. \ucf58\ucf54\ubc14\ub974 \uc655\uc758 \uc544\ub0b4(\uc989 \ucfe0 \ud6cc\ub9b0\uc5d0\uac8c\ub294 \uc678\uc219\ubaa8) \ubb34\uac04 \uc655\ube44\uac00 \uc774\uba58 \ub9c8\ud558\uc758 \uc5ec\uc790\ub4e4\uc744 \uc774\ub04c\uace0 \ub098\uc640 \uc6c3\uc637\uc744 \uae4c\uace0 \uc816\uac00\uc2b4\uc744 \ub4dc\ub7ec\ub0b4 \ubcf4\uc600\ub2e4. \uc774\uac83\uc744 \ubcf8 \ucfe0 \ud6cc\ub9b0\uc740 \ubd80\ub044\ub7ec\uc6c0\uc5d0 \ub208\uc744 \ub3cc\ub838\uace0, \uc774\ub54c \ub0a8\uc790\ub4e4\uc774 \ub2ec\ub824\ub4e4\uc5b4 \ubab8\uc2f8\uc6c0 \ub05d\uc5d0 \uadf8\ub97c \ub0c9\uc218\uac00 \ub4e4\uc5b4\uc788\ub294 \ubb3c\ud1b5\uc5d0 \ucc98\ubc15\uc558\ub2e4. \ucfe0 \ud6cc\ub9b0\uc774 \ubc1c\uc0b0\ud558\ub294 \uad11\ub780\uc758 \uc5f4\uae30 \ub54c\ubb38\uc5d0 \ubb3c\ud1b5\uc774 \ud130\uc838 \ubc84\ub838\ub2e4. \uc6b8\ub77c \ub0a8\uc790\ub4e4\uc740 \ub450 \ubc88\uc9f8 \ubb3c\ud1b5\uc5d0 \ub2e4\uc2dc \uadf8\ub97c \ucc98\ubc15\uc558\uace0, \uc774\ubc88\uc5d0\ub294 \ubb3c\uc774 \ud384\ud384 \ub053\uc5b4\uc62c\ub790\ub2e4. \uc138 \ubc88\uc9f8 \ubb3c\ud1b5\uc5d0 \ub2e4\uc2dc \ucc98\ubc15\uc790 \uaca8\uc6b0 \uc801\ub2f9\ud55c \uc628\ub3c4\ub85c \uc5f4\uc744 \uc2dd\ud790 \uc218 \uc788\uc5c8\ub2e4.", "sentence_answer": "\ubb34\uae30\uc640 \ud0c8\uac83\uc744 \uac00\uc9c0\uac8c \ub41c \ucfe0 \ud6cc\ub9b0\uc740 \u2018\uc9c0\uae08 \uc0b4\uc544\uc788\ub294 \uc6b8\ub77c\uc778\ub4e4\ubcf4\ub2e4 \ub354 \ub9ce\uc740 \uc6b8\ub77c\uc778\ub4e4\uc744 \uc8fd\uc600\ub2e4\u2019\uace0 \uc790\ub791\ud558\uace0 \ub2e4\ub2c8\ub358 \ub124\ud06c\ud0c4 \uc2a4\ucf00\ub124 \uc758 \uc138 \uc544\ub4e4\ub4e4\uc744 \uae09\uc2b5\ud574\uc11c \ubaa8\uc870\ub9ac \uc8fd\uc600\ub2e4.", "paragraph_id": "6562782-5-1", "paragraph_question": "question: \uc9c0\uae08 \uc0b4\uc544\uc788\ub294 \uc6b8\ub9ac\uc778\ub4e4\ubcf4\ub2e4 \ub354 \ub9ce\uc740 \uc6b8\ub77c\uc778\ub4e4\uc744 \uc8fd\uc600\ub2e4\uace0 \ub9d0\ud55c \ub0a8\uc790\ub4e4\uc740 \ub204\uad6c\uc758 \uc544\ub4e4\ub4e4\uc778\uac00?, context: \uc774\uba58 \ub9c8\ud558\uc758 \uc5b4\ub290 \ub0a0, \ucfe0 \ud6cc\ub9b0\uc740 \uce74\ud750\ubc14\ub4dc\uac00 \ud559\uc0dd\ub4e4\uc744 \uac00\ub974\uce58\ub294 \uac83\uc744 \uc6b0\uc5f0\ud788 \ub4e3\uac8c \ub418\uc5c8\ub2e4. \ud55c \ud559\uc0dd\uc774 \uce74\ud750\ubc14\ub4dc\uc5d0\uac8c \uc624\ub298\uc774 \uc5b4\ub5a4 \uc885\ub958\uc758 \uc0c1\uc11c\ub85c\uc6b4 \ub0a0\uc774\ub0d0\uace0 \ubb3c\uc5c8\ub2e4. \uce74\ud750\ubc14\ub4dc\ub294 \uc624\ub298 \ubb34\uae30\ub97c \uac16\uac8c \ub418\ub294 \uc804\uc0ac\ub294 \ubd88\uba78\uc758 \uba85\uc131\uc744 \uac00\uc9c0\uac8c \ub420 \uac83\uc774\ub77c\uace0 \ub300\ub2f5\ud588\ub2e4. \uc774\ub54c \uace0\uc791 7\uc0b4\uc774\uc5c8\ub358 \ucfe0 \ud6cc\ub9b0\uc740 \ucf58\ucf54\ubc14\ub974 \uc655\uc5d0\uac8c \uac00\uc11c \ubb34\uae30\ub97c \ub2ec\ub77c\uace0 \ud588\ub2e4. \ubb34\uae30\ub4e4\uc774 \ucfe0 \ud6cc\ub9b0\uc758 \ud798\uc744 \uacac\ub38c\ub0b4\uc9c0 \ubabb\ud558\uc790 \ucf58\ucf54\ubc14\ub974\ub294 \uc790\uae30 \ubb34\uae30\ub97c \uadf8\uc5d0\uac8c \uc8fc\uc5c8\ub2e4. \uadf8\ub7ec\ub098 \uc774 \uaf34\uc744 \ubcf8 \uce74\ud750\ubc14\ub4dc\ub294 \uc560\ud1b5\ud574\ud588\ub2e4. \ucfe0 \ud6cc\ub9b0\uc740 \uce74\ud750\ubc14\ub4dc\uc758 \uc608\uc5b8\uc744 \uc55e\ubd80\ubd84\ub9cc \ub4e3\uace0 \uac00\ubc84\ub838\ub358 \uac83\uc774\ub2e4. \uc608\uc5b8\uc758 \uc804\uccb4 \ub0b4\uc6a9\uc740 \uc624\ub298 \ubb34\uae30\ub97c \uac16\uac8c \ub418\ub294 \uc804\uc0ac\ub294 \ubd88\uba78\uc758 \uba85\uc131\uc744 \uac00\uc9c0\uac8c \ub420 \uac83\uc774\ub098 \uadf8\uc758 \uc218\uba85\uc740 \uc9e7\uc744 \uac83\uc774\ub77c\ub294 \uac83\uc774\uc5c8\ub2e4. \uc5bc\ub9c8 \ub4a4 \uce74\ud750\ubc14\ub4dc\uc758 \ub2e4\ub978 \uc608\uc5b8\uc744 \ub610 \ub4e4\uc740 \ucfe0 \ud6cc\ub9b0\uc740 \ucf58\ucf54\ubc14\ub974\uc5d0\uac8c \uc804\ucc28\ub97c \ub2ec\ub77c\uace0 \ud588\uace0, \uc774\ubc88\uc5d0\ub3c4 \ucf58\ucf54\ubc14\ub974 \uc655\uc758 \uc804\ucc28\ub9cc\uc774 \uadf8\uc758 \ud798\uc744 \uacac\ub38c\ub0bc \uc218 \uc788\uc5c8\ub2e4. \ubb34\uae30\uc640 \ud0c8\uac83\uc744 \uac00\uc9c0\uac8c \ub41c \ucfe0 \ud6cc\ub9b0\uc740 \u2018\uc9c0\uae08 \uc0b4\uc544\uc788\ub294 \uc6b8\ub77c\uc778\ub4e4\ubcf4\ub2e4 \ub354 \ub9ce\uc740 \uc6b8\ub77c\uc778\ub4e4\uc744 \uc8fd\uc600\ub2e4\u2019\uace0 \uc790\ub791\ud558\uace0 \ub2e4\ub2c8\ub358 \ub124\ud06c\ud0c4 \uc2a4\ucf00\ub124\uc758 \uc138 \uc544\ub4e4\ub4e4\uc744 \uae09\uc2b5\ud574\uc11c \ubaa8\uc870\ub9ac \uc8fd\uc600\ub2e4. \uadf8\ub294 \uc804\ub9ac\ud488\uc744 \ucc59\uaca8\uc11c \uc774\uba58 \ub9c8\ud558\ub85c \ub3cc\uc544\uc654\uace0, \uc6b8\ub77c\uc778\ub4e4\uc740 \ub9ac\uc2a4\ud2b8\ub77c\ub4dc \uc0c1\ud0dc\uc5d0 \ube60\uc9c4 \ucfe0 \ud6cc\ub9b0\uc774 \uc790\uc2e0\ub4e4\uae4c\uc9c0 \ubaa8\ub450 \uc8fd\uc774\uc9c0 \uc54a\uc744\uc9c0 \ub450\ub824\uc6cc\ud588\ub2e4. \ucf58\ucf54\ubc14\ub974 \uc655\uc758 \uc544\ub0b4(\uc989 \ucfe0 \ud6cc\ub9b0\uc5d0\uac8c\ub294 \uc678\uc219\ubaa8) \ubb34\uac04 \uc655\ube44\uac00 \uc774\uba58 \ub9c8\ud558\uc758 \uc5ec\uc790\ub4e4\uc744 \uc774\ub04c\uace0 \ub098\uc640 \uc6c3\uc637\uc744 \uae4c\uace0 \uc816\uac00\uc2b4\uc744 \ub4dc\ub7ec\ub0b4 \ubcf4\uc600\ub2e4. \uc774\uac83\uc744 \ubcf8 \ucfe0 \ud6cc\ub9b0\uc740 \ubd80\ub044\ub7ec\uc6c0\uc5d0 \ub208\uc744 \ub3cc\ub838\uace0, \uc774\ub54c \ub0a8\uc790\ub4e4\uc774 \ub2ec\ub824\ub4e4\uc5b4 \ubab8\uc2f8\uc6c0 \ub05d\uc5d0 \uadf8\ub97c \ub0c9\uc218\uac00 \ub4e4\uc5b4\uc788\ub294 \ubb3c\ud1b5\uc5d0 \ucc98\ubc15\uc558\ub2e4. \ucfe0 \ud6cc\ub9b0\uc774 \ubc1c\uc0b0\ud558\ub294 \uad11\ub780\uc758 \uc5f4\uae30 \ub54c\ubb38\uc5d0 \ubb3c\ud1b5\uc774 \ud130\uc838 \ubc84\ub838\ub2e4. \uc6b8\ub77c \ub0a8\uc790\ub4e4\uc740 \ub450 \ubc88\uc9f8 \ubb3c\ud1b5\uc5d0 \ub2e4\uc2dc \uadf8\ub97c \ucc98\ubc15\uc558\uace0, \uc774\ubc88\uc5d0\ub294 \ubb3c\uc774 \ud384\ud384 \ub053\uc5b4\uc62c\ub790\ub2e4. \uc138 \ubc88\uc9f8 \ubb3c\ud1b5\uc5d0 \ub2e4\uc2dc \ucc98\ubc15\uc790 \uaca8\uc6b0 \uc801\ub2f9\ud55c \uc628\ub3c4\ub85c \uc5f4\uc744 \uc2dd\ud790 \uc218 \uc788\uc5c8\ub2e4."} {"question": "\uc804\ud0dc\uc77c\uc774 \uc0ac\ub9dd\ud558\uae30 \uc9c1\uc804 \ud588\ub358 \ub9d0\uc740?", "paragraph": "1970\ub144 11\uc6d4 13\uc77c \uc624\ud6c4 1\uc2dc \uc544\ub4e4 \uc804\ud0dc\uc77c\uc774 \ud3c9\ud654\uc2dc\uc7a5 \uc55e \uac70\ub9ac\uc5d0\uc11c \"\uadfc\ub85c\uae30\uc900\ubc95\uc744 \uc900\uc218\ud558\ub77c\", \"\uc6b0\ub9ac\ub294 \uae30\uacc4\uac00 \uc544\ub2c8\ub2e4\", \"\uc77c\uc694\uc77c\uc5d4 \uc26c\uac8c \ud558\ub77c\"\uc640 \uac19\uc740 \uad6c\ud638\ub97c \uc678\uce58\uba70 \ubd84\uc2e0\ud558\uc600\ub2e4. \uc774\uc18c\uc120\uc740 \uad6c\uc5ed \uc608\ubc30\ub97c \ub3cc\uace0 \uc788\ub2e4 \ub3d9\ub124 \uc8fc\ubbfc\uc774 \uac74\ub0b4\uc900 \ub77c\ub514\uc624\uc5d0\uc11c \ub098\uc624\ub294 \ub274\uc2a4\ub97c \ub4e3\uace0 \uc774 \uc0ac\uc2e4\uc744 \uc54c\uc558\ub2e4. \uc804\ud0dc\uc77c\uc740 \uba54\ub514\uceec \ubcd1\uc6d0\uc5d0\uc11c \uc131\ubaa8\ubcd1\uc6d0\uc73c\ub85c \uc62e\uaca8\uc84c\uace0 \uc774\ubbf8 \uac00\ub9dd\uc774 \uc5c6\uc5c8\ub2e4. \uc804\ud0dc\uc77c\uc740 \ub2ec\ub824\uc628 \uce5c\uad6c\ub4e4\uacfc \uc5b4\uba38\ub2c8\uc5d0\uac8c \uc790\uc2e0\uc774 \ubabb\ub2e4\ud55c \uc77c\uc744 \ub2e4 \ud574 \ub2ec\ub77c\uace0 \ubd80\ud0c1\ud558\uc600\ub2e4. \uc804\ud0dc\uc77c\uc740 \uc774\uc18c\uc120\uc5d0\uac8c \uac70\ub4ed \uc5b4\ub5a4 \uc720\ud639\uc5d0\ub3c4 \uad74\ud558\uc9c0 \ub9d0\uace0 \uc790\uc2e0\uc758 \ubd80\ud0c1\uc744 \ub4e4\uc5b4\ub2ec\ub77c \uac04\uccad\ud558\uc600\uace0 \ud070 \uc18c\ub9ac\ub85c \ub300\ub2f5\ud558\uac8c \ud558\uc600\ub2e4. \uc774\uc18c\uc120\uc740 \"\ub0b4 \ubab8\uc774 \uac00\ub8e8\uac00 \ub418\uc5b4\ub3c4 \ub2c8\uac00 \uc6d0\ud558\ub294 \uac70 \ub05d\uae4c\uc9c0 \ud560 \uac70\ub2e4!\"\ub77c\uace0 \ud06c\uac8c \ub300\ub2f5\ud558\uc600\ub2e4. \uadf8\ub0a0 \uc624\ud6c4 10\uc2dc \uc804\ud0dc\uc77c\uc740 \"\ubc30\uac00 \uace0\ud504\ub2e4\"\ub294 \ub9d0\uc744 \ub0a8\uae30\uace0 \uc0ac\ub9dd\ud558\uc600\ub2e4.", "answer": "\ubc30\uac00 \uace0\ud504\ub2e4", "sentence": "\uadf8\ub0a0 \uc624\ud6c4 10\uc2dc \uc804\ud0dc\uc77c\uc740 \" \ubc30\uac00 \uace0\ud504\ub2e4 \"\ub294 \ub9d0\uc744 \ub0a8\uae30\uace0 \uc0ac\ub9dd\ud558\uc600\ub2e4.", "paragraph_sentence": "1970\ub144 11\uc6d4 13\uc77c \uc624\ud6c4 1\uc2dc \uc544\ub4e4 \uc804\ud0dc\uc77c\uc774 \ud3c9\ud654\uc2dc\uc7a5 \uc55e \uac70\ub9ac\uc5d0\uc11c \"\uadfc\ub85c\uae30\uc900\ubc95\uc744 \uc900\uc218\ud558\ub77c\", \"\uc6b0\ub9ac\ub294 \uae30\uacc4\uac00 \uc544\ub2c8\ub2e4\", \"\uc77c\uc694\uc77c\uc5d4 \uc26c\uac8c \ud558\ub77c\"\uc640 \uac19\uc740 \uad6c\ud638\ub97c \uc678\uce58\uba70 \ubd84\uc2e0\ud558\uc600\ub2e4. \uc774\uc18c\uc120\uc740 \uad6c\uc5ed \uc608\ubc30\ub97c \ub3cc\uace0 \uc788\ub2e4 \ub3d9\ub124 \uc8fc\ubbfc\uc774 \uac74\ub0b4\uc900 \ub77c\ub514\uc624\uc5d0\uc11c \ub098\uc624\ub294 \ub274\uc2a4\ub97c \ub4e3\uace0 \uc774 \uc0ac\uc2e4\uc744 \uc54c\uc558\ub2e4. \uc804\ud0dc\uc77c\uc740 \uba54\ub514\uceec \ubcd1\uc6d0\uc5d0\uc11c \uc131\ubaa8\ubcd1\uc6d0\uc73c\ub85c \uc62e\uaca8\uc84c\uace0 \uc774\ubbf8 \uac00\ub9dd\uc774 \uc5c6\uc5c8\ub2e4. \uc804\ud0dc\uc77c\uc740 \ub2ec\ub824\uc628 \uce5c\uad6c\ub4e4\uacfc \uc5b4\uba38\ub2c8\uc5d0\uac8c \uc790\uc2e0\uc774 \ubabb\ub2e4\ud55c \uc77c\uc744 \ub2e4 \ud574 \ub2ec\ub77c\uace0 \ubd80\ud0c1\ud558\uc600\ub2e4. \uc804\ud0dc\uc77c\uc740 \uc774\uc18c\uc120\uc5d0\uac8c \uac70\ub4ed \uc5b4\ub5a4 \uc720\ud639\uc5d0\ub3c4 \uad74\ud558\uc9c0 \ub9d0\uace0 \uc790\uc2e0\uc758 \ubd80\ud0c1\uc744 \ub4e4\uc5b4\ub2ec\ub77c \uac04\uccad\ud558\uc600\uace0 \ud070 \uc18c\ub9ac\ub85c \ub300\ub2f5\ud558\uac8c \ud558\uc600\ub2e4. \uc774\uc18c\uc120\uc740 \"\ub0b4 \ubab8\uc774 \uac00\ub8e8\uac00 \ub418\uc5b4\ub3c4 \ub2c8\uac00 \uc6d0\ud558\ub294 \uac70 \ub05d\uae4c\uc9c0 \ud560 \uac70\ub2e4!\"\ub77c\uace0 \ud06c\uac8c \ub300\ub2f5\ud558\uc600\ub2e4. \uadf8\ub0a0 \uc624\ud6c4 10\uc2dc \uc804\ud0dc\uc77c\uc740 \" \ubc30\uac00 \uace0\ud504\ub2e4 \"\ub294 \ub9d0\uc744 \ub0a8\uae30\uace0 \uc0ac\ub9dd\ud558\uc600\ub2e4. ", "paragraph_answer": "1970\ub144 11\uc6d4 13\uc77c \uc624\ud6c4 1\uc2dc \uc544\ub4e4 \uc804\ud0dc\uc77c\uc774 \ud3c9\ud654\uc2dc\uc7a5 \uc55e \uac70\ub9ac\uc5d0\uc11c \"\uadfc\ub85c\uae30\uc900\ubc95\uc744 \uc900\uc218\ud558\ub77c\", \"\uc6b0\ub9ac\ub294 \uae30\uacc4\uac00 \uc544\ub2c8\ub2e4\", \"\uc77c\uc694\uc77c\uc5d4 \uc26c\uac8c \ud558\ub77c\"\uc640 \uac19\uc740 \uad6c\ud638\ub97c \uc678\uce58\uba70 \ubd84\uc2e0\ud558\uc600\ub2e4. \uc774\uc18c\uc120\uc740 \uad6c\uc5ed \uc608\ubc30\ub97c \ub3cc\uace0 \uc788\ub2e4 \ub3d9\ub124 \uc8fc\ubbfc\uc774 \uac74\ub0b4\uc900 \ub77c\ub514\uc624\uc5d0\uc11c \ub098\uc624\ub294 \ub274\uc2a4\ub97c \ub4e3\uace0 \uc774 \uc0ac\uc2e4\uc744 \uc54c\uc558\ub2e4. \uc804\ud0dc\uc77c\uc740 \uba54\ub514\uceec \ubcd1\uc6d0\uc5d0\uc11c \uc131\ubaa8\ubcd1\uc6d0\uc73c\ub85c \uc62e\uaca8\uc84c\uace0 \uc774\ubbf8 \uac00\ub9dd\uc774 \uc5c6\uc5c8\ub2e4. \uc804\ud0dc\uc77c\uc740 \ub2ec\ub824\uc628 \uce5c\uad6c\ub4e4\uacfc \uc5b4\uba38\ub2c8\uc5d0\uac8c \uc790\uc2e0\uc774 \ubabb\ub2e4\ud55c \uc77c\uc744 \ub2e4 \ud574 \ub2ec\ub77c\uace0 \ubd80\ud0c1\ud558\uc600\ub2e4. \uc804\ud0dc\uc77c\uc740 \uc774\uc18c\uc120\uc5d0\uac8c \uac70\ub4ed \uc5b4\ub5a4 \uc720\ud639\uc5d0\ub3c4 \uad74\ud558\uc9c0 \ub9d0\uace0 \uc790\uc2e0\uc758 \ubd80\ud0c1\uc744 \ub4e4\uc5b4\ub2ec\ub77c \uac04\uccad\ud558\uc600\uace0 \ud070 \uc18c\ub9ac\ub85c \ub300\ub2f5\ud558\uac8c \ud558\uc600\ub2e4. \uc774\uc18c\uc120\uc740 \"\ub0b4 \ubab8\uc774 \uac00\ub8e8\uac00 \ub418\uc5b4\ub3c4 \ub2c8\uac00 \uc6d0\ud558\ub294 \uac70 \ub05d\uae4c\uc9c0 \ud560 \uac70\ub2e4!\"\ub77c\uace0 \ud06c\uac8c \ub300\ub2f5\ud558\uc600\ub2e4. \uadf8\ub0a0 \uc624\ud6c4 10\uc2dc \uc804\ud0dc\uc77c\uc740 \" \ubc30\uac00 \uace0\ud504\ub2e4 \"\ub294 \ub9d0\uc744 \ub0a8\uae30\uace0 \uc0ac\ub9dd\ud558\uc600\ub2e4.", "sentence_answer": "\uadf8\ub0a0 \uc624\ud6c4 10\uc2dc \uc804\ud0dc\uc77c\uc740 \" \ubc30\uac00 \uace0\ud504\ub2e4 \"\ub294 \ub9d0\uc744 \ub0a8\uae30\uace0 \uc0ac\ub9dd\ud558\uc600\ub2e4.", "paragraph_id": "6464150-4-1", "paragraph_question": "question: \uc804\ud0dc\uc77c\uc774 \uc0ac\ub9dd\ud558\uae30 \uc9c1\uc804 \ud588\ub358 \ub9d0\uc740?, context: 1970\ub144 11\uc6d4 13\uc77c \uc624\ud6c4 1\uc2dc \uc544\ub4e4 \uc804\ud0dc\uc77c\uc774 \ud3c9\ud654\uc2dc\uc7a5 \uc55e \uac70\ub9ac\uc5d0\uc11c \"\uadfc\ub85c\uae30\uc900\ubc95\uc744 \uc900\uc218\ud558\ub77c\", \"\uc6b0\ub9ac\ub294 \uae30\uacc4\uac00 \uc544\ub2c8\ub2e4\", \"\uc77c\uc694\uc77c\uc5d4 \uc26c\uac8c \ud558\ub77c\"\uc640 \uac19\uc740 \uad6c\ud638\ub97c \uc678\uce58\uba70 \ubd84\uc2e0\ud558\uc600\ub2e4. \uc774\uc18c\uc120\uc740 \uad6c\uc5ed \uc608\ubc30\ub97c \ub3cc\uace0 \uc788\ub2e4 \ub3d9\ub124 \uc8fc\ubbfc\uc774 \uac74\ub0b4\uc900 \ub77c\ub514\uc624\uc5d0\uc11c \ub098\uc624\ub294 \ub274\uc2a4\ub97c \ub4e3\uace0 \uc774 \uc0ac\uc2e4\uc744 \uc54c\uc558\ub2e4. \uc804\ud0dc\uc77c\uc740 \uba54\ub514\uceec \ubcd1\uc6d0\uc5d0\uc11c \uc131\ubaa8\ubcd1\uc6d0\uc73c\ub85c \uc62e\uaca8\uc84c\uace0 \uc774\ubbf8 \uac00\ub9dd\uc774 \uc5c6\uc5c8\ub2e4. \uc804\ud0dc\uc77c\uc740 \ub2ec\ub824\uc628 \uce5c\uad6c\ub4e4\uacfc \uc5b4\uba38\ub2c8\uc5d0\uac8c \uc790\uc2e0\uc774 \ubabb\ub2e4\ud55c \uc77c\uc744 \ub2e4 \ud574 \ub2ec\ub77c\uace0 \ubd80\ud0c1\ud558\uc600\ub2e4. \uc804\ud0dc\uc77c\uc740 \uc774\uc18c\uc120\uc5d0\uac8c \uac70\ub4ed \uc5b4\ub5a4 \uc720\ud639\uc5d0\ub3c4 \uad74\ud558\uc9c0 \ub9d0\uace0 \uc790\uc2e0\uc758 \ubd80\ud0c1\uc744 \ub4e4\uc5b4\ub2ec\ub77c \uac04\uccad\ud558\uc600\uace0 \ud070 \uc18c\ub9ac\ub85c \ub300\ub2f5\ud558\uac8c \ud558\uc600\ub2e4. \uc774\uc18c\uc120\uc740 \"\ub0b4 \ubab8\uc774 \uac00\ub8e8\uac00 \ub418\uc5b4\ub3c4 \ub2c8\uac00 \uc6d0\ud558\ub294 \uac70 \ub05d\uae4c\uc9c0 \ud560 \uac70\ub2e4!\"\ub77c\uace0 \ud06c\uac8c \ub300\ub2f5\ud558\uc600\ub2e4. \uadf8\ub0a0 \uc624\ud6c4 10\uc2dc \uc804\ud0dc\uc77c\uc740 \"\ubc30\uac00 \uace0\ud504\ub2e4\"\ub294 \ub9d0\uc744 \ub0a8\uae30\uace0 \uc0ac\ub9dd\ud558\uc600\ub2e4."} {"question": "19\uc138\uae30 \ud6c4\ubc18 \ub178\ub798, \ucda4, \uc2dc \ub4f1 \uc5ec\ub7ec\uac00\uc9c0 \uc608\uc220 \ubd84\uc57c\ub97c \uaca8\ub8e8\ub294 \ub300\ud68c\uc758 \uc774\ub984\uc740?", "paragraph": "\uc6e8\uc77c\uc2a4\ub294 \uc601\uad6d\uc758 \ub2e4\ub978 \uc9c0\uc5ed\uacfc \uc815\uce58\uc801, \uc0ac\ud68c\uc801 \uc5ed\uc0ac\ub97c \uae34\ubc00\ud558\uac8c \uacf5\uc720\ud558\uace0 \uc788\uc9c0\ub9cc \uc778\uad6c\uc758 \ub300\ub2e4\uc218\ub294 \uc601\uc5b4\ub97c \uad6c\uc0ac\ud55c\ub2e4. \uc6e8\uc77c\uc2a4\ub294 \ub3c5\ud2b9\ud55c \ubb38\ud654\uc801 \uc815\uccb4\uc131\uc744 \uac00\uc9c0\uace0 \uc788\uc73c\uba70, \uacf5\uc2dd\uc801\uc73c\ub85c \uc774\uc911 \uc5b8\uc5b4 \uccb4\uc81c\uc774\ub2e4. \uc6e8\uc77c\uc2a4 \ub0b4\uc5d0\ub294 560,000\uba85 \uc774\uc0c1\uc758 \uc6e8\uc77c\uc2a4\uc5b4 \uc0ac\uc6a9\uc790\uac00 \uac70\uc8fc\ud558\uace0 \uc788\uc73c\uba70 \uc774\ub4e4\uc740 \uc8fc\ub85c \uc6e8\uc77c\uc2a4\uc758 \ubd81\ubd80\uc640 \uc11c\ubd80\uc5d0 \uac70\uc8fc\ud55c\ub2e4. 19\uc138\uae30 \ud6c4\ubc18\ubd80\ud130\ub294 \ub178\ub798, \ucda4, \uc2dc \ub0ad\uc1a1, \ubb38\ud559, \uc545\uae30, \uc791\uace1, \ubba4\uc9c0\uceec, \uc5f0\uadf9 \ub4f1\uc744 \uaca8\ub8e8\ub294 \ub300\ud68c\uc778 \uc774\uc2a4\ud14c\ub4dc\ubcf4\ub4dc\ub97c \ud1b5\ud574, \uc6e8\uc77c\uc2a4\ub294 '\ub178\ub798\uc758 \ub098\ub77c'\uc758 \uc774\ubbf8\uc9c0\ub85c \ub110\ub9ac \uc54c\ub824\uc9c0\uac8c \ub41c\ub2e4. \uc774\uc2a4\ud14c\ub4dc\ubcf4\ub4dc\ub294 1176\ub144\uc5d0 \ucd5c\ucd08 \ub300\ud68c\uac00 \uac1c\ucd5c\ub418\uc5b4 \ud604\uc7ac\uc5d0 \uc774\ub974\ub294 \uc720\uad6c\ud55c \uc804\ud1b5\uc744 \uc9c0\ub2c8\uace0 \uc788\ub2e4. \uc804\ud1b5\uc801\uc73c\ub85c \uc6e8\uc77c\uc2a4\uc778\ub4e4\uc740 \ub9e4\uc6b0 \ud65c\ubc1c\ud558\uace0 \uacbd\ucf8c\ud55c \uc0ac\ub78c\ub4e4\ub85c \uc54c\ub824\uc838 \uc654\uc73c\uba70, \uadf8\ub4e4\uc740 \uc774\uc6c3\uacfc \uc5b4\uc6b8\ub824 \ub178\ub798\ud558\uace0 \ucda4\ucd94\ub294 \uac83\uc744 \uc990\uacbc\ub2e4. \ub610\ud55c \ud558\ud504 \uc74c\uc545\ubb38\ud654\uac00 \ub9e4\uc6b0 \ubc1c\ub2ec\ud588\uace0 \uc74c\uc720\uc2dc\uc778\ub4e4\uc740 \uc6e8\uc77c\uc2a4\uc5b4\ub85c \uc2dc\ub97c \uc37c\ub2e4. \uc815\uae30\uc801\uc73c\ub85c \uc74c\uc720\uc2dc\uc778 \uacbd\uc5f0\ub300\ud68c\uac00 \uac1c\ucd5c\ub418\uc5c8\ub2e4. \uadf8\ub7ec\ub098 \uc774\ub7ec\ud55c \uc6e8\uc77c\uc2a4 \uace0\uc720\uc758 \ubb38\ud654\ub294 17\uc138\uae30 \ud6c4\ubc18\ubd80\ud130 \uc789\uae00\ub79c\ub4dc\uc758 \ubb38\ud654\uac00 \uc720\uc785\ub418\uae30 \uc2dc\uc791\ud558\uba74\uc11c \uc11c\uc11c\ud788 \uc0ac\ub77c\uc84c\ub2e4. \ud2b9\ud788 18\uc138\uae30 \ucd08\ubc18\uc5d0\ub294 \uc5c4\uccad\ub09c \uc18d\ub3c4\ub85c \uc6e8\uc77c\uc2a4\uc758 \uc804\ud1b5\ubb38\ud654\uac00 \ud1f4\uc0c9\ub418\uc5c8\ub2e4. 18\uc138\uae30 \ucd08\ubc18 \uc6e8\uc77c\uc2a4\uc5b4\ub85c \ub41c \uc11c\uc801\uc740 \uc790\ucde8\ub97c \uac10\ucd94\uc5c8\uace0 \ud558\ud504 \uac00\ub77d\uc5d0 \ub9de\ucdb0 \ub178\ub798\ud558\ub294 \uac83\uc740 \uc800\uc9c8\uc2a4\ub7f0 \uac83\uc73c\ub85c \uc778\uc2dd\uc774 \ub9cc\uc5f0\ud574\uc84c\ub2e4. \ud604\ub300\uc758 \uc6e8\uc77c\uc2a4 \ubb38\ud654\ub294 18\uc138\uae30 \ucd08\ubc18 \uc774\ud6c4 \uc789\uae00\ub79c\ub4dc \ubb38\ud654\uc640 \uc11e\uc784\uc5d0 \ub530\ub77c \uc789\uae00\ub79c\ub4dc \ubb38\ud654\uc640 \ud070 \ucc28\uc774\uac00 \uc5c6\uc5b4\uc84c\ub2e4.", "answer": "\uc774\uc2a4\ud14c\ub4dc\ubcf4\ub4dc", "sentence": "19\uc138\uae30 \ud6c4\ubc18\ubd80\ud130\ub294 \ub178\ub798, \ucda4, \uc2dc \ub0ad\uc1a1, \ubb38\ud559, \uc545\uae30, \uc791\uace1, \ubba4\uc9c0\uceec, \uc5f0\uadf9 \ub4f1\uc744 \uaca8\ub8e8\ub294 \ub300\ud68c\uc778 \uc774\uc2a4\ud14c\ub4dc\ubcf4\ub4dc \ub97c \ud1b5\ud574, \uc6e8\uc77c\uc2a4\ub294 '\ub178\ub798\uc758 \ub098\ub77c'\uc758 \uc774\ubbf8\uc9c0\ub85c \ub110\ub9ac \uc54c\ub824\uc9c0\uac8c \ub41c\ub2e4.", "paragraph_sentence": "\uc6e8\uc77c\uc2a4\ub294 \uc601\uad6d\uc758 \ub2e4\ub978 \uc9c0\uc5ed\uacfc \uc815\uce58\uc801, \uc0ac\ud68c\uc801 \uc5ed\uc0ac\ub97c \uae34\ubc00\ud558\uac8c \uacf5\uc720\ud558\uace0 \uc788\uc9c0\ub9cc \uc778\uad6c\uc758 \ub300\ub2e4\uc218\ub294 \uc601\uc5b4\ub97c \uad6c\uc0ac\ud55c\ub2e4. \uc6e8\uc77c\uc2a4\ub294 \ub3c5\ud2b9\ud55c \ubb38\ud654\uc801 \uc815\uccb4\uc131\uc744 \uac00\uc9c0\uace0 \uc788\uc73c\uba70, \uacf5\uc2dd\uc801\uc73c\ub85c \uc774\uc911 \uc5b8\uc5b4 \uccb4\uc81c\uc774\ub2e4. \uc6e8\uc77c\uc2a4 \ub0b4\uc5d0\ub294 560,000\uba85 \uc774\uc0c1\uc758 \uc6e8\uc77c\uc2a4\uc5b4 \uc0ac\uc6a9\uc790\uac00 \uac70\uc8fc\ud558\uace0 \uc788\uc73c\uba70 \uc774\ub4e4\uc740 \uc8fc\ub85c \uc6e8\uc77c\uc2a4\uc758 \ubd81\ubd80\uc640 \uc11c\ubd80\uc5d0 \uac70\uc8fc\ud55c\ub2e4. 19\uc138\uae30 \ud6c4\ubc18\ubd80\ud130\ub294 \ub178\ub798, \ucda4, \uc2dc \ub0ad\uc1a1, \ubb38\ud559, \uc545\uae30, \uc791\uace1, \ubba4\uc9c0\uceec, \uc5f0\uadf9 \ub4f1\uc744 \uaca8\ub8e8\ub294 \ub300\ud68c\uc778 \uc774\uc2a4\ud14c\ub4dc\ubcf4\ub4dc \ub97c \ud1b5\ud574, \uc6e8\uc77c\uc2a4\ub294 '\ub178\ub798\uc758 \ub098\ub77c'\uc758 \uc774\ubbf8\uc9c0\ub85c \ub110\ub9ac \uc54c\ub824\uc9c0\uac8c \ub41c\ub2e4. \uc774\uc2a4\ud14c\ub4dc\ubcf4\ub4dc\ub294 1176\ub144\uc5d0 \ucd5c\ucd08 \ub300\ud68c\uac00 \uac1c\ucd5c\ub418\uc5b4 \ud604\uc7ac\uc5d0 \uc774\ub974\ub294 \uc720\uad6c\ud55c \uc804\ud1b5\uc744 \uc9c0\ub2c8\uace0 \uc788\ub2e4. \uc804\ud1b5\uc801\uc73c\ub85c \uc6e8\uc77c\uc2a4\uc778\ub4e4\uc740 \ub9e4\uc6b0 \ud65c\ubc1c\ud558\uace0 \uacbd\ucf8c\ud55c \uc0ac\ub78c\ub4e4\ub85c \uc54c\ub824\uc838 \uc654\uc73c\uba70, \uadf8\ub4e4\uc740 \uc774\uc6c3\uacfc \uc5b4\uc6b8\ub824 \ub178\ub798\ud558\uace0 \ucda4\ucd94\ub294 \uac83\uc744 \uc990\uacbc\ub2e4. \ub610\ud55c \ud558\ud504 \uc74c\uc545\ubb38\ud654\uac00 \ub9e4\uc6b0 \ubc1c\ub2ec\ud588\uace0 \uc74c\uc720\uc2dc\uc778\ub4e4\uc740 \uc6e8\uc77c\uc2a4\uc5b4\ub85c \uc2dc\ub97c \uc37c\ub2e4. \uc815\uae30\uc801\uc73c\ub85c \uc74c\uc720\uc2dc\uc778 \uacbd\uc5f0\ub300\ud68c\uac00 \uac1c\ucd5c\ub418\uc5c8\ub2e4. \uadf8\ub7ec\ub098 \uc774\ub7ec\ud55c \uc6e8\uc77c\uc2a4 \uace0\uc720\uc758 \ubb38\ud654\ub294 17\uc138\uae30 \ud6c4\ubc18\ubd80\ud130 \uc789\uae00\ub79c\ub4dc\uc758 \ubb38\ud654\uac00 \uc720\uc785\ub418\uae30 \uc2dc\uc791\ud558\uba74\uc11c \uc11c\uc11c\ud788 \uc0ac\ub77c\uc84c\ub2e4. \ud2b9\ud788 18\uc138\uae30 \ucd08\ubc18\uc5d0\ub294 \uc5c4\uccad\ub09c \uc18d\ub3c4\ub85c \uc6e8\uc77c\uc2a4\uc758 \uc804\ud1b5\ubb38\ud654\uac00 \ud1f4\uc0c9\ub418\uc5c8\ub2e4. 18\uc138\uae30 \ucd08\ubc18 \uc6e8\uc77c\uc2a4\uc5b4\ub85c \ub41c \uc11c\uc801\uc740 \uc790\ucde8\ub97c \uac10\ucd94\uc5c8\uace0 \ud558\ud504 \uac00\ub77d\uc5d0 \ub9de\ucdb0 \ub178\ub798\ud558\ub294 \uac83\uc740 \uc800\uc9c8\uc2a4\ub7f0 \uac83\uc73c\ub85c \uc778\uc2dd\uc774 \ub9cc\uc5f0\ud574\uc84c\ub2e4. \ud604\ub300\uc758 \uc6e8\uc77c\uc2a4 \ubb38\ud654\ub294 18\uc138\uae30 \ucd08\ubc18 \uc774\ud6c4 \uc789\uae00\ub79c\ub4dc \ubb38\ud654\uc640 \uc11e\uc784\uc5d0 \ub530\ub77c \uc789\uae00\ub79c\ub4dc \ubb38\ud654\uc640 \ud070 \ucc28\uc774\uac00 \uc5c6\uc5b4\uc84c\ub2e4.", "paragraph_answer": "\uc6e8\uc77c\uc2a4\ub294 \uc601\uad6d\uc758 \ub2e4\ub978 \uc9c0\uc5ed\uacfc \uc815\uce58\uc801, \uc0ac\ud68c\uc801 \uc5ed\uc0ac\ub97c \uae34\ubc00\ud558\uac8c \uacf5\uc720\ud558\uace0 \uc788\uc9c0\ub9cc \uc778\uad6c\uc758 \ub300\ub2e4\uc218\ub294 \uc601\uc5b4\ub97c \uad6c\uc0ac\ud55c\ub2e4. \uc6e8\uc77c\uc2a4\ub294 \ub3c5\ud2b9\ud55c \ubb38\ud654\uc801 \uc815\uccb4\uc131\uc744 \uac00\uc9c0\uace0 \uc788\uc73c\uba70, \uacf5\uc2dd\uc801\uc73c\ub85c \uc774\uc911 \uc5b8\uc5b4 \uccb4\uc81c\uc774\ub2e4. \uc6e8\uc77c\uc2a4 \ub0b4\uc5d0\ub294 560,000\uba85 \uc774\uc0c1\uc758 \uc6e8\uc77c\uc2a4\uc5b4 \uc0ac\uc6a9\uc790\uac00 \uac70\uc8fc\ud558\uace0 \uc788\uc73c\uba70 \uc774\ub4e4\uc740 \uc8fc\ub85c \uc6e8\uc77c\uc2a4\uc758 \ubd81\ubd80\uc640 \uc11c\ubd80\uc5d0 \uac70\uc8fc\ud55c\ub2e4. 19\uc138\uae30 \ud6c4\ubc18\ubd80\ud130\ub294 \ub178\ub798, \ucda4, \uc2dc \ub0ad\uc1a1, \ubb38\ud559, \uc545\uae30, \uc791\uace1, \ubba4\uc9c0\uceec, \uc5f0\uadf9 \ub4f1\uc744 \uaca8\ub8e8\ub294 \ub300\ud68c\uc778 \uc774\uc2a4\ud14c\ub4dc\ubcf4\ub4dc \ub97c \ud1b5\ud574, \uc6e8\uc77c\uc2a4\ub294 '\ub178\ub798\uc758 \ub098\ub77c'\uc758 \uc774\ubbf8\uc9c0\ub85c \ub110\ub9ac \uc54c\ub824\uc9c0\uac8c \ub41c\ub2e4. \uc774\uc2a4\ud14c\ub4dc\ubcf4\ub4dc\ub294 1176\ub144\uc5d0 \ucd5c\ucd08 \ub300\ud68c\uac00 \uac1c\ucd5c\ub418\uc5b4 \ud604\uc7ac\uc5d0 \uc774\ub974\ub294 \uc720\uad6c\ud55c \uc804\ud1b5\uc744 \uc9c0\ub2c8\uace0 \uc788\ub2e4. \uc804\ud1b5\uc801\uc73c\ub85c \uc6e8\uc77c\uc2a4\uc778\ub4e4\uc740 \ub9e4\uc6b0 \ud65c\ubc1c\ud558\uace0 \uacbd\ucf8c\ud55c \uc0ac\ub78c\ub4e4\ub85c \uc54c\ub824\uc838 \uc654\uc73c\uba70, \uadf8\ub4e4\uc740 \uc774\uc6c3\uacfc \uc5b4\uc6b8\ub824 \ub178\ub798\ud558\uace0 \ucda4\ucd94\ub294 \uac83\uc744 \uc990\uacbc\ub2e4. \ub610\ud55c \ud558\ud504 \uc74c\uc545\ubb38\ud654\uac00 \ub9e4\uc6b0 \ubc1c\ub2ec\ud588\uace0 \uc74c\uc720\uc2dc\uc778\ub4e4\uc740 \uc6e8\uc77c\uc2a4\uc5b4\ub85c \uc2dc\ub97c \uc37c\ub2e4. \uc815\uae30\uc801\uc73c\ub85c \uc74c\uc720\uc2dc\uc778 \uacbd\uc5f0\ub300\ud68c\uac00 \uac1c\ucd5c\ub418\uc5c8\ub2e4. \uadf8\ub7ec\ub098 \uc774\ub7ec\ud55c \uc6e8\uc77c\uc2a4 \uace0\uc720\uc758 \ubb38\ud654\ub294 17\uc138\uae30 \ud6c4\ubc18\ubd80\ud130 \uc789\uae00\ub79c\ub4dc\uc758 \ubb38\ud654\uac00 \uc720\uc785\ub418\uae30 \uc2dc\uc791\ud558\uba74\uc11c \uc11c\uc11c\ud788 \uc0ac\ub77c\uc84c\ub2e4. \ud2b9\ud788 18\uc138\uae30 \ucd08\ubc18\uc5d0\ub294 \uc5c4\uccad\ub09c \uc18d\ub3c4\ub85c \uc6e8\uc77c\uc2a4\uc758 \uc804\ud1b5\ubb38\ud654\uac00 \ud1f4\uc0c9\ub418\uc5c8\ub2e4. 18\uc138\uae30 \ucd08\ubc18 \uc6e8\uc77c\uc2a4\uc5b4\ub85c \ub41c \uc11c\uc801\uc740 \uc790\ucde8\ub97c \uac10\ucd94\uc5c8\uace0 \ud558\ud504 \uac00\ub77d\uc5d0 \ub9de\ucdb0 \ub178\ub798\ud558\ub294 \uac83\uc740 \uc800\uc9c8\uc2a4\ub7f0 \uac83\uc73c\ub85c \uc778\uc2dd\uc774 \ub9cc\uc5f0\ud574\uc84c\ub2e4. \ud604\ub300\uc758 \uc6e8\uc77c\uc2a4 \ubb38\ud654\ub294 18\uc138\uae30 \ucd08\ubc18 \uc774\ud6c4 \uc789\uae00\ub79c\ub4dc \ubb38\ud654\uc640 \uc11e\uc784\uc5d0 \ub530\ub77c \uc789\uae00\ub79c\ub4dc \ubb38\ud654\uc640 \ud070 \ucc28\uc774\uac00 \uc5c6\uc5b4\uc84c\ub2e4.", "sentence_answer": "19\uc138\uae30 \ud6c4\ubc18\ubd80\ud130\ub294 \ub178\ub798, \ucda4, \uc2dc \ub0ad\uc1a1, \ubb38\ud559, \uc545\uae30, \uc791\uace1, \ubba4\uc9c0\uceec, \uc5f0\uadf9 \ub4f1\uc744 \uaca8\ub8e8\ub294 \ub300\ud68c\uc778 \uc774\uc2a4\ud14c\ub4dc\ubcf4\ub4dc \ub97c \ud1b5\ud574, \uc6e8\uc77c\uc2a4\ub294 '\ub178\ub798\uc758 \ub098\ub77c'\uc758 \uc774\ubbf8\uc9c0\ub85c \ub110\ub9ac \uc54c\ub824\uc9c0\uac8c \ub41c\ub2e4.", "paragraph_id": "5829149-2-1", "paragraph_question": "question: 19\uc138\uae30 \ud6c4\ubc18 \ub178\ub798, \ucda4, \uc2dc \ub4f1 \uc5ec\ub7ec\uac00\uc9c0 \uc608\uc220 \ubd84\uc57c\ub97c \uaca8\ub8e8\ub294 \ub300\ud68c\uc758 \uc774\ub984\uc740?, context: \uc6e8\uc77c\uc2a4\ub294 \uc601\uad6d\uc758 \ub2e4\ub978 \uc9c0\uc5ed\uacfc \uc815\uce58\uc801, \uc0ac\ud68c\uc801 \uc5ed\uc0ac\ub97c \uae34\ubc00\ud558\uac8c \uacf5\uc720\ud558\uace0 \uc788\uc9c0\ub9cc \uc778\uad6c\uc758 \ub300\ub2e4\uc218\ub294 \uc601\uc5b4\ub97c \uad6c\uc0ac\ud55c\ub2e4. \uc6e8\uc77c\uc2a4\ub294 \ub3c5\ud2b9\ud55c \ubb38\ud654\uc801 \uc815\uccb4\uc131\uc744 \uac00\uc9c0\uace0 \uc788\uc73c\uba70, \uacf5\uc2dd\uc801\uc73c\ub85c \uc774\uc911 \uc5b8\uc5b4 \uccb4\uc81c\uc774\ub2e4. \uc6e8\uc77c\uc2a4 \ub0b4\uc5d0\ub294 560,000\uba85 \uc774\uc0c1\uc758 \uc6e8\uc77c\uc2a4\uc5b4 \uc0ac\uc6a9\uc790\uac00 \uac70\uc8fc\ud558\uace0 \uc788\uc73c\uba70 \uc774\ub4e4\uc740 \uc8fc\ub85c \uc6e8\uc77c\uc2a4\uc758 \ubd81\ubd80\uc640 \uc11c\ubd80\uc5d0 \uac70\uc8fc\ud55c\ub2e4. 19\uc138\uae30 \ud6c4\ubc18\ubd80\ud130\ub294 \ub178\ub798, \ucda4, \uc2dc \ub0ad\uc1a1, \ubb38\ud559, \uc545\uae30, \uc791\uace1, \ubba4\uc9c0\uceec, \uc5f0\uadf9 \ub4f1\uc744 \uaca8\ub8e8\ub294 \ub300\ud68c\uc778 \uc774\uc2a4\ud14c\ub4dc\ubcf4\ub4dc\ub97c \ud1b5\ud574, \uc6e8\uc77c\uc2a4\ub294 '\ub178\ub798\uc758 \ub098\ub77c'\uc758 \uc774\ubbf8\uc9c0\ub85c \ub110\ub9ac \uc54c\ub824\uc9c0\uac8c \ub41c\ub2e4. \uc774\uc2a4\ud14c\ub4dc\ubcf4\ub4dc\ub294 1176\ub144\uc5d0 \ucd5c\ucd08 \ub300\ud68c\uac00 \uac1c\ucd5c\ub418\uc5b4 \ud604\uc7ac\uc5d0 \uc774\ub974\ub294 \uc720\uad6c\ud55c \uc804\ud1b5\uc744 \uc9c0\ub2c8\uace0 \uc788\ub2e4. \uc804\ud1b5\uc801\uc73c\ub85c \uc6e8\uc77c\uc2a4\uc778\ub4e4\uc740 \ub9e4\uc6b0 \ud65c\ubc1c\ud558\uace0 \uacbd\ucf8c\ud55c \uc0ac\ub78c\ub4e4\ub85c \uc54c\ub824\uc838 \uc654\uc73c\uba70, \uadf8\ub4e4\uc740 \uc774\uc6c3\uacfc \uc5b4\uc6b8\ub824 \ub178\ub798\ud558\uace0 \ucda4\ucd94\ub294 \uac83\uc744 \uc990\uacbc\ub2e4. \ub610\ud55c \ud558\ud504 \uc74c\uc545\ubb38\ud654\uac00 \ub9e4\uc6b0 \ubc1c\ub2ec\ud588\uace0 \uc74c\uc720\uc2dc\uc778\ub4e4\uc740 \uc6e8\uc77c\uc2a4\uc5b4\ub85c \uc2dc\ub97c \uc37c\ub2e4. \uc815\uae30\uc801\uc73c\ub85c \uc74c\uc720\uc2dc\uc778 \uacbd\uc5f0\ub300\ud68c\uac00 \uac1c\ucd5c\ub418\uc5c8\ub2e4. \uadf8\ub7ec\ub098 \uc774\ub7ec\ud55c \uc6e8\uc77c\uc2a4 \uace0\uc720\uc758 \ubb38\ud654\ub294 17\uc138\uae30 \ud6c4\ubc18\ubd80\ud130 \uc789\uae00\ub79c\ub4dc\uc758 \ubb38\ud654\uac00 \uc720\uc785\ub418\uae30 \uc2dc\uc791\ud558\uba74\uc11c \uc11c\uc11c\ud788 \uc0ac\ub77c\uc84c\ub2e4. \ud2b9\ud788 18\uc138\uae30 \ucd08\ubc18\uc5d0\ub294 \uc5c4\uccad\ub09c \uc18d\ub3c4\ub85c \uc6e8\uc77c\uc2a4\uc758 \uc804\ud1b5\ubb38\ud654\uac00 \ud1f4\uc0c9\ub418\uc5c8\ub2e4. 18\uc138\uae30 \ucd08\ubc18 \uc6e8\uc77c\uc2a4\uc5b4\ub85c \ub41c \uc11c\uc801\uc740 \uc790\ucde8\ub97c \uac10\ucd94\uc5c8\uace0 \ud558\ud504 \uac00\ub77d\uc5d0 \ub9de\ucdb0 \ub178\ub798\ud558\ub294 \uac83\uc740 \uc800\uc9c8\uc2a4\ub7f0 \uac83\uc73c\ub85c \uc778\uc2dd\uc774 \ub9cc\uc5f0\ud574\uc84c\ub2e4. \ud604\ub300\uc758 \uc6e8\uc77c\uc2a4 \ubb38\ud654\ub294 18\uc138\uae30 \ucd08\ubc18 \uc774\ud6c4 \uc789\uae00\ub79c\ub4dc \ubb38\ud654\uc640 \uc11e\uc784\uc5d0 \ub530\ub77c \uc789\uae00\ub79c\ub4dc \ubb38\ud654\uc640 \ud070 \ucc28\uc774\uac00 \uc5c6\uc5b4\uc84c\ub2e4."} {"question": "\uc774\uc2ac\ub78c\ud559\uad50\uc758 \ud788\ud504\uc988 \ucf54\uc2a4 \uc774\uc678\uc758 \ub2e4\ub978 \ucf54\uc2a4\ub294 \ubb34\uc5c7\uc785\ub2c8\uae4c?", "paragraph": "\uc601\uc5b4\uc5d0\uc11c \ub9c8\ub4dc\ub77c\uc0ac\ub77c\ub294 \ub2e8\uc5b4\ub294 \uc77c\ubc18\uc801\uc73c\ub85c \u2018\uc774\uc2ac\ub78c \uad50\uc721\uae30\uad00\u2019\uc73c\ub85c \ubc88\uc5ed\ub41c\ub2e4. \uc77c\ubc18\uc801\uc778 \uc774\uc2ac\ub78c \ud559\uad50\ub294 \ubcf4\ud1b5 \ub450 \uac00\uc9c0\uc758 \uad50\uc721 \ucf54\uc2a4\ub97c \uc81c\uacf5\ud55c\ub2e4. \ud788\ud504\uc988(\u1e25if\u1e93) \ucf54\uc2a4\ub294 \ucfe0\ub780\uc744 \uae30\uc5b5\ud558\ub3c4\ub85d \uad50\uc721\ud558\ub294 \ucf54\uc2a4\uc774\uba70, \uc644\uc804\ud55c \ucfe0\ub780\uc744 \uae30\uc5b5\ud558\uace0 \uc788\ub294 \uc0ac\ub78c\uc778 \ud558\ud53c\uc988(\u1e25\u0101fi\u1e93)\uac00 \ub418\ub294 \uac83\uc744 \ubaa9\uc801\uc73c\ub85c \ud55c\ub2e4. \ub2e4\ub978 \ucf54\uc2a4\ub294 \uc6b8\ub77c\ub9c8(\u02bf\u0101lim) \ucf54\uc2a4\uc778\ub370, \ud55c \uacf5\ub3d9\uccb4\uc5d0\uc11c \ud559\uc790\ub85c \uc9c0\uba85 \ubc1b\uae30\ub97c \uc6d0\ud558\ub294 \uc790\ub97c \ub300\uc0c1\uc73c\ub85c \ud558\ub294 \uad50\uc721\uacfc\uc815\uc774\ub2e4. \ub9c8\ub4dc\ub77c\uc0ac\uc5d0\uc11c\uc758 \uc77c\ubc18\uc801\uc778 \uad50\uc721\uacfc\uc815\uc740 \uc544\ub78d\uc5b4, \ud0c0\ud504\uc2dc\ub974(\u062a\u0641\u0633\u064a\u0631; \ucfe0\ub780\uc758 \ub0b4\uc6a9\uc5d0 \ub300\ud55c \ud574\uc11d), \uc0e4\ub9ac\uc544(\uc774\uc2ac\ub78c \uc728\ubc95), \ud558\ub514\uc2a4(\uc608\uc5b8\uc790 \ubb34\ud568\ub9c8\ub4dc\uc758 \uc5b8\ud589), \ub17c\ub9ac\ud559, \uc774\uc2ac\ub78c \uc5ed\uc0ac \ub4f1\uc73c\ub85c \uc774\ub8e8\uc5b4\uc838 \uc788\ub2e4. \uc774 \uc911 \ud558\ub514\uc2a4\ub294 \uadfc\ub300 \uc624\uc2a4\ub9cc \ud280\ub974\ud06c \uc81c\uad6d\uc758 \uc290\ub808\uc774\ub9cc 1\uc138\uc5d0 \uc758\ud574 \ub3c4\uc785\ub41c \uad50\uacfc\ubaa9\uc774\ub2e4. \uc77c\ubd80 \ub9c8\ub4dc\ub77c\uc0ac\uc5d0\uc11c\ub294 \ud559\uc0dd\ub4e4\uc758 \uc694\uccad\uc5d0 \uc758\ud574 \uc544\ub78d\ubb38\ud559, \uacfc\ud559, \uc138\uacc4\uc0ac, \uc601\uc5b4\ub098 \ud504\ub791\uc2a4\uc5b4\uc640 \uac19\uc740 \uc678\uad6d\uc5b4\ub97c \ubd80\uac00\uc801\uc73c\ub85c \uad50\uc721\ud558\uae30\ub3c4 \ud55c\ub2e4. \uc624\uc2a4\ub9cc \ud280\ub974\ud06c \uc81c\uad6d\uc758 \ub9c8\ub4dc\ub77c\uc0ac\ub294 \uc885\uad50 \uad50\uc721\uc744 \ud589\ud558\uba74\uc11c \uc791\ubb38, \ubb38\ubc95, \ud1b5\uc0ac\ub860, \uc2dc\ubb38\ud559, \uc815\uc11c\ubc95, \uc790\uc5f0\uacfc\ud559, \uc815\uce58\ud559, \uc0dd\ud65c\uc608\ubc95\uc744 \uad50\uc721\ud558\uae30\ub3c4 \ud588\uc5c8\ub2e4.", "answer": "\uc6b8\ub77c\ub9c8", "sentence": "\ub2e4\ub978 \ucf54\uc2a4\ub294 \uc6b8\ub77c\ub9c8 (\u02bf\u0101lim) \ucf54\uc2a4\uc778\ub370, \ud55c \uacf5\ub3d9\uccb4\uc5d0\uc11c \ud559\uc790\ub85c \uc9c0\uba85 \ubc1b\uae30\ub97c \uc6d0\ud558\ub294 \uc790\ub97c \ub300\uc0c1\uc73c\ub85c \ud558\ub294 \uad50\uc721\uacfc\uc815\uc774\ub2e4.", "paragraph_sentence": "\uc601\uc5b4\uc5d0\uc11c \ub9c8\ub4dc\ub77c\uc0ac\ub77c\ub294 \ub2e8\uc5b4\ub294 \uc77c\ubc18\uc801\uc73c\ub85c \u2018\uc774\uc2ac\ub78c \uad50\uc721\uae30\uad00\u2019\uc73c\ub85c \ubc88\uc5ed\ub41c\ub2e4. \uc77c\ubc18\uc801\uc778 \uc774\uc2ac\ub78c \ud559\uad50\ub294 \ubcf4\ud1b5 \ub450 \uac00\uc9c0\uc758 \uad50\uc721 \ucf54\uc2a4\ub97c \uc81c\uacf5\ud55c\ub2e4. \ud788\ud504\uc988(\u1e25if\u1e93) \ucf54\uc2a4\ub294 \ucfe0\ub780\uc744 \uae30\uc5b5\ud558\ub3c4\ub85d \uad50\uc721\ud558\ub294 \ucf54\uc2a4\uc774\uba70, \uc644\uc804\ud55c \ucfe0\ub780\uc744 \uae30\uc5b5\ud558\uace0 \uc788\ub294 \uc0ac\ub78c\uc778 \ud558\ud53c\uc988(\u1e25\u0101fi\u1e93)\uac00 \ub418\ub294 \uac83\uc744 \ubaa9\uc801\uc73c\ub85c \ud55c\ub2e4. \ub2e4\ub978 \ucf54\uc2a4\ub294 \uc6b8\ub77c\ub9c8 (\u02bf\u0101lim) \ucf54\uc2a4\uc778\ub370, \ud55c \uacf5\ub3d9\uccb4\uc5d0\uc11c \ud559\uc790\ub85c \uc9c0\uba85 \ubc1b\uae30\ub97c \uc6d0\ud558\ub294 \uc790\ub97c \ub300\uc0c1\uc73c\ub85c \ud558\ub294 \uad50\uc721\uacfc\uc815\uc774\ub2e4. \ub9c8\ub4dc\ub77c\uc0ac\uc5d0\uc11c\uc758 \uc77c\ubc18\uc801\uc778 \uad50\uc721\uacfc\uc815\uc740 \uc544\ub78d\uc5b4, \ud0c0\ud504\uc2dc\ub974(\u062a\u0641\u0633\u064a\u0631; \ucfe0\ub780\uc758 \ub0b4\uc6a9\uc5d0 \ub300\ud55c \ud574\uc11d), \uc0e4\ub9ac\uc544(\uc774\uc2ac\ub78c \uc728\ubc95), \ud558\ub514\uc2a4(\uc608\uc5b8\uc790 \ubb34\ud568\ub9c8\ub4dc\uc758 \uc5b8\ud589), \ub17c\ub9ac\ud559, \uc774\uc2ac\ub78c \uc5ed\uc0ac \ub4f1\uc73c\ub85c \uc774\ub8e8\uc5b4\uc838 \uc788\ub2e4. \uc774 \uc911 \ud558\ub514\uc2a4\ub294 \uadfc\ub300 \uc624\uc2a4\ub9cc \ud280\ub974\ud06c \uc81c\uad6d\uc758 \uc290\ub808\uc774\ub9cc 1\uc138\uc5d0 \uc758\ud574 \ub3c4\uc785\ub41c \uad50\uacfc\ubaa9\uc774\ub2e4. \uc77c\ubd80 \ub9c8\ub4dc\ub77c\uc0ac\uc5d0\uc11c\ub294 \ud559\uc0dd\ub4e4\uc758 \uc694\uccad\uc5d0 \uc758\ud574 \uc544\ub78d\ubb38\ud559, \uacfc\ud559, \uc138\uacc4\uc0ac, \uc601\uc5b4\ub098 \ud504\ub791\uc2a4\uc5b4\uc640 \uac19\uc740 \uc678\uad6d\uc5b4\ub97c \ubd80\uac00\uc801\uc73c\ub85c \uad50\uc721\ud558\uae30\ub3c4 \ud55c\ub2e4. \uc624\uc2a4\ub9cc \ud280\ub974\ud06c \uc81c\uad6d\uc758 \ub9c8\ub4dc\ub77c\uc0ac\ub294 \uc885\uad50 \uad50\uc721\uc744 \ud589\ud558\uba74\uc11c \uc791\ubb38, \ubb38\ubc95, \ud1b5\uc0ac\ub860, \uc2dc\ubb38\ud559, \uc815\uc11c\ubc95, \uc790\uc5f0\uacfc\ud559, \uc815\uce58\ud559, \uc0dd\ud65c\uc608\ubc95\uc744 \uad50\uc721\ud558\uae30\ub3c4 \ud588\uc5c8\ub2e4.", "paragraph_answer": "\uc601\uc5b4\uc5d0\uc11c \ub9c8\ub4dc\ub77c\uc0ac\ub77c\ub294 \ub2e8\uc5b4\ub294 \uc77c\ubc18\uc801\uc73c\ub85c \u2018\uc774\uc2ac\ub78c \uad50\uc721\uae30\uad00\u2019\uc73c\ub85c \ubc88\uc5ed\ub41c\ub2e4. \uc77c\ubc18\uc801\uc778 \uc774\uc2ac\ub78c \ud559\uad50\ub294 \ubcf4\ud1b5 \ub450 \uac00\uc9c0\uc758 \uad50\uc721 \ucf54\uc2a4\ub97c \uc81c\uacf5\ud55c\ub2e4. \ud788\ud504\uc988(\u1e25if\u1e93) \ucf54\uc2a4\ub294 \ucfe0\ub780\uc744 \uae30\uc5b5\ud558\ub3c4\ub85d \uad50\uc721\ud558\ub294 \ucf54\uc2a4\uc774\uba70, \uc644\uc804\ud55c \ucfe0\ub780\uc744 \uae30\uc5b5\ud558\uace0 \uc788\ub294 \uc0ac\ub78c\uc778 \ud558\ud53c\uc988(\u1e25\u0101fi\u1e93)\uac00 \ub418\ub294 \uac83\uc744 \ubaa9\uc801\uc73c\ub85c \ud55c\ub2e4. \ub2e4\ub978 \ucf54\uc2a4\ub294 \uc6b8\ub77c\ub9c8 (\u02bf\u0101lim) \ucf54\uc2a4\uc778\ub370, \ud55c \uacf5\ub3d9\uccb4\uc5d0\uc11c \ud559\uc790\ub85c \uc9c0\uba85 \ubc1b\uae30\ub97c \uc6d0\ud558\ub294 \uc790\ub97c \ub300\uc0c1\uc73c\ub85c \ud558\ub294 \uad50\uc721\uacfc\uc815\uc774\ub2e4. \ub9c8\ub4dc\ub77c\uc0ac\uc5d0\uc11c\uc758 \uc77c\ubc18\uc801\uc778 \uad50\uc721\uacfc\uc815\uc740 \uc544\ub78d\uc5b4, \ud0c0\ud504\uc2dc\ub974(\u062a\u0641\u0633\u064a\u0631; \ucfe0\ub780\uc758 \ub0b4\uc6a9\uc5d0 \ub300\ud55c \ud574\uc11d), \uc0e4\ub9ac\uc544(\uc774\uc2ac\ub78c \uc728\ubc95), \ud558\ub514\uc2a4(\uc608\uc5b8\uc790 \ubb34\ud568\ub9c8\ub4dc\uc758 \uc5b8\ud589), \ub17c\ub9ac\ud559, \uc774\uc2ac\ub78c \uc5ed\uc0ac \ub4f1\uc73c\ub85c \uc774\ub8e8\uc5b4\uc838 \uc788\ub2e4. \uc774 \uc911 \ud558\ub514\uc2a4\ub294 \uadfc\ub300 \uc624\uc2a4\ub9cc \ud280\ub974\ud06c \uc81c\uad6d\uc758 \uc290\ub808\uc774\ub9cc 1\uc138\uc5d0 \uc758\ud574 \ub3c4\uc785\ub41c \uad50\uacfc\ubaa9\uc774\ub2e4. \uc77c\ubd80 \ub9c8\ub4dc\ub77c\uc0ac\uc5d0\uc11c\ub294 \ud559\uc0dd\ub4e4\uc758 \uc694\uccad\uc5d0 \uc758\ud574 \uc544\ub78d\ubb38\ud559, \uacfc\ud559, \uc138\uacc4\uc0ac, \uc601\uc5b4\ub098 \ud504\ub791\uc2a4\uc5b4\uc640 \uac19\uc740 \uc678\uad6d\uc5b4\ub97c \ubd80\uac00\uc801\uc73c\ub85c \uad50\uc721\ud558\uae30\ub3c4 \ud55c\ub2e4. \uc624\uc2a4\ub9cc \ud280\ub974\ud06c \uc81c\uad6d\uc758 \ub9c8\ub4dc\ub77c\uc0ac\ub294 \uc885\uad50 \uad50\uc721\uc744 \ud589\ud558\uba74\uc11c \uc791\ubb38, \ubb38\ubc95, \ud1b5\uc0ac\ub860, \uc2dc\ubb38\ud559, \uc815\uc11c\ubc95, \uc790\uc5f0\uacfc\ud559, \uc815\uce58\ud559, \uc0dd\ud65c\uc608\ubc95\uc744 \uad50\uc721\ud558\uae30\ub3c4 \ud588\uc5c8\ub2e4.", "sentence_answer": "\ub2e4\ub978 \ucf54\uc2a4\ub294 \uc6b8\ub77c\ub9c8 (\u02bf\u0101lim) \ucf54\uc2a4\uc778\ub370, \ud55c \uacf5\ub3d9\uccb4\uc5d0\uc11c \ud559\uc790\ub85c \uc9c0\uba85 \ubc1b\uae30\ub97c \uc6d0\ud558\ub294 \uc790\ub97c \ub300\uc0c1\uc73c\ub85c \ud558\ub294 \uad50\uc721\uacfc\uc815\uc774\ub2e4.", "paragraph_id": "6498964-1-1", "paragraph_question": "question: \uc774\uc2ac\ub78c\ud559\uad50\uc758 \ud788\ud504\uc988 \ucf54\uc2a4 \uc774\uc678\uc758 \ub2e4\ub978 \ucf54\uc2a4\ub294 \ubb34\uc5c7\uc785\ub2c8\uae4c?, context: \uc601\uc5b4\uc5d0\uc11c \ub9c8\ub4dc\ub77c\uc0ac\ub77c\ub294 \ub2e8\uc5b4\ub294 \uc77c\ubc18\uc801\uc73c\ub85c \u2018\uc774\uc2ac\ub78c \uad50\uc721\uae30\uad00\u2019\uc73c\ub85c \ubc88\uc5ed\ub41c\ub2e4. \uc77c\ubc18\uc801\uc778 \uc774\uc2ac\ub78c \ud559\uad50\ub294 \ubcf4\ud1b5 \ub450 \uac00\uc9c0\uc758 \uad50\uc721 \ucf54\uc2a4\ub97c \uc81c\uacf5\ud55c\ub2e4. \ud788\ud504\uc988(\u1e25if\u1e93) \ucf54\uc2a4\ub294 \ucfe0\ub780\uc744 \uae30\uc5b5\ud558\ub3c4\ub85d \uad50\uc721\ud558\ub294 \ucf54\uc2a4\uc774\uba70, \uc644\uc804\ud55c \ucfe0\ub780\uc744 \uae30\uc5b5\ud558\uace0 \uc788\ub294 \uc0ac\ub78c\uc778 \ud558\ud53c\uc988(\u1e25\u0101fi\u1e93)\uac00 \ub418\ub294 \uac83\uc744 \ubaa9\uc801\uc73c\ub85c \ud55c\ub2e4. \ub2e4\ub978 \ucf54\uc2a4\ub294 \uc6b8\ub77c\ub9c8(\u02bf\u0101lim) \ucf54\uc2a4\uc778\ub370, \ud55c \uacf5\ub3d9\uccb4\uc5d0\uc11c \ud559\uc790\ub85c \uc9c0\uba85 \ubc1b\uae30\ub97c \uc6d0\ud558\ub294 \uc790\ub97c \ub300\uc0c1\uc73c\ub85c \ud558\ub294 \uad50\uc721\uacfc\uc815\uc774\ub2e4. \ub9c8\ub4dc\ub77c\uc0ac\uc5d0\uc11c\uc758 \uc77c\ubc18\uc801\uc778 \uad50\uc721\uacfc\uc815\uc740 \uc544\ub78d\uc5b4, \ud0c0\ud504\uc2dc\ub974(\u062a\u0641\u0633\u064a\u0631; \ucfe0\ub780\uc758 \ub0b4\uc6a9\uc5d0 \ub300\ud55c \ud574\uc11d), \uc0e4\ub9ac\uc544(\uc774\uc2ac\ub78c \uc728\ubc95), \ud558\ub514\uc2a4(\uc608\uc5b8\uc790 \ubb34\ud568\ub9c8\ub4dc\uc758 \uc5b8\ud589), \ub17c\ub9ac\ud559, \uc774\uc2ac\ub78c \uc5ed\uc0ac \ub4f1\uc73c\ub85c \uc774\ub8e8\uc5b4\uc838 \uc788\ub2e4. \uc774 \uc911 \ud558\ub514\uc2a4\ub294 \uadfc\ub300 \uc624\uc2a4\ub9cc \ud280\ub974\ud06c \uc81c\uad6d\uc758 \uc290\ub808\uc774\ub9cc 1\uc138\uc5d0 \uc758\ud574 \ub3c4\uc785\ub41c \uad50\uacfc\ubaa9\uc774\ub2e4. \uc77c\ubd80 \ub9c8\ub4dc\ub77c\uc0ac\uc5d0\uc11c\ub294 \ud559\uc0dd\ub4e4\uc758 \uc694\uccad\uc5d0 \uc758\ud574 \uc544\ub78d\ubb38\ud559, \uacfc\ud559, \uc138\uacc4\uc0ac, \uc601\uc5b4\ub098 \ud504\ub791\uc2a4\uc5b4\uc640 \uac19\uc740 \uc678\uad6d\uc5b4\ub97c \ubd80\uac00\uc801\uc73c\ub85c \uad50\uc721\ud558\uae30\ub3c4 \ud55c\ub2e4. \uc624\uc2a4\ub9cc \ud280\ub974\ud06c \uc81c\uad6d\uc758 \ub9c8\ub4dc\ub77c\uc0ac\ub294 \uc885\uad50 \uad50\uc721\uc744 \ud589\ud558\uba74\uc11c \uc791\ubb38, \ubb38\ubc95, \ud1b5\uc0ac\ub860, \uc2dc\ubb38\ud559, \uc815\uc11c\ubc95, \uc790\uc5f0\uacfc\ud559, \uc815\uce58\ud559, \uc0dd\ud65c\uc608\ubc95\uc744 \uad50\uc721\ud558\uae30\ub3c4 \ud588\uc5c8\ub2e4."} {"question": "\uae40\ud55c\uc194\uc758 \ucd9c\uc0dd\uc77c\uc740?", "paragraph": "\uae40\ud55c\uc194\uc740 1995\ub144 6\uc6d4 16\uc77c\uc5d0 \ud3c9\uc591\uc5d0\uc11c \ud0dc\uc5b4\ub0ac\ub2e4. \uc544\ubc84\uc9c0 \uae40\uc815\ub0a8\uc774 \ud6c4\uacc4 \uad6c\ub3c4\uc5d0\uc11c \ubc00\ub9ac\uc790 \uc5ec\ub7ec\ub098\ub77c\ub97c \ub5a0\ub3cc\uc544 \ub2e4\ub154\uace0, \uc774\ud6c4 2011\ub144 10\uc6d4 \ubcf4\uc2a4\ub2c8\uc544 \ud5e4\ub974\uccb4\uace0\ube44\ub098\uc758 \uad6d\uc81c\ud559\uad50\uc5d0 \uc785\ud559\ud588\uace0 \ubcf4\uc2a4\ub2c8\uc544 \ub2f9\uad6d\uc774 \ucde8\ud559 \ube44\uc790\ub97c \ubc1c\uae09\ud574 \uc8fc\uc5c8\ub2e4. \uadf8\ub294 \ub2f9\ucd08 \ud64d\ucf69\uc758 \ub9ac\ud3ec\ucd98 \uc720\ub098\uc774\ud2f0\ub4dc \uc6d4\ub4dc\uce7c\ub9ac\uc9c0(Li Po Chun United World College)\uc5d0 \ub4f1\ub85d\ud560 \uc608\uc815\uc774\uc5c8\uc9c0\ub9cc \ud64d\ucf69 \ub2f9\uad6d\uc774 \ube44\uc790 \ubc1c\uae09\uc744 \uac70\uc808\ud558\uc5ec 2012\ub144\ubd80\ud130 \uc720\ub7fd\uc5d0\uc11c \ud559\uad50\ub97c \ub2e4\ub2c8\uae30\ub85c \ud558\uc600\uace0, \ubcf4\uc2a4\ub2c8\uc544\ud589\uc744 \uacb0\uc815\ud558\uc600\ub2e4\uace0 \ud55c\ub2e4. 2013\ub144 5\uc6d4 30\uc77c\uc5d0 \uad6d\uc81c\ud559\uad50\ub97c \uc878\uc5c5\ud588\uace0 \uadf8\ud574 2013\ub144 8\uc6d4\uc5d0 \ud30c\ub9ac \uc815\uce58 \ub300\ud559\uc5d0 \uc9c4\ud559, 2016\ub144 \uc878\uc5c5\ud588\ub2e4. 2017\ub144 1\uc6d4\uc758 \uc544\ubc84\uc9c0 \uae40\uc815\ub0a8\uc774 \ubd81\ud55c\uc758 \uc0ac\uc8fc\ub97c \ubc1b\uc740 \uac83\uc73c\ub85c \ucd94\uc815\ub418\ub294 \ubca0\ud2b8\ub0a8 \uc778\ub3c4\ub124\uc2dc\uc544 \uad6d\uc801\uc758 \uc5ec\uc131\ub4e4\uc5d0\uac8c \uc0b4\ud574\ub2f9\ud55c \ud6c4\uc5d0 \ub9dd\uba85\uc744 \ud558\uc600\uc9c0\ub9cc, 2018\ub144 \ud604\uc7ac\uae4c\uc9c0 \uadf8\uc758 \uc18c\uc7ac\ub294 \uba85\ud655\ud558\uac8c \ud655\uc778\ub418\uace0 \uc788\uc9c0 \uc54a\ub2e4.", "answer": "1995\ub144 6\uc6d4 16\uc77c", "sentence": "\uae40\ud55c\uc194\uc740 1995\ub144 6\uc6d4 16\uc77c \uc5d0 \ud3c9\uc591\uc5d0\uc11c \ud0dc\uc5b4\ub0ac\ub2e4.", "paragraph_sentence": " \uae40\ud55c\uc194\uc740 1995\ub144 6\uc6d4 16\uc77c \uc5d0 \ud3c9\uc591\uc5d0\uc11c \ud0dc\uc5b4\ub0ac\ub2e4. \uc544\ubc84\uc9c0 \uae40\uc815\ub0a8\uc774 \ud6c4\uacc4 \uad6c\ub3c4\uc5d0\uc11c \ubc00\ub9ac\uc790 \uc5ec\ub7ec\ub098\ub77c\ub97c \ub5a0\ub3cc\uc544 \ub2e4\ub154\uace0, \uc774\ud6c4 2011\ub144 10\uc6d4 \ubcf4\uc2a4\ub2c8\uc544 \ud5e4\ub974\uccb4\uace0\ube44\ub098\uc758 \uad6d\uc81c\ud559\uad50\uc5d0 \uc785\ud559\ud588\uace0 \ubcf4\uc2a4\ub2c8\uc544 \ub2f9\uad6d\uc774 \ucde8\ud559 \ube44\uc790\ub97c \ubc1c\uae09\ud574 \uc8fc\uc5c8\ub2e4. \uadf8\ub294 \ub2f9\ucd08 \ud64d\ucf69\uc758 \ub9ac\ud3ec\ucd98 \uc720\ub098\uc774\ud2f0\ub4dc \uc6d4\ub4dc\uce7c\ub9ac\uc9c0(Li Po Chun United World College)\uc5d0 \ub4f1\ub85d\ud560 \uc608\uc815\uc774\uc5c8\uc9c0\ub9cc \ud64d\ucf69 \ub2f9\uad6d\uc774 \ube44\uc790 \ubc1c\uae09\uc744 \uac70\uc808\ud558\uc5ec 2012\ub144\ubd80\ud130 \uc720\ub7fd\uc5d0\uc11c \ud559\uad50\ub97c \ub2e4\ub2c8\uae30\ub85c \ud558\uc600\uace0, \ubcf4\uc2a4\ub2c8\uc544\ud589\uc744 \uacb0\uc815\ud558\uc600\ub2e4\uace0 \ud55c\ub2e4. 2013\ub144 5\uc6d4 30\uc77c\uc5d0 \uad6d\uc81c\ud559\uad50\ub97c \uc878\uc5c5\ud588\uace0 \uadf8\ud574 2013\ub144 8\uc6d4\uc5d0 \ud30c\ub9ac \uc815\uce58 \ub300\ud559\uc5d0 \uc9c4\ud559, 2016\ub144 \uc878\uc5c5\ud588\ub2e4. 2017\ub144 1\uc6d4\uc758 \uc544\ubc84\uc9c0 \uae40\uc815\ub0a8\uc774 \ubd81\ud55c\uc758 \uc0ac\uc8fc\ub97c \ubc1b\uc740 \uac83\uc73c\ub85c \ucd94\uc815\ub418\ub294 \ubca0\ud2b8\ub0a8 \uc778\ub3c4\ub124\uc2dc\uc544 \uad6d\uc801\uc758 \uc5ec\uc131\ub4e4\uc5d0\uac8c \uc0b4\ud574\ub2f9\ud55c \ud6c4\uc5d0 \ub9dd\uba85\uc744 \ud558\uc600\uc9c0\ub9cc, 2018\ub144 \ud604\uc7ac\uae4c\uc9c0 \uadf8\uc758 \uc18c\uc7ac\ub294 \uba85\ud655\ud558\uac8c \ud655\uc778\ub418\uace0 \uc788\uc9c0 \uc54a\ub2e4.", "paragraph_answer": "\uae40\ud55c\uc194\uc740 1995\ub144 6\uc6d4 16\uc77c \uc5d0 \ud3c9\uc591\uc5d0\uc11c \ud0dc\uc5b4\ub0ac\ub2e4. \uc544\ubc84\uc9c0 \uae40\uc815\ub0a8\uc774 \ud6c4\uacc4 \uad6c\ub3c4\uc5d0\uc11c \ubc00\ub9ac\uc790 \uc5ec\ub7ec\ub098\ub77c\ub97c \ub5a0\ub3cc\uc544 \ub2e4\ub154\uace0, \uc774\ud6c4 2011\ub144 10\uc6d4 \ubcf4\uc2a4\ub2c8\uc544 \ud5e4\ub974\uccb4\uace0\ube44\ub098\uc758 \uad6d\uc81c\ud559\uad50\uc5d0 \uc785\ud559\ud588\uace0 \ubcf4\uc2a4\ub2c8\uc544 \ub2f9\uad6d\uc774 \ucde8\ud559 \ube44\uc790\ub97c \ubc1c\uae09\ud574 \uc8fc\uc5c8\ub2e4. \uadf8\ub294 \ub2f9\ucd08 \ud64d\ucf69\uc758 \ub9ac\ud3ec\ucd98 \uc720\ub098\uc774\ud2f0\ub4dc \uc6d4\ub4dc\uce7c\ub9ac\uc9c0(Li Po Chun United World College)\uc5d0 \ub4f1\ub85d\ud560 \uc608\uc815\uc774\uc5c8\uc9c0\ub9cc \ud64d\ucf69 \ub2f9\uad6d\uc774 \ube44\uc790 \ubc1c\uae09\uc744 \uac70\uc808\ud558\uc5ec 2012\ub144\ubd80\ud130 \uc720\ub7fd\uc5d0\uc11c \ud559\uad50\ub97c \ub2e4\ub2c8\uae30\ub85c \ud558\uc600\uace0, \ubcf4\uc2a4\ub2c8\uc544\ud589\uc744 \uacb0\uc815\ud558\uc600\ub2e4\uace0 \ud55c\ub2e4. 2013\ub144 5\uc6d4 30\uc77c\uc5d0 \uad6d\uc81c\ud559\uad50\ub97c \uc878\uc5c5\ud588\uace0 \uadf8\ud574 2013\ub144 8\uc6d4\uc5d0 \ud30c\ub9ac \uc815\uce58 \ub300\ud559\uc5d0 \uc9c4\ud559, 2016\ub144 \uc878\uc5c5\ud588\ub2e4. 2017\ub144 1\uc6d4\uc758 \uc544\ubc84\uc9c0 \uae40\uc815\ub0a8\uc774 \ubd81\ud55c\uc758 \uc0ac\uc8fc\ub97c \ubc1b\uc740 \uac83\uc73c\ub85c \ucd94\uc815\ub418\ub294 \ubca0\ud2b8\ub0a8 \uc778\ub3c4\ub124\uc2dc\uc544 \uad6d\uc801\uc758 \uc5ec\uc131\ub4e4\uc5d0\uac8c \uc0b4\ud574\ub2f9\ud55c \ud6c4\uc5d0 \ub9dd\uba85\uc744 \ud558\uc600\uc9c0\ub9cc, 2018\ub144 \ud604\uc7ac\uae4c\uc9c0 \uadf8\uc758 \uc18c\uc7ac\ub294 \uba85\ud655\ud558\uac8c \ud655\uc778\ub418\uace0 \uc788\uc9c0 \uc54a\ub2e4.", "sentence_answer": "\uae40\ud55c\uc194\uc740 1995\ub144 6\uc6d4 16\uc77c \uc5d0 \ud3c9\uc591\uc5d0\uc11c \ud0dc\uc5b4\ub0ac\ub2e4.", "paragraph_id": "6482923-0-0", "paragraph_question": "question: \uae40\ud55c\uc194\uc758 \ucd9c\uc0dd\uc77c\uc740?, context: \uae40\ud55c\uc194\uc740 1995\ub144 6\uc6d4 16\uc77c\uc5d0 \ud3c9\uc591\uc5d0\uc11c \ud0dc\uc5b4\ub0ac\ub2e4. \uc544\ubc84\uc9c0 \uae40\uc815\ub0a8\uc774 \ud6c4\uacc4 \uad6c\ub3c4\uc5d0\uc11c \ubc00\ub9ac\uc790 \uc5ec\ub7ec\ub098\ub77c\ub97c \ub5a0\ub3cc\uc544 \ub2e4\ub154\uace0, \uc774\ud6c4 2011\ub144 10\uc6d4 \ubcf4\uc2a4\ub2c8\uc544 \ud5e4\ub974\uccb4\uace0\ube44\ub098\uc758 \uad6d\uc81c\ud559\uad50\uc5d0 \uc785\ud559\ud588\uace0 \ubcf4\uc2a4\ub2c8\uc544 \ub2f9\uad6d\uc774 \ucde8\ud559 \ube44\uc790\ub97c \ubc1c\uae09\ud574 \uc8fc\uc5c8\ub2e4. \uadf8\ub294 \ub2f9\ucd08 \ud64d\ucf69\uc758 \ub9ac\ud3ec\ucd98 \uc720\ub098\uc774\ud2f0\ub4dc \uc6d4\ub4dc\uce7c\ub9ac\uc9c0(Li Po Chun United World College)\uc5d0 \ub4f1\ub85d\ud560 \uc608\uc815\uc774\uc5c8\uc9c0\ub9cc \ud64d\ucf69 \ub2f9\uad6d\uc774 \ube44\uc790 \ubc1c\uae09\uc744 \uac70\uc808\ud558\uc5ec 2012\ub144\ubd80\ud130 \uc720\ub7fd\uc5d0\uc11c \ud559\uad50\ub97c \ub2e4\ub2c8\uae30\ub85c \ud558\uc600\uace0, \ubcf4\uc2a4\ub2c8\uc544\ud589\uc744 \uacb0\uc815\ud558\uc600\ub2e4\uace0 \ud55c\ub2e4. 2013\ub144 5\uc6d4 30\uc77c\uc5d0 \uad6d\uc81c\ud559\uad50\ub97c \uc878\uc5c5\ud588\uace0 \uadf8\ud574 2013\ub144 8\uc6d4\uc5d0 \ud30c\ub9ac \uc815\uce58 \ub300\ud559\uc5d0 \uc9c4\ud559, 2016\ub144 \uc878\uc5c5\ud588\ub2e4. 2017\ub144 1\uc6d4\uc758 \uc544\ubc84\uc9c0 \uae40\uc815\ub0a8\uc774 \ubd81\ud55c\uc758 \uc0ac\uc8fc\ub97c \ubc1b\uc740 \uac83\uc73c\ub85c \ucd94\uc815\ub418\ub294 \ubca0\ud2b8\ub0a8 \uc778\ub3c4\ub124\uc2dc\uc544 \uad6d\uc801\uc758 \uc5ec\uc131\ub4e4\uc5d0\uac8c \uc0b4\ud574\ub2f9\ud55c \ud6c4\uc5d0 \ub9dd\uba85\uc744 \ud558\uc600\uc9c0\ub9cc, 2018\ub144 \ud604\uc7ac\uae4c\uc9c0 \uadf8\uc758 \uc18c\uc7ac\ub294 \uba85\ud655\ud558\uac8c \ud655\uc778\ub418\uace0 \uc788\uc9c0 \uc54a\ub2e4."} {"question": "\uc120\uc870\ub294 1591\ub144 2\uc6d4 \uc774\uc21c\uc2e0\uc744 \uc815\uc74d\ud604\uac10\uc5d0\uc11c \ubb34\uc5c7\uc73c\ub85c \uc2b9\uc9c4\uc2dc\ucf30\uc2b5\ub2c8\uae4c?", "paragraph": "1591\ub144 2\uc6d4\uc5d0 \uc120\uc870\ub294 \uc774\ucc9c\u00b7\uc774\uc5b5\uae30\u00b7\uc591\uc751\uc9c0\u00b7\uc774\uc21c\uc2e0\uc744 \ub0a8\ucabd \uc694\ud574\uc9c0\uc5d0 \uc784\uba85\ud558\uc5ec \uacf5\uc744 \uc138\uc6b0\uac8c \ud558\ub77c\ub294 \uc804\uad50\ub97c \ub0b4\ub838\ub2e4. \uc120\uc870\ub294 \uc2e0\ud558\ub4e4\uc758 \ubc18\ubc1c\uacfc \ub17c\ud575\uc744 \ud53c\ud558\uae30 \uc704\ud574 \ubcbc\uc2ac\uc758 \uac01 \ub2e8\uacc4\ub9c8\ub2e4 \uc784\uba85\ud558\uc5ec \uc81c\uc218\ud558\uace0 \uc2b9\uc9c4\uc2dc\ud0a4\ub294 \ubc29\ubc95\uc744 \uc368\uc11c 1591\ub144 2\uc6d4 \uc774\uc21c\uc2e0\uc744 \uc815\uc74d\ud604\uac10\uc5d0\uc11c \uc9c4\ub3c4\uad70\uc218(\u73cd\u5cf6\u90e1\u5b88)\ub85c \uc2b9\uc9c4\uc2dc\ucf30\uace0, \uadf8\uac00 \ubd80\uc784\uc9c0\uc5d0 \ubd80\uc784\ud558\uae30\ub3c4 \uc804\uc5d0 \uac00\ub9ac\ud3ec\ucca8\uc808\uc81c\uc0ac(\u52a0\u91cc\u6d66\u50c9\u7bc0\u5236\u4f7f)\ub85c \uc804\uc784\uc2dc\ucf30\ub2e4. \uc774\uc5b4 \uc120\uc870\ub294 \uc774\uc21c\uc2e0\uc774 \uac00\ub9ac\ud3ec\uc5d0 \ubd80\uc784\ud558\uae30\ub3c4 \uc804\uc5d0 \ub2e4\uc2dc \uc804\ub77c\uc88c\uc218\uc0ac\uc5d0 \uc804\uc784\ud558\uac8c \ud558\uc600\ub2e4. \ub610\ud55c, \uc120\uc870\ub294 \uc774\uc21c\uc2e0\uacfc \uac19\uc774 \ubc31\uc758\uc885\uad70\uc744 \ud558\uc600\ub358 \uc774\uacbd\ub85d\ub3c4 \uc804\ub77c\ub3c4\uc758 \uc694\uc9c0\uc778 \ub098\uc8fc\ubaa9\uc0ac\uc5d0 \uc81c\uc218\ud558\uc600\ub2e4. \uacc4\uc18d\ub418\ub294 \ub300\uc2e0\ub4e4\uc758 \ubc18\ub300\uc5d0\ub3c4 \ubd88\uad6c\ud558\uace0 \uc120\uc870\ub294 \uc774\uc21c\uc2e0\uc744 \uc218\uc0ac\ub85c \uc0bc\uc73c\ub824\ub294 \uc2ec\uc9c0\ub97c \uad73\ud614\ub2e4. \uadf8\ub294 \uc774\uacbd\ub85d\uc758 \ubc1c\ud0c1\uc740 \ucc9c\ucc9c\ud788 \uc0dd\uac01\ud558\uc5ec \uacb0\uc815\ud558\uaca0\ub2e4\uace0 \ud558\uba74\uc11c\ub3c4 \uc774\uc21c\uc2e0\uc758 \uc218\uc0ac \ubc1c\ud0c1\uc740 \uac1c\uc815\uc740 \ud560 \uc218 \uc5c6\ub2e4\uace0 \ud558\uba70 \ubc00\uc5b4\ubd99\uc600\ub2e4.", "answer": "\uc9c4\ub3c4\uad70\uc218", "sentence": "\uc120\uc870\ub294 \uc2e0\ud558\ub4e4\uc758 \ubc18\ubc1c\uacfc \ub17c\ud575\uc744 \ud53c\ud558\uae30 \uc704\ud574 \ubcbc\uc2ac\uc758 \uac01 \ub2e8\uacc4\ub9c8\ub2e4 \uc784\uba85\ud558\uc5ec \uc81c\uc218\ud558\uace0 \uc2b9\uc9c4\uc2dc\ud0a4\ub294 \ubc29\ubc95\uc744 \uc368\uc11c 1591\ub144 2\uc6d4 \uc774\uc21c\uc2e0\uc744 \uc815\uc74d\ud604\uac10\uc5d0\uc11c \uc9c4\ub3c4\uad70\uc218 (\u73cd\u5cf6\u90e1\u5b88)\ub85c \uc2b9\uc9c4\uc2dc\ucf30\uace0, \uadf8\uac00 \ubd80\uc784\uc9c0\uc5d0 \ubd80\uc784\ud558\uae30\ub3c4 \uc804\uc5d0 \uac00\ub9ac\ud3ec\ucca8\uc808\uc81c\uc0ac(\u52a0\u91cc\u6d66\u50c9\u7bc0\u5236\u4f7f)\ub85c \uc804\uc784\uc2dc\ucf30\ub2e4.", "paragraph_sentence": "1591\ub144 2\uc6d4\uc5d0 \uc120\uc870\ub294 \uc774\ucc9c\u00b7\uc774\uc5b5\uae30\u00b7\uc591\uc751\uc9c0\u00b7\uc774\uc21c\uc2e0\uc744 \ub0a8\ucabd \uc694\ud574\uc9c0\uc5d0 \uc784\uba85\ud558\uc5ec \uacf5\uc744 \uc138\uc6b0\uac8c \ud558\ub77c\ub294 \uc804\uad50\ub97c \ub0b4\ub838\ub2e4. \uc120\uc870\ub294 \uc2e0\ud558\ub4e4\uc758 \ubc18\ubc1c\uacfc \ub17c\ud575\uc744 \ud53c\ud558\uae30 \uc704\ud574 \ubcbc\uc2ac\uc758 \uac01 \ub2e8\uacc4\ub9c8\ub2e4 \uc784\uba85\ud558\uc5ec \uc81c\uc218\ud558\uace0 \uc2b9\uc9c4\uc2dc\ud0a4\ub294 \ubc29\ubc95\uc744 \uc368\uc11c 1591\ub144 2\uc6d4 \uc774\uc21c\uc2e0\uc744 \uc815\uc74d\ud604\uac10\uc5d0\uc11c \uc9c4\ub3c4\uad70\uc218 (\u73cd\u5cf6\u90e1\u5b88)\ub85c \uc2b9\uc9c4\uc2dc\ucf30\uace0, \uadf8\uac00 \ubd80\uc784\uc9c0\uc5d0 \ubd80\uc784\ud558\uae30\ub3c4 \uc804\uc5d0 \uac00\ub9ac\ud3ec\ucca8\uc808\uc81c\uc0ac(\u52a0\u91cc\u6d66\u50c9\u7bc0\u5236\u4f7f)\ub85c \uc804\uc784\uc2dc\ucf30\ub2e4. \uc774\uc5b4 \uc120\uc870\ub294 \uc774\uc21c\uc2e0\uc774 \uac00\ub9ac\ud3ec\uc5d0 \ubd80\uc784\ud558\uae30\ub3c4 \uc804\uc5d0 \ub2e4\uc2dc \uc804\ub77c\uc88c\uc218\uc0ac\uc5d0 \uc804\uc784\ud558\uac8c \ud558\uc600\ub2e4. \ub610\ud55c, \uc120\uc870\ub294 \uc774\uc21c\uc2e0\uacfc \uac19\uc774 \ubc31\uc758\uc885\uad70\uc744 \ud558\uc600\ub358 \uc774\uacbd\ub85d\ub3c4 \uc804\ub77c\ub3c4\uc758 \uc694\uc9c0\uc778 \ub098\uc8fc\ubaa9\uc0ac\uc5d0 \uc81c\uc218\ud558\uc600\ub2e4. \uacc4\uc18d\ub418\ub294 \ub300\uc2e0\ub4e4\uc758 \ubc18\ub300\uc5d0\ub3c4 \ubd88\uad6c\ud558\uace0 \uc120\uc870\ub294 \uc774\uc21c\uc2e0\uc744 \uc218\uc0ac\ub85c \uc0bc\uc73c\ub824\ub294 \uc2ec\uc9c0\ub97c \uad73\ud614\ub2e4. \uadf8\ub294 \uc774\uacbd\ub85d\uc758 \ubc1c\ud0c1\uc740 \ucc9c\ucc9c\ud788 \uc0dd\uac01\ud558\uc5ec \uacb0\uc815\ud558\uaca0\ub2e4\uace0 \ud558\uba74\uc11c\ub3c4 \uc774\uc21c\uc2e0\uc758 \uc218\uc0ac \ubc1c\ud0c1\uc740 \uac1c\uc815\uc740 \ud560 \uc218 \uc5c6\ub2e4\uace0 \ud558\uba70 \ubc00\uc5b4\ubd99\uc600\ub2e4.", "paragraph_answer": "1591\ub144 2\uc6d4\uc5d0 \uc120\uc870\ub294 \uc774\ucc9c\u00b7\uc774\uc5b5\uae30\u00b7\uc591\uc751\uc9c0\u00b7\uc774\uc21c\uc2e0\uc744 \ub0a8\ucabd \uc694\ud574\uc9c0\uc5d0 \uc784\uba85\ud558\uc5ec \uacf5\uc744 \uc138\uc6b0\uac8c \ud558\ub77c\ub294 \uc804\uad50\ub97c \ub0b4\ub838\ub2e4. \uc120\uc870\ub294 \uc2e0\ud558\ub4e4\uc758 \ubc18\ubc1c\uacfc \ub17c\ud575\uc744 \ud53c\ud558\uae30 \uc704\ud574 \ubcbc\uc2ac\uc758 \uac01 \ub2e8\uacc4\ub9c8\ub2e4 \uc784\uba85\ud558\uc5ec \uc81c\uc218\ud558\uace0 \uc2b9\uc9c4\uc2dc\ud0a4\ub294 \ubc29\ubc95\uc744 \uc368\uc11c 1591\ub144 2\uc6d4 \uc774\uc21c\uc2e0\uc744 \uc815\uc74d\ud604\uac10\uc5d0\uc11c \uc9c4\ub3c4\uad70\uc218 (\u73cd\u5cf6\u90e1\u5b88)\ub85c \uc2b9\uc9c4\uc2dc\ucf30\uace0, \uadf8\uac00 \ubd80\uc784\uc9c0\uc5d0 \ubd80\uc784\ud558\uae30\ub3c4 \uc804\uc5d0 \uac00\ub9ac\ud3ec\ucca8\uc808\uc81c\uc0ac(\u52a0\u91cc\u6d66\u50c9\u7bc0\u5236\u4f7f)\ub85c \uc804\uc784\uc2dc\ucf30\ub2e4. \uc774\uc5b4 \uc120\uc870\ub294 \uc774\uc21c\uc2e0\uc774 \uac00\ub9ac\ud3ec\uc5d0 \ubd80\uc784\ud558\uae30\ub3c4 \uc804\uc5d0 \ub2e4\uc2dc \uc804\ub77c\uc88c\uc218\uc0ac\uc5d0 \uc804\uc784\ud558\uac8c \ud558\uc600\ub2e4. \ub610\ud55c, \uc120\uc870\ub294 \uc774\uc21c\uc2e0\uacfc \uac19\uc774 \ubc31\uc758\uc885\uad70\uc744 \ud558\uc600\ub358 \uc774\uacbd\ub85d\ub3c4 \uc804\ub77c\ub3c4\uc758 \uc694\uc9c0\uc778 \ub098\uc8fc\ubaa9\uc0ac\uc5d0 \uc81c\uc218\ud558\uc600\ub2e4. \uacc4\uc18d\ub418\ub294 \ub300\uc2e0\ub4e4\uc758 \ubc18\ub300\uc5d0\ub3c4 \ubd88\uad6c\ud558\uace0 \uc120\uc870\ub294 \uc774\uc21c\uc2e0\uc744 \uc218\uc0ac\ub85c \uc0bc\uc73c\ub824\ub294 \uc2ec\uc9c0\ub97c \uad73\ud614\ub2e4. \uadf8\ub294 \uc774\uacbd\ub85d\uc758 \ubc1c\ud0c1\uc740 \ucc9c\ucc9c\ud788 \uc0dd\uac01\ud558\uc5ec \uacb0\uc815\ud558\uaca0\ub2e4\uace0 \ud558\uba74\uc11c\ub3c4 \uc774\uc21c\uc2e0\uc758 \uc218\uc0ac \ubc1c\ud0c1\uc740 \uac1c\uc815\uc740 \ud560 \uc218 \uc5c6\ub2e4\uace0 \ud558\uba70 \ubc00\uc5b4\ubd99\uc600\ub2e4.", "sentence_answer": "\uc120\uc870\ub294 \uc2e0\ud558\ub4e4\uc758 \ubc18\ubc1c\uacfc \ub17c\ud575\uc744 \ud53c\ud558\uae30 \uc704\ud574 \ubcbc\uc2ac\uc758 \uac01 \ub2e8\uacc4\ub9c8\ub2e4 \uc784\uba85\ud558\uc5ec \uc81c\uc218\ud558\uace0 \uc2b9\uc9c4\uc2dc\ud0a4\ub294 \ubc29\ubc95\uc744 \uc368\uc11c 1591\ub144 2\uc6d4 \uc774\uc21c\uc2e0\uc744 \uc815\uc74d\ud604\uac10\uc5d0\uc11c \uc9c4\ub3c4\uad70\uc218 (\u73cd\u5cf6\u90e1\u5b88)\ub85c \uc2b9\uc9c4\uc2dc\ucf30\uace0, \uadf8\uac00 \ubd80\uc784\uc9c0\uc5d0 \ubd80\uc784\ud558\uae30\ub3c4 \uc804\uc5d0 \uac00\ub9ac\ud3ec\ucca8\uc808\uc81c\uc0ac(\u52a0\u91cc\u6d66\u50c9\u7bc0\u5236\u4f7f)\ub85c \uc804\uc784\uc2dc\ucf30\ub2e4.", "paragraph_id": "6545635-2-0", "paragraph_question": "question: \uc120\uc870\ub294 1591\ub144 2\uc6d4 \uc774\uc21c\uc2e0\uc744 \uc815\uc74d\ud604\uac10\uc5d0\uc11c \ubb34\uc5c7\uc73c\ub85c \uc2b9\uc9c4\uc2dc\ucf30\uc2b5\ub2c8\uae4c?, context: 1591\ub144 2\uc6d4\uc5d0 \uc120\uc870\ub294 \uc774\ucc9c\u00b7\uc774\uc5b5\uae30\u00b7\uc591\uc751\uc9c0\u00b7\uc774\uc21c\uc2e0\uc744 \ub0a8\ucabd \uc694\ud574\uc9c0\uc5d0 \uc784\uba85\ud558\uc5ec \uacf5\uc744 \uc138\uc6b0\uac8c \ud558\ub77c\ub294 \uc804\uad50\ub97c \ub0b4\ub838\ub2e4. \uc120\uc870\ub294 \uc2e0\ud558\ub4e4\uc758 \ubc18\ubc1c\uacfc \ub17c\ud575\uc744 \ud53c\ud558\uae30 \uc704\ud574 \ubcbc\uc2ac\uc758 \uac01 \ub2e8\uacc4\ub9c8\ub2e4 \uc784\uba85\ud558\uc5ec \uc81c\uc218\ud558\uace0 \uc2b9\uc9c4\uc2dc\ud0a4\ub294 \ubc29\ubc95\uc744 \uc368\uc11c 1591\ub144 2\uc6d4 \uc774\uc21c\uc2e0\uc744 \uc815\uc74d\ud604\uac10\uc5d0\uc11c \uc9c4\ub3c4\uad70\uc218(\u73cd\u5cf6\u90e1\u5b88)\ub85c \uc2b9\uc9c4\uc2dc\ucf30\uace0, \uadf8\uac00 \ubd80\uc784\uc9c0\uc5d0 \ubd80\uc784\ud558\uae30\ub3c4 \uc804\uc5d0 \uac00\ub9ac\ud3ec\ucca8\uc808\uc81c\uc0ac(\u52a0\u91cc\u6d66\u50c9\u7bc0\u5236\u4f7f)\ub85c \uc804\uc784\uc2dc\ucf30\ub2e4. \uc774\uc5b4 \uc120\uc870\ub294 \uc774\uc21c\uc2e0\uc774 \uac00\ub9ac\ud3ec\uc5d0 \ubd80\uc784\ud558\uae30\ub3c4 \uc804\uc5d0 \ub2e4\uc2dc \uc804\ub77c\uc88c\uc218\uc0ac\uc5d0 \uc804\uc784\ud558\uac8c \ud558\uc600\ub2e4. \ub610\ud55c, \uc120\uc870\ub294 \uc774\uc21c\uc2e0\uacfc \uac19\uc774 \ubc31\uc758\uc885\uad70\uc744 \ud558\uc600\ub358 \uc774\uacbd\ub85d\ub3c4 \uc804\ub77c\ub3c4\uc758 \uc694\uc9c0\uc778 \ub098\uc8fc\ubaa9\uc0ac\uc5d0 \uc81c\uc218\ud558\uc600\ub2e4. \uacc4\uc18d\ub418\ub294 \ub300\uc2e0\ub4e4\uc758 \ubc18\ub300\uc5d0\ub3c4 \ubd88\uad6c\ud558\uace0 \uc120\uc870\ub294 \uc774\uc21c\uc2e0\uc744 \uc218\uc0ac\ub85c \uc0bc\uc73c\ub824\ub294 \uc2ec\uc9c0\ub97c \uad73\ud614\ub2e4. \uadf8\ub294 \uc774\uacbd\ub85d\uc758 \ubc1c\ud0c1\uc740 \ucc9c\ucc9c\ud788 \uc0dd\uac01\ud558\uc5ec \uacb0\uc815\ud558\uaca0\ub2e4\uace0 \ud558\uba74\uc11c\ub3c4 \uc774\uc21c\uc2e0\uc758 \uc218\uc0ac \ubc1c\ud0c1\uc740 \uac1c\uc815\uc740 \ud560 \uc218 \uc5c6\ub2e4\uace0 \ud558\uba70 \ubc00\uc5b4\ubd99\uc600\ub2e4."} {"question": "\uacbd\uc2e0\ud658\uad6d \uc774\ud6c4 \ub2e4\uc2dc \uc9d1\uad8c\ud55c \uc11c\uc778\uc740 \ubb34\uc5c7\uc73c\ub85c \ub0a8\uc778\uc758 \uc7ac\uae30\ub97c \ub9c9\uc558\ub294\uac00?", "paragraph": "\uadf8\ub7ec\ub098 \uacbd\uc2e0\ud658\uad6d\uc5d0 \uc758\ud558\uc5ec 1680\ub144 \ub0a8\uc778\uc774 \uc2e4\uac01\ud558\uace0 \uc11c\uc778 \uc815\uad8c\uc774 \ub2e4\uc2dc \uc218\ub9bd\ub418\uba74\uc11c \ubd95\ub2f9 \uc0ac\uc774\uc758 \ub300\ub9bd\uc591\uc0c1\uc740 \ud06c\uac8c \ub2ec\ub77c\uc838 \uac14\ub2e4. \uc989 \ub2e4\uc2dc \uc9d1\uad8c\ud55c \uc11c\uc778\uc740 \ucca0\uc800\ud55c \ud0c4\uc555\uc73c\ub85c \ub0a8\uc778\uc758 \uc7ac\uae30\ub97c \ub9c9\uc558\ub2e4. \uc774\ub54c, \uc11c\uc778\uc740 \uc544\uc608 \ub0a8\uc778\uc744 \uc644\uc804\ud788 \ucd95\ucd9c\ud558\uc790\ub294 \ub178\ub860\uacfc \ub0a8\uc778\uacfc\uc758 \ud654\ud574\ub97c \uc8fc\uc7a5\ud558\ub294 \uc18c\ub860\uc73c\ub85c \ubd84\uc5f4\ub418\uc5c8\uc73c\uba70, \ub450 \uc138\ub825 \uc0ac\uc774\uc758 \ub300\ub9bd\uc73c\ub85c \uc815\uad6d\uc758 \ubc18\uc804\uc774 \uac70\ub4ed\ub418\uc5c8\ub2e4. \uc774\ub85c\ubd80\ud130 \uacac\uc81c\uc640 \uade0\ud615, \uacf5\ub860\uc5d0 \ud1a0\ub300\ud55c \ubd95\ub2f9\uc815\uce58\uc758 \uae30\ubcf8 \uc6d0\ub9ac\ub294 \ubb34\ub108\uc9c0\uace0, \uc0c1\ub300 \uc138\ub825\uc758 \uc874\uc7ac \uc790\uccb4\ub97c \uc544\uc608 \uc778\uc815\ud558\ub824 \ub4e4\uc9c0 \uc54a\ub294 \uc77c\ub2f9 \uc804\uc81c\ud654\uc758 \ucd94\uc138\uac00 \ub098\ud0c0\ub098\uae30 \uc2dc\uc791\ud558\uc600\ub2e4. \uc0c1\ub300 \ub2f9\uc5d0 \ub300\ud55c \ubcf4\ubcf5\uc73c\ub85c \uc0ac\uc0ac(\u8cdc\u6b7b)\uac00 \ube48\ubc88\ud558\uc600\uace0, \uc815\uc7c1\uc758 \ucd08\uc810\uc774 \uc655\uc704 \uacc4\uc2b9 \ubb38\uc81c\ub85c \ube44\ud654\ub418\ub294 \ub4f1 \ubd95\ub2f9\uc815\uce58\uac00 \uc815\uc0c1\uc801\uc73c\ub85c \uc6b4\uc601\ub418\uc9c0 \ubabb\ud558\uc600\ub2e4.", "answer": "\ucca0\uc800\ud55c \ud0c4\uc555", "sentence": "\uc989 \ub2e4\uc2dc \uc9d1\uad8c\ud55c \uc11c\uc778\uc740 \ucca0\uc800\ud55c \ud0c4\uc555 \uc73c\ub85c \ub0a8\uc778\uc758 \uc7ac\uae30\ub97c \ub9c9\uc558\ub2e4.", "paragraph_sentence": "\uadf8\ub7ec\ub098 \uacbd\uc2e0\ud658\uad6d\uc5d0 \uc758\ud558\uc5ec 1680\ub144 \ub0a8\uc778\uc774 \uc2e4\uac01\ud558\uace0 \uc11c\uc778 \uc815\uad8c\uc774 \ub2e4\uc2dc \uc218\ub9bd\ub418\uba74\uc11c \ubd95\ub2f9 \uc0ac\uc774\uc758 \ub300\ub9bd\uc591\uc0c1\uc740 \ud06c\uac8c \ub2ec\ub77c\uc838 \uac14\ub2e4. \uc989 \ub2e4\uc2dc \uc9d1\uad8c\ud55c \uc11c\uc778\uc740 \ucca0\uc800\ud55c \ud0c4\uc555 \uc73c\ub85c \ub0a8\uc778\uc758 \uc7ac\uae30\ub97c \ub9c9\uc558\ub2e4. \uc774\ub54c, \uc11c\uc778\uc740 \uc544\uc608 \ub0a8\uc778\uc744 \uc644\uc804\ud788 \ucd95\ucd9c\ud558\uc790\ub294 \ub178\ub860\uacfc \ub0a8\uc778\uacfc\uc758 \ud654\ud574\ub97c \uc8fc\uc7a5\ud558\ub294 \uc18c\ub860\uc73c\ub85c \ubd84\uc5f4\ub418\uc5c8\uc73c\uba70, \ub450 \uc138\ub825 \uc0ac\uc774\uc758 \ub300\ub9bd\uc73c\ub85c \uc815\uad6d\uc758 \ubc18\uc804\uc774 \uac70\ub4ed\ub418\uc5c8\ub2e4. \uc774\ub85c\ubd80\ud130 \uacac\uc81c\uc640 \uade0\ud615, \uacf5\ub860\uc5d0 \ud1a0\ub300\ud55c \ubd95\ub2f9\uc815\uce58\uc758 \uae30\ubcf8 \uc6d0\ub9ac\ub294 \ubb34\ub108\uc9c0\uace0, \uc0c1\ub300 \uc138\ub825\uc758 \uc874\uc7ac \uc790\uccb4\ub97c \uc544\uc608 \uc778\uc815\ud558\ub824 \ub4e4\uc9c0 \uc54a\ub294 \uc77c\ub2f9 \uc804\uc81c\ud654\uc758 \ucd94\uc138\uac00 \ub098\ud0c0\ub098\uae30 \uc2dc\uc791\ud558\uc600\ub2e4. \uc0c1\ub300 \ub2f9\uc5d0 \ub300\ud55c \ubcf4\ubcf5\uc73c\ub85c \uc0ac\uc0ac(\u8cdc\u6b7b)\uac00 \ube48\ubc88\ud558\uc600\uace0, \uc815\uc7c1\uc758 \ucd08\uc810\uc774 \uc655\uc704 \uacc4\uc2b9 \ubb38\uc81c\ub85c \ube44\ud654\ub418\ub294 \ub4f1 \ubd95\ub2f9\uc815\uce58\uac00 \uc815\uc0c1\uc801\uc73c\ub85c \uc6b4\uc601\ub418\uc9c0 \ubabb\ud558\uc600\ub2e4.", "paragraph_answer": "\uadf8\ub7ec\ub098 \uacbd\uc2e0\ud658\uad6d\uc5d0 \uc758\ud558\uc5ec 1680\ub144 \ub0a8\uc778\uc774 \uc2e4\uac01\ud558\uace0 \uc11c\uc778 \uc815\uad8c\uc774 \ub2e4\uc2dc \uc218\ub9bd\ub418\uba74\uc11c \ubd95\ub2f9 \uc0ac\uc774\uc758 \ub300\ub9bd\uc591\uc0c1\uc740 \ud06c\uac8c \ub2ec\ub77c\uc838 \uac14\ub2e4. \uc989 \ub2e4\uc2dc \uc9d1\uad8c\ud55c \uc11c\uc778\uc740 \ucca0\uc800\ud55c \ud0c4\uc555 \uc73c\ub85c \ub0a8\uc778\uc758 \uc7ac\uae30\ub97c \ub9c9\uc558\ub2e4. \uc774\ub54c, \uc11c\uc778\uc740 \uc544\uc608 \ub0a8\uc778\uc744 \uc644\uc804\ud788 \ucd95\ucd9c\ud558\uc790\ub294 \ub178\ub860\uacfc \ub0a8\uc778\uacfc\uc758 \ud654\ud574\ub97c \uc8fc\uc7a5\ud558\ub294 \uc18c\ub860\uc73c\ub85c \ubd84\uc5f4\ub418\uc5c8\uc73c\uba70, \ub450 \uc138\ub825 \uc0ac\uc774\uc758 \ub300\ub9bd\uc73c\ub85c \uc815\uad6d\uc758 \ubc18\uc804\uc774 \uac70\ub4ed\ub418\uc5c8\ub2e4. \uc774\ub85c\ubd80\ud130 \uacac\uc81c\uc640 \uade0\ud615, \uacf5\ub860\uc5d0 \ud1a0\ub300\ud55c \ubd95\ub2f9\uc815\uce58\uc758 \uae30\ubcf8 \uc6d0\ub9ac\ub294 \ubb34\ub108\uc9c0\uace0, \uc0c1\ub300 \uc138\ub825\uc758 \uc874\uc7ac \uc790\uccb4\ub97c \uc544\uc608 \uc778\uc815\ud558\ub824 \ub4e4\uc9c0 \uc54a\ub294 \uc77c\ub2f9 \uc804\uc81c\ud654\uc758 \ucd94\uc138\uac00 \ub098\ud0c0\ub098\uae30 \uc2dc\uc791\ud558\uc600\ub2e4. \uc0c1\ub300 \ub2f9\uc5d0 \ub300\ud55c \ubcf4\ubcf5\uc73c\ub85c \uc0ac\uc0ac(\u8cdc\u6b7b)\uac00 \ube48\ubc88\ud558\uc600\uace0, \uc815\uc7c1\uc758 \ucd08\uc810\uc774 \uc655\uc704 \uacc4\uc2b9 \ubb38\uc81c\ub85c \ube44\ud654\ub418\ub294 \ub4f1 \ubd95\ub2f9\uc815\uce58\uac00 \uc815\uc0c1\uc801\uc73c\ub85c \uc6b4\uc601\ub418\uc9c0 \ubabb\ud558\uc600\ub2e4.", "sentence_answer": "\uc989 \ub2e4\uc2dc \uc9d1\uad8c\ud55c \uc11c\uc778\uc740 \ucca0\uc800\ud55c \ud0c4\uc555 \uc73c\ub85c \ub0a8\uc778\uc758 \uc7ac\uae30\ub97c \ub9c9\uc558\ub2e4.", "paragraph_id": "6187890-10-1", "paragraph_question": "question: \uacbd\uc2e0\ud658\uad6d \uc774\ud6c4 \ub2e4\uc2dc \uc9d1\uad8c\ud55c \uc11c\uc778\uc740 \ubb34\uc5c7\uc73c\ub85c \ub0a8\uc778\uc758 \uc7ac\uae30\ub97c \ub9c9\uc558\ub294\uac00?, context: \uadf8\ub7ec\ub098 \uacbd\uc2e0\ud658\uad6d\uc5d0 \uc758\ud558\uc5ec 1680\ub144 \ub0a8\uc778\uc774 \uc2e4\uac01\ud558\uace0 \uc11c\uc778 \uc815\uad8c\uc774 \ub2e4\uc2dc \uc218\ub9bd\ub418\uba74\uc11c \ubd95\ub2f9 \uc0ac\uc774\uc758 \ub300\ub9bd\uc591\uc0c1\uc740 \ud06c\uac8c \ub2ec\ub77c\uc838 \uac14\ub2e4. \uc989 \ub2e4\uc2dc \uc9d1\uad8c\ud55c \uc11c\uc778\uc740 \ucca0\uc800\ud55c \ud0c4\uc555\uc73c\ub85c \ub0a8\uc778\uc758 \uc7ac\uae30\ub97c \ub9c9\uc558\ub2e4. \uc774\ub54c, \uc11c\uc778\uc740 \uc544\uc608 \ub0a8\uc778\uc744 \uc644\uc804\ud788 \ucd95\ucd9c\ud558\uc790\ub294 \ub178\ub860\uacfc \ub0a8\uc778\uacfc\uc758 \ud654\ud574\ub97c \uc8fc\uc7a5\ud558\ub294 \uc18c\ub860\uc73c\ub85c \ubd84\uc5f4\ub418\uc5c8\uc73c\uba70, \ub450 \uc138\ub825 \uc0ac\uc774\uc758 \ub300\ub9bd\uc73c\ub85c \uc815\uad6d\uc758 \ubc18\uc804\uc774 \uac70\ub4ed\ub418\uc5c8\ub2e4. \uc774\ub85c\ubd80\ud130 \uacac\uc81c\uc640 \uade0\ud615, \uacf5\ub860\uc5d0 \ud1a0\ub300\ud55c \ubd95\ub2f9\uc815\uce58\uc758 \uae30\ubcf8 \uc6d0\ub9ac\ub294 \ubb34\ub108\uc9c0\uace0, \uc0c1\ub300 \uc138\ub825\uc758 \uc874\uc7ac \uc790\uccb4\ub97c \uc544\uc608 \uc778\uc815\ud558\ub824 \ub4e4\uc9c0 \uc54a\ub294 \uc77c\ub2f9 \uc804\uc81c\ud654\uc758 \ucd94\uc138\uac00 \ub098\ud0c0\ub098\uae30 \uc2dc\uc791\ud558\uc600\ub2e4. \uc0c1\ub300 \ub2f9\uc5d0 \ub300\ud55c \ubcf4\ubcf5\uc73c\ub85c \uc0ac\uc0ac(\u8cdc\u6b7b)\uac00 \ube48\ubc88\ud558\uc600\uace0, \uc815\uc7c1\uc758 \ucd08\uc810\uc774 \uc655\uc704 \uacc4\uc2b9 \ubb38\uc81c\ub85c \ube44\ud654\ub418\ub294 \ub4f1 \ubd95\ub2f9\uc815\uce58\uac00 \uc815\uc0c1\uc801\uc73c\ub85c \uc6b4\uc601\ub418\uc9c0 \ubabb\ud558\uc600\ub2e4."} {"question": "\uc544\uc720\uac00 \uc544\ub97c \uc544\ube44\ub1fd\uc5d0\uc11c \ubc30\uce58\ub41c \ud3ec\uc9c0\uc158\uc740 \uc5b4\ub514\uc778\uac00?", "paragraph": "2\ub2ec \ud6c4\uc778 2009\ub144 8\uc6d4 31\uc77c, \uc774\uc801\uc2dc\uc7a5 \ub9c8\uc9c0\ub9c9 \ub0a0\uc5d0, \uc2e0\uc784 \uac10\ub3c5 \ub514\ub514\uc5d0 \ub370\uc0f9\uc740 \ub9ac\uadf8 2\uc5d0 \uc0c8\ub85c \uc2b9\uaca9\ub41c AC \uc544\ub97c \uc544\ube44\ub1fd\uc73c\ub85c 2009-10 \uc2dc\uc98c\ub3d9\uc548 \uc784\ub300\ub420 \uac83\uc774\ub77c\uace0 \ubc1d\ud614\ub2e4. \uc544\ub97c \uc544\ube44\ub1fd\uc5d0\uc11c \uadf8\ub294 10\ubc88 \ub4f1\ubc88\ud638\ub97c \ubc1b\uc558\uace0, \uadf8\uac00 \uc120\ud638\ud558\ub294 \ub77c\uc774\ud2b8 \uc719\uc5d0 \ubc30\uce58\ub418\uc5c8\ub2e4. 9\uc6d4 11\uc77c, \uc559\uc81c SCO\uc804\uc5d0\uc11c \uad50\uccb4 \ucd9c\uc7a5\ud558\uba70 \uc544\ube44\ub1fd \ub370\ubdd4\uc804\uc744 \uce58\ub8e8\uc5c8\uace0, \ud300\uc740 1-1 \ubb34\uc2b9\ubd80\ub97c \uac70\ub450\uc5c8\ub2e4. \uadf8 \ub2e4\uc74c\uc8fc, \uadf8\ub294 \ud22c\ub974 FC\uc804\uc5d0\uc11c \uace8\uc744 \uae30\ub85d\ud558\uc600\uc73c\ub098, \uc544\ube44\ub1fd\uc740 2-4\ub85c \ud328\ud558\uc600\ub2e4. \uadf8\ub294 \uc804\ubc18\uae30\uc5d0 \ub9e4\uacbd\uae30 \ucd9c\uc7a5\ud558\uc600\uc73c\ub098, 2010\ub144 \uc544\ud504\ub9ac\uce74 \ub124\uc774\uc158\uc2a4\ucef5\uc73c\ub85c \uc778\ud574 1\uc6d4 \ud55c\ub2ec\ub3d9\uc548 \uc18c\uc18d\ud300 \uacbd\uae30\uc5d0 \ucd9c\uc7a5\ud558\uc9c0 \ubabb\ud558\uc600\ub2e4. \uc544\uc720\ub294 2010\ub144 2\uc6d4 5\uc77c\uc5d0 \uc18c\uc18d\ud300\uc73c\ub85c \ub3cc\uc544\uc640 \uc794\uc5ec\uae30\uac04\ub3d9\uc548 \ubaa8\ub4e0 \uacbd\uae30\uc5d0 \uc120\ubc1c\ub85c \ucd9c\uc804\ud558\uc600\ub2e4. 2010\ub144 4\uc6d4 9\uc77c, \uc2b9\uaca9 \uacbd\uc7c1\uc5d0\uc11c, \uc544\uc720\ub294 \ub974\uc544\ube0c\ub974 AC\uc804\uc5d0\uc11c 2\uace8\uc744 \uae30\ub85d\ud558\uc600\uace0, \ud300\uc758 2-1 \uc2b9\ub9ac \uc8fc\uc5ed\uc774 \ub418\uc5c8\ub2e4. \uadf8 \ub2e4\uc74c\uc8fc, \uadf8\ub294 EA \uac31\uac15\uc804\uc5d0\uc11c \uc120\uc81c\uace8\uc744 \uae30\ub85d\ud558\uc600\uace0, \uc18c\uc18d\ud300\uc740 1-1 \ubb34\uc2b9\ubd80\ub97c \uac70\ub450\uc5c8\ub2e4. 5\uc6d4 14\uc77c\uc758 \ud074\ub808\ub974\ubabd \ud478\ud2b8\uc804\uc5d0\uc11c\uc758 1-0 \uc2b9\ub9ac\ub294 \uc544\ub97c \uc544\ube44\ub1fd\uc758 \uc2b9\uaca9\uc744 \ud655\uc815\uc2dc\ucf30\ub2e4. \uc544\uc720\ub294 \uc774 \uacbd\uae30\uc5d0 \uc120\ubc1c\ub85c \ucd9c\uc7a5\ud558\uc5ec \ud480\ud0c0\uc784\uc744 \uc18c\ud654\ud558\uc600\ub2e4. \uadf8\ub294 \uc544\ub97c \uc544\ube44\ub1fd \uc18c\uc18d\uc73c\ub85c 26\uacbd\uae30\uc5d0 \ucd9c\uc7a5\ud558\uc5ec 4\ub4dd\uc810\uc744 \uae30\ub85d\ud558\uc600\ub2e4.", "answer": "\ub77c\uc774\ud2b8 \uc719", "sentence": "\uc544\ub97c \uc544\ube44\ub1fd\uc5d0\uc11c \uadf8\ub294 10\ubc88 \ub4f1\ubc88\ud638\ub97c \ubc1b\uc558\uace0, \uadf8\uac00 \uc120\ud638\ud558\ub294 \ub77c\uc774\ud2b8 \uc719 \uc5d0 \ubc30\uce58\ub418\uc5c8\ub2e4.", "paragraph_sentence": "2\ub2ec \ud6c4\uc778 2009\ub144 8\uc6d4 31\uc77c, \uc774\uc801\uc2dc\uc7a5 \ub9c8\uc9c0\ub9c9 \ub0a0\uc5d0, \uc2e0\uc784 \uac10\ub3c5 \ub514\ub514\uc5d0 \ub370\uc0f9\uc740 \ub9ac\uadf8 2\uc5d0 \uc0c8\ub85c \uc2b9\uaca9\ub41c AC \uc544\ub97c \uc544\ube44\ub1fd\uc73c\ub85c 2009-10 \uc2dc\uc98c\ub3d9\uc548 \uc784\ub300\ub420 \uac83\uc774\ub77c\uace0 \ubc1d\ud614\ub2e4. \uc544\ub97c \uc544\ube44\ub1fd\uc5d0\uc11c \uadf8\ub294 10\ubc88 \ub4f1\ubc88\ud638\ub97c \ubc1b\uc558\uace0, \uadf8\uac00 \uc120\ud638\ud558\ub294 \ub77c\uc774\ud2b8 \uc719 \uc5d0 \ubc30\uce58\ub418\uc5c8\ub2e4. 9\uc6d4 11\uc77c, \uc559\uc81c SCO\uc804\uc5d0\uc11c \uad50\uccb4 \ucd9c\uc7a5\ud558\uba70 \uc544\ube44\ub1fd \ub370\ubdd4\uc804\uc744 \uce58\ub8e8\uc5c8\uace0, \ud300\uc740 1-1 \ubb34\uc2b9\ubd80\ub97c \uac70\ub450\uc5c8\ub2e4. \uadf8 \ub2e4\uc74c\uc8fc, \uadf8\ub294 \ud22c\ub974 FC\uc804\uc5d0\uc11c \uace8\uc744 \uae30\ub85d\ud558\uc600\uc73c\ub098, \uc544\ube44\ub1fd\uc740 2-4\ub85c \ud328\ud558\uc600\ub2e4. \uadf8\ub294 \uc804\ubc18\uae30\uc5d0 \ub9e4\uacbd\uae30 \ucd9c\uc7a5\ud558\uc600\uc73c\ub098, 2010\ub144 \uc544\ud504\ub9ac\uce74 \ub124\uc774\uc158\uc2a4\ucef5\uc73c\ub85c \uc778\ud574 1\uc6d4 \ud55c\ub2ec\ub3d9\uc548 \uc18c\uc18d\ud300 \uacbd\uae30\uc5d0 \ucd9c\uc7a5\ud558\uc9c0 \ubabb\ud558\uc600\ub2e4. \uc544\uc720\ub294 2010\ub144 2\uc6d4 5\uc77c\uc5d0 \uc18c\uc18d\ud300\uc73c\ub85c \ub3cc\uc544\uc640 \uc794\uc5ec\uae30\uac04\ub3d9\uc548 \ubaa8\ub4e0 \uacbd\uae30\uc5d0 \uc120\ubc1c\ub85c \ucd9c\uc804\ud558\uc600\ub2e4. 2010\ub144 4\uc6d4 9\uc77c, \uc2b9\uaca9 \uacbd\uc7c1\uc5d0\uc11c, \uc544\uc720\ub294 \ub974\uc544\ube0c\ub974 AC\uc804\uc5d0\uc11c 2\uace8\uc744 \uae30\ub85d\ud558\uc600\uace0, \ud300\uc758 2-1 \uc2b9\ub9ac \uc8fc\uc5ed\uc774 \ub418\uc5c8\ub2e4. \uadf8 \ub2e4\uc74c\uc8fc, \uadf8\ub294 EA \uac31\uac15\uc804\uc5d0\uc11c \uc120\uc81c\uace8\uc744 \uae30\ub85d\ud558\uc600\uace0, \uc18c\uc18d\ud300\uc740 1-1 \ubb34\uc2b9\ubd80\ub97c \uac70\ub450\uc5c8\ub2e4. 5\uc6d4 14\uc77c\uc758 \ud074\ub808\ub974\ubabd \ud478\ud2b8\uc804\uc5d0\uc11c\uc758 1-0 \uc2b9\ub9ac\ub294 \uc544\ub97c \uc544\ube44\ub1fd\uc758 \uc2b9\uaca9\uc744 \ud655\uc815\uc2dc\ucf30\ub2e4. \uc544\uc720\ub294 \uc774 \uacbd\uae30\uc5d0 \uc120\ubc1c\ub85c \ucd9c\uc7a5\ud558\uc5ec \ud480\ud0c0\uc784\uc744 \uc18c\ud654\ud558\uc600\ub2e4. \uadf8\ub294 \uc544\ub97c \uc544\ube44\ub1fd \uc18c\uc18d\uc73c\ub85c 26\uacbd\uae30\uc5d0 \ucd9c\uc7a5\ud558\uc5ec 4\ub4dd\uc810\uc744 \uae30\ub85d\ud558\uc600\ub2e4.", "paragraph_answer": "2\ub2ec \ud6c4\uc778 2009\ub144 8\uc6d4 31\uc77c, \uc774\uc801\uc2dc\uc7a5 \ub9c8\uc9c0\ub9c9 \ub0a0\uc5d0, \uc2e0\uc784 \uac10\ub3c5 \ub514\ub514\uc5d0 \ub370\uc0f9\uc740 \ub9ac\uadf8 2\uc5d0 \uc0c8\ub85c \uc2b9\uaca9\ub41c AC \uc544\ub97c \uc544\ube44\ub1fd\uc73c\ub85c 2009-10 \uc2dc\uc98c\ub3d9\uc548 \uc784\ub300\ub420 \uac83\uc774\ub77c\uace0 \ubc1d\ud614\ub2e4. \uc544\ub97c \uc544\ube44\ub1fd\uc5d0\uc11c \uadf8\ub294 10\ubc88 \ub4f1\ubc88\ud638\ub97c \ubc1b\uc558\uace0, \uadf8\uac00 \uc120\ud638\ud558\ub294 \ub77c\uc774\ud2b8 \uc719 \uc5d0 \ubc30\uce58\ub418\uc5c8\ub2e4. 9\uc6d4 11\uc77c, \uc559\uc81c SCO\uc804\uc5d0\uc11c \uad50\uccb4 \ucd9c\uc7a5\ud558\uba70 \uc544\ube44\ub1fd \ub370\ubdd4\uc804\uc744 \uce58\ub8e8\uc5c8\uace0, \ud300\uc740 1-1 \ubb34\uc2b9\ubd80\ub97c \uac70\ub450\uc5c8\ub2e4. \uadf8 \ub2e4\uc74c\uc8fc, \uadf8\ub294 \ud22c\ub974 FC\uc804\uc5d0\uc11c \uace8\uc744 \uae30\ub85d\ud558\uc600\uc73c\ub098, \uc544\ube44\ub1fd\uc740 2-4\ub85c \ud328\ud558\uc600\ub2e4. \uadf8\ub294 \uc804\ubc18\uae30\uc5d0 \ub9e4\uacbd\uae30 \ucd9c\uc7a5\ud558\uc600\uc73c\ub098, 2010\ub144 \uc544\ud504\ub9ac\uce74 \ub124\uc774\uc158\uc2a4\ucef5\uc73c\ub85c \uc778\ud574 1\uc6d4 \ud55c\ub2ec\ub3d9\uc548 \uc18c\uc18d\ud300 \uacbd\uae30\uc5d0 \ucd9c\uc7a5\ud558\uc9c0 \ubabb\ud558\uc600\ub2e4. \uc544\uc720\ub294 2010\ub144 2\uc6d4 5\uc77c\uc5d0 \uc18c\uc18d\ud300\uc73c\ub85c \ub3cc\uc544\uc640 \uc794\uc5ec\uae30\uac04\ub3d9\uc548 \ubaa8\ub4e0 \uacbd\uae30\uc5d0 \uc120\ubc1c\ub85c \ucd9c\uc804\ud558\uc600\ub2e4. 2010\ub144 4\uc6d4 9\uc77c, \uc2b9\uaca9 \uacbd\uc7c1\uc5d0\uc11c, \uc544\uc720\ub294 \ub974\uc544\ube0c\ub974 AC\uc804\uc5d0\uc11c 2\uace8\uc744 \uae30\ub85d\ud558\uc600\uace0, \ud300\uc758 2-1 \uc2b9\ub9ac \uc8fc\uc5ed\uc774 \ub418\uc5c8\ub2e4. \uadf8 \ub2e4\uc74c\uc8fc, \uadf8\ub294 EA \uac31\uac15\uc804\uc5d0\uc11c \uc120\uc81c\uace8\uc744 \uae30\ub85d\ud558\uc600\uace0, \uc18c\uc18d\ud300\uc740 1-1 \ubb34\uc2b9\ubd80\ub97c \uac70\ub450\uc5c8\ub2e4. 5\uc6d4 14\uc77c\uc758 \ud074\ub808\ub974\ubabd \ud478\ud2b8\uc804\uc5d0\uc11c\uc758 1-0 \uc2b9\ub9ac\ub294 \uc544\ub97c \uc544\ube44\ub1fd\uc758 \uc2b9\uaca9\uc744 \ud655\uc815\uc2dc\ucf30\ub2e4. \uc544\uc720\ub294 \uc774 \uacbd\uae30\uc5d0 \uc120\ubc1c\ub85c \ucd9c\uc7a5\ud558\uc5ec \ud480\ud0c0\uc784\uc744 \uc18c\ud654\ud558\uc600\ub2e4. \uadf8\ub294 \uc544\ub97c \uc544\ube44\ub1fd \uc18c\uc18d\uc73c\ub85c 26\uacbd\uae30\uc5d0 \ucd9c\uc7a5\ud558\uc5ec 4\ub4dd\uc810\uc744 \uae30\ub85d\ud558\uc600\ub2e4.", "sentence_answer": "\uc544\ub97c \uc544\ube44\ub1fd\uc5d0\uc11c \uadf8\ub294 10\ubc88 \ub4f1\ubc88\ud638\ub97c \ubc1b\uc558\uace0, \uadf8\uac00 \uc120\ud638\ud558\ub294 \ub77c\uc774\ud2b8 \uc719 \uc5d0 \ubc30\uce58\ub418\uc5c8\ub2e4.", "paragraph_id": "6466047-3-1", "paragraph_question": "question: \uc544\uc720\uac00 \uc544\ub97c \uc544\ube44\ub1fd\uc5d0\uc11c \ubc30\uce58\ub41c \ud3ec\uc9c0\uc158\uc740 \uc5b4\ub514\uc778\uac00?, context: 2\ub2ec \ud6c4\uc778 2009\ub144 8\uc6d4 31\uc77c, \uc774\uc801\uc2dc\uc7a5 \ub9c8\uc9c0\ub9c9 \ub0a0\uc5d0, \uc2e0\uc784 \uac10\ub3c5 \ub514\ub514\uc5d0 \ub370\uc0f9\uc740 \ub9ac\uadf8 2\uc5d0 \uc0c8\ub85c \uc2b9\uaca9\ub41c AC \uc544\ub97c \uc544\ube44\ub1fd\uc73c\ub85c 2009-10 \uc2dc\uc98c\ub3d9\uc548 \uc784\ub300\ub420 \uac83\uc774\ub77c\uace0 \ubc1d\ud614\ub2e4. \uc544\ub97c \uc544\ube44\ub1fd\uc5d0\uc11c \uadf8\ub294 10\ubc88 \ub4f1\ubc88\ud638\ub97c \ubc1b\uc558\uace0, \uadf8\uac00 \uc120\ud638\ud558\ub294 \ub77c\uc774\ud2b8 \uc719\uc5d0 \ubc30\uce58\ub418\uc5c8\ub2e4. 9\uc6d4 11\uc77c, \uc559\uc81c SCO\uc804\uc5d0\uc11c \uad50\uccb4 \ucd9c\uc7a5\ud558\uba70 \uc544\ube44\ub1fd \ub370\ubdd4\uc804\uc744 \uce58\ub8e8\uc5c8\uace0, \ud300\uc740 1-1 \ubb34\uc2b9\ubd80\ub97c \uac70\ub450\uc5c8\ub2e4. \uadf8 \ub2e4\uc74c\uc8fc, \uadf8\ub294 \ud22c\ub974 FC\uc804\uc5d0\uc11c \uace8\uc744 \uae30\ub85d\ud558\uc600\uc73c\ub098, \uc544\ube44\ub1fd\uc740 2-4\ub85c \ud328\ud558\uc600\ub2e4. \uadf8\ub294 \uc804\ubc18\uae30\uc5d0 \ub9e4\uacbd\uae30 \ucd9c\uc7a5\ud558\uc600\uc73c\ub098, 2010\ub144 \uc544\ud504\ub9ac\uce74 \ub124\uc774\uc158\uc2a4\ucef5\uc73c\ub85c \uc778\ud574 1\uc6d4 \ud55c\ub2ec\ub3d9\uc548 \uc18c\uc18d\ud300 \uacbd\uae30\uc5d0 \ucd9c\uc7a5\ud558\uc9c0 \ubabb\ud558\uc600\ub2e4. \uc544\uc720\ub294 2010\ub144 2\uc6d4 5\uc77c\uc5d0 \uc18c\uc18d\ud300\uc73c\ub85c \ub3cc\uc544\uc640 \uc794\uc5ec\uae30\uac04\ub3d9\uc548 \ubaa8\ub4e0 \uacbd\uae30\uc5d0 \uc120\ubc1c\ub85c \ucd9c\uc804\ud558\uc600\ub2e4. 2010\ub144 4\uc6d4 9\uc77c, \uc2b9\uaca9 \uacbd\uc7c1\uc5d0\uc11c, \uc544\uc720\ub294 \ub974\uc544\ube0c\ub974 AC\uc804\uc5d0\uc11c 2\uace8\uc744 \uae30\ub85d\ud558\uc600\uace0, \ud300\uc758 2-1 \uc2b9\ub9ac \uc8fc\uc5ed\uc774 \ub418\uc5c8\ub2e4. \uadf8 \ub2e4\uc74c\uc8fc, \uadf8\ub294 EA \uac31\uac15\uc804\uc5d0\uc11c \uc120\uc81c\uace8\uc744 \uae30\ub85d\ud558\uc600\uace0, \uc18c\uc18d\ud300\uc740 1-1 \ubb34\uc2b9\ubd80\ub97c \uac70\ub450\uc5c8\ub2e4. 5\uc6d4 14\uc77c\uc758 \ud074\ub808\ub974\ubabd \ud478\ud2b8\uc804\uc5d0\uc11c\uc758 1-0 \uc2b9\ub9ac\ub294 \uc544\ub97c \uc544\ube44\ub1fd\uc758 \uc2b9\uaca9\uc744 \ud655\uc815\uc2dc\ucf30\ub2e4. \uc544\uc720\ub294 \uc774 \uacbd\uae30\uc5d0 \uc120\ubc1c\ub85c \ucd9c\uc7a5\ud558\uc5ec \ud480\ud0c0\uc784\uc744 \uc18c\ud654\ud558\uc600\ub2e4. \uadf8\ub294 \uc544\ub97c \uc544\ube44\ub1fd \uc18c\uc18d\uc73c\ub85c 26\uacbd\uae30\uc5d0 \ucd9c\uc7a5\ud558\uc5ec 4\ub4dd\uc810\uc744 \uae30\ub85d\ud558\uc600\ub2e4."} {"question": "0.23.5 \uc5c5\ub370\uc774\ud2b8\uc5d0\uc11c \ucf5c\ub77c\ubcf4\ub808\uc774\uc158 \ud55c \uacf3\uc740 \uc5b4\ub514\uc778\uac00?", "paragraph": "0.23.5 \uc5c5\ub370\uc774\ud2b8\uc5d0\uc11c\ub294 \ubbf8 \ud56d\uacf5\uc6b0\uc8fc\uad6d NASA\uc640\uc758 \ucf5c\ub77c\ubcf4\ub808\uc774\uc158\uc744 \ud1b5\ud574 \uc0c8\ub85c\uc6b4 \ucc9c\uccb4\uc778 \uc18c\ud589\uc131\uacfc \uc2e4\uc81c NASA\uc5d0\uc11c \uacc4\ud68d \uc911\uc778 \u2018\uc18c\ud589\uc131 \ud3ec\ud68d \uacc4\ud68d (Asteroid Redirect Mission, ARM)\u2019\uc744 \uac8c\uc784 \uc0c1\uc5d0\uc11c \uc218\ud589 \uac00\ub2a5\ucf00 \ud574\uc8fc\ub294 \ub2e4\uc591\ud55c \ud30c\uce20\ub4e4\uc774 \ucd94\uac00\ub418\uc5c8\ub2e4. \ud50c\ub808\uc774\uc5b4\ub294 \uad00\uce21\uc18c\uc5d0\uc11c \ubbf8\ud655\uc778\ucc9c\uccb4\ub85c \ubc1c\uacac\ub418\ub294 \uc18c\ud589\uc131\ub4e4\uc744 \ucd94\uc801, \uad00\ucc30\ud558\uc5ec \uc815\ubcf4\ub97c \uc218\uc9d1\ud55c \ub4a4, \uc801\uc808\ud55c \uacc4\ud68d\uc744 \uad6c\uc0c1\ud574\uc57c\ud55c\ub2e4. \uc18c\ud589\uc131\uc740 \uc791\uac8c\ub294 \uc57d 10t \ub0a8\uc9d3(A\ub4f1\uae09)\ubd80\ud130 \ucd5c\ub300 \uc57d 4000t(E\ub4f1\uae09)\uae4c\uc9c0 \ud06c\uae30, \ubaa8\uc591, \uada4\ub3c4 \ubaa8\ub450 \ubb34\uc791\uc704\ub85c \uc0dd\uc131\ub418\uba70, \ub300\ubd80\ubd84 \ucee4\ube48 \uc8fc\ubcc0\uc5d0 \ub4f1\uc7a5\ud574\uc11c \uc11c\uc11c\ud788 \uc811\uadfc\ud558\ub294 \uada4\ub3c4\ub97c \uadf8\ub9b0\ub2e4. \ucee4\ube48\uc758 \uc778\ub825\uad8c\uc73c\ub85c \ub4e4\uc5b4\uc628 \uc18c\ud589\uc131\ub4e4\uc774 \uc5b4\ub5a4 \uada4\ub3c4\ub97c \uadf8\ub9b4\uc9c0\ub294 \ucc9c\ucc28\ub9cc\ubcc4\uc778\ub370, \uad00\uce21\uc18c\uc5d0\uc11c \uc608\uc0c1\uada4\ub3c4\ub97c \ud655\uc778 \ud55c \ub4a4, NASA\uc758 \ucc28\uc138\ub300 SLS \ub85c\ucf13\uc744 \ubcf8 \ub530 \ub9cc\ub4e0 \uc5ed\ub300 \ucd5c\uace0 \ucd94\ub825\uc758 \uc0c8\ub85c\uc6b4 \ucd08\ub300\ud615 \ubc1c\uc0ac\uccb4 \ud30c\uce20\uc640, \uc5b4\ub5a4 \ubb3c\uccb4\ub4e0\uc9c0 \uc6c0\ucf1c\uc958 \uc218 \uc788\ub294 \ud3ec\ud68d \ud30c\uce20\ub97c \uc774\uc6a9\ud574 \uc18c\ud589\uc131\uacfc \ub791\ub370\ubd80\ud560 \uc218 \uc788\ub2e4. \ucee4\ube48\uc744 \uc2a4\uccd0 \uc9c0\ub098\uac00\ub294 \uc18c\ud589\uc131\uc744 \ubd99\uc7a1\uc544\uc11c \uc81c3\uc758 \uc704\uc131\uc73c\ub85c \ub9cc\ub4e0\ub2e4\ub4e0\uc9c0, \ucda9\ub3cc\uada4\ub3c4\ub85c \ub3cc\uc9c4\ud558\ub294 \uac70\ub300\uc18c\ud589\uc131\uc744 \uc0ac\ub825\uc744 \ub2e4\ud574 \ubc00\uc5b4 \ub0b8\ub2e4\ub4e0\uc9c0, \uc544\ub2c8\uba74 \ud638\uae30\uc2ec\uacfc \ub2a5\ub825\uc5d0 \ub530\ub77c \ucee4\ube48\uc73c\ub85c \ub04c\uace0 \uc640\uc11c \uc6b0\uc8fc\uc13c\ud130 \uc55e\uc5d0 \ucc29\uc9c0 \uc2dc\ud0ac\uc9c0\ub294 \ucee4\ubc8c\ub4e4\uc774 \uc544\ub2cc \uc804\uc801\uc73c\ub85c \ud50c\ub808\uc774\uc5b4\uc758 \ub9c8\uc74c\uc774\ub2e4.", "answer": "NASA", "sentence": "0.23.5 \uc5c5\ub370\uc774\ud2b8\uc5d0\uc11c\ub294 \ubbf8 \ud56d\uacf5\uc6b0\uc8fc\uad6d NASA \uc640\uc758 \ucf5c\ub77c\ubcf4\ub808\uc774\uc158\uc744 \ud1b5\ud574 \uc0c8\ub85c\uc6b4 \ucc9c\uccb4\uc778 \uc18c\ud589\uc131\uacfc \uc2e4\uc81c NASA\uc5d0\uc11c \uacc4\ud68d \uc911\uc778 \u2018\uc18c\ud589\uc131 \ud3ec\ud68d \uacc4\ud68d (Asteroid Redirect Mission, ARM)\u2019\uc744 \uac8c\uc784 \uc0c1\uc5d0\uc11c \uc218\ud589 \uac00\ub2a5\ucf00 \ud574\uc8fc\ub294 \ub2e4\uc591\ud55c \ud30c\uce20\ub4e4\uc774 \ucd94\uac00\ub418\uc5c8\ub2e4.", "paragraph_sentence": " 0.23.5 \uc5c5\ub370\uc774\ud2b8\uc5d0\uc11c\ub294 \ubbf8 \ud56d\uacf5\uc6b0\uc8fc\uad6d NASA \uc640\uc758 \ucf5c\ub77c\ubcf4\ub808\uc774\uc158\uc744 \ud1b5\ud574 \uc0c8\ub85c\uc6b4 \ucc9c\uccb4\uc778 \uc18c\ud589\uc131\uacfc \uc2e4\uc81c NASA\uc5d0\uc11c \uacc4\ud68d \uc911\uc778 \u2018\uc18c\ud589\uc131 \ud3ec\ud68d \uacc4\ud68d (Asteroid Redirect Mission, ARM)\u2019\uc744 \uac8c\uc784 \uc0c1\uc5d0\uc11c \uc218\ud589 \uac00\ub2a5\ucf00 \ud574\uc8fc\ub294 \ub2e4\uc591\ud55c \ud30c\uce20\ub4e4\uc774 \ucd94\uac00\ub418\uc5c8\ub2e4. \ud50c\ub808\uc774\uc5b4\ub294 \uad00\uce21\uc18c\uc5d0\uc11c \ubbf8\ud655\uc778\ucc9c\uccb4\ub85c \ubc1c\uacac\ub418\ub294 \uc18c\ud589\uc131\ub4e4\uc744 \ucd94\uc801, \uad00\ucc30\ud558\uc5ec \uc815\ubcf4\ub97c \uc218\uc9d1\ud55c \ub4a4, \uc801\uc808\ud55c \uacc4\ud68d\uc744 \uad6c\uc0c1\ud574\uc57c\ud55c\ub2e4. \uc18c\ud589\uc131\uc740 \uc791\uac8c\ub294 \uc57d 10t \ub0a8\uc9d3(A\ub4f1\uae09)\ubd80\ud130 \ucd5c\ub300 \uc57d 4000t(E\ub4f1\uae09)\uae4c\uc9c0 \ud06c\uae30, \ubaa8\uc591, \uada4\ub3c4 \ubaa8\ub450 \ubb34\uc791\uc704\ub85c \uc0dd\uc131\ub418\uba70, \ub300\ubd80\ubd84 \ucee4\ube48 \uc8fc\ubcc0\uc5d0 \ub4f1\uc7a5\ud574\uc11c \uc11c\uc11c\ud788 \uc811\uadfc\ud558\ub294 \uada4\ub3c4\ub97c \uadf8\ub9b0\ub2e4. \ucee4\ube48\uc758 \uc778\ub825\uad8c\uc73c\ub85c \ub4e4\uc5b4\uc628 \uc18c\ud589\uc131\ub4e4\uc774 \uc5b4\ub5a4 \uada4\ub3c4\ub97c \uadf8\ub9b4\uc9c0\ub294 \ucc9c\ucc28\ub9cc\ubcc4\uc778\ub370, \uad00\uce21\uc18c\uc5d0\uc11c \uc608\uc0c1\uada4\ub3c4\ub97c \ud655\uc778 \ud55c \ub4a4, NASA\uc758 \ucc28\uc138\ub300 SLS \ub85c\ucf13\uc744 \ubcf8 \ub530 \ub9cc\ub4e0 \uc5ed\ub300 \ucd5c\uace0 \ucd94\ub825\uc758 \uc0c8\ub85c\uc6b4 \ucd08\ub300\ud615 \ubc1c\uc0ac\uccb4 \ud30c\uce20\uc640, \uc5b4\ub5a4 \ubb3c\uccb4\ub4e0\uc9c0 \uc6c0\ucf1c\uc958 \uc218 \uc788\ub294 \ud3ec\ud68d \ud30c\uce20\ub97c \uc774\uc6a9\ud574 \uc18c\ud589\uc131\uacfc \ub791\ub370\ubd80\ud560 \uc218 \uc788\ub2e4. \ucee4\ube48\uc744 \uc2a4\uccd0 \uc9c0\ub098\uac00\ub294 \uc18c\ud589\uc131\uc744 \ubd99\uc7a1\uc544\uc11c \uc81c3\uc758 \uc704\uc131\uc73c\ub85c \ub9cc\ub4e0\ub2e4\ub4e0\uc9c0, \ucda9\ub3cc\uada4\ub3c4\ub85c \ub3cc\uc9c4\ud558\ub294 \uac70\ub300\uc18c\ud589\uc131\uc744 \uc0ac\ub825\uc744 \ub2e4\ud574 \ubc00\uc5b4 \ub0b8\ub2e4\ub4e0\uc9c0, \uc544\ub2c8\uba74 \ud638\uae30\uc2ec\uacfc \ub2a5\ub825\uc5d0 \ub530\ub77c \ucee4\ube48\uc73c\ub85c \ub04c\uace0 \uc640\uc11c \uc6b0\uc8fc\uc13c\ud130 \uc55e\uc5d0 \ucc29\uc9c0 \uc2dc\ud0ac\uc9c0\ub294 \ucee4\ubc8c\ub4e4\uc774 \uc544\ub2cc \uc804\uc801\uc73c\ub85c \ud50c\ub808\uc774\uc5b4\uc758 \ub9c8\uc74c\uc774\ub2e4.", "paragraph_answer": "0.23.5 \uc5c5\ub370\uc774\ud2b8\uc5d0\uc11c\ub294 \ubbf8 \ud56d\uacf5\uc6b0\uc8fc\uad6d NASA \uc640\uc758 \ucf5c\ub77c\ubcf4\ub808\uc774\uc158\uc744 \ud1b5\ud574 \uc0c8\ub85c\uc6b4 \ucc9c\uccb4\uc778 \uc18c\ud589\uc131\uacfc \uc2e4\uc81c NASA\uc5d0\uc11c \uacc4\ud68d \uc911\uc778 \u2018\uc18c\ud589\uc131 \ud3ec\ud68d \uacc4\ud68d (Asteroid Redirect Mission, ARM)\u2019\uc744 \uac8c\uc784 \uc0c1\uc5d0\uc11c \uc218\ud589 \uac00\ub2a5\ucf00 \ud574\uc8fc\ub294 \ub2e4\uc591\ud55c \ud30c\uce20\ub4e4\uc774 \ucd94\uac00\ub418\uc5c8\ub2e4. \ud50c\ub808\uc774\uc5b4\ub294 \uad00\uce21\uc18c\uc5d0\uc11c \ubbf8\ud655\uc778\ucc9c\uccb4\ub85c \ubc1c\uacac\ub418\ub294 \uc18c\ud589\uc131\ub4e4\uc744 \ucd94\uc801, \uad00\ucc30\ud558\uc5ec \uc815\ubcf4\ub97c \uc218\uc9d1\ud55c \ub4a4, \uc801\uc808\ud55c \uacc4\ud68d\uc744 \uad6c\uc0c1\ud574\uc57c\ud55c\ub2e4. \uc18c\ud589\uc131\uc740 \uc791\uac8c\ub294 \uc57d 10t \ub0a8\uc9d3(A\ub4f1\uae09)\ubd80\ud130 \ucd5c\ub300 \uc57d 4000t(E\ub4f1\uae09)\uae4c\uc9c0 \ud06c\uae30, \ubaa8\uc591, \uada4\ub3c4 \ubaa8\ub450 \ubb34\uc791\uc704\ub85c \uc0dd\uc131\ub418\uba70, \ub300\ubd80\ubd84 \ucee4\ube48 \uc8fc\ubcc0\uc5d0 \ub4f1\uc7a5\ud574\uc11c \uc11c\uc11c\ud788 \uc811\uadfc\ud558\ub294 \uada4\ub3c4\ub97c \uadf8\ub9b0\ub2e4. \ucee4\ube48\uc758 \uc778\ub825\uad8c\uc73c\ub85c \ub4e4\uc5b4\uc628 \uc18c\ud589\uc131\ub4e4\uc774 \uc5b4\ub5a4 \uada4\ub3c4\ub97c \uadf8\ub9b4\uc9c0\ub294 \ucc9c\ucc28\ub9cc\ubcc4\uc778\ub370, \uad00\uce21\uc18c\uc5d0\uc11c \uc608\uc0c1\uada4\ub3c4\ub97c \ud655\uc778 \ud55c \ub4a4, NASA\uc758 \ucc28\uc138\ub300 SLS \ub85c\ucf13\uc744 \ubcf8 \ub530 \ub9cc\ub4e0 \uc5ed\ub300 \ucd5c\uace0 \ucd94\ub825\uc758 \uc0c8\ub85c\uc6b4 \ucd08\ub300\ud615 \ubc1c\uc0ac\uccb4 \ud30c\uce20\uc640, \uc5b4\ub5a4 \ubb3c\uccb4\ub4e0\uc9c0 \uc6c0\ucf1c\uc958 \uc218 \uc788\ub294 \ud3ec\ud68d \ud30c\uce20\ub97c \uc774\uc6a9\ud574 \uc18c\ud589\uc131\uacfc \ub791\ub370\ubd80\ud560 \uc218 \uc788\ub2e4. \ucee4\ube48\uc744 \uc2a4\uccd0 \uc9c0\ub098\uac00\ub294 \uc18c\ud589\uc131\uc744 \ubd99\uc7a1\uc544\uc11c \uc81c3\uc758 \uc704\uc131\uc73c\ub85c \ub9cc\ub4e0\ub2e4\ub4e0\uc9c0, \ucda9\ub3cc\uada4\ub3c4\ub85c \ub3cc\uc9c4\ud558\ub294 \uac70\ub300\uc18c\ud589\uc131\uc744 \uc0ac\ub825\uc744 \ub2e4\ud574 \ubc00\uc5b4 \ub0b8\ub2e4\ub4e0\uc9c0, \uc544\ub2c8\uba74 \ud638\uae30\uc2ec\uacfc \ub2a5\ub825\uc5d0 \ub530\ub77c \ucee4\ube48\uc73c\ub85c \ub04c\uace0 \uc640\uc11c \uc6b0\uc8fc\uc13c\ud130 \uc55e\uc5d0 \ucc29\uc9c0 \uc2dc\ud0ac\uc9c0\ub294 \ucee4\ubc8c\ub4e4\uc774 \uc544\ub2cc \uc804\uc801\uc73c\ub85c \ud50c\ub808\uc774\uc5b4\uc758 \ub9c8\uc74c\uc774\ub2e4.", "sentence_answer": "0.23.5 \uc5c5\ub370\uc774\ud2b8\uc5d0\uc11c\ub294 \ubbf8 \ud56d\uacf5\uc6b0\uc8fc\uad6d NASA \uc640\uc758 \ucf5c\ub77c\ubcf4\ub808\uc774\uc158\uc744 \ud1b5\ud574 \uc0c8\ub85c\uc6b4 \ucc9c\uccb4\uc778 \uc18c\ud589\uc131\uacfc \uc2e4\uc81c NASA\uc5d0\uc11c \uacc4\ud68d \uc911\uc778 \u2018\uc18c\ud589\uc131 \ud3ec\ud68d \uacc4\ud68d (Asteroid Redirect Mission, ARM)\u2019\uc744 \uac8c\uc784 \uc0c1\uc5d0\uc11c \uc218\ud589 \uac00\ub2a5\ucf00 \ud574\uc8fc\ub294 \ub2e4\uc591\ud55c \ud30c\uce20\ub4e4\uc774 \ucd94\uac00\ub418\uc5c8\ub2e4.", "paragraph_id": "6561656-1-0", "paragraph_question": "question: 0.23.5 \uc5c5\ub370\uc774\ud2b8\uc5d0\uc11c \ucf5c\ub77c\ubcf4\ub808\uc774\uc158 \ud55c \uacf3\uc740 \uc5b4\ub514\uc778\uac00?, context: 0.23.5 \uc5c5\ub370\uc774\ud2b8\uc5d0\uc11c\ub294 \ubbf8 \ud56d\uacf5\uc6b0\uc8fc\uad6d NASA\uc640\uc758 \ucf5c\ub77c\ubcf4\ub808\uc774\uc158\uc744 \ud1b5\ud574 \uc0c8\ub85c\uc6b4 \ucc9c\uccb4\uc778 \uc18c\ud589\uc131\uacfc \uc2e4\uc81c NASA\uc5d0\uc11c \uacc4\ud68d \uc911\uc778 \u2018\uc18c\ud589\uc131 \ud3ec\ud68d \uacc4\ud68d (Asteroid Redirect Mission, ARM)\u2019\uc744 \uac8c\uc784 \uc0c1\uc5d0\uc11c \uc218\ud589 \uac00\ub2a5\ucf00 \ud574\uc8fc\ub294 \ub2e4\uc591\ud55c \ud30c\uce20\ub4e4\uc774 \ucd94\uac00\ub418\uc5c8\ub2e4. \ud50c\ub808\uc774\uc5b4\ub294 \uad00\uce21\uc18c\uc5d0\uc11c \ubbf8\ud655\uc778\ucc9c\uccb4\ub85c \ubc1c\uacac\ub418\ub294 \uc18c\ud589\uc131\ub4e4\uc744 \ucd94\uc801, \uad00\ucc30\ud558\uc5ec \uc815\ubcf4\ub97c \uc218\uc9d1\ud55c \ub4a4, \uc801\uc808\ud55c \uacc4\ud68d\uc744 \uad6c\uc0c1\ud574\uc57c\ud55c\ub2e4. \uc18c\ud589\uc131\uc740 \uc791\uac8c\ub294 \uc57d 10t \ub0a8\uc9d3(A\ub4f1\uae09)\ubd80\ud130 \ucd5c\ub300 \uc57d 4000t(E\ub4f1\uae09)\uae4c\uc9c0 \ud06c\uae30, \ubaa8\uc591, \uada4\ub3c4 \ubaa8\ub450 \ubb34\uc791\uc704\ub85c \uc0dd\uc131\ub418\uba70, \ub300\ubd80\ubd84 \ucee4\ube48 \uc8fc\ubcc0\uc5d0 \ub4f1\uc7a5\ud574\uc11c \uc11c\uc11c\ud788 \uc811\uadfc\ud558\ub294 \uada4\ub3c4\ub97c \uadf8\ub9b0\ub2e4. \ucee4\ube48\uc758 \uc778\ub825\uad8c\uc73c\ub85c \ub4e4\uc5b4\uc628 \uc18c\ud589\uc131\ub4e4\uc774 \uc5b4\ub5a4 \uada4\ub3c4\ub97c \uadf8\ub9b4\uc9c0\ub294 \ucc9c\ucc28\ub9cc\ubcc4\uc778\ub370, \uad00\uce21\uc18c\uc5d0\uc11c \uc608\uc0c1\uada4\ub3c4\ub97c \ud655\uc778 \ud55c \ub4a4, NASA\uc758 \ucc28\uc138\ub300 SLS \ub85c\ucf13\uc744 \ubcf8 \ub530 \ub9cc\ub4e0 \uc5ed\ub300 \ucd5c\uace0 \ucd94\ub825\uc758 \uc0c8\ub85c\uc6b4 \ucd08\ub300\ud615 \ubc1c\uc0ac\uccb4 \ud30c\uce20\uc640, \uc5b4\ub5a4 \ubb3c\uccb4\ub4e0\uc9c0 \uc6c0\ucf1c\uc958 \uc218 \uc788\ub294 \ud3ec\ud68d \ud30c\uce20\ub97c \uc774\uc6a9\ud574 \uc18c\ud589\uc131\uacfc \ub791\ub370\ubd80\ud560 \uc218 \uc788\ub2e4. \ucee4\ube48\uc744 \uc2a4\uccd0 \uc9c0\ub098\uac00\ub294 \uc18c\ud589\uc131\uc744 \ubd99\uc7a1\uc544\uc11c \uc81c3\uc758 \uc704\uc131\uc73c\ub85c \ub9cc\ub4e0\ub2e4\ub4e0\uc9c0, \ucda9\ub3cc\uada4\ub3c4\ub85c \ub3cc\uc9c4\ud558\ub294 \uac70\ub300\uc18c\ud589\uc131\uc744 \uc0ac\ub825\uc744 \ub2e4\ud574 \ubc00\uc5b4 \ub0b8\ub2e4\ub4e0\uc9c0, \uc544\ub2c8\uba74 \ud638\uae30\uc2ec\uacfc \ub2a5\ub825\uc5d0 \ub530\ub77c \ucee4\ube48\uc73c\ub85c \ub04c\uace0 \uc640\uc11c \uc6b0\uc8fc\uc13c\ud130 \uc55e\uc5d0 \ucc29\uc9c0 \uc2dc\ud0ac\uc9c0\ub294 \ucee4\ubc8c\ub4e4\uc774 \uc544\ub2cc \uc804\uc801\uc73c\ub85c \ud50c\ub808\uc774\uc5b4\uc758 \ub9c8\uc74c\uc774\ub2e4."} {"question": "\ub2ec\ub9c8\ud2f0\uc544\uc758 \ubd80\uc871\ub4e4\uacfc \ubc8c\uc600\ub358 \uc804\uc7c1 \uc774\ud6c4 \ub85c\ub9c8\uad70\uc774 \ucc29\uc2e4\ud788 \uc9c4\uaca9\ud558\ub358 \uacf3\uc740?", "paragraph": "\ud30c\ub974\ud2f0\uc544\uac00 \uc5b8\uc81c\ub098 \uc704\ud611\uc801\uc778 \uc0c1\ub300\uc774\uae30\ub294 \ud588\uc9c0\ub9cc, \uc2e4\uc81c \uc804\uc7c1\uc740 \uac8c\ub974\ub9cc \uc871\uc744 \uc0c1\ub300\ub85c \ub300\ubd80\ubd84 \ub77c\uc778 \uac15, \ub3c4\ub098\uc6b0 \uac15 \uadfc\uad50\uc5d0\uc11c \ubc8c\uc5b4\uc84c\ub2e4. \uc548\ud1a0\ub2c8\uc6b0\uc2a4\uc640 \ucd5c\uc885 \uc804\ud22c\ub97c \ubc8c\uc774\uae30 \uc804\uc5d0 \ub2ec\ub9c8\ud2f0\uc544\uc758 \ubd80\uc871\ub4e4\uacfc \ubc8c\uc600\ub358 \uc804\uc7c1 \uc774\ud6c4 \ub85c\ub9c8\uad70\uc740 \ucc29\uc2e4\ud788 \ub3c4\ub098\uc6b0 \uac15\uc744 \ud5a5\ud574 \uc9c4\uaca9\ud558\uae30 \uc2dc\uc791\ud588\ub2e4. \ud558\uc9c0\ub9cc \uc5ec\ub7ec \ucc28\ub840 \uc804\uc7c1\uc5d0\uc11c \uc2b9\ub9ac\ud588\uc74c\uc5d0\ub3c4 \ubd88\uad6c\ud558\uace0 \uac8c\ub974\ub9c8\ub2c8\uc544 \uc9c0\uc5ed\uc740 \ub85c\ub9c8\ud654\ud558\ub294 \ub370 \uc2e4\ud328\ud558\uc600\ub2e4. 9\ub144\uc5d0 \ud1a0\uc774\ud1a0\ubd80\ub974\ud06c \uc232\uc5d0\uc11c \ub2f9\ud55c \ucc38\ud328\uac00 \uc774\ub97c \uc0c1\uc9d5\uc801\uc73c\ub85c \ubcf4\uc5ec\uc8fc\ub294 \uc0ac\uac74\uc774\ub2e4. \uac8c\ub974\ub9c8\ub2c8\uc544\uc758 \ucd1d\ub3c5\uc778 \ubc14\ub8e8\uc2a4\uac00 \uc774\ub044\ub294 3\uac1c \uad70\ub2e8\uc774 \ucf00\ub8e8\uc2a4\ud0a4 \uc871 \ucd9c\uc2e0\uc758 \uc544\ub974\ubbf8\ub2c8\uc6b0\uc2a4\uac00 \uc774\ub044\ub294 \uac8c\ub974\ub9cc \uc871\uc5d0\uac8c \uc804\uba78\ud55c\ub2e4. \uc544\uc6b0\uad6c\uc2a4\ud22c\uc2a4\ub294 \uc0ac\ud0dc\ub97c \uc218\uc2b5\ud558\ub824 \ud588\uace0, \ud2f0\ubca0\ub9ac\uc6b0\uc2a4\ub294 \uc774\ud6c4 \uc5ec\ub7ec \ucc28\ub840 \ub77c\uc778\ub780\ud2b8\ub85c \uc9c4\uaca9\ud558\uc5ec \uc2b9\ub9ac\ub97c \uac70\ub450\uc5c8\ub2e4. \ud558\uc9c0\ub9cc \uc544\uc6b0\uad6c\uc2a4\ud22c\uc2a4\uc758 \uc8fd\uc740 \ud6c4\uc5d0 \uacc4\uc2b9\uc790\uc778 \ud2f0\ubca0\ub9ac\uc6b0\uc2a4\ub294 \uac8c\ub974\ub9c8\ub2c8\uc544\uc5d0\uc11c \ucca0\uc218\ud558\uae30\ub85c \uacb0\uc815\ud558\uc600\uace0, \uc774\ud6c4 \ub85c\ub9c8\uad70\uc740 \ub77c\uc778 \uac15\uacfc \ub3c4\ub098\uc6b0 \uac15\uc744 \uc8fc \ubc29\uc5b4\uc120\uc73c\ub85c \uc0bc\uc544 \ubc29\uc704 \uccb4\uacc4\ub97c \uad6c\ucd95\ud55c\ub2e4.", "answer": "\ub3c4\ub098\uc6b0 \uac15", "sentence": "\ud30c\ub974\ud2f0\uc544\uac00 \uc5b8\uc81c\ub098 \uc704\ud611\uc801\uc778 \uc0c1\ub300\uc774\uae30\ub294 \ud588\uc9c0\ub9cc, \uc2e4\uc81c \uc804\uc7c1\uc740 \uac8c\ub974\ub9cc \uc871\uc744 \uc0c1\ub300\ub85c \ub300\ubd80\ubd84 \ub77c\uc778 \uac15, \ub3c4\ub098\uc6b0 \uac15 \uadfc\uad50\uc5d0\uc11c \ubc8c\uc5b4\uc84c\ub2e4.", "paragraph_sentence": " \ud30c\ub974\ud2f0\uc544\uac00 \uc5b8\uc81c\ub098 \uc704\ud611\uc801\uc778 \uc0c1\ub300\uc774\uae30\ub294 \ud588\uc9c0\ub9cc, \uc2e4\uc81c \uc804\uc7c1\uc740 \uac8c\ub974\ub9cc \uc871\uc744 \uc0c1\ub300\ub85c \ub300\ubd80\ubd84 \ub77c\uc778 \uac15, \ub3c4\ub098\uc6b0 \uac15 \uadfc\uad50\uc5d0\uc11c \ubc8c\uc5b4\uc84c\ub2e4. \uc548\ud1a0\ub2c8\uc6b0\uc2a4\uc640 \ucd5c\uc885 \uc804\ud22c\ub97c \ubc8c\uc774\uae30 \uc804\uc5d0 \ub2ec\ub9c8\ud2f0\uc544\uc758 \ubd80\uc871\ub4e4\uacfc \ubc8c\uc600\ub358 \uc804\uc7c1 \uc774\ud6c4 \ub85c\ub9c8\uad70\uc740 \ucc29\uc2e4\ud788 \ub3c4\ub098\uc6b0 \uac15\uc744 \ud5a5\ud574 \uc9c4\uaca9\ud558\uae30 \uc2dc\uc791\ud588\ub2e4. \ud558\uc9c0\ub9cc \uc5ec\ub7ec \ucc28\ub840 \uc804\uc7c1\uc5d0\uc11c \uc2b9\ub9ac\ud588\uc74c\uc5d0\ub3c4 \ubd88\uad6c\ud558\uace0 \uac8c\ub974\ub9c8\ub2c8\uc544 \uc9c0\uc5ed\uc740 \ub85c\ub9c8\ud654\ud558\ub294 \ub370 \uc2e4\ud328\ud558\uc600\ub2e4. 9\ub144\uc5d0 \ud1a0\uc774\ud1a0\ubd80\ub974\ud06c \uc232\uc5d0\uc11c \ub2f9\ud55c \ucc38\ud328\uac00 \uc774\ub97c \uc0c1\uc9d5\uc801\uc73c\ub85c \ubcf4\uc5ec\uc8fc\ub294 \uc0ac\uac74\uc774\ub2e4. \uac8c\ub974\ub9c8\ub2c8\uc544\uc758 \ucd1d\ub3c5\uc778 \ubc14\ub8e8\uc2a4\uac00 \uc774\ub044\ub294 3\uac1c \uad70\ub2e8\uc774 \ucf00\ub8e8\uc2a4\ud0a4 \uc871 \ucd9c\uc2e0\uc758 \uc544\ub974\ubbf8\ub2c8\uc6b0\uc2a4\uac00 \uc774\ub044\ub294 \uac8c\ub974\ub9cc \uc871\uc5d0\uac8c \uc804\uba78\ud55c\ub2e4. \uc544\uc6b0\uad6c\uc2a4\ud22c\uc2a4\ub294 \uc0ac\ud0dc\ub97c \uc218\uc2b5\ud558\ub824 \ud588\uace0, \ud2f0\ubca0\ub9ac\uc6b0\uc2a4\ub294 \uc774\ud6c4 \uc5ec\ub7ec \ucc28\ub840 \ub77c\uc778\ub780\ud2b8\ub85c \uc9c4\uaca9\ud558\uc5ec \uc2b9\ub9ac\ub97c \uac70\ub450\uc5c8\ub2e4. \ud558\uc9c0\ub9cc \uc544\uc6b0\uad6c\uc2a4\ud22c\uc2a4\uc758 \uc8fd\uc740 \ud6c4\uc5d0 \uacc4\uc2b9\uc790\uc778 \ud2f0\ubca0\ub9ac\uc6b0\uc2a4\ub294 \uac8c\ub974\ub9c8\ub2c8\uc544\uc5d0\uc11c \ucca0\uc218\ud558\uae30\ub85c \uacb0\uc815\ud558\uc600\uace0, \uc774\ud6c4 \ub85c\ub9c8\uad70\uc740 \ub77c\uc778 \uac15\uacfc \ub3c4\ub098\uc6b0 \uac15\uc744 \uc8fc \ubc29\uc5b4\uc120\uc73c\ub85c \uc0bc\uc544 \ubc29\uc704 \uccb4\uacc4\ub97c \uad6c\ucd95\ud55c\ub2e4.", "paragraph_answer": "\ud30c\ub974\ud2f0\uc544\uac00 \uc5b8\uc81c\ub098 \uc704\ud611\uc801\uc778 \uc0c1\ub300\uc774\uae30\ub294 \ud588\uc9c0\ub9cc, \uc2e4\uc81c \uc804\uc7c1\uc740 \uac8c\ub974\ub9cc \uc871\uc744 \uc0c1\ub300\ub85c \ub300\ubd80\ubd84 \ub77c\uc778 \uac15, \ub3c4\ub098\uc6b0 \uac15 \uadfc\uad50\uc5d0\uc11c \ubc8c\uc5b4\uc84c\ub2e4. \uc548\ud1a0\ub2c8\uc6b0\uc2a4\uc640 \ucd5c\uc885 \uc804\ud22c\ub97c \ubc8c\uc774\uae30 \uc804\uc5d0 \ub2ec\ub9c8\ud2f0\uc544\uc758 \ubd80\uc871\ub4e4\uacfc \ubc8c\uc600\ub358 \uc804\uc7c1 \uc774\ud6c4 \ub85c\ub9c8\uad70\uc740 \ucc29\uc2e4\ud788 \ub3c4\ub098\uc6b0 \uac15\uc744 \ud5a5\ud574 \uc9c4\uaca9\ud558\uae30 \uc2dc\uc791\ud588\ub2e4. \ud558\uc9c0\ub9cc \uc5ec\ub7ec \ucc28\ub840 \uc804\uc7c1\uc5d0\uc11c \uc2b9\ub9ac\ud588\uc74c\uc5d0\ub3c4 \ubd88\uad6c\ud558\uace0 \uac8c\ub974\ub9c8\ub2c8\uc544 \uc9c0\uc5ed\uc740 \ub85c\ub9c8\ud654\ud558\ub294 \ub370 \uc2e4\ud328\ud558\uc600\ub2e4. 9\ub144\uc5d0 \ud1a0\uc774\ud1a0\ubd80\ub974\ud06c \uc232\uc5d0\uc11c \ub2f9\ud55c \ucc38\ud328\uac00 \uc774\ub97c \uc0c1\uc9d5\uc801\uc73c\ub85c \ubcf4\uc5ec\uc8fc\ub294 \uc0ac\uac74\uc774\ub2e4. \uac8c\ub974\ub9c8\ub2c8\uc544\uc758 \ucd1d\ub3c5\uc778 \ubc14\ub8e8\uc2a4\uac00 \uc774\ub044\ub294 3\uac1c \uad70\ub2e8\uc774 \ucf00\ub8e8\uc2a4\ud0a4 \uc871 \ucd9c\uc2e0\uc758 \uc544\ub974\ubbf8\ub2c8\uc6b0\uc2a4\uac00 \uc774\ub044\ub294 \uac8c\ub974\ub9cc \uc871\uc5d0\uac8c \uc804\uba78\ud55c\ub2e4. \uc544\uc6b0\uad6c\uc2a4\ud22c\uc2a4\ub294 \uc0ac\ud0dc\ub97c \uc218\uc2b5\ud558\ub824 \ud588\uace0, \ud2f0\ubca0\ub9ac\uc6b0\uc2a4\ub294 \uc774\ud6c4 \uc5ec\ub7ec \ucc28\ub840 \ub77c\uc778\ub780\ud2b8\ub85c \uc9c4\uaca9\ud558\uc5ec \uc2b9\ub9ac\ub97c \uac70\ub450\uc5c8\ub2e4. \ud558\uc9c0\ub9cc \uc544\uc6b0\uad6c\uc2a4\ud22c\uc2a4\uc758 \uc8fd\uc740 \ud6c4\uc5d0 \uacc4\uc2b9\uc790\uc778 \ud2f0\ubca0\ub9ac\uc6b0\uc2a4\ub294 \uac8c\ub974\ub9c8\ub2c8\uc544\uc5d0\uc11c \ucca0\uc218\ud558\uae30\ub85c \uacb0\uc815\ud558\uc600\uace0, \uc774\ud6c4 \ub85c\ub9c8\uad70\uc740 \ub77c\uc778 \uac15\uacfc \ub3c4\ub098\uc6b0 \uac15\uc744 \uc8fc \ubc29\uc5b4\uc120\uc73c\ub85c \uc0bc\uc544 \ubc29\uc704 \uccb4\uacc4\ub97c \uad6c\ucd95\ud55c\ub2e4.", "sentence_answer": "\ud30c\ub974\ud2f0\uc544\uac00 \uc5b8\uc81c\ub098 \uc704\ud611\uc801\uc778 \uc0c1\ub300\uc774\uae30\ub294 \ud588\uc9c0\ub9cc, \uc2e4\uc81c \uc804\uc7c1\uc740 \uac8c\ub974\ub9cc \uc871\uc744 \uc0c1\ub300\ub85c \ub300\ubd80\ubd84 \ub77c\uc778 \uac15, \ub3c4\ub098\uc6b0 \uac15 \uadfc\uad50\uc5d0\uc11c \ubc8c\uc5b4\uc84c\ub2e4.", "paragraph_id": "6520922-28-0", "paragraph_question": "question: \ub2ec\ub9c8\ud2f0\uc544\uc758 \ubd80\uc871\ub4e4\uacfc \ubc8c\uc600\ub358 \uc804\uc7c1 \uc774\ud6c4 \ub85c\ub9c8\uad70\uc774 \ucc29\uc2e4\ud788 \uc9c4\uaca9\ud558\ub358 \uacf3\uc740?, context: \ud30c\ub974\ud2f0\uc544\uac00 \uc5b8\uc81c\ub098 \uc704\ud611\uc801\uc778 \uc0c1\ub300\uc774\uae30\ub294 \ud588\uc9c0\ub9cc, \uc2e4\uc81c \uc804\uc7c1\uc740 \uac8c\ub974\ub9cc \uc871\uc744 \uc0c1\ub300\ub85c \ub300\ubd80\ubd84 \ub77c\uc778 \uac15, \ub3c4\ub098\uc6b0 \uac15 \uadfc\uad50\uc5d0\uc11c \ubc8c\uc5b4\uc84c\ub2e4. \uc548\ud1a0\ub2c8\uc6b0\uc2a4\uc640 \ucd5c\uc885 \uc804\ud22c\ub97c \ubc8c\uc774\uae30 \uc804\uc5d0 \ub2ec\ub9c8\ud2f0\uc544\uc758 \ubd80\uc871\ub4e4\uacfc \ubc8c\uc600\ub358 \uc804\uc7c1 \uc774\ud6c4 \ub85c\ub9c8\uad70\uc740 \ucc29\uc2e4\ud788 \ub3c4\ub098\uc6b0 \uac15\uc744 \ud5a5\ud574 \uc9c4\uaca9\ud558\uae30 \uc2dc\uc791\ud588\ub2e4. \ud558\uc9c0\ub9cc \uc5ec\ub7ec \ucc28\ub840 \uc804\uc7c1\uc5d0\uc11c \uc2b9\ub9ac\ud588\uc74c\uc5d0\ub3c4 \ubd88\uad6c\ud558\uace0 \uac8c\ub974\ub9c8\ub2c8\uc544 \uc9c0\uc5ed\uc740 \ub85c\ub9c8\ud654\ud558\ub294 \ub370 \uc2e4\ud328\ud558\uc600\ub2e4. 9\ub144\uc5d0 \ud1a0\uc774\ud1a0\ubd80\ub974\ud06c \uc232\uc5d0\uc11c \ub2f9\ud55c \ucc38\ud328\uac00 \uc774\ub97c \uc0c1\uc9d5\uc801\uc73c\ub85c \ubcf4\uc5ec\uc8fc\ub294 \uc0ac\uac74\uc774\ub2e4. \uac8c\ub974\ub9c8\ub2c8\uc544\uc758 \ucd1d\ub3c5\uc778 \ubc14\ub8e8\uc2a4\uac00 \uc774\ub044\ub294 3\uac1c \uad70\ub2e8\uc774 \ucf00\ub8e8\uc2a4\ud0a4 \uc871 \ucd9c\uc2e0\uc758 \uc544\ub974\ubbf8\ub2c8\uc6b0\uc2a4\uac00 \uc774\ub044\ub294 \uac8c\ub974\ub9cc \uc871\uc5d0\uac8c \uc804\uba78\ud55c\ub2e4. \uc544\uc6b0\uad6c\uc2a4\ud22c\uc2a4\ub294 \uc0ac\ud0dc\ub97c \uc218\uc2b5\ud558\ub824 \ud588\uace0, \ud2f0\ubca0\ub9ac\uc6b0\uc2a4\ub294 \uc774\ud6c4 \uc5ec\ub7ec \ucc28\ub840 \ub77c\uc778\ub780\ud2b8\ub85c \uc9c4\uaca9\ud558\uc5ec \uc2b9\ub9ac\ub97c \uac70\ub450\uc5c8\ub2e4. \ud558\uc9c0\ub9cc \uc544\uc6b0\uad6c\uc2a4\ud22c\uc2a4\uc758 \uc8fd\uc740 \ud6c4\uc5d0 \uacc4\uc2b9\uc790\uc778 \ud2f0\ubca0\ub9ac\uc6b0\uc2a4\ub294 \uac8c\ub974\ub9c8\ub2c8\uc544\uc5d0\uc11c \ucca0\uc218\ud558\uae30\ub85c \uacb0\uc815\ud558\uc600\uace0, \uc774\ud6c4 \ub85c\ub9c8\uad70\uc740 \ub77c\uc778 \uac15\uacfc \ub3c4\ub098\uc6b0 \uac15\uc744 \uc8fc \ubc29\uc5b4\uc120\uc73c\ub85c \uc0bc\uc544 \ubc29\uc704 \uccb4\uacc4\ub97c \uad6c\ucd95\ud55c\ub2e4."} {"question": "\ucd5c\ucd08\uc758 \uc0ac\ud68c\ud559\uc790\ub85c \uc5ec\uaca8\uc9c0\uace0 \uc788\ub294 \uc778\ubb3c\uc740?", "paragraph": "\uc774\ube10 \ud560\ub454\uc740 \ucd5c\ucd08\uc758 \uc0ac\ud68c\ud559\uc790\ub85c \uc5ec\uaca8\uc9c0\uace0 \uc788\ub2e4. \ud604\ub300\uc801\uc778 \uc0ac\ud68c\ud559\uc740 19\uc138\uae30 \ucd08\ubc18 \uc138\uacc4\uc801\uc73c\ub85c \ud559\ubb38 \ud604\ub300\ud654 \ubc14\ub78c\uc774 \ubd88 \ub54c \ub098\ud0c0\ub098\uac8c \ub418\uc5c8\ub2e4. \ub4a4\ub974\ucf10\uacfc \uac19\uc740 \ucd08\uae30 \uc0ac\ud68c\ud559\uc790\ub4e4\uc740 \uc0ac\ud68c\ud559\uc758 \ubaa9\ud45c\ub97c \uad6c\uc870 \uae30\ub2a5\uc8fc\uc758(Structural Functionalism)\uc758 \ud655\ub9bd\uc744 \ud1b5\ud55c \uc0ac\ud68c\uc801 \ub2e8\uccb4\uc758 \ubcf5\uc7a1\ud55c \uacb0\ud569\uc744 \uc774\ud574\ud558\uace0 \uadf8 \uacb0\ud569 \uad00\uacc4\ub97c \ud574\uc11d\ud558\ub294 \uac83\uc73c\ub85c \uc124\uc815\ud558\uc600\ub2e4. \ubca0\ubc84\ub294 \uc0ac\ud68c\ud604\uc0c1\uc5d0 \ub300\ud574 \uac1c\uc778 \uac04\uc758 \uc778\uc2dd \ucc28\uc774\ub97c \ud574\uc18c\ud558\ub294 \uc6a9\uc5b4 \uc815\uc758\uc640 \uac19\uc740 \uc774\ub860\ud654(Rationalization)\ub97c \ud1b5\ud574 \uc0ac\ud68c\ub97c \ubc1c\uc804\uc2dc\ud0ac \uc218 \uc788\ub2e4\uace0 \uc8fc\uc7a5\ud558\uc600\ub2e4. \uac8c\uc624\ub974\ud06c \uc9c0\uba5c, \ub4c0\ubcf4\uc774\uc2a4(W. E. B. Du Bois)\uc640 \uac19\uc740 \uc0ac\ud68c\ud559\uc790\ub4e4\uc740 \ubbf8\uc2dc\uc0ac\ud68c\ud559(Microsociology)\uacfc \uc815\ub7c9\ud654\uc758 \uae38\uc744 \uc5f4\uc5c8\ub2e4. \ubbf8\ub4dc\uc640 \uadf8\uc758 \uc870\uad50 \ud5c8\ubc84\ud2b8 \ube14\ub8e8\uba38(Herbert Blumer)\uac00 \ud589\ud55c \ubbf8\uad6d \uc0ac\ud68c\uc758 \uc911\uc694 \uc5ed\ud560\uc5d0 \ub300\ud55c \ubbf8\uc2dc\uc801 \uc811\uadfc\uc740 \uc0c1\uc9d5\uc801 \uc0c1\ud638\uc791\uc6a9\ub860(Symbolic Interactionism)\uc774\ub77c\ub294 \uc0c8\ub85c\uc6b4 \uc0ac\ud68c\ud559\uc801 \ubd84\uc57c\ub97c \ud30c\uc0dd\ud558\uc600\ub2e4.", "answer": "\uc774\ube10 \ud560\ub454", "sentence": "\uc774\ube10 \ud560\ub454 \uc740 \ucd5c\ucd08\uc758 \uc0ac\ud68c\ud559\uc790\ub85c \uc5ec\uaca8\uc9c0\uace0 \uc788\ub2e4.", "paragraph_sentence": " \uc774\ube10 \ud560\ub454 \uc740 \ucd5c\ucd08\uc758 \uc0ac\ud68c\ud559\uc790\ub85c \uc5ec\uaca8\uc9c0\uace0 \uc788\ub2e4. \ud604\ub300\uc801\uc778 \uc0ac\ud68c\ud559\uc740 19\uc138\uae30 \ucd08\ubc18 \uc138\uacc4\uc801\uc73c\ub85c \ud559\ubb38 \ud604\ub300\ud654 \ubc14\ub78c\uc774 \ubd88 \ub54c \ub098\ud0c0\ub098\uac8c \ub418\uc5c8\ub2e4. \ub4a4\ub974\ucf10\uacfc \uac19\uc740 \ucd08\uae30 \uc0ac\ud68c\ud559\uc790\ub4e4\uc740 \uc0ac\ud68c\ud559\uc758 \ubaa9\ud45c\ub97c \uad6c\uc870 \uae30\ub2a5\uc8fc\uc758(Structural Functionalism)\uc758 \ud655\ub9bd\uc744 \ud1b5\ud55c \uc0ac\ud68c\uc801 \ub2e8\uccb4\uc758 \ubcf5\uc7a1\ud55c \uacb0\ud569\uc744 \uc774\ud574\ud558\uace0 \uadf8 \uacb0\ud569 \uad00\uacc4\ub97c \ud574\uc11d\ud558\ub294 \uac83\uc73c\ub85c \uc124\uc815\ud558\uc600\ub2e4. \ubca0\ubc84\ub294 \uc0ac\ud68c\ud604\uc0c1\uc5d0 \ub300\ud574 \uac1c\uc778 \uac04\uc758 \uc778\uc2dd \ucc28\uc774\ub97c \ud574\uc18c\ud558\ub294 \uc6a9\uc5b4 \uc815\uc758\uc640 \uac19\uc740 \uc774\ub860\ud654(Rationalization)\ub97c \ud1b5\ud574 \uc0ac\ud68c\ub97c \ubc1c\uc804\uc2dc\ud0ac \uc218 \uc788\ub2e4\uace0 \uc8fc\uc7a5\ud558\uc600\ub2e4. \uac8c\uc624\ub974\ud06c \uc9c0\uba5c, \ub4c0\ubcf4\uc774\uc2a4(W. E. B. Du Bois)\uc640 \uac19\uc740 \uc0ac\ud68c\ud559\uc790\ub4e4\uc740 \ubbf8\uc2dc\uc0ac\ud68c\ud559(Microsociology)\uacfc \uc815\ub7c9\ud654\uc758 \uae38\uc744 \uc5f4\uc5c8\ub2e4. \ubbf8\ub4dc\uc640 \uadf8\uc758 \uc870\uad50 \ud5c8\ubc84\ud2b8 \ube14\ub8e8\uba38(Herbert Blumer)\uac00 \ud589\ud55c \ubbf8\uad6d \uc0ac\ud68c\uc758 \uc911\uc694 \uc5ed\ud560\uc5d0 \ub300\ud55c \ubbf8\uc2dc\uc801 \uc811\uadfc\uc740 \uc0c1\uc9d5\uc801 \uc0c1\ud638\uc791\uc6a9\ub860(Symbolic Interactionism)\uc774\ub77c\ub294 \uc0c8\ub85c\uc6b4 \uc0ac\ud68c\ud559\uc801 \ubd84\uc57c\ub97c \ud30c\uc0dd\ud558\uc600\ub2e4.", "paragraph_answer": " \uc774\ube10 \ud560\ub454 \uc740 \ucd5c\ucd08\uc758 \uc0ac\ud68c\ud559\uc790\ub85c \uc5ec\uaca8\uc9c0\uace0 \uc788\ub2e4. \ud604\ub300\uc801\uc778 \uc0ac\ud68c\ud559\uc740 19\uc138\uae30 \ucd08\ubc18 \uc138\uacc4\uc801\uc73c\ub85c \ud559\ubb38 \ud604\ub300\ud654 \ubc14\ub78c\uc774 \ubd88 \ub54c \ub098\ud0c0\ub098\uac8c \ub418\uc5c8\ub2e4. \ub4a4\ub974\ucf10\uacfc \uac19\uc740 \ucd08\uae30 \uc0ac\ud68c\ud559\uc790\ub4e4\uc740 \uc0ac\ud68c\ud559\uc758 \ubaa9\ud45c\ub97c \uad6c\uc870 \uae30\ub2a5\uc8fc\uc758(Structural Functionalism)\uc758 \ud655\ub9bd\uc744 \ud1b5\ud55c \uc0ac\ud68c\uc801 \ub2e8\uccb4\uc758 \ubcf5\uc7a1\ud55c \uacb0\ud569\uc744 \uc774\ud574\ud558\uace0 \uadf8 \uacb0\ud569 \uad00\uacc4\ub97c \ud574\uc11d\ud558\ub294 \uac83\uc73c\ub85c \uc124\uc815\ud558\uc600\ub2e4. \ubca0\ubc84\ub294 \uc0ac\ud68c\ud604\uc0c1\uc5d0 \ub300\ud574 \uac1c\uc778 \uac04\uc758 \uc778\uc2dd \ucc28\uc774\ub97c \ud574\uc18c\ud558\ub294 \uc6a9\uc5b4 \uc815\uc758\uc640 \uac19\uc740 \uc774\ub860\ud654(Rationalization)\ub97c \ud1b5\ud574 \uc0ac\ud68c\ub97c \ubc1c\uc804\uc2dc\ud0ac \uc218 \uc788\ub2e4\uace0 \uc8fc\uc7a5\ud558\uc600\ub2e4. \uac8c\uc624\ub974\ud06c \uc9c0\uba5c, \ub4c0\ubcf4\uc774\uc2a4(W. E. B. Du Bois)\uc640 \uac19\uc740 \uc0ac\ud68c\ud559\uc790\ub4e4\uc740 \ubbf8\uc2dc\uc0ac\ud68c\ud559(Microsociology)\uacfc \uc815\ub7c9\ud654\uc758 \uae38\uc744 \uc5f4\uc5c8\ub2e4. \ubbf8\ub4dc\uc640 \uadf8\uc758 \uc870\uad50 \ud5c8\ubc84\ud2b8 \ube14\ub8e8\uba38(Herbert Blumer)\uac00 \ud589\ud55c \ubbf8\uad6d \uc0ac\ud68c\uc758 \uc911\uc694 \uc5ed\ud560\uc5d0 \ub300\ud55c \ubbf8\uc2dc\uc801 \uc811\uadfc\uc740 \uc0c1\uc9d5\uc801 \uc0c1\ud638\uc791\uc6a9\ub860(Symbolic Interactionism)\uc774\ub77c\ub294 \uc0c8\ub85c\uc6b4 \uc0ac\ud68c\ud559\uc801 \ubd84\uc57c\ub97c \ud30c\uc0dd\ud558\uc600\ub2e4.", "sentence_answer": " \uc774\ube10 \ud560\ub454 \uc740 \ucd5c\ucd08\uc758 \uc0ac\ud68c\ud559\uc790\ub85c \uc5ec\uaca8\uc9c0\uace0 \uc788\ub2e4.", "paragraph_id": "6489929-32-0", "paragraph_question": "question: \ucd5c\ucd08\uc758 \uc0ac\ud68c\ud559\uc790\ub85c \uc5ec\uaca8\uc9c0\uace0 \uc788\ub294 \uc778\ubb3c\uc740?, context: \uc774\ube10 \ud560\ub454\uc740 \ucd5c\ucd08\uc758 \uc0ac\ud68c\ud559\uc790\ub85c \uc5ec\uaca8\uc9c0\uace0 \uc788\ub2e4. \ud604\ub300\uc801\uc778 \uc0ac\ud68c\ud559\uc740 19\uc138\uae30 \ucd08\ubc18 \uc138\uacc4\uc801\uc73c\ub85c \ud559\ubb38 \ud604\ub300\ud654 \ubc14\ub78c\uc774 \ubd88 \ub54c \ub098\ud0c0\ub098\uac8c \ub418\uc5c8\ub2e4. \ub4a4\ub974\ucf10\uacfc \uac19\uc740 \ucd08\uae30 \uc0ac\ud68c\ud559\uc790\ub4e4\uc740 \uc0ac\ud68c\ud559\uc758 \ubaa9\ud45c\ub97c \uad6c\uc870 \uae30\ub2a5\uc8fc\uc758(Structural Functionalism)\uc758 \ud655\ub9bd\uc744 \ud1b5\ud55c \uc0ac\ud68c\uc801 \ub2e8\uccb4\uc758 \ubcf5\uc7a1\ud55c \uacb0\ud569\uc744 \uc774\ud574\ud558\uace0 \uadf8 \uacb0\ud569 \uad00\uacc4\ub97c \ud574\uc11d\ud558\ub294 \uac83\uc73c\ub85c \uc124\uc815\ud558\uc600\ub2e4. \ubca0\ubc84\ub294 \uc0ac\ud68c\ud604\uc0c1\uc5d0 \ub300\ud574 \uac1c\uc778 \uac04\uc758 \uc778\uc2dd \ucc28\uc774\ub97c \ud574\uc18c\ud558\ub294 \uc6a9\uc5b4 \uc815\uc758\uc640 \uac19\uc740 \uc774\ub860\ud654(Rationalization)\ub97c \ud1b5\ud574 \uc0ac\ud68c\ub97c \ubc1c\uc804\uc2dc\ud0ac \uc218 \uc788\ub2e4\uace0 \uc8fc\uc7a5\ud558\uc600\ub2e4. \uac8c\uc624\ub974\ud06c \uc9c0\uba5c, \ub4c0\ubcf4\uc774\uc2a4(W. E. B. Du Bois)\uc640 \uac19\uc740 \uc0ac\ud68c\ud559\uc790\ub4e4\uc740 \ubbf8\uc2dc\uc0ac\ud68c\ud559(Microsociology)\uacfc \uc815\ub7c9\ud654\uc758 \uae38\uc744 \uc5f4\uc5c8\ub2e4. \ubbf8\ub4dc\uc640 \uadf8\uc758 \uc870\uad50 \ud5c8\ubc84\ud2b8 \ube14\ub8e8\uba38(Herbert Blumer)\uac00 \ud589\ud55c \ubbf8\uad6d \uc0ac\ud68c\uc758 \uc911\uc694 \uc5ed\ud560\uc5d0 \ub300\ud55c \ubbf8\uc2dc\uc801 \uc811\uadfc\uc740 \uc0c1\uc9d5\uc801 \uc0c1\ud638\uc791\uc6a9\ub860(Symbolic Interactionism)\uc774\ub77c\ub294 \uc0c8\ub85c\uc6b4 \uc0ac\ud68c\ud559\uc801 \ubd84\uc57c\ub97c \ud30c\uc0dd\ud558\uc600\ub2e4."} {"question": "\uc548\ub4dc\ub808\uc774 \ubcf4\uc988\ub124\uc13c\uc2a4\ud0a4\ub294 \uc5b4\ub9b0\uc2dc\uc808\uc744 \uc8fc\ub85c \uc5b4\ub514\uc5d0\uc11c \ubcf4\ub0c8\ub294\uac00?", "paragraph": "1933\ub144 5\uc6d4 12\uc77c \uc18c\ube44\uc5d0\ud2b8 \uc5f0\ubc29 \ubaa8\uc2a4\ud06c\ubc14\uc5d0\uc11c \ud0dc\uc5b4\ub0ac\ub2e4. \uadf8\ub294 \uc5b4\ub9b0 \uc2dc\uc808\uc744 \uc8fc\ub85c \uc18c\ube44\uc5d0\ud2b8 \uc5f0\ubc29\uc758 \ub3c4\uc2dc \ube14\ub77c\ub514\ubbf8\ub974\uc5d0\uc11c \ubcf4\ub0c8\ub2e4. \uc81c2\ucc28 \uc138\uacc4 \ub300\uc804 \uc911\uc5d0\ub294 \uc5b4\uba38\ub2c8\uc640 \ud568\uaed8 \uc6b0\ub784 \uc0b0\ub9e5\uc5d0 \uc788\ub294 \ucfe0\ub974\uac04 \uc9c0\ubc29\uc5d0\uc11c \uc0b4\uae30\ub3c4 \ud588\ub2e4. \uadf8\uc758 \uc591\uce5c\uc740 \ubaa8\ub450 \ubb38\ud559\uacfc \uc608\uc220\uc5d0 \uae4a\uc740 \uad00\uc2ec\uc774 \uc788\uc5c8\ub2e4. \uadf8\uc758 \uc5b4\uba38\ub2c8\ub294 \uc5b4\ub9b0 \uc2dc\uc808\ubd80\ud130 \uadf8\uc5d0\uac8c \ubcf4\ub9ac\uc2a4 \ud30c\uc2a4\ud14c\ub974\ub098\ud06c\uc758 \uc2dc\ub97c \uc77d\uc5b4\uc8fc\uc5c8\ub2e4. \uadf8\uc758 \uc544\ubc84\uc9c0\ub294 \ub808\ub2cc\uadf8\ub77c\ub4dc\uc5d0\uc11c \uacf5\ud559 \uad50\uc218\ub85c \uc77c\ud588\ub2e4. \ubcf4\uc988\ub124\uc13c\uc2a4\ud0a4\ub294 \uc804\uc7c1 \uc911 \uc804\uc120\uc73c\ub85c \ub3cc\uc544\uac00\ub358 \uc544\ubc84\uc9c0\uac00 \ucfe0\ub974\uac04\uc5d0 \ub4e4\ub800\ub358 \ub0a0\uc744 \ud68c\uc0c1\ud55c\ub2e4. \uadf8\uc758 \uc544\ubc84\uc9c0\ub294 \uba74\ub3c4\ub97c \ud558\uc9c0 \uc54a\uc544 \ucd08\ucdcc\ud55c \ubaa8\uc2b5\uc73c\ub85c \uc57d\uac04\uc758 \uc2dd\ub7c9\uc774 \ub4e4\uc5b4 \uc788\ub294 \ubc30\ub0ad\uacfc \uace0\uc57c\uc758 \uc791\ud488\uc9d1\uc744 \uac00\uc838\uc654\ub2e4. \uace0\uc57c\uc758 \uadf8\ub9bc\uc740 \ud654\uac00\uac00 \ub418\uace0\uc790 \ud588\ub358 \uafc8 \ub9ce\uc740 \uc5b4\ub9b0 \uc18c\ub144\uc5d0\uac8c \uae4a\uc740 \uac10\uba85\uc744 \uc8fc\uc5c8\ub2e4. \ubcf4\uc988\ub124\uc13c\uc2a4\ud0a4\ub294 \uace0\uc57c\uc758 \uadf8\ub85c\ud14c\uc2a4\ud06c\ud558\uace0 \ubb34\uc2dc\ubb34\uc2dc\ud55c \uc804\uc7c1 \uadf8\ub9bc\uc744 \ud1b5\ud574\uc11c \uc804\uc7c1\uc758 \ucc38\uc0c1\uc744 \uc774\ud574\ud588\ub2e4. \ubc14\ub85c \uadf8\uc758 \uc720\uba85\ud55c \uc2dc \uff1c\ub098\ub294 \uace0\uc57c\uff1e(1957)\uac00 \uc804\uc7c1\uc5d0 \ub300\ud55c \uc2dc\uc778\uc758 \uc774\ud574\ub97c \ubc18\uc601\ud55c \uc791\ud488\uc774\ub2e4.", "answer": "\ube14\ub77c\ub514\ubbf8\ub974", "sentence": "\ub3c4\uc2dc \ube14\ub77c\ub514\ubbf8\ub974 \uc5d0\uc11c \ubcf4\ub0c8\ub2e4.", "paragraph_sentence": "1933\ub144 5\uc6d4 12\uc77c \uc18c\ube44\uc5d0\ud2b8 \uc5f0\ubc29 \ubaa8\uc2a4\ud06c\ubc14\uc5d0\uc11c \ud0dc\uc5b4\ub0ac\ub2e4. \uadf8\ub294 \uc5b4\ub9b0 \uc2dc\uc808\uc744 \uc8fc\ub85c \uc18c\ube44\uc5d0\ud2b8 \uc5f0\ubc29\uc758 \ub3c4\uc2dc \ube14\ub77c\ub514\ubbf8\ub974 \uc5d0\uc11c \ubcf4\ub0c8\ub2e4. \uc81c2\ucc28 \uc138\uacc4 \ub300\uc804 \uc911\uc5d0\ub294 \uc5b4\uba38\ub2c8\uc640 \ud568\uaed8 \uc6b0\ub784 \uc0b0\ub9e5\uc5d0 \uc788\ub294 \ucfe0\ub974\uac04 \uc9c0\ubc29\uc5d0\uc11c \uc0b4\uae30\ub3c4 \ud588\ub2e4. \uadf8\uc758 \uc591\uce5c\uc740 \ubaa8\ub450 \ubb38\ud559\uacfc \uc608\uc220\uc5d0 \uae4a\uc740 \uad00\uc2ec\uc774 \uc788\uc5c8\ub2e4. \uadf8\uc758 \uc5b4\uba38\ub2c8\ub294 \uc5b4\ub9b0 \uc2dc\uc808\ubd80\ud130 \uadf8\uc5d0\uac8c \ubcf4\ub9ac\uc2a4 \ud30c\uc2a4\ud14c\ub974\ub098\ud06c\uc758 \uc2dc\ub97c \uc77d\uc5b4\uc8fc\uc5c8\ub2e4. \uadf8\uc758 \uc544\ubc84\uc9c0\ub294 \ub808\ub2cc\uadf8\ub77c\ub4dc\uc5d0\uc11c \uacf5\ud559 \uad50\uc218\ub85c \uc77c\ud588\ub2e4. \ubcf4\uc988\ub124\uc13c\uc2a4\ud0a4\ub294 \uc804\uc7c1 \uc911 \uc804\uc120\uc73c\ub85c \ub3cc\uc544\uac00\ub358 \uc544\ubc84\uc9c0\uac00 \ucfe0\ub974\uac04\uc5d0 \ub4e4\ub800\ub358 \ub0a0\uc744 \ud68c\uc0c1\ud55c\ub2e4. \uadf8\uc758 \uc544\ubc84\uc9c0\ub294 \uba74\ub3c4\ub97c \ud558\uc9c0 \uc54a\uc544 \ucd08\ucdcc\ud55c \ubaa8\uc2b5\uc73c\ub85c \uc57d\uac04\uc758 \uc2dd\ub7c9\uc774 \ub4e4\uc5b4 \uc788\ub294 \ubc30\ub0ad\uacfc \uace0\uc57c\uc758 \uc791\ud488\uc9d1\uc744 \uac00\uc838\uc654\ub2e4. \uace0\uc57c\uc758 \uadf8\ub9bc\uc740 \ud654\uac00\uac00 \ub418\uace0\uc790 \ud588\ub358 \uafc8 \ub9ce\uc740 \uc5b4\ub9b0 \uc18c\ub144\uc5d0\uac8c \uae4a\uc740 \uac10\uba85\uc744 \uc8fc\uc5c8\ub2e4. \ubcf4\uc988\ub124\uc13c\uc2a4\ud0a4\ub294 \uace0\uc57c\uc758 \uadf8\ub85c\ud14c\uc2a4\ud06c\ud558\uace0 \ubb34\uc2dc\ubb34\uc2dc\ud55c \uc804\uc7c1 \uadf8\ub9bc\uc744 \ud1b5\ud574\uc11c \uc804\uc7c1\uc758 \ucc38\uc0c1\uc744 \uc774\ud574\ud588\ub2e4. \ubc14\ub85c \uadf8\uc758 \uc720\uba85\ud55c \uc2dc \uff1c\ub098\ub294 \uace0\uc57c\uff1e(1957)\uac00 \uc804\uc7c1\uc5d0 \ub300\ud55c \uc2dc\uc778\uc758 \uc774\ud574\ub97c \ubc18\uc601\ud55c \uc791\ud488\uc774\ub2e4.", "paragraph_answer": "1933\ub144 5\uc6d4 12\uc77c \uc18c\ube44\uc5d0\ud2b8 \uc5f0\ubc29 \ubaa8\uc2a4\ud06c\ubc14\uc5d0\uc11c \ud0dc\uc5b4\ub0ac\ub2e4. \uadf8\ub294 \uc5b4\ub9b0 \uc2dc\uc808\uc744 \uc8fc\ub85c \uc18c\ube44\uc5d0\ud2b8 \uc5f0\ubc29\uc758 \ub3c4\uc2dc \ube14\ub77c\ub514\ubbf8\ub974 \uc5d0\uc11c \ubcf4\ub0c8\ub2e4. \uc81c2\ucc28 \uc138\uacc4 \ub300\uc804 \uc911\uc5d0\ub294 \uc5b4\uba38\ub2c8\uc640 \ud568\uaed8 \uc6b0\ub784 \uc0b0\ub9e5\uc5d0 \uc788\ub294 \ucfe0\ub974\uac04 \uc9c0\ubc29\uc5d0\uc11c \uc0b4\uae30\ub3c4 \ud588\ub2e4. \uadf8\uc758 \uc591\uce5c\uc740 \ubaa8\ub450 \ubb38\ud559\uacfc \uc608\uc220\uc5d0 \uae4a\uc740 \uad00\uc2ec\uc774 \uc788\uc5c8\ub2e4. \uadf8\uc758 \uc5b4\uba38\ub2c8\ub294 \uc5b4\ub9b0 \uc2dc\uc808\ubd80\ud130 \uadf8\uc5d0\uac8c \ubcf4\ub9ac\uc2a4 \ud30c\uc2a4\ud14c\ub974\ub098\ud06c\uc758 \uc2dc\ub97c \uc77d\uc5b4\uc8fc\uc5c8\ub2e4. \uadf8\uc758 \uc544\ubc84\uc9c0\ub294 \ub808\ub2cc\uadf8\ub77c\ub4dc\uc5d0\uc11c \uacf5\ud559 \uad50\uc218\ub85c \uc77c\ud588\ub2e4. \ubcf4\uc988\ub124\uc13c\uc2a4\ud0a4\ub294 \uc804\uc7c1 \uc911 \uc804\uc120\uc73c\ub85c \ub3cc\uc544\uac00\ub358 \uc544\ubc84\uc9c0\uac00 \ucfe0\ub974\uac04\uc5d0 \ub4e4\ub800\ub358 \ub0a0\uc744 \ud68c\uc0c1\ud55c\ub2e4. \uadf8\uc758 \uc544\ubc84\uc9c0\ub294 \uba74\ub3c4\ub97c \ud558\uc9c0 \uc54a\uc544 \ucd08\ucdcc\ud55c \ubaa8\uc2b5\uc73c\ub85c \uc57d\uac04\uc758 \uc2dd\ub7c9\uc774 \ub4e4\uc5b4 \uc788\ub294 \ubc30\ub0ad\uacfc \uace0\uc57c\uc758 \uc791\ud488\uc9d1\uc744 \uac00\uc838\uc654\ub2e4. \uace0\uc57c\uc758 \uadf8\ub9bc\uc740 \ud654\uac00\uac00 \ub418\uace0\uc790 \ud588\ub358 \uafc8 \ub9ce\uc740 \uc5b4\ub9b0 \uc18c\ub144\uc5d0\uac8c \uae4a\uc740 \uac10\uba85\uc744 \uc8fc\uc5c8\ub2e4. \ubcf4\uc988\ub124\uc13c\uc2a4\ud0a4\ub294 \uace0\uc57c\uc758 \uadf8\ub85c\ud14c\uc2a4\ud06c\ud558\uace0 \ubb34\uc2dc\ubb34\uc2dc\ud55c \uc804\uc7c1 \uadf8\ub9bc\uc744 \ud1b5\ud574\uc11c \uc804\uc7c1\uc758 \ucc38\uc0c1\uc744 \uc774\ud574\ud588\ub2e4. \ubc14\ub85c \uadf8\uc758 \uc720\uba85\ud55c \uc2dc \uff1c\ub098\ub294 \uace0\uc57c\uff1e(1957)\uac00 \uc804\uc7c1\uc5d0 \ub300\ud55c \uc2dc\uc778\uc758 \uc774\ud574\ub97c \ubc18\uc601\ud55c \uc791\ud488\uc774\ub2e4.", "sentence_answer": "\ub3c4\uc2dc \ube14\ub77c\ub514\ubbf8\ub974 \uc5d0\uc11c \ubcf4\ub0c8\ub2e4.", "paragraph_id": "6521455-0-0", "paragraph_question": "question: \uc548\ub4dc\ub808\uc774 \ubcf4\uc988\ub124\uc13c\uc2a4\ud0a4\ub294 \uc5b4\ub9b0\uc2dc\uc808\uc744 \uc8fc\ub85c \uc5b4\ub514\uc5d0\uc11c \ubcf4\ub0c8\ub294\uac00?, context: 1933\ub144 5\uc6d4 12\uc77c \uc18c\ube44\uc5d0\ud2b8 \uc5f0\ubc29 \ubaa8\uc2a4\ud06c\ubc14\uc5d0\uc11c \ud0dc\uc5b4\ub0ac\ub2e4. \uadf8\ub294 \uc5b4\ub9b0 \uc2dc\uc808\uc744 \uc8fc\ub85c \uc18c\ube44\uc5d0\ud2b8 \uc5f0\ubc29\uc758 \ub3c4\uc2dc \ube14\ub77c\ub514\ubbf8\ub974\uc5d0\uc11c \ubcf4\ub0c8\ub2e4. \uc81c2\ucc28 \uc138\uacc4 \ub300\uc804 \uc911\uc5d0\ub294 \uc5b4\uba38\ub2c8\uc640 \ud568\uaed8 \uc6b0\ub784 \uc0b0\ub9e5\uc5d0 \uc788\ub294 \ucfe0\ub974\uac04 \uc9c0\ubc29\uc5d0\uc11c \uc0b4\uae30\ub3c4 \ud588\ub2e4. \uadf8\uc758 \uc591\uce5c\uc740 \ubaa8\ub450 \ubb38\ud559\uacfc \uc608\uc220\uc5d0 \uae4a\uc740 \uad00\uc2ec\uc774 \uc788\uc5c8\ub2e4. \uadf8\uc758 \uc5b4\uba38\ub2c8\ub294 \uc5b4\ub9b0 \uc2dc\uc808\ubd80\ud130 \uadf8\uc5d0\uac8c \ubcf4\ub9ac\uc2a4 \ud30c\uc2a4\ud14c\ub974\ub098\ud06c\uc758 \uc2dc\ub97c \uc77d\uc5b4\uc8fc\uc5c8\ub2e4. \uadf8\uc758 \uc544\ubc84\uc9c0\ub294 \ub808\ub2cc\uadf8\ub77c\ub4dc\uc5d0\uc11c \uacf5\ud559 \uad50\uc218\ub85c \uc77c\ud588\ub2e4. \ubcf4\uc988\ub124\uc13c\uc2a4\ud0a4\ub294 \uc804\uc7c1 \uc911 \uc804\uc120\uc73c\ub85c \ub3cc\uc544\uac00\ub358 \uc544\ubc84\uc9c0\uac00 \ucfe0\ub974\uac04\uc5d0 \ub4e4\ub800\ub358 \ub0a0\uc744 \ud68c\uc0c1\ud55c\ub2e4. \uadf8\uc758 \uc544\ubc84\uc9c0\ub294 \uba74\ub3c4\ub97c \ud558\uc9c0 \uc54a\uc544 \ucd08\ucdcc\ud55c \ubaa8\uc2b5\uc73c\ub85c \uc57d\uac04\uc758 \uc2dd\ub7c9\uc774 \ub4e4\uc5b4 \uc788\ub294 \ubc30\ub0ad\uacfc \uace0\uc57c\uc758 \uc791\ud488\uc9d1\uc744 \uac00\uc838\uc654\ub2e4. \uace0\uc57c\uc758 \uadf8\ub9bc\uc740 \ud654\uac00\uac00 \ub418\uace0\uc790 \ud588\ub358 \uafc8 \ub9ce\uc740 \uc5b4\ub9b0 \uc18c\ub144\uc5d0\uac8c \uae4a\uc740 \uac10\uba85\uc744 \uc8fc\uc5c8\ub2e4. \ubcf4\uc988\ub124\uc13c\uc2a4\ud0a4\ub294 \uace0\uc57c\uc758 \uadf8\ub85c\ud14c\uc2a4\ud06c\ud558\uace0 \ubb34\uc2dc\ubb34\uc2dc\ud55c \uc804\uc7c1 \uadf8\ub9bc\uc744 \ud1b5\ud574\uc11c \uc804\uc7c1\uc758 \ucc38\uc0c1\uc744 \uc774\ud574\ud588\ub2e4. \ubc14\ub85c \uadf8\uc758 \uc720\uba85\ud55c \uc2dc \uff1c\ub098\ub294 \uace0\uc57c\uff1e(1957)\uac00 \uc804\uc7c1\uc5d0 \ub300\ud55c \uc2dc\uc778\uc758 \uc774\ud574\ub97c \ubc18\uc601\ud55c \uc791\ud488\uc774\ub2e4."} {"question": "\uc624\uc2a4\ud2b8\ub808\uc77c\ub9ac\uc544\uccad\uac1c\uad6c\ub9ac\uac00 \ube44\ub9cc\uc774 \ub418\uba74 \uba38\ub9ac\uc640 \ubab8\ud1b5\uc5d0 \ubb34\uc5c7\uc774 \ub204\uc801\ub418\ub294\uac00?", "paragraph": "\uc624\uc2a4\ud2b8\ub808\uc77c\ub9ac\uc544\uccad\uac1c\uad6c\ub9ac\ub294 \uc138\uacc4\uc801\uc73c\ub85c \uac00\uc7a5 \uc778\uae30\uc788\ub294 \uc560\uc644 \uac1c\uad6c\ub9ac \uc911 \ud558\ub098\uc774\ub2e4. \uc131\uaca9\uc774 \uc720\uc21c\ud558\uace0 \uc218\uba85\uc774 \uae38\ub2e4\ub294 \uc810\uc740 \uc560\uc644\ub3d9\ubb3c\ub85c\uc11c \ub9e4\uc6b0 \uc801\uc808\ud558\uace0 \uc720\ub9ac\ud55c \ud2b9\uc9d5\uc774\ub2e4. \ub610\ud55c \ud0a4\uc6b0\uae30\ub3c4 \uc27d\ub2e4. \uba39\uc744 \uc218 \uc788\ub294 \uba39\uc774\uc758 \ubc94\uc704\ub3c4 \ub2e4\uc591\ud558\uace0 \uc9c8\ubcd1\uc5d0\ub3c4 \uac15\ud558\ub2e4. \ud55c \uac00\uc9c0 \ubb38\uc81c\ub294 \uacfc\uc2dd\uc73c\ub85c, \uc624\uc2a4\ud2b8\ub808\uc77c\ub9ac\uc544\uccad\uac1c\uad6c\ub9ac\ub294 \uacfc\uc2dd\ud558\uba74 \ube44\ub9cc\ud558\uae30 \uc27d\ub2e4. \uc57c\uc0dd\uc5d0\uc11c\ub294 \uba39\uc774\ub97c \ucc3e\uae30 \uc704\ud574 \uc774\ub9ac\uc800\ub9ac \ub6f0\uc5b4\ub2e4\ub2c8\uba70 \uc5d0\ub108\uc9c0\ub97c \uc18c\ubaa8\ud558\uc9c0\ub9cc, \ud3ec\ud68d \uc0c1\ud0dc\uc5d0\uc11c\ub294 \uc881\uc740 \uacf5\uac04\uc5d0\uc11c \uba39\uc774\uac00 \uc8fc\uc5b4\uc9c0\uae30\uc5d0 \uba39\uc774\ub97c \uba39\uae30 \uc704\ud574 \ud544\uc694\ud55c \ud65c\ub3d9\ub7c9\uc774 \uc904\uc5b4\ub4e4\uace0 \ubab8\ubb34\uac8c\uac00 \ubd88\uc5b4\ub098\uac8c \ub41c\ub2e4. \ube44\ub9cc\uc774 \ub41c \uc624\uc2a4\ud2b8\ub808\uc77c\ub9ac\uc544\uccad\uac1c\uad6c\ub9ac\ub294 \uba38\ub9ac\uc640 \ubab8\ud1b5\uc5d0 \uc9c0\ubc29\uce35\uc774 \ub204\uc801\ub418\uc5b4 \ub545\ub538\ub9c9\ud558\uace0 \"\uc2dc\ubb34\ub8e9\ud55c(dumpy)\" \ud45c\uc815\uc744 \uac16\uac8c \ub41c\ub2e4. \ub54c\ubb38\uc5d0 \"\uc2dc\ubb34\ub8e9\uccad\uac1c\uad6c\ub9ac(dumpy tree frog)\"\ub77c\ub294 \ubcc4\uba85\ub3c4 \ubd99\uc5c8\ub2e4.", "answer": "\uc9c0\ubc29\uce35", "sentence": "\ube44\ub9cc\uc774 \ub41c \uc624\uc2a4\ud2b8\ub808\uc77c\ub9ac\uc544\uccad\uac1c\uad6c\ub9ac\ub294 \uba38\ub9ac\uc640 \ubab8\ud1b5\uc5d0 \uc9c0\ubc29\uce35 \uc774 \ub204\uc801\ub418\uc5b4 \ub545\ub538\ub9c9\ud558\uace0 \"\uc2dc\ubb34\ub8e9\ud55c(dumpy)\" \ud45c\uc815\uc744 \uac16\uac8c \ub41c\ub2e4.", "paragraph_sentence": "\uc624\uc2a4\ud2b8\ub808\uc77c\ub9ac\uc544\uccad\uac1c\uad6c\ub9ac\ub294 \uc138\uacc4\uc801\uc73c\ub85c \uac00\uc7a5 \uc778\uae30\uc788\ub294 \uc560\uc644 \uac1c\uad6c\ub9ac \uc911 \ud558\ub098\uc774\ub2e4. \uc131\uaca9\uc774 \uc720\uc21c\ud558\uace0 \uc218\uba85\uc774 \uae38\ub2e4\ub294 \uc810\uc740 \uc560\uc644\ub3d9\ubb3c\ub85c\uc11c \ub9e4\uc6b0 \uc801\uc808\ud558\uace0 \uc720\ub9ac\ud55c \ud2b9\uc9d5\uc774\ub2e4. \ub610\ud55c \ud0a4\uc6b0\uae30\ub3c4 \uc27d\ub2e4. \uba39\uc744 \uc218 \uc788\ub294 \uba39\uc774\uc758 \ubc94\uc704\ub3c4 \ub2e4\uc591\ud558\uace0 \uc9c8\ubcd1\uc5d0\ub3c4 \uac15\ud558\ub2e4. \ud55c \uac00\uc9c0 \ubb38\uc81c\ub294 \uacfc\uc2dd\uc73c\ub85c, \uc624\uc2a4\ud2b8\ub808\uc77c\ub9ac\uc544\uccad\uac1c\uad6c\ub9ac\ub294 \uacfc\uc2dd\ud558\uba74 \ube44\ub9cc\ud558\uae30 \uc27d\ub2e4. \uc57c\uc0dd\uc5d0\uc11c\ub294 \uba39\uc774\ub97c \ucc3e\uae30 \uc704\ud574 \uc774\ub9ac\uc800\ub9ac \ub6f0\uc5b4\ub2e4\ub2c8\uba70 \uc5d0\ub108\uc9c0\ub97c \uc18c\ubaa8\ud558\uc9c0\ub9cc, \ud3ec\ud68d \uc0c1\ud0dc\uc5d0\uc11c\ub294 \uc881\uc740 \uacf5\uac04\uc5d0\uc11c \uba39\uc774\uac00 \uc8fc\uc5b4\uc9c0\uae30\uc5d0 \uba39\uc774\ub97c \uba39\uae30 \uc704\ud574 \ud544\uc694\ud55c \ud65c\ub3d9\ub7c9\uc774 \uc904\uc5b4\ub4e4\uace0 \ubab8\ubb34\uac8c\uac00 \ubd88\uc5b4\ub098\uac8c \ub41c\ub2e4. \ube44\ub9cc\uc774 \ub41c \uc624\uc2a4\ud2b8\ub808\uc77c\ub9ac\uc544\uccad\uac1c\uad6c\ub9ac\ub294 \uba38\ub9ac\uc640 \ubab8\ud1b5\uc5d0 \uc9c0\ubc29\uce35 \uc774 \ub204\uc801\ub418\uc5b4 \ub545\ub538\ub9c9\ud558\uace0 \"\uc2dc\ubb34\ub8e9\ud55c(dumpy)\" \ud45c\uc815\uc744 \uac16\uac8c \ub41c\ub2e4. \ub54c\ubb38\uc5d0 \"\uc2dc\ubb34\ub8e9\uccad\uac1c\uad6c\ub9ac(dumpy tree frog)\"\ub77c\ub294 \ubcc4\uba85\ub3c4 \ubd99\uc5c8\ub2e4.", "paragraph_answer": "\uc624\uc2a4\ud2b8\ub808\uc77c\ub9ac\uc544\uccad\uac1c\uad6c\ub9ac\ub294 \uc138\uacc4\uc801\uc73c\ub85c \uac00\uc7a5 \uc778\uae30\uc788\ub294 \uc560\uc644 \uac1c\uad6c\ub9ac \uc911 \ud558\ub098\uc774\ub2e4. \uc131\uaca9\uc774 \uc720\uc21c\ud558\uace0 \uc218\uba85\uc774 \uae38\ub2e4\ub294 \uc810\uc740 \uc560\uc644\ub3d9\ubb3c\ub85c\uc11c \ub9e4\uc6b0 \uc801\uc808\ud558\uace0 \uc720\ub9ac\ud55c \ud2b9\uc9d5\uc774\ub2e4. \ub610\ud55c \ud0a4\uc6b0\uae30\ub3c4 \uc27d\ub2e4. \uba39\uc744 \uc218 \uc788\ub294 \uba39\uc774\uc758 \ubc94\uc704\ub3c4 \ub2e4\uc591\ud558\uace0 \uc9c8\ubcd1\uc5d0\ub3c4 \uac15\ud558\ub2e4. \ud55c \uac00\uc9c0 \ubb38\uc81c\ub294 \uacfc\uc2dd\uc73c\ub85c, \uc624\uc2a4\ud2b8\ub808\uc77c\ub9ac\uc544\uccad\uac1c\uad6c\ub9ac\ub294 \uacfc\uc2dd\ud558\uba74 \ube44\ub9cc\ud558\uae30 \uc27d\ub2e4. \uc57c\uc0dd\uc5d0\uc11c\ub294 \uba39\uc774\ub97c \ucc3e\uae30 \uc704\ud574 \uc774\ub9ac\uc800\ub9ac \ub6f0\uc5b4\ub2e4\ub2c8\uba70 \uc5d0\ub108\uc9c0\ub97c \uc18c\ubaa8\ud558\uc9c0\ub9cc, \ud3ec\ud68d \uc0c1\ud0dc\uc5d0\uc11c\ub294 \uc881\uc740 \uacf5\uac04\uc5d0\uc11c \uba39\uc774\uac00 \uc8fc\uc5b4\uc9c0\uae30\uc5d0 \uba39\uc774\ub97c \uba39\uae30 \uc704\ud574 \ud544\uc694\ud55c \ud65c\ub3d9\ub7c9\uc774 \uc904\uc5b4\ub4e4\uace0 \ubab8\ubb34\uac8c\uac00 \ubd88\uc5b4\ub098\uac8c \ub41c\ub2e4. \ube44\ub9cc\uc774 \ub41c \uc624\uc2a4\ud2b8\ub808\uc77c\ub9ac\uc544\uccad\uac1c\uad6c\ub9ac\ub294 \uba38\ub9ac\uc640 \ubab8\ud1b5\uc5d0 \uc9c0\ubc29\uce35 \uc774 \ub204\uc801\ub418\uc5b4 \ub545\ub538\ub9c9\ud558\uace0 \"\uc2dc\ubb34\ub8e9\ud55c(dumpy)\" \ud45c\uc815\uc744 \uac16\uac8c \ub41c\ub2e4. \ub54c\ubb38\uc5d0 \"\uc2dc\ubb34\ub8e9\uccad\uac1c\uad6c\ub9ac(dumpy tree frog)\"\ub77c\ub294 \ubcc4\uba85\ub3c4 \ubd99\uc5c8\ub2e4.", "sentence_answer": "\ube44\ub9cc\uc774 \ub41c \uc624\uc2a4\ud2b8\ub808\uc77c\ub9ac\uc544\uccad\uac1c\uad6c\ub9ac\ub294 \uba38\ub9ac\uc640 \ubab8\ud1b5\uc5d0 \uc9c0\ubc29\uce35 \uc774 \ub204\uc801\ub418\uc5b4 \ub545\ub538\ub9c9\ud558\uace0 \"\uc2dc\ubb34\ub8e9\ud55c(dumpy)\" \ud45c\uc815\uc744 \uac16\uac8c \ub41c\ub2e4.", "paragraph_id": "6587749-4-0", "paragraph_question": "question: \uc624\uc2a4\ud2b8\ub808\uc77c\ub9ac\uc544\uccad\uac1c\uad6c\ub9ac\uac00 \ube44\ub9cc\uc774 \ub418\uba74 \uba38\ub9ac\uc640 \ubab8\ud1b5\uc5d0 \ubb34\uc5c7\uc774 \ub204\uc801\ub418\ub294\uac00?, context: \uc624\uc2a4\ud2b8\ub808\uc77c\ub9ac\uc544\uccad\uac1c\uad6c\ub9ac\ub294 \uc138\uacc4\uc801\uc73c\ub85c \uac00\uc7a5 \uc778\uae30\uc788\ub294 \uc560\uc644 \uac1c\uad6c\ub9ac \uc911 \ud558\ub098\uc774\ub2e4. \uc131\uaca9\uc774 \uc720\uc21c\ud558\uace0 \uc218\uba85\uc774 \uae38\ub2e4\ub294 \uc810\uc740 \uc560\uc644\ub3d9\ubb3c\ub85c\uc11c \ub9e4\uc6b0 \uc801\uc808\ud558\uace0 \uc720\ub9ac\ud55c \ud2b9\uc9d5\uc774\ub2e4. \ub610\ud55c \ud0a4\uc6b0\uae30\ub3c4 \uc27d\ub2e4. \uba39\uc744 \uc218 \uc788\ub294 \uba39\uc774\uc758 \ubc94\uc704\ub3c4 \ub2e4\uc591\ud558\uace0 \uc9c8\ubcd1\uc5d0\ub3c4 \uac15\ud558\ub2e4. \ud55c \uac00\uc9c0 \ubb38\uc81c\ub294 \uacfc\uc2dd\uc73c\ub85c, \uc624\uc2a4\ud2b8\ub808\uc77c\ub9ac\uc544\uccad\uac1c\uad6c\ub9ac\ub294 \uacfc\uc2dd\ud558\uba74 \ube44\ub9cc\ud558\uae30 \uc27d\ub2e4. \uc57c\uc0dd\uc5d0\uc11c\ub294 \uba39\uc774\ub97c \ucc3e\uae30 \uc704\ud574 \uc774\ub9ac\uc800\ub9ac \ub6f0\uc5b4\ub2e4\ub2c8\uba70 \uc5d0\ub108\uc9c0\ub97c \uc18c\ubaa8\ud558\uc9c0\ub9cc, \ud3ec\ud68d \uc0c1\ud0dc\uc5d0\uc11c\ub294 \uc881\uc740 \uacf5\uac04\uc5d0\uc11c \uba39\uc774\uac00 \uc8fc\uc5b4\uc9c0\uae30\uc5d0 \uba39\uc774\ub97c \uba39\uae30 \uc704\ud574 \ud544\uc694\ud55c \ud65c\ub3d9\ub7c9\uc774 \uc904\uc5b4\ub4e4\uace0 \ubab8\ubb34\uac8c\uac00 \ubd88\uc5b4\ub098\uac8c \ub41c\ub2e4. \ube44\ub9cc\uc774 \ub41c \uc624\uc2a4\ud2b8\ub808\uc77c\ub9ac\uc544\uccad\uac1c\uad6c\ub9ac\ub294 \uba38\ub9ac\uc640 \ubab8\ud1b5\uc5d0 \uc9c0\ubc29\uce35\uc774 \ub204\uc801\ub418\uc5b4 \ub545\ub538\ub9c9\ud558\uace0 \"\uc2dc\ubb34\ub8e9\ud55c(dumpy)\" \ud45c\uc815\uc744 \uac16\uac8c \ub41c\ub2e4. \ub54c\ubb38\uc5d0 \"\uc2dc\ubb34\ub8e9\uccad\uac1c\uad6c\ub9ac(dumpy tree frog)\"\ub77c\ub294 \ubcc4\uba85\ub3c4 \ubd99\uc5c8\ub2e4."} {"question": "<\uc5b4\uc81c \ub0b4\ub9b0 \ube44>\uc5d0\uc11c \uc0c9\uc18c\ud3f0\uc744 \uc5f0\uc8fc\ud55c \uc0ac\ub78c\uc740 \ub204\uad6c\uc785\ub2c8\uae4c?", "paragraph": "\uc0c8\ub85c\uc6b4 \uc77c\ub809\ud2b8\ub9ad \ud604\uc545 4\uc911\uc8fc \u3008\ub0b4 \ub098\ub77c \uc81c\uc77c\ub85c \uc88b\uc544\u3009\uc5d0\uc11c\ub294 \uc911\uac04\uc5d0 \uc7a5\uc0c8\ub0a9\uc758 \ub3c5\uc8fc\uac00 \uc774\uc5b4\uc84c\uc73c\uba70, \ubaa8\ub780\ubd09\uc545\ub2e8\uc758 \uc120\uc6b0\ud5a5\ud76c \ub610\ub294 \ucc28\uc601\ubbf8\uac00 \uc5f0\uc8fc\ud558\uc600\ub358 \uae30\uc874 \u3008\ub0b4 \ub098\ub77c \uc81c\uc77c\ub85c \uc88b\uc544\u3009\uac00 \uc544\ub2cc \ub610 \ub2e4\ub978 \uc5f0\uc8fc\uc790\ub4e4\uc758 \uc77c\ub809\ud2b8\ub9ad \uc2a4\ud2b8\ub9c1\uacfc \uc804\ud1b5\uc545\uae30 \uc7a5\uc0c8\ub0a9\uc758 \uc5f0\uc8fc\ub294 \ub098\ub984 \uc0c8\ub85c\uc6b4 \uc870\ud654\ub97c \ubcf4\uc5ec\uc8fc\uc5c8\ub2e4. \u3008\ub2ec\ub824\uac00\uc790 \ubbf8\ub798\ub85c\u3009\uc5d0\uc11c \ub9ac\uc218\uacbd\u00b7\uae40\uccad\u00b7\uae40\uc8fc\ud5a5\u00b7\uc1a1\uc601\u00b7\ub85c\uacbd\ubbf8\ub294 \uc9e7\uc740 \ud56b\ud32c\uce20\ub97c \uc785\uace0 \ub178\ub798\ud558\uba74\uc11c \uc0c1\ub2f9\ud788 \uaca9\ub82c\ud55c \ucda4\uc744 \ucd94\uc5c8\ub294\ub370, \uc9e7\uc740 \ud56b\ud32c\uce20\ub294 \uae30\uc874\uc758 \ud3c9\uc591 \uacf5\uc5f0\uc5d0\uc11c \uacb0\ucf54 \uc0c1\uc0c1\ud560 \uc218 \uc5c6\uc5c8\ub358 \ubaa8\uc2b5\uc774\uc5c8\ub2e4. \ub610\ud55c \uae40\uc8fc\ud5a5\uc758 \uac00\uc2b4 \uae4a\uac8c \ud328\uc778 \ub4dc\ub808\uc2a4\ub3c4 \ud3c9\uc591 \uacf5\uc5f0\uc5d0\uc11c\ub294 \ubcfc \uc218 \uc5c6\ub358 \uc7a5\uba74\uc774\uc5c8\ub2e4. \uc57d 20\uc5ec \ubd84\uc5d0 \uac78\uccd0 \ud074\ub798\uc2dd\u00b7\ubbfc\uc694 \uba85\uace1 \ubaa8\uc74c \uad00\ud604\uc545 \u3008\uce5c\uadfc\ud55c \uc120\uc728\u3009\uc744 \uc5f0\uc8fc\ud558\uc600\ub294\ub370 \uc0bc\uc9c0\uc5f0\uc545\ub2e8 \uc2dc\uc808\ubcf4\ub2e4 \ub300\ud3ed \ubcf4\uac15\ub41c \uad00\u00b7\ud604\uc545 \ud30c\ud2b8\ub294 \ub354\uc6b1 \uc138\ub828\ub418\uace0 \ud48d\uc131\ud55c \uc0ac\uc6b4\ub4dc\ub97c \ub4e4\ub824\uc8fc\uc5c8\ub2e4. \uc815\uc131\uc870\uc758 \u3008\uc5b4\uc81c \ub0b4\ub9b0 \ube44\u3009\ub294 \uacf5\ud6c8\uad6d\uac00\ud569\ucc3d\ub2e8\uc758 \uc120\uc6b0\uc815\ud601\uacfc \ub610 \ub2e4\ub978 \uc0c9\uc18c\ud3f0 \uc5f0\uc8fc\uc790\uc5d0 \uc758\ud574 \ucd94\uc801\ucd94\uc801 \ube44\uac00 \ub0b4\ub9ac\ub294 \ub4ef \uc7ac\ud574\uc11d\ub418\uc5c8\ub294\ub370, \uc120\uc6b0\uc815\ud601\uc740 2009\ub144 10\uc6d4 \uc0bc\uc9c0\uc5f0\uc545\ub2e8 \uacf5\uc5f0\uc758 \uc0c9\uc18c\ud3f0 \uc5f0\uc8fc\uc790\ub85c \ubaa8\uc2b5\uc744 \ubcf4\uc600\uc73c\uba70 2016\ub144 12\uc6d4 28\uc77c\uc758 \ubaa8\ub780\ubd09\uc545\ub2e8 \uacf5\uc5f0\uc5d0\uc11c\ub294 \ubaa8\ub780\ubd09\uc545\ub2e8\uacfc \uac19\uc740 \uc81c\ubcf5\uc744 \uc785\uace0 \ubaa8\ub780\ubd09\uc545\ub2e8 \uc790\ub9ac\uc5d0 \ucd9c\ud604\ud55c \ucd5c\ucd08\uc758 \ub0a8\uc131 \uc5f0\uc8fc\uc790\ub4e4 \uc911 \ud558\ub098\uc774\uc5c8\ub2e4. \uc774\ud6c4 \u3008\ucd5c\uc9c4\uc0ac\ub301 \uc14b\uc9f8 \ub538\u3009, \u3008\ud640\ub85c\uc544\ub9ac\ub791\u3009\uac19\uc740 \uce5c\uadfc\ud55c \ub0a8\uce21 \ub178\ub798 \uba54\ub4e4\ub9ac\uc640 \ud1b5\uc77c \ub178\ub798\uac00 \uc774\uc5b4\uc84c\uace0 \u3008\ub2e4\uc2dc \ub9cc\ub0a9\uc2dc\ub2e4\u3009\ub85c \uacf5\uc5f0\uc758 \ub05d\uc744 \ub9fa\uc5c8\ub2e4", "answer": "\uc120\uc6b0\uc815\ud601", "sentence": "\uc815\uc131\uc870\uc758 \u3008\uc5b4\uc81c \ub0b4\ub9b0 \ube44\u3009\ub294 \uacf5\ud6c8\uad6d\uac00\ud569\ucc3d\ub2e8\uc758 \uc120\uc6b0\uc815\ud601 \uacfc \ub610 \ub2e4\ub978 \uc0c9\uc18c\ud3f0 \uc5f0\uc8fc\uc790\uc5d0 \uc758\ud574 \ucd94\uc801\ucd94\uc801 \ube44\uac00 \ub0b4\ub9ac\ub294 \ub4ef \uc7ac\ud574\uc11d\ub418\uc5c8\ub294\ub370, \uc120\uc6b0\uc815\ud601\uc740 2009\ub144 10\uc6d4 \uc0bc\uc9c0\uc5f0\uc545\ub2e8 \uacf5\uc5f0\uc758 \uc0c9\uc18c\ud3f0 \uc5f0\uc8fc\uc790\ub85c \ubaa8\uc2b5\uc744 \ubcf4\uc600\uc73c\uba70 2016\ub144 12\uc6d4 28\uc77c\uc758 \ubaa8\ub780\ubd09\uc545\ub2e8 \uacf5\uc5f0\uc5d0\uc11c\ub294 \ubaa8\ub780\ubd09\uc545\ub2e8\uacfc \uac19\uc740 \uc81c\ubcf5\uc744 \uc785\uace0 \ubaa8\ub780\ubd09\uc545\ub2e8 \uc790\ub9ac\uc5d0 \ucd9c\ud604\ud55c \ucd5c\ucd08\uc758 \ub0a8\uc131 \uc5f0\uc8fc\uc790\ub4e4 \uc911 \ud558\ub098\uc774\uc5c8\ub2e4.", "paragraph_sentence": "\uc0c8\ub85c\uc6b4 \uc77c\ub809\ud2b8\ub9ad \ud604\uc545 4\uc911\uc8fc \u3008\ub0b4 \ub098\ub77c \uc81c\uc77c\ub85c \uc88b\uc544\u3009\uc5d0\uc11c\ub294 \uc911\uac04\uc5d0 \uc7a5\uc0c8\ub0a9\uc758 \ub3c5\uc8fc\uac00 \uc774\uc5b4\uc84c\uc73c\uba70, \ubaa8\ub780\ubd09\uc545\ub2e8\uc758 \uc120\uc6b0\ud5a5\ud76c \ub610\ub294 \ucc28\uc601\ubbf8\uac00 \uc5f0\uc8fc\ud558\uc600\ub358 \uae30\uc874 \u3008\ub0b4 \ub098\ub77c \uc81c\uc77c\ub85c \uc88b\uc544\u3009\uac00 \uc544\ub2cc \ub610 \ub2e4\ub978 \uc5f0\uc8fc\uc790\ub4e4\uc758 \uc77c\ub809\ud2b8\ub9ad \uc2a4\ud2b8\ub9c1\uacfc \uc804\ud1b5\uc545\uae30 \uc7a5\uc0c8\ub0a9\uc758 \uc5f0\uc8fc\ub294 \ub098\ub984 \uc0c8\ub85c\uc6b4 \uc870\ud654\ub97c \ubcf4\uc5ec\uc8fc\uc5c8\ub2e4. \u3008\ub2ec\ub824\uac00\uc790 \ubbf8\ub798\ub85c\u3009\uc5d0\uc11c \ub9ac\uc218\uacbd\u00b7\uae40\uccad\u00b7\uae40\uc8fc\ud5a5\u00b7\uc1a1\uc601\u00b7\ub85c\uacbd\ubbf8\ub294 \uc9e7\uc740 \ud56b\ud32c\uce20\ub97c \uc785\uace0 \ub178\ub798\ud558\uba74\uc11c \uc0c1\ub2f9\ud788 \uaca9\ub82c\ud55c \ucda4\uc744 \ucd94\uc5c8\ub294\ub370, \uc9e7\uc740 \ud56b\ud32c\uce20\ub294 \uae30\uc874\uc758 \ud3c9\uc591 \uacf5\uc5f0\uc5d0\uc11c \uacb0\ucf54 \uc0c1\uc0c1\ud560 \uc218 \uc5c6\uc5c8\ub358 \ubaa8\uc2b5\uc774\uc5c8\ub2e4. \ub610\ud55c \uae40\uc8fc\ud5a5\uc758 \uac00\uc2b4 \uae4a\uac8c \ud328\uc778 \ub4dc\ub808\uc2a4\ub3c4 \ud3c9\uc591 \uacf5\uc5f0\uc5d0\uc11c\ub294 \ubcfc \uc218 \uc5c6\ub358 \uc7a5\uba74\uc774\uc5c8\ub2e4. \uc57d 20\uc5ec \ubd84\uc5d0 \uac78\uccd0 \ud074\ub798\uc2dd\u00b7\ubbfc\uc694 \uba85\uace1 \ubaa8\uc74c \uad00\ud604\uc545 \u3008\uce5c\uadfc\ud55c \uc120\uc728\u3009\uc744 \uc5f0\uc8fc\ud558\uc600\ub294\ub370 \uc0bc\uc9c0\uc5f0\uc545\ub2e8 \uc2dc\uc808\ubcf4\ub2e4 \ub300\ud3ed \ubcf4\uac15\ub41c \uad00\u00b7\ud604\uc545 \ud30c\ud2b8\ub294 \ub354\uc6b1 \uc138\ub828\ub418\uace0 \ud48d\uc131\ud55c \uc0ac\uc6b4\ub4dc\ub97c \ub4e4\ub824\uc8fc\uc5c8\ub2e4. \uc815\uc131\uc870\uc758 \u3008\uc5b4\uc81c \ub0b4\ub9b0 \ube44\u3009\ub294 \uacf5\ud6c8\uad6d\uac00\ud569\ucc3d\ub2e8\uc758 \uc120\uc6b0\uc815\ud601 \uacfc \ub610 \ub2e4\ub978 \uc0c9\uc18c\ud3f0 \uc5f0\uc8fc\uc790\uc5d0 \uc758\ud574 \ucd94\uc801\ucd94\uc801 \ube44\uac00 \ub0b4\ub9ac\ub294 \ub4ef \uc7ac\ud574\uc11d\ub418\uc5c8\ub294\ub370, \uc120\uc6b0\uc815\ud601\uc740 2009\ub144 10\uc6d4 \uc0bc\uc9c0\uc5f0\uc545\ub2e8 \uacf5\uc5f0\uc758 \uc0c9\uc18c\ud3f0 \uc5f0\uc8fc\uc790\ub85c \ubaa8\uc2b5\uc744 \ubcf4\uc600\uc73c\uba70 2016\ub144 12\uc6d4 28\uc77c\uc758 \ubaa8\ub780\ubd09\uc545\ub2e8 \uacf5\uc5f0\uc5d0\uc11c\ub294 \ubaa8\ub780\ubd09\uc545\ub2e8\uacfc \uac19\uc740 \uc81c\ubcf5\uc744 \uc785\uace0 \ubaa8\ub780\ubd09\uc545\ub2e8 \uc790\ub9ac\uc5d0 \ucd9c\ud604\ud55c \ucd5c\ucd08\uc758 \ub0a8\uc131 \uc5f0\uc8fc\uc790\ub4e4 \uc911 \ud558\ub098\uc774\uc5c8\ub2e4. \uc774\ud6c4 \u3008\ucd5c\uc9c4\uc0ac\ub301 \uc14b\uc9f8 \ub538\u3009, \u3008\ud640\ub85c\uc544\ub9ac\ub791\u3009\uac19\uc740 \uce5c\uadfc\ud55c \ub0a8\uce21 \ub178\ub798 \uba54\ub4e4\ub9ac\uc640 \ud1b5\uc77c \ub178\ub798\uac00 \uc774\uc5b4\uc84c\uace0 \u3008\ub2e4\uc2dc \ub9cc\ub0a9\uc2dc\ub2e4\u3009\ub85c \uacf5\uc5f0\uc758 \ub05d\uc744 \ub9fa\uc5c8\ub2e4", "paragraph_answer": "\uc0c8\ub85c\uc6b4 \uc77c\ub809\ud2b8\ub9ad \ud604\uc545 4\uc911\uc8fc \u3008\ub0b4 \ub098\ub77c \uc81c\uc77c\ub85c \uc88b\uc544\u3009\uc5d0\uc11c\ub294 \uc911\uac04\uc5d0 \uc7a5\uc0c8\ub0a9\uc758 \ub3c5\uc8fc\uac00 \uc774\uc5b4\uc84c\uc73c\uba70, \ubaa8\ub780\ubd09\uc545\ub2e8\uc758 \uc120\uc6b0\ud5a5\ud76c \ub610\ub294 \ucc28\uc601\ubbf8\uac00 \uc5f0\uc8fc\ud558\uc600\ub358 \uae30\uc874 \u3008\ub0b4 \ub098\ub77c \uc81c\uc77c\ub85c \uc88b\uc544\u3009\uac00 \uc544\ub2cc \ub610 \ub2e4\ub978 \uc5f0\uc8fc\uc790\ub4e4\uc758 \uc77c\ub809\ud2b8\ub9ad \uc2a4\ud2b8\ub9c1\uacfc \uc804\ud1b5\uc545\uae30 \uc7a5\uc0c8\ub0a9\uc758 \uc5f0\uc8fc\ub294 \ub098\ub984 \uc0c8\ub85c\uc6b4 \uc870\ud654\ub97c \ubcf4\uc5ec\uc8fc\uc5c8\ub2e4. \u3008\ub2ec\ub824\uac00\uc790 \ubbf8\ub798\ub85c\u3009\uc5d0\uc11c \ub9ac\uc218\uacbd\u00b7\uae40\uccad\u00b7\uae40\uc8fc\ud5a5\u00b7\uc1a1\uc601\u00b7\ub85c\uacbd\ubbf8\ub294 \uc9e7\uc740 \ud56b\ud32c\uce20\ub97c \uc785\uace0 \ub178\ub798\ud558\uba74\uc11c \uc0c1\ub2f9\ud788 \uaca9\ub82c\ud55c \ucda4\uc744 \ucd94\uc5c8\ub294\ub370, \uc9e7\uc740 \ud56b\ud32c\uce20\ub294 \uae30\uc874\uc758 \ud3c9\uc591 \uacf5\uc5f0\uc5d0\uc11c \uacb0\ucf54 \uc0c1\uc0c1\ud560 \uc218 \uc5c6\uc5c8\ub358 \ubaa8\uc2b5\uc774\uc5c8\ub2e4. \ub610\ud55c \uae40\uc8fc\ud5a5\uc758 \uac00\uc2b4 \uae4a\uac8c \ud328\uc778 \ub4dc\ub808\uc2a4\ub3c4 \ud3c9\uc591 \uacf5\uc5f0\uc5d0\uc11c\ub294 \ubcfc \uc218 \uc5c6\ub358 \uc7a5\uba74\uc774\uc5c8\ub2e4. \uc57d 20\uc5ec \ubd84\uc5d0 \uac78\uccd0 \ud074\ub798\uc2dd\u00b7\ubbfc\uc694 \uba85\uace1 \ubaa8\uc74c \uad00\ud604\uc545 \u3008\uce5c\uadfc\ud55c \uc120\uc728\u3009\uc744 \uc5f0\uc8fc\ud558\uc600\ub294\ub370 \uc0bc\uc9c0\uc5f0\uc545\ub2e8 \uc2dc\uc808\ubcf4\ub2e4 \ub300\ud3ed \ubcf4\uac15\ub41c \uad00\u00b7\ud604\uc545 \ud30c\ud2b8\ub294 \ub354\uc6b1 \uc138\ub828\ub418\uace0 \ud48d\uc131\ud55c \uc0ac\uc6b4\ub4dc\ub97c \ub4e4\ub824\uc8fc\uc5c8\ub2e4. \uc815\uc131\uc870\uc758 \u3008\uc5b4\uc81c \ub0b4\ub9b0 \ube44\u3009\ub294 \uacf5\ud6c8\uad6d\uac00\ud569\ucc3d\ub2e8\uc758 \uc120\uc6b0\uc815\ud601 \uacfc \ub610 \ub2e4\ub978 \uc0c9\uc18c\ud3f0 \uc5f0\uc8fc\uc790\uc5d0 \uc758\ud574 \ucd94\uc801\ucd94\uc801 \ube44\uac00 \ub0b4\ub9ac\ub294 \ub4ef \uc7ac\ud574\uc11d\ub418\uc5c8\ub294\ub370, \uc120\uc6b0\uc815\ud601\uc740 2009\ub144 10\uc6d4 \uc0bc\uc9c0\uc5f0\uc545\ub2e8 \uacf5\uc5f0\uc758 \uc0c9\uc18c\ud3f0 \uc5f0\uc8fc\uc790\ub85c \ubaa8\uc2b5\uc744 \ubcf4\uc600\uc73c\uba70 2016\ub144 12\uc6d4 28\uc77c\uc758 \ubaa8\ub780\ubd09\uc545\ub2e8 \uacf5\uc5f0\uc5d0\uc11c\ub294 \ubaa8\ub780\ubd09\uc545\ub2e8\uacfc \uac19\uc740 \uc81c\ubcf5\uc744 \uc785\uace0 \ubaa8\ub780\ubd09\uc545\ub2e8 \uc790\ub9ac\uc5d0 \ucd9c\ud604\ud55c \ucd5c\ucd08\uc758 \ub0a8\uc131 \uc5f0\uc8fc\uc790\ub4e4 \uc911 \ud558\ub098\uc774\uc5c8\ub2e4. \uc774\ud6c4 \u3008\ucd5c\uc9c4\uc0ac\ub301 \uc14b\uc9f8 \ub538\u3009, \u3008\ud640\ub85c\uc544\ub9ac\ub791\u3009\uac19\uc740 \uce5c\uadfc\ud55c \ub0a8\uce21 \ub178\ub798 \uba54\ub4e4\ub9ac\uc640 \ud1b5\uc77c \ub178\ub798\uac00 \uc774\uc5b4\uc84c\uace0 \u3008\ub2e4\uc2dc \ub9cc\ub0a9\uc2dc\ub2e4\u3009\ub85c \uacf5\uc5f0\uc758 \ub05d\uc744 \ub9fa\uc5c8\ub2e4", "sentence_answer": "\uc815\uc131\uc870\uc758 \u3008\uc5b4\uc81c \ub0b4\ub9b0 \ube44\u3009\ub294 \uacf5\ud6c8\uad6d\uac00\ud569\ucc3d\ub2e8\uc758 \uc120\uc6b0\uc815\ud601 \uacfc \ub610 \ub2e4\ub978 \uc0c9\uc18c\ud3f0 \uc5f0\uc8fc\uc790\uc5d0 \uc758\ud574 \ucd94\uc801\ucd94\uc801 \ube44\uac00 \ub0b4\ub9ac\ub294 \ub4ef \uc7ac\ud574\uc11d\ub418\uc5c8\ub294\ub370, \uc120\uc6b0\uc815\ud601\uc740 2009\ub144 10\uc6d4 \uc0bc\uc9c0\uc5f0\uc545\ub2e8 \uacf5\uc5f0\uc758 \uc0c9\uc18c\ud3f0 \uc5f0\uc8fc\uc790\ub85c \ubaa8\uc2b5\uc744 \ubcf4\uc600\uc73c\uba70 2016\ub144 12\uc6d4 28\uc77c\uc758 \ubaa8\ub780\ubd09\uc545\ub2e8 \uacf5\uc5f0\uc5d0\uc11c\ub294 \ubaa8\ub780\ubd09\uc545\ub2e8\uacfc \uac19\uc740 \uc81c\ubcf5\uc744 \uc785\uace0 \ubaa8\ub780\ubd09\uc545\ub2e8 \uc790\ub9ac\uc5d0 \ucd9c\ud604\ud55c \ucd5c\ucd08\uc758 \ub0a8\uc131 \uc5f0\uc8fc\uc790\ub4e4 \uc911 \ud558\ub098\uc774\uc5c8\ub2e4.", "paragraph_id": "6136088-2-1", "paragraph_question": "question: <\uc5b4\uc81c \ub0b4\ub9b0 \ube44>\uc5d0\uc11c \uc0c9\uc18c\ud3f0\uc744 \uc5f0\uc8fc\ud55c \uc0ac\ub78c\uc740 \ub204\uad6c\uc785\ub2c8\uae4c?, context: \uc0c8\ub85c\uc6b4 \uc77c\ub809\ud2b8\ub9ad \ud604\uc545 4\uc911\uc8fc \u3008\ub0b4 \ub098\ub77c \uc81c\uc77c\ub85c \uc88b\uc544\u3009\uc5d0\uc11c\ub294 \uc911\uac04\uc5d0 \uc7a5\uc0c8\ub0a9\uc758 \ub3c5\uc8fc\uac00 \uc774\uc5b4\uc84c\uc73c\uba70, \ubaa8\ub780\ubd09\uc545\ub2e8\uc758 \uc120\uc6b0\ud5a5\ud76c \ub610\ub294 \ucc28\uc601\ubbf8\uac00 \uc5f0\uc8fc\ud558\uc600\ub358 \uae30\uc874 \u3008\ub0b4 \ub098\ub77c \uc81c\uc77c\ub85c \uc88b\uc544\u3009\uac00 \uc544\ub2cc \ub610 \ub2e4\ub978 \uc5f0\uc8fc\uc790\ub4e4\uc758 \uc77c\ub809\ud2b8\ub9ad \uc2a4\ud2b8\ub9c1\uacfc \uc804\ud1b5\uc545\uae30 \uc7a5\uc0c8\ub0a9\uc758 \uc5f0\uc8fc\ub294 \ub098\ub984 \uc0c8\ub85c\uc6b4 \uc870\ud654\ub97c \ubcf4\uc5ec\uc8fc\uc5c8\ub2e4. \u3008\ub2ec\ub824\uac00\uc790 \ubbf8\ub798\ub85c\u3009\uc5d0\uc11c \ub9ac\uc218\uacbd\u00b7\uae40\uccad\u00b7\uae40\uc8fc\ud5a5\u00b7\uc1a1\uc601\u00b7\ub85c\uacbd\ubbf8\ub294 \uc9e7\uc740 \ud56b\ud32c\uce20\ub97c \uc785\uace0 \ub178\ub798\ud558\uba74\uc11c \uc0c1\ub2f9\ud788 \uaca9\ub82c\ud55c \ucda4\uc744 \ucd94\uc5c8\ub294\ub370, \uc9e7\uc740 \ud56b\ud32c\uce20\ub294 \uae30\uc874\uc758 \ud3c9\uc591 \uacf5\uc5f0\uc5d0\uc11c \uacb0\ucf54 \uc0c1\uc0c1\ud560 \uc218 \uc5c6\uc5c8\ub358 \ubaa8\uc2b5\uc774\uc5c8\ub2e4. \ub610\ud55c \uae40\uc8fc\ud5a5\uc758 \uac00\uc2b4 \uae4a\uac8c \ud328\uc778 \ub4dc\ub808\uc2a4\ub3c4 \ud3c9\uc591 \uacf5\uc5f0\uc5d0\uc11c\ub294 \ubcfc \uc218 \uc5c6\ub358 \uc7a5\uba74\uc774\uc5c8\ub2e4. \uc57d 20\uc5ec \ubd84\uc5d0 \uac78\uccd0 \ud074\ub798\uc2dd\u00b7\ubbfc\uc694 \uba85\uace1 \ubaa8\uc74c \uad00\ud604\uc545 \u3008\uce5c\uadfc\ud55c \uc120\uc728\u3009\uc744 \uc5f0\uc8fc\ud558\uc600\ub294\ub370 \uc0bc\uc9c0\uc5f0\uc545\ub2e8 \uc2dc\uc808\ubcf4\ub2e4 \ub300\ud3ed \ubcf4\uac15\ub41c \uad00\u00b7\ud604\uc545 \ud30c\ud2b8\ub294 \ub354\uc6b1 \uc138\ub828\ub418\uace0 \ud48d\uc131\ud55c \uc0ac\uc6b4\ub4dc\ub97c \ub4e4\ub824\uc8fc\uc5c8\ub2e4. \uc815\uc131\uc870\uc758 \u3008\uc5b4\uc81c \ub0b4\ub9b0 \ube44\u3009\ub294 \uacf5\ud6c8\uad6d\uac00\ud569\ucc3d\ub2e8\uc758 \uc120\uc6b0\uc815\ud601\uacfc \ub610 \ub2e4\ub978 \uc0c9\uc18c\ud3f0 \uc5f0\uc8fc\uc790\uc5d0 \uc758\ud574 \ucd94\uc801\ucd94\uc801 \ube44\uac00 \ub0b4\ub9ac\ub294 \ub4ef \uc7ac\ud574\uc11d\ub418\uc5c8\ub294\ub370, \uc120\uc6b0\uc815\ud601\uc740 2009\ub144 10\uc6d4 \uc0bc\uc9c0\uc5f0\uc545\ub2e8 \uacf5\uc5f0\uc758 \uc0c9\uc18c\ud3f0 \uc5f0\uc8fc\uc790\ub85c \ubaa8\uc2b5\uc744 \ubcf4\uc600\uc73c\uba70 2016\ub144 12\uc6d4 28\uc77c\uc758 \ubaa8\ub780\ubd09\uc545\ub2e8 \uacf5\uc5f0\uc5d0\uc11c\ub294 \ubaa8\ub780\ubd09\uc545\ub2e8\uacfc \uac19\uc740 \uc81c\ubcf5\uc744 \uc785\uace0 \ubaa8\ub780\ubd09\uc545\ub2e8 \uc790\ub9ac\uc5d0 \ucd9c\ud604\ud55c \ucd5c\ucd08\uc758 \ub0a8\uc131 \uc5f0\uc8fc\uc790\ub4e4 \uc911 \ud558\ub098\uc774\uc5c8\ub2e4. \uc774\ud6c4 \u3008\ucd5c\uc9c4\uc0ac\ub301 \uc14b\uc9f8 \ub538\u3009, \u3008\ud640\ub85c\uc544\ub9ac\ub791\u3009\uac19\uc740 \uce5c\uadfc\ud55c \ub0a8\uce21 \ub178\ub798 \uba54\ub4e4\ub9ac\uc640 \ud1b5\uc77c \ub178\ub798\uac00 \uc774\uc5b4\uc84c\uace0 \u3008\ub2e4\uc2dc \ub9cc\ub0a9\uc2dc\ub2e4\u3009\ub85c \uacf5\uc5f0\uc758 \ub05d\uc744 \ub9fa\uc5c8\ub2e4"} {"question": "\ucd5c\uadfc \uc5f0\uad6c\uc5d0\uc11c\ub294 \uac00\uac00\uc758 \uc601\ud5a5\uc73c\ub85c \uc601\uad6d\uc758 \ub2c8\ucee4\uc2a4 \uc18c\ube44\uac00 \uba87 % \uc99d\uac00\ud558\uc600\ub2e4\uace0 \ubc1c\ud45c\ub418\uc5c8\ub294\uac00?", "paragraph": "\uadf8\ub140\uc758 \uc774\ubbf8\uc9c0\uc640 \ud328\uc158 \uac10\uac01\uc740 \ud328\ub9ac\uc2a4 \ud790\ud2bc\uacfc \ub2c8\ucf5c \ub9ac\uce58\uc640 \uac19\uc740 \uba85\uc0ac\ub4e4\uc5d0\uac8c \ub9ce\uc740 \uc601\ud5a5\uc744 \uc8fc\uc5c8\ub2e4. \ucd5c\uadfc\uc758 \uc5f0\uad6c\uc5d0\uc11c\ub294 \uac00\uac00\uc758 \uc601\ud5a5\uc73c\ub85c \uc601\uad6d\uc758 \ub2c8\ucee4\uc2a4 \uc18c\ube44\uac00 25% \uc99d\uac00\ud558\uc600\ub2e4\ub294 \uacb0\uacfc\uac00 \ubc1c\ud45c\ub418\uc5c8\ub2e4. \ub780\uc81c\ub9ac \uc804\ubb38\uac00 \uc544\ub124\ud2b8 \uc6cc\ubc84\ud2bc\uc5d0 \ub530\ub974\uba74, 16\uc138 \uc774\ud558\uc758 \uc5ec\uc131 \uccad\uc18c\ub144\ub4e4\uc774 \uac80\uc740 \uc18d\uc637\uc744 \uad6c\uc785\ud558\uc5ec \ud56b \ud32c\uce20\ucc98\ub7fc \uc785\ub294 \uac83\uc740 \uac00\uac00\uc758 \uc2a4\ud0c0\uc77c\uc744 \ubcf8\ub72c \uac83\uc774\ub77c\uace0 \ud55c\ub2e4. \ud55c\ud3b8, 2010\ub144 9\uc6d4 \uc5f4\ub9b0 2010 MTV \ube44\ub514\uc624 \ubba4\uc9c1 \uc5b4\uc6cc\ub4dc\uc5d0\uc11c \uc0dd\uace0\uae30 \ub4dc\ub808\uc2a4\ub97c \uc785\uace0 \ub098\uc640 \uc138\uacc4\uc801\uc73c\ub85c \ud070 \ud654\uc81c\uac00 \ub418\uc5c8\ub294\ub370, \uc774 \uc0dd\uace0\uae30 \ub4dc\ub808\uc2a4\ub294 \ub274\uc695\ud0c0\uc784\uc2a4\uac00 \uc120\uc815\ud55c 2010\ub144 \uc62c\ud574\uc758 \uc544\uc774\ub514\uc5b4\ub85c \uc120\uc815\ub418\uc5c8\uace0, \ubbf8\uad6d \ub85c\ud070\ub864 \uba85\uc608\uc758 \uc804\ub2f9\uc5d0 \ubc15\uc81c\ub97c \uac70\uccd0 \uac00\uac00\uc758 \uc5b4\ub9b0 \uc2dc\uc808 \ud53c\uc544\ub178\uc640 \ud568\uaed8 \"Women Who Rock: Vision, Passion, Power\"\ub77c\ub294 \uc774\ub984\uc73c\ub85c \ubcf4\uc874\ud558\uace0\uc788\ub2e4.", "answer": "25%", "sentence": "\uc601\uad6d\uc758 \ub2c8\ucee4\uc2a4 \uc18c\ube44\uac00 25% \uc99d\uac00\ud558\uc600\ub2e4\ub294 \uacb0\uacfc\uac00 \ubc1c\ud45c\ub418\uc5c8\ub2e4.", "paragraph_sentence": "\uadf8\ub140\uc758 \uc774\ubbf8\uc9c0\uc640 \ud328\uc158 \uac10\uac01\uc740 \ud328\ub9ac\uc2a4 \ud790\ud2bc\uacfc \ub2c8\ucf5c \ub9ac\uce58\uc640 \uac19\uc740 \uba85\uc0ac\ub4e4\uc5d0\uac8c \ub9ce\uc740 \uc601\ud5a5\uc744 \uc8fc\uc5c8\ub2e4. \ucd5c\uadfc\uc758 \uc5f0\uad6c\uc5d0\uc11c\ub294 \uac00\uac00\uc758 \uc601\ud5a5\uc73c\ub85c \uc601\uad6d\uc758 \ub2c8\ucee4\uc2a4 \uc18c\ube44\uac00 25% \uc99d\uac00\ud558\uc600\ub2e4\ub294 \uacb0\uacfc\uac00 \ubc1c\ud45c\ub418\uc5c8\ub2e4. \ub780\uc81c\ub9ac \uc804\ubb38\uac00 \uc544\ub124\ud2b8 \uc6cc\ubc84\ud2bc\uc5d0 \ub530\ub974\uba74, 16\uc138 \uc774\ud558\uc758 \uc5ec\uc131 \uccad\uc18c\ub144\ub4e4\uc774 \uac80\uc740 \uc18d\uc637\uc744 \uad6c\uc785\ud558\uc5ec \ud56b \ud32c\uce20\ucc98\ub7fc \uc785\ub294 \uac83\uc740 \uac00\uac00\uc758 \uc2a4\ud0c0\uc77c\uc744 \ubcf8\ub72c \uac83\uc774\ub77c\uace0 \ud55c\ub2e4. \ud55c\ud3b8, 2010\ub144 9\uc6d4 \uc5f4\ub9b0 2010 MTV \ube44\ub514\uc624 \ubba4\uc9c1 \uc5b4\uc6cc\ub4dc\uc5d0\uc11c \uc0dd\uace0\uae30 \ub4dc\ub808\uc2a4\ub97c \uc785\uace0 \ub098\uc640 \uc138\uacc4\uc801\uc73c\ub85c \ud070 \ud654\uc81c\uac00 \ub418\uc5c8\ub294\ub370, \uc774 \uc0dd\uace0\uae30 \ub4dc\ub808\uc2a4\ub294 \ub274\uc695\ud0c0\uc784\uc2a4\uac00 \uc120\uc815\ud55c 2010\ub144 \uc62c\ud574\uc758 \uc544\uc774\ub514\uc5b4\ub85c \uc120\uc815\ub418\uc5c8\uace0, \ubbf8\uad6d \ub85c\ud070\ub864 \uba85\uc608\uc758 \uc804\ub2f9\uc5d0 \ubc15\uc81c\ub97c \uac70\uccd0 \uac00\uac00\uc758 \uc5b4\ub9b0 \uc2dc\uc808 \ud53c\uc544\ub178\uc640 \ud568\uaed8 \"Women Who Rock: Vision, Passion, Power\"\ub77c\ub294 \uc774\ub984\uc73c\ub85c \ubcf4\uc874\ud558\uace0\uc788\ub2e4.", "paragraph_answer": "\uadf8\ub140\uc758 \uc774\ubbf8\uc9c0\uc640 \ud328\uc158 \uac10\uac01\uc740 \ud328\ub9ac\uc2a4 \ud790\ud2bc\uacfc \ub2c8\ucf5c \ub9ac\uce58\uc640 \uac19\uc740 \uba85\uc0ac\ub4e4\uc5d0\uac8c \ub9ce\uc740 \uc601\ud5a5\uc744 \uc8fc\uc5c8\ub2e4. \ucd5c\uadfc\uc758 \uc5f0\uad6c\uc5d0\uc11c\ub294 \uac00\uac00\uc758 \uc601\ud5a5\uc73c\ub85c \uc601\uad6d\uc758 \ub2c8\ucee4\uc2a4 \uc18c\ube44\uac00 25% \uc99d\uac00\ud558\uc600\ub2e4\ub294 \uacb0\uacfc\uac00 \ubc1c\ud45c\ub418\uc5c8\ub2e4. \ub780\uc81c\ub9ac \uc804\ubb38\uac00 \uc544\ub124\ud2b8 \uc6cc\ubc84\ud2bc\uc5d0 \ub530\ub974\uba74, 16\uc138 \uc774\ud558\uc758 \uc5ec\uc131 \uccad\uc18c\ub144\ub4e4\uc774 \uac80\uc740 \uc18d\uc637\uc744 \uad6c\uc785\ud558\uc5ec \ud56b \ud32c\uce20\ucc98\ub7fc \uc785\ub294 \uac83\uc740 \uac00\uac00\uc758 \uc2a4\ud0c0\uc77c\uc744 \ubcf8\ub72c \uac83\uc774\ub77c\uace0 \ud55c\ub2e4. \ud55c\ud3b8, 2010\ub144 9\uc6d4 \uc5f4\ub9b0 2010 MTV \ube44\ub514\uc624 \ubba4\uc9c1 \uc5b4\uc6cc\ub4dc\uc5d0\uc11c \uc0dd\uace0\uae30 \ub4dc\ub808\uc2a4\ub97c \uc785\uace0 \ub098\uc640 \uc138\uacc4\uc801\uc73c\ub85c \ud070 \ud654\uc81c\uac00 \ub418\uc5c8\ub294\ub370, \uc774 \uc0dd\uace0\uae30 \ub4dc\ub808\uc2a4\ub294 \ub274\uc695\ud0c0\uc784\uc2a4\uac00 \uc120\uc815\ud55c 2010\ub144 \uc62c\ud574\uc758 \uc544\uc774\ub514\uc5b4\ub85c \uc120\uc815\ub418\uc5c8\uace0, \ubbf8\uad6d \ub85c\ud070\ub864 \uba85\uc608\uc758 \uc804\ub2f9\uc5d0 \ubc15\uc81c\ub97c \uac70\uccd0 \uac00\uac00\uc758 \uc5b4\ub9b0 \uc2dc\uc808 \ud53c\uc544\ub178\uc640 \ud568\uaed8 \"Women Who Rock: Vision, Passion, Power\"\ub77c\ub294 \uc774\ub984\uc73c\ub85c \ubcf4\uc874\ud558\uace0\uc788\ub2e4.", "sentence_answer": "\uc601\uad6d\uc758 \ub2c8\ucee4\uc2a4 \uc18c\ube44\uac00 25% \uc99d\uac00\ud558\uc600\ub2e4\ub294 \uacb0\uacfc\uac00 \ubc1c\ud45c\ub418\uc5c8\ub2e4.", "paragraph_id": "6513681-26-0", "paragraph_question": "question: \ucd5c\uadfc \uc5f0\uad6c\uc5d0\uc11c\ub294 \uac00\uac00\uc758 \uc601\ud5a5\uc73c\ub85c \uc601\uad6d\uc758 \ub2c8\ucee4\uc2a4 \uc18c\ube44\uac00 \uba87 % \uc99d\uac00\ud558\uc600\ub2e4\uace0 \ubc1c\ud45c\ub418\uc5c8\ub294\uac00?, context: \uadf8\ub140\uc758 \uc774\ubbf8\uc9c0\uc640 \ud328\uc158 \uac10\uac01\uc740 \ud328\ub9ac\uc2a4 \ud790\ud2bc\uacfc \ub2c8\ucf5c \ub9ac\uce58\uc640 \uac19\uc740 \uba85\uc0ac\ub4e4\uc5d0\uac8c \ub9ce\uc740 \uc601\ud5a5\uc744 \uc8fc\uc5c8\ub2e4. \ucd5c\uadfc\uc758 \uc5f0\uad6c\uc5d0\uc11c\ub294 \uac00\uac00\uc758 \uc601\ud5a5\uc73c\ub85c \uc601\uad6d\uc758 \ub2c8\ucee4\uc2a4 \uc18c\ube44\uac00 25% \uc99d\uac00\ud558\uc600\ub2e4\ub294 \uacb0\uacfc\uac00 \ubc1c\ud45c\ub418\uc5c8\ub2e4. \ub780\uc81c\ub9ac \uc804\ubb38\uac00 \uc544\ub124\ud2b8 \uc6cc\ubc84\ud2bc\uc5d0 \ub530\ub974\uba74, 16\uc138 \uc774\ud558\uc758 \uc5ec\uc131 \uccad\uc18c\ub144\ub4e4\uc774 \uac80\uc740 \uc18d\uc637\uc744 \uad6c\uc785\ud558\uc5ec \ud56b \ud32c\uce20\ucc98\ub7fc \uc785\ub294 \uac83\uc740 \uac00\uac00\uc758 \uc2a4\ud0c0\uc77c\uc744 \ubcf8\ub72c \uac83\uc774\ub77c\uace0 \ud55c\ub2e4. \ud55c\ud3b8, 2010\ub144 9\uc6d4 \uc5f4\ub9b0 2010 MTV \ube44\ub514\uc624 \ubba4\uc9c1 \uc5b4\uc6cc\ub4dc\uc5d0\uc11c \uc0dd\uace0\uae30 \ub4dc\ub808\uc2a4\ub97c \uc785\uace0 \ub098\uc640 \uc138\uacc4\uc801\uc73c\ub85c \ud070 \ud654\uc81c\uac00 \ub418\uc5c8\ub294\ub370, \uc774 \uc0dd\uace0\uae30 \ub4dc\ub808\uc2a4\ub294 \ub274\uc695\ud0c0\uc784\uc2a4\uac00 \uc120\uc815\ud55c 2010\ub144 \uc62c\ud574\uc758 \uc544\uc774\ub514\uc5b4\ub85c \uc120\uc815\ub418\uc5c8\uace0, \ubbf8\uad6d \ub85c\ud070\ub864 \uba85\uc608\uc758 \uc804\ub2f9\uc5d0 \ubc15\uc81c\ub97c \uac70\uccd0 \uac00\uac00\uc758 \uc5b4\ub9b0 \uc2dc\uc808 \ud53c\uc544\ub178\uc640 \ud568\uaed8 \"Women Who Rock: Vision, Passion, Power\"\ub77c\ub294 \uc774\ub984\uc73c\ub85c \ubcf4\uc874\ud558\uace0\uc788\ub2e4."} {"question": "\uc815\ub3d9\uc815\uc774 \uc2e0\uc124\ub41c \uc11c\ub300\ubb38\uad6c\uc5d0 \uc18d\ud588\ub358 \uc2dc\uae30\ub294?", "paragraph": "\uc774\uc21c\uc6b0\uc758 \uc5f0\uad6c\uc5d0 \ub530\ub974\uba74, \ub300\uc815\ub3d9\uacfc \uc18c\uc815\ub3d9\uc758 \uac1c\ub150\uacfc \ubc94\uc704\uac00 \uc644\uc804\ud788 \ubb34\ub825\ud654\ud55c \ub54c\ub294 \uc77c\uc81c\uac15\uc810\uae30 1914\ub144\uc774\ub2e4. \uc774 \ub2f9\uc2dc \uc870\uc120\ucd1d\ub3c5\ubd80\uac00 \uc804\uad6d\uc801\uc778 \ud589\uc815\uad6c\uc5ed\uac1c\ud3b8\uc744 \ub2e8\ud589\ud558\uba74\uc11c \uac1c\ubcc4 \ub2e8\uc704\uc758 \ub3d9\ub124\ub4e4\uc744 \uc790\uc2e0\ub4e4\uc758 \ud3b8\uc758\ub300\ub85c \uad6c\uc5ed\uc744 \uc7ac\ud3b8\ud558\uac70\ub098 \uba85\uce6d\uc744 \ubcc0\uacbd\ud558\uc600\ub2e4. \u300e\uc870\uc120\ucd1d\ub3c5\ubd80 \uad00\ubcf4\u300f1914\ub144 4\uc6d4 27\uc77c\uc790\uc5d0 \uac8c\uc81c\ub41c '\uc870\uc120\ucd1d\ub3c5\ubd80 \uacbd\uae30\ub3c4\uace0\uc2dc \uc81c7\ud638 \uacbd\uc131\ubd80 \uc815\ub3d9\uc758 \uba85\uce6d \ubc0f \uad6c\uc5ed'(\uace0\uc2dc\uc77c\uc790 1914\ub144 4\uc6d4 1\uc77c)\uc5d0\uc11c \uadf8 \uacb0\uacfc\ub97c \ud655\uc778\ud560 \uc218 \uc788\ub2e4. 1914\ub144 \ud589\uc815\uad6c\uc5ed \uac1c\ud3b8 \ub54c \uc0c8\ub85c\uc6b4 \uc815\ub3d9(\u8c9e\u6d1e)\uc774 \ub300\uc815\ub3d9\uacfc \uc18c\uc815\ub3d9\uc744 \ud558\ub098\ub85c \ubb36\uae30\ub294 \ud558\uc600\uc73c\ub098 \ub300\ubd80\ubd84\uc758 \uc794\uc5ec\uc9c0\uc5ed\uc774 \uc778\uc811\ud558\ub294 \ud0dc\ud3c9\ud1b5(\uc9c0\uae08\uc758 \ud0dc\ud3c9\ub85c), \uc11c\uc18c\ubb38\uc815(\uc9c0\uae08\uc758 \uc11c\uc18c\ubb38\ub3d9), \ubb34\uad50\uc815(\uc9c0\uae08\uc758 \ubb34\uad50\ub3d9) \ub4f1\uc9c0\ub85c \ud769\uc5b4\uc838 \ud3b8\uc785\ub418\uc5c8\ub2e4. \ud1a0\uc9c0\uc870\uc0ac\uc0ac\uc5c5\uacb0\uacfc \uc0c8\ub85c\uc6b4 \uc9c0\ubc88\uc774 \ubd80\uc5ec\ub41c \uc774\ub798\ub85c \ud589\uc815\uad6c\uc5ed\uc0c1 \uc815\ub3d9\uc5d0 \uc874\uc18d\ud588\ub358 \uc9c0\ubc88\uc740 '\uc815\ub3d9 1\ubc88\uc9c0'\ubd80\ud130 '\uc815\ub3d9 34\ubc88\uc9c0'\uc5d0 \uc774\ub974\ub294 \uad6c\uc131\uc774\uc5c8\ub2e4. 1936\ub144 \ud589\uc815\uad6c\uc5ed \uac1c\ud3b8 \ub54c \uc815\ub3d9\uc815(\u8c9e\u6d1e\u753a)\uc774 \ub418\uc5c8\uace0, \uc815\ub3d9\uc815\uc740 1943\ub144 6\uc6d4 \uc2e0\uc124\ub41c \uc11c\ub300\ubb38\uad6c\uc5d0 \uc18d\ud558\uc600\uace0, 1946\ub144 \uc815\ub3d9\uc73c\ub85c \uc774\ub984\uc774 \ubc14\ub00c\uc5b4 \uc9c0\uae08\uc5d0 \uc774\ub974\uba70, 1975\ub144\uc5d0\ub294 \uc911\uad6c \uad00\ud560\uc774 \ub418\uc5c8\ub2e4.", "answer": "1943\ub144 6\uc6d4", "sentence": "\uc815\ub3d9\uc815\uc740 1943\ub144 6\uc6d4 \uc2e0\uc124\ub41c \uc11c\ub300\ubb38\uad6c\uc5d0 \uc18d\ud558\uc600\uace0, 1946\ub144 \uc815\ub3d9\uc73c\ub85c \uc774\ub984\uc774 \ubc14\ub00c\uc5b4 \uc9c0\uae08\uc5d0 \uc774\ub974\uba70, 1975\ub144\uc5d0\ub294 \uc911\uad6c \uad00\ud560\uc774 \ub418\uc5c8\ub2e4.", "paragraph_sentence": "\uc774\uc21c\uc6b0\uc758 \uc5f0\uad6c\uc5d0 \ub530\ub974\uba74, \ub300\uc815\ub3d9\uacfc \uc18c\uc815\ub3d9\uc758 \uac1c\ub150\uacfc \ubc94\uc704\uac00 \uc644\uc804\ud788 \ubb34\ub825\ud654\ud55c \ub54c\ub294 \uc77c\uc81c\uac15\uc810\uae30 1914\ub144\uc774\ub2e4. \uc774 \ub2f9\uc2dc \uc870\uc120\ucd1d\ub3c5\ubd80\uac00 \uc804\uad6d\uc801\uc778 \ud589\uc815\uad6c\uc5ed\uac1c\ud3b8\uc744 \ub2e8\ud589\ud558\uba74\uc11c \uac1c\ubcc4 \ub2e8\uc704\uc758 \ub3d9\ub124\ub4e4\uc744 \uc790\uc2e0\ub4e4\uc758 \ud3b8\uc758\ub300\ub85c \uad6c\uc5ed\uc744 \uc7ac\ud3b8\ud558\uac70\ub098 \uba85\uce6d\uc744 \ubcc0\uacbd\ud558\uc600\ub2e4. \u300e\uc870\uc120\ucd1d\ub3c5\ubd80 \uad00\ubcf4\u300f1914\ub144 4\uc6d4 27\uc77c\uc790\uc5d0 \uac8c\uc81c\ub41c '\uc870\uc120\ucd1d\ub3c5\ubd80 \uacbd\uae30\ub3c4\uace0\uc2dc \uc81c7\ud638 \uacbd\uc131\ubd80 \uc815\ub3d9\uc758 \uba85\uce6d \ubc0f \uad6c\uc5ed'(\uace0\uc2dc\uc77c\uc790 1914\ub144 4\uc6d4 1\uc77c)\uc5d0\uc11c \uadf8 \uacb0\uacfc\ub97c \ud655\uc778\ud560 \uc218 \uc788\ub2e4. 1914\ub144 \ud589\uc815\uad6c\uc5ed \uac1c\ud3b8 \ub54c \uc0c8\ub85c\uc6b4 \uc815\ub3d9(\u8c9e\u6d1e)\uc774 \ub300\uc815\ub3d9\uacfc \uc18c\uc815\ub3d9\uc744 \ud558\ub098\ub85c \ubb36\uae30\ub294 \ud558\uc600\uc73c\ub098 \ub300\ubd80\ubd84\uc758 \uc794\uc5ec\uc9c0\uc5ed\uc774 \uc778\uc811\ud558\ub294 \ud0dc\ud3c9\ud1b5(\uc9c0\uae08\uc758 \ud0dc\ud3c9\ub85c), \uc11c\uc18c\ubb38\uc815(\uc9c0\uae08\uc758 \uc11c\uc18c\ubb38\ub3d9), \ubb34\uad50\uc815(\uc9c0\uae08\uc758 \ubb34\uad50\ub3d9) \ub4f1\uc9c0\ub85c \ud769\uc5b4\uc838 \ud3b8\uc785\ub418\uc5c8\ub2e4. \ud1a0\uc9c0\uc870\uc0ac\uc0ac\uc5c5\uacb0\uacfc \uc0c8\ub85c\uc6b4 \uc9c0\ubc88\uc774 \ubd80\uc5ec\ub41c \uc774\ub798\ub85c \ud589\uc815\uad6c\uc5ed\uc0c1 \uc815\ub3d9\uc5d0 \uc874\uc18d\ud588\ub358 \uc9c0\ubc88\uc740 '\uc815\ub3d9 1\ubc88\uc9c0'\ubd80\ud130 '\uc815\ub3d9 34\ubc88\uc9c0'\uc5d0 \uc774\ub974\ub294 \uad6c\uc131\uc774\uc5c8\ub2e4. 1936\ub144 \ud589\uc815\uad6c\uc5ed \uac1c\ud3b8 \ub54c \uc815\ub3d9\uc815(\u8c9e\u6d1e\u753a)\uc774 \ub418\uc5c8\uace0, \uc815\ub3d9\uc815\uc740 1943\ub144 6\uc6d4 \uc2e0\uc124\ub41c \uc11c\ub300\ubb38\uad6c\uc5d0 \uc18d\ud558\uc600\uace0, 1946\ub144 \uc815\ub3d9\uc73c\ub85c \uc774\ub984\uc774 \ubc14\ub00c\uc5b4 \uc9c0\uae08\uc5d0 \uc774\ub974\uba70, 1975\ub144\uc5d0\ub294 \uc911\uad6c \uad00\ud560\uc774 \ub418\uc5c8\ub2e4. ", "paragraph_answer": "\uc774\uc21c\uc6b0\uc758 \uc5f0\uad6c\uc5d0 \ub530\ub974\uba74, \ub300\uc815\ub3d9\uacfc \uc18c\uc815\ub3d9\uc758 \uac1c\ub150\uacfc \ubc94\uc704\uac00 \uc644\uc804\ud788 \ubb34\ub825\ud654\ud55c \ub54c\ub294 \uc77c\uc81c\uac15\uc810\uae30 1914\ub144\uc774\ub2e4. \uc774 \ub2f9\uc2dc \uc870\uc120\ucd1d\ub3c5\ubd80\uac00 \uc804\uad6d\uc801\uc778 \ud589\uc815\uad6c\uc5ed\uac1c\ud3b8\uc744 \ub2e8\ud589\ud558\uba74\uc11c \uac1c\ubcc4 \ub2e8\uc704\uc758 \ub3d9\ub124\ub4e4\uc744 \uc790\uc2e0\ub4e4\uc758 \ud3b8\uc758\ub300\ub85c \uad6c\uc5ed\uc744 \uc7ac\ud3b8\ud558\uac70\ub098 \uba85\uce6d\uc744 \ubcc0\uacbd\ud558\uc600\ub2e4. \u300e\uc870\uc120\ucd1d\ub3c5\ubd80 \uad00\ubcf4\u300f1914\ub144 4\uc6d4 27\uc77c\uc790\uc5d0 \uac8c\uc81c\ub41c '\uc870\uc120\ucd1d\ub3c5\ubd80 \uacbd\uae30\ub3c4\uace0\uc2dc \uc81c7\ud638 \uacbd\uc131\ubd80 \uc815\ub3d9\uc758 \uba85\uce6d \ubc0f \uad6c\uc5ed'(\uace0\uc2dc\uc77c\uc790 1914\ub144 4\uc6d4 1\uc77c)\uc5d0\uc11c \uadf8 \uacb0\uacfc\ub97c \ud655\uc778\ud560 \uc218 \uc788\ub2e4. 1914\ub144 \ud589\uc815\uad6c\uc5ed \uac1c\ud3b8 \ub54c \uc0c8\ub85c\uc6b4 \uc815\ub3d9(\u8c9e\u6d1e)\uc774 \ub300\uc815\ub3d9\uacfc \uc18c\uc815\ub3d9\uc744 \ud558\ub098\ub85c \ubb36\uae30\ub294 \ud558\uc600\uc73c\ub098 \ub300\ubd80\ubd84\uc758 \uc794\uc5ec\uc9c0\uc5ed\uc774 \uc778\uc811\ud558\ub294 \ud0dc\ud3c9\ud1b5(\uc9c0\uae08\uc758 \ud0dc\ud3c9\ub85c), \uc11c\uc18c\ubb38\uc815(\uc9c0\uae08\uc758 \uc11c\uc18c\ubb38\ub3d9), \ubb34\uad50\uc815(\uc9c0\uae08\uc758 \ubb34\uad50\ub3d9) \ub4f1\uc9c0\ub85c \ud769\uc5b4\uc838 \ud3b8\uc785\ub418\uc5c8\ub2e4. \ud1a0\uc9c0\uc870\uc0ac\uc0ac\uc5c5\uacb0\uacfc \uc0c8\ub85c\uc6b4 \uc9c0\ubc88\uc774 \ubd80\uc5ec\ub41c \uc774\ub798\ub85c \ud589\uc815\uad6c\uc5ed\uc0c1 \uc815\ub3d9\uc5d0 \uc874\uc18d\ud588\ub358 \uc9c0\ubc88\uc740 '\uc815\ub3d9 1\ubc88\uc9c0'\ubd80\ud130 '\uc815\ub3d9 34\ubc88\uc9c0'\uc5d0 \uc774\ub974\ub294 \uad6c\uc131\uc774\uc5c8\ub2e4. 1936\ub144 \ud589\uc815\uad6c\uc5ed \uac1c\ud3b8 \ub54c \uc815\ub3d9\uc815(\u8c9e\u6d1e\u753a)\uc774 \ub418\uc5c8\uace0, \uc815\ub3d9\uc815\uc740 1943\ub144 6\uc6d4 \uc2e0\uc124\ub41c \uc11c\ub300\ubb38\uad6c\uc5d0 \uc18d\ud558\uc600\uace0, 1946\ub144 \uc815\ub3d9\uc73c\ub85c \uc774\ub984\uc774 \ubc14\ub00c\uc5b4 \uc9c0\uae08\uc5d0 \uc774\ub974\uba70, 1975\ub144\uc5d0\ub294 \uc911\uad6c \uad00\ud560\uc774 \ub418\uc5c8\ub2e4.", "sentence_answer": "\uc815\ub3d9\uc815\uc740 1943\ub144 6\uc6d4 \uc2e0\uc124\ub41c \uc11c\ub300\ubb38\uad6c\uc5d0 \uc18d\ud558\uc600\uace0, 1946\ub144 \uc815\ub3d9\uc73c\ub85c \uc774\ub984\uc774 \ubc14\ub00c\uc5b4 \uc9c0\uae08\uc5d0 \uc774\ub974\uba70, 1975\ub144\uc5d0\ub294 \uc911\uad6c \uad00\ud560\uc774 \ub418\uc5c8\ub2e4.", "paragraph_id": "6550027-1-1", "paragraph_question": "question: \uc815\ub3d9\uc815\uc774 \uc2e0\uc124\ub41c \uc11c\ub300\ubb38\uad6c\uc5d0 \uc18d\ud588\ub358 \uc2dc\uae30\ub294?, context: \uc774\uc21c\uc6b0\uc758 \uc5f0\uad6c\uc5d0 \ub530\ub974\uba74, \ub300\uc815\ub3d9\uacfc \uc18c\uc815\ub3d9\uc758 \uac1c\ub150\uacfc \ubc94\uc704\uac00 \uc644\uc804\ud788 \ubb34\ub825\ud654\ud55c \ub54c\ub294 \uc77c\uc81c\uac15\uc810\uae30 1914\ub144\uc774\ub2e4. \uc774 \ub2f9\uc2dc \uc870\uc120\ucd1d\ub3c5\ubd80\uac00 \uc804\uad6d\uc801\uc778 \ud589\uc815\uad6c\uc5ed\uac1c\ud3b8\uc744 \ub2e8\ud589\ud558\uba74\uc11c \uac1c\ubcc4 \ub2e8\uc704\uc758 \ub3d9\ub124\ub4e4\uc744 \uc790\uc2e0\ub4e4\uc758 \ud3b8\uc758\ub300\ub85c \uad6c\uc5ed\uc744 \uc7ac\ud3b8\ud558\uac70\ub098 \uba85\uce6d\uc744 \ubcc0\uacbd\ud558\uc600\ub2e4. \u300e\uc870\uc120\ucd1d\ub3c5\ubd80 \uad00\ubcf4\u300f1914\ub144 4\uc6d4 27\uc77c\uc790\uc5d0 \uac8c\uc81c\ub41c '\uc870\uc120\ucd1d\ub3c5\ubd80 \uacbd\uae30\ub3c4\uace0\uc2dc \uc81c7\ud638 \uacbd\uc131\ubd80 \uc815\ub3d9\uc758 \uba85\uce6d \ubc0f \uad6c\uc5ed'(\uace0\uc2dc\uc77c\uc790 1914\ub144 4\uc6d4 1\uc77c)\uc5d0\uc11c \uadf8 \uacb0\uacfc\ub97c \ud655\uc778\ud560 \uc218 \uc788\ub2e4. 1914\ub144 \ud589\uc815\uad6c\uc5ed \uac1c\ud3b8 \ub54c \uc0c8\ub85c\uc6b4 \uc815\ub3d9(\u8c9e\u6d1e)\uc774 \ub300\uc815\ub3d9\uacfc \uc18c\uc815\ub3d9\uc744 \ud558\ub098\ub85c \ubb36\uae30\ub294 \ud558\uc600\uc73c\ub098 \ub300\ubd80\ubd84\uc758 \uc794\uc5ec\uc9c0\uc5ed\uc774 \uc778\uc811\ud558\ub294 \ud0dc\ud3c9\ud1b5(\uc9c0\uae08\uc758 \ud0dc\ud3c9\ub85c), \uc11c\uc18c\ubb38\uc815(\uc9c0\uae08\uc758 \uc11c\uc18c\ubb38\ub3d9), \ubb34\uad50\uc815(\uc9c0\uae08\uc758 \ubb34\uad50\ub3d9) \ub4f1\uc9c0\ub85c \ud769\uc5b4\uc838 \ud3b8\uc785\ub418\uc5c8\ub2e4. \ud1a0\uc9c0\uc870\uc0ac\uc0ac\uc5c5\uacb0\uacfc \uc0c8\ub85c\uc6b4 \uc9c0\ubc88\uc774 \ubd80\uc5ec\ub41c \uc774\ub798\ub85c \ud589\uc815\uad6c\uc5ed\uc0c1 \uc815\ub3d9\uc5d0 \uc874\uc18d\ud588\ub358 \uc9c0\ubc88\uc740 '\uc815\ub3d9 1\ubc88\uc9c0'\ubd80\ud130 '\uc815\ub3d9 34\ubc88\uc9c0'\uc5d0 \uc774\ub974\ub294 \uad6c\uc131\uc774\uc5c8\ub2e4. 1936\ub144 \ud589\uc815\uad6c\uc5ed \uac1c\ud3b8 \ub54c \uc815\ub3d9\uc815(\u8c9e\u6d1e\u753a)\uc774 \ub418\uc5c8\uace0, \uc815\ub3d9\uc815\uc740 1943\ub144 6\uc6d4 \uc2e0\uc124\ub41c \uc11c\ub300\ubb38\uad6c\uc5d0 \uc18d\ud558\uc600\uace0, 1946\ub144 \uc815\ub3d9\uc73c\ub85c \uc774\ub984\uc774 \ubc14\ub00c\uc5b4 \uc9c0\uae08\uc5d0 \uc774\ub974\uba70, 1975\ub144\uc5d0\ub294 \uc911\uad6c \uad00\ud560\uc774 \ub418\uc5c8\ub2e4."} {"question": "\uae38\ubc84\ud2b8\uac00 \uba55\uc2dc\ucf54 \ud0c0\ub9c8\uc6b8\ub9ac\ud30c\uc2a4 \uc8fc\uc5d0 \uc0c1\ub959\ud55c \ub54c\ub294?", "paragraph": "\ud5c8\ub9ac\ucf00\uc778 \uae38\ubc84\ud2b8\ub294 9\uc6d4 14\uc77c\uc5d0 \uc0ac\ud53c\uc5b4-\uc2ec\uc2a8 \ud5c8\ub9ac\ucf00\uc778 \ub4f1\uae09 \uc81c 5\ub4f1\uae09\uc758 \uc138\ub825\uc73c\ub85c \ucf54\uc8fc\ubc00 \uc12c\uc5d0 \ub450 \ubc88\uc9f8\ub85c, \uadf8\ub9ac\uace0 \uba55\uc2dc\ucf54 \uc720\uce74\ud0c4 \ubc18\ub3c4\uc5d0 \uc138 \ubc88\uc9f8\ub85c \uc0c1\ub959\ud588\ub2e4. \uc81c5\ub4f1\uae09\uc758 \uc138\ub825\uc73c\ub85c \uc0c1\ub959\ud55c \ud5c8\ub9ac\ucf00\uc778\uc740 1979\ub144\uc758 \ud5c8\ub9ac\ucf00\uc778 \ub370\uc774\ube44\ub4dc\uac00 \ud788\uc2a4\ud30c\ub2c8\uc62c\ub77c \uc12c\uc5d0 \uc0c1\ub959\ud55c \uc774\ud6c4 \ucc98\uc74c \uc77c\uc5b4\ub0ac\ub358 \uc77c\uc774\uc5c8\ub2e4. \ub610\ud55c \uc0c1\ub959\ud560 \ub2f9\uc2dc \ucf54\uc8fc\ubc00 \uc12c\uc758 \uc911\uc2ec\uae30\uc555\uc740 900hPa\uc73c\ub85c \uc608\uc0c1\ub418\uc5c8\ub2e4. \uc790\uc5f0\ud788 \ub0b4\ub959\uc73c\ub85c \ub4e4\uc5b4\uc624\uba74\uc11c \ud5c8\ub9ac\ucf00\uc778\uc740 \ube60\ub974\uac8c \uc57d\ud574\uc84c\uace0 \ub2e4\uc2dc \uba55\uc2dc\ucf54 \ub9cc\uc5d0 \uc81c2\ub4f1\uae09\uc758 \uc138\ub825\uc73c\ub85c \uc9c4\ucd9c\ud558\uae30 \uc804\uae4c\uc9c0 \uacc4\uc18d \uc57d\ud654\ub418\uc5c8\ub2e4. \uadf8\ub7ec\ub098 \ub2e4\uc2dc \ubc14\ub2e4\ub85c \ub098\uc628 \uae38\ubc84\ud2b8\ub294 \ub9e4\uc6b0 \ube60\ub974\uac8c \uac15\ud574\uc84c\uc73c\ub098 9\uc6d4 16\uc77c \uba55\uc2dc\ucf54 \ud0c0\ub9c8\uc6b8\ub9ac\ud30c\uc2a4 \uc8fc\uc5d0 \ub124 \ubc88\uc9f8\ub85c \uc0c1\ub959\ud558\uba74\uc11c \uae38\ubc84\ud2b8\uac00 \uac15\ud574\uc9c8 \uc218 \uc788\ub294 \uc694\uac74\uc744 \uc644\uc804\ud788 \uc81c\uac70\ud574 \ubc84\ub9b0 \uc0c1\ud0dc\uac00 \ub418\uc5b4\ubc84\ub838\ub2e4.", "answer": "9\uc6d4 16\uc77c", "sentence": "\uadf8\ub7ec\ub098 \ub2e4\uc2dc \ubc14\ub2e4\ub85c \ub098\uc628 \uae38\ubc84\ud2b8\ub294 \ub9e4\uc6b0 \ube60\ub974\uac8c \uac15\ud574\uc84c\uc73c\ub098 9\uc6d4 16\uc77c \uba55\uc2dc\ucf54 \ud0c0\ub9c8\uc6b8\ub9ac\ud30c\uc2a4 \uc8fc\uc5d0 \ub124 \ubc88\uc9f8\ub85c \uc0c1\ub959\ud558\uba74\uc11c \uae38\ubc84\ud2b8\uac00 \uac15\ud574\uc9c8 \uc218 \uc788\ub294 \uc694\uac74\uc744 \uc644\uc804\ud788 \uc81c\uac70\ud574 \ubc84\ub9b0 \uc0c1\ud0dc\uac00 \ub418\uc5b4\ubc84\ub838\ub2e4.", "paragraph_sentence": "\ud5c8\ub9ac\ucf00\uc778 \uae38\ubc84\ud2b8\ub294 9\uc6d4 14\uc77c\uc5d0 \uc0ac\ud53c\uc5b4-\uc2ec\uc2a8 \ud5c8\ub9ac\ucf00\uc778 \ub4f1\uae09 \uc81c 5\ub4f1\uae09\uc758 \uc138\ub825\uc73c\ub85c \ucf54\uc8fc\ubc00 \uc12c\uc5d0 \ub450 \ubc88\uc9f8\ub85c, \uadf8\ub9ac\uace0 \uba55\uc2dc\ucf54 \uc720\uce74\ud0c4 \ubc18\ub3c4\uc5d0 \uc138 \ubc88\uc9f8\ub85c \uc0c1\ub959\ud588\ub2e4. \uc81c5\ub4f1\uae09\uc758 \uc138\ub825\uc73c\ub85c \uc0c1\ub959\ud55c \ud5c8\ub9ac\ucf00\uc778\uc740 1979\ub144\uc758 \ud5c8\ub9ac\ucf00\uc778 \ub370\uc774\ube44\ub4dc\uac00 \ud788\uc2a4\ud30c\ub2c8\uc62c\ub77c \uc12c\uc5d0 \uc0c1\ub959\ud55c \uc774\ud6c4 \ucc98\uc74c \uc77c\uc5b4\ub0ac\ub358 \uc77c\uc774\uc5c8\ub2e4. \ub610\ud55c \uc0c1\ub959\ud560 \ub2f9\uc2dc \ucf54\uc8fc\ubc00 \uc12c\uc758 \uc911\uc2ec\uae30\uc555\uc740 900hPa\uc73c\ub85c \uc608\uc0c1\ub418\uc5c8\ub2e4. \uc790\uc5f0\ud788 \ub0b4\ub959\uc73c\ub85c \ub4e4\uc5b4\uc624\uba74\uc11c \ud5c8\ub9ac\ucf00\uc778\uc740 \ube60\ub974\uac8c \uc57d\ud574\uc84c\uace0 \ub2e4\uc2dc \uba55\uc2dc\ucf54 \ub9cc\uc5d0 \uc81c2\ub4f1\uae09\uc758 \uc138\ub825\uc73c\ub85c \uc9c4\ucd9c\ud558\uae30 \uc804\uae4c\uc9c0 \uacc4\uc18d \uc57d\ud654\ub418\uc5c8\ub2e4. \uadf8\ub7ec\ub098 \ub2e4\uc2dc \ubc14\ub2e4\ub85c \ub098\uc628 \uae38\ubc84\ud2b8\ub294 \ub9e4\uc6b0 \ube60\ub974\uac8c \uac15\ud574\uc84c\uc73c\ub098 9\uc6d4 16\uc77c \uba55\uc2dc\ucf54 \ud0c0\ub9c8\uc6b8\ub9ac\ud30c\uc2a4 \uc8fc\uc5d0 \ub124 \ubc88\uc9f8\ub85c \uc0c1\ub959\ud558\uba74\uc11c \uae38\ubc84\ud2b8\uac00 \uac15\ud574\uc9c8 \uc218 \uc788\ub294 \uc694\uac74\uc744 \uc644\uc804\ud788 \uc81c\uac70\ud574 \ubc84\ub9b0 \uc0c1\ud0dc\uac00 \ub418\uc5b4\ubc84\ub838\ub2e4. ", "paragraph_answer": "\ud5c8\ub9ac\ucf00\uc778 \uae38\ubc84\ud2b8\ub294 9\uc6d4 14\uc77c\uc5d0 \uc0ac\ud53c\uc5b4-\uc2ec\uc2a8 \ud5c8\ub9ac\ucf00\uc778 \ub4f1\uae09 \uc81c 5\ub4f1\uae09\uc758 \uc138\ub825\uc73c\ub85c \ucf54\uc8fc\ubc00 \uc12c\uc5d0 \ub450 \ubc88\uc9f8\ub85c, \uadf8\ub9ac\uace0 \uba55\uc2dc\ucf54 \uc720\uce74\ud0c4 \ubc18\ub3c4\uc5d0 \uc138 \ubc88\uc9f8\ub85c \uc0c1\ub959\ud588\ub2e4. \uc81c5\ub4f1\uae09\uc758 \uc138\ub825\uc73c\ub85c \uc0c1\ub959\ud55c \ud5c8\ub9ac\ucf00\uc778\uc740 1979\ub144\uc758 \ud5c8\ub9ac\ucf00\uc778 \ub370\uc774\ube44\ub4dc\uac00 \ud788\uc2a4\ud30c\ub2c8\uc62c\ub77c \uc12c\uc5d0 \uc0c1\ub959\ud55c \uc774\ud6c4 \ucc98\uc74c \uc77c\uc5b4\ub0ac\ub358 \uc77c\uc774\uc5c8\ub2e4. \ub610\ud55c \uc0c1\ub959\ud560 \ub2f9\uc2dc \ucf54\uc8fc\ubc00 \uc12c\uc758 \uc911\uc2ec\uae30\uc555\uc740 900hPa\uc73c\ub85c \uc608\uc0c1\ub418\uc5c8\ub2e4. \uc790\uc5f0\ud788 \ub0b4\ub959\uc73c\ub85c \ub4e4\uc5b4\uc624\uba74\uc11c \ud5c8\ub9ac\ucf00\uc778\uc740 \ube60\ub974\uac8c \uc57d\ud574\uc84c\uace0 \ub2e4\uc2dc \uba55\uc2dc\ucf54 \ub9cc\uc5d0 \uc81c2\ub4f1\uae09\uc758 \uc138\ub825\uc73c\ub85c \uc9c4\ucd9c\ud558\uae30 \uc804\uae4c\uc9c0 \uacc4\uc18d \uc57d\ud654\ub418\uc5c8\ub2e4. \uadf8\ub7ec\ub098 \ub2e4\uc2dc \ubc14\ub2e4\ub85c \ub098\uc628 \uae38\ubc84\ud2b8\ub294 \ub9e4\uc6b0 \ube60\ub974\uac8c \uac15\ud574\uc84c\uc73c\ub098 9\uc6d4 16\uc77c \uba55\uc2dc\ucf54 \ud0c0\ub9c8\uc6b8\ub9ac\ud30c\uc2a4 \uc8fc\uc5d0 \ub124 \ubc88\uc9f8\ub85c \uc0c1\ub959\ud558\uba74\uc11c \uae38\ubc84\ud2b8\uac00 \uac15\ud574\uc9c8 \uc218 \uc788\ub294 \uc694\uac74\uc744 \uc644\uc804\ud788 \uc81c\uac70\ud574 \ubc84\ub9b0 \uc0c1\ud0dc\uac00 \ub418\uc5b4\ubc84\ub838\ub2e4.", "sentence_answer": "\uadf8\ub7ec\ub098 \ub2e4\uc2dc \ubc14\ub2e4\ub85c \ub098\uc628 \uae38\ubc84\ud2b8\ub294 \ub9e4\uc6b0 \ube60\ub974\uac8c \uac15\ud574\uc84c\uc73c\ub098 9\uc6d4 16\uc77c \uba55\uc2dc\ucf54 \ud0c0\ub9c8\uc6b8\ub9ac\ud30c\uc2a4 \uc8fc\uc5d0 \ub124 \ubc88\uc9f8\ub85c \uc0c1\ub959\ud558\uba74\uc11c \uae38\ubc84\ud2b8\uac00 \uac15\ud574\uc9c8 \uc218 \uc788\ub294 \uc694\uac74\uc744 \uc644\uc804\ud788 \uc81c\uac70\ud574 \ubc84\ub9b0 \uc0c1\ud0dc\uac00 \ub418\uc5b4\ubc84\ub838\ub2e4.", "paragraph_id": "6571114-1-1", "paragraph_question": "question: \uae38\ubc84\ud2b8\uac00 \uba55\uc2dc\ucf54 \ud0c0\ub9c8\uc6b8\ub9ac\ud30c\uc2a4 \uc8fc\uc5d0 \uc0c1\ub959\ud55c \ub54c\ub294?, context: \ud5c8\ub9ac\ucf00\uc778 \uae38\ubc84\ud2b8\ub294 9\uc6d4 14\uc77c\uc5d0 \uc0ac\ud53c\uc5b4-\uc2ec\uc2a8 \ud5c8\ub9ac\ucf00\uc778 \ub4f1\uae09 \uc81c 5\ub4f1\uae09\uc758 \uc138\ub825\uc73c\ub85c \ucf54\uc8fc\ubc00 \uc12c\uc5d0 \ub450 \ubc88\uc9f8\ub85c, \uadf8\ub9ac\uace0 \uba55\uc2dc\ucf54 \uc720\uce74\ud0c4 \ubc18\ub3c4\uc5d0 \uc138 \ubc88\uc9f8\ub85c \uc0c1\ub959\ud588\ub2e4. \uc81c5\ub4f1\uae09\uc758 \uc138\ub825\uc73c\ub85c \uc0c1\ub959\ud55c \ud5c8\ub9ac\ucf00\uc778\uc740 1979\ub144\uc758 \ud5c8\ub9ac\ucf00\uc778 \ub370\uc774\ube44\ub4dc\uac00 \ud788\uc2a4\ud30c\ub2c8\uc62c\ub77c \uc12c\uc5d0 \uc0c1\ub959\ud55c \uc774\ud6c4 \ucc98\uc74c \uc77c\uc5b4\ub0ac\ub358 \uc77c\uc774\uc5c8\ub2e4. \ub610\ud55c \uc0c1\ub959\ud560 \ub2f9\uc2dc \ucf54\uc8fc\ubc00 \uc12c\uc758 \uc911\uc2ec\uae30\uc555\uc740 900hPa\uc73c\ub85c \uc608\uc0c1\ub418\uc5c8\ub2e4. \uc790\uc5f0\ud788 \ub0b4\ub959\uc73c\ub85c \ub4e4\uc5b4\uc624\uba74\uc11c \ud5c8\ub9ac\ucf00\uc778\uc740 \ube60\ub974\uac8c \uc57d\ud574\uc84c\uace0 \ub2e4\uc2dc \uba55\uc2dc\ucf54 \ub9cc\uc5d0 \uc81c2\ub4f1\uae09\uc758 \uc138\ub825\uc73c\ub85c \uc9c4\ucd9c\ud558\uae30 \uc804\uae4c\uc9c0 \uacc4\uc18d \uc57d\ud654\ub418\uc5c8\ub2e4. \uadf8\ub7ec\ub098 \ub2e4\uc2dc \ubc14\ub2e4\ub85c \ub098\uc628 \uae38\ubc84\ud2b8\ub294 \ub9e4\uc6b0 \ube60\ub974\uac8c \uac15\ud574\uc84c\uc73c\ub098 9\uc6d4 16\uc77c \uba55\uc2dc\ucf54 \ud0c0\ub9c8\uc6b8\ub9ac\ud30c\uc2a4 \uc8fc\uc5d0 \ub124 \ubc88\uc9f8\ub85c \uc0c1\ub959\ud558\uba74\uc11c \uae38\ubc84\ud2b8\uac00 \uac15\ud574\uc9c8 \uc218 \uc788\ub294 \uc694\uac74\uc744 \uc644\uc804\ud788 \uc81c\uac70\ud574 \ubc84\ub9b0 \uc0c1\ud0dc\uac00 \ub418\uc5b4\ubc84\ub838\ub2e4."} {"question": "\uc8fc\uc138\ud398\uac00 \uc911\uad6d\uc5b4\ub97c \ud559\uc2b5\ud55c \uc9c0\uc5ed\uc740?", "paragraph": "\uadf8\ub294 \ub9c8\uce74\uc624\uc5d0\uc11c \uc911\uad6d\uc5b4\ub97c \ud559\uc2b5\ud558\uc600\uace0, \uc911\uad6d\uc5d0\uc11c \ub0ad\uc138\ub155\uc774\ub77c\ub294 \uc774\ub984\uc73c\ub85c \ud65c\ub3d9\ud558\uc600\ub2e4. \uac15\ud76c\uc81c(\uc7ac\uc704: 1661\ub144 ~ 1722\ub144), \uc639\uc815\uc81c(\uc7ac\uc704: 1722\ub144 ~ 1735\ub144), \uac74\ub96d\uc81c(\uc7ac\uc704: 1735\ub144 ~ 1796\ub144)\uc758 \uc8fc\ubaa9\uc744 \ubc1b\uc558\uace0, \ud669\uc2e4 \uc6a9\ud488 \uc81c\uc791 \uae30\uad00\uc774\uc5c8\ub358 \uc870\ud310\ucc98(\u9020\u8fa6\u8655) \ub0b4\uc758 \ud68c\ud654 \uc81c\uc791 \ubd80\uc11c\uc778 \uc5ec\uc758\uad00(\u5982\u610f\u9928)\uc5d0 \uc18c\uc18d\ub41c \uad81\uc815 \ud654\uac00\ub85c 50\uc5ec \ub144\uac04 \ud65c\uc57d\ud558\uc600\ub2e4. \uc774\ud0c8\ub9ac\uc544\uc5d0\uc11c \uae30\ucd08\ub97c \ub2e6\uc740 \uc11c\uc591\ud654 \uae30\ubc95\uc744 \ubc14\ud0d5\uc73c\ub85c \uc911\uad6d\uc5d0 \uc640\uc11c \ub2e4\uc2dc \uc911\uad6d\ud654\ub97c \ubc30\uc6e0\uace0, \ucd5c\ucd08\ub85c \ub450 \uc591\uc2dd\uc744 \ud63c\ud569\ud55c \uc0c8\ub85c\uc6b4 \ud615\ud0dc\uc758 \uadf8\ub9bc\uc744 \ub9cc\ub4e4\uc5b4\ub0c8\ub2e4. \uc911\uad6d\uc5d0 \uc120\uad50\uc0ac\ub85c \ud30c\uacac\ub418\uc5c8\uc9c0\ub9cc \uc120\uad50\uc0ac\ubcf4\ub2e4\ub294 \uad81\uc815 \ud654\uac00\ub85c\uc11c \ud65c\ub3d9\ud558\uc600\uace0, \uc639\uc815\uc81c\u00b7\uac74\ub96d\uc81c \ub54c\uc5d0 \uadf8\ub9ac\uc2a4\ub3c4\uad50\uc5d0 \ub300\ud55c \ubc15\ud574\uc5d0\ub3c4 \ubd88\uad6c\ud558\uace0 \ud669\uc81c\uc758 \ucd1d\uc560 \ub54c\ubb38\uc5d0 \ubaa8\uba74\ud560 \uc218 \uc788\uc5c8\ub2e4.", "answer": "\ub9c8\uce74\uc624", "sentence": "\uadf8\ub294 \ub9c8\uce74\uc624 \uc5d0\uc11c \uc911\uad6d\uc5b4\ub97c \ud559\uc2b5\ud558\uc600\uace0, \uc911\uad6d\uc5d0\uc11c \ub0ad\uc138\ub155\uc774\ub77c\ub294 \uc774\ub984\uc73c\ub85c \ud65c\ub3d9\ud558\uc600\ub2e4.", "paragraph_sentence": " \uadf8\ub294 \ub9c8\uce74\uc624 \uc5d0\uc11c \uc911\uad6d\uc5b4\ub97c \ud559\uc2b5\ud558\uc600\uace0, \uc911\uad6d\uc5d0\uc11c \ub0ad\uc138\ub155\uc774\ub77c\ub294 \uc774\ub984\uc73c\ub85c \ud65c\ub3d9\ud558\uc600\ub2e4. \uac15\ud76c\uc81c(\uc7ac\uc704: 1661\ub144 ~ 1722\ub144), \uc639\uc815\uc81c(\uc7ac\uc704: 1722\ub144 ~ 1735\ub144), \uac74\ub96d\uc81c(\uc7ac\uc704: 1735\ub144 ~ 1796\ub144)\uc758 \uc8fc\ubaa9\uc744 \ubc1b\uc558\uace0, \ud669\uc2e4 \uc6a9\ud488 \uc81c\uc791 \uae30\uad00\uc774\uc5c8\ub358 \uc870\ud310\ucc98(\u9020\u8fa6\u8655) \ub0b4\uc758 \ud68c\ud654 \uc81c\uc791 \ubd80\uc11c\uc778 \uc5ec\uc758\uad00(\u5982\u610f\u9928)\uc5d0 \uc18c\uc18d\ub41c \uad81\uc815 \ud654\uac00\ub85c 50\uc5ec \ub144\uac04 \ud65c\uc57d\ud558\uc600\ub2e4. \uc774\ud0c8\ub9ac\uc544\uc5d0\uc11c \uae30\ucd08\ub97c \ub2e6\uc740 \uc11c\uc591\ud654 \uae30\ubc95\uc744 \ubc14\ud0d5\uc73c\ub85c \uc911\uad6d\uc5d0 \uc640\uc11c \ub2e4\uc2dc \uc911\uad6d\ud654\ub97c \ubc30\uc6e0\uace0, \ucd5c\ucd08\ub85c \ub450 \uc591\uc2dd\uc744 \ud63c\ud569\ud55c \uc0c8\ub85c\uc6b4 \ud615\ud0dc\uc758 \uadf8\ub9bc\uc744 \ub9cc\ub4e4\uc5b4\ub0c8\ub2e4. \uc911\uad6d\uc5d0 \uc120\uad50\uc0ac\ub85c \ud30c\uacac\ub418\uc5c8\uc9c0\ub9cc \uc120\uad50\uc0ac\ubcf4\ub2e4\ub294 \uad81\uc815 \ud654\uac00\ub85c\uc11c \ud65c\ub3d9\ud558\uc600\uace0, \uc639\uc815\uc81c\u00b7\uac74\ub96d\uc81c \ub54c\uc5d0 \uadf8\ub9ac\uc2a4\ub3c4\uad50\uc5d0 \ub300\ud55c \ubc15\ud574\uc5d0\ub3c4 \ubd88\uad6c\ud558\uace0 \ud669\uc81c\uc758 \ucd1d\uc560 \ub54c\ubb38\uc5d0 \ubaa8\uba74\ud560 \uc218 \uc788\uc5c8\ub2e4.", "paragraph_answer": "\uadf8\ub294 \ub9c8\uce74\uc624 \uc5d0\uc11c \uc911\uad6d\uc5b4\ub97c \ud559\uc2b5\ud558\uc600\uace0, \uc911\uad6d\uc5d0\uc11c \ub0ad\uc138\ub155\uc774\ub77c\ub294 \uc774\ub984\uc73c\ub85c \ud65c\ub3d9\ud558\uc600\ub2e4. \uac15\ud76c\uc81c(\uc7ac\uc704: 1661\ub144 ~ 1722\ub144), \uc639\uc815\uc81c(\uc7ac\uc704: 1722\ub144 ~ 1735\ub144), \uac74\ub96d\uc81c(\uc7ac\uc704: 1735\ub144 ~ 1796\ub144)\uc758 \uc8fc\ubaa9\uc744 \ubc1b\uc558\uace0, \ud669\uc2e4 \uc6a9\ud488 \uc81c\uc791 \uae30\uad00\uc774\uc5c8\ub358 \uc870\ud310\ucc98(\u9020\u8fa6\u8655) \ub0b4\uc758 \ud68c\ud654 \uc81c\uc791 \ubd80\uc11c\uc778 \uc5ec\uc758\uad00(\u5982\u610f\u9928)\uc5d0 \uc18c\uc18d\ub41c \uad81\uc815 \ud654\uac00\ub85c 50\uc5ec \ub144\uac04 \ud65c\uc57d\ud558\uc600\ub2e4. \uc774\ud0c8\ub9ac\uc544\uc5d0\uc11c \uae30\ucd08\ub97c \ub2e6\uc740 \uc11c\uc591\ud654 \uae30\ubc95\uc744 \ubc14\ud0d5\uc73c\ub85c \uc911\uad6d\uc5d0 \uc640\uc11c \ub2e4\uc2dc \uc911\uad6d\ud654\ub97c \ubc30\uc6e0\uace0, \ucd5c\ucd08\ub85c \ub450 \uc591\uc2dd\uc744 \ud63c\ud569\ud55c \uc0c8\ub85c\uc6b4 \ud615\ud0dc\uc758 \uadf8\ub9bc\uc744 \ub9cc\ub4e4\uc5b4\ub0c8\ub2e4. \uc911\uad6d\uc5d0 \uc120\uad50\uc0ac\ub85c \ud30c\uacac\ub418\uc5c8\uc9c0\ub9cc \uc120\uad50\uc0ac\ubcf4\ub2e4\ub294 \uad81\uc815 \ud654\uac00\ub85c\uc11c \ud65c\ub3d9\ud558\uc600\uace0, \uc639\uc815\uc81c\u00b7\uac74\ub96d\uc81c \ub54c\uc5d0 \uadf8\ub9ac\uc2a4\ub3c4\uad50\uc5d0 \ub300\ud55c \ubc15\ud574\uc5d0\ub3c4 \ubd88\uad6c\ud558\uace0 \ud669\uc81c\uc758 \ucd1d\uc560 \ub54c\ubb38\uc5d0 \ubaa8\uba74\ud560 \uc218 \uc788\uc5c8\ub2e4.", "sentence_answer": "\uadf8\ub294 \ub9c8\uce74\uc624 \uc5d0\uc11c \uc911\uad6d\uc5b4\ub97c \ud559\uc2b5\ud558\uc600\uace0, \uc911\uad6d\uc5d0\uc11c \ub0ad\uc138\ub155\uc774\ub77c\ub294 \uc774\ub984\uc73c\ub85c \ud65c\ub3d9\ud558\uc600\ub2e4.", "paragraph_id": "6555956-0-1", "paragraph_question": "question: \uc8fc\uc138\ud398\uac00 \uc911\uad6d\uc5b4\ub97c \ud559\uc2b5\ud55c \uc9c0\uc5ed\uc740?, context: \uadf8\ub294 \ub9c8\uce74\uc624\uc5d0\uc11c \uc911\uad6d\uc5b4\ub97c \ud559\uc2b5\ud558\uc600\uace0, \uc911\uad6d\uc5d0\uc11c \ub0ad\uc138\ub155\uc774\ub77c\ub294 \uc774\ub984\uc73c\ub85c \ud65c\ub3d9\ud558\uc600\ub2e4. \uac15\ud76c\uc81c(\uc7ac\uc704: 1661\ub144 ~ 1722\ub144), \uc639\uc815\uc81c(\uc7ac\uc704: 1722\ub144 ~ 1735\ub144), \uac74\ub96d\uc81c(\uc7ac\uc704: 1735\ub144 ~ 1796\ub144)\uc758 \uc8fc\ubaa9\uc744 \ubc1b\uc558\uace0, \ud669\uc2e4 \uc6a9\ud488 \uc81c\uc791 \uae30\uad00\uc774\uc5c8\ub358 \uc870\ud310\ucc98(\u9020\u8fa6\u8655) \ub0b4\uc758 \ud68c\ud654 \uc81c\uc791 \ubd80\uc11c\uc778 \uc5ec\uc758\uad00(\u5982\u610f\u9928)\uc5d0 \uc18c\uc18d\ub41c \uad81\uc815 \ud654\uac00\ub85c 50\uc5ec \ub144\uac04 \ud65c\uc57d\ud558\uc600\ub2e4. \uc774\ud0c8\ub9ac\uc544\uc5d0\uc11c \uae30\ucd08\ub97c \ub2e6\uc740 \uc11c\uc591\ud654 \uae30\ubc95\uc744 \ubc14\ud0d5\uc73c\ub85c \uc911\uad6d\uc5d0 \uc640\uc11c \ub2e4\uc2dc \uc911\uad6d\ud654\ub97c \ubc30\uc6e0\uace0, \ucd5c\ucd08\ub85c \ub450 \uc591\uc2dd\uc744 \ud63c\ud569\ud55c \uc0c8\ub85c\uc6b4 \ud615\ud0dc\uc758 \uadf8\ub9bc\uc744 \ub9cc\ub4e4\uc5b4\ub0c8\ub2e4. \uc911\uad6d\uc5d0 \uc120\uad50\uc0ac\ub85c \ud30c\uacac\ub418\uc5c8\uc9c0\ub9cc \uc120\uad50\uc0ac\ubcf4\ub2e4\ub294 \uad81\uc815 \ud654\uac00\ub85c\uc11c \ud65c\ub3d9\ud558\uc600\uace0, \uc639\uc815\uc81c\u00b7\uac74\ub96d\uc81c \ub54c\uc5d0 \uadf8\ub9ac\uc2a4\ub3c4\uad50\uc5d0 \ub300\ud55c \ubc15\ud574\uc5d0\ub3c4 \ubd88\uad6c\ud558\uace0 \ud669\uc81c\uc758 \ucd1d\uc560 \ub54c\ubb38\uc5d0 \ubaa8\uba74\ud560 \uc218 \uc788\uc5c8\ub2e4."} {"question": "\uc548\uc0b0\uc2dc\uac00 \uc9c0\uae08\uc758 \uc548\uc0b0\uc2dc\ub85c \uc2b9\uaca9\ub41c \ub54c\ub294?", "paragraph": "\ud604\uc7ac \uc548\uc0b0\uc2dc\uc758 \uc77c\ubd80\ub85c \uad00\ub9ac\ub418\uace0 \uc788\ub294 \ud48d\ub3c4\uc5d0\uc11c\ub294 \uc870\uc120 \ub9d0 \uccad\uc77c \uc804\uc7c1\uc5d0\uc11c \uc77c\ubcf8\uacfc \uccad\ub098\ub77c \uac04\uc5d0 \ud48d\ub3c4\ud574\uc804\uc774 \uc788\uc5c8\ub2e4. \uc77c\uc81c \uac15\uc810\uae30\uc5d0\ub294 1914\ub144 \ud589\uc815\uad6c\uc5ed \uac1c\ud3b8\uc5d0 \ub530\ub77c \uc2dc\ud765\uad70 \uc218\uc554\uba74, \uad70\uc790\uba74\uc73c\ub85c \uc804\ud658\ub418\uc5c8\uc73c\uba70, \uc77c\ubd80 \uc9c0\uc5ed(\ubc18\uc6d4\uba74)\uc744 \uc218\uc6d0\uad70\uc5d0 \uc591\ub3c4\ud558\uc600\ub2e4. 1937\ub144\uc5d0\ub294 \uc548\uc0b0\uc744 \uad00\ud1b5\ud558\uc5ec \uc218\uc6d0\uacfc \uc778\ucc9c \uac04\uc744 \uc5f0\uacb0\ud558\ub294 \uc218\uc778\uc120 \ud611\uada4 \ucca0\ub3c4\uac00 \uc6b4\ud589\ud558\uae30 \uc2dc\uc791\ud558\uc600\ub2e4. \uadf8\ub7ec\ub098 1972\ub144 \ub300\ud55c\ubbfc\uad6d \uc815\ubd80\uc758 \ucca0\ub3c4 \ub178\uc120 \uc815\ub9ac\uc640 1988\ub144 \uc548\uc0b0\uc120 \uc804\ucca0 \uac1c\ud1b5 \uc774\ud6c4 \uc6b4\uc1a1\uc774 \uae09\uac10\ud558\ub2e4\uac00, 1995\ub144 12\uc6d4 31\uc77c\uc744 \ub05d\uc73c\ub85c \uc6b4\ud589\uc744 \uc911\ub2e8\ud558\uac8c \ub418\uc5c8\ub2e4. \ud55c\ud3b8\uc73c\ub85c 1970\ub144\ub300 \ud6c4\ubc18\ubd80\ud130 \uc548\uc0b0\uc2dc\uc758 \uacf5\uc5c5 \uc704\uc131\ub3c4\uc2dc\ud654\uac00 \uac00\uc18d\ub418\uae30 \uc2dc\uc791\ud558\uc5ec, 1976\ub144 12\uc6d4 \uc2dc\ud765\uad70\uacfc \ud654\uc131\uad70\uc758 \uc77c\ubd80\uc600\ub358 \uc548\uc0b0 \uc77c\ub300\uc5d0 \u2018\ubc18\uc6d4\uc2e0\uacf5\uc5c5\ub3c4\uc2dc \uac74\uc124\uacc4\ud68d\u2019\uc744 \ubc1c\ud45c\ud558\uc600\uc73c\uba70, 1979\ub144 8\uc6d4 31\uc77c\uc5d0 \uacbd\uae30\ub3c4 \ubc18\uc6d4\ucd9c\uc7a5\uc18c\ub97c \uc124\uce58\ud558\uc600\uace0, 1986\ub144 1\uc6d4 1\uc77c\uc5d0\ub294 \ucd9c\uc7a5\uc18c\uc5d0\uc11c \uc9c0\uae08\uc758 \uc548\uc0b0\uc2dc\ub85c \uc2b9\uaca9\ub418\uc5c8\ub2e4. 2002\ub144\uc5d0\ub294 \uc0c1\ub85d\uad6c\uc640 \ub2e8\uc6d0\uad6c \ub4f1 2\uac1c \uad6c\ub97c \uc0c8\ub85c \uc124\uce58\ud558\uba74\uc11c \uad6c\uc81c(\u5340\u5236)\ub97c \uc2e4\uc2dc\ud558\uc600\ub2e4.", "answer": "1986\ub144 1\uc6d4 1\uc77c", "sentence": "\ud55c\ud3b8\uc73c\ub85c 1970\ub144\ub300 \ud6c4\ubc18\ubd80\ud130 \uc548\uc0b0\uc2dc\uc758 \uacf5\uc5c5 \uc704\uc131\ub3c4\uc2dc\ud654\uac00 \uac00\uc18d\ub418\uae30 \uc2dc\uc791\ud558\uc5ec, 1976\ub144 12\uc6d4 \uc2dc\ud765\uad70\uacfc \ud654\uc131\uad70\uc758 \uc77c\ubd80\uc600\ub358 \uc548\uc0b0 \uc77c\ub300\uc5d0 \u2018\ubc18\uc6d4\uc2e0\uacf5\uc5c5\ub3c4\uc2dc \uac74\uc124\uacc4\ud68d\u2019\uc744 \ubc1c\ud45c\ud558\uc600\uc73c\uba70, 1979\ub144 8\uc6d4 31\uc77c\uc5d0 \uacbd\uae30\ub3c4 \ubc18\uc6d4\ucd9c\uc7a5\uc18c\ub97c \uc124\uce58\ud558\uc600\uace0, 1986\ub144 1\uc6d4 1\uc77c \uc5d0\ub294 \ucd9c\uc7a5\uc18c\uc5d0\uc11c \uc9c0\uae08\uc758 \uc548\uc0b0\uc2dc\ub85c \uc2b9\uaca9\ub418\uc5c8\ub2e4.", "paragraph_sentence": "\ud604\uc7ac \uc548\uc0b0\uc2dc\uc758 \uc77c\ubd80\ub85c \uad00\ub9ac\ub418\uace0 \uc788\ub294 \ud48d\ub3c4\uc5d0\uc11c\ub294 \uc870\uc120 \ub9d0 \uccad\uc77c \uc804\uc7c1\uc5d0\uc11c \uc77c\ubcf8\uacfc \uccad\ub098\ub77c \uac04\uc5d0 \ud48d\ub3c4\ud574\uc804\uc774 \uc788\uc5c8\ub2e4. \uc77c\uc81c \uac15\uc810\uae30\uc5d0\ub294 1914\ub144 \ud589\uc815\uad6c\uc5ed \uac1c\ud3b8\uc5d0 \ub530\ub77c \uc2dc\ud765\uad70 \uc218\uc554\uba74, \uad70\uc790\uba74\uc73c\ub85c \uc804\ud658\ub418\uc5c8\uc73c\uba70, \uc77c\ubd80 \uc9c0\uc5ed(\ubc18\uc6d4\uba74)\uc744 \uc218\uc6d0\uad70\uc5d0 \uc591\ub3c4\ud558\uc600\ub2e4. 1937\ub144\uc5d0\ub294 \uc548\uc0b0\uc744 \uad00\ud1b5\ud558\uc5ec \uc218\uc6d0\uacfc \uc778\ucc9c \uac04\uc744 \uc5f0\uacb0\ud558\ub294 \uc218\uc778\uc120 \ud611\uada4 \ucca0\ub3c4\uac00 \uc6b4\ud589\ud558\uae30 \uc2dc\uc791\ud558\uc600\ub2e4. \uadf8\ub7ec\ub098 1972\ub144 \ub300\ud55c\ubbfc\uad6d \uc815\ubd80\uc758 \ucca0\ub3c4 \ub178\uc120 \uc815\ub9ac\uc640 1988\ub144 \uc548\uc0b0\uc120 \uc804\ucca0 \uac1c\ud1b5 \uc774\ud6c4 \uc6b4\uc1a1\uc774 \uae09\uac10\ud558\ub2e4\uac00, 1995\ub144 12\uc6d4 31\uc77c\uc744 \ub05d\uc73c\ub85c \uc6b4\ud589\uc744 \uc911\ub2e8\ud558\uac8c \ub418\uc5c8\ub2e4. \ud55c\ud3b8\uc73c\ub85c 1970\ub144\ub300 \ud6c4\ubc18\ubd80\ud130 \uc548\uc0b0\uc2dc\uc758 \uacf5\uc5c5 \uc704\uc131\ub3c4\uc2dc\ud654\uac00 \uac00\uc18d\ub418\uae30 \uc2dc\uc791\ud558\uc5ec, 1976\ub144 12\uc6d4 \uc2dc\ud765\uad70\uacfc \ud654\uc131\uad70\uc758 \uc77c\ubd80\uc600\ub358 \uc548\uc0b0 \uc77c\ub300\uc5d0 \u2018\ubc18\uc6d4\uc2e0\uacf5\uc5c5\ub3c4\uc2dc \uac74\uc124\uacc4\ud68d\u2019\uc744 \ubc1c\ud45c\ud558\uc600\uc73c\uba70, 1979\ub144 8\uc6d4 31\uc77c\uc5d0 \uacbd\uae30\ub3c4 \ubc18\uc6d4\ucd9c\uc7a5\uc18c\ub97c \uc124\uce58\ud558\uc600\uace0, 1986\ub144 1\uc6d4 1\uc77c \uc5d0\ub294 \ucd9c\uc7a5\uc18c\uc5d0\uc11c \uc9c0\uae08\uc758 \uc548\uc0b0\uc2dc\ub85c \uc2b9\uaca9\ub418\uc5c8\ub2e4. 2002\ub144\uc5d0\ub294 \uc0c1\ub85d\uad6c\uc640 \ub2e8\uc6d0\uad6c \ub4f1 2\uac1c \uad6c\ub97c \uc0c8\ub85c \uc124\uce58\ud558\uba74\uc11c \uad6c\uc81c(\u5340\u5236)\ub97c \uc2e4\uc2dc\ud558\uc600\ub2e4.", "paragraph_answer": "\ud604\uc7ac \uc548\uc0b0\uc2dc\uc758 \uc77c\ubd80\ub85c \uad00\ub9ac\ub418\uace0 \uc788\ub294 \ud48d\ub3c4\uc5d0\uc11c\ub294 \uc870\uc120 \ub9d0 \uccad\uc77c \uc804\uc7c1\uc5d0\uc11c \uc77c\ubcf8\uacfc \uccad\ub098\ub77c \uac04\uc5d0 \ud48d\ub3c4\ud574\uc804\uc774 \uc788\uc5c8\ub2e4. \uc77c\uc81c \uac15\uc810\uae30\uc5d0\ub294 1914\ub144 \ud589\uc815\uad6c\uc5ed \uac1c\ud3b8\uc5d0 \ub530\ub77c \uc2dc\ud765\uad70 \uc218\uc554\uba74, \uad70\uc790\uba74\uc73c\ub85c \uc804\ud658\ub418\uc5c8\uc73c\uba70, \uc77c\ubd80 \uc9c0\uc5ed(\ubc18\uc6d4\uba74)\uc744 \uc218\uc6d0\uad70\uc5d0 \uc591\ub3c4\ud558\uc600\ub2e4. 1937\ub144\uc5d0\ub294 \uc548\uc0b0\uc744 \uad00\ud1b5\ud558\uc5ec \uc218\uc6d0\uacfc \uc778\ucc9c \uac04\uc744 \uc5f0\uacb0\ud558\ub294 \uc218\uc778\uc120 \ud611\uada4 \ucca0\ub3c4\uac00 \uc6b4\ud589\ud558\uae30 \uc2dc\uc791\ud558\uc600\ub2e4. \uadf8\ub7ec\ub098 1972\ub144 \ub300\ud55c\ubbfc\uad6d \uc815\ubd80\uc758 \ucca0\ub3c4 \ub178\uc120 \uc815\ub9ac\uc640 1988\ub144 \uc548\uc0b0\uc120 \uc804\ucca0 \uac1c\ud1b5 \uc774\ud6c4 \uc6b4\uc1a1\uc774 \uae09\uac10\ud558\ub2e4\uac00, 1995\ub144 12\uc6d4 31\uc77c\uc744 \ub05d\uc73c\ub85c \uc6b4\ud589\uc744 \uc911\ub2e8\ud558\uac8c \ub418\uc5c8\ub2e4. \ud55c\ud3b8\uc73c\ub85c 1970\ub144\ub300 \ud6c4\ubc18\ubd80\ud130 \uc548\uc0b0\uc2dc\uc758 \uacf5\uc5c5 \uc704\uc131\ub3c4\uc2dc\ud654\uac00 \uac00\uc18d\ub418\uae30 \uc2dc\uc791\ud558\uc5ec, 1976\ub144 12\uc6d4 \uc2dc\ud765\uad70\uacfc \ud654\uc131\uad70\uc758 \uc77c\ubd80\uc600\ub358 \uc548\uc0b0 \uc77c\ub300\uc5d0 \u2018\ubc18\uc6d4\uc2e0\uacf5\uc5c5\ub3c4\uc2dc \uac74\uc124\uacc4\ud68d\u2019\uc744 \ubc1c\ud45c\ud558\uc600\uc73c\uba70, 1979\ub144 8\uc6d4 31\uc77c\uc5d0 \uacbd\uae30\ub3c4 \ubc18\uc6d4\ucd9c\uc7a5\uc18c\ub97c \uc124\uce58\ud558\uc600\uace0, 1986\ub144 1\uc6d4 1\uc77c \uc5d0\ub294 \ucd9c\uc7a5\uc18c\uc5d0\uc11c \uc9c0\uae08\uc758 \uc548\uc0b0\uc2dc\ub85c \uc2b9\uaca9\ub418\uc5c8\ub2e4. 2002\ub144\uc5d0\ub294 \uc0c1\ub85d\uad6c\uc640 \ub2e8\uc6d0\uad6c \ub4f1 2\uac1c \uad6c\ub97c \uc0c8\ub85c \uc124\uce58\ud558\uba74\uc11c \uad6c\uc81c(\u5340\u5236)\ub97c \uc2e4\uc2dc\ud558\uc600\ub2e4.", "sentence_answer": "\ud55c\ud3b8\uc73c\ub85c 1970\ub144\ub300 \ud6c4\ubc18\ubd80\ud130 \uc548\uc0b0\uc2dc\uc758 \uacf5\uc5c5 \uc704\uc131\ub3c4\uc2dc\ud654\uac00 \uac00\uc18d\ub418\uae30 \uc2dc\uc791\ud558\uc5ec, 1976\ub144 12\uc6d4 \uc2dc\ud765\uad70\uacfc \ud654\uc131\uad70\uc758 \uc77c\ubd80\uc600\ub358 \uc548\uc0b0 \uc77c\ub300\uc5d0 \u2018\ubc18\uc6d4\uc2e0\uacf5\uc5c5\ub3c4\uc2dc \uac74\uc124\uacc4\ud68d\u2019\uc744 \ubc1c\ud45c\ud558\uc600\uc73c\uba70, 1979\ub144 8\uc6d4 31\uc77c\uc5d0 \uacbd\uae30\ub3c4 \ubc18\uc6d4\ucd9c\uc7a5\uc18c\ub97c \uc124\uce58\ud558\uc600\uace0, 1986\ub144 1\uc6d4 1\uc77c \uc5d0\ub294 \ucd9c\uc7a5\uc18c\uc5d0\uc11c \uc9c0\uae08\uc758 \uc548\uc0b0\uc2dc\ub85c \uc2b9\uaca9\ub418\uc5c8\ub2e4.", "paragraph_id": "6538946-4-2", "paragraph_question": "question: \uc548\uc0b0\uc2dc\uac00 \uc9c0\uae08\uc758 \uc548\uc0b0\uc2dc\ub85c \uc2b9\uaca9\ub41c \ub54c\ub294?, context: \ud604\uc7ac \uc548\uc0b0\uc2dc\uc758 \uc77c\ubd80\ub85c \uad00\ub9ac\ub418\uace0 \uc788\ub294 \ud48d\ub3c4\uc5d0\uc11c\ub294 \uc870\uc120 \ub9d0 \uccad\uc77c \uc804\uc7c1\uc5d0\uc11c \uc77c\ubcf8\uacfc \uccad\ub098\ub77c \uac04\uc5d0 \ud48d\ub3c4\ud574\uc804\uc774 \uc788\uc5c8\ub2e4. \uc77c\uc81c \uac15\uc810\uae30\uc5d0\ub294 1914\ub144 \ud589\uc815\uad6c\uc5ed \uac1c\ud3b8\uc5d0 \ub530\ub77c \uc2dc\ud765\uad70 \uc218\uc554\uba74, \uad70\uc790\uba74\uc73c\ub85c \uc804\ud658\ub418\uc5c8\uc73c\uba70, \uc77c\ubd80 \uc9c0\uc5ed(\ubc18\uc6d4\uba74)\uc744 \uc218\uc6d0\uad70\uc5d0 \uc591\ub3c4\ud558\uc600\ub2e4. 1937\ub144\uc5d0\ub294 \uc548\uc0b0\uc744 \uad00\ud1b5\ud558\uc5ec \uc218\uc6d0\uacfc \uc778\ucc9c \uac04\uc744 \uc5f0\uacb0\ud558\ub294 \uc218\uc778\uc120 \ud611\uada4 \ucca0\ub3c4\uac00 \uc6b4\ud589\ud558\uae30 \uc2dc\uc791\ud558\uc600\ub2e4. \uadf8\ub7ec\ub098 1972\ub144 \ub300\ud55c\ubbfc\uad6d \uc815\ubd80\uc758 \ucca0\ub3c4 \ub178\uc120 \uc815\ub9ac\uc640 1988\ub144 \uc548\uc0b0\uc120 \uc804\ucca0 \uac1c\ud1b5 \uc774\ud6c4 \uc6b4\uc1a1\uc774 \uae09\uac10\ud558\ub2e4\uac00, 1995\ub144 12\uc6d4 31\uc77c\uc744 \ub05d\uc73c\ub85c \uc6b4\ud589\uc744 \uc911\ub2e8\ud558\uac8c \ub418\uc5c8\ub2e4. \ud55c\ud3b8\uc73c\ub85c 1970\ub144\ub300 \ud6c4\ubc18\ubd80\ud130 \uc548\uc0b0\uc2dc\uc758 \uacf5\uc5c5 \uc704\uc131\ub3c4\uc2dc\ud654\uac00 \uac00\uc18d\ub418\uae30 \uc2dc\uc791\ud558\uc5ec, 1976\ub144 12\uc6d4 \uc2dc\ud765\uad70\uacfc \ud654\uc131\uad70\uc758 \uc77c\ubd80\uc600\ub358 \uc548\uc0b0 \uc77c\ub300\uc5d0 \u2018\ubc18\uc6d4\uc2e0\uacf5\uc5c5\ub3c4\uc2dc \uac74\uc124\uacc4\ud68d\u2019\uc744 \ubc1c\ud45c\ud558\uc600\uc73c\uba70, 1979\ub144 8\uc6d4 31\uc77c\uc5d0 \uacbd\uae30\ub3c4 \ubc18\uc6d4\ucd9c\uc7a5\uc18c\ub97c \uc124\uce58\ud558\uc600\uace0, 1986\ub144 1\uc6d4 1\uc77c\uc5d0\ub294 \ucd9c\uc7a5\uc18c\uc5d0\uc11c \uc9c0\uae08\uc758 \uc548\uc0b0\uc2dc\ub85c \uc2b9\uaca9\ub418\uc5c8\ub2e4. 2002\ub144\uc5d0\ub294 \uc0c1\ub85d\uad6c\uc640 \ub2e8\uc6d0\uad6c \ub4f1 2\uac1c \uad6c\ub97c \uc0c8\ub85c \uc124\uce58\ud558\uba74\uc11c \uad6c\uc81c(\u5340\u5236)\ub97c \uc2e4\uc2dc\ud558\uc600\ub2e4."} {"question": "\ud604\uc7ac \ubbf8\uad6d\uc5d0\uc11c \uc720\uc77c\ud558\uac8c \ud574\ub2ec\uc0ac\ub0e5\uc744 \ud560 \uc218 \uc788\ub294 \uc774\ub4e4\uc740?", "paragraph": "20\uc138\uae30\uc5d0 \uac78\uccd0 \uc61b \uc11c\uc2dd\uc9c0\uc758 3\ubd84\uc758 2\uc5d0 \ud574\ub2f9\ud558\ub294 \uc9c0\uc5ed\uc5d0\uc11c \ud574\ub2ec\uc758 \uac1c\uccb4\uc218\uac00 \uc99d\uac00\ud558\uc600\uc73c\uba70, \uc774 \uac1c\uccb4\uc218 \ubcf5\uc6d0\uc740 \ud574\uc591 \ud658\uacbd\ubcf4\uc874\uc758 \uac00\uc7a5 \uc131\uacf5\uc801\uc778 \uc0ac\ub840\ub85c \uc0dd\uac01\ub418\uace0 \uc788\ub2e4. \uadf8\ub7ec\ub098 IUCN\uc740 \uc5ec\uc804\ud788 \ud574\ub2ec\uc744 \uba78\uc885 \uc704\uae30\uc885\uc73c\ub85c \ubd84\ub958\ud558\uace0 \uc788\uc73c\uba70, \ud574\ub2ec\uc5d0\uac8c \uc2ec\uac01\ud55c \uc704\ud611\uc73c\ub85c \uc791\uc6a9\ud558\ub294 \uc694\uc18c\ub85c \uae30\ub984 \uc720\ucd9c, \ubc94\uace0\ub798\uc758 \ud3ec\uc2dd, \ubc00\uc5b4(\u5bc6\u6f01; poaching), \uc778\uac04\uc758 \uc5b4\uc5c5 \ud65c\ub3d9\uacfc\uc758 \ucda9\ub3cc\uc744 \ub4e4\uc5c8\ub2e4. \ud574\ub2ec\uc774 \ub09a\uc2dc\ub3c4\uad6c\uc5d0 \uc5bd\ud600\uc11c \uc775\uc0ac\ud558\ub294 \uacbd\uc6b0\ub3c4 \uc788\uae30 \ub54c\ubb38\uc774\ub2e4. \uc774\uc81c \ubbf8\uad6d\uc5d0\uc11c \ud574\ub2ec \uc0ac\ub0e5\uc740 \uc6d0\uc8fc\ubbfc\ub4e4\uc758 \uc81c\ud55c\uc801 \uc218\ub835 \ud589\uc704\ub97c \uc81c\uc678\ud558\uba74 \ubaa8\ub450 \ubd88\ubc95\uc774\ub2e4. 1991\ub144\uc5d0 \uc18c\ub828\uc774 \ubd95\uad34\ud55c \uc774\ud6c4 \ub7ec\uc2dc\uc544 \uadf9\ub3d9\uc5d0\uc11c\ub294 \ubc00\uc5b4 \ud589\uc704\uac00 \uc2ec\uac01\ud55c \uac71\uc815\uac70\ub9ac\uc600\ub2e4. \ud558\uc9c0\ub9cc \ubc95\uc801 \uc81c\uc7ac\uac00 \uac15\ud654\ub418\uace0 \uacbd\uc81c\uc0ac\uc815\uc774 \ud638\uc804\ub418\uba74\uc11c \ubc00\uc5b4 \ud589\uc704\ub294 \ub208\uc5d0 \ub744\uac8c \uc904\uc5b4\ub4e4\uc5c8\ub2e4.", "answer": "\uc6d0\uc8fc\ubbfc", "sentence": "\uc774\uc81c \ubbf8\uad6d\uc5d0\uc11c \ud574\ub2ec \uc0ac\ub0e5\uc740 \uc6d0\uc8fc\ubbfc \ub4e4\uc758 \uc81c\ud55c\uc801 \uc218\ub835 \ud589\uc704\ub97c \uc81c\uc678\ud558\uba74 \ubaa8\ub450 \ubd88\ubc95\uc774\ub2e4.", "paragraph_sentence": "20\uc138\uae30\uc5d0 \uac78\uccd0 \uc61b \uc11c\uc2dd\uc9c0\uc758 3\ubd84\uc758 2\uc5d0 \ud574\ub2f9\ud558\ub294 \uc9c0\uc5ed\uc5d0\uc11c \ud574\ub2ec\uc758 \uac1c\uccb4\uc218\uac00 \uc99d\uac00\ud558\uc600\uc73c\uba70, \uc774 \uac1c\uccb4\uc218 \ubcf5\uc6d0\uc740 \ud574\uc591 \ud658\uacbd\ubcf4\uc874\uc758 \uac00\uc7a5 \uc131\uacf5\uc801\uc778 \uc0ac\ub840\ub85c \uc0dd\uac01\ub418\uace0 \uc788\ub2e4. \uadf8\ub7ec\ub098 IUCN\uc740 \uc5ec\uc804\ud788 \ud574\ub2ec\uc744 \uba78\uc885 \uc704\uae30\uc885\uc73c\ub85c \ubd84\ub958\ud558\uace0 \uc788\uc73c\uba70, \ud574\ub2ec\uc5d0\uac8c \uc2ec\uac01\ud55c \uc704\ud611\uc73c\ub85c \uc791\uc6a9\ud558\ub294 \uc694\uc18c\ub85c \uae30\ub984 \uc720\ucd9c, \ubc94\uace0\ub798\uc758 \ud3ec\uc2dd, \ubc00\uc5b4(\u5bc6\u6f01; poaching), \uc778\uac04\uc758 \uc5b4\uc5c5 \ud65c\ub3d9\uacfc\uc758 \ucda9\ub3cc\uc744 \ub4e4\uc5c8\ub2e4. \ud574\ub2ec\uc774 \ub09a\uc2dc\ub3c4\uad6c\uc5d0 \uc5bd\ud600\uc11c \uc775\uc0ac\ud558\ub294 \uacbd\uc6b0\ub3c4 \uc788\uae30 \ub54c\ubb38\uc774\ub2e4. \uc774\uc81c \ubbf8\uad6d\uc5d0\uc11c \ud574\ub2ec \uc0ac\ub0e5\uc740 \uc6d0\uc8fc\ubbfc \ub4e4\uc758 \uc81c\ud55c\uc801 \uc218\ub835 \ud589\uc704\ub97c \uc81c\uc678\ud558\uba74 \ubaa8\ub450 \ubd88\ubc95\uc774\ub2e4. 1991\ub144\uc5d0 \uc18c\ub828\uc774 \ubd95\uad34\ud55c \uc774\ud6c4 \ub7ec\uc2dc\uc544 \uadf9\ub3d9\uc5d0\uc11c\ub294 \ubc00\uc5b4 \ud589\uc704\uac00 \uc2ec\uac01\ud55c \uac71\uc815\uac70\ub9ac\uc600\ub2e4. \ud558\uc9c0\ub9cc \ubc95\uc801 \uc81c\uc7ac\uac00 \uac15\ud654\ub418\uace0 \uacbd\uc81c\uc0ac\uc815\uc774 \ud638\uc804\ub418\uba74\uc11c \ubc00\uc5b4 \ud589\uc704\ub294 \ub208\uc5d0 \ub744\uac8c \uc904\uc5b4\ub4e4\uc5c8\ub2e4.", "paragraph_answer": "20\uc138\uae30\uc5d0 \uac78\uccd0 \uc61b \uc11c\uc2dd\uc9c0\uc758 3\ubd84\uc758 2\uc5d0 \ud574\ub2f9\ud558\ub294 \uc9c0\uc5ed\uc5d0\uc11c \ud574\ub2ec\uc758 \uac1c\uccb4\uc218\uac00 \uc99d\uac00\ud558\uc600\uc73c\uba70, \uc774 \uac1c\uccb4\uc218 \ubcf5\uc6d0\uc740 \ud574\uc591 \ud658\uacbd\ubcf4\uc874\uc758 \uac00\uc7a5 \uc131\uacf5\uc801\uc778 \uc0ac\ub840\ub85c \uc0dd\uac01\ub418\uace0 \uc788\ub2e4. \uadf8\ub7ec\ub098 IUCN\uc740 \uc5ec\uc804\ud788 \ud574\ub2ec\uc744 \uba78\uc885 \uc704\uae30\uc885\uc73c\ub85c \ubd84\ub958\ud558\uace0 \uc788\uc73c\uba70, \ud574\ub2ec\uc5d0\uac8c \uc2ec\uac01\ud55c \uc704\ud611\uc73c\ub85c \uc791\uc6a9\ud558\ub294 \uc694\uc18c\ub85c \uae30\ub984 \uc720\ucd9c, \ubc94\uace0\ub798\uc758 \ud3ec\uc2dd, \ubc00\uc5b4(\u5bc6\u6f01; poaching), \uc778\uac04\uc758 \uc5b4\uc5c5 \ud65c\ub3d9\uacfc\uc758 \ucda9\ub3cc\uc744 \ub4e4\uc5c8\ub2e4. \ud574\ub2ec\uc774 \ub09a\uc2dc\ub3c4\uad6c\uc5d0 \uc5bd\ud600\uc11c \uc775\uc0ac\ud558\ub294 \uacbd\uc6b0\ub3c4 \uc788\uae30 \ub54c\ubb38\uc774\ub2e4. \uc774\uc81c \ubbf8\uad6d\uc5d0\uc11c \ud574\ub2ec \uc0ac\ub0e5\uc740 \uc6d0\uc8fc\ubbfc \ub4e4\uc758 \uc81c\ud55c\uc801 \uc218\ub835 \ud589\uc704\ub97c \uc81c\uc678\ud558\uba74 \ubaa8\ub450 \ubd88\ubc95\uc774\ub2e4. 1991\ub144\uc5d0 \uc18c\ub828\uc774 \ubd95\uad34\ud55c \uc774\ud6c4 \ub7ec\uc2dc\uc544 \uadf9\ub3d9\uc5d0\uc11c\ub294 \ubc00\uc5b4 \ud589\uc704\uac00 \uc2ec\uac01\ud55c \uac71\uc815\uac70\ub9ac\uc600\ub2e4. \ud558\uc9c0\ub9cc \ubc95\uc801 \uc81c\uc7ac\uac00 \uac15\ud654\ub418\uace0 \uacbd\uc81c\uc0ac\uc815\uc774 \ud638\uc804\ub418\uba74\uc11c \ubc00\uc5b4 \ud589\uc704\ub294 \ub208\uc5d0 \ub744\uac8c \uc904\uc5b4\ub4e4\uc5c8\ub2e4.", "sentence_answer": "\uc774\uc81c \ubbf8\uad6d\uc5d0\uc11c \ud574\ub2ec \uc0ac\ub0e5\uc740 \uc6d0\uc8fc\ubbfc \ub4e4\uc758 \uc81c\ud55c\uc801 \uc218\ub835 \ud589\uc704\ub97c \uc81c\uc678\ud558\uba74 \ubaa8\ub450 \ubd88\ubc95\uc774\ub2e4.", "paragraph_id": "6570452-18-2", "paragraph_question": "question: \ud604\uc7ac \ubbf8\uad6d\uc5d0\uc11c \uc720\uc77c\ud558\uac8c \ud574\ub2ec\uc0ac\ub0e5\uc744 \ud560 \uc218 \uc788\ub294 \uc774\ub4e4\uc740?, context: 20\uc138\uae30\uc5d0 \uac78\uccd0 \uc61b \uc11c\uc2dd\uc9c0\uc758 3\ubd84\uc758 2\uc5d0 \ud574\ub2f9\ud558\ub294 \uc9c0\uc5ed\uc5d0\uc11c \ud574\ub2ec\uc758 \uac1c\uccb4\uc218\uac00 \uc99d\uac00\ud558\uc600\uc73c\uba70, \uc774 \uac1c\uccb4\uc218 \ubcf5\uc6d0\uc740 \ud574\uc591 \ud658\uacbd\ubcf4\uc874\uc758 \uac00\uc7a5 \uc131\uacf5\uc801\uc778 \uc0ac\ub840\ub85c \uc0dd\uac01\ub418\uace0 \uc788\ub2e4. \uadf8\ub7ec\ub098 IUCN\uc740 \uc5ec\uc804\ud788 \ud574\ub2ec\uc744 \uba78\uc885 \uc704\uae30\uc885\uc73c\ub85c \ubd84\ub958\ud558\uace0 \uc788\uc73c\uba70, \ud574\ub2ec\uc5d0\uac8c \uc2ec\uac01\ud55c \uc704\ud611\uc73c\ub85c \uc791\uc6a9\ud558\ub294 \uc694\uc18c\ub85c \uae30\ub984 \uc720\ucd9c, \ubc94\uace0\ub798\uc758 \ud3ec\uc2dd, \ubc00\uc5b4(\u5bc6\u6f01; poaching), \uc778\uac04\uc758 \uc5b4\uc5c5 \ud65c\ub3d9\uacfc\uc758 \ucda9\ub3cc\uc744 \ub4e4\uc5c8\ub2e4. \ud574\ub2ec\uc774 \ub09a\uc2dc\ub3c4\uad6c\uc5d0 \uc5bd\ud600\uc11c \uc775\uc0ac\ud558\ub294 \uacbd\uc6b0\ub3c4 \uc788\uae30 \ub54c\ubb38\uc774\ub2e4. \uc774\uc81c \ubbf8\uad6d\uc5d0\uc11c \ud574\ub2ec \uc0ac\ub0e5\uc740 \uc6d0\uc8fc\ubbfc\ub4e4\uc758 \uc81c\ud55c\uc801 \uc218\ub835 \ud589\uc704\ub97c \uc81c\uc678\ud558\uba74 \ubaa8\ub450 \ubd88\ubc95\uc774\ub2e4. 1991\ub144\uc5d0 \uc18c\ub828\uc774 \ubd95\uad34\ud55c \uc774\ud6c4 \ub7ec\uc2dc\uc544 \uadf9\ub3d9\uc5d0\uc11c\ub294 \ubc00\uc5b4 \ud589\uc704\uac00 \uc2ec\uac01\ud55c \uac71\uc815\uac70\ub9ac\uc600\ub2e4. \ud558\uc9c0\ub9cc \ubc95\uc801 \uc81c\uc7ac\uac00 \uac15\ud654\ub418\uace0 \uacbd\uc81c\uc0ac\uc815\uc774 \ud638\uc804\ub418\uba74\uc11c \ubc00\uc5b4 \ud589\uc704\ub294 \ub208\uc5d0 \ub744\uac8c \uc904\uc5b4\ub4e4\uc5c8\ub2e4."} {"question": "\uc21c\uc885\ud669\uc81c\uac00 \uc9c4\uc885, \ud5cc\uc885, \ucca0\uc885\uc744 \ud669\uc81c\ub85c \ucd94\uc22d\ud55c \uac83\uc740 \ubb34\uc5c7\uc5d0 \ub530\ub978 \uac83\uc774\uc5c8\ub098?", "paragraph": "\uc774\uc5b4 1897\ub144(\uad11\ubb34 \uc6d0\ub144), \uad6d\ud638\ub97c \ub300\ud55c\uc81c\uad6d\uc73c\ub85c \uc120\ud3ec\ud558\uace0 \ud669\uc81c\uc5d0 \uc624\ub978 \uace0\uc885\ud669\uc81c\ub294 \ud669\uac00\uc758 \uc815\ud1b5\uc131\uc744 \uc704\ud574 \ud669\uac00\uc758 \uc9c1\uacc4 \uc870\uc0c1\uc778 \uc7a5\uc885, \uc815\uc885, \uc21c\uc870, \uc775\uc885\uc744 \ubaa8\ub450 \ud669\uc81c\ub85c \ub192\uc774\uace0, \uc7a5\uc885, \uc815\uc885, \uc775\uc885\uc758 \ubb18\ud638 \ub610\ud55c \ub192\uc5ec \uac01\uac01 \uc7a5\uc870, \uc815\uc870, \ubb38\uc870\ub85c \uace0\ucce4\ub2e4. \ub610\ud55c 1908\ub144(\uc735\ud76c 2\ub144)\uc5d0 \uc21c\uc885\ud669\uc81c\ub294 \ud669\uc81c\uad6d\uc758 \uc885\ubb18 \uc81c\ub3c4\uc778 \uce60\ubb18\uc81c\uc5d0 \ub530\ub77c \uc9c4\uc885, \ud5cc\uc885, \ucca0\uc885\ub3c4 \ubaa8\ub450 \ud669\uc81c\ub85c \ucd94\uc22d\ud558\uc600\ub2e4. 1910\ub144(\uc735\ud76c 4\ub144), \ud55c\uc77c\ud569\ubcd1\uc870\uc57d\uc73c\ub85c \uc778\ud574 \ub300\ud55c\uc81c\uad6d\uc774 \uba78\ub9dd\ud55c \ud6c4 \uc774\ubbf8 \ubb3c\ub7ec\ub098\uc788\ub358 \uace0\uc885\uacfc \uc21c\uc885\uc740 \uac01\uac01 \ub355\uc218\uad81 \uc774\ud0dc\uc655, \ucc3d\ub355\uad81 \uc774\uc655 \ub4f1\uc73c\ub85c \uaca9\ud558\ub418\uc5c8\ub2e4. 1919\ub144\uc5d0 \uace0\uc885\uc774, 1926\ub144\uc5d0 \uc21c\uc885\uc774 \uc138\uc0c1\uc744 \ub5a0\ub0a0 \ub54c, \ubcf8\ub798 \ub9dd\uad6d\uc758 \uad70\uc8fc\uc5d0\uac8c \ubb18\ud638\ub97c \ubd99\uc774\ub294 \uc77c\uc740 \ub4dc\ubb3c\uc5c8\uc73c\ub098 \uc774\uc655\uc9c1\uacfc \uc870\uc120\ucd1d\ub3c5\ubd80\uc5d0\uc11c \uc870\uc120 \uc655\uc2e4\uc758 \uc608\uc640 \ud615\uc2dd\uc5d0 \ub530\ub77c \ubb18\ud638\ub97c \uc62c\ub9ac\uae30\ub3c4 \ud588\ub2e4.", "answer": "\uce60\ubb18\uc81c", "sentence": "\ub610\ud55c 1908\ub144(\uc735\ud76c 2\ub144)\uc5d0 \uc21c\uc885\ud669\uc81c\ub294 \ud669\uc81c\uad6d\uc758 \uc885\ubb18 \uc81c\ub3c4\uc778 \uce60\ubb18\uc81c \uc5d0 \ub530\ub77c \uc9c4\uc885, \ud5cc\uc885, \ucca0\uc885\ub3c4 \ubaa8\ub450 \ud669\uc81c\ub85c \ucd94\uc22d\ud558\uc600\ub2e4.", "paragraph_sentence": "\uc774\uc5b4 1897\ub144(\uad11\ubb34 \uc6d0\ub144), \uad6d\ud638\ub97c \ub300\ud55c\uc81c\uad6d\uc73c\ub85c \uc120\ud3ec\ud558\uace0 \ud669\uc81c\uc5d0 \uc624\ub978 \uace0\uc885\ud669\uc81c\ub294 \ud669\uac00\uc758 \uc815\ud1b5\uc131\uc744 \uc704\ud574 \ud669\uac00\uc758 \uc9c1\uacc4 \uc870\uc0c1\uc778 \uc7a5\uc885, \uc815\uc885, \uc21c\uc870, \uc775\uc885\uc744 \ubaa8\ub450 \ud669\uc81c\ub85c \ub192\uc774\uace0, \uc7a5\uc885, \uc815\uc885, \uc775\uc885\uc758 \ubb18\ud638 \ub610\ud55c \ub192\uc5ec \uac01\uac01 \uc7a5\uc870, \uc815\uc870, \ubb38\uc870\ub85c \uace0\ucce4\ub2e4. \ub610\ud55c 1908\ub144(\uc735\ud76c 2\ub144)\uc5d0 \uc21c\uc885\ud669\uc81c\ub294 \ud669\uc81c\uad6d\uc758 \uc885\ubb18 \uc81c\ub3c4\uc778 \uce60\ubb18\uc81c \uc5d0 \ub530\ub77c \uc9c4\uc885, \ud5cc\uc885, \ucca0\uc885\ub3c4 \ubaa8\ub450 \ud669\uc81c\ub85c \ucd94\uc22d\ud558\uc600\ub2e4. 1910\ub144(\uc735\ud76c 4\ub144), \ud55c\uc77c\ud569\ubcd1\uc870\uc57d\uc73c\ub85c \uc778\ud574 \ub300\ud55c\uc81c\uad6d\uc774 \uba78\ub9dd\ud55c \ud6c4 \uc774\ubbf8 \ubb3c\ub7ec\ub098\uc788\ub358 \uace0\uc885\uacfc \uc21c\uc885\uc740 \uac01\uac01 \ub355\uc218\uad81 \uc774\ud0dc\uc655, \ucc3d\ub355\uad81 \uc774\uc655 \ub4f1\uc73c\ub85c \uaca9\ud558\ub418\uc5c8\ub2e4. 1919\ub144\uc5d0 \uace0\uc885\uc774, 1926\ub144\uc5d0 \uc21c\uc885\uc774 \uc138\uc0c1\uc744 \ub5a0\ub0a0 \ub54c, \ubcf8\ub798 \ub9dd\uad6d\uc758 \uad70\uc8fc\uc5d0\uac8c \ubb18\ud638\ub97c \ubd99\uc774\ub294 \uc77c\uc740 \ub4dc\ubb3c\uc5c8\uc73c\ub098 \uc774\uc655\uc9c1\uacfc \uc870\uc120\ucd1d\ub3c5\ubd80\uc5d0\uc11c \uc870\uc120 \uc655\uc2e4\uc758 \uc608\uc640 \ud615\uc2dd\uc5d0 \ub530\ub77c \ubb18\ud638\ub97c \uc62c\ub9ac\uae30\ub3c4 \ud588\ub2e4.", "paragraph_answer": "\uc774\uc5b4 1897\ub144(\uad11\ubb34 \uc6d0\ub144), \uad6d\ud638\ub97c \ub300\ud55c\uc81c\uad6d\uc73c\ub85c \uc120\ud3ec\ud558\uace0 \ud669\uc81c\uc5d0 \uc624\ub978 \uace0\uc885\ud669\uc81c\ub294 \ud669\uac00\uc758 \uc815\ud1b5\uc131\uc744 \uc704\ud574 \ud669\uac00\uc758 \uc9c1\uacc4 \uc870\uc0c1\uc778 \uc7a5\uc885, \uc815\uc885, \uc21c\uc870, \uc775\uc885\uc744 \ubaa8\ub450 \ud669\uc81c\ub85c \ub192\uc774\uace0, \uc7a5\uc885, \uc815\uc885, \uc775\uc885\uc758 \ubb18\ud638 \ub610\ud55c \ub192\uc5ec \uac01\uac01 \uc7a5\uc870, \uc815\uc870, \ubb38\uc870\ub85c \uace0\ucce4\ub2e4. \ub610\ud55c 1908\ub144(\uc735\ud76c 2\ub144)\uc5d0 \uc21c\uc885\ud669\uc81c\ub294 \ud669\uc81c\uad6d\uc758 \uc885\ubb18 \uc81c\ub3c4\uc778 \uce60\ubb18\uc81c \uc5d0 \ub530\ub77c \uc9c4\uc885, \ud5cc\uc885, \ucca0\uc885\ub3c4 \ubaa8\ub450 \ud669\uc81c\ub85c \ucd94\uc22d\ud558\uc600\ub2e4. 1910\ub144(\uc735\ud76c 4\ub144), \ud55c\uc77c\ud569\ubcd1\uc870\uc57d\uc73c\ub85c \uc778\ud574 \ub300\ud55c\uc81c\uad6d\uc774 \uba78\ub9dd\ud55c \ud6c4 \uc774\ubbf8 \ubb3c\ub7ec\ub098\uc788\ub358 \uace0\uc885\uacfc \uc21c\uc885\uc740 \uac01\uac01 \ub355\uc218\uad81 \uc774\ud0dc\uc655, \ucc3d\ub355\uad81 \uc774\uc655 \ub4f1\uc73c\ub85c \uaca9\ud558\ub418\uc5c8\ub2e4. 1919\ub144\uc5d0 \uace0\uc885\uc774, 1926\ub144\uc5d0 \uc21c\uc885\uc774 \uc138\uc0c1\uc744 \ub5a0\ub0a0 \ub54c, \ubcf8\ub798 \ub9dd\uad6d\uc758 \uad70\uc8fc\uc5d0\uac8c \ubb18\ud638\ub97c \ubd99\uc774\ub294 \uc77c\uc740 \ub4dc\ubb3c\uc5c8\uc73c\ub098 \uc774\uc655\uc9c1\uacfc \uc870\uc120\ucd1d\ub3c5\ubd80\uc5d0\uc11c \uc870\uc120 \uc655\uc2e4\uc758 \uc608\uc640 \ud615\uc2dd\uc5d0 \ub530\ub77c \ubb18\ud638\ub97c \uc62c\ub9ac\uae30\ub3c4 \ud588\ub2e4.", "sentence_answer": "\ub610\ud55c 1908\ub144(\uc735\ud76c 2\ub144)\uc5d0 \uc21c\uc885\ud669\uc81c\ub294 \ud669\uc81c\uad6d\uc758 \uc885\ubb18 \uc81c\ub3c4\uc778 \uce60\ubb18\uc81c \uc5d0 \ub530\ub77c \uc9c4\uc885, \ud5cc\uc885, \ucca0\uc885\ub3c4 \ubaa8\ub450 \ud669\uc81c\ub85c \ucd94\uc22d\ud558\uc600\ub2e4.", "paragraph_id": "6504692-11-1", "paragraph_question": "question: \uc21c\uc885\ud669\uc81c\uac00 \uc9c4\uc885, \ud5cc\uc885, \ucca0\uc885\uc744 \ud669\uc81c\ub85c \ucd94\uc22d\ud55c \uac83\uc740 \ubb34\uc5c7\uc5d0 \ub530\ub978 \uac83\uc774\uc5c8\ub098?, context: \uc774\uc5b4 1897\ub144(\uad11\ubb34 \uc6d0\ub144), \uad6d\ud638\ub97c \ub300\ud55c\uc81c\uad6d\uc73c\ub85c \uc120\ud3ec\ud558\uace0 \ud669\uc81c\uc5d0 \uc624\ub978 \uace0\uc885\ud669\uc81c\ub294 \ud669\uac00\uc758 \uc815\ud1b5\uc131\uc744 \uc704\ud574 \ud669\uac00\uc758 \uc9c1\uacc4 \uc870\uc0c1\uc778 \uc7a5\uc885, \uc815\uc885, \uc21c\uc870, \uc775\uc885\uc744 \ubaa8\ub450 \ud669\uc81c\ub85c \ub192\uc774\uace0, \uc7a5\uc885, \uc815\uc885, \uc775\uc885\uc758 \ubb18\ud638 \ub610\ud55c \ub192\uc5ec \uac01\uac01 \uc7a5\uc870, \uc815\uc870, \ubb38\uc870\ub85c \uace0\ucce4\ub2e4. \ub610\ud55c 1908\ub144(\uc735\ud76c 2\ub144)\uc5d0 \uc21c\uc885\ud669\uc81c\ub294 \ud669\uc81c\uad6d\uc758 \uc885\ubb18 \uc81c\ub3c4\uc778 \uce60\ubb18\uc81c\uc5d0 \ub530\ub77c \uc9c4\uc885, \ud5cc\uc885, \ucca0\uc885\ub3c4 \ubaa8\ub450 \ud669\uc81c\ub85c \ucd94\uc22d\ud558\uc600\ub2e4. 1910\ub144(\uc735\ud76c 4\ub144), \ud55c\uc77c\ud569\ubcd1\uc870\uc57d\uc73c\ub85c \uc778\ud574 \ub300\ud55c\uc81c\uad6d\uc774 \uba78\ub9dd\ud55c \ud6c4 \uc774\ubbf8 \ubb3c\ub7ec\ub098\uc788\ub358 \uace0\uc885\uacfc \uc21c\uc885\uc740 \uac01\uac01 \ub355\uc218\uad81 \uc774\ud0dc\uc655, \ucc3d\ub355\uad81 \uc774\uc655 \ub4f1\uc73c\ub85c \uaca9\ud558\ub418\uc5c8\ub2e4. 1919\ub144\uc5d0 \uace0\uc885\uc774, 1926\ub144\uc5d0 \uc21c\uc885\uc774 \uc138\uc0c1\uc744 \ub5a0\ub0a0 \ub54c, \ubcf8\ub798 \ub9dd\uad6d\uc758 \uad70\uc8fc\uc5d0\uac8c \ubb18\ud638\ub97c \ubd99\uc774\ub294 \uc77c\uc740 \ub4dc\ubb3c\uc5c8\uc73c\ub098 \uc774\uc655\uc9c1\uacfc \uc870\uc120\ucd1d\ub3c5\ubd80\uc5d0\uc11c \uc870\uc120 \uc655\uc2e4\uc758 \uc608\uc640 \ud615\uc2dd\uc5d0 \ub530\ub77c \ubb18\ud638\ub97c \uc62c\ub9ac\uae30\ub3c4 \ud588\ub2e4."} {"question": "\uc11c\uc7ac\ud544\uc774 \ub098\ub77c\uc758 \uc8fc\uc778\uc774\uba70 \uc815\uce58\uc5d0 \ucc38\uc5ec\ud560 \uad8c\ub9ac\uac00 \uc8fc\uc5b4\uc838\uc57c \ud55c\ub2e4\uace0 \uc8fc\uc7a5\ud55c \ub300\uc0c1\uc740?", "paragraph": "\uac11\uc2e0\uc815\ubcc0\uc5d0 \uac00\ub2f4\ud588\ub2e4\uac00 \uc2e4\ud328\ud558\uc5ec \ubbf8\uad6d\uc73c\ub85c \ub9dd\uba85\ud588\ub358 \uc11c\uc7ac\ud544\uc740 \uac11\uc2e0\uc815\ubcc0 \uc138\ub825\uc5d0 \ub300\ud55c \uc0ac\uba74\ub839\uc774 \ub0b4\ub824\uc9c0\uc790 11\ub144 \ub9cc\uc5d0 \uadc0\uad6d\ud55c\ub2e4. \uc11c\uc7ac\ud544\uc740 1894\ub144 12\uc6d4 \uc815\ubd80\uc758 \uc0ac\uba74\ub839\uc774 \ub0b4\ub824\uc9c0\uace0 1895\ub144 \ubc15\uc601\ud6a8, \uc720\uae38\uc900 \ub4f1\uc758 \uad8c\uace0\ub85c \uadc0\uad6d\ud574 1896\ub144 4\uc6d4 7\uc77c \ub3c5\ub9bd\uc2e0\ubb38\uc744 \ubc1c\uac04\ud588\ub2e4. \uadc0\uad6d \uc9c1\ud6c4 \uc11c\uc7ac\ud544\uc740 \ubc31\uc131\uc774 \ub098\ub77c\uc758 \uc8fc\uc778\uc774\uba70 \ubc31\uc131\ub4e4\uc774 \uc815\uce58\uc5d0 \ucc38\uc5ec\ud560 \uad8c\ub9ac\uac00 \uc8fc\uc5b4\uc838\uc57c \ud55c\ub2e4\uace0 \uc5ed\uc124\ud558\uace0, \ubc31\uc131\ub4e4\uc774 \uc815\uce58\uc5d0 \ucc38\uc5ec\ud558\ub824\uba74 \uc6b0\uc120 \ub9d0\uacfc \uae00, \uad6d\ub0b4\uc678 \uc0ac\uc815\uc744 \uc54c\uc544\uc57c \ub418\ubbc0\ub85c \uc2e0\ubb38\uc744 \ubc1c\uac04\ud560 \uac83\uc744 \uc8fc\uc7a5\ud558\uc5ec 1\ub144\ub9cc\uc778 1896\ub144\uc5d0 \ub3c5\ub9bd\uc2e0\ubb38\uc744 \ubc1c\uac04\ud55c\ub2e4. \uadf8\ub7ec\ub098 \ub3c5\ub9bd\uc2e0\ubb38\uc758 \ubc1c\uac04 \uc678\uc5d0\ub3c4 \uc11c\uc7ac\ud544\uc740 \ub9d0\uacfc \uae00, \uad6d\ub0b4\uc678 \uc0ac\uc815\uc744 \uc548\ub2e4\uace0 \ud574\ub3c4 \ubc31\uc131\ub4e4 \uc2a4\uc2a4\ub85c\uac00 \uc790\uae30 \uad8c\ub9ac\ub97c \uc8fc\uc7a5\ud558\uc9c0 \uc54a\ub294\ub2e4\uba74 \ub204\uad6c\ub3c4 \ubc31\uc131\ub4e4\uc5d0\uac8c \uad8c\ub9ac\ub97c \uc8fc\uc9c0 \uc54a\ub294\ub2e4, \uc790\uc720\uc640 \uad8c\ub9ac\ub294 \uacf5\uc9dc\ub85c \uc8fc\uc5b4\uc9c0\ub294 \uac83\uc774 \uc544\ub2c8\ub77c\uba70 \uc870\uc815\uacfc \uad00\uccad\uc744 \ud5a5\ud574 \ubbfc\uc911\uc758 \ubaa9\uc18c\ub9ac\ub97c \ub0bc \uc218 \uc788\ub294 \uae30\uad00\uc744 \ub9cc\ub4e4 \uac83\uc744 \uc5ed\uc124\ud588\ub2e4. \uc774\uc5d0 \uc724\uce58\ud638, \uc774\uc0c1\uc7ac, \uc720\uae38\uc900 \ub4f1\uc740 \uc801\uadf9 \ub3d9\uc870\ud588\ub2e4.", "answer": "\ubc31\uc131", "sentence": "\uadc0\uad6d \uc9c1\ud6c4 \uc11c\uc7ac\ud544\uc740 \ubc31\uc131 \uc774 \ub098\ub77c\uc758 \uc8fc\uc778\uc774\uba70 \ubc31\uc131\ub4e4\uc774 \uc815\uce58\uc5d0 \ucc38\uc5ec\ud560 \uad8c\ub9ac\uac00 \uc8fc\uc5b4\uc838\uc57c \ud55c\ub2e4\uace0 \uc5ed\uc124\ud558\uace0, \ubc31\uc131\ub4e4\uc774 \uc815\uce58\uc5d0 \ucc38\uc5ec\ud558\ub824\uba74 \uc6b0\uc120 \ub9d0\uacfc \uae00, \uad6d\ub0b4\uc678 \uc0ac\uc815\uc744 \uc54c\uc544\uc57c \ub418\ubbc0\ub85c \uc2e0\ubb38\uc744 \ubc1c\uac04\ud560 \uac83\uc744 \uc8fc\uc7a5\ud558\uc5ec 1\ub144\ub9cc\uc778 1896\ub144\uc5d0 \ub3c5\ub9bd\uc2e0\ubb38\uc744 \ubc1c\uac04\ud55c\ub2e4.", "paragraph_sentence": "\uac11\uc2e0\uc815\ubcc0\uc5d0 \uac00\ub2f4\ud588\ub2e4\uac00 \uc2e4\ud328\ud558\uc5ec \ubbf8\uad6d\uc73c\ub85c \ub9dd\uba85\ud588\ub358 \uc11c\uc7ac\ud544\uc740 \uac11\uc2e0\uc815\ubcc0 \uc138\ub825\uc5d0 \ub300\ud55c \uc0ac\uba74\ub839\uc774 \ub0b4\ub824\uc9c0\uc790 11\ub144 \ub9cc\uc5d0 \uadc0\uad6d\ud55c\ub2e4. \uc11c\uc7ac\ud544\uc740 1894\ub144 12\uc6d4 \uc815\ubd80\uc758 \uc0ac\uba74\ub839\uc774 \ub0b4\ub824\uc9c0\uace0 1895\ub144 \ubc15\uc601\ud6a8, \uc720\uae38\uc900 \ub4f1\uc758 \uad8c\uace0\ub85c \uadc0\uad6d\ud574 1896\ub144 4\uc6d4 7\uc77c \ub3c5\ub9bd\uc2e0\ubb38\uc744 \ubc1c\uac04\ud588\ub2e4. \uadc0\uad6d \uc9c1\ud6c4 \uc11c\uc7ac\ud544\uc740 \ubc31\uc131 \uc774 \ub098\ub77c\uc758 \uc8fc\uc778\uc774\uba70 \ubc31\uc131\ub4e4\uc774 \uc815\uce58\uc5d0 \ucc38\uc5ec\ud560 \uad8c\ub9ac\uac00 \uc8fc\uc5b4\uc838\uc57c \ud55c\ub2e4\uace0 \uc5ed\uc124\ud558\uace0, \ubc31\uc131\ub4e4\uc774 \uc815\uce58\uc5d0 \ucc38\uc5ec\ud558\ub824\uba74 \uc6b0\uc120 \ub9d0\uacfc \uae00, \uad6d\ub0b4\uc678 \uc0ac\uc815\uc744 \uc54c\uc544\uc57c \ub418\ubbc0\ub85c \uc2e0\ubb38\uc744 \ubc1c\uac04\ud560 \uac83\uc744 \uc8fc\uc7a5\ud558\uc5ec 1\ub144\ub9cc\uc778 1896\ub144\uc5d0 \ub3c5\ub9bd\uc2e0\ubb38\uc744 \ubc1c\uac04\ud55c\ub2e4. \uadf8\ub7ec\ub098 \ub3c5\ub9bd\uc2e0\ubb38\uc758 \ubc1c\uac04 \uc678\uc5d0\ub3c4 \uc11c\uc7ac\ud544\uc740 \ub9d0\uacfc \uae00, \uad6d\ub0b4\uc678 \uc0ac\uc815\uc744 \uc548\ub2e4\uace0 \ud574\ub3c4 \ubc31\uc131\ub4e4 \uc2a4\uc2a4\ub85c\uac00 \uc790\uae30 \uad8c\ub9ac\ub97c \uc8fc\uc7a5\ud558\uc9c0 \uc54a\ub294\ub2e4\uba74 \ub204\uad6c\ub3c4 \ubc31\uc131\ub4e4\uc5d0\uac8c \uad8c\ub9ac\ub97c \uc8fc\uc9c0 \uc54a\ub294\ub2e4, \uc790\uc720\uc640 \uad8c\ub9ac\ub294 \uacf5\uc9dc\ub85c \uc8fc\uc5b4\uc9c0\ub294 \uac83\uc774 \uc544\ub2c8\ub77c\uba70 \uc870\uc815\uacfc \uad00\uccad\uc744 \ud5a5\ud574 \ubbfc\uc911\uc758 \ubaa9\uc18c\ub9ac\ub97c \ub0bc \uc218 \uc788\ub294 \uae30\uad00\uc744 \ub9cc\ub4e4 \uac83\uc744 \uc5ed\uc124\ud588\ub2e4. \uc774\uc5d0 \uc724\uce58\ud638, \uc774\uc0c1\uc7ac, \uc720\uae38\uc900 \ub4f1\uc740 \uc801\uadf9 \ub3d9\uc870\ud588\ub2e4.", "paragraph_answer": "\uac11\uc2e0\uc815\ubcc0\uc5d0 \uac00\ub2f4\ud588\ub2e4\uac00 \uc2e4\ud328\ud558\uc5ec \ubbf8\uad6d\uc73c\ub85c \ub9dd\uba85\ud588\ub358 \uc11c\uc7ac\ud544\uc740 \uac11\uc2e0\uc815\ubcc0 \uc138\ub825\uc5d0 \ub300\ud55c \uc0ac\uba74\ub839\uc774 \ub0b4\ub824\uc9c0\uc790 11\ub144 \ub9cc\uc5d0 \uadc0\uad6d\ud55c\ub2e4. \uc11c\uc7ac\ud544\uc740 1894\ub144 12\uc6d4 \uc815\ubd80\uc758 \uc0ac\uba74\ub839\uc774 \ub0b4\ub824\uc9c0\uace0 1895\ub144 \ubc15\uc601\ud6a8, \uc720\uae38\uc900 \ub4f1\uc758 \uad8c\uace0\ub85c \uadc0\uad6d\ud574 1896\ub144 4\uc6d4 7\uc77c \ub3c5\ub9bd\uc2e0\ubb38\uc744 \ubc1c\uac04\ud588\ub2e4. \uadc0\uad6d \uc9c1\ud6c4 \uc11c\uc7ac\ud544\uc740 \ubc31\uc131 \uc774 \ub098\ub77c\uc758 \uc8fc\uc778\uc774\uba70 \ubc31\uc131\ub4e4\uc774 \uc815\uce58\uc5d0 \ucc38\uc5ec\ud560 \uad8c\ub9ac\uac00 \uc8fc\uc5b4\uc838\uc57c \ud55c\ub2e4\uace0 \uc5ed\uc124\ud558\uace0, \ubc31\uc131\ub4e4\uc774 \uc815\uce58\uc5d0 \ucc38\uc5ec\ud558\ub824\uba74 \uc6b0\uc120 \ub9d0\uacfc \uae00, \uad6d\ub0b4\uc678 \uc0ac\uc815\uc744 \uc54c\uc544\uc57c \ub418\ubbc0\ub85c \uc2e0\ubb38\uc744 \ubc1c\uac04\ud560 \uac83\uc744 \uc8fc\uc7a5\ud558\uc5ec 1\ub144\ub9cc\uc778 1896\ub144\uc5d0 \ub3c5\ub9bd\uc2e0\ubb38\uc744 \ubc1c\uac04\ud55c\ub2e4. \uadf8\ub7ec\ub098 \ub3c5\ub9bd\uc2e0\ubb38\uc758 \ubc1c\uac04 \uc678\uc5d0\ub3c4 \uc11c\uc7ac\ud544\uc740 \ub9d0\uacfc \uae00, \uad6d\ub0b4\uc678 \uc0ac\uc815\uc744 \uc548\ub2e4\uace0 \ud574\ub3c4 \ubc31\uc131\ub4e4 \uc2a4\uc2a4\ub85c\uac00 \uc790\uae30 \uad8c\ub9ac\ub97c \uc8fc\uc7a5\ud558\uc9c0 \uc54a\ub294\ub2e4\uba74 \ub204\uad6c\ub3c4 \ubc31\uc131\ub4e4\uc5d0\uac8c \uad8c\ub9ac\ub97c \uc8fc\uc9c0 \uc54a\ub294\ub2e4, \uc790\uc720\uc640 \uad8c\ub9ac\ub294 \uacf5\uc9dc\ub85c \uc8fc\uc5b4\uc9c0\ub294 \uac83\uc774 \uc544\ub2c8\ub77c\uba70 \uc870\uc815\uacfc \uad00\uccad\uc744 \ud5a5\ud574 \ubbfc\uc911\uc758 \ubaa9\uc18c\ub9ac\ub97c \ub0bc \uc218 \uc788\ub294 \uae30\uad00\uc744 \ub9cc\ub4e4 \uac83\uc744 \uc5ed\uc124\ud588\ub2e4. \uc774\uc5d0 \uc724\uce58\ud638, \uc774\uc0c1\uc7ac, \uc720\uae38\uc900 \ub4f1\uc740 \uc801\uadf9 \ub3d9\uc870\ud588\ub2e4.", "sentence_answer": "\uadc0\uad6d \uc9c1\ud6c4 \uc11c\uc7ac\ud544\uc740 \ubc31\uc131 \uc774 \ub098\ub77c\uc758 \uc8fc\uc778\uc774\uba70 \ubc31\uc131\ub4e4\uc774 \uc815\uce58\uc5d0 \ucc38\uc5ec\ud560 \uad8c\ub9ac\uac00 \uc8fc\uc5b4\uc838\uc57c \ud55c\ub2e4\uace0 \uc5ed\uc124\ud558\uace0, \ubc31\uc131\ub4e4\uc774 \uc815\uce58\uc5d0 \ucc38\uc5ec\ud558\ub824\uba74 \uc6b0\uc120 \ub9d0\uacfc \uae00, \uad6d\ub0b4\uc678 \uc0ac\uc815\uc744 \uc54c\uc544\uc57c \ub418\ubbc0\ub85c \uc2e0\ubb38\uc744 \ubc1c\uac04\ud560 \uac83\uc744 \uc8fc\uc7a5\ud558\uc5ec 1\ub144\ub9cc\uc778 1896\ub144\uc5d0 \ub3c5\ub9bd\uc2e0\ubb38\uc744 \ubc1c\uac04\ud55c\ub2e4.", "paragraph_id": "6534017-1-2", "paragraph_question": "question: \uc11c\uc7ac\ud544\uc774 \ub098\ub77c\uc758 \uc8fc\uc778\uc774\uba70 \uc815\uce58\uc5d0 \ucc38\uc5ec\ud560 \uad8c\ub9ac\uac00 \uc8fc\uc5b4\uc838\uc57c \ud55c\ub2e4\uace0 \uc8fc\uc7a5\ud55c \ub300\uc0c1\uc740?, context: \uac11\uc2e0\uc815\ubcc0\uc5d0 \uac00\ub2f4\ud588\ub2e4\uac00 \uc2e4\ud328\ud558\uc5ec \ubbf8\uad6d\uc73c\ub85c \ub9dd\uba85\ud588\ub358 \uc11c\uc7ac\ud544\uc740 \uac11\uc2e0\uc815\ubcc0 \uc138\ub825\uc5d0 \ub300\ud55c \uc0ac\uba74\ub839\uc774 \ub0b4\ub824\uc9c0\uc790 11\ub144 \ub9cc\uc5d0 \uadc0\uad6d\ud55c\ub2e4. \uc11c\uc7ac\ud544\uc740 1894\ub144 12\uc6d4 \uc815\ubd80\uc758 \uc0ac\uba74\ub839\uc774 \ub0b4\ub824\uc9c0\uace0 1895\ub144 \ubc15\uc601\ud6a8, \uc720\uae38\uc900 \ub4f1\uc758 \uad8c\uace0\ub85c \uadc0\uad6d\ud574 1896\ub144 4\uc6d4 7\uc77c \ub3c5\ub9bd\uc2e0\ubb38\uc744 \ubc1c\uac04\ud588\ub2e4. \uadc0\uad6d \uc9c1\ud6c4 \uc11c\uc7ac\ud544\uc740 \ubc31\uc131\uc774 \ub098\ub77c\uc758 \uc8fc\uc778\uc774\uba70 \ubc31\uc131\ub4e4\uc774 \uc815\uce58\uc5d0 \ucc38\uc5ec\ud560 \uad8c\ub9ac\uac00 \uc8fc\uc5b4\uc838\uc57c \ud55c\ub2e4\uace0 \uc5ed\uc124\ud558\uace0, \ubc31\uc131\ub4e4\uc774 \uc815\uce58\uc5d0 \ucc38\uc5ec\ud558\ub824\uba74 \uc6b0\uc120 \ub9d0\uacfc \uae00, \uad6d\ub0b4\uc678 \uc0ac\uc815\uc744 \uc54c\uc544\uc57c \ub418\ubbc0\ub85c \uc2e0\ubb38\uc744 \ubc1c\uac04\ud560 \uac83\uc744 \uc8fc\uc7a5\ud558\uc5ec 1\ub144\ub9cc\uc778 1896\ub144\uc5d0 \ub3c5\ub9bd\uc2e0\ubb38\uc744 \ubc1c\uac04\ud55c\ub2e4. \uadf8\ub7ec\ub098 \ub3c5\ub9bd\uc2e0\ubb38\uc758 \ubc1c\uac04 \uc678\uc5d0\ub3c4 \uc11c\uc7ac\ud544\uc740 \ub9d0\uacfc \uae00, \uad6d\ub0b4\uc678 \uc0ac\uc815\uc744 \uc548\ub2e4\uace0 \ud574\ub3c4 \ubc31\uc131\ub4e4 \uc2a4\uc2a4\ub85c\uac00 \uc790\uae30 \uad8c\ub9ac\ub97c \uc8fc\uc7a5\ud558\uc9c0 \uc54a\ub294\ub2e4\uba74 \ub204\uad6c\ub3c4 \ubc31\uc131\ub4e4\uc5d0\uac8c \uad8c\ub9ac\ub97c \uc8fc\uc9c0 \uc54a\ub294\ub2e4, \uc790\uc720\uc640 \uad8c\ub9ac\ub294 \uacf5\uc9dc\ub85c \uc8fc\uc5b4\uc9c0\ub294 \uac83\uc774 \uc544\ub2c8\ub77c\uba70 \uc870\uc815\uacfc \uad00\uccad\uc744 \ud5a5\ud574 \ubbfc\uc911\uc758 \ubaa9\uc18c\ub9ac\ub97c \ub0bc \uc218 \uc788\ub294 \uae30\uad00\uc744 \ub9cc\ub4e4 \uac83\uc744 \uc5ed\uc124\ud588\ub2e4. \uc774\uc5d0 \uc724\uce58\ud638, \uc774\uc0c1\uc7ac, \uc720\uae38\uc900 \ub4f1\uc740 \uc801\uadf9 \ub3d9\uc870\ud588\ub2e4."} {"question": "\ud0d1 \ud0ac\uc744 \uc704\ud574 \uc0ac\uc6a9\ub41c \uc9c4\ud759\uc744 \ub9cc\ub4e4\uae30 \uc704\ud574 \ubb3c\uacfc \ubbf8\ub124\ub784\uacfc \ud568\uaed8 \uc0ac\uc6a9\ub41c \uc7ac\ub8cc\ub294?", "paragraph": "5\uc6d4 11\uc77c\uc5d0 \uc880 \ub354 \uc791\uc740 \ub69c\uaed1\uc744 \uc2ec\ud574\ub85c \ub0b4\ub824\ubcf4\ub0b4\ub294 \uc2dc\ub3c4\uac00 \uc774\ub8e8\uc5b4\uc84c\uc73c\ub098, \uc774 \uc5ed\uc2dc \ud6a8\uacfc\ub97c \ubcf4\uc9c0 \ubabb\ud588\ub2e4. 5\uc6d4 14\uc77c\uc5d0 \ubd80\ub7ec\uc9c4 \ud30c\uc774\ud504\uc5d0 \uc791\uc740 \ud30c\uc774\ud504\ub97c \uaf42\uc544\uc11c \uc720\ucd9c\ub418\ub294 \uac00\uc2a4\ub97c \ubf51\uc544\ub0b4\uae30 \uc2dc\uc791\ud55c\ub2e4. \uc720\ucd9c\ub7c9\uc744 \uc904\uc774\uae30 \uc704\ud55c \uc2dc\ub3c4\uc774\uba70, \uc720\ucd9c\uad6c\uba4d\uc744 \uc644\uc804\ud788 \ub36e\uc5b4\uc11c \uc7a0\uadf8\ub294 \uc2dc\ub3c4\ub97c \ud560\ub54c\uae4c\uc9c0 \ud558\ub8e8 1\ucc9c~5\ucc9c \ubc30\ub7f4\uc529 \ube7c\ub0c8\uc5c8\ub2e4. 5\uc6d4 25\uc77c\uc5d0 \ubb34\uac70\uc6b4 \ub4dc\ub9b4\ub9c1 \uc9c4\ud759\uc744 \ud38c\ud504\ub85c \uc720\ucd9c\uad6c\uba4d\uc5d0 \ub123\uc5b4 \uc720\ucd9c \uc555\ub825\uc744 \uc904\uc774\uace0, \uc2dc\uba58\ud2b8\ub85c \ubc1c\ub77c\uc11c \ucd5c\uc885\uc801\uc73c\ub85c \uad6c\uba4d\uc744 \ub9c9\uc544\ubc84\ub9ac\ub294 \"\ud0d1 \ud0ac\"\uc774\ub77c\uace0 \uc54c\ub824\uc9c4 \ubc29\uc2dd\uc744 \uc2dc\ub3c4\ud55c\ub2e4. \uc77c\ubc18 \uac00\uc815\uc5d0\uc11c\uc758 \uc4f0\ub808\uae30\ub97c \uc774\uc6a9\ud558\ub294 \uac83\uc774\uc5c8\ub2e4. \ub2e8\ub2e8\ud55c \ubb3c\uc9c8\uacfc \ubd80\ub4dc\ub7ec\uc6b4 \ubb3c\uc9c8\uc758 \uacb0\ud569\uc774 \uad6c\uba4d\uc744 \ucc28\ub2e8\ud558\ub294\ub370 \ud6a8\uacfc\uac00 \uc788\uc744\uac83\uc73c\ub85c \uae30\ub300\ud588\ub2e4. \ub2e4\uc74c\uc740 \ubc29\ub300\ud55c \uc591\uc758 \uc9c4\ud759\uc744 \uc774\uc6a9\ud558\ub294 \uac83\uc73c\ub85c \uc77c\ubc18\uc801\uc778 \uc9c4\ud759\uc774 \uc544\ub2cc \uc2dc\ucd94\ud55c \uc9c4\ud759\uc744 \ubb3c\uacfc \ubbf8\ub124\ub784, \uc911\uc815\uc11d \ub4f1\uc73c\ub85c \ud63c\ud569\ud55c \uac83\uc73c\ub85c \uc11d\uc720\ubcf4\ub2e4 \uac15\ud55c \uc810\uc131\uc744 \ub098\ud0c0\ub0c8\ub2e4. \ucda9\ubd84\ud55c \uc9c4\ud759\uc774 \uc720\uc815\uc744 \ub36e\uac8c \ub418\uba74 \uc601\uad6c\uc801\uc73c\ub85c \ubd84\ucd9c\uc744 \uba48\ucd94\uac8c \ud558\ub294 \uac83\uc774\uc5c8\ub2e4. \ubc14\ub2e4 1,500\ubbf8\ud130 \uc544\ub798 BP\ub294 \uac70\ub300\ud55c \uace0\ucca0 \ucee8\ud14c\uc774\ub108\ub97c \uac16\ub2e4\ub193\uc558\ub2e4. \uc4f0\ub808\uae30\ub4e4\uc740 \ud30c\uc774\ud504\ub97c \ub530\ub77c BOP\ub97c \uac70\uccd0 \uc720\uc815 \uc544\ub798\ub85c \ub0b4\ub824\uac00\uac8c \ub41c\ub2e4. \ub4a4\uc774\uc5b4 \uc9c4\ud759\uc774 \ubc14\ub2e4\uc704 \ubc30\ub85c\ubd80\ud130 \uacf5\uae09\ub41c\ub2e4. \ucc98\uc74c\uc5d0\ub294 \uc791\uc5c5\uc774 \uc798 \uc9c4\ud589\ub418\ub294 \uac83\uc73c\ub85c \ubcf4\uc600\ub2e4. \ud558\uc9c0\ub9cc \ud604\uc2e4\uc740 \ub2ec\ub790\ub2e4. \uc4f0\ub808\uae30\uc5d0 \uac00\ud574\uc9c0\ub294 \uc555\ub825\uc774 \ub108\ubb34 \uac15\ud588\ub358 \uac83\uc774\ub2e4. \uc774\ud2c0\ub3d9\uc548 BP\ub294 5\ubc88\uc774\ub098 \uc4f0\ub808\uae30\uc640 \uc9c4\ud759\uc744 \ub0b4\ubcf4\ub0c8\ub2e4. \uc9c4\ud759\uc744 \ub0b4\ubcf4\ub0bc\ub54c\ub9c8\ub2e4 \uc6d0\uc720\uc640 \ud568\uaed8 \ubaa8\ub4e0\uac83\uc774 \ub2e4\uc2dc \ubc00\ub824\uc62c\ub77c\uc654\ub2e4. \ud0d1 \ud0ac\uc774 \uc2e4\ud328\ub77c\ub294 \uac83\uc774 \uc99d\uba85\ub420\ub54c\uae4c\uc9c0 445\ub9cc\ub9ac\ud130\uc758 \uc9c4\ud759\uc774 \ud22c\uc785\ub410\ub2e4. \ub354 \uace0\uc555\uc73c\ub85c \ud0d1 \ud0ac\uc744 \uc2dc\ub3c4\ud558\uba74 \uc6d0\uc720\uac00 \ud574\uc800\ub85c \ud758\ub7ec\ub4e4\uc5b4\uac00\uba74\uc11c \uc2dc\ucd94\uad6c \uc678\uc758 \uad6c\uc5ed\uc5d0\uc11c \ud130\uc9c8\uc9c0\ub3c4 \ubab0\ub77c\uc11c \uc704\ud5d8\ud558\ub2e4\uace0 \ud310\ub2e8\ud558\uc5ec \ub354\uc774\uc0c1 \uc2dc\ub3c4\ud558\uc9c0 \uc54a\uc558\ub2e4.", "answer": "\uc911\uc815\uc11d", "sentence": "\ub2e4\uc74c\uc740 \ubc29\ub300\ud55c \uc591\uc758 \uc9c4\ud759\uc744 \uc774\uc6a9\ud558\ub294 \uac83\uc73c\ub85c \uc77c\ubc18\uc801\uc778 \uc9c4\ud759\uc774 \uc544\ub2cc \uc2dc\ucd94\ud55c \uc9c4\ud759\uc744 \ubb3c\uacfc \ubbf8\ub124\ub784, \uc911\uc815\uc11d \ub4f1\uc73c\ub85c \ud63c\ud569\ud55c \uac83\uc73c\ub85c \uc11d\uc720\ubcf4\ub2e4 \uac15\ud55c \uc810\uc131\uc744 \ub098\ud0c0\ub0c8\ub2e4.", "paragraph_sentence": "5\uc6d4 11\uc77c\uc5d0 \uc880 \ub354 \uc791\uc740 \ub69c\uaed1\uc744 \uc2ec\ud574\ub85c \ub0b4\ub824\ubcf4\ub0b4\ub294 \uc2dc\ub3c4\uac00 \uc774\ub8e8\uc5b4\uc84c\uc73c\ub098, \uc774 \uc5ed\uc2dc \ud6a8\uacfc\ub97c \ubcf4\uc9c0 \ubabb\ud588\ub2e4. 5\uc6d4 14\uc77c\uc5d0 \ubd80\ub7ec\uc9c4 \ud30c\uc774\ud504\uc5d0 \uc791\uc740 \ud30c\uc774\ud504\ub97c \uaf42\uc544\uc11c \uc720\ucd9c\ub418\ub294 \uac00\uc2a4\ub97c \ubf51\uc544\ub0b4\uae30 \uc2dc\uc791\ud55c\ub2e4. \uc720\ucd9c\ub7c9\uc744 \uc904\uc774\uae30 \uc704\ud55c \uc2dc\ub3c4\uc774\uba70, \uc720\ucd9c\uad6c\uba4d\uc744 \uc644\uc804\ud788 \ub36e\uc5b4\uc11c \uc7a0\uadf8\ub294 \uc2dc\ub3c4\ub97c \ud560\ub54c\uae4c\uc9c0 \ud558\ub8e8 1\ucc9c~5\ucc9c \ubc30\ub7f4\uc529 \ube7c\ub0c8\uc5c8\ub2e4. 5\uc6d4 25\uc77c\uc5d0 \ubb34\uac70\uc6b4 \ub4dc\ub9b4\ub9c1 \uc9c4\ud759\uc744 \ud38c\ud504\ub85c \uc720\ucd9c\uad6c\uba4d\uc5d0 \ub123\uc5b4 \uc720\ucd9c \uc555\ub825\uc744 \uc904\uc774\uace0, \uc2dc\uba58\ud2b8\ub85c \ubc1c\ub77c\uc11c \ucd5c\uc885\uc801\uc73c\ub85c \uad6c\uba4d\uc744 \ub9c9\uc544\ubc84\ub9ac\ub294 \"\ud0d1 \ud0ac\"\uc774\ub77c\uace0 \uc54c\ub824\uc9c4 \ubc29\uc2dd\uc744 \uc2dc\ub3c4\ud55c\ub2e4. \uc77c\ubc18 \uac00\uc815\uc5d0\uc11c\uc758 \uc4f0\ub808\uae30\ub97c \uc774\uc6a9\ud558\ub294 \uac83\uc774\uc5c8\ub2e4. \ub2e8\ub2e8\ud55c \ubb3c\uc9c8\uacfc \ubd80\ub4dc\ub7ec\uc6b4 \ubb3c\uc9c8\uc758 \uacb0\ud569\uc774 \uad6c\uba4d\uc744 \ucc28\ub2e8\ud558\ub294\ub370 \ud6a8\uacfc\uac00 \uc788\uc744\uac83\uc73c\ub85c \uae30\ub300\ud588\ub2e4. \ub2e4\uc74c\uc740 \ubc29\ub300\ud55c \uc591\uc758 \uc9c4\ud759\uc744 \uc774\uc6a9\ud558\ub294 \uac83\uc73c\ub85c \uc77c\ubc18\uc801\uc778 \uc9c4\ud759\uc774 \uc544\ub2cc \uc2dc\ucd94\ud55c \uc9c4\ud759\uc744 \ubb3c\uacfc \ubbf8\ub124\ub784, \uc911\uc815\uc11d \ub4f1\uc73c\ub85c \ud63c\ud569\ud55c \uac83\uc73c\ub85c \uc11d\uc720\ubcf4\ub2e4 \uac15\ud55c \uc810\uc131\uc744 \ub098\ud0c0\ub0c8\ub2e4. \ucda9\ubd84\ud55c \uc9c4\ud759\uc774 \uc720\uc815\uc744 \ub36e\uac8c \ub418\uba74 \uc601\uad6c\uc801\uc73c\ub85c \ubd84\ucd9c\uc744 \uba48\ucd94\uac8c \ud558\ub294 \uac83\uc774\uc5c8\ub2e4. \ubc14\ub2e4 1,500\ubbf8\ud130 \uc544\ub798 BP\ub294 \uac70\ub300\ud55c \uace0\ucca0 \ucee8\ud14c\uc774\ub108\ub97c \uac16\ub2e4\ub193\uc558\ub2e4. \uc4f0\ub808\uae30\ub4e4\uc740 \ud30c\uc774\ud504\ub97c \ub530\ub77c BOP\ub97c \uac70\uccd0 \uc720\uc815 \uc544\ub798\ub85c \ub0b4\ub824\uac00\uac8c \ub41c\ub2e4. \ub4a4\uc774\uc5b4 \uc9c4\ud759\uc774 \ubc14\ub2e4\uc704 \ubc30\ub85c\ubd80\ud130 \uacf5\uae09\ub41c\ub2e4. \ucc98\uc74c\uc5d0\ub294 \uc791\uc5c5\uc774 \uc798 \uc9c4\ud589\ub418\ub294 \uac83\uc73c\ub85c \ubcf4\uc600\ub2e4. \ud558\uc9c0\ub9cc \ud604\uc2e4\uc740 \ub2ec\ub790\ub2e4. \uc4f0\ub808\uae30\uc5d0 \uac00\ud574\uc9c0\ub294 \uc555\ub825\uc774 \ub108\ubb34 \uac15\ud588\ub358 \uac83\uc774\ub2e4. \uc774\ud2c0\ub3d9\uc548 BP\ub294 5\ubc88\uc774\ub098 \uc4f0\ub808\uae30\uc640 \uc9c4\ud759\uc744 \ub0b4\ubcf4\ub0c8\ub2e4. \uc9c4\ud759\uc744 \ub0b4\ubcf4\ub0bc\ub54c\ub9c8\ub2e4 \uc6d0\uc720\uc640 \ud568\uaed8 \ubaa8\ub4e0\uac83\uc774 \ub2e4\uc2dc \ubc00\ub824\uc62c\ub77c\uc654\ub2e4. \ud0d1 \ud0ac\uc774 \uc2e4\ud328\ub77c\ub294 \uac83\uc774 \uc99d\uba85\ub420\ub54c\uae4c\uc9c0 445\ub9cc\ub9ac\ud130\uc758 \uc9c4\ud759\uc774 \ud22c\uc785\ub410\ub2e4. \ub354 \uace0\uc555\uc73c\ub85c \ud0d1 \ud0ac\uc744 \uc2dc\ub3c4\ud558\uba74 \uc6d0\uc720\uac00 \ud574\uc800\ub85c \ud758\ub7ec\ub4e4\uc5b4\uac00\uba74\uc11c \uc2dc\ucd94\uad6c \uc678\uc758 \uad6c\uc5ed\uc5d0\uc11c \ud130\uc9c8\uc9c0\ub3c4 \ubab0\ub77c\uc11c \uc704\ud5d8\ud558\ub2e4\uace0 \ud310\ub2e8\ud558\uc5ec \ub354\uc774\uc0c1 \uc2dc\ub3c4\ud558\uc9c0 \uc54a\uc558\ub2e4.", "paragraph_answer": "5\uc6d4 11\uc77c\uc5d0 \uc880 \ub354 \uc791\uc740 \ub69c\uaed1\uc744 \uc2ec\ud574\ub85c \ub0b4\ub824\ubcf4\ub0b4\ub294 \uc2dc\ub3c4\uac00 \uc774\ub8e8\uc5b4\uc84c\uc73c\ub098, \uc774 \uc5ed\uc2dc \ud6a8\uacfc\ub97c \ubcf4\uc9c0 \ubabb\ud588\ub2e4. 5\uc6d4 14\uc77c\uc5d0 \ubd80\ub7ec\uc9c4 \ud30c\uc774\ud504\uc5d0 \uc791\uc740 \ud30c\uc774\ud504\ub97c \uaf42\uc544\uc11c \uc720\ucd9c\ub418\ub294 \uac00\uc2a4\ub97c \ubf51\uc544\ub0b4\uae30 \uc2dc\uc791\ud55c\ub2e4. \uc720\ucd9c\ub7c9\uc744 \uc904\uc774\uae30 \uc704\ud55c \uc2dc\ub3c4\uc774\uba70, \uc720\ucd9c\uad6c\uba4d\uc744 \uc644\uc804\ud788 \ub36e\uc5b4\uc11c \uc7a0\uadf8\ub294 \uc2dc\ub3c4\ub97c \ud560\ub54c\uae4c\uc9c0 \ud558\ub8e8 1\ucc9c~5\ucc9c \ubc30\ub7f4\uc529 \ube7c\ub0c8\uc5c8\ub2e4. 5\uc6d4 25\uc77c\uc5d0 \ubb34\uac70\uc6b4 \ub4dc\ub9b4\ub9c1 \uc9c4\ud759\uc744 \ud38c\ud504\ub85c \uc720\ucd9c\uad6c\uba4d\uc5d0 \ub123\uc5b4 \uc720\ucd9c \uc555\ub825\uc744 \uc904\uc774\uace0, \uc2dc\uba58\ud2b8\ub85c \ubc1c\ub77c\uc11c \ucd5c\uc885\uc801\uc73c\ub85c \uad6c\uba4d\uc744 \ub9c9\uc544\ubc84\ub9ac\ub294 \"\ud0d1 \ud0ac\"\uc774\ub77c\uace0 \uc54c\ub824\uc9c4 \ubc29\uc2dd\uc744 \uc2dc\ub3c4\ud55c\ub2e4. \uc77c\ubc18 \uac00\uc815\uc5d0\uc11c\uc758 \uc4f0\ub808\uae30\ub97c \uc774\uc6a9\ud558\ub294 \uac83\uc774\uc5c8\ub2e4. \ub2e8\ub2e8\ud55c \ubb3c\uc9c8\uacfc \ubd80\ub4dc\ub7ec\uc6b4 \ubb3c\uc9c8\uc758 \uacb0\ud569\uc774 \uad6c\uba4d\uc744 \ucc28\ub2e8\ud558\ub294\ub370 \ud6a8\uacfc\uac00 \uc788\uc744\uac83\uc73c\ub85c \uae30\ub300\ud588\ub2e4. \ub2e4\uc74c\uc740 \ubc29\ub300\ud55c \uc591\uc758 \uc9c4\ud759\uc744 \uc774\uc6a9\ud558\ub294 \uac83\uc73c\ub85c \uc77c\ubc18\uc801\uc778 \uc9c4\ud759\uc774 \uc544\ub2cc \uc2dc\ucd94\ud55c \uc9c4\ud759\uc744 \ubb3c\uacfc \ubbf8\ub124\ub784, \uc911\uc815\uc11d \ub4f1\uc73c\ub85c \ud63c\ud569\ud55c \uac83\uc73c\ub85c \uc11d\uc720\ubcf4\ub2e4 \uac15\ud55c \uc810\uc131\uc744 \ub098\ud0c0\ub0c8\ub2e4. \ucda9\ubd84\ud55c \uc9c4\ud759\uc774 \uc720\uc815\uc744 \ub36e\uac8c \ub418\uba74 \uc601\uad6c\uc801\uc73c\ub85c \ubd84\ucd9c\uc744 \uba48\ucd94\uac8c \ud558\ub294 \uac83\uc774\uc5c8\ub2e4. \ubc14\ub2e4 1,500\ubbf8\ud130 \uc544\ub798 BP\ub294 \uac70\ub300\ud55c \uace0\ucca0 \ucee8\ud14c\uc774\ub108\ub97c \uac16\ub2e4\ub193\uc558\ub2e4. \uc4f0\ub808\uae30\ub4e4\uc740 \ud30c\uc774\ud504\ub97c \ub530\ub77c BOP\ub97c \uac70\uccd0 \uc720\uc815 \uc544\ub798\ub85c \ub0b4\ub824\uac00\uac8c \ub41c\ub2e4. \ub4a4\uc774\uc5b4 \uc9c4\ud759\uc774 \ubc14\ub2e4\uc704 \ubc30\ub85c\ubd80\ud130 \uacf5\uae09\ub41c\ub2e4. \ucc98\uc74c\uc5d0\ub294 \uc791\uc5c5\uc774 \uc798 \uc9c4\ud589\ub418\ub294 \uac83\uc73c\ub85c \ubcf4\uc600\ub2e4. \ud558\uc9c0\ub9cc \ud604\uc2e4\uc740 \ub2ec\ub790\ub2e4. \uc4f0\ub808\uae30\uc5d0 \uac00\ud574\uc9c0\ub294 \uc555\ub825\uc774 \ub108\ubb34 \uac15\ud588\ub358 \uac83\uc774\ub2e4. \uc774\ud2c0\ub3d9\uc548 BP\ub294 5\ubc88\uc774\ub098 \uc4f0\ub808\uae30\uc640 \uc9c4\ud759\uc744 \ub0b4\ubcf4\ub0c8\ub2e4. \uc9c4\ud759\uc744 \ub0b4\ubcf4\ub0bc\ub54c\ub9c8\ub2e4 \uc6d0\uc720\uc640 \ud568\uaed8 \ubaa8\ub4e0\uac83\uc774 \ub2e4\uc2dc \ubc00\ub824\uc62c\ub77c\uc654\ub2e4. \ud0d1 \ud0ac\uc774 \uc2e4\ud328\ub77c\ub294 \uac83\uc774 \uc99d\uba85\ub420\ub54c\uae4c\uc9c0 445\ub9cc\ub9ac\ud130\uc758 \uc9c4\ud759\uc774 \ud22c\uc785\ub410\ub2e4. \ub354 \uace0\uc555\uc73c\ub85c \ud0d1 \ud0ac\uc744 \uc2dc\ub3c4\ud558\uba74 \uc6d0\uc720\uac00 \ud574\uc800\ub85c \ud758\ub7ec\ub4e4\uc5b4\uac00\uba74\uc11c \uc2dc\ucd94\uad6c \uc678\uc758 \uad6c\uc5ed\uc5d0\uc11c \ud130\uc9c8\uc9c0\ub3c4 \ubab0\ub77c\uc11c \uc704\ud5d8\ud558\ub2e4\uace0 \ud310\ub2e8\ud558\uc5ec \ub354\uc774\uc0c1 \uc2dc\ub3c4\ud558\uc9c0 \uc54a\uc558\ub2e4.", "sentence_answer": "\ub2e4\uc74c\uc740 \ubc29\ub300\ud55c \uc591\uc758 \uc9c4\ud759\uc744 \uc774\uc6a9\ud558\ub294 \uac83\uc73c\ub85c \uc77c\ubc18\uc801\uc778 \uc9c4\ud759\uc774 \uc544\ub2cc \uc2dc\ucd94\ud55c \uc9c4\ud759\uc744 \ubb3c\uacfc \ubbf8\ub124\ub784, \uc911\uc815\uc11d \ub4f1\uc73c\ub85c \ud63c\ud569\ud55c \uac83\uc73c\ub85c \uc11d\uc720\ubcf4\ub2e4 \uac15\ud55c \uc810\uc131\uc744 \ub098\ud0c0\ub0c8\ub2e4.", "paragraph_id": "6513252-6-2", "paragraph_question": "question: \ud0d1 \ud0ac\uc744 \uc704\ud574 \uc0ac\uc6a9\ub41c \uc9c4\ud759\uc744 \ub9cc\ub4e4\uae30 \uc704\ud574 \ubb3c\uacfc \ubbf8\ub124\ub784\uacfc \ud568\uaed8 \uc0ac\uc6a9\ub41c \uc7ac\ub8cc\ub294?, context: 5\uc6d4 11\uc77c\uc5d0 \uc880 \ub354 \uc791\uc740 \ub69c\uaed1\uc744 \uc2ec\ud574\ub85c \ub0b4\ub824\ubcf4\ub0b4\ub294 \uc2dc\ub3c4\uac00 \uc774\ub8e8\uc5b4\uc84c\uc73c\ub098, \uc774 \uc5ed\uc2dc \ud6a8\uacfc\ub97c \ubcf4\uc9c0 \ubabb\ud588\ub2e4. 5\uc6d4 14\uc77c\uc5d0 \ubd80\ub7ec\uc9c4 \ud30c\uc774\ud504\uc5d0 \uc791\uc740 \ud30c\uc774\ud504\ub97c \uaf42\uc544\uc11c \uc720\ucd9c\ub418\ub294 \uac00\uc2a4\ub97c \ubf51\uc544\ub0b4\uae30 \uc2dc\uc791\ud55c\ub2e4. \uc720\ucd9c\ub7c9\uc744 \uc904\uc774\uae30 \uc704\ud55c \uc2dc\ub3c4\uc774\uba70, \uc720\ucd9c\uad6c\uba4d\uc744 \uc644\uc804\ud788 \ub36e\uc5b4\uc11c \uc7a0\uadf8\ub294 \uc2dc\ub3c4\ub97c \ud560\ub54c\uae4c\uc9c0 \ud558\ub8e8 1\ucc9c~5\ucc9c \ubc30\ub7f4\uc529 \ube7c\ub0c8\uc5c8\ub2e4. 5\uc6d4 25\uc77c\uc5d0 \ubb34\uac70\uc6b4 \ub4dc\ub9b4\ub9c1 \uc9c4\ud759\uc744 \ud38c\ud504\ub85c \uc720\ucd9c\uad6c\uba4d\uc5d0 \ub123\uc5b4 \uc720\ucd9c \uc555\ub825\uc744 \uc904\uc774\uace0, \uc2dc\uba58\ud2b8\ub85c \ubc1c\ub77c\uc11c \ucd5c\uc885\uc801\uc73c\ub85c \uad6c\uba4d\uc744 \ub9c9\uc544\ubc84\ub9ac\ub294 \"\ud0d1 \ud0ac\"\uc774\ub77c\uace0 \uc54c\ub824\uc9c4 \ubc29\uc2dd\uc744 \uc2dc\ub3c4\ud55c\ub2e4. \uc77c\ubc18 \uac00\uc815\uc5d0\uc11c\uc758 \uc4f0\ub808\uae30\ub97c \uc774\uc6a9\ud558\ub294 \uac83\uc774\uc5c8\ub2e4. \ub2e8\ub2e8\ud55c \ubb3c\uc9c8\uacfc \ubd80\ub4dc\ub7ec\uc6b4 \ubb3c\uc9c8\uc758 \uacb0\ud569\uc774 \uad6c\uba4d\uc744 \ucc28\ub2e8\ud558\ub294\ub370 \ud6a8\uacfc\uac00 \uc788\uc744\uac83\uc73c\ub85c \uae30\ub300\ud588\ub2e4. \ub2e4\uc74c\uc740 \ubc29\ub300\ud55c \uc591\uc758 \uc9c4\ud759\uc744 \uc774\uc6a9\ud558\ub294 \uac83\uc73c\ub85c \uc77c\ubc18\uc801\uc778 \uc9c4\ud759\uc774 \uc544\ub2cc \uc2dc\ucd94\ud55c \uc9c4\ud759\uc744 \ubb3c\uacfc \ubbf8\ub124\ub784, \uc911\uc815\uc11d \ub4f1\uc73c\ub85c \ud63c\ud569\ud55c \uac83\uc73c\ub85c \uc11d\uc720\ubcf4\ub2e4 \uac15\ud55c \uc810\uc131\uc744 \ub098\ud0c0\ub0c8\ub2e4. \ucda9\ubd84\ud55c \uc9c4\ud759\uc774 \uc720\uc815\uc744 \ub36e\uac8c \ub418\uba74 \uc601\uad6c\uc801\uc73c\ub85c \ubd84\ucd9c\uc744 \uba48\ucd94\uac8c \ud558\ub294 \uac83\uc774\uc5c8\ub2e4. \ubc14\ub2e4 1,500\ubbf8\ud130 \uc544\ub798 BP\ub294 \uac70\ub300\ud55c \uace0\ucca0 \ucee8\ud14c\uc774\ub108\ub97c \uac16\ub2e4\ub193\uc558\ub2e4. \uc4f0\ub808\uae30\ub4e4\uc740 \ud30c\uc774\ud504\ub97c \ub530\ub77c BOP\ub97c \uac70\uccd0 \uc720\uc815 \uc544\ub798\ub85c \ub0b4\ub824\uac00\uac8c \ub41c\ub2e4. \ub4a4\uc774\uc5b4 \uc9c4\ud759\uc774 \ubc14\ub2e4\uc704 \ubc30\ub85c\ubd80\ud130 \uacf5\uae09\ub41c\ub2e4. \ucc98\uc74c\uc5d0\ub294 \uc791\uc5c5\uc774 \uc798 \uc9c4\ud589\ub418\ub294 \uac83\uc73c\ub85c \ubcf4\uc600\ub2e4. \ud558\uc9c0\ub9cc \ud604\uc2e4\uc740 \ub2ec\ub790\ub2e4. \uc4f0\ub808\uae30\uc5d0 \uac00\ud574\uc9c0\ub294 \uc555\ub825\uc774 \ub108\ubb34 \uac15\ud588\ub358 \uac83\uc774\ub2e4. \uc774\ud2c0\ub3d9\uc548 BP\ub294 5\ubc88\uc774\ub098 \uc4f0\ub808\uae30\uc640 \uc9c4\ud759\uc744 \ub0b4\ubcf4\ub0c8\ub2e4. \uc9c4\ud759\uc744 \ub0b4\ubcf4\ub0bc\ub54c\ub9c8\ub2e4 \uc6d0\uc720\uc640 \ud568\uaed8 \ubaa8\ub4e0\uac83\uc774 \ub2e4\uc2dc \ubc00\ub824\uc62c\ub77c\uc654\ub2e4. \ud0d1 \ud0ac\uc774 \uc2e4\ud328\ub77c\ub294 \uac83\uc774 \uc99d\uba85\ub420\ub54c\uae4c\uc9c0 445\ub9cc\ub9ac\ud130\uc758 \uc9c4\ud759\uc774 \ud22c\uc785\ub410\ub2e4. \ub354 \uace0\uc555\uc73c\ub85c \ud0d1 \ud0ac\uc744 \uc2dc\ub3c4\ud558\uba74 \uc6d0\uc720\uac00 \ud574\uc800\ub85c \ud758\ub7ec\ub4e4\uc5b4\uac00\uba74\uc11c \uc2dc\ucd94\uad6c \uc678\uc758 \uad6c\uc5ed\uc5d0\uc11c \ud130\uc9c8\uc9c0\ub3c4 \ubab0\ub77c\uc11c \uc704\ud5d8\ud558\ub2e4\uace0 \ud310\ub2e8\ud558\uc5ec \ub354\uc774\uc0c1 \uc2dc\ub3c4\ud558\uc9c0 \uc54a\uc558\ub2e4."} {"question": "\ucd5c\uc9c4\uc2e4\uc758 \ucd94\ucc9c\uc73c\ub85c \ud1b1\uc2a4\ud0c0\uac00 \ub41c \ub300\ud45c\uc801\uc778 \uc778\ubb3c\uc740?", "paragraph": "\ubc29\uc1a1\uc778 \uc720\uc7ac\uc11d\uc740 \ucd5c\uc9c4\uc2e4\uc758 \ucd94\ucc9c\uc73c\ub85c \ud1b1\uc2a4\ud0c0\uac00 \ub41c \ub300\ud45c\uc801\uc778 \uc608\uc774\ub2e4. 2000\ub144, \ucd5c\uc9c4\uc2e4\uc740 MBC\uc758 \ud55c \uace0\uc704\uae09 \uc5f0\ucd9c\uac00\uc5d0\uac8c \uc720\uc7ac\uc11d\uc744 MBC \u300a\ubaa9\ud45c \ub2ec\uc131 \ud1a0\uc694\uc77c - \uc2a4\ud0c0 \uc11c\ubc14\uc774\ubc8c \ub3d9\uac70\ub3d9\ub77d\u300b\uc758 \uba54\uc778 MC\ub85c \ucd94\ucc9c\ud558\uc600\ub2e4. \ub2f9\uc2dc \ud06c\uac8c \uc720\uba85\ud558\uc9c0 \uc54a\uc558\ub358 \uc720\uc7ac\uc11d\uc740 \ucd5c\uc9c4\uc2e4\uacfc\ub294 \uce5c\ubd84\uc774 \uc5c6\uc5c8\ub2e4. \ucd5c\uc9c4\uc2e4\uc740 \uc720\uc7ac\uc11d\uc758 \ubcc4\uba85\uc778 '\uba54\ub69c\uae30'\ub294 \uc54c\uc544\ub3c4 \uc2e4\uba85\uc740 \uc54c\uc9c0 \ubabb\ud588\uc9c0\ub9cc KBS \u300a\uc11c\uc138\uc6d0 \uc1fc\u300b\uc5d0 \ucd9c\uc5f0\ud588\ub358 \uc720\uc7ac\uc11d\uc744 \uae30\uc5b5\ud558\uace0 \uc788\uc5c8\uace0, \ucd5c\uc9c4\uc2e4\uc774 \uadf8 \uace0\uc704\uae09 \uc5f0\ucd9c\uac00\uc5d0\uac8c \"\uc0c8 \ud504\ub85c\uadf8\ub7a8\uc774 \uc788\uc73c\uba74 '\uba54\ub69c\uae30'\ub97c \uc4f0\ub77c\"\ub77c\uace0 \uac15\ub825\ud558\uac8c \ucd94\ucc9c\ud558\uba74\uc11c \u300a\uc2a4\ud0c0 \uc11c\ubc14\uc774\ubc8c \ub3d9\uac70\ub3d9\ub77d\u300b\uc758 \uba54\uc778 MC\ub85c \uc720\uc7ac\uc11d\uc774 \ubc1c\ud0c1\ub418\uc5c8\ub2e4. \uc720\uc7ac\uc11d\uc740 \u300a\uc2a4\ud0c0 \uc11c\ubc14\uc774\ubc8c \ub3d9\uace0\ub3d9\ub77d\u300b\uc758 \uba54\uc778 MC\ub85c \ud65c\ub3d9\ud558\uba74\uc11c \ub300\uc911\uc73c\ub85c\ubd80\ud130 \ub9ce\uc740 \uc8fc\ubaa9\uc744 \ubc1b\uc558\uace0 \ud1b1\uc2a4\ud0c0\ub85c \uc778\uc815\ubc1b\ub294 \uc804\uae30\ub97c \ub9c8\ub828\ud558\uc600\ub2e4. \uc720\uc7ac\uc11d\uc740 \uadf8 \ud6c4 \ud55c\ucc38\uc774 \uc9c0\ub098\uc11c\uc57c \ucd5c\uc9c4\uc2e4\uc774 \uc790\uc2e0\uc744 \ucd94\ucc9c\ud588\ub2e4\ub294 \uc0ac\uc2e4\uc744 \uc54c\uac8c \ub418\uc5c8\ub2e4.", "answer": "\uc720\uc7ac\uc11d", "sentence": "\ubc29\uc1a1\uc778 \uc720\uc7ac\uc11d \uc740 \ucd5c\uc9c4\uc2e4\uc758 \ucd94\ucc9c\uc73c\ub85c \ud1b1\uc2a4\ud0c0\uac00 \ub41c \ub300\ud45c\uc801\uc778 \uc608\uc774\ub2e4.", "paragraph_sentence": " \ubc29\uc1a1\uc778 \uc720\uc7ac\uc11d \uc740 \ucd5c\uc9c4\uc2e4\uc758 \ucd94\ucc9c\uc73c\ub85c \ud1b1\uc2a4\ud0c0\uac00 \ub41c \ub300\ud45c\uc801\uc778 \uc608\uc774\ub2e4. 2000\ub144, \ucd5c\uc9c4\uc2e4\uc740 MBC\uc758 \ud55c \uace0\uc704\uae09 \uc5f0\ucd9c\uac00\uc5d0\uac8c \uc720\uc7ac\uc11d\uc744 MBC \u300a\ubaa9\ud45c \ub2ec\uc131 \ud1a0\uc694\uc77c - \uc2a4\ud0c0 \uc11c\ubc14\uc774\ubc8c \ub3d9\uac70\ub3d9\ub77d\u300b\uc758 \uba54\uc778 MC\ub85c \ucd94\ucc9c\ud558\uc600\ub2e4. \ub2f9\uc2dc \ud06c\uac8c \uc720\uba85\ud558\uc9c0 \uc54a\uc558\ub358 \uc720\uc7ac\uc11d\uc740 \ucd5c\uc9c4\uc2e4\uacfc\ub294 \uce5c\ubd84\uc774 \uc5c6\uc5c8\ub2e4. \ucd5c\uc9c4\uc2e4\uc740 \uc720\uc7ac\uc11d\uc758 \ubcc4\uba85\uc778 '\uba54\ub69c\uae30'\ub294 \uc54c\uc544\ub3c4 \uc2e4\uba85\uc740 \uc54c\uc9c0 \ubabb\ud588\uc9c0\ub9cc KBS \u300a\uc11c\uc138\uc6d0 \uc1fc\u300b\uc5d0 \ucd9c\uc5f0\ud588\ub358 \uc720\uc7ac\uc11d\uc744 \uae30\uc5b5\ud558\uace0 \uc788\uc5c8\uace0, \ucd5c\uc9c4\uc2e4\uc774 \uadf8 \uace0\uc704\uae09 \uc5f0\ucd9c\uac00\uc5d0\uac8c \"\uc0c8 \ud504\ub85c\uadf8\ub7a8\uc774 \uc788\uc73c\uba74 '\uba54\ub69c\uae30'\ub97c \uc4f0\ub77c\"\ub77c\uace0 \uac15\ub825\ud558\uac8c \ucd94\ucc9c\ud558\uba74\uc11c \u300a\uc2a4\ud0c0 \uc11c\ubc14\uc774\ubc8c \ub3d9\uac70\ub3d9\ub77d\u300b\uc758 \uba54\uc778 MC\ub85c \uc720\uc7ac\uc11d\uc774 \ubc1c\ud0c1\ub418\uc5c8\ub2e4. \uc720\uc7ac\uc11d\uc740 \u300a\uc2a4\ud0c0 \uc11c\ubc14\uc774\ubc8c \ub3d9\uace0\ub3d9\ub77d\u300b\uc758 \uba54\uc778 MC\ub85c \ud65c\ub3d9\ud558\uba74\uc11c \ub300\uc911\uc73c\ub85c\ubd80\ud130 \ub9ce\uc740 \uc8fc\ubaa9\uc744 \ubc1b\uc558\uace0 \ud1b1\uc2a4\ud0c0\ub85c \uc778\uc815\ubc1b\ub294 \uc804\uae30\ub97c \ub9c8\ub828\ud558\uc600\ub2e4. \uc720\uc7ac\uc11d\uc740 \uadf8 \ud6c4 \ud55c\ucc38\uc774 \uc9c0\ub098\uc11c\uc57c \ucd5c\uc9c4\uc2e4\uc774 \uc790\uc2e0\uc744 \ucd94\ucc9c\ud588\ub2e4\ub294 \uc0ac\uc2e4\uc744 \uc54c\uac8c \ub418\uc5c8\ub2e4.", "paragraph_answer": "\ubc29\uc1a1\uc778 \uc720\uc7ac\uc11d \uc740 \ucd5c\uc9c4\uc2e4\uc758 \ucd94\ucc9c\uc73c\ub85c \ud1b1\uc2a4\ud0c0\uac00 \ub41c \ub300\ud45c\uc801\uc778 \uc608\uc774\ub2e4. 2000\ub144, \ucd5c\uc9c4\uc2e4\uc740 MBC\uc758 \ud55c \uace0\uc704\uae09 \uc5f0\ucd9c\uac00\uc5d0\uac8c \uc720\uc7ac\uc11d\uc744 MBC \u300a\ubaa9\ud45c \ub2ec\uc131 \ud1a0\uc694\uc77c - \uc2a4\ud0c0 \uc11c\ubc14\uc774\ubc8c \ub3d9\uac70\ub3d9\ub77d\u300b\uc758 \uba54\uc778 MC\ub85c \ucd94\ucc9c\ud558\uc600\ub2e4. \ub2f9\uc2dc \ud06c\uac8c \uc720\uba85\ud558\uc9c0 \uc54a\uc558\ub358 \uc720\uc7ac\uc11d\uc740 \ucd5c\uc9c4\uc2e4\uacfc\ub294 \uce5c\ubd84\uc774 \uc5c6\uc5c8\ub2e4. \ucd5c\uc9c4\uc2e4\uc740 \uc720\uc7ac\uc11d\uc758 \ubcc4\uba85\uc778 '\uba54\ub69c\uae30'\ub294 \uc54c\uc544\ub3c4 \uc2e4\uba85\uc740 \uc54c\uc9c0 \ubabb\ud588\uc9c0\ub9cc KBS \u300a\uc11c\uc138\uc6d0 \uc1fc\u300b\uc5d0 \ucd9c\uc5f0\ud588\ub358 \uc720\uc7ac\uc11d\uc744 \uae30\uc5b5\ud558\uace0 \uc788\uc5c8\uace0, \ucd5c\uc9c4\uc2e4\uc774 \uadf8 \uace0\uc704\uae09 \uc5f0\ucd9c\uac00\uc5d0\uac8c \"\uc0c8 \ud504\ub85c\uadf8\ub7a8\uc774 \uc788\uc73c\uba74 '\uba54\ub69c\uae30'\ub97c \uc4f0\ub77c\"\ub77c\uace0 \uac15\ub825\ud558\uac8c \ucd94\ucc9c\ud558\uba74\uc11c \u300a\uc2a4\ud0c0 \uc11c\ubc14\uc774\ubc8c \ub3d9\uac70\ub3d9\ub77d\u300b\uc758 \uba54\uc778 MC\ub85c \uc720\uc7ac\uc11d\uc774 \ubc1c\ud0c1\ub418\uc5c8\ub2e4. \uc720\uc7ac\uc11d\uc740 \u300a\uc2a4\ud0c0 \uc11c\ubc14\uc774\ubc8c \ub3d9\uace0\ub3d9\ub77d\u300b\uc758 \uba54\uc778 MC\ub85c \ud65c\ub3d9\ud558\uba74\uc11c \ub300\uc911\uc73c\ub85c\ubd80\ud130 \ub9ce\uc740 \uc8fc\ubaa9\uc744 \ubc1b\uc558\uace0 \ud1b1\uc2a4\ud0c0\ub85c \uc778\uc815\ubc1b\ub294 \uc804\uae30\ub97c \ub9c8\ub828\ud558\uc600\ub2e4. \uc720\uc7ac\uc11d\uc740 \uadf8 \ud6c4 \ud55c\ucc38\uc774 \uc9c0\ub098\uc11c\uc57c \ucd5c\uc9c4\uc2e4\uc774 \uc790\uc2e0\uc744 \ucd94\ucc9c\ud588\ub2e4\ub294 \uc0ac\uc2e4\uc744 \uc54c\uac8c \ub418\uc5c8\ub2e4.", "sentence_answer": "\ubc29\uc1a1\uc778 \uc720\uc7ac\uc11d \uc740 \ucd5c\uc9c4\uc2e4\uc758 \ucd94\ucc9c\uc73c\ub85c \ud1b1\uc2a4\ud0c0\uac00 \ub41c \ub300\ud45c\uc801\uc778 \uc608\uc774\ub2e4.", "paragraph_id": "6560193-5-0", "paragraph_question": "question: \ucd5c\uc9c4\uc2e4\uc758 \ucd94\ucc9c\uc73c\ub85c \ud1b1\uc2a4\ud0c0\uac00 \ub41c \ub300\ud45c\uc801\uc778 \uc778\ubb3c\uc740?, context: \ubc29\uc1a1\uc778 \uc720\uc7ac\uc11d\uc740 \ucd5c\uc9c4\uc2e4\uc758 \ucd94\ucc9c\uc73c\ub85c \ud1b1\uc2a4\ud0c0\uac00 \ub41c \ub300\ud45c\uc801\uc778 \uc608\uc774\ub2e4. 2000\ub144, \ucd5c\uc9c4\uc2e4\uc740 MBC\uc758 \ud55c \uace0\uc704\uae09 \uc5f0\ucd9c\uac00\uc5d0\uac8c \uc720\uc7ac\uc11d\uc744 MBC \u300a\ubaa9\ud45c \ub2ec\uc131 \ud1a0\uc694\uc77c - \uc2a4\ud0c0 \uc11c\ubc14\uc774\ubc8c \ub3d9\uac70\ub3d9\ub77d\u300b\uc758 \uba54\uc778 MC\ub85c \ucd94\ucc9c\ud558\uc600\ub2e4. \ub2f9\uc2dc \ud06c\uac8c \uc720\uba85\ud558\uc9c0 \uc54a\uc558\ub358 \uc720\uc7ac\uc11d\uc740 \ucd5c\uc9c4\uc2e4\uacfc\ub294 \uce5c\ubd84\uc774 \uc5c6\uc5c8\ub2e4. \ucd5c\uc9c4\uc2e4\uc740 \uc720\uc7ac\uc11d\uc758 \ubcc4\uba85\uc778 '\uba54\ub69c\uae30'\ub294 \uc54c\uc544\ub3c4 \uc2e4\uba85\uc740 \uc54c\uc9c0 \ubabb\ud588\uc9c0\ub9cc KBS \u300a\uc11c\uc138\uc6d0 \uc1fc\u300b\uc5d0 \ucd9c\uc5f0\ud588\ub358 \uc720\uc7ac\uc11d\uc744 \uae30\uc5b5\ud558\uace0 \uc788\uc5c8\uace0, \ucd5c\uc9c4\uc2e4\uc774 \uadf8 \uace0\uc704\uae09 \uc5f0\ucd9c\uac00\uc5d0\uac8c \"\uc0c8 \ud504\ub85c\uadf8\ub7a8\uc774 \uc788\uc73c\uba74 '\uba54\ub69c\uae30'\ub97c \uc4f0\ub77c\"\ub77c\uace0 \uac15\ub825\ud558\uac8c \ucd94\ucc9c\ud558\uba74\uc11c \u300a\uc2a4\ud0c0 \uc11c\ubc14\uc774\ubc8c \ub3d9\uac70\ub3d9\ub77d\u300b\uc758 \uba54\uc778 MC\ub85c \uc720\uc7ac\uc11d\uc774 \ubc1c\ud0c1\ub418\uc5c8\ub2e4. \uc720\uc7ac\uc11d\uc740 \u300a\uc2a4\ud0c0 \uc11c\ubc14\uc774\ubc8c \ub3d9\uace0\ub3d9\ub77d\u300b\uc758 \uba54\uc778 MC\ub85c \ud65c\ub3d9\ud558\uba74\uc11c \ub300\uc911\uc73c\ub85c\ubd80\ud130 \ub9ce\uc740 \uc8fc\ubaa9\uc744 \ubc1b\uc558\uace0 \ud1b1\uc2a4\ud0c0\ub85c \uc778\uc815\ubc1b\ub294 \uc804\uae30\ub97c \ub9c8\ub828\ud558\uc600\ub2e4. \uc720\uc7ac\uc11d\uc740 \uadf8 \ud6c4 \ud55c\ucc38\uc774 \uc9c0\ub098\uc11c\uc57c \ucd5c\uc9c4\uc2e4\uc774 \uc790\uc2e0\uc744 \ucd94\ucc9c\ud588\ub2e4\ub294 \uc0ac\uc2e4\uc744 \uc54c\uac8c \ub418\uc5c8\ub2e4."} {"question": "\uc6d0\uc791\uc758 \uacf5\uc800\uc790\uc774\uc790 \uadf8\ub9bc \uc791\uac00\ub294 \ub204\uad6c\uc778\uac00?", "paragraph": "\ubb34\uc5b4\ub294 \uc790\uc2e0\uc758 \ub9cc\ud654\ub97c \uc601\ud654\ud654\ud560 \ub54c \ub9c8\ub2e4\uc758 \ubd84\uc7c1\uc740 \ubb3c\ub860, \uc790\uc2e0\uc774 \uc601\ud654\uc758 \uac01\ubcf8 \ubc0f \uc5f0\ucd9c\uc5d0 \ucc38\uc5ec\ud558\uc9c0 \ubabb\ud588\uae30 \ub54c\ubb38\uc5d0 \ub178\uace8\uc801\uc73c\ub85c \uc601\ud654\uc640 \uad00\uacc4\ub97c \ub04a\uc5c8\ub2e4. \uadf8\ub294 \uc18c\uc18d \ucd9c\ud310\uc0ac\uc778 DC \ucf54\ubbf9\uc2a4\uc758 \ubaa8\ud68c\uc0ac \uc6cc\ub108 \ube0c\ub77c\ub354\uc2a4\uac00 \ubb34\uc5b4\uac00 \uc601\ud654\ub97c \ucd94\ucc9c\ud588\ub2e4\ub294 \ubc1c\uc5b8\uc744 \ucca0\ud68c\ud558\uc9c0 \uc54a\uc790 DC \ucf54\ubbf9\uc2a4\uc640\uc758 \ud611\ub3d9 \uc791\uc5c5\uc744 \ub05d\ub0b4 \ubc84\ub838\ub2e4. \ubb34\uc5b4\ub294 \uac01\ubcf8\uc758 \ud50c\ub86f\uc5d0\ub294 \uad6c\uba4d\uc774 \ub098 \uc788\uace0 \uc6d0\uc791\uc758 \ud14c\ub9c8\ub85c \uc591 \uadf9\ub2e8\uc758 \uc815\uce58 \uc774\ub370\uc62c\ub85c\uae30(\ud30c\uc2dc\uc998\uacfc \uc544\ub098\ud0a4\uc998)\ub97c \uc124\uc815\ud588\ub294\ub370\uadf8\uac83\uc744 \"\uc694\uc0c8 \uc720\ud589\ud558\ub294 \ubbf8\uad6d\uc2dd \uc2e0\ubcf4\uc218\uc8fc\uc758 vs. \ubbf8\uad6d\uc2dd \uc790\uc720\uc8fc\uc758\"\ub85c \uac1c\uc870\ud558\uace0 \ub9d0\uc558\ub2e4\uace0 \uc8fc\uc7a5\ud588\ub2e4. \ubb34\uc5b4 \ubcf8\uc778\uc758 \ud76c\ub9dd\uc5d0 \ub530\ub77c, \ubb34\uc5b4\uc758 \uc774\ub984\uc740 \uc601\ud654\uc758 \uc5d4\ub529 \ud06c\ub808\ub527\uc5d0 \ubcf4\uc774\uc9c0 \uc54a\ub294\ub2e4. \ud558\uc9c0\ub9cc \uacf5\uc800\uc790\uc774\uc790 \uadf8\ub9bc \uc791\uac00 \ub370\uc774\ube44\ub4dc \ub85c\uc774\ub4dc\ub294 \uc601\ud654\ud654\uc5d0 \ucc2c\uc131\ud558\uba74\uc11c, \uac01\ubcf8\uc774 \ub9e4\uc6b0 \uc88b\uc73c\uba70, \ubb34\uc5b4\ub294 \ucc45\uc758 \ub0b4\uc6a9 \uc804\uccb4\uac00 \uc601\ud654\uac00 \ub418\uc5b4\uc57c\ub9cc \ub9cc\uc871\ud560 \uac83\uc774\ub77c\uace0 \ud588\ub2e4.", "answer": "\ub370\uc774\ube44\ub4dc \ub85c\uc774\ub4dc", "sentence": "\ud558\uc9c0\ub9cc \uacf5\uc800\uc790\uc774\uc790 \uadf8\ub9bc \uc791\uac00 \ub370\uc774\ube44\ub4dc \ub85c\uc774\ub4dc \ub294 \uc601\ud654\ud654\uc5d0 \ucc2c\uc131\ud558\uba74\uc11c, \uac01\ubcf8\uc774 \ub9e4\uc6b0 \uc88b\uc73c\uba70, \ubb34\uc5b4\ub294 \ucc45\uc758 \ub0b4\uc6a9 \uc804\uccb4\uac00 \uc601\ud654\uac00 \ub418\uc5b4\uc57c\ub9cc \ub9cc\uc871\ud560 \uac83\uc774\ub77c\uace0 \ud588\ub2e4.", "paragraph_sentence": "\ubb34\uc5b4\ub294 \uc790\uc2e0\uc758 \ub9cc\ud654\ub97c \uc601\ud654\ud654\ud560 \ub54c \ub9c8\ub2e4\uc758 \ubd84\uc7c1\uc740 \ubb3c\ub860, \uc790\uc2e0\uc774 \uc601\ud654\uc758 \uac01\ubcf8 \ubc0f \uc5f0\ucd9c\uc5d0 \ucc38\uc5ec\ud558\uc9c0 \ubabb\ud588\uae30 \ub54c\ubb38\uc5d0 \ub178\uace8\uc801\uc73c\ub85c \uc601\ud654\uc640 \uad00\uacc4\ub97c \ub04a\uc5c8\ub2e4. \uadf8\ub294 \uc18c\uc18d \ucd9c\ud310\uc0ac\uc778 DC \ucf54\ubbf9\uc2a4\uc758 \ubaa8\ud68c\uc0ac \uc6cc\ub108 \ube0c\ub77c\ub354\uc2a4\uac00 \ubb34\uc5b4\uac00 \uc601\ud654\ub97c \ucd94\ucc9c\ud588\ub2e4\ub294 \ubc1c\uc5b8\uc744 \ucca0\ud68c\ud558\uc9c0 \uc54a\uc790 DC \ucf54\ubbf9\uc2a4\uc640\uc758 \ud611\ub3d9 \uc791\uc5c5\uc744 \ub05d\ub0b4 \ubc84\ub838\ub2e4. \ubb34\uc5b4\ub294 \uac01\ubcf8\uc758 \ud50c\ub86f\uc5d0\ub294 \uad6c\uba4d\uc774 \ub098 \uc788\uace0 \uc6d0\uc791\uc758 \ud14c\ub9c8\ub85c \uc591 \uadf9\ub2e8\uc758 \uc815\uce58 \uc774\ub370\uc62c\ub85c\uae30(\ud30c\uc2dc\uc998\uacfc \uc544\ub098\ud0a4\uc998)\ub97c \uc124\uc815\ud588\ub294\ub370\uadf8\uac83\uc744 \"\uc694\uc0c8 \uc720\ud589\ud558\ub294 \ubbf8\uad6d\uc2dd \uc2e0\ubcf4\uc218\uc8fc\uc758 vs. \ubbf8\uad6d\uc2dd \uc790\uc720\uc8fc\uc758\"\ub85c \uac1c\uc870\ud558\uace0 \ub9d0\uc558\ub2e4\uace0 \uc8fc\uc7a5\ud588\ub2e4. \ubb34\uc5b4 \ubcf8\uc778\uc758 \ud76c\ub9dd\uc5d0 \ub530\ub77c, \ubb34\uc5b4\uc758 \uc774\ub984\uc740 \uc601\ud654\uc758 \uc5d4\ub529 \ud06c\ub808\ub527\uc5d0 \ubcf4\uc774\uc9c0 \uc54a\ub294\ub2e4. \ud558\uc9c0\ub9cc \uacf5\uc800\uc790\uc774\uc790 \uadf8\ub9bc \uc791\uac00 \ub370\uc774\ube44\ub4dc \ub85c\uc774\ub4dc \ub294 \uc601\ud654\ud654\uc5d0 \ucc2c\uc131\ud558\uba74\uc11c, \uac01\ubcf8\uc774 \ub9e4\uc6b0 \uc88b\uc73c\uba70, \ubb34\uc5b4\ub294 \ucc45\uc758 \ub0b4\uc6a9 \uc804\uccb4\uac00 \uc601\ud654\uac00 \ub418\uc5b4\uc57c\ub9cc \ub9cc\uc871\ud560 \uac83\uc774\ub77c\uace0 \ud588\ub2e4. ", "paragraph_answer": "\ubb34\uc5b4\ub294 \uc790\uc2e0\uc758 \ub9cc\ud654\ub97c \uc601\ud654\ud654\ud560 \ub54c \ub9c8\ub2e4\uc758 \ubd84\uc7c1\uc740 \ubb3c\ub860, \uc790\uc2e0\uc774 \uc601\ud654\uc758 \uac01\ubcf8 \ubc0f \uc5f0\ucd9c\uc5d0 \ucc38\uc5ec\ud558\uc9c0 \ubabb\ud588\uae30 \ub54c\ubb38\uc5d0 \ub178\uace8\uc801\uc73c\ub85c \uc601\ud654\uc640 \uad00\uacc4\ub97c \ub04a\uc5c8\ub2e4. \uadf8\ub294 \uc18c\uc18d \ucd9c\ud310\uc0ac\uc778 DC \ucf54\ubbf9\uc2a4\uc758 \ubaa8\ud68c\uc0ac \uc6cc\ub108 \ube0c\ub77c\ub354\uc2a4\uac00 \ubb34\uc5b4\uac00 \uc601\ud654\ub97c \ucd94\ucc9c\ud588\ub2e4\ub294 \ubc1c\uc5b8\uc744 \ucca0\ud68c\ud558\uc9c0 \uc54a\uc790 DC \ucf54\ubbf9\uc2a4\uc640\uc758 \ud611\ub3d9 \uc791\uc5c5\uc744 \ub05d\ub0b4 \ubc84\ub838\ub2e4. \ubb34\uc5b4\ub294 \uac01\ubcf8\uc758 \ud50c\ub86f\uc5d0\ub294 \uad6c\uba4d\uc774 \ub098 \uc788\uace0 \uc6d0\uc791\uc758 \ud14c\ub9c8\ub85c \uc591 \uadf9\ub2e8\uc758 \uc815\uce58 \uc774\ub370\uc62c\ub85c\uae30(\ud30c\uc2dc\uc998\uacfc \uc544\ub098\ud0a4\uc998)\ub97c \uc124\uc815\ud588\ub294\ub370\uadf8\uac83\uc744 \"\uc694\uc0c8 \uc720\ud589\ud558\ub294 \ubbf8\uad6d\uc2dd \uc2e0\ubcf4\uc218\uc8fc\uc758 vs. \ubbf8\uad6d\uc2dd \uc790\uc720\uc8fc\uc758\"\ub85c \uac1c\uc870\ud558\uace0 \ub9d0\uc558\ub2e4\uace0 \uc8fc\uc7a5\ud588\ub2e4. \ubb34\uc5b4 \ubcf8\uc778\uc758 \ud76c\ub9dd\uc5d0 \ub530\ub77c, \ubb34\uc5b4\uc758 \uc774\ub984\uc740 \uc601\ud654\uc758 \uc5d4\ub529 \ud06c\ub808\ub527\uc5d0 \ubcf4\uc774\uc9c0 \uc54a\ub294\ub2e4. \ud558\uc9c0\ub9cc \uacf5\uc800\uc790\uc774\uc790 \uadf8\ub9bc \uc791\uac00 \ub370\uc774\ube44\ub4dc \ub85c\uc774\ub4dc \ub294 \uc601\ud654\ud654\uc5d0 \ucc2c\uc131\ud558\uba74\uc11c, \uac01\ubcf8\uc774 \ub9e4\uc6b0 \uc88b\uc73c\uba70, \ubb34\uc5b4\ub294 \ucc45\uc758 \ub0b4\uc6a9 \uc804\uccb4\uac00 \uc601\ud654\uac00 \ub418\uc5b4\uc57c\ub9cc \ub9cc\uc871\ud560 \uac83\uc774\ub77c\uace0 \ud588\ub2e4.", "sentence_answer": "\ud558\uc9c0\ub9cc \uacf5\uc800\uc790\uc774\uc790 \uadf8\ub9bc \uc791\uac00 \ub370\uc774\ube44\ub4dc \ub85c\uc774\ub4dc \ub294 \uc601\ud654\ud654\uc5d0 \ucc2c\uc131\ud558\uba74\uc11c, \uac01\ubcf8\uc774 \ub9e4\uc6b0 \uc88b\uc73c\uba70, \ubb34\uc5b4\ub294 \ucc45\uc758 \ub0b4\uc6a9 \uc804\uccb4\uac00 \uc601\ud654\uac00 \ub418\uc5b4\uc57c\ub9cc \ub9cc\uc871\ud560 \uac83\uc774\ub77c\uace0 \ud588\ub2e4.", "paragraph_id": "6583521-6-1", "paragraph_question": "question: \uc6d0\uc791\uc758 \uacf5\uc800\uc790\uc774\uc790 \uadf8\ub9bc \uc791\uac00\ub294 \ub204\uad6c\uc778\uac00?, context: \ubb34\uc5b4\ub294 \uc790\uc2e0\uc758 \ub9cc\ud654\ub97c \uc601\ud654\ud654\ud560 \ub54c \ub9c8\ub2e4\uc758 \ubd84\uc7c1\uc740 \ubb3c\ub860, \uc790\uc2e0\uc774 \uc601\ud654\uc758 \uac01\ubcf8 \ubc0f \uc5f0\ucd9c\uc5d0 \ucc38\uc5ec\ud558\uc9c0 \ubabb\ud588\uae30 \ub54c\ubb38\uc5d0 \ub178\uace8\uc801\uc73c\ub85c \uc601\ud654\uc640 \uad00\uacc4\ub97c \ub04a\uc5c8\ub2e4. \uadf8\ub294 \uc18c\uc18d \ucd9c\ud310\uc0ac\uc778 DC \ucf54\ubbf9\uc2a4\uc758 \ubaa8\ud68c\uc0ac \uc6cc\ub108 \ube0c\ub77c\ub354\uc2a4\uac00 \ubb34\uc5b4\uac00 \uc601\ud654\ub97c \ucd94\ucc9c\ud588\ub2e4\ub294 \ubc1c\uc5b8\uc744 \ucca0\ud68c\ud558\uc9c0 \uc54a\uc790 DC \ucf54\ubbf9\uc2a4\uc640\uc758 \ud611\ub3d9 \uc791\uc5c5\uc744 \ub05d\ub0b4 \ubc84\ub838\ub2e4. \ubb34\uc5b4\ub294 \uac01\ubcf8\uc758 \ud50c\ub86f\uc5d0\ub294 \uad6c\uba4d\uc774 \ub098 \uc788\uace0 \uc6d0\uc791\uc758 \ud14c\ub9c8\ub85c \uc591 \uadf9\ub2e8\uc758 \uc815\uce58 \uc774\ub370\uc62c\ub85c\uae30(\ud30c\uc2dc\uc998\uacfc \uc544\ub098\ud0a4\uc998)\ub97c \uc124\uc815\ud588\ub294\ub370\uadf8\uac83\uc744 \"\uc694\uc0c8 \uc720\ud589\ud558\ub294 \ubbf8\uad6d\uc2dd \uc2e0\ubcf4\uc218\uc8fc\uc758 vs. \ubbf8\uad6d\uc2dd \uc790\uc720\uc8fc\uc758\"\ub85c \uac1c\uc870\ud558\uace0 \ub9d0\uc558\ub2e4\uace0 \uc8fc\uc7a5\ud588\ub2e4. \ubb34\uc5b4 \ubcf8\uc778\uc758 \ud76c\ub9dd\uc5d0 \ub530\ub77c, \ubb34\uc5b4\uc758 \uc774\ub984\uc740 \uc601\ud654\uc758 \uc5d4\ub529 \ud06c\ub808\ub527\uc5d0 \ubcf4\uc774\uc9c0 \uc54a\ub294\ub2e4. \ud558\uc9c0\ub9cc \uacf5\uc800\uc790\uc774\uc790 \uadf8\ub9bc \uc791\uac00 \ub370\uc774\ube44\ub4dc \ub85c\uc774\ub4dc\ub294 \uc601\ud654\ud654\uc5d0 \ucc2c\uc131\ud558\uba74\uc11c, \uac01\ubcf8\uc774 \ub9e4\uc6b0 \uc88b\uc73c\uba70, \ubb34\uc5b4\ub294 \ucc45\uc758 \ub0b4\uc6a9 \uc804\uccb4\uac00 \uc601\ud654\uac00 \ub418\uc5b4\uc57c\ub9cc \ub9cc\uc871\ud560 \uac83\uc774\ub77c\uace0 \ud588\ub2e4."} {"question": "\ube14\ub808\uc774\ud06c \ub77c\uc774\ube14\ub9ac\uac00 \ubc84\ubc45\ud06c \uace0\ub4f1\ud559\uad50\uc5d0\uc11c \ud68c\uc7a5\uc744 \ub9e1\uc558\ub358 \uadf8\ub8f9\uc740?", "paragraph": "\ub77c\uc774\ube14\ub9ac\uc758 \uc5b4\ub9b0 \uc2dc\uc808, \uadf8\ub140\uc758 \ubd80\ubaa8\ub2d8\uc740 \uadf8\ub140\ub97c \ubca0\uc774\ube44\uc2dc\ud130\uc640 \ud568\uaed8 \uc9d1\uc5d0 \ub0a8\uaca8\ub450\uace0 \uc2f6\uc9c0 \uc54a\uc558\uae30 \ub54c\ubb38\uc5d0 \ubd80\ubaa8\ub2d8\uacfc \ud568\uaed8 \uadf8\ub140\uc640 \ud568\uaed8 \ud589\ub3d9 \uac15\uc2b5\ud68c\uc5d0 \ub370\ub824\uac14\uc2b5\ub2c8\ub2e4. \ub77c\uc774\ube14\ub9ac\ub294 \uc5b4\ub838\uc744 \ub54c\ubd80\ud130 \uadf8\ub140\uc758 \ubd80\ubaa8\uac00 \uc5f0\uae30 \uc218\uc5c5\uc744 \uc9c0\ub3c4\ud558\ub294 \uac83\uc744 \ubcf4\uace0 \uc5f0\uae30\uc5d0 \ub300\ud574 \ubc30\uc6b0\uac8c \ub418\uc5c8\uace0, \ucc28\uce30 \uadf8\ub140\uac00 \ub098\uc774\uac00 \ub4e4\uba74\uc11c \uc5f0\uae30\ud65c\ub3d9\uc5d0 \ub300\ud55c \ud655\uc2e0\uc744 \uac16\ub294\ub370 \ub3c4\uc6c0\uc774 \ub418\uc5c8\ub2e4\uace0 \ud68c\uc0c1\ud558\uc600\ub2e4. \uadf8\ub140\ub294 \ub2f9\ucd08 \ucc98\uc74c\uc5d0\ub294 \uc5f0\uae30\uc5d0 \uad00\uc2ec\uc774 \uc5c6\uc5c8\uace0, \uc2a4\ud0e0\ud3ec\ub4dc \ub300\ud559\uc5d0 \uc785\ud559\ud558\uae30 \ud76c\ub9dd\ud558\uace0 \uc788\uc5c8\ub2e4. \uadf8\ub140\uac00 \uc911\ud559\uad50 1\ud559\ub144\uacfc \uace0\ub4f1\ud559\uad50 2\ud559\ub144 \uc0ac\uc774\uc758 \uc5ec\ub984\uae30\uac04 \ub3d9\uc548 \uadf8\ub140\uc758 \uc624\ube60 \uc5d0\ub9ad\uc740 \uc5d0\uc774\uc804\ud2b8\uc5d0 \uba87 \ub2ec \ub3d9\uc548 \uba87 \ucc28\ub840\uc758 \uc624\ub514\uc158\uc744 \ubcf4\ub0c8\ub2e4. \uc774 \uc624\ub514\uc158\uc73c\ub85c \uadf8\ub140\ub294 \uc601\ud654 \u300a\uccad\ubc14\uc9c0 \ub3cc\ub824 \uc785\uae30\u300b\uc758 \ube0c\ub9ac\uc9d3 \ube0c\ub9b4\ub79c\ub4dc \uc5ed\ud560\ub85c \uce90\uc2a4\ud305 \ub418\uc5c8\ub2e4. \ub77c\uc774\ube14\ub9ac\ub294 \uadf8\ub140\uac00 \ubc84\ubc45\ud06c \uace0\ub4f1\ud559\uad50 2\ud559\ub144\uacfc \uc878\uc5c5\uc744 \uc55e\ub454 \uc2dc\uae30 \ub3d9\uc548 \uc774 \uc601\ud654\uc758 \ucd2c\uc601\uc744 \ud588\ub2e4. \ubc84\ubc45\ud06c(Burbank)\uc5d0\uc11c \ub77c\uc774\ube14\ub9ac\ub294 \uc2dc\ub2c8\uc5b4 \ud074\ub798\uc2a4\uc758 \ud68c\uc7a5, \uce58\uc5b4\ub9ac\ub354 \ubc0f \ucc54\ud53c\uc5b8 \uc27d \ud569\ucc3d\ub2e8 \uba64\ubc84\ub85c \ud65c\ub3d9\ud588\ub2e4.", "answer": "\uc2dc\ub2c8\uc5b4 \ud074\ub798\uc2a4", "sentence": "\ubc84\ubc45\ud06c(Burbank)\uc5d0\uc11c \ub77c\uc774\ube14\ub9ac\ub294 \uc2dc\ub2c8\uc5b4 \ud074\ub798\uc2a4 \uc758 \ud68c\uc7a5, \uce58\uc5b4\ub9ac\ub354 \ubc0f \ucc54\ud53c\uc5b8 \uc27d \ud569\ucc3d\ub2e8 \uba64\ubc84\ub85c \ud65c\ub3d9\ud588\ub2e4.", "paragraph_sentence": "\ub77c\uc774\ube14\ub9ac\uc758 \uc5b4\ub9b0 \uc2dc\uc808, \uadf8\ub140\uc758 \ubd80\ubaa8\ub2d8\uc740 \uadf8\ub140\ub97c \ubca0\uc774\ube44\uc2dc\ud130\uc640 \ud568\uaed8 \uc9d1\uc5d0 \ub0a8\uaca8\ub450\uace0 \uc2f6\uc9c0 \uc54a\uc558\uae30 \ub54c\ubb38\uc5d0 \ubd80\ubaa8\ub2d8\uacfc \ud568\uaed8 \uadf8\ub140\uc640 \ud568\uaed8 \ud589\ub3d9 \uac15\uc2b5\ud68c\uc5d0 \ub370\ub824\uac14\uc2b5\ub2c8\ub2e4. \ub77c\uc774\ube14\ub9ac\ub294 \uc5b4\ub838\uc744 \ub54c\ubd80\ud130 \uadf8\ub140\uc758 \ubd80\ubaa8\uac00 \uc5f0\uae30 \uc218\uc5c5\uc744 \uc9c0\ub3c4\ud558\ub294 \uac83\uc744 \ubcf4\uace0 \uc5f0\uae30\uc5d0 \ub300\ud574 \ubc30\uc6b0\uac8c \ub418\uc5c8\uace0, \ucc28\uce30 \uadf8\ub140\uac00 \ub098\uc774\uac00 \ub4e4\uba74\uc11c \uc5f0\uae30\ud65c\ub3d9\uc5d0 \ub300\ud55c \ud655\uc2e0\uc744 \uac16\ub294\ub370 \ub3c4\uc6c0\uc774 \ub418\uc5c8\ub2e4\uace0 \ud68c\uc0c1\ud558\uc600\ub2e4. \uadf8\ub140\ub294 \ub2f9\ucd08 \ucc98\uc74c\uc5d0\ub294 \uc5f0\uae30\uc5d0 \uad00\uc2ec\uc774 \uc5c6\uc5c8\uace0, \uc2a4\ud0e0\ud3ec\ub4dc \ub300\ud559\uc5d0 \uc785\ud559\ud558\uae30 \ud76c\ub9dd\ud558\uace0 \uc788\uc5c8\ub2e4. \uadf8\ub140\uac00 \uc911\ud559\uad50 1\ud559\ub144\uacfc \uace0\ub4f1\ud559\uad50 2\ud559\ub144 \uc0ac\uc774\uc758 \uc5ec\ub984\uae30\uac04 \ub3d9\uc548 \uadf8\ub140\uc758 \uc624\ube60 \uc5d0\ub9ad\uc740 \uc5d0\uc774\uc804\ud2b8\uc5d0 \uba87 \ub2ec \ub3d9\uc548 \uba87 \ucc28\ub840\uc758 \uc624\ub514\uc158\uc744 \ubcf4\ub0c8\ub2e4. \uc774 \uc624\ub514\uc158\uc73c\ub85c \uadf8\ub140\ub294 \uc601\ud654 \u300a\uccad\ubc14\uc9c0 \ub3cc\ub824 \uc785\uae30\u300b\uc758 \ube0c\ub9ac\uc9d3 \ube0c\ub9b4\ub79c\ub4dc \uc5ed\ud560\ub85c \uce90\uc2a4\ud305 \ub418\uc5c8\ub2e4. \ub77c\uc774\ube14\ub9ac\ub294 \uadf8\ub140\uac00 \ubc84\ubc45\ud06c \uace0\ub4f1\ud559\uad50 2\ud559\ub144\uacfc \uc878\uc5c5\uc744 \uc55e\ub454 \uc2dc\uae30 \ub3d9\uc548 \uc774 \uc601\ud654\uc758 \ucd2c\uc601\uc744 \ud588\ub2e4. \ubc84\ubc45\ud06c(Burbank)\uc5d0\uc11c \ub77c\uc774\ube14\ub9ac\ub294 \uc2dc\ub2c8\uc5b4 \ud074\ub798\uc2a4 \uc758 \ud68c\uc7a5, \uce58\uc5b4\ub9ac\ub354 \ubc0f \ucc54\ud53c\uc5b8 \uc27d \ud569\ucc3d\ub2e8 \uba64\ubc84\ub85c \ud65c\ub3d9\ud588\ub2e4. ", "paragraph_answer": "\ub77c\uc774\ube14\ub9ac\uc758 \uc5b4\ub9b0 \uc2dc\uc808, \uadf8\ub140\uc758 \ubd80\ubaa8\ub2d8\uc740 \uadf8\ub140\ub97c \ubca0\uc774\ube44\uc2dc\ud130\uc640 \ud568\uaed8 \uc9d1\uc5d0 \ub0a8\uaca8\ub450\uace0 \uc2f6\uc9c0 \uc54a\uc558\uae30 \ub54c\ubb38\uc5d0 \ubd80\ubaa8\ub2d8\uacfc \ud568\uaed8 \uadf8\ub140\uc640 \ud568\uaed8 \ud589\ub3d9 \uac15\uc2b5\ud68c\uc5d0 \ub370\ub824\uac14\uc2b5\ub2c8\ub2e4. \ub77c\uc774\ube14\ub9ac\ub294 \uc5b4\ub838\uc744 \ub54c\ubd80\ud130 \uadf8\ub140\uc758 \ubd80\ubaa8\uac00 \uc5f0\uae30 \uc218\uc5c5\uc744 \uc9c0\ub3c4\ud558\ub294 \uac83\uc744 \ubcf4\uace0 \uc5f0\uae30\uc5d0 \ub300\ud574 \ubc30\uc6b0\uac8c \ub418\uc5c8\uace0, \ucc28\uce30 \uadf8\ub140\uac00 \ub098\uc774\uac00 \ub4e4\uba74\uc11c \uc5f0\uae30\ud65c\ub3d9\uc5d0 \ub300\ud55c \ud655\uc2e0\uc744 \uac16\ub294\ub370 \ub3c4\uc6c0\uc774 \ub418\uc5c8\ub2e4\uace0 \ud68c\uc0c1\ud558\uc600\ub2e4. \uadf8\ub140\ub294 \ub2f9\ucd08 \ucc98\uc74c\uc5d0\ub294 \uc5f0\uae30\uc5d0 \uad00\uc2ec\uc774 \uc5c6\uc5c8\uace0, \uc2a4\ud0e0\ud3ec\ub4dc \ub300\ud559\uc5d0 \uc785\ud559\ud558\uae30 \ud76c\ub9dd\ud558\uace0 \uc788\uc5c8\ub2e4. \uadf8\ub140\uac00 \uc911\ud559\uad50 1\ud559\ub144\uacfc \uace0\ub4f1\ud559\uad50 2\ud559\ub144 \uc0ac\uc774\uc758 \uc5ec\ub984\uae30\uac04 \ub3d9\uc548 \uadf8\ub140\uc758 \uc624\ube60 \uc5d0\ub9ad\uc740 \uc5d0\uc774\uc804\ud2b8\uc5d0 \uba87 \ub2ec \ub3d9\uc548 \uba87 \ucc28\ub840\uc758 \uc624\ub514\uc158\uc744 \ubcf4\ub0c8\ub2e4. \uc774 \uc624\ub514\uc158\uc73c\ub85c \uadf8\ub140\ub294 \uc601\ud654 \u300a\uccad\ubc14\uc9c0 \ub3cc\ub824 \uc785\uae30\u300b\uc758 \ube0c\ub9ac\uc9d3 \ube0c\ub9b4\ub79c\ub4dc \uc5ed\ud560\ub85c \uce90\uc2a4\ud305 \ub418\uc5c8\ub2e4. \ub77c\uc774\ube14\ub9ac\ub294 \uadf8\ub140\uac00 \ubc84\ubc45\ud06c \uace0\ub4f1\ud559\uad50 2\ud559\ub144\uacfc \uc878\uc5c5\uc744 \uc55e\ub454 \uc2dc\uae30 \ub3d9\uc548 \uc774 \uc601\ud654\uc758 \ucd2c\uc601\uc744 \ud588\ub2e4. \ubc84\ubc45\ud06c(Burbank)\uc5d0\uc11c \ub77c\uc774\ube14\ub9ac\ub294 \uc2dc\ub2c8\uc5b4 \ud074\ub798\uc2a4 \uc758 \ud68c\uc7a5, \uce58\uc5b4\ub9ac\ub354 \ubc0f \ucc54\ud53c\uc5b8 \uc27d \ud569\ucc3d\ub2e8 \uba64\ubc84\ub85c \ud65c\ub3d9\ud588\ub2e4.", "sentence_answer": "\ubc84\ubc45\ud06c(Burbank)\uc5d0\uc11c \ub77c\uc774\ube14\ub9ac\ub294 \uc2dc\ub2c8\uc5b4 \ud074\ub798\uc2a4 \uc758 \ud68c\uc7a5, \uce58\uc5b4\ub9ac\ub354 \ubc0f \ucc54\ud53c\uc5b8 \uc27d \ud569\ucc3d\ub2e8 \uba64\ubc84\ub85c \ud65c\ub3d9\ud588\ub2e4.", "paragraph_id": "6509731-1-1", "paragraph_question": "question: \ube14\ub808\uc774\ud06c \ub77c\uc774\ube14\ub9ac\uac00 \ubc84\ubc45\ud06c \uace0\ub4f1\ud559\uad50\uc5d0\uc11c \ud68c\uc7a5\uc744 \ub9e1\uc558\ub358 \uadf8\ub8f9\uc740?, context: \ub77c\uc774\ube14\ub9ac\uc758 \uc5b4\ub9b0 \uc2dc\uc808, \uadf8\ub140\uc758 \ubd80\ubaa8\ub2d8\uc740 \uadf8\ub140\ub97c \ubca0\uc774\ube44\uc2dc\ud130\uc640 \ud568\uaed8 \uc9d1\uc5d0 \ub0a8\uaca8\ub450\uace0 \uc2f6\uc9c0 \uc54a\uc558\uae30 \ub54c\ubb38\uc5d0 \ubd80\ubaa8\ub2d8\uacfc \ud568\uaed8 \uadf8\ub140\uc640 \ud568\uaed8 \ud589\ub3d9 \uac15\uc2b5\ud68c\uc5d0 \ub370\ub824\uac14\uc2b5\ub2c8\ub2e4. \ub77c\uc774\ube14\ub9ac\ub294 \uc5b4\ub838\uc744 \ub54c\ubd80\ud130 \uadf8\ub140\uc758 \ubd80\ubaa8\uac00 \uc5f0\uae30 \uc218\uc5c5\uc744 \uc9c0\ub3c4\ud558\ub294 \uac83\uc744 \ubcf4\uace0 \uc5f0\uae30\uc5d0 \ub300\ud574 \ubc30\uc6b0\uac8c \ub418\uc5c8\uace0, \ucc28\uce30 \uadf8\ub140\uac00 \ub098\uc774\uac00 \ub4e4\uba74\uc11c \uc5f0\uae30\ud65c\ub3d9\uc5d0 \ub300\ud55c \ud655\uc2e0\uc744 \uac16\ub294\ub370 \ub3c4\uc6c0\uc774 \ub418\uc5c8\ub2e4\uace0 \ud68c\uc0c1\ud558\uc600\ub2e4. \uadf8\ub140\ub294 \ub2f9\ucd08 \ucc98\uc74c\uc5d0\ub294 \uc5f0\uae30\uc5d0 \uad00\uc2ec\uc774 \uc5c6\uc5c8\uace0, \uc2a4\ud0e0\ud3ec\ub4dc \ub300\ud559\uc5d0 \uc785\ud559\ud558\uae30 \ud76c\ub9dd\ud558\uace0 \uc788\uc5c8\ub2e4. \uadf8\ub140\uac00 \uc911\ud559\uad50 1\ud559\ub144\uacfc \uace0\ub4f1\ud559\uad50 2\ud559\ub144 \uc0ac\uc774\uc758 \uc5ec\ub984\uae30\uac04 \ub3d9\uc548 \uadf8\ub140\uc758 \uc624\ube60 \uc5d0\ub9ad\uc740 \uc5d0\uc774\uc804\ud2b8\uc5d0 \uba87 \ub2ec \ub3d9\uc548 \uba87 \ucc28\ub840\uc758 \uc624\ub514\uc158\uc744 \ubcf4\ub0c8\ub2e4. \uc774 \uc624\ub514\uc158\uc73c\ub85c \uadf8\ub140\ub294 \uc601\ud654 \u300a\uccad\ubc14\uc9c0 \ub3cc\ub824 \uc785\uae30\u300b\uc758 \ube0c\ub9ac\uc9d3 \ube0c\ub9b4\ub79c\ub4dc \uc5ed\ud560\ub85c \uce90\uc2a4\ud305 \ub418\uc5c8\ub2e4. \ub77c\uc774\ube14\ub9ac\ub294 \uadf8\ub140\uac00 \ubc84\ubc45\ud06c \uace0\ub4f1\ud559\uad50 2\ud559\ub144\uacfc \uc878\uc5c5\uc744 \uc55e\ub454 \uc2dc\uae30 \ub3d9\uc548 \uc774 \uc601\ud654\uc758 \ucd2c\uc601\uc744 \ud588\ub2e4. \ubc84\ubc45\ud06c(Burbank)\uc5d0\uc11c \ub77c\uc774\ube14\ub9ac\ub294 \uc2dc\ub2c8\uc5b4 \ud074\ub798\uc2a4\uc758 \ud68c\uc7a5, \uce58\uc5b4\ub9ac\ub354 \ubc0f \ucc54\ud53c\uc5b8 \uc27d \ud569\ucc3d\ub2e8 \uba64\ubc84\ub85c \ud65c\ub3d9\ud588\ub2e4."} {"question": "\ud06c\ub808\ud0c0\ub294 \ubb34\uc2ac\ub9bc\ub4e4\uc774 \uc0b4\uae30 \uc2dc\uc791\ud55c \ub54c\ub294 \uc5b8\uc81c\uc778\uac00?", "paragraph": "\uc544\ub78d\uc778\uc758 \ub300\uc815\ubcf5 \ub54c\ubd80\ud130 \ud06c\ub808\ud0c0\uc5d0 \uc774\ubbf8 \ubb34\uc2ac\ub9bc\ub4e4\uc774 \uc0b4\uc558\uc9c0\ub9cc, \uc624\uc2a4\ub9cc \uc81c\uad6d\uc758 \uc9c0\ubc30\ub97c \ubc1b\uc73c\uba74\uc11c \ubb34\uc2ac\ub9bc\uc774 \ub354 \ub298\uc5b4\ub0ac\ub2e4. \ub300\ubd80\ubd84\uc758 \ud06c\ub808\ud0c0\uc778 \ubb34\uc2ac\ub9bc\ub4e4\uc740 \ud06c\ub808\ud0c0 \uadf8\ub9ac\uc2a4\uc5b4\ub97c \uc4f0\ub358 \uc9c0\uc5ed \uadf8\ub9ac\uc2a4 \uac1c\uc885\uc790\ub4e4\uc774\uc5c8\ub2e4. \uadf8\ub7ec\ub098 19\uc138\uae30 \ud06c\ub808\ud0c0\uc758 \uc815\uce58 \uc0c1\ud669\uc5d0\uc11c \uadf8\ub9ac\uc2a4\ub3c4 \uad50\ub3c4\ub4e4\uc740 \uc774\ub4e4\uc744 \ud130\ud0a4\uc778\uc73c\ub85c \uce58\ubd80\ud558\uac8c \ub41c\ub2e4. \uadf8\ub9ac\uc2a4 \ub3c5\ub9bd\uc804\uc7c1 \uc9c1\uc804\uc5d0 \ud06c\ub808\ud0c0 \uc12c \uc778\uad6c\uc758 45%\uac00 \ubb34\uc2ac\ub9bc\uc774\uc5c8\ub2e4. \uadf8\ub4e4 \uc911 \uc0c1\ub2f9\uc218\uac00 \ud06c\ub808\ud0c0 \uadf8\ub9ac\uc2a4\ub3c4 \uad50\ub3c4 \ucd9c\uc2e0\uc774\uc5c8\ub2e4. \ub098\uc911\uc5d0 \ud130\ud0a4, \ub85c\ub3c4\uc2a4, \uc2dc\ub9ac\uc544 \ub4f1\uc9c0\uc758 \uc0ac\ud68c \ubd88\uc548\uc73c\ub85c \ub9ce\uc740 \ubb34\uc2ac\ub9bc\ub4e4\uc774 \uc12c\uc744 \ub5a0\ub098\uc790 \ub0a8\uc740 \uc790\ub4e4\uc740 \ub2e4\uc2dc \uadf8\ub9ac\uc2a4\ub3c4\uad50\ub85c \ub3cc\uc544\uc624\uac8c \ub41c\ub2e4. 1900\ub144\uc5d0 \uc12c \uc778\uad6c\uc758 11%\ub9cc\uc774 \ubb34\uc2ac\ub9bc\uc774\uc5c8\ub2e4. \ub0a8\uc740 \uc0ac\ub78c\ub4e4\uc740 1924\ub144 \ud130\ud0a4-\uadf8\ub9ac\uc2a4 \uc778\uad6c \uad50\ud658\ub54c \uc12c\uc744 \ub5a0\ub098\ub3c4\ub85d \uac15\uc694\ubc1b\uc558\ub2e4.", "answer": "\uc544\ub78d\uc778\uc758 \ub300\uc815\ubcf5", "sentence": "\uc544\ub78d\uc778\uc758 \ub300\uc815\ubcf5 \ub54c\ubd80\ud130 \ud06c\ub808\ud0c0\uc5d0 \uc774\ubbf8 \ubb34\uc2ac\ub9bc\ub4e4\uc774 \uc0b4\uc558\uc9c0\ub9cc, \uc624\uc2a4\ub9cc \uc81c\uad6d\uc758 \uc9c0\ubc30\ub97c \ubc1b\uc73c\uba74\uc11c \ubb34\uc2ac\ub9bc\uc774 \ub354 \ub298\uc5b4\ub0ac\ub2e4.", "paragraph_sentence": " \uc544\ub78d\uc778\uc758 \ub300\uc815\ubcf5 \ub54c\ubd80\ud130 \ud06c\ub808\ud0c0\uc5d0 \uc774\ubbf8 \ubb34\uc2ac\ub9bc\ub4e4\uc774 \uc0b4\uc558\uc9c0\ub9cc, \uc624\uc2a4\ub9cc \uc81c\uad6d\uc758 \uc9c0\ubc30\ub97c \ubc1b\uc73c\uba74\uc11c \ubb34\uc2ac\ub9bc\uc774 \ub354 \ub298\uc5b4\ub0ac\ub2e4. \ub300\ubd80\ubd84\uc758 \ud06c\ub808\ud0c0\uc778 \ubb34\uc2ac\ub9bc\ub4e4\uc740 \ud06c\ub808\ud0c0 \uadf8\ub9ac\uc2a4\uc5b4\ub97c \uc4f0\ub358 \uc9c0\uc5ed \uadf8\ub9ac\uc2a4 \uac1c\uc885\uc790\ub4e4\uc774\uc5c8\ub2e4. \uadf8\ub7ec\ub098 19\uc138\uae30 \ud06c\ub808\ud0c0\uc758 \uc815\uce58 \uc0c1\ud669\uc5d0\uc11c \uadf8\ub9ac\uc2a4\ub3c4 \uad50\ub3c4\ub4e4\uc740 \uc774\ub4e4\uc744 \ud130\ud0a4\uc778\uc73c\ub85c \uce58\ubd80\ud558\uac8c \ub41c\ub2e4. \uadf8\ub9ac\uc2a4 \ub3c5\ub9bd\uc804\uc7c1 \uc9c1\uc804\uc5d0 \ud06c\ub808\ud0c0 \uc12c \uc778\uad6c\uc758 45%\uac00 \ubb34\uc2ac\ub9bc\uc774\uc5c8\ub2e4. \uadf8\ub4e4 \uc911 \uc0c1\ub2f9\uc218\uac00 \ud06c\ub808\ud0c0 \uadf8\ub9ac\uc2a4\ub3c4 \uad50\ub3c4 \ucd9c\uc2e0\uc774\uc5c8\ub2e4. \ub098\uc911\uc5d0 \ud130\ud0a4, \ub85c\ub3c4\uc2a4, \uc2dc\ub9ac\uc544 \ub4f1\uc9c0\uc758 \uc0ac\ud68c \ubd88\uc548\uc73c\ub85c \ub9ce\uc740 \ubb34\uc2ac\ub9bc\ub4e4\uc774 \uc12c\uc744 \ub5a0\ub098\uc790 \ub0a8\uc740 \uc790\ub4e4\uc740 \ub2e4\uc2dc \uadf8\ub9ac\uc2a4\ub3c4\uad50\ub85c \ub3cc\uc544\uc624\uac8c \ub41c\ub2e4. 1900\ub144\uc5d0 \uc12c \uc778\uad6c\uc758 11%\ub9cc\uc774 \ubb34\uc2ac\ub9bc\uc774\uc5c8\ub2e4. \ub0a8\uc740 \uc0ac\ub78c\ub4e4\uc740 1924\ub144 \ud130\ud0a4-\uadf8\ub9ac\uc2a4 \uc778\uad6c \uad50\ud658\ub54c \uc12c\uc744 \ub5a0\ub098\ub3c4\ub85d \uac15\uc694\ubc1b\uc558\ub2e4.", "paragraph_answer": " \uc544\ub78d\uc778\uc758 \ub300\uc815\ubcf5 \ub54c\ubd80\ud130 \ud06c\ub808\ud0c0\uc5d0 \uc774\ubbf8 \ubb34\uc2ac\ub9bc\ub4e4\uc774 \uc0b4\uc558\uc9c0\ub9cc, \uc624\uc2a4\ub9cc \uc81c\uad6d\uc758 \uc9c0\ubc30\ub97c \ubc1b\uc73c\uba74\uc11c \ubb34\uc2ac\ub9bc\uc774 \ub354 \ub298\uc5b4\ub0ac\ub2e4. \ub300\ubd80\ubd84\uc758 \ud06c\ub808\ud0c0\uc778 \ubb34\uc2ac\ub9bc\ub4e4\uc740 \ud06c\ub808\ud0c0 \uadf8\ub9ac\uc2a4\uc5b4\ub97c \uc4f0\ub358 \uc9c0\uc5ed \uadf8\ub9ac\uc2a4 \uac1c\uc885\uc790\ub4e4\uc774\uc5c8\ub2e4. \uadf8\ub7ec\ub098 19\uc138\uae30 \ud06c\ub808\ud0c0\uc758 \uc815\uce58 \uc0c1\ud669\uc5d0\uc11c \uadf8\ub9ac\uc2a4\ub3c4 \uad50\ub3c4\ub4e4\uc740 \uc774\ub4e4\uc744 \ud130\ud0a4\uc778\uc73c\ub85c \uce58\ubd80\ud558\uac8c \ub41c\ub2e4. \uadf8\ub9ac\uc2a4 \ub3c5\ub9bd\uc804\uc7c1 \uc9c1\uc804\uc5d0 \ud06c\ub808\ud0c0 \uc12c \uc778\uad6c\uc758 45%\uac00 \ubb34\uc2ac\ub9bc\uc774\uc5c8\ub2e4. \uadf8\ub4e4 \uc911 \uc0c1\ub2f9\uc218\uac00 \ud06c\ub808\ud0c0 \uadf8\ub9ac\uc2a4\ub3c4 \uad50\ub3c4 \ucd9c\uc2e0\uc774\uc5c8\ub2e4. \ub098\uc911\uc5d0 \ud130\ud0a4, \ub85c\ub3c4\uc2a4, \uc2dc\ub9ac\uc544 \ub4f1\uc9c0\uc758 \uc0ac\ud68c \ubd88\uc548\uc73c\ub85c \ub9ce\uc740 \ubb34\uc2ac\ub9bc\ub4e4\uc774 \uc12c\uc744 \ub5a0\ub098\uc790 \ub0a8\uc740 \uc790\ub4e4\uc740 \ub2e4\uc2dc \uadf8\ub9ac\uc2a4\ub3c4\uad50\ub85c \ub3cc\uc544\uc624\uac8c \ub41c\ub2e4. 1900\ub144\uc5d0 \uc12c \uc778\uad6c\uc758 11%\ub9cc\uc774 \ubb34\uc2ac\ub9bc\uc774\uc5c8\ub2e4. \ub0a8\uc740 \uc0ac\ub78c\ub4e4\uc740 1924\ub144 \ud130\ud0a4-\uadf8\ub9ac\uc2a4 \uc778\uad6c \uad50\ud658\ub54c \uc12c\uc744 \ub5a0\ub098\ub3c4\ub85d \uac15\uc694\ubc1b\uc558\ub2e4.", "sentence_answer": " \uc544\ub78d\uc778\uc758 \ub300\uc815\ubcf5 \ub54c\ubd80\ud130 \ud06c\ub808\ud0c0\uc5d0 \uc774\ubbf8 \ubb34\uc2ac\ub9bc\ub4e4\uc774 \uc0b4\uc558\uc9c0\ub9cc, \uc624\uc2a4\ub9cc \uc81c\uad6d\uc758 \uc9c0\ubc30\ub97c \ubc1b\uc73c\uba74\uc11c \ubb34\uc2ac\ub9bc\uc774 \ub354 \ub298\uc5b4\ub0ac\ub2e4.", "paragraph_id": "6563340-2-0", "paragraph_question": "question: \ud06c\ub808\ud0c0\ub294 \ubb34\uc2ac\ub9bc\ub4e4\uc774 \uc0b4\uae30 \uc2dc\uc791\ud55c \ub54c\ub294 \uc5b8\uc81c\uc778\uac00?, context: \uc544\ub78d\uc778\uc758 \ub300\uc815\ubcf5 \ub54c\ubd80\ud130 \ud06c\ub808\ud0c0\uc5d0 \uc774\ubbf8 \ubb34\uc2ac\ub9bc\ub4e4\uc774 \uc0b4\uc558\uc9c0\ub9cc, \uc624\uc2a4\ub9cc \uc81c\uad6d\uc758 \uc9c0\ubc30\ub97c \ubc1b\uc73c\uba74\uc11c \ubb34\uc2ac\ub9bc\uc774 \ub354 \ub298\uc5b4\ub0ac\ub2e4. \ub300\ubd80\ubd84\uc758 \ud06c\ub808\ud0c0\uc778 \ubb34\uc2ac\ub9bc\ub4e4\uc740 \ud06c\ub808\ud0c0 \uadf8\ub9ac\uc2a4\uc5b4\ub97c \uc4f0\ub358 \uc9c0\uc5ed \uadf8\ub9ac\uc2a4 \uac1c\uc885\uc790\ub4e4\uc774\uc5c8\ub2e4. \uadf8\ub7ec\ub098 19\uc138\uae30 \ud06c\ub808\ud0c0\uc758 \uc815\uce58 \uc0c1\ud669\uc5d0\uc11c \uadf8\ub9ac\uc2a4\ub3c4 \uad50\ub3c4\ub4e4\uc740 \uc774\ub4e4\uc744 \ud130\ud0a4\uc778\uc73c\ub85c \uce58\ubd80\ud558\uac8c \ub41c\ub2e4. \uadf8\ub9ac\uc2a4 \ub3c5\ub9bd\uc804\uc7c1 \uc9c1\uc804\uc5d0 \ud06c\ub808\ud0c0 \uc12c \uc778\uad6c\uc758 45%\uac00 \ubb34\uc2ac\ub9bc\uc774\uc5c8\ub2e4. \uadf8\ub4e4 \uc911 \uc0c1\ub2f9\uc218\uac00 \ud06c\ub808\ud0c0 \uadf8\ub9ac\uc2a4\ub3c4 \uad50\ub3c4 \ucd9c\uc2e0\uc774\uc5c8\ub2e4. \ub098\uc911\uc5d0 \ud130\ud0a4, \ub85c\ub3c4\uc2a4, \uc2dc\ub9ac\uc544 \ub4f1\uc9c0\uc758 \uc0ac\ud68c \ubd88\uc548\uc73c\ub85c \ub9ce\uc740 \ubb34\uc2ac\ub9bc\ub4e4\uc774 \uc12c\uc744 \ub5a0\ub098\uc790 \ub0a8\uc740 \uc790\ub4e4\uc740 \ub2e4\uc2dc \uadf8\ub9ac\uc2a4\ub3c4\uad50\ub85c \ub3cc\uc544\uc624\uac8c \ub41c\ub2e4. 1900\ub144\uc5d0 \uc12c \uc778\uad6c\uc758 11%\ub9cc\uc774 \ubb34\uc2ac\ub9bc\uc774\uc5c8\ub2e4. \ub0a8\uc740 \uc0ac\ub78c\ub4e4\uc740 1924\ub144 \ud130\ud0a4-\uadf8\ub9ac\uc2a4 \uc778\uad6c \uad50\ud658\ub54c \uc12c\uc744 \ub5a0\ub098\ub3c4\ub85d \uac15\uc694\ubc1b\uc558\ub2e4."} {"question": "\ub808\uc624\ub12c\ub85c\ub294 \uc2e4\ub825\uc788\ub294 \uc815\uce58\uac00\uc600\uace0 \ud398\ub77c\ub77c\ub0b4 \uacbd\uc81c\uc801 \ubc88\uc601\uc5d0 \uc9d1\uc911\ud588\ub294\ub370 \ud398\ub77c\ub77c \ucd5c\ucd08\uc758 \ubb34\uc5c7\uc744 \uac74\uc124\ud558\ub294\ub370 \uad00\uc5ec\ud588\ub294\uac00?", "paragraph": "\ub2c8\ucf5c\ub85c 3\uc138 \ud1b5\uce58 \uc2dc\uae30\uc5d0 \uc774\ub7ec\ud55c \uacfc\uc815\ub4e4\uc774 \uc5c6\uc5c8\ub358 \uac83\ub4e4\uc744 \ub808\uc624\ub12c\ub85c\ub294 \uc815\uce58\uc801 \uc77c\ub4e4\uacfc \ud398\ub77c\ub77c\ub0b4 \uacbd\uc81c\uc801 \ubc88\uc601\uc5d0 \ud55c\uce35 \ub354 \uc9d1\uc911\ud558\uc600\ub2e4. \ub808\uc624\ub12c\ub85c\ub294 \uc2e4\ub825\uc788\ub294 \uc815\uce58\uac00\uc600\uace0 \ud398\ub77c\ub77c \ucd5c\ucd08\uc758 \ubcd1\uc6d0 \uac74\uc124\uc5d0 \uad00\uc5ec\ub97c \ud588\uc5c8\ub2e4. \ud558\uc9c0\ub9cc \uadf8\ub294 \uc8fc\ub85c \uc2a4\uc2a4\ub85c\ub97c \ubb38\ud654\uc778\uc73c\ub85c\uc11c \uad6c\ubd84\uc9c0\uc5c8\ub2e4. \ub808\uc628 \ubc14\ud2f0\uc2a4\ud0c0 \uc54c\ubca0\ub974\ud2f0\ub294 \ub808\uc624\ub12c\ub85c\uc758 \uc758\ub8b0\uc5d0 \uc758\ud574 \uac74\ucd95\ub860\uc744 \uc800\uc220\ud558\uc600\uc73c\uba70, \ud398\ub77c\ub77c\uc758 \uad81\uc804\uc5d0\ub294 \ud53c\uc0ac\ub12c\ub85c, \uc57c\ucf54\ud3ec \ubca8\ub9ac\ub2c8, \uc870\ubc18\ub2c8 \ub2e4 \uc624\ub9ac\uc62c\ub85c, \uc548\ub4dc\ub808\uc544 \ub9cc\ud14c\ub0d0, \ud53c\uc5d0\ub85c \ub378\ub77c \ud504\ub780\uccb4\uc2a4\uce74, \ub124\ub35c\ub780\ub4dc \ucd9c\uc2e0\uc758 \ub85c\ud788\uc5b4\ub974 \ud310 \ub370\ub974 \ubca0\uc774\ub358 \ub4f1\uc774 \uc608\uc220\uac00\ub4e4\uc774 \uc788\uc5c8\ub2e4. \uadf8\uc758 \uac1c\uc778 \uc131\ubb34\uc77c\uacfc\uc11c\ub294 \ubd84\ud560\ub41c \uc0c1\ud0dc\ub85c 1958\ub144 \ud06c\ub9ac\uc2a4\ud2f0 \uacbd\ub9e4\uc5d0\uc11c \ub791\uac00\ud1a1(Llangattock) \ub0a8\uc791\uc5d0\uac8c \ud310\ub9e4\ub418\uae30\ub3c4 \ud558\uc600\ub2e4. \uc774\ud6c4\ub85c \uc774 \uc11c\uc801\uc740 \ub791\uac00\ud1a1 \uc131\ubb34\uc77c\uacfc\uc11c\ub85c \uc54c\ub824\uc9c0\uac8c \ub418\uc5c8\ub2e4. \uc774\ub294 \uc870\ub974\uc870 \ub2ec\ub808\ub9c8\ub0d0(Giorgio d\u00b4Alemagna)\uc758 \uc608\uc220\uc801 \ubc29\ud5a5\ud558\uc5d0\uc11c \ub9cc\ub4e4\uc5b4\uc84c\uace0 \ub9c8\ud14c\uc624 \ub370 \ud30c\uc2a4\ud2f0(Matteo De Pasti)\uc640 \uc57c\ucf54\ud3ec \ub9c8\ub0d0\ub2c8\ubaa8(Jacopo Magnanimo) \uac19\uc740 \ud654\uac00\ub4e4\ub3c4 \uc5ec\uae30\uc5d0 \uae30\uc5ec\ub97c \ud558\uc600\ub2e4. \ud30c\ub9ac\uc758 \ub8e8\ube0c\ub974, \ub374\ub9c8\ud06c \uad6d\ub9bd \ub3c4\uc11c\uad00, \uc77c\ubd80 \uac1c\uc778 \uc218\uc9d1\uac00\ub4e4\uc5d0\uac8c \ud769\uc5b4\uc838\uc788\ub294 \uc0c1\ud669\uc774\ub2e4.", "answer": "\ubcd1\uc6d0", "sentence": "\ub808\uc624\ub12c\ub85c\ub294 \uc2e4\ub825\uc788\ub294 \uc815\uce58\uac00\uc600\uace0 \ud398\ub77c\ub77c \ucd5c\ucd08\uc758 \ubcd1\uc6d0 \uac74\uc124\uc5d0 \uad00\uc5ec\ub97c \ud588\uc5c8\ub2e4.", "paragraph_sentence": "\ub2c8\ucf5c\ub85c 3\uc138 \ud1b5\uce58 \uc2dc\uae30\uc5d0 \uc774\ub7ec\ud55c \uacfc\uc815\ub4e4\uc774 \uc5c6\uc5c8\ub358 \uac83\ub4e4\uc744 \ub808\uc624\ub12c\ub85c\ub294 \uc815\uce58\uc801 \uc77c\ub4e4\uacfc \ud398\ub77c\ub77c\ub0b4 \uacbd\uc81c\uc801 \ubc88\uc601\uc5d0 \ud55c\uce35 \ub354 \uc9d1\uc911\ud558\uc600\ub2e4. \ub808\uc624\ub12c\ub85c\ub294 \uc2e4\ub825\uc788\ub294 \uc815\uce58\uac00\uc600\uace0 \ud398\ub77c\ub77c \ucd5c\ucd08\uc758 \ubcd1\uc6d0 \uac74\uc124\uc5d0 \uad00\uc5ec\ub97c \ud588\uc5c8\ub2e4. \ud558\uc9c0\ub9cc \uadf8\ub294 \uc8fc\ub85c \uc2a4\uc2a4\ub85c\ub97c \ubb38\ud654\uc778\uc73c\ub85c\uc11c \uad6c\ubd84\uc9c0\uc5c8\ub2e4. \ub808\uc628 \ubc14\ud2f0\uc2a4\ud0c0 \uc54c\ubca0\ub974\ud2f0\ub294 \ub808\uc624\ub12c\ub85c\uc758 \uc758\ub8b0\uc5d0 \uc758\ud574 \uac74\ucd95\ub860\uc744 \uc800\uc220\ud558\uc600\uc73c\uba70, \ud398\ub77c\ub77c\uc758 \uad81\uc804\uc5d0\ub294 \ud53c\uc0ac\ub12c\ub85c, \uc57c\ucf54\ud3ec \ubca8\ub9ac\ub2c8, \uc870\ubc18\ub2c8 \ub2e4 \uc624\ub9ac\uc62c\ub85c, \uc548\ub4dc\ub808\uc544 \ub9cc\ud14c\ub0d0, \ud53c\uc5d0\ub85c \ub378\ub77c \ud504\ub780\uccb4\uc2a4\uce74, \ub124\ub35c\ub780\ub4dc \ucd9c\uc2e0\uc758 \ub85c\ud788\uc5b4\ub974 \ud310 \ub370\ub974 \ubca0\uc774\ub358 \ub4f1\uc774 \uc608\uc220\uac00\ub4e4\uc774 \uc788\uc5c8\ub2e4. \uadf8\uc758 \uac1c\uc778 \uc131\ubb34\uc77c\uacfc\uc11c\ub294 \ubd84\ud560\ub41c \uc0c1\ud0dc\ub85c 1958\ub144 \ud06c\ub9ac\uc2a4\ud2f0 \uacbd\ub9e4\uc5d0\uc11c \ub791\uac00\ud1a1(Llangattock) \ub0a8\uc791\uc5d0\uac8c \ud310\ub9e4\ub418\uae30\ub3c4 \ud558\uc600\ub2e4. \uc774\ud6c4\ub85c \uc774 \uc11c\uc801\uc740 \ub791\uac00\ud1a1 \uc131\ubb34\uc77c\uacfc\uc11c\ub85c \uc54c\ub824\uc9c0\uac8c \ub418\uc5c8\ub2e4. \uc774\ub294 \uc870\ub974\uc870 \ub2ec\ub808\ub9c8\ub0d0(Giorgio d\u00b4Alemagna)\uc758 \uc608\uc220\uc801 \ubc29\ud5a5\ud558\uc5d0\uc11c \ub9cc\ub4e4\uc5b4\uc84c\uace0 \ub9c8\ud14c\uc624 \ub370 \ud30c\uc2a4\ud2f0(Matteo De Pasti)\uc640 \uc57c\ucf54\ud3ec \ub9c8\ub0d0\ub2c8\ubaa8(Jacopo Magnanimo) \uac19\uc740 \ud654\uac00\ub4e4\ub3c4 \uc5ec\uae30\uc5d0 \uae30\uc5ec\ub97c \ud558\uc600\ub2e4. \ud30c\ub9ac\uc758 \ub8e8\ube0c\ub974, \ub374\ub9c8\ud06c \uad6d\ub9bd \ub3c4\uc11c\uad00, \uc77c\ubd80 \uac1c\uc778 \uc218\uc9d1\uac00\ub4e4\uc5d0\uac8c \ud769\uc5b4\uc838\uc788\ub294 \uc0c1\ud669\uc774\ub2e4.", "paragraph_answer": "\ub2c8\ucf5c\ub85c 3\uc138 \ud1b5\uce58 \uc2dc\uae30\uc5d0 \uc774\ub7ec\ud55c \uacfc\uc815\ub4e4\uc774 \uc5c6\uc5c8\ub358 \uac83\ub4e4\uc744 \ub808\uc624\ub12c\ub85c\ub294 \uc815\uce58\uc801 \uc77c\ub4e4\uacfc \ud398\ub77c\ub77c\ub0b4 \uacbd\uc81c\uc801 \ubc88\uc601\uc5d0 \ud55c\uce35 \ub354 \uc9d1\uc911\ud558\uc600\ub2e4. \ub808\uc624\ub12c\ub85c\ub294 \uc2e4\ub825\uc788\ub294 \uc815\uce58\uac00\uc600\uace0 \ud398\ub77c\ub77c \ucd5c\ucd08\uc758 \ubcd1\uc6d0 \uac74\uc124\uc5d0 \uad00\uc5ec\ub97c \ud588\uc5c8\ub2e4. \ud558\uc9c0\ub9cc \uadf8\ub294 \uc8fc\ub85c \uc2a4\uc2a4\ub85c\ub97c \ubb38\ud654\uc778\uc73c\ub85c\uc11c \uad6c\ubd84\uc9c0\uc5c8\ub2e4. \ub808\uc628 \ubc14\ud2f0\uc2a4\ud0c0 \uc54c\ubca0\ub974\ud2f0\ub294 \ub808\uc624\ub12c\ub85c\uc758 \uc758\ub8b0\uc5d0 \uc758\ud574 \uac74\ucd95\ub860\uc744 \uc800\uc220\ud558\uc600\uc73c\uba70, \ud398\ub77c\ub77c\uc758 \uad81\uc804\uc5d0\ub294 \ud53c\uc0ac\ub12c\ub85c, \uc57c\ucf54\ud3ec \ubca8\ub9ac\ub2c8, \uc870\ubc18\ub2c8 \ub2e4 \uc624\ub9ac\uc62c\ub85c, \uc548\ub4dc\ub808\uc544 \ub9cc\ud14c\ub0d0, \ud53c\uc5d0\ub85c \ub378\ub77c \ud504\ub780\uccb4\uc2a4\uce74, \ub124\ub35c\ub780\ub4dc \ucd9c\uc2e0\uc758 \ub85c\ud788\uc5b4\ub974 \ud310 \ub370\ub974 \ubca0\uc774\ub358 \ub4f1\uc774 \uc608\uc220\uac00\ub4e4\uc774 \uc788\uc5c8\ub2e4. \uadf8\uc758 \uac1c\uc778 \uc131\ubb34\uc77c\uacfc\uc11c\ub294 \ubd84\ud560\ub41c \uc0c1\ud0dc\ub85c 1958\ub144 \ud06c\ub9ac\uc2a4\ud2f0 \uacbd\ub9e4\uc5d0\uc11c \ub791\uac00\ud1a1(Llangattock) \ub0a8\uc791\uc5d0\uac8c \ud310\ub9e4\ub418\uae30\ub3c4 \ud558\uc600\ub2e4. \uc774\ud6c4\ub85c \uc774 \uc11c\uc801\uc740 \ub791\uac00\ud1a1 \uc131\ubb34\uc77c\uacfc\uc11c\ub85c \uc54c\ub824\uc9c0\uac8c \ub418\uc5c8\ub2e4. \uc774\ub294 \uc870\ub974\uc870 \ub2ec\ub808\ub9c8\ub0d0(Giorgio d\u00b4Alemagna)\uc758 \uc608\uc220\uc801 \ubc29\ud5a5\ud558\uc5d0\uc11c \ub9cc\ub4e4\uc5b4\uc84c\uace0 \ub9c8\ud14c\uc624 \ub370 \ud30c\uc2a4\ud2f0(Matteo De Pasti)\uc640 \uc57c\ucf54\ud3ec \ub9c8\ub0d0\ub2c8\ubaa8(Jacopo Magnanimo) \uac19\uc740 \ud654\uac00\ub4e4\ub3c4 \uc5ec\uae30\uc5d0 \uae30\uc5ec\ub97c \ud558\uc600\ub2e4. \ud30c\ub9ac\uc758 \ub8e8\ube0c\ub974, \ub374\ub9c8\ud06c \uad6d\ub9bd \ub3c4\uc11c\uad00, \uc77c\ubd80 \uac1c\uc778 \uc218\uc9d1\uac00\ub4e4\uc5d0\uac8c \ud769\uc5b4\uc838\uc788\ub294 \uc0c1\ud669\uc774\ub2e4.", "sentence_answer": "\ub808\uc624\ub12c\ub85c\ub294 \uc2e4\ub825\uc788\ub294 \uc815\uce58\uac00\uc600\uace0 \ud398\ub77c\ub77c \ucd5c\ucd08\uc758 \ubcd1\uc6d0 \uac74\uc124\uc5d0 \uad00\uc5ec\ub97c \ud588\uc5c8\ub2e4.", "paragraph_id": "6306261-2-0", "paragraph_question": "question: \ub808\uc624\ub12c\ub85c\ub294 \uc2e4\ub825\uc788\ub294 \uc815\uce58\uac00\uc600\uace0 \ud398\ub77c\ub77c\ub0b4 \uacbd\uc81c\uc801 \ubc88\uc601\uc5d0 \uc9d1\uc911\ud588\ub294\ub370 \ud398\ub77c\ub77c \ucd5c\ucd08\uc758 \ubb34\uc5c7\uc744 \uac74\uc124\ud558\ub294\ub370 \uad00\uc5ec\ud588\ub294\uac00?, context: \ub2c8\ucf5c\ub85c 3\uc138 \ud1b5\uce58 \uc2dc\uae30\uc5d0 \uc774\ub7ec\ud55c \uacfc\uc815\ub4e4\uc774 \uc5c6\uc5c8\ub358 \uac83\ub4e4\uc744 \ub808\uc624\ub12c\ub85c\ub294 \uc815\uce58\uc801 \uc77c\ub4e4\uacfc \ud398\ub77c\ub77c\ub0b4 \uacbd\uc81c\uc801 \ubc88\uc601\uc5d0 \ud55c\uce35 \ub354 \uc9d1\uc911\ud558\uc600\ub2e4. \ub808\uc624\ub12c\ub85c\ub294 \uc2e4\ub825\uc788\ub294 \uc815\uce58\uac00\uc600\uace0 \ud398\ub77c\ub77c \ucd5c\ucd08\uc758 \ubcd1\uc6d0 \uac74\uc124\uc5d0 \uad00\uc5ec\ub97c \ud588\uc5c8\ub2e4. \ud558\uc9c0\ub9cc \uadf8\ub294 \uc8fc\ub85c \uc2a4\uc2a4\ub85c\ub97c \ubb38\ud654\uc778\uc73c\ub85c\uc11c \uad6c\ubd84\uc9c0\uc5c8\ub2e4. \ub808\uc628 \ubc14\ud2f0\uc2a4\ud0c0 \uc54c\ubca0\ub974\ud2f0\ub294 \ub808\uc624\ub12c\ub85c\uc758 \uc758\ub8b0\uc5d0 \uc758\ud574 \uac74\ucd95\ub860\uc744 \uc800\uc220\ud558\uc600\uc73c\uba70, \ud398\ub77c\ub77c\uc758 \uad81\uc804\uc5d0\ub294 \ud53c\uc0ac\ub12c\ub85c, \uc57c\ucf54\ud3ec \ubca8\ub9ac\ub2c8, \uc870\ubc18\ub2c8 \ub2e4 \uc624\ub9ac\uc62c\ub85c, \uc548\ub4dc\ub808\uc544 \ub9cc\ud14c\ub0d0, \ud53c\uc5d0\ub85c \ub378\ub77c \ud504\ub780\uccb4\uc2a4\uce74, \ub124\ub35c\ub780\ub4dc \ucd9c\uc2e0\uc758 \ub85c\ud788\uc5b4\ub974 \ud310 \ub370\ub974 \ubca0\uc774\ub358 \ub4f1\uc774 \uc608\uc220\uac00\ub4e4\uc774 \uc788\uc5c8\ub2e4. \uadf8\uc758 \uac1c\uc778 \uc131\ubb34\uc77c\uacfc\uc11c\ub294 \ubd84\ud560\ub41c \uc0c1\ud0dc\ub85c 1958\ub144 \ud06c\ub9ac\uc2a4\ud2f0 \uacbd\ub9e4\uc5d0\uc11c \ub791\uac00\ud1a1(Llangattock) \ub0a8\uc791\uc5d0\uac8c \ud310\ub9e4\ub418\uae30\ub3c4 \ud558\uc600\ub2e4. \uc774\ud6c4\ub85c \uc774 \uc11c\uc801\uc740 \ub791\uac00\ud1a1 \uc131\ubb34\uc77c\uacfc\uc11c\ub85c \uc54c\ub824\uc9c0\uac8c \ub418\uc5c8\ub2e4. \uc774\ub294 \uc870\ub974\uc870 \ub2ec\ub808\ub9c8\ub0d0(Giorgio d\u00b4Alemagna)\uc758 \uc608\uc220\uc801 \ubc29\ud5a5\ud558\uc5d0\uc11c \ub9cc\ub4e4\uc5b4\uc84c\uace0 \ub9c8\ud14c\uc624 \ub370 \ud30c\uc2a4\ud2f0(Matteo De Pasti)\uc640 \uc57c\ucf54\ud3ec \ub9c8\ub0d0\ub2c8\ubaa8(Jacopo Magnanimo) \uac19\uc740 \ud654\uac00\ub4e4\ub3c4 \uc5ec\uae30\uc5d0 \uae30\uc5ec\ub97c \ud558\uc600\ub2e4. \ud30c\ub9ac\uc758 \ub8e8\ube0c\ub974, \ub374\ub9c8\ud06c \uad6d\ub9bd \ub3c4\uc11c\uad00, \uc77c\ubd80 \uac1c\uc778 \uc218\uc9d1\uac00\ub4e4\uc5d0\uac8c \ud769\uc5b4\uc838\uc788\ub294 \uc0c1\ud669\uc774\ub2e4."} {"question": "\ud070\uace0\ub798\uc758 \uc218\uc5fc\ud310\uc740 \ubb34\uc5c7\uc73c\ub85c \uc774\ub8e8\uc5b4\uc838 \uc788\ub294\uac00?", "paragraph": "\ud070\uace0\ub798\ub294 \uc218\uc5fc\uc744 \uc774\uc6a9\ud574 \uba39\uc774\ub97c \uac78\ub7ec \uba39\uc73c\uba70, \uadf8\ub4e4\uc758 \uba39\uc774\uac00 \ub418\ub294 \ub3d9\ubb3c\uc740 \uc791\uc740 \ubb3c\uace0\uae30, \uc624\uc9d5\uc5b4, \ud06c\ub9b4, \uac11\uac01\ub958 \ub4f1\uc774\ub2e4. \uba39\uc744 \ub54c\ub294 \ube44\uad50\uc801 \ube60\ub974\uac8c \ud5e4\uc5c4\uce58\uba74\uc11c(\uc2dc\uc18d 11\ud0ac\ub85c\ubbf8\ud130) \ub3d9\uc2dc\uc5d0 \ud131\uc744 \ud06c\uac8c \ubc8c\ub9ac\uba70, \uc774\uc5d0 \ub530\ub77c \uc57d 70 \uc138\uc81c\uacf1\ubbf8\ud130\uc758 \ubc14\ub2f7\ubb3c\uc744 \uba38\uae08\ub294\ub2e4. \ubc14\ub2f7\ubb3c\uc744 \uba38\uae08\uc740 \ud6c4\uc5d0\ub294 \uc774\ub97c \uc218\uc5fc\uc744 \ud1b5\ud574\uc11c \ubc49\uc5b4\ub0b4\uba70, \uadf8 \ud6c4 \uc785 \uc18d\uc5d0\ub294 \uba39\uc774\uac00 \ub0a8\ub294\ub2e4. \uc131\uccb4\ub294 \uc785\uc758 \uc591\ucabd\uc5d0 262\uc5d0\uc11c 473\uac1c\uc758 \uc218\uc5fc\ud310\uc774 \uc788\ub2e4. \uac01 \ud310\uc740 \ucf00\ub77c\ud2f4\uc73c\ub85c \uc774\ub8e8\uc5b4\uc838 \uc788\uc73c\uba70, \uc5ec\uae30\uc5d0\uc11c \ubd80\ub4dc\ub7ec\uc6b4 \ud138 \uac19\uc740 \uc218\uc5fc\uc774 \uc0dd\uc131\ub41c\ub2e4. \uac01 \ud310\uc758 \uae38\uc774\ub294 76\uc13c\ud2f0\ubbf8\ud130\uc774\uba70, \ub108\ube44\ub294 30\uc13c\ud2f0\ubbf8\ud130 \uc815\ub3c4\uc774\ub2e4. \uc774\ub4e4\uc740 \uc885\uc885 200\ubbf8\ud130 \uc774\uc0c1\uae4c\uc9c0 \uc7a0\uc218\ud558\ub294\ub370, \uc5ec\uae30\uc11c \uc8fc\ub85c \ud06c\ub9b4 \ub5bc\ub97c \uc12d\ucde8\ud55c\ub2e4. \ud55c\ubc88 \uba38\uae08\uc744 \ub54c\ub9c8\ub2e4 10\ud0ac\ub85c\uadf8\ub7a8 \uc815\ub3c4\uc758 \ud06c\ub9b4\uc744 \uc5bb\uc744 \uc218 \uc788\ub2e4. \ud55c \uac1c\uccb4\ub294 \ucd5c\ub300 1,800\ud0ac\ub85c\uadf8\ub7a8\uc758 \uba39\uc774\ub97c \uba39\uc744 \uc218 \uc788\uc73c\uba70, \uadf8\uc5d0 \ub530\ub77c \uacfc\ud559\uc790\ub4e4\uc740 \uc774\ub4e4\uc774 \uc778\uac04\uacfc \ube44\uc2b7\ud558\uac8c \ud558\ub8e8 3\uc2dc\uac04\uc744 \ud558\ub8e8\uc5d0 \ud544\uc694\ud55c \ub9cc\ud07c\uc758 \uc74c\uc2dd\uc744 \uc12d\ucde8\ud558\ub294 \ub370\uc5d0 \uc0ac\uc6a9\ud55c\ub2e4\uace0 \uacb0\ub860\uc9c0\uc5c8\ub2e4. \uba39\uc774\uc758 \ubc00\ub3c4\uac00 \ub192\uc9c0 \uc54a\uac70\ub098 \ub108\ubb34 \uae4a\uc740 \uc218\uc2ec\uc5d0 \uc788\uc73c\uba74 \ub354 \uc801\uc808\ud55c \uba39\uc774\ub97c \ucc3e\uc73c\ub7ec \ub3cc\uc544\ub2e4\ub2cc\ub2e4. \ub3cc\uace0\ub798\ucc98\ub7fc \ud070\uace0\ub798 \ub610\ud55c \ube60\ub978 \uc18d\ub3c4\ub85c \ubb3c\uace0\uae30 \ub5bc\ub97c \ud55c \uacf3\uc73c\ub85c \ubab0\uc544 \uc12d\ucde8\ud558\ub294 \uac83\uc774 \ubaa9\uaca9\ub418\uc5c8\ub2e4.", "answer": "\ucf00\ub77c\ud2f4", "sentence": "\uac01 \ud310\uc740 \ucf00\ub77c\ud2f4 \uc73c\ub85c \uc774\ub8e8\uc5b4\uc838 \uc788\uc73c\uba70, \uc5ec\uae30\uc5d0\uc11c \ubd80\ub4dc\ub7ec\uc6b4 \ud138 \uac19\uc740 \uc218\uc5fc\uc774 \uc0dd\uc131\ub41c\ub2e4.", "paragraph_sentence": "\ud070\uace0\ub798\ub294 \uc218\uc5fc\uc744 \uc774\uc6a9\ud574 \uba39\uc774\ub97c \uac78\ub7ec \uba39\uc73c\uba70, \uadf8\ub4e4\uc758 \uba39\uc774\uac00 \ub418\ub294 \ub3d9\ubb3c\uc740 \uc791\uc740 \ubb3c\uace0\uae30, \uc624\uc9d5\uc5b4, \ud06c\ub9b4, \uac11\uac01\ub958 \ub4f1\uc774\ub2e4. \uba39\uc744 \ub54c\ub294 \ube44\uad50\uc801 \ube60\ub974\uac8c \ud5e4\uc5c4\uce58\uba74\uc11c(\uc2dc\uc18d 11\ud0ac\ub85c\ubbf8\ud130) \ub3d9\uc2dc\uc5d0 \ud131\uc744 \ud06c\uac8c \ubc8c\ub9ac\uba70, \uc774\uc5d0 \ub530\ub77c \uc57d 70 \uc138\uc81c\uacf1\ubbf8\ud130\uc758 \ubc14\ub2f7\ubb3c\uc744 \uba38\uae08\ub294\ub2e4. \ubc14\ub2f7\ubb3c\uc744 \uba38\uae08\uc740 \ud6c4\uc5d0\ub294 \uc774\ub97c \uc218\uc5fc\uc744 \ud1b5\ud574\uc11c \ubc49\uc5b4\ub0b4\uba70, \uadf8 \ud6c4 \uc785 \uc18d\uc5d0\ub294 \uba39\uc774\uac00 \ub0a8\ub294\ub2e4. \uc131\uccb4\ub294 \uc785\uc758 \uc591\ucabd\uc5d0 262\uc5d0\uc11c 473\uac1c\uc758 \uc218\uc5fc\ud310\uc774 \uc788\ub2e4. \uac01 \ud310\uc740 \ucf00\ub77c\ud2f4 \uc73c\ub85c \uc774\ub8e8\uc5b4\uc838 \uc788\uc73c\uba70, \uc5ec\uae30\uc5d0\uc11c \ubd80\ub4dc\ub7ec\uc6b4 \ud138 \uac19\uc740 \uc218\uc5fc\uc774 \uc0dd\uc131\ub41c\ub2e4. \uac01 \ud310\uc758 \uae38\uc774\ub294 76\uc13c\ud2f0\ubbf8\ud130\uc774\uba70, \ub108\ube44\ub294 30\uc13c\ud2f0\ubbf8\ud130 \uc815\ub3c4\uc774\ub2e4. \uc774\ub4e4\uc740 \uc885\uc885 200\ubbf8\ud130 \uc774\uc0c1\uae4c\uc9c0 \uc7a0\uc218\ud558\ub294\ub370, \uc5ec\uae30\uc11c \uc8fc\ub85c \ud06c\ub9b4 \ub5bc\ub97c \uc12d\ucde8\ud55c\ub2e4. \ud55c\ubc88 \uba38\uae08\uc744 \ub54c\ub9c8\ub2e4 10\ud0ac\ub85c\uadf8\ub7a8 \uc815\ub3c4\uc758 \ud06c\ub9b4\uc744 \uc5bb\uc744 \uc218 \uc788\ub2e4. \ud55c \uac1c\uccb4\ub294 \ucd5c\ub300 1,800\ud0ac\ub85c\uadf8\ub7a8\uc758 \uba39\uc774\ub97c \uba39\uc744 \uc218 \uc788\uc73c\uba70, \uadf8\uc5d0 \ub530\ub77c \uacfc\ud559\uc790\ub4e4\uc740 \uc774\ub4e4\uc774 \uc778\uac04\uacfc \ube44\uc2b7\ud558\uac8c \ud558\ub8e8 3\uc2dc\uac04\uc744 \ud558\ub8e8\uc5d0 \ud544\uc694\ud55c \ub9cc\ud07c\uc758 \uc74c\uc2dd\uc744 \uc12d\ucde8\ud558\ub294 \ub370\uc5d0 \uc0ac\uc6a9\ud55c\ub2e4\uace0 \uacb0\ub860\uc9c0\uc5c8\ub2e4. \uba39\uc774\uc758 \ubc00\ub3c4\uac00 \ub192\uc9c0 \uc54a\uac70\ub098 \ub108\ubb34 \uae4a\uc740 \uc218\uc2ec\uc5d0 \uc788\uc73c\uba74 \ub354 \uc801\uc808\ud55c \uba39\uc774\ub97c \ucc3e\uc73c\ub7ec \ub3cc\uc544\ub2e4\ub2cc\ub2e4. \ub3cc\uace0\ub798\ucc98\ub7fc \ud070\uace0\ub798 \ub610\ud55c \ube60\ub978 \uc18d\ub3c4\ub85c \ubb3c\uace0\uae30 \ub5bc\ub97c \ud55c \uacf3\uc73c\ub85c \ubab0\uc544 \uc12d\ucde8\ud558\ub294 \uac83\uc774 \ubaa9\uaca9\ub418\uc5c8\ub2e4.", "paragraph_answer": "\ud070\uace0\ub798\ub294 \uc218\uc5fc\uc744 \uc774\uc6a9\ud574 \uba39\uc774\ub97c \uac78\ub7ec \uba39\uc73c\uba70, \uadf8\ub4e4\uc758 \uba39\uc774\uac00 \ub418\ub294 \ub3d9\ubb3c\uc740 \uc791\uc740 \ubb3c\uace0\uae30, \uc624\uc9d5\uc5b4, \ud06c\ub9b4, \uac11\uac01\ub958 \ub4f1\uc774\ub2e4. \uba39\uc744 \ub54c\ub294 \ube44\uad50\uc801 \ube60\ub974\uac8c \ud5e4\uc5c4\uce58\uba74\uc11c(\uc2dc\uc18d 11\ud0ac\ub85c\ubbf8\ud130) \ub3d9\uc2dc\uc5d0 \ud131\uc744 \ud06c\uac8c \ubc8c\ub9ac\uba70, \uc774\uc5d0 \ub530\ub77c \uc57d 70 \uc138\uc81c\uacf1\ubbf8\ud130\uc758 \ubc14\ub2f7\ubb3c\uc744 \uba38\uae08\ub294\ub2e4. \ubc14\ub2f7\ubb3c\uc744 \uba38\uae08\uc740 \ud6c4\uc5d0\ub294 \uc774\ub97c \uc218\uc5fc\uc744 \ud1b5\ud574\uc11c \ubc49\uc5b4\ub0b4\uba70, \uadf8 \ud6c4 \uc785 \uc18d\uc5d0\ub294 \uba39\uc774\uac00 \ub0a8\ub294\ub2e4. \uc131\uccb4\ub294 \uc785\uc758 \uc591\ucabd\uc5d0 262\uc5d0\uc11c 473\uac1c\uc758 \uc218\uc5fc\ud310\uc774 \uc788\ub2e4. \uac01 \ud310\uc740 \ucf00\ub77c\ud2f4 \uc73c\ub85c \uc774\ub8e8\uc5b4\uc838 \uc788\uc73c\uba70, \uc5ec\uae30\uc5d0\uc11c \ubd80\ub4dc\ub7ec\uc6b4 \ud138 \uac19\uc740 \uc218\uc5fc\uc774 \uc0dd\uc131\ub41c\ub2e4. \uac01 \ud310\uc758 \uae38\uc774\ub294 76\uc13c\ud2f0\ubbf8\ud130\uc774\uba70, \ub108\ube44\ub294 30\uc13c\ud2f0\ubbf8\ud130 \uc815\ub3c4\uc774\ub2e4. \uc774\ub4e4\uc740 \uc885\uc885 200\ubbf8\ud130 \uc774\uc0c1\uae4c\uc9c0 \uc7a0\uc218\ud558\ub294\ub370, \uc5ec\uae30\uc11c \uc8fc\ub85c \ud06c\ub9b4 \ub5bc\ub97c \uc12d\ucde8\ud55c\ub2e4. \ud55c\ubc88 \uba38\uae08\uc744 \ub54c\ub9c8\ub2e4 10\ud0ac\ub85c\uadf8\ub7a8 \uc815\ub3c4\uc758 \ud06c\ub9b4\uc744 \uc5bb\uc744 \uc218 \uc788\ub2e4. \ud55c \uac1c\uccb4\ub294 \ucd5c\ub300 1,800\ud0ac\ub85c\uadf8\ub7a8\uc758 \uba39\uc774\ub97c \uba39\uc744 \uc218 \uc788\uc73c\uba70, \uadf8\uc5d0 \ub530\ub77c \uacfc\ud559\uc790\ub4e4\uc740 \uc774\ub4e4\uc774 \uc778\uac04\uacfc \ube44\uc2b7\ud558\uac8c \ud558\ub8e8 3\uc2dc\uac04\uc744 \ud558\ub8e8\uc5d0 \ud544\uc694\ud55c \ub9cc\ud07c\uc758 \uc74c\uc2dd\uc744 \uc12d\ucde8\ud558\ub294 \ub370\uc5d0 \uc0ac\uc6a9\ud55c\ub2e4\uace0 \uacb0\ub860\uc9c0\uc5c8\ub2e4. \uba39\uc774\uc758 \ubc00\ub3c4\uac00 \ub192\uc9c0 \uc54a\uac70\ub098 \ub108\ubb34 \uae4a\uc740 \uc218\uc2ec\uc5d0 \uc788\uc73c\uba74 \ub354 \uc801\uc808\ud55c \uba39\uc774\ub97c \ucc3e\uc73c\ub7ec \ub3cc\uc544\ub2e4\ub2cc\ub2e4. \ub3cc\uace0\ub798\ucc98\ub7fc \ud070\uace0\ub798 \ub610\ud55c \ube60\ub978 \uc18d\ub3c4\ub85c \ubb3c\uace0\uae30 \ub5bc\ub97c \ud55c \uacf3\uc73c\ub85c \ubab0\uc544 \uc12d\ucde8\ud558\ub294 \uac83\uc774 \ubaa9\uaca9\ub418\uc5c8\ub2e4.", "sentence_answer": "\uac01 \ud310\uc740 \ucf00\ub77c\ud2f4 \uc73c\ub85c \uc774\ub8e8\uc5b4\uc838 \uc788\uc73c\uba70, \uc5ec\uae30\uc5d0\uc11c \ubd80\ub4dc\ub7ec\uc6b4 \ud138 \uac19\uc740 \uc218\uc5fc\uc774 \uc0dd\uc131\ub41c\ub2e4.", "paragraph_id": "5910097-4-2", "paragraph_question": "question: \ud070\uace0\ub798\uc758 \uc218\uc5fc\ud310\uc740 \ubb34\uc5c7\uc73c\ub85c \uc774\ub8e8\uc5b4\uc838 \uc788\ub294\uac00?, context: \ud070\uace0\ub798\ub294 \uc218\uc5fc\uc744 \uc774\uc6a9\ud574 \uba39\uc774\ub97c \uac78\ub7ec \uba39\uc73c\uba70, \uadf8\ub4e4\uc758 \uba39\uc774\uac00 \ub418\ub294 \ub3d9\ubb3c\uc740 \uc791\uc740 \ubb3c\uace0\uae30, \uc624\uc9d5\uc5b4, \ud06c\ub9b4, \uac11\uac01\ub958 \ub4f1\uc774\ub2e4. \uba39\uc744 \ub54c\ub294 \ube44\uad50\uc801 \ube60\ub974\uac8c \ud5e4\uc5c4\uce58\uba74\uc11c(\uc2dc\uc18d 11\ud0ac\ub85c\ubbf8\ud130) \ub3d9\uc2dc\uc5d0 \ud131\uc744 \ud06c\uac8c \ubc8c\ub9ac\uba70, \uc774\uc5d0 \ub530\ub77c \uc57d 70 \uc138\uc81c\uacf1\ubbf8\ud130\uc758 \ubc14\ub2f7\ubb3c\uc744 \uba38\uae08\ub294\ub2e4. \ubc14\ub2f7\ubb3c\uc744 \uba38\uae08\uc740 \ud6c4\uc5d0\ub294 \uc774\ub97c \uc218\uc5fc\uc744 \ud1b5\ud574\uc11c \ubc49\uc5b4\ub0b4\uba70, \uadf8 \ud6c4 \uc785 \uc18d\uc5d0\ub294 \uba39\uc774\uac00 \ub0a8\ub294\ub2e4. \uc131\uccb4\ub294 \uc785\uc758 \uc591\ucabd\uc5d0 262\uc5d0\uc11c 473\uac1c\uc758 \uc218\uc5fc\ud310\uc774 \uc788\ub2e4. \uac01 \ud310\uc740 \ucf00\ub77c\ud2f4\uc73c\ub85c \uc774\ub8e8\uc5b4\uc838 \uc788\uc73c\uba70, \uc5ec\uae30\uc5d0\uc11c \ubd80\ub4dc\ub7ec\uc6b4 \ud138 \uac19\uc740 \uc218\uc5fc\uc774 \uc0dd\uc131\ub41c\ub2e4. \uac01 \ud310\uc758 \uae38\uc774\ub294 76\uc13c\ud2f0\ubbf8\ud130\uc774\uba70, \ub108\ube44\ub294 30\uc13c\ud2f0\ubbf8\ud130 \uc815\ub3c4\uc774\ub2e4. \uc774\ub4e4\uc740 \uc885\uc885 200\ubbf8\ud130 \uc774\uc0c1\uae4c\uc9c0 \uc7a0\uc218\ud558\ub294\ub370, \uc5ec\uae30\uc11c \uc8fc\ub85c \ud06c\ub9b4 \ub5bc\ub97c \uc12d\ucde8\ud55c\ub2e4. \ud55c\ubc88 \uba38\uae08\uc744 \ub54c\ub9c8\ub2e4 10\ud0ac\ub85c\uadf8\ub7a8 \uc815\ub3c4\uc758 \ud06c\ub9b4\uc744 \uc5bb\uc744 \uc218 \uc788\ub2e4. \ud55c \uac1c\uccb4\ub294 \ucd5c\ub300 1,800\ud0ac\ub85c\uadf8\ub7a8\uc758 \uba39\uc774\ub97c \uba39\uc744 \uc218 \uc788\uc73c\uba70, \uadf8\uc5d0 \ub530\ub77c \uacfc\ud559\uc790\ub4e4\uc740 \uc774\ub4e4\uc774 \uc778\uac04\uacfc \ube44\uc2b7\ud558\uac8c \ud558\ub8e8 3\uc2dc\uac04\uc744 \ud558\ub8e8\uc5d0 \ud544\uc694\ud55c \ub9cc\ud07c\uc758 \uc74c\uc2dd\uc744 \uc12d\ucde8\ud558\ub294 \ub370\uc5d0 \uc0ac\uc6a9\ud55c\ub2e4\uace0 \uacb0\ub860\uc9c0\uc5c8\ub2e4. \uba39\uc774\uc758 \ubc00\ub3c4\uac00 \ub192\uc9c0 \uc54a\uac70\ub098 \ub108\ubb34 \uae4a\uc740 \uc218\uc2ec\uc5d0 \uc788\uc73c\uba74 \ub354 \uc801\uc808\ud55c \uba39\uc774\ub97c \ucc3e\uc73c\ub7ec \ub3cc\uc544\ub2e4\ub2cc\ub2e4. \ub3cc\uace0\ub798\ucc98\ub7fc \ud070\uace0\ub798 \ub610\ud55c \ube60\ub978 \uc18d\ub3c4\ub85c \ubb3c\uace0\uae30 \ub5bc\ub97c \ud55c \uacf3\uc73c\ub85c \ubab0\uc544 \uc12d\ucde8\ud558\ub294 \uac83\uc774 \ubaa9\uaca9\ub418\uc5c8\ub2e4."} {"question": "\ub2ec\ub77c\uc774 \ub77c\ub9c8 13\uc138\uc758 \ubca0\uc774\uc9d5 \uccb4\ub958 \uc911\uc5d0 \uccad\ub098\ub77c\uac00 \ud2f0\ubca0\ud2b8 \uc601\uc8fc\ub85c \uc639\ub9bd\ud558\ub824 \ud55c \uc0ac\ub78c\uc740 \ub204\uad6c\uc778\uac00?", "paragraph": "\uc601\uad6d\uc5d0 \ucad3\uae34 \ub2ec\ub77c\uc774 \ub77c\ub9c8 13\uc138\ub294 1904\ub144 \ub9d0\uc5d0 \ubabd\uace8\uc758 \uc6b0\ub974\uac00(\ud604\uc7ac\uc758 \uc6b8\ub780\ubc14\ud1a0\ub974)\uc5d0 \ub9dd\uba85\ud558\uace0 \ud604\uc9c0\uc758 \uac94 \ub8e8\ud06c\ud30c\uc7a5\uc5d0\uc11c \uc655\uc774\uae30\ub3c4 \ud55c \uc9c0\uc5d0\ud504\uce35\ub2f9\ubc14\u00b7\ud638\ud1a1\ud1a0 8\uc138\uc758 \uc2e0\uc138\ub97c \uc84c\ub2e4. 13\uc138 \uccb4\ub958 \ub54c \ub7ec\uc2dc\uc544\ub294 \ubabd\uace8 \uc601\uc0ac\uc640 \ubd80\uc784 \ub3c4\uc911\uc758 \uc911\uad6d \ubca0\uc774\uc9d5 \uc8fc\uc7ac \ub300\uc0ac \ub4f1\uc744 13\uc138\uc5d0 \uc811\ucd09\uc2dc\ucf1c\uc11c \uad00\uacc4 \uac15\ud654\ub97c \uaf80\ud558\uace0 \uc788\ub2e4. \uadf8\ub7ec\ub098 \uac80\uc18c\ud55c \uc0dd\ud65c\uc744 \uc88b\uc544\ud558\ub294 13\uc138\uc640 \uc9c0\uc5d0\ud504\uce35\ub2f9\ubc14\u00b7\ud638\ud1a1\ud1a0 8\uc138\uc640 \ub73b\uc774 \ub9de\uc9c0 \uc54a\uace0 13\uc138\ub294 1906\ub144 \uac00\uc744\uc5d0\ub294 \ubabd\uace8\uc744 \ub5a0\ub098\uace0 \ucff5\ubd90\uc0ac\uc5d0 \uc7a0\uc2dc \uba38\ubb38 \ub4a4 1908\ub144 4\uc6d4\uc5d0 \uc911\uad6d \uc0b0\uc2dc \uc131\uc758 \uc624\ub300\uc0b0\uc5d0 \ub3c4\ucc29\ud588\ub2e4. \uc624\ub300\uc0b0\uc5d0\uc11c \ubbf8\uad6d \ub300\uc0ac \ub85d \ud790, \ub2c8\uc2dc\ud63c\uac04\uc9c0\uc758 \uc624\uc624\ud0c0\ub2c8 \uc190 \uc720\uc6b0 \ub4f1\uacfc \uba74\ub2f4\ud558\uace0 \uc788\ub2e4. \uc774\uc5b4 10\uc6d4 14\uc77c \ubca0\uc774\uc9d5\uc5d0\uc11c \uc11c\ud0dc\ud6c4, \uad11\uc11c\uc81c\uc640 \uba74\ub2f4\ud558\uace0 \uc788\ub2e4. \uc774 \ub54c, \uc77c\ubcf8 \ub300\uc0ac\uad00\uc5d0\ub3c4 1\uc8fc\uc77c \uba38\ubb3c\uace0 \uc788\ub2e4. \uc911\uad6d \uc8fc\uc7ac \ub300\uc0ac\uc758 \ud558\uc57c\uc2dc \uace4\uc2a4\ucf00\uac00 \uadf9\uc9c4\ud788 \ub300\uc811\ud55c \uac83\uacfc \uc624\ub300\uc0b0\uc5d0\uc11c \uc624\uc624\ud0c0\ub2c8 \uc190 \uc720\uc6b0\uac00 \uc77c\ubcf8\uc774 \ubd88\uad50 \uad6d\uac00\uc784\uc744 \uc124\ud30c\ud55c \ud0d3\uc5d0 \uc774\ud6c4 13\uc138\ub294 \uc0c1\ub2f9\ud55c \uc77c\ubcf8 \ud3b8\ub4e4\uae30\uc5d0 \ub41c\ub2e4. 13\uc138\uc758 \ubca0\uc774\uc9d5 \uccb4\ub958 \uc911 \uccad\ub098\ub77c\ub294 \ud2f0\ubca0\ud2b8 \uc601\uc8fc\ub85c\uc11c \ud310\uccb8 \ub77c\ub9c8 9\uc138\ub97c \uc639\ub9bd\ud558\ub824 \ud588\ub2e4. \uadf8\ub798\uc11c 13\uc138\ub294 \ubab0\ub798 \uc601\uad6d, \ub7ec\uc2dc\uc544, \uc77c\ubcf8\uc758 \uc911\uad6d \ubca0\uc774\uc9d5 \uc8fc\uc7ac \ub300\uc0ac\uc5d0 \ubc00\uc0ac\ub97c \ubcf4\ub0b4\uace0 \ub3c4\uc6c0\uc744 \ubc1b\uace0 \uc788\ub2e4.1908\ub144 12\uc6d4 \ubca0\uc774\uc9d5\uc744 \uc11c\uc11c 1909\ub144 12\uc6d4 \ub77c\uc0ac\uc5d0 \ub3cc\uc544\uc628\ub2e4.", "answer": "\ub77c\ub9c8 9\uc138", "sentence": "13\uc138\uc758 \ubca0\uc774\uc9d5 \uccb4\ub958 \uc911 \uccad\ub098\ub77c\ub294 \ud2f0\ubca0\ud2b8 \uc601\uc8fc\ub85c\uc11c \ud310\uccb8 \ub77c\ub9c8 9\uc138 \ub97c \uc639\ub9bd\ud558\ub824 \ud588\ub2e4.", "paragraph_sentence": "\uc601\uad6d\uc5d0 \ucad3\uae34 \ub2ec\ub77c\uc774 \ub77c\ub9c8 13\uc138\ub294 1904\ub144 \ub9d0\uc5d0 \ubabd\uace8\uc758 \uc6b0\ub974\uac00(\ud604\uc7ac\uc758 \uc6b8\ub780\ubc14\ud1a0\ub974)\uc5d0 \ub9dd\uba85\ud558\uace0 \ud604\uc9c0\uc758 \uac94 \ub8e8\ud06c\ud30c\uc7a5\uc5d0\uc11c \uc655\uc774\uae30\ub3c4 \ud55c \uc9c0\uc5d0\ud504\uce35\ub2f9\ubc14\u00b7\ud638\ud1a1\ud1a0 8\uc138\uc758 \uc2e0\uc138\ub97c \uc84c\ub2e4. 13\uc138 \uccb4\ub958 \ub54c \ub7ec\uc2dc\uc544\ub294 \ubabd\uace8 \uc601\uc0ac\uc640 \ubd80\uc784 \ub3c4\uc911\uc758 \uc911\uad6d \ubca0\uc774\uc9d5 \uc8fc\uc7ac \ub300\uc0ac \ub4f1\uc744 13\uc138\uc5d0 \uc811\ucd09\uc2dc\ucf1c\uc11c \uad00\uacc4 \uac15\ud654\ub97c \uaf80\ud558\uace0 \uc788\ub2e4. \uadf8\ub7ec\ub098 \uac80\uc18c\ud55c \uc0dd\ud65c\uc744 \uc88b\uc544\ud558\ub294 13\uc138\uc640 \uc9c0\uc5d0\ud504\uce35\ub2f9\ubc14\u00b7\ud638\ud1a1\ud1a0 8\uc138\uc640 \ub73b\uc774 \ub9de\uc9c0 \uc54a\uace0 13\uc138\ub294 1906\ub144 \uac00\uc744\uc5d0\ub294 \ubabd\uace8\uc744 \ub5a0\ub098\uace0 \ucff5\ubd90\uc0ac\uc5d0 \uc7a0\uc2dc \uba38\ubb38 \ub4a4 1908\ub144 4\uc6d4\uc5d0 \uc911\uad6d \uc0b0\uc2dc \uc131\uc758 \uc624\ub300\uc0b0\uc5d0 \ub3c4\ucc29\ud588\ub2e4. \uc624\ub300\uc0b0\uc5d0\uc11c \ubbf8\uad6d \ub300\uc0ac \ub85d \ud790, \ub2c8\uc2dc\ud63c\uac04\uc9c0\uc758 \uc624\uc624\ud0c0\ub2c8 \uc190 \uc720\uc6b0 \ub4f1\uacfc \uba74\ub2f4\ud558\uace0 \uc788\ub2e4. \uc774\uc5b4 10\uc6d4 14\uc77c \ubca0\uc774\uc9d5\uc5d0\uc11c \uc11c\ud0dc\ud6c4, \uad11\uc11c\uc81c\uc640 \uba74\ub2f4\ud558\uace0 \uc788\ub2e4. \uc774 \ub54c, \uc77c\ubcf8 \ub300\uc0ac\uad00\uc5d0\ub3c4 1\uc8fc\uc77c \uba38\ubb3c\uace0 \uc788\ub2e4. \uc911\uad6d \uc8fc\uc7ac \ub300\uc0ac\uc758 \ud558\uc57c\uc2dc \uace4\uc2a4\ucf00\uac00 \uadf9\uc9c4\ud788 \ub300\uc811\ud55c \uac83\uacfc \uc624\ub300\uc0b0\uc5d0\uc11c \uc624\uc624\ud0c0\ub2c8 \uc190 \uc720\uc6b0\uac00 \uc77c\ubcf8\uc774 \ubd88\uad50 \uad6d\uac00\uc784\uc744 \uc124\ud30c\ud55c \ud0d3\uc5d0 \uc774\ud6c4 13\uc138\ub294 \uc0c1\ub2f9\ud55c \uc77c\ubcf8 \ud3b8\ub4e4\uae30\uc5d0 \ub41c\ub2e4. 13\uc138\uc758 \ubca0\uc774\uc9d5 \uccb4\ub958 \uc911 \uccad\ub098\ub77c\ub294 \ud2f0\ubca0\ud2b8 \uc601\uc8fc\ub85c\uc11c \ud310\uccb8 \ub77c\ub9c8 9\uc138 \ub97c \uc639\ub9bd\ud558\ub824 \ud588\ub2e4. \uadf8\ub798\uc11c 13\uc138\ub294 \ubab0\ub798 \uc601\uad6d, \ub7ec\uc2dc\uc544, \uc77c\ubcf8\uc758 \uc911\uad6d \ubca0\uc774\uc9d5 \uc8fc\uc7ac \ub300\uc0ac\uc5d0 \ubc00\uc0ac\ub97c \ubcf4\ub0b4\uace0 \ub3c4\uc6c0\uc744 \ubc1b\uace0 \uc788\ub2e4.1908\ub144 12\uc6d4 \ubca0\uc774\uc9d5\uc744 \uc11c\uc11c 1909\ub144 12\uc6d4 \ub77c\uc0ac\uc5d0 \ub3cc\uc544\uc628\ub2e4.", "paragraph_answer": "\uc601\uad6d\uc5d0 \ucad3\uae34 \ub2ec\ub77c\uc774 \ub77c\ub9c8 13\uc138\ub294 1904\ub144 \ub9d0\uc5d0 \ubabd\uace8\uc758 \uc6b0\ub974\uac00(\ud604\uc7ac\uc758 \uc6b8\ub780\ubc14\ud1a0\ub974)\uc5d0 \ub9dd\uba85\ud558\uace0 \ud604\uc9c0\uc758 \uac94 \ub8e8\ud06c\ud30c\uc7a5\uc5d0\uc11c \uc655\uc774\uae30\ub3c4 \ud55c \uc9c0\uc5d0\ud504\uce35\ub2f9\ubc14\u00b7\ud638\ud1a1\ud1a0 8\uc138\uc758 \uc2e0\uc138\ub97c \uc84c\ub2e4. 13\uc138 \uccb4\ub958 \ub54c \ub7ec\uc2dc\uc544\ub294 \ubabd\uace8 \uc601\uc0ac\uc640 \ubd80\uc784 \ub3c4\uc911\uc758 \uc911\uad6d \ubca0\uc774\uc9d5 \uc8fc\uc7ac \ub300\uc0ac \ub4f1\uc744 13\uc138\uc5d0 \uc811\ucd09\uc2dc\ucf1c\uc11c \uad00\uacc4 \uac15\ud654\ub97c \uaf80\ud558\uace0 \uc788\ub2e4. \uadf8\ub7ec\ub098 \uac80\uc18c\ud55c \uc0dd\ud65c\uc744 \uc88b\uc544\ud558\ub294 13\uc138\uc640 \uc9c0\uc5d0\ud504\uce35\ub2f9\ubc14\u00b7\ud638\ud1a1\ud1a0 8\uc138\uc640 \ub73b\uc774 \ub9de\uc9c0 \uc54a\uace0 13\uc138\ub294 1906\ub144 \uac00\uc744\uc5d0\ub294 \ubabd\uace8\uc744 \ub5a0\ub098\uace0 \ucff5\ubd90\uc0ac\uc5d0 \uc7a0\uc2dc \uba38\ubb38 \ub4a4 1908\ub144 4\uc6d4\uc5d0 \uc911\uad6d \uc0b0\uc2dc \uc131\uc758 \uc624\ub300\uc0b0\uc5d0 \ub3c4\ucc29\ud588\ub2e4. \uc624\ub300\uc0b0\uc5d0\uc11c \ubbf8\uad6d \ub300\uc0ac \ub85d \ud790, \ub2c8\uc2dc\ud63c\uac04\uc9c0\uc758 \uc624\uc624\ud0c0\ub2c8 \uc190 \uc720\uc6b0 \ub4f1\uacfc \uba74\ub2f4\ud558\uace0 \uc788\ub2e4. \uc774\uc5b4 10\uc6d4 14\uc77c \ubca0\uc774\uc9d5\uc5d0\uc11c \uc11c\ud0dc\ud6c4, \uad11\uc11c\uc81c\uc640 \uba74\ub2f4\ud558\uace0 \uc788\ub2e4. \uc774 \ub54c, \uc77c\ubcf8 \ub300\uc0ac\uad00\uc5d0\ub3c4 1\uc8fc\uc77c \uba38\ubb3c\uace0 \uc788\ub2e4. \uc911\uad6d \uc8fc\uc7ac \ub300\uc0ac\uc758 \ud558\uc57c\uc2dc \uace4\uc2a4\ucf00\uac00 \uadf9\uc9c4\ud788 \ub300\uc811\ud55c \uac83\uacfc \uc624\ub300\uc0b0\uc5d0\uc11c \uc624\uc624\ud0c0\ub2c8 \uc190 \uc720\uc6b0\uac00 \uc77c\ubcf8\uc774 \ubd88\uad50 \uad6d\uac00\uc784\uc744 \uc124\ud30c\ud55c \ud0d3\uc5d0 \uc774\ud6c4 13\uc138\ub294 \uc0c1\ub2f9\ud55c \uc77c\ubcf8 \ud3b8\ub4e4\uae30\uc5d0 \ub41c\ub2e4. 13\uc138\uc758 \ubca0\uc774\uc9d5 \uccb4\ub958 \uc911 \uccad\ub098\ub77c\ub294 \ud2f0\ubca0\ud2b8 \uc601\uc8fc\ub85c\uc11c \ud310\uccb8 \ub77c\ub9c8 9\uc138 \ub97c \uc639\ub9bd\ud558\ub824 \ud588\ub2e4. \uadf8\ub798\uc11c 13\uc138\ub294 \ubab0\ub798 \uc601\uad6d, \ub7ec\uc2dc\uc544, \uc77c\ubcf8\uc758 \uc911\uad6d \ubca0\uc774\uc9d5 \uc8fc\uc7ac \ub300\uc0ac\uc5d0 \ubc00\uc0ac\ub97c \ubcf4\ub0b4\uace0 \ub3c4\uc6c0\uc744 \ubc1b\uace0 \uc788\ub2e4.1908\ub144 12\uc6d4 \ubca0\uc774\uc9d5\uc744 \uc11c\uc11c 1909\ub144 12\uc6d4 \ub77c\uc0ac\uc5d0 \ub3cc\uc544\uc628\ub2e4.", "sentence_answer": "13\uc138\uc758 \ubca0\uc774\uc9d5 \uccb4\ub958 \uc911 \uccad\ub098\ub77c\ub294 \ud2f0\ubca0\ud2b8 \uc601\uc8fc\ub85c\uc11c \ud310\uccb8 \ub77c\ub9c8 9\uc138 \ub97c \uc639\ub9bd\ud558\ub824 \ud588\ub2e4.", "paragraph_id": "6594166-9-1", "paragraph_question": "question: \ub2ec\ub77c\uc774 \ub77c\ub9c8 13\uc138\uc758 \ubca0\uc774\uc9d5 \uccb4\ub958 \uc911\uc5d0 \uccad\ub098\ub77c\uac00 \ud2f0\ubca0\ud2b8 \uc601\uc8fc\ub85c \uc639\ub9bd\ud558\ub824 \ud55c \uc0ac\ub78c\uc740 \ub204\uad6c\uc778\uac00?, context: \uc601\uad6d\uc5d0 \ucad3\uae34 \ub2ec\ub77c\uc774 \ub77c\ub9c8 13\uc138\ub294 1904\ub144 \ub9d0\uc5d0 \ubabd\uace8\uc758 \uc6b0\ub974\uac00(\ud604\uc7ac\uc758 \uc6b8\ub780\ubc14\ud1a0\ub974)\uc5d0 \ub9dd\uba85\ud558\uace0 \ud604\uc9c0\uc758 \uac94 \ub8e8\ud06c\ud30c\uc7a5\uc5d0\uc11c \uc655\uc774\uae30\ub3c4 \ud55c \uc9c0\uc5d0\ud504\uce35\ub2f9\ubc14\u00b7\ud638\ud1a1\ud1a0 8\uc138\uc758 \uc2e0\uc138\ub97c \uc84c\ub2e4. 13\uc138 \uccb4\ub958 \ub54c \ub7ec\uc2dc\uc544\ub294 \ubabd\uace8 \uc601\uc0ac\uc640 \ubd80\uc784 \ub3c4\uc911\uc758 \uc911\uad6d \ubca0\uc774\uc9d5 \uc8fc\uc7ac \ub300\uc0ac \ub4f1\uc744 13\uc138\uc5d0 \uc811\ucd09\uc2dc\ucf1c\uc11c \uad00\uacc4 \uac15\ud654\ub97c \uaf80\ud558\uace0 \uc788\ub2e4. \uadf8\ub7ec\ub098 \uac80\uc18c\ud55c \uc0dd\ud65c\uc744 \uc88b\uc544\ud558\ub294 13\uc138\uc640 \uc9c0\uc5d0\ud504\uce35\ub2f9\ubc14\u00b7\ud638\ud1a1\ud1a0 8\uc138\uc640 \ub73b\uc774 \ub9de\uc9c0 \uc54a\uace0 13\uc138\ub294 1906\ub144 \uac00\uc744\uc5d0\ub294 \ubabd\uace8\uc744 \ub5a0\ub098\uace0 \ucff5\ubd90\uc0ac\uc5d0 \uc7a0\uc2dc \uba38\ubb38 \ub4a4 1908\ub144 4\uc6d4\uc5d0 \uc911\uad6d \uc0b0\uc2dc \uc131\uc758 \uc624\ub300\uc0b0\uc5d0 \ub3c4\ucc29\ud588\ub2e4. \uc624\ub300\uc0b0\uc5d0\uc11c \ubbf8\uad6d \ub300\uc0ac \ub85d \ud790, \ub2c8\uc2dc\ud63c\uac04\uc9c0\uc758 \uc624\uc624\ud0c0\ub2c8 \uc190 \uc720\uc6b0 \ub4f1\uacfc \uba74\ub2f4\ud558\uace0 \uc788\ub2e4. \uc774\uc5b4 10\uc6d4 14\uc77c \ubca0\uc774\uc9d5\uc5d0\uc11c \uc11c\ud0dc\ud6c4, \uad11\uc11c\uc81c\uc640 \uba74\ub2f4\ud558\uace0 \uc788\ub2e4. \uc774 \ub54c, \uc77c\ubcf8 \ub300\uc0ac\uad00\uc5d0\ub3c4 1\uc8fc\uc77c \uba38\ubb3c\uace0 \uc788\ub2e4. \uc911\uad6d \uc8fc\uc7ac \ub300\uc0ac\uc758 \ud558\uc57c\uc2dc \uace4\uc2a4\ucf00\uac00 \uadf9\uc9c4\ud788 \ub300\uc811\ud55c \uac83\uacfc \uc624\ub300\uc0b0\uc5d0\uc11c \uc624\uc624\ud0c0\ub2c8 \uc190 \uc720\uc6b0\uac00 \uc77c\ubcf8\uc774 \ubd88\uad50 \uad6d\uac00\uc784\uc744 \uc124\ud30c\ud55c \ud0d3\uc5d0 \uc774\ud6c4 13\uc138\ub294 \uc0c1\ub2f9\ud55c \uc77c\ubcf8 \ud3b8\ub4e4\uae30\uc5d0 \ub41c\ub2e4. 13\uc138\uc758 \ubca0\uc774\uc9d5 \uccb4\ub958 \uc911 \uccad\ub098\ub77c\ub294 \ud2f0\ubca0\ud2b8 \uc601\uc8fc\ub85c\uc11c \ud310\uccb8 \ub77c\ub9c8 9\uc138\ub97c \uc639\ub9bd\ud558\ub824 \ud588\ub2e4. \uadf8\ub798\uc11c 13\uc138\ub294 \ubab0\ub798 \uc601\uad6d, \ub7ec\uc2dc\uc544, \uc77c\ubcf8\uc758 \uc911\uad6d \ubca0\uc774\uc9d5 \uc8fc\uc7ac \ub300\uc0ac\uc5d0 \ubc00\uc0ac\ub97c \ubcf4\ub0b4\uace0 \ub3c4\uc6c0\uc744 \ubc1b\uace0 \uc788\ub2e4.1908\ub144 12\uc6d4 \ubca0\uc774\uc9d5\uc744 \uc11c\uc11c 1909\ub144 12\uc6d4 \ub77c\uc0ac\uc5d0 \ub3cc\uc544\uc628\ub2e4."} {"question": "\uc2a4\uc2a4\ubb34\uc758 \ud615\uc778 \ub9c8\ubaa8\ub8e8\uac00 \uad6c\ud55c \uc0ac\ub78c\uc758 \uc774\ub984\uc740?", "paragraph": "\ud55c\ud3b8, \ub9c8\ubaa8\ub8e8\uc758 \ub3d9\uc0dd \ucf54\ub2e4\uc774 \uc2a4\uc2a4\ubb34\ub294 \ubc29\uc0ac\ub2a5\uc5d0 \uc624\uc5fc\ub41c \uc9c0\uad6c\uc5d0\uc11c \uc548\uc804\ud55c \uc9c0\ud558\uc5d0\uc11c \uac70\uc8fc\ud558\uba70 \ub9e4\uc77c \uc9c0\uc0c1\uc73c\ub85c \ub098\uc640 \uc9c0\uad6c \ud45c\uba74\uc5d0\uc11c \ub808\uc5b4 \uba54\ud0c8\uc744 \ucc3e\uace0 \uc788\uc5c8\ub2e4. \ubcf4\ud1b5\uacfc \ub2e4\ub984\uc5c6\ub358 \uc5b4\ub290 \ub0a0, \uc2a4\uc2a4\ubb34\ub294 \uc608\uc804\uacfc \uac19\uc774 \uc9c0\uad6c \ud45c\uba74\uc744 \uc870\uc0ac\ud558\uace0 \uc788\uc5c8\ub2e4. \uadf8\ub7f0\ub370 \uadf8 \ub54c \uac11\uc790\uae30 \ud55c \ubb3c\uccb4\uac00 \uadfc\ucc98\uc5d0\uc11c \uc9c0\uad6c\uc5d0 \ucda9\ub3cc\ud558\uace0, \uadf8 \uc5ec\ud30c\ub85c \uc2a4\uc2a4\ubb34\ub294 \uc7a0\uc2dc \uc758\uc2dd\uc744 \uc783\uace0 \ubc29\uc0ac\ub2a5\uc5d0\uc11c \uadf8\ub97c \ubcf4\ud638\ud574\uc8fc\ub358 \ud5ec\uba67\uc740 \ub0a0\uc544\uac00 \ubc84\ub9b0\ub2e4. \uc7a0\uc2dc \ub4a4, \uadf8\ub294 \uae68\uc5b4\ub098\uc11c \uc815\uc2e0\uc744 \ucc28\ub838\ub294\ub370, \uadf8 \uacf3\uc5d0\uc11c \uc678\uacc4\uc778\uc758 \uba54\uc2dc\uc9c0\uac00 \ub2f4\uae34 \ucea1\uc290\uc744 \ubc1c\uacac\ud558\uac8c \ub418\uace0, \uadf8\uc758 \uc8fc\ubcc0\uc5d0 \ubc29\uc0ac\ub2a5\uc774 \ud604\uc800\ud788 \uc904\uc5b4\ub4e0 \uac83\uc744 \uc54c\uc544\ucc28\ub9b0\ub2e4. \uc2a4\uc2a4\ubb34\ub294 \uad6c\uc870\ub418\uc5b4 \uc624\ud0a4\ub2e4\uc758 \ud568\uc120 \uc548\uc73c\ub85c \uc62e\uaca8\uc9c0\uace0, \ud568\uc120\uc5d0 \ud0c0\uace0 \uc788\ub358 \uc120\uc6d0\ub4e4\uc740 \uc2a4\uc2a4\ubb34\uac00 \ubc1c\uacac\ud588\ub358 \ucea1\uc290\uc5d0 \uc5b4\ub5a4 \uc124\uacc4\ub3c4\uac00 \ub4e4\uc5b4\uc788\ub294 \uac83\uc744 \ubc1c\uacac\ud55c\ub2e4. \uadf8 \uc124\uacc4\ub3c4\uc5d0\ub294 \uc774\uc2a4\uce78\ub378\uc774\ub77c\uace0 \ud558\ub294 \ud589\uc131\uc758 \uc88c\ud45c\uc640 \uc9c0\uad6c\uc5d0\uc11c \uadf8 \ud589\uc131\uae4c\uc9c0\uc758 \ud56d\ub85c\uac00 \ub2f4\uaca8\uc838 \uc788\uc5c8\ub2e4. \ud55c\ud3b8 \uc2a4\uc2a4\ubb34\ub294 \uc790\uc2e0\uc758 \ud615\uc774 \uc624\ud0a4\ub2e4\ub97c \uad6c\ud558\ub294 \ub300\uc2e0 \ubaa9\uc228\uc744 \uc783\uc740 \uac83\uc744 \uc54c\uace0 \uc624\ud0a4\ub2e4\uc5d0\uac8c \uc9c1\uc811 \uac00\uc11c \uc2ec\ud558\uac8c \ud654\ub97c \ub0b4\uc9c0\ub9cc, \uc120\uc6d0 \uc720\ud0a4 \ubaa8\ub9ac\uac00 \ud798\uc73c\ub85c \uadf8\ub97c \uc81c\uc9c0\ud55c\ub2e4.", "answer": "\uc624\ud0a4\ub2e4", "sentence": "\uc2a4\uc2a4\ubb34\ub294 \uad6c\uc870\ub418\uc5b4 \uc624\ud0a4\ub2e4 \uc758 \ud568\uc120 \uc548\uc73c\ub85c \uc62e\uaca8\uc9c0\uace0, \ud568\uc120\uc5d0 \ud0c0\uace0 \uc788\ub358 \uc120\uc6d0\ub4e4\uc740 \uc2a4\uc2a4\ubb34\uac00 \ubc1c\uacac\ud588\ub358 \ucea1\uc290\uc5d0 \uc5b4\ub5a4 \uc124\uacc4\ub3c4\uac00 \ub4e4\uc5b4\uc788\ub294 \uac83\uc744 \ubc1c\uacac\ud55c\ub2e4.", "paragraph_sentence": "\ud55c\ud3b8, \ub9c8\ubaa8\ub8e8\uc758 \ub3d9\uc0dd \ucf54\ub2e4\uc774 \uc2a4\uc2a4\ubb34\ub294 \ubc29\uc0ac\ub2a5\uc5d0 \uc624\uc5fc\ub41c \uc9c0\uad6c\uc5d0\uc11c \uc548\uc804\ud55c \uc9c0\ud558\uc5d0\uc11c \uac70\uc8fc\ud558\uba70 \ub9e4\uc77c \uc9c0\uc0c1\uc73c\ub85c \ub098\uc640 \uc9c0\uad6c \ud45c\uba74\uc5d0\uc11c \ub808\uc5b4 \uba54\ud0c8\uc744 \ucc3e\uace0 \uc788\uc5c8\ub2e4. \ubcf4\ud1b5\uacfc \ub2e4\ub984\uc5c6\ub358 \uc5b4\ub290 \ub0a0, \uc2a4\uc2a4\ubb34\ub294 \uc608\uc804\uacfc \uac19\uc774 \uc9c0\uad6c \ud45c\uba74\uc744 \uc870\uc0ac\ud558\uace0 \uc788\uc5c8\ub2e4. \uadf8\ub7f0\ub370 \uadf8 \ub54c \uac11\uc790\uae30 \ud55c \ubb3c\uccb4\uac00 \uadfc\ucc98\uc5d0\uc11c \uc9c0\uad6c\uc5d0 \ucda9\ub3cc\ud558\uace0, \uadf8 \uc5ec\ud30c\ub85c \uc2a4\uc2a4\ubb34\ub294 \uc7a0\uc2dc \uc758\uc2dd\uc744 \uc783\uace0 \ubc29\uc0ac\ub2a5\uc5d0\uc11c \uadf8\ub97c \ubcf4\ud638\ud574\uc8fc\ub358 \ud5ec\uba67\uc740 \ub0a0\uc544\uac00 \ubc84\ub9b0\ub2e4. \uc7a0\uc2dc \ub4a4, \uadf8\ub294 \uae68\uc5b4\ub098\uc11c \uc815\uc2e0\uc744 \ucc28\ub838\ub294\ub370, \uadf8 \uacf3\uc5d0\uc11c \uc678\uacc4\uc778\uc758 \uba54\uc2dc\uc9c0\uac00 \ub2f4\uae34 \ucea1\uc290\uc744 \ubc1c\uacac\ud558\uac8c \ub418\uace0, \uadf8\uc758 \uc8fc\ubcc0\uc5d0 \ubc29\uc0ac\ub2a5\uc774 \ud604\uc800\ud788 \uc904\uc5b4\ub4e0 \uac83\uc744 \uc54c\uc544\ucc28\ub9b0\ub2e4. \uc2a4\uc2a4\ubb34\ub294 \uad6c\uc870\ub418\uc5b4 \uc624\ud0a4\ub2e4 \uc758 \ud568\uc120 \uc548\uc73c\ub85c \uc62e\uaca8\uc9c0\uace0, \ud568\uc120\uc5d0 \ud0c0\uace0 \uc788\ub358 \uc120\uc6d0\ub4e4\uc740 \uc2a4\uc2a4\ubb34\uac00 \ubc1c\uacac\ud588\ub358 \ucea1\uc290\uc5d0 \uc5b4\ub5a4 \uc124\uacc4\ub3c4\uac00 \ub4e4\uc5b4\uc788\ub294 \uac83\uc744 \ubc1c\uacac\ud55c\ub2e4. \uadf8 \uc124\uacc4\ub3c4\uc5d0\ub294 \uc774\uc2a4\uce78\ub378\uc774\ub77c\uace0 \ud558\ub294 \ud589\uc131\uc758 \uc88c\ud45c\uc640 \uc9c0\uad6c\uc5d0\uc11c \uadf8 \ud589\uc131\uae4c\uc9c0\uc758 \ud56d\ub85c\uac00 \ub2f4\uaca8\uc838 \uc788\uc5c8\ub2e4. \ud55c\ud3b8 \uc2a4\uc2a4\ubb34\ub294 \uc790\uc2e0\uc758 \ud615\uc774 \uc624\ud0a4\ub2e4\ub97c \uad6c\ud558\ub294 \ub300\uc2e0 \ubaa9\uc228\uc744 \uc783\uc740 \uac83\uc744 \uc54c\uace0 \uc624\ud0a4\ub2e4\uc5d0\uac8c \uc9c1\uc811 \uac00\uc11c \uc2ec\ud558\uac8c \ud654\ub97c \ub0b4\uc9c0\ub9cc, \uc120\uc6d0 \uc720\ud0a4 \ubaa8\ub9ac\uac00 \ud798\uc73c\ub85c \uadf8\ub97c \uc81c\uc9c0\ud55c\ub2e4.", "paragraph_answer": "\ud55c\ud3b8, \ub9c8\ubaa8\ub8e8\uc758 \ub3d9\uc0dd \ucf54\ub2e4\uc774 \uc2a4\uc2a4\ubb34\ub294 \ubc29\uc0ac\ub2a5\uc5d0 \uc624\uc5fc\ub41c \uc9c0\uad6c\uc5d0\uc11c \uc548\uc804\ud55c \uc9c0\ud558\uc5d0\uc11c \uac70\uc8fc\ud558\uba70 \ub9e4\uc77c \uc9c0\uc0c1\uc73c\ub85c \ub098\uc640 \uc9c0\uad6c \ud45c\uba74\uc5d0\uc11c \ub808\uc5b4 \uba54\ud0c8\uc744 \ucc3e\uace0 \uc788\uc5c8\ub2e4. \ubcf4\ud1b5\uacfc \ub2e4\ub984\uc5c6\ub358 \uc5b4\ub290 \ub0a0, \uc2a4\uc2a4\ubb34\ub294 \uc608\uc804\uacfc \uac19\uc774 \uc9c0\uad6c \ud45c\uba74\uc744 \uc870\uc0ac\ud558\uace0 \uc788\uc5c8\ub2e4. \uadf8\ub7f0\ub370 \uadf8 \ub54c \uac11\uc790\uae30 \ud55c \ubb3c\uccb4\uac00 \uadfc\ucc98\uc5d0\uc11c \uc9c0\uad6c\uc5d0 \ucda9\ub3cc\ud558\uace0, \uadf8 \uc5ec\ud30c\ub85c \uc2a4\uc2a4\ubb34\ub294 \uc7a0\uc2dc \uc758\uc2dd\uc744 \uc783\uace0 \ubc29\uc0ac\ub2a5\uc5d0\uc11c \uadf8\ub97c \ubcf4\ud638\ud574\uc8fc\ub358 \ud5ec\uba67\uc740 \ub0a0\uc544\uac00 \ubc84\ub9b0\ub2e4. \uc7a0\uc2dc \ub4a4, \uadf8\ub294 \uae68\uc5b4\ub098\uc11c \uc815\uc2e0\uc744 \ucc28\ub838\ub294\ub370, \uadf8 \uacf3\uc5d0\uc11c \uc678\uacc4\uc778\uc758 \uba54\uc2dc\uc9c0\uac00 \ub2f4\uae34 \ucea1\uc290\uc744 \ubc1c\uacac\ud558\uac8c \ub418\uace0, \uadf8\uc758 \uc8fc\ubcc0\uc5d0 \ubc29\uc0ac\ub2a5\uc774 \ud604\uc800\ud788 \uc904\uc5b4\ub4e0 \uac83\uc744 \uc54c\uc544\ucc28\ub9b0\ub2e4. \uc2a4\uc2a4\ubb34\ub294 \uad6c\uc870\ub418\uc5b4 \uc624\ud0a4\ub2e4 \uc758 \ud568\uc120 \uc548\uc73c\ub85c \uc62e\uaca8\uc9c0\uace0, \ud568\uc120\uc5d0 \ud0c0\uace0 \uc788\ub358 \uc120\uc6d0\ub4e4\uc740 \uc2a4\uc2a4\ubb34\uac00 \ubc1c\uacac\ud588\ub358 \ucea1\uc290\uc5d0 \uc5b4\ub5a4 \uc124\uacc4\ub3c4\uac00 \ub4e4\uc5b4\uc788\ub294 \uac83\uc744 \ubc1c\uacac\ud55c\ub2e4. \uadf8 \uc124\uacc4\ub3c4\uc5d0\ub294 \uc774\uc2a4\uce78\ub378\uc774\ub77c\uace0 \ud558\ub294 \ud589\uc131\uc758 \uc88c\ud45c\uc640 \uc9c0\uad6c\uc5d0\uc11c \uadf8 \ud589\uc131\uae4c\uc9c0\uc758 \ud56d\ub85c\uac00 \ub2f4\uaca8\uc838 \uc788\uc5c8\ub2e4. \ud55c\ud3b8 \uc2a4\uc2a4\ubb34\ub294 \uc790\uc2e0\uc758 \ud615\uc774 \uc624\ud0a4\ub2e4\ub97c \uad6c\ud558\ub294 \ub300\uc2e0 \ubaa9\uc228\uc744 \uc783\uc740 \uac83\uc744 \uc54c\uace0 \uc624\ud0a4\ub2e4\uc5d0\uac8c \uc9c1\uc811 \uac00\uc11c \uc2ec\ud558\uac8c \ud654\ub97c \ub0b4\uc9c0\ub9cc, \uc120\uc6d0 \uc720\ud0a4 \ubaa8\ub9ac\uac00 \ud798\uc73c\ub85c \uadf8\ub97c \uc81c\uc9c0\ud55c\ub2e4.", "sentence_answer": "\uc2a4\uc2a4\ubb34\ub294 \uad6c\uc870\ub418\uc5b4 \uc624\ud0a4\ub2e4 \uc758 \ud568\uc120 \uc548\uc73c\ub85c \uc62e\uaca8\uc9c0\uace0, \ud568\uc120\uc5d0 \ud0c0\uace0 \uc788\ub358 \uc120\uc6d0\ub4e4\uc740 \uc2a4\uc2a4\ubb34\uac00 \ubc1c\uacac\ud588\ub358 \ucea1\uc290\uc5d0 \uc5b4\ub5a4 \uc124\uacc4\ub3c4\uac00 \ub4e4\uc5b4\uc788\ub294 \uac83\uc744 \ubc1c\uacac\ud55c\ub2e4.", "paragraph_id": "6526332-2-1", "paragraph_question": "question: \uc2a4\uc2a4\ubb34\uc758 \ud615\uc778 \ub9c8\ubaa8\ub8e8\uac00 \uad6c\ud55c \uc0ac\ub78c\uc758 \uc774\ub984\uc740?, context: \ud55c\ud3b8, \ub9c8\ubaa8\ub8e8\uc758 \ub3d9\uc0dd \ucf54\ub2e4\uc774 \uc2a4\uc2a4\ubb34\ub294 \ubc29\uc0ac\ub2a5\uc5d0 \uc624\uc5fc\ub41c \uc9c0\uad6c\uc5d0\uc11c \uc548\uc804\ud55c \uc9c0\ud558\uc5d0\uc11c \uac70\uc8fc\ud558\uba70 \ub9e4\uc77c \uc9c0\uc0c1\uc73c\ub85c \ub098\uc640 \uc9c0\uad6c \ud45c\uba74\uc5d0\uc11c \ub808\uc5b4 \uba54\ud0c8\uc744 \ucc3e\uace0 \uc788\uc5c8\ub2e4. \ubcf4\ud1b5\uacfc \ub2e4\ub984\uc5c6\ub358 \uc5b4\ub290 \ub0a0, \uc2a4\uc2a4\ubb34\ub294 \uc608\uc804\uacfc \uac19\uc774 \uc9c0\uad6c \ud45c\uba74\uc744 \uc870\uc0ac\ud558\uace0 \uc788\uc5c8\ub2e4. \uadf8\ub7f0\ub370 \uadf8 \ub54c \uac11\uc790\uae30 \ud55c \ubb3c\uccb4\uac00 \uadfc\ucc98\uc5d0\uc11c \uc9c0\uad6c\uc5d0 \ucda9\ub3cc\ud558\uace0, \uadf8 \uc5ec\ud30c\ub85c \uc2a4\uc2a4\ubb34\ub294 \uc7a0\uc2dc \uc758\uc2dd\uc744 \uc783\uace0 \ubc29\uc0ac\ub2a5\uc5d0\uc11c \uadf8\ub97c \ubcf4\ud638\ud574\uc8fc\ub358 \ud5ec\uba67\uc740 \ub0a0\uc544\uac00 \ubc84\ub9b0\ub2e4. \uc7a0\uc2dc \ub4a4, \uadf8\ub294 \uae68\uc5b4\ub098\uc11c \uc815\uc2e0\uc744 \ucc28\ub838\ub294\ub370, \uadf8 \uacf3\uc5d0\uc11c \uc678\uacc4\uc778\uc758 \uba54\uc2dc\uc9c0\uac00 \ub2f4\uae34 \ucea1\uc290\uc744 \ubc1c\uacac\ud558\uac8c \ub418\uace0, \uadf8\uc758 \uc8fc\ubcc0\uc5d0 \ubc29\uc0ac\ub2a5\uc774 \ud604\uc800\ud788 \uc904\uc5b4\ub4e0 \uac83\uc744 \uc54c\uc544\ucc28\ub9b0\ub2e4. \uc2a4\uc2a4\ubb34\ub294 \uad6c\uc870\ub418\uc5b4 \uc624\ud0a4\ub2e4\uc758 \ud568\uc120 \uc548\uc73c\ub85c \uc62e\uaca8\uc9c0\uace0, \ud568\uc120\uc5d0 \ud0c0\uace0 \uc788\ub358 \uc120\uc6d0\ub4e4\uc740 \uc2a4\uc2a4\ubb34\uac00 \ubc1c\uacac\ud588\ub358 \ucea1\uc290\uc5d0 \uc5b4\ub5a4 \uc124\uacc4\ub3c4\uac00 \ub4e4\uc5b4\uc788\ub294 \uac83\uc744 \ubc1c\uacac\ud55c\ub2e4. \uadf8 \uc124\uacc4\ub3c4\uc5d0\ub294 \uc774\uc2a4\uce78\ub378\uc774\ub77c\uace0 \ud558\ub294 \ud589\uc131\uc758 \uc88c\ud45c\uc640 \uc9c0\uad6c\uc5d0\uc11c \uadf8 \ud589\uc131\uae4c\uc9c0\uc758 \ud56d\ub85c\uac00 \ub2f4\uaca8\uc838 \uc788\uc5c8\ub2e4. \ud55c\ud3b8 \uc2a4\uc2a4\ubb34\ub294 \uc790\uc2e0\uc758 \ud615\uc774 \uc624\ud0a4\ub2e4\ub97c \uad6c\ud558\ub294 \ub300\uc2e0 \ubaa9\uc228\uc744 \uc783\uc740 \uac83\uc744 \uc54c\uace0 \uc624\ud0a4\ub2e4\uc5d0\uac8c \uc9c1\uc811 \uac00\uc11c \uc2ec\ud558\uac8c \ud654\ub97c \ub0b4\uc9c0\ub9cc, \uc120\uc6d0 \uc720\ud0a4 \ubaa8\ub9ac\uac00 \ud798\uc73c\ub85c \uadf8\ub97c \uc81c\uc9c0\ud55c\ub2e4."} {"question": "50\ub144\ub300 \uc774\ub798\ub85c \uc778\uacf5\uc9c0\ub2a5 \ubd84\uc57c\uc5d0\uc11c \uc9c0\ub2a5\uc744 \ud310\ub2e8\ud558\ub294 \uae30\uc900\uc73c\ub85c \ub110\ub9ac \ucc44\uc6a9\ud55c \uac83\uc740?", "paragraph": "\uc874 \uc124\uc758 \uc911\uad6d\uc5b4 \ubc29 \ub17c\ubcc0\uc740 50\ub144\ub300 \uc774\ub798\ub85c \uc778\uacf5\uc9c0\ub2a5 \ubd84\uc57c\uc5d0\uc11c \uc9c0\ub2a5\uc744 \ud310\uac00\ub984\ud558\ub294 \uae30\uc900\uc73c\ub85c\uc11c \ub110\ub9ac \ucc44\uc6a9\ud574 \uc628 \ud29c\ub9c1 \ud14c\uc2a4\ud2b8\uc5d0 \ub300\ud55c \ubc18\uc99d \ub17c\ubcc0\uc73c\ub85c \uac04\uc8fc\ub418\uc5b4 \uc654\ub2e4. \uc9c0\ub2a5\uc774 \ubb34\uc5c7\uc774\ub0d0\uace0 \ud558\ub294 \ucd94\uc0c1\uc801\uc778 \ub17c\uc758\ub294 \ub05d\uc5c6\uc774 \ubc18\ubcf5\ub418\uae30 \ub54c\ubb38\uc5d0 \uc9c0\ub2a5\uc758 \uc2e4\uc81c\uc801 \uae30\uc900\uc744 \uc81c\uc2dc\ud558\uae30 \uc704\ud574 \uc5b4\ub5a4 \uc2dc\uc2a4\ud15c\uc774 \uc9c0\ub2a5\uc744 \uac00\uc84c\ub294\uc9c0\uc758 \uc5ec\ubd80\ub97c \uac80\uc0ac\ud560 \uc218 \uc788\ub294 \uac00\uc124\uc744 \uace0\uc548\ud574 \ub0c8\ub358 \uac83\uc774\ub2e4. \ud29c\ub9c1 \ud14c\uc2a4\ud2b8\ub294 \uc9c0\ub2a5\uc744 \ud589\uc704\uc5d0 \uc758\ud574 \uaddc\uc815\ud558\ub824\ub294 \ud589\ub3d9\uc8fc\uc758\ub97c \uae30\ubcf8\uc0ac\uc0c1\uc73c\ub85c \ud558\uace0 \uc788\uc73c\uba70 \uc774\uc5d0 \uc758\uac70\ud558\uc5ec \uc0dd\uac01\ud558\ub294 \ucef4\ud4e8\ud130\uc758 \ucd9c\ud604\uc774 \uac00\ub2a5\ud558\ub2e4\ub294 \uc774\ub978 \ubc14 \uac15\ud55c \uc778\uacf5\uc9c0\ub2a5\uc744 \uc9c0\uc9c0\ud55c\ub2e4. \uc774\uc5d0 \ubc18\ud574 \uc874 \uc124\uc758 \uc911\uad6d\uc5b4 \ubc29 \ub17c\ubcc0\uc5d0 \uc758\ud558\uba74 \ub514\uc9c0\ud138 \ucef4\ud4e8\ud130\ub294 \ud615\uc2dd\uc801\uc774\uace0 \uad6c\ubb38\ub860\uc801\uc774\uae30 \ub54c\ubb38\uc5d0 \uae30\uc220\uc774 \uc544\ubb34\ub9ac \ubc1c\uc804\ud574\ub3c4 \uc2ec\ub9ac\uc801 \uacfc\uc815\uc774\ub77c\ub294 \uc758\ubbf8\ub860\uc801 \ub0b4\uc6a9\uc744 \uc9c0\ub2cc \uc9c0\ub2a5\uc758 \uc77c\uc744 \ud574\ub0bc \uc218 \uc5c6\ub2e4\ub294 \uac83\uc774\ub2e4. \uc911\uad6d\uc5b4 \ubc29 \uc0ac\ub78c\uc774 \uc81c \uc544\ubb34\ub9ac \uc911\uad6d\uc5b4\ub85c \ud574\ub2f5\uc744 \uc798 \ud574\ub0b8\ub2e4\uace0 \ud558\ub354\ub77c\ub3c4 \uaddc\uc815\uc744 \ub2e4\ub8e8\ub294 \uc808\ucc28\ub9cc \uc775\ud78c \uadf8\uac00 \uc911\uad6d\uc5b4\ub97c \uc774\ud574\ud558\uace0 \uc788\ub2e4\uace0\ub294 \ud560 \uc218 \uc5c6\uae30 \ub54c\ubb38\uc774\ub2e4.", "answer": "\ud29c\ub9c1 \ud14c\uc2a4\ud2b8", "sentence": "\uc874 \uc124\uc758 \uc911\uad6d\uc5b4 \ubc29 \ub17c\ubcc0\uc740 50\ub144\ub300 \uc774\ub798\ub85c \uc778\uacf5\uc9c0\ub2a5 \ubd84\uc57c\uc5d0\uc11c \uc9c0\ub2a5\uc744 \ud310\uac00\ub984\ud558\ub294 \uae30\uc900\uc73c\ub85c\uc11c \ub110\ub9ac \ucc44\uc6a9\ud574 \uc628 \ud29c\ub9c1 \ud14c\uc2a4\ud2b8 \uc5d0 \ub300\ud55c \ubc18\uc99d \ub17c\ubcc0\uc73c\ub85c \uac04\uc8fc\ub418\uc5b4 \uc654\ub2e4.", "paragraph_sentence": " \uc874 \uc124\uc758 \uc911\uad6d\uc5b4 \ubc29 \ub17c\ubcc0\uc740 50\ub144\ub300 \uc774\ub798\ub85c \uc778\uacf5\uc9c0\ub2a5 \ubd84\uc57c\uc5d0\uc11c \uc9c0\ub2a5\uc744 \ud310\uac00\ub984\ud558\ub294 \uae30\uc900\uc73c\ub85c\uc11c \ub110\ub9ac \ucc44\uc6a9\ud574 \uc628 \ud29c\ub9c1 \ud14c\uc2a4\ud2b8 \uc5d0 \ub300\ud55c \ubc18\uc99d \ub17c\ubcc0\uc73c\ub85c \uac04\uc8fc\ub418\uc5b4 \uc654\ub2e4. \uc9c0\ub2a5\uc774 \ubb34\uc5c7\uc774\ub0d0\uace0 \ud558\ub294 \ucd94\uc0c1\uc801\uc778 \ub17c\uc758\ub294 \ub05d\uc5c6\uc774 \ubc18\ubcf5\ub418\uae30 \ub54c\ubb38\uc5d0 \uc9c0\ub2a5\uc758 \uc2e4\uc81c\uc801 \uae30\uc900\uc744 \uc81c\uc2dc\ud558\uae30 \uc704\ud574 \uc5b4\ub5a4 \uc2dc\uc2a4\ud15c\uc774 \uc9c0\ub2a5\uc744 \uac00\uc84c\ub294\uc9c0\uc758 \uc5ec\ubd80\ub97c \uac80\uc0ac\ud560 \uc218 \uc788\ub294 \uac00\uc124\uc744 \uace0\uc548\ud574 \ub0c8\ub358 \uac83\uc774\ub2e4. \ud29c\ub9c1 \ud14c\uc2a4\ud2b8\ub294 \uc9c0\ub2a5\uc744 \ud589\uc704\uc5d0 \uc758\ud574 \uaddc\uc815\ud558\ub824\ub294 \ud589\ub3d9\uc8fc\uc758\ub97c \uae30\ubcf8\uc0ac\uc0c1\uc73c\ub85c \ud558\uace0 \uc788\uc73c\uba70 \uc774\uc5d0 \uc758\uac70\ud558\uc5ec \uc0dd\uac01\ud558\ub294 \ucef4\ud4e8\ud130\uc758 \ucd9c\ud604\uc774 \uac00\ub2a5\ud558\ub2e4\ub294 \uc774\ub978 \ubc14 \uac15\ud55c \uc778\uacf5\uc9c0\ub2a5\uc744 \uc9c0\uc9c0\ud55c\ub2e4. \uc774\uc5d0 \ubc18\ud574 \uc874 \uc124\uc758 \uc911\uad6d\uc5b4 \ubc29 \ub17c\ubcc0\uc5d0 \uc758\ud558\uba74 \ub514\uc9c0\ud138 \ucef4\ud4e8\ud130\ub294 \ud615\uc2dd\uc801\uc774\uace0 \uad6c\ubb38\ub860\uc801\uc774\uae30 \ub54c\ubb38\uc5d0 \uae30\uc220\uc774 \uc544\ubb34\ub9ac \ubc1c\uc804\ud574\ub3c4 \uc2ec\ub9ac\uc801 \uacfc\uc815\uc774\ub77c\ub294 \uc758\ubbf8\ub860\uc801 \ub0b4\uc6a9\uc744 \uc9c0\ub2cc \uc9c0\ub2a5\uc758 \uc77c\uc744 \ud574\ub0bc \uc218 \uc5c6\ub2e4\ub294 \uac83\uc774\ub2e4. \uc911\uad6d\uc5b4 \ubc29 \uc0ac\ub78c\uc774 \uc81c \uc544\ubb34\ub9ac \uc911\uad6d\uc5b4\ub85c \ud574\ub2f5\uc744 \uc798 \ud574\ub0b8\ub2e4\uace0 \ud558\ub354\ub77c\ub3c4 \uaddc\uc815\uc744 \ub2e4\ub8e8\ub294 \uc808\ucc28\ub9cc \uc775\ud78c \uadf8\uac00 \uc911\uad6d\uc5b4\ub97c \uc774\ud574\ud558\uace0 \uc788\ub2e4\uace0\ub294 \ud560 \uc218 \uc5c6\uae30 \ub54c\ubb38\uc774\ub2e4.", "paragraph_answer": "\uc874 \uc124\uc758 \uc911\uad6d\uc5b4 \ubc29 \ub17c\ubcc0\uc740 50\ub144\ub300 \uc774\ub798\ub85c \uc778\uacf5\uc9c0\ub2a5 \ubd84\uc57c\uc5d0\uc11c \uc9c0\ub2a5\uc744 \ud310\uac00\ub984\ud558\ub294 \uae30\uc900\uc73c\ub85c\uc11c \ub110\ub9ac \ucc44\uc6a9\ud574 \uc628 \ud29c\ub9c1 \ud14c\uc2a4\ud2b8 \uc5d0 \ub300\ud55c \ubc18\uc99d \ub17c\ubcc0\uc73c\ub85c \uac04\uc8fc\ub418\uc5b4 \uc654\ub2e4. \uc9c0\ub2a5\uc774 \ubb34\uc5c7\uc774\ub0d0\uace0 \ud558\ub294 \ucd94\uc0c1\uc801\uc778 \ub17c\uc758\ub294 \ub05d\uc5c6\uc774 \ubc18\ubcf5\ub418\uae30 \ub54c\ubb38\uc5d0 \uc9c0\ub2a5\uc758 \uc2e4\uc81c\uc801 \uae30\uc900\uc744 \uc81c\uc2dc\ud558\uae30 \uc704\ud574 \uc5b4\ub5a4 \uc2dc\uc2a4\ud15c\uc774 \uc9c0\ub2a5\uc744 \uac00\uc84c\ub294\uc9c0\uc758 \uc5ec\ubd80\ub97c \uac80\uc0ac\ud560 \uc218 \uc788\ub294 \uac00\uc124\uc744 \uace0\uc548\ud574 \ub0c8\ub358 \uac83\uc774\ub2e4. \ud29c\ub9c1 \ud14c\uc2a4\ud2b8\ub294 \uc9c0\ub2a5\uc744 \ud589\uc704\uc5d0 \uc758\ud574 \uaddc\uc815\ud558\ub824\ub294 \ud589\ub3d9\uc8fc\uc758\ub97c \uae30\ubcf8\uc0ac\uc0c1\uc73c\ub85c \ud558\uace0 \uc788\uc73c\uba70 \uc774\uc5d0 \uc758\uac70\ud558\uc5ec \uc0dd\uac01\ud558\ub294 \ucef4\ud4e8\ud130\uc758 \ucd9c\ud604\uc774 \uac00\ub2a5\ud558\ub2e4\ub294 \uc774\ub978 \ubc14 \uac15\ud55c \uc778\uacf5\uc9c0\ub2a5\uc744 \uc9c0\uc9c0\ud55c\ub2e4. \uc774\uc5d0 \ubc18\ud574 \uc874 \uc124\uc758 \uc911\uad6d\uc5b4 \ubc29 \ub17c\ubcc0\uc5d0 \uc758\ud558\uba74 \ub514\uc9c0\ud138 \ucef4\ud4e8\ud130\ub294 \ud615\uc2dd\uc801\uc774\uace0 \uad6c\ubb38\ub860\uc801\uc774\uae30 \ub54c\ubb38\uc5d0 \uae30\uc220\uc774 \uc544\ubb34\ub9ac \ubc1c\uc804\ud574\ub3c4 \uc2ec\ub9ac\uc801 \uacfc\uc815\uc774\ub77c\ub294 \uc758\ubbf8\ub860\uc801 \ub0b4\uc6a9\uc744 \uc9c0\ub2cc \uc9c0\ub2a5\uc758 \uc77c\uc744 \ud574\ub0bc \uc218 \uc5c6\ub2e4\ub294 \uac83\uc774\ub2e4. \uc911\uad6d\uc5b4 \ubc29 \uc0ac\ub78c\uc774 \uc81c \uc544\ubb34\ub9ac \uc911\uad6d\uc5b4\ub85c \ud574\ub2f5\uc744 \uc798 \ud574\ub0b8\ub2e4\uace0 \ud558\ub354\ub77c\ub3c4 \uaddc\uc815\uc744 \ub2e4\ub8e8\ub294 \uc808\ucc28\ub9cc \uc775\ud78c \uadf8\uac00 \uc911\uad6d\uc5b4\ub97c \uc774\ud574\ud558\uace0 \uc788\ub2e4\uace0\ub294 \ud560 \uc218 \uc5c6\uae30 \ub54c\ubb38\uc774\ub2e4.", "sentence_answer": "\uc874 \uc124\uc758 \uc911\uad6d\uc5b4 \ubc29 \ub17c\ubcc0\uc740 50\ub144\ub300 \uc774\ub798\ub85c \uc778\uacf5\uc9c0\ub2a5 \ubd84\uc57c\uc5d0\uc11c \uc9c0\ub2a5\uc744 \ud310\uac00\ub984\ud558\ub294 \uae30\uc900\uc73c\ub85c\uc11c \ub110\ub9ac \ucc44\uc6a9\ud574 \uc628 \ud29c\ub9c1 \ud14c\uc2a4\ud2b8 \uc5d0 \ub300\ud55c \ubc18\uc99d \ub17c\ubcc0\uc73c\ub85c \uac04\uc8fc\ub418\uc5b4 \uc654\ub2e4.", "paragraph_id": "6601749-9-0", "paragraph_question": "question: 50\ub144\ub300 \uc774\ub798\ub85c \uc778\uacf5\uc9c0\ub2a5 \ubd84\uc57c\uc5d0\uc11c \uc9c0\ub2a5\uc744 \ud310\ub2e8\ud558\ub294 \uae30\uc900\uc73c\ub85c \ub110\ub9ac \ucc44\uc6a9\ud55c \uac83\uc740?, context: \uc874 \uc124\uc758 \uc911\uad6d\uc5b4 \ubc29 \ub17c\ubcc0\uc740 50\ub144\ub300 \uc774\ub798\ub85c \uc778\uacf5\uc9c0\ub2a5 \ubd84\uc57c\uc5d0\uc11c \uc9c0\ub2a5\uc744 \ud310\uac00\ub984\ud558\ub294 \uae30\uc900\uc73c\ub85c\uc11c \ub110\ub9ac \ucc44\uc6a9\ud574 \uc628 \ud29c\ub9c1 \ud14c\uc2a4\ud2b8\uc5d0 \ub300\ud55c \ubc18\uc99d \ub17c\ubcc0\uc73c\ub85c \uac04\uc8fc\ub418\uc5b4 \uc654\ub2e4. \uc9c0\ub2a5\uc774 \ubb34\uc5c7\uc774\ub0d0\uace0 \ud558\ub294 \ucd94\uc0c1\uc801\uc778 \ub17c\uc758\ub294 \ub05d\uc5c6\uc774 \ubc18\ubcf5\ub418\uae30 \ub54c\ubb38\uc5d0 \uc9c0\ub2a5\uc758 \uc2e4\uc81c\uc801 \uae30\uc900\uc744 \uc81c\uc2dc\ud558\uae30 \uc704\ud574 \uc5b4\ub5a4 \uc2dc\uc2a4\ud15c\uc774 \uc9c0\ub2a5\uc744 \uac00\uc84c\ub294\uc9c0\uc758 \uc5ec\ubd80\ub97c \uac80\uc0ac\ud560 \uc218 \uc788\ub294 \uac00\uc124\uc744 \uace0\uc548\ud574 \ub0c8\ub358 \uac83\uc774\ub2e4. \ud29c\ub9c1 \ud14c\uc2a4\ud2b8\ub294 \uc9c0\ub2a5\uc744 \ud589\uc704\uc5d0 \uc758\ud574 \uaddc\uc815\ud558\ub824\ub294 \ud589\ub3d9\uc8fc\uc758\ub97c \uae30\ubcf8\uc0ac\uc0c1\uc73c\ub85c \ud558\uace0 \uc788\uc73c\uba70 \uc774\uc5d0 \uc758\uac70\ud558\uc5ec \uc0dd\uac01\ud558\ub294 \ucef4\ud4e8\ud130\uc758 \ucd9c\ud604\uc774 \uac00\ub2a5\ud558\ub2e4\ub294 \uc774\ub978 \ubc14 \uac15\ud55c \uc778\uacf5\uc9c0\ub2a5\uc744 \uc9c0\uc9c0\ud55c\ub2e4. \uc774\uc5d0 \ubc18\ud574 \uc874 \uc124\uc758 \uc911\uad6d\uc5b4 \ubc29 \ub17c\ubcc0\uc5d0 \uc758\ud558\uba74 \ub514\uc9c0\ud138 \ucef4\ud4e8\ud130\ub294 \ud615\uc2dd\uc801\uc774\uace0 \uad6c\ubb38\ub860\uc801\uc774\uae30 \ub54c\ubb38\uc5d0 \uae30\uc220\uc774 \uc544\ubb34\ub9ac \ubc1c\uc804\ud574\ub3c4 \uc2ec\ub9ac\uc801 \uacfc\uc815\uc774\ub77c\ub294 \uc758\ubbf8\ub860\uc801 \ub0b4\uc6a9\uc744 \uc9c0\ub2cc \uc9c0\ub2a5\uc758 \uc77c\uc744 \ud574\ub0bc \uc218 \uc5c6\ub2e4\ub294 \uac83\uc774\ub2e4. \uc911\uad6d\uc5b4 \ubc29 \uc0ac\ub78c\uc774 \uc81c \uc544\ubb34\ub9ac \uc911\uad6d\uc5b4\ub85c \ud574\ub2f5\uc744 \uc798 \ud574\ub0b8\ub2e4\uace0 \ud558\ub354\ub77c\ub3c4 \uaddc\uc815\uc744 \ub2e4\ub8e8\ub294 \uc808\ucc28\ub9cc \uc775\ud78c \uadf8\uac00 \uc911\uad6d\uc5b4\ub97c \uc774\ud574\ud558\uace0 \uc788\ub2e4\uace0\ub294 \ud560 \uc218 \uc5c6\uae30 \ub54c\ubb38\uc774\ub2e4."} {"question": "\ud5e4\ud30c\uc774\uc2a4\ud1a0\uc2a4 \uc2e0\uc804(\ud14c\uc138\uc774\uc628)\uacfc \ud30c\ub974\ud14c\ub17c \uc2e0\uc804\uc774 \ub3d9\uc2dc\uc5d0 \uc788\ub294 \uadf8\ub9ac\uc2a4\uc758 \ub3c4\uc2dc\ub294?", "paragraph": "\uad6c\uc870\uc0c1\uc758 \ubc1c\uc804\uc5d0\uc11c\ub294 \uc218\uc138\uae30 \uc0ac\uc774\uc5d0 \ud2b9\ubcc4\ud55c \ubcc0\ud654\ub97c \ubcf4\uc774\uc9c0 \uc54a\uc558\ub2e4. \uadf8\ub7f0\ub370 \ud615\ud0dc\uc0c1\uc758 \uc9c4\ubcf4 \ubc1c\ub2ec\uc758 \ud754\uc801\uc740 \uc801\uc5b4\ub3c4 \ub3c4\ub9ac\uc2a4\uc2dd \uc2e0\uc804\uc5d0\uc11c\ub3c4 \ubd84\uba85\ud788 \ub354\ub4ec\uc5b4 \uc0dd\uac01\ud560 \uc218 \uc788\ub2e4. \uae30\uc6d0\uc804 6\uc138\uae30\uc5d0 \uc18d\ud55c\ub2e4\ub294 \uc140\ub9ac\ub17c\ud0dc(\uace0\ub300\uc758 \uc140\ub9ac\ub204\uc2a4) \uc18c\uc704 C\u00b7D\u00b7F\u00b7G \uc81c \uc2e0\uc804(\u8af8\u795e\u6bbf), \ud398\uc2a4\ud23c(\uace0\ub300\uc758 \ud3ec\uc138\uc774\ub3c4\ub2c8\uc544)\uc758 \uc774\ub978\ubc14 <\ubc14\uc2e4\ub9ac\uce74> \ubc0f <\ucf00\ub808\uc2a4 \uc2e0\uc804> \ub4f1\uc758 \uc911\ud6c4(\u91cd\u539a)\ud55c \ud615\uc2dd, \uae30\uc6d0\uc804 5\uc138\uae30 \uc804\ubc18\uc5d0 \uc18d\ud558\ub294 \uc5d0\uae30\ub098\uc758 <\uc544\ud30c\uc774\uc544 \uc2e0\uc804>, \uc62c\ub9bc\ud53c\uc544\uc758 <\uc81c\uc6b0\uc2a4 \uc2e0\uc804>, \ud398\uc2a4\ud23c\uc758 <\ud3ec\uc138\uc774\ub3c8 \uc2e0\uc804> \ub4f1 \uc5c4\uc815\ud55c \uc591\uc2dd\uc744 \uac70\uccd0, \uc774\uc73d\uace0 \ud074\ub798\uc2dd\uc758 \uc644\uc131\uc73c\ub85c \ubc1c\uc804\ud558\uc600\ub2e4. \ud074\ub798\uc2dd\uc758 \uc644\uc131\uc740 \uae30\uc6d0\uc804 5\uc138\uae30 \uc911\uc5fd\uc5d0 \uc138\uc6cc\uc9c4 \uc544\ud14c\ub124\uc758 \ud30c\ub974\ud14c\ub17c \uc2e0\uc804, \ud639\uc740 \uac19\uc740 \uc138\uae30 \ud6c4\ubc18\uc5d0 \ub9cc\ub4e4\uc5b4\uc9c4 \uc544\ud14c\ub124\uc758 \ud5e4\ud30c\uc774\uc2a4\ud1a0\uc2a4 \uc2e0\uc804(\uc77c\ubc18\uc801\uc73c\ub85c \ud14c\uc138\uc774\uc628\uc774\ub77c \uc54c\ub824\uc84c\ub2e4) \ud53c\uac08\ub9ac\uc544 \ubd80\uadfc\uc758 \ubc14\uc0ac\uc774\uc758 <\uc544\ud3f4\ub85c \uc2e0\uc804> \ub4f1\uc5d0 \uc758\ud574 \ub2ec\uc131\ub418\uc5c8\ub2e4.", "answer": "\uc544\ud14c\ub124", "sentence": "\ud074\ub798\uc2dd\uc758 \uc644\uc131\uc740 \uae30\uc6d0\uc804 5\uc138\uae30 \uc911\uc5fd\uc5d0 \uc138\uc6cc\uc9c4 \uc544\ud14c\ub124 \uc758 \ud30c\ub974\ud14c\ub17c \uc2e0\uc804, \ud639\uc740 \uac19\uc740 \uc138\uae30 \ud6c4\ubc18\uc5d0 \ub9cc\ub4e4\uc5b4\uc9c4 \uc544\ud14c\ub124\uc758 \ud5e4\ud30c\uc774\uc2a4\ud1a0\uc2a4 \uc2e0\uc804(\uc77c\ubc18\uc801\uc73c\ub85c \ud14c\uc138\uc774\uc628\uc774\ub77c \uc54c\ub824\uc84c\ub2e4) \ud53c\uac08\ub9ac\uc544 \ubd80\uadfc\uc758 \ubc14\uc0ac\uc774\uc758 <\uc544\ud3f4\ub85c \uc2e0\uc804> \ub4f1\uc5d0 \uc758\ud574 \ub2ec\uc131\ub418\uc5c8\ub2e4.", "paragraph_sentence": "\uad6c\uc870\uc0c1\uc758 \ubc1c\uc804\uc5d0\uc11c\ub294 \uc218\uc138\uae30 \uc0ac\uc774\uc5d0 \ud2b9\ubcc4\ud55c \ubcc0\ud654\ub97c \ubcf4\uc774\uc9c0 \uc54a\uc558\ub2e4. \uadf8\ub7f0\ub370 \ud615\ud0dc\uc0c1\uc758 \uc9c4\ubcf4 \ubc1c\ub2ec\uc758 \ud754\uc801\uc740 \uc801\uc5b4\ub3c4 \ub3c4\ub9ac\uc2a4\uc2dd \uc2e0\uc804\uc5d0\uc11c\ub3c4 \ubd84\uba85\ud788 \ub354\ub4ec\uc5b4 \uc0dd\uac01\ud560 \uc218 \uc788\ub2e4. \uae30\uc6d0\uc804 6\uc138\uae30\uc5d0 \uc18d\ud55c\ub2e4\ub294 \uc140\ub9ac\ub17c\ud0dc(\uace0\ub300\uc758 \uc140\ub9ac\ub204\uc2a4) \uc18c\uc704 C\u00b7D\u00b7F\u00b7G \uc81c \uc2e0\uc804(\u8af8\u795e\u6bbf), \ud398\uc2a4\ud23c(\uace0\ub300\uc758 \ud3ec\uc138\uc774\ub3c4\ub2c8\uc544)\uc758 \uc774\ub978\ubc14 <\ubc14\uc2e4\ub9ac\uce74> \ubc0f <\ucf00\ub808\uc2a4 \uc2e0\uc804> \ub4f1\uc758 \uc911\ud6c4(\u91cd\u539a)\ud55c \ud615\uc2dd, \uae30\uc6d0\uc804 5\uc138\uae30 \uc804\ubc18\uc5d0 \uc18d\ud558\ub294 \uc5d0\uae30\ub098\uc758 <\uc544\ud30c\uc774\uc544 \uc2e0\uc804>, \uc62c\ub9bc\ud53c\uc544\uc758 <\uc81c\uc6b0\uc2a4 \uc2e0\uc804>, \ud398\uc2a4\ud23c\uc758 <\ud3ec\uc138\uc774\ub3c8 \uc2e0\uc804> \ub4f1 \uc5c4\uc815\ud55c \uc591\uc2dd\uc744 \uac70\uccd0, \uc774\uc73d\uace0 \ud074\ub798\uc2dd\uc758 \uc644\uc131\uc73c\ub85c \ubc1c\uc804\ud558\uc600\ub2e4. \ud074\ub798\uc2dd\uc758 \uc644\uc131\uc740 \uae30\uc6d0\uc804 5\uc138\uae30 \uc911\uc5fd\uc5d0 \uc138\uc6cc\uc9c4 \uc544\ud14c\ub124 \uc758 \ud30c\ub974\ud14c\ub17c \uc2e0\uc804, \ud639\uc740 \uac19\uc740 \uc138\uae30 \ud6c4\ubc18\uc5d0 \ub9cc\ub4e4\uc5b4\uc9c4 \uc544\ud14c\ub124\uc758 \ud5e4\ud30c\uc774\uc2a4\ud1a0\uc2a4 \uc2e0\uc804(\uc77c\ubc18\uc801\uc73c\ub85c \ud14c\uc138\uc774\uc628\uc774\ub77c \uc54c\ub824\uc84c\ub2e4) \ud53c\uac08\ub9ac\uc544 \ubd80\uadfc\uc758 \ubc14\uc0ac\uc774\uc758 <\uc544\ud3f4\ub85c \uc2e0\uc804> \ub4f1\uc5d0 \uc758\ud574 \ub2ec\uc131\ub418\uc5c8\ub2e4. ", "paragraph_answer": "\uad6c\uc870\uc0c1\uc758 \ubc1c\uc804\uc5d0\uc11c\ub294 \uc218\uc138\uae30 \uc0ac\uc774\uc5d0 \ud2b9\ubcc4\ud55c \ubcc0\ud654\ub97c \ubcf4\uc774\uc9c0 \uc54a\uc558\ub2e4. \uadf8\ub7f0\ub370 \ud615\ud0dc\uc0c1\uc758 \uc9c4\ubcf4 \ubc1c\ub2ec\uc758 \ud754\uc801\uc740 \uc801\uc5b4\ub3c4 \ub3c4\ub9ac\uc2a4\uc2dd \uc2e0\uc804\uc5d0\uc11c\ub3c4 \ubd84\uba85\ud788 \ub354\ub4ec\uc5b4 \uc0dd\uac01\ud560 \uc218 \uc788\ub2e4. \uae30\uc6d0\uc804 6\uc138\uae30\uc5d0 \uc18d\ud55c\ub2e4\ub294 \uc140\ub9ac\ub17c\ud0dc(\uace0\ub300\uc758 \uc140\ub9ac\ub204\uc2a4) \uc18c\uc704 C\u00b7D\u00b7F\u00b7G \uc81c \uc2e0\uc804(\u8af8\u795e\u6bbf), \ud398\uc2a4\ud23c(\uace0\ub300\uc758 \ud3ec\uc138\uc774\ub3c4\ub2c8\uc544)\uc758 \uc774\ub978\ubc14 <\ubc14\uc2e4\ub9ac\uce74> \ubc0f <\ucf00\ub808\uc2a4 \uc2e0\uc804> \ub4f1\uc758 \uc911\ud6c4(\u91cd\u539a)\ud55c \ud615\uc2dd, \uae30\uc6d0\uc804 5\uc138\uae30 \uc804\ubc18\uc5d0 \uc18d\ud558\ub294 \uc5d0\uae30\ub098\uc758 <\uc544\ud30c\uc774\uc544 \uc2e0\uc804>, \uc62c\ub9bc\ud53c\uc544\uc758 <\uc81c\uc6b0\uc2a4 \uc2e0\uc804>, \ud398\uc2a4\ud23c\uc758 <\ud3ec\uc138\uc774\ub3c8 \uc2e0\uc804> \ub4f1 \uc5c4\uc815\ud55c \uc591\uc2dd\uc744 \uac70\uccd0, \uc774\uc73d\uace0 \ud074\ub798\uc2dd\uc758 \uc644\uc131\uc73c\ub85c \ubc1c\uc804\ud558\uc600\ub2e4. \ud074\ub798\uc2dd\uc758 \uc644\uc131\uc740 \uae30\uc6d0\uc804 5\uc138\uae30 \uc911\uc5fd\uc5d0 \uc138\uc6cc\uc9c4 \uc544\ud14c\ub124 \uc758 \ud30c\ub974\ud14c\ub17c \uc2e0\uc804, \ud639\uc740 \uac19\uc740 \uc138\uae30 \ud6c4\ubc18\uc5d0 \ub9cc\ub4e4\uc5b4\uc9c4 \uc544\ud14c\ub124\uc758 \ud5e4\ud30c\uc774\uc2a4\ud1a0\uc2a4 \uc2e0\uc804(\uc77c\ubc18\uc801\uc73c\ub85c \ud14c\uc138\uc774\uc628\uc774\ub77c \uc54c\ub824\uc84c\ub2e4) \ud53c\uac08\ub9ac\uc544 \ubd80\uadfc\uc758 \ubc14\uc0ac\uc774\uc758 <\uc544\ud3f4\ub85c \uc2e0\uc804> \ub4f1\uc5d0 \uc758\ud574 \ub2ec\uc131\ub418\uc5c8\ub2e4.", "sentence_answer": "\ud074\ub798\uc2dd\uc758 \uc644\uc131\uc740 \uae30\uc6d0\uc804 5\uc138\uae30 \uc911\uc5fd\uc5d0 \uc138\uc6cc\uc9c4 \uc544\ud14c\ub124 \uc758 \ud30c\ub974\ud14c\ub17c \uc2e0\uc804, \ud639\uc740 \uac19\uc740 \uc138\uae30 \ud6c4\ubc18\uc5d0 \ub9cc\ub4e4\uc5b4\uc9c4 \uc544\ud14c\ub124\uc758 \ud5e4\ud30c\uc774\uc2a4\ud1a0\uc2a4 \uc2e0\uc804(\uc77c\ubc18\uc801\uc73c\ub85c \ud14c\uc138\uc774\uc628\uc774\ub77c \uc54c\ub824\uc84c\ub2e4) \ud53c\uac08\ub9ac\uc544 \ubd80\uadfc\uc758 \ubc14\uc0ac\uc774\uc758 <\uc544\ud3f4\ub85c \uc2e0\uc804> \ub4f1\uc5d0 \uc758\ud574 \ub2ec\uc131\ub418\uc5c8\ub2e4.", "paragraph_id": "6492006-1-2", "paragraph_question": "question: \ud5e4\ud30c\uc774\uc2a4\ud1a0\uc2a4 \uc2e0\uc804(\ud14c\uc138\uc774\uc628)\uacfc \ud30c\ub974\ud14c\ub17c \uc2e0\uc804\uc774 \ub3d9\uc2dc\uc5d0 \uc788\ub294 \uadf8\ub9ac\uc2a4\uc758 \ub3c4\uc2dc\ub294?, context: \uad6c\uc870\uc0c1\uc758 \ubc1c\uc804\uc5d0\uc11c\ub294 \uc218\uc138\uae30 \uc0ac\uc774\uc5d0 \ud2b9\ubcc4\ud55c \ubcc0\ud654\ub97c \ubcf4\uc774\uc9c0 \uc54a\uc558\ub2e4. \uadf8\ub7f0\ub370 \ud615\ud0dc\uc0c1\uc758 \uc9c4\ubcf4 \ubc1c\ub2ec\uc758 \ud754\uc801\uc740 \uc801\uc5b4\ub3c4 \ub3c4\ub9ac\uc2a4\uc2dd \uc2e0\uc804\uc5d0\uc11c\ub3c4 \ubd84\uba85\ud788 \ub354\ub4ec\uc5b4 \uc0dd\uac01\ud560 \uc218 \uc788\ub2e4. \uae30\uc6d0\uc804 6\uc138\uae30\uc5d0 \uc18d\ud55c\ub2e4\ub294 \uc140\ub9ac\ub17c\ud0dc(\uace0\ub300\uc758 \uc140\ub9ac\ub204\uc2a4) \uc18c\uc704 C\u00b7D\u00b7F\u00b7G \uc81c \uc2e0\uc804(\u8af8\u795e\u6bbf), \ud398\uc2a4\ud23c(\uace0\ub300\uc758 \ud3ec\uc138\uc774\ub3c4\ub2c8\uc544)\uc758 \uc774\ub978\ubc14 <\ubc14\uc2e4\ub9ac\uce74> \ubc0f <\ucf00\ub808\uc2a4 \uc2e0\uc804> \ub4f1\uc758 \uc911\ud6c4(\u91cd\u539a)\ud55c \ud615\uc2dd, \uae30\uc6d0\uc804 5\uc138\uae30 \uc804\ubc18\uc5d0 \uc18d\ud558\ub294 \uc5d0\uae30\ub098\uc758 <\uc544\ud30c\uc774\uc544 \uc2e0\uc804>, \uc62c\ub9bc\ud53c\uc544\uc758 <\uc81c\uc6b0\uc2a4 \uc2e0\uc804>, \ud398\uc2a4\ud23c\uc758 <\ud3ec\uc138\uc774\ub3c8 \uc2e0\uc804> \ub4f1 \uc5c4\uc815\ud55c \uc591\uc2dd\uc744 \uac70\uccd0, \uc774\uc73d\uace0 \ud074\ub798\uc2dd\uc758 \uc644\uc131\uc73c\ub85c \ubc1c\uc804\ud558\uc600\ub2e4. \ud074\ub798\uc2dd\uc758 \uc644\uc131\uc740 \uae30\uc6d0\uc804 5\uc138\uae30 \uc911\uc5fd\uc5d0 \uc138\uc6cc\uc9c4 \uc544\ud14c\ub124\uc758 \ud30c\ub974\ud14c\ub17c \uc2e0\uc804, \ud639\uc740 \uac19\uc740 \uc138\uae30 \ud6c4\ubc18\uc5d0 \ub9cc\ub4e4\uc5b4\uc9c4 \uc544\ud14c\ub124\uc758 \ud5e4\ud30c\uc774\uc2a4\ud1a0\uc2a4 \uc2e0\uc804(\uc77c\ubc18\uc801\uc73c\ub85c \ud14c\uc138\uc774\uc628\uc774\ub77c \uc54c\ub824\uc84c\ub2e4) \ud53c\uac08\ub9ac\uc544 \ubd80\uadfc\uc758 \ubc14\uc0ac\uc774\uc758 <\uc544\ud3f4\ub85c \uc2e0\uc804> \ub4f1\uc5d0 \uc758\ud574 \ub2ec\uc131\ub418\uc5c8\ub2e4."} {"question": "\ubca4\ud5c8 \uc81c2\uc81c\uc791\uc9c4\uc758 \uc120\uc784 \uc870\uac10\ub3c5\uc740?", "paragraph": "\uc564\ub4dc\ub958 \ub9c8\ud2bc\uacfc \uc57c\ud0a4\ub9c8 \uce74\ub204\ud2b8\uac00 \uc601\ud654 \ud6c4\ubc18\ubd80 \uc804\ucc28\uacbd\uc8fc \uc7a5\uba74 \uc81c\uc791\uc744 \uc9c0\ud718\ud588\ub294\ub370 \ub458 \ub2e4 \uc774\uc804\ubd80\ud130 \ud0c0 \uc791\ud488\uc5d0\uc11c \uc81c2 \uc81c\uc791\uc9c4 \uac10\ub3c5 \uc5ed\ud560\uc744 \uc790\uc8fc \ub9e1\uc740 \uacbd\ub825\uc790\uc600\ub2e4. \ub458\uc740 \uac01\uc790 \ud718\ud558\uc5d0 \uc870\uac10\ub3c5 \ud55c \uba85\uc529\uc744 \uac70\ub290\ub838\ub294\ub370 \uc774\ub4e4\uc740 \ucd94\uac00 \uc7a5\uba74\uc744 \ucc0d\ub294 \uc77c\uc744 \ub9e1\uc558\ub2e4. \uc81c2 \uc81c\uc791\uc9c4\uc5d0\ub294 \uc138\ub974\uc870 \ub808\uc624\ub124\uac00 \uc788\uc5c8\ub294\ub370 \uadf8\ub294 \uc120\uc784 \uc870\uac10\ub3c5 \uc9c1\ubd84\uc774\uc5c8\uc73c\uba70 \uc7ac\ucd2c\uc601 \uc791\uc5c5\uc744 \ub9e1\uc558\ub2e4. \uc640\uc77c\ub7ec \uac10\ub3c5\uc740 \uacbd\uc8fc \uc804 \ub3c4\uc5f4 \uc608\uc2dd \uc7a5\uba74, \ud658\ud638\ud558\ub294 \uad70\uc911\ub4e4, \ubca4\ud5c8\uc758 \uc804\ucc28\uac00 \uacb0\uc2b9\uc120\uc744 \ud1b5\uacfc\ud588\uc744 \ub54c\uc758 \uacbd\uae30\uc7a5 \ubaa8\uc2b5 \ub4f1\uc744 \ucd2c\uc601\ud588\ub2e4. \ub3c4\uc5f4 \uc7a5\uba74\uc740 1925\ub144 \ubb34\uc131\uc601\ud654 \ubc84\uc804\uc758 \uac19\uc740 \ubd80\ubd84\uc744 \ud55c \uc7a5\uba74 \ud55c \uc7a5\uba74\ubcc4\ub85c \ub9ac\uba54\uc774\ud06c\ud55c \uac83\uc774\ub2e4. \uc640\uc77c\ub7ec\ub294 \uc804\ucc28\uacbd\uae30\uac00 \ub300\ubd80\ubd84 \uc911\uac04 \uac70\ub9ac \ubc0f \uadfc\uac70\ub9ac \uc2dc\uc810\uc73c\ub85c \uc9c4\ud589\ub418\uae30 \ub54c\ubb38\uc5d0 \uad00\uac1d\ub4e4\uc774 \uacbd\uae30\uc7a5\uc758 \uc6c5\uc7a5\ud568\uc744 \uc2e4\uac10\ud560 \uc218 \uc788\ub3c4\ub85d \uc6d0\uac70\ub9ac\uc5d0\uc11c\uc758 \uc804\ucc28\uc218 \ud589\uc9c4 \uc7a5\uba74\uc744 \uc0bd\uc785\ud588\ub2e4.(\ub2e4\ub9cc \ub3c4\uc5f4 \uc608\uc2dd\uc740 \uace0\uc99d\uc0c1 \uc815\ud655\ud55c \uc7ac\ud604\uc740 \uc544\ub2c8\uc5c8\ub2e4.)", "answer": "\uc138\ub974\uc870 \ub808\uc624\ub124", "sentence": "\uc81c2 \uc81c\uc791\uc9c4\uc5d0\ub294 \uc138\ub974\uc870 \ub808\uc624\ub124 \uac00 \uc788\uc5c8\ub294\ub370 \uadf8\ub294 \uc120\uc784 \uc870\uac10\ub3c5 \uc9c1\ubd84\uc774\uc5c8\uc73c\uba70 \uc7ac\ucd2c\uc601 \uc791\uc5c5\uc744 \ub9e1\uc558\ub2e4.", "paragraph_sentence": "\uc564\ub4dc\ub958 \ub9c8\ud2bc\uacfc \uc57c\ud0a4\ub9c8 \uce74\ub204\ud2b8\uac00 \uc601\ud654 \ud6c4\ubc18\ubd80 \uc804\ucc28\uacbd\uc8fc \uc7a5\uba74 \uc81c\uc791\uc744 \uc9c0\ud718\ud588\ub294\ub370 \ub458 \ub2e4 \uc774\uc804\ubd80\ud130 \ud0c0 \uc791\ud488\uc5d0\uc11c \uc81c2 \uc81c\uc791\uc9c4 \uac10\ub3c5 \uc5ed\ud560\uc744 \uc790\uc8fc \ub9e1\uc740 \uacbd\ub825\uc790\uc600\ub2e4. \ub458\uc740 \uac01\uc790 \ud718\ud558\uc5d0 \uc870\uac10\ub3c5 \ud55c \uba85\uc529\uc744 \uac70\ub290\ub838\ub294\ub370 \uc774\ub4e4\uc740 \ucd94\uac00 \uc7a5\uba74\uc744 \ucc0d\ub294 \uc77c\uc744 \ub9e1\uc558\ub2e4. \uc81c2 \uc81c\uc791\uc9c4\uc5d0\ub294 \uc138\ub974\uc870 \ub808\uc624\ub124 \uac00 \uc788\uc5c8\ub294\ub370 \uadf8\ub294 \uc120\uc784 \uc870\uac10\ub3c5 \uc9c1\ubd84\uc774\uc5c8\uc73c\uba70 \uc7ac\ucd2c\uc601 \uc791\uc5c5\uc744 \ub9e1\uc558\ub2e4. \uc640\uc77c\ub7ec \uac10\ub3c5\uc740 \uacbd\uc8fc \uc804 \ub3c4\uc5f4 \uc608\uc2dd \uc7a5\uba74, \ud658\ud638\ud558\ub294 \uad70\uc911\ub4e4, \ubca4\ud5c8\uc758 \uc804\ucc28\uac00 \uacb0\uc2b9\uc120\uc744 \ud1b5\uacfc\ud588\uc744 \ub54c\uc758 \uacbd\uae30\uc7a5 \ubaa8\uc2b5 \ub4f1\uc744 \ucd2c\uc601\ud588\ub2e4. \ub3c4\uc5f4 \uc7a5\uba74\uc740 1925\ub144 \ubb34\uc131\uc601\ud654 \ubc84\uc804\uc758 \uac19\uc740 \ubd80\ubd84\uc744 \ud55c \uc7a5\uba74 \ud55c \uc7a5\uba74\ubcc4\ub85c \ub9ac\uba54\uc774\ud06c\ud55c \uac83\uc774\ub2e4. \uc640\uc77c\ub7ec\ub294 \uc804\ucc28\uacbd\uae30\uac00 \ub300\ubd80\ubd84 \uc911\uac04 \uac70\ub9ac \ubc0f \uadfc\uac70\ub9ac \uc2dc\uc810\uc73c\ub85c \uc9c4\ud589\ub418\uae30 \ub54c\ubb38\uc5d0 \uad00\uac1d\ub4e4\uc774 \uacbd\uae30\uc7a5\uc758 \uc6c5\uc7a5\ud568\uc744 \uc2e4\uac10\ud560 \uc218 \uc788\ub3c4\ub85d \uc6d0\uac70\ub9ac\uc5d0\uc11c\uc758 \uc804\ucc28\uc218 \ud589\uc9c4 \uc7a5\uba74\uc744 \uc0bd\uc785\ud588\ub2e4. (\ub2e4\ub9cc \ub3c4\uc5f4 \uc608\uc2dd\uc740 \uace0\uc99d\uc0c1 \uc815\ud655\ud55c \uc7ac\ud604\uc740 \uc544\ub2c8\uc5c8\ub2e4. )", "paragraph_answer": "\uc564\ub4dc\ub958 \ub9c8\ud2bc\uacfc \uc57c\ud0a4\ub9c8 \uce74\ub204\ud2b8\uac00 \uc601\ud654 \ud6c4\ubc18\ubd80 \uc804\ucc28\uacbd\uc8fc \uc7a5\uba74 \uc81c\uc791\uc744 \uc9c0\ud718\ud588\ub294\ub370 \ub458 \ub2e4 \uc774\uc804\ubd80\ud130 \ud0c0 \uc791\ud488\uc5d0\uc11c \uc81c2 \uc81c\uc791\uc9c4 \uac10\ub3c5 \uc5ed\ud560\uc744 \uc790\uc8fc \ub9e1\uc740 \uacbd\ub825\uc790\uc600\ub2e4. \ub458\uc740 \uac01\uc790 \ud718\ud558\uc5d0 \uc870\uac10\ub3c5 \ud55c \uba85\uc529\uc744 \uac70\ub290\ub838\ub294\ub370 \uc774\ub4e4\uc740 \ucd94\uac00 \uc7a5\uba74\uc744 \ucc0d\ub294 \uc77c\uc744 \ub9e1\uc558\ub2e4. \uc81c2 \uc81c\uc791\uc9c4\uc5d0\ub294 \uc138\ub974\uc870 \ub808\uc624\ub124 \uac00 \uc788\uc5c8\ub294\ub370 \uadf8\ub294 \uc120\uc784 \uc870\uac10\ub3c5 \uc9c1\ubd84\uc774\uc5c8\uc73c\uba70 \uc7ac\ucd2c\uc601 \uc791\uc5c5\uc744 \ub9e1\uc558\ub2e4. \uc640\uc77c\ub7ec \uac10\ub3c5\uc740 \uacbd\uc8fc \uc804 \ub3c4\uc5f4 \uc608\uc2dd \uc7a5\uba74, \ud658\ud638\ud558\ub294 \uad70\uc911\ub4e4, \ubca4\ud5c8\uc758 \uc804\ucc28\uac00 \uacb0\uc2b9\uc120\uc744 \ud1b5\uacfc\ud588\uc744 \ub54c\uc758 \uacbd\uae30\uc7a5 \ubaa8\uc2b5 \ub4f1\uc744 \ucd2c\uc601\ud588\ub2e4. \ub3c4\uc5f4 \uc7a5\uba74\uc740 1925\ub144 \ubb34\uc131\uc601\ud654 \ubc84\uc804\uc758 \uac19\uc740 \ubd80\ubd84\uc744 \ud55c \uc7a5\uba74 \ud55c \uc7a5\uba74\ubcc4\ub85c \ub9ac\uba54\uc774\ud06c\ud55c \uac83\uc774\ub2e4. \uc640\uc77c\ub7ec\ub294 \uc804\ucc28\uacbd\uae30\uac00 \ub300\ubd80\ubd84 \uc911\uac04 \uac70\ub9ac \ubc0f \uadfc\uac70\ub9ac \uc2dc\uc810\uc73c\ub85c \uc9c4\ud589\ub418\uae30 \ub54c\ubb38\uc5d0 \uad00\uac1d\ub4e4\uc774 \uacbd\uae30\uc7a5\uc758 \uc6c5\uc7a5\ud568\uc744 \uc2e4\uac10\ud560 \uc218 \uc788\ub3c4\ub85d \uc6d0\uac70\ub9ac\uc5d0\uc11c\uc758 \uc804\ucc28\uc218 \ud589\uc9c4 \uc7a5\uba74\uc744 \uc0bd\uc785\ud588\ub2e4.(\ub2e4\ub9cc \ub3c4\uc5f4 \uc608\uc2dd\uc740 \uace0\uc99d\uc0c1 \uc815\ud655\ud55c \uc7ac\ud604\uc740 \uc544\ub2c8\uc5c8\ub2e4.)", "sentence_answer": "\uc81c2 \uc81c\uc791\uc9c4\uc5d0\ub294 \uc138\ub974\uc870 \ub808\uc624\ub124 \uac00 \uc788\uc5c8\ub294\ub370 \uadf8\ub294 \uc120\uc784 \uc870\uac10\ub3c5 \uc9c1\ubd84\uc774\uc5c8\uc73c\uba70 \uc7ac\ucd2c\uc601 \uc791\uc5c5\uc744 \ub9e1\uc558\ub2e4.", "paragraph_id": "6530302-29-1", "paragraph_question": "question: \ubca4\ud5c8 \uc81c2\uc81c\uc791\uc9c4\uc758 \uc120\uc784 \uc870\uac10\ub3c5\uc740?, context: \uc564\ub4dc\ub958 \ub9c8\ud2bc\uacfc \uc57c\ud0a4\ub9c8 \uce74\ub204\ud2b8\uac00 \uc601\ud654 \ud6c4\ubc18\ubd80 \uc804\ucc28\uacbd\uc8fc \uc7a5\uba74 \uc81c\uc791\uc744 \uc9c0\ud718\ud588\ub294\ub370 \ub458 \ub2e4 \uc774\uc804\ubd80\ud130 \ud0c0 \uc791\ud488\uc5d0\uc11c \uc81c2 \uc81c\uc791\uc9c4 \uac10\ub3c5 \uc5ed\ud560\uc744 \uc790\uc8fc \ub9e1\uc740 \uacbd\ub825\uc790\uc600\ub2e4. \ub458\uc740 \uac01\uc790 \ud718\ud558\uc5d0 \uc870\uac10\ub3c5 \ud55c \uba85\uc529\uc744 \uac70\ub290\ub838\ub294\ub370 \uc774\ub4e4\uc740 \ucd94\uac00 \uc7a5\uba74\uc744 \ucc0d\ub294 \uc77c\uc744 \ub9e1\uc558\ub2e4. \uc81c2 \uc81c\uc791\uc9c4\uc5d0\ub294 \uc138\ub974\uc870 \ub808\uc624\ub124\uac00 \uc788\uc5c8\ub294\ub370 \uadf8\ub294 \uc120\uc784 \uc870\uac10\ub3c5 \uc9c1\ubd84\uc774\uc5c8\uc73c\uba70 \uc7ac\ucd2c\uc601 \uc791\uc5c5\uc744 \ub9e1\uc558\ub2e4. \uc640\uc77c\ub7ec \uac10\ub3c5\uc740 \uacbd\uc8fc \uc804 \ub3c4\uc5f4 \uc608\uc2dd \uc7a5\uba74, \ud658\ud638\ud558\ub294 \uad70\uc911\ub4e4, \ubca4\ud5c8\uc758 \uc804\ucc28\uac00 \uacb0\uc2b9\uc120\uc744 \ud1b5\uacfc\ud588\uc744 \ub54c\uc758 \uacbd\uae30\uc7a5 \ubaa8\uc2b5 \ub4f1\uc744 \ucd2c\uc601\ud588\ub2e4. \ub3c4\uc5f4 \uc7a5\uba74\uc740 1925\ub144 \ubb34\uc131\uc601\ud654 \ubc84\uc804\uc758 \uac19\uc740 \ubd80\ubd84\uc744 \ud55c \uc7a5\uba74 \ud55c \uc7a5\uba74\ubcc4\ub85c \ub9ac\uba54\uc774\ud06c\ud55c \uac83\uc774\ub2e4. \uc640\uc77c\ub7ec\ub294 \uc804\ucc28\uacbd\uae30\uac00 \ub300\ubd80\ubd84 \uc911\uac04 \uac70\ub9ac \ubc0f \uadfc\uac70\ub9ac \uc2dc\uc810\uc73c\ub85c \uc9c4\ud589\ub418\uae30 \ub54c\ubb38\uc5d0 \uad00\uac1d\ub4e4\uc774 \uacbd\uae30\uc7a5\uc758 \uc6c5\uc7a5\ud568\uc744 \uc2e4\uac10\ud560 \uc218 \uc788\ub3c4\ub85d \uc6d0\uac70\ub9ac\uc5d0\uc11c\uc758 \uc804\ucc28\uc218 \ud589\uc9c4 \uc7a5\uba74\uc744 \uc0bd\uc785\ud588\ub2e4.(\ub2e4\ub9cc \ub3c4\uc5f4 \uc608\uc2dd\uc740 \uace0\uc99d\uc0c1 \uc815\ud655\ud55c \uc7ac\ud604\uc740 \uc544\ub2c8\uc5c8\ub2e4.)"} {"question": "\ubd88\uad50\uc5d0\uc11c \ucd95\uc801\ub41c \uc120\uc5c5\uc5d0 \uc758\ud574 \uc0dd\ucc9c \ud558\uae30 \uc704\ud574 \ud589\ud574\uc57c \ud558\ub294 \uac83\uc740?", "paragraph": "\uc608\ub97c \ub4e4\uc5b4, \ud78c\ub450\uad50\uc758 \uc815\ud1b5 6\ud30c\ucca0\ud559 \uc911\uc758 \ud558\ub098\uc778 \ubbf8\ub9d8\uc0ac \ud559\ud30c\uc5d0 \ub530\ub974\uba74, \u300a\ubca0\ub2e4\u300b\uc5d0 \uaddc\uc815\ub418\uc5b4 \uc788\ub294 \ud2b9\uc815\ud55c \uc81c\uc0ac(\u796d\u7940: \ud78c\ub450\uad50\uc758 \uc785\uc7a5\uc5d0\uc11c\ub294 \ub2e8\uc21c\ud55c \uc81c\uc0ac\uac00 \uc544\ub2cc \uae30\ub3c4\uc640 \uc9c4\uc5b8\uacfc \uc9d1\uc911 \ub4f1\uc774 \uc218\ubc18\ub41c \ud2b9\uc815\ud55c \uc218\ud589\ubc95 \ub610\ub294 \uc218\ud589\uccb4\uacc4\uc758 \uc2e4\ucc9c\uc774\ub77c\uace0 \ud560 \uc218 \uc788\uc74c, \ub9d0\ud558\uc790\uba74 \uac00\ud1a8\ub9ad\uc758 \ubbf8\uc0ac\ub098 \uac1c\uc2e0\uad50\uc758 \uc608\ubc30, \ubd88\uad50\uc758 \uacf5\uc591 \ub610\ub294 \uc608\ubd88 \ub610\ub294 \uc5fc\ubd88\uc5d0 \ud574\ub2f9\ud558\ub294 \uac83\uc774\ub77c\uace0 \ud560 \uc218 \uc788\uc74c)\ub97c \uc2e4\ud589\ud558\uba74 \uadf8 \uc2e4\ud589\uc790\uc5d0\uac8c \uc77c\uc885\uc758 \uc5ec\ub825\uc774 \ub0a8\uc544 \uadf8\uac83\uc774 \ub098\uc911\uc5d0 \uacfc\ubcf4\ub97c \uac00\uc838\uc628\ub2e4. \uc774 \ub54c\uc758 \uc5ec\ub825\uc744 \uc2e0\ub4dd\ub825(\u65b0\u5f97\u529b: \uc0c8\ub85c\uc774 \uc5bb\uc740 \ud798)\uc774\ub77c \ud558\ub294\ub370, \ubbf8\ub9d8\uc0ac \ud559\ud30c\uc5d0\uc11c\ub294 \uc774 \uc2e0\ub4dd\ub825\uc758 \ucd1d\ud569\uc774 \ubc14\ub974\uace0 \ucc29\ud558\uba74 \ub2e4\uc74c \uc0dd\uc5d0\uc11c \uc0dd\ucc9c(\u751f\u5929: \ucc9c\uacc4\uc5d0 \ud0dc\uc5b4\ub0a8) \ub4f1\uc758 \ubc88\uc601(\u7e41\u69ae, abhyudaya)\uc758 \uacfc\ubcf4\ub97c \uc5bb\uc5b4 \ubcf5\ub77d\uc744 \ud5a5\uc218\ud55c\ub2e4\ub294 \uad50\uc758\ub97c \uac00\ub974\ucce4\ub2e4. \uc0dd\ucc9c(\u751f\u5929)\uc5d0 \ub300\ud55c \uc0ac\uc0c1\uc774 \uc788\uc5c8\ub2e4\ub294 \uac83\uc5d0\ub294 \ubbf8\ub9d8\uc0ac \ud559\ud30c\uc640 \ubd88\uad50\uac04\uc5d0 \uc11c\ub85c \uc77c\uce58\ud558\ub294 \uc0ac\ud56d\uc774\uc9c0\ub9cc, \ubbf8\ub9d8\uc0ac \ud559\ud30c\uac00 \u300a\ubca0\ub2e4\u300b\uc5d0 \ub530\ub978 \uc81c\uc0ac\uc758 \uc2e4\ud589\uc744 \ud1b5\ud574 \ucd95\uc801\ub41c \uc2e0\ub4dd\ub825(\u65b0\u5f97\u529b: \uc0c8\ub85c\uc774 \uc5bb\uc740 \ud798, \uc989 \ucda9\ubd84\ud788 \ucd95\uc801\ub41c \uacbd\uc6b0 \uc774\uc804\uc758 \uc5c5\uc744 \ubc14\uafc0 \uc218 \uc788\ub294 \uc0c8\ub85c\uc6b4 \ud798)\uc5d0 \uc758\ud574 \uc0dd\ucc9c(\u751f\u5929)\ud560 \uc218 \uc788\ub2e4\uace0 \uc8fc\uc7a5\ud55c \ubc18\uba74, \ubd88\uad50\uc5d0\uc11c\ub294 5\uacc4 \u00b7 10\uc120 \u00b7 \ubcf4\uc2dc \ub4f1\uc758 \uc138\uac04\uc758 \uc120\ubc95[\u4e16\u9593\u5584\u6cd5]\uc744 \ud589\ud568\uc73c\ub85c\uc368 \ucd95\uc801\ub41c \uc120\uc5c5(\u5584\u696d: \uc120\ud55c \uce74\ub974\ub9c8, \uc989 \uc120\ud55c \uc7a0\uc7ac\ub825 \ub610\ub294 \ud615\uc131\ub825)\uc5d0 \uc758\ud574 \uc0dd\ucc9c(\u751f\u5929)\ud560 \uc218 \uc788\ub2e4\uace0 \uc8fc\uc7a5\ud55c \uac83\uc740 \uc11c\ub85c \uac04\uc758 \ucc28\uc774\uc810\uc774\ub2e4.", "answer": "\uc138\uac04\uc758 \uc120\ubc95", "sentence": "\uc0dd\ucc9c(\u751f\u5929)\uc5d0 \ub300\ud55c \uc0ac\uc0c1\uc774 \uc788\uc5c8\ub2e4\ub294 \uac83\uc5d0\ub294 \ubbf8\ub9d8\uc0ac \ud559\ud30c\uc640 \ubd88\uad50\uac04\uc5d0 \uc11c\ub85c \uc77c\uce58\ud558\ub294 \uc0ac\ud56d\uc774\uc9c0\ub9cc, \ubbf8\ub9d8\uc0ac \ud559\ud30c\uac00 \u300a\ubca0\ub2e4\u300b\uc5d0 \ub530\ub978 \uc81c\uc0ac\uc758 \uc2e4\ud589\uc744 \ud1b5\ud574 \ucd95\uc801\ub41c \uc2e0\ub4dd\ub825(\u65b0\u5f97\u529b: \uc0c8\ub85c\uc774 \uc5bb\uc740 \ud798, \uc989 \ucda9\ubd84\ud788 \ucd95\uc801\ub41c \uacbd\uc6b0 \uc774\uc804\uc758 \uc5c5\uc744 \ubc14\uafc0 \uc218 \uc788\ub294 \uc0c8\ub85c\uc6b4 \ud798)\uc5d0 \uc758\ud574 \uc0dd\ucc9c(\u751f\u5929)\ud560 \uc218 \uc788\ub2e4\uace0 \uc8fc\uc7a5\ud55c \ubc18\uba74, \ubd88\uad50\uc5d0\uc11c\ub294 5\uacc4 \u00b7 10\uc120 \u00b7 \ubcf4\uc2dc \ub4f1\uc758 \uc138\uac04\uc758 \uc120\ubc95 [\u4e16\u9593\u5584\u6cd5]\uc744 \ud589\ud568\uc73c\ub85c\uc368 \ucd95\uc801\ub41c \uc120\uc5c5(\u5584\u696d: \uc120\ud55c \uce74\ub974\ub9c8, \uc989 \uc120\ud55c \uc7a0\uc7ac\ub825 \ub610\ub294 \ud615\uc131\ub825)\uc5d0 \uc758\ud574 \uc0dd\ucc9c(\u751f\u5929)\ud560 \uc218 \uc788\ub2e4\uace0 \uc8fc\uc7a5\ud55c \uac83\uc740 \uc11c\ub85c \uac04\uc758 \ucc28\uc774\uc810\uc774\ub2e4.", "paragraph_sentence": "\uc608\ub97c \ub4e4\uc5b4, \ud78c\ub450\uad50\uc758 \uc815\ud1b5 6\ud30c\ucca0\ud559 \uc911\uc758 \ud558\ub098\uc778 \ubbf8\ub9d8\uc0ac \ud559\ud30c\uc5d0 \ub530\ub974\uba74, \u300a\ubca0\ub2e4\u300b\uc5d0 \uaddc\uc815\ub418\uc5b4 \uc788\ub294 \ud2b9\uc815\ud55c \uc81c\uc0ac(\u796d\u7940: \ud78c\ub450\uad50\uc758 \uc785\uc7a5\uc5d0\uc11c\ub294 \ub2e8\uc21c\ud55c \uc81c\uc0ac\uac00 \uc544\ub2cc \uae30\ub3c4\uc640 \uc9c4\uc5b8\uacfc \uc9d1\uc911 \ub4f1\uc774 \uc218\ubc18\ub41c \ud2b9\uc815\ud55c \uc218\ud589\ubc95 \ub610\ub294 \uc218\ud589\uccb4\uacc4\uc758 \uc2e4\ucc9c\uc774\ub77c\uace0 \ud560 \uc218 \uc788\uc74c, \ub9d0\ud558\uc790\uba74 \uac00\ud1a8\ub9ad\uc758 \ubbf8\uc0ac\ub098 \uac1c\uc2e0\uad50\uc758 \uc608\ubc30, \ubd88\uad50\uc758 \uacf5\uc591 \ub610\ub294 \uc608\ubd88 \ub610\ub294 \uc5fc\ubd88\uc5d0 \ud574\ub2f9\ud558\ub294 \uac83\uc774\ub77c\uace0 \ud560 \uc218 \uc788\uc74c)\ub97c \uc2e4\ud589\ud558\uba74 \uadf8 \uc2e4\ud589\uc790\uc5d0\uac8c \uc77c\uc885\uc758 \uc5ec\ub825\uc774 \ub0a8\uc544 \uadf8\uac83\uc774 \ub098\uc911\uc5d0 \uacfc\ubcf4\ub97c \uac00\uc838\uc628\ub2e4. \uc774 \ub54c\uc758 \uc5ec\ub825\uc744 \uc2e0\ub4dd\ub825(\u65b0\u5f97\u529b: \uc0c8\ub85c\uc774 \uc5bb\uc740 \ud798)\uc774\ub77c \ud558\ub294\ub370, \ubbf8\ub9d8\uc0ac \ud559\ud30c\uc5d0\uc11c\ub294 \uc774 \uc2e0\ub4dd\ub825\uc758 \ucd1d\ud569\uc774 \ubc14\ub974\uace0 \ucc29\ud558\uba74 \ub2e4\uc74c \uc0dd\uc5d0\uc11c \uc0dd\ucc9c(\u751f\u5929: \ucc9c\uacc4\uc5d0 \ud0dc\uc5b4\ub0a8) \ub4f1\uc758 \ubc88\uc601(\u7e41\u69ae, abhyudaya)\uc758 \uacfc\ubcf4\ub97c \uc5bb\uc5b4 \ubcf5\ub77d\uc744 \ud5a5\uc218\ud55c\ub2e4\ub294 \uad50\uc758\ub97c \uac00\ub974\ucce4\ub2e4. \uc0dd\ucc9c(\u751f\u5929)\uc5d0 \ub300\ud55c \uc0ac\uc0c1\uc774 \uc788\uc5c8\ub2e4\ub294 \uac83\uc5d0\ub294 \ubbf8\ub9d8\uc0ac \ud559\ud30c\uc640 \ubd88\uad50\uac04\uc5d0 \uc11c\ub85c \uc77c\uce58\ud558\ub294 \uc0ac\ud56d\uc774\uc9c0\ub9cc, \ubbf8\ub9d8\uc0ac \ud559\ud30c\uac00 \u300a\ubca0\ub2e4\u300b\uc5d0 \ub530\ub978 \uc81c\uc0ac\uc758 \uc2e4\ud589\uc744 \ud1b5\ud574 \ucd95\uc801\ub41c \uc2e0\ub4dd\ub825(\u65b0\u5f97\u529b: \uc0c8\ub85c\uc774 \uc5bb\uc740 \ud798, \uc989 \ucda9\ubd84\ud788 \ucd95\uc801\ub41c \uacbd\uc6b0 \uc774\uc804\uc758 \uc5c5\uc744 \ubc14\uafc0 \uc218 \uc788\ub294 \uc0c8\ub85c\uc6b4 \ud798)\uc5d0 \uc758\ud574 \uc0dd\ucc9c(\u751f\u5929)\ud560 \uc218 \uc788\ub2e4\uace0 \uc8fc\uc7a5\ud55c \ubc18\uba74, \ubd88\uad50\uc5d0\uc11c\ub294 5\uacc4 \u00b7 10\uc120 \u00b7 \ubcf4\uc2dc \ub4f1\uc758 \uc138\uac04\uc758 \uc120\ubc95 [\u4e16\u9593\u5584\u6cd5]\uc744 \ud589\ud568\uc73c\ub85c\uc368 \ucd95\uc801\ub41c \uc120\uc5c5(\u5584\u696d: \uc120\ud55c \uce74\ub974\ub9c8, \uc989 \uc120\ud55c \uc7a0\uc7ac\ub825 \ub610\ub294 \ud615\uc131\ub825)\uc5d0 \uc758\ud574 \uc0dd\ucc9c(\u751f\u5929)\ud560 \uc218 \uc788\ub2e4\uace0 \uc8fc\uc7a5\ud55c \uac83\uc740 \uc11c\ub85c \uac04\uc758 \ucc28\uc774\uc810\uc774\ub2e4. ", "paragraph_answer": "\uc608\ub97c \ub4e4\uc5b4, \ud78c\ub450\uad50\uc758 \uc815\ud1b5 6\ud30c\ucca0\ud559 \uc911\uc758 \ud558\ub098\uc778 \ubbf8\ub9d8\uc0ac \ud559\ud30c\uc5d0 \ub530\ub974\uba74, \u300a\ubca0\ub2e4\u300b\uc5d0 \uaddc\uc815\ub418\uc5b4 \uc788\ub294 \ud2b9\uc815\ud55c \uc81c\uc0ac(\u796d\u7940: \ud78c\ub450\uad50\uc758 \uc785\uc7a5\uc5d0\uc11c\ub294 \ub2e8\uc21c\ud55c \uc81c\uc0ac\uac00 \uc544\ub2cc \uae30\ub3c4\uc640 \uc9c4\uc5b8\uacfc \uc9d1\uc911 \ub4f1\uc774 \uc218\ubc18\ub41c \ud2b9\uc815\ud55c \uc218\ud589\ubc95 \ub610\ub294 \uc218\ud589\uccb4\uacc4\uc758 \uc2e4\ucc9c\uc774\ub77c\uace0 \ud560 \uc218 \uc788\uc74c, \ub9d0\ud558\uc790\uba74 \uac00\ud1a8\ub9ad\uc758 \ubbf8\uc0ac\ub098 \uac1c\uc2e0\uad50\uc758 \uc608\ubc30, \ubd88\uad50\uc758 \uacf5\uc591 \ub610\ub294 \uc608\ubd88 \ub610\ub294 \uc5fc\ubd88\uc5d0 \ud574\ub2f9\ud558\ub294 \uac83\uc774\ub77c\uace0 \ud560 \uc218 \uc788\uc74c)\ub97c \uc2e4\ud589\ud558\uba74 \uadf8 \uc2e4\ud589\uc790\uc5d0\uac8c \uc77c\uc885\uc758 \uc5ec\ub825\uc774 \ub0a8\uc544 \uadf8\uac83\uc774 \ub098\uc911\uc5d0 \uacfc\ubcf4\ub97c \uac00\uc838\uc628\ub2e4. \uc774 \ub54c\uc758 \uc5ec\ub825\uc744 \uc2e0\ub4dd\ub825(\u65b0\u5f97\u529b: \uc0c8\ub85c\uc774 \uc5bb\uc740 \ud798)\uc774\ub77c \ud558\ub294\ub370, \ubbf8\ub9d8\uc0ac \ud559\ud30c\uc5d0\uc11c\ub294 \uc774 \uc2e0\ub4dd\ub825\uc758 \ucd1d\ud569\uc774 \ubc14\ub974\uace0 \ucc29\ud558\uba74 \ub2e4\uc74c \uc0dd\uc5d0\uc11c \uc0dd\ucc9c(\u751f\u5929: \ucc9c\uacc4\uc5d0 \ud0dc\uc5b4\ub0a8) \ub4f1\uc758 \ubc88\uc601(\u7e41\u69ae, abhyudaya)\uc758 \uacfc\ubcf4\ub97c \uc5bb\uc5b4 \ubcf5\ub77d\uc744 \ud5a5\uc218\ud55c\ub2e4\ub294 \uad50\uc758\ub97c \uac00\ub974\ucce4\ub2e4. \uc0dd\ucc9c(\u751f\u5929)\uc5d0 \ub300\ud55c \uc0ac\uc0c1\uc774 \uc788\uc5c8\ub2e4\ub294 \uac83\uc5d0\ub294 \ubbf8\ub9d8\uc0ac \ud559\ud30c\uc640 \ubd88\uad50\uac04\uc5d0 \uc11c\ub85c \uc77c\uce58\ud558\ub294 \uc0ac\ud56d\uc774\uc9c0\ub9cc, \ubbf8\ub9d8\uc0ac \ud559\ud30c\uac00 \u300a\ubca0\ub2e4\u300b\uc5d0 \ub530\ub978 \uc81c\uc0ac\uc758 \uc2e4\ud589\uc744 \ud1b5\ud574 \ucd95\uc801\ub41c \uc2e0\ub4dd\ub825(\u65b0\u5f97\u529b: \uc0c8\ub85c\uc774 \uc5bb\uc740 \ud798, \uc989 \ucda9\ubd84\ud788 \ucd95\uc801\ub41c \uacbd\uc6b0 \uc774\uc804\uc758 \uc5c5\uc744 \ubc14\uafc0 \uc218 \uc788\ub294 \uc0c8\ub85c\uc6b4 \ud798)\uc5d0 \uc758\ud574 \uc0dd\ucc9c(\u751f\u5929)\ud560 \uc218 \uc788\ub2e4\uace0 \uc8fc\uc7a5\ud55c \ubc18\uba74, \ubd88\uad50\uc5d0\uc11c\ub294 5\uacc4 \u00b7 10\uc120 \u00b7 \ubcf4\uc2dc \ub4f1\uc758 \uc138\uac04\uc758 \uc120\ubc95 [\u4e16\u9593\u5584\u6cd5]\uc744 \ud589\ud568\uc73c\ub85c\uc368 \ucd95\uc801\ub41c \uc120\uc5c5(\u5584\u696d: \uc120\ud55c \uce74\ub974\ub9c8, \uc989 \uc120\ud55c \uc7a0\uc7ac\ub825 \ub610\ub294 \ud615\uc131\ub825)\uc5d0 \uc758\ud574 \uc0dd\ucc9c(\u751f\u5929)\ud560 \uc218 \uc788\ub2e4\uace0 \uc8fc\uc7a5\ud55c \uac83\uc740 \uc11c\ub85c \uac04\uc758 \ucc28\uc774\uc810\uc774\ub2e4.", "sentence_answer": "\uc0dd\ucc9c(\u751f\u5929)\uc5d0 \ub300\ud55c \uc0ac\uc0c1\uc774 \uc788\uc5c8\ub2e4\ub294 \uac83\uc5d0\ub294 \ubbf8\ub9d8\uc0ac \ud559\ud30c\uc640 \ubd88\uad50\uac04\uc5d0 \uc11c\ub85c \uc77c\uce58\ud558\ub294 \uc0ac\ud56d\uc774\uc9c0\ub9cc, \ubbf8\ub9d8\uc0ac \ud559\ud30c\uac00 \u300a\ubca0\ub2e4\u300b\uc5d0 \ub530\ub978 \uc81c\uc0ac\uc758 \uc2e4\ud589\uc744 \ud1b5\ud574 \ucd95\uc801\ub41c \uc2e0\ub4dd\ub825(\u65b0\u5f97\u529b: \uc0c8\ub85c\uc774 \uc5bb\uc740 \ud798, \uc989 \ucda9\ubd84\ud788 \ucd95\uc801\ub41c \uacbd\uc6b0 \uc774\uc804\uc758 \uc5c5\uc744 \ubc14\uafc0 \uc218 \uc788\ub294 \uc0c8\ub85c\uc6b4 \ud798)\uc5d0 \uc758\ud574 \uc0dd\ucc9c(\u751f\u5929)\ud560 \uc218 \uc788\ub2e4\uace0 \uc8fc\uc7a5\ud55c \ubc18\uba74, \ubd88\uad50\uc5d0\uc11c\ub294 5\uacc4 \u00b7 10\uc120 \u00b7 \ubcf4\uc2dc \ub4f1\uc758 \uc138\uac04\uc758 \uc120\ubc95 [\u4e16\u9593\u5584\u6cd5]\uc744 \ud589\ud568\uc73c\ub85c\uc368 \ucd95\uc801\ub41c \uc120\uc5c5(\u5584\u696d: \uc120\ud55c \uce74\ub974\ub9c8, \uc989 \uc120\ud55c \uc7a0\uc7ac\ub825 \ub610\ub294 \ud615\uc131\ub825)\uc5d0 \uc758\ud574 \uc0dd\ucc9c(\u751f\u5929)\ud560 \uc218 \uc788\ub2e4\uace0 \uc8fc\uc7a5\ud55c \uac83\uc740 \uc11c\ub85c \uac04\uc758 \ucc28\uc774\uc810\uc774\ub2e4.", "paragraph_id": "6596528-2-0", "paragraph_question": "question: \ubd88\uad50\uc5d0\uc11c \ucd95\uc801\ub41c \uc120\uc5c5\uc5d0 \uc758\ud574 \uc0dd\ucc9c \ud558\uae30 \uc704\ud574 \ud589\ud574\uc57c \ud558\ub294 \uac83\uc740?, context: \uc608\ub97c \ub4e4\uc5b4, \ud78c\ub450\uad50\uc758 \uc815\ud1b5 6\ud30c\ucca0\ud559 \uc911\uc758 \ud558\ub098\uc778 \ubbf8\ub9d8\uc0ac \ud559\ud30c\uc5d0 \ub530\ub974\uba74, \u300a\ubca0\ub2e4\u300b\uc5d0 \uaddc\uc815\ub418\uc5b4 \uc788\ub294 \ud2b9\uc815\ud55c \uc81c\uc0ac(\u796d\u7940: \ud78c\ub450\uad50\uc758 \uc785\uc7a5\uc5d0\uc11c\ub294 \ub2e8\uc21c\ud55c \uc81c\uc0ac\uac00 \uc544\ub2cc \uae30\ub3c4\uc640 \uc9c4\uc5b8\uacfc \uc9d1\uc911 \ub4f1\uc774 \uc218\ubc18\ub41c \ud2b9\uc815\ud55c \uc218\ud589\ubc95 \ub610\ub294 \uc218\ud589\uccb4\uacc4\uc758 \uc2e4\ucc9c\uc774\ub77c\uace0 \ud560 \uc218 \uc788\uc74c, \ub9d0\ud558\uc790\uba74 \uac00\ud1a8\ub9ad\uc758 \ubbf8\uc0ac\ub098 \uac1c\uc2e0\uad50\uc758 \uc608\ubc30, \ubd88\uad50\uc758 \uacf5\uc591 \ub610\ub294 \uc608\ubd88 \ub610\ub294 \uc5fc\ubd88\uc5d0 \ud574\ub2f9\ud558\ub294 \uac83\uc774\ub77c\uace0 \ud560 \uc218 \uc788\uc74c)\ub97c \uc2e4\ud589\ud558\uba74 \uadf8 \uc2e4\ud589\uc790\uc5d0\uac8c \uc77c\uc885\uc758 \uc5ec\ub825\uc774 \ub0a8\uc544 \uadf8\uac83\uc774 \ub098\uc911\uc5d0 \uacfc\ubcf4\ub97c \uac00\uc838\uc628\ub2e4. \uc774 \ub54c\uc758 \uc5ec\ub825\uc744 \uc2e0\ub4dd\ub825(\u65b0\u5f97\u529b: \uc0c8\ub85c\uc774 \uc5bb\uc740 \ud798)\uc774\ub77c \ud558\ub294\ub370, \ubbf8\ub9d8\uc0ac \ud559\ud30c\uc5d0\uc11c\ub294 \uc774 \uc2e0\ub4dd\ub825\uc758 \ucd1d\ud569\uc774 \ubc14\ub974\uace0 \ucc29\ud558\uba74 \ub2e4\uc74c \uc0dd\uc5d0\uc11c \uc0dd\ucc9c(\u751f\u5929: \ucc9c\uacc4\uc5d0 \ud0dc\uc5b4\ub0a8) \ub4f1\uc758 \ubc88\uc601(\u7e41\u69ae, abhyudaya)\uc758 \uacfc\ubcf4\ub97c \uc5bb\uc5b4 \ubcf5\ub77d\uc744 \ud5a5\uc218\ud55c\ub2e4\ub294 \uad50\uc758\ub97c \uac00\ub974\ucce4\ub2e4. \uc0dd\ucc9c(\u751f\u5929)\uc5d0 \ub300\ud55c \uc0ac\uc0c1\uc774 \uc788\uc5c8\ub2e4\ub294 \uac83\uc5d0\ub294 \ubbf8\ub9d8\uc0ac \ud559\ud30c\uc640 \ubd88\uad50\uac04\uc5d0 \uc11c\ub85c \uc77c\uce58\ud558\ub294 \uc0ac\ud56d\uc774\uc9c0\ub9cc, \ubbf8\ub9d8\uc0ac \ud559\ud30c\uac00 \u300a\ubca0\ub2e4\u300b\uc5d0 \ub530\ub978 \uc81c\uc0ac\uc758 \uc2e4\ud589\uc744 \ud1b5\ud574 \ucd95\uc801\ub41c \uc2e0\ub4dd\ub825(\u65b0\u5f97\u529b: \uc0c8\ub85c\uc774 \uc5bb\uc740 \ud798, \uc989 \ucda9\ubd84\ud788 \ucd95\uc801\ub41c \uacbd\uc6b0 \uc774\uc804\uc758 \uc5c5\uc744 \ubc14\uafc0 \uc218 \uc788\ub294 \uc0c8\ub85c\uc6b4 \ud798)\uc5d0 \uc758\ud574 \uc0dd\ucc9c(\u751f\u5929)\ud560 \uc218 \uc788\ub2e4\uace0 \uc8fc\uc7a5\ud55c \ubc18\uba74, \ubd88\uad50\uc5d0\uc11c\ub294 5\uacc4 \u00b7 10\uc120 \u00b7 \ubcf4\uc2dc \ub4f1\uc758 \uc138\uac04\uc758 \uc120\ubc95[\u4e16\u9593\u5584\u6cd5]\uc744 \ud589\ud568\uc73c\ub85c\uc368 \ucd95\uc801\ub41c \uc120\uc5c5(\u5584\u696d: \uc120\ud55c \uce74\ub974\ub9c8, \uc989 \uc120\ud55c \uc7a0\uc7ac\ub825 \ub610\ub294 \ud615\uc131\ub825)\uc5d0 \uc758\ud574 \uc0dd\ucc9c(\u751f\u5929)\ud560 \uc218 \uc788\ub2e4\uace0 \uc8fc\uc7a5\ud55c \uac83\uc740 \uc11c\ub85c \uac04\uc758 \ucc28\uc774\uc810\uc774\ub2e4."} {"question": "\uc784\uc2dc\ud568\uc120 \uac74\uc870\ubd80\ub97c \uc124\uce58\ud55c \uacf3\uc740 \uc5b4\ub514\uc778\uac00\uc694?", "paragraph": "\uc81c1\ucc28 \uc138\uacc4 \ub300\uc804\uc774 \ub05d\ub098\uace0 \ub098\uc11c \ubd88\ud669\uc774 \uc654\uc744 \ub54c\ub3c4 \ub610 \ub2e4\uc2dc \uacbd\uc601 \uc704\uae30\uc5d0 \ube60\uc84c\ub2e4. \uc774\ub54c\ub294 1919\ub144\uc758 \ub178\ub3d9\uc7c1\uc758, 1922\ub144\uc5d0 \uc6cc\uc2f1\ud134 \uad70\ucd95 \uc870\uc57d, 1923\ub144\uc758 \uad00\ub3d9\ub300\uc9c0\uc9c4\uc774 \uacb9\ucce4\ub2e4. \ub610\ud55c 1927\ub144 \uae08\uc735 \uacf5\ud669\uc5d0 \uc758\ud574 \uc8fc\uac70\ub798 \uc740\ud589 15\uac1c \uc740\ud589\uc774 \uc784\uc2dc \ud734\uc5c5\ud558\uba74\uc11c \uc790\uae08 \uc0ac\uc815\uc774 \uae09\uaca9\ud788 \uc545\ud654\ud588\ub2e4. \uc815\ubd80\uc640 \uace0\ubca0 \uc2dc\uc758 \uc6d0\uc870\ub85c \uc9c1\uc6d0 \uc57d 3,500\uba85\uc758 \ud574\uace0 \ud734\uc5c5, \uc784\uc6d0 \ubcf4\uc218 \uc0ad\uac10, \ub9c8\uce20\uce74\ud0c0 \uac00\ubb38\uacfc \uac00\uc640\uc0ac\ud0a4 \uac00\ubb38\uc5d0\uc11c \uc0ac\uc7ac\ub97c \ucd9c\uc5f0\ud558\ub294 \ub4f1\uc744 \ud1b5\ud574 \uacbd\uc601 \uc7ac\uac74\uc744 \uc2dc\ub3c4\ud588\ub2e4. \ub610\ud55c \ud6a8\uace0 \uacf5\uc7a5\uc744 \ubd84\ub9ac\ud558\uc5ec \u3008\uac00\uc640\uc0ac\ud0a4 \ucc28\ub7c9\u3009\uc73c\ub85c \ub3c5\ub9bd\uc801 \uc800\ub2f9\uad8c\uc744 \uc124\uc815\ud558\uace0, \ubcf8\uc0ac \uacf5\uc7a5\uacfc \u847a\u5408\uacf5\uc7a5\uc744 \ub2f4\ubcf4\ub85c \ub300\ucd9c\uc744 \ubc1b\uc558\ub2e4. \uc77c\ubcf8 \uc81c\uad6d \ud574\uad70\ub3c4 \uce78\uc138\uc774 \ubcf8\ubd80\uc5d0 \uc784\uc2dc\ud568\uc120 \uac74\uc870\ubd80\ub97c \uc124\uce58\ud558\uace0 \ud568\uc815 \uac74\uc870\ub97c \uacc4\uc18d\ud560 \uc218 \uc788\ub3c4\ub85d \ub3c4\uc654\ub2e4.", "answer": "\uc77c\ubcf8 \uc81c\uad6d \ud574\uad70\ub3c4 \uce78\uc138\uc774 \ubcf8\ubd80", "sentence": "\uc77c\ubcf8 \uc81c\uad6d \ud574\uad70\ub3c4 \uce78\uc138\uc774 \ubcf8\ubd80 \uc5d0 \uc784\uc2dc\ud568\uc120 \uac74\uc870\ubd80\ub97c \uc124\uce58\ud558\uace0 \ud568\uc815 \uac74\uc870\ub97c \uacc4\uc18d\ud560 \uc218 \uc788\ub3c4\ub85d \ub3c4\uc654\ub2e4.", "paragraph_sentence": "\uc81c1\ucc28 \uc138\uacc4 \ub300\uc804\uc774 \ub05d\ub098\uace0 \ub098\uc11c \ubd88\ud669\uc774 \uc654\uc744 \ub54c\ub3c4 \ub610 \ub2e4\uc2dc \uacbd\uc601 \uc704\uae30\uc5d0 \ube60\uc84c\ub2e4. \uc774\ub54c\ub294 1919\ub144\uc758 \ub178\ub3d9\uc7c1\uc758, 1922\ub144\uc5d0 \uc6cc\uc2f1\ud134 \uad70\ucd95 \uc870\uc57d, 1923\ub144\uc758 \uad00\ub3d9\ub300\uc9c0\uc9c4\uc774 \uacb9\ucce4\ub2e4. \ub610\ud55c 1927\ub144 \uae08\uc735 \uacf5\ud669\uc5d0 \uc758\ud574 \uc8fc\uac70\ub798 \uc740\ud589 15\uac1c \uc740\ud589\uc774 \uc784\uc2dc \ud734\uc5c5\ud558\uba74\uc11c \uc790\uae08 \uc0ac\uc815\uc774 \uae09\uaca9\ud788 \uc545\ud654\ud588\ub2e4. \uc815\ubd80\uc640 \uace0\ubca0 \uc2dc\uc758 \uc6d0\uc870\ub85c \uc9c1\uc6d0 \uc57d 3,500\uba85\uc758 \ud574\uace0 \ud734\uc5c5, \uc784\uc6d0 \ubcf4\uc218 \uc0ad\uac10, \ub9c8\uce20\uce74\ud0c0 \uac00\ubb38\uacfc \uac00\uc640\uc0ac\ud0a4 \uac00\ubb38\uc5d0\uc11c \uc0ac\uc7ac\ub97c \ucd9c\uc5f0\ud558\ub294 \ub4f1\uc744 \ud1b5\ud574 \uacbd\uc601 \uc7ac\uac74\uc744 \uc2dc\ub3c4\ud588\ub2e4. \ub610\ud55c \ud6a8\uace0 \uacf5\uc7a5\uc744 \ubd84\ub9ac\ud558\uc5ec \u3008\uac00\uc640\uc0ac\ud0a4 \ucc28\ub7c9\u3009\uc73c\ub85c \ub3c5\ub9bd\uc801 \uc800\ub2f9\uad8c\uc744 \uc124\uc815\ud558\uace0, \ubcf8\uc0ac \uacf5\uc7a5\uacfc \u847a\u5408\uacf5\uc7a5\uc744 \ub2f4\ubcf4\ub85c \ub300\ucd9c\uc744 \ubc1b\uc558\ub2e4. \uc77c\ubcf8 \uc81c\uad6d \ud574\uad70\ub3c4 \uce78\uc138\uc774 \ubcf8\ubd80 \uc5d0 \uc784\uc2dc\ud568\uc120 \uac74\uc870\ubd80\ub97c \uc124\uce58\ud558\uace0 \ud568\uc815 \uac74\uc870\ub97c \uacc4\uc18d\ud560 \uc218 \uc788\ub3c4\ub85d \ub3c4\uc654\ub2e4. ", "paragraph_answer": "\uc81c1\ucc28 \uc138\uacc4 \ub300\uc804\uc774 \ub05d\ub098\uace0 \ub098\uc11c \ubd88\ud669\uc774 \uc654\uc744 \ub54c\ub3c4 \ub610 \ub2e4\uc2dc \uacbd\uc601 \uc704\uae30\uc5d0 \ube60\uc84c\ub2e4. \uc774\ub54c\ub294 1919\ub144\uc758 \ub178\ub3d9\uc7c1\uc758, 1922\ub144\uc5d0 \uc6cc\uc2f1\ud134 \uad70\ucd95 \uc870\uc57d, 1923\ub144\uc758 \uad00\ub3d9\ub300\uc9c0\uc9c4\uc774 \uacb9\ucce4\ub2e4. \ub610\ud55c 1927\ub144 \uae08\uc735 \uacf5\ud669\uc5d0 \uc758\ud574 \uc8fc\uac70\ub798 \uc740\ud589 15\uac1c \uc740\ud589\uc774 \uc784\uc2dc \ud734\uc5c5\ud558\uba74\uc11c \uc790\uae08 \uc0ac\uc815\uc774 \uae09\uaca9\ud788 \uc545\ud654\ud588\ub2e4. \uc815\ubd80\uc640 \uace0\ubca0 \uc2dc\uc758 \uc6d0\uc870\ub85c \uc9c1\uc6d0 \uc57d 3,500\uba85\uc758 \ud574\uace0 \ud734\uc5c5, \uc784\uc6d0 \ubcf4\uc218 \uc0ad\uac10, \ub9c8\uce20\uce74\ud0c0 \uac00\ubb38\uacfc \uac00\uc640\uc0ac\ud0a4 \uac00\ubb38\uc5d0\uc11c \uc0ac\uc7ac\ub97c \ucd9c\uc5f0\ud558\ub294 \ub4f1\uc744 \ud1b5\ud574 \uacbd\uc601 \uc7ac\uac74\uc744 \uc2dc\ub3c4\ud588\ub2e4. \ub610\ud55c \ud6a8\uace0 \uacf5\uc7a5\uc744 \ubd84\ub9ac\ud558\uc5ec \u3008\uac00\uc640\uc0ac\ud0a4 \ucc28\ub7c9\u3009\uc73c\ub85c \ub3c5\ub9bd\uc801 \uc800\ub2f9\uad8c\uc744 \uc124\uc815\ud558\uace0, \ubcf8\uc0ac \uacf5\uc7a5\uacfc \u847a\u5408\uacf5\uc7a5\uc744 \ub2f4\ubcf4\ub85c \ub300\ucd9c\uc744 \ubc1b\uc558\ub2e4. \uc77c\ubcf8 \uc81c\uad6d \ud574\uad70\ub3c4 \uce78\uc138\uc774 \ubcf8\ubd80 \uc5d0 \uc784\uc2dc\ud568\uc120 \uac74\uc870\ubd80\ub97c \uc124\uce58\ud558\uace0 \ud568\uc815 \uac74\uc870\ub97c \uacc4\uc18d\ud560 \uc218 \uc788\ub3c4\ub85d \ub3c4\uc654\ub2e4.", "sentence_answer": " \uc77c\ubcf8 \uc81c\uad6d \ud574\uad70\ub3c4 \uce78\uc138\uc774 \ubcf8\ubd80 \uc5d0 \uc784\uc2dc\ud568\uc120 \uac74\uc870\ubd80\ub97c \uc124\uce58\ud558\uace0 \ud568\uc815 \uac74\uc870\ub97c \uacc4\uc18d\ud560 \uc218 \uc788\ub3c4\ub85d \ub3c4\uc654\ub2e4.", "paragraph_id": "6472155-2-1", "paragraph_question": "question: \uc784\uc2dc\ud568\uc120 \uac74\uc870\ubd80\ub97c \uc124\uce58\ud55c \uacf3\uc740 \uc5b4\ub514\uc778\uac00\uc694?, context: \uc81c1\ucc28 \uc138\uacc4 \ub300\uc804\uc774 \ub05d\ub098\uace0 \ub098\uc11c \ubd88\ud669\uc774 \uc654\uc744 \ub54c\ub3c4 \ub610 \ub2e4\uc2dc \uacbd\uc601 \uc704\uae30\uc5d0 \ube60\uc84c\ub2e4. \uc774\ub54c\ub294 1919\ub144\uc758 \ub178\ub3d9\uc7c1\uc758, 1922\ub144\uc5d0 \uc6cc\uc2f1\ud134 \uad70\ucd95 \uc870\uc57d, 1923\ub144\uc758 \uad00\ub3d9\ub300\uc9c0\uc9c4\uc774 \uacb9\ucce4\ub2e4. \ub610\ud55c 1927\ub144 \uae08\uc735 \uacf5\ud669\uc5d0 \uc758\ud574 \uc8fc\uac70\ub798 \uc740\ud589 15\uac1c \uc740\ud589\uc774 \uc784\uc2dc \ud734\uc5c5\ud558\uba74\uc11c \uc790\uae08 \uc0ac\uc815\uc774 \uae09\uaca9\ud788 \uc545\ud654\ud588\ub2e4. \uc815\ubd80\uc640 \uace0\ubca0 \uc2dc\uc758 \uc6d0\uc870\ub85c \uc9c1\uc6d0 \uc57d 3,500\uba85\uc758 \ud574\uace0 \ud734\uc5c5, \uc784\uc6d0 \ubcf4\uc218 \uc0ad\uac10, \ub9c8\uce20\uce74\ud0c0 \uac00\ubb38\uacfc \uac00\uc640\uc0ac\ud0a4 \uac00\ubb38\uc5d0\uc11c \uc0ac\uc7ac\ub97c \ucd9c\uc5f0\ud558\ub294 \ub4f1\uc744 \ud1b5\ud574 \uacbd\uc601 \uc7ac\uac74\uc744 \uc2dc\ub3c4\ud588\ub2e4. \ub610\ud55c \ud6a8\uace0 \uacf5\uc7a5\uc744 \ubd84\ub9ac\ud558\uc5ec \u3008\uac00\uc640\uc0ac\ud0a4 \ucc28\ub7c9\u3009\uc73c\ub85c \ub3c5\ub9bd\uc801 \uc800\ub2f9\uad8c\uc744 \uc124\uc815\ud558\uace0, \ubcf8\uc0ac \uacf5\uc7a5\uacfc \u847a\u5408\uacf5\uc7a5\uc744 \ub2f4\ubcf4\ub85c \ub300\ucd9c\uc744 \ubc1b\uc558\ub2e4. \uc77c\ubcf8 \uc81c\uad6d \ud574\uad70\ub3c4 \uce78\uc138\uc774 \ubcf8\ubd80\uc5d0 \uc784\uc2dc\ud568\uc120 \uac74\uc870\ubd80\ub97c \uc124\uce58\ud558\uace0 \ud568\uc815 \uac74\uc870\ub97c \uacc4\uc18d\ud560 \uc218 \uc788\ub3c4\ub85d \ub3c4\uc654\ub2e4."} {"question": "\ub370\uc774\ube44\ub4dc \ubca0\ucef4\uc758 \ud6c8\ub828 \ubd88\ucc38\uc744 \ud5c8\ub77d\ud55c \ub0a0 \ube45\ud1a0\ub9ac\uc544 \uc560\ub364\uc2a4\uac00 \ucc38\uc5ec\ud55c \ud589\uc0ac\ub294?", "paragraph": "2000\ub144\ub300 \ucd08, \ud37c\uac70\uc2a8\uacfc \ubca0\ucef4 \uc0ac\uc774\uc758 \uad00\uacc4\ub294 \ubca0\ucef4\uc758 \uba85\uc131\uacfc \ucd95\uad6c \uc678 \uc601\uc5ed\uc5d0\uc11c\uc758 \ud65c\ub3d9\uc73c\ub85c \uc545\ud654\ub418\uc5c8\ub2e4. 2000\ub144, \ubca0\ucef4\uc740 \uc7a5\uc5fc\uc744 \uc553\ub358 \uc544\ub4e4 \ube0c\ub8e8\ud074\ub9b0\uc744 \ub3cc\ubcf4\uae30 \uc704\ud574 \ud6c8\ub828\uc5d0\uc11c \ube60\uc9c8 \uc218 \uc788\uac8c \ud5c8\ub77d\uc744 \ubc1b\uc558\uc9c0\ub9cc, \ud37c\uac70\uc2a8\uc740 \uadf8\ub0a0 \uc800\ub141 \ube45\ud1a0\ub9ac\uc544 \uc560\ub364\uc2a4\uac00 \ub7f0\ub358 \ud328\uc158\uc758 \uc8fc \ud589\uc0ac\uc5d0\uc11c \ucd2c\uc601\ub41c \uac83\uc744 \ubcf4\uace0 \ub178\ud588\uace0, \ube45\ud1a0\ub9ac\uc544\uac00 \uadf8\ub0a0 \ube0c\ub8e8\ud074\ub9b0\uc744 \ub3cc\ubd24\ub2e4\uba74 \ubca0\ucef4\uc774 \ud6c8\ub828\uc5d0 \ucc38\uac00\ud560 \uc218 \uc788\uc5c8\uc744 \uac83\uc774\ub77c\uace0 \uc8fc\uc7a5\ud588\ub2e4. \ud37c\uac70\uc2a8 \uac10\ub3c5\uc740 \ubca0\ucef4\uc5d0 \ud5c8\uc6a9\ud560 \uc218 \uc788\ub294 \ucd5c\uace0\uc561\uc758 \ubc8c\uae08 (\ub2f9\uc2dc 2\uc8fc\uce58 \uc8fc\uae09\uc5d0 \ud574\ub2f9\ud558\ub294 \u00a350,000) \uc744 \ubd80\uacfc\ud558\uace0 \uc720\ub098\uc774\ud2f0\ub4dc\uc758 \uc219\uc801 \ub9ac\uc988 \uc720\ub098\uc774\ud2f0\ub4dc\uc640\uc758 \uc911\uc694\ud55c \uacbd\uae30\uc5d0 \uacb0\uc7a5\uc2dc\ucf1c \uc751\uc218\ud588\ub2e4. \ud37c\uac70\uc2a8 \uac10\ub3c5\uc740 \uc790\uc11c\uc804\uc5d0 \"\uadf8\uc758 \ub3d9\ub8cc\uc5d0 \uacf5\ud3c9\ud558\uc9c0 \ubabb\ud588\uc5c8\ub2e4\"\ub77c\uace0 \uc8fc\uc7a5\ud558\uba70 \ubca0\ucef4\uc744 \ube44\ub09c\ud588\ub2e4. \uc774\uc5d0\ub3c4 \ubd88\uad6c\ud558\uace0 \ubca0\ucef4\uc740 \uc720\ub098\uc774\ud2f0\ub4dc\uc758 \ud504\ub9ac\ubbf8\uc5b4\ub9ac\uadf8 \uc5ed\ub300 \ucd5c\ub2e4 \uc2b9\uc810\ucc28 \uc6b0\uc2b9\uc744 \uc774\ub8e9\ud574\ub0b4\uba70 \uc18c\uc18d \uad6c\ub2e8\uc5d0\uc11c \ud6cc\ub96d\ud55c \ud55c \ud574\ub97c \ubcf4\ub0c8\ub2e4.", "answer": "\ub7f0\ub358 \ud328\uc158\uc758 \uc8fc \ud589\uc0ac", "sentence": "2000\ub144, \ubca0\ucef4\uc740 \uc7a5\uc5fc\uc744 \uc553\ub358 \uc544\ub4e4 \ube0c\ub8e8\ud074\ub9b0\uc744 \ub3cc\ubcf4\uae30 \uc704\ud574 \ud6c8\ub828\uc5d0\uc11c \ube60\uc9c8 \uc218 \uc788\uac8c \ud5c8\ub77d\uc744 \ubc1b\uc558\uc9c0\ub9cc, \ud37c\uac70\uc2a8\uc740 \uadf8\ub0a0 \uc800\ub141 \ube45\ud1a0\ub9ac\uc544 \uc560\ub364\uc2a4\uac00 \ub7f0\ub358 \ud328\uc158\uc758 \uc8fc \ud589\uc0ac \uc5d0\uc11c \ucd2c\uc601\ub41c \uac83\uc744 \ubcf4\uace0 \ub178\ud588\uace0, \ube45\ud1a0\ub9ac\uc544\uac00 \uadf8\ub0a0 \ube0c\ub8e8\ud074\ub9b0\uc744 \ub3cc\ubd24\ub2e4\uba74 \ubca0\ucef4\uc774 \ud6c8\ub828\uc5d0 \ucc38\uac00\ud560 \uc218 \uc788\uc5c8\uc744 \uac83\uc774\ub77c\uace0 \uc8fc\uc7a5\ud588\ub2e4.", "paragraph_sentence": "2000\ub144\ub300 \ucd08, \ud37c\uac70\uc2a8\uacfc \ubca0\ucef4 \uc0ac\uc774\uc758 \uad00\uacc4\ub294 \ubca0\ucef4\uc758 \uba85\uc131\uacfc \ucd95\uad6c \uc678 \uc601\uc5ed\uc5d0\uc11c\uc758 \ud65c\ub3d9\uc73c\ub85c \uc545\ud654\ub418\uc5c8\ub2e4. 2000\ub144, \ubca0\ucef4\uc740 \uc7a5\uc5fc\uc744 \uc553\ub358 \uc544\ub4e4 \ube0c\ub8e8\ud074\ub9b0\uc744 \ub3cc\ubcf4\uae30 \uc704\ud574 \ud6c8\ub828\uc5d0\uc11c \ube60\uc9c8 \uc218 \uc788\uac8c \ud5c8\ub77d\uc744 \ubc1b\uc558\uc9c0\ub9cc, \ud37c\uac70\uc2a8\uc740 \uadf8\ub0a0 \uc800\ub141 \ube45\ud1a0\ub9ac\uc544 \uc560\ub364\uc2a4\uac00 \ub7f0\ub358 \ud328\uc158\uc758 \uc8fc \ud589\uc0ac \uc5d0\uc11c \ucd2c\uc601\ub41c \uac83\uc744 \ubcf4\uace0 \ub178\ud588\uace0, \ube45\ud1a0\ub9ac\uc544\uac00 \uadf8\ub0a0 \ube0c\ub8e8\ud074\ub9b0\uc744 \ub3cc\ubd24\ub2e4\uba74 \ubca0\ucef4\uc774 \ud6c8\ub828\uc5d0 \ucc38\uac00\ud560 \uc218 \uc788\uc5c8\uc744 \uac83\uc774\ub77c\uace0 \uc8fc\uc7a5\ud588\ub2e4. \ud37c\uac70\uc2a8 \uac10\ub3c5\uc740 \ubca0\ucef4\uc5d0 \ud5c8\uc6a9\ud560 \uc218 \uc788\ub294 \ucd5c\uace0\uc561\uc758 \ubc8c\uae08 (\ub2f9\uc2dc 2\uc8fc\uce58 \uc8fc\uae09\uc5d0 \ud574\ub2f9\ud558\ub294 \u00a350,000) \uc744 \ubd80\uacfc\ud558\uace0 \uc720\ub098\uc774\ud2f0\ub4dc\uc758 \uc219\uc801 \ub9ac\uc988 \uc720\ub098\uc774\ud2f0\ub4dc\uc640\uc758 \uc911\uc694\ud55c \uacbd\uae30\uc5d0 \uacb0\uc7a5\uc2dc\ucf1c \uc751\uc218\ud588\ub2e4. \ud37c\uac70\uc2a8 \uac10\ub3c5\uc740 \uc790\uc11c\uc804\uc5d0 \"\uadf8\uc758 \ub3d9\ub8cc\uc5d0 \uacf5\ud3c9\ud558\uc9c0 \ubabb\ud588\uc5c8\ub2e4\"\ub77c\uace0 \uc8fc\uc7a5\ud558\uba70 \ubca0\ucef4\uc744 \ube44\ub09c\ud588\ub2e4. \uc774\uc5d0\ub3c4 \ubd88\uad6c\ud558\uace0 \ubca0\ucef4\uc740 \uc720\ub098\uc774\ud2f0\ub4dc\uc758 \ud504\ub9ac\ubbf8\uc5b4\ub9ac\uadf8 \uc5ed\ub300 \ucd5c\ub2e4 \uc2b9\uc810\ucc28 \uc6b0\uc2b9\uc744 \uc774\ub8e9\ud574\ub0b4\uba70 \uc18c\uc18d \uad6c\ub2e8\uc5d0\uc11c \ud6cc\ub96d\ud55c \ud55c \ud574\ub97c \ubcf4\ub0c8\ub2e4.", "paragraph_answer": "2000\ub144\ub300 \ucd08, \ud37c\uac70\uc2a8\uacfc \ubca0\ucef4 \uc0ac\uc774\uc758 \uad00\uacc4\ub294 \ubca0\ucef4\uc758 \uba85\uc131\uacfc \ucd95\uad6c \uc678 \uc601\uc5ed\uc5d0\uc11c\uc758 \ud65c\ub3d9\uc73c\ub85c \uc545\ud654\ub418\uc5c8\ub2e4. 2000\ub144, \ubca0\ucef4\uc740 \uc7a5\uc5fc\uc744 \uc553\ub358 \uc544\ub4e4 \ube0c\ub8e8\ud074\ub9b0\uc744 \ub3cc\ubcf4\uae30 \uc704\ud574 \ud6c8\ub828\uc5d0\uc11c \ube60\uc9c8 \uc218 \uc788\uac8c \ud5c8\ub77d\uc744 \ubc1b\uc558\uc9c0\ub9cc, \ud37c\uac70\uc2a8\uc740 \uadf8\ub0a0 \uc800\ub141 \ube45\ud1a0\ub9ac\uc544 \uc560\ub364\uc2a4\uac00 \ub7f0\ub358 \ud328\uc158\uc758 \uc8fc \ud589\uc0ac \uc5d0\uc11c \ucd2c\uc601\ub41c \uac83\uc744 \ubcf4\uace0 \ub178\ud588\uace0, \ube45\ud1a0\ub9ac\uc544\uac00 \uadf8\ub0a0 \ube0c\ub8e8\ud074\ub9b0\uc744 \ub3cc\ubd24\ub2e4\uba74 \ubca0\ucef4\uc774 \ud6c8\ub828\uc5d0 \ucc38\uac00\ud560 \uc218 \uc788\uc5c8\uc744 \uac83\uc774\ub77c\uace0 \uc8fc\uc7a5\ud588\ub2e4. \ud37c\uac70\uc2a8 \uac10\ub3c5\uc740 \ubca0\ucef4\uc5d0 \ud5c8\uc6a9\ud560 \uc218 \uc788\ub294 \ucd5c\uace0\uc561\uc758 \ubc8c\uae08 (\ub2f9\uc2dc 2\uc8fc\uce58 \uc8fc\uae09\uc5d0 \ud574\ub2f9\ud558\ub294 \u00a350,000) \uc744 \ubd80\uacfc\ud558\uace0 \uc720\ub098\uc774\ud2f0\ub4dc\uc758 \uc219\uc801 \ub9ac\uc988 \uc720\ub098\uc774\ud2f0\ub4dc\uc640\uc758 \uc911\uc694\ud55c \uacbd\uae30\uc5d0 \uacb0\uc7a5\uc2dc\ucf1c \uc751\uc218\ud588\ub2e4. \ud37c\uac70\uc2a8 \uac10\ub3c5\uc740 \uc790\uc11c\uc804\uc5d0 \"\uadf8\uc758 \ub3d9\ub8cc\uc5d0 \uacf5\ud3c9\ud558\uc9c0 \ubabb\ud588\uc5c8\ub2e4\"\ub77c\uace0 \uc8fc\uc7a5\ud558\uba70 \ubca0\ucef4\uc744 \ube44\ub09c\ud588\ub2e4. \uc774\uc5d0\ub3c4 \ubd88\uad6c\ud558\uace0 \ubca0\ucef4\uc740 \uc720\ub098\uc774\ud2f0\ub4dc\uc758 \ud504\ub9ac\ubbf8\uc5b4\ub9ac\uadf8 \uc5ed\ub300 \ucd5c\ub2e4 \uc2b9\uc810\ucc28 \uc6b0\uc2b9\uc744 \uc774\ub8e9\ud574\ub0b4\uba70 \uc18c\uc18d \uad6c\ub2e8\uc5d0\uc11c \ud6cc\ub96d\ud55c \ud55c \ud574\ub97c \ubcf4\ub0c8\ub2e4.", "sentence_answer": "2000\ub144, \ubca0\ucef4\uc740 \uc7a5\uc5fc\uc744 \uc553\ub358 \uc544\ub4e4 \ube0c\ub8e8\ud074\ub9b0\uc744 \ub3cc\ubcf4\uae30 \uc704\ud574 \ud6c8\ub828\uc5d0\uc11c \ube60\uc9c8 \uc218 \uc788\uac8c \ud5c8\ub77d\uc744 \ubc1b\uc558\uc9c0\ub9cc, \ud37c\uac70\uc2a8\uc740 \uadf8\ub0a0 \uc800\ub141 \ube45\ud1a0\ub9ac\uc544 \uc560\ub364\uc2a4\uac00 \ub7f0\ub358 \ud328\uc158\uc758 \uc8fc \ud589\uc0ac \uc5d0\uc11c \ucd2c\uc601\ub41c \uac83\uc744 \ubcf4\uace0 \ub178\ud588\uace0, \ube45\ud1a0\ub9ac\uc544\uac00 \uadf8\ub0a0 \ube0c\ub8e8\ud074\ub9b0\uc744 \ub3cc\ubd24\ub2e4\uba74 \ubca0\ucef4\uc774 \ud6c8\ub828\uc5d0 \ucc38\uac00\ud560 \uc218 \uc788\uc5c8\uc744 \uac83\uc774\ub77c\uace0 \uc8fc\uc7a5\ud588\ub2e4.", "paragraph_id": "6528559-9-0", "paragraph_question": "question: \ub370\uc774\ube44\ub4dc \ubca0\ucef4\uc758 \ud6c8\ub828 \ubd88\ucc38\uc744 \ud5c8\ub77d\ud55c \ub0a0 \ube45\ud1a0\ub9ac\uc544 \uc560\ub364\uc2a4\uac00 \ucc38\uc5ec\ud55c \ud589\uc0ac\ub294?, context: 2000\ub144\ub300 \ucd08, \ud37c\uac70\uc2a8\uacfc \ubca0\ucef4 \uc0ac\uc774\uc758 \uad00\uacc4\ub294 \ubca0\ucef4\uc758 \uba85\uc131\uacfc \ucd95\uad6c \uc678 \uc601\uc5ed\uc5d0\uc11c\uc758 \ud65c\ub3d9\uc73c\ub85c \uc545\ud654\ub418\uc5c8\ub2e4. 2000\ub144, \ubca0\ucef4\uc740 \uc7a5\uc5fc\uc744 \uc553\ub358 \uc544\ub4e4 \ube0c\ub8e8\ud074\ub9b0\uc744 \ub3cc\ubcf4\uae30 \uc704\ud574 \ud6c8\ub828\uc5d0\uc11c \ube60\uc9c8 \uc218 \uc788\uac8c \ud5c8\ub77d\uc744 \ubc1b\uc558\uc9c0\ub9cc, \ud37c\uac70\uc2a8\uc740 \uadf8\ub0a0 \uc800\ub141 \ube45\ud1a0\ub9ac\uc544 \uc560\ub364\uc2a4\uac00 \ub7f0\ub358 \ud328\uc158\uc758 \uc8fc \ud589\uc0ac\uc5d0\uc11c \ucd2c\uc601\ub41c \uac83\uc744 \ubcf4\uace0 \ub178\ud588\uace0, \ube45\ud1a0\ub9ac\uc544\uac00 \uadf8\ub0a0 \ube0c\ub8e8\ud074\ub9b0\uc744 \ub3cc\ubd24\ub2e4\uba74 \ubca0\ucef4\uc774 \ud6c8\ub828\uc5d0 \ucc38\uac00\ud560 \uc218 \uc788\uc5c8\uc744 \uac83\uc774\ub77c\uace0 \uc8fc\uc7a5\ud588\ub2e4. \ud37c\uac70\uc2a8 \uac10\ub3c5\uc740 \ubca0\ucef4\uc5d0 \ud5c8\uc6a9\ud560 \uc218 \uc788\ub294 \ucd5c\uace0\uc561\uc758 \ubc8c\uae08 (\ub2f9\uc2dc 2\uc8fc\uce58 \uc8fc\uae09\uc5d0 \ud574\ub2f9\ud558\ub294 \u00a350,000) \uc744 \ubd80\uacfc\ud558\uace0 \uc720\ub098\uc774\ud2f0\ub4dc\uc758 \uc219\uc801 \ub9ac\uc988 \uc720\ub098\uc774\ud2f0\ub4dc\uc640\uc758 \uc911\uc694\ud55c \uacbd\uae30\uc5d0 \uacb0\uc7a5\uc2dc\ucf1c \uc751\uc218\ud588\ub2e4. \ud37c\uac70\uc2a8 \uac10\ub3c5\uc740 \uc790\uc11c\uc804\uc5d0 \"\uadf8\uc758 \ub3d9\ub8cc\uc5d0 \uacf5\ud3c9\ud558\uc9c0 \ubabb\ud588\uc5c8\ub2e4\"\ub77c\uace0 \uc8fc\uc7a5\ud558\uba70 \ubca0\ucef4\uc744 \ube44\ub09c\ud588\ub2e4. \uc774\uc5d0\ub3c4 \ubd88\uad6c\ud558\uace0 \ubca0\ucef4\uc740 \uc720\ub098\uc774\ud2f0\ub4dc\uc758 \ud504\ub9ac\ubbf8\uc5b4\ub9ac\uadf8 \uc5ed\ub300 \ucd5c\ub2e4 \uc2b9\uc810\ucc28 \uc6b0\uc2b9\uc744 \uc774\ub8e9\ud574\ub0b4\uba70 \uc18c\uc18d \uad6c\ub2e8\uc5d0\uc11c \ud6cc\ub96d\ud55c \ud55c \ud574\ub97c \ubcf4\ub0c8\ub2e4."} {"question": "2016\ub144 2\uc6d4 5\uc77c \uc138\uc6d4\ud638 \uc9c4\uc0c1 \uaddc\uba85\uacfc \uc5ed\uc0ac\uad50\uacfc\uc11c \uc815\uc0c1\ud654 \ub4f1\uc744 \uc704\ud55c \uc11c\uba85\uc6b4\ub3d9\uc774 \uc5f4\ub9b0 \uc7a5\uc18c\ub294?", "paragraph": "2016\ub144 2\uc6d4 5\uc77c \uc624\uc804 11\uc2dc \ubb34\ub835 \uc11c\uc6b8\uc5ed \uc785\uad6c\uc5d0\uc11c \uc138\uc6d4\ud638 \uc9c4\uc0c1 \uaddc\uba85\uacfc \uc5ed\uc0ac\uad50\uacfc\uc11c \uc815\uc0c1\ud654 \ub4f1\uc744 \uc704\ud55c \uc11c\uba85 \uc6b4\ub3d9\uc744 \uc9c4\ud589\ud558\ub294 \uc2dc\ubbfc\uc0ac\ud68c\ub2e8\uccb4\uc758 \ud14c\uc774\ube14 \uc55e\uc73c\ub85c \uc5b4\ubc84\uc774\ub2e8\uccb4\ub97c \ud3ec\ud568\ud55c \ubcf4\uc218 \ub2e8\uccb4 \ud68c\uc6d0 30~40\uc5ec \uba85\uc774 \uc695\uc124\uacfc \uace0\ud568\uc744 \uce58\uba70 \ub098\ud0c0\ub0ac\uace0 \uadf8\ub4e4\uc744 \uc138\uc6d4\ud638 \uc720\uac00\uc871\uc774 \uc788\ub294 \uc11c\uba85\ub300\uc5d0\ub3c4 \"\ubd88\uc21c\ubd84\uc790\ub4e4\"\uc774\ub77c\ub294 \uc2ec\ud55c \ub9c9\ub9d0\uc744 \uc3df\uc558\ub2e4. \uac8c\ub2e4\uac00 \ud68c\uc6d0 \uc911 \ud55c \uba85\uc740 \uc2dc\ubbfc\ub2e8\uccb4\uc758 \uc190\ud33b\ub9d0\uc744 \ubc1c\ub85c \ubc1f\uace0 \ub09c\ub3d9\uc744 \ubc8c\uc778 \uc77c\ubd80 \ubcf4\uc218\ub2e8\uccb4 \ud68c\uc6d0\ub4e4\uc758 \uc190\uc5d0\ub294 '\uacbd\uc81c\ud65c\uc131\ud654 \uc704\ud55c \ucc9c\ub9cc \uc11c\uba85 \uc6b4\ub3d9'\ub4f1\uc758 \uc190\ud33b\ub9d0\uc774 \uc788\uc5c8\uace0 11\uc2dc 10\ubd84 \uc2dc\ubbfc\uc0ac\ud68c\ub2e8\uccb4\uc758 \ud569\ub3d9 \uae30\uc790\ud68c\uacac \ud604\uc7a5\uc744 \ubc29\ud574\ud558\uc790 \uacbd\ucc30\uc774 \ucd9c\ub3d9\ud588\uc73c\ub098 \uc18c\ub780\uc744 \uba48\ucd94\uc9c0 \uc54a\uace0 \ubab8\uc2f8\uc6c0\uae4c\uc9c0 \ubc8c\uc774\uc790 \uc2dc\ubbfc\uc0ac\ud68c\ub2e8\uccb4 \uad00\uacc4\uc790\uac00 \uc5b4\ubc84\uc774\uc5f0\ud569 \ud68c\uc6d0\uc744 \ub9c9\uc544\uc11c\uc790 \uc774\uc5d0 \ubd84\ub178\ud55c \uc5b4\ubc84\uc774\uc5f0\ud569 \ud68c\uc6d0\uc774 \uad00\uacc4\uc790\uc758 \uc5bc\uad74\uc744 \uc8fc\uba39\uc73c\ub85c \uc138 \ucc28\ub840 \uac00\uaca9\ud588\uace0 \ud2b9\ubcc4\ud55c \ubc29\uc5b4\ub3c4 \uc5c6\uc774 \ud3ed\ub825\uc744 \ub2f9\ud55c \uc774 \uad00\uacc4\uc790\ub294 \ucf54\uc640 \uc785\uc774 \ud130\uc9c0\ub294 \ub4f1 \ubd80\uc0c1\uc744 \ub2f9\ud588\uc73c\uba70 \ud604\uc7ac \uacbd\ucc30\uc758 \uc870\uc0ac \uc911\uc5d0 \uc788\ub2e4.", "answer": "\uc11c\uc6b8\uc5ed \uc785\uad6c", "sentence": "2016\ub144 2\uc6d4 5\uc77c \uc624\uc804 11\uc2dc \ubb34\ub835 \uc11c\uc6b8\uc5ed \uc785\uad6c \uc5d0\uc11c \uc138\uc6d4\ud638 \uc9c4\uc0c1 \uaddc\uba85\uacfc \uc5ed\uc0ac\uad50\uacfc\uc11c \uc815\uc0c1\ud654 \ub4f1\uc744 \uc704\ud55c \uc11c\uba85 \uc6b4\ub3d9\uc744 \uc9c4\ud589\ud558\ub294 \uc2dc\ubbfc\uc0ac\ud68c\ub2e8\uccb4\uc758 \ud14c\uc774\ube14 \uc55e\uc73c\ub85c \uc5b4\ubc84\uc774\ub2e8\uccb4\ub97c \ud3ec\ud568\ud55c \ubcf4\uc218 \ub2e8\uccb4 \ud68c\uc6d0 30~40\uc5ec \uba85\uc774 \uc695\uc124\uacfc \uace0\ud568\uc744 \uce58\uba70 \ub098\ud0c0\ub0ac\uace0 \uadf8\ub4e4\uc744 \uc138\uc6d4\ud638 \uc720\uac00\uc871\uc774 \uc788\ub294 \uc11c\uba85\ub300\uc5d0\ub3c4 \"\ubd88\uc21c\ubd84\uc790\ub4e4\"\uc774\ub77c\ub294 \uc2ec\ud55c \ub9c9\ub9d0\uc744 \uc3df\uc558\ub2e4.", "paragraph_sentence": " 2016\ub144 2\uc6d4 5\uc77c \uc624\uc804 11\uc2dc \ubb34\ub835 \uc11c\uc6b8\uc5ed \uc785\uad6c \uc5d0\uc11c \uc138\uc6d4\ud638 \uc9c4\uc0c1 \uaddc\uba85\uacfc \uc5ed\uc0ac\uad50\uacfc\uc11c \uc815\uc0c1\ud654 \ub4f1\uc744 \uc704\ud55c \uc11c\uba85 \uc6b4\ub3d9\uc744 \uc9c4\ud589\ud558\ub294 \uc2dc\ubbfc\uc0ac\ud68c\ub2e8\uccb4\uc758 \ud14c\uc774\ube14 \uc55e\uc73c\ub85c \uc5b4\ubc84\uc774\ub2e8\uccb4\ub97c \ud3ec\ud568\ud55c \ubcf4\uc218 \ub2e8\uccb4 \ud68c\uc6d0 30~40\uc5ec \uba85\uc774 \uc695\uc124\uacfc \uace0\ud568\uc744 \uce58\uba70 \ub098\ud0c0\ub0ac\uace0 \uadf8\ub4e4\uc744 \uc138\uc6d4\ud638 \uc720\uac00\uc871\uc774 \uc788\ub294 \uc11c\uba85\ub300\uc5d0\ub3c4 \"\ubd88\uc21c\ubd84\uc790\ub4e4\"\uc774\ub77c\ub294 \uc2ec\ud55c \ub9c9\ub9d0\uc744 \uc3df\uc558\ub2e4. \uac8c\ub2e4\uac00 \ud68c\uc6d0 \uc911 \ud55c \uba85\uc740 \uc2dc\ubbfc\ub2e8\uccb4\uc758 \uc190\ud33b\ub9d0\uc744 \ubc1c\ub85c \ubc1f\uace0 \ub09c\ub3d9\uc744 \ubc8c\uc778 \uc77c\ubd80 \ubcf4\uc218\ub2e8\uccb4 \ud68c\uc6d0\ub4e4\uc758 \uc190\uc5d0\ub294 '\uacbd\uc81c\ud65c\uc131\ud654 \uc704\ud55c \ucc9c\ub9cc \uc11c\uba85 \uc6b4\ub3d9'\ub4f1\uc758 \uc190\ud33b\ub9d0\uc774 \uc788\uc5c8\uace0 11\uc2dc 10\ubd84 \uc2dc\ubbfc\uc0ac\ud68c\ub2e8\uccb4\uc758 \ud569\ub3d9 \uae30\uc790\ud68c\uacac \ud604\uc7a5\uc744 \ubc29\ud574\ud558\uc790 \uacbd\ucc30\uc774 \ucd9c\ub3d9\ud588\uc73c\ub098 \uc18c\ub780\uc744 \uba48\ucd94\uc9c0 \uc54a\uace0 \ubab8\uc2f8\uc6c0\uae4c\uc9c0 \ubc8c\uc774\uc790 \uc2dc\ubbfc\uc0ac\ud68c\ub2e8\uccb4 \uad00\uacc4\uc790\uac00 \uc5b4\ubc84\uc774\uc5f0\ud569 \ud68c\uc6d0\uc744 \ub9c9\uc544\uc11c\uc790 \uc774\uc5d0 \ubd84\ub178\ud55c \uc5b4\ubc84\uc774\uc5f0\ud569 \ud68c\uc6d0\uc774 \uad00\uacc4\uc790\uc758 \uc5bc\uad74\uc744 \uc8fc\uba39\uc73c\ub85c \uc138 \ucc28\ub840 \uac00\uaca9\ud588\uace0 \ud2b9\ubcc4\ud55c \ubc29\uc5b4\ub3c4 \uc5c6\uc774 \ud3ed\ub825\uc744 \ub2f9\ud55c \uc774 \uad00\uacc4\uc790\ub294 \ucf54\uc640 \uc785\uc774 \ud130\uc9c0\ub294 \ub4f1 \ubd80\uc0c1\uc744 \ub2f9\ud588\uc73c\uba70 \ud604\uc7ac \uacbd\ucc30\uc758 \uc870\uc0ac \uc911\uc5d0 \uc788\ub2e4.", "paragraph_answer": "2016\ub144 2\uc6d4 5\uc77c \uc624\uc804 11\uc2dc \ubb34\ub835 \uc11c\uc6b8\uc5ed \uc785\uad6c \uc5d0\uc11c \uc138\uc6d4\ud638 \uc9c4\uc0c1 \uaddc\uba85\uacfc \uc5ed\uc0ac\uad50\uacfc\uc11c \uc815\uc0c1\ud654 \ub4f1\uc744 \uc704\ud55c \uc11c\uba85 \uc6b4\ub3d9\uc744 \uc9c4\ud589\ud558\ub294 \uc2dc\ubbfc\uc0ac\ud68c\ub2e8\uccb4\uc758 \ud14c\uc774\ube14 \uc55e\uc73c\ub85c \uc5b4\ubc84\uc774\ub2e8\uccb4\ub97c \ud3ec\ud568\ud55c \ubcf4\uc218 \ub2e8\uccb4 \ud68c\uc6d0 30~40\uc5ec \uba85\uc774 \uc695\uc124\uacfc \uace0\ud568\uc744 \uce58\uba70 \ub098\ud0c0\ub0ac\uace0 \uadf8\ub4e4\uc744 \uc138\uc6d4\ud638 \uc720\uac00\uc871\uc774 \uc788\ub294 \uc11c\uba85\ub300\uc5d0\ub3c4 \"\ubd88\uc21c\ubd84\uc790\ub4e4\"\uc774\ub77c\ub294 \uc2ec\ud55c \ub9c9\ub9d0\uc744 \uc3df\uc558\ub2e4. \uac8c\ub2e4\uac00 \ud68c\uc6d0 \uc911 \ud55c \uba85\uc740 \uc2dc\ubbfc\ub2e8\uccb4\uc758 \uc190\ud33b\ub9d0\uc744 \ubc1c\ub85c \ubc1f\uace0 \ub09c\ub3d9\uc744 \ubc8c\uc778 \uc77c\ubd80 \ubcf4\uc218\ub2e8\uccb4 \ud68c\uc6d0\ub4e4\uc758 \uc190\uc5d0\ub294 '\uacbd\uc81c\ud65c\uc131\ud654 \uc704\ud55c \ucc9c\ub9cc \uc11c\uba85 \uc6b4\ub3d9'\ub4f1\uc758 \uc190\ud33b\ub9d0\uc774 \uc788\uc5c8\uace0 11\uc2dc 10\ubd84 \uc2dc\ubbfc\uc0ac\ud68c\ub2e8\uccb4\uc758 \ud569\ub3d9 \uae30\uc790\ud68c\uacac \ud604\uc7a5\uc744 \ubc29\ud574\ud558\uc790 \uacbd\ucc30\uc774 \ucd9c\ub3d9\ud588\uc73c\ub098 \uc18c\ub780\uc744 \uba48\ucd94\uc9c0 \uc54a\uace0 \ubab8\uc2f8\uc6c0\uae4c\uc9c0 \ubc8c\uc774\uc790 \uc2dc\ubbfc\uc0ac\ud68c\ub2e8\uccb4 \uad00\uacc4\uc790\uac00 \uc5b4\ubc84\uc774\uc5f0\ud569 \ud68c\uc6d0\uc744 \ub9c9\uc544\uc11c\uc790 \uc774\uc5d0 \ubd84\ub178\ud55c \uc5b4\ubc84\uc774\uc5f0\ud569 \ud68c\uc6d0\uc774 \uad00\uacc4\uc790\uc758 \uc5bc\uad74\uc744 \uc8fc\uba39\uc73c\ub85c \uc138 \ucc28\ub840 \uac00\uaca9\ud588\uace0 \ud2b9\ubcc4\ud55c \ubc29\uc5b4\ub3c4 \uc5c6\uc774 \ud3ed\ub825\uc744 \ub2f9\ud55c \uc774 \uad00\uacc4\uc790\ub294 \ucf54\uc640 \uc785\uc774 \ud130\uc9c0\ub294 \ub4f1 \ubd80\uc0c1\uc744 \ub2f9\ud588\uc73c\uba70 \ud604\uc7ac \uacbd\ucc30\uc758 \uc870\uc0ac \uc911\uc5d0 \uc788\ub2e4.", "sentence_answer": "2016\ub144 2\uc6d4 5\uc77c \uc624\uc804 11\uc2dc \ubb34\ub835 \uc11c\uc6b8\uc5ed \uc785\uad6c \uc5d0\uc11c \uc138\uc6d4\ud638 \uc9c4\uc0c1 \uaddc\uba85\uacfc \uc5ed\uc0ac\uad50\uacfc\uc11c \uc815\uc0c1\ud654 \ub4f1\uc744 \uc704\ud55c \uc11c\uba85 \uc6b4\ub3d9\uc744 \uc9c4\ud589\ud558\ub294 \uc2dc\ubbfc\uc0ac\ud68c\ub2e8\uccb4\uc758 \ud14c\uc774\ube14 \uc55e\uc73c\ub85c \uc5b4\ubc84\uc774\ub2e8\uccb4\ub97c \ud3ec\ud568\ud55c \ubcf4\uc218 \ub2e8\uccb4 \ud68c\uc6d0 30~40\uc5ec \uba85\uc774 \uc695\uc124\uacfc \uace0\ud568\uc744 \uce58\uba70 \ub098\ud0c0\ub0ac\uace0 \uadf8\ub4e4\uc744 \uc138\uc6d4\ud638 \uc720\uac00\uc871\uc774 \uc788\ub294 \uc11c\uba85\ub300\uc5d0\ub3c4 \"\ubd88\uc21c\ubd84\uc790\ub4e4\"\uc774\ub77c\ub294 \uc2ec\ud55c \ub9c9\ub9d0\uc744 \uc3df\uc558\ub2e4.", "paragraph_id": "6577727-0-2", "paragraph_question": "question: 2016\ub144 2\uc6d4 5\uc77c \uc138\uc6d4\ud638 \uc9c4\uc0c1 \uaddc\uba85\uacfc \uc5ed\uc0ac\uad50\uacfc\uc11c \uc815\uc0c1\ud654 \ub4f1\uc744 \uc704\ud55c \uc11c\uba85\uc6b4\ub3d9\uc774 \uc5f4\ub9b0 \uc7a5\uc18c\ub294?, context: 2016\ub144 2\uc6d4 5\uc77c \uc624\uc804 11\uc2dc \ubb34\ub835 \uc11c\uc6b8\uc5ed \uc785\uad6c\uc5d0\uc11c \uc138\uc6d4\ud638 \uc9c4\uc0c1 \uaddc\uba85\uacfc \uc5ed\uc0ac\uad50\uacfc\uc11c \uc815\uc0c1\ud654 \ub4f1\uc744 \uc704\ud55c \uc11c\uba85 \uc6b4\ub3d9\uc744 \uc9c4\ud589\ud558\ub294 \uc2dc\ubbfc\uc0ac\ud68c\ub2e8\uccb4\uc758 \ud14c\uc774\ube14 \uc55e\uc73c\ub85c \uc5b4\ubc84\uc774\ub2e8\uccb4\ub97c \ud3ec\ud568\ud55c \ubcf4\uc218 \ub2e8\uccb4 \ud68c\uc6d0 30~40\uc5ec \uba85\uc774 \uc695\uc124\uacfc \uace0\ud568\uc744 \uce58\uba70 \ub098\ud0c0\ub0ac\uace0 \uadf8\ub4e4\uc744 \uc138\uc6d4\ud638 \uc720\uac00\uc871\uc774 \uc788\ub294 \uc11c\uba85\ub300\uc5d0\ub3c4 \"\ubd88\uc21c\ubd84\uc790\ub4e4\"\uc774\ub77c\ub294 \uc2ec\ud55c \ub9c9\ub9d0\uc744 \uc3df\uc558\ub2e4. \uac8c\ub2e4\uac00 \ud68c\uc6d0 \uc911 \ud55c \uba85\uc740 \uc2dc\ubbfc\ub2e8\uccb4\uc758 \uc190\ud33b\ub9d0\uc744 \ubc1c\ub85c \ubc1f\uace0 \ub09c\ub3d9\uc744 \ubc8c\uc778 \uc77c\ubd80 \ubcf4\uc218\ub2e8\uccb4 \ud68c\uc6d0\ub4e4\uc758 \uc190\uc5d0\ub294 '\uacbd\uc81c\ud65c\uc131\ud654 \uc704\ud55c \ucc9c\ub9cc \uc11c\uba85 \uc6b4\ub3d9'\ub4f1\uc758 \uc190\ud33b\ub9d0\uc774 \uc788\uc5c8\uace0 11\uc2dc 10\ubd84 \uc2dc\ubbfc\uc0ac\ud68c\ub2e8\uccb4\uc758 \ud569\ub3d9 \uae30\uc790\ud68c\uacac \ud604\uc7a5\uc744 \ubc29\ud574\ud558\uc790 \uacbd\ucc30\uc774 \ucd9c\ub3d9\ud588\uc73c\ub098 \uc18c\ub780\uc744 \uba48\ucd94\uc9c0 \uc54a\uace0 \ubab8\uc2f8\uc6c0\uae4c\uc9c0 \ubc8c\uc774\uc790 \uc2dc\ubbfc\uc0ac\ud68c\ub2e8\uccb4 \uad00\uacc4\uc790\uac00 \uc5b4\ubc84\uc774\uc5f0\ud569 \ud68c\uc6d0\uc744 \ub9c9\uc544\uc11c\uc790 \uc774\uc5d0 \ubd84\ub178\ud55c \uc5b4\ubc84\uc774\uc5f0\ud569 \ud68c\uc6d0\uc774 \uad00\uacc4\uc790\uc758 \uc5bc\uad74\uc744 \uc8fc\uba39\uc73c\ub85c \uc138 \ucc28\ub840 \uac00\uaca9\ud588\uace0 \ud2b9\ubcc4\ud55c \ubc29\uc5b4\ub3c4 \uc5c6\uc774 \ud3ed\ub825\uc744 \ub2f9\ud55c \uc774 \uad00\uacc4\uc790\ub294 \ucf54\uc640 \uc785\uc774 \ud130\uc9c0\ub294 \ub4f1 \ubd80\uc0c1\uc744 \ub2f9\ud588\uc73c\uba70 \ud604\uc7ac \uacbd\ucc30\uc758 \uc870\uc0ac \uc911\uc5d0 \uc788\ub2e4."} {"question": "Single Ladies\ub97c \uc790\uc2e0\ub9cc\uc758 \ubc84\uc804\uc73c\ub85c \ub9cc\ub4e0 \uc624\ubc14\ub9c8\uc758 \ub2ee\uc740 \uaf34\uc740?", "paragraph": "\uac00\uc218 \uc874 \ub808\uc804\ub4dc\uac00 \ub9cc\ub4e0 \ube44\ub514\uc624\uc5d0\uc11c \ubbf8\uad6d\uc758 \ub300\ud1b5\ub839 \ubc84\ub77d \uc624\ubc14\ub9c8\uac00 \uc640\uc774\ud504 \ubbf8\uc258 \uc624\ubc14\ub9c8\uc640 \ud568\uaed8 \ucd9c\uc5f0\ud574 \u3008Single Ladies\u3009\uc758 \uc548\ubb34\ub97c \ucdc4\ub2e4. \ub610\ud55c \uc624\ubc14\ub9c8\ub294 \ucde8\uc784\uc2dd\uc5d0\uc11c \ub178\ub798 \uc548\ubb34 \uc911 \ud558\ub098\uc778 \uc190\uc744 \uaf2c\ub294 \ub3d9\uc791\uc744 \uc7a0\uc2dc \ud588\ub2e4. \uc774\ub9cc \ud06c\ub85c\uc2a8\uc774\ub77c\ub294 \uc0ac\ub78c\uc740 \uc624\ubc14\ub9c8\ub97c \ub2ee\uc740\uaf34\ub85c \uc720\uba85\ud55c\ub370, \u3008Single Ladies\u3009 \uc548\ubb34\ub97c \uc790\uc2e0\ub9cc\uc758 \ubc84\uc804\uc73c\ub85c \ub9cc\ub4e4\uc5c8\ub2e4. \ubbf8\uad6d\uc758 \ud658\uacbd\uc6b4\ub3d9\uac00\uc774\uc790 \uc815\uce58\uc778 \uc870 \ub124\uc774\uc158\uacfc \ubbf8\uad6d\uc758 \ubc30\uc6b0 \ud1b0 \ud589\ud06c\uc2a4\uc640 \uac19\uc774 \uc798 \uc54c\ub824\uc9c4 \uc720\uba85\uc778\ub4e4\ub3c4 \ub304\uc2a4\ub97c \ub530\ub77c \ud588\ub2e4. DJ \uc6f9\uc2a4\ud0c0\uc640 \uc9d0 \uc870\ub098\uc2a4\uc758 \"Dancin on Me\" \ubba4\uc9c1\ube44\ub514\uc624\uc5d0\uc11c \u3008Single Ladies\u3009\ub97c \ubaa8\ubc29\ud55c \uc138 \uba85\uc758 \uc5ec\uc790\uac00 \ubc30\uacbd\uc5d0\uc11c \ucda4\uc744 \ucd94\ub294 \uc7a5\uba74\uc774 \uc788\ub2e4. \ucf00\uc774\ud2f0 \ube0c\ub79c\ub4dc\ub294 2010\ub144 3\uc6d4 12\uc77c \uc5f4\ub9b0 \uc790\uc120 \ucf54\ubbf9 \ub9b4\ub9ac\ud504\uc758 \uae30\uae08 \ubaa8\uae08\uc744 \uc704\ud55c BBC One\uc758 Let's Dance for Comic Relief\uc5d0\uc11c \uac80\uc815\uc0c9 \ub9ac\uc5b4\ud0c0\ub4dc \uc758\uc0c1\uacfc \uace8\ub4dc\uae00\ub85c\ube0c\ub97c \ub07c\uace0 \u3008Single Ladies\u3009 \uacf5\uc5f0\uc744 \ud3bc\ucce4\ub2e4. \uc81c\ub098 \uc5b4\uc2dc\ucf54\ube44\uce20, \ud06c\ub9ac\uc2a4 \ucf5c\ud37c, \ud5e4\ub354 \ubaa8\ub9ac\uc2a4\ub294 2011\ub144 6\uc6d4 \uc5f4\ub9b0 \uae00\ub9ac \ub77c\uc774\ube0c \ucf58\uc11c\ud2b8\uc5d0\uc11c \u3008Single Ladies\u3009 \ub304\uc2a4\ub97c \ucdc4\ub2e4. \uc544\uce78\uc18c \uc8fc \ub9ac\ud2c0\ub85d\uc5d0 \uc788\ub294 \uac00\uc774\uc5b4\uc2a4\ud504\ub9c1\uc2a4\uc81c\uc77c\uce68\ub840\uad50\ud68c\uc758 \uc74c\uc545\ubaa9\uc0ac\ub294 \ud569\ucc3d\ub2e8\uc6d0\ub4e4\uc5d0\uac8c \u3008Single Ladies\u3009 \ub9ac\ubbf9\uc2a4 \ubc84\uc804\uc73c\ub85c \ucda4\uc744 \ucd94\uba74 \uc8fc\ubaa9\uc744 \ub04c \uac83\uc774\ub77c\uba70 \"\ud6cc\ub96d\ud55c \uc544\uc774\ub514\uc5b4\"\ub77c\uace0 \ud588\ub2e4. \uac00\uc774\uc5b4\uc2a4\ud504\ub9c1\uc2a4\uc81c\uc77c\uce68\ub840\uad50\ud68c\uc758 \uc74c\uc545\ubaa9\uc0ac \ubcf4\uc870 \uc2e0\ub514 \uc6d4\ucee4\uc2a8\uc774 2011\ub144 8\uc6d4 29\uc77c \uc720\ud29c\ube0c\uc5d0 \ub3d9\uc601\uc0c1\uc744 \uc62c\ub838\ub2e4. \uc2f8\uc774\ub294 2013\ub144 4\uc6d4 13\uc77c\uc5d0\ub294 \uc11c\uc6b8\uc5d0\uc11c \uc5f4\ub9b0 \ucf58\uc11c\ud2b8 \ud574\ud504\ub2dd \ub3c4\uc911 \ube68\uac04\uc0c9 \ub9ac\uc5b4\ud0c0\ub4dc \uc758\uc0c1\uacfc \ubd80\uce20\ub97c\uc2e0\uace0 \u3008Single Ladies\u3009\ub97c \ubd88\ub800\uace0, 6\uc6d4 16\uc77c\uc5d0\ub294 \uce90\ub098\ub2e4\uc758 \uc74c\uc545 \uc2dc\uc0c1\uc2dd \uba38\uce58 \ubba4\uc9c1 \ube44\ub514\uc624 \uc5b4\uc6cc\uc988\uc5d0\uc11c \uc548\ubb34\ub97c \ucdc4\ub2e4. \uc288\ud37c\uc8fc\ub2c8\uc5b4\uc758 \uc2e0\ub3d9, \ub3d9\ud574, \uc740\ud601\uc740 2010\ub144\ubd80\ud130 2011\ub144\uae4c\uc9c0 \uc5f4\ub9b0 \uc790\uc2e0\ub4e4\uc758 \ucf58\uc11c\ud2b8 \ud22c\uc5b4 SUPER SHOW 3\uc5d0\uc11c \u3008Single Ladies\u3009 \ud328\ub7ec\ub514 \uacf5\uc5f0\uc744 \ud588\ub2e4. \uc815\uc8fc\ub9ac\uc640 \ub808\uc778\ubcf4\uc6b0\ub294 2010\ub144 2\uc6d4 14\uc77c MBC \uc124\ud2b9\uc9d1 \u300a\uc2a4\ud0c0 \ub304\uc2a4 \ub300\uaca9\ub3cc \ucda4\ubd24\ub2e4!\u300b\uc5d0\uc11c \ud569\ub3d9\ubb34\ub300\ub97c \ud3bc\ucce4\ub2e4.", "answer": "\uc774\ub9cc \ud06c\ub85c\uc2a8", "sentence": "\uc774\ub9cc \ud06c\ub85c\uc2a8 \uc774\ub77c\ub294 \uc0ac\ub78c\uc740 \uc624\ubc14\ub9c8\ub97c \ub2ee\uc740\uaf34\ub85c \uc720\uba85\ud55c\ub370, \u3008Single Ladies\u3009 \uc548\ubb34\ub97c \uc790\uc2e0\ub9cc\uc758 \ubc84\uc804\uc73c\ub85c \ub9cc\ub4e4\uc5c8\ub2e4.", "paragraph_sentence": "\uac00\uc218 \uc874 \ub808\uc804\ub4dc\uac00 \ub9cc\ub4e0 \ube44\ub514\uc624\uc5d0\uc11c \ubbf8\uad6d\uc758 \ub300\ud1b5\ub839 \ubc84\ub77d \uc624\ubc14\ub9c8\uac00 \uc640\uc774\ud504 \ubbf8\uc258 \uc624\ubc14\ub9c8\uc640 \ud568\uaed8 \ucd9c\uc5f0\ud574 \u3008Single Ladies\u3009\uc758 \uc548\ubb34\ub97c \ucdc4\ub2e4. \ub610\ud55c \uc624\ubc14\ub9c8\ub294 \ucde8\uc784\uc2dd\uc5d0\uc11c \ub178\ub798 \uc548\ubb34 \uc911 \ud558\ub098\uc778 \uc190\uc744 \uaf2c\ub294 \ub3d9\uc791\uc744 \uc7a0\uc2dc \ud588\ub2e4. \uc774\ub9cc \ud06c\ub85c\uc2a8 \uc774\ub77c\ub294 \uc0ac\ub78c\uc740 \uc624\ubc14\ub9c8\ub97c \ub2ee\uc740\uaf34\ub85c \uc720\uba85\ud55c\ub370, \u3008Single Ladies\u3009 \uc548\ubb34\ub97c \uc790\uc2e0\ub9cc\uc758 \ubc84\uc804\uc73c\ub85c \ub9cc\ub4e4\uc5c8\ub2e4. \ubbf8\uad6d\uc758 \ud658\uacbd\uc6b4\ub3d9\uac00\uc774\uc790 \uc815\uce58\uc778 \uc870 \ub124\uc774\uc158\uacfc \ubbf8\uad6d\uc758 \ubc30\uc6b0 \ud1b0 \ud589\ud06c\uc2a4\uc640 \uac19\uc774 \uc798 \uc54c\ub824\uc9c4 \uc720\uba85\uc778\ub4e4\ub3c4 \ub304\uc2a4\ub97c \ub530\ub77c \ud588\ub2e4. DJ \uc6f9\uc2a4\ud0c0\uc640 \uc9d0 \uc870\ub098\uc2a4\uc758 \"Dancin on Me\" \ubba4\uc9c1\ube44\ub514\uc624\uc5d0\uc11c \u3008Single Ladies\u3009\ub97c \ubaa8\ubc29\ud55c \uc138 \uba85\uc758 \uc5ec\uc790\uac00 \ubc30\uacbd\uc5d0\uc11c \ucda4\uc744 \ucd94\ub294 \uc7a5\uba74\uc774 \uc788\ub2e4. \ucf00\uc774\ud2f0 \ube0c\ub79c\ub4dc\ub294 2010\ub144 3\uc6d4 12\uc77c \uc5f4\ub9b0 \uc790\uc120 \ucf54\ubbf9 \ub9b4\ub9ac\ud504\uc758 \uae30\uae08 \ubaa8\uae08\uc744 \uc704\ud55c BBC One\uc758 Let's Dance for Comic Relief\uc5d0\uc11c \uac80\uc815\uc0c9 \ub9ac\uc5b4\ud0c0\ub4dc \uc758\uc0c1\uacfc \uace8\ub4dc\uae00\ub85c\ube0c\ub97c \ub07c\uace0 \u3008Single Ladies\u3009 \uacf5\uc5f0\uc744 \ud3bc\ucce4\ub2e4. \uc81c\ub098 \uc5b4\uc2dc\ucf54\ube44\uce20, \ud06c\ub9ac\uc2a4 \ucf5c\ud37c, \ud5e4\ub354 \ubaa8\ub9ac\uc2a4\ub294 2011\ub144 6\uc6d4 \uc5f4\ub9b0 \uae00\ub9ac \ub77c\uc774\ube0c \ucf58\uc11c\ud2b8\uc5d0\uc11c \u3008Single Ladies\u3009 \ub304\uc2a4\ub97c \ucdc4\ub2e4. \uc544\uce78\uc18c \uc8fc \ub9ac\ud2c0\ub85d\uc5d0 \uc788\ub294 \uac00\uc774\uc5b4\uc2a4\ud504\ub9c1\uc2a4\uc81c\uc77c\uce68\ub840\uad50\ud68c\uc758 \uc74c\uc545\ubaa9\uc0ac\ub294 \ud569\ucc3d\ub2e8\uc6d0\ub4e4\uc5d0\uac8c \u3008Single Ladies\u3009 \ub9ac\ubbf9\uc2a4 \ubc84\uc804\uc73c\ub85c \ucda4\uc744 \ucd94\uba74 \uc8fc\ubaa9\uc744 \ub04c \uac83\uc774\ub77c\uba70 \"\ud6cc\ub96d\ud55c \uc544\uc774\ub514\uc5b4\"\ub77c\uace0 \ud588\ub2e4. \uac00\uc774\uc5b4\uc2a4\ud504\ub9c1\uc2a4\uc81c\uc77c\uce68\ub840\uad50\ud68c\uc758 \uc74c\uc545\ubaa9\uc0ac \ubcf4\uc870 \uc2e0\ub514 \uc6d4\ucee4\uc2a8\uc774 2011\ub144 8\uc6d4 29\uc77c \uc720\ud29c\ube0c\uc5d0 \ub3d9\uc601\uc0c1\uc744 \uc62c\ub838\ub2e4. \uc2f8\uc774\ub294 2013\ub144 4\uc6d4 13\uc77c\uc5d0\ub294 \uc11c\uc6b8\uc5d0\uc11c \uc5f4\ub9b0 \ucf58\uc11c\ud2b8 \ud574\ud504\ub2dd \ub3c4\uc911 \ube68\uac04\uc0c9 \ub9ac\uc5b4\ud0c0\ub4dc \uc758\uc0c1\uacfc \ubd80\uce20\ub97c\uc2e0\uace0 \u3008Single Ladies\u3009\ub97c \ubd88\ub800\uace0, 6\uc6d4 16\uc77c\uc5d0\ub294 \uce90\ub098\ub2e4\uc758 \uc74c\uc545 \uc2dc\uc0c1\uc2dd \uba38\uce58 \ubba4\uc9c1 \ube44\ub514\uc624 \uc5b4\uc6cc\uc988\uc5d0\uc11c \uc548\ubb34\ub97c \ucdc4\ub2e4. \uc288\ud37c\uc8fc\ub2c8\uc5b4\uc758 \uc2e0\ub3d9, \ub3d9\ud574, \uc740\ud601\uc740 2010\ub144\ubd80\ud130 2011\ub144\uae4c\uc9c0 \uc5f4\ub9b0 \uc790\uc2e0\ub4e4\uc758 \ucf58\uc11c\ud2b8 \ud22c\uc5b4 SUPER SHOW 3\uc5d0\uc11c \u3008Single Ladies\u3009 \ud328\ub7ec\ub514 \uacf5\uc5f0\uc744 \ud588\ub2e4. \uc815\uc8fc\ub9ac\uc640 \ub808\uc778\ubcf4\uc6b0\ub294 2010\ub144 2\uc6d4 14\uc77c MBC \uc124\ud2b9\uc9d1 \u300a\uc2a4\ud0c0 \ub304\uc2a4 \ub300\uaca9\ub3cc \ucda4\ubd24\ub2e4!\u300b\uc5d0\uc11c \ud569\ub3d9\ubb34\ub300\ub97c \ud3bc\ucce4\ub2e4.", "paragraph_answer": "\uac00\uc218 \uc874 \ub808\uc804\ub4dc\uac00 \ub9cc\ub4e0 \ube44\ub514\uc624\uc5d0\uc11c \ubbf8\uad6d\uc758 \ub300\ud1b5\ub839 \ubc84\ub77d \uc624\ubc14\ub9c8\uac00 \uc640\uc774\ud504 \ubbf8\uc258 \uc624\ubc14\ub9c8\uc640 \ud568\uaed8 \ucd9c\uc5f0\ud574 \u3008Single Ladies\u3009\uc758 \uc548\ubb34\ub97c \ucdc4\ub2e4. \ub610\ud55c \uc624\ubc14\ub9c8\ub294 \ucde8\uc784\uc2dd\uc5d0\uc11c \ub178\ub798 \uc548\ubb34 \uc911 \ud558\ub098\uc778 \uc190\uc744 \uaf2c\ub294 \ub3d9\uc791\uc744 \uc7a0\uc2dc \ud588\ub2e4. \uc774\ub9cc \ud06c\ub85c\uc2a8 \uc774\ub77c\ub294 \uc0ac\ub78c\uc740 \uc624\ubc14\ub9c8\ub97c \ub2ee\uc740\uaf34\ub85c \uc720\uba85\ud55c\ub370, \u3008Single Ladies\u3009 \uc548\ubb34\ub97c \uc790\uc2e0\ub9cc\uc758 \ubc84\uc804\uc73c\ub85c \ub9cc\ub4e4\uc5c8\ub2e4. \ubbf8\uad6d\uc758 \ud658\uacbd\uc6b4\ub3d9\uac00\uc774\uc790 \uc815\uce58\uc778 \uc870 \ub124\uc774\uc158\uacfc \ubbf8\uad6d\uc758 \ubc30\uc6b0 \ud1b0 \ud589\ud06c\uc2a4\uc640 \uac19\uc774 \uc798 \uc54c\ub824\uc9c4 \uc720\uba85\uc778\ub4e4\ub3c4 \ub304\uc2a4\ub97c \ub530\ub77c \ud588\ub2e4. DJ \uc6f9\uc2a4\ud0c0\uc640 \uc9d0 \uc870\ub098\uc2a4\uc758 \"Dancin on Me\" \ubba4\uc9c1\ube44\ub514\uc624\uc5d0\uc11c \u3008Single Ladies\u3009\ub97c \ubaa8\ubc29\ud55c \uc138 \uba85\uc758 \uc5ec\uc790\uac00 \ubc30\uacbd\uc5d0\uc11c \ucda4\uc744 \ucd94\ub294 \uc7a5\uba74\uc774 \uc788\ub2e4. \ucf00\uc774\ud2f0 \ube0c\ub79c\ub4dc\ub294 2010\ub144 3\uc6d4 12\uc77c \uc5f4\ub9b0 \uc790\uc120 \ucf54\ubbf9 \ub9b4\ub9ac\ud504\uc758 \uae30\uae08 \ubaa8\uae08\uc744 \uc704\ud55c BBC One\uc758 Let's Dance for Comic Relief\uc5d0\uc11c \uac80\uc815\uc0c9 \ub9ac\uc5b4\ud0c0\ub4dc \uc758\uc0c1\uacfc \uace8\ub4dc\uae00\ub85c\ube0c\ub97c \ub07c\uace0 \u3008Single Ladies\u3009 \uacf5\uc5f0\uc744 \ud3bc\ucce4\ub2e4. \uc81c\ub098 \uc5b4\uc2dc\ucf54\ube44\uce20, \ud06c\ub9ac\uc2a4 \ucf5c\ud37c, \ud5e4\ub354 \ubaa8\ub9ac\uc2a4\ub294 2011\ub144 6\uc6d4 \uc5f4\ub9b0 \uae00\ub9ac \ub77c\uc774\ube0c \ucf58\uc11c\ud2b8\uc5d0\uc11c \u3008Single Ladies\u3009 \ub304\uc2a4\ub97c \ucdc4\ub2e4. \uc544\uce78\uc18c \uc8fc \ub9ac\ud2c0\ub85d\uc5d0 \uc788\ub294 \uac00\uc774\uc5b4\uc2a4\ud504\ub9c1\uc2a4\uc81c\uc77c\uce68\ub840\uad50\ud68c\uc758 \uc74c\uc545\ubaa9\uc0ac\ub294 \ud569\ucc3d\ub2e8\uc6d0\ub4e4\uc5d0\uac8c \u3008Single Ladies\u3009 \ub9ac\ubbf9\uc2a4 \ubc84\uc804\uc73c\ub85c \ucda4\uc744 \ucd94\uba74 \uc8fc\ubaa9\uc744 \ub04c \uac83\uc774\ub77c\uba70 \"\ud6cc\ub96d\ud55c \uc544\uc774\ub514\uc5b4\"\ub77c\uace0 \ud588\ub2e4. \uac00\uc774\uc5b4\uc2a4\ud504\ub9c1\uc2a4\uc81c\uc77c\uce68\ub840\uad50\ud68c\uc758 \uc74c\uc545\ubaa9\uc0ac \ubcf4\uc870 \uc2e0\ub514 \uc6d4\ucee4\uc2a8\uc774 2011\ub144 8\uc6d4 29\uc77c \uc720\ud29c\ube0c\uc5d0 \ub3d9\uc601\uc0c1\uc744 \uc62c\ub838\ub2e4. \uc2f8\uc774\ub294 2013\ub144 4\uc6d4 13\uc77c\uc5d0\ub294 \uc11c\uc6b8\uc5d0\uc11c \uc5f4\ub9b0 \ucf58\uc11c\ud2b8 \ud574\ud504\ub2dd \ub3c4\uc911 \ube68\uac04\uc0c9 \ub9ac\uc5b4\ud0c0\ub4dc \uc758\uc0c1\uacfc \ubd80\uce20\ub97c\uc2e0\uace0 \u3008Single Ladies\u3009\ub97c \ubd88\ub800\uace0, 6\uc6d4 16\uc77c\uc5d0\ub294 \uce90\ub098\ub2e4\uc758 \uc74c\uc545 \uc2dc\uc0c1\uc2dd \uba38\uce58 \ubba4\uc9c1 \ube44\ub514\uc624 \uc5b4\uc6cc\uc988\uc5d0\uc11c \uc548\ubb34\ub97c \ucdc4\ub2e4. \uc288\ud37c\uc8fc\ub2c8\uc5b4\uc758 \uc2e0\ub3d9, \ub3d9\ud574, \uc740\ud601\uc740 2010\ub144\ubd80\ud130 2011\ub144\uae4c\uc9c0 \uc5f4\ub9b0 \uc790\uc2e0\ub4e4\uc758 \ucf58\uc11c\ud2b8 \ud22c\uc5b4 SUPER SHOW 3\uc5d0\uc11c \u3008Single Ladies\u3009 \ud328\ub7ec\ub514 \uacf5\uc5f0\uc744 \ud588\ub2e4. \uc815\uc8fc\ub9ac\uc640 \ub808\uc778\ubcf4\uc6b0\ub294 2010\ub144 2\uc6d4 14\uc77c MBC \uc124\ud2b9\uc9d1 \u300a\uc2a4\ud0c0 \ub304\uc2a4 \ub300\uaca9\ub3cc \ucda4\ubd24\ub2e4!\u300b\uc5d0\uc11c \ud569\ub3d9\ubb34\ub300\ub97c \ud3bc\ucce4\ub2e4.", "sentence_answer": " \uc774\ub9cc \ud06c\ub85c\uc2a8 \uc774\ub77c\ub294 \uc0ac\ub78c\uc740 \uc624\ubc14\ub9c8\ub97c \ub2ee\uc740\uaf34\ub85c \uc720\uba85\ud55c\ub370, \u3008Single Ladies\u3009 \uc548\ubb34\ub97c \uc790\uc2e0\ub9cc\uc758 \ubc84\uc804\uc73c\ub85c \ub9cc\ub4e4\uc5c8\ub2e4.", "paragraph_id": "5808934-28-1", "paragraph_question": "question: Single Ladies\ub97c \uc790\uc2e0\ub9cc\uc758 \ubc84\uc804\uc73c\ub85c \ub9cc\ub4e0 \uc624\ubc14\ub9c8\uc758 \ub2ee\uc740 \uaf34\uc740?, context: \uac00\uc218 \uc874 \ub808\uc804\ub4dc\uac00 \ub9cc\ub4e0 \ube44\ub514\uc624\uc5d0\uc11c \ubbf8\uad6d\uc758 \ub300\ud1b5\ub839 \ubc84\ub77d \uc624\ubc14\ub9c8\uac00 \uc640\uc774\ud504 \ubbf8\uc258 \uc624\ubc14\ub9c8\uc640 \ud568\uaed8 \ucd9c\uc5f0\ud574 \u3008Single Ladies\u3009\uc758 \uc548\ubb34\ub97c \ucdc4\ub2e4. \ub610\ud55c \uc624\ubc14\ub9c8\ub294 \ucde8\uc784\uc2dd\uc5d0\uc11c \ub178\ub798 \uc548\ubb34 \uc911 \ud558\ub098\uc778 \uc190\uc744 \uaf2c\ub294 \ub3d9\uc791\uc744 \uc7a0\uc2dc \ud588\ub2e4. \uc774\ub9cc \ud06c\ub85c\uc2a8\uc774\ub77c\ub294 \uc0ac\ub78c\uc740 \uc624\ubc14\ub9c8\ub97c \ub2ee\uc740\uaf34\ub85c \uc720\uba85\ud55c\ub370, \u3008Single Ladies\u3009 \uc548\ubb34\ub97c \uc790\uc2e0\ub9cc\uc758 \ubc84\uc804\uc73c\ub85c \ub9cc\ub4e4\uc5c8\ub2e4. \ubbf8\uad6d\uc758 \ud658\uacbd\uc6b4\ub3d9\uac00\uc774\uc790 \uc815\uce58\uc778 \uc870 \ub124\uc774\uc158\uacfc \ubbf8\uad6d\uc758 \ubc30\uc6b0 \ud1b0 \ud589\ud06c\uc2a4\uc640 \uac19\uc774 \uc798 \uc54c\ub824\uc9c4 \uc720\uba85\uc778\ub4e4\ub3c4 \ub304\uc2a4\ub97c \ub530\ub77c \ud588\ub2e4. DJ \uc6f9\uc2a4\ud0c0\uc640 \uc9d0 \uc870\ub098\uc2a4\uc758 \"Dancin on Me\" \ubba4\uc9c1\ube44\ub514\uc624\uc5d0\uc11c \u3008Single Ladies\u3009\ub97c \ubaa8\ubc29\ud55c \uc138 \uba85\uc758 \uc5ec\uc790\uac00 \ubc30\uacbd\uc5d0\uc11c \ucda4\uc744 \ucd94\ub294 \uc7a5\uba74\uc774 \uc788\ub2e4. \ucf00\uc774\ud2f0 \ube0c\ub79c\ub4dc\ub294 2010\ub144 3\uc6d4 12\uc77c \uc5f4\ub9b0 \uc790\uc120 \ucf54\ubbf9 \ub9b4\ub9ac\ud504\uc758 \uae30\uae08 \ubaa8\uae08\uc744 \uc704\ud55c BBC One\uc758 Let's Dance for Comic Relief\uc5d0\uc11c \uac80\uc815\uc0c9 \ub9ac\uc5b4\ud0c0\ub4dc \uc758\uc0c1\uacfc \uace8\ub4dc\uae00\ub85c\ube0c\ub97c \ub07c\uace0 \u3008Single Ladies\u3009 \uacf5\uc5f0\uc744 \ud3bc\ucce4\ub2e4. \uc81c\ub098 \uc5b4\uc2dc\ucf54\ube44\uce20, \ud06c\ub9ac\uc2a4 \ucf5c\ud37c, \ud5e4\ub354 \ubaa8\ub9ac\uc2a4\ub294 2011\ub144 6\uc6d4 \uc5f4\ub9b0 \uae00\ub9ac \ub77c\uc774\ube0c \ucf58\uc11c\ud2b8\uc5d0\uc11c \u3008Single Ladies\u3009 \ub304\uc2a4\ub97c \ucdc4\ub2e4. \uc544\uce78\uc18c \uc8fc \ub9ac\ud2c0\ub85d\uc5d0 \uc788\ub294 \uac00\uc774\uc5b4\uc2a4\ud504\ub9c1\uc2a4\uc81c\uc77c\uce68\ub840\uad50\ud68c\uc758 \uc74c\uc545\ubaa9\uc0ac\ub294 \ud569\ucc3d\ub2e8\uc6d0\ub4e4\uc5d0\uac8c \u3008Single Ladies\u3009 \ub9ac\ubbf9\uc2a4 \ubc84\uc804\uc73c\ub85c \ucda4\uc744 \ucd94\uba74 \uc8fc\ubaa9\uc744 \ub04c \uac83\uc774\ub77c\uba70 \"\ud6cc\ub96d\ud55c \uc544\uc774\ub514\uc5b4\"\ub77c\uace0 \ud588\ub2e4. \uac00\uc774\uc5b4\uc2a4\ud504\ub9c1\uc2a4\uc81c\uc77c\uce68\ub840\uad50\ud68c\uc758 \uc74c\uc545\ubaa9\uc0ac \ubcf4\uc870 \uc2e0\ub514 \uc6d4\ucee4\uc2a8\uc774 2011\ub144 8\uc6d4 29\uc77c \uc720\ud29c\ube0c\uc5d0 \ub3d9\uc601\uc0c1\uc744 \uc62c\ub838\ub2e4. \uc2f8\uc774\ub294 2013\ub144 4\uc6d4 13\uc77c\uc5d0\ub294 \uc11c\uc6b8\uc5d0\uc11c \uc5f4\ub9b0 \ucf58\uc11c\ud2b8 \ud574\ud504\ub2dd \ub3c4\uc911 \ube68\uac04\uc0c9 \ub9ac\uc5b4\ud0c0\ub4dc \uc758\uc0c1\uacfc \ubd80\uce20\ub97c\uc2e0\uace0 \u3008Single Ladies\u3009\ub97c \ubd88\ub800\uace0, 6\uc6d4 16\uc77c\uc5d0\ub294 \uce90\ub098\ub2e4\uc758 \uc74c\uc545 \uc2dc\uc0c1\uc2dd \uba38\uce58 \ubba4\uc9c1 \ube44\ub514\uc624 \uc5b4\uc6cc\uc988\uc5d0\uc11c \uc548\ubb34\ub97c \ucdc4\ub2e4. \uc288\ud37c\uc8fc\ub2c8\uc5b4\uc758 \uc2e0\ub3d9, \ub3d9\ud574, \uc740\ud601\uc740 2010\ub144\ubd80\ud130 2011\ub144\uae4c\uc9c0 \uc5f4\ub9b0 \uc790\uc2e0\ub4e4\uc758 \ucf58\uc11c\ud2b8 \ud22c\uc5b4 SUPER SHOW 3\uc5d0\uc11c \u3008Single Ladies\u3009 \ud328\ub7ec\ub514 \uacf5\uc5f0\uc744 \ud588\ub2e4. \uc815\uc8fc\ub9ac\uc640 \ub808\uc778\ubcf4\uc6b0\ub294 2010\ub144 2\uc6d4 14\uc77c MBC \uc124\ud2b9\uc9d1 \u300a\uc2a4\ud0c0 \ub304\uc2a4 \ub300\uaca9\ub3cc \ucda4\ubd24\ub2e4!\u300b\uc5d0\uc11c \ud569\ub3d9\ubb34\ub300\ub97c \ud3bc\ucce4\ub2e4."} {"question": "\ubd80\uc790 \uac10\uc138 \ub17c\ub780\uc744 \uc77c\uc73c\ud0a4\uba70 \uc774\uba85\ubc15 \uc815\ubd80 \ub2f9\uc2dc \ud3d0\uc9c0\ub41c \uc138\uae08\uc740?", "paragraph": "2007\ub144 \uce58\ub7ec\uc9c4 \uc81c17\ub300 \ub300\uc120\uc5d0\uc11c \ud55c\ub098\ub77c\ub2f9\uc758 \uc774\uba85\ubc15 \ud6c4\ubcf4\uac00 \ub300\ud1b5\ub839\uc5d0 \ub2f9\uc120\ub418\uc5c8\ub2e4. \uc774\uba85\ubc15 \uc815\ubd80\ub294 7\ub300 \uac15\uad6d, 4\ub9cc\ub2ec\ub7ec \uc2dc\ub300, 7% \uc131\uc7a5('747 \uacf5\uc57d')\uc744 \ub0b4\uc138\uc6e0\ub2e4. \ud558\uc9c0\ub9cc \uc9d1\uad8c \ucd08\uc778 2008\ub144 \ubbf8\uad6d\ubc1c \uc138\uacc4 \uae08\uc735 \uc704\uae30\uc758 \uc5ec\ud30c\ub85c \uc778\ud55c \uacbd\uc81c\uc801 \uc704\uae30\uc640, \uad11\uc6b0\ubcd1\uc18c \uc218\uc785 \ubc18\ub300 \ucd1b\ubd88\uc9d1\ud68c\ub85c \uc778\ud55c \uc815\uce58\uc801 \uc704\uae30\ub97c \ub3d9\uc2dc\uc5d0 \uacaa\uc5c8\ub2e4. \uae08\uc735\uc704\uae30\ub97c \uadf9\ubcf5\ud558\uae30 \uc704\ud574 \uc774\uba85\ubc15 \uc815\ubd80\ub294 \uc5ec\ub7ec \uc815\ucc45\uc744 \uc2dc\ub3c4\ud558\ub294\ub370, \uadf8 \uc911 \ub300\ud45c\uc801\uc778 \uac83\uc774 4\ub300\uac15 \uc0ac\uc5c5\uc774\ub2e4. 4\ub300\uac15 \uc0ac\uc5c5\uc740 \uac74\uc124\uacbd\uae30\ub97c \uc0b4\ub838\uace0 \ud64d\uc218 \ud53c\ud574\ub97c \uc904\uc600\ub2e4\ub294 \uae0d\uc815\uc801 \ud3c9\uac00\uc640 \uc138\uae08 \ub0ad\ube44\uc640 \ud658\uacbd\ud30c\uad34\ub77c\ub294 \ubd80\uc815\uc801 \ud3c9\uac00\uac00 \uac01\uac01 \uc874\uc7ac\ud55c\ub2e4. 2010\ub144\uc5d0\ub294 G20\uc815\uc0c1\ud68c\uc758 \uac1c\ucd5c\uc640, 2012\ub144 \ud575\uc548\ubcf4\uc815\uc0c1\ud68c\uc758 \uac1c\ucd5c\ud558\uc5ec \uc678\uad50\uc801 \uc131\uacfc\ub97c \uc774\ub918\uace0, \uae08\uc735\uc704\uae30 \uadf9\ubcf5, \uad6d\uac00\uc2e0\uc6a9\ub4f1\uae09 \ud5a5\uc0c1, \uc6d0\uc804 \uc218\uc8fc \ub4f1\uc758 \uc131\uacfc\ub3c4 \uac70\ub450\uc5c8\ub2e4\ub294 \uae0d\uc815\uc801\uc778 \ud3c9\uac00\uac00 \uc874\uc7ac\ud55c\ub2e4. \ud558\uc9c0\ub9cc \ubbf8\ub514\uc5b4\ubc95 \uac1c\uc815\uc73c\ub85c \uc5b8\ub860\uc7a5\uc545\uc774 \uc544\ub2c8\ub0d0\ub294 \ube44\ud310\uc801 \uc2dc\uac01\uc774 \uc874\uc7ac\ud558\uba70, \uc138\uc885\uc2dc \uc218\uc815\uc548\uacfc \uac19\uc774 \uc5ec\ub7ec \uc815\ucc45\ub4e4\uc758 \ucd94\uc9c4\uc5d0 \ub300\ud55c \ucc2c\ubc18 \uc591\ub860\uc774 \uc874\uc7ac\ud55c\ub2e4. \ub610\ud55c, \uc885\ud569\ubd80\ub3d9\uc0b0\uc138 \ud3d0\uc9c0\ub85c \ubd80\uc790 \uac10\uc138 \uc758\ud639\uc774 \uc81c\uae30\ub418\uc5c8\uace0, \uc57c\uc2ec\ucc28\uac8c \ucd94\uc9c4\ud55c \uc790\uc6d0\uc678\uad50\uc5d0 \ub300\ud574\uc11c\ub294 \uc138\uae08 \ub0ad\ube44\uc640 \ube44\ub9ac \uc758\ud639\uc774 \uc81c\uae30\ub418\uc5b4 \ub17c\ub780\uc774 \ub418\uace0\uc788\ub2e4. \ud55c\ud3b8, \uc774\uba85\ubc15 \uc815\ubd80\ub294 \ud5cc\uc815 \uc0ac\uc0c1 \ucd5c\ucd08\ub85c \ub300\ud1b5\ub839\uc774 \uc784\uae30 \ub9d0 \ud0c8\ub2f9\uc744 \ud558\uc9c0 \uc54a\uace0 \uc815\uad8c \uc7ac\ucc3d\ucd9c\uc744 \ud55c \uc815\ubd80\uc774\uae30\ub3c4 \ud558\ub2e4.", "answer": "\uc885\ud569\ubd80\ub3d9\uc0b0\uc138", "sentence": "\ub610\ud55c, \uc885\ud569\ubd80\ub3d9\uc0b0\uc138 \ud3d0\uc9c0\ub85c \ubd80\uc790 \uac10\uc138 \uc758\ud639\uc774 \uc81c\uae30\ub418\uc5c8\uace0, \uc57c\uc2ec\ucc28\uac8c \ucd94\uc9c4\ud55c \uc790\uc6d0\uc678\uad50\uc5d0 \ub300\ud574\uc11c\ub294 \uc138\uae08 \ub0ad\ube44\uc640 \ube44\ub9ac \uc758\ud639\uc774 \uc81c\uae30\ub418\uc5b4 \ub17c\ub780\uc774 \ub418\uace0\uc788\ub2e4.", "paragraph_sentence": "2007\ub144 \uce58\ub7ec\uc9c4 \uc81c17\ub300 \ub300\uc120\uc5d0\uc11c \ud55c\ub098\ub77c\ub2f9\uc758 \uc774\uba85\ubc15 \ud6c4\ubcf4\uac00 \ub300\ud1b5\ub839\uc5d0 \ub2f9\uc120\ub418\uc5c8\ub2e4. \uc774\uba85\ubc15 \uc815\ubd80\ub294 7\ub300 \uac15\uad6d, 4\ub9cc\ub2ec\ub7ec \uc2dc\ub300, 7% \uc131\uc7a5('747 \uacf5\uc57d')\uc744 \ub0b4\uc138\uc6e0\ub2e4. \ud558\uc9c0\ub9cc \uc9d1\uad8c \ucd08\uc778 2008\ub144 \ubbf8\uad6d\ubc1c \uc138\uacc4 \uae08\uc735 \uc704\uae30\uc758 \uc5ec\ud30c\ub85c \uc778\ud55c \uacbd\uc81c\uc801 \uc704\uae30\uc640, \uad11\uc6b0\ubcd1\uc18c \uc218\uc785 \ubc18\ub300 \ucd1b\ubd88\uc9d1\ud68c\ub85c \uc778\ud55c \uc815\uce58\uc801 \uc704\uae30\ub97c \ub3d9\uc2dc\uc5d0 \uacaa\uc5c8\ub2e4. \uae08\uc735\uc704\uae30\ub97c \uadf9\ubcf5\ud558\uae30 \uc704\ud574 \uc774\uba85\ubc15 \uc815\ubd80\ub294 \uc5ec\ub7ec \uc815\ucc45\uc744 \uc2dc\ub3c4\ud558\ub294\ub370, \uadf8 \uc911 \ub300\ud45c\uc801\uc778 \uac83\uc774 4\ub300\uac15 \uc0ac\uc5c5\uc774\ub2e4. 4\ub300\uac15 \uc0ac\uc5c5\uc740 \uac74\uc124\uacbd\uae30\ub97c \uc0b4\ub838\uace0 \ud64d\uc218 \ud53c\ud574\ub97c \uc904\uc600\ub2e4\ub294 \uae0d\uc815\uc801 \ud3c9\uac00\uc640 \uc138\uae08 \ub0ad\ube44\uc640 \ud658\uacbd\ud30c\uad34\ub77c\ub294 \ubd80\uc815\uc801 \ud3c9\uac00\uac00 \uac01\uac01 \uc874\uc7ac\ud55c\ub2e4. 2010\ub144\uc5d0\ub294 G20\uc815\uc0c1\ud68c\uc758 \uac1c\ucd5c\uc640, 2012\ub144 \ud575\uc548\ubcf4\uc815\uc0c1\ud68c\uc758 \uac1c\ucd5c\ud558\uc5ec \uc678\uad50\uc801 \uc131\uacfc\ub97c \uc774\ub918\uace0, \uae08\uc735\uc704\uae30 \uadf9\ubcf5, \uad6d\uac00\uc2e0\uc6a9\ub4f1\uae09 \ud5a5\uc0c1, \uc6d0\uc804 \uc218\uc8fc \ub4f1\uc758 \uc131\uacfc\ub3c4 \uac70\ub450\uc5c8\ub2e4\ub294 \uae0d\uc815\uc801\uc778 \ud3c9\uac00\uac00 \uc874\uc7ac\ud55c\ub2e4. \ud558\uc9c0\ub9cc \ubbf8\ub514\uc5b4\ubc95 \uac1c\uc815\uc73c\ub85c \uc5b8\ub860\uc7a5\uc545\uc774 \uc544\ub2c8\ub0d0\ub294 \ube44\ud310\uc801 \uc2dc\uac01\uc774 \uc874\uc7ac\ud558\uba70, \uc138\uc885\uc2dc \uc218\uc815\uc548\uacfc \uac19\uc774 \uc5ec\ub7ec \uc815\ucc45\ub4e4\uc758 \ucd94\uc9c4\uc5d0 \ub300\ud55c \ucc2c\ubc18 \uc591\ub860\uc774 \uc874\uc7ac\ud55c\ub2e4. \ub610\ud55c, \uc885\ud569\ubd80\ub3d9\uc0b0\uc138 \ud3d0\uc9c0\ub85c \ubd80\uc790 \uac10\uc138 \uc758\ud639\uc774 \uc81c\uae30\ub418\uc5c8\uace0, \uc57c\uc2ec\ucc28\uac8c \ucd94\uc9c4\ud55c \uc790\uc6d0\uc678\uad50\uc5d0 \ub300\ud574\uc11c\ub294 \uc138\uae08 \ub0ad\ube44\uc640 \ube44\ub9ac \uc758\ud639\uc774 \uc81c\uae30\ub418\uc5b4 \ub17c\ub780\uc774 \ub418\uace0\uc788\ub2e4. \ud55c\ud3b8, \uc774\uba85\ubc15 \uc815\ubd80\ub294 \ud5cc\uc815 \uc0ac\uc0c1 \ucd5c\ucd08\ub85c \ub300\ud1b5\ub839\uc774 \uc784\uae30 \ub9d0 \ud0c8\ub2f9\uc744 \ud558\uc9c0 \uc54a\uace0 \uc815\uad8c \uc7ac\ucc3d\ucd9c\uc744 \ud55c \uc815\ubd80\uc774\uae30\ub3c4 \ud558\ub2e4.", "paragraph_answer": "2007\ub144 \uce58\ub7ec\uc9c4 \uc81c17\ub300 \ub300\uc120\uc5d0\uc11c \ud55c\ub098\ub77c\ub2f9\uc758 \uc774\uba85\ubc15 \ud6c4\ubcf4\uac00 \ub300\ud1b5\ub839\uc5d0 \ub2f9\uc120\ub418\uc5c8\ub2e4. \uc774\uba85\ubc15 \uc815\ubd80\ub294 7\ub300 \uac15\uad6d, 4\ub9cc\ub2ec\ub7ec \uc2dc\ub300, 7% \uc131\uc7a5('747 \uacf5\uc57d')\uc744 \ub0b4\uc138\uc6e0\ub2e4. \ud558\uc9c0\ub9cc \uc9d1\uad8c \ucd08\uc778 2008\ub144 \ubbf8\uad6d\ubc1c \uc138\uacc4 \uae08\uc735 \uc704\uae30\uc758 \uc5ec\ud30c\ub85c \uc778\ud55c \uacbd\uc81c\uc801 \uc704\uae30\uc640, \uad11\uc6b0\ubcd1\uc18c \uc218\uc785 \ubc18\ub300 \ucd1b\ubd88\uc9d1\ud68c\ub85c \uc778\ud55c \uc815\uce58\uc801 \uc704\uae30\ub97c \ub3d9\uc2dc\uc5d0 \uacaa\uc5c8\ub2e4. \uae08\uc735\uc704\uae30\ub97c \uadf9\ubcf5\ud558\uae30 \uc704\ud574 \uc774\uba85\ubc15 \uc815\ubd80\ub294 \uc5ec\ub7ec \uc815\ucc45\uc744 \uc2dc\ub3c4\ud558\ub294\ub370, \uadf8 \uc911 \ub300\ud45c\uc801\uc778 \uac83\uc774 4\ub300\uac15 \uc0ac\uc5c5\uc774\ub2e4. 4\ub300\uac15 \uc0ac\uc5c5\uc740 \uac74\uc124\uacbd\uae30\ub97c \uc0b4\ub838\uace0 \ud64d\uc218 \ud53c\ud574\ub97c \uc904\uc600\ub2e4\ub294 \uae0d\uc815\uc801 \ud3c9\uac00\uc640 \uc138\uae08 \ub0ad\ube44\uc640 \ud658\uacbd\ud30c\uad34\ub77c\ub294 \ubd80\uc815\uc801 \ud3c9\uac00\uac00 \uac01\uac01 \uc874\uc7ac\ud55c\ub2e4. 2010\ub144\uc5d0\ub294 G20\uc815\uc0c1\ud68c\uc758 \uac1c\ucd5c\uc640, 2012\ub144 \ud575\uc548\ubcf4\uc815\uc0c1\ud68c\uc758 \uac1c\ucd5c\ud558\uc5ec \uc678\uad50\uc801 \uc131\uacfc\ub97c \uc774\ub918\uace0, \uae08\uc735\uc704\uae30 \uadf9\ubcf5, \uad6d\uac00\uc2e0\uc6a9\ub4f1\uae09 \ud5a5\uc0c1, \uc6d0\uc804 \uc218\uc8fc \ub4f1\uc758 \uc131\uacfc\ub3c4 \uac70\ub450\uc5c8\ub2e4\ub294 \uae0d\uc815\uc801\uc778 \ud3c9\uac00\uac00 \uc874\uc7ac\ud55c\ub2e4. \ud558\uc9c0\ub9cc \ubbf8\ub514\uc5b4\ubc95 \uac1c\uc815\uc73c\ub85c \uc5b8\ub860\uc7a5\uc545\uc774 \uc544\ub2c8\ub0d0\ub294 \ube44\ud310\uc801 \uc2dc\uac01\uc774 \uc874\uc7ac\ud558\uba70, \uc138\uc885\uc2dc \uc218\uc815\uc548\uacfc \uac19\uc774 \uc5ec\ub7ec \uc815\ucc45\ub4e4\uc758 \ucd94\uc9c4\uc5d0 \ub300\ud55c \ucc2c\ubc18 \uc591\ub860\uc774 \uc874\uc7ac\ud55c\ub2e4. \ub610\ud55c, \uc885\ud569\ubd80\ub3d9\uc0b0\uc138 \ud3d0\uc9c0\ub85c \ubd80\uc790 \uac10\uc138 \uc758\ud639\uc774 \uc81c\uae30\ub418\uc5c8\uace0, \uc57c\uc2ec\ucc28\uac8c \ucd94\uc9c4\ud55c \uc790\uc6d0\uc678\uad50\uc5d0 \ub300\ud574\uc11c\ub294 \uc138\uae08 \ub0ad\ube44\uc640 \ube44\ub9ac \uc758\ud639\uc774 \uc81c\uae30\ub418\uc5b4 \ub17c\ub780\uc774 \ub418\uace0\uc788\ub2e4. \ud55c\ud3b8, \uc774\uba85\ubc15 \uc815\ubd80\ub294 \ud5cc\uc815 \uc0ac\uc0c1 \ucd5c\ucd08\ub85c \ub300\ud1b5\ub839\uc774 \uc784\uae30 \ub9d0 \ud0c8\ub2f9\uc744 \ud558\uc9c0 \uc54a\uace0 \uc815\uad8c \uc7ac\ucc3d\ucd9c\uc744 \ud55c \uc815\ubd80\uc774\uae30\ub3c4 \ud558\ub2e4.", "sentence_answer": "\ub610\ud55c, \uc885\ud569\ubd80\ub3d9\uc0b0\uc138 \ud3d0\uc9c0\ub85c \ubd80\uc790 \uac10\uc138 \uc758\ud639\uc774 \uc81c\uae30\ub418\uc5c8\uace0, \uc57c\uc2ec\ucc28\uac8c \ucd94\uc9c4\ud55c \uc790\uc6d0\uc678\uad50\uc5d0 \ub300\ud574\uc11c\ub294 \uc138\uae08 \ub0ad\ube44\uc640 \ube44\ub9ac \uc758\ud639\uc774 \uc81c\uae30\ub418\uc5b4 \ub17c\ub780\uc774 \ub418\uace0\uc788\ub2e4.", "paragraph_id": "6485683-15-1", "paragraph_question": "question: \ubd80\uc790 \uac10\uc138 \ub17c\ub780\uc744 \uc77c\uc73c\ud0a4\uba70 \uc774\uba85\ubc15 \uc815\ubd80 \ub2f9\uc2dc \ud3d0\uc9c0\ub41c \uc138\uae08\uc740?, context: 2007\ub144 \uce58\ub7ec\uc9c4 \uc81c17\ub300 \ub300\uc120\uc5d0\uc11c \ud55c\ub098\ub77c\ub2f9\uc758 \uc774\uba85\ubc15 \ud6c4\ubcf4\uac00 \ub300\ud1b5\ub839\uc5d0 \ub2f9\uc120\ub418\uc5c8\ub2e4. \uc774\uba85\ubc15 \uc815\ubd80\ub294 7\ub300 \uac15\uad6d, 4\ub9cc\ub2ec\ub7ec \uc2dc\ub300, 7% \uc131\uc7a5('747 \uacf5\uc57d')\uc744 \ub0b4\uc138\uc6e0\ub2e4. \ud558\uc9c0\ub9cc \uc9d1\uad8c \ucd08\uc778 2008\ub144 \ubbf8\uad6d\ubc1c \uc138\uacc4 \uae08\uc735 \uc704\uae30\uc758 \uc5ec\ud30c\ub85c \uc778\ud55c \uacbd\uc81c\uc801 \uc704\uae30\uc640, \uad11\uc6b0\ubcd1\uc18c \uc218\uc785 \ubc18\ub300 \ucd1b\ubd88\uc9d1\ud68c\ub85c \uc778\ud55c \uc815\uce58\uc801 \uc704\uae30\ub97c \ub3d9\uc2dc\uc5d0 \uacaa\uc5c8\ub2e4. \uae08\uc735\uc704\uae30\ub97c \uadf9\ubcf5\ud558\uae30 \uc704\ud574 \uc774\uba85\ubc15 \uc815\ubd80\ub294 \uc5ec\ub7ec \uc815\ucc45\uc744 \uc2dc\ub3c4\ud558\ub294\ub370, \uadf8 \uc911 \ub300\ud45c\uc801\uc778 \uac83\uc774 4\ub300\uac15 \uc0ac\uc5c5\uc774\ub2e4. 4\ub300\uac15 \uc0ac\uc5c5\uc740 \uac74\uc124\uacbd\uae30\ub97c \uc0b4\ub838\uace0 \ud64d\uc218 \ud53c\ud574\ub97c \uc904\uc600\ub2e4\ub294 \uae0d\uc815\uc801 \ud3c9\uac00\uc640 \uc138\uae08 \ub0ad\ube44\uc640 \ud658\uacbd\ud30c\uad34\ub77c\ub294 \ubd80\uc815\uc801 \ud3c9\uac00\uac00 \uac01\uac01 \uc874\uc7ac\ud55c\ub2e4. 2010\ub144\uc5d0\ub294 G20\uc815\uc0c1\ud68c\uc758 \uac1c\ucd5c\uc640, 2012\ub144 \ud575\uc548\ubcf4\uc815\uc0c1\ud68c\uc758 \uac1c\ucd5c\ud558\uc5ec \uc678\uad50\uc801 \uc131\uacfc\ub97c \uc774\ub918\uace0, \uae08\uc735\uc704\uae30 \uadf9\ubcf5, \uad6d\uac00\uc2e0\uc6a9\ub4f1\uae09 \ud5a5\uc0c1, \uc6d0\uc804 \uc218\uc8fc \ub4f1\uc758 \uc131\uacfc\ub3c4 \uac70\ub450\uc5c8\ub2e4\ub294 \uae0d\uc815\uc801\uc778 \ud3c9\uac00\uac00 \uc874\uc7ac\ud55c\ub2e4. \ud558\uc9c0\ub9cc \ubbf8\ub514\uc5b4\ubc95 \uac1c\uc815\uc73c\ub85c \uc5b8\ub860\uc7a5\uc545\uc774 \uc544\ub2c8\ub0d0\ub294 \ube44\ud310\uc801 \uc2dc\uac01\uc774 \uc874\uc7ac\ud558\uba70, \uc138\uc885\uc2dc \uc218\uc815\uc548\uacfc \uac19\uc774 \uc5ec\ub7ec \uc815\ucc45\ub4e4\uc758 \ucd94\uc9c4\uc5d0 \ub300\ud55c \ucc2c\ubc18 \uc591\ub860\uc774 \uc874\uc7ac\ud55c\ub2e4. \ub610\ud55c, \uc885\ud569\ubd80\ub3d9\uc0b0\uc138 \ud3d0\uc9c0\ub85c \ubd80\uc790 \uac10\uc138 \uc758\ud639\uc774 \uc81c\uae30\ub418\uc5c8\uace0, \uc57c\uc2ec\ucc28\uac8c \ucd94\uc9c4\ud55c \uc790\uc6d0\uc678\uad50\uc5d0 \ub300\ud574\uc11c\ub294 \uc138\uae08 \ub0ad\ube44\uc640 \ube44\ub9ac \uc758\ud639\uc774 \uc81c\uae30\ub418\uc5b4 \ub17c\ub780\uc774 \ub418\uace0\uc788\ub2e4. \ud55c\ud3b8, \uc774\uba85\ubc15 \uc815\ubd80\ub294 \ud5cc\uc815 \uc0ac\uc0c1 \ucd5c\ucd08\ub85c \ub300\ud1b5\ub839\uc774 \uc784\uae30 \ub9d0 \ud0c8\ub2f9\uc744 \ud558\uc9c0 \uc54a\uace0 \uc815\uad8c \uc7ac\ucc3d\ucd9c\uc744 \ud55c \uc815\ubd80\uc774\uae30\ub3c4 \ud558\ub2e4."} {"question": "\uc5c4\uc8fc\uba85\uc740 \ub204\uad6c\uc758 \uc870\uce74\uc778\uac00?", "paragraph": "\uc774\uc740\uc758 \uc77c\ubcf8 \uc720\ud559\uc5d0 \uacbd\ud559\uc6d0 \ud559\uc0dd \uc870\ub300\ud638\uc640 \uc11c\ubcd1\uac11(\u5f90\u4e19\u7532), \uc5c4\uc8fc\uba85\uc774 \ub3d9\ud589\ud558\uc600\ub2e4. \uc5c4\uc8fc\uba85\uc740 \ud669\uadc0\ube44 \uc5c4\uc528\uc758 \uc870\uce74\uc774\uc790 \uc774\uc740\uacfc \ub098\uc774\ub3c4 \ub3d9\uac11\uc778 \uc808\uce5c\ud55c \uc0ac\uc774\uc5ec\uc11c \uc774\uc740\uc774 \uc5c4\uc8fc\uba85\uc758 \ub3d9\ud589\uc744 \uac04\uccad\ud588\uace0, \ud669\uadc0\ube44 \uc5c4\uc528\uc758 \ub73b\uc774 \ubc18\uc601\ub418\uc5b4 \uc5c4\uc8fc\uba85\uc758 \ub3d9\ud589\uc740 \uae09\uc791\uc2a4\ub7fd\uac8c \uc774\ub8e8\uc5b4\uc84c\ub2e4. 1907\ub144 12\uc6d4 5\uc77c\uc5d0 \ud55c\uc131\uc744 \ucd9c\ubc1c\ud558\uc5ec 12\uc6d4 15\uc77c\uc5d0 \uc77c\ubcf8 \ub3c4\ucfc4 \uc2e0\ubc14\uc2dc(\u65b0\u6a4b) \uc815\uac70\uc7a5\uc5d0 \ub3c4\ucc29\ud558\uc600\ub2e4. \uc2dc\ubc14 \uc774\uad81\uc5d0\uc11c \uba38\ubb3c\ub2e4\uac00 1908\ub144 2\uc6d4 9\uc77c\uc5d0 \uc544\uc790\ubd80 \ub3c4\ub9ac\uc774\uc790\uce74(\u9ebb\u5e03\u9ce5\u5c45\u5742)\uc758 \uc800\ud0dd\uc73c\ub85c \uac70\ucc98\ub97c \uc62e\uacbc\uc73c\uba70, \uc800\ud0dd \ub0b4\uc5d0 \uc5b4\ud559\ubb38\uc18c\uac00 \uc124\uce58\ub418\uc5b4 \uac1c\uc778 \uad50\uc2b5\uc744 \ubc1b\uc558\ub2e4. \uac19\uc740 \ud574 8\uc6d4\uc5d0 \uc77c\ubcf8 \uae34\ud0a4 \uc9c0\ubc29\uc744 \uc5ec\ud589\ud558\uc600\ub2e4. 1909\ub144 7\uc6d4 27\uc77c\uc5d0 \uba54\uc774\uc9c0 \ucc9c\ud669\uc740 \uc774\ud1a0 \ud788\ub85c\ubd80\ubbf8\ub97c \ud669\ud0dc\uc790\uc758 \uc720\ud559\uc744 \ub2f4\ub2f9\ud558\ub294 \ubcf4\uc721\ucd1d\uc7ac\ub85c \uc784\uba85\ud558\uc600\ub2e4. \uac19\uc740 \ud574 8\uc6d4\uc5d0 \uc77c\ubcf8 \ub3c4\ud638\ucfe0 \uc9c0\ubc29\uc744 \uc5ec\ud589\ud558\uc600\uc73c\uba70, 9\uc6d4 8\uc77c\uc5d0 \ub300\ud55c\uc81c\uad6d \uc721\uad70 \ubcf4\ubcd1 \ubd80\uc704\uc5d0 \uc784\uba85\ub418\uc5c8\ub2e4. 1909\ub144 10\uc6d4 26\uc77c\uc5d0 \uc774\ud1a0 \ud788\ub85c\ubd80\ubbf8\uac00 \uc548\uc911\uadfc\uc5d0 \ud53c\uaca9\ub2f9\ud558\uc5ec \uc0ac\ub9dd\ud558\uc790 \uba54\uc774\uc9c0 \ucc9c\ud669\uc740 \uac19\uc740 \ud574 11\uc6d4 17\uc77c\uc5d0 \uc774\uc640\ucfe0\ub77c \ub3c4\ubaa8\uc0ac\ub2e4\ub97c \ubcf4\uc721\ucd1d\uc7ac\ub85c \uc784\uba85\ud558\uc600\ub2e4. 1910\ub144 5\uc6d4\uc5d0 \ub098\uace0\uc57c\uc5d0 \uac00\uc11c \uacf5\uc9c4\ud68c\ub97c \uad00\ub78c\ud558\uc600\uace0, \uac19\uc740 \ud574 7\uc6d4\uc5d0\ub294 \uc77c\ubcf8 \uc0b0\uc778\ub3c4\uc640 \uc0b0\uc694\ub3c4 \uc9c0\ubc29\uc744 \uc5ec\ud589\ud558\uc600\ub2e4.", "answer": "\ud669\uadc0\ube44 \uc5c4\uc528", "sentence": "\uc5c4\uc8fc\uba85\uc740 \ud669\uadc0\ube44 \uc5c4\uc528 \uc758 \uc870\uce74\uc774\uc790 \uc774\uc740\uacfc \ub098\uc774\ub3c4 \ub3d9\uac11\uc778 \uc808\uce5c\ud55c \uc0ac\uc774\uc5ec\uc11c \uc774\uc740\uc774 \uc5c4\uc8fc\uba85\uc758 \ub3d9\ud589\uc744 \uac04\uccad\ud588\uace0, \ud669\uadc0\ube44 \uc5c4\uc528\uc758 \ub73b\uc774 \ubc18\uc601\ub418\uc5b4 \uc5c4\uc8fc\uba85\uc758 \ub3d9\ud589\uc740 \uae09\uc791\uc2a4\ub7fd\uac8c \uc774\ub8e8\uc5b4\uc84c\ub2e4.", "paragraph_sentence": "\uc774\uc740\uc758 \uc77c\ubcf8 \uc720\ud559\uc5d0 \uacbd\ud559\uc6d0 \ud559\uc0dd \uc870\ub300\ud638\uc640 \uc11c\ubcd1\uac11(\u5f90\u4e19\u7532), \uc5c4\uc8fc\uba85\uc774 \ub3d9\ud589\ud558\uc600\ub2e4. \uc5c4\uc8fc\uba85\uc740 \ud669\uadc0\ube44 \uc5c4\uc528 \uc758 \uc870\uce74\uc774\uc790 \uc774\uc740\uacfc \ub098\uc774\ub3c4 \ub3d9\uac11\uc778 \uc808\uce5c\ud55c \uc0ac\uc774\uc5ec\uc11c \uc774\uc740\uc774 \uc5c4\uc8fc\uba85\uc758 \ub3d9\ud589\uc744 \uac04\uccad\ud588\uace0, \ud669\uadc0\ube44 \uc5c4\uc528\uc758 \ub73b\uc774 \ubc18\uc601\ub418\uc5b4 \uc5c4\uc8fc\uba85\uc758 \ub3d9\ud589\uc740 \uae09\uc791\uc2a4\ub7fd\uac8c \uc774\ub8e8\uc5b4\uc84c\ub2e4. 1907\ub144 12\uc6d4 5\uc77c\uc5d0 \ud55c\uc131\uc744 \ucd9c\ubc1c\ud558\uc5ec 12\uc6d4 15\uc77c\uc5d0 \uc77c\ubcf8 \ub3c4\ucfc4 \uc2e0\ubc14\uc2dc(\u65b0\u6a4b) \uc815\uac70\uc7a5\uc5d0 \ub3c4\ucc29\ud558\uc600\ub2e4. \uc2dc\ubc14 \uc774\uad81\uc5d0\uc11c \uba38\ubb3c\ub2e4\uac00 1908\ub144 2\uc6d4 9\uc77c\uc5d0 \uc544\uc790\ubd80 \ub3c4\ub9ac\uc774\uc790\uce74(\u9ebb\u5e03\u9ce5\u5c45\u5742)\uc758 \uc800\ud0dd\uc73c\ub85c \uac70\ucc98\ub97c \uc62e\uacbc\uc73c\uba70, \uc800\ud0dd \ub0b4\uc5d0 \uc5b4\ud559\ubb38\uc18c\uac00 \uc124\uce58\ub418\uc5b4 \uac1c\uc778 \uad50\uc2b5\uc744 \ubc1b\uc558\ub2e4. \uac19\uc740 \ud574 8\uc6d4\uc5d0 \uc77c\ubcf8 \uae34\ud0a4 \uc9c0\ubc29\uc744 \uc5ec\ud589\ud558\uc600\ub2e4. 1909\ub144 7\uc6d4 27\uc77c\uc5d0 \uba54\uc774\uc9c0 \ucc9c\ud669\uc740 \uc774\ud1a0 \ud788\ub85c\ubd80\ubbf8\ub97c \ud669\ud0dc\uc790\uc758 \uc720\ud559\uc744 \ub2f4\ub2f9\ud558\ub294 \ubcf4\uc721\ucd1d\uc7ac\ub85c \uc784\uba85\ud558\uc600\ub2e4. \uac19\uc740 \ud574 8\uc6d4\uc5d0 \uc77c\ubcf8 \ub3c4\ud638\ucfe0 \uc9c0\ubc29\uc744 \uc5ec\ud589\ud558\uc600\uc73c\uba70, 9\uc6d4 8\uc77c\uc5d0 \ub300\ud55c\uc81c\uad6d \uc721\uad70 \ubcf4\ubcd1 \ubd80\uc704\uc5d0 \uc784\uba85\ub418\uc5c8\ub2e4. 1909\ub144 10\uc6d4 26\uc77c\uc5d0 \uc774\ud1a0 \ud788\ub85c\ubd80\ubbf8\uac00 \uc548\uc911\uadfc\uc5d0 \ud53c\uaca9\ub2f9\ud558\uc5ec \uc0ac\ub9dd\ud558\uc790 \uba54\uc774\uc9c0 \ucc9c\ud669\uc740 \uac19\uc740 \ud574 11\uc6d4 17\uc77c\uc5d0 \uc774\uc640\ucfe0\ub77c \ub3c4\ubaa8\uc0ac\ub2e4\ub97c \ubcf4\uc721\ucd1d\uc7ac\ub85c \uc784\uba85\ud558\uc600\ub2e4. 1910\ub144 5\uc6d4\uc5d0 \ub098\uace0\uc57c\uc5d0 \uac00\uc11c \uacf5\uc9c4\ud68c\ub97c \uad00\ub78c\ud558\uc600\uace0, \uac19\uc740 \ud574 7\uc6d4\uc5d0\ub294 \uc77c\ubcf8 \uc0b0\uc778\ub3c4\uc640 \uc0b0\uc694\ub3c4 \uc9c0\ubc29\uc744 \uc5ec\ud589\ud558\uc600\ub2e4.", "paragraph_answer": "\uc774\uc740\uc758 \uc77c\ubcf8 \uc720\ud559\uc5d0 \uacbd\ud559\uc6d0 \ud559\uc0dd \uc870\ub300\ud638\uc640 \uc11c\ubcd1\uac11(\u5f90\u4e19\u7532), \uc5c4\uc8fc\uba85\uc774 \ub3d9\ud589\ud558\uc600\ub2e4. \uc5c4\uc8fc\uba85\uc740 \ud669\uadc0\ube44 \uc5c4\uc528 \uc758 \uc870\uce74\uc774\uc790 \uc774\uc740\uacfc \ub098\uc774\ub3c4 \ub3d9\uac11\uc778 \uc808\uce5c\ud55c \uc0ac\uc774\uc5ec\uc11c \uc774\uc740\uc774 \uc5c4\uc8fc\uba85\uc758 \ub3d9\ud589\uc744 \uac04\uccad\ud588\uace0, \ud669\uadc0\ube44 \uc5c4\uc528\uc758 \ub73b\uc774 \ubc18\uc601\ub418\uc5b4 \uc5c4\uc8fc\uba85\uc758 \ub3d9\ud589\uc740 \uae09\uc791\uc2a4\ub7fd\uac8c \uc774\ub8e8\uc5b4\uc84c\ub2e4. 1907\ub144 12\uc6d4 5\uc77c\uc5d0 \ud55c\uc131\uc744 \ucd9c\ubc1c\ud558\uc5ec 12\uc6d4 15\uc77c\uc5d0 \uc77c\ubcf8 \ub3c4\ucfc4 \uc2e0\ubc14\uc2dc(\u65b0\u6a4b) \uc815\uac70\uc7a5\uc5d0 \ub3c4\ucc29\ud558\uc600\ub2e4. \uc2dc\ubc14 \uc774\uad81\uc5d0\uc11c \uba38\ubb3c\ub2e4\uac00 1908\ub144 2\uc6d4 9\uc77c\uc5d0 \uc544\uc790\ubd80 \ub3c4\ub9ac\uc774\uc790\uce74(\u9ebb\u5e03\u9ce5\u5c45\u5742)\uc758 \uc800\ud0dd\uc73c\ub85c \uac70\ucc98\ub97c \uc62e\uacbc\uc73c\uba70, \uc800\ud0dd \ub0b4\uc5d0 \uc5b4\ud559\ubb38\uc18c\uac00 \uc124\uce58\ub418\uc5b4 \uac1c\uc778 \uad50\uc2b5\uc744 \ubc1b\uc558\ub2e4. \uac19\uc740 \ud574 8\uc6d4\uc5d0 \uc77c\ubcf8 \uae34\ud0a4 \uc9c0\ubc29\uc744 \uc5ec\ud589\ud558\uc600\ub2e4. 1909\ub144 7\uc6d4 27\uc77c\uc5d0 \uba54\uc774\uc9c0 \ucc9c\ud669\uc740 \uc774\ud1a0 \ud788\ub85c\ubd80\ubbf8\ub97c \ud669\ud0dc\uc790\uc758 \uc720\ud559\uc744 \ub2f4\ub2f9\ud558\ub294 \ubcf4\uc721\ucd1d\uc7ac\ub85c \uc784\uba85\ud558\uc600\ub2e4. \uac19\uc740 \ud574 8\uc6d4\uc5d0 \uc77c\ubcf8 \ub3c4\ud638\ucfe0 \uc9c0\ubc29\uc744 \uc5ec\ud589\ud558\uc600\uc73c\uba70, 9\uc6d4 8\uc77c\uc5d0 \ub300\ud55c\uc81c\uad6d \uc721\uad70 \ubcf4\ubcd1 \ubd80\uc704\uc5d0 \uc784\uba85\ub418\uc5c8\ub2e4. 1909\ub144 10\uc6d4 26\uc77c\uc5d0 \uc774\ud1a0 \ud788\ub85c\ubd80\ubbf8\uac00 \uc548\uc911\uadfc\uc5d0 \ud53c\uaca9\ub2f9\ud558\uc5ec \uc0ac\ub9dd\ud558\uc790 \uba54\uc774\uc9c0 \ucc9c\ud669\uc740 \uac19\uc740 \ud574 11\uc6d4 17\uc77c\uc5d0 \uc774\uc640\ucfe0\ub77c \ub3c4\ubaa8\uc0ac\ub2e4\ub97c \ubcf4\uc721\ucd1d\uc7ac\ub85c \uc784\uba85\ud558\uc600\ub2e4. 1910\ub144 5\uc6d4\uc5d0 \ub098\uace0\uc57c\uc5d0 \uac00\uc11c \uacf5\uc9c4\ud68c\ub97c \uad00\ub78c\ud558\uc600\uace0, \uac19\uc740 \ud574 7\uc6d4\uc5d0\ub294 \uc77c\ubcf8 \uc0b0\uc778\ub3c4\uc640 \uc0b0\uc694\ub3c4 \uc9c0\ubc29\uc744 \uc5ec\ud589\ud558\uc600\ub2e4.", "sentence_answer": "\uc5c4\uc8fc\uba85\uc740 \ud669\uadc0\ube44 \uc5c4\uc528 \uc758 \uc870\uce74\uc774\uc790 \uc774\uc740\uacfc \ub098\uc774\ub3c4 \ub3d9\uac11\uc778 \uc808\uce5c\ud55c \uc0ac\uc774\uc5ec\uc11c \uc774\uc740\uc774 \uc5c4\uc8fc\uba85\uc758 \ub3d9\ud589\uc744 \uac04\uccad\ud588\uace0, \ud669\uadc0\ube44 \uc5c4\uc528\uc758 \ub73b\uc774 \ubc18\uc601\ub418\uc5b4 \uc5c4\uc8fc\uba85\uc758 \ub3d9\ud589\uc740 \uae09\uc791\uc2a4\ub7fd\uac8c \uc774\ub8e8\uc5b4\uc84c\ub2e4.", "paragraph_id": "5836625-3-0", "paragraph_question": "question: \uc5c4\uc8fc\uba85\uc740 \ub204\uad6c\uc758 \uc870\uce74\uc778\uac00?, context: \uc774\uc740\uc758 \uc77c\ubcf8 \uc720\ud559\uc5d0 \uacbd\ud559\uc6d0 \ud559\uc0dd \uc870\ub300\ud638\uc640 \uc11c\ubcd1\uac11(\u5f90\u4e19\u7532), \uc5c4\uc8fc\uba85\uc774 \ub3d9\ud589\ud558\uc600\ub2e4. \uc5c4\uc8fc\uba85\uc740 \ud669\uadc0\ube44 \uc5c4\uc528\uc758 \uc870\uce74\uc774\uc790 \uc774\uc740\uacfc \ub098\uc774\ub3c4 \ub3d9\uac11\uc778 \uc808\uce5c\ud55c \uc0ac\uc774\uc5ec\uc11c \uc774\uc740\uc774 \uc5c4\uc8fc\uba85\uc758 \ub3d9\ud589\uc744 \uac04\uccad\ud588\uace0, \ud669\uadc0\ube44 \uc5c4\uc528\uc758 \ub73b\uc774 \ubc18\uc601\ub418\uc5b4 \uc5c4\uc8fc\uba85\uc758 \ub3d9\ud589\uc740 \uae09\uc791\uc2a4\ub7fd\uac8c \uc774\ub8e8\uc5b4\uc84c\ub2e4. 1907\ub144 12\uc6d4 5\uc77c\uc5d0 \ud55c\uc131\uc744 \ucd9c\ubc1c\ud558\uc5ec 12\uc6d4 15\uc77c\uc5d0 \uc77c\ubcf8 \ub3c4\ucfc4 \uc2e0\ubc14\uc2dc(\u65b0\u6a4b) \uc815\uac70\uc7a5\uc5d0 \ub3c4\ucc29\ud558\uc600\ub2e4. \uc2dc\ubc14 \uc774\uad81\uc5d0\uc11c \uba38\ubb3c\ub2e4\uac00 1908\ub144 2\uc6d4 9\uc77c\uc5d0 \uc544\uc790\ubd80 \ub3c4\ub9ac\uc774\uc790\uce74(\u9ebb\u5e03\u9ce5\u5c45\u5742)\uc758 \uc800\ud0dd\uc73c\ub85c \uac70\ucc98\ub97c \uc62e\uacbc\uc73c\uba70, \uc800\ud0dd \ub0b4\uc5d0 \uc5b4\ud559\ubb38\uc18c\uac00 \uc124\uce58\ub418\uc5b4 \uac1c\uc778 \uad50\uc2b5\uc744 \ubc1b\uc558\ub2e4. \uac19\uc740 \ud574 8\uc6d4\uc5d0 \uc77c\ubcf8 \uae34\ud0a4 \uc9c0\ubc29\uc744 \uc5ec\ud589\ud558\uc600\ub2e4. 1909\ub144 7\uc6d4 27\uc77c\uc5d0 \uba54\uc774\uc9c0 \ucc9c\ud669\uc740 \uc774\ud1a0 \ud788\ub85c\ubd80\ubbf8\ub97c \ud669\ud0dc\uc790\uc758 \uc720\ud559\uc744 \ub2f4\ub2f9\ud558\ub294 \ubcf4\uc721\ucd1d\uc7ac\ub85c \uc784\uba85\ud558\uc600\ub2e4. \uac19\uc740 \ud574 8\uc6d4\uc5d0 \uc77c\ubcf8 \ub3c4\ud638\ucfe0 \uc9c0\ubc29\uc744 \uc5ec\ud589\ud558\uc600\uc73c\uba70, 9\uc6d4 8\uc77c\uc5d0 \ub300\ud55c\uc81c\uad6d \uc721\uad70 \ubcf4\ubcd1 \ubd80\uc704\uc5d0 \uc784\uba85\ub418\uc5c8\ub2e4. 1909\ub144 10\uc6d4 26\uc77c\uc5d0 \uc774\ud1a0 \ud788\ub85c\ubd80\ubbf8\uac00 \uc548\uc911\uadfc\uc5d0 \ud53c\uaca9\ub2f9\ud558\uc5ec \uc0ac\ub9dd\ud558\uc790 \uba54\uc774\uc9c0 \ucc9c\ud669\uc740 \uac19\uc740 \ud574 11\uc6d4 17\uc77c\uc5d0 \uc774\uc640\ucfe0\ub77c \ub3c4\ubaa8\uc0ac\ub2e4\ub97c \ubcf4\uc721\ucd1d\uc7ac\ub85c \uc784\uba85\ud558\uc600\ub2e4. 1910\ub144 5\uc6d4\uc5d0 \ub098\uace0\uc57c\uc5d0 \uac00\uc11c \uacf5\uc9c4\ud68c\ub97c \uad00\ub78c\ud558\uc600\uace0, \uac19\uc740 \ud574 7\uc6d4\uc5d0\ub294 \uc77c\ubcf8 \uc0b0\uc778\ub3c4\uc640 \uc0b0\uc694\ub3c4 \uc9c0\ubc29\uc744 \uc5ec\ud589\ud558\uc600\ub2e4."} {"question": "\ub85c\uc774\ud0b4\uc5d0\uac8c APAN \uc2a4\ud0c0 \uc5b4\uc6cc\uc988 \ubca0\uc2a4\ud2b8 OST\uc0c1\uc744 \uc548\uaca8\uc900 \ub4dc\ub77c\ub9c8 \uc81c\ubaa9\uc740?", "paragraph": "\uc815\uc2dd \ub370\ubdd4\uace1 <\ubd04\ubd04\ubd04>\uc744 \ud3ec\ud568\ud558\uc5ec 2013\ub144 6\uc6d4 \ubc1c\ud45c\ud55c \uc815\uaddc 1\uc9d1 Love Love Love\ub85c \u300aMnet \uc544\uc2dc\uc548 \ubba4\uc9c1 \uc5b4\uc6cc\ub4dc\u300b\uc640 \u300a\uace8\ub4e0 \ub514\uc2a4\ud06c\u300b\uc5d0\uc11c \uadf8\ud574 \ub0a8\uc790 \uc2e0\uc778\uc0c1\ub4f1\uc744 \uc218\uc0c1 \ud588\uc73c\uba70, 2014\ub144 2\uc9d1 Home\uc5d0 \uc774\uc5b4 2015\ub144 3\uc9d1 \ubd81\ub450\uce60\uc131, 2017\ub144 EP \uac1c\ud654\uae30(\u958b\u82b1\u671f)\ub4f1 \uc790\uc791\uace1 \uc815\uaddc\uc74c\ubc18 3\uc7a5\uacfc EP 1\uc7a5\uc744 \ubc1c\ud45c\ud558\uc600\uc73c\uba70, 2018\ub144 <\uadf8\ub54c \ud5e4\uc5b4\uc9c0\uba74 \ub3fc>\ub97c \ube44\ub86f \ub2e4\uc218\uc758 \uc2f1\uae00\ub3c4 \ubc1c\ud45c\ud558\uc600\ub2e4. 2015\ub144 \ub4dc\ub77c\ub9c8 \ud53c\ub178\ud0a4\uc624\uc758 OST \ud53c\ub178\ud0a4\uc624\ub85c \ub300\ub9cc Hito \ubba4\uc9c1 \uc5b4\uc6cc\uc988\uc5d0\uc11c \ud574\uc678 \uc544\ud2f0\uc2a4\ud2b8 \uc778\uae30\uc0c1\uacfc APAN \uc2a4\ud0c0 \uc5b4\uc6cc\uc988 \ubca0\uc2a4\ud2b8 OST\uc0c1\uc744 \uc218\uc0c1\ud558\uc600\uc73c\uba70, 2016\ub144\uc5d0\ub294 \ub300\ub9cc \ud788\ud2b8\uc1a1 \u5929\u9ed1\u9ed1(\ucc9c\ud751\ud751)\ub97c \uc911\uad6d\uc5b4\ub85c \ub9ac\uba54\ud06c\ud55c \ub97c \ubc1c\ud45c\ud558\uace0 \ub300\ub9cc \ub2e8\ub3c5\ucf58\uc11c\ud2b8\uc640 \ub9d0\ub808\uc774\uc2dc\uc544 \ud504\ub85c\ubaa8\uc158 \ud22c\uc5b4\ub97c \uac1c\ucd5c\ud558\uc600\ub2e4.. 2018\ub144 \ub9ac\ud328\ud0a4\uc9c0\uc568\ubc94 \u300aROY KIM BEST\u300b\ub85c \uc77c\ubcf8 \uc815\uc2dd \ub370\ubdd4\ub97c \uc55e\ub450\uace0 \uc788\ub2e4. \uc568\ubc94, \uad11\uace0\uc74c\uc545, \ub85c\uace0\uc1a1, \ucea0\ud398\uc778\uc1a1, OST\ub4f1\uacfc \ub354\ubd88\uc5b4 \ud65c\ubc1c\ud55c \ucf58\uc11c\ud2b8 \ud65c\ub3d9\uc744 \ud3bc\uce58\uace0 \uc788\uc73c\uba70 \ud604\uc7ac \uc870\uc9c0\ud0c0\uc6b4 \ub300\ud559\uad50\uc5d0 \uc7ac\ud559 \uc911\uc73c\ub85c \uc74c\uc545 \ud65c\ub3d9\uacfc \ud559\uc5c5\uc744 \ubcd1\ud589\ud558\uace0 \uc788\ub2e4.", "answer": "\ud53c\ub178\ud0a4\uc624", "sentence": "2015\ub144 \ub4dc\ub77c\ub9c8 \ud53c\ub178\ud0a4\uc624 \uc758 OST \ud53c\ub178\ud0a4\uc624\ub85c \ub300\ub9cc Hito \ubba4\uc9c1 \uc5b4\uc6cc\uc988\uc5d0\uc11c \ud574\uc678 \uc544\ud2f0\uc2a4\ud2b8 \uc778\uae30\uc0c1\uacfc APAN \uc2a4\ud0c0 \uc5b4\uc6cc\uc988 \ubca0\uc2a4\ud2b8 OST\uc0c1\uc744 \uc218\uc0c1\ud558\uc600\uc73c\uba70, 2016\ub144\uc5d0\ub294 \ub300\ub9cc \ud788\ud2b8\uc1a1 \u5929\u9ed1\u9ed1(\ucc9c\ud751\ud751)\ub97c \uc911\uad6d\uc5b4\ub85c \ub9ac\uba54\ud06c\ud55c \ub97c \ubc1c\ud45c\ud558\uace0 \ub300\ub9cc \ub2e8\ub3c5\ucf58\uc11c\ud2b8\uc640 \ub9d0\ub808\uc774\uc2dc\uc544 \ud504\ub85c\ubaa8\uc158 \ud22c\uc5b4\ub97c \uac1c\ucd5c\ud558\uc600\ub2e4.. 2018\ub144 \ub9ac\ud328\ud0a4\uc9c0\uc568\ubc94 \u300aROY KIM BEST\u300b\ub85c \uc77c\ubcf8 \uc815\uc2dd \ub370\ubdd4\ub97c \uc55e\ub450\uace0 \uc788\ub2e4.", "paragraph_sentence": "\uc815\uc2dd \ub370\ubdd4\uace1 <\ubd04\ubd04\ubd04>\uc744 \ud3ec\ud568\ud558\uc5ec 2013\ub144 6\uc6d4 \ubc1c\ud45c\ud55c \uc815\uaddc 1\uc9d1 Love Love Love\ub85c \u300aMnet \uc544\uc2dc\uc548 \ubba4\uc9c1 \uc5b4\uc6cc\ub4dc\u300b\uc640 \u300a\uace8\ub4e0 \ub514\uc2a4\ud06c\u300b\uc5d0\uc11c \uadf8\ud574 \ub0a8\uc790 \uc2e0\uc778\uc0c1\ub4f1\uc744 \uc218\uc0c1 \ud588\uc73c\uba70, 2014\ub144 2\uc9d1 Home\uc5d0 \uc774\uc5b4 2015\ub144 3\uc9d1 \ubd81\ub450\uce60\uc131, 2017\ub144 EP \uac1c\ud654\uae30(\u958b\u82b1\u671f)\ub4f1 \uc790\uc791\uace1 \uc815\uaddc\uc74c\ubc18 3\uc7a5\uacfc EP 1\uc7a5\uc744 \ubc1c\ud45c\ud558\uc600\uc73c\uba70, 2018\ub144 <\uadf8\ub54c \ud5e4\uc5b4\uc9c0\uba74 \ub3fc>\ub97c \ube44\ub86f \ub2e4\uc218\uc758 \uc2f1\uae00\ub3c4 \ubc1c\ud45c\ud558\uc600\ub2e4. 2015\ub144 \ub4dc\ub77c\ub9c8 \ud53c\ub178\ud0a4\uc624 \uc758 OST \ud53c\ub178\ud0a4\uc624\ub85c \ub300\ub9cc Hito \ubba4\uc9c1 \uc5b4\uc6cc\uc988\uc5d0\uc11c \ud574\uc678 \uc544\ud2f0\uc2a4\ud2b8 \uc778\uae30\uc0c1\uacfc APAN \uc2a4\ud0c0 \uc5b4\uc6cc\uc988 \ubca0\uc2a4\ud2b8 OST\uc0c1\uc744 \uc218\uc0c1\ud558\uc600\uc73c\uba70, 2016\ub144\uc5d0\ub294 \ub300\ub9cc \ud788\ud2b8\uc1a1 \u5929\u9ed1\u9ed1(\ucc9c\ud751\ud751)\ub97c \uc911\uad6d\uc5b4\ub85c \ub9ac\uba54\ud06c\ud55c \ub97c \ubc1c\ud45c\ud558\uace0 \ub300\ub9cc \ub2e8\ub3c5\ucf58\uc11c\ud2b8\uc640 \ub9d0\ub808\uc774\uc2dc\uc544 \ud504\ub85c\ubaa8\uc158 \ud22c\uc5b4\ub97c \uac1c\ucd5c\ud558\uc600\ub2e4.. 2018\ub144 \ub9ac\ud328\ud0a4\uc9c0\uc568\ubc94 \u300aROY KIM BEST\u300b\ub85c \uc77c\ubcf8 \uc815\uc2dd \ub370\ubdd4\ub97c \uc55e\ub450\uace0 \uc788\ub2e4. \uc568\ubc94, \uad11\uace0\uc74c\uc545, \ub85c\uace0\uc1a1, \ucea0\ud398\uc778\uc1a1, OST\ub4f1\uacfc \ub354\ubd88\uc5b4 \ud65c\ubc1c\ud55c \ucf58\uc11c\ud2b8 \ud65c\ub3d9\uc744 \ud3bc\uce58\uace0 \uc788\uc73c\uba70 \ud604\uc7ac \uc870\uc9c0\ud0c0\uc6b4 \ub300\ud559\uad50\uc5d0 \uc7ac\ud559 \uc911\uc73c\ub85c \uc74c\uc545 \ud65c\ub3d9\uacfc \ud559\uc5c5\uc744 \ubcd1\ud589\ud558\uace0 \uc788\ub2e4.", "paragraph_answer": "\uc815\uc2dd \ub370\ubdd4\uace1 <\ubd04\ubd04\ubd04>\uc744 \ud3ec\ud568\ud558\uc5ec 2013\ub144 6\uc6d4 \ubc1c\ud45c\ud55c \uc815\uaddc 1\uc9d1 Love Love Love\ub85c \u300aMnet \uc544\uc2dc\uc548 \ubba4\uc9c1 \uc5b4\uc6cc\ub4dc\u300b\uc640 \u300a\uace8\ub4e0 \ub514\uc2a4\ud06c\u300b\uc5d0\uc11c \uadf8\ud574 \ub0a8\uc790 \uc2e0\uc778\uc0c1\ub4f1\uc744 \uc218\uc0c1 \ud588\uc73c\uba70, 2014\ub144 2\uc9d1 Home\uc5d0 \uc774\uc5b4 2015\ub144 3\uc9d1 \ubd81\ub450\uce60\uc131, 2017\ub144 EP \uac1c\ud654\uae30(\u958b\u82b1\u671f)\ub4f1 \uc790\uc791\uace1 \uc815\uaddc\uc74c\ubc18 3\uc7a5\uacfc EP 1\uc7a5\uc744 \ubc1c\ud45c\ud558\uc600\uc73c\uba70, 2018\ub144 <\uadf8\ub54c \ud5e4\uc5b4\uc9c0\uba74 \ub3fc>\ub97c \ube44\ub86f \ub2e4\uc218\uc758 \uc2f1\uae00\ub3c4 \ubc1c\ud45c\ud558\uc600\ub2e4. 2015\ub144 \ub4dc\ub77c\ub9c8 \ud53c\ub178\ud0a4\uc624 \uc758 OST \ud53c\ub178\ud0a4\uc624\ub85c \ub300\ub9cc Hito \ubba4\uc9c1 \uc5b4\uc6cc\uc988\uc5d0\uc11c \ud574\uc678 \uc544\ud2f0\uc2a4\ud2b8 \uc778\uae30\uc0c1\uacfc APAN \uc2a4\ud0c0 \uc5b4\uc6cc\uc988 \ubca0\uc2a4\ud2b8 OST\uc0c1\uc744 \uc218\uc0c1\ud558\uc600\uc73c\uba70, 2016\ub144\uc5d0\ub294 \ub300\ub9cc \ud788\ud2b8\uc1a1 \u5929\u9ed1\u9ed1(\ucc9c\ud751\ud751)\ub97c \uc911\uad6d\uc5b4\ub85c \ub9ac\uba54\ud06c\ud55c \ub97c \ubc1c\ud45c\ud558\uace0 \ub300\ub9cc \ub2e8\ub3c5\ucf58\uc11c\ud2b8\uc640 \ub9d0\ub808\uc774\uc2dc\uc544 \ud504\ub85c\ubaa8\uc158 \ud22c\uc5b4\ub97c \uac1c\ucd5c\ud558\uc600\ub2e4.. 2018\ub144 \ub9ac\ud328\ud0a4\uc9c0\uc568\ubc94 \u300aROY KIM BEST\u300b\ub85c \uc77c\ubcf8 \uc815\uc2dd \ub370\ubdd4\ub97c \uc55e\ub450\uace0 \uc788\ub2e4. \uc568\ubc94, \uad11\uace0\uc74c\uc545, \ub85c\uace0\uc1a1, \ucea0\ud398\uc778\uc1a1, OST\ub4f1\uacfc \ub354\ubd88\uc5b4 \ud65c\ubc1c\ud55c \ucf58\uc11c\ud2b8 \ud65c\ub3d9\uc744 \ud3bc\uce58\uace0 \uc788\uc73c\uba70 \ud604\uc7ac \uc870\uc9c0\ud0c0\uc6b4 \ub300\ud559\uad50\uc5d0 \uc7ac\ud559 \uc911\uc73c\ub85c \uc74c\uc545 \ud65c\ub3d9\uacfc \ud559\uc5c5\uc744 \ubcd1\ud589\ud558\uace0 \uc788\ub2e4.", "sentence_answer": "2015\ub144 \ub4dc\ub77c\ub9c8 \ud53c\ub178\ud0a4\uc624 \uc758 OST \ud53c\ub178\ud0a4\uc624\ub85c \ub300\ub9cc Hito \ubba4\uc9c1 \uc5b4\uc6cc\uc988\uc5d0\uc11c \ud574\uc678 \uc544\ud2f0\uc2a4\ud2b8 \uc778\uae30\uc0c1\uacfc APAN \uc2a4\ud0c0 \uc5b4\uc6cc\uc988 \ubca0\uc2a4\ud2b8 OST\uc0c1\uc744 \uc218\uc0c1\ud558\uc600\uc73c\uba70, 2016\ub144\uc5d0\ub294 \ub300\ub9cc \ud788\ud2b8\uc1a1 \u5929\u9ed1\u9ed1(\ucc9c\ud751\ud751)\ub97c \uc911\uad6d\uc5b4\ub85c \ub9ac\uba54\ud06c\ud55c \ub97c \ubc1c\ud45c\ud558\uace0 \ub300\ub9cc \ub2e8\ub3c5\ucf58\uc11c\ud2b8\uc640 \ub9d0\ub808\uc774\uc2dc\uc544 \ud504\ub85c\ubaa8\uc158 \ud22c\uc5b4\ub97c \uac1c\ucd5c\ud558\uc600\ub2e4.. 2018\ub144 \ub9ac\ud328\ud0a4\uc9c0\uc568\ubc94 \u300aROY KIM BEST\u300b\ub85c \uc77c\ubcf8 \uc815\uc2dd \ub370\ubdd4\ub97c \uc55e\ub450\uace0 \uc788\ub2e4.", "paragraph_id": "6496334-2-1", "paragraph_question": "question: \ub85c\uc774\ud0b4\uc5d0\uac8c APAN \uc2a4\ud0c0 \uc5b4\uc6cc\uc988 \ubca0\uc2a4\ud2b8 OST\uc0c1\uc744 \uc548\uaca8\uc900 \ub4dc\ub77c\ub9c8 \uc81c\ubaa9\uc740?, context: \uc815\uc2dd \ub370\ubdd4\uace1 <\ubd04\ubd04\ubd04>\uc744 \ud3ec\ud568\ud558\uc5ec 2013\ub144 6\uc6d4 \ubc1c\ud45c\ud55c \uc815\uaddc 1\uc9d1 Love Love Love\ub85c \u300aMnet \uc544\uc2dc\uc548 \ubba4\uc9c1 \uc5b4\uc6cc\ub4dc\u300b\uc640 \u300a\uace8\ub4e0 \ub514\uc2a4\ud06c\u300b\uc5d0\uc11c \uadf8\ud574 \ub0a8\uc790 \uc2e0\uc778\uc0c1\ub4f1\uc744 \uc218\uc0c1 \ud588\uc73c\uba70, 2014\ub144 2\uc9d1 Home\uc5d0 \uc774\uc5b4 2015\ub144 3\uc9d1 \ubd81\ub450\uce60\uc131, 2017\ub144 EP \uac1c\ud654\uae30(\u958b\u82b1\u671f)\ub4f1 \uc790\uc791\uace1 \uc815\uaddc\uc74c\ubc18 3\uc7a5\uacfc EP 1\uc7a5\uc744 \ubc1c\ud45c\ud558\uc600\uc73c\uba70, 2018\ub144 <\uadf8\ub54c \ud5e4\uc5b4\uc9c0\uba74 \ub3fc>\ub97c \ube44\ub86f \ub2e4\uc218\uc758 \uc2f1\uae00\ub3c4 \ubc1c\ud45c\ud558\uc600\ub2e4. 2015\ub144 \ub4dc\ub77c\ub9c8 \ud53c\ub178\ud0a4\uc624\uc758 OST \ud53c\ub178\ud0a4\uc624\ub85c \ub300\ub9cc Hito \ubba4\uc9c1 \uc5b4\uc6cc\uc988\uc5d0\uc11c \ud574\uc678 \uc544\ud2f0\uc2a4\ud2b8 \uc778\uae30\uc0c1\uacfc APAN \uc2a4\ud0c0 \uc5b4\uc6cc\uc988 \ubca0\uc2a4\ud2b8 OST\uc0c1\uc744 \uc218\uc0c1\ud558\uc600\uc73c\uba70, 2016\ub144\uc5d0\ub294 \ub300\ub9cc \ud788\ud2b8\uc1a1 \u5929\u9ed1\u9ed1(\ucc9c\ud751\ud751)\ub97c \uc911\uad6d\uc5b4\ub85c \ub9ac\uba54\ud06c\ud55c \ub97c \ubc1c\ud45c\ud558\uace0 \ub300\ub9cc \ub2e8\ub3c5\ucf58\uc11c\ud2b8\uc640 \ub9d0\ub808\uc774\uc2dc\uc544 \ud504\ub85c\ubaa8\uc158 \ud22c\uc5b4\ub97c \uac1c\ucd5c\ud558\uc600\ub2e4.. 2018\ub144 \ub9ac\ud328\ud0a4\uc9c0\uc568\ubc94 \u300aROY KIM BEST\u300b\ub85c \uc77c\ubcf8 \uc815\uc2dd \ub370\ubdd4\ub97c \uc55e\ub450\uace0 \uc788\ub2e4. \uc568\ubc94, \uad11\uace0\uc74c\uc545, \ub85c\uace0\uc1a1, \ucea0\ud398\uc778\uc1a1, OST\ub4f1\uacfc \ub354\ubd88\uc5b4 \ud65c\ubc1c\ud55c \ucf58\uc11c\ud2b8 \ud65c\ub3d9\uc744 \ud3bc\uce58\uace0 \uc788\uc73c\uba70 \ud604\uc7ac \uc870\uc9c0\ud0c0\uc6b4 \ub300\ud559\uad50\uc5d0 \uc7ac\ud559 \uc911\uc73c\ub85c \uc74c\uc545 \ud65c\ub3d9\uacfc \ud559\uc5c5\uc744 \ubcd1\ud589\ud558\uace0 \uc788\ub2e4."} {"question": "\uac74\ub96d\uc81c \ub54c \uc120\uad50\uc0ac\ub4e4\uc758 \ub3c4\uc6c0\uc73c\ub85c \uccad\ub098\ub77c\uc758 \uac15\uc5ed\uc744 \uc138\ubc00\ud788 \uadf8\ub9b0 \uc11c\uc591\uc2dd \uc9c0\ub3c4\ub294?", "paragraph": "\uac74\ub96d\uc81c\ub294 \uc774\ubbf8 \uc804\uc784 \ud669\uc81c\ub4e4\uc778 \uac15\ud76c\uc81c, \uc639\uc815\uc81c\uc640 \uac19\uc774 \uad81\uc815\uc758 \uc120\uad50\uc0ac\ub4e4\uc744 \uc6b0\ub300\ud558\uba70 \uadf8\ub4e4\uc5d0\uac8c \ubd80\ubd84\uc801\uc778 \uc120\uad50 \ud65c\ub3d9\uc744 \ud5c8\ub77d\ud558\uc600\ub2e4. \ud558\uc9c0\ub9cc \uac74\ub96d\uc81c \uc790\uc2e0\uc740 \uc870\ubd80\uc640 \ubd80\ud669\uacfc \uac19\uc774 \uac00\ud1a8\ub9ad\uad50\ud68c\uc758 \uac00\ub974\uce68\uc5d0\ub294 \uc804\ud600 \uad00\uc2ec\uc774 \uc5c6\uc5c8\uace0 \uc120\uad50\uc0ac\ub4e4\uc758 \uc608\uc220\uc801, \uacfc\ud559\uc801 \uc9c0\uc2dd\ub9cc\uc744 \uc774\uc6a9\ud558\ub824\uace0 \ud558\uc600\ub2e4. \uac74\ub96d\uc81c\uc758 \uc99d\uc870\ubd80\uc778 \uc21c\uce58\uc81c\ubd80\ud130 \uc774\ubbf8 \uc544\ub2f4 \uc0ec \ud3f0 \ubca8\uc744 \uc2dc\ucd08\ub85c \uac74\ub96d\uc81c \ub54c\uc5d0\ub3c4 \uc774\uadf8\ub098\uce20 \ucfa8\uae00\ub7ec, \uc548\ud1a0 \uace0\uac00\uc774\uc2ac \ub4f1\uc758 \uc608\uc218\ud68c \uc120\uad50\uc0ac\ub4e4\uc774 \ucc9c\ubb38\ub300\uc778 \ud760\ucc9c\uac10\uc744 \uc774\ub04c\uba70 \uc911\uad6d\uc758 \ucc9c\ubb38\ud559\uacfc \uc9c0\ub9ac\ud559\uc5d0 \ub3c4\uc6c0\uc744 \uc8fc\uc5c8\ub2e4. 1769\ub144(\uac74\ub96d 34\ub144) \uc774\ub4e4\uc758 \ub3c4\uc6c0\uc73c\ub85c \uc644\uc131\ub41c \u300a\uac74\ub96d\ud669\uc5ec\uc804\ub78c\ub3c4\u300b(\u4e7e\u9686\u7687\u8207\u5168\u89bd\u5716)\ub294 \uc11c\uc591\uc2dd\uc73c\ub85c \uadf8\ub9b0 \uc9c0\ub3c4\ub85c \uccad\ub098\ub77c\uc758 \uac15\uc5ed\uc744 \uc138\ubc00\ud788 \uadf8\ub824\ub0c8\ub2e4. \ud504\ub791\uc2a4 \uc120\uad50\uc0ac\uc778 \uc870\uc81c\ud504\ub9c8\ub9ac \uc544\ubbf8\uc624\ub294 \uc5ec\ub7ec \uac1c\uc758 \ud669\uc2e4 \uc815\uc6d0\uc744 \ud504\ub791\uc2a4\uc2dd\uc73c\ub85c \uafb8\uba70\uc8fc\uace0 \u300a\uc190\uc790\ubcd1\ubc95\u300b\uacfc \uac19\uc740 \uc911\uad6d\uc758 \uc720\uba85\ud55c \uace0\uc11c\uc801\uc774\ub098 \uac74\ub96d\uc81c\uac00 \uc9c0\uc740 \uc2dc \ub4f1\uc744 \ud504\ub791\uc2a4\uc5b4\ub85c \ubc88\uc5ed\ud558\uc5ec \ucd9c\ud310\ud558\uae30\ub3c4 \ud558\uc600\ub2e4.", "answer": "\uac74\ub96d\ud669\uc5ec\uc804\ub78c\ub3c4", "sentence": "1769\ub144(\uac74\ub96d 34\ub144) \uc774\ub4e4\uc758 \ub3c4\uc6c0\uc73c\ub85c \uc644\uc131\ub41c \u300a \uac74\ub96d\ud669\uc5ec\uc804\ub78c\ub3c4 \u300b(\u4e7e\u9686\u7687\u8207\u5168\u89bd\u5716)\ub294 \uc11c\uc591\uc2dd\uc73c\ub85c \uadf8\ub9b0 \uc9c0\ub3c4\ub85c \uccad\ub098\ub77c\uc758 \uac15\uc5ed\uc744 \uc138\ubc00\ud788 \uadf8\ub824\ub0c8\ub2e4.", "paragraph_sentence": "\uac74\ub96d\uc81c\ub294 \uc774\ubbf8 \uc804\uc784 \ud669\uc81c\ub4e4\uc778 \uac15\ud76c\uc81c, \uc639\uc815\uc81c\uc640 \uac19\uc774 \uad81\uc815\uc758 \uc120\uad50\uc0ac\ub4e4\uc744 \uc6b0\ub300\ud558\uba70 \uadf8\ub4e4\uc5d0\uac8c \ubd80\ubd84\uc801\uc778 \uc120\uad50 \ud65c\ub3d9\uc744 \ud5c8\ub77d\ud558\uc600\ub2e4. \ud558\uc9c0\ub9cc \uac74\ub96d\uc81c \uc790\uc2e0\uc740 \uc870\ubd80\uc640 \ubd80\ud669\uacfc \uac19\uc774 \uac00\ud1a8\ub9ad\uad50\ud68c\uc758 \uac00\ub974\uce68\uc5d0\ub294 \uc804\ud600 \uad00\uc2ec\uc774 \uc5c6\uc5c8\uace0 \uc120\uad50\uc0ac\ub4e4\uc758 \uc608\uc220\uc801, \uacfc\ud559\uc801 \uc9c0\uc2dd\ub9cc\uc744 \uc774\uc6a9\ud558\ub824\uace0 \ud558\uc600\ub2e4. \uac74\ub96d\uc81c\uc758 \uc99d\uc870\ubd80\uc778 \uc21c\uce58\uc81c\ubd80\ud130 \uc774\ubbf8 \uc544\ub2f4 \uc0ec \ud3f0 \ubca8\uc744 \uc2dc\ucd08\ub85c \uac74\ub96d\uc81c \ub54c\uc5d0\ub3c4 \uc774\uadf8\ub098\uce20 \ucfa8\uae00\ub7ec, \uc548\ud1a0 \uace0\uac00\uc774\uc2ac \ub4f1\uc758 \uc608\uc218\ud68c \uc120\uad50\uc0ac\ub4e4\uc774 \ucc9c\ubb38\ub300\uc778 \ud760\ucc9c\uac10\uc744 \uc774\ub04c\uba70 \uc911\uad6d\uc758 \ucc9c\ubb38\ud559\uacfc \uc9c0\ub9ac\ud559\uc5d0 \ub3c4\uc6c0\uc744 \uc8fc\uc5c8\ub2e4. 1769\ub144(\uac74\ub96d 34\ub144) \uc774\ub4e4\uc758 \ub3c4\uc6c0\uc73c\ub85c \uc644\uc131\ub41c \u300a \uac74\ub96d\ud669\uc5ec\uc804\ub78c\ub3c4 \u300b(\u4e7e\u9686\u7687\u8207\u5168\u89bd\u5716)\ub294 \uc11c\uc591\uc2dd\uc73c\ub85c \uadf8\ub9b0 \uc9c0\ub3c4\ub85c \uccad\ub098\ub77c\uc758 \uac15\uc5ed\uc744 \uc138\ubc00\ud788 \uadf8\ub824\ub0c8\ub2e4. \ud504\ub791\uc2a4 \uc120\uad50\uc0ac\uc778 \uc870\uc81c\ud504\ub9c8\ub9ac \uc544\ubbf8\uc624\ub294 \uc5ec\ub7ec \uac1c\uc758 \ud669\uc2e4 \uc815\uc6d0\uc744 \ud504\ub791\uc2a4\uc2dd\uc73c\ub85c \uafb8\uba70\uc8fc\uace0 \u300a\uc190\uc790\ubcd1\ubc95\u300b\uacfc \uac19\uc740 \uc911\uad6d\uc758 \uc720\uba85\ud55c \uace0\uc11c\uc801\uc774\ub098 \uac74\ub96d\uc81c\uac00 \uc9c0\uc740 \uc2dc \ub4f1\uc744 \ud504\ub791\uc2a4\uc5b4\ub85c \ubc88\uc5ed\ud558\uc5ec \ucd9c\ud310\ud558\uae30\ub3c4 \ud558\uc600\ub2e4.", "paragraph_answer": "\uac74\ub96d\uc81c\ub294 \uc774\ubbf8 \uc804\uc784 \ud669\uc81c\ub4e4\uc778 \uac15\ud76c\uc81c, \uc639\uc815\uc81c\uc640 \uac19\uc774 \uad81\uc815\uc758 \uc120\uad50\uc0ac\ub4e4\uc744 \uc6b0\ub300\ud558\uba70 \uadf8\ub4e4\uc5d0\uac8c \ubd80\ubd84\uc801\uc778 \uc120\uad50 \ud65c\ub3d9\uc744 \ud5c8\ub77d\ud558\uc600\ub2e4. \ud558\uc9c0\ub9cc \uac74\ub96d\uc81c \uc790\uc2e0\uc740 \uc870\ubd80\uc640 \ubd80\ud669\uacfc \uac19\uc774 \uac00\ud1a8\ub9ad\uad50\ud68c\uc758 \uac00\ub974\uce68\uc5d0\ub294 \uc804\ud600 \uad00\uc2ec\uc774 \uc5c6\uc5c8\uace0 \uc120\uad50\uc0ac\ub4e4\uc758 \uc608\uc220\uc801, \uacfc\ud559\uc801 \uc9c0\uc2dd\ub9cc\uc744 \uc774\uc6a9\ud558\ub824\uace0 \ud558\uc600\ub2e4. \uac74\ub96d\uc81c\uc758 \uc99d\uc870\ubd80\uc778 \uc21c\uce58\uc81c\ubd80\ud130 \uc774\ubbf8 \uc544\ub2f4 \uc0ec \ud3f0 \ubca8\uc744 \uc2dc\ucd08\ub85c \uac74\ub96d\uc81c \ub54c\uc5d0\ub3c4 \uc774\uadf8\ub098\uce20 \ucfa8\uae00\ub7ec, \uc548\ud1a0 \uace0\uac00\uc774\uc2ac \ub4f1\uc758 \uc608\uc218\ud68c \uc120\uad50\uc0ac\ub4e4\uc774 \ucc9c\ubb38\ub300\uc778 \ud760\ucc9c\uac10\uc744 \uc774\ub04c\uba70 \uc911\uad6d\uc758 \ucc9c\ubb38\ud559\uacfc \uc9c0\ub9ac\ud559\uc5d0 \ub3c4\uc6c0\uc744 \uc8fc\uc5c8\ub2e4. 1769\ub144(\uac74\ub96d 34\ub144) \uc774\ub4e4\uc758 \ub3c4\uc6c0\uc73c\ub85c \uc644\uc131\ub41c \u300a \uac74\ub96d\ud669\uc5ec\uc804\ub78c\ub3c4 \u300b(\u4e7e\u9686\u7687\u8207\u5168\u89bd\u5716)\ub294 \uc11c\uc591\uc2dd\uc73c\ub85c \uadf8\ub9b0 \uc9c0\ub3c4\ub85c \uccad\ub098\ub77c\uc758 \uac15\uc5ed\uc744 \uc138\ubc00\ud788 \uadf8\ub824\ub0c8\ub2e4. \ud504\ub791\uc2a4 \uc120\uad50\uc0ac\uc778 \uc870\uc81c\ud504\ub9c8\ub9ac \uc544\ubbf8\uc624\ub294 \uc5ec\ub7ec \uac1c\uc758 \ud669\uc2e4 \uc815\uc6d0\uc744 \ud504\ub791\uc2a4\uc2dd\uc73c\ub85c \uafb8\uba70\uc8fc\uace0 \u300a\uc190\uc790\ubcd1\ubc95\u300b\uacfc \uac19\uc740 \uc911\uad6d\uc758 \uc720\uba85\ud55c \uace0\uc11c\uc801\uc774\ub098 \uac74\ub96d\uc81c\uac00 \uc9c0\uc740 \uc2dc \ub4f1\uc744 \ud504\ub791\uc2a4\uc5b4\ub85c \ubc88\uc5ed\ud558\uc5ec \ucd9c\ud310\ud558\uae30\ub3c4 \ud558\uc600\ub2e4.", "sentence_answer": "1769\ub144(\uac74\ub96d 34\ub144) \uc774\ub4e4\uc758 \ub3c4\uc6c0\uc73c\ub85c \uc644\uc131\ub41c \u300a \uac74\ub96d\ud669\uc5ec\uc804\ub78c\ub3c4 \u300b(\u4e7e\u9686\u7687\u8207\u5168\u89bd\u5716)\ub294 \uc11c\uc591\uc2dd\uc73c\ub85c \uadf8\ub9b0 \uc9c0\ub3c4\ub85c \uccad\ub098\ub77c\uc758 \uac15\uc5ed\uc744 \uc138\ubc00\ud788 \uadf8\ub824\ub0c8\ub2e4.", "paragraph_id": "6486904-17-0", "paragraph_question": "question: \uac74\ub96d\uc81c \ub54c \uc120\uad50\uc0ac\ub4e4\uc758 \ub3c4\uc6c0\uc73c\ub85c \uccad\ub098\ub77c\uc758 \uac15\uc5ed\uc744 \uc138\ubc00\ud788 \uadf8\ub9b0 \uc11c\uc591\uc2dd \uc9c0\ub3c4\ub294?, context: \uac74\ub96d\uc81c\ub294 \uc774\ubbf8 \uc804\uc784 \ud669\uc81c\ub4e4\uc778 \uac15\ud76c\uc81c, \uc639\uc815\uc81c\uc640 \uac19\uc774 \uad81\uc815\uc758 \uc120\uad50\uc0ac\ub4e4\uc744 \uc6b0\ub300\ud558\uba70 \uadf8\ub4e4\uc5d0\uac8c \ubd80\ubd84\uc801\uc778 \uc120\uad50 \ud65c\ub3d9\uc744 \ud5c8\ub77d\ud558\uc600\ub2e4. \ud558\uc9c0\ub9cc \uac74\ub96d\uc81c \uc790\uc2e0\uc740 \uc870\ubd80\uc640 \ubd80\ud669\uacfc \uac19\uc774 \uac00\ud1a8\ub9ad\uad50\ud68c\uc758 \uac00\ub974\uce68\uc5d0\ub294 \uc804\ud600 \uad00\uc2ec\uc774 \uc5c6\uc5c8\uace0 \uc120\uad50\uc0ac\ub4e4\uc758 \uc608\uc220\uc801, \uacfc\ud559\uc801 \uc9c0\uc2dd\ub9cc\uc744 \uc774\uc6a9\ud558\ub824\uace0 \ud558\uc600\ub2e4. \uac74\ub96d\uc81c\uc758 \uc99d\uc870\ubd80\uc778 \uc21c\uce58\uc81c\ubd80\ud130 \uc774\ubbf8 \uc544\ub2f4 \uc0ec \ud3f0 \ubca8\uc744 \uc2dc\ucd08\ub85c \uac74\ub96d\uc81c \ub54c\uc5d0\ub3c4 \uc774\uadf8\ub098\uce20 \ucfa8\uae00\ub7ec, \uc548\ud1a0 \uace0\uac00\uc774\uc2ac \ub4f1\uc758 \uc608\uc218\ud68c \uc120\uad50\uc0ac\ub4e4\uc774 \ucc9c\ubb38\ub300\uc778 \ud760\ucc9c\uac10\uc744 \uc774\ub04c\uba70 \uc911\uad6d\uc758 \ucc9c\ubb38\ud559\uacfc \uc9c0\ub9ac\ud559\uc5d0 \ub3c4\uc6c0\uc744 \uc8fc\uc5c8\ub2e4. 1769\ub144(\uac74\ub96d 34\ub144) \uc774\ub4e4\uc758 \ub3c4\uc6c0\uc73c\ub85c \uc644\uc131\ub41c \u300a\uac74\ub96d\ud669\uc5ec\uc804\ub78c\ub3c4\u300b(\u4e7e\u9686\u7687\u8207\u5168\u89bd\u5716)\ub294 \uc11c\uc591\uc2dd\uc73c\ub85c \uadf8\ub9b0 \uc9c0\ub3c4\ub85c \uccad\ub098\ub77c\uc758 \uac15\uc5ed\uc744 \uc138\ubc00\ud788 \uadf8\ub824\ub0c8\ub2e4. \ud504\ub791\uc2a4 \uc120\uad50\uc0ac\uc778 \uc870\uc81c\ud504\ub9c8\ub9ac \uc544\ubbf8\uc624\ub294 \uc5ec\ub7ec \uac1c\uc758 \ud669\uc2e4 \uc815\uc6d0\uc744 \ud504\ub791\uc2a4\uc2dd\uc73c\ub85c \uafb8\uba70\uc8fc\uace0 \u300a\uc190\uc790\ubcd1\ubc95\u300b\uacfc \uac19\uc740 \uc911\uad6d\uc758 \uc720\uba85\ud55c \uace0\uc11c\uc801\uc774\ub098 \uac74\ub96d\uc81c\uac00 \uc9c0\uc740 \uc2dc \ub4f1\uc744 \ud504\ub791\uc2a4\uc5b4\ub85c \ubc88\uc5ed\ud558\uc5ec \ucd9c\ud310\ud558\uae30\ub3c4 \ud558\uc600\ub2e4."} {"question": "1680\ub144 \uc219\uc885\uc774 \uad8c\ub825\uc758 \ud575\uc2ec\uc744 \uc7a5\uc545\ud55c \uacf3\uc740?", "paragraph": "\uadf8\ub7ec\ub098 \uc218\uc138\uc5d0 \ubab0\ub838\ub358 \uc11c\uc778\uc740 1680\ub144(\uc219\uc885 6\ub144) \ub0a8\uc778 \uc601\uc218 \ud5c8\uc801\uc774 \ub300\ud765\uc0b0\uc131\uc758 \uad70\uc778\uc744 \ub3d9\uc6d0\ud574 \uc5ed\ubaa8\ub97c \uafb8\uba84\ub2e4\uace0 \uace0\ubc1c\ud558\uc5ec, \ud5c8\uc801\u00b7\uc724\ud734 \ub4f1\uc744 \uc0ac\ud615\uc2dc\ud0a4\uace0 \ub098\uba38\uc9c0 \ub0a8\uc778\ub4e4\ub3c4 \ucd95\ucd9c\ud588\ub2e4. \uc774 \uc0ac\uac74\uc744 \u2018\uacbd\uc2e0\ud658\uad6d\u2019\uc774\ub77c \ud55c\ub2e4. \uc774 \ubb34\ub835 \uc11c\uc778\uc740 \uc790\uccb4 \ubd84\uc5f4\uc744 \uc77c\uc73c\ucf1c \uc1a1\uc2dc\uc5f4\uc744 \uc601\uc218\ub85c \ud558\ub294 \ub178\ub860\uacfc \uc724\uc99d\uc744 \uc911\uc2ec\uc73c\ub85c \ud558\ub294 \uc18c\ub860\uc73c\ub85c \uac08\ub77c\uc84c\ub2e4(1683\ub144). \ub178\ub860\uc740 \ub300\uc758\uba85\ubd84\uc744 \uc874\uc911\ud558\uace0, \ub0b4\uc218\uc678\uc591, \uc989 \ubbfc\uc0dd\uc548\uc815\uacfc \uc790\uce58\uc790\uac15\uc744 \uac15\uc870\ud558\uc600\uc73c\uba70, \uc18c\ub860\uc740 \uc2e4\ub9ac\ub97c \uc911\uc2dc\ud558\uace0 \uc801\uadf9\uc801\uc778 \ubd81\ubc29\uac1c\ucc99\uc744 \uc8fc\uc7a5\ud55c \uc810\uc5d0\uc11c \uc815\ucc45\uc801 \ucc28\uc774\uac00 \uc788\uc5c8\ub2e4. \uc655\uc740 \uc591\ud30c\ub97c \uc5f0\ub9bd\uc2dc\ucf30\uc73c\ub098 \uad8c\ub825\uc758 \ud575\uc2ec\uc744 \uc7a5\uc545\ud55c \uac83\uc740 \ub178\ub860\uc73c\ub85c\uc11c, \uc1a1\uc2dc\uc5f4\uacfc \uc0bc\ucc99\uc73c\ub85c \ubd88\ub9ac\ub358 \uc655\uc2e4\uc758 \uc678\ucc99, \uc989 \uae40\uc11d\uc8fc \u00b7 \uae40\ub9cc\uae30 \u00b7 \ubbfc\uc815\uc911\uc774 \uc5f0\ud569\ud558\uc5ec \uc815\uce58\ub97c \uc8fc\ub3c4\ud588\ub2e4.", "answer": "\ub178\ub860", "sentence": "\uc774 \ubb34\ub835 \uc11c\uc778\uc740 \uc790\uccb4 \ubd84\uc5f4\uc744 \uc77c\uc73c\ucf1c \uc1a1\uc2dc\uc5f4\uc744 \uc601\uc218\ub85c \ud558\ub294 \ub178\ub860 \uacfc \uc724\uc99d\uc744 \uc911\uc2ec\uc73c\ub85c \ud558\ub294 \uc18c\ub860\uc73c\ub85c \uac08\ub77c\uc84c\ub2e4(1683\ub144).", "paragraph_sentence": "\uadf8\ub7ec\ub098 \uc218\uc138\uc5d0 \ubab0\ub838\ub358 \uc11c\uc778\uc740 1680\ub144(\uc219\uc885 6\ub144) \ub0a8\uc778 \uc601\uc218 \ud5c8\uc801\uc774 \ub300\ud765\uc0b0\uc131\uc758 \uad70\uc778\uc744 \ub3d9\uc6d0\ud574 \uc5ed\ubaa8\ub97c \uafb8\uba84\ub2e4\uace0 \uace0\ubc1c\ud558\uc5ec, \ud5c8\uc801\u00b7\uc724\ud734 \ub4f1\uc744 \uc0ac\ud615\uc2dc\ud0a4\uace0 \ub098\uba38\uc9c0 \ub0a8\uc778\ub4e4\ub3c4 \ucd95\ucd9c\ud588\ub2e4. \uc774 \uc0ac\uac74\uc744 \u2018\uacbd\uc2e0\ud658\uad6d\u2019\uc774\ub77c \ud55c\ub2e4. \uc774 \ubb34\ub835 \uc11c\uc778\uc740 \uc790\uccb4 \ubd84\uc5f4\uc744 \uc77c\uc73c\ucf1c \uc1a1\uc2dc\uc5f4\uc744 \uc601\uc218\ub85c \ud558\ub294 \ub178\ub860 \uacfc \uc724\uc99d\uc744 \uc911\uc2ec\uc73c\ub85c \ud558\ub294 \uc18c\ub860\uc73c\ub85c \uac08\ub77c\uc84c\ub2e4(1683\ub144). \ub178\ub860\uc740 \ub300\uc758\uba85\ubd84\uc744 \uc874\uc911\ud558\uace0, \ub0b4\uc218\uc678\uc591, \uc989 \ubbfc\uc0dd\uc548\uc815\uacfc \uc790\uce58\uc790\uac15\uc744 \uac15\uc870\ud558\uc600\uc73c\uba70, \uc18c\ub860\uc740 \uc2e4\ub9ac\ub97c \uc911\uc2dc\ud558\uace0 \uc801\uadf9\uc801\uc778 \ubd81\ubc29\uac1c\ucc99\uc744 \uc8fc\uc7a5\ud55c \uc810\uc5d0\uc11c \uc815\ucc45\uc801 \ucc28\uc774\uac00 \uc788\uc5c8\ub2e4. \uc655\uc740 \uc591\ud30c\ub97c \uc5f0\ub9bd\uc2dc\ucf30\uc73c\ub098 \uad8c\ub825\uc758 \ud575\uc2ec\uc744 \uc7a5\uc545\ud55c \uac83\uc740 \ub178\ub860\uc73c\ub85c\uc11c, \uc1a1\uc2dc\uc5f4\uacfc \uc0bc\ucc99\uc73c\ub85c \ubd88\ub9ac\ub358 \uc655\uc2e4\uc758 \uc678\ucc99, \uc989 \uae40\uc11d\uc8fc \u00b7 \uae40\ub9cc\uae30 \u00b7 \ubbfc\uc815\uc911\uc774 \uc5f0\ud569\ud558\uc5ec \uc815\uce58\ub97c \uc8fc\ub3c4\ud588\ub2e4.", "paragraph_answer": "\uadf8\ub7ec\ub098 \uc218\uc138\uc5d0 \ubab0\ub838\ub358 \uc11c\uc778\uc740 1680\ub144(\uc219\uc885 6\ub144) \ub0a8\uc778 \uc601\uc218 \ud5c8\uc801\uc774 \ub300\ud765\uc0b0\uc131\uc758 \uad70\uc778\uc744 \ub3d9\uc6d0\ud574 \uc5ed\ubaa8\ub97c \uafb8\uba84\ub2e4\uace0 \uace0\ubc1c\ud558\uc5ec, \ud5c8\uc801\u00b7\uc724\ud734 \ub4f1\uc744 \uc0ac\ud615\uc2dc\ud0a4\uace0 \ub098\uba38\uc9c0 \ub0a8\uc778\ub4e4\ub3c4 \ucd95\ucd9c\ud588\ub2e4. \uc774 \uc0ac\uac74\uc744 \u2018\uacbd\uc2e0\ud658\uad6d\u2019\uc774\ub77c \ud55c\ub2e4. \uc774 \ubb34\ub835 \uc11c\uc778\uc740 \uc790\uccb4 \ubd84\uc5f4\uc744 \uc77c\uc73c\ucf1c \uc1a1\uc2dc\uc5f4\uc744 \uc601\uc218\ub85c \ud558\ub294 \ub178\ub860 \uacfc \uc724\uc99d\uc744 \uc911\uc2ec\uc73c\ub85c \ud558\ub294 \uc18c\ub860\uc73c\ub85c \uac08\ub77c\uc84c\ub2e4(1683\ub144). \ub178\ub860\uc740 \ub300\uc758\uba85\ubd84\uc744 \uc874\uc911\ud558\uace0, \ub0b4\uc218\uc678\uc591, \uc989 \ubbfc\uc0dd\uc548\uc815\uacfc \uc790\uce58\uc790\uac15\uc744 \uac15\uc870\ud558\uc600\uc73c\uba70, \uc18c\ub860\uc740 \uc2e4\ub9ac\ub97c \uc911\uc2dc\ud558\uace0 \uc801\uadf9\uc801\uc778 \ubd81\ubc29\uac1c\ucc99\uc744 \uc8fc\uc7a5\ud55c \uc810\uc5d0\uc11c \uc815\ucc45\uc801 \ucc28\uc774\uac00 \uc788\uc5c8\ub2e4. \uc655\uc740 \uc591\ud30c\ub97c \uc5f0\ub9bd\uc2dc\ucf30\uc73c\ub098 \uad8c\ub825\uc758 \ud575\uc2ec\uc744 \uc7a5\uc545\ud55c \uac83\uc740 \ub178\ub860\uc73c\ub85c\uc11c, \uc1a1\uc2dc\uc5f4\uacfc \uc0bc\ucc99\uc73c\ub85c \ubd88\ub9ac\ub358 \uc655\uc2e4\uc758 \uc678\ucc99, \uc989 \uae40\uc11d\uc8fc \u00b7 \uae40\ub9cc\uae30 \u00b7 \ubbfc\uc815\uc911\uc774 \uc5f0\ud569\ud558\uc5ec \uc815\uce58\ub97c \uc8fc\ub3c4\ud588\ub2e4.", "sentence_answer": "\uc774 \ubb34\ub835 \uc11c\uc778\uc740 \uc790\uccb4 \ubd84\uc5f4\uc744 \uc77c\uc73c\ucf1c \uc1a1\uc2dc\uc5f4\uc744 \uc601\uc218\ub85c \ud558\ub294 \ub178\ub860 \uacfc \uc724\uc99d\uc744 \uc911\uc2ec\uc73c\ub85c \ud558\ub294 \uc18c\ub860\uc73c\ub85c \uac08\ub77c\uc84c\ub2e4(1683\ub144).", "paragraph_id": "6551283-2-2", "paragraph_question": "question: 1680\ub144 \uc219\uc885\uc774 \uad8c\ub825\uc758 \ud575\uc2ec\uc744 \uc7a5\uc545\ud55c \uacf3\uc740?, context: \uadf8\ub7ec\ub098 \uc218\uc138\uc5d0 \ubab0\ub838\ub358 \uc11c\uc778\uc740 1680\ub144(\uc219\uc885 6\ub144) \ub0a8\uc778 \uc601\uc218 \ud5c8\uc801\uc774 \ub300\ud765\uc0b0\uc131\uc758 \uad70\uc778\uc744 \ub3d9\uc6d0\ud574 \uc5ed\ubaa8\ub97c \uafb8\uba84\ub2e4\uace0 \uace0\ubc1c\ud558\uc5ec, \ud5c8\uc801\u00b7\uc724\ud734 \ub4f1\uc744 \uc0ac\ud615\uc2dc\ud0a4\uace0 \ub098\uba38\uc9c0 \ub0a8\uc778\ub4e4\ub3c4 \ucd95\ucd9c\ud588\ub2e4. \uc774 \uc0ac\uac74\uc744 \u2018\uacbd\uc2e0\ud658\uad6d\u2019\uc774\ub77c \ud55c\ub2e4. \uc774 \ubb34\ub835 \uc11c\uc778\uc740 \uc790\uccb4 \ubd84\uc5f4\uc744 \uc77c\uc73c\ucf1c \uc1a1\uc2dc\uc5f4\uc744 \uc601\uc218\ub85c \ud558\ub294 \ub178\ub860\uacfc \uc724\uc99d\uc744 \uc911\uc2ec\uc73c\ub85c \ud558\ub294 \uc18c\ub860\uc73c\ub85c \uac08\ub77c\uc84c\ub2e4(1683\ub144). \ub178\ub860\uc740 \ub300\uc758\uba85\ubd84\uc744 \uc874\uc911\ud558\uace0, \ub0b4\uc218\uc678\uc591, \uc989 \ubbfc\uc0dd\uc548\uc815\uacfc \uc790\uce58\uc790\uac15\uc744 \uac15\uc870\ud558\uc600\uc73c\uba70, \uc18c\ub860\uc740 \uc2e4\ub9ac\ub97c \uc911\uc2dc\ud558\uace0 \uc801\uadf9\uc801\uc778 \ubd81\ubc29\uac1c\ucc99\uc744 \uc8fc\uc7a5\ud55c \uc810\uc5d0\uc11c \uc815\ucc45\uc801 \ucc28\uc774\uac00 \uc788\uc5c8\ub2e4. \uc655\uc740 \uc591\ud30c\ub97c \uc5f0\ub9bd\uc2dc\ucf30\uc73c\ub098 \uad8c\ub825\uc758 \ud575\uc2ec\uc744 \uc7a5\uc545\ud55c \uac83\uc740 \ub178\ub860\uc73c\ub85c\uc11c, \uc1a1\uc2dc\uc5f4\uacfc \uc0bc\ucc99\uc73c\ub85c \ubd88\ub9ac\ub358 \uc655\uc2e4\uc758 \uc678\ucc99, \uc989 \uae40\uc11d\uc8fc \u00b7 \uae40\ub9cc\uae30 \u00b7 \ubbfc\uc815\uc911\uc774 \uc5f0\ud569\ud558\uc5ec \uc815\uce58\ub97c \uc8fc\ub3c4\ud588\ub2e4."} {"question": "\uac15\ud638\ub3d9\uc5d0 \uc774\uc5b4 1\ubc15 2\uc77c \ud558\ucc28 \uc120\uc5b8\ud55c \uc0ac\ub78c\uc740?", "paragraph": "2011\ub144 8\uc6d4 10\uc77c, \uba54\uc778 MC\uc600\ub358 \uac15\ud638\ub3d9\uc774 \ud558\ucc28\ud55c\ub2e4\ub294 \uc758\uc0ac\ub97c \ubc1d\ud600 \ub124\ud2f0\uc98c\ub4e4\uc744 \ub180\ub77c\uac8c \ud588\ub2e4. \uc774 \uc18c\uc2dd\uc774 \uc54c\ub824\uc9c4 \ud6c4 \ub124\ud2f0\uc98c\ub4e4\uc740 \uccad\uc6d0 \uac8c\uc2dc\ud310\uc5d0 \ud558\ucc28\ubc18\ub300 \uae00\uc744 \uac8c\uc7ac\ud574 \uc218\ub9ce\uc740 \ub124\ud2f0\uc98c\ub4e4\uc774 \uc11c\uba85 \uc6b4\ub3d9\uc744 \ud588\ub2e4. \uadf8\ub9ac\uace0 \ubc14\ub85c \ub2e4\uc74c \ub0a0, \uc81c\uc791\uc9c4 \uce21\uc740 \"\uac15\ud638\ub3d9\uc758 \ud558\ucc28\uc758\uc0ac\ub97c \uc804\ub2ec \ubc1b\uc740 \uac83\uc740 \uc0ac\uc2e4\uc774\ub77c \ud588\uc9c0\ub9cc \uc544\uc9c1 \uc544\ubb34\uac83\ub3c4 \uacb0\uc815\ub41c \uac83\uc740 \uc5c6\ub2e4.\" \uace0 \uacf5\uc2dd\uc801\uc778 \uc785\uc7a5\uc744 \ub4dc\ub7ec\ub0c8\ub2e4. \uadf8\ub9ac\uace0 \uadf8 \ub2e4\uc74c \ub0a0\uc778 8\uc6d4 12\uc77c \uc624\uc804, \uac15\ud638\ub3d9\uc740 \ucd2c\uc601\uc744 \uc55e\ub450\uace0 \uc11c\uc6b8 \uc5ec\uc758\ub3c4 KBS \uc2e0\uad00 \uc778\uadfc\uc5d0\uc11c \ucde8\uc7ac\uc9c4\ub4e4\uc5d0\uac8c \uacf5\uc2dd \uc785\uc7a5\uc744 \uc804\ud588\ub2e4. \uac15\ud638\ub3d9\uc740 \"\uc81c\uc791\uc9c4\uacfc \ud604\uc7ac \ub9ce\uc740 \uc774\uc57c\uae30\ub97c \ub098\ub204\uace0 \uc788\ub2e4.\"\uba70 \"\ud558\ucc28\uc5d0 \ub300\ud574 \uacb0\uc815\ub41c \uac83\uc740 \uc5c6\ub2e4. \ub354 \ub9ce\uc740 \uc598\uae30\ub97c \ub098\ub208 \ud6c4 \uacb0\uacfc\uac00 \ub098\uc624\uba74 \ub9d0\ud574 \uc8fc\uaca0\ub2e4.\"\uace0 \uc790\uc2e0\uc758 \uc785\uc7a5\uc744 \ubc1d\ud614\ub2e4. \uac15\ud638\ub3d9\uc774 1\ubc152\uc77c \ud558\ucc28\ub97c \uc120\uc5b8 \ud55c\uc9c0 1\uc8fc\uc77c \uc9f8\uc778 8\uc6d4 17\uc77c, 1\ubc152\uc77c PD \ub098\uc601\uc11d \ub610\ud55c 1\ubc152\uc77c\uc744 \ud558\ucc28\ud55c\ub2e4\uace0 \ubc1d\ud600\uc84c\ub2e4. \ub098\uc601\uc11d PD\ub294 \"1\ubc152\uc77c\uc744 \ub5a0\ub098\uc9c0 \uc54a\uc744 \uac83\"\uc774\ub77c\uba70 \"\uc885\ud3b8 \ubfd0\ub9cc\uc774 \uc544\ub2c8\ub77c \ubc29\uc1a1\uad6d\uc73c\ub85c\ub3c4 \uc774\uc801\ud558\uc9c0 \uc54a\uc744\uac83\"\uc774\ub77c\uace0 \uc77c\ub2e8 \ud558\ucc28\uc124\uc640 \uc774\uc801\uc124\uc744 \uac15\ub825\ud788 \ubd80\uc778\ud588\ub2e4. \uadf8\ub7ec\ub098 2011\ub144 8\uc6d4 19\uc77c, 1\ubc152\uc77c\uc774 2012\ub144 2\uc6d4\uc5d0 \uacf5\uc2dd \ud3d0\uc9c0\ub428\uc744 \ubc1d\ud614\ub2e4. \uadf8 \uc774\uc720\ub294 \uac15\ud638\ub3d9 \ud3ec\ud568 \uba64\ubc84 \uc804\uc6d0\uc774 6\uac1c\uc6d4\uac04 \ucd2c\uc601\uc744 \ub05d\uae4c\uc9c0 \ud568\uaed8 \ud558\uace0 \uc720\uc885\uc758 \ubbf8\ub97c \uac70\ub450\ub294 \uac83\uc73c\ub85c \ud569\uc758\ud55c \uad00\uacc4\ub85c \uc804\uaca9 \ud3d0\uc9c0\ud558\ub294 \uac83\uc73c\ub85c \uacb0\ub860\uc774 \ub0ac\uae30 \ub54c\ubb38\uc774\ub2e4. \ud558\uc9c0\ub9cc 2011\ub144 9\uc6d4 5\uc77c, \uac15\ud638\ub3d9\uc774 \uc138\ubb34\uc870\uc0ac\uc5d0\uc11c \uc138\uae08\ud3ec\ud0c8 \ud610\uc758\uac00 \ub4dc\ub7ec\ub098 \ub124\ud2f0\uc98c\ub4e4\uc5d0\uac8c \ud070 \ucda9\uaca9\uc744 \uc8fc\uc5c8\ub2e4. \uc774 \uc0ac\ud0dc\ub85c \uac15\ud638\ub3d9\uc758 \ud1f4\ucd9c\uc6b4\ub3d9\uae4c\uc9c0 \uc77c\uc5b4\ub098 \ub124\ud2f0\uc98c\ub4e4\uc5d0\uac8c \ud070 \ube44\ud310\uc744 \ubc1b\uc558\ub2e4. \uacb0\uad6d \uac15\ud638\ub3d9\uc740 2011\ub144 9\uc6d4 9\uc77c, \uae34\uae09 \uae30\uc790\ud68c\uacac\uc5d0\uc11c \uc790\uc9c4 \uc740\ud1f4\ub97c \uc120\uc5b8\ud558\uc600\ub2e4. \uac15\ud638\ub3d9\uc758 \uac11\uc791\uc2a4\ub7ec\uc6b4 \uc740\ud1f4\ub85c \uc778\ud558\uc5ec \uc608\ub2a5\uad6d \uc804\uccb4\uac00 \ub2e4 \ube44\uc0c1\uc774 \uacb0\ub838\ub2e4. \ub610\ud55c KBS\ub294 \"\uac15\ud638\ub3d9\uc758 \ube48\uc790\ub9ac\ub97c \ub300\uccb4\ud560\ub9cc\ud55c \uc778\ubb3c\uc774 \uc5c6\uace0, 2012\ub144 2\uc6d4 \ub2e4\uac19\uc774 \uc720\uc885\uc758 \ubbf8\ub97c \uac70\ub450\uae30\ub85c \uc2dc\uccad\uc790\ub4e4\uaed8 \uc57d\uc18d\ud55c \ub9cc\ud07c \ub300\uccb4 \uc778\ubb3c \ud22c\uc785\uc740 \uc801\uc808\uce58 \uc54a\ub2e4\uace0 \ubcf4\uace0 \uc788\ub2e4.\"\ub77c\uace0 \uc804\ud558\uc600\ub2e4. \uadf8\ub7ec\ud55c \uc774\uc720\ub85c 1\ubc15 2\uc77c\uc740 \uac15\ud638\ub3d9 \uc5c6\uc774 5\uc778 \uccb4\uc81c\ub85c \ub098\uba38\uc9c0 \ubc29\uc1a1\ubd84\uc744 \uc9c4\ud589\ud558\uc600\ub2e4.", "answer": "\ub098\uc601\uc11d", "sentence": "\uac15\ud638\ub3d9\uc774 1\ubc152\uc77c \ud558\ucc28\ub97c \uc120\uc5b8 \ud55c\uc9c0 1\uc8fc\uc77c \uc9f8\uc778 8\uc6d4 17\uc77c, 1\ubc152\uc77c PD \ub098\uc601\uc11d \ub610\ud55c 1\ubc152\uc77c\uc744 \ud558\ucc28\ud55c\ub2e4\uace0 \ubc1d\ud600\uc84c\ub2e4.", "paragraph_sentence": "2011\ub144 8\uc6d4 10\uc77c, \uba54\uc778 MC\uc600\ub358 \uac15\ud638\ub3d9\uc774 \ud558\ucc28\ud55c\ub2e4\ub294 \uc758\uc0ac\ub97c \ubc1d\ud600 \ub124\ud2f0\uc98c\ub4e4\uc744 \ub180\ub77c\uac8c \ud588\ub2e4. \uc774 \uc18c\uc2dd\uc774 \uc54c\ub824\uc9c4 \ud6c4 \ub124\ud2f0\uc98c\ub4e4\uc740 \uccad\uc6d0 \uac8c\uc2dc\ud310\uc5d0 \ud558\ucc28\ubc18\ub300 \uae00\uc744 \uac8c\uc7ac\ud574 \uc218\ub9ce\uc740 \ub124\ud2f0\uc98c\ub4e4\uc774 \uc11c\uba85 \uc6b4\ub3d9\uc744 \ud588\ub2e4. \uadf8\ub9ac\uace0 \ubc14\ub85c \ub2e4\uc74c \ub0a0, \uc81c\uc791\uc9c4 \uce21\uc740 \"\uac15\ud638\ub3d9\uc758 \ud558\ucc28\uc758\uc0ac\ub97c \uc804\ub2ec \ubc1b\uc740 \uac83\uc740 \uc0ac\uc2e4\uc774\ub77c \ud588\uc9c0\ub9cc \uc544\uc9c1 \uc544\ubb34\uac83\ub3c4 \uacb0\uc815\ub41c \uac83\uc740 \uc5c6\ub2e4. \" \uace0 \uacf5\uc2dd\uc801\uc778 \uc785\uc7a5\uc744 \ub4dc\ub7ec\ub0c8\ub2e4. \uadf8\ub9ac\uace0 \uadf8 \ub2e4\uc74c \ub0a0\uc778 8\uc6d4 12\uc77c \uc624\uc804, \uac15\ud638\ub3d9\uc740 \ucd2c\uc601\uc744 \uc55e\ub450\uace0 \uc11c\uc6b8 \uc5ec\uc758\ub3c4 KBS \uc2e0\uad00 \uc778\uadfc\uc5d0\uc11c \ucde8\uc7ac\uc9c4\ub4e4\uc5d0\uac8c \uacf5\uc2dd \uc785\uc7a5\uc744 \uc804\ud588\ub2e4. \uac15\ud638\ub3d9\uc740 \"\uc81c\uc791\uc9c4\uacfc \ud604\uc7ac \ub9ce\uc740 \uc774\uc57c\uae30\ub97c \ub098\ub204\uace0 \uc788\ub2e4. \"\uba70 \"\ud558\ucc28\uc5d0 \ub300\ud574 \uacb0\uc815\ub41c \uac83\uc740 \uc5c6\ub2e4. \ub354 \ub9ce\uc740 \uc598\uae30\ub97c \ub098\ub208 \ud6c4 \uacb0\uacfc\uac00 \ub098\uc624\uba74 \ub9d0\ud574 \uc8fc\uaca0\ub2e4. \"\uace0 \uc790\uc2e0\uc758 \uc785\uc7a5\uc744 \ubc1d\ud614\ub2e4. \uac15\ud638\ub3d9\uc774 1\ubc152\uc77c \ud558\ucc28\ub97c \uc120\uc5b8 \ud55c\uc9c0 1\uc8fc\uc77c \uc9f8\uc778 8\uc6d4 17\uc77c, 1\ubc152\uc77c PD \ub098\uc601\uc11d \ub610\ud55c 1\ubc152\uc77c\uc744 \ud558\ucc28\ud55c\ub2e4\uace0 \ubc1d\ud600\uc84c\ub2e4. \ub098\uc601\uc11d PD\ub294 \"1\ubc152\uc77c\uc744 \ub5a0\ub098\uc9c0 \uc54a\uc744 \uac83\"\uc774\ub77c\uba70 \"\uc885\ud3b8 \ubfd0\ub9cc\uc774 \uc544\ub2c8\ub77c \ubc29\uc1a1\uad6d\uc73c\ub85c\ub3c4 \uc774\uc801\ud558\uc9c0 \uc54a\uc744\uac83 \"\uc774\ub77c\uace0 \uc77c\ub2e8 \ud558\ucc28\uc124\uc640 \uc774\uc801\uc124\uc744 \uac15\ub825\ud788 \ubd80\uc778\ud588\ub2e4. \uadf8\ub7ec\ub098 2011\ub144 8\uc6d4 19\uc77c, 1\ubc152\uc77c\uc774 2012\ub144 2\uc6d4\uc5d0 \uacf5\uc2dd \ud3d0\uc9c0\ub428\uc744 \ubc1d\ud614\ub2e4. \uadf8 \uc774\uc720\ub294 \uac15\ud638\ub3d9 \ud3ec\ud568 \uba64\ubc84 \uc804\uc6d0\uc774 6\uac1c\uc6d4\uac04 \ucd2c\uc601\uc744 \ub05d\uae4c\uc9c0 \ud568\uaed8 \ud558\uace0 \uc720\uc885\uc758 \ubbf8\ub97c \uac70\ub450\ub294 \uac83\uc73c\ub85c \ud569\uc758\ud55c \uad00\uacc4\ub85c \uc804\uaca9 \ud3d0\uc9c0\ud558\ub294 \uac83\uc73c\ub85c \uacb0\ub860\uc774 \ub0ac\uae30 \ub54c\ubb38\uc774\ub2e4. \ud558\uc9c0\ub9cc 2011\ub144 9\uc6d4 5\uc77c, \uac15\ud638\ub3d9\uc774 \uc138\ubb34\uc870\uc0ac\uc5d0\uc11c \uc138\uae08\ud3ec\ud0c8 \ud610\uc758\uac00 \ub4dc\ub7ec\ub098 \ub124\ud2f0\uc98c\ub4e4\uc5d0\uac8c \ud070 \ucda9\uaca9\uc744 \uc8fc\uc5c8\ub2e4. \uc774 \uc0ac\ud0dc\ub85c \uac15\ud638\ub3d9\uc758 \ud1f4\ucd9c\uc6b4\ub3d9\uae4c\uc9c0 \uc77c\uc5b4\ub098 \ub124\ud2f0\uc98c\ub4e4\uc5d0\uac8c \ud070 \ube44\ud310\uc744 \ubc1b\uc558\ub2e4. \uacb0\uad6d \uac15\ud638\ub3d9\uc740 2011\ub144 9\uc6d4 9\uc77c, \uae34\uae09 \uae30\uc790\ud68c\uacac\uc5d0\uc11c \uc790\uc9c4 \uc740\ud1f4\ub97c \uc120\uc5b8\ud558\uc600\ub2e4. \uac15\ud638\ub3d9\uc758 \uac11\uc791\uc2a4\ub7ec\uc6b4 \uc740\ud1f4\ub85c \uc778\ud558\uc5ec \uc608\ub2a5\uad6d \uc804\uccb4\uac00 \ub2e4 \ube44\uc0c1\uc774 \uacb0\ub838\ub2e4. \ub610\ud55c KBS\ub294 \"\uac15\ud638\ub3d9\uc758 \ube48\uc790\ub9ac\ub97c \ub300\uccb4\ud560\ub9cc\ud55c \uc778\ubb3c\uc774 \uc5c6\uace0, 2012\ub144 2\uc6d4 \ub2e4\uac19\uc774 \uc720\uc885\uc758 \ubbf8\ub97c \uac70\ub450\uae30\ub85c \uc2dc\uccad\uc790\ub4e4\uaed8 \uc57d\uc18d\ud55c \ub9cc\ud07c \ub300\uccb4 \uc778\ubb3c \ud22c\uc785\uc740 \uc801\uc808\uce58 \uc54a\ub2e4\uace0 \ubcf4\uace0 \uc788\ub2e4. \"\ub77c\uace0 \uc804\ud558\uc600\ub2e4. \uadf8\ub7ec\ud55c \uc774\uc720\ub85c 1\ubc15 2\uc77c\uc740 \uac15\ud638\ub3d9 \uc5c6\uc774 5\uc778 \uccb4\uc81c\ub85c \ub098\uba38\uc9c0 \ubc29\uc1a1\ubd84\uc744 \uc9c4\ud589\ud558\uc600\ub2e4.", "paragraph_answer": "2011\ub144 8\uc6d4 10\uc77c, \uba54\uc778 MC\uc600\ub358 \uac15\ud638\ub3d9\uc774 \ud558\ucc28\ud55c\ub2e4\ub294 \uc758\uc0ac\ub97c \ubc1d\ud600 \ub124\ud2f0\uc98c\ub4e4\uc744 \ub180\ub77c\uac8c \ud588\ub2e4. \uc774 \uc18c\uc2dd\uc774 \uc54c\ub824\uc9c4 \ud6c4 \ub124\ud2f0\uc98c\ub4e4\uc740 \uccad\uc6d0 \uac8c\uc2dc\ud310\uc5d0 \ud558\ucc28\ubc18\ub300 \uae00\uc744 \uac8c\uc7ac\ud574 \uc218\ub9ce\uc740 \ub124\ud2f0\uc98c\ub4e4\uc774 \uc11c\uba85 \uc6b4\ub3d9\uc744 \ud588\ub2e4. \uadf8\ub9ac\uace0 \ubc14\ub85c \ub2e4\uc74c \ub0a0, \uc81c\uc791\uc9c4 \uce21\uc740 \"\uac15\ud638\ub3d9\uc758 \ud558\ucc28\uc758\uc0ac\ub97c \uc804\ub2ec \ubc1b\uc740 \uac83\uc740 \uc0ac\uc2e4\uc774\ub77c \ud588\uc9c0\ub9cc \uc544\uc9c1 \uc544\ubb34\uac83\ub3c4 \uacb0\uc815\ub41c \uac83\uc740 \uc5c6\ub2e4.\" \uace0 \uacf5\uc2dd\uc801\uc778 \uc785\uc7a5\uc744 \ub4dc\ub7ec\ub0c8\ub2e4. \uadf8\ub9ac\uace0 \uadf8 \ub2e4\uc74c \ub0a0\uc778 8\uc6d4 12\uc77c \uc624\uc804, \uac15\ud638\ub3d9\uc740 \ucd2c\uc601\uc744 \uc55e\ub450\uace0 \uc11c\uc6b8 \uc5ec\uc758\ub3c4 KBS \uc2e0\uad00 \uc778\uadfc\uc5d0\uc11c \ucde8\uc7ac\uc9c4\ub4e4\uc5d0\uac8c \uacf5\uc2dd \uc785\uc7a5\uc744 \uc804\ud588\ub2e4. \uac15\ud638\ub3d9\uc740 \"\uc81c\uc791\uc9c4\uacfc \ud604\uc7ac \ub9ce\uc740 \uc774\uc57c\uae30\ub97c \ub098\ub204\uace0 \uc788\ub2e4.\"\uba70 \"\ud558\ucc28\uc5d0 \ub300\ud574 \uacb0\uc815\ub41c \uac83\uc740 \uc5c6\ub2e4. \ub354 \ub9ce\uc740 \uc598\uae30\ub97c \ub098\ub208 \ud6c4 \uacb0\uacfc\uac00 \ub098\uc624\uba74 \ub9d0\ud574 \uc8fc\uaca0\ub2e4.\"\uace0 \uc790\uc2e0\uc758 \uc785\uc7a5\uc744 \ubc1d\ud614\ub2e4. \uac15\ud638\ub3d9\uc774 1\ubc152\uc77c \ud558\ucc28\ub97c \uc120\uc5b8 \ud55c\uc9c0 1\uc8fc\uc77c \uc9f8\uc778 8\uc6d4 17\uc77c, 1\ubc152\uc77c PD \ub098\uc601\uc11d \ub610\ud55c 1\ubc152\uc77c\uc744 \ud558\ucc28\ud55c\ub2e4\uace0 \ubc1d\ud600\uc84c\ub2e4. \ub098\uc601\uc11d PD\ub294 \"1\ubc152\uc77c\uc744 \ub5a0\ub098\uc9c0 \uc54a\uc744 \uac83\"\uc774\ub77c\uba70 \"\uc885\ud3b8 \ubfd0\ub9cc\uc774 \uc544\ub2c8\ub77c \ubc29\uc1a1\uad6d\uc73c\ub85c\ub3c4 \uc774\uc801\ud558\uc9c0 \uc54a\uc744\uac83\"\uc774\ub77c\uace0 \uc77c\ub2e8 \ud558\ucc28\uc124\uc640 \uc774\uc801\uc124\uc744 \uac15\ub825\ud788 \ubd80\uc778\ud588\ub2e4. \uadf8\ub7ec\ub098 2011\ub144 8\uc6d4 19\uc77c, 1\ubc152\uc77c\uc774 2012\ub144 2\uc6d4\uc5d0 \uacf5\uc2dd \ud3d0\uc9c0\ub428\uc744 \ubc1d\ud614\ub2e4. \uadf8 \uc774\uc720\ub294 \uac15\ud638\ub3d9 \ud3ec\ud568 \uba64\ubc84 \uc804\uc6d0\uc774 6\uac1c\uc6d4\uac04 \ucd2c\uc601\uc744 \ub05d\uae4c\uc9c0 \ud568\uaed8 \ud558\uace0 \uc720\uc885\uc758 \ubbf8\ub97c \uac70\ub450\ub294 \uac83\uc73c\ub85c \ud569\uc758\ud55c \uad00\uacc4\ub85c \uc804\uaca9 \ud3d0\uc9c0\ud558\ub294 \uac83\uc73c\ub85c \uacb0\ub860\uc774 \ub0ac\uae30 \ub54c\ubb38\uc774\ub2e4. \ud558\uc9c0\ub9cc 2011\ub144 9\uc6d4 5\uc77c, \uac15\ud638\ub3d9\uc774 \uc138\ubb34\uc870\uc0ac\uc5d0\uc11c \uc138\uae08\ud3ec\ud0c8 \ud610\uc758\uac00 \ub4dc\ub7ec\ub098 \ub124\ud2f0\uc98c\ub4e4\uc5d0\uac8c \ud070 \ucda9\uaca9\uc744 \uc8fc\uc5c8\ub2e4. \uc774 \uc0ac\ud0dc\ub85c \uac15\ud638\ub3d9\uc758 \ud1f4\ucd9c\uc6b4\ub3d9\uae4c\uc9c0 \uc77c\uc5b4\ub098 \ub124\ud2f0\uc98c\ub4e4\uc5d0\uac8c \ud070 \ube44\ud310\uc744 \ubc1b\uc558\ub2e4. \uacb0\uad6d \uac15\ud638\ub3d9\uc740 2011\ub144 9\uc6d4 9\uc77c, \uae34\uae09 \uae30\uc790\ud68c\uacac\uc5d0\uc11c \uc790\uc9c4 \uc740\ud1f4\ub97c \uc120\uc5b8\ud558\uc600\ub2e4. \uac15\ud638\ub3d9\uc758 \uac11\uc791\uc2a4\ub7ec\uc6b4 \uc740\ud1f4\ub85c \uc778\ud558\uc5ec \uc608\ub2a5\uad6d \uc804\uccb4\uac00 \ub2e4 \ube44\uc0c1\uc774 \uacb0\ub838\ub2e4. \ub610\ud55c KBS\ub294 \"\uac15\ud638\ub3d9\uc758 \ube48\uc790\ub9ac\ub97c \ub300\uccb4\ud560\ub9cc\ud55c \uc778\ubb3c\uc774 \uc5c6\uace0, 2012\ub144 2\uc6d4 \ub2e4\uac19\uc774 \uc720\uc885\uc758 \ubbf8\ub97c \uac70\ub450\uae30\ub85c \uc2dc\uccad\uc790\ub4e4\uaed8 \uc57d\uc18d\ud55c \ub9cc\ud07c \ub300\uccb4 \uc778\ubb3c \ud22c\uc785\uc740 \uc801\uc808\uce58 \uc54a\ub2e4\uace0 \ubcf4\uace0 \uc788\ub2e4.\"\ub77c\uace0 \uc804\ud558\uc600\ub2e4. \uadf8\ub7ec\ud55c \uc774\uc720\ub85c 1\ubc15 2\uc77c\uc740 \uac15\ud638\ub3d9 \uc5c6\uc774 5\uc778 \uccb4\uc81c\ub85c \ub098\uba38\uc9c0 \ubc29\uc1a1\ubd84\uc744 \uc9c4\ud589\ud558\uc600\ub2e4.", "sentence_answer": "\uac15\ud638\ub3d9\uc774 1\ubc152\uc77c \ud558\ucc28\ub97c \uc120\uc5b8 \ud55c\uc9c0 1\uc8fc\uc77c \uc9f8\uc778 8\uc6d4 17\uc77c, 1\ubc152\uc77c PD \ub098\uc601\uc11d \ub610\ud55c 1\ubc152\uc77c\uc744 \ud558\ucc28\ud55c\ub2e4\uace0 \ubc1d\ud600\uc84c\ub2e4.", "paragraph_id": "6469291-4-0", "paragraph_question": "question: \uac15\ud638\ub3d9\uc5d0 \uc774\uc5b4 1\ubc15 2\uc77c \ud558\ucc28 \uc120\uc5b8\ud55c \uc0ac\ub78c\uc740?, context: 2011\ub144 8\uc6d4 10\uc77c, \uba54\uc778 MC\uc600\ub358 \uac15\ud638\ub3d9\uc774 \ud558\ucc28\ud55c\ub2e4\ub294 \uc758\uc0ac\ub97c \ubc1d\ud600 \ub124\ud2f0\uc98c\ub4e4\uc744 \ub180\ub77c\uac8c \ud588\ub2e4. \uc774 \uc18c\uc2dd\uc774 \uc54c\ub824\uc9c4 \ud6c4 \ub124\ud2f0\uc98c\ub4e4\uc740 \uccad\uc6d0 \uac8c\uc2dc\ud310\uc5d0 \ud558\ucc28\ubc18\ub300 \uae00\uc744 \uac8c\uc7ac\ud574 \uc218\ub9ce\uc740 \ub124\ud2f0\uc98c\ub4e4\uc774 \uc11c\uba85 \uc6b4\ub3d9\uc744 \ud588\ub2e4. \uadf8\ub9ac\uace0 \ubc14\ub85c \ub2e4\uc74c \ub0a0, \uc81c\uc791\uc9c4 \uce21\uc740 \"\uac15\ud638\ub3d9\uc758 \ud558\ucc28\uc758\uc0ac\ub97c \uc804\ub2ec \ubc1b\uc740 \uac83\uc740 \uc0ac\uc2e4\uc774\ub77c \ud588\uc9c0\ub9cc \uc544\uc9c1 \uc544\ubb34\uac83\ub3c4 \uacb0\uc815\ub41c \uac83\uc740 \uc5c6\ub2e4.\" \uace0 \uacf5\uc2dd\uc801\uc778 \uc785\uc7a5\uc744 \ub4dc\ub7ec\ub0c8\ub2e4. \uadf8\ub9ac\uace0 \uadf8 \ub2e4\uc74c \ub0a0\uc778 8\uc6d4 12\uc77c \uc624\uc804, \uac15\ud638\ub3d9\uc740 \ucd2c\uc601\uc744 \uc55e\ub450\uace0 \uc11c\uc6b8 \uc5ec\uc758\ub3c4 KBS \uc2e0\uad00 \uc778\uadfc\uc5d0\uc11c \ucde8\uc7ac\uc9c4\ub4e4\uc5d0\uac8c \uacf5\uc2dd \uc785\uc7a5\uc744 \uc804\ud588\ub2e4. \uac15\ud638\ub3d9\uc740 \"\uc81c\uc791\uc9c4\uacfc \ud604\uc7ac \ub9ce\uc740 \uc774\uc57c\uae30\ub97c \ub098\ub204\uace0 \uc788\ub2e4.\"\uba70 \"\ud558\ucc28\uc5d0 \ub300\ud574 \uacb0\uc815\ub41c \uac83\uc740 \uc5c6\ub2e4. \ub354 \ub9ce\uc740 \uc598\uae30\ub97c \ub098\ub208 \ud6c4 \uacb0\uacfc\uac00 \ub098\uc624\uba74 \ub9d0\ud574 \uc8fc\uaca0\ub2e4.\"\uace0 \uc790\uc2e0\uc758 \uc785\uc7a5\uc744 \ubc1d\ud614\ub2e4. \uac15\ud638\ub3d9\uc774 1\ubc152\uc77c \ud558\ucc28\ub97c \uc120\uc5b8 \ud55c\uc9c0 1\uc8fc\uc77c \uc9f8\uc778 8\uc6d4 17\uc77c, 1\ubc152\uc77c PD \ub098\uc601\uc11d \ub610\ud55c 1\ubc152\uc77c\uc744 \ud558\ucc28\ud55c\ub2e4\uace0 \ubc1d\ud600\uc84c\ub2e4. \ub098\uc601\uc11d PD\ub294 \"1\ubc152\uc77c\uc744 \ub5a0\ub098\uc9c0 \uc54a\uc744 \uac83\"\uc774\ub77c\uba70 \"\uc885\ud3b8 \ubfd0\ub9cc\uc774 \uc544\ub2c8\ub77c \ubc29\uc1a1\uad6d\uc73c\ub85c\ub3c4 \uc774\uc801\ud558\uc9c0 \uc54a\uc744\uac83\"\uc774\ub77c\uace0 \uc77c\ub2e8 \ud558\ucc28\uc124\uc640 \uc774\uc801\uc124\uc744 \uac15\ub825\ud788 \ubd80\uc778\ud588\ub2e4. \uadf8\ub7ec\ub098 2011\ub144 8\uc6d4 19\uc77c, 1\ubc152\uc77c\uc774 2012\ub144 2\uc6d4\uc5d0 \uacf5\uc2dd \ud3d0\uc9c0\ub428\uc744 \ubc1d\ud614\ub2e4. \uadf8 \uc774\uc720\ub294 \uac15\ud638\ub3d9 \ud3ec\ud568 \uba64\ubc84 \uc804\uc6d0\uc774 6\uac1c\uc6d4\uac04 \ucd2c\uc601\uc744 \ub05d\uae4c\uc9c0 \ud568\uaed8 \ud558\uace0 \uc720\uc885\uc758 \ubbf8\ub97c \uac70\ub450\ub294 \uac83\uc73c\ub85c \ud569\uc758\ud55c \uad00\uacc4\ub85c \uc804\uaca9 \ud3d0\uc9c0\ud558\ub294 \uac83\uc73c\ub85c \uacb0\ub860\uc774 \ub0ac\uae30 \ub54c\ubb38\uc774\ub2e4. \ud558\uc9c0\ub9cc 2011\ub144 9\uc6d4 5\uc77c, \uac15\ud638\ub3d9\uc774 \uc138\ubb34\uc870\uc0ac\uc5d0\uc11c \uc138\uae08\ud3ec\ud0c8 \ud610\uc758\uac00 \ub4dc\ub7ec\ub098 \ub124\ud2f0\uc98c\ub4e4\uc5d0\uac8c \ud070 \ucda9\uaca9\uc744 \uc8fc\uc5c8\ub2e4. \uc774 \uc0ac\ud0dc\ub85c \uac15\ud638\ub3d9\uc758 \ud1f4\ucd9c\uc6b4\ub3d9\uae4c\uc9c0 \uc77c\uc5b4\ub098 \ub124\ud2f0\uc98c\ub4e4\uc5d0\uac8c \ud070 \ube44\ud310\uc744 \ubc1b\uc558\ub2e4. \uacb0\uad6d \uac15\ud638\ub3d9\uc740 2011\ub144 9\uc6d4 9\uc77c, \uae34\uae09 \uae30\uc790\ud68c\uacac\uc5d0\uc11c \uc790\uc9c4 \uc740\ud1f4\ub97c \uc120\uc5b8\ud558\uc600\ub2e4. \uac15\ud638\ub3d9\uc758 \uac11\uc791\uc2a4\ub7ec\uc6b4 \uc740\ud1f4\ub85c \uc778\ud558\uc5ec \uc608\ub2a5\uad6d \uc804\uccb4\uac00 \ub2e4 \ube44\uc0c1\uc774 \uacb0\ub838\ub2e4. \ub610\ud55c KBS\ub294 \"\uac15\ud638\ub3d9\uc758 \ube48\uc790\ub9ac\ub97c \ub300\uccb4\ud560\ub9cc\ud55c \uc778\ubb3c\uc774 \uc5c6\uace0, 2012\ub144 2\uc6d4 \ub2e4\uac19\uc774 \uc720\uc885\uc758 \ubbf8\ub97c \uac70\ub450\uae30\ub85c \uc2dc\uccad\uc790\ub4e4\uaed8 \uc57d\uc18d\ud55c \ub9cc\ud07c \ub300\uccb4 \uc778\ubb3c \ud22c\uc785\uc740 \uc801\uc808\uce58 \uc54a\ub2e4\uace0 \ubcf4\uace0 \uc788\ub2e4.\"\ub77c\uace0 \uc804\ud558\uc600\ub2e4. \uadf8\ub7ec\ud55c \uc774\uc720\ub85c 1\ubc15 2\uc77c\uc740 \uac15\ud638\ub3d9 \uc5c6\uc774 5\uc778 \uccb4\uc81c\ub85c \ub098\uba38\uc9c0 \ubc29\uc1a1\ubd84\uc744 \uc9c4\ud589\ud558\uc600\ub2e4."} {"question": "1930\ub144\ub300\ubd80\ud130 1940\ub144\ub300 \ucd08\ubc18\uc5d0 \ub2f4\ubc30 \ubc18\ub300 \uc6b4\ub3d9\uc774 \uc804\uac1c\ub420 \ub2f9\uc2dc\uc758 \ub3c5\uc77c\uc744 \ud1b5\uce58\ud558\ub358 \ub2f9\uc740?", "paragraph": "\ub3c5\uc77c\uc758 \uc758\uc0ac\ub4e4\uc774 \ud761\uc5f0\uacfc \ud3d0\uc554\uc758 \uad00\uacc4\ub97c \ucc98\uc74c \ud655\uc778\ud55c \uc774\ud6c4\ubd80\ud130 \ub098\uce58 \ub3c5\uc77c\uc740 \uac15\ub825\ud55c \ubc18\ub2f4\ubc30 \uc6b4\ub3d9 \uacfc \ud604\ub300\uc0ac \ucd5c\ucd08\uc758 \uacf5\uc775\uc801\uc778 \uae08\uc5f0 \uc6b4\ub3d9\uc744 \uc804\uac1c\ud588\ub2e4. \ub2f4\ubc30 \ubc18\ub300 \uc6b4\ub3d9\uc740 20\uc138\uae30 \ucd08\ubc18\ubd80\ud130 \ub9ce\uc740 \ub098\ub77c\uc5d0\uc11c \uc804\uac1c\ub418\uc5b4 \uc654\uc9c0\ub9cc \uad6d\uac00\uc0ac\ud68c\uc8fc\uc758 \ub3c5\uc77c \ub178\ub3d9\uc790\ub2f9(\ub098\uce58\ub2f9)\uc758 \ud1b5\uce58 \ud558\uc5d0 \uc788\ub358 \ub3c5\uc77c \uc774\uc678\uc758 \ub098\ub77c\uc5d0\uc11c\ub294 \ud070 \uc131\uacf5\uc744 \uac70\ub450\uc9c0 \ubabb\ud588\ub2e4. \ub098\uce58 \ub3c5\uc77c\uc758 \uae08\uc5f0 \uc6b4\ub3d9\uc740 1930\ub144\ub300\uc640 1940\ub144\ub300 \ucd08\ubc18\uc5d0 \uc788\uc5c8\ub358 \uae08\uc5f0 \uc6b4\ub3d9 \uac00\uc6b4\ub370 \uc138\uacc4\uc5d0\uc11c \uac00\uc7a5 \uac15\ub825\ud588\ub358 \uae08\uc5f0 \uc6b4\ub3d9\uc774\uc5c8\ub2e4. \ub098\uce58\ub2f9\uc758 \uc9c0\ub3c4\uc790\ub4e4\uc740 \ud761\uc5f0\uc744 \ube44\ub09c\ud588\uc73c\uba70 \uc774\ub4e4 \uc911 \uc77c\ubd80\ub294 \ub2f4\ubc30 \uc18c\ube44\ub97c \uacf5\uacf5\uc5f0\ud788 \ube44\ub09c\ud558\uae30\ub3c4 \ud588\ub2e4. \ud761\uc5f0\uc774 \uac74\uac15\uc5d0 \ubbf8\uce58\ub294 \uc601\ud5a5\uc5d0 \uad00\ud55c \uc5f0\uad6c\uac00 \ub098\uce58\ub2f9\uc758 \uc9c0\ub3c4 \ud558\uc5d0 \uc9c4\ud589\ub418\uc5c8\ub294\ub370 \uc774\ub294 \ub2f9\uc2dc \ub098\uce58 \ub3c5\uc77c\uc5d0\uc11c \uac00\uc7a5 \uc911\uc694\ud55c \uc5f0\uad6c \uc8fc\uc81c\uac00 \ub418\uae30\ub3c4 \ud558\uc600\ub2e4. \ub2f4\ubc30\ub97c \uc2eb\uc5b4\ud558\ub358 \uc544\ub3cc\ud504 \ud788\ud2c0\ub7ec\uc640 \ub098\uce58 \ub3c5\uc77c\uc758 \ucd9c\uc0b0 \uc7a5\ub824 \uc815\ucc45\uc740 \uae08\uc5f0 \uc6b4\ub3d9\uc744 \uc790\uadf9\ud558\ub294 \uc6d0\uc778\uc774 \ub418\uc5c8\ub294\ub370 \uc774\ub294 \uc778\uc885 \ucc28\ubcc4, \ubc18\uc720\ub300\uc8fc\uc758\uc640 \uc5f0\uad00\ub418\uc5b4 \uc788\uc5c8\ub2e4.", "answer": "\ub178\ub3d9\uc790\ub2f9", "sentence": "\uad6d\uac00\uc0ac\ud68c\uc8fc\uc758 \ub3c5\uc77c \ub178\ub3d9\uc790\ub2f9 (\ub098\uce58\ub2f9)", "paragraph_sentence": "\ub3c5\uc77c\uc758 \uc758\uc0ac\ub4e4\uc774 \ud761\uc5f0\uacfc \ud3d0\uc554\uc758 \uad00\uacc4\ub97c \ucc98\uc74c \ud655\uc778\ud55c \uc774\ud6c4\ubd80\ud130 \ub098\uce58 \ub3c5\uc77c\uc740 \uac15\ub825\ud55c \ubc18\ub2f4\ubc30 \uc6b4\ub3d9 \uacfc \ud604\ub300\uc0ac \ucd5c\ucd08\uc758 \uacf5\uc775\uc801\uc778 \uae08\uc5f0 \uc6b4\ub3d9\uc744 \uc804\uac1c\ud588\ub2e4. \ub2f4\ubc30 \ubc18\ub300 \uc6b4\ub3d9\uc740 20\uc138\uae30 \ucd08\ubc18\ubd80\ud130 \ub9ce\uc740 \ub098\ub77c\uc5d0\uc11c \uc804\uac1c\ub418\uc5b4 \uc654\uc9c0\ub9cc \uad6d\uac00\uc0ac\ud68c\uc8fc\uc758 \ub3c5\uc77c \ub178\ub3d9\uc790\ub2f9 (\ub098\uce58\ub2f9) \uc758 \ud1b5\uce58 \ud558\uc5d0 \uc788\ub358 \ub3c5\uc77c \uc774\uc678\uc758 \ub098\ub77c\uc5d0\uc11c\ub294 \ud070 \uc131\uacf5\uc744 \uac70\ub450\uc9c0 \ubabb\ud588\ub2e4. \ub098\uce58 \ub3c5\uc77c\uc758 \uae08\uc5f0 \uc6b4\ub3d9\uc740 1930\ub144\ub300\uc640 1940\ub144\ub300 \ucd08\ubc18\uc5d0 \uc788\uc5c8\ub358 \uae08\uc5f0 \uc6b4\ub3d9 \uac00\uc6b4\ub370 \uc138\uacc4\uc5d0\uc11c \uac00\uc7a5 \uac15\ub825\ud588\ub358 \uae08\uc5f0 \uc6b4\ub3d9\uc774\uc5c8\ub2e4. \ub098\uce58\ub2f9\uc758 \uc9c0\ub3c4\uc790\ub4e4\uc740 \ud761\uc5f0\uc744 \ube44\ub09c\ud588\uc73c\uba70 \uc774\ub4e4 \uc911 \uc77c\ubd80\ub294 \ub2f4\ubc30 \uc18c\ube44\ub97c \uacf5\uacf5\uc5f0\ud788 \ube44\ub09c\ud558\uae30\ub3c4 \ud588\ub2e4. \ud761\uc5f0\uc774 \uac74\uac15\uc5d0 \ubbf8\uce58\ub294 \uc601\ud5a5\uc5d0 \uad00\ud55c \uc5f0\uad6c\uac00 \ub098\uce58\ub2f9\uc758 \uc9c0\ub3c4 \ud558\uc5d0 \uc9c4\ud589\ub418\uc5c8\ub294\ub370 \uc774\ub294 \ub2f9\uc2dc \ub098\uce58 \ub3c5\uc77c\uc5d0\uc11c \uac00\uc7a5 \uc911\uc694\ud55c \uc5f0\uad6c \uc8fc\uc81c\uac00 \ub418\uae30\ub3c4 \ud558\uc600\ub2e4. \ub2f4\ubc30\ub97c \uc2eb\uc5b4\ud558\ub358 \uc544\ub3cc\ud504 \ud788\ud2c0\ub7ec\uc640 \ub098\uce58 \ub3c5\uc77c\uc758 \ucd9c\uc0b0 \uc7a5\ub824 \uc815\ucc45\uc740 \uae08\uc5f0 \uc6b4\ub3d9\uc744 \uc790\uadf9\ud558\ub294 \uc6d0\uc778\uc774 \ub418\uc5c8\ub294\ub370 \uc774\ub294 \uc778\uc885 \ucc28\ubcc4, \ubc18\uc720\ub300\uc8fc\uc758\uc640 \uc5f0\uad00\ub418\uc5b4 \uc788\uc5c8\ub2e4.", "paragraph_answer": "\ub3c5\uc77c\uc758 \uc758\uc0ac\ub4e4\uc774 \ud761\uc5f0\uacfc \ud3d0\uc554\uc758 \uad00\uacc4\ub97c \ucc98\uc74c \ud655\uc778\ud55c \uc774\ud6c4\ubd80\ud130 \ub098\uce58 \ub3c5\uc77c\uc740 \uac15\ub825\ud55c \ubc18\ub2f4\ubc30 \uc6b4\ub3d9 \uacfc \ud604\ub300\uc0ac \ucd5c\ucd08\uc758 \uacf5\uc775\uc801\uc778 \uae08\uc5f0 \uc6b4\ub3d9\uc744 \uc804\uac1c\ud588\ub2e4. \ub2f4\ubc30 \ubc18\ub300 \uc6b4\ub3d9\uc740 20\uc138\uae30 \ucd08\ubc18\ubd80\ud130 \ub9ce\uc740 \ub098\ub77c\uc5d0\uc11c \uc804\uac1c\ub418\uc5b4 \uc654\uc9c0\ub9cc \uad6d\uac00\uc0ac\ud68c\uc8fc\uc758 \ub3c5\uc77c \ub178\ub3d9\uc790\ub2f9 (\ub098\uce58\ub2f9)\uc758 \ud1b5\uce58 \ud558\uc5d0 \uc788\ub358 \ub3c5\uc77c \uc774\uc678\uc758 \ub098\ub77c\uc5d0\uc11c\ub294 \ud070 \uc131\uacf5\uc744 \uac70\ub450\uc9c0 \ubabb\ud588\ub2e4. \ub098\uce58 \ub3c5\uc77c\uc758 \uae08\uc5f0 \uc6b4\ub3d9\uc740 1930\ub144\ub300\uc640 1940\ub144\ub300 \ucd08\ubc18\uc5d0 \uc788\uc5c8\ub358 \uae08\uc5f0 \uc6b4\ub3d9 \uac00\uc6b4\ub370 \uc138\uacc4\uc5d0\uc11c \uac00\uc7a5 \uac15\ub825\ud588\ub358 \uae08\uc5f0 \uc6b4\ub3d9\uc774\uc5c8\ub2e4. \ub098\uce58\ub2f9\uc758 \uc9c0\ub3c4\uc790\ub4e4\uc740 \ud761\uc5f0\uc744 \ube44\ub09c\ud588\uc73c\uba70 \uc774\ub4e4 \uc911 \uc77c\ubd80\ub294 \ub2f4\ubc30 \uc18c\ube44\ub97c \uacf5\uacf5\uc5f0\ud788 \ube44\ub09c\ud558\uae30\ub3c4 \ud588\ub2e4. \ud761\uc5f0\uc774 \uac74\uac15\uc5d0 \ubbf8\uce58\ub294 \uc601\ud5a5\uc5d0 \uad00\ud55c \uc5f0\uad6c\uac00 \ub098\uce58\ub2f9\uc758 \uc9c0\ub3c4 \ud558\uc5d0 \uc9c4\ud589\ub418\uc5c8\ub294\ub370 \uc774\ub294 \ub2f9\uc2dc \ub098\uce58 \ub3c5\uc77c\uc5d0\uc11c \uac00\uc7a5 \uc911\uc694\ud55c \uc5f0\uad6c \uc8fc\uc81c\uac00 \ub418\uae30\ub3c4 \ud558\uc600\ub2e4. \ub2f4\ubc30\ub97c \uc2eb\uc5b4\ud558\ub358 \uc544\ub3cc\ud504 \ud788\ud2c0\ub7ec\uc640 \ub098\uce58 \ub3c5\uc77c\uc758 \ucd9c\uc0b0 \uc7a5\ub824 \uc815\ucc45\uc740 \uae08\uc5f0 \uc6b4\ub3d9\uc744 \uc790\uadf9\ud558\ub294 \uc6d0\uc778\uc774 \ub418\uc5c8\ub294\ub370 \uc774\ub294 \uc778\uc885 \ucc28\ubcc4, \ubc18\uc720\ub300\uc8fc\uc758\uc640 \uc5f0\uad00\ub418\uc5b4 \uc788\uc5c8\ub2e4.", "sentence_answer": "\uad6d\uac00\uc0ac\ud68c\uc8fc\uc758 \ub3c5\uc77c \ub178\ub3d9\uc790\ub2f9 (\ub098\uce58\ub2f9)", "paragraph_id": "6488722-0-0", "paragraph_question": "question: 1930\ub144\ub300\ubd80\ud130 1940\ub144\ub300 \ucd08\ubc18\uc5d0 \ub2f4\ubc30 \ubc18\ub300 \uc6b4\ub3d9\uc774 \uc804\uac1c\ub420 \ub2f9\uc2dc\uc758 \ub3c5\uc77c\uc744 \ud1b5\uce58\ud558\ub358 \ub2f9\uc740?, context: \ub3c5\uc77c\uc758 \uc758\uc0ac\ub4e4\uc774 \ud761\uc5f0\uacfc \ud3d0\uc554\uc758 \uad00\uacc4\ub97c \ucc98\uc74c \ud655\uc778\ud55c \uc774\ud6c4\ubd80\ud130 \ub098\uce58 \ub3c5\uc77c\uc740 \uac15\ub825\ud55c \ubc18\ub2f4\ubc30 \uc6b4\ub3d9 \uacfc \ud604\ub300\uc0ac \ucd5c\ucd08\uc758 \uacf5\uc775\uc801\uc778 \uae08\uc5f0 \uc6b4\ub3d9\uc744 \uc804\uac1c\ud588\ub2e4. \ub2f4\ubc30 \ubc18\ub300 \uc6b4\ub3d9\uc740 20\uc138\uae30 \ucd08\ubc18\ubd80\ud130 \ub9ce\uc740 \ub098\ub77c\uc5d0\uc11c \uc804\uac1c\ub418\uc5b4 \uc654\uc9c0\ub9cc \uad6d\uac00\uc0ac\ud68c\uc8fc\uc758 \ub3c5\uc77c \ub178\ub3d9\uc790\ub2f9(\ub098\uce58\ub2f9)\uc758 \ud1b5\uce58 \ud558\uc5d0 \uc788\ub358 \ub3c5\uc77c \uc774\uc678\uc758 \ub098\ub77c\uc5d0\uc11c\ub294 \ud070 \uc131\uacf5\uc744 \uac70\ub450\uc9c0 \ubabb\ud588\ub2e4. \ub098\uce58 \ub3c5\uc77c\uc758 \uae08\uc5f0 \uc6b4\ub3d9\uc740 1930\ub144\ub300\uc640 1940\ub144\ub300 \ucd08\ubc18\uc5d0 \uc788\uc5c8\ub358 \uae08\uc5f0 \uc6b4\ub3d9 \uac00\uc6b4\ub370 \uc138\uacc4\uc5d0\uc11c \uac00\uc7a5 \uac15\ub825\ud588\ub358 \uae08\uc5f0 \uc6b4\ub3d9\uc774\uc5c8\ub2e4. \ub098\uce58\ub2f9\uc758 \uc9c0\ub3c4\uc790\ub4e4\uc740 \ud761\uc5f0\uc744 \ube44\ub09c\ud588\uc73c\uba70 \uc774\ub4e4 \uc911 \uc77c\ubd80\ub294 \ub2f4\ubc30 \uc18c\ube44\ub97c \uacf5\uacf5\uc5f0\ud788 \ube44\ub09c\ud558\uae30\ub3c4 \ud588\ub2e4. \ud761\uc5f0\uc774 \uac74\uac15\uc5d0 \ubbf8\uce58\ub294 \uc601\ud5a5\uc5d0 \uad00\ud55c \uc5f0\uad6c\uac00 \ub098\uce58\ub2f9\uc758 \uc9c0\ub3c4 \ud558\uc5d0 \uc9c4\ud589\ub418\uc5c8\ub294\ub370 \uc774\ub294 \ub2f9\uc2dc \ub098\uce58 \ub3c5\uc77c\uc5d0\uc11c \uac00\uc7a5 \uc911\uc694\ud55c \uc5f0\uad6c \uc8fc\uc81c\uac00 \ub418\uae30\ub3c4 \ud558\uc600\ub2e4. \ub2f4\ubc30\ub97c \uc2eb\uc5b4\ud558\ub358 \uc544\ub3cc\ud504 \ud788\ud2c0\ub7ec\uc640 \ub098\uce58 \ub3c5\uc77c\uc758 \ucd9c\uc0b0 \uc7a5\ub824 \uc815\ucc45\uc740 \uae08\uc5f0 \uc6b4\ub3d9\uc744 \uc790\uadf9\ud558\ub294 \uc6d0\uc778\uc774 \ub418\uc5c8\ub294\ub370 \uc774\ub294 \uc778\uc885 \ucc28\ubcc4, \ubc18\uc720\ub300\uc8fc\uc758\uc640 \uc5f0\uad00\ub418\uc5b4 \uc788\uc5c8\ub2e4."} {"question": "\uc678\uad6d\ub3c4 \uc18c\ubc29\uc740 \uc9c0\ubc29\uc0ac\ubb34\uc774\uace0 1\ucc28 \ud654\uc7ac\ub300\uc751\ucc98\uac00 \uc9c0\uc790\uccb4\ub77c\uba70 \uc18c\ubc29\uc9c1\uc744 \uad6d\uac00\uc9c1\uc73c\ub85c \ud558\ub294 \uac83\uc774 \ube44\ud6a8\uc728\uc801\uc774\ub77c\uace0 \ub9d0\ud55c \ubd80\ucc98\ub294?", "paragraph": "\uadf8\ub7ec\ub098 2014\ub144 5\uc6d4 28\uc77c \uc548\uc804\ud589\uc815\ubd80\ub294 \"\uc678\uad6d\ub3c4 \uc18c\ubc29\uc740 \uc9c0\ubc29\uc0ac\ubb34\uc774\uace0, \ud654\uc7ac\uc5d0 \uc77c\ucc28\ub85c \ub300\uc751\ud558\ub294 \uacf3\uc774 \uc9c0\ucc28\uccb4\uc774\uba70, \uad6d\uac00\uc548\uc804\ucc98\uc758 \uc2e0\uc124\ub85c \uc18c\ubc29\uc9c1\uc744 \uad6d\uac00\uc9c1\uc73c\ub85c \ud558\ub294 \uac83\uc740 \ube44\ud6a8\uc728\uc774\ub2e4\"\ub77c\uace0 \ub9d0\ud588\uc73c\ub098, \uc18c\ubc29\ubc29\uc7ac\uccad\uc758 \ud574\uccb4\ub97c \ub9c9\uc544\ub2ec\ub77c\ub294 \uae00\uc744 \uc62c\ub9b0 \uc18c\ubc29\uad00\uc740 \"\uac00\uc7a5 \uc120\uc9c4\ud654\ub41c \ubbf8\uad6d\uacfc \uc77c\ubcf8\uc758 \uc18c\ubc29\uc744 \ub9d0\ud558\uc790\uba74, \ubbf8\uad6d\uc740 \uc8fc \uc0ac\ubb34\uc778\ub370, \uadf8 \uc8fc\ub294 \ub300\ud55c\ubbfc\uad6d\ubcf4\ub2e4 \ud070 \uc8fc\uac00 \ub300\ubd80\ubd84\uc774\ub77c\ub294 \uac83\uc740 \uc0c1\uc2dd\"\uc774\ub77c\uba74\uc11c \"\uc8fc\ub97c \uad6d\uac00 \uac1c\ub150\uc73c\ub85c \ubd10\uc57c \ud55c\ub2e4\"\uace0 \uc8fc\uc7a5\ud588\ub2e4. \uc774\uc5b4\uc11c \"\uc77c\ubcf8\uc740 \uc2dc\u00b7\uc815\u00b7\ucd0c\uc758 \uc18c\ubc29\uc11c\uac00 \uac01\uc885 \uc7ac\ub09c\uc7ac\ud574\uc5d0 \ubaa8\ub450 \ub300\uc751\ud55c\ub2e4\"\uba74\uc11c \"\ub300\ud55c\ubbfc\uad6d\uacfc\ub294 \ub2e4\ub974\ub2e4. \uc18c\ubc29\uc11c\uac00 \ubaa8\ub4e0 \ubc29\uc7ac\uc5c5\ubb34\ub97c \ud3ec\uad04\ud574 \uc774\ub8e8\uc5b4\uc9c0\ub294 \uac15\ub825\ud55c \uc18c\ubc29\ud589\uc815\uccb4\uc81c\ub2e4. \uc548\uc804\ucd1d\uad04\uacfc\ub77c\ub294 \ud589\uc815\uad00\ub8cc\ub4e4\ub85c \uc774\ub8e8\uc5b4\uc9c4 \uc870\uc9c1\uc774 '\uc790\ub8cc \uc81c\ucd9c\ud558\ub77c, \uacc4\ud68d \uc218\ub9bd\ud558\ub77c'\ub294 '\ud398\uc774\ud37c \uc6cc\ud06c' \ud558\ub294 \uac83\uacfc \ud574\uc591\uc7ac\ub09c\uc740 \ud574\uacbd, \uc721\uc0c1\uc7ac\ub09c\uc740 \uc18c\ubc29\uc73c\ub85c \ubd84\ub9ac\ud55c \uac83\uacfc\ub294 \ucc38 \ub9ce\uc774 \ub2e4\ub974\ub2e4\"\uace0 \uc8fc\uc7a5\ud588\ub2e4.", "answer": "\uc548\uc804\ud589\uc815\ubd80", "sentence": "\uadf8\ub7ec\ub098 2014\ub144 5\uc6d4 28\uc77c \uc548\uc804\ud589\uc815\ubd80 \ub294 \"\uc678\uad6d\ub3c4 \uc18c\ubc29\uc740 \uc9c0\ubc29\uc0ac\ubb34\uc774\uace0, \ud654\uc7ac\uc5d0 \uc77c\ucc28\ub85c \ub300\uc751\ud558\ub294 \uacf3\uc774 \uc9c0\ucc28\uccb4\uc774\uba70, \uad6d\uac00\uc548\uc804\ucc98\uc758 \uc2e0\uc124\ub85c \uc18c\ubc29\uc9c1\uc744 \uad6d\uac00\uc9c1\uc73c\ub85c \ud558\ub294 \uac83\uc740 \ube44\ud6a8\uc728\uc774\ub2e4\"\ub77c\uace0 \ub9d0\ud588\uc73c\ub098, \uc18c\ubc29\ubc29\uc7ac\uccad\uc758 \ud574\uccb4\ub97c \ub9c9\uc544\ub2ec\ub77c\ub294 \uae00\uc744 \uc62c\ub9b0 \uc18c\ubc29\uad00\uc740 \"\uac00\uc7a5 \uc120\uc9c4\ud654\ub41c \ubbf8\uad6d\uacfc \uc77c\ubcf8\uc758 \uc18c\ubc29\uc744 \ub9d0\ud558\uc790\uba74, \ubbf8\uad6d\uc740 \uc8fc \uc0ac\ubb34\uc778\ub370, \uadf8 \uc8fc\ub294 \ub300\ud55c\ubbfc\uad6d\ubcf4\ub2e4 \ud070 \uc8fc\uac00 \ub300\ubd80\ubd84\uc774\ub77c\ub294 \uac83\uc740 \uc0c1\uc2dd\"\uc774\ub77c\uba74\uc11c \"\uc8fc\ub97c \uad6d\uac00 \uac1c\ub150\uc73c\ub85c \ubd10\uc57c \ud55c\ub2e4\"\uace0 \uc8fc\uc7a5\ud588\ub2e4.", "paragraph_sentence": " \uadf8\ub7ec\ub098 2014\ub144 5\uc6d4 28\uc77c \uc548\uc804\ud589\uc815\ubd80 \ub294 \"\uc678\uad6d\ub3c4 \uc18c\ubc29\uc740 \uc9c0\ubc29\uc0ac\ubb34\uc774\uace0, \ud654\uc7ac\uc5d0 \uc77c\ucc28\ub85c \ub300\uc751\ud558\ub294 \uacf3\uc774 \uc9c0\ucc28\uccb4\uc774\uba70, \uad6d\uac00\uc548\uc804\ucc98\uc758 \uc2e0\uc124\ub85c \uc18c\ubc29\uc9c1\uc744 \uad6d\uac00\uc9c1\uc73c\ub85c \ud558\ub294 \uac83\uc740 \ube44\ud6a8\uc728\uc774\ub2e4\"\ub77c\uace0 \ub9d0\ud588\uc73c\ub098, \uc18c\ubc29\ubc29\uc7ac\uccad\uc758 \ud574\uccb4\ub97c \ub9c9\uc544\ub2ec\ub77c\ub294 \uae00\uc744 \uc62c\ub9b0 \uc18c\ubc29\uad00\uc740 \"\uac00\uc7a5 \uc120\uc9c4\ud654\ub41c \ubbf8\uad6d\uacfc \uc77c\ubcf8\uc758 \uc18c\ubc29\uc744 \ub9d0\ud558\uc790\uba74, \ubbf8\uad6d\uc740 \uc8fc \uc0ac\ubb34\uc778\ub370, \uadf8 \uc8fc\ub294 \ub300\ud55c\ubbfc\uad6d\ubcf4\ub2e4 \ud070 \uc8fc\uac00 \ub300\ubd80\ubd84\uc774\ub77c\ub294 \uac83\uc740 \uc0c1\uc2dd\"\uc774\ub77c\uba74\uc11c \"\uc8fc\ub97c \uad6d\uac00 \uac1c\ub150\uc73c\ub85c \ubd10\uc57c \ud55c\ub2e4\"\uace0 \uc8fc\uc7a5\ud588\ub2e4. \uc774\uc5b4\uc11c \"\uc77c\ubcf8\uc740 \uc2dc\u00b7\uc815\u00b7\ucd0c\uc758 \uc18c\ubc29\uc11c\uac00 \uac01\uc885 \uc7ac\ub09c\uc7ac\ud574\uc5d0 \ubaa8\ub450 \ub300\uc751\ud55c\ub2e4\"\uba74\uc11c \"\ub300\ud55c\ubbfc\uad6d\uacfc\ub294 \ub2e4\ub974\ub2e4. \uc18c\ubc29\uc11c\uac00 \ubaa8\ub4e0 \ubc29\uc7ac\uc5c5\ubb34\ub97c \ud3ec\uad04\ud574 \uc774\ub8e8\uc5b4\uc9c0\ub294 \uac15\ub825\ud55c \uc18c\ubc29\ud589\uc815\uccb4\uc81c\ub2e4. \uc548\uc804\ucd1d\uad04\uacfc\ub77c\ub294 \ud589\uc815\uad00\ub8cc\ub4e4\ub85c \uc774\ub8e8\uc5b4\uc9c4 \uc870\uc9c1\uc774 '\uc790\ub8cc \uc81c\ucd9c\ud558\ub77c, \uacc4\ud68d \uc218\ub9bd\ud558\ub77c'\ub294 '\ud398\uc774\ud37c \uc6cc\ud06c' \ud558\ub294 \uac83\uacfc \ud574\uc591\uc7ac\ub09c\uc740 \ud574\uacbd, \uc721\uc0c1\uc7ac\ub09c\uc740 \uc18c\ubc29\uc73c\ub85c \ubd84\ub9ac\ud55c \uac83\uacfc\ub294 \ucc38 \ub9ce\uc774 \ub2e4\ub974\ub2e4\"\uace0 \uc8fc\uc7a5\ud588\ub2e4.", "paragraph_answer": "\uadf8\ub7ec\ub098 2014\ub144 5\uc6d4 28\uc77c \uc548\uc804\ud589\uc815\ubd80 \ub294 \"\uc678\uad6d\ub3c4 \uc18c\ubc29\uc740 \uc9c0\ubc29\uc0ac\ubb34\uc774\uace0, \ud654\uc7ac\uc5d0 \uc77c\ucc28\ub85c \ub300\uc751\ud558\ub294 \uacf3\uc774 \uc9c0\ucc28\uccb4\uc774\uba70, \uad6d\uac00\uc548\uc804\ucc98\uc758 \uc2e0\uc124\ub85c \uc18c\ubc29\uc9c1\uc744 \uad6d\uac00\uc9c1\uc73c\ub85c \ud558\ub294 \uac83\uc740 \ube44\ud6a8\uc728\uc774\ub2e4\"\ub77c\uace0 \ub9d0\ud588\uc73c\ub098, \uc18c\ubc29\ubc29\uc7ac\uccad\uc758 \ud574\uccb4\ub97c \ub9c9\uc544\ub2ec\ub77c\ub294 \uae00\uc744 \uc62c\ub9b0 \uc18c\ubc29\uad00\uc740 \"\uac00\uc7a5 \uc120\uc9c4\ud654\ub41c \ubbf8\uad6d\uacfc \uc77c\ubcf8\uc758 \uc18c\ubc29\uc744 \ub9d0\ud558\uc790\uba74, \ubbf8\uad6d\uc740 \uc8fc \uc0ac\ubb34\uc778\ub370, \uadf8 \uc8fc\ub294 \ub300\ud55c\ubbfc\uad6d\ubcf4\ub2e4 \ud070 \uc8fc\uac00 \ub300\ubd80\ubd84\uc774\ub77c\ub294 \uac83\uc740 \uc0c1\uc2dd\"\uc774\ub77c\uba74\uc11c \"\uc8fc\ub97c \uad6d\uac00 \uac1c\ub150\uc73c\ub85c \ubd10\uc57c \ud55c\ub2e4\"\uace0 \uc8fc\uc7a5\ud588\ub2e4. \uc774\uc5b4\uc11c \"\uc77c\ubcf8\uc740 \uc2dc\u00b7\uc815\u00b7\ucd0c\uc758 \uc18c\ubc29\uc11c\uac00 \uac01\uc885 \uc7ac\ub09c\uc7ac\ud574\uc5d0 \ubaa8\ub450 \ub300\uc751\ud55c\ub2e4\"\uba74\uc11c \"\ub300\ud55c\ubbfc\uad6d\uacfc\ub294 \ub2e4\ub974\ub2e4. \uc18c\ubc29\uc11c\uac00 \ubaa8\ub4e0 \ubc29\uc7ac\uc5c5\ubb34\ub97c \ud3ec\uad04\ud574 \uc774\ub8e8\uc5b4\uc9c0\ub294 \uac15\ub825\ud55c \uc18c\ubc29\ud589\uc815\uccb4\uc81c\ub2e4. \uc548\uc804\ucd1d\uad04\uacfc\ub77c\ub294 \ud589\uc815\uad00\ub8cc\ub4e4\ub85c \uc774\ub8e8\uc5b4\uc9c4 \uc870\uc9c1\uc774 '\uc790\ub8cc \uc81c\ucd9c\ud558\ub77c, \uacc4\ud68d \uc218\ub9bd\ud558\ub77c'\ub294 '\ud398\uc774\ud37c \uc6cc\ud06c' \ud558\ub294 \uac83\uacfc \ud574\uc591\uc7ac\ub09c\uc740 \ud574\uacbd, \uc721\uc0c1\uc7ac\ub09c\uc740 \uc18c\ubc29\uc73c\ub85c \ubd84\ub9ac\ud55c \uac83\uacfc\ub294 \ucc38 \ub9ce\uc774 \ub2e4\ub974\ub2e4\"\uace0 \uc8fc\uc7a5\ud588\ub2e4.", "sentence_answer": "\uadf8\ub7ec\ub098 2014\ub144 5\uc6d4 28\uc77c \uc548\uc804\ud589\uc815\ubd80 \ub294 \"\uc678\uad6d\ub3c4 \uc18c\ubc29\uc740 \uc9c0\ubc29\uc0ac\ubb34\uc774\uace0, \ud654\uc7ac\uc5d0 \uc77c\ucc28\ub85c \ub300\uc751\ud558\ub294 \uacf3\uc774 \uc9c0\ucc28\uccb4\uc774\uba70, \uad6d\uac00\uc548\uc804\ucc98\uc758 \uc2e0\uc124\ub85c \uc18c\ubc29\uc9c1\uc744 \uad6d\uac00\uc9c1\uc73c\ub85c \ud558\ub294 \uac83\uc740 \ube44\ud6a8\uc728\uc774\ub2e4\"\ub77c\uace0 \ub9d0\ud588\uc73c\ub098, \uc18c\ubc29\ubc29\uc7ac\uccad\uc758 \ud574\uccb4\ub97c \ub9c9\uc544\ub2ec\ub77c\ub294 \uae00\uc744 \uc62c\ub9b0 \uc18c\ubc29\uad00\uc740 \"\uac00\uc7a5 \uc120\uc9c4\ud654\ub41c \ubbf8\uad6d\uacfc \uc77c\ubcf8\uc758 \uc18c\ubc29\uc744 \ub9d0\ud558\uc790\uba74, \ubbf8\uad6d\uc740 \uc8fc \uc0ac\ubb34\uc778\ub370, \uadf8 \uc8fc\ub294 \ub300\ud55c\ubbfc\uad6d\ubcf4\ub2e4 \ud070 \uc8fc\uac00 \ub300\ubd80\ubd84\uc774\ub77c\ub294 \uac83\uc740 \uc0c1\uc2dd\"\uc774\ub77c\uba74\uc11c \"\uc8fc\ub97c \uad6d\uac00 \uac1c\ub150\uc73c\ub85c \ubd10\uc57c \ud55c\ub2e4\"\uace0 \uc8fc\uc7a5\ud588\ub2e4.", "paragraph_id": "6486393-7-0", "paragraph_question": "question: \uc678\uad6d\ub3c4 \uc18c\ubc29\uc740 \uc9c0\ubc29\uc0ac\ubb34\uc774\uace0 1\ucc28 \ud654\uc7ac\ub300\uc751\ucc98\uac00 \uc9c0\uc790\uccb4\ub77c\uba70 \uc18c\ubc29\uc9c1\uc744 \uad6d\uac00\uc9c1\uc73c\ub85c \ud558\ub294 \uac83\uc774 \ube44\ud6a8\uc728\uc801\uc774\ub77c\uace0 \ub9d0\ud55c \ubd80\ucc98\ub294?, context: \uadf8\ub7ec\ub098 2014\ub144 5\uc6d4 28\uc77c \uc548\uc804\ud589\uc815\ubd80\ub294 \"\uc678\uad6d\ub3c4 \uc18c\ubc29\uc740 \uc9c0\ubc29\uc0ac\ubb34\uc774\uace0, \ud654\uc7ac\uc5d0 \uc77c\ucc28\ub85c \ub300\uc751\ud558\ub294 \uacf3\uc774 \uc9c0\ucc28\uccb4\uc774\uba70, \uad6d\uac00\uc548\uc804\ucc98\uc758 \uc2e0\uc124\ub85c \uc18c\ubc29\uc9c1\uc744 \uad6d\uac00\uc9c1\uc73c\ub85c \ud558\ub294 \uac83\uc740 \ube44\ud6a8\uc728\uc774\ub2e4\"\ub77c\uace0 \ub9d0\ud588\uc73c\ub098, \uc18c\ubc29\ubc29\uc7ac\uccad\uc758 \ud574\uccb4\ub97c \ub9c9\uc544\ub2ec\ub77c\ub294 \uae00\uc744 \uc62c\ub9b0 \uc18c\ubc29\uad00\uc740 \"\uac00\uc7a5 \uc120\uc9c4\ud654\ub41c \ubbf8\uad6d\uacfc \uc77c\ubcf8\uc758 \uc18c\ubc29\uc744 \ub9d0\ud558\uc790\uba74, \ubbf8\uad6d\uc740 \uc8fc \uc0ac\ubb34\uc778\ub370, \uadf8 \uc8fc\ub294 \ub300\ud55c\ubbfc\uad6d\ubcf4\ub2e4 \ud070 \uc8fc\uac00 \ub300\ubd80\ubd84\uc774\ub77c\ub294 \uac83\uc740 \uc0c1\uc2dd\"\uc774\ub77c\uba74\uc11c \"\uc8fc\ub97c \uad6d\uac00 \uac1c\ub150\uc73c\ub85c \ubd10\uc57c \ud55c\ub2e4\"\uace0 \uc8fc\uc7a5\ud588\ub2e4. \uc774\uc5b4\uc11c \"\uc77c\ubcf8\uc740 \uc2dc\u00b7\uc815\u00b7\ucd0c\uc758 \uc18c\ubc29\uc11c\uac00 \uac01\uc885 \uc7ac\ub09c\uc7ac\ud574\uc5d0 \ubaa8\ub450 \ub300\uc751\ud55c\ub2e4\"\uba74\uc11c \"\ub300\ud55c\ubbfc\uad6d\uacfc\ub294 \ub2e4\ub974\ub2e4. \uc18c\ubc29\uc11c\uac00 \ubaa8\ub4e0 \ubc29\uc7ac\uc5c5\ubb34\ub97c \ud3ec\uad04\ud574 \uc774\ub8e8\uc5b4\uc9c0\ub294 \uac15\ub825\ud55c \uc18c\ubc29\ud589\uc815\uccb4\uc81c\ub2e4. \uc548\uc804\ucd1d\uad04\uacfc\ub77c\ub294 \ud589\uc815\uad00\ub8cc\ub4e4\ub85c \uc774\ub8e8\uc5b4\uc9c4 \uc870\uc9c1\uc774 '\uc790\ub8cc \uc81c\ucd9c\ud558\ub77c, \uacc4\ud68d \uc218\ub9bd\ud558\ub77c'\ub294 '\ud398\uc774\ud37c \uc6cc\ud06c' \ud558\ub294 \uac83\uacfc \ud574\uc591\uc7ac\ub09c\uc740 \ud574\uacbd, \uc721\uc0c1\uc7ac\ub09c\uc740 \uc18c\ubc29\uc73c\ub85c \ubd84\ub9ac\ud55c \uac83\uacfc\ub294 \ucc38 \ub9ce\uc774 \ub2e4\ub974\ub2e4\"\uace0 \uc8fc\uc7a5\ud588\ub2e4."} {"question": "\ud45c\ucc3d\uc6d0\uc758 \ubc15\uc0ac\ud559\uc704 \ub17c\ubb38\uc774 \ud45c\uc808\uc774\ub77c\uace0 \uc758\ud639\uc744 \uc81c\uae30\ud55c \uc0ac\ub78c\uc740?", "paragraph": "2013\ub144 7\uc6d4, \ud45c\ucc3d\uc6d0\uc740 \ubcc0\ud76c\uc7ac \ub4f1\uc774 \uc81c\uae30\ud55c \ubc15\uc0ac\ud559\uc704 \ub17c\ubb38 \ud45c\uc808 \uc758\ud639\uc774 \uc0ac\uc2e4\uc774\ub77c\uace0 \uc778\uc815\ud588\ub2e4. \ub17c\ubb38 \uc911 \uc77c\ubd80 \ubd80\ubd84\uc774 \uc778\uc6a9\ud45c\uae30\uc5d0 \uc624\ub958\uac00 \uc788\uc5c8\ub2e4. 2013\ub144 7\uc6d4 7\uc77c, \ud45c\ucc3d\uc6d0\uc740 \"\uc9c1\uc811\uc801\uc778 \uc778\uc6a9\uc744 \ucd9c\ucc98 \ud45c\uc2dc\ub9cc \ud55c \ucc44 \uac04\uc811\uc778\uc6a9 \ud615\ud0dc\ub85c \uc798\ubabb \ud45c\uae30\ud588\ub2e4. \uc81c \ubc15\uc0ac\ub17c\ubb38\uc5d0 \ud45c\uc808\ubd80\ubd84\uc774 \uc788\ub2e4\ub294 \uc0ac\uc2e4\uc5d0 \uc2e4\ub9dd\ud558\uace0 \ubd84\ub178\ud55c \ubd84\uc774 \uacc4\uc2dc\ub2e4\uba74 \uc815\uc911\ud558\uac8c \uc0ac\uacfc\ub4dc\ub9ac\uaca0\ub2e4\"\uace0 \ubc1d\ud614\ub2e4. 2013\ub144 7\uc6d4 9\uc77c, \ud45c\ucc3d\uc6d0\uc740 \uc790\uc2e0\uc758 \ube14\ub85c\uadf8\ub97c \ud1b5\ud574 \uc790\uc2e0\uc758 \ubc15\uc0ac\ud559\uc704 \uc9c0\ub3c4\uad50\uc218\uc600\ub358 \uc601\uad6d \uc5d1\uc11c\ud130 \ub300\ud559\uad50\uc758 \ub9c8\uc774\ud074 \ub7ec\uc26c\ub85c\ubd80\ud130 \"\ubb38\uc81c\uac00 \ubc1c\uc0dd\ud558\uc9c0 \uc54a\uc744 \uac83\uc73c\ub85c \uc0dd\uac01\ub41c\ub2e4. \uc815\uc624\ud45c\ub97c \uc81c\ucd9c\ud558\uac70\ub098 \uc628\ub77c\uc778\uc0c1 \ub17c\ubb38\uc744 \uc218\uc815\ud558\ub294 \uac83\uc73c\ub85c \ucda9\ubd84\ud558\ub2e4\uace0 \ud310\ub2e8\ud55c\ub2e4\"\ub294 \ub2f5\ubcc0\uc744 \ubc1b\uc558\ub2e4\uace0 \ubc1d\ud614\ub2e4. \uc774\ud6c4\uc5d0\ub3c4 \ubcc0\ud76c\uc7ac\ub294 \ucd94\uac00 \ub17c\ubb38 \ud45c\uc808\uc744 \ubc1c\uacac\ud558\uace0 \uc774\ub97c \ud45c\ucc3d\uc6d0\uc5d0\uac8c \uc54c\ub838\uc9c0\ub9cc, \ud45c\ucc3d\uc6d0\uc740 \uc774\ub294 \uc774\ubbf8 \uc815\ub9ac\ub41c \ubb38\uc81c\ub77c \ud558\uace0 \ub2ec\ub9ac \ubc18\uc751\uc740 \uc548\ud588\ub2e4.", "answer": "\ubcc0\ud76c\uc7ac", "sentence": "2013\ub144 7\uc6d4, \ud45c\ucc3d\uc6d0\uc740 \ubcc0\ud76c\uc7ac \ub4f1\uc774 \uc81c\uae30\ud55c \ubc15\uc0ac\ud559\uc704 \ub17c\ubb38 \ud45c\uc808 \uc758\ud639\uc774 \uc0ac\uc2e4\uc774\ub77c\uace0 \uc778\uc815\ud588\ub2e4.", "paragraph_sentence": " 2013\ub144 7\uc6d4, \ud45c\ucc3d\uc6d0\uc740 \ubcc0\ud76c\uc7ac \ub4f1\uc774 \uc81c\uae30\ud55c \ubc15\uc0ac\ud559\uc704 \ub17c\ubb38 \ud45c\uc808 \uc758\ud639\uc774 \uc0ac\uc2e4\uc774\ub77c\uace0 \uc778\uc815\ud588\ub2e4. \ub17c\ubb38 \uc911 \uc77c\ubd80 \ubd80\ubd84\uc774 \uc778\uc6a9\ud45c\uae30\uc5d0 \uc624\ub958\uac00 \uc788\uc5c8\ub2e4. 2013\ub144 7\uc6d4 7\uc77c, \ud45c\ucc3d\uc6d0\uc740 \"\uc9c1\uc811\uc801\uc778 \uc778\uc6a9\uc744 \ucd9c\ucc98 \ud45c\uc2dc\ub9cc \ud55c \ucc44 \uac04\uc811\uc778\uc6a9 \ud615\ud0dc\ub85c \uc798\ubabb \ud45c\uae30\ud588\ub2e4. \uc81c \ubc15\uc0ac\ub17c\ubb38\uc5d0 \ud45c\uc808\ubd80\ubd84\uc774 \uc788\ub2e4\ub294 \uc0ac\uc2e4\uc5d0 \uc2e4\ub9dd\ud558\uace0 \ubd84\ub178\ud55c \ubd84\uc774 \uacc4\uc2dc\ub2e4\uba74 \uc815\uc911\ud558\uac8c \uc0ac\uacfc\ub4dc\ub9ac\uaca0\ub2e4\"\uace0 \ubc1d\ud614\ub2e4. 2013\ub144 7\uc6d4 9\uc77c, \ud45c\ucc3d\uc6d0\uc740 \uc790\uc2e0\uc758 \ube14\ub85c\uadf8\ub97c \ud1b5\ud574 \uc790\uc2e0\uc758 \ubc15\uc0ac\ud559\uc704 \uc9c0\ub3c4\uad50\uc218\uc600\ub358 \uc601\uad6d \uc5d1\uc11c\ud130 \ub300\ud559\uad50\uc758 \ub9c8\uc774\ud074 \ub7ec\uc26c\ub85c\ubd80\ud130 \"\ubb38\uc81c\uac00 \ubc1c\uc0dd\ud558\uc9c0 \uc54a\uc744 \uac83\uc73c\ub85c \uc0dd\uac01\ub41c\ub2e4. \uc815\uc624\ud45c\ub97c \uc81c\ucd9c\ud558\uac70\ub098 \uc628\ub77c\uc778\uc0c1 \ub17c\ubb38\uc744 \uc218\uc815\ud558\ub294 \uac83\uc73c\ub85c \ucda9\ubd84\ud558\ub2e4\uace0 \ud310\ub2e8\ud55c\ub2e4\"\ub294 \ub2f5\ubcc0\uc744 \ubc1b\uc558\ub2e4\uace0 \ubc1d\ud614\ub2e4. \uc774\ud6c4\uc5d0\ub3c4 \ubcc0\ud76c\uc7ac\ub294 \ucd94\uac00 \ub17c\ubb38 \ud45c\uc808\uc744 \ubc1c\uacac\ud558\uace0 \uc774\ub97c \ud45c\ucc3d\uc6d0\uc5d0\uac8c \uc54c\ub838\uc9c0\ub9cc, \ud45c\ucc3d\uc6d0\uc740 \uc774\ub294 \uc774\ubbf8 \uc815\ub9ac\ub41c \ubb38\uc81c\ub77c \ud558\uace0 \ub2ec\ub9ac \ubc18\uc751\uc740 \uc548\ud588\ub2e4.", "paragraph_answer": "2013\ub144 7\uc6d4, \ud45c\ucc3d\uc6d0\uc740 \ubcc0\ud76c\uc7ac \ub4f1\uc774 \uc81c\uae30\ud55c \ubc15\uc0ac\ud559\uc704 \ub17c\ubb38 \ud45c\uc808 \uc758\ud639\uc774 \uc0ac\uc2e4\uc774\ub77c\uace0 \uc778\uc815\ud588\ub2e4. \ub17c\ubb38 \uc911 \uc77c\ubd80 \ubd80\ubd84\uc774 \uc778\uc6a9\ud45c\uae30\uc5d0 \uc624\ub958\uac00 \uc788\uc5c8\ub2e4. 2013\ub144 7\uc6d4 7\uc77c, \ud45c\ucc3d\uc6d0\uc740 \"\uc9c1\uc811\uc801\uc778 \uc778\uc6a9\uc744 \ucd9c\ucc98 \ud45c\uc2dc\ub9cc \ud55c \ucc44 \uac04\uc811\uc778\uc6a9 \ud615\ud0dc\ub85c \uc798\ubabb \ud45c\uae30\ud588\ub2e4. \uc81c \ubc15\uc0ac\ub17c\ubb38\uc5d0 \ud45c\uc808\ubd80\ubd84\uc774 \uc788\ub2e4\ub294 \uc0ac\uc2e4\uc5d0 \uc2e4\ub9dd\ud558\uace0 \ubd84\ub178\ud55c \ubd84\uc774 \uacc4\uc2dc\ub2e4\uba74 \uc815\uc911\ud558\uac8c \uc0ac\uacfc\ub4dc\ub9ac\uaca0\ub2e4\"\uace0 \ubc1d\ud614\ub2e4. 2013\ub144 7\uc6d4 9\uc77c, \ud45c\ucc3d\uc6d0\uc740 \uc790\uc2e0\uc758 \ube14\ub85c\uadf8\ub97c \ud1b5\ud574 \uc790\uc2e0\uc758 \ubc15\uc0ac\ud559\uc704 \uc9c0\ub3c4\uad50\uc218\uc600\ub358 \uc601\uad6d \uc5d1\uc11c\ud130 \ub300\ud559\uad50\uc758 \ub9c8\uc774\ud074 \ub7ec\uc26c\ub85c\ubd80\ud130 \"\ubb38\uc81c\uac00 \ubc1c\uc0dd\ud558\uc9c0 \uc54a\uc744 \uac83\uc73c\ub85c \uc0dd\uac01\ub41c\ub2e4. \uc815\uc624\ud45c\ub97c \uc81c\ucd9c\ud558\uac70\ub098 \uc628\ub77c\uc778\uc0c1 \ub17c\ubb38\uc744 \uc218\uc815\ud558\ub294 \uac83\uc73c\ub85c \ucda9\ubd84\ud558\ub2e4\uace0 \ud310\ub2e8\ud55c\ub2e4\"\ub294 \ub2f5\ubcc0\uc744 \ubc1b\uc558\ub2e4\uace0 \ubc1d\ud614\ub2e4. \uc774\ud6c4\uc5d0\ub3c4 \ubcc0\ud76c\uc7ac\ub294 \ucd94\uac00 \ub17c\ubb38 \ud45c\uc808\uc744 \ubc1c\uacac\ud558\uace0 \uc774\ub97c \ud45c\ucc3d\uc6d0\uc5d0\uac8c \uc54c\ub838\uc9c0\ub9cc, \ud45c\ucc3d\uc6d0\uc740 \uc774\ub294 \uc774\ubbf8 \uc815\ub9ac\ub41c \ubb38\uc81c\ub77c \ud558\uace0 \ub2ec\ub9ac \ubc18\uc751\uc740 \uc548\ud588\ub2e4.", "sentence_answer": "2013\ub144 7\uc6d4, \ud45c\ucc3d\uc6d0\uc740 \ubcc0\ud76c\uc7ac \ub4f1\uc774 \uc81c\uae30\ud55c \ubc15\uc0ac\ud559\uc704 \ub17c\ubb38 \ud45c\uc808 \uc758\ud639\uc774 \uc0ac\uc2e4\uc774\ub77c\uace0 \uc778\uc815\ud588\ub2e4.", "paragraph_id": "6465714-2-1", "paragraph_question": "question: \ud45c\ucc3d\uc6d0\uc758 \ubc15\uc0ac\ud559\uc704 \ub17c\ubb38\uc774 \ud45c\uc808\uc774\ub77c\uace0 \uc758\ud639\uc744 \uc81c\uae30\ud55c \uc0ac\ub78c\uc740?, context: 2013\ub144 7\uc6d4, \ud45c\ucc3d\uc6d0\uc740 \ubcc0\ud76c\uc7ac \ub4f1\uc774 \uc81c\uae30\ud55c \ubc15\uc0ac\ud559\uc704 \ub17c\ubb38 \ud45c\uc808 \uc758\ud639\uc774 \uc0ac\uc2e4\uc774\ub77c\uace0 \uc778\uc815\ud588\ub2e4. \ub17c\ubb38 \uc911 \uc77c\ubd80 \ubd80\ubd84\uc774 \uc778\uc6a9\ud45c\uae30\uc5d0 \uc624\ub958\uac00 \uc788\uc5c8\ub2e4. 2013\ub144 7\uc6d4 7\uc77c, \ud45c\ucc3d\uc6d0\uc740 \"\uc9c1\uc811\uc801\uc778 \uc778\uc6a9\uc744 \ucd9c\ucc98 \ud45c\uc2dc\ub9cc \ud55c \ucc44 \uac04\uc811\uc778\uc6a9 \ud615\ud0dc\ub85c \uc798\ubabb \ud45c\uae30\ud588\ub2e4. \uc81c \ubc15\uc0ac\ub17c\ubb38\uc5d0 \ud45c\uc808\ubd80\ubd84\uc774 \uc788\ub2e4\ub294 \uc0ac\uc2e4\uc5d0 \uc2e4\ub9dd\ud558\uace0 \ubd84\ub178\ud55c \ubd84\uc774 \uacc4\uc2dc\ub2e4\uba74 \uc815\uc911\ud558\uac8c \uc0ac\uacfc\ub4dc\ub9ac\uaca0\ub2e4\"\uace0 \ubc1d\ud614\ub2e4. 2013\ub144 7\uc6d4 9\uc77c, \ud45c\ucc3d\uc6d0\uc740 \uc790\uc2e0\uc758 \ube14\ub85c\uadf8\ub97c \ud1b5\ud574 \uc790\uc2e0\uc758 \ubc15\uc0ac\ud559\uc704 \uc9c0\ub3c4\uad50\uc218\uc600\ub358 \uc601\uad6d \uc5d1\uc11c\ud130 \ub300\ud559\uad50\uc758 \ub9c8\uc774\ud074 \ub7ec\uc26c\ub85c\ubd80\ud130 \"\ubb38\uc81c\uac00 \ubc1c\uc0dd\ud558\uc9c0 \uc54a\uc744 \uac83\uc73c\ub85c \uc0dd\uac01\ub41c\ub2e4. \uc815\uc624\ud45c\ub97c \uc81c\ucd9c\ud558\uac70\ub098 \uc628\ub77c\uc778\uc0c1 \ub17c\ubb38\uc744 \uc218\uc815\ud558\ub294 \uac83\uc73c\ub85c \ucda9\ubd84\ud558\ub2e4\uace0 \ud310\ub2e8\ud55c\ub2e4\"\ub294 \ub2f5\ubcc0\uc744 \ubc1b\uc558\ub2e4\uace0 \ubc1d\ud614\ub2e4. \uc774\ud6c4\uc5d0\ub3c4 \ubcc0\ud76c\uc7ac\ub294 \ucd94\uac00 \ub17c\ubb38 \ud45c\uc808\uc744 \ubc1c\uacac\ud558\uace0 \uc774\ub97c \ud45c\ucc3d\uc6d0\uc5d0\uac8c \uc54c\ub838\uc9c0\ub9cc, \ud45c\ucc3d\uc6d0\uc740 \uc774\ub294 \uc774\ubbf8 \uc815\ub9ac\ub41c \ubb38\uc81c\ub77c \ud558\uace0 \ub2ec\ub9ac \ubc18\uc751\uc740 \uc548\ud588\ub2e4."} {"question": "\uc67c\ucabd \uc190\ubaa9 \uace8\uc808 \uc774\ud6c4 \uc7ac\ud65c\ud6c8\ub828\uc5d0\uc11c \uc5b4\ub5a4 \ud22c\uad6c\ub97c \uc774\uc6a9\ud574 \ub358\uc84c\ub294\uac00?", "paragraph": "1992\ub144 4\uc6d4 19\uc77c\uc5d0 \uc5f4\ub9b0 \uc694\ubbf8\uc6b0\ub9ac\uc804\uc5d0\uc11c \ub545\ubcfc \ud0c0\uad6c\uac00 \uc67c\ucabd \uc190\ubaa9\uc5d0 \ub9de\uc544\uc11c \uace8\uc808\ub418\ub294 \ubd80\uc0c1\uc744 \ub2f9\ud574 3\uac1c\uc6d4 \ub4a4\uc778 \uc5ec\ub984\uc5d0 \ubcf5\uadc0\ud588\ub2e4. \uace8\uc808\uc774 \ub098\uc740 \uc9c1\ud6c4\uc5d0\ub294 \uc545\ub825\uc774 \uc880\ucc98\ub7fc \ub3cc\uc544\uc624\uc9c0 \uc54a\uc544\uc11c \uac00\ubccd\uac8c \uce90\uce58\ubcfc\uc744 \ud574\ub3c4 \uc190\ubaa9\uc5d0 \ub0a0\uce74\ub85c\uc6b4 \ud1b5\uc99d\uc774 \uc788\uc5c8\ub2e4. \uadf8\ub7ec\ub098 \uc660\uc9c0 \ucee4\ube0c\ub9cc\uc740 \ud1b5\uc99d \uc5c6\uc774 \ub358\uc9c8 \uc218 \uc788\uc5c8\uae30 \ub54c\ubb38\uc5d0 \uc7ac\ud65c\ud6c8\ub828\uc5d0\uc11c\ub294 \ucee4\ube0c\ub9cc\uc744 \uc0ac\uc6a9\ud55c \uba40\ub9ac\ub358\uc9c0\ub294 \ubc29\uc2dd\uc758 \uce90\uce58\ubcfc\uc744 \uacc4\uc18d\ud588\ub2e4. \ucc98\uc74c \ud55c\ub3d9\uc548\uc5d4 \ucee8\ud2b8\ub864\uc774 \uc7a1\ud788\uc9c0 \uc54a\uc558\uc9c0\ub9cc \uc810\uc810 \uc694\ub839\uc774 \uc7a1\ud600\uc11c \uc774\uac83\uc774 \uc774\ub9c8\ub098\uce74\uc758 \uacb0\uc815\uad6c\ub77c\uace0\ub3c4 \ud560 \uc218 \uc788\ub294 \uc2ac\ub85c \ucee4\ube0c\ub97c \uc2b5\ub4dd\ud55c \uacc4\uae30\uac00 \ub410\ub2e4\uace0 \ud55c\ub2e4. \ub610 \uc544\ub7ab\ud314\uc758 \uadfc\uc721\uc744 \ub2e8\ub828\ud558\ub294 \uac00\uc6b4\ub370 \uc67c\ucabd \ud314\uafc8\uce58\uc758 \uc720\ub9ac\uc5f0\uace8\uc774 \uc6b0\uc5f0\ud788 \ub4e4\ub7ec\ubd99\uc5c8\ub2e4\uace0\ub3c4 \ub9d0\ud588\ub2e4. \uadf8 \ud574\uc5d0\ub294 11\uacbd\uae30\uc5d0 \ub4f1\ud310\uc5d0\ub9cc \uadf8\ucce4\uc9c0\ub9cc 8\uc2b9 2\ud328\ub85c \ud3c9\uade0 \uc790\ucc45\uc810 1.77\uc758 \uc131\uc801\uc744 \ub0a8\uacbc\ub2e4.", "answer": "\ucee4\ube0c", "sentence": "\uadf8\ub7ec\ub098 \uc660\uc9c0 \ucee4\ube0c \ub9cc\uc740 \ud1b5\uc99d \uc5c6\uc774 \ub358\uc9c8 \uc218 \uc788\uc5c8\uae30 \ub54c\ubb38\uc5d0 \uc7ac\ud65c\ud6c8\ub828\uc5d0\uc11c\ub294 \ucee4\ube0c\ub9cc\uc744 \uc0ac\uc6a9\ud55c \uba40\ub9ac\ub358\uc9c0\ub294 \ubc29\uc2dd\uc758 \uce90\uce58\ubcfc\uc744 \uacc4\uc18d\ud588\ub2e4.", "paragraph_sentence": "1992\ub144 4\uc6d4 19\uc77c\uc5d0 \uc5f4\ub9b0 \uc694\ubbf8\uc6b0\ub9ac\uc804\uc5d0\uc11c \ub545\ubcfc \ud0c0\uad6c\uac00 \uc67c\ucabd \uc190\ubaa9\uc5d0 \ub9de\uc544\uc11c \uace8\uc808\ub418\ub294 \ubd80\uc0c1\uc744 \ub2f9\ud574 3\uac1c\uc6d4 \ub4a4\uc778 \uc5ec\ub984\uc5d0 \ubcf5\uadc0\ud588\ub2e4. \uace8\uc808\uc774 \ub098\uc740 \uc9c1\ud6c4\uc5d0\ub294 \uc545\ub825\uc774 \uc880\ucc98\ub7fc \ub3cc\uc544\uc624\uc9c0 \uc54a\uc544\uc11c \uac00\ubccd\uac8c \uce90\uce58\ubcfc\uc744 \ud574\ub3c4 \uc190\ubaa9\uc5d0 \ub0a0\uce74\ub85c\uc6b4 \ud1b5\uc99d\uc774 \uc788\uc5c8\ub2e4. \uadf8\ub7ec\ub098 \uc660\uc9c0 \ucee4\ube0c \ub9cc\uc740 \ud1b5\uc99d \uc5c6\uc774 \ub358\uc9c8 \uc218 \uc788\uc5c8\uae30 \ub54c\ubb38\uc5d0 \uc7ac\ud65c\ud6c8\ub828\uc5d0\uc11c\ub294 \ucee4\ube0c\ub9cc\uc744 \uc0ac\uc6a9\ud55c \uba40\ub9ac\ub358\uc9c0\ub294 \ubc29\uc2dd\uc758 \uce90\uce58\ubcfc\uc744 \uacc4\uc18d\ud588\ub2e4. \ucc98\uc74c \ud55c\ub3d9\uc548\uc5d4 \ucee8\ud2b8\ub864\uc774 \uc7a1\ud788\uc9c0 \uc54a\uc558\uc9c0\ub9cc \uc810\uc810 \uc694\ub839\uc774 \uc7a1\ud600\uc11c \uc774\uac83\uc774 \uc774\ub9c8\ub098\uce74\uc758 \uacb0\uc815\uad6c\ub77c\uace0\ub3c4 \ud560 \uc218 \uc788\ub294 \uc2ac\ub85c \ucee4\ube0c\ub97c \uc2b5\ub4dd\ud55c \uacc4\uae30\uac00 \ub410\ub2e4\uace0 \ud55c\ub2e4. \ub610 \uc544\ub7ab\ud314\uc758 \uadfc\uc721\uc744 \ub2e8\ub828\ud558\ub294 \uac00\uc6b4\ub370 \uc67c\ucabd \ud314\uafc8\uce58\uc758 \uc720\ub9ac\uc5f0\uace8\uc774 \uc6b0\uc5f0\ud788 \ub4e4\ub7ec\ubd99\uc5c8\ub2e4\uace0\ub3c4 \ub9d0\ud588\ub2e4. \uadf8 \ud574\uc5d0\ub294 11\uacbd\uae30\uc5d0 \ub4f1\ud310\uc5d0\ub9cc \uadf8\ucce4\uc9c0\ub9cc 8\uc2b9 2\ud328\ub85c \ud3c9\uade0 \uc790\ucc45\uc810 1.77\uc758 \uc131\uc801\uc744 \ub0a8\uacbc\ub2e4.", "paragraph_answer": "1992\ub144 4\uc6d4 19\uc77c\uc5d0 \uc5f4\ub9b0 \uc694\ubbf8\uc6b0\ub9ac\uc804\uc5d0\uc11c \ub545\ubcfc \ud0c0\uad6c\uac00 \uc67c\ucabd \uc190\ubaa9\uc5d0 \ub9de\uc544\uc11c \uace8\uc808\ub418\ub294 \ubd80\uc0c1\uc744 \ub2f9\ud574 3\uac1c\uc6d4 \ub4a4\uc778 \uc5ec\ub984\uc5d0 \ubcf5\uadc0\ud588\ub2e4. \uace8\uc808\uc774 \ub098\uc740 \uc9c1\ud6c4\uc5d0\ub294 \uc545\ub825\uc774 \uc880\ucc98\ub7fc \ub3cc\uc544\uc624\uc9c0 \uc54a\uc544\uc11c \uac00\ubccd\uac8c \uce90\uce58\ubcfc\uc744 \ud574\ub3c4 \uc190\ubaa9\uc5d0 \ub0a0\uce74\ub85c\uc6b4 \ud1b5\uc99d\uc774 \uc788\uc5c8\ub2e4. \uadf8\ub7ec\ub098 \uc660\uc9c0 \ucee4\ube0c \ub9cc\uc740 \ud1b5\uc99d \uc5c6\uc774 \ub358\uc9c8 \uc218 \uc788\uc5c8\uae30 \ub54c\ubb38\uc5d0 \uc7ac\ud65c\ud6c8\ub828\uc5d0\uc11c\ub294 \ucee4\ube0c\ub9cc\uc744 \uc0ac\uc6a9\ud55c \uba40\ub9ac\ub358\uc9c0\ub294 \ubc29\uc2dd\uc758 \uce90\uce58\ubcfc\uc744 \uacc4\uc18d\ud588\ub2e4. \ucc98\uc74c \ud55c\ub3d9\uc548\uc5d4 \ucee8\ud2b8\ub864\uc774 \uc7a1\ud788\uc9c0 \uc54a\uc558\uc9c0\ub9cc \uc810\uc810 \uc694\ub839\uc774 \uc7a1\ud600\uc11c \uc774\uac83\uc774 \uc774\ub9c8\ub098\uce74\uc758 \uacb0\uc815\uad6c\ub77c\uace0\ub3c4 \ud560 \uc218 \uc788\ub294 \uc2ac\ub85c \ucee4\ube0c\ub97c \uc2b5\ub4dd\ud55c \uacc4\uae30\uac00 \ub410\ub2e4\uace0 \ud55c\ub2e4. \ub610 \uc544\ub7ab\ud314\uc758 \uadfc\uc721\uc744 \ub2e8\ub828\ud558\ub294 \uac00\uc6b4\ub370 \uc67c\ucabd \ud314\uafc8\uce58\uc758 \uc720\ub9ac\uc5f0\uace8\uc774 \uc6b0\uc5f0\ud788 \ub4e4\ub7ec\ubd99\uc5c8\ub2e4\uace0\ub3c4 \ub9d0\ud588\ub2e4. \uadf8 \ud574\uc5d0\ub294 11\uacbd\uae30\uc5d0 \ub4f1\ud310\uc5d0\ub9cc \uadf8\ucce4\uc9c0\ub9cc 8\uc2b9 2\ud328\ub85c \ud3c9\uade0 \uc790\ucc45\uc810 1.77\uc758 \uc131\uc801\uc744 \ub0a8\uacbc\ub2e4.", "sentence_answer": "\uadf8\ub7ec\ub098 \uc660\uc9c0 \ucee4\ube0c \ub9cc\uc740 \ud1b5\uc99d \uc5c6\uc774 \ub358\uc9c8 \uc218 \uc788\uc5c8\uae30 \ub54c\ubb38\uc5d0 \uc7ac\ud65c\ud6c8\ub828\uc5d0\uc11c\ub294 \ucee4\ube0c\ub9cc\uc744 \uc0ac\uc6a9\ud55c \uba40\ub9ac\ub358\uc9c0\ub294 \ubc29\uc2dd\uc758 \uce90\uce58\ubcfc\uc744 \uacc4\uc18d\ud588\ub2e4.", "paragraph_id": "6545003-5-1", "paragraph_question": "question: \uc67c\ucabd \uc190\ubaa9 \uace8\uc808 \uc774\ud6c4 \uc7ac\ud65c\ud6c8\ub828\uc5d0\uc11c \uc5b4\ub5a4 \ud22c\uad6c\ub97c \uc774\uc6a9\ud574 \ub358\uc84c\ub294\uac00?, context: 1992\ub144 4\uc6d4 19\uc77c\uc5d0 \uc5f4\ub9b0 \uc694\ubbf8\uc6b0\ub9ac\uc804\uc5d0\uc11c \ub545\ubcfc \ud0c0\uad6c\uac00 \uc67c\ucabd \uc190\ubaa9\uc5d0 \ub9de\uc544\uc11c \uace8\uc808\ub418\ub294 \ubd80\uc0c1\uc744 \ub2f9\ud574 3\uac1c\uc6d4 \ub4a4\uc778 \uc5ec\ub984\uc5d0 \ubcf5\uadc0\ud588\ub2e4. \uace8\uc808\uc774 \ub098\uc740 \uc9c1\ud6c4\uc5d0\ub294 \uc545\ub825\uc774 \uc880\ucc98\ub7fc \ub3cc\uc544\uc624\uc9c0 \uc54a\uc544\uc11c \uac00\ubccd\uac8c \uce90\uce58\ubcfc\uc744 \ud574\ub3c4 \uc190\ubaa9\uc5d0 \ub0a0\uce74\ub85c\uc6b4 \ud1b5\uc99d\uc774 \uc788\uc5c8\ub2e4. \uadf8\ub7ec\ub098 \uc660\uc9c0 \ucee4\ube0c\ub9cc\uc740 \ud1b5\uc99d \uc5c6\uc774 \ub358\uc9c8 \uc218 \uc788\uc5c8\uae30 \ub54c\ubb38\uc5d0 \uc7ac\ud65c\ud6c8\ub828\uc5d0\uc11c\ub294 \ucee4\ube0c\ub9cc\uc744 \uc0ac\uc6a9\ud55c \uba40\ub9ac\ub358\uc9c0\ub294 \ubc29\uc2dd\uc758 \uce90\uce58\ubcfc\uc744 \uacc4\uc18d\ud588\ub2e4. \ucc98\uc74c \ud55c\ub3d9\uc548\uc5d4 \ucee8\ud2b8\ub864\uc774 \uc7a1\ud788\uc9c0 \uc54a\uc558\uc9c0\ub9cc \uc810\uc810 \uc694\ub839\uc774 \uc7a1\ud600\uc11c \uc774\uac83\uc774 \uc774\ub9c8\ub098\uce74\uc758 \uacb0\uc815\uad6c\ub77c\uace0\ub3c4 \ud560 \uc218 \uc788\ub294 \uc2ac\ub85c \ucee4\ube0c\ub97c \uc2b5\ub4dd\ud55c \uacc4\uae30\uac00 \ub410\ub2e4\uace0 \ud55c\ub2e4. \ub610 \uc544\ub7ab\ud314\uc758 \uadfc\uc721\uc744 \ub2e8\ub828\ud558\ub294 \uac00\uc6b4\ub370 \uc67c\ucabd \ud314\uafc8\uce58\uc758 \uc720\ub9ac\uc5f0\uace8\uc774 \uc6b0\uc5f0\ud788 \ub4e4\ub7ec\ubd99\uc5c8\ub2e4\uace0\ub3c4 \ub9d0\ud588\ub2e4. \uadf8 \ud574\uc5d0\ub294 11\uacbd\uae30\uc5d0 \ub4f1\ud310\uc5d0\ub9cc \uadf8\ucce4\uc9c0\ub9cc 8\uc2b9 2\ud328\ub85c \ud3c9\uade0 \uc790\ucc45\uc810 1.77\uc758 \uc131\uc801\uc744 \ub0a8\uacbc\ub2e4."} {"question": "6\uc6d4 \ud56d\uc7c1 \uc911 \uaddc\ud0c4\ub300\ud68c\ub97c \uac16\uae30\ub85c \ud55c \ub0a0\uc740?", "paragraph": "1987\ub144 5\uc6d4 17\uc77c, \ub178\ub3d9\uc790\uc600\ub358 \ud669\ubcf4\uc601\uad6d\uc740 \ubd80\uc0b0\uc0c1\uace0(\ud604 \uac1c\uc131\uace0) \uc55e\uc5d0\uc11c '\ub3c5\uc7ac\ud0c0\ub3c4' \ub4f1\uc744 \uc678\uce58\uba70 \ubd84\uc2e0\ud588\uc73c\uba70, \uc77c\uc8fc\uc77c \ub4a4 \uc0ac\ub9dd\ud558\uc600\ub2e4. \uc774\ud2bf\ub0a0\uc778 5\uc6d4 18\uc77c \uba85\ub3d9\uc131\ub2f9\uc5d0\uc11c \uad11\uc8fc\ud56d\uc7c1 7\uc8fc\ub144 \ubbf8\uc0ac\uc5d0 \uc815\uc758\uad6c\ud604\uc0ac\uc81c\ub2e8 \uae40\uc2b9\ud6c8 \uc2e0\ubd80\uac00 \ubc15\uc885\ucca0 \uace0\ubb38 \uce58\uc0ac \uc0ac\uac74\uc774 \uacbd\ucc30\uc5d0 \uc758\ud574 \ucd95\uc18c\u00b7\uc740\ud3d0\ub418\uc5c8\uc74c\uc744 \ud3ed\ub85c\ud558\uc600\ub2e4. \uc774\uc5d0 \uc81c5\uacf5\ud654\uad6d \uc815\uad8c\uc744 \ube44\ud310\ud558\ub358 \uad6d\ubbfc\ub4e4\uc740 \uc804\ub450\ud658 \uad70\uc0ac\ub3c5\uc7ac\uc815\uad8c\uc758 \uc633\uc9c0 \ubabb\ud568\uc5d0 \ud06c\uac8c \ubd84\ub178\ud558\uc600\uace0, \uc774\ud6c4 \ubbfc\uc8fc\ud654\ub97c \uc694\uad6c\ud558\ub294 \uc2dc\uc704\uac00 \uc804\uad6d\uc5d0\uc11c \uc790\uc8fc \uc77c\uc5b4\ub0ac\ub2e4. \uc774\ud6c4 5\uc6d4 23\uc77c \"\ubc15\uc885\ucca0 \uace0\ubb38\uc0b4\uc778\uc740\ud3d0\uc870\uc791\uaddc\ud0c4 \ubc94\uad6d\ubbfc\ub300\ud68c \uc900\ube44\uc704\uc6d0\ud68c\"\uac00 \uacb0\uc131\ub418\uc5c8\uace0, \uc774\ub4e4\uc740 6\uc6d4 10\uc77c\uc5d0 \uaddc\ud0c4\ub300\ud68c\ub97c \uac16\uae30\ub85c \uacb0\uc815\ud558\uc600\ub2e4(\uadf8\ub0a0\uc740 \ub178\ud0dc\uc6b0\uac00 \ubbfc\uc815\ub2f9 \ub300\uc120 \ud6c4\ubcf4\ub85c \uc120\ucd9c\ub41c \ub0a0\uc774\uae30\ub3c4 \ud558\ub2e4).", "answer": "6\uc6d4 10\uc77c", "sentence": "\uc774\ub4e4\uc740 6\uc6d4 10\uc77c \uc5d0 \uaddc\ud0c4\ub300\ud68c\ub97c \uac16\uae30\ub85c \uacb0\uc815\ud558\uc600\ub2e4(\uadf8\ub0a0\uc740 \ub178\ud0dc\uc6b0\uac00 \ubbfc\uc815\ub2f9 \ub300\uc120 \ud6c4\ubcf4\ub85c \uc120\ucd9c\ub41c \ub0a0\uc774\uae30\ub3c4 \ud558\ub2e4).", "paragraph_sentence": "1987\ub144 5\uc6d4 17\uc77c, \ub178\ub3d9\uc790\uc600\ub358 \ud669\ubcf4\uc601\uad6d\uc740 \ubd80\uc0b0\uc0c1\uace0(\ud604 \uac1c\uc131\uace0) \uc55e\uc5d0\uc11c '\ub3c5\uc7ac\ud0c0\ub3c4' \ub4f1\uc744 \uc678\uce58\uba70 \ubd84\uc2e0\ud588\uc73c\uba70, \uc77c\uc8fc\uc77c \ub4a4 \uc0ac\ub9dd\ud558\uc600\ub2e4. \uc774\ud2bf\ub0a0\uc778 5\uc6d4 18\uc77c \uba85\ub3d9\uc131\ub2f9\uc5d0\uc11c \uad11\uc8fc\ud56d\uc7c1 7\uc8fc\ub144 \ubbf8\uc0ac\uc5d0 \uc815\uc758\uad6c\ud604\uc0ac\uc81c\ub2e8 \uae40\uc2b9\ud6c8 \uc2e0\ubd80\uac00 \ubc15\uc885\ucca0 \uace0\ubb38 \uce58\uc0ac \uc0ac\uac74\uc774 \uacbd\ucc30\uc5d0 \uc758\ud574 \ucd95\uc18c\u00b7\uc740\ud3d0\ub418\uc5c8\uc74c\uc744 \ud3ed\ub85c\ud558\uc600\ub2e4. \uc774\uc5d0 \uc81c5\uacf5\ud654\uad6d \uc815\uad8c\uc744 \ube44\ud310\ud558\ub358 \uad6d\ubbfc\ub4e4\uc740 \uc804\ub450\ud658 \uad70\uc0ac\ub3c5\uc7ac\uc815\uad8c\uc758 \uc633\uc9c0 \ubabb\ud568\uc5d0 \ud06c\uac8c \ubd84\ub178\ud558\uc600\uace0, \uc774\ud6c4 \ubbfc\uc8fc\ud654\ub97c \uc694\uad6c\ud558\ub294 \uc2dc\uc704\uac00 \uc804\uad6d\uc5d0\uc11c \uc790\uc8fc \uc77c\uc5b4\ub0ac\ub2e4. \uc774\ud6c4 5\uc6d4 23\uc77c \"\ubc15\uc885\ucca0 \uace0\ubb38\uc0b4\uc778\uc740\ud3d0\uc870\uc791\uaddc\ud0c4 \ubc94\uad6d\ubbfc\ub300\ud68c \uc900\ube44\uc704\uc6d0\ud68c\"\uac00 \uacb0\uc131\ub418\uc5c8\uace0, \uc774\ub4e4\uc740 6\uc6d4 10\uc77c \uc5d0 \uaddc\ud0c4\ub300\ud68c\ub97c \uac16\uae30\ub85c \uacb0\uc815\ud558\uc600\ub2e4(\uadf8\ub0a0\uc740 \ub178\ud0dc\uc6b0\uac00 \ubbfc\uc815\ub2f9 \ub300\uc120 \ud6c4\ubcf4\ub85c \uc120\ucd9c\ub41c \ub0a0\uc774\uae30\ub3c4 \ud558\ub2e4). ", "paragraph_answer": "1987\ub144 5\uc6d4 17\uc77c, \ub178\ub3d9\uc790\uc600\ub358 \ud669\ubcf4\uc601\uad6d\uc740 \ubd80\uc0b0\uc0c1\uace0(\ud604 \uac1c\uc131\uace0) \uc55e\uc5d0\uc11c '\ub3c5\uc7ac\ud0c0\ub3c4' \ub4f1\uc744 \uc678\uce58\uba70 \ubd84\uc2e0\ud588\uc73c\uba70, \uc77c\uc8fc\uc77c \ub4a4 \uc0ac\ub9dd\ud558\uc600\ub2e4. \uc774\ud2bf\ub0a0\uc778 5\uc6d4 18\uc77c \uba85\ub3d9\uc131\ub2f9\uc5d0\uc11c \uad11\uc8fc\ud56d\uc7c1 7\uc8fc\ub144 \ubbf8\uc0ac\uc5d0 \uc815\uc758\uad6c\ud604\uc0ac\uc81c\ub2e8 \uae40\uc2b9\ud6c8 \uc2e0\ubd80\uac00 \ubc15\uc885\ucca0 \uace0\ubb38 \uce58\uc0ac \uc0ac\uac74\uc774 \uacbd\ucc30\uc5d0 \uc758\ud574 \ucd95\uc18c\u00b7\uc740\ud3d0\ub418\uc5c8\uc74c\uc744 \ud3ed\ub85c\ud558\uc600\ub2e4. \uc774\uc5d0 \uc81c5\uacf5\ud654\uad6d \uc815\uad8c\uc744 \ube44\ud310\ud558\ub358 \uad6d\ubbfc\ub4e4\uc740 \uc804\ub450\ud658 \uad70\uc0ac\ub3c5\uc7ac\uc815\uad8c\uc758 \uc633\uc9c0 \ubabb\ud568\uc5d0 \ud06c\uac8c \ubd84\ub178\ud558\uc600\uace0, \uc774\ud6c4 \ubbfc\uc8fc\ud654\ub97c \uc694\uad6c\ud558\ub294 \uc2dc\uc704\uac00 \uc804\uad6d\uc5d0\uc11c \uc790\uc8fc \uc77c\uc5b4\ub0ac\ub2e4. \uc774\ud6c4 5\uc6d4 23\uc77c \"\ubc15\uc885\ucca0 \uace0\ubb38\uc0b4\uc778\uc740\ud3d0\uc870\uc791\uaddc\ud0c4 \ubc94\uad6d\ubbfc\ub300\ud68c \uc900\ube44\uc704\uc6d0\ud68c\"\uac00 \uacb0\uc131\ub418\uc5c8\uace0, \uc774\ub4e4\uc740 6\uc6d4 10\uc77c \uc5d0 \uaddc\ud0c4\ub300\ud68c\ub97c \uac16\uae30\ub85c \uacb0\uc815\ud558\uc600\ub2e4(\uadf8\ub0a0\uc740 \ub178\ud0dc\uc6b0\uac00 \ubbfc\uc815\ub2f9 \ub300\uc120 \ud6c4\ubcf4\ub85c \uc120\ucd9c\ub41c \ub0a0\uc774\uae30\ub3c4 \ud558\ub2e4).", "sentence_answer": "\uc774\ub4e4\uc740 6\uc6d4 10\uc77c \uc5d0 \uaddc\ud0c4\ub300\ud68c\ub97c \uac16\uae30\ub85c \uacb0\uc815\ud558\uc600\ub2e4(\uadf8\ub0a0\uc740 \ub178\ud0dc\uc6b0\uac00 \ubbfc\uc815\ub2f9 \ub300\uc120 \ud6c4\ubcf4\ub85c \uc120\ucd9c\ub41c \ub0a0\uc774\uae30\ub3c4 \ud558\ub2e4).", "paragraph_id": "6499551-0-0", "paragraph_question": "question: 6\uc6d4 \ud56d\uc7c1 \uc911 \uaddc\ud0c4\ub300\ud68c\ub97c \uac16\uae30\ub85c \ud55c \ub0a0\uc740?, context: 1987\ub144 5\uc6d4 17\uc77c, \ub178\ub3d9\uc790\uc600\ub358 \ud669\ubcf4\uc601\uad6d\uc740 \ubd80\uc0b0\uc0c1\uace0(\ud604 \uac1c\uc131\uace0) \uc55e\uc5d0\uc11c '\ub3c5\uc7ac\ud0c0\ub3c4' \ub4f1\uc744 \uc678\uce58\uba70 \ubd84\uc2e0\ud588\uc73c\uba70, \uc77c\uc8fc\uc77c \ub4a4 \uc0ac\ub9dd\ud558\uc600\ub2e4. \uc774\ud2bf\ub0a0\uc778 5\uc6d4 18\uc77c \uba85\ub3d9\uc131\ub2f9\uc5d0\uc11c \uad11\uc8fc\ud56d\uc7c1 7\uc8fc\ub144 \ubbf8\uc0ac\uc5d0 \uc815\uc758\uad6c\ud604\uc0ac\uc81c\ub2e8 \uae40\uc2b9\ud6c8 \uc2e0\ubd80\uac00 \ubc15\uc885\ucca0 \uace0\ubb38 \uce58\uc0ac \uc0ac\uac74\uc774 \uacbd\ucc30\uc5d0 \uc758\ud574 \ucd95\uc18c\u00b7\uc740\ud3d0\ub418\uc5c8\uc74c\uc744 \ud3ed\ub85c\ud558\uc600\ub2e4. \uc774\uc5d0 \uc81c5\uacf5\ud654\uad6d \uc815\uad8c\uc744 \ube44\ud310\ud558\ub358 \uad6d\ubbfc\ub4e4\uc740 \uc804\ub450\ud658 \uad70\uc0ac\ub3c5\uc7ac\uc815\uad8c\uc758 \uc633\uc9c0 \ubabb\ud568\uc5d0 \ud06c\uac8c \ubd84\ub178\ud558\uc600\uace0, \uc774\ud6c4 \ubbfc\uc8fc\ud654\ub97c \uc694\uad6c\ud558\ub294 \uc2dc\uc704\uac00 \uc804\uad6d\uc5d0\uc11c \uc790\uc8fc \uc77c\uc5b4\ub0ac\ub2e4. \uc774\ud6c4 5\uc6d4 23\uc77c \"\ubc15\uc885\ucca0 \uace0\ubb38\uc0b4\uc778\uc740\ud3d0\uc870\uc791\uaddc\ud0c4 \ubc94\uad6d\ubbfc\ub300\ud68c \uc900\ube44\uc704\uc6d0\ud68c\"\uac00 \uacb0\uc131\ub418\uc5c8\uace0, \uc774\ub4e4\uc740 6\uc6d4 10\uc77c\uc5d0 \uaddc\ud0c4\ub300\ud68c\ub97c \uac16\uae30\ub85c \uacb0\uc815\ud558\uc600\ub2e4(\uadf8\ub0a0\uc740 \ub178\ud0dc\uc6b0\uac00 \ubbfc\uc815\ub2f9 \ub300\uc120 \ud6c4\ubcf4\ub85c \uc120\ucd9c\ub41c \ub0a0\uc774\uae30\ub3c4 \ud558\ub2e4)."} {"question": "\uac74\uc870\ub41c \uc811\ucc29\uc81c\ub294 \ubb34\uc5c7\uc744 \uc0dd\uc131\ud558\ub294\uac00?", "paragraph": "\uac10\uc555\uc2dd \uc811\ucc29\uc81c\ub294 \uc561\uccb4 \uce90\ub9ac\uc5b4 \ud615\ud0dc \ud639\uc740 100% \uace0\uccb4 \ud615\ud0dc\ub85c \uc81c\uc791\ub41c\ub2e4. \ud14c\uc774\ud504\ub098 \ub77c\ubca8\uacfc \uac19\uc740 \ubb3c\ud488\ub4e4\uc740 \uc811\ucc29\uc81c\ub97c \uc9c0\uc9c0\ub300 \ubd80\ubd84\uc5d0 \ucf54\ud305\ud558\uace0 \uc720\uae30 \uc6a9\ub9e4\ub098 \uc6cc\ud130 \uce90\ub9ac\uc5b4\ub97c \ub728\uac70\uc6b4 \uacf5\uae30\ub97c \uc774\uc6a9\ud558\ub294 \uac74\uc870\uae30\ub97c \uc774\uc6a9\ud558\uc5ec \uc99d\ubc1c \uc2dc\ud0a4\ub294 \ubc29\uc2dd\uc73c\ub85c \uc561\uccb4 \uac10\uc555 \uc811\ucc29\uc81c\ub97c \uc774\uc6a9\ud558\uc5ec \ub9cc\ub4e4\uc5b4\uc9c4\ub2e4. \uac74\uc870\ub41c \uc811\ucc29\uc81c\ub294 \uad50\ucc28 \uacb0\ud569\uc744 \uc0dd\uc131\ud558\uace0 \ubd84\uc790 \uc911\ub7c9\uc744 \uc99d\uac00\uc2dc\ud0a4\uae30 \uc704\ud574 \ub354 \uac00\uc5f4\ub41c\ub2e4. 100% \uace0\uccb4 \uac10\uc555 \uc811\ucc29\uc81c\uc5d0\ub294 \ucf54\ud305\uc774 \ub418\uba74\uc11c \ubcf5\uc0ac\ub97c \ubc1b\uc544 \ubd84\uc790 \uc9c8\ub7c9\uc774 \uc99d\uac00\ud558\uace0 \uc811\ucc29\uc81c\uac00 \uc0dd\uc0b0\ub418\ub294 \ub0ae\uc740 \uc810\ub3c4\uc758 \ud3f4\ub9ac\uba38\ub85c \ub41c \uac83\ub4e4\ub3c4 \uc788\ub2e4. \ub610\ub294, \ub192\uc740 \uc810\uc131\uc744 \uac16\uace0 \uac00\uc5f4\ub418\uba74\uc11c \ucf54\ud305\uc774 \ub420 \uc218 \uc788\uc744\ub9cc\ud07c \uc810\uc131\uc774 \ub0ae\uc544\uc9c0\uace0 \uc2dd\uc73c\uba74\uc11c \ucd5c\uc885 \ud615\ud0dc\ub85c \uace0\uc815\ub418\ub294 \uac83\ub4e4\ub3c4 \uc788\ub2e4(\ud56b \uba5c\ud2b8 \ud615 \uc811\ucc29\uc81c, HMPSA).", "answer": "\uad50\ucc28 \uacb0\ud569", "sentence": "\uac74\uc870\ub41c \uc811\ucc29\uc81c\ub294 \uad50\ucc28 \uacb0\ud569 \uc744 \uc0dd\uc131\ud558\uace0 \ubd84\uc790 \uc911\ub7c9\uc744 \uc99d\uac00\uc2dc\ud0a4\uae30 \uc704\ud574 \ub354 \uac00\uc5f4\ub41c\ub2e4.", "paragraph_sentence": "\uac10\uc555\uc2dd \uc811\ucc29\uc81c\ub294 \uc561\uccb4 \uce90\ub9ac\uc5b4 \ud615\ud0dc \ud639\uc740 100% \uace0\uccb4 \ud615\ud0dc\ub85c \uc81c\uc791\ub41c\ub2e4. \ud14c\uc774\ud504\ub098 \ub77c\ubca8\uacfc \uac19\uc740 \ubb3c\ud488\ub4e4\uc740 \uc811\ucc29\uc81c\ub97c \uc9c0\uc9c0\ub300 \ubd80\ubd84\uc5d0 \ucf54\ud305\ud558\uace0 \uc720\uae30 \uc6a9\ub9e4\ub098 \uc6cc\ud130 \uce90\ub9ac\uc5b4\ub97c \ub728\uac70\uc6b4 \uacf5\uae30\ub97c \uc774\uc6a9\ud558\ub294 \uac74\uc870\uae30\ub97c \uc774\uc6a9\ud558\uc5ec \uc99d\ubc1c \uc2dc\ud0a4\ub294 \ubc29\uc2dd\uc73c\ub85c \uc561\uccb4 \uac10\uc555 \uc811\ucc29\uc81c\ub97c \uc774\uc6a9\ud558\uc5ec \ub9cc\ub4e4\uc5b4\uc9c4\ub2e4. \uac74\uc870\ub41c \uc811\ucc29\uc81c\ub294 \uad50\ucc28 \uacb0\ud569 \uc744 \uc0dd\uc131\ud558\uace0 \ubd84\uc790 \uc911\ub7c9\uc744 \uc99d\uac00\uc2dc\ud0a4\uae30 \uc704\ud574 \ub354 \uac00\uc5f4\ub41c\ub2e4. 100% \uace0\uccb4 \uac10\uc555 \uc811\ucc29\uc81c\uc5d0\ub294 \ucf54\ud305\uc774 \ub418\uba74\uc11c \ubcf5\uc0ac\ub97c \ubc1b\uc544 \ubd84\uc790 \uc9c8\ub7c9\uc774 \uc99d\uac00\ud558\uace0 \uc811\ucc29\uc81c\uac00 \uc0dd\uc0b0\ub418\ub294 \ub0ae\uc740 \uc810\ub3c4\uc758 \ud3f4\ub9ac\uba38\ub85c \ub41c \uac83\ub4e4\ub3c4 \uc788\ub2e4. \ub610\ub294, \ub192\uc740 \uc810\uc131\uc744 \uac16\uace0 \uac00\uc5f4\ub418\uba74\uc11c \ucf54\ud305\uc774 \ub420 \uc218 \uc788\uc744\ub9cc\ud07c \uc810\uc131\uc774 \ub0ae\uc544\uc9c0\uace0 \uc2dd\uc73c\uba74\uc11c \ucd5c\uc885 \ud615\ud0dc\ub85c \uace0\uc815\ub418\ub294 \uac83\ub4e4\ub3c4 \uc788\ub2e4(\ud56b \uba5c\ud2b8 \ud615 \uc811\ucc29\uc81c, HMPSA).", "paragraph_answer": "\uac10\uc555\uc2dd \uc811\ucc29\uc81c\ub294 \uc561\uccb4 \uce90\ub9ac\uc5b4 \ud615\ud0dc \ud639\uc740 100% \uace0\uccb4 \ud615\ud0dc\ub85c \uc81c\uc791\ub41c\ub2e4. \ud14c\uc774\ud504\ub098 \ub77c\ubca8\uacfc \uac19\uc740 \ubb3c\ud488\ub4e4\uc740 \uc811\ucc29\uc81c\ub97c \uc9c0\uc9c0\ub300 \ubd80\ubd84\uc5d0 \ucf54\ud305\ud558\uace0 \uc720\uae30 \uc6a9\ub9e4\ub098 \uc6cc\ud130 \uce90\ub9ac\uc5b4\ub97c \ub728\uac70\uc6b4 \uacf5\uae30\ub97c \uc774\uc6a9\ud558\ub294 \uac74\uc870\uae30\ub97c \uc774\uc6a9\ud558\uc5ec \uc99d\ubc1c \uc2dc\ud0a4\ub294 \ubc29\uc2dd\uc73c\ub85c \uc561\uccb4 \uac10\uc555 \uc811\ucc29\uc81c\ub97c \uc774\uc6a9\ud558\uc5ec \ub9cc\ub4e4\uc5b4\uc9c4\ub2e4. \uac74\uc870\ub41c \uc811\ucc29\uc81c\ub294 \uad50\ucc28 \uacb0\ud569 \uc744 \uc0dd\uc131\ud558\uace0 \ubd84\uc790 \uc911\ub7c9\uc744 \uc99d\uac00\uc2dc\ud0a4\uae30 \uc704\ud574 \ub354 \uac00\uc5f4\ub41c\ub2e4. 100% \uace0\uccb4 \uac10\uc555 \uc811\ucc29\uc81c\uc5d0\ub294 \ucf54\ud305\uc774 \ub418\uba74\uc11c \ubcf5\uc0ac\ub97c \ubc1b\uc544 \ubd84\uc790 \uc9c8\ub7c9\uc774 \uc99d\uac00\ud558\uace0 \uc811\ucc29\uc81c\uac00 \uc0dd\uc0b0\ub418\ub294 \ub0ae\uc740 \uc810\ub3c4\uc758 \ud3f4\ub9ac\uba38\ub85c \ub41c \uac83\ub4e4\ub3c4 \uc788\ub2e4. \ub610\ub294, \ub192\uc740 \uc810\uc131\uc744 \uac16\uace0 \uac00\uc5f4\ub418\uba74\uc11c \ucf54\ud305\uc774 \ub420 \uc218 \uc788\uc744\ub9cc\ud07c \uc810\uc131\uc774 \ub0ae\uc544\uc9c0\uace0 \uc2dd\uc73c\uba74\uc11c \ucd5c\uc885 \ud615\ud0dc\ub85c \uace0\uc815\ub418\ub294 \uac83\ub4e4\ub3c4 \uc788\ub2e4(\ud56b \uba5c\ud2b8 \ud615 \uc811\ucc29\uc81c, HMPSA).", "sentence_answer": "\uac74\uc870\ub41c \uc811\ucc29\uc81c\ub294 \uad50\ucc28 \uacb0\ud569 \uc744 \uc0dd\uc131\ud558\uace0 \ubd84\uc790 \uc911\ub7c9\uc744 \uc99d\uac00\uc2dc\ud0a4\uae30 \uc704\ud574 \ub354 \uac00\uc5f4\ub41c\ub2e4.", "paragraph_id": "6469315-2-0", "paragraph_question": "question: \uac74\uc870\ub41c \uc811\ucc29\uc81c\ub294 \ubb34\uc5c7\uc744 \uc0dd\uc131\ud558\ub294\uac00?, context: \uac10\uc555\uc2dd \uc811\ucc29\uc81c\ub294 \uc561\uccb4 \uce90\ub9ac\uc5b4 \ud615\ud0dc \ud639\uc740 100% \uace0\uccb4 \ud615\ud0dc\ub85c \uc81c\uc791\ub41c\ub2e4. \ud14c\uc774\ud504\ub098 \ub77c\ubca8\uacfc \uac19\uc740 \ubb3c\ud488\ub4e4\uc740 \uc811\ucc29\uc81c\ub97c \uc9c0\uc9c0\ub300 \ubd80\ubd84\uc5d0 \ucf54\ud305\ud558\uace0 \uc720\uae30 \uc6a9\ub9e4\ub098 \uc6cc\ud130 \uce90\ub9ac\uc5b4\ub97c \ub728\uac70\uc6b4 \uacf5\uae30\ub97c \uc774\uc6a9\ud558\ub294 \uac74\uc870\uae30\ub97c \uc774\uc6a9\ud558\uc5ec \uc99d\ubc1c \uc2dc\ud0a4\ub294 \ubc29\uc2dd\uc73c\ub85c \uc561\uccb4 \uac10\uc555 \uc811\ucc29\uc81c\ub97c \uc774\uc6a9\ud558\uc5ec \ub9cc\ub4e4\uc5b4\uc9c4\ub2e4. \uac74\uc870\ub41c \uc811\ucc29\uc81c\ub294 \uad50\ucc28 \uacb0\ud569\uc744 \uc0dd\uc131\ud558\uace0 \ubd84\uc790 \uc911\ub7c9\uc744 \uc99d\uac00\uc2dc\ud0a4\uae30 \uc704\ud574 \ub354 \uac00\uc5f4\ub41c\ub2e4. 100% \uace0\uccb4 \uac10\uc555 \uc811\ucc29\uc81c\uc5d0\ub294 \ucf54\ud305\uc774 \ub418\uba74\uc11c \ubcf5\uc0ac\ub97c \ubc1b\uc544 \ubd84\uc790 \uc9c8\ub7c9\uc774 \uc99d\uac00\ud558\uace0 \uc811\ucc29\uc81c\uac00 \uc0dd\uc0b0\ub418\ub294 \ub0ae\uc740 \uc810\ub3c4\uc758 \ud3f4\ub9ac\uba38\ub85c \ub41c \uac83\ub4e4\ub3c4 \uc788\ub2e4. \ub610\ub294, \ub192\uc740 \uc810\uc131\uc744 \uac16\uace0 \uac00\uc5f4\ub418\uba74\uc11c \ucf54\ud305\uc774 \ub420 \uc218 \uc788\uc744\ub9cc\ud07c \uc810\uc131\uc774 \ub0ae\uc544\uc9c0\uace0 \uc2dd\uc73c\uba74\uc11c \ucd5c\uc885 \ud615\ud0dc\ub85c \uace0\uc815\ub418\ub294 \uac83\ub4e4\ub3c4 \uc788\ub2e4(\ud56b \uba5c\ud2b8 \ud615 \uc811\ucc29\uc81c, HMPSA)."} {"question": "\ud64d\uac00\uad8c\uc740 \uc5ec\ub7ec \uc11c\ub300\uc5d0 \uac78\uccd0 \ub0b4\ub824\uc624\uba74\uc11c, \ub204\uad6c\uc5d0\uac8c \uc804\uc218 \ub418\ub098\uc694?", "paragraph": "\uc774\ud6c4 \ud64d\uac00\uad8c\uc740 \uc5ec\ub7ec \ub300\uc5d0 \uac70\uccd0 \ub0b4\ub824\uc624\uba74\uc11c \ub9c8\uce68\ub0b4 \ud669\ube44\ud64d(\u9ec3\u98db\u9d3b)\uc5d0\uac8c \uc804\uc218\ub418\uae30\uc5d0 \uc774\ub978\ub2e4. \ud669\ube44\ud64d\uc740 \uadf8\uc758 \uc544\ubc84\uc9c0 \ud669\uae30\uc601(\u9ec3\u9a65\u82f1)\uc73c\ub85c\ubd80\ud130 \uac00\ubb38 \ub300\ub300\ub85c \ub0b4\ub824\uc628 \ud64d\uac00\uad8c\uc744 \uc775\ud600 15\uc138\uc758 \ub098\uc774\uc5d0 \ub3c4\uc7a5\uc744 \uc6b4\uc601\ud558\uc600\ub294\ub370, \uadf8\uc758 \uc544\ubc84\uc9c0 \ud669\uae30\uc601\uc740 \uad11\ub3d9\uc2ed\ud638(\u5ee3\u6771\u5341\u864e)\uc758 \ubc18\uc5f4\uc5d0 \uc624\ub978 \uace0\uc218\uc600\ub2e4. \ud669\ube44\ud64d\uc740 \uc804\uc218\ubc1b\uc740 \ud64d\uac00\uad8c\uc758 \uad8c\ubc95\ub4e4\uc744 \ub2e4\uc2dc \ud55c\ubc88 \uac1c\ub7c9\ud558\uc5ec \ubd88\ud544\uc694\ud55c \ub3d9\uc791\uc744 \uc5c6\uc560\uace0 \uc790\uc2e0\uc774 \ub9cc\ub4e0 \uc0c8\ub85c\uc6b4 \ud615\ud0dc\ub97c \ucd94\uac00\ud558\uc600\ub2e4. \uc774\ub54c \uacf5\uc790\ubcf5\ud638\uad8c\uacfc \ud638\ud559\uc30d\ud615\uad8c\uc774 \ud06c\uac8c \uac1c\ub7c9\ub418\uc5c8\ub294\ub370 \ud5db\uc810\uc774 \ub9ce\uc740 \ub3d9\uc791\ub4e4\uc740 \ubaa8\ub450 \ubc30\uc81c\uc2dc\ud0a4\uace0 \ud6a8\uacfc\uc801\uc774\uace0 \uc989\uc2e4\uc131\uc774 \ub192\uc740 \ub3d9\uc791\ub4e4\ub85c \uc0c8\ub85c \uac1c\ud3b8\ub418\uc5c8\ub2e4\uace0 \ud55c\ub2e4. \ub610\ud55c \ud669\ube44\ud64d\uc740 \uc18c\uac78\uc544(\u8607\u4e5e\u5152)\ub85c\ubd80\ud130\u300a\ucde8\ud314\uc120\uad8c\u300b(\u9189\u516b\u4ed9\u62f3)\uc744, \ucca0\uad50\uc0bc(\u9435\u6a4b\u4e09)\uc758 \uc81c\uc790\uc778 \uc784\ubcf5\uc131(\u6797\u798f\u6210)\uc73c\ub85c\ubd80\ud130\ub294 \u300a\ucca0\uc120\uad8c\u300b(\u9435\u7dda\u62f3)\uacfc \u300a\ube44\ud0c0\u300b(\u98db\u7823)\ub97c, \uc1a1\ud718\ub2f9(\u5b8b\u8f1d\u93dc)\uc73c\ub85c\ubd80\ud130 \u300a\ubb34\uc601\uac01\u300b(\u7121\u5f71\u811a)\uc744 \uc804\uc218\ubc1b\uc544 \ud64d\uac00\uad8c\uc5d0 \ud3b8\uc785\uc2dc\ud0b4\uc73c\ub85c\uc368 \ub9c8\uce68\ub0b4 \uc624\ub298\ub0a0 \ud64d\uac00\uad8c\uc758 \ud615\ud0dc\ub97c \uc644\uc131\uc2dc\ucf30\ub2e4. \uc774\ub7ec\ud55c \uac1c\ub7c9\uacfc \ud3b8\uc785 \uacfc\uc815\uc5d0\uc11c \uc0c8\ub86d\uac8c \ucd94\uac00\ub41c \uad8c\ubc95\uc774 \ub9ce\uc544\uc9c0\uace0 \ud64d\uac00\uad8c\uc758 \uad50\uc2b5 \uccb4\uacc4\uac00 \ube44\ub85c\uc18c \uc644\ube44\ub418\uc5b4 \uadfc\ub300 \ud64d\uac00\uad8c\uc758 \ub9e5\uc744 \ud615\uc131\ud558\uc600\uc73c\ubbc0\ub85c \ud604\uc7ac \ud669\ube44\ud64d\uc774 \uacc4\ubcf4\ub97c \uc787\ub294 \ud64d\uac00\ubb38 \uc81c\uc790\ub4e4\uc740 \ud669\ube44\ud64d\uc744 \uc0ac\uc2e4\uc0c1 \uadfc\ub300 \ud64d\uac00\uad8c\uc758 \uc2dc\uc870\ub85c \ubc1b\ub4e4\uace0 \uc788\ub2e4. \ub610\ud55c \ud669\ube44\ud64d\uc740 \ucc44\ub9ac\ubd88\uad8c(\u8521\u674e\u4f5b\u62f3), \uc601\ucd98\uad8c(\u8a60\u6625\u62f3) \ub4f1 \ub0a8\ud30c \uc9c0\uc5ed\uc5d0\uc11c \uc131\ud589\ud558\ub294 \ubaa8\ub4e0 \uad8c\ubc95\uc744 \ub204\ub974\uace0 \ud64d\uac00\uad8c\uc744 \uc601\ub0a8 \ubb34\uc220 \uc911 \uc81c\uc77c\ubb34\uc220\uc758 \ubc18\uc5f4\uc5d0 \ub2e4\uc2dc \uc62c\ub824\ub193\uc544 \uc774\uc804\ubcf4\ub2e4 \ub354\uc6b1 \uba85\uc131\uc744 \ub192\uac8c \ud558\uc600\uc73c\ubbc0\ub85c \uc624\ub298\ub0a0\uc758 \ubb34\uc220\uac00\ub4e4\uc740 \ud669\ube44\ud64d\uc744 \ud64d\uac00\uad8c\uc758 \uc77c\ub300\uc885\uc0ac(\u4e00\u4ee3\u5b97\u5e2b)\uc774\uc790 \uc911\ud765\uc870\ub85c \ubc1b\ub4e4\uace0 \uc788\ub2e4.", "answer": "\ud669\ube44\ud64d", "sentence": "\ub9c8\uce68\ub0b4 \ud669\ube44\ud64d (\u9ec3\u98db\u9d3b)\uc5d0\uac8c \uc804\uc218\ub418\uae30\uc5d0 \uc774\ub978\ub2e4.", "paragraph_sentence": "\uc774\ud6c4 \ud64d\uac00\uad8c\uc740 \uc5ec\ub7ec \ub300\uc5d0 \uac70\uccd0 \ub0b4\ub824\uc624\uba74\uc11c \ub9c8\uce68\ub0b4 \ud669\ube44\ud64d (\u9ec3\u98db\u9d3b)\uc5d0\uac8c \uc804\uc218\ub418\uae30\uc5d0 \uc774\ub978\ub2e4. \ud669\ube44\ud64d\uc740 \uadf8\uc758 \uc544\ubc84\uc9c0 \ud669\uae30\uc601(\u9ec3\u9a65\u82f1)\uc73c\ub85c\ubd80\ud130 \uac00\ubb38 \ub300\ub300\ub85c \ub0b4\ub824\uc628 \ud64d\uac00\uad8c\uc744 \uc775\ud600 15\uc138\uc758 \ub098\uc774\uc5d0 \ub3c4\uc7a5\uc744 \uc6b4\uc601\ud558\uc600\ub294\ub370, \uadf8\uc758 \uc544\ubc84\uc9c0 \ud669\uae30\uc601\uc740 \uad11\ub3d9\uc2ed\ud638(\u5ee3\u6771\u5341\u864e)\uc758 \ubc18\uc5f4\uc5d0 \uc624\ub978 \uace0\uc218\uc600\ub2e4. \ud669\ube44\ud64d\uc740 \uc804\uc218\ubc1b\uc740 \ud64d\uac00\uad8c\uc758 \uad8c\ubc95\ub4e4\uc744 \ub2e4\uc2dc \ud55c\ubc88 \uac1c\ub7c9\ud558\uc5ec \ubd88\ud544\uc694\ud55c \ub3d9\uc791\uc744 \uc5c6\uc560\uace0 \uc790\uc2e0\uc774 \ub9cc\ub4e0 \uc0c8\ub85c\uc6b4 \ud615\ud0dc\ub97c \ucd94\uac00\ud558\uc600\ub2e4. \uc774\ub54c \uacf5\uc790\ubcf5\ud638\uad8c\uacfc \ud638\ud559\uc30d\ud615\uad8c\uc774 \ud06c\uac8c \uac1c\ub7c9\ub418\uc5c8\ub294\ub370 \ud5db\uc810\uc774 \ub9ce\uc740 \ub3d9\uc791\ub4e4\uc740 \ubaa8\ub450 \ubc30\uc81c\uc2dc\ud0a4\uace0 \ud6a8\uacfc\uc801\uc774\uace0 \uc989\uc2e4\uc131\uc774 \ub192\uc740 \ub3d9\uc791\ub4e4\ub85c \uc0c8\ub85c \uac1c\ud3b8\ub418\uc5c8\ub2e4\uace0 \ud55c\ub2e4. \ub610\ud55c \ud669\ube44\ud64d\uc740 \uc18c\uac78\uc544(\u8607\u4e5e\u5152)\ub85c\ubd80\ud130\u300a\ucde8\ud314\uc120\uad8c\u300b(\u9189\u516b\u4ed9\u62f3)\uc744, \ucca0\uad50\uc0bc(\u9435\u6a4b\u4e09)\uc758 \uc81c\uc790\uc778 \uc784\ubcf5\uc131(\u6797\u798f\u6210)\uc73c\ub85c\ubd80\ud130\ub294 \u300a\ucca0\uc120\uad8c\u300b(\u9435\u7dda\u62f3)\uacfc \u300a\ube44\ud0c0\u300b(\u98db\u7823)\ub97c, \uc1a1\ud718\ub2f9(\u5b8b\u8f1d\u93dc)\uc73c\ub85c\ubd80\ud130 \u300a\ubb34\uc601\uac01\u300b(\u7121\u5f71\u811a)\uc744 \uc804\uc218\ubc1b\uc544 \ud64d\uac00\uad8c\uc5d0 \ud3b8\uc785\uc2dc\ud0b4\uc73c\ub85c\uc368 \ub9c8\uce68\ub0b4 \uc624\ub298\ub0a0 \ud64d\uac00\uad8c\uc758 \ud615\ud0dc\ub97c \uc644\uc131\uc2dc\ucf30\ub2e4. \uc774\ub7ec\ud55c \uac1c\ub7c9\uacfc \ud3b8\uc785 \uacfc\uc815\uc5d0\uc11c \uc0c8\ub86d\uac8c \ucd94\uac00\ub41c \uad8c\ubc95\uc774 \ub9ce\uc544\uc9c0\uace0 \ud64d\uac00\uad8c\uc758 \uad50\uc2b5 \uccb4\uacc4\uac00 \ube44\ub85c\uc18c \uc644\ube44\ub418\uc5b4 \uadfc\ub300 \ud64d\uac00\uad8c\uc758 \ub9e5\uc744 \ud615\uc131\ud558\uc600\uc73c\ubbc0\ub85c \ud604\uc7ac \ud669\ube44\ud64d\uc774 \uacc4\ubcf4\ub97c \uc787\ub294 \ud64d\uac00\ubb38 \uc81c\uc790\ub4e4\uc740 \ud669\ube44\ud64d\uc744 \uc0ac\uc2e4\uc0c1 \uadfc\ub300 \ud64d\uac00\uad8c\uc758 \uc2dc\uc870\ub85c \ubc1b\ub4e4\uace0 \uc788\ub2e4. \ub610\ud55c \ud669\ube44\ud64d\uc740 \ucc44\ub9ac\ubd88\uad8c(\u8521\u674e\u4f5b\u62f3), \uc601\ucd98\uad8c(\u8a60\u6625\u62f3) \ub4f1 \ub0a8\ud30c \uc9c0\uc5ed\uc5d0\uc11c \uc131\ud589\ud558\ub294 \ubaa8\ub4e0 \uad8c\ubc95\uc744 \ub204\ub974\uace0 \ud64d\uac00\uad8c\uc744 \uc601\ub0a8 \ubb34\uc220 \uc911 \uc81c\uc77c\ubb34\uc220\uc758 \ubc18\uc5f4\uc5d0 \ub2e4\uc2dc \uc62c\ub824\ub193\uc544 \uc774\uc804\ubcf4\ub2e4 \ub354\uc6b1 \uba85\uc131\uc744 \ub192\uac8c \ud558\uc600\uc73c\ubbc0\ub85c \uc624\ub298\ub0a0\uc758 \ubb34\uc220\uac00\ub4e4\uc740 \ud669\ube44\ud64d\uc744 \ud64d\uac00\uad8c\uc758 \uc77c\ub300\uc885\uc0ac(\u4e00\u4ee3\u5b97\u5e2b)\uc774\uc790 \uc911\ud765\uc870\ub85c \ubc1b\ub4e4\uace0 \uc788\ub2e4.", "paragraph_answer": "\uc774\ud6c4 \ud64d\uac00\uad8c\uc740 \uc5ec\ub7ec \ub300\uc5d0 \uac70\uccd0 \ub0b4\ub824\uc624\uba74\uc11c \ub9c8\uce68\ub0b4 \ud669\ube44\ud64d (\u9ec3\u98db\u9d3b)\uc5d0\uac8c \uc804\uc218\ub418\uae30\uc5d0 \uc774\ub978\ub2e4. \ud669\ube44\ud64d\uc740 \uadf8\uc758 \uc544\ubc84\uc9c0 \ud669\uae30\uc601(\u9ec3\u9a65\u82f1)\uc73c\ub85c\ubd80\ud130 \uac00\ubb38 \ub300\ub300\ub85c \ub0b4\ub824\uc628 \ud64d\uac00\uad8c\uc744 \uc775\ud600 15\uc138\uc758 \ub098\uc774\uc5d0 \ub3c4\uc7a5\uc744 \uc6b4\uc601\ud558\uc600\ub294\ub370, \uadf8\uc758 \uc544\ubc84\uc9c0 \ud669\uae30\uc601\uc740 \uad11\ub3d9\uc2ed\ud638(\u5ee3\u6771\u5341\u864e)\uc758 \ubc18\uc5f4\uc5d0 \uc624\ub978 \uace0\uc218\uc600\ub2e4. \ud669\ube44\ud64d\uc740 \uc804\uc218\ubc1b\uc740 \ud64d\uac00\uad8c\uc758 \uad8c\ubc95\ub4e4\uc744 \ub2e4\uc2dc \ud55c\ubc88 \uac1c\ub7c9\ud558\uc5ec \ubd88\ud544\uc694\ud55c \ub3d9\uc791\uc744 \uc5c6\uc560\uace0 \uc790\uc2e0\uc774 \ub9cc\ub4e0 \uc0c8\ub85c\uc6b4 \ud615\ud0dc\ub97c \ucd94\uac00\ud558\uc600\ub2e4. \uc774\ub54c \uacf5\uc790\ubcf5\ud638\uad8c\uacfc \ud638\ud559\uc30d\ud615\uad8c\uc774 \ud06c\uac8c \uac1c\ub7c9\ub418\uc5c8\ub294\ub370 \ud5db\uc810\uc774 \ub9ce\uc740 \ub3d9\uc791\ub4e4\uc740 \ubaa8\ub450 \ubc30\uc81c\uc2dc\ud0a4\uace0 \ud6a8\uacfc\uc801\uc774\uace0 \uc989\uc2e4\uc131\uc774 \ub192\uc740 \ub3d9\uc791\ub4e4\ub85c \uc0c8\ub85c \uac1c\ud3b8\ub418\uc5c8\ub2e4\uace0 \ud55c\ub2e4. \ub610\ud55c \ud669\ube44\ud64d\uc740 \uc18c\uac78\uc544(\u8607\u4e5e\u5152)\ub85c\ubd80\ud130\u300a\ucde8\ud314\uc120\uad8c\u300b(\u9189\u516b\u4ed9\u62f3)\uc744, \ucca0\uad50\uc0bc(\u9435\u6a4b\u4e09)\uc758 \uc81c\uc790\uc778 \uc784\ubcf5\uc131(\u6797\u798f\u6210)\uc73c\ub85c\ubd80\ud130\ub294 \u300a\ucca0\uc120\uad8c\u300b(\u9435\u7dda\u62f3)\uacfc \u300a\ube44\ud0c0\u300b(\u98db\u7823)\ub97c, \uc1a1\ud718\ub2f9(\u5b8b\u8f1d\u93dc)\uc73c\ub85c\ubd80\ud130 \u300a\ubb34\uc601\uac01\u300b(\u7121\u5f71\u811a)\uc744 \uc804\uc218\ubc1b\uc544 \ud64d\uac00\uad8c\uc5d0 \ud3b8\uc785\uc2dc\ud0b4\uc73c\ub85c\uc368 \ub9c8\uce68\ub0b4 \uc624\ub298\ub0a0 \ud64d\uac00\uad8c\uc758 \ud615\ud0dc\ub97c \uc644\uc131\uc2dc\ucf30\ub2e4. \uc774\ub7ec\ud55c \uac1c\ub7c9\uacfc \ud3b8\uc785 \uacfc\uc815\uc5d0\uc11c \uc0c8\ub86d\uac8c \ucd94\uac00\ub41c \uad8c\ubc95\uc774 \ub9ce\uc544\uc9c0\uace0 \ud64d\uac00\uad8c\uc758 \uad50\uc2b5 \uccb4\uacc4\uac00 \ube44\ub85c\uc18c \uc644\ube44\ub418\uc5b4 \uadfc\ub300 \ud64d\uac00\uad8c\uc758 \ub9e5\uc744 \ud615\uc131\ud558\uc600\uc73c\ubbc0\ub85c \ud604\uc7ac \ud669\ube44\ud64d\uc774 \uacc4\ubcf4\ub97c \uc787\ub294 \ud64d\uac00\ubb38 \uc81c\uc790\ub4e4\uc740 \ud669\ube44\ud64d\uc744 \uc0ac\uc2e4\uc0c1 \uadfc\ub300 \ud64d\uac00\uad8c\uc758 \uc2dc\uc870\ub85c \ubc1b\ub4e4\uace0 \uc788\ub2e4. \ub610\ud55c \ud669\ube44\ud64d\uc740 \ucc44\ub9ac\ubd88\uad8c(\u8521\u674e\u4f5b\u62f3), \uc601\ucd98\uad8c(\u8a60\u6625\u62f3) \ub4f1 \ub0a8\ud30c \uc9c0\uc5ed\uc5d0\uc11c \uc131\ud589\ud558\ub294 \ubaa8\ub4e0 \uad8c\ubc95\uc744 \ub204\ub974\uace0 \ud64d\uac00\uad8c\uc744 \uc601\ub0a8 \ubb34\uc220 \uc911 \uc81c\uc77c\ubb34\uc220\uc758 \ubc18\uc5f4\uc5d0 \ub2e4\uc2dc \uc62c\ub824\ub193\uc544 \uc774\uc804\ubcf4\ub2e4 \ub354\uc6b1 \uba85\uc131\uc744 \ub192\uac8c \ud558\uc600\uc73c\ubbc0\ub85c \uc624\ub298\ub0a0\uc758 \ubb34\uc220\uac00\ub4e4\uc740 \ud669\ube44\ud64d\uc744 \ud64d\uac00\uad8c\uc758 \uc77c\ub300\uc885\uc0ac(\u4e00\u4ee3\u5b97\u5e2b)\uc774\uc790 \uc911\ud765\uc870\ub85c \ubc1b\ub4e4\uace0 \uc788\ub2e4.", "sentence_answer": "\ub9c8\uce68\ub0b4 \ud669\ube44\ud64d (\u9ec3\u98db\u9d3b)\uc5d0\uac8c \uc804\uc218\ub418\uae30\uc5d0 \uc774\ub978\ub2e4.", "paragraph_id": "6571977-0-0", "paragraph_question": "question: \ud64d\uac00\uad8c\uc740 \uc5ec\ub7ec \uc11c\ub300\uc5d0 \uac78\uccd0 \ub0b4\ub824\uc624\uba74\uc11c, \ub204\uad6c\uc5d0\uac8c \uc804\uc218 \ub418\ub098\uc694?, context: \uc774\ud6c4 \ud64d\uac00\uad8c\uc740 \uc5ec\ub7ec \ub300\uc5d0 \uac70\uccd0 \ub0b4\ub824\uc624\uba74\uc11c \ub9c8\uce68\ub0b4 \ud669\ube44\ud64d(\u9ec3\u98db\u9d3b)\uc5d0\uac8c \uc804\uc218\ub418\uae30\uc5d0 \uc774\ub978\ub2e4. \ud669\ube44\ud64d\uc740 \uadf8\uc758 \uc544\ubc84\uc9c0 \ud669\uae30\uc601(\u9ec3\u9a65\u82f1)\uc73c\ub85c\ubd80\ud130 \uac00\ubb38 \ub300\ub300\ub85c \ub0b4\ub824\uc628 \ud64d\uac00\uad8c\uc744 \uc775\ud600 15\uc138\uc758 \ub098\uc774\uc5d0 \ub3c4\uc7a5\uc744 \uc6b4\uc601\ud558\uc600\ub294\ub370, \uadf8\uc758 \uc544\ubc84\uc9c0 \ud669\uae30\uc601\uc740 \uad11\ub3d9\uc2ed\ud638(\u5ee3\u6771\u5341\u864e)\uc758 \ubc18\uc5f4\uc5d0 \uc624\ub978 \uace0\uc218\uc600\ub2e4. \ud669\ube44\ud64d\uc740 \uc804\uc218\ubc1b\uc740 \ud64d\uac00\uad8c\uc758 \uad8c\ubc95\ub4e4\uc744 \ub2e4\uc2dc \ud55c\ubc88 \uac1c\ub7c9\ud558\uc5ec \ubd88\ud544\uc694\ud55c \ub3d9\uc791\uc744 \uc5c6\uc560\uace0 \uc790\uc2e0\uc774 \ub9cc\ub4e0 \uc0c8\ub85c\uc6b4 \ud615\ud0dc\ub97c \ucd94\uac00\ud558\uc600\ub2e4. \uc774\ub54c \uacf5\uc790\ubcf5\ud638\uad8c\uacfc \ud638\ud559\uc30d\ud615\uad8c\uc774 \ud06c\uac8c \uac1c\ub7c9\ub418\uc5c8\ub294\ub370 \ud5db\uc810\uc774 \ub9ce\uc740 \ub3d9\uc791\ub4e4\uc740 \ubaa8\ub450 \ubc30\uc81c\uc2dc\ud0a4\uace0 \ud6a8\uacfc\uc801\uc774\uace0 \uc989\uc2e4\uc131\uc774 \ub192\uc740 \ub3d9\uc791\ub4e4\ub85c \uc0c8\ub85c \uac1c\ud3b8\ub418\uc5c8\ub2e4\uace0 \ud55c\ub2e4. \ub610\ud55c \ud669\ube44\ud64d\uc740 \uc18c\uac78\uc544(\u8607\u4e5e\u5152)\ub85c\ubd80\ud130\u300a\ucde8\ud314\uc120\uad8c\u300b(\u9189\u516b\u4ed9\u62f3)\uc744, \ucca0\uad50\uc0bc(\u9435\u6a4b\u4e09)\uc758 \uc81c\uc790\uc778 \uc784\ubcf5\uc131(\u6797\u798f\u6210)\uc73c\ub85c\ubd80\ud130\ub294 \u300a\ucca0\uc120\uad8c\u300b(\u9435\u7dda\u62f3)\uacfc \u300a\ube44\ud0c0\u300b(\u98db\u7823)\ub97c, \uc1a1\ud718\ub2f9(\u5b8b\u8f1d\u93dc)\uc73c\ub85c\ubd80\ud130 \u300a\ubb34\uc601\uac01\u300b(\u7121\u5f71\u811a)\uc744 \uc804\uc218\ubc1b\uc544 \ud64d\uac00\uad8c\uc5d0 \ud3b8\uc785\uc2dc\ud0b4\uc73c\ub85c\uc368 \ub9c8\uce68\ub0b4 \uc624\ub298\ub0a0 \ud64d\uac00\uad8c\uc758 \ud615\ud0dc\ub97c \uc644\uc131\uc2dc\ucf30\ub2e4. \uc774\ub7ec\ud55c \uac1c\ub7c9\uacfc \ud3b8\uc785 \uacfc\uc815\uc5d0\uc11c \uc0c8\ub86d\uac8c \ucd94\uac00\ub41c \uad8c\ubc95\uc774 \ub9ce\uc544\uc9c0\uace0 \ud64d\uac00\uad8c\uc758 \uad50\uc2b5 \uccb4\uacc4\uac00 \ube44\ub85c\uc18c \uc644\ube44\ub418\uc5b4 \uadfc\ub300 \ud64d\uac00\uad8c\uc758 \ub9e5\uc744 \ud615\uc131\ud558\uc600\uc73c\ubbc0\ub85c \ud604\uc7ac \ud669\ube44\ud64d\uc774 \uacc4\ubcf4\ub97c \uc787\ub294 \ud64d\uac00\ubb38 \uc81c\uc790\ub4e4\uc740 \ud669\ube44\ud64d\uc744 \uc0ac\uc2e4\uc0c1 \uadfc\ub300 \ud64d\uac00\uad8c\uc758 \uc2dc\uc870\ub85c \ubc1b\ub4e4\uace0 \uc788\ub2e4. \ub610\ud55c \ud669\ube44\ud64d\uc740 \ucc44\ub9ac\ubd88\uad8c(\u8521\u674e\u4f5b\u62f3), \uc601\ucd98\uad8c(\u8a60\u6625\u62f3) \ub4f1 \ub0a8\ud30c \uc9c0\uc5ed\uc5d0\uc11c \uc131\ud589\ud558\ub294 \ubaa8\ub4e0 \uad8c\ubc95\uc744 \ub204\ub974\uace0 \ud64d\uac00\uad8c\uc744 \uc601\ub0a8 \ubb34\uc220 \uc911 \uc81c\uc77c\ubb34\uc220\uc758 \ubc18\uc5f4\uc5d0 \ub2e4\uc2dc \uc62c\ub824\ub193\uc544 \uc774\uc804\ubcf4\ub2e4 \ub354\uc6b1 \uba85\uc131\uc744 \ub192\uac8c \ud558\uc600\uc73c\ubbc0\ub85c \uc624\ub298\ub0a0\uc758 \ubb34\uc220\uac00\ub4e4\uc740 \ud669\ube44\ud64d\uc744 \ud64d\uac00\uad8c\uc758 \uc77c\ub300\uc885\uc0ac(\u4e00\u4ee3\u5b97\u5e2b)\uc774\uc790 \uc911\ud765\uc870\ub85c \ubc1b\ub4e4\uace0 \uc788\ub2e4."} {"question": "\uc778\ub958\uac00 \uc774\uc6a9\ud574 \uc628 \uac00\uc7a5 \uc624\ub798\ub41c \uc57d\uc7ac \uc911 \ud558\ub098\uc774\uba70 \ud604\ub300\uc5d0\ub294\uc624\ub77d\uc774\ub098 \uc758\ub8cc\uc6a9\uc73c\ub85c \uc0ac\uc6a9\ub418\ub294 \uac83\uc740?", "paragraph": "\ub300\ub9c8\ucd08\ub294 \uc778\ub958\uac00 \uc774\uc6a9\ud574 \uc628 \uac00\uc7a5 \uc624\ub798\ub41c \uc57d\uc81c \uc911 \ud558\ub098\ub85c, \ud604\ub300\uc5d0\ub294 \uc8fc\ub85c \uc624\ub77d\uc801, \uc885\uad50\uc801(\uc601\uc801) \ub610\ub294 \uc758\ub8cc\uc801 \ubaa9\uc801\uc73c\ub85c \uc0ac\uc6a9\ub418\uace0 \uc788\ub2e4. \uad6d\uc81c \uc5f0\ud569(UN)\uc774 2004\ub144\uc5d0 \ucd94\uc815\ud55c \ubc14\uc5d0 \ub530\ub974\uba74, \uc804 \uc138\uacc4 \uc131\ub144 \uc778\uad6c\uc758 4% \uc815\ub3c4(\uc57d 1\uc5b56,200\ub9cc\uba85)\uac00 \ub300\ub9c8\ucd08\ub97c 1\ub144\uc5d0 1\ud68c \uc774\uc0c1 \ud761\uc5f0\ud558\uace0 \uc788\uc73c\uba70, \uc804 \uc138\uacc4 \uc131\ub144 \uc778\uad6c\uc758 0.6% \uc815\ub3c4(\uc57d 2,250\ub9cc\uba85)\uac00 1\uc77c 1\ud68c \uc774\uc0c1 \ud761\uc5f0\ud558\uace0 \uc788\ub2e4\uace0 \ud55c\ub2e4. \ub300\ub9c8\ucd08\uc758 \uc18c\uc9c0\ub098 \uc0ac\uc6a9, \ud310\ub9e4\ud589\uc704\ub294 \uadf8\uac83\uc774 \uac16\uace0 \uc788\ub294 \uac01\uc131 \ud6a8\uacfc \ub54c\ubb38\uc5d0 20\uc138\uae30 \ucd08\ubd80\ud130 \uc804 \uc138\uacc4 \uac70\uc758 \ub300\ubd80\ubd84\uc758 \ub098\ub77c\uc5d0\uc11c \ubd88\ubc95\uc73c\ub85c \uaddc\uc815\ub418\uc5c8\ub2e4. \uc804 \uc138\uacc4 \ub300\ubd80\ubd84 \uad6d\uac00\uc5d0\uc11c \ub300\ub9c8\ucd08\uc758 \ud761\uc5f0\uc740 \ubd88\ubc95\uc774\uace0 \uadf8 \uc7ac\ubc30\uc640 \uc720\ud1b5\uc740 \uc5c4\uaca9\ud55c \ud1b5\uc81c\ub97c \ubc1b\uc9c0\ub9cc, \uc77c\ubd80 \uad6d\uac00\uc5d0\uc11c\ub294 \uadf8\ub7ec\ud55c \uaddc\uc81c\ub97c \uc644\ud654\ud558\uace0 \uc788\ub2e4.", "answer": "\ub300\ub9c8\ucd08", "sentence": "\ub300\ub9c8\ucd08 \ub294 \uc778\ub958\uac00 \uc774\uc6a9\ud574 \uc628 \uac00\uc7a5 \uc624\ub798\ub41c \uc57d\uc81c \uc911 \ud558\ub098\ub85c, \ud604\ub300\uc5d0\ub294 \uc8fc\ub85c \uc624\ub77d\uc801, \uc885\uad50\uc801(\uc601\uc801) \ub610\ub294 \uc758\ub8cc\uc801 \ubaa9\uc801\uc73c\ub85c \uc0ac\uc6a9\ub418\uace0 \uc788\ub2e4.", "paragraph_sentence": " \ub300\ub9c8\ucd08 \ub294 \uc778\ub958\uac00 \uc774\uc6a9\ud574 \uc628 \uac00\uc7a5 \uc624\ub798\ub41c \uc57d\uc81c \uc911 \ud558\ub098\ub85c, \ud604\ub300\uc5d0\ub294 \uc8fc\ub85c \uc624\ub77d\uc801, \uc885\uad50\uc801(\uc601\uc801) \ub610\ub294 \uc758\ub8cc\uc801 \ubaa9\uc801\uc73c\ub85c \uc0ac\uc6a9\ub418\uace0 \uc788\ub2e4. \uad6d\uc81c \uc5f0\ud569(UN)\uc774 2004\ub144\uc5d0 \ucd94\uc815\ud55c \ubc14\uc5d0 \ub530\ub974\uba74, \uc804 \uc138\uacc4 \uc131\ub144 \uc778\uad6c\uc758 4% \uc815\ub3c4(\uc57d 1\uc5b56,200\ub9cc\uba85)\uac00 \ub300\ub9c8\ucd08\ub97c 1\ub144\uc5d0 1\ud68c \uc774\uc0c1 \ud761\uc5f0\ud558\uace0 \uc788\uc73c\uba70, \uc804 \uc138\uacc4 \uc131\ub144 \uc778\uad6c\uc758 0.6% \uc815\ub3c4(\uc57d 2,250\ub9cc\uba85)\uac00 1\uc77c 1\ud68c \uc774\uc0c1 \ud761\uc5f0\ud558\uace0 \uc788\ub2e4\uace0 \ud55c\ub2e4. \ub300\ub9c8\ucd08\uc758 \uc18c\uc9c0\ub098 \uc0ac\uc6a9, \ud310\ub9e4\ud589\uc704\ub294 \uadf8\uac83\uc774 \uac16\uace0 \uc788\ub294 \uac01\uc131 \ud6a8\uacfc \ub54c\ubb38\uc5d0 20\uc138\uae30 \ucd08\ubd80\ud130 \uc804 \uc138\uacc4 \uac70\uc758 \ub300\ubd80\ubd84\uc758 \ub098\ub77c\uc5d0\uc11c \ubd88\ubc95\uc73c\ub85c \uaddc\uc815\ub418\uc5c8\ub2e4. \uc804 \uc138\uacc4 \ub300\ubd80\ubd84 \uad6d\uac00\uc5d0\uc11c \ub300\ub9c8\ucd08\uc758 \ud761\uc5f0\uc740 \ubd88\ubc95\uc774\uace0 \uadf8 \uc7ac\ubc30\uc640 \uc720\ud1b5\uc740 \uc5c4\uaca9\ud55c \ud1b5\uc81c\ub97c \ubc1b\uc9c0\ub9cc, \uc77c\ubd80 \uad6d\uac00\uc5d0\uc11c\ub294 \uadf8\ub7ec\ud55c \uaddc\uc81c\ub97c \uc644\ud654\ud558\uace0 \uc788\ub2e4.", "paragraph_answer": " \ub300\ub9c8\ucd08 \ub294 \uc778\ub958\uac00 \uc774\uc6a9\ud574 \uc628 \uac00\uc7a5 \uc624\ub798\ub41c \uc57d\uc81c \uc911 \ud558\ub098\ub85c, \ud604\ub300\uc5d0\ub294 \uc8fc\ub85c \uc624\ub77d\uc801, \uc885\uad50\uc801(\uc601\uc801) \ub610\ub294 \uc758\ub8cc\uc801 \ubaa9\uc801\uc73c\ub85c \uc0ac\uc6a9\ub418\uace0 \uc788\ub2e4. \uad6d\uc81c \uc5f0\ud569(UN)\uc774 2004\ub144\uc5d0 \ucd94\uc815\ud55c \ubc14\uc5d0 \ub530\ub974\uba74, \uc804 \uc138\uacc4 \uc131\ub144 \uc778\uad6c\uc758 4% \uc815\ub3c4(\uc57d 1\uc5b56,200\ub9cc\uba85)\uac00 \ub300\ub9c8\ucd08\ub97c 1\ub144\uc5d0 1\ud68c \uc774\uc0c1 \ud761\uc5f0\ud558\uace0 \uc788\uc73c\uba70, \uc804 \uc138\uacc4 \uc131\ub144 \uc778\uad6c\uc758 0.6% \uc815\ub3c4(\uc57d 2,250\ub9cc\uba85)\uac00 1\uc77c 1\ud68c \uc774\uc0c1 \ud761\uc5f0\ud558\uace0 \uc788\ub2e4\uace0 \ud55c\ub2e4. \ub300\ub9c8\ucd08\uc758 \uc18c\uc9c0\ub098 \uc0ac\uc6a9, \ud310\ub9e4\ud589\uc704\ub294 \uadf8\uac83\uc774 \uac16\uace0 \uc788\ub294 \uac01\uc131 \ud6a8\uacfc \ub54c\ubb38\uc5d0 20\uc138\uae30 \ucd08\ubd80\ud130 \uc804 \uc138\uacc4 \uac70\uc758 \ub300\ubd80\ubd84\uc758 \ub098\ub77c\uc5d0\uc11c \ubd88\ubc95\uc73c\ub85c \uaddc\uc815\ub418\uc5c8\ub2e4. \uc804 \uc138\uacc4 \ub300\ubd80\ubd84 \uad6d\uac00\uc5d0\uc11c \ub300\ub9c8\ucd08\uc758 \ud761\uc5f0\uc740 \ubd88\ubc95\uc774\uace0 \uadf8 \uc7ac\ubc30\uc640 \uc720\ud1b5\uc740 \uc5c4\uaca9\ud55c \ud1b5\uc81c\ub97c \ubc1b\uc9c0\ub9cc, \uc77c\ubd80 \uad6d\uac00\uc5d0\uc11c\ub294 \uadf8\ub7ec\ud55c \uaddc\uc81c\ub97c \uc644\ud654\ud558\uace0 \uc788\ub2e4.", "sentence_answer": " \ub300\ub9c8\ucd08 \ub294 \uc778\ub958\uac00 \uc774\uc6a9\ud574 \uc628 \uac00\uc7a5 \uc624\ub798\ub41c \uc57d\uc81c \uc911 \ud558\ub098\ub85c, \ud604\ub300\uc5d0\ub294 \uc8fc\ub85c \uc624\ub77d\uc801, \uc885\uad50\uc801(\uc601\uc801) \ub610\ub294 \uc758\ub8cc\uc801 \ubaa9\uc801\uc73c\ub85c \uc0ac\uc6a9\ub418\uace0 \uc788\ub2e4.", "paragraph_id": "6485405-0-0", "paragraph_question": "question: \uc778\ub958\uac00 \uc774\uc6a9\ud574 \uc628 \uac00\uc7a5 \uc624\ub798\ub41c \uc57d\uc7ac \uc911 \ud558\ub098\uc774\uba70 \ud604\ub300\uc5d0\ub294\uc624\ub77d\uc774\ub098 \uc758\ub8cc\uc6a9\uc73c\ub85c \uc0ac\uc6a9\ub418\ub294 \uac83\uc740?, context: \ub300\ub9c8\ucd08\ub294 \uc778\ub958\uac00 \uc774\uc6a9\ud574 \uc628 \uac00\uc7a5 \uc624\ub798\ub41c \uc57d\uc81c \uc911 \ud558\ub098\ub85c, \ud604\ub300\uc5d0\ub294 \uc8fc\ub85c \uc624\ub77d\uc801, \uc885\uad50\uc801(\uc601\uc801) \ub610\ub294 \uc758\ub8cc\uc801 \ubaa9\uc801\uc73c\ub85c \uc0ac\uc6a9\ub418\uace0 \uc788\ub2e4. \uad6d\uc81c \uc5f0\ud569(UN)\uc774 2004\ub144\uc5d0 \ucd94\uc815\ud55c \ubc14\uc5d0 \ub530\ub974\uba74, \uc804 \uc138\uacc4 \uc131\ub144 \uc778\uad6c\uc758 4% \uc815\ub3c4(\uc57d 1\uc5b56,200\ub9cc\uba85)\uac00 \ub300\ub9c8\ucd08\ub97c 1\ub144\uc5d0 1\ud68c \uc774\uc0c1 \ud761\uc5f0\ud558\uace0 \uc788\uc73c\uba70, \uc804 \uc138\uacc4 \uc131\ub144 \uc778\uad6c\uc758 0.6% \uc815\ub3c4(\uc57d 2,250\ub9cc\uba85)\uac00 1\uc77c 1\ud68c \uc774\uc0c1 \ud761\uc5f0\ud558\uace0 \uc788\ub2e4\uace0 \ud55c\ub2e4. \ub300\ub9c8\ucd08\uc758 \uc18c\uc9c0\ub098 \uc0ac\uc6a9, \ud310\ub9e4\ud589\uc704\ub294 \uadf8\uac83\uc774 \uac16\uace0 \uc788\ub294 \uac01\uc131 \ud6a8\uacfc \ub54c\ubb38\uc5d0 20\uc138\uae30 \ucd08\ubd80\ud130 \uc804 \uc138\uacc4 \uac70\uc758 \ub300\ubd80\ubd84\uc758 \ub098\ub77c\uc5d0\uc11c \ubd88\ubc95\uc73c\ub85c \uaddc\uc815\ub418\uc5c8\ub2e4. \uc804 \uc138\uacc4 \ub300\ubd80\ubd84 \uad6d\uac00\uc5d0\uc11c \ub300\ub9c8\ucd08\uc758 \ud761\uc5f0\uc740 \ubd88\ubc95\uc774\uace0 \uadf8 \uc7ac\ubc30\uc640 \uc720\ud1b5\uc740 \uc5c4\uaca9\ud55c \ud1b5\uc81c\ub97c \ubc1b\uc9c0\ub9cc, \uc77c\ubd80 \uad6d\uac00\uc5d0\uc11c\ub294 \uadf8\ub7ec\ud55c \uaddc\uc81c\ub97c \uc644\ud654\ud558\uace0 \uc788\ub2e4."} {"question": "\ud3fc\ud398\uc774\uc6b0\uc2a4\uc758 \ud568\ub300\ub97c \uae30\uc6d0\uc804 36\ub144 \ub098\uc6b0\ub85c\ucfe0\uc2a4 \uc804\ud22c\uc5d0\uc11c \uc644\ud30c\uc2dc\ud0a8 \uad70\ub300\uc758 \uc218\uc7a5\uc740 \ub204\uad6c\uc778\uac00?", "paragraph": "\uae30\uc6d0\uc804 36\ub144 \uc625\ud0c0\ube44\uc544\ub204\uc2a4\uc640 \ub808\ud53c\ub450\uc2a4\ub294 \uc139\uc2a4\ud22c\uc2a4 \ud3fc\ud398\uc774\uc6b0\uc2a4\ub97c \uce58\uae30 \uc704\ud574 \ud569\ub3d9 \uc791\uc804\uc744 \uac1c\uc2dc\ud558\uc600\ub2e4. \uc625\ud0c0\ube44\uc544\ub204\uc2a4\ub97c \uc774\uacbc\uc74c\uc5d0\ub3c4 \ubd88\uad6c\ud558\uace0 \uc139\uc2a4\ud22c\uc2a4 \ud3fc\ud398\uc774\uc6b0\uc2a4\uc758 \ud568\ub300\ub294 \uae30\uc6d0\uc804 36\ub144 9\uc6d4 3\uc77c \ub098\uc6b0\ub85c\ucfe0\uc2a4 \uc804\ud22c\uc5d0\uc11c \uc544\uadf8\ub9ac\ud30c\uc758 \uad70\ub300\uc5d0\uac8c \uac70\uc758 \uc644\ud30c\ub418\uc5c8\ub2e4. \uc139\uc2a4\ud22c\uc2a4\ub294 \ub0a8\uc740 \uad70\ub300\ub97c \uc774\ub04c\uace0 \ub3d9\ucabd\uc73c\ub85c \ud1f4\uac01\ud558\uc600\ub294\ub370, \uadf8 \ub2e4\uc74c\ud574\uc5d0 \ud3fc\ud398\uc774\uc6b0\uc2a4\ub294 \uc548\ud1a0\ub2c8\uc6b0\uc2a4 \ud718\ud558\uc758 \uc7a5\uad70\uc5d0\uac8c \uc7a1\ud600 \ucc98\ud615\ub418\uc5c8\ub2e4. \uc625\ud0c0\ube44\uc544\ub204\uc2a4\uc640 \ub808\ud53c\ub450\uc2a4\ub294 \ud56d\ubcf5\ud55c \ud3fc\ud398\uc774\uc6b0\uc2a4\uc758 \ubcd1\uc0ac\ub97c \uc190\uc5d0 \ub123\uc5c8\ub294\ub370 \uba38\uc9c0\uc54a\uc544 \ub808\ud53c\ub450\uc2a4\ub294 \uc790\uc2e0\uc774 \uc2dc\uce60\ub9ac\uc544\ub97c \ub2e4\uc2a4\ub9b4 \ucda9\ubd84\ud55c \ub2a5\ub825\uc774 \ub41c\ub2e4 \uc790\ubd80\ud558\uace0 \uc625\ud0c0\ube44\uc544\ub204\uc2a4\uc5d0\uac8c \ub5a0\ub098\ub77c \uba85\ub839\ud558\uc600\ub2e4. \uadf8\ub7ec\ub098 \ub808\ud53c\ub450\uc2a4\uc758 \uad70\ub300\ub294 \uc790\uc2e0\ub4e4\uc774 \uc2f8\uc6b0\ub294 \ub370 \uc9c0\uce58\uace0 \uc625\ud0c0\ube44\uc544\ub204\uc2a4\uac00 \uadf8\ub4e4\uc744 \ub3c8\uc73c\ub85c \uc720\ud639\ud558\uc790 \ub808\ud53c\ub450\uc2a4\ub97c \ubc84\ub9ac\uace0 \uc625\ud0c0\ube44\uc544\ub204\uc2a4\uc5d0\uac8c\ub85c \ud22c\ud56d\ud558\uc600\ub2e4. \ub808\ud53c\ub450\uc2a4\ub294 \uc625\ud0c0\ube44\uc544\ub204\uc2a4\uc5d0\uac8c \ud56d\ubcf5\ud558\uace0 \ucd5c\uace0 \uc81c\uc0ac\uc7a5(pontifex maximus)\uc758 \uc9c1\ud568\uc744 \uc0ac\uc6a9\ud560 \uc218 \uc788\ub3c4\ub85d \ud5c8\ub77d\ubc1b\uc558\uc73c\ub098 \uc0bc\ub450 \uc815\uce58 \uccb4\uc81c\uc5d0\uc11c \ucad3\uaca8\ub098\uace0 \uacf5\uc9c1 \uc0dd\ud65c \uc5ed\uc2dc \ub05d\uc774 \ub098\uba74\uc11c \uc774\ud0c8\ub9ac\uc544\uc758 \uce74\ud398 \ud0a4\ub974\ucf00\uc774\uc5d0 \uc788\ub294 \uc7a5\uc6d0\uc73c\ub85c \uc720\ubc30\ub418\uc5c8\ub2e4. \ub85c\ub9c8\uc758 \ud1b5\uce58\uad8c\uc740 \uc774\uc81c \uc11c\ubc29\uc758 \uc625\ud0c0\ube44\uc544\ub204\uc2a4\uc640 \ub3d9\ubc29\uc758 \uc548\ud1a0\ub2c8\uc6b0\uc2a4\uc5d0\uac8c\ub85c \uc8fc\uc5b4\uc84c\uace0 \uc591\ubd84\ub418\uc5c8\ub2e4. \uacf5\ud654\uad6d\uc758 \ud3c9\ud654\uc640 \uc548\uc815\uc744 \uc9c0\ud0a4\uae30 \uc704\ud574 \uc625\ud0c0\ube44\uc544\ub204\uc2a4\ub294 \ub85c\ub9c8 \uc2dc\ubbfc\uc758 \uc7ac\uc0b0\uad8c\uc744 \ubcf4\uc7a5\ud574 \uc8fc\uc5c8\ub2e4. \uc625\ud0c0\ube44\uc544\ub204\uc2a4\uac00 \uc804\uc5ed\ud55c \ubcd1\uc0ac\ub4e4\uc5d0\uac8c \uc774\ud0c8\ub9ac\uc544 \uc678\uacfd\uc5d0 \uc790\ub9ac\ub97c \uc7a1\uac8c \ud574\uc8fc\ub294 \uc0ac\uc774, \uacfc\uac70 \ud3fc\ud398\uc774\uc6b0\uc2a4\uc758 \uad70\ub300\uc5d0 \ucc38\uac00\ud558\uae30 \uc704\ud574 \ub85c\ub9c8\ub97c \ub5a0\ub0ac\ub358 3\ub9cc\uc5ec \uba85\uc758 \ub178\uc608\ub97c \ubaa8\ub450 \uc8fc\uc778\uc5d0\uac8c \ub3cc\ub824\uc8fc\uc5c8\ub2e4. \uc625\ud0c0\ube44\uc544\ub204\uc2a4\ub294 \ub85c\ub9c8\ub85c \ub3cc\uc544\uc624\uc790 \uc790\uc2e0\uacfc \ub9ac\ube44\uc544, \uc625\ud0c0\ube44\uc544\uc758 \uc2e0\ubcc0 \uc548\uc804\uc758 \ubcf4\uc7a5\uc744 \uc6d0\ub85c\uc6d0\uc5d0 \uc694\uccad\ud558\uc600\ub2e4. \uadf8 \uacb0\uacfc \uc625\ud0c0\ube44\uc544\ub204\uc2a4\uc640 \uadf8\uc758 \uc544\ub0b4 \ub9ac\ube44\uc544, \ub204\uc774\uc778 \uc625\ud0c0\ube44\uc544, \ub538\uc778 \uc728\ub9ac\uc544\ub294 \uc8fc\uad8c\uba74\uc81c\uad8c\uc744 \ubd80\uc5ec\ubc1b\uc558\ub2e4.", "answer": "\uc544\uadf8\ub9ac\ud30c", "sentence": "\uc139\uc2a4\ud22c\uc2a4 \ud3fc\ud398\uc774\uc6b0\uc2a4\uc758 \ud568\ub300\ub294 \uae30\uc6d0\uc804 36\ub144 9\uc6d4 3\uc77c \ub098\uc6b0\ub85c\ucfe0\uc2a4 \uc804\ud22c\uc5d0\uc11c \uc544\uadf8\ub9ac\ud30c \uc758 \uad70\ub300\uc5d0\uac8c \uac70\uc758 \uc644\ud30c\ub418\uc5c8\ub2e4.", "paragraph_sentence": "\uae30\uc6d0\uc804 36\ub144 \uc625\ud0c0\ube44\uc544\ub204\uc2a4\uc640 \ub808\ud53c\ub450\uc2a4\ub294 \uc139\uc2a4\ud22c\uc2a4 \ud3fc\ud398\uc774\uc6b0\uc2a4\ub97c \uce58\uae30 \uc704\ud574 \ud569\ub3d9 \uc791\uc804\uc744 \uac1c\uc2dc\ud558\uc600\ub2e4. \uc625\ud0c0\ube44\uc544\ub204\uc2a4\ub97c \uc774\uacbc\uc74c\uc5d0\ub3c4 \ubd88\uad6c\ud558\uace0 \uc139\uc2a4\ud22c\uc2a4 \ud3fc\ud398\uc774\uc6b0\uc2a4\uc758 \ud568\ub300\ub294 \uae30\uc6d0\uc804 36\ub144 9\uc6d4 3\uc77c \ub098\uc6b0\ub85c\ucfe0\uc2a4 \uc804\ud22c\uc5d0\uc11c \uc544\uadf8\ub9ac\ud30c \uc758 \uad70\ub300\uc5d0\uac8c \uac70\uc758 \uc644\ud30c\ub418\uc5c8\ub2e4. \uc139\uc2a4\ud22c\uc2a4\ub294 \ub0a8\uc740 \uad70\ub300\ub97c \uc774\ub04c\uace0 \ub3d9\ucabd\uc73c\ub85c \ud1f4\uac01\ud558\uc600\ub294\ub370, \uadf8 \ub2e4\uc74c\ud574\uc5d0 \ud3fc\ud398\uc774\uc6b0\uc2a4\ub294 \uc548\ud1a0\ub2c8\uc6b0\uc2a4 \ud718\ud558\uc758 \uc7a5\uad70\uc5d0\uac8c \uc7a1\ud600 \ucc98\ud615\ub418\uc5c8\ub2e4. \uc625\ud0c0\ube44\uc544\ub204\uc2a4\uc640 \ub808\ud53c\ub450\uc2a4\ub294 \ud56d\ubcf5\ud55c \ud3fc\ud398\uc774\uc6b0\uc2a4\uc758 \ubcd1\uc0ac\ub97c \uc190\uc5d0 \ub123\uc5c8\ub294\ub370 \uba38\uc9c0\uc54a\uc544 \ub808\ud53c\ub450\uc2a4\ub294 \uc790\uc2e0\uc774 \uc2dc\uce60\ub9ac\uc544\ub97c \ub2e4\uc2a4\ub9b4 \ucda9\ubd84\ud55c \ub2a5\ub825\uc774 \ub41c\ub2e4 \uc790\ubd80\ud558\uace0 \uc625\ud0c0\ube44\uc544\ub204\uc2a4\uc5d0\uac8c \ub5a0\ub098\ub77c \uba85\ub839\ud558\uc600\ub2e4. \uadf8\ub7ec\ub098 \ub808\ud53c\ub450\uc2a4\uc758 \uad70\ub300\ub294 \uc790\uc2e0\ub4e4\uc774 \uc2f8\uc6b0\ub294 \ub370 \uc9c0\uce58\uace0 \uc625\ud0c0\ube44\uc544\ub204\uc2a4\uac00 \uadf8\ub4e4\uc744 \ub3c8\uc73c\ub85c \uc720\ud639\ud558\uc790 \ub808\ud53c\ub450\uc2a4\ub97c \ubc84\ub9ac\uace0 \uc625\ud0c0\ube44\uc544\ub204\uc2a4\uc5d0\uac8c\ub85c \ud22c\ud56d\ud558\uc600\ub2e4. \ub808\ud53c\ub450\uc2a4\ub294 \uc625\ud0c0\ube44\uc544\ub204\uc2a4\uc5d0\uac8c \ud56d\ubcf5\ud558\uace0 \ucd5c\uace0 \uc81c\uc0ac\uc7a5(pontifex maximus)\uc758 \uc9c1\ud568\uc744 \uc0ac\uc6a9\ud560 \uc218 \uc788\ub3c4\ub85d \ud5c8\ub77d\ubc1b\uc558\uc73c\ub098 \uc0bc\ub450 \uc815\uce58 \uccb4\uc81c\uc5d0\uc11c \ucad3\uaca8\ub098\uace0 \uacf5\uc9c1 \uc0dd\ud65c \uc5ed\uc2dc \ub05d\uc774 \ub098\uba74\uc11c \uc774\ud0c8\ub9ac\uc544\uc758 \uce74\ud398 \ud0a4\ub974\ucf00\uc774\uc5d0 \uc788\ub294 \uc7a5\uc6d0\uc73c\ub85c \uc720\ubc30\ub418\uc5c8\ub2e4. \ub85c\ub9c8\uc758 \ud1b5\uce58\uad8c\uc740 \uc774\uc81c \uc11c\ubc29\uc758 \uc625\ud0c0\ube44\uc544\ub204\uc2a4\uc640 \ub3d9\ubc29\uc758 \uc548\ud1a0\ub2c8\uc6b0\uc2a4\uc5d0\uac8c\ub85c \uc8fc\uc5b4\uc84c\uace0 \uc591\ubd84\ub418\uc5c8\ub2e4. \uacf5\ud654\uad6d\uc758 \ud3c9\ud654\uc640 \uc548\uc815\uc744 \uc9c0\ud0a4\uae30 \uc704\ud574 \uc625\ud0c0\ube44\uc544\ub204\uc2a4\ub294 \ub85c\ub9c8 \uc2dc\ubbfc\uc758 \uc7ac\uc0b0\uad8c\uc744 \ubcf4\uc7a5\ud574 \uc8fc\uc5c8\ub2e4. \uc625\ud0c0\ube44\uc544\ub204\uc2a4\uac00 \uc804\uc5ed\ud55c \ubcd1\uc0ac\ub4e4\uc5d0\uac8c \uc774\ud0c8\ub9ac\uc544 \uc678\uacfd\uc5d0 \uc790\ub9ac\ub97c \uc7a1\uac8c \ud574\uc8fc\ub294 \uc0ac\uc774, \uacfc\uac70 \ud3fc\ud398\uc774\uc6b0\uc2a4\uc758 \uad70\ub300\uc5d0 \ucc38\uac00\ud558\uae30 \uc704\ud574 \ub85c\ub9c8\ub97c \ub5a0\ub0ac\ub358 3\ub9cc\uc5ec \uba85\uc758 \ub178\uc608\ub97c \ubaa8\ub450 \uc8fc\uc778\uc5d0\uac8c \ub3cc\ub824\uc8fc\uc5c8\ub2e4. \uc625\ud0c0\ube44\uc544\ub204\uc2a4\ub294 \ub85c\ub9c8\ub85c \ub3cc\uc544\uc624\uc790 \uc790\uc2e0\uacfc \ub9ac\ube44\uc544, \uc625\ud0c0\ube44\uc544\uc758 \uc2e0\ubcc0 \uc548\uc804\uc758 \ubcf4\uc7a5\uc744 \uc6d0\ub85c\uc6d0\uc5d0 \uc694\uccad\ud558\uc600\ub2e4. \uadf8 \uacb0\uacfc \uc625\ud0c0\ube44\uc544\ub204\uc2a4\uc640 \uadf8\uc758 \uc544\ub0b4 \ub9ac\ube44\uc544, \ub204\uc774\uc778 \uc625\ud0c0\ube44\uc544, \ub538\uc778 \uc728\ub9ac\uc544\ub294 \uc8fc\uad8c\uba74\uc81c\uad8c\uc744 \ubd80\uc5ec\ubc1b\uc558\ub2e4.", "paragraph_answer": "\uae30\uc6d0\uc804 36\ub144 \uc625\ud0c0\ube44\uc544\ub204\uc2a4\uc640 \ub808\ud53c\ub450\uc2a4\ub294 \uc139\uc2a4\ud22c\uc2a4 \ud3fc\ud398\uc774\uc6b0\uc2a4\ub97c \uce58\uae30 \uc704\ud574 \ud569\ub3d9 \uc791\uc804\uc744 \uac1c\uc2dc\ud558\uc600\ub2e4. \uc625\ud0c0\ube44\uc544\ub204\uc2a4\ub97c \uc774\uacbc\uc74c\uc5d0\ub3c4 \ubd88\uad6c\ud558\uace0 \uc139\uc2a4\ud22c\uc2a4 \ud3fc\ud398\uc774\uc6b0\uc2a4\uc758 \ud568\ub300\ub294 \uae30\uc6d0\uc804 36\ub144 9\uc6d4 3\uc77c \ub098\uc6b0\ub85c\ucfe0\uc2a4 \uc804\ud22c\uc5d0\uc11c \uc544\uadf8\ub9ac\ud30c \uc758 \uad70\ub300\uc5d0\uac8c \uac70\uc758 \uc644\ud30c\ub418\uc5c8\ub2e4. \uc139\uc2a4\ud22c\uc2a4\ub294 \ub0a8\uc740 \uad70\ub300\ub97c \uc774\ub04c\uace0 \ub3d9\ucabd\uc73c\ub85c \ud1f4\uac01\ud558\uc600\ub294\ub370, \uadf8 \ub2e4\uc74c\ud574\uc5d0 \ud3fc\ud398\uc774\uc6b0\uc2a4\ub294 \uc548\ud1a0\ub2c8\uc6b0\uc2a4 \ud718\ud558\uc758 \uc7a5\uad70\uc5d0\uac8c \uc7a1\ud600 \ucc98\ud615\ub418\uc5c8\ub2e4. \uc625\ud0c0\ube44\uc544\ub204\uc2a4\uc640 \ub808\ud53c\ub450\uc2a4\ub294 \ud56d\ubcf5\ud55c \ud3fc\ud398\uc774\uc6b0\uc2a4\uc758 \ubcd1\uc0ac\ub97c \uc190\uc5d0 \ub123\uc5c8\ub294\ub370 \uba38\uc9c0\uc54a\uc544 \ub808\ud53c\ub450\uc2a4\ub294 \uc790\uc2e0\uc774 \uc2dc\uce60\ub9ac\uc544\ub97c \ub2e4\uc2a4\ub9b4 \ucda9\ubd84\ud55c \ub2a5\ub825\uc774 \ub41c\ub2e4 \uc790\ubd80\ud558\uace0 \uc625\ud0c0\ube44\uc544\ub204\uc2a4\uc5d0\uac8c \ub5a0\ub098\ub77c \uba85\ub839\ud558\uc600\ub2e4. \uadf8\ub7ec\ub098 \ub808\ud53c\ub450\uc2a4\uc758 \uad70\ub300\ub294 \uc790\uc2e0\ub4e4\uc774 \uc2f8\uc6b0\ub294 \ub370 \uc9c0\uce58\uace0 \uc625\ud0c0\ube44\uc544\ub204\uc2a4\uac00 \uadf8\ub4e4\uc744 \ub3c8\uc73c\ub85c \uc720\ud639\ud558\uc790 \ub808\ud53c\ub450\uc2a4\ub97c \ubc84\ub9ac\uace0 \uc625\ud0c0\ube44\uc544\ub204\uc2a4\uc5d0\uac8c\ub85c \ud22c\ud56d\ud558\uc600\ub2e4. \ub808\ud53c\ub450\uc2a4\ub294 \uc625\ud0c0\ube44\uc544\ub204\uc2a4\uc5d0\uac8c \ud56d\ubcf5\ud558\uace0 \ucd5c\uace0 \uc81c\uc0ac\uc7a5(pontifex maximus)\uc758 \uc9c1\ud568\uc744 \uc0ac\uc6a9\ud560 \uc218 \uc788\ub3c4\ub85d \ud5c8\ub77d\ubc1b\uc558\uc73c\ub098 \uc0bc\ub450 \uc815\uce58 \uccb4\uc81c\uc5d0\uc11c \ucad3\uaca8\ub098\uace0 \uacf5\uc9c1 \uc0dd\ud65c \uc5ed\uc2dc \ub05d\uc774 \ub098\uba74\uc11c \uc774\ud0c8\ub9ac\uc544\uc758 \uce74\ud398 \ud0a4\ub974\ucf00\uc774\uc5d0 \uc788\ub294 \uc7a5\uc6d0\uc73c\ub85c \uc720\ubc30\ub418\uc5c8\ub2e4. \ub85c\ub9c8\uc758 \ud1b5\uce58\uad8c\uc740 \uc774\uc81c \uc11c\ubc29\uc758 \uc625\ud0c0\ube44\uc544\ub204\uc2a4\uc640 \ub3d9\ubc29\uc758 \uc548\ud1a0\ub2c8\uc6b0\uc2a4\uc5d0\uac8c\ub85c \uc8fc\uc5b4\uc84c\uace0 \uc591\ubd84\ub418\uc5c8\ub2e4. \uacf5\ud654\uad6d\uc758 \ud3c9\ud654\uc640 \uc548\uc815\uc744 \uc9c0\ud0a4\uae30 \uc704\ud574 \uc625\ud0c0\ube44\uc544\ub204\uc2a4\ub294 \ub85c\ub9c8 \uc2dc\ubbfc\uc758 \uc7ac\uc0b0\uad8c\uc744 \ubcf4\uc7a5\ud574 \uc8fc\uc5c8\ub2e4. \uc625\ud0c0\ube44\uc544\ub204\uc2a4\uac00 \uc804\uc5ed\ud55c \ubcd1\uc0ac\ub4e4\uc5d0\uac8c \uc774\ud0c8\ub9ac\uc544 \uc678\uacfd\uc5d0 \uc790\ub9ac\ub97c \uc7a1\uac8c \ud574\uc8fc\ub294 \uc0ac\uc774, \uacfc\uac70 \ud3fc\ud398\uc774\uc6b0\uc2a4\uc758 \uad70\ub300\uc5d0 \ucc38\uac00\ud558\uae30 \uc704\ud574 \ub85c\ub9c8\ub97c \ub5a0\ub0ac\ub358 3\ub9cc\uc5ec \uba85\uc758 \ub178\uc608\ub97c \ubaa8\ub450 \uc8fc\uc778\uc5d0\uac8c \ub3cc\ub824\uc8fc\uc5c8\ub2e4. \uc625\ud0c0\ube44\uc544\ub204\uc2a4\ub294 \ub85c\ub9c8\ub85c \ub3cc\uc544\uc624\uc790 \uc790\uc2e0\uacfc \ub9ac\ube44\uc544, \uc625\ud0c0\ube44\uc544\uc758 \uc2e0\ubcc0 \uc548\uc804\uc758 \ubcf4\uc7a5\uc744 \uc6d0\ub85c\uc6d0\uc5d0 \uc694\uccad\ud558\uc600\ub2e4. \uadf8 \uacb0\uacfc \uc625\ud0c0\ube44\uc544\ub204\uc2a4\uc640 \uadf8\uc758 \uc544\ub0b4 \ub9ac\ube44\uc544, \ub204\uc774\uc778 \uc625\ud0c0\ube44\uc544, \ub538\uc778 \uc728\ub9ac\uc544\ub294 \uc8fc\uad8c\uba74\uc81c\uad8c\uc744 \ubd80\uc5ec\ubc1b\uc558\ub2e4.", "sentence_answer": "\uc139\uc2a4\ud22c\uc2a4 \ud3fc\ud398\uc774\uc6b0\uc2a4\uc758 \ud568\ub300\ub294 \uae30\uc6d0\uc804 36\ub144 9\uc6d4 3\uc77c \ub098\uc6b0\ub85c\ucfe0\uc2a4 \uc804\ud22c\uc5d0\uc11c \uc544\uadf8\ub9ac\ud30c \uc758 \uad70\ub300\uc5d0\uac8c \uac70\uc758 \uc644\ud30c\ub418\uc5c8\ub2e4.", "paragraph_id": "6657258-16-0", "paragraph_question": "question: \ud3fc\ud398\uc774\uc6b0\uc2a4\uc758 \ud568\ub300\ub97c \uae30\uc6d0\uc804 36\ub144 \ub098\uc6b0\ub85c\ucfe0\uc2a4 \uc804\ud22c\uc5d0\uc11c \uc644\ud30c\uc2dc\ud0a8 \uad70\ub300\uc758 \uc218\uc7a5\uc740 \ub204\uad6c\uc778\uac00?, context: \uae30\uc6d0\uc804 36\ub144 \uc625\ud0c0\ube44\uc544\ub204\uc2a4\uc640 \ub808\ud53c\ub450\uc2a4\ub294 \uc139\uc2a4\ud22c\uc2a4 \ud3fc\ud398\uc774\uc6b0\uc2a4\ub97c \uce58\uae30 \uc704\ud574 \ud569\ub3d9 \uc791\uc804\uc744 \uac1c\uc2dc\ud558\uc600\ub2e4. \uc625\ud0c0\ube44\uc544\ub204\uc2a4\ub97c \uc774\uacbc\uc74c\uc5d0\ub3c4 \ubd88\uad6c\ud558\uace0 \uc139\uc2a4\ud22c\uc2a4 \ud3fc\ud398\uc774\uc6b0\uc2a4\uc758 \ud568\ub300\ub294 \uae30\uc6d0\uc804 36\ub144 9\uc6d4 3\uc77c \ub098\uc6b0\ub85c\ucfe0\uc2a4 \uc804\ud22c\uc5d0\uc11c \uc544\uadf8\ub9ac\ud30c\uc758 \uad70\ub300\uc5d0\uac8c \uac70\uc758 \uc644\ud30c\ub418\uc5c8\ub2e4. \uc139\uc2a4\ud22c\uc2a4\ub294 \ub0a8\uc740 \uad70\ub300\ub97c \uc774\ub04c\uace0 \ub3d9\ucabd\uc73c\ub85c \ud1f4\uac01\ud558\uc600\ub294\ub370, \uadf8 \ub2e4\uc74c\ud574\uc5d0 \ud3fc\ud398\uc774\uc6b0\uc2a4\ub294 \uc548\ud1a0\ub2c8\uc6b0\uc2a4 \ud718\ud558\uc758 \uc7a5\uad70\uc5d0\uac8c \uc7a1\ud600 \ucc98\ud615\ub418\uc5c8\ub2e4. \uc625\ud0c0\ube44\uc544\ub204\uc2a4\uc640 \ub808\ud53c\ub450\uc2a4\ub294 \ud56d\ubcf5\ud55c \ud3fc\ud398\uc774\uc6b0\uc2a4\uc758 \ubcd1\uc0ac\ub97c \uc190\uc5d0 \ub123\uc5c8\ub294\ub370 \uba38\uc9c0\uc54a\uc544 \ub808\ud53c\ub450\uc2a4\ub294 \uc790\uc2e0\uc774 \uc2dc\uce60\ub9ac\uc544\ub97c \ub2e4\uc2a4\ub9b4 \ucda9\ubd84\ud55c \ub2a5\ub825\uc774 \ub41c\ub2e4 \uc790\ubd80\ud558\uace0 \uc625\ud0c0\ube44\uc544\ub204\uc2a4\uc5d0\uac8c \ub5a0\ub098\ub77c \uba85\ub839\ud558\uc600\ub2e4. \uadf8\ub7ec\ub098 \ub808\ud53c\ub450\uc2a4\uc758 \uad70\ub300\ub294 \uc790\uc2e0\ub4e4\uc774 \uc2f8\uc6b0\ub294 \ub370 \uc9c0\uce58\uace0 \uc625\ud0c0\ube44\uc544\ub204\uc2a4\uac00 \uadf8\ub4e4\uc744 \ub3c8\uc73c\ub85c \uc720\ud639\ud558\uc790 \ub808\ud53c\ub450\uc2a4\ub97c \ubc84\ub9ac\uace0 \uc625\ud0c0\ube44\uc544\ub204\uc2a4\uc5d0\uac8c\ub85c \ud22c\ud56d\ud558\uc600\ub2e4. \ub808\ud53c\ub450\uc2a4\ub294 \uc625\ud0c0\ube44\uc544\ub204\uc2a4\uc5d0\uac8c \ud56d\ubcf5\ud558\uace0 \ucd5c\uace0 \uc81c\uc0ac\uc7a5(pontifex maximus)\uc758 \uc9c1\ud568\uc744 \uc0ac\uc6a9\ud560 \uc218 \uc788\ub3c4\ub85d \ud5c8\ub77d\ubc1b\uc558\uc73c\ub098 \uc0bc\ub450 \uc815\uce58 \uccb4\uc81c\uc5d0\uc11c \ucad3\uaca8\ub098\uace0 \uacf5\uc9c1 \uc0dd\ud65c \uc5ed\uc2dc \ub05d\uc774 \ub098\uba74\uc11c \uc774\ud0c8\ub9ac\uc544\uc758 \uce74\ud398 \ud0a4\ub974\ucf00\uc774\uc5d0 \uc788\ub294 \uc7a5\uc6d0\uc73c\ub85c \uc720\ubc30\ub418\uc5c8\ub2e4. \ub85c\ub9c8\uc758 \ud1b5\uce58\uad8c\uc740 \uc774\uc81c \uc11c\ubc29\uc758 \uc625\ud0c0\ube44\uc544\ub204\uc2a4\uc640 \ub3d9\ubc29\uc758 \uc548\ud1a0\ub2c8\uc6b0\uc2a4\uc5d0\uac8c\ub85c \uc8fc\uc5b4\uc84c\uace0 \uc591\ubd84\ub418\uc5c8\ub2e4. \uacf5\ud654\uad6d\uc758 \ud3c9\ud654\uc640 \uc548\uc815\uc744 \uc9c0\ud0a4\uae30 \uc704\ud574 \uc625\ud0c0\ube44\uc544\ub204\uc2a4\ub294 \ub85c\ub9c8 \uc2dc\ubbfc\uc758 \uc7ac\uc0b0\uad8c\uc744 \ubcf4\uc7a5\ud574 \uc8fc\uc5c8\ub2e4. \uc625\ud0c0\ube44\uc544\ub204\uc2a4\uac00 \uc804\uc5ed\ud55c \ubcd1\uc0ac\ub4e4\uc5d0\uac8c \uc774\ud0c8\ub9ac\uc544 \uc678\uacfd\uc5d0 \uc790\ub9ac\ub97c \uc7a1\uac8c \ud574\uc8fc\ub294 \uc0ac\uc774, \uacfc\uac70 \ud3fc\ud398\uc774\uc6b0\uc2a4\uc758 \uad70\ub300\uc5d0 \ucc38\uac00\ud558\uae30 \uc704\ud574 \ub85c\ub9c8\ub97c \ub5a0\ub0ac\ub358 3\ub9cc\uc5ec \uba85\uc758 \ub178\uc608\ub97c \ubaa8\ub450 \uc8fc\uc778\uc5d0\uac8c \ub3cc\ub824\uc8fc\uc5c8\ub2e4. \uc625\ud0c0\ube44\uc544\ub204\uc2a4\ub294 \ub85c\ub9c8\ub85c \ub3cc\uc544\uc624\uc790 \uc790\uc2e0\uacfc \ub9ac\ube44\uc544, \uc625\ud0c0\ube44\uc544\uc758 \uc2e0\ubcc0 \uc548\uc804\uc758 \ubcf4\uc7a5\uc744 \uc6d0\ub85c\uc6d0\uc5d0 \uc694\uccad\ud558\uc600\ub2e4. \uadf8 \uacb0\uacfc \uc625\ud0c0\ube44\uc544\ub204\uc2a4\uc640 \uadf8\uc758 \uc544\ub0b4 \ub9ac\ube44\uc544, \ub204\uc774\uc778 \uc625\ud0c0\ube44\uc544, \ub538\uc778 \uc728\ub9ac\uc544\ub294 \uc8fc\uad8c\uba74\uc81c\uad8c\uc744 \ubd80\uc5ec\ubc1b\uc558\ub2e4."} {"question": "\uc911\uad6d\uc758 \uc870\uc120\uc5d0 \ub300\ud55c \uae30\ub85d\uc774 \uc788\ub294 \ubb38\ud5cc \uc911 \uac00\uc7a5 \uc624\ub798\ub41c \uac83\uc740?", "paragraph": "\ud604\uc804\ud558\ub294 \uc911\uad6d\uc758 \uc870\uc120\uc5d0 \ub300\ud55c \uae30\ub85d\uc73c\ub85c\ub294 \uae30\uc6d0\uc804 7\uc138\uae30 \ubb34\ub835\uc758 \ubb38\ud5cc\uc778 \u300a\uad00\uc790\u300b\uc5d0 \ub4f1\uc7a5\ud558\ub294 \uac83\uc774 \uac00\uc7a5 \uc624\ub79c \uac83\uc774\ub2e4. \uae30\uc6d0\uc804 323\ub144 \uc911\uad6d \uc804\uad6d\uc2dc\ub300\uc758 \uc5f0\uacfc \uc678\uad50\uc801 \ub9c8\ucc30\uc774 \uc788\uc5c8\uc73c\uba70, \uc11c\uae30\uc804 3\uc138\uae30 \ucd08\uc5d0 \uc5f0\uc758 \uacf5\uaca9\uc73c\ub85c 2\ucc9c\uc5ec \ub9ac\uc758 \uac15\ud1a0\ub97c \uc0c1\uc2e4\ud558\uace0 \ub9cc\ubc88\ud55c\uc744 \uacbd\uacc4\ub85c \uc0bc\uc558\ub2e4. \uc774 \ubb34\ub835\uc5d0 \uace0\uc870\uc120\uc758 \uc911\uc2ec\uc9c0\uac00 \uc694\ub3d9\uc5d0\uc11c \ud3c9\uc591\uc73c\ub85c \uc62e\uaca8 \uac14\ub2e4\ub294 \uc8fc\uc7a5\uc774 \uc788\ub2e4. \u300a\uc704\ub7b5\u300b\uc758 \uae30\ub85d\uc5d0 \ub530\ub974\uba74 \uc11c\uae30\uc804 2\uc138\uae30 \ub9d0 \uc9c4\uc2dc\ud669\uc774 \uc911\uad6d\uc744 \ud1b5\uc77c\ud558\uc790 \uc870\uc120\uc655 \ubd80(\u5426)\uac00 \ud615\uc2dd\uc801\uc73c\ub85c \uc9c4\ub098\ub77c\uc5d0 \ubcf5\uc18d\ud558\uc600\ub2e4\uace0 \ud55c\ub2e4. \uc774\ud6c4 \uc9c4 \ub9d0\uae30\uc758 \ud63c\ub780\uae30\uc5d0 \uc911\uad6d \uc720\ubbfc\ub4e4\uc744 \ub9ce\uc774 \ubc1b\uc544\ub4e4\uc600\uc73c\uba70 \ud2b9\ud788 \uae30\uc6d0\uc804 195\ub144, \uc704\ub9cc\uc758 \ub9dd\uba85\uc744 \ubc1b\uc544\ub4e4\uc5ec \uc11c\ucabd \ubcc0\uacbd\uc758 \uc218\ube44\ub97c \ub9e1\uacbc\ub2e4. \uae30\uc6d0\uc804 194\ub144, \uc704\ub9cc\uc740 \uc900\uc655\uc744 \ucad3\uc544\ub0b4\uace0 \uc655\uc704\ub97c \ucc2c\ud0c8\ud558\uc5ec \uc704\ub9cc\uc870\uc120\uc744 \uc138\uc6e0\ub2e4.", "answer": "\uad00\uc790", "sentence": "\ud604\uc804\ud558\ub294 \uc911\uad6d\uc758 \uc870\uc120\uc5d0 \ub300\ud55c \uae30\ub85d\uc73c\ub85c\ub294 \uae30\uc6d0\uc804 7\uc138\uae30 \ubb34\ub835\uc758 \ubb38\ud5cc\uc778 \u300a \uad00\uc790 \u300b\uc5d0 \ub4f1\uc7a5\ud558\ub294 \uac83\uc774 \uac00\uc7a5 \uc624\ub79c \uac83\uc774\ub2e4.", "paragraph_sentence": " \ud604\uc804\ud558\ub294 \uc911\uad6d\uc758 \uc870\uc120\uc5d0 \ub300\ud55c \uae30\ub85d\uc73c\ub85c\ub294 \uae30\uc6d0\uc804 7\uc138\uae30 \ubb34\ub835\uc758 \ubb38\ud5cc\uc778 \u300a \uad00\uc790 \u300b\uc5d0 \ub4f1\uc7a5\ud558\ub294 \uac83\uc774 \uac00\uc7a5 \uc624\ub79c \uac83\uc774\ub2e4. \uae30\uc6d0\uc804 323\ub144 \uc911\uad6d \uc804\uad6d\uc2dc\ub300\uc758 \uc5f0\uacfc \uc678\uad50\uc801 \ub9c8\ucc30\uc774 \uc788\uc5c8\uc73c\uba70, \uc11c\uae30\uc804 3\uc138\uae30 \ucd08\uc5d0 \uc5f0\uc758 \uacf5\uaca9\uc73c\ub85c 2\ucc9c\uc5ec \ub9ac\uc758 \uac15\ud1a0\ub97c \uc0c1\uc2e4\ud558\uace0 \ub9cc\ubc88\ud55c\uc744 \uacbd\uacc4\ub85c \uc0bc\uc558\ub2e4. \uc774 \ubb34\ub835\uc5d0 \uace0\uc870\uc120\uc758 \uc911\uc2ec\uc9c0\uac00 \uc694\ub3d9\uc5d0\uc11c \ud3c9\uc591\uc73c\ub85c \uc62e\uaca8 \uac14\ub2e4\ub294 \uc8fc\uc7a5\uc774 \uc788\ub2e4. \u300a\uc704\ub7b5\u300b\uc758 \uae30\ub85d\uc5d0 \ub530\ub974\uba74 \uc11c\uae30\uc804 2\uc138\uae30 \ub9d0 \uc9c4\uc2dc\ud669\uc774 \uc911\uad6d\uc744 \ud1b5\uc77c\ud558\uc790 \uc870\uc120\uc655 \ubd80(\u5426)\uac00 \ud615\uc2dd\uc801\uc73c\ub85c \uc9c4\ub098\ub77c\uc5d0 \ubcf5\uc18d\ud558\uc600\ub2e4\uace0 \ud55c\ub2e4. \uc774\ud6c4 \uc9c4 \ub9d0\uae30\uc758 \ud63c\ub780\uae30\uc5d0 \uc911\uad6d \uc720\ubbfc\ub4e4\uc744 \ub9ce\uc774 \ubc1b\uc544\ub4e4\uc600\uc73c\uba70 \ud2b9\ud788 \uae30\uc6d0\uc804 195\ub144, \uc704\ub9cc\uc758 \ub9dd\uba85\uc744 \ubc1b\uc544\ub4e4\uc5ec \uc11c\ucabd \ubcc0\uacbd\uc758 \uc218\ube44\ub97c \ub9e1\uacbc\ub2e4. \uae30\uc6d0\uc804 194\ub144, \uc704\ub9cc\uc740 \uc900\uc655\uc744 \ucad3\uc544\ub0b4\uace0 \uc655\uc704\ub97c \ucc2c\ud0c8\ud558\uc5ec \uc704\ub9cc\uc870\uc120\uc744 \uc138\uc6e0\ub2e4.", "paragraph_answer": "\ud604\uc804\ud558\ub294 \uc911\uad6d\uc758 \uc870\uc120\uc5d0 \ub300\ud55c \uae30\ub85d\uc73c\ub85c\ub294 \uae30\uc6d0\uc804 7\uc138\uae30 \ubb34\ub835\uc758 \ubb38\ud5cc\uc778 \u300a \uad00\uc790 \u300b\uc5d0 \ub4f1\uc7a5\ud558\ub294 \uac83\uc774 \uac00\uc7a5 \uc624\ub79c \uac83\uc774\ub2e4. \uae30\uc6d0\uc804 323\ub144 \uc911\uad6d \uc804\uad6d\uc2dc\ub300\uc758 \uc5f0\uacfc \uc678\uad50\uc801 \ub9c8\ucc30\uc774 \uc788\uc5c8\uc73c\uba70, \uc11c\uae30\uc804 3\uc138\uae30 \ucd08\uc5d0 \uc5f0\uc758 \uacf5\uaca9\uc73c\ub85c 2\ucc9c\uc5ec \ub9ac\uc758 \uac15\ud1a0\ub97c \uc0c1\uc2e4\ud558\uace0 \ub9cc\ubc88\ud55c\uc744 \uacbd\uacc4\ub85c \uc0bc\uc558\ub2e4. \uc774 \ubb34\ub835\uc5d0 \uace0\uc870\uc120\uc758 \uc911\uc2ec\uc9c0\uac00 \uc694\ub3d9\uc5d0\uc11c \ud3c9\uc591\uc73c\ub85c \uc62e\uaca8 \uac14\ub2e4\ub294 \uc8fc\uc7a5\uc774 \uc788\ub2e4. \u300a\uc704\ub7b5\u300b\uc758 \uae30\ub85d\uc5d0 \ub530\ub974\uba74 \uc11c\uae30\uc804 2\uc138\uae30 \ub9d0 \uc9c4\uc2dc\ud669\uc774 \uc911\uad6d\uc744 \ud1b5\uc77c\ud558\uc790 \uc870\uc120\uc655 \ubd80(\u5426)\uac00 \ud615\uc2dd\uc801\uc73c\ub85c \uc9c4\ub098\ub77c\uc5d0 \ubcf5\uc18d\ud558\uc600\ub2e4\uace0 \ud55c\ub2e4. \uc774\ud6c4 \uc9c4 \ub9d0\uae30\uc758 \ud63c\ub780\uae30\uc5d0 \uc911\uad6d \uc720\ubbfc\ub4e4\uc744 \ub9ce\uc774 \ubc1b\uc544\ub4e4\uc600\uc73c\uba70 \ud2b9\ud788 \uae30\uc6d0\uc804 195\ub144, \uc704\ub9cc\uc758 \ub9dd\uba85\uc744 \ubc1b\uc544\ub4e4\uc5ec \uc11c\ucabd \ubcc0\uacbd\uc758 \uc218\ube44\ub97c \ub9e1\uacbc\ub2e4. \uae30\uc6d0\uc804 194\ub144, \uc704\ub9cc\uc740 \uc900\uc655\uc744 \ucad3\uc544\ub0b4\uace0 \uc655\uc704\ub97c \ucc2c\ud0c8\ud558\uc5ec \uc704\ub9cc\uc870\uc120\uc744 \uc138\uc6e0\ub2e4.", "sentence_answer": "\ud604\uc804\ud558\ub294 \uc911\uad6d\uc758 \uc870\uc120\uc5d0 \ub300\ud55c \uae30\ub85d\uc73c\ub85c\ub294 \uae30\uc6d0\uc804 7\uc138\uae30 \ubb34\ub835\uc758 \ubb38\ud5cc\uc778 \u300a \uad00\uc790 \u300b\uc5d0 \ub4f1\uc7a5\ud558\ub294 \uac83\uc774 \uac00\uc7a5 \uc624\ub79c \uac83\uc774\ub2e4.", "paragraph_id": "6490822-1-0", "paragraph_question": "question: \uc911\uad6d\uc758 \uc870\uc120\uc5d0 \ub300\ud55c \uae30\ub85d\uc774 \uc788\ub294 \ubb38\ud5cc \uc911 \uac00\uc7a5 \uc624\ub798\ub41c \uac83\uc740?, context: \ud604\uc804\ud558\ub294 \uc911\uad6d\uc758 \uc870\uc120\uc5d0 \ub300\ud55c \uae30\ub85d\uc73c\ub85c\ub294 \uae30\uc6d0\uc804 7\uc138\uae30 \ubb34\ub835\uc758 \ubb38\ud5cc\uc778 \u300a\uad00\uc790\u300b\uc5d0 \ub4f1\uc7a5\ud558\ub294 \uac83\uc774 \uac00\uc7a5 \uc624\ub79c \uac83\uc774\ub2e4. \uae30\uc6d0\uc804 323\ub144 \uc911\uad6d \uc804\uad6d\uc2dc\ub300\uc758 \uc5f0\uacfc \uc678\uad50\uc801 \ub9c8\ucc30\uc774 \uc788\uc5c8\uc73c\uba70, \uc11c\uae30\uc804 3\uc138\uae30 \ucd08\uc5d0 \uc5f0\uc758 \uacf5\uaca9\uc73c\ub85c 2\ucc9c\uc5ec \ub9ac\uc758 \uac15\ud1a0\ub97c \uc0c1\uc2e4\ud558\uace0 \ub9cc\ubc88\ud55c\uc744 \uacbd\uacc4\ub85c \uc0bc\uc558\ub2e4. \uc774 \ubb34\ub835\uc5d0 \uace0\uc870\uc120\uc758 \uc911\uc2ec\uc9c0\uac00 \uc694\ub3d9\uc5d0\uc11c \ud3c9\uc591\uc73c\ub85c \uc62e\uaca8 \uac14\ub2e4\ub294 \uc8fc\uc7a5\uc774 \uc788\ub2e4. \u300a\uc704\ub7b5\u300b\uc758 \uae30\ub85d\uc5d0 \ub530\ub974\uba74 \uc11c\uae30\uc804 2\uc138\uae30 \ub9d0 \uc9c4\uc2dc\ud669\uc774 \uc911\uad6d\uc744 \ud1b5\uc77c\ud558\uc790 \uc870\uc120\uc655 \ubd80(\u5426)\uac00 \ud615\uc2dd\uc801\uc73c\ub85c \uc9c4\ub098\ub77c\uc5d0 \ubcf5\uc18d\ud558\uc600\ub2e4\uace0 \ud55c\ub2e4. \uc774\ud6c4 \uc9c4 \ub9d0\uae30\uc758 \ud63c\ub780\uae30\uc5d0 \uc911\uad6d \uc720\ubbfc\ub4e4\uc744 \ub9ce\uc774 \ubc1b\uc544\ub4e4\uc600\uc73c\uba70 \ud2b9\ud788 \uae30\uc6d0\uc804 195\ub144, \uc704\ub9cc\uc758 \ub9dd\uba85\uc744 \ubc1b\uc544\ub4e4\uc5ec \uc11c\ucabd \ubcc0\uacbd\uc758 \uc218\ube44\ub97c \ub9e1\uacbc\ub2e4. \uae30\uc6d0\uc804 194\ub144, \uc704\ub9cc\uc740 \uc900\uc655\uc744 \ucad3\uc544\ub0b4\uace0 \uc655\uc704\ub97c \ucc2c\ud0c8\ud558\uc5ec \uc704\ub9cc\uc870\uc120\uc744 \uc138\uc6e0\ub2e4."} {"question": "8\uccb4\uc9c8 \uc911 \uace0\uacf5 \ub2e4\uc774\ube59\uacfc \uc2ec\ud574\uc7a0\uc218 \uc2a4\ucfe0\ubc84 \ub2e4\uc774\ube59\uc740 \uc5b4\ub5a4 \uccb4\uc9c8\uc5d0 \uc801\ud569\ud55c\uac00?", "paragraph": "8\uccb4\uc9c8\uc758\ud559\uc758 \ud0dc\ub3d9\uacfc \ub354\ubd88\uc5b4 8\uccb4\uc9c8\ud2b9\uc131\uc758 \ubc14\ud0d5\uc73c\ub85c \uc18c\uac1c\ub41c \ub0b4\uc6a9\uc744 \ubcf4\uba74 \uccb4\uc9c8\ubcc4\ub85c \ub300\ud45c \uc6b4\ub3d9 \uc885\ubaa9\uc774 \uc5b8\uae09\ub41c\ub2e4. 12\ucabd \ucc38\uc870. \uacf5\uc744 \ub2e4\ub8e8\ub294 \uc57c\uad6c, \ucd95\uad6c, \uace8\ud504\ub294 \ubaa9\uc74c\uccb4\uc9c8\uc5d0 \uc801\ud569\ud55c \uad6c\uae30\uc885\ubaa9\uc774\uba70, \ub9c8\ub77c\ud1a4\uc740 \ud3d0\uae30\uc6b4\uacfc \uc9c0\uad6c\ub825\uc774 \uac15\ud55c \uae08\uc74c\uccb4\uc9c8\uc758 \uace0\uc720\ud55c \ub2a5\ub825\uc744 \ub4dc\ub7ec\ub0b4\ub294 \uc6b4\ub3d9\uc774\ub2e4. \uc774\uc5d0 \ub300\ud574 \ucd94\uac00\uc801 \uc5f0\uad6c\uc640 \uad00\ucc30\uc744 \ud1b5\ud574 \ubcf4\ub2e4 \ub2e4\uc591\ud558\uace0 \uad6c\uccb4\uc801\uc778 \uccb4\uc9c8\uc801 \ubd84\ub958\uac00 \uc2dc\ub3c4\ub418\uace0 \uc788\ub2e4.[24] \uc774 \uc678\uc5d0\ub3c4 \ub18d\uad6c\uc640 \ud14c\ub2c8\uc2a4\uc758 \ubaa9\uc74c\uccb4\uc9c8, \uc7a5\uac70\ub9ac \ub2ec\ub9ac\uae30\uc640 \uc11c\ucee4\uc2a4 \ubc0f \uae30\uacc4\uccb4\uc870\uc758 \uae08\uc74c\uccb4\uc9c8, \ubd81\uad6d\uacf0 \uc218\uc601\uacfc \uace0\uacf5 \ub2e4\uc774\ube59\uacfc \uc2ec\ud574\uc7a0\uc218 \uc2a4\ucfe0\ubc84 \ub2e4\uc774\ube59\uc758 \uae08\uc591\uccb4\uc9c8, \uac00\uc7a5 \uc794\uc778\ud55c \uc774\uc885\uaca9\ud22c\uae30\uc758 \uc81c\uc655\uc801 \ud2b9\uc131\uc73c\ub85c \ubaa9\uc591\uccb4\uc9c8, \ub208\uc124\ubbf8\uac00 \ube60\ub974\uace0 \uc190\ubaa9\uc758 \ud798\uc774 \uac15\ud55c \ud0c1\uad6c\uc640 \uc544\uc774\uc2a4\ud558\ud0a4\uc640 \ubc30\ub4dc\ubbfc\ud134 \uadf8\ub9ac\uace0 \uac80\ub3c4\uc758 \ud1a0\uc591\uccb4\uc9c8, \uc77c\ubcf8\uc5d0\uc11c \ubc1c\ub2ec\ud55c \uc2a4\ubaa8\uc120\uc218\uc758 \ud1a0\uc74c\uccb4\uc9c8, \ud0dc\uad8c\ub3c4\uc640 \uc591\uad81\uc5d0 \uae30\ub7c9\uc744 \ub3cb\ubcf4\uc774\ub294 \uc218\uc74c\uccb4\uc9c8, \ub9ac\ub4ec\uccb4\uc870\uc640 \uac19\uc740 \uac00\ubcbc\uc6b4 \ub3d9\uc791\uc758 \uade0\ud615\ubbf8\ub97c \ubc1c\uc0b0\ud558\ub294 \uc218\uc591\uccb4\uc9c8 \ub4f1 \uc7a5\ubd80\uac15\uc57d\uc758 \uc6d0\ub9ac\uc5d0 \ub530\ub978 \uc2e0\uccb4\ubc1c\ub2ec\uc744 \uc801\uc6a9\ud558\ub294 8\uccb4\uc9c8\uc758 \uc6b4\ub3d9\uc801 \uae30\ub7c9\uc740 \ub69c\ub837\ud558\uac8c \ub098\ub258\uc5b4 \uc9c4\ub2e4.", "answer": "\uae08\uc591\uccb4\uc9c8", "sentence": "\uc774\uc5d0 \ub300\ud574 \ucd94\uac00\uc801 \uc5f0\uad6c\uc640 \uad00\ucc30\uc744 \ud1b5\ud574 \ubcf4\ub2e4 \ub2e4\uc591\ud558\uace0 \uad6c\uccb4\uc801\uc778 \uccb4\uc9c8\uc801 \ubd84\ub958\uac00 \uc2dc\ub3c4\ub418\uace0 \uc788\ub2e4.[24] \uc774 \uc678\uc5d0\ub3c4 \ub18d\uad6c\uc640 \ud14c\ub2c8\uc2a4\uc758 \ubaa9\uc74c\uccb4\uc9c8, \uc7a5\uac70\ub9ac \ub2ec\ub9ac\uae30\uc640 \uc11c\ucee4\uc2a4 \ubc0f \uae30\uacc4\uccb4\uc870\uc758 \uae08\uc74c\uccb4\uc9c8, \ubd81\uad6d\uacf0 \uc218\uc601\uacfc \uace0\uacf5 \ub2e4\uc774\ube59\uacfc \uc2ec\ud574\uc7a0\uc218 \uc2a4\ucfe0\ubc84 \ub2e4\uc774\ube59\uc758 \uae08\uc591\uccb4\uc9c8 ,", "paragraph_sentence": "8\uccb4\uc9c8\uc758\ud559\uc758 \ud0dc\ub3d9\uacfc \ub354\ubd88\uc5b4 8\uccb4\uc9c8\ud2b9\uc131\uc758 \ubc14\ud0d5\uc73c\ub85c \uc18c\uac1c\ub41c \ub0b4\uc6a9\uc744 \ubcf4\uba74 \uccb4\uc9c8\ubcc4\ub85c \ub300\ud45c \uc6b4\ub3d9 \uc885\ubaa9\uc774 \uc5b8\uae09\ub41c\ub2e4. 12\ucabd \ucc38\uc870. \uacf5\uc744 \ub2e4\ub8e8\ub294 \uc57c\uad6c, \ucd95\uad6c, \uace8\ud504\ub294 \ubaa9\uc74c\uccb4\uc9c8\uc5d0 \uc801\ud569\ud55c \uad6c\uae30\uc885\ubaa9\uc774\uba70, \ub9c8\ub77c\ud1a4\uc740 \ud3d0\uae30\uc6b4\uacfc \uc9c0\uad6c\ub825\uc774 \uac15\ud55c \uae08\uc74c\uccb4\uc9c8\uc758 \uace0\uc720\ud55c \ub2a5\ub825\uc744 \ub4dc\ub7ec\ub0b4\ub294 \uc6b4\ub3d9\uc774\ub2e4. \uc774\uc5d0 \ub300\ud574 \ucd94\uac00\uc801 \uc5f0\uad6c\uc640 \uad00\ucc30\uc744 \ud1b5\ud574 \ubcf4\ub2e4 \ub2e4\uc591\ud558\uace0 \uad6c\uccb4\uc801\uc778 \uccb4\uc9c8\uc801 \ubd84\ub958\uac00 \uc2dc\ub3c4\ub418\uace0 \uc788\ub2e4.[24] \uc774 \uc678\uc5d0\ub3c4 \ub18d\uad6c\uc640 \ud14c\ub2c8\uc2a4\uc758 \ubaa9\uc74c\uccb4\uc9c8, \uc7a5\uac70\ub9ac \ub2ec\ub9ac\uae30\uc640 \uc11c\ucee4\uc2a4 \ubc0f \uae30\uacc4\uccb4\uc870\uc758 \uae08\uc74c\uccb4\uc9c8, \ubd81\uad6d\uacf0 \uc218\uc601\uacfc \uace0\uacf5 \ub2e4\uc774\ube59\uacfc \uc2ec\ud574\uc7a0\uc218 \uc2a4\ucfe0\ubc84 \ub2e4\uc774\ube59\uc758 \uae08\uc591\uccb4\uc9c8 , \uac00\uc7a5 \uc794\uc778\ud55c \uc774\uc885\uaca9\ud22c\uae30\uc758 \uc81c\uc655\uc801 \ud2b9\uc131\uc73c\ub85c \ubaa9\uc591\uccb4\uc9c8, \ub208\uc124\ubbf8\uac00 \ube60\ub974\uace0 \uc190\ubaa9\uc758 \ud798\uc774 \uac15\ud55c \ud0c1\uad6c\uc640 \uc544\uc774\uc2a4\ud558\ud0a4\uc640 \ubc30\ub4dc\ubbfc\ud134 \uadf8\ub9ac\uace0 \uac80\ub3c4\uc758 \ud1a0\uc591\uccb4\uc9c8, \uc77c\ubcf8\uc5d0\uc11c \ubc1c\ub2ec\ud55c \uc2a4\ubaa8\uc120\uc218\uc758 \ud1a0\uc74c\uccb4\uc9c8, \ud0dc\uad8c\ub3c4\uc640 \uc591\uad81\uc5d0 \uae30\ub7c9\uc744 \ub3cb\ubcf4\uc774\ub294 \uc218\uc74c\uccb4\uc9c8, \ub9ac\ub4ec\uccb4\uc870\uc640 \uac19\uc740 \uac00\ubcbc\uc6b4 \ub3d9\uc791\uc758 \uade0\ud615\ubbf8\ub97c \ubc1c\uc0b0\ud558\ub294 \uc218\uc591\uccb4\uc9c8 \ub4f1 \uc7a5\ubd80\uac15\uc57d\uc758 \uc6d0\ub9ac\uc5d0 \ub530\ub978 \uc2e0\uccb4\ubc1c\ub2ec\uc744 \uc801\uc6a9\ud558\ub294 8\uccb4\uc9c8\uc758 \uc6b4\ub3d9\uc801 \uae30\ub7c9\uc740 \ub69c\ub837\ud558\uac8c \ub098\ub258\uc5b4 \uc9c4\ub2e4.", "paragraph_answer": "8\uccb4\uc9c8\uc758\ud559\uc758 \ud0dc\ub3d9\uacfc \ub354\ubd88\uc5b4 8\uccb4\uc9c8\ud2b9\uc131\uc758 \ubc14\ud0d5\uc73c\ub85c \uc18c\uac1c\ub41c \ub0b4\uc6a9\uc744 \ubcf4\uba74 \uccb4\uc9c8\ubcc4\ub85c \ub300\ud45c \uc6b4\ub3d9 \uc885\ubaa9\uc774 \uc5b8\uae09\ub41c\ub2e4. 12\ucabd \ucc38\uc870. \uacf5\uc744 \ub2e4\ub8e8\ub294 \uc57c\uad6c, \ucd95\uad6c, \uace8\ud504\ub294 \ubaa9\uc74c\uccb4\uc9c8\uc5d0 \uc801\ud569\ud55c \uad6c\uae30\uc885\ubaa9\uc774\uba70, \ub9c8\ub77c\ud1a4\uc740 \ud3d0\uae30\uc6b4\uacfc \uc9c0\uad6c\ub825\uc774 \uac15\ud55c \uae08\uc74c\uccb4\uc9c8\uc758 \uace0\uc720\ud55c \ub2a5\ub825\uc744 \ub4dc\ub7ec\ub0b4\ub294 \uc6b4\ub3d9\uc774\ub2e4. \uc774\uc5d0 \ub300\ud574 \ucd94\uac00\uc801 \uc5f0\uad6c\uc640 \uad00\ucc30\uc744 \ud1b5\ud574 \ubcf4\ub2e4 \ub2e4\uc591\ud558\uace0 \uad6c\uccb4\uc801\uc778 \uccb4\uc9c8\uc801 \ubd84\ub958\uac00 \uc2dc\ub3c4\ub418\uace0 \uc788\ub2e4.[24] \uc774 \uc678\uc5d0\ub3c4 \ub18d\uad6c\uc640 \ud14c\ub2c8\uc2a4\uc758 \ubaa9\uc74c\uccb4\uc9c8, \uc7a5\uac70\ub9ac \ub2ec\ub9ac\uae30\uc640 \uc11c\ucee4\uc2a4 \ubc0f \uae30\uacc4\uccb4\uc870\uc758 \uae08\uc74c\uccb4\uc9c8, \ubd81\uad6d\uacf0 \uc218\uc601\uacfc \uace0\uacf5 \ub2e4\uc774\ube59\uacfc \uc2ec\ud574\uc7a0\uc218 \uc2a4\ucfe0\ubc84 \ub2e4\uc774\ube59\uc758 \uae08\uc591\uccb4\uc9c8 , \uac00\uc7a5 \uc794\uc778\ud55c \uc774\uc885\uaca9\ud22c\uae30\uc758 \uc81c\uc655\uc801 \ud2b9\uc131\uc73c\ub85c \ubaa9\uc591\uccb4\uc9c8, \ub208\uc124\ubbf8\uac00 \ube60\ub974\uace0 \uc190\ubaa9\uc758 \ud798\uc774 \uac15\ud55c \ud0c1\uad6c\uc640 \uc544\uc774\uc2a4\ud558\ud0a4\uc640 \ubc30\ub4dc\ubbfc\ud134 \uadf8\ub9ac\uace0 \uac80\ub3c4\uc758 \ud1a0\uc591\uccb4\uc9c8, \uc77c\ubcf8\uc5d0\uc11c \ubc1c\ub2ec\ud55c \uc2a4\ubaa8\uc120\uc218\uc758 \ud1a0\uc74c\uccb4\uc9c8, \ud0dc\uad8c\ub3c4\uc640 \uc591\uad81\uc5d0 \uae30\ub7c9\uc744 \ub3cb\ubcf4\uc774\ub294 \uc218\uc74c\uccb4\uc9c8, \ub9ac\ub4ec\uccb4\uc870\uc640 \uac19\uc740 \uac00\ubcbc\uc6b4 \ub3d9\uc791\uc758 \uade0\ud615\ubbf8\ub97c \ubc1c\uc0b0\ud558\ub294 \uc218\uc591\uccb4\uc9c8 \ub4f1 \uc7a5\ubd80\uac15\uc57d\uc758 \uc6d0\ub9ac\uc5d0 \ub530\ub978 \uc2e0\uccb4\ubc1c\ub2ec\uc744 \uc801\uc6a9\ud558\ub294 8\uccb4\uc9c8\uc758 \uc6b4\ub3d9\uc801 \uae30\ub7c9\uc740 \ub69c\ub837\ud558\uac8c \ub098\ub258\uc5b4 \uc9c4\ub2e4.", "sentence_answer": "\uc774\uc5d0 \ub300\ud574 \ucd94\uac00\uc801 \uc5f0\uad6c\uc640 \uad00\ucc30\uc744 \ud1b5\ud574 \ubcf4\ub2e4 \ub2e4\uc591\ud558\uace0 \uad6c\uccb4\uc801\uc778 \uccb4\uc9c8\uc801 \ubd84\ub958\uac00 \uc2dc\ub3c4\ub418\uace0 \uc788\ub2e4.[24] \uc774 \uc678\uc5d0\ub3c4 \ub18d\uad6c\uc640 \ud14c\ub2c8\uc2a4\uc758 \ubaa9\uc74c\uccb4\uc9c8, \uc7a5\uac70\ub9ac \ub2ec\ub9ac\uae30\uc640 \uc11c\ucee4\uc2a4 \ubc0f \uae30\uacc4\uccb4\uc870\uc758 \uae08\uc74c\uccb4\uc9c8, \ubd81\uad6d\uacf0 \uc218\uc601\uacfc \uace0\uacf5 \ub2e4\uc774\ube59\uacfc \uc2ec\ud574\uc7a0\uc218 \uc2a4\ucfe0\ubc84 \ub2e4\uc774\ube59\uc758 \uae08\uc591\uccb4\uc9c8 ,", "paragraph_id": "6566734-12-1", "paragraph_question": "question: 8\uccb4\uc9c8 \uc911 \uace0\uacf5 \ub2e4\uc774\ube59\uacfc \uc2ec\ud574\uc7a0\uc218 \uc2a4\ucfe0\ubc84 \ub2e4\uc774\ube59\uc740 \uc5b4\ub5a4 \uccb4\uc9c8\uc5d0 \uc801\ud569\ud55c\uac00?, context: 8\uccb4\uc9c8\uc758\ud559\uc758 \ud0dc\ub3d9\uacfc \ub354\ubd88\uc5b4 8\uccb4\uc9c8\ud2b9\uc131\uc758 \ubc14\ud0d5\uc73c\ub85c \uc18c\uac1c\ub41c \ub0b4\uc6a9\uc744 \ubcf4\uba74 \uccb4\uc9c8\ubcc4\ub85c \ub300\ud45c \uc6b4\ub3d9 \uc885\ubaa9\uc774 \uc5b8\uae09\ub41c\ub2e4. 12\ucabd \ucc38\uc870. \uacf5\uc744 \ub2e4\ub8e8\ub294 \uc57c\uad6c, \ucd95\uad6c, \uace8\ud504\ub294 \ubaa9\uc74c\uccb4\uc9c8\uc5d0 \uc801\ud569\ud55c \uad6c\uae30\uc885\ubaa9\uc774\uba70, \ub9c8\ub77c\ud1a4\uc740 \ud3d0\uae30\uc6b4\uacfc \uc9c0\uad6c\ub825\uc774 \uac15\ud55c \uae08\uc74c\uccb4\uc9c8\uc758 \uace0\uc720\ud55c \ub2a5\ub825\uc744 \ub4dc\ub7ec\ub0b4\ub294 \uc6b4\ub3d9\uc774\ub2e4. \uc774\uc5d0 \ub300\ud574 \ucd94\uac00\uc801 \uc5f0\uad6c\uc640 \uad00\ucc30\uc744 \ud1b5\ud574 \ubcf4\ub2e4 \ub2e4\uc591\ud558\uace0 \uad6c\uccb4\uc801\uc778 \uccb4\uc9c8\uc801 \ubd84\ub958\uac00 \uc2dc\ub3c4\ub418\uace0 \uc788\ub2e4.[24] \uc774 \uc678\uc5d0\ub3c4 \ub18d\uad6c\uc640 \ud14c\ub2c8\uc2a4\uc758 \ubaa9\uc74c\uccb4\uc9c8, \uc7a5\uac70\ub9ac \ub2ec\ub9ac\uae30\uc640 \uc11c\ucee4\uc2a4 \ubc0f \uae30\uacc4\uccb4\uc870\uc758 \uae08\uc74c\uccb4\uc9c8, \ubd81\uad6d\uacf0 \uc218\uc601\uacfc \uace0\uacf5 \ub2e4\uc774\ube59\uacfc \uc2ec\ud574\uc7a0\uc218 \uc2a4\ucfe0\ubc84 \ub2e4\uc774\ube59\uc758 \uae08\uc591\uccb4\uc9c8, \uac00\uc7a5 \uc794\uc778\ud55c \uc774\uc885\uaca9\ud22c\uae30\uc758 \uc81c\uc655\uc801 \ud2b9\uc131\uc73c\ub85c \ubaa9\uc591\uccb4\uc9c8, \ub208\uc124\ubbf8\uac00 \ube60\ub974\uace0 \uc190\ubaa9\uc758 \ud798\uc774 \uac15\ud55c \ud0c1\uad6c\uc640 \uc544\uc774\uc2a4\ud558\ud0a4\uc640 \ubc30\ub4dc\ubbfc\ud134 \uadf8\ub9ac\uace0 \uac80\ub3c4\uc758 \ud1a0\uc591\uccb4\uc9c8, \uc77c\ubcf8\uc5d0\uc11c \ubc1c\ub2ec\ud55c \uc2a4\ubaa8\uc120\uc218\uc758 \ud1a0\uc74c\uccb4\uc9c8, \ud0dc\uad8c\ub3c4\uc640 \uc591\uad81\uc5d0 \uae30\ub7c9\uc744 \ub3cb\ubcf4\uc774\ub294 \uc218\uc74c\uccb4\uc9c8, \ub9ac\ub4ec\uccb4\uc870\uc640 \uac19\uc740 \uac00\ubcbc\uc6b4 \ub3d9\uc791\uc758 \uade0\ud615\ubbf8\ub97c \ubc1c\uc0b0\ud558\ub294 \uc218\uc591\uccb4\uc9c8 \ub4f1 \uc7a5\ubd80\uac15\uc57d\uc758 \uc6d0\ub9ac\uc5d0 \ub530\ub978 \uc2e0\uccb4\ubc1c\ub2ec\uc744 \uc801\uc6a9\ud558\ub294 8\uccb4\uc9c8\uc758 \uc6b4\ub3d9\uc801 \uae30\ub7c9\uc740 \ub69c\ub837\ud558\uac8c \ub098\ub258\uc5b4 \uc9c4\ub2e4."} {"question": "\uc120\uc218 \uc0dd\ud65c\uc744 \ud558\ub294 \ub3d9\uc548 4\uac1c\uc758 \uba54\uc774\uc800\ub300\ud68c\uc5d0\uc11c \ud55c\ubc88 \uc774\uc0c1\uc744 \uc6b0\uc2b9\ud558\ub294 \uacbd\uc6b0 \ubb34\uc5c7\uc774\ub77c\uace0 \ud558\ub294\uac00?", "paragraph": "\uc120\uc218\uc0dd\ud65c \uc911\uc5d0 4\uac1c\uc758 \uba54\uc774\uc800 \ub300\ud68c\ub97c \ubaa8\ub450 \ud55c \ubc88 \uc774\uc0c1 \uc6b0\uc2b9\ud558\ub294 \uac83\uc744 \ucee4\ub9ac\uc5b4 \uadf8\ub79c\ub4dc \uc2ac\ub7a8(Career Grand Slam)\uc774\ub77c \ud55c\ub2e4(\uc704\uc5d0\uc11c \ud55c \ud574\uc5d0 4\uac1c \uba54\uc774\uc800 \ub300\ud68c\ub97c \ubaa8\ub450 \uc6b0\uc2b9\ud55c \uc120\uc218\ub4e4\ub3c4 \ud574\ub2f9\ub428). \uc9c0\uae08\uae4c\uc9c0 7\uba85\uc758 \ub0a8\uc790 \uc120\uc218\uc640 10\uba85\uc758 \uc5ec\uc790 \uc120\uc218\uac00 \ub2e8\uc2dd\uc5d0\uc11c \ucee4\ub9ac\uc5b4 \uadf8\ub79c\ub4dc \uc2ac\ub7a8\uc744 \ub2ec\uc131\ud558\uc600\ub2e4. \uadf8\ub7ec\ub098 \uc624\ud508 \uc2dc\ub300 \uc774\ud6c4\ub85c \ud55c\uc815\ud558\uba74 \uc624\uc9c1 5 \uba85\uc758 \ub0a8\uc790 \uc120\uc218(\ub85c\ub4dc \ub808\uc774\ubc84, \uc548\ub4dc\ub808 \uc544\uac00\uc2dc, \ub85c\uc800 \ud398\ub354\ub7ec, \ub77c\ud30c\uc5d8 \ub098\ub2ec, \ub178\ubc14\ud06c \uc870\ucf54\ube44\uce58)\uc640 6\uba85\uc758 \uc5ec\uc790 \uc120\uc218(\ub9c8\uac00\ub81b \ucf54\ud2b8, \ud06c\ub9ac\uc2a4 \uc5d0\ubc84\ud2b8, \ub9c8\ub974\ud2f0\ub098 \ub098\ube0c\ub77c\ud2f8\ub85c\ubc14, \uc288\ud14c\ud53c \uadf8\ub77c\ud504, \uc138\ub808\ub098 \uc70c\ub9ac\uc5c4\uc2a4, \ub9c8\ub9ac\uc544 \uc0e4\ub77c\ud3ec\ubc14)\ub9cc\uc774 \uc5ec\uae30\uc5d0 \ud574\ub2f9\ub41c\ub2e4. \uc218\ub9ce\uc740 \uc120\uc218\ub4e4\uc774 \ubd80\uc0c1\uc774\ub098 \ud2b9\uc815 \ucf54\ud2b8\uc5d0 \ub300\ud55c \uc57d\uc810 \ub54c\ubb38\uc5d0 \ucee4\ub9ac\uc5b4 \uadf8\ub79c\ub4dc \uc2ac\ub7a8\uc744 \ub2ec\uc131\ud558\ub294\ub370 \uc2e4\ud328\ud588\ub2e4. \ube44\uc678\ub978 \ubcf4\ub9ac\ub294 US \uc624\ud508\uacfc \ud638\uc8fc \uc624\ud508\uc5d0\uc11c \ud55c \ubc88\ub3c4 \uc6b0\uc2b9\ud558\uc9c0 \ubabb\ud588\ub2e4. \uc874 \uba54\ucf04\ub85c\ub294 \ud638\uc8fc \uc624\ud508\uacfc \ud504\ub791\uc2a4 \uc624\ud508\uc5d0\uc11c \ud55c \ubc88\ub3c4 \uc6b0\uc2b9\ud558\uc9c0 \ubabb\ud588\ub2e4. \ucf04 \ub85c\uc988\uc6d4, \uae30\uc608\ub974\ubaa8 \ube4c\ub77c\uc2a4, \uc774\ubc18 \ub80c\ub4e4, \ubaa8\ub2c8\uce74 \uc140\ub808\uc2a4, \uc950\uc2a4\ud2f4 \uc5d0\ub139 \uadf8\ub9ac\uace0 \ub9e4\uce20 \ube4c\ub79c\ub354\ub294 \uc714\ube14\ub358\uc5d0\uc11c \uc6b0\uc2b9\ud558\uc9c0 \ubabb\ud588\ub2e4. \uc874 \ub274\ucef4, \uc544\uc11c \uc560\uc2dc, \uc9c0\ubbf8 \ucf54\ub108\uc2a4, \ubcf4\ub9ac\uc2a4 \ubca0\ucee4, \uc2a4\ud14c\ud310 \uc5d0\ub4dc\ubca0\ub9ac, \ud53c\ud2b8 \uc0d8\ud504\ub77c\uc2a4, \ub9c8\ub974\ud2f0\ub098 \ud79d\uae30\uc2a4, \ub9b0\uc81c\uc774 \ub370\uc774\ube10\ud3ec\ud2b8\ub294 \ud504\ub791\uc2a4 \uc624\ud508\uc5d0\uc11c \uc6b0\uc2b9\ud558\uc9c0 \ubabb\ud588\ub2e4. \ube44\ub108\uc2a4 \uc70c\ub9ac\uc5c4\uc2a4\ub294 \ud638\uc8fc \uc624\ud508\uacfc \ud504\ub791\uc2a4 \uc624\ud508\uc5d0\uc11c \uc6b0\uc2b9\ud558\uc9c0 \ubabb\ud588\ub2e4. \ub2e4\uc74c\uc740 \uc120\uc218\uc0dd\ud65c \uc911 4\uac1c\uc758 \uadf8\ub79c\ub4dc \uc2ac\ub7a8 \ub300\ud68c\uc5d0\uc11c \ubaa8\ub450 \uc6b0\uc2b9\ud55c \uc120\uc218\ub4e4\uc758 \ubaa9\ub85d\uc774\ub2e4. \ub9e8 \uc55e\uc758 \uc22b\uc790\ub294 \uccab \uadf8\ub79c\ub4dc \uc2ac\ub7a8 \uc6b0\uc2b9 \uc5f0\ub3c4\uc774\uba70, \uadf8 \ub2e4\uc74c\uc5d0 \ub098\uc624\ub294 \uc22b\uc790\ub294 \ucee4\ub9ac\uc5b4 \uadf8\ub79c\ub4dc \uc2ac\ub7a8\uc744 \ub2ec\uc131\ud558\ub294 \ub370 \ud544\uc694\ud55c \ub098\uba38\uc9c0 \ub300\ud68c\ub97c \uc6b0\uc2b9\ud55c \uc5f0\ub3c4\ub4e4\uc774\ub2e4. \ub9e8 \ub4a4 \ube14\ub7ad\ud0b7 \uae30\ud638 \uc548\uc5d0 \ud45c\uae30\ub41c \uc22b\uc790\ub294 \ucee4\ub9ac\uc5b4 \uadf8\ub79c\ub4dc \uc2ac\ub7a8\uc744 \ub2ec\uc131\ud588\uc744 \ub2f9\uc2dc\uc758 \ub098\uc774\ub97c \ub098\ud0c0\ub0b8\ub2e4.", "answer": "\ucee4\ub9ac\uc5b4 \uadf8\ub79c\ub4dc \uc2ac\ub7a8", "sentence": "\uc120\uc218\uc0dd\ud65c \uc911\uc5d0 4\uac1c\uc758 \uba54\uc774\uc800 \ub300\ud68c\ub97c \ubaa8\ub450 \ud55c \ubc88 \uc774\uc0c1 \uc6b0\uc2b9\ud558\ub294 \uac83\uc744 \ucee4\ub9ac\uc5b4 \uadf8\ub79c\ub4dc \uc2ac\ub7a8 (Career Grand Slam)", "paragraph_sentence": " \uc120\uc218\uc0dd\ud65c \uc911\uc5d0 4\uac1c\uc758 \uba54\uc774\uc800 \ub300\ud68c\ub97c \ubaa8\ub450 \ud55c \ubc88 \uc774\uc0c1 \uc6b0\uc2b9\ud558\ub294 \uac83\uc744 \ucee4\ub9ac\uc5b4 \uadf8\ub79c\ub4dc \uc2ac\ub7a8 (Career Grand Slam) \uc774\ub77c \ud55c\ub2e4(\uc704\uc5d0\uc11c \ud55c \ud574\uc5d0 4\uac1c \uba54\uc774\uc800 \ub300\ud68c\ub97c \ubaa8\ub450 \uc6b0\uc2b9\ud55c \uc120\uc218\ub4e4\ub3c4 \ud574\ub2f9\ub428). \uc9c0\uae08\uae4c\uc9c0 7\uba85\uc758 \ub0a8\uc790 \uc120\uc218\uc640 10\uba85\uc758 \uc5ec\uc790 \uc120\uc218\uac00 \ub2e8\uc2dd\uc5d0\uc11c \ucee4\ub9ac\uc5b4 \uadf8\ub79c\ub4dc \uc2ac\ub7a8\uc744 \ub2ec\uc131\ud558\uc600\ub2e4. \uadf8\ub7ec\ub098 \uc624\ud508 \uc2dc\ub300 \uc774\ud6c4\ub85c \ud55c\uc815\ud558\uba74 \uc624\uc9c1 5 \uba85\uc758 \ub0a8\uc790 \uc120\uc218(\ub85c\ub4dc \ub808\uc774\ubc84, \uc548\ub4dc\ub808 \uc544\uac00\uc2dc, \ub85c\uc800 \ud398\ub354\ub7ec, \ub77c\ud30c\uc5d8 \ub098\ub2ec, \ub178\ubc14\ud06c \uc870\ucf54\ube44\uce58)\uc640 6\uba85\uc758 \uc5ec\uc790 \uc120\uc218(\ub9c8\uac00\ub81b \ucf54\ud2b8, \ud06c\ub9ac\uc2a4 \uc5d0\ubc84\ud2b8, \ub9c8\ub974\ud2f0\ub098 \ub098\ube0c\ub77c\ud2f8\ub85c\ubc14, \uc288\ud14c\ud53c \uadf8\ub77c\ud504, \uc138\ub808\ub098 \uc70c\ub9ac\uc5c4\uc2a4, \ub9c8\ub9ac\uc544 \uc0e4\ub77c\ud3ec\ubc14)\ub9cc\uc774 \uc5ec\uae30\uc5d0 \ud574\ub2f9\ub41c\ub2e4. \uc218\ub9ce\uc740 \uc120\uc218\ub4e4\uc774 \ubd80\uc0c1\uc774\ub098 \ud2b9\uc815 \ucf54\ud2b8\uc5d0 \ub300\ud55c \uc57d\uc810 \ub54c\ubb38\uc5d0 \ucee4\ub9ac\uc5b4 \uadf8\ub79c\ub4dc \uc2ac\ub7a8\uc744 \ub2ec\uc131\ud558\ub294\ub370 \uc2e4\ud328\ud588\ub2e4. \ube44\uc678\ub978 \ubcf4\ub9ac\ub294 US \uc624\ud508\uacfc \ud638\uc8fc \uc624\ud508\uc5d0\uc11c \ud55c \ubc88\ub3c4 \uc6b0\uc2b9\ud558\uc9c0 \ubabb\ud588\ub2e4. \uc874 \uba54\ucf04\ub85c\ub294 \ud638\uc8fc \uc624\ud508\uacfc \ud504\ub791\uc2a4 \uc624\ud508\uc5d0\uc11c \ud55c \ubc88\ub3c4 \uc6b0\uc2b9\ud558\uc9c0 \ubabb\ud588\ub2e4. \ucf04 \ub85c\uc988\uc6d4, \uae30\uc608\ub974\ubaa8 \ube4c\ub77c\uc2a4, \uc774\ubc18 \ub80c\ub4e4, \ubaa8\ub2c8\uce74 \uc140\ub808\uc2a4, \uc950\uc2a4\ud2f4 \uc5d0\ub139 \uadf8\ub9ac\uace0 \ub9e4\uce20 \ube4c\ub79c\ub354\ub294 \uc714\ube14\ub358\uc5d0\uc11c \uc6b0\uc2b9\ud558\uc9c0 \ubabb\ud588\ub2e4. \uc874 \ub274\ucef4, \uc544\uc11c \uc560\uc2dc, \uc9c0\ubbf8 \ucf54\ub108\uc2a4, \ubcf4\ub9ac\uc2a4 \ubca0\ucee4, \uc2a4\ud14c\ud310 \uc5d0\ub4dc\ubca0\ub9ac, \ud53c\ud2b8 \uc0d8\ud504\ub77c\uc2a4, \ub9c8\ub974\ud2f0\ub098 \ud79d\uae30\uc2a4, \ub9b0\uc81c\uc774 \ub370\uc774\ube10\ud3ec\ud2b8\ub294 \ud504\ub791\uc2a4 \uc624\ud508\uc5d0\uc11c \uc6b0\uc2b9\ud558\uc9c0 \ubabb\ud588\ub2e4. \ube44\ub108\uc2a4 \uc70c\ub9ac\uc5c4\uc2a4\ub294 \ud638\uc8fc \uc624\ud508\uacfc \ud504\ub791\uc2a4 \uc624\ud508\uc5d0\uc11c \uc6b0\uc2b9\ud558\uc9c0 \ubabb\ud588\ub2e4. \ub2e4\uc74c\uc740 \uc120\uc218\uc0dd\ud65c \uc911 4\uac1c\uc758 \uadf8\ub79c\ub4dc \uc2ac\ub7a8 \ub300\ud68c\uc5d0\uc11c \ubaa8\ub450 \uc6b0\uc2b9\ud55c \uc120\uc218\ub4e4\uc758 \ubaa9\ub85d\uc774\ub2e4. \ub9e8 \uc55e\uc758 \uc22b\uc790\ub294 \uccab \uadf8\ub79c\ub4dc \uc2ac\ub7a8 \uc6b0\uc2b9 \uc5f0\ub3c4\uc774\uba70, \uadf8 \ub2e4\uc74c\uc5d0 \ub098\uc624\ub294 \uc22b\uc790\ub294 \ucee4\ub9ac\uc5b4 \uadf8\ub79c\ub4dc \uc2ac\ub7a8\uc744 \ub2ec\uc131\ud558\ub294 \ub370 \ud544\uc694\ud55c \ub098\uba38\uc9c0 \ub300\ud68c\ub97c \uc6b0\uc2b9\ud55c \uc5f0\ub3c4\ub4e4\uc774\ub2e4. \ub9e8 \ub4a4 \ube14\ub7ad\ud0b7 \uae30\ud638 \uc548\uc5d0 \ud45c\uae30\ub41c \uc22b\uc790\ub294 \ucee4\ub9ac\uc5b4 \uadf8\ub79c\ub4dc \uc2ac\ub7a8\uc744 \ub2ec\uc131\ud588\uc744 \ub2f9\uc2dc\uc758 \ub098\uc774\ub97c \ub098\ud0c0\ub0b8\ub2e4.", "paragraph_answer": "\uc120\uc218\uc0dd\ud65c \uc911\uc5d0 4\uac1c\uc758 \uba54\uc774\uc800 \ub300\ud68c\ub97c \ubaa8\ub450 \ud55c \ubc88 \uc774\uc0c1 \uc6b0\uc2b9\ud558\ub294 \uac83\uc744 \ucee4\ub9ac\uc5b4 \uadf8\ub79c\ub4dc \uc2ac\ub7a8 (Career Grand Slam)\uc774\ub77c \ud55c\ub2e4(\uc704\uc5d0\uc11c \ud55c \ud574\uc5d0 4\uac1c \uba54\uc774\uc800 \ub300\ud68c\ub97c \ubaa8\ub450 \uc6b0\uc2b9\ud55c \uc120\uc218\ub4e4\ub3c4 \ud574\ub2f9\ub428). \uc9c0\uae08\uae4c\uc9c0 7\uba85\uc758 \ub0a8\uc790 \uc120\uc218\uc640 10\uba85\uc758 \uc5ec\uc790 \uc120\uc218\uac00 \ub2e8\uc2dd\uc5d0\uc11c \ucee4\ub9ac\uc5b4 \uadf8\ub79c\ub4dc \uc2ac\ub7a8\uc744 \ub2ec\uc131\ud558\uc600\ub2e4. \uadf8\ub7ec\ub098 \uc624\ud508 \uc2dc\ub300 \uc774\ud6c4\ub85c \ud55c\uc815\ud558\uba74 \uc624\uc9c1 5 \uba85\uc758 \ub0a8\uc790 \uc120\uc218(\ub85c\ub4dc \ub808\uc774\ubc84, \uc548\ub4dc\ub808 \uc544\uac00\uc2dc, \ub85c\uc800 \ud398\ub354\ub7ec, \ub77c\ud30c\uc5d8 \ub098\ub2ec, \ub178\ubc14\ud06c \uc870\ucf54\ube44\uce58)\uc640 6\uba85\uc758 \uc5ec\uc790 \uc120\uc218(\ub9c8\uac00\ub81b \ucf54\ud2b8, \ud06c\ub9ac\uc2a4 \uc5d0\ubc84\ud2b8, \ub9c8\ub974\ud2f0\ub098 \ub098\ube0c\ub77c\ud2f8\ub85c\ubc14, \uc288\ud14c\ud53c \uadf8\ub77c\ud504, \uc138\ub808\ub098 \uc70c\ub9ac\uc5c4\uc2a4, \ub9c8\ub9ac\uc544 \uc0e4\ub77c\ud3ec\ubc14)\ub9cc\uc774 \uc5ec\uae30\uc5d0 \ud574\ub2f9\ub41c\ub2e4. \uc218\ub9ce\uc740 \uc120\uc218\ub4e4\uc774 \ubd80\uc0c1\uc774\ub098 \ud2b9\uc815 \ucf54\ud2b8\uc5d0 \ub300\ud55c \uc57d\uc810 \ub54c\ubb38\uc5d0 \ucee4\ub9ac\uc5b4 \uadf8\ub79c\ub4dc \uc2ac\ub7a8\uc744 \ub2ec\uc131\ud558\ub294\ub370 \uc2e4\ud328\ud588\ub2e4. \ube44\uc678\ub978 \ubcf4\ub9ac\ub294 US \uc624\ud508\uacfc \ud638\uc8fc \uc624\ud508\uc5d0\uc11c \ud55c \ubc88\ub3c4 \uc6b0\uc2b9\ud558\uc9c0 \ubabb\ud588\ub2e4. \uc874 \uba54\ucf04\ub85c\ub294 \ud638\uc8fc \uc624\ud508\uacfc \ud504\ub791\uc2a4 \uc624\ud508\uc5d0\uc11c \ud55c \ubc88\ub3c4 \uc6b0\uc2b9\ud558\uc9c0 \ubabb\ud588\ub2e4. \ucf04 \ub85c\uc988\uc6d4, \uae30\uc608\ub974\ubaa8 \ube4c\ub77c\uc2a4, \uc774\ubc18 \ub80c\ub4e4, \ubaa8\ub2c8\uce74 \uc140\ub808\uc2a4, \uc950\uc2a4\ud2f4 \uc5d0\ub139 \uadf8\ub9ac\uace0 \ub9e4\uce20 \ube4c\ub79c\ub354\ub294 \uc714\ube14\ub358\uc5d0\uc11c \uc6b0\uc2b9\ud558\uc9c0 \ubabb\ud588\ub2e4. \uc874 \ub274\ucef4, \uc544\uc11c \uc560\uc2dc, \uc9c0\ubbf8 \ucf54\ub108\uc2a4, \ubcf4\ub9ac\uc2a4 \ubca0\ucee4, \uc2a4\ud14c\ud310 \uc5d0\ub4dc\ubca0\ub9ac, \ud53c\ud2b8 \uc0d8\ud504\ub77c\uc2a4, \ub9c8\ub974\ud2f0\ub098 \ud79d\uae30\uc2a4, \ub9b0\uc81c\uc774 \ub370\uc774\ube10\ud3ec\ud2b8\ub294 \ud504\ub791\uc2a4 \uc624\ud508\uc5d0\uc11c \uc6b0\uc2b9\ud558\uc9c0 \ubabb\ud588\ub2e4. \ube44\ub108\uc2a4 \uc70c\ub9ac\uc5c4\uc2a4\ub294 \ud638\uc8fc \uc624\ud508\uacfc \ud504\ub791\uc2a4 \uc624\ud508\uc5d0\uc11c \uc6b0\uc2b9\ud558\uc9c0 \ubabb\ud588\ub2e4. \ub2e4\uc74c\uc740 \uc120\uc218\uc0dd\ud65c \uc911 4\uac1c\uc758 \uadf8\ub79c\ub4dc \uc2ac\ub7a8 \ub300\ud68c\uc5d0\uc11c \ubaa8\ub450 \uc6b0\uc2b9\ud55c \uc120\uc218\ub4e4\uc758 \ubaa9\ub85d\uc774\ub2e4. \ub9e8 \uc55e\uc758 \uc22b\uc790\ub294 \uccab \uadf8\ub79c\ub4dc \uc2ac\ub7a8 \uc6b0\uc2b9 \uc5f0\ub3c4\uc774\uba70, \uadf8 \ub2e4\uc74c\uc5d0 \ub098\uc624\ub294 \uc22b\uc790\ub294 \ucee4\ub9ac\uc5b4 \uadf8\ub79c\ub4dc \uc2ac\ub7a8\uc744 \ub2ec\uc131\ud558\ub294 \ub370 \ud544\uc694\ud55c \ub098\uba38\uc9c0 \ub300\ud68c\ub97c \uc6b0\uc2b9\ud55c \uc5f0\ub3c4\ub4e4\uc774\ub2e4. \ub9e8 \ub4a4 \ube14\ub7ad\ud0b7 \uae30\ud638 \uc548\uc5d0 \ud45c\uae30\ub41c \uc22b\uc790\ub294 \ucee4\ub9ac\uc5b4 \uadf8\ub79c\ub4dc \uc2ac\ub7a8\uc744 \ub2ec\uc131\ud588\uc744 \ub2f9\uc2dc\uc758 \ub098\uc774\ub97c \ub098\ud0c0\ub0b8\ub2e4.", "sentence_answer": "\uc120\uc218\uc0dd\ud65c \uc911\uc5d0 4\uac1c\uc758 \uba54\uc774\uc800 \ub300\ud68c\ub97c \ubaa8\ub450 \ud55c \ubc88 \uc774\uc0c1 \uc6b0\uc2b9\ud558\ub294 \uac83\uc744 \ucee4\ub9ac\uc5b4 \uadf8\ub79c\ub4dc \uc2ac\ub7a8 (Career Grand Slam)", "paragraph_id": "5974268-2-0", "paragraph_question": "question: \uc120\uc218 \uc0dd\ud65c\uc744 \ud558\ub294 \ub3d9\uc548 4\uac1c\uc758 \uba54\uc774\uc800\ub300\ud68c\uc5d0\uc11c \ud55c\ubc88 \uc774\uc0c1\uc744 \uc6b0\uc2b9\ud558\ub294 \uacbd\uc6b0 \ubb34\uc5c7\uc774\ub77c\uace0 \ud558\ub294\uac00?, context: \uc120\uc218\uc0dd\ud65c \uc911\uc5d0 4\uac1c\uc758 \uba54\uc774\uc800 \ub300\ud68c\ub97c \ubaa8\ub450 \ud55c \ubc88 \uc774\uc0c1 \uc6b0\uc2b9\ud558\ub294 \uac83\uc744 \ucee4\ub9ac\uc5b4 \uadf8\ub79c\ub4dc \uc2ac\ub7a8(Career Grand Slam)\uc774\ub77c \ud55c\ub2e4(\uc704\uc5d0\uc11c \ud55c \ud574\uc5d0 4\uac1c \uba54\uc774\uc800 \ub300\ud68c\ub97c \ubaa8\ub450 \uc6b0\uc2b9\ud55c \uc120\uc218\ub4e4\ub3c4 \ud574\ub2f9\ub428). \uc9c0\uae08\uae4c\uc9c0 7\uba85\uc758 \ub0a8\uc790 \uc120\uc218\uc640 10\uba85\uc758 \uc5ec\uc790 \uc120\uc218\uac00 \ub2e8\uc2dd\uc5d0\uc11c \ucee4\ub9ac\uc5b4 \uadf8\ub79c\ub4dc \uc2ac\ub7a8\uc744 \ub2ec\uc131\ud558\uc600\ub2e4. \uadf8\ub7ec\ub098 \uc624\ud508 \uc2dc\ub300 \uc774\ud6c4\ub85c \ud55c\uc815\ud558\uba74 \uc624\uc9c1 5 \uba85\uc758 \ub0a8\uc790 \uc120\uc218(\ub85c\ub4dc \ub808\uc774\ubc84, \uc548\ub4dc\ub808 \uc544\uac00\uc2dc, \ub85c\uc800 \ud398\ub354\ub7ec, \ub77c\ud30c\uc5d8 \ub098\ub2ec, \ub178\ubc14\ud06c \uc870\ucf54\ube44\uce58)\uc640 6\uba85\uc758 \uc5ec\uc790 \uc120\uc218(\ub9c8\uac00\ub81b \ucf54\ud2b8, \ud06c\ub9ac\uc2a4 \uc5d0\ubc84\ud2b8, \ub9c8\ub974\ud2f0\ub098 \ub098\ube0c\ub77c\ud2f8\ub85c\ubc14, \uc288\ud14c\ud53c \uadf8\ub77c\ud504, \uc138\ub808\ub098 \uc70c\ub9ac\uc5c4\uc2a4, \ub9c8\ub9ac\uc544 \uc0e4\ub77c\ud3ec\ubc14)\ub9cc\uc774 \uc5ec\uae30\uc5d0 \ud574\ub2f9\ub41c\ub2e4. \uc218\ub9ce\uc740 \uc120\uc218\ub4e4\uc774 \ubd80\uc0c1\uc774\ub098 \ud2b9\uc815 \ucf54\ud2b8\uc5d0 \ub300\ud55c \uc57d\uc810 \ub54c\ubb38\uc5d0 \ucee4\ub9ac\uc5b4 \uadf8\ub79c\ub4dc \uc2ac\ub7a8\uc744 \ub2ec\uc131\ud558\ub294\ub370 \uc2e4\ud328\ud588\ub2e4. \ube44\uc678\ub978 \ubcf4\ub9ac\ub294 US \uc624\ud508\uacfc \ud638\uc8fc \uc624\ud508\uc5d0\uc11c \ud55c \ubc88\ub3c4 \uc6b0\uc2b9\ud558\uc9c0 \ubabb\ud588\ub2e4. \uc874 \uba54\ucf04\ub85c\ub294 \ud638\uc8fc \uc624\ud508\uacfc \ud504\ub791\uc2a4 \uc624\ud508\uc5d0\uc11c \ud55c \ubc88\ub3c4 \uc6b0\uc2b9\ud558\uc9c0 \ubabb\ud588\ub2e4. \ucf04 \ub85c\uc988\uc6d4, \uae30\uc608\ub974\ubaa8 \ube4c\ub77c\uc2a4, \uc774\ubc18 \ub80c\ub4e4, \ubaa8\ub2c8\uce74 \uc140\ub808\uc2a4, \uc950\uc2a4\ud2f4 \uc5d0\ub139 \uadf8\ub9ac\uace0 \ub9e4\uce20 \ube4c\ub79c\ub354\ub294 \uc714\ube14\ub358\uc5d0\uc11c \uc6b0\uc2b9\ud558\uc9c0 \ubabb\ud588\ub2e4. \uc874 \ub274\ucef4, \uc544\uc11c \uc560\uc2dc, \uc9c0\ubbf8 \ucf54\ub108\uc2a4, \ubcf4\ub9ac\uc2a4 \ubca0\ucee4, \uc2a4\ud14c\ud310 \uc5d0\ub4dc\ubca0\ub9ac, \ud53c\ud2b8 \uc0d8\ud504\ub77c\uc2a4, \ub9c8\ub974\ud2f0\ub098 \ud79d\uae30\uc2a4, \ub9b0\uc81c\uc774 \ub370\uc774\ube10\ud3ec\ud2b8\ub294 \ud504\ub791\uc2a4 \uc624\ud508\uc5d0\uc11c \uc6b0\uc2b9\ud558\uc9c0 \ubabb\ud588\ub2e4. \ube44\ub108\uc2a4 \uc70c\ub9ac\uc5c4\uc2a4\ub294 \ud638\uc8fc \uc624\ud508\uacfc \ud504\ub791\uc2a4 \uc624\ud508\uc5d0\uc11c \uc6b0\uc2b9\ud558\uc9c0 \ubabb\ud588\ub2e4. \ub2e4\uc74c\uc740 \uc120\uc218\uc0dd\ud65c \uc911 4\uac1c\uc758 \uadf8\ub79c\ub4dc \uc2ac\ub7a8 \ub300\ud68c\uc5d0\uc11c \ubaa8\ub450 \uc6b0\uc2b9\ud55c \uc120\uc218\ub4e4\uc758 \ubaa9\ub85d\uc774\ub2e4. \ub9e8 \uc55e\uc758 \uc22b\uc790\ub294 \uccab \uadf8\ub79c\ub4dc \uc2ac\ub7a8 \uc6b0\uc2b9 \uc5f0\ub3c4\uc774\uba70, \uadf8 \ub2e4\uc74c\uc5d0 \ub098\uc624\ub294 \uc22b\uc790\ub294 \ucee4\ub9ac\uc5b4 \uadf8\ub79c\ub4dc \uc2ac\ub7a8\uc744 \ub2ec\uc131\ud558\ub294 \ub370 \ud544\uc694\ud55c \ub098\uba38\uc9c0 \ub300\ud68c\ub97c \uc6b0\uc2b9\ud55c \uc5f0\ub3c4\ub4e4\uc774\ub2e4. \ub9e8 \ub4a4 \ube14\ub7ad\ud0b7 \uae30\ud638 \uc548\uc5d0 \ud45c\uae30\ub41c \uc22b\uc790\ub294 \ucee4\ub9ac\uc5b4 \uadf8\ub79c\ub4dc \uc2ac\ub7a8\uc744 \ub2ec\uc131\ud588\uc744 \ub2f9\uc2dc\uc758 \ub098\uc774\ub97c \ub098\ud0c0\ub0b8\ub2e4."} {"question": "\uc870\uc9c0\uc544 \uacf5\ud654\uad6d\uc774\ub77c\ub294 \uc0c8\ub85c\uc6b4 \ub3c5\ub9bd\uad6d\uc774 \ub41c \ub144\ub3c4\ub294?", "paragraph": "\uc18c\ub828 \uc2dc\uc808, \uc870\uc9c0\uc544\uc758 \ubbfc\uc871\uc6b4\ub3d9\uc740 \uc5b5\uc555\ub418\uc5c8\uc73c\ub098, \ud55c\ud3b8\uc73c\ub85c\ub294 \uc18c\ub828\uc758 \uc911\uc559 \uc815\ubd80\uc640 \ud0c0\ud611\ud558\uc5ec \uc18c\ub828 \uc911\uc559 \uc815\uce58\ubb34\ub300\uc5d0 \uace0\uc704\uc9c1\uc73c\ub85c \ub4f1\uc7a5\ud558\ub294 \uc870\uc9c0\uc544 \ucd9c\uc2e0 \uc778\ubb3c\ub3c4 \uc788\uc5c8\ub294\ub370, \uadf8\ub8e8\uc9c0\uc57c \uacf5\uc0b0\ub2f9 \uc81c1\uc11c\uae30\ub97c \uc9c0\ub0b4\ub2e4\uac00 1985\ub144 \uace0\ub974\ubc14\ucd08\ud504 \uc815\uad8c \ucd9c\ubc94 \ud6c4 \uc18c\ub828 \uc678\ubb34\uc7a5\uad00\uc774 \ub41c \uc5d0\ub450\uc544\ub974\ub4dc \uc170\ubc14\ub974\ub4dc\ub098\uc81c\uac00 \ub300\ud45c\uc801\uc774\ub2e4. 1970\ub144\ub300\uc5d0 \uc18c\ub828\uc5d0 \ubc18\ubc1c\ud558\ub294 \uc6c0\uc9c1\uc784\uc774 \uc788\uc5b4\uc654\uace0, \ub3d9\uc2dc\uc5d0 \uacf5\ud654\uad6d \ub0b4\uc758 \uc18c\uc218\ubbfc\uc871\uc778 \uc555\ud558\uc2a4\ub098 \ub0a8\uc624\uc138\ud2f0\uc57c\uc640\uc758 \uac08\ub4f1\ub3c4 \ub5a0\uc624\ub974\uae30 \uc2dc\uc791\ud588\ub2e4. \uc774\ub7ec\ud55c \ubbfc\uc871 \ub300\ub9bd\uc740 1980\ub144\ub300 \uace0\ub974\ubc14\ucd08\ud504 \uc2dc\ub300 \uc774\ud6c4 \ub354\uc6b1 \uc2ec\ud574\uc84c\uc73c\uba70 \uc870\uc9c0\uc544\uc778\ub4e4\uc740 \uc870\uc9c0\uc544\uc758 \uc18c\ub828 \ub3c5\ub9bd\uc744 \uc2dc\ub3c4\ud558\uace0 \ub3d9\uc2dc\uc5d0 \uc555\ud558\uc2a4\uc758 \ubd84\ub9ac \ub3c5\ub9bd \uc6b4\ub3d9\uc744 \uc800\uc9c0\ud558\ub824\uace0 \ud588\ub2e4. \ubcf5\uc7a1\ud55c \uc815\uad6d \uc18d\uc5d0 1989\ub144 \ud2b8\ube4c\ub9ac\uc2dc\uc5d0\uc11c \uc2dc\uc704 \uc911\uc774\ub358 \uad70\uc911\uc5d0\uac8c \uc18c\ub828\uad70\uc774 \ubc1c\ud3ec\ud558\uc5ec \uc218\uc2ed\uba85\uc774 \uc0ac\ub9dd\ud558\ub294 \uc0ac\uac74\uc774 \uc77c\uc5b4\ub098 \ub3c5\ub9bd \uc6b4\ub3d9\uc740 \ub354\uc6b1 \uac00\uc18d\ud654\ub418\uc5c8\ub2e4. \uac1c\ud601, \uac1c\ubc29 \uc5f4\ud48d \uc18d\uc5d0 1990\ub144 \uc870\uc9c0\uc544 \uc5ed\uc0ac\uc0c1 \ucd5c\ucd08\uc758 \ub2e4\ub2f9\uc81c \uc120\uac70\uac00 \uc2e4\uc2dc\ub410\uace0 \uce5c\ub7ec\uc2dc\uc544 \uc131\ud5a5\uc758 \uc555\ud558\uc9c0\uc57c\uc640 \ub0a8\uc624\uc138\ud2f0\uc57c \uc9c0\uc5ed\uc5d0\uc11c\ub294 \uc18c\ub828 \uc794\ub958\ub97c \uc6d0\ud558\uba70 \ub3c5\ub9bd \ub2e4\ud23c\uc774 \uc77c\uc5b4\ub0ac\uc73c\ub098 \uc870\uc9c0\uc544\ub294 \uc18c\ub828 \ud574\uccb4\uc5d0 \uc55e\uc11c 1991\ub144 4\uc6d4 9\uc77c \ub3c5\ub9bd\uc744 \uc120\uc5b8\ud588\ub2e4. \uc18c\ub828 \ud574\uccb4\uac00 \ud655\uc815\ub418\uace0 1991\ub144 12\uc6d4 25\uc77c \uc870\uc9c0\uc544\ub294 \uc644\uc804\ud788 \ub3c5\ub9bd\ud558\uc5ec \uc870\uc9c0\uc544 \uacf5\ud654\uad6d\uc774\ub77c\ub294 \uc0c8\ub85c\uc6b4 \ub3c5\ub9bd\uad6d\uc774 \ub418\uc5c8\uc73c\ub098, \uad6d\ub0b4\uc758 \uc815\ud30c \ub300\ub9bd\uacfc \ubbfc\uc871 \ubb38\uc81c\ub85c 1992\ub144 ~ 1994\ub144 \ub0b4\uc804\uc5d0 \ub3cc\uc785\ud558\uc5ec \uc0ac\uc2e4\uc0c1 \ubb34\uc815\ubd80 \uc0c1\ud0dc\uc5d0 \ube60\uc84c\ub2e4.", "answer": "1991\ub144", "sentence": "\uac1c\ud601, \uac1c\ubc29 \uc5f4\ud48d \uc18d\uc5d0 1990\ub144 \uc870\uc9c0\uc544 \uc5ed\uc0ac\uc0c1 \ucd5c\ucd08\uc758 \ub2e4\ub2f9\uc81c \uc120\uac70\uac00 \uc2e4\uc2dc\ub410\uace0 \uce5c\ub7ec\uc2dc\uc544 \uc131\ud5a5\uc758 \uc555\ud558\uc9c0\uc57c\uc640 \ub0a8\uc624\uc138\ud2f0\uc57c \uc9c0\uc5ed\uc5d0\uc11c\ub294 \uc18c\ub828 \uc794\ub958\ub97c \uc6d0\ud558\uba70 \ub3c5\ub9bd \ub2e4\ud23c\uc774 \uc77c\uc5b4\ub0ac\uc73c\ub098 \uc870\uc9c0\uc544\ub294 \uc18c\ub828 \ud574\uccb4\uc5d0 \uc55e\uc11c 1991\ub144 4\uc6d4 9\uc77c \ub3c5\ub9bd\uc744 \uc120\uc5b8\ud588\ub2e4.", "paragraph_sentence": "\uc18c\ub828 \uc2dc\uc808, \uc870\uc9c0\uc544\uc758 \ubbfc\uc871\uc6b4\ub3d9\uc740 \uc5b5\uc555\ub418\uc5c8\uc73c\ub098, \ud55c\ud3b8\uc73c\ub85c\ub294 \uc18c\ub828\uc758 \uc911\uc559 \uc815\ubd80\uc640 \ud0c0\ud611\ud558\uc5ec \uc18c\ub828 \uc911\uc559 \uc815\uce58\ubb34\ub300\uc5d0 \uace0\uc704\uc9c1\uc73c\ub85c \ub4f1\uc7a5\ud558\ub294 \uc870\uc9c0\uc544 \ucd9c\uc2e0 \uc778\ubb3c\ub3c4 \uc788\uc5c8\ub294\ub370, \uadf8\ub8e8\uc9c0\uc57c \uacf5\uc0b0\ub2f9 \uc81c1\uc11c\uae30\ub97c \uc9c0\ub0b4\ub2e4\uac00 1985\ub144 \uace0\ub974\ubc14\ucd08\ud504 \uc815\uad8c \ucd9c\ubc94 \ud6c4 \uc18c\ub828 \uc678\ubb34\uc7a5\uad00\uc774 \ub41c \uc5d0\ub450\uc544\ub974\ub4dc \uc170\ubc14\ub974\ub4dc\ub098\uc81c\uac00 \ub300\ud45c\uc801\uc774\ub2e4. 1970\ub144\ub300\uc5d0 \uc18c\ub828\uc5d0 \ubc18\ubc1c\ud558\ub294 \uc6c0\uc9c1\uc784\uc774 \uc788\uc5b4\uc654\uace0, \ub3d9\uc2dc\uc5d0 \uacf5\ud654\uad6d \ub0b4\uc758 \uc18c\uc218\ubbfc\uc871\uc778 \uc555\ud558\uc2a4\ub098 \ub0a8\uc624\uc138\ud2f0\uc57c\uc640\uc758 \uac08\ub4f1\ub3c4 \ub5a0\uc624\ub974\uae30 \uc2dc\uc791\ud588\ub2e4. \uc774\ub7ec\ud55c \ubbfc\uc871 \ub300\ub9bd\uc740 1980\ub144\ub300 \uace0\ub974\ubc14\ucd08\ud504 \uc2dc\ub300 \uc774\ud6c4 \ub354\uc6b1 \uc2ec\ud574\uc84c\uc73c\uba70 \uc870\uc9c0\uc544\uc778\ub4e4\uc740 \uc870\uc9c0\uc544\uc758 \uc18c\ub828 \ub3c5\ub9bd\uc744 \uc2dc\ub3c4\ud558\uace0 \ub3d9\uc2dc\uc5d0 \uc555\ud558\uc2a4\uc758 \ubd84\ub9ac \ub3c5\ub9bd \uc6b4\ub3d9\uc744 \uc800\uc9c0\ud558\ub824\uace0 \ud588\ub2e4. \ubcf5\uc7a1\ud55c \uc815\uad6d \uc18d\uc5d0 1989\ub144 \ud2b8\ube4c\ub9ac\uc2dc\uc5d0\uc11c \uc2dc\uc704 \uc911\uc774\ub358 \uad70\uc911\uc5d0\uac8c \uc18c\ub828\uad70\uc774 \ubc1c\ud3ec\ud558\uc5ec \uc218\uc2ed\uba85\uc774 \uc0ac\ub9dd\ud558\ub294 \uc0ac\uac74\uc774 \uc77c\uc5b4\ub098 \ub3c5\ub9bd \uc6b4\ub3d9\uc740 \ub354\uc6b1 \uac00\uc18d\ud654\ub418\uc5c8\ub2e4. \uac1c\ud601, \uac1c\ubc29 \uc5f4\ud48d \uc18d\uc5d0 1990\ub144 \uc870\uc9c0\uc544 \uc5ed\uc0ac\uc0c1 \ucd5c\ucd08\uc758 \ub2e4\ub2f9\uc81c \uc120\uac70\uac00 \uc2e4\uc2dc\ub410\uace0 \uce5c\ub7ec\uc2dc\uc544 \uc131\ud5a5\uc758 \uc555\ud558\uc9c0\uc57c\uc640 \ub0a8\uc624\uc138\ud2f0\uc57c \uc9c0\uc5ed\uc5d0\uc11c\ub294 \uc18c\ub828 \uc794\ub958\ub97c \uc6d0\ud558\uba70 \ub3c5\ub9bd \ub2e4\ud23c\uc774 \uc77c\uc5b4\ub0ac\uc73c\ub098 \uc870\uc9c0\uc544\ub294 \uc18c\ub828 \ud574\uccb4\uc5d0 \uc55e\uc11c 1991\ub144 4\uc6d4 9\uc77c \ub3c5\ub9bd\uc744 \uc120\uc5b8\ud588\ub2e4. \uc18c\ub828 \ud574\uccb4\uac00 \ud655\uc815\ub418\uace0 1991\ub144 12\uc6d4 25\uc77c \uc870\uc9c0\uc544\ub294 \uc644\uc804\ud788 \ub3c5\ub9bd\ud558\uc5ec \uc870\uc9c0\uc544 \uacf5\ud654\uad6d\uc774\ub77c\ub294 \uc0c8\ub85c\uc6b4 \ub3c5\ub9bd\uad6d\uc774 \ub418\uc5c8\uc73c\ub098, \uad6d\ub0b4\uc758 \uc815\ud30c \ub300\ub9bd\uacfc \ubbfc\uc871 \ubb38\uc81c\ub85c 1992\ub144 ~ 1994\ub144 \ub0b4\uc804\uc5d0 \ub3cc\uc785\ud558\uc5ec \uc0ac\uc2e4\uc0c1 \ubb34\uc815\ubd80 \uc0c1\ud0dc\uc5d0 \ube60\uc84c\ub2e4.", "paragraph_answer": "\uc18c\ub828 \uc2dc\uc808, \uc870\uc9c0\uc544\uc758 \ubbfc\uc871\uc6b4\ub3d9\uc740 \uc5b5\uc555\ub418\uc5c8\uc73c\ub098, \ud55c\ud3b8\uc73c\ub85c\ub294 \uc18c\ub828\uc758 \uc911\uc559 \uc815\ubd80\uc640 \ud0c0\ud611\ud558\uc5ec \uc18c\ub828 \uc911\uc559 \uc815\uce58\ubb34\ub300\uc5d0 \uace0\uc704\uc9c1\uc73c\ub85c \ub4f1\uc7a5\ud558\ub294 \uc870\uc9c0\uc544 \ucd9c\uc2e0 \uc778\ubb3c\ub3c4 \uc788\uc5c8\ub294\ub370, \uadf8\ub8e8\uc9c0\uc57c \uacf5\uc0b0\ub2f9 \uc81c1\uc11c\uae30\ub97c \uc9c0\ub0b4\ub2e4\uac00 1985\ub144 \uace0\ub974\ubc14\ucd08\ud504 \uc815\uad8c \ucd9c\ubc94 \ud6c4 \uc18c\ub828 \uc678\ubb34\uc7a5\uad00\uc774 \ub41c \uc5d0\ub450\uc544\ub974\ub4dc \uc170\ubc14\ub974\ub4dc\ub098\uc81c\uac00 \ub300\ud45c\uc801\uc774\ub2e4. 1970\ub144\ub300\uc5d0 \uc18c\ub828\uc5d0 \ubc18\ubc1c\ud558\ub294 \uc6c0\uc9c1\uc784\uc774 \uc788\uc5b4\uc654\uace0, \ub3d9\uc2dc\uc5d0 \uacf5\ud654\uad6d \ub0b4\uc758 \uc18c\uc218\ubbfc\uc871\uc778 \uc555\ud558\uc2a4\ub098 \ub0a8\uc624\uc138\ud2f0\uc57c\uc640\uc758 \uac08\ub4f1\ub3c4 \ub5a0\uc624\ub974\uae30 \uc2dc\uc791\ud588\ub2e4. \uc774\ub7ec\ud55c \ubbfc\uc871 \ub300\ub9bd\uc740 1980\ub144\ub300 \uace0\ub974\ubc14\ucd08\ud504 \uc2dc\ub300 \uc774\ud6c4 \ub354\uc6b1 \uc2ec\ud574\uc84c\uc73c\uba70 \uc870\uc9c0\uc544\uc778\ub4e4\uc740 \uc870\uc9c0\uc544\uc758 \uc18c\ub828 \ub3c5\ub9bd\uc744 \uc2dc\ub3c4\ud558\uace0 \ub3d9\uc2dc\uc5d0 \uc555\ud558\uc2a4\uc758 \ubd84\ub9ac \ub3c5\ub9bd \uc6b4\ub3d9\uc744 \uc800\uc9c0\ud558\ub824\uace0 \ud588\ub2e4. \ubcf5\uc7a1\ud55c \uc815\uad6d \uc18d\uc5d0 1989\ub144 \ud2b8\ube4c\ub9ac\uc2dc\uc5d0\uc11c \uc2dc\uc704 \uc911\uc774\ub358 \uad70\uc911\uc5d0\uac8c \uc18c\ub828\uad70\uc774 \ubc1c\ud3ec\ud558\uc5ec \uc218\uc2ed\uba85\uc774 \uc0ac\ub9dd\ud558\ub294 \uc0ac\uac74\uc774 \uc77c\uc5b4\ub098 \ub3c5\ub9bd \uc6b4\ub3d9\uc740 \ub354\uc6b1 \uac00\uc18d\ud654\ub418\uc5c8\ub2e4. \uac1c\ud601, \uac1c\ubc29 \uc5f4\ud48d \uc18d\uc5d0 1990\ub144 \uc870\uc9c0\uc544 \uc5ed\uc0ac\uc0c1 \ucd5c\ucd08\uc758 \ub2e4\ub2f9\uc81c \uc120\uac70\uac00 \uc2e4\uc2dc\ub410\uace0 \uce5c\ub7ec\uc2dc\uc544 \uc131\ud5a5\uc758 \uc555\ud558\uc9c0\uc57c\uc640 \ub0a8\uc624\uc138\ud2f0\uc57c \uc9c0\uc5ed\uc5d0\uc11c\ub294 \uc18c\ub828 \uc794\ub958\ub97c \uc6d0\ud558\uba70 \ub3c5\ub9bd \ub2e4\ud23c\uc774 \uc77c\uc5b4\ub0ac\uc73c\ub098 \uc870\uc9c0\uc544\ub294 \uc18c\ub828 \ud574\uccb4\uc5d0 \uc55e\uc11c 1991\ub144 4\uc6d4 9\uc77c \ub3c5\ub9bd\uc744 \uc120\uc5b8\ud588\ub2e4. \uc18c\ub828 \ud574\uccb4\uac00 \ud655\uc815\ub418\uace0 1991\ub144 12\uc6d4 25\uc77c \uc870\uc9c0\uc544\ub294 \uc644\uc804\ud788 \ub3c5\ub9bd\ud558\uc5ec \uc870\uc9c0\uc544 \uacf5\ud654\uad6d\uc774\ub77c\ub294 \uc0c8\ub85c\uc6b4 \ub3c5\ub9bd\uad6d\uc774 \ub418\uc5c8\uc73c\ub098, \uad6d\ub0b4\uc758 \uc815\ud30c \ub300\ub9bd\uacfc \ubbfc\uc871 \ubb38\uc81c\ub85c 1992\ub144 ~ 1994\ub144 \ub0b4\uc804\uc5d0 \ub3cc\uc785\ud558\uc5ec \uc0ac\uc2e4\uc0c1 \ubb34\uc815\ubd80 \uc0c1\ud0dc\uc5d0 \ube60\uc84c\ub2e4.", "sentence_answer": "\uac1c\ud601, \uac1c\ubc29 \uc5f4\ud48d \uc18d\uc5d0 1990\ub144 \uc870\uc9c0\uc544 \uc5ed\uc0ac\uc0c1 \ucd5c\ucd08\uc758 \ub2e4\ub2f9\uc81c \uc120\uac70\uac00 \uc2e4\uc2dc\ub410\uace0 \uce5c\ub7ec\uc2dc\uc544 \uc131\ud5a5\uc758 \uc555\ud558\uc9c0\uc57c\uc640 \ub0a8\uc624\uc138\ud2f0\uc57c \uc9c0\uc5ed\uc5d0\uc11c\ub294 \uc18c\ub828 \uc794\ub958\ub97c \uc6d0\ud558\uba70 \ub3c5\ub9bd \ub2e4\ud23c\uc774 \uc77c\uc5b4\ub0ac\uc73c\ub098 \uc870\uc9c0\uc544\ub294 \uc18c\ub828 \ud574\uccb4\uc5d0 \uc55e\uc11c 1991\ub144 4\uc6d4 9\uc77c \ub3c5\ub9bd\uc744 \uc120\uc5b8\ud588\ub2e4.", "paragraph_id": "5885698-2-0", "paragraph_question": "question: \uc870\uc9c0\uc544 \uacf5\ud654\uad6d\uc774\ub77c\ub294 \uc0c8\ub85c\uc6b4 \ub3c5\ub9bd\uad6d\uc774 \ub41c \ub144\ub3c4\ub294?, context: \uc18c\ub828 \uc2dc\uc808, \uc870\uc9c0\uc544\uc758 \ubbfc\uc871\uc6b4\ub3d9\uc740 \uc5b5\uc555\ub418\uc5c8\uc73c\ub098, \ud55c\ud3b8\uc73c\ub85c\ub294 \uc18c\ub828\uc758 \uc911\uc559 \uc815\ubd80\uc640 \ud0c0\ud611\ud558\uc5ec \uc18c\ub828 \uc911\uc559 \uc815\uce58\ubb34\ub300\uc5d0 \uace0\uc704\uc9c1\uc73c\ub85c \ub4f1\uc7a5\ud558\ub294 \uc870\uc9c0\uc544 \ucd9c\uc2e0 \uc778\ubb3c\ub3c4 \uc788\uc5c8\ub294\ub370, \uadf8\ub8e8\uc9c0\uc57c \uacf5\uc0b0\ub2f9 \uc81c1\uc11c\uae30\ub97c \uc9c0\ub0b4\ub2e4\uac00 1985\ub144 \uace0\ub974\ubc14\ucd08\ud504 \uc815\uad8c \ucd9c\ubc94 \ud6c4 \uc18c\ub828 \uc678\ubb34\uc7a5\uad00\uc774 \ub41c \uc5d0\ub450\uc544\ub974\ub4dc \uc170\ubc14\ub974\ub4dc\ub098\uc81c\uac00 \ub300\ud45c\uc801\uc774\ub2e4. 1970\ub144\ub300\uc5d0 \uc18c\ub828\uc5d0 \ubc18\ubc1c\ud558\ub294 \uc6c0\uc9c1\uc784\uc774 \uc788\uc5b4\uc654\uace0, \ub3d9\uc2dc\uc5d0 \uacf5\ud654\uad6d \ub0b4\uc758 \uc18c\uc218\ubbfc\uc871\uc778 \uc555\ud558\uc2a4\ub098 \ub0a8\uc624\uc138\ud2f0\uc57c\uc640\uc758 \uac08\ub4f1\ub3c4 \ub5a0\uc624\ub974\uae30 \uc2dc\uc791\ud588\ub2e4. \uc774\ub7ec\ud55c \ubbfc\uc871 \ub300\ub9bd\uc740 1980\ub144\ub300 \uace0\ub974\ubc14\ucd08\ud504 \uc2dc\ub300 \uc774\ud6c4 \ub354\uc6b1 \uc2ec\ud574\uc84c\uc73c\uba70 \uc870\uc9c0\uc544\uc778\ub4e4\uc740 \uc870\uc9c0\uc544\uc758 \uc18c\ub828 \ub3c5\ub9bd\uc744 \uc2dc\ub3c4\ud558\uace0 \ub3d9\uc2dc\uc5d0 \uc555\ud558\uc2a4\uc758 \ubd84\ub9ac \ub3c5\ub9bd \uc6b4\ub3d9\uc744 \uc800\uc9c0\ud558\ub824\uace0 \ud588\ub2e4. \ubcf5\uc7a1\ud55c \uc815\uad6d \uc18d\uc5d0 1989\ub144 \ud2b8\ube4c\ub9ac\uc2dc\uc5d0\uc11c \uc2dc\uc704 \uc911\uc774\ub358 \uad70\uc911\uc5d0\uac8c \uc18c\ub828\uad70\uc774 \ubc1c\ud3ec\ud558\uc5ec \uc218\uc2ed\uba85\uc774 \uc0ac\ub9dd\ud558\ub294 \uc0ac\uac74\uc774 \uc77c\uc5b4\ub098 \ub3c5\ub9bd \uc6b4\ub3d9\uc740 \ub354\uc6b1 \uac00\uc18d\ud654\ub418\uc5c8\ub2e4. \uac1c\ud601, \uac1c\ubc29 \uc5f4\ud48d \uc18d\uc5d0 1990\ub144 \uc870\uc9c0\uc544 \uc5ed\uc0ac\uc0c1 \ucd5c\ucd08\uc758 \ub2e4\ub2f9\uc81c \uc120\uac70\uac00 \uc2e4\uc2dc\ub410\uace0 \uce5c\ub7ec\uc2dc\uc544 \uc131\ud5a5\uc758 \uc555\ud558\uc9c0\uc57c\uc640 \ub0a8\uc624\uc138\ud2f0\uc57c \uc9c0\uc5ed\uc5d0\uc11c\ub294 \uc18c\ub828 \uc794\ub958\ub97c \uc6d0\ud558\uba70 \ub3c5\ub9bd \ub2e4\ud23c\uc774 \uc77c\uc5b4\ub0ac\uc73c\ub098 \uc870\uc9c0\uc544\ub294 \uc18c\ub828 \ud574\uccb4\uc5d0 \uc55e\uc11c 1991\ub144 4\uc6d4 9\uc77c \ub3c5\ub9bd\uc744 \uc120\uc5b8\ud588\ub2e4. \uc18c\ub828 \ud574\uccb4\uac00 \ud655\uc815\ub418\uace0 1991\ub144 12\uc6d4 25\uc77c \uc870\uc9c0\uc544\ub294 \uc644\uc804\ud788 \ub3c5\ub9bd\ud558\uc5ec \uc870\uc9c0\uc544 \uacf5\ud654\uad6d\uc774\ub77c\ub294 \uc0c8\ub85c\uc6b4 \ub3c5\ub9bd\uad6d\uc774 \ub418\uc5c8\uc73c\ub098, \uad6d\ub0b4\uc758 \uc815\ud30c \ub300\ub9bd\uacfc \ubbfc\uc871 \ubb38\uc81c\ub85c 1992\ub144 ~ 1994\ub144 \ub0b4\uc804\uc5d0 \ub3cc\uc785\ud558\uc5ec \uc0ac\uc2e4\uc0c1 \ubb34\uc815\ubd80 \uc0c1\ud0dc\uc5d0 \ube60\uc84c\ub2e4."} {"question": "\ud5c8\ube14 \uc6b0\uc8fc \ub9dd\uc6d0\uacbd\uc758 \uad11\uac01 \ud589\uc131 \uce74\uba54\ub77c\ub97c \uc774\uc6a9\ud55c \uc778\uc811 \ud56d\uc131 \uc5f0\uad6c\uac00 \uc5b8\uc81c \uc885\ub8cc\ub418\uc5c8\ub294\uac00?", "paragraph": "\uace0\ub798\uc790\ub9ac \ud0c0\uc6b0\ub294 \uc2dc\uc120 \uc18d\ub3c4 \uad00\uce21\uc744 \ud1b5\ud55c \ud589\uc131 \ud0d0\uc0ac\uc758 \ub300\uc0c1 \uc911 \ud558\ub098\uc600\ub2e4. 1988\ub144 \ub2f9\uc2dc \uad00\uce21\uc5d0 \ub530\ub974\uba74 \ud56d\uc131 \uc8fc\uc704\uc5d0\uc11c \uc5b4\ub5a4 \uc8fc\uae30\uc801\uc778 \uc2dc\uc120\uc18d\ub3c4 \ubcc0\ud654\ub7c9\ub3c4 \ubc1c\uacac\ub418\uc9c0 \uc54a\uc558\ub294\ub370, \uc774\ub294 \ubaa9\uc131\ubcf4\ub2e4 \uac00\uae4c\uc6b4 \uac70\ub9ac \ub0b4\uc5d0 \ubb34\uac70\uc6b4 \ud589\uc131\uc774 \uc874\uc7ac\ud558\uc9c0 \uc54a\ub294\ub2e4\ub294 \uc758\ubbf8\uc600\ub2e4. \ub354 \uc815\uad50\ud55c \uad00\uce21\uae30\uad6c\ub4e4\uc774 \uac1c\ubc1c\ub418\uc5c8\uc73c\ub098, \uc774\ub4e4\uc744 \uc774\uc6a9\ud558\uc5ec \ud0c0\uc6b0\ub97c \uad00\uce21\ud574\ub3c4 \ubb34\uac70\uc6b4 \ud589\uc131\uc740 \uc5ec\uc804\ud788 \ubc1c\uacac\ub418\uc9c0 \uc54a\uc558\ub2e4. \ub300\uc2e0 \uad00\uce21\uae30\uae30\uc758 \uc815\uad50\ud568 \ub355\uc5d0 \ud0c0\uc6b0\ubcc4\uc774 5\ub144 \uc8fc\uae30\ub85c 11 m/s \uc2dc\uc120\uc18d\ub3c4 \ubcc0\ud654\ub97c \ubcf4\uc784\uc744 \ubc1d\ud600\ub0c8\ub2e4. \uc774 \uacb0\uacfc\ub97c \ud1b5\ud574 \ud0c0\uc6b0 \uc8fc\uc704\uc5d0\ub294 \ub728\uac70\uc6b4 \ubaa9\uc131\uc774 \uc5c6\uc73c\uba70 \uacf5\uc804 \uc8fc\uae30 15\ub144 \uc774\ud558\uc758, \ubaa9\uc131 \uc815\ub3c4 \uc9c8\ub7c9 \uc774\uc0c1\uc758 \ud589\uc131\uc740 \uc5c6\uc74c\uc744 \uc54c \uc218 \uc788\ub2e4. \ub367\ubd99\uc5ec \ud5c8\ube14 \uc6b0\uc8fc \ub9dd\uc6d0\uacbd\uc758 \uad11\uac01 \ud589\uc131 \uce74\uba54\ub77c\ub97c \uc774\uc6a9\ud55c \uc778\uc811 \ud56d\uc131 \uc5f0\uad6c\uac00 1999\ub144\uc5d0 \uc885\ub8cc\ub418\uc5c8\ub294\ub370 \uc5ec\uae30\uc5d0\ub294 \uace0\ub798\uc790\ub9ac \ud0c0\uc6b0\ub97c \ub3c4\ub294 \ud76c\ubbf8\ud55c \ucc9c\uccb4\uac00 \uc788\ub294\uc9c0\uc758 \uc5ec\ubd80\ub97c \uc54c\uc544\ub0b4\ub294 \uac83\ub3c4 \ud3ec\ud568\ub418\uc5b4 \uc788\uc5c8\ub2e4. \uacb0\uacfc\ub294 \ub9dd\uc6d0\uacbd\uc758 \ubd84\ud574\ub2a5 \ud55c\ub3c4 \ub0b4\uc5d0\uc11c \ubc1c\uacac\ub41c \uac83\uc740 \uc544\ubb34\uac83\ub3c4 \uc5c6\uc5c8\ub2e4.", "answer": "1999\ub144", "sentence": "\ub367\ubd99\uc5ec \ud5c8\ube14 \uc6b0\uc8fc \ub9dd\uc6d0\uacbd\uc758 \uad11\uac01 \ud589\uc131 \uce74\uba54\ub77c\ub97c \uc774\uc6a9\ud55c \uc778\uc811 \ud56d\uc131 \uc5f0\uad6c\uac00 1999\ub144 \uc5d0 \uc885\ub8cc\ub418\uc5c8\ub294\ub370 \uc5ec\uae30\uc5d0\ub294 \uace0\ub798\uc790\ub9ac \ud0c0\uc6b0\ub97c \ub3c4\ub294 \ud76c\ubbf8\ud55c \ucc9c\uccb4\uac00 \uc788\ub294\uc9c0\uc758 \uc5ec\ubd80\ub97c \uc54c\uc544\ub0b4\ub294 \uac83\ub3c4 \ud3ec\ud568\ub418\uc5b4 \uc788\uc5c8\ub2e4.", "paragraph_sentence": "\uace0\ub798\uc790\ub9ac \ud0c0\uc6b0\ub294 \uc2dc\uc120 \uc18d\ub3c4 \uad00\uce21\uc744 \ud1b5\ud55c \ud589\uc131 \ud0d0\uc0ac\uc758 \ub300\uc0c1 \uc911 \ud558\ub098\uc600\ub2e4. 1988\ub144 \ub2f9\uc2dc \uad00\uce21\uc5d0 \ub530\ub974\uba74 \ud56d\uc131 \uc8fc\uc704\uc5d0\uc11c \uc5b4\ub5a4 \uc8fc\uae30\uc801\uc778 \uc2dc\uc120\uc18d\ub3c4 \ubcc0\ud654\ub7c9\ub3c4 \ubc1c\uacac\ub418\uc9c0 \uc54a\uc558\ub294\ub370, \uc774\ub294 \ubaa9\uc131\ubcf4\ub2e4 \uac00\uae4c\uc6b4 \uac70\ub9ac \ub0b4\uc5d0 \ubb34\uac70\uc6b4 \ud589\uc131\uc774 \uc874\uc7ac\ud558\uc9c0 \uc54a\ub294\ub2e4\ub294 \uc758\ubbf8\uc600\ub2e4. \ub354 \uc815\uad50\ud55c \uad00\uce21\uae30\uad6c\ub4e4\uc774 \uac1c\ubc1c\ub418\uc5c8\uc73c\ub098, \uc774\ub4e4\uc744 \uc774\uc6a9\ud558\uc5ec \ud0c0\uc6b0\ub97c \uad00\uce21\ud574\ub3c4 \ubb34\uac70\uc6b4 \ud589\uc131\uc740 \uc5ec\uc804\ud788 \ubc1c\uacac\ub418\uc9c0 \uc54a\uc558\ub2e4. \ub300\uc2e0 \uad00\uce21\uae30\uae30\uc758 \uc815\uad50\ud568 \ub355\uc5d0 \ud0c0\uc6b0\ubcc4\uc774 5\ub144 \uc8fc\uae30\ub85c 11 m/s \uc2dc\uc120\uc18d\ub3c4 \ubcc0\ud654\ub97c \ubcf4\uc784\uc744 \ubc1d\ud600\ub0c8\ub2e4. \uc774 \uacb0\uacfc\ub97c \ud1b5\ud574 \ud0c0\uc6b0 \uc8fc\uc704\uc5d0\ub294 \ub728\uac70\uc6b4 \ubaa9\uc131\uc774 \uc5c6\uc73c\uba70 \uacf5\uc804 \uc8fc\uae30 15\ub144 \uc774\ud558\uc758, \ubaa9\uc131 \uc815\ub3c4 \uc9c8\ub7c9 \uc774\uc0c1\uc758 \ud589\uc131\uc740 \uc5c6\uc74c\uc744 \uc54c \uc218 \uc788\ub2e4. \ub367\ubd99\uc5ec \ud5c8\ube14 \uc6b0\uc8fc \ub9dd\uc6d0\uacbd\uc758 \uad11\uac01 \ud589\uc131 \uce74\uba54\ub77c\ub97c \uc774\uc6a9\ud55c \uc778\uc811 \ud56d\uc131 \uc5f0\uad6c\uac00 1999\ub144 \uc5d0 \uc885\ub8cc\ub418\uc5c8\ub294\ub370 \uc5ec\uae30\uc5d0\ub294 \uace0\ub798\uc790\ub9ac \ud0c0\uc6b0\ub97c \ub3c4\ub294 \ud76c\ubbf8\ud55c \ucc9c\uccb4\uac00 \uc788\ub294\uc9c0\uc758 \uc5ec\ubd80\ub97c \uc54c\uc544\ub0b4\ub294 \uac83\ub3c4 \ud3ec\ud568\ub418\uc5b4 \uc788\uc5c8\ub2e4. \uacb0\uacfc\ub294 \ub9dd\uc6d0\uacbd\uc758 \ubd84\ud574\ub2a5 \ud55c\ub3c4 \ub0b4\uc5d0\uc11c \ubc1c\uacac\ub41c \uac83\uc740 \uc544\ubb34\uac83\ub3c4 \uc5c6\uc5c8\ub2e4.", "paragraph_answer": "\uace0\ub798\uc790\ub9ac \ud0c0\uc6b0\ub294 \uc2dc\uc120 \uc18d\ub3c4 \uad00\uce21\uc744 \ud1b5\ud55c \ud589\uc131 \ud0d0\uc0ac\uc758 \ub300\uc0c1 \uc911 \ud558\ub098\uc600\ub2e4. 1988\ub144 \ub2f9\uc2dc \uad00\uce21\uc5d0 \ub530\ub974\uba74 \ud56d\uc131 \uc8fc\uc704\uc5d0\uc11c \uc5b4\ub5a4 \uc8fc\uae30\uc801\uc778 \uc2dc\uc120\uc18d\ub3c4 \ubcc0\ud654\ub7c9\ub3c4 \ubc1c\uacac\ub418\uc9c0 \uc54a\uc558\ub294\ub370, \uc774\ub294 \ubaa9\uc131\ubcf4\ub2e4 \uac00\uae4c\uc6b4 \uac70\ub9ac \ub0b4\uc5d0 \ubb34\uac70\uc6b4 \ud589\uc131\uc774 \uc874\uc7ac\ud558\uc9c0 \uc54a\ub294\ub2e4\ub294 \uc758\ubbf8\uc600\ub2e4. \ub354 \uc815\uad50\ud55c \uad00\uce21\uae30\uad6c\ub4e4\uc774 \uac1c\ubc1c\ub418\uc5c8\uc73c\ub098, \uc774\ub4e4\uc744 \uc774\uc6a9\ud558\uc5ec \ud0c0\uc6b0\ub97c \uad00\uce21\ud574\ub3c4 \ubb34\uac70\uc6b4 \ud589\uc131\uc740 \uc5ec\uc804\ud788 \ubc1c\uacac\ub418\uc9c0 \uc54a\uc558\ub2e4. \ub300\uc2e0 \uad00\uce21\uae30\uae30\uc758 \uc815\uad50\ud568 \ub355\uc5d0 \ud0c0\uc6b0\ubcc4\uc774 5\ub144 \uc8fc\uae30\ub85c 11 m/s \uc2dc\uc120\uc18d\ub3c4 \ubcc0\ud654\ub97c \ubcf4\uc784\uc744 \ubc1d\ud600\ub0c8\ub2e4. \uc774 \uacb0\uacfc\ub97c \ud1b5\ud574 \ud0c0\uc6b0 \uc8fc\uc704\uc5d0\ub294 \ub728\uac70\uc6b4 \ubaa9\uc131\uc774 \uc5c6\uc73c\uba70 \uacf5\uc804 \uc8fc\uae30 15\ub144 \uc774\ud558\uc758, \ubaa9\uc131 \uc815\ub3c4 \uc9c8\ub7c9 \uc774\uc0c1\uc758 \ud589\uc131\uc740 \uc5c6\uc74c\uc744 \uc54c \uc218 \uc788\ub2e4. \ub367\ubd99\uc5ec \ud5c8\ube14 \uc6b0\uc8fc \ub9dd\uc6d0\uacbd\uc758 \uad11\uac01 \ud589\uc131 \uce74\uba54\ub77c\ub97c \uc774\uc6a9\ud55c \uc778\uc811 \ud56d\uc131 \uc5f0\uad6c\uac00 1999\ub144 \uc5d0 \uc885\ub8cc\ub418\uc5c8\ub294\ub370 \uc5ec\uae30\uc5d0\ub294 \uace0\ub798\uc790\ub9ac \ud0c0\uc6b0\ub97c \ub3c4\ub294 \ud76c\ubbf8\ud55c \ucc9c\uccb4\uac00 \uc788\ub294\uc9c0\uc758 \uc5ec\ubd80\ub97c \uc54c\uc544\ub0b4\ub294 \uac83\ub3c4 \ud3ec\ud568\ub418\uc5b4 \uc788\uc5c8\ub2e4. \uacb0\uacfc\ub294 \ub9dd\uc6d0\uacbd\uc758 \ubd84\ud574\ub2a5 \ud55c\ub3c4 \ub0b4\uc5d0\uc11c \ubc1c\uacac\ub41c \uac83\uc740 \uc544\ubb34\uac83\ub3c4 \uc5c6\uc5c8\ub2e4.", "sentence_answer": "\ub367\ubd99\uc5ec \ud5c8\ube14 \uc6b0\uc8fc \ub9dd\uc6d0\uacbd\uc758 \uad11\uac01 \ud589\uc131 \uce74\uba54\ub77c\ub97c \uc774\uc6a9\ud55c \uc778\uc811 \ud56d\uc131 \uc5f0\uad6c\uac00 1999\ub144 \uc5d0 \uc885\ub8cc\ub418\uc5c8\ub294\ub370 \uc5ec\uae30\uc5d0\ub294 \uace0\ub798\uc790\ub9ac \ud0c0\uc6b0\ub97c \ub3c4\ub294 \ud76c\ubbf8\ud55c \ucc9c\uccb4\uac00 \uc788\ub294\uc9c0\uc758 \uc5ec\ubd80\ub97c \uc54c\uc544\ub0b4\ub294 \uac83\ub3c4 \ud3ec\ud568\ub418\uc5b4 \uc788\uc5c8\ub2e4.", "paragraph_id": "6471214-1-0", "paragraph_question": "question: \ud5c8\ube14 \uc6b0\uc8fc \ub9dd\uc6d0\uacbd\uc758 \uad11\uac01 \ud589\uc131 \uce74\uba54\ub77c\ub97c \uc774\uc6a9\ud55c \uc778\uc811 \ud56d\uc131 \uc5f0\uad6c\uac00 \uc5b8\uc81c \uc885\ub8cc\ub418\uc5c8\ub294\uac00?, context: \uace0\ub798\uc790\ub9ac \ud0c0\uc6b0\ub294 \uc2dc\uc120 \uc18d\ub3c4 \uad00\uce21\uc744 \ud1b5\ud55c \ud589\uc131 \ud0d0\uc0ac\uc758 \ub300\uc0c1 \uc911 \ud558\ub098\uc600\ub2e4. 1988\ub144 \ub2f9\uc2dc \uad00\uce21\uc5d0 \ub530\ub974\uba74 \ud56d\uc131 \uc8fc\uc704\uc5d0\uc11c \uc5b4\ub5a4 \uc8fc\uae30\uc801\uc778 \uc2dc\uc120\uc18d\ub3c4 \ubcc0\ud654\ub7c9\ub3c4 \ubc1c\uacac\ub418\uc9c0 \uc54a\uc558\ub294\ub370, \uc774\ub294 \ubaa9\uc131\ubcf4\ub2e4 \uac00\uae4c\uc6b4 \uac70\ub9ac \ub0b4\uc5d0 \ubb34\uac70\uc6b4 \ud589\uc131\uc774 \uc874\uc7ac\ud558\uc9c0 \uc54a\ub294\ub2e4\ub294 \uc758\ubbf8\uc600\ub2e4. \ub354 \uc815\uad50\ud55c \uad00\uce21\uae30\uad6c\ub4e4\uc774 \uac1c\ubc1c\ub418\uc5c8\uc73c\ub098, \uc774\ub4e4\uc744 \uc774\uc6a9\ud558\uc5ec \ud0c0\uc6b0\ub97c \uad00\uce21\ud574\ub3c4 \ubb34\uac70\uc6b4 \ud589\uc131\uc740 \uc5ec\uc804\ud788 \ubc1c\uacac\ub418\uc9c0 \uc54a\uc558\ub2e4. \ub300\uc2e0 \uad00\uce21\uae30\uae30\uc758 \uc815\uad50\ud568 \ub355\uc5d0 \ud0c0\uc6b0\ubcc4\uc774 5\ub144 \uc8fc\uae30\ub85c 11 m/s \uc2dc\uc120\uc18d\ub3c4 \ubcc0\ud654\ub97c \ubcf4\uc784\uc744 \ubc1d\ud600\ub0c8\ub2e4. \uc774 \uacb0\uacfc\ub97c \ud1b5\ud574 \ud0c0\uc6b0 \uc8fc\uc704\uc5d0\ub294 \ub728\uac70\uc6b4 \ubaa9\uc131\uc774 \uc5c6\uc73c\uba70 \uacf5\uc804 \uc8fc\uae30 15\ub144 \uc774\ud558\uc758, \ubaa9\uc131 \uc815\ub3c4 \uc9c8\ub7c9 \uc774\uc0c1\uc758 \ud589\uc131\uc740 \uc5c6\uc74c\uc744 \uc54c \uc218 \uc788\ub2e4. \ub367\ubd99\uc5ec \ud5c8\ube14 \uc6b0\uc8fc \ub9dd\uc6d0\uacbd\uc758 \uad11\uac01 \ud589\uc131 \uce74\uba54\ub77c\ub97c \uc774\uc6a9\ud55c \uc778\uc811 \ud56d\uc131 \uc5f0\uad6c\uac00 1999\ub144\uc5d0 \uc885\ub8cc\ub418\uc5c8\ub294\ub370 \uc5ec\uae30\uc5d0\ub294 \uace0\ub798\uc790\ub9ac \ud0c0\uc6b0\ub97c \ub3c4\ub294 \ud76c\ubbf8\ud55c \ucc9c\uccb4\uac00 \uc788\ub294\uc9c0\uc758 \uc5ec\ubd80\ub97c \uc54c\uc544\ub0b4\ub294 \uac83\ub3c4 \ud3ec\ud568\ub418\uc5b4 \uc788\uc5c8\ub2e4. \uacb0\uacfc\ub294 \ub9dd\uc6d0\uacbd\uc758 \ubd84\ud574\ub2a5 \ud55c\ub3c4 \ub0b4\uc5d0\uc11c \ubc1c\uacac\ub41c \uac83\uc740 \uc544\ubb34\uac83\ub3c4 \uc5c6\uc5c8\ub2e4."} {"question": "\uacbd\uc81c\ud611\ub825\uac1c\ubc1c\uae30\uad6c\uac00 \uc0dd\uc0b0\uac00\ub2a5\uc778\uad6c \ub300\ube44 \ub178\uc778 \ubd80\uc591\uc778\uad6c\uc5d0 \ub300\ud574 \uc77c\ubcf8\uc744 \ub6f0\uc5b4\ub118\uc744 \uac83\uc774\ub77c\uace0 \ubcf8 \ud574\ub294?", "paragraph": "2015\ub144 \uacbd\uc81c\ud611\ub825\uac1c\ubc1c\uae30\uad6c \ud1b5\uacc4\uc5d0 \ub530\ub974\uba74 \ud604\uc7ac \ud55c\uad6d\uc740 \uc0dd\uc0b0\uac00\ub2a5\uc778\uad6c 5.1\uba85\uc774 \ub178\uc778 1\uba85\uc744 \ubd80\uc591\ud558\uace0 \uc788\uc73c\uba70, 2075\ub144\uc5d0\ub294 \uc77c\ubcf8\uc744 \ub6f0\uc5b4\ub118\uc5b4 \uc0dd\uc0b0\uac00\ub2a5\uc778\uad6c 1.25\uba85\uc774 \ub178\uc778 1\uba85\uc744 \ubd80\uc591\ud558\uac8c \ub41c\ub2e4\uace0 \ubc1d\ud614\ub2e4. \ud604\uc7ac\ub294 OECD \ud3c9\uade0\ubcf4\ub2e4\ub3c4 \ub0ae\uc740 \uc218\uc900\uc774\uc9c0\ub9cc \uae30\ub300\uc218\uba85\uc774 \uc0c1\uc2b9\ud568\uc5d0 \ub530\ub77c \uc77c\ubcf8\ub9c8\uc800 \ub118\uac8c \ub418\ub294 \uac83\uc778\ub370 \uc774\ub294 \uc55e\uc73c\ub85c \ub178\uc778 \ubd80\uc591 \ubd80\ub2f4\uc774 \uae09\uc18d\ud558\uac8c \ucee4\uc9d0\uc744 \uc758\ubbf8\ud55c\ub2e4. \ub610\ud55c OECD \ud68c\uc6d0\uad6d\uc758 \ub178\uc778 \uc18c\ub4dd\uc740 \uc804\uccb4\uc778\uad6c \uc18c\ub4dd\uc758 86.6%\uc600\uc9c0\ub9cc \ud55c\uad6d\uc740 60.1%\uc5d0 \uadf8\uce58\ub294 \ub4f1 \ub178\uc778 \ubb38\uc81c\uc758 \uc2ec\uac01\uc131\uc744 \uac15\uc870\ud588\ub2e4. \ud558\uc9c0\ub9cc \ub178\uc778 \ube48\uace4 \ubb38\uc81c\ub97c \ud574\uacb0\ud558\uae30\uc5d0 \ud604\uc7ac\uc758 \uacf5\uc801\uc5f0\uae08 \uc81c\ub3c4\ub294 \ub108\ubb34 \ucde8\uc57d\ud558\uc5ec \uc900\ubcf4\ud3b8\uc801 \ub178\ud6c4\uc18c\ub4dd \ubcf4\uc7a5\uccb4\uacc4\uc778 \uae30\ucd08\uc5f0\uae08 \ub4f1\uc744 \uac15\ud654\ud574\uc57c \ud560 \ud544\uc694\uc131\uc774 \uc81c\uae30\ub418\uc5b4 \uc654\ub2e4. 2011\ub144 \uae30\uc900\uc73c\ub85c \uad6d\ub0b4\ucd1d\uc0dd\uc0b0 \ub300\ube44 \uc9c0\ucd9c \uc218\uc900\uc774 \ud55c\uad6d\uc758 \uc5f0\uae08 \uc9c0\ucd9c\uc740 2.2%\uc5d0 \ubd88\uacfc\ud574 OECD \ud3c9\uade0\uc778 8.1%\uc5d0 \ube44\ud574 \ub9e4\uc6b0 \ub0ae\uc740 \uc218\uc900\uc774\uae30 \ub54c\ubb38\uc774\ub2e4. \ud3c9\uade0 \uc218\uae09\uc561\ub3c4 \uc6d4\ud3c9\uade0 70\ub9cc \uc6d0\uc5d0 \ubbf8\uce58\uc9c0\ub3c4 \ubabb\ud588\ub2e4.", "answer": "2075\ub144", "sentence": "2015\ub144 \uacbd\uc81c\ud611\ub825\uac1c\ubc1c\uae30\uad6c \ud1b5\uacc4\uc5d0 \ub530\ub974\uba74 \ud604\uc7ac \ud55c\uad6d\uc740 \uc0dd\uc0b0\uac00\ub2a5\uc778\uad6c 5.1\uba85\uc774 \ub178\uc778 1\uba85\uc744 \ubd80\uc591\ud558\uace0 \uc788\uc73c\uba70, 2075\ub144 \uc5d0\ub294 \uc77c\ubcf8\uc744 \ub6f0\uc5b4\ub118\uc5b4 \uc0dd\uc0b0\uac00\ub2a5\uc778\uad6c 1.25\uba85\uc774 \ub178\uc778 1\uba85\uc744 \ubd80\uc591\ud558\uac8c \ub41c\ub2e4\uace0 \ubc1d\ud614\ub2e4.", "paragraph_sentence": " 2015\ub144 \uacbd\uc81c\ud611\ub825\uac1c\ubc1c\uae30\uad6c \ud1b5\uacc4\uc5d0 \ub530\ub974\uba74 \ud604\uc7ac \ud55c\uad6d\uc740 \uc0dd\uc0b0\uac00\ub2a5\uc778\uad6c 5.1\uba85\uc774 \ub178\uc778 1\uba85\uc744 \ubd80\uc591\ud558\uace0 \uc788\uc73c\uba70, 2075\ub144 \uc5d0\ub294 \uc77c\ubcf8\uc744 \ub6f0\uc5b4\ub118\uc5b4 \uc0dd\uc0b0\uac00\ub2a5\uc778\uad6c 1.25\uba85\uc774 \ub178\uc778 1\uba85\uc744 \ubd80\uc591\ud558\uac8c \ub41c\ub2e4\uace0 \ubc1d\ud614\ub2e4. \ud604\uc7ac\ub294 OECD \ud3c9\uade0\ubcf4\ub2e4\ub3c4 \ub0ae\uc740 \uc218\uc900\uc774\uc9c0\ub9cc \uae30\ub300\uc218\uba85\uc774 \uc0c1\uc2b9\ud568\uc5d0 \ub530\ub77c \uc77c\ubcf8\ub9c8\uc800 \ub118\uac8c \ub418\ub294 \uac83\uc778\ub370 \uc774\ub294 \uc55e\uc73c\ub85c \ub178\uc778 \ubd80\uc591 \ubd80\ub2f4\uc774 \uae09\uc18d\ud558\uac8c \ucee4\uc9d0\uc744 \uc758\ubbf8\ud55c\ub2e4. \ub610\ud55c OECD \ud68c\uc6d0\uad6d\uc758 \ub178\uc778 \uc18c\ub4dd\uc740 \uc804\uccb4\uc778\uad6c \uc18c\ub4dd\uc758 86.6%\uc600\uc9c0\ub9cc \ud55c\uad6d\uc740 60.1%\uc5d0 \uadf8\uce58\ub294 \ub4f1 \ub178\uc778 \ubb38\uc81c\uc758 \uc2ec\uac01\uc131\uc744 \uac15\uc870\ud588\ub2e4. \ud558\uc9c0\ub9cc \ub178\uc778 \ube48\uace4 \ubb38\uc81c\ub97c \ud574\uacb0\ud558\uae30\uc5d0 \ud604\uc7ac\uc758 \uacf5\uc801\uc5f0\uae08 \uc81c\ub3c4\ub294 \ub108\ubb34 \ucde8\uc57d\ud558\uc5ec \uc900\ubcf4\ud3b8\uc801 \ub178\ud6c4\uc18c\ub4dd \ubcf4\uc7a5\uccb4\uacc4\uc778 \uae30\ucd08\uc5f0\uae08 \ub4f1\uc744 \uac15\ud654\ud574\uc57c \ud560 \ud544\uc694\uc131\uc774 \uc81c\uae30\ub418\uc5b4 \uc654\ub2e4. 2011\ub144 \uae30\uc900\uc73c\ub85c \uad6d\ub0b4\ucd1d\uc0dd\uc0b0 \ub300\ube44 \uc9c0\ucd9c \uc218\uc900\uc774 \ud55c\uad6d\uc758 \uc5f0\uae08 \uc9c0\ucd9c\uc740 2.2%\uc5d0 \ubd88\uacfc\ud574 OECD \ud3c9\uade0\uc778 8.1%\uc5d0 \ube44\ud574 \ub9e4\uc6b0 \ub0ae\uc740 \uc218\uc900\uc774\uae30 \ub54c\ubb38\uc774\ub2e4. \ud3c9\uade0 \uc218\uae09\uc561\ub3c4 \uc6d4\ud3c9\uade0 70\ub9cc \uc6d0\uc5d0 \ubbf8\uce58\uc9c0\ub3c4 \ubabb\ud588\ub2e4.", "paragraph_answer": "2015\ub144 \uacbd\uc81c\ud611\ub825\uac1c\ubc1c\uae30\uad6c \ud1b5\uacc4\uc5d0 \ub530\ub974\uba74 \ud604\uc7ac \ud55c\uad6d\uc740 \uc0dd\uc0b0\uac00\ub2a5\uc778\uad6c 5.1\uba85\uc774 \ub178\uc778 1\uba85\uc744 \ubd80\uc591\ud558\uace0 \uc788\uc73c\uba70, 2075\ub144 \uc5d0\ub294 \uc77c\ubcf8\uc744 \ub6f0\uc5b4\ub118\uc5b4 \uc0dd\uc0b0\uac00\ub2a5\uc778\uad6c 1.25\uba85\uc774 \ub178\uc778 1\uba85\uc744 \ubd80\uc591\ud558\uac8c \ub41c\ub2e4\uace0 \ubc1d\ud614\ub2e4. \ud604\uc7ac\ub294 OECD \ud3c9\uade0\ubcf4\ub2e4\ub3c4 \ub0ae\uc740 \uc218\uc900\uc774\uc9c0\ub9cc \uae30\ub300\uc218\uba85\uc774 \uc0c1\uc2b9\ud568\uc5d0 \ub530\ub77c \uc77c\ubcf8\ub9c8\uc800 \ub118\uac8c \ub418\ub294 \uac83\uc778\ub370 \uc774\ub294 \uc55e\uc73c\ub85c \ub178\uc778 \ubd80\uc591 \ubd80\ub2f4\uc774 \uae09\uc18d\ud558\uac8c \ucee4\uc9d0\uc744 \uc758\ubbf8\ud55c\ub2e4. \ub610\ud55c OECD \ud68c\uc6d0\uad6d\uc758 \ub178\uc778 \uc18c\ub4dd\uc740 \uc804\uccb4\uc778\uad6c \uc18c\ub4dd\uc758 86.6%\uc600\uc9c0\ub9cc \ud55c\uad6d\uc740 60.1%\uc5d0 \uadf8\uce58\ub294 \ub4f1 \ub178\uc778 \ubb38\uc81c\uc758 \uc2ec\uac01\uc131\uc744 \uac15\uc870\ud588\ub2e4. \ud558\uc9c0\ub9cc \ub178\uc778 \ube48\uace4 \ubb38\uc81c\ub97c \ud574\uacb0\ud558\uae30\uc5d0 \ud604\uc7ac\uc758 \uacf5\uc801\uc5f0\uae08 \uc81c\ub3c4\ub294 \ub108\ubb34 \ucde8\uc57d\ud558\uc5ec \uc900\ubcf4\ud3b8\uc801 \ub178\ud6c4\uc18c\ub4dd \ubcf4\uc7a5\uccb4\uacc4\uc778 \uae30\ucd08\uc5f0\uae08 \ub4f1\uc744 \uac15\ud654\ud574\uc57c \ud560 \ud544\uc694\uc131\uc774 \uc81c\uae30\ub418\uc5b4 \uc654\ub2e4. 2011\ub144 \uae30\uc900\uc73c\ub85c \uad6d\ub0b4\ucd1d\uc0dd\uc0b0 \ub300\ube44 \uc9c0\ucd9c \uc218\uc900\uc774 \ud55c\uad6d\uc758 \uc5f0\uae08 \uc9c0\ucd9c\uc740 2.2%\uc5d0 \ubd88\uacfc\ud574 OECD \ud3c9\uade0\uc778 8.1%\uc5d0 \ube44\ud574 \ub9e4\uc6b0 \ub0ae\uc740 \uc218\uc900\uc774\uae30 \ub54c\ubb38\uc774\ub2e4. \ud3c9\uade0 \uc218\uae09\uc561\ub3c4 \uc6d4\ud3c9\uade0 70\ub9cc \uc6d0\uc5d0 \ubbf8\uce58\uc9c0\ub3c4 \ubabb\ud588\ub2e4.", "sentence_answer": "2015\ub144 \uacbd\uc81c\ud611\ub825\uac1c\ubc1c\uae30\uad6c \ud1b5\uacc4\uc5d0 \ub530\ub974\uba74 \ud604\uc7ac \ud55c\uad6d\uc740 \uc0dd\uc0b0\uac00\ub2a5\uc778\uad6c 5.1\uba85\uc774 \ub178\uc778 1\uba85\uc744 \ubd80\uc591\ud558\uace0 \uc788\uc73c\uba70, 2075\ub144 \uc5d0\ub294 \uc77c\ubcf8\uc744 \ub6f0\uc5b4\ub118\uc5b4 \uc0dd\uc0b0\uac00\ub2a5\uc778\uad6c 1.25\uba85\uc774 \ub178\uc778 1\uba85\uc744 \ubd80\uc591\ud558\uac8c \ub41c\ub2e4\uace0 \ubc1d\ud614\ub2e4.", "paragraph_id": "6580792-115-1", "paragraph_question": "question: \uacbd\uc81c\ud611\ub825\uac1c\ubc1c\uae30\uad6c\uac00 \uc0dd\uc0b0\uac00\ub2a5\uc778\uad6c \ub300\ube44 \ub178\uc778 \ubd80\uc591\uc778\uad6c\uc5d0 \ub300\ud574 \uc77c\ubcf8\uc744 \ub6f0\uc5b4\ub118\uc744 \uac83\uc774\ub77c\uace0 \ubcf8 \ud574\ub294?, context: 2015\ub144 \uacbd\uc81c\ud611\ub825\uac1c\ubc1c\uae30\uad6c \ud1b5\uacc4\uc5d0 \ub530\ub974\uba74 \ud604\uc7ac \ud55c\uad6d\uc740 \uc0dd\uc0b0\uac00\ub2a5\uc778\uad6c 5.1\uba85\uc774 \ub178\uc778 1\uba85\uc744 \ubd80\uc591\ud558\uace0 \uc788\uc73c\uba70, 2075\ub144\uc5d0\ub294 \uc77c\ubcf8\uc744 \ub6f0\uc5b4\ub118\uc5b4 \uc0dd\uc0b0\uac00\ub2a5\uc778\uad6c 1.25\uba85\uc774 \ub178\uc778 1\uba85\uc744 \ubd80\uc591\ud558\uac8c \ub41c\ub2e4\uace0 \ubc1d\ud614\ub2e4. \ud604\uc7ac\ub294 OECD \ud3c9\uade0\ubcf4\ub2e4\ub3c4 \ub0ae\uc740 \uc218\uc900\uc774\uc9c0\ub9cc \uae30\ub300\uc218\uba85\uc774 \uc0c1\uc2b9\ud568\uc5d0 \ub530\ub77c \uc77c\ubcf8\ub9c8\uc800 \ub118\uac8c \ub418\ub294 \uac83\uc778\ub370 \uc774\ub294 \uc55e\uc73c\ub85c \ub178\uc778 \ubd80\uc591 \ubd80\ub2f4\uc774 \uae09\uc18d\ud558\uac8c \ucee4\uc9d0\uc744 \uc758\ubbf8\ud55c\ub2e4. \ub610\ud55c OECD \ud68c\uc6d0\uad6d\uc758 \ub178\uc778 \uc18c\ub4dd\uc740 \uc804\uccb4\uc778\uad6c \uc18c\ub4dd\uc758 86.6%\uc600\uc9c0\ub9cc \ud55c\uad6d\uc740 60.1%\uc5d0 \uadf8\uce58\ub294 \ub4f1 \ub178\uc778 \ubb38\uc81c\uc758 \uc2ec\uac01\uc131\uc744 \uac15\uc870\ud588\ub2e4. \ud558\uc9c0\ub9cc \ub178\uc778 \ube48\uace4 \ubb38\uc81c\ub97c \ud574\uacb0\ud558\uae30\uc5d0 \ud604\uc7ac\uc758 \uacf5\uc801\uc5f0\uae08 \uc81c\ub3c4\ub294 \ub108\ubb34 \ucde8\uc57d\ud558\uc5ec \uc900\ubcf4\ud3b8\uc801 \ub178\ud6c4\uc18c\ub4dd \ubcf4\uc7a5\uccb4\uacc4\uc778 \uae30\ucd08\uc5f0\uae08 \ub4f1\uc744 \uac15\ud654\ud574\uc57c \ud560 \ud544\uc694\uc131\uc774 \uc81c\uae30\ub418\uc5b4 \uc654\ub2e4. 2011\ub144 \uae30\uc900\uc73c\ub85c \uad6d\ub0b4\ucd1d\uc0dd\uc0b0 \ub300\ube44 \uc9c0\ucd9c \uc218\uc900\uc774 \ud55c\uad6d\uc758 \uc5f0\uae08 \uc9c0\ucd9c\uc740 2.2%\uc5d0 \ubd88\uacfc\ud574 OECD \ud3c9\uade0\uc778 8.1%\uc5d0 \ube44\ud574 \ub9e4\uc6b0 \ub0ae\uc740 \uc218\uc900\uc774\uae30 \ub54c\ubb38\uc774\ub2e4. \ud3c9\uade0 \uc218\uae09\uc561\ub3c4 \uc6d4\ud3c9\uade0 70\ub9cc \uc6d0\uc5d0 \ubbf8\uce58\uc9c0\ub3c4 \ubabb\ud588\ub2e4."} {"question": "\ubca0\ub9ac\uac00 \uc218\uac10\ub418\uae30 \uc804 \ub9c8\uc9c0\ub9c9\uc73c\ub85c \ubc1c\ud45c\ud55c \uc2f1\uae00\uc740 \ubb34\uc5c7\uc778\uac00?", "paragraph": "1950\ub144\ub300 \ub9d0\uc758 \ubca0\ub9ac\ub294 \ud788\ud2b8 \ub808\ucf54\ub4dc \uc5ec\ub7ff\uc744 \ubc30\ucd9c\ud558\uace0 \uc601\ud654 \ucd9c\uc5f0\uacfc \uc131\uacf5\uc801\uc778 \ud22c\uc5b4 \ud65c\ub3d9\uc744 \ud1b5\ud574 \uc138\uac04\uc758 \uc774\ubaa9\uc744 \ud55c \ubab8\uc5d0 \ubc1b\ub294 \ub300\uc2a4\ud0c0\ub85c\uc11c\uc758 \uc785\uc9c0\ub97c \uad73\ud614\ub2e4. \uadf8\ub294 \uc778\uc885 \uc81c\uc57d\uc774 \uc5c6\ub294 \uc138\uc778\ud2b8 \ub8e8\uc774\uc2a4 \ub098\uc774\ud2b8 \ud074\ub7fd\uc778 \ubca0\ub9ac\uc2a4 \ud074\ub7fd \ubc34\ub4dc\uc2a4\ud0e0\ub4dc(Berry's Club Bandstand)\ub97c \uac1c\uc7a5\ud588\uace0 \uc790\uc2e0\uc758 \uc9c4\uc9dc \uc0ac\uc720\uc9c0\uc5d0 \ud22c\uc790\ub97c \ud558\uae30\ub3c4 \ud588\ub2e4. \ud558\uc9c0\ub9cc 1959\ub144 12\uc6d4, \uadf8\ub294 14\uc138 \uc544\ud30c\uce58 \uc871 \uc6e8\uc774\ud2b8\ub9ac\uc2a4 \uc7ac\ub2c8\uc2a4 \uc5d0\uc2a4\uce7c\ub780\ud14c\ub97c \uc790\uc2e0\uc758 \ud074\ub7fd\uc5d0\uc11c \ud734\ub300\ud488 \ubcf4\uad00\uc18c \uc5ec\uc9c1\uc6d0\uc73c\ub85c \ucc44\uc6a9\ud558\uae30 \uc704\ud574 \ud568\uaed8 \uc8fc \uacbd\uacc4\ub97c \ub118\uc5b4\uac00\ub824\ub358 \uc911 \uc131\uad00\uacc4\ub97c \ub9fa\uc5c8\ub2e4\ub294 \ud610\uc758\ub85c \ub9e8\ubc95\uc5d0 \uc758\ud574 \uccb4\ud3ec\ub418\uc5c8\ub2e4. 1960\ub144 3\uc6d4\uc758 2\uc8fc \ub3d9\uc548 \uc7ac\ud310\uc5d0 \ubd88\ub9bc\uc744 \ubc1b\uc740 \uadf8\ub294 \uacb0\uad6d \uc720\uc8c4\ub97c \uc120\uace0\ubc1b\uc544 5\ub144\uc758 \uac10\uc625\uc0b4\uc774\ub97c \ud558\uac8c \ub41c \uac83\uc740 \ubb3c\ub860 5,000\ub2ec\ub7ec\uc758 \ubc8c\uae08\uc774 \ub9e4\uaca8\uc84c\ub2e4. \uadf8\ub294 \uc7ac\ud310\uad00\uc758 \ubc1c\uc5b8\uc774 \uc778\uc885\ucc28\ubcc4\uc801\uc774\uc5c8\uc73c\uba70 \ubc30\uc2ec\uc6d0\ub2e8\uc774 \uc790\uc2e0\uc5d0\uac8c \ud3b8\uacac\uc744 \uac00\uc9c0\uace0 \uc788\uc5c8\ub2e4\uace0 \ud56d\uc18c\ud588\ub2e4. \uc774 \uc8fc\uc7a5\uc740 \ubc1b\uc544\ub4e4\uc5ec\uc84c\uc73c\uba70 \uc774\uc2ec\uc740 1961\ub144 5\uc6d4\uacfc 6\uc6d4\uc5d0 \uc788\uc5c8\ub2e4. \uadf8\ub7ec\ub098 \uc720\uc8c4\ub97c \uba74\ud558\uc9c0 \ubabb\ud558\uc5ec 3\ub144\uc758 \uc9d5\uc5ed\ud615\uc774 \ub0b4\ub824\uc84c\ub2e4. \ub610\ub2e4\uc2dc \ud56d\uc18c\ub97c \ud588\uc9c0\ub9cc \ubc1b\uc544\ub4e4\uc5ec\uc9c0\uc9c0 \uc54a\uc544 \ubca0\ub9ac\ub294 1962\ub144 2\uc6d4\uc5d0\uc11c 1963\ub144 10\uc6d4\uae4c\uc9c0 \uac10\uc625\uc5d0\uc11c \uc9c0\ub0b4\uac8c \ub41c\ub2e4. \uadf8\ub294 \uc7ac\ud310 \uc2dc\uae30\uc5d0\ub3c4 \ub179\uc74c\uacfc \uacf5\uc5f0\uc744 \uc774\uc5b4\ub098\uac14\uc9c0\ub9cc, \uadf8\uc758 \uc778\uae30\ub294 \uc11c\uc11c\ud788 \ud558\ub77d\uc138\ub85c \ub3cc\uc544\uc130\ub2e4. \uc218\uac10\ub418\uae30 \uc804\uc5d0 \ub9c8\uc9c0\ub9c9\uc73c\ub85c \ubc1c\ud45c\ud55c \uc2f1\uae00\uc740 \u3008Come On\u3009\uc774\uc5c8\ub2e4.", "answer": "Come On", "sentence": "\uc218\uac10\ub418\uae30 \uc804\uc5d0 \ub9c8\uc9c0\ub9c9\uc73c\ub85c \ubc1c\ud45c\ud55c \uc2f1\uae00\uc740 \u3008 Come On \u3009\uc774\uc5c8\ub2e4.", "paragraph_sentence": "1950\ub144\ub300 \ub9d0\uc758 \ubca0\ub9ac\ub294 \ud788\ud2b8 \ub808\ucf54\ub4dc \uc5ec\ub7ff\uc744 \ubc30\ucd9c\ud558\uace0 \uc601\ud654 \ucd9c\uc5f0\uacfc \uc131\uacf5\uc801\uc778 \ud22c\uc5b4 \ud65c\ub3d9\uc744 \ud1b5\ud574 \uc138\uac04\uc758 \uc774\ubaa9\uc744 \ud55c \ubab8\uc5d0 \ubc1b\ub294 \ub300\uc2a4\ud0c0\ub85c\uc11c\uc758 \uc785\uc9c0\ub97c \uad73\ud614\ub2e4. \uadf8\ub294 \uc778\uc885 \uc81c\uc57d\uc774 \uc5c6\ub294 \uc138\uc778\ud2b8 \ub8e8\uc774\uc2a4 \ub098\uc774\ud2b8 \ud074\ub7fd\uc778 \ubca0\ub9ac\uc2a4 \ud074\ub7fd \ubc34\ub4dc\uc2a4\ud0e0\ub4dc(Berry's Club Bandstand)\ub97c \uac1c\uc7a5\ud588\uace0 \uc790\uc2e0\uc758 \uc9c4\uc9dc \uc0ac\uc720\uc9c0\uc5d0 \ud22c\uc790\ub97c \ud558\uae30\ub3c4 \ud588\ub2e4. \ud558\uc9c0\ub9cc 1959\ub144 12\uc6d4, \uadf8\ub294 14\uc138 \uc544\ud30c\uce58 \uc871 \uc6e8\uc774\ud2b8\ub9ac\uc2a4 \uc7ac\ub2c8\uc2a4 \uc5d0\uc2a4\uce7c\ub780\ud14c\ub97c \uc790\uc2e0\uc758 \ud074\ub7fd\uc5d0\uc11c \ud734\ub300\ud488 \ubcf4\uad00\uc18c \uc5ec\uc9c1\uc6d0\uc73c\ub85c \ucc44\uc6a9\ud558\uae30 \uc704\ud574 \ud568\uaed8 \uc8fc \uacbd\uacc4\ub97c \ub118\uc5b4\uac00\ub824\ub358 \uc911 \uc131\uad00\uacc4\ub97c \ub9fa\uc5c8\ub2e4\ub294 \ud610\uc758\ub85c \ub9e8\ubc95\uc5d0 \uc758\ud574 \uccb4\ud3ec\ub418\uc5c8\ub2e4. 1960\ub144 3\uc6d4\uc758 2\uc8fc \ub3d9\uc548 \uc7ac\ud310\uc5d0 \ubd88\ub9bc\uc744 \ubc1b\uc740 \uadf8\ub294 \uacb0\uad6d \uc720\uc8c4\ub97c \uc120\uace0\ubc1b\uc544 5\ub144\uc758 \uac10\uc625\uc0b4\uc774\ub97c \ud558\uac8c \ub41c \uac83\uc740 \ubb3c\ub860 5,000\ub2ec\ub7ec\uc758 \ubc8c\uae08\uc774 \ub9e4\uaca8\uc84c\ub2e4. \uadf8\ub294 \uc7ac\ud310\uad00\uc758 \ubc1c\uc5b8\uc774 \uc778\uc885\ucc28\ubcc4\uc801\uc774\uc5c8\uc73c\uba70 \ubc30\uc2ec\uc6d0\ub2e8\uc774 \uc790\uc2e0\uc5d0\uac8c \ud3b8\uacac\uc744 \uac00\uc9c0\uace0 \uc788\uc5c8\ub2e4\uace0 \ud56d\uc18c\ud588\ub2e4. \uc774 \uc8fc\uc7a5\uc740 \ubc1b\uc544\ub4e4\uc5ec\uc84c\uc73c\uba70 \uc774\uc2ec\uc740 1961\ub144 5\uc6d4\uacfc 6\uc6d4\uc5d0 \uc788\uc5c8\ub2e4. \uadf8\ub7ec\ub098 \uc720\uc8c4\ub97c \uba74\ud558\uc9c0 \ubabb\ud558\uc5ec 3\ub144\uc758 \uc9d5\uc5ed\ud615\uc774 \ub0b4\ub824\uc84c\ub2e4. \ub610\ub2e4\uc2dc \ud56d\uc18c\ub97c \ud588\uc9c0\ub9cc \ubc1b\uc544\ub4e4\uc5ec\uc9c0\uc9c0 \uc54a\uc544 \ubca0\ub9ac\ub294 1962\ub144 2\uc6d4\uc5d0\uc11c 1963\ub144 10\uc6d4\uae4c\uc9c0 \uac10\uc625\uc5d0\uc11c \uc9c0\ub0b4\uac8c \ub41c\ub2e4. \uadf8\ub294 \uc7ac\ud310 \uc2dc\uae30\uc5d0\ub3c4 \ub179\uc74c\uacfc \uacf5\uc5f0\uc744 \uc774\uc5b4\ub098\uac14\uc9c0\ub9cc, \uadf8\uc758 \uc778\uae30\ub294 \uc11c\uc11c\ud788 \ud558\ub77d\uc138\ub85c \ub3cc\uc544\uc130\ub2e4. \uc218\uac10\ub418\uae30 \uc804\uc5d0 \ub9c8\uc9c0\ub9c9\uc73c\ub85c \ubc1c\ud45c\ud55c \uc2f1\uae00\uc740 \u3008 Come On \u3009\uc774\uc5c8\ub2e4. ", "paragraph_answer": "1950\ub144\ub300 \ub9d0\uc758 \ubca0\ub9ac\ub294 \ud788\ud2b8 \ub808\ucf54\ub4dc \uc5ec\ub7ff\uc744 \ubc30\ucd9c\ud558\uace0 \uc601\ud654 \ucd9c\uc5f0\uacfc \uc131\uacf5\uc801\uc778 \ud22c\uc5b4 \ud65c\ub3d9\uc744 \ud1b5\ud574 \uc138\uac04\uc758 \uc774\ubaa9\uc744 \ud55c \ubab8\uc5d0 \ubc1b\ub294 \ub300\uc2a4\ud0c0\ub85c\uc11c\uc758 \uc785\uc9c0\ub97c \uad73\ud614\ub2e4. \uadf8\ub294 \uc778\uc885 \uc81c\uc57d\uc774 \uc5c6\ub294 \uc138\uc778\ud2b8 \ub8e8\uc774\uc2a4 \ub098\uc774\ud2b8 \ud074\ub7fd\uc778 \ubca0\ub9ac\uc2a4 \ud074\ub7fd \ubc34\ub4dc\uc2a4\ud0e0\ub4dc(Berry's Club Bandstand)\ub97c \uac1c\uc7a5\ud588\uace0 \uc790\uc2e0\uc758 \uc9c4\uc9dc \uc0ac\uc720\uc9c0\uc5d0 \ud22c\uc790\ub97c \ud558\uae30\ub3c4 \ud588\ub2e4. \ud558\uc9c0\ub9cc 1959\ub144 12\uc6d4, \uadf8\ub294 14\uc138 \uc544\ud30c\uce58 \uc871 \uc6e8\uc774\ud2b8\ub9ac\uc2a4 \uc7ac\ub2c8\uc2a4 \uc5d0\uc2a4\uce7c\ub780\ud14c\ub97c \uc790\uc2e0\uc758 \ud074\ub7fd\uc5d0\uc11c \ud734\ub300\ud488 \ubcf4\uad00\uc18c \uc5ec\uc9c1\uc6d0\uc73c\ub85c \ucc44\uc6a9\ud558\uae30 \uc704\ud574 \ud568\uaed8 \uc8fc \uacbd\uacc4\ub97c \ub118\uc5b4\uac00\ub824\ub358 \uc911 \uc131\uad00\uacc4\ub97c \ub9fa\uc5c8\ub2e4\ub294 \ud610\uc758\ub85c \ub9e8\ubc95\uc5d0 \uc758\ud574 \uccb4\ud3ec\ub418\uc5c8\ub2e4. 1960\ub144 3\uc6d4\uc758 2\uc8fc \ub3d9\uc548 \uc7ac\ud310\uc5d0 \ubd88\ub9bc\uc744 \ubc1b\uc740 \uadf8\ub294 \uacb0\uad6d \uc720\uc8c4\ub97c \uc120\uace0\ubc1b\uc544 5\ub144\uc758 \uac10\uc625\uc0b4\uc774\ub97c \ud558\uac8c \ub41c \uac83\uc740 \ubb3c\ub860 5,000\ub2ec\ub7ec\uc758 \ubc8c\uae08\uc774 \ub9e4\uaca8\uc84c\ub2e4. \uadf8\ub294 \uc7ac\ud310\uad00\uc758 \ubc1c\uc5b8\uc774 \uc778\uc885\ucc28\ubcc4\uc801\uc774\uc5c8\uc73c\uba70 \ubc30\uc2ec\uc6d0\ub2e8\uc774 \uc790\uc2e0\uc5d0\uac8c \ud3b8\uacac\uc744 \uac00\uc9c0\uace0 \uc788\uc5c8\ub2e4\uace0 \ud56d\uc18c\ud588\ub2e4. \uc774 \uc8fc\uc7a5\uc740 \ubc1b\uc544\ub4e4\uc5ec\uc84c\uc73c\uba70 \uc774\uc2ec\uc740 1961\ub144 5\uc6d4\uacfc 6\uc6d4\uc5d0 \uc788\uc5c8\ub2e4. \uadf8\ub7ec\ub098 \uc720\uc8c4\ub97c \uba74\ud558\uc9c0 \ubabb\ud558\uc5ec 3\ub144\uc758 \uc9d5\uc5ed\ud615\uc774 \ub0b4\ub824\uc84c\ub2e4. \ub610\ub2e4\uc2dc \ud56d\uc18c\ub97c \ud588\uc9c0\ub9cc \ubc1b\uc544\ub4e4\uc5ec\uc9c0\uc9c0 \uc54a\uc544 \ubca0\ub9ac\ub294 1962\ub144 2\uc6d4\uc5d0\uc11c 1963\ub144 10\uc6d4\uae4c\uc9c0 \uac10\uc625\uc5d0\uc11c \uc9c0\ub0b4\uac8c \ub41c\ub2e4. \uadf8\ub294 \uc7ac\ud310 \uc2dc\uae30\uc5d0\ub3c4 \ub179\uc74c\uacfc \uacf5\uc5f0\uc744 \uc774\uc5b4\ub098\uac14\uc9c0\ub9cc, \uadf8\uc758 \uc778\uae30\ub294 \uc11c\uc11c\ud788 \ud558\ub77d\uc138\ub85c \ub3cc\uc544\uc130\ub2e4. \uc218\uac10\ub418\uae30 \uc804\uc5d0 \ub9c8\uc9c0\ub9c9\uc73c\ub85c \ubc1c\ud45c\ud55c \uc2f1\uae00\uc740 \u3008 Come On \u3009\uc774\uc5c8\ub2e4.", "sentence_answer": "\uc218\uac10\ub418\uae30 \uc804\uc5d0 \ub9c8\uc9c0\ub9c9\uc73c\ub85c \ubc1c\ud45c\ud55c \uc2f1\uae00\uc740 \u3008 Come On \u3009\uc774\uc5c8\ub2e4.", "paragraph_id": "6596458-8-1", "paragraph_question": "question: \ubca0\ub9ac\uac00 \uc218\uac10\ub418\uae30 \uc804 \ub9c8\uc9c0\ub9c9\uc73c\ub85c \ubc1c\ud45c\ud55c \uc2f1\uae00\uc740 \ubb34\uc5c7\uc778\uac00?, context: 1950\ub144\ub300 \ub9d0\uc758 \ubca0\ub9ac\ub294 \ud788\ud2b8 \ub808\ucf54\ub4dc \uc5ec\ub7ff\uc744 \ubc30\ucd9c\ud558\uace0 \uc601\ud654 \ucd9c\uc5f0\uacfc \uc131\uacf5\uc801\uc778 \ud22c\uc5b4 \ud65c\ub3d9\uc744 \ud1b5\ud574 \uc138\uac04\uc758 \uc774\ubaa9\uc744 \ud55c \ubab8\uc5d0 \ubc1b\ub294 \ub300\uc2a4\ud0c0\ub85c\uc11c\uc758 \uc785\uc9c0\ub97c \uad73\ud614\ub2e4. \uadf8\ub294 \uc778\uc885 \uc81c\uc57d\uc774 \uc5c6\ub294 \uc138\uc778\ud2b8 \ub8e8\uc774\uc2a4 \ub098\uc774\ud2b8 \ud074\ub7fd\uc778 \ubca0\ub9ac\uc2a4 \ud074\ub7fd \ubc34\ub4dc\uc2a4\ud0e0\ub4dc(Berry's Club Bandstand)\ub97c \uac1c\uc7a5\ud588\uace0 \uc790\uc2e0\uc758 \uc9c4\uc9dc \uc0ac\uc720\uc9c0\uc5d0 \ud22c\uc790\ub97c \ud558\uae30\ub3c4 \ud588\ub2e4. \ud558\uc9c0\ub9cc 1959\ub144 12\uc6d4, \uadf8\ub294 14\uc138 \uc544\ud30c\uce58 \uc871 \uc6e8\uc774\ud2b8\ub9ac\uc2a4 \uc7ac\ub2c8\uc2a4 \uc5d0\uc2a4\uce7c\ub780\ud14c\ub97c \uc790\uc2e0\uc758 \ud074\ub7fd\uc5d0\uc11c \ud734\ub300\ud488 \ubcf4\uad00\uc18c \uc5ec\uc9c1\uc6d0\uc73c\ub85c \ucc44\uc6a9\ud558\uae30 \uc704\ud574 \ud568\uaed8 \uc8fc \uacbd\uacc4\ub97c \ub118\uc5b4\uac00\ub824\ub358 \uc911 \uc131\uad00\uacc4\ub97c \ub9fa\uc5c8\ub2e4\ub294 \ud610\uc758\ub85c \ub9e8\ubc95\uc5d0 \uc758\ud574 \uccb4\ud3ec\ub418\uc5c8\ub2e4. 1960\ub144 3\uc6d4\uc758 2\uc8fc \ub3d9\uc548 \uc7ac\ud310\uc5d0 \ubd88\ub9bc\uc744 \ubc1b\uc740 \uadf8\ub294 \uacb0\uad6d \uc720\uc8c4\ub97c \uc120\uace0\ubc1b\uc544 5\ub144\uc758 \uac10\uc625\uc0b4\uc774\ub97c \ud558\uac8c \ub41c \uac83\uc740 \ubb3c\ub860 5,000\ub2ec\ub7ec\uc758 \ubc8c\uae08\uc774 \ub9e4\uaca8\uc84c\ub2e4. \uadf8\ub294 \uc7ac\ud310\uad00\uc758 \ubc1c\uc5b8\uc774 \uc778\uc885\ucc28\ubcc4\uc801\uc774\uc5c8\uc73c\uba70 \ubc30\uc2ec\uc6d0\ub2e8\uc774 \uc790\uc2e0\uc5d0\uac8c \ud3b8\uacac\uc744 \uac00\uc9c0\uace0 \uc788\uc5c8\ub2e4\uace0 \ud56d\uc18c\ud588\ub2e4. \uc774 \uc8fc\uc7a5\uc740 \ubc1b\uc544\ub4e4\uc5ec\uc84c\uc73c\uba70 \uc774\uc2ec\uc740 1961\ub144 5\uc6d4\uacfc 6\uc6d4\uc5d0 \uc788\uc5c8\ub2e4. \uadf8\ub7ec\ub098 \uc720\uc8c4\ub97c \uba74\ud558\uc9c0 \ubabb\ud558\uc5ec 3\ub144\uc758 \uc9d5\uc5ed\ud615\uc774 \ub0b4\ub824\uc84c\ub2e4. \ub610\ub2e4\uc2dc \ud56d\uc18c\ub97c \ud588\uc9c0\ub9cc \ubc1b\uc544\ub4e4\uc5ec\uc9c0\uc9c0 \uc54a\uc544 \ubca0\ub9ac\ub294 1962\ub144 2\uc6d4\uc5d0\uc11c 1963\ub144 10\uc6d4\uae4c\uc9c0 \uac10\uc625\uc5d0\uc11c \uc9c0\ub0b4\uac8c \ub41c\ub2e4. \uadf8\ub294 \uc7ac\ud310 \uc2dc\uae30\uc5d0\ub3c4 \ub179\uc74c\uacfc \uacf5\uc5f0\uc744 \uc774\uc5b4\ub098\uac14\uc9c0\ub9cc, \uadf8\uc758 \uc778\uae30\ub294 \uc11c\uc11c\ud788 \ud558\ub77d\uc138\ub85c \ub3cc\uc544\uc130\ub2e4. \uc218\uac10\ub418\uae30 \uc804\uc5d0 \ub9c8\uc9c0\ub9c9\uc73c\ub85c \ubc1c\ud45c\ud55c \uc2f1\uae00\uc740 \u3008Come On\u3009\uc774\uc5c8\ub2e4."} {"question": "\ubb34\uc545\ud559\uc0ac\ub294 \uba87\uac1c\uc758 \ub3d9\uc73c\ub85c \uc790\ub9ac \uc7a1\uace0 \uc788\ub098?", "paragraph": "\ubc31\uc591\ub85c\uc758 \ub05d\uc5d0\ub294 \ub300\ud559 \ubcf8\ubd80\uac00 \uc704\uce58\ud55c \uc5b8\ub354\uc6b0\ub4dc\uad00\uc744 \uc911\uc2ec\uc73c\ub85c \uc67c\ud3b8\uc5d0 \uc2a4\ud300\uc2a8\uad00\uacfc \uc624\ub978\ud3b8\uc5d0 \uc544\ud39c\uc824\ub7ec\uad00\uc774 \uc704\uce58\uc5d0 \uc788\uace0 \uc138 \uac74\ubb3c\uc758 \uc911\uc559\uc5d0\ub294 \ud559\uad50\uc758 \ucc3d\uc124\uc790\uc774\uba70 \ucd08\uc11d\uc744 \uc774\ub8ec \uc5b8\ub354\uc6b0\ub4dc\uc758 \ub3d9\uc0c1\uc774 \uc11c \uc788\ub2e4. \uc2a4\ud300\uc2a8\uad00 \ub4a4\ud3b8 \uc5b8\ub355\uc704 \uc67c\ud3b8\uc5d0\ub294 \ud55c\uacbd\uad00\uacfc \ud540\uc2a8\uad00\uc774 \ub098\ub780\ud788 \uc11c \uc788\uace0, \uadf8 \ub4a4\ub85c \uc2e0\ud559\uad00\uc774 \ub3d9\ud5a5\uc744 \ubc14\ub77c\ubcf4\uba70 \uc790\ub9ac \uc7a1\uace0 \uc788\ub2e4. \uc5b8\ub354\uc6b0\ub4dc\uad00\uc758 \ub4a4\ud3b8\uc73c\ub85c \uc0ac\ud68c\uacfc\ud559\ub300\ud559\uc774 \uc790\ub9ac\ud55c \uc5f0\ud76c\uad00\uc774 \uc11c \uc788\uc73c\uba70 \uc624\ub978\ud3b8\uc5d0 \uc131\uc554\uad00, \uc67c\ud3b8\uc5d0 \uc720\uc5b5\uacb8\uae30\ub150\uad00, \uadf8 \ub4a4\ub85c \ub300\uc6b0\uad00\uc774 \uc788\ub2e4. \uc720\uc5b5\uacb8\uae30\ub150\uad00 \ub4a4\ud3b8 \uc624\ub978\ucabd\uc5d0\ub294 \ube4c\ub9c1\uc2ac\ub9ac\uad00\uc774 \uc790\ub9ac \uc7a1\uace0 \uc788\uc73c\uba70 \ub4b7\uc0b0 \uc911\ud131\uc5d0\ub294 \uc885\ud569\uad50\uc2e4\ub2e8\uacfc \uc678\uc194\uad00, \uc704\ub2f9\uad00\uc774 \uc11c \uc788\ub2e4. \uc131\uc554\uad00 \ub4b7\uae38\uc744 \ub530\ub77c \ub3d9\ubb38\ubc29\ud5a5\uc73c\ub85c \uac00\uba74 \uc6b0\uce21\uc5d0 \uc0c8\ucc9c\ub144\uad00, \uad6d\uc81c\ud559\uc0ac, \uc5b8\uc5b4\uc5f0\uad6c\uad50\uc721\uc6d0, \uc81c\uc911\ud559\uc0ac\uac00 \uc788\uace0, \ub300\uc6b0\uad00 \ub4a4\ud3b8 \uace0\uac1c\ub97c \ub118\uc5b4 \ubd81\ubb38\ubc29\ud5a5\uc73c\ub85c \uac00\uba74 \ubb34\uc545\ud559\uc0ac(\ud559\uc0dd\uae30\uc219\uc0ac) 5\uac1c\ub3d9\uc774 \uc790\ub9ac \uc7a1\uace0 \uc788\ub2e4.", "answer": "5\uac1c\ub3d9", "sentence": "\uc131\uc554\uad00 \ub4b7\uae38\uc744 \ub530\ub77c \ub3d9\ubb38\ubc29\ud5a5\uc73c\ub85c \uac00\uba74 \uc6b0\uce21\uc5d0 \uc0c8\ucc9c\ub144\uad00, \uad6d\uc81c\ud559\uc0ac, \uc5b8\uc5b4\uc5f0\uad6c\uad50\uc721\uc6d0, \uc81c\uc911\ud559\uc0ac\uac00 \uc788\uace0, \ub300\uc6b0\uad00 \ub4a4\ud3b8 \uace0\uac1c\ub97c \ub118\uc5b4 \ubd81\ubb38\ubc29\ud5a5\uc73c\ub85c \uac00\uba74 \ubb34\uc545\ud559\uc0ac(\ud559\uc0dd\uae30\uc219\uc0ac) 5\uac1c\ub3d9 \uc774 \uc790\ub9ac \uc7a1\uace0 \uc788\ub2e4.", "paragraph_sentence": "\ubc31\uc591\ub85c\uc758 \ub05d\uc5d0\ub294 \ub300\ud559 \ubcf8\ubd80\uac00 \uc704\uce58\ud55c \uc5b8\ub354\uc6b0\ub4dc\uad00\uc744 \uc911\uc2ec\uc73c\ub85c \uc67c\ud3b8\uc5d0 \uc2a4\ud300\uc2a8\uad00\uacfc \uc624\ub978\ud3b8\uc5d0 \uc544\ud39c\uc824\ub7ec\uad00\uc774 \uc704\uce58\uc5d0 \uc788\uace0 \uc138 \uac74\ubb3c\uc758 \uc911\uc559\uc5d0\ub294 \ud559\uad50\uc758 \ucc3d\uc124\uc790\uc774\uba70 \ucd08\uc11d\uc744 \uc774\ub8ec \uc5b8\ub354\uc6b0\ub4dc\uc758 \ub3d9\uc0c1\uc774 \uc11c \uc788\ub2e4. \uc2a4\ud300\uc2a8\uad00 \ub4a4\ud3b8 \uc5b8\ub355\uc704 \uc67c\ud3b8\uc5d0\ub294 \ud55c\uacbd\uad00\uacfc \ud540\uc2a8\uad00\uc774 \ub098\ub780\ud788 \uc11c \uc788\uace0, \uadf8 \ub4a4\ub85c \uc2e0\ud559\uad00\uc774 \ub3d9\ud5a5\uc744 \ubc14\ub77c\ubcf4\uba70 \uc790\ub9ac \uc7a1\uace0 \uc788\ub2e4. \uc5b8\ub354\uc6b0\ub4dc\uad00\uc758 \ub4a4\ud3b8\uc73c\ub85c \uc0ac\ud68c\uacfc\ud559\ub300\ud559\uc774 \uc790\ub9ac\ud55c \uc5f0\ud76c\uad00\uc774 \uc11c \uc788\uc73c\uba70 \uc624\ub978\ud3b8\uc5d0 \uc131\uc554\uad00, \uc67c\ud3b8\uc5d0 \uc720\uc5b5\uacb8\uae30\ub150\uad00, \uadf8 \ub4a4\ub85c \ub300\uc6b0\uad00\uc774 \uc788\ub2e4. \uc720\uc5b5\uacb8\uae30\ub150\uad00 \ub4a4\ud3b8 \uc624\ub978\ucabd\uc5d0\ub294 \ube4c\ub9c1\uc2ac\ub9ac\uad00\uc774 \uc790\ub9ac \uc7a1\uace0 \uc788\uc73c\uba70 \ub4b7\uc0b0 \uc911\ud131\uc5d0\ub294 \uc885\ud569\uad50\uc2e4\ub2e8\uacfc \uc678\uc194\uad00, \uc704\ub2f9\uad00\uc774 \uc11c \uc788\ub2e4. \uc131\uc554\uad00 \ub4b7\uae38\uc744 \ub530\ub77c \ub3d9\ubb38\ubc29\ud5a5\uc73c\ub85c \uac00\uba74 \uc6b0\uce21\uc5d0 \uc0c8\ucc9c\ub144\uad00, \uad6d\uc81c\ud559\uc0ac, \uc5b8\uc5b4\uc5f0\uad6c\uad50\uc721\uc6d0, \uc81c\uc911\ud559\uc0ac\uac00 \uc788\uace0, \ub300\uc6b0\uad00 \ub4a4\ud3b8 \uace0\uac1c\ub97c \ub118\uc5b4 \ubd81\ubb38\ubc29\ud5a5\uc73c\ub85c \uac00\uba74 \ubb34\uc545\ud559\uc0ac(\ud559\uc0dd\uae30\uc219\uc0ac) 5\uac1c\ub3d9 \uc774 \uc790\ub9ac \uc7a1\uace0 \uc788\ub2e4. ", "paragraph_answer": "\ubc31\uc591\ub85c\uc758 \ub05d\uc5d0\ub294 \ub300\ud559 \ubcf8\ubd80\uac00 \uc704\uce58\ud55c \uc5b8\ub354\uc6b0\ub4dc\uad00\uc744 \uc911\uc2ec\uc73c\ub85c \uc67c\ud3b8\uc5d0 \uc2a4\ud300\uc2a8\uad00\uacfc \uc624\ub978\ud3b8\uc5d0 \uc544\ud39c\uc824\ub7ec\uad00\uc774 \uc704\uce58\uc5d0 \uc788\uace0 \uc138 \uac74\ubb3c\uc758 \uc911\uc559\uc5d0\ub294 \ud559\uad50\uc758 \ucc3d\uc124\uc790\uc774\uba70 \ucd08\uc11d\uc744 \uc774\ub8ec \uc5b8\ub354\uc6b0\ub4dc\uc758 \ub3d9\uc0c1\uc774 \uc11c \uc788\ub2e4. \uc2a4\ud300\uc2a8\uad00 \ub4a4\ud3b8 \uc5b8\ub355\uc704 \uc67c\ud3b8\uc5d0\ub294 \ud55c\uacbd\uad00\uacfc \ud540\uc2a8\uad00\uc774 \ub098\ub780\ud788 \uc11c \uc788\uace0, \uadf8 \ub4a4\ub85c \uc2e0\ud559\uad00\uc774 \ub3d9\ud5a5\uc744 \ubc14\ub77c\ubcf4\uba70 \uc790\ub9ac \uc7a1\uace0 \uc788\ub2e4. \uc5b8\ub354\uc6b0\ub4dc\uad00\uc758 \ub4a4\ud3b8\uc73c\ub85c \uc0ac\ud68c\uacfc\ud559\ub300\ud559\uc774 \uc790\ub9ac\ud55c \uc5f0\ud76c\uad00\uc774 \uc11c \uc788\uc73c\uba70 \uc624\ub978\ud3b8\uc5d0 \uc131\uc554\uad00, \uc67c\ud3b8\uc5d0 \uc720\uc5b5\uacb8\uae30\ub150\uad00, \uadf8 \ub4a4\ub85c \ub300\uc6b0\uad00\uc774 \uc788\ub2e4. \uc720\uc5b5\uacb8\uae30\ub150\uad00 \ub4a4\ud3b8 \uc624\ub978\ucabd\uc5d0\ub294 \ube4c\ub9c1\uc2ac\ub9ac\uad00\uc774 \uc790\ub9ac \uc7a1\uace0 \uc788\uc73c\uba70 \ub4b7\uc0b0 \uc911\ud131\uc5d0\ub294 \uc885\ud569\uad50\uc2e4\ub2e8\uacfc \uc678\uc194\uad00, \uc704\ub2f9\uad00\uc774 \uc11c \uc788\ub2e4. \uc131\uc554\uad00 \ub4b7\uae38\uc744 \ub530\ub77c \ub3d9\ubb38\ubc29\ud5a5\uc73c\ub85c \uac00\uba74 \uc6b0\uce21\uc5d0 \uc0c8\ucc9c\ub144\uad00, \uad6d\uc81c\ud559\uc0ac, \uc5b8\uc5b4\uc5f0\uad6c\uad50\uc721\uc6d0, \uc81c\uc911\ud559\uc0ac\uac00 \uc788\uace0, \ub300\uc6b0\uad00 \ub4a4\ud3b8 \uace0\uac1c\ub97c \ub118\uc5b4 \ubd81\ubb38\ubc29\ud5a5\uc73c\ub85c \uac00\uba74 \ubb34\uc545\ud559\uc0ac(\ud559\uc0dd\uae30\uc219\uc0ac) 5\uac1c\ub3d9 \uc774 \uc790\ub9ac \uc7a1\uace0 \uc788\ub2e4.", "sentence_answer": "\uc131\uc554\uad00 \ub4b7\uae38\uc744 \ub530\ub77c \ub3d9\ubb38\ubc29\ud5a5\uc73c\ub85c \uac00\uba74 \uc6b0\uce21\uc5d0 \uc0c8\ucc9c\ub144\uad00, \uad6d\uc81c\ud559\uc0ac, \uc5b8\uc5b4\uc5f0\uad6c\uad50\uc721\uc6d0, \uc81c\uc911\ud559\uc0ac\uac00 \uc788\uace0, \ub300\uc6b0\uad00 \ub4a4\ud3b8 \uace0\uac1c\ub97c \ub118\uc5b4 \ubd81\ubb38\ubc29\ud5a5\uc73c\ub85c \uac00\uba74 \ubb34\uc545\ud559\uc0ac(\ud559\uc0dd\uae30\uc219\uc0ac) 5\uac1c\ub3d9 \uc774 \uc790\ub9ac \uc7a1\uace0 \uc788\ub2e4.", "paragraph_id": "6458470-6-2", "paragraph_question": "question: \ubb34\uc545\ud559\uc0ac\ub294 \uba87\uac1c\uc758 \ub3d9\uc73c\ub85c \uc790\ub9ac \uc7a1\uace0 \uc788\ub098?, context: \ubc31\uc591\ub85c\uc758 \ub05d\uc5d0\ub294 \ub300\ud559 \ubcf8\ubd80\uac00 \uc704\uce58\ud55c \uc5b8\ub354\uc6b0\ub4dc\uad00\uc744 \uc911\uc2ec\uc73c\ub85c \uc67c\ud3b8\uc5d0 \uc2a4\ud300\uc2a8\uad00\uacfc \uc624\ub978\ud3b8\uc5d0 \uc544\ud39c\uc824\ub7ec\uad00\uc774 \uc704\uce58\uc5d0 \uc788\uace0 \uc138 \uac74\ubb3c\uc758 \uc911\uc559\uc5d0\ub294 \ud559\uad50\uc758 \ucc3d\uc124\uc790\uc774\uba70 \ucd08\uc11d\uc744 \uc774\ub8ec \uc5b8\ub354\uc6b0\ub4dc\uc758 \ub3d9\uc0c1\uc774 \uc11c \uc788\ub2e4. \uc2a4\ud300\uc2a8\uad00 \ub4a4\ud3b8 \uc5b8\ub355\uc704 \uc67c\ud3b8\uc5d0\ub294 \ud55c\uacbd\uad00\uacfc \ud540\uc2a8\uad00\uc774 \ub098\ub780\ud788 \uc11c \uc788\uace0, \uadf8 \ub4a4\ub85c \uc2e0\ud559\uad00\uc774 \ub3d9\ud5a5\uc744 \ubc14\ub77c\ubcf4\uba70 \uc790\ub9ac \uc7a1\uace0 \uc788\ub2e4. \uc5b8\ub354\uc6b0\ub4dc\uad00\uc758 \ub4a4\ud3b8\uc73c\ub85c \uc0ac\ud68c\uacfc\ud559\ub300\ud559\uc774 \uc790\ub9ac\ud55c \uc5f0\ud76c\uad00\uc774 \uc11c \uc788\uc73c\uba70 \uc624\ub978\ud3b8\uc5d0 \uc131\uc554\uad00, \uc67c\ud3b8\uc5d0 \uc720\uc5b5\uacb8\uae30\ub150\uad00, \uadf8 \ub4a4\ub85c \ub300\uc6b0\uad00\uc774 \uc788\ub2e4. \uc720\uc5b5\uacb8\uae30\ub150\uad00 \ub4a4\ud3b8 \uc624\ub978\ucabd\uc5d0\ub294 \ube4c\ub9c1\uc2ac\ub9ac\uad00\uc774 \uc790\ub9ac \uc7a1\uace0 \uc788\uc73c\uba70 \ub4b7\uc0b0 \uc911\ud131\uc5d0\ub294 \uc885\ud569\uad50\uc2e4\ub2e8\uacfc \uc678\uc194\uad00, \uc704\ub2f9\uad00\uc774 \uc11c \uc788\ub2e4. \uc131\uc554\uad00 \ub4b7\uae38\uc744 \ub530\ub77c \ub3d9\ubb38\ubc29\ud5a5\uc73c\ub85c \uac00\uba74 \uc6b0\uce21\uc5d0 \uc0c8\ucc9c\ub144\uad00, \uad6d\uc81c\ud559\uc0ac, \uc5b8\uc5b4\uc5f0\uad6c\uad50\uc721\uc6d0, \uc81c\uc911\ud559\uc0ac\uac00 \uc788\uace0, \ub300\uc6b0\uad00 \ub4a4\ud3b8 \uace0\uac1c\ub97c \ub118\uc5b4 \ubd81\ubb38\ubc29\ud5a5\uc73c\ub85c \uac00\uba74 \ubb34\uc545\ud559\uc0ac(\ud559\uc0dd\uae30\uc219\uc0ac) 5\uac1c\ub3d9\uc774 \uc790\ub9ac \uc7a1\uace0 \uc788\ub2e4."} {"question": "\uc601\uad6d \ucc28\ud2b8 1\uc704\ub97c \uae30\ub85d\ud55c \ud1b0 \uc18c\ub839 \uce90\ub9ad\ud130\uac00 \ub4f1\uc7a5\ud558\ub294 \uc2f1\uae00 \uc568\ubc94\uc740?", "paragraph": "\u300aScary Monsters and Super Creeps\u300b (1980)\ub294 \ucc28\ud2b8 1\uc704 \uc2f1\uae00 \u3008Ashes to Ashes\u3009\ub97c \ubc30\ucd9c\ud588\ub2e4. \ub178\ub798\uc5d0\ub294 \uae30\ud0c0 \ud569\uc131\ubc95\uc744 \ub2e4\ub8e8\ub358 \ucc99 \ud574\uba38\uc758 \uad6c\uc870\uc801 \uc791\uc5c5\uc774 \ub4e4\uc5b4\uac14\uc73c\uba70, \u3008Space Oddity\u3009\uc758 \ud1b0 \uc18c\ub839 \uce90\ub9ad\ud130\uac00 \uc7ac\ub4f1\uc7a5\ud588\ub2e4. \ub178\ub798\ub294 \uc218\uba74 \uc544\ub798\uc758 \ub274 \ub85c\ub9e8\ud2f1 \uc6b4\ub3d9\uc73c\ub85c \uad6d\uc81c\uc801\uc73c\ub85c \uc54c\ub824\uc9c4\ub2e4. \ubcf4\uc704\ub294 \ub274 \ub85c\ub9e8\ud2f1\uc758 \uac70\uc810\uc774\uc5c8\ub358 \ub7f0\ub358 \ud074\ub7fd \"\ube14\ub9ac\uce20\"\uc5d0 \ubc29\ubb38\ud558\uc5ec \ubba4\uc9c1\ube44\ub514\uc624\uc5d0 \ucd9c\uc5f0\ud560 \uc0ac\ub78c\uc744 \ubf51\uc558\uace0, \uadf8 \uc911 \ubc34\ub4dc \ube44\uc0ac\uc9c0\uc758 \uc2a4\ud2f0\ube0c \uc2a4\ud2b8\ub79c\uc9c0\ub3c4 \uc788\uc5c8\ub2e4. \uc774 \ubba4\uc9c1\ube44\ub514\uc624\ub294 \uadf8 \ub2f9\uc2dc \uac00\uc7a5 \ud601\uc2e0\uc801\uc778 \uac83\uc73c\ub85c \uc774\ub984\uc744 \uc54c\ub838\ub2e4. \ubca0\ub97c\ub9b0 \uc2dc\uae30 \uad73\uc5b4\uc9c4 \uc791\ubc95\uc744 \uc0ac\uc6a9\ud55c \u300aScary Monsters\u300b\ub294 \uc74c\uc545, \uac00\uc0ac\uac00 \ub354\uc6b1 \uc9c1\uc811\uc801\uc73c\ub85c \uc804\ub2ec\ub418\uc5c8\ub2e4\uace0 \ube44\ud3c9\uac00\ub4e4\uc740 \uc9c4\ub2e8\ud588\ub2e4. \uc74c\ubc18\uc758 \ub0a0\uce74\ub85c\uc6b4 \ud558\ub4dc \ub85d\uc740 \ub85c\ubc84\ud2b8 \ud504\ub9bd, \ucc99 \ud574\uba38, \ud53c\ud2b8 \ud0c0\uc6b4\uc820\ub4dc\uc758 \ub69c\ub837\ud55c \uae30\ud0c0\uac00 \ud3ec\ud568\ub418\uc5b4 \uc788\uc5c8\ub2e4. \u3008Ashes to Ashes\u3009\uac00 \uc601\uad6d \ucc28\ud2b8 1\uc704\ub97c \uae30\ub85d\ud558\uc790 \ubcf4\uc704\ub294 \ube0c\ub85c\ub4dc\uc6e8\uc774\uc5d0\uc11c 9\uc6d4 24\uc77c\ubd80\ud130 \uc2dc\uc791, 3\ub2ec \ub3d9\uc548 \uc5f0\uadf9 \u300a\uc5d8\ub9ac\ud380\ud2b8 \ub9e8\u300b\uc5d0 \ucd9c\uc5f0\ud588\ub2e4. \uadf8\ub294 \ubb34\ub824 157\ud68c\uc758 \uacf5\uc5f0\uc744 \uc131\uacf5\ub9ac\uc5d0 \ub9c8\ucce4\ub2e4.", "answer": "Ashes to Ashes", "sentence": "\u300aScary Monsters and Super Creeps\u300b (1980)\ub294 \ucc28\ud2b8 1\uc704 \uc2f1\uae00 \u3008 Ashes to Ashes \u3009\ub97c \ubc30\ucd9c\ud588\ub2e4.", "paragraph_sentence": " \u300aScary Monsters and Super Creeps\u300b (1980)\ub294 \ucc28\ud2b8 1\uc704 \uc2f1\uae00 \u3008 Ashes to Ashes \u3009\ub97c \ubc30\ucd9c\ud588\ub2e4. \ub178\ub798\uc5d0\ub294 \uae30\ud0c0 \ud569\uc131\ubc95\uc744 \ub2e4\ub8e8\ub358 \ucc99 \ud574\uba38\uc758 \uad6c\uc870\uc801 \uc791\uc5c5\uc774 \ub4e4\uc5b4\uac14\uc73c\uba70, \u3008Space Oddity\u3009\uc758 \ud1b0 \uc18c\ub839 \uce90\ub9ad\ud130\uac00 \uc7ac\ub4f1\uc7a5\ud588\ub2e4. \ub178\ub798\ub294 \uc218\uba74 \uc544\ub798\uc758 \ub274 \ub85c\ub9e8\ud2f1 \uc6b4\ub3d9\uc73c\ub85c \uad6d\uc81c\uc801\uc73c\ub85c \uc54c\ub824\uc9c4\ub2e4. \ubcf4\uc704\ub294 \ub274 \ub85c\ub9e8\ud2f1\uc758 \uac70\uc810\uc774\uc5c8\ub358 \ub7f0\ub358 \ud074\ub7fd \"\ube14\ub9ac\uce20\"\uc5d0 \ubc29\ubb38\ud558\uc5ec \ubba4\uc9c1\ube44\ub514\uc624\uc5d0 \ucd9c\uc5f0\ud560 \uc0ac\ub78c\uc744 \ubf51\uc558\uace0, \uadf8 \uc911 \ubc34\ub4dc \ube44\uc0ac\uc9c0\uc758 \uc2a4\ud2f0\ube0c \uc2a4\ud2b8\ub79c\uc9c0\ub3c4 \uc788\uc5c8\ub2e4. \uc774 \ubba4\uc9c1\ube44\ub514\uc624\ub294 \uadf8 \ub2f9\uc2dc \uac00\uc7a5 \ud601\uc2e0\uc801\uc778 \uac83\uc73c\ub85c \uc774\ub984\uc744 \uc54c\ub838\ub2e4. \ubca0\ub97c\ub9b0 \uc2dc\uae30 \uad73\uc5b4\uc9c4 \uc791\ubc95\uc744 \uc0ac\uc6a9\ud55c \u300aScary Monsters\u300b\ub294 \uc74c\uc545, \uac00\uc0ac\uac00 \ub354\uc6b1 \uc9c1\uc811\uc801\uc73c\ub85c \uc804\ub2ec\ub418\uc5c8\ub2e4\uace0 \ube44\ud3c9\uac00\ub4e4\uc740 \uc9c4\ub2e8\ud588\ub2e4. \uc74c\ubc18\uc758 \ub0a0\uce74\ub85c\uc6b4 \ud558\ub4dc \ub85d\uc740 \ub85c\ubc84\ud2b8 \ud504\ub9bd, \ucc99 \ud574\uba38, \ud53c\ud2b8 \ud0c0\uc6b4\uc820\ub4dc\uc758 \ub69c\ub837\ud55c \uae30\ud0c0\uac00 \ud3ec\ud568\ub418\uc5b4 \uc788\uc5c8\ub2e4. \u3008Ashes to Ashes\u3009\uac00 \uc601\uad6d \ucc28\ud2b8 1\uc704\ub97c \uae30\ub85d\ud558\uc790 \ubcf4\uc704\ub294 \ube0c\ub85c\ub4dc\uc6e8\uc774\uc5d0\uc11c 9\uc6d4 24\uc77c\ubd80\ud130 \uc2dc\uc791, 3\ub2ec \ub3d9\uc548 \uc5f0\uadf9 \u300a\uc5d8\ub9ac\ud380\ud2b8 \ub9e8\u300b\uc5d0 \ucd9c\uc5f0\ud588\ub2e4. \uadf8\ub294 \ubb34\ub824 157\ud68c\uc758 \uacf5\uc5f0\uc744 \uc131\uacf5\ub9ac\uc5d0 \ub9c8\ucce4\ub2e4.", "paragraph_answer": "\u300aScary Monsters and Super Creeps\u300b (1980)\ub294 \ucc28\ud2b8 1\uc704 \uc2f1\uae00 \u3008 Ashes to Ashes \u3009\ub97c \ubc30\ucd9c\ud588\ub2e4. \ub178\ub798\uc5d0\ub294 \uae30\ud0c0 \ud569\uc131\ubc95\uc744 \ub2e4\ub8e8\ub358 \ucc99 \ud574\uba38\uc758 \uad6c\uc870\uc801 \uc791\uc5c5\uc774 \ub4e4\uc5b4\uac14\uc73c\uba70, \u3008Space Oddity\u3009\uc758 \ud1b0 \uc18c\ub839 \uce90\ub9ad\ud130\uac00 \uc7ac\ub4f1\uc7a5\ud588\ub2e4. \ub178\ub798\ub294 \uc218\uba74 \uc544\ub798\uc758 \ub274 \ub85c\ub9e8\ud2f1 \uc6b4\ub3d9\uc73c\ub85c \uad6d\uc81c\uc801\uc73c\ub85c \uc54c\ub824\uc9c4\ub2e4. \ubcf4\uc704\ub294 \ub274 \ub85c\ub9e8\ud2f1\uc758 \uac70\uc810\uc774\uc5c8\ub358 \ub7f0\ub358 \ud074\ub7fd \"\ube14\ub9ac\uce20\"\uc5d0 \ubc29\ubb38\ud558\uc5ec \ubba4\uc9c1\ube44\ub514\uc624\uc5d0 \ucd9c\uc5f0\ud560 \uc0ac\ub78c\uc744 \ubf51\uc558\uace0, \uadf8 \uc911 \ubc34\ub4dc \ube44\uc0ac\uc9c0\uc758 \uc2a4\ud2f0\ube0c \uc2a4\ud2b8\ub79c\uc9c0\ub3c4 \uc788\uc5c8\ub2e4. \uc774 \ubba4\uc9c1\ube44\ub514\uc624\ub294 \uadf8 \ub2f9\uc2dc \uac00\uc7a5 \ud601\uc2e0\uc801\uc778 \uac83\uc73c\ub85c \uc774\ub984\uc744 \uc54c\ub838\ub2e4. \ubca0\ub97c\ub9b0 \uc2dc\uae30 \uad73\uc5b4\uc9c4 \uc791\ubc95\uc744 \uc0ac\uc6a9\ud55c \u300aScary Monsters\u300b\ub294 \uc74c\uc545, \uac00\uc0ac\uac00 \ub354\uc6b1 \uc9c1\uc811\uc801\uc73c\ub85c \uc804\ub2ec\ub418\uc5c8\ub2e4\uace0 \ube44\ud3c9\uac00\ub4e4\uc740 \uc9c4\ub2e8\ud588\ub2e4. \uc74c\ubc18\uc758 \ub0a0\uce74\ub85c\uc6b4 \ud558\ub4dc \ub85d\uc740 \ub85c\ubc84\ud2b8 \ud504\ub9bd, \ucc99 \ud574\uba38, \ud53c\ud2b8 \ud0c0\uc6b4\uc820\ub4dc\uc758 \ub69c\ub837\ud55c \uae30\ud0c0\uac00 \ud3ec\ud568\ub418\uc5b4 \uc788\uc5c8\ub2e4. \u3008Ashes to Ashes\u3009\uac00 \uc601\uad6d \ucc28\ud2b8 1\uc704\ub97c \uae30\ub85d\ud558\uc790 \ubcf4\uc704\ub294 \ube0c\ub85c\ub4dc\uc6e8\uc774\uc5d0\uc11c 9\uc6d4 24\uc77c\ubd80\ud130 \uc2dc\uc791, 3\ub2ec \ub3d9\uc548 \uc5f0\uadf9 \u300a\uc5d8\ub9ac\ud380\ud2b8 \ub9e8\u300b\uc5d0 \ucd9c\uc5f0\ud588\ub2e4. \uadf8\ub294 \ubb34\ub824 157\ud68c\uc758 \uacf5\uc5f0\uc744 \uc131\uacf5\ub9ac\uc5d0 \ub9c8\ucce4\ub2e4.", "sentence_answer": "\u300aScary Monsters and Super Creeps\u300b (1980)\ub294 \ucc28\ud2b8 1\uc704 \uc2f1\uae00 \u3008 Ashes to Ashes \u3009\ub97c \ubc30\ucd9c\ud588\ub2e4.", "paragraph_id": "6578410-28-1", "paragraph_question": "question: \uc601\uad6d \ucc28\ud2b8 1\uc704\ub97c \uae30\ub85d\ud55c \ud1b0 \uc18c\ub839 \uce90\ub9ad\ud130\uac00 \ub4f1\uc7a5\ud558\ub294 \uc2f1\uae00 \uc568\ubc94\uc740?, context: \u300aScary Monsters and Super Creeps\u300b (1980)\ub294 \ucc28\ud2b8 1\uc704 \uc2f1\uae00 \u3008Ashes to Ashes\u3009\ub97c \ubc30\ucd9c\ud588\ub2e4. \ub178\ub798\uc5d0\ub294 \uae30\ud0c0 \ud569\uc131\ubc95\uc744 \ub2e4\ub8e8\ub358 \ucc99 \ud574\uba38\uc758 \uad6c\uc870\uc801 \uc791\uc5c5\uc774 \ub4e4\uc5b4\uac14\uc73c\uba70, \u3008Space Oddity\u3009\uc758 \ud1b0 \uc18c\ub839 \uce90\ub9ad\ud130\uac00 \uc7ac\ub4f1\uc7a5\ud588\ub2e4. \ub178\ub798\ub294 \uc218\uba74 \uc544\ub798\uc758 \ub274 \ub85c\ub9e8\ud2f1 \uc6b4\ub3d9\uc73c\ub85c \uad6d\uc81c\uc801\uc73c\ub85c \uc54c\ub824\uc9c4\ub2e4. \ubcf4\uc704\ub294 \ub274 \ub85c\ub9e8\ud2f1\uc758 \uac70\uc810\uc774\uc5c8\ub358 \ub7f0\ub358 \ud074\ub7fd \"\ube14\ub9ac\uce20\"\uc5d0 \ubc29\ubb38\ud558\uc5ec \ubba4\uc9c1\ube44\ub514\uc624\uc5d0 \ucd9c\uc5f0\ud560 \uc0ac\ub78c\uc744 \ubf51\uc558\uace0, \uadf8 \uc911 \ubc34\ub4dc \ube44\uc0ac\uc9c0\uc758 \uc2a4\ud2f0\ube0c \uc2a4\ud2b8\ub79c\uc9c0\ub3c4 \uc788\uc5c8\ub2e4. \uc774 \ubba4\uc9c1\ube44\ub514\uc624\ub294 \uadf8 \ub2f9\uc2dc \uac00\uc7a5 \ud601\uc2e0\uc801\uc778 \uac83\uc73c\ub85c \uc774\ub984\uc744 \uc54c\ub838\ub2e4. \ubca0\ub97c\ub9b0 \uc2dc\uae30 \uad73\uc5b4\uc9c4 \uc791\ubc95\uc744 \uc0ac\uc6a9\ud55c \u300aScary Monsters\u300b\ub294 \uc74c\uc545, \uac00\uc0ac\uac00 \ub354\uc6b1 \uc9c1\uc811\uc801\uc73c\ub85c \uc804\ub2ec\ub418\uc5c8\ub2e4\uace0 \ube44\ud3c9\uac00\ub4e4\uc740 \uc9c4\ub2e8\ud588\ub2e4. \uc74c\ubc18\uc758 \ub0a0\uce74\ub85c\uc6b4 \ud558\ub4dc \ub85d\uc740 \ub85c\ubc84\ud2b8 \ud504\ub9bd, \ucc99 \ud574\uba38, \ud53c\ud2b8 \ud0c0\uc6b4\uc820\ub4dc\uc758 \ub69c\ub837\ud55c \uae30\ud0c0\uac00 \ud3ec\ud568\ub418\uc5b4 \uc788\uc5c8\ub2e4. \u3008Ashes to Ashes\u3009\uac00 \uc601\uad6d \ucc28\ud2b8 1\uc704\ub97c \uae30\ub85d\ud558\uc790 \ubcf4\uc704\ub294 \ube0c\ub85c\ub4dc\uc6e8\uc774\uc5d0\uc11c 9\uc6d4 24\uc77c\ubd80\ud130 \uc2dc\uc791, 3\ub2ec \ub3d9\uc548 \uc5f0\uadf9 \u300a\uc5d8\ub9ac\ud380\ud2b8 \ub9e8\u300b\uc5d0 \ucd9c\uc5f0\ud588\ub2e4. \uadf8\ub294 \ubb34\ub824 157\ud68c\uc758 \uacf5\uc5f0\uc744 \uc131\uacf5\ub9ac\uc5d0 \ub9c8\ucce4\ub2e4."} {"question": "\uc601\uad6d\uad70\uacfc \ud504\ub791\uc2a4\uad70\uc774 1916\ub144 7\uc6d4\uc5d0\uc11c 11\uc6d4\uc5d0 \uac78\uccd0 \ub3c5\uc77c\uad70\uc744 \ubab0\uc544\ubd99\uc5ec \uaca8\uc6b8\uc5d4 \uc5b4\ub290 \ubc29\uc5b4\uc120\uae4c\uc9c0 \ubc00\uc5b4\ub0c8\ub098?", "paragraph": "1915\ub144 6\uc6d4 17\uc77c \ub9cc\uc288\ud0c0\uc778\uc740 \ubcf5\uadc0\ud558\uc5ec \ub9c9\uc2a4 \ud3f0 \uac08\ube44\uce20\uac00 \uc9c0\ud718\ud558\ub294 \uc81c10\uad70 \uc791\uc804\ucc38\ubaa8\uc7a5\uad50\uac00 \ub418\uc5c8\ub2e4. \uc5bc\ub9c8 \ub4a4 \ub300\uc704\ub85c \uc2b9\uc9c4\ud55c \ub9cc\uc288\ud0c0\uc778\uc740 \uc81c10\uad70\uc774 \ud3f4\ub780\ub4dc, \ub9ac\ud22c\uc544\ub2c8\uc544, \ubaac\ud14c\ub124\uadf8\ub85c, \uc54c\ubc14\ub2c8\uc544\ub97c \uc131\uacf5\uc801\uc73c\ub85c \uacf5\uaca9\ud558\ub294 \uacfc\uc815\uc5d0\uc11c \uacf5\uc138\uc791\uc804 \uacc4\ud68d \ubc0f \uc218\ud589\uc744 \uacbd\ud5d8\uc73c\ub85c \uccb4\ub4dd\ud588\ub2e4. 1916\ub144 \ucd08 \ubca0\ub974\ub42d\uc5d0\uc11c \uacf5\uc138\uc791\uc804\uc744 \uc218\ud589\ud558\uace0 \uadf8 \ub4a4\uc5d0\ub294 \uc19c \uac15 \uadfc\ucc98\uc758 \uc0ac\ub839\ubd80\uc5d0\uc11c \ud504\ub9ac\uce20 \ud3f0 \ubca8\ub85c\ube0c \uc7a5\uad70\uacfc \ud504\ub9ac\uce20 \ud3f0 \ub85c\uc2a4\ubca0\ub974\ud06c \ucc38\ubaa8\uc7a5 \ud718\ud558\uc5d0 \ubcf4\uae09\uc7a5\uad50\ub85c \uc77c\ud588\ub2e4. \uc774 \uc77c\ub300\ub294 1\ucc28 \ub300\uc804 \ub0b4\ub0b4 \uac00\uc7a5 \uaca9\ub82c\ud55c \uc804\ud22c\uac00 \ubc8c\uc5b4\uc9c4 \uc804\uc7a5\uc774\uc5c8\ub2e4. \uc601\uad6d\uad70\uacfc \ud504\ub791\uc2a4\uad70\uc740 1916\ub144 7\uc6d4\uc5d0\uc11c 11\uc6d4\uc5d0 \uac78\uccd0 \uacf5\uc138\ub97c \ud3bc\uccd0 \uaca8\uc6b8\ucbe4\uc5d0\ub294 \ub3c5\uc77c\uad70\uc744 \ubca0\ub974\ub42d-\ub791\uc2a4 \uc0ac\uc774\uc5d0 \uad6c\ucd95\ub41c \ubc29\uc5b4\uc120\uc778 \ud78c\ub374\ubd80\ub974\ud06c \uc120\uae4c\uc9c0 \ubc00\uc5b4\ubd99\uc600\ub2e4. \ub9cc\uc288\ud0c0\uc778 \ub300\uc704\ub294 1917\ub144 10\uc6d4\uae4c\uc9c0 \uacc4\uc18d \ubca8\ub85c\ube0c \uc218\ud558\ub85c \uc77c\ud558\ub2e4\uac00 \ub9ac\uac00\ub97c \uc810\ub839\uc911\uc778 \uc81c4\uae30\ubcd1\uc0ac\ub2e8 \ucc38\ubaa8\uc7a5\uc73c\ub85c \uc784\uba85\ubc1b\uc544 \uadf8\ub9ac\ub85c \uc62e\uaca8\uac14\ub2e4. 1918\ub144 3\uc6d4 \ube0c\ub808\uc2a4\ud2b8-\ub9ac\ud1a0\ud504\uc2a4\ud06c \uc870\uc57d\uc73c\ub85c \ub3d9\ubd80\uc804\uc120\uc774 \ub3c5\uc77c\uc758 \uc2b9\ub9ac\ub85c \ub05d\ub098\uc790 \ub9cc\uc288\ud0c0\uc778\uc758 \ubd80\ub300\ub294 \ub3d9\ubd80\uc804\uc120\uc5d0 \ub354 \uba38\ubb34\ub97c \ud544\uc694\uac00 \uc5c6\uc5b4\uc84c\uace0, \ub9cc\uc288\ud0c0\uc778\uc740 \ub2e4\uc2dc \ub7ad\uc2a4 \uadfc\uad50\uc758 \uc81c213\ubcf4\ubcd1\uc0ac\ub2e8\uc73c\ub85c \uac70\ucc98\ub97c \uc62e\uacbc\ub2e4. \ub3c5\uc77c\uad70\uc740 \uc11c\ubd80\uc804\uc120\uc5d0\uc11c \uc77c\ubd80 \uc131\uacfc\ub97c \uac70\ub450\uc5c8\uc73c\ub098 \uacb0\uad6d \uc804\uc7c1\uc5d0\uc11c \uc84c\uace0, 1918\ub144 11\uc6d4 11\uc77c \ud734\uc804\ud611\uc815\uc774 \uc870\uc778(1918\ub144 11\uc6d4 11\uc77c \ud734\uc804)\ub41c\ub2e4.", "answer": "\ud78c\ub374\ubd80\ub974\ud06c", "sentence": "\uc601\uad6d\uad70\uacfc \ud504\ub791\uc2a4\uad70\uc740 1916\ub144 7\uc6d4\uc5d0\uc11c 11\uc6d4\uc5d0 \uac78\uccd0 \uacf5\uc138\ub97c \ud3bc\uccd0 \uaca8\uc6b8\ucbe4\uc5d0\ub294 \ub3c5\uc77c\uad70\uc744 \ubca0\ub974\ub42d-\ub791\uc2a4 \uc0ac\uc774\uc5d0 \uad6c\ucd95\ub41c \ubc29\uc5b4\uc120\uc778 \ud78c\ub374\ubd80\ub974\ud06c \uc120\uae4c\uc9c0 \ubc00\uc5b4\ubd99\uc600\ub2e4.", "paragraph_sentence": "1915\ub144 6\uc6d4 17\uc77c \ub9cc\uc288\ud0c0\uc778\uc740 \ubcf5\uadc0\ud558\uc5ec \ub9c9\uc2a4 \ud3f0 \uac08\ube44\uce20\uac00 \uc9c0\ud718\ud558\ub294 \uc81c10\uad70 \uc791\uc804\ucc38\ubaa8\uc7a5\uad50\uac00 \ub418\uc5c8\ub2e4. \uc5bc\ub9c8 \ub4a4 \ub300\uc704\ub85c \uc2b9\uc9c4\ud55c \ub9cc\uc288\ud0c0\uc778\uc740 \uc81c10\uad70\uc774 \ud3f4\ub780\ub4dc, \ub9ac\ud22c\uc544\ub2c8\uc544, \ubaac\ud14c\ub124\uadf8\ub85c, \uc54c\ubc14\ub2c8\uc544\ub97c \uc131\uacf5\uc801\uc73c\ub85c \uacf5\uaca9\ud558\ub294 \uacfc\uc815\uc5d0\uc11c \uacf5\uc138\uc791\uc804 \uacc4\ud68d \ubc0f \uc218\ud589\uc744 \uacbd\ud5d8\uc73c\ub85c \uccb4\ub4dd\ud588\ub2e4. 1916\ub144 \ucd08 \ubca0\ub974\ub42d\uc5d0\uc11c \uacf5\uc138\uc791\uc804\uc744 \uc218\ud589\ud558\uace0 \uadf8 \ub4a4\uc5d0\ub294 \uc19c \uac15 \uadfc\ucc98\uc758 \uc0ac\ub839\ubd80\uc5d0\uc11c \ud504\ub9ac\uce20 \ud3f0 \ubca8\ub85c\ube0c \uc7a5\uad70\uacfc \ud504\ub9ac\uce20 \ud3f0 \ub85c\uc2a4\ubca0\ub974\ud06c \ucc38\ubaa8\uc7a5 \ud718\ud558\uc5d0 \ubcf4\uae09\uc7a5\uad50\ub85c \uc77c\ud588\ub2e4. \uc774 \uc77c\ub300\ub294 1\ucc28 \ub300\uc804 \ub0b4\ub0b4 \uac00\uc7a5 \uaca9\ub82c\ud55c \uc804\ud22c\uac00 \ubc8c\uc5b4\uc9c4 \uc804\uc7a5\uc774\uc5c8\ub2e4. \uc601\uad6d\uad70\uacfc \ud504\ub791\uc2a4\uad70\uc740 1916\ub144 7\uc6d4\uc5d0\uc11c 11\uc6d4\uc5d0 \uac78\uccd0 \uacf5\uc138\ub97c \ud3bc\uccd0 \uaca8\uc6b8\ucbe4\uc5d0\ub294 \ub3c5\uc77c\uad70\uc744 \ubca0\ub974\ub42d-\ub791\uc2a4 \uc0ac\uc774\uc5d0 \uad6c\ucd95\ub41c \ubc29\uc5b4\uc120\uc778 \ud78c\ub374\ubd80\ub974\ud06c \uc120\uae4c\uc9c0 \ubc00\uc5b4\ubd99\uc600\ub2e4. \ub9cc\uc288\ud0c0\uc778 \ub300\uc704\ub294 1917\ub144 10\uc6d4\uae4c\uc9c0 \uacc4\uc18d \ubca8\ub85c\ube0c \uc218\ud558\ub85c \uc77c\ud558\ub2e4\uac00 \ub9ac\uac00\ub97c \uc810\ub839\uc911\uc778 \uc81c4\uae30\ubcd1\uc0ac\ub2e8 \ucc38\ubaa8\uc7a5\uc73c\ub85c \uc784\uba85\ubc1b\uc544 \uadf8\ub9ac\ub85c \uc62e\uaca8\uac14\ub2e4. 1918\ub144 3\uc6d4 \ube0c\ub808\uc2a4\ud2b8-\ub9ac\ud1a0\ud504\uc2a4\ud06c \uc870\uc57d\uc73c\ub85c \ub3d9\ubd80\uc804\uc120\uc774 \ub3c5\uc77c\uc758 \uc2b9\ub9ac\ub85c \ub05d\ub098\uc790 \ub9cc\uc288\ud0c0\uc778\uc758 \ubd80\ub300\ub294 \ub3d9\ubd80\uc804\uc120\uc5d0 \ub354 \uba38\ubb34\ub97c \ud544\uc694\uac00 \uc5c6\uc5b4\uc84c\uace0, \ub9cc\uc288\ud0c0\uc778\uc740 \ub2e4\uc2dc \ub7ad\uc2a4 \uadfc\uad50\uc758 \uc81c213\ubcf4\ubcd1\uc0ac\ub2e8\uc73c\ub85c \uac70\ucc98\ub97c \uc62e\uacbc\ub2e4. \ub3c5\uc77c\uad70\uc740 \uc11c\ubd80\uc804\uc120\uc5d0\uc11c \uc77c\ubd80 \uc131\uacfc\ub97c \uac70\ub450\uc5c8\uc73c\ub098 \uacb0\uad6d \uc804\uc7c1\uc5d0\uc11c \uc84c\uace0, 1918\ub144 11\uc6d4 11\uc77c \ud734\uc804\ud611\uc815\uc774 \uc870\uc778(1918\ub144 11\uc6d4 11\uc77c \ud734\uc804)\ub41c\ub2e4.", "paragraph_answer": "1915\ub144 6\uc6d4 17\uc77c \ub9cc\uc288\ud0c0\uc778\uc740 \ubcf5\uadc0\ud558\uc5ec \ub9c9\uc2a4 \ud3f0 \uac08\ube44\uce20\uac00 \uc9c0\ud718\ud558\ub294 \uc81c10\uad70 \uc791\uc804\ucc38\ubaa8\uc7a5\uad50\uac00 \ub418\uc5c8\ub2e4. \uc5bc\ub9c8 \ub4a4 \ub300\uc704\ub85c \uc2b9\uc9c4\ud55c \ub9cc\uc288\ud0c0\uc778\uc740 \uc81c10\uad70\uc774 \ud3f4\ub780\ub4dc, \ub9ac\ud22c\uc544\ub2c8\uc544, \ubaac\ud14c\ub124\uadf8\ub85c, \uc54c\ubc14\ub2c8\uc544\ub97c \uc131\uacf5\uc801\uc73c\ub85c \uacf5\uaca9\ud558\ub294 \uacfc\uc815\uc5d0\uc11c \uacf5\uc138\uc791\uc804 \uacc4\ud68d \ubc0f \uc218\ud589\uc744 \uacbd\ud5d8\uc73c\ub85c \uccb4\ub4dd\ud588\ub2e4. 1916\ub144 \ucd08 \ubca0\ub974\ub42d\uc5d0\uc11c \uacf5\uc138\uc791\uc804\uc744 \uc218\ud589\ud558\uace0 \uadf8 \ub4a4\uc5d0\ub294 \uc19c \uac15 \uadfc\ucc98\uc758 \uc0ac\ub839\ubd80\uc5d0\uc11c \ud504\ub9ac\uce20 \ud3f0 \ubca8\ub85c\ube0c \uc7a5\uad70\uacfc \ud504\ub9ac\uce20 \ud3f0 \ub85c\uc2a4\ubca0\ub974\ud06c \ucc38\ubaa8\uc7a5 \ud718\ud558\uc5d0 \ubcf4\uae09\uc7a5\uad50\ub85c \uc77c\ud588\ub2e4. \uc774 \uc77c\ub300\ub294 1\ucc28 \ub300\uc804 \ub0b4\ub0b4 \uac00\uc7a5 \uaca9\ub82c\ud55c \uc804\ud22c\uac00 \ubc8c\uc5b4\uc9c4 \uc804\uc7a5\uc774\uc5c8\ub2e4. \uc601\uad6d\uad70\uacfc \ud504\ub791\uc2a4\uad70\uc740 1916\ub144 7\uc6d4\uc5d0\uc11c 11\uc6d4\uc5d0 \uac78\uccd0 \uacf5\uc138\ub97c \ud3bc\uccd0 \uaca8\uc6b8\ucbe4\uc5d0\ub294 \ub3c5\uc77c\uad70\uc744 \ubca0\ub974\ub42d-\ub791\uc2a4 \uc0ac\uc774\uc5d0 \uad6c\ucd95\ub41c \ubc29\uc5b4\uc120\uc778 \ud78c\ub374\ubd80\ub974\ud06c \uc120\uae4c\uc9c0 \ubc00\uc5b4\ubd99\uc600\ub2e4. \ub9cc\uc288\ud0c0\uc778 \ub300\uc704\ub294 1917\ub144 10\uc6d4\uae4c\uc9c0 \uacc4\uc18d \ubca8\ub85c\ube0c \uc218\ud558\ub85c \uc77c\ud558\ub2e4\uac00 \ub9ac\uac00\ub97c \uc810\ub839\uc911\uc778 \uc81c4\uae30\ubcd1\uc0ac\ub2e8 \ucc38\ubaa8\uc7a5\uc73c\ub85c \uc784\uba85\ubc1b\uc544 \uadf8\ub9ac\ub85c \uc62e\uaca8\uac14\ub2e4. 1918\ub144 3\uc6d4 \ube0c\ub808\uc2a4\ud2b8-\ub9ac\ud1a0\ud504\uc2a4\ud06c \uc870\uc57d\uc73c\ub85c \ub3d9\ubd80\uc804\uc120\uc774 \ub3c5\uc77c\uc758 \uc2b9\ub9ac\ub85c \ub05d\ub098\uc790 \ub9cc\uc288\ud0c0\uc778\uc758 \ubd80\ub300\ub294 \ub3d9\ubd80\uc804\uc120\uc5d0 \ub354 \uba38\ubb34\ub97c \ud544\uc694\uac00 \uc5c6\uc5b4\uc84c\uace0, \ub9cc\uc288\ud0c0\uc778\uc740 \ub2e4\uc2dc \ub7ad\uc2a4 \uadfc\uad50\uc758 \uc81c213\ubcf4\ubcd1\uc0ac\ub2e8\uc73c\ub85c \uac70\ucc98\ub97c \uc62e\uacbc\ub2e4. \ub3c5\uc77c\uad70\uc740 \uc11c\ubd80\uc804\uc120\uc5d0\uc11c \uc77c\ubd80 \uc131\uacfc\ub97c \uac70\ub450\uc5c8\uc73c\ub098 \uacb0\uad6d \uc804\uc7c1\uc5d0\uc11c \uc84c\uace0, 1918\ub144 11\uc6d4 11\uc77c \ud734\uc804\ud611\uc815\uc774 \uc870\uc778(1918\ub144 11\uc6d4 11\uc77c \ud734\uc804)\ub41c\ub2e4.", "sentence_answer": "\uc601\uad6d\uad70\uacfc \ud504\ub791\uc2a4\uad70\uc740 1916\ub144 7\uc6d4\uc5d0\uc11c 11\uc6d4\uc5d0 \uac78\uccd0 \uacf5\uc138\ub97c \ud3bc\uccd0 \uaca8\uc6b8\ucbe4\uc5d0\ub294 \ub3c5\uc77c\uad70\uc744 \ubca0\ub974\ub42d-\ub791\uc2a4 \uc0ac\uc774\uc5d0 \uad6c\ucd95\ub41c \ubc29\uc5b4\uc120\uc778 \ud78c\ub374\ubd80\ub974\ud06c \uc120\uae4c\uc9c0 \ubc00\uc5b4\ubd99\uc600\ub2e4.", "paragraph_id": "6540644-4-1", "paragraph_question": "question: \uc601\uad6d\uad70\uacfc \ud504\ub791\uc2a4\uad70\uc774 1916\ub144 7\uc6d4\uc5d0\uc11c 11\uc6d4\uc5d0 \uac78\uccd0 \ub3c5\uc77c\uad70\uc744 \ubab0\uc544\ubd99\uc5ec \uaca8\uc6b8\uc5d4 \uc5b4\ub290 \ubc29\uc5b4\uc120\uae4c\uc9c0 \ubc00\uc5b4\ub0c8\ub098?, context: 1915\ub144 6\uc6d4 17\uc77c \ub9cc\uc288\ud0c0\uc778\uc740 \ubcf5\uadc0\ud558\uc5ec \ub9c9\uc2a4 \ud3f0 \uac08\ube44\uce20\uac00 \uc9c0\ud718\ud558\ub294 \uc81c10\uad70 \uc791\uc804\ucc38\ubaa8\uc7a5\uad50\uac00 \ub418\uc5c8\ub2e4. \uc5bc\ub9c8 \ub4a4 \ub300\uc704\ub85c \uc2b9\uc9c4\ud55c \ub9cc\uc288\ud0c0\uc778\uc740 \uc81c10\uad70\uc774 \ud3f4\ub780\ub4dc, \ub9ac\ud22c\uc544\ub2c8\uc544, \ubaac\ud14c\ub124\uadf8\ub85c, \uc54c\ubc14\ub2c8\uc544\ub97c \uc131\uacf5\uc801\uc73c\ub85c \uacf5\uaca9\ud558\ub294 \uacfc\uc815\uc5d0\uc11c \uacf5\uc138\uc791\uc804 \uacc4\ud68d \ubc0f \uc218\ud589\uc744 \uacbd\ud5d8\uc73c\ub85c \uccb4\ub4dd\ud588\ub2e4. 1916\ub144 \ucd08 \ubca0\ub974\ub42d\uc5d0\uc11c \uacf5\uc138\uc791\uc804\uc744 \uc218\ud589\ud558\uace0 \uadf8 \ub4a4\uc5d0\ub294 \uc19c \uac15 \uadfc\ucc98\uc758 \uc0ac\ub839\ubd80\uc5d0\uc11c \ud504\ub9ac\uce20 \ud3f0 \ubca8\ub85c\ube0c \uc7a5\uad70\uacfc \ud504\ub9ac\uce20 \ud3f0 \ub85c\uc2a4\ubca0\ub974\ud06c \ucc38\ubaa8\uc7a5 \ud718\ud558\uc5d0 \ubcf4\uae09\uc7a5\uad50\ub85c \uc77c\ud588\ub2e4. \uc774 \uc77c\ub300\ub294 1\ucc28 \ub300\uc804 \ub0b4\ub0b4 \uac00\uc7a5 \uaca9\ub82c\ud55c \uc804\ud22c\uac00 \ubc8c\uc5b4\uc9c4 \uc804\uc7a5\uc774\uc5c8\ub2e4. \uc601\uad6d\uad70\uacfc \ud504\ub791\uc2a4\uad70\uc740 1916\ub144 7\uc6d4\uc5d0\uc11c 11\uc6d4\uc5d0 \uac78\uccd0 \uacf5\uc138\ub97c \ud3bc\uccd0 \uaca8\uc6b8\ucbe4\uc5d0\ub294 \ub3c5\uc77c\uad70\uc744 \ubca0\ub974\ub42d-\ub791\uc2a4 \uc0ac\uc774\uc5d0 \uad6c\ucd95\ub41c \ubc29\uc5b4\uc120\uc778 \ud78c\ub374\ubd80\ub974\ud06c \uc120\uae4c\uc9c0 \ubc00\uc5b4\ubd99\uc600\ub2e4. \ub9cc\uc288\ud0c0\uc778 \ub300\uc704\ub294 1917\ub144 10\uc6d4\uae4c\uc9c0 \uacc4\uc18d \ubca8\ub85c\ube0c \uc218\ud558\ub85c \uc77c\ud558\ub2e4\uac00 \ub9ac\uac00\ub97c \uc810\ub839\uc911\uc778 \uc81c4\uae30\ubcd1\uc0ac\ub2e8 \ucc38\ubaa8\uc7a5\uc73c\ub85c \uc784\uba85\ubc1b\uc544 \uadf8\ub9ac\ub85c \uc62e\uaca8\uac14\ub2e4. 1918\ub144 3\uc6d4 \ube0c\ub808\uc2a4\ud2b8-\ub9ac\ud1a0\ud504\uc2a4\ud06c \uc870\uc57d\uc73c\ub85c \ub3d9\ubd80\uc804\uc120\uc774 \ub3c5\uc77c\uc758 \uc2b9\ub9ac\ub85c \ub05d\ub098\uc790 \ub9cc\uc288\ud0c0\uc778\uc758 \ubd80\ub300\ub294 \ub3d9\ubd80\uc804\uc120\uc5d0 \ub354 \uba38\ubb34\ub97c \ud544\uc694\uac00 \uc5c6\uc5b4\uc84c\uace0, \ub9cc\uc288\ud0c0\uc778\uc740 \ub2e4\uc2dc \ub7ad\uc2a4 \uadfc\uad50\uc758 \uc81c213\ubcf4\ubcd1\uc0ac\ub2e8\uc73c\ub85c \uac70\ucc98\ub97c \uc62e\uacbc\ub2e4. \ub3c5\uc77c\uad70\uc740 \uc11c\ubd80\uc804\uc120\uc5d0\uc11c \uc77c\ubd80 \uc131\uacfc\ub97c \uac70\ub450\uc5c8\uc73c\ub098 \uacb0\uad6d \uc804\uc7c1\uc5d0\uc11c \uc84c\uace0, 1918\ub144 11\uc6d4 11\uc77c \ud734\uc804\ud611\uc815\uc774 \uc870\uc778(1918\ub144 11\uc6d4 11\uc77c \ud734\uc804)\ub41c\ub2e4."} {"question": "\uc639\uc815\uc81c \ud6c4\uc5d0 \uc989\uc704\ud55c \uc0ac\ub78c\uc758 \uc774\ub984\uc740?", "paragraph": "\uc639\uc815\uc81c\uc758 \ub4a4\ub97c \uc774\uc5b4 \uc989\uc704\ud55c \uac74\ub96d\uc81c\ub294 \uad81\uc815 \ud68c\ud654\uc758 \ubc1c\uc804\uc744 \uc911\uc2dc\ud558\uc600\uace0, \uac15\ud76c\uc81c \ub54c\ubd80\ud130 \uad81\uc815\uc5d0 \ub4e4\uc5b4\uc640 \ud65c\uc57d\ud55c \uce74\uc2a4\ud2f8\ub9ac\uc624\ub124\ub97c \uc911\uc6a9\ud558\uc5ec\uc11c \uad81\uc815 \ud654\uac00 \uc911\uc5d0\uc11c \uac78\ucd9c\ud55c \uc778\ubb3c\ub85c \uc131\uacf5\ud558\uac8c \ub41c\ub2e4. \uac74\ub96d\uc81c\ub294 \ub9e4\uc77c \uadf8\uc758 \ud654\uc2e4\uc5d0 \uac00\uc11c \uc791\uc5c5\ud558\ub294 \ubaa8\uc2b5\uc744 \ubcf4\uc558\ub2e4. \ud504\ub791\uc2a4 \uc120\uad50\uc0ac \uc544\ubbf8\uc624(Jean-Joseph Marie Amiot, \uc911\uad6d\uba85: \u9322\u5fb7\u660e, 1718\ub144 ~ 1793\ub144)\ub294 \u201c\uc120\uad50\uc0ac\ub4e4\uc774 \ubca0\uc774\uc9d5\uc73c\ub85c \uc628 \uc774\ub798 \uac74\ub96d\uc81c\ub9cc\ud07c \uc120\uad50\uc0ac\ub4e4\uc758 \ubcf5\ubb34\ub97c \uc774\uc6a9\ud55c \ud669\uc81c\ub294 \uc5c6\uc5c8\uc73c\uba70 \uac74\ub96d\uc81c\ub9cc\ud07c \uc120\uad50\uc0ac\ub4e4\uc744 \ud639\uc0ac\uc2dc\ud0a8 \ud669\uc81c\ub3c4 \uc5c6\uc5c8\ub2e4\u201d\uace0 \ud3c9\ud55c \ubc14 \uc788\ub2e4. 1736\ub144(\uac74\ub96d \uc6d0\ub144)\uc5d0 \uac74\ub96d\uc81c\uc640 \ud669\ud6c4\uc778 \ud6a8\ud604\uc21c\ud669\ud6c4, \uadf8\ub9ac\uace0 11\uba85\uc758 \ud6c4\uad81\ub4e4\uc758 \ucd08\uc0c1\ud654\uc778 \u300a\uc2ec\uc0ac\uce58\ud3c9\ub3c4\u300b(\u5fc3\u5beb\u6cbb\u5e73\u5716)\ub97c \uadf8\ub9ac\uac8c \ub41c\ub2e4. \uadf8\uac00 \uadf8\ub9b0 \uc774\ubc31\uc5ec \ud3ed\uc758 \uc778\ubb3c \ucd08\uc0c1 \uc911\uc5d0\uc11c \uc720\uba85\ud55c \uc791\ud488 \uc911\uc758 \ud558\ub098\uc774\ub2e4. \uac74\ub96d\uc81c\uc758 \uba85\uc5d0 \uc758\ud574\uc11c \uc6d0\uba85\uc6d0\uc5d0 1747\ub144(\uac74\ub96d 12\ub144)\ubd80\ud130 1759\ub144(\uac74\ub96d 24\ub144)\uae4c\uc9c0 \ubca0\ub974\uc0ac\uc720 \uad81\uc804\uc744 \ubaa8\ubc29\ud55c \uac74\ubb3c\ub4e4\uc744 \uc124\uacc4\ud558\uace0 \uc2dc\uacf5\ud558\ub294\ub370 \ucc38\uc5ec\ud558\uc600\ub2e4. \uc774\ubb34\ub835 \uadf8\ub294 \uc6d0\uba85\uc6d0 \uacf5\uc0ac\uc640 \uad00\ub828\ud558\uc5ec \ud669\uc2e4 \uc6d0\ub9bc\uc744 \uad00\uc7a5\ud558\ub294 \uc9c1\ubb34\uc778 \uc8153\ud488 \ubd09\uc2e0\uc6d0\uacbd(\u5949\u5bb8\u82d1\u537f)\uc5d0 \uc624\ub978\ub2e4.", "answer": "\uac74\ub96d\uc81c", "sentence": "\uc639\uc815\uc81c\uc758 \ub4a4\ub97c \uc774\uc5b4 \uc989\uc704\ud55c \uac74\ub96d\uc81c \ub294 \uad81\uc815 \ud68c\ud654\uc758 \ubc1c\uc804\uc744 \uc911\uc2dc\ud558\uc600\uace0, \uac15\ud76c\uc81c \ub54c\ubd80\ud130 \uad81\uc815\uc5d0 \ub4e4\uc5b4\uc640 \ud65c\uc57d\ud55c \uce74\uc2a4\ud2f8\ub9ac\uc624\ub124\ub97c \uc911\uc6a9\ud558\uc5ec\uc11c \uad81\uc815 \ud654\uac00 \uc911\uc5d0\uc11c \uac78\ucd9c\ud55c \uc778\ubb3c\ub85c \uc131\uacf5\ud558\uac8c \ub41c\ub2e4.", "paragraph_sentence": " \uc639\uc815\uc81c\uc758 \ub4a4\ub97c \uc774\uc5b4 \uc989\uc704\ud55c \uac74\ub96d\uc81c \ub294 \uad81\uc815 \ud68c\ud654\uc758 \ubc1c\uc804\uc744 \uc911\uc2dc\ud558\uc600\uace0, \uac15\ud76c\uc81c \ub54c\ubd80\ud130 \uad81\uc815\uc5d0 \ub4e4\uc5b4\uc640 \ud65c\uc57d\ud55c \uce74\uc2a4\ud2f8\ub9ac\uc624\ub124\ub97c \uc911\uc6a9\ud558\uc5ec\uc11c \uad81\uc815 \ud654\uac00 \uc911\uc5d0\uc11c \uac78\ucd9c\ud55c \uc778\ubb3c\ub85c \uc131\uacf5\ud558\uac8c \ub41c\ub2e4. \uac74\ub96d\uc81c\ub294 \ub9e4\uc77c \uadf8\uc758 \ud654\uc2e4\uc5d0 \uac00\uc11c \uc791\uc5c5\ud558\ub294 \ubaa8\uc2b5\uc744 \ubcf4\uc558\ub2e4. \ud504\ub791\uc2a4 \uc120\uad50\uc0ac \uc544\ubbf8\uc624(Jean-Joseph Marie Amiot, \uc911\uad6d\uba85: \u9322\u5fb7\u660e, 1718\ub144 ~ 1793\ub144)\ub294 \u201c\uc120\uad50\uc0ac\ub4e4\uc774 \ubca0\uc774\uc9d5\uc73c\ub85c \uc628 \uc774\ub798 \uac74\ub96d\uc81c\ub9cc\ud07c \uc120\uad50\uc0ac\ub4e4\uc758 \ubcf5\ubb34\ub97c \uc774\uc6a9\ud55c \ud669\uc81c\ub294 \uc5c6\uc5c8\uc73c\uba70 \uac74\ub96d\uc81c\ub9cc\ud07c \uc120\uad50\uc0ac\ub4e4\uc744 \ud639\uc0ac\uc2dc\ud0a8 \ud669\uc81c\ub3c4 \uc5c6\uc5c8\ub2e4\u201d\uace0 \ud3c9\ud55c \ubc14 \uc788\ub2e4. 1736\ub144(\uac74\ub96d \uc6d0\ub144)\uc5d0 \uac74\ub96d\uc81c\uc640 \ud669\ud6c4\uc778 \ud6a8\ud604\uc21c\ud669\ud6c4, \uadf8\ub9ac\uace0 11\uba85\uc758 \ud6c4\uad81\ub4e4\uc758 \ucd08\uc0c1\ud654\uc778 \u300a\uc2ec\uc0ac\uce58\ud3c9\ub3c4\u300b(\u5fc3\u5beb\u6cbb\u5e73\u5716)\ub97c \uadf8\ub9ac\uac8c \ub41c\ub2e4. \uadf8\uac00 \uadf8\ub9b0 \uc774\ubc31\uc5ec \ud3ed\uc758 \uc778\ubb3c \ucd08\uc0c1 \uc911\uc5d0\uc11c \uc720\uba85\ud55c \uc791\ud488 \uc911\uc758 \ud558\ub098\uc774\ub2e4. \uac74\ub96d\uc81c\uc758 \uba85\uc5d0 \uc758\ud574\uc11c \uc6d0\uba85\uc6d0\uc5d0 1747\ub144(\uac74\ub96d 12\ub144)\ubd80\ud130 1759\ub144(\uac74\ub96d 24\ub144)\uae4c\uc9c0 \ubca0\ub974\uc0ac\uc720 \uad81\uc804\uc744 \ubaa8\ubc29\ud55c \uac74\ubb3c\ub4e4\uc744 \uc124\uacc4\ud558\uace0 \uc2dc\uacf5\ud558\ub294\ub370 \ucc38\uc5ec\ud558\uc600\ub2e4. \uc774\ubb34\ub835 \uadf8\ub294 \uc6d0\uba85\uc6d0 \uacf5\uc0ac\uc640 \uad00\ub828\ud558\uc5ec \ud669\uc2e4 \uc6d0\ub9bc\uc744 \uad00\uc7a5\ud558\ub294 \uc9c1\ubb34\uc778 \uc8153\ud488 \ubd09\uc2e0\uc6d0\uacbd(\u5949\u5bb8\u82d1\u537f)\uc5d0 \uc624\ub978\ub2e4.", "paragraph_answer": "\uc639\uc815\uc81c\uc758 \ub4a4\ub97c \uc774\uc5b4 \uc989\uc704\ud55c \uac74\ub96d\uc81c \ub294 \uad81\uc815 \ud68c\ud654\uc758 \ubc1c\uc804\uc744 \uc911\uc2dc\ud558\uc600\uace0, \uac15\ud76c\uc81c \ub54c\ubd80\ud130 \uad81\uc815\uc5d0 \ub4e4\uc5b4\uc640 \ud65c\uc57d\ud55c \uce74\uc2a4\ud2f8\ub9ac\uc624\ub124\ub97c \uc911\uc6a9\ud558\uc5ec\uc11c \uad81\uc815 \ud654\uac00 \uc911\uc5d0\uc11c \uac78\ucd9c\ud55c \uc778\ubb3c\ub85c \uc131\uacf5\ud558\uac8c \ub41c\ub2e4. \uac74\ub96d\uc81c\ub294 \ub9e4\uc77c \uadf8\uc758 \ud654\uc2e4\uc5d0 \uac00\uc11c \uc791\uc5c5\ud558\ub294 \ubaa8\uc2b5\uc744 \ubcf4\uc558\ub2e4. \ud504\ub791\uc2a4 \uc120\uad50\uc0ac \uc544\ubbf8\uc624(Jean-Joseph Marie Amiot, \uc911\uad6d\uba85: \u9322\u5fb7\u660e, 1718\ub144 ~ 1793\ub144)\ub294 \u201c\uc120\uad50\uc0ac\ub4e4\uc774 \ubca0\uc774\uc9d5\uc73c\ub85c \uc628 \uc774\ub798 \uac74\ub96d\uc81c\ub9cc\ud07c \uc120\uad50\uc0ac\ub4e4\uc758 \ubcf5\ubb34\ub97c \uc774\uc6a9\ud55c \ud669\uc81c\ub294 \uc5c6\uc5c8\uc73c\uba70 \uac74\ub96d\uc81c\ub9cc\ud07c \uc120\uad50\uc0ac\ub4e4\uc744 \ud639\uc0ac\uc2dc\ud0a8 \ud669\uc81c\ub3c4 \uc5c6\uc5c8\ub2e4\u201d\uace0 \ud3c9\ud55c \ubc14 \uc788\ub2e4. 1736\ub144(\uac74\ub96d \uc6d0\ub144)\uc5d0 \uac74\ub96d\uc81c\uc640 \ud669\ud6c4\uc778 \ud6a8\ud604\uc21c\ud669\ud6c4, \uadf8\ub9ac\uace0 11\uba85\uc758 \ud6c4\uad81\ub4e4\uc758 \ucd08\uc0c1\ud654\uc778 \u300a\uc2ec\uc0ac\uce58\ud3c9\ub3c4\u300b(\u5fc3\u5beb\u6cbb\u5e73\u5716)\ub97c \uadf8\ub9ac\uac8c \ub41c\ub2e4. \uadf8\uac00 \uadf8\ub9b0 \uc774\ubc31\uc5ec \ud3ed\uc758 \uc778\ubb3c \ucd08\uc0c1 \uc911\uc5d0\uc11c \uc720\uba85\ud55c \uc791\ud488 \uc911\uc758 \ud558\ub098\uc774\ub2e4. \uac74\ub96d\uc81c\uc758 \uba85\uc5d0 \uc758\ud574\uc11c \uc6d0\uba85\uc6d0\uc5d0 1747\ub144(\uac74\ub96d 12\ub144)\ubd80\ud130 1759\ub144(\uac74\ub96d 24\ub144)\uae4c\uc9c0 \ubca0\ub974\uc0ac\uc720 \uad81\uc804\uc744 \ubaa8\ubc29\ud55c \uac74\ubb3c\ub4e4\uc744 \uc124\uacc4\ud558\uace0 \uc2dc\uacf5\ud558\ub294\ub370 \ucc38\uc5ec\ud558\uc600\ub2e4. \uc774\ubb34\ub835 \uadf8\ub294 \uc6d0\uba85\uc6d0 \uacf5\uc0ac\uc640 \uad00\ub828\ud558\uc5ec \ud669\uc2e4 \uc6d0\ub9bc\uc744 \uad00\uc7a5\ud558\ub294 \uc9c1\ubb34\uc778 \uc8153\ud488 \ubd09\uc2e0\uc6d0\uacbd(\u5949\u5bb8\u82d1\u537f)\uc5d0 \uc624\ub978\ub2e4.", "sentence_answer": "\uc639\uc815\uc81c\uc758 \ub4a4\ub97c \uc774\uc5b4 \uc989\uc704\ud55c \uac74\ub96d\uc81c \ub294 \uad81\uc815 \ud68c\ud654\uc758 \ubc1c\uc804\uc744 \uc911\uc2dc\ud558\uc600\uace0, \uac15\ud76c\uc81c \ub54c\ubd80\ud130 \uad81\uc815\uc5d0 \ub4e4\uc5b4\uc640 \ud65c\uc57d\ud55c \uce74\uc2a4\ud2f8\ub9ac\uc624\ub124\ub97c \uc911\uc6a9\ud558\uc5ec\uc11c \uad81\uc815 \ud654\uac00 \uc911\uc5d0\uc11c \uac78\ucd9c\ud55c \uc778\ubb3c\ub85c \uc131\uacf5\ud558\uac8c \ub41c\ub2e4.", "paragraph_id": "6555956-2-0", "paragraph_question": "question: \uc639\uc815\uc81c \ud6c4\uc5d0 \uc989\uc704\ud55c \uc0ac\ub78c\uc758 \uc774\ub984\uc740?, context: \uc639\uc815\uc81c\uc758 \ub4a4\ub97c \uc774\uc5b4 \uc989\uc704\ud55c \uac74\ub96d\uc81c\ub294 \uad81\uc815 \ud68c\ud654\uc758 \ubc1c\uc804\uc744 \uc911\uc2dc\ud558\uc600\uace0, \uac15\ud76c\uc81c \ub54c\ubd80\ud130 \uad81\uc815\uc5d0 \ub4e4\uc5b4\uc640 \ud65c\uc57d\ud55c \uce74\uc2a4\ud2f8\ub9ac\uc624\ub124\ub97c \uc911\uc6a9\ud558\uc5ec\uc11c \uad81\uc815 \ud654\uac00 \uc911\uc5d0\uc11c \uac78\ucd9c\ud55c \uc778\ubb3c\ub85c \uc131\uacf5\ud558\uac8c \ub41c\ub2e4. \uac74\ub96d\uc81c\ub294 \ub9e4\uc77c \uadf8\uc758 \ud654\uc2e4\uc5d0 \uac00\uc11c \uc791\uc5c5\ud558\ub294 \ubaa8\uc2b5\uc744 \ubcf4\uc558\ub2e4. \ud504\ub791\uc2a4 \uc120\uad50\uc0ac \uc544\ubbf8\uc624(Jean-Joseph Marie Amiot, \uc911\uad6d\uba85: \u9322\u5fb7\u660e, 1718\ub144 ~ 1793\ub144)\ub294 \u201c\uc120\uad50\uc0ac\ub4e4\uc774 \ubca0\uc774\uc9d5\uc73c\ub85c \uc628 \uc774\ub798 \uac74\ub96d\uc81c\ub9cc\ud07c \uc120\uad50\uc0ac\ub4e4\uc758 \ubcf5\ubb34\ub97c \uc774\uc6a9\ud55c \ud669\uc81c\ub294 \uc5c6\uc5c8\uc73c\uba70 \uac74\ub96d\uc81c\ub9cc\ud07c \uc120\uad50\uc0ac\ub4e4\uc744 \ud639\uc0ac\uc2dc\ud0a8 \ud669\uc81c\ub3c4 \uc5c6\uc5c8\ub2e4\u201d\uace0 \ud3c9\ud55c \ubc14 \uc788\ub2e4. 1736\ub144(\uac74\ub96d \uc6d0\ub144)\uc5d0 \uac74\ub96d\uc81c\uc640 \ud669\ud6c4\uc778 \ud6a8\ud604\uc21c\ud669\ud6c4, \uadf8\ub9ac\uace0 11\uba85\uc758 \ud6c4\uad81\ub4e4\uc758 \ucd08\uc0c1\ud654\uc778 \u300a\uc2ec\uc0ac\uce58\ud3c9\ub3c4\u300b(\u5fc3\u5beb\u6cbb\u5e73\u5716)\ub97c \uadf8\ub9ac\uac8c \ub41c\ub2e4. \uadf8\uac00 \uadf8\ub9b0 \uc774\ubc31\uc5ec \ud3ed\uc758 \uc778\ubb3c \ucd08\uc0c1 \uc911\uc5d0\uc11c \uc720\uba85\ud55c \uc791\ud488 \uc911\uc758 \ud558\ub098\uc774\ub2e4. \uac74\ub96d\uc81c\uc758 \uba85\uc5d0 \uc758\ud574\uc11c \uc6d0\uba85\uc6d0\uc5d0 1747\ub144(\uac74\ub96d 12\ub144)\ubd80\ud130 1759\ub144(\uac74\ub96d 24\ub144)\uae4c\uc9c0 \ubca0\ub974\uc0ac\uc720 \uad81\uc804\uc744 \ubaa8\ubc29\ud55c \uac74\ubb3c\ub4e4\uc744 \uc124\uacc4\ud558\uace0 \uc2dc\uacf5\ud558\ub294\ub370 \ucc38\uc5ec\ud558\uc600\ub2e4. \uc774\ubb34\ub835 \uadf8\ub294 \uc6d0\uba85\uc6d0 \uacf5\uc0ac\uc640 \uad00\ub828\ud558\uc5ec \ud669\uc2e4 \uc6d0\ub9bc\uc744 \uad00\uc7a5\ud558\ub294 \uc9c1\ubb34\uc778 \uc8153\ud488 \ubd09\uc2e0\uc6d0\uacbd(\u5949\u5bb8\u82d1\u537f)\uc5d0 \uc624\ub978\ub2e4."} {"question": "\ub9cc\uc288\ud0c0\uc778\uc740 \ub8ec\ud2b8\uc288\ud14c\ud2b8\uc758 \uc791\uc804\ucc38\ubaa8\uc778 \uc5b4\ub290 \ub300\ub839\uacfc \ud568\uaed8 \uc791\uc804\uacc4\ud68d\uc744 \uc218\ub9bd\ud588\ub294\uac00?", "paragraph": "1939\ub144 8\uc6d4 18\uc77c, \ud3f4\ub780\ub4dc\ub97c \uce68\uacf5\ud558\ub294 \ubc31\uc0c9 \uc791\uc804\uc744 \uc900\ube44\ud558\ub358 \ub9cc\uc288\ud0c0\uc778\uc740 \uac8c\ub974\ud2b8 \ud3f0 \ub8ec\ud2b8\uc288\ud14c\ud2b8\uc758 \ub0a8\ubd80 \uc9d1\ub2e8\uad70 \ucc38\ubaa8\uc7a5\uc73c\ub85c \ubd80\uc784\ud55c\ub2e4. \ub9cc\uc288\ud0c0\uc778\uc740 \ub8ec\ud2b8\uc288\ud14c\ud2b8\uc758 \uc791\uc804\ucc38\ubaa8 \uadc4\ud130 \ube14\ub8e8\uba58\ud2b8\ub9ac\ud2b8 \ub300\ub839\uacfc \ud568\uaed8 \uc791\uc804\uacc4\ud68d \uc218\ub9bd\uc5d0 \ucc38\uc5ec\ud588\ub2e4. \ub9cc\uc288\ud0c0\uc778\uc740 \uc9d1\ub2e8\uad70\uc758 \uae30\uac11\ubcd1\ub825 \ub2e4\uc218\ub97c \ubc1c\ud130 \ud3f0 \ub77c\uc774\ud5e4\ub098\uc6b0\uc758 \uc81c10\uad70\uc5d0 \uc9d1\uc911\uc2dc\ucf1c \uc8fc\uba74\uc11c \ub3cc\ud30c\uad6c\ub97c \ub9c8\ub828, \ud3f4\ub780\ub4dc\uad70\uc744 \ube44\uc2a4\ud234\ub77c \uac15 \uc11c\uc548\uc5d0 \ud3ec\uc704\uc2dc\ud0a4\uace0, \ub77c\uc774\ud5e4\ub098\uc6b0\uc758 \uc81c10\uad70\uc774 \uae30\uac11\uae30\ub3d9\uc758 \ud798\uc73c\ub85c \ud3f4\ub780\ub4dc\uc758 \uc218\ub3c4 \ubc14\ub974\uc0e4\ubc14\uae4c\uc9c0 \ub3cc\uc9c4\ud558\ub294 \ub3d9\uc548 \ube4c\ud5ec\ub984 \ub9ac\uc2a4\ud2b8\uc758 \uc81c14\uad70\uacfc \uc694\ud558\ub124\uc2a4 \ube14\ub77c\ucf54\ube44\uce20\uc758 \uc81c8\uad70\uc740 \uce21\uba74\uc9c0\uc6d0\uc744 \ub9e1\ub294\ub2e4\ub294 \uacc4\ud68d\uc744 \uc791\uc131\ud588\uace0, \ub8ec\ud2b8\uc288\ud14c\ud2b8\ub294 \uc774\ub97c \ubc1b\uc544\ub4e4\uc600\ub2e4. \uc0ac\uc2e4 \ub9cc\uc288\ud0c0\uc778\uc740 \uac1c\uc778\uc801\uc73c\ub85c \ud3f4\ub780\ub4dc\ub97c \ub3c5\uc77c\uacfc \uc18c\ub828 \uc0ac\uc774\uc758 \uc644\ucda9\uc9c0\ub300\ub85c \ub0a8\uaca8\ub450\ub294 \ud3b8\uc774 \uc88b\ub2e4\uace0 \uc0dd\uac01\ud588\uae30\uc5d0 \ub300\ud3f4\ub780\ub4dc \uad70\uc0ac \uc791\uc804\uc5d0 \ubbf8\uc628\uc801\uc774\uc5c8\ub2e4. \ubfd0\ub9cc \uc544\ub2c8\ub77c \ud3f4\ub780\ub4dc \uce68\uacf5\uc774 \uc644\ub8cc\ub418\uc9c0 \uc54a\uc558\uc744 \ub54c \uc11c\ucabd\uc5d0\uc11c \uc5f0\ud569\uad70\uc774 \ub4a4\ud1b5\uc218\ub97c \uccd0\uc11c \ub3c5\uc77c\uc774 \uc591\uba74\uc804\uc7c1\uc5d0 \ube60\uc9c0\uac8c \ub418\ub294 \uac83\ub3c4 \uac71\uc815\uac70\ub9ac\uc600\ub2e4.", "answer": "\ube14\ub8e8\uba58\ud2b8\ub9ac\ud2b8 \ub300\ub839", "sentence": "\ub9cc\uc288\ud0c0\uc778\uc740 \ub8ec\ud2b8\uc288\ud14c\ud2b8\uc758 \uc791\uc804\ucc38\ubaa8 \uadc4\ud130 \ube14\ub8e8\uba58\ud2b8\ub9ac\ud2b8 \ub300\ub839 \uacfc \ud568\uaed8 \uc791\uc804\uacc4\ud68d \uc218\ub9bd\uc5d0 \ucc38\uc5ec\ud588\ub2e4.", "paragraph_sentence": "1939\ub144 8\uc6d4 18\uc77c, \ud3f4\ub780\ub4dc\ub97c \uce68\uacf5\ud558\ub294 \ubc31\uc0c9 \uc791\uc804\uc744 \uc900\ube44\ud558\ub358 \ub9cc\uc288\ud0c0\uc778\uc740 \uac8c\ub974\ud2b8 \ud3f0 \ub8ec\ud2b8\uc288\ud14c\ud2b8\uc758 \ub0a8\ubd80 \uc9d1\ub2e8\uad70 \ucc38\ubaa8\uc7a5\uc73c\ub85c \ubd80\uc784\ud55c\ub2e4. \ub9cc\uc288\ud0c0\uc778\uc740 \ub8ec\ud2b8\uc288\ud14c\ud2b8\uc758 \uc791\uc804\ucc38\ubaa8 \uadc4\ud130 \ube14\ub8e8\uba58\ud2b8\ub9ac\ud2b8 \ub300\ub839 \uacfc \ud568\uaed8 \uc791\uc804\uacc4\ud68d \uc218\ub9bd\uc5d0 \ucc38\uc5ec\ud588\ub2e4. \ub9cc\uc288\ud0c0\uc778\uc740 \uc9d1\ub2e8\uad70\uc758 \uae30\uac11\ubcd1\ub825 \ub2e4\uc218\ub97c \ubc1c\ud130 \ud3f0 \ub77c\uc774\ud5e4\ub098\uc6b0\uc758 \uc81c10\uad70\uc5d0 \uc9d1\uc911\uc2dc\ucf1c \uc8fc\uba74\uc11c \ub3cc\ud30c\uad6c\ub97c \ub9c8\ub828, \ud3f4\ub780\ub4dc\uad70\uc744 \ube44\uc2a4\ud234\ub77c \uac15 \uc11c\uc548\uc5d0 \ud3ec\uc704\uc2dc\ud0a4\uace0, \ub77c\uc774\ud5e4\ub098\uc6b0\uc758 \uc81c10\uad70\uc774 \uae30\uac11\uae30\ub3d9\uc758 \ud798\uc73c\ub85c \ud3f4\ub780\ub4dc\uc758 \uc218\ub3c4 \ubc14\ub974\uc0e4\ubc14\uae4c\uc9c0 \ub3cc\uc9c4\ud558\ub294 \ub3d9\uc548 \ube4c\ud5ec\ub984 \ub9ac\uc2a4\ud2b8\uc758 \uc81c14\uad70\uacfc \uc694\ud558\ub124\uc2a4 \ube14\ub77c\ucf54\ube44\uce20\uc758 \uc81c8\uad70\uc740 \uce21\uba74\uc9c0\uc6d0\uc744 \ub9e1\ub294\ub2e4\ub294 \uacc4\ud68d\uc744 \uc791\uc131\ud588\uace0, \ub8ec\ud2b8\uc288\ud14c\ud2b8\ub294 \uc774\ub97c \ubc1b\uc544\ub4e4\uc600\ub2e4. \uc0ac\uc2e4 \ub9cc\uc288\ud0c0\uc778\uc740 \uac1c\uc778\uc801\uc73c\ub85c \ud3f4\ub780\ub4dc\ub97c \ub3c5\uc77c\uacfc \uc18c\ub828 \uc0ac\uc774\uc758 \uc644\ucda9\uc9c0\ub300\ub85c \ub0a8\uaca8\ub450\ub294 \ud3b8\uc774 \uc88b\ub2e4\uace0 \uc0dd\uac01\ud588\uae30\uc5d0 \ub300\ud3f4\ub780\ub4dc \uad70\uc0ac \uc791\uc804\uc5d0 \ubbf8\uc628\uc801\uc774\uc5c8\ub2e4. \ubfd0\ub9cc \uc544\ub2c8\ub77c \ud3f4\ub780\ub4dc \uce68\uacf5\uc774 \uc644\ub8cc\ub418\uc9c0 \uc54a\uc558\uc744 \ub54c \uc11c\ucabd\uc5d0\uc11c \uc5f0\ud569\uad70\uc774 \ub4a4\ud1b5\uc218\ub97c \uccd0\uc11c \ub3c5\uc77c\uc774 \uc591\uba74\uc804\uc7c1\uc5d0 \ube60\uc9c0\uac8c \ub418\ub294 \uac83\ub3c4 \uac71\uc815\uac70\ub9ac\uc600\ub2e4.", "paragraph_answer": "1939\ub144 8\uc6d4 18\uc77c, \ud3f4\ub780\ub4dc\ub97c \uce68\uacf5\ud558\ub294 \ubc31\uc0c9 \uc791\uc804\uc744 \uc900\ube44\ud558\ub358 \ub9cc\uc288\ud0c0\uc778\uc740 \uac8c\ub974\ud2b8 \ud3f0 \ub8ec\ud2b8\uc288\ud14c\ud2b8\uc758 \ub0a8\ubd80 \uc9d1\ub2e8\uad70 \ucc38\ubaa8\uc7a5\uc73c\ub85c \ubd80\uc784\ud55c\ub2e4. \ub9cc\uc288\ud0c0\uc778\uc740 \ub8ec\ud2b8\uc288\ud14c\ud2b8\uc758 \uc791\uc804\ucc38\ubaa8 \uadc4\ud130 \ube14\ub8e8\uba58\ud2b8\ub9ac\ud2b8 \ub300\ub839 \uacfc \ud568\uaed8 \uc791\uc804\uacc4\ud68d \uc218\ub9bd\uc5d0 \ucc38\uc5ec\ud588\ub2e4. \ub9cc\uc288\ud0c0\uc778\uc740 \uc9d1\ub2e8\uad70\uc758 \uae30\uac11\ubcd1\ub825 \ub2e4\uc218\ub97c \ubc1c\ud130 \ud3f0 \ub77c\uc774\ud5e4\ub098\uc6b0\uc758 \uc81c10\uad70\uc5d0 \uc9d1\uc911\uc2dc\ucf1c \uc8fc\uba74\uc11c \ub3cc\ud30c\uad6c\ub97c \ub9c8\ub828, \ud3f4\ub780\ub4dc\uad70\uc744 \ube44\uc2a4\ud234\ub77c \uac15 \uc11c\uc548\uc5d0 \ud3ec\uc704\uc2dc\ud0a4\uace0, \ub77c\uc774\ud5e4\ub098\uc6b0\uc758 \uc81c10\uad70\uc774 \uae30\uac11\uae30\ub3d9\uc758 \ud798\uc73c\ub85c \ud3f4\ub780\ub4dc\uc758 \uc218\ub3c4 \ubc14\ub974\uc0e4\ubc14\uae4c\uc9c0 \ub3cc\uc9c4\ud558\ub294 \ub3d9\uc548 \ube4c\ud5ec\ub984 \ub9ac\uc2a4\ud2b8\uc758 \uc81c14\uad70\uacfc \uc694\ud558\ub124\uc2a4 \ube14\ub77c\ucf54\ube44\uce20\uc758 \uc81c8\uad70\uc740 \uce21\uba74\uc9c0\uc6d0\uc744 \ub9e1\ub294\ub2e4\ub294 \uacc4\ud68d\uc744 \uc791\uc131\ud588\uace0, \ub8ec\ud2b8\uc288\ud14c\ud2b8\ub294 \uc774\ub97c \ubc1b\uc544\ub4e4\uc600\ub2e4. \uc0ac\uc2e4 \ub9cc\uc288\ud0c0\uc778\uc740 \uac1c\uc778\uc801\uc73c\ub85c \ud3f4\ub780\ub4dc\ub97c \ub3c5\uc77c\uacfc \uc18c\ub828 \uc0ac\uc774\uc758 \uc644\ucda9\uc9c0\ub300\ub85c \ub0a8\uaca8\ub450\ub294 \ud3b8\uc774 \uc88b\ub2e4\uace0 \uc0dd\uac01\ud588\uae30\uc5d0 \ub300\ud3f4\ub780\ub4dc \uad70\uc0ac \uc791\uc804\uc5d0 \ubbf8\uc628\uc801\uc774\uc5c8\ub2e4. \ubfd0\ub9cc \uc544\ub2c8\ub77c \ud3f4\ub780\ub4dc \uce68\uacf5\uc774 \uc644\ub8cc\ub418\uc9c0 \uc54a\uc558\uc744 \ub54c \uc11c\ucabd\uc5d0\uc11c \uc5f0\ud569\uad70\uc774 \ub4a4\ud1b5\uc218\ub97c \uccd0\uc11c \ub3c5\uc77c\uc774 \uc591\uba74\uc804\uc7c1\uc5d0 \ube60\uc9c0\uac8c \ub418\ub294 \uac83\ub3c4 \uac71\uc815\uac70\ub9ac\uc600\ub2e4.", "sentence_answer": "\ub9cc\uc288\ud0c0\uc778\uc740 \ub8ec\ud2b8\uc288\ud14c\ud2b8\uc758 \uc791\uc804\ucc38\ubaa8 \uadc4\ud130 \ube14\ub8e8\uba58\ud2b8\ub9ac\ud2b8 \ub300\ub839 \uacfc \ud568\uaed8 \uc791\uc804\uacc4\ud68d \uc218\ub9bd\uc5d0 \ucc38\uc5ec\ud588\ub2e4.", "paragraph_id": "6586570-9-1", "paragraph_question": "question: \ub9cc\uc288\ud0c0\uc778\uc740 \ub8ec\ud2b8\uc288\ud14c\ud2b8\uc758 \uc791\uc804\ucc38\ubaa8\uc778 \uc5b4\ub290 \ub300\ub839\uacfc \ud568\uaed8 \uc791\uc804\uacc4\ud68d\uc744 \uc218\ub9bd\ud588\ub294\uac00?, context: 1939\ub144 8\uc6d4 18\uc77c, \ud3f4\ub780\ub4dc\ub97c \uce68\uacf5\ud558\ub294 \ubc31\uc0c9 \uc791\uc804\uc744 \uc900\ube44\ud558\ub358 \ub9cc\uc288\ud0c0\uc778\uc740 \uac8c\ub974\ud2b8 \ud3f0 \ub8ec\ud2b8\uc288\ud14c\ud2b8\uc758 \ub0a8\ubd80 \uc9d1\ub2e8\uad70 \ucc38\ubaa8\uc7a5\uc73c\ub85c \ubd80\uc784\ud55c\ub2e4. \ub9cc\uc288\ud0c0\uc778\uc740 \ub8ec\ud2b8\uc288\ud14c\ud2b8\uc758 \uc791\uc804\ucc38\ubaa8 \uadc4\ud130 \ube14\ub8e8\uba58\ud2b8\ub9ac\ud2b8 \ub300\ub839\uacfc \ud568\uaed8 \uc791\uc804\uacc4\ud68d \uc218\ub9bd\uc5d0 \ucc38\uc5ec\ud588\ub2e4. \ub9cc\uc288\ud0c0\uc778\uc740 \uc9d1\ub2e8\uad70\uc758 \uae30\uac11\ubcd1\ub825 \ub2e4\uc218\ub97c \ubc1c\ud130 \ud3f0 \ub77c\uc774\ud5e4\ub098\uc6b0\uc758 \uc81c10\uad70\uc5d0 \uc9d1\uc911\uc2dc\ucf1c \uc8fc\uba74\uc11c \ub3cc\ud30c\uad6c\ub97c \ub9c8\ub828, \ud3f4\ub780\ub4dc\uad70\uc744 \ube44\uc2a4\ud234\ub77c \uac15 \uc11c\uc548\uc5d0 \ud3ec\uc704\uc2dc\ud0a4\uace0, \ub77c\uc774\ud5e4\ub098\uc6b0\uc758 \uc81c10\uad70\uc774 \uae30\uac11\uae30\ub3d9\uc758 \ud798\uc73c\ub85c \ud3f4\ub780\ub4dc\uc758 \uc218\ub3c4 \ubc14\ub974\uc0e4\ubc14\uae4c\uc9c0 \ub3cc\uc9c4\ud558\ub294 \ub3d9\uc548 \ube4c\ud5ec\ub984 \ub9ac\uc2a4\ud2b8\uc758 \uc81c14\uad70\uacfc \uc694\ud558\ub124\uc2a4 \ube14\ub77c\ucf54\ube44\uce20\uc758 \uc81c8\uad70\uc740 \uce21\uba74\uc9c0\uc6d0\uc744 \ub9e1\ub294\ub2e4\ub294 \uacc4\ud68d\uc744 \uc791\uc131\ud588\uace0, \ub8ec\ud2b8\uc288\ud14c\ud2b8\ub294 \uc774\ub97c \ubc1b\uc544\ub4e4\uc600\ub2e4. \uc0ac\uc2e4 \ub9cc\uc288\ud0c0\uc778\uc740 \uac1c\uc778\uc801\uc73c\ub85c \ud3f4\ub780\ub4dc\ub97c \ub3c5\uc77c\uacfc \uc18c\ub828 \uc0ac\uc774\uc758 \uc644\ucda9\uc9c0\ub300\ub85c \ub0a8\uaca8\ub450\ub294 \ud3b8\uc774 \uc88b\ub2e4\uace0 \uc0dd\uac01\ud588\uae30\uc5d0 \ub300\ud3f4\ub780\ub4dc \uad70\uc0ac \uc791\uc804\uc5d0 \ubbf8\uc628\uc801\uc774\uc5c8\ub2e4. \ubfd0\ub9cc \uc544\ub2c8\ub77c \ud3f4\ub780\ub4dc \uce68\uacf5\uc774 \uc644\ub8cc\ub418\uc9c0 \uc54a\uc558\uc744 \ub54c \uc11c\ucabd\uc5d0\uc11c \uc5f0\ud569\uad70\uc774 \ub4a4\ud1b5\uc218\ub97c \uccd0\uc11c \ub3c5\uc77c\uc774 \uc591\uba74\uc804\uc7c1\uc5d0 \ube60\uc9c0\uac8c \ub418\ub294 \uac83\ub3c4 \uac71\uc815\uac70\ub9ac\uc600\ub2e4."} {"question": "\uc54c\uce20\ud558\uc774\uba38\uc758 \uc815\ud655\ud55c \uc9c4\ub2e8\uc744 \uc704\ud574\uc11c \uc5b4\ub5a4 \uc2e0\uacbd\uc5d0 \ub300\ud55c \uc870\uc0ac\uac00 \ud544\uc694\ud55c\uac00?", "paragraph": "\uc9c8\ubcd1\uc758 \ud2b9\uc131\uc740 \uac1c\uac1c\uc778\ub9c8\ub2e4 \ub2e4\ub974\uac8c \ub098\ud0c0\ub098\uc9c0\ub9cc \uc77c\ubd80 \uacf5\ud1b5\uc801\uc778 \uc99d\uc0c1\uc744 \uac00\uc9c4\ub2e4. \ucd08\uae30\uc99d\uc0c1\uc740 \ub178\ud654\uc5d0 \uc758\ud55c \ub2e8\uc21c\ud55c \uc99d\uc0c1\uc774\ub098 \uc2a4\ud2b8\ub808\uc2a4\uc5d0 \uc758\ud55c \uc99d\uc0c1\uc73c\ub85c \uc624\ud574\ub418\ub294 \uacbd\ud5a5\uc774 \uc788\ub2e4. \ubc1c\ubcd1 \ucd08\uae30\uc5d0\ub294 \uc774\ub984\u00b7\ub0a0\uc9dc\u00b7\uc7a5\uc18c\uc640 \uac19\uc740 \uac83\ub4e4\uc774 \uae30\uc5b5\uc5d0\uc11c \uc0ac\ub77c\uc9c0\ub294 \uacf5\ud1b5\ub41c \ub2e8\uae30 \uae30\uc5b5 \uc0c1\uc2e4\uc744 \uacaa\ub294\ub2e4. \uc9c8\ubcd1\uc774 \uc545\ud654\ub418\uba74, \ud63c\ub780, \uaca9\ud55c \ud589\ub3d9, \uc870\uc6b8\uc99d, \uc5b8\uc5b4\uc7a5\uc560, \uc7a5\uae30\uae30\uc5b5\uc0c1\uc2e4\ub4f1\uc758 \uc99d\uc0c1\uc774 \ub098\ud0c0\ub09c\ub2e4. \uacb0\uacfc\uc801\uc73c\ub85c \uc2e0\uccb4\uae30\ub2a5\uc774 \uc0c1\uc2e4\ub418\uace0, \uce58\uc0ac\uc5d0 \uc774\ub978\ub2e4. \uac1c\uac1c\uc778\ub9c8\ub2e4 \uc99d\uc0c1\uc774 \ub2e4\ub974\uae30 \ub54c\ubb38\uc5d0 \uc9c8\ubcd1\uc774 \uc5b4\ub5bb\uac8c \uc601\ud5a5\uc744 \ubbf8\uce60\uc9c0 \uc608\uce21\ud558\ub294 \uac83\uc740 \uc5b4\ub835\ub2e4. \uc54c\uce20\ud558\uc774\uba38\ubcd1\uc774 \uc758\uc2ec\ub418\uba74 \ubcf4\ud1b5 \uc0dd\uac01\ud558\uac70\ub098 \ud589\ub3d9\ud558\ub294 \ub2a5\ub825\uc744 \uac80\uc0ac\ud558\ub294 \uc9c4\ub2e8\uc774 \ud589\ud574\uc9c0\uace0 \uac00\ub2a5\ud55c\uacbd\uc6b0 \ub1cc\uac80\uc0ac\uac00 \uc774\ub8e8\uc5b4\uc9c4\ub2e4. \ud558\uc9c0\ub9cc \uc815\ud655\ud55c \uc9c4\ub2e8\uc744 \uc704\ud574\uc11c\ub294 \ub1cc\uc2e0\uacbd\uc5d0 \ub300\ud55c \uc870\uc0ac\uac00 \ud544\uc694\ud558\ub2e4. \uc54c\uce20\ud558\uc774\uba38\ubcd1\uc774 \ubc1c\ubcd1\ud574\ub3c4 \uc644\uc804\ud788 \ud655\uc778\ub420\ub54c\uae4c\uc9c0\ub294 \ubcf4\ud1b5 \ub9ce\uc740 \uc2dc\uac04\uc774 \uc18c\uc694\ub418\uc5b4, \uc9c4\ub2e8\uc774 \ub0b4\ub824\uc9c0\uc9c0 \uc54a\uc740 \ucc44\ub85c \uc218\ub144\uac04 \ubcd1\uc774 \uc9c4\ud589\ub420 \uc218 \uc788\ub2e4. \ubc1c\ubcd1\ud560 \uacbd\uc6b0 \ud3c9\uade0 \uae30\ub300 \uc218\uba85\uc740 7\ub144\uc774\uba70 3%\ubbf8\ub9cc\uc774 \uc9c4\ub2e8 \uc774\ud6c4 14\ub144\uc744 \ub118\uae34\ub2e4.", "answer": "\ub1cc\uc2e0\uacbd", "sentence": "\ud558\uc9c0\ub9cc \uc815\ud655\ud55c \uc9c4\ub2e8\uc744 \uc704\ud574\uc11c\ub294 \ub1cc\uc2e0\uacbd \uc5d0 \ub300\ud55c \uc870\uc0ac\uac00 \ud544\uc694\ud558\ub2e4.", "paragraph_sentence": "\uc9c8\ubcd1\uc758 \ud2b9\uc131\uc740 \uac1c\uac1c\uc778\ub9c8\ub2e4 \ub2e4\ub974\uac8c \ub098\ud0c0\ub098\uc9c0\ub9cc \uc77c\ubd80 \uacf5\ud1b5\uc801\uc778 \uc99d\uc0c1\uc744 \uac00\uc9c4\ub2e4. \ucd08\uae30\uc99d\uc0c1\uc740 \ub178\ud654\uc5d0 \uc758\ud55c \ub2e8\uc21c\ud55c \uc99d\uc0c1\uc774\ub098 \uc2a4\ud2b8\ub808\uc2a4\uc5d0 \uc758\ud55c \uc99d\uc0c1\uc73c\ub85c \uc624\ud574\ub418\ub294 \uacbd\ud5a5\uc774 \uc788\ub2e4. \ubc1c\ubcd1 \ucd08\uae30\uc5d0\ub294 \uc774\ub984\u00b7\ub0a0\uc9dc\u00b7\uc7a5\uc18c\uc640 \uac19\uc740 \uac83\ub4e4\uc774 \uae30\uc5b5\uc5d0\uc11c \uc0ac\ub77c\uc9c0\ub294 \uacf5\ud1b5\ub41c \ub2e8\uae30 \uae30\uc5b5 \uc0c1\uc2e4\uc744 \uacaa\ub294\ub2e4. \uc9c8\ubcd1\uc774 \uc545\ud654\ub418\uba74, \ud63c\ub780, \uaca9\ud55c \ud589\ub3d9, \uc870\uc6b8\uc99d, \uc5b8\uc5b4\uc7a5\uc560, \uc7a5\uae30\uae30\uc5b5\uc0c1\uc2e4\ub4f1\uc758 \uc99d\uc0c1\uc774 \ub098\ud0c0\ub09c\ub2e4. \uacb0\uacfc\uc801\uc73c\ub85c \uc2e0\uccb4\uae30\ub2a5\uc774 \uc0c1\uc2e4\ub418\uace0, \uce58\uc0ac\uc5d0 \uc774\ub978\ub2e4. \uac1c\uac1c\uc778\ub9c8\ub2e4 \uc99d\uc0c1\uc774 \ub2e4\ub974\uae30 \ub54c\ubb38\uc5d0 \uc9c8\ubcd1\uc774 \uc5b4\ub5bb\uac8c \uc601\ud5a5\uc744 \ubbf8\uce60\uc9c0 \uc608\uce21\ud558\ub294 \uac83\uc740 \uc5b4\ub835\ub2e4. \uc54c\uce20\ud558\uc774\uba38\ubcd1\uc774 \uc758\uc2ec\ub418\uba74 \ubcf4\ud1b5 \uc0dd\uac01\ud558\uac70\ub098 \ud589\ub3d9\ud558\ub294 \ub2a5\ub825\uc744 \uac80\uc0ac\ud558\ub294 \uc9c4\ub2e8\uc774 \ud589\ud574\uc9c0\uace0 \uac00\ub2a5\ud55c\uacbd\uc6b0 \ub1cc\uac80\uc0ac\uac00 \uc774\ub8e8\uc5b4\uc9c4\ub2e4. \ud558\uc9c0\ub9cc \uc815\ud655\ud55c \uc9c4\ub2e8\uc744 \uc704\ud574\uc11c\ub294 \ub1cc\uc2e0\uacbd \uc5d0 \ub300\ud55c \uc870\uc0ac\uac00 \ud544\uc694\ud558\ub2e4. \uc54c\uce20\ud558\uc774\uba38\ubcd1\uc774 \ubc1c\ubcd1\ud574\ub3c4 \uc644\uc804\ud788 \ud655\uc778\ub420\ub54c\uae4c\uc9c0\ub294 \ubcf4\ud1b5 \ub9ce\uc740 \uc2dc\uac04\uc774 \uc18c\uc694\ub418\uc5b4, \uc9c4\ub2e8\uc774 \ub0b4\ub824\uc9c0\uc9c0 \uc54a\uc740 \ucc44\ub85c \uc218\ub144\uac04 \ubcd1\uc774 \uc9c4\ud589\ub420 \uc218 \uc788\ub2e4. \ubc1c\ubcd1\ud560 \uacbd\uc6b0 \ud3c9\uade0 \uae30\ub300 \uc218\uba85\uc740 7\ub144\uc774\uba70 3%\ubbf8\ub9cc\uc774 \uc9c4\ub2e8 \uc774\ud6c4 14\ub144\uc744 \ub118\uae34\ub2e4.", "paragraph_answer": "\uc9c8\ubcd1\uc758 \ud2b9\uc131\uc740 \uac1c\uac1c\uc778\ub9c8\ub2e4 \ub2e4\ub974\uac8c \ub098\ud0c0\ub098\uc9c0\ub9cc \uc77c\ubd80 \uacf5\ud1b5\uc801\uc778 \uc99d\uc0c1\uc744 \uac00\uc9c4\ub2e4. \ucd08\uae30\uc99d\uc0c1\uc740 \ub178\ud654\uc5d0 \uc758\ud55c \ub2e8\uc21c\ud55c \uc99d\uc0c1\uc774\ub098 \uc2a4\ud2b8\ub808\uc2a4\uc5d0 \uc758\ud55c \uc99d\uc0c1\uc73c\ub85c \uc624\ud574\ub418\ub294 \uacbd\ud5a5\uc774 \uc788\ub2e4. \ubc1c\ubcd1 \ucd08\uae30\uc5d0\ub294 \uc774\ub984\u00b7\ub0a0\uc9dc\u00b7\uc7a5\uc18c\uc640 \uac19\uc740 \uac83\ub4e4\uc774 \uae30\uc5b5\uc5d0\uc11c \uc0ac\ub77c\uc9c0\ub294 \uacf5\ud1b5\ub41c \ub2e8\uae30 \uae30\uc5b5 \uc0c1\uc2e4\uc744 \uacaa\ub294\ub2e4. \uc9c8\ubcd1\uc774 \uc545\ud654\ub418\uba74, \ud63c\ub780, \uaca9\ud55c \ud589\ub3d9, \uc870\uc6b8\uc99d, \uc5b8\uc5b4\uc7a5\uc560, \uc7a5\uae30\uae30\uc5b5\uc0c1\uc2e4\ub4f1\uc758 \uc99d\uc0c1\uc774 \ub098\ud0c0\ub09c\ub2e4. \uacb0\uacfc\uc801\uc73c\ub85c \uc2e0\uccb4\uae30\ub2a5\uc774 \uc0c1\uc2e4\ub418\uace0, \uce58\uc0ac\uc5d0 \uc774\ub978\ub2e4. \uac1c\uac1c\uc778\ub9c8\ub2e4 \uc99d\uc0c1\uc774 \ub2e4\ub974\uae30 \ub54c\ubb38\uc5d0 \uc9c8\ubcd1\uc774 \uc5b4\ub5bb\uac8c \uc601\ud5a5\uc744 \ubbf8\uce60\uc9c0 \uc608\uce21\ud558\ub294 \uac83\uc740 \uc5b4\ub835\ub2e4. \uc54c\uce20\ud558\uc774\uba38\ubcd1\uc774 \uc758\uc2ec\ub418\uba74 \ubcf4\ud1b5 \uc0dd\uac01\ud558\uac70\ub098 \ud589\ub3d9\ud558\ub294 \ub2a5\ub825\uc744 \uac80\uc0ac\ud558\ub294 \uc9c4\ub2e8\uc774 \ud589\ud574\uc9c0\uace0 \uac00\ub2a5\ud55c\uacbd\uc6b0 \ub1cc\uac80\uc0ac\uac00 \uc774\ub8e8\uc5b4\uc9c4\ub2e4. \ud558\uc9c0\ub9cc \uc815\ud655\ud55c \uc9c4\ub2e8\uc744 \uc704\ud574\uc11c\ub294 \ub1cc\uc2e0\uacbd \uc5d0 \ub300\ud55c \uc870\uc0ac\uac00 \ud544\uc694\ud558\ub2e4. \uc54c\uce20\ud558\uc774\uba38\ubcd1\uc774 \ubc1c\ubcd1\ud574\ub3c4 \uc644\uc804\ud788 \ud655\uc778\ub420\ub54c\uae4c\uc9c0\ub294 \ubcf4\ud1b5 \ub9ce\uc740 \uc2dc\uac04\uc774 \uc18c\uc694\ub418\uc5b4, \uc9c4\ub2e8\uc774 \ub0b4\ub824\uc9c0\uc9c0 \uc54a\uc740 \ucc44\ub85c \uc218\ub144\uac04 \ubcd1\uc774 \uc9c4\ud589\ub420 \uc218 \uc788\ub2e4. \ubc1c\ubcd1\ud560 \uacbd\uc6b0 \ud3c9\uade0 \uae30\ub300 \uc218\uba85\uc740 7\ub144\uc774\uba70 3%\ubbf8\ub9cc\uc774 \uc9c4\ub2e8 \uc774\ud6c4 14\ub144\uc744 \ub118\uae34\ub2e4.", "sentence_answer": "\ud558\uc9c0\ub9cc \uc815\ud655\ud55c \uc9c4\ub2e8\uc744 \uc704\ud574\uc11c\ub294 \ub1cc\uc2e0\uacbd \uc5d0 \ub300\ud55c \uc870\uc0ac\uac00 \ud544\uc694\ud558\ub2e4.", "paragraph_id": "6539479-1-0", "paragraph_question": "question: \uc54c\uce20\ud558\uc774\uba38\uc758 \uc815\ud655\ud55c \uc9c4\ub2e8\uc744 \uc704\ud574\uc11c \uc5b4\ub5a4 \uc2e0\uacbd\uc5d0 \ub300\ud55c \uc870\uc0ac\uac00 \ud544\uc694\ud55c\uac00?, context: \uc9c8\ubcd1\uc758 \ud2b9\uc131\uc740 \uac1c\uac1c\uc778\ub9c8\ub2e4 \ub2e4\ub974\uac8c \ub098\ud0c0\ub098\uc9c0\ub9cc \uc77c\ubd80 \uacf5\ud1b5\uc801\uc778 \uc99d\uc0c1\uc744 \uac00\uc9c4\ub2e4. \ucd08\uae30\uc99d\uc0c1\uc740 \ub178\ud654\uc5d0 \uc758\ud55c \ub2e8\uc21c\ud55c \uc99d\uc0c1\uc774\ub098 \uc2a4\ud2b8\ub808\uc2a4\uc5d0 \uc758\ud55c \uc99d\uc0c1\uc73c\ub85c \uc624\ud574\ub418\ub294 \uacbd\ud5a5\uc774 \uc788\ub2e4. \ubc1c\ubcd1 \ucd08\uae30\uc5d0\ub294 \uc774\ub984\u00b7\ub0a0\uc9dc\u00b7\uc7a5\uc18c\uc640 \uac19\uc740 \uac83\ub4e4\uc774 \uae30\uc5b5\uc5d0\uc11c \uc0ac\ub77c\uc9c0\ub294 \uacf5\ud1b5\ub41c \ub2e8\uae30 \uae30\uc5b5 \uc0c1\uc2e4\uc744 \uacaa\ub294\ub2e4. \uc9c8\ubcd1\uc774 \uc545\ud654\ub418\uba74, \ud63c\ub780, \uaca9\ud55c \ud589\ub3d9, \uc870\uc6b8\uc99d, \uc5b8\uc5b4\uc7a5\uc560, \uc7a5\uae30\uae30\uc5b5\uc0c1\uc2e4\ub4f1\uc758 \uc99d\uc0c1\uc774 \ub098\ud0c0\ub09c\ub2e4. \uacb0\uacfc\uc801\uc73c\ub85c \uc2e0\uccb4\uae30\ub2a5\uc774 \uc0c1\uc2e4\ub418\uace0, \uce58\uc0ac\uc5d0 \uc774\ub978\ub2e4. \uac1c\uac1c\uc778\ub9c8\ub2e4 \uc99d\uc0c1\uc774 \ub2e4\ub974\uae30 \ub54c\ubb38\uc5d0 \uc9c8\ubcd1\uc774 \uc5b4\ub5bb\uac8c \uc601\ud5a5\uc744 \ubbf8\uce60\uc9c0 \uc608\uce21\ud558\ub294 \uac83\uc740 \uc5b4\ub835\ub2e4. \uc54c\uce20\ud558\uc774\uba38\ubcd1\uc774 \uc758\uc2ec\ub418\uba74 \ubcf4\ud1b5 \uc0dd\uac01\ud558\uac70\ub098 \ud589\ub3d9\ud558\ub294 \ub2a5\ub825\uc744 \uac80\uc0ac\ud558\ub294 \uc9c4\ub2e8\uc774 \ud589\ud574\uc9c0\uace0 \uac00\ub2a5\ud55c\uacbd\uc6b0 \ub1cc\uac80\uc0ac\uac00 \uc774\ub8e8\uc5b4\uc9c4\ub2e4. \ud558\uc9c0\ub9cc \uc815\ud655\ud55c \uc9c4\ub2e8\uc744 \uc704\ud574\uc11c\ub294 \ub1cc\uc2e0\uacbd\uc5d0 \ub300\ud55c \uc870\uc0ac\uac00 \ud544\uc694\ud558\ub2e4. \uc54c\uce20\ud558\uc774\uba38\ubcd1\uc774 \ubc1c\ubcd1\ud574\ub3c4 \uc644\uc804\ud788 \ud655\uc778\ub420\ub54c\uae4c\uc9c0\ub294 \ubcf4\ud1b5 \ub9ce\uc740 \uc2dc\uac04\uc774 \uc18c\uc694\ub418\uc5b4, \uc9c4\ub2e8\uc774 \ub0b4\ub824\uc9c0\uc9c0 \uc54a\uc740 \ucc44\ub85c \uc218\ub144\uac04 \ubcd1\uc774 \uc9c4\ud589\ub420 \uc218 \uc788\ub2e4. \ubc1c\ubcd1\ud560 \uacbd\uc6b0 \ud3c9\uade0 \uae30\ub300 \uc218\uba85\uc740 7\ub144\uc774\uba70 3%\ubbf8\ub9cc\uc774 \uc9c4\ub2e8 \uc774\ud6c4 14\ub144\uc744 \ub118\uae34\ub2e4."} {"question": "\uc870\uc120\uc740 \ucd08\ubc18 \uc5b4\ub5a4 \uccb4\uc81c\uac00 \uc81c\ub300\ub85c \ubc1c\ud718\ub418\uc9c0 \ubabb\ud558\uc600\uc2b5\ub2c8\uae4c?", "paragraph": "1592\ub144, \uc77c\ubcf8\uc744 \ud1b5\uc77c\ud55c \ub3c4\uc694\ud1a0\ubbf8 \ud788\ub370\uc694\uc2dc(\ud48d\uc2e0\uc218\uae38)\ub294 20\ub9cc \ubcd1\ub825\uc758 \uc77c\ubcf8\uad70\uc744 \uc870\uc120\uc5d0 \ub300\uac70 \ubcf4\ub0b4\uc5b4 \uc870\uc120\uc744 \uce68\ub7b5\ud588\ub2e4. \uc870\uc120\uc740 \ucd08\ubc18 \uc81c\uc2b9\ubc29\ub7b5\uccb4\uc81c\uac00 \uc81c\ub300\ub85c \ubc1c\ud718\ub418\uc9c0 \ubabb\ud558\uc600\uace0, \uc77c\ubcf8\uad70\uc758 \uc2e0\ubb34\uae30\uc778 \uc870\ucd1d\uc5d0 \ud06c\uac8c \uace0\uc804\ud558\uc600\ub2e4. \ubd80\uc0b0\uc9c4\uc131\uc758 \uc815\ubc1c\uacfc \ub3d9\ub798\uc131\uc758 \uc1a1\uc0c1\ud604\uc774 \uc0bd\uc2dc\uac04 \ub9cc\uc5d0 \uc77c\ubcf8\uad70\uc5d0\uac8c \ubb34\ub108\uc84c\ub2e4. \uc774\ub54c\uae4c\uc9c0\ub9cc \ud574\ub3c4 \uad00\uad70\uc740 \uc798 \uc2f8\uc6e0\uace0 \uc77c\ubcf8\uad70\uc758 \ud76c\uc0dd\uc790\ub294 \uc0ac\ub9dd \uc57d 100\uba85, \uc911\uc0c1 \uc57d 500\uba85\uc774\uc5c8\ub2e4. \uadf8\ub9ac\uace0 \uc0c1\uc8fc\uc758 \uc774\uc77c\uacfc \ucda9\uc8fc \ud0c4\uae08\ub300\uc5d0\uc11c \uc9c4\uc744 \uce58\uace0 \uc788\ub358 \uc2e0\ub9bd\uc7a5\uad70\uc758 \uad70\ub300\ub3c4 \uc77c\ubcf8\uad70\uc774 \ubb34\uc7a5\ud55c \uc870\ucd1d\uc5d0 \uc758\ud574 \uada4\uba78\ub2f9\ud55c\ub2e4. \uc0c1\ud669\uc774 \uc774\ub807\uac8c \ub418\uc790 \uc120\uc870\ub294 \uc218\ub3c4 \ud55c\uc591\uc744 \ubc84\ub9ac\uace0 \uc555\ub85d\uac15\ubcc0\uc5d0 \uc788\ub294 \uc758\uc8fc\uae4c\uc9c0 \ud53c\ub09c\uc744 \uac00\uc57c \ud588\ub2e4. \uc77c\ubcf8\uad70\uc740 \uac1c\uc804 20\uc5ec \uc77c \ub9cc\uc5d0 \ud55c\uc591\uc5d0 \uc785\uc131\ud558\uc600\uace0, \uc784\uc9c4\uac15 \ubc29\uc5b4\uc120\ub3c4 \uac04\ub2e8\ud788 \ub6ab\uc740 \ucc44 \ud3c9\uc591\uae4c\uc9c0 \uc190\uc27d\uac8c \uc810\ub839\ud558\uc600\ub2e4.", "answer": "\uc81c\uc2b9\ubc29\ub7b5\uccb4\uc81c", "sentence": "\uc870\uc120\uc740 \ucd08\ubc18 \uc81c\uc2b9\ubc29\ub7b5\uccb4\uc81c \uac00 \uc81c\ub300\ub85c \ubc1c\ud718\ub418\uc9c0 \ubabb\ud558\uc600\uace0, \uc77c\ubcf8\uad70\uc758 \uc2e0\ubb34\uae30\uc778 \uc870\ucd1d\uc5d0 \ud06c\uac8c \uace0\uc804\ud558\uc600\ub2e4.", "paragraph_sentence": "1592\ub144, \uc77c\ubcf8\uc744 \ud1b5\uc77c\ud55c \ub3c4\uc694\ud1a0\ubbf8 \ud788\ub370\uc694\uc2dc(\ud48d\uc2e0\uc218\uae38)\ub294 20\ub9cc \ubcd1\ub825\uc758 \uc77c\ubcf8\uad70\uc744 \uc870\uc120\uc5d0 \ub300\uac70 \ubcf4\ub0b4\uc5b4 \uc870\uc120\uc744 \uce68\ub7b5\ud588\ub2e4. \uc870\uc120\uc740 \ucd08\ubc18 \uc81c\uc2b9\ubc29\ub7b5\uccb4\uc81c \uac00 \uc81c\ub300\ub85c \ubc1c\ud718\ub418\uc9c0 \ubabb\ud558\uc600\uace0, \uc77c\ubcf8\uad70\uc758 \uc2e0\ubb34\uae30\uc778 \uc870\ucd1d\uc5d0 \ud06c\uac8c \uace0\uc804\ud558\uc600\ub2e4. \ubd80\uc0b0\uc9c4\uc131\uc758 \uc815\ubc1c\uacfc \ub3d9\ub798\uc131\uc758 \uc1a1\uc0c1\ud604\uc774 \uc0bd\uc2dc\uac04 \ub9cc\uc5d0 \uc77c\ubcf8\uad70\uc5d0\uac8c \ubb34\ub108\uc84c\ub2e4. \uc774\ub54c\uae4c\uc9c0\ub9cc \ud574\ub3c4 \uad00\uad70\uc740 \uc798 \uc2f8\uc6e0\uace0 \uc77c\ubcf8\uad70\uc758 \ud76c\uc0dd\uc790\ub294 \uc0ac\ub9dd \uc57d 100\uba85, \uc911\uc0c1 \uc57d 500\uba85\uc774\uc5c8\ub2e4. \uadf8\ub9ac\uace0 \uc0c1\uc8fc\uc758 \uc774\uc77c\uacfc \ucda9\uc8fc \ud0c4\uae08\ub300\uc5d0\uc11c \uc9c4\uc744 \uce58\uace0 \uc788\ub358 \uc2e0\ub9bd\uc7a5\uad70\uc758 \uad70\ub300\ub3c4 \uc77c\ubcf8\uad70\uc774 \ubb34\uc7a5\ud55c \uc870\ucd1d\uc5d0 \uc758\ud574 \uada4\uba78\ub2f9\ud55c\ub2e4. \uc0c1\ud669\uc774 \uc774\ub807\uac8c \ub418\uc790 \uc120\uc870\ub294 \uc218\ub3c4 \ud55c\uc591\uc744 \ubc84\ub9ac\uace0 \uc555\ub85d\uac15\ubcc0\uc5d0 \uc788\ub294 \uc758\uc8fc\uae4c\uc9c0 \ud53c\ub09c\uc744 \uac00\uc57c \ud588\ub2e4. \uc77c\ubcf8\uad70\uc740 \uac1c\uc804 20\uc5ec \uc77c \ub9cc\uc5d0 \ud55c\uc591\uc5d0 \uc785\uc131\ud558\uc600\uace0, \uc784\uc9c4\uac15 \ubc29\uc5b4\uc120\ub3c4 \uac04\ub2e8\ud788 \ub6ab\uc740 \ucc44 \ud3c9\uc591\uae4c\uc9c0 \uc190\uc27d\uac8c \uc810\ub839\ud558\uc600\ub2e4.", "paragraph_answer": "1592\ub144, \uc77c\ubcf8\uc744 \ud1b5\uc77c\ud55c \ub3c4\uc694\ud1a0\ubbf8 \ud788\ub370\uc694\uc2dc(\ud48d\uc2e0\uc218\uae38)\ub294 20\ub9cc \ubcd1\ub825\uc758 \uc77c\ubcf8\uad70\uc744 \uc870\uc120\uc5d0 \ub300\uac70 \ubcf4\ub0b4\uc5b4 \uc870\uc120\uc744 \uce68\ub7b5\ud588\ub2e4. \uc870\uc120\uc740 \ucd08\ubc18 \uc81c\uc2b9\ubc29\ub7b5\uccb4\uc81c \uac00 \uc81c\ub300\ub85c \ubc1c\ud718\ub418\uc9c0 \ubabb\ud558\uc600\uace0, \uc77c\ubcf8\uad70\uc758 \uc2e0\ubb34\uae30\uc778 \uc870\ucd1d\uc5d0 \ud06c\uac8c \uace0\uc804\ud558\uc600\ub2e4. \ubd80\uc0b0\uc9c4\uc131\uc758 \uc815\ubc1c\uacfc \ub3d9\ub798\uc131\uc758 \uc1a1\uc0c1\ud604\uc774 \uc0bd\uc2dc\uac04 \ub9cc\uc5d0 \uc77c\ubcf8\uad70\uc5d0\uac8c \ubb34\ub108\uc84c\ub2e4. \uc774\ub54c\uae4c\uc9c0\ub9cc \ud574\ub3c4 \uad00\uad70\uc740 \uc798 \uc2f8\uc6e0\uace0 \uc77c\ubcf8\uad70\uc758 \ud76c\uc0dd\uc790\ub294 \uc0ac\ub9dd \uc57d 100\uba85, \uc911\uc0c1 \uc57d 500\uba85\uc774\uc5c8\ub2e4. \uadf8\ub9ac\uace0 \uc0c1\uc8fc\uc758 \uc774\uc77c\uacfc \ucda9\uc8fc \ud0c4\uae08\ub300\uc5d0\uc11c \uc9c4\uc744 \uce58\uace0 \uc788\ub358 \uc2e0\ub9bd\uc7a5\uad70\uc758 \uad70\ub300\ub3c4 \uc77c\ubcf8\uad70\uc774 \ubb34\uc7a5\ud55c \uc870\ucd1d\uc5d0 \uc758\ud574 \uada4\uba78\ub2f9\ud55c\ub2e4. \uc0c1\ud669\uc774 \uc774\ub807\uac8c \ub418\uc790 \uc120\uc870\ub294 \uc218\ub3c4 \ud55c\uc591\uc744 \ubc84\ub9ac\uace0 \uc555\ub85d\uac15\ubcc0\uc5d0 \uc788\ub294 \uc758\uc8fc\uae4c\uc9c0 \ud53c\ub09c\uc744 \uac00\uc57c \ud588\ub2e4. \uc77c\ubcf8\uad70\uc740 \uac1c\uc804 20\uc5ec \uc77c \ub9cc\uc5d0 \ud55c\uc591\uc5d0 \uc785\uc131\ud558\uc600\uace0, \uc784\uc9c4\uac15 \ubc29\uc5b4\uc120\ub3c4 \uac04\ub2e8\ud788 \ub6ab\uc740 \ucc44 \ud3c9\uc591\uae4c\uc9c0 \uc190\uc27d\uac8c \uc810\ub839\ud558\uc600\ub2e4.", "sentence_answer": "\uc870\uc120\uc740 \ucd08\ubc18 \uc81c\uc2b9\ubc29\ub7b5\uccb4\uc81c \uac00 \uc81c\ub300\ub85c \ubc1c\ud718\ub418\uc9c0 \ubabb\ud558\uc600\uace0, \uc77c\ubcf8\uad70\uc758 \uc2e0\ubb34\uae30\uc778 \uc870\ucd1d\uc5d0 \ud06c\uac8c \uace0\uc804\ud558\uc600\ub2e4.", "paragraph_id": "6187890-6-1", "paragraph_question": "question: \uc870\uc120\uc740 \ucd08\ubc18 \uc5b4\ub5a4 \uccb4\uc81c\uac00 \uc81c\ub300\ub85c \ubc1c\ud718\ub418\uc9c0 \ubabb\ud558\uc600\uc2b5\ub2c8\uae4c?, context: 1592\ub144, \uc77c\ubcf8\uc744 \ud1b5\uc77c\ud55c \ub3c4\uc694\ud1a0\ubbf8 \ud788\ub370\uc694\uc2dc(\ud48d\uc2e0\uc218\uae38)\ub294 20\ub9cc \ubcd1\ub825\uc758 \uc77c\ubcf8\uad70\uc744 \uc870\uc120\uc5d0 \ub300\uac70 \ubcf4\ub0b4\uc5b4 \uc870\uc120\uc744 \uce68\ub7b5\ud588\ub2e4. \uc870\uc120\uc740 \ucd08\ubc18 \uc81c\uc2b9\ubc29\ub7b5\uccb4\uc81c\uac00 \uc81c\ub300\ub85c \ubc1c\ud718\ub418\uc9c0 \ubabb\ud558\uc600\uace0, \uc77c\ubcf8\uad70\uc758 \uc2e0\ubb34\uae30\uc778 \uc870\ucd1d\uc5d0 \ud06c\uac8c \uace0\uc804\ud558\uc600\ub2e4. \ubd80\uc0b0\uc9c4\uc131\uc758 \uc815\ubc1c\uacfc \ub3d9\ub798\uc131\uc758 \uc1a1\uc0c1\ud604\uc774 \uc0bd\uc2dc\uac04 \ub9cc\uc5d0 \uc77c\ubcf8\uad70\uc5d0\uac8c \ubb34\ub108\uc84c\ub2e4. \uc774\ub54c\uae4c\uc9c0\ub9cc \ud574\ub3c4 \uad00\uad70\uc740 \uc798 \uc2f8\uc6e0\uace0 \uc77c\ubcf8\uad70\uc758 \ud76c\uc0dd\uc790\ub294 \uc0ac\ub9dd \uc57d 100\uba85, \uc911\uc0c1 \uc57d 500\uba85\uc774\uc5c8\ub2e4. \uadf8\ub9ac\uace0 \uc0c1\uc8fc\uc758 \uc774\uc77c\uacfc \ucda9\uc8fc \ud0c4\uae08\ub300\uc5d0\uc11c \uc9c4\uc744 \uce58\uace0 \uc788\ub358 \uc2e0\ub9bd\uc7a5\uad70\uc758 \uad70\ub300\ub3c4 \uc77c\ubcf8\uad70\uc774 \ubb34\uc7a5\ud55c \uc870\ucd1d\uc5d0 \uc758\ud574 \uada4\uba78\ub2f9\ud55c\ub2e4. \uc0c1\ud669\uc774 \uc774\ub807\uac8c \ub418\uc790 \uc120\uc870\ub294 \uc218\ub3c4 \ud55c\uc591\uc744 \ubc84\ub9ac\uace0 \uc555\ub85d\uac15\ubcc0\uc5d0 \uc788\ub294 \uc758\uc8fc\uae4c\uc9c0 \ud53c\ub09c\uc744 \uac00\uc57c \ud588\ub2e4. \uc77c\ubcf8\uad70\uc740 \uac1c\uc804 20\uc5ec \uc77c \ub9cc\uc5d0 \ud55c\uc591\uc5d0 \uc785\uc131\ud558\uc600\uace0, \uc784\uc9c4\uac15 \ubc29\uc5b4\uc120\ub3c4 \uac04\ub2e8\ud788 \ub6ab\uc740 \ucc44 \ud3c9\uc591\uae4c\uc9c0 \uc190\uc27d\uac8c \uc810\ub839\ud558\uc600\ub2e4."} {"question": "\ub2e4\uc2dc \ub9f9\uacf5\uc744 \uac00\ud574 \ucd1d\uac80\uc804\uc744 \ubc8c\uc5ec \ub3d9\ub9f9\uad70\uc744 \uace0\uc9c0\ub300\uc5d0\uc11c \ucad3\uc544\ub0b8 \ubcd1\uc0ac\ub4e4\uc740 \uc5b4\ub5a4 \ubd80\ub300\uc5d0 \uc18d\ud574\uc788\ub294\uac00?", "paragraph": "\uc9d9\uc740 \uc548\uac1c\uc758 \ub3c4\uc6c0\uacfc \uc2dc\uc57c\uc758 \ud750\ub984\uc73c\ub85c \uc778\ud574 \uc870\uc2ec\uc2a4\ub7fd\uac8c \uc0dd. \uc77c\ub808\ub974(St. Hilaire) \uc0ac\ub2e8\uc774 \uc9c4\uad70\ud588\uc73c\ub098, \uadf8\ub4e4\uc774 \ub0b4\ub9ac\ub9c9\uae38\uc5d0 \uac14\uc744 \ub54c \uc800 \uc720\uba85\ud55c '\uc544\uc6b0\uc2a4\ud130\ub9ac\uce20\uc758 \ud0dc\uc591'(Sun of Austerlitz)\uc774 \uc548\uac1c\ub97c \uc5c6\uc560\ubc84\ub824, \uc774\ub4e4\uc774 \uc9c4\uad70\ud558\ub3c4\ub85d \uace0\ubb34\uc2dc\ucf30\ub2e4. \uace0\uc9c0\ub300 \uc704\uc5d0 \uc788\ub358 \ub7ec\uc2dc\uc544 \uc7a5\uad70\ub4e4\uacfc \ubcd1\uc0ac\ub4e4\uc740 \uadf8\ub4e4\uc744 \ud5a5\ud574 \ub2e4\uac00\uc624\ub294 \ub9ce\uc740 \uc218\uc758 \ud504\ub791\uc2a4\uad70\uc744 \ubcf4\uace0 \uad73\uc5b4\ubc84\ub9ac\uace0 \ub9d0\uc558\ub2e4. \ub3d9\ub9f9\uad70 \uc0ac\ub839\uad00\ub4e4\uc740 \uc774\uc81c 4\uac1c \uc885\ub300\uc758 \ubcc4\ub3d9\ub300\uc911 \uc774\ub3d9\uc774 \ub2a6\uc5c8\ub358 \ubd80\ub300\ub4e4\uc758 \uc77c\ubd80\ub97c \uadf8\ub4e4\uc758 \uce58\uc5f4\ud55c \uc804\ud22c\uc5d0 \ucc38\uc5ec\uc2dc\ud0ac \uc218 \uc788\uc5c8\ub2e4. \ud55c \uc2dc\uac04\uc758 \uc804\ud22c\ub3d9\uc548 \ub3d9\ub9f9\uad70 \uc911\uc559\uc758 \ubcd1\uc0ac\ub4e4\uc740 \uac70\uc758 \uc804\uba78\ud558\uc600\ub2e4. \ub450 \ubc88\uc9f8 \uc885\ub300\uc5d0\uc11c \ucc28\ucd9c\ub41c, \uac70\uc758 \ub300\ubd80\ubd84 \uc219\ub828\ub41c \uc624\uc2a4\ud2b8\ub9ac\uc544\uad70\uc73c\ub85c \uad6c\uc131\ub41c \uc9c0\uc6d0 \ubd80\ub300\uac00 \uc804\ud22c\uc5d0 \ucc38\uc5ec\ud558\uc5ec, \ud504\ub791\uc2a4\uad70\uc5d0\uc11c \uac00\uc7a5 \uc798 \uc2f8\uc6b0\ub358 \ubd80\ub300\uc911 \ud558\ub098\uc5d0 \uacf5\uaca9\uc744 \uac00\ud558\uc600\uace0, \uacb0\uacfc\uc801\uc73c\ub85c \uadf8\ub4e4\uc744 \uacbd\uc0ac\uc9c4 \uacf3\uc73c\ub85c \ubab0\uc544\ub123\ub294 \ub370\uc5d0 \uc131\uacf5\ud588\ub2e4. \uadf8\ub7ec\ub098 \uc808\ub9dd\uc5d0 \uc0ac\ub85c\uc7a1\ud78c \uc0dd. \ud2f8\ub808\ub974 \ubd80\ub300\uc758 \ubcd1\uc0ac\ub4e4\uc740 \ub2e4\uc2dc \ud55c \ubc88 \ub9f9\uacf5\uc744 \uac00\ud588\uace0, \ucd1d\uac80\uc804\uc744 \ubc8c\uc5ec \ub3d9\ub9f9\uad70\uc744 \uace0\uc9c0\ub300\uc5d0\uc11c \ucad3\uc544\ub0b4\uc5c8\ub2e4. \ubd81\ucabd\uc5d0\uc11c\ub294 \ubc29\ub2f4(Vandamme)\uc7a5\uad70\uc758 \ubcc4\ub3d9\ub300\uac00 \uc2a4\ud0c0\ub808 \ube44\ub178\ud750\ub77c\ub514(Star\u00e9 Vinohrady)\ub77c \ubd88\ub9ac\ub294 \uc9c0\uc5ed\uc744 \uacf5\uaca9\ud588\uace0, \uba87 \ucc28\ub840\uc758 \uad50\uc804\uacfc \uac15\ub825\ud55c \uc77c\uc81c\uc0ac\uaca9\uc744 \ud1b5\ud574 \ub3d9\ub9f9\uad70\uc744 \uada4\uc8fc\uc2dc\ucf30\ub2e4.", "answer": "\uc0dd. \ud2f8\ub808\ub974 \ubd80\ub300", "sentence": "\uadf8\ub7ec\ub098 \uc808\ub9dd\uc5d0 \uc0ac\ub85c\uc7a1\ud78c \uc0dd. \ud2f8\ub808\ub974 \ubd80\ub300 \uc758 \ubcd1\uc0ac\ub4e4\uc740 \ub2e4\uc2dc \ud55c \ubc88 \ub9f9\uacf5\uc744 \uac00\ud588\uace0, \ucd1d\uac80\uc804\uc744 \ubc8c\uc5ec \ub3d9\ub9f9\uad70\uc744 \uace0\uc9c0\ub300\uc5d0\uc11c \ucad3\uc544\ub0b4\uc5c8\ub2e4.", "paragraph_sentence": "\uc9d9\uc740 \uc548\uac1c\uc758 \ub3c4\uc6c0\uacfc \uc2dc\uc57c\uc758 \ud750\ub984\uc73c\ub85c \uc778\ud574 \uc870\uc2ec\uc2a4\ub7fd\uac8c \uc0dd. \uc77c\ub808\ub974(St. Hilaire) \uc0ac\ub2e8\uc774 \uc9c4\uad70\ud588\uc73c\ub098, \uadf8\ub4e4\uc774 \ub0b4\ub9ac\ub9c9\uae38\uc5d0 \uac14\uc744 \ub54c \uc800 \uc720\uba85\ud55c '\uc544\uc6b0\uc2a4\ud130\ub9ac\uce20\uc758 \ud0dc\uc591'(Sun of Austerlitz)\uc774 \uc548\uac1c\ub97c \uc5c6\uc560\ubc84\ub824, \uc774\ub4e4\uc774 \uc9c4\uad70\ud558\ub3c4\ub85d \uace0\ubb34\uc2dc\ucf30\ub2e4. \uace0\uc9c0\ub300 \uc704\uc5d0 \uc788\ub358 \ub7ec\uc2dc\uc544 \uc7a5\uad70\ub4e4\uacfc \ubcd1\uc0ac\ub4e4\uc740 \uadf8\ub4e4\uc744 \ud5a5\ud574 \ub2e4\uac00\uc624\ub294 \ub9ce\uc740 \uc218\uc758 \ud504\ub791\uc2a4\uad70\uc744 \ubcf4\uace0 \uad73\uc5b4\ubc84\ub9ac\uace0 \ub9d0\uc558\ub2e4. \ub3d9\ub9f9\uad70 \uc0ac\ub839\uad00\ub4e4\uc740 \uc774\uc81c 4\uac1c \uc885\ub300\uc758 \ubcc4\ub3d9\ub300\uc911 \uc774\ub3d9\uc774 \ub2a6\uc5c8\ub358 \ubd80\ub300\ub4e4\uc758 \uc77c\ubd80\ub97c \uadf8\ub4e4\uc758 \uce58\uc5f4\ud55c \uc804\ud22c\uc5d0 \ucc38\uc5ec\uc2dc\ud0ac \uc218 \uc788\uc5c8\ub2e4. \ud55c \uc2dc\uac04\uc758 \uc804\ud22c\ub3d9\uc548 \ub3d9\ub9f9\uad70 \uc911\uc559\uc758 \ubcd1\uc0ac\ub4e4\uc740 \uac70\uc758 \uc804\uba78\ud558\uc600\ub2e4. \ub450 \ubc88\uc9f8 \uc885\ub300\uc5d0\uc11c \ucc28\ucd9c\ub41c, \uac70\uc758 \ub300\ubd80\ubd84 \uc219\ub828\ub41c \uc624\uc2a4\ud2b8\ub9ac\uc544\uad70\uc73c\ub85c \uad6c\uc131\ub41c \uc9c0\uc6d0 \ubd80\ub300\uac00 \uc804\ud22c\uc5d0 \ucc38\uc5ec\ud558\uc5ec, \ud504\ub791\uc2a4\uad70\uc5d0\uc11c \uac00\uc7a5 \uc798 \uc2f8\uc6b0\ub358 \ubd80\ub300\uc911 \ud558\ub098\uc5d0 \uacf5\uaca9\uc744 \uac00\ud558\uc600\uace0, \uacb0\uacfc\uc801\uc73c\ub85c \uadf8\ub4e4\uc744 \uacbd\uc0ac\uc9c4 \uacf3\uc73c\ub85c \ubab0\uc544\ub123\ub294 \ub370\uc5d0 \uc131\uacf5\ud588\ub2e4. \uadf8\ub7ec\ub098 \uc808\ub9dd\uc5d0 \uc0ac\ub85c\uc7a1\ud78c \uc0dd. \ud2f8\ub808\ub974 \ubd80\ub300 \uc758 \ubcd1\uc0ac\ub4e4\uc740 \ub2e4\uc2dc \ud55c \ubc88 \ub9f9\uacf5\uc744 \uac00\ud588\uace0, \ucd1d\uac80\uc804\uc744 \ubc8c\uc5ec \ub3d9\ub9f9\uad70\uc744 \uace0\uc9c0\ub300\uc5d0\uc11c \ucad3\uc544\ub0b4\uc5c8\ub2e4. \ubd81\ucabd\uc5d0\uc11c\ub294 \ubc29\ub2f4(Vandamme)\uc7a5\uad70\uc758 \ubcc4\ub3d9\ub300\uac00 \uc2a4\ud0c0\ub808 \ube44\ub178\ud750\ub77c\ub514(Star\u00e9 Vinohrady)\ub77c \ubd88\ub9ac\ub294 \uc9c0\uc5ed\uc744 \uacf5\uaca9\ud588\uace0, \uba87 \ucc28\ub840\uc758 \uad50\uc804\uacfc \uac15\ub825\ud55c \uc77c\uc81c\uc0ac\uaca9\uc744 \ud1b5\ud574 \ub3d9\ub9f9\uad70\uc744 \uada4\uc8fc\uc2dc\ucf30\ub2e4.", "paragraph_answer": "\uc9d9\uc740 \uc548\uac1c\uc758 \ub3c4\uc6c0\uacfc \uc2dc\uc57c\uc758 \ud750\ub984\uc73c\ub85c \uc778\ud574 \uc870\uc2ec\uc2a4\ub7fd\uac8c \uc0dd. \uc77c\ub808\ub974(St. Hilaire) \uc0ac\ub2e8\uc774 \uc9c4\uad70\ud588\uc73c\ub098, \uadf8\ub4e4\uc774 \ub0b4\ub9ac\ub9c9\uae38\uc5d0 \uac14\uc744 \ub54c \uc800 \uc720\uba85\ud55c '\uc544\uc6b0\uc2a4\ud130\ub9ac\uce20\uc758 \ud0dc\uc591'(Sun of Austerlitz)\uc774 \uc548\uac1c\ub97c \uc5c6\uc560\ubc84\ub824, \uc774\ub4e4\uc774 \uc9c4\uad70\ud558\ub3c4\ub85d \uace0\ubb34\uc2dc\ucf30\ub2e4. \uace0\uc9c0\ub300 \uc704\uc5d0 \uc788\ub358 \ub7ec\uc2dc\uc544 \uc7a5\uad70\ub4e4\uacfc \ubcd1\uc0ac\ub4e4\uc740 \uadf8\ub4e4\uc744 \ud5a5\ud574 \ub2e4\uac00\uc624\ub294 \ub9ce\uc740 \uc218\uc758 \ud504\ub791\uc2a4\uad70\uc744 \ubcf4\uace0 \uad73\uc5b4\ubc84\ub9ac\uace0 \ub9d0\uc558\ub2e4. \ub3d9\ub9f9\uad70 \uc0ac\ub839\uad00\ub4e4\uc740 \uc774\uc81c 4\uac1c \uc885\ub300\uc758 \ubcc4\ub3d9\ub300\uc911 \uc774\ub3d9\uc774 \ub2a6\uc5c8\ub358 \ubd80\ub300\ub4e4\uc758 \uc77c\ubd80\ub97c \uadf8\ub4e4\uc758 \uce58\uc5f4\ud55c \uc804\ud22c\uc5d0 \ucc38\uc5ec\uc2dc\ud0ac \uc218 \uc788\uc5c8\ub2e4. \ud55c \uc2dc\uac04\uc758 \uc804\ud22c\ub3d9\uc548 \ub3d9\ub9f9\uad70 \uc911\uc559\uc758 \ubcd1\uc0ac\ub4e4\uc740 \uac70\uc758 \uc804\uba78\ud558\uc600\ub2e4. \ub450 \ubc88\uc9f8 \uc885\ub300\uc5d0\uc11c \ucc28\ucd9c\ub41c, \uac70\uc758 \ub300\ubd80\ubd84 \uc219\ub828\ub41c \uc624\uc2a4\ud2b8\ub9ac\uc544\uad70\uc73c\ub85c \uad6c\uc131\ub41c \uc9c0\uc6d0 \ubd80\ub300\uac00 \uc804\ud22c\uc5d0 \ucc38\uc5ec\ud558\uc5ec, \ud504\ub791\uc2a4\uad70\uc5d0\uc11c \uac00\uc7a5 \uc798 \uc2f8\uc6b0\ub358 \ubd80\ub300\uc911 \ud558\ub098\uc5d0 \uacf5\uaca9\uc744 \uac00\ud558\uc600\uace0, \uacb0\uacfc\uc801\uc73c\ub85c \uadf8\ub4e4\uc744 \uacbd\uc0ac\uc9c4 \uacf3\uc73c\ub85c \ubab0\uc544\ub123\ub294 \ub370\uc5d0 \uc131\uacf5\ud588\ub2e4. \uadf8\ub7ec\ub098 \uc808\ub9dd\uc5d0 \uc0ac\ub85c\uc7a1\ud78c \uc0dd. \ud2f8\ub808\ub974 \ubd80\ub300 \uc758 \ubcd1\uc0ac\ub4e4\uc740 \ub2e4\uc2dc \ud55c \ubc88 \ub9f9\uacf5\uc744 \uac00\ud588\uace0, \ucd1d\uac80\uc804\uc744 \ubc8c\uc5ec \ub3d9\ub9f9\uad70\uc744 \uace0\uc9c0\ub300\uc5d0\uc11c \ucad3\uc544\ub0b4\uc5c8\ub2e4. \ubd81\ucabd\uc5d0\uc11c\ub294 \ubc29\ub2f4(Vandamme)\uc7a5\uad70\uc758 \ubcc4\ub3d9\ub300\uac00 \uc2a4\ud0c0\ub808 \ube44\ub178\ud750\ub77c\ub514(Star\u00e9 Vinohrady)\ub77c \ubd88\ub9ac\ub294 \uc9c0\uc5ed\uc744 \uacf5\uaca9\ud588\uace0, \uba87 \ucc28\ub840\uc758 \uad50\uc804\uacfc \uac15\ub825\ud55c \uc77c\uc81c\uc0ac\uaca9\uc744 \ud1b5\ud574 \ub3d9\ub9f9\uad70\uc744 \uada4\uc8fc\uc2dc\ucf30\ub2e4.", "sentence_answer": "\uadf8\ub7ec\ub098 \uc808\ub9dd\uc5d0 \uc0ac\ub85c\uc7a1\ud78c \uc0dd. \ud2f8\ub808\ub974 \ubd80\ub300 \uc758 \ubcd1\uc0ac\ub4e4\uc740 \ub2e4\uc2dc \ud55c \ubc88 \ub9f9\uacf5\uc744 \uac00\ud588\uace0, \ucd1d\uac80\uc804\uc744 \ubc8c\uc5ec \ub3d9\ub9f9\uad70\uc744 \uace0\uc9c0\ub300\uc5d0\uc11c \ucad3\uc544\ub0b4\uc5c8\ub2e4.", "paragraph_id": "6533833-12-1", "paragraph_question": "question: \ub2e4\uc2dc \ub9f9\uacf5\uc744 \uac00\ud574 \ucd1d\uac80\uc804\uc744 \ubc8c\uc5ec \ub3d9\ub9f9\uad70\uc744 \uace0\uc9c0\ub300\uc5d0\uc11c \ucad3\uc544\ub0b8 \ubcd1\uc0ac\ub4e4\uc740 \uc5b4\ub5a4 \ubd80\ub300\uc5d0 \uc18d\ud574\uc788\ub294\uac00?, context: \uc9d9\uc740 \uc548\uac1c\uc758 \ub3c4\uc6c0\uacfc \uc2dc\uc57c\uc758 \ud750\ub984\uc73c\ub85c \uc778\ud574 \uc870\uc2ec\uc2a4\ub7fd\uac8c \uc0dd. \uc77c\ub808\ub974(St. Hilaire) \uc0ac\ub2e8\uc774 \uc9c4\uad70\ud588\uc73c\ub098, \uadf8\ub4e4\uc774 \ub0b4\ub9ac\ub9c9\uae38\uc5d0 \uac14\uc744 \ub54c \uc800 \uc720\uba85\ud55c '\uc544\uc6b0\uc2a4\ud130\ub9ac\uce20\uc758 \ud0dc\uc591'(Sun of Austerlitz)\uc774 \uc548\uac1c\ub97c \uc5c6\uc560\ubc84\ub824, \uc774\ub4e4\uc774 \uc9c4\uad70\ud558\ub3c4\ub85d \uace0\ubb34\uc2dc\ucf30\ub2e4. \uace0\uc9c0\ub300 \uc704\uc5d0 \uc788\ub358 \ub7ec\uc2dc\uc544 \uc7a5\uad70\ub4e4\uacfc \ubcd1\uc0ac\ub4e4\uc740 \uadf8\ub4e4\uc744 \ud5a5\ud574 \ub2e4\uac00\uc624\ub294 \ub9ce\uc740 \uc218\uc758 \ud504\ub791\uc2a4\uad70\uc744 \ubcf4\uace0 \uad73\uc5b4\ubc84\ub9ac\uace0 \ub9d0\uc558\ub2e4. \ub3d9\ub9f9\uad70 \uc0ac\ub839\uad00\ub4e4\uc740 \uc774\uc81c 4\uac1c \uc885\ub300\uc758 \ubcc4\ub3d9\ub300\uc911 \uc774\ub3d9\uc774 \ub2a6\uc5c8\ub358 \ubd80\ub300\ub4e4\uc758 \uc77c\ubd80\ub97c \uadf8\ub4e4\uc758 \uce58\uc5f4\ud55c \uc804\ud22c\uc5d0 \ucc38\uc5ec\uc2dc\ud0ac \uc218 \uc788\uc5c8\ub2e4. \ud55c \uc2dc\uac04\uc758 \uc804\ud22c\ub3d9\uc548 \ub3d9\ub9f9\uad70 \uc911\uc559\uc758 \ubcd1\uc0ac\ub4e4\uc740 \uac70\uc758 \uc804\uba78\ud558\uc600\ub2e4. \ub450 \ubc88\uc9f8 \uc885\ub300\uc5d0\uc11c \ucc28\ucd9c\ub41c, \uac70\uc758 \ub300\ubd80\ubd84 \uc219\ub828\ub41c \uc624\uc2a4\ud2b8\ub9ac\uc544\uad70\uc73c\ub85c \uad6c\uc131\ub41c \uc9c0\uc6d0 \ubd80\ub300\uac00 \uc804\ud22c\uc5d0 \ucc38\uc5ec\ud558\uc5ec, \ud504\ub791\uc2a4\uad70\uc5d0\uc11c \uac00\uc7a5 \uc798 \uc2f8\uc6b0\ub358 \ubd80\ub300\uc911 \ud558\ub098\uc5d0 \uacf5\uaca9\uc744 \uac00\ud558\uc600\uace0, \uacb0\uacfc\uc801\uc73c\ub85c \uadf8\ub4e4\uc744 \uacbd\uc0ac\uc9c4 \uacf3\uc73c\ub85c \ubab0\uc544\ub123\ub294 \ub370\uc5d0 \uc131\uacf5\ud588\ub2e4. \uadf8\ub7ec\ub098 \uc808\ub9dd\uc5d0 \uc0ac\ub85c\uc7a1\ud78c \uc0dd. \ud2f8\ub808\ub974 \ubd80\ub300\uc758 \ubcd1\uc0ac\ub4e4\uc740 \ub2e4\uc2dc \ud55c \ubc88 \ub9f9\uacf5\uc744 \uac00\ud588\uace0, \ucd1d\uac80\uc804\uc744 \ubc8c\uc5ec \ub3d9\ub9f9\uad70\uc744 \uace0\uc9c0\ub300\uc5d0\uc11c \ucad3\uc544\ub0b4\uc5c8\ub2e4. \ubd81\ucabd\uc5d0\uc11c\ub294 \ubc29\ub2f4(Vandamme)\uc7a5\uad70\uc758 \ubcc4\ub3d9\ub300\uac00 \uc2a4\ud0c0\ub808 \ube44\ub178\ud750\ub77c\ub514(Star\u00e9 Vinohrady)\ub77c \ubd88\ub9ac\ub294 \uc9c0\uc5ed\uc744 \uacf5\uaca9\ud588\uace0, \uba87 \ucc28\ub840\uc758 \uad50\uc804\uacfc \uac15\ub825\ud55c \uc77c\uc81c\uc0ac\uaca9\uc744 \ud1b5\ud574 \ub3d9\ub9f9\uad70\uc744 \uada4\uc8fc\uc2dc\ucf30\ub2e4."} {"question": "DJ\uc815\uad8c\uc774 \uc0dd\uc0b0\uc801 \ubcf5\uc9c0\ub77c\ub294 \uac1c\ub150\uc744 \ud1b5\ud574 \uc0ac\ud68c\uc548\uc804\ub9dd\uc744 \uad6c\ucd95\ud558\ub294\ub370 \uc8fc\ub825\ud588\ub2e4\uace0 \ud3c9\uac00\ud55c \uc0ac\ub78c\uc740?", "paragraph": "\uc774\ub0b4\uc601 \uace0\ub824\ub300\ud559\uad50 \uad50\uc218\ub294 \"DJ \uc815\uad8c\uc740 \ucd5c\ucd08\uc758 \uac1c\ud601\uc815\uad8c\"\uc774\ub77c\uba70 \"\uc0dd\uc0b0\uc801 \ubcf5\uc9c0\ub77c\ub294 \uac1c\ub150\uc744 \ud1b5\ud574 \uc18c\uc678\uacc4\uce35, \uc11c\ubbfc\ub4e4\uc744 \uc704\ud55c \uc0ac\ud68c\uc548\uc804\ub9dd\uc744 \uad6c\ucd95\ud558\ub294\ub370 \uc8fc\ub825\ud588\ub2e4\"\uba70 \"\uadf8 \uc804\uae4c\uc9c0\ub9cc \ud574\ub3c4 \ubcf5\uc9c0\ubb38\uc81c\ub294 \uc720\ub7fd\ucc98\ub7fc \uc0dd\uc0b0\uc131\uc774\ub098 \uacbd\uc7c1\ub825\uc744 \uc57d\ud654\uc2dc\ud0a4\ub294 \uc77c\ubc29\uc801 \uac1c\ub150\uc5d0 \uba38\ubb3c\ub800\ub2e4\"\uba70 \"\ubcf5\uc9c0\uc99d\uc9c4\uc744 \uacbd\uc81c\ubc1c\uc804, \uacbd\uc7c1\ub825 \ud5a5\uc0c1\uacfc \ub3d9\uc2dc\uc5d0 \ucd94\uad6c, \uacfc\uac70\uc640 \ucc28\ubcc4\ud654\ud55c \uc815\ucc45\uc744 \ubcf4\uc5ec\uc92c\ub2e4\"\ub77c\uace0 \ud3c9\uac00\ud588\ub2e4. \uc774\uc815\ud76c \ud55c\uad6d\uc678\uad6d\uc5b4\ub300\ud559\uad50 \uad50\uc218\ub294 \"\uace0\uc778\uc740 \ub178\ubca8 \ud3c9\ud654\uc0c1\uc744 \uc218\uc0c1\ud560 \ub9cc\ud07c \uc138\uacc4\uc801 \ubc18\uc5f4\uc5d0 \uc120 \uc9c0\ub3c4\uc790\uc600\uace0 \ud55c\uad6d\uc758 \uc704\uc0c1\uc744 \ub192\uc600\ub2e4\"\ub77c\uace0 \ud3c9\uac00\ud588\ub2e4. \uc131\uade0\uad00\ub300\ud559\uad50 \uae40\uc131\uc8fc \uad50\uc218\ub294 \"\uc804\uc9c1 \ub300\ud1b5\ub839\uc744 \ud3ec\ud568, \ub204\uad6c \ud558\ub098 \ucc45\uc784\uc9c0\uc9c0 \uc54a\ub294 \uc0c1\ud669\uc5d0\uc11c DJ\ub294 \uad6d\uac00\ud30c\uc0b0\uc73c\ub85c \uc774\uc5b4\uc9c8 \uc218 \uc788\uc5c8\ub358 \uad6d\ub09c \uadf9\ubcf5\uc744 \uc704\ud574 \ud63c\uc2e0\uc758 \ud798\uc744 \ub2e4\ud588\ub2e4. \uadf8 \uacb0\uacfc 2\ub144 \ub9cc\uc5d0 \uc678\ud658\uc704\uae30\ub97c \ub118\uae38 \uc218 \uc788\uc5c8\ub2e4. \uc774\ub294 IMF \uad00\ub9ac\uccb4\uc81c \uadf9\ubcf5\uc744 \uc815\ucc45\uc758 \ucd5c\uc6b0\uc120\uc73c\ub85c \ud55c \uae40\ub300\uc911 \uc815\ubd80\uc640 \ub178\uc0ac \uac04 \ud654\ud574 \ub178\ub825\uc73c\ub85c \uac00\ub2a5\ud560 \uc218 \uc788\uc5c8\ub2e4. \ub610\ud55c DJ\ub294 \uad6d\ubbfc\ud654\ud569\uc744 \uc704\ud574 \uc6a9\uc11c\uc640 \ud654\ud574\uc758 \uc815\uce58\ub97c \ud3c8\ub2e4\"\ub77c\uace0 \ud3c9\uac00\ud588\ub2e4.", "answer": "\uc774\ub0b4\uc601", "sentence": "\uc774\ub0b4\uc601 \uace0\ub824\ub300\ud559\uad50 \uad50\uc218\ub294 \"DJ \uc815\uad8c\uc740 \ucd5c\ucd08\uc758 \uac1c\ud601\uc815\uad8c\"\uc774\ub77c\uba70 \"\uc0dd\uc0b0\uc801 \ubcf5\uc9c0\ub77c\ub294 \uac1c\ub150\uc744 \ud1b5\ud574 \uc18c\uc678\uacc4\uce35, \uc11c\ubbfc\ub4e4\uc744 \uc704\ud55c \uc0ac\ud68c\uc548\uc804\ub9dd\uc744 \uad6c\ucd95\ud558\ub294\ub370 \uc8fc\ub825\ud588\ub2e4\"\uba70 \"\uadf8 \uc804\uae4c\uc9c0\ub9cc \ud574\ub3c4 \ubcf5\uc9c0\ubb38\uc81c\ub294 \uc720\ub7fd\ucc98\ub7fc \uc0dd\uc0b0\uc131\uc774\ub098 \uacbd\uc7c1\ub825\uc744 \uc57d\ud654\uc2dc\ud0a4\ub294 \uc77c\ubc29\uc801 \uac1c\ub150\uc5d0 \uba38\ubb3c\ub800\ub2e4\"\uba70 \"\ubcf5\uc9c0\uc99d\uc9c4\uc744 \uacbd\uc81c\ubc1c\uc804, \uacbd\uc7c1\ub825 \ud5a5\uc0c1\uacfc \ub3d9\uc2dc\uc5d0 \ucd94\uad6c, \uacfc\uac70\uc640 \ucc28\ubcc4\ud654\ud55c \uc815\ucc45\uc744 \ubcf4\uc5ec\uc92c\ub2e4\"\ub77c\uace0 \ud3c9\uac00\ud588\ub2e4.", "paragraph_sentence": " \uc774\ub0b4\uc601 \uace0\ub824\ub300\ud559\uad50 \uad50\uc218\ub294 \"DJ \uc815\uad8c\uc740 \ucd5c\ucd08\uc758 \uac1c\ud601\uc815\uad8c\"\uc774\ub77c\uba70 \"\uc0dd\uc0b0\uc801 \ubcf5\uc9c0\ub77c\ub294 \uac1c\ub150\uc744 \ud1b5\ud574 \uc18c\uc678\uacc4\uce35, \uc11c\ubbfc\ub4e4\uc744 \uc704\ud55c \uc0ac\ud68c\uc548\uc804\ub9dd\uc744 \uad6c\ucd95\ud558\ub294\ub370 \uc8fc\ub825\ud588\ub2e4\"\uba70 \"\uadf8 \uc804\uae4c\uc9c0\ub9cc \ud574\ub3c4 \ubcf5\uc9c0\ubb38\uc81c\ub294 \uc720\ub7fd\ucc98\ub7fc \uc0dd\uc0b0\uc131\uc774\ub098 \uacbd\uc7c1\ub825\uc744 \uc57d\ud654\uc2dc\ud0a4\ub294 \uc77c\ubc29\uc801 \uac1c\ub150\uc5d0 \uba38\ubb3c\ub800\ub2e4\"\uba70 \"\ubcf5\uc9c0\uc99d\uc9c4\uc744 \uacbd\uc81c\ubc1c\uc804, \uacbd\uc7c1\ub825 \ud5a5\uc0c1\uacfc \ub3d9\uc2dc\uc5d0 \ucd94\uad6c, \uacfc\uac70\uc640 \ucc28\ubcc4\ud654\ud55c \uc815\ucc45\uc744 \ubcf4\uc5ec\uc92c\ub2e4\"\ub77c\uace0 \ud3c9\uac00\ud588\ub2e4. \uc774\uc815\ud76c \ud55c\uad6d\uc678\uad6d\uc5b4\ub300\ud559\uad50 \uad50\uc218\ub294 \"\uace0\uc778\uc740 \ub178\ubca8 \ud3c9\ud654\uc0c1\uc744 \uc218\uc0c1\ud560 \ub9cc\ud07c \uc138\uacc4\uc801 \ubc18\uc5f4\uc5d0 \uc120 \uc9c0\ub3c4\uc790\uc600\uace0 \ud55c\uad6d\uc758 \uc704\uc0c1\uc744 \ub192\uc600\ub2e4\"\ub77c\uace0 \ud3c9\uac00\ud588\ub2e4. \uc131\uade0\uad00\ub300\ud559\uad50 \uae40\uc131\uc8fc \uad50\uc218\ub294 \"\uc804\uc9c1 \ub300\ud1b5\ub839\uc744 \ud3ec\ud568, \ub204\uad6c \ud558\ub098 \ucc45\uc784\uc9c0\uc9c0 \uc54a\ub294 \uc0c1\ud669\uc5d0\uc11c DJ\ub294 \uad6d\uac00\ud30c\uc0b0\uc73c\ub85c \uc774\uc5b4\uc9c8 \uc218 \uc788\uc5c8\ub358 \uad6d\ub09c \uadf9\ubcf5\uc744 \uc704\ud574 \ud63c\uc2e0\uc758 \ud798\uc744 \ub2e4\ud588\ub2e4. \uadf8 \uacb0\uacfc 2\ub144 \ub9cc\uc5d0 \uc678\ud658\uc704\uae30\ub97c \ub118\uae38 \uc218 \uc788\uc5c8\ub2e4. \uc774\ub294 IMF \uad00\ub9ac\uccb4\uc81c \uadf9\ubcf5\uc744 \uc815\ucc45\uc758 \ucd5c\uc6b0\uc120\uc73c\ub85c \ud55c \uae40\ub300\uc911 \uc815\ubd80\uc640 \ub178\uc0ac \uac04 \ud654\ud574 \ub178\ub825\uc73c\ub85c \uac00\ub2a5\ud560 \uc218 \uc788\uc5c8\ub2e4. \ub610\ud55c DJ\ub294 \uad6d\ubbfc\ud654\ud569\uc744 \uc704\ud574 \uc6a9\uc11c\uc640 \ud654\ud574\uc758 \uc815\uce58\ub97c \ud3c8\ub2e4\"\ub77c\uace0 \ud3c9\uac00\ud588\ub2e4.", "paragraph_answer": " \uc774\ub0b4\uc601 \uace0\ub824\ub300\ud559\uad50 \uad50\uc218\ub294 \"DJ \uc815\uad8c\uc740 \ucd5c\ucd08\uc758 \uac1c\ud601\uc815\uad8c\"\uc774\ub77c\uba70 \"\uc0dd\uc0b0\uc801 \ubcf5\uc9c0\ub77c\ub294 \uac1c\ub150\uc744 \ud1b5\ud574 \uc18c\uc678\uacc4\uce35, \uc11c\ubbfc\ub4e4\uc744 \uc704\ud55c \uc0ac\ud68c\uc548\uc804\ub9dd\uc744 \uad6c\ucd95\ud558\ub294\ub370 \uc8fc\ub825\ud588\ub2e4\"\uba70 \"\uadf8 \uc804\uae4c\uc9c0\ub9cc \ud574\ub3c4 \ubcf5\uc9c0\ubb38\uc81c\ub294 \uc720\ub7fd\ucc98\ub7fc \uc0dd\uc0b0\uc131\uc774\ub098 \uacbd\uc7c1\ub825\uc744 \uc57d\ud654\uc2dc\ud0a4\ub294 \uc77c\ubc29\uc801 \uac1c\ub150\uc5d0 \uba38\ubb3c\ub800\ub2e4\"\uba70 \"\ubcf5\uc9c0\uc99d\uc9c4\uc744 \uacbd\uc81c\ubc1c\uc804, \uacbd\uc7c1\ub825 \ud5a5\uc0c1\uacfc \ub3d9\uc2dc\uc5d0 \ucd94\uad6c, \uacfc\uac70\uc640 \ucc28\ubcc4\ud654\ud55c \uc815\ucc45\uc744 \ubcf4\uc5ec\uc92c\ub2e4\"\ub77c\uace0 \ud3c9\uac00\ud588\ub2e4. \uc774\uc815\ud76c \ud55c\uad6d\uc678\uad6d\uc5b4\ub300\ud559\uad50 \uad50\uc218\ub294 \"\uace0\uc778\uc740 \ub178\ubca8 \ud3c9\ud654\uc0c1\uc744 \uc218\uc0c1\ud560 \ub9cc\ud07c \uc138\uacc4\uc801 \ubc18\uc5f4\uc5d0 \uc120 \uc9c0\ub3c4\uc790\uc600\uace0 \ud55c\uad6d\uc758 \uc704\uc0c1\uc744 \ub192\uc600\ub2e4\"\ub77c\uace0 \ud3c9\uac00\ud588\ub2e4. \uc131\uade0\uad00\ub300\ud559\uad50 \uae40\uc131\uc8fc \uad50\uc218\ub294 \"\uc804\uc9c1 \ub300\ud1b5\ub839\uc744 \ud3ec\ud568, \ub204\uad6c \ud558\ub098 \ucc45\uc784\uc9c0\uc9c0 \uc54a\ub294 \uc0c1\ud669\uc5d0\uc11c DJ\ub294 \uad6d\uac00\ud30c\uc0b0\uc73c\ub85c \uc774\uc5b4\uc9c8 \uc218 \uc788\uc5c8\ub358 \uad6d\ub09c \uadf9\ubcf5\uc744 \uc704\ud574 \ud63c\uc2e0\uc758 \ud798\uc744 \ub2e4\ud588\ub2e4. \uadf8 \uacb0\uacfc 2\ub144 \ub9cc\uc5d0 \uc678\ud658\uc704\uae30\ub97c \ub118\uae38 \uc218 \uc788\uc5c8\ub2e4. \uc774\ub294 IMF \uad00\ub9ac\uccb4\uc81c \uadf9\ubcf5\uc744 \uc815\ucc45\uc758 \ucd5c\uc6b0\uc120\uc73c\ub85c \ud55c \uae40\ub300\uc911 \uc815\ubd80\uc640 \ub178\uc0ac \uac04 \ud654\ud574 \ub178\ub825\uc73c\ub85c \uac00\ub2a5\ud560 \uc218 \uc788\uc5c8\ub2e4. \ub610\ud55c DJ\ub294 \uad6d\ubbfc\ud654\ud569\uc744 \uc704\ud574 \uc6a9\uc11c\uc640 \ud654\ud574\uc758 \uc815\uce58\ub97c \ud3c8\ub2e4\"\ub77c\uace0 \ud3c9\uac00\ud588\ub2e4.", "sentence_answer": " \uc774\ub0b4\uc601 \uace0\ub824\ub300\ud559\uad50 \uad50\uc218\ub294 \"DJ \uc815\uad8c\uc740 \ucd5c\ucd08\uc758 \uac1c\ud601\uc815\uad8c\"\uc774\ub77c\uba70 \"\uc0dd\uc0b0\uc801 \ubcf5\uc9c0\ub77c\ub294 \uac1c\ub150\uc744 \ud1b5\ud574 \uc18c\uc678\uacc4\uce35, \uc11c\ubbfc\ub4e4\uc744 \uc704\ud55c \uc0ac\ud68c\uc548\uc804\ub9dd\uc744 \uad6c\ucd95\ud558\ub294\ub370 \uc8fc\ub825\ud588\ub2e4\"\uba70 \"\uadf8 \uc804\uae4c\uc9c0\ub9cc \ud574\ub3c4 \ubcf5\uc9c0\ubb38\uc81c\ub294 \uc720\ub7fd\ucc98\ub7fc \uc0dd\uc0b0\uc131\uc774\ub098 \uacbd\uc7c1\ub825\uc744 \uc57d\ud654\uc2dc\ud0a4\ub294 \uc77c\ubc29\uc801 \uac1c\ub150\uc5d0 \uba38\ubb3c\ub800\ub2e4\"\uba70 \"\ubcf5\uc9c0\uc99d\uc9c4\uc744 \uacbd\uc81c\ubc1c\uc804, \uacbd\uc7c1\ub825 \ud5a5\uc0c1\uacfc \ub3d9\uc2dc\uc5d0 \ucd94\uad6c, \uacfc\uac70\uc640 \ucc28\ubcc4\ud654\ud55c \uc815\ucc45\uc744 \ubcf4\uc5ec\uc92c\ub2e4\"\ub77c\uace0 \ud3c9\uac00\ud588\ub2e4.", "paragraph_id": "6576830-55-0", "paragraph_question": "question: DJ\uc815\uad8c\uc774 \uc0dd\uc0b0\uc801 \ubcf5\uc9c0\ub77c\ub294 \uac1c\ub150\uc744 \ud1b5\ud574 \uc0ac\ud68c\uc548\uc804\ub9dd\uc744 \uad6c\ucd95\ud558\ub294\ub370 \uc8fc\ub825\ud588\ub2e4\uace0 \ud3c9\uac00\ud55c \uc0ac\ub78c\uc740?, context: \uc774\ub0b4\uc601 \uace0\ub824\ub300\ud559\uad50 \uad50\uc218\ub294 \"DJ \uc815\uad8c\uc740 \ucd5c\ucd08\uc758 \uac1c\ud601\uc815\uad8c\"\uc774\ub77c\uba70 \"\uc0dd\uc0b0\uc801 \ubcf5\uc9c0\ub77c\ub294 \uac1c\ub150\uc744 \ud1b5\ud574 \uc18c\uc678\uacc4\uce35, \uc11c\ubbfc\ub4e4\uc744 \uc704\ud55c \uc0ac\ud68c\uc548\uc804\ub9dd\uc744 \uad6c\ucd95\ud558\ub294\ub370 \uc8fc\ub825\ud588\ub2e4\"\uba70 \"\uadf8 \uc804\uae4c\uc9c0\ub9cc \ud574\ub3c4 \ubcf5\uc9c0\ubb38\uc81c\ub294 \uc720\ub7fd\ucc98\ub7fc \uc0dd\uc0b0\uc131\uc774\ub098 \uacbd\uc7c1\ub825\uc744 \uc57d\ud654\uc2dc\ud0a4\ub294 \uc77c\ubc29\uc801 \uac1c\ub150\uc5d0 \uba38\ubb3c\ub800\ub2e4\"\uba70 \"\ubcf5\uc9c0\uc99d\uc9c4\uc744 \uacbd\uc81c\ubc1c\uc804, \uacbd\uc7c1\ub825 \ud5a5\uc0c1\uacfc \ub3d9\uc2dc\uc5d0 \ucd94\uad6c, \uacfc\uac70\uc640 \ucc28\ubcc4\ud654\ud55c \uc815\ucc45\uc744 \ubcf4\uc5ec\uc92c\ub2e4\"\ub77c\uace0 \ud3c9\uac00\ud588\ub2e4. \uc774\uc815\ud76c \ud55c\uad6d\uc678\uad6d\uc5b4\ub300\ud559\uad50 \uad50\uc218\ub294 \"\uace0\uc778\uc740 \ub178\ubca8 \ud3c9\ud654\uc0c1\uc744 \uc218\uc0c1\ud560 \ub9cc\ud07c \uc138\uacc4\uc801 \ubc18\uc5f4\uc5d0 \uc120 \uc9c0\ub3c4\uc790\uc600\uace0 \ud55c\uad6d\uc758 \uc704\uc0c1\uc744 \ub192\uc600\ub2e4\"\ub77c\uace0 \ud3c9\uac00\ud588\ub2e4. \uc131\uade0\uad00\ub300\ud559\uad50 \uae40\uc131\uc8fc \uad50\uc218\ub294 \"\uc804\uc9c1 \ub300\ud1b5\ub839\uc744 \ud3ec\ud568, \ub204\uad6c \ud558\ub098 \ucc45\uc784\uc9c0\uc9c0 \uc54a\ub294 \uc0c1\ud669\uc5d0\uc11c DJ\ub294 \uad6d\uac00\ud30c\uc0b0\uc73c\ub85c \uc774\uc5b4\uc9c8 \uc218 \uc788\uc5c8\ub358 \uad6d\ub09c \uadf9\ubcf5\uc744 \uc704\ud574 \ud63c\uc2e0\uc758 \ud798\uc744 \ub2e4\ud588\ub2e4. \uadf8 \uacb0\uacfc 2\ub144 \ub9cc\uc5d0 \uc678\ud658\uc704\uae30\ub97c \ub118\uae38 \uc218 \uc788\uc5c8\ub2e4. \uc774\ub294 IMF \uad00\ub9ac\uccb4\uc81c \uadf9\ubcf5\uc744 \uc815\ucc45\uc758 \ucd5c\uc6b0\uc120\uc73c\ub85c \ud55c \uae40\ub300\uc911 \uc815\ubd80\uc640 \ub178\uc0ac \uac04 \ud654\ud574 \ub178\ub825\uc73c\ub85c \uac00\ub2a5\ud560 \uc218 \uc788\uc5c8\ub2e4. \ub610\ud55c DJ\ub294 \uad6d\ubbfc\ud654\ud569\uc744 \uc704\ud574 \uc6a9\uc11c\uc640 \ud654\ud574\uc758 \uc815\uce58\ub97c \ud3c8\ub2e4\"\ub77c\uace0 \ud3c9\uac00\ud588\ub2e4."} {"question": "\uc5ed\uc0ac\ud559\uacc4 \uc8fc\ub958\uc5d0\uc11c \uc1a1\ub098\ub77c\ub2f9\uc2dc\uc758 \uc870\uc815\uc774 \ub204\uad6c\ub97c \uc6b0\ub300\ud558\uc5ec \ubb38\uc57d\ud574\uc84c\ub2e4\uace0 \uc8fc\uc7a5\ud558\uc600\ub098?", "paragraph": "\ud669\uc81c\ub294 \uc815\ucc45\uc744 \uc218\ub9bd\ud560 \uc218 \uc788\uc5c8\uace0, \ub3c4\ud559 \uc815\uce58\uc640 \uc81c\ub840\ub97c \ud589\ud558\uac70\ub098 \ucde8\ubbf8 \uadf8\ub9ac\uace0 \uc5ec\uc790 \ub610\ud55c \uc790\uc2e0 \ub9c8\uc74c\ub300\ub85c \uc815\ud558\uace0 \uace0\ub97c \uc218 \uc788\uc5c8\ub2e4. \uadf8\ub7ec\ub098 \ud504\ub808\ub354\ub9ad W. \ubaa8\ud2b8(Frederick W. Mote)\ub294 \ub300\ubd80\ubd84\uc758 \uc1a1\ub098\ub77c \ud669\uc81c\ub4e4\uc774 \uc720\ub144 \uc2dc\uc808\uc744 \ud654\ub824\ud558\uc9c0\ub9cc \ucca0\uc800\ud788 \uace0\ub9bd\ub41c \uad6c\uc911\uad81\uad90\uc5d0\uc11c \uad81\uc911 \uc0dd\ud65c\uc744 \ud558\ub2e4\ubcf4\ub2c8 \uc548\ub77d\ud55c \uc0dd\ud65c\ub9cc\uc744 \ucd94\uad6c\ud558\uc600\uace0 \uc774\ub85c \uc778\ud574 \uc815\uce58\uc5d0 \uad00\uc2ec\uc744 \uc783\uc5b4 \uc815\uc0ac \uc77c\uccb4\ub97c \uc870\uc815\uc5d0 \ub5a0\ub118\uae30\uace0 \uadf8\uc5d0 \uc758\uc9c0\ud588\ub2e4\uace0 \uc8fc\uc7a5\ud55c\ub2e4. \uc5ed\uc0ac\ud559\uacc4 \uc8fc\ub958\uc5d0\uc11c\ub294 \uc8fc\ub85c \uc1a1\ub098\ub77c \ub2f9\uc2dc\uc758 \uc870\uc815\uc774 \uc911\uad6d\uc758 \uc5b4\ub290 \uc655\uc870\ubcf4\ub2e4 \ubb38\uad00\ub4e4\uc744 \uc6b0\ub300\ud558\uc5ec \uc1a1\ub098\ub77c\uac00 \uc774\ub85c \uc778\ud574 \ubb38\uc57d\ud574\uc84c\ub2e4\ub294 \uc2dc\uac01\uc744 \uacac\uc9c0\ud558\uc9c0\ub9cc, \ub610\ub2e4\ub978 \uc758\uacac\uc73c\ub85c\ub294 \uc870\ud68c\uc5d0\uc11c \ud669\uc81c\ub97c \ub300\ud560 \ub54c, \ub300\uc2e0\ub4e4\uc758 \uc9c0\ub098\uce5c \uc874\uc911\uacfc \uc608\uc6b0\uac00 \uacb0\uacfc\uc801\uc73c\ub85c \ud669\uc81c\uac00 \uc815\uce58\uc5d0 \uad00\uc2ec\uc744 \uc783\uc5b4 \ud669\uc81c\uc640 \uc870\uc815 \uc2e0\ub8cc\uac04\uc758 \uae34\ubc00\ud55c \uad00\uacc4\ub97c \uc57d\ud654\uc2dc\ucf30\ub2e4\uace0\ub3c4 \ubcf8\ub2e4.", "answer": "\ubb38\uad00", "sentence": "\uc5ed\uc0ac\ud559\uacc4 \uc8fc\ub958\uc5d0\uc11c\ub294 \uc8fc\ub85c \uc1a1\ub098\ub77c \ub2f9\uc2dc\uc758 \uc870\uc815\uc774 \uc911\uad6d\uc758 \uc5b4\ub290 \uc655\uc870\ubcf4\ub2e4 \ubb38\uad00 \ub4e4\uc744 \uc6b0\ub300\ud558\uc5ec \uc1a1\ub098\ub77c\uac00 \uc774\ub85c \uc778\ud574 \ubb38\uc57d\ud574\uc84c\ub2e4\ub294 \uc2dc\uac01\uc744 \uacac\uc9c0\ud558\uc9c0\ub9cc, \ub610\ub2e4\ub978 \uc758\uacac\uc73c\ub85c\ub294 \uc870\ud68c\uc5d0\uc11c \ud669\uc81c\ub97c \ub300\ud560 \ub54c, \ub300\uc2e0\ub4e4\uc758 \uc9c0\ub098\uce5c \uc874\uc911\uacfc \uc608\uc6b0\uac00 \uacb0\uacfc\uc801\uc73c\ub85c \ud669\uc81c\uac00 \uc815\uce58\uc5d0 \uad00\uc2ec\uc744 \uc783\uc5b4 \ud669\uc81c\uc640 \uc870\uc815 \uc2e0\ub8cc\uac04\uc758 \uae34\ubc00\ud55c \uad00\uacc4\ub97c \uc57d\ud654\uc2dc\ucf30\ub2e4\uace0\ub3c4 \ubcf8\ub2e4.", "paragraph_sentence": "\ud669\uc81c\ub294 \uc815\ucc45\uc744 \uc218\ub9bd\ud560 \uc218 \uc788\uc5c8\uace0, \ub3c4\ud559 \uc815\uce58\uc640 \uc81c\ub840\ub97c \ud589\ud558\uac70\ub098 \ucde8\ubbf8 \uadf8\ub9ac\uace0 \uc5ec\uc790 \ub610\ud55c \uc790\uc2e0 \ub9c8\uc74c\ub300\ub85c \uc815\ud558\uace0 \uace0\ub97c \uc218 \uc788\uc5c8\ub2e4. \uadf8\ub7ec\ub098 \ud504\ub808\ub354\ub9ad W. \ubaa8\ud2b8(Frederick W. Mote)\ub294 \ub300\ubd80\ubd84\uc758 \uc1a1\ub098\ub77c \ud669\uc81c\ub4e4\uc774 \uc720\ub144 \uc2dc\uc808\uc744 \ud654\ub824\ud558\uc9c0\ub9cc \ucca0\uc800\ud788 \uace0\ub9bd\ub41c \uad6c\uc911\uad81\uad90\uc5d0\uc11c \uad81\uc911 \uc0dd\ud65c\uc744 \ud558\ub2e4\ubcf4\ub2c8 \uc548\ub77d\ud55c \uc0dd\ud65c\ub9cc\uc744 \ucd94\uad6c\ud558\uc600\uace0 \uc774\ub85c \uc778\ud574 \uc815\uce58\uc5d0 \uad00\uc2ec\uc744 \uc783\uc5b4 \uc815\uc0ac \uc77c\uccb4\ub97c \uc870\uc815\uc5d0 \ub5a0\ub118\uae30\uace0 \uadf8\uc5d0 \uc758\uc9c0\ud588\ub2e4\uace0 \uc8fc\uc7a5\ud55c\ub2e4. \uc5ed\uc0ac\ud559\uacc4 \uc8fc\ub958\uc5d0\uc11c\ub294 \uc8fc\ub85c \uc1a1\ub098\ub77c \ub2f9\uc2dc\uc758 \uc870\uc815\uc774 \uc911\uad6d\uc758 \uc5b4\ub290 \uc655\uc870\ubcf4\ub2e4 \ubb38\uad00 \ub4e4\uc744 \uc6b0\ub300\ud558\uc5ec \uc1a1\ub098\ub77c\uac00 \uc774\ub85c \uc778\ud574 \ubb38\uc57d\ud574\uc84c\ub2e4\ub294 \uc2dc\uac01\uc744 \uacac\uc9c0\ud558\uc9c0\ub9cc, \ub610\ub2e4\ub978 \uc758\uacac\uc73c\ub85c\ub294 \uc870\ud68c\uc5d0\uc11c \ud669\uc81c\ub97c \ub300\ud560 \ub54c, \ub300\uc2e0\ub4e4\uc758 \uc9c0\ub098\uce5c \uc874\uc911\uacfc \uc608\uc6b0\uac00 \uacb0\uacfc\uc801\uc73c\ub85c \ud669\uc81c\uac00 \uc815\uce58\uc5d0 \uad00\uc2ec\uc744 \uc783\uc5b4 \ud669\uc81c\uc640 \uc870\uc815 \uc2e0\ub8cc\uac04\uc758 \uae34\ubc00\ud55c \uad00\uacc4\ub97c \uc57d\ud654\uc2dc\ucf30\ub2e4\uace0\ub3c4 \ubcf8\ub2e4. ", "paragraph_answer": "\ud669\uc81c\ub294 \uc815\ucc45\uc744 \uc218\ub9bd\ud560 \uc218 \uc788\uc5c8\uace0, \ub3c4\ud559 \uc815\uce58\uc640 \uc81c\ub840\ub97c \ud589\ud558\uac70\ub098 \ucde8\ubbf8 \uadf8\ub9ac\uace0 \uc5ec\uc790 \ub610\ud55c \uc790\uc2e0 \ub9c8\uc74c\ub300\ub85c \uc815\ud558\uace0 \uace0\ub97c \uc218 \uc788\uc5c8\ub2e4. \uadf8\ub7ec\ub098 \ud504\ub808\ub354\ub9ad W. \ubaa8\ud2b8(Frederick W. Mote)\ub294 \ub300\ubd80\ubd84\uc758 \uc1a1\ub098\ub77c \ud669\uc81c\ub4e4\uc774 \uc720\ub144 \uc2dc\uc808\uc744 \ud654\ub824\ud558\uc9c0\ub9cc \ucca0\uc800\ud788 \uace0\ub9bd\ub41c \uad6c\uc911\uad81\uad90\uc5d0\uc11c \uad81\uc911 \uc0dd\ud65c\uc744 \ud558\ub2e4\ubcf4\ub2c8 \uc548\ub77d\ud55c \uc0dd\ud65c\ub9cc\uc744 \ucd94\uad6c\ud558\uc600\uace0 \uc774\ub85c \uc778\ud574 \uc815\uce58\uc5d0 \uad00\uc2ec\uc744 \uc783\uc5b4 \uc815\uc0ac \uc77c\uccb4\ub97c \uc870\uc815\uc5d0 \ub5a0\ub118\uae30\uace0 \uadf8\uc5d0 \uc758\uc9c0\ud588\ub2e4\uace0 \uc8fc\uc7a5\ud55c\ub2e4. \uc5ed\uc0ac\ud559\uacc4 \uc8fc\ub958\uc5d0\uc11c\ub294 \uc8fc\ub85c \uc1a1\ub098\ub77c \ub2f9\uc2dc\uc758 \uc870\uc815\uc774 \uc911\uad6d\uc758 \uc5b4\ub290 \uc655\uc870\ubcf4\ub2e4 \ubb38\uad00 \ub4e4\uc744 \uc6b0\ub300\ud558\uc5ec \uc1a1\ub098\ub77c\uac00 \uc774\ub85c \uc778\ud574 \ubb38\uc57d\ud574\uc84c\ub2e4\ub294 \uc2dc\uac01\uc744 \uacac\uc9c0\ud558\uc9c0\ub9cc, \ub610\ub2e4\ub978 \uc758\uacac\uc73c\ub85c\ub294 \uc870\ud68c\uc5d0\uc11c \ud669\uc81c\ub97c \ub300\ud560 \ub54c, \ub300\uc2e0\ub4e4\uc758 \uc9c0\ub098\uce5c \uc874\uc911\uacfc \uc608\uc6b0\uac00 \uacb0\uacfc\uc801\uc73c\ub85c \ud669\uc81c\uac00 \uc815\uce58\uc5d0 \uad00\uc2ec\uc744 \uc783\uc5b4 \ud669\uc81c\uc640 \uc870\uc815 \uc2e0\ub8cc\uac04\uc758 \uae34\ubc00\ud55c \uad00\uacc4\ub97c \uc57d\ud654\uc2dc\ucf30\ub2e4\uace0\ub3c4 \ubcf8\ub2e4.", "sentence_answer": "\uc5ed\uc0ac\ud559\uacc4 \uc8fc\ub958\uc5d0\uc11c\ub294 \uc8fc\ub85c \uc1a1\ub098\ub77c \ub2f9\uc2dc\uc758 \uc870\uc815\uc774 \uc911\uad6d\uc758 \uc5b4\ub290 \uc655\uc870\ubcf4\ub2e4 \ubb38\uad00 \ub4e4\uc744 \uc6b0\ub300\ud558\uc5ec \uc1a1\ub098\ub77c\uac00 \uc774\ub85c \uc778\ud574 \ubb38\uc57d\ud574\uc84c\ub2e4\ub294 \uc2dc\uac01\uc744 \uacac\uc9c0\ud558\uc9c0\ub9cc, \ub610\ub2e4\ub978 \uc758\uacac\uc73c\ub85c\ub294 \uc870\ud68c\uc5d0\uc11c \ud669\uc81c\ub97c \ub300\ud560 \ub54c, \ub300\uc2e0\ub4e4\uc758 \uc9c0\ub098\uce5c \uc874\uc911\uacfc \uc608\uc6b0\uac00 \uacb0\uacfc\uc801\uc73c\ub85c \ud669\uc81c\uac00 \uc815\uce58\uc5d0 \uad00\uc2ec\uc744 \uc783\uc5b4 \ud669\uc81c\uc640 \uc870\uc815 \uc2e0\ub8cc\uac04\uc758 \uae34\ubc00\ud55c \uad00\uacc4\ub97c \uc57d\ud654\uc2dc\ucf30\ub2e4\uace0\ub3c4 \ubcf8\ub2e4.", "paragraph_id": "6518418-6-1", "paragraph_question": "question: \uc5ed\uc0ac\ud559\uacc4 \uc8fc\ub958\uc5d0\uc11c \uc1a1\ub098\ub77c\ub2f9\uc2dc\uc758 \uc870\uc815\uc774 \ub204\uad6c\ub97c \uc6b0\ub300\ud558\uc5ec \ubb38\uc57d\ud574\uc84c\ub2e4\uace0 \uc8fc\uc7a5\ud558\uc600\ub098?, context: \ud669\uc81c\ub294 \uc815\ucc45\uc744 \uc218\ub9bd\ud560 \uc218 \uc788\uc5c8\uace0, \ub3c4\ud559 \uc815\uce58\uc640 \uc81c\ub840\ub97c \ud589\ud558\uac70\ub098 \ucde8\ubbf8 \uadf8\ub9ac\uace0 \uc5ec\uc790 \ub610\ud55c \uc790\uc2e0 \ub9c8\uc74c\ub300\ub85c \uc815\ud558\uace0 \uace0\ub97c \uc218 \uc788\uc5c8\ub2e4. \uadf8\ub7ec\ub098 \ud504\ub808\ub354\ub9ad W. \ubaa8\ud2b8(Frederick W. Mote)\ub294 \ub300\ubd80\ubd84\uc758 \uc1a1\ub098\ub77c \ud669\uc81c\ub4e4\uc774 \uc720\ub144 \uc2dc\uc808\uc744 \ud654\ub824\ud558\uc9c0\ub9cc \ucca0\uc800\ud788 \uace0\ub9bd\ub41c \uad6c\uc911\uad81\uad90\uc5d0\uc11c \uad81\uc911 \uc0dd\ud65c\uc744 \ud558\ub2e4\ubcf4\ub2c8 \uc548\ub77d\ud55c \uc0dd\ud65c\ub9cc\uc744 \ucd94\uad6c\ud558\uc600\uace0 \uc774\ub85c \uc778\ud574 \uc815\uce58\uc5d0 \uad00\uc2ec\uc744 \uc783\uc5b4 \uc815\uc0ac \uc77c\uccb4\ub97c \uc870\uc815\uc5d0 \ub5a0\ub118\uae30\uace0 \uadf8\uc5d0 \uc758\uc9c0\ud588\ub2e4\uace0 \uc8fc\uc7a5\ud55c\ub2e4. \uc5ed\uc0ac\ud559\uacc4 \uc8fc\ub958\uc5d0\uc11c\ub294 \uc8fc\ub85c \uc1a1\ub098\ub77c \ub2f9\uc2dc\uc758 \uc870\uc815\uc774 \uc911\uad6d\uc758 \uc5b4\ub290 \uc655\uc870\ubcf4\ub2e4 \ubb38\uad00\ub4e4\uc744 \uc6b0\ub300\ud558\uc5ec \uc1a1\ub098\ub77c\uac00 \uc774\ub85c \uc778\ud574 \ubb38\uc57d\ud574\uc84c\ub2e4\ub294 \uc2dc\uac01\uc744 \uacac\uc9c0\ud558\uc9c0\ub9cc, \ub610\ub2e4\ub978 \uc758\uacac\uc73c\ub85c\ub294 \uc870\ud68c\uc5d0\uc11c \ud669\uc81c\ub97c \ub300\ud560 \ub54c, \ub300\uc2e0\ub4e4\uc758 \uc9c0\ub098\uce5c \uc874\uc911\uacfc \uc608\uc6b0\uac00 \uacb0\uacfc\uc801\uc73c\ub85c \ud669\uc81c\uac00 \uc815\uce58\uc5d0 \uad00\uc2ec\uc744 \uc783\uc5b4 \ud669\uc81c\uc640 \uc870\uc815 \uc2e0\ub8cc\uac04\uc758 \uae34\ubc00\ud55c \uad00\uacc4\ub97c \uc57d\ud654\uc2dc\ucf30\ub2e4\uace0\ub3c4 \ubcf8\ub2e4."} {"question": "\uc774\uc778\uc81c\uac00 \ucd9c\ub9c8\ud55c \ub300\ud1b5\ub839 \uc120\uac70\ub294 \uba87 \ub300\uc778\uac00?", "paragraph": "1997\ub144 \uc81c15\ub300 \ub300\ud1b5\ub839 \uc120\uac70\uc5d0 \ucd9c\ub9c8\ub97c \uc120\uc5b8\ud558\uc600\uace0, \uc774\ud6c4 \uc2e0\ud55c\uad6d\ub2f9 \ub300\ud1b5\ub839 \ud6c4\ubcf4 \uacbd\uc120\uc5d0 \ucc38\uac00\ud558\uc600\ub2e4. \ucd08\ubc18\uc5d0\ub294 \uc774\ud68c\ucc3d\uc758 \uc555\ub3c4\uc801 \uc6b0\uc138\uac00 \uacc4\uc18d\ub418\uc5c8\uc73c\ub098, \uc911\ubc18 \uc774\ud6c4 \uc774\uc778\uc81c\uac00 \uae09\uc18d\ud788 \ubd80\uc0c1\ud558\uba74\uc11c \uac08\ub4f1\uc774 \ucee4\uc84c\ub2e4. \uadf8\ub294 1\ucc28 \ud22c\ud45c\uc5d0\uc11c 14.7%\ub97c \uc5bb\uc5b4 2\uc704\ub97c \uae30\ub85d, \uacb0\uc120 \ud22c\ud45c\uc5d0 \uc9c4\ucd9c\ud558\uac8c \ub418\uc5c8\uace0 \uc774\ud6c4 \uc774\ud68c\ucc3d\uacfc \ub300\uacb0\ud558\uc600\uc73c\ub098 \ub099\uc120\ud558\uc600\ub2e4. \uacb0\uad6d \uadf8\ub294 \uac19\uc740 \ud574 9\uc6d4 13\uc77c, \uc2e0\ud55c\uad6d\ub2f9\uc744 \ud0c8\ub2f9\ud558\uc600\ub2e4. \uadf8\ub294 \uc790\uc2e0\uc774 \ud0c8\ub2f9\ud55c \uac83\uc5d0 \ub300\ud574 '\uc138\ub300\uad50\uccb4\ub9cc\uc774 30\ub144\uc758 \ub0a1\uace0 \ubcd1\ub4e0 3\uae40 \uc815\uce58\uad6c\uc870\ub97c \uccad\uc0b0\ud560 \uc218 \uc788\uae30 \ub54c\ubb38'\uc774\ub77c\uace0 \ud558\uc600\uc73c\ub098, \uc2e4\uc81c\ub85c\ub294 \uacbd\uc120 \uacfc\uc815\uc5d0\uc11c \uc0dd\uae34 \uc559\uae08\uacfc \uc544\ub4e4\uc758 \ubcd1\uc5ed\ubb38\uc81c\ub85c \uc774\ud68c\ucc3d\uc758 \uc9c0\uc9c0\uc728\uc774 \ud558\ub77d\ud55c \uac83\uc774 \ud06c\uac8c \uc791\uc6a9\ud558\uc600\ub2e4. \uc774\ub85c\uc368 \ubcf4\uc218\uc138\ub825\uc740 \ubd84\uc5f4\ub418\uace0 \ub9cc\ub2e4. \ub300\uc2e0 \uadf8\ub294 \uad6d\ubbfc\uc2e0\ub2f9\uc744 \ucc3d\ub2f9\ud558\uc5ec \uc81c15\ub300 \ub300\ud1b5\ub839 \uc120\uac70\uc5d0 \ucd9c\ub9c8, \ud6c4\ubcf4\ub85c \uc120\ucd9c\ub418\uc5c8\ub2e4. \uadf8\ub294 \"\uc80a\uc740 \ud55c\uad6d, \uac15\ud55c \ub098\ub77c\"\ub97c \uc2ac\ub85c\uac74\uc73c\ub85c \ub0b4\uc138\uc6e0\ub2e4.", "answer": "\uc81c15\ub300", "sentence": "1997\ub144 \uc81c15\ub300 \ub300\ud1b5\ub839 \uc120\uac70\uc5d0 \ucd9c\ub9c8\ub97c \uc120\uc5b8\ud558\uc600\uace0, \uc774\ud6c4 \uc2e0\ud55c\uad6d\ub2f9 \ub300\ud1b5\ub839 \ud6c4\ubcf4 \uacbd\uc120\uc5d0 \ucc38\uac00\ud558\uc600\ub2e4.", "paragraph_sentence": " 1997\ub144 \uc81c15\ub300 \ub300\ud1b5\ub839 \uc120\uac70\uc5d0 \ucd9c\ub9c8\ub97c \uc120\uc5b8\ud558\uc600\uace0, \uc774\ud6c4 \uc2e0\ud55c\uad6d\ub2f9 \ub300\ud1b5\ub839 \ud6c4\ubcf4 \uacbd\uc120\uc5d0 \ucc38\uac00\ud558\uc600\ub2e4. \ucd08\ubc18\uc5d0\ub294 \uc774\ud68c\ucc3d\uc758 \uc555\ub3c4\uc801 \uc6b0\uc138\uac00 \uacc4\uc18d\ub418\uc5c8\uc73c\ub098, \uc911\ubc18 \uc774\ud6c4 \uc774\uc778\uc81c\uac00 \uae09\uc18d\ud788 \ubd80\uc0c1\ud558\uba74\uc11c \uac08\ub4f1\uc774 \ucee4\uc84c\ub2e4. \uadf8\ub294 1\ucc28 \ud22c\ud45c\uc5d0\uc11c 14.7%\ub97c \uc5bb\uc5b4 2\uc704\ub97c \uae30\ub85d, \uacb0\uc120 \ud22c\ud45c\uc5d0 \uc9c4\ucd9c\ud558\uac8c \ub418\uc5c8\uace0 \uc774\ud6c4 \uc774\ud68c\ucc3d\uacfc \ub300\uacb0\ud558\uc600\uc73c\ub098 \ub099\uc120\ud558\uc600\ub2e4. \uacb0\uad6d \uadf8\ub294 \uac19\uc740 \ud574 9\uc6d4 13\uc77c, \uc2e0\ud55c\uad6d\ub2f9\uc744 \ud0c8\ub2f9\ud558\uc600\ub2e4. \uadf8\ub294 \uc790\uc2e0\uc774 \ud0c8\ub2f9\ud55c \uac83\uc5d0 \ub300\ud574 '\uc138\ub300\uad50\uccb4\ub9cc\uc774 30\ub144\uc758 \ub0a1\uace0 \ubcd1\ub4e0 3\uae40 \uc815\uce58\uad6c\uc870\ub97c \uccad\uc0b0\ud560 \uc218 \uc788\uae30 \ub54c\ubb38'\uc774\ub77c\uace0 \ud558\uc600\uc73c\ub098, \uc2e4\uc81c\ub85c\ub294 \uacbd\uc120 \uacfc\uc815\uc5d0\uc11c \uc0dd\uae34 \uc559\uae08\uacfc \uc544\ub4e4\uc758 \ubcd1\uc5ed\ubb38\uc81c\ub85c \uc774\ud68c\ucc3d\uc758 \uc9c0\uc9c0\uc728\uc774 \ud558\ub77d\ud55c \uac83\uc774 \ud06c\uac8c \uc791\uc6a9\ud558\uc600\ub2e4. \uc774\ub85c\uc368 \ubcf4\uc218\uc138\ub825\uc740 \ubd84\uc5f4\ub418\uace0 \ub9cc\ub2e4. \ub300\uc2e0 \uadf8\ub294 \uad6d\ubbfc\uc2e0\ub2f9\uc744 \ucc3d\ub2f9\ud558\uc5ec \uc81c15\ub300 \ub300\ud1b5\ub839 \uc120\uac70\uc5d0 \ucd9c\ub9c8, \ud6c4\ubcf4\ub85c \uc120\ucd9c\ub418\uc5c8\ub2e4. \uadf8\ub294 \"\uc80a\uc740 \ud55c\uad6d, \uac15\ud55c \ub098\ub77c\"\ub97c \uc2ac\ub85c\uac74\uc73c\ub85c \ub0b4\uc138\uc6e0\ub2e4.", "paragraph_answer": "1997\ub144 \uc81c15\ub300 \ub300\ud1b5\ub839 \uc120\uac70\uc5d0 \ucd9c\ub9c8\ub97c \uc120\uc5b8\ud558\uc600\uace0, \uc774\ud6c4 \uc2e0\ud55c\uad6d\ub2f9 \ub300\ud1b5\ub839 \ud6c4\ubcf4 \uacbd\uc120\uc5d0 \ucc38\uac00\ud558\uc600\ub2e4. \ucd08\ubc18\uc5d0\ub294 \uc774\ud68c\ucc3d\uc758 \uc555\ub3c4\uc801 \uc6b0\uc138\uac00 \uacc4\uc18d\ub418\uc5c8\uc73c\ub098, \uc911\ubc18 \uc774\ud6c4 \uc774\uc778\uc81c\uac00 \uae09\uc18d\ud788 \ubd80\uc0c1\ud558\uba74\uc11c \uac08\ub4f1\uc774 \ucee4\uc84c\ub2e4. \uadf8\ub294 1\ucc28 \ud22c\ud45c\uc5d0\uc11c 14.7%\ub97c \uc5bb\uc5b4 2\uc704\ub97c \uae30\ub85d, \uacb0\uc120 \ud22c\ud45c\uc5d0 \uc9c4\ucd9c\ud558\uac8c \ub418\uc5c8\uace0 \uc774\ud6c4 \uc774\ud68c\ucc3d\uacfc \ub300\uacb0\ud558\uc600\uc73c\ub098 \ub099\uc120\ud558\uc600\ub2e4. \uacb0\uad6d \uadf8\ub294 \uac19\uc740 \ud574 9\uc6d4 13\uc77c, \uc2e0\ud55c\uad6d\ub2f9\uc744 \ud0c8\ub2f9\ud558\uc600\ub2e4. \uadf8\ub294 \uc790\uc2e0\uc774 \ud0c8\ub2f9\ud55c \uac83\uc5d0 \ub300\ud574 '\uc138\ub300\uad50\uccb4\ub9cc\uc774 30\ub144\uc758 \ub0a1\uace0 \ubcd1\ub4e0 3\uae40 \uc815\uce58\uad6c\uc870\ub97c \uccad\uc0b0\ud560 \uc218 \uc788\uae30 \ub54c\ubb38'\uc774\ub77c\uace0 \ud558\uc600\uc73c\ub098, \uc2e4\uc81c\ub85c\ub294 \uacbd\uc120 \uacfc\uc815\uc5d0\uc11c \uc0dd\uae34 \uc559\uae08\uacfc \uc544\ub4e4\uc758 \ubcd1\uc5ed\ubb38\uc81c\ub85c \uc774\ud68c\ucc3d\uc758 \uc9c0\uc9c0\uc728\uc774 \ud558\ub77d\ud55c \uac83\uc774 \ud06c\uac8c \uc791\uc6a9\ud558\uc600\ub2e4. \uc774\ub85c\uc368 \ubcf4\uc218\uc138\ub825\uc740 \ubd84\uc5f4\ub418\uace0 \ub9cc\ub2e4. \ub300\uc2e0 \uadf8\ub294 \uad6d\ubbfc\uc2e0\ub2f9\uc744 \ucc3d\ub2f9\ud558\uc5ec \uc81c15\ub300 \ub300\ud1b5\ub839 \uc120\uac70\uc5d0 \ucd9c\ub9c8, \ud6c4\ubcf4\ub85c \uc120\ucd9c\ub418\uc5c8\ub2e4. \uadf8\ub294 \"\uc80a\uc740 \ud55c\uad6d, \uac15\ud55c \ub098\ub77c\"\ub97c \uc2ac\ub85c\uac74\uc73c\ub85c \ub0b4\uc138\uc6e0\ub2e4.", "sentence_answer": "1997\ub144 \uc81c15\ub300 \ub300\ud1b5\ub839 \uc120\uac70\uc5d0 \ucd9c\ub9c8\ub97c \uc120\uc5b8\ud558\uc600\uace0, \uc774\ud6c4 \uc2e0\ud55c\uad6d\ub2f9 \ub300\ud1b5\ub839 \ud6c4\ubcf4 \uacbd\uc120\uc5d0 \ucc38\uac00\ud558\uc600\ub2e4.", "paragraph_id": "5840727-0-1", "paragraph_question": "question: \uc774\uc778\uc81c\uac00 \ucd9c\ub9c8\ud55c \ub300\ud1b5\ub839 \uc120\uac70\ub294 \uba87 \ub300\uc778\uac00?, context: 1997\ub144 \uc81c15\ub300 \ub300\ud1b5\ub839 \uc120\uac70\uc5d0 \ucd9c\ub9c8\ub97c \uc120\uc5b8\ud558\uc600\uace0, \uc774\ud6c4 \uc2e0\ud55c\uad6d\ub2f9 \ub300\ud1b5\ub839 \ud6c4\ubcf4 \uacbd\uc120\uc5d0 \ucc38\uac00\ud558\uc600\ub2e4. \ucd08\ubc18\uc5d0\ub294 \uc774\ud68c\ucc3d\uc758 \uc555\ub3c4\uc801 \uc6b0\uc138\uac00 \uacc4\uc18d\ub418\uc5c8\uc73c\ub098, \uc911\ubc18 \uc774\ud6c4 \uc774\uc778\uc81c\uac00 \uae09\uc18d\ud788 \ubd80\uc0c1\ud558\uba74\uc11c \uac08\ub4f1\uc774 \ucee4\uc84c\ub2e4. \uadf8\ub294 1\ucc28 \ud22c\ud45c\uc5d0\uc11c 14.7%\ub97c \uc5bb\uc5b4 2\uc704\ub97c \uae30\ub85d, \uacb0\uc120 \ud22c\ud45c\uc5d0 \uc9c4\ucd9c\ud558\uac8c \ub418\uc5c8\uace0 \uc774\ud6c4 \uc774\ud68c\ucc3d\uacfc \ub300\uacb0\ud558\uc600\uc73c\ub098 \ub099\uc120\ud558\uc600\ub2e4. \uacb0\uad6d \uadf8\ub294 \uac19\uc740 \ud574 9\uc6d4 13\uc77c, \uc2e0\ud55c\uad6d\ub2f9\uc744 \ud0c8\ub2f9\ud558\uc600\ub2e4. \uadf8\ub294 \uc790\uc2e0\uc774 \ud0c8\ub2f9\ud55c \uac83\uc5d0 \ub300\ud574 '\uc138\ub300\uad50\uccb4\ub9cc\uc774 30\ub144\uc758 \ub0a1\uace0 \ubcd1\ub4e0 3\uae40 \uc815\uce58\uad6c\uc870\ub97c \uccad\uc0b0\ud560 \uc218 \uc788\uae30 \ub54c\ubb38'\uc774\ub77c\uace0 \ud558\uc600\uc73c\ub098, \uc2e4\uc81c\ub85c\ub294 \uacbd\uc120 \uacfc\uc815\uc5d0\uc11c \uc0dd\uae34 \uc559\uae08\uacfc \uc544\ub4e4\uc758 \ubcd1\uc5ed\ubb38\uc81c\ub85c \uc774\ud68c\ucc3d\uc758 \uc9c0\uc9c0\uc728\uc774 \ud558\ub77d\ud55c \uac83\uc774 \ud06c\uac8c \uc791\uc6a9\ud558\uc600\ub2e4. \uc774\ub85c\uc368 \ubcf4\uc218\uc138\ub825\uc740 \ubd84\uc5f4\ub418\uace0 \ub9cc\ub2e4. \ub300\uc2e0 \uadf8\ub294 \uad6d\ubbfc\uc2e0\ub2f9\uc744 \ucc3d\ub2f9\ud558\uc5ec \uc81c15\ub300 \ub300\ud1b5\ub839 \uc120\uac70\uc5d0 \ucd9c\ub9c8, \ud6c4\ubcf4\ub85c \uc120\ucd9c\ub418\uc5c8\ub2e4. \uadf8\ub294 \"\uc80a\uc740 \ud55c\uad6d, \uac15\ud55c \ub098\ub77c\"\ub97c \uc2ac\ub85c\uac74\uc73c\ub85c \ub0b4\uc138\uc6e0\ub2e4."} {"question": "\uae40\ubcf4\ub984\uc774 \uc6b0\uc2b9\ud55c \ub300\ud68c \uc774\ub984\uc740?", "paragraph": "\uae40\ubcf4\ub984\uc740 \uc138\uacc4\uc120\uc218\uad8c\ub300\ud68c\uc640 \uc6d4\ub4dc\ucef5 \ub9e4\uc2a4\uc2a4\ud0c0\ud2b8 \uc885\ud569\uc5d0\uc11c \uc6b0\uc2b9\ud558\uba70 \uc5ec\uc81c\uc758 \uadc0\ud658\uc744 \uc54c\ub838\ub2e4. \uc0c1\uc9d5\uc801\uc778 \uc138\uacc4\uc120\uc218\uad8c\ub300\ud68c\uc5d0\uc11c\ub294 1\ub300 \uc774\ub9ac\ub124 \uc0e4\uc6b0\ud134(\ub124\ub35c\ub780\ub4dc), 2\ub300 \uc774\ubc14\ub2c8 \ube14\ub871\ub315(\uce90\ub098\ub2e4)\uc5d0 \uc774\uc5b4 3\ub300 \ucc54\ud53c\uc5b8\uc73c\ub85c \uae40\ubcf4\ub984\uc774 \ub4f1\uadf9\ud588\ub2e4. \uc9c4\uc815\ud55c \uac15\uc790\uac00 \uc0b4\uc544\ub0a8\ub294 \uc2dc\uc98c \uc6d4\ub4dc\ucef5\uc5d0\uc11c\ub294 1\ub300 \ub9c8\ub9ac\uc2a4\uce74 \ud6c4\uc774\uc2a4\ub9cc(\ub124\ub35c\ub780\ub4dc), 2\ub300 \uae40\ubcf4\ub984, 3\ub300 \ud504\ub780\uccb4\uc2a4\uce74 \ub864\ub85c\ube0c\ub9ac\uae30\ub2e4(\uc774\ud0c8\ub9ac\uc544), 4\ub300 \ube14\ub871\ub315, 5\ub300 \uc0e4\uc6b0\ud134\uc5d0 \uc774\uc5b4 6\ub300 \ucc54\ud53c\uc5b8\uc73c\ub85c \uae40\ubcf4\ub984\uc774 \ub4f1\uadf9\ud588\ub2e4. 2011-2012 \uc2dc\uc98c \uc6d4\ub4dc\ucef5\uc5d0 \uccab \uc120\uc744 \ubcf4\uc774\uba70 \ub9e4\uc2a4\uc2a4\ud0c0\ud2b8\uac00 \uc138\uacc4\ub300\ud68c\uc5d0 \ub4f1\uc7a5\ud55c \uc774\ud6c4 2011-2012 \uc2dc\uc98c\uc740 \uc6d4\ub4dc\ucef5 \ucc54\ud53c\uc5b8 \ud6c4\uc774\uc2a4\ub9cc\uc774 \uc555\ub3c4\ud588\ub2e4. 2012-2013 \uc2dc\uc98c\uc740 \uc6d4\ub4dc\ucef5 \ucc54\ud53c\uc5b8 \uae40\ubcf4\ub984\uc774 \uc555\ub3c4\ud588\ub2e4. 2013-2014 \uc2dc\uc98c\uc740 \uc6d4\ub4dc\ucef5 \ucc54\ud53c\uc5b8 \ub864\ub85c\ube0c\ub9ac\uae30\ub2e4\uac00 \uc555\ub3c4\ud588\ub2e4. 2014-2015 \uc2dc\uc98c\uc740 \uc6d4\ub4dc\ucef5 \ucc54\ud53c\uc5b8 \ube14\ub871\ub315\uacfc \uc120\uc218\uad8c \ucc54\ud53c\uc5b8 \uc0e4\uc6b0\ud134\uc774 \uc591\ubd84\ud588\ub2e4. 2015-2016 \uc2dc\uc98c\ub3c4 \uc6d4\ub4dc\ucef5 \ucc54\ud53c\uc5b8 \uc0e4\uc6b0\ud134\uacfc \uc120\uc218\uad8c \ucc54\ud53c\uc5b8 \ube14\ub871\ub315\uc774 \uc591\ubd84\ud588\ub2e4. 2016-2017 \uc2dc\uc98c\uc740 \uae40\ubcf4\ub984\uc774 \ucd5c\ucd08\ub85c \uc6d4\ub4dc\ucef5\uacfc \uc120\uc218\uad8c 2\uad00\uc655\uc744 \ucc28\uc9c0\ud558\uba70 \uc555\ub3c4\ud588\ub2e4. \uae40\ubcf4\ub984\uc740 \ub610\ud55c \ucd5c\ucd08\ub85c \uc6d4\ub4dc\ucef5 \uc885\ud569 \ucc54\ud53c\uc5b8 2\ud68c\uc5d0 \uc624\ub974\ub294 \uc704\uc5c5\ub3c4 \ub2ec\uc131\ud588\ub2e4. \uae4a\uc740 \ub0b4\uc0c1\uc744 \uc785\uace0 \uc740\uac70\uc790\uc911\ud558\ub294 \ub3d9\uc548 \uc783\uc5b4\ubc84\ub838\ub358 \uc601\ud1a0\ub97c \uc5ec\uc81c\uac00 \ub3cc\uc544\uc640 \ub418\ucc3e\uc73c\ub2c8 \ubb47 \uc81c\ud6c4\ub4e4\uc774 \ud55c \ubc1c \ubb3c\ub7ec\uc11c\uba70 \uace0\uac1c\ub97c \uc219\uc600\ub2e4.", "answer": "\uc138\uacc4\uc120\uc218\uad8c\ub300\ud68c", "sentence": "\uae40\ubcf4\ub984\uc740 \uc138\uacc4\uc120\uc218\uad8c\ub300\ud68c \uc640 \uc6d4\ub4dc\ucef5 \ub9e4\uc2a4\uc2a4\ud0c0\ud2b8 \uc885\ud569\uc5d0\uc11c \uc6b0\uc2b9\ud558\uba70 \uc5ec\uc81c\uc758 \uadc0\ud658\uc744 \uc54c\ub838\ub2e4.", "paragraph_sentence": " \uae40\ubcf4\ub984\uc740 \uc138\uacc4\uc120\uc218\uad8c\ub300\ud68c \uc640 \uc6d4\ub4dc\ucef5 \ub9e4\uc2a4\uc2a4\ud0c0\ud2b8 \uc885\ud569\uc5d0\uc11c \uc6b0\uc2b9\ud558\uba70 \uc5ec\uc81c\uc758 \uadc0\ud658\uc744 \uc54c\ub838\ub2e4. \uc0c1\uc9d5\uc801\uc778 \uc138\uacc4\uc120\uc218\uad8c\ub300\ud68c\uc5d0\uc11c\ub294 1\ub300 \uc774\ub9ac\ub124 \uc0e4\uc6b0\ud134(\ub124\ub35c\ub780\ub4dc), 2\ub300 \uc774\ubc14\ub2c8 \ube14\ub871\ub315(\uce90\ub098\ub2e4)\uc5d0 \uc774\uc5b4 3\ub300 \ucc54\ud53c\uc5b8\uc73c\ub85c \uae40\ubcf4\ub984\uc774 \ub4f1\uadf9\ud588\ub2e4. \uc9c4\uc815\ud55c \uac15\uc790\uac00 \uc0b4\uc544\ub0a8\ub294 \uc2dc\uc98c \uc6d4\ub4dc\ucef5\uc5d0\uc11c\ub294 1\ub300 \ub9c8\ub9ac\uc2a4\uce74 \ud6c4\uc774\uc2a4\ub9cc(\ub124\ub35c\ub780\ub4dc), 2\ub300 \uae40\ubcf4\ub984, 3\ub300 \ud504\ub780\uccb4\uc2a4\uce74 \ub864\ub85c\ube0c\ub9ac\uae30\ub2e4(\uc774\ud0c8\ub9ac\uc544), 4\ub300 \ube14\ub871\ub315, 5\ub300 \uc0e4\uc6b0\ud134\uc5d0 \uc774\uc5b4 6\ub300 \ucc54\ud53c\uc5b8\uc73c\ub85c \uae40\ubcf4\ub984\uc774 \ub4f1\uadf9\ud588\ub2e4. 2011-2012 \uc2dc\uc98c \uc6d4\ub4dc\ucef5\uc5d0 \uccab \uc120\uc744 \ubcf4\uc774\uba70 \ub9e4\uc2a4\uc2a4\ud0c0\ud2b8\uac00 \uc138\uacc4\ub300\ud68c\uc5d0 \ub4f1\uc7a5\ud55c \uc774\ud6c4 2011-2012 \uc2dc\uc98c\uc740 \uc6d4\ub4dc\ucef5 \ucc54\ud53c\uc5b8 \ud6c4\uc774\uc2a4\ub9cc\uc774 \uc555\ub3c4\ud588\ub2e4. 2012-2013 \uc2dc\uc98c\uc740 \uc6d4\ub4dc\ucef5 \ucc54\ud53c\uc5b8 \uae40\ubcf4\ub984\uc774 \uc555\ub3c4\ud588\ub2e4. 2013-2014 \uc2dc\uc98c\uc740 \uc6d4\ub4dc\ucef5 \ucc54\ud53c\uc5b8 \ub864\ub85c\ube0c\ub9ac\uae30\ub2e4\uac00 \uc555\ub3c4\ud588\ub2e4. 2014-2015 \uc2dc\uc98c\uc740 \uc6d4\ub4dc\ucef5 \ucc54\ud53c\uc5b8 \ube14\ub871\ub315\uacfc \uc120\uc218\uad8c \ucc54\ud53c\uc5b8 \uc0e4\uc6b0\ud134\uc774 \uc591\ubd84\ud588\ub2e4. 2015-2016 \uc2dc\uc98c\ub3c4 \uc6d4\ub4dc\ucef5 \ucc54\ud53c\uc5b8 \uc0e4\uc6b0\ud134\uacfc \uc120\uc218\uad8c \ucc54\ud53c\uc5b8 \ube14\ub871\ub315\uc774 \uc591\ubd84\ud588\ub2e4. 2016-2017 \uc2dc\uc98c\uc740 \uae40\ubcf4\ub984\uc774 \ucd5c\ucd08\ub85c \uc6d4\ub4dc\ucef5\uacfc \uc120\uc218\uad8c 2\uad00\uc655\uc744 \ucc28\uc9c0\ud558\uba70 \uc555\ub3c4\ud588\ub2e4. \uae40\ubcf4\ub984\uc740 \ub610\ud55c \ucd5c\ucd08\ub85c \uc6d4\ub4dc\ucef5 \uc885\ud569 \ucc54\ud53c\uc5b8 2\ud68c\uc5d0 \uc624\ub974\ub294 \uc704\uc5c5\ub3c4 \ub2ec\uc131\ud588\ub2e4. \uae4a\uc740 \ub0b4\uc0c1\uc744 \uc785\uace0 \uc740\uac70\uc790\uc911\ud558\ub294 \ub3d9\uc548 \uc783\uc5b4\ubc84\ub838\ub358 \uc601\ud1a0\ub97c \uc5ec\uc81c\uac00 \ub3cc\uc544\uc640 \ub418\ucc3e\uc73c\ub2c8 \ubb47 \uc81c\ud6c4\ub4e4\uc774 \ud55c \ubc1c \ubb3c\ub7ec\uc11c\uba70 \uace0\uac1c\ub97c \uc219\uc600\ub2e4.", "paragraph_answer": "\uae40\ubcf4\ub984\uc740 \uc138\uacc4\uc120\uc218\uad8c\ub300\ud68c \uc640 \uc6d4\ub4dc\ucef5 \ub9e4\uc2a4\uc2a4\ud0c0\ud2b8 \uc885\ud569\uc5d0\uc11c \uc6b0\uc2b9\ud558\uba70 \uc5ec\uc81c\uc758 \uadc0\ud658\uc744 \uc54c\ub838\ub2e4. \uc0c1\uc9d5\uc801\uc778 \uc138\uacc4\uc120\uc218\uad8c\ub300\ud68c\uc5d0\uc11c\ub294 1\ub300 \uc774\ub9ac\ub124 \uc0e4\uc6b0\ud134(\ub124\ub35c\ub780\ub4dc), 2\ub300 \uc774\ubc14\ub2c8 \ube14\ub871\ub315(\uce90\ub098\ub2e4)\uc5d0 \uc774\uc5b4 3\ub300 \ucc54\ud53c\uc5b8\uc73c\ub85c \uae40\ubcf4\ub984\uc774 \ub4f1\uadf9\ud588\ub2e4. \uc9c4\uc815\ud55c \uac15\uc790\uac00 \uc0b4\uc544\ub0a8\ub294 \uc2dc\uc98c \uc6d4\ub4dc\ucef5\uc5d0\uc11c\ub294 1\ub300 \ub9c8\ub9ac\uc2a4\uce74 \ud6c4\uc774\uc2a4\ub9cc(\ub124\ub35c\ub780\ub4dc), 2\ub300 \uae40\ubcf4\ub984, 3\ub300 \ud504\ub780\uccb4\uc2a4\uce74 \ub864\ub85c\ube0c\ub9ac\uae30\ub2e4(\uc774\ud0c8\ub9ac\uc544), 4\ub300 \ube14\ub871\ub315, 5\ub300 \uc0e4\uc6b0\ud134\uc5d0 \uc774\uc5b4 6\ub300 \ucc54\ud53c\uc5b8\uc73c\ub85c \uae40\ubcf4\ub984\uc774 \ub4f1\uadf9\ud588\ub2e4. 2011-2012 \uc2dc\uc98c \uc6d4\ub4dc\ucef5\uc5d0 \uccab \uc120\uc744 \ubcf4\uc774\uba70 \ub9e4\uc2a4\uc2a4\ud0c0\ud2b8\uac00 \uc138\uacc4\ub300\ud68c\uc5d0 \ub4f1\uc7a5\ud55c \uc774\ud6c4 2011-2012 \uc2dc\uc98c\uc740 \uc6d4\ub4dc\ucef5 \ucc54\ud53c\uc5b8 \ud6c4\uc774\uc2a4\ub9cc\uc774 \uc555\ub3c4\ud588\ub2e4. 2012-2013 \uc2dc\uc98c\uc740 \uc6d4\ub4dc\ucef5 \ucc54\ud53c\uc5b8 \uae40\ubcf4\ub984\uc774 \uc555\ub3c4\ud588\ub2e4. 2013-2014 \uc2dc\uc98c\uc740 \uc6d4\ub4dc\ucef5 \ucc54\ud53c\uc5b8 \ub864\ub85c\ube0c\ub9ac\uae30\ub2e4\uac00 \uc555\ub3c4\ud588\ub2e4. 2014-2015 \uc2dc\uc98c\uc740 \uc6d4\ub4dc\ucef5 \ucc54\ud53c\uc5b8 \ube14\ub871\ub315\uacfc \uc120\uc218\uad8c \ucc54\ud53c\uc5b8 \uc0e4\uc6b0\ud134\uc774 \uc591\ubd84\ud588\ub2e4. 2015-2016 \uc2dc\uc98c\ub3c4 \uc6d4\ub4dc\ucef5 \ucc54\ud53c\uc5b8 \uc0e4\uc6b0\ud134\uacfc \uc120\uc218\uad8c \ucc54\ud53c\uc5b8 \ube14\ub871\ub315\uc774 \uc591\ubd84\ud588\ub2e4. 2016-2017 \uc2dc\uc98c\uc740 \uae40\ubcf4\ub984\uc774 \ucd5c\ucd08\ub85c \uc6d4\ub4dc\ucef5\uacfc \uc120\uc218\uad8c 2\uad00\uc655\uc744 \ucc28\uc9c0\ud558\uba70 \uc555\ub3c4\ud588\ub2e4. \uae40\ubcf4\ub984\uc740 \ub610\ud55c \ucd5c\ucd08\ub85c \uc6d4\ub4dc\ucef5 \uc885\ud569 \ucc54\ud53c\uc5b8 2\ud68c\uc5d0 \uc624\ub974\ub294 \uc704\uc5c5\ub3c4 \ub2ec\uc131\ud588\ub2e4. \uae4a\uc740 \ub0b4\uc0c1\uc744 \uc785\uace0 \uc740\uac70\uc790\uc911\ud558\ub294 \ub3d9\uc548 \uc783\uc5b4\ubc84\ub838\ub358 \uc601\ud1a0\ub97c \uc5ec\uc81c\uac00 \ub3cc\uc544\uc640 \ub418\ucc3e\uc73c\ub2c8 \ubb47 \uc81c\ud6c4\ub4e4\uc774 \ud55c \ubc1c \ubb3c\ub7ec\uc11c\uba70 \uace0\uac1c\ub97c \uc219\uc600\ub2e4.", "sentence_answer": "\uae40\ubcf4\ub984\uc740 \uc138\uacc4\uc120\uc218\uad8c\ub300\ud68c \uc640 \uc6d4\ub4dc\ucef5 \ub9e4\uc2a4\uc2a4\ud0c0\ud2b8 \uc885\ud569\uc5d0\uc11c \uc6b0\uc2b9\ud558\uba70 \uc5ec\uc81c\uc758 \uadc0\ud658\uc744 \uc54c\ub838\ub2e4.", "paragraph_id": "6267639-14-0", "paragraph_question": "question: \uae40\ubcf4\ub984\uc774 \uc6b0\uc2b9\ud55c \ub300\ud68c \uc774\ub984\uc740?, context: \uae40\ubcf4\ub984\uc740 \uc138\uacc4\uc120\uc218\uad8c\ub300\ud68c\uc640 \uc6d4\ub4dc\ucef5 \ub9e4\uc2a4\uc2a4\ud0c0\ud2b8 \uc885\ud569\uc5d0\uc11c \uc6b0\uc2b9\ud558\uba70 \uc5ec\uc81c\uc758 \uadc0\ud658\uc744 \uc54c\ub838\ub2e4. \uc0c1\uc9d5\uc801\uc778 \uc138\uacc4\uc120\uc218\uad8c\ub300\ud68c\uc5d0\uc11c\ub294 1\ub300 \uc774\ub9ac\ub124 \uc0e4\uc6b0\ud134(\ub124\ub35c\ub780\ub4dc), 2\ub300 \uc774\ubc14\ub2c8 \ube14\ub871\ub315(\uce90\ub098\ub2e4)\uc5d0 \uc774\uc5b4 3\ub300 \ucc54\ud53c\uc5b8\uc73c\ub85c \uae40\ubcf4\ub984\uc774 \ub4f1\uadf9\ud588\ub2e4. \uc9c4\uc815\ud55c \uac15\uc790\uac00 \uc0b4\uc544\ub0a8\ub294 \uc2dc\uc98c \uc6d4\ub4dc\ucef5\uc5d0\uc11c\ub294 1\ub300 \ub9c8\ub9ac\uc2a4\uce74 \ud6c4\uc774\uc2a4\ub9cc(\ub124\ub35c\ub780\ub4dc), 2\ub300 \uae40\ubcf4\ub984, 3\ub300 \ud504\ub780\uccb4\uc2a4\uce74 \ub864\ub85c\ube0c\ub9ac\uae30\ub2e4(\uc774\ud0c8\ub9ac\uc544), 4\ub300 \ube14\ub871\ub315, 5\ub300 \uc0e4\uc6b0\ud134\uc5d0 \uc774\uc5b4 6\ub300 \ucc54\ud53c\uc5b8\uc73c\ub85c \uae40\ubcf4\ub984\uc774 \ub4f1\uadf9\ud588\ub2e4. 2011-2012 \uc2dc\uc98c \uc6d4\ub4dc\ucef5\uc5d0 \uccab \uc120\uc744 \ubcf4\uc774\uba70 \ub9e4\uc2a4\uc2a4\ud0c0\ud2b8\uac00 \uc138\uacc4\ub300\ud68c\uc5d0 \ub4f1\uc7a5\ud55c \uc774\ud6c4 2011-2012 \uc2dc\uc98c\uc740 \uc6d4\ub4dc\ucef5 \ucc54\ud53c\uc5b8 \ud6c4\uc774\uc2a4\ub9cc\uc774 \uc555\ub3c4\ud588\ub2e4. 2012-2013 \uc2dc\uc98c\uc740 \uc6d4\ub4dc\ucef5 \ucc54\ud53c\uc5b8 \uae40\ubcf4\ub984\uc774 \uc555\ub3c4\ud588\ub2e4. 2013-2014 \uc2dc\uc98c\uc740 \uc6d4\ub4dc\ucef5 \ucc54\ud53c\uc5b8 \ub864\ub85c\ube0c\ub9ac\uae30\ub2e4\uac00 \uc555\ub3c4\ud588\ub2e4. 2014-2015 \uc2dc\uc98c\uc740 \uc6d4\ub4dc\ucef5 \ucc54\ud53c\uc5b8 \ube14\ub871\ub315\uacfc \uc120\uc218\uad8c \ucc54\ud53c\uc5b8 \uc0e4\uc6b0\ud134\uc774 \uc591\ubd84\ud588\ub2e4. 2015-2016 \uc2dc\uc98c\ub3c4 \uc6d4\ub4dc\ucef5 \ucc54\ud53c\uc5b8 \uc0e4\uc6b0\ud134\uacfc \uc120\uc218\uad8c \ucc54\ud53c\uc5b8 \ube14\ub871\ub315\uc774 \uc591\ubd84\ud588\ub2e4. 2016-2017 \uc2dc\uc98c\uc740 \uae40\ubcf4\ub984\uc774 \ucd5c\ucd08\ub85c \uc6d4\ub4dc\ucef5\uacfc \uc120\uc218\uad8c 2\uad00\uc655\uc744 \ucc28\uc9c0\ud558\uba70 \uc555\ub3c4\ud588\ub2e4. \uae40\ubcf4\ub984\uc740 \ub610\ud55c \ucd5c\ucd08\ub85c \uc6d4\ub4dc\ucef5 \uc885\ud569 \ucc54\ud53c\uc5b8 2\ud68c\uc5d0 \uc624\ub974\ub294 \uc704\uc5c5\ub3c4 \ub2ec\uc131\ud588\ub2e4. \uae4a\uc740 \ub0b4\uc0c1\uc744 \uc785\uace0 \uc740\uac70\uc790\uc911\ud558\ub294 \ub3d9\uc548 \uc783\uc5b4\ubc84\ub838\ub358 \uc601\ud1a0\ub97c \uc5ec\uc81c\uac00 \ub3cc\uc544\uc640 \ub418\ucc3e\uc73c\ub2c8 \ubb47 \uc81c\ud6c4\ub4e4\uc774 \ud55c \ubc1c \ubb3c\ub7ec\uc11c\uba70 \uace0\uac1c\ub97c \uc219\uc600\ub2e4."} {"question": "\ub18d\ub9bc\uc218\uc0b0\uc2dd\ud488\ubd80\uc5d0\uc11c\ub294 \ub9e4\ub144 \uc5bc\ub9c8\uc758 \uc300 \uc0dd\uc0b0\uc774 \uac10\uc18c\ud560 \uac83\uc774\ub77c\uace0 \uae30\ub300\ud558\uc600\ub294\uac00?", "paragraph": "\uc774\uc640 \uac19\uc740 \uc2dc\ubc94\uc0ac\uc5c5\uc744 \uac70\uce5c \ub18d\ub9bc\uc218\uc0b0\uc2dd\ud488\ubd80\ub294 2011\ub144\uc5d0 \ubcf8\uaca9\uc801\uc73c\ub85c \ub9e4\ub144 \ub17c 40,000ha\ub97c \ub300\uc0c1\uc73c\ub85c \ud558\ub294 \ub17c \uc18c\ub4dd\uae30\ubc18 \ub2e4\uc591\ud654 \uc0ac\uc5c5\uc744 \ucd94\uc9c4\ud558\uc600\ub2e4. \ub17c \uc18c\ub4dd\uae30\ubc18 \ub2e4\uc591\ud654 \uc0ac\uc5c5\uc740 \uc2dc\ubc94\uc0ac\uc5c5\uacfc \ub9c8\ucc2c\uac00\uc9c0\ub85c \ub17c\uc5d0 \ud0c0\uc791\ubb3c(2011\ub144\uc5d0\ub294 1\ub144\uc0dd \ubc0f \ub2e4\ub144\uc0dd \ud0c0\uc791\ubb3c\uc774\uc5c8\uace0, 2012\ub144\uc5d0\ub294 \ucf69, \uc870\uc0ac\ub8cc, \uac00\uacf5\uc6a9 \ubcbc\ub85c \ud55c\uc815)\uc744 \uc7ac\ubc30\ud558\ub294 \ub18d\uc5c5\uc778\uc744 \ub300\uc0c1\uc73c\ub85c \ud558\ub294 \uac83\uc73c\ub85c \uc2dc\uc7a5\u00b7\uad70\uc218\uc640 \ub17c \uc18c\ub4dd\uae30\ubc18 \ub2e4\uc591\ud654\uc0ac\uc5c5 \uc57d\uc815\uc744 \uccb4\uacb0\ud558\uace0, \uc57d\uc815 \uc0ac\ud56d\uc744 \uc774\ud589\ud55c \ub18d\uc5c5\uc778\uc5d0 \ub300\ud574 \uc0ac\uc5c5\ub144\ub3c4\uc758 12\uc6d4\uc5d0 10a \ub2f9 30\ub9cc \uc6d0(\uac00\uacf5\uc6a9 \ubcbc\ub294 20\ub9cc \uc6d0)\uc744 \uc804\uc561 \uad6d\uace0(\ub18d\uc5b4\ucd0c\uad6c\uc870\uac1c\uc120 \ud2b9\ubcc4\ud68c\uacc4)\uc5d0\uc11c \uc9c0\uae09\ud558\uc600\ub2e4. \ub18d\ub9bc\uc218\uc0b0\uc2dd\ud488\ubd80\ub294 \uc774 \uc0ac\uc5c5\uc774 \uc81c\ub300\ub85c \uc774\ub8e8\uc5b4\uc9c8 \uacbd\uc6b0 \ub9e4\ub144 \uc300 \uc0dd\uc0b0\uc774 20\ub9cc \ud1a4 \uc774\uc0c1 \uac10\uc18c\ud560 \uac83\uc73c\ub85c \uae30\ub300\ud558\uc600\ub2e4.", "answer": "20\ub9cc \ud1a4 \uc774\uc0c1", "sentence": "\ub18d\ub9bc\uc218\uc0b0\uc2dd\ud488\ubd80\ub294 \uc774 \uc0ac\uc5c5\uc774 \uc81c\ub300\ub85c \uc774\ub8e8\uc5b4\uc9c8 \uacbd\uc6b0 \ub9e4\ub144 \uc300 \uc0dd\uc0b0\uc774 20\ub9cc \ud1a4 \uc774\uc0c1 \uac10\uc18c\ud560 \uac83\uc73c\ub85c \uae30\ub300\ud558\uc600\ub2e4.", "paragraph_sentence": "\uc774\uc640 \uac19\uc740 \uc2dc\ubc94\uc0ac\uc5c5\uc744 \uac70\uce5c \ub18d\ub9bc\uc218\uc0b0\uc2dd\ud488\ubd80\ub294 2011\ub144\uc5d0 \ubcf8\uaca9\uc801\uc73c\ub85c \ub9e4\ub144 \ub17c 40,000ha\ub97c \ub300\uc0c1\uc73c\ub85c \ud558\ub294 \ub17c \uc18c\ub4dd\uae30\ubc18 \ub2e4\uc591\ud654 \uc0ac\uc5c5\uc744 \ucd94\uc9c4\ud558\uc600\ub2e4. \ub17c \uc18c\ub4dd\uae30\ubc18 \ub2e4\uc591\ud654 \uc0ac\uc5c5\uc740 \uc2dc\ubc94\uc0ac\uc5c5\uacfc \ub9c8\ucc2c\uac00\uc9c0\ub85c \ub17c\uc5d0 \ud0c0\uc791\ubb3c(2011\ub144\uc5d0\ub294 1\ub144\uc0dd \ubc0f \ub2e4\ub144\uc0dd \ud0c0\uc791\ubb3c\uc774\uc5c8\uace0, 2012\ub144\uc5d0\ub294 \ucf69, \uc870\uc0ac\ub8cc, \uac00\uacf5\uc6a9 \ubcbc\ub85c \ud55c\uc815)\uc744 \uc7ac\ubc30\ud558\ub294 \ub18d\uc5c5\uc778\uc744 \ub300\uc0c1\uc73c\ub85c \ud558\ub294 \uac83\uc73c\ub85c \uc2dc\uc7a5\u00b7\uad70\uc218\uc640 \ub17c \uc18c\ub4dd\uae30\ubc18 \ub2e4\uc591\ud654\uc0ac\uc5c5 \uc57d\uc815\uc744 \uccb4\uacb0\ud558\uace0, \uc57d\uc815 \uc0ac\ud56d\uc744 \uc774\ud589\ud55c \ub18d\uc5c5\uc778\uc5d0 \ub300\ud574 \uc0ac\uc5c5\ub144\ub3c4\uc758 12\uc6d4\uc5d0 10a \ub2f9 30\ub9cc \uc6d0(\uac00\uacf5\uc6a9 \ubcbc\ub294 20\ub9cc \uc6d0)\uc744 \uc804\uc561 \uad6d\uace0(\ub18d\uc5b4\ucd0c\uad6c\uc870\uac1c\uc120 \ud2b9\ubcc4\ud68c\uacc4)\uc5d0\uc11c \uc9c0\uae09\ud558\uc600\ub2e4. \ub18d\ub9bc\uc218\uc0b0\uc2dd\ud488\ubd80\ub294 \uc774 \uc0ac\uc5c5\uc774 \uc81c\ub300\ub85c \uc774\ub8e8\uc5b4\uc9c8 \uacbd\uc6b0 \ub9e4\ub144 \uc300 \uc0dd\uc0b0\uc774 20\ub9cc \ud1a4 \uc774\uc0c1 \uac10\uc18c\ud560 \uac83\uc73c\ub85c \uae30\ub300\ud558\uc600\ub2e4. ", "paragraph_answer": "\uc774\uc640 \uac19\uc740 \uc2dc\ubc94\uc0ac\uc5c5\uc744 \uac70\uce5c \ub18d\ub9bc\uc218\uc0b0\uc2dd\ud488\ubd80\ub294 2011\ub144\uc5d0 \ubcf8\uaca9\uc801\uc73c\ub85c \ub9e4\ub144 \ub17c 40,000ha\ub97c \ub300\uc0c1\uc73c\ub85c \ud558\ub294 \ub17c \uc18c\ub4dd\uae30\ubc18 \ub2e4\uc591\ud654 \uc0ac\uc5c5\uc744 \ucd94\uc9c4\ud558\uc600\ub2e4. \ub17c \uc18c\ub4dd\uae30\ubc18 \ub2e4\uc591\ud654 \uc0ac\uc5c5\uc740 \uc2dc\ubc94\uc0ac\uc5c5\uacfc \ub9c8\ucc2c\uac00\uc9c0\ub85c \ub17c\uc5d0 \ud0c0\uc791\ubb3c(2011\ub144\uc5d0\ub294 1\ub144\uc0dd \ubc0f \ub2e4\ub144\uc0dd \ud0c0\uc791\ubb3c\uc774\uc5c8\uace0, 2012\ub144\uc5d0\ub294 \ucf69, \uc870\uc0ac\ub8cc, \uac00\uacf5\uc6a9 \ubcbc\ub85c \ud55c\uc815)\uc744 \uc7ac\ubc30\ud558\ub294 \ub18d\uc5c5\uc778\uc744 \ub300\uc0c1\uc73c\ub85c \ud558\ub294 \uac83\uc73c\ub85c \uc2dc\uc7a5\u00b7\uad70\uc218\uc640 \ub17c \uc18c\ub4dd\uae30\ubc18 \ub2e4\uc591\ud654\uc0ac\uc5c5 \uc57d\uc815\uc744 \uccb4\uacb0\ud558\uace0, \uc57d\uc815 \uc0ac\ud56d\uc744 \uc774\ud589\ud55c \ub18d\uc5c5\uc778\uc5d0 \ub300\ud574 \uc0ac\uc5c5\ub144\ub3c4\uc758 12\uc6d4\uc5d0 10a \ub2f9 30\ub9cc \uc6d0(\uac00\uacf5\uc6a9 \ubcbc\ub294 20\ub9cc \uc6d0)\uc744 \uc804\uc561 \uad6d\uace0(\ub18d\uc5b4\ucd0c\uad6c\uc870\uac1c\uc120 \ud2b9\ubcc4\ud68c\uacc4)\uc5d0\uc11c \uc9c0\uae09\ud558\uc600\ub2e4. \ub18d\ub9bc\uc218\uc0b0\uc2dd\ud488\ubd80\ub294 \uc774 \uc0ac\uc5c5\uc774 \uc81c\ub300\ub85c \uc774\ub8e8\uc5b4\uc9c8 \uacbd\uc6b0 \ub9e4\ub144 \uc300 \uc0dd\uc0b0\uc774 20\ub9cc \ud1a4 \uc774\uc0c1 \uac10\uc18c\ud560 \uac83\uc73c\ub85c \uae30\ub300\ud558\uc600\ub2e4.", "sentence_answer": "\ub18d\ub9bc\uc218\uc0b0\uc2dd\ud488\ubd80\ub294 \uc774 \uc0ac\uc5c5\uc774 \uc81c\ub300\ub85c \uc774\ub8e8\uc5b4\uc9c8 \uacbd\uc6b0 \ub9e4\ub144 \uc300 \uc0dd\uc0b0\uc774 20\ub9cc \ud1a4 \uc774\uc0c1 \uac10\uc18c\ud560 \uac83\uc73c\ub85c \uae30\ub300\ud558\uc600\ub2e4.", "paragraph_id": "6538576-1-1", "paragraph_question": "question: \ub18d\ub9bc\uc218\uc0b0\uc2dd\ud488\ubd80\uc5d0\uc11c\ub294 \ub9e4\ub144 \uc5bc\ub9c8\uc758 \uc300 \uc0dd\uc0b0\uc774 \uac10\uc18c\ud560 \uac83\uc774\ub77c\uace0 \uae30\ub300\ud558\uc600\ub294\uac00?, context: \uc774\uc640 \uac19\uc740 \uc2dc\ubc94\uc0ac\uc5c5\uc744 \uac70\uce5c \ub18d\ub9bc\uc218\uc0b0\uc2dd\ud488\ubd80\ub294 2011\ub144\uc5d0 \ubcf8\uaca9\uc801\uc73c\ub85c \ub9e4\ub144 \ub17c 40,000ha\ub97c \ub300\uc0c1\uc73c\ub85c \ud558\ub294 \ub17c \uc18c\ub4dd\uae30\ubc18 \ub2e4\uc591\ud654 \uc0ac\uc5c5\uc744 \ucd94\uc9c4\ud558\uc600\ub2e4. \ub17c \uc18c\ub4dd\uae30\ubc18 \ub2e4\uc591\ud654 \uc0ac\uc5c5\uc740 \uc2dc\ubc94\uc0ac\uc5c5\uacfc \ub9c8\ucc2c\uac00\uc9c0\ub85c \ub17c\uc5d0 \ud0c0\uc791\ubb3c(2011\ub144\uc5d0\ub294 1\ub144\uc0dd \ubc0f \ub2e4\ub144\uc0dd \ud0c0\uc791\ubb3c\uc774\uc5c8\uace0, 2012\ub144\uc5d0\ub294 \ucf69, \uc870\uc0ac\ub8cc, \uac00\uacf5\uc6a9 \ubcbc\ub85c \ud55c\uc815)\uc744 \uc7ac\ubc30\ud558\ub294 \ub18d\uc5c5\uc778\uc744 \ub300\uc0c1\uc73c\ub85c \ud558\ub294 \uac83\uc73c\ub85c \uc2dc\uc7a5\u00b7\uad70\uc218\uc640 \ub17c \uc18c\ub4dd\uae30\ubc18 \ub2e4\uc591\ud654\uc0ac\uc5c5 \uc57d\uc815\uc744 \uccb4\uacb0\ud558\uace0, \uc57d\uc815 \uc0ac\ud56d\uc744 \uc774\ud589\ud55c \ub18d\uc5c5\uc778\uc5d0 \ub300\ud574 \uc0ac\uc5c5\ub144\ub3c4\uc758 12\uc6d4\uc5d0 10a \ub2f9 30\ub9cc \uc6d0(\uac00\uacf5\uc6a9 \ubcbc\ub294 20\ub9cc \uc6d0)\uc744 \uc804\uc561 \uad6d\uace0(\ub18d\uc5b4\ucd0c\uad6c\uc870\uac1c\uc120 \ud2b9\ubcc4\ud68c\uacc4)\uc5d0\uc11c \uc9c0\uae09\ud558\uc600\ub2e4. \ub18d\ub9bc\uc218\uc0b0\uc2dd\ud488\ubd80\ub294 \uc774 \uc0ac\uc5c5\uc774 \uc81c\ub300\ub85c \uc774\ub8e8\uc5b4\uc9c8 \uacbd\uc6b0 \ub9e4\ub144 \uc300 \uc0dd\uc0b0\uc774 20\ub9cc \ud1a4 \uc774\uc0c1 \uac10\uc18c\ud560 \uac83\uc73c\ub85c \uae30\ub300\ud558\uc600\ub2e4."} {"question": "\uba74\uc811\uad00\uc5d0\uac8c \uc815\uc720\ub77c\ub97c \ubf51\uc73c\ub77c\uace0 \ud2b9\uc815\ud558\uc5ec \uc9c0\uc2dc\ud55c \uc774\ud654\uc5ec\uc790\ub300\ud559\uad50\uc758 \uc785\ud559 \ucc98\uc7a5\uc740 \ub204\uad6c\uc778\uac00?", "paragraph": "\uc774\ud654\uc5ec\uc790\ub300\ud559\uad50\ub294 \ucd5c\uc21c\uc2e4\uacfc \uc815\uc724\ud68c \uc0ac\uc774\uc758 \uc790\ub140\uc778 \uc815\uc720\ub77c\ub97c \uccb4\uc721 \ud2b9\uae30\uc790\ub85c \ubd80\uc815\uc785\ud559\uc2dc\ud0a8 \ud6c4 \ud559\uc0ac \uad00\ub9ac\ub97c \uc18c\ud640\ud788 \ud574\uc654\uc74c\uc774 \ucd5c\uc21c\uc2e4 \uac8c\uc774\ud2b8\uc758 \uccad\ubb38\ud68c\ub97c \ud1b5\ud574\uc11c \ubc1d\ud600\uc84c\ub2e4. \uccad\ubb38\ud68c\ub97c \ud1b5\ud574\uc11c \uc815\uc720\ub77c\ub294 \"\ucd5c\uacbd\ud76c \uc804 \ucd1d\uc7a5\uacfc \uae40\uacbd\uc219 \uc804 \ud559\uc7a5 \ub4f1 \uad50\uc218\ub4e4\uc758 \ud2b9\ud61c\uc640 \ub3c4\uc6c0\"\uc73c\ub85c \uc774\ud654\uc5ec\ub300\uc5d0 \uc785\ud559\ud588\uc74c\uc774 \ub4dc\ub7ec\ub0ac\uc73c\uba70, \uc785\ud559 \uba74\uc811\uc5d0\uc11c \uc804 \uc785\ud559 \ucc98\uc7a5 \ub0a8\uad81\uace4\uc740 \uba74\uc811\uad00\uc5d0\uac8c \uc815\uc720\ub77c\ub97c \ubf51\uc73c\ub77c\uace0 \ud2b9\uc815\ud558\uc5ec \uc9c0\uc2dc\ud55c \uc815\ud669\ub3c4 \ub4dc\ub7ec\ub0ac\ub2e4. \uc774\ud6c4 \ucd5c\uacbd\ud76c \uc804 \uc774\ud654\uc5ec\ub300 \ucd1d\uc7a5 \ub4f1 \uc804\ud604\uc9c1 \uad50\uc6d0\ub4e4 \uc911 \uc77c\ubd80\ub294 \uad6c\uc18d \uae30\uc18c\ub418\uc5c8\uc73c\uba70, \uccad\ubb38\ud68c\uc5d0 \ucd9c\ub450\ud558\uc5ec \uc99d\uc5b8\ud558\uc600\ub2e4. \uccad\ubb38\ud68c \uacfc\uc815\uc5d0\uc11c \uc815\uc720\ub77c\ub294 \uc218\uc5c5\uc5d0 \ucd9c\uc11d\ud558\uc9c0 \uc54a\uac70\ub098 \uacfc\uc81c\ubb3c\uc744 \uc81c\ucd9c\ud558\uc9c0 \uc54a\uc544\ub3c4 \uc88b\uc740 \ud559\uc810\uc744 \ubc1b\uace0, \uad50\uc218\uac00 \uacf5\ud56d\uc73c\ub85c \ub9c8\uc911\uc744 \ub098\uc624\ub294 \ub4f1 \ub2e4\ub978 \ud559\uc0dd\ub4e4\uacfc \ub69c\ub837\ud558\uac8c \ub2e4\ub978 \ub300\uc6b0\ub97c \ubc1b\uc73c\uba70 \ud559\uad50\ub97c \ub2e4\ub154\uc74c\uc774 \ubc1d\ud600\uc84c\ub2e4.", "answer": "\ub0a8\uad81\uace4", "sentence": "\uc785\ud559 \uba74\uc811\uc5d0\uc11c \uc804 \uc785\ud559 \ucc98\uc7a5 \ub0a8\uad81\uace4 \uc740 \uba74\uc811\uad00\uc5d0\uac8c \uc815\uc720\ub77c\ub97c \ubf51\uc73c\ub77c\uace0 \ud2b9\uc815\ud558\uc5ec \uc9c0\uc2dc\ud55c \uc815\ud669\ub3c4 \ub4dc\ub7ec\ub0ac\ub2e4.", "paragraph_sentence": "\uc774\ud654\uc5ec\uc790\ub300\ud559\uad50\ub294 \ucd5c\uc21c\uc2e4\uacfc \uc815\uc724\ud68c \uc0ac\uc774\uc758 \uc790\ub140\uc778 \uc815\uc720\ub77c\ub97c \uccb4\uc721 \ud2b9\uae30\uc790\ub85c \ubd80\uc815\uc785\ud559\uc2dc\ud0a8 \ud6c4 \ud559\uc0ac \uad00\ub9ac\ub97c \uc18c\ud640\ud788 \ud574\uc654\uc74c\uc774 \ucd5c\uc21c\uc2e4 \uac8c\uc774\ud2b8\uc758 \uccad\ubb38\ud68c\ub97c \ud1b5\ud574\uc11c \ubc1d\ud600\uc84c\ub2e4. \uccad\ubb38\ud68c\ub97c \ud1b5\ud574\uc11c \uc815\uc720\ub77c\ub294 \"\ucd5c\uacbd\ud76c \uc804 \ucd1d\uc7a5\uacfc \uae40\uacbd\uc219 \uc804 \ud559\uc7a5 \ub4f1 \uad50\uc218\ub4e4\uc758 \ud2b9\ud61c\uc640 \ub3c4\uc6c0\"\uc73c\ub85c \uc774\ud654\uc5ec\ub300\uc5d0 \uc785\ud559\ud588\uc74c\uc774 \ub4dc\ub7ec\ub0ac\uc73c\uba70, \uc785\ud559 \uba74\uc811\uc5d0\uc11c \uc804 \uc785\ud559 \ucc98\uc7a5 \ub0a8\uad81\uace4 \uc740 \uba74\uc811\uad00\uc5d0\uac8c \uc815\uc720\ub77c\ub97c \ubf51\uc73c\ub77c\uace0 \ud2b9\uc815\ud558\uc5ec \uc9c0\uc2dc\ud55c \uc815\ud669\ub3c4 \ub4dc\ub7ec\ub0ac\ub2e4. \uc774\ud6c4 \ucd5c\uacbd\ud76c \uc804 \uc774\ud654\uc5ec\ub300 \ucd1d\uc7a5 \ub4f1 \uc804\ud604\uc9c1 \uad50\uc6d0\ub4e4 \uc911 \uc77c\ubd80\ub294 \uad6c\uc18d \uae30\uc18c\ub418\uc5c8\uc73c\uba70, \uccad\ubb38\ud68c\uc5d0 \ucd9c\ub450\ud558\uc5ec \uc99d\uc5b8\ud558\uc600\ub2e4. \uccad\ubb38\ud68c \uacfc\uc815\uc5d0\uc11c \uc815\uc720\ub77c\ub294 \uc218\uc5c5\uc5d0 \ucd9c\uc11d\ud558\uc9c0 \uc54a\uac70\ub098 \uacfc\uc81c\ubb3c\uc744 \uc81c\ucd9c\ud558\uc9c0 \uc54a\uc544\ub3c4 \uc88b\uc740 \ud559\uc810\uc744 \ubc1b\uace0, \uad50\uc218\uac00 \uacf5\ud56d\uc73c\ub85c \ub9c8\uc911\uc744 \ub098\uc624\ub294 \ub4f1 \ub2e4\ub978 \ud559\uc0dd\ub4e4\uacfc \ub69c\ub837\ud558\uac8c \ub2e4\ub978 \ub300\uc6b0\ub97c \ubc1b\uc73c\uba70 \ud559\uad50\ub97c \ub2e4\ub154\uc74c\uc774 \ubc1d\ud600\uc84c\ub2e4.", "paragraph_answer": "\uc774\ud654\uc5ec\uc790\ub300\ud559\uad50\ub294 \ucd5c\uc21c\uc2e4\uacfc \uc815\uc724\ud68c \uc0ac\uc774\uc758 \uc790\ub140\uc778 \uc815\uc720\ub77c\ub97c \uccb4\uc721 \ud2b9\uae30\uc790\ub85c \ubd80\uc815\uc785\ud559\uc2dc\ud0a8 \ud6c4 \ud559\uc0ac \uad00\ub9ac\ub97c \uc18c\ud640\ud788 \ud574\uc654\uc74c\uc774 \ucd5c\uc21c\uc2e4 \uac8c\uc774\ud2b8\uc758 \uccad\ubb38\ud68c\ub97c \ud1b5\ud574\uc11c \ubc1d\ud600\uc84c\ub2e4. \uccad\ubb38\ud68c\ub97c \ud1b5\ud574\uc11c \uc815\uc720\ub77c\ub294 \"\ucd5c\uacbd\ud76c \uc804 \ucd1d\uc7a5\uacfc \uae40\uacbd\uc219 \uc804 \ud559\uc7a5 \ub4f1 \uad50\uc218\ub4e4\uc758 \ud2b9\ud61c\uc640 \ub3c4\uc6c0\"\uc73c\ub85c \uc774\ud654\uc5ec\ub300\uc5d0 \uc785\ud559\ud588\uc74c\uc774 \ub4dc\ub7ec\ub0ac\uc73c\uba70, \uc785\ud559 \uba74\uc811\uc5d0\uc11c \uc804 \uc785\ud559 \ucc98\uc7a5 \ub0a8\uad81\uace4 \uc740 \uba74\uc811\uad00\uc5d0\uac8c \uc815\uc720\ub77c\ub97c \ubf51\uc73c\ub77c\uace0 \ud2b9\uc815\ud558\uc5ec \uc9c0\uc2dc\ud55c \uc815\ud669\ub3c4 \ub4dc\ub7ec\ub0ac\ub2e4. \uc774\ud6c4 \ucd5c\uacbd\ud76c \uc804 \uc774\ud654\uc5ec\ub300 \ucd1d\uc7a5 \ub4f1 \uc804\ud604\uc9c1 \uad50\uc6d0\ub4e4 \uc911 \uc77c\ubd80\ub294 \uad6c\uc18d \uae30\uc18c\ub418\uc5c8\uc73c\uba70, \uccad\ubb38\ud68c\uc5d0 \ucd9c\ub450\ud558\uc5ec \uc99d\uc5b8\ud558\uc600\ub2e4. \uccad\ubb38\ud68c \uacfc\uc815\uc5d0\uc11c \uc815\uc720\ub77c\ub294 \uc218\uc5c5\uc5d0 \ucd9c\uc11d\ud558\uc9c0 \uc54a\uac70\ub098 \uacfc\uc81c\ubb3c\uc744 \uc81c\ucd9c\ud558\uc9c0 \uc54a\uc544\ub3c4 \uc88b\uc740 \ud559\uc810\uc744 \ubc1b\uace0, \uad50\uc218\uac00 \uacf5\ud56d\uc73c\ub85c \ub9c8\uc911\uc744 \ub098\uc624\ub294 \ub4f1 \ub2e4\ub978 \ud559\uc0dd\ub4e4\uacfc \ub69c\ub837\ud558\uac8c \ub2e4\ub978 \ub300\uc6b0\ub97c \ubc1b\uc73c\uba70 \ud559\uad50\ub97c \ub2e4\ub154\uc74c\uc774 \ubc1d\ud600\uc84c\ub2e4.", "sentence_answer": "\uc785\ud559 \uba74\uc811\uc5d0\uc11c \uc804 \uc785\ud559 \ucc98\uc7a5 \ub0a8\uad81\uace4 \uc740 \uba74\uc811\uad00\uc5d0\uac8c \uc815\uc720\ub77c\ub97c \ubf51\uc73c\ub77c\uace0 \ud2b9\uc815\ud558\uc5ec \uc9c0\uc2dc\ud55c \uc815\ud669\ub3c4 \ub4dc\ub7ec\ub0ac\ub2e4.", "paragraph_id": "6581158-26-1", "paragraph_question": "question: \uba74\uc811\uad00\uc5d0\uac8c \uc815\uc720\ub77c\ub97c \ubf51\uc73c\ub77c\uace0 \ud2b9\uc815\ud558\uc5ec \uc9c0\uc2dc\ud55c \uc774\ud654\uc5ec\uc790\ub300\ud559\uad50\uc758 \uc785\ud559 \ucc98\uc7a5\uc740 \ub204\uad6c\uc778\uac00?, context: \uc774\ud654\uc5ec\uc790\ub300\ud559\uad50\ub294 \ucd5c\uc21c\uc2e4\uacfc \uc815\uc724\ud68c \uc0ac\uc774\uc758 \uc790\ub140\uc778 \uc815\uc720\ub77c\ub97c \uccb4\uc721 \ud2b9\uae30\uc790\ub85c \ubd80\uc815\uc785\ud559\uc2dc\ud0a8 \ud6c4 \ud559\uc0ac \uad00\ub9ac\ub97c \uc18c\ud640\ud788 \ud574\uc654\uc74c\uc774 \ucd5c\uc21c\uc2e4 \uac8c\uc774\ud2b8\uc758 \uccad\ubb38\ud68c\ub97c \ud1b5\ud574\uc11c \ubc1d\ud600\uc84c\ub2e4. \uccad\ubb38\ud68c\ub97c \ud1b5\ud574\uc11c \uc815\uc720\ub77c\ub294 \"\ucd5c\uacbd\ud76c \uc804 \ucd1d\uc7a5\uacfc \uae40\uacbd\uc219 \uc804 \ud559\uc7a5 \ub4f1 \uad50\uc218\ub4e4\uc758 \ud2b9\ud61c\uc640 \ub3c4\uc6c0\"\uc73c\ub85c \uc774\ud654\uc5ec\ub300\uc5d0 \uc785\ud559\ud588\uc74c\uc774 \ub4dc\ub7ec\ub0ac\uc73c\uba70, \uc785\ud559 \uba74\uc811\uc5d0\uc11c \uc804 \uc785\ud559 \ucc98\uc7a5 \ub0a8\uad81\uace4\uc740 \uba74\uc811\uad00\uc5d0\uac8c \uc815\uc720\ub77c\ub97c \ubf51\uc73c\ub77c\uace0 \ud2b9\uc815\ud558\uc5ec \uc9c0\uc2dc\ud55c \uc815\ud669\ub3c4 \ub4dc\ub7ec\ub0ac\ub2e4. \uc774\ud6c4 \ucd5c\uacbd\ud76c \uc804 \uc774\ud654\uc5ec\ub300 \ucd1d\uc7a5 \ub4f1 \uc804\ud604\uc9c1 \uad50\uc6d0\ub4e4 \uc911 \uc77c\ubd80\ub294 \uad6c\uc18d \uae30\uc18c\ub418\uc5c8\uc73c\uba70, \uccad\ubb38\ud68c\uc5d0 \ucd9c\ub450\ud558\uc5ec \uc99d\uc5b8\ud558\uc600\ub2e4. \uccad\ubb38\ud68c \uacfc\uc815\uc5d0\uc11c \uc815\uc720\ub77c\ub294 \uc218\uc5c5\uc5d0 \ucd9c\uc11d\ud558\uc9c0 \uc54a\uac70\ub098 \uacfc\uc81c\ubb3c\uc744 \uc81c\ucd9c\ud558\uc9c0 \uc54a\uc544\ub3c4 \uc88b\uc740 \ud559\uc810\uc744 \ubc1b\uace0, \uad50\uc218\uac00 \uacf5\ud56d\uc73c\ub85c \ub9c8\uc911\uc744 \ub098\uc624\ub294 \ub4f1 \ub2e4\ub978 \ud559\uc0dd\ub4e4\uacfc \ub69c\ub837\ud558\uac8c \ub2e4\ub978 \ub300\uc6b0\ub97c \ubc1b\uc73c\uba70 \ud559\uad50\ub97c \ub2e4\ub154\uc74c\uc774 \ubc1d\ud600\uc84c\ub2e4."} {"question": "\uc21c\ucc9c\ub300\ud559\uad50\uac00 \uac1c\uad50 70\uc8fc\ub144 \uae30\ub150\uad00 \uc124\ub9bd\uc0ac\uc5c5\uc744 \ucd94\uc9c4\ud558\uae30 \uc704\ud574 \ud22c\uc790\ud55c \uc0ac\uc5c5\ube44\ub294 \ucd1d \uc5bc\ub9c8\uc778\uac00?", "paragraph": "\uc21c\ucc9c\ub300\ud559\uad50\ub294 \uac1c\uad50 70\uc8fc\ub144\uc744 \uae30\ub150\ud558\uae30 \uc704\ud574 2003\ub144 7\uc6d4 \ucd1d \uc0ac\uc5c5\ube44 86\uc5b5\uc6d0\uc744 \ud22c\uc790\ud574 \uac1c\uad50 70\uc8fc\ub144 \uae30\ub150\uad00 \uac74\ub9bd\uc0ac\uc5c5\uc744 \uc2dc\uc791\ud588\ub2e4. 70\uc8fc\ub144 \uae30\ub150\uad00\uc740 \uad6d\uc81c\ud559\uc220\ud589\uc0ac, \ubb38\ud654\ud589\uc0ac, \uc608\uc220\ud589\uc0ac\ub97c \ud560 \uc218 \uc788\ub294 \ub2e4\ubaa9\uc801 \ubcf5\ud569 \uac74\ubb3c\ub85c \uc9c0\ud5581\uce35, \uc9c0\uc0c13\uce35, \ucd1d \uba74\uc801 6,425m(1,943\ud3c9)\ub85c \uac74\ub9bd\ub410\ub2e4. \uc9c0\ud558 1\uce35\uc5d0\ub294 \ub18d\ud611, \uad6d\uc81c\uad50\ub958\uc13c\ud130, \uc9c0\uc0c1 1\uce35\uc5d0\ub294 \uc21c\ucc9c\ub300\ud559\uad50 \uc5ed\uc0ac \uc790\ub8cc\uc2e4\uacfc \ub300\uaddc\ubaa8 \ud589\uc0ac\ub97c \uc704\ud55c 433\uc11d \uaddc\ubaa8\uc758 \uc6b0\uc11d\ud640, \ub3d9\ucc3d\ud68c\uc2e4, \uc804\uc2dc\uc2e4\uc774 \uc704\uce58\ud574 \uc788\ub2e4. \ub610\ud55c \uc9c0\uc0c1 2\uce35\uc5d0\ub294 \ub3d9\uc2dc\ud1b5\uc5ed\uc2dc\uc124\uc774 \uc644\ube44\ub41c 121\uc11d \uaddc\ubaa8\uc758 \ud559\uc220\ud589\uc0ac\uc6a9 \ub300\ud68c\uc758\uc2e4\uacfc \uc911\ud68c\uc758\uc2e4, \uc9c0\uc0c1 3\uce35\uc5d0\ub294 \uc911,\uc18c\ud68c\uc758\uc2e4, \uad50\uc9c1\uc6d0\uc2dd\ub2f9, \uce74\ud398 \ub4f1\uc774 \uad6c\ube44\ub3fc \uc788\ub2e4. \uc6b0\uc11d\ud640\uc774\ub77c \ubd80\ub974\uae30\ub3c4 \ud558\uba70, \uc9c0\uc5ed\uc8fc\ubbfc\ub4e4\uc744 \uc704\ud55c \uc5ec\ub7ec\uac00\uc9c0 \ud504\ub85c\uadf8\ub7a8\uc744 \uc6b4\uc601\ud558\uace0 \uc788\ub2e4.", "answer": "86\uc5b5\uc6d0", "sentence": "2003\ub144 7\uc6d4 \ucd1d \uc0ac\uc5c5\ube44 86\uc5b5\uc6d0 \uc744 \ud22c\uc790\ud574 \uac1c\uad50 70\uc8fc\ub144 \uae30\ub150\uad00 \uac74\ub9bd\uc0ac\uc5c5\uc744 \uc2dc\uc791\ud588\ub2e4.", "paragraph_sentence": "\uc21c\ucc9c\ub300\ud559\uad50\ub294 \uac1c\uad50 70\uc8fc\ub144\uc744 \uae30\ub150\ud558\uae30 \uc704\ud574 2003\ub144 7\uc6d4 \ucd1d \uc0ac\uc5c5\ube44 86\uc5b5\uc6d0 \uc744 \ud22c\uc790\ud574 \uac1c\uad50 70\uc8fc\ub144 \uae30\ub150\uad00 \uac74\ub9bd\uc0ac\uc5c5\uc744 \uc2dc\uc791\ud588\ub2e4. 70\uc8fc\ub144 \uae30\ub150\uad00\uc740 \uad6d\uc81c\ud559\uc220\ud589\uc0ac, \ubb38\ud654\ud589\uc0ac, \uc608\uc220\ud589\uc0ac\ub97c \ud560 \uc218 \uc788\ub294 \ub2e4\ubaa9\uc801 \ubcf5\ud569 \uac74\ubb3c\ub85c \uc9c0\ud5581\uce35, \uc9c0\uc0c13\uce35, \ucd1d \uba74\uc801 6,425m(1,943\ud3c9)\ub85c \uac74\ub9bd\ub410\ub2e4. \uc9c0\ud558 1\uce35\uc5d0\ub294 \ub18d\ud611, \uad6d\uc81c\uad50\ub958\uc13c\ud130, \uc9c0\uc0c1 1\uce35\uc5d0\ub294 \uc21c\ucc9c\ub300\ud559\uad50 \uc5ed\uc0ac \uc790\ub8cc\uc2e4\uacfc \ub300\uaddc\ubaa8 \ud589\uc0ac\ub97c \uc704\ud55c 433\uc11d \uaddc\ubaa8\uc758 \uc6b0\uc11d\ud640, \ub3d9\ucc3d\ud68c\uc2e4, \uc804\uc2dc\uc2e4\uc774 \uc704\uce58\ud574 \uc788\ub2e4. \ub610\ud55c \uc9c0\uc0c1 2\uce35\uc5d0\ub294 \ub3d9\uc2dc\ud1b5\uc5ed\uc2dc\uc124\uc774 \uc644\ube44\ub41c 121\uc11d \uaddc\ubaa8\uc758 \ud559\uc220\ud589\uc0ac\uc6a9 \ub300\ud68c\uc758\uc2e4\uacfc \uc911\ud68c\uc758\uc2e4, \uc9c0\uc0c1 3\uce35\uc5d0\ub294 \uc911,\uc18c\ud68c\uc758\uc2e4, \uad50\uc9c1\uc6d0\uc2dd\ub2f9, \uce74\ud398 \ub4f1\uc774 \uad6c\ube44\ub3fc \uc788\ub2e4. \uc6b0\uc11d\ud640\uc774\ub77c \ubd80\ub974\uae30\ub3c4 \ud558\uba70, \uc9c0\uc5ed\uc8fc\ubbfc\ub4e4\uc744 \uc704\ud55c \uc5ec\ub7ec\uac00\uc9c0 \ud504\ub85c\uadf8\ub7a8\uc744 \uc6b4\uc601\ud558\uace0 \uc788\ub2e4.", "paragraph_answer": "\uc21c\ucc9c\ub300\ud559\uad50\ub294 \uac1c\uad50 70\uc8fc\ub144\uc744 \uae30\ub150\ud558\uae30 \uc704\ud574 2003\ub144 7\uc6d4 \ucd1d \uc0ac\uc5c5\ube44 86\uc5b5\uc6d0 \uc744 \ud22c\uc790\ud574 \uac1c\uad50 70\uc8fc\ub144 \uae30\ub150\uad00 \uac74\ub9bd\uc0ac\uc5c5\uc744 \uc2dc\uc791\ud588\ub2e4. 70\uc8fc\ub144 \uae30\ub150\uad00\uc740 \uad6d\uc81c\ud559\uc220\ud589\uc0ac, \ubb38\ud654\ud589\uc0ac, \uc608\uc220\ud589\uc0ac\ub97c \ud560 \uc218 \uc788\ub294 \ub2e4\ubaa9\uc801 \ubcf5\ud569 \uac74\ubb3c\ub85c \uc9c0\ud5581\uce35, \uc9c0\uc0c13\uce35, \ucd1d \uba74\uc801 6,425m(1,943\ud3c9)\ub85c \uac74\ub9bd\ub410\ub2e4. \uc9c0\ud558 1\uce35\uc5d0\ub294 \ub18d\ud611, \uad6d\uc81c\uad50\ub958\uc13c\ud130, \uc9c0\uc0c1 1\uce35\uc5d0\ub294 \uc21c\ucc9c\ub300\ud559\uad50 \uc5ed\uc0ac \uc790\ub8cc\uc2e4\uacfc \ub300\uaddc\ubaa8 \ud589\uc0ac\ub97c \uc704\ud55c 433\uc11d \uaddc\ubaa8\uc758 \uc6b0\uc11d\ud640, \ub3d9\ucc3d\ud68c\uc2e4, \uc804\uc2dc\uc2e4\uc774 \uc704\uce58\ud574 \uc788\ub2e4. \ub610\ud55c \uc9c0\uc0c1 2\uce35\uc5d0\ub294 \ub3d9\uc2dc\ud1b5\uc5ed\uc2dc\uc124\uc774 \uc644\ube44\ub41c 121\uc11d \uaddc\ubaa8\uc758 \ud559\uc220\ud589\uc0ac\uc6a9 \ub300\ud68c\uc758\uc2e4\uacfc \uc911\ud68c\uc758\uc2e4, \uc9c0\uc0c1 3\uce35\uc5d0\ub294 \uc911,\uc18c\ud68c\uc758\uc2e4, \uad50\uc9c1\uc6d0\uc2dd\ub2f9, \uce74\ud398 \ub4f1\uc774 \uad6c\ube44\ub3fc \uc788\ub2e4. \uc6b0\uc11d\ud640\uc774\ub77c \ubd80\ub974\uae30\ub3c4 \ud558\uba70, \uc9c0\uc5ed\uc8fc\ubbfc\ub4e4\uc744 \uc704\ud55c \uc5ec\ub7ec\uac00\uc9c0 \ud504\ub85c\uadf8\ub7a8\uc744 \uc6b4\uc601\ud558\uace0 \uc788\ub2e4.", "sentence_answer": "2003\ub144 7\uc6d4 \ucd1d \uc0ac\uc5c5\ube44 86\uc5b5\uc6d0 \uc744 \ud22c\uc790\ud574 \uac1c\uad50 70\uc8fc\ub144 \uae30\ub150\uad00 \uac74\ub9bd\uc0ac\uc5c5\uc744 \uc2dc\uc791\ud588\ub2e4.", "paragraph_id": "6537159-6-0", "paragraph_question": "question: \uc21c\ucc9c\ub300\ud559\uad50\uac00 \uac1c\uad50 70\uc8fc\ub144 \uae30\ub150\uad00 \uc124\ub9bd\uc0ac\uc5c5\uc744 \ucd94\uc9c4\ud558\uae30 \uc704\ud574 \ud22c\uc790\ud55c \uc0ac\uc5c5\ube44\ub294 \ucd1d \uc5bc\ub9c8\uc778\uac00?, context: \uc21c\ucc9c\ub300\ud559\uad50\ub294 \uac1c\uad50 70\uc8fc\ub144\uc744 \uae30\ub150\ud558\uae30 \uc704\ud574 2003\ub144 7\uc6d4 \ucd1d \uc0ac\uc5c5\ube44 86\uc5b5\uc6d0\uc744 \ud22c\uc790\ud574 \uac1c\uad50 70\uc8fc\ub144 \uae30\ub150\uad00 \uac74\ub9bd\uc0ac\uc5c5\uc744 \uc2dc\uc791\ud588\ub2e4. 70\uc8fc\ub144 \uae30\ub150\uad00\uc740 \uad6d\uc81c\ud559\uc220\ud589\uc0ac, \ubb38\ud654\ud589\uc0ac, \uc608\uc220\ud589\uc0ac\ub97c \ud560 \uc218 \uc788\ub294 \ub2e4\ubaa9\uc801 \ubcf5\ud569 \uac74\ubb3c\ub85c \uc9c0\ud5581\uce35, \uc9c0\uc0c13\uce35, \ucd1d \uba74\uc801 6,425m(1,943\ud3c9)\ub85c \uac74\ub9bd\ub410\ub2e4. \uc9c0\ud558 1\uce35\uc5d0\ub294 \ub18d\ud611, \uad6d\uc81c\uad50\ub958\uc13c\ud130, \uc9c0\uc0c1 1\uce35\uc5d0\ub294 \uc21c\ucc9c\ub300\ud559\uad50 \uc5ed\uc0ac \uc790\ub8cc\uc2e4\uacfc \ub300\uaddc\ubaa8 \ud589\uc0ac\ub97c \uc704\ud55c 433\uc11d \uaddc\ubaa8\uc758 \uc6b0\uc11d\ud640, \ub3d9\ucc3d\ud68c\uc2e4, \uc804\uc2dc\uc2e4\uc774 \uc704\uce58\ud574 \uc788\ub2e4. \ub610\ud55c \uc9c0\uc0c1 2\uce35\uc5d0\ub294 \ub3d9\uc2dc\ud1b5\uc5ed\uc2dc\uc124\uc774 \uc644\ube44\ub41c 121\uc11d \uaddc\ubaa8\uc758 \ud559\uc220\ud589\uc0ac\uc6a9 \ub300\ud68c\uc758\uc2e4\uacfc \uc911\ud68c\uc758\uc2e4, \uc9c0\uc0c1 3\uce35\uc5d0\ub294 \uc911,\uc18c\ud68c\uc758\uc2e4, \uad50\uc9c1\uc6d0\uc2dd\ub2f9, \uce74\ud398 \ub4f1\uc774 \uad6c\ube44\ub3fc \uc788\ub2e4. \uc6b0\uc11d\ud640\uc774\ub77c \ubd80\ub974\uae30\ub3c4 \ud558\uba70, \uc9c0\uc5ed\uc8fc\ubbfc\ub4e4\uc744 \uc704\ud55c \uc5ec\ub7ec\uac00\uc9c0 \ud504\ub85c\uadf8\ub7a8\uc744 \uc6b4\uc601\ud558\uace0 \uc788\ub2e4."} {"question": "\uc77c\ubcf8\uc0ac\ud68c\ub2f9\uc758 \uccb4\uc81c\uac00 \uc131\ub9bd\ub41c \uc2dc\uae30\ub294?", "paragraph": "55\ub144 \uccb4\uc81c\uac00 \uc131\ub9bd\ud55c \ub2f9\uc2dc\ub294 \uc0ac\ud68c\ub2f9\ub3c4 \uc815\uad8c \ud68d\ub4dd\uc744 \ubaa9\ud45c\ub85c \ud588\uc73c\uba70, \uc9c0\uc9c0\uc790\uc758 \uc0c1\ub2f9\uc218\ub3c4 \uc0ac\ud68c\ub2f9 \uc815\uad8c\uc744 \ubc14\ub77c\uace0 \uc788\uc5c8\ub2e4. \uadf8\ub7ec\ub098 \uc790\ubbfc\ub2f9\uc758 \uc77c\ub2f9 \uc6b0\uc704\uccb4\uc81c\uac00 \uc9c0\uc18d\ub418\uba74\uc11c, \uc0ac\ud68c\ub2f9\uc758 \uc815\uad8c \ud68d\ub4dd\uc5d0 \ub300\ud55c \uc5f4\ub9dd\uc740 \ud76c\ubbf8\ud574\uc9c0\uae30 \uc2dc\uc791\ud588\ub2e4. \ub178\ub3d9\uc870\ud569\uc744 \uae30\ubc18\uc73c\ub85c \ud55c \uc0ac\ud68c\ub2f9\uc774\uc9c0\ub9cc, \uc0ac\ud68c \uad6c\uc870\uac00 \ubcc0\ud654\ud558\uba74\uc11c \uc720\uad8c\uc790\ub4e4\uc740 \uc0ac\ud68c\uc8fc\uc758\uc5d0 \ud765\ubbf8\ub97c \uc783\uae30 \uc2dc\uc791\ud588\uace0, \uc0ac\ud68c\ub2f9 \ub610\ud55c \ud604\uc0c1 \uc720\uc9c0\uc5d0 \uae09\uae09\ud558\uba74\uc11c \uc7a5\uae30\uc801\uc73c\ub85c\ub294 \ub0ae\uc740 \ud558\ub77d\uc138\ub97c \ubcf4\uc774\uae30 \uc2dc\uc791\ud588\ub2e4. \uc0ac\ud68c\ub2f9\uc740 \u2018\ub9cc\ub144 \uc57c\ub2f9\u2019\uc73c\ub85c \ubd88\ub9ac\uba70, \uc9c0\uc9c0\uc790\ub4e4 \ub610\ud55c \uc790\ubbfc\ub2f9 \uc815\uad8c\uc774 \uc9c0\uc18d\ub418\ub294 \uac83\uc744 \uc804\uc81c\ub85c \ud558\ub294 \uc778\uc2dd\uc774 \ud37c\uc9c0\uae30 \uc2dc\uc791\ud588\ub2e4. \ud2b9\ud788 \ub3c4\uc2dc\uc9c0\uc5ed\uc5d0\uc11c \uc0ac\ud68c\ub2f9\uc774 \ubc00\ub824\ub098\uae30 \uc2dc\uc791\ud55c \uac83\uc740 \uc0c1\ub2f9\ud574, \ub3c4\ucfc4\ub3c4 \uc758\ud68c\uc5d0\uc11c\ub294 \uc790\ubbfc\ub2f9 \ubfd0\ub9cc \uc544\ub2c8\ub77c \uacf5\uba85\ub2f9\uacfc \uc77c\ubcf8\uacf5\uc0b0\ub2f9\uc5d0\ub3c4 \uc758\uc11d\uc774 \ubbf8\uce58\uc9c0 \ubabb\ud574 \uc81c4\ub2f9\uc73c\ub85c \uc804\ub77d\ud588\ub2e4. \uadf8\ub098\ub9c8 \uc9c0\ubc29\uc5d0\uc11c\ub294 \uc790\ubbfc\ub2f9\uacfc \uc0ac\ud68c\ub2f9\uc758 \uc591\ub2f9 \uccb4\uc81c\uac00 \uc720\uc9c0\ub418\uc5c8\uae30 \ub54c\ubb38\uc5d0, \uc0ac\ud68c\ub2f9\uc740 \ub3c4\uc2dc\ud615 \uc815\ub2f9\uc5d0\uc11c \ub18d\ucd0c\ud615 \uc815\ub2f9\uc73c\ub85c \uc804\ub77d\ud574 \uac14\ub2e4.", "answer": "55\ub144", "sentence": "55\ub144 \uccb4\uc81c\uac00 \uc131\ub9bd\ud55c \ub2f9\uc2dc\ub294 \uc0ac\ud68c\ub2f9\ub3c4 \uc815\uad8c \ud68d\ub4dd\uc744 \ubaa9\ud45c\ub85c \ud588\uc73c\uba70, \uc9c0\uc9c0\uc790\uc758 \uc0c1\ub2f9\uc218\ub3c4 \uc0ac\ud68c\ub2f9 \uc815\uad8c\uc744 \ubc14\ub77c\uace0 \uc788\uc5c8\ub2e4.", "paragraph_sentence": " 55\ub144 \uccb4\uc81c\uac00 \uc131\ub9bd\ud55c \ub2f9\uc2dc\ub294 \uc0ac\ud68c\ub2f9\ub3c4 \uc815\uad8c \ud68d\ub4dd\uc744 \ubaa9\ud45c\ub85c \ud588\uc73c\uba70, \uc9c0\uc9c0\uc790\uc758 \uc0c1\ub2f9\uc218\ub3c4 \uc0ac\ud68c\ub2f9 \uc815\uad8c\uc744 \ubc14\ub77c\uace0 \uc788\uc5c8\ub2e4. \uadf8\ub7ec\ub098 \uc790\ubbfc\ub2f9\uc758 \uc77c\ub2f9 \uc6b0\uc704\uccb4\uc81c\uac00 \uc9c0\uc18d\ub418\uba74\uc11c, \uc0ac\ud68c\ub2f9\uc758 \uc815\uad8c \ud68d\ub4dd\uc5d0 \ub300\ud55c \uc5f4\ub9dd\uc740 \ud76c\ubbf8\ud574\uc9c0\uae30 \uc2dc\uc791\ud588\ub2e4. \ub178\ub3d9\uc870\ud569\uc744 \uae30\ubc18\uc73c\ub85c \ud55c \uc0ac\ud68c\ub2f9\uc774\uc9c0\ub9cc, \uc0ac\ud68c \uad6c\uc870\uac00 \ubcc0\ud654\ud558\uba74\uc11c \uc720\uad8c\uc790\ub4e4\uc740 \uc0ac\ud68c\uc8fc\uc758\uc5d0 \ud765\ubbf8\ub97c \uc783\uae30 \uc2dc\uc791\ud588\uace0, \uc0ac\ud68c\ub2f9 \ub610\ud55c \ud604\uc0c1 \uc720\uc9c0\uc5d0 \uae09\uae09\ud558\uba74\uc11c \uc7a5\uae30\uc801\uc73c\ub85c\ub294 \ub0ae\uc740 \ud558\ub77d\uc138\ub97c \ubcf4\uc774\uae30 \uc2dc\uc791\ud588\ub2e4. \uc0ac\ud68c\ub2f9\uc740 \u2018\ub9cc\ub144 \uc57c\ub2f9\u2019\uc73c\ub85c \ubd88\ub9ac\uba70, \uc9c0\uc9c0\uc790\ub4e4 \ub610\ud55c \uc790\ubbfc\ub2f9 \uc815\uad8c\uc774 \uc9c0\uc18d\ub418\ub294 \uac83\uc744 \uc804\uc81c\ub85c \ud558\ub294 \uc778\uc2dd\uc774 \ud37c\uc9c0\uae30 \uc2dc\uc791\ud588\ub2e4. \ud2b9\ud788 \ub3c4\uc2dc\uc9c0\uc5ed\uc5d0\uc11c \uc0ac\ud68c\ub2f9\uc774 \ubc00\ub824\ub098\uae30 \uc2dc\uc791\ud55c \uac83\uc740 \uc0c1\ub2f9\ud574, \ub3c4\ucfc4\ub3c4 \uc758\ud68c\uc5d0\uc11c\ub294 \uc790\ubbfc\ub2f9 \ubfd0\ub9cc \uc544\ub2c8\ub77c \uacf5\uba85\ub2f9\uacfc \uc77c\ubcf8\uacf5\uc0b0\ub2f9\uc5d0\ub3c4 \uc758\uc11d\uc774 \ubbf8\uce58\uc9c0 \ubabb\ud574 \uc81c4\ub2f9\uc73c\ub85c \uc804\ub77d\ud588\ub2e4. \uadf8\ub098\ub9c8 \uc9c0\ubc29\uc5d0\uc11c\ub294 \uc790\ubbfc\ub2f9\uacfc \uc0ac\ud68c\ub2f9\uc758 \uc591\ub2f9 \uccb4\uc81c\uac00 \uc720\uc9c0\ub418\uc5c8\uae30 \ub54c\ubb38\uc5d0, \uc0ac\ud68c\ub2f9\uc740 \ub3c4\uc2dc\ud615 \uc815\ub2f9\uc5d0\uc11c \ub18d\ucd0c\ud615 \uc815\ub2f9\uc73c\ub85c \uc804\ub77d\ud574 \uac14\ub2e4.", "paragraph_answer": " 55\ub144 \uccb4\uc81c\uac00 \uc131\ub9bd\ud55c \ub2f9\uc2dc\ub294 \uc0ac\ud68c\ub2f9\ub3c4 \uc815\uad8c \ud68d\ub4dd\uc744 \ubaa9\ud45c\ub85c \ud588\uc73c\uba70, \uc9c0\uc9c0\uc790\uc758 \uc0c1\ub2f9\uc218\ub3c4 \uc0ac\ud68c\ub2f9 \uc815\uad8c\uc744 \ubc14\ub77c\uace0 \uc788\uc5c8\ub2e4. \uadf8\ub7ec\ub098 \uc790\ubbfc\ub2f9\uc758 \uc77c\ub2f9 \uc6b0\uc704\uccb4\uc81c\uac00 \uc9c0\uc18d\ub418\uba74\uc11c, \uc0ac\ud68c\ub2f9\uc758 \uc815\uad8c \ud68d\ub4dd\uc5d0 \ub300\ud55c \uc5f4\ub9dd\uc740 \ud76c\ubbf8\ud574\uc9c0\uae30 \uc2dc\uc791\ud588\ub2e4. \ub178\ub3d9\uc870\ud569\uc744 \uae30\ubc18\uc73c\ub85c \ud55c \uc0ac\ud68c\ub2f9\uc774\uc9c0\ub9cc, \uc0ac\ud68c \uad6c\uc870\uac00 \ubcc0\ud654\ud558\uba74\uc11c \uc720\uad8c\uc790\ub4e4\uc740 \uc0ac\ud68c\uc8fc\uc758\uc5d0 \ud765\ubbf8\ub97c \uc783\uae30 \uc2dc\uc791\ud588\uace0, \uc0ac\ud68c\ub2f9 \ub610\ud55c \ud604\uc0c1 \uc720\uc9c0\uc5d0 \uae09\uae09\ud558\uba74\uc11c \uc7a5\uae30\uc801\uc73c\ub85c\ub294 \ub0ae\uc740 \ud558\ub77d\uc138\ub97c \ubcf4\uc774\uae30 \uc2dc\uc791\ud588\ub2e4. \uc0ac\ud68c\ub2f9\uc740 \u2018\ub9cc\ub144 \uc57c\ub2f9\u2019\uc73c\ub85c \ubd88\ub9ac\uba70, \uc9c0\uc9c0\uc790\ub4e4 \ub610\ud55c \uc790\ubbfc\ub2f9 \uc815\uad8c\uc774 \uc9c0\uc18d\ub418\ub294 \uac83\uc744 \uc804\uc81c\ub85c \ud558\ub294 \uc778\uc2dd\uc774 \ud37c\uc9c0\uae30 \uc2dc\uc791\ud588\ub2e4. \ud2b9\ud788 \ub3c4\uc2dc\uc9c0\uc5ed\uc5d0\uc11c \uc0ac\ud68c\ub2f9\uc774 \ubc00\ub824\ub098\uae30 \uc2dc\uc791\ud55c \uac83\uc740 \uc0c1\ub2f9\ud574, \ub3c4\ucfc4\ub3c4 \uc758\ud68c\uc5d0\uc11c\ub294 \uc790\ubbfc\ub2f9 \ubfd0\ub9cc \uc544\ub2c8\ub77c \uacf5\uba85\ub2f9\uacfc \uc77c\ubcf8\uacf5\uc0b0\ub2f9\uc5d0\ub3c4 \uc758\uc11d\uc774 \ubbf8\uce58\uc9c0 \ubabb\ud574 \uc81c4\ub2f9\uc73c\ub85c \uc804\ub77d\ud588\ub2e4. \uadf8\ub098\ub9c8 \uc9c0\ubc29\uc5d0\uc11c\ub294 \uc790\ubbfc\ub2f9\uacfc \uc0ac\ud68c\ub2f9\uc758 \uc591\ub2f9 \uccb4\uc81c\uac00 \uc720\uc9c0\ub418\uc5c8\uae30 \ub54c\ubb38\uc5d0, \uc0ac\ud68c\ub2f9\uc740 \ub3c4\uc2dc\ud615 \uc815\ub2f9\uc5d0\uc11c \ub18d\ucd0c\ud615 \uc815\ub2f9\uc73c\ub85c \uc804\ub77d\ud574 \uac14\ub2e4.", "sentence_answer": " 55\ub144 \uccb4\uc81c\uac00 \uc131\ub9bd\ud55c \ub2f9\uc2dc\ub294 \uc0ac\ud68c\ub2f9\ub3c4 \uc815\uad8c \ud68d\ub4dd\uc744 \ubaa9\ud45c\ub85c \ud588\uc73c\uba70, \uc9c0\uc9c0\uc790\uc758 \uc0c1\ub2f9\uc218\ub3c4 \uc0ac\ud68c\ub2f9 \uc815\uad8c\uc744 \ubc14\ub77c\uace0 \uc788\uc5c8\ub2e4.", "paragraph_id": "6591960-2-0", "paragraph_question": "question: \uc77c\ubcf8\uc0ac\ud68c\ub2f9\uc758 \uccb4\uc81c\uac00 \uc131\ub9bd\ub41c \uc2dc\uae30\ub294?, context: 55\ub144 \uccb4\uc81c\uac00 \uc131\ub9bd\ud55c \ub2f9\uc2dc\ub294 \uc0ac\ud68c\ub2f9\ub3c4 \uc815\uad8c \ud68d\ub4dd\uc744 \ubaa9\ud45c\ub85c \ud588\uc73c\uba70, \uc9c0\uc9c0\uc790\uc758 \uc0c1\ub2f9\uc218\ub3c4 \uc0ac\ud68c\ub2f9 \uc815\uad8c\uc744 \ubc14\ub77c\uace0 \uc788\uc5c8\ub2e4. \uadf8\ub7ec\ub098 \uc790\ubbfc\ub2f9\uc758 \uc77c\ub2f9 \uc6b0\uc704\uccb4\uc81c\uac00 \uc9c0\uc18d\ub418\uba74\uc11c, \uc0ac\ud68c\ub2f9\uc758 \uc815\uad8c \ud68d\ub4dd\uc5d0 \ub300\ud55c \uc5f4\ub9dd\uc740 \ud76c\ubbf8\ud574\uc9c0\uae30 \uc2dc\uc791\ud588\ub2e4. \ub178\ub3d9\uc870\ud569\uc744 \uae30\ubc18\uc73c\ub85c \ud55c \uc0ac\ud68c\ub2f9\uc774\uc9c0\ub9cc, \uc0ac\ud68c \uad6c\uc870\uac00 \ubcc0\ud654\ud558\uba74\uc11c \uc720\uad8c\uc790\ub4e4\uc740 \uc0ac\ud68c\uc8fc\uc758\uc5d0 \ud765\ubbf8\ub97c \uc783\uae30 \uc2dc\uc791\ud588\uace0, \uc0ac\ud68c\ub2f9 \ub610\ud55c \ud604\uc0c1 \uc720\uc9c0\uc5d0 \uae09\uae09\ud558\uba74\uc11c \uc7a5\uae30\uc801\uc73c\ub85c\ub294 \ub0ae\uc740 \ud558\ub77d\uc138\ub97c \ubcf4\uc774\uae30 \uc2dc\uc791\ud588\ub2e4. \uc0ac\ud68c\ub2f9\uc740 \u2018\ub9cc\ub144 \uc57c\ub2f9\u2019\uc73c\ub85c \ubd88\ub9ac\uba70, \uc9c0\uc9c0\uc790\ub4e4 \ub610\ud55c \uc790\ubbfc\ub2f9 \uc815\uad8c\uc774 \uc9c0\uc18d\ub418\ub294 \uac83\uc744 \uc804\uc81c\ub85c \ud558\ub294 \uc778\uc2dd\uc774 \ud37c\uc9c0\uae30 \uc2dc\uc791\ud588\ub2e4. \ud2b9\ud788 \ub3c4\uc2dc\uc9c0\uc5ed\uc5d0\uc11c \uc0ac\ud68c\ub2f9\uc774 \ubc00\ub824\ub098\uae30 \uc2dc\uc791\ud55c \uac83\uc740 \uc0c1\ub2f9\ud574, \ub3c4\ucfc4\ub3c4 \uc758\ud68c\uc5d0\uc11c\ub294 \uc790\ubbfc\ub2f9 \ubfd0\ub9cc \uc544\ub2c8\ub77c \uacf5\uba85\ub2f9\uacfc \uc77c\ubcf8\uacf5\uc0b0\ub2f9\uc5d0\ub3c4 \uc758\uc11d\uc774 \ubbf8\uce58\uc9c0 \ubabb\ud574 \uc81c4\ub2f9\uc73c\ub85c \uc804\ub77d\ud588\ub2e4. \uadf8\ub098\ub9c8 \uc9c0\ubc29\uc5d0\uc11c\ub294 \uc790\ubbfc\ub2f9\uacfc \uc0ac\ud68c\ub2f9\uc758 \uc591\ub2f9 \uccb4\uc81c\uac00 \uc720\uc9c0\ub418\uc5c8\uae30 \ub54c\ubb38\uc5d0, \uc0ac\ud68c\ub2f9\uc740 \ub3c4\uc2dc\ud615 \uc815\ub2f9\uc5d0\uc11c \ub18d\ucd0c\ud615 \uc815\ub2f9\uc73c\ub85c \uc804\ub77d\ud574 \uac14\ub2e4."} {"question": "\uadf8\ub9ac\ud53c\uc2a4\uc758 \uc2e4\ud5d8\uc744 \ud1b5\ud558\uc5ec \uc54c\uac8c\ub41c \ud615\uc9c8\ubcc0\ud658\uc758 \uc6d0\uc778\uc740 \ubb34\uc5c7\uc778\uac00?", "paragraph": "\uc5fc\uc0c9\uccb4\ub294 DNA\uc640 \ub2e8\ubc31\uc9c8\uc774 \uc5c9\ucf1c \uc788\ub294 \uad6c\uc870\ub2e4. \uc720\uc804\uacfc \uad00\ub828\ud55c \ubb3c\uc9c8\uc774 \uc5fc\uc0c9\uccb4\uc5d0 \uc788\ub2e4\ub294 \uac83\uc744 \ubc1c\uacac\ud55c \uc774\ud6c4\uc5d0\ub3c4 \uacfc\ud559\uc790\ub4e4\uc740 \uc815\ud655\ud788 \uc5fc\uc0c9\uccb4\uc758 \uc5b4\ub5a4 \uc131\ubd84\uc774 \uc720\uc804\uc5d0 \uad00\uc5ec\ud558\ub294\uc9c0 \ubc1d\ud600\ub0b4\uc9c0 \ubabb\ud558\uace0 \uc788\uc5c8\ub2e4. 1928\ub144 \uadf8\ub9ac\ud53c\uc2a4\ub294 \uadf8\ub9ac\ud53c\uc2a4 \uc2e4\ud5d8\uc744 \ud1b5\ud574 \ubc15\ud14c\ub9ac\uc544\uc758 \ud615\uc9c8\uc804\ud658\uc744 \ubc1c\uacac\ud558\uc600\ub2e4. \uadf8\uc758 \uc2e4\ud5d8\uc740 \uc720\ub3c5\ud55c \ud3d0\ub834\uc30d\uad6c\uade0(S\ud615)\uc5d0 \uc5f4\uc744 \uac00\ud558\uc5ec \ud30c\uad34\ud558\uba74 \ub3c5\uc131\uc774 \uc0ac\ub77c\uc9c0\uc9c0\ub9cc, \ubb34\ud574\ud55c \ud3d0\ub834\uc30d\uad6c\uade0(R\ud615)\uc5d0 \uc774\ubbf8 \uc5f4\ucc98\ub9ac\ud558\uc5ec \ub3c5\uc131\uc774 \uc0ac\ub77c\uc9c4 S\ud615 \uade0\uc744 \ub123\uc790 \ubaa8\ub450 \ub3c5\uc131\uc744 \uc9c0\ub2c8\uac8c \ub418\ub294 \uac83\uc744 \uad00\ucc30\ud558\uc600\ub2e4. \uadf8\ub9ac\ud53c\uc2a4\ub294 S\ud615 \uade0\uc758 \uc5b4\ub5a4 \uc131\ubd84\uc774 R\ud615\uc5d0 \uc601\ud5a5\uc744 \uc8fc\uc5b4 \ud615\uc9c8\uc804\ud658\uc774 \uc77c\uc5b4\ub0ac\ub2e4\ub294 \uac83\uc744 \uc54c\uc558\uc73c\ub098 \ubb34\uc5c7\uc774 \uadf8\ub7ec\ud55c \ubcc0\ud658\uc744 \uc77c\uc73c\ud0a4\ub294\uc9c0\ub294 \ubc1d\ud600\ub0b4\uc9c0 \ubabb\ud588\ub2e4. 1944\ub144 \uc5d0\uc774\ubc84\ub9ac\ub294 \uadf8\ub9ac\ud53c\uc2a4\uc758 \uc2e4\ud5d8\uc744 \ud6e8\uc52c \uc815\uad50\ud558\uac8c \ud1b5\uc81c\ud558\uc5ec \uc5f4\ucc98\ub9ac\ud55c S\ud615 \uade0\uc744 \ud0c4\uc218\ud654\ubb3c, \ub2e8\ubc31\uc9c8, DNA\ub85c \uad6c\ubd84\ud558\uc5ec R\ud615 \uade0\uc5d0 \ud22c\uc785\ud558\uc600\uace0, \uadf8 \uacb0\uacfc DNA\uac00 \ud615\uc9c8\ubcc0\ud658\uc758 \uc6d0\uc778\uc784\uc744 \ubc1d\ud600\ub0b4\uc5c8\ub2e4. 1952\ub144 \ud5c8\uc2dc\uc640 \uccb4\uc774\uc2a4\ub294 \ubc15\ud14c\ub9ac\uc624\ud30c\uc9c0\ub97c \uc774\uc6a9\ud55c \ud5c8\uc2dc-\uccb4\uc774\uc2a4 \uc2e4\ud5d8\uc744 \ud1b5\ud574 DNA\uac00 \uc720\uc804\ubb3c\uc9c8\uc784\uc744 \ubc1d\ud614\ub2e4.\ud5c8\uc2dc\ub294 \uc774 \uc2e4\ud5d8\uc758 \uacf5\ub85c\ub85c 1969\ub144 \ub178\ubca8 \uc0dd\ub9ac\ud559\u00b7\uc758\ud559\uc0c1\uc744 \uc218\uc0c1\ud558\uc600\ub2e4.", "answer": "DNA", "sentence": "\uc5fc\uc0c9\uccb4\ub294 DNA \uc640 \ub2e8\ubc31\uc9c8\uc774 \uc5c9\ucf1c \uc788\ub294 \uad6c\uc870\ub2e4.", "paragraph_sentence": " \uc5fc\uc0c9\uccb4\ub294 DNA \uc640 \ub2e8\ubc31\uc9c8\uc774 \uc5c9\ucf1c \uc788\ub294 \uad6c\uc870\ub2e4. \uc720\uc804\uacfc \uad00\ub828\ud55c \ubb3c\uc9c8\uc774 \uc5fc\uc0c9\uccb4\uc5d0 \uc788\ub2e4\ub294 \uac83\uc744 \ubc1c\uacac\ud55c \uc774\ud6c4\uc5d0\ub3c4 \uacfc\ud559\uc790\ub4e4\uc740 \uc815\ud655\ud788 \uc5fc\uc0c9\uccb4\uc758 \uc5b4\ub5a4 \uc131\ubd84\uc774 \uc720\uc804\uc5d0 \uad00\uc5ec\ud558\ub294\uc9c0 \ubc1d\ud600\ub0b4\uc9c0 \ubabb\ud558\uace0 \uc788\uc5c8\ub2e4. 1928\ub144 \uadf8\ub9ac\ud53c\uc2a4\ub294 \uadf8\ub9ac\ud53c\uc2a4 \uc2e4\ud5d8\uc744 \ud1b5\ud574 \ubc15\ud14c\ub9ac\uc544\uc758 \ud615\uc9c8\uc804\ud658\uc744 \ubc1c\uacac\ud558\uc600\ub2e4. \uadf8\uc758 \uc2e4\ud5d8\uc740 \uc720\ub3c5\ud55c \ud3d0\ub834\uc30d\uad6c\uade0(S\ud615)\uc5d0 \uc5f4\uc744 \uac00\ud558\uc5ec \ud30c\uad34\ud558\uba74 \ub3c5\uc131\uc774 \uc0ac\ub77c\uc9c0\uc9c0\ub9cc, \ubb34\ud574\ud55c \ud3d0\ub834\uc30d\uad6c\uade0(R\ud615)\uc5d0 \uc774\ubbf8 \uc5f4\ucc98\ub9ac\ud558\uc5ec \ub3c5\uc131\uc774 \uc0ac\ub77c\uc9c4 S\ud615 \uade0\uc744 \ub123\uc790 \ubaa8\ub450 \ub3c5\uc131\uc744 \uc9c0\ub2c8\uac8c \ub418\ub294 \uac83\uc744 \uad00\ucc30\ud558\uc600\ub2e4. \uadf8\ub9ac\ud53c\uc2a4\ub294 S\ud615 \uade0\uc758 \uc5b4\ub5a4 \uc131\ubd84\uc774 R\ud615\uc5d0 \uc601\ud5a5\uc744 \uc8fc\uc5b4 \ud615\uc9c8\uc804\ud658\uc774 \uc77c\uc5b4\ub0ac\ub2e4\ub294 \uac83\uc744 \uc54c\uc558\uc73c\ub098 \ubb34\uc5c7\uc774 \uadf8\ub7ec\ud55c \ubcc0\ud658\uc744 \uc77c\uc73c\ud0a4\ub294\uc9c0\ub294 \ubc1d\ud600\ub0b4\uc9c0 \ubabb\ud588\ub2e4. 1944\ub144 \uc5d0\uc774\ubc84\ub9ac\ub294 \uadf8\ub9ac\ud53c\uc2a4\uc758 \uc2e4\ud5d8\uc744 \ud6e8\uc52c \uc815\uad50\ud558\uac8c \ud1b5\uc81c\ud558\uc5ec \uc5f4\ucc98\ub9ac\ud55c S\ud615 \uade0\uc744 \ud0c4\uc218\ud654\ubb3c, \ub2e8\ubc31\uc9c8, DNA\ub85c \uad6c\ubd84\ud558\uc5ec R\ud615 \uade0\uc5d0 \ud22c\uc785\ud558\uc600\uace0, \uadf8 \uacb0\uacfc DNA\uac00 \ud615\uc9c8\ubcc0\ud658\uc758 \uc6d0\uc778\uc784\uc744 \ubc1d\ud600\ub0b4\uc5c8\ub2e4. 1952\ub144 \ud5c8\uc2dc\uc640 \uccb4\uc774\uc2a4\ub294 \ubc15\ud14c\ub9ac\uc624\ud30c\uc9c0\ub97c \uc774\uc6a9\ud55c \ud5c8\uc2dc-\uccb4\uc774\uc2a4 \uc2e4\ud5d8\uc744 \ud1b5\ud574 DNA\uac00 \uc720\uc804\ubb3c\uc9c8\uc784\uc744 \ubc1d\ud614\ub2e4.\ud5c8\uc2dc\ub294 \uc774 \uc2e4\ud5d8\uc758 \uacf5\ub85c\ub85c 1969\ub144 \ub178\ubca8 \uc0dd\ub9ac\ud559\u00b7\uc758\ud559\uc0c1\uc744 \uc218\uc0c1\ud558\uc600\ub2e4.", "paragraph_answer": "\uc5fc\uc0c9\uccb4\ub294 DNA \uc640 \ub2e8\ubc31\uc9c8\uc774 \uc5c9\ucf1c \uc788\ub294 \uad6c\uc870\ub2e4. \uc720\uc804\uacfc \uad00\ub828\ud55c \ubb3c\uc9c8\uc774 \uc5fc\uc0c9\uccb4\uc5d0 \uc788\ub2e4\ub294 \uac83\uc744 \ubc1c\uacac\ud55c \uc774\ud6c4\uc5d0\ub3c4 \uacfc\ud559\uc790\ub4e4\uc740 \uc815\ud655\ud788 \uc5fc\uc0c9\uccb4\uc758 \uc5b4\ub5a4 \uc131\ubd84\uc774 \uc720\uc804\uc5d0 \uad00\uc5ec\ud558\ub294\uc9c0 \ubc1d\ud600\ub0b4\uc9c0 \ubabb\ud558\uace0 \uc788\uc5c8\ub2e4. 1928\ub144 \uadf8\ub9ac\ud53c\uc2a4\ub294 \uadf8\ub9ac\ud53c\uc2a4 \uc2e4\ud5d8\uc744 \ud1b5\ud574 \ubc15\ud14c\ub9ac\uc544\uc758 \ud615\uc9c8\uc804\ud658\uc744 \ubc1c\uacac\ud558\uc600\ub2e4. \uadf8\uc758 \uc2e4\ud5d8\uc740 \uc720\ub3c5\ud55c \ud3d0\ub834\uc30d\uad6c\uade0(S\ud615)\uc5d0 \uc5f4\uc744 \uac00\ud558\uc5ec \ud30c\uad34\ud558\uba74 \ub3c5\uc131\uc774 \uc0ac\ub77c\uc9c0\uc9c0\ub9cc, \ubb34\ud574\ud55c \ud3d0\ub834\uc30d\uad6c\uade0(R\ud615)\uc5d0 \uc774\ubbf8 \uc5f4\ucc98\ub9ac\ud558\uc5ec \ub3c5\uc131\uc774 \uc0ac\ub77c\uc9c4 S\ud615 \uade0\uc744 \ub123\uc790 \ubaa8\ub450 \ub3c5\uc131\uc744 \uc9c0\ub2c8\uac8c \ub418\ub294 \uac83\uc744 \uad00\ucc30\ud558\uc600\ub2e4. \uadf8\ub9ac\ud53c\uc2a4\ub294 S\ud615 \uade0\uc758 \uc5b4\ub5a4 \uc131\ubd84\uc774 R\ud615\uc5d0 \uc601\ud5a5\uc744 \uc8fc\uc5b4 \ud615\uc9c8\uc804\ud658\uc774 \uc77c\uc5b4\ub0ac\ub2e4\ub294 \uac83\uc744 \uc54c\uc558\uc73c\ub098 \ubb34\uc5c7\uc774 \uadf8\ub7ec\ud55c \ubcc0\ud658\uc744 \uc77c\uc73c\ud0a4\ub294\uc9c0\ub294 \ubc1d\ud600\ub0b4\uc9c0 \ubabb\ud588\ub2e4. 1944\ub144 \uc5d0\uc774\ubc84\ub9ac\ub294 \uadf8\ub9ac\ud53c\uc2a4\uc758 \uc2e4\ud5d8\uc744 \ud6e8\uc52c \uc815\uad50\ud558\uac8c \ud1b5\uc81c\ud558\uc5ec \uc5f4\ucc98\ub9ac\ud55c S\ud615 \uade0\uc744 \ud0c4\uc218\ud654\ubb3c, \ub2e8\ubc31\uc9c8, DNA\ub85c \uad6c\ubd84\ud558\uc5ec R\ud615 \uade0\uc5d0 \ud22c\uc785\ud558\uc600\uace0, \uadf8 \uacb0\uacfc DNA\uac00 \ud615\uc9c8\ubcc0\ud658\uc758 \uc6d0\uc778\uc784\uc744 \ubc1d\ud600\ub0b4\uc5c8\ub2e4. 1952\ub144 \ud5c8\uc2dc\uc640 \uccb4\uc774\uc2a4\ub294 \ubc15\ud14c\ub9ac\uc624\ud30c\uc9c0\ub97c \uc774\uc6a9\ud55c \ud5c8\uc2dc-\uccb4\uc774\uc2a4 \uc2e4\ud5d8\uc744 \ud1b5\ud574 DNA\uac00 \uc720\uc804\ubb3c\uc9c8\uc784\uc744 \ubc1d\ud614\ub2e4.\ud5c8\uc2dc\ub294 \uc774 \uc2e4\ud5d8\uc758 \uacf5\ub85c\ub85c 1969\ub144 \ub178\ubca8 \uc0dd\ub9ac\ud559\u00b7\uc758\ud559\uc0c1\uc744 \uc218\uc0c1\ud558\uc600\ub2e4.", "sentence_answer": "\uc5fc\uc0c9\uccb4\ub294 DNA \uc640 \ub2e8\ubc31\uc9c8\uc774 \uc5c9\ucf1c \uc788\ub294 \uad6c\uc870\ub2e4.", "paragraph_id": "6543400-1-1", "paragraph_question": "question: \uadf8\ub9ac\ud53c\uc2a4\uc758 \uc2e4\ud5d8\uc744 \ud1b5\ud558\uc5ec \uc54c\uac8c\ub41c \ud615\uc9c8\ubcc0\ud658\uc758 \uc6d0\uc778\uc740 \ubb34\uc5c7\uc778\uac00?, context: \uc5fc\uc0c9\uccb4\ub294 DNA\uc640 \ub2e8\ubc31\uc9c8\uc774 \uc5c9\ucf1c \uc788\ub294 \uad6c\uc870\ub2e4. \uc720\uc804\uacfc \uad00\ub828\ud55c \ubb3c\uc9c8\uc774 \uc5fc\uc0c9\uccb4\uc5d0 \uc788\ub2e4\ub294 \uac83\uc744 \ubc1c\uacac\ud55c \uc774\ud6c4\uc5d0\ub3c4 \uacfc\ud559\uc790\ub4e4\uc740 \uc815\ud655\ud788 \uc5fc\uc0c9\uccb4\uc758 \uc5b4\ub5a4 \uc131\ubd84\uc774 \uc720\uc804\uc5d0 \uad00\uc5ec\ud558\ub294\uc9c0 \ubc1d\ud600\ub0b4\uc9c0 \ubabb\ud558\uace0 \uc788\uc5c8\ub2e4. 1928\ub144 \uadf8\ub9ac\ud53c\uc2a4\ub294 \uadf8\ub9ac\ud53c\uc2a4 \uc2e4\ud5d8\uc744 \ud1b5\ud574 \ubc15\ud14c\ub9ac\uc544\uc758 \ud615\uc9c8\uc804\ud658\uc744 \ubc1c\uacac\ud558\uc600\ub2e4. \uadf8\uc758 \uc2e4\ud5d8\uc740 \uc720\ub3c5\ud55c \ud3d0\ub834\uc30d\uad6c\uade0(S\ud615)\uc5d0 \uc5f4\uc744 \uac00\ud558\uc5ec \ud30c\uad34\ud558\uba74 \ub3c5\uc131\uc774 \uc0ac\ub77c\uc9c0\uc9c0\ub9cc, \ubb34\ud574\ud55c \ud3d0\ub834\uc30d\uad6c\uade0(R\ud615)\uc5d0 \uc774\ubbf8 \uc5f4\ucc98\ub9ac\ud558\uc5ec \ub3c5\uc131\uc774 \uc0ac\ub77c\uc9c4 S\ud615 \uade0\uc744 \ub123\uc790 \ubaa8\ub450 \ub3c5\uc131\uc744 \uc9c0\ub2c8\uac8c \ub418\ub294 \uac83\uc744 \uad00\ucc30\ud558\uc600\ub2e4. \uadf8\ub9ac\ud53c\uc2a4\ub294 S\ud615 \uade0\uc758 \uc5b4\ub5a4 \uc131\ubd84\uc774 R\ud615\uc5d0 \uc601\ud5a5\uc744 \uc8fc\uc5b4 \ud615\uc9c8\uc804\ud658\uc774 \uc77c\uc5b4\ub0ac\ub2e4\ub294 \uac83\uc744 \uc54c\uc558\uc73c\ub098 \ubb34\uc5c7\uc774 \uadf8\ub7ec\ud55c \ubcc0\ud658\uc744 \uc77c\uc73c\ud0a4\ub294\uc9c0\ub294 \ubc1d\ud600\ub0b4\uc9c0 \ubabb\ud588\ub2e4. 1944\ub144 \uc5d0\uc774\ubc84\ub9ac\ub294 \uadf8\ub9ac\ud53c\uc2a4\uc758 \uc2e4\ud5d8\uc744 \ud6e8\uc52c \uc815\uad50\ud558\uac8c \ud1b5\uc81c\ud558\uc5ec \uc5f4\ucc98\ub9ac\ud55c S\ud615 \uade0\uc744 \ud0c4\uc218\ud654\ubb3c, \ub2e8\ubc31\uc9c8, DNA\ub85c \uad6c\ubd84\ud558\uc5ec R\ud615 \uade0\uc5d0 \ud22c\uc785\ud558\uc600\uace0, \uadf8 \uacb0\uacfc DNA\uac00 \ud615\uc9c8\ubcc0\ud658\uc758 \uc6d0\uc778\uc784\uc744 \ubc1d\ud600\ub0b4\uc5c8\ub2e4. 1952\ub144 \ud5c8\uc2dc\uc640 \uccb4\uc774\uc2a4\ub294 \ubc15\ud14c\ub9ac\uc624\ud30c\uc9c0\ub97c \uc774\uc6a9\ud55c \ud5c8\uc2dc-\uccb4\uc774\uc2a4 \uc2e4\ud5d8\uc744 \ud1b5\ud574 DNA\uac00 \uc720\uc804\ubb3c\uc9c8\uc784\uc744 \ubc1d\ud614\ub2e4.\ud5c8\uc2dc\ub294 \uc774 \uc2e4\ud5d8\uc758 \uacf5\ub85c\ub85c 1969\ub144 \ub178\ubca8 \uc0dd\ub9ac\ud559\u00b7\uc758\ud559\uc0c1\uc744 \uc218\uc0c1\ud558\uc600\ub2e4."} {"question": "\uc11d\uc601\uc720\ub9ac\ub294 \uc5f0\ud654\ud558\ub294 \uc628\ub3c4 \ub54c\ubb38\uc5d0 \uc5b4\ub5a4 \uc791\uc5c5\uc774 \uc5b4\ub824\uc6b4\uac00?", "paragraph": "\uc720\ub9ac \uad6c\uc870\ub294 \uaddc\uc18c\uc6d0\uc790\uc640 \uc0b0\uc18c\uc6d0\uc790\uac00 \uacb0\ud569\ud558\uc5ec \ub9cc\ub4e0 \uadf8\ubb3c\ucf54\uc5d0 \uad70\ub370\uad70\ub370 \uad6c\uba4d\uc774 \ub0a8\uaca8\uc9c4 \uac83 \uac19\uc740 \ubaa8\uc591\uc73c\ub85c \uc774\ub8e8\uc5b4\uc84c\ub2e4. \uc21c\uc218\ud55c \uaddc\uc18c\uc640 \uc0b0\uc18c\ub9cc\uc758 \uadf8\ubb3c\ucf54\ub85c \ub41c \uc720\ub9ac\ub97c \uc11d\uc601\uc720\ub9ac\ub77c\uace0 \ubd80\ub974\ub294\ub370 \uc5f0\ud654\uc810(\u8edf\u5316\u9ede)\uc774 1,500\u2103 \uc774\uc0c1(\ubcf4\ud1b5 \uc18c\ub2e4\uc11d\ud68c\uc720\ub9ac\ub294 \uc57d 600\u2103)\uc73c\ub85c\uc11c \uc57d\ud488\uc5d0 \uc758\ud574 \uce68\uc2dd\ub418\uc9c0\ub3c4 \uc54a\uc744 \ubfd0 \uc544\ub2c8\ub77c 1,000\u2103 \uc815\ub3c4\uc758 \uc628\ub3c4\ucc28\ub85c \uae09\uc5f4\u00b7\uae09\ub7ad\ub418\uc5b4\ub3c4 \uae68\uc9c0\uc9c0 \uc54a\ub294\ub2e4. \uc11d\uc601\uc720\ub9ac\ub294 \uc774\ucc98\ub7fc \uc6b0\uc218\ud55c \uc5ec\ub7ec \uc131\uc9c8\uc744 \uac00\uc84c\uc73c\ub098 \uc5f0\ud654\ud558\ub294 \uc628\ub3c4\uac00 1,500\u2103 \uc774\uc0c1\uc774\ubbc0\ub85c \uac00\uacf5\u00b7\uc131\ud615\uc774 \uadf9\ud788 \uc5b4\ub835\ub2e4. \ub610\ud55c \uc6d0\ub8cc\ub97c \uc720\ub9ac\ud654\ud558\ub824\uba74 1,717\u2103 \uc774\uc0c1\uc758 \uace0\uc628\uc744 \ub9cc\ub4e4\uc5b4 \uc8fc\uc5b4\uc57c \ud558\ubbc0\ub85c \ub3c4\uac00\ub2c8\uc758 \uc7ac\ub8cc, \uace0\uc628\uc744 \ub9cc\ub4dc\ub294 \ubc29\ubc95 \ub4f1 \uae30\uc220\uc801\uc73c\ub85c \uace4\ub780\ud55c \uc810\uc774 \ub9ce\uc73c\uba70, \ub530\ub77c\uc11c \uac12\uc774 \ubab9\uc2dc \ube44\uc2f8 \ud2b9\uc218\ud55c \uacbd\uc6b0 \uc774\uc678\ub294 \uc0ac\uc6a9\ub418\uc9c0 \uc54a\ub294\ub2e4.", "answer": "\uac00\uacf5\u00b7\uc131\ud615", "sentence": "\uc11d\uc601\uc720\ub9ac\ub294 \uc774\ucc98\ub7fc \uc6b0\uc218\ud55c \uc5ec\ub7ec \uc131\uc9c8\uc744 \uac00\uc84c\uc73c\ub098 \uc5f0\ud654\ud558\ub294 \uc628\ub3c4\uac00 1,500\u2103 \uc774\uc0c1\uc774\ubbc0\ub85c \uac00\uacf5\u00b7\uc131\ud615 \uc774 \uadf9\ud788 \uc5b4\ub835\ub2e4.", "paragraph_sentence": "\uc720\ub9ac \uad6c\uc870\ub294 \uaddc\uc18c\uc6d0\uc790\uc640 \uc0b0\uc18c\uc6d0\uc790\uac00 \uacb0\ud569\ud558\uc5ec \ub9cc\ub4e0 \uadf8\ubb3c\ucf54\uc5d0 \uad70\ub370\uad70\ub370 \uad6c\uba4d\uc774 \ub0a8\uaca8\uc9c4 \uac83 \uac19\uc740 \ubaa8\uc591\uc73c\ub85c \uc774\ub8e8\uc5b4\uc84c\ub2e4. \uc21c\uc218\ud55c \uaddc\uc18c\uc640 \uc0b0\uc18c\ub9cc\uc758 \uadf8\ubb3c\ucf54\ub85c \ub41c \uc720\ub9ac\ub97c \uc11d\uc601\uc720\ub9ac\ub77c\uace0 \ubd80\ub974\ub294\ub370 \uc5f0\ud654\uc810(\u8edf\u5316\u9ede)\uc774 1,500\u2103 \uc774\uc0c1(\ubcf4\ud1b5 \uc18c\ub2e4\uc11d\ud68c\uc720\ub9ac\ub294 \uc57d 600\u2103)\uc73c\ub85c\uc11c \uc57d\ud488\uc5d0 \uc758\ud574 \uce68\uc2dd\ub418\uc9c0\ub3c4 \uc54a\uc744 \ubfd0 \uc544\ub2c8\ub77c 1,000\u2103 \uc815\ub3c4\uc758 \uc628\ub3c4\ucc28\ub85c \uae09\uc5f4\u00b7\uae09\ub7ad\ub418\uc5b4\ub3c4 \uae68\uc9c0\uc9c0 \uc54a\ub294\ub2e4. \uc11d\uc601\uc720\ub9ac\ub294 \uc774\ucc98\ub7fc \uc6b0\uc218\ud55c \uc5ec\ub7ec \uc131\uc9c8\uc744 \uac00\uc84c\uc73c\ub098 \uc5f0\ud654\ud558\ub294 \uc628\ub3c4\uac00 1,500\u2103 \uc774\uc0c1\uc774\ubbc0\ub85c \uac00\uacf5\u00b7\uc131\ud615 \uc774 \uadf9\ud788 \uc5b4\ub835\ub2e4. \ub610\ud55c \uc6d0\ub8cc\ub97c \uc720\ub9ac\ud654\ud558\ub824\uba74 1,717\u2103 \uc774\uc0c1\uc758 \uace0\uc628\uc744 \ub9cc\ub4e4\uc5b4 \uc8fc\uc5b4\uc57c \ud558\ubbc0\ub85c \ub3c4\uac00\ub2c8\uc758 \uc7ac\ub8cc, \uace0\uc628\uc744 \ub9cc\ub4dc\ub294 \ubc29\ubc95 \ub4f1 \uae30\uc220\uc801\uc73c\ub85c \uace4\ub780\ud55c \uc810\uc774 \ub9ce\uc73c\uba70, \ub530\ub77c\uc11c \uac12\uc774 \ubab9\uc2dc \ube44\uc2f8 \ud2b9\uc218\ud55c \uacbd\uc6b0 \uc774\uc678\ub294 \uc0ac\uc6a9\ub418\uc9c0 \uc54a\ub294\ub2e4.", "paragraph_answer": "\uc720\ub9ac \uad6c\uc870\ub294 \uaddc\uc18c\uc6d0\uc790\uc640 \uc0b0\uc18c\uc6d0\uc790\uac00 \uacb0\ud569\ud558\uc5ec \ub9cc\ub4e0 \uadf8\ubb3c\ucf54\uc5d0 \uad70\ub370\uad70\ub370 \uad6c\uba4d\uc774 \ub0a8\uaca8\uc9c4 \uac83 \uac19\uc740 \ubaa8\uc591\uc73c\ub85c \uc774\ub8e8\uc5b4\uc84c\ub2e4. \uc21c\uc218\ud55c \uaddc\uc18c\uc640 \uc0b0\uc18c\ub9cc\uc758 \uadf8\ubb3c\ucf54\ub85c \ub41c \uc720\ub9ac\ub97c \uc11d\uc601\uc720\ub9ac\ub77c\uace0 \ubd80\ub974\ub294\ub370 \uc5f0\ud654\uc810(\u8edf\u5316\u9ede)\uc774 1,500\u2103 \uc774\uc0c1(\ubcf4\ud1b5 \uc18c\ub2e4\uc11d\ud68c\uc720\ub9ac\ub294 \uc57d 600\u2103)\uc73c\ub85c\uc11c \uc57d\ud488\uc5d0 \uc758\ud574 \uce68\uc2dd\ub418\uc9c0\ub3c4 \uc54a\uc744 \ubfd0 \uc544\ub2c8\ub77c 1,000\u2103 \uc815\ub3c4\uc758 \uc628\ub3c4\ucc28\ub85c \uae09\uc5f4\u00b7\uae09\ub7ad\ub418\uc5b4\ub3c4 \uae68\uc9c0\uc9c0 \uc54a\ub294\ub2e4. \uc11d\uc601\uc720\ub9ac\ub294 \uc774\ucc98\ub7fc \uc6b0\uc218\ud55c \uc5ec\ub7ec \uc131\uc9c8\uc744 \uac00\uc84c\uc73c\ub098 \uc5f0\ud654\ud558\ub294 \uc628\ub3c4\uac00 1,500\u2103 \uc774\uc0c1\uc774\ubbc0\ub85c \uac00\uacf5\u00b7\uc131\ud615 \uc774 \uadf9\ud788 \uc5b4\ub835\ub2e4. \ub610\ud55c \uc6d0\ub8cc\ub97c \uc720\ub9ac\ud654\ud558\ub824\uba74 1,717\u2103 \uc774\uc0c1\uc758 \uace0\uc628\uc744 \ub9cc\ub4e4\uc5b4 \uc8fc\uc5b4\uc57c \ud558\ubbc0\ub85c \ub3c4\uac00\ub2c8\uc758 \uc7ac\ub8cc, \uace0\uc628\uc744 \ub9cc\ub4dc\ub294 \ubc29\ubc95 \ub4f1 \uae30\uc220\uc801\uc73c\ub85c \uace4\ub780\ud55c \uc810\uc774 \ub9ce\uc73c\uba70, \ub530\ub77c\uc11c \uac12\uc774 \ubab9\uc2dc \ube44\uc2f8 \ud2b9\uc218\ud55c \uacbd\uc6b0 \uc774\uc678\ub294 \uc0ac\uc6a9\ub418\uc9c0 \uc54a\ub294\ub2e4.", "sentence_answer": "\uc11d\uc601\uc720\ub9ac\ub294 \uc774\ucc98\ub7fc \uc6b0\uc218\ud55c \uc5ec\ub7ec \uc131\uc9c8\uc744 \uac00\uc84c\uc73c\ub098 \uc5f0\ud654\ud558\ub294 \uc628\ub3c4\uac00 1,500\u2103 \uc774\uc0c1\uc774\ubbc0\ub85c \uac00\uacf5\u00b7\uc131\ud615 \uc774 \uadf9\ud788 \uc5b4\ub835\ub2e4.", "paragraph_id": "6147943-6-1", "paragraph_question": "question: \uc11d\uc601\uc720\ub9ac\ub294 \uc5f0\ud654\ud558\ub294 \uc628\ub3c4 \ub54c\ubb38\uc5d0 \uc5b4\ub5a4 \uc791\uc5c5\uc774 \uc5b4\ub824\uc6b4\uac00?, context: \uc720\ub9ac \uad6c\uc870\ub294 \uaddc\uc18c\uc6d0\uc790\uc640 \uc0b0\uc18c\uc6d0\uc790\uac00 \uacb0\ud569\ud558\uc5ec \ub9cc\ub4e0 \uadf8\ubb3c\ucf54\uc5d0 \uad70\ub370\uad70\ub370 \uad6c\uba4d\uc774 \ub0a8\uaca8\uc9c4 \uac83 \uac19\uc740 \ubaa8\uc591\uc73c\ub85c \uc774\ub8e8\uc5b4\uc84c\ub2e4. \uc21c\uc218\ud55c \uaddc\uc18c\uc640 \uc0b0\uc18c\ub9cc\uc758 \uadf8\ubb3c\ucf54\ub85c \ub41c \uc720\ub9ac\ub97c \uc11d\uc601\uc720\ub9ac\ub77c\uace0 \ubd80\ub974\ub294\ub370 \uc5f0\ud654\uc810(\u8edf\u5316\u9ede)\uc774 1,500\u2103 \uc774\uc0c1(\ubcf4\ud1b5 \uc18c\ub2e4\uc11d\ud68c\uc720\ub9ac\ub294 \uc57d 600\u2103)\uc73c\ub85c\uc11c \uc57d\ud488\uc5d0 \uc758\ud574 \uce68\uc2dd\ub418\uc9c0\ub3c4 \uc54a\uc744 \ubfd0 \uc544\ub2c8\ub77c 1,000\u2103 \uc815\ub3c4\uc758 \uc628\ub3c4\ucc28\ub85c \uae09\uc5f4\u00b7\uae09\ub7ad\ub418\uc5b4\ub3c4 \uae68\uc9c0\uc9c0 \uc54a\ub294\ub2e4. \uc11d\uc601\uc720\ub9ac\ub294 \uc774\ucc98\ub7fc \uc6b0\uc218\ud55c \uc5ec\ub7ec \uc131\uc9c8\uc744 \uac00\uc84c\uc73c\ub098 \uc5f0\ud654\ud558\ub294 \uc628\ub3c4\uac00 1,500\u2103 \uc774\uc0c1\uc774\ubbc0\ub85c \uac00\uacf5\u00b7\uc131\ud615\uc774 \uadf9\ud788 \uc5b4\ub835\ub2e4. \ub610\ud55c \uc6d0\ub8cc\ub97c \uc720\ub9ac\ud654\ud558\ub824\uba74 1,717\u2103 \uc774\uc0c1\uc758 \uace0\uc628\uc744 \ub9cc\ub4e4\uc5b4 \uc8fc\uc5b4\uc57c \ud558\ubbc0\ub85c \ub3c4\uac00\ub2c8\uc758 \uc7ac\ub8cc, \uace0\uc628\uc744 \ub9cc\ub4dc\ub294 \ubc29\ubc95 \ub4f1 \uae30\uc220\uc801\uc73c\ub85c \uace4\ub780\ud55c \uc810\uc774 \ub9ce\uc73c\uba70, \ub530\ub77c\uc11c \uac12\uc774 \ubab9\uc2dc \ube44\uc2f8 \ud2b9\uc218\ud55c \uacbd\uc6b0 \uc774\uc678\ub294 \uc0ac\uc6a9\ub418\uc9c0 \uc54a\ub294\ub2e4."} {"question": "\ub9cc\uc288\ud0c0\uc778\uc740 \ub204\uad6c\uc5d0\uac8c \uc785\uc591 \ub418\uc5c8\ub294\uac00?", "paragraph": "\ub9cc\uc288\ud0c0\uc778\uc740 1887\ub144 \ubca0\ub97c\ub9b0\uc5d0\uc11c \ud504\ub85c\uc774\uc13c\uacc4 \uadc0\uc871\uc774\uc790 \ud3ec\ubcd1\ub300\uc7a5\uc778 \uc5d0\ub450\uc544\ub974\ud2b8 \ud3f0 \ub808\ube48\uc2a4\ud0a4(1829\ub144 ~ 1906\ub144)\uc640 \uadf8 \uc544\ub0b4 \ud5ec\ub808\ub124 \ud3f0 \uc288\ud398\ub97c\ub9c1(1847\ub144 ~ 1910\ub144) \uc0ac\uc774\uc758 \uc5f4\uc9f8 \uc544\ub4e4 \ud504\ub9ac\uce20 \uc5d0\ub9ac\ud788 \uac8c\uc624\ub974\ud06c \uc5d0\ub450\uc544\ub974\ud2b8 \ud3f0 \ub808\ube48\uc2a4\ud0a4\ub77c\ub294 \uc774\ub984\uc73c\ub85c \ud0dc\uc5b4\ub0ac\ub2e4. \ud3f0 \ub808\ube48\uc2a4\ud0a4 \uac00\ubb38\uc740 \uce74\uc288\ube44\uc544\uc778 \ud608\ud1b5\uc774\uc5c8\uc73c\uba70 \ube0c\ub85c\ud750\ube44\uce20 \ubb38\uc7a5\uc744 \uc0ac\uc6a9\ud558\ub294 \uadc0\uc871\uac00\uc600\ub2e4. \uc5d0\ub9ac\ud788\uc758 \uc774\ubaa8 \ud5e4\ub4dc\ube44\ud788 \ud3f0 \uc288\ud398\ub97c\ub9c1(1852\ub144 ~ 1925\ub144)\uc774 \uc911\uc7a5 \uac8c\uc624\ub974\ud06c \ud3f0 \ub9cc\uc288\ud0c0\uc778(1844\ub144 ~ 1913\ub144)\uacfc \uacb0\ud63c\ud588\uc73c\ub098 \uc2ac\ud558\uc5d0 \uc790\ub140\uac00 \uc5c6\uc5c8\uae30\uc5d0 \uc5d0\ub9ac\ud788\ub97c \uc785\uc591\ud568\uc73c\ub85c\uc368 \uc5d0\ub9ac\ud788\ub294 \ud3f0 \ub9cc\uc288\ud0c0\uc778\uc774 \ub418\uc5c8\ub2e4. \ub9cc\uc288\ud0c0\uc778 \uc9d1\uc548\uc740 \uadf8\uc804\uc5d0\ub3c4 \uc5d0\ub9ac\ud788\uc758 \uc0ac\ucd0c \ub9c8\ub974\ud0c0(\ud5ec\ub808\ub124 \ud5e4\ub4dc\ube44\ud788 \uc790\ub9e4\uc758 \uc8fd\uc740 \ud615\uc81c\uc758 \ub538)\ub97c \uc785\uc591\ud588\uc5c8\ub2e4.", "answer": "\uc774\ubaa8", "sentence": "\uc5d0\ub9ac\ud788\uc758 \uc774\ubaa8 \ud5e4\ub4dc\ube44\ud788 \ud3f0 \uc288\ud398\ub97c\ub9c1(1852\ub144 ~ 1925\ub144)\uc774 \uc911\uc7a5 \uac8c\uc624\ub974\ud06c \ud3f0 \ub9cc\uc288\ud0c0\uc778(1844\ub144 ~ 1913\ub144)\uacfc \uacb0\ud63c\ud588\uc73c\ub098 \uc2ac\ud558\uc5d0 \uc790\ub140\uac00 \uc5c6\uc5c8\uae30\uc5d0 \uc5d0\ub9ac\ud788\ub97c \uc785\uc591\ud568\uc73c\ub85c\uc368 \uc5d0\ub9ac\ud788\ub294 \ud3f0 \ub9cc\uc288\ud0c0\uc778\uc774 \ub418\uc5c8\ub2e4.", "paragraph_sentence": "\ub9cc\uc288\ud0c0\uc778\uc740 1887\ub144 \ubca0\ub97c\ub9b0\uc5d0\uc11c \ud504\ub85c\uc774\uc13c\uacc4 \uadc0\uc871\uc774\uc790 \ud3ec\ubcd1\ub300\uc7a5\uc778 \uc5d0\ub450\uc544\ub974\ud2b8 \ud3f0 \ub808\ube48\uc2a4\ud0a4(1829\ub144 ~ 1906\ub144)\uc640 \uadf8 \uc544\ub0b4 \ud5ec\ub808\ub124 \ud3f0 \uc288\ud398\ub97c\ub9c1(1847\ub144 ~ 1910\ub144) \uc0ac\uc774\uc758 \uc5f4\uc9f8 \uc544\ub4e4 \ud504\ub9ac\uce20 \uc5d0\ub9ac\ud788 \uac8c\uc624\ub974\ud06c \uc5d0\ub450\uc544\ub974\ud2b8 \ud3f0 \ub808\ube48\uc2a4\ud0a4\ub77c\ub294 \uc774\ub984\uc73c\ub85c \ud0dc\uc5b4\ub0ac\ub2e4. \ud3f0 \ub808\ube48\uc2a4\ud0a4 \uac00\ubb38\uc740 \uce74\uc288\ube44\uc544\uc778 \ud608\ud1b5\uc774\uc5c8\uc73c\uba70 \ube0c\ub85c\ud750\ube44\uce20 \ubb38\uc7a5\uc744 \uc0ac\uc6a9\ud558\ub294 \uadc0\uc871\uac00\uc600\ub2e4. \uc5d0\ub9ac\ud788\uc758 \uc774\ubaa8 \ud5e4\ub4dc\ube44\ud788 \ud3f0 \uc288\ud398\ub97c\ub9c1(1852\ub144 ~ 1925\ub144)\uc774 \uc911\uc7a5 \uac8c\uc624\ub974\ud06c \ud3f0 \ub9cc\uc288\ud0c0\uc778(1844\ub144 ~ 1913\ub144)\uacfc \uacb0\ud63c\ud588\uc73c\ub098 \uc2ac\ud558\uc5d0 \uc790\ub140\uac00 \uc5c6\uc5c8\uae30\uc5d0 \uc5d0\ub9ac\ud788\ub97c \uc785\uc591\ud568\uc73c\ub85c\uc368 \uc5d0\ub9ac\ud788\ub294 \ud3f0 \ub9cc\uc288\ud0c0\uc778\uc774 \ub418\uc5c8\ub2e4. \ub9cc\uc288\ud0c0\uc778 \uc9d1\uc548\uc740 \uadf8\uc804\uc5d0\ub3c4 \uc5d0\ub9ac\ud788\uc758 \uc0ac\ucd0c \ub9c8\ub974\ud0c0(\ud5ec\ub808\ub124 \ud5e4\ub4dc\ube44\ud788 \uc790\ub9e4\uc758 \uc8fd\uc740 \ud615\uc81c\uc758 \ub538)\ub97c \uc785\uc591\ud588\uc5c8\ub2e4.", "paragraph_answer": "\ub9cc\uc288\ud0c0\uc778\uc740 1887\ub144 \ubca0\ub97c\ub9b0\uc5d0\uc11c \ud504\ub85c\uc774\uc13c\uacc4 \uadc0\uc871\uc774\uc790 \ud3ec\ubcd1\ub300\uc7a5\uc778 \uc5d0\ub450\uc544\ub974\ud2b8 \ud3f0 \ub808\ube48\uc2a4\ud0a4(1829\ub144 ~ 1906\ub144)\uc640 \uadf8 \uc544\ub0b4 \ud5ec\ub808\ub124 \ud3f0 \uc288\ud398\ub97c\ub9c1(1847\ub144 ~ 1910\ub144) \uc0ac\uc774\uc758 \uc5f4\uc9f8 \uc544\ub4e4 \ud504\ub9ac\uce20 \uc5d0\ub9ac\ud788 \uac8c\uc624\ub974\ud06c \uc5d0\ub450\uc544\ub974\ud2b8 \ud3f0 \ub808\ube48\uc2a4\ud0a4\ub77c\ub294 \uc774\ub984\uc73c\ub85c \ud0dc\uc5b4\ub0ac\ub2e4. \ud3f0 \ub808\ube48\uc2a4\ud0a4 \uac00\ubb38\uc740 \uce74\uc288\ube44\uc544\uc778 \ud608\ud1b5\uc774\uc5c8\uc73c\uba70 \ube0c\ub85c\ud750\ube44\uce20 \ubb38\uc7a5\uc744 \uc0ac\uc6a9\ud558\ub294 \uadc0\uc871\uac00\uc600\ub2e4. \uc5d0\ub9ac\ud788\uc758 \uc774\ubaa8 \ud5e4\ub4dc\ube44\ud788 \ud3f0 \uc288\ud398\ub97c\ub9c1(1852\ub144 ~ 1925\ub144)\uc774 \uc911\uc7a5 \uac8c\uc624\ub974\ud06c \ud3f0 \ub9cc\uc288\ud0c0\uc778(1844\ub144 ~ 1913\ub144)\uacfc \uacb0\ud63c\ud588\uc73c\ub098 \uc2ac\ud558\uc5d0 \uc790\ub140\uac00 \uc5c6\uc5c8\uae30\uc5d0 \uc5d0\ub9ac\ud788\ub97c \uc785\uc591\ud568\uc73c\ub85c\uc368 \uc5d0\ub9ac\ud788\ub294 \ud3f0 \ub9cc\uc288\ud0c0\uc778\uc774 \ub418\uc5c8\ub2e4. \ub9cc\uc288\ud0c0\uc778 \uc9d1\uc548\uc740 \uadf8\uc804\uc5d0\ub3c4 \uc5d0\ub9ac\ud788\uc758 \uc0ac\ucd0c \ub9c8\ub974\ud0c0(\ud5ec\ub808\ub124 \ud5e4\ub4dc\ube44\ud788 \uc790\ub9e4\uc758 \uc8fd\uc740 \ud615\uc81c\uc758 \ub538)\ub97c \uc785\uc591\ud588\uc5c8\ub2e4.", "sentence_answer": "\uc5d0\ub9ac\ud788\uc758 \uc774\ubaa8 \ud5e4\ub4dc\ube44\ud788 \ud3f0 \uc288\ud398\ub97c\ub9c1(1852\ub144 ~ 1925\ub144)\uc774 \uc911\uc7a5 \uac8c\uc624\ub974\ud06c \ud3f0 \ub9cc\uc288\ud0c0\uc778(1844\ub144 ~ 1913\ub144)\uacfc \uacb0\ud63c\ud588\uc73c\ub098 \uc2ac\ud558\uc5d0 \uc790\ub140\uac00 \uc5c6\uc5c8\uae30\uc5d0 \uc5d0\ub9ac\ud788\ub97c \uc785\uc591\ud568\uc73c\ub85c\uc368 \uc5d0\ub9ac\ud788\ub294 \ud3f0 \ub9cc\uc288\ud0c0\uc778\uc774 \ub418\uc5c8\ub2e4.", "paragraph_id": "6586570-2-2", "paragraph_question": "question: \ub9cc\uc288\ud0c0\uc778\uc740 \ub204\uad6c\uc5d0\uac8c \uc785\uc591 \ub418\uc5c8\ub294\uac00?, context: \ub9cc\uc288\ud0c0\uc778\uc740 1887\ub144 \ubca0\ub97c\ub9b0\uc5d0\uc11c \ud504\ub85c\uc774\uc13c\uacc4 \uadc0\uc871\uc774\uc790 \ud3ec\ubcd1\ub300\uc7a5\uc778 \uc5d0\ub450\uc544\ub974\ud2b8 \ud3f0 \ub808\ube48\uc2a4\ud0a4(1829\ub144 ~ 1906\ub144)\uc640 \uadf8 \uc544\ub0b4 \ud5ec\ub808\ub124 \ud3f0 \uc288\ud398\ub97c\ub9c1(1847\ub144 ~ 1910\ub144) \uc0ac\uc774\uc758 \uc5f4\uc9f8 \uc544\ub4e4 \ud504\ub9ac\uce20 \uc5d0\ub9ac\ud788 \uac8c\uc624\ub974\ud06c \uc5d0\ub450\uc544\ub974\ud2b8 \ud3f0 \ub808\ube48\uc2a4\ud0a4\ub77c\ub294 \uc774\ub984\uc73c\ub85c \ud0dc\uc5b4\ub0ac\ub2e4. \ud3f0 \ub808\ube48\uc2a4\ud0a4 \uac00\ubb38\uc740 \uce74\uc288\ube44\uc544\uc778 \ud608\ud1b5\uc774\uc5c8\uc73c\uba70 \ube0c\ub85c\ud750\ube44\uce20 \ubb38\uc7a5\uc744 \uc0ac\uc6a9\ud558\ub294 \uadc0\uc871\uac00\uc600\ub2e4. \uc5d0\ub9ac\ud788\uc758 \uc774\ubaa8 \ud5e4\ub4dc\ube44\ud788 \ud3f0 \uc288\ud398\ub97c\ub9c1(1852\ub144 ~ 1925\ub144)\uc774 \uc911\uc7a5 \uac8c\uc624\ub974\ud06c \ud3f0 \ub9cc\uc288\ud0c0\uc778(1844\ub144 ~ 1913\ub144)\uacfc \uacb0\ud63c\ud588\uc73c\ub098 \uc2ac\ud558\uc5d0 \uc790\ub140\uac00 \uc5c6\uc5c8\uae30\uc5d0 \uc5d0\ub9ac\ud788\ub97c \uc785\uc591\ud568\uc73c\ub85c\uc368 \uc5d0\ub9ac\ud788\ub294 \ud3f0 \ub9cc\uc288\ud0c0\uc778\uc774 \ub418\uc5c8\ub2e4. \ub9cc\uc288\ud0c0\uc778 \uc9d1\uc548\uc740 \uadf8\uc804\uc5d0\ub3c4 \uc5d0\ub9ac\ud788\uc758 \uc0ac\ucd0c \ub9c8\ub974\ud0c0(\ud5ec\ub808\ub124 \ud5e4\ub4dc\ube44\ud788 \uc790\ub9e4\uc758 \uc8fd\uc740 \ud615\uc81c\uc758 \ub538)\ub97c \uc785\uc591\ud588\uc5c8\ub2e4."} {"question": "5\uc6d4 18\uc77c \uad11\uc8fc \uc2dc\ub0b4\uc5d0 \ud22c\uc785\ub3fc \uc6b4\ub3d9\uad8c \ub300\ud559\uc0dd\ubfd0\ub9cc \uc544\ub2c8\ub77c \uc2dc\uc704\uc5d0 \ucc38\uac00\ud558\uc9c0 \uc54a\uc740 \ubb34\uace0\ud55c \uc2dc\ubbfc\uae4c\uc9c0 \ud3ed\ud589\ud55c \uad70\uc778\ub4e4\uc758 \uc18c\uc18d\uc740?", "paragraph": "5\uc6d4 18\uc77c 16\uc2dc \uc774\ud6c4 \uad11\uc8fc \uc2dc\ub0b4\uc5d0 \ud22c\uc785\ub41c \uacf5\uc218\ubd80\ub300\uc6d0\uc774 \uc6b4\ub3d9\uad8c \ub300\ud559\uc0dd\ubfd0\ub9cc \uc544\ub2c8\ub77c \uc2dc\uc704\uc5d0 \ucc38\uc5ec\ud558\uc9c0 \uc54a\uc740 \ubb34\uace0\ud55c \uc2dc\ubbfc\uae4c\uc9c0 \ub2e5\uce58\ub294 \ub300\ub85c \uc0b4\uc0c1\u00b7\ud3ed\ud589\ud558\ub294 \uac83\uc744 \ubaa9\uaca9\ud55c \uad11\uc8fc\uc2dc\ubbfc\ub4e4\uc740 \ub450\ub824\uc6c0\uc744 \ub118\uc5b4 \ubd84\ub178\ub97c \ub290\uaf08\uace0, \uadf8 \uacb0\uacfc \uc911\uc7a5\ub144\uce35\ubfd0\ub9cc \uc544\ub2c8\ub77c 10\ub300 \uccad\uc18c\ub144\uae4c\uc9c0 \uac70\ub9ac\ub85c \ub098\uc11c \uc2dc\uc704\uc5d0 \ucc38\uc5ec\ud558\uba74\uc11c 5\u00b718 \uad11\uc8fc \ubbfc\uc8fc\ud654 \uc6b4\ub3d9\uc740 \uac77\uc7a1\uc744 \uc218 \uc5c6\uc774 \ubc88\uc84c\ub2e4. \uad11\uc8fc \uc2dc\ubbfc\ub4e4\uc758 \uaca9\ub82c\ud55c \uc800\ud56d\uc5d0 \ubd80\ub52a\ud78c \uacc4\uc5c4\uad70\uc740 5\uc6d4 21\uc77c 13\uc2dc\uacbd \uc804\ub0a8\ub300\ud559\uad50\uc640 \uc804\ub0a8\ub3c4\uccad \uc55e\uc5d0\uc11c \uc9d1\ub2e8 \ubc1c\ud3ec\ub97c \ud55c \ud6c4 \ucca0\uc218\ud588\ub2e4. \uc774 \ub0a0 \uc800\ub141 \uad11\uc8fc\uc2dc \uc678\uacfd\uc73c\ub85c \ucca0\uc218\ud55c \uacc4\uc5c4\uad70\uc740 \uad11\uc8fc \uc678\uacfd\ub3c4\ub85c \ubd09\uc1c4\uc791\uc804\uc744 \ud3bc\ucce4\uc73c\uba70, \uc774 \uacfc\uc815\uc5d0\uc11c \ucc28\ub7c9 \ud1b5\ud589\uc790\ub098 \uc9c0\uc5ed \uc8fc\ubbfc\ub4e4\uc758 \ud76c\uc0dd\uc774 \ubc1c\uc0dd\ud588\ub2e4. 5\uc6d4 27\uc77c 0\uc2dc\ub97c \uae30\ud574 \uacc4\uc5c4\uad70\uc740 \uc0c1\ubb34\ucda9\uc815\uc791\uc804\uc744 \uc2e4\uc2dc\ud574 \ubb34\ub825\uc73c\ub85c \uc804\ub0a8\ub3c4\uccad\uc744 \uc810\ub839\ud588\ub2e4.", "answer": "\uacf5\uc218\ubd80\ub300\uc6d0", "sentence": "5\uc6d4 18\uc77c 16\uc2dc \uc774\ud6c4 \uad11\uc8fc \uc2dc\ub0b4\uc5d0 \ud22c\uc785\ub41c \uacf5\uc218\ubd80\ub300\uc6d0 \uc774 \uc6b4\ub3d9\uad8c \ub300\ud559\uc0dd\ubfd0\ub9cc \uc544\ub2c8\ub77c \uc2dc\uc704\uc5d0 \ucc38\uc5ec\ud558\uc9c0 \uc54a\uc740 \ubb34\uace0\ud55c \uc2dc\ubbfc\uae4c\uc9c0 \ub2e5\uce58\ub294 \ub300\ub85c \uc0b4\uc0c1\u00b7\ud3ed\ud589\ud558\ub294 \uac83\uc744 \ubaa9\uaca9\ud55c \uad11\uc8fc\uc2dc\ubbfc\ub4e4\uc740 \ub450\ub824\uc6c0\uc744 \ub118\uc5b4 \ubd84\ub178\ub97c \ub290\uaf08\uace0, \uadf8 \uacb0\uacfc \uc911\uc7a5\ub144\uce35\ubfd0\ub9cc \uc544\ub2c8\ub77c 10\ub300 \uccad\uc18c\ub144\uae4c\uc9c0 \uac70\ub9ac\ub85c \ub098\uc11c \uc2dc\uc704\uc5d0 \ucc38\uc5ec\ud558\uba74\uc11c 5\u00b718 \uad11\uc8fc \ubbfc\uc8fc\ud654 \uc6b4\ub3d9\uc740 \uac77\uc7a1\uc744 \uc218 \uc5c6\uc774 \ubc88\uc84c\ub2e4.", "paragraph_sentence": " 5\uc6d4 18\uc77c 16\uc2dc \uc774\ud6c4 \uad11\uc8fc \uc2dc\ub0b4\uc5d0 \ud22c\uc785\ub41c \uacf5\uc218\ubd80\ub300\uc6d0 \uc774 \uc6b4\ub3d9\uad8c \ub300\ud559\uc0dd\ubfd0\ub9cc \uc544\ub2c8\ub77c \uc2dc\uc704\uc5d0 \ucc38\uc5ec\ud558\uc9c0 \uc54a\uc740 \ubb34\uace0\ud55c \uc2dc\ubbfc\uae4c\uc9c0 \ub2e5\uce58\ub294 \ub300\ub85c \uc0b4\uc0c1\u00b7\ud3ed\ud589\ud558\ub294 \uac83\uc744 \ubaa9\uaca9\ud55c \uad11\uc8fc\uc2dc\ubbfc\ub4e4\uc740 \ub450\ub824\uc6c0\uc744 \ub118\uc5b4 \ubd84\ub178\ub97c \ub290\uaf08\uace0, \uadf8 \uacb0\uacfc \uc911\uc7a5\ub144\uce35\ubfd0\ub9cc \uc544\ub2c8\ub77c 10\ub300 \uccad\uc18c\ub144\uae4c\uc9c0 \uac70\ub9ac\ub85c \ub098\uc11c \uc2dc\uc704\uc5d0 \ucc38\uc5ec\ud558\uba74\uc11c 5\u00b718 \uad11\uc8fc \ubbfc\uc8fc\ud654 \uc6b4\ub3d9\uc740 \uac77\uc7a1\uc744 \uc218 \uc5c6\uc774 \ubc88\uc84c\ub2e4. \uad11\uc8fc \uc2dc\ubbfc\ub4e4\uc758 \uaca9\ub82c\ud55c \uc800\ud56d\uc5d0 \ubd80\ub52a\ud78c \uacc4\uc5c4\uad70\uc740 5\uc6d4 21\uc77c 13\uc2dc\uacbd \uc804\ub0a8\ub300\ud559\uad50\uc640 \uc804\ub0a8\ub3c4\uccad \uc55e\uc5d0\uc11c \uc9d1\ub2e8 \ubc1c\ud3ec\ub97c \ud55c \ud6c4 \ucca0\uc218\ud588\ub2e4. \uc774 \ub0a0 \uc800\ub141 \uad11\uc8fc\uc2dc \uc678\uacfd\uc73c\ub85c \ucca0\uc218\ud55c \uacc4\uc5c4\uad70\uc740 \uad11\uc8fc \uc678\uacfd\ub3c4\ub85c \ubd09\uc1c4\uc791\uc804\uc744 \ud3bc\ucce4\uc73c\uba70, \uc774 \uacfc\uc815\uc5d0\uc11c \ucc28\ub7c9 \ud1b5\ud589\uc790\ub098 \uc9c0\uc5ed \uc8fc\ubbfc\ub4e4\uc758 \ud76c\uc0dd\uc774 \ubc1c\uc0dd\ud588\ub2e4. 5\uc6d4 27\uc77c 0\uc2dc\ub97c \uae30\ud574 \uacc4\uc5c4\uad70\uc740 \uc0c1\ubb34\ucda9\uc815\uc791\uc804\uc744 \uc2e4\uc2dc\ud574 \ubb34\ub825\uc73c\ub85c \uc804\ub0a8\ub3c4\uccad\uc744 \uc810\ub839\ud588\ub2e4.", "paragraph_answer": "5\uc6d4 18\uc77c 16\uc2dc \uc774\ud6c4 \uad11\uc8fc \uc2dc\ub0b4\uc5d0 \ud22c\uc785\ub41c \uacf5\uc218\ubd80\ub300\uc6d0 \uc774 \uc6b4\ub3d9\uad8c \ub300\ud559\uc0dd\ubfd0\ub9cc \uc544\ub2c8\ub77c \uc2dc\uc704\uc5d0 \ucc38\uc5ec\ud558\uc9c0 \uc54a\uc740 \ubb34\uace0\ud55c \uc2dc\ubbfc\uae4c\uc9c0 \ub2e5\uce58\ub294 \ub300\ub85c \uc0b4\uc0c1\u00b7\ud3ed\ud589\ud558\ub294 \uac83\uc744 \ubaa9\uaca9\ud55c \uad11\uc8fc\uc2dc\ubbfc\ub4e4\uc740 \ub450\ub824\uc6c0\uc744 \ub118\uc5b4 \ubd84\ub178\ub97c \ub290\uaf08\uace0, \uadf8 \uacb0\uacfc \uc911\uc7a5\ub144\uce35\ubfd0\ub9cc \uc544\ub2c8\ub77c 10\ub300 \uccad\uc18c\ub144\uae4c\uc9c0 \uac70\ub9ac\ub85c \ub098\uc11c \uc2dc\uc704\uc5d0 \ucc38\uc5ec\ud558\uba74\uc11c 5\u00b718 \uad11\uc8fc \ubbfc\uc8fc\ud654 \uc6b4\ub3d9\uc740 \uac77\uc7a1\uc744 \uc218 \uc5c6\uc774 \ubc88\uc84c\ub2e4. \uad11\uc8fc \uc2dc\ubbfc\ub4e4\uc758 \uaca9\ub82c\ud55c \uc800\ud56d\uc5d0 \ubd80\ub52a\ud78c \uacc4\uc5c4\uad70\uc740 5\uc6d4 21\uc77c 13\uc2dc\uacbd \uc804\ub0a8\ub300\ud559\uad50\uc640 \uc804\ub0a8\ub3c4\uccad \uc55e\uc5d0\uc11c \uc9d1\ub2e8 \ubc1c\ud3ec\ub97c \ud55c \ud6c4 \ucca0\uc218\ud588\ub2e4. \uc774 \ub0a0 \uc800\ub141 \uad11\uc8fc\uc2dc \uc678\uacfd\uc73c\ub85c \ucca0\uc218\ud55c \uacc4\uc5c4\uad70\uc740 \uad11\uc8fc \uc678\uacfd\ub3c4\ub85c \ubd09\uc1c4\uc791\uc804\uc744 \ud3bc\ucce4\uc73c\uba70, \uc774 \uacfc\uc815\uc5d0\uc11c \ucc28\ub7c9 \ud1b5\ud589\uc790\ub098 \uc9c0\uc5ed \uc8fc\ubbfc\ub4e4\uc758 \ud76c\uc0dd\uc774 \ubc1c\uc0dd\ud588\ub2e4. 5\uc6d4 27\uc77c 0\uc2dc\ub97c \uae30\ud574 \uacc4\uc5c4\uad70\uc740 \uc0c1\ubb34\ucda9\uc815\uc791\uc804\uc744 \uc2e4\uc2dc\ud574 \ubb34\ub825\uc73c\ub85c \uc804\ub0a8\ub3c4\uccad\uc744 \uc810\ub839\ud588\ub2e4.", "sentence_answer": "5\uc6d4 18\uc77c 16\uc2dc \uc774\ud6c4 \uad11\uc8fc \uc2dc\ub0b4\uc5d0 \ud22c\uc785\ub41c \uacf5\uc218\ubd80\ub300\uc6d0 \uc774 \uc6b4\ub3d9\uad8c \ub300\ud559\uc0dd\ubfd0\ub9cc \uc544\ub2c8\ub77c \uc2dc\uc704\uc5d0 \ucc38\uc5ec\ud558\uc9c0 \uc54a\uc740 \ubb34\uace0\ud55c \uc2dc\ubbfc\uae4c\uc9c0 \ub2e5\uce58\ub294 \ub300\ub85c \uc0b4\uc0c1\u00b7\ud3ed\ud589\ud558\ub294 \uac83\uc744 \ubaa9\uaca9\ud55c \uad11\uc8fc\uc2dc\ubbfc\ub4e4\uc740 \ub450\ub824\uc6c0\uc744 \ub118\uc5b4 \ubd84\ub178\ub97c \ub290\uaf08\uace0, \uadf8 \uacb0\uacfc \uc911\uc7a5\ub144\uce35\ubfd0\ub9cc \uc544\ub2c8\ub77c 10\ub300 \uccad\uc18c\ub144\uae4c\uc9c0 \uac70\ub9ac\ub85c \ub098\uc11c \uc2dc\uc704\uc5d0 \ucc38\uc5ec\ud558\uba74\uc11c 5\u00b718 \uad11\uc8fc \ubbfc\uc8fc\ud654 \uc6b4\ub3d9\uc740 \uac77\uc7a1\uc744 \uc218 \uc5c6\uc774 \ubc88\uc84c\ub2e4.", "paragraph_id": "6568325-1-1", "paragraph_question": "question: 5\uc6d4 18\uc77c \uad11\uc8fc \uc2dc\ub0b4\uc5d0 \ud22c\uc785\ub3fc \uc6b4\ub3d9\uad8c \ub300\ud559\uc0dd\ubfd0\ub9cc \uc544\ub2c8\ub77c \uc2dc\uc704\uc5d0 \ucc38\uac00\ud558\uc9c0 \uc54a\uc740 \ubb34\uace0\ud55c \uc2dc\ubbfc\uae4c\uc9c0 \ud3ed\ud589\ud55c \uad70\uc778\ub4e4\uc758 \uc18c\uc18d\uc740?, context: 5\uc6d4 18\uc77c 16\uc2dc \uc774\ud6c4 \uad11\uc8fc \uc2dc\ub0b4\uc5d0 \ud22c\uc785\ub41c \uacf5\uc218\ubd80\ub300\uc6d0\uc774 \uc6b4\ub3d9\uad8c \ub300\ud559\uc0dd\ubfd0\ub9cc \uc544\ub2c8\ub77c \uc2dc\uc704\uc5d0 \ucc38\uc5ec\ud558\uc9c0 \uc54a\uc740 \ubb34\uace0\ud55c \uc2dc\ubbfc\uae4c\uc9c0 \ub2e5\uce58\ub294 \ub300\ub85c \uc0b4\uc0c1\u00b7\ud3ed\ud589\ud558\ub294 \uac83\uc744 \ubaa9\uaca9\ud55c \uad11\uc8fc\uc2dc\ubbfc\ub4e4\uc740 \ub450\ub824\uc6c0\uc744 \ub118\uc5b4 \ubd84\ub178\ub97c \ub290\uaf08\uace0, \uadf8 \uacb0\uacfc \uc911\uc7a5\ub144\uce35\ubfd0\ub9cc \uc544\ub2c8\ub77c 10\ub300 \uccad\uc18c\ub144\uae4c\uc9c0 \uac70\ub9ac\ub85c \ub098\uc11c \uc2dc\uc704\uc5d0 \ucc38\uc5ec\ud558\uba74\uc11c 5\u00b718 \uad11\uc8fc \ubbfc\uc8fc\ud654 \uc6b4\ub3d9\uc740 \uac77\uc7a1\uc744 \uc218 \uc5c6\uc774 \ubc88\uc84c\ub2e4. \uad11\uc8fc \uc2dc\ubbfc\ub4e4\uc758 \uaca9\ub82c\ud55c \uc800\ud56d\uc5d0 \ubd80\ub52a\ud78c \uacc4\uc5c4\uad70\uc740 5\uc6d4 21\uc77c 13\uc2dc\uacbd \uc804\ub0a8\ub300\ud559\uad50\uc640 \uc804\ub0a8\ub3c4\uccad \uc55e\uc5d0\uc11c \uc9d1\ub2e8 \ubc1c\ud3ec\ub97c \ud55c \ud6c4 \ucca0\uc218\ud588\ub2e4. \uc774 \ub0a0 \uc800\ub141 \uad11\uc8fc\uc2dc \uc678\uacfd\uc73c\ub85c \ucca0\uc218\ud55c \uacc4\uc5c4\uad70\uc740 \uad11\uc8fc \uc678\uacfd\ub3c4\ub85c \ubd09\uc1c4\uc791\uc804\uc744 \ud3bc\ucce4\uc73c\uba70, \uc774 \uacfc\uc815\uc5d0\uc11c \ucc28\ub7c9 \ud1b5\ud589\uc790\ub098 \uc9c0\uc5ed \uc8fc\ubbfc\ub4e4\uc758 \ud76c\uc0dd\uc774 \ubc1c\uc0dd\ud588\ub2e4. 5\uc6d4 27\uc77c 0\uc2dc\ub97c \uae30\ud574 \uacc4\uc5c4\uad70\uc740 \uc0c1\ubb34\ucda9\uc815\uc791\uc804\uc744 \uc2e4\uc2dc\ud574 \ubb34\ub825\uc73c\ub85c \uc804\ub0a8\ub3c4\uccad\uc744 \uc810\ub839\ud588\ub2e4."} {"question": "\ub9ce\uc740 \ub524\ubbf8\ub4e4\uc774 \ubd88\uacbd\ud55c \ud589\uc704\ub97c \uc774\uc720\ub85c \ub2f9\ud55c \uc751\uc9d5 \uc911 \ud280\ub2c8\uc2a4\uc5d0\uc11c \uc77c\uc5b4\ub09c \ud574\ub294?", "paragraph": "\ub9ce\uc740 \ub524\ubbf8\ub4e4\uc774 \ubd88\uacbd\uc744 \ubc94\ud588\ub2e4\ub294 \uc774\uc720\ub85c \ubaa9\uc228\uc744 \uc783\uc5c8\ub2e4. \uc77c\ubd80 \ub524\ubbf8\ub4e4\uc774 \uc21c\uad50\ub97c \ud0dd\ud558\uae30\ub3c4 \ud588\uc9c0\ub9cc \ub300\ubd80\ubd84 \uc220\uc774 \ucde8\ud558\uac70\ub098 \uc815\uc2e0 \uc774\uc0c1\uc790\uc758 \uacbd\uc6b0\uac00 \ub9ce\uc558\ub2e4\uace0 \ud558\ub294\ub370 \uc815\uce58\uc801\uc778 \uc774\uc720 \ud639\uc740 \uc0ac\uc801\uc778 \uc6d0\ud55c \ub4f1\uc73c\ub85c \uc77c\ubc29\uc801\uc778 \uc0ac\ud615\uc744 \uc218\ud589\ud558\ub294 \uacbd\uc6b0\ub294 \ud754\uce58 \uc54a\uc558\ub2e4. \uc0ac\ud615\uac10\uc778 \ubaa8\ub3c5 \ud589\uc704\ub294 \ubb34\uc2ac\ub9bc\ub4e4\uc5d0\uac8c \uc544\ucca8\uc744 \ud558\uac70\ub098 \ub450\ub824\uc6c0\uc5d0 \ub5a8\uc5b4\uc57c \ud588\ub358 \uc774\uc720\uc600\ub2e4. \uc5d0\ub4dc\uc6cc\ub4dc \uc70c\ub9ac\uc5c4 \ub808\uc778\uc774 \uc774\uc9d1\ud2b8 \uc5ec\ud589\uae30\uc5d0\uc11c \uc37c\ub4ef\uc774 \ub2f9\uc2dc \uc720\ub300\uc778\ub4e4\uc740 \uacb0\ucf54 \ub9d0\uc744 \ud568\ubd80\ub85c \ub0b4\ubc49\uc9c0 \uc54a\uc558\uc73c\uba70 \ud22c\ub974\ud06c\uc871\uacfc \uc544\ub78d\uc778\uc744 \ud5a5\ud55c \ubaa8\uc695\uc801\uc778 \ub9d0\uc744 \uc785\uc5d0 \ub2f4\uc9c0\ub3c4 \uc54a\uc558\ub2e4. \uc774\ub294 \uc218\ub9ce\uc740 \uc0ac\ub78c\ub4e4\uc774 \uc624\ud574\ub97c \uc0ac\uac70\ub098, \uc2e4\uc218\ub85c \uad81\uc9c0\uc5d0 \ubab0\ub824 \uafb8\ub780\uc774\ub098 \uc608\uc5b8\uc790\ub97c \uc695\ub418\uac8c \ud588\ub2e4\ub294 \uc774\uc720\ub85c \uc0ac\ud615\ub2f9\ud588\uae30 \ub54c\ubb38\uc774\ub2e4. \ubd88\uacbd\ud55c \ud589\uc704\uc5d0 \ub300\ud55c \uc751\uc9d5\uc740 \ub3c4\ucc98\uc5d0\uc11c \uc77c\uc5b4\ub0ac\ub2e4 : \ud280\ub2c8\uc2a4(1876\ub144), \ud558\ub9c8\ub2e8(1876\ub144), \uc54c\ub808\ud3ec(1889\ub144), \ud14c\ud5e4\ub780(1895\ub144), \ubaa8\uc220(1911\ub144)", "answer": "1876\ub144", "sentence": "\ubd88\uacbd\ud55c \ud589\uc704\uc5d0 \ub300\ud55c \uc751\uc9d5\uc740 \ub3c4\ucc98\uc5d0\uc11c \uc77c\uc5b4\ub0ac\ub2e4 : \ud280\ub2c8\uc2a4( 1876\ub144 ),", "paragraph_sentence": "\ub9ce\uc740 \ub524\ubbf8\ub4e4\uc774 \ubd88\uacbd\uc744 \ubc94\ud588\ub2e4\ub294 \uc774\uc720\ub85c \ubaa9\uc228\uc744 \uc783\uc5c8\ub2e4. \uc77c\ubd80 \ub524\ubbf8\ub4e4\uc774 \uc21c\uad50\ub97c \ud0dd\ud558\uae30\ub3c4 \ud588\uc9c0\ub9cc \ub300\ubd80\ubd84 \uc220\uc774 \ucde8\ud558\uac70\ub098 \uc815\uc2e0 \uc774\uc0c1\uc790\uc758 \uacbd\uc6b0\uac00 \ub9ce\uc558\ub2e4\uace0 \ud558\ub294\ub370 \uc815\uce58\uc801\uc778 \uc774\uc720 \ud639\uc740 \uc0ac\uc801\uc778 \uc6d0\ud55c \ub4f1\uc73c\ub85c \uc77c\ubc29\uc801\uc778 \uc0ac\ud615\uc744 \uc218\ud589\ud558\ub294 \uacbd\uc6b0\ub294 \ud754\uce58 \uc54a\uc558\ub2e4. \uc0ac\ud615\uac10\uc778 \ubaa8\ub3c5 \ud589\uc704\ub294 \ubb34\uc2ac\ub9bc\ub4e4\uc5d0\uac8c \uc544\ucca8\uc744 \ud558\uac70\ub098 \ub450\ub824\uc6c0\uc5d0 \ub5a8\uc5b4\uc57c \ud588\ub358 \uc774\uc720\uc600\ub2e4. \uc5d0\ub4dc\uc6cc\ub4dc \uc70c\ub9ac\uc5c4 \ub808\uc778\uc774 \uc774\uc9d1\ud2b8 \uc5ec\ud589\uae30\uc5d0\uc11c \uc37c\ub4ef\uc774 \ub2f9\uc2dc \uc720\ub300\uc778\ub4e4\uc740 \uacb0\ucf54 \ub9d0\uc744 \ud568\ubd80\ub85c \ub0b4\ubc49\uc9c0 \uc54a\uc558\uc73c\uba70 \ud22c\ub974\ud06c\uc871\uacfc \uc544\ub78d\uc778\uc744 \ud5a5\ud55c \ubaa8\uc695\uc801\uc778 \ub9d0\uc744 \uc785\uc5d0 \ub2f4\uc9c0\ub3c4 \uc54a\uc558\ub2e4. \uc774\ub294 \uc218\ub9ce\uc740 \uc0ac\ub78c\ub4e4\uc774 \uc624\ud574\ub97c \uc0ac\uac70\ub098, \uc2e4\uc218\ub85c \uad81\uc9c0\uc5d0 \ubab0\ub824 \uafb8\ub780\uc774\ub098 \uc608\uc5b8\uc790\ub97c \uc695\ub418\uac8c \ud588\ub2e4\ub294 \uc774\uc720\ub85c \uc0ac\ud615\ub2f9\ud588\uae30 \ub54c\ubb38\uc774\ub2e4. \ubd88\uacbd\ud55c \ud589\uc704\uc5d0 \ub300\ud55c \uc751\uc9d5\uc740 \ub3c4\ucc98\uc5d0\uc11c \uc77c\uc5b4\ub0ac\ub2e4 : \ud280\ub2c8\uc2a4( 1876\ub144 ), \ud558\ub9c8\ub2e8(1876\ub144), \uc54c\ub808\ud3ec(1889\ub144), \ud14c\ud5e4\ub780(1895\ub144), \ubaa8\uc220(1911\ub144)", "paragraph_answer": "\ub9ce\uc740 \ub524\ubbf8\ub4e4\uc774 \ubd88\uacbd\uc744 \ubc94\ud588\ub2e4\ub294 \uc774\uc720\ub85c \ubaa9\uc228\uc744 \uc783\uc5c8\ub2e4. \uc77c\ubd80 \ub524\ubbf8\ub4e4\uc774 \uc21c\uad50\ub97c \ud0dd\ud558\uae30\ub3c4 \ud588\uc9c0\ub9cc \ub300\ubd80\ubd84 \uc220\uc774 \ucde8\ud558\uac70\ub098 \uc815\uc2e0 \uc774\uc0c1\uc790\uc758 \uacbd\uc6b0\uac00 \ub9ce\uc558\ub2e4\uace0 \ud558\ub294\ub370 \uc815\uce58\uc801\uc778 \uc774\uc720 \ud639\uc740 \uc0ac\uc801\uc778 \uc6d0\ud55c \ub4f1\uc73c\ub85c \uc77c\ubc29\uc801\uc778 \uc0ac\ud615\uc744 \uc218\ud589\ud558\ub294 \uacbd\uc6b0\ub294 \ud754\uce58 \uc54a\uc558\ub2e4. \uc0ac\ud615\uac10\uc778 \ubaa8\ub3c5 \ud589\uc704\ub294 \ubb34\uc2ac\ub9bc\ub4e4\uc5d0\uac8c \uc544\ucca8\uc744 \ud558\uac70\ub098 \ub450\ub824\uc6c0\uc5d0 \ub5a8\uc5b4\uc57c \ud588\ub358 \uc774\uc720\uc600\ub2e4. \uc5d0\ub4dc\uc6cc\ub4dc \uc70c\ub9ac\uc5c4 \ub808\uc778\uc774 \uc774\uc9d1\ud2b8 \uc5ec\ud589\uae30\uc5d0\uc11c \uc37c\ub4ef\uc774 \ub2f9\uc2dc \uc720\ub300\uc778\ub4e4\uc740 \uacb0\ucf54 \ub9d0\uc744 \ud568\ubd80\ub85c \ub0b4\ubc49\uc9c0 \uc54a\uc558\uc73c\uba70 \ud22c\ub974\ud06c\uc871\uacfc \uc544\ub78d\uc778\uc744 \ud5a5\ud55c \ubaa8\uc695\uc801\uc778 \ub9d0\uc744 \uc785\uc5d0 \ub2f4\uc9c0\ub3c4 \uc54a\uc558\ub2e4. \uc774\ub294 \uc218\ub9ce\uc740 \uc0ac\ub78c\ub4e4\uc774 \uc624\ud574\ub97c \uc0ac\uac70\ub098, \uc2e4\uc218\ub85c \uad81\uc9c0\uc5d0 \ubab0\ub824 \uafb8\ub780\uc774\ub098 \uc608\uc5b8\uc790\ub97c \uc695\ub418\uac8c \ud588\ub2e4\ub294 \uc774\uc720\ub85c \uc0ac\ud615\ub2f9\ud588\uae30 \ub54c\ubb38\uc774\ub2e4. \ubd88\uacbd\ud55c \ud589\uc704\uc5d0 \ub300\ud55c \uc751\uc9d5\uc740 \ub3c4\ucc98\uc5d0\uc11c \uc77c\uc5b4\ub0ac\ub2e4 : \ud280\ub2c8\uc2a4( 1876\ub144 ), \ud558\ub9c8\ub2e8(1876\ub144), \uc54c\ub808\ud3ec(1889\ub144), \ud14c\ud5e4\ub780(1895\ub144), \ubaa8\uc220(1911\ub144)", "sentence_answer": "\ubd88\uacbd\ud55c \ud589\uc704\uc5d0 \ub300\ud55c \uc751\uc9d5\uc740 \ub3c4\ucc98\uc5d0\uc11c \uc77c\uc5b4\ub0ac\ub2e4 : \ud280\ub2c8\uc2a4( 1876\ub144 ),", "paragraph_id": "6513164-9-2", "paragraph_question": "question: \ub9ce\uc740 \ub524\ubbf8\ub4e4\uc774 \ubd88\uacbd\ud55c \ud589\uc704\ub97c \uc774\uc720\ub85c \ub2f9\ud55c \uc751\uc9d5 \uc911 \ud280\ub2c8\uc2a4\uc5d0\uc11c \uc77c\uc5b4\ub09c \ud574\ub294?, context: \ub9ce\uc740 \ub524\ubbf8\ub4e4\uc774 \ubd88\uacbd\uc744 \ubc94\ud588\ub2e4\ub294 \uc774\uc720\ub85c \ubaa9\uc228\uc744 \uc783\uc5c8\ub2e4. \uc77c\ubd80 \ub524\ubbf8\ub4e4\uc774 \uc21c\uad50\ub97c \ud0dd\ud558\uae30\ub3c4 \ud588\uc9c0\ub9cc \ub300\ubd80\ubd84 \uc220\uc774 \ucde8\ud558\uac70\ub098 \uc815\uc2e0 \uc774\uc0c1\uc790\uc758 \uacbd\uc6b0\uac00 \ub9ce\uc558\ub2e4\uace0 \ud558\ub294\ub370 \uc815\uce58\uc801\uc778 \uc774\uc720 \ud639\uc740 \uc0ac\uc801\uc778 \uc6d0\ud55c \ub4f1\uc73c\ub85c \uc77c\ubc29\uc801\uc778 \uc0ac\ud615\uc744 \uc218\ud589\ud558\ub294 \uacbd\uc6b0\ub294 \ud754\uce58 \uc54a\uc558\ub2e4. \uc0ac\ud615\uac10\uc778 \ubaa8\ub3c5 \ud589\uc704\ub294 \ubb34\uc2ac\ub9bc\ub4e4\uc5d0\uac8c \uc544\ucca8\uc744 \ud558\uac70\ub098 \ub450\ub824\uc6c0\uc5d0 \ub5a8\uc5b4\uc57c \ud588\ub358 \uc774\uc720\uc600\ub2e4. \uc5d0\ub4dc\uc6cc\ub4dc \uc70c\ub9ac\uc5c4 \ub808\uc778\uc774 \uc774\uc9d1\ud2b8 \uc5ec\ud589\uae30\uc5d0\uc11c \uc37c\ub4ef\uc774 \ub2f9\uc2dc \uc720\ub300\uc778\ub4e4\uc740 \uacb0\ucf54 \ub9d0\uc744 \ud568\ubd80\ub85c \ub0b4\ubc49\uc9c0 \uc54a\uc558\uc73c\uba70 \ud22c\ub974\ud06c\uc871\uacfc \uc544\ub78d\uc778\uc744 \ud5a5\ud55c \ubaa8\uc695\uc801\uc778 \ub9d0\uc744 \uc785\uc5d0 \ub2f4\uc9c0\ub3c4 \uc54a\uc558\ub2e4. \uc774\ub294 \uc218\ub9ce\uc740 \uc0ac\ub78c\ub4e4\uc774 \uc624\ud574\ub97c \uc0ac\uac70\ub098, \uc2e4\uc218\ub85c \uad81\uc9c0\uc5d0 \ubab0\ub824 \uafb8\ub780\uc774\ub098 \uc608\uc5b8\uc790\ub97c \uc695\ub418\uac8c \ud588\ub2e4\ub294 \uc774\uc720\ub85c \uc0ac\ud615\ub2f9\ud588\uae30 \ub54c\ubb38\uc774\ub2e4. \ubd88\uacbd\ud55c \ud589\uc704\uc5d0 \ub300\ud55c \uc751\uc9d5\uc740 \ub3c4\ucc98\uc5d0\uc11c \uc77c\uc5b4\ub0ac\ub2e4 : \ud280\ub2c8\uc2a4(1876\ub144), \ud558\ub9c8\ub2e8(1876\ub144), \uc54c\ub808\ud3ec(1889\ub144), \ud14c\ud5e4\ub780(1895\ub144), \ubaa8\uc220(1911\ub144)"} {"question": "\ud5dd\uac00\ub9ac\uc758 \ubca0\ud2c0\ub80c \uac00\ubcf4\ub974\ub97c \uc9c0\uc6d0\ud574\uc8fc\ub358 \ub098\ub77c\ub294?", "paragraph": "\ud2b8\ub780\uc2e4\ubc14\ub2c8\uc544\uc758 \uacf5\uc791\uc774\uc790, \ud5dd\uac00\ub9ac \uac1c\uc2e0\uad50\ub3c4\uc758 \uc655\uc774\uc5c8\ub358 \ubca0\ud2c0\ub80c \uac00\ubcf4\ub974\ub294 \ud569\uc2a4\ubd80\ub974\ud06c \ud5dd\uac00\ub9ac\ub85c \uc9c4\uaca9\ud588\ub2e4. \uc774 \ub54c \uadf8\ub294 \uc624\uc2a4\ub9cc \uc81c\uad6d\uc758 \uc220\ud0c4 \uc624\uc2a4\ub9cc 2\uc138\uc758 \uc9c0\uc6d0\uc744 \ubc1b\uace0 \uc788\uc5c8\ub2e4. \ud398\ub974\ub514\ub09c\ud2b8 2\uc138\uc758 \uac00\ud1a8\ub9ad \uacf5\ud3ec \uc815\uce58\ub85c \ubca0\ud2c0\ub80c\uc740 \uc624\uc2a4\ub9cc 2\uc138\uc5d0\uac8c \ubcf4\ud638\ub97c \ud574\ub2ec\ub77c \ubd80\ud0c1\ud588\uace0, \uc624\uc2a4\ub9cc \uc81c\uad6d\uc740 \uc774\uc5d0 \ub530\ub77c \ud569\uc2a4\ubd80\ub974\ud06c\uc758 \ud1b5\uce58\uc5d0 \ubc18\uae30\ub97c \uc77c\uc73c\ud0a4\uace0 \uac1c\uc2e0\uad50\ub3c4\uc758 \uc655\uc73c\ub85c \ud504\ub9ac\ub4dc\ub9ac\ud788 5\uc138\ub97c \uc120\ucd9c\ud55c \uc774\ud6c4 \ubcf4\ud5e4\ubbf8\uc544 \ub0b4 \uc5ec\ub7ec \uad6d\uac00\ub4e4\uc774 \ud798\uc744 \ubc1c\ud718\ud560 \uc218 \uc788\ub294 \uc0c1\ud669\uc744 \ub9cc\ub4e4 \uc218 \uc788\ub294 \ubcf4\ud5e4\ubbf8\uc544\uc758 \uc720\uc77c\ud55c \ub3d9\ub9f9\uad6d\uc774 \ub418\uc5c8\ub2e4. \ud558\uc778\ub9ac\ud788 \ube44\ud130\uac00 \uc774\uc2a4\ud0c4\ubd88\uc744 1620\ub144 1\uc6d4 \ubc29\ubb38\ud588\uace0, \uba54\uba54\ud2b8 \uc544\uac00\uac00 1620\ub144 7\uc6d4 \ud504\ub77c\ud558\ub97c \ubc29\ubb38\ud588\ub2e4. \uc624\uc2a4\ub9cc \uc81c\uad6d \uae30\ubcd1\uc740 \ud504\ub9ac\ub4dc\ub9ac\ud788\uc5d0\uac8c 6\ub9cc \uba85\uc758 \uae30\ubcd1\uc744 \uc8fc\uc5c8\uace0, 40\ub9cc \uba85\uc758 \uad70\ub300\uac00 \ud3f4\ub780\ub4dc\ub97c \uce68\uacf5\ud560 \uacc4\ud68d\uc744 \uc138\uc6e0\ub2e4. \uc774\ub294 \uc220\ud0c4\uc5d0\uac8c \ubc14\uce58\ub294 \uc5f0\uac04 \uc870\uacf5\uc5d0 \ub300\ud55c \ub2f5\ub840\uc774\uae30\ub3c4 \ud588\ub2e4. \uc774 \ud611\uc0c1\uc740 1620\ub144\ubd80\ud130 1621\ub144\uae4c\uc9c0 \ubc1c\ubc1c\ud55c \ud3f4\ub780\ub4dc-\uc624\uc2a4\ub9cc \uc804\uc7c1\uc744 \ucd09\ubc1c\uc2dc\ucf30\ub2e4. \uc624\uc2a4\ub9cc \uc81c\uad6d\uad70\uc740 1620\ub144 9\uc6d4\ubd80\ud130 10\uc6d4\uae4c\uc9c0 30\ub144 \uc804\uc7c1 \ub2f9\uc2dc \ud569\uc2a4\ubd80\ub974\ud06c \uac00\ubb38\uc744 \uc9c0\uc6d0\ud558\uace0 \uc788\ub358 \ud3f4\ub780\ub4dc-\ub9ac\ud22c\uc544\ub2c8\uc544 \uc5f0\ubc29\uc744 \uccb4\ucd08\ub77c \uc804\ud22c\uc5d0\uc11c \ud328\ubc30\uc2dc\ucf30\ub2e4. \uadf8\ub7ec\ub098 1620\ub144 11\uc6d4 \ubc31\uc0b0 \uc804\ud22c\uc5d0\uc11c \ubcf4\ud5e4\ubbf8\uc544\uad70\uc774 \ud328\ubc30\ud560 \ub54c\ub294 \uc624\uc2a4\ub9cc \uc81c\uad6d\uc740 \uac1c\uc785\ud560 \uc218 \uc5c6\uc5c8\ub2e4. 1621\ub144 \ud3f4\ub780\ub4dc\uad70\uc774 \uc7ac\uc815\ube44\ud574 \ucd08\ud2f4 \uc804\ud22c\uc5d0\uc11c \uc624\uc2a4\ub9cc \uc81c\uad6d\uad70\uc744 \ud328\ubc30\uc2dc\ucf30\uace0 \uc591\uad6d\uc758 \uc804\uc7c1\uc740 \uc6d0\uc0c1\ubcf5\uadc0\ub418\uc5c8\ub2e4.", "answer": "\uc624\uc2a4\ub9cc \uc81c\uad6d", "sentence": "\uc774 \ub54c \uadf8\ub294 \uc624\uc2a4\ub9cc \uc81c\uad6d \uc758 \uc220\ud0c4 \uc624\uc2a4\ub9cc 2\uc138\uc758 \uc9c0\uc6d0\uc744 \ubc1b\uace0 \uc788\uc5c8\ub2e4.", "paragraph_sentence": "\ud2b8\ub780\uc2e4\ubc14\ub2c8\uc544\uc758 \uacf5\uc791\uc774\uc790, \ud5dd\uac00\ub9ac \uac1c\uc2e0\uad50\ub3c4\uc758 \uc655\uc774\uc5c8\ub358 \ubca0\ud2c0\ub80c \uac00\ubcf4\ub974\ub294 \ud569\uc2a4\ubd80\ub974\ud06c \ud5dd\uac00\ub9ac\ub85c \uc9c4\uaca9\ud588\ub2e4. \uc774 \ub54c \uadf8\ub294 \uc624\uc2a4\ub9cc \uc81c\uad6d \uc758 \uc220\ud0c4 \uc624\uc2a4\ub9cc 2\uc138\uc758 \uc9c0\uc6d0\uc744 \ubc1b\uace0 \uc788\uc5c8\ub2e4. \ud398\ub974\ub514\ub09c\ud2b8 2\uc138\uc758 \uac00\ud1a8\ub9ad \uacf5\ud3ec \uc815\uce58\ub85c \ubca0\ud2c0\ub80c\uc740 \uc624\uc2a4\ub9cc 2\uc138\uc5d0\uac8c \ubcf4\ud638\ub97c \ud574\ub2ec\ub77c \ubd80\ud0c1\ud588\uace0, \uc624\uc2a4\ub9cc \uc81c\uad6d\uc740 \uc774\uc5d0 \ub530\ub77c \ud569\uc2a4\ubd80\ub974\ud06c\uc758 \ud1b5\uce58\uc5d0 \ubc18\uae30\ub97c \uc77c\uc73c\ud0a4\uace0 \uac1c\uc2e0\uad50\ub3c4\uc758 \uc655\uc73c\ub85c \ud504\ub9ac\ub4dc\ub9ac\ud788 5\uc138\ub97c \uc120\ucd9c\ud55c \uc774\ud6c4 \ubcf4\ud5e4\ubbf8\uc544 \ub0b4 \uc5ec\ub7ec \uad6d\uac00\ub4e4\uc774 \ud798\uc744 \ubc1c\ud718\ud560 \uc218 \uc788\ub294 \uc0c1\ud669\uc744 \ub9cc\ub4e4 \uc218 \uc788\ub294 \ubcf4\ud5e4\ubbf8\uc544\uc758 \uc720\uc77c\ud55c \ub3d9\ub9f9\uad6d\uc774 \ub418\uc5c8\ub2e4. \ud558\uc778\ub9ac\ud788 \ube44\ud130\uac00 \uc774\uc2a4\ud0c4\ubd88\uc744 1620\ub144 1\uc6d4 \ubc29\ubb38\ud588\uace0, \uba54\uba54\ud2b8 \uc544\uac00\uac00 1620\ub144 7\uc6d4 \ud504\ub77c\ud558\ub97c \ubc29\ubb38\ud588\ub2e4. \uc624\uc2a4\ub9cc \uc81c\uad6d \uae30\ubcd1\uc740 \ud504\ub9ac\ub4dc\ub9ac\ud788\uc5d0\uac8c 6\ub9cc \uba85\uc758 \uae30\ubcd1\uc744 \uc8fc\uc5c8\uace0, 40\ub9cc \uba85\uc758 \uad70\ub300\uac00 \ud3f4\ub780\ub4dc\ub97c \uce68\uacf5\ud560 \uacc4\ud68d\uc744 \uc138\uc6e0\ub2e4. \uc774\ub294 \uc220\ud0c4\uc5d0\uac8c \ubc14\uce58\ub294 \uc5f0\uac04 \uc870\uacf5\uc5d0 \ub300\ud55c \ub2f5\ub840\uc774\uae30\ub3c4 \ud588\ub2e4. \uc774 \ud611\uc0c1\uc740 1620\ub144\ubd80\ud130 1621\ub144\uae4c\uc9c0 \ubc1c\ubc1c\ud55c \ud3f4\ub780\ub4dc-\uc624\uc2a4\ub9cc \uc804\uc7c1\uc744 \ucd09\ubc1c\uc2dc\ucf30\ub2e4. \uc624\uc2a4\ub9cc \uc81c\uad6d\uad70\uc740 1620\ub144 9\uc6d4\ubd80\ud130 10\uc6d4\uae4c\uc9c0 30\ub144 \uc804\uc7c1 \ub2f9\uc2dc \ud569\uc2a4\ubd80\ub974\ud06c \uac00\ubb38\uc744 \uc9c0\uc6d0\ud558\uace0 \uc788\ub358 \ud3f4\ub780\ub4dc-\ub9ac\ud22c\uc544\ub2c8\uc544 \uc5f0\ubc29\uc744 \uccb4\ucd08\ub77c \uc804\ud22c\uc5d0\uc11c \ud328\ubc30\uc2dc\ucf30\ub2e4. \uadf8\ub7ec\ub098 1620\ub144 11\uc6d4 \ubc31\uc0b0 \uc804\ud22c\uc5d0\uc11c \ubcf4\ud5e4\ubbf8\uc544\uad70\uc774 \ud328\ubc30\ud560 \ub54c\ub294 \uc624\uc2a4\ub9cc \uc81c\uad6d\uc740 \uac1c\uc785\ud560 \uc218 \uc5c6\uc5c8\ub2e4. 1621\ub144 \ud3f4\ub780\ub4dc\uad70\uc774 \uc7ac\uc815\ube44\ud574 \ucd08\ud2f4 \uc804\ud22c\uc5d0\uc11c \uc624\uc2a4\ub9cc \uc81c\uad6d\uad70\uc744 \ud328\ubc30\uc2dc\ucf30\uace0 \uc591\uad6d\uc758 \uc804\uc7c1\uc740 \uc6d0\uc0c1\ubcf5\uadc0\ub418\uc5c8\ub2e4.", "paragraph_answer": "\ud2b8\ub780\uc2e4\ubc14\ub2c8\uc544\uc758 \uacf5\uc791\uc774\uc790, \ud5dd\uac00\ub9ac \uac1c\uc2e0\uad50\ub3c4\uc758 \uc655\uc774\uc5c8\ub358 \ubca0\ud2c0\ub80c \uac00\ubcf4\ub974\ub294 \ud569\uc2a4\ubd80\ub974\ud06c \ud5dd\uac00\ub9ac\ub85c \uc9c4\uaca9\ud588\ub2e4. \uc774 \ub54c \uadf8\ub294 \uc624\uc2a4\ub9cc \uc81c\uad6d \uc758 \uc220\ud0c4 \uc624\uc2a4\ub9cc 2\uc138\uc758 \uc9c0\uc6d0\uc744 \ubc1b\uace0 \uc788\uc5c8\ub2e4. \ud398\ub974\ub514\ub09c\ud2b8 2\uc138\uc758 \uac00\ud1a8\ub9ad \uacf5\ud3ec \uc815\uce58\ub85c \ubca0\ud2c0\ub80c\uc740 \uc624\uc2a4\ub9cc 2\uc138\uc5d0\uac8c \ubcf4\ud638\ub97c \ud574\ub2ec\ub77c \ubd80\ud0c1\ud588\uace0, \uc624\uc2a4\ub9cc \uc81c\uad6d\uc740 \uc774\uc5d0 \ub530\ub77c \ud569\uc2a4\ubd80\ub974\ud06c\uc758 \ud1b5\uce58\uc5d0 \ubc18\uae30\ub97c \uc77c\uc73c\ud0a4\uace0 \uac1c\uc2e0\uad50\ub3c4\uc758 \uc655\uc73c\ub85c \ud504\ub9ac\ub4dc\ub9ac\ud788 5\uc138\ub97c \uc120\ucd9c\ud55c \uc774\ud6c4 \ubcf4\ud5e4\ubbf8\uc544 \ub0b4 \uc5ec\ub7ec \uad6d\uac00\ub4e4\uc774 \ud798\uc744 \ubc1c\ud718\ud560 \uc218 \uc788\ub294 \uc0c1\ud669\uc744 \ub9cc\ub4e4 \uc218 \uc788\ub294 \ubcf4\ud5e4\ubbf8\uc544\uc758 \uc720\uc77c\ud55c \ub3d9\ub9f9\uad6d\uc774 \ub418\uc5c8\ub2e4. \ud558\uc778\ub9ac\ud788 \ube44\ud130\uac00 \uc774\uc2a4\ud0c4\ubd88\uc744 1620\ub144 1\uc6d4 \ubc29\ubb38\ud588\uace0, \uba54\uba54\ud2b8 \uc544\uac00\uac00 1620\ub144 7\uc6d4 \ud504\ub77c\ud558\ub97c \ubc29\ubb38\ud588\ub2e4. \uc624\uc2a4\ub9cc \uc81c\uad6d \uae30\ubcd1\uc740 \ud504\ub9ac\ub4dc\ub9ac\ud788\uc5d0\uac8c 6\ub9cc \uba85\uc758 \uae30\ubcd1\uc744 \uc8fc\uc5c8\uace0, 40\ub9cc \uba85\uc758 \uad70\ub300\uac00 \ud3f4\ub780\ub4dc\ub97c \uce68\uacf5\ud560 \uacc4\ud68d\uc744 \uc138\uc6e0\ub2e4. \uc774\ub294 \uc220\ud0c4\uc5d0\uac8c \ubc14\uce58\ub294 \uc5f0\uac04 \uc870\uacf5\uc5d0 \ub300\ud55c \ub2f5\ub840\uc774\uae30\ub3c4 \ud588\ub2e4. \uc774 \ud611\uc0c1\uc740 1620\ub144\ubd80\ud130 1621\ub144\uae4c\uc9c0 \ubc1c\ubc1c\ud55c \ud3f4\ub780\ub4dc-\uc624\uc2a4\ub9cc \uc804\uc7c1\uc744 \ucd09\ubc1c\uc2dc\ucf30\ub2e4. \uc624\uc2a4\ub9cc \uc81c\uad6d\uad70\uc740 1620\ub144 9\uc6d4\ubd80\ud130 10\uc6d4\uae4c\uc9c0 30\ub144 \uc804\uc7c1 \ub2f9\uc2dc \ud569\uc2a4\ubd80\ub974\ud06c \uac00\ubb38\uc744 \uc9c0\uc6d0\ud558\uace0 \uc788\ub358 \ud3f4\ub780\ub4dc-\ub9ac\ud22c\uc544\ub2c8\uc544 \uc5f0\ubc29\uc744 \uccb4\ucd08\ub77c \uc804\ud22c\uc5d0\uc11c \ud328\ubc30\uc2dc\ucf30\ub2e4. \uadf8\ub7ec\ub098 1620\ub144 11\uc6d4 \ubc31\uc0b0 \uc804\ud22c\uc5d0\uc11c \ubcf4\ud5e4\ubbf8\uc544\uad70\uc774 \ud328\ubc30\ud560 \ub54c\ub294 \uc624\uc2a4\ub9cc \uc81c\uad6d\uc740 \uac1c\uc785\ud560 \uc218 \uc5c6\uc5c8\ub2e4. 1621\ub144 \ud3f4\ub780\ub4dc\uad70\uc774 \uc7ac\uc815\ube44\ud574 \ucd08\ud2f4 \uc804\ud22c\uc5d0\uc11c \uc624\uc2a4\ub9cc \uc81c\uad6d\uad70\uc744 \ud328\ubc30\uc2dc\ucf30\uace0 \uc591\uad6d\uc758 \uc804\uc7c1\uc740 \uc6d0\uc0c1\ubcf5\uadc0\ub418\uc5c8\ub2e4.", "sentence_answer": "\uc774 \ub54c \uadf8\ub294 \uc624\uc2a4\ub9cc \uc81c\uad6d \uc758 \uc220\ud0c4 \uc624\uc2a4\ub9cc 2\uc138\uc758 \uc9c0\uc6d0\uc744 \ubc1b\uace0 \uc788\uc5c8\ub2e4.", "paragraph_id": "5782587-13-0", "paragraph_question": "question: \ud5dd\uac00\ub9ac\uc758 \ubca0\ud2c0\ub80c \uac00\ubcf4\ub974\ub97c \uc9c0\uc6d0\ud574\uc8fc\ub358 \ub098\ub77c\ub294?, context: \ud2b8\ub780\uc2e4\ubc14\ub2c8\uc544\uc758 \uacf5\uc791\uc774\uc790, \ud5dd\uac00\ub9ac \uac1c\uc2e0\uad50\ub3c4\uc758 \uc655\uc774\uc5c8\ub358 \ubca0\ud2c0\ub80c \uac00\ubcf4\ub974\ub294 \ud569\uc2a4\ubd80\ub974\ud06c \ud5dd\uac00\ub9ac\ub85c \uc9c4\uaca9\ud588\ub2e4. \uc774 \ub54c \uadf8\ub294 \uc624\uc2a4\ub9cc \uc81c\uad6d\uc758 \uc220\ud0c4 \uc624\uc2a4\ub9cc 2\uc138\uc758 \uc9c0\uc6d0\uc744 \ubc1b\uace0 \uc788\uc5c8\ub2e4. \ud398\ub974\ub514\ub09c\ud2b8 2\uc138\uc758 \uac00\ud1a8\ub9ad \uacf5\ud3ec \uc815\uce58\ub85c \ubca0\ud2c0\ub80c\uc740 \uc624\uc2a4\ub9cc 2\uc138\uc5d0\uac8c \ubcf4\ud638\ub97c \ud574\ub2ec\ub77c \ubd80\ud0c1\ud588\uace0, \uc624\uc2a4\ub9cc \uc81c\uad6d\uc740 \uc774\uc5d0 \ub530\ub77c \ud569\uc2a4\ubd80\ub974\ud06c\uc758 \ud1b5\uce58\uc5d0 \ubc18\uae30\ub97c \uc77c\uc73c\ud0a4\uace0 \uac1c\uc2e0\uad50\ub3c4\uc758 \uc655\uc73c\ub85c \ud504\ub9ac\ub4dc\ub9ac\ud788 5\uc138\ub97c \uc120\ucd9c\ud55c \uc774\ud6c4 \ubcf4\ud5e4\ubbf8\uc544 \ub0b4 \uc5ec\ub7ec \uad6d\uac00\ub4e4\uc774 \ud798\uc744 \ubc1c\ud718\ud560 \uc218 \uc788\ub294 \uc0c1\ud669\uc744 \ub9cc\ub4e4 \uc218 \uc788\ub294 \ubcf4\ud5e4\ubbf8\uc544\uc758 \uc720\uc77c\ud55c \ub3d9\ub9f9\uad6d\uc774 \ub418\uc5c8\ub2e4. \ud558\uc778\ub9ac\ud788 \ube44\ud130\uac00 \uc774\uc2a4\ud0c4\ubd88\uc744 1620\ub144 1\uc6d4 \ubc29\ubb38\ud588\uace0, \uba54\uba54\ud2b8 \uc544\uac00\uac00 1620\ub144 7\uc6d4 \ud504\ub77c\ud558\ub97c \ubc29\ubb38\ud588\ub2e4. \uc624\uc2a4\ub9cc \uc81c\uad6d \uae30\ubcd1\uc740 \ud504\ub9ac\ub4dc\ub9ac\ud788\uc5d0\uac8c 6\ub9cc \uba85\uc758 \uae30\ubcd1\uc744 \uc8fc\uc5c8\uace0, 40\ub9cc \uba85\uc758 \uad70\ub300\uac00 \ud3f4\ub780\ub4dc\ub97c \uce68\uacf5\ud560 \uacc4\ud68d\uc744 \uc138\uc6e0\ub2e4. \uc774\ub294 \uc220\ud0c4\uc5d0\uac8c \ubc14\uce58\ub294 \uc5f0\uac04 \uc870\uacf5\uc5d0 \ub300\ud55c \ub2f5\ub840\uc774\uae30\ub3c4 \ud588\ub2e4. \uc774 \ud611\uc0c1\uc740 1620\ub144\ubd80\ud130 1621\ub144\uae4c\uc9c0 \ubc1c\ubc1c\ud55c \ud3f4\ub780\ub4dc-\uc624\uc2a4\ub9cc \uc804\uc7c1\uc744 \ucd09\ubc1c\uc2dc\ucf30\ub2e4. \uc624\uc2a4\ub9cc \uc81c\uad6d\uad70\uc740 1620\ub144 9\uc6d4\ubd80\ud130 10\uc6d4\uae4c\uc9c0 30\ub144 \uc804\uc7c1 \ub2f9\uc2dc \ud569\uc2a4\ubd80\ub974\ud06c \uac00\ubb38\uc744 \uc9c0\uc6d0\ud558\uace0 \uc788\ub358 \ud3f4\ub780\ub4dc-\ub9ac\ud22c\uc544\ub2c8\uc544 \uc5f0\ubc29\uc744 \uccb4\ucd08\ub77c \uc804\ud22c\uc5d0\uc11c \ud328\ubc30\uc2dc\ucf30\ub2e4. \uadf8\ub7ec\ub098 1620\ub144 11\uc6d4 \ubc31\uc0b0 \uc804\ud22c\uc5d0\uc11c \ubcf4\ud5e4\ubbf8\uc544\uad70\uc774 \ud328\ubc30\ud560 \ub54c\ub294 \uc624\uc2a4\ub9cc \uc81c\uad6d\uc740 \uac1c\uc785\ud560 \uc218 \uc5c6\uc5c8\ub2e4. 1621\ub144 \ud3f4\ub780\ub4dc\uad70\uc774 \uc7ac\uc815\ube44\ud574 \ucd08\ud2f4 \uc804\ud22c\uc5d0\uc11c \uc624\uc2a4\ub9cc \uc81c\uad6d\uad70\uc744 \ud328\ubc30\uc2dc\ucf30\uace0 \uc591\uad6d\uc758 \uc804\uc7c1\uc740 \uc6d0\uc0c1\ubcf5\uadc0\ub418\uc5c8\ub2e4."} {"question": "\uad6d\ubbfc\uc758 \uc5b5\uc6b8\ud55c \uc8fd\uc74c\uc744 \ud648\uc1fc\ud551\uc774\ub77c\ub294 \uc774\ub984\uc744 \ub0b4\uac78\uace0 \ub0b4\ubcf4\ub0c8\ub2e4\uace0 \uc8fc\uc7a5\ud55c \uad6d\ubbfc\uc758\ub2f9 \ucd5c\uace0\uc704\uc6d0\uc740?", "paragraph": "\uad6d\ubbfc\uc758\ub2f9 \uc7a5\uc9c4\uc601 \ucd5c\uace0\uc704\uc6d0\ub3c4 \ucd5c\uace0\uc704\uc6d0\ud68c\uc758\uc5d0\uc11c \"\uad6d\ubbfc\uc758 \uc5b5\uc6b8\ud55c \uc8fd\uc74c\uc744 \ud648\uc1fc\ud551\uc774\ub77c\ub294 \uc774\ub984\uc744 \ub0b4\uac78\uace0 \ub0b4\ubcf4\ub0c8\ub2e4\"\uace0 \uc9c0\uc801\ud558\uba70, \"\ubb38\uc7ac\uc778 \uc815\ubd80\uc758 \uc1fc\ud1b5\uc774 \ub3c4\ub97c \ub118\uc5c8\ub2e4\"\uace0 \ube44\ud310\ud558\uc600\ub2e4. \uc790\uc720\ud55c\uad6d\ub2f9 \uc2e0\ubcf4\ub77c \uc6d0\ub0b4\ub300\ubcc0\uc778\uc740 \ub17c\ud3c9\uc744 \ud1b5\ud574 \"\uc7ac\ub09c \ucc38\uc0ac \ud76c\uc0dd\uc790\ub97c \uc774\uc6a9\ud55c \ub300\ud1b5\ub839\uc758 '\uc1fc\ud1b5 \ud64d\ubcf4'\uac00 \uac00\uad00\uc774\ub2e4\"\ub77c\uba70, \"\uad6d\ubbfc \uc138\uae08\uc73c\ub85c \uc6b4\uc601\ub418\ub294 \ubc29\uc1a1\uc774 \ub300\ud1b5\ub839 \uce6d\uc1a1 \ub9e4\uccb4\ub85c \uc804\ub77d\ud55c \ub9cc\ud07c \ubb38\uc7ac\uc778 \ub300\ud1b5\ub839\uc774 \uace0\uac1c \uc219\uc5ec \uc0ac\uacfc\ud574\uc57c \ud55c\ub2e4\"\uace0 \ubc1d\ud614\ub2e4. \ub9cc\ud654\uac00 \uc724\uc11c\uc778\uc740 \ucc38\uc0ac\ub97c \ud648\uc1fc\ud551 \ubc29\uc1a1 \ud615\uc2dd\uc73c\ub85c \ubcf4\ub3c4\ud55c \uad6d\ubbfc\ubc29\uc1a1\uc5d0 \ub300\ud574 \"\uc774 \uc545\ub9c8\ub4e4\uc544\"\ub77c\uba70 \ubd84\uac1c\ud588\ub2e4. \ud55c\ud3b8 \ubcf4\ub3c4\uac00 \ub17c\ub780\uc774 \ub418\uc790 \uad6d\ubbfc\ubc29\uc1a1\uc774 \uc9e7\uc740 \uc0ac\uacfc \ubc29\uc1a1\uc744 \ud588\uc9c0\ub9cc \ubb34\uc131\uc758\ud558\ub2e4\ub294 \ube44\ud310\uc744 \ubc1b\uc558\uc73c\uba70, \uacb0\uad6d '\uc774\ub2c8\ud2b9\ubcc4\uc804'\uc744 \ud3d0\uc9c0\ud558\uace0 \ucc45\uc784\uc790\ub97c \uc9d5\uacc4\ud55c\ub2e4 \ubc1d\ud614\ub2e4.", "answer": "\uc7a5\uc9c4\uc601", "sentence": "\uad6d\ubbfc\uc758\ub2f9 \uc7a5\uc9c4\uc601 \ucd5c\uace0\uc704\uc6d0\ub3c4 \ucd5c\uace0\uc704\uc6d0\ud68c\uc758\uc5d0\uc11c \"\uad6d\ubbfc\uc758 \uc5b5\uc6b8\ud55c \uc8fd\uc74c\uc744 \ud648\uc1fc\ud551\uc774\ub77c\ub294 \uc774\ub984\uc744 \ub0b4\uac78\uace0 \ub0b4\ubcf4\ub0c8\ub2e4\"\uace0 \uc9c0\uc801\ud558\uba70, \"\ubb38\uc7ac\uc778 \uc815\ubd80\uc758 \uc1fc\ud1b5\uc774 \ub3c4\ub97c \ub118\uc5c8\ub2e4\"\uace0 \ube44\ud310\ud558\uc600\ub2e4.", "paragraph_sentence": " \uad6d\ubbfc\uc758\ub2f9 \uc7a5\uc9c4\uc601 \ucd5c\uace0\uc704\uc6d0\ub3c4 \ucd5c\uace0\uc704\uc6d0\ud68c\uc758\uc5d0\uc11c \"\uad6d\ubbfc\uc758 \uc5b5\uc6b8\ud55c \uc8fd\uc74c\uc744 \ud648\uc1fc\ud551\uc774\ub77c\ub294 \uc774\ub984\uc744 \ub0b4\uac78\uace0 \ub0b4\ubcf4\ub0c8\ub2e4\"\uace0 \uc9c0\uc801\ud558\uba70, \"\ubb38\uc7ac\uc778 \uc815\ubd80\uc758 \uc1fc\ud1b5\uc774 \ub3c4\ub97c \ub118\uc5c8\ub2e4\"\uace0 \ube44\ud310\ud558\uc600\ub2e4. \uc790\uc720\ud55c\uad6d\ub2f9 \uc2e0\ubcf4\ub77c \uc6d0\ub0b4\ub300\ubcc0\uc778\uc740 \ub17c\ud3c9\uc744 \ud1b5\ud574 \"\uc7ac\ub09c \ucc38\uc0ac \ud76c\uc0dd\uc790\ub97c \uc774\uc6a9\ud55c \ub300\ud1b5\ub839\uc758 '\uc1fc\ud1b5 \ud64d\ubcf4'\uac00 \uac00\uad00\uc774\ub2e4\"\ub77c\uba70, \"\uad6d\ubbfc \uc138\uae08\uc73c\ub85c \uc6b4\uc601\ub418\ub294 \ubc29\uc1a1\uc774 \ub300\ud1b5\ub839 \uce6d\uc1a1 \ub9e4\uccb4\ub85c \uc804\ub77d\ud55c \ub9cc\ud07c \ubb38\uc7ac\uc778 \ub300\ud1b5\ub839\uc774 \uace0\uac1c \uc219\uc5ec \uc0ac\uacfc\ud574\uc57c \ud55c\ub2e4\"\uace0 \ubc1d\ud614\ub2e4. \ub9cc\ud654\uac00 \uc724\uc11c\uc778\uc740 \ucc38\uc0ac\ub97c \ud648\uc1fc\ud551 \ubc29\uc1a1 \ud615\uc2dd\uc73c\ub85c \ubcf4\ub3c4\ud55c \uad6d\ubbfc\ubc29\uc1a1\uc5d0 \ub300\ud574 \"\uc774 \uc545\ub9c8\ub4e4\uc544\"\ub77c\uba70 \ubd84\uac1c\ud588\ub2e4. \ud55c\ud3b8 \ubcf4\ub3c4\uac00 \ub17c\ub780\uc774 \ub418\uc790 \uad6d\ubbfc\ubc29\uc1a1\uc774 \uc9e7\uc740 \uc0ac\uacfc \ubc29\uc1a1\uc744 \ud588\uc9c0\ub9cc \ubb34\uc131\uc758\ud558\ub2e4\ub294 \ube44\ud310\uc744 \ubc1b\uc558\uc73c\uba70, \uacb0\uad6d '\uc774\ub2c8\ud2b9\ubcc4\uc804'\uc744 \ud3d0\uc9c0\ud558\uace0 \ucc45\uc784\uc790\ub97c \uc9d5\uacc4\ud55c\ub2e4 \ubc1d\ud614\ub2e4.", "paragraph_answer": "\uad6d\ubbfc\uc758\ub2f9 \uc7a5\uc9c4\uc601 \ucd5c\uace0\uc704\uc6d0\ub3c4 \ucd5c\uace0\uc704\uc6d0\ud68c\uc758\uc5d0\uc11c \"\uad6d\ubbfc\uc758 \uc5b5\uc6b8\ud55c \uc8fd\uc74c\uc744 \ud648\uc1fc\ud551\uc774\ub77c\ub294 \uc774\ub984\uc744 \ub0b4\uac78\uace0 \ub0b4\ubcf4\ub0c8\ub2e4\"\uace0 \uc9c0\uc801\ud558\uba70, \"\ubb38\uc7ac\uc778 \uc815\ubd80\uc758 \uc1fc\ud1b5\uc774 \ub3c4\ub97c \ub118\uc5c8\ub2e4\"\uace0 \ube44\ud310\ud558\uc600\ub2e4. \uc790\uc720\ud55c\uad6d\ub2f9 \uc2e0\ubcf4\ub77c \uc6d0\ub0b4\ub300\ubcc0\uc778\uc740 \ub17c\ud3c9\uc744 \ud1b5\ud574 \"\uc7ac\ub09c \ucc38\uc0ac \ud76c\uc0dd\uc790\ub97c \uc774\uc6a9\ud55c \ub300\ud1b5\ub839\uc758 '\uc1fc\ud1b5 \ud64d\ubcf4'\uac00 \uac00\uad00\uc774\ub2e4\"\ub77c\uba70, \"\uad6d\ubbfc \uc138\uae08\uc73c\ub85c \uc6b4\uc601\ub418\ub294 \ubc29\uc1a1\uc774 \ub300\ud1b5\ub839 \uce6d\uc1a1 \ub9e4\uccb4\ub85c \uc804\ub77d\ud55c \ub9cc\ud07c \ubb38\uc7ac\uc778 \ub300\ud1b5\ub839\uc774 \uace0\uac1c \uc219\uc5ec \uc0ac\uacfc\ud574\uc57c \ud55c\ub2e4\"\uace0 \ubc1d\ud614\ub2e4. \ub9cc\ud654\uac00 \uc724\uc11c\uc778\uc740 \ucc38\uc0ac\ub97c \ud648\uc1fc\ud551 \ubc29\uc1a1 \ud615\uc2dd\uc73c\ub85c \ubcf4\ub3c4\ud55c \uad6d\ubbfc\ubc29\uc1a1\uc5d0 \ub300\ud574 \"\uc774 \uc545\ub9c8\ub4e4\uc544\"\ub77c\uba70 \ubd84\uac1c\ud588\ub2e4. \ud55c\ud3b8 \ubcf4\ub3c4\uac00 \ub17c\ub780\uc774 \ub418\uc790 \uad6d\ubbfc\ubc29\uc1a1\uc774 \uc9e7\uc740 \uc0ac\uacfc \ubc29\uc1a1\uc744 \ud588\uc9c0\ub9cc \ubb34\uc131\uc758\ud558\ub2e4\ub294 \ube44\ud310\uc744 \ubc1b\uc558\uc73c\uba70, \uacb0\uad6d '\uc774\ub2c8\ud2b9\ubcc4\uc804'\uc744 \ud3d0\uc9c0\ud558\uace0 \ucc45\uc784\uc790\ub97c \uc9d5\uacc4\ud55c\ub2e4 \ubc1d\ud614\ub2e4.", "sentence_answer": "\uad6d\ubbfc\uc758\ub2f9 \uc7a5\uc9c4\uc601 \ucd5c\uace0\uc704\uc6d0\ub3c4 \ucd5c\uace0\uc704\uc6d0\ud68c\uc758\uc5d0\uc11c \"\uad6d\ubbfc\uc758 \uc5b5\uc6b8\ud55c \uc8fd\uc74c\uc744 \ud648\uc1fc\ud551\uc774\ub77c\ub294 \uc774\ub984\uc744 \ub0b4\uac78\uace0 \ub0b4\ubcf4\ub0c8\ub2e4\"\uace0 \uc9c0\uc801\ud558\uba70, \"\ubb38\uc7ac\uc778 \uc815\ubd80\uc758 \uc1fc\ud1b5\uc774 \ub3c4\ub97c \ub118\uc5c8\ub2e4\"\uace0 \ube44\ud310\ud558\uc600\ub2e4.", "paragraph_id": "6551082-3-0", "paragraph_question": "question: \uad6d\ubbfc\uc758 \uc5b5\uc6b8\ud55c \uc8fd\uc74c\uc744 \ud648\uc1fc\ud551\uc774\ub77c\ub294 \uc774\ub984\uc744 \ub0b4\uac78\uace0 \ub0b4\ubcf4\ub0c8\ub2e4\uace0 \uc8fc\uc7a5\ud55c \uad6d\ubbfc\uc758\ub2f9 \ucd5c\uace0\uc704\uc6d0\uc740?, context: \uad6d\ubbfc\uc758\ub2f9 \uc7a5\uc9c4\uc601 \ucd5c\uace0\uc704\uc6d0\ub3c4 \ucd5c\uace0\uc704\uc6d0\ud68c\uc758\uc5d0\uc11c \"\uad6d\ubbfc\uc758 \uc5b5\uc6b8\ud55c \uc8fd\uc74c\uc744 \ud648\uc1fc\ud551\uc774\ub77c\ub294 \uc774\ub984\uc744 \ub0b4\uac78\uace0 \ub0b4\ubcf4\ub0c8\ub2e4\"\uace0 \uc9c0\uc801\ud558\uba70, \"\ubb38\uc7ac\uc778 \uc815\ubd80\uc758 \uc1fc\ud1b5\uc774 \ub3c4\ub97c \ub118\uc5c8\ub2e4\"\uace0 \ube44\ud310\ud558\uc600\ub2e4. \uc790\uc720\ud55c\uad6d\ub2f9 \uc2e0\ubcf4\ub77c \uc6d0\ub0b4\ub300\ubcc0\uc778\uc740 \ub17c\ud3c9\uc744 \ud1b5\ud574 \"\uc7ac\ub09c \ucc38\uc0ac \ud76c\uc0dd\uc790\ub97c \uc774\uc6a9\ud55c \ub300\ud1b5\ub839\uc758 '\uc1fc\ud1b5 \ud64d\ubcf4'\uac00 \uac00\uad00\uc774\ub2e4\"\ub77c\uba70, \"\uad6d\ubbfc \uc138\uae08\uc73c\ub85c \uc6b4\uc601\ub418\ub294 \ubc29\uc1a1\uc774 \ub300\ud1b5\ub839 \uce6d\uc1a1 \ub9e4\uccb4\ub85c \uc804\ub77d\ud55c \ub9cc\ud07c \ubb38\uc7ac\uc778 \ub300\ud1b5\ub839\uc774 \uace0\uac1c \uc219\uc5ec \uc0ac\uacfc\ud574\uc57c \ud55c\ub2e4\"\uace0 \ubc1d\ud614\ub2e4. \ub9cc\ud654\uac00 \uc724\uc11c\uc778\uc740 \ucc38\uc0ac\ub97c \ud648\uc1fc\ud551 \ubc29\uc1a1 \ud615\uc2dd\uc73c\ub85c \ubcf4\ub3c4\ud55c \uad6d\ubbfc\ubc29\uc1a1\uc5d0 \ub300\ud574 \"\uc774 \uc545\ub9c8\ub4e4\uc544\"\ub77c\uba70 \ubd84\uac1c\ud588\ub2e4. \ud55c\ud3b8 \ubcf4\ub3c4\uac00 \ub17c\ub780\uc774 \ub418\uc790 \uad6d\ubbfc\ubc29\uc1a1\uc774 \uc9e7\uc740 \uc0ac\uacfc \ubc29\uc1a1\uc744 \ud588\uc9c0\ub9cc \ubb34\uc131\uc758\ud558\ub2e4\ub294 \ube44\ud310\uc744 \ubc1b\uc558\uc73c\uba70, \uacb0\uad6d '\uc774\ub2c8\ud2b9\ubcc4\uc804'\uc744 \ud3d0\uc9c0\ud558\uace0 \ucc45\uc784\uc790\ub97c \uc9d5\uacc4\ud55c\ub2e4 \ubc1d\ud614\ub2e4."} {"question": "\uc194\ub85c\ub300\ucca9 \ub2f9\uc2dc \uacbd\ucc30 \uce21 \ucd94\uc0b0 \uc5ec\uc758\ub3c4 \uacf5\uc6d0\uc5d0 \uc628 \ub0a8 \ub140\uc758 \uc218\ub294?", "paragraph": "\ub300\ud55c\ubbfc\uad6d \ucd5c\ucd08 \ud2b9\ubcc4\ud55c \ub0a0 \"\uc194\ub85c\ub300\ucca9\" \ud589\uc0ac\ub294 \uc11c\uc6b8\ud2b9\ubcc4\uc2dc\uc758 \uacbd\uc6b0 \uc2dc\uc98c\uc81c \ud06c\ub9ac\uc2a4\ub9c8\uc2a4 \uae30\uac04 \uc911 2012\ub144 12\uc6d4 24\uc77c\uc6d4\uc694\uc77c \ud3c9\uc77c/\uc784\uc2dc \uc790\uc728 1\uc77c \ud734\uc77c\uc81c \ud06c\ub9ac\uc2a4\ub9c8\uc2a4 \uc774\ube0c*\uc131\ud0c4\uc804\uc57c\uc81c \ub370\uc774* \uc624\ud6c43~5\uc2dc(\uc57d2\uc2dc\uac04 \ub3d9\uc548)\uc5d0 \ub0a8\uc790\ub4e4\uc740 \ud558\uc580\uc0c9, \uc5ec\uc790\ub4e4\uc740 \ubd89\uc740\uc0c9, \ucee4\ud50c\uc740 \ucd08\ub85d\uc0c9 \uacc4\uc5f4\uc758 \uc637\uc744 \uc785\uace0 \uc11c\uc6b8\uc5ec\uc758\ub3c4\uacf5\uc6d0\uc73c\ub85c \ubaa8\uc774\ub294 \uac83\uc73c\ub85c \uc2dc\uc791\ud558\uc600\ub2e4. \uacf5\uc6d0\uc5d0 \ubaa8\uc778 \ub0a8\ub140\ub294 \uc2e0\ud638 \uc591\ud3b8\uc5d0\uc11c \ub300\uae30\ud558\ub2e4\uac00 \uc9c4\ud589\uc790\uac00 \uc2e0\ud638\ub97c \ubcf4\ub0b4\uba74 \ub9c8\uc74c\uc5d0 \ub4dc\ub294 \uc774\uc131(\uc5ec\ub294 \ub0a8\uc73c\ub85c, \ub0a8\uc740 \uc5ec\ub85c)\uc744 \ud5a5\ud574 \ub2ec\ub824\uac00 \uc190\uc744 \uc7a1\ub294 \ud615\uc2dd\uc73c\ub85c \uc9c4\ud589\ub418\uc5c8\ub2e4. \uc2e4\uc81c \uc774 \ud589\uc0ac\ub97c \ud1b5\ud574 \uba87 \uc30d\uc758 \ucee4\ud50c\uc774 \ud0c4\uc0dd\ud558\uae30\ub3c4 \ud558\uc600\ub2e4. \ud558\uc9c0\ub9cc \uc548\uc804\uc0c1\uc758 \uc774\uc720 \ub4f1\uc73c\ub85c \uc0c1\ub2f9\uc218\uc758 \uc5ec\uc131 \ucc38\uac00 \uc2e0\uccad\uc790\ub4e4\uc774 \uc2e4\uc81c \ud589\uc0ac\uc5d0 \ucc38\uac00\ud558\uc9c0 \uc54a\uc74c\uc73c\ub85c \uc778\ud574, \ud589\uc0ac \ucc38\uac00\uc790 \uc57d 3,000\uba85 \uc911 \ub300\ubd80\ubd84\uc774 \ub0a8\uc131\uc73c\ub85c \ucc44\uc6cc\uc9c0\ub294 \ud604\uc0c1\uc774 \ubc1c\uc0dd\ud588\ub2e4. \ud55c\ud3b8 \uacbd\ucc30 \uce21 \ucd94\uc0b0\uc5d0 \ub530\ub974\uba74 \uc774\ub0a0 \uc11c\uc6b8 \uc5ec\uc758\ub3c4 \uacf5\uc6d0\uc5d0\uc11c \uc9c4\ud589\ub41c \ud589\uc0ac\uc5d0\ub294 \ub0a8\uc131 700\uba85\uacfc \uc5ec\uc131 300\uba85 \ub4f1 \uc57d 1\ucc9c\uc5ec \uba85\uc774 \ucc38\uac00\ud55c \uac83\uc73c\ub85c \uc804\ud574\uc84c\ub2e4. \u201c\ub0a8\uc790\uc640 \uc5ec\uc790\uac00 \ub300\uce58\ud55c \uac83\uc774 \uc544\ub2c8\ub77c \uacbd\ucc30\uacfc \ube44\ub458\uae30\uac00 \ub300\uce58\ud558\ub294 \ubd84\uc704\uae30\u201d\ub77c\ub294 \uae00\uc774 \uc62c\ub77c\uc624\uba70 \ub2f9\uc2dc \uadf9\uba85\ud55c \ub0a8\ub140 \ube44\uc728\uc744 \uc124\uba85\ud558\uae30\ub3c4 \ud588\ub2e4. \uc9dd\uc744 \ucc3e\uc9c0 \ubabb\ud55c \ub300\ubd80\ubd84\uc758 \ucc38\uac00\uc790\ub4e4\uc774 \uadfc\ucc98 \uc74c\uc2dd\uc810, \uc220\uc9d1 \ub4f1\uc73c\ub85c \ud769\uc5b4\uc84c\ub2e4. \uc774 \ud589\uc0ac\uc5d0 \ucc38\uac00\ud588\ub358 \uac1c\uadf8\ub9e8 \ubc15\ud718\uc21c\uc740 \"\uc194\ub85c\ub300\ucca9\uc774 \uc220\ub85c\ub300\ucca9\uc73c\ub85c \uc774\uc5b4\uc9c0\uaca0\ub124\uc694\"\ub77c\ub294 \uae00\uc744 \ub0a8\uae30\uae30\ub3c4 \ud588\ub2e4. \uc218\ub3c4\uad8c\uacfc \uc9c0\ubc29\uc758 \uacbd\uc6b0 \uc5ec\uc131 \ucc38\uac00\uc790\uac00 \ub354 \uc801\uc5b4 \ud589\uc0ac \uc790\uccb4\uac00 \uc81c\ub300\ub85c \uc9c4\ud589\ub418\uc9c0 \ubabb\ud55c \uacbd\uc6b0\ub3c4 \uc788\uc5c8\ub2e4. \ucc9c\uc548 \uc194\ub85c\ub300\ucca9\uc758 \uacbd\uc6b0 \uc5ec\uc131 \ucc38\uac00\uc790\uac00 10\uc5ec \uba85 \uc815\ub3c4\uc5d0 \ubd88\uacfc\ud574 \ud589\uc0ac\ub97c \uce58\ub8e8\uc9c0 \ubabb\ud588\uc9c0\ub9cc, \uac1c\uc778\uc801\uc73c\ub85c \ubc88\ud638\ub97c \uc8fc\uace0\ubc1b\uc544 1\ud638 \ucee4\ud50c\uc774 \ud798\uacb9\uac8c \ud0c4\uc0dd\ub418\uae30\ub3c4 \ud588\ub2e4. \ud55c\ud3b8 \ub300\ubd80\ubd84\uc758 \uc5ec\uc131 \uc5f0\uc608\uc778\ub4e4\uc774 \uc548\uc804\uc0c1\uc758 \uc774\uc720\ub85c \ud589\uc0ac \ubd88\ucc38\uc744 \uc120\uc5b8\ud55c \uac00\uc6b4\ub370, \uc2e0\uc778 \uac78\uadf8\ub8f9 \ube44\ud0a4\ub2c8\uc758 \uba64\ubc84\uc778 \ud574\uc774\uac00 \uc194\ub85c\ub300\ucca9\uc5d0 \ucc38\uac00\ud558\uc5ec \uc2e4\uc2dc\uac04 \uac80\uc0c9 1\uc704\ub97c \ucc28\uc9c0\ud558\uae30\ub3c4 \ud558\uc600\ub2e4.", "answer": "\ub0a8\uc131 700\uba85\uacfc \uc5ec\uc131 300\uba85", "sentence": "\ud55c\ud3b8 \uacbd\ucc30 \uce21 \ucd94\uc0b0\uc5d0 \ub530\ub974\uba74 \uc774\ub0a0 \uc11c\uc6b8 \uc5ec\uc758\ub3c4 \uacf5\uc6d0\uc5d0\uc11c \uc9c4\ud589\ub41c \ud589\uc0ac\uc5d0\ub294 \ub0a8\uc131 700\uba85\uacfc \uc5ec\uc131 300\uba85 \ub4f1 \uc57d 1\ucc9c\uc5ec \uba85\uc774 \ucc38\uac00\ud55c \uac83\uc73c\ub85c \uc804\ud574\uc84c\ub2e4.", "paragraph_sentence": "\ub300\ud55c\ubbfc\uad6d \ucd5c\ucd08 \ud2b9\ubcc4\ud55c \ub0a0 \"\uc194\ub85c\ub300\ucca9\" \ud589\uc0ac\ub294 \uc11c\uc6b8\ud2b9\ubcc4\uc2dc\uc758 \uacbd\uc6b0 \uc2dc\uc98c\uc81c \ud06c\ub9ac\uc2a4\ub9c8\uc2a4 \uae30\uac04 \uc911 2012\ub144 12\uc6d4 24\uc77c\uc6d4\uc694\uc77c \ud3c9\uc77c/\uc784\uc2dc \uc790\uc728 1\uc77c \ud734\uc77c\uc81c \ud06c\ub9ac\uc2a4\ub9c8\uc2a4 \uc774\ube0c*\uc131\ud0c4\uc804\uc57c\uc81c \ub370\uc774* \uc624\ud6c43~5\uc2dc(\uc57d2\uc2dc\uac04 \ub3d9\uc548)\uc5d0 \ub0a8\uc790\ub4e4\uc740 \ud558\uc580\uc0c9, \uc5ec\uc790\ub4e4\uc740 \ubd89\uc740\uc0c9, \ucee4\ud50c\uc740 \ucd08\ub85d\uc0c9 \uacc4\uc5f4\uc758 \uc637\uc744 \uc785\uace0 \uc11c\uc6b8\uc5ec\uc758\ub3c4\uacf5\uc6d0\uc73c\ub85c \ubaa8\uc774\ub294 \uac83\uc73c\ub85c \uc2dc\uc791\ud558\uc600\ub2e4. \uacf5\uc6d0\uc5d0 \ubaa8\uc778 \ub0a8\ub140\ub294 \uc2e0\ud638 \uc591\ud3b8\uc5d0\uc11c \ub300\uae30\ud558\ub2e4\uac00 \uc9c4\ud589\uc790\uac00 \uc2e0\ud638\ub97c \ubcf4\ub0b4\uba74 \ub9c8\uc74c\uc5d0 \ub4dc\ub294 \uc774\uc131(\uc5ec\ub294 \ub0a8\uc73c\ub85c, \ub0a8\uc740 \uc5ec\ub85c)\uc744 \ud5a5\ud574 \ub2ec\ub824\uac00 \uc190\uc744 \uc7a1\ub294 \ud615\uc2dd\uc73c\ub85c \uc9c4\ud589\ub418\uc5c8\ub2e4. \uc2e4\uc81c \uc774 \ud589\uc0ac\ub97c \ud1b5\ud574 \uba87 \uc30d\uc758 \ucee4\ud50c\uc774 \ud0c4\uc0dd\ud558\uae30\ub3c4 \ud558\uc600\ub2e4. \ud558\uc9c0\ub9cc \uc548\uc804\uc0c1\uc758 \uc774\uc720 \ub4f1\uc73c\ub85c \uc0c1\ub2f9\uc218\uc758 \uc5ec\uc131 \ucc38\uac00 \uc2e0\uccad\uc790\ub4e4\uc774 \uc2e4\uc81c \ud589\uc0ac\uc5d0 \ucc38\uac00\ud558\uc9c0 \uc54a\uc74c\uc73c\ub85c \uc778\ud574, \ud589\uc0ac \ucc38\uac00\uc790 \uc57d 3,000\uba85 \uc911 \ub300\ubd80\ubd84\uc774 \ub0a8\uc131\uc73c\ub85c \ucc44\uc6cc\uc9c0\ub294 \ud604\uc0c1\uc774 \ubc1c\uc0dd\ud588\ub2e4. \ud55c\ud3b8 \uacbd\ucc30 \uce21 \ucd94\uc0b0\uc5d0 \ub530\ub974\uba74 \uc774\ub0a0 \uc11c\uc6b8 \uc5ec\uc758\ub3c4 \uacf5\uc6d0\uc5d0\uc11c \uc9c4\ud589\ub41c \ud589\uc0ac\uc5d0\ub294 \ub0a8\uc131 700\uba85\uacfc \uc5ec\uc131 300\uba85 \ub4f1 \uc57d 1\ucc9c\uc5ec \uba85\uc774 \ucc38\uac00\ud55c \uac83\uc73c\ub85c \uc804\ud574\uc84c\ub2e4. \u201c\ub0a8\uc790\uc640 \uc5ec\uc790\uac00 \ub300\uce58\ud55c \uac83\uc774 \uc544\ub2c8\ub77c \uacbd\ucc30\uacfc \ube44\ub458\uae30\uac00 \ub300\uce58\ud558\ub294 \ubd84\uc704\uae30\u201d\ub77c\ub294 \uae00\uc774 \uc62c\ub77c\uc624\uba70 \ub2f9\uc2dc \uadf9\uba85\ud55c \ub0a8\ub140 \ube44\uc728\uc744 \uc124\uba85\ud558\uae30\ub3c4 \ud588\ub2e4. \uc9dd\uc744 \ucc3e\uc9c0 \ubabb\ud55c \ub300\ubd80\ubd84\uc758 \ucc38\uac00\uc790\ub4e4\uc774 \uadfc\ucc98 \uc74c\uc2dd\uc810, \uc220\uc9d1 \ub4f1\uc73c\ub85c \ud769\uc5b4\uc84c\ub2e4. \uc774 \ud589\uc0ac\uc5d0 \ucc38\uac00\ud588\ub358 \uac1c\uadf8\ub9e8 \ubc15\ud718\uc21c\uc740 \"\uc194\ub85c\ub300\ucca9\uc774 \uc220\ub85c\ub300\ucca9\uc73c\ub85c \uc774\uc5b4\uc9c0\uaca0\ub124\uc694\"\ub77c\ub294 \uae00\uc744 \ub0a8\uae30\uae30\ub3c4 \ud588\ub2e4. \uc218\ub3c4\uad8c\uacfc \uc9c0\ubc29\uc758 \uacbd\uc6b0 \uc5ec\uc131 \ucc38\uac00\uc790\uac00 \ub354 \uc801\uc5b4 \ud589\uc0ac \uc790\uccb4\uac00 \uc81c\ub300\ub85c \uc9c4\ud589\ub418\uc9c0 \ubabb\ud55c \uacbd\uc6b0\ub3c4 \uc788\uc5c8\ub2e4. \ucc9c\uc548 \uc194\ub85c\ub300\ucca9\uc758 \uacbd\uc6b0 \uc5ec\uc131 \ucc38\uac00\uc790\uac00 10\uc5ec \uba85 \uc815\ub3c4\uc5d0 \ubd88\uacfc\ud574 \ud589\uc0ac\ub97c \uce58\ub8e8\uc9c0 \ubabb\ud588\uc9c0\ub9cc, \uac1c\uc778\uc801\uc73c\ub85c \ubc88\ud638\ub97c \uc8fc\uace0\ubc1b\uc544 1\ud638 \ucee4\ud50c\uc774 \ud798\uacb9\uac8c \ud0c4\uc0dd\ub418\uae30\ub3c4 \ud588\ub2e4. \ud55c\ud3b8 \ub300\ubd80\ubd84\uc758 \uc5ec\uc131 \uc5f0\uc608\uc778\ub4e4\uc774 \uc548\uc804\uc0c1\uc758 \uc774\uc720\ub85c \ud589\uc0ac \ubd88\ucc38\uc744 \uc120\uc5b8\ud55c \uac00\uc6b4\ub370, \uc2e0\uc778 \uac78\uadf8\ub8f9 \ube44\ud0a4\ub2c8\uc758 \uba64\ubc84\uc778 \ud574\uc774\uac00 \uc194\ub85c\ub300\ucca9\uc5d0 \ucc38\uac00\ud558\uc5ec \uc2e4\uc2dc\uac04 \uac80\uc0c9 1\uc704\ub97c \ucc28\uc9c0\ud558\uae30\ub3c4 \ud558\uc600\ub2e4.", "paragraph_answer": "\ub300\ud55c\ubbfc\uad6d \ucd5c\ucd08 \ud2b9\ubcc4\ud55c \ub0a0 \"\uc194\ub85c\ub300\ucca9\" \ud589\uc0ac\ub294 \uc11c\uc6b8\ud2b9\ubcc4\uc2dc\uc758 \uacbd\uc6b0 \uc2dc\uc98c\uc81c \ud06c\ub9ac\uc2a4\ub9c8\uc2a4 \uae30\uac04 \uc911 2012\ub144 12\uc6d4 24\uc77c\uc6d4\uc694\uc77c \ud3c9\uc77c/\uc784\uc2dc \uc790\uc728 1\uc77c \ud734\uc77c\uc81c \ud06c\ub9ac\uc2a4\ub9c8\uc2a4 \uc774\ube0c*\uc131\ud0c4\uc804\uc57c\uc81c \ub370\uc774* \uc624\ud6c43~5\uc2dc(\uc57d2\uc2dc\uac04 \ub3d9\uc548)\uc5d0 \ub0a8\uc790\ub4e4\uc740 \ud558\uc580\uc0c9, \uc5ec\uc790\ub4e4\uc740 \ubd89\uc740\uc0c9, \ucee4\ud50c\uc740 \ucd08\ub85d\uc0c9 \uacc4\uc5f4\uc758 \uc637\uc744 \uc785\uace0 \uc11c\uc6b8\uc5ec\uc758\ub3c4\uacf5\uc6d0\uc73c\ub85c \ubaa8\uc774\ub294 \uac83\uc73c\ub85c \uc2dc\uc791\ud558\uc600\ub2e4. \uacf5\uc6d0\uc5d0 \ubaa8\uc778 \ub0a8\ub140\ub294 \uc2e0\ud638 \uc591\ud3b8\uc5d0\uc11c \ub300\uae30\ud558\ub2e4\uac00 \uc9c4\ud589\uc790\uac00 \uc2e0\ud638\ub97c \ubcf4\ub0b4\uba74 \ub9c8\uc74c\uc5d0 \ub4dc\ub294 \uc774\uc131(\uc5ec\ub294 \ub0a8\uc73c\ub85c, \ub0a8\uc740 \uc5ec\ub85c)\uc744 \ud5a5\ud574 \ub2ec\ub824\uac00 \uc190\uc744 \uc7a1\ub294 \ud615\uc2dd\uc73c\ub85c \uc9c4\ud589\ub418\uc5c8\ub2e4. \uc2e4\uc81c \uc774 \ud589\uc0ac\ub97c \ud1b5\ud574 \uba87 \uc30d\uc758 \ucee4\ud50c\uc774 \ud0c4\uc0dd\ud558\uae30\ub3c4 \ud558\uc600\ub2e4. \ud558\uc9c0\ub9cc \uc548\uc804\uc0c1\uc758 \uc774\uc720 \ub4f1\uc73c\ub85c \uc0c1\ub2f9\uc218\uc758 \uc5ec\uc131 \ucc38\uac00 \uc2e0\uccad\uc790\ub4e4\uc774 \uc2e4\uc81c \ud589\uc0ac\uc5d0 \ucc38\uac00\ud558\uc9c0 \uc54a\uc74c\uc73c\ub85c \uc778\ud574, \ud589\uc0ac \ucc38\uac00\uc790 \uc57d 3,000\uba85 \uc911 \ub300\ubd80\ubd84\uc774 \ub0a8\uc131\uc73c\ub85c \ucc44\uc6cc\uc9c0\ub294 \ud604\uc0c1\uc774 \ubc1c\uc0dd\ud588\ub2e4. \ud55c\ud3b8 \uacbd\ucc30 \uce21 \ucd94\uc0b0\uc5d0 \ub530\ub974\uba74 \uc774\ub0a0 \uc11c\uc6b8 \uc5ec\uc758\ub3c4 \uacf5\uc6d0\uc5d0\uc11c \uc9c4\ud589\ub41c \ud589\uc0ac\uc5d0\ub294 \ub0a8\uc131 700\uba85\uacfc \uc5ec\uc131 300\uba85 \ub4f1 \uc57d 1\ucc9c\uc5ec \uba85\uc774 \ucc38\uac00\ud55c \uac83\uc73c\ub85c \uc804\ud574\uc84c\ub2e4. \u201c\ub0a8\uc790\uc640 \uc5ec\uc790\uac00 \ub300\uce58\ud55c \uac83\uc774 \uc544\ub2c8\ub77c \uacbd\ucc30\uacfc \ube44\ub458\uae30\uac00 \ub300\uce58\ud558\ub294 \ubd84\uc704\uae30\u201d\ub77c\ub294 \uae00\uc774 \uc62c\ub77c\uc624\uba70 \ub2f9\uc2dc \uadf9\uba85\ud55c \ub0a8\ub140 \ube44\uc728\uc744 \uc124\uba85\ud558\uae30\ub3c4 \ud588\ub2e4. \uc9dd\uc744 \ucc3e\uc9c0 \ubabb\ud55c \ub300\ubd80\ubd84\uc758 \ucc38\uac00\uc790\ub4e4\uc774 \uadfc\ucc98 \uc74c\uc2dd\uc810, \uc220\uc9d1 \ub4f1\uc73c\ub85c \ud769\uc5b4\uc84c\ub2e4. \uc774 \ud589\uc0ac\uc5d0 \ucc38\uac00\ud588\ub358 \uac1c\uadf8\ub9e8 \ubc15\ud718\uc21c\uc740 \"\uc194\ub85c\ub300\ucca9\uc774 \uc220\ub85c\ub300\ucca9\uc73c\ub85c \uc774\uc5b4\uc9c0\uaca0\ub124\uc694\"\ub77c\ub294 \uae00\uc744 \ub0a8\uae30\uae30\ub3c4 \ud588\ub2e4. \uc218\ub3c4\uad8c\uacfc \uc9c0\ubc29\uc758 \uacbd\uc6b0 \uc5ec\uc131 \ucc38\uac00\uc790\uac00 \ub354 \uc801\uc5b4 \ud589\uc0ac \uc790\uccb4\uac00 \uc81c\ub300\ub85c \uc9c4\ud589\ub418\uc9c0 \ubabb\ud55c \uacbd\uc6b0\ub3c4 \uc788\uc5c8\ub2e4. \ucc9c\uc548 \uc194\ub85c\ub300\ucca9\uc758 \uacbd\uc6b0 \uc5ec\uc131 \ucc38\uac00\uc790\uac00 10\uc5ec \uba85 \uc815\ub3c4\uc5d0 \ubd88\uacfc\ud574 \ud589\uc0ac\ub97c \uce58\ub8e8\uc9c0 \ubabb\ud588\uc9c0\ub9cc, \uac1c\uc778\uc801\uc73c\ub85c \ubc88\ud638\ub97c \uc8fc\uace0\ubc1b\uc544 1\ud638 \ucee4\ud50c\uc774 \ud798\uacb9\uac8c \ud0c4\uc0dd\ub418\uae30\ub3c4 \ud588\ub2e4. \ud55c\ud3b8 \ub300\ubd80\ubd84\uc758 \uc5ec\uc131 \uc5f0\uc608\uc778\ub4e4\uc774 \uc548\uc804\uc0c1\uc758 \uc774\uc720\ub85c \ud589\uc0ac \ubd88\ucc38\uc744 \uc120\uc5b8\ud55c \uac00\uc6b4\ub370, \uc2e0\uc778 \uac78\uadf8\ub8f9 \ube44\ud0a4\ub2c8\uc758 \uba64\ubc84\uc778 \ud574\uc774\uac00 \uc194\ub85c\ub300\ucca9\uc5d0 \ucc38\uac00\ud558\uc5ec \uc2e4\uc2dc\uac04 \uac80\uc0c9 1\uc704\ub97c \ucc28\uc9c0\ud558\uae30\ub3c4 \ud558\uc600\ub2e4.", "sentence_answer": "\ud55c\ud3b8 \uacbd\ucc30 \uce21 \ucd94\uc0b0\uc5d0 \ub530\ub974\uba74 \uc774\ub0a0 \uc11c\uc6b8 \uc5ec\uc758\ub3c4 \uacf5\uc6d0\uc5d0\uc11c \uc9c4\ud589\ub41c \ud589\uc0ac\uc5d0\ub294 \ub0a8\uc131 700\uba85\uacfc \uc5ec\uc131 300\uba85 \ub4f1 \uc57d 1\ucc9c\uc5ec \uba85\uc774 \ucc38\uac00\ud55c \uac83\uc73c\ub85c \uc804\ud574\uc84c\ub2e4.", "paragraph_id": "6518390-2-1", "paragraph_question": "question: \uc194\ub85c\ub300\ucca9 \ub2f9\uc2dc \uacbd\ucc30 \uce21 \ucd94\uc0b0 \uc5ec\uc758\ub3c4 \uacf5\uc6d0\uc5d0 \uc628 \ub0a8 \ub140\uc758 \uc218\ub294?, context: \ub300\ud55c\ubbfc\uad6d \ucd5c\ucd08 \ud2b9\ubcc4\ud55c \ub0a0 \"\uc194\ub85c\ub300\ucca9\" \ud589\uc0ac\ub294 \uc11c\uc6b8\ud2b9\ubcc4\uc2dc\uc758 \uacbd\uc6b0 \uc2dc\uc98c\uc81c \ud06c\ub9ac\uc2a4\ub9c8\uc2a4 \uae30\uac04 \uc911 2012\ub144 12\uc6d4 24\uc77c\uc6d4\uc694\uc77c \ud3c9\uc77c/\uc784\uc2dc \uc790\uc728 1\uc77c \ud734\uc77c\uc81c \ud06c\ub9ac\uc2a4\ub9c8\uc2a4 \uc774\ube0c*\uc131\ud0c4\uc804\uc57c\uc81c \ub370\uc774* \uc624\ud6c43~5\uc2dc(\uc57d2\uc2dc\uac04 \ub3d9\uc548)\uc5d0 \ub0a8\uc790\ub4e4\uc740 \ud558\uc580\uc0c9, \uc5ec\uc790\ub4e4\uc740 \ubd89\uc740\uc0c9, \ucee4\ud50c\uc740 \ucd08\ub85d\uc0c9 \uacc4\uc5f4\uc758 \uc637\uc744 \uc785\uace0 \uc11c\uc6b8\uc5ec\uc758\ub3c4\uacf5\uc6d0\uc73c\ub85c \ubaa8\uc774\ub294 \uac83\uc73c\ub85c \uc2dc\uc791\ud558\uc600\ub2e4. \uacf5\uc6d0\uc5d0 \ubaa8\uc778 \ub0a8\ub140\ub294 \uc2e0\ud638 \uc591\ud3b8\uc5d0\uc11c \ub300\uae30\ud558\ub2e4\uac00 \uc9c4\ud589\uc790\uac00 \uc2e0\ud638\ub97c \ubcf4\ub0b4\uba74 \ub9c8\uc74c\uc5d0 \ub4dc\ub294 \uc774\uc131(\uc5ec\ub294 \ub0a8\uc73c\ub85c, \ub0a8\uc740 \uc5ec\ub85c)\uc744 \ud5a5\ud574 \ub2ec\ub824\uac00 \uc190\uc744 \uc7a1\ub294 \ud615\uc2dd\uc73c\ub85c \uc9c4\ud589\ub418\uc5c8\ub2e4. \uc2e4\uc81c \uc774 \ud589\uc0ac\ub97c \ud1b5\ud574 \uba87 \uc30d\uc758 \ucee4\ud50c\uc774 \ud0c4\uc0dd\ud558\uae30\ub3c4 \ud558\uc600\ub2e4. \ud558\uc9c0\ub9cc \uc548\uc804\uc0c1\uc758 \uc774\uc720 \ub4f1\uc73c\ub85c \uc0c1\ub2f9\uc218\uc758 \uc5ec\uc131 \ucc38\uac00 \uc2e0\uccad\uc790\ub4e4\uc774 \uc2e4\uc81c \ud589\uc0ac\uc5d0 \ucc38\uac00\ud558\uc9c0 \uc54a\uc74c\uc73c\ub85c \uc778\ud574, \ud589\uc0ac \ucc38\uac00\uc790 \uc57d 3,000\uba85 \uc911 \ub300\ubd80\ubd84\uc774 \ub0a8\uc131\uc73c\ub85c \ucc44\uc6cc\uc9c0\ub294 \ud604\uc0c1\uc774 \ubc1c\uc0dd\ud588\ub2e4. \ud55c\ud3b8 \uacbd\ucc30 \uce21 \ucd94\uc0b0\uc5d0 \ub530\ub974\uba74 \uc774\ub0a0 \uc11c\uc6b8 \uc5ec\uc758\ub3c4 \uacf5\uc6d0\uc5d0\uc11c \uc9c4\ud589\ub41c \ud589\uc0ac\uc5d0\ub294 \ub0a8\uc131 700\uba85\uacfc \uc5ec\uc131 300\uba85 \ub4f1 \uc57d 1\ucc9c\uc5ec \uba85\uc774 \ucc38\uac00\ud55c \uac83\uc73c\ub85c \uc804\ud574\uc84c\ub2e4. \u201c\ub0a8\uc790\uc640 \uc5ec\uc790\uac00 \ub300\uce58\ud55c \uac83\uc774 \uc544\ub2c8\ub77c \uacbd\ucc30\uacfc \ube44\ub458\uae30\uac00 \ub300\uce58\ud558\ub294 \ubd84\uc704\uae30\u201d\ub77c\ub294 \uae00\uc774 \uc62c\ub77c\uc624\uba70 \ub2f9\uc2dc \uadf9\uba85\ud55c \ub0a8\ub140 \ube44\uc728\uc744 \uc124\uba85\ud558\uae30\ub3c4 \ud588\ub2e4. \uc9dd\uc744 \ucc3e\uc9c0 \ubabb\ud55c \ub300\ubd80\ubd84\uc758 \ucc38\uac00\uc790\ub4e4\uc774 \uadfc\ucc98 \uc74c\uc2dd\uc810, \uc220\uc9d1 \ub4f1\uc73c\ub85c \ud769\uc5b4\uc84c\ub2e4. \uc774 \ud589\uc0ac\uc5d0 \ucc38\uac00\ud588\ub358 \uac1c\uadf8\ub9e8 \ubc15\ud718\uc21c\uc740 \"\uc194\ub85c\ub300\ucca9\uc774 \uc220\ub85c\ub300\ucca9\uc73c\ub85c \uc774\uc5b4\uc9c0\uaca0\ub124\uc694\"\ub77c\ub294 \uae00\uc744 \ub0a8\uae30\uae30\ub3c4 \ud588\ub2e4. \uc218\ub3c4\uad8c\uacfc \uc9c0\ubc29\uc758 \uacbd\uc6b0 \uc5ec\uc131 \ucc38\uac00\uc790\uac00 \ub354 \uc801\uc5b4 \ud589\uc0ac \uc790\uccb4\uac00 \uc81c\ub300\ub85c \uc9c4\ud589\ub418\uc9c0 \ubabb\ud55c \uacbd\uc6b0\ub3c4 \uc788\uc5c8\ub2e4. \ucc9c\uc548 \uc194\ub85c\ub300\ucca9\uc758 \uacbd\uc6b0 \uc5ec\uc131 \ucc38\uac00\uc790\uac00 10\uc5ec \uba85 \uc815\ub3c4\uc5d0 \ubd88\uacfc\ud574 \ud589\uc0ac\ub97c \uce58\ub8e8\uc9c0 \ubabb\ud588\uc9c0\ub9cc, \uac1c\uc778\uc801\uc73c\ub85c \ubc88\ud638\ub97c \uc8fc\uace0\ubc1b\uc544 1\ud638 \ucee4\ud50c\uc774 \ud798\uacb9\uac8c \ud0c4\uc0dd\ub418\uae30\ub3c4 \ud588\ub2e4. \ud55c\ud3b8 \ub300\ubd80\ubd84\uc758 \uc5ec\uc131 \uc5f0\uc608\uc778\ub4e4\uc774 \uc548\uc804\uc0c1\uc758 \uc774\uc720\ub85c \ud589\uc0ac \ubd88\ucc38\uc744 \uc120\uc5b8\ud55c \uac00\uc6b4\ub370, \uc2e0\uc778 \uac78\uadf8\ub8f9 \ube44\ud0a4\ub2c8\uc758 \uba64\ubc84\uc778 \ud574\uc774\uac00 \uc194\ub85c\ub300\ucca9\uc5d0 \ucc38\uac00\ud558\uc5ec \uc2e4\uc2dc\uac04 \uac80\uc0c9 1\uc704\ub97c \ucc28\uc9c0\ud558\uae30\ub3c4 \ud558\uc600\ub2e4."} {"question": "IPTV\uc640 \ub2ec\ub9ac 280\uc6d0\uc758 \uc7ac\uc1a1\uc2e0 \ub300\uac00\ub97c \uac70\ubd80\ud588\ub358 \ud154\ub808\ube44\uc804 \uc0ac\uc5c5\uc790\ub4e4\uc740 \uc5b4\ub5a4 \uc0ac\uc5c5\uc790\ub4e4\uc778\uac00?", "paragraph": "\uc9c0\uc0c1\ud30c \ud154\ub808\ube44\uc804 \ubc29\uc1a1 3\uc0ac\uc640 \ucf00\uc774\ube14 \ud154\ub808\ube44\uc804 \ubc29\uc1a1\uc0ac\ub294 2007\ub144\ubd80\ud130 \uc7ac\uc1a1\uc2e0 \ub300\uac00 \ub4f1\uc744 \uc774\uc720\ub85c \uac08\ub4f1\uc744 \ube5a\uc5b4\uc654\ub2e4. 2008\ub144\uc5d0 \uc9c0\uc0c1\ud30c \ubc29\uc1a1\uc0ac\ub294 IPTV \uc0ac\uc5c5\uc790\uc640\uc758 \uacfc\uae08 \ud611\uc0c1\uc744 \uac00\uc785\uc790\ub2f9 280\uc6d0\uc73c\ub85c \ud0c0\uacb0\ud558\ub294 \ub370\uc5d0\ub294 \uc131\uacf5\ud588\uc9c0\ub9cc, \ucf00\uc774\ube14 TV\uc640\uc758 \ud611\uc0c1\uc740 \uacb0\ub82c\ub418\uc5c8\ub2e4. 2009\ub144 11\uc6d4\uc5d0 \uc9c0\uc0c1\ud30c \ud154\ub808\ube44\uc804 \ubc29\uc1a1 3\uc0ac\ub294 5\ub300 \ucf00\uc774\ube14 \ud154\ub808\ube44\uc804 \ubc29\uc1a1\uc0ac\uc5d0 \ub300\ud574 \uc9c0\uc0c1\ud30c \uc7ac\uc1a1\uc2e0\uc744 \uae08\uc9c0\ud558\ub294 \ubbfc\uc0ac\ubcf8\uc548 \uc18c\uc1a1\uc744 \uc81c\uae30\ud55c\ub2e4. 2010\ub144 9\uc6d4\uc5d0 1\uc2ec \ud310\uacb0\uc5d0\uc11c \uc9c0\uc0c1\ud30c \ud154\ub808\ube44\uc804 \ubc29\uc1a1 3\uc0ac\uc758 \uc800\uc791\uad8c\uc774 \uc778\uc815\ub418\uc5c8\uc73c\uba70, 2009\ub144 12\uc6d4 18\uc77c \uc774\ud6c4 \ucf00\uc774\ube14 \ud154\ub808\ube44\uc804 \ubc29\uc1a1\uc0ac\uc5d0 \uac00\uc785\ud55c \uc0ac\ub78c\ub4e4\uc5d0 \ub300\ud574 \uc9c0\uc0c1\ud30c \ub3d9\uc2dc \uc7ac\uc1a1\uc2e0 \ud589\uc704\ub97c \uae08\uc9c0\ud55c\ub2e4\ub294 \ud310\uacb0\uc774 \ub0ac\ub2e4. \uc774\uc5d0 \ub300\ud574 \ubc29\uc1a1\ud1b5\uc2e0\uc704\uc6d0\ud68c\ub294 \uc81c\ub3c4\uac1c\uc120 \uc804\ub2f4\ubc18\uc744 \uad6c\uc131\ud558\uace0 \uc6b4\uc601\ud558\uae30\ub85c \ud558\uc600\ub2e4. \uadf8 \ud6c4, 2\uc2ec\uc5d0\uc11c\ub3c4 \uc774\uc640 \ube44\uc2b7\ud55c \ud310\uacb0\uc774 \ub098\uc654\uc9c0\ub9cc \uc9c0\uc0c1\ud30c \ud154\ub808\ube44\uc804 \ubc29\uc1a1\uc0ac\ub4e4\uc774 \ucf00\uc774\ube14 \ud154\ub808\ube44\uc804 \uc0ac\uc5c5\uc790\uc5d0\uac8c \uc7ac\uc1a1\uc2e0 \ub300\uac00\ub97c \uac15\uc81c\ud574\uc11c\ub294 \uc548\ub41c\ub2e4\ub294 \ud310\uacb0\uc774 \ub098\uc654\ub2e4. \uc9c0\uc0c1\ud30c \ud154\ub808\ube44\uc804 \ubc29\uc1a1 3\uc0ac\uc640 \ucf00\uc774\ube14 \ud154\ub808\ube44\uc804 \ubc29\uc1a1\uc0ac\ub294 \uc7ac\uc1a1\uc2e0 \ub300\uac00 \uae08\uc561 \ud611\uc0c1\uc744 \uacc4\uc18d \uc2dc\ub3c4\ud588\uc73c\ub098, \uc591 \uce21 \uc0ac\uc774\uc758 \uc774\uacac\uc758 \ud3ed\uc73c\ub85c \ud30c\ud589\uc774 \uc9c0\uc18d\ub418\uc5c8\ub2e4. \uc9c0\uc0c1\ud30c \ubc29\uc1a1\uc0ac\ub294 IPTV\uc640 \ub9c8\ucc2c\uac00\uc9c0\ub85c, \uc7ac\uc1a1\uc2e0 \uc694\uae08\uc73c\ub85c \uac00\uc785\uc790\ub2f9 280\uc6d0 \uc815\ub3c4\ub97c \uc694\uad6c\ud558\uc600\uc9c0\ub9cc, \ucf00\uc774\ube14 \ubc29\uc1a1\uc0ac\ub294 \uc7ac\uc1a1\uc2e0 \uc694\uae08\uc73c\ub85c \uac00\uc785\uc790\ub2f9 100\uc6d0\uc73c\ub85c \ub9de\uc11c\uc654\ub2e4.", "answer": "\ucf00\uc774\ube14 \ud154\ub808\ube44\uc804 \uc0ac\uc5c5\uc790", "sentence": "\uadf8 \ud6c4, 2\uc2ec\uc5d0\uc11c\ub3c4 \uc774\uc640 \ube44\uc2b7\ud55c \ud310\uacb0\uc774 \ub098\uc654\uc9c0\ub9cc \uc9c0\uc0c1\ud30c \ud154\ub808\ube44\uc804 \ubc29\uc1a1\uc0ac\ub4e4\uc774 \ucf00\uc774\ube14 \ud154\ub808\ube44\uc804 \uc0ac\uc5c5\uc790 \uc5d0\uac8c \uc7ac\uc1a1\uc2e0 \ub300\uac00\ub97c \uac15\uc81c\ud574\uc11c\ub294 \uc548\ub41c\ub2e4\ub294 \ud310\uacb0\uc774 \ub098\uc654\ub2e4.", "paragraph_sentence": "\uc9c0\uc0c1\ud30c \ud154\ub808\ube44\uc804 \ubc29\uc1a1 3\uc0ac\uc640 \ucf00\uc774\ube14 \ud154\ub808\ube44\uc804 \ubc29\uc1a1\uc0ac\ub294 2007\ub144\ubd80\ud130 \uc7ac\uc1a1\uc2e0 \ub300\uac00 \ub4f1\uc744 \uc774\uc720\ub85c \uac08\ub4f1\uc744 \ube5a\uc5b4\uc654\ub2e4. 2008\ub144\uc5d0 \uc9c0\uc0c1\ud30c \ubc29\uc1a1\uc0ac\ub294 IPTV \uc0ac\uc5c5\uc790\uc640\uc758 \uacfc\uae08 \ud611\uc0c1\uc744 \uac00\uc785\uc790\ub2f9 280\uc6d0\uc73c\ub85c \ud0c0\uacb0\ud558\ub294 \ub370\uc5d0\ub294 \uc131\uacf5\ud588\uc9c0\ub9cc, \ucf00\uc774\ube14 TV\uc640\uc758 \ud611\uc0c1\uc740 \uacb0\ub82c\ub418\uc5c8\ub2e4. 2009\ub144 11\uc6d4\uc5d0 \uc9c0\uc0c1\ud30c \ud154\ub808\ube44\uc804 \ubc29\uc1a1 3\uc0ac\ub294 5\ub300 \ucf00\uc774\ube14 \ud154\ub808\ube44\uc804 \ubc29\uc1a1\uc0ac\uc5d0 \ub300\ud574 \uc9c0\uc0c1\ud30c \uc7ac\uc1a1\uc2e0\uc744 \uae08\uc9c0\ud558\ub294 \ubbfc\uc0ac\ubcf8\uc548 \uc18c\uc1a1\uc744 \uc81c\uae30\ud55c\ub2e4. 2010\ub144 9\uc6d4\uc5d0 1\uc2ec \ud310\uacb0\uc5d0\uc11c \uc9c0\uc0c1\ud30c \ud154\ub808\ube44\uc804 \ubc29\uc1a1 3\uc0ac\uc758 \uc800\uc791\uad8c\uc774 \uc778\uc815\ub418\uc5c8\uc73c\uba70, 2009\ub144 12\uc6d4 18\uc77c \uc774\ud6c4 \ucf00\uc774\ube14 \ud154\ub808\ube44\uc804 \ubc29\uc1a1\uc0ac\uc5d0 \uac00\uc785\ud55c \uc0ac\ub78c\ub4e4\uc5d0 \ub300\ud574 \uc9c0\uc0c1\ud30c \ub3d9\uc2dc \uc7ac\uc1a1\uc2e0 \ud589\uc704\ub97c \uae08\uc9c0\ud55c\ub2e4\ub294 \ud310\uacb0\uc774 \ub0ac\ub2e4. \uc774\uc5d0 \ub300\ud574 \ubc29\uc1a1\ud1b5\uc2e0\uc704\uc6d0\ud68c\ub294 \uc81c\ub3c4\uac1c\uc120 \uc804\ub2f4\ubc18\uc744 \uad6c\uc131\ud558\uace0 \uc6b4\uc601\ud558\uae30\ub85c \ud558\uc600\ub2e4. \uadf8 \ud6c4, 2\uc2ec\uc5d0\uc11c\ub3c4 \uc774\uc640 \ube44\uc2b7\ud55c \ud310\uacb0\uc774 \ub098\uc654\uc9c0\ub9cc \uc9c0\uc0c1\ud30c \ud154\ub808\ube44\uc804 \ubc29\uc1a1\uc0ac\ub4e4\uc774 \ucf00\uc774\ube14 \ud154\ub808\ube44\uc804 \uc0ac\uc5c5\uc790 \uc5d0\uac8c \uc7ac\uc1a1\uc2e0 \ub300\uac00\ub97c \uac15\uc81c\ud574\uc11c\ub294 \uc548\ub41c\ub2e4\ub294 \ud310\uacb0\uc774 \ub098\uc654\ub2e4. \uc9c0\uc0c1\ud30c \ud154\ub808\ube44\uc804 \ubc29\uc1a1 3\uc0ac\uc640 \ucf00\uc774\ube14 \ud154\ub808\ube44\uc804 \ubc29\uc1a1\uc0ac\ub294 \uc7ac\uc1a1\uc2e0 \ub300\uac00 \uae08\uc561 \ud611\uc0c1\uc744 \uacc4\uc18d \uc2dc\ub3c4\ud588\uc73c\ub098, \uc591 \uce21 \uc0ac\uc774\uc758 \uc774\uacac\uc758 \ud3ed\uc73c\ub85c \ud30c\ud589\uc774 \uc9c0\uc18d\ub418\uc5c8\ub2e4. \uc9c0\uc0c1\ud30c \ubc29\uc1a1\uc0ac\ub294 IPTV\uc640 \ub9c8\ucc2c\uac00\uc9c0\ub85c, \uc7ac\uc1a1\uc2e0 \uc694\uae08\uc73c\ub85c \uac00\uc785\uc790\ub2f9 280\uc6d0 \uc815\ub3c4\ub97c \uc694\uad6c\ud558\uc600\uc9c0\ub9cc, \ucf00\uc774\ube14 \ubc29\uc1a1\uc0ac\ub294 \uc7ac\uc1a1\uc2e0 \uc694\uae08\uc73c\ub85c \uac00\uc785\uc790\ub2f9 100\uc6d0\uc73c\ub85c \ub9de\uc11c\uc654\ub2e4.", "paragraph_answer": "\uc9c0\uc0c1\ud30c \ud154\ub808\ube44\uc804 \ubc29\uc1a1 3\uc0ac\uc640 \ucf00\uc774\ube14 \ud154\ub808\ube44\uc804 \ubc29\uc1a1\uc0ac\ub294 2007\ub144\ubd80\ud130 \uc7ac\uc1a1\uc2e0 \ub300\uac00 \ub4f1\uc744 \uc774\uc720\ub85c \uac08\ub4f1\uc744 \ube5a\uc5b4\uc654\ub2e4. 2008\ub144\uc5d0 \uc9c0\uc0c1\ud30c \ubc29\uc1a1\uc0ac\ub294 IPTV \uc0ac\uc5c5\uc790\uc640\uc758 \uacfc\uae08 \ud611\uc0c1\uc744 \uac00\uc785\uc790\ub2f9 280\uc6d0\uc73c\ub85c \ud0c0\uacb0\ud558\ub294 \ub370\uc5d0\ub294 \uc131\uacf5\ud588\uc9c0\ub9cc, \ucf00\uc774\ube14 TV\uc640\uc758 \ud611\uc0c1\uc740 \uacb0\ub82c\ub418\uc5c8\ub2e4. 2009\ub144 11\uc6d4\uc5d0 \uc9c0\uc0c1\ud30c \ud154\ub808\ube44\uc804 \ubc29\uc1a1 3\uc0ac\ub294 5\ub300 \ucf00\uc774\ube14 \ud154\ub808\ube44\uc804 \ubc29\uc1a1\uc0ac\uc5d0 \ub300\ud574 \uc9c0\uc0c1\ud30c \uc7ac\uc1a1\uc2e0\uc744 \uae08\uc9c0\ud558\ub294 \ubbfc\uc0ac\ubcf8\uc548 \uc18c\uc1a1\uc744 \uc81c\uae30\ud55c\ub2e4. 2010\ub144 9\uc6d4\uc5d0 1\uc2ec \ud310\uacb0\uc5d0\uc11c \uc9c0\uc0c1\ud30c \ud154\ub808\ube44\uc804 \ubc29\uc1a1 3\uc0ac\uc758 \uc800\uc791\uad8c\uc774 \uc778\uc815\ub418\uc5c8\uc73c\uba70, 2009\ub144 12\uc6d4 18\uc77c \uc774\ud6c4 \ucf00\uc774\ube14 \ud154\ub808\ube44\uc804 \ubc29\uc1a1\uc0ac\uc5d0 \uac00\uc785\ud55c \uc0ac\ub78c\ub4e4\uc5d0 \ub300\ud574 \uc9c0\uc0c1\ud30c \ub3d9\uc2dc \uc7ac\uc1a1\uc2e0 \ud589\uc704\ub97c \uae08\uc9c0\ud55c\ub2e4\ub294 \ud310\uacb0\uc774 \ub0ac\ub2e4. \uc774\uc5d0 \ub300\ud574 \ubc29\uc1a1\ud1b5\uc2e0\uc704\uc6d0\ud68c\ub294 \uc81c\ub3c4\uac1c\uc120 \uc804\ub2f4\ubc18\uc744 \uad6c\uc131\ud558\uace0 \uc6b4\uc601\ud558\uae30\ub85c \ud558\uc600\ub2e4. \uadf8 \ud6c4, 2\uc2ec\uc5d0\uc11c\ub3c4 \uc774\uc640 \ube44\uc2b7\ud55c \ud310\uacb0\uc774 \ub098\uc654\uc9c0\ub9cc \uc9c0\uc0c1\ud30c \ud154\ub808\ube44\uc804 \ubc29\uc1a1\uc0ac\ub4e4\uc774 \ucf00\uc774\ube14 \ud154\ub808\ube44\uc804 \uc0ac\uc5c5\uc790 \uc5d0\uac8c \uc7ac\uc1a1\uc2e0 \ub300\uac00\ub97c \uac15\uc81c\ud574\uc11c\ub294 \uc548\ub41c\ub2e4\ub294 \ud310\uacb0\uc774 \ub098\uc654\ub2e4. \uc9c0\uc0c1\ud30c \ud154\ub808\ube44\uc804 \ubc29\uc1a1 3\uc0ac\uc640 \ucf00\uc774\ube14 \ud154\ub808\ube44\uc804 \ubc29\uc1a1\uc0ac\ub294 \uc7ac\uc1a1\uc2e0 \ub300\uac00 \uae08\uc561 \ud611\uc0c1\uc744 \uacc4\uc18d \uc2dc\ub3c4\ud588\uc73c\ub098, \uc591 \uce21 \uc0ac\uc774\uc758 \uc774\uacac\uc758 \ud3ed\uc73c\ub85c \ud30c\ud589\uc774 \uc9c0\uc18d\ub418\uc5c8\ub2e4. \uc9c0\uc0c1\ud30c \ubc29\uc1a1\uc0ac\ub294 IPTV\uc640 \ub9c8\ucc2c\uac00\uc9c0\ub85c, \uc7ac\uc1a1\uc2e0 \uc694\uae08\uc73c\ub85c \uac00\uc785\uc790\ub2f9 280\uc6d0 \uc815\ub3c4\ub97c \uc694\uad6c\ud558\uc600\uc9c0\ub9cc, \ucf00\uc774\ube14 \ubc29\uc1a1\uc0ac\ub294 \uc7ac\uc1a1\uc2e0 \uc694\uae08\uc73c\ub85c \uac00\uc785\uc790\ub2f9 100\uc6d0\uc73c\ub85c \ub9de\uc11c\uc654\ub2e4.", "sentence_answer": "\uadf8 \ud6c4, 2\uc2ec\uc5d0\uc11c\ub3c4 \uc774\uc640 \ube44\uc2b7\ud55c \ud310\uacb0\uc774 \ub098\uc654\uc9c0\ub9cc \uc9c0\uc0c1\ud30c \ud154\ub808\ube44\uc804 \ubc29\uc1a1\uc0ac\ub4e4\uc774 \ucf00\uc774\ube14 \ud154\ub808\ube44\uc804 \uc0ac\uc5c5\uc790 \uc5d0\uac8c \uc7ac\uc1a1\uc2e0 \ub300\uac00\ub97c \uac15\uc81c\ud574\uc11c\ub294 \uc548\ub41c\ub2e4\ub294 \ud310\uacb0\uc774 \ub098\uc654\ub2e4.", "paragraph_id": "6486479-0-1", "paragraph_question": "question: IPTV\uc640 \ub2ec\ub9ac 280\uc6d0\uc758 \uc7ac\uc1a1\uc2e0 \ub300\uac00\ub97c \uac70\ubd80\ud588\ub358 \ud154\ub808\ube44\uc804 \uc0ac\uc5c5\uc790\ub4e4\uc740 \uc5b4\ub5a4 \uc0ac\uc5c5\uc790\ub4e4\uc778\uac00?, context: \uc9c0\uc0c1\ud30c \ud154\ub808\ube44\uc804 \ubc29\uc1a1 3\uc0ac\uc640 \ucf00\uc774\ube14 \ud154\ub808\ube44\uc804 \ubc29\uc1a1\uc0ac\ub294 2007\ub144\ubd80\ud130 \uc7ac\uc1a1\uc2e0 \ub300\uac00 \ub4f1\uc744 \uc774\uc720\ub85c \uac08\ub4f1\uc744 \ube5a\uc5b4\uc654\ub2e4. 2008\ub144\uc5d0 \uc9c0\uc0c1\ud30c \ubc29\uc1a1\uc0ac\ub294 IPTV \uc0ac\uc5c5\uc790\uc640\uc758 \uacfc\uae08 \ud611\uc0c1\uc744 \uac00\uc785\uc790\ub2f9 280\uc6d0\uc73c\ub85c \ud0c0\uacb0\ud558\ub294 \ub370\uc5d0\ub294 \uc131\uacf5\ud588\uc9c0\ub9cc, \ucf00\uc774\ube14 TV\uc640\uc758 \ud611\uc0c1\uc740 \uacb0\ub82c\ub418\uc5c8\ub2e4. 2009\ub144 11\uc6d4\uc5d0 \uc9c0\uc0c1\ud30c \ud154\ub808\ube44\uc804 \ubc29\uc1a1 3\uc0ac\ub294 5\ub300 \ucf00\uc774\ube14 \ud154\ub808\ube44\uc804 \ubc29\uc1a1\uc0ac\uc5d0 \ub300\ud574 \uc9c0\uc0c1\ud30c \uc7ac\uc1a1\uc2e0\uc744 \uae08\uc9c0\ud558\ub294 \ubbfc\uc0ac\ubcf8\uc548 \uc18c\uc1a1\uc744 \uc81c\uae30\ud55c\ub2e4. 2010\ub144 9\uc6d4\uc5d0 1\uc2ec \ud310\uacb0\uc5d0\uc11c \uc9c0\uc0c1\ud30c \ud154\ub808\ube44\uc804 \ubc29\uc1a1 3\uc0ac\uc758 \uc800\uc791\uad8c\uc774 \uc778\uc815\ub418\uc5c8\uc73c\uba70, 2009\ub144 12\uc6d4 18\uc77c \uc774\ud6c4 \ucf00\uc774\ube14 \ud154\ub808\ube44\uc804 \ubc29\uc1a1\uc0ac\uc5d0 \uac00\uc785\ud55c \uc0ac\ub78c\ub4e4\uc5d0 \ub300\ud574 \uc9c0\uc0c1\ud30c \ub3d9\uc2dc \uc7ac\uc1a1\uc2e0 \ud589\uc704\ub97c \uae08\uc9c0\ud55c\ub2e4\ub294 \ud310\uacb0\uc774 \ub0ac\ub2e4. \uc774\uc5d0 \ub300\ud574 \ubc29\uc1a1\ud1b5\uc2e0\uc704\uc6d0\ud68c\ub294 \uc81c\ub3c4\uac1c\uc120 \uc804\ub2f4\ubc18\uc744 \uad6c\uc131\ud558\uace0 \uc6b4\uc601\ud558\uae30\ub85c \ud558\uc600\ub2e4. \uadf8 \ud6c4, 2\uc2ec\uc5d0\uc11c\ub3c4 \uc774\uc640 \ube44\uc2b7\ud55c \ud310\uacb0\uc774 \ub098\uc654\uc9c0\ub9cc \uc9c0\uc0c1\ud30c \ud154\ub808\ube44\uc804 \ubc29\uc1a1\uc0ac\ub4e4\uc774 \ucf00\uc774\ube14 \ud154\ub808\ube44\uc804 \uc0ac\uc5c5\uc790\uc5d0\uac8c \uc7ac\uc1a1\uc2e0 \ub300\uac00\ub97c \uac15\uc81c\ud574\uc11c\ub294 \uc548\ub41c\ub2e4\ub294 \ud310\uacb0\uc774 \ub098\uc654\ub2e4. \uc9c0\uc0c1\ud30c \ud154\ub808\ube44\uc804 \ubc29\uc1a1 3\uc0ac\uc640 \ucf00\uc774\ube14 \ud154\ub808\ube44\uc804 \ubc29\uc1a1\uc0ac\ub294 \uc7ac\uc1a1\uc2e0 \ub300\uac00 \uae08\uc561 \ud611\uc0c1\uc744 \uacc4\uc18d \uc2dc\ub3c4\ud588\uc73c\ub098, \uc591 \uce21 \uc0ac\uc774\uc758 \uc774\uacac\uc758 \ud3ed\uc73c\ub85c \ud30c\ud589\uc774 \uc9c0\uc18d\ub418\uc5c8\ub2e4. \uc9c0\uc0c1\ud30c \ubc29\uc1a1\uc0ac\ub294 IPTV\uc640 \ub9c8\ucc2c\uac00\uc9c0\ub85c, \uc7ac\uc1a1\uc2e0 \uc694\uae08\uc73c\ub85c \uac00\uc785\uc790\ub2f9 280\uc6d0 \uc815\ub3c4\ub97c \uc694\uad6c\ud558\uc600\uc9c0\ub9cc, \ucf00\uc774\ube14 \ubc29\uc1a1\uc0ac\ub294 \uc7ac\uc1a1\uc2e0 \uc694\uae08\uc73c\ub85c \uac00\uc785\uc790\ub2f9 100\uc6d0\uc73c\ub85c \ub9de\uc11c\uc654\ub2e4."} {"question": "\ub9d0\ucf64 \uc568\ub9ac\uc2a8 \uac10\ub3c5\uc774 \ub9e8\uccb4\uc2a4\ud130 \uc2dc\ud2f0 \uc720\ub2c8\ud3fc\uc744 \ube68\uac04 \uc0c1\uc758\uc640 \uac80\uc815 \ud558\uc758\ub85c \uc120\ud0dd\ud55c \ubc14\ud0d5\uc774 \ub418\ub294 \ud300\uc740?", "paragraph": "\ub9e8\uccb4\uc2a4\ud130 \uc2dc\ud2f0\uc758 \ud648 \uc720\ub2c8\ud3fc \uc0c9\uc740 \ud558\ub298\uc0c9 \uc0c1\uc758\uc640 \ud770\uc0c9 \ud558\uc758\uc774\ub2e4. \uc608\uc804\uc758 \uc6d0\uc815 \ud558\uc758\ub294 \uc0c1\ud558\uc758 \ubaa8\ub450 \uc801\uac08\uc0c9 \uc774\uac70\ub098 \ube68\uac04\uc0c9 \uc0c1\uc758\uc640 \uac80\uc815\uc0c9 \ud558\uc758 \uc774\uae30\ub3c4 \ud558\uc600\ub2e4. \ud074\ub7fd\uc758 \uc624\ub9ac\uc9c0\ub110 \ud648 \uc720\ub2c8\ud3fc\uc740 \ubd88\uba85\ud655\ud558\uae34 \ud558\uc9c0\ub9cc 1892\ub144\uc744 \uc804\ud6c4\ub85c \ud30c\ub780\uc0c9\uc744 \uc785\uc5c8\uc5c8\ub2e4\ub294 \uc99d\uac70\uac00 \uc788\ub2e4. '\uc720\uba85\ud55c \ucd95\uad6c \ud074\ub7fd\ub4e4 - \ub9e8\uccb4\uc2a4\ud130 \uc2dc\ud2f0'\ub77c \uc9c0\uc5b4\uc9c4 1940\ub144\ub300\uc5d0 \ud3b4\ub0b8 \uc774 \uc18c\ucc45\uc790\uc5d0\uc11c\ub294 \uacfc\uac70 \uc6e8\uc2a4\ud2b8 \uace0\ud2bc (\uc138\uc778\ud2b8 \ub9c8\ud06c\uc2a4) \uc774 \ubcf8\ub798 \uc554\uc801\uc0c9 \uc0c1\uc758\uc640 \uac80\uc815\uc0c9 \ud558\uc758\ub97c \uc785\uc5c8\ub2e4\uace0 \uba85\uc2dc\ub418\uc5b4 \uc788\uc73c\uba70, 1884\ub144\uc5d0\ub294 \ud074\ub7fd \ucc3d\ub2e8\uc774 \uad50\ud68c\uc5d0\uc11c\ubd80\ud130 \ube44\ub86f\ub418\uc5c8\ub2e4\ub294 \uac83\uc744 \ubcf4\uc5ec\uc8fc\uae30 \uc704\ud574 \ubc31\uc2ed\uc790 (white cross) \uac00 \uc0c8\uaca8\uc9c4 \uac80\uc815\uc0c9 \uc720\ub2c8\ud3fc\uc744 \uc785\uc5c8\uc5c8\ub2e4\uace0 \uc804\ud574\uc838 \uc788\ub2e4. \ube68\uac04\uc0c9 \uc0c1\uc758\uc640 \uac80\uc815\uc0c9 \ud558\uc758 \uc6d0\uc815 \uc720\ub2c8\ud3fc\uc740 \u524d \uc5b4\uc2dc\uc2a4\ud134\ud2b8 \ucf54\uce58\uc774\uc790 \uac10\ub3c5\uc774\uc5c8\ub358 \ub9d0\ucf64 \uc568\ub9ac\uc2a8 \uc2dc\uc808\ubd80\ud130 \ub098\uc654\uc73c\uba70, \uadf8\ub294 \uc774\ud0c8\ub9ac\uc544\uc758 \uba85\ubb38\ud074\ub7fd\uc778 AC \ubc00\ub780\uc758 \uc720\ub2c8\ud3fc\uc744 \ucc44\ud0dd\ud558\uc5ec \uadf8\ub7f0 \uc0c9\uae54\uc758 \uc720\ub2c8\ud3fc\uc774 \ub9e8\uccb4\uc2a4\ud130 \uc2dc\ud2f0\uc758 \uc131\uacf5\uc744 \ubd88\uc5b4\ub123\uc5b4\uc904 \uac78\ub85c \ubbff\uace0 \uc788\uc5c8\ub2e4.", "answer": "AC \ubc00\ub780", "sentence": "\uadf8\ub294 \uc774\ud0c8\ub9ac\uc544\uc758 \uba85\ubb38\ud074\ub7fd\uc778 AC \ubc00\ub780 \uc758 \uc720\ub2c8\ud3fc\uc744 \ucc44\ud0dd\ud558\uc5ec \uadf8\ub7f0 \uc0c9\uae54\uc758 \uc720\ub2c8\ud3fc\uc774 \ub9e8\uccb4\uc2a4\ud130 \uc2dc\ud2f0\uc758 \uc131\uacf5\uc744 \ubd88\uc5b4\ub123\uc5b4\uc904 \uac78\ub85c \ubbff\uace0 \uc788\uc5c8\ub2e4.", "paragraph_sentence": "\ub9e8\uccb4\uc2a4\ud130 \uc2dc\ud2f0\uc758 \ud648 \uc720\ub2c8\ud3fc \uc0c9\uc740 \ud558\ub298\uc0c9 \uc0c1\uc758\uc640 \ud770\uc0c9 \ud558\uc758\uc774\ub2e4. \uc608\uc804\uc758 \uc6d0\uc815 \ud558\uc758\ub294 \uc0c1\ud558\uc758 \ubaa8\ub450 \uc801\uac08\uc0c9 \uc774\uac70\ub098 \ube68\uac04\uc0c9 \uc0c1\uc758\uc640 \uac80\uc815\uc0c9 \ud558\uc758 \uc774\uae30\ub3c4 \ud558\uc600\ub2e4. \ud074\ub7fd\uc758 \uc624\ub9ac\uc9c0\ub110 \ud648 \uc720\ub2c8\ud3fc\uc740 \ubd88\uba85\ud655\ud558\uae34 \ud558\uc9c0\ub9cc 1892\ub144\uc744 \uc804\ud6c4\ub85c \ud30c\ub780\uc0c9\uc744 \uc785\uc5c8\uc5c8\ub2e4\ub294 \uc99d\uac70\uac00 \uc788\ub2e4. '\uc720\uba85\ud55c \ucd95\uad6c \ud074\ub7fd\ub4e4 - \ub9e8\uccb4\uc2a4\ud130 \uc2dc\ud2f0'\ub77c \uc9c0\uc5b4\uc9c4 1940\ub144\ub300\uc5d0 \ud3b4\ub0b8 \uc774 \uc18c\ucc45\uc790\uc5d0\uc11c\ub294 \uacfc\uac70 \uc6e8\uc2a4\ud2b8 \uace0\ud2bc (\uc138\uc778\ud2b8 \ub9c8\ud06c\uc2a4) \uc774 \ubcf8\ub798 \uc554\uc801\uc0c9 \uc0c1\uc758\uc640 \uac80\uc815\uc0c9 \ud558\uc758\ub97c \uc785\uc5c8\ub2e4\uace0 \uba85\uc2dc\ub418\uc5b4 \uc788\uc73c\uba70, 1884\ub144\uc5d0\ub294 \ud074\ub7fd \ucc3d\ub2e8\uc774 \uad50\ud68c\uc5d0\uc11c\ubd80\ud130 \ube44\ub86f\ub418\uc5c8\ub2e4\ub294 \uac83\uc744 \ubcf4\uc5ec\uc8fc\uae30 \uc704\ud574 \ubc31\uc2ed\uc790 (white cross) \uac00 \uc0c8\uaca8\uc9c4 \uac80\uc815\uc0c9 \uc720\ub2c8\ud3fc\uc744 \uc785\uc5c8\uc5c8\ub2e4\uace0 \uc804\ud574\uc838 \uc788\ub2e4. \ube68\uac04\uc0c9 \uc0c1\uc758\uc640 \uac80\uc815\uc0c9 \ud558\uc758 \uc6d0\uc815 \uc720\ub2c8\ud3fc\uc740 \u524d \uc5b4\uc2dc\uc2a4\ud134\ud2b8 \ucf54\uce58\uc774\uc790 \uac10\ub3c5\uc774\uc5c8\ub358 \ub9d0\ucf64 \uc568\ub9ac\uc2a8 \uc2dc\uc808\ubd80\ud130 \ub098\uc654\uc73c\uba70, \uadf8\ub294 \uc774\ud0c8\ub9ac\uc544\uc758 \uba85\ubb38\ud074\ub7fd\uc778 AC \ubc00\ub780 \uc758 \uc720\ub2c8\ud3fc\uc744 \ucc44\ud0dd\ud558\uc5ec \uadf8\ub7f0 \uc0c9\uae54\uc758 \uc720\ub2c8\ud3fc\uc774 \ub9e8\uccb4\uc2a4\ud130 \uc2dc\ud2f0\uc758 \uc131\uacf5\uc744 \ubd88\uc5b4\ub123\uc5b4\uc904 \uac78\ub85c \ubbff\uace0 \uc788\uc5c8\ub2e4. ", "paragraph_answer": "\ub9e8\uccb4\uc2a4\ud130 \uc2dc\ud2f0\uc758 \ud648 \uc720\ub2c8\ud3fc \uc0c9\uc740 \ud558\ub298\uc0c9 \uc0c1\uc758\uc640 \ud770\uc0c9 \ud558\uc758\uc774\ub2e4. \uc608\uc804\uc758 \uc6d0\uc815 \ud558\uc758\ub294 \uc0c1\ud558\uc758 \ubaa8\ub450 \uc801\uac08\uc0c9 \uc774\uac70\ub098 \ube68\uac04\uc0c9 \uc0c1\uc758\uc640 \uac80\uc815\uc0c9 \ud558\uc758 \uc774\uae30\ub3c4 \ud558\uc600\ub2e4. \ud074\ub7fd\uc758 \uc624\ub9ac\uc9c0\ub110 \ud648 \uc720\ub2c8\ud3fc\uc740 \ubd88\uba85\ud655\ud558\uae34 \ud558\uc9c0\ub9cc 1892\ub144\uc744 \uc804\ud6c4\ub85c \ud30c\ub780\uc0c9\uc744 \uc785\uc5c8\uc5c8\ub2e4\ub294 \uc99d\uac70\uac00 \uc788\ub2e4. '\uc720\uba85\ud55c \ucd95\uad6c \ud074\ub7fd\ub4e4 - \ub9e8\uccb4\uc2a4\ud130 \uc2dc\ud2f0'\ub77c \uc9c0\uc5b4\uc9c4 1940\ub144\ub300\uc5d0 \ud3b4\ub0b8 \uc774 \uc18c\ucc45\uc790\uc5d0\uc11c\ub294 \uacfc\uac70 \uc6e8\uc2a4\ud2b8 \uace0\ud2bc (\uc138\uc778\ud2b8 \ub9c8\ud06c\uc2a4) \uc774 \ubcf8\ub798 \uc554\uc801\uc0c9 \uc0c1\uc758\uc640 \uac80\uc815\uc0c9 \ud558\uc758\ub97c \uc785\uc5c8\ub2e4\uace0 \uba85\uc2dc\ub418\uc5b4 \uc788\uc73c\uba70, 1884\ub144\uc5d0\ub294 \ud074\ub7fd \ucc3d\ub2e8\uc774 \uad50\ud68c\uc5d0\uc11c\ubd80\ud130 \ube44\ub86f\ub418\uc5c8\ub2e4\ub294 \uac83\uc744 \ubcf4\uc5ec\uc8fc\uae30 \uc704\ud574 \ubc31\uc2ed\uc790 (white cross) \uac00 \uc0c8\uaca8\uc9c4 \uac80\uc815\uc0c9 \uc720\ub2c8\ud3fc\uc744 \uc785\uc5c8\uc5c8\ub2e4\uace0 \uc804\ud574\uc838 \uc788\ub2e4. \ube68\uac04\uc0c9 \uc0c1\uc758\uc640 \uac80\uc815\uc0c9 \ud558\uc758 \uc6d0\uc815 \uc720\ub2c8\ud3fc\uc740 \u524d \uc5b4\uc2dc\uc2a4\ud134\ud2b8 \ucf54\uce58\uc774\uc790 \uac10\ub3c5\uc774\uc5c8\ub358 \ub9d0\ucf64 \uc568\ub9ac\uc2a8 \uc2dc\uc808\ubd80\ud130 \ub098\uc654\uc73c\uba70, \uadf8\ub294 \uc774\ud0c8\ub9ac\uc544\uc758 \uba85\ubb38\ud074\ub7fd\uc778 AC \ubc00\ub780 \uc758 \uc720\ub2c8\ud3fc\uc744 \ucc44\ud0dd\ud558\uc5ec \uadf8\ub7f0 \uc0c9\uae54\uc758 \uc720\ub2c8\ud3fc\uc774 \ub9e8\uccb4\uc2a4\ud130 \uc2dc\ud2f0\uc758 \uc131\uacf5\uc744 \ubd88\uc5b4\ub123\uc5b4\uc904 \uac78\ub85c \ubbff\uace0 \uc788\uc5c8\ub2e4.", "sentence_answer": "\uadf8\ub294 \uc774\ud0c8\ub9ac\uc544\uc758 \uba85\ubb38\ud074\ub7fd\uc778 AC \ubc00\ub780 \uc758 \uc720\ub2c8\ud3fc\uc744 \ucc44\ud0dd\ud558\uc5ec \uadf8\ub7f0 \uc0c9\uae54\uc758 \uc720\ub2c8\ud3fc\uc774 \ub9e8\uccb4\uc2a4\ud130 \uc2dc\ud2f0\uc758 \uc131\uacf5\uc744 \ubd88\uc5b4\ub123\uc5b4\uc904 \uac78\ub85c \ubbff\uace0 \uc788\uc5c8\ub2e4.", "paragraph_id": "6502151-17-1", "paragraph_question": "question: \ub9d0\ucf64 \uc568\ub9ac\uc2a8 \uac10\ub3c5\uc774 \ub9e8\uccb4\uc2a4\ud130 \uc2dc\ud2f0 \uc720\ub2c8\ud3fc\uc744 \ube68\uac04 \uc0c1\uc758\uc640 \uac80\uc815 \ud558\uc758\ub85c \uc120\ud0dd\ud55c \ubc14\ud0d5\uc774 \ub418\ub294 \ud300\uc740?, context: \ub9e8\uccb4\uc2a4\ud130 \uc2dc\ud2f0\uc758 \ud648 \uc720\ub2c8\ud3fc \uc0c9\uc740 \ud558\ub298\uc0c9 \uc0c1\uc758\uc640 \ud770\uc0c9 \ud558\uc758\uc774\ub2e4. \uc608\uc804\uc758 \uc6d0\uc815 \ud558\uc758\ub294 \uc0c1\ud558\uc758 \ubaa8\ub450 \uc801\uac08\uc0c9 \uc774\uac70\ub098 \ube68\uac04\uc0c9 \uc0c1\uc758\uc640 \uac80\uc815\uc0c9 \ud558\uc758 \uc774\uae30\ub3c4 \ud558\uc600\ub2e4. \ud074\ub7fd\uc758 \uc624\ub9ac\uc9c0\ub110 \ud648 \uc720\ub2c8\ud3fc\uc740 \ubd88\uba85\ud655\ud558\uae34 \ud558\uc9c0\ub9cc 1892\ub144\uc744 \uc804\ud6c4\ub85c \ud30c\ub780\uc0c9\uc744 \uc785\uc5c8\uc5c8\ub2e4\ub294 \uc99d\uac70\uac00 \uc788\ub2e4. '\uc720\uba85\ud55c \ucd95\uad6c \ud074\ub7fd\ub4e4 - \ub9e8\uccb4\uc2a4\ud130 \uc2dc\ud2f0'\ub77c \uc9c0\uc5b4\uc9c4 1940\ub144\ub300\uc5d0 \ud3b4\ub0b8 \uc774 \uc18c\ucc45\uc790\uc5d0\uc11c\ub294 \uacfc\uac70 \uc6e8\uc2a4\ud2b8 \uace0\ud2bc (\uc138\uc778\ud2b8 \ub9c8\ud06c\uc2a4) \uc774 \ubcf8\ub798 \uc554\uc801\uc0c9 \uc0c1\uc758\uc640 \uac80\uc815\uc0c9 \ud558\uc758\ub97c \uc785\uc5c8\ub2e4\uace0 \uba85\uc2dc\ub418\uc5b4 \uc788\uc73c\uba70, 1884\ub144\uc5d0\ub294 \ud074\ub7fd \ucc3d\ub2e8\uc774 \uad50\ud68c\uc5d0\uc11c\ubd80\ud130 \ube44\ub86f\ub418\uc5c8\ub2e4\ub294 \uac83\uc744 \ubcf4\uc5ec\uc8fc\uae30 \uc704\ud574 \ubc31\uc2ed\uc790 (white cross) \uac00 \uc0c8\uaca8\uc9c4 \uac80\uc815\uc0c9 \uc720\ub2c8\ud3fc\uc744 \uc785\uc5c8\uc5c8\ub2e4\uace0 \uc804\ud574\uc838 \uc788\ub2e4. \ube68\uac04\uc0c9 \uc0c1\uc758\uc640 \uac80\uc815\uc0c9 \ud558\uc758 \uc6d0\uc815 \uc720\ub2c8\ud3fc\uc740 \u524d \uc5b4\uc2dc\uc2a4\ud134\ud2b8 \ucf54\uce58\uc774\uc790 \uac10\ub3c5\uc774\uc5c8\ub358 \ub9d0\ucf64 \uc568\ub9ac\uc2a8 \uc2dc\uc808\ubd80\ud130 \ub098\uc654\uc73c\uba70, \uadf8\ub294 \uc774\ud0c8\ub9ac\uc544\uc758 \uba85\ubb38\ud074\ub7fd\uc778 AC \ubc00\ub780\uc758 \uc720\ub2c8\ud3fc\uc744 \ucc44\ud0dd\ud558\uc5ec \uadf8\ub7f0 \uc0c9\uae54\uc758 \uc720\ub2c8\ud3fc\uc774 \ub9e8\uccb4\uc2a4\ud130 \uc2dc\ud2f0\uc758 \uc131\uacf5\uc744 \ubd88\uc5b4\ub123\uc5b4\uc904 \uac78\ub85c \ubbff\uace0 \uc788\uc5c8\ub2e4."} {"question": "\uc5ec\uc131 \uc131\uae30 \uc808\uc81c \uc2dc\uc220\uc5d0\uc11c \ubd09\uc1c4\uc220\uc5d0 \uc5b4\ub5a4 \ub098\ubb34\uc758 \uac00\uc2dc\uac00 \uc0b4\uc744 \ub6ab\ub294\ub370 \uc0ac\uc6a9\ub418\ub098?", "paragraph": "\ud0dc\uc5b4\ub098\uc11c\ubd80\ud130 \uccab \uc6d4\uacbd\uae4c\uc9c0\uc758 \uc2dc\uac04 \uc911\uc5d0 \uc5ec\uc131\ud560\ub840\ub97c \ubc1b\uac8c \ub418\uc9c0\ub9cc \ub300\ubd80\ubd84 4-8\uc138\uc5d0 \uc5ec\uc131\uc131\uae30\uc808\uc81c\uc220\uc744 \ubc1b\uac8c \ub41c\ub2e4. \uc5ec\uc131 \uc131\uae30\uc758 \uc808\uc81c \uc21c\uc11c\uc758 \uc2e4\uc81c \ubc29\ubc95\uc740 \uc778\uc885 \uc9d1\ub2e8\uacfc \ub3c4\uc2dc \uadf8\ub9ac\uace0 \ub18d\ucd0c\uc9c0\uc5ed \ub4f1\uc5d0 \ub530\ub77c \ub2e4\uc591\ud55c \uc808\ucc28\ub4e4\uc774 \uc788\ub2e4. \uc0ac\ud68c \uacbd\uc81c\uc801 \uc0c1\ud0dc\uc5d0 \ub530\ub77c\uc11c\ub3c4 \ub2e4\ub974\uac8c \ub098\ud0c0\ub09c\ub2e4.\ub3c4\uc2dc \uc9c0\uc5ed\uc5d0\uc11c\ub294 \ubcd1\uc6d0\uc5d0\uc11c\ub3c4 \uc774\ub8e8\uc5b4\uc9c0\uba70 \uc804\ubb38\uc758\ub4e4\uc774 \ub2f4\ub2f9\ud558\uae30\ub3c4 \ud558\uc9c0\ub9cc \uc774 \uc2dc\uc220\uc740 \uc8fc\ub85c \ub098\uc774 \ub9ce\uc740 \uc5ec\uc131\uc774\ub098 \uc870\uc0b0\uc790, \uc0b0\ud30c\ub4e4\uc774 \ud589\ud55c\ub2e4. \ub9c8\ucde8 \uc5c6\uc774 \uc2dc\uc220\ub418\ub294 \uacbd\uc6b0\uac00 \ub9ce\ub2e4.\uc18c\ub140\ub4e4\uc758 \ube44\uba85 \uc18c\ub9ac\uac00 \ub4e4\ub9ac\uc9c0 \uc54a\ub3c4\ub85d, \ud560\ub840 \uc758\uc2dd\uc740 \ub3d9 \ud2b8\uae30 \uc9c1\uc804\uc5d0 \uac70\ud589\ub41c\ub2e4. \uc18c\ub140\ub4e4\uc740 \ub098\uc774\ub4e0 \uc5ec\uc790\ub4e4\uc5d0 \uc758\ud574 \uc6c0\uc9c1\uc77c \uc218 \uc5c6\uac8c \ud3c9\ud3c9\ud55c \uacf3\uc5d0 \ub215\ud600\uc838 \uc2dc\uc220\uc744 \ubc1b\uac8c \ub41c\ub2e4.\ub3c4\uad6c\ub294 \uc0ac\uc6a9 \uc804\ud6c4\uc5d0 \uc18c\ub3c5\ub418\uc9c0 \uc54a\ub294 \uacbd\uc6b0\uac00 \ub9ce\uc73c\uba70 \uba74\ub3c4\ub0a0, \uce7c, \uac00\uc704, \uae68\uc9c4 \uc720\ub9ac \uc870\uac01, \ub0a0\uce74\ub85c\uc6b4 \ub3cc \ub4f1\uc774 \ub3c4\uad6c\ub85c \uc0ac\uc6a9\ub41c\ub2e4. \uc5b4\ub5a4 \uc9c0\uc5ed\uc5d0\uc11c\ub294 \uc774\ube68\uc744 \uc0ac\uc6a9\ud558\uae30\ub3c4 \ud55c\ub2e4.\ud2b9\ud788 \ubd09\uc1c4\uc220\uc758 \uacbd\uc6b0\uc5d0\ub294 \uc544\uce74\uc2dc\uc544 \ub098\ubb34\uc758 \uac00\uc2dc\ub85c \uc0b4\uc5d0 \uad6c\uba4d\uc744 \uc5ec\ub7ec \uac1c \ub6ab\uc5b4 \uadf8 \uad6c\uba4d\uc744 \ud76c\uace0 \uc9c8\uae34 \uc2e4\ub85c \uaff0\ub9e8\ub2e4. \uc5ec\uc131 \uc131\uae30 \uc808\uc81c \uc2dc\uc220\uc774 \ub05d\ub098\uba74 \ub2e4\ub9ac\ub97c \uc6c0\uc9c1\uc774\uc9c0 \ubabb\ud558\ub3c4\ub85d \ubc1c\ubaa9\uc5d0\uc11c \uace8\ubc18\uae4c\uc9c0 \ucc9c\uc73c\ub85c \uaf41\uaf41 \ubb36\uc5b4 \ub193\ub294\ub2e4.\uc0c1\ucc98\uac00 \uc5b4\ub290 \uc815\ub3c4 \ud68c\ubcf5\uc774 \ub420 \ub54c\uae4c\uc9c0 \uc9d1\uc5d0\uc11c \ub5a8\uc5b4\uc838 \uc791\uc740 \uc6c0\ub9c9\uc5d0\uc11c \uc9c0\ub0b4\uac8c \ub41c\ub2e4. \uc0c1\ucc98\uac00 \uc544\ubb3c \ub54c\uae4c\uc9c0 \ub2e4\ub9ac\ub294 \uacc4\uc18d \ubb36\uc5ec \uc788\uac8c \ub41c\ub2e4. \uc774 \uc808\ucc28\ub294 \uc18c\ub140\uc5d0\uac8c \uc5ec\ub7ec \ubd80\uc791\uc6a9\uc744 \uac00\uc838\uc624\uba70 \uc8fd\uc74c\uc744 \ucd08\ub798\ud558\uae30\ub3c4 \ud55c\ub2e4. \ub610\ud55c \uc0ac\ub9dd\ud558\uc9c0 \uc54a\ub294\ub2e4 \ud558\ub354\ub77c\ub3c4 \ub2e4\ub7c9\uc758 \ucd9c\ud608, \ud328\ud608\uc99d, \ud30c\uc0c1\ud48d, \uc1fc\ud06c, \uace0\ud1b5, \uac10\uc5fc, \uc694\ud3d0(Urine retention), \uc778\uadfc \uc870\uc9c1\uc758 \uc0c1\ucc98 \ub4f1\uc774 \uc0dd\uae30\uba70 HIV\uc758 \uc7a0\uc7ac\uc801\uc778 \uac10\uc5fc\ud1b5\ub85c\uac00 \ub420 \uc218 \uc788\ub2e4.", "answer": "\uc544\uce74\uc2dc\uc544 \ub098\ubb34", "sentence": "\uc5b4\ub5a4 \uc9c0\uc5ed\uc5d0\uc11c\ub294 \uc774\ube68\uc744 \uc0ac\uc6a9\ud558\uae30\ub3c4 \ud55c\ub2e4.\ud2b9\ud788 \ubd09\uc1c4\uc220\uc758 \uacbd\uc6b0\uc5d0\ub294 \uc544\uce74\uc2dc\uc544 \ub098\ubb34 \uc758 \uac00\uc2dc\ub85c \uc0b4\uc5d0 \uad6c\uba4d\uc744 \uc5ec\ub7ec \uac1c \ub6ab\uc5b4 \uadf8 \uad6c\uba4d\uc744 \ud76c\uace0 \uc9c8\uae34 \uc2e4\ub85c \uaff0\ub9e8\ub2e4.", "paragraph_sentence": "\ud0dc\uc5b4\ub098\uc11c\ubd80\ud130 \uccab \uc6d4\uacbd\uae4c\uc9c0\uc758 \uc2dc\uac04 \uc911\uc5d0 \uc5ec\uc131\ud560\ub840\ub97c \ubc1b\uac8c \ub418\uc9c0\ub9cc \ub300\ubd80\ubd84 4-8\uc138\uc5d0 \uc5ec\uc131\uc131\uae30\uc808\uc81c\uc220\uc744 \ubc1b\uac8c \ub41c\ub2e4. \uc5ec\uc131 \uc131\uae30\uc758 \uc808\uc81c \uc21c\uc11c\uc758 \uc2e4\uc81c \ubc29\ubc95\uc740 \uc778\uc885 \uc9d1\ub2e8\uacfc \ub3c4\uc2dc \uadf8\ub9ac\uace0 \ub18d\ucd0c\uc9c0\uc5ed \ub4f1\uc5d0 \ub530\ub77c \ub2e4\uc591\ud55c \uc808\ucc28\ub4e4\uc774 \uc788\ub2e4. \uc0ac\ud68c \uacbd\uc81c\uc801 \uc0c1\ud0dc\uc5d0 \ub530\ub77c\uc11c\ub3c4 \ub2e4\ub974\uac8c \ub098\ud0c0\ub09c\ub2e4.\ub3c4\uc2dc \uc9c0\uc5ed\uc5d0\uc11c\ub294 \ubcd1\uc6d0\uc5d0\uc11c\ub3c4 \uc774\ub8e8\uc5b4\uc9c0\uba70 \uc804\ubb38\uc758\ub4e4\uc774 \ub2f4\ub2f9\ud558\uae30\ub3c4 \ud558\uc9c0\ub9cc \uc774 \uc2dc\uc220\uc740 \uc8fc\ub85c \ub098\uc774 \ub9ce\uc740 \uc5ec\uc131\uc774\ub098 \uc870\uc0b0\uc790, \uc0b0\ud30c\ub4e4\uc774 \ud589\ud55c\ub2e4. \ub9c8\ucde8 \uc5c6\uc774 \uc2dc\uc220\ub418\ub294 \uacbd\uc6b0\uac00 \ub9ce\ub2e4.\uc18c\ub140\ub4e4\uc758 \ube44\uba85 \uc18c\ub9ac\uac00 \ub4e4\ub9ac\uc9c0 \uc54a\ub3c4\ub85d, \ud560\ub840 \uc758\uc2dd\uc740 \ub3d9 \ud2b8\uae30 \uc9c1\uc804\uc5d0 \uac70\ud589\ub41c\ub2e4. \uc18c\ub140\ub4e4\uc740 \ub098\uc774\ub4e0 \uc5ec\uc790\ub4e4\uc5d0 \uc758\ud574 \uc6c0\uc9c1\uc77c \uc218 \uc5c6\uac8c \ud3c9\ud3c9\ud55c \uacf3\uc5d0 \ub215\ud600\uc838 \uc2dc\uc220\uc744 \ubc1b\uac8c \ub41c\ub2e4.\ub3c4\uad6c\ub294 \uc0ac\uc6a9 \uc804\ud6c4\uc5d0 \uc18c\ub3c5\ub418\uc9c0 \uc54a\ub294 \uacbd\uc6b0\uac00 \ub9ce\uc73c\uba70 \uba74\ub3c4\ub0a0, \uce7c, \uac00\uc704, \uae68\uc9c4 \uc720\ub9ac \uc870\uac01, \ub0a0\uce74\ub85c\uc6b4 \ub3cc \ub4f1\uc774 \ub3c4\uad6c\ub85c \uc0ac\uc6a9\ub41c\ub2e4. \uc5b4\ub5a4 \uc9c0\uc5ed\uc5d0\uc11c\ub294 \uc774\ube68\uc744 \uc0ac\uc6a9\ud558\uae30\ub3c4 \ud55c\ub2e4.\ud2b9\ud788 \ubd09\uc1c4\uc220\uc758 \uacbd\uc6b0\uc5d0\ub294 \uc544\uce74\uc2dc\uc544 \ub098\ubb34 \uc758 \uac00\uc2dc\ub85c \uc0b4\uc5d0 \uad6c\uba4d\uc744 \uc5ec\ub7ec \uac1c \ub6ab\uc5b4 \uadf8 \uad6c\uba4d\uc744 \ud76c\uace0 \uc9c8\uae34 \uc2e4\ub85c \uaff0\ub9e8\ub2e4. \uc5ec\uc131 \uc131\uae30 \uc808\uc81c \uc2dc\uc220\uc774 \ub05d\ub098\uba74 \ub2e4\ub9ac\ub97c \uc6c0\uc9c1\uc774\uc9c0 \ubabb\ud558\ub3c4\ub85d \ubc1c\ubaa9\uc5d0\uc11c \uace8\ubc18\uae4c\uc9c0 \ucc9c\uc73c\ub85c \uaf41\uaf41 \ubb36\uc5b4 \ub193\ub294\ub2e4.\uc0c1\ucc98\uac00 \uc5b4\ub290 \uc815\ub3c4 \ud68c\ubcf5\uc774 \ub420 \ub54c\uae4c\uc9c0 \uc9d1\uc5d0\uc11c \ub5a8\uc5b4\uc838 \uc791\uc740 \uc6c0\ub9c9\uc5d0\uc11c \uc9c0\ub0b4\uac8c \ub41c\ub2e4. \uc0c1\ucc98\uac00 \uc544\ubb3c \ub54c\uae4c\uc9c0 \ub2e4\ub9ac\ub294 \uacc4\uc18d \ubb36\uc5ec \uc788\uac8c \ub41c\ub2e4. \uc774 \uc808\ucc28\ub294 \uc18c\ub140\uc5d0\uac8c \uc5ec\ub7ec \ubd80\uc791\uc6a9\uc744 \uac00\uc838\uc624\uba70 \uc8fd\uc74c\uc744 \ucd08\ub798\ud558\uae30\ub3c4 \ud55c\ub2e4. \ub610\ud55c \uc0ac\ub9dd\ud558\uc9c0 \uc54a\ub294\ub2e4 \ud558\ub354\ub77c\ub3c4 \ub2e4\ub7c9\uc758 \ucd9c\ud608, \ud328\ud608\uc99d, \ud30c\uc0c1\ud48d, \uc1fc\ud06c, \uace0\ud1b5, \uac10\uc5fc, \uc694\ud3d0(Urine retention), \uc778\uadfc \uc870\uc9c1\uc758 \uc0c1\ucc98 \ub4f1\uc774 \uc0dd\uae30\uba70 HIV\uc758 \uc7a0\uc7ac\uc801\uc778 \uac10\uc5fc\ud1b5\ub85c\uac00 \ub420 \uc218 \uc788\ub2e4.", "paragraph_answer": "\ud0dc\uc5b4\ub098\uc11c\ubd80\ud130 \uccab \uc6d4\uacbd\uae4c\uc9c0\uc758 \uc2dc\uac04 \uc911\uc5d0 \uc5ec\uc131\ud560\ub840\ub97c \ubc1b\uac8c \ub418\uc9c0\ub9cc \ub300\ubd80\ubd84 4-8\uc138\uc5d0 \uc5ec\uc131\uc131\uae30\uc808\uc81c\uc220\uc744 \ubc1b\uac8c \ub41c\ub2e4. \uc5ec\uc131 \uc131\uae30\uc758 \uc808\uc81c \uc21c\uc11c\uc758 \uc2e4\uc81c \ubc29\ubc95\uc740 \uc778\uc885 \uc9d1\ub2e8\uacfc \ub3c4\uc2dc \uadf8\ub9ac\uace0 \ub18d\ucd0c\uc9c0\uc5ed \ub4f1\uc5d0 \ub530\ub77c \ub2e4\uc591\ud55c \uc808\ucc28\ub4e4\uc774 \uc788\ub2e4. \uc0ac\ud68c \uacbd\uc81c\uc801 \uc0c1\ud0dc\uc5d0 \ub530\ub77c\uc11c\ub3c4 \ub2e4\ub974\uac8c \ub098\ud0c0\ub09c\ub2e4.\ub3c4\uc2dc \uc9c0\uc5ed\uc5d0\uc11c\ub294 \ubcd1\uc6d0\uc5d0\uc11c\ub3c4 \uc774\ub8e8\uc5b4\uc9c0\uba70 \uc804\ubb38\uc758\ub4e4\uc774 \ub2f4\ub2f9\ud558\uae30\ub3c4 \ud558\uc9c0\ub9cc \uc774 \uc2dc\uc220\uc740 \uc8fc\ub85c \ub098\uc774 \ub9ce\uc740 \uc5ec\uc131\uc774\ub098 \uc870\uc0b0\uc790, \uc0b0\ud30c\ub4e4\uc774 \ud589\ud55c\ub2e4. \ub9c8\ucde8 \uc5c6\uc774 \uc2dc\uc220\ub418\ub294 \uacbd\uc6b0\uac00 \ub9ce\ub2e4.\uc18c\ub140\ub4e4\uc758 \ube44\uba85 \uc18c\ub9ac\uac00 \ub4e4\ub9ac\uc9c0 \uc54a\ub3c4\ub85d, \ud560\ub840 \uc758\uc2dd\uc740 \ub3d9 \ud2b8\uae30 \uc9c1\uc804\uc5d0 \uac70\ud589\ub41c\ub2e4. \uc18c\ub140\ub4e4\uc740 \ub098\uc774\ub4e0 \uc5ec\uc790\ub4e4\uc5d0 \uc758\ud574 \uc6c0\uc9c1\uc77c \uc218 \uc5c6\uac8c \ud3c9\ud3c9\ud55c \uacf3\uc5d0 \ub215\ud600\uc838 \uc2dc\uc220\uc744 \ubc1b\uac8c \ub41c\ub2e4.\ub3c4\uad6c\ub294 \uc0ac\uc6a9 \uc804\ud6c4\uc5d0 \uc18c\ub3c5\ub418\uc9c0 \uc54a\ub294 \uacbd\uc6b0\uac00 \ub9ce\uc73c\uba70 \uba74\ub3c4\ub0a0, \uce7c, \uac00\uc704, \uae68\uc9c4 \uc720\ub9ac \uc870\uac01, \ub0a0\uce74\ub85c\uc6b4 \ub3cc \ub4f1\uc774 \ub3c4\uad6c\ub85c \uc0ac\uc6a9\ub41c\ub2e4. \uc5b4\ub5a4 \uc9c0\uc5ed\uc5d0\uc11c\ub294 \uc774\ube68\uc744 \uc0ac\uc6a9\ud558\uae30\ub3c4 \ud55c\ub2e4.\ud2b9\ud788 \ubd09\uc1c4\uc220\uc758 \uacbd\uc6b0\uc5d0\ub294 \uc544\uce74\uc2dc\uc544 \ub098\ubb34 \uc758 \uac00\uc2dc\ub85c \uc0b4\uc5d0 \uad6c\uba4d\uc744 \uc5ec\ub7ec \uac1c \ub6ab\uc5b4 \uadf8 \uad6c\uba4d\uc744 \ud76c\uace0 \uc9c8\uae34 \uc2e4\ub85c \uaff0\ub9e8\ub2e4. \uc5ec\uc131 \uc131\uae30 \uc808\uc81c \uc2dc\uc220\uc774 \ub05d\ub098\uba74 \ub2e4\ub9ac\ub97c \uc6c0\uc9c1\uc774\uc9c0 \ubabb\ud558\ub3c4\ub85d \ubc1c\ubaa9\uc5d0\uc11c \uace8\ubc18\uae4c\uc9c0 \ucc9c\uc73c\ub85c \uaf41\uaf41 \ubb36\uc5b4 \ub193\ub294\ub2e4.\uc0c1\ucc98\uac00 \uc5b4\ub290 \uc815\ub3c4 \ud68c\ubcf5\uc774 \ub420 \ub54c\uae4c\uc9c0 \uc9d1\uc5d0\uc11c \ub5a8\uc5b4\uc838 \uc791\uc740 \uc6c0\ub9c9\uc5d0\uc11c \uc9c0\ub0b4\uac8c \ub41c\ub2e4. \uc0c1\ucc98\uac00 \uc544\ubb3c \ub54c\uae4c\uc9c0 \ub2e4\ub9ac\ub294 \uacc4\uc18d \ubb36\uc5ec \uc788\uac8c \ub41c\ub2e4. \uc774 \uc808\ucc28\ub294 \uc18c\ub140\uc5d0\uac8c \uc5ec\ub7ec \ubd80\uc791\uc6a9\uc744 \uac00\uc838\uc624\uba70 \uc8fd\uc74c\uc744 \ucd08\ub798\ud558\uae30\ub3c4 \ud55c\ub2e4. \ub610\ud55c \uc0ac\ub9dd\ud558\uc9c0 \uc54a\ub294\ub2e4 \ud558\ub354\ub77c\ub3c4 \ub2e4\ub7c9\uc758 \ucd9c\ud608, \ud328\ud608\uc99d, \ud30c\uc0c1\ud48d, \uc1fc\ud06c, \uace0\ud1b5, \uac10\uc5fc, \uc694\ud3d0(Urine retention), \uc778\uadfc \uc870\uc9c1\uc758 \uc0c1\ucc98 \ub4f1\uc774 \uc0dd\uae30\uba70 HIV\uc758 \uc7a0\uc7ac\uc801\uc778 \uac10\uc5fc\ud1b5\ub85c\uac00 \ub420 \uc218 \uc788\ub2e4.", "sentence_answer": "\uc5b4\ub5a4 \uc9c0\uc5ed\uc5d0\uc11c\ub294 \uc774\ube68\uc744 \uc0ac\uc6a9\ud558\uae30\ub3c4 \ud55c\ub2e4.\ud2b9\ud788 \ubd09\uc1c4\uc220\uc758 \uacbd\uc6b0\uc5d0\ub294 \uc544\uce74\uc2dc\uc544 \ub098\ubb34 \uc758 \uac00\uc2dc\ub85c \uc0b4\uc5d0 \uad6c\uba4d\uc744 \uc5ec\ub7ec \uac1c \ub6ab\uc5b4 \uadf8 \uad6c\uba4d\uc744 \ud76c\uace0 \uc9c8\uae34 \uc2e4\ub85c \uaff0\ub9e8\ub2e4.", "paragraph_id": "6523816-1-1", "paragraph_question": "question: \uc5ec\uc131 \uc131\uae30 \uc808\uc81c \uc2dc\uc220\uc5d0\uc11c \ubd09\uc1c4\uc220\uc5d0 \uc5b4\ub5a4 \ub098\ubb34\uc758 \uac00\uc2dc\uac00 \uc0b4\uc744 \ub6ab\ub294\ub370 \uc0ac\uc6a9\ub418\ub098?, context: \ud0dc\uc5b4\ub098\uc11c\ubd80\ud130 \uccab \uc6d4\uacbd\uae4c\uc9c0\uc758 \uc2dc\uac04 \uc911\uc5d0 \uc5ec\uc131\ud560\ub840\ub97c \ubc1b\uac8c \ub418\uc9c0\ub9cc \ub300\ubd80\ubd84 4-8\uc138\uc5d0 \uc5ec\uc131\uc131\uae30\uc808\uc81c\uc220\uc744 \ubc1b\uac8c \ub41c\ub2e4. \uc5ec\uc131 \uc131\uae30\uc758 \uc808\uc81c \uc21c\uc11c\uc758 \uc2e4\uc81c \ubc29\ubc95\uc740 \uc778\uc885 \uc9d1\ub2e8\uacfc \ub3c4\uc2dc \uadf8\ub9ac\uace0 \ub18d\ucd0c\uc9c0\uc5ed \ub4f1\uc5d0 \ub530\ub77c \ub2e4\uc591\ud55c \uc808\ucc28\ub4e4\uc774 \uc788\ub2e4. \uc0ac\ud68c \uacbd\uc81c\uc801 \uc0c1\ud0dc\uc5d0 \ub530\ub77c\uc11c\ub3c4 \ub2e4\ub974\uac8c \ub098\ud0c0\ub09c\ub2e4.\ub3c4\uc2dc \uc9c0\uc5ed\uc5d0\uc11c\ub294 \ubcd1\uc6d0\uc5d0\uc11c\ub3c4 \uc774\ub8e8\uc5b4\uc9c0\uba70 \uc804\ubb38\uc758\ub4e4\uc774 \ub2f4\ub2f9\ud558\uae30\ub3c4 \ud558\uc9c0\ub9cc \uc774 \uc2dc\uc220\uc740 \uc8fc\ub85c \ub098\uc774 \ub9ce\uc740 \uc5ec\uc131\uc774\ub098 \uc870\uc0b0\uc790, \uc0b0\ud30c\ub4e4\uc774 \ud589\ud55c\ub2e4. \ub9c8\ucde8 \uc5c6\uc774 \uc2dc\uc220\ub418\ub294 \uacbd\uc6b0\uac00 \ub9ce\ub2e4.\uc18c\ub140\ub4e4\uc758 \ube44\uba85 \uc18c\ub9ac\uac00 \ub4e4\ub9ac\uc9c0 \uc54a\ub3c4\ub85d, \ud560\ub840 \uc758\uc2dd\uc740 \ub3d9 \ud2b8\uae30 \uc9c1\uc804\uc5d0 \uac70\ud589\ub41c\ub2e4. \uc18c\ub140\ub4e4\uc740 \ub098\uc774\ub4e0 \uc5ec\uc790\ub4e4\uc5d0 \uc758\ud574 \uc6c0\uc9c1\uc77c \uc218 \uc5c6\uac8c \ud3c9\ud3c9\ud55c \uacf3\uc5d0 \ub215\ud600\uc838 \uc2dc\uc220\uc744 \ubc1b\uac8c \ub41c\ub2e4.\ub3c4\uad6c\ub294 \uc0ac\uc6a9 \uc804\ud6c4\uc5d0 \uc18c\ub3c5\ub418\uc9c0 \uc54a\ub294 \uacbd\uc6b0\uac00 \ub9ce\uc73c\uba70 \uba74\ub3c4\ub0a0, \uce7c, \uac00\uc704, \uae68\uc9c4 \uc720\ub9ac \uc870\uac01, \ub0a0\uce74\ub85c\uc6b4 \ub3cc \ub4f1\uc774 \ub3c4\uad6c\ub85c \uc0ac\uc6a9\ub41c\ub2e4. \uc5b4\ub5a4 \uc9c0\uc5ed\uc5d0\uc11c\ub294 \uc774\ube68\uc744 \uc0ac\uc6a9\ud558\uae30\ub3c4 \ud55c\ub2e4.\ud2b9\ud788 \ubd09\uc1c4\uc220\uc758 \uacbd\uc6b0\uc5d0\ub294 \uc544\uce74\uc2dc\uc544 \ub098\ubb34\uc758 \uac00\uc2dc\ub85c \uc0b4\uc5d0 \uad6c\uba4d\uc744 \uc5ec\ub7ec \uac1c \ub6ab\uc5b4 \uadf8 \uad6c\uba4d\uc744 \ud76c\uace0 \uc9c8\uae34 \uc2e4\ub85c \uaff0\ub9e8\ub2e4. \uc5ec\uc131 \uc131\uae30 \uc808\uc81c \uc2dc\uc220\uc774 \ub05d\ub098\uba74 \ub2e4\ub9ac\ub97c \uc6c0\uc9c1\uc774\uc9c0 \ubabb\ud558\ub3c4\ub85d \ubc1c\ubaa9\uc5d0\uc11c \uace8\ubc18\uae4c\uc9c0 \ucc9c\uc73c\ub85c \uaf41\uaf41 \ubb36\uc5b4 \ub193\ub294\ub2e4.\uc0c1\ucc98\uac00 \uc5b4\ub290 \uc815\ub3c4 \ud68c\ubcf5\uc774 \ub420 \ub54c\uae4c\uc9c0 \uc9d1\uc5d0\uc11c \ub5a8\uc5b4\uc838 \uc791\uc740 \uc6c0\ub9c9\uc5d0\uc11c \uc9c0\ub0b4\uac8c \ub41c\ub2e4. \uc0c1\ucc98\uac00 \uc544\ubb3c \ub54c\uae4c\uc9c0 \ub2e4\ub9ac\ub294 \uacc4\uc18d \ubb36\uc5ec \uc788\uac8c \ub41c\ub2e4. \uc774 \uc808\ucc28\ub294 \uc18c\ub140\uc5d0\uac8c \uc5ec\ub7ec \ubd80\uc791\uc6a9\uc744 \uac00\uc838\uc624\uba70 \uc8fd\uc74c\uc744 \ucd08\ub798\ud558\uae30\ub3c4 \ud55c\ub2e4. \ub610\ud55c \uc0ac\ub9dd\ud558\uc9c0 \uc54a\ub294\ub2e4 \ud558\ub354\ub77c\ub3c4 \ub2e4\ub7c9\uc758 \ucd9c\ud608, \ud328\ud608\uc99d, \ud30c\uc0c1\ud48d, \uc1fc\ud06c, \uace0\ud1b5, \uac10\uc5fc, \uc694\ud3d0(Urine retention), \uc778\uadfc \uc870\uc9c1\uc758 \uc0c1\ucc98 \ub4f1\uc774 \uc0dd\uae30\uba70 HIV\uc758 \uc7a0\uc7ac\uc801\uc778 \uac10\uc5fc\ud1b5\ub85c\uac00 \ub420 \uc218 \uc788\ub2e4."} {"question": "\uc21c\uc870\uac00 \uc560\ucd08\uc758 \ubb18\ud638\uc778 \uc21c\uc885\uc774 \uc544\ub2c8\ub77c \uc21c\uc870\ub85c \ubd88\ub9b0 \uac83\uc740, \uc21c\uc870\uac00 \ub204\uad6c\ub85c\ubd80\ud130 \ub098\ub77c\ub97c \uc9c0\ucf1c\ub0b8 \uacf5\uc744 \uc778\uc815 \ubc1b\uc544\uc11c\uc778\uac00?", "paragraph": "\uc138\uc870\uc758 \uacbd\uc6b0\ub294 \ubcf8\ub798\ub294 \uc2e0\uc885(\u795e\u5b97), \uc608\uc885(\u777f\u5b97), \uc131\uc885(\u8056\u5b97)\uc911\uc5d0\uc11c \ud55c \uac1c\uc758 \ubb18\ud638\ub97c \uace0\ub974\ub294 \uac83\uc774\uc600\uc73c\ub098, \ud6c4\uacc4\ub97c \uc774\uc740 \uc608\uc885\uc774 \ubd80\uc655\uc758 \uacf5\ub355\uc774 \uac00\ud788 \uc138\uc885\uc744 \ub6f0\uc5b4\ub118\uc73c\uba70 \ud55c\ub098\ub77c\uc5d0\ub3c4 \uc55e\uc5d0\ub294 \uc138\uc885\uacfc, \ud6c4\ub300\uc5d0 \uc138\uc870\uac00 \uc788\uc5c8\ub2e4\ub294 \uace0\uc0ac\ub97c \uc608\ub85c \ub4e4\uba70 \uc138\uc870\ub85c \uc815\ud560 \uac83\uc744 \uac15\ub825\ud788 \uc8fc\uc7a5\ud558\uc600\ub2e4. \uc120\uc870\uc640 \uc601\uc870, \uc21c\uc870\uc758 \uacbd\uc6b0 \ubcf8\ub798 \ubb18\ud638\ub294 \uc120\uc885(\u5ba3\u5b97), \uc601\uc885(\u82f1\u5b97), \uc21c\uc885(\u7d14\u5b97)\uc774\uc5c8\ub2e4. \uadf8\ub7ec\ub098 \uacf5\uc774 \uc788\ub2e4 \ud558\uc5ec \ubaa8\ub450 \uc870\uc758 \uc885\ud638\ub97c \ubc1b\uc544 \uaca9\uc0c1\ub418\uc5c8\ub294\ub370, \uc120\uc870\ub294 \uc784\uc9c4\uc65c\ub780\uacfc \uc815\uc720\uc7ac\ub780 \ub2f9\uc2dc, \ub098\ub77c\ub97c \uc65c\uc801\uc758 \uc190\uc73c\ub85c\ubd80\ud130 \uad6c\ud574\ub0b8 \uacf5\uc73c\ub85c, \uc601\uc870\ub294 \uc624\ub798 \uc7ac\uc704\ud558\uba70 \ube5b\ub098\ub294 \uc5c5\uc801\uc744 \uc313\uc544\uc11c, \uadf8\ub9ac\uace0 \uc21c\uc870\ub294 \uc0ac\ud559\uc778 \ucc9c\uc8fc\uad50\uc758 \uc138\ub85c\ubd80\ud130 \ub098\ub77c\ub97c \uc9c0\ucf1c\ub0c8\ub2e4\ub294 \uacf5\uc744 \uc778\uc815\ubc1b\uc558\uae30 \ub54c\ubb38\uc774\ub2e4. \ubcf8\ub798 \uc911\uc885 \uc5ed\uc2dc \uc544\ub4e4\uc778 \uc778\uc885\uc758 \uac15\ub825\ud55c \uc8fc\uc7a5\uc73c\ub85c \uc911\uc885\uc774 \uc544\ub2cc \uc911\uc870(\u4e2d\u7956)\ub85c \uc815\ud560 \uac83\uc744 \uc6d0\ud588\uc73c\ub098, \uc2e0\ub8cc\ub4e4\uc774 \uc120\uc655\uc740 \uc131\uc885\uc744 \uc774\uc740 \uad70\uc8fc\ub85c\uc11c \uc131\uc885\ubcf4\ub2e4 \ub354 \ub192\uc774 \ubc1b\ub4e4 \uc218 \uc5c6\ub2e4 \ud558\uc5ec \ubb34\uc0b0\ub41c \uc77c\ub3c4 \uc788\uc5c8\ub2e4.", "answer": "\ucc9c\uc8fc\uad50", "sentence": "\uadf8\ub7ec\ub098 \uacf5\uc774 \uc788\ub2e4 \ud558\uc5ec \ubaa8\ub450 \uc870\uc758 \uc885\ud638\ub97c \ubc1b\uc544 \uaca9\uc0c1\ub418\uc5c8\ub294\ub370, \uc120\uc870\ub294 \uc784\uc9c4\uc65c\ub780\uacfc \uc815\uc720\uc7ac\ub780 \ub2f9\uc2dc, \ub098\ub77c\ub97c \uc65c\uc801\uc758 \uc190\uc73c\ub85c\ubd80\ud130 \uad6c\ud574\ub0b8 \uacf5\uc73c\ub85c, \uc601\uc870\ub294 \uc624\ub798 \uc7ac\uc704\ud558\uba70 \ube5b\ub098\ub294 \uc5c5\uc801\uc744 \uc313\uc544\uc11c, \uadf8\ub9ac\uace0 \uc21c\uc870\ub294 \uc0ac\ud559\uc778 \ucc9c\uc8fc\uad50 \uc758 \uc138\ub85c\ubd80\ud130 \ub098\ub77c\ub97c \uc9c0\ucf1c\ub0c8\ub2e4\ub294 \uacf5\uc744 \uc778\uc815\ubc1b\uc558\uae30 \ub54c\ubb38\uc774\ub2e4.", "paragraph_sentence": "\uc138\uc870\uc758 \uacbd\uc6b0\ub294 \ubcf8\ub798\ub294 \uc2e0\uc885(\u795e\u5b97), \uc608\uc885(\u777f\u5b97), \uc131\uc885(\u8056\u5b97)\uc911\uc5d0\uc11c \ud55c \uac1c\uc758 \ubb18\ud638\ub97c \uace0\ub974\ub294 \uac83\uc774\uc600\uc73c\ub098, \ud6c4\uacc4\ub97c \uc774\uc740 \uc608\uc885\uc774 \ubd80\uc655\uc758 \uacf5\ub355\uc774 \uac00\ud788 \uc138\uc885\uc744 \ub6f0\uc5b4\ub118\uc73c\uba70 \ud55c\ub098\ub77c\uc5d0\ub3c4 \uc55e\uc5d0\ub294 \uc138\uc885\uacfc, \ud6c4\ub300\uc5d0 \uc138\uc870\uac00 \uc788\uc5c8\ub2e4\ub294 \uace0\uc0ac\ub97c \uc608\ub85c \ub4e4\uba70 \uc138\uc870\ub85c \uc815\ud560 \uac83\uc744 \uac15\ub825\ud788 \uc8fc\uc7a5\ud558\uc600\ub2e4. \uc120\uc870\uc640 \uc601\uc870, \uc21c\uc870\uc758 \uacbd\uc6b0 \ubcf8\ub798 \ubb18\ud638\ub294 \uc120\uc885(\u5ba3\u5b97), \uc601\uc885(\u82f1\u5b97), \uc21c\uc885(\u7d14\u5b97)\uc774\uc5c8\ub2e4. \uadf8\ub7ec\ub098 \uacf5\uc774 \uc788\ub2e4 \ud558\uc5ec \ubaa8\ub450 \uc870\uc758 \uc885\ud638\ub97c \ubc1b\uc544 \uaca9\uc0c1\ub418\uc5c8\ub294\ub370, \uc120\uc870\ub294 \uc784\uc9c4\uc65c\ub780\uacfc \uc815\uc720\uc7ac\ub780 \ub2f9\uc2dc, \ub098\ub77c\ub97c \uc65c\uc801\uc758 \uc190\uc73c\ub85c\ubd80\ud130 \uad6c\ud574\ub0b8 \uacf5\uc73c\ub85c, \uc601\uc870\ub294 \uc624\ub798 \uc7ac\uc704\ud558\uba70 \ube5b\ub098\ub294 \uc5c5\uc801\uc744 \uc313\uc544\uc11c, \uadf8\ub9ac\uace0 \uc21c\uc870\ub294 \uc0ac\ud559\uc778 \ucc9c\uc8fc\uad50 \uc758 \uc138\ub85c\ubd80\ud130 \ub098\ub77c\ub97c \uc9c0\ucf1c\ub0c8\ub2e4\ub294 \uacf5\uc744 \uc778\uc815\ubc1b\uc558\uae30 \ub54c\ubb38\uc774\ub2e4. \ubcf8\ub798 \uc911\uc885 \uc5ed\uc2dc \uc544\ub4e4\uc778 \uc778\uc885\uc758 \uac15\ub825\ud55c \uc8fc\uc7a5\uc73c\ub85c \uc911\uc885\uc774 \uc544\ub2cc \uc911\uc870(\u4e2d\u7956)\ub85c \uc815\ud560 \uac83\uc744 \uc6d0\ud588\uc73c\ub098, \uc2e0\ub8cc\ub4e4\uc774 \uc120\uc655\uc740 \uc131\uc885\uc744 \uc774\uc740 \uad70\uc8fc\ub85c\uc11c \uc131\uc885\ubcf4\ub2e4 \ub354 \ub192\uc774 \ubc1b\ub4e4 \uc218 \uc5c6\ub2e4 \ud558\uc5ec \ubb34\uc0b0\ub41c \uc77c\ub3c4 \uc788\uc5c8\ub2e4.", "paragraph_answer": "\uc138\uc870\uc758 \uacbd\uc6b0\ub294 \ubcf8\ub798\ub294 \uc2e0\uc885(\u795e\u5b97), \uc608\uc885(\u777f\u5b97), \uc131\uc885(\u8056\u5b97)\uc911\uc5d0\uc11c \ud55c \uac1c\uc758 \ubb18\ud638\ub97c \uace0\ub974\ub294 \uac83\uc774\uc600\uc73c\ub098, \ud6c4\uacc4\ub97c \uc774\uc740 \uc608\uc885\uc774 \ubd80\uc655\uc758 \uacf5\ub355\uc774 \uac00\ud788 \uc138\uc885\uc744 \ub6f0\uc5b4\ub118\uc73c\uba70 \ud55c\ub098\ub77c\uc5d0\ub3c4 \uc55e\uc5d0\ub294 \uc138\uc885\uacfc, \ud6c4\ub300\uc5d0 \uc138\uc870\uac00 \uc788\uc5c8\ub2e4\ub294 \uace0\uc0ac\ub97c \uc608\ub85c \ub4e4\uba70 \uc138\uc870\ub85c \uc815\ud560 \uac83\uc744 \uac15\ub825\ud788 \uc8fc\uc7a5\ud558\uc600\ub2e4. \uc120\uc870\uc640 \uc601\uc870, \uc21c\uc870\uc758 \uacbd\uc6b0 \ubcf8\ub798 \ubb18\ud638\ub294 \uc120\uc885(\u5ba3\u5b97), \uc601\uc885(\u82f1\u5b97), \uc21c\uc885(\u7d14\u5b97)\uc774\uc5c8\ub2e4. \uadf8\ub7ec\ub098 \uacf5\uc774 \uc788\ub2e4 \ud558\uc5ec \ubaa8\ub450 \uc870\uc758 \uc885\ud638\ub97c \ubc1b\uc544 \uaca9\uc0c1\ub418\uc5c8\ub294\ub370, \uc120\uc870\ub294 \uc784\uc9c4\uc65c\ub780\uacfc \uc815\uc720\uc7ac\ub780 \ub2f9\uc2dc, \ub098\ub77c\ub97c \uc65c\uc801\uc758 \uc190\uc73c\ub85c\ubd80\ud130 \uad6c\ud574\ub0b8 \uacf5\uc73c\ub85c, \uc601\uc870\ub294 \uc624\ub798 \uc7ac\uc704\ud558\uba70 \ube5b\ub098\ub294 \uc5c5\uc801\uc744 \uc313\uc544\uc11c, \uadf8\ub9ac\uace0 \uc21c\uc870\ub294 \uc0ac\ud559\uc778 \ucc9c\uc8fc\uad50 \uc758 \uc138\ub85c\ubd80\ud130 \ub098\ub77c\ub97c \uc9c0\ucf1c\ub0c8\ub2e4\ub294 \uacf5\uc744 \uc778\uc815\ubc1b\uc558\uae30 \ub54c\ubb38\uc774\ub2e4. \ubcf8\ub798 \uc911\uc885 \uc5ed\uc2dc \uc544\ub4e4\uc778 \uc778\uc885\uc758 \uac15\ub825\ud55c \uc8fc\uc7a5\uc73c\ub85c \uc911\uc885\uc774 \uc544\ub2cc \uc911\uc870(\u4e2d\u7956)\ub85c \uc815\ud560 \uac83\uc744 \uc6d0\ud588\uc73c\ub098, \uc2e0\ub8cc\ub4e4\uc774 \uc120\uc655\uc740 \uc131\uc885\uc744 \uc774\uc740 \uad70\uc8fc\ub85c\uc11c \uc131\uc885\ubcf4\ub2e4 \ub354 \ub192\uc774 \ubc1b\ub4e4 \uc218 \uc5c6\ub2e4 \ud558\uc5ec \ubb34\uc0b0\ub41c \uc77c\ub3c4 \uc788\uc5c8\ub2e4.", "sentence_answer": "\uadf8\ub7ec\ub098 \uacf5\uc774 \uc788\ub2e4 \ud558\uc5ec \ubaa8\ub450 \uc870\uc758 \uc885\ud638\ub97c \ubc1b\uc544 \uaca9\uc0c1\ub418\uc5c8\ub294\ub370, \uc120\uc870\ub294 \uc784\uc9c4\uc65c\ub780\uacfc \uc815\uc720\uc7ac\ub780 \ub2f9\uc2dc, \ub098\ub77c\ub97c \uc65c\uc801\uc758 \uc190\uc73c\ub85c\ubd80\ud130 \uad6c\ud574\ub0b8 \uacf5\uc73c\ub85c, \uc601\uc870\ub294 \uc624\ub798 \uc7ac\uc704\ud558\uba70 \ube5b\ub098\ub294 \uc5c5\uc801\uc744 \uc313\uc544\uc11c, \uadf8\ub9ac\uace0 \uc21c\uc870\ub294 \uc0ac\ud559\uc778 \ucc9c\uc8fc\uad50 \uc758 \uc138\ub85c\ubd80\ud130 \ub098\ub77c\ub97c \uc9c0\ucf1c\ub0c8\ub2e4\ub294 \uacf5\uc744 \uc778\uc815\ubc1b\uc558\uae30 \ub54c\ubb38\uc774\ub2e4.", "paragraph_id": "6504692-17-1", "paragraph_question": "question: \uc21c\uc870\uac00 \uc560\ucd08\uc758 \ubb18\ud638\uc778 \uc21c\uc885\uc774 \uc544\ub2c8\ub77c \uc21c\uc870\ub85c \ubd88\ub9b0 \uac83\uc740, \uc21c\uc870\uac00 \ub204\uad6c\ub85c\ubd80\ud130 \ub098\ub77c\ub97c \uc9c0\ucf1c\ub0b8 \uacf5\uc744 \uc778\uc815 \ubc1b\uc544\uc11c\uc778\uac00?, context: \uc138\uc870\uc758 \uacbd\uc6b0\ub294 \ubcf8\ub798\ub294 \uc2e0\uc885(\u795e\u5b97), \uc608\uc885(\u777f\u5b97), \uc131\uc885(\u8056\u5b97)\uc911\uc5d0\uc11c \ud55c \uac1c\uc758 \ubb18\ud638\ub97c \uace0\ub974\ub294 \uac83\uc774\uc600\uc73c\ub098, \ud6c4\uacc4\ub97c \uc774\uc740 \uc608\uc885\uc774 \ubd80\uc655\uc758 \uacf5\ub355\uc774 \uac00\ud788 \uc138\uc885\uc744 \ub6f0\uc5b4\ub118\uc73c\uba70 \ud55c\ub098\ub77c\uc5d0\ub3c4 \uc55e\uc5d0\ub294 \uc138\uc885\uacfc, \ud6c4\ub300\uc5d0 \uc138\uc870\uac00 \uc788\uc5c8\ub2e4\ub294 \uace0\uc0ac\ub97c \uc608\ub85c \ub4e4\uba70 \uc138\uc870\ub85c \uc815\ud560 \uac83\uc744 \uac15\ub825\ud788 \uc8fc\uc7a5\ud558\uc600\ub2e4. \uc120\uc870\uc640 \uc601\uc870, \uc21c\uc870\uc758 \uacbd\uc6b0 \ubcf8\ub798 \ubb18\ud638\ub294 \uc120\uc885(\u5ba3\u5b97), \uc601\uc885(\u82f1\u5b97), \uc21c\uc885(\u7d14\u5b97)\uc774\uc5c8\ub2e4. \uadf8\ub7ec\ub098 \uacf5\uc774 \uc788\ub2e4 \ud558\uc5ec \ubaa8\ub450 \uc870\uc758 \uc885\ud638\ub97c \ubc1b\uc544 \uaca9\uc0c1\ub418\uc5c8\ub294\ub370, \uc120\uc870\ub294 \uc784\uc9c4\uc65c\ub780\uacfc \uc815\uc720\uc7ac\ub780 \ub2f9\uc2dc, \ub098\ub77c\ub97c \uc65c\uc801\uc758 \uc190\uc73c\ub85c\ubd80\ud130 \uad6c\ud574\ub0b8 \uacf5\uc73c\ub85c, \uc601\uc870\ub294 \uc624\ub798 \uc7ac\uc704\ud558\uba70 \ube5b\ub098\ub294 \uc5c5\uc801\uc744 \uc313\uc544\uc11c, \uadf8\ub9ac\uace0 \uc21c\uc870\ub294 \uc0ac\ud559\uc778 \ucc9c\uc8fc\uad50\uc758 \uc138\ub85c\ubd80\ud130 \ub098\ub77c\ub97c \uc9c0\ucf1c\ub0c8\ub2e4\ub294 \uacf5\uc744 \uc778\uc815\ubc1b\uc558\uae30 \ub54c\ubb38\uc774\ub2e4. \ubcf8\ub798 \uc911\uc885 \uc5ed\uc2dc \uc544\ub4e4\uc778 \uc778\uc885\uc758 \uac15\ub825\ud55c \uc8fc\uc7a5\uc73c\ub85c \uc911\uc885\uc774 \uc544\ub2cc \uc911\uc870(\u4e2d\u7956)\ub85c \uc815\ud560 \uac83\uc744 \uc6d0\ud588\uc73c\ub098, \uc2e0\ub8cc\ub4e4\uc774 \uc120\uc655\uc740 \uc131\uc885\uc744 \uc774\uc740 \uad70\uc8fc\ub85c\uc11c \uc131\uc885\ubcf4\ub2e4 \ub354 \ub192\uc774 \ubc1b\ub4e4 \uc218 \uc5c6\ub2e4 \ud558\uc5ec \ubb34\uc0b0\ub41c \uc77c\ub3c4 \uc788\uc5c8\ub2e4."} {"question": "1996\ub144 \ub2f9\uc2dc \uacbd\ud76c\ub300\ud559\uad50 \ubb34\uc6a9\uacfc\uc5d0 \uc7ac\ud559\uc911\uc774 \ub358 \uae40\uc774\uc9c0\ub294 \uc5b4\ub5a0\ud55c \uacbd\ub85c\ub97c \ud1b5\ud558\uc5ec \ubca0\uc774\ube44\ubcf5\uc2a4\uc5d0 \uce90\uc2a4\ud305 \ub418\uc5c8\ub294\uac00?", "paragraph": "1996\ub144, \uad6d\ub0b4\uac00\uc218\ub4e4\uc758 \uacf5\uc5f0 \uae30\ud68d\uc744 \ub2f4\ub2f9\ud558\ub358 \ub4f1\ub8e1 \uae30\ud68d(\ud604 DR\ubba4\uc9c1)\uc758 \ub300\ud45c \uc724\ub4f1\ub8e1\uc740 \ub2f9\uc2dc \uc601\uad6d\uc5d0\uc11c\ubd80\ud130 \ubbf8\uad6d\uae4c\uc9c0 \uc804\uc138\uacc4\uc801\uc73c\ub85c \uc120\ud48d\uc801\uc778 \uc778\uae30\ub97c \ub04c\ub358 \uac78 \uadf8\ub8f9 \uc2a4\ud30c\uc774\uc2a4 \uac78\uc2a4\ub97c \ud45c\ubc29\ud558\ub294 \uac78\uadf8\ub8f9\uc744 \uae30\ud68d\ud55c\ub2e4. \uba3c\uc800 \ubd80\uc0b0\uc5d0\uc11c \ubaa8\ub378\ub85c \ud65c\ub3d9\ud558\ub358 \uc815\uc2dc\uc6b4\uacfc \uc815\ud604\uc804\uc744 \uce90\uc2a4\ud305\ud558\uace0 \uce98\ub9ac\ud3ec\ub2c8\uc544 \ucd9c\uc2e0\uc774\uc790 \uc791\uace1\uac00 \uc2e0\uc131\ud638\uc758 \uce5c\ucc99 \uad00\uacc4\uc600\ub358 \ucc28\uc720\ubbf8\ub97c \uc18c\uac1c\ubc1b\uc544 \ud300\uc758 \ub9ac\ub4dc \ubcf4\uceec\ub85c \ud569\ub958\uc2dc\ud0a8\ub2e4. \uadf8\ub9ac\uace0 \uacbd\ud76c\ub300\ud559\uad50\uc5d0\uc11c \ubb34\uc6a9\uacfc\ub97c \uc7ac\ud559 \uc911\uc774\ub358 \uae40\uc774\uc9c0\ub97c \uae38\uac70\ub9ac \uce90\uc2a4\ud305\uc744 \ud1b5\ud574 \ud569\ub958\uc2dc\ud0a8 \ud6c4 \uc11c\uc6b8 \ub3d9\ub300\ubb38\uc5d0\uc11c \ub958\uc2dc\uc6d0\uc758 \ub77c\ub514\uc624 \uacf5\uac1c\ubc29\uc1a1\uc744 \uad6c\uacbd\ud558\uace0\uc788\ub358 \uc774\ud76c\uc9c4\uc744 \ubc29\uc1a1\uc778 \uc724\uc815\uc218\uc640 \uac00\uc218 \uc591\ud30c\uc758 \ub9e4\ub2c8\uc800, \uadf8\ub9ac\uace0 DR \ubba4\uc9c1 \uc724\ub4f1\ub8e1 \ub300\ud45c\uac00 \ub85c\ub4dc \uce90\uc2a4\ud305\ud558\uba74\uc11c \ub2e4\uc12f \uba85\uc758 \ub77c\uc778\uc5c5\uc744 \uc644\uc131\uc2dc\ud0a8\ub2e4.", "answer": "\uae38\uac70\ub9ac \uce90\uc2a4\ud305", "sentence": "\uae40\uc774\uc9c0\ub97c \uae38\uac70\ub9ac \uce90\uc2a4\ud305 \uc744 \ud1b5\ud574 \ud569\ub958\uc2dc\ud0a8 \ud6c4 \uc11c\uc6b8 \ub3d9\ub300\ubb38\uc5d0\uc11c \ub958\uc2dc\uc6d0\uc758 \ub77c\ub514\uc624 \uacf5\uac1c\ubc29\uc1a1\uc744 \uad6c\uacbd\ud558\uace0\uc788\ub358 \uc774\ud76c\uc9c4\uc744 \ubc29\uc1a1\uc778 \uc724\uc815\uc218\uc640 \uac00\uc218 \uc591\ud30c\uc758 \ub9e4\ub2c8\uc800, \uadf8\ub9ac\uace0 DR \ubba4\uc9c1 \uc724\ub4f1\ub8e1 \ub300\ud45c\uac00 \ub85c\ub4dc \uce90\uc2a4\ud305\ud558\uba74\uc11c \ub2e4\uc12f \uba85\uc758 \ub77c\uc778\uc5c5\uc744 \uc644\uc131\uc2dc\ud0a8\ub2e4.", "paragraph_sentence": "1996\ub144, \uad6d\ub0b4\uac00\uc218\ub4e4\uc758 \uacf5\uc5f0 \uae30\ud68d\uc744 \ub2f4\ub2f9\ud558\ub358 \ub4f1\ub8e1 \uae30\ud68d(\ud604 DR\ubba4\uc9c1)\uc758 \ub300\ud45c \uc724\ub4f1\ub8e1\uc740 \ub2f9\uc2dc \uc601\uad6d\uc5d0\uc11c\ubd80\ud130 \ubbf8\uad6d\uae4c\uc9c0 \uc804\uc138\uacc4\uc801\uc73c\ub85c \uc120\ud48d\uc801\uc778 \uc778\uae30\ub97c \ub04c\ub358 \uac78 \uadf8\ub8f9 \uc2a4\ud30c\uc774\uc2a4 \uac78\uc2a4\ub97c \ud45c\ubc29\ud558\ub294 \uac78\uadf8\ub8f9\uc744 \uae30\ud68d\ud55c\ub2e4. \uba3c\uc800 \ubd80\uc0b0\uc5d0\uc11c \ubaa8\ub378\ub85c \ud65c\ub3d9\ud558\ub358 \uc815\uc2dc\uc6b4\uacfc \uc815\ud604\uc804\uc744 \uce90\uc2a4\ud305\ud558\uace0 \uce98\ub9ac\ud3ec\ub2c8\uc544 \ucd9c\uc2e0\uc774\uc790 \uc791\uace1\uac00 \uc2e0\uc131\ud638\uc758 \uce5c\ucc99 \uad00\uacc4\uc600\ub358 \ucc28\uc720\ubbf8\ub97c \uc18c\uac1c\ubc1b\uc544 \ud300\uc758 \ub9ac\ub4dc \ubcf4\uceec\ub85c \ud569\ub958\uc2dc\ud0a8\ub2e4. \uadf8\ub9ac\uace0 \uacbd\ud76c\ub300\ud559\uad50\uc5d0\uc11c \ubb34\uc6a9\uacfc\ub97c \uc7ac\ud559 \uc911\uc774\ub358 \uae40\uc774\uc9c0\ub97c \uae38\uac70\ub9ac \uce90\uc2a4\ud305 \uc744 \ud1b5\ud574 \ud569\ub958\uc2dc\ud0a8 \ud6c4 \uc11c\uc6b8 \ub3d9\ub300\ubb38\uc5d0\uc11c \ub958\uc2dc\uc6d0\uc758 \ub77c\ub514\uc624 \uacf5\uac1c\ubc29\uc1a1\uc744 \uad6c\uacbd\ud558\uace0\uc788\ub358 \uc774\ud76c\uc9c4\uc744 \ubc29\uc1a1\uc778 \uc724\uc815\uc218\uc640 \uac00\uc218 \uc591\ud30c\uc758 \ub9e4\ub2c8\uc800, \uadf8\ub9ac\uace0 DR \ubba4\uc9c1 \uc724\ub4f1\ub8e1 \ub300\ud45c\uac00 \ub85c\ub4dc \uce90\uc2a4\ud305\ud558\uba74\uc11c \ub2e4\uc12f \uba85\uc758 \ub77c\uc778\uc5c5\uc744 \uc644\uc131\uc2dc\ud0a8\ub2e4. ", "paragraph_answer": "1996\ub144, \uad6d\ub0b4\uac00\uc218\ub4e4\uc758 \uacf5\uc5f0 \uae30\ud68d\uc744 \ub2f4\ub2f9\ud558\ub358 \ub4f1\ub8e1 \uae30\ud68d(\ud604 DR\ubba4\uc9c1)\uc758 \ub300\ud45c \uc724\ub4f1\ub8e1\uc740 \ub2f9\uc2dc \uc601\uad6d\uc5d0\uc11c\ubd80\ud130 \ubbf8\uad6d\uae4c\uc9c0 \uc804\uc138\uacc4\uc801\uc73c\ub85c \uc120\ud48d\uc801\uc778 \uc778\uae30\ub97c \ub04c\ub358 \uac78 \uadf8\ub8f9 \uc2a4\ud30c\uc774\uc2a4 \uac78\uc2a4\ub97c \ud45c\ubc29\ud558\ub294 \uac78\uadf8\ub8f9\uc744 \uae30\ud68d\ud55c\ub2e4. \uba3c\uc800 \ubd80\uc0b0\uc5d0\uc11c \ubaa8\ub378\ub85c \ud65c\ub3d9\ud558\ub358 \uc815\uc2dc\uc6b4\uacfc \uc815\ud604\uc804\uc744 \uce90\uc2a4\ud305\ud558\uace0 \uce98\ub9ac\ud3ec\ub2c8\uc544 \ucd9c\uc2e0\uc774\uc790 \uc791\uace1\uac00 \uc2e0\uc131\ud638\uc758 \uce5c\ucc99 \uad00\uacc4\uc600\ub358 \ucc28\uc720\ubbf8\ub97c \uc18c\uac1c\ubc1b\uc544 \ud300\uc758 \ub9ac\ub4dc \ubcf4\uceec\ub85c \ud569\ub958\uc2dc\ud0a8\ub2e4. \uadf8\ub9ac\uace0 \uacbd\ud76c\ub300\ud559\uad50\uc5d0\uc11c \ubb34\uc6a9\uacfc\ub97c \uc7ac\ud559 \uc911\uc774\ub358 \uae40\uc774\uc9c0\ub97c \uae38\uac70\ub9ac \uce90\uc2a4\ud305 \uc744 \ud1b5\ud574 \ud569\ub958\uc2dc\ud0a8 \ud6c4 \uc11c\uc6b8 \ub3d9\ub300\ubb38\uc5d0\uc11c \ub958\uc2dc\uc6d0\uc758 \ub77c\ub514\uc624 \uacf5\uac1c\ubc29\uc1a1\uc744 \uad6c\uacbd\ud558\uace0\uc788\ub358 \uc774\ud76c\uc9c4\uc744 \ubc29\uc1a1\uc778 \uc724\uc815\uc218\uc640 \uac00\uc218 \uc591\ud30c\uc758 \ub9e4\ub2c8\uc800, \uadf8\ub9ac\uace0 DR \ubba4\uc9c1 \uc724\ub4f1\ub8e1 \ub300\ud45c\uac00 \ub85c\ub4dc \uce90\uc2a4\ud305\ud558\uba74\uc11c \ub2e4\uc12f \uba85\uc758 \ub77c\uc778\uc5c5\uc744 \uc644\uc131\uc2dc\ud0a8\ub2e4.", "sentence_answer": "\uae40\uc774\uc9c0\ub97c \uae38\uac70\ub9ac \uce90\uc2a4\ud305 \uc744 \ud1b5\ud574 \ud569\ub958\uc2dc\ud0a8 \ud6c4 \uc11c\uc6b8 \ub3d9\ub300\ubb38\uc5d0\uc11c \ub958\uc2dc\uc6d0\uc758 \ub77c\ub514\uc624 \uacf5\uac1c\ubc29\uc1a1\uc744 \uad6c\uacbd\ud558\uace0\uc788\ub358 \uc774\ud76c\uc9c4\uc744 \ubc29\uc1a1\uc778 \uc724\uc815\uc218\uc640 \uac00\uc218 \uc591\ud30c\uc758 \ub9e4\ub2c8\uc800, \uadf8\ub9ac\uace0 DR \ubba4\uc9c1 \uc724\ub4f1\ub8e1 \ub300\ud45c\uac00 \ub85c\ub4dc \uce90\uc2a4\ud305\ud558\uba74\uc11c \ub2e4\uc12f \uba85\uc758 \ub77c\uc778\uc5c5\uc744 \uc644\uc131\uc2dc\ud0a8\ub2e4.", "paragraph_id": "6110374-3-1", "paragraph_question": "question: 1996\ub144 \ub2f9\uc2dc \uacbd\ud76c\ub300\ud559\uad50 \ubb34\uc6a9\uacfc\uc5d0 \uc7ac\ud559\uc911\uc774 \ub358 \uae40\uc774\uc9c0\ub294 \uc5b4\ub5a0\ud55c \uacbd\ub85c\ub97c \ud1b5\ud558\uc5ec \ubca0\uc774\ube44\ubcf5\uc2a4\uc5d0 \uce90\uc2a4\ud305 \ub418\uc5c8\ub294\uac00?, context: 1996\ub144, \uad6d\ub0b4\uac00\uc218\ub4e4\uc758 \uacf5\uc5f0 \uae30\ud68d\uc744 \ub2f4\ub2f9\ud558\ub358 \ub4f1\ub8e1 \uae30\ud68d(\ud604 DR\ubba4\uc9c1)\uc758 \ub300\ud45c \uc724\ub4f1\ub8e1\uc740 \ub2f9\uc2dc \uc601\uad6d\uc5d0\uc11c\ubd80\ud130 \ubbf8\uad6d\uae4c\uc9c0 \uc804\uc138\uacc4\uc801\uc73c\ub85c \uc120\ud48d\uc801\uc778 \uc778\uae30\ub97c \ub04c\ub358 \uac78 \uadf8\ub8f9 \uc2a4\ud30c\uc774\uc2a4 \uac78\uc2a4\ub97c \ud45c\ubc29\ud558\ub294 \uac78\uadf8\ub8f9\uc744 \uae30\ud68d\ud55c\ub2e4. \uba3c\uc800 \ubd80\uc0b0\uc5d0\uc11c \ubaa8\ub378\ub85c \ud65c\ub3d9\ud558\ub358 \uc815\uc2dc\uc6b4\uacfc \uc815\ud604\uc804\uc744 \uce90\uc2a4\ud305\ud558\uace0 \uce98\ub9ac\ud3ec\ub2c8\uc544 \ucd9c\uc2e0\uc774\uc790 \uc791\uace1\uac00 \uc2e0\uc131\ud638\uc758 \uce5c\ucc99 \uad00\uacc4\uc600\ub358 \ucc28\uc720\ubbf8\ub97c \uc18c\uac1c\ubc1b\uc544 \ud300\uc758 \ub9ac\ub4dc \ubcf4\uceec\ub85c \ud569\ub958\uc2dc\ud0a8\ub2e4. \uadf8\ub9ac\uace0 \uacbd\ud76c\ub300\ud559\uad50\uc5d0\uc11c \ubb34\uc6a9\uacfc\ub97c \uc7ac\ud559 \uc911\uc774\ub358 \uae40\uc774\uc9c0\ub97c \uae38\uac70\ub9ac \uce90\uc2a4\ud305\uc744 \ud1b5\ud574 \ud569\ub958\uc2dc\ud0a8 \ud6c4 \uc11c\uc6b8 \ub3d9\ub300\ubb38\uc5d0\uc11c \ub958\uc2dc\uc6d0\uc758 \ub77c\ub514\uc624 \uacf5\uac1c\ubc29\uc1a1\uc744 \uad6c\uacbd\ud558\uace0\uc788\ub358 \uc774\ud76c\uc9c4\uc744 \ubc29\uc1a1\uc778 \uc724\uc815\uc218\uc640 \uac00\uc218 \uc591\ud30c\uc758 \ub9e4\ub2c8\uc800, \uadf8\ub9ac\uace0 DR \ubba4\uc9c1 \uc724\ub4f1\ub8e1 \ub300\ud45c\uac00 \ub85c\ub4dc \uce90\uc2a4\ud305\ud558\uba74\uc11c \ub2e4\uc12f \uba85\uc758 \ub77c\uc778\uc5c5\uc744 \uc644\uc131\uc2dc\ud0a8\ub2e4."} {"question": "\ubc1c\ub80c\uc288\ud0c0\uc778\uc744 \uc8fd\uc778 \ubb34\uae30\ub294?", "paragraph": "\uc0c1\ub2f9\ud55c \uc7ac\uc815\uc801\u00b7\uc601\ud1a0\uc801 \ubcf4\uc0c1\uc744 \uae30\ub300\ud558\uba74\uc11c, \uc0ac\ud1f4\uc11c\ub97c \uc81c\ucd9c\ud55c \ubc1c\ub80c\uc288\ud0c0\uc778\uc758 \ub9c8\uc9c0\ub9c9 \uc804\ud5a5\uc774 \ube48\uc5d0\uc11c \ubb34\uc2dc\ub2f9\ud588\uc73c\uba70, \ubc1c\ub80c\uc288\ud0c0\uc778\uc774 32\uba85\uc758 \uc5f0\ub300\uc7a5\ub4e4\uacfc \ud568\uaed8 \ud669\uc81c\uc5d0\uac8c \ucda9\uc131\uc744 \ub9f9\uc138\ud55c \uc81c2\uc758 \ud544\uc820 \uc120\uc5b8(1634\ub144 2\uc6d4 20\uc77c)\ub3c4 \uc5ed\uc2dc \ub9c8\ucc2c\uac00\uc9c0\uc600\ub2e4. \ud504\ub77c\ud558\uac00 \ud669\uc81c\ub97c \uc9c0\uc9c0\ud55c\ub2e4\uace0 \uc120\uc5b8\ud588\uc744\ub54c, \ubc1c\ub80c\uc288\ud0c0\uc778\uc740 \uc2a4\uc6e8\ub374\uc778\uacfc \uc791\uc13c\uc778\ub4e4\uacfc \ud798\uc744 \ud569\uccd0\ubcf4\uaca0\ub2e4\ub294 \uc0dd\uac01\uc73c\ub85c \ud544\uc820\uc744 \ub5a0\ub098 \uc5d0\uac70(\uc624\ub298\ub0a0 \uccb4\ucf54\uc758 \ud5e4\ud504)\ub85c \ud5a5\ud588\ub2e4. 2\uc6d4 25\uc77c, \uac70\uae30\uc11c \uadf8\uc640 \ud2b8\ub85c\uce20\uce74, \uc77c\ub85c, \uadf8 \ubc16\uc758 \uc7a5\uad50\ub4e4\uc740 \uc544\uc77c\ub79c\ub4dc\uc778 \uc6d4\ud130 \ubc84\ud2c0\ub7ec \uc7a5\uad70, \uadf8\ub9ac\uace0 \uc2a4\ucf54\ud2c0\ub79c\ub4dc \ucd9c\uc2e0 \uc5f0\ub300\uc7a5\uc778 \uc6d4\ud130 \ub808\uc2ac\ub9ac\uc640 \uc874 \uace0\ub4e0\uc774 \uc9c0\ud718\ud558\ub294 \ubcd1\uc0ac\ub4e4\uc5d0\uac8c \uc0b4\ud574\ub418\uc5c8\ub2e4. \uc601\uad6d\uc778 \ub300\uc7a5 \uc6d4\ud130 \ub370\ubc84\ub8e8\uac00 \uc7a0\uacb0\uc5d0 \uc77c\uc5b4\ub098 \uc0b4\ub824\ub2ec\ub77c\uace0 \uc560\uc6d0\ud558\ub294 \ubc1c\ub80c\uc288\ud0c0\uc778\uc744 \ubbf8\ub298\ucc3d\uc73c\ub85c \uc0b4\ud574\ud588\ub2e4.", "answer": "\ubbf8\ub298\ucc3d", "sentence": "\uc601\uad6d\uc778 \ub300\uc7a5 \uc6d4\ud130 \ub370\ubc84\ub8e8\uac00 \uc7a0\uacb0\uc5d0 \uc77c\uc5b4\ub098 \uc0b4\ub824\ub2ec\ub77c\uace0 \uc560\uc6d0\ud558\ub294 \ubc1c\ub80c\uc288\ud0c0\uc778\uc744 \ubbf8\ub298\ucc3d \uc73c\ub85c \uc0b4\ud574\ud588\ub2e4.", "paragraph_sentence": "\uc0c1\ub2f9\ud55c \uc7ac\uc815\uc801\u00b7\uc601\ud1a0\uc801 \ubcf4\uc0c1\uc744 \uae30\ub300\ud558\uba74\uc11c, \uc0ac\ud1f4\uc11c\ub97c \uc81c\ucd9c\ud55c \ubc1c\ub80c\uc288\ud0c0\uc778\uc758 \ub9c8\uc9c0\ub9c9 \uc804\ud5a5\uc774 \ube48\uc5d0\uc11c \ubb34\uc2dc\ub2f9\ud588\uc73c\uba70, \ubc1c\ub80c\uc288\ud0c0\uc778\uc774 32\uba85\uc758 \uc5f0\ub300\uc7a5\ub4e4\uacfc \ud568\uaed8 \ud669\uc81c\uc5d0\uac8c \ucda9\uc131\uc744 \ub9f9\uc138\ud55c \uc81c2\uc758 \ud544\uc820 \uc120\uc5b8(1634\ub144 2\uc6d4 20\uc77c)\ub3c4 \uc5ed\uc2dc \ub9c8\ucc2c\uac00\uc9c0\uc600\ub2e4. \ud504\ub77c\ud558\uac00 \ud669\uc81c\ub97c \uc9c0\uc9c0\ud55c\ub2e4\uace0 \uc120\uc5b8\ud588\uc744\ub54c, \ubc1c\ub80c\uc288\ud0c0\uc778\uc740 \uc2a4\uc6e8\ub374\uc778\uacfc \uc791\uc13c\uc778\ub4e4\uacfc \ud798\uc744 \ud569\uccd0\ubcf4\uaca0\ub2e4\ub294 \uc0dd\uac01\uc73c\ub85c \ud544\uc820\uc744 \ub5a0\ub098 \uc5d0\uac70(\uc624\ub298\ub0a0 \uccb4\ucf54\uc758 \ud5e4\ud504)\ub85c \ud5a5\ud588\ub2e4. 2\uc6d4 25\uc77c, \uac70\uae30\uc11c \uadf8\uc640 \ud2b8\ub85c\uce20\uce74, \uc77c\ub85c, \uadf8 \ubc16\uc758 \uc7a5\uad50\ub4e4\uc740 \uc544\uc77c\ub79c\ub4dc\uc778 \uc6d4\ud130 \ubc84\ud2c0\ub7ec \uc7a5\uad70, \uadf8\ub9ac\uace0 \uc2a4\ucf54\ud2c0\ub79c\ub4dc \ucd9c\uc2e0 \uc5f0\ub300\uc7a5\uc778 \uc6d4\ud130 \ub808\uc2ac\ub9ac\uc640 \uc874 \uace0\ub4e0\uc774 \uc9c0\ud718\ud558\ub294 \ubcd1\uc0ac\ub4e4\uc5d0\uac8c \uc0b4\ud574\ub418\uc5c8\ub2e4. \uc601\uad6d\uc778 \ub300\uc7a5 \uc6d4\ud130 \ub370\ubc84\ub8e8\uac00 \uc7a0\uacb0\uc5d0 \uc77c\uc5b4\ub098 \uc0b4\ub824\ub2ec\ub77c\uace0 \uc560\uc6d0\ud558\ub294 \ubc1c\ub80c\uc288\ud0c0\uc778\uc744 \ubbf8\ub298\ucc3d \uc73c\ub85c \uc0b4\ud574\ud588\ub2e4. ", "paragraph_answer": "\uc0c1\ub2f9\ud55c \uc7ac\uc815\uc801\u00b7\uc601\ud1a0\uc801 \ubcf4\uc0c1\uc744 \uae30\ub300\ud558\uba74\uc11c, \uc0ac\ud1f4\uc11c\ub97c \uc81c\ucd9c\ud55c \ubc1c\ub80c\uc288\ud0c0\uc778\uc758 \ub9c8\uc9c0\ub9c9 \uc804\ud5a5\uc774 \ube48\uc5d0\uc11c \ubb34\uc2dc\ub2f9\ud588\uc73c\uba70, \ubc1c\ub80c\uc288\ud0c0\uc778\uc774 32\uba85\uc758 \uc5f0\ub300\uc7a5\ub4e4\uacfc \ud568\uaed8 \ud669\uc81c\uc5d0\uac8c \ucda9\uc131\uc744 \ub9f9\uc138\ud55c \uc81c2\uc758 \ud544\uc820 \uc120\uc5b8(1634\ub144 2\uc6d4 20\uc77c)\ub3c4 \uc5ed\uc2dc \ub9c8\ucc2c\uac00\uc9c0\uc600\ub2e4. \ud504\ub77c\ud558\uac00 \ud669\uc81c\ub97c \uc9c0\uc9c0\ud55c\ub2e4\uace0 \uc120\uc5b8\ud588\uc744\ub54c, \ubc1c\ub80c\uc288\ud0c0\uc778\uc740 \uc2a4\uc6e8\ub374\uc778\uacfc \uc791\uc13c\uc778\ub4e4\uacfc \ud798\uc744 \ud569\uccd0\ubcf4\uaca0\ub2e4\ub294 \uc0dd\uac01\uc73c\ub85c \ud544\uc820\uc744 \ub5a0\ub098 \uc5d0\uac70(\uc624\ub298\ub0a0 \uccb4\ucf54\uc758 \ud5e4\ud504)\ub85c \ud5a5\ud588\ub2e4. 2\uc6d4 25\uc77c, \uac70\uae30\uc11c \uadf8\uc640 \ud2b8\ub85c\uce20\uce74, \uc77c\ub85c, \uadf8 \ubc16\uc758 \uc7a5\uad50\ub4e4\uc740 \uc544\uc77c\ub79c\ub4dc\uc778 \uc6d4\ud130 \ubc84\ud2c0\ub7ec \uc7a5\uad70, \uadf8\ub9ac\uace0 \uc2a4\ucf54\ud2c0\ub79c\ub4dc \ucd9c\uc2e0 \uc5f0\ub300\uc7a5\uc778 \uc6d4\ud130 \ub808\uc2ac\ub9ac\uc640 \uc874 \uace0\ub4e0\uc774 \uc9c0\ud718\ud558\ub294 \ubcd1\uc0ac\ub4e4\uc5d0\uac8c \uc0b4\ud574\ub418\uc5c8\ub2e4. \uc601\uad6d\uc778 \ub300\uc7a5 \uc6d4\ud130 \ub370\ubc84\ub8e8\uac00 \uc7a0\uacb0\uc5d0 \uc77c\uc5b4\ub098 \uc0b4\ub824\ub2ec\ub77c\uace0 \uc560\uc6d0\ud558\ub294 \ubc1c\ub80c\uc288\ud0c0\uc778\uc744 \ubbf8\ub298\ucc3d \uc73c\ub85c \uc0b4\ud574\ud588\ub2e4.", "sentence_answer": "\uc601\uad6d\uc778 \ub300\uc7a5 \uc6d4\ud130 \ub370\ubc84\ub8e8\uac00 \uc7a0\uacb0\uc5d0 \uc77c\uc5b4\ub098 \uc0b4\ub824\ub2ec\ub77c\uace0 \uc560\uc6d0\ud558\ub294 \ubc1c\ub80c\uc288\ud0c0\uc778\uc744 \ubbf8\ub298\ucc3d \uc73c\ub85c \uc0b4\ud574\ud588\ub2e4.", "paragraph_id": "6465447-5-1", "paragraph_question": "question: \ubc1c\ub80c\uc288\ud0c0\uc778\uc744 \uc8fd\uc778 \ubb34\uae30\ub294?, context: \uc0c1\ub2f9\ud55c \uc7ac\uc815\uc801\u00b7\uc601\ud1a0\uc801 \ubcf4\uc0c1\uc744 \uae30\ub300\ud558\uba74\uc11c, \uc0ac\ud1f4\uc11c\ub97c \uc81c\ucd9c\ud55c \ubc1c\ub80c\uc288\ud0c0\uc778\uc758 \ub9c8\uc9c0\ub9c9 \uc804\ud5a5\uc774 \ube48\uc5d0\uc11c \ubb34\uc2dc\ub2f9\ud588\uc73c\uba70, \ubc1c\ub80c\uc288\ud0c0\uc778\uc774 32\uba85\uc758 \uc5f0\ub300\uc7a5\ub4e4\uacfc \ud568\uaed8 \ud669\uc81c\uc5d0\uac8c \ucda9\uc131\uc744 \ub9f9\uc138\ud55c \uc81c2\uc758 \ud544\uc820 \uc120\uc5b8(1634\ub144 2\uc6d4 20\uc77c)\ub3c4 \uc5ed\uc2dc \ub9c8\ucc2c\uac00\uc9c0\uc600\ub2e4. \ud504\ub77c\ud558\uac00 \ud669\uc81c\ub97c \uc9c0\uc9c0\ud55c\ub2e4\uace0 \uc120\uc5b8\ud588\uc744\ub54c, \ubc1c\ub80c\uc288\ud0c0\uc778\uc740 \uc2a4\uc6e8\ub374\uc778\uacfc \uc791\uc13c\uc778\ub4e4\uacfc \ud798\uc744 \ud569\uccd0\ubcf4\uaca0\ub2e4\ub294 \uc0dd\uac01\uc73c\ub85c \ud544\uc820\uc744 \ub5a0\ub098 \uc5d0\uac70(\uc624\ub298\ub0a0 \uccb4\ucf54\uc758 \ud5e4\ud504)\ub85c \ud5a5\ud588\ub2e4. 2\uc6d4 25\uc77c, \uac70\uae30\uc11c \uadf8\uc640 \ud2b8\ub85c\uce20\uce74, \uc77c\ub85c, \uadf8 \ubc16\uc758 \uc7a5\uad50\ub4e4\uc740 \uc544\uc77c\ub79c\ub4dc\uc778 \uc6d4\ud130 \ubc84\ud2c0\ub7ec \uc7a5\uad70, \uadf8\ub9ac\uace0 \uc2a4\ucf54\ud2c0\ub79c\ub4dc \ucd9c\uc2e0 \uc5f0\ub300\uc7a5\uc778 \uc6d4\ud130 \ub808\uc2ac\ub9ac\uc640 \uc874 \uace0\ub4e0\uc774 \uc9c0\ud718\ud558\ub294 \ubcd1\uc0ac\ub4e4\uc5d0\uac8c \uc0b4\ud574\ub418\uc5c8\ub2e4. \uc601\uad6d\uc778 \ub300\uc7a5 \uc6d4\ud130 \ub370\ubc84\ub8e8\uac00 \uc7a0\uacb0\uc5d0 \uc77c\uc5b4\ub098 \uc0b4\ub824\ub2ec\ub77c\uace0 \uc560\uc6d0\ud558\ub294 \ubc1c\ub80c\uc288\ud0c0\uc778\uc744 \ubbf8\ub298\ucc3d\uc73c\ub85c \uc0b4\ud574\ud588\ub2e4."} {"question": "\uc218\ub2e8\uc758 \uc8fc\uc694 \ub18d\uc0b0\ubb3c\uc740 \ubb34\uc5c7\uc778\uac00?", "paragraph": "1996-2005\ub144 \uae30\uac04 \ub3d9\uc548 \uc5f0\ud3c9\uade0 6.3%\uc758 \uacbd\uc81c \uc131\uc7a5\uc744 \uae30\ub85d\ud558\uc600\ub2e4. \ucd1d\ub178\ub3d9\uc778\uad6c\uc758 80%\uac00 \ub18d\uc5c5 \ub610\ub294 \ubaa9\ucd95\uc5c5\uc5d0 \uc885\uc0ac\ud558\uace0 \uc788\ub2e4. \uacbd\uc9c0 \uba74\uc801\uc740 800\ub9cc\u33ca\ub85c \uc8fc\ub85c \ub098\uc77c\uac15 \uc720\uc5ed\uc5d0 \ubd84\ud3ec\ud55c\ub2e4. \uc8fc\uc694 \ub18d\uc0b0\ubb3c\uc740 \uba74\ud654\uc778\ub370, \uc8fc\ub85c \uc601\uad6d\uc778\uc774 \uac74\uc124\ud55c \uc13c\ub108\ub310\uc5d0 \uc758\ud574 \uad00\uac1c(\u704c\u6f11)\ub41c \ucf00\ub514\ub77c \uc9c0\uad6c\uc5d0\uc11c \uc7ac\ubc30\ub41c\ub2e4. \uba74\ud654 \uc774\uc678\uc758 \uacbd\uc791\ubb3c\ub85c\ub294 \uc544\ub77c\ube44\uc544\uace0\ubb34, \uae68, \ub099\ud654\uc0dd, \uc0ac\ud0d5\uc218\uc218, \uc870, \ubc00 \ub4f1\uc774 \uc788\ub2e4. \uacf5\uc5c5\uc5d0 \uc788\uc5b4\uc11c\ub294 \ub3c5\ub9bd \ud6c4 \uc12c\uc720, \uc2dd\ud488, \uc81c\ub2f9, \uc81c\uc9c0, \uc815\uc720, \uc2dc\uba58\ud2b8 \ub4f1\uc758 \uacf5\uc7a5\uc774 \uac74\uc124\ub418\uc5c8\ub2e4. \ud604\uc7ac \uc218\ub2e8\uc5d0\uc11c \uacf5\uc5c5\ud654\uc758 \uc7a5\uc560\uac00 \ub418\uace0 \uc788\ub294 \uac83\uc740 \uc790\ubcf8\uc758 \ubd80\uc871, \uad50\ud1b5\uc2dc\uc124\uc758 \ubbf8\ube44, \uae30\uc220\uc758 \ud6c4\uc9c4\uc131 \ub4f1\uc774\ub2e4. \uc9c0\ud558\uc790\uc6d0\uc73c\ub85c\ub294 \ucca0\uad11\uc11d\u00b7\ub9dd\uac04\u00b7\ub3d9 \ub4f1\uc774 \uc788\ub2e4. 2002\ub144 \uc218\ucd9c 17\uc5b5 1,770\ub9cc \ub2ec\ub7ec, \uc218\uc785 19\uc5b5 1,480\ub9cc \ub2ec\ub7ec\ub97c \uae30\ub85d\ud588\ub2e4. \uc8fc\uc694 \uc218\ucd9c\ud488\uc740 \uba74\ud654\uc640 \ub099\ud654\uc0dd, \uc544\ub77c\ube44\uc544\uace0\ubb34 \ub4f1\uc774\ub2e4. \ub300\ud3ed\uc801\uc778 \ubb34\uc5ed\uc801\uc790\ub294 \uc678\uad6d\uc758 \uac01\uc885 \uc6d0\uc870\uc640 \uc544\ub78d\uc0b0\uc720\uad6d\uc5d0 \ucde8\uc5c5\ud55c \ub178\ub3d9\uc790\uc758 \ubcf8\uad6d \uc1a1\uae08\uc73c\ub85c \uba54\uc6b4\ub2e4.", "answer": "\uba74\ud654", "sentence": "\uc8fc\uc694 \ub18d\uc0b0\ubb3c\uc740 \uba74\ud654 \uc778\ub370, \uc8fc\ub85c \uc601\uad6d\uc778\uc774 \uac74\uc124\ud55c \uc13c\ub108\ub310\uc5d0 \uc758\ud574 \uad00\uac1c(\u704c\u6f11)\ub41c \ucf00\ub514\ub77c \uc9c0\uad6c\uc5d0\uc11c \uc7ac\ubc30\ub41c\ub2e4.", "paragraph_sentence": "1996-2005\ub144 \uae30\uac04 \ub3d9\uc548 \uc5f0\ud3c9\uade0 6.3%\uc758 \uacbd\uc81c \uc131\uc7a5\uc744 \uae30\ub85d\ud558\uc600\ub2e4. \ucd1d\ub178\ub3d9\uc778\uad6c\uc758 80%\uac00 \ub18d\uc5c5 \ub610\ub294 \ubaa9\ucd95\uc5c5\uc5d0 \uc885\uc0ac\ud558\uace0 \uc788\ub2e4. \uacbd\uc9c0 \uba74\uc801\uc740 800\ub9cc\u33ca\ub85c \uc8fc\ub85c \ub098\uc77c\uac15 \uc720\uc5ed\uc5d0 \ubd84\ud3ec\ud55c\ub2e4. \uc8fc\uc694 \ub18d\uc0b0\ubb3c\uc740 \uba74\ud654 \uc778\ub370, \uc8fc\ub85c \uc601\uad6d\uc778\uc774 \uac74\uc124\ud55c \uc13c\ub108\ub310\uc5d0 \uc758\ud574 \uad00\uac1c(\u704c\u6f11)\ub41c \ucf00\ub514\ub77c \uc9c0\uad6c\uc5d0\uc11c \uc7ac\ubc30\ub41c\ub2e4. \uba74\ud654 \uc774\uc678\uc758 \uacbd\uc791\ubb3c\ub85c\ub294 \uc544\ub77c\ube44\uc544\uace0\ubb34, \uae68, \ub099\ud654\uc0dd, \uc0ac\ud0d5\uc218\uc218, \uc870, \ubc00 \ub4f1\uc774 \uc788\ub2e4. \uacf5\uc5c5\uc5d0 \uc788\uc5b4\uc11c\ub294 \ub3c5\ub9bd \ud6c4 \uc12c\uc720, \uc2dd\ud488, \uc81c\ub2f9, \uc81c\uc9c0, \uc815\uc720, \uc2dc\uba58\ud2b8 \ub4f1\uc758 \uacf5\uc7a5\uc774 \uac74\uc124\ub418\uc5c8\ub2e4. \ud604\uc7ac \uc218\ub2e8\uc5d0\uc11c \uacf5\uc5c5\ud654\uc758 \uc7a5\uc560\uac00 \ub418\uace0 \uc788\ub294 \uac83\uc740 \uc790\ubcf8\uc758 \ubd80\uc871, \uad50\ud1b5\uc2dc\uc124\uc758 \ubbf8\ube44, \uae30\uc220\uc758 \ud6c4\uc9c4\uc131 \ub4f1\uc774\ub2e4. \uc9c0\ud558\uc790\uc6d0\uc73c\ub85c\ub294 \ucca0\uad11\uc11d\u00b7\ub9dd\uac04\u00b7\ub3d9 \ub4f1\uc774 \uc788\ub2e4. 2002\ub144 \uc218\ucd9c 17\uc5b5 1,770\ub9cc \ub2ec\ub7ec, \uc218\uc785 19\uc5b5 1,480\ub9cc \ub2ec\ub7ec\ub97c \uae30\ub85d\ud588\ub2e4. \uc8fc\uc694 \uc218\ucd9c\ud488\uc740 \uba74\ud654\uc640 \ub099\ud654\uc0dd, \uc544\ub77c\ube44\uc544\uace0\ubb34 \ub4f1\uc774\ub2e4. \ub300\ud3ed\uc801\uc778 \ubb34\uc5ed\uc801\uc790\ub294 \uc678\uad6d\uc758 \uac01\uc885 \uc6d0\uc870\uc640 \uc544\ub78d\uc0b0\uc720\uad6d\uc5d0 \ucde8\uc5c5\ud55c \ub178\ub3d9\uc790\uc758 \ubcf8\uad6d \uc1a1\uae08\uc73c\ub85c \uba54\uc6b4\ub2e4.", "paragraph_answer": "1996-2005\ub144 \uae30\uac04 \ub3d9\uc548 \uc5f0\ud3c9\uade0 6.3%\uc758 \uacbd\uc81c \uc131\uc7a5\uc744 \uae30\ub85d\ud558\uc600\ub2e4. \ucd1d\ub178\ub3d9\uc778\uad6c\uc758 80%\uac00 \ub18d\uc5c5 \ub610\ub294 \ubaa9\ucd95\uc5c5\uc5d0 \uc885\uc0ac\ud558\uace0 \uc788\ub2e4. \uacbd\uc9c0 \uba74\uc801\uc740 800\ub9cc\u33ca\ub85c \uc8fc\ub85c \ub098\uc77c\uac15 \uc720\uc5ed\uc5d0 \ubd84\ud3ec\ud55c\ub2e4. \uc8fc\uc694 \ub18d\uc0b0\ubb3c\uc740 \uba74\ud654 \uc778\ub370, \uc8fc\ub85c \uc601\uad6d\uc778\uc774 \uac74\uc124\ud55c \uc13c\ub108\ub310\uc5d0 \uc758\ud574 \uad00\uac1c(\u704c\u6f11)\ub41c \ucf00\ub514\ub77c \uc9c0\uad6c\uc5d0\uc11c \uc7ac\ubc30\ub41c\ub2e4. \uba74\ud654 \uc774\uc678\uc758 \uacbd\uc791\ubb3c\ub85c\ub294 \uc544\ub77c\ube44\uc544\uace0\ubb34, \uae68, \ub099\ud654\uc0dd, \uc0ac\ud0d5\uc218\uc218, \uc870, \ubc00 \ub4f1\uc774 \uc788\ub2e4. \uacf5\uc5c5\uc5d0 \uc788\uc5b4\uc11c\ub294 \ub3c5\ub9bd \ud6c4 \uc12c\uc720, \uc2dd\ud488, \uc81c\ub2f9, \uc81c\uc9c0, \uc815\uc720, \uc2dc\uba58\ud2b8 \ub4f1\uc758 \uacf5\uc7a5\uc774 \uac74\uc124\ub418\uc5c8\ub2e4. \ud604\uc7ac \uc218\ub2e8\uc5d0\uc11c \uacf5\uc5c5\ud654\uc758 \uc7a5\uc560\uac00 \ub418\uace0 \uc788\ub294 \uac83\uc740 \uc790\ubcf8\uc758 \ubd80\uc871, \uad50\ud1b5\uc2dc\uc124\uc758 \ubbf8\ube44, \uae30\uc220\uc758 \ud6c4\uc9c4\uc131 \ub4f1\uc774\ub2e4. \uc9c0\ud558\uc790\uc6d0\uc73c\ub85c\ub294 \ucca0\uad11\uc11d\u00b7\ub9dd\uac04\u00b7\ub3d9 \ub4f1\uc774 \uc788\ub2e4. 2002\ub144 \uc218\ucd9c 17\uc5b5 1,770\ub9cc \ub2ec\ub7ec, \uc218\uc785 19\uc5b5 1,480\ub9cc \ub2ec\ub7ec\ub97c \uae30\ub85d\ud588\ub2e4. \uc8fc\uc694 \uc218\ucd9c\ud488\uc740 \uba74\ud654\uc640 \ub099\ud654\uc0dd, \uc544\ub77c\ube44\uc544\uace0\ubb34 \ub4f1\uc774\ub2e4. \ub300\ud3ed\uc801\uc778 \ubb34\uc5ed\uc801\uc790\ub294 \uc678\uad6d\uc758 \uac01\uc885 \uc6d0\uc870\uc640 \uc544\ub78d\uc0b0\uc720\uad6d\uc5d0 \ucde8\uc5c5\ud55c \ub178\ub3d9\uc790\uc758 \ubcf8\uad6d \uc1a1\uae08\uc73c\ub85c \uba54\uc6b4\ub2e4.", "sentence_answer": "\uc8fc\uc694 \ub18d\uc0b0\ubb3c\uc740 \uba74\ud654 \uc778\ub370, \uc8fc\ub85c \uc601\uad6d\uc778\uc774 \uac74\uc124\ud55c \uc13c\ub108\ub310\uc5d0 \uc758\ud574 \uad00\uac1c(\u704c\u6f11)\ub41c \ucf00\ub514\ub77c \uc9c0\uad6c\uc5d0\uc11c \uc7ac\ubc30\ub41c\ub2e4.", "paragraph_id": "6532482-1-0", "paragraph_question": "question: \uc218\ub2e8\uc758 \uc8fc\uc694 \ub18d\uc0b0\ubb3c\uc740 \ubb34\uc5c7\uc778\uac00?, context: 1996-2005\ub144 \uae30\uac04 \ub3d9\uc548 \uc5f0\ud3c9\uade0 6.3%\uc758 \uacbd\uc81c \uc131\uc7a5\uc744 \uae30\ub85d\ud558\uc600\ub2e4. \ucd1d\ub178\ub3d9\uc778\uad6c\uc758 80%\uac00 \ub18d\uc5c5 \ub610\ub294 \ubaa9\ucd95\uc5c5\uc5d0 \uc885\uc0ac\ud558\uace0 \uc788\ub2e4. \uacbd\uc9c0 \uba74\uc801\uc740 800\ub9cc\u33ca\ub85c \uc8fc\ub85c \ub098\uc77c\uac15 \uc720\uc5ed\uc5d0 \ubd84\ud3ec\ud55c\ub2e4. \uc8fc\uc694 \ub18d\uc0b0\ubb3c\uc740 \uba74\ud654\uc778\ub370, \uc8fc\ub85c \uc601\uad6d\uc778\uc774 \uac74\uc124\ud55c \uc13c\ub108\ub310\uc5d0 \uc758\ud574 \uad00\uac1c(\u704c\u6f11)\ub41c \ucf00\ub514\ub77c \uc9c0\uad6c\uc5d0\uc11c \uc7ac\ubc30\ub41c\ub2e4. \uba74\ud654 \uc774\uc678\uc758 \uacbd\uc791\ubb3c\ub85c\ub294 \uc544\ub77c\ube44\uc544\uace0\ubb34, \uae68, \ub099\ud654\uc0dd, \uc0ac\ud0d5\uc218\uc218, \uc870, \ubc00 \ub4f1\uc774 \uc788\ub2e4. \uacf5\uc5c5\uc5d0 \uc788\uc5b4\uc11c\ub294 \ub3c5\ub9bd \ud6c4 \uc12c\uc720, \uc2dd\ud488, \uc81c\ub2f9, \uc81c\uc9c0, \uc815\uc720, \uc2dc\uba58\ud2b8 \ub4f1\uc758 \uacf5\uc7a5\uc774 \uac74\uc124\ub418\uc5c8\ub2e4. \ud604\uc7ac \uc218\ub2e8\uc5d0\uc11c \uacf5\uc5c5\ud654\uc758 \uc7a5\uc560\uac00 \ub418\uace0 \uc788\ub294 \uac83\uc740 \uc790\ubcf8\uc758 \ubd80\uc871, \uad50\ud1b5\uc2dc\uc124\uc758 \ubbf8\ube44, \uae30\uc220\uc758 \ud6c4\uc9c4\uc131 \ub4f1\uc774\ub2e4. \uc9c0\ud558\uc790\uc6d0\uc73c\ub85c\ub294 \ucca0\uad11\uc11d\u00b7\ub9dd\uac04\u00b7\ub3d9 \ub4f1\uc774 \uc788\ub2e4. 2002\ub144 \uc218\ucd9c 17\uc5b5 1,770\ub9cc \ub2ec\ub7ec, \uc218\uc785 19\uc5b5 1,480\ub9cc \ub2ec\ub7ec\ub97c \uae30\ub85d\ud588\ub2e4. \uc8fc\uc694 \uc218\ucd9c\ud488\uc740 \uba74\ud654\uc640 \ub099\ud654\uc0dd, \uc544\ub77c\ube44\uc544\uace0\ubb34 \ub4f1\uc774\ub2e4. \ub300\ud3ed\uc801\uc778 \ubb34\uc5ed\uc801\uc790\ub294 \uc678\uad6d\uc758 \uac01\uc885 \uc6d0\uc870\uc640 \uc544\ub78d\uc0b0\uc720\uad6d\uc5d0 \ucde8\uc5c5\ud55c \ub178\ub3d9\uc790\uc758 \ubcf8\uad6d \uc1a1\uae08\uc73c\ub85c \uba54\uc6b4\ub2e4."} {"question": "\ucf00\ud50c\ub7ec\ub294 \ube44\ud22c\uc2a4 \ubb90\ub7ec\uc758 \ubc11\uc5d0\uc11c \ubb34\uc5c7\uc744 \uacf5\ubd80\ud558\uc600\ub098?", "paragraph": "1589\ub144, \ucd08\ub4f1 \ud559\uad50, \ub77c\ud2f4\uc5b4 \uc911\ub4f1 \ud559\uad50, \ubdd4\ub974\ud15c\ubca0\ub974\ud06c\uc758 \uad6d\uc601 \uac1c\uc2e0\uad50 \uc2e0\ud559\uad50\ub97c \uac70\uce5c \ub4a4, \ucf00\ud50c\ub7ec\ub294 \ud280\ube59\uac90 \ub300\ud559\uad50\uc5d0 \uc2e0\ud559 \ud559\uc0dd\uc73c\ub85c \ub4e4\uc5b4\uac00 \ube44\ud22c\uc2a4 \ubb90\ub7ec \ubc11\uc5d0\uc11c \ucca0\ud559\uc744 \uacf5\ubd80\ud588\ub2e4. \uad50\uacfc \uacfc\uc815 \uc911\uc5d0\ub294 \uc218\ud559\uacfc \ucc9c\ubb38\ud559\ub3c4 \uc788\uc5c8\ub2e4. \uadf8\ub294 \uc790\uc2e0\uc774 \uad49\uc7a5\ud55c \uc2e4\ub825\uc758 \uc218\ud559\ub3c4\uc784\uc744 \uc99d\uba85\ud588\uace0 \ub2a5\uc219\ud55c \uc810\uc131\uc220\uc0ac\ub77c\ub294 \ud3c9\ud310\uc744 \ub4e4\uc5c8\uc73c\uba70, \ub3d9\ub144\ubc30 \ud559\uc0dd\ub4e4\uc5d0\uac8c \ubcc4\uc810\uc744 \uccd0\uc8fc\uace4 \ud588\ub2e4. \ub300\ud559\uc758 \uc218\ud559 \uad50\uc218\uc600\ub358 \ubbf8\ud558\uc5d8 \ub9e4\uc2a4\ud2c0\ub9b0\uc758 \uc9c0\ub3c4\ub97c \ubc1b\uc73c\uba70, \ucf00\ud50c\ub7ec\ub294 \ud589\uc131\uc6b4\ub3d9\uc5d0 \uad00\ud558\uc5ec \uac1c\uc2e0\uad50\uc5d0\uc11c \uc758\ubb34\uc801\uc73c\ub85c \uac00\ub974\uccd0\uc57c \ud558\ub294 \ud504\ud1a8\ub808\ub9c8\uc774\uc624\uc2a4\uc124\uacfc \uc77c\ubd80 \uc81c\uc790\ub4e4\uc5d0\uac8c\ub9cc \uac1c\uc778\uc801\uc73c\ub85c \uac00\ub974\ucce4\ub358 \ucf54\ud398\ub974\ub2c8\ucfe0\uc2a4\uc124 \uc591\ucabd\uc744 \ubaa8\ub450 \ubc30\uc6e0\ub2e4. \uadf8\ub294 \uc774 \uc2dc\uc810\ubd80\ud130 \ucf54\ud398\ub974\ub2c8\ucfe0\uc2a4\uc8fc\uc758\uc790\uac00 \ub418\uc5c8\ub2e4. \ud559\uc0dd\ub4e4 \uac04\uc5d0 \ud1a0\ub860\uc774 \ubc8c\uc5b4\uc9c0\uba74, \ucf00\ud50c\ub7ec\ub294 \uc774\ub860\uc801\uc778 \uad00\uc810\uacfc \uc2e0\ud559\uc801\uc778 \uad00\uc810 \ubaa8\ub450\uc5d0\uc11c \uc9c0\ub3d9\uc124\uc744 \uc639\ud638\ud588\uace0, \ud0dc\uc591\uc774 \uc6b0\uc8fc\ub97c \uc6c0\uc9c1\uc774\ub294 \uc8fc\uc694\ud55c \uc6d0\ucc9c\uc774\ub77c\uace0 \uc8fc\uc7a5\ud588\ub2e4. \ubaa9\uc0ac\uac00 \ub418\uaca0\ub2e4\ub294 \ubc14\ub78c\uc5d0\ub3c4 \ubd88\uad6c\ud558\uace0, \ud559\uc5c5\uc758 \ub9c9\ubc14\uc9c0\uc5d0 \uc774\ub974\ub7ec \ucf00\ud50c\ub7ec\ub294 \uc624\uc2a4\ud2b8\ub9ac\uc544 \uadf8\ub77c\uce20\uc758 \uac1c\uc2e0\uad50 \ud559\uad50(\uadf8\ub77c\uce20 \ub300\ud559\uc758 \uc804\uc2e0)\uc758 \uc218\ud559\uacfc \ucc9c\ubb38\ud559 \uad50\uc0ac \uc790\ub9ac\ub85c \ucc9c\uac70\ub418\uc5c8\ub2e4. \uadf8\ub294 1594\ub144 4\uc6d4, \ub098\uc774 23\uc138 \ub54c \uadf8 \uc790\ub9ac\ub97c \ubc1b\uc544\ub4e4\uc600\ub2e4. \ud558\uc9c0\ub9cc \ucf00\ud50c\ub7ec\ub294 \ud6cc\ub96d\ud55c \ud559\uc790\uc774\uae34 \ud588\uc5b4\ub3c4 \ud6cc\ub96d\ud55c \uad50\uc721\uc790\ub294 \ub418\uc9c0 \ubabb\ud588\ub2e4. \ubaa9\uc18c\ub9ac\ub294 \uc6c5\uc5bc\uac70\ub838\uace0, \ube48\ubc88\ud788 \uac15\uc758 \uc8fc\uc81c\ub97c \ubc97\uc5b4\ub098 \uc606\uae38\ub85c \ube60\uc84c\uc73c\uba70, \uadf8\ub77c\uce20\uc5d0\uc11c\uc758 \ub450 \ubc88\uc9f8 \ud574\uc5d0\ub294 \uc218\uac15\uc0dd\uc774 \ud55c \uba85\ub3c4 \uc5c6\uc5c8\uc744 \uc815\ub3c4\ub85c \ub9d0 \ubabb\ud558\uac8c \uc9c0\ub8e8\ud55c \uac15\uc758\uc600\ub2e4\uace0 \ud55c\ub2e4.", "answer": "\ucca0\ud559", "sentence": "1589\ub144, \ucd08\ub4f1 \ud559\uad50, \ub77c\ud2f4\uc5b4 \uc911\ub4f1 \ud559\uad50, \ubdd4\ub974\ud15c\ubca0\ub974\ud06c\uc758 \uad6d\uc601 \uac1c\uc2e0\uad50 \uc2e0\ud559\uad50\ub97c \uac70\uce5c \ub4a4, \ucf00\ud50c\ub7ec\ub294 \ud280\ube59\uac90 \ub300\ud559\uad50\uc5d0 \uc2e0\ud559 \ud559\uc0dd\uc73c\ub85c \ub4e4\uc5b4\uac00 \ube44\ud22c\uc2a4 \ubb90\ub7ec \ubc11\uc5d0\uc11c \ucca0\ud559 \uc744 \uacf5\ubd80\ud588\ub2e4.", "paragraph_sentence": " 1589\ub144, \ucd08\ub4f1 \ud559\uad50, \ub77c\ud2f4\uc5b4 \uc911\ub4f1 \ud559\uad50, \ubdd4\ub974\ud15c\ubca0\ub974\ud06c\uc758 \uad6d\uc601 \uac1c\uc2e0\uad50 \uc2e0\ud559\uad50\ub97c \uac70\uce5c \ub4a4, \ucf00\ud50c\ub7ec\ub294 \ud280\ube59\uac90 \ub300\ud559\uad50\uc5d0 \uc2e0\ud559 \ud559\uc0dd\uc73c\ub85c \ub4e4\uc5b4\uac00 \ube44\ud22c\uc2a4 \ubb90\ub7ec \ubc11\uc5d0\uc11c \ucca0\ud559 \uc744 \uacf5\ubd80\ud588\ub2e4. \uad50\uacfc \uacfc\uc815 \uc911\uc5d0\ub294 \uc218\ud559\uacfc \ucc9c\ubb38\ud559\ub3c4 \uc788\uc5c8\ub2e4. \uadf8\ub294 \uc790\uc2e0\uc774 \uad49\uc7a5\ud55c \uc2e4\ub825\uc758 \uc218\ud559\ub3c4\uc784\uc744 \uc99d\uba85\ud588\uace0 \ub2a5\uc219\ud55c \uc810\uc131\uc220\uc0ac\ub77c\ub294 \ud3c9\ud310\uc744 \ub4e4\uc5c8\uc73c\uba70, \ub3d9\ub144\ubc30 \ud559\uc0dd\ub4e4\uc5d0\uac8c \ubcc4\uc810\uc744 \uccd0\uc8fc\uace4 \ud588\ub2e4. \ub300\ud559\uc758 \uc218\ud559 \uad50\uc218\uc600\ub358 \ubbf8\ud558\uc5d8 \ub9e4\uc2a4\ud2c0\ub9b0\uc758 \uc9c0\ub3c4\ub97c \ubc1b\uc73c\uba70, \ucf00\ud50c\ub7ec\ub294 \ud589\uc131\uc6b4\ub3d9\uc5d0 \uad00\ud558\uc5ec \uac1c\uc2e0\uad50\uc5d0\uc11c \uc758\ubb34\uc801\uc73c\ub85c \uac00\ub974\uccd0\uc57c \ud558\ub294 \ud504\ud1a8\ub808\ub9c8\uc774\uc624\uc2a4\uc124\uacfc \uc77c\ubd80 \uc81c\uc790\ub4e4\uc5d0\uac8c\ub9cc \uac1c\uc778\uc801\uc73c\ub85c \uac00\ub974\ucce4\ub358 \ucf54\ud398\ub974\ub2c8\ucfe0\uc2a4\uc124 \uc591\ucabd\uc744 \ubaa8\ub450 \ubc30\uc6e0\ub2e4. \uadf8\ub294 \uc774 \uc2dc\uc810\ubd80\ud130 \ucf54\ud398\ub974\ub2c8\ucfe0\uc2a4\uc8fc\uc758\uc790\uac00 \ub418\uc5c8\ub2e4. \ud559\uc0dd\ub4e4 \uac04\uc5d0 \ud1a0\ub860\uc774 \ubc8c\uc5b4\uc9c0\uba74, \ucf00\ud50c\ub7ec\ub294 \uc774\ub860\uc801\uc778 \uad00\uc810\uacfc \uc2e0\ud559\uc801\uc778 \uad00\uc810 \ubaa8\ub450\uc5d0\uc11c \uc9c0\ub3d9\uc124\uc744 \uc639\ud638\ud588\uace0, \ud0dc\uc591\uc774 \uc6b0\uc8fc\ub97c \uc6c0\uc9c1\uc774\ub294 \uc8fc\uc694\ud55c \uc6d0\ucc9c\uc774\ub77c\uace0 \uc8fc\uc7a5\ud588\ub2e4. \ubaa9\uc0ac\uac00 \ub418\uaca0\ub2e4\ub294 \ubc14\ub78c\uc5d0\ub3c4 \ubd88\uad6c\ud558\uace0, \ud559\uc5c5\uc758 \ub9c9\ubc14\uc9c0\uc5d0 \uc774\ub974\ub7ec \ucf00\ud50c\ub7ec\ub294 \uc624\uc2a4\ud2b8\ub9ac\uc544 \uadf8\ub77c\uce20\uc758 \uac1c\uc2e0\uad50 \ud559\uad50(\uadf8\ub77c\uce20 \ub300\ud559\uc758 \uc804\uc2e0)\uc758 \uc218\ud559\uacfc \ucc9c\ubb38\ud559 \uad50\uc0ac \uc790\ub9ac\ub85c \ucc9c\uac70\ub418\uc5c8\ub2e4. \uadf8\ub294 1594\ub144 4\uc6d4, \ub098\uc774 23\uc138 \ub54c \uadf8 \uc790\ub9ac\ub97c \ubc1b\uc544\ub4e4\uc600\ub2e4. \ud558\uc9c0\ub9cc \ucf00\ud50c\ub7ec\ub294 \ud6cc\ub96d\ud55c \ud559\uc790\uc774\uae34 \ud588\uc5b4\ub3c4 \ud6cc\ub96d\ud55c \uad50\uc721\uc790\ub294 \ub418\uc9c0 \ubabb\ud588\ub2e4. \ubaa9\uc18c\ub9ac\ub294 \uc6c5\uc5bc\uac70\ub838\uace0, \ube48\ubc88\ud788 \uac15\uc758 \uc8fc\uc81c\ub97c \ubc97\uc5b4\ub098 \uc606\uae38\ub85c \ube60\uc84c\uc73c\uba70, \uadf8\ub77c\uce20\uc5d0\uc11c\uc758 \ub450 \ubc88\uc9f8 \ud574\uc5d0\ub294 \uc218\uac15\uc0dd\uc774 \ud55c \uba85\ub3c4 \uc5c6\uc5c8\uc744 \uc815\ub3c4\ub85c \ub9d0 \ubabb\ud558\uac8c \uc9c0\ub8e8\ud55c \uac15\uc758\uc600\ub2e4\uace0 \ud55c\ub2e4.", "paragraph_answer": "1589\ub144, \ucd08\ub4f1 \ud559\uad50, \ub77c\ud2f4\uc5b4 \uc911\ub4f1 \ud559\uad50, \ubdd4\ub974\ud15c\ubca0\ub974\ud06c\uc758 \uad6d\uc601 \uac1c\uc2e0\uad50 \uc2e0\ud559\uad50\ub97c \uac70\uce5c \ub4a4, \ucf00\ud50c\ub7ec\ub294 \ud280\ube59\uac90 \ub300\ud559\uad50\uc5d0 \uc2e0\ud559 \ud559\uc0dd\uc73c\ub85c \ub4e4\uc5b4\uac00 \ube44\ud22c\uc2a4 \ubb90\ub7ec \ubc11\uc5d0\uc11c \ucca0\ud559 \uc744 \uacf5\ubd80\ud588\ub2e4. \uad50\uacfc \uacfc\uc815 \uc911\uc5d0\ub294 \uc218\ud559\uacfc \ucc9c\ubb38\ud559\ub3c4 \uc788\uc5c8\ub2e4. \uadf8\ub294 \uc790\uc2e0\uc774 \uad49\uc7a5\ud55c \uc2e4\ub825\uc758 \uc218\ud559\ub3c4\uc784\uc744 \uc99d\uba85\ud588\uace0 \ub2a5\uc219\ud55c \uc810\uc131\uc220\uc0ac\ub77c\ub294 \ud3c9\ud310\uc744 \ub4e4\uc5c8\uc73c\uba70, \ub3d9\ub144\ubc30 \ud559\uc0dd\ub4e4\uc5d0\uac8c \ubcc4\uc810\uc744 \uccd0\uc8fc\uace4 \ud588\ub2e4. \ub300\ud559\uc758 \uc218\ud559 \uad50\uc218\uc600\ub358 \ubbf8\ud558\uc5d8 \ub9e4\uc2a4\ud2c0\ub9b0\uc758 \uc9c0\ub3c4\ub97c \ubc1b\uc73c\uba70, \ucf00\ud50c\ub7ec\ub294 \ud589\uc131\uc6b4\ub3d9\uc5d0 \uad00\ud558\uc5ec \uac1c\uc2e0\uad50\uc5d0\uc11c \uc758\ubb34\uc801\uc73c\ub85c \uac00\ub974\uccd0\uc57c \ud558\ub294 \ud504\ud1a8\ub808\ub9c8\uc774\uc624\uc2a4\uc124\uacfc \uc77c\ubd80 \uc81c\uc790\ub4e4\uc5d0\uac8c\ub9cc \uac1c\uc778\uc801\uc73c\ub85c \uac00\ub974\ucce4\ub358 \ucf54\ud398\ub974\ub2c8\ucfe0\uc2a4\uc124 \uc591\ucabd\uc744 \ubaa8\ub450 \ubc30\uc6e0\ub2e4. \uadf8\ub294 \uc774 \uc2dc\uc810\ubd80\ud130 \ucf54\ud398\ub974\ub2c8\ucfe0\uc2a4\uc8fc\uc758\uc790\uac00 \ub418\uc5c8\ub2e4. \ud559\uc0dd\ub4e4 \uac04\uc5d0 \ud1a0\ub860\uc774 \ubc8c\uc5b4\uc9c0\uba74, \ucf00\ud50c\ub7ec\ub294 \uc774\ub860\uc801\uc778 \uad00\uc810\uacfc \uc2e0\ud559\uc801\uc778 \uad00\uc810 \ubaa8\ub450\uc5d0\uc11c \uc9c0\ub3d9\uc124\uc744 \uc639\ud638\ud588\uace0, \ud0dc\uc591\uc774 \uc6b0\uc8fc\ub97c \uc6c0\uc9c1\uc774\ub294 \uc8fc\uc694\ud55c \uc6d0\ucc9c\uc774\ub77c\uace0 \uc8fc\uc7a5\ud588\ub2e4. \ubaa9\uc0ac\uac00 \ub418\uaca0\ub2e4\ub294 \ubc14\ub78c\uc5d0\ub3c4 \ubd88\uad6c\ud558\uace0, \ud559\uc5c5\uc758 \ub9c9\ubc14\uc9c0\uc5d0 \uc774\ub974\ub7ec \ucf00\ud50c\ub7ec\ub294 \uc624\uc2a4\ud2b8\ub9ac\uc544 \uadf8\ub77c\uce20\uc758 \uac1c\uc2e0\uad50 \ud559\uad50(\uadf8\ub77c\uce20 \ub300\ud559\uc758 \uc804\uc2e0)\uc758 \uc218\ud559\uacfc \ucc9c\ubb38\ud559 \uad50\uc0ac \uc790\ub9ac\ub85c \ucc9c\uac70\ub418\uc5c8\ub2e4. \uadf8\ub294 1594\ub144 4\uc6d4, \ub098\uc774 23\uc138 \ub54c \uadf8 \uc790\ub9ac\ub97c \ubc1b\uc544\ub4e4\uc600\ub2e4. \ud558\uc9c0\ub9cc \ucf00\ud50c\ub7ec\ub294 \ud6cc\ub96d\ud55c \ud559\uc790\uc774\uae34 \ud588\uc5b4\ub3c4 \ud6cc\ub96d\ud55c \uad50\uc721\uc790\ub294 \ub418\uc9c0 \ubabb\ud588\ub2e4. \ubaa9\uc18c\ub9ac\ub294 \uc6c5\uc5bc\uac70\ub838\uace0, \ube48\ubc88\ud788 \uac15\uc758 \uc8fc\uc81c\ub97c \ubc97\uc5b4\ub098 \uc606\uae38\ub85c \ube60\uc84c\uc73c\uba70, \uadf8\ub77c\uce20\uc5d0\uc11c\uc758 \ub450 \ubc88\uc9f8 \ud574\uc5d0\ub294 \uc218\uac15\uc0dd\uc774 \ud55c \uba85\ub3c4 \uc5c6\uc5c8\uc744 \uc815\ub3c4\ub85c \ub9d0 \ubabb\ud558\uac8c \uc9c0\ub8e8\ud55c \uac15\uc758\uc600\ub2e4\uace0 \ud55c\ub2e4.", "sentence_answer": "1589\ub144, \ucd08\ub4f1 \ud559\uad50, \ub77c\ud2f4\uc5b4 \uc911\ub4f1 \ud559\uad50, \ubdd4\ub974\ud15c\ubca0\ub974\ud06c\uc758 \uad6d\uc601 \uac1c\uc2e0\uad50 \uc2e0\ud559\uad50\ub97c \uac70\uce5c \ub4a4, \ucf00\ud50c\ub7ec\ub294 \ud280\ube59\uac90 \ub300\ud559\uad50\uc5d0 \uc2e0\ud559 \ud559\uc0dd\uc73c\ub85c \ub4e4\uc5b4\uac00 \ube44\ud22c\uc2a4 \ubb90\ub7ec \ubc11\uc5d0\uc11c \ucca0\ud559 \uc744 \uacf5\ubd80\ud588\ub2e4.", "paragraph_id": "6541867-2-0", "paragraph_question": "question: \ucf00\ud50c\ub7ec\ub294 \ube44\ud22c\uc2a4 \ubb90\ub7ec\uc758 \ubc11\uc5d0\uc11c \ubb34\uc5c7\uc744 \uacf5\ubd80\ud558\uc600\ub098?, context: 1589\ub144, \ucd08\ub4f1 \ud559\uad50, \ub77c\ud2f4\uc5b4 \uc911\ub4f1 \ud559\uad50, \ubdd4\ub974\ud15c\ubca0\ub974\ud06c\uc758 \uad6d\uc601 \uac1c\uc2e0\uad50 \uc2e0\ud559\uad50\ub97c \uac70\uce5c \ub4a4, \ucf00\ud50c\ub7ec\ub294 \ud280\ube59\uac90 \ub300\ud559\uad50\uc5d0 \uc2e0\ud559 \ud559\uc0dd\uc73c\ub85c \ub4e4\uc5b4\uac00 \ube44\ud22c\uc2a4 \ubb90\ub7ec \ubc11\uc5d0\uc11c \ucca0\ud559\uc744 \uacf5\ubd80\ud588\ub2e4. \uad50\uacfc \uacfc\uc815 \uc911\uc5d0\ub294 \uc218\ud559\uacfc \ucc9c\ubb38\ud559\ub3c4 \uc788\uc5c8\ub2e4. \uadf8\ub294 \uc790\uc2e0\uc774 \uad49\uc7a5\ud55c \uc2e4\ub825\uc758 \uc218\ud559\ub3c4\uc784\uc744 \uc99d\uba85\ud588\uace0 \ub2a5\uc219\ud55c \uc810\uc131\uc220\uc0ac\ub77c\ub294 \ud3c9\ud310\uc744 \ub4e4\uc5c8\uc73c\uba70, \ub3d9\ub144\ubc30 \ud559\uc0dd\ub4e4\uc5d0\uac8c \ubcc4\uc810\uc744 \uccd0\uc8fc\uace4 \ud588\ub2e4. \ub300\ud559\uc758 \uc218\ud559 \uad50\uc218\uc600\ub358 \ubbf8\ud558\uc5d8 \ub9e4\uc2a4\ud2c0\ub9b0\uc758 \uc9c0\ub3c4\ub97c \ubc1b\uc73c\uba70, \ucf00\ud50c\ub7ec\ub294 \ud589\uc131\uc6b4\ub3d9\uc5d0 \uad00\ud558\uc5ec \uac1c\uc2e0\uad50\uc5d0\uc11c \uc758\ubb34\uc801\uc73c\ub85c \uac00\ub974\uccd0\uc57c \ud558\ub294 \ud504\ud1a8\ub808\ub9c8\uc774\uc624\uc2a4\uc124\uacfc \uc77c\ubd80 \uc81c\uc790\ub4e4\uc5d0\uac8c\ub9cc \uac1c\uc778\uc801\uc73c\ub85c \uac00\ub974\ucce4\ub358 \ucf54\ud398\ub974\ub2c8\ucfe0\uc2a4\uc124 \uc591\ucabd\uc744 \ubaa8\ub450 \ubc30\uc6e0\ub2e4. \uadf8\ub294 \uc774 \uc2dc\uc810\ubd80\ud130 \ucf54\ud398\ub974\ub2c8\ucfe0\uc2a4\uc8fc\uc758\uc790\uac00 \ub418\uc5c8\ub2e4. \ud559\uc0dd\ub4e4 \uac04\uc5d0 \ud1a0\ub860\uc774 \ubc8c\uc5b4\uc9c0\uba74, \ucf00\ud50c\ub7ec\ub294 \uc774\ub860\uc801\uc778 \uad00\uc810\uacfc \uc2e0\ud559\uc801\uc778 \uad00\uc810 \ubaa8\ub450\uc5d0\uc11c \uc9c0\ub3d9\uc124\uc744 \uc639\ud638\ud588\uace0, \ud0dc\uc591\uc774 \uc6b0\uc8fc\ub97c \uc6c0\uc9c1\uc774\ub294 \uc8fc\uc694\ud55c \uc6d0\ucc9c\uc774\ub77c\uace0 \uc8fc\uc7a5\ud588\ub2e4. \ubaa9\uc0ac\uac00 \ub418\uaca0\ub2e4\ub294 \ubc14\ub78c\uc5d0\ub3c4 \ubd88\uad6c\ud558\uace0, \ud559\uc5c5\uc758 \ub9c9\ubc14\uc9c0\uc5d0 \uc774\ub974\ub7ec \ucf00\ud50c\ub7ec\ub294 \uc624\uc2a4\ud2b8\ub9ac\uc544 \uadf8\ub77c\uce20\uc758 \uac1c\uc2e0\uad50 \ud559\uad50(\uadf8\ub77c\uce20 \ub300\ud559\uc758 \uc804\uc2e0)\uc758 \uc218\ud559\uacfc \ucc9c\ubb38\ud559 \uad50\uc0ac \uc790\ub9ac\ub85c \ucc9c\uac70\ub418\uc5c8\ub2e4. \uadf8\ub294 1594\ub144 4\uc6d4, \ub098\uc774 23\uc138 \ub54c \uadf8 \uc790\ub9ac\ub97c \ubc1b\uc544\ub4e4\uc600\ub2e4. \ud558\uc9c0\ub9cc \ucf00\ud50c\ub7ec\ub294 \ud6cc\ub96d\ud55c \ud559\uc790\uc774\uae34 \ud588\uc5b4\ub3c4 \ud6cc\ub96d\ud55c \uad50\uc721\uc790\ub294 \ub418\uc9c0 \ubabb\ud588\ub2e4. \ubaa9\uc18c\ub9ac\ub294 \uc6c5\uc5bc\uac70\ub838\uace0, \ube48\ubc88\ud788 \uac15\uc758 \uc8fc\uc81c\ub97c \ubc97\uc5b4\ub098 \uc606\uae38\ub85c \ube60\uc84c\uc73c\uba70, \uadf8\ub77c\uce20\uc5d0\uc11c\uc758 \ub450 \ubc88\uc9f8 \ud574\uc5d0\ub294 \uc218\uac15\uc0dd\uc774 \ud55c \uba85\ub3c4 \uc5c6\uc5c8\uc744 \uc815\ub3c4\ub85c \ub9d0 \ubabb\ud558\uac8c \uc9c0\ub8e8\ud55c \uac15\uc758\uc600\ub2e4\uace0 \ud55c\ub2e4."} {"question": "\ub300\ud55c\ubbfc\uad6d\uc758 \uad6d\uac00\uc720\uacf5\uc790\ub294 \ub3c4\uc2dc\ucca0\ub3c4 \uc694\uae08\uc744 \uc5bc\ub9c8\ub098 \ub0b4\uace0 \ud0d1\uc2b9\ud558\ub294\uac00?", "paragraph": "\ub300\ud55c\ubbfc\uad6d\uc758 \ub3c4\uc2dc\ucca0\ub3c4\ub294 \uc218\ub3c4\uad8c\uc9c0\uc5ed\uacfc \ubd80\uc0b0\uc9c0\uc5ed, \ub300\uad6c\uad11\uc5ed\uc2dc, \ub300\uc804\uad11\uc5ed\uc2dc, \uad11\uc8fc\uad11\uc5ed\uc2dc \ub4f1\uc5d0\uc11c \uc6b4\ud589\ub418\uace0 \uc788\ub2e4. \uc2dc\ub0b4\ub97c \uc6b4\ud589\ud558\ub294 \ub3c4\uc2dc\ucca0\ub3c4\ub97c \uc77c\uceec\uc5b4 '\uc9c0\ud558\ucca0' \ub610\ub294 '\uc804\ucca0' \ub4f1\uc73c\ub85c \ubd80\ub974\uae30\ub3c4 \ud55c\ub2e4. 1974\ub144 8\uc6d4 15\uc77c \ucc98\uc74c\uc73c\ub85c \uac1c\ud1b5\ub41c \uc218\ub3c4\uad8c\uc9c0\uc5ed\uc740 1~9\ud638\uc120\uacfc \ubd84\ub2f9\uc120, \uc911\uc559\uc120, \uacbd\uc758\uc120, \uacbd\ucd98\uc120, \uc778\ucc9c 1\ud638\uc120, \uc778\ucc9c\uad6d\uc81c\uacf5\ud56d\ucca0\ub3c4, \uc2e0\ubd84\ub2f9\uc120, \uc218\uc778\uc120, \uc6a9\uc778\uacbd\uc804\ucca0, \uc758\uc815\ubd80\uacbd\uc804\ucca0 \uae4c\uc9c0 \ucd1d 19\uac1c \ub178\uc120\uc774 \uc6b4\ud589\ub418\uace0 \uc788\uc73c\uba70 \uc804\uad6d\uc5d0\uc11c \uac00\uc7a5 \uaddc\ubaa8\uac00 \ud06c\ub2e4. \uae30\ubcf8\uc694\uae08\uc740 \ud3c9\uade0\uc801\uc73c\ub85c \uad50\ud1b5\uce74\ub4dc \uc774\uc6a9\uc2dc \ucd5c\ucd08 10km\uae4c\uc9c0 1050\uc6d0, \uadf8\uc774\ud6c4 5;km \ucd08\uacfc\uc2dc\ub9c8\ub2e4 100\uc6d0\uc529 \ucd94\uac00\ub41c\ub2e4. \uadf8\ub7ec\ub098 \ubbfc\uc601 \ub3c4\uc2dc\ucca0\ub3c4\uc778 \uc2e0\ubd84\ub2f9\uc120\uc740 \uae30\ubcf8\uc694\uae08\uc774 1,750\uc6d0\uc774\uace0, \ub2e4\ub978 \ub178\uc120\uc5d0\uc11c \uc2e0\ubd84\ub2f9\uc120\uc73c\ub85c \ud658\uc2b9\ud560 \uacbd\uc6b0 700\uc6d0\uc774 \ucd94\uac00\ub41c\ub2e4. \uc218\ub3c4\uad8c \uc804\ucca0\uc740 \uc778\ucc9c \ub3c4\uc2dc\ucca0\ub3c4, \uc778\ucc9c\uacf5\ud56d, \uae40\ud3ec\uacf5\ud56d\uacfc\ub3c4 \uc5f0\uacb0\ub41c\ub2e4. 1985\ub144 \uac1c\ud1b5\ub41c \ub450 \ubc88\uc9f8\ub85c \ud070 \ubd80\uc0b0 \ub3c4\uc2dc\ucca0\ub3c4\ub294 \ucd1d \uae38\uc774 131.7km\uc5d0 1\ud638\uc120, 2\ud638\uc120, 3\ud638\uc120, 4\ud638\uc120, \ubd80\uc0b0\uae40\ud574\uacbd\uc804\ucca0 5\uac1c \ub178\uc120\uc774 \uc6b4\ud589\ub418\uace0 \uc788\uc73c\uba70, \uc774\uc678\uc5d0\ub3c4 \ub300\uad6c\uc9c0\ud558\ucca0 1\ud638\uc120, 2\ud638\uc120 2\uac1c \ub178\uc120\uc774 \uc6b4\ud589\ub418\uace0 \ub300\uc804\uc9c0\ud558\ucca0 1\ud638\uc120 1\uac1c\ub178\uc120\uc774 \uc6b4\uc601, \uad11\uc8fc1\ud638\uc120 1\uac1c \ub178\uc120 \uc6b4\ud589 \ub4f1 \uc77c\ubd80 \uad11\uc5ed\uc2dc\ub4e4\uc744 \uc911\uc2ec\uc73c\ub85c \ub3c4\uc2dc\ucca0\ub3c4\uac00 \uacc4\uc18d\ub418\uc5b4 \uac74\uc124\uc911\uc774\ub2e4. \uc694\uae08\uc740 \uac01 \uc9c0\uc790\uccb4\uc5d0\uc11c \ub530\ub85c \uaddc\uc815\ud558\uc5ec \uc870\uae08\uc529 \ub2e4\ub974\ub2e4. \ubcf5\uc9c0 \uc815\ucc45\uc73c\ub85c 65\uc138 \uc774\uc0c1\uc758 \ub178\uc778\uacfc \uc7a5\uc560\uc778, \uad6d\uac00\uc720\uacf5\uc790\ub294 \ubb34\uc784\uc73c\ub85c \uc2b9\ucc28\ud560 \uc218 \uc788\ub2e4.(\ubd80\uc0b0-\uae40\ud574 \uacbd\uc804\ucca0\uc740 \ub178\uc778\ub3c4 \uc720\uc784\uc2b9\ucc28)", "answer": "\ubb34\uc784", "sentence": "\ubcf5\uc9c0 \uc815\ucc45\uc73c\ub85c 65\uc138 \uc774\uc0c1\uc758 \ub178\uc778\uacfc \uc7a5\uc560\uc778, \uad6d\uac00\uc720\uacf5\uc790\ub294 \ubb34\uc784 \uc73c\ub85c \uc2b9\ucc28\ud560 \uc218 \uc788\ub2e4.", "paragraph_sentence": "\ub300\ud55c\ubbfc\uad6d\uc758 \ub3c4\uc2dc\ucca0\ub3c4\ub294 \uc218\ub3c4\uad8c\uc9c0\uc5ed\uacfc \ubd80\uc0b0\uc9c0\uc5ed, \ub300\uad6c\uad11\uc5ed\uc2dc, \ub300\uc804\uad11\uc5ed\uc2dc, \uad11\uc8fc\uad11\uc5ed\uc2dc \ub4f1\uc5d0\uc11c \uc6b4\ud589\ub418\uace0 \uc788\ub2e4. \uc2dc\ub0b4\ub97c \uc6b4\ud589\ud558\ub294 \ub3c4\uc2dc\ucca0\ub3c4\ub97c \uc77c\uceec\uc5b4 '\uc9c0\ud558\ucca0' \ub610\ub294 '\uc804\ucca0' \ub4f1\uc73c\ub85c \ubd80\ub974\uae30\ub3c4 \ud55c\ub2e4. 1974\ub144 8\uc6d4 15\uc77c \ucc98\uc74c\uc73c\ub85c \uac1c\ud1b5\ub41c \uc218\ub3c4\uad8c\uc9c0\uc5ed\uc740 1~9\ud638\uc120\uacfc \ubd84\ub2f9\uc120, \uc911\uc559\uc120, \uacbd\uc758\uc120, \uacbd\ucd98\uc120, \uc778\ucc9c 1\ud638\uc120, \uc778\ucc9c\uad6d\uc81c\uacf5\ud56d\ucca0\ub3c4, \uc2e0\ubd84\ub2f9\uc120, \uc218\uc778\uc120, \uc6a9\uc778\uacbd\uc804\ucca0, \uc758\uc815\ubd80\uacbd\uc804\ucca0 \uae4c\uc9c0 \ucd1d 19\uac1c \ub178\uc120\uc774 \uc6b4\ud589\ub418\uace0 \uc788\uc73c\uba70 \uc804\uad6d\uc5d0\uc11c \uac00\uc7a5 \uaddc\ubaa8\uac00 \ud06c\ub2e4. \uae30\ubcf8\uc694\uae08\uc740 \ud3c9\uade0\uc801\uc73c\ub85c \uad50\ud1b5\uce74\ub4dc \uc774\uc6a9\uc2dc \ucd5c\ucd08 10km\uae4c\uc9c0 1050\uc6d0, \uadf8\uc774\ud6c4 5;km \ucd08\uacfc\uc2dc\ub9c8\ub2e4 100\uc6d0\uc529 \ucd94\uac00\ub41c\ub2e4. \uadf8\ub7ec\ub098 \ubbfc\uc601 \ub3c4\uc2dc\ucca0\ub3c4\uc778 \uc2e0\ubd84\ub2f9\uc120\uc740 \uae30\ubcf8\uc694\uae08\uc774 1,750\uc6d0\uc774\uace0, \ub2e4\ub978 \ub178\uc120\uc5d0\uc11c \uc2e0\ubd84\ub2f9\uc120\uc73c\ub85c \ud658\uc2b9\ud560 \uacbd\uc6b0 700\uc6d0\uc774 \ucd94\uac00\ub41c\ub2e4. \uc218\ub3c4\uad8c \uc804\ucca0\uc740 \uc778\ucc9c \ub3c4\uc2dc\ucca0\ub3c4, \uc778\ucc9c\uacf5\ud56d, \uae40\ud3ec\uacf5\ud56d\uacfc\ub3c4 \uc5f0\uacb0\ub41c\ub2e4. 1985\ub144 \uac1c\ud1b5\ub41c \ub450 \ubc88\uc9f8\ub85c \ud070 \ubd80\uc0b0 \ub3c4\uc2dc\ucca0\ub3c4\ub294 \ucd1d \uae38\uc774 131.7km\uc5d0 1\ud638\uc120, 2\ud638\uc120, 3\ud638\uc120, 4\ud638\uc120, \ubd80\uc0b0\uae40\ud574\uacbd\uc804\ucca0 5\uac1c \ub178\uc120\uc774 \uc6b4\ud589\ub418\uace0 \uc788\uc73c\uba70, \uc774\uc678\uc5d0\ub3c4 \ub300\uad6c\uc9c0\ud558\ucca0 1\ud638\uc120, 2\ud638\uc120 2\uac1c \ub178\uc120\uc774 \uc6b4\ud589\ub418\uace0 \ub300\uc804\uc9c0\ud558\ucca0 1\ud638\uc120 1\uac1c\ub178\uc120\uc774 \uc6b4\uc601, \uad11\uc8fc1\ud638\uc120 1\uac1c \ub178\uc120 \uc6b4\ud589 \ub4f1 \uc77c\ubd80 \uad11\uc5ed\uc2dc\ub4e4\uc744 \uc911\uc2ec\uc73c\ub85c \ub3c4\uc2dc\ucca0\ub3c4\uac00 \uacc4\uc18d\ub418\uc5b4 \uac74\uc124\uc911\uc774\ub2e4. \uc694\uae08\uc740 \uac01 \uc9c0\uc790\uccb4\uc5d0\uc11c \ub530\ub85c \uaddc\uc815\ud558\uc5ec \uc870\uae08\uc529 \ub2e4\ub974\ub2e4. \ubcf5\uc9c0 \uc815\ucc45\uc73c\ub85c 65\uc138 \uc774\uc0c1\uc758 \ub178\uc778\uacfc \uc7a5\uc560\uc778, \uad6d\uac00\uc720\uacf5\uc790\ub294 \ubb34\uc784 \uc73c\ub85c \uc2b9\ucc28\ud560 \uc218 \uc788\ub2e4. (\ubd80\uc0b0-\uae40\ud574 \uacbd\uc804\ucca0\uc740 \ub178\uc778\ub3c4 \uc720\uc784\uc2b9\ucc28)", "paragraph_answer": "\ub300\ud55c\ubbfc\uad6d\uc758 \ub3c4\uc2dc\ucca0\ub3c4\ub294 \uc218\ub3c4\uad8c\uc9c0\uc5ed\uacfc \ubd80\uc0b0\uc9c0\uc5ed, \ub300\uad6c\uad11\uc5ed\uc2dc, \ub300\uc804\uad11\uc5ed\uc2dc, \uad11\uc8fc\uad11\uc5ed\uc2dc \ub4f1\uc5d0\uc11c \uc6b4\ud589\ub418\uace0 \uc788\ub2e4. \uc2dc\ub0b4\ub97c \uc6b4\ud589\ud558\ub294 \ub3c4\uc2dc\ucca0\ub3c4\ub97c \uc77c\uceec\uc5b4 '\uc9c0\ud558\ucca0' \ub610\ub294 '\uc804\ucca0' \ub4f1\uc73c\ub85c \ubd80\ub974\uae30\ub3c4 \ud55c\ub2e4. 1974\ub144 8\uc6d4 15\uc77c \ucc98\uc74c\uc73c\ub85c \uac1c\ud1b5\ub41c \uc218\ub3c4\uad8c\uc9c0\uc5ed\uc740 1~9\ud638\uc120\uacfc \ubd84\ub2f9\uc120, \uc911\uc559\uc120, \uacbd\uc758\uc120, \uacbd\ucd98\uc120, \uc778\ucc9c 1\ud638\uc120, \uc778\ucc9c\uad6d\uc81c\uacf5\ud56d\ucca0\ub3c4, \uc2e0\ubd84\ub2f9\uc120, \uc218\uc778\uc120, \uc6a9\uc778\uacbd\uc804\ucca0, \uc758\uc815\ubd80\uacbd\uc804\ucca0 \uae4c\uc9c0 \ucd1d 19\uac1c \ub178\uc120\uc774 \uc6b4\ud589\ub418\uace0 \uc788\uc73c\uba70 \uc804\uad6d\uc5d0\uc11c \uac00\uc7a5 \uaddc\ubaa8\uac00 \ud06c\ub2e4. \uae30\ubcf8\uc694\uae08\uc740 \ud3c9\uade0\uc801\uc73c\ub85c \uad50\ud1b5\uce74\ub4dc \uc774\uc6a9\uc2dc \ucd5c\ucd08 10km\uae4c\uc9c0 1050\uc6d0, \uadf8\uc774\ud6c4 5;km \ucd08\uacfc\uc2dc\ub9c8\ub2e4 100\uc6d0\uc529 \ucd94\uac00\ub41c\ub2e4. \uadf8\ub7ec\ub098 \ubbfc\uc601 \ub3c4\uc2dc\ucca0\ub3c4\uc778 \uc2e0\ubd84\ub2f9\uc120\uc740 \uae30\ubcf8\uc694\uae08\uc774 1,750\uc6d0\uc774\uace0, \ub2e4\ub978 \ub178\uc120\uc5d0\uc11c \uc2e0\ubd84\ub2f9\uc120\uc73c\ub85c \ud658\uc2b9\ud560 \uacbd\uc6b0 700\uc6d0\uc774 \ucd94\uac00\ub41c\ub2e4. \uc218\ub3c4\uad8c \uc804\ucca0\uc740 \uc778\ucc9c \ub3c4\uc2dc\ucca0\ub3c4, \uc778\ucc9c\uacf5\ud56d, \uae40\ud3ec\uacf5\ud56d\uacfc\ub3c4 \uc5f0\uacb0\ub41c\ub2e4. 1985\ub144 \uac1c\ud1b5\ub41c \ub450 \ubc88\uc9f8\ub85c \ud070 \ubd80\uc0b0 \ub3c4\uc2dc\ucca0\ub3c4\ub294 \ucd1d \uae38\uc774 131.7km\uc5d0 1\ud638\uc120, 2\ud638\uc120, 3\ud638\uc120, 4\ud638\uc120, \ubd80\uc0b0\uae40\ud574\uacbd\uc804\ucca0 5\uac1c \ub178\uc120\uc774 \uc6b4\ud589\ub418\uace0 \uc788\uc73c\uba70, \uc774\uc678\uc5d0\ub3c4 \ub300\uad6c\uc9c0\ud558\ucca0 1\ud638\uc120, 2\ud638\uc120 2\uac1c \ub178\uc120\uc774 \uc6b4\ud589\ub418\uace0 \ub300\uc804\uc9c0\ud558\ucca0 1\ud638\uc120 1\uac1c\ub178\uc120\uc774 \uc6b4\uc601, \uad11\uc8fc1\ud638\uc120 1\uac1c \ub178\uc120 \uc6b4\ud589 \ub4f1 \uc77c\ubd80 \uad11\uc5ed\uc2dc\ub4e4\uc744 \uc911\uc2ec\uc73c\ub85c \ub3c4\uc2dc\ucca0\ub3c4\uac00 \uacc4\uc18d\ub418\uc5b4 \uac74\uc124\uc911\uc774\ub2e4. \uc694\uae08\uc740 \uac01 \uc9c0\uc790\uccb4\uc5d0\uc11c \ub530\ub85c \uaddc\uc815\ud558\uc5ec \uc870\uae08\uc529 \ub2e4\ub974\ub2e4. \ubcf5\uc9c0 \uc815\ucc45\uc73c\ub85c 65\uc138 \uc774\uc0c1\uc758 \ub178\uc778\uacfc \uc7a5\uc560\uc778, \uad6d\uac00\uc720\uacf5\uc790\ub294 \ubb34\uc784 \uc73c\ub85c \uc2b9\ucc28\ud560 \uc218 \uc788\ub2e4.(\ubd80\uc0b0-\uae40\ud574 \uacbd\uc804\ucca0\uc740 \ub178\uc778\ub3c4 \uc720\uc784\uc2b9\ucc28)", "sentence_answer": "\ubcf5\uc9c0 \uc815\ucc45\uc73c\ub85c 65\uc138 \uc774\uc0c1\uc758 \ub178\uc778\uacfc \uc7a5\uc560\uc778, \uad6d\uac00\uc720\uacf5\uc790\ub294 \ubb34\uc784 \uc73c\ub85c \uc2b9\ucc28\ud560 \uc218 \uc788\ub2e4.", "paragraph_id": "6031861-27-2", "paragraph_question": "question: \ub300\ud55c\ubbfc\uad6d\uc758 \uad6d\uac00\uc720\uacf5\uc790\ub294 \ub3c4\uc2dc\ucca0\ub3c4 \uc694\uae08\uc744 \uc5bc\ub9c8\ub098 \ub0b4\uace0 \ud0d1\uc2b9\ud558\ub294\uac00?, context: \ub300\ud55c\ubbfc\uad6d\uc758 \ub3c4\uc2dc\ucca0\ub3c4\ub294 \uc218\ub3c4\uad8c\uc9c0\uc5ed\uacfc \ubd80\uc0b0\uc9c0\uc5ed, \ub300\uad6c\uad11\uc5ed\uc2dc, \ub300\uc804\uad11\uc5ed\uc2dc, \uad11\uc8fc\uad11\uc5ed\uc2dc \ub4f1\uc5d0\uc11c \uc6b4\ud589\ub418\uace0 \uc788\ub2e4. \uc2dc\ub0b4\ub97c \uc6b4\ud589\ud558\ub294 \ub3c4\uc2dc\ucca0\ub3c4\ub97c \uc77c\uceec\uc5b4 '\uc9c0\ud558\ucca0' \ub610\ub294 '\uc804\ucca0' \ub4f1\uc73c\ub85c \ubd80\ub974\uae30\ub3c4 \ud55c\ub2e4. 1974\ub144 8\uc6d4 15\uc77c \ucc98\uc74c\uc73c\ub85c \uac1c\ud1b5\ub41c \uc218\ub3c4\uad8c\uc9c0\uc5ed\uc740 1~9\ud638\uc120\uacfc \ubd84\ub2f9\uc120, \uc911\uc559\uc120, \uacbd\uc758\uc120, \uacbd\ucd98\uc120, \uc778\ucc9c 1\ud638\uc120, \uc778\ucc9c\uad6d\uc81c\uacf5\ud56d\ucca0\ub3c4, \uc2e0\ubd84\ub2f9\uc120, \uc218\uc778\uc120, \uc6a9\uc778\uacbd\uc804\ucca0, \uc758\uc815\ubd80\uacbd\uc804\ucca0 \uae4c\uc9c0 \ucd1d 19\uac1c \ub178\uc120\uc774 \uc6b4\ud589\ub418\uace0 \uc788\uc73c\uba70 \uc804\uad6d\uc5d0\uc11c \uac00\uc7a5 \uaddc\ubaa8\uac00 \ud06c\ub2e4. \uae30\ubcf8\uc694\uae08\uc740 \ud3c9\uade0\uc801\uc73c\ub85c \uad50\ud1b5\uce74\ub4dc \uc774\uc6a9\uc2dc \ucd5c\ucd08 10km\uae4c\uc9c0 1050\uc6d0, \uadf8\uc774\ud6c4 5;km \ucd08\uacfc\uc2dc\ub9c8\ub2e4 100\uc6d0\uc529 \ucd94\uac00\ub41c\ub2e4. \uadf8\ub7ec\ub098 \ubbfc\uc601 \ub3c4\uc2dc\ucca0\ub3c4\uc778 \uc2e0\ubd84\ub2f9\uc120\uc740 \uae30\ubcf8\uc694\uae08\uc774 1,750\uc6d0\uc774\uace0, \ub2e4\ub978 \ub178\uc120\uc5d0\uc11c \uc2e0\ubd84\ub2f9\uc120\uc73c\ub85c \ud658\uc2b9\ud560 \uacbd\uc6b0 700\uc6d0\uc774 \ucd94\uac00\ub41c\ub2e4. \uc218\ub3c4\uad8c \uc804\ucca0\uc740 \uc778\ucc9c \ub3c4\uc2dc\ucca0\ub3c4, \uc778\ucc9c\uacf5\ud56d, \uae40\ud3ec\uacf5\ud56d\uacfc\ub3c4 \uc5f0\uacb0\ub41c\ub2e4. 1985\ub144 \uac1c\ud1b5\ub41c \ub450 \ubc88\uc9f8\ub85c \ud070 \ubd80\uc0b0 \ub3c4\uc2dc\ucca0\ub3c4\ub294 \ucd1d \uae38\uc774 131.7km\uc5d0 1\ud638\uc120, 2\ud638\uc120, 3\ud638\uc120, 4\ud638\uc120, \ubd80\uc0b0\uae40\ud574\uacbd\uc804\ucca0 5\uac1c \ub178\uc120\uc774 \uc6b4\ud589\ub418\uace0 \uc788\uc73c\uba70, \uc774\uc678\uc5d0\ub3c4 \ub300\uad6c\uc9c0\ud558\ucca0 1\ud638\uc120, 2\ud638\uc120 2\uac1c \ub178\uc120\uc774 \uc6b4\ud589\ub418\uace0 \ub300\uc804\uc9c0\ud558\ucca0 1\ud638\uc120 1\uac1c\ub178\uc120\uc774 \uc6b4\uc601, \uad11\uc8fc1\ud638\uc120 1\uac1c \ub178\uc120 \uc6b4\ud589 \ub4f1 \uc77c\ubd80 \uad11\uc5ed\uc2dc\ub4e4\uc744 \uc911\uc2ec\uc73c\ub85c \ub3c4\uc2dc\ucca0\ub3c4\uac00 \uacc4\uc18d\ub418\uc5b4 \uac74\uc124\uc911\uc774\ub2e4. \uc694\uae08\uc740 \uac01 \uc9c0\uc790\uccb4\uc5d0\uc11c \ub530\ub85c \uaddc\uc815\ud558\uc5ec \uc870\uae08\uc529 \ub2e4\ub974\ub2e4. \ubcf5\uc9c0 \uc815\ucc45\uc73c\ub85c 65\uc138 \uc774\uc0c1\uc758 \ub178\uc778\uacfc \uc7a5\uc560\uc778, \uad6d\uac00\uc720\uacf5\uc790\ub294 \ubb34\uc784\uc73c\ub85c \uc2b9\ucc28\ud560 \uc218 \uc788\ub2e4.(\ubd80\uc0b0-\uae40\ud574 \uacbd\uc804\ucca0\uc740 \ub178\uc778\ub3c4 \uc720\uc784\uc2b9\ucc28)"} {"question": "2007\ub144\ub3c4 \uc804\uc778\ubc94\uc774 \ucd08\uccad\ubc1b\uc740 \ud558\uc640\uc774 \ud638\ub180\ub8f0\uc5d0\uc11c \uc5f4\ub9b0 \uc2ec\ud3ec\uc9c0\uc5c4\uc740??", "paragraph": "2017\ub144 5\uc6d4 23\uc77c\ubd80\ud130 25\uc77c\uae4c\uc9c0 \ubbf8\uad6d \ud558\uc640\uc774 \ud638\ub180\ub8f0\ub8e8\uc5d0\uc11c \uc5f4\ub9b0 \uc544\uc2dc\uc544\ud0dc\ud3c9\uc591\uc9c0\uc0c1\uad70(LANPAC) \uc2ec\ud3ec\uc9c0\uc5c4\uc5d0 \ucd08\uccad\uc744 \ubc1b\uc544 \ud55c\ubbf8\ub3d9\ub9f9\uc758 \uc911\uc694\uc131\uacfc \ubc1c\uc804 \ubc29\uc548\uc5d0 \ub300\ud574 \ubc1c\ud45c\ud558\uc600\ub2e4. \uc804\uc778\ubc94 \uc911\uc7a5\uc740 \uc774\ub840\uc801\uc73c\ub85c \uc608\ube44\uc5ed \ud55c\uad6d \uc7a5\uad70\uc784\uc5d0\ub3c4 \ubd88\uad6c\ud558\uace0 \uac01 \uad6d\uc758 \ud604\uc9c1 \uc721\uad70 \ucd5c\uace0 \uc218\ub1cc\ubd80\uac00 \ucc38\uc11d\ud558\ub294 \uc774 \uc2ec\ud3ec\uc9c0\uc5c4\uc5d0 \ucd08\uccad\uc744 \ubc1b\uc544 \uc608\ube44\uc5ed\uc73c\ub85c\ub294 \uc720\uc77c\ud558\uac8c \ubc1c\ud45c\ud588\ub294\ub370, \ud604\uc9c1 \ubbf8\uad70 \uc8fc\uc694 \uc9c0\ud718\uad00\ub4e4\uc740 \ubb3c\ub860 \uc804\uc9c1 \ubbf8\uad70 \uc218\ub1cc\ubd80\ub4e4\ub85c\ubd80\ud130 \ud55c\u00b7\ubbf8 \uad70\uc0ac\ub3d9\ub9f9\uacfc \ud55c\u00b7\ubbf8 \uc5f0\ud569\uc804\ub825 \uadf9\ub300\ud654 \ud574\ubc95\uc5d0 \ub300\ud55c \uac00\uc7a5 \uc2e0\ub8b0\ud560 \uc218 \uc788\ub294 \ud55c\uad6d\uad70 \ucd9c\uc2e0\uc73c\ub85c \ud3c9\uac00\ubc1b\uace0 \uc788\uc5b4 \uc774 \uc2ec\ud3ec\uc9c0\uc5c4\uc5d0\ub3c4 \ucd08\uccad\ub418\uc5c8\ub2e4. \uadf8\ub294 \ubc1c\ud45c\ubb38\uc5d0\uc11c \u201c\ud55c\ubbf8\uac04 \uae34\ubc00\ud55c \uc5f0\ud569\uc791\uc804\uacfc \uc9c0\ud718\ud1b5\uc81c\ub97c \uac15\ud654\ud558\uae30 \uc704\ud574\uc11c\ub294 \ud6a8\uc728\uc801\uc778 \uc804\uc220\uc9c0\ud718\ud1b5\uc81c\uc790\ub3d9\ud654\uccb4\uacc4(C\u2074I)\uac00 \uae30\ubc18\ub418\uc5b4\uc57c \ud558\uba70 \uc591\uad6d\uac04 \uc5b8\uc5b4\uc7a5\ubcbd\uc758 \ubb38\uc81c\ub97c \ubcf4\ub2e4 \uae4a\uc774 \uc788\uac8c \uc774\ud574\ud558\uace0 \ubb38\ud654\uc758 \ucc28\uc774\ub97c \uc774\ud574\ud558\ub294 \ub4f1 \uc0c1\ud638 \uc774\ud574\ub97c \uc99d\uc9c4\uc2dc\ud0a4\uae30 \uc704\ud55c \ub178\ub825\uc774 \ud544\uc694\ud558\ub2e4\u201d\ub294 \uc810\uc744 \uc5ed\uc124\ud558\uc600\uace0, \u201c\ubbf8\uad6d\uc774 \uc6b0\ub9ac\ub098\ub77c\uc758 \uc8fc\ub3c4\uc801 \uc5ed\ud560\uc744 \uc778\uc815\ud558\uace0 \uc6b0\ub9ac\ub098\ub77c\ub294 \uc8fc\ub3c4\uc801 \uc5ed\ud560\uc744 \ud560 \uc218 \uc788\ub294 \ub2a5\ub825\uc744 \ud0a4\uc6cc\uc57c \ud55c\ub2e4\u201d\uace0 \uc8fc\uc7a5\ud588\ub2e4. \ub610\ud55c \uc2ec\ud3ec\uc9c0\uc5c4\uc774 \ub05d\ub098\uace0 \uc5f0\ud569\ub274\uc2a4\uc640 \uac00\uc9c4 \uc778\ud130\ubdf0\uc5d0\uc11c \u201c\ubbf8\uad70 \ubc1c\ud45c\uc790\ub4e4 \ub300\ubd80\ubd84\uc740 \uae30\uc870\uc5f0\uc124\uacfc \ubc1c\ud45c\ubb38\uc5d0\uc11c \ubd81\ud55c\uc744 \u2018\ubd80\uc0c1\ud558\ub294 \uc801(rising adversary)\u2019\uc73c\ub85c \uc9c0\uce6d\ud588\ub2e4\u201d\uba74\uc11c \u201c\ubbf8\uad6d\uc774 \ud575\uacfc \ubbf8\uc0ac\uc77c \uac1c\ubc1c\uc5d0 \uc9c8\uc8fc\ud558\ub294 \ubd81\ud55c\uc744 \uc5b4\ub5bb\uac8c \uc778\uc2dd\ud558\uace0 \uc788\ub294\uc9c0\ub97c \uc798 \ubcf4\uc5ec\uc8fc\ub294 \ub9d0\u201d\uc774\ub77c\uace0 \uc804\ud588\ub2e4.", "answer": "\uc544\uc2dc\uc544\ud0dc\ud3c9\uc591\uc9c0\uc0c1\uad70", "sentence": "2017\ub144 5\uc6d4 23\uc77c\ubd80\ud130 25\uc77c\uae4c\uc9c0 \ubbf8\uad6d \ud558\uc640\uc774 \ud638\ub180\ub8f0\ub8e8\uc5d0\uc11c \uc5f4\ub9b0 \uc544\uc2dc\uc544\ud0dc\ud3c9\uc591\uc9c0\uc0c1\uad70 (LANPAC) \uc2ec\ud3ec\uc9c0\uc5c4\uc5d0 \ucd08\uccad\uc744 \ubc1b\uc544 \ud55c\ubbf8\ub3d9\ub9f9\uc758 \uc911\uc694\uc131\uacfc \ubc1c\uc804 \ubc29\uc548\uc5d0 \ub300\ud574 \ubc1c\ud45c\ud558\uc600\ub2e4.", "paragraph_sentence": " 2017\ub144 5\uc6d4 23\uc77c\ubd80\ud130 25\uc77c\uae4c\uc9c0 \ubbf8\uad6d \ud558\uc640\uc774 \ud638\ub180\ub8f0\ub8e8\uc5d0\uc11c \uc5f4\ub9b0 \uc544\uc2dc\uc544\ud0dc\ud3c9\uc591\uc9c0\uc0c1\uad70 (LANPAC) \uc2ec\ud3ec\uc9c0\uc5c4\uc5d0 \ucd08\uccad\uc744 \ubc1b\uc544 \ud55c\ubbf8\ub3d9\ub9f9\uc758 \uc911\uc694\uc131\uacfc \ubc1c\uc804 \ubc29\uc548\uc5d0 \ub300\ud574 \ubc1c\ud45c\ud558\uc600\ub2e4. \uc804\uc778\ubc94 \uc911\uc7a5\uc740 \uc774\ub840\uc801\uc73c\ub85c \uc608\ube44\uc5ed \ud55c\uad6d \uc7a5\uad70\uc784\uc5d0\ub3c4 \ubd88\uad6c\ud558\uace0 \uac01 \uad6d\uc758 \ud604\uc9c1 \uc721\uad70 \ucd5c\uace0 \uc218\ub1cc\ubd80\uac00 \ucc38\uc11d\ud558\ub294 \uc774 \uc2ec\ud3ec\uc9c0\uc5c4\uc5d0 \ucd08\uccad\uc744 \ubc1b\uc544 \uc608\ube44\uc5ed\uc73c\ub85c\ub294 \uc720\uc77c\ud558\uac8c \ubc1c\ud45c\ud588\ub294\ub370, \ud604\uc9c1 \ubbf8\uad70 \uc8fc\uc694 \uc9c0\ud718\uad00\ub4e4\uc740 \ubb3c\ub860 \uc804\uc9c1 \ubbf8\uad70 \uc218\ub1cc\ubd80\ub4e4\ub85c\ubd80\ud130 \ud55c\u00b7\ubbf8 \uad70\uc0ac\ub3d9\ub9f9\uacfc \ud55c\u00b7\ubbf8 \uc5f0\ud569\uc804\ub825 \uadf9\ub300\ud654 \ud574\ubc95\uc5d0 \ub300\ud55c \uac00\uc7a5 \uc2e0\ub8b0\ud560 \uc218 \uc788\ub294 \ud55c\uad6d\uad70 \ucd9c\uc2e0\uc73c\ub85c \ud3c9\uac00\ubc1b\uace0 \uc788\uc5b4 \uc774 \uc2ec\ud3ec\uc9c0\uc5c4\uc5d0\ub3c4 \ucd08\uccad\ub418\uc5c8\ub2e4. \uadf8\ub294 \ubc1c\ud45c\ubb38\uc5d0\uc11c \u201c\ud55c\ubbf8\uac04 \uae34\ubc00\ud55c \uc5f0\ud569\uc791\uc804\uacfc \uc9c0\ud718\ud1b5\uc81c\ub97c \uac15\ud654\ud558\uae30 \uc704\ud574\uc11c\ub294 \ud6a8\uc728\uc801\uc778 \uc804\uc220\uc9c0\ud718\ud1b5\uc81c\uc790\ub3d9\ud654\uccb4\uacc4(C\u2074I)\uac00 \uae30\ubc18\ub418\uc5b4\uc57c \ud558\uba70 \uc591\uad6d\uac04 \uc5b8\uc5b4\uc7a5\ubcbd\uc758 \ubb38\uc81c\ub97c \ubcf4\ub2e4 \uae4a\uc774 \uc788\uac8c \uc774\ud574\ud558\uace0 \ubb38\ud654\uc758 \ucc28\uc774\ub97c \uc774\ud574\ud558\ub294 \ub4f1 \uc0c1\ud638 \uc774\ud574\ub97c \uc99d\uc9c4\uc2dc\ud0a4\uae30 \uc704\ud55c \ub178\ub825\uc774 \ud544\uc694\ud558\ub2e4\u201d\ub294 \uc810\uc744 \uc5ed\uc124\ud558\uc600\uace0, \u201c\ubbf8\uad6d\uc774 \uc6b0\ub9ac\ub098\ub77c\uc758 \uc8fc\ub3c4\uc801 \uc5ed\ud560\uc744 \uc778\uc815\ud558\uace0 \uc6b0\ub9ac\ub098\ub77c\ub294 \uc8fc\ub3c4\uc801 \uc5ed\ud560\uc744 \ud560 \uc218 \uc788\ub294 \ub2a5\ub825\uc744 \ud0a4\uc6cc\uc57c \ud55c\ub2e4\u201d\uace0 \uc8fc\uc7a5\ud588\ub2e4. \ub610\ud55c \uc2ec\ud3ec\uc9c0\uc5c4\uc774 \ub05d\ub098\uace0 \uc5f0\ud569\ub274\uc2a4\uc640 \uac00\uc9c4 \uc778\ud130\ubdf0\uc5d0\uc11c \u201c\ubbf8\uad70 \ubc1c\ud45c\uc790\ub4e4 \ub300\ubd80\ubd84\uc740 \uae30\uc870\uc5f0\uc124\uacfc \ubc1c\ud45c\ubb38\uc5d0\uc11c \ubd81\ud55c\uc744 \u2018\ubd80\uc0c1\ud558\ub294 \uc801(rising adversary)\u2019\uc73c\ub85c \uc9c0\uce6d\ud588\ub2e4\u201d\uba74\uc11c \u201c\ubbf8\uad6d\uc774 \ud575\uacfc \ubbf8\uc0ac\uc77c \uac1c\ubc1c\uc5d0 \uc9c8\uc8fc\ud558\ub294 \ubd81\ud55c\uc744 \uc5b4\ub5bb\uac8c \uc778\uc2dd\ud558\uace0 \uc788\ub294\uc9c0\ub97c \uc798 \ubcf4\uc5ec\uc8fc\ub294 \ub9d0\u201d\uc774\ub77c\uace0 \uc804\ud588\ub2e4.", "paragraph_answer": "2017\ub144 5\uc6d4 23\uc77c\ubd80\ud130 25\uc77c\uae4c\uc9c0 \ubbf8\uad6d \ud558\uc640\uc774 \ud638\ub180\ub8f0\ub8e8\uc5d0\uc11c \uc5f4\ub9b0 \uc544\uc2dc\uc544\ud0dc\ud3c9\uc591\uc9c0\uc0c1\uad70 (LANPAC) \uc2ec\ud3ec\uc9c0\uc5c4\uc5d0 \ucd08\uccad\uc744 \ubc1b\uc544 \ud55c\ubbf8\ub3d9\ub9f9\uc758 \uc911\uc694\uc131\uacfc \ubc1c\uc804 \ubc29\uc548\uc5d0 \ub300\ud574 \ubc1c\ud45c\ud558\uc600\ub2e4. \uc804\uc778\ubc94 \uc911\uc7a5\uc740 \uc774\ub840\uc801\uc73c\ub85c \uc608\ube44\uc5ed \ud55c\uad6d \uc7a5\uad70\uc784\uc5d0\ub3c4 \ubd88\uad6c\ud558\uace0 \uac01 \uad6d\uc758 \ud604\uc9c1 \uc721\uad70 \ucd5c\uace0 \uc218\ub1cc\ubd80\uac00 \ucc38\uc11d\ud558\ub294 \uc774 \uc2ec\ud3ec\uc9c0\uc5c4\uc5d0 \ucd08\uccad\uc744 \ubc1b\uc544 \uc608\ube44\uc5ed\uc73c\ub85c\ub294 \uc720\uc77c\ud558\uac8c \ubc1c\ud45c\ud588\ub294\ub370, \ud604\uc9c1 \ubbf8\uad70 \uc8fc\uc694 \uc9c0\ud718\uad00\ub4e4\uc740 \ubb3c\ub860 \uc804\uc9c1 \ubbf8\uad70 \uc218\ub1cc\ubd80\ub4e4\ub85c\ubd80\ud130 \ud55c\u00b7\ubbf8 \uad70\uc0ac\ub3d9\ub9f9\uacfc \ud55c\u00b7\ubbf8 \uc5f0\ud569\uc804\ub825 \uadf9\ub300\ud654 \ud574\ubc95\uc5d0 \ub300\ud55c \uac00\uc7a5 \uc2e0\ub8b0\ud560 \uc218 \uc788\ub294 \ud55c\uad6d\uad70 \ucd9c\uc2e0\uc73c\ub85c \ud3c9\uac00\ubc1b\uace0 \uc788\uc5b4 \uc774 \uc2ec\ud3ec\uc9c0\uc5c4\uc5d0\ub3c4 \ucd08\uccad\ub418\uc5c8\ub2e4. \uadf8\ub294 \ubc1c\ud45c\ubb38\uc5d0\uc11c \u201c\ud55c\ubbf8\uac04 \uae34\ubc00\ud55c \uc5f0\ud569\uc791\uc804\uacfc \uc9c0\ud718\ud1b5\uc81c\ub97c \uac15\ud654\ud558\uae30 \uc704\ud574\uc11c\ub294 \ud6a8\uc728\uc801\uc778 \uc804\uc220\uc9c0\ud718\ud1b5\uc81c\uc790\ub3d9\ud654\uccb4\uacc4(C\u2074I)\uac00 \uae30\ubc18\ub418\uc5b4\uc57c \ud558\uba70 \uc591\uad6d\uac04 \uc5b8\uc5b4\uc7a5\ubcbd\uc758 \ubb38\uc81c\ub97c \ubcf4\ub2e4 \uae4a\uc774 \uc788\uac8c \uc774\ud574\ud558\uace0 \ubb38\ud654\uc758 \ucc28\uc774\ub97c \uc774\ud574\ud558\ub294 \ub4f1 \uc0c1\ud638 \uc774\ud574\ub97c \uc99d\uc9c4\uc2dc\ud0a4\uae30 \uc704\ud55c \ub178\ub825\uc774 \ud544\uc694\ud558\ub2e4\u201d\ub294 \uc810\uc744 \uc5ed\uc124\ud558\uc600\uace0, \u201c\ubbf8\uad6d\uc774 \uc6b0\ub9ac\ub098\ub77c\uc758 \uc8fc\ub3c4\uc801 \uc5ed\ud560\uc744 \uc778\uc815\ud558\uace0 \uc6b0\ub9ac\ub098\ub77c\ub294 \uc8fc\ub3c4\uc801 \uc5ed\ud560\uc744 \ud560 \uc218 \uc788\ub294 \ub2a5\ub825\uc744 \ud0a4\uc6cc\uc57c \ud55c\ub2e4\u201d\uace0 \uc8fc\uc7a5\ud588\ub2e4. \ub610\ud55c \uc2ec\ud3ec\uc9c0\uc5c4\uc774 \ub05d\ub098\uace0 \uc5f0\ud569\ub274\uc2a4\uc640 \uac00\uc9c4 \uc778\ud130\ubdf0\uc5d0\uc11c \u201c\ubbf8\uad70 \ubc1c\ud45c\uc790\ub4e4 \ub300\ubd80\ubd84\uc740 \uae30\uc870\uc5f0\uc124\uacfc \ubc1c\ud45c\ubb38\uc5d0\uc11c \ubd81\ud55c\uc744 \u2018\ubd80\uc0c1\ud558\ub294 \uc801(rising adversary)\u2019\uc73c\ub85c \uc9c0\uce6d\ud588\ub2e4\u201d\uba74\uc11c \u201c\ubbf8\uad6d\uc774 \ud575\uacfc \ubbf8\uc0ac\uc77c \uac1c\ubc1c\uc5d0 \uc9c8\uc8fc\ud558\ub294 \ubd81\ud55c\uc744 \uc5b4\ub5bb\uac8c \uc778\uc2dd\ud558\uace0 \uc788\ub294\uc9c0\ub97c \uc798 \ubcf4\uc5ec\uc8fc\ub294 \ub9d0\u201d\uc774\ub77c\uace0 \uc804\ud588\ub2e4.", "sentence_answer": "2017\ub144 5\uc6d4 23\uc77c\ubd80\ud130 25\uc77c\uae4c\uc9c0 \ubbf8\uad6d \ud558\uc640\uc774 \ud638\ub180\ub8f0\ub8e8\uc5d0\uc11c \uc5f4\ub9b0 \uc544\uc2dc\uc544\ud0dc\ud3c9\uc591\uc9c0\uc0c1\uad70 (LANPAC) \uc2ec\ud3ec\uc9c0\uc5c4\uc5d0 \ucd08\uccad\uc744 \ubc1b\uc544 \ud55c\ubbf8\ub3d9\ub9f9\uc758 \uc911\uc694\uc131\uacfc \ubc1c\uc804 \ubc29\uc548\uc5d0 \ub300\ud574 \ubc1c\ud45c\ud558\uc600\ub2e4.", "paragraph_id": "6593261-15-0", "paragraph_question": "question: 2007\ub144\ub3c4 \uc804\uc778\ubc94\uc774 \ucd08\uccad\ubc1b\uc740 \ud558\uc640\uc774 \ud638\ub180\ub8f0\uc5d0\uc11c \uc5f4\ub9b0 \uc2ec\ud3ec\uc9c0\uc5c4\uc740??, context: 2017\ub144 5\uc6d4 23\uc77c\ubd80\ud130 25\uc77c\uae4c\uc9c0 \ubbf8\uad6d \ud558\uc640\uc774 \ud638\ub180\ub8f0\ub8e8\uc5d0\uc11c \uc5f4\ub9b0 \uc544\uc2dc\uc544\ud0dc\ud3c9\uc591\uc9c0\uc0c1\uad70(LANPAC) \uc2ec\ud3ec\uc9c0\uc5c4\uc5d0 \ucd08\uccad\uc744 \ubc1b\uc544 \ud55c\ubbf8\ub3d9\ub9f9\uc758 \uc911\uc694\uc131\uacfc \ubc1c\uc804 \ubc29\uc548\uc5d0 \ub300\ud574 \ubc1c\ud45c\ud558\uc600\ub2e4. \uc804\uc778\ubc94 \uc911\uc7a5\uc740 \uc774\ub840\uc801\uc73c\ub85c \uc608\ube44\uc5ed \ud55c\uad6d \uc7a5\uad70\uc784\uc5d0\ub3c4 \ubd88\uad6c\ud558\uace0 \uac01 \uad6d\uc758 \ud604\uc9c1 \uc721\uad70 \ucd5c\uace0 \uc218\ub1cc\ubd80\uac00 \ucc38\uc11d\ud558\ub294 \uc774 \uc2ec\ud3ec\uc9c0\uc5c4\uc5d0 \ucd08\uccad\uc744 \ubc1b\uc544 \uc608\ube44\uc5ed\uc73c\ub85c\ub294 \uc720\uc77c\ud558\uac8c \ubc1c\ud45c\ud588\ub294\ub370, \ud604\uc9c1 \ubbf8\uad70 \uc8fc\uc694 \uc9c0\ud718\uad00\ub4e4\uc740 \ubb3c\ub860 \uc804\uc9c1 \ubbf8\uad70 \uc218\ub1cc\ubd80\ub4e4\ub85c\ubd80\ud130 \ud55c\u00b7\ubbf8 \uad70\uc0ac\ub3d9\ub9f9\uacfc \ud55c\u00b7\ubbf8 \uc5f0\ud569\uc804\ub825 \uadf9\ub300\ud654 \ud574\ubc95\uc5d0 \ub300\ud55c \uac00\uc7a5 \uc2e0\ub8b0\ud560 \uc218 \uc788\ub294 \ud55c\uad6d\uad70 \ucd9c\uc2e0\uc73c\ub85c \ud3c9\uac00\ubc1b\uace0 \uc788\uc5b4 \uc774 \uc2ec\ud3ec\uc9c0\uc5c4\uc5d0\ub3c4 \ucd08\uccad\ub418\uc5c8\ub2e4. \uadf8\ub294 \ubc1c\ud45c\ubb38\uc5d0\uc11c \u201c\ud55c\ubbf8\uac04 \uae34\ubc00\ud55c \uc5f0\ud569\uc791\uc804\uacfc \uc9c0\ud718\ud1b5\uc81c\ub97c \uac15\ud654\ud558\uae30 \uc704\ud574\uc11c\ub294 \ud6a8\uc728\uc801\uc778 \uc804\uc220\uc9c0\ud718\ud1b5\uc81c\uc790\ub3d9\ud654\uccb4\uacc4(C\u2074I)\uac00 \uae30\ubc18\ub418\uc5b4\uc57c \ud558\uba70 \uc591\uad6d\uac04 \uc5b8\uc5b4\uc7a5\ubcbd\uc758 \ubb38\uc81c\ub97c \ubcf4\ub2e4 \uae4a\uc774 \uc788\uac8c \uc774\ud574\ud558\uace0 \ubb38\ud654\uc758 \ucc28\uc774\ub97c \uc774\ud574\ud558\ub294 \ub4f1 \uc0c1\ud638 \uc774\ud574\ub97c \uc99d\uc9c4\uc2dc\ud0a4\uae30 \uc704\ud55c \ub178\ub825\uc774 \ud544\uc694\ud558\ub2e4\u201d\ub294 \uc810\uc744 \uc5ed\uc124\ud558\uc600\uace0, \u201c\ubbf8\uad6d\uc774 \uc6b0\ub9ac\ub098\ub77c\uc758 \uc8fc\ub3c4\uc801 \uc5ed\ud560\uc744 \uc778\uc815\ud558\uace0 \uc6b0\ub9ac\ub098\ub77c\ub294 \uc8fc\ub3c4\uc801 \uc5ed\ud560\uc744 \ud560 \uc218 \uc788\ub294 \ub2a5\ub825\uc744 \ud0a4\uc6cc\uc57c \ud55c\ub2e4\u201d\uace0 \uc8fc\uc7a5\ud588\ub2e4. \ub610\ud55c \uc2ec\ud3ec\uc9c0\uc5c4\uc774 \ub05d\ub098\uace0 \uc5f0\ud569\ub274\uc2a4\uc640 \uac00\uc9c4 \uc778\ud130\ubdf0\uc5d0\uc11c \u201c\ubbf8\uad70 \ubc1c\ud45c\uc790\ub4e4 \ub300\ubd80\ubd84\uc740 \uae30\uc870\uc5f0\uc124\uacfc \ubc1c\ud45c\ubb38\uc5d0\uc11c \ubd81\ud55c\uc744 \u2018\ubd80\uc0c1\ud558\ub294 \uc801(rising adversary)\u2019\uc73c\ub85c \uc9c0\uce6d\ud588\ub2e4\u201d\uba74\uc11c \u201c\ubbf8\uad6d\uc774 \ud575\uacfc \ubbf8\uc0ac\uc77c \uac1c\ubc1c\uc5d0 \uc9c8\uc8fc\ud558\ub294 \ubd81\ud55c\uc744 \uc5b4\ub5bb\uac8c \uc778\uc2dd\ud558\uace0 \uc788\ub294\uc9c0\ub97c \uc798 \ubcf4\uc5ec\uc8fc\ub294 \ub9d0\u201d\uc774\ub77c\uace0 \uc804\ud588\ub2e4."} {"question": "\ub354\ubd88\uc5b4\ubbfc\uc8fc\ub2f9\uacfc \uad6d\ubbfc\uc758\ub2f9\uacfc \ub2ec\ub9ac \ub2f9 \ucc28\uc6d0\uc758 \uc9d1\ud68c \ucc38\uc5ec\uc640 \ub2f9 \uc9c0\ub3c4\ubd80 \ubaa8\ub450\uac00 \ucc38\uc11d\ud558\uaca0\ub2e4\ub294 \ub2f9 \uc774\ub984\uc740?", "paragraph": "\uc9d1\ud68c\uc5d0 \uc55e\uc11c \ub354\ubd88\uc5b4\ubbfc\uc8fc\ub2f9\uacfc \uad6d\ubbfc\uc758\ub2f9 \uc9c0\ub3c4\ubd80\ub294 \ub2f9 \ucc28\uc6d0\uc5d0\uc11c \ucc38\uc11d\ud558\uc9c0 \uc54a\ub294\ub2e4\ub294 \uc785\uc7a5\uc744 \ubc1d\ud614\ub2e4. \uc6b0\uc0c1\ud638 \ubbfc\uc8fc\ub2f9 \ub300\ud45c\ub294 \"\uc544\uc9c1 \uc7a5\uc678\ub85c\ub294 \ub098\uc11c\uc9c0 \uc54a\uaca0\ub2e4\"\ub294 \uc785\uc7a5\uc744 \ubc1d\ud614\uace0, \ubc15\uc9c0\uc6d0 \uad6d\ubbfc\uc758\ub2f9 \uc6d0\ub0b4\ub300\ud45c\ub3c4 \"\uc544\uc9c1 \uc815\uce58\uad8c\uc5d0\uc11c \uadf8\ub807\uac8c \ub098\uc11c\uc11c\ub294 \uc548\ub41c\ub2e4\ub294 \uc0dd\uac01\"\uc774\ub77c\uace0 \ubc1d\ud614\ub2e4. \uc774\ubc16\uc5d0\ub3c4 \ubc15\uc6d0\uc21c \uc11c\uc6b8\uc2dc\uc7a5, \ubb38\uc7ac\uc778 \ub354\ubd88\uc5b4\ubbfc\uc8fc\ub2f9 \uc804 \ub300\ud45c, \uc548\ud76c\uc815 \ucda9\uccad\ub0a8\ub3c4\uc9c0\uc0ac \ub4f1\ub3c4 \ubd88\ucc38 \uc758\uc0ac\ub97c \ubc1d\ud614\ub2e4. \uc774\uc5d0 \ub300\ud574\uc11c\ub294 \"\ubb34\uc870\uac74\uc801\uc778 \ube44\ud310\ubcf4\ub2e4 \uc218\uad8c \uc815\ub2f9\uc73c\ub85c\uc11c \ub300\uc548\uc744 \uc81c\uc2dc\ud574\uc57c \ud55c\ub2e4\"\ub294 \ucc45\uc784\uac10\uacfc, \"\ud0c4\ud575\uc774\ub098 \ud558\uc57c \ub4f1 \uc9d1\ud68c \ucc38\uac00\uc790\ub4e4\uc758 \uc694\uad6c\uc5d0 \ub300\ud55c \uc815\uce58\uc801 \ubd80\ub2f4 \ub54c\ubb38\"\uc774\ub77c\ub294 \uc2dc\uac01\ub3c4 \uc788\uc5c8\ub2e4. \ubc18\uba74 \uc815\uc758\ub2f9\uc740 \ub2f9 \ucc28\uc6d0\uc5d0\uc11c \uc9d1\ud68c \ucc38\uc5ec\ub97c \uacb0\uc815\ud558\uace0, \ub2f9 \uc9c0\ub3c4\ubd80\uac00 \ubaa8\ub450 \ucc38\uc11d\ud558\uaca0\ub2e4\ub294 \uc785\uc7a5\uc744 \ubc1d\ud614\ub2e4.", "answer": "\uc815\uc758\ub2f9", "sentence": "\ubc18\uba74 \uc815\uc758\ub2f9 \uc740 \ub2f9 \ucc28\uc6d0\uc5d0\uc11c \uc9d1\ud68c \ucc38\uc5ec\ub97c \uacb0\uc815\ud558\uace0, \ub2f9 \uc9c0\ub3c4\ubd80\uac00 \ubaa8\ub450 \ucc38\uc11d\ud558\uaca0\ub2e4\ub294 \uc785\uc7a5\uc744 \ubc1d\ud614\ub2e4.", "paragraph_sentence": "\uc9d1\ud68c\uc5d0 \uc55e\uc11c \ub354\ubd88\uc5b4\ubbfc\uc8fc\ub2f9\uacfc \uad6d\ubbfc\uc758\ub2f9 \uc9c0\ub3c4\ubd80\ub294 \ub2f9 \ucc28\uc6d0\uc5d0\uc11c \ucc38\uc11d\ud558\uc9c0 \uc54a\ub294\ub2e4\ub294 \uc785\uc7a5\uc744 \ubc1d\ud614\ub2e4. \uc6b0\uc0c1\ud638 \ubbfc\uc8fc\ub2f9 \ub300\ud45c\ub294 \"\uc544\uc9c1 \uc7a5\uc678\ub85c\ub294 \ub098\uc11c\uc9c0 \uc54a\uaca0\ub2e4\"\ub294 \uc785\uc7a5\uc744 \ubc1d\ud614\uace0, \ubc15\uc9c0\uc6d0 \uad6d\ubbfc\uc758\ub2f9 \uc6d0\ub0b4\ub300\ud45c\ub3c4 \"\uc544\uc9c1 \uc815\uce58\uad8c\uc5d0\uc11c \uadf8\ub807\uac8c \ub098\uc11c\uc11c\ub294 \uc548\ub41c\ub2e4\ub294 \uc0dd\uac01\"\uc774\ub77c\uace0 \ubc1d\ud614\ub2e4. \uc774\ubc16\uc5d0\ub3c4 \ubc15\uc6d0\uc21c \uc11c\uc6b8\uc2dc\uc7a5, \ubb38\uc7ac\uc778 \ub354\ubd88\uc5b4\ubbfc\uc8fc\ub2f9 \uc804 \ub300\ud45c, \uc548\ud76c\uc815 \ucda9\uccad\ub0a8\ub3c4\uc9c0\uc0ac \ub4f1\ub3c4 \ubd88\ucc38 \uc758\uc0ac\ub97c \ubc1d\ud614\ub2e4. \uc774\uc5d0 \ub300\ud574\uc11c\ub294 \"\ubb34\uc870\uac74\uc801\uc778 \ube44\ud310\ubcf4\ub2e4 \uc218\uad8c \uc815\ub2f9\uc73c\ub85c\uc11c \ub300\uc548\uc744 \uc81c\uc2dc\ud574\uc57c \ud55c\ub2e4\"\ub294 \ucc45\uc784\uac10\uacfc, \"\ud0c4\ud575\uc774\ub098 \ud558\uc57c \ub4f1 \uc9d1\ud68c \ucc38\uac00\uc790\ub4e4\uc758 \uc694\uad6c\uc5d0 \ub300\ud55c \uc815\uce58\uc801 \ubd80\ub2f4 \ub54c\ubb38\"\uc774\ub77c\ub294 \uc2dc\uac01\ub3c4 \uc788\uc5c8\ub2e4. \ubc18\uba74 \uc815\uc758\ub2f9 \uc740 \ub2f9 \ucc28\uc6d0\uc5d0\uc11c \uc9d1\ud68c \ucc38\uc5ec\ub97c \uacb0\uc815\ud558\uace0, \ub2f9 \uc9c0\ub3c4\ubd80\uac00 \ubaa8\ub450 \ucc38\uc11d\ud558\uaca0\ub2e4\ub294 \uc785\uc7a5\uc744 \ubc1d\ud614\ub2e4. ", "paragraph_answer": "\uc9d1\ud68c\uc5d0 \uc55e\uc11c \ub354\ubd88\uc5b4\ubbfc\uc8fc\ub2f9\uacfc \uad6d\ubbfc\uc758\ub2f9 \uc9c0\ub3c4\ubd80\ub294 \ub2f9 \ucc28\uc6d0\uc5d0\uc11c \ucc38\uc11d\ud558\uc9c0 \uc54a\ub294\ub2e4\ub294 \uc785\uc7a5\uc744 \ubc1d\ud614\ub2e4. \uc6b0\uc0c1\ud638 \ubbfc\uc8fc\ub2f9 \ub300\ud45c\ub294 \"\uc544\uc9c1 \uc7a5\uc678\ub85c\ub294 \ub098\uc11c\uc9c0 \uc54a\uaca0\ub2e4\"\ub294 \uc785\uc7a5\uc744 \ubc1d\ud614\uace0, \ubc15\uc9c0\uc6d0 \uad6d\ubbfc\uc758\ub2f9 \uc6d0\ub0b4\ub300\ud45c\ub3c4 \"\uc544\uc9c1 \uc815\uce58\uad8c\uc5d0\uc11c \uadf8\ub807\uac8c \ub098\uc11c\uc11c\ub294 \uc548\ub41c\ub2e4\ub294 \uc0dd\uac01\"\uc774\ub77c\uace0 \ubc1d\ud614\ub2e4. \uc774\ubc16\uc5d0\ub3c4 \ubc15\uc6d0\uc21c \uc11c\uc6b8\uc2dc\uc7a5, \ubb38\uc7ac\uc778 \ub354\ubd88\uc5b4\ubbfc\uc8fc\ub2f9 \uc804 \ub300\ud45c, \uc548\ud76c\uc815 \ucda9\uccad\ub0a8\ub3c4\uc9c0\uc0ac \ub4f1\ub3c4 \ubd88\ucc38 \uc758\uc0ac\ub97c \ubc1d\ud614\ub2e4. \uc774\uc5d0 \ub300\ud574\uc11c\ub294 \"\ubb34\uc870\uac74\uc801\uc778 \ube44\ud310\ubcf4\ub2e4 \uc218\uad8c \uc815\ub2f9\uc73c\ub85c\uc11c \ub300\uc548\uc744 \uc81c\uc2dc\ud574\uc57c \ud55c\ub2e4\"\ub294 \ucc45\uc784\uac10\uacfc, \"\ud0c4\ud575\uc774\ub098 \ud558\uc57c \ub4f1 \uc9d1\ud68c \ucc38\uac00\uc790\ub4e4\uc758 \uc694\uad6c\uc5d0 \ub300\ud55c \uc815\uce58\uc801 \ubd80\ub2f4 \ub54c\ubb38\"\uc774\ub77c\ub294 \uc2dc\uac01\ub3c4 \uc788\uc5c8\ub2e4. \ubc18\uba74 \uc815\uc758\ub2f9 \uc740 \ub2f9 \ucc28\uc6d0\uc5d0\uc11c \uc9d1\ud68c \ucc38\uc5ec\ub97c \uacb0\uc815\ud558\uace0, \ub2f9 \uc9c0\ub3c4\ubd80\uac00 \ubaa8\ub450 \ucc38\uc11d\ud558\uaca0\ub2e4\ub294 \uc785\uc7a5\uc744 \ubc1d\ud614\ub2e4.", "sentence_answer": "\ubc18\uba74 \uc815\uc758\ub2f9 \uc740 \ub2f9 \ucc28\uc6d0\uc5d0\uc11c \uc9d1\ud68c \ucc38\uc5ec\ub97c \uacb0\uc815\ud558\uace0, \ub2f9 \uc9c0\ub3c4\ubd80\uac00 \ubaa8\ub450 \ucc38\uc11d\ud558\uaca0\ub2e4\ub294 \uc785\uc7a5\uc744 \ubc1d\ud614\ub2e4.", "paragraph_id": "6657023-4-0", "paragraph_question": "question: \ub354\ubd88\uc5b4\ubbfc\uc8fc\ub2f9\uacfc \uad6d\ubbfc\uc758\ub2f9\uacfc \ub2ec\ub9ac \ub2f9 \ucc28\uc6d0\uc758 \uc9d1\ud68c \ucc38\uc5ec\uc640 \ub2f9 \uc9c0\ub3c4\ubd80 \ubaa8\ub450\uac00 \ucc38\uc11d\ud558\uaca0\ub2e4\ub294 \ub2f9 \uc774\ub984\uc740?, context: \uc9d1\ud68c\uc5d0 \uc55e\uc11c \ub354\ubd88\uc5b4\ubbfc\uc8fc\ub2f9\uacfc \uad6d\ubbfc\uc758\ub2f9 \uc9c0\ub3c4\ubd80\ub294 \ub2f9 \ucc28\uc6d0\uc5d0\uc11c \ucc38\uc11d\ud558\uc9c0 \uc54a\ub294\ub2e4\ub294 \uc785\uc7a5\uc744 \ubc1d\ud614\ub2e4. \uc6b0\uc0c1\ud638 \ubbfc\uc8fc\ub2f9 \ub300\ud45c\ub294 \"\uc544\uc9c1 \uc7a5\uc678\ub85c\ub294 \ub098\uc11c\uc9c0 \uc54a\uaca0\ub2e4\"\ub294 \uc785\uc7a5\uc744 \ubc1d\ud614\uace0, \ubc15\uc9c0\uc6d0 \uad6d\ubbfc\uc758\ub2f9 \uc6d0\ub0b4\ub300\ud45c\ub3c4 \"\uc544\uc9c1 \uc815\uce58\uad8c\uc5d0\uc11c \uadf8\ub807\uac8c \ub098\uc11c\uc11c\ub294 \uc548\ub41c\ub2e4\ub294 \uc0dd\uac01\"\uc774\ub77c\uace0 \ubc1d\ud614\ub2e4. \uc774\ubc16\uc5d0\ub3c4 \ubc15\uc6d0\uc21c \uc11c\uc6b8\uc2dc\uc7a5, \ubb38\uc7ac\uc778 \ub354\ubd88\uc5b4\ubbfc\uc8fc\ub2f9 \uc804 \ub300\ud45c, \uc548\ud76c\uc815 \ucda9\uccad\ub0a8\ub3c4\uc9c0\uc0ac \ub4f1\ub3c4 \ubd88\ucc38 \uc758\uc0ac\ub97c \ubc1d\ud614\ub2e4. \uc774\uc5d0 \ub300\ud574\uc11c\ub294 \"\ubb34\uc870\uac74\uc801\uc778 \ube44\ud310\ubcf4\ub2e4 \uc218\uad8c \uc815\ub2f9\uc73c\ub85c\uc11c \ub300\uc548\uc744 \uc81c\uc2dc\ud574\uc57c \ud55c\ub2e4\"\ub294 \ucc45\uc784\uac10\uacfc, \"\ud0c4\ud575\uc774\ub098 \ud558\uc57c \ub4f1 \uc9d1\ud68c \ucc38\uac00\uc790\ub4e4\uc758 \uc694\uad6c\uc5d0 \ub300\ud55c \uc815\uce58\uc801 \ubd80\ub2f4 \ub54c\ubb38\"\uc774\ub77c\ub294 \uc2dc\uac01\ub3c4 \uc788\uc5c8\ub2e4. \ubc18\uba74 \uc815\uc758\ub2f9\uc740 \ub2f9 \ucc28\uc6d0\uc5d0\uc11c \uc9d1\ud68c \ucc38\uc5ec\ub97c \uacb0\uc815\ud558\uace0, \ub2f9 \uc9c0\ub3c4\ubd80\uac00 \ubaa8\ub450 \ucc38\uc11d\ud558\uaca0\ub2e4\ub294 \uc785\uc7a5\uc744 \ubc1d\ud614\ub2e4."} {"question": "\uace0\ub798\uc0c1\uc5b4\uc758 \ub208 \ub4a4\uc5d0\ub294 \ubb34\uc5c7\uc774 \uc788\ub294\uac00?", "paragraph": "\uace0\ub798\uc0c1\uc5b4\ub294 \ud3ed\uc774 1.5m\uae4c\uc9c0 \ub298\uc5b4\ub0a0 \uc218 \uc788\ub294 \uc785\uc744 \uac00\uc84c\uace0, 300\uc5d0\uc11c 350\uac1c\uc758 \uc791\uc740 \uc774\ube68\uc744 \uac00\uc84c\uc73c\uba70, 10\uac1c\uc758 \uc5ec\uacfc \uae30\uad00\uc774 \uc788\ub2e4. \uace0\ub798\uc0c1\uc5b4\ub294 10\uac1c\uc758 \ud070 \uc544\uac00\ubbf8\ub97c \uac00\uc9c0\uace0 \uc788\ub2e4. \uba38\ub9ac\ub294 \ub113\uace0 \ub0a9\uc791\ud558\uba70 \uc55e\uc5d0 2\uac1c\uc758 \uc791\uc740 \ub208\uc774 \ub2ec\ub838\ub2e4.\uace0\ub798\uc0c1\uc5b4\ub294 \uc804\uccb4\uc801\uc73c\ub85c \ud68c\uc0c9\uc778 \ud3b8\uc774\uba70, \ud558\uc580\uc0c9 \ubcf5\ubd80\ub97c \uac00\uc9c0\uace0 \uc788\ub2e4. \ud53c\ubd80\uc5d0\ub294 \uac1c\uccb4\ub9c8\ub2e4 \ub2e4\ub978 \uc605\uc740 \ub178\ub780\uc0c9\uc758 \ubc18\uc810\uacfc \uc904\ubb34\ub2ac\ub97c \uac00\uc9c0\uace0 \uc788\ub2e4. \uace0\ub798\uc0c1\uc5b4\ub294 \uc606\uba74\uc5d0 3\uac1c\uc758 \ub3cc\ucd9c\ub41c \ubd80\ubd84\uc774 \uc788\ub2e4. \ud53c\ubd80\uc758 \ub450\uaed8\ub294 \uc57d 10cm \uc815\ub3c4 \ud55c\ub2e4. \ud55c \uc30d\uc758 \ub4f1\uc9c0\ub290\ub7ec\ubbf8\uc640 \uac00\uc2b4\uc9c0\ub290\ub7ec\ubbf8\ub97c \uac00\uc9c0\uace0 \uc788\uae30\ub3c4 \ud558\ub2e4. \uc131\uc778\uc758 \uaf2c\ub9ac\ub294 \uc57d\uac04 \ucd08\uc2b9\ub2ec \ubaa8\uc591\uc778 \ubc18\uba74, \uce58\uc5b4\uc758 \uaf2c\ub9ac\ub294 \ub0ae\uc740 \ucabd\uc5d0 \uc788\ub294 \uc9c0\ub290\ub7ec\ubbf8\uac00 \uc704\ucabd\uc5d0 \uc788\ub294 \uc9c0\ub290\ub7ec\ubbf8\ubcf4\ub2e4 \uc791\uc740 \ubaa8\uc591\uc774\ub2e4. \uace0\ub798\uc0c1\uc5b4\uc758 \ubd84\uc218\uacf5\uc740 \ub208 \ubc14\ub85c \ub4a4\uc5d0 \uc788\ub2e4.", "answer": "\ubd84\uc218\uacf5", "sentence": "\uace0\ub798\uc0c1\uc5b4\uc758 \ubd84\uc218\uacf5 \uc740 \ub208 \ubc14\ub85c \ub4a4\uc5d0 \uc788\ub2e4.", "paragraph_sentence": "\uace0\ub798\uc0c1\uc5b4\ub294 \ud3ed\uc774 1.5m\uae4c\uc9c0 \ub298\uc5b4\ub0a0 \uc218 \uc788\ub294 \uc785\uc744 \uac00\uc84c\uace0, 300\uc5d0\uc11c 350\uac1c\uc758 \uc791\uc740 \uc774\ube68\uc744 \uac00\uc84c\uc73c\uba70, 10\uac1c\uc758 \uc5ec\uacfc \uae30\uad00\uc774 \uc788\ub2e4. \uace0\ub798\uc0c1\uc5b4\ub294 10\uac1c\uc758 \ud070 \uc544\uac00\ubbf8\ub97c \uac00\uc9c0\uace0 \uc788\ub2e4. \uba38\ub9ac\ub294 \ub113\uace0 \ub0a9\uc791\ud558\uba70 \uc55e\uc5d0 2\uac1c\uc758 \uc791\uc740 \ub208\uc774 \ub2ec\ub838\ub2e4.\uace0\ub798\uc0c1\uc5b4\ub294 \uc804\uccb4\uc801\uc73c\ub85c \ud68c\uc0c9\uc778 \ud3b8\uc774\uba70, \ud558\uc580\uc0c9 \ubcf5\ubd80\ub97c \uac00\uc9c0\uace0 \uc788\ub2e4. \ud53c\ubd80\uc5d0\ub294 \uac1c\uccb4\ub9c8\ub2e4 \ub2e4\ub978 \uc605\uc740 \ub178\ub780\uc0c9\uc758 \ubc18\uc810\uacfc \uc904\ubb34\ub2ac\ub97c \uac00\uc9c0\uace0 \uc788\ub2e4. \uace0\ub798\uc0c1\uc5b4\ub294 \uc606\uba74\uc5d0 3\uac1c\uc758 \ub3cc\ucd9c\ub41c \ubd80\ubd84\uc774 \uc788\ub2e4. \ud53c\ubd80\uc758 \ub450\uaed8\ub294 \uc57d 10cm \uc815\ub3c4 \ud55c\ub2e4. \ud55c \uc30d\uc758 \ub4f1\uc9c0\ub290\ub7ec\ubbf8\uc640 \uac00\uc2b4\uc9c0\ub290\ub7ec\ubbf8\ub97c \uac00\uc9c0\uace0 \uc788\uae30\ub3c4 \ud558\ub2e4. \uc131\uc778\uc758 \uaf2c\ub9ac\ub294 \uc57d\uac04 \ucd08\uc2b9\ub2ec \ubaa8\uc591\uc778 \ubc18\uba74, \uce58\uc5b4\uc758 \uaf2c\ub9ac\ub294 \ub0ae\uc740 \ucabd\uc5d0 \uc788\ub294 \uc9c0\ub290\ub7ec\ubbf8\uac00 \uc704\ucabd\uc5d0 \uc788\ub294 \uc9c0\ub290\ub7ec\ubbf8\ubcf4\ub2e4 \uc791\uc740 \ubaa8\uc591\uc774\ub2e4. \uace0\ub798\uc0c1\uc5b4\uc758 \ubd84\uc218\uacf5 \uc740 \ub208 \ubc14\ub85c \ub4a4\uc5d0 \uc788\ub2e4. ", "paragraph_answer": "\uace0\ub798\uc0c1\uc5b4\ub294 \ud3ed\uc774 1.5m\uae4c\uc9c0 \ub298\uc5b4\ub0a0 \uc218 \uc788\ub294 \uc785\uc744 \uac00\uc84c\uace0, 300\uc5d0\uc11c 350\uac1c\uc758 \uc791\uc740 \uc774\ube68\uc744 \uac00\uc84c\uc73c\uba70, 10\uac1c\uc758 \uc5ec\uacfc \uae30\uad00\uc774 \uc788\ub2e4. \uace0\ub798\uc0c1\uc5b4\ub294 10\uac1c\uc758 \ud070 \uc544\uac00\ubbf8\ub97c \uac00\uc9c0\uace0 \uc788\ub2e4. \uba38\ub9ac\ub294 \ub113\uace0 \ub0a9\uc791\ud558\uba70 \uc55e\uc5d0 2\uac1c\uc758 \uc791\uc740 \ub208\uc774 \ub2ec\ub838\ub2e4.\uace0\ub798\uc0c1\uc5b4\ub294 \uc804\uccb4\uc801\uc73c\ub85c \ud68c\uc0c9\uc778 \ud3b8\uc774\uba70, \ud558\uc580\uc0c9 \ubcf5\ubd80\ub97c \uac00\uc9c0\uace0 \uc788\ub2e4. \ud53c\ubd80\uc5d0\ub294 \uac1c\uccb4\ub9c8\ub2e4 \ub2e4\ub978 \uc605\uc740 \ub178\ub780\uc0c9\uc758 \ubc18\uc810\uacfc \uc904\ubb34\ub2ac\ub97c \uac00\uc9c0\uace0 \uc788\ub2e4. \uace0\ub798\uc0c1\uc5b4\ub294 \uc606\uba74\uc5d0 3\uac1c\uc758 \ub3cc\ucd9c\ub41c \ubd80\ubd84\uc774 \uc788\ub2e4. \ud53c\ubd80\uc758 \ub450\uaed8\ub294 \uc57d 10cm \uc815\ub3c4 \ud55c\ub2e4. \ud55c \uc30d\uc758 \ub4f1\uc9c0\ub290\ub7ec\ubbf8\uc640 \uac00\uc2b4\uc9c0\ub290\ub7ec\ubbf8\ub97c \uac00\uc9c0\uace0 \uc788\uae30\ub3c4 \ud558\ub2e4. \uc131\uc778\uc758 \uaf2c\ub9ac\ub294 \uc57d\uac04 \ucd08\uc2b9\ub2ec \ubaa8\uc591\uc778 \ubc18\uba74, \uce58\uc5b4\uc758 \uaf2c\ub9ac\ub294 \ub0ae\uc740 \ucabd\uc5d0 \uc788\ub294 \uc9c0\ub290\ub7ec\ubbf8\uac00 \uc704\ucabd\uc5d0 \uc788\ub294 \uc9c0\ub290\ub7ec\ubbf8\ubcf4\ub2e4 \uc791\uc740 \ubaa8\uc591\uc774\ub2e4. \uace0\ub798\uc0c1\uc5b4\uc758 \ubd84\uc218\uacf5 \uc740 \ub208 \ubc14\ub85c \ub4a4\uc5d0 \uc788\ub2e4.", "sentence_answer": "\uace0\ub798\uc0c1\uc5b4\uc758 \ubd84\uc218\uacf5 \uc740 \ub208 \ubc14\ub85c \ub4a4\uc5d0 \uc788\ub2e4.", "paragraph_id": "6487061-1-2", "paragraph_question": "question: \uace0\ub798\uc0c1\uc5b4\uc758 \ub208 \ub4a4\uc5d0\ub294 \ubb34\uc5c7\uc774 \uc788\ub294\uac00?, context: \uace0\ub798\uc0c1\uc5b4\ub294 \ud3ed\uc774 1.5m\uae4c\uc9c0 \ub298\uc5b4\ub0a0 \uc218 \uc788\ub294 \uc785\uc744 \uac00\uc84c\uace0, 300\uc5d0\uc11c 350\uac1c\uc758 \uc791\uc740 \uc774\ube68\uc744 \uac00\uc84c\uc73c\uba70, 10\uac1c\uc758 \uc5ec\uacfc \uae30\uad00\uc774 \uc788\ub2e4. \uace0\ub798\uc0c1\uc5b4\ub294 10\uac1c\uc758 \ud070 \uc544\uac00\ubbf8\ub97c \uac00\uc9c0\uace0 \uc788\ub2e4. \uba38\ub9ac\ub294 \ub113\uace0 \ub0a9\uc791\ud558\uba70 \uc55e\uc5d0 2\uac1c\uc758 \uc791\uc740 \ub208\uc774 \ub2ec\ub838\ub2e4.\uace0\ub798\uc0c1\uc5b4\ub294 \uc804\uccb4\uc801\uc73c\ub85c \ud68c\uc0c9\uc778 \ud3b8\uc774\uba70, \ud558\uc580\uc0c9 \ubcf5\ubd80\ub97c \uac00\uc9c0\uace0 \uc788\ub2e4. \ud53c\ubd80\uc5d0\ub294 \uac1c\uccb4\ub9c8\ub2e4 \ub2e4\ub978 \uc605\uc740 \ub178\ub780\uc0c9\uc758 \ubc18\uc810\uacfc \uc904\ubb34\ub2ac\ub97c \uac00\uc9c0\uace0 \uc788\ub2e4. \uace0\ub798\uc0c1\uc5b4\ub294 \uc606\uba74\uc5d0 3\uac1c\uc758 \ub3cc\ucd9c\ub41c \ubd80\ubd84\uc774 \uc788\ub2e4. \ud53c\ubd80\uc758 \ub450\uaed8\ub294 \uc57d 10cm \uc815\ub3c4 \ud55c\ub2e4. \ud55c \uc30d\uc758 \ub4f1\uc9c0\ub290\ub7ec\ubbf8\uc640 \uac00\uc2b4\uc9c0\ub290\ub7ec\ubbf8\ub97c \uac00\uc9c0\uace0 \uc788\uae30\ub3c4 \ud558\ub2e4. \uc131\uc778\uc758 \uaf2c\ub9ac\ub294 \uc57d\uac04 \ucd08\uc2b9\ub2ec \ubaa8\uc591\uc778 \ubc18\uba74, \uce58\uc5b4\uc758 \uaf2c\ub9ac\ub294 \ub0ae\uc740 \ucabd\uc5d0 \uc788\ub294 \uc9c0\ub290\ub7ec\ubbf8\uac00 \uc704\ucabd\uc5d0 \uc788\ub294 \uc9c0\ub290\ub7ec\ubbf8\ubcf4\ub2e4 \uc791\uc740 \ubaa8\uc591\uc774\ub2e4. \uace0\ub798\uc0c1\uc5b4\uc758 \ubd84\uc218\uacf5\uc740 \ub208 \ubc14\ub85c \ub4a4\uc5d0 \uc788\ub2e4."} {"question": "\ucf00\uc774\ud2b8\uc758 \uc124\uc815\uc740 \ub204\uad6c\uc5d0\uac8c \ub118\uc5b4\uac14\ub294\uac00?", "paragraph": "\uc2dc\uc98c 1\uc5d0\uc11c\ub294 \ub2f9\ucd08 \uce90\uc2a4\ud305 \uacc4\ud68d\uacfc \ub9ce\uc740 \ubd80\ubd84\uc774 \ub2ec\ub77c\uc84c\ub2e4. \uc8fc\uc694 \uc778\ubb3c\uc778 \uc7ad\uc740 \uc6d0\ub798 \ud30c\uc77c\ub7ff \uc5d0\ud53c\uc18c\ub4dc\uc5d0\uc11c \uc8fd\uc5b4\uac00\uace0 \uc788\ub358 \uac83\uc73c\ub85c \uc124\uc815\ud558\uace0 \ub9c8\uc774\ud074 \ud0a4\ud2bc\uc774 \ub9e1\uc744 \uac83\uc73c\ub85c \uacc4\ud68d\ub418\uc5c8\uc73c\ub098 ABC\uc5d0\uc11c \uc7ad\uc774 \uadf8\ub0e5 \uc0b4\uc544\uc788\ub294 \uac83\uc73c\ub85c \ud558\uace0 \ubc30\uc5ed\ub3c4 \ubc14\uafb8\uac8c \ud588\ub2e4. \ucf00\uc774\ud2b8\ub3c4 \uc6d0\ub798\ub294 \uc0dd\uc874\uc790\ub4e4\uc758 \uc218\uc7a5\uc744 \ub9e1\uae30\uae30\ub85c \ub418\uc5b4 \uc788\uc5c8\uc73c\uba70 \uc911\ub144\uc758 \ucee4\ub9ac\uc5b4\uc6b0\uba3c\uc774\uc790 \ub0a8\ud3b8\uc774 \uad50\ud1b5 \uc0ac\uace0\ub85c \uc0ac\ub9dd\ud55c \uac83\uc73c\ub85c \uc124\uc815\ud560 \uacc4\ud68d\uc774\uc600\uc73c\ub098 \uc774 \uc124\uc815\uc740 \ub85c\uc988\uc5d0\uac8c \ub118\uaca8\uc9c0\uac8c \ub418\uc5c8\ub2e4. \ub3c4\ubbf8\ub2c9 \ubaa8\ub098\uadf8\ud55c\uacfc \uc870\uc9c0 \uac00\ub974\uc2dc\uc544\ub3c4 \uc6d0\ub798 \uc804\uc9c1 \ub77d \uc2a4\ud0c0\ub85c \uc124\uc815\ub418\uc5c8\ub358 \uc18c\uc774\uc5b4\ub97c \uc5fc\ub450\uc5d0 \ub450\uace0 \uc624\ub514\uc158\uc744 \ubcf4\uc558\uc73c\ub098 \uc774 \ub77d \uc2a4\ud0c0\uc758 \uc124\uc815\uc740 \ucc30\ub9ac \ud398\uc774\uc2a4\ub85c \ub118\uc5b4\uac14\uace0 \uc5ed\ud560\ub3c4 \ub3c4\ubbf8\ub2c9 \ubaa8\ub098\uadf8\ud55c\uc774 \ub9e1\uac8c \ub418\uc5c8\uc73c\uba70 \uc870\uc9c0 \uac00\ub974\uc2dc\uc544\uc5d0\uac8c\ub294 \ud734\uace0 \ud5d0\ub9ac \ub808\uc608\uc2a4 \uc5ed\ud560\uc744 \ub9e1\uae30\ub85c \ub418\uc5c8\ub2e4. \ub098\uc911\uc5d0 \uc18c\uc774\uc5b4 \uc5ed\ud560\uc744 \ub9e1\uac8c \ub41c \uc870\uc2dc \ud560\ub85c\uc6e8\uc774\ub3c4 \ucc98\uc74c\uc5d0\ub294 \uc790\uc2e0\uc758 \ub0a8\ubd80 \uc5b5\uc591\uc744 \uc228\uae30\ub824\uace0 \uc560\ub97c \uc37c\uc5c8\ub294\ub370, J. J. \uc5d0\uc774\ube0c\ub7fc\uc2a4\uac00 \ub0a8\ubd80 \uc5b5\uc591\ub3c4 \uce90\uc2a4\ud305\ud55c \uc774\uc720 \uc911\uc5d0 \ud558\ub098\ub2c8\uae4c \uadf8\ub0e5 \uc788\ub294 \uadf8\ub300\ub85c \uc5f0\uae30\ud558\ub77c\uace0 \uc8fc\ubb38\ud588\ub2e4. \uc6d0\ub798\uc758 \uc18c\uc774\uc5b4 \uce90\ub9ad\ud130 \uc124\uc815\uc740 \uae54\ub054\ud55c \ub3c4\uc2dc\uc758 \uc815\uc7a5 \ucc28\ub9bc\uc758 \ud611\uc7a1\uafbc\uc774\uc5c8\ub2e4\uac00 \uc624\ub514\uc158\uc5d0\uc11c \ud560\ub85c\uc6e8\uc774\uac00 \ub300\uc0ac\ub97c \uae4c\uba39\uc740 \ubc14\ub78c\uc5d0 \ud667\uae40\uc5d0 \uc758\uc790\ub97c \uac77\uc5b4\ucc3c\uace0, \uadf8\uac78 \ubcf8 \uc791\uac00\ub4e4\uc774 \ub9c8\uc74c\uc5d0 \ub4e4\uc5b4\uc11c \uac70\uae30\uc5d0 \ub9de\uac8c \uc124\uc815\uc774 \ubcc0\uacbd\ub418\uc5c8\ub2e4. \uae40\uc724\uc9c4\uc740 \uc6d0\ub798 \ucf00\uc774\ud2b8 \uc5ed\ud560\uc744 \uc5fc\ub450\uc5d0 \ub450\uace0 \uc624\ub514\uc158\uc744 \ubcf4\uc558\uc73c\ub098 \uc81c\uc791\uc9c4\uc5d0\uc11c\ub294 \ucf00\uc774\ud2b8 \uc5ed\ud560\uc740 \uc54c\ub9de\uc9c0 \uc54a\ub2e4\uace0 \uc0dd\uac01\ud588\uace0 \uadf8 \ub300\uc2e0 \uc120 \uc5ed\ud560\uc744 \uc0c8\ub85c \ub9cc\ub4e4\uace0 \uc9c4\ub3c4 \uc0c8\ub85c \ub9cc\ub4e4\uc5b4 \ub2e4\ub2c8\uc5d8 \ub300 \ud0b4\uc774 \uadf8 \uc5ed\ud560\uc744 \ub9e1\uac8c \ud558\uace0 \ub458\uc744 \ubd80\ubd80\ub85c \uc5ee\uac8c \ud588\ub2e4. \ub098\ube48 \uc564\ub4dc\ub958\uac00 \ub9e1\uc740 \uc0ac\uc774\ub3c4\ub3c4 \uc6d0\ub798 \uacc4\ud68d\uc0c1\uc5d0\uc11c\ub294 \uc5c6\uc5c8\ub2e4. \ub85c\ud06c\uc640 \ub9c8\uc774\ud074\ub3c4 \ubc30\uc5ed\uc744 \ub9e1\uc740 \ubc30\uc6b0\uc758 \ub73b\uc5d0 \ub530\ub77c \ub300\ubcf8\uc774 \uc791\uc131\ub418\uc5c8\uc73c\uba70 \uc5d0\ubc00\ub9ac \ub4dc \ub77c\ube48\uc774 \ub9e1\uc740 \ud074\ub808\uc5b4\ub3c4 \uc6d0\ub798\ub294 \uc870\uc5f0\ub9cc \ub9e1\uc744 \uacc4\ud68d\uc774\uc600\ub2e4. \uc2dc\uc98c 2\uc5d0\uc11c \ub9c8\uc774\ud074 \uc5d0\uba40\uc2a8\uc740 \ub2f9\ucd08\uc5d0\ub294 \ud5e8\ub9ac \uac8c\uc77c \uc5ed\ud560\uc744 \ub9e1\uc544 \ucd1d 3\ubc88\ub9cc \ucd9c\uc5f0\ud560 \uc608\uc815\uc774\uc600\uc73c\ub098 \uc5f0\uae30\ub825\uc774 \ub6f0\uc5b4\ub098\ub2e4\ub294 \ud3c9\uac00\ub97c \ubc1b\uc544 \ucd1d 8\ud3b8\uc73c\ub85c \uc5f0\uc7a5\ud558\uace0, \uadf8 \ud6c4 \uc2dc\uc98c 3, \ub9c8\uc9c0\ub9c9 \uc2dc\uc98c\uae4c\uc9c0 \ub4f1\uc7a5\ud558\uac8c \ub41c\ub2e4.", "answer": "\ub85c\uc988", "sentence": "\uc774 \uc124\uc815\uc740 \ub85c\uc988 \uc5d0\uac8c \ub118\uaca8\uc9c0\uac8c \ub418\uc5c8\ub2e4.", "paragraph_sentence": "\uc2dc\uc98c 1\uc5d0\uc11c\ub294 \ub2f9\ucd08 \uce90\uc2a4\ud305 \uacc4\ud68d\uacfc \ub9ce\uc740 \ubd80\ubd84\uc774 \ub2ec\ub77c\uc84c\ub2e4. \uc8fc\uc694 \uc778\ubb3c\uc778 \uc7ad\uc740 \uc6d0\ub798 \ud30c\uc77c\ub7ff \uc5d0\ud53c\uc18c\ub4dc\uc5d0\uc11c \uc8fd\uc5b4\uac00\uace0 \uc788\ub358 \uac83\uc73c\ub85c \uc124\uc815\ud558\uace0 \ub9c8\uc774\ud074 \ud0a4\ud2bc\uc774 \ub9e1\uc744 \uac83\uc73c\ub85c \uacc4\ud68d\ub418\uc5c8\uc73c\ub098 ABC\uc5d0\uc11c \uc7ad\uc774 \uadf8\ub0e5 \uc0b4\uc544\uc788\ub294 \uac83\uc73c\ub85c \ud558\uace0 \ubc30\uc5ed\ub3c4 \ubc14\uafb8\uac8c \ud588\ub2e4. \ucf00\uc774\ud2b8\ub3c4 \uc6d0\ub798\ub294 \uc0dd\uc874\uc790\ub4e4\uc758 \uc218\uc7a5\uc744 \ub9e1\uae30\uae30\ub85c \ub418\uc5b4 \uc788\uc5c8\uc73c\uba70 \uc911\ub144\uc758 \ucee4\ub9ac\uc5b4\uc6b0\uba3c\uc774\uc790 \ub0a8\ud3b8\uc774 \uad50\ud1b5 \uc0ac\uace0\ub85c \uc0ac\ub9dd\ud55c \uac83\uc73c\ub85c \uc124\uc815\ud560 \uacc4\ud68d\uc774\uc600\uc73c\ub098 \uc774 \uc124\uc815\uc740 \ub85c\uc988 \uc5d0\uac8c \ub118\uaca8\uc9c0\uac8c \ub418\uc5c8\ub2e4. \ub3c4\ubbf8\ub2c9 \ubaa8\ub098\uadf8\ud55c\uacfc \uc870\uc9c0 \uac00\ub974\uc2dc\uc544\ub3c4 \uc6d0\ub798 \uc804\uc9c1 \ub77d \uc2a4\ud0c0\ub85c \uc124\uc815\ub418\uc5c8\ub358 \uc18c\uc774\uc5b4\ub97c \uc5fc\ub450\uc5d0 \ub450\uace0 \uc624\ub514\uc158\uc744 \ubcf4\uc558\uc73c\ub098 \uc774 \ub77d \uc2a4\ud0c0\uc758 \uc124\uc815\uc740 \ucc30\ub9ac \ud398\uc774\uc2a4\ub85c \ub118\uc5b4\uac14\uace0 \uc5ed\ud560\ub3c4 \ub3c4\ubbf8\ub2c9 \ubaa8\ub098\uadf8\ud55c\uc774 \ub9e1\uac8c \ub418\uc5c8\uc73c\uba70 \uc870\uc9c0 \uac00\ub974\uc2dc\uc544\uc5d0\uac8c\ub294 \ud734\uace0 \ud5d0\ub9ac \ub808\uc608\uc2a4 \uc5ed\ud560\uc744 \ub9e1\uae30\ub85c \ub418\uc5c8\ub2e4. \ub098\uc911\uc5d0 \uc18c\uc774\uc5b4 \uc5ed\ud560\uc744 \ub9e1\uac8c \ub41c \uc870\uc2dc \ud560\ub85c\uc6e8\uc774\ub3c4 \ucc98\uc74c\uc5d0\ub294 \uc790\uc2e0\uc758 \ub0a8\ubd80 \uc5b5\uc591\uc744 \uc228\uae30\ub824\uace0 \uc560\ub97c \uc37c\uc5c8\ub294\ub370, J. J. \uc5d0\uc774\ube0c\ub7fc\uc2a4\uac00 \ub0a8\ubd80 \uc5b5\uc591\ub3c4 \uce90\uc2a4\ud305\ud55c \uc774\uc720 \uc911\uc5d0 \ud558\ub098\ub2c8\uae4c \uadf8\ub0e5 \uc788\ub294 \uadf8\ub300\ub85c \uc5f0\uae30\ud558\ub77c\uace0 \uc8fc\ubb38\ud588\ub2e4. \uc6d0\ub798\uc758 \uc18c\uc774\uc5b4 \uce90\ub9ad\ud130 \uc124\uc815\uc740 \uae54\ub054\ud55c \ub3c4\uc2dc\uc758 \uc815\uc7a5 \ucc28\ub9bc\uc758 \ud611\uc7a1\uafbc\uc774\uc5c8\ub2e4\uac00 \uc624\ub514\uc158\uc5d0\uc11c \ud560\ub85c\uc6e8\uc774\uac00 \ub300\uc0ac\ub97c \uae4c\uba39\uc740 \ubc14\ub78c\uc5d0 \ud667\uae40\uc5d0 \uc758\uc790\ub97c \uac77\uc5b4\ucc3c\uace0, \uadf8\uac78 \ubcf8 \uc791\uac00\ub4e4\uc774 \ub9c8\uc74c\uc5d0 \ub4e4\uc5b4\uc11c \uac70\uae30\uc5d0 \ub9de\uac8c \uc124\uc815\uc774 \ubcc0\uacbd\ub418\uc5c8\ub2e4. \uae40\uc724\uc9c4\uc740 \uc6d0\ub798 \ucf00\uc774\ud2b8 \uc5ed\ud560\uc744 \uc5fc\ub450\uc5d0 \ub450\uace0 \uc624\ub514\uc158\uc744 \ubcf4\uc558\uc73c\ub098 \uc81c\uc791\uc9c4\uc5d0\uc11c\ub294 \ucf00\uc774\ud2b8 \uc5ed\ud560\uc740 \uc54c\ub9de\uc9c0 \uc54a\ub2e4\uace0 \uc0dd\uac01\ud588\uace0 \uadf8 \ub300\uc2e0 \uc120 \uc5ed\ud560\uc744 \uc0c8\ub85c \ub9cc\ub4e4\uace0 \uc9c4\ub3c4 \uc0c8\ub85c \ub9cc\ub4e4\uc5b4 \ub2e4\ub2c8\uc5d8 \ub300 \ud0b4\uc774 \uadf8 \uc5ed\ud560\uc744 \ub9e1\uac8c \ud558\uace0 \ub458\uc744 \ubd80\ubd80\ub85c \uc5ee\uac8c \ud588\ub2e4. \ub098\ube48 \uc564\ub4dc\ub958\uac00 \ub9e1\uc740 \uc0ac\uc774\ub3c4\ub3c4 \uc6d0\ub798 \uacc4\ud68d\uc0c1\uc5d0\uc11c\ub294 \uc5c6\uc5c8\ub2e4. \ub85c\ud06c\uc640 \ub9c8\uc774\ud074\ub3c4 \ubc30\uc5ed\uc744 \ub9e1\uc740 \ubc30\uc6b0\uc758 \ub73b\uc5d0 \ub530\ub77c \ub300\ubcf8\uc774 \uc791\uc131\ub418\uc5c8\uc73c\uba70 \uc5d0\ubc00\ub9ac \ub4dc \ub77c\ube48\uc774 \ub9e1\uc740 \ud074\ub808\uc5b4\ub3c4 \uc6d0\ub798\ub294 \uc870\uc5f0\ub9cc \ub9e1\uc744 \uacc4\ud68d\uc774\uc600\ub2e4. \uc2dc\uc98c 2\uc5d0\uc11c \ub9c8\uc774\ud074 \uc5d0\uba40\uc2a8\uc740 \ub2f9\ucd08\uc5d0\ub294 \ud5e8\ub9ac \uac8c\uc77c \uc5ed\ud560\uc744 \ub9e1\uc544 \ucd1d 3\ubc88\ub9cc \ucd9c\uc5f0\ud560 \uc608\uc815\uc774\uc600\uc73c\ub098 \uc5f0\uae30\ub825\uc774 \ub6f0\uc5b4\ub098\ub2e4\ub294 \ud3c9\uac00\ub97c \ubc1b\uc544 \ucd1d 8\ud3b8\uc73c\ub85c \uc5f0\uc7a5\ud558\uace0, \uadf8 \ud6c4 \uc2dc\uc98c 3, \ub9c8\uc9c0\ub9c9 \uc2dc\uc98c\uae4c\uc9c0 \ub4f1\uc7a5\ud558\uac8c \ub41c\ub2e4.", "paragraph_answer": "\uc2dc\uc98c 1\uc5d0\uc11c\ub294 \ub2f9\ucd08 \uce90\uc2a4\ud305 \uacc4\ud68d\uacfc \ub9ce\uc740 \ubd80\ubd84\uc774 \ub2ec\ub77c\uc84c\ub2e4. \uc8fc\uc694 \uc778\ubb3c\uc778 \uc7ad\uc740 \uc6d0\ub798 \ud30c\uc77c\ub7ff \uc5d0\ud53c\uc18c\ub4dc\uc5d0\uc11c \uc8fd\uc5b4\uac00\uace0 \uc788\ub358 \uac83\uc73c\ub85c \uc124\uc815\ud558\uace0 \ub9c8\uc774\ud074 \ud0a4\ud2bc\uc774 \ub9e1\uc744 \uac83\uc73c\ub85c \uacc4\ud68d\ub418\uc5c8\uc73c\ub098 ABC\uc5d0\uc11c \uc7ad\uc774 \uadf8\ub0e5 \uc0b4\uc544\uc788\ub294 \uac83\uc73c\ub85c \ud558\uace0 \ubc30\uc5ed\ub3c4 \ubc14\uafb8\uac8c \ud588\ub2e4. \ucf00\uc774\ud2b8\ub3c4 \uc6d0\ub798\ub294 \uc0dd\uc874\uc790\ub4e4\uc758 \uc218\uc7a5\uc744 \ub9e1\uae30\uae30\ub85c \ub418\uc5b4 \uc788\uc5c8\uc73c\uba70 \uc911\ub144\uc758 \ucee4\ub9ac\uc5b4\uc6b0\uba3c\uc774\uc790 \ub0a8\ud3b8\uc774 \uad50\ud1b5 \uc0ac\uace0\ub85c \uc0ac\ub9dd\ud55c \uac83\uc73c\ub85c \uc124\uc815\ud560 \uacc4\ud68d\uc774\uc600\uc73c\ub098 \uc774 \uc124\uc815\uc740 \ub85c\uc988 \uc5d0\uac8c \ub118\uaca8\uc9c0\uac8c \ub418\uc5c8\ub2e4. \ub3c4\ubbf8\ub2c9 \ubaa8\ub098\uadf8\ud55c\uacfc \uc870\uc9c0 \uac00\ub974\uc2dc\uc544\ub3c4 \uc6d0\ub798 \uc804\uc9c1 \ub77d \uc2a4\ud0c0\ub85c \uc124\uc815\ub418\uc5c8\ub358 \uc18c\uc774\uc5b4\ub97c \uc5fc\ub450\uc5d0 \ub450\uace0 \uc624\ub514\uc158\uc744 \ubcf4\uc558\uc73c\ub098 \uc774 \ub77d \uc2a4\ud0c0\uc758 \uc124\uc815\uc740 \ucc30\ub9ac \ud398\uc774\uc2a4\ub85c \ub118\uc5b4\uac14\uace0 \uc5ed\ud560\ub3c4 \ub3c4\ubbf8\ub2c9 \ubaa8\ub098\uadf8\ud55c\uc774 \ub9e1\uac8c \ub418\uc5c8\uc73c\uba70 \uc870\uc9c0 \uac00\ub974\uc2dc\uc544\uc5d0\uac8c\ub294 \ud734\uace0 \ud5d0\ub9ac \ub808\uc608\uc2a4 \uc5ed\ud560\uc744 \ub9e1\uae30\ub85c \ub418\uc5c8\ub2e4. \ub098\uc911\uc5d0 \uc18c\uc774\uc5b4 \uc5ed\ud560\uc744 \ub9e1\uac8c \ub41c \uc870\uc2dc \ud560\ub85c\uc6e8\uc774\ub3c4 \ucc98\uc74c\uc5d0\ub294 \uc790\uc2e0\uc758 \ub0a8\ubd80 \uc5b5\uc591\uc744 \uc228\uae30\ub824\uace0 \uc560\ub97c \uc37c\uc5c8\ub294\ub370, J. J. \uc5d0\uc774\ube0c\ub7fc\uc2a4\uac00 \ub0a8\ubd80 \uc5b5\uc591\ub3c4 \uce90\uc2a4\ud305\ud55c \uc774\uc720 \uc911\uc5d0 \ud558\ub098\ub2c8\uae4c \uadf8\ub0e5 \uc788\ub294 \uadf8\ub300\ub85c \uc5f0\uae30\ud558\ub77c\uace0 \uc8fc\ubb38\ud588\ub2e4. \uc6d0\ub798\uc758 \uc18c\uc774\uc5b4 \uce90\ub9ad\ud130 \uc124\uc815\uc740 \uae54\ub054\ud55c \ub3c4\uc2dc\uc758 \uc815\uc7a5 \ucc28\ub9bc\uc758 \ud611\uc7a1\uafbc\uc774\uc5c8\ub2e4\uac00 \uc624\ub514\uc158\uc5d0\uc11c \ud560\ub85c\uc6e8\uc774\uac00 \ub300\uc0ac\ub97c \uae4c\uba39\uc740 \ubc14\ub78c\uc5d0 \ud667\uae40\uc5d0 \uc758\uc790\ub97c \uac77\uc5b4\ucc3c\uace0, \uadf8\uac78 \ubcf8 \uc791\uac00\ub4e4\uc774 \ub9c8\uc74c\uc5d0 \ub4e4\uc5b4\uc11c \uac70\uae30\uc5d0 \ub9de\uac8c \uc124\uc815\uc774 \ubcc0\uacbd\ub418\uc5c8\ub2e4. \uae40\uc724\uc9c4\uc740 \uc6d0\ub798 \ucf00\uc774\ud2b8 \uc5ed\ud560\uc744 \uc5fc\ub450\uc5d0 \ub450\uace0 \uc624\ub514\uc158\uc744 \ubcf4\uc558\uc73c\ub098 \uc81c\uc791\uc9c4\uc5d0\uc11c\ub294 \ucf00\uc774\ud2b8 \uc5ed\ud560\uc740 \uc54c\ub9de\uc9c0 \uc54a\ub2e4\uace0 \uc0dd\uac01\ud588\uace0 \uadf8 \ub300\uc2e0 \uc120 \uc5ed\ud560\uc744 \uc0c8\ub85c \ub9cc\ub4e4\uace0 \uc9c4\ub3c4 \uc0c8\ub85c \ub9cc\ub4e4\uc5b4 \ub2e4\ub2c8\uc5d8 \ub300 \ud0b4\uc774 \uadf8 \uc5ed\ud560\uc744 \ub9e1\uac8c \ud558\uace0 \ub458\uc744 \ubd80\ubd80\ub85c \uc5ee\uac8c \ud588\ub2e4. \ub098\ube48 \uc564\ub4dc\ub958\uac00 \ub9e1\uc740 \uc0ac\uc774\ub3c4\ub3c4 \uc6d0\ub798 \uacc4\ud68d\uc0c1\uc5d0\uc11c\ub294 \uc5c6\uc5c8\ub2e4. \ub85c\ud06c\uc640 \ub9c8\uc774\ud074\ub3c4 \ubc30\uc5ed\uc744 \ub9e1\uc740 \ubc30\uc6b0\uc758 \ub73b\uc5d0 \ub530\ub77c \ub300\ubcf8\uc774 \uc791\uc131\ub418\uc5c8\uc73c\uba70 \uc5d0\ubc00\ub9ac \ub4dc \ub77c\ube48\uc774 \ub9e1\uc740 \ud074\ub808\uc5b4\ub3c4 \uc6d0\ub798\ub294 \uc870\uc5f0\ub9cc \ub9e1\uc744 \uacc4\ud68d\uc774\uc600\ub2e4. \uc2dc\uc98c 2\uc5d0\uc11c \ub9c8\uc774\ud074 \uc5d0\uba40\uc2a8\uc740 \ub2f9\ucd08\uc5d0\ub294 \ud5e8\ub9ac \uac8c\uc77c \uc5ed\ud560\uc744 \ub9e1\uc544 \ucd1d 3\ubc88\ub9cc \ucd9c\uc5f0\ud560 \uc608\uc815\uc774\uc600\uc73c\ub098 \uc5f0\uae30\ub825\uc774 \ub6f0\uc5b4\ub098\ub2e4\ub294 \ud3c9\uac00\ub97c \ubc1b\uc544 \ucd1d 8\ud3b8\uc73c\ub85c \uc5f0\uc7a5\ud558\uace0, \uadf8 \ud6c4 \uc2dc\uc98c 3, \ub9c8\uc9c0\ub9c9 \uc2dc\uc98c\uae4c\uc9c0 \ub4f1\uc7a5\ud558\uac8c \ub41c\ub2e4.", "sentence_answer": "\uc774 \uc124\uc815\uc740 \ub85c\uc988 \uc5d0\uac8c \ub118\uaca8\uc9c0\uac8c \ub418\uc5c8\ub2e4.", "paragraph_id": "6496300-5-0", "paragraph_question": "question: \ucf00\uc774\ud2b8\uc758 \uc124\uc815\uc740 \ub204\uad6c\uc5d0\uac8c \ub118\uc5b4\uac14\ub294\uac00?, context: \uc2dc\uc98c 1\uc5d0\uc11c\ub294 \ub2f9\ucd08 \uce90\uc2a4\ud305 \uacc4\ud68d\uacfc \ub9ce\uc740 \ubd80\ubd84\uc774 \ub2ec\ub77c\uc84c\ub2e4. \uc8fc\uc694 \uc778\ubb3c\uc778 \uc7ad\uc740 \uc6d0\ub798 \ud30c\uc77c\ub7ff \uc5d0\ud53c\uc18c\ub4dc\uc5d0\uc11c \uc8fd\uc5b4\uac00\uace0 \uc788\ub358 \uac83\uc73c\ub85c \uc124\uc815\ud558\uace0 \ub9c8\uc774\ud074 \ud0a4\ud2bc\uc774 \ub9e1\uc744 \uac83\uc73c\ub85c \uacc4\ud68d\ub418\uc5c8\uc73c\ub098 ABC\uc5d0\uc11c \uc7ad\uc774 \uadf8\ub0e5 \uc0b4\uc544\uc788\ub294 \uac83\uc73c\ub85c \ud558\uace0 \ubc30\uc5ed\ub3c4 \ubc14\uafb8\uac8c \ud588\ub2e4. \ucf00\uc774\ud2b8\ub3c4 \uc6d0\ub798\ub294 \uc0dd\uc874\uc790\ub4e4\uc758 \uc218\uc7a5\uc744 \ub9e1\uae30\uae30\ub85c \ub418\uc5b4 \uc788\uc5c8\uc73c\uba70 \uc911\ub144\uc758 \ucee4\ub9ac\uc5b4\uc6b0\uba3c\uc774\uc790 \ub0a8\ud3b8\uc774 \uad50\ud1b5 \uc0ac\uace0\ub85c \uc0ac\ub9dd\ud55c \uac83\uc73c\ub85c \uc124\uc815\ud560 \uacc4\ud68d\uc774\uc600\uc73c\ub098 \uc774 \uc124\uc815\uc740 \ub85c\uc988\uc5d0\uac8c \ub118\uaca8\uc9c0\uac8c \ub418\uc5c8\ub2e4. \ub3c4\ubbf8\ub2c9 \ubaa8\ub098\uadf8\ud55c\uacfc \uc870\uc9c0 \uac00\ub974\uc2dc\uc544\ub3c4 \uc6d0\ub798 \uc804\uc9c1 \ub77d \uc2a4\ud0c0\ub85c \uc124\uc815\ub418\uc5c8\ub358 \uc18c\uc774\uc5b4\ub97c \uc5fc\ub450\uc5d0 \ub450\uace0 \uc624\ub514\uc158\uc744 \ubcf4\uc558\uc73c\ub098 \uc774 \ub77d \uc2a4\ud0c0\uc758 \uc124\uc815\uc740 \ucc30\ub9ac \ud398\uc774\uc2a4\ub85c \ub118\uc5b4\uac14\uace0 \uc5ed\ud560\ub3c4 \ub3c4\ubbf8\ub2c9 \ubaa8\ub098\uadf8\ud55c\uc774 \ub9e1\uac8c \ub418\uc5c8\uc73c\uba70 \uc870\uc9c0 \uac00\ub974\uc2dc\uc544\uc5d0\uac8c\ub294 \ud734\uace0 \ud5d0\ub9ac \ub808\uc608\uc2a4 \uc5ed\ud560\uc744 \ub9e1\uae30\ub85c \ub418\uc5c8\ub2e4. \ub098\uc911\uc5d0 \uc18c\uc774\uc5b4 \uc5ed\ud560\uc744 \ub9e1\uac8c \ub41c \uc870\uc2dc \ud560\ub85c\uc6e8\uc774\ub3c4 \ucc98\uc74c\uc5d0\ub294 \uc790\uc2e0\uc758 \ub0a8\ubd80 \uc5b5\uc591\uc744 \uc228\uae30\ub824\uace0 \uc560\ub97c \uc37c\uc5c8\ub294\ub370, J. J. \uc5d0\uc774\ube0c\ub7fc\uc2a4\uac00 \ub0a8\ubd80 \uc5b5\uc591\ub3c4 \uce90\uc2a4\ud305\ud55c \uc774\uc720 \uc911\uc5d0 \ud558\ub098\ub2c8\uae4c \uadf8\ub0e5 \uc788\ub294 \uadf8\ub300\ub85c \uc5f0\uae30\ud558\ub77c\uace0 \uc8fc\ubb38\ud588\ub2e4. \uc6d0\ub798\uc758 \uc18c\uc774\uc5b4 \uce90\ub9ad\ud130 \uc124\uc815\uc740 \uae54\ub054\ud55c \ub3c4\uc2dc\uc758 \uc815\uc7a5 \ucc28\ub9bc\uc758 \ud611\uc7a1\uafbc\uc774\uc5c8\ub2e4\uac00 \uc624\ub514\uc158\uc5d0\uc11c \ud560\ub85c\uc6e8\uc774\uac00 \ub300\uc0ac\ub97c \uae4c\uba39\uc740 \ubc14\ub78c\uc5d0 \ud667\uae40\uc5d0 \uc758\uc790\ub97c \uac77\uc5b4\ucc3c\uace0, \uadf8\uac78 \ubcf8 \uc791\uac00\ub4e4\uc774 \ub9c8\uc74c\uc5d0 \ub4e4\uc5b4\uc11c \uac70\uae30\uc5d0 \ub9de\uac8c \uc124\uc815\uc774 \ubcc0\uacbd\ub418\uc5c8\ub2e4. \uae40\uc724\uc9c4\uc740 \uc6d0\ub798 \ucf00\uc774\ud2b8 \uc5ed\ud560\uc744 \uc5fc\ub450\uc5d0 \ub450\uace0 \uc624\ub514\uc158\uc744 \ubcf4\uc558\uc73c\ub098 \uc81c\uc791\uc9c4\uc5d0\uc11c\ub294 \ucf00\uc774\ud2b8 \uc5ed\ud560\uc740 \uc54c\ub9de\uc9c0 \uc54a\ub2e4\uace0 \uc0dd\uac01\ud588\uace0 \uadf8 \ub300\uc2e0 \uc120 \uc5ed\ud560\uc744 \uc0c8\ub85c \ub9cc\ub4e4\uace0 \uc9c4\ub3c4 \uc0c8\ub85c \ub9cc\ub4e4\uc5b4 \ub2e4\ub2c8\uc5d8 \ub300 \ud0b4\uc774 \uadf8 \uc5ed\ud560\uc744 \ub9e1\uac8c \ud558\uace0 \ub458\uc744 \ubd80\ubd80\ub85c \uc5ee\uac8c \ud588\ub2e4. \ub098\ube48 \uc564\ub4dc\ub958\uac00 \ub9e1\uc740 \uc0ac\uc774\ub3c4\ub3c4 \uc6d0\ub798 \uacc4\ud68d\uc0c1\uc5d0\uc11c\ub294 \uc5c6\uc5c8\ub2e4. \ub85c\ud06c\uc640 \ub9c8\uc774\ud074\ub3c4 \ubc30\uc5ed\uc744 \ub9e1\uc740 \ubc30\uc6b0\uc758 \ub73b\uc5d0 \ub530\ub77c \ub300\ubcf8\uc774 \uc791\uc131\ub418\uc5c8\uc73c\uba70 \uc5d0\ubc00\ub9ac \ub4dc \ub77c\ube48\uc774 \ub9e1\uc740 \ud074\ub808\uc5b4\ub3c4 \uc6d0\ub798\ub294 \uc870\uc5f0\ub9cc \ub9e1\uc744 \uacc4\ud68d\uc774\uc600\ub2e4. \uc2dc\uc98c 2\uc5d0\uc11c \ub9c8\uc774\ud074 \uc5d0\uba40\uc2a8\uc740 \ub2f9\ucd08\uc5d0\ub294 \ud5e8\ub9ac \uac8c\uc77c \uc5ed\ud560\uc744 \ub9e1\uc544 \ucd1d 3\ubc88\ub9cc \ucd9c\uc5f0\ud560 \uc608\uc815\uc774\uc600\uc73c\ub098 \uc5f0\uae30\ub825\uc774 \ub6f0\uc5b4\ub098\ub2e4\ub294 \ud3c9\uac00\ub97c \ubc1b\uc544 \ucd1d 8\ud3b8\uc73c\ub85c \uc5f0\uc7a5\ud558\uace0, \uadf8 \ud6c4 \uc2dc\uc98c 3, \ub9c8\uc9c0\ub9c9 \uc2dc\uc98c\uae4c\uc9c0 \ub4f1\uc7a5\ud558\uac8c \ub41c\ub2e4."} {"question": "\ud0dc\uc591\uacc4 \uc9c4\ud654\uacfc\uc815\uc758 \uc2dc\uc810\uc5d0\uc11c \ud0dc\uc591\uc740 \ubb34\uc2a8 \ud615\ud0dc\uc600\uc744\uaebc\ub77c \ucd94\uce21\ud558\uc600\ub098?", "paragraph": "\ud0dc\uc591\uacc4\uc758 \uc9c4\ud654 \uacfc\uc815\uc5d0\uc11c, \uc774 \uc2dc\uc810\uc758 \ud0dc\uc591\uc740 \ud669\uc18c\uc790\ub9ac T\ud615 \ud56d\uc131 \ud615\ud0dc\uc600\uc744 \uac83\uc73c\ub85c \ucd94\uce21\ub41c\ub2e4. \ub2e4\ub978 \ud669\uc18c\uc790\ub9ac T\ud615 \ud56d\uc131\ub4e4\uc744 \uc5f0\uad6c\ud55c \uacb0\uacfc, \uc774\ub4e4\uc740 \ud0dc\uc591 \uc9c8\ub7c9\uc758 0.001\ubc30\uc5d0\uc11c 0.1\ubc30\uc5d0 \uc774\ub974\ub294 \uc6d0\uc2dc \ud589\uc131\uacc4 \uc6d0\ubc18\uc744 \uac70\ub290\ub9b0 \uacbd\uc6b0\uac00 \ub9ce\uc740 \uac83\uc73c\ub85c \ub4dc\ub7ec\ub0ac\ub2e4. \uc774\ub7ec\ud55c \uc6d0\ubc18\ub4e4\uc740 \uc218\ubc31 \ucc9c\ubb38\ub2e8\uc704\uae4c\uc9c0 \ud3bc\uccd0\uc838 \uc788\uc73c\uba70(\ud5c8\ube14 \uc6b0\uc8fc \ub9dd\uc6d0\uacbd\uc774 \uc624\ub9ac\uc628\uc131\uc6b4\uc758 \ud56d\uc131 \uc0dd\uc131 \uc601\uc5ed\uc744 \uad00\uce21\ud55c \uacb0\uacfc 1,000\ucc9c\ubb38\ub2e8\uc704\uae4c\uc9c0 \ud3bc\uccd0\uc9c4 \uc6d0\uc2dc \ud589\uc131\uacc4 \uc6d0\ubc18\ub4e4\uc744 \uad00\uce21\ud560 \uc218 \uc788\uc5c8\ub2e4.), \uc628\ub3c4\ub294 \uadf8\ub9ac \ub192\uc9c0 \uc54a\uace0, \uac00\uc7a5 \ub728\uac70\uc6b4 \uacf3\ub3c4 \uc218\ucc9c \ucf08\ube48 \uc815\ub3c4\uc5d0 \ubd88\uacfc\ud588\ub2e4. \uc774\ud6c4 5\ucc9c\ub9cc \ub144\uc774 \ud758\ub7ec\uc11c, \ud0dc\uc591\uc758 \uc911\uc2ec\ud575 \ubd80\ubd84\uc758 \uc628\ub3c4 \ubc0f \uc555\ub825\uc740 \ub9e4\uc6b0 \ub192\uc544\uc9c0\uac70\ub098 \ucee4\uc838\uc11c \uc218\uc18c\ub294 \uc735\ud569 \uc791\uc6a9\uc744 \uc77c\uc73c\ud0a4\uae30 \uc2dc\uc791\ud55c\ub2e4. \ud0dc\uc591\uc758 \ub0b4\ubd80\uc5d0\uc11c \uc735\ud569 \uc791\uc6a9\uc744 \ud1b5\ud574 \ubc1c\uc0dd\ud558\ub294 \ubcf5\uc0ac\uc555\uacfc \uc911\ub825\uc73c\ub85c \ub9d0\ubbf8\uc554\uc544 \uc218\ucd95\ud558\ub824\ub294 \ud798\uc774 \ud3c9\ud615\uc744 \uc774\ub8e8\uba74\uc11c \uc720\uccb4 \uc815\uc5ed\ud559\uc801 \uade0\ud615 \uc0c1\ud0dc\uac00 \ub9cc\ub4e4\uc5b4\uc9c0\uac8c \ub418\uc5c8\ub2e4. \uc774 \uc2dc\uc810\uc5d0\uc11c \ud0dc\uc591\uc740 \uc5b4\uc5ff\ud55c \uc8fc\uacc4\uc5f4\uc131\uc758 \ub2e8\uacc4\uc5d0 \uc811\uc5b4\ub4e4\uac8c \ub41c\ub2e4. \uc8fc\uacc4\uc5f4\uc131\uc740 \uc911\uc2ec\ud575 \ubd80\ubd84\uc5d0 \uc788\ub294 \uc218\uc18c\ub97c \ud0dc\uc6cc \ud5ec\ub968\uc73c\ub85c \ubc14\uafb8\ub294 \uc874\uc7ac\uc774\ub2e4. \ud604\uc7ac \ud0dc\uc591\uc740 \uc8fc\uacc4\uc5f4\uc131 \uc0c1\ud0dc\uc5d0 \uc788\ub2e4.", "answer": "\ud669\uc18c\uc790\ub9ac T\ud615 \ud56d\uc131 \ud615\ud0dc", "sentence": "\ud0dc\uc591\uacc4\uc758 \uc9c4\ud654 \uacfc\uc815\uc5d0\uc11c, \uc774 \uc2dc\uc810\uc758 \ud0dc\uc591\uc740 \ud669\uc18c\uc790\ub9ac T\ud615 \ud56d\uc131 \ud615\ud0dc \uc600\uc744 \uac83\uc73c\ub85c \ucd94\uce21\ub41c\ub2e4.", "paragraph_sentence": " \ud0dc\uc591\uacc4\uc758 \uc9c4\ud654 \uacfc\uc815\uc5d0\uc11c, \uc774 \uc2dc\uc810\uc758 \ud0dc\uc591\uc740 \ud669\uc18c\uc790\ub9ac T\ud615 \ud56d\uc131 \ud615\ud0dc \uc600\uc744 \uac83\uc73c\ub85c \ucd94\uce21\ub41c\ub2e4. \ub2e4\ub978 \ud669\uc18c\uc790\ub9ac T\ud615 \ud56d\uc131\ub4e4\uc744 \uc5f0\uad6c\ud55c \uacb0\uacfc, \uc774\ub4e4\uc740 \ud0dc\uc591 \uc9c8\ub7c9\uc758 0.001\ubc30\uc5d0\uc11c 0.1\ubc30\uc5d0 \uc774\ub974\ub294 \uc6d0\uc2dc \ud589\uc131\uacc4 \uc6d0\ubc18\uc744 \uac70\ub290\ub9b0 \uacbd\uc6b0\uac00 \ub9ce\uc740 \uac83\uc73c\ub85c \ub4dc\ub7ec\ub0ac\ub2e4. \uc774\ub7ec\ud55c \uc6d0\ubc18\ub4e4\uc740 \uc218\ubc31 \ucc9c\ubb38\ub2e8\uc704\uae4c\uc9c0 \ud3bc\uccd0\uc838 \uc788\uc73c\uba70(\ud5c8\ube14 \uc6b0\uc8fc \ub9dd\uc6d0\uacbd\uc774 \uc624\ub9ac\uc628\uc131\uc6b4\uc758 \ud56d\uc131 \uc0dd\uc131 \uc601\uc5ed\uc744 \uad00\uce21\ud55c \uacb0\uacfc 1,000\ucc9c\ubb38\ub2e8\uc704\uae4c\uc9c0 \ud3bc\uccd0\uc9c4 \uc6d0\uc2dc \ud589\uc131\uacc4 \uc6d0\ubc18\ub4e4\uc744 \uad00\uce21\ud560 \uc218 \uc788\uc5c8\ub2e4. ) , \uc628\ub3c4\ub294 \uadf8\ub9ac \ub192\uc9c0 \uc54a\uace0, \uac00\uc7a5 \ub728\uac70\uc6b4 \uacf3\ub3c4 \uc218\ucc9c \ucf08\ube48 \uc815\ub3c4\uc5d0 \ubd88\uacfc\ud588\ub2e4. \uc774\ud6c4 5\ucc9c\ub9cc \ub144\uc774 \ud758\ub7ec\uc11c, \ud0dc\uc591\uc758 \uc911\uc2ec\ud575 \ubd80\ubd84\uc758 \uc628\ub3c4 \ubc0f \uc555\ub825\uc740 \ub9e4\uc6b0 \ub192\uc544\uc9c0\uac70\ub098 \ucee4\uc838\uc11c \uc218\uc18c\ub294 \uc735\ud569 \uc791\uc6a9\uc744 \uc77c\uc73c\ud0a4\uae30 \uc2dc\uc791\ud55c\ub2e4. \ud0dc\uc591\uc758 \ub0b4\ubd80\uc5d0\uc11c \uc735\ud569 \uc791\uc6a9\uc744 \ud1b5\ud574 \ubc1c\uc0dd\ud558\ub294 \ubcf5\uc0ac\uc555\uacfc \uc911\ub825\uc73c\ub85c \ub9d0\ubbf8\uc554\uc544 \uc218\ucd95\ud558\ub824\ub294 \ud798\uc774 \ud3c9\ud615\uc744 \uc774\ub8e8\uba74\uc11c \uc720\uccb4 \uc815\uc5ed\ud559\uc801 \uade0\ud615 \uc0c1\ud0dc\uac00 \ub9cc\ub4e4\uc5b4\uc9c0\uac8c \ub418\uc5c8\ub2e4. \uc774 \uc2dc\uc810\uc5d0\uc11c \ud0dc\uc591\uc740 \uc5b4\uc5ff\ud55c \uc8fc\uacc4\uc5f4\uc131\uc758 \ub2e8\uacc4\uc5d0 \uc811\uc5b4\ub4e4\uac8c \ub41c\ub2e4. \uc8fc\uacc4\uc5f4\uc131\uc740 \uc911\uc2ec\ud575 \ubd80\ubd84\uc5d0 \uc788\ub294 \uc218\uc18c\ub97c \ud0dc\uc6cc \ud5ec\ub968\uc73c\ub85c \ubc14\uafb8\ub294 \uc874\uc7ac\uc774\ub2e4. \ud604\uc7ac \ud0dc\uc591\uc740 \uc8fc\uacc4\uc5f4\uc131 \uc0c1\ud0dc\uc5d0 \uc788\ub2e4.", "paragraph_answer": "\ud0dc\uc591\uacc4\uc758 \uc9c4\ud654 \uacfc\uc815\uc5d0\uc11c, \uc774 \uc2dc\uc810\uc758 \ud0dc\uc591\uc740 \ud669\uc18c\uc790\ub9ac T\ud615 \ud56d\uc131 \ud615\ud0dc \uc600\uc744 \uac83\uc73c\ub85c \ucd94\uce21\ub41c\ub2e4. \ub2e4\ub978 \ud669\uc18c\uc790\ub9ac T\ud615 \ud56d\uc131\ub4e4\uc744 \uc5f0\uad6c\ud55c \uacb0\uacfc, \uc774\ub4e4\uc740 \ud0dc\uc591 \uc9c8\ub7c9\uc758 0.001\ubc30\uc5d0\uc11c 0.1\ubc30\uc5d0 \uc774\ub974\ub294 \uc6d0\uc2dc \ud589\uc131\uacc4 \uc6d0\ubc18\uc744 \uac70\ub290\ub9b0 \uacbd\uc6b0\uac00 \ub9ce\uc740 \uac83\uc73c\ub85c \ub4dc\ub7ec\ub0ac\ub2e4. \uc774\ub7ec\ud55c \uc6d0\ubc18\ub4e4\uc740 \uc218\ubc31 \ucc9c\ubb38\ub2e8\uc704\uae4c\uc9c0 \ud3bc\uccd0\uc838 \uc788\uc73c\uba70(\ud5c8\ube14 \uc6b0\uc8fc \ub9dd\uc6d0\uacbd\uc774 \uc624\ub9ac\uc628\uc131\uc6b4\uc758 \ud56d\uc131 \uc0dd\uc131 \uc601\uc5ed\uc744 \uad00\uce21\ud55c \uacb0\uacfc 1,000\ucc9c\ubb38\ub2e8\uc704\uae4c\uc9c0 \ud3bc\uccd0\uc9c4 \uc6d0\uc2dc \ud589\uc131\uacc4 \uc6d0\ubc18\ub4e4\uc744 \uad00\uce21\ud560 \uc218 \uc788\uc5c8\ub2e4.), \uc628\ub3c4\ub294 \uadf8\ub9ac \ub192\uc9c0 \uc54a\uace0, \uac00\uc7a5 \ub728\uac70\uc6b4 \uacf3\ub3c4 \uc218\ucc9c \ucf08\ube48 \uc815\ub3c4\uc5d0 \ubd88\uacfc\ud588\ub2e4. \uc774\ud6c4 5\ucc9c\ub9cc \ub144\uc774 \ud758\ub7ec\uc11c, \ud0dc\uc591\uc758 \uc911\uc2ec\ud575 \ubd80\ubd84\uc758 \uc628\ub3c4 \ubc0f \uc555\ub825\uc740 \ub9e4\uc6b0 \ub192\uc544\uc9c0\uac70\ub098 \ucee4\uc838\uc11c \uc218\uc18c\ub294 \uc735\ud569 \uc791\uc6a9\uc744 \uc77c\uc73c\ud0a4\uae30 \uc2dc\uc791\ud55c\ub2e4. \ud0dc\uc591\uc758 \ub0b4\ubd80\uc5d0\uc11c \uc735\ud569 \uc791\uc6a9\uc744 \ud1b5\ud574 \ubc1c\uc0dd\ud558\ub294 \ubcf5\uc0ac\uc555\uacfc \uc911\ub825\uc73c\ub85c \ub9d0\ubbf8\uc554\uc544 \uc218\ucd95\ud558\ub824\ub294 \ud798\uc774 \ud3c9\ud615\uc744 \uc774\ub8e8\uba74\uc11c \uc720\uccb4 \uc815\uc5ed\ud559\uc801 \uade0\ud615 \uc0c1\ud0dc\uac00 \ub9cc\ub4e4\uc5b4\uc9c0\uac8c \ub418\uc5c8\ub2e4. \uc774 \uc2dc\uc810\uc5d0\uc11c \ud0dc\uc591\uc740 \uc5b4\uc5ff\ud55c \uc8fc\uacc4\uc5f4\uc131\uc758 \ub2e8\uacc4\uc5d0 \uc811\uc5b4\ub4e4\uac8c \ub41c\ub2e4. \uc8fc\uacc4\uc5f4\uc131\uc740 \uc911\uc2ec\ud575 \ubd80\ubd84\uc5d0 \uc788\ub294 \uc218\uc18c\ub97c \ud0dc\uc6cc \ud5ec\ub968\uc73c\ub85c \ubc14\uafb8\ub294 \uc874\uc7ac\uc774\ub2e4. \ud604\uc7ac \ud0dc\uc591\uc740 \uc8fc\uacc4\uc5f4\uc131 \uc0c1\ud0dc\uc5d0 \uc788\ub2e4.", "sentence_answer": "\ud0dc\uc591\uacc4\uc758 \uc9c4\ud654 \uacfc\uc815\uc5d0\uc11c, \uc774 \uc2dc\uc810\uc758 \ud0dc\uc591\uc740 \ud669\uc18c\uc790\ub9ac T\ud615 \ud56d\uc131 \ud615\ud0dc \uc600\uc744 \uac83\uc73c\ub85c \ucd94\uce21\ub41c\ub2e4.", "paragraph_id": "5911343-3-0", "paragraph_question": "question: \ud0dc\uc591\uacc4 \uc9c4\ud654\uacfc\uc815\uc758 \uc2dc\uc810\uc5d0\uc11c \ud0dc\uc591\uc740 \ubb34\uc2a8 \ud615\ud0dc\uc600\uc744\uaebc\ub77c \ucd94\uce21\ud558\uc600\ub098?, context: \ud0dc\uc591\uacc4\uc758 \uc9c4\ud654 \uacfc\uc815\uc5d0\uc11c, \uc774 \uc2dc\uc810\uc758 \ud0dc\uc591\uc740 \ud669\uc18c\uc790\ub9ac T\ud615 \ud56d\uc131 \ud615\ud0dc\uc600\uc744 \uac83\uc73c\ub85c \ucd94\uce21\ub41c\ub2e4. \ub2e4\ub978 \ud669\uc18c\uc790\ub9ac T\ud615 \ud56d\uc131\ub4e4\uc744 \uc5f0\uad6c\ud55c \uacb0\uacfc, \uc774\ub4e4\uc740 \ud0dc\uc591 \uc9c8\ub7c9\uc758 0.001\ubc30\uc5d0\uc11c 0.1\ubc30\uc5d0 \uc774\ub974\ub294 \uc6d0\uc2dc \ud589\uc131\uacc4 \uc6d0\ubc18\uc744 \uac70\ub290\ub9b0 \uacbd\uc6b0\uac00 \ub9ce\uc740 \uac83\uc73c\ub85c \ub4dc\ub7ec\ub0ac\ub2e4. \uc774\ub7ec\ud55c \uc6d0\ubc18\ub4e4\uc740 \uc218\ubc31 \ucc9c\ubb38\ub2e8\uc704\uae4c\uc9c0 \ud3bc\uccd0\uc838 \uc788\uc73c\uba70(\ud5c8\ube14 \uc6b0\uc8fc \ub9dd\uc6d0\uacbd\uc774 \uc624\ub9ac\uc628\uc131\uc6b4\uc758 \ud56d\uc131 \uc0dd\uc131 \uc601\uc5ed\uc744 \uad00\uce21\ud55c \uacb0\uacfc 1,000\ucc9c\ubb38\ub2e8\uc704\uae4c\uc9c0 \ud3bc\uccd0\uc9c4 \uc6d0\uc2dc \ud589\uc131\uacc4 \uc6d0\ubc18\ub4e4\uc744 \uad00\uce21\ud560 \uc218 \uc788\uc5c8\ub2e4.), \uc628\ub3c4\ub294 \uadf8\ub9ac \ub192\uc9c0 \uc54a\uace0, \uac00\uc7a5 \ub728\uac70\uc6b4 \uacf3\ub3c4 \uc218\ucc9c \ucf08\ube48 \uc815\ub3c4\uc5d0 \ubd88\uacfc\ud588\ub2e4. \uc774\ud6c4 5\ucc9c\ub9cc \ub144\uc774 \ud758\ub7ec\uc11c, \ud0dc\uc591\uc758 \uc911\uc2ec\ud575 \ubd80\ubd84\uc758 \uc628\ub3c4 \ubc0f \uc555\ub825\uc740 \ub9e4\uc6b0 \ub192\uc544\uc9c0\uac70\ub098 \ucee4\uc838\uc11c \uc218\uc18c\ub294 \uc735\ud569 \uc791\uc6a9\uc744 \uc77c\uc73c\ud0a4\uae30 \uc2dc\uc791\ud55c\ub2e4. \ud0dc\uc591\uc758 \ub0b4\ubd80\uc5d0\uc11c \uc735\ud569 \uc791\uc6a9\uc744 \ud1b5\ud574 \ubc1c\uc0dd\ud558\ub294 \ubcf5\uc0ac\uc555\uacfc \uc911\ub825\uc73c\ub85c \ub9d0\ubbf8\uc554\uc544 \uc218\ucd95\ud558\ub824\ub294 \ud798\uc774 \ud3c9\ud615\uc744 \uc774\ub8e8\uba74\uc11c \uc720\uccb4 \uc815\uc5ed\ud559\uc801 \uade0\ud615 \uc0c1\ud0dc\uac00 \ub9cc\ub4e4\uc5b4\uc9c0\uac8c \ub418\uc5c8\ub2e4. \uc774 \uc2dc\uc810\uc5d0\uc11c \ud0dc\uc591\uc740 \uc5b4\uc5ff\ud55c \uc8fc\uacc4\uc5f4\uc131\uc758 \ub2e8\uacc4\uc5d0 \uc811\uc5b4\ub4e4\uac8c \ub41c\ub2e4. \uc8fc\uacc4\uc5f4\uc131\uc740 \uc911\uc2ec\ud575 \ubd80\ubd84\uc5d0 \uc788\ub294 \uc218\uc18c\ub97c \ud0dc\uc6cc \ud5ec\ub968\uc73c\ub85c \ubc14\uafb8\ub294 \uc874\uc7ac\uc774\ub2e4. \ud604\uc7ac \ud0dc\uc591\uc740 \uc8fc\uacc4\uc5f4\uc131 \uc0c1\ud0dc\uc5d0 \uc788\ub2e4."} {"question": "\ub9ac\ud50c\ub9ac\uac00 \uc5d0\uc774\ub9ac\uc5b8\uc744 \uac00\ub450\ub824\uace0 \ud55c \uacf3\uc740 \uc5b4\ub514\uc778\uac00?", "paragraph": "\uadf8\ub9ac\uace0 \uadf8 \ud6c4 \uc8c4\uc218\ub4e4\uc774 \ud558\ub098\ub458\uc529 \uc5d0\uc774\ub9ac\uc5b8\ub4e4\uc5d0\uac8c \uc0b4\ud574\ub2f9\ud558\uae30 \uc2dc\uc791\ud558\uace0, \ub3d9\ub8cc \uc8c4\uc218\ub4e4\ub85c\ubd80\ud130 \uc800\ub2a5\uc544 \ucde8\uae09\uc744 \ubc1b\ub358 \uc8c4\uc218 \uace8\ub9ad\uc774 \uc5d0\uc774\ub9ac\uc5b8\uc744 \ubcf4\uc558\ub2e4\uace0 \ub9d0\ud558\uc9c0\ub9cc \ub9ac\ud50c\ub9ac\ub97c \uc81c\uc678\ud55c \uc564\ub4dc\ub958\uc2a4 \uad50\ub3c4\uc18c\uc7a5\uc744 \ube44\ub86f\ud574 \ub2e4\ub978 \uc774\ub4e4\uc740 \uadf8 \uc774\uc57c\uae30\ub97c \ubbff\uc9c0 \uc54a\ub294\ub2e4. \uc624\ud788\ub824 \uac19\uc774 \uc815\uc2e0\ub098\uac04 \uc0ac\ub78c \ucde8\uae09\uc744 \ubc1b\uc73c\uba70 \uc758\ubb34\uc2e4\uc5d0 \uad6c\uae08\ub41c \ub9ac\ud50c\ub9ac\ub294 \ud074\ub808\uba58\uc2a4\uac00 \uc758\ubb34\uc2e4\uc5d0 \ub4e4\uc5b4\uc628 \uc5d0\uc774\ub9ac\uc5b8\uc5d0\uac8c \uc0b4\ud574\ub2f9\ud558\uc790, \uc564\ub4dc\ub958\uc2a4 \uad50\ub3c4\uc18c\uc7a5\uc5d0\uac8c \ub2ec\ub824\uac00 \uc0ac\uc2e4\uc744 \uc54c\ub9ac\uc9c0\ub9cc \uc5d0\uc774\ub9ac\uc5b8\uc774 \uadf8\ub9c8\uc800 \uc8c4\uc218\ub4e4\uc774 \ubcf4\ub294 \uc55e\uc5d0\uc11c \uc0b4\ud574\ud55c\ub2e4. \ud55c\uc21c\uac04\uc5d0 \uad6c\uc2ec\uc810\uc744 \uc783\uc740 \uc8c4\uc218\ub4e4\uc740 \uadf8\ub4e4 \uc0ac\uc774\uc5d0\uc11c \uc2e0\uc559\uc2ec\uc73c\ub85c \ub2e8\uacb0\uc744 \uc720\uc9c0\ud558\ub294\ub370 \uc55e\uc7a5\uc130\ub358 \uc8c4\uc218 \ub51c\ub860\uc744 \ub9ac\ub354\ub85c \uc0bc\uae38 \uc6d0\ud558\ub098, \ub51c\ub860\uc740 \uac70\ubd80\ud558\uace0 \ub9ac\ud50c\ub9ac\uac00 \ub9ac\ub354\uac00 \ub418\uae38 \uad8c\uc720\ud55c\ub2e4. \ub9ac\ud50c\ub9ac\ub294 \ubb34\uae30\uac00 \uc5c6\ub294 \uad50\ub3c4\uc18c\ub0b4 \ud2b9\uc131\uc0c1, \uc720\ub3c5\uc131 \uc778\ud654\ubb3c\uc9c8\uc744 \uc774\uc6a9\ud574 \uc5d0\uc774\ub9ac\uc5b8\uc744 \ud3d0\uae30\ubb3c \uc800\uc7a5\uace0\ub85c \uc720\uc778\ud574 \uac00\ub46c\ub458 \uacc4\ud68d\uc744 \uc138\uc6b4\ub2e4. \ud558\uc9c0\ub9cc \uc2e0\ud638\ud0c4\uc744 \uac00\uc9c4 \uc8c4\uc218 \uc911 \ud558\ub098\uac00 \uc5d0\uc774\ub9ac\uc5b8\uc5d0\uac8c \uacf5\uaca9\ubc1b\uc73c\uba74\uc11c \uc778\ud654\ubb3c\uc9c8\uc5d0 \ubd88\uc774 \ubd99\uc5b4 \ud3ed\ubc1c\ud574 \ubc84\ub9ac\ub294 \ubc14\ub78c\uc5d0 \uc8c4\uc218\ub4e4 \ubc18\uc808\uc740 \ubd88\uc5d0 \ud0c0 \uc8fd\uace0, \ub0a8\uc740 \uc8c4\uc218\ub4e4\ub3c4 \uc2ec\uac01\ud55c \ubd80\uc0c1\uc744 \uc785\ub294\ub2e4. \uc7ac\ud3b8\uc9d1\ud310\uc5d0\uc11c \ucd94\uac00\ub41c \uc7a5\uba74\uc5d0\uc11c\ub294 \uc774 \ud3ed\ubc1c \uc640\uc911\uc5d0 \uc8c4\uc218 \ud55c\uba85\uc774 \uc790\uc2e0\uc744 \ud76c\uc0dd\ud574 \ud3d0\uae30\ubb3c \uc800\uc7a5\uace0\ub85c \uc5d0\uc774\ub9ac\uc5b8\uc744 \uc720\uc778\ud558\uace0, \uc548\uc5d0 \uac19\uc774 \uac19\ud600\uc11c \uc5d0\uc774\ub9ac\uc5b8\uc744 \uac00\ub450\ub294\ub370 \uc131\uacf5\ud55c\ub2e4. \ud558\uc9c0\ub9cc \uc55e\uc11c \uc5d0\uc774\ub9ac\uc5b8\uc744 \uc758\ubb34\uc2e4\uc5d0\uc11c \ubcf4\uace0 \uc815\uc2e0\uc774 \uc774\uc0c1\ud574\uc9c4 \uace8\ub9ad\uc740 \ud3d0\uae30\ubb3c \uc800\uc7a5\uace0 \uc55e\uc5d0\uc11c \uc5d0\uc774\ub9ac\uc5b8\uc744 \uc9c0\ud0a4\ub358 \ub3d9\ub8cc \uc8c4\uc218\ub97c \uc8fd\uc5ec\uc11c \uc548\uc5d0 \uac07\ud78c \uc5d0\uc774\ub9ac\uc5b8\uc744 \ub3c4\ub85c \ud480\uc5b4\uc8fc\uace0 \uc790\uc2e0\ub3c4 \uc8fd\uace0 \ub9cc\ub2e4.", "answer": "\ud3d0\uae30\ubb3c \uc800\uc7a5\uace0", "sentence": "\ub9ac\ud50c\ub9ac\ub294 \ubb34\uae30\uac00 \uc5c6\ub294 \uad50\ub3c4\uc18c\ub0b4 \ud2b9\uc131\uc0c1, \uc720\ub3c5\uc131 \uc778\ud654\ubb3c\uc9c8\uc744 \uc774\uc6a9\ud574 \uc5d0\uc774\ub9ac\uc5b8\uc744 \ud3d0\uae30\ubb3c \uc800\uc7a5\uace0 \ub85c \uc720\uc778\ud574 \uac00\ub46c\ub458 \uacc4\ud68d\uc744 \uc138\uc6b4\ub2e4.", "paragraph_sentence": "\uadf8\ub9ac\uace0 \uadf8 \ud6c4 \uc8c4\uc218\ub4e4\uc774 \ud558\ub098\ub458\uc529 \uc5d0\uc774\ub9ac\uc5b8\ub4e4\uc5d0\uac8c \uc0b4\ud574\ub2f9\ud558\uae30 \uc2dc\uc791\ud558\uace0, \ub3d9\ub8cc \uc8c4\uc218\ub4e4\ub85c\ubd80\ud130 \uc800\ub2a5\uc544 \ucde8\uae09\uc744 \ubc1b\ub358 \uc8c4\uc218 \uace8\ub9ad\uc774 \uc5d0\uc774\ub9ac\uc5b8\uc744 \ubcf4\uc558\ub2e4\uace0 \ub9d0\ud558\uc9c0\ub9cc \ub9ac\ud50c\ub9ac\ub97c \uc81c\uc678\ud55c \uc564\ub4dc\ub958\uc2a4 \uad50\ub3c4\uc18c\uc7a5\uc744 \ube44\ub86f\ud574 \ub2e4\ub978 \uc774\ub4e4\uc740 \uadf8 \uc774\uc57c\uae30\ub97c \ubbff\uc9c0 \uc54a\ub294\ub2e4. \uc624\ud788\ub824 \uac19\uc774 \uc815\uc2e0\ub098\uac04 \uc0ac\ub78c \ucde8\uae09\uc744 \ubc1b\uc73c\uba70 \uc758\ubb34\uc2e4\uc5d0 \uad6c\uae08\ub41c \ub9ac\ud50c\ub9ac\ub294 \ud074\ub808\uba58\uc2a4\uac00 \uc758\ubb34\uc2e4\uc5d0 \ub4e4\uc5b4\uc628 \uc5d0\uc774\ub9ac\uc5b8\uc5d0\uac8c \uc0b4\ud574\ub2f9\ud558\uc790, \uc564\ub4dc\ub958\uc2a4 \uad50\ub3c4\uc18c\uc7a5\uc5d0\uac8c \ub2ec\ub824\uac00 \uc0ac\uc2e4\uc744 \uc54c\ub9ac\uc9c0\ub9cc \uc5d0\uc774\ub9ac\uc5b8\uc774 \uadf8\ub9c8\uc800 \uc8c4\uc218\ub4e4\uc774 \ubcf4\ub294 \uc55e\uc5d0\uc11c \uc0b4\ud574\ud55c\ub2e4. \ud55c\uc21c\uac04\uc5d0 \uad6c\uc2ec\uc810\uc744 \uc783\uc740 \uc8c4\uc218\ub4e4\uc740 \uadf8\ub4e4 \uc0ac\uc774\uc5d0\uc11c \uc2e0\uc559\uc2ec\uc73c\ub85c \ub2e8\uacb0\uc744 \uc720\uc9c0\ud558\ub294\ub370 \uc55e\uc7a5\uc130\ub358 \uc8c4\uc218 \ub51c\ub860\uc744 \ub9ac\ub354\ub85c \uc0bc\uae38 \uc6d0\ud558\ub098, \ub51c\ub860\uc740 \uac70\ubd80\ud558\uace0 \ub9ac\ud50c\ub9ac\uac00 \ub9ac\ub354\uac00 \ub418\uae38 \uad8c\uc720\ud55c\ub2e4. \ub9ac\ud50c\ub9ac\ub294 \ubb34\uae30\uac00 \uc5c6\ub294 \uad50\ub3c4\uc18c\ub0b4 \ud2b9\uc131\uc0c1, \uc720\ub3c5\uc131 \uc778\ud654\ubb3c\uc9c8\uc744 \uc774\uc6a9\ud574 \uc5d0\uc774\ub9ac\uc5b8\uc744 \ud3d0\uae30\ubb3c \uc800\uc7a5\uace0 \ub85c \uc720\uc778\ud574 \uac00\ub46c\ub458 \uacc4\ud68d\uc744 \uc138\uc6b4\ub2e4. \ud558\uc9c0\ub9cc \uc2e0\ud638\ud0c4\uc744 \uac00\uc9c4 \uc8c4\uc218 \uc911 \ud558\ub098\uac00 \uc5d0\uc774\ub9ac\uc5b8\uc5d0\uac8c \uacf5\uaca9\ubc1b\uc73c\uba74\uc11c \uc778\ud654\ubb3c\uc9c8\uc5d0 \ubd88\uc774 \ubd99\uc5b4 \ud3ed\ubc1c\ud574 \ubc84\ub9ac\ub294 \ubc14\ub78c\uc5d0 \uc8c4\uc218\ub4e4 \ubc18\uc808\uc740 \ubd88\uc5d0 \ud0c0 \uc8fd\uace0, \ub0a8\uc740 \uc8c4\uc218\ub4e4\ub3c4 \uc2ec\uac01\ud55c \ubd80\uc0c1\uc744 \uc785\ub294\ub2e4. \uc7ac\ud3b8\uc9d1\ud310\uc5d0\uc11c \ucd94\uac00\ub41c \uc7a5\uba74\uc5d0\uc11c\ub294 \uc774 \ud3ed\ubc1c \uc640\uc911\uc5d0 \uc8c4\uc218 \ud55c\uba85\uc774 \uc790\uc2e0\uc744 \ud76c\uc0dd\ud574 \ud3d0\uae30\ubb3c \uc800\uc7a5\uace0\ub85c \uc5d0\uc774\ub9ac\uc5b8\uc744 \uc720\uc778\ud558\uace0, \uc548\uc5d0 \uac19\uc774 \uac19\ud600\uc11c \uc5d0\uc774\ub9ac\uc5b8\uc744 \uac00\ub450\ub294\ub370 \uc131\uacf5\ud55c\ub2e4. \ud558\uc9c0\ub9cc \uc55e\uc11c \uc5d0\uc774\ub9ac\uc5b8\uc744 \uc758\ubb34\uc2e4\uc5d0\uc11c \ubcf4\uace0 \uc815\uc2e0\uc774 \uc774\uc0c1\ud574\uc9c4 \uace8\ub9ad\uc740 \ud3d0\uae30\ubb3c \uc800\uc7a5\uace0 \uc55e\uc5d0\uc11c \uc5d0\uc774\ub9ac\uc5b8\uc744 \uc9c0\ud0a4\ub358 \ub3d9\ub8cc \uc8c4\uc218\ub97c \uc8fd\uc5ec\uc11c \uc548\uc5d0 \uac07\ud78c \uc5d0\uc774\ub9ac\uc5b8\uc744 \ub3c4\ub85c \ud480\uc5b4\uc8fc\uace0 \uc790\uc2e0\ub3c4 \uc8fd\uace0 \ub9cc\ub2e4.", "paragraph_answer": "\uadf8\ub9ac\uace0 \uadf8 \ud6c4 \uc8c4\uc218\ub4e4\uc774 \ud558\ub098\ub458\uc529 \uc5d0\uc774\ub9ac\uc5b8\ub4e4\uc5d0\uac8c \uc0b4\ud574\ub2f9\ud558\uae30 \uc2dc\uc791\ud558\uace0, \ub3d9\ub8cc \uc8c4\uc218\ub4e4\ub85c\ubd80\ud130 \uc800\ub2a5\uc544 \ucde8\uae09\uc744 \ubc1b\ub358 \uc8c4\uc218 \uace8\ub9ad\uc774 \uc5d0\uc774\ub9ac\uc5b8\uc744 \ubcf4\uc558\ub2e4\uace0 \ub9d0\ud558\uc9c0\ub9cc \ub9ac\ud50c\ub9ac\ub97c \uc81c\uc678\ud55c \uc564\ub4dc\ub958\uc2a4 \uad50\ub3c4\uc18c\uc7a5\uc744 \ube44\ub86f\ud574 \ub2e4\ub978 \uc774\ub4e4\uc740 \uadf8 \uc774\uc57c\uae30\ub97c \ubbff\uc9c0 \uc54a\ub294\ub2e4. \uc624\ud788\ub824 \uac19\uc774 \uc815\uc2e0\ub098\uac04 \uc0ac\ub78c \ucde8\uae09\uc744 \ubc1b\uc73c\uba70 \uc758\ubb34\uc2e4\uc5d0 \uad6c\uae08\ub41c \ub9ac\ud50c\ub9ac\ub294 \ud074\ub808\uba58\uc2a4\uac00 \uc758\ubb34\uc2e4\uc5d0 \ub4e4\uc5b4\uc628 \uc5d0\uc774\ub9ac\uc5b8\uc5d0\uac8c \uc0b4\ud574\ub2f9\ud558\uc790, \uc564\ub4dc\ub958\uc2a4 \uad50\ub3c4\uc18c\uc7a5\uc5d0\uac8c \ub2ec\ub824\uac00 \uc0ac\uc2e4\uc744 \uc54c\ub9ac\uc9c0\ub9cc \uc5d0\uc774\ub9ac\uc5b8\uc774 \uadf8\ub9c8\uc800 \uc8c4\uc218\ub4e4\uc774 \ubcf4\ub294 \uc55e\uc5d0\uc11c \uc0b4\ud574\ud55c\ub2e4. \ud55c\uc21c\uac04\uc5d0 \uad6c\uc2ec\uc810\uc744 \uc783\uc740 \uc8c4\uc218\ub4e4\uc740 \uadf8\ub4e4 \uc0ac\uc774\uc5d0\uc11c \uc2e0\uc559\uc2ec\uc73c\ub85c \ub2e8\uacb0\uc744 \uc720\uc9c0\ud558\ub294\ub370 \uc55e\uc7a5\uc130\ub358 \uc8c4\uc218 \ub51c\ub860\uc744 \ub9ac\ub354\ub85c \uc0bc\uae38 \uc6d0\ud558\ub098, \ub51c\ub860\uc740 \uac70\ubd80\ud558\uace0 \ub9ac\ud50c\ub9ac\uac00 \ub9ac\ub354\uac00 \ub418\uae38 \uad8c\uc720\ud55c\ub2e4. \ub9ac\ud50c\ub9ac\ub294 \ubb34\uae30\uac00 \uc5c6\ub294 \uad50\ub3c4\uc18c\ub0b4 \ud2b9\uc131\uc0c1, \uc720\ub3c5\uc131 \uc778\ud654\ubb3c\uc9c8\uc744 \uc774\uc6a9\ud574 \uc5d0\uc774\ub9ac\uc5b8\uc744 \ud3d0\uae30\ubb3c \uc800\uc7a5\uace0 \ub85c \uc720\uc778\ud574 \uac00\ub46c\ub458 \uacc4\ud68d\uc744 \uc138\uc6b4\ub2e4. \ud558\uc9c0\ub9cc \uc2e0\ud638\ud0c4\uc744 \uac00\uc9c4 \uc8c4\uc218 \uc911 \ud558\ub098\uac00 \uc5d0\uc774\ub9ac\uc5b8\uc5d0\uac8c \uacf5\uaca9\ubc1b\uc73c\uba74\uc11c \uc778\ud654\ubb3c\uc9c8\uc5d0 \ubd88\uc774 \ubd99\uc5b4 \ud3ed\ubc1c\ud574 \ubc84\ub9ac\ub294 \ubc14\ub78c\uc5d0 \uc8c4\uc218\ub4e4 \ubc18\uc808\uc740 \ubd88\uc5d0 \ud0c0 \uc8fd\uace0, \ub0a8\uc740 \uc8c4\uc218\ub4e4\ub3c4 \uc2ec\uac01\ud55c \ubd80\uc0c1\uc744 \uc785\ub294\ub2e4. \uc7ac\ud3b8\uc9d1\ud310\uc5d0\uc11c \ucd94\uac00\ub41c \uc7a5\uba74\uc5d0\uc11c\ub294 \uc774 \ud3ed\ubc1c \uc640\uc911\uc5d0 \uc8c4\uc218 \ud55c\uba85\uc774 \uc790\uc2e0\uc744 \ud76c\uc0dd\ud574 \ud3d0\uae30\ubb3c \uc800\uc7a5\uace0\ub85c \uc5d0\uc774\ub9ac\uc5b8\uc744 \uc720\uc778\ud558\uace0, \uc548\uc5d0 \uac19\uc774 \uac19\ud600\uc11c \uc5d0\uc774\ub9ac\uc5b8\uc744 \uac00\ub450\ub294\ub370 \uc131\uacf5\ud55c\ub2e4. \ud558\uc9c0\ub9cc \uc55e\uc11c \uc5d0\uc774\ub9ac\uc5b8\uc744 \uc758\ubb34\uc2e4\uc5d0\uc11c \ubcf4\uace0 \uc815\uc2e0\uc774 \uc774\uc0c1\ud574\uc9c4 \uace8\ub9ad\uc740 \ud3d0\uae30\ubb3c \uc800\uc7a5\uace0 \uc55e\uc5d0\uc11c \uc5d0\uc774\ub9ac\uc5b8\uc744 \uc9c0\ud0a4\ub358 \ub3d9\ub8cc \uc8c4\uc218\ub97c \uc8fd\uc5ec\uc11c \uc548\uc5d0 \uac07\ud78c \uc5d0\uc774\ub9ac\uc5b8\uc744 \ub3c4\ub85c \ud480\uc5b4\uc8fc\uace0 \uc790\uc2e0\ub3c4 \uc8fd\uace0 \ub9cc\ub2e4.", "sentence_answer": "\ub9ac\ud50c\ub9ac\ub294 \ubb34\uae30\uac00 \uc5c6\ub294 \uad50\ub3c4\uc18c\ub0b4 \ud2b9\uc131\uc0c1, \uc720\ub3c5\uc131 \uc778\ud654\ubb3c\uc9c8\uc744 \uc774\uc6a9\ud574 \uc5d0\uc774\ub9ac\uc5b8\uc744 \ud3d0\uae30\ubb3c \uc800\uc7a5\uace0 \ub85c \uc720\uc778\ud574 \uac00\ub46c\ub458 \uacc4\ud68d\uc744 \uc138\uc6b4\ub2e4.", "paragraph_id": "6540728-1-1", "paragraph_question": "question: \ub9ac\ud50c\ub9ac\uac00 \uc5d0\uc774\ub9ac\uc5b8\uc744 \uac00\ub450\ub824\uace0 \ud55c \uacf3\uc740 \uc5b4\ub514\uc778\uac00?, context: \uadf8\ub9ac\uace0 \uadf8 \ud6c4 \uc8c4\uc218\ub4e4\uc774 \ud558\ub098\ub458\uc529 \uc5d0\uc774\ub9ac\uc5b8\ub4e4\uc5d0\uac8c \uc0b4\ud574\ub2f9\ud558\uae30 \uc2dc\uc791\ud558\uace0, \ub3d9\ub8cc \uc8c4\uc218\ub4e4\ub85c\ubd80\ud130 \uc800\ub2a5\uc544 \ucde8\uae09\uc744 \ubc1b\ub358 \uc8c4\uc218 \uace8\ub9ad\uc774 \uc5d0\uc774\ub9ac\uc5b8\uc744 \ubcf4\uc558\ub2e4\uace0 \ub9d0\ud558\uc9c0\ub9cc \ub9ac\ud50c\ub9ac\ub97c \uc81c\uc678\ud55c \uc564\ub4dc\ub958\uc2a4 \uad50\ub3c4\uc18c\uc7a5\uc744 \ube44\ub86f\ud574 \ub2e4\ub978 \uc774\ub4e4\uc740 \uadf8 \uc774\uc57c\uae30\ub97c \ubbff\uc9c0 \uc54a\ub294\ub2e4. \uc624\ud788\ub824 \uac19\uc774 \uc815\uc2e0\ub098\uac04 \uc0ac\ub78c \ucde8\uae09\uc744 \ubc1b\uc73c\uba70 \uc758\ubb34\uc2e4\uc5d0 \uad6c\uae08\ub41c \ub9ac\ud50c\ub9ac\ub294 \ud074\ub808\uba58\uc2a4\uac00 \uc758\ubb34\uc2e4\uc5d0 \ub4e4\uc5b4\uc628 \uc5d0\uc774\ub9ac\uc5b8\uc5d0\uac8c \uc0b4\ud574\ub2f9\ud558\uc790, \uc564\ub4dc\ub958\uc2a4 \uad50\ub3c4\uc18c\uc7a5\uc5d0\uac8c \ub2ec\ub824\uac00 \uc0ac\uc2e4\uc744 \uc54c\ub9ac\uc9c0\ub9cc \uc5d0\uc774\ub9ac\uc5b8\uc774 \uadf8\ub9c8\uc800 \uc8c4\uc218\ub4e4\uc774 \ubcf4\ub294 \uc55e\uc5d0\uc11c \uc0b4\ud574\ud55c\ub2e4. \ud55c\uc21c\uac04\uc5d0 \uad6c\uc2ec\uc810\uc744 \uc783\uc740 \uc8c4\uc218\ub4e4\uc740 \uadf8\ub4e4 \uc0ac\uc774\uc5d0\uc11c \uc2e0\uc559\uc2ec\uc73c\ub85c \ub2e8\uacb0\uc744 \uc720\uc9c0\ud558\ub294\ub370 \uc55e\uc7a5\uc130\ub358 \uc8c4\uc218 \ub51c\ub860\uc744 \ub9ac\ub354\ub85c \uc0bc\uae38 \uc6d0\ud558\ub098, \ub51c\ub860\uc740 \uac70\ubd80\ud558\uace0 \ub9ac\ud50c\ub9ac\uac00 \ub9ac\ub354\uac00 \ub418\uae38 \uad8c\uc720\ud55c\ub2e4. \ub9ac\ud50c\ub9ac\ub294 \ubb34\uae30\uac00 \uc5c6\ub294 \uad50\ub3c4\uc18c\ub0b4 \ud2b9\uc131\uc0c1, \uc720\ub3c5\uc131 \uc778\ud654\ubb3c\uc9c8\uc744 \uc774\uc6a9\ud574 \uc5d0\uc774\ub9ac\uc5b8\uc744 \ud3d0\uae30\ubb3c \uc800\uc7a5\uace0\ub85c \uc720\uc778\ud574 \uac00\ub46c\ub458 \uacc4\ud68d\uc744 \uc138\uc6b4\ub2e4. \ud558\uc9c0\ub9cc \uc2e0\ud638\ud0c4\uc744 \uac00\uc9c4 \uc8c4\uc218 \uc911 \ud558\ub098\uac00 \uc5d0\uc774\ub9ac\uc5b8\uc5d0\uac8c \uacf5\uaca9\ubc1b\uc73c\uba74\uc11c \uc778\ud654\ubb3c\uc9c8\uc5d0 \ubd88\uc774 \ubd99\uc5b4 \ud3ed\ubc1c\ud574 \ubc84\ub9ac\ub294 \ubc14\ub78c\uc5d0 \uc8c4\uc218\ub4e4 \ubc18\uc808\uc740 \ubd88\uc5d0 \ud0c0 \uc8fd\uace0, \ub0a8\uc740 \uc8c4\uc218\ub4e4\ub3c4 \uc2ec\uac01\ud55c \ubd80\uc0c1\uc744 \uc785\ub294\ub2e4. \uc7ac\ud3b8\uc9d1\ud310\uc5d0\uc11c \ucd94\uac00\ub41c \uc7a5\uba74\uc5d0\uc11c\ub294 \uc774 \ud3ed\ubc1c \uc640\uc911\uc5d0 \uc8c4\uc218 \ud55c\uba85\uc774 \uc790\uc2e0\uc744 \ud76c\uc0dd\ud574 \ud3d0\uae30\ubb3c \uc800\uc7a5\uace0\ub85c \uc5d0\uc774\ub9ac\uc5b8\uc744 \uc720\uc778\ud558\uace0, \uc548\uc5d0 \uac19\uc774 \uac19\ud600\uc11c \uc5d0\uc774\ub9ac\uc5b8\uc744 \uac00\ub450\ub294\ub370 \uc131\uacf5\ud55c\ub2e4. \ud558\uc9c0\ub9cc \uc55e\uc11c \uc5d0\uc774\ub9ac\uc5b8\uc744 \uc758\ubb34\uc2e4\uc5d0\uc11c \ubcf4\uace0 \uc815\uc2e0\uc774 \uc774\uc0c1\ud574\uc9c4 \uace8\ub9ad\uc740 \ud3d0\uae30\ubb3c \uc800\uc7a5\uace0 \uc55e\uc5d0\uc11c \uc5d0\uc774\ub9ac\uc5b8\uc744 \uc9c0\ud0a4\ub358 \ub3d9\ub8cc \uc8c4\uc218\ub97c \uc8fd\uc5ec\uc11c \uc548\uc5d0 \uac07\ud78c \uc5d0\uc774\ub9ac\uc5b8\uc744 \ub3c4\ub85c \ud480\uc5b4\uc8fc\uace0 \uc790\uc2e0\ub3c4 \uc8fd\uace0 \ub9cc\ub2e4."} {"question": "\ud5e8\ub9ac\uc5d0\ud0c0\uac00 \ubb3c\uc744 \ub9c8\uc2dc\uace0 \uac11\uc790\uae30 \uc4f0\ub7ec\uc9c4 \ub54c\ub294?", "paragraph": "1668\ub144 \ub8e8\uc774 14\uc138\ub294 \ud5e8\ub9ac\uc5d0\ud0c0\uc5d0\uac8c \ud504\ub791\uc2a4-\uc601\uad6d\uac04\uc758 \ub3d9\ub9f9\uc744 \uc704\ud55c \ud611\ub825\uc744 \ubd80\ud0c1\ud588\ub2e4. \uc655\uc758 \uc694\uccad\uc744 \ubc1b\uace0 \uc601\uad6d\uc73c\ub85c \ud5a5\ud55c \ud5e8\ub9ac\uc5d0\ud0c0\ub294 \ud3c9\uc18c \uc0ac\uc774\uac00 \uc88b\ub358 \uc624\ube60 \ucc30\uc2a4 2\uc138\ub97c \ub9cc\ub098 \ubb34\uc0ac\ud788 \ub3c4\ubc84 \ubc00\uc57d\uc744 \ub9fa\ub294 \ub370 \uc131\uacf5\ud588\ub2e4. \ud504\ub791\uc2a4\ub85c \ub3cc\uc544\uc628 \uadf8 \ud574 6\uc6d4 29\uc77c \uc624\ud6c4, \ud5e8\ub9ac\uc5d0\ud0c0\ub294 \uce58\ucee4\ub9ac\uac00 \ub4e0 \ubb3c\uc744 \ub9c8\uc2dc\uace0 \uac11\uc790\uae30 \uc4f0\ub7ec\uc838 \ubab9\uc2dc \uad34\ub85c\uc6cc\ud558\ub2e4 \uc774\ud2bf\ub0a0 \uc0c8\ubcbd \uc758\ubb38\uc758 \uc8fd\uc74c\uc744 \ub9de\uc558\ub2e4. \ud544\ub9ac\ud504\uc758 \uc560\uc778 \ub85c\ub80c \uacf5\uc774 \ud5e8\ub9ac\uc5d0\ud0c0\uc758 \ucc3b\uc794\uc5d0 \ub3c5\uc744 \ud0d4\ub2e4\ub294 \uc18c\ubb38\uc774 \ub3cc\uc558\uace0 \ud5e8\ub9ac\uc5d0\ud0c0\uc758 \uc2dc\uc2e0\uc740 \ud574\ubd80\ub418\uc5c8\uc73c\ub098 \uc758\uc0ac\ub4e4\uc740 \ub3c5\uc0b4\uc758 \ud754\uc801\uc744 \ud655\uc778\ud560 \uc218 \uc5c6\ub2e4\ub294\ub370 \uc758\uacac\uc744 \uc77c\uce58\uc2dc\ucf30\ub2e4. \uadf8\ub140\uc758 \uc0ac\uc778\uc5d0 \ub300\ud574\uc11c\ub294 \uc758\uc0ac\ub4e4\ub3c4 \uc758\uacac\uc774 \ubd84\ubd84\ud588\uace0 \uacf5\uc2dd\uc801\uc73c\ub85c\ub294 \ucf5c\ub808\ub77c\ub77c\uace0 \ubc1c\ud45c\ub418\uc5c8\uc73c\ub098 \uc2e4\uc81c\ub85c\ub294 \ubcf5\ub9c9\uc5fc\uc774\uc5c8\ub2e4\uace0 \uc5ec\uaca8\uc9c0\uace0 \uc788\ub2e4.", "answer": "6\uc6d4 29\uc77c", "sentence": "\ud504\ub791\uc2a4\ub85c \ub3cc\uc544\uc628 \uadf8 \ud574 6\uc6d4 29\uc77c \uc624\ud6c4, \ud5e8\ub9ac\uc5d0\ud0c0\ub294 \uce58\ucee4\ub9ac\uac00 \ub4e0 \ubb3c\uc744 \ub9c8\uc2dc\uace0 \uac11\uc790\uae30 \uc4f0\ub7ec\uc838 \ubab9\uc2dc \uad34\ub85c\uc6cc\ud558\ub2e4 \uc774\ud2bf\ub0a0 \uc0c8\ubcbd \uc758\ubb38\uc758 \uc8fd\uc74c\uc744 \ub9de\uc558\ub2e4.", "paragraph_sentence": "1668\ub144 \ub8e8\uc774 14\uc138\ub294 \ud5e8\ub9ac\uc5d0\ud0c0\uc5d0\uac8c \ud504\ub791\uc2a4-\uc601\uad6d\uac04\uc758 \ub3d9\ub9f9\uc744 \uc704\ud55c \ud611\ub825\uc744 \ubd80\ud0c1\ud588\ub2e4. \uc655\uc758 \uc694\uccad\uc744 \ubc1b\uace0 \uc601\uad6d\uc73c\ub85c \ud5a5\ud55c \ud5e8\ub9ac\uc5d0\ud0c0\ub294 \ud3c9\uc18c \uc0ac\uc774\uac00 \uc88b\ub358 \uc624\ube60 \ucc30\uc2a4 2\uc138\ub97c \ub9cc\ub098 \ubb34\uc0ac\ud788 \ub3c4\ubc84 \ubc00\uc57d\uc744 \ub9fa\ub294 \ub370 \uc131\uacf5\ud588\ub2e4. \ud504\ub791\uc2a4\ub85c \ub3cc\uc544\uc628 \uadf8 \ud574 6\uc6d4 29\uc77c \uc624\ud6c4, \ud5e8\ub9ac\uc5d0\ud0c0\ub294 \uce58\ucee4\ub9ac\uac00 \ub4e0 \ubb3c\uc744 \ub9c8\uc2dc\uace0 \uac11\uc790\uae30 \uc4f0\ub7ec\uc838 \ubab9\uc2dc \uad34\ub85c\uc6cc\ud558\ub2e4 \uc774\ud2bf\ub0a0 \uc0c8\ubcbd \uc758\ubb38\uc758 \uc8fd\uc74c\uc744 \ub9de\uc558\ub2e4. \ud544\ub9ac\ud504\uc758 \uc560\uc778 \ub85c\ub80c \uacf5\uc774 \ud5e8\ub9ac\uc5d0\ud0c0\uc758 \ucc3b\uc794\uc5d0 \ub3c5\uc744 \ud0d4\ub2e4\ub294 \uc18c\ubb38\uc774 \ub3cc\uc558\uace0 \ud5e8\ub9ac\uc5d0\ud0c0\uc758 \uc2dc\uc2e0\uc740 \ud574\ubd80\ub418\uc5c8\uc73c\ub098 \uc758\uc0ac\ub4e4\uc740 \ub3c5\uc0b4\uc758 \ud754\uc801\uc744 \ud655\uc778\ud560 \uc218 \uc5c6\ub2e4\ub294\ub370 \uc758\uacac\uc744 \uc77c\uce58\uc2dc\ucf30\ub2e4. \uadf8\ub140\uc758 \uc0ac\uc778\uc5d0 \ub300\ud574\uc11c\ub294 \uc758\uc0ac\ub4e4\ub3c4 \uc758\uacac\uc774 \ubd84\ubd84\ud588\uace0 \uacf5\uc2dd\uc801\uc73c\ub85c\ub294 \ucf5c\ub808\ub77c\ub77c\uace0 \ubc1c\ud45c\ub418\uc5c8\uc73c\ub098 \uc2e4\uc81c\ub85c\ub294 \ubcf5\ub9c9\uc5fc\uc774\uc5c8\ub2e4\uace0 \uc5ec\uaca8\uc9c0\uace0 \uc788\ub2e4.", "paragraph_answer": "1668\ub144 \ub8e8\uc774 14\uc138\ub294 \ud5e8\ub9ac\uc5d0\ud0c0\uc5d0\uac8c \ud504\ub791\uc2a4-\uc601\uad6d\uac04\uc758 \ub3d9\ub9f9\uc744 \uc704\ud55c \ud611\ub825\uc744 \ubd80\ud0c1\ud588\ub2e4. \uc655\uc758 \uc694\uccad\uc744 \ubc1b\uace0 \uc601\uad6d\uc73c\ub85c \ud5a5\ud55c \ud5e8\ub9ac\uc5d0\ud0c0\ub294 \ud3c9\uc18c \uc0ac\uc774\uac00 \uc88b\ub358 \uc624\ube60 \ucc30\uc2a4 2\uc138\ub97c \ub9cc\ub098 \ubb34\uc0ac\ud788 \ub3c4\ubc84 \ubc00\uc57d\uc744 \ub9fa\ub294 \ub370 \uc131\uacf5\ud588\ub2e4. \ud504\ub791\uc2a4\ub85c \ub3cc\uc544\uc628 \uadf8 \ud574 6\uc6d4 29\uc77c \uc624\ud6c4, \ud5e8\ub9ac\uc5d0\ud0c0\ub294 \uce58\ucee4\ub9ac\uac00 \ub4e0 \ubb3c\uc744 \ub9c8\uc2dc\uace0 \uac11\uc790\uae30 \uc4f0\ub7ec\uc838 \ubab9\uc2dc \uad34\ub85c\uc6cc\ud558\ub2e4 \uc774\ud2bf\ub0a0 \uc0c8\ubcbd \uc758\ubb38\uc758 \uc8fd\uc74c\uc744 \ub9de\uc558\ub2e4. \ud544\ub9ac\ud504\uc758 \uc560\uc778 \ub85c\ub80c \uacf5\uc774 \ud5e8\ub9ac\uc5d0\ud0c0\uc758 \ucc3b\uc794\uc5d0 \ub3c5\uc744 \ud0d4\ub2e4\ub294 \uc18c\ubb38\uc774 \ub3cc\uc558\uace0 \ud5e8\ub9ac\uc5d0\ud0c0\uc758 \uc2dc\uc2e0\uc740 \ud574\ubd80\ub418\uc5c8\uc73c\ub098 \uc758\uc0ac\ub4e4\uc740 \ub3c5\uc0b4\uc758 \ud754\uc801\uc744 \ud655\uc778\ud560 \uc218 \uc5c6\ub2e4\ub294\ub370 \uc758\uacac\uc744 \uc77c\uce58\uc2dc\ucf30\ub2e4. \uadf8\ub140\uc758 \uc0ac\uc778\uc5d0 \ub300\ud574\uc11c\ub294 \uc758\uc0ac\ub4e4\ub3c4 \uc758\uacac\uc774 \ubd84\ubd84\ud588\uace0 \uacf5\uc2dd\uc801\uc73c\ub85c\ub294 \ucf5c\ub808\ub77c\ub77c\uace0 \ubc1c\ud45c\ub418\uc5c8\uc73c\ub098 \uc2e4\uc81c\ub85c\ub294 \ubcf5\ub9c9\uc5fc\uc774\uc5c8\ub2e4\uace0 \uc5ec\uaca8\uc9c0\uace0 \uc788\ub2e4.", "sentence_answer": "\ud504\ub791\uc2a4\ub85c \ub3cc\uc544\uc628 \uadf8 \ud574 6\uc6d4 29\uc77c \uc624\ud6c4, \ud5e8\ub9ac\uc5d0\ud0c0\ub294 \uce58\ucee4\ub9ac\uac00 \ub4e0 \ubb3c\uc744 \ub9c8\uc2dc\uace0 \uac11\uc790\uae30 \uc4f0\ub7ec\uc838 \ubab9\uc2dc \uad34\ub85c\uc6cc\ud558\ub2e4 \uc774\ud2bf\ub0a0 \uc0c8\ubcbd \uc758\ubb38\uc758 \uc8fd\uc74c\uc744 \ub9de\uc558\ub2e4.", "paragraph_id": "6586368-2-2", "paragraph_question": "question: \ud5e8\ub9ac\uc5d0\ud0c0\uac00 \ubb3c\uc744 \ub9c8\uc2dc\uace0 \uac11\uc790\uae30 \uc4f0\ub7ec\uc9c4 \ub54c\ub294?, context: 1668\ub144 \ub8e8\uc774 14\uc138\ub294 \ud5e8\ub9ac\uc5d0\ud0c0\uc5d0\uac8c \ud504\ub791\uc2a4-\uc601\uad6d\uac04\uc758 \ub3d9\ub9f9\uc744 \uc704\ud55c \ud611\ub825\uc744 \ubd80\ud0c1\ud588\ub2e4. \uc655\uc758 \uc694\uccad\uc744 \ubc1b\uace0 \uc601\uad6d\uc73c\ub85c \ud5a5\ud55c \ud5e8\ub9ac\uc5d0\ud0c0\ub294 \ud3c9\uc18c \uc0ac\uc774\uac00 \uc88b\ub358 \uc624\ube60 \ucc30\uc2a4 2\uc138\ub97c \ub9cc\ub098 \ubb34\uc0ac\ud788 \ub3c4\ubc84 \ubc00\uc57d\uc744 \ub9fa\ub294 \ub370 \uc131\uacf5\ud588\ub2e4. \ud504\ub791\uc2a4\ub85c \ub3cc\uc544\uc628 \uadf8 \ud574 6\uc6d4 29\uc77c \uc624\ud6c4, \ud5e8\ub9ac\uc5d0\ud0c0\ub294 \uce58\ucee4\ub9ac\uac00 \ub4e0 \ubb3c\uc744 \ub9c8\uc2dc\uace0 \uac11\uc790\uae30 \uc4f0\ub7ec\uc838 \ubab9\uc2dc \uad34\ub85c\uc6cc\ud558\ub2e4 \uc774\ud2bf\ub0a0 \uc0c8\ubcbd \uc758\ubb38\uc758 \uc8fd\uc74c\uc744 \ub9de\uc558\ub2e4. \ud544\ub9ac\ud504\uc758 \uc560\uc778 \ub85c\ub80c \uacf5\uc774 \ud5e8\ub9ac\uc5d0\ud0c0\uc758 \ucc3b\uc794\uc5d0 \ub3c5\uc744 \ud0d4\ub2e4\ub294 \uc18c\ubb38\uc774 \ub3cc\uc558\uace0 \ud5e8\ub9ac\uc5d0\ud0c0\uc758 \uc2dc\uc2e0\uc740 \ud574\ubd80\ub418\uc5c8\uc73c\ub098 \uc758\uc0ac\ub4e4\uc740 \ub3c5\uc0b4\uc758 \ud754\uc801\uc744 \ud655\uc778\ud560 \uc218 \uc5c6\ub2e4\ub294\ub370 \uc758\uacac\uc744 \uc77c\uce58\uc2dc\ucf30\ub2e4. \uadf8\ub140\uc758 \uc0ac\uc778\uc5d0 \ub300\ud574\uc11c\ub294 \uc758\uc0ac\ub4e4\ub3c4 \uc758\uacac\uc774 \ubd84\ubd84\ud588\uace0 \uacf5\uc2dd\uc801\uc73c\ub85c\ub294 \ucf5c\ub808\ub77c\ub77c\uace0 \ubc1c\ud45c\ub418\uc5c8\uc73c\ub098 \uc2e4\uc81c\ub85c\ub294 \ubcf5\ub9c9\uc5fc\uc774\uc5c8\ub2e4\uace0 \uc5ec\uaca8\uc9c0\uace0 \uc788\ub2e4."} {"question": "\uc7ad \ube14\ub799\uc774 \uc131\uc6b0\ub97c \ub9e1\uc544 \uac1c\ubd09\ud55c \ucd1d 20.3\ubc00\ub9ac\uc5b8\ub2ec\ub7ec\ub97c \uae30\ub85d\ud55c \uc560\ub2c8\uba54\uc774\uc158\uc740?", "paragraph": "\uadf8\ub294 \uace7 \uc8fc\uc5f0\uae09\uc758 \ubc30\uc6b0\uac00 \ub418\uc5b4 \u300a\ub0b4\uac90 \ub108\ubb34 \uac00\ubcbc\uc6b4 \uadf8\ub140\u300b, \u300a\uc2a4\ucfe8\uc624\ube0c\ub77d\u300b, \u300a\ub098\ucd08\ub9ac\ube0c\ub808\u300b, \u300a\ud130\ub124\uc774\uc154\uc2a4 D: \uc6b4\uba85\uc758 \ud53c\ud06c\u300b, \u300a\ud0b9\ucf69\u300b, \u300a\uc774\uc5b4 \uc6d0\u300b, \u300a\uac78\ub9ac\ubc84 \uc5ec\ud589\uae30\u300b \ub4f1\uc5d0 \ucd9c\uc5f0\ud558\uc600\ub2e4. \ub610\ud55c \u300a\ucff5\ud478 \ud32c\ub354\u300b\uc5d0\uc11c \uc131\uc6b0\ub97c \ub9e1\uc544 \uac1c\ubd09 \uccab\ub0a0\uc778 2008\ub144 6\uc6d4 6\uc77c, \ucd1d 20.3\ubc00\ub9ac\uc5b8 \ub2ec\ub7ec\ub97c \uae30\ub85d\ud558\uc600\ub2e4. \uadf8\ub294 \uc774 \uc601\ud654\uc5d0\uc11c \ub450 \ubc88\uc758 \uc544\uce74\ub370\ubbf8\uc0c1\uc744 \uc218\uc0c1\ud55c \ub354\uc2a4\ud2f4 \ud638\ud504\uba3c\uacfc \uacf5\ub3d9 \uc8fc\uc5f0\uc744 \ub9e1\uc558\ub2e4. \uadf8\uc758 \ub2e4\uc74c \uc601\ud654 \u300a\ube45\uc774\uc5b4\u300b\ub294 2011\ub144 10\uc6d4 \uac1c\ubd09\ub418\uc5c8\uace0, \uc624\uc5b8 \uc70c\uc2a8, \uc2a4\ud2f0\ube0c \ub9c8\ud2f4, \uc870\ubca0\uc2a4 \uc70c\ub9ac\uc5c4\uc2a4 \ub4f1\uacfc \uacf5\ub3d9 \uce90\uc2a4\ud305 \ub418\uc5c8\ub2e4. \uadf8\ub294 \ucf54\ubbf8\ub514 \ubc30\uc6b0 \ubaa8\uc784\uc778 \ud504\ub7ab\ud329\uc758 \uc77c\uc6d0\uc73c\ub85c \uc77c\uceec\uc5b4\uc9c0\uba70, \uc774 \ubaa8\uc784\uc740 \uc790\uc8fc \ud568\uaed8 \uacf5\uc5f0\uc744 \uac16\ub294 \ubc30\uc6b0\ub4e4\ub85c \uc774\ub8e8\uc5b4\uc838 \uc788\ub2e4. \uba64\ubc84\ub4e4\ub85c\ub294 \uc624\uc5b8 \uc70c\uc2a8, \ubca4 \uc2a4\ud2f8\ub7ec, \uc70c \ud398\ub7f4, \ube48\uc2a4 \ubcf8, \uc2a4\ud2f0\ube0c \ucee4\ub810 \ub4f1\uc774 \uc788\ub2e4.", "answer": "\ucff5\ud478 \ud32c\ub354", "sentence": "\ub610\ud55c \u300a \ucff5\ud478 \ud32c\ub354 \u300b\uc5d0\uc11c \uc131\uc6b0\ub97c \ub9e1\uc544 \uac1c\ubd09 \uccab\ub0a0\uc778 2008\ub144 6\uc6d4 6\uc77c, \ucd1d 20.3\ubc00\ub9ac\uc5b8 \ub2ec\ub7ec\ub97c \uae30\ub85d\ud558\uc600\ub2e4.", "paragraph_sentence": "\uadf8\ub294 \uace7 \uc8fc\uc5f0\uae09\uc758 \ubc30\uc6b0\uac00 \ub418\uc5b4 \u300a\ub0b4\uac90 \ub108\ubb34 \uac00\ubcbc\uc6b4 \uadf8\ub140\u300b, \u300a\uc2a4\ucfe8\uc624\ube0c\ub77d\u300b, \u300a\ub098\ucd08\ub9ac\ube0c\ub808\u300b, \u300a\ud130\ub124\uc774\uc154\uc2a4 D: \uc6b4\uba85\uc758 \ud53c\ud06c\u300b, \u300a\ud0b9\ucf69\u300b, \u300a\uc774\uc5b4 \uc6d0\u300b, \u300a\uac78\ub9ac\ubc84 \uc5ec\ud589\uae30\u300b \ub4f1\uc5d0 \ucd9c\uc5f0\ud558\uc600\ub2e4. \ub610\ud55c \u300a \ucff5\ud478 \ud32c\ub354 \u300b\uc5d0\uc11c \uc131\uc6b0\ub97c \ub9e1\uc544 \uac1c\ubd09 \uccab\ub0a0\uc778 2008\ub144 6\uc6d4 6\uc77c, \ucd1d 20.3\ubc00\ub9ac\uc5b8 \ub2ec\ub7ec\ub97c \uae30\ub85d\ud558\uc600\ub2e4. \uadf8\ub294 \uc774 \uc601\ud654\uc5d0\uc11c \ub450 \ubc88\uc758 \uc544\uce74\ub370\ubbf8\uc0c1\uc744 \uc218\uc0c1\ud55c \ub354\uc2a4\ud2f4 \ud638\ud504\uba3c\uacfc \uacf5\ub3d9 \uc8fc\uc5f0\uc744 \ub9e1\uc558\ub2e4. \uadf8\uc758 \ub2e4\uc74c \uc601\ud654 \u300a\ube45\uc774\uc5b4\u300b\ub294 2011\ub144 10\uc6d4 \uac1c\ubd09\ub418\uc5c8\uace0, \uc624\uc5b8 \uc70c\uc2a8, \uc2a4\ud2f0\ube0c \ub9c8\ud2f4, \uc870\ubca0\uc2a4 \uc70c\ub9ac\uc5c4\uc2a4 \ub4f1\uacfc \uacf5\ub3d9 \uce90\uc2a4\ud305 \ub418\uc5c8\ub2e4. \uadf8\ub294 \ucf54\ubbf8\ub514 \ubc30\uc6b0 \ubaa8\uc784\uc778 \ud504\ub7ab\ud329\uc758 \uc77c\uc6d0\uc73c\ub85c \uc77c\uceec\uc5b4\uc9c0\uba70, \uc774 \ubaa8\uc784\uc740 \uc790\uc8fc \ud568\uaed8 \uacf5\uc5f0\uc744 \uac16\ub294 \ubc30\uc6b0\ub4e4\ub85c \uc774\ub8e8\uc5b4\uc838 \uc788\ub2e4. \uba64\ubc84\ub4e4\ub85c\ub294 \uc624\uc5b8 \uc70c\uc2a8, \ubca4 \uc2a4\ud2f8\ub7ec, \uc70c \ud398\ub7f4, \ube48\uc2a4 \ubcf8, \uc2a4\ud2f0\ube0c \ucee4\ub810 \ub4f1\uc774 \uc788\ub2e4.", "paragraph_answer": "\uadf8\ub294 \uace7 \uc8fc\uc5f0\uae09\uc758 \ubc30\uc6b0\uac00 \ub418\uc5b4 \u300a\ub0b4\uac90 \ub108\ubb34 \uac00\ubcbc\uc6b4 \uadf8\ub140\u300b, \u300a\uc2a4\ucfe8\uc624\ube0c\ub77d\u300b, \u300a\ub098\ucd08\ub9ac\ube0c\ub808\u300b, \u300a\ud130\ub124\uc774\uc154\uc2a4 D: \uc6b4\uba85\uc758 \ud53c\ud06c\u300b, \u300a\ud0b9\ucf69\u300b, \u300a\uc774\uc5b4 \uc6d0\u300b, \u300a\uac78\ub9ac\ubc84 \uc5ec\ud589\uae30\u300b \ub4f1\uc5d0 \ucd9c\uc5f0\ud558\uc600\ub2e4. \ub610\ud55c \u300a \ucff5\ud478 \ud32c\ub354 \u300b\uc5d0\uc11c \uc131\uc6b0\ub97c \ub9e1\uc544 \uac1c\ubd09 \uccab\ub0a0\uc778 2008\ub144 6\uc6d4 6\uc77c, \ucd1d 20.3\ubc00\ub9ac\uc5b8 \ub2ec\ub7ec\ub97c \uae30\ub85d\ud558\uc600\ub2e4. \uadf8\ub294 \uc774 \uc601\ud654\uc5d0\uc11c \ub450 \ubc88\uc758 \uc544\uce74\ub370\ubbf8\uc0c1\uc744 \uc218\uc0c1\ud55c \ub354\uc2a4\ud2f4 \ud638\ud504\uba3c\uacfc \uacf5\ub3d9 \uc8fc\uc5f0\uc744 \ub9e1\uc558\ub2e4. \uadf8\uc758 \ub2e4\uc74c \uc601\ud654 \u300a\ube45\uc774\uc5b4\u300b\ub294 2011\ub144 10\uc6d4 \uac1c\ubd09\ub418\uc5c8\uace0, \uc624\uc5b8 \uc70c\uc2a8, \uc2a4\ud2f0\ube0c \ub9c8\ud2f4, \uc870\ubca0\uc2a4 \uc70c\ub9ac\uc5c4\uc2a4 \ub4f1\uacfc \uacf5\ub3d9 \uce90\uc2a4\ud305 \ub418\uc5c8\ub2e4. \uadf8\ub294 \ucf54\ubbf8\ub514 \ubc30\uc6b0 \ubaa8\uc784\uc778 \ud504\ub7ab\ud329\uc758 \uc77c\uc6d0\uc73c\ub85c \uc77c\uceec\uc5b4\uc9c0\uba70, \uc774 \ubaa8\uc784\uc740 \uc790\uc8fc \ud568\uaed8 \uacf5\uc5f0\uc744 \uac16\ub294 \ubc30\uc6b0\ub4e4\ub85c \uc774\ub8e8\uc5b4\uc838 \uc788\ub2e4. \uba64\ubc84\ub4e4\ub85c\ub294 \uc624\uc5b8 \uc70c\uc2a8, \ubca4 \uc2a4\ud2f8\ub7ec, \uc70c \ud398\ub7f4, \ube48\uc2a4 \ubcf8, \uc2a4\ud2f0\ube0c \ucee4\ub810 \ub4f1\uc774 \uc788\ub2e4.", "sentence_answer": "\ub610\ud55c \u300a \ucff5\ud478 \ud32c\ub354 \u300b\uc5d0\uc11c \uc131\uc6b0\ub97c \ub9e1\uc544 \uac1c\ubd09 \uccab\ub0a0\uc778 2008\ub144 6\uc6d4 6\uc77c, \ucd1d 20.3\ubc00\ub9ac\uc5b8 \ub2ec\ub7ec\ub97c \uae30\ub85d\ud558\uc600\ub2e4.", "paragraph_id": "5860330-1-0", "paragraph_question": "question: \uc7ad \ube14\ub799\uc774 \uc131\uc6b0\ub97c \ub9e1\uc544 \uac1c\ubd09\ud55c \ucd1d 20.3\ubc00\ub9ac\uc5b8\ub2ec\ub7ec\ub97c \uae30\ub85d\ud55c \uc560\ub2c8\uba54\uc774\uc158\uc740?, context: \uadf8\ub294 \uace7 \uc8fc\uc5f0\uae09\uc758 \ubc30\uc6b0\uac00 \ub418\uc5b4 \u300a\ub0b4\uac90 \ub108\ubb34 \uac00\ubcbc\uc6b4 \uadf8\ub140\u300b, \u300a\uc2a4\ucfe8\uc624\ube0c\ub77d\u300b, \u300a\ub098\ucd08\ub9ac\ube0c\ub808\u300b, \u300a\ud130\ub124\uc774\uc154\uc2a4 D: \uc6b4\uba85\uc758 \ud53c\ud06c\u300b, \u300a\ud0b9\ucf69\u300b, \u300a\uc774\uc5b4 \uc6d0\u300b, \u300a\uac78\ub9ac\ubc84 \uc5ec\ud589\uae30\u300b \ub4f1\uc5d0 \ucd9c\uc5f0\ud558\uc600\ub2e4. \ub610\ud55c \u300a\ucff5\ud478 \ud32c\ub354\u300b\uc5d0\uc11c \uc131\uc6b0\ub97c \ub9e1\uc544 \uac1c\ubd09 \uccab\ub0a0\uc778 2008\ub144 6\uc6d4 6\uc77c, \ucd1d 20.3\ubc00\ub9ac\uc5b8 \ub2ec\ub7ec\ub97c \uae30\ub85d\ud558\uc600\ub2e4. \uadf8\ub294 \uc774 \uc601\ud654\uc5d0\uc11c \ub450 \ubc88\uc758 \uc544\uce74\ub370\ubbf8\uc0c1\uc744 \uc218\uc0c1\ud55c \ub354\uc2a4\ud2f4 \ud638\ud504\uba3c\uacfc \uacf5\ub3d9 \uc8fc\uc5f0\uc744 \ub9e1\uc558\ub2e4. \uadf8\uc758 \ub2e4\uc74c \uc601\ud654 \u300a\ube45\uc774\uc5b4\u300b\ub294 2011\ub144 10\uc6d4 \uac1c\ubd09\ub418\uc5c8\uace0, \uc624\uc5b8 \uc70c\uc2a8, \uc2a4\ud2f0\ube0c \ub9c8\ud2f4, \uc870\ubca0\uc2a4 \uc70c\ub9ac\uc5c4\uc2a4 \ub4f1\uacfc \uacf5\ub3d9 \uce90\uc2a4\ud305 \ub418\uc5c8\ub2e4. \uadf8\ub294 \ucf54\ubbf8\ub514 \ubc30\uc6b0 \ubaa8\uc784\uc778 \ud504\ub7ab\ud329\uc758 \uc77c\uc6d0\uc73c\ub85c \uc77c\uceec\uc5b4\uc9c0\uba70, \uc774 \ubaa8\uc784\uc740 \uc790\uc8fc \ud568\uaed8 \uacf5\uc5f0\uc744 \uac16\ub294 \ubc30\uc6b0\ub4e4\ub85c \uc774\ub8e8\uc5b4\uc838 \uc788\ub2e4. \uba64\ubc84\ub4e4\ub85c\ub294 \uc624\uc5b8 \uc70c\uc2a8, \ubca4 \uc2a4\ud2f8\ub7ec, \uc70c \ud398\ub7f4, \ube48\uc2a4 \ubcf8, \uc2a4\ud2f0\ube0c \ucee4\ub810 \ub4f1\uc774 \uc788\ub2e4."} {"question": "\ud0dc\ud48d \ub9e4\ubbf8\uac00 \ud55c\ubc18\ub3c4\uc5d0 \ub3c4\ucc29\ud588\uc744 \ub54c\uc758 \uc774\ub3d9\uc18d\ub3c4\ub294 \uba87 km/h \uc778\uac00?", "paragraph": "\uc774\uc640 \uac19\uc774 \uae30\ub85d\uc801\uc778 \uac15\ud48d\uc774 \uc77c\uc5c8\ub358 \uc6d0\uc778\uc740 \ubb3c\ub860 \uac15\ub825\ud588\ub358 \ud0dc\ud48d\uc758 \uc138\ub825\uc774 \uc77c\ucc28\uc801\uc774\uc9c0\ub9cc, \ubd80\uac00\uc801\uc778 \uc694\uc778\uc73c\ub85c\uc11c \ub450 \uac00\uc9c0\ub97c \ub354 \uaf3d\uc744 \uc218 \uc788\ub2e4. \uba3c\uc800 \ud55c\ubc18\ub3c4 \uc0c1\ub959 \uc2dc 45 km/h \uc815\ub3c4\uc758 \uc0c1\ub2f9\ud788 \ube68\ub790\ub358 \ud0dc\ud48d\uc758 \uc774\ub3d9\uc18d\ub3c4\ub97c \ub4e4 \uc218 \uc788\ub294\ub370, \uc774 \uc774\ub3d9\uc18d\ub3c4\uac00 \ud0dc\ud48d\uc758 \ud48d\uc18d (\uc0c1\ub959 \uc2dc \ucd5c\ub300 \ud48d\uc18d 40 m/s) \uc5d0 \ub354\ud574\uc838 \uc704\ud5d8\ubc18\uc6d0\uc5d0\uc11c\uc758 \ubc14\ub78c\uc758 \ud798\uc744 \uadf8\ub9cc\ud07c \ub354 \uac15\ud558\uac8c \ud588\ub2e4. \ubc18\uc2dc\uacc4 \ubc29\ud5a5\uc73c\ub85c \ud68c\uc804\ud558\ub294 \ud0dc\ud48d\uc758 \ud2b9\uc131\uc0c1 \uc9c4\ud589\ubc29\ud5a5\uc758 \uc624\ub978\ucabd, \ub2e4\uc2dc \ub9d0\ud574 \uc704\ud5d8\ubc18\uc6d0\uc5d0\uc11c\ub294 \ud0dc\ud48d\uc758 \ud68c\uc804\uacfc \uc9c4\ud589\ubc29\ud5a5\uc774 \uc911\ucca9\ub418\uc5b4, \ud0dc\ud48d\uc758 \uc774\ub3d9\uc18d\ub3c4\uac00 \uadf8\ub300\ub85c \ud48d\uc18d\uc5d0 \ub354\ud574\uc9c0\uae30 \ub54c\ubb38\uc774\ub2e4. \uac8c\ub2e4\uac00 \ud0dc\ud48d \uc0c1\ub959 \uc77c\uc778 9\uc6d4 12\uc77c \uc624\ud6c4\uc758 \ud55c\ubc18\ub3c4 \uc8fc\ubcc0 \uae30\uc555\ubc30\uce58\ub294 \ub0a8\ucabd\uc73c\ub85c\ubd80\ud130 \ubd81\uc0c1\ud558\ub294 \ub9e4\uc6b0 \ub0ae\uc740 \uc911\uc2ec \uae30\uc555\uc758 \ud0dc\ud48d\uacfc \ubd81\ucabd\uc758 \uace0\uae30\uc555\uc774 \ub9c8\uc8fc\uce58\ub294 \ud615\uad6d\uc774 \ub418\uc5b4 \uc788\uc5b4, \uc774 \ubd80\uadfc\uc758 \uae30\uc555\uacbd\ub3c4\ub97c \uae09\uaca9\ud558\uac8c \ub9cc\ub4e6\uc5d0 \ub530\ub77c \uac15\ud48d\uc744 \uc720\ubc1c\uc2dc\ud0a4\ub294 \ub610 \ud558\ub098\uc758 \uc870\uac74\uc744 \ud615\uc131\ud588\ub2e4. \uc989, \uc774 \ub450\uac00\uc9c0 \uc694\uc18c\uac00 \ud568\uaed8 \uc791\uc6a9\ud558\uc5ec \ud0dc\ud48d \uadf8 \uc790\uccb4\ub9cc\uc73c\ub85c\ub3c4 \ucda9\ubd84\ud788 \uac15\ub825\ud588\ub358 \ubc14\ub78c\uc758 \ud798\uc744 \uc774\uc911 \uc0bc\uc911\uc73c\ub85c \uac15\ud654\uc2dc\ud0a8 \uac83\uc774\ub2e4.", "answer": "45 km/h", "sentence": "\uba3c\uc800 \ud55c\ubc18\ub3c4 \uc0c1\ub959 \uc2dc 45 km/h \uc815\ub3c4\uc758 \uc0c1\ub2f9\ud788 \ube68\ub790\ub358 \ud0dc\ud48d\uc758 \uc774\ub3d9\uc18d\ub3c4\ub97c \ub4e4 \uc218 \uc788\ub294\ub370, \uc774 \uc774\ub3d9\uc18d\ub3c4\uac00 \ud0dc\ud48d\uc758 \ud48d\uc18d (\uc0c1\ub959 \uc2dc \ucd5c\ub300 \ud48d\uc18d 40 m/s) \uc5d0 \ub354\ud574\uc838 \uc704\ud5d8\ubc18\uc6d0\uc5d0\uc11c\uc758 \ubc14\ub78c\uc758 \ud798\uc744 \uadf8\ub9cc\ud07c \ub354 \uac15\ud558\uac8c \ud588\ub2e4.", "paragraph_sentence": "\uc774\uc640 \uac19\uc774 \uae30\ub85d\uc801\uc778 \uac15\ud48d\uc774 \uc77c\uc5c8\ub358 \uc6d0\uc778\uc740 \ubb3c\ub860 \uac15\ub825\ud588\ub358 \ud0dc\ud48d\uc758 \uc138\ub825\uc774 \uc77c\ucc28\uc801\uc774\uc9c0\ub9cc, \ubd80\uac00\uc801\uc778 \uc694\uc778\uc73c\ub85c\uc11c \ub450 \uac00\uc9c0\ub97c \ub354 \uaf3d\uc744 \uc218 \uc788\ub2e4. \uba3c\uc800 \ud55c\ubc18\ub3c4 \uc0c1\ub959 \uc2dc 45 km/h \uc815\ub3c4\uc758 \uc0c1\ub2f9\ud788 \ube68\ub790\ub358 \ud0dc\ud48d\uc758 \uc774\ub3d9\uc18d\ub3c4\ub97c \ub4e4 \uc218 \uc788\ub294\ub370, \uc774 \uc774\ub3d9\uc18d\ub3c4\uac00 \ud0dc\ud48d\uc758 \ud48d\uc18d (\uc0c1\ub959 \uc2dc \ucd5c\ub300 \ud48d\uc18d 40 m/s) \uc5d0 \ub354\ud574\uc838 \uc704\ud5d8\ubc18\uc6d0\uc5d0\uc11c\uc758 \ubc14\ub78c\uc758 \ud798\uc744 \uadf8\ub9cc\ud07c \ub354 \uac15\ud558\uac8c \ud588\ub2e4. \ubc18\uc2dc\uacc4 \ubc29\ud5a5\uc73c\ub85c \ud68c\uc804\ud558\ub294 \ud0dc\ud48d\uc758 \ud2b9\uc131\uc0c1 \uc9c4\ud589\ubc29\ud5a5\uc758 \uc624\ub978\ucabd, \ub2e4\uc2dc \ub9d0\ud574 \uc704\ud5d8\ubc18\uc6d0\uc5d0\uc11c\ub294 \ud0dc\ud48d\uc758 \ud68c\uc804\uacfc \uc9c4\ud589\ubc29\ud5a5\uc774 \uc911\ucca9\ub418\uc5b4, \ud0dc\ud48d\uc758 \uc774\ub3d9\uc18d\ub3c4\uac00 \uadf8\ub300\ub85c \ud48d\uc18d\uc5d0 \ub354\ud574\uc9c0\uae30 \ub54c\ubb38\uc774\ub2e4. \uac8c\ub2e4\uac00 \ud0dc\ud48d \uc0c1\ub959 \uc77c\uc778 9\uc6d4 12\uc77c \uc624\ud6c4\uc758 \ud55c\ubc18\ub3c4 \uc8fc\ubcc0 \uae30\uc555\ubc30\uce58\ub294 \ub0a8\ucabd\uc73c\ub85c\ubd80\ud130 \ubd81\uc0c1\ud558\ub294 \ub9e4\uc6b0 \ub0ae\uc740 \uc911\uc2ec \uae30\uc555\uc758 \ud0dc\ud48d\uacfc \ubd81\ucabd\uc758 \uace0\uae30\uc555\uc774 \ub9c8\uc8fc\uce58\ub294 \ud615\uad6d\uc774 \ub418\uc5b4 \uc788\uc5b4, \uc774 \ubd80\uadfc\uc758 \uae30\uc555\uacbd\ub3c4\ub97c \uae09\uaca9\ud558\uac8c \ub9cc\ub4e6\uc5d0 \ub530\ub77c \uac15\ud48d\uc744 \uc720\ubc1c\uc2dc\ud0a4\ub294 \ub610 \ud558\ub098\uc758 \uc870\uac74\uc744 \ud615\uc131\ud588\ub2e4. \uc989, \uc774 \ub450\uac00\uc9c0 \uc694\uc18c\uac00 \ud568\uaed8 \uc791\uc6a9\ud558\uc5ec \ud0dc\ud48d \uadf8 \uc790\uccb4\ub9cc\uc73c\ub85c\ub3c4 \ucda9\ubd84\ud788 \uac15\ub825\ud588\ub358 \ubc14\ub78c\uc758 \ud798\uc744 \uc774\uc911 \uc0bc\uc911\uc73c\ub85c \uac15\ud654\uc2dc\ud0a8 \uac83\uc774\ub2e4.", "paragraph_answer": "\uc774\uc640 \uac19\uc774 \uae30\ub85d\uc801\uc778 \uac15\ud48d\uc774 \uc77c\uc5c8\ub358 \uc6d0\uc778\uc740 \ubb3c\ub860 \uac15\ub825\ud588\ub358 \ud0dc\ud48d\uc758 \uc138\ub825\uc774 \uc77c\ucc28\uc801\uc774\uc9c0\ub9cc, \ubd80\uac00\uc801\uc778 \uc694\uc778\uc73c\ub85c\uc11c \ub450 \uac00\uc9c0\ub97c \ub354 \uaf3d\uc744 \uc218 \uc788\ub2e4. \uba3c\uc800 \ud55c\ubc18\ub3c4 \uc0c1\ub959 \uc2dc 45 km/h \uc815\ub3c4\uc758 \uc0c1\ub2f9\ud788 \ube68\ub790\ub358 \ud0dc\ud48d\uc758 \uc774\ub3d9\uc18d\ub3c4\ub97c \ub4e4 \uc218 \uc788\ub294\ub370, \uc774 \uc774\ub3d9\uc18d\ub3c4\uac00 \ud0dc\ud48d\uc758 \ud48d\uc18d (\uc0c1\ub959 \uc2dc \ucd5c\ub300 \ud48d\uc18d 40 m/s) \uc5d0 \ub354\ud574\uc838 \uc704\ud5d8\ubc18\uc6d0\uc5d0\uc11c\uc758 \ubc14\ub78c\uc758 \ud798\uc744 \uadf8\ub9cc\ud07c \ub354 \uac15\ud558\uac8c \ud588\ub2e4. \ubc18\uc2dc\uacc4 \ubc29\ud5a5\uc73c\ub85c \ud68c\uc804\ud558\ub294 \ud0dc\ud48d\uc758 \ud2b9\uc131\uc0c1 \uc9c4\ud589\ubc29\ud5a5\uc758 \uc624\ub978\ucabd, \ub2e4\uc2dc \ub9d0\ud574 \uc704\ud5d8\ubc18\uc6d0\uc5d0\uc11c\ub294 \ud0dc\ud48d\uc758 \ud68c\uc804\uacfc \uc9c4\ud589\ubc29\ud5a5\uc774 \uc911\ucca9\ub418\uc5b4, \ud0dc\ud48d\uc758 \uc774\ub3d9\uc18d\ub3c4\uac00 \uadf8\ub300\ub85c \ud48d\uc18d\uc5d0 \ub354\ud574\uc9c0\uae30 \ub54c\ubb38\uc774\ub2e4. \uac8c\ub2e4\uac00 \ud0dc\ud48d \uc0c1\ub959 \uc77c\uc778 9\uc6d4 12\uc77c \uc624\ud6c4\uc758 \ud55c\ubc18\ub3c4 \uc8fc\ubcc0 \uae30\uc555\ubc30\uce58\ub294 \ub0a8\ucabd\uc73c\ub85c\ubd80\ud130 \ubd81\uc0c1\ud558\ub294 \ub9e4\uc6b0 \ub0ae\uc740 \uc911\uc2ec \uae30\uc555\uc758 \ud0dc\ud48d\uacfc \ubd81\ucabd\uc758 \uace0\uae30\uc555\uc774 \ub9c8\uc8fc\uce58\ub294 \ud615\uad6d\uc774 \ub418\uc5b4 \uc788\uc5b4, \uc774 \ubd80\uadfc\uc758 \uae30\uc555\uacbd\ub3c4\ub97c \uae09\uaca9\ud558\uac8c \ub9cc\ub4e6\uc5d0 \ub530\ub77c \uac15\ud48d\uc744 \uc720\ubc1c\uc2dc\ud0a4\ub294 \ub610 \ud558\ub098\uc758 \uc870\uac74\uc744 \ud615\uc131\ud588\ub2e4. \uc989, \uc774 \ub450\uac00\uc9c0 \uc694\uc18c\uac00 \ud568\uaed8 \uc791\uc6a9\ud558\uc5ec \ud0dc\ud48d \uadf8 \uc790\uccb4\ub9cc\uc73c\ub85c\ub3c4 \ucda9\ubd84\ud788 \uac15\ub825\ud588\ub358 \ubc14\ub78c\uc758 \ud798\uc744 \uc774\uc911 \uc0bc\uc911\uc73c\ub85c \uac15\ud654\uc2dc\ud0a8 \uac83\uc774\ub2e4.", "sentence_answer": "\uba3c\uc800 \ud55c\ubc18\ub3c4 \uc0c1\ub959 \uc2dc 45 km/h \uc815\ub3c4\uc758 \uc0c1\ub2f9\ud788 \ube68\ub790\ub358 \ud0dc\ud48d\uc758 \uc774\ub3d9\uc18d\ub3c4\ub97c \ub4e4 \uc218 \uc788\ub294\ub370, \uc774 \uc774\ub3d9\uc18d\ub3c4\uac00 \ud0dc\ud48d\uc758 \ud48d\uc18d (\uc0c1\ub959 \uc2dc \ucd5c\ub300 \ud48d\uc18d 40 m/s) \uc5d0 \ub354\ud574\uc838 \uc704\ud5d8\ubc18\uc6d0\uc5d0\uc11c\uc758 \ubc14\ub78c\uc758 \ud798\uc744 \uadf8\ub9cc\ud07c \ub354 \uac15\ud558\uac8c \ud588\ub2e4.", "paragraph_id": "6565385-3-0", "paragraph_question": "question: \ud0dc\ud48d \ub9e4\ubbf8\uac00 \ud55c\ubc18\ub3c4\uc5d0 \ub3c4\ucc29\ud588\uc744 \ub54c\uc758 \uc774\ub3d9\uc18d\ub3c4\ub294 \uba87 km/h \uc778\uac00?, context: \uc774\uc640 \uac19\uc774 \uae30\ub85d\uc801\uc778 \uac15\ud48d\uc774 \uc77c\uc5c8\ub358 \uc6d0\uc778\uc740 \ubb3c\ub860 \uac15\ub825\ud588\ub358 \ud0dc\ud48d\uc758 \uc138\ub825\uc774 \uc77c\ucc28\uc801\uc774\uc9c0\ub9cc, \ubd80\uac00\uc801\uc778 \uc694\uc778\uc73c\ub85c\uc11c \ub450 \uac00\uc9c0\ub97c \ub354 \uaf3d\uc744 \uc218 \uc788\ub2e4. \uba3c\uc800 \ud55c\ubc18\ub3c4 \uc0c1\ub959 \uc2dc 45 km/h \uc815\ub3c4\uc758 \uc0c1\ub2f9\ud788 \ube68\ub790\ub358 \ud0dc\ud48d\uc758 \uc774\ub3d9\uc18d\ub3c4\ub97c \ub4e4 \uc218 \uc788\ub294\ub370, \uc774 \uc774\ub3d9\uc18d\ub3c4\uac00 \ud0dc\ud48d\uc758 \ud48d\uc18d (\uc0c1\ub959 \uc2dc \ucd5c\ub300 \ud48d\uc18d 40 m/s) \uc5d0 \ub354\ud574\uc838 \uc704\ud5d8\ubc18\uc6d0\uc5d0\uc11c\uc758 \ubc14\ub78c\uc758 \ud798\uc744 \uadf8\ub9cc\ud07c \ub354 \uac15\ud558\uac8c \ud588\ub2e4. \ubc18\uc2dc\uacc4 \ubc29\ud5a5\uc73c\ub85c \ud68c\uc804\ud558\ub294 \ud0dc\ud48d\uc758 \ud2b9\uc131\uc0c1 \uc9c4\ud589\ubc29\ud5a5\uc758 \uc624\ub978\ucabd, \ub2e4\uc2dc \ub9d0\ud574 \uc704\ud5d8\ubc18\uc6d0\uc5d0\uc11c\ub294 \ud0dc\ud48d\uc758 \ud68c\uc804\uacfc \uc9c4\ud589\ubc29\ud5a5\uc774 \uc911\ucca9\ub418\uc5b4, \ud0dc\ud48d\uc758 \uc774\ub3d9\uc18d\ub3c4\uac00 \uadf8\ub300\ub85c \ud48d\uc18d\uc5d0 \ub354\ud574\uc9c0\uae30 \ub54c\ubb38\uc774\ub2e4. \uac8c\ub2e4\uac00 \ud0dc\ud48d \uc0c1\ub959 \uc77c\uc778 9\uc6d4 12\uc77c \uc624\ud6c4\uc758 \ud55c\ubc18\ub3c4 \uc8fc\ubcc0 \uae30\uc555\ubc30\uce58\ub294 \ub0a8\ucabd\uc73c\ub85c\ubd80\ud130 \ubd81\uc0c1\ud558\ub294 \ub9e4\uc6b0 \ub0ae\uc740 \uc911\uc2ec \uae30\uc555\uc758 \ud0dc\ud48d\uacfc \ubd81\ucabd\uc758 \uace0\uae30\uc555\uc774 \ub9c8\uc8fc\uce58\ub294 \ud615\uad6d\uc774 \ub418\uc5b4 \uc788\uc5b4, \uc774 \ubd80\uadfc\uc758 \uae30\uc555\uacbd\ub3c4\ub97c \uae09\uaca9\ud558\uac8c \ub9cc\ub4e6\uc5d0 \ub530\ub77c \uac15\ud48d\uc744 \uc720\ubc1c\uc2dc\ud0a4\ub294 \ub610 \ud558\ub098\uc758 \uc870\uac74\uc744 \ud615\uc131\ud588\ub2e4. \uc989, \uc774 \ub450\uac00\uc9c0 \uc694\uc18c\uac00 \ud568\uaed8 \uc791\uc6a9\ud558\uc5ec \ud0dc\ud48d \uadf8 \uc790\uccb4\ub9cc\uc73c\ub85c\ub3c4 \ucda9\ubd84\ud788 \uac15\ub825\ud588\ub358 \ubc14\ub78c\uc758 \ud798\uc744 \uc774\uc911 \uc0bc\uc911\uc73c\ub85c \uac15\ud654\uc2dc\ud0a8 \uac83\uc774\ub2e4."} {"question": "\uc5d0\ub179\uc740 \uba87\uc0b4\uae4c\uc9c0 \uc0b4\uc558\ub294\uac00?", "paragraph": "\uc5d0\ub178\uc2a4\ub294 \uad6c\uc2ed \uc138\uc5d0 \uac8c\ub09c\uc744 \ub0b3\uc558\uace0 \uac8c\ub09c\uc744 \ub0b3\uc740 \ud6c4 \ud314\ubc31\uc2ed\uc624 \ub144\uc744 \uc9c0\ub0b4\uba70 \uc790\ub140\ub4e4\uc744 \ub0b3\uc558\uc73c\uba70 \uadf8\ub294 \uad6c\ubc31\uc624 \uc138\ub97c \uc0b4\uace0 \uc8fd\uc5c8\ub354\ub77c \uac8c\ub09c\uc740 \uce60\uc2ed \uc138\uc5d0 \ub9c8\ud560\ub784\ub810\uc744 \ub0b3\uc558\uace0 \ub9c8\ud560\ub784\ub810\uc744 \ub0b3\uc740 \ud6c4 \ud314\ubc31\uc0ac\uc2ed \ub144\uc744 \uc9c0\ub0b4\uba70 \uc790\ub140\ub4e4\uc744 \ub0b3\uc558\uc73c\uba70 \uadf8\ub294 \uad6c\ubc31\uc2ed \uc138\ub97c \uc0b4\uace0 \uc8fd\uc5c8\ub354\ub77c \ub9c8\ud560\ub784\ub810\uc740 \uc721\uc2ed\uc624 \uc138\uc5d0 \uc57c\ub81b\uc744 \ub0b3\uc558\uace0 \uc57c\ub81b\uc744 \ub0b3\uc740 \ud6c4 \ud314\ubc31\uc0bc\uc2ed \ub144\uc744 \uc9c0\ub0b4\uba70 \uc790\ub140\ub97c \ub0b3\uc558\uc73c\uba70 \uadf8\ub294 \ud314\ubc31\uad6c\uc2ed\uc624 \uc138\ub97c \uc0b4\uace0 \uc8fd\uc5c8\ub354\ub77c \uc57c\ub81b\uc740 \ubc31\uc721\uc2ed\uc774 \uc138\uc5d0 \uc5d0\ub179\uc744 \ub0b3\uc558\uace0 \uc5d0\ub179\uc744 \ub0b3\uc740 \ud6c4 \ud314\ubc31 \ub144\uc744 \uc9c0\ub0b4\uba70 \uc790\ub140\ub4e4\uc744 \ub0b3\uc558\uc73c\uba70 \uadf8\ub294 \uad6c\ubc31\uc721\uc2ed\uc774 \uc138\ub97c \uc0b4\uace0 \uc8fd\uc5c8\ub354\ub77c \uc5d0\ub179\uc740 \uc721\uc2ed\uc624 \uc138\uc5d0 \ubbc0\ub450\uc140\ub77c\ub97c \ub0b3\uc558\uace0 \ubbc0\ub450\uc140\ub77c\ub97c \ub0b3\uc740 \ud6c4 \uc0bc\ubc31 \ub144\uc744 \ud558\ub098\ub2d8\uacfc \ub3d9\ud589\ud558\uba70 \uc790\ub140\ub4e4\uc744 \ub0b3\uc558\uc73c\uba70 \uadf8\ub294 \uc0bc\ubc31\uc721\uc2ed\uc624 \uc138\ub97c \uc0b4\uc558\ub354\ub77c \uc5d0\ub179\uc774 \ud558\ub098\ub2d8\uacfc \ub3d9\ud589\ud558\ub354\ub2c8 \ud558\ub098\ub2d8\uc774 \uadf8\ub97c \ub370\ub824\uac00\uc2dc\ubbc0\ub85c \uc138\uc0c1\uc5d0 \uc788\uc9c0 \uc544\ub2c8\ud558\uc600\ub354\ub77c \ubbc0\ub450\uc140\ub77c\ub294 \ubc31\ud314\uc2ed\uce60 \uc138\uc5d0 \ub77c\uba55\uc744 \ub0b3\uc558\uace0 \ub77c\uba55\uc744 \ub0b3\uc740 \ud6c4 \uce60\ubc31\ud314\uc2ed\uc774 \ub144\uc744 \uc9c0\ub0b4\uba70 \uc790\ub140\ub97c \ub0b3\uc558\uc73c\uba70 \uadf8\ub294 \uad6c\ubc31\uc721\uc2ed\uad6c \uc138\ub97c \uc0b4\uace0 \uc8fd\uc5c8\ub354\ub77c \ub77c\uba55\uc740 \ubc31\ud314\uc2ed\uc774 \uc138\uc5d0 \uc544\ub4e4\uc744 \ub0b3\uace0 \uc774\ub984\uc744 \ub178\uc544\ub77c \ud558\uc5ec \uc774\ub974\ub418 \uc5ec\ud638\uc640\uaed8\uc11c \ub545\uc744 \uc800\uc8fc\ud558\uc2dc\ubbc0\ub85c \uc218\uace0\ub86d\uac8c \uc77c\ud558\ub294 \uc6b0\ub9ac\ub97c \uc774 \uc544\ub4e4\uc774 \uc548\uc704\ud558\ub9ac\ub77c \ud558\uc600\ub354\ub77c \ub77c\uba55\uc740 \ub178\uc544\ub97c \ub0b3\uc740 \ud6c4 \uc624\ubc31\uad6c\uc2ed\uc624 \ub144\uc744 \uc9c0\ub0b4\uba70 \uc790\ub140\ub4e4\uc744 \ub0b3\uc558\uc73c\uba70", "answer": "\uc0bc\ubc31\uc721\uc2ed\uc624 \uc138", "sentence": "\uadf8\ub294 \uc0bc\ubc31\uc721\uc2ed\uc624 \uc138 \ub97c \uc0b4\uc558\ub354\ub77c \uc5d0\ub179\uc774 \ud558\ub098\ub2d8\uacfc \ub3d9\ud589\ud558\ub354\ub2c8 \ud558\ub098\ub2d8\uc774 \uadf8\ub97c \ub370\ub824\uac00\uc2dc\ubbc0\ub85c \uc138\uc0c1\uc5d0 \uc788\uc9c0 \uc544\ub2c8\ud558\uc600\ub354\ub77c \ubbc0\ub450\uc140\ub77c\ub294 \ubc31\ud314\uc2ed\uce60 \uc138\uc5d0 \ub77c\uba55\uc744 \ub0b3\uc558\uace0 \ub77c\uba55\uc744 \ub0b3\uc740 \ud6c4 \uce60\ubc31\ud314\uc2ed\uc774 \ub144\uc744 \uc9c0\ub0b4\uba70 \uc790\ub140\ub97c \ub0b3\uc558\uc73c\uba70 \uadf8\ub294 \uad6c\ubc31\uc721\uc2ed\uad6c \uc138\ub97c \uc0b4\uace0 \uc8fd\uc5c8\ub354\ub77c \ub77c\uba55\uc740 \ubc31\ud314\uc2ed\uc774 \uc138\uc5d0 \uc544\ub4e4\uc744 \ub0b3\uace0 \uc774\ub984\uc744 \ub178\uc544\ub77c \ud558\uc5ec \uc774\ub974\ub418 \uc5ec\ud638\uc640\uaed8\uc11c \ub545\uc744 \uc800\uc8fc\ud558\uc2dc\ubbc0\ub85c \uc218\uace0\ub86d\uac8c \uc77c\ud558\ub294 \uc6b0\ub9ac\ub97c \uc774 \uc544\ub4e4\uc774 \uc548\uc704\ud558\ub9ac\ub77c \ud558\uc600\ub354\ub77c \ub77c\uba55\uc740 \ub178\uc544\ub97c \ub0b3\uc740 \ud6c4 \uc624\ubc31\uad6c\uc2ed\uc624 \ub144\uc744 \uc9c0\ub0b4\uba70 \uc790\ub140\ub4e4\uc744 \ub0b3\uc558\uc73c\uba70", "paragraph_sentence": "\uc5d0\ub178\uc2a4\ub294 \uad6c\uc2ed \uc138\uc5d0 \uac8c\ub09c\uc744 \ub0b3\uc558\uace0 \uac8c\ub09c\uc744 \ub0b3\uc740 \ud6c4 \ud314\ubc31\uc2ed\uc624 \ub144\uc744 \uc9c0\ub0b4\uba70 \uc790\ub140\ub4e4\uc744 \ub0b3\uc558\uc73c\uba70 \uadf8\ub294 \uad6c\ubc31\uc624 \uc138\ub97c \uc0b4\uace0 \uc8fd\uc5c8\ub354\ub77c \uac8c\ub09c\uc740 \uce60\uc2ed \uc138\uc5d0 \ub9c8\ud560\ub784\ub810\uc744 \ub0b3\uc558\uace0 \ub9c8\ud560\ub784\ub810\uc744 \ub0b3\uc740 \ud6c4 \ud314\ubc31\uc0ac\uc2ed \ub144\uc744 \uc9c0\ub0b4\uba70 \uc790\ub140\ub4e4\uc744 \ub0b3\uc558\uc73c\uba70 \uadf8\ub294 \uad6c\ubc31\uc2ed \uc138\ub97c \uc0b4\uace0 \uc8fd\uc5c8\ub354\ub77c \ub9c8\ud560\ub784\ub810\uc740 \uc721\uc2ed\uc624 \uc138\uc5d0 \uc57c\ub81b\uc744 \ub0b3\uc558\uace0 \uc57c\ub81b\uc744 \ub0b3\uc740 \ud6c4 \ud314\ubc31\uc0bc\uc2ed \ub144\uc744 \uc9c0\ub0b4\uba70 \uc790\ub140\ub97c \ub0b3\uc558\uc73c\uba70 \uadf8\ub294 \ud314\ubc31\uad6c\uc2ed\uc624 \uc138\ub97c \uc0b4\uace0 \uc8fd\uc5c8\ub354\ub77c \uc57c\ub81b\uc740 \ubc31\uc721\uc2ed\uc774 \uc138\uc5d0 \uc5d0\ub179\uc744 \ub0b3\uc558\uace0 \uc5d0\ub179\uc744 \ub0b3\uc740 \ud6c4 \ud314\ubc31 \ub144\uc744 \uc9c0\ub0b4\uba70 \uc790\ub140\ub4e4\uc744 \ub0b3\uc558\uc73c\uba70 \uadf8\ub294 \uad6c\ubc31\uc721\uc2ed\uc774 \uc138\ub97c \uc0b4\uace0 \uc8fd\uc5c8\ub354\ub77c \uc5d0\ub179\uc740 \uc721\uc2ed\uc624 \uc138\uc5d0 \ubbc0\ub450\uc140\ub77c\ub97c \ub0b3\uc558\uace0 \ubbc0\ub450\uc140\ub77c\ub97c \ub0b3\uc740 \ud6c4 \uc0bc\ubc31 \ub144\uc744 \ud558\ub098\ub2d8\uacfc \ub3d9\ud589\ud558\uba70 \uc790\ub140\ub4e4\uc744 \ub0b3\uc558\uc73c\uba70 \uadf8\ub294 \uc0bc\ubc31\uc721\uc2ed\uc624 \uc138 \ub97c \uc0b4\uc558\ub354\ub77c \uc5d0\ub179\uc774 \ud558\ub098\ub2d8\uacfc \ub3d9\ud589\ud558\ub354\ub2c8 \ud558\ub098\ub2d8\uc774 \uadf8\ub97c \ub370\ub824\uac00\uc2dc\ubbc0\ub85c \uc138\uc0c1\uc5d0 \uc788\uc9c0 \uc544\ub2c8\ud558\uc600\ub354\ub77c \ubbc0\ub450\uc140\ub77c\ub294 \ubc31\ud314\uc2ed\uce60 \uc138\uc5d0 \ub77c\uba55\uc744 \ub0b3\uc558\uace0 \ub77c\uba55\uc744 \ub0b3\uc740 \ud6c4 \uce60\ubc31\ud314\uc2ed\uc774 \ub144\uc744 \uc9c0\ub0b4\uba70 \uc790\ub140\ub97c \ub0b3\uc558\uc73c\uba70 \uadf8\ub294 \uad6c\ubc31\uc721\uc2ed\uad6c \uc138\ub97c \uc0b4\uace0 \uc8fd\uc5c8\ub354\ub77c \ub77c\uba55\uc740 \ubc31\ud314\uc2ed\uc774 \uc138\uc5d0 \uc544\ub4e4\uc744 \ub0b3\uace0 \uc774\ub984\uc744 \ub178\uc544\ub77c \ud558\uc5ec \uc774\ub974\ub418 \uc5ec\ud638\uc640\uaed8\uc11c \ub545\uc744 \uc800\uc8fc\ud558\uc2dc\ubbc0\ub85c \uc218\uace0\ub86d\uac8c \uc77c\ud558\ub294 \uc6b0\ub9ac\ub97c \uc774 \uc544\ub4e4\uc774 \uc548\uc704\ud558\ub9ac\ub77c \ud558\uc600\ub354\ub77c \ub77c\uba55\uc740 \ub178\uc544\ub97c \ub0b3\uc740 \ud6c4 \uc624\ubc31\uad6c\uc2ed\uc624 \ub144\uc744 \uc9c0\ub0b4\uba70 \uc790\ub140\ub4e4\uc744 \ub0b3\uc558\uc73c\uba70 ", "paragraph_answer": "\uc5d0\ub178\uc2a4\ub294 \uad6c\uc2ed \uc138\uc5d0 \uac8c\ub09c\uc744 \ub0b3\uc558\uace0 \uac8c\ub09c\uc744 \ub0b3\uc740 \ud6c4 \ud314\ubc31\uc2ed\uc624 \ub144\uc744 \uc9c0\ub0b4\uba70 \uc790\ub140\ub4e4\uc744 \ub0b3\uc558\uc73c\uba70 \uadf8\ub294 \uad6c\ubc31\uc624 \uc138\ub97c \uc0b4\uace0 \uc8fd\uc5c8\ub354\ub77c \uac8c\ub09c\uc740 \uce60\uc2ed \uc138\uc5d0 \ub9c8\ud560\ub784\ub810\uc744 \ub0b3\uc558\uace0 \ub9c8\ud560\ub784\ub810\uc744 \ub0b3\uc740 \ud6c4 \ud314\ubc31\uc0ac\uc2ed \ub144\uc744 \uc9c0\ub0b4\uba70 \uc790\ub140\ub4e4\uc744 \ub0b3\uc558\uc73c\uba70 \uadf8\ub294 \uad6c\ubc31\uc2ed \uc138\ub97c \uc0b4\uace0 \uc8fd\uc5c8\ub354\ub77c \ub9c8\ud560\ub784\ub810\uc740 \uc721\uc2ed\uc624 \uc138\uc5d0 \uc57c\ub81b\uc744 \ub0b3\uc558\uace0 \uc57c\ub81b\uc744 \ub0b3\uc740 \ud6c4 \ud314\ubc31\uc0bc\uc2ed \ub144\uc744 \uc9c0\ub0b4\uba70 \uc790\ub140\ub97c \ub0b3\uc558\uc73c\uba70 \uadf8\ub294 \ud314\ubc31\uad6c\uc2ed\uc624 \uc138\ub97c \uc0b4\uace0 \uc8fd\uc5c8\ub354\ub77c \uc57c\ub81b\uc740 \ubc31\uc721\uc2ed\uc774 \uc138\uc5d0 \uc5d0\ub179\uc744 \ub0b3\uc558\uace0 \uc5d0\ub179\uc744 \ub0b3\uc740 \ud6c4 \ud314\ubc31 \ub144\uc744 \uc9c0\ub0b4\uba70 \uc790\ub140\ub4e4\uc744 \ub0b3\uc558\uc73c\uba70 \uadf8\ub294 \uad6c\ubc31\uc721\uc2ed\uc774 \uc138\ub97c \uc0b4\uace0 \uc8fd\uc5c8\ub354\ub77c \uc5d0\ub179\uc740 \uc721\uc2ed\uc624 \uc138\uc5d0 \ubbc0\ub450\uc140\ub77c\ub97c \ub0b3\uc558\uace0 \ubbc0\ub450\uc140\ub77c\ub97c \ub0b3\uc740 \ud6c4 \uc0bc\ubc31 \ub144\uc744 \ud558\ub098\ub2d8\uacfc \ub3d9\ud589\ud558\uba70 \uc790\ub140\ub4e4\uc744 \ub0b3\uc558\uc73c\uba70 \uadf8\ub294 \uc0bc\ubc31\uc721\uc2ed\uc624 \uc138 \ub97c \uc0b4\uc558\ub354\ub77c \uc5d0\ub179\uc774 \ud558\ub098\ub2d8\uacfc \ub3d9\ud589\ud558\ub354\ub2c8 \ud558\ub098\ub2d8\uc774 \uadf8\ub97c \ub370\ub824\uac00\uc2dc\ubbc0\ub85c \uc138\uc0c1\uc5d0 \uc788\uc9c0 \uc544\ub2c8\ud558\uc600\ub354\ub77c \ubbc0\ub450\uc140\ub77c\ub294 \ubc31\ud314\uc2ed\uce60 \uc138\uc5d0 \ub77c\uba55\uc744 \ub0b3\uc558\uace0 \ub77c\uba55\uc744 \ub0b3\uc740 \ud6c4 \uce60\ubc31\ud314\uc2ed\uc774 \ub144\uc744 \uc9c0\ub0b4\uba70 \uc790\ub140\ub97c \ub0b3\uc558\uc73c\uba70 \uadf8\ub294 \uad6c\ubc31\uc721\uc2ed\uad6c \uc138\ub97c \uc0b4\uace0 \uc8fd\uc5c8\ub354\ub77c \ub77c\uba55\uc740 \ubc31\ud314\uc2ed\uc774 \uc138\uc5d0 \uc544\ub4e4\uc744 \ub0b3\uace0 \uc774\ub984\uc744 \ub178\uc544\ub77c \ud558\uc5ec \uc774\ub974\ub418 \uc5ec\ud638\uc640\uaed8\uc11c \ub545\uc744 \uc800\uc8fc\ud558\uc2dc\ubbc0\ub85c \uc218\uace0\ub86d\uac8c \uc77c\ud558\ub294 \uc6b0\ub9ac\ub97c \uc774 \uc544\ub4e4\uc774 \uc548\uc704\ud558\ub9ac\ub77c \ud558\uc600\ub354\ub77c \ub77c\uba55\uc740 \ub178\uc544\ub97c \ub0b3\uc740 \ud6c4 \uc624\ubc31\uad6c\uc2ed\uc624 \ub144\uc744 \uc9c0\ub0b4\uba70 \uc790\ub140\ub4e4\uc744 \ub0b3\uc558\uc73c\uba70", "sentence_answer": "\uadf8\ub294 \uc0bc\ubc31\uc721\uc2ed\uc624 \uc138 \ub97c \uc0b4\uc558\ub354\ub77c \uc5d0\ub179\uc774 \ud558\ub098\ub2d8\uacfc \ub3d9\ud589\ud558\ub354\ub2c8 \ud558\ub098\ub2d8\uc774 \uadf8\ub97c \ub370\ub824\uac00\uc2dc\ubbc0\ub85c \uc138\uc0c1\uc5d0 \uc788\uc9c0 \uc544\ub2c8\ud558\uc600\ub354\ub77c \ubbc0\ub450\uc140\ub77c\ub294 \ubc31\ud314\uc2ed\uce60 \uc138\uc5d0 \ub77c\uba55\uc744 \ub0b3\uc558\uace0 \ub77c\uba55\uc744 \ub0b3\uc740 \ud6c4 \uce60\ubc31\ud314\uc2ed\uc774 \ub144\uc744 \uc9c0\ub0b4\uba70 \uc790\ub140\ub97c \ub0b3\uc558\uc73c\uba70 \uadf8\ub294 \uad6c\ubc31\uc721\uc2ed\uad6c \uc138\ub97c \uc0b4\uace0 \uc8fd\uc5c8\ub354\ub77c \ub77c\uba55\uc740 \ubc31\ud314\uc2ed\uc774 \uc138\uc5d0 \uc544\ub4e4\uc744 \ub0b3\uace0 \uc774\ub984\uc744 \ub178\uc544\ub77c \ud558\uc5ec \uc774\ub974\ub418 \uc5ec\ud638\uc640\uaed8\uc11c \ub545\uc744 \uc800\uc8fc\ud558\uc2dc\ubbc0\ub85c \uc218\uace0\ub86d\uac8c \uc77c\ud558\ub294 \uc6b0\ub9ac\ub97c \uc774 \uc544\ub4e4\uc774 \uc548\uc704\ud558\ub9ac\ub77c \ud558\uc600\ub354\ub77c \ub77c\uba55\uc740 \ub178\uc544\ub97c \ub0b3\uc740 \ud6c4 \uc624\ubc31\uad6c\uc2ed\uc624 \ub144\uc744 \uc9c0\ub0b4\uba70 \uc790\ub140\ub4e4\uc744 \ub0b3\uc558\uc73c\uba70", "paragraph_id": "6484097-0-1", "paragraph_question": "question: \uc5d0\ub179\uc740 \uba87\uc0b4\uae4c\uc9c0 \uc0b4\uc558\ub294\uac00?, context: \uc5d0\ub178\uc2a4\ub294 \uad6c\uc2ed \uc138\uc5d0 \uac8c\ub09c\uc744 \ub0b3\uc558\uace0 \uac8c\ub09c\uc744 \ub0b3\uc740 \ud6c4 \ud314\ubc31\uc2ed\uc624 \ub144\uc744 \uc9c0\ub0b4\uba70 \uc790\ub140\ub4e4\uc744 \ub0b3\uc558\uc73c\uba70 \uadf8\ub294 \uad6c\ubc31\uc624 \uc138\ub97c \uc0b4\uace0 \uc8fd\uc5c8\ub354\ub77c \uac8c\ub09c\uc740 \uce60\uc2ed \uc138\uc5d0 \ub9c8\ud560\ub784\ub810\uc744 \ub0b3\uc558\uace0 \ub9c8\ud560\ub784\ub810\uc744 \ub0b3\uc740 \ud6c4 \ud314\ubc31\uc0ac\uc2ed \ub144\uc744 \uc9c0\ub0b4\uba70 \uc790\ub140\ub4e4\uc744 \ub0b3\uc558\uc73c\uba70 \uadf8\ub294 \uad6c\ubc31\uc2ed \uc138\ub97c \uc0b4\uace0 \uc8fd\uc5c8\ub354\ub77c \ub9c8\ud560\ub784\ub810\uc740 \uc721\uc2ed\uc624 \uc138\uc5d0 \uc57c\ub81b\uc744 \ub0b3\uc558\uace0 \uc57c\ub81b\uc744 \ub0b3\uc740 \ud6c4 \ud314\ubc31\uc0bc\uc2ed \ub144\uc744 \uc9c0\ub0b4\uba70 \uc790\ub140\ub97c \ub0b3\uc558\uc73c\uba70 \uadf8\ub294 \ud314\ubc31\uad6c\uc2ed\uc624 \uc138\ub97c \uc0b4\uace0 \uc8fd\uc5c8\ub354\ub77c \uc57c\ub81b\uc740 \ubc31\uc721\uc2ed\uc774 \uc138\uc5d0 \uc5d0\ub179\uc744 \ub0b3\uc558\uace0 \uc5d0\ub179\uc744 \ub0b3\uc740 \ud6c4 \ud314\ubc31 \ub144\uc744 \uc9c0\ub0b4\uba70 \uc790\ub140\ub4e4\uc744 \ub0b3\uc558\uc73c\uba70 \uadf8\ub294 \uad6c\ubc31\uc721\uc2ed\uc774 \uc138\ub97c \uc0b4\uace0 \uc8fd\uc5c8\ub354\ub77c \uc5d0\ub179\uc740 \uc721\uc2ed\uc624 \uc138\uc5d0 \ubbc0\ub450\uc140\ub77c\ub97c \ub0b3\uc558\uace0 \ubbc0\ub450\uc140\ub77c\ub97c \ub0b3\uc740 \ud6c4 \uc0bc\ubc31 \ub144\uc744 \ud558\ub098\ub2d8\uacfc \ub3d9\ud589\ud558\uba70 \uc790\ub140\ub4e4\uc744 \ub0b3\uc558\uc73c\uba70 \uadf8\ub294 \uc0bc\ubc31\uc721\uc2ed\uc624 \uc138\ub97c \uc0b4\uc558\ub354\ub77c \uc5d0\ub179\uc774 \ud558\ub098\ub2d8\uacfc \ub3d9\ud589\ud558\ub354\ub2c8 \ud558\ub098\ub2d8\uc774 \uadf8\ub97c \ub370\ub824\uac00\uc2dc\ubbc0\ub85c \uc138\uc0c1\uc5d0 \uc788\uc9c0 \uc544\ub2c8\ud558\uc600\ub354\ub77c \ubbc0\ub450\uc140\ub77c\ub294 \ubc31\ud314\uc2ed\uce60 \uc138\uc5d0 \ub77c\uba55\uc744 \ub0b3\uc558\uace0 \ub77c\uba55\uc744 \ub0b3\uc740 \ud6c4 \uce60\ubc31\ud314\uc2ed\uc774 \ub144\uc744 \uc9c0\ub0b4\uba70 \uc790\ub140\ub97c \ub0b3\uc558\uc73c\uba70 \uadf8\ub294 \uad6c\ubc31\uc721\uc2ed\uad6c \uc138\ub97c \uc0b4\uace0 \uc8fd\uc5c8\ub354\ub77c \ub77c\uba55\uc740 \ubc31\ud314\uc2ed\uc774 \uc138\uc5d0 \uc544\ub4e4\uc744 \ub0b3\uace0 \uc774\ub984\uc744 \ub178\uc544\ub77c \ud558\uc5ec \uc774\ub974\ub418 \uc5ec\ud638\uc640\uaed8\uc11c \ub545\uc744 \uc800\uc8fc\ud558\uc2dc\ubbc0\ub85c \uc218\uace0\ub86d\uac8c \uc77c\ud558\ub294 \uc6b0\ub9ac\ub97c \uc774 \uc544\ub4e4\uc774 \uc548\uc704\ud558\ub9ac\ub77c \ud558\uc600\ub354\ub77c \ub77c\uba55\uc740 \ub178\uc544\ub97c \ub0b3\uc740 \ud6c4 \uc624\ubc31\uad6c\uc2ed\uc624 \ub144\uc744 \uc9c0\ub0b4\uba70 \uc790\ub140\ub4e4\uc744 \ub0b3\uc558\uc73c\uba70"} {"question": "\ub9c8\ub4dc\ud5e4\uc2dc\uc778\uc774 \uc0ac\ud68c\ubb38\ud654\uc801\uc73c\ub85c \uae4a\uc740 \uc720\ub300\uad00\uacc4\ub97c \uac16\uace0 \uc788\ub358 \uc8fc\ub294 \uc5b4\ub5a4 \uc8fc \uc778\uac00?", "paragraph": "1950\ub144 \ub9fa\uc5b4\uc9c4 \ub124\ud314-\uc778\ub3c4 \ud3c9\ud654\uc640 \uce5c\uad50 \uc870\uc57d\uc5d0 \ub530\ub77c \uc591\uad6d \uad6d\ubbfc\uc740 \ub124\ud314\uacfc \uc778\ub3c4 \uc0ac\uc774\ub97c \uc5ec\uad8c \uc5c6\uc774 \ub118\ub098\ub4e4\uace0 \uc77c\uc744 \ud560 \uc218 \uc788\uc5c8\uc73c\uba70, \uc774\uc5d0 \ub530\ub77c\uc11c \ub124\ud314-\uc778\ub3c4 \uc0ac\uc774 \uc0ac\ub78c\ub4e4\uc758 \uc6c0\uc9c1\uc784\uc744 \ucd94\uc801\ud558\uae30 \uc5b4\ub824\uc6e0\ub2e4. \uc774\ub7ec\ud55c \uc778\ub3c4 \ub0b4\uc758 \uc7a0\uc2dd \ud604\uc0c1\uc740 \ub124\ud314-\uc778\ub3c4 \uad00\uacc4\uc758 \uac00\uc7a5 \ud070 \ubb38\uc81c\uc600\ub2e4. \ub9c8\ub4dc\ud5e4\uc2dc\uc778\uc740 \uc778\uadfc\uc758 \uc778\ub3c4 \ube44\ud558\ub974 \uc8fc \ubc0f \uc6b0\ud0c0\ub974\ud504\ub77c\ub370\uc2dc \uc8fc\uc640 \uae34\ubc00\ud55c \uc0ac\ud68c\ubb38\ud654\uc801 \uc720\ub300\uac10\uc744 \uc774\ub8e8\uace0 \uc788\uc5c8\ub2e4. \uc778\ub3c4\ub294 \uc774\ub7ec\ud55c \ud3ed\ub825\uc2dc\uc704 \uc0ac\ud0dc\uc5d0 \ub300\ud574 \uc6b0\ub824\ub97c \ud45c\ud558\uace0 \ub124\ud314\uc5d0\uac8c \ub9c8\ub4dc\ud5e4\uc2dc\uc778\ub4e4\uc758 \uc785\uc7a5\uc744 \uace0\ub824\ud558\ub824 \uc904 \uac83\uc744 \ub2f9\ubd80\ud558\uc600\ub2e4. \ub124\ud314 \uc815\ubd80\uc640 \ubbf8\ub514\uc5b4\uc5d0\uc11c\ub294 \ub9c8\ub4dc\ud5e4\uc2dc\uc778\uc758 \uc6b4\ub3d9\uc744 \uc778\ub3c4 \uce68\uc785\uc790\ub4e4\uc774 \uc77c\uc73c\ud0a8 \uac83\uc774\ub77c\uace0 \ubb18\uc0ac\ud588\uace0, \uc5ec\uae30\uc5d0\ub2e4 \ub124\ud314\uc778 \ub2e4\uc218\uc778 \uc2dc\ud0b4 \uc8fc\uc758 \uc0ac\ub840\ub97c \ub4e4\uc5b4 \ub124\ud314 \uc804\uccb4 \ub610\ub294 \uc77c\ubd80 \uc9c0\uc5ed\uc744 \uc778\ub3c4\uac00 \ud761\uc218\ud569\ubcd1\ud560\ub824\ub294 \uc74c\ubaa8\ub77c\uace0 \uc0dd\uac01\ud558\ub294 \uc8fc\uc7a5\uc774 \ud798\uc744 \uc5bb\uc5c8\ub2e4.", "answer": "\uc778\ub3c4 \ube44\ud558\ub974 \uc8fc \ubc0f \uc6b0\ud0c0\ub974\ud504\ub77c\ub370\uc2dc \uc8fc", "sentence": "\ub9c8\ub4dc\ud5e4\uc2dc\uc778\uc740 \uc778\uadfc\uc758 \uc778\ub3c4 \ube44\ud558\ub974 \uc8fc \ubc0f \uc6b0\ud0c0\ub974\ud504\ub77c\ub370\uc2dc \uc8fc \uc640 \uae34\ubc00\ud55c \uc0ac\ud68c\ubb38\ud654\uc801 \uc720\ub300\uac10\uc744 \uc774\ub8e8\uace0 \uc788\uc5c8\ub2e4.", "paragraph_sentence": "1950\ub144 \ub9fa\uc5b4\uc9c4 \ub124\ud314-\uc778\ub3c4 \ud3c9\ud654\uc640 \uce5c\uad50 \uc870\uc57d\uc5d0 \ub530\ub77c \uc591\uad6d \uad6d\ubbfc\uc740 \ub124\ud314\uacfc \uc778\ub3c4 \uc0ac\uc774\ub97c \uc5ec\uad8c \uc5c6\uc774 \ub118\ub098\ub4e4\uace0 \uc77c\uc744 \ud560 \uc218 \uc788\uc5c8\uc73c\uba70, \uc774\uc5d0 \ub530\ub77c\uc11c \ub124\ud314-\uc778\ub3c4 \uc0ac\uc774 \uc0ac\ub78c\ub4e4\uc758 \uc6c0\uc9c1\uc784\uc744 \ucd94\uc801\ud558\uae30 \uc5b4\ub824\uc6e0\ub2e4. \uc774\ub7ec\ud55c \uc778\ub3c4 \ub0b4\uc758 \uc7a0\uc2dd \ud604\uc0c1\uc740 \ub124\ud314-\uc778\ub3c4 \uad00\uacc4\uc758 \uac00\uc7a5 \ud070 \ubb38\uc81c\uc600\ub2e4. \ub9c8\ub4dc\ud5e4\uc2dc\uc778\uc740 \uc778\uadfc\uc758 \uc778\ub3c4 \ube44\ud558\ub974 \uc8fc \ubc0f \uc6b0\ud0c0\ub974\ud504\ub77c\ub370\uc2dc \uc8fc \uc640 \uae34\ubc00\ud55c \uc0ac\ud68c\ubb38\ud654\uc801 \uc720\ub300\uac10\uc744 \uc774\ub8e8\uace0 \uc788\uc5c8\ub2e4. \uc778\ub3c4\ub294 \uc774\ub7ec\ud55c \ud3ed\ub825\uc2dc\uc704 \uc0ac\ud0dc\uc5d0 \ub300\ud574 \uc6b0\ub824\ub97c \ud45c\ud558\uace0 \ub124\ud314\uc5d0\uac8c \ub9c8\ub4dc\ud5e4\uc2dc\uc778\ub4e4\uc758 \uc785\uc7a5\uc744 \uace0\ub824\ud558\ub824 \uc904 \uac83\uc744 \ub2f9\ubd80\ud558\uc600\ub2e4. \ub124\ud314 \uc815\ubd80\uc640 \ubbf8\ub514\uc5b4\uc5d0\uc11c\ub294 \ub9c8\ub4dc\ud5e4\uc2dc\uc778\uc758 \uc6b4\ub3d9\uc744 \uc778\ub3c4 \uce68\uc785\uc790\ub4e4\uc774 \uc77c\uc73c\ud0a8 \uac83\uc774\ub77c\uace0 \ubb18\uc0ac\ud588\uace0, \uc5ec\uae30\uc5d0\ub2e4 \ub124\ud314\uc778 \ub2e4\uc218\uc778 \uc2dc\ud0b4 \uc8fc\uc758 \uc0ac\ub840\ub97c \ub4e4\uc5b4 \ub124\ud314 \uc804\uccb4 \ub610\ub294 \uc77c\ubd80 \uc9c0\uc5ed\uc744 \uc778\ub3c4\uac00 \ud761\uc218\ud569\ubcd1\ud560\ub824\ub294 \uc74c\ubaa8\ub77c\uace0 \uc0dd\uac01\ud558\ub294 \uc8fc\uc7a5\uc774 \ud798\uc744 \uc5bb\uc5c8\ub2e4.", "paragraph_answer": "1950\ub144 \ub9fa\uc5b4\uc9c4 \ub124\ud314-\uc778\ub3c4 \ud3c9\ud654\uc640 \uce5c\uad50 \uc870\uc57d\uc5d0 \ub530\ub77c \uc591\uad6d \uad6d\ubbfc\uc740 \ub124\ud314\uacfc \uc778\ub3c4 \uc0ac\uc774\ub97c \uc5ec\uad8c \uc5c6\uc774 \ub118\ub098\ub4e4\uace0 \uc77c\uc744 \ud560 \uc218 \uc788\uc5c8\uc73c\uba70, \uc774\uc5d0 \ub530\ub77c\uc11c \ub124\ud314-\uc778\ub3c4 \uc0ac\uc774 \uc0ac\ub78c\ub4e4\uc758 \uc6c0\uc9c1\uc784\uc744 \ucd94\uc801\ud558\uae30 \uc5b4\ub824\uc6e0\ub2e4. \uc774\ub7ec\ud55c \uc778\ub3c4 \ub0b4\uc758 \uc7a0\uc2dd \ud604\uc0c1\uc740 \ub124\ud314-\uc778\ub3c4 \uad00\uacc4\uc758 \uac00\uc7a5 \ud070 \ubb38\uc81c\uc600\ub2e4. \ub9c8\ub4dc\ud5e4\uc2dc\uc778\uc740 \uc778\uadfc\uc758 \uc778\ub3c4 \ube44\ud558\ub974 \uc8fc \ubc0f \uc6b0\ud0c0\ub974\ud504\ub77c\ub370\uc2dc \uc8fc \uc640 \uae34\ubc00\ud55c \uc0ac\ud68c\ubb38\ud654\uc801 \uc720\ub300\uac10\uc744 \uc774\ub8e8\uace0 \uc788\uc5c8\ub2e4. \uc778\ub3c4\ub294 \uc774\ub7ec\ud55c \ud3ed\ub825\uc2dc\uc704 \uc0ac\ud0dc\uc5d0 \ub300\ud574 \uc6b0\ub824\ub97c \ud45c\ud558\uace0 \ub124\ud314\uc5d0\uac8c \ub9c8\ub4dc\ud5e4\uc2dc\uc778\ub4e4\uc758 \uc785\uc7a5\uc744 \uace0\ub824\ud558\ub824 \uc904 \uac83\uc744 \ub2f9\ubd80\ud558\uc600\ub2e4. \ub124\ud314 \uc815\ubd80\uc640 \ubbf8\ub514\uc5b4\uc5d0\uc11c\ub294 \ub9c8\ub4dc\ud5e4\uc2dc\uc778\uc758 \uc6b4\ub3d9\uc744 \uc778\ub3c4 \uce68\uc785\uc790\ub4e4\uc774 \uc77c\uc73c\ud0a8 \uac83\uc774\ub77c\uace0 \ubb18\uc0ac\ud588\uace0, \uc5ec\uae30\uc5d0\ub2e4 \ub124\ud314\uc778 \ub2e4\uc218\uc778 \uc2dc\ud0b4 \uc8fc\uc758 \uc0ac\ub840\ub97c \ub4e4\uc5b4 \ub124\ud314 \uc804\uccb4 \ub610\ub294 \uc77c\ubd80 \uc9c0\uc5ed\uc744 \uc778\ub3c4\uac00 \ud761\uc218\ud569\ubcd1\ud560\ub824\ub294 \uc74c\ubaa8\ub77c\uace0 \uc0dd\uac01\ud558\ub294 \uc8fc\uc7a5\uc774 \ud798\uc744 \uc5bb\uc5c8\ub2e4.", "sentence_answer": "\ub9c8\ub4dc\ud5e4\uc2dc\uc778\uc740 \uc778\uadfc\uc758 \uc778\ub3c4 \ube44\ud558\ub974 \uc8fc \ubc0f \uc6b0\ud0c0\ub974\ud504\ub77c\ub370\uc2dc \uc8fc \uc640 \uae34\ubc00\ud55c \uc0ac\ud68c\ubb38\ud654\uc801 \uc720\ub300\uac10\uc744 \uc774\ub8e8\uace0 \uc788\uc5c8\ub2e4.", "paragraph_id": "5780897-3-1", "paragraph_question": "question: \ub9c8\ub4dc\ud5e4\uc2dc\uc778\uc774 \uc0ac\ud68c\ubb38\ud654\uc801\uc73c\ub85c \uae4a\uc740 \uc720\ub300\uad00\uacc4\ub97c \uac16\uace0 \uc788\ub358 \uc8fc\ub294 \uc5b4\ub5a4 \uc8fc \uc778\uac00?, context: 1950\ub144 \ub9fa\uc5b4\uc9c4 \ub124\ud314-\uc778\ub3c4 \ud3c9\ud654\uc640 \uce5c\uad50 \uc870\uc57d\uc5d0 \ub530\ub77c \uc591\uad6d \uad6d\ubbfc\uc740 \ub124\ud314\uacfc \uc778\ub3c4 \uc0ac\uc774\ub97c \uc5ec\uad8c \uc5c6\uc774 \ub118\ub098\ub4e4\uace0 \uc77c\uc744 \ud560 \uc218 \uc788\uc5c8\uc73c\uba70, \uc774\uc5d0 \ub530\ub77c\uc11c \ub124\ud314-\uc778\ub3c4 \uc0ac\uc774 \uc0ac\ub78c\ub4e4\uc758 \uc6c0\uc9c1\uc784\uc744 \ucd94\uc801\ud558\uae30 \uc5b4\ub824\uc6e0\ub2e4. \uc774\ub7ec\ud55c \uc778\ub3c4 \ub0b4\uc758 \uc7a0\uc2dd \ud604\uc0c1\uc740 \ub124\ud314-\uc778\ub3c4 \uad00\uacc4\uc758 \uac00\uc7a5 \ud070 \ubb38\uc81c\uc600\ub2e4. \ub9c8\ub4dc\ud5e4\uc2dc\uc778\uc740 \uc778\uadfc\uc758 \uc778\ub3c4 \ube44\ud558\ub974 \uc8fc \ubc0f \uc6b0\ud0c0\ub974\ud504\ub77c\ub370\uc2dc \uc8fc\uc640 \uae34\ubc00\ud55c \uc0ac\ud68c\ubb38\ud654\uc801 \uc720\ub300\uac10\uc744 \uc774\ub8e8\uace0 \uc788\uc5c8\ub2e4. \uc778\ub3c4\ub294 \uc774\ub7ec\ud55c \ud3ed\ub825\uc2dc\uc704 \uc0ac\ud0dc\uc5d0 \ub300\ud574 \uc6b0\ub824\ub97c \ud45c\ud558\uace0 \ub124\ud314\uc5d0\uac8c \ub9c8\ub4dc\ud5e4\uc2dc\uc778\ub4e4\uc758 \uc785\uc7a5\uc744 \uace0\ub824\ud558\ub824 \uc904 \uac83\uc744 \ub2f9\ubd80\ud558\uc600\ub2e4. \ub124\ud314 \uc815\ubd80\uc640 \ubbf8\ub514\uc5b4\uc5d0\uc11c\ub294 \ub9c8\ub4dc\ud5e4\uc2dc\uc778\uc758 \uc6b4\ub3d9\uc744 \uc778\ub3c4 \uce68\uc785\uc790\ub4e4\uc774 \uc77c\uc73c\ud0a8 \uac83\uc774\ub77c\uace0 \ubb18\uc0ac\ud588\uace0, \uc5ec\uae30\uc5d0\ub2e4 \ub124\ud314\uc778 \ub2e4\uc218\uc778 \uc2dc\ud0b4 \uc8fc\uc758 \uc0ac\ub840\ub97c \ub4e4\uc5b4 \ub124\ud314 \uc804\uccb4 \ub610\ub294 \uc77c\ubd80 \uc9c0\uc5ed\uc744 \uc778\ub3c4\uac00 \ud761\uc218\ud569\ubcd1\ud560\ub824\ub294 \uc74c\ubaa8\ub77c\uace0 \uc0dd\uac01\ud558\ub294 \uc8fc\uc7a5\uc774 \ud798\uc744 \uc5bb\uc5c8\ub2e4."} {"question": "\uce7c\ub9ac\uc2a4\ud1a0\uc758 \ud070 \ucda9\ub3cc\uad6c\ub294 \ubb34\uc5c7\ucc98\ub7fc \uac00\uc6b4\ub370 \ud070 \ubd09\uc6b0\ub9ac\uac00 \uc544\ub2cc \uad6c\ub369\uc774\uac00 \uc788\ub098\uc694?", "paragraph": "\ucda9\ub3cc\uad6c \uc9c0\ub984\uc758 \ubc94\uc704\ub294 0.1 km(\uc0ac\uc9c4 \ud574\uc0c1\ub3c4\ub85c \uacb0\uc815\ub41c \ud55c\uacc4)\uc5d0\uc11c 100 km \uc815\ub3c4\uc774\uace0, \uc774 \uac12\uc740 \ub2e4\ud658 \ucda9\ub3cc\uad6c\ub294 \uc138\uc9c0 \uc54a\uc740 \uac12\uc774\ub2e4. \uc9c0\ub984\uc774 5 km\uac00 \ub418\uc9c0 \uc54a\ub294 \uc791\uc740 \ucda9\ub3cc\uad6c\ub4e4\uc740 \ub2e8\uc21c\ud558\uac8c, \uc6b0\ubb35\ud558\uac70\ub098 \ubc14\ub2e5\uc774 \ud3c9\ud3c9\ud55c \ubaa8\uc591\uc744 \uac00\uc9c0\uace0 \uc788\ub2e4. 5~40 km \uc815\ub3c4\uc758 \ucda9\ub3cc\uad6c\ub4e4\uc740 \ubcf4\ud1b5 \uac00\uc6b4\ub370\uc5d0 \ubd09\uc6b0\ub9ac\uac00 \uc874\uc7ac\ud55c\ub2e4. 25~100 km \uc815\ub3c4\uc758 \ud070 \ucda9\ub3cc\uad6c\ub4e4\uc740, \ud2f4\ub4dc\ub974\ucc98\ub7fc \uac00\uc6b4\ub370\uc5d0 \ubd09\uc6b0\ub9ac \ub300\uc2e0 \uad6c\ub369\uc774\uac00 \uc874\uc7ac\ud55c\ub2e4. \uac00\uc6b4\ub370\uc5d0 \ubd09\uc6b0\ub9ac\ub97c \uac00\uc9c0\uace0 \uc788\ub294 \uac00\uc7a5 \ud070 \ucda9\ub3cc\uad6c\uc778, \uc9c0\ub984 60 km\uc9dc\ub9ac \ucda9\ub3cc\uad6c\ub4e4\uc740 \ucda9\ub3cc \ud6c4 \uc911\uc2ec\ubd80\uc5d0\uc11c\uc758 \uad6c\uc870\uc801 \uc735\uae30\ub85c \uc778\ud55c \uac83\uc73c\ub85c \ucd94\uce21\ub41c\ub2e4. \uc608\uc2dc\ub85c \uc544\uc2a4\uac00\ub974\ub4dc\ub098 \ud558\ub974\uac00 \uc788\ub2e4. \ub9e4\uc6b0 \ud06c\uace0(\uc9c0\ub984 100 km \uc774\uc0c1), \ubc1d\uc740 \ucda9\ub3cc\uad6c\ub4e4 \uc911 \uc77c\ubd80\ub294 \uc774\ub840\uc801\uc73c\ub85c \ub3d4 \ubaa8\uc591\uc758 \uc9c0\uc9c8 \uad6c\uc870\ub97c \ubcf4\uc5ec\uc900\ub2e4. \uc774\ub7ec\ud55c \ud604\uc0c1\uc740 \ud754\ud558\uc9c0 \uc54a\uace0, \ub85c\ud508\ucc98\ub7fc \ub2e4\ud658 \ucda9\ub3cc\uad6c\ub85c \ubcc0\ud654\ud558\ub294 \uc9c0\ud615\uc77c \uc218\ub3c4 \uc788\ub2e4. \uce7c\ub9ac\uc2a4\ud1a0\uc758 \ucda9\ub3cc\uad6c\ub4e4\uc740 \uc77c\ubc18\uc801\uc73c\ub85c \ub2ec\uc5d0 \uc788\ub294 \ucda9\ub3cc\uad6c\ub4e4\ubcf4\ub2e4 \uae4a\uc774\uac00 \uc595\ub2e4.", "answer": "\ud2f4\ub4dc\ub974", "sentence": "25~100 km \uc815\ub3c4\uc758 \ud070 \ucda9\ub3cc\uad6c\ub4e4\uc740, \ud2f4\ub4dc\ub974 \ucc98\ub7fc \uac00\uc6b4\ub370\uc5d0 \ubd09\uc6b0\ub9ac \ub300\uc2e0 \uad6c\ub369\uc774\uac00 \uc874\uc7ac\ud55c\ub2e4.", "paragraph_sentence": "\ucda9\ub3cc\uad6c \uc9c0\ub984\uc758 \ubc94\uc704\ub294 0.1 km(\uc0ac\uc9c4 \ud574\uc0c1\ub3c4\ub85c \uacb0\uc815\ub41c \ud55c\uacc4)\uc5d0\uc11c 100 km \uc815\ub3c4\uc774\uace0, \uc774 \uac12\uc740 \ub2e4\ud658 \ucda9\ub3cc\uad6c\ub294 \uc138\uc9c0 \uc54a\uc740 \uac12\uc774\ub2e4. \uc9c0\ub984\uc774 5 km\uac00 \ub418\uc9c0 \uc54a\ub294 \uc791\uc740 \ucda9\ub3cc\uad6c\ub4e4\uc740 \ub2e8\uc21c\ud558\uac8c, \uc6b0\ubb35\ud558\uac70\ub098 \ubc14\ub2e5\uc774 \ud3c9\ud3c9\ud55c \ubaa8\uc591\uc744 \uac00\uc9c0\uace0 \uc788\ub2e4. 5~40 km \uc815\ub3c4\uc758 \ucda9\ub3cc\uad6c\ub4e4\uc740 \ubcf4\ud1b5 \uac00\uc6b4\ub370\uc5d0 \ubd09\uc6b0\ub9ac\uac00 \uc874\uc7ac\ud55c\ub2e4. 25~100 km \uc815\ub3c4\uc758 \ud070 \ucda9\ub3cc\uad6c\ub4e4\uc740, \ud2f4\ub4dc\ub974 \ucc98\ub7fc \uac00\uc6b4\ub370\uc5d0 \ubd09\uc6b0\ub9ac \ub300\uc2e0 \uad6c\ub369\uc774\uac00 \uc874\uc7ac\ud55c\ub2e4. \uac00\uc6b4\ub370\uc5d0 \ubd09\uc6b0\ub9ac\ub97c \uac00\uc9c0\uace0 \uc788\ub294 \uac00\uc7a5 \ud070 \ucda9\ub3cc\uad6c\uc778, \uc9c0\ub984 60 km\uc9dc\ub9ac \ucda9\ub3cc\uad6c\ub4e4\uc740 \ucda9\ub3cc \ud6c4 \uc911\uc2ec\ubd80\uc5d0\uc11c\uc758 \uad6c\uc870\uc801 \uc735\uae30\ub85c \uc778\ud55c \uac83\uc73c\ub85c \ucd94\uce21\ub41c\ub2e4. \uc608\uc2dc\ub85c \uc544\uc2a4\uac00\ub974\ub4dc\ub098 \ud558\ub974\uac00 \uc788\ub2e4. \ub9e4\uc6b0 \ud06c\uace0(\uc9c0\ub984 100 km \uc774\uc0c1), \ubc1d\uc740 \ucda9\ub3cc\uad6c\ub4e4 \uc911 \uc77c\ubd80\ub294 \uc774\ub840\uc801\uc73c\ub85c \ub3d4 \ubaa8\uc591\uc758 \uc9c0\uc9c8 \uad6c\uc870\ub97c \ubcf4\uc5ec\uc900\ub2e4. \uc774\ub7ec\ud55c \ud604\uc0c1\uc740 \ud754\ud558\uc9c0 \uc54a\uace0, \ub85c\ud508\ucc98\ub7fc \ub2e4\ud658 \ucda9\ub3cc\uad6c\ub85c \ubcc0\ud654\ud558\ub294 \uc9c0\ud615\uc77c \uc218\ub3c4 \uc788\ub2e4. \uce7c\ub9ac\uc2a4\ud1a0\uc758 \ucda9\ub3cc\uad6c\ub4e4\uc740 \uc77c\ubc18\uc801\uc73c\ub85c \ub2ec\uc5d0 \uc788\ub294 \ucda9\ub3cc\uad6c\ub4e4\ubcf4\ub2e4 \uae4a\uc774\uac00 \uc595\ub2e4.", "paragraph_answer": "\ucda9\ub3cc\uad6c \uc9c0\ub984\uc758 \ubc94\uc704\ub294 0.1 km(\uc0ac\uc9c4 \ud574\uc0c1\ub3c4\ub85c \uacb0\uc815\ub41c \ud55c\uacc4)\uc5d0\uc11c 100 km \uc815\ub3c4\uc774\uace0, \uc774 \uac12\uc740 \ub2e4\ud658 \ucda9\ub3cc\uad6c\ub294 \uc138\uc9c0 \uc54a\uc740 \uac12\uc774\ub2e4. \uc9c0\ub984\uc774 5 km\uac00 \ub418\uc9c0 \uc54a\ub294 \uc791\uc740 \ucda9\ub3cc\uad6c\ub4e4\uc740 \ub2e8\uc21c\ud558\uac8c, \uc6b0\ubb35\ud558\uac70\ub098 \ubc14\ub2e5\uc774 \ud3c9\ud3c9\ud55c \ubaa8\uc591\uc744 \uac00\uc9c0\uace0 \uc788\ub2e4. 5~40 km \uc815\ub3c4\uc758 \ucda9\ub3cc\uad6c\ub4e4\uc740 \ubcf4\ud1b5 \uac00\uc6b4\ub370\uc5d0 \ubd09\uc6b0\ub9ac\uac00 \uc874\uc7ac\ud55c\ub2e4. 25~100 km \uc815\ub3c4\uc758 \ud070 \ucda9\ub3cc\uad6c\ub4e4\uc740, \ud2f4\ub4dc\ub974 \ucc98\ub7fc \uac00\uc6b4\ub370\uc5d0 \ubd09\uc6b0\ub9ac \ub300\uc2e0 \uad6c\ub369\uc774\uac00 \uc874\uc7ac\ud55c\ub2e4. \uac00\uc6b4\ub370\uc5d0 \ubd09\uc6b0\ub9ac\ub97c \uac00\uc9c0\uace0 \uc788\ub294 \uac00\uc7a5 \ud070 \ucda9\ub3cc\uad6c\uc778, \uc9c0\ub984 60 km\uc9dc\ub9ac \ucda9\ub3cc\uad6c\ub4e4\uc740 \ucda9\ub3cc \ud6c4 \uc911\uc2ec\ubd80\uc5d0\uc11c\uc758 \uad6c\uc870\uc801 \uc735\uae30\ub85c \uc778\ud55c \uac83\uc73c\ub85c \ucd94\uce21\ub41c\ub2e4. \uc608\uc2dc\ub85c \uc544\uc2a4\uac00\ub974\ub4dc\ub098 \ud558\ub974\uac00 \uc788\ub2e4. \ub9e4\uc6b0 \ud06c\uace0(\uc9c0\ub984 100 km \uc774\uc0c1), \ubc1d\uc740 \ucda9\ub3cc\uad6c\ub4e4 \uc911 \uc77c\ubd80\ub294 \uc774\ub840\uc801\uc73c\ub85c \ub3d4 \ubaa8\uc591\uc758 \uc9c0\uc9c8 \uad6c\uc870\ub97c \ubcf4\uc5ec\uc900\ub2e4. \uc774\ub7ec\ud55c \ud604\uc0c1\uc740 \ud754\ud558\uc9c0 \uc54a\uace0, \ub85c\ud508\ucc98\ub7fc \ub2e4\ud658 \ucda9\ub3cc\uad6c\ub85c \ubcc0\ud654\ud558\ub294 \uc9c0\ud615\uc77c \uc218\ub3c4 \uc788\ub2e4. \uce7c\ub9ac\uc2a4\ud1a0\uc758 \ucda9\ub3cc\uad6c\ub4e4\uc740 \uc77c\ubc18\uc801\uc73c\ub85c \ub2ec\uc5d0 \uc788\ub294 \ucda9\ub3cc\uad6c\ub4e4\ubcf4\ub2e4 \uae4a\uc774\uac00 \uc595\ub2e4.", "sentence_answer": "25~100 km \uc815\ub3c4\uc758 \ud070 \ucda9\ub3cc\uad6c\ub4e4\uc740, \ud2f4\ub4dc\ub974 \ucc98\ub7fc \uac00\uc6b4\ub370\uc5d0 \ubd09\uc6b0\ub9ac \ub300\uc2e0 \uad6c\ub369\uc774\uac00 \uc874\uc7ac\ud55c\ub2e4.", "paragraph_id": "6561311-5-1", "paragraph_question": "question: \uce7c\ub9ac\uc2a4\ud1a0\uc758 \ud070 \ucda9\ub3cc\uad6c\ub294 \ubb34\uc5c7\ucc98\ub7fc \uac00\uc6b4\ub370 \ud070 \ubd09\uc6b0\ub9ac\uac00 \uc544\ub2cc \uad6c\ub369\uc774\uac00 \uc788\ub098\uc694?, context: \ucda9\ub3cc\uad6c \uc9c0\ub984\uc758 \ubc94\uc704\ub294 0.1 km(\uc0ac\uc9c4 \ud574\uc0c1\ub3c4\ub85c \uacb0\uc815\ub41c \ud55c\uacc4)\uc5d0\uc11c 100 km \uc815\ub3c4\uc774\uace0, \uc774 \uac12\uc740 \ub2e4\ud658 \ucda9\ub3cc\uad6c\ub294 \uc138\uc9c0 \uc54a\uc740 \uac12\uc774\ub2e4. \uc9c0\ub984\uc774 5 km\uac00 \ub418\uc9c0 \uc54a\ub294 \uc791\uc740 \ucda9\ub3cc\uad6c\ub4e4\uc740 \ub2e8\uc21c\ud558\uac8c, \uc6b0\ubb35\ud558\uac70\ub098 \ubc14\ub2e5\uc774 \ud3c9\ud3c9\ud55c \ubaa8\uc591\uc744 \uac00\uc9c0\uace0 \uc788\ub2e4. 5~40 km \uc815\ub3c4\uc758 \ucda9\ub3cc\uad6c\ub4e4\uc740 \ubcf4\ud1b5 \uac00\uc6b4\ub370\uc5d0 \ubd09\uc6b0\ub9ac\uac00 \uc874\uc7ac\ud55c\ub2e4. 25~100 km \uc815\ub3c4\uc758 \ud070 \ucda9\ub3cc\uad6c\ub4e4\uc740, \ud2f4\ub4dc\ub974\ucc98\ub7fc \uac00\uc6b4\ub370\uc5d0 \ubd09\uc6b0\ub9ac \ub300\uc2e0 \uad6c\ub369\uc774\uac00 \uc874\uc7ac\ud55c\ub2e4. \uac00\uc6b4\ub370\uc5d0 \ubd09\uc6b0\ub9ac\ub97c \uac00\uc9c0\uace0 \uc788\ub294 \uac00\uc7a5 \ud070 \ucda9\ub3cc\uad6c\uc778, \uc9c0\ub984 60 km\uc9dc\ub9ac \ucda9\ub3cc\uad6c\ub4e4\uc740 \ucda9\ub3cc \ud6c4 \uc911\uc2ec\ubd80\uc5d0\uc11c\uc758 \uad6c\uc870\uc801 \uc735\uae30\ub85c \uc778\ud55c \uac83\uc73c\ub85c \ucd94\uce21\ub41c\ub2e4. \uc608\uc2dc\ub85c \uc544\uc2a4\uac00\ub974\ub4dc\ub098 \ud558\ub974\uac00 \uc788\ub2e4. \ub9e4\uc6b0 \ud06c\uace0(\uc9c0\ub984 100 km \uc774\uc0c1), \ubc1d\uc740 \ucda9\ub3cc\uad6c\ub4e4 \uc911 \uc77c\ubd80\ub294 \uc774\ub840\uc801\uc73c\ub85c \ub3d4 \ubaa8\uc591\uc758 \uc9c0\uc9c8 \uad6c\uc870\ub97c \ubcf4\uc5ec\uc900\ub2e4. \uc774\ub7ec\ud55c \ud604\uc0c1\uc740 \ud754\ud558\uc9c0 \uc54a\uace0, \ub85c\ud508\ucc98\ub7fc \ub2e4\ud658 \ucda9\ub3cc\uad6c\ub85c \ubcc0\ud654\ud558\ub294 \uc9c0\ud615\uc77c \uc218\ub3c4 \uc788\ub2e4. \uce7c\ub9ac\uc2a4\ud1a0\uc758 \ucda9\ub3cc\uad6c\ub4e4\uc740 \uc77c\ubc18\uc801\uc73c\ub85c \ub2ec\uc5d0 \uc788\ub294 \ucda9\ub3cc\uad6c\ub4e4\ubcf4\ub2e4 \uae4a\uc774\uac00 \uc595\ub2e4."} {"question": "\ubbf8\uad6d\uc758 \ub3c4\uc2dc \uc911 \uace0\ud559\ub825 \uc0ac\ub78c\ub4e4\uc774 \ubaa8\uc778 \ub3c4\uc2dc \uc911 \ud558\ub098\ub85c \ubbf8\uad6d\uc5d0\uc11c \ub300\ud559\uc0dd\uc774 \uac00\uc7a5 \ub9ce\uc740 \ub3c4\uc2dc\ub294?", "paragraph": "\ub274\uc695\uc758 \uacf5\ub9bd\ud559\uad50 \uc2dc\uc2a4\ud15c\uc740 \ub274\uc695 \uad50\uc721\uad6d\uc5d0 \uc758\ud574 \uc6b4\uc601\ub418\uace0 \uc788\uc73c\uba70, \ubbf8\uad6d \ucd5c\ub300\uc774\ub2e4. \uc57d 110\ub9cc \uba85\uc758 \ud559\uc0dd\uc774 1,200\uac1c\uc758 \ucd08\ub4f1\ud559\uad50, \uc911\ud559\uad50\uc5d0\uc11c \uad50\uc721\uc744 \ubc1b\uace0 \uc788\ub2e4. \uadf8 \uc678, \uc885\uad50\uacc4 \ub4f1\uc758 \ud559\uad50\ub97c \ud569\uce58\uba74 \uc57d 900\uac1c\uc758 \uc0ac\ub9bd\ud559\uad50\uac00 \uc788\ub2e4. \ub274\uc695\uc774 \ub300\ud559 \ub3c4\uc2dc\ub77c\ub294 \uc774\ubbf8\uc9c0\ub294 \ubcc4\ub85c \uc5c6\uc9c0\ub9cc, \uc2e4\uc740 \uc57d 59\ub9cc 4000\uba85\uc758 \ub300\ud559\uc0dd\uc774 \uc788\uc73c\uba70, \uc774\uac83\uc740 \ubbf8\uad6d\uc5d0\uc11c \ucd5c\ub300\uc774\ub2e4. 2005\ub144, \ub9e8\ud574\ud2bc\uc758 \uc0ac\ub294 \uc0ac\ub78c 5\uba85 \uc911 3\uba85\uc740 \ub300\ud559 \uc878\uc5c5\uc0dd\uc774\uba70, 4\uba85 \uc911 1\uba85\uc740 \ub354 \ub192\uc740 \ud559\uc704\ub97c \uac00\uc9c0\uace0 \uc788\uc73c\uba70, \ubbf8\uad6d\uc758 \ub3c4\uc2dc \uc911\uc5d0\uc11c\ub3c4 \uace0\ud559\ub825 \uc0ac\ub78c\ub4e4\uc774 \ubaa8\uc778 \ub3c4\uc2dc \uc911 \ud558\ub098\uc774\ub2e4. \ub274\uc695\uc5d0\ub294 \uceec\ub7fc\ube44\uc544 \ub300\ud559\uad50, \ub274\uc695 \ub300\ud559\uad50, \ud3ec\ub4dc\ud584 \ub300\ud559\uad50, \uc138\uc778\ud2b8 \uc874\uc2a4 \ub300\ud559\uad50, \uc608\uc2dc\ubc14 \ub300\ud559\uad50, \ub274 \uc2a4\ucfe8, \ubc14\ub108\ub4dc \ub300\ud559\uad50, \uc904\ub9ac\uc544\ub4dc \ud559\uad50, \ud398\uc774\uc2a4 \ub300\ud559\uad50, \ucfe0\ud37c \uc720\ub2c8\uc5b8 \ub4f1 \uc885\uad50\uacc4\uc640 \uc804\ubb38\uacc4 \ub4f1\uc744 \ud3ec\ud568\ud55c \ub9ce\uc740 \uba85\ubb38 \uc0ac\ub9bd \ub300\ud559\uc774 \uc788\ub2e4. \ub610\ud55c \uacf5\uc801\uc778 \uad50\uc721 \uae30\uad00\uc73c\ub85c\ub294 \ubbf8\uad6d \uc81c 3\uc704 \uaddc\ubaa8\uc758 \uacf5\ub9bd \ub300\ud559\uc778 \ub274\uc695 \uc2dc\ub9bd \ub300\ud559\uad50, \ub274\uc695 \uc8fc\ub9bd \ub300\ud559\uad50\uac00 \uc788\ub2e4.", "answer": "\ub274\uc695", "sentence": "\ub274\uc695 \uc758 \uacf5\ub9bd\ud559\uad50 \uc2dc\uc2a4\ud15c\uc740 \ub274\uc695 \uad50\uc721\uad6d\uc5d0 \uc758\ud574 \uc6b4\uc601\ub418\uace0 \uc788\uc73c\uba70, \ubbf8\uad6d \ucd5c\ub300\uc774\ub2e4.", "paragraph_sentence": " \ub274\uc695 \uc758 \uacf5\ub9bd\ud559\uad50 \uc2dc\uc2a4\ud15c\uc740 \ub274\uc695 \uad50\uc721\uad6d\uc5d0 \uc758\ud574 \uc6b4\uc601\ub418\uace0 \uc788\uc73c\uba70, \ubbf8\uad6d \ucd5c\ub300\uc774\ub2e4. \uc57d 110\ub9cc \uba85\uc758 \ud559\uc0dd\uc774 1,200\uac1c\uc758 \ucd08\ub4f1\ud559\uad50, \uc911\ud559\uad50\uc5d0\uc11c \uad50\uc721\uc744 \ubc1b\uace0 \uc788\ub2e4. \uadf8 \uc678, \uc885\uad50\uacc4 \ub4f1\uc758 \ud559\uad50\ub97c \ud569\uce58\uba74 \uc57d 900\uac1c\uc758 \uc0ac\ub9bd\ud559\uad50\uac00 \uc788\ub2e4. \ub274\uc695\uc774 \ub300\ud559 \ub3c4\uc2dc\ub77c\ub294 \uc774\ubbf8\uc9c0\ub294 \ubcc4\ub85c \uc5c6\uc9c0\ub9cc, \uc2e4\uc740 \uc57d 59\ub9cc 4000\uba85\uc758 \ub300\ud559\uc0dd\uc774 \uc788\uc73c\uba70, \uc774\uac83\uc740 \ubbf8\uad6d\uc5d0\uc11c \ucd5c\ub300\uc774\ub2e4. 2005\ub144, \ub9e8\ud574\ud2bc\uc758 \uc0ac\ub294 \uc0ac\ub78c 5\uba85 \uc911 3\uba85\uc740 \ub300\ud559 \uc878\uc5c5\uc0dd\uc774\uba70, 4\uba85 \uc911 1\uba85\uc740 \ub354 \ub192\uc740 \ud559\uc704\ub97c \uac00\uc9c0\uace0 \uc788\uc73c\uba70, \ubbf8\uad6d\uc758 \ub3c4\uc2dc \uc911\uc5d0\uc11c\ub3c4 \uace0\ud559\ub825 \uc0ac\ub78c\ub4e4\uc774 \ubaa8\uc778 \ub3c4\uc2dc \uc911 \ud558\ub098\uc774\ub2e4. \ub274\uc695\uc5d0\ub294 \uceec\ub7fc\ube44\uc544 \ub300\ud559\uad50, \ub274\uc695 \ub300\ud559\uad50, \ud3ec\ub4dc\ud584 \ub300\ud559\uad50, \uc138\uc778\ud2b8 \uc874\uc2a4 \ub300\ud559\uad50, \uc608\uc2dc\ubc14 \ub300\ud559\uad50, \ub274 \uc2a4\ucfe8, \ubc14\ub108\ub4dc \ub300\ud559\uad50, \uc904\ub9ac\uc544\ub4dc \ud559\uad50, \ud398\uc774\uc2a4 \ub300\ud559\uad50, \ucfe0\ud37c \uc720\ub2c8\uc5b8 \ub4f1 \uc885\uad50\uacc4\uc640 \uc804\ubb38\uacc4 \ub4f1\uc744 \ud3ec\ud568\ud55c \ub9ce\uc740 \uba85\ubb38 \uc0ac\ub9bd \ub300\ud559\uc774 \uc788\ub2e4. \ub610\ud55c \uacf5\uc801\uc778 \uad50\uc721 \uae30\uad00\uc73c\ub85c\ub294 \ubbf8\uad6d \uc81c 3\uc704 \uaddc\ubaa8\uc758 \uacf5\ub9bd \ub300\ud559\uc778 \ub274\uc695 \uc2dc\ub9bd \ub300\ud559\uad50, \ub274\uc695 \uc8fc\ub9bd \ub300\ud559\uad50\uac00 \uc788\ub2e4.", "paragraph_answer": " \ub274\uc695 \uc758 \uacf5\ub9bd\ud559\uad50 \uc2dc\uc2a4\ud15c\uc740 \ub274\uc695 \uad50\uc721\uad6d\uc5d0 \uc758\ud574 \uc6b4\uc601\ub418\uace0 \uc788\uc73c\uba70, \ubbf8\uad6d \ucd5c\ub300\uc774\ub2e4. \uc57d 110\ub9cc \uba85\uc758 \ud559\uc0dd\uc774 1,200\uac1c\uc758 \ucd08\ub4f1\ud559\uad50, \uc911\ud559\uad50\uc5d0\uc11c \uad50\uc721\uc744 \ubc1b\uace0 \uc788\ub2e4. \uadf8 \uc678, \uc885\uad50\uacc4 \ub4f1\uc758 \ud559\uad50\ub97c \ud569\uce58\uba74 \uc57d 900\uac1c\uc758 \uc0ac\ub9bd\ud559\uad50\uac00 \uc788\ub2e4. \ub274\uc695\uc774 \ub300\ud559 \ub3c4\uc2dc\ub77c\ub294 \uc774\ubbf8\uc9c0\ub294 \ubcc4\ub85c \uc5c6\uc9c0\ub9cc, \uc2e4\uc740 \uc57d 59\ub9cc 4000\uba85\uc758 \ub300\ud559\uc0dd\uc774 \uc788\uc73c\uba70, \uc774\uac83\uc740 \ubbf8\uad6d\uc5d0\uc11c \ucd5c\ub300\uc774\ub2e4. 2005\ub144, \ub9e8\ud574\ud2bc\uc758 \uc0ac\ub294 \uc0ac\ub78c 5\uba85 \uc911 3\uba85\uc740 \ub300\ud559 \uc878\uc5c5\uc0dd\uc774\uba70, 4\uba85 \uc911 1\uba85\uc740 \ub354 \ub192\uc740 \ud559\uc704\ub97c \uac00\uc9c0\uace0 \uc788\uc73c\uba70, \ubbf8\uad6d\uc758 \ub3c4\uc2dc \uc911\uc5d0\uc11c\ub3c4 \uace0\ud559\ub825 \uc0ac\ub78c\ub4e4\uc774 \ubaa8\uc778 \ub3c4\uc2dc \uc911 \ud558\ub098\uc774\ub2e4. \ub274\uc695\uc5d0\ub294 \uceec\ub7fc\ube44\uc544 \ub300\ud559\uad50, \ub274\uc695 \ub300\ud559\uad50, \ud3ec\ub4dc\ud584 \ub300\ud559\uad50, \uc138\uc778\ud2b8 \uc874\uc2a4 \ub300\ud559\uad50, \uc608\uc2dc\ubc14 \ub300\ud559\uad50, \ub274 \uc2a4\ucfe8, \ubc14\ub108\ub4dc \ub300\ud559\uad50, \uc904\ub9ac\uc544\ub4dc \ud559\uad50, \ud398\uc774\uc2a4 \ub300\ud559\uad50, \ucfe0\ud37c \uc720\ub2c8\uc5b8 \ub4f1 \uc885\uad50\uacc4\uc640 \uc804\ubb38\uacc4 \ub4f1\uc744 \ud3ec\ud568\ud55c \ub9ce\uc740 \uba85\ubb38 \uc0ac\ub9bd \ub300\ud559\uc774 \uc788\ub2e4. \ub610\ud55c \uacf5\uc801\uc778 \uad50\uc721 \uae30\uad00\uc73c\ub85c\ub294 \ubbf8\uad6d \uc81c 3\uc704 \uaddc\ubaa8\uc758 \uacf5\ub9bd \ub300\ud559\uc778 \ub274\uc695 \uc2dc\ub9bd \ub300\ud559\uad50, \ub274\uc695 \uc8fc\ub9bd \ub300\ud559\uad50\uac00 \uc788\ub2e4.", "sentence_answer": " \ub274\uc695 \uc758 \uacf5\ub9bd\ud559\uad50 \uc2dc\uc2a4\ud15c\uc740 \ub274\uc695 \uad50\uc721\uad6d\uc5d0 \uc758\ud574 \uc6b4\uc601\ub418\uace0 \uc788\uc73c\uba70, \ubbf8\uad6d \ucd5c\ub300\uc774\ub2e4.", "paragraph_id": "6546073-14-0", "paragraph_question": "question: \ubbf8\uad6d\uc758 \ub3c4\uc2dc \uc911 \uace0\ud559\ub825 \uc0ac\ub78c\ub4e4\uc774 \ubaa8\uc778 \ub3c4\uc2dc \uc911 \ud558\ub098\ub85c \ubbf8\uad6d\uc5d0\uc11c \ub300\ud559\uc0dd\uc774 \uac00\uc7a5 \ub9ce\uc740 \ub3c4\uc2dc\ub294?, context: \ub274\uc695\uc758 \uacf5\ub9bd\ud559\uad50 \uc2dc\uc2a4\ud15c\uc740 \ub274\uc695 \uad50\uc721\uad6d\uc5d0 \uc758\ud574 \uc6b4\uc601\ub418\uace0 \uc788\uc73c\uba70, \ubbf8\uad6d \ucd5c\ub300\uc774\ub2e4. \uc57d 110\ub9cc \uba85\uc758 \ud559\uc0dd\uc774 1,200\uac1c\uc758 \ucd08\ub4f1\ud559\uad50, \uc911\ud559\uad50\uc5d0\uc11c \uad50\uc721\uc744 \ubc1b\uace0 \uc788\ub2e4. \uadf8 \uc678, \uc885\uad50\uacc4 \ub4f1\uc758 \ud559\uad50\ub97c \ud569\uce58\uba74 \uc57d 900\uac1c\uc758 \uc0ac\ub9bd\ud559\uad50\uac00 \uc788\ub2e4. \ub274\uc695\uc774 \ub300\ud559 \ub3c4\uc2dc\ub77c\ub294 \uc774\ubbf8\uc9c0\ub294 \ubcc4\ub85c \uc5c6\uc9c0\ub9cc, \uc2e4\uc740 \uc57d 59\ub9cc 4000\uba85\uc758 \ub300\ud559\uc0dd\uc774 \uc788\uc73c\uba70, \uc774\uac83\uc740 \ubbf8\uad6d\uc5d0\uc11c \ucd5c\ub300\uc774\ub2e4. 2005\ub144, \ub9e8\ud574\ud2bc\uc758 \uc0ac\ub294 \uc0ac\ub78c 5\uba85 \uc911 3\uba85\uc740 \ub300\ud559 \uc878\uc5c5\uc0dd\uc774\uba70, 4\uba85 \uc911 1\uba85\uc740 \ub354 \ub192\uc740 \ud559\uc704\ub97c \uac00\uc9c0\uace0 \uc788\uc73c\uba70, \ubbf8\uad6d\uc758 \ub3c4\uc2dc \uc911\uc5d0\uc11c\ub3c4 \uace0\ud559\ub825 \uc0ac\ub78c\ub4e4\uc774 \ubaa8\uc778 \ub3c4\uc2dc \uc911 \ud558\ub098\uc774\ub2e4. \ub274\uc695\uc5d0\ub294 \uceec\ub7fc\ube44\uc544 \ub300\ud559\uad50, \ub274\uc695 \ub300\ud559\uad50, \ud3ec\ub4dc\ud584 \ub300\ud559\uad50, \uc138\uc778\ud2b8 \uc874\uc2a4 \ub300\ud559\uad50, \uc608\uc2dc\ubc14 \ub300\ud559\uad50, \ub274 \uc2a4\ucfe8, \ubc14\ub108\ub4dc \ub300\ud559\uad50, \uc904\ub9ac\uc544\ub4dc \ud559\uad50, \ud398\uc774\uc2a4 \ub300\ud559\uad50, \ucfe0\ud37c \uc720\ub2c8\uc5b8 \ub4f1 \uc885\uad50\uacc4\uc640 \uc804\ubb38\uacc4 \ub4f1\uc744 \ud3ec\ud568\ud55c \ub9ce\uc740 \uba85\ubb38 \uc0ac\ub9bd \ub300\ud559\uc774 \uc788\ub2e4. \ub610\ud55c \uacf5\uc801\uc778 \uad50\uc721 \uae30\uad00\uc73c\ub85c\ub294 \ubbf8\uad6d \uc81c 3\uc704 \uaddc\ubaa8\uc758 \uacf5\ub9bd \ub300\ud559\uc778 \ub274\uc695 \uc2dc\ub9bd \ub300\ud559\uad50, \ub274\uc695 \uc8fc\ub9bd \ub300\ud559\uad50\uac00 \uc788\ub2e4."} {"question": "'\ubc84\ucd94\uc5bc \ub9ac\uc5bc\ub9ac\ud2f0'\ub77c\ub294 \ub2e8\uc5b4\uc758 \uae30\uc6d0\uc774 \ub41c \ucc45\uc744 \uc4f4 \uc0ac\ub78c\uc740?", "paragraph": "'\ubc84\ucd94\uc5bc\ub9ac\uc5bc\ub9ac\ud2f0(Virtual Reality; VR)'\ub77c\ub294 \ub2e8\uc5b4\uc758 \uae30\uc6d0\uc740 \ud504\ub791\uc2a4\uc758 \uadf9\uc791\uac00, \uc2dc\uc778, \ubc30\uc6b0\uc774\uc790 \uc5f0\ucd9c\uac00\uc778 \uc559\ud1a0\ub0c9 \uc544\ub974\ud1a0(Antonin Artaud)\uc758 \ucc45\uc5d0\uc11c \uadf8\uac00 \uadf9\uc7a5\uc744 \ubb18\uc0ac\ud558\ub294 \ub2e8\uc5b4\ub85c '\ubc84\ucd94\uc5bc \ub9ac\uc5bc\ub9ac\ud2f0'\ub97c \uc0ac\uc6a9\ud55c \uac83\uc774\ub2e4. \ud604\uc7ac\uc758 '\uac00\uc0c1\ud604\uc2e4' \uc758\ubbf8\uc640 \uac00\uae4c\uc6b4 '\uc778\uacf5 \ud604\uc2e4(artificial reality)'\uc774\ub77c\ub294 \ub2e8\uc5b4\ub294 1970\ub144 \ub300\uc5d0 \uccab \uc138\ub300 \uac00\uc0c1\ud604\uc2e4 \uc5f0\uad6c\uac00 \uc911 \ud558\ub098\uc778 \ub9c8\uc774\ub7f0 \ud06c\ub8e8\uac70(Myron Krueger)\uc5d0 \uc758\ud574 \ub9cc\ub4e4\uc5b4\uc84c\ub2e4. \uadf8 \ud6c4 1980\ub144\ub300 \ud6c4\ubc18\uc5d0 \ubbf8\uad6d\uc758 \ucef4\ud4e8\ud130 \uacfc\ud559\uc790\uc778 \uc7ac\ub7f0 \ub798\ub2c8\uc5b4(Jaron Lanier)\uc5d0 \uc758\ud574 \ud604\uc7ac\uc758 \uac00\uc0c1\ud604\uc2e4 \uac1c\ub150\uc744 \ub73b\ud558\ub294 \ub2e8\uc5b4\uc778 '\ubc84\ucd94\uc5bc \ub9ac\uc5bc\ub9ac\ud2f0'\uac00 \ub110\ub9ac \uc4f0\uc774\uac8c \ub418\uc5c8\ub2e4. \ud55c\uad6d\uc5b4\ub85c '\ubc84\ucd94\uc5bc \ub9ac\uc5bc\ub9ac\ud2f0'\ub294 '\uac00\uc0c1\ud604\uc2e4'\uc774\ub77c\uace0 \ubc88\uc5ed\ub418\ub294\ub370, \uc774\ub294 \uc77c\ubcf8\uc5b4 \ubc88\uc5ed\uc744 \uadf8\ub300\ub85c \ubcf8\ub72c \uac83\uc774\ub77c \uc54c\ub824\uc838 \uc788\ub2e4. \ucef4\ud4e8\ud130\uc758 \ucca8\ub2e8\uae30\uc220\uc744 \ub3d9\uc6d0\ud574 \uc778\uac04\uc758 \uc624\uac10\uc744 \ucc3d\uc870\ud558\ub294 \uac83\uc774 \ubc84\ucd94\uc5bc \ub9ac\uc5bc\ub9ac\ud2f0\uc758 \uac1c\ub150\uc778\ub370 \ub9c8\uce58 \ud604\uc2e4\uacfc \uc644\uc804\ud788 \ub300\uce6d\uc801 \uac1c\ub150\uc73c\ub85c \uc4f0\uc778 \ub4ef\ud55c '\uac00\uc0c1\ud604\uc2e4' \uc774\ub77c\ub294 \ubc88\uc5ed\uc774 \uc798\ubabb\ub418\uc5c8\ub2e4\ub294 \uc758\uacac\ub3c4 \uc788\ub2e4.", "answer": "\uc559\ud1a0\ub0c9 \uc544\ub974\ud1a0", "sentence": "'\ubc84\ucd94\uc5bc\ub9ac\uc5bc\ub9ac\ud2f0(Virtual Reality; VR)'\ub77c\ub294 \ub2e8\uc5b4\uc758 \uae30\uc6d0\uc740 \ud504\ub791\uc2a4\uc758 \uadf9\uc791\uac00, \uc2dc\uc778, \ubc30\uc6b0\uc774\uc790 \uc5f0\ucd9c\uac00\uc778 \uc559\ud1a0\ub0c9 \uc544\ub974\ud1a0 (Antonin Artaud)", "paragraph_sentence": " '\ubc84\ucd94\uc5bc\ub9ac\uc5bc\ub9ac\ud2f0(Virtual Reality; VR)'\ub77c\ub294 \ub2e8\uc5b4\uc758 \uae30\uc6d0\uc740 \ud504\ub791\uc2a4\uc758 \uadf9\uc791\uac00, \uc2dc\uc778, \ubc30\uc6b0\uc774\uc790 \uc5f0\ucd9c\uac00\uc778 \uc559\ud1a0\ub0c9 \uc544\ub974\ud1a0 (Antonin Artaud) \uc758 \ucc45\uc5d0\uc11c \uadf8\uac00 \uadf9\uc7a5\uc744 \ubb18\uc0ac\ud558\ub294 \ub2e8\uc5b4\ub85c '\ubc84\ucd94\uc5bc \ub9ac\uc5bc\ub9ac\ud2f0'\ub97c \uc0ac\uc6a9\ud55c \uac83\uc774\ub2e4. \ud604\uc7ac\uc758 '\uac00\uc0c1\ud604\uc2e4' \uc758\ubbf8\uc640 \uac00\uae4c\uc6b4 '\uc778\uacf5 \ud604\uc2e4(artificial reality)'\uc774\ub77c\ub294 \ub2e8\uc5b4\ub294 1970\ub144 \ub300\uc5d0 \uccab \uc138\ub300 \uac00\uc0c1\ud604\uc2e4 \uc5f0\uad6c\uac00 \uc911 \ud558\ub098\uc778 \ub9c8\uc774\ub7f0 \ud06c\ub8e8\uac70(Myron Krueger)\uc5d0 \uc758\ud574 \ub9cc\ub4e4\uc5b4\uc84c\ub2e4. \uadf8 \ud6c4 1980\ub144\ub300 \ud6c4\ubc18\uc5d0 \ubbf8\uad6d\uc758 \ucef4\ud4e8\ud130 \uacfc\ud559\uc790\uc778 \uc7ac\ub7f0 \ub798\ub2c8\uc5b4(Jaron Lanier)\uc5d0 \uc758\ud574 \ud604\uc7ac\uc758 \uac00\uc0c1\ud604\uc2e4 \uac1c\ub150\uc744 \ub73b\ud558\ub294 \ub2e8\uc5b4\uc778 '\ubc84\ucd94\uc5bc \ub9ac\uc5bc\ub9ac\ud2f0'\uac00 \ub110\ub9ac \uc4f0\uc774\uac8c \ub418\uc5c8\ub2e4. \ud55c\uad6d\uc5b4\ub85c '\ubc84\ucd94\uc5bc \ub9ac\uc5bc\ub9ac\ud2f0'\ub294 '\uac00\uc0c1\ud604\uc2e4'\uc774\ub77c\uace0 \ubc88\uc5ed\ub418\ub294\ub370, \uc774\ub294 \uc77c\ubcf8\uc5b4 \ubc88\uc5ed\uc744 \uadf8\ub300\ub85c \ubcf8\ub72c \uac83\uc774\ub77c \uc54c\ub824\uc838 \uc788\ub2e4. \ucef4\ud4e8\ud130\uc758 \ucca8\ub2e8\uae30\uc220\uc744 \ub3d9\uc6d0\ud574 \uc778\uac04\uc758 \uc624\uac10\uc744 \ucc3d\uc870\ud558\ub294 \uac83\uc774 \ubc84\ucd94\uc5bc \ub9ac\uc5bc\ub9ac\ud2f0\uc758 \uac1c\ub150\uc778\ub370 \ub9c8\uce58 \ud604\uc2e4\uacfc \uc644\uc804\ud788 \ub300\uce6d\uc801 \uac1c\ub150\uc73c\ub85c \uc4f0\uc778 \ub4ef\ud55c '\uac00\uc0c1\ud604\uc2e4' \uc774\ub77c\ub294 \ubc88\uc5ed\uc774 \uc798\ubabb\ub418\uc5c8\ub2e4\ub294 \uc758\uacac\ub3c4 \uc788\ub2e4.", "paragraph_answer": "'\ubc84\ucd94\uc5bc\ub9ac\uc5bc\ub9ac\ud2f0(Virtual Reality; VR)'\ub77c\ub294 \ub2e8\uc5b4\uc758 \uae30\uc6d0\uc740 \ud504\ub791\uc2a4\uc758 \uadf9\uc791\uac00, \uc2dc\uc778, \ubc30\uc6b0\uc774\uc790 \uc5f0\ucd9c\uac00\uc778 \uc559\ud1a0\ub0c9 \uc544\ub974\ud1a0 (Antonin Artaud)\uc758 \ucc45\uc5d0\uc11c \uadf8\uac00 \uadf9\uc7a5\uc744 \ubb18\uc0ac\ud558\ub294 \ub2e8\uc5b4\ub85c '\ubc84\ucd94\uc5bc \ub9ac\uc5bc\ub9ac\ud2f0'\ub97c \uc0ac\uc6a9\ud55c \uac83\uc774\ub2e4. \ud604\uc7ac\uc758 '\uac00\uc0c1\ud604\uc2e4' \uc758\ubbf8\uc640 \uac00\uae4c\uc6b4 '\uc778\uacf5 \ud604\uc2e4(artificial reality)'\uc774\ub77c\ub294 \ub2e8\uc5b4\ub294 1970\ub144 \ub300\uc5d0 \uccab \uc138\ub300 \uac00\uc0c1\ud604\uc2e4 \uc5f0\uad6c\uac00 \uc911 \ud558\ub098\uc778 \ub9c8\uc774\ub7f0 \ud06c\ub8e8\uac70(Myron Krueger)\uc5d0 \uc758\ud574 \ub9cc\ub4e4\uc5b4\uc84c\ub2e4. \uadf8 \ud6c4 1980\ub144\ub300 \ud6c4\ubc18\uc5d0 \ubbf8\uad6d\uc758 \ucef4\ud4e8\ud130 \uacfc\ud559\uc790\uc778 \uc7ac\ub7f0 \ub798\ub2c8\uc5b4(Jaron Lanier)\uc5d0 \uc758\ud574 \ud604\uc7ac\uc758 \uac00\uc0c1\ud604\uc2e4 \uac1c\ub150\uc744 \ub73b\ud558\ub294 \ub2e8\uc5b4\uc778 '\ubc84\ucd94\uc5bc \ub9ac\uc5bc\ub9ac\ud2f0'\uac00 \ub110\ub9ac \uc4f0\uc774\uac8c \ub418\uc5c8\ub2e4. \ud55c\uad6d\uc5b4\ub85c '\ubc84\ucd94\uc5bc \ub9ac\uc5bc\ub9ac\ud2f0'\ub294 '\uac00\uc0c1\ud604\uc2e4'\uc774\ub77c\uace0 \ubc88\uc5ed\ub418\ub294\ub370, \uc774\ub294 \uc77c\ubcf8\uc5b4 \ubc88\uc5ed\uc744 \uadf8\ub300\ub85c \ubcf8\ub72c \uac83\uc774\ub77c \uc54c\ub824\uc838 \uc788\ub2e4. \ucef4\ud4e8\ud130\uc758 \ucca8\ub2e8\uae30\uc220\uc744 \ub3d9\uc6d0\ud574 \uc778\uac04\uc758 \uc624\uac10\uc744 \ucc3d\uc870\ud558\ub294 \uac83\uc774 \ubc84\ucd94\uc5bc \ub9ac\uc5bc\ub9ac\ud2f0\uc758 \uac1c\ub150\uc778\ub370 \ub9c8\uce58 \ud604\uc2e4\uacfc \uc644\uc804\ud788 \ub300\uce6d\uc801 \uac1c\ub150\uc73c\ub85c \uc4f0\uc778 \ub4ef\ud55c '\uac00\uc0c1\ud604\uc2e4' \uc774\ub77c\ub294 \ubc88\uc5ed\uc774 \uc798\ubabb\ub418\uc5c8\ub2e4\ub294 \uc758\uacac\ub3c4 \uc788\ub2e4.", "sentence_answer": "'\ubc84\ucd94\uc5bc\ub9ac\uc5bc\ub9ac\ud2f0(Virtual Reality; VR)'\ub77c\ub294 \ub2e8\uc5b4\uc758 \uae30\uc6d0\uc740 \ud504\ub791\uc2a4\uc758 \uadf9\uc791\uac00, \uc2dc\uc778, \ubc30\uc6b0\uc774\uc790 \uc5f0\ucd9c\uac00\uc778 \uc559\ud1a0\ub0c9 \uc544\ub974\ud1a0 (Antonin Artaud)", "paragraph_id": "5782945-1-0", "paragraph_question": "question: '\ubc84\ucd94\uc5bc \ub9ac\uc5bc\ub9ac\ud2f0'\ub77c\ub294 \ub2e8\uc5b4\uc758 \uae30\uc6d0\uc774 \ub41c \ucc45\uc744 \uc4f4 \uc0ac\ub78c\uc740?, context: '\ubc84\ucd94\uc5bc\ub9ac\uc5bc\ub9ac\ud2f0(Virtual Reality; VR)'\ub77c\ub294 \ub2e8\uc5b4\uc758 \uae30\uc6d0\uc740 \ud504\ub791\uc2a4\uc758 \uadf9\uc791\uac00, \uc2dc\uc778, \ubc30\uc6b0\uc774\uc790 \uc5f0\ucd9c\uac00\uc778 \uc559\ud1a0\ub0c9 \uc544\ub974\ud1a0(Antonin Artaud)\uc758 \ucc45\uc5d0\uc11c \uadf8\uac00 \uadf9\uc7a5\uc744 \ubb18\uc0ac\ud558\ub294 \ub2e8\uc5b4\ub85c '\ubc84\ucd94\uc5bc \ub9ac\uc5bc\ub9ac\ud2f0'\ub97c \uc0ac\uc6a9\ud55c \uac83\uc774\ub2e4. \ud604\uc7ac\uc758 '\uac00\uc0c1\ud604\uc2e4' \uc758\ubbf8\uc640 \uac00\uae4c\uc6b4 '\uc778\uacf5 \ud604\uc2e4(artificial reality)'\uc774\ub77c\ub294 \ub2e8\uc5b4\ub294 1970\ub144 \ub300\uc5d0 \uccab \uc138\ub300 \uac00\uc0c1\ud604\uc2e4 \uc5f0\uad6c\uac00 \uc911 \ud558\ub098\uc778 \ub9c8\uc774\ub7f0 \ud06c\ub8e8\uac70(Myron Krueger)\uc5d0 \uc758\ud574 \ub9cc\ub4e4\uc5b4\uc84c\ub2e4. \uadf8 \ud6c4 1980\ub144\ub300 \ud6c4\ubc18\uc5d0 \ubbf8\uad6d\uc758 \ucef4\ud4e8\ud130 \uacfc\ud559\uc790\uc778 \uc7ac\ub7f0 \ub798\ub2c8\uc5b4(Jaron Lanier)\uc5d0 \uc758\ud574 \ud604\uc7ac\uc758 \uac00\uc0c1\ud604\uc2e4 \uac1c\ub150\uc744 \ub73b\ud558\ub294 \ub2e8\uc5b4\uc778 '\ubc84\ucd94\uc5bc \ub9ac\uc5bc\ub9ac\ud2f0'\uac00 \ub110\ub9ac \uc4f0\uc774\uac8c \ub418\uc5c8\ub2e4. \ud55c\uad6d\uc5b4\ub85c '\ubc84\ucd94\uc5bc \ub9ac\uc5bc\ub9ac\ud2f0'\ub294 '\uac00\uc0c1\ud604\uc2e4'\uc774\ub77c\uace0 \ubc88\uc5ed\ub418\ub294\ub370, \uc774\ub294 \uc77c\ubcf8\uc5b4 \ubc88\uc5ed\uc744 \uadf8\ub300\ub85c \ubcf8\ub72c \uac83\uc774\ub77c \uc54c\ub824\uc838 \uc788\ub2e4. \ucef4\ud4e8\ud130\uc758 \ucca8\ub2e8\uae30\uc220\uc744 \ub3d9\uc6d0\ud574 \uc778\uac04\uc758 \uc624\uac10\uc744 \ucc3d\uc870\ud558\ub294 \uac83\uc774 \ubc84\ucd94\uc5bc \ub9ac\uc5bc\ub9ac\ud2f0\uc758 \uac1c\ub150\uc778\ub370 \ub9c8\uce58 \ud604\uc2e4\uacfc \uc644\uc804\ud788 \ub300\uce6d\uc801 \uac1c\ub150\uc73c\ub85c \uc4f0\uc778 \ub4ef\ud55c '\uac00\uc0c1\ud604\uc2e4' \uc774\ub77c\ub294 \ubc88\uc5ed\uc774 \uc798\ubabb\ub418\uc5c8\ub2e4\ub294 \uc758\uacac\ub3c4 \uc788\ub2e4."} {"question": "\ub2e4\ucf00\ub2e4 \uc694\uc2dc\uc988\ubbf8\ub294 \uc544\uc2dc\uce74\uac00 \uc694\uc2dc\uc544\ud0a4\uc5d0\uac8c \uc5b4\ub5a4 \uad00\uacc4\uc778\uac00?", "paragraph": "\uc544\uc2dc\uce74\uac00 \uc694\uc2dc\uc544\ud0a4\uac00 \ub9e4\ubd80\uc778 \ub2e4\ucf00\ub2e4 \uc694\uc2dc\uc988\ubbf8\ub97c \uc758\uc9c0\ud574 \uc640\uce74\uc0ac\ub85c \uc624\uace0, \uadf8 \ud6c4 \uc5d0\uce58\uc820 \uc544\uc0ac\ucfe0\ub77c\uac00\ub85c \ub3c4\ub9dd\uc624\uac8c \ub41c\ub2e4. \uc774\ub54c \ubbf8\uc4f0\ud788\ub370\ub294 \uc694\uc2dc\uc544\ud0a4\uc640 \uc811\ucd09\uc744 \uac16\ub294\ub2e4. \ubbf8\uc4f0\ud788\ub370\uc758 \uc5b4\uba38\ub2c8\ub294 \ub2e4\ucf00\ub2e4 \uc694\uc2dc\uc988\ubbf8\uc640 \ub0a8\ub9e4\uad00\uacc4\ub85c \uc54c\ub824\uc838 \uc788\ub2e4. \ub610, \uc544\uc0ac\ucfe0\ub77c \uc694\uc2dc\uce74\uac8c\uc758 \uc5b4\uba38\ub2c8\ub3c4 \uc640\uce74\uc0ac \ub2e4\ucf00\ub2e4\uac00 \ucd9c\uc2e0\uc73c\ub85c \uc788\uc5c8\uace0, \uc774\ub7f0 \uc5f0\ud6c4\ub85c \uc694\uc2dc\uc544\ud0a4 \uc811\ub300\uc5ed\uc73c\ub85c \uc784\uba85\ub418\uc5c8\ub2e4\uace0 \uc0dd\uac01\ub41c\ub2e4. \uc694\uc2dc\uc544\ud0a4\ub294 \uc544\uc0ac\ucfe0\ub77c\uc758 \uc0c1\uacbd\uc744 \uae30\ub300\ud588\uc9c0\ub9cc, \uc694\uc2dc\uce74\uac8c\ub294 \ubbf8\ub3d9\ub3c4 \ud558\uc9c0 \uc54a\ub294\ub2e4. \uadf8\ub798\uc11c, \ubbf8\uc4f0\ud788\ub370\ub97c \ud1b5\ud574 \uc624\ub2e4 \ub178\ubd80\ub098\uac00\uac00 \uad50\ud1a0\ub97c \uacf5\uaca9 \uc790\uc2e0\uc744 \uc138\uc774\uc774\ud0c0\uc774\uc1fc\uad70\uc73c\ub85c \uc639\ub9bd\ub418\ub3c4\ub85d \uc694\uccad\ud55c\ub2e4. \ubbf8\uc4f0\ud788\ub370\uc758 \uc219\ubaa8\uac00 \uc0ac\uc774\ud1a0 \ub3c4\uc0b0\uc758 \ubd80\uc778\uc73c\ub85c, \ub178\ubd80\ub098\uac00\uc758 \uc815\uc2e4\uc778 \uc0ac\uc774\ud1a0 \ub3c4\uc0b0\uc758 \ub538(\ub178\ud788\uba54)\uacfc\ub294 \uc0ac\ucd0c\uad00\uacc4\ub85c \uc788\uc744 \uac00\ub2a5\uc131\uc774 \uc788\ub294 \uac83 \uac19\ub2e4. \uadf8 \uc778\uc5f0\uc73c\ub85c \uc694\uc2dc\uc544\ud0a4\uc758 \ubd80\ud0c1\uc744 \ub178\ubd80\ub098\uac00\uc5d0\uac8c \uccad\ud588\ub2e4\uace0 \uc804\ud574\uc9c4\ub2e4.", "answer": "\ub9e4\ubd80", "sentence": "\uc544\uc2dc\uce74\uac00 \uc694\uc2dc\uc544\ud0a4\uac00 \ub9e4\ubd80 \uc778 \ub2e4\ucf00\ub2e4 \uc694\uc2dc\uc988\ubbf8\ub97c \uc758\uc9c0\ud574 \uc640\uce74\uc0ac\ub85c \uc624\uace0, \uadf8 \ud6c4 \uc5d0\uce58\uc820 \uc544\uc0ac\ucfe0\ub77c\uac00\ub85c \ub3c4\ub9dd\uc624\uac8c \ub41c\ub2e4.", "paragraph_sentence": " \uc544\uc2dc\uce74\uac00 \uc694\uc2dc\uc544\ud0a4\uac00 \ub9e4\ubd80 \uc778 \ub2e4\ucf00\ub2e4 \uc694\uc2dc\uc988\ubbf8\ub97c \uc758\uc9c0\ud574 \uc640\uce74\uc0ac\ub85c \uc624\uace0, \uadf8 \ud6c4 \uc5d0\uce58\uc820 \uc544\uc0ac\ucfe0\ub77c\uac00\ub85c \ub3c4\ub9dd\uc624\uac8c \ub41c\ub2e4. \uc774\ub54c \ubbf8\uc4f0\ud788\ub370\ub294 \uc694\uc2dc\uc544\ud0a4\uc640 \uc811\ucd09\uc744 \uac16\ub294\ub2e4. \ubbf8\uc4f0\ud788\ub370\uc758 \uc5b4\uba38\ub2c8\ub294 \ub2e4\ucf00\ub2e4 \uc694\uc2dc\uc988\ubbf8\uc640 \ub0a8\ub9e4\uad00\uacc4\ub85c \uc54c\ub824\uc838 \uc788\ub2e4. \ub610, \uc544\uc0ac\ucfe0\ub77c \uc694\uc2dc\uce74\uac8c\uc758 \uc5b4\uba38\ub2c8\ub3c4 \uc640\uce74\uc0ac \ub2e4\ucf00\ub2e4\uac00 \ucd9c\uc2e0\uc73c\ub85c \uc788\uc5c8\uace0, \uc774\ub7f0 \uc5f0\ud6c4\ub85c \uc694\uc2dc\uc544\ud0a4 \uc811\ub300\uc5ed\uc73c\ub85c \uc784\uba85\ub418\uc5c8\ub2e4\uace0 \uc0dd\uac01\ub41c\ub2e4. \uc694\uc2dc\uc544\ud0a4\ub294 \uc544\uc0ac\ucfe0\ub77c\uc758 \uc0c1\uacbd\uc744 \uae30\ub300\ud588\uc9c0\ub9cc, \uc694\uc2dc\uce74\uac8c\ub294 \ubbf8\ub3d9\ub3c4 \ud558\uc9c0 \uc54a\ub294\ub2e4. \uadf8\ub798\uc11c, \ubbf8\uc4f0\ud788\ub370\ub97c \ud1b5\ud574 \uc624\ub2e4 \ub178\ubd80\ub098\uac00\uac00 \uad50\ud1a0\ub97c \uacf5\uaca9 \uc790\uc2e0\uc744 \uc138\uc774\uc774\ud0c0\uc774\uc1fc\uad70\uc73c\ub85c \uc639\ub9bd\ub418\ub3c4\ub85d \uc694\uccad\ud55c\ub2e4. \ubbf8\uc4f0\ud788\ub370\uc758 \uc219\ubaa8\uac00 \uc0ac\uc774\ud1a0 \ub3c4\uc0b0\uc758 \ubd80\uc778\uc73c\ub85c, \ub178\ubd80\ub098\uac00\uc758 \uc815\uc2e4\uc778 \uc0ac\uc774\ud1a0 \ub3c4\uc0b0\uc758 \ub538(\ub178\ud788\uba54)\uacfc\ub294 \uc0ac\ucd0c\uad00\uacc4\ub85c \uc788\uc744 \uac00\ub2a5\uc131\uc774 \uc788\ub294 \uac83 \uac19\ub2e4. \uadf8 \uc778\uc5f0\uc73c\ub85c \uc694\uc2dc\uc544\ud0a4\uc758 \ubd80\ud0c1\uc744 \ub178\ubd80\ub098\uac00\uc5d0\uac8c \uccad\ud588\ub2e4\uace0 \uc804\ud574\uc9c4\ub2e4.", "paragraph_answer": "\uc544\uc2dc\uce74\uac00 \uc694\uc2dc\uc544\ud0a4\uac00 \ub9e4\ubd80 \uc778 \ub2e4\ucf00\ub2e4 \uc694\uc2dc\uc988\ubbf8\ub97c \uc758\uc9c0\ud574 \uc640\uce74\uc0ac\ub85c \uc624\uace0, \uadf8 \ud6c4 \uc5d0\uce58\uc820 \uc544\uc0ac\ucfe0\ub77c\uac00\ub85c \ub3c4\ub9dd\uc624\uac8c \ub41c\ub2e4. \uc774\ub54c \ubbf8\uc4f0\ud788\ub370\ub294 \uc694\uc2dc\uc544\ud0a4\uc640 \uc811\ucd09\uc744 \uac16\ub294\ub2e4. \ubbf8\uc4f0\ud788\ub370\uc758 \uc5b4\uba38\ub2c8\ub294 \ub2e4\ucf00\ub2e4 \uc694\uc2dc\uc988\ubbf8\uc640 \ub0a8\ub9e4\uad00\uacc4\ub85c \uc54c\ub824\uc838 \uc788\ub2e4. \ub610, \uc544\uc0ac\ucfe0\ub77c \uc694\uc2dc\uce74\uac8c\uc758 \uc5b4\uba38\ub2c8\ub3c4 \uc640\uce74\uc0ac \ub2e4\ucf00\ub2e4\uac00 \ucd9c\uc2e0\uc73c\ub85c \uc788\uc5c8\uace0, \uc774\ub7f0 \uc5f0\ud6c4\ub85c \uc694\uc2dc\uc544\ud0a4 \uc811\ub300\uc5ed\uc73c\ub85c \uc784\uba85\ub418\uc5c8\ub2e4\uace0 \uc0dd\uac01\ub41c\ub2e4. \uc694\uc2dc\uc544\ud0a4\ub294 \uc544\uc0ac\ucfe0\ub77c\uc758 \uc0c1\uacbd\uc744 \uae30\ub300\ud588\uc9c0\ub9cc, \uc694\uc2dc\uce74\uac8c\ub294 \ubbf8\ub3d9\ub3c4 \ud558\uc9c0 \uc54a\ub294\ub2e4. \uadf8\ub798\uc11c, \ubbf8\uc4f0\ud788\ub370\ub97c \ud1b5\ud574 \uc624\ub2e4 \ub178\ubd80\ub098\uac00\uac00 \uad50\ud1a0\ub97c \uacf5\uaca9 \uc790\uc2e0\uc744 \uc138\uc774\uc774\ud0c0\uc774\uc1fc\uad70\uc73c\ub85c \uc639\ub9bd\ub418\ub3c4\ub85d \uc694\uccad\ud55c\ub2e4. \ubbf8\uc4f0\ud788\ub370\uc758 \uc219\ubaa8\uac00 \uc0ac\uc774\ud1a0 \ub3c4\uc0b0\uc758 \ubd80\uc778\uc73c\ub85c, \ub178\ubd80\ub098\uac00\uc758 \uc815\uc2e4\uc778 \uc0ac\uc774\ud1a0 \ub3c4\uc0b0\uc758 \ub538(\ub178\ud788\uba54)\uacfc\ub294 \uc0ac\ucd0c\uad00\uacc4\ub85c \uc788\uc744 \uac00\ub2a5\uc131\uc774 \uc788\ub294 \uac83 \uac19\ub2e4. \uadf8 \uc778\uc5f0\uc73c\ub85c \uc694\uc2dc\uc544\ud0a4\uc758 \ubd80\ud0c1\uc744 \ub178\ubd80\ub098\uac00\uc5d0\uac8c \uccad\ud588\ub2e4\uace0 \uc804\ud574\uc9c4\ub2e4.", "sentence_answer": "\uc544\uc2dc\uce74\uac00 \uc694\uc2dc\uc544\ud0a4\uac00 \ub9e4\ubd80 \uc778 \ub2e4\ucf00\ub2e4 \uc694\uc2dc\uc988\ubbf8\ub97c \uc758\uc9c0\ud574 \uc640\uce74\uc0ac\ub85c \uc624\uace0, \uadf8 \ud6c4 \uc5d0\uce58\uc820 \uc544\uc0ac\ucfe0\ub77c\uac00\ub85c \ub3c4\ub9dd\uc624\uac8c \ub41c\ub2e4.", "paragraph_id": "6538744-0-0", "paragraph_question": "question: \ub2e4\ucf00\ub2e4 \uc694\uc2dc\uc988\ubbf8\ub294 \uc544\uc2dc\uce74\uac00 \uc694\uc2dc\uc544\ud0a4\uc5d0\uac8c \uc5b4\ub5a4 \uad00\uacc4\uc778\uac00?, context: \uc544\uc2dc\uce74\uac00 \uc694\uc2dc\uc544\ud0a4\uac00 \ub9e4\ubd80\uc778 \ub2e4\ucf00\ub2e4 \uc694\uc2dc\uc988\ubbf8\ub97c \uc758\uc9c0\ud574 \uc640\uce74\uc0ac\ub85c \uc624\uace0, \uadf8 \ud6c4 \uc5d0\uce58\uc820 \uc544\uc0ac\ucfe0\ub77c\uac00\ub85c \ub3c4\ub9dd\uc624\uac8c \ub41c\ub2e4. \uc774\ub54c \ubbf8\uc4f0\ud788\ub370\ub294 \uc694\uc2dc\uc544\ud0a4\uc640 \uc811\ucd09\uc744 \uac16\ub294\ub2e4. \ubbf8\uc4f0\ud788\ub370\uc758 \uc5b4\uba38\ub2c8\ub294 \ub2e4\ucf00\ub2e4 \uc694\uc2dc\uc988\ubbf8\uc640 \ub0a8\ub9e4\uad00\uacc4\ub85c \uc54c\ub824\uc838 \uc788\ub2e4. \ub610, \uc544\uc0ac\ucfe0\ub77c \uc694\uc2dc\uce74\uac8c\uc758 \uc5b4\uba38\ub2c8\ub3c4 \uc640\uce74\uc0ac \ub2e4\ucf00\ub2e4\uac00 \ucd9c\uc2e0\uc73c\ub85c \uc788\uc5c8\uace0, \uc774\ub7f0 \uc5f0\ud6c4\ub85c \uc694\uc2dc\uc544\ud0a4 \uc811\ub300\uc5ed\uc73c\ub85c \uc784\uba85\ub418\uc5c8\ub2e4\uace0 \uc0dd\uac01\ub41c\ub2e4. \uc694\uc2dc\uc544\ud0a4\ub294 \uc544\uc0ac\ucfe0\ub77c\uc758 \uc0c1\uacbd\uc744 \uae30\ub300\ud588\uc9c0\ub9cc, \uc694\uc2dc\uce74\uac8c\ub294 \ubbf8\ub3d9\ub3c4 \ud558\uc9c0 \uc54a\ub294\ub2e4. \uadf8\ub798\uc11c, \ubbf8\uc4f0\ud788\ub370\ub97c \ud1b5\ud574 \uc624\ub2e4 \ub178\ubd80\ub098\uac00\uac00 \uad50\ud1a0\ub97c \uacf5\uaca9 \uc790\uc2e0\uc744 \uc138\uc774\uc774\ud0c0\uc774\uc1fc\uad70\uc73c\ub85c \uc639\ub9bd\ub418\ub3c4\ub85d \uc694\uccad\ud55c\ub2e4. \ubbf8\uc4f0\ud788\ub370\uc758 \uc219\ubaa8\uac00 \uc0ac\uc774\ud1a0 \ub3c4\uc0b0\uc758 \ubd80\uc778\uc73c\ub85c, \ub178\ubd80\ub098\uac00\uc758 \uc815\uc2e4\uc778 \uc0ac\uc774\ud1a0 \ub3c4\uc0b0\uc758 \ub538(\ub178\ud788\uba54)\uacfc\ub294 \uc0ac\ucd0c\uad00\uacc4\ub85c \uc788\uc744 \uac00\ub2a5\uc131\uc774 \uc788\ub294 \uac83 \uac19\ub2e4. \uadf8 \uc778\uc5f0\uc73c\ub85c \uc694\uc2dc\uc544\ud0a4\uc758 \ubd80\ud0c1\uc744 \ub178\ubd80\ub098\uac00\uc5d0\uac8c \uccad\ud588\ub2e4\uace0 \uc804\ud574\uc9c4\ub2e4."} {"question": "\uc911\ub3d9\uacfc \uc720\ub7fd \uc0ac\uc774\uc758 \uacc4\uc18d\ub41c \ub2e4\ud23c\uc758 \uc804\ud658\uc810\uc774 \ub41c \uc804\ud22c\ub294?", "paragraph": "10\uc6d4 7\uc77c \uc77c\uc694\uc77c \uc544\uce68, \ub808\ud310\ud1a0 \ud574\uad70 \uae30\uc9c0\uc5d0\uc11c \uc11c\ucabd\uc73c\ub85c \ucd9c\ud56d\ud55c \uc624\uc2a4\ub9cc \ud568\ub300\uc640 \uba54\uc2dc\ub098\uc5d0\uc11c \ucd9c\ud56d\ud55c \uc2e0\uc131 \ub3d9\ub9f9\uc758 \ud568\ub300\uac00 \uadf8\ub9ac\uc2a4 \uc11c\ubd80\uc758 \ubc14\uae65\ucabd\uc5d0 \uc788\ub294 \ud30c\ud2b8\ub77c\uc2a4 \ub9cc\uc758 \ub044\ud2b8\uba38\ub9ac\uc5d0\uc11c \ub9cc\ub098\uba74\uc11c 5\uc2dc\uac04\uc5d0 \uac78\uce5c \uc804\ud22c\uac00 \uc2dc\uc791\ub418\uc5c8\ub2e4. \uc804\ud22c \uc774\ud6c4 \uc2e0\uc131 \ub3d9\ub9f9\uc740 \uc77c\uc2dc\uc801\uc73c\ub85c \uc9c0\uc911\ud574\uc758 \ud328\uad8c\uc744 \uc7a5\uc545\ud558\uc600\uc73c\uba70, \ub85c\ub9c8\ub97c \uc624\uc2a4\ub9cc\uc758 \uce68\ub7b5\uc73c\ub85c\ubd80\ud130 \ubcf4\ud638\ud558\uace0, \uc720\ub7fd\uc744 \ud5a5\ud55c \uc624\uc2a4\ub9cc\uc758 \ud33d\ucc3d\uc744 \uc800\uc9c0\ud558\uc600\ub2e4. \uc624\uc9c1 \ub178\ub97c \uc813\ub294 \uc804\ud568\ub4e4\ub9cc\uc73c\ub85c \uce58\ub7ec\uc9c4 \uc774 \ub9c8\uc9c0\ub9c9 \uc911\uc694\ud55c \ud574\uc0c1 \uc804\ud22c\ub294 \uc138\uacc4\uc801\uc73c\ub85c \uc720\uba85\ud55c \uacb0\uc815\uc801\uc778 \uc804\ud22c \uac00\uc6b4\ub370 \ud558\ub098\ub85c \u201c\ub808\ud310\ud1a0 \uc804\ud22c \uc774\ud6c4 \uc138\uacc4\ub97c \uc6c0\uc9c1\uc774\ub294 \ucd94\ub294 \ub2e4\ub978 \ucabd\uc73c\ub85c \ud754\ub4e4\ub9ac\uae30 \uc2dc\uc791\ud574, \ubd80\uc720\ud568\uc740 \ub3d9\ucabd\uc5d0\uc11c \uc11c\ucabd\uc73c\ub85c \uc774\ub3d9\ud574\uac00\uc11c, \uc624\ub298\ub0a0\uae4c\uc9c0 \uacc4\uc18d\ub418\ub294 \uc138\uacc4\uc758 \ud328\ud134\uc744 \uac16\ucd94\uac8c \ub418\uc5c8\ub2e4.\u201d \ub354\ubd88\uc5b4 \u201c\uc911\ub3d9\uacfc \uc720\ub7fd \uc0ac\uc774\uc758 \uacc4\uc18d\ub41c \ub2e4\ud23c\uc758 \uc804\ud658\uc810\uc774\uae30\ub3c4 \ud588\uc73c\uba70, \uc774\ub294 \uc544\uc9c1\uae4c\uc9c0\ub3c4 \uc644\ubcbd\ud558\uac8c \ud574\uc18c\ub418\uc5b4\uc788\uc9c0 \uc54a\ub2e4.\u201d", "answer": "\ub808\ud310\ud1a0 \uc804\ud22c", "sentence": "\uc624\uc9c1 \ub178\ub97c \uc813\ub294 \uc804\ud568\ub4e4\ub9cc\uc73c\ub85c \uce58\ub7ec\uc9c4 \uc774 \ub9c8\uc9c0\ub9c9 \uc911\uc694\ud55c \ud574\uc0c1 \uc804\ud22c\ub294 \uc138\uacc4\uc801\uc73c\ub85c \uc720\uba85\ud55c \uacb0\uc815\uc801\uc778 \uc804\ud22c \uac00\uc6b4\ub370 \ud558\ub098\ub85c \u201c \ub808\ud310\ud1a0 \uc804\ud22c \uc774\ud6c4 \uc138\uacc4\ub97c \uc6c0\uc9c1\uc774\ub294 \ucd94\ub294 \ub2e4\ub978 \ucabd\uc73c\ub85c \ud754\ub4e4\ub9ac\uae30 \uc2dc\uc791\ud574, \ubd80\uc720\ud568\uc740 \ub3d9\ucabd\uc5d0\uc11c \uc11c\ucabd\uc73c\ub85c \uc774\ub3d9\ud574\uac00\uc11c, \uc624\ub298\ub0a0\uae4c\uc9c0 \uacc4\uc18d\ub418\ub294 \uc138\uacc4\uc758 \ud328\ud134\uc744 \uac16\ucd94\uac8c \ub418\uc5c8\ub2e4.", "paragraph_sentence": "10\uc6d4 7\uc77c \uc77c\uc694\uc77c \uc544\uce68, \ub808\ud310\ud1a0 \ud574\uad70 \uae30\uc9c0\uc5d0\uc11c \uc11c\ucabd\uc73c\ub85c \ucd9c\ud56d\ud55c \uc624\uc2a4\ub9cc \ud568\ub300\uc640 \uba54\uc2dc\ub098\uc5d0\uc11c \ucd9c\ud56d\ud55c \uc2e0\uc131 \ub3d9\ub9f9\uc758 \ud568\ub300\uac00 \uadf8\ub9ac\uc2a4 \uc11c\ubd80\uc758 \ubc14\uae65\ucabd\uc5d0 \uc788\ub294 \ud30c\ud2b8\ub77c\uc2a4 \ub9cc\uc758 \ub044\ud2b8\uba38\ub9ac\uc5d0\uc11c \ub9cc\ub098\uba74\uc11c 5\uc2dc\uac04\uc5d0 \uac78\uce5c \uc804\ud22c\uac00 \uc2dc\uc791\ub418\uc5c8\ub2e4. \uc804\ud22c \uc774\ud6c4 \uc2e0\uc131 \ub3d9\ub9f9\uc740 \uc77c\uc2dc\uc801\uc73c\ub85c \uc9c0\uc911\ud574\uc758 \ud328\uad8c\uc744 \uc7a5\uc545\ud558\uc600\uc73c\uba70, \ub85c\ub9c8\ub97c \uc624\uc2a4\ub9cc\uc758 \uce68\ub7b5\uc73c\ub85c\ubd80\ud130 \ubcf4\ud638\ud558\uace0, \uc720\ub7fd\uc744 \ud5a5\ud55c \uc624\uc2a4\ub9cc\uc758 \ud33d\ucc3d\uc744 \uc800\uc9c0\ud558\uc600\ub2e4. \uc624\uc9c1 \ub178\ub97c \uc813\ub294 \uc804\ud568\ub4e4\ub9cc\uc73c\ub85c \uce58\ub7ec\uc9c4 \uc774 \ub9c8\uc9c0\ub9c9 \uc911\uc694\ud55c \ud574\uc0c1 \uc804\ud22c\ub294 \uc138\uacc4\uc801\uc73c\ub85c \uc720\uba85\ud55c \uacb0\uc815\uc801\uc778 \uc804\ud22c \uac00\uc6b4\ub370 \ud558\ub098\ub85c \u201c \ub808\ud310\ud1a0 \uc804\ud22c \uc774\ud6c4 \uc138\uacc4\ub97c \uc6c0\uc9c1\uc774\ub294 \ucd94\ub294 \ub2e4\ub978 \ucabd\uc73c\ub85c \ud754\ub4e4\ub9ac\uae30 \uc2dc\uc791\ud574, \ubd80\uc720\ud568\uc740 \ub3d9\ucabd\uc5d0\uc11c \uc11c\ucabd\uc73c\ub85c \uc774\ub3d9\ud574\uac00\uc11c, \uc624\ub298\ub0a0\uae4c\uc9c0 \uacc4\uc18d\ub418\ub294 \uc138\uacc4\uc758 \ud328\ud134\uc744 \uac16\ucd94\uac8c \ub418\uc5c8\ub2e4. \u201d \ub354\ubd88\uc5b4 \u201c\uc911\ub3d9\uacfc \uc720\ub7fd \uc0ac\uc774\uc758 \uacc4\uc18d\ub41c \ub2e4\ud23c\uc758 \uc804\ud658\uc810\uc774\uae30\ub3c4 \ud588\uc73c\uba70, \uc774\ub294 \uc544\uc9c1\uae4c\uc9c0\ub3c4 \uc644\ubcbd\ud558\uac8c \ud574\uc18c\ub418\uc5b4\uc788\uc9c0 \uc54a\ub2e4. \u201d", "paragraph_answer": "10\uc6d4 7\uc77c \uc77c\uc694\uc77c \uc544\uce68, \ub808\ud310\ud1a0 \ud574\uad70 \uae30\uc9c0\uc5d0\uc11c \uc11c\ucabd\uc73c\ub85c \ucd9c\ud56d\ud55c \uc624\uc2a4\ub9cc \ud568\ub300\uc640 \uba54\uc2dc\ub098\uc5d0\uc11c \ucd9c\ud56d\ud55c \uc2e0\uc131 \ub3d9\ub9f9\uc758 \ud568\ub300\uac00 \uadf8\ub9ac\uc2a4 \uc11c\ubd80\uc758 \ubc14\uae65\ucabd\uc5d0 \uc788\ub294 \ud30c\ud2b8\ub77c\uc2a4 \ub9cc\uc758 \ub044\ud2b8\uba38\ub9ac\uc5d0\uc11c \ub9cc\ub098\uba74\uc11c 5\uc2dc\uac04\uc5d0 \uac78\uce5c \uc804\ud22c\uac00 \uc2dc\uc791\ub418\uc5c8\ub2e4. \uc804\ud22c \uc774\ud6c4 \uc2e0\uc131 \ub3d9\ub9f9\uc740 \uc77c\uc2dc\uc801\uc73c\ub85c \uc9c0\uc911\ud574\uc758 \ud328\uad8c\uc744 \uc7a5\uc545\ud558\uc600\uc73c\uba70, \ub85c\ub9c8\ub97c \uc624\uc2a4\ub9cc\uc758 \uce68\ub7b5\uc73c\ub85c\ubd80\ud130 \ubcf4\ud638\ud558\uace0, \uc720\ub7fd\uc744 \ud5a5\ud55c \uc624\uc2a4\ub9cc\uc758 \ud33d\ucc3d\uc744 \uc800\uc9c0\ud558\uc600\ub2e4. \uc624\uc9c1 \ub178\ub97c \uc813\ub294 \uc804\ud568\ub4e4\ub9cc\uc73c\ub85c \uce58\ub7ec\uc9c4 \uc774 \ub9c8\uc9c0\ub9c9 \uc911\uc694\ud55c \ud574\uc0c1 \uc804\ud22c\ub294 \uc138\uacc4\uc801\uc73c\ub85c \uc720\uba85\ud55c \uacb0\uc815\uc801\uc778 \uc804\ud22c \uac00\uc6b4\ub370 \ud558\ub098\ub85c \u201c \ub808\ud310\ud1a0 \uc804\ud22c \uc774\ud6c4 \uc138\uacc4\ub97c \uc6c0\uc9c1\uc774\ub294 \ucd94\ub294 \ub2e4\ub978 \ucabd\uc73c\ub85c \ud754\ub4e4\ub9ac\uae30 \uc2dc\uc791\ud574, \ubd80\uc720\ud568\uc740 \ub3d9\ucabd\uc5d0\uc11c \uc11c\ucabd\uc73c\ub85c \uc774\ub3d9\ud574\uac00\uc11c, \uc624\ub298\ub0a0\uae4c\uc9c0 \uacc4\uc18d\ub418\ub294 \uc138\uacc4\uc758 \ud328\ud134\uc744 \uac16\ucd94\uac8c \ub418\uc5c8\ub2e4.\u201d \ub354\ubd88\uc5b4 \u201c\uc911\ub3d9\uacfc \uc720\ub7fd \uc0ac\uc774\uc758 \uacc4\uc18d\ub41c \ub2e4\ud23c\uc758 \uc804\ud658\uc810\uc774\uae30\ub3c4 \ud588\uc73c\uba70, \uc774\ub294 \uc544\uc9c1\uae4c\uc9c0\ub3c4 \uc644\ubcbd\ud558\uac8c \ud574\uc18c\ub418\uc5b4\uc788\uc9c0 \uc54a\ub2e4.\u201d", "sentence_answer": "\uc624\uc9c1 \ub178\ub97c \uc813\ub294 \uc804\ud568\ub4e4\ub9cc\uc73c\ub85c \uce58\ub7ec\uc9c4 \uc774 \ub9c8\uc9c0\ub9c9 \uc911\uc694\ud55c \ud574\uc0c1 \uc804\ud22c\ub294 \uc138\uacc4\uc801\uc73c\ub85c \uc720\uba85\ud55c \uacb0\uc815\uc801\uc778 \uc804\ud22c \uac00\uc6b4\ub370 \ud558\ub098\ub85c \u201c \ub808\ud310\ud1a0 \uc804\ud22c \uc774\ud6c4 \uc138\uacc4\ub97c \uc6c0\uc9c1\uc774\ub294 \ucd94\ub294 \ub2e4\ub978 \ucabd\uc73c\ub85c \ud754\ub4e4\ub9ac\uae30 \uc2dc\uc791\ud574, \ubd80\uc720\ud568\uc740 \ub3d9\ucabd\uc5d0\uc11c \uc11c\ucabd\uc73c\ub85c \uc774\ub3d9\ud574\uac00\uc11c, \uc624\ub298\ub0a0\uae4c\uc9c0 \uacc4\uc18d\ub418\ub294 \uc138\uacc4\uc758 \ud328\ud134\uc744 \uac16\ucd94\uac8c \ub418\uc5c8\ub2e4.", "paragraph_id": "6495950-0-0", "paragraph_question": "question: \uc911\ub3d9\uacfc \uc720\ub7fd \uc0ac\uc774\uc758 \uacc4\uc18d\ub41c \ub2e4\ud23c\uc758 \uc804\ud658\uc810\uc774 \ub41c \uc804\ud22c\ub294?, context: 10\uc6d4 7\uc77c \uc77c\uc694\uc77c \uc544\uce68, \ub808\ud310\ud1a0 \ud574\uad70 \uae30\uc9c0\uc5d0\uc11c \uc11c\ucabd\uc73c\ub85c \ucd9c\ud56d\ud55c \uc624\uc2a4\ub9cc \ud568\ub300\uc640 \uba54\uc2dc\ub098\uc5d0\uc11c \ucd9c\ud56d\ud55c \uc2e0\uc131 \ub3d9\ub9f9\uc758 \ud568\ub300\uac00 \uadf8\ub9ac\uc2a4 \uc11c\ubd80\uc758 \ubc14\uae65\ucabd\uc5d0 \uc788\ub294 \ud30c\ud2b8\ub77c\uc2a4 \ub9cc\uc758 \ub044\ud2b8\uba38\ub9ac\uc5d0\uc11c \ub9cc\ub098\uba74\uc11c 5\uc2dc\uac04\uc5d0 \uac78\uce5c \uc804\ud22c\uac00 \uc2dc\uc791\ub418\uc5c8\ub2e4. \uc804\ud22c \uc774\ud6c4 \uc2e0\uc131 \ub3d9\ub9f9\uc740 \uc77c\uc2dc\uc801\uc73c\ub85c \uc9c0\uc911\ud574\uc758 \ud328\uad8c\uc744 \uc7a5\uc545\ud558\uc600\uc73c\uba70, \ub85c\ub9c8\ub97c \uc624\uc2a4\ub9cc\uc758 \uce68\ub7b5\uc73c\ub85c\ubd80\ud130 \ubcf4\ud638\ud558\uace0, \uc720\ub7fd\uc744 \ud5a5\ud55c \uc624\uc2a4\ub9cc\uc758 \ud33d\ucc3d\uc744 \uc800\uc9c0\ud558\uc600\ub2e4. \uc624\uc9c1 \ub178\ub97c \uc813\ub294 \uc804\ud568\ub4e4\ub9cc\uc73c\ub85c \uce58\ub7ec\uc9c4 \uc774 \ub9c8\uc9c0\ub9c9 \uc911\uc694\ud55c \ud574\uc0c1 \uc804\ud22c\ub294 \uc138\uacc4\uc801\uc73c\ub85c \uc720\uba85\ud55c \uacb0\uc815\uc801\uc778 \uc804\ud22c \uac00\uc6b4\ub370 \ud558\ub098\ub85c \u201c\ub808\ud310\ud1a0 \uc804\ud22c \uc774\ud6c4 \uc138\uacc4\ub97c \uc6c0\uc9c1\uc774\ub294 \ucd94\ub294 \ub2e4\ub978 \ucabd\uc73c\ub85c \ud754\ub4e4\ub9ac\uae30 \uc2dc\uc791\ud574, \ubd80\uc720\ud568\uc740 \ub3d9\ucabd\uc5d0\uc11c \uc11c\ucabd\uc73c\ub85c \uc774\ub3d9\ud574\uac00\uc11c, \uc624\ub298\ub0a0\uae4c\uc9c0 \uacc4\uc18d\ub418\ub294 \uc138\uacc4\uc758 \ud328\ud134\uc744 \uac16\ucd94\uac8c \ub418\uc5c8\ub2e4.\u201d \ub354\ubd88\uc5b4 \u201c\uc911\ub3d9\uacfc \uc720\ub7fd \uc0ac\uc774\uc758 \uacc4\uc18d\ub41c \ub2e4\ud23c\uc758 \uc804\ud658\uc810\uc774\uae30\ub3c4 \ud588\uc73c\uba70, \uc774\ub294 \uc544\uc9c1\uae4c\uc9c0\ub3c4 \uc644\ubcbd\ud558\uac8c \ud574\uc18c\ub418\uc5b4\uc788\uc9c0 \uc54a\ub2e4.\u201d"} {"question": "\ubba4\uc9c0\uceec \ube4c\ub9ac \uc5d8\ub9ac\uc5b4\ud2b8\uc758 \uc81c\uc791\uc0ac\ub294 \ubb34\uc5c7\uc778\uac00?", "paragraph": "\ubba4\uc9c0\uceec \ube4c\ub9ac \uc5d8\ub9ac\uc5b4\ud2b8\ub294 \uc601\uad6d \uc6e8\uc2a4\ud2b8\uc5d4\ub4dc \uc18c\uc7ac\uc758 \ube45\ud1a0\ub9ac\uc544 \ud330\ub9ac\uc2a4 \uc2dc\uc5b4\ud130\uc5d0\uc11c 2005\ub144 3\uc6d4 31\uc77c \ud504\ub9ac\ubdf0\uc640 \ud568\uaed8 2005\ub144 5\uc6d4 11\uc77c \uc815\uc2dd \ucd08\uc5f0\ub418\uc5b4 \uc544\uc9c1\uae4c\uc9c0 \uacf5\uc5f0\uc911\uc774\ub2e4. \uc804\ud558\ub294 \ubc14\uc5d0 \ub530\ub974\uba74 550\ub9cc \ud30c\uc6b4\ub4dc(\ud55c\ud654 \uc57d 100\uc5b5)\uc5d0 \ub2ec\ud558\ub294 \uc218\uc775\uc744 \uc62c\ub838\ub2e4. \uc6d0\uc791\uacfc \uac19\uc774 \uc81c\uc791\uc0ac\ub294 \uc6cc\ud0b9 \ud0c0\uc774\ud2c0\uc774\uba70, \uc2a4\ud2f0\ube10 \ub2ec\ub4dc\ub9ac\uac00 \uc5f0\ucd9c\uc744 \ub9e1\uc558\uace0, \ud53c\ub354 \ub2ec\ub9c1\uc774 \uc548\ubb34\ub97c \ub2f4\ub2f9\ud588\ub2e4. \ub9ac\uc554 \ubaa8\uc5b4, \uc81c\uc784\uc2a4 \ub85c\ub9c8\uc2a4, \uc870\uc9c0 \ub9e5\uacfc\uc774\uc5b4\uac00 \ube4c\ub9ac \uc5d8\ub9ac\uc5b4\ud2b8\ub97c, \ud558\uc774\ub4e0 \uadf8\uc708\uacfc \ud300 \ud790\uc774 \uac01\uac01 \uc70c\ud0a8\uc2a8 \ubd80\uc778\uacfc \ube4c\ub9ac \uc544\ube60 \uc5ed\uc744 \uc5f0\uae30\ud588\ub2e4. \uc138\ud2b8\ub294 \uc774\uc548 \ub9e5\ub2d0\uc774 \ub514\uc790\uc778\ud588\uace0 \uc758\uc0c1\uc740 \ub2c8\ud0a4 \uae00\ub9ac\ube14\ub79c\ub4dc\uac00 \ub9e1\uc558\ub2e4. \uc870\uba85\uc740 \ub9ad \ud53c\uc154, \uc0ac\uc6b4\ub4dc\ub294 \ud3f4 \uc544\ub974\ub514\ub760\uac00 \ub9e1\uc558\ub2e4. \uc624\ub9ac\uc9c0\ub110 \uce90\uc2a4\ud2b8 \uc568\ubc94\uc740 2006\ub144 1\uc6d4 10\uc77c\uc5d0 \ubc1c\ub9e4\ub410\ub2e4.", "answer": "\uc6cc\ud0b9 \ud0c0\uc774\ud2c0", "sentence": "\uc6d0\uc791\uacfc \uac19\uc774 \uc81c\uc791\uc0ac\ub294 \uc6cc\ud0b9 \ud0c0\uc774\ud2c0 \uc774\uba70, \uc2a4\ud2f0\ube10 \ub2ec\ub4dc\ub9ac\uac00 \uc5f0\ucd9c\uc744 \ub9e1\uc558\uace0, \ud53c\ub354 \ub2ec\ub9c1\uc774 \uc548\ubb34\ub97c \ub2f4\ub2f9\ud588\ub2e4.", "paragraph_sentence": "\ubba4\uc9c0\uceec \ube4c\ub9ac \uc5d8\ub9ac\uc5b4\ud2b8\ub294 \uc601\uad6d \uc6e8\uc2a4\ud2b8\uc5d4\ub4dc \uc18c\uc7ac\uc758 \ube45\ud1a0\ub9ac\uc544 \ud330\ub9ac\uc2a4 \uc2dc\uc5b4\ud130\uc5d0\uc11c 2005\ub144 3\uc6d4 31\uc77c \ud504\ub9ac\ubdf0\uc640 \ud568\uaed8 2005\ub144 5\uc6d4 11\uc77c \uc815\uc2dd \ucd08\uc5f0\ub418\uc5b4 \uc544\uc9c1\uae4c\uc9c0 \uacf5\uc5f0\uc911\uc774\ub2e4. \uc804\ud558\ub294 \ubc14\uc5d0 \ub530\ub974\uba74 550\ub9cc \ud30c\uc6b4\ub4dc(\ud55c\ud654 \uc57d 100\uc5b5)\uc5d0 \ub2ec\ud558\ub294 \uc218\uc775\uc744 \uc62c\ub838\ub2e4. \uc6d0\uc791\uacfc \uac19\uc774 \uc81c\uc791\uc0ac\ub294 \uc6cc\ud0b9 \ud0c0\uc774\ud2c0 \uc774\uba70, \uc2a4\ud2f0\ube10 \ub2ec\ub4dc\ub9ac\uac00 \uc5f0\ucd9c\uc744 \ub9e1\uc558\uace0, \ud53c\ub354 \ub2ec\ub9c1\uc774 \uc548\ubb34\ub97c \ub2f4\ub2f9\ud588\ub2e4. \ub9ac\uc554 \ubaa8\uc5b4, \uc81c\uc784\uc2a4 \ub85c\ub9c8\uc2a4, \uc870\uc9c0 \ub9e5\uacfc\uc774\uc5b4\uac00 \ube4c\ub9ac \uc5d8\ub9ac\uc5b4\ud2b8\ub97c, \ud558\uc774\ub4e0 \uadf8\uc708\uacfc \ud300 \ud790\uc774 \uac01\uac01 \uc70c\ud0a8\uc2a8 \ubd80\uc778\uacfc \ube4c\ub9ac \uc544\ube60 \uc5ed\uc744 \uc5f0\uae30\ud588\ub2e4. \uc138\ud2b8\ub294 \uc774\uc548 \ub9e5\ub2d0\uc774 \ub514\uc790\uc778\ud588\uace0 \uc758\uc0c1\uc740 \ub2c8\ud0a4 \uae00\ub9ac\ube14\ub79c\ub4dc\uac00 \ub9e1\uc558\ub2e4. \uc870\uba85\uc740 \ub9ad \ud53c\uc154, \uc0ac\uc6b4\ub4dc\ub294 \ud3f4 \uc544\ub974\ub514\ub760\uac00 \ub9e1\uc558\ub2e4. \uc624\ub9ac\uc9c0\ub110 \uce90\uc2a4\ud2b8 \uc568\ubc94\uc740 2006\ub144 1\uc6d4 10\uc77c\uc5d0 \ubc1c\ub9e4\ub410\ub2e4.", "paragraph_answer": "\ubba4\uc9c0\uceec \ube4c\ub9ac \uc5d8\ub9ac\uc5b4\ud2b8\ub294 \uc601\uad6d \uc6e8\uc2a4\ud2b8\uc5d4\ub4dc \uc18c\uc7ac\uc758 \ube45\ud1a0\ub9ac\uc544 \ud330\ub9ac\uc2a4 \uc2dc\uc5b4\ud130\uc5d0\uc11c 2005\ub144 3\uc6d4 31\uc77c \ud504\ub9ac\ubdf0\uc640 \ud568\uaed8 2005\ub144 5\uc6d4 11\uc77c \uc815\uc2dd \ucd08\uc5f0\ub418\uc5b4 \uc544\uc9c1\uae4c\uc9c0 \uacf5\uc5f0\uc911\uc774\ub2e4. \uc804\ud558\ub294 \ubc14\uc5d0 \ub530\ub974\uba74 550\ub9cc \ud30c\uc6b4\ub4dc(\ud55c\ud654 \uc57d 100\uc5b5)\uc5d0 \ub2ec\ud558\ub294 \uc218\uc775\uc744 \uc62c\ub838\ub2e4. \uc6d0\uc791\uacfc \uac19\uc774 \uc81c\uc791\uc0ac\ub294 \uc6cc\ud0b9 \ud0c0\uc774\ud2c0 \uc774\uba70, \uc2a4\ud2f0\ube10 \ub2ec\ub4dc\ub9ac\uac00 \uc5f0\ucd9c\uc744 \ub9e1\uc558\uace0, \ud53c\ub354 \ub2ec\ub9c1\uc774 \uc548\ubb34\ub97c \ub2f4\ub2f9\ud588\ub2e4. \ub9ac\uc554 \ubaa8\uc5b4, \uc81c\uc784\uc2a4 \ub85c\ub9c8\uc2a4, \uc870\uc9c0 \ub9e5\uacfc\uc774\uc5b4\uac00 \ube4c\ub9ac \uc5d8\ub9ac\uc5b4\ud2b8\ub97c, \ud558\uc774\ub4e0 \uadf8\uc708\uacfc \ud300 \ud790\uc774 \uac01\uac01 \uc70c\ud0a8\uc2a8 \ubd80\uc778\uacfc \ube4c\ub9ac \uc544\ube60 \uc5ed\uc744 \uc5f0\uae30\ud588\ub2e4. \uc138\ud2b8\ub294 \uc774\uc548 \ub9e5\ub2d0\uc774 \ub514\uc790\uc778\ud588\uace0 \uc758\uc0c1\uc740 \ub2c8\ud0a4 \uae00\ub9ac\ube14\ub79c\ub4dc\uac00 \ub9e1\uc558\ub2e4. \uc870\uba85\uc740 \ub9ad \ud53c\uc154, \uc0ac\uc6b4\ub4dc\ub294 \ud3f4 \uc544\ub974\ub514\ub760\uac00 \ub9e1\uc558\ub2e4. \uc624\ub9ac\uc9c0\ub110 \uce90\uc2a4\ud2b8 \uc568\ubc94\uc740 2006\ub144 1\uc6d4 10\uc77c\uc5d0 \ubc1c\ub9e4\ub410\ub2e4.", "sentence_answer": "\uc6d0\uc791\uacfc \uac19\uc774 \uc81c\uc791\uc0ac\ub294 \uc6cc\ud0b9 \ud0c0\uc774\ud2c0 \uc774\uba70, \uc2a4\ud2f0\ube10 \ub2ec\ub4dc\ub9ac\uac00 \uc5f0\ucd9c\uc744 \ub9e1\uc558\uace0, \ud53c\ub354 \ub2ec\ub9c1\uc774 \uc548\ubb34\ub97c \ub2f4\ub2f9\ud588\ub2e4.", "paragraph_id": "6531194-0-1", "paragraph_question": "question: \ubba4\uc9c0\uceec \ube4c\ub9ac \uc5d8\ub9ac\uc5b4\ud2b8\uc758 \uc81c\uc791\uc0ac\ub294 \ubb34\uc5c7\uc778\uac00?, context: \ubba4\uc9c0\uceec \ube4c\ub9ac \uc5d8\ub9ac\uc5b4\ud2b8\ub294 \uc601\uad6d \uc6e8\uc2a4\ud2b8\uc5d4\ub4dc \uc18c\uc7ac\uc758 \ube45\ud1a0\ub9ac\uc544 \ud330\ub9ac\uc2a4 \uc2dc\uc5b4\ud130\uc5d0\uc11c 2005\ub144 3\uc6d4 31\uc77c \ud504\ub9ac\ubdf0\uc640 \ud568\uaed8 2005\ub144 5\uc6d4 11\uc77c \uc815\uc2dd \ucd08\uc5f0\ub418\uc5b4 \uc544\uc9c1\uae4c\uc9c0 \uacf5\uc5f0\uc911\uc774\ub2e4. \uc804\ud558\ub294 \ubc14\uc5d0 \ub530\ub974\uba74 550\ub9cc \ud30c\uc6b4\ub4dc(\ud55c\ud654 \uc57d 100\uc5b5)\uc5d0 \ub2ec\ud558\ub294 \uc218\uc775\uc744 \uc62c\ub838\ub2e4. \uc6d0\uc791\uacfc \uac19\uc774 \uc81c\uc791\uc0ac\ub294 \uc6cc\ud0b9 \ud0c0\uc774\ud2c0\uc774\uba70, \uc2a4\ud2f0\ube10 \ub2ec\ub4dc\ub9ac\uac00 \uc5f0\ucd9c\uc744 \ub9e1\uc558\uace0, \ud53c\ub354 \ub2ec\ub9c1\uc774 \uc548\ubb34\ub97c \ub2f4\ub2f9\ud588\ub2e4. \ub9ac\uc554 \ubaa8\uc5b4, \uc81c\uc784\uc2a4 \ub85c\ub9c8\uc2a4, \uc870\uc9c0 \ub9e5\uacfc\uc774\uc5b4\uac00 \ube4c\ub9ac \uc5d8\ub9ac\uc5b4\ud2b8\ub97c, \ud558\uc774\ub4e0 \uadf8\uc708\uacfc \ud300 \ud790\uc774 \uac01\uac01 \uc70c\ud0a8\uc2a8 \ubd80\uc778\uacfc \ube4c\ub9ac \uc544\ube60 \uc5ed\uc744 \uc5f0\uae30\ud588\ub2e4. \uc138\ud2b8\ub294 \uc774\uc548 \ub9e5\ub2d0\uc774 \ub514\uc790\uc778\ud588\uace0 \uc758\uc0c1\uc740 \ub2c8\ud0a4 \uae00\ub9ac\ube14\ub79c\ub4dc\uac00 \ub9e1\uc558\ub2e4. \uc870\uba85\uc740 \ub9ad \ud53c\uc154, \uc0ac\uc6b4\ub4dc\ub294 \ud3f4 \uc544\ub974\ub514\ub760\uac00 \ub9e1\uc558\ub2e4. \uc624\ub9ac\uc9c0\ub110 \uce90\uc2a4\ud2b8 \uc568\ubc94\uc740 2006\ub144 1\uc6d4 10\uc77c\uc5d0 \ubc1c\ub9e4\ub410\ub2e4."} {"question": "\ud14d\uc0ac\uc2a4 \uc8fc\uc758 \uc120\uac70\uc778\uc774 \ub300\ud1b5\ub839\uc73c\ub85c \ub204\uad6c\ub97c \ud22c\ud45c\ud558\uc600\ub294\uc9c0 \uc54c \uc218 \uc5c6\ub294 \uc774\uc720\ub294?", "paragraph": "\uc6cc\uc2f1\ud134 \uc8fc\uc758 \uc120\uac70\uc778 \uc138 \uba85\uc740 \uc120\ucd9c\ud558\uae30\ub85c \ud55c \ud790\ub7ec\ub9ac \ud074\ub9b0\ud134 \ub300\uc2e0 \ucf5c\ub9b0 \ud30c\uc6d4 \uc804 \uc7a5\uad00\uc744 \ub300\ud1b5\ub839\uc73c\ub85c \ud22c\ud45c\ud558\uc600\ub2e4. \uc774\ub4e4\uc740 \uac01\uac01 \uc218\uc804 \ucf5c\ub9b0\uc2a4, \ub9c8\ub9ac\uc544 \uce94\ud2b8\uc6f0, \uc5d8\ub9ac\uc790\ubca0\uc2a4 \uc6cc\ub7f0\uc744 \ubd80\ud1b5\ub839\uc73c\ub85c \ud22c\ud45c\ud558\uc600\ub2e4. \ud558\uc640\uc774 \uc8fc\uc758 \ud55c \uc120\uac70\uc778\uc740 \uc120\ucd9c\ud558\uae30\ub85c \ud55c \ud790\ub7ec\ub9ac \ud074\ub9b0\ud134 \ub300\uc2e0 \ubc84\ub2c8 \uc0cc\ub354\uc2a4\ub97c \ub300\ud1b5\ub839\uc73c\ub85c \uc5d8\ub9ac\uc790\ubca0\uc2a4 \uc6cc\ub7f0\uc744 \ubd80\ud1b5\ub839\uc73c\ub85c \ud22c\ud45c\ud558\uc600\ub2e4. \uc6cc\uc2f1\ud134 \uc8fc\uc758 \ub610\ub2e4\ub978 \ud55c \uba85\uc740 \uc120\ucd9c\ud558\uae30\ub85c \ud55c \ud790\ub7ec\ub9ac \ud074\ub9b0\ud134 \ub300\uc2e0 \ubbf8\uad6d \uc6d0\uc8fc\ubbfc \ud658\uacbd\uc6b4\ub3d9\uac00 \ud398\uc774\uc2a4 \uc2a4\ud30c\ud2f0\ub4dc \uc774\uae00\uc744 \ub300\ud1b5\ub839\uc73c\ub85c, \uc6d0\uc8fc\ubbfc\uacfc \ubc31\uc778\uc758 \ud63c\ud608\uc778 \ud658\uacbd\uc6b4\ub3d9\uac00 \uc704\ub178\uc544 \ub77c\ub450\ud06c\ub97c \ubd80\ud1b5\ub839\uc73c\ub85c \ud22c\ud45c\ud558\uc600\ub2e4. \ud14d\uc0ac\uc2a4 \uc8fc\uc758 \ud55c \uc120\uac70\uc778\uc740 \uc874 \ucf00\uc774\uc2dd\uc744, \ud55c \uc120\uac70\uc778\uc740 \ub860 \ud3f4\uc744 \ub300\ud1b5\ub839\uc73c\ub85c \ud22c\ud45c\ud558\uc600\uace0 \ud55c \uba85\uc740 \uce7c\ub9ac \ud53c\uc624\ub9ac\ub098\ub97c \ubd80\ud1b5\ub839\uc73c\ub85c \ud22c\ud45c\ud558\uc600\ub2e4. \ubd80\ud1b5\ub839\uc73c\ub85c \ud53c\uc624\ub9ac\ub098\ub97c \ucc0d\uc740 \ud14d\uc0ac\uc2a4 \uc8fc\uc758 \uc120\uac70\uc778\uc774 \ub300\ud1b5\ub839\uc73c\ub85c \ub204\uad6c\ub97c \ucc0d\uc5c8\ub294\uc9c0\ub294 \uc120\uac70 \uacb0\uacfc\ub97c \ud1b5\ud574\uc11c\ub294 \ud655\uc778\ub418\uc9c0 \uc54a\uc558\ub2e4. \ud14d\uc0ac\uc2a4 \uc8fc\uc758 \uc120\uac70\uc778\uc740 \ub300\ud1b5\ub839\uacfc \ubd80\ud1b5\ub839\uc744 \uac01\uac01 \ub2e4\ub978 \ud22c\ud45c\uc6a9\uc9c0\uc5d0 \uc801\uc5b4 \ube44\ubc00\ud22c\ud45c\ub97c \ud558\uae30 \ub54c\ubb38\uc774\ub2e4. \uadf8\ub7ec\ub098 \uc774\ud6c4 \ubc18\ub780\ud45c\ub97c \ub0b8 \uc120\uac70\uc778\ub4e4\uacfc\uc758 \uc778\ud130\ubdf0\ub97c \ud1b5\ud574 \ub300\ud1b5\ub839\uc73c\ub85c \ucf00\uc774\uc2dd\uc744 \uc120\ucd9c\ud55c \uc120\uac70\uc778\uc774 \ubd80\ud1b5\ub839\uc73c\ub85c \ud53c\uc624\ub9ac\ub098\ub97c \uc120\ucd9c\ud588\uace0, \ud3f4\uc744 \ub300\ud1b5\ub839\uc73c\ub85c \uc120\ucd9c\ud55c \uc120\uac70\uc778\uc740 \ubd80\ud1b5\ub839 \ud22c\ud45c\ub294 \uc790\uae30\uac00 \ud22c\ud45c\ud574\uc57c \ud560 \ub9c8\uc774\ud06c \ud39c\uc2a4\uc5d0\uac8c \ud22c\ud45c\ud55c \uac83\uc73c\ub85c \ubc1d\ud600\uc84c\ub2e4. \ucf5c\ub9b0 \ud30c\uc6d4\uacfc \ubc84\ub2c8 \uc0cc\ub354\uc2a4, \ucf00\uc774\uc2dd, \ud3f4\uc740 \uae30\uba85\ud6c4\ubcf4\ub85c \ub4f1\ub85d\ub418\uc5b4 \uc720\uad8c\uc790\uc758 \ud45c\ub97c \uc5bb\uae30\ub3c4 \ud588\ub294\ub370 \uc0cc\ub354\uc2a4\ub294 \uae30\uba85\ud22c\ud45c\uc5d0\uc11c 10\ub9cc \ud45c \uc774\uc0c1\uc744 \ub4dd\ud45c\ud588\ub2e4.", "answer": "\ube44\ubc00\ud22c\ud45c", "sentence": "\ud14d\uc0ac\uc2a4 \uc8fc\uc758 \uc120\uac70\uc778\uc740 \ub300\ud1b5\ub839\uacfc \ubd80\ud1b5\ub839\uc744 \uac01\uac01 \ub2e4\ub978 \ud22c\ud45c\uc6a9\uc9c0\uc5d0 \uc801\uc5b4 \ube44\ubc00\ud22c\ud45c \ub97c \ud558\uae30 \ub54c\ubb38\uc774\ub2e4.", "paragraph_sentence": "\uc6cc\uc2f1\ud134 \uc8fc\uc758 \uc120\uac70\uc778 \uc138 \uba85\uc740 \uc120\ucd9c\ud558\uae30\ub85c \ud55c \ud790\ub7ec\ub9ac \ud074\ub9b0\ud134 \ub300\uc2e0 \ucf5c\ub9b0 \ud30c\uc6d4 \uc804 \uc7a5\uad00\uc744 \ub300\ud1b5\ub839\uc73c\ub85c \ud22c\ud45c\ud558\uc600\ub2e4. \uc774\ub4e4\uc740 \uac01\uac01 \uc218\uc804 \ucf5c\ub9b0\uc2a4, \ub9c8\ub9ac\uc544 \uce94\ud2b8\uc6f0, \uc5d8\ub9ac\uc790\ubca0\uc2a4 \uc6cc\ub7f0\uc744 \ubd80\ud1b5\ub839\uc73c\ub85c \ud22c\ud45c\ud558\uc600\ub2e4. \ud558\uc640\uc774 \uc8fc\uc758 \ud55c \uc120\uac70\uc778\uc740 \uc120\ucd9c\ud558\uae30\ub85c \ud55c \ud790\ub7ec\ub9ac \ud074\ub9b0\ud134 \ub300\uc2e0 \ubc84\ub2c8 \uc0cc\ub354\uc2a4\ub97c \ub300\ud1b5\ub839\uc73c\ub85c \uc5d8\ub9ac\uc790\ubca0\uc2a4 \uc6cc\ub7f0\uc744 \ubd80\ud1b5\ub839\uc73c\ub85c \ud22c\ud45c\ud558\uc600\ub2e4. \uc6cc\uc2f1\ud134 \uc8fc\uc758 \ub610\ub2e4\ub978 \ud55c \uba85\uc740 \uc120\ucd9c\ud558\uae30\ub85c \ud55c \ud790\ub7ec\ub9ac \ud074\ub9b0\ud134 \ub300\uc2e0 \ubbf8\uad6d \uc6d0\uc8fc\ubbfc \ud658\uacbd\uc6b4\ub3d9\uac00 \ud398\uc774\uc2a4 \uc2a4\ud30c\ud2f0\ub4dc \uc774\uae00\uc744 \ub300\ud1b5\ub839\uc73c\ub85c, \uc6d0\uc8fc\ubbfc\uacfc \ubc31\uc778\uc758 \ud63c\ud608\uc778 \ud658\uacbd\uc6b4\ub3d9\uac00 \uc704\ub178\uc544 \ub77c\ub450\ud06c\ub97c \ubd80\ud1b5\ub839\uc73c\ub85c \ud22c\ud45c\ud558\uc600\ub2e4. \ud14d\uc0ac\uc2a4 \uc8fc\uc758 \ud55c \uc120\uac70\uc778\uc740 \uc874 \ucf00\uc774\uc2dd\uc744, \ud55c \uc120\uac70\uc778\uc740 \ub860 \ud3f4\uc744 \ub300\ud1b5\ub839\uc73c\ub85c \ud22c\ud45c\ud558\uc600\uace0 \ud55c \uba85\uc740 \uce7c\ub9ac \ud53c\uc624\ub9ac\ub098\ub97c \ubd80\ud1b5\ub839\uc73c\ub85c \ud22c\ud45c\ud558\uc600\ub2e4. \ubd80\ud1b5\ub839\uc73c\ub85c \ud53c\uc624\ub9ac\ub098\ub97c \ucc0d\uc740 \ud14d\uc0ac\uc2a4 \uc8fc\uc758 \uc120\uac70\uc778\uc774 \ub300\ud1b5\ub839\uc73c\ub85c \ub204\uad6c\ub97c \ucc0d\uc5c8\ub294\uc9c0\ub294 \uc120\uac70 \uacb0\uacfc\ub97c \ud1b5\ud574\uc11c\ub294 \ud655\uc778\ub418\uc9c0 \uc54a\uc558\ub2e4. \ud14d\uc0ac\uc2a4 \uc8fc\uc758 \uc120\uac70\uc778\uc740 \ub300\ud1b5\ub839\uacfc \ubd80\ud1b5\ub839\uc744 \uac01\uac01 \ub2e4\ub978 \ud22c\ud45c\uc6a9\uc9c0\uc5d0 \uc801\uc5b4 \ube44\ubc00\ud22c\ud45c \ub97c \ud558\uae30 \ub54c\ubb38\uc774\ub2e4. \uadf8\ub7ec\ub098 \uc774\ud6c4 \ubc18\ub780\ud45c\ub97c \ub0b8 \uc120\uac70\uc778\ub4e4\uacfc\uc758 \uc778\ud130\ubdf0\ub97c \ud1b5\ud574 \ub300\ud1b5\ub839\uc73c\ub85c \ucf00\uc774\uc2dd\uc744 \uc120\ucd9c\ud55c \uc120\uac70\uc778\uc774 \ubd80\ud1b5\ub839\uc73c\ub85c \ud53c\uc624\ub9ac\ub098\ub97c \uc120\ucd9c\ud588\uace0, \ud3f4\uc744 \ub300\ud1b5\ub839\uc73c\ub85c \uc120\ucd9c\ud55c \uc120\uac70\uc778\uc740 \ubd80\ud1b5\ub839 \ud22c\ud45c\ub294 \uc790\uae30\uac00 \ud22c\ud45c\ud574\uc57c \ud560 \ub9c8\uc774\ud06c \ud39c\uc2a4\uc5d0\uac8c \ud22c\ud45c\ud55c \uac83\uc73c\ub85c \ubc1d\ud600\uc84c\ub2e4. \ucf5c\ub9b0 \ud30c\uc6d4\uacfc \ubc84\ub2c8 \uc0cc\ub354\uc2a4, \ucf00\uc774\uc2dd, \ud3f4\uc740 \uae30\uba85\ud6c4\ubcf4\ub85c \ub4f1\ub85d\ub418\uc5b4 \uc720\uad8c\uc790\uc758 \ud45c\ub97c \uc5bb\uae30\ub3c4 \ud588\ub294\ub370 \uc0cc\ub354\uc2a4\ub294 \uae30\uba85\ud22c\ud45c\uc5d0\uc11c 10\ub9cc \ud45c \uc774\uc0c1\uc744 \ub4dd\ud45c\ud588\ub2e4.", "paragraph_answer": "\uc6cc\uc2f1\ud134 \uc8fc\uc758 \uc120\uac70\uc778 \uc138 \uba85\uc740 \uc120\ucd9c\ud558\uae30\ub85c \ud55c \ud790\ub7ec\ub9ac \ud074\ub9b0\ud134 \ub300\uc2e0 \ucf5c\ub9b0 \ud30c\uc6d4 \uc804 \uc7a5\uad00\uc744 \ub300\ud1b5\ub839\uc73c\ub85c \ud22c\ud45c\ud558\uc600\ub2e4. \uc774\ub4e4\uc740 \uac01\uac01 \uc218\uc804 \ucf5c\ub9b0\uc2a4, \ub9c8\ub9ac\uc544 \uce94\ud2b8\uc6f0, \uc5d8\ub9ac\uc790\ubca0\uc2a4 \uc6cc\ub7f0\uc744 \ubd80\ud1b5\ub839\uc73c\ub85c \ud22c\ud45c\ud558\uc600\ub2e4. \ud558\uc640\uc774 \uc8fc\uc758 \ud55c \uc120\uac70\uc778\uc740 \uc120\ucd9c\ud558\uae30\ub85c \ud55c \ud790\ub7ec\ub9ac \ud074\ub9b0\ud134 \ub300\uc2e0 \ubc84\ub2c8 \uc0cc\ub354\uc2a4\ub97c \ub300\ud1b5\ub839\uc73c\ub85c \uc5d8\ub9ac\uc790\ubca0\uc2a4 \uc6cc\ub7f0\uc744 \ubd80\ud1b5\ub839\uc73c\ub85c \ud22c\ud45c\ud558\uc600\ub2e4. \uc6cc\uc2f1\ud134 \uc8fc\uc758 \ub610\ub2e4\ub978 \ud55c \uba85\uc740 \uc120\ucd9c\ud558\uae30\ub85c \ud55c \ud790\ub7ec\ub9ac \ud074\ub9b0\ud134 \ub300\uc2e0 \ubbf8\uad6d \uc6d0\uc8fc\ubbfc \ud658\uacbd\uc6b4\ub3d9\uac00 \ud398\uc774\uc2a4 \uc2a4\ud30c\ud2f0\ub4dc \uc774\uae00\uc744 \ub300\ud1b5\ub839\uc73c\ub85c, \uc6d0\uc8fc\ubbfc\uacfc \ubc31\uc778\uc758 \ud63c\ud608\uc778 \ud658\uacbd\uc6b4\ub3d9\uac00 \uc704\ub178\uc544 \ub77c\ub450\ud06c\ub97c \ubd80\ud1b5\ub839\uc73c\ub85c \ud22c\ud45c\ud558\uc600\ub2e4. \ud14d\uc0ac\uc2a4 \uc8fc\uc758 \ud55c \uc120\uac70\uc778\uc740 \uc874 \ucf00\uc774\uc2dd\uc744, \ud55c \uc120\uac70\uc778\uc740 \ub860 \ud3f4\uc744 \ub300\ud1b5\ub839\uc73c\ub85c \ud22c\ud45c\ud558\uc600\uace0 \ud55c \uba85\uc740 \uce7c\ub9ac \ud53c\uc624\ub9ac\ub098\ub97c \ubd80\ud1b5\ub839\uc73c\ub85c \ud22c\ud45c\ud558\uc600\ub2e4. \ubd80\ud1b5\ub839\uc73c\ub85c \ud53c\uc624\ub9ac\ub098\ub97c \ucc0d\uc740 \ud14d\uc0ac\uc2a4 \uc8fc\uc758 \uc120\uac70\uc778\uc774 \ub300\ud1b5\ub839\uc73c\ub85c \ub204\uad6c\ub97c \ucc0d\uc5c8\ub294\uc9c0\ub294 \uc120\uac70 \uacb0\uacfc\ub97c \ud1b5\ud574\uc11c\ub294 \ud655\uc778\ub418\uc9c0 \uc54a\uc558\ub2e4. \ud14d\uc0ac\uc2a4 \uc8fc\uc758 \uc120\uac70\uc778\uc740 \ub300\ud1b5\ub839\uacfc \ubd80\ud1b5\ub839\uc744 \uac01\uac01 \ub2e4\ub978 \ud22c\ud45c\uc6a9\uc9c0\uc5d0 \uc801\uc5b4 \ube44\ubc00\ud22c\ud45c \ub97c \ud558\uae30 \ub54c\ubb38\uc774\ub2e4. \uadf8\ub7ec\ub098 \uc774\ud6c4 \ubc18\ub780\ud45c\ub97c \ub0b8 \uc120\uac70\uc778\ub4e4\uacfc\uc758 \uc778\ud130\ubdf0\ub97c \ud1b5\ud574 \ub300\ud1b5\ub839\uc73c\ub85c \ucf00\uc774\uc2dd\uc744 \uc120\ucd9c\ud55c \uc120\uac70\uc778\uc774 \ubd80\ud1b5\ub839\uc73c\ub85c \ud53c\uc624\ub9ac\ub098\ub97c \uc120\ucd9c\ud588\uace0, \ud3f4\uc744 \ub300\ud1b5\ub839\uc73c\ub85c \uc120\ucd9c\ud55c \uc120\uac70\uc778\uc740 \ubd80\ud1b5\ub839 \ud22c\ud45c\ub294 \uc790\uae30\uac00 \ud22c\ud45c\ud574\uc57c \ud560 \ub9c8\uc774\ud06c \ud39c\uc2a4\uc5d0\uac8c \ud22c\ud45c\ud55c \uac83\uc73c\ub85c \ubc1d\ud600\uc84c\ub2e4. \ucf5c\ub9b0 \ud30c\uc6d4\uacfc \ubc84\ub2c8 \uc0cc\ub354\uc2a4, \ucf00\uc774\uc2dd, \ud3f4\uc740 \uae30\uba85\ud6c4\ubcf4\ub85c \ub4f1\ub85d\ub418\uc5b4 \uc720\uad8c\uc790\uc758 \ud45c\ub97c \uc5bb\uae30\ub3c4 \ud588\ub294\ub370 \uc0cc\ub354\uc2a4\ub294 \uae30\uba85\ud22c\ud45c\uc5d0\uc11c 10\ub9cc \ud45c \uc774\uc0c1\uc744 \ub4dd\ud45c\ud588\ub2e4.", "sentence_answer": "\ud14d\uc0ac\uc2a4 \uc8fc\uc758 \uc120\uac70\uc778\uc740 \ub300\ud1b5\ub839\uacfc \ubd80\ud1b5\ub839\uc744 \uac01\uac01 \ub2e4\ub978 \ud22c\ud45c\uc6a9\uc9c0\uc5d0 \uc801\uc5b4 \ube44\ubc00\ud22c\ud45c \ub97c \ud558\uae30 \ub54c\ubb38\uc774\ub2e4.", "paragraph_id": "6469516-0-1", "paragraph_question": "question: \ud14d\uc0ac\uc2a4 \uc8fc\uc758 \uc120\uac70\uc778\uc774 \ub300\ud1b5\ub839\uc73c\ub85c \ub204\uad6c\ub97c \ud22c\ud45c\ud558\uc600\ub294\uc9c0 \uc54c \uc218 \uc5c6\ub294 \uc774\uc720\ub294?, context: \uc6cc\uc2f1\ud134 \uc8fc\uc758 \uc120\uac70\uc778 \uc138 \uba85\uc740 \uc120\ucd9c\ud558\uae30\ub85c \ud55c \ud790\ub7ec\ub9ac \ud074\ub9b0\ud134 \ub300\uc2e0 \ucf5c\ub9b0 \ud30c\uc6d4 \uc804 \uc7a5\uad00\uc744 \ub300\ud1b5\ub839\uc73c\ub85c \ud22c\ud45c\ud558\uc600\ub2e4. \uc774\ub4e4\uc740 \uac01\uac01 \uc218\uc804 \ucf5c\ub9b0\uc2a4, \ub9c8\ub9ac\uc544 \uce94\ud2b8\uc6f0, \uc5d8\ub9ac\uc790\ubca0\uc2a4 \uc6cc\ub7f0\uc744 \ubd80\ud1b5\ub839\uc73c\ub85c \ud22c\ud45c\ud558\uc600\ub2e4. \ud558\uc640\uc774 \uc8fc\uc758 \ud55c \uc120\uac70\uc778\uc740 \uc120\ucd9c\ud558\uae30\ub85c \ud55c \ud790\ub7ec\ub9ac \ud074\ub9b0\ud134 \ub300\uc2e0 \ubc84\ub2c8 \uc0cc\ub354\uc2a4\ub97c \ub300\ud1b5\ub839\uc73c\ub85c \uc5d8\ub9ac\uc790\ubca0\uc2a4 \uc6cc\ub7f0\uc744 \ubd80\ud1b5\ub839\uc73c\ub85c \ud22c\ud45c\ud558\uc600\ub2e4. \uc6cc\uc2f1\ud134 \uc8fc\uc758 \ub610\ub2e4\ub978 \ud55c \uba85\uc740 \uc120\ucd9c\ud558\uae30\ub85c \ud55c \ud790\ub7ec\ub9ac \ud074\ub9b0\ud134 \ub300\uc2e0 \ubbf8\uad6d \uc6d0\uc8fc\ubbfc \ud658\uacbd\uc6b4\ub3d9\uac00 \ud398\uc774\uc2a4 \uc2a4\ud30c\ud2f0\ub4dc \uc774\uae00\uc744 \ub300\ud1b5\ub839\uc73c\ub85c, \uc6d0\uc8fc\ubbfc\uacfc \ubc31\uc778\uc758 \ud63c\ud608\uc778 \ud658\uacbd\uc6b4\ub3d9\uac00 \uc704\ub178\uc544 \ub77c\ub450\ud06c\ub97c \ubd80\ud1b5\ub839\uc73c\ub85c \ud22c\ud45c\ud558\uc600\ub2e4. \ud14d\uc0ac\uc2a4 \uc8fc\uc758 \ud55c \uc120\uac70\uc778\uc740 \uc874 \ucf00\uc774\uc2dd\uc744, \ud55c \uc120\uac70\uc778\uc740 \ub860 \ud3f4\uc744 \ub300\ud1b5\ub839\uc73c\ub85c \ud22c\ud45c\ud558\uc600\uace0 \ud55c \uba85\uc740 \uce7c\ub9ac \ud53c\uc624\ub9ac\ub098\ub97c \ubd80\ud1b5\ub839\uc73c\ub85c \ud22c\ud45c\ud558\uc600\ub2e4. \ubd80\ud1b5\ub839\uc73c\ub85c \ud53c\uc624\ub9ac\ub098\ub97c \ucc0d\uc740 \ud14d\uc0ac\uc2a4 \uc8fc\uc758 \uc120\uac70\uc778\uc774 \ub300\ud1b5\ub839\uc73c\ub85c \ub204\uad6c\ub97c \ucc0d\uc5c8\ub294\uc9c0\ub294 \uc120\uac70 \uacb0\uacfc\ub97c \ud1b5\ud574\uc11c\ub294 \ud655\uc778\ub418\uc9c0 \uc54a\uc558\ub2e4. \ud14d\uc0ac\uc2a4 \uc8fc\uc758 \uc120\uac70\uc778\uc740 \ub300\ud1b5\ub839\uacfc \ubd80\ud1b5\ub839\uc744 \uac01\uac01 \ub2e4\ub978 \ud22c\ud45c\uc6a9\uc9c0\uc5d0 \uc801\uc5b4 \ube44\ubc00\ud22c\ud45c\ub97c \ud558\uae30 \ub54c\ubb38\uc774\ub2e4. \uadf8\ub7ec\ub098 \uc774\ud6c4 \ubc18\ub780\ud45c\ub97c \ub0b8 \uc120\uac70\uc778\ub4e4\uacfc\uc758 \uc778\ud130\ubdf0\ub97c \ud1b5\ud574 \ub300\ud1b5\ub839\uc73c\ub85c \ucf00\uc774\uc2dd\uc744 \uc120\ucd9c\ud55c \uc120\uac70\uc778\uc774 \ubd80\ud1b5\ub839\uc73c\ub85c \ud53c\uc624\ub9ac\ub098\ub97c \uc120\ucd9c\ud588\uace0, \ud3f4\uc744 \ub300\ud1b5\ub839\uc73c\ub85c \uc120\ucd9c\ud55c \uc120\uac70\uc778\uc740 \ubd80\ud1b5\ub839 \ud22c\ud45c\ub294 \uc790\uae30\uac00 \ud22c\ud45c\ud574\uc57c \ud560 \ub9c8\uc774\ud06c \ud39c\uc2a4\uc5d0\uac8c \ud22c\ud45c\ud55c \uac83\uc73c\ub85c \ubc1d\ud600\uc84c\ub2e4. \ucf5c\ub9b0 \ud30c\uc6d4\uacfc \ubc84\ub2c8 \uc0cc\ub354\uc2a4, \ucf00\uc774\uc2dd, \ud3f4\uc740 \uae30\uba85\ud6c4\ubcf4\ub85c \ub4f1\ub85d\ub418\uc5b4 \uc720\uad8c\uc790\uc758 \ud45c\ub97c \uc5bb\uae30\ub3c4 \ud588\ub294\ub370 \uc0cc\ub354\uc2a4\ub294 \uae30\uba85\ud22c\ud45c\uc5d0\uc11c 10\ub9cc \ud45c \uc774\uc0c1\uc744 \ub4dd\ud45c\ud588\ub2e4."} {"question": "\uc2a4\ud0c0\ud384\uc2a4\uac00 \ubc1c\ud45c\ud55c 2000\ub144\ub300 \uac00\uc7a5 \uc704\ub300\ud55c \ub178\ub798 \ud0d1 10 \uc911 single ladies\ub294 \uba87 \uc704\uc5d0 \uc62c\ub790\ub294\uac00?", "paragraph": "\u300a\ub864\ub9c1 \uc2a4\ud1a4\u300b\uc740 \u3008Single Ladies\u3009\ub97c 2008\ub144 \uc62c\ud574 \ucd5c\uace0\uc758 \ub178\ub798\ub85c \ubf51\uc73c\uba70 \"\ube44\ud2b8\ub294 \ud65c\uae30\uac00 \ub118\uccd0 \uac70\ubd80\ud560 \uc218 \uc5c6\uace0, \ubcf4\uceec \ud6c4\ud06c\ub294 \uaca9\ub82c\ud558\uba74\uc11c\ub3c4 \uc9c4\uba74\ubaa9\uc744 \ubcf4\uc5ec\uc900\ub2e4\"\uace0 \uc124\uba85\ud588\ub2e4. 2009\ub144 \ub3c5\uc790\ub4e4\uc744 \ub300\uc0c1\uc73c\ub85c \uc870\uc0ac\ud55c 2000\ub144\ub300 \ucd5c\uace0\uc758 \ub178\ub798 \ub450 \ubc88\uc9f8\uc5d0 \uc120\uc815\ub418\uc5c8\uace0, \u300a\ub864\ub9c1 \uc2a4\ud1a4\u300b \ud3c9\ub860\uac00\ub4e4\uc774 \uc120\uc815\ud55c 2000\ub144\ub300 \ucd5c\uace0\uc758 \ub178\ub798 100 \uc21c\uc704 \uc911 50\uc704\uc5d0 \uc120\uc815\ub418\uc5c8\ub2e4. MTV \ub274\uc2a4\uc758 2008\ub144 \ucd5c\uace0\uc758 \ub178\ub798 2\uc704\uc5d0 \uc120\uc815\ub418\uc5c8\uace0, \uc81c\uc784\uc2a4 \ubabd\uace0\uba54\ub9ac\ub294 \"\ub0b4\uac00 \uc0c1\uc0c1\ub3c4 \ubabb \ud560 \uc815\ub3c4\ub85c \uc9c0\ub098\uce58\uac8c \ud65c\ub3d9\uc801\uc774\uace0 \uacfc\uc7a5\ub418\uc5c8\ub2e4. \ub178\ub798\ub294 \ud55c \ud3b8\uc758 \uc11c\uc0ac\uc2dc\uac19\uae30\ub3c4 \ud558\uba70 \uc139\uc2dc\ud558\uace0 \uc2ec\uc9c0\uc5b4 \uc57d\uac04 \uc2ac\ud504\uae30\uae4c\uc9c0 \ud558\ub2e4\"\uace0 \ub9d0\ud588\ub2e4. \"\ube44\uc698\uc138\uac00 \uc774\uc640 \uac19\uc740 \uc2f1\uae00\uc744 \ub2e4\uc2dc \ubc1c\ub9e4\ud560 \ud655\ub960\uc740 \uc808\ub300 \uc5c6\ub2e4\"\uace0 \ubabd\uace0\uba54\ub9ac\ub294 \uacb0\ub860\uc744 \ub0b4\ub838\ub2e4. \u300a\ud0c0\uc784\u300b\uc758 \ud3c9\ub860\uac00 \uc870\uc26c \ud0c0\uc774\ub79c\uc9c0\uc5d8\uc740 \"\ube60\ub974\uac8c \uc911\ub3c5\ub41c\ub2e4\"\uace0 \uc124\uba85\ud558\uba70 2008\ub144 \ucd5c\uace0\uc758 \ub178\ub798 7\uc704\uc5d0 \uc120\uc815\ud588\ub2e4. \uac19\uc740 \uc7a1\uc9c0\uc758 \ub354\uae00\ub77c\uc2a4 \uc6b8\ud504\ub294 \uc790\uc2e0\uc774 \ubf51\uc740 \uc5ed\uc0ac\uc0c1 \ucd5c\uace0\uc758 \ub178\ub798 100 \uc21c\uc704 \uc911 9\uc704\uc5d0 \uc120\uc815\ud588\ub2e4. \u300a\uc2a4\ud0c0\ud384\uc2a4\u300b\uc5d0\uc11c\ub294 2000\ub144\ub300 \uac00\uc7a5 \uc704\ub300\ud55c \ub178\ub798 \ud0d1 10\uace1\uc744 \ubc1c\ud45c\ud588\uace0, \u3008Single Ladies\u3009\ub294 \uc2ec\uc0ac\uc704\uc6d0\ub4e4\uc5d0\uac8c \ubbf8\uad6d\uc801\uc774\ub780 \uace1\uc774\ub780 \uc810\uc5d0\uc11c \ub192\uc740 \uc810\uc218\ub97c \ubc1b\uc73c\uba70 10\uc704\uc5d0 \uc62c\ub790\ub2e4.", "answer": "10\uc704", "sentence": "\u300a\uc2a4\ud0c0\ud384\uc2a4\u300b\uc5d0\uc11c\ub294 2000\ub144\ub300 \uac00\uc7a5 \uc704\ub300\ud55c \ub178\ub798 \ud0d1 10\uace1\uc744 \ubc1c\ud45c\ud588\uace0, \u3008Single Ladies\u3009\ub294 \uc2ec\uc0ac\uc704\uc6d0\ub4e4\uc5d0\uac8c \ubbf8\uad6d\uc801\uc774\ub780 \uace1\uc774\ub780 \uc810\uc5d0\uc11c \ub192\uc740 \uc810\uc218\ub97c \ubc1b\uc73c\uba70 10\uc704 \uc5d0 \uc62c\ub790\ub2e4.", "paragraph_sentence": "\u300a\ub864\ub9c1 \uc2a4\ud1a4\u300b\uc740 \u3008Single Ladies\u3009\ub97c 2008\ub144 \uc62c\ud574 \ucd5c\uace0\uc758 \ub178\ub798\ub85c \ubf51\uc73c\uba70 \"\ube44\ud2b8\ub294 \ud65c\uae30\uac00 \ub118\uccd0 \uac70\ubd80\ud560 \uc218 \uc5c6\uace0, \ubcf4\uceec \ud6c4\ud06c\ub294 \uaca9\ub82c\ud558\uba74\uc11c\ub3c4 \uc9c4\uba74\ubaa9\uc744 \ubcf4\uc5ec\uc900\ub2e4\"\uace0 \uc124\uba85\ud588\ub2e4. 2009\ub144 \ub3c5\uc790\ub4e4\uc744 \ub300\uc0c1\uc73c\ub85c \uc870\uc0ac\ud55c 2000\ub144\ub300 \ucd5c\uace0\uc758 \ub178\ub798 \ub450 \ubc88\uc9f8\uc5d0 \uc120\uc815\ub418\uc5c8\uace0, \u300a\ub864\ub9c1 \uc2a4\ud1a4\u300b \ud3c9\ub860\uac00\ub4e4\uc774 \uc120\uc815\ud55c 2000\ub144\ub300 \ucd5c\uace0\uc758 \ub178\ub798 100 \uc21c\uc704 \uc911 50\uc704\uc5d0 \uc120\uc815\ub418\uc5c8\ub2e4. MTV \ub274\uc2a4\uc758 2008\ub144 \ucd5c\uace0\uc758 \ub178\ub798 2\uc704\uc5d0 \uc120\uc815\ub418\uc5c8\uace0, \uc81c\uc784\uc2a4 \ubabd\uace0\uba54\ub9ac\ub294 \"\ub0b4\uac00 \uc0c1\uc0c1\ub3c4 \ubabb \ud560 \uc815\ub3c4\ub85c \uc9c0\ub098\uce58\uac8c \ud65c\ub3d9\uc801\uc774\uace0 \uacfc\uc7a5\ub418\uc5c8\ub2e4. \ub178\ub798\ub294 \ud55c \ud3b8\uc758 \uc11c\uc0ac\uc2dc\uac19\uae30\ub3c4 \ud558\uba70 \uc139\uc2dc\ud558\uace0 \uc2ec\uc9c0\uc5b4 \uc57d\uac04 \uc2ac\ud504\uae30\uae4c\uc9c0 \ud558\ub2e4\"\uace0 \ub9d0\ud588\ub2e4. \"\ube44\uc698\uc138\uac00 \uc774\uc640 \uac19\uc740 \uc2f1\uae00\uc744 \ub2e4\uc2dc \ubc1c\ub9e4\ud560 \ud655\ub960\uc740 \uc808\ub300 \uc5c6\ub2e4\"\uace0 \ubabd\uace0\uba54\ub9ac\ub294 \uacb0\ub860\uc744 \ub0b4\ub838\ub2e4. \u300a\ud0c0\uc784\u300b\uc758 \ud3c9\ub860\uac00 \uc870\uc26c \ud0c0\uc774\ub79c\uc9c0\uc5d8\uc740 \"\ube60\ub974\uac8c \uc911\ub3c5\ub41c\ub2e4\"\uace0 \uc124\uba85\ud558\uba70 2008\ub144 \ucd5c\uace0\uc758 \ub178\ub798 7\uc704\uc5d0 \uc120\uc815\ud588\ub2e4. \uac19\uc740 \uc7a1\uc9c0\uc758 \ub354\uae00\ub77c\uc2a4 \uc6b8\ud504\ub294 \uc790\uc2e0\uc774 \ubf51\uc740 \uc5ed\uc0ac\uc0c1 \ucd5c\uace0\uc758 \ub178\ub798 100 \uc21c\uc704 \uc911 9\uc704\uc5d0 \uc120\uc815\ud588\ub2e4. \u300a\uc2a4\ud0c0\ud384\uc2a4\u300b\uc5d0\uc11c\ub294 2000\ub144\ub300 \uac00\uc7a5 \uc704\ub300\ud55c \ub178\ub798 \ud0d1 10\uace1\uc744 \ubc1c\ud45c\ud588\uace0, \u3008Single Ladies\u3009\ub294 \uc2ec\uc0ac\uc704\uc6d0\ub4e4\uc5d0\uac8c \ubbf8\uad6d\uc801\uc774\ub780 \uace1\uc774\ub780 \uc810\uc5d0\uc11c \ub192\uc740 \uc810\uc218\ub97c \ubc1b\uc73c\uba70 10\uc704 \uc5d0 \uc62c\ub790\ub2e4. ", "paragraph_answer": "\u300a\ub864\ub9c1 \uc2a4\ud1a4\u300b\uc740 \u3008Single Ladies\u3009\ub97c 2008\ub144 \uc62c\ud574 \ucd5c\uace0\uc758 \ub178\ub798\ub85c \ubf51\uc73c\uba70 \"\ube44\ud2b8\ub294 \ud65c\uae30\uac00 \ub118\uccd0 \uac70\ubd80\ud560 \uc218 \uc5c6\uace0, \ubcf4\uceec \ud6c4\ud06c\ub294 \uaca9\ub82c\ud558\uba74\uc11c\ub3c4 \uc9c4\uba74\ubaa9\uc744 \ubcf4\uc5ec\uc900\ub2e4\"\uace0 \uc124\uba85\ud588\ub2e4. 2009\ub144 \ub3c5\uc790\ub4e4\uc744 \ub300\uc0c1\uc73c\ub85c \uc870\uc0ac\ud55c 2000\ub144\ub300 \ucd5c\uace0\uc758 \ub178\ub798 \ub450 \ubc88\uc9f8\uc5d0 \uc120\uc815\ub418\uc5c8\uace0, \u300a\ub864\ub9c1 \uc2a4\ud1a4\u300b \ud3c9\ub860\uac00\ub4e4\uc774 \uc120\uc815\ud55c 2000\ub144\ub300 \ucd5c\uace0\uc758 \ub178\ub798 100 \uc21c\uc704 \uc911 50\uc704\uc5d0 \uc120\uc815\ub418\uc5c8\ub2e4. MTV \ub274\uc2a4\uc758 2008\ub144 \ucd5c\uace0\uc758 \ub178\ub798 2\uc704\uc5d0 \uc120\uc815\ub418\uc5c8\uace0, \uc81c\uc784\uc2a4 \ubabd\uace0\uba54\ub9ac\ub294 \"\ub0b4\uac00 \uc0c1\uc0c1\ub3c4 \ubabb \ud560 \uc815\ub3c4\ub85c \uc9c0\ub098\uce58\uac8c \ud65c\ub3d9\uc801\uc774\uace0 \uacfc\uc7a5\ub418\uc5c8\ub2e4. \ub178\ub798\ub294 \ud55c \ud3b8\uc758 \uc11c\uc0ac\uc2dc\uac19\uae30\ub3c4 \ud558\uba70 \uc139\uc2dc\ud558\uace0 \uc2ec\uc9c0\uc5b4 \uc57d\uac04 \uc2ac\ud504\uae30\uae4c\uc9c0 \ud558\ub2e4\"\uace0 \ub9d0\ud588\ub2e4. \"\ube44\uc698\uc138\uac00 \uc774\uc640 \uac19\uc740 \uc2f1\uae00\uc744 \ub2e4\uc2dc \ubc1c\ub9e4\ud560 \ud655\ub960\uc740 \uc808\ub300 \uc5c6\ub2e4\"\uace0 \ubabd\uace0\uba54\ub9ac\ub294 \uacb0\ub860\uc744 \ub0b4\ub838\ub2e4. \u300a\ud0c0\uc784\u300b\uc758 \ud3c9\ub860\uac00 \uc870\uc26c \ud0c0\uc774\ub79c\uc9c0\uc5d8\uc740 \"\ube60\ub974\uac8c \uc911\ub3c5\ub41c\ub2e4\"\uace0 \uc124\uba85\ud558\uba70 2008\ub144 \ucd5c\uace0\uc758 \ub178\ub798 7\uc704\uc5d0 \uc120\uc815\ud588\ub2e4. \uac19\uc740 \uc7a1\uc9c0\uc758 \ub354\uae00\ub77c\uc2a4 \uc6b8\ud504\ub294 \uc790\uc2e0\uc774 \ubf51\uc740 \uc5ed\uc0ac\uc0c1 \ucd5c\uace0\uc758 \ub178\ub798 100 \uc21c\uc704 \uc911 9\uc704\uc5d0 \uc120\uc815\ud588\ub2e4. \u300a\uc2a4\ud0c0\ud384\uc2a4\u300b\uc5d0\uc11c\ub294 2000\ub144\ub300 \uac00\uc7a5 \uc704\ub300\ud55c \ub178\ub798 \ud0d1 10\uace1\uc744 \ubc1c\ud45c\ud588\uace0, \u3008Single Ladies\u3009\ub294 \uc2ec\uc0ac\uc704\uc6d0\ub4e4\uc5d0\uac8c \ubbf8\uad6d\uc801\uc774\ub780 \uace1\uc774\ub780 \uc810\uc5d0\uc11c \ub192\uc740 \uc810\uc218\ub97c \ubc1b\uc73c\uba70 10\uc704 \uc5d0 \uc62c\ub790\ub2e4.", "sentence_answer": "\u300a\uc2a4\ud0c0\ud384\uc2a4\u300b\uc5d0\uc11c\ub294 2000\ub144\ub300 \uac00\uc7a5 \uc704\ub300\ud55c \ub178\ub798 \ud0d1 10\uace1\uc744 \ubc1c\ud45c\ud588\uace0, \u3008Single Ladies\u3009\ub294 \uc2ec\uc0ac\uc704\uc6d0\ub4e4\uc5d0\uac8c \ubbf8\uad6d\uc801\uc774\ub780 \uace1\uc774\ub780 \uc810\uc5d0\uc11c \ub192\uc740 \uc810\uc218\ub97c \ubc1b\uc73c\uba70 10\uc704 \uc5d0 \uc62c\ub790\ub2e4.", "paragraph_id": "6488992-11-2", "paragraph_question": "question: \uc2a4\ud0c0\ud384\uc2a4\uac00 \ubc1c\ud45c\ud55c 2000\ub144\ub300 \uac00\uc7a5 \uc704\ub300\ud55c \ub178\ub798 \ud0d1 10 \uc911 single ladies\ub294 \uba87 \uc704\uc5d0 \uc62c\ub790\ub294\uac00?, context: \u300a\ub864\ub9c1 \uc2a4\ud1a4\u300b\uc740 \u3008Single Ladies\u3009\ub97c 2008\ub144 \uc62c\ud574 \ucd5c\uace0\uc758 \ub178\ub798\ub85c \ubf51\uc73c\uba70 \"\ube44\ud2b8\ub294 \ud65c\uae30\uac00 \ub118\uccd0 \uac70\ubd80\ud560 \uc218 \uc5c6\uace0, \ubcf4\uceec \ud6c4\ud06c\ub294 \uaca9\ub82c\ud558\uba74\uc11c\ub3c4 \uc9c4\uba74\ubaa9\uc744 \ubcf4\uc5ec\uc900\ub2e4\"\uace0 \uc124\uba85\ud588\ub2e4. 2009\ub144 \ub3c5\uc790\ub4e4\uc744 \ub300\uc0c1\uc73c\ub85c \uc870\uc0ac\ud55c 2000\ub144\ub300 \ucd5c\uace0\uc758 \ub178\ub798 \ub450 \ubc88\uc9f8\uc5d0 \uc120\uc815\ub418\uc5c8\uace0, \u300a\ub864\ub9c1 \uc2a4\ud1a4\u300b \ud3c9\ub860\uac00\ub4e4\uc774 \uc120\uc815\ud55c 2000\ub144\ub300 \ucd5c\uace0\uc758 \ub178\ub798 100 \uc21c\uc704 \uc911 50\uc704\uc5d0 \uc120\uc815\ub418\uc5c8\ub2e4. MTV \ub274\uc2a4\uc758 2008\ub144 \ucd5c\uace0\uc758 \ub178\ub798 2\uc704\uc5d0 \uc120\uc815\ub418\uc5c8\uace0, \uc81c\uc784\uc2a4 \ubabd\uace0\uba54\ub9ac\ub294 \"\ub0b4\uac00 \uc0c1\uc0c1\ub3c4 \ubabb \ud560 \uc815\ub3c4\ub85c \uc9c0\ub098\uce58\uac8c \ud65c\ub3d9\uc801\uc774\uace0 \uacfc\uc7a5\ub418\uc5c8\ub2e4. \ub178\ub798\ub294 \ud55c \ud3b8\uc758 \uc11c\uc0ac\uc2dc\uac19\uae30\ub3c4 \ud558\uba70 \uc139\uc2dc\ud558\uace0 \uc2ec\uc9c0\uc5b4 \uc57d\uac04 \uc2ac\ud504\uae30\uae4c\uc9c0 \ud558\ub2e4\"\uace0 \ub9d0\ud588\ub2e4. \"\ube44\uc698\uc138\uac00 \uc774\uc640 \uac19\uc740 \uc2f1\uae00\uc744 \ub2e4\uc2dc \ubc1c\ub9e4\ud560 \ud655\ub960\uc740 \uc808\ub300 \uc5c6\ub2e4\"\uace0 \ubabd\uace0\uba54\ub9ac\ub294 \uacb0\ub860\uc744 \ub0b4\ub838\ub2e4. \u300a\ud0c0\uc784\u300b\uc758 \ud3c9\ub860\uac00 \uc870\uc26c \ud0c0\uc774\ub79c\uc9c0\uc5d8\uc740 \"\ube60\ub974\uac8c \uc911\ub3c5\ub41c\ub2e4\"\uace0 \uc124\uba85\ud558\uba70 2008\ub144 \ucd5c\uace0\uc758 \ub178\ub798 7\uc704\uc5d0 \uc120\uc815\ud588\ub2e4. \uac19\uc740 \uc7a1\uc9c0\uc758 \ub354\uae00\ub77c\uc2a4 \uc6b8\ud504\ub294 \uc790\uc2e0\uc774 \ubf51\uc740 \uc5ed\uc0ac\uc0c1 \ucd5c\uace0\uc758 \ub178\ub798 100 \uc21c\uc704 \uc911 9\uc704\uc5d0 \uc120\uc815\ud588\ub2e4. \u300a\uc2a4\ud0c0\ud384\uc2a4\u300b\uc5d0\uc11c\ub294 2000\ub144\ub300 \uac00\uc7a5 \uc704\ub300\ud55c \ub178\ub798 \ud0d1 10\uace1\uc744 \ubc1c\ud45c\ud588\uace0, \u3008Single Ladies\u3009\ub294 \uc2ec\uc0ac\uc704\uc6d0\ub4e4\uc5d0\uac8c \ubbf8\uad6d\uc801\uc774\ub780 \uace1\uc774\ub780 \uc810\uc5d0\uc11c \ub192\uc740 \uc810\uc218\ub97c \ubc1b\uc73c\uba70 10\uc704\uc5d0 \uc62c\ub790\ub2e4."} {"question": "\uc5ec\ub7ec\ubc88\uc758 \uc804\uc7c1\uc758 \uc2b9\ub9ac\uc5d0\ub3c4 \ubd88\uad6c\ud558\uace0 \ub85c\ub9c8\ud654 \uc2dc\ud0a4\ub294\ub370 \uc2e4\ud328\ud55c \uc9c0\uc5ed\uc740 \uc5b4\ub514\uc778\uac00?", "paragraph": "\ud30c\ub974\ud2f0\uc544\uac00 \uc5b8\uc81c\ub098 \uc704\ud611\uc801\uc778 \uc0c1\ub300\uc774\uae30\ub294 \ud588\uc9c0\ub9cc, \uc2e4\uc81c \uc804\uc7c1\uc740 \uac8c\ub974\ub9cc \uc871\uc744 \uc0c1\ub300\ub85c \ub300\ubd80\ubd84 \ub77c\uc778 \uac15, \ub3c4\ub098\uc6b0 \uac15 \uadfc\uad50\uc5d0\uc11c \ubc8c\uc5b4\uc84c\ub2e4. \uc548\ud1a0\ub2c8\uc6b0\uc2a4\uc640 \ucd5c\uc885 \uc804\ud22c\ub97c \ubc8c\uc774\uae30 \uc804\uc5d0 \ub2ec\ub9c8\ud2f0\uc544\uc758 \ubd80\uc871\ub4e4\uacfc \ubc8c\uc600\ub358 \uc804\uc7c1 \uc774\ud6c4 \ub85c\ub9c8\uad70\uc740 \ucc29\uc2e4\ud788 \ub3c4\ub098\uc6b0 \uac15\uc744 \ud5a5\ud574 \uc9c4\uaca9\ud558\uae30 \uc2dc\uc791\ud588\ub2e4. \ud558\uc9c0\ub9cc \uc5ec\ub7ec \ucc28\ub840 \uc804\uc7c1\uc5d0\uc11c \uc2b9\ub9ac\ud588\uc74c\uc5d0\ub3c4 \ubd88\uad6c\ud558\uace0 \uac8c\ub974\ub9c8\ub2c8\uc544 \uc9c0\uc5ed\uc740 \ub85c\ub9c8\ud654\ud558\ub294 \ub370 \uc2e4\ud328\ud558\uc600\ub2e4. 9\ub144\uc5d0 \ud1a0\uc774\ud1a0\ubd80\ub974\ud06c \uc232\uc5d0\uc11c \ub2f9\ud55c \ucc38\ud328\uac00 \uc774\ub97c \uc0c1\uc9d5\uc801\uc73c\ub85c \ubcf4\uc5ec\uc8fc\ub294 \uc0ac\uac74\uc774\ub2e4. \uac8c\ub974\ub9c8\ub2c8\uc544\uc758 \ucd1d\ub3c5\uc778 \ubc14\ub8e8\uc2a4\uac00 \uc774\ub044\ub294 3\uac1c \uad70\ub2e8\uc774 \ucf00\ub8e8\uc2a4\ud0a4 \uc871 \ucd9c\uc2e0\uc758 \uc544\ub974\ubbf8\ub2c8\uc6b0\uc2a4\uac00 \uc774\ub044\ub294 \uac8c\ub974\ub9cc \uc871\uc5d0\uac8c \uc804\uba78\ud55c\ub2e4. \uc544\uc6b0\uad6c\uc2a4\ud22c\uc2a4\ub294 \uc0ac\ud0dc\ub97c \uc218\uc2b5\ud558\ub824 \ud588\uace0, \ud2f0\ubca0\ub9ac\uc6b0\uc2a4\ub294 \uc774\ud6c4 \uc5ec\ub7ec \ucc28\ub840 \ub77c\uc778\ub780\ud2b8\ub85c \uc9c4\uaca9\ud558\uc5ec \uc2b9\ub9ac\ub97c \uac70\ub450\uc5c8\ub2e4. \ud558\uc9c0\ub9cc \uc544\uc6b0\uad6c\uc2a4\ud22c\uc2a4\uc758 \uc8fd\uc740 \ud6c4\uc5d0 \uacc4\uc2b9\uc790\uc778 \ud2f0\ubca0\ub9ac\uc6b0\uc2a4\ub294 \uac8c\ub974\ub9c8\ub2c8\uc544\uc5d0\uc11c \ucca0\uc218\ud558\uae30\ub85c \uacb0\uc815\ud558\uc600\uace0, \uc774\ud6c4 \ub85c\ub9c8\uad70\uc740 \ub77c\uc778 \uac15\uacfc \ub3c4\ub098\uc6b0 \uac15\uc744 \uc8fc \ubc29\uc5b4\uc120\uc73c\ub85c \uc0bc\uc544 \ubc29\uc704 \uccb4\uacc4\ub97c \uad6c\ucd95\ud55c\ub2e4.", "answer": "\uac8c\ub974\ub9c8\ub2c8\uc544", "sentence": "\ud558\uc9c0\ub9cc \uc5ec\ub7ec \ucc28\ub840 \uc804\uc7c1\uc5d0\uc11c \uc2b9\ub9ac\ud588\uc74c\uc5d0\ub3c4 \ubd88\uad6c\ud558\uace0 \uac8c\ub974\ub9c8\ub2c8\uc544 \uc9c0\uc5ed\uc740 \ub85c\ub9c8\ud654\ud558\ub294 \ub370 \uc2e4\ud328\ud558\uc600\ub2e4.", "paragraph_sentence": "\ud30c\ub974\ud2f0\uc544\uac00 \uc5b8\uc81c\ub098 \uc704\ud611\uc801\uc778 \uc0c1\ub300\uc774\uae30\ub294 \ud588\uc9c0\ub9cc, \uc2e4\uc81c \uc804\uc7c1\uc740 \uac8c\ub974\ub9cc \uc871\uc744 \uc0c1\ub300\ub85c \ub300\ubd80\ubd84 \ub77c\uc778 \uac15, \ub3c4\ub098\uc6b0 \uac15 \uadfc\uad50\uc5d0\uc11c \ubc8c\uc5b4\uc84c\ub2e4. \uc548\ud1a0\ub2c8\uc6b0\uc2a4\uc640 \ucd5c\uc885 \uc804\ud22c\ub97c \ubc8c\uc774\uae30 \uc804\uc5d0 \ub2ec\ub9c8\ud2f0\uc544\uc758 \ubd80\uc871\ub4e4\uacfc \ubc8c\uc600\ub358 \uc804\uc7c1 \uc774\ud6c4 \ub85c\ub9c8\uad70\uc740 \ucc29\uc2e4\ud788 \ub3c4\ub098\uc6b0 \uac15\uc744 \ud5a5\ud574 \uc9c4\uaca9\ud558\uae30 \uc2dc\uc791\ud588\ub2e4. \ud558\uc9c0\ub9cc \uc5ec\ub7ec \ucc28\ub840 \uc804\uc7c1\uc5d0\uc11c \uc2b9\ub9ac\ud588\uc74c\uc5d0\ub3c4 \ubd88\uad6c\ud558\uace0 \uac8c\ub974\ub9c8\ub2c8\uc544 \uc9c0\uc5ed\uc740 \ub85c\ub9c8\ud654\ud558\ub294 \ub370 \uc2e4\ud328\ud558\uc600\ub2e4. 9\ub144\uc5d0 \ud1a0\uc774\ud1a0\ubd80\ub974\ud06c \uc232\uc5d0\uc11c \ub2f9\ud55c \ucc38\ud328\uac00 \uc774\ub97c \uc0c1\uc9d5\uc801\uc73c\ub85c \ubcf4\uc5ec\uc8fc\ub294 \uc0ac\uac74\uc774\ub2e4. \uac8c\ub974\ub9c8\ub2c8\uc544\uc758 \ucd1d\ub3c5\uc778 \ubc14\ub8e8\uc2a4\uac00 \uc774\ub044\ub294 3\uac1c \uad70\ub2e8\uc774 \ucf00\ub8e8\uc2a4\ud0a4 \uc871 \ucd9c\uc2e0\uc758 \uc544\ub974\ubbf8\ub2c8\uc6b0\uc2a4\uac00 \uc774\ub044\ub294 \uac8c\ub974\ub9cc \uc871\uc5d0\uac8c \uc804\uba78\ud55c\ub2e4. \uc544\uc6b0\uad6c\uc2a4\ud22c\uc2a4\ub294 \uc0ac\ud0dc\ub97c \uc218\uc2b5\ud558\ub824 \ud588\uace0, \ud2f0\ubca0\ub9ac\uc6b0\uc2a4\ub294 \uc774\ud6c4 \uc5ec\ub7ec \ucc28\ub840 \ub77c\uc778\ub780\ud2b8\ub85c \uc9c4\uaca9\ud558\uc5ec \uc2b9\ub9ac\ub97c \uac70\ub450\uc5c8\ub2e4. \ud558\uc9c0\ub9cc \uc544\uc6b0\uad6c\uc2a4\ud22c\uc2a4\uc758 \uc8fd\uc740 \ud6c4\uc5d0 \uacc4\uc2b9\uc790\uc778 \ud2f0\ubca0\ub9ac\uc6b0\uc2a4\ub294 \uac8c\ub974\ub9c8\ub2c8\uc544\uc5d0\uc11c \ucca0\uc218\ud558\uae30\ub85c \uacb0\uc815\ud558\uc600\uace0, \uc774\ud6c4 \ub85c\ub9c8\uad70\uc740 \ub77c\uc778 \uac15\uacfc \ub3c4\ub098\uc6b0 \uac15\uc744 \uc8fc \ubc29\uc5b4\uc120\uc73c\ub85c \uc0bc\uc544 \ubc29\uc704 \uccb4\uacc4\ub97c \uad6c\ucd95\ud55c\ub2e4.", "paragraph_answer": "\ud30c\ub974\ud2f0\uc544\uac00 \uc5b8\uc81c\ub098 \uc704\ud611\uc801\uc778 \uc0c1\ub300\uc774\uae30\ub294 \ud588\uc9c0\ub9cc, \uc2e4\uc81c \uc804\uc7c1\uc740 \uac8c\ub974\ub9cc \uc871\uc744 \uc0c1\ub300\ub85c \ub300\ubd80\ubd84 \ub77c\uc778 \uac15, \ub3c4\ub098\uc6b0 \uac15 \uadfc\uad50\uc5d0\uc11c \ubc8c\uc5b4\uc84c\ub2e4. \uc548\ud1a0\ub2c8\uc6b0\uc2a4\uc640 \ucd5c\uc885 \uc804\ud22c\ub97c \ubc8c\uc774\uae30 \uc804\uc5d0 \ub2ec\ub9c8\ud2f0\uc544\uc758 \ubd80\uc871\ub4e4\uacfc \ubc8c\uc600\ub358 \uc804\uc7c1 \uc774\ud6c4 \ub85c\ub9c8\uad70\uc740 \ucc29\uc2e4\ud788 \ub3c4\ub098\uc6b0 \uac15\uc744 \ud5a5\ud574 \uc9c4\uaca9\ud558\uae30 \uc2dc\uc791\ud588\ub2e4. \ud558\uc9c0\ub9cc \uc5ec\ub7ec \ucc28\ub840 \uc804\uc7c1\uc5d0\uc11c \uc2b9\ub9ac\ud588\uc74c\uc5d0\ub3c4 \ubd88\uad6c\ud558\uace0 \uac8c\ub974\ub9c8\ub2c8\uc544 \uc9c0\uc5ed\uc740 \ub85c\ub9c8\ud654\ud558\ub294 \ub370 \uc2e4\ud328\ud558\uc600\ub2e4. 9\ub144\uc5d0 \ud1a0\uc774\ud1a0\ubd80\ub974\ud06c \uc232\uc5d0\uc11c \ub2f9\ud55c \ucc38\ud328\uac00 \uc774\ub97c \uc0c1\uc9d5\uc801\uc73c\ub85c \ubcf4\uc5ec\uc8fc\ub294 \uc0ac\uac74\uc774\ub2e4. \uac8c\ub974\ub9c8\ub2c8\uc544\uc758 \ucd1d\ub3c5\uc778 \ubc14\ub8e8\uc2a4\uac00 \uc774\ub044\ub294 3\uac1c \uad70\ub2e8\uc774 \ucf00\ub8e8\uc2a4\ud0a4 \uc871 \ucd9c\uc2e0\uc758 \uc544\ub974\ubbf8\ub2c8\uc6b0\uc2a4\uac00 \uc774\ub044\ub294 \uac8c\ub974\ub9cc \uc871\uc5d0\uac8c \uc804\uba78\ud55c\ub2e4. \uc544\uc6b0\uad6c\uc2a4\ud22c\uc2a4\ub294 \uc0ac\ud0dc\ub97c \uc218\uc2b5\ud558\ub824 \ud588\uace0, \ud2f0\ubca0\ub9ac\uc6b0\uc2a4\ub294 \uc774\ud6c4 \uc5ec\ub7ec \ucc28\ub840 \ub77c\uc778\ub780\ud2b8\ub85c \uc9c4\uaca9\ud558\uc5ec \uc2b9\ub9ac\ub97c \uac70\ub450\uc5c8\ub2e4. \ud558\uc9c0\ub9cc \uc544\uc6b0\uad6c\uc2a4\ud22c\uc2a4\uc758 \uc8fd\uc740 \ud6c4\uc5d0 \uacc4\uc2b9\uc790\uc778 \ud2f0\ubca0\ub9ac\uc6b0\uc2a4\ub294 \uac8c\ub974\ub9c8\ub2c8\uc544\uc5d0\uc11c \ucca0\uc218\ud558\uae30\ub85c \uacb0\uc815\ud558\uc600\uace0, \uc774\ud6c4 \ub85c\ub9c8\uad70\uc740 \ub77c\uc778 \uac15\uacfc \ub3c4\ub098\uc6b0 \uac15\uc744 \uc8fc \ubc29\uc5b4\uc120\uc73c\ub85c \uc0bc\uc544 \ubc29\uc704 \uccb4\uacc4\ub97c \uad6c\ucd95\ud55c\ub2e4.", "sentence_answer": "\ud558\uc9c0\ub9cc \uc5ec\ub7ec \ucc28\ub840 \uc804\uc7c1\uc5d0\uc11c \uc2b9\ub9ac\ud588\uc74c\uc5d0\ub3c4 \ubd88\uad6c\ud558\uace0 \uac8c\ub974\ub9c8\ub2c8\uc544 \uc9c0\uc5ed\uc740 \ub85c\ub9c8\ud654\ud558\ub294 \ub370 \uc2e4\ud328\ud558\uc600\ub2e4.", "paragraph_id": "6657258-28-0", "paragraph_question": "question: \uc5ec\ub7ec\ubc88\uc758 \uc804\uc7c1\uc758 \uc2b9\ub9ac\uc5d0\ub3c4 \ubd88\uad6c\ud558\uace0 \ub85c\ub9c8\ud654 \uc2dc\ud0a4\ub294\ub370 \uc2e4\ud328\ud55c \uc9c0\uc5ed\uc740 \uc5b4\ub514\uc778\uac00?, context: \ud30c\ub974\ud2f0\uc544\uac00 \uc5b8\uc81c\ub098 \uc704\ud611\uc801\uc778 \uc0c1\ub300\uc774\uae30\ub294 \ud588\uc9c0\ub9cc, \uc2e4\uc81c \uc804\uc7c1\uc740 \uac8c\ub974\ub9cc \uc871\uc744 \uc0c1\ub300\ub85c \ub300\ubd80\ubd84 \ub77c\uc778 \uac15, \ub3c4\ub098\uc6b0 \uac15 \uadfc\uad50\uc5d0\uc11c \ubc8c\uc5b4\uc84c\ub2e4. \uc548\ud1a0\ub2c8\uc6b0\uc2a4\uc640 \ucd5c\uc885 \uc804\ud22c\ub97c \ubc8c\uc774\uae30 \uc804\uc5d0 \ub2ec\ub9c8\ud2f0\uc544\uc758 \ubd80\uc871\ub4e4\uacfc \ubc8c\uc600\ub358 \uc804\uc7c1 \uc774\ud6c4 \ub85c\ub9c8\uad70\uc740 \ucc29\uc2e4\ud788 \ub3c4\ub098\uc6b0 \uac15\uc744 \ud5a5\ud574 \uc9c4\uaca9\ud558\uae30 \uc2dc\uc791\ud588\ub2e4. \ud558\uc9c0\ub9cc \uc5ec\ub7ec \ucc28\ub840 \uc804\uc7c1\uc5d0\uc11c \uc2b9\ub9ac\ud588\uc74c\uc5d0\ub3c4 \ubd88\uad6c\ud558\uace0 \uac8c\ub974\ub9c8\ub2c8\uc544 \uc9c0\uc5ed\uc740 \ub85c\ub9c8\ud654\ud558\ub294 \ub370 \uc2e4\ud328\ud558\uc600\ub2e4. 9\ub144\uc5d0 \ud1a0\uc774\ud1a0\ubd80\ub974\ud06c \uc232\uc5d0\uc11c \ub2f9\ud55c \ucc38\ud328\uac00 \uc774\ub97c \uc0c1\uc9d5\uc801\uc73c\ub85c \ubcf4\uc5ec\uc8fc\ub294 \uc0ac\uac74\uc774\ub2e4. \uac8c\ub974\ub9c8\ub2c8\uc544\uc758 \ucd1d\ub3c5\uc778 \ubc14\ub8e8\uc2a4\uac00 \uc774\ub044\ub294 3\uac1c \uad70\ub2e8\uc774 \ucf00\ub8e8\uc2a4\ud0a4 \uc871 \ucd9c\uc2e0\uc758 \uc544\ub974\ubbf8\ub2c8\uc6b0\uc2a4\uac00 \uc774\ub044\ub294 \uac8c\ub974\ub9cc \uc871\uc5d0\uac8c \uc804\uba78\ud55c\ub2e4. \uc544\uc6b0\uad6c\uc2a4\ud22c\uc2a4\ub294 \uc0ac\ud0dc\ub97c \uc218\uc2b5\ud558\ub824 \ud588\uace0, \ud2f0\ubca0\ub9ac\uc6b0\uc2a4\ub294 \uc774\ud6c4 \uc5ec\ub7ec \ucc28\ub840 \ub77c\uc778\ub780\ud2b8\ub85c \uc9c4\uaca9\ud558\uc5ec \uc2b9\ub9ac\ub97c \uac70\ub450\uc5c8\ub2e4. \ud558\uc9c0\ub9cc \uc544\uc6b0\uad6c\uc2a4\ud22c\uc2a4\uc758 \uc8fd\uc740 \ud6c4\uc5d0 \uacc4\uc2b9\uc790\uc778 \ud2f0\ubca0\ub9ac\uc6b0\uc2a4\ub294 \uac8c\ub974\ub9c8\ub2c8\uc544\uc5d0\uc11c \ucca0\uc218\ud558\uae30\ub85c \uacb0\uc815\ud558\uc600\uace0, \uc774\ud6c4 \ub85c\ub9c8\uad70\uc740 \ub77c\uc778 \uac15\uacfc \ub3c4\ub098\uc6b0 \uac15\uc744 \uc8fc \ubc29\uc5b4\uc120\uc73c\ub85c \uc0bc\uc544 \ubc29\uc704 \uccb4\uacc4\ub97c \uad6c\ucd95\ud55c\ub2e4."} {"question": "\ud64d\ucf69 \uc785\ubc95\ud68c\uc758 \uc784\uae30\ub294?", "paragraph": "\ud64d\ucf69\uc758 \uc785\ubc95\ubd80\uc5d0 \ud574\ub2f9\ud558\ub294 \uae30\uad00\uc740 \ud64d\ucf69 \uc785\ubc95\ud68c(\u9999\u6e2f\u7acb\u6cd5\u6703)\uc774\ub2e4. \ud604\uc7ac 60\uba85\uc758 \uc758\uc11d\uc774 \uc788\uace0, \uc784\uae30\ub294 4\ub144\uc774\ub2e4. \uc81c3\ud68c \uc785\ubc95\ud68c \uc911 60\uac1c\uc758 \uc758\uc11d \uc911\uc5d0 30\uc11d\uc740 5\uac1c\uc758 \uc9c0\ubc29\ubcc4\ub85c \ube44\ub840 \ub300\ud45c\uc81c\uc758 \uc9c1\uc811 \uc120\uac70\ub97c \ud1b5\ud574 \ubf51\ud788\uac8c \ub418\uace0, \uadf8 \ub098\uba38\uc9c0 30\uba85\uc740 28\uac1c\uc758 \uae30\ub2a5\ubcc4\ub85c \ub2e8\uccb4 \ud639\uc740 1\uc77c 1\uc77c\uc758 \uc120\uac70\ub97c \ud1b5\ud574 \uc120\ucd9c\ub41c\ub2e4. \ubaa8\ub4e0 \uc120\uac70 \uc720\uad8c\uc790\ub294 \uc9c0\ubc29 \uc9c1\uc811 \uc120\uac70\uc5d0\ub294 \ubaa8\ub4e0 \uc0ac\ub78c\uc5d0\uac8c \ud55c \ud45c\uac00 \uc8fc\uc5b4\uc9c0\uc9c0\ub9cc \uae30\ub2a5\ubcc4\ub85c \uc870\uc9c1\ub418\ub294 \uc758\uc11d\uc740 15\ub9cc \uba85\uc758 \uc120\uac70 \uc720\uad8c\uc790\ub9cc\uc774 \ud22c\ud45c\uad8c\uc774 \uc788\ub2e4. \uc77c\ubc18\uc801\uc778 \uc0c1\ud669\uc5d0\uc11c, \ud64d\ucf69 \ubc95\ub840\uc5d0\ub294 \uc9c1\uc811 \uc120\uac70 \ubc29\uc2dd\uc758 \uc758\uc6d0\uacfc \uae30\ub2a5\ubcc4\ub85c \uc870\uc9c1 \ub41c \uc758\uc6d0\uc5d0\uc11c \uacfc\ubc18\uc218\uc758 \ucc2c\uc131\uc774 \ud544\uc694\ud558\uace0, \ub610 \ud589\uc815\uc7a5\uad00\uc758 \ube44\uc900\uc744 \ud1b5\ud574\uc11c \ud6a8\ub825\uc774 \uc0dd\uae34\ub2e4. \ud64d\ucf69 \ubc18\ud658 \uc774\ud6c4, \ud604\uc7ac 3\ucc28 \uc785\ubc95\ud68c \uc120\uac70\uac00 \uac01\uac01 1998\ub144, 2000\ub144, 2004\ub144\uc5d0 \uac70\ud589\ub418\uc5c8\ub2e4.", "answer": "4\ub144", "sentence": "\uc784\uae30\ub294 4\ub144 \uc774\ub2e4.", "paragraph_sentence": "\ud64d\ucf69\uc758 \uc785\ubc95\ubd80\uc5d0 \ud574\ub2f9\ud558\ub294 \uae30\uad00\uc740 \ud64d\ucf69 \uc785\ubc95\ud68c(\u9999\u6e2f\u7acb\u6cd5\u6703)\uc774\ub2e4. \ud604\uc7ac 60\uba85\uc758 \uc758\uc11d\uc774 \uc788\uace0, \uc784\uae30\ub294 4\ub144 \uc774\ub2e4. \uc81c3\ud68c \uc785\ubc95\ud68c \uc911 60\uac1c\uc758 \uc758\uc11d \uc911\uc5d0 30\uc11d\uc740 5\uac1c\uc758 \uc9c0\ubc29\ubcc4\ub85c \ube44\ub840 \ub300\ud45c\uc81c\uc758 \uc9c1\uc811 \uc120\uac70\ub97c \ud1b5\ud574 \ubf51\ud788\uac8c \ub418\uace0, \uadf8 \ub098\uba38\uc9c0 30\uba85\uc740 28\uac1c\uc758 \uae30\ub2a5\ubcc4\ub85c \ub2e8\uccb4 \ud639\uc740 1\uc77c 1\uc77c\uc758 \uc120\uac70\ub97c \ud1b5\ud574 \uc120\ucd9c\ub41c\ub2e4. \ubaa8\ub4e0 \uc120\uac70 \uc720\uad8c\uc790\ub294 \uc9c0\ubc29 \uc9c1\uc811 \uc120\uac70\uc5d0\ub294 \ubaa8\ub4e0 \uc0ac\ub78c\uc5d0\uac8c \ud55c \ud45c\uac00 \uc8fc\uc5b4\uc9c0\uc9c0\ub9cc \uae30\ub2a5\ubcc4\ub85c \uc870\uc9c1\ub418\ub294 \uc758\uc11d\uc740 15\ub9cc \uba85\uc758 \uc120\uac70 \uc720\uad8c\uc790\ub9cc\uc774 \ud22c\ud45c\uad8c\uc774 \uc788\ub2e4. \uc77c\ubc18\uc801\uc778 \uc0c1\ud669\uc5d0\uc11c, \ud64d\ucf69 \ubc95\ub840\uc5d0\ub294 \uc9c1\uc811 \uc120\uac70 \ubc29\uc2dd\uc758 \uc758\uc6d0\uacfc \uae30\ub2a5\ubcc4\ub85c \uc870\uc9c1 \ub41c \uc758\uc6d0\uc5d0\uc11c \uacfc\ubc18\uc218\uc758 \ucc2c\uc131\uc774 \ud544\uc694\ud558\uace0, \ub610 \ud589\uc815\uc7a5\uad00\uc758 \ube44\uc900\uc744 \ud1b5\ud574\uc11c \ud6a8\ub825\uc774 \uc0dd\uae34\ub2e4. \ud64d\ucf69 \ubc18\ud658 \uc774\ud6c4, \ud604\uc7ac 3\ucc28 \uc785\ubc95\ud68c \uc120\uac70\uac00 \uac01\uac01 1998\ub144, 2000\ub144, 2004\ub144\uc5d0 \uac70\ud589\ub418\uc5c8\ub2e4.", "paragraph_answer": "\ud64d\ucf69\uc758 \uc785\ubc95\ubd80\uc5d0 \ud574\ub2f9\ud558\ub294 \uae30\uad00\uc740 \ud64d\ucf69 \uc785\ubc95\ud68c(\u9999\u6e2f\u7acb\u6cd5\u6703)\uc774\ub2e4. \ud604\uc7ac 60\uba85\uc758 \uc758\uc11d\uc774 \uc788\uace0, \uc784\uae30\ub294 4\ub144 \uc774\ub2e4. \uc81c3\ud68c \uc785\ubc95\ud68c \uc911 60\uac1c\uc758 \uc758\uc11d \uc911\uc5d0 30\uc11d\uc740 5\uac1c\uc758 \uc9c0\ubc29\ubcc4\ub85c \ube44\ub840 \ub300\ud45c\uc81c\uc758 \uc9c1\uc811 \uc120\uac70\ub97c \ud1b5\ud574 \ubf51\ud788\uac8c \ub418\uace0, \uadf8 \ub098\uba38\uc9c0 30\uba85\uc740 28\uac1c\uc758 \uae30\ub2a5\ubcc4\ub85c \ub2e8\uccb4 \ud639\uc740 1\uc77c 1\uc77c\uc758 \uc120\uac70\ub97c \ud1b5\ud574 \uc120\ucd9c\ub41c\ub2e4. \ubaa8\ub4e0 \uc120\uac70 \uc720\uad8c\uc790\ub294 \uc9c0\ubc29 \uc9c1\uc811 \uc120\uac70\uc5d0\ub294 \ubaa8\ub4e0 \uc0ac\ub78c\uc5d0\uac8c \ud55c \ud45c\uac00 \uc8fc\uc5b4\uc9c0\uc9c0\ub9cc \uae30\ub2a5\ubcc4\ub85c \uc870\uc9c1\ub418\ub294 \uc758\uc11d\uc740 15\ub9cc \uba85\uc758 \uc120\uac70 \uc720\uad8c\uc790\ub9cc\uc774 \ud22c\ud45c\uad8c\uc774 \uc788\ub2e4. \uc77c\ubc18\uc801\uc778 \uc0c1\ud669\uc5d0\uc11c, \ud64d\ucf69 \ubc95\ub840\uc5d0\ub294 \uc9c1\uc811 \uc120\uac70 \ubc29\uc2dd\uc758 \uc758\uc6d0\uacfc \uae30\ub2a5\ubcc4\ub85c \uc870\uc9c1 \ub41c \uc758\uc6d0\uc5d0\uc11c \uacfc\ubc18\uc218\uc758 \ucc2c\uc131\uc774 \ud544\uc694\ud558\uace0, \ub610 \ud589\uc815\uc7a5\uad00\uc758 \ube44\uc900\uc744 \ud1b5\ud574\uc11c \ud6a8\ub825\uc774 \uc0dd\uae34\ub2e4. \ud64d\ucf69 \ubc18\ud658 \uc774\ud6c4, \ud604\uc7ac 3\ucc28 \uc785\ubc95\ud68c \uc120\uac70\uac00 \uac01\uac01 1998\ub144, 2000\ub144, 2004\ub144\uc5d0 \uac70\ud589\ub418\uc5c8\ub2e4.", "sentence_answer": "\uc784\uae30\ub294 4\ub144 \uc774\ub2e4.", "paragraph_id": "6572281-4-1", "paragraph_question": "question: \ud64d\ucf69 \uc785\ubc95\ud68c\uc758 \uc784\uae30\ub294?, context: \ud64d\ucf69\uc758 \uc785\ubc95\ubd80\uc5d0 \ud574\ub2f9\ud558\ub294 \uae30\uad00\uc740 \ud64d\ucf69 \uc785\ubc95\ud68c(\u9999\u6e2f\u7acb\u6cd5\u6703)\uc774\ub2e4. \ud604\uc7ac 60\uba85\uc758 \uc758\uc11d\uc774 \uc788\uace0, \uc784\uae30\ub294 4\ub144\uc774\ub2e4. \uc81c3\ud68c \uc785\ubc95\ud68c \uc911 60\uac1c\uc758 \uc758\uc11d \uc911\uc5d0 30\uc11d\uc740 5\uac1c\uc758 \uc9c0\ubc29\ubcc4\ub85c \ube44\ub840 \ub300\ud45c\uc81c\uc758 \uc9c1\uc811 \uc120\uac70\ub97c \ud1b5\ud574 \ubf51\ud788\uac8c \ub418\uace0, \uadf8 \ub098\uba38\uc9c0 30\uba85\uc740 28\uac1c\uc758 \uae30\ub2a5\ubcc4\ub85c \ub2e8\uccb4 \ud639\uc740 1\uc77c 1\uc77c\uc758 \uc120\uac70\ub97c \ud1b5\ud574 \uc120\ucd9c\ub41c\ub2e4. \ubaa8\ub4e0 \uc120\uac70 \uc720\uad8c\uc790\ub294 \uc9c0\ubc29 \uc9c1\uc811 \uc120\uac70\uc5d0\ub294 \ubaa8\ub4e0 \uc0ac\ub78c\uc5d0\uac8c \ud55c \ud45c\uac00 \uc8fc\uc5b4\uc9c0\uc9c0\ub9cc \uae30\ub2a5\ubcc4\ub85c \uc870\uc9c1\ub418\ub294 \uc758\uc11d\uc740 15\ub9cc \uba85\uc758 \uc120\uac70 \uc720\uad8c\uc790\ub9cc\uc774 \ud22c\ud45c\uad8c\uc774 \uc788\ub2e4. \uc77c\ubc18\uc801\uc778 \uc0c1\ud669\uc5d0\uc11c, \ud64d\ucf69 \ubc95\ub840\uc5d0\ub294 \uc9c1\uc811 \uc120\uac70 \ubc29\uc2dd\uc758 \uc758\uc6d0\uacfc \uae30\ub2a5\ubcc4\ub85c \uc870\uc9c1 \ub41c \uc758\uc6d0\uc5d0\uc11c \uacfc\ubc18\uc218\uc758 \ucc2c\uc131\uc774 \ud544\uc694\ud558\uace0, \ub610 \ud589\uc815\uc7a5\uad00\uc758 \ube44\uc900\uc744 \ud1b5\ud574\uc11c \ud6a8\ub825\uc774 \uc0dd\uae34\ub2e4. \ud64d\ucf69 \ubc18\ud658 \uc774\ud6c4, \ud604\uc7ac 3\ucc28 \uc785\ubc95\ud68c \uc120\uac70\uac00 \uac01\uac01 1998\ub144, 2000\ub144, 2004\ub144\uc5d0 \uac70\ud589\ub418\uc5c8\ub2e4."} {"question": "\ubbfc\ub9bd\ub300\ud559\uc124\ub9bd\uc6b4\ub3d9\uc774 \ubd80\ud65c\ud55c \ub54c\ub294?", "paragraph": "\uc77c\ubcf8 \uc81c\uad6d\uc758 \ud0c4\uc555\uc73c\ub85c \uc88c\uc808\ub41c \ud638\ub0a8 \uc9c0\uc5ed\uc758 \ubbfc\ub9bd\ub300\ud559\uc124\ub9bd\uc6b4\ub3d9(\u6c11\u7acb\u5927\u5b78\u8a2d\u7acb\u904b\u52d5)\uc740 \ud574\ubc29 \uc9c1\ud6c4\uc778 1946\ub144 5\uc6d4 \uc870\uc120\ub300\ud559 \uc124\ub9bd\ub3d9\uc9c0\ud68c[\u671d\u9bae\u5927\u5b78 \u8a2d\u7acb\u540c\u5fd7\u6703]\uc640 \ucc3d\ub9bd\uc900\ube44\uc704\uc6d0\ud68c\uac00 \uacb0\uc131\ud558\uba74\uc11c \ubd80\ud65c\ud558\uc600\ub2e4. \uc124\ub9bd\ub3d9\uc9c0\ud68c\uc6d0\ub4e4\uc740 12\uc6d4\ubd80\ud130 \ud2b8\ub7ed\uc744 \ud0c0\uace0 \uc804\ub77c\ub3c4 \uac01 \uc9c0\uc5ed\uc744 \ub3cc\uc544\ub2e4\ub2c8\uba74\uc11c \ubaa8\uae08\ud558\uc600\ub294\ub370 \uc774\uc5d0 \uad70\uc218, \uacbd\ucc30\uc11c\uc7a5, \uba74\uc7a5, \uc2dc\uace8\uc774\uc7a5\uc740 \ubb3c\ub860 \uae30\uc0dd\uae4c\uc9c0 \ub2f9\uc2dc \uc300 2\ub9d0\uac12\uc5d0 \ud574\ub2f9\ud558\ub294 100\uc6d0\uc9dc\ub9ac \uc124\ub9bd\ub3d9\uc9c0\ud68c\uc6d0\uad8c\uc744 \uc0c0\uace0 \uac00\ub09c\ud588\ub358 \ub18d\ucd0c\ubbfc\ub3c4 \ub300\ud559\uc744 \uc138\uc6b4\ub2e4\ub294 \ub9d0\uc5d0 \uae68, \uae40, \ubbf8\uc5ed\uc744 \ub0b4\ub193\uc73c\uba74\uc11c \ub300\ud559 \uc124\ub9bd\uc5d0 \ud798\uc744 \ubcf4\ud0dc\ub294, \uc124\ub9bd\ub3d9\uc9c0\ud68c\uc6d0 \uc790\uaca9\uc740 \ud574\ubc29\ub41c \uc870\uad6d\uc744 \uac74\uc124\ud558\ub824\ub294 \uc2e0\ub150\uc5d0 \ubd88\ud0c0\ub294 \uc9c0\uc2dd\uc778, \uad00\ub9ac, \uc9c0\uc8fc\uc5d0\uc11c \uba38\uc2b4\uacfc \ucd0c\ubd80\uae4c\uc9c0 \uacc4\uce35, \ud559\ub825, \ube48\ubd80 \uaca9\ucc28\ub97c \ub118\uc5b4\uc11c \ud638\ub0a8 \uc9c0\uc5ed \uc804 \ub300\uc911\ubfd0\ub9cc \uc544\ub2c8\ub77c \uba40\ub9ac\ub294 \uc81c\uc8fc\ub3c4\uc5d0\uc11c \ucda9\uccad\ub3c4\uc640 \uc218\ub3c4\uad8c\uc5d0\uae4c\uc9c0 \uc644\uc804\ud788 \uac1c\ubc29\ub418\uc5c8\ub2e4. \ub2f9\uc2dc \uc804\ub77c\ub0a8\ub3c4\uc9c0\uc0ac\uc600\ub358 \uc11c\ubbfc\ud638\ub294 \uc77c\ubcf8 \uba54\uc774\uc9c0\ub300\ud559\uad50(\u660e\u6cbb\u5927\u5b66)\ub97c \uc878\uc5c5\ud558\uace0 \uc77c\ubcf8 \ubb38\uad50\uc0ac\ud68c\uad6d\uc5d0\uc11c \uadfc\ubb34\ud55c \uacbd\ud5d8\uc774 \uc788\ub294 \ubc15\ucca0\uc6c5\uc5d0\uac8c \uc124\ub9bd\ub3d9\uc9c0\ud68c\uc7a5\uc9c1\uc744 \ub9e1\uac8c \ud558\uc600\uace0 \ub300\ud559 \uc124\ub9bd\uc744 \uc900\ube44\ud558\uace0\uc790 \ub3c4\uccad\uc774 \uc18c\uc720\ud55c \ucc28\ub7c9\uc744 \uc0ac\uc6a9\ud558\uac8c \ud558\uace0 \ub3c4\uc9c0\uc0ac \uba85\uc758\ub85c \uc0ac\uc7a5\uacfc \uad70\uc218\uc5d0\uac8c \ud611\uc870 \uc9c0\uc2dc \uacf5\ubb38\uc744 \ubcf4\ub0b4\ub294, \uc124\ub9bd\uc5d0\uc11c \uc77c\uc120\uc5d0\uc11c \ud070 \uc5ed\ud560\uc744 \ud558\uc600\ub2e4.", "answer": "1946\ub144 5\uc6d4", "sentence": "\uc77c\ubcf8 \uc81c\uad6d\uc758 \ud0c4\uc555\uc73c\ub85c \uc88c\uc808\ub41c \ud638\ub0a8 \uc9c0\uc5ed\uc758 \ubbfc\ub9bd\ub300\ud559\uc124\ub9bd\uc6b4\ub3d9(\u6c11\u7acb\u5927\u5b78\u8a2d\u7acb\u904b\u52d5)\uc740 \ud574\ubc29 \uc9c1\ud6c4\uc778 1946\ub144 5\uc6d4 \uc870\uc120\ub300\ud559 \uc124\ub9bd\ub3d9\uc9c0\ud68c[\u671d\u9bae\u5927\u5b78 \u8a2d\u7acb\u540c\u5fd7\u6703]\uc640 \ucc3d\ub9bd\uc900\ube44\uc704\uc6d0\ud68c\uac00 \uacb0\uc131\ud558\uba74\uc11c \ubd80\ud65c\ud558\uc600\ub2e4.", "paragraph_sentence": " \uc77c\ubcf8 \uc81c\uad6d\uc758 \ud0c4\uc555\uc73c\ub85c \uc88c\uc808\ub41c \ud638\ub0a8 \uc9c0\uc5ed\uc758 \ubbfc\ub9bd\ub300\ud559\uc124\ub9bd\uc6b4\ub3d9(\u6c11\u7acb\u5927\u5b78\u8a2d\u7acb\u904b\u52d5)\uc740 \ud574\ubc29 \uc9c1\ud6c4\uc778 1946\ub144 5\uc6d4 \uc870\uc120\ub300\ud559 \uc124\ub9bd\ub3d9\uc9c0\ud68c[\u671d\u9bae\u5927\u5b78 \u8a2d\u7acb\u540c\u5fd7\u6703]\uc640 \ucc3d\ub9bd\uc900\ube44\uc704\uc6d0\ud68c\uac00 \uacb0\uc131\ud558\uba74\uc11c \ubd80\ud65c\ud558\uc600\ub2e4. \uc124\ub9bd\ub3d9\uc9c0\ud68c\uc6d0\ub4e4\uc740 12\uc6d4\ubd80\ud130 \ud2b8\ub7ed\uc744 \ud0c0\uace0 \uc804\ub77c\ub3c4 \uac01 \uc9c0\uc5ed\uc744 \ub3cc\uc544\ub2e4\ub2c8\uba74\uc11c \ubaa8\uae08\ud558\uc600\ub294\ub370 \uc774\uc5d0 \uad70\uc218, \uacbd\ucc30\uc11c\uc7a5, \uba74\uc7a5, \uc2dc\uace8\uc774\uc7a5\uc740 \ubb3c\ub860 \uae30\uc0dd\uae4c\uc9c0 \ub2f9\uc2dc \uc300 2\ub9d0\uac12\uc5d0 \ud574\ub2f9\ud558\ub294 100\uc6d0\uc9dc\ub9ac \uc124\ub9bd\ub3d9\uc9c0\ud68c\uc6d0\uad8c\uc744 \uc0c0\uace0 \uac00\ub09c\ud588\ub358 \ub18d\ucd0c\ubbfc\ub3c4 \ub300\ud559\uc744 \uc138\uc6b4\ub2e4\ub294 \ub9d0\uc5d0 \uae68, \uae40, \ubbf8\uc5ed\uc744 \ub0b4\ub193\uc73c\uba74\uc11c \ub300\ud559 \uc124\ub9bd\uc5d0 \ud798\uc744 \ubcf4\ud0dc\ub294, \uc124\ub9bd\ub3d9\uc9c0\ud68c\uc6d0 \uc790\uaca9\uc740 \ud574\ubc29\ub41c \uc870\uad6d\uc744 \uac74\uc124\ud558\ub824\ub294 \uc2e0\ub150\uc5d0 \ubd88\ud0c0\ub294 \uc9c0\uc2dd\uc778, \uad00\ub9ac, \uc9c0\uc8fc\uc5d0\uc11c \uba38\uc2b4\uacfc \ucd0c\ubd80\uae4c\uc9c0 \uacc4\uce35, \ud559\ub825, \ube48\ubd80 \uaca9\ucc28\ub97c \ub118\uc5b4\uc11c \ud638\ub0a8 \uc9c0\uc5ed \uc804 \ub300\uc911\ubfd0\ub9cc \uc544\ub2c8\ub77c \uba40\ub9ac\ub294 \uc81c\uc8fc\ub3c4\uc5d0\uc11c \ucda9\uccad\ub3c4\uc640 \uc218\ub3c4\uad8c\uc5d0\uae4c\uc9c0 \uc644\uc804\ud788 \uac1c\ubc29\ub418\uc5c8\ub2e4. \ub2f9\uc2dc \uc804\ub77c\ub0a8\ub3c4\uc9c0\uc0ac\uc600\ub358 \uc11c\ubbfc\ud638\ub294 \uc77c\ubcf8 \uba54\uc774\uc9c0\ub300\ud559\uad50(\u660e\u6cbb\u5927\u5b66)\ub97c \uc878\uc5c5\ud558\uace0 \uc77c\ubcf8 \ubb38\uad50\uc0ac\ud68c\uad6d\uc5d0\uc11c \uadfc\ubb34\ud55c \uacbd\ud5d8\uc774 \uc788\ub294 \ubc15\ucca0\uc6c5\uc5d0\uac8c \uc124\ub9bd\ub3d9\uc9c0\ud68c\uc7a5\uc9c1\uc744 \ub9e1\uac8c \ud558\uc600\uace0 \ub300\ud559 \uc124\ub9bd\uc744 \uc900\ube44\ud558\uace0\uc790 \ub3c4\uccad\uc774 \uc18c\uc720\ud55c \ucc28\ub7c9\uc744 \uc0ac\uc6a9\ud558\uac8c \ud558\uace0 \ub3c4\uc9c0\uc0ac \uba85\uc758\ub85c \uc0ac\uc7a5\uacfc \uad70\uc218\uc5d0\uac8c \ud611\uc870 \uc9c0\uc2dc \uacf5\ubb38\uc744 \ubcf4\ub0b4\ub294, \uc124\ub9bd\uc5d0\uc11c \uc77c\uc120\uc5d0\uc11c \ud070 \uc5ed\ud560\uc744 \ud558\uc600\ub2e4.", "paragraph_answer": "\uc77c\ubcf8 \uc81c\uad6d\uc758 \ud0c4\uc555\uc73c\ub85c \uc88c\uc808\ub41c \ud638\ub0a8 \uc9c0\uc5ed\uc758 \ubbfc\ub9bd\ub300\ud559\uc124\ub9bd\uc6b4\ub3d9(\u6c11\u7acb\u5927\u5b78\u8a2d\u7acb\u904b\u52d5)\uc740 \ud574\ubc29 \uc9c1\ud6c4\uc778 1946\ub144 5\uc6d4 \uc870\uc120\ub300\ud559 \uc124\ub9bd\ub3d9\uc9c0\ud68c[\u671d\u9bae\u5927\u5b78 \u8a2d\u7acb\u540c\u5fd7\u6703]\uc640 \ucc3d\ub9bd\uc900\ube44\uc704\uc6d0\ud68c\uac00 \uacb0\uc131\ud558\uba74\uc11c \ubd80\ud65c\ud558\uc600\ub2e4. \uc124\ub9bd\ub3d9\uc9c0\ud68c\uc6d0\ub4e4\uc740 12\uc6d4\ubd80\ud130 \ud2b8\ub7ed\uc744 \ud0c0\uace0 \uc804\ub77c\ub3c4 \uac01 \uc9c0\uc5ed\uc744 \ub3cc\uc544\ub2e4\ub2c8\uba74\uc11c \ubaa8\uae08\ud558\uc600\ub294\ub370 \uc774\uc5d0 \uad70\uc218, \uacbd\ucc30\uc11c\uc7a5, \uba74\uc7a5, \uc2dc\uace8\uc774\uc7a5\uc740 \ubb3c\ub860 \uae30\uc0dd\uae4c\uc9c0 \ub2f9\uc2dc \uc300 2\ub9d0\uac12\uc5d0 \ud574\ub2f9\ud558\ub294 100\uc6d0\uc9dc\ub9ac \uc124\ub9bd\ub3d9\uc9c0\ud68c\uc6d0\uad8c\uc744 \uc0c0\uace0 \uac00\ub09c\ud588\ub358 \ub18d\ucd0c\ubbfc\ub3c4 \ub300\ud559\uc744 \uc138\uc6b4\ub2e4\ub294 \ub9d0\uc5d0 \uae68, \uae40, \ubbf8\uc5ed\uc744 \ub0b4\ub193\uc73c\uba74\uc11c \ub300\ud559 \uc124\ub9bd\uc5d0 \ud798\uc744 \ubcf4\ud0dc\ub294, \uc124\ub9bd\ub3d9\uc9c0\ud68c\uc6d0 \uc790\uaca9\uc740 \ud574\ubc29\ub41c \uc870\uad6d\uc744 \uac74\uc124\ud558\ub824\ub294 \uc2e0\ub150\uc5d0 \ubd88\ud0c0\ub294 \uc9c0\uc2dd\uc778, \uad00\ub9ac, \uc9c0\uc8fc\uc5d0\uc11c \uba38\uc2b4\uacfc \ucd0c\ubd80\uae4c\uc9c0 \uacc4\uce35, \ud559\ub825, \ube48\ubd80 \uaca9\ucc28\ub97c \ub118\uc5b4\uc11c \ud638\ub0a8 \uc9c0\uc5ed \uc804 \ub300\uc911\ubfd0\ub9cc \uc544\ub2c8\ub77c \uba40\ub9ac\ub294 \uc81c\uc8fc\ub3c4\uc5d0\uc11c \ucda9\uccad\ub3c4\uc640 \uc218\ub3c4\uad8c\uc5d0\uae4c\uc9c0 \uc644\uc804\ud788 \uac1c\ubc29\ub418\uc5c8\ub2e4. \ub2f9\uc2dc \uc804\ub77c\ub0a8\ub3c4\uc9c0\uc0ac\uc600\ub358 \uc11c\ubbfc\ud638\ub294 \uc77c\ubcf8 \uba54\uc774\uc9c0\ub300\ud559\uad50(\u660e\u6cbb\u5927\u5b66)\ub97c \uc878\uc5c5\ud558\uace0 \uc77c\ubcf8 \ubb38\uad50\uc0ac\ud68c\uad6d\uc5d0\uc11c \uadfc\ubb34\ud55c \uacbd\ud5d8\uc774 \uc788\ub294 \ubc15\ucca0\uc6c5\uc5d0\uac8c \uc124\ub9bd\ub3d9\uc9c0\ud68c\uc7a5\uc9c1\uc744 \ub9e1\uac8c \ud558\uc600\uace0 \ub300\ud559 \uc124\ub9bd\uc744 \uc900\ube44\ud558\uace0\uc790 \ub3c4\uccad\uc774 \uc18c\uc720\ud55c \ucc28\ub7c9\uc744 \uc0ac\uc6a9\ud558\uac8c \ud558\uace0 \ub3c4\uc9c0\uc0ac \uba85\uc758\ub85c \uc0ac\uc7a5\uacfc \uad70\uc218\uc5d0\uac8c \ud611\uc870 \uc9c0\uc2dc \uacf5\ubb38\uc744 \ubcf4\ub0b4\ub294, \uc124\ub9bd\uc5d0\uc11c \uc77c\uc120\uc5d0\uc11c \ud070 \uc5ed\ud560\uc744 \ud558\uc600\ub2e4.", "sentence_answer": "\uc77c\ubcf8 \uc81c\uad6d\uc758 \ud0c4\uc555\uc73c\ub85c \uc88c\uc808\ub41c \ud638\ub0a8 \uc9c0\uc5ed\uc758 \ubbfc\ub9bd\ub300\ud559\uc124\ub9bd\uc6b4\ub3d9(\u6c11\u7acb\u5927\u5b78\u8a2d\u7acb\u904b\u52d5)\uc740 \ud574\ubc29 \uc9c1\ud6c4\uc778 1946\ub144 5\uc6d4 \uc870\uc120\ub300\ud559 \uc124\ub9bd\ub3d9\uc9c0\ud68c[\u671d\u9bae\u5927\u5b78 \u8a2d\u7acb\u540c\u5fd7\u6703]\uc640 \ucc3d\ub9bd\uc900\ube44\uc704\uc6d0\ud68c\uac00 \uacb0\uc131\ud558\uba74\uc11c \ubd80\ud65c\ud558\uc600\ub2e4.", "paragraph_id": "6551328-1-1", "paragraph_question": "question: \ubbfc\ub9bd\ub300\ud559\uc124\ub9bd\uc6b4\ub3d9\uc774 \ubd80\ud65c\ud55c \ub54c\ub294?, context: \uc77c\ubcf8 \uc81c\uad6d\uc758 \ud0c4\uc555\uc73c\ub85c \uc88c\uc808\ub41c \ud638\ub0a8 \uc9c0\uc5ed\uc758 \ubbfc\ub9bd\ub300\ud559\uc124\ub9bd\uc6b4\ub3d9(\u6c11\u7acb\u5927\u5b78\u8a2d\u7acb\u904b\u52d5)\uc740 \ud574\ubc29 \uc9c1\ud6c4\uc778 1946\ub144 5\uc6d4 \uc870\uc120\ub300\ud559 \uc124\ub9bd\ub3d9\uc9c0\ud68c[\u671d\u9bae\u5927\u5b78 \u8a2d\u7acb\u540c\u5fd7\u6703]\uc640 \ucc3d\ub9bd\uc900\ube44\uc704\uc6d0\ud68c\uac00 \uacb0\uc131\ud558\uba74\uc11c \ubd80\ud65c\ud558\uc600\ub2e4. \uc124\ub9bd\ub3d9\uc9c0\ud68c\uc6d0\ub4e4\uc740 12\uc6d4\ubd80\ud130 \ud2b8\ub7ed\uc744 \ud0c0\uace0 \uc804\ub77c\ub3c4 \uac01 \uc9c0\uc5ed\uc744 \ub3cc\uc544\ub2e4\ub2c8\uba74\uc11c \ubaa8\uae08\ud558\uc600\ub294\ub370 \uc774\uc5d0 \uad70\uc218, \uacbd\ucc30\uc11c\uc7a5, \uba74\uc7a5, \uc2dc\uace8\uc774\uc7a5\uc740 \ubb3c\ub860 \uae30\uc0dd\uae4c\uc9c0 \ub2f9\uc2dc \uc300 2\ub9d0\uac12\uc5d0 \ud574\ub2f9\ud558\ub294 100\uc6d0\uc9dc\ub9ac \uc124\ub9bd\ub3d9\uc9c0\ud68c\uc6d0\uad8c\uc744 \uc0c0\uace0 \uac00\ub09c\ud588\ub358 \ub18d\ucd0c\ubbfc\ub3c4 \ub300\ud559\uc744 \uc138\uc6b4\ub2e4\ub294 \ub9d0\uc5d0 \uae68, \uae40, \ubbf8\uc5ed\uc744 \ub0b4\ub193\uc73c\uba74\uc11c \ub300\ud559 \uc124\ub9bd\uc5d0 \ud798\uc744 \ubcf4\ud0dc\ub294, \uc124\ub9bd\ub3d9\uc9c0\ud68c\uc6d0 \uc790\uaca9\uc740 \ud574\ubc29\ub41c \uc870\uad6d\uc744 \uac74\uc124\ud558\ub824\ub294 \uc2e0\ub150\uc5d0 \ubd88\ud0c0\ub294 \uc9c0\uc2dd\uc778, \uad00\ub9ac, \uc9c0\uc8fc\uc5d0\uc11c \uba38\uc2b4\uacfc \ucd0c\ubd80\uae4c\uc9c0 \uacc4\uce35, \ud559\ub825, \ube48\ubd80 \uaca9\ucc28\ub97c \ub118\uc5b4\uc11c \ud638\ub0a8 \uc9c0\uc5ed \uc804 \ub300\uc911\ubfd0\ub9cc \uc544\ub2c8\ub77c \uba40\ub9ac\ub294 \uc81c\uc8fc\ub3c4\uc5d0\uc11c \ucda9\uccad\ub3c4\uc640 \uc218\ub3c4\uad8c\uc5d0\uae4c\uc9c0 \uc644\uc804\ud788 \uac1c\ubc29\ub418\uc5c8\ub2e4. \ub2f9\uc2dc \uc804\ub77c\ub0a8\ub3c4\uc9c0\uc0ac\uc600\ub358 \uc11c\ubbfc\ud638\ub294 \uc77c\ubcf8 \uba54\uc774\uc9c0\ub300\ud559\uad50(\u660e\u6cbb\u5927\u5b66)\ub97c \uc878\uc5c5\ud558\uace0 \uc77c\ubcf8 \ubb38\uad50\uc0ac\ud68c\uad6d\uc5d0\uc11c \uadfc\ubb34\ud55c \uacbd\ud5d8\uc774 \uc788\ub294 \ubc15\ucca0\uc6c5\uc5d0\uac8c \uc124\ub9bd\ub3d9\uc9c0\ud68c\uc7a5\uc9c1\uc744 \ub9e1\uac8c \ud558\uc600\uace0 \ub300\ud559 \uc124\ub9bd\uc744 \uc900\ube44\ud558\uace0\uc790 \ub3c4\uccad\uc774 \uc18c\uc720\ud55c \ucc28\ub7c9\uc744 \uc0ac\uc6a9\ud558\uac8c \ud558\uace0 \ub3c4\uc9c0\uc0ac \uba85\uc758\ub85c \uc0ac\uc7a5\uacfc \uad70\uc218\uc5d0\uac8c \ud611\uc870 \uc9c0\uc2dc \uacf5\ubb38\uc744 \ubcf4\ub0b4\ub294, \uc124\ub9bd\uc5d0\uc11c \uc77c\uc120\uc5d0\uc11c \ud070 \uc5ed\ud560\uc744 \ud558\uc600\ub2e4."}